SUBJECT: Re: [ciencialist] Re: Seria Einstein uma farsa?
FROM: "Cyberlander" <mrcyberlander@i12.com>
TO: <ciencialist@yahoogrupos.com.br>
DATE: 14/12/2004 11:00

Eu conheci o César Lattes quando fiz engª química na UFRJ, um sujeito meio exc~entrico, arrastando as snadálias de dedo pelos corredores do bloco "A", se escondendo pelos cantos (d~e-se o desconto que era a época da ditadura). Ele gostacva quando as meninas calouras vinham puxar papo com ele na cantina (o lado nromal dele hehehehe). Acho que o que estragou ele foi a indicação pro Nobel.O cara é um gênio dedicado, assiti a algumas palestras dele (com projeção filmes super-8 pois na época não existia powerpoint). Ele agora é octogenário(o que no Brasil, para homens é quase sinônimo de caduquice), só lhe resta cultuar seu passado.
[ ]'s
D.C.

CYBERLANDER

Ama a realidade que constróis,
que nem a morte deterá teu voo · ·


----- Original Message -----

From: JVictor
To: ciencialist@yahoogrupos.com.br
Sent: Monday, December 13, 2004 11:24 PM
Subject: Re: [ciencialist] Re: Seria Einstein uma farsa?


Tá aí, Takata, falou e disse. Mas o Professor nem chega a morder!

Victor.
----- Original Message -----
From: rmtakata
To: ciencialist@yahoogrupos.com.br
Sent: Monday, December 13, 2004 4:41 PM
Subject: [ciencialist] Re: Seria Einstein uma farsa?



--- Em ciencialist@yahoogrupos.com.br, Rodrigo Marques <rodmarq72@y...>
> Mas porque então César Lattes, que pelo que sei é um físico competente
> falou estas coisas? Rabugice? Ou porque a inveja é realmente uma m...?

Se tivessem dado um Nobel para Lattes e' possivel q. ele nao mordesse tto.

[]s,

Roberto Takata





##### ##### #####

Para saber mais visite
http://www.ciencialist.hpg.ig.com.br


##### ##### ##### #####


Yahoo! Grupos, um serviço oferecido por:







------------------------------------------------------------------------------
Links do Yahoo! Grupos

a.. Para visitar o site do seu grupo na web, acesse:
http://br.groups.yahoo.com/group/ciencialist/

b.. Para sair deste grupo, envie um e-mail para:
ciencialist-unsubscribe@yahoogrupos.com.br

c.. O uso que você faz do Yahoo! Grupos está sujeito aos Termos do Serviço do Yahoo!.



[As partes desta mensagem que não continham texto foram removidas]



##### ##### #####

Para saber mais visite
http://www.ciencialist.hpg.ig.com.br


##### ##### ##### #####


Yahoo! Grupos, um serviço oferecido por:

São Paulo Rio de Janeiro Curitiba Porto Alegre Belo Horizonte Brasília




------------------------------------------------------------------------------
Links do Yahoo! Grupos

a.. Para visitar o site do seu grupo na web, acesse:
http://br.groups.yahoo.com/group/ciencialist/

b.. Para sair deste grupo, envie um e-mail para:
ciencialist-unsubscribe@yahoogrupos.com.br

c.. O uso que você faz do Yahoo! Grupos está sujeito aos Termos do Serviço do Yahoo!.



[As partes desta mensagem que não continham texto foram removidas]



SUBJECT: Re: [ciencialist] Re: Seria Einstein uma farsa?
FROM: "Alvaro Augusto - Lunabay" <alvaro@lunabay.com.br>
TO: <ciencialist@yahoogrupos.com.br>
DATE: 14/12/2004 12:22

Mas as críticas de Lattes não são de hoje! Logo, a hipótese da caduquice não
se aplica. Eu tenho até hoje uma exemplar de uma revista que durou apenas
três exemplares, de cujo nome não me lembro. Ela deve ter sido publicada em
1981 e trazia na capa uma ilustração de Lattes esganando Einstein. Na
reportagem, Lattes já desfiava suas críticas altamente embasadas ("Einstein
é uma besta", "defecou sobre o tapete persa da física", etc)...

[ ]s

Alvaro Augusto


----- Original Message -----
From: "Cyberlander" <mrcyberlander@i12.com>
To: <ciencialist@yahoogrupos.com.br>
Sent: Tuesday, December 14, 2004 11:00 AM
Subject: Re: [ciencialist] Re: Seria Einstein uma farsa?



Eu conheci o César Lattes quando fiz engª química na UFRJ, um sujeito meio
exc~entrico, arrastando as snadálias de dedo pelos corredores do bloco "A",
se escondendo pelos cantos (d~e-se o desconto que era a época da ditadura).
Ele gostacva quando as meninas calouras vinham puxar papo com ele na cantina
(o lado nromal dele hehehehe). Acho que o que estragou ele foi a indicação
pro Nobel.O cara é um gênio dedicado, assiti a algumas palestras dele (com
projeção filmes super-8 pois na época não existia powerpoint). Ele agora é
octogenário(o que no Brasil, para homens é quase sinônimo de caduquice), só
lhe resta cultuar seu passado.
[ ]'s
D.C.

CYBERLANDER

Ama a realidade que constróis,
que nem a morte deterá teu voo · ·


----- Original Message -----

From: JVictor
To: ciencialist@yahoogrupos.com.br
Sent: Monday, December 13, 2004 11:24 PM
Subject: Re: [ciencialist] Re: Seria Einstein uma farsa?


Tá aí, Takata, falou e disse. Mas o Professor nem chega a morder!

Victor.
----- Original Message -----
From: rmtakata
To: ciencialist@yahoogrupos.com.br
Sent: Monday, December 13, 2004 4:41 PM
Subject: [ciencialist] Re: Seria Einstein uma farsa?



--- Em ciencialist@yahoogrupos.com.br, Rodrigo Marques <rodmarq72@y...>
> Mas porque então César Lattes, que pelo que sei é um físico competente
> falou estas coisas? Rabugice? Ou porque a inveja é realmente uma m...?

Se tivessem dado um Nobel para Lattes e' possivel q. ele nao mordesse
tto.

[]s,

Roberto Takata





##### ##### #####

Para saber mais visite
http://www.ciencialist.hpg.ig.com.br


##### ##### ##### #####


Yahoo! Grupos, um serviço oferecido por:







--------------------------------------------------------------------------
----
Links do Yahoo! Grupos

a.. Para visitar o site do seu grupo na web, acesse:
http://br.groups.yahoo.com/group/ciencialist/

b.. Para sair deste grupo, envie um e-mail para:
ciencialist-unsubscribe@yahoogrupos.com.br

c.. O uso que você faz do Yahoo! Grupos está sujeito aos Termos do
Serviço do Yahoo!.



[As partes desta mensagem que não continham texto foram removidas]



##### ##### #####

Para saber mais visite
http://www.ciencialist.hpg.ig.com.br


##### ##### ##### #####


Yahoo! Grupos, um serviço oferecido por:

São Paulo Rio de Janeiro Curitiba Porto Alegre Belo
Horizonte Brasília




----------------------------------------------------------------------------
--
Links do Yahoo! Grupos

a.. Para visitar o site do seu grupo na web, acesse:
http://br.groups.yahoo.com/group/ciencialist/

b.. Para sair deste grupo, envie um e-mail para:
ciencialist-unsubscribe@yahoogrupos.com.br

c.. O uso que você faz do Yahoo! Grupos está sujeito aos Termos do
Serviço do Yahoo!.



[As partes desta mensagem que não continham texto foram removidas]



##### ##### #####

Para saber mais visite
http://www.ciencialist.hpg.ig.com.br


##### ##### ##### #####
Links do Yahoo! Grupos












SUBJECT: Re: Entrevista com Dawkins e evolução dos humanos
FROM: "rmtakata" <rmtakata@altavista.net>
TO: ciencialist@yahoogrupos.com.br
DATE: 14/12/2004 12:31


--- Em ciencialist@yahoogrupos.com.br, "murilo filo"
> Basta vc relêr o texto com atenção, e ver o ''a menos''...
> Por mim tudo bem, já que êle sugere naves p/colonizar o espaço. Já
> vão tarde.
> Isto é melhor do que a ''seleção administrada'' tentada na II
> Guerra. abr/M.

Continuo sem saber o q. isso tem a ver com nazismo. Ter programa
espacial e' ideologia nazista? Q. eu saiba, nao.

[]s,

Roberto Takata





SUBJECT: Re: Entrevista com Dawkins e evolução dos humanos
FROM: "rmtakata" <rmtakata@altavista.net>
TO: ciencialist@yahoogrupos.com.br
DATE: 14/12/2004 12:40


--- Em ciencialist@yahoogrupos.com.br, "Alberto Mesquita Filho"
> Pela pequena amostra apresentada, não irá demorar muito e os
> neonazistas mais fanáticos estarão recorrendo a esse pensamento do
> Dawkins para justificar a mentalidade assassina de seu ícone Adolf
> Hitler.

Um neonazista e' capaz de 'justificar' seus pensamentos idiotas ate'
com afirmacoes opostas.

Note-se q. Dawkins nao disse q. a extincao se dara' por causa do fluxo
genico. E sim q. nao havera' evolucao biologica (ou ela dar-se-a' em
um ritmo muito mais lento) em funcao do fluxo genico (o q. seria
errado, mas nada tem de nazista).

> Digo apenas que esse pensamento realmente parece um "treco"
> nazista, como afirmou o Murilo.

Realmente nao sei como se parece.

> ********* início da citação ************
> "Eu não acho que os humanos irão evoluir mais -nós seremos extintos
> antes. A menos que nos mudemos para o espaço e nos separemos em
> colônias, que é o que é preciso para que a evolução divergente
> aconteça. Neste momento, há fluxo gênico em todo o mundo, e isso
> evita a evolução." (Richard Dawkins)
> ********* final da citação ************

[]s,

Roberto Takata





SUBJECT: O Codigo Genético
FROM: "rmtakata" <rmtakata@altavista.net>
TO: ciencialist@yahoogrupos.com.br
DATE: 14/12/2004 13:32


Cientistas desvendam código genético do arroz - diz o título da
matéria da Folha online
(http://www1.folha.uol.com.br/folha/ciencia/ult306u12732.shtml - texto
distribuído pela Agência Lusa)

Detalhe: o código genético foi desvendado em 1961! O q. foi feito é o
*seqüenciamento* do material genético do arroz.

Tudo isso apenas para indicar um sítio muito interessante:

Deciphering the Genetic Code
http://history.nih.gov/exhibits/nirenberg/

(Eu visualizo em futuro bem próximo q. as mensagens secretas serão
codificados em pequenos trechos de ADN - q. poderão ser guardadas na
unha, dente, nas células dos espiões ou simplesmente transmitidas via
internet como inocentes resultados de sequenciamento.)

[]s,

Roberto Takata





SUBJECT: Site inacreditável
FROM: Rodrigo Marques <rodmarq72@yahoo.com.br>
TO: Ceticismo Aberto <ceticismoaberto@yahoogrupos.com.br>, CienciaList <ciencialist@yahoogrupos.com.br>, Sociedade Brasileira de "C�ticos" e Racionalistas <sbcr@yahoogrupos.com.br>, Sociedade da Terra redonda <strbrasil@yahoogrupos.com.br>
DATE: 14/12/2004 13:46

Pessoal, quem tiver estômago veja este site:
http://www.showdalua.com/

Aqui o cara defende as inacreditáveis teorias:
-nunca houve uma viagem à Lua;
-a bomba atômica não existe;
-a Terra não gira ao redor do Sol;
-a Terra e todos os planetas tem uma órbita independente;
-propõe uma nova teoria da gravidade (a gravidade é uma força externa que vem da lua e planetas sem lua não tem gravidade)
-buracos negros não existem;
-Einstein, Galileu e Copérnico erraram.
Confiram...


---------------------------------
Yahoo! Mail - Agora com 250MB de espaço gratuito. Abra uma conta agora!

[As partes desta mensagem que não continham texto foram removidas]



SUBJECT: Re: Site inacreditável
FROM: "brudna" <lrb@iq.ufrgs.br>
TO: ciencialist@yahoogrupos.com.br
DATE: 14/12/2004 13:56


Algo intrigante eh que varios sites conspiracionistas tem textos
escritos em vermelho e de varios tamanhos. Sao desorganizado e com
imagens feias. :-) (devem ser memes que acompanham as piracoes)

Ateh
Luis Brudna

--- Em ciencialist@yahoogrupos.com.br, Rodrigo Marques
<rodmarq72@y...> escreveu
> Pessoal, quem tiver estômago veja este site:
> http://www.showdalua.com/
>
> Aqui o cara defende as inacreditáveis teorias:
> -nunca houve uma viagem à Lua;
> -a bomba atômica não existe;
> -a Terra não gira ao redor do Sol;
> -a Terra e todos os planetas tem uma órbita independente;
> -propõe uma nova teoria da gravidade (a gravidade é uma força
externa que vem da lua e planetas sem lua não tem gravidade)
> -buracos negros não existem;
> -Einstein, Galileu e Copérnico erraram.
> Confiram...






SUBJECT: Re: um paradoxo e dois bobos, sera?
FROM: Hélio Ricardo Carvalho <hrc@fis.puc-rio.br>
TO: ciencialist@yahoogrupos.com.br
DATE: 14/12/2004 13:59




> . . . . . .

> Esta placa fica com distribuição de cargas nas suas duas
> superfícies. Lembre que ainda existe um campo elétrico interno que
> deve ser neutralizado no corpo da placa interna do capacitor
> cilíndrico.


Continuando

Bem, neste momento a esfera de tranf. está encostada na sup. interna
da placa interna do capacitor e a carga total deste conjunto (esfera
+ esta superf.) é (quase) igual (em módulo) a carga da atual da
esfera do gerador que, por sua vez, já não é mais a mesma que
inicialmente. Não é mais a mesma porque a capacitância mudou.
A sup. externa da placa interna está com uma carga negativa de
módulo igual a soma da carga da esfera do gerador e da carga
transferida. A placa externa está com uma carga positiva de módulo
igual a esta soma.

As principais falhas do artigo são:
1 - Considerar a capacitância da esfera do gerador como independente
de todas as outras placas metálicas e distribuições de cargas do
espaço. (que são pertencentes ao circuito elétrico da bateria).
2 - Misturar geometria cilíndrica (capacitor cilíndrico) com
esférica (esfera do gerador) e com coisas totalmente assimétricas
(esfera de transferência fora do centro). Isto dificulta os
cálculos. Aquela expressões mostradas no artigo não se aplicam à
esta geometria mista.

Poderia ser ou tudo esférico, ou tudo cilíndrico de comprimento
infinito ou tudo plano de área infinita.

Ex., tudo esférico:
Esfera do gerador, capacitor esférico envolvendo e uma membrana
metálica esférica cujo raio variasse desde o da esfera do gerador
até a interna do capacitor.

Vou tentar refazer os cálculos dele nesta situação e depois (talvez)
eu conto aqui.


Hélio









SUBJECT: Re: [ciencialist] Site inacreditável
FROM: "Alvaro Augusto - Lunabay" <alvaro@lunabay.com.br>
TO: <ciencialist@yahoogrupos.com.br>
DATE: 14/12/2004 14:46

Esse André Mauro até que começou bem, mostrando que era apenas levemente
maluco e instigando os cientistas a mostrarem que a viagem à Lua realmente
ocorreu. Mas agora ele enfiou o pé na jaca de vez!

[ ]s

Alvaro Augusto

----- Original Message -----
From: "Rodrigo Marques" <rodmarq72@yahoo.com.br>
To: "Ceticismo Aberto" <ceticismoaberto@yahoogrupos.com.br>; "CienciaList"
<ciencialist@yahoogrupos.com.br>; "Sociedade Brasileira de Céticos e
Racionalistas" <sbcr@yahoogrupos.com.br>; "Sociedade da Terra redonda"
<strbrasil@yahoogrupos.com.br>
Sent: Tuesday, December 14, 2004 1:46 PM
Subject: [ciencialist] Site inacreditável



Pessoal, quem tiver estômago veja este site:
http://www.showdalua.com/

Aqui o cara defende as inacreditáveis teorias:
-nunca houve uma viagem à Lua;
-a bomba atômica não existe;
-a Terra não gira ao redor do Sol;
-a Terra e todos os planetas tem uma órbita independente;
-propõe uma nova teoria da gravidade (a gravidade é uma força externa que
vem da lua e planetas sem lua não tem gravidade)
-buracos negros não existem;
-Einstein, Galileu e Copérnico erraram.
Confiram...




SUBJECT: Re: um paradoxo e dois bobos, sera?
FROM: "rosevena3" <rosevena3@yahoo.com.br>
TO: ciencialist@yahoogrupos.com.br
DATE: 14/12/2004 15:00


--- Em ciencialist@yahoogrupos.com.br, Hélio Ricardo Carvalho
<hrc@f...> escreveu
>
>
>
> > . . . . . .

> Vou tentar refazer os cálculos dele nesta situação e depois
(talvez)
> eu conto aqui.
>
>
> Hélio

Gostei muito dessa sua continuidade explicativa, pois só assim
não
restarão duvidas a respeito do assunto.
As vezes é necessário a perda de um pouco mais de tempo com um
assunto, banal ou ele pode originar uma nova teoria errônea,
não?
(memória da água)
Agradeço e espero
Eu por exemplo estou muito preocupado, pois...
O contraditório tem que calar o cérebro, ou...
Solicito permissão para pedir que alguém me explique o
seguinte,
NUMA ESFERA CARREGADA, SENDO O POTENCIAL CONSTANTE EM QUALQUER PONTO
INTERNO E A SUPERFICIE, PORQUE ENTÃO A CARGA ELETRICA EM EXCESSO
NÃO
PODE ESTAR EM QUALQUER PONTO,(POTENCIAL CONSTANTE NÃO É COMO
EQUILIBRIO) MAS APENAS NA SUPERFICIE DO CONDUTOR? É UMA
CONVENÇÃO?
Explicando: se deposito uma carga extra no interior do condutor e ela
vai para a superfície, é porque uma força esta agindo sobre
a carga,
ou não? E se tem uma força agindo é desequilíbrio, se
é
desequilíbrio, o potencial não é o mesmo, entende?






SUBJECT: Re: [ciencialist] Site inacreditável
FROM: "Oraculo" <oraculo@atibaia.com.br>
TO: <ciencialist@yahoogrupos.com.br>
DATE: 14/12/2004 15:14

Olá

Já que estamos falando de sites inacreditáveis, deem uma olhada neste, é ótimo (e é sobre ciência, Brudna, já que é um artefato "científico" para permitir a vida eterna..:-)

http://www.alexchiu.com/

Um abraço.

Homero


----- Original Message -----
From: Rodrigo Marques
To: Ceticismo Aberto ; CienciaList ; Sociedade Brasileira de Céticos e Racionalistas ; Sociedade da Terra redonda
Sent: Tuesday, December 14, 2004 1:46 PM
Subject: [ciencialist] Site inacreditável


Pessoal, quem tiver estômago veja este site:
http://www.showdalua.com/

Aqui o cara defende as inacreditáveis teorias:
-nunca houve uma viagem à Lua;
-a bomba atômica não existe;
-a Terra não gira ao redor do Sol;
-a Terra e todos os planetas tem uma órbita independente;
-propõe uma nova teoria da gravidade (a gravidade é uma força externa que vem da lua e planetas sem lua não tem gravidade)
-buracos negros não existem;
-Einstein, Galileu e Copérnico erraram.
Confiram...


---------------------------------
Yahoo! Mail - Agora com 250MB de espaço gratuito. Abra uma conta agora!

[As partes desta mensagem que não continham texto foram removidas]



##### ##### #####

Para saber mais visite
http://www.ciencialist.hpg.ig.com.br


##### ##### ##### #####


Yahoo! Grupos, um serviço oferecido por:







------------------------------------------------------------------------------
Links do Yahoo! Grupos

a.. Para visitar o site do seu grupo na web, acesse:
http://br.groups.yahoo.com/group/ciencialist/

b.. Para sair deste grupo, envie um e-mail para:
ciencialist-unsubscribe@yahoogrupos.com.br

c.. O uso que você faz do Yahoo! Grupos está sujeito aos Termos do Serviço do Yahoo!.



[As partes desta mensagem que não continham texto foram removidas]



SUBJECT: Água em Marte (era Site inacreditável p/ Alberto)
FROM: "Oraculo" <oraculo@atibaia.com.br>
TO: <ciencialist@yahoogrupos.com.br>
DATE: 14/12/2004 15:40

Olá Alberto

Esta mensagem é sobre a descoberta de novas evidencias de água em Marte, mas aproveito para dizer que adorei a resposta as afirmações do site inacreditavel, em especial o sashimi estragado, tô rindo até agora..:-)

Um abraço.

Homero
_______________________________________
http://noticias.terra.com.br/ciencia/interna/0,,OI438801-EI301,00.html

Sonda acha nova prova de água em Marte


O robô americano Spirit descobriu uma nova prova de que houve água em Marte. O anúncio foi feito nesta segunda-feira pela Nasa.
"Os cientistas identificaram um mineral chamado goethita sobre rochas examinadas pelo Spirit", declarou o Jet Propulsion Laboratory (JPL), da Nasa, em um comunicado. "A goethita, assim como a jarosita encontrada pelo Opportunity, é uma prova significativa da presença, no passado, de água em Marte", disse o especialista Goestar Klingelhoefer, da Universidade de Mainz, na Alemanha. Esse mineral "se forma apenas na presença de água, seja em sua forma líquida, sólida ou gasosa", explicou a Agência.

A sonda gêmea Opportunity, que está do lado oposto de Marte, em uma região chamada Meridiani Planum, foi a primeira a estabelecer há meses a presença de água salgada sobre a superfície do Planeta Vermelho. Os dois robôs estão na superfície marciana há cerca de um ano. A dupla terminou sua missão inicial em abril, mas continuou funcionando normalmente além da duração prevista de sua vida útil.


----- Original Message -----
From: Alberto Mesquita Filho
To: ciencialist@yahoogrupos.com.br
Sent: Tuesday, December 14, 2004 4:26 PM
Subject: Re: [ciencialist] Site inacreditável


----- Original Message -----
From: "Rodrigo Marques"
Sent: Tuesday, December 14, 2004 12:46 PM
Subject: [ciencialist] Site inacreditável

> Pessoal, quem tiver estômago veja este site:
> http://www.showdalua.com/
> Aqui o cara defende as inacreditáveis teorias:

> -nunca houve uma viagem à Lua;

Como não?!!! Os físicos "modernos" vivem no mundo da Lua desde o início do
século passado.

> -a bomba atômica não existe;

De fato. Isso é invenção de japonês. Em 1945 houve duas epidemias (uma em
Hiroshima e outra em Nagasaki) devidas ao "sashimi" estragado e o governo
japonês decretou a existência da bomba atômica, botando a culpa nos
angelicais estadunidenses.

> -a Terra não gira ao redor do Sol;

Isso todo o mundo sabe e faz parte da sabedoria milenar. A Terra está
situada sobre um prato dourado apoiado nas costas de vários elefantes e
estes equilibram-se sobre o casco de uma tartaruga gigante. Foi a partir
dessa constatação científica que surgiu a expressão popular: "É preciso ter
um saco de elefante".

Existe uma teoria alternativa e bastante promissora, a dizer que a Terra é
ôca e que nós vivemos na superfície interna de uma casca esférica. O Sol
estaria situado no centro da esfera e emitiria luz apenas para um dos lados,
tal e qual um farol náutico . Os que acreditam nessa teoria são obrigados a
assumir ou o giro do farol ou o giro da casca esférica, afim de explicarem a
alternância dos dias e das noites.

> -a Terra e todos os planetas tem uma órbita independente;

Nesse caso discordo. A órbita da Terra não é independente, mas determinada
por Vixenu, o deus da água e que nada mais é senão a tartaruga gigante da
teoria hindu, acima referida. Se a órbita fosse independente não haveria
como explicar o dilúvio universal, resultante de um porre da tartaruga.

> -propõe uma nova teoria da gravidade (a gravidade é uma força externa que
> vem da lua e planetas sem lua não tem gravidade)

Essa teoria promete, mas ainda está na fase das especulações. Por ora é
melhor acreditar que os corpos evoluem para os seus locais naturais,
conforme comprovado cientificamente por Aristóteles.

> -buracos negros não existem;

Essa foi uma hipótese propugnada pelos preconceituosos agentes da
Ku-Klux-Klan, mas não chegou a vingar.

> -Einstein, Galileu e Copérnico erraram.

Essa eu nem vou comentar pois não conheço nenhum dos três. De qualquer
forma, e haja vista que errar é humano, eles parecem estar no bom caminho.

[ ]´s
Alberto
http://ecientificocultural.com/indice.htm
Mas indiferentemente a tudo isso, o neutrino tem massa, o elétron não é
uma carga elétrica coulombiana e a Terra se move. E a história se repetirá.



##### ##### #####

Para saber mais visite
http://www.ciencialist.hpg.ig.com.br


##### ##### ##### #####


Yahoo! Grupos, um serviço oferecido por:

São Paulo Rio de Janeiro Curitiba Porto Alegre Belo Horizonte Brasília




------------------------------------------------------------------------------
Links do Yahoo! Grupos

a.. Para visitar o site do seu grupo na web, acesse:
http://br.groups.yahoo.com/group/ciencialist/

b.. Para sair deste grupo, envie um e-mail para:
ciencialist-unsubscribe@yahoogrupos.com.br

c.. O uso que você faz do Yahoo! Grupos está sujeito aos Termos do Serviço do Yahoo!.



[As partes desta mensagem que não continham texto foram removidas]



SUBJECT: Re: [ciencialist] Site inacreditável
FROM: Italo <itscefet03@yahoo.com.br>
TO: ciencialist@yahoogrupos.com.br
DATE: 14/12/2004 15:41

Sites inacreditáveis va-lá! Mas vida eterna já é SPAM .

Ítalo

Oraculo <oraculo@atibaia.com.br> wrote:
Olá

Já que estamos falando de sites inacreditáveis, deem uma olhada neste, é ótimo (e é sobre ciência, Brudna, já que é um artefato "científico" para permitir a vida eterna..:-)

http://www.alexchiu.com/

Um abraço.

Homero




---------------------------------
Yahoo! Mail - Agora com 250MB de espaço gratuito. Abra uma conta agora!

[As partes desta mensagem que não continham texto foram removidas]



SUBJECT: Re: [ciencialist] Site inacreditável
FROM: "Oraculo" <oraculo@atibaia.com.br>
TO: <ciencialist@yahoogrupos.com.br>
DATE: 14/12/2004 15:55

Olá Italo

Por favor, eu não estava fazendo propaganda do sistema de vida eterna..:-) Apenas achei que, por ser um dispositivo "científico" seria do interesse da lista..:-)

Senão como física ou biologia (o dispositivo certamente é uma fraude..:-), pelo menos na área de psicologia, ao demonstrar a capacidade de acreditar em quase qualquer coisa, que a mente humana parece possuir. Vida eterna através de anéis com imãs e vida eterna em paraisos sobrenaturais, a distância é pequena em termos de credulidade..:-)

Um abraço.

Homero

----- Original Message -----
From: Italo
To: ciencialist@yahoogrupos.com.br
Sent: Tuesday, December 14, 2004 3:41 PM
Subject: Re: [ciencialist] Site inacreditável


Sites inacreditáveis va-lá! Mas vida eterna já é SPAM .

Ítalo

Oraculo <oraculo@atibaia.com.br> wrote:
Olá

Já que estamos falando de sites inacreditáveis, deem uma olhada neste, é ótimo (e é sobre ciência, Brudna, já que é um artefato "científico" para permitir a vida eterna..:-)

http://www.alexchiu.com/

Um abraço.

Homero




---------------------------------
Yahoo! Mail - Agora com 250MB de espaço gratuito. Abra uma conta agora!

[As partes desta mensagem que não continham texto foram removidas]



##### ##### #####

Para saber mais visite
http://www.ciencialist.hpg.ig.com.br


##### ##### ##### #####


Yahoo! Grupos, um serviço oferecido por:

São Paulo Rio de Janeiro Curitiba Porto Alegre Belo Horizonte Brasília




------------------------------------------------------------------------------
Links do Yahoo! Grupos

a.. Para visitar o site do seu grupo na web, acesse:
http://br.groups.yahoo.com/group/ciencialist/

b.. Para sair deste grupo, envie um e-mail para:
ciencialist-unsubscribe@yahoogrupos.com.br

c.. O uso que você faz do Yahoo! Grupos está sujeito aos Termos do Serviço do Yahoo!.



[As partes desta mensagem que não continham texto foram removidas]



SUBJECT: Re: [ciencialist] Site inacreditável
FROM: Italo <itscefet03@yahoo.com.br>
TO: ciencialist@yahoogrupos.com.br
DATE: 14/12/2004 16:20

Falei brincando, desculpe-me, Homero, se "peguei pesado" (talvez no seu browser não tenha aparecido o smile que eu coloquei, para "denunciar" o tom irônico....).

Ítalo

Oraculo <oraculo@atibaia.com.br> wrote:
Olá Italo

Por favor, eu não estava fazendo propaganda do sistema de vida eterna..:-) Apenas achei que, por ser um dispositivo "científico" seria do interesse da lista..:-)

Senão como física ou biologia (o dispositivo certamente é uma fraude..:-), pelo menos na área de psicologia, ao demonstrar a capacidade de acreditar em quase qualquer coisa, que a mente humana parece possuir. Vida eterna através de anéis com imãs e vida eterna em paraisos sobrenaturais, a distância é pequena em termos de credulidade..:-)

Um abraço.

Homero


---------------------------------
Yahoo! Mail - Agora com 250MB de espaço gratuito. Abra uma conta agora!

[As partes desta mensagem que não continham texto foram removidas]



SUBJECT: Re: [ciencialist] Site inacreditável
FROM: "Alberto Mesquita Filho" <albmesq@uol.com.br>
TO: <ciencialist@yahoogrupos.com.br>
DATE: 14/12/2004 16:26

----- Original Message -----
From: "Rodrigo Marques"
Sent: Tuesday, December 14, 2004 12:46 PM
Subject: [ciencialist] Site inacreditável

> Pessoal, quem tiver estômago veja este site:
> http://www.showdalua.com/
> Aqui o cara defende as inacreditáveis teorias:

> -nunca houve uma viagem à Lua;

Como não?!!! Os físicos "modernos" vivem no mundo da Lua desde o início do
século passado.

> -a bomba atômica não existe;

De fato. Isso é invenção de japonês. Em 1945 houve duas epidemias (uma em
Hiroshima e outra em Nagasaki) devidas ao "sashimi" estragado e o governo
japonês decretou a existência da bomba atômica, botando a culpa nos
angelicais estadunidenses.

> -a Terra não gira ao redor do Sol;

Isso todo o mundo sabe e faz parte da sabedoria milenar. A Terra está
situada sobre um prato dourado apoiado nas costas de vários elefantes e
estes equilibram-se sobre o casco de uma tartaruga gigante. Foi a partir
dessa constatação científica que surgiu a expressão popular: "É preciso ter
um saco de elefante".

Existe uma teoria alternativa e bastante promissora, a dizer que a Terra é
ôca e que nós vivemos na superfície interna de uma casca esférica. O Sol
estaria situado no centro da esfera e emitiria luz apenas para um dos lados,
tal e qual um farol náutico . Os que acreditam nessa teoria são obrigados a
assumir ou o giro do farol ou o giro da casca esférica, afim de explicarem a
alternância dos dias e das noites.

> -a Terra e todos os planetas tem uma órbita independente;

Nesse caso discordo. A órbita da Terra não é independente, mas determinada
por Vixenu, o deus da água e que nada mais é senão a tartaruga gigante da
teoria hindu, acima referida. Se a órbita fosse independente não haveria
como explicar o dilúvio universal, resultante de um porre da tartaruga.

> -propõe uma nova teoria da gravidade (a gravidade é uma força externa que
> vem da lua e planetas sem lua não tem gravidade)

Essa teoria promete, mas ainda está na fase das especulações. Por ora é
melhor acreditar que os corpos evoluem para os seus locais naturais,
conforme comprovado cientificamente por Aristóteles.

> -buracos negros não existem;

Essa foi uma hipótese propugnada pelos preconceituosos agentes da
Ku-Klux-Klan, mas não chegou a vingar.

> -Einstein, Galileu e Copérnico erraram.

Essa eu nem vou comentar pois não conheço nenhum dos três. De qualquer
forma, e haja vista que errar é humano, eles parecem estar no bom caminho.

[ ]´s
Alberto
http://ecientificocultural.com/indice.htm
Mas indiferentemente a tudo isso, o neutrino tem massa, o elétron não é
uma carga elétrica coulombiana e a Terra se move. E a história se repetirá.



SUBJECT: Re: Site inacreditável
FROM: "rosevena3" <rosevena3@yahoo.com.br>
TO: ciencialist@yahoogrupos.com.br
DATE: 14/12/2004 16:35


Ola.
Com licença de por minha colher nesse angu.
Só vi a parte que trata da terra não girar em torno do sol, não
entendi nada pois não sou entendido no assunto, então pergunto:
O CARA PODE TER ALGUMA RAZÃO, OU É LOUCO MESMO?
Para mim seria muito fácil ficar com a maioria, mas penso que toda
unanimidade é burra, e eu tenho que entender o cara para poder
julgar, e confesso que não entendo nada para poder saber PORQUE A
CONSTELAÇÃO DEVERIA ESTAR EM UM SÓ PONTO UMA VEZ SÓ POR ANO?
ALGUEM PODE EXPLICAR A LOGICA DO FULANO SE É QUE TEM ALGUMA,
GRATO






SUBJECT: Site inacreditável p/ rosevena3
FROM: "Oraculo" <oraculo@atibaia.com.br>
TO: <ciencialist@yahoogrupos.com.br>
DATE: 14/12/2004 17:03

Olá rosevena3

Esse é o problema, a lógica do autor é pessoal, impossível de ser replicada por outra pessoa..:-) Assim, é impossível refutar um argumento baseado nessa lógica não convencional..:-)

E o dito "toda unanimidade é burra" deve ser aplicado a opiniões, não a toda afirmação ou conhecimento. A unanimidade conhecida como ciência (ou o conhecimento por ela produzida, que tende a se tornar unanime) não deriva de opiniões mas de conclusões embasadas. E pode (e deve) sempre mudar, claro, mas exige, para tanto, que o motivo da mudança siga algumas regras rígidas, conhecidas como método cientifico. E a mudança, na maioria absoluta das vezes, é mais um ajuste, uma melhora, que uma refutação ou exclusão.

Veja o caso da Terra que gira ao redor do Sol, alegação com a qual o autor não concorda. Essa unanimidade não é burra, é a conclusão de centenas de evidencias, experimentos e calculos realizados ao longo do tempo, que suportam e explicam diversos fenomenos e desdobramentos a respeito do universo físico. Não corremos o risco de descobrir, amanhã ou depois, que na verdade estavamos errados e que a Terra se apoia em um circulo de matal que se apoia sobre elefantes nas costas de uma tartaruga..:-) Mesmo você, se fizer uma replica do sistema solar mecanica (talvez um experimento apra a feira de ciencias da escola..:-), se repetir, em escla, as posições e velocidades de cada planeta e satelite, verá que este modelo se comporta e repete todos os fenomenso que observamos diariamente, dia e noite, ano, eclipses, trajeto dos planetas, etc, etc, etc. Se alguém deseja refutar esse conhecimento, tem de explicar porque isso acontece, não?

Claro que não é possível, para nós leigos e cidadãos comuns, testar TODA afirmação ou teoria que se apresenta. Assim, uma escolha sábia (e não burra), seria decidir acompanhar as alegações e conhecimentos produzidos por um sistema que tem se mostrado confiável a bastante tempo, a ciência. Olhe para os lados o que vê? Desdobramentos confiáveis desse conhecimento. Aviões, computadores, remedios, espectativa de vida de mais de 80 anos, pouca mortalidade infantil, e esta conversa entre nós dois.

As vezes a ciência vai se enganar, e nós junto com ela, mas, estatisticamente, vamos acertar muito mais, muito mesmo, que se escolhermos autores malucos como esse..:-)

O computador em que você escreve e que me envia esta mensagem, assim como a rede Internet que permite isso, é totalmente fruto do conhecimento cientifico. Não há nada nesta conversa que não seja resultado de um conehcimento confiável e cientifico. Seu automóvel, sua espectativa de vida, sua viajem em aviões, sua TV, seu aparelho de som, seu bem estar físico, tudo deriva de um conehcimento confiável, dentro do limite máximo da incerteza deste universo.

Então, se a ciência, dentro de um método, diz que a Terra gira ao redor do Sol, se os desdobramentos disso são visiveis (seu telefone celular fala com o Japão, instantaneamente, se desejar, porque existem satelites em órbita, e estes só podem ficar lá se as equações e cálculos sobre a gravidade e forças físicas estiverem corretos, as mesmas equações e forças fisicas que explicam a órbita da Terra..:-). Mas, as afirmações de pesosas como o autor da página inacreditável o que tem para sustentá-las? Uma lógica pessoal e improvável, afirmações as mais estapafurdias e teorias conspiratórias antigas.

Onde colocaria, digamos, seu dinheiro, se tivesse de apostar em uma delas?..:-)

Um abraço.

Homero



----- Original Message -----
From: rosevena3
To: ciencialist@yahoogrupos.com.br
Sent: Tuesday, December 14, 2004 4:35 PM
Subject: [ciencialist] Re: Site inacreditável



Ola.
Com licença de por minha colher nesse angu.
Só vi a parte que trata da terra não girar em torno do sol, não
entendi nada pois não sou entendido no assunto, então pergunto:
O CARA PODE TER ALGUMA RAZÃO, OU É LOUCO MESMO?
Para mim seria muito fácil ficar com a maioria, mas penso que toda
unanimidade é burra, e eu tenho que entender o cara para poder
julgar, e confesso que não entendo nada para poder saber PORQUE A
CONSTELAÇÃO DEVERIA ESTAR EM UM SÓ PONTO UMA VEZ SÓ POR ANO?
ALGUEM PODE EXPLICAR A LOGICA DO FULANO SE É QUE TEM ALGUMA,
GRATO






##### ##### #####

Para saber mais visite
http://www.ciencialist.hpg.ig.com.br


##### ##### ##### #####


Yahoo! Grupos, um serviço oferecido por:







------------------------------------------------------------------------------
Links do Yahoo! Grupos

a.. Para visitar o site do seu grupo na web, acesse:
http://br.groups.yahoo.com/group/ciencialist/

b.. Para sair deste grupo, envie um e-mail para:
ciencialist-unsubscribe@yahoogrupos.com.br

c.. O uso que você faz do Yahoo! Grupos está sujeito aos Termos do Serviço do Yahoo!.



[As partes desta mensagem que não continham texto foram removidas]



SUBJECT: eletron não tão nebuloso
FROM: "rayfisica" <rayfisica@yahoo.com.br>
TO: ciencialist@yahoogrupos.com.br
DATE: 14/12/2004 17:04


Alberto
Li acho que nos três primeiros minutos ou no universo elegante, o
seguinte:
`O repórter pergunta a um estudioso, sobre o fato de apenas três
pessoas compreenderem a teoria da relatividade, dizem que após pensar
um pouco o cientista responde – quem poderia ser a terceira pessoa'.
Acho que dá pra entender o que estou tentando passar.
Pergunto lhe:
TODAS AS PESSOAS QUE TE CRITICAM REALMENTE ENTENDERAM SUAS IDEIAS OU
ALGUMAS ESTÃO APENAS REPETINDO O PENSAMENTO VIGENTE?
QUANTO POR CENTO DAS PESSOAS QUE TIVERAM CONTATO COM SUA TEORIA A
ENTENDEU?
Estou lendo o seu trabalho, depois volto a tc para ti.






SUBJECT: Re: um paradoxo e dois bobos, sera?
FROM: Hélio Ricardo Carvalho <hrc@fis.puc-rio.br>
TO: ciencialist@yahoogrupos.com.br
DATE: 14/12/2004 17:55


"rosevena3"


Vou responder rápido pois tenho que sair logo.

A teoria eletromagnética (quando surgiu com tudo que aprendemos
ainda hoje em física III) desconhece o elétron. Logo, qq carga
colocada no interior do condutor iria se separar em pedaços de carga
ainda menores que se afastariam uma das outras por serem de mesmo
sinal.

:-)


Hélio



<rosevena3@y...> escreveu
>
> --- Em ciencialist@yahoogrupos.com.br, Hélio Ricardo Carvalho
> <hrc@f...> escreveu
> >
> >
> >
> > > . . . . . .
>
> > Vou tentar refazer os cálculos dele nesta situação e depois
> (talvez)
> > eu conto aqui.
> >
> >
> > Hélio
>
> Gostei muito dessa sua continuidade explicativa, pois só assim
> não
> restarão duvidas a respeito do assunto.
> As vezes é necessário a perda de um pouco mais de tempo com um
> assunto, banal ou ele pode originar uma nova teoria errônea,
> não?
> (memória da água)
> Agradeço e espero
> Eu por exemplo estou muito preocupado, pois...
> O contraditório tem que calar o cérebro, ou...
> Solicito permissão para pedir que alguém me explique o
> seguinte,
> NUMA ESFERA CARREGADA, SENDO O POTENCIAL CONSTANTE EM QUALQUER
PONTO
> INTERNO E A SUPERFICIE, PORQUE ENTÃO A CARGA ELETRICA EM EXCESSO
> NÃO
> PODE ESTAR EM QUALQUER PONTO,(POTENCIAL CONSTANTE NÃO É COMO
> EQUILIBRIO) MAS APENAS NA SUPERFICIE DO CONDUTOR? É UMA
> CONVENÇÃO?
> Explicando: se deposito uma carga extra no interior do condutor e
ela
> vai para a superfície, é porque uma força esta agindo sobre
> a carga,
> ou não? E se tem uma força agindo é desequilíbrio, se
> é
> desequilíbrio, o potencial não é o mesmo, entende?





SUBJECT: Re: Site inacreditável
FROM: Maria Natália <grasdic@hotmail.com>
TO: ciencialist@yahoogrupos.com.br
DATE: 14/12/2004 18:40


Rodrigo:
Se um país não tiver asilos para doidinhos, onde eles se sentirão à
vontade? Na net, nem mais!
Portanto acho que este site quer fazer concorrência ao de humor em
Ciência. Mas não consegue. A ergonomia do site deixa muito a
desejar: faz lembrar uma drogaria ou feira da ladra (marchè aux
pouces). Nem para aulas dá.
Feio porco e mau
Um abraço
Maria Natália

--- Em ciencialist@yahoogrupos.com.br, Rodrigo Marques
<rodmarq72@y...> escreveu
> Pessoal, quem tiver estômago veja este site:
> http://www.showdalua.com/
>
> Aqui o cara defende as inacreditáveis teorias:
> -nunca houve uma viagem à Lua;
> -a bomba atômica não existe;
> -a Terra não gira ao redor do Sol;
> -a Terra e todos os planetas tem uma órbita independente;
> -propõe uma nova teoria da gravidade (a gravidade é uma força
externa que vem da lua e planetas sem lua não tem gravidade)
> -buracos negros não existem;
> -Einstein, Galileu e Copérnico erraram.
> Confiram...
>
>
> ---------------------------------
> Yahoo! Mail - Agora com 250MB de espaço gratuito. Abra uma conta
agora!
>
> [As partes desta mensagem que não continham texto foram removidas]





SUBJECT: Re: Site inacreditável
FROM: Maria Natália <grasdic@hotmail.com>
TO: ciencialist@yahoogrupos.com.br
DATE: 14/12/2004 19:06


Rosevena:
O cara é mesmo pessoa com deficit em atenção. Pois quem é astrónomo
da lista já lhe deu descrédito. E existe o Hubble situado fora da
Terra e observando o Universo.
Já eras vivo quando o Homem chegou à lua? E teu avô? Por aí já podes
analisar o tal Marco. Aliás este site é a tradução de um americano.
Nem original é desgraçado é. E me parece que nem pertença à academia
brasileirra de Ciência. Depois nenhuma universidade apoio o tipo.
Nem vontade de rir dá pois é prova de ignorância total e acho que
não nos devemos rir com a ignorância. Quem estudou física a sério
topa.
Para antídoto desta página te recomendo esta do Phil Plait
http://www.badastronomy.com/
Um abraço
Maria Natália
--- Em ciencialist@yahoogrupos.com.br, "rosevena3" <rosevena3@y...>
escreveu
>
> Ola.
> Com licença de por minha colher nesse angu.
> Só vi a parte que trata da terra não girar em torno do sol, não
> entendi nada pois não sou entendido no assunto, então pergunto:
> O CARA PODE TER ALGUMA RAZÃO, OU É LOUCO MESMO?
> Para mim seria muito fácil ficar com a maioria, mas penso que toda
> unanimidade é burra, e eu tenho que entender o cara para poder
> julgar, e confesso que não entendo nada para poder saber PORQUE A
> CONSTELAÇÃO DEVERIA ESTAR EM UM SÓ PONTO UMA VEZ SÓ POR ANO?
> ALGUEM PODE EXPLICAR A LOGICA DO FULANO SE É QUE TEM ALGUMA,
> GRATO





SUBJECT: RES: [ciencialist] Re: importação de livros
FROM: "pablo" <pablo@unidavi.edu.br>
TO: <ciencialist@yahoogrupos.com.br>
DATE: 14/12/2004 19:58

Mirko,

Aqui vão alguns links:

http://www.receita.fazenda.gov.br/Legislacao/Leis/2004/lei10865.htm a
lei
http://jbonline.terra.com.br/jb/papel/cadernob/2004/06/03/jorcab20040603
001.html de junho/04
http://www.interlegis.gov.br/20040709090901/comunidade/casas_legislativa
s/federal/c2/20040709090624 de julho/04
http://www1.folha.uol.com.br/folha/dimenstein/noticias/gd040604a.htm de
junho/04

Pelo que li agora, parece que os técnicos foram isentados da taxação de
PIS e COFINS.

[ ]



-----Mensagem original-----
De: Mirko Lerotic Filho [mailto:mleroticf@yahoo.com]
Enviada em: terça-feira, 14 de dezembro de 2004 01:36
Para: ciencialist@yahoogrupos.com.br
Assunto: [ciencialist] Re: importação de livros



Pablo,

Você poderia dar uma referência sobre esse projeto que você citou, de
taxação de importação de livros? Desconheço...

Até onde sei, vale a lei número 10.753, de 30 de outubro de 2003, que
institui a Política Nacional do livro, que diz, em seu artigo número
4:

Art. 4o É livre a entrada no País de livros em língua estrangeira ou
portuguesa, isentos de imposto de importação ou de qualquer taxa,
independente de licença alfandegária prévia.

Peço livros pela Amazon há muitos anos e nunca paguei imposto sobre
importação; o frete, esse sim, pode ficar um pouco caro.

Esse tema já foi discutido aqui na lista. Veja por exemplo a thread:
http://br.groups.yahoo.com/group/ciencialist/message/39325
http://br.groups.yahoo.com/group/ciencialist/message/39326
http://br.groups.yahoo.com/group/ciencialist/message/39327
http://br.groups.yahoo.com/group/ciencialist/message/39319
http://br.groups.yahoo.com/group/ciencialist/message/39320

Saudações,
Mirko Lerotic Filho




--- Em ciencialist@yahoogrupos.com.br, "pablo" <pablo@u...> escreveu
> Prezados,
>
> sei que há ou havia um projeto para se taxar a importação de
livros, e
> que iria sofrer uma modificação para excluir os livros técnicos.
Alguém
> sabe me dizer como ficou esta situação?
> Quero comprar um livro técnico na bn.com ou amazon.com, como o fiz
> algumas vezes até o ano passado. Mas, se irei pagar 60% em cima do
valor
> do livro + frete, desisto.
> E outro questionamento, se excluíram os livros técnicos da taxação
(n
> sei), quem irá e como serão classificados em técnico e não
técnicos? Por
> exemplo quero comprar um livro de nutrição (Krause), será que corro
o
> risco de tê-lo classificado juntamente com aqueles sobre dietas do
Dr.
> Atkins??? Ou, ainda pior, o Atkins seria classificado como técnico?
>
> abraço,
>
> Pablo
>
>
>
> [As partes desta mensagem que não continham texto foram removidas]





##### ##### #####

Para saber mais visite
http://www.ciencialist.hpg.ig.com.br


##### ##### ##### #####



Yahoo! Grupos, um serviço oferecido por:

PUBLICIDADE

<http://br.rd.yahoo.com/SIG=12aiihcqa/M=264379.5078783.6203979.1588051/D
=brclubs/S=2137111528:HM/EXP=1103081773/A=2332652/R=0/id=noscript/SIG=11
9058f8i/*http://br.download.yahoo.com/messenger/>


_____

Links do Yahoo! Grupos


* Para visitar o site do seu grupo na web, acesse:
http://br.groups.yahoo.com/group/ciencialist/


* Para sair deste grupo, envie um e-mail para:
ciencialist-unsubscribe@yahoogrupos.com.br
<mailto:ciencialist-unsubscribe@yahoogrupos.com.br?subject=Unsubscribe>


* O uso que você faz do Yahoo! Grupos está sujeito aos Termos do
Serviço do Yahoo! <http://br.yahoo.com/info/utos.html> .




[As partes desta mensagem que não continham texto foram removidas]



SUBJECT: Antídoto para site porcaria
FROM: Maria Natália <grasdic@hotmail.com>
TO: ciencialist@yahoogrupos.com.br
DATE: 14/12/2004 20:23


E para quem é novo aqui:
http://www.badastronomy.com/
na parte de astronomia e espaço
Boas leituras
Maria Natália





SUBJECT: Re: Site inacreditável
FROM: Maria Natália <grasdic@hotmail.com>
TO: ciencialist@yahoogrupos.com.br
DATE: 14/12/2004 21:26


Alberto:
E agora com a tua resposta já poderei dar o site dos "despassarados
na lua" aos alunos... mas para trabalho interdisciplinar Português/
História/ F e Q na secção de teatro. Sim deste-me ideia para uma
peça de teatro. Vamos ver se os alunos apoiam. Fazem os textos, o
cenário e o guarda roupa. Vai ser interessante e como a ideia veio
daqui se escreverá: Ideia da loja C-List com apoio de A.M Filho.(por
extenso, claro)
Posso usar estas ideias?
Obrigada
Um abraço
Maria Natália
--- Em ciencialist@yahoogrupos.com.br, "Alberto Mesquita Filho"
<albmesq@u...> escreveu
> ----- Original Message -----
> From: "Rodrigo Marques"
> Sent: Tuesday, December 14, 2004 12:46 PM
> Subject: [ciencialist] Site inacreditável
>
> > Pessoal, quem tiver estômago veja este site:
> > http://www.showdalua.com/
> > Aqui o cara defende as inacreditáveis teorias:
>
> > -nunca houve uma viagem à Lua;
>
> Como não?!!! Os físicos "modernos" vivem no mundo da Lua desde o
início do
> século passado.
>
> > -a bomba atômica não existe;
>
> De fato. Isso é invenção de japonês. Em 1945 houve duas epidemias
(uma em
> Hiroshima e outra em Nagasaki) devidas ao "sashimi" estragado e o
governo
> japonês decretou a existência da bomba atômica, botando a culpa nos
> angelicais estadunidenses.
>
> > -a Terra não gira ao redor do Sol;
>
> Isso todo o mundo sabe e faz parte da sabedoria milenar. A Terra
está
> situada sobre um prato dourado apoiado nas costas de vários
elefantes e
> estes equilibram-se sobre o casco de uma tartaruga gigante. Foi a
partir
> dessa constatação científica que surgiu a expressão popular: "É
preciso ter
> um saco de elefante".
>
> Existe uma teoria alternativa e bastante promissora, a dizer que a
Terra é
> ôca e que nós vivemos na superfície interna de uma casca esférica.
O Sol
> estaria situado no centro da esfera e emitiria luz apenas para um
dos lados,
> tal e qual um farol náutico . Os que acreditam nessa teoria são
obrigados a
> assumir ou o giro do farol ou o giro da casca esférica, afim de
explicarem a
> alternância dos dias e das noites.
>
> > -a Terra e todos os planetas tem uma órbita independente;
>
> Nesse caso discordo. A órbita da Terra não é independente, mas
determinada
> por Vixenu, o deus da água e que nada mais é senão a tartaruga
gigante da
> teoria hindu, acima referida. Se a órbita fosse independente não
haveria
> como explicar o dilúvio universal, resultante de um porre da
tartaruga.
>
> > -propõe uma nova teoria da gravidade (a gravidade é uma força
externa que
> > vem da lua e planetas sem lua não tem gravidade)
>
> Essa teoria promete, mas ainda está na fase das especulações. Por
ora é
> melhor acreditar que os corpos evoluem para os seus locais
naturais,
> conforme comprovado cientificamente por Aristóteles.
>
> > -buracos negros não existem;
>
> Essa foi uma hipótese propugnada pelos preconceituosos agentes da
> Ku-Klux-Klan, mas não chegou a vingar.
>
> > -Einstein, Galileu e Copérnico erraram.
>
> Essa eu nem vou comentar pois não conheço nenhum dos três. De
qualquer
> forma, e haja vista que errar é humano, eles parecem estar no bom
caminho.
>
> [ ]´s
> Alberto
> http://ecientificocultural.com/indice.htm
> Mas indiferentemente a tudo isso, o neutrino tem massa, o elétron
não é
> uma carga elétrica coulombiana e a Terra se move. E a história se
repetirá.





SUBJECT: Re: [ciencialist] Re: Site inacreditável
FROM: "Alberto Mesquita Filho" <albmesq@uol.com.br>
TO: <ciencialist@yahoogrupos.com.br>
DATE: 14/12/2004 23:33

Olá Maria Natália

Sem problemas. Sinto-me lisonjeado com isso. Agradeço em meu nome e não
tenho dúvidas de que os demais membros da C-List endossarão o meu
agradecimento, na base do "quem cala consente" [Não há necessidade de se
manifestarem favoravelmente, pois do contrário receberemos mais de 500 msgs
quase idênticas e o Brudna será obrigado a colocar todo o mundo em
moderação].

[ ]´s
Alberto
http://ecientificocultural.com/indice.htm
Mas indiferentemente a tudo isso, o neutrino tem massa, o elétron não é
uma carga elétrica coulombiana e a Terra se move. E a história se repetirá.


----- Original Message -----
From: "Maria Natália" <grasdic@hotmail.com>
To: <ciencialist@yahoogrupos.com.br>
Sent: Tuesday, December 14, 2004 8:26 PM
Subject: [ciencialist] Re: Site inacreditável


Alberto:
E agora com a tua resposta já poderei dar o site dos "despassarados
na lua" aos alunos... mas para trabalho interdisciplinar Português/
História/ F e Q na secção de teatro. Sim deste-me ideia para uma
peça de teatro. Vamos ver se os alunos apoiam. Fazem os textos, o
cenário e o guarda roupa. Vai ser interessante e como a ideia veio
daqui se escreverá: Ideia da loja C-List com apoio de A.M Filho.(por
extenso, claro)
Posso usar estas ideias?
Obrigada
Um abraço
Maria Natália



SUBJECT: Re: [ciencialist] eletron não tão nebuloso
FROM: "Alberto Mesquita Filho" <albmesq@uol.com.br>
TO: <ciencialist@yahoogrupos.com.br>
DATE: 15/12/2004 01:12

Prezado "rayfisica"

Hoje em dia é muito raro eu receber alguma mensagem relacionada a meus
estudos em física e que consiga me comover, e você, com pouquíssimas
palavras, chegou próximo a isso. Em virtude disso, acredito que você mereça
mais do que uma resposta seca a cada um dos seus quesitos principais.

A título de introdução, diria que em 1983, quando comecei a teorizar em
física, senti-me realmente frente a um problema. A partir do "insight" então
ocorrido [que rememorei com poucas palavras na msg 3610 da lista de
discussão de filosofia Acrópolis -- vide
http://ecientificocultural.com/ECC2/Dialogos/varios/ac_3612.htm ] até a
apresentação de um opúsculo, que escrevi em 1992 e intitulado "A equação do
elétron e o eletromagnetismo" (1993), eu não estava mais tão somente frente
a um único problema, mas de posse de uma infinidade de questionamentos
interelacionados: alguns de resposta óbvia, outros absurdos e a grande
maioria a desafiar minha perspicácia, ao mesmo tempo que colocava em xeque
quase tudo o que eu fora obrigado a acreditar através do ensino massificante
pelo qual somos obrigados a passar.

Resumindo, até 1992 eu estava frente a um grande problema (exposto em
dois livros, um publicado em 1985 e outro em 1987) e com a obsessão para
resolvê-los. Em 1993, com a publicação do terceiro livro da série, o
problema deixou de ser meu. É bem verdade que surgiram os problemas-filhos,
como citado por Popper...

" Mas ainda que encontremos uma solução, poderemos
descobrir, para nossa satisfação, a existência de toda uma família de
encantadores, se bem que talvez difíceis, problemas-filhos, para cujo
bem-estar poderemos trabalhar, com uma finalidade em vista, até ao fim dos
nossos dias." [Karl Popper]

... mas a verdade é que a obsessão desapareceu e, sob esse aspecto, sinto-me
um ser bastante realizado e satisfeito comigo mesmo. A sociedade, a esse
respeito, não poderá me cobrar mais nada, mesmo porque realizei esse
trabalho de maneira 100% amadorística. Digo então que o problema deixou de
ser meu, pois se eu tiver um pingo de razão, aqueles que me ignoraram,
enquanto recebiam verbas polpudas para zelar pela aceitação, ou não, de
idéias de nossos compatriotas, hão de ser cobrados por *mais esse* descaso.

Vamos então a sua mensagem.

Fico lisonjeado com a comparação entre a minha teoria e a relatividade, mas
é bom lembrar que essa comparação somente fará sentido "a posteriori" e se,
e somente se, a minha teoria conseguir vingar, coisa que ainda não
aconteceu. A despeito disso, vou efetuar algumas considerações:

Em primeiro lugar, eu diria que o paralelismo que você traçou com a
compreensão da relatividade não é perfeito. O jovem Einstein teve a
felicidade de encontrar pela sua frente um editor de periódico respeitável e
chamado Max Planck, aquele mesmo que, na palavras de Thomas Khun, teria dito
o seguinte:

"Uma verdade científica nova não é geralmente apresentada de maneira a
convencer os que se opõem a ela... simplesmente a pouco e pouco eles morrem,
e nova geração que se forma familiariza-se com a verdade desde o princípio."
[Max Planck]

Você (ou alguém aqui na Ciencialist) conhece algum editor nacional e/ou
avaliador por pares (referee) que pensa desta maneira? E que em virtude
disso seria capaz de avaliar e aceitar para publicação algo em que não
acredita, a exemplo do que Planck fez, a despeito de discordar da
interpretação que Einstein deu ao efeito fotoelétrico?

Em segundo lugar, a frase que você cita refere-se à relatividade geral, algo
que surgiu cerca de uma década após a relatividade restrita já estar
praticamente consagrada.

Em terceiro lugar, o que consagrou a relatividade restrita foi o fato dela
responder a algumas perguntas que até então os físicos não tinham resposta
nenhuma. Os físicos de hoje sequer se deram conta que "estão perdidos num
mato e sem cachorro". Eles estão plenamente satisfeitos com a matemática que
dá certo sem que saibam quando, onde, como e nem porquê.

E em quarto lugar, ambas as relatividades --e essa idéia é citada até mesmo
por Einstein-- vão contra o "senso-comum adotado pelos físicos da época" (e,
sob muitos aspectos, quero crer que até mesmo para os físicos da
atualidade). A minha teoria, pelo contrário, tem a pretensão de devolver a
credibilidade à nossa maneira de perceber ou apreciar a fenomenologia
natural, bem ao estilo newtoniano. Sob esse aspecto, é difícil dizer que
alguém realmente chegou a compreender a teoria da relatividade, pois o
próprio Einstein chegou a chamar a atenção para que não se esforçassem muito
neste sentido. Compreender a teoria da relatividade geral, no sentido
apresentado no texto citado, seria equivalente a perceber que ela está
dotada de uma matemática que dá certo sem que se saiba o porquê, a não ser
assumindo uma recorrência a artifícios matemáticos outros e dotados da mesma
falácia, até chegarmos no princípio da relatividade geral e que, como
princípio, é aceito "a priori".

Vamos agora ao cerne da questão.

É difícil dizer quantas pessoas "realmente" entenderam as minhas idéias. Eu
diria que alguns (poucos) torcem pelo meu sucesso, mas fico com a impressão
de que até o momento ninguém chegou sequer a perceber porque eu acho que
esta seria a única saída para a física da atualidade. Desde os tempos em que
militava na medicina observei que o acadêmico brasileiro dificilmente
valoriza temas nossos e menos ainda idéias nossas. Quando publiquei
aquele opúsculo citado acima, em 1993, recebi um conselho do professor
Newton da Costa, emérito professor do Instituto de Matemática e Estatística
(IME) da Usp e que era mais ou menos assim: "Este livro, editado em
português e aqui, no Brasil, está fadado a não ser lido por alguém em
condições de criticá-lo. Por que você não faz uma síntese e a publica no
exterior?" Como cheguei a concluir posteriormente, "este pensamento não
reflete uma opinião solitária, mas sim uma tendência geral, que é até mesmo
defendida em aulas de cursos de graduação da USP" (esse trecho entre aspas
retirei de um dos meus editoriais).

Eu diria então que é bem provável que praticamente ninguém tenha lido a
minha teoria, pelo menos com a disposição de entender. Uma exceção é o
Chalub que participou de um diálogo no news uol há muito tempo, e creio que
você deve ter lido alguma coisa a respeito, pois a thread tinha o nome que
você escolheu para esta da Ciencialist: Um elétron não tão nebuloso [Se
alguém se interessar, poderá acompanhar esse diálogo a partir de
http://ecientificocultural.com/ECC2/Dialogos/uol1998a.htm ]. Fiquei com a
impressão que ele não entendeu da maneira que seria necessária para a
evolução do diálogo. Alguns chegaram a ler algumas sínteses e/ou alguns
"problemas-filhos", mas não me parece que, além do Chalub, alguém tenha se
esforçado por entender a teoria.

Essa desconsideração para com as minhas idéias eu já flagrei inúmeras vezes
aqui na Ciencialist, e creio ter ficado patente em novembro de 2000, quando
a pedidos de alguns interessados eu me dispus a destrinchar minha teoria
item por item (vide
http://ecientificocultural.com/ECC2/Dialogos/dsfcvm2.htm ). Ninguém aqui
parece querer arcar com o ônus de ter de repensar a física.
Conseqûentemente, ninguém está interessado em conhecer as minhas idéias,
embora queiram que eu fique sempre repisando no que penso a respeito da
física "moderna". Quando digo o que penso, atendendo a esses insistentes
apelos, eles se ofendem e ficam balbuciando coisas sem sentido.
Eventualmente conseguem se fazer entendidos, e não é raro receberem o troco
devido, ocasião em que acabam "enfiando o rabo entre as pernas", pois jamais
esperavam que alguém pudesse dar uma resposta para coisas que aprenderam na
universidade que não tinha resposta nenhuma [explicar um efeito
aparentemente quântico por um raciocínio 100% clássico].

Em 1992/3 eu solicitei a opinião de muitos eméritos professores da Usp.
Quase todos receberam-me muito bem e acredito que mal saí de suas salas
devem ter jogado meu trabalho na cesta de lixo. Um desses eméritos
professores foi mais delicado e chamou um de seus orientandos de
pós-graduação, solicitando ao mesmo que lê-se a teoria para posterior
discussão. Passado um certo tempo esse orientando começou a me evitar, e eu
logo adivinhei o motivo e parei de procurá-lo.

Em 1994 ou 1995 fui convidado pelo Secretário da Cultura de Bauru para expor
minha teoria numa palestra da Unesp dessa cidade. Soube pelo Secretário que
os professores de física da Unesp foram PROIBIDOS de comparecer ao evento.
Com muita insistência o Secretário conseguiu convencer o Reitor a que
preenchesse a platéia com alunos da Faculdade de Física, ao lado de um dos
professores que foi escolhido (quiçá sorteado) para ouvir as heresias que eu
tinha para apresentar. Essa proibição me foi relatada pelo próprio
Secretário da Cultura, que lamentou bastante o acontecido e quero crer que
sequer entendeu o que estava a acontecer nos bastidores do Olimpo Acadêmico.

Que mais dizer?

[ ]´s
Alberto
http://ecientificocultural.com/indice.htm
Mas indiferentemente a tudo isso, o neutrino tem massa, o elétron não é
uma carga elétrica coulombiana e a Terra se move. E a história se repetirá.



----- Original Message -----
From: "rayfisica"
Sent: Tuesday, December 14, 2004 4:04 PM
Subject: [ciencialist] eletron não tão nebuloso


Alberto
Li acho que nos três primeiros minutos ou no universo elegante, o
seguinte:
`O repórter pergunta a um estudioso, sobre o fato de apenas três
pessoas compreenderem a teoria da relatividade, dizem que após pensar
um pouco o cientista responde - quem poderia ser a terceira pessoa'.
Acho que dá pra entender o que estou tentando passar.
Pergunto lhe:
TODAS AS PESSOAS QUE TE CRITICAM REALMENTE ENTENDERAM SUAS IDEIAS OU
ALGUMAS ESTÃO APENAS REPETINDO O PENSAMENTO VIGENTE?
QUANTO POR CENTO DAS PESSOAS QUE TIVERAM CONTATO COM SUA TEORIA A
ENTENDEU?
Estou lendo o seu trabalho, depois volto a tc para ti.



SUBJECT: Re: Seria Eintein uma farsa? - entrevista com César Lattes
FROM: Manuel Bulcão <manuelbulcao@uol.com.br>
TO: ciencialist@yahoogrupos.com.br
DATE: 15/12/2004 01:40


--- Em ciencialist@yahoogrupos.com.br, "JVictor" <jvoneto@u...>
escreveu

> Quem quiser pode ler, mas trata-se do mesmo lenga lenga. Depois
não digam que não avisei.

Manuel: Nessa entrevista, o César Lattes afirma:... A Teoria da
Relatividade não é invenção dele [do Einstein]. Já existe há
séculos. Vem da Renascença, de Leonardo Da Vinci, Galileu e Giordano
Bruno. Ele não inventou a relatividade.

Manuel: Pois eu vou ainda mais longe: esse estória de que o espaço-
tempo é curvo é uma idéia que remonta aos Eleatas: Parmênides e
Zenão afirmavam que o Ser é esférico e que a sensação de que o tempo
flui não passa de uma ilusão. A conclusão, portanto, é que Einstein
plagiou Parmênides e seus discípulos.

Claro que não concordo com isso (depois explico por que não
concordo).

Esse tipo de crítica que fazem ao físico alemão, alguns céticos
porretas (que acham cheirosíssimos todos os peidos do Einstein)
também o fazem a Freud.

Dois pesos, duas medidas.

Depois que ficou provada a consistência da hipótese do inconsciente,
agora afirmam que a idéia do inconsciente não é do Freud: que,
muuuuuuuuuuuuuuuuuito antes dele, vários literatos e filósofos já
especulavam sobre a existência de um inconsciente.

Estranhamente, esses cientificistas não conseguem perceber as
diferenças que existem entre uma idéia literária, uma conjuntura
filosófica e uma hipótese científica.

Mas só não percebem quando não lhes convém perceber. Desonestidade.

Ósculos,
Manuel Bulcão





SUBJECT: O Ensino da Física nas Escolas
FROM: Maria Natália <grasdic@hotmail.com>
TO: ciencialist@yahoogrupos.com.br
DATE: 15/12/2004 03:52


Vai decorrer na Universidade do Minho em 21 de Janeiro de 2005
É o melhor modo de se começar o Ano da Física: preocupando-nos com o
que se anda a ensinar nas escolas aos moços desde os 11 aos 18 anos,
7º ao 12º anos. E uma vez que o currículo é Nacional...vais sair
dali documento. Só que Braga é longe e serão dois dias e com tudo
pago de nosso bolso...
Vejam o programa em
http://www.spf.pt/ensino_fisica_escolas.pdf
Maria Natália






SUBJECT: TIMSS e PIRLS, International Study Center
FROM: Maria Natália <grasdic@hotmail.com>
TO: ciencialist@yahoogrupos.com.br
DATE: 15/12/2004 03:58


E me parece que nos EUA quem está a ter melhores classificações não
são os americanos...
http://www.timss.org/
E por aqui também se pode saber o que é isso do TIMSS e do futuro
PIRLS
desculpe Leo
Um abraço
Maria Natália






SUBJECT: Re: TIMSS e PIRLS, International Study Center
FROM: Maria Natália <grasdic@hotmail.com>
TO: ciencialist@yahoogrupos.com.br
DATE: 15/12/2004 04:07


Mas pode ler aqui in english:

"TIMSS 2003: Eighth-Grade Performance Up, Fourth-Grade Scores Flat
2004-12-14 - NSTA Reports

American eighth graders - especially African American students -
showed significant gains in both science and math over the last
eight years, while scores for US fourth graders remained relatively
flat in both subjects, says the 2003 Trends in International Math
and Science Study - commonly known as TIMSS - which was released on
December 14.

US Eighth-grade Scores Show Significant Improvement in Science

The 2003 TIMSS assessed the mathematics and science achievement of
half a million students in 49 countries at either the fourth- or
eighth-grade levels. In 2003, the United States was one of 13
countries that demonstrated a significant increase in science
achievement at the eighth grade and was the sole nation of the
participating G-8 countries that showed improvement in science at
this grade level. US eighth graders scored 12 points higher on the
science portion (527 on average) than in 1999 and 15 points higher
than in 1995.

In 2003, the United States ranked ninth out of 45 countries in
student science achievement at the eighth grade. US students scored
lower than students from consistently high-scoring countries, such
as Singapore, Japan, and Korea.

American boys and girls showed significant improvement in eighth-
grade science scores, and gains for minority groups were also
impressive: The average score for African American students was 40
points higher than in 1995, while Hispanic students posted a 36-
point increase since 1995.

"Although much more work needs to be done to further narrow the
achievement gap, the important strides made at the eighth-grade
level can be attributed to work by science teachers, scientists, and
other stakeholders over the past decade to implement the National
Science Education Standards. These standards present a clear vision
of what our students are expected to know and be able to do in
science at all grade levels," said NSTA Executive Director Gerald
Wheeler. "Most states have successfully incorporated the national
standards into state-specific benchmarks, and it appears that
efforts by our teachers and others to adapt standards-based learning
goals into middle-level classroom instruction are working."

Average Science Scores for US Fourth Graders Flat Since 1995

Although US fourth graders came in above international averages in
the 1995 TIMSS, 2003 TIMSS results suggest fourth-grade scores have
not changed much in the past eight years.

In 1995, American fourth graders scored 542 on TIMSS. In 2003, that
average score fell to 536, a decline considered not statistically
significant when compared with 1995. In 1995, only fourth graders in
Japan outperformed US fourth graders, and US students scored higher
their peers in 13 countries. In 2003, US fourth graders were
outperformed by students in two countries (Singapore and Japan), and
they scored higher than students in eight countries.

Both boys and white students at this grade level scored lower in
2003. Although African American fourth graders posted a 25-point
increase since 1995, scores for Hispanic fourth graders remained
largely flat. The gender gap narrowed slightly in the fourth grade.

US Students Show Gains in Eighth-grade Math; Fourth-grade Math
Scores Flat

The average scores of US fourth and eighth graders on the
mathematics portion of TIMSS virtually mirror the science
achievements posted by students.

In both 2003 and 1995, US fourth graders scored 518 on average in
mathematics. Overall, the standing of US fourth graders relative to
their peers in other counties was lower in 2003 than in 1995. While
African American students posted gains, scores for Hispanic fourth
graders remained flat.

US eighth graders, however, showed a significant improvement in
mathematics performance between 1995 and 2003, with scores up 12
points during this time. Both boys and girls showed improvements
since 1995, as did African American students and Hispanic students,
with gains of 29 points and 22 points respectively.

This is the third TIMSS comparison of math and science achievement
worldwide since 1995. The TIMSS 2003 used nationally representative
samples of fourth and eighth graders in all countries. In the United
States, 248 schools and 10,795 students participated in the fourth-
grade assessment, and 232 schools and 8,912 students participated in
the eighth-grade assessment. The assessment entailed a 72-minute
paper-and-pencil assessment at grade four and a 90-minute assessment
in grade eight, and it included questionnaires for students,
administrators, and teachers.

Media outlets, policy makers, and others often cite the TIMSS study
in discussions of science and math achievement. To read the entire
study, go to www.timss.org. See page 19 for additional charts that
accompany this article.""
Maria Natália









--- Em ciencialist@yahoogrupos.com.br, Maria Natália <grasdic@h...>
escreveu
>
> E me parece que nos EUA quem está a ter melhores classificações
não
> são os americanos...
> http://www.timss.org/
> E por aqui também se pode saber o que é isso do TIMSS e do futuro
> PIRLS
> desculpe Leo
> Um abraço
> Maria Natália





SUBJECT: Mergulho final e fatal dia 15 de Dezembro!
FROM: Maria Natália <grasdic@hotmail.com>
TO: ciencialist@yahoogrupos.com.br
DATE: 15/12/2004 05:58


E a chegada de Huygens a Titã está para breve. Veja se tem hipótes
de assistir via TV. Pessoal de telecomunicações não consegue?:

"The next transmission of the ESA TV Service will be:
15-Dec-04 10:00 - 10:15 GMT
Huygens Ready for the Final Plunge
ESA TV Exchanges

Background information on the transmission:
Replay I: 15 December 15:00-15:15 GMT

Replay II: 18 December 11:30 -11:45 GMT

Europe's Huygens probe has covered billions of kilometres, during a
seven-year flight from Earth to Saturn. Now, the moment of the
release of
Huygens onto a trajectory to Titan is approaching, and scientists
and
engineers need maximum confidence in the readyness of the space
probe
for its short but critical mission.

Today's programme provides a detailed overview of the tests that
were
performed and the safeguards built into the system to maximise
chances
for misison success. It includes images from a recent check-out at
ESOC,
where the functioning of Huygens systems was tested.

The script is online under
http://television.esa.int/photos/ESATV15122004.pdf

Transmission of this Exchange on 15 December is not on EbS (because
of
the EuroTop) but on Eutelsat W1 - for the transmission details, see
below.

The second replay on 18 December will be on EbS - for the
transmission
details, see the ESA TV Website.

More background information can be found on
www.esa.int/SPECIALS/Cassini-Huygens.

Transmission details:On 15 December only:

Eutelsat W1 at 10 degrees east, Transponder B4 Middle 1 (SCPC),
vertical, F=11079 MHz, SR=5.632 MS/sec, FEC=3/4

For further information, visit our website at
http://television.esa.int. For other enquires, contact Claus
Habfast, Tel +31 71 565 3838, Fax
+31 71 565 6340, e-mail claus.habfast@esa.int."
Maria Natália





SUBJECT: Re: Telescópios da Nasa descobrem sistema solar em formação
FROM: Maria Natália <grasdic@hotmail.com>
TO: ciencialist@yahoogrupos.com.br
DATE: 15/12/2004 06:20


Amauri:
Não me esqueci de tua dúvida e seguiu para investigador.
Entrtanto tens aqueles sites em protuguês.
Um abraço
Maria Natália

--- Em ciencialist@yahoogrupos.com.br, "Amauri Jr"
<amaurijunior2@y...> escreveu
> Maria
>
> Minha duvida é: há pesquisas que mostram que tudo que se originou
a vida veio da poeira cósmica que algumas estrelas mortas deixaram,
por que esta procura?
>
> Amauri
> ----- Original Message -----
> From: Maria Natália
> To: ciencialist@yahoogrupos.com.br
> Sent: Sunday, December 12, 2004 8:44 PM
> Subject: [ciencialist] Re: Telescópios da Nasa descobrem sistema
solar em formação
>
>
>
> Amauri:
> Se existe um sistema solar como o nosso, existe um 4º calhau a
> contar da estrela central ou um 5º. Tudo depende da estrela.
> Portanto posso pensar em condições semelhantes á da Terra. Um
> planeta que estará na mesma situação da que a Terra viveu. Toda
a
> história se repetiria.A probabilidade 50% de existir ou não. Mas
> como não sou bióloga nem antropocêntrica...
> Mas quando falo de vida até essa com suporte não em C, carbono,
> penso. Penso em vida com suporte silício, Si, que também origina
> compostos de cadeia. Mas em que vida é diferente da da Terra.
Sem
> água mas amónia por exemplo. E Vida para mim não é só a
inteligente.
> um virus e uma bactéria me servem como vida. E te lembra que
essa
> vida já existe em Marte.
> Para mim a noção de vida é mais ampla: é haver a hipótese de
> transmissão de um código. E não será transmissão de código o que
se
> faz em feixes ópticos de compostos de Si? Tudo o que se tem
> aprendido em ciência de materiais? Em nano estruturas?
> Não, não leio romances FC. Mas isto até já foi tratado por
pessoal
> de Ciência em ficção científica.
> Para mim a vida cria-se num tubo de ensaio e nem tenho de me
> preocupar com a alma. O químico é um bocado mais frio que um
biólogo
> e se ele for de inorgânica, está tudo dito.
> Sobre outras formas de vida tenho muito mas é em inglês de
> discussões noutros listas americanas. O site honesto é:
> http://www.universetoday.com e nela tens de ir aos foruns onde
mesmo
> sem estares registado podes busacar a sala de discussão
> E outra página mas que deves ler como ficção é esta pois eu com
> ufologia...hum hum
> http://www.gpua.ubbi.com.br/artigos/vida_outros_planetas.htm
> A 2ª vai porque está em português
> E deum modo mais geral e sobre astronomia em português tens este
bom:
> http://www.on.br/pergunte_astro/indice_tema.php
> e ainda abrangendo questões de física:
> http://www.astro.iag.usp.br/perguntas.html
> E creio te ter esclarecido sob o ponto de vista da química
> Um abraço
> Maria Natália
>
> --- Em ciencialist@yahoogrupos.com.br, "Amauri Jr"
> <amaurijunior2@y...> escreveu
> > Oi Natalia
> >
> > O que tem haver descobrir vida em outro sistema solar e
conhecer
> ciência? Ora, posso analisar isto como uma grande base para o
> progresso de descobertas do sentindo de descobrir vida. Mas a
> ciência ao mesmo tempo, tem que analisar que seres vivos também
> podem respirar e viver de formas diversas, como a pouco
descobriram
> bactérias que respiram gás metano.
> >
> > Bjs
> > Amauri
> > ----- Original Message -----
> > From: Maria Natália
> > To: ciencialist@yahoogrupos.com.br
> > Sent: Saturday, December 11, 2004 6:05 PM
> > Subject: [ciencialist] Re: Telescópios da Nasa descobrem
sistema
> solar em formação
> >
> >
> >
> > E a tradução é de confiança?
> > Esta descoberta vem mostrar que pode haver Vida como a nossa
ou
> > diferente. Mas nunca "eles estão entre nós". Continuo a
pensar
> que
> > as dificuldades de contacto são de cerca de 99.9789% de tal
> facto se
> > vir a dar.
> > E que interesse teriam eles em conhcer pessoal mais atrasado
e
> que
> > ainda não se entende nem na comunidade científica? daqui a
> 300000
> > talvez...
> > Uma abraço
> > Maria Natália
> >
> >
> >
> > --- Em ciencialist@yahoogrupos.com.br, "Oraculo"
<oraculo@a...>
> > escreveu
> > > Telescópios da Nasa descobrem sistema solar em formação
> > >
> > > http://noticias.terra.com.br/ciencia/interna/0,,OI436946-
> > EI301,00.html
> > >
> > > Dois observatórios espaciais da Nasa descobriram um
sistema
> solar
> > em plena formação, que inclui meteoritos e pó cósmico em
torno
> de
> > estrelas do tamanho do Sol, informou hoje o Laboratório de
> Propulsão
> > a Jato (JPL). O JPL afirmou em comunicado que o
descobrimento
> foi
> > feito de maneira conjunta pelos telescópios Spitzer e
Hubble,
> que
> > operam em órbita em torno da Terra.
> > > Trata-se da imagem mais clara de que se tem notícia das
fases
> > iniciais da formação de planetas, segundo o laboratório da
> Agência
> > de Aeronáutica e o Espaço dos EUA (Nasa) em Pasadena, na
> Califórnia.
> > >
> > > Os planetas nascem da união de discos de pó e gás
concentrados
> em
> > torno de uma estrela. As imagens são uma espécie de
recriação da
> > evolução do nosso Sistema Solar, precisou a Nasa.
> > >
> > > Segundo Charles Beichman, um astrônomo do JPL, o Hubble
> descobriu
> > a estrela central, de aproximadamente 50 a 25 milhões de
anos, e
> o
> > Spitzer, uma grande nuvem de poeira, "o que mostra que os
> > telescópios podem se complementar". "O Spitzer estabeleceu
pela
> > primeira vez um vínculo direto entre os planetas e os
discos.
> Agora
> > podemos estudar essa relação", disse.
> > >
> > > "A nova imagem do Hubble nos permite lançar o melhor olhar
> > possível sobre a luz refletida de um disco que gira em torno
da
> > massa de um sol", afirmou David Ardila, diretor do estudo e
> > astrônomo da Universidade Johns Hopkins.
> > >
> > > O disco de escombros captado pelo Hubble foi identificado
como
> HG
> > 107146 e fica a uma distância de 88 anos luz da Terra.
> > >
> > >
> > > [As partes desta mensagem que não continham texto foram
> removidas]
> >
> >
> >
> >
> >
> > ##### ##### #####
> >
> > Para saber mais visite
> > http://www.ciencialist.hpg.ig.com.br
> >
> >
> > ##### ##### ##### #####
> >
> >
> > Yahoo! Grupos, um serviço oferecido por:
> >
> >
> >
> >
> >
> >
> >
> > ---------------------------------------------------------------
----
> -----------
> > Links do Yahoo! Grupos
> >
> > a.. Para visitar o site do seu grupo na web, acesse:
> > http://br.groups.yahoo.com/group/ciencialist/
> >
> > b.. Para sair deste grupo, envie um e-mail para:
> > ciencialist-unsubscribe@yahoogrupos.com.br
> >
> > c.. O uso que você faz do Yahoo! Grupos está sujeito aos
> Termos do Serviço do Yahoo!.
> >
> >
> >
> > [As partes desta mensagem que não continham texto foram
removidas]
>
>
>
>
>
> ##### ##### #####
>
> Para saber mais visite
> http://www.ciencialist.hpg.ig.com.br
>
>
> ##### ##### ##### #####
>
>
> Yahoo! Grupos, um serviço oferecido por:
>
>
>
>
>
>
>
> -------------------------------------------------------------------
-----------
> Links do Yahoo! Grupos
>
> a.. Para visitar o site do seu grupo na web, acesse:
> http://br.groups.yahoo.com/group/ciencialist/
>
> b.. Para sair deste grupo, envie um e-mail para:
> ciencialist-unsubscribe@yahoogrupos.com.br
>
> c.. O uso que você faz do Yahoo! Grupos está sujeito aos
Termos do Serviço do Yahoo!.
>
>
>
> [As partes desta mensagem que não continham texto foram removidas]





SUBJECT: Células de sangue transferirndo momento angular
FROM: Maria Natália <grasdic@hotmail.com>
TO: ciencialist@yahoogrupos.com.br
DATE: 15/12/2004 06:30


E a vêm a caminho uns biosensores e micro máquinas:

"Red blood cells are go! (Dec 14)
http://physicsweb.org/article/news/8/12/8
Physicists in India have shown that red blood cells can transfer
the angular momentum in a circularly polarized laser beam into
rotational motion. The "motor" developed by Deepak Mathur and
colleagues at the Tata Institute of Fundamental Research (TIFR) in
Mumbai could find use in a variety of applications, including
biosensors and cellular micromachines (J A Dharmadhikari et al. 2004
Appl. Phys. Lett. 85 6048)."
Maria Natália






SUBJECT: Re: Site inacreditável p/ rosevena3
FROM: "rosevena3" <rosevena3@yahoo.com.br>
TO: ciencialist@yahoogrupos.com.br
DATE: 15/12/2004 07:51


--- Em ciencialist@yahoogrupos.com.br, "Oraculo" <oraculo@a...>
escreveu
> Olá rosevena3
>

>
> E o dito "toda unanimidade é burra" deve ser aplicado a opiniões,
não a toda afirmação ou conhecimento. A unanimidade conhecida como
ciência (ou o conhecimento por ela produzida, que tende a se tornar
unanime) não deriva de opiniões mas de conclusões embasadas. E pode
(e deve) sempre mudar, claro, mas exige, para tanto, que o motivo da
mudança siga algumas regras rígidas, conhecidas como método
cientifico. E a mudança, na maioria absoluta das vezes, é mais um
ajuste, uma melhora, que uma refutação ou exclusão.
>

Agradeço, pois tenho um enorme prazer por me permitirem estar aqui.
É lógico que a finalidade é provocar e, portanto volto a frisar:
Toda unanimidade é burra e não que são burros os que pensão de forma
unânime, é que depois de algum tempo o conhecimento por ser unânime
vira crença, ai meu amigo pra economizar tempo a gente engole.
Por exemplo, o éter universal nos 800s.



> Claro que não é possível, para nós leigos e cidadãos comuns, testar
TODA afirmação ou teoria que se apresenta. Assim, uma escolha sábia
(e não burra), seria decidir acompanhar as alegações e conhecimentos
produzidos por um sistema que tem se mostrado confiável a bastante
tempo, a ciência. Olhe para os lados o que vê? Desdobramentos
confiáveis desse conhecimento. Aviões, computadores, remedios,
espectativa de vida de mais de 80 anos, pouca mortalidade infantil, e
esta conversa entre nós dois.
>

Ah a ciência, quem me dera se tivesse permanecido pura, porem caiu na
mão dos homens. (hahaha)
Exatamente por ser muito difícil se não impossível a nós mortais
conferirmos o dito, é que devemos ficar atento.
Dou dois exemplos;
• O santo sudário é verdadeiro ou falso. O método cientifico
levou a duas conclusões diferentes a respeito dele, de acordo com
interesses dos cientistas que utilizaram ele, o método é ferramenta e
não parâmetros rígidos.
• O neutrino, é tão dispendioso o experimento que detecta-o,
que não há exatamente a repetitividade que apregoa o próprio método
cientifico, estamos depositando nossa crença em poucas pessoas, e ó,
por um Nobel...
Nesses dois exemplos, a ciência esta na mão de pessoas, é o poder que
determina o método e não o contrario.
Por exemplo, lembram se da fusão a frio, era tão simples que no dia
seguinte até as universidades brasileiras estavam realizando o
experimento.




> Onde colocaria, digamos, seu dinheiro, se tivesse de apostar em uma
delas?..:-)
>

já ouvi falar nisso

> Um abraço.
>
> Homero
>
>
>
idem





SUBJECT: Re: Site inacreditável
FROM: Hélio Ricardo Carvalho <hrc@fis.puc-rio.br>
TO: ciencialist@yahoogrupos.com.br
DATE: 15/12/2004 10:59



Esta teoria não é do Andre Mauro mas de um tal de Davino Servídio
que o Andre Mauro escutou e engoliu direto. Andre Mauro é cineastra,
não é cientista (Davino também não). Pelo que narra na página me
parece que (na melhor das hipóteses) isto foi um erro de
interpretação dos dados observacionais cometidos pelo Sr. Davino e
que o Andre Mauro engoliu sem questionamento. (?????)

Ele usa como teste as populares "três marias" que fazem parte da
constelação chamada Orion que não estão na linha do "zodíaco" mas
estão mais para o sul (quando vistas do equador). No início da
primavera e do outono a inclinação de ~23o da Terra não é obstáculo
para a visão de estrelas mais ao sul ou mais ao norte celeste. Ele
pode ter se confundido na hora de procurar as três marias.
Não fiz o teste para ver se ele disse a verdade sobre as
observações. Mas estou só supondo que ele (Davino) não usou de má fé
mas sinplesmente errou por ignorancia na hora de interpretar as
observações.


Hélio

Ps.: Eu não sou astrônomo amador (e não tive tempo para rodar
programas de mapa do céu) e posso ter errado algo aqui. Maria
Natália, me socorra.


--- Em ciencialist@yahoogrupos.com.br, "rosevena3" <rosevena3@y...>
escreveu
>
> Ola.
> Com licença de por minha colher nesse angu.
> Só vi a parte que trata da terra não girar em torno do sol, não
> entendi nada pois não sou entendido no assunto, então pergunto:
> O CARA PODE TER ALGUMA RAZÃO, OU É LOUCO MESMO?
> Para mim seria muito fácil ficar com a maioria, mas penso que toda
> unanimidade é burra, e eu tenho que entender o cara para poder
> julgar, e confesso que não entendo nada para poder saber PORQUE A
> CONSTELAÇÃO DEVERIA ESTAR EM UM SÓ PONTO UMA VEZ SÓ POR ANO?
> ALGUEM PODE EXPLICAR A LOGICA DO FULANO SE É QUE TEM ALGUMA,
> GRATO





SUBJECT: Re: [ciencialist] Re: um paradoxo e dois bobos, sera?
FROM: "Luiz Ferraz Netto" <leobarretos@uol.com.br>
TO: <ciencialist@yahoogrupos.com.br>
DATE: 15/12/2004 12:11

Vamos ao texto abaixo:
===========================
Luiz Ferraz Netto [Léo]
leobarretos@uol.com.br
http://www.feiradeciencias.com.br
===========================
"rosevena3"

Solicito permissão para pedir que alguém me explique o
seguinte,

"NUMA ESFERA CARREGADA, SENDO O POTENCIAL CONSTANTE EM QUALQUER PONTO INTERNO E A SUPERFICIE,

Léo: Atenção --- essa afirmação só será verdadeira após restabelecido o equilíbrio, ou seja, o potencial eletrostático só será constante (conforme se afirma nessa primeira parte) após estabelecido o equilíbrio de cargas.

PORQUE ENTÃO A CARGA ELETRICA EM EXCESSO NÃO
PODE ESTAR EM QUALQUER PONTO,

Léo: Existe uma fase transitória entre os estados inicial "esfera neutra" e final "esfera eletrizada e em equilíbrio de cargas". Nessa fase transitória podemos discernir as etapas de 'eletrização e busca do equilíbrio'. Assim que for colocada "a segunda carga em excesso" na esfera, elas se repelem (forças coulombianas) e se afastam o máximo possível, na matéria condutora, o mais na 'periferia' possível --- daí nasce a pressão eletrostática na superfície.
Assim, na fase de eletrização há um contínuo movimento de cargas até a obtenção do equilíbrio eletrostático final --- ai sim podemos afirmar que o potencial elétrico é o mesmo em todos os pontos da esfera (superfície e interior --- mesmo nos pontos da parte oca onde não há material algum!).

(POTENCIAL CONSTANTE NÃO É COMO
EQUILIBRIO) MAS APENAS NA SUPERFICIE DO CONDUTOR? É UMA
CONVENÇÃO?

Léo: Não há convenção alguma; as cargas realmente, na fase transitória, deslocam-se sob ação de forças coulombianas e se afastam o máximo possível uma das outras. A tendência seria mesmo abandonarem o corpo. Depois sossegam ... é o equilíbrio em relação a um referencial ligado ao corpo.

[]'


SUBJECT: Re: [ciencialist] Re: Site inacreditável
FROM: "Luiz Ferraz Netto" <leobarretos@uol.com.br>
TO: <ciencialist@yahoogrupos.com.br>
DATE: 15/12/2004 12:32

Olá Rosevena3,

O problema todo é que "não é a Terra que gira ao redor do Sol" e sim "é o SOL que gira ao redor da TERRA"; todo mundo pode constatar isso, todos os dias.

O 'cara' não é louco, apenas não tem razão! Assim como muitos outros, é um 'cara' de apenas 'meias-verdades'; assim como meus professores da 5a série que me ensinaram que a "Terra atrai a Lua" --- nunca me ensinaram a outra metade da 'verdade'.

Esse pessoal gosta muito de esconder as verdades, pois elas são 'doídas'. Veja, dizem que a órbita da Terra é uma elipse. Ora, elipse é uma curva fechada e a trajetória da Terra ao redor do Sol não é fechada (jamais a Terra passará duas vezes pelo mesmo ponto!), pois o Sol se desloca em direção à VEGA, arrastando consigo todos os seus filhotes. Se a Terra nunca poderá estar no mesmo lugar, nós, provisórios moradores da superfície, nunca poderemos ver qqer coisa (constelações, por exemplo) no 'céu', duas vezes, no mesmo ponto.
Ponto final.

aquele abraço,


===========================
Luiz Ferraz Netto [Léo]
leobarretos@uol.com.br
http://www.feiradeciencias.com.br
===========================
-----Mensagem Original-----
De: "rosevena3" <rosevena3@yahoo.com.br>
Para: <ciencialist@yahoogrupos.com.br>
Enviada em: terça-feira, 14 de dezembro de 2004 16:35
Assunto: [ciencialist] Re: Site inacreditável




Ola.
Com licença de por minha colher nesse angu.
Só vi a parte que trata da terra não girar em torno do sol, não
entendi nada pois não sou entendido no assunto, então pergunto:
O CARA PODE TER ALGUMA RAZÃO, OU É LOUCO MESMO?
Para mim seria muito fácil ficar com a maioria, mas penso que toda
unanimidade é burra, e eu tenho que entender o cara para poder
julgar, e confesso que não entendo nada para poder saber PORQUE A
CONSTELAÇÃO DEVERIA ESTAR EM UM SÓ PONTO UMA VEZ SÓ POR ANO?
ALGUEM PODE EXPLICAR A LOGICA DO FULANO SE É QUE TEM ALGUMA,
GRATO




SUBJECT: Blog com dicas de sites (sobre ciência)
FROM: Luis Brudna <luisbrudna@gmail.com>
TO: ciencialist@yahoogrupos.com.br
DATE: 15/12/2004 13:47

Estou tentando manter um blog com dicas de sites sobre ciência. O
objetivo é ampliar a idéia para algo mais completo, mas por enquanto é
o que consigo fazer com o site hospedado gratuitamente.

O endereco eh
http://antiparticula.webcindario.com

Um dos problemas do script WordPress (para fazer blogs) é que as
categorias começam a ficar com muitos ´posts´ e não tem como paginar
automaticamente. :-( Vou ter que encontrar outra plataforma.

Até
Luís Brudna


SUBJECT: Re: [ciencialist] Re: Site inacreditável
FROM: Luis Brudna <luisbrudna@gmail.com>
TO: ciencialist@yahoogrupos.com.br
DATE: 15/12/2004 13:52

O quão ´não-circular´ (ou elíptica :-) ) é a órbita da Terra? Qual é
o motivo disso? (pergunta de repórter, se fazendo de bobo pra
aproveitar a genialidade do entrevistado ;-) )

Até
Luís Brudna

On Wed, 15 Dec 2004 12:32:59 -0200, Luiz Ferraz Netto
<leobarretos@uol.com.br> wrote:
> Esse pessoal gosta muito de esconder as verdades, pois elas são 'doídas'. Veja, dizem que a órbita da Terra é uma elipse. Ora, elipse é uma curva fechada e a trajetória da Terra ao redor do Sol não é fechada (jamais a Terra passará duas vezes pelo mesmo ponto!), pois o Sol se desloca em direção à VEGA, arrastando consigo todos os seus filhotes.
> Luiz Ferraz Netto [Léo]
> leobarretos@uol.com.br
> http://www.feiradeciencias.com.br


SUBJECT: Re: Site inacreditável
FROM: "Rodrigo Marques" <rodmarq72@yahoo.com.br>
TO: ciencialist@yahoogrupos.com.br
DATE: 15/12/2004 14:31


Aproveito também para perguntar sobre esse movimento do Sol em
direção à Vega. Como ele é? O Sol vai se chocar com ela?
--- Em ciencialist@yahoogrupos.com.br, Luis Brudna <luisbrudna@g...>
escreveu
> O quão ´não-circular´ (ou elíptica :-) ) é a órbita da Terra?
Qual é
> o motivo disso? (pergunta de repórter, se fazendo de bobo pra
> aproveitar a genialidade do entrevistado ;-) )
>
> Até
> Luís Brudna






SUBJECT: Re: eletron não tão nebuloso
FROM: Hélio Ricardo Carvalho <hrc@fis.puc-rio.br>
TO: ciencialist@yahoogrupos.com.br
DATE: 15/12/2004 15:02


Oi Grande Alberto,

--- Em ciencialist@yahoogrupos.com.br, "Alberto Mesquita Filho"
<albmesq@u...> escreveu
> Prezado "rayfisica"
>
> Hoje em dia é muito raro eu receber alguma mensagem relacionada a
meus
> estudos em física e que consiga me comover, e você, com
pouquíssimas
> palavras, chegou próximo a isso. ...
> ...
> ...
> ... Vamos agora ao cerne da questão.
>
> É difícil dizer quantas pessoas "realmente" entenderam as minhas
idéias. Eu
> diria que alguns (poucos) torcem pelo meu sucesso,


Eu! Pode me colocar na lista dos que torcem!


>...mas fico com a impressão
> de que até o momento ninguém chegou sequer a perceber porque eu
acho que
> esta seria a única saída para a física da atualidade.
> ...
> ...
> ... Eu diria então que é bem provável que praticamente ninguém
tenha lido a
> minha teoria, pelo menos com a disposição de entender. Uma exceção
é o
> Chalub que participou de um diálogo no news uol há muito tempo, e
creio que
> você deve ter lido alguma coisa a respeito, pois a thread tinha o
nome que
> você escolheu para esta da Ciencialist: Um elétron não tão
nebuloso [Se
> alguém se interessar, poderá acompanhar esse diálogo a partir de
> http://ecientificocultural.com/ECC2/Dialogos/uol1998a.htm ].


Eu estava lendo com disposição de entender mas parei por uma série
de motivos.
Começamos a trocar msg em pvt e, confesso, falei mais das minhas
heresias e quase nada das suas :-).
Espero poder, no início do ano que vem, recomeçar um diálogo
contigo. Primeiro falando (e criticando) das suas teorias.


> ...
> ...
> ...
> Essa desconsideração para com as minhas idéias eu já flagrei
inúmeras vezes
> aqui na Ciencialist, e creio ter ficado patente em novembro de
2000, quando
> a pedidos de alguns interessados eu me dispus a destrinchar minha
teoria
> item por item (vide
> http://ecientificocultural.com/ECC2/Dialogos/dsfcvm2.htm ).


Em nov. de 2000 eu ainda não estava na lista :-).

Seu fã (não duvide disto)
Hélio






SUBJECT: Química aqui e lá
FROM: "brudna" <lrb@iq.ufrgs.br>
TO: ciencialist@yahoogrupos.com.br
DATE: 15/12/2004 15:50


O quão enfáticos podemos ser quanto as propriedades dos elementos da
tabela periódica?

Pensei nisso ao ler o comentário no ´Jorna da ciência´

--início -
Leitora comenta artigo `Presença de água em Marte no passado não
comprova que tenha havido vida naquele planeta', de Ricardo Ferreira

Será que não seria mais correto afirmar que `Por tudo que sabemos das
suas propriedades, sem água a vida seria impossível... na Terra'

Mensagem de Brancilene de Araújo, do Depto. de Química do Centro de
Ciências Exatas e Naturais da Universidade Federal de Alagoas:

`Tenho acompanhado com interesse e certa curiosidade a discussão sobre
a água em marte e a existência de vida naquele planeta e no resto do
universo. E, uma vez que sou fascinada pela astronomia de uma maneira
geral, me surgiu a questão.

Será mesmo que a água é essencial para a vida em outros planetas ou
sistemas planetários... ou nós estamos apenas procurando evidência de
vida segundo nossos padrões?

Será mesmo que alguns metais constantes da tabela periódica e minerais
essenciais para a vida na Terra, o seriam em outro planeta?

Quem sabe não há outros elementos químicos que nós desconhecemos lá
fora tão importantes para a vida quanto os que aqui existem?

Será que não seria mais correto afirmar que `Por tudo que sabemos das
suas propriedades, sem água a vida seria impossível... na Terra'
http://www.jornaldaciencia.org.br/Detalhe.jsp?id=24059
------fim ----

Até
Luís Brudna






SUBJECT: Re: Química aqui e lá
FROM: "rmtakata" <rmtakata@altavista.net>
TO: ciencialist@yahoogrupos.com.br
DATE: 15/12/2004 16:13


--- Em ciencialist@yahoogrupos.com.br, "brudna" <lrb@i...> escreveu
> O quão enfáticos podemos ser quanto as propriedades dos elementos da
> tabela periódica?

Podemos com certeza dizer q. hidrogenio tera' um proton em seu nucleo
em qq lugar do universo - por definicao. Em relação 'as propriedades,
elas serao como as leis da fisica: se as leis da fisica forem
universais (o universo for isotropico), as propriedades quimicas serao
universais.

> --início -
> Mensagem de Brancilene de Araújo, do Depto. de Química do Centro de
> Ciências Exatas e Naturais da Universidade Federal de Alagoas:

> Será mesmo que a água é essencial para a vida em outros planetas ou
> sistemas planetários... ou nós estamos apenas procurando evidência
> de vida segundo nossos padrões?

A Nasa esta' *declaradamente* procurando por indicios de vida segundo
os padroes de vida q. encontramos na Terra. Dificilmente poderia ser
diferente, ja' q. ela e' a unica q. conhecemos.

> Quem sabe não há outros elementos químicos que nós desconhecemos lá
> fora tão importantes para a vida quanto os que aqui existem?

Ainda estao procurando elementos transuranicos estaveis.

> Será que não seria mais correto afirmar que `Por tudo que sabemos
> das suas propriedades, sem água a vida seria impossível... na Terra'

Nem essa afirmacao seria rigorosamente correta. Teriamos q. dizer: a
vida, como a conhecemos, seria impossivel sem a presenca de agua
liquida. Se a Terra nao tivesse agua talvez outras formas de vida
poderiam ter emergido.

[]s,

Roberto Takata





SUBJECT: Re: eletron não tão nebuloso
FROM: César A. K. Grossmann <cesarakg@bol.com.br>
TO: ciencialist@yahoogrupos.com.br
DATE: 15/12/2004 17:18


--- Em ciencialist@yahoogrupos.com.br, "Alberto Mesquita Filho"
<albmesq@u...> escreveu
>
> É difícil dizer quantas pessoas "realmente" entenderam as minhas
idéias. Eu

Minha dificuldade é que todo o site ecientifico pare ser uma coletânea
de mensagens postadas em grupos de discussão e de listas como esta. Se
você tiver uma página em que você aborda de forma mais organizada e,
de preferência, capaz de ser entendida por alguém que tenha completado
o segundo grau, seria ótimo. Tenho curiosidade sempre que vejo tuas
mensagens, principalmente por que você não tem a atitude que se vê em
certos "cientistas" que defendem idéias "alternativas". Pelo
contrário, nunca vi você perder o tino ou o tom com quem quer que seja...

> tinha para apresentar. Essa proibição me foi relatada pelo próprio
> Secretário da Cultura, que lamentou bastante o acontecido e quero
crer que
> sequer entendeu o que estava a acontecer nos bastidores do Olimpo
Acadêmico.

Seria interessante você escrever um "diário de guerra", sem relatar
uma linha da tua teoria, apenas o tipo de recepção que você tem
recebido. Já pensou? E se você conseguir a mesma popularidade daquele
maluco que vende o livro sobre o planeta Hercóbulus (tenho a impressão
que a polêmica, neste caso, vai ser positiva), vai conseguir financiar
a publicação do que quer que seja...

[]s
--
César A. K. Grossmann
http://www.LinuxByGrossmann.cjb.net/





SUBJECT: Re: Site inacreditável
FROM: Maria Natália <grasdic@hotmail.com>
TO: ciencialist@yahoogrupos.com.br
DATE: 15/12/2004 20:21


Rodrigo:
O chocar, melhor dizendo, colidir, não é a nossa ideia de colisão
como em desastre de automoveis. Mesmo entre partículas a colisão tem
de ser vista como aproximação pois que nesta já ha interferências,
sobreposições de campos sejam eles gravíticos, magnéticos, eléctricos
Que a galáxia de Andrómeda e a Via Láctea estão em "recta de
colisão " já é conhecido e a previsão é que tal irá suceder antes do
nosso sol se finar. E como disse atrás a colisão é interferência de
campos e duas hipóteses se pôem ou se forma um BN ou a Terra sai
disparada para outras "quintas". Em qualquer dos casos a vida na
Terra tal qual a conhecemos hoje (e se nessa época se mantiver, o
que duvido)deixará de existir pela falta de condições boas de 4º
calhau a contar de uma Estrela tipo Sol.
Mas vá ao ON por a questão pois melhor lhe explicarão como se chegou
a esta ideia de colisão
Se Vega pertence á nossa galáxia basta pensares...Agora se é por
Vega ou por Antares a nossa aproximação Isso pouco contará.
[]'
Maria Natália

--- Em ciencialist@yahoogrupos.com.br, "Rodrigo Marques"
<rodmarq72@y...> escreveu
>
> Aproveito também para perguntar sobre esse movimento do Sol em
> direção à Vega. Como ele é? O Sol vai se chocar com ela?
> --- Em ciencialist@yahoogrupos.com.br, Luis Brudna
<luisbrudna@g...>
> escreveu
> > O quão ´não-circular´ (ou elíptica :-) ) é a órbita da Terra?
> Qual é
> > o motivo disso? (pergunta de repórter, se fazendo de bobo pra
> > aproveitar a genialidade do entrevistado ;-) )
> >
> > Até
> > Luís Brudna





SUBJECT: Re: Química aqui e lá
FROM: Maria Natália <grasdic@hotmail.com>
TO: ciencialist@yahoogrupos.com.br
DATE: 15/12/2004 21:54


Luis:
A Tabela Periódica é a ÙNICA coisa que será sempre válida em todo o
Universo!
As roupas as modas o aspecto das pessoas mudam mas isto NUNCA muda.
Tu sabes que as propriedades dos elementos varia consoante a pressão
e tempertaura a que se encontram. O que é o estado Sólido? e o
Líquido? e os comportamentos estão lá na TP. 4g de hidrogénio a
reagir com 32g de oxigénio dão água se a T e a P forem ...? Quem me
garante que se gastem mesmo aqueles 4g mais 32g? É tudo
probabilidade. E a lei de Lavoisiser e que muitos blá bla´nem um
décimo dos que a sabem digeriram, é probabilidade!
Quanto à VIDA concordo com o que as pessoas da tua mensagem
disseram. Vida pode ser diferente. Não me considero a excepção e nem
nossa VIDA um milagre. Surgiu obra de probabilidade de
circunstâncias de Físca e de Química. Aqui foi, lá naquele
extrasolar e em 4º ou 5º calhau será outra coisa. Quem mo dirá? Só
um tal encontro do 7º grau.
A engenharia de materiais muito nos está a desvendar acerca do
comportamento da matéria. E aquilo que é impensável hoje será
trivial amanhã. E surgirão avestruzes (me perdõem as aves) a dizer
que o Homem nunca esteve em Marte e robots isso era o deus deles e
religiosidades várias. E nós seremos os "quadrados" e obscurantistas
químicos ou de Ciência que acreditavamos em coisas obsoletas e
ultrapassadas. Bibliotecas de Alexandria serão queimadas, povos
dizimados e só uma poeira de nós chegará lá em 2800.
Péssimista? Não. Será o decurso normal da nossa vida. Creio que não
estamos preparados para colonizar nenhum planeta se nem na Terra ou
numa lista conseguimos viver em Paz.
E a falta de água em Marte, O ter-se esgotado, partido para o Espaço
pode ser uma das fases de um ciclo que levará a essa outra vida e
que eu neste estado de minha ciência não vejo nem sei. Como se
estivesse a falar antes do BB: "Não tenho física que se aplique como
vou saber?" A minha biologia não o permite.
Um abraço metaquímico da Natália

--- Em ciencialist@yahoogrupos.com.br, "brudna" <lrb@i...> escreveu
>
> O quão enfáticos podemos ser quanto as propriedades dos
elementos da
> tabela periódica?
>
> Pensei nisso ao ler o comentário no ´Jorna da ciência´
>
> --início -
> Leitora comenta artigo `Presença de água em Marte no passado não
> comprova que tenha havido vida naquele planeta', de Ricardo
Ferreira
>
> Será que não seria mais correto afirmar que `Por tudo que sabemos
das
> suas propriedades, sem água a vida seria impossível... na Terra'
>
> Mensagem de Brancilene de Araújo, do Depto. de Química do Centro de
> Ciências Exatas e Naturais da Universidade Federal de Alagoas:
>
> `Tenho acompanhado com interesse e certa curiosidade a discussão
sobre
> a água em marte e a existência de vida naquele planeta e no resto
do
> universo. E, uma vez que sou fascinada pela astronomia de uma
maneira
> geral, me surgiu a questão.
>
> Será mesmo que a água é essencial para a vida em outros planetas ou
> sistemas planetários... ou nós estamos apenas procurando evidência
de
> vida segundo nossos padrões?
>
> Será mesmo que alguns metais constantes da tabela periódica e
minerais
> essenciais para a vida na Terra, o seriam em outro planeta?
>
> Quem sabe não há outros elementos químicos que nós desconhecemos lá
> fora tão importantes para a vida quanto os que aqui existem?
>
> Será que não seria mais correto afirmar que `Por tudo que sabemos
das
> suas propriedades, sem água a vida seria impossível... na Terra'
> http://www.jornaldaciencia.org.br/Detalhe.jsp?id=24059
> ------fim ----
>
> Até
> Luís Brudna





SUBJECT: Re: eletron não tão nebuloso
FROM: Maria Natália <grasdic@hotmail.com>
TO: ciencialist@yahoogrupos.com.br
DATE: 15/12/2004 22:12


Cesar:

Não sabia que essa coisa também andava aí pelo Brasil esse tal Herco
qualquer coisa...Como as doenças se propagam e atravessam o oceano!
Tens algum site dessa "coisa"?
E a ideia do relato de Alberto sobre a "censura2 de seus pares e com
a calma a quele nos habituou e ...ensina e comunica acho que era de
fazer.
Um abraço?
Maria Natália
--- Em ciencialist@yahoogrupos.com.br, César A. K. Grossmann
<cesarakg@b...> escreveu
>
> --- Em ciencialist@yahoogrupos.com.br, "Alberto Mesquita Filho"
> <albmesq@u...> escreveu
> >
> > É difícil dizer quantas pessoas "realmente" entenderam as minhas
> idéias. Eu
>
> Minha dificuldade é que todo o site ecientifico pare ser uma
coletânea
> de mensagens postadas em grupos de discussão e de listas como
esta. Se
> você tiver uma página em que você aborda de forma mais organizada
e,
> de preferência, capaz de ser entendida por alguém que tenha
completado
> o segundo grau, seria ótimo. Tenho curiosidade sempre que vejo tuas
> mensagens, principalmente por que você não tem a atitude que se vê
em
> certos "cientistas" que defendem idéias "alternativas". Pelo
> contrário, nunca vi você perder o tino ou o tom com quem quer que
seja...
>
> > tinha para apresentar. Essa proibição me foi relatada pelo
próprio
> > Secretário da Cultura, que lamentou bastante o acontecido e quero
> crer que
> > sequer entendeu o que estava a acontecer nos bastidores do Olimpo
> Acadêmico.
>
> Seria interessante você escrever um "diário de guerra", sem relatar
> uma linha da tua teoria, apenas o tipo de recepção que você tem
> recebido. Já pensou? E se você conseguir a mesma popularidade
daquele
> maluco que vende o livro sobre o planeta Hercóbulus (tenho a
impressão
> que a polêmica, neste caso, vai ser positiva), vai conseguir
financiar
> a publicação do que quer que seja...
>
> []s
> --
> César A. K. Grossmann
> http://www.LinuxByGrossmann.cjb.net/





SUBJECT: Experiência em Fusão a "quente" vai começar
FROM: Maria Natália <grasdic@hotmail.com>
TO: ciencialist@yahoogrupos.com.br
DATE: 15/12/2004 22:20



No MIT e na Universidade de Colúmbia estudantes e investigadores
começaram os preparativos para um nova experiência que os levar´ao
confinamento de de plasma (gás ionizado a alta temperatura) usando
campos magnéticos bastante fortes (ói Leo)de um anel supercondutor
de meia tonelada dentro de um recipiente proveniente de um navio.
Esta experiência será a primeira no género para se tentar chegar ao
sonho da energia do futuro.
Site original em: http://web.mit.edu/newsoffice/2004/ldx.html

"MIT and Columbia University students and researchers have begun
operation of a novel experiment that confines high-temperature
ionized gas, called plasma, using the strong magnetic fields from a
half-ton superconducting ring inside a huge vessel reminiscent of a
spaceship. The experiment, the first of its kind, will test whether
nature's way of confining high-temperature gas might lead to a new
source of energy for the world.
First results from the Levitated Dipole Experiment (LDX) were
presented at a meeting of the American Physical Society the week of
Nov. 15. Scientists and students described more than 100 plasma
discharges created within the new device, each lasting from five to
10 seconds. X-ray spectroscopy and visible photography recorded
spectacular images of the hot, confined plasma and of the dynamics
of matter confined by strong magnetic force fields.

A dedication for LDX, the United States' newest approach to nuclear
fusion, was held in late October. Fusion energy is advantageous
because its hydrogen fuel is practically limitless and the resulting
energy would be clean and would not contribute to global warming as
does the burning of fossil fuels.

Scientists using the LDX experiment will conduct basic studies of
confined high-temperature matter and investigate whether the plasma
may someday be used to produce fusion energy on Earth. Fusion energy
is the energy source of the sun and stars. At high temperature and
pressure, light elements like hydrogen are fused together to make
heavier elements, such as helium, in a process that releases large
amounts of energy.

Powerful magnets, such as the ring in LDX, provide the magnetic
fields needed to initiate, sustain and control the plasma in which
fusion occurs. Because the shape of the magnetic force fields
determines the properties of the confined plasma, several different
fusion research experiments are under way throughout the world,
including a second experiment at MIT, the Alcator C-Mod, and the HBT-
EP experiment at Columbia University.

LDX tackles fusion with a unique approach, taking its cue from
nature. The primary confining fields are created by a powerful
superconducting ring about the size of a truck tire and weighing
more than a half-ton that will ultimately be levitated within a
large vacuum chamber. A second superconducting magnet located above
the vacuum chamber provides the force necessary to support the
weight of the floating coil. The resulting force field resembles the
fields of the magnetized planets, such as Earth and Jupiter.
Satellites have observed how these fields can confine plasma at
hundreds of millions of degrees.

The LDX research team is led by Jay Kesner, senior scientist at
MIT's Plasma Science and Fusion Center who earned his Ph.D. from
Columbia University in 1970, and Michael Mauel, a professor of
applied physics at Columbia University who earned his degrees from
MIT (S.B. 1978, S.M., Sc.D.)."
Sonhemos então pois quando o Homem sonha a Vida pula e avança
Maria Natália









SUBJECT: 4 questões "chatinhas" e pôr físicos e químicos a BRINCAR!
FROM: Maria Natália <grasdic@hotmail.com>
TO: ciencialist@yahoogrupos.com.br
DATE: 15/12/2004 22:33


I
1--Uma chama em atmosfera de Oxigénio, no espaço, que direcção toma?
2--O Vácuo, vazio, é dieléctrico.De onde vem então a impedância para
a carga fluir?
3- Se em órbita a velocidade de um satélite diminuir o que acontece?
(Esta é fácil)
4-Qual a explicação para o campo eléctrico ser zero dentro de um
condutor? (esta já foi aqui tratada?)
II
Agora para brincadeira:

1--http://physicsweb.org/articles/world/16/3/1/1

From: Engineering.com

2--"DNA Computers"

http://www.engineering.com/content/ContentDisplay?contentId=41010014

2--From: PhysLink.com:

"Genetic Altered Animals"

http://www.physlink.com/Community/Forums/viewmessages.cfm?
Forum=8&Topic=1877

3--"Maurice Wilkins"

http://www.physlink.com/Community/Forums/viewmessages.cfm?
Forum=8&Topic=2973

4--"Ethics Of Nanotechnology"

http://www.physlink.com/Community/Forums/viewmessages.cfm?
Forum=8&Topic=2737

Divirtam-se pois eu HOJE NÃO IREI À CAMA!!!
Maria Natália






SUBJECT: It 's a COOL COOL site!
FROM: Maria Natália <grasdic@hotmail.com>
TO: ciencialist@yahoogrupos.com.br
DATE: 15/12/2004 22:37


http://www.scienceiq.com/ UAU!!!!!!!!!!!!!!!!!!!
Apenas...No coments
Um abraço
Maria Natália





SUBJECT: E se encontrarmos Vida, aliens, num planeta?
FROM: Maria Natália <grasdic@hotmail.com>
TO: ciencialist@yahoogrupos.com.br
DATE: 15/12/2004 23:05


Como falaríamos com eles?
Quanto a mim seria um problema idêntico ao de Colombo ou ao de
Alvares Cabral ao chegar a uma América ou Brasil,não?
Eu vou pelo uso das mãos e claro no bom sentido, pessoal...Nada de
por a mão no indígena!
Um abraço
Matia Natália





SUBJECT: Re: [ciencialist] E se encontrarmos Vida, aliens, num planeta?
FROM: "Cyberlander" <mrcyberlander@i12.com>
TO: <ciencialist@yahoogrupos.com.br>
DATE: 16/12/2004 00:18

Se fosse uma alien jeitosinha, sei lá, pegava na teta pra ver a consistência, a textura, o calor __ calro tudo dentro do espírito científico de investigação hehehehehe.

Mas agora falando sério, nos filmes de sci-fi a primeira providência seria dissecar o(a) sujeito(a)...
Se houvesse amistosidade (caso contrário daria no pe´) eu tentaria algum tipo de diálogo e talvez fazer alguma coisa junto para aproximação (andar, apreciar paisagens , o que fosse possível), pois acredito que compartilhar momentos é uma forma de trocar experiência. Em resumo, tentaria aprender/ensinar algo para/com nosso anfitrião.
[ ]'s
D.C.

CYBERLANDER

Ama a realidade que constróis,
que nem a morte deterá teu voo · ·


----- Original Message -----

From: Maria Natália
To: ciencialist@yahoogrupos.com.br
Sent: Wednesday, December 15, 2004 11:05 PM
Subject: [ciencialist] E se encontrarmos Vida, aliens, num planeta?



Como falaríamos com eles?
Quanto a mim seria um problema idêntico ao de Colombo ou ao de
Alvares Cabral ao chegar a uma América ou Brasil,não?
Eu vou pelo uso das mãos e claro no bom sentido, pessoal...Nada de
por a mão no indígena!
Um abraço
Matia Natália





##### ##### #####

Para saber mais visite
http://www.ciencialist.hpg.ig.com.br


##### ##### ##### #####


Yahoo! Grupos, um serviço oferecido por:







------------------------------------------------------------------------------
Links do Yahoo! Grupos

a.. Para visitar o site do seu grupo na web, acesse:
http://br.groups.yahoo.com/group/ciencialist/

b.. Para sair deste grupo, envie um e-mail para:
ciencialist-unsubscribe@yahoogrupos.com.br

c.. O uso que você faz do Yahoo! Grupos está sujeito aos Termos do Serviço do Yahoo!.



[As partes desta mensagem que não continham texto foram removidas]



SUBJECT: A experiência de Shahriar - dupla fenda revisitada
FROM: "Alvaro Augusto - Lunabay" <alvaro@lunabay.com.br>
TO: <ciencialist@yahoogrupos.com.br>
DATE: 16/12/2004 01:25

Não sei se isso já circulou por aqui. Será que Alberto Mesquita está certo,
afinal?

[ ]s

Alvaro Augusto
________________________________________

In March 2004, Shahriar S. Afshar announced at Harvard University the
results of a variation on the similar two-pin-hole "which-way" experiment,
in which he claims to have disproved Bohr's Principle of Complementarity,
also reported in the July 24 edition of New Scientist.

Using his experiment, it is possible to detect interference fringes even
when observing the path of a photon stream, indicating that the wavefunction
does not collapse. If his results are verified, it has far-reaching
implications for the understanding of the quantum world, and invalidates the
Copenhagen interpretation. It would also seem to falsify the Many-worlds
interpretation, which predicts that there should be no interference between
wave functions in universes that are physically distinguishable. However, it
lends support to the Transactional interpretation, which is consistent with
the results.

More information at http://www.irims.org/quant-ph/030503/



SUBJECT: a nebulosidade do eletron ... p/ Alberto
FROM: "dfahlb" <dfahlb@yahoo.com>
TO: ciencialist@yahoogrupos.com.br
DATE: 16/12/2004 03:52


Olá Alberto,

Antes de começar a minha resposta, gostaria de parabenizá-lo por se
esforçar em tentar explicar o que a mecânica quântica já explica.
Sabe-se que a quântica previu pioneiramente vários fenômenos a nível
de partículas, núcleo atômico, atômico e molecular ou ainda no nível
macroscópico, por exemplo, condensado de Bose-Einstein, bandas de
valência dos condutores (gás de férmions), efeito Casimir, etc. Você
não foi o primeiro a tentar explicar vários fenômenos usando apenas a
Mecânica Clássica, e não será o último.

É salutar revisitar os conceitos, a parte histórica, etc, na
espectativa de tentar encontrar algo que possa ter passado
desapercebido por milhares de físicos em mais de 100 anos de
história. Ainda que seja bastante difícil, vale a pena o esforço,
principalmente se conquistado :o) pois o Prêmio Nobel te espera!. (Li
em algum lugar que só o Brasil formaria por volta de 300 doutores em
física por ano, o que deveria equivaler a uns 1000 físicos (sem
mestrado ou doutorado) formados por ano).

Outra coisa é que é realmente lamentável algumas posturas que alguns
dos físicos os quais você se expôs te trataram mal; mas, fazendo o
papel de advogado do diabo, eu levantaria a hipótese de que os
físicos te "dão as costas" não por uma questão de um establishment ou
algo assim.

Pode ser que você tenha a "fama" de ser alguém que vem ficado com a
mesma idéia durante décadas (você disse que é desde 1983). Daí, SE
você estivesse errado (eu disse SE) sugeriria que não valha a pena
discutir com você, afinal, em 30 anos vários físicos já devem ter
discutido com você e você não mudou de idéia daí sugeriria que seria
um esforço em vão. Está certo que na época em que você diz ter
conversado com o pessoal da USP, você só tinha 10 anos de teoria. Mas
hoje já vão 30.

Veja que não estou dizendo que sua teoria está errada e nem que foi
isso que passou pela cabeça dos físicos os quais você conversou. Só
estou fazendo o papel de advogado do diabo mas sem querer ofender ou
concluir, apenas "hipotetizar" com o que pode ter passado pela cabeça
dos físicos (que te trataram mal).Imagino que não tenha sido todos ou
ainda um ou outro não pôde parar para te explicar tim-tim por tim-tim
todos os tijolos elementares nos quais a Física é construída.

Mas o melhor mesmo é ir perguntar a quem "de teu as costas" ou foi
agressivo em todos esses anos. Tipo perguntando:
"Você me agrediu (ou me ignorou) por que?
a) Você acha que não vale a pena parar o teu trabalho para me
explicar a física mais elementar?
b) Você acha que eu sou do tipo que não mudaria de idéia?
c) Etc."

Enquanto você não pergunta (para termos os dados "experimentais")
então ficamos a "hipotetizar" sobre o comportamento deles :o)

Você tem que ver que não é uma coisa fácil para um físico ensinar a
você todos as diciplinas e explicar tudo o que um estudante vê em 4
anos de curso. Ainda mais se você supusesse de antemão que toda a
Física atual estivesse errada e não estivesse a fim de estudá-la.
Agora, talvez, as idéias mais gerais possam ser esboçadas em alguns
poucos dias de conversa :o) Mas muitos físicos achariam ingrata a
missão de resumir 4 anos de estudo em algumas conversar ou nalguns
emails. Imagine: 4 anos de umas 20 horas-aula semanais resumidas para
você o trabalhão que não daria :o)

Mudando um pouco de assunto, talvez seja interessante você ver como
os cientistas em geral pensam ou teorizam. Grosso modo, para eles uma
teoria científica é tida como tendo duas partes:
a ) explica o que já foi observado experimentalmente e
b ) prevê alguma medida que ainda não fora feita.

Daí, sendo feita tal medida e confirmada na experiência as hipóteses
bem como o modelo que as englobem, então, ele, o modelo, ganha o
status de teoria científica. Ou seja, antes de alcançar o status de
teoria, são apenas conjecturas ou hipóteses que possam estar
ocorrendo. Então você teria:

.hipóteses, e mais gerais que eles:
.modelos ou teorias fenomenológicas, e mais geral do que eles:
.teorias ou teorias gerais.

Ainda que eu tenha simplificado um pouco, isso é como um cientista
vai construindo sua teoria. Pode ser que não se concorde com isso ou
que Popper ou Kuhn não goste dessas definições :o) de qualquer forma,
não vou querer entrar no lado epistemológico da questão pois isso
deixo para os filósofos da ciência ou críticos dela. Prefiro me ater
mais a modelos, teorias, concomitantemente com o lado experimental
(ou o "lado prático").

Não sou especialista no lado epistemológico da Ciência. Tento, às
vezes, pretensiosamente, esboçar o que seriam as rotas dela. No
momento, sou um reles aluno com pretensão de ser algum dia professor-
pesquisador (cientista brasileiro). Mas acho que posso conseguir dar
uma definição, ainda que geral, da visão de como se faz teorias
científicas ou o que elas devem respeitar. Talvez eu no futuro me
reduza a um mero crítico de teorias que, como um crítico de poesias
que pode ter a capacidade de dizer que poesia é boa ou ruim mas não
tem talento em escrevê-las (as poesias), seria mal-sucessido em criar
teorias que `funcionem'.

Voltando a epistemologia, se você quer que seu modelo novo esteja bem
próximo da teria geral ainda não obtida pela Ciência, ou seja, que
ele possa estar contido nela, mas de tal maneira que ele seja o mais
completo possível, é interessante que se lance hipóteses que sejam
testáveis experimentalmente e não simplesmente axiomas. Um belo
exemplo disso é a Teoria da Relatividade Restrita na qual com apenas
duas hipóteses (testáveis experimentalmente) Einstein construiu toda
uma teoria! No final do email falo um pouco disso.

Isso posto, acho que dá para você, Alberto, mostrar aos físicos o
porquê de tua teoria estar certa (caso esteja). Sabendo de antemão
que os físicos vão te cobrar resultados experimentais (o "lado
prático" da teoria) é interessante você mostrar a eles algo que
eles "deixaram passar". Me explico:

Falo agora da Teoria Quântica. Mas não falo de Planck
(http://nobelprize.org/physics/laureates/1918/index.html ) com
primeira quantização do campo eletromagnético - os fótons - e nem de
Einstein (http://nobelprize.org/physics/laureates/1921/index.html )do
efeito fotoelétrico e nem o modelo das órbitas de Bohr (
http://nobelprize.org/physics/laureates/1922/index.html ), pois foram
apenas uma aproximação da teoria funcionando mais como modelos
fenomenológicos. Claro que modelos que explicam certas
particularidades devem poder serem englobados por teoria mais
completa.

A primeira Teoria Quântica que surgiu foi a de Schroedinger
(http://nobelprize.org/physics/laureates/1933/index.html ) mais tarde
interpretada fisicamente por Max Born (
http://nobelprize.org/physics/laureates/1954/index.html ). Essa
teoria fez previsões confirmadas experimentalmente (ou seja, "na
prática") para vários átomos; mas antes, explicou os valores
alcançados experimentalmente até o momento.

Então o que eu sugiro para que você convença os físicos (mostrando
também aos demais cientistas como químicos, etc, ou outros
profissionais que lidam com espectroscopia de átomos) é que você
comece explicando a eles, pelo menos, o comportamento de um dos
sistemas mais simples que existe no Universo: o átomo de hidrogênio.
E por que tentar explicar o comportamento do elétron em tal átomo?
Porque ele é o átomo mais simples (contém 1 próton e 1 elétron).
Vamos os resultados experimentais desse átomo? Eles são:

E1 = -13,6 eV
E2 = -3,39 eV
E3 = -1,51 eV
E4 = -0,85 eV
.
.
.

Esses valores são as energias possíveis para o elétron no átomo do
hidrogênio. Repare que eles são discretos (quantizados). Então o que
você fará é o seguinte: pegue a(s) sua(s) equação(ões) que descrevam
o comportamento do elétron no átomo de hidrogênio e tente explicá-la!

Na teoria quântica você pega 2 termos: um é a energia cinética do
elétron (você começa supondo que o elétron não está parado no átomo e
sim com posição variando no tempo) e o outro termo é a energia
potencial entre o elétron (de carga negativa) e o próton (de carga
positiva) pois eles estariam sujeitos a uma força atrativa (a energia
potencial é negativa em sistemas ligados). A esses dois termos
somados você chama da "energia total do sistema" ou
ainda "hamiltoniana do sistema", uma homenagem a Hamilton que propôs
uma maneira diferente de lidar com sistemas físicos na Físicas
Clássica, que seria descrever os sistemas como a soma das energias
dos constituintes dos sistema (Newton pensara mais em termos de
forças e resultantes das forças).

Então, segundo a teoria quântica, uma vez de posse desses 2 termos,
você troca as definições, de uma partícula pontual e clássica, do que
seja momento (massa vezes velocidade) e do que seja posição (x, y, z)
para as definições de operadores posição e momento de uma partícula
que seja descrita com uma função das posições ou momento.

Para uma visão mais quantitativa veja em
http://walet.phy.umist.ac.uk/QM/LectureNotes/node61.html
para o operador momento (equação 11.1 do texto), a Hamiltoniana
clássica de um sistema que contenha energia cinética e potencial
(equação 11.3) e o operador Hamiltoniano (eq. 11.5). O operador
Hamiltoniano tem que operar em algo, na equação 11.5 poderia ser a
função de Schroedinger (a famosa função de onda) no espaço das
configurações (das posições), ou seja, o famoso "psi (x)".

Passo essa referência pois foi citada pelo físico, também vencedor do
Prêmio Nobel, t'Hooft
(http://nobelprize.org/physics/laureates/1999/index.html ) no site
dele http://www.phys.uu.nl/~thooft/theorist.html aonde ele passa
várias referências de como fazer Física pela internet.

Com essa teoria, você consegue explicar, entre outros resultados
experimentais, o espectro do átomo de hidrogênio:
E1 = -13,6 eV
E2 = -3,39 eV
E3 = -1,51 eV
E4 = -0,85 eV
.
.
.
então, tente fazer tua teoria explicar pelo menos esses resultados
experimentais.

Repare que estou dando uma dica apenas: a de você tente explicar os
resultados obtidos experimentalmente de um dos sistemas ligados mais
simples que existe no Universo e daí parta para explicar os mais
complexos que este; afinal, como você deve saber, a quântica explica
átomos mais complexos também ( como disse no começo, a teoria
quântica explica a nível de partículas, nuclear, atômico e molecular
ou ainda no nível macroscópico, por exemplo, condensado de Bose-
Einstein, bandas de valência dos condutores (gás de férmions), efeito
Casimir, etc.)

E sobre o fato de "torcer a favor" ou "torcer contra" você, eu diria
que acho ser possível encontrar outra forma de descrever os fenômenos
a nível de partículas, nuclear, atômico, etc, ainda que você teria
que evocar em algum momento algum atributo não-clássico ao sistema.
Mas é apenas a opinião deste estudante.

Agora, não me surpreenderia o fato de você estar certo mas de certo
me surpreenderia com a nova explicação, imagino. Nem acho que você
não vai conseguir explicar tal sistema simples (elétron e próton se
interagindo) pois afinal você vem a 30 anos pensando nesse problema
ou outros do tipo.

Tendo sua teoria explicando com precisão e "na prática" ou seja, um
caso experimental, tem outra vantagem: ficaria mais fácil para o
leigo em tua teoria entendê-la partindo de uma explicação e cálculo
de tal sistema simples.

Ou seja, se você não calculou, vale a dica e se sim, seria um bom
ponto de partida para explicar a tua teoria. Agora, se você ainda não
explicou ou se tua teoria não conseguisse explicar, os mais céticos à
sua idéia começariam a ter razão. Assim, os físicos que te trataram
mal, estariam completamente errados em te destratar ou ignorar, mas
certos em achar que tua teoria esteja errada.

Agora, estando uma teoria errada, acho que não haveria problemas em
descartá-la, afinal a História mostra que até os 'geniais' erram
(erraram) também. Ou seja, em pior das situações para a tua teoria, o
autor dela, ou seja você, ainda poderia inovar e contribuir
enormemente com outra teoria na Física! Evidente.

Até mais.

P.S.: Na Teoria da Relatividade Restrita com apenas duas hipóteses
(testáveis experimentalmente) Einstein construiu toda uma teoria:

Hipótese 1: a velocidade da luz é a mesma para todos os observadores
[independendo da velocidade deles ou da fonte dessa luz (cuidado aqui
pois estamos comparando a velocidade da luz no mesmo meio e não em
meios diferentes pois sabe-se que a velocidade da luz no vácuo, por
exemplo, é diferente da da luz na água, ar, etc, assim, estaríamos
sempre colocando fontes de luz e observadores no mesmo meio)].
Hipótese 2: as Leis da Física são as mesmas para todos os
referenciais (na Teoria da Relatividade de Galileu, era conhecido
apenas as Leis da Mecânica e as do Eletromagnetismo, ainda não).

Com essas duas hipótese que foram testadas à exaustão pelos físicos
experimentais e ainda o serão durante várias décadas, ou seja,
hipóteses até então confirmadas em laboratório até os dias de hoje,
Einstein conseguiu chegar nas Transformações de Lorentz. Lorentz
ainda supunha um éter luminífero e encontrou as transformações que
levam o seu nome ao tentar explicar o experimento de Michelson-
Morley. Einstein que em suas notas biográficas disse não conhecer as
transformações de Lorentz, as obteve de uma maneira que trouxe uma
nova Física.

Ainda que a Teoria da Relatividade Restrita fosse mais geral que os
modelos fenomenológicos que existiam até então, ela não era geral
pois tratava apenas de corpos em movimento retilíneo e uniforme. Esse
foi um dos motivos de Einstein não ter ganho o Nobel pela elaboração
dela (http://nobelprize.org/physics/laureates/1921/index.html ). Para
referenciais acelerados, Einstein criou a Teoria da Relatividade
Geral mas foi apenas depois de ele ter ganho o Nobel que houveram um
número bem grande de medidas que confirmassem a teoria (antes haviam
dados, mas eram poucos, muito embora os dados confirmassem).


--- Em ciencialist@yahoogrupos.com.br, "Alberto Mesquita Filho"
<albmesq@u...> escreveu
> Prezado "rayfisica"
>
> Hoje em dia é muito raro eu receber alguma mensagem relacionada a
meus
> estudos em física e que consiga me comover, e você, com pouquíssimas
> palavras, chegou próximo a isso. Em virtude disso, acredito que
você mereça
> mais do que uma resposta seca a cada um dos seus quesitos
principais.
>
> A título de introdução, diria que em 1983, quando comecei a
teorizar em
> física, senti-me realmente frente a um problema. A partir
do "insight" então
> ocorrido [que rememorei com poucas palavras na msg 3610 da lista de
> discussão de filosofia Acrópolis -- vide
> http://ecientificocultural.com/ECC2/Dialogos/varios/ac_3612.htm ]
até a
> apresentação de um opúsculo, que escrevi em 1992 e intitulado "A
equação do
> elétron e o eletromagnetismo" (1993), eu não estava mais tão
somente frente
> a um único problema, mas de posse de uma infinidade de
questionamentos
> interelacionados: alguns de resposta óbvia, outros absurdos e a
grande
> maioria a desafiar minha perspicácia, ao mesmo tempo que colocava
em xeque
> quase tudo o que eu fora obrigado a acreditar através do ensino
massificante
> pelo qual somos obrigados a passar.
>
> Resumindo, até 1992 eu estava frente a um grande problema (exposto
em
> dois livros, um publicado em 1985 e outro em 1987) e com a obsessão
para
> resolvê-los. Em 1993, com a publicação do terceiro livro da série, o
> problema deixou de ser meu. É bem verdade que surgiram os problemas-
filhos,
> como citado por Popper...
>
> " Mas ainda que encontremos uma solução, poderemos
> descobrir, para nossa satisfação, a existência de toda uma família
de
> encantadores, se bem que talvez difíceis, problemas-filhos, para
cujo
> bem-estar poderemos trabalhar, com uma finalidade em vista, até ao
fim dos
> nossos dias." [Karl Popper]
>
> ... mas a verdade é que a obsessão desapareceu e, sob esse aspecto,
sinto-me
> um ser bastante realizado e satisfeito comigo mesmo. A sociedade, a
esse
> respeito, não poderá me cobrar mais nada, mesmo porque realizei esse
> trabalho de maneira 100% amadorística. Digo então que o problema
deixou de
> ser meu, pois se eu tiver um pingo de razão, aqueles que me
ignoraram,
> enquanto recebiam verbas polpudas para zelar pela aceitação, ou
não, de
> idéias de nossos compatriotas, hão de ser cobrados por *mais esse*
descaso.
>
> Vamos então a sua mensagem.
>
> Fico lisonjeado com a comparação entre a minha teoria e a
relatividade, mas
> é bom lembrar que essa comparação somente fará sentido "a
posteriori" e se,
> e somente se, a minha teoria conseguir vingar, coisa que ainda não
> aconteceu. A despeito disso, vou efetuar algumas considerações:
>
> Em primeiro lugar, eu diria que o paralelismo que você traçou com a
> compreensão da relatividade não é perfeito. O jovem Einstein teve a
> felicidade de encontrar pela sua frente um editor de periódico
respeitável e
> chamado Max Planck, aquele mesmo que, na palavras de Thomas Khun,
teria dito
> o seguinte:
>
> "Uma verdade científica nova não é geralmente apresentada de
maneira a
> convencer os que se opõem a ela... simplesmente a pouco e pouco
eles morrem,
> e nova geração que se forma familiariza-se com a verdade desde o
princípio."
> [Max Planck]
>
> Você (ou alguém aqui na Ciencialist) conhece algum editor nacional
e/ou
> avaliador por pares (referee) que pensa desta maneira? E que em
virtude
> disso seria capaz de avaliar e aceitar para publicação algo em que
não
> acredita, a exemplo do que Planck fez, a despeito de discordar da
> interpretação que Einstein deu ao efeito fotoelétrico?
>
> Em segundo lugar, a frase que você cita refere-se à relatividade
geral, algo
> que surgiu cerca de uma década após a relatividade restrita já estar
> praticamente consagrada.
>
> Em terceiro lugar, o que consagrou a relatividade restrita foi o
fato dela
> responder a algumas perguntas que até então os físicos não tinham
resposta
> nenhuma. Os físicos de hoje sequer se deram conta que "estão
perdidos num
> mato e sem cachorro". Eles estão plenamente satisfeitos com a
matemática que
> dá certo sem que saibam quando, onde, como e nem porquê.
>
> E em quarto lugar, ambas as relatividades --e essa idéia é citada
até mesmo
> por Einstein-- vão contra o "senso-comum adotado pelos físicos da
época" (e,
> sob muitos aspectos, quero crer que até mesmo para os físicos da
> atualidade). A minha teoria, pelo contrário, tem a pretensão de
devolver a
> credibilidade à nossa maneira de perceber ou apreciar a
fenomenologia
> natural, bem ao estilo newtoniano. Sob esse aspecto, é difícil
dizer que
> alguém realmente chegou a compreender a teoria da relatividade,
pois o
> próprio Einstein chegou a chamar a atenção para que não se
esforçassem muito
> neste sentido. Compreender a teoria da relatividade geral, no
sentido
> apresentado no texto citado, seria equivalente a perceber que ela
está
> dotada de uma matemática que dá certo sem que se saiba o porquê, a
não ser
> assumindo uma recorrência a artifícios matemáticos outros e dotados
da mesma
> falácia, até chegarmos no princípio da relatividade geral e que,
como
> princípio, é aceito "a priori".
>
> Vamos agora ao cerne da questão.
>
> É difícil dizer quantas pessoas "realmente" entenderam as minhas
idéias. Eu
> diria que alguns (poucos) torcem pelo meu sucesso, mas fico com a
impressão
> de que até o momento ninguém chegou sequer a perceber porque eu
acho que
> esta seria a única saída para a física da atualidade. Desde os
tempos em que
> militava na medicina observei que o acadêmico brasileiro
dificilmente
> valoriza temas nossos e menos ainda idéias nossas. Quando publiquei
> aquele opúsculo citado acima, em 1993, recebi um conselho do
professor
> Newton da Costa, emérito professor do Instituto de Matemática e
Estatística
> (IME) da Usp e que era mais ou menos assim: "Este livro, editado em
> português e aqui, no Brasil, está fadado a não ser lido por alguém
em
> condições de criticá-lo. Por que você não faz uma síntese e a
publica no
> exterior?" Como cheguei a concluir posteriormente, "este pensamento
não
> reflete uma opinião solitária, mas sim uma tendência geral, que é
até mesmo
> defendida em aulas de cursos de graduação da USP" (esse trecho
entre aspas
> retirei de um dos meus editoriais).
>
> Eu diria então que é bem provável que praticamente ninguém tenha
lido a
> minha teoria, pelo menos com a disposição de entender. Uma exceção
é o
> Chalub que participou de um diálogo no news uol há muito tempo, e
creio que
> você deve ter lido alguma coisa a respeito, pois a thread tinha o
nome que
> você escolheu para esta da Ciencialist: Um elétron não tão nebuloso
[Se
> alguém se interessar, poderá acompanhar esse diálogo a partir de
> http://ecientificocultural.com/ECC2/Dialogos/uol1998a.htm ]. Fiquei
com a
> impressão que ele não entendeu da maneira que seria necessária para
a
> evolução do diálogo. Alguns chegaram a ler algumas sínteses e/ou
alguns
> "problemas-filhos", mas não me parece que, além do Chalub, alguém
tenha se
> esforçado por entender a teoria.
>
> Essa desconsideração para com as minhas idéias eu já flagrei
inúmeras vezes
> aqui na Ciencialist, e creio ter ficado patente em novembro de
2000, quando
> a pedidos de alguns interessados eu me dispus a destrinchar minha
teoria
> item por item (vide
> http://ecientificocultural.com/ECC2/Dialogos/dsfcvm2.htm ). Ninguém
aqui
> parece querer arcar com o ônus de ter de repensar a física.
> Conseqûentemente, ninguém está interessado em conhecer as minhas
idéias,
> embora queiram que eu fique sempre repisando no que penso a
respeito da
> física "moderna". Quando digo o que penso, atendendo a esses
insistentes
> apelos, eles se ofendem e ficam balbuciando coisas sem sentido.
> Eventualmente conseguem se fazer entendidos, e não é raro receberem
o troco
> devido, ocasião em que acabam "enfiando o rabo entre as pernas",
pois jamais
> esperavam que alguém pudesse dar uma resposta para coisas que
aprenderam na
> universidade que não tinha resposta nenhuma [explicar um efeito
> aparentemente quântico por um raciocínio 100% clássico].
>
> Em 1992/3 eu solicitei a opinião de muitos eméritos professores da
Usp.
> Quase todos receberam-me muito bem e acredito que mal saí de suas
salas
> devem ter jogado meu trabalho na cesta de lixo. Um desses eméritos
> professores foi mais delicado e chamou um de seus orientandos de
> pós-graduação, solicitando ao mesmo que lê-se a teoria para
posterior
> discussão. Passado um certo tempo esse orientando começou a me
evitar, e eu
> logo adivinhei o motivo e parei de procurá-lo.
>
> Em 1994 ou 1995 fui convidado pelo Secretário da Cultura de Bauru
para expor
> minha teoria numa palestra da Unesp dessa cidade. Soube pelo
Secretário que
> os professores de física da Unesp foram PROIBIDOS de comparecer ao
evento.
> Com muita insistência o Secretário conseguiu convencer o Reitor a
que
> preenchesse a platéia com alunos da Faculdade de Física, ao lado de
um dos
> professores que foi escolhido (quiçá sorteado) para ouvir as
heresias que eu
> tinha para apresentar. Essa proibição me foi relatada pelo próprio
> Secretário da Cultura, que lamentou bastante o acontecido e quero
crer que
> sequer entendeu o que estava a acontecer nos bastidores do Olimpo
Acadêmico.
>
> Que mais dizer?
>
> [ ]´s
> Alberto
> http://ecientificocultural.com/indice.htm
> Mas indiferentemente a tudo isso, o neutrino tem massa, o elétron
não é
> uma carga elétrica coulombiana e a Terra se move. E a história se
repetirá.
>
>
>
> ----- Original Message -----
> From: "rayfisica"
> Sent: Tuesday, December 14, 2004 4:04 PM
> Subject: [ciencialist] eletron não tão nebuloso
>
>
> Alberto
> Li acho que nos três primeiros minutos ou no universo elegante, o
> seguinte:
> `O repórter pergunta a um estudioso, sobre o fato de apenas três
> pessoas compreenderem a teoria da relatividade, dizem que após
pensar
> um pouco o cientista responde - quem poderia ser a terceira pessoa'.
> Acho que dá pra entender o que estou tentando passar.
> Pergunto lhe:
> TODAS AS PESSOAS QUE TE CRITICAM REALMENTE ENTENDERAM SUAS IDEIAS OU
> ALGUMAS ESTÃO APENAS REPETINDO O PENSAMENTO VIGENTE?
> QUANTO POR CENTO DAS PESSOAS QUE TIVERAM CONTATO COM SUA TEORIA A
> ENTENDEU?
> Estou lendo o seu trabalho, depois volto a tc para ti.





SUBJECT: Re: E se encontrarmos Vida, aliens, num planeta?
FROM: "dfahlb" <dfahlb@yahoo.com>
TO: ciencialist@yahoogrupos.com.br
DATE: 16/12/2004 03:55


--- Em ciencialist@yahoogrupos.com.br, Maria Natália <grasdic@h...>
escreveu
>
> Como falaríamos com eles?

Imagine se estivéssemos num estado de linguagem completamente
diferente, tal que não haveria, pelo menos no contato, de estabelecer
uma linguagem? Ou ainda que nós não nos déssemos conta de que
estávamos tento contato com outra forma de vida? Ou ainda que
esse "ET" tentasse comunicar conosco de uma maneira bem estranha?

Veja o filme "Solaris". Ele discute essas questões. Ou ainda leia o
livro :o) mas o livro não li. Só vi o filme.

> Quanto a mim seria um problema idêntico ao de Colombo ou ao de
> Alvares Cabral ao chegar a uma América ou Brasil,não?
> Eu vou pelo uso das mãos e claro no bom sentido, pessoal...Nada de
> por a mão no indígena!
> Um abraço
> Matia Natália





SUBJECT: Re: [ciencialist] A experiência de Shahriar - dupla fenda revisitada
FROM: "Alberto Mesquita Filho" <albmesq@uol.com.br>
TO: <ciencialist@yahoogrupos.com.br>
DATE: 16/12/2004 06:04

----- Original Message -----
From: "Alvaro Augusto - Lunabay"
Sent: Thursday, December 16, 2004 12:25 AM
Subject: [ciencialist] A experiência de Shahriar - dupla fenda revisitada

> Não sei se isso [ http://www.irims.org/quant-ph/030503/ ] já circulou por
> aqui.

Olá Álvaro

Não, ainda não circulou. Os defensores da ficção quântica ainda estão de
queixo caído, logo tão cedo não irão comentar coisa alguma, e muito menos
aqui no Brasil. É bem possível que o Taborda se manifeste, e não é difícil
imaginar o que ele possa vir a dizer, mas não quero quebrar o impacto de sua
mensagem. Quanto aos donos da bola, diria que os membros da gang britânica
do Hawkings já estão se armando, e muito em breve estarão se bombardeando
entre si com o famoso "fogo amigo". O William G. Unruh já começou a se
manifestar, mas os ecos ainda são poucos.

> Será que Alberto Mesquita está certo, afinal?

Não, ainda é muito cedo para isso. O apocalipse quântico está próximo, mas
essa física ficcionista ainda vingará por mais alguns anos. Por enquanto
apenas alguns alicerces, que já estavam podres, começaram a dar sinais de
insolvência. A interpretação de Copenhagen "está indo prá cucuia" e levando
consigo os universos paralelos do Everett. Mas isso ainda é muito pouco, é
tão somente o princípio do fim. Como diz o "alternativo quântico" John G.
Cramer, "The quantum formalism has no problem in predicting the Afshar
result. A simple quantum mechanical calculation using the standard formalism
shows that the wires should intercept only a very small fraction of the
light. The problem encountered by the Copenhagen and Many-Worlds
Interpretations is that the Afshar Experiment has identified a situation in
which these popular interpretations of quantum mechanics are inconsistent
with the quantum formalism itself." [vide
http://www.analogsf.com/0409/altview2.shtml ]

De qualquer maneira, tem muito fanático quântico que está sem dormir, desde
que o Afshar os colocou numa "sinuca de bico".

Grato pela lembrança.

[ ]´s
Alberto
Nobre e Excelso Cavaleiro do Apocalipse Quântico
http://ecientificocultural.com/indice.htm

PS: Se alguém quiser ver uma ilustração didática, com figuras em cores,
sugiro que vá para a seguinte página do site do Afshar:
http://www.sciencefriday.com/images/shows/2004/073004/AfsharExperimentSmall.jpg




SUBJECT: Livros para todos e á borliu
FROM: Maria Natália <grasdic@hotmail.com>
TO: ciencialist@yahoogrupos.com.br
DATE: 16/12/2004 06:06


Caríssimos:

Os livros de 5 grandes bibliotecas norteamericanas vão ser
colocadas na íntegra no maior motor de busca da Internet Google. A
versão digitalizada de um milhão de obras cujos direitos de autor já
tinham caído no domínio público vai estar disponível para
leitura "on-line", anunciou o Google. Para já, quatro bibliotecas
universitárias (Harvard, Osford, Michigan e Stanford) e a Biblioteca
Pública de Nova Yorque participam no projecto. Os livros cujos
direitos já sejam públicos vão ficar disponíveis na sua versão
integral, enquanto das restantes obras apenas serão colocados na
internet excertos, de modo a preservar os direitos de autor de quem
as assina, adiantou a AFP. Para já, o Google vai digitalizar cerca
de UM MILHÃO de obras que passam a poder ser consultadas em qualquer
local com acesso à net, assegurando que os únicos "links" desses
documentos derão para a livraria Amazon e para algumas bibliotecas
publicas, diz a Lusa.
É lá vêm os livros de Newton...
Um abraço
Maria Natália







SUBJECT: Re: um paradoxo e dois bobos, sera?
FROM: "rosevena3" <rosevena3@yahoo.com.br>
TO: ciencialist@yahoogrupos.com.br
DATE: 16/12/2004 06:08


--- Em ciencialist@yahoogrupos.com.br, "Luiz Ferraz Netto"
<leobarretos@u...> escreveu
> Vamos ao texto abaixo:
> ===========================
> Luiz Ferraz Netto [Léo]
> leobarretos@u...
> http://www.feiradeciencias.com.br
> ===========================
>



Ola
Inicio repetindo "toda unanimidade é burra".
E imploro aos garimpeiros, incluindo ai os cultivadores de ostras,
que peguem leve, por favor.
Há dois tipos de aprendizes, o que não sabe e o que sabe diferente
(incluindo ai o que sabe errado), um determinado assunto.
Ao primeiro basta apenas preencher o vazio, já ao segundo é
necessário demonstrar que agora é o certo que se ensina e as vezes
para isso basta apenas a relação de admiração...
Quando então até o errado passa se por certo (essa ferramenta é
utilizada, por exemplo, pelos ditadores, os religiosos etc.).
Quando o nego já levou tantas bordoadas, que pensa ser apenas um
processo biológico, numa singularidade, desconfia de tudo.
Não que alguém tenha alguma coisa a ver com isso, apenas no sentido
de justificar.
Eu insisto...
Estando o cilindro, esfera ou cubo etc. oco ou não, de material
condutor no nosso exemplo, eletrizado ou não, o campo elétrico em seu
interior é nulo, quer a origem de algum eventual campo elétrico
esteja à superfície ou fora do condutor, não?
Nulo significa que a soma dos vetores é zero, não? (isso em qualquer
ponto interno)
Pelo que aprendi e acredito o campo elétrico existe independente da
carga de prova estar ali ou não, e potencial elétrico é uma
característica (nem sei se o termo é esse) do campo elétrico, sendo
superfície equipotencial, nesse caso região equipotencial, todos os
pontos onde o potencial elétrico tem o mesmo valor em relação a um
referencial, não?
Simplificando ainda mais, devido a minha ignorância e incapacidade de
expressão (uma carga elétrica de prova X depositada num ponto Y de um
campo elétrico adquire energia potencial elétrica W), assim sendo
potencial elétrico é uma característica do campo elétrico, campo
elétrico é uma influencia, alteração (xí) da carga elétrica no meio,
portanto não dependendo da presença de uma segunda carga elétrica, já
energia potencial elétrica é uma relação entre no mínimo duas cargas
elétricas.
Assim sendo caso essa mesma carga elétrica de prova seja depositada
em qualquer ponto de uma "região" equipotencial, sua energia
potencial elétrica será a mesma, ora só que o potencial elétrico é
uma relação entre dois pontos de um campo elétrico, assim sendo não
há potencial elétrico entre dois pontos equipotenciais de um campo,
assim sendo uma carga elétrica depositada em uma região equipotencial
de um campo elétrico, não tem porque mover se para um determinado
ponto dessa região.
Peço que faça se um esforço para entender o que estou tentando dizer.
Repito uma carga elétrica depositada em um campo elétrico nulo não
tem porque mover se para qualquer outro ponto ao mesmo potencial
elétrico, pois todo movimento espontâneo de uma carga elétrica em um
campo elétrico é no sentido de diminuir sua energia potencial
elétrica, e caso todas as cargas movam se espontaneamente para uma
região AO MESMO POTENCIAL ELETRICO, não seria uma violação da
entropia?
Depois volto a questioná-los a respeito da introdução de uma carga
elétrica dentro de um condutor carregado.
Até.






SUBJECT: Re: A experiência de Shahriar - dupla fenda revisitada
FROM: "rayfisica" <rayfisica@yahoo.com.br>
TO: ciencialist@yahoogrupos.com.br
DATE: 16/12/2004 06:29


--- Em ciencialist@yahoogrupos.com.br, "Alberto Mesquita Filho"
<albmesq@u...> escreveu
> ----- Original Message -----
> From: "Alvaro Augusto - Lunabay"
> Sent: Thursday, December 16, 2004 12:25 AM
> Subject: [ciencialist] A experiência de Shahriar - dupla fenda
revisitada


O problema penso eu é que a coisa anda muito imprevisível, andam até
chamando a coisa de religião, gostaria de algum comentário dos
senhores a respeito do paradoxo EPR e da teoria das cordas (Alberto,
eu li no universo elegante que ninguém, entendeu e nem vai entender
essa, nos próximos trezentos anos, é mole).





SUBJECT: Re: [ciencialist] a nebulosidade do eletron ... p/ Alberto
FROM: "Alberto Mesquita Filho" <albmesq@uol.com.br>
TO: <ciencialist@yahoogrupos.com.br>
DATE: 16/12/2004 07:02

----- Original Message -----
From: "dfahlb"
Sent: Thursday, December 16, 2004 2:52 AM
Subject: [ciencialist] a nebulosidade do eletron ... p/ Alberto

Olá "dfahlb"

Grato pela mensagem de certo modo carinhosa e, por outro lado, bastante
realista. Não vou entrar em pormenores pois trata-se do que poderia chamar
uma "mensagem conselheira". Conselhos a gente avalia e, uma vez aprovados,
guarda no nosso arsenal de conhecimentos para utilizá-los no momento
oportuno. Conseqüentemente, só me resta agradecer pela distinção.

Aproveito o ensejo para agradecer também as mensagens solidárias dos amigos
Hélio, Murilo e César (na ordem de chegada).

Quanto à última msg que enviei, em resposta ao Álvaro, peço aos novos
membros não levarem muito a sério minha irreverência com respeito ao
"apocalipse quântico" bem como ao "Nobre e Excelso Cavaleiro...". Isso é uma
brincadeira que os mais antigos Ciencialisteiros [até mesmo os "fanáticos"
quânticos :-)] entendem, mas os novos membros as vezes se assustam com a
mesma.

[ ]´s
Alberto
http://ecientificocultural.com/indice.htm
Mas indiferentemente a tudo isso, o neutrino tem massa, o elétron não é
uma carga elétrica coulombiana e a Terra se move. E a história se repetirá.



SUBJECT: Re: E se encontrarmos Vida, aliens, num planeta?
FROM: Maria Natália <grasdic@hotmail.com>
TO: ciencialist@yahoogrupos.com.br
DATE: 16/12/2004 07:19


Caro dfahlb:
Bem o filme vi que até foi discutido e não gostei. Dizem que os
livros são sempre difernetes e para melhor. Mas calma já digo porque
não gostei. Porque os filmes de FC são para mim mesmo de Ciência e
para se usar em aulas. E aquele dava para aulas de psicologia que
não é de minha área.
Acho que este problema da linguagem nem se vai pôr porque se já
estaremos tão avançados para viagens não robóticas quase de certeza
que teremos equipamento feito pelos colegas de software que servem
de tradutores automáticos. Quiça um ship implantado na zona da
interpretação sons no cérebro. E se eles emitem ondas que vêm até
nós e nem precisaremos de falar fazendo lembrar a teoria da luz de
Aristóteles: os olhos emitiam raios que iriam colher informações
nopsobjectos e as trariam ao cérebros para interpretação. E há
muitas ideias mas todas por enquanto ficcionadas.
Esta discussão está bem viva em:
http://www.universetoday.com/forum/index.php?showtopic=5697
Foi um intervalo na Física dura dos quânticos
Um abraço
Maria Natália


--- Em ciencialist@yahoogrupos.com.br, "dfahlb" <dfahlb@y...>
escreveu
>
> --- Em ciencialist@yahoogrupos.com.br, Maria Natália
<grasdic@h...>
> escreveu
> >
> > Como falaríamos com eles?
>
> Imagine se estivéssemos num estado de linguagem completamente
> diferente, tal que não haveria, pelo menos no contato, de
estabelecer
> uma linguagem? Ou ainda que nós não nos déssemos conta de que
> estávamos tento contato com outra forma de vida? Ou ainda que
> esse "ET" tentasse comunicar conosco de uma maneira bem estranha?
>
> Veja o filme "Solaris". Ele discute essas questões. Ou ainda leia
o
> livro :o) mas o livro não li. Só vi o filme.
>
> > Quanto a mim seria um problema idêntico ao de Colombo ou ao de
> > Alvares Cabral ao chegar a uma América ou Brasil,não?
> > Eu vou pelo uso das mãos e claro no bom sentido, pessoal...Nada
de
> > por a mão no indígena!
> > Um abraço
> > Matia Natália





SUBJECT: Re: [ciencialist] Re: um paradoxo e dois bobos, sera?
FROM: "Alberto Mesquita Filho" <albmesq@uol.com.br>
TO: <ciencialist@yahoogrupos.com.br>
DATE: 16/12/2004 08:01

----- Original Message -----
From: "rosevena3"
Sent: Thursday, December 16, 2004 5:08 AM
Subject: [ciencialist] Re: um paradoxo e dois bobos, sera?

Olá "rosevena3"

Estou pegando o bonde andando, pois não lembro das msgs anteriores e estou
sem tempo para consultá-las, mas assim mesmo vou palpitar.

> Estando o cilindro, esfera ou cubo etc. oco ou não, de material
> condutor no nosso exemplo, eletrizado ou não, o campo elétrico em seu
> interior é nulo, quer a origem de algum eventual campo elétrico
> esteja à superfície ou fora do condutor, não?

Eu diria que o campo elétrico "E" da teoria de Maxwell é nulo em todos os
pontos do interior do condutor, e o potencial elétrico "fi" da mesma teoria
é constante.

> Nulo significa que a soma dos vetores é zero, não? (isso em qualquer
> ponto interno)

Perfeito.

> Pelo que aprendi e acredito o campo elétrico existe independente da
> carga de prova estar ali ou não, e potencial elétrico é uma
> característica (nem sei se o termo é esse) do campo elétrico, sendo
> superfície equipotencial, nesse caso região equipotencial, todos os
> pontos onde o potencial elétrico tem o mesmo valor em relação a um
> referencial, não?

Cuidado. Estamos aqui frente a dois campos que se superpõem. Podemos sempre
raciocinar em termos de dois campos como se eles existissem isoladamente
(princípio da superposição), logo o seu raciocínio até aqui está correto.
Mas enquanto existir uma carga no interior do condutor, não podemos mais
dizer que o campo resultante seja nulo, pois esta carga também está gerando
um campo. Podemos sim, em teoria (e creio que seria difícil justificar, a
não ser apelando para o princípio da superposição, verificável em
circunstâncias outras), assumir que o campo da carga externa não desempenha
*inicialmente* nenhuma ação sobre a carga interna. Ou seja, a carga, como um
todo, está numa região equipotencial. Mas cada fração da carga interna
estará sob o efeito do campo das outras frações dessa mesma carga, logo cada
fração não estará mais numa região equipotencial. Se esta carga estiver
mergulhada no vácuo (condutor ôco), ela permanecerá estacionária. Do
contrário, a carga se esfacela em partículas que se dispersam (por repulsão
entre suas frações), ganhando a periferia do condutor. Creio que o Hélio,
numa msg anterior, chegou a comentar algo do tipo.

Em qualquer caso, a carga externa estará sob a ação do campo da carga
interna, e graças a isso as suas partículas irão se redistribuir. Ocorrendo
essa redistribuição, a nova carga externa exercerá o que poderíamos chamar
uma "reação" a exercer algum efeito sobre a carga interna.

> Simplificando ainda mais, devido a minha ignorância e incapacidade de
> expressão (uma carga elétrica de prova X depositada num ponto Y de um
> campo elétrico adquire energia potencial elétrica W), assim sendo
> potencial elétrico é uma característica do campo elétrico, campo
> elétrico é uma influencia, alteração (xí) da carga elétrica no meio,
> portanto não dependendo da presença de uma segunda carga elétrica, já
> energia potencial elétrica é uma relação entre no mínimo duas cargas
> elétricas.

Se a carga Y for indivisível (ou inquebrável) e dotada de frações fixas em
seus pontos (por exemplo, frações aprisionadas numa rede dielétrica), ela
permanecerá estacionária, mas cada um desses pontos (microcargas) serão
afetados pela reação descrita acima. Veja bem, estou raciocinando com
microcargas do eletromagnetismo clássico de Maxwell-Lorentz (e não com a
minha teoria). Ou seja, essa rede dielétrica, após brevíssimo intervalo de
tempo, estará dotada de um grande número de pequenos dipolos elétricos.

> Assim sendo caso essa mesma carga elétrica de prova seja depositada
> em qualquer ponto de uma "região" equipotencial, sua energia
> potencial elétrica será a mesma, ora só que o potencial elétrico é
> uma relação entre dois pontos de um campo elétrico, assim sendo não
> há potencial elétrico entre dois pontos equipotenciais de um campo,
> assim sendo uma carga elétrica depositada em uma região equipotencial
> de um campo elétrico, não tem porque mover se para um determinado
> ponto dessa região.

A carga como um todo não se move, mas suas frações sim, desde que estejam
imersas num meio condutor. E se movem não sob o efeito primeiro do campo da
carga externa, mas do campo de suas frações constituintes. Secundariamente a
carga externa poderá ter um certo efeito sobre as frações da carga interna,
ainda que a resultante sobre a carga toda permaneça nula.

> Peço que faça se um esforço para entender o que estou tentando dizer.
> Repito uma carga elétrica depositada em um campo elétrico nulo não
> tem porque mover se para qualquer outro ponto ao mesmo potencial
> elétrico, pois todo movimento espontâneo de uma carga elétrica em um
> campo elétrico é no sentido de diminuir sua energia potencial
> elétrica, e caso todas as cargas movam se espontaneamente para uma
> região AO MESMO POTENCIAL ELETRICO, não seria uma violação da
> entropia?

Não, a entropia aumentará, da mesma maneira que aumenta quando duas cargas
elétricas de mesmo sinal se afastam em obediência à lei de Coulomb. O
problema é que o meio condutor desfaz a idéia de uma carga elétrica única.
Na realidade são milhares de cargas se repelindo entre si, e nem precisaria
existir nenhuma carga elétrica externa para que o afastamento se
verificasse.

[ ]´s
Alberto
http://ecientificocultural.com/indice.htm
Mas indiferentemente a tudo isso, o neutrino tem massa, o elétron não é
uma carga elétrica coulombiana e a Terra se move. E a história se repetirá.



SUBJECT: Re: [ciencialist] Re: A experiência de Shahriar - dupla fenda revisitada
FROM: "Alberto Mesquita Filho" <albmesq@uol.com.br>
TO: <ciencialist@yahoogrupos.com.br>
DATE: 16/12/2004 08:30

----- Original Message -----
From: "rayfisica"
Sent: Thursday, December 16, 2004 5:29 AM
Subject: [ciencialist] Re: A experiência de Shahriar - dupla fenda
revisitada

> gostaria de algum comentário dos senhores a respeito do paradoxo EPR...

Eu já disse o que penso a respeito na msg C-List 4790 (abril/2000) que
também pode ser lida em
http://ecientificocultural.com/ECC2/Dialogos/varios/ci4790.htm . Como
trata-se da mensagem final de um diálogo bastante longo (na realidade é uma
síntese do que foi discutido), coloco-me a disposição para o esclarecimento
de alguma dúvida. O desfecho escrito pelo meu net-amigo e oponente Belisário
pode ser lido em http://br.groups.yahoo.com/group/ciencialist/message/4848 .
O diálogo todo pode ser acompanhado a partir de
http://ecientificocultural.com/ECC2/Dialogos/tra.htm

> ...e da teoria das cordas (Alberto,
> eu li no universo elegante que ninguém, entendeu e nem vai entender
> essa, nos próximos trezentos anos, é mole).

Mui amigo!!!! :-)) O que você quer que eu comente? Quantos anos eles levarão
para se enforcar nas supercordas? Eu acho que será antes de 2010, mas esse é
só um palpite sem valor científico. Quanto ao mais, eu diria que 300 anos é
pouco quando comparado à eternidade. ;-))

[ ]´s
Alberto
http://ecientificocultural.com/indice.htm
Mas indiferentemente a tudo isso, o neutrino tem massa, o elétron não é
uma carga elétrica coulombiana e a Terra se move. E a história se repetirá.



SUBJECT: Google porá acervo de bibliotecas na internet FSP 15Dez04
FROM: "L.E.R.de Carvalho" <lecarvalho@infolink.com.br>
TO: ciencialist@yahoogrupos.com.br
DATE: 16/12/2004 08:47


Universidades como Harvard, Stanford e Oxford terão livros digitalizados;
usuários poderão fazer buscas gratuitas

Google porá acervo de bibliotecas na internet

JOHN MARKOFF
DO "NEW YORK TIMES"

A Google, operadora do serviço de buscas mais popular da internet, anunciou
ontem acordo com algumas das maiores bibliotecas dos EUA e a da
Universidade de Oxford (Reino Unido) para converter seus acervos em
arquivos digitais, que serão postos na rede.
O objetivo da colaboração entre a Google e as instituições de pesquisa -que
incluem as universidades de Michigan, Harvard e Stanford e a Biblioteca
Pública de Nova York- é criar um acervo de livros, estudos acadêmicos e
coleções especiais no qual usuários poderão fazer buscas gratuitas que
levem em conta não apenas título, autor e assunto das obras mas também seu
conteúdo.
Já é possível acessar parte do acervo de várias bibliotecas do mundo, e
muitos sites publicam obras de domínio público, mas, na maioria dos casos,
a busca imita a consulta a um catálogo em bibliotecas "reais", ou seja, não
é possível incluir o texto das obras na pesquisa.
Sites como o A9 (<http://www.a9.com>www.a9.com), da Amazon.com, e o próprio
Google já permitem a busca dentro de livros à venda cujas editoras
"abriram" as obras, mas o usuário só pode ler trechos dos textos. O Google
Scholar (<http://scholar.google.com>scholar.google.com) e o A9 também acham
palavras e expressões dentro de artigos acadêmicos, mas a leitura integral
dos textos só é permitida a usuários e instituições assinantes das revistas
que os publicam.
A Google deve escanear e digitalizar dezenas de milhares de páginas por dia
em cada uma das bibliotecas. Executivos da empresa não quiseram divulgar o
custo da tarefa, mas analistas estimam que cerca de US$ 10 serão gastos
para cada uma das mais de 15 milhões de obras incluídas no acordo.
Bibliotecários envolvidos no projeto dizem que ele deverá ser concluído em
uma década.
A Google vai publicar o texto integral dos livros que já estão em domínio
público. Os livros cujo direito autoral é privado serão escaneados
integralmente, mas só pequenos trechos de cada obra poderão ser lidos on-line.
Os acervos das universidades de Michigan e Stanford, que, somados, chegam a
15 milhões de obras, deverão ser totalmente digitalizados. O projeto em
Harvard será inicialmente limitado a 40 mil livros. Em Oxford, apenas
livros publicados antes de 1900 serão escaneados, e a Biblioteca Pública de
Nova York só permitiu que materiais frágeis e sob domínio público sejam
digitalizados.
A iniciativa deverá dar início a uma corrida com gigantes como a Amazon, a
Microsoft e a Yahoo!. Como a Google, essas empresas podem comprar o direito
de oferecer acesso aos livros e obter lucros por meio de venda e inserção
de anúncios nos portais de acesso.
Anteontem, a Biblioteca do Congresso dos EUA e bibliotecas de Canadá,
Egito, China e Holanda anunciaram um plano de digitalizar e dar acesso
gratuito via internet a 1 milhão de livros.


[As partes desta mensagem que não continham texto foram removidas]



SUBJECT: Re: [ciencialist] Re: Site inacreditável
FROM: "Luiz Ferraz Netto" <leobarretos@uol.com.br>
TO: <ciencialist@yahoogrupos.com.br>
DATE: 16/12/2004 09:03

De: "Luis Brudna"


O quão ´não-circular´ (ou elíptica :-) ) é a órbita da Terra? Qual é
o motivo disso? (pergunta de repórter, se fazendo de bobo pra
aproveitar a genialidade do entrevistado ;-) )

O motivo disso são as 'forças centrais'.

[]'
Léo


SUBJECT: Re: um paradoxo e dois bobos, sera?
FROM: "rosevena3" <rosevena3@yahoo.com.br>
TO: ciencialist@yahoogrupos.com.br
DATE: 16/12/2004 10:34


--- Em ciencialist@yahoogrupos.com.br, "Alberto Mesquita Filho"
<albmesq@u...> escreveu
> ----- Original Message -----
Puxa!
Agora ficou claro.
Por favor, aproveita e disserte caso tenhas tempo, realizando uma
comparação entre a teoria vigente e a sua, objetivando responder por
que, nesse caso claro, sua teoria encaixa se melhor, penso que assim
eu e acho que outros, num dialogo intimista, poderemos entender e até
quiçá, aprender um pouco mais.
Estou sendo sincero.
Quer acredites ou não






SUBJECT: Actualización en Perspectivas
FROM: "Kentaro Mori" <kentaro.mori@itelefonica.com.br>
TO: ciencialist@yahoogrupos.com.br
DATE: 16/12/2004 11:38


CIENCIA

El lado oscuro de la Luna, la Gran Coincidencia Cósmica y una
explicación heterodoxa
No todos se detienen a preguntar por qué la Luna exhibe siempre las
mismas manchas, identificadas por diversas culturas ya sea como un
hombre, un conejo o una mujer cargando un bebé, entre otras muchas
interpretaciones. Piénselo bien ya que es algo intrigante: la Luna
podría girar y al hacerlo mostraría sus otras manchas; nuestros
ancestros podrían haber visto las manchas diferentes de otras partes
del satélite; lo mismo de diferentes partes del mundo podrían ser
vistas diferentes zonas de la Luna?
http://www.perspectivas.com.mx/ci/luna.htm

La marea roja
Por siglos nadie pudo explicarla satisfactoriamente. Los antiguos
griegos la atribuían a la furia de Neptuno. En la Edad media se
hablaba de una ?purgación? de los lechos marinos. Algunos
naturalistas estimaban que era el resultado de influencias lunares.
Otros hablaban de silenciosas erupciones volcánicas, y no faltó el
químico despistado que tratara de resolver el enigma con base en
confusas explicaciones sobre sustancias químicas venenosas
producidas bajo el efecto del fuego central en las entrañas del
planeta?
http://www.perspectivas.com.mx/ci/marea_roja.htm

Anillos de hadas
En otras épocas, la presencia de estas bandas se atribuía a diversos
agentes, tales como los relámpagos, las hormigas, los caracoles, las
pacas de heno que se apilaban en los campos, etcétera. Para la
mentalidad supersticiosa de la Edad Media esto era obra del diablo
o ?cosas de brujas?. Se decía que eran generados por bocanadas de
vapores subterráneos, provenientes del infierno, o que el diablo
mismo había caminado, durante la noche, dejando su huella mientras
batía su mantequilla.
http://www.perspectivas.com.mx/ci/anillos_de_hadas.htm





SUBJECT: Re: A experiência de Shahriar - dupla fenda revisitada
FROM: "rayfisica" <rayfisica@yahoo.com.br>
TO: ciencialist@yahoogrupos.com.br
DATE: 16/12/2004 11:59


--- Em ciencialist@yahoogrupos.com.br, "Alberto Mesquita Filho"
<albmesq@u...> escreveu
> ----- Original Message -----
> From: "rayfisica"
> Sent: Thursday, December 16, 2004 5:29 AM
> Subject: [ciencialist] Re: A experiência de Shahriar - dupla fenda
> revisitada
>> > ...e da teoria das cordas (Alberto,
> > eu li no universo elegante que ninguém, entendeu e nem vai
entender
> > essa, nos próximos trezentos anos, é mole).
>
> Mui amigo!!!! :-)) O que você quer que eu comente? Quantos anos
eles levarão
> para se enforcar nas supercordas? Eu acho que será antes de 2010,
mas esse é
> só um palpite sem valor científico. Quanto ao mais, eu diria que
300 anos é
> pouco quando comparado à eternidade. ;-))


Desculpem me acho que não me fiz entender.
Penso que uma, duas ou dez pessoas compreendendo teoria ela é digamos
COMPREENDIDA, entendeu?
Agora! Quando ninguém entende o porque dos por quês...
Ou é hospício ou é brincadeira, mas nunca ciência.(literalmente-
CIÊNCIA: "Conjunto de conhecimentos socialmente adquiridos ou
produzidos, historicamente acumulados, dotados de universalidade e
objetividade que permitem sua transmissão, e estruturados com
métodos, teorias e linguagens próprias, que visam compreender e,
possibilitam orientar a natureza e as atividades humanas."
[Dicionário Aurélio - Século XXI])
Seria a isso que o velho sábio senhor Albert Einstein chamou loucura.






SUBJECT: Re: um paradoxo e dois bobos, sera?
FROM: Hélio Ricardo Carvalho <hrc@fis.puc-rio.br>
TO: ciencialist@yahoogrupos.com.br
DATE: 16/12/2004 13:27


Olá Alberto e "rosevena3"

Comentários no meio do texto.
Assim como o Alberto vou usar eletromagnetismo clássico de
Maxwell-Lorentz (e não a minha teoria mesmo porque ainda não a tenho
formulada)[:-)].
Isto significa misturar um pouco o eletromagnetismo de Maxwell (que
funciona quase sempre bem numa visão macroscópica) com um pouco de
Lorentz que é uma tentativa de salvar Maxwell para o micro.
(interpretação particular minha, com alguma influencia do Alberto).

"rosevena3" disse:
> > ... Estando o cilindro, esfera ou cubo etc. oco ou não, de
material
> > condutor no nosso exemplo, eletrizado ou não, o campo elétrico
em seu
> > interior é nulo, quer a origem de algum eventual campo elétrico
> > esteja à superfície ou fora do condutor, não?
>

Depois do equilíbrio eletrostático, sim!
A transição para o equilíbrio é um tempo muito curto ~10E-15s
(dizem).

Alberto disse:
> Eu diria que o campo elétrico "E" da teoria de Maxwell é nulo em
todos os
> pontos do interior do condutor, e o potencial elétrico "fi" da
mesma teoria
> é constante.
> ...
> ...

"rosevena3" disse:
> > ... potencial elétrico é uma
> > característica (nem sei se o termo é esse) do campo elétrico,
sendo
> > superfície equipotencial, nesse caso região equipotencial, todos
os
> > pontos onde o potencial elétrico tem o mesmo valor em relação a
um
> > referencial, não?

Alberto resp.:
> Cuidado. Estamos aqui frente a dois campos que se superpõem.
Podemos sempre
> raciocinar em termos de dois campos como se eles existissem
isoladamente
> (princípio da superposição), logo o seu raciocínio até aqui está
correto.
> Mas enquanto existir uma carga no interior do condutor, não
podemos mais
> dizer que o campo resultante seja nulo, pois esta carga também
está gerando
> um campo. Podemos sim, em teoria (e creio que seria difícil
justificar, a
> não ser apelando para o princípio da superposição, verificável em
> circunstâncias outras), assumir que o campo da carga externa não
desempenha
> *inicialmente* nenhuma ação sobre a carga interna. Ou seja, a
carga, como um
> todo, está numa região equipotencial. Mas cada fração da carga
interna
> estará sob o efeito do campo das outras frações dessa mesma carga,
logo cada
> fração não estará mais numa região equipotencial. Se esta carga
estiver
> mergulhada no vácuo (condutor ôco), ela permanecerá estacionária.
Do
> contrário, a carga se esfacela em partículas que se dispersam (por
repulsão
> entre suas frações), ganhando a periferia do condutor. Creio que o
Hélio,
> numa msg anterior, chegou a comentar algo do tipo.

Sim eu comentei isto pois estava somente usando a visão de cargas
como um fluido elétrico que é a visão original do EM e na qual se
basearam para dizer que o campo no interior de um condutor em
equilíbrio é zero. Um fluido pode ser sempre mais dividido. A visão
que veio depois diz que um elétron É uma carga elétrica não
divisível. O Alberto (e de certa forma eu também) não concorda com
isto.

Alberto disse:
>
> Em qualquer caso, a carga externa estará sob a ação do campo da
carga
> interna, e graças a isso as suas partículas irão se redistribuir.
Ocorrendo
> essa redistribuição, a nova carga externa exercerá o que
poderíamos chamar
> uma "reação" a exercer algum efeito sobre a carga interna.
>

Perfeito Alberto. Você sintetizou em poucas palavras o que acontece
(será que é assim mesmo que acontece????).
Desculpa se serei repetitivo. Se você, "rosevena3", á entendeu não
precisa ler o próximo parágrafo.

Ao colocar um elétron extra no interior do condutor (não sei como
faze-lo aparecer ali sem ter percorrido todo caminho de fora para
dentro mas Vá Lá) em qq ponto fora do centro, ele no início não
sofrerá ação mas será fonte de campo (campo dentro do condutor) que
agirá nos seus "companheiros". Todos vão se afastar (inclusive os
ligados aos núcleos vão tentar isto). Isto muda a distribuição de
cargas na superfície do condutor e agora o campo gerado pelos outros
elétrons (e prótons) no "nosso amiginho" não será mais nulo. Até que
ele chegue à superfície (na verdade ele pode assumir a posição de um
vizinho que por sua vez assumi a de outro e assim por diante até a
superfície). Isto tudo dizem que acontece em aprox. 10e-15s e o
condutor volta a seu estado de equilíbrio eletrostático.
Se for exatamente no centro não tem diferença pois dizem que o
elétron não fica sossegado num lugar só mesmo sem forças agindo
nele :-).

Hélio





SUBJECT: Re: um paradoxo e dois bobos, sera?
FROM: Hélio Ricardo Carvalho <hrc@fis.puc-rio.br>
TO: ciencialist@yahoogrupos.com.br
DATE: 16/12/2004 14:19


Apenas para corrigir umas pequenas incorreções na msg anterior.

Num metal os elétrons ligados ao núcleo não desempenham um papel
muito determinante. Os eventos acontecem com os ditos "elétrons
livres" que por sua fraca energia de ligação são deslocados por
campo mesmo pequeno. E é por isto que acontece aquele trem de
elétrons em movimento (corrente) que mencionei.

Curiosidade: se uma carga Q é aparece (do nada) no centro de um
condutor esférico de 1m de raio, quando as cargas superficiais irão
sentir sua presença?
Assumindo que o efeito de um ligamento de campo eletrostático se
propaga na velocidade da luz:

1 m / c = ~ 3 x 10-9 s

Se o raio for de 1 mm --> ~ 10-12 s


Hélio





SUBJECT: Re: eletron não tão nebuloso
FROM: César A. K. Grossmann <cesarakg@bol.com.br>
TO: ciencialist@yahoogrupos.com.br
DATE: 16/12/2004 15:03


--- Em ciencialist@yahoogrupos.com.br, Maria Natália <grasdic@h...>
escreveu
>
> Cesar:
>
> Não sabia que essa coisa também andava aí pelo Brasil esse tal Herco
> qualquer coisa...Como as doenças se propagam e atravessam o oceano!
> Tens algum site dessa "coisa"?

Sim e não. Eu anotei o nome do tal planeta em um cartaz que vi na rua,
e fiz o que sempre faço quando me lembro: pesquisei no Google:

http://tinyurl.com/5pwo8

De quebra, fiquei então conhecendo o ZetaTalk.com e o
www.badastronomy.com. Pelo menos uma coisa de bom sai destas incursões
pelo estranho mundo dos que ganham dinheiro em cima da ignorância
alheia...

> E a ideia do relato de Alberto sobre a "censura2 de seus pares e com
> a calma a quele nos habituou e ...ensina e comunica acho que era de
> fazer.

O Alberto, só pela forma tranqüila que apresenta suas idéias, pela
disposição ao debate, já merece todo o respeito... Lembra as conversas
de Martinho Lutero e do Papa, Lutero dizia "raciocinemos sobre isto",
e o Papa retorquia "submete-te ou serás excomungado". Embora eu ache
que o Alberto também seja mais facilmente identificado com Galileu...

"Eppur si muove.."

[]s
--
César A. K. Grossmann
http://www.LinuxByGrossmann.cjb.net/





SUBJECT: Vídeos do CERN e procura do Yahoo
FROM: "brudna" <lrb@iq.ufrgs.br>
TO: ciencialist@yahoogrupos.com.br
DATE: 16/12/2004 15:36


Interessante

http://livefromcern.web.cern.ch/livefromcern/antimatter/webcast/AM-webcast06.html

ou http://tinyurl.com/6ndyj

Gostei do
" CERN in 2 minutes
In only two minutes, a description of CERN and it's main scientific
activities. " :-)

Agora o Yahoo também tem um sistema de procura de vídeos (beta)
http://video.search.yahoo.com/

Outros serviços que já tinham procura de vídeos
http://www.alltheweb.com/?cat=vid
http://www.altavista.com/video/default

Até
Luís Brudna





SUBJECT: Re: [ciencialist] Re: A experiência de Shahriar - dupla fenda revisitada
FROM: "Alberto Mesquita Filho" <albmesq@uol.com.br>
TO: <ciencialist@yahoogrupos.com.br>
DATE: 16/12/2004 15:42

----- Original Message -----
From: "rayfisica"
Sent: Thursday, December 16, 2004 10:59 AM
Subject: [ciencialist] Re: A experiência de Shahriar - dupla fenda
revisitada

> > > ...e da teoria das cordas (Alberto,
> > > eu li no universo elegante que ninguém, entendeu e nem vai
> > > entender essa, nos próximos trezentos anos, é mole).

> > Mui amigo!!!! :-)) O que você quer que eu comente? Quantos anos
> > eles levarão para se enforcar nas supercordas? Eu acho que será antes de
> > 2010, mas esse é só um palpite sem valor científico. Quanto ao mais, eu
> > diria que 300 anos é pouco quando comparado à eternidade. ;-))

> Desculpem me acho que não me fiz entender.

Eu entendi, apenas quis brincar com os "meus amigos" quânticos, e acabei
gerando uma possível confusão. Desculpe-me pelo mal entendido.

> Penso que uma, duas ou dez pessoas compreendendo teoria ela é digamos
> COMPREENDIDA, entendeu?

De fato, o número não é importante. Ou a teoria é compreensível ou não é.
Ela pode ser compreensível e não ser compreendida pela maioria.

> Agora! Quando ninguém entende o porque dos por quês...
> Ou é hospício ou é brincadeira, mas nunca ciência.(literalmente-
> CIÊNCIA:

Concordo plenamente, e quando me refiro à "ficção" quântica", além da
brincadeira que estou a fazer com os quânticos, e a residir no entre-aspas
(e não aquela brincadeira que você está a se referir), estou também
procurando realçar esse aspecto, ou seja, nem é tão brincadeira assim [Toda
brincadeira tem um fundo de verdade].

Mas eles "não estão nem aí" e consideram essa minha maneira de interpretar a
realidade como "obsoleta". Ou seja, eles costumam condenar esse realismo,
que chamam de medieval, e optam pelo utilitarismo ou pragmatismo defendido
por Bohr [Se a coisa funciona, não seria necessário saber o porquê dela
funcionar].

Eu não sou contra o utilitarismo, acho apenas que ele é uma etapa do
processo do conhecimento. Já os mais fanáticos seguidores de Bohr adotam
esse utilitarismo como um fim e iludem-se com a decretação da
impossibilidade da compreensão da natureza íntima da matéria, como se fosse
um dogma. É nesse sentido que a teoria das cordas será para sempre
incompreensível, pois ela foi semeada por cima desse dogma. Quando disse que
"300 anos é pouco quando comparado à eternidade", estava pretendendo ampliar
a previsão do autor citado, pois creio que a teoria das cordas jamais será
entendida. Felizmente em poucos anos ninguém irá mais se preocupar com essa
incompreensão, pois ela irá ocupar o local para a qual foi destinada, no
setor
de história da ciência das bibliotecas, juntamente com a teoria quântica e
as teorias do epiciclo, do flogisto e do calórico, entre outras.

> Seria a isso que o velho sábio senhor Albert Einstein chamou loucura.

Sim, o Einstein realista do famoso paradoxo EPR chegou a dizer algo do
gênero.

[ ]´s
Alberto
http://ecientificocultural.com/indice.htm
Mas indiferentemente a tudo isso, o neutrino tem massa, o elétron não é
uma carga elétrica coulombiana e a Terra se move. E a história se repetirá.



SUBJECT: Shahriar Afshar--Quantum Rebel?
FROM: "dfahlb" <dfahlb@yahoo.com>
TO: ciencialist@yahoogrupos.com.br
DATE: 16/12/2004 16:26


--- Em ciencialist@yahoogrupos.com.br, "Alvaro Augusto - Lunabay"
<alvaro@l...> escreveu
> Não sei se isso já circulou por aqui. Será que Alberto Mesquita
está certo,
> afinal?
>
> [ ]s
>
> Alvaro Augusto
> ________________________________________

O físico Unruh teria respondido a essa suposta violação do princípio
da complementaridade de Borh.

Veja em:
http://axion.physics.ubc.ca/rebel.html

Sobre a complementaridade de Bohr, parece que não caiu ainda :o)





SUBJECT: Ação e ReAção
FROM: marcelo ferrari <emailferrari@yahoo.com.br>
TO: emailferrari@yahoo.com.br
DATE: 16/12/2004 18:04

Até onde vai uma ação?
Onde termina a ação e começa a reação?


__________________________________________________
Converse com seus amigos em tempo real com o Yahoo! Messenger
http://br.download.yahoo.com/messenger/

[As partes desta mensagem que não continham texto foram removidas]



SUBJECT: Tudo é energia
FROM: "brudna" <lrb@iq.ufrgs.br>
TO: ciencialist@yahoogrupos.com.br
DATE: 16/12/2004 18:35


Fico confuso com o argumento ´Tudo é energia´.

Onde estão os erros? Que conceitos falhos pode ter esse argumento?
Como aproveitar essa oportunidade para ensinar?


Até
Luís Brudna






SUBJECT: Re: Tudo é energia
FROM: "rmtakata" <rmtakata@altavista.net>
TO: ciencialist@yahoogrupos.com.br
DATE: 16/12/2004 19:00


--- Em ciencialist@yahoogrupos.com.br, "Amauri Jr"
> Tudo é energia quando se movimenta ou quando as células se
> reproduzem, mas não é só isso, quando os peixes nadam geram energia e
> quando se tem a fecundação dos animais se gera energia. Tudo que há no
> universo é de alguma forma gerador de energia, o sol com sua fusão
> nuclear é um gerador de energia, um planeta fazendo sua
> rotação....enfim tudo que há movimento gera energia.

Sob essa perspectiva nas verdade nada gera energia. No maximo
transforma energia de uma para outra forma.

[]s,

Roberto Takata





SUBJECT: Re: [ciencialist] Tudo é energia
FROM: "Oraculo" <oraculo@atibaia.com.br>
TO: <ciencialist@yahoogrupos.com.br>
DATE: 16/12/2004 19:16

Olá

O problema é a definição de energia. Em geral, uma confusão com os diversos sentidos do termo energia causa problemas ou permite frases de alto impacto como essa "tudo é energia", embora vazia de significado..:-) Como na famosa "navegar é preciso, viver não é preciso" onde os diversos sentidos de precisão podem criar diversas interpretações do poema.

A energia de uma bateria, a energia de um corpo em movimento, a energia produzida pela usina hidroelétrica, nada tem a ver com a energia de um espetáculo de teatro, acordar se sentidno "enérgico", ou a palestra enérgica do professor de filosofia.

Se usarmos um conceito estrito para o termo energia (strictu senso) acho que fica mais fácil analisar a frase "tudo é energia". Primeiro, a capacidade de produzir trabalho é energia. Se produz trabalho, cabe o uso do termo, se não produz, não cabe. Assim, eletricidade é energia, mas uma palestra ou mesmo um elemento químico não são.

Mesmo agora que sabemos que E=mc2, isso não significa que podemos abandonar os conceitos e definições e considerar "tudo" a mesma coisa..:-) Tudo o que, cara-pálid, eu perguntaria ao autor..:-) O fato de que é possível convertar algo em outra coisa não torna as duas uma só.

Acho que o problema da frase "tudo é energia" é mesmo as decorrencias da mesma: e daí? O que o autor ou quem a apresenta quer dizer com isso, quais as implicações decorrentes, caso se aceite como verdade a afirmação? Tudo é energia ENTÃO podemos nos comunicar telepaticamente? Tudo é energia, ENTÃO a homeopatia, usando o "tudo" e a energia, funciona? Ou apenas tudo é energia, e sabemos como relacionar a matéria e a energia em uma equação matematica correta (e, como tudo que existe neste universo é matéria ou energia, esse é um saber consideravel..:-)?

Tudo é energia em geral é uma afirmação-escapatória, usada esotericametne na falta de evidencias ou argumentos confiáveis..:-)

Um abraço.

Homero

----- Original Message -----
From: brudna
To: ciencialist@yahoogrupos.com.br
Sent: Thursday, December 16, 2004 6:35 PM
Subject: [ciencialist] Tudo é energia



Fico confuso com o argumento ´Tudo é energia´.

Onde estão os erros? Que conceitos falhos pode ter esse argumento?
Como aproveitar essa oportunidade para ensinar?


Até
Luís Brudna






##### ##### #####

Para saber mais visite
http://www.ciencialist.hpg.ig.com.br


##### ##### ##### #####


Yahoo! Grupos, um serviço oferecido por:







------------------------------------------------------------------------------
Links do Yahoo! Grupos

a.. Para visitar o site do seu grupo na web, acesse:
http://br.groups.yahoo.com/group/ciencialist/

b.. Para sair deste grupo, envie um e-mail para:
ciencialist-unsubscribe@yahoogrupos.com.br

c.. O uso que você faz do Yahoo! Grupos está sujeito aos Termos do Serviço do Yahoo!.



[As partes desta mensagem que não continham texto foram removidas]



SUBJECT: Re: Tudo é energia
FROM: "rmtakata" <rmtakata@altavista.net>
TO: ciencialist@yahoogrupos.com.br
DATE: 16/12/2004 19:16


--- Em ciencialist@yahoogrupos.com.br, "brudna" <lrb@i...> escreveu
> Fico confuso com o argumento ´Tudo é energia´.
> Onde estão os erros? Que conceitos falhos pode ter esse argumento?
> Como aproveitar essa oportunidade para ensinar?

Depende de como se está definindo energia.

Se se parte do principio de q. realmente tudo e' energia, entao, *por
definicao*, tudo e' energia: tempo, espaco, materia, configuracoes dos
elementos no espaco e no tempo (informacao), as cores, os cheiros, a
carga eletrica, os sabores...

Mas se se parte de uma definicao particular de energia e se procura
generalizar para todos os aspectos do mundo (ou talvez fora dele),
entao e' necessario inicialmente mostrar as interrelacoes (inclusive
quantitativas) entre esses outros aspectos e a energia. Por exemplo, a
famosa formula einsteniana: E = m.c^2. (Claro q. ainda existira' a
discussao de se essa relacao significa identidadade. Por exemplo,
podemos ter uma relacao US$ 1 = R$ 2,70 - o q. nao significa q. dolar
e real sejam identicos, sejam a mesma coisa, embora possamos falar q.,
por exemplo, o poder de compra [em uma dada regiao, em um dado
momento] sejam identicos.)

"Tudo e' energia" noveval guarda uma estreita relacao com "tudo e'
numero" pitagorico.

[]s,

Roberto Takata





SUBJECT: Shahriar Afshar -- Quantum Rebel?
FROM: "dfahlb" <dfahlb@yahoo.com>
TO: ciencialist@yahoogrupos.com.br
DATE: 16/12/2004 20:49


Olá pessoal,

É de grande valia fazer como S. Afshar ou W Unruh no sentido de
colocar mais de duas fendas ou ainda adicionar várias lentes, vários
espelhos, prismas, etc, para tentar resolver um problema mais
complexo.

Entretanto, num nível mais elementar você não consegue ter outra
explicação sem evocar uma função do tipo Schroedinger governando o
comportamento de uma partícula.

Experiências simples já evidenciam isso. Videm, por exemplo, em
http://physicsweb.org/articles/world/15/9/1/1/bologna-image
aonde tem-se o resultado experimental de 1 elétron por vez! passando
por uma fenda dupla. Espera-se que o filme fique um tempo grande de
esposição (ou seja, após muitos elétrons tendo passado 1 a 1 e sem
interferirem 1 com o outro) e tem-se a figura de interferência!

A interpretação mais aceita seria que você tem uma função não-local
que governa a partícula e daí as regiões de interferência mesmo para
uma única partícula! Recentemente tem-se feito esperiências com 1
única molécula passando por vez e vê-se também o comportamento
quântico.

Esperiências simples também evidenciam o fato de que ou bloqueando
uma das fendas ou tentando detectar por qual a partícula passou, o
padrão de interferência é destruído. Ou seja, faz-se assim para cada
elétron que vai passando e no final, no filme, ao invés de ter aquela
figura com regiões de interferência, a figura é contínua (aquela
mancha branca - que indica aonde um elétron ou vários colidiram -
fica sem aquelas falhas).

Então, os cientistas tem agora ou aumentado o tamanho (indo para
átomos, depois polímeros (moléculas grandes)) estão querendo
descobrir aonde é que a Mecânica Clássica estaria "voltando" ou seja,
que os efeitos quânticos fossem desprezíveis ou indetectáveis. Alguns
estão propondo lançar vírus por uma fenda dupla.

Outros, como Afshar, preferem "complicar" o sistema de tal forma que
uma outra interpretação também valesse. Outros ainda tendem
investigar o efeito do tempo na medida e por aí vai. Entretanto, acho
que para entender a mecânica quântica bastaria analisar a fenda dupla
sem ir colocando lentes, espelhos, prismas e coisas a mais. Claro que
estes cientistas estão interessados no "ponto de ruptura" da
quântica.

Mas, até o momento, uma coisa podemos dizer com alguma certeza:
experiências mais simples (fundamentais, portanto) não puderam ser
interpretadas de outra maneira.

Até.





SUBJECT: Re: [ciencialist] Shahriar Afshar--Quantum Rebel?
FROM: "Alberto Mesquita Filho" <albmesq@uol.com.br>
TO: <ciencialist@yahoogrupos.com.br>
DATE: 16/12/2004 20:55

----- Original Message -----
From: "dfahlb"
Sent: Thursday, December 16, 2004 3:26 PM
Subject: [ciencialist] Shahriar Afshar--Quantum Rebel?

> O físico Unruh teria respondido a essa suposta violação do princípio
> da complementaridade de Borh.
> Veja em: http://axion.physics.ubc.ca/rebel.html
> Sobre a complementaridade de Bohr, parece que não caiu ainda :o)

Apenas para colocar mais lenha na fogueira ;-), diria que vale a pena
analisar a resposta que o Shahriar S. Afshar deu ao Unruh em 19/ago/2004.
Não reproduzo aqui pois essa resposta está protegida por copyright, mas pode
ser lida em http://users.rowan.edu/~afshar/FAQ.htm (é bem pequena, por
sinal -- estou aguardando uma resposta mais extensa, conforme prometido pelo
autor).

Pelo visto, e supondo que a acusação feita pelo Afshar proceda, o Unruh está
tentando cientifizar aquele velho costume político-britânico de propalar
"meias-verdades".

[ ]´s
Alberto
http://ecientificocultural.com/indice.htm
Mas indiferentemente a tudo isso, o neutrino tem massa, o elétron não é
uma carga elétrica coulombiana e a Terra se move. E a história se repetirá.



SUBJECT: Electrão como um vector
FROM: "Sergio M. M. Taborda" <sergiotaborda@yahoo.com.br>
TO: ciencialist@yahoogrupos.com.br
DATE: 16/12/2004 21:59


Sim, Alberto esta é para si.

Eu andei pensando nas hipoteses que vc usa na sua teoria e não chego a
nenhuma conclusão do pq vc supor que o eletrão pode ser caracterizado
por um vector. Tenho algumas perguntas em relação a isso. A primiera é
exactamente Pq um vector ?
Aceitando que é um vector, será um vector ou um co-vector ?
Depois vc caracteriza o vector como W(w_x,w_y,w_z) = K w1 , onde w1 é
um vector unitário. Isto significa que podemos reduzir o estudo ao K
que não é um vector. Ora, isto não contraria a propria hipotese ?

Depois, eu suponho que W dependa de x,y,z ou seja que as derivadas
parciais am ordem a estes não sejam nulas. O que por conseguinte
significa que as derivadas de K tb dependem.

Depois, se W é um vector no espaço, ele muda a sua orientação ? Ou
seja, como são as derivadas em ordem ao tempo ?

Outra coisa que não consegui encontrar no seu site é a relação entre o
vector do electrão e a carga do electrão.

Se o electrão A cria um campo de efeitos EA e este é captado por um
electrão B que tem o seu campo EB , qual é o efeito de A em B , ou
seja, o que acontece a B ? Sabemos que cargas opostas se repetem, pelo
que EA aplicado em B deverá cria um força em B que faz o electrão se
afastar F=dp/dt. Como encontrar esta força a partir das definições dos
efeitos electricos e magneticos?

Basicamente, como se relaciona a carga , com os campos de efeitos e
estes dois com as forças repulsão e atracção ?

Outra interrogação é se a teoria so se aplica a electrões e protoes ou
a qq particula com carga , e nesse caso como entram as particulas sem
carga ?





SUBJECT: Seqüenciamento do Genoma do Arroz chega ao fim
FROM: Paula Fernandez <paulayaya@yahoo.com.br>
TO: ciencialist@yahoogrupos.com.br
DATE: 17/12/2004 00:24

fonte:http://www.ufpel.tche.br/noticias.php

Seqüenciamento do Genoma do Arroz chega ao fim

Pesquisa poderá contribuir para o fim da fome no mundo



Está decodificada a seqüência do genoma do arroz. O
anúncio do final do trabalho foi feito simultaneamente
em todos os dez países membros da rede que realizou a
pesquisa, que são Japão, Estados Unidos, França,
China, Índia, Coréia, Taiwan, Tailândia, Brasil e
Reino Unido. No Brasil, o anúncio foi realizado nesta
segunda-feira(13) pelo ministro da Educação, Tarso
Genro, na condição de representante máximo da
instituição envolvida, a Universidade Federal de
Pelotas.

O professor Antônio Costa de Oliveira, da Faculdade de
Agronomia Eliseu Maciel(Faem) da UFPel é o responsável
pela equipe brasileira. Ele está participando de
encontro que reúne 60 pesquisadores dos dez países,
até 20 de dezembro, no Japão. Oliveira é o único
cientista brasileiro na reunião e a fazer parte da
rede internacional que seqüenciou o genoma.

A seqüência completa dos 12 cromossomos do arroz cobre
um total de 390 milhões de pares de bases. Por volta
de 40 mil genes foram identificados. “Sua composição e
distribuição clareiam as principais características do
genoma do arroz”, afirmou o professor Oliveira. Para
ele, o feito não é importante somente para o
melhoramento do arroz, mas fornecerá também valiosas
informações para a análise de outros cereais, como o
milho e o trigo. “Mais importante que isto, é que o
genoma do arroz servirá como pedra de Rosseta para
salvar a fome no mundo, através de sua ampla aplicação
na agricultura e em campos correlatos”, comemorou.

A codificação do genoma do grão começou a
ser realizada em 1997. Antônio Oliveira ingressou na
rede de pesquisadores em 2000. Com o fim do
seqüenciamento, novas etapas do trabalho já estão
planejadas. “Conhecemos as seqüências mas não sabemos
suas funções”, diz Oliveira. Neste sentido, linhas de
pesquisa estão desenhadas na UFPel para dar
prosseguimento às investigações. Entre elas,
tolerância a ácidos orgânicos e ao frio, linhagem para
produção de híbridos e outros gens de importância
agronômica.

Dentro desta área chamada de genômica
funcional, Oliveira levou ao 2º Simpósio Internacional
de Genômica Funcional de Arroz, realizado em novembro
na Universidade do Arizona, Estados Unidos, o trabalho
Expressão diferencial de gens de raiz de arroz sob
condições de hipóxia.

O objetivo da pesquisa é, através da identificação e
estudo dos gens de arroz responsáveis pela sua
tolerância ao encharcamento, transferir esta
característica para outros produtos, como trigo, aveia
e milho para, desta forma, permitir seus cultivos em
áreas onde hoje só se pode plantar arroz. O trabalho
está sendo desenvolvido no Centro de Genômica e
Fitomelhoramento, dentro do Programa de Pós-graduação
em Agronomia da Faem. Estima-se que em dois anos se
tenha algum resultado.

Participa das pesquisas na UFPel o professor Fernando
Irajá Carvalho, líder do grupo de pesquisa em cereais
da Universidade.







_______________________________________________________
Yahoo! Mail - Agora com 250MB de espaço gratuito. Abra
uma conta agora! http://br.info.mail.yahoo.com/


SUBJECT: Re: Tudo é energia
FROM: Maria Natália <grasdic@hotmail.com>
TO: ciencialist@yahoogrupos.com.br
DATE: 17/12/2004 01:24


Homero:
E o químico dirá que mesmo aquela cadeira parada, em repouso
elativo, tem energia. Energia interna, energia potencial química e
energia das oscilações, ligações, vibrações, rotacões moléculares.
Um astrofísico falará da energia do vácuo e por aí fora.
É como o orientador de estágio pegar num sapato que coloca em cima
de secretária do estagiário dizendo: e agora com base neste sapato
faça a planificação para aula de química nível 11º ano. Difícil será
introduzir em programa existente, mas faz-se.
E aquele sapato tem tudo e até aula de português permite fazer.
A energia assim falada permite tudo o que a creatividade do
professor quiser.
Definição de energia dá tanta raiva como a definição de força. É que
se necessita do conceito bem esclarecido. Só na universidade se
entende tais definições.
Energia é assim uma expressão em verso de poeta.

Energia é tudo:
Está contigo
foge com a bola
Cavalga no som
no grão de trigo
é núcleo desnudo
dá cabo da tola
De físico bombom*

Um abraço
Maria Natália
* físico muita BOM

--- Em ciencialist@yahoogrupos.com.br, "Oraculo" <oraculo@a...>
escreveu
> Olá
>
> O problema é a definição de energia. Em geral, uma confusão com os
diversos sentidos do termo energia causa problemas ou permite frases
de alto impacto como essa "tudo é energia", embora vazia de
significado..:-) Como na famosa "navegar é preciso, viver não é
preciso" onde os diversos sentidos de precisão podem criar diversas
interpretações do poema.
>
> A energia de uma bateria, a energia de um corpo em movimento, a
energia produzida pela usina hidroelétrica, nada tem a ver com a
energia de um espetáculo de teatro, acordar se sentidno "enérgico",
ou a palestra enérgica do professor de filosofia.
>
> Se usarmos um conceito estrito para o termo energia (strictu
senso) acho que fica mais fácil analisar a frase "tudo é energia".
Primeiro, a capacidade de produzir trabalho é energia. Se produz
trabalho, cabe o uso do termo, se não produz, não cabe. Assim,
eletricidade é energia, mas uma palestra ou mesmo um elemento
químico não são.
>
> Mesmo agora que sabemos que E=mc2, isso não significa que podemos
abandonar os conceitos e definições e considerar "tudo" a mesma
coisa..:-) Tudo o que, cara-pálid, eu perguntaria ao autor..:-) O
fato de que é possível convertar algo em outra coisa não torna as
duas uma só.
>
> Acho que o problema da frase "tudo é energia" é mesmo as
decorrencias da mesma: e daí? O que o autor ou quem a apresenta quer
dizer com isso, quais as implicações decorrentes, caso se aceite
como verdade a afirmação? Tudo é energia ENTÃO podemos nos comunicar
telepaticamente? Tudo é energia, ENTÃO a homeopatia, usando o "tudo"
e a energia, funciona? Ou apenas tudo é energia, e sabemos como
relacionar a matéria e a energia em uma equação matematica correta
(e, como tudo que existe neste universo é matéria ou energia, esse é
um saber consideravel..:-)?
>
> Tudo é energia em geral é uma afirmação-escapatória, usada
esotericametne na falta de evidencias ou argumentos confiáveis..:-)
>
> Um abraço.
>
> Homero
>
> ----- Original Message -----
> From: brudna
> To: ciencialist@yahoogrupos.com.br
> Sent: Thursday, December 16, 2004 6:35 PM
> Subject: [ciencialist] Tudo é energia
>
>
>
> Fico confuso com o argumento ´Tudo é energia´.
>
> Onde estão os erros? Que conceitos falhos pode ter esse
argumento?
> Como aproveitar essa oportunidade para ensinar?
>
>
> Até
> Luís Brudna
>
>
>
>
>
>
> ##### ##### #####
>
> Para saber mais visite
> http://www.ciencialist.hpg.ig.com.br
>
>
> ##### ##### ##### #####
>
>
> Yahoo! Grupos, um serviço oferecido por:
>
>
>
>
>
>
>
> -------------------------------------------------------------------
-----------
> Links do Yahoo! Grupos
>
> a.. Para visitar o site do seu grupo na web, acesse:
> http://br.groups.yahoo.com/group/ciencialist/
>
> b.. Para sair deste grupo, envie um e-mail para:
> ciencialist-unsubscribe@yahoogrupos.com.br
>
> c.. O uso que você faz do Yahoo! Grupos está sujeito aos
Termos do Serviço do Yahoo!.
>
>
>
> [As partes desta mensagem que não continham texto foram removidas]





SUBJECT: Mais rápido que a luz?
FROM: "rosevena3" <rosevena3@yahoo.com.br>
TO: ciencialist@yahoogrupos.com.br
DATE: 17/12/2004 06:15


Pessoal não sei se o assunto já foi tratado aqui se já,
digam me onde.
Caso não peço ajuda para entender
------------------------------------
http://www.fisicaju.com.br/fisica/luz.htm

Experimento americano pode ter superado velocidade da luz

Quinta, 20 de julho de 2000, 11h08min

Um grupo de físicos americanos pode ter descoberto uma
velocidade
superior ao que antes se suponha ser a velocidade máxima do
universo –
a velocidade da luz. Durante gerações, os físicos supuseram
que não
havia nada mais veloz do que o movimento da luz no vazio, uma
velocidade de 300 mil quilômetros por segundo. Mas, em uma
experiência da Universidade de Princeton, Nova Jersey (EUA),
físicos
enviaram um raio de luz de laser através de vapor de césio numa
velocidade tão alta que saiu da câmera antes de terminar de
entrar.
Os cientistas dizem que esta foi a demonstração mais convincente
de
que a velocidade da luz pode ser superada, ao menos sob determinadas
circunstâncias em laboratório.
Os resultados serão publicados na edição de amanhã da
revista
Nature. O feito ainda não tem aplicações práticas imediatas,
mas
experimentos similares têm gerado entusiasmo na comunidade
internacional de físicos teóricos e ópticos. A
experiência de
Princeton põe à prova os limites da teoria da relatividade que
Albert
Einstein descobriu no príncipio do século, segundo a qual a
velocidade das partículas de luz no vazio é a única medida
absoluta
do universo. A velocidade de tudo mais seria relativa ao observador.
Nas circunstâncias cotidianas, um objeto não pode viajar numa
velocidade maior do que a da luz. Em possíveis aplicações
práticas, o
experimento poderia contribuir no desenvolvimento de computadores
mais velozes, que transportariam informações em partículas de
luz.

Agência RBS
Relatividade não foi violada
O experimento de Lijun Wang, A. Kuzmich e A. Dogariu, do NEC
Research Institute, publicado na Nature, 406, 277, de 20 jul 2000,
consiste na propagação de um pulso de laser com 3,7 microns de
extensão por uma cápsula de 6 cm contendo átomos de césio
especialmente excitados ao segundo nível, em uma situação
programada
para que o índice de refração esteja mudando rapidamente. Como
um
pulso de curta duração é necessariamente formado por um grande
número
de ondas com freqüências diferentes, o que se mede é a
interferência
das diferentes freqüências componentes do pulso, que mudam de fase
devido à variação do índice de refração. A fase relativa
muda fazendo
que a parte central do pulso se adiante 1,7% de sua extensão. Como
o
início do pulso inicial ocorre antes do centro do pico de
saída, não
há qualquer violação da casualidade. Como a velocidade de fase
da luz
não muda, só a velocidade de grupo do pulso, não há
qualquer violação
da relatividade. O próprio Lijun Wang, líder da equipe
explica: "Falando precisamente, é a velocidade de transferência
da
informação que é limitada pela velocidade da luz no vácuo."
Toda a
informação necessária sobre o pulso está contida em sua
minúscula
borda frontal. Assim que essa parte do pulso entra na câmara, os
átomos especialmente preparados podem começar a reproduzir
outro
pulso idêntico no outro lado da câmara. (27 jul 2000)

Fonte: Departamento de Astronomia do Instituto de Física da UFRGS
------------------------------------------
PERGUNTO:
Da pra ultrapassar c ou não?
Pois o "Falando precisamente, é a velocidade de transferência
da
informação que é limitada pela velocidade da luz no vácuo."
Me deixou
com a pulga atrás da orelha, junto com a pulga do neutrino.







SUBJECT: Gravidade
FROM: "tipoalgo" <tipoalgo@bol.com.br>
TO: ciencialist@yahoogrupos.com.br
DATE: 17/12/2004 06:45


Bom dia a todos,

Alguém poderia esclarecer se o professor Fran de Aquino é realmente
brasileiro?
A pergunta é porque ele na Internet ele é quase todo inglês!

E o que dizer de suas pesquisas?
Um brasileiro já domina a gravidade e poucos se aperceberam?

Algum mestre aqui da c_list poderia fazer um convite para ele nos
brindar com os seus conhecimentos?

Mais informações em:

http://users.elo.com.br/~deaquino//

(em algo em português)
http://users.elo.com.br/~deaquino//PORT.htm

"· Born: October 1, 1950 · Place of birth: Brazil · Residence: São
Luís - MA, Brazil
· Affiliation: Physics Department, Maranhão State University, São
Luís - MA, Brazil"


Abraços a todos

Tipoalgo





SUBJECT: Re: [ciencialist] Re: um paradoxo e dois bobos, sera?
FROM: "Alberto Mesquita Filho" <albmesq@uol.com.br>
TO: <ciencialist@yahoogrupos.com.br>
DATE: 17/12/2004 06:55

----- Original Message -----
From: "rosevena3"
Sent: Thursday, December 16, 2004 9:34 AM
Subject: [ciencialist] Re: um paradoxo e dois bobos, sera?

> Por favor, aproveita e disserte caso tenhas tempo, realizando uma
> comparação entre a teoria vigente e a sua, objetivando responder por
> que, nesse caso claro, sua teoria encaixa se melhor,...

Caro "rosevena3"

Eu diria que a minha teoria não é melhor nem pior, simplesmente visa a
outros objetivos. A teoria de Maxwell é a melhor dentro dos objetivos para
os quais foi proposta, qual seja, para explicar o eletromagnetismo
macroscópico.

Exemplifico com uma analogia: A mecânica dos fluidos, com sua equação de
Navier Stokes, tem-se mostrado ser uma teoria excelente para a resolução de
uma infinidade de problemas em física. Não obstante, jamais conseguiríamos,
com essa teoria, ou equação, estudar a interação entre, por exemplo, duas
moléculas de água, pelo simples fato de que a molécula de água é uma
entidade totalmente diferente dos "elementos infinitesimais de volume" da
mecânica dos fluidos (esses elementos são entidades matemáticas e
imaginárias).

A teoria de Maxwell nada mais é senão o que poderia ser chamado a
eletrodinâmica dos fluidos elétricos. Como tal, ela vale somente para
elementos de volume desse fluido (elementos de carga e elementos de corrente
elétrica), mas é incapaz de resolver qualquer problema no âmbito do
microcosmo, pois as partículas que fazem parte desse *suposto* fluido são
totalmente diferentes dos elementos considerados. Um elétron NÃO É a fração
infinitesimal dos fluidos elétricos, e esse foi, a meu ver, o maior erro
cometido em toda a história da física. Como diria o filósofo Ronald Golias,
:-), "Ouça o que eu digo, pois o que eu digo não está nos livros." Em
decorrência desse erro surgiu a física que, segundo os físicos do início do
século XX, ia contra o senso comum (relatividade e quântica).

Por outro lado, tentar resolver o problema proposto (espalhamento do fluido
ou das partículas elementares através de um condutor) através de uma teoria
válida para o microcosmo não é nada impossível, mas seria equivalente a
tentar matar um mosquito com um canhão (estou parafraseando os matemáticos).
Seria também equivalente a utilizar o formalismo lagrangeano para resolver
um problema de física desses ensinados no segundo grau; ou então estudar uma
colisão de duas bolas de bilhar decompondo as bolas em um número enorme de
moléculas e, a partir daí, tentar aplicar a mecânica estatística ou algo
equivalente. Volto a dizer: é possível, mas não é prático e nem mesmo fácil.
Além do que eu teria antes que firmar as bases da minha teoria, pois ninguém
aqui parece conhecê-las. É possível que na thread que o Taborda inaugurou
(Electrão como vetor) eu consiga evoluir nessa direção, mas não garanto,
pois discutir com o Taborda é algo fantástico e gratificante, mas bastante
imprevisível (as dúvidas vão se acumulando como bola de neve e a gente acaba
se dispersando).

E por falar no Sérgio Taborda, quero apenas dar um alô e dizer que vou
demorar um pouco para responder a sua mensagem, pois ela é longa, delicada e
o tempo não está me ajudando (nessa semana tive muitos compromissos
acadêmicos).

> Estou sendo sincero. Quer acredites ou não

Eu sempre acreditei na boa fé da maioria dos net-amigos da Ciencialist, até
mesmo aqueles com quem tenho "quebrado o pau". É possível que eu tenha sido
mal interpretado quando afirmei recentemente o seguinte (msg 43184): "essa
desconsideração para com as minhas idéias eu já flagrei inúmeras vezes aqui
na Ciencialist...". O termo "desconsideração" parece-me ter sido pesado
demais, pois o que pretendi dizer era que não havia um interesse por novas
idéias, em especial as nacionais (independentemente de serem minhas ou de
terceiros). Tanto assim é que conclui esse pensamento com as seguintes
palavras: "Ninguém aqui parece querer arcar com o ônus de ter de repensar a
física." E esta sim, parece-me retratar uma grande "verdade", não só aqui na
Ciencialist, mas nos meios acadêmicos do Brasil e do mundo (a
pseudofilosofia paradigmática de Thomas Kuhn apóia-se nessa "verdade").

[ ]´s
Alberto
http://ecientificocultural.com/indice.htm
Mas indiferentemente a tudo isso, o neutrino tem massa, o elétron não é
uma carga elétrica coulombiana e a Terra se move. E a história se repetirá.



SUBJECT: Re: [ciencialist] Tudo é energia
FROM: "JVictor" <jvoneto@uol.com.br>
TO: <ciencialist@yahoogrupos.com.br>
DATE: 17/12/2004 06:58


Amauri,

Creio que você quis dizer que todas as ações são realizadas às custas de energia e não que a energia é gerada, criada, nos processos. A energia do universo é constante, não muda. O que há são transformações de uma forma de energia para outra. E as formas são muitas. Mas, no final, a soma de todas elas é sempre a mesma. Sempre. E esse sempre está ancorado em aspectos da natureza, que determinam a razão pela qual a energia se conserva: as simetrias. Estas características da natureza, as simetrias, dizem quais as coisas que nuncam mudam, quando de uma transformação qualquer; igualmente dizem quais coisas nunca podem acontecer expontâneamente, sem uma ajudinha externa. É isto que determina, impõe e define as leis da natureza, que não mudam com o tempo. Pois as simetrias não mudam. A isotropia e a homogeneidade do espaço são os culpados. Se estas últimas propriedades variassem de ponto para ponto ou para direções diferentes um mesmo corpo teria quantidade de movimento diferente a depender da direção do movimento, um comprimento teria medidas diferentes na mesa do moderador Brudna e na sala de Takata, os ponteiros de seus relógios ocupariam posições diferentes em coordenadas diferentes, e por aí afora. A energia do universo variável seria uma indicação, um aviso, de que estas últimas varinhas de condão invariantes deixariam de ser constantes, o que implicaria em leis da natureza também variáveis, uma para cada dia da semana ou para cada casa. A física seria impossível. Os físicos estariam desempregados, os químicos e todos os cientistas, idem; e eu não teria a satisfação de participar deste fórum, aprendendo tanta coisa bonita com tanta gente inteligente. Então, viva as simetrias da natureza, os deuses regulamentadores do que é possível e do que não é possível, nem que as vaca tussa. Os físicos deveriam bradar: meu salário por uma simetria!
É por causa disso que é impossível o MP. Culpem, pois, as simetrias.

Sds,

Victor.

----- Original Message -----
From: Amauri Jr
To: ciencialist@yahoogrupos.com.br
Sent: Friday, December 17, 2004 6:57 PM
Subject: Re: [ciencialist] Tudo é energia


Tudo é energia quando se movimenta ou quando as células se reproduzem, mas não é só isso, quando os peixes nadam geram energia e quando se tem a fecundação dos animais se gera energia. Tudo que há no universo é de alguma forma gerador de energia, o sol com sua fusão nuclear é um gerador de energia, um planeta fazendo sua rotação....enfim tudo que há movimento gera energia.

Abraços
Amauri
----- Original Message -----
From: brudna
To: ciencialist@yahoogrupos.com.br
Sent: Thursday, December 16, 2004 6:35 PM
Subject: [ciencialist] Tudo é energia



Fico confuso com o argumento ´Tudo é energia´.

Onde estão os erros? Que conceitos falhos pode ter esse argumento?
Como aproveitar essa oportunidade para ensinar?


Até
Luís Brudna






##### ##### #####

Para saber mais visite
http://www.ciencialist.hpg.ig.com.br


##### ##### ##### #####


Yahoo! Grupos, um serviço oferecido por:







------------------------------------------------------------------------------
Links do Yahoo! Grupos

a.. Para visitar o site do seu grupo na web, acesse:
http://br.groups.yahoo.com/group/ciencialist/

b.. Para sair deste grupo, envie um e-mail para:
ciencialist-unsubscribe@yahoogrupos.com.br

c.. O uso que você faz do Yahoo! Grupos está sujeito aos Termos do Serviço do Yahoo!.



[As partes desta mensagem que não continham texto foram removidas]



##### ##### #####

Para saber mais visite
http://www.ciencialist.hpg.ig.com.br


##### ##### ##### #####


Yahoo! Grupos, um serviço oferecido por:

São Paulo Rio de Janeiro Curitiba Porto Alegre Belo Horizonte Brasília




------------------------------------------------------------------------------
Links do Yahoo! Grupos

a.. Para visitar o site do seu grupo na web, acesse:
http://br.groups.yahoo.com/group/ciencialist/

b.. Para sair deste grupo, envie um e-mail para:
ciencialist-unsubscribe@yahoogrupos.com.br

c.. O uso que você faz do Yahoo! Grupos está sujeito aos Termos do Serviço do Yahoo!.



[As partes desta mensagem que não continham texto foram removidas]



SUBJECT: Re: [ciencialist] Re: Tudo é energia
FROM: "JVictor" <jvoneto@uol.com.br>
TO: <ciencialist@yahoogrupos.com.br>
DATE: 17/12/2004 07:30

Natália,

Se você colocar em uma mesa redonda 5 gênios da física e da química para discutirem o que é energia, no sentido mesmo das verdades essenciais por trás desse intuitivo conceito, você acha que todos darão a mesma resposta, embora todos concordem com a conservação dela?
Uma boa experiência, com estudantes do nível médio, acho, é dar-lhes uma tarefa: estudarem o conceito de energia, usando diversas fontes, mas que eles estudem em separado. Depois, juntá-los numa espécie de seminário e pedir-lhes explanarem o que cada um entendeu.
Se eu ainda labutasse na área," juro" que faria algo assim.
Quem sabe, com os jovens, vazios de conceitos batidos e dogmas, não pudéssemos aprender algo mais, a respeito.

Sds,

Victor.

----- Original Message -----
From: Maria Natália
To: ciencialist@yahoogrupos.com.br
Sent: Friday, December 17, 2004 1:24 AM
Subject: [ciencialist] Re: Tudo é energia



Homero:
E o químico dirá que mesmo aquela cadeira parada, em repouso
elativo, tem energia. Energia interna, energia potencial química e
energia das oscilações, ligações, vibrações, rotacões moléculares.
Um astrofísico falará da energia do vácuo e por aí fora.
É como o orientador de estágio pegar num sapato que coloca em cima
de secretária do estagiário dizendo: e agora com base neste sapato
faça a planificação para aula de química nível 11º ano. Difícil será
introduzir em programa existente, mas faz-se.
E aquele sapato tem tudo e até aula de português permite fazer.
A energia assim falada permite tudo o que a creatividade do
professor quiser.
Definição de energia dá tanta raiva como a definição de força. É que
se necessita do conceito bem esclarecido. Só na universidade se
entende tais definições.
Energia é assim uma expressão em verso de poeta.

Energia é tudo:
Está contigo
foge com a bola
Cavalga no som
no grão de trigo
é núcleo desnudo
dá cabo da tola
De físico bombom*

Um abraço
Maria Natália
* físico muita BOM

--- Em ciencialist@yahoogrupos.com.br, "Oraculo" <oraculo@a...>
escreveu
> Olá
>
> O problema é a definição de energia. Em geral, uma confusão com os
diversos sentidos do termo energia causa problemas ou permite frases
de alto impacto como essa "tudo é energia", embora vazia de
significado..:-) Como na famosa "navegar é preciso, viver não é
preciso" onde os diversos sentidos de precisão podem criar diversas
interpretações do poema.
>
> A energia de uma bateria, a energia de um corpo em movimento, a
energia produzida pela usina hidroelétrica, nada tem a ver com a
energia de um espetáculo de teatro, acordar se sentidno "enérgico",
ou a palestra enérgica do professor de filosofia.
>
> Se usarmos um conceito estrito para o termo energia (strictu
senso) acho que fica mais fácil analisar a frase "tudo é energia".
Primeiro, a capacidade de produzir trabalho é energia. Se produz
trabalho, cabe o uso do termo, se não produz, não cabe. Assim,
eletricidade é energia, mas uma palestra ou mesmo um elemento
químico não são.
>
> Mesmo agora que sabemos que E=mc2, isso não significa que podemos
abandonar os conceitos e definições e considerar "tudo" a mesma
coisa..:-) Tudo o que, cara-pálid, eu perguntaria ao autor..:-) O
fato de que é possível convertar algo em outra coisa não torna as
duas uma só.
>
> Acho que o problema da frase "tudo é energia" é mesmo as
decorrencias da mesma: e daí? O que o autor ou quem a apresenta quer
dizer com isso, quais as implicações decorrentes, caso se aceite
como verdade a afirmação? Tudo é energia ENTÃO podemos nos comunicar
telepaticamente? Tudo é energia, ENTÃO a homeopatia, usando o "tudo"
e a energia, funciona? Ou apenas tudo é energia, e sabemos como
relacionar a matéria e a energia em uma equação matematica correta
(e, como tudo que existe neste universo é matéria ou energia, esse é
um saber consideravel..:-)?
>
> Tudo é energia em geral é uma afirmação-escapatória, usada
esotericametne na falta de evidencias ou argumentos confiáveis..:-)
>
> Um abraço.
>
> Homero
>
> ----- Original Message -----
> From: brudna
> To: ciencialist@yahoogrupos.com.br
> Sent: Thursday, December 16, 2004 6:35 PM
> Subject: [ciencialist] Tudo é energia
>
>
>
> Fico confuso com o argumento ´Tudo é energia´.
>
> Onde estão os erros? Que conceitos falhos pode ter esse
argumento?
> Como aproveitar essa oportunidade para ensinar?
>
>
> Até
> Luís Brudna
>
>
>
>
>
>
> ##### ##### #####
>
> Para saber mais visite
> http://www.ciencialist.hpg.ig.com.br
>
>
> ##### ##### ##### #####
>
>
> Yahoo! Grupos, um serviço oferecido por:
>
>
>
>
>
>
>
> -------------------------------------------------------------------
-----------
> Links do Yahoo! Grupos
>
> a.. Para visitar o site do seu grupo na web, acesse:
> http://br.groups.yahoo.com/group/ciencialist/
>
> b.. Para sair deste grupo, envie um e-mail para:
> ciencialist-unsubscribe@yahoogrupos.com.br
>
> c.. O uso que você faz do Yahoo! Grupos está sujeito aos
Termos do Serviço do Yahoo!.
>
>
>
> [As partes desta mensagem que não continham texto foram removidas]





##### ##### #####

Para saber mais visite
http://www.ciencialist.hpg.ig.com.br


##### ##### ##### #####


Yahoo! Grupos, um serviço oferecido por:

São Paulo Rio de Janeiro Curitiba Porto Alegre Belo Horizonte Brasília




------------------------------------------------------------------------------
Links do Yahoo! Grupos

a.. Para visitar o site do seu grupo na web, acesse:
http://br.groups.yahoo.com/group/ciencialist/

b.. Para sair deste grupo, envie um e-mail para:
ciencialist-unsubscribe@yahoogrupos.com.br

c.. O uso que você faz do Yahoo! Grupos está sujeito aos Termos do Serviço do Yahoo!.



[As partes desta mensagem que não continham texto foram removidas]



SUBJECT: Re: [ciencialist] Gravidade
FROM: "JVictor" <jvoneto@uol.com.br>
TO: <ciencialist@yahoogrupos.com.br>
DATE: 17/12/2004 08:19

O Fran de Aquino é brasileiro e é ligado, como Professor, à Universidade informada abaixo. Acho que é maranhense também. Suas teorias são bastante questionáveis, embora bem embasadas matematicamente, via formalismo hamiltoniano. Ele estabelece a inercia dos corpos em termos de efeitos gravitacionais e da influência eletromagnética, postulando que massa inercial e massa gravitacional são diferentes, devido aos efeitos eletromgnéticos, não levados em consideração quando de suas medidas. A partir de uma expresão relacionando as duas massas, estabele novos conceitos de energia, envolvendo esse novo dado, o E.M. Do bojo de sua teoria, ele deduz a TRG, como caso particular! Encontra também que a massa pode ser negativa, e avança todo um conjunto de idéias baseado nesse "descoberta". A partir daí, o céu é o limite, em termos de imaginação. Coisas como produzir energia, pular para cima, contra a gravidade, sem esforço externo, revolução de toda a tecnologia, construção de naves que viajem a velocidades inimagináveis sem nem espichar as bochechas, explicação do "fato" de discos voadores estarem aquí e depois lá, a zilhões de anos luz, e outros etecéteras inconcebíveis, dentro de nossa estrutura de conhecimento científico. Certa vez consultei o Daniel Doro Ferrante e ele me disse que as teorias de Fran de Aquino não gozam de credibilidade junto aos pesquisadores dessas áreas. Particularmente, não aprofundei nada a respeito, por não ter tempo nem ânimo para coisas assim. Apenas fiz uma leitura rápida para sentir o que vem por lá. Por isso, a primeira opinião acima.
Você encontra a teoria completa em seu site.

Sds,

Victor.

----- Original Message -----
From: tipoalgo
To: ciencialist@yahoogrupos.com.br
Sent: Friday, December 17, 2004 6:45 AM
Subject: [ciencialist] Gravidade



Bom dia a todos,

Alguém poderia esclarecer se o professor Fran de Aquino é realmente
brasileiro?
A pergunta é porque ele na Internet ele é quase todo inglês!

E o que dizer de suas pesquisas?
Um brasileiro já domina a gravidade e poucos se aperceberam?

Algum mestre aqui da c_list poderia fazer um convite para ele nos
brindar com os seus conhecimentos?

Mais informações em:

http://users.elo.com.br/~deaquino//

(em algo em português)
http://users.elo.com.br/~deaquino//PORT.htm

"· Born: October 1, 1950 · Place of birth: Brazil · Residence: São
Luís - MA, Brazil
· Affiliation: Physics Department, Maranhão State University, São
Luís - MA, Brazil"


Abraços a todos

Tipoalgo





##### ##### #####

Para saber mais visite
http://www.ciencialist.hpg.ig.com.br


##### ##### ##### #####


Yahoo! Grupos, um serviço oferecido por:







------------------------------------------------------------------------------
Links do Yahoo! Grupos

a.. Para visitar o site do seu grupo na web, acesse:
http://br.groups.yahoo.com/group/ciencialist/

b.. Para sair deste grupo, envie um e-mail para:
ciencialist-unsubscribe@yahoogrupos.com.br

c.. O uso que você faz do Yahoo! Grupos está sujeito aos Termos do Serviço do Yahoo!.



[As partes desta mensagem que não continham texto foram removidas]



SUBJECT: Re: [ciencialist] Re: Tudo é energia
FROM: "E m i l i a n o C h e m e l l o" <chemelloe@yahoo.com.br>
TO: <ciencialist@yahoogrupos.com.br>
DATE: 17/12/2004 08:20

Caros amigos (e amigas),

Sugiro que se faça uma (re)leitura do texto retirado do famoso "The
Feynman Lectures on Physics" disponível on-line no endereço:

O que é energia segundo Richard Feynman
http://www.ucs.br/ccet/defq/naeq/material_didatico/textos_interativos_05.htm

Dizer que tudo "É"energia é um engano. Na minha opinião, podemos dizer que
tudo (este tudo se referindo a matéria) POSSUI alguma forma de energia.

[ ] 's do Emiliano Chemello
emiliano@quimica.net
http://www.quimica.net/emiliano
http://www.ucs.br/ccet/defq/naeq

" Rien ne se perd, rien ne se crée,
tout se transforme."

Antoine Laurent de Lavoisier (químico francês, 1743 - 1794)

----- Original Message -----
From: Maria Natália
To: ciencialist@yahoogrupos.com.br
Sent: Friday, December 17, 2004 1:24 AM
Subject: [ciencialist] Re: Tudo é energia



Homero:
E o químico dirá que mesmo aquela cadeira parada, em repouso
elativo, tem energia. Energia interna, energia potencial química e
energia das oscilações, ligações, vibrações, rotacões moléculares.
Um astrofísico falará da energia do vácuo e por aí fora.
É como o orientador de estágio pegar num sapato que coloca em cima
de secretária do estagiário dizendo: e agora com base neste sapato
faça a planificação para aula de química nível 11º ano. Difícil será
introduzir em programa existente, mas faz-se.
E aquele sapato tem tudo e até aula de português permite fazer.
A energia assim falada permite tudo o que a creatividade do
professor quiser.
Definição de energia dá tanta raiva como a definição de força. É que
se necessita do conceito bem esclarecido. Só na universidade se
entende tais definições.
Energia é assim uma expressão em verso de poeta.

Energia é tudo:
Está contigo
foge com a bola
Cavalga no som
no grão de trigo
é núcleo desnudo
dá cabo da tola
De físico bombom*

Um abraço
Maria Natália
* físico muita BOM

--- Em ciencialist@yahoogrupos.com.br, "Oraculo" <oraculo@a...>
escreveu
> Olá
>
> O problema é a definição de energia. Em geral, uma confusão com os
diversos sentidos do termo energia causa problemas ou permite frases
de alto impacto como essa "tudo é energia", embora vazia de
significado..:-) Como na famosa "navegar é preciso, viver não é
preciso" onde os diversos sentidos de precisão podem criar diversas
interpretações do poema.
>
> A energia de uma bateria, a energia de um corpo em movimento, a
energia produzida pela usina hidroelétrica, nada tem a ver com a
energia de um espetáculo de teatro, acordar se sentidno "enérgico",
ou a palestra enérgica do professor de filosofia.
>
> Se usarmos um conceito estrito para o termo energia (strictu
senso) acho que fica mais fácil analisar a frase "tudo é energia".
Primeiro, a capacidade de produzir trabalho é energia. Se produz
trabalho, cabe o uso do termo, se não produz, não cabe. Assim,
eletricidade é energia, mas uma palestra ou mesmo um elemento
químico não são.
>
> Mesmo agora que sabemos que E=mc2, isso não significa que podemos
abandonar os conceitos e definições e considerar "tudo" a mesma
coisa..:-) Tudo o que, cara-pálid, eu perguntaria ao autor..:-) O
fato de que é possível convertar algo em outra coisa não torna as
duas uma só.
>
> Acho que o problema da frase "tudo é energia" é mesmo as
decorrencias da mesma: e daí? O que o autor ou quem a apresenta quer
dizer com isso, quais as implicações decorrentes, caso se aceite
como verdade a afirmação? Tudo é energia ENTÃO podemos nos comunicar
telepaticamente? Tudo é energia, ENTÃO a homeopatia, usando o "tudo"
e a energia, funciona? Ou apenas tudo é energia, e sabemos como
relacionar a matéria e a energia em uma equação matematica correta
(e, como tudo que existe neste universo é matéria ou energia, esse é
um saber consideravel..:-)?
>
> Tudo é energia em geral é uma afirmação-escapatória, usada
esotericametne na falta de evidencias ou argumentos confiáveis..:-)
>
> Um abraço.
>
> Homero
>
> ----- Original Message -----
> From: brudna
> To: ciencialist@yahoogrupos.com.br
> Sent: Thursday, December 16, 2004 6:35 PM
> Subject: [ciencialist] Tudo é energia
>
>
>
> Fico confuso com o argumento ´Tudo é energia´.
>
> Onde estão os erros? Que conceitos falhos pode ter esse
argumento?
> Como aproveitar essa oportunidade para ensinar?
>
>
> Até
> Luís Brudna
>
>
>
>
>
>
> ##### ##### #####
>
> Para saber mais visite
> http://www.ciencialist.hpg.ig.com.br
>
>
> ##### ##### ##### #####
>
>
> Yahoo! Grupos, um serviço oferecido por:
>
>
>
>
>
>
>
> -------------------------------------------------------------------
-----------
> Links do Yahoo! Grupos
>
> a.. Para visitar o site do seu grupo na web, acesse:
> http://br.groups.yahoo.com/group/ciencialist/
>
> b.. Para sair deste grupo, envie um e-mail para:
> ciencialist-unsubscribe@yahoogrupos.com.br
>
> c.. O uso que você faz do Yahoo! Grupos está sujeito aos
Termos do Serviço do Yahoo!.
>
>
>
> [As partes desta mensagem que não continham texto foram removidas]





##### ##### #####

Para saber mais visite
http://www.ciencialist.hpg.ig.com.br


##### ##### ##### #####


Yahoo! Grupos, um serviço oferecido por:

São Paulo Rio de Janeiro Curitiba Porto Alegre Belo Horizonte Brasília





Links do Yahoo! Grupos

Para visitar o site do seu grupo na web, acesse:
http://br.groups.yahoo.com/group/ciencialist/

Para sair deste grupo, envie um e-mail para:
ciencialist-unsubscribe@yahoogrupos.com.br

O uso que você faz do Yahoo! Grupos está sujeito aos Termos do Serviço do
Yahoo!.



SUBJECT: Re: [ciencialist] Mais rápido que a luz?
FROM: "Alvaro Augusto - Lunabay" <alvaro@lunabay.com.br>
TO: <ciencialist@yahoogrupos.com.br>
DATE: 17/12/2004 10:02

Parece mais uma das célebres confusões entre velocidade de grupo e de
fase... O texto da Agência RBS diz que não houve violação da "casualidade".
O que é isso? Alguém ir trabalhar de terno e gravata na sexta-feira? :-)

[ ]s

Alvaro Augusto



----- Original Message -----
From: "rosevena3" <rosevena3@yahoo.com.br>
To: <ciencialist@yahoogrupos.com.br>
Sent: Friday, December 17, 2004 6:15 AM
Subject: [ciencialist] Mais rápido que a luz?




Pessoal não sei se o assunto já foi tratado aqui se já,
digam me onde.
Caso não peço ajuda para entender
------------------------------------
http://www.fisicaju.com.br/fisica/luz.htm

Experimento americano pode ter superado velocidade da luz

Quinta, 20 de julho de 2000, 11h08min

Um grupo de físicos americanos pode ter descoberto uma
velocidade
superior ao que antes se suponha ser a velocidade máxima do
universo -
a velocidade da luz. Durante gerações, os físicos supuseram
que não
havia nada mais veloz do que o movimento da luz no vazio, uma
velocidade de 300 mil quilômetros por segundo. Mas, em uma
experiência da Universidade de Princeton, Nova Jersey (EUA),
físicos
enviaram um raio de luz de laser através de vapor de césio numa
velocidade tão alta que saiu da câmera antes de terminar de
entrar.
Os cientistas dizem que esta foi a demonstração mais convincente
de
que a velocidade da luz pode ser superada, ao menos sob determinadas
circunstâncias em laboratório.
Os resultados serão publicados na edição de amanhã da
revista
Nature. O feito ainda não tem aplicações práticas imediatas,
mas
experimentos similares têm gerado entusiasmo na comunidade
internacional de físicos teóricos e ópticos. A
experiência de
Princeton põe à prova os limites da teoria da relatividade que
Albert
Einstein descobriu no príncipio do século, segundo a qual a
velocidade das partículas de luz no vazio é a única medida
absoluta
do universo. A velocidade de tudo mais seria relativa ao observador.
Nas circunstâncias cotidianas, um objeto não pode viajar numa
velocidade maior do que a da luz. Em possíveis aplicações
práticas, o
experimento poderia contribuir no desenvolvimento de computadores
mais velozes, que transportariam informações em partículas de
luz.

Agência RBS
Relatividade não foi violada
O experimento de Lijun Wang, A. Kuzmich e A. Dogariu, do NEC
Research Institute, publicado na Nature, 406, 277, de 20 jul 2000,
consiste na propagação de um pulso de laser com 3,7 microns de
extensão por uma cápsula de 6 cm contendo átomos de césio
especialmente excitados ao segundo nível, em uma situação
programada
para que o índice de refração esteja mudando rapidamente. Como
um
pulso de curta duração é necessariamente formado por um grande
número
de ondas com freqüências diferentes, o que se mede é a
interferência
das diferentes freqüências componentes do pulso, que mudam de fase
devido à variação do índice de refração. A fase relativa
muda fazendo
que a parte central do pulso se adiante 1,7% de sua extensão. Como
o
início do pulso inicial ocorre antes do centro do pico de
saída, não
há qualquer violação da casualidade. Como a velocidade de fase
da luz
não muda, só a velocidade de grupo do pulso, não há
qualquer violação
da relatividade. O próprio Lijun Wang, líder da equipe
explica: "Falando precisamente, é a velocidade de transferência
da
informação que é limitada pela velocidade da luz no vácuo."
Toda a
informação necessária sobre o pulso está contida em sua
minúscula
borda frontal. Assim que essa parte do pulso entra na câmara, os
átomos especialmente preparados podem começar a reproduzir
outro
pulso idêntico no outro lado da câmara. (27 jul 2000)

Fonte: Departamento de Astronomia do Instituto de Física da UFRGS
------------------------------------------
PERGUNTO:
Da pra ultrapassar c ou não?
Pois o "Falando precisamente, é a velocidade de transferência
da
informação que é limitada pela velocidade da luz no vácuo."
Me deixou
com a pulga atrás da orelha, junto com a pulga do neutrino.







##### ##### #####

Para saber mais visite
http://www.ciencialist.hpg.ig.com.br


##### ##### ##### #####
Links do Yahoo! Grupos












SUBJECT: eletrofluor
FROM: "E m i l i a n o C h e m e l l o" <chemelloe@yahoo.com.br>
TO: <ciencialist@yahoogrupos.com.br>
DATE: 17/12/2004 10:17

mais uma missão para o Sir Takata! : )

Contato Naeq:
Nome: Giulia
Email: giulia_bc@yahoo.com.br
Mensagem: Gostaria de saber se vocês teriam alguma informação sobre
eletrofluor relacionado à odontologia.Uma amiga,dentista, vai prestar
concurso este domingo(19/12) e não pudemos encontrar nada a respeito.
Provavelmente, o nome do referido em ingles é muito diferente, pois não
encontramos nada em arquivos de revistas odontológicas britânicas e
americanas.
Obrigada,
Giulia

[ ] 's do Emiliano Chemello
emiliano@quimica.net
http://www.quimica.net/emiliano
http://www.ucs.br/ccet/defq/naeq

" Rien ne se perd, rien ne se crée,
tout se transforme."

Antoine Laurent de Lavoisier (químico francês, 1743 - 1794)



SUBJECT: Re: Mais rápido que a luz?
FROM: Maria Natália <grasdic@hotmail.com>
TO: ciencialist@yahoogrupos.com.br
DATE: 17/12/2004 10:22


Rosevena:
´
Ah! João Magueijo? Ya meu. Malta, pessoal e galera falar muito dele
aqui.
Tu já leu?
Então toma e uma vez que as mensagens devem estar a meses luz e dá
trabalho:

http://tinyurl.com/6s5my

http://www.meguimaraes.com/imagensepalavras/arquivo/002838.html
http://frontwheeldrive.com/joao_magueijo.html
http://www-ctp.mit.edu/cosmo/y0001/magueijo_abs.html
http://revistagalileu.globo.com/Galileu/0,6993,ECT656750-1724,00.html
http://www.cvil.wustl.edu/~dawei/life/astronomy.htm

Entretanto tua dúvida seguiu para João
Um abraço
Maria Natália


--- Em ciencialist@yahoogrupos.com.br, "rosevena3" <rosevena3@y...>
escreveu
>
> Pessoal não sei se o assunto já foi tratado aqui se já,
> digam me onde.
> Caso não peço ajuda para entender
> ------------------------------------
> http://www.fisicaju.com.br/fisica/luz.htm
>
> Experimento americano pode ter superado velocidade da luz
>
> Quinta, 20 de julho de 2000, 11h08min
>
> Um grupo de físicos americanos pode ter descoberto uma
> velocidade
> superior ao que antes se suponha ser a velocidade máxima do
> universo –
> a velocidade da luz. Durante gerações, os físicos supuseram
> que não
> havia nada mais veloz do que o movimento da luz no vazio, uma
> velocidade de 300 mil quilômetros por segundo. Mas, em uma
> experiência da Universidade de Princeton, Nova Jersey (EUA),
> físicos
> enviaram um raio de luz de laser através de vapor de césio numa
> velocidade tão alta que saiu da câmera antes de terminar de
> entrar.
> Os cientistas dizem que esta foi a demonstração mais convincente
> de
> que a velocidade da luz pode ser superada, ao menos sob
determinadas
> circunstâncias em laboratório.
> Os resultados serão publicados na edição de amanhã da
> revista
> Nature. O feito ainda não tem aplicações práticas imediatas,
> mas
> experimentos similares têm gerado entusiasmo na comunidade
> internacional de físicos teóricos e ópticos. A
> experiência de
> Princeton põe à prova os limites da teoria da relatividade que
> Albert
> Einstein descobriu no príncipio do século, segundo a qual a
> velocidade das partículas de luz no vazio é a única medida
> absoluta
> do universo. A velocidade de tudo mais seria relativa ao
observador.
> Nas circunstâncias cotidianas, um objeto não pode viajar numa
> velocidade maior do que a da luz. Em possíveis aplicações
> práticas, o
> experimento poderia contribuir no desenvolvimento de computadores
> mais velozes, que transportariam informações em partículas de
> luz.
>
> Agência RBS
> Relatividade não foi violada
> O experimento de Lijun Wang, A. Kuzmich e A. Dogariu, do NEC
> Research Institute, publicado na Nature, 406, 277, de 20 jul 2000,
> consiste na propagação de um pulso de laser com 3,7 microns de
> extensão por uma cápsula de 6 cm contendo átomos de césio
> especialmente excitados ao segundo nível, em uma situação
> programada
> para que o índice de refração esteja mudando rapidamente. Como
> um
> pulso de curta duração é necessariamente formado por um grande
> número
> de ondas com freqüências diferentes, o que se mede é a
> interferência
> das diferentes freqüências componentes do pulso, que mudam de fase
> devido à variação do índice de refração. A fase relativa
> muda fazendo
> que a parte central do pulso se adiante 1,7% de sua extensão. Como
> o
> início do pulso inicial ocorre antes do centro do pico de
> saída, não
> há qualquer violação da casualidade. Como a velocidade de fase
> da luz
> não muda, só a velocidade de grupo do pulso, não há
> qualquer violação
> da relatividade. O próprio Lijun Wang, líder da equipe
> explica: "Falando precisamente, é a velocidade de transferência
> da
> informação que é limitada pela velocidade da luz no vácuo."
> Toda a
> informação necessária sobre o pulso está contida em sua
> minúscula
> borda frontal. Assim que essa parte do pulso entra na câmara, os
> átomos especialmente preparados podem começar a reproduzir
> outro
> pulso idêntico no outro lado da câmara. (27 jul 2000)
>
> Fonte: Departamento de Astronomia do Instituto de Física da UFRGS
> ------------------------------------------
> PERGUNTO:
> Da pra ultrapassar c ou não?
> Pois o "Falando precisamente, é a velocidade de transferência
> da
> informação que é limitada pela velocidade da luz no vácuo."
> Me deixou
> com a pulga atrás da orelha, junto com a pulga do neutrino.





SUBJECT: Re: [ciencialist] Mais rápido que a luz?
FROM: "JVictor" <jvoneto@uol.com.br>
TO: <ciencialist@yahoogrupos.com.br>
DATE: 17/12/2004 10:27

Álvaro,

Particularmente, não tenho a menor dúvida. Esses buscadores de prêmios e de causas perdidas deveriam desviar as verbas para coisas realmente produtivas, e não "descobrir" milagres que permitam trabalhar com um computador ultra, ultra rápido, antes mesmos que eles sejam construídos!...É nisso que dá violar a causalidade, tão impossível quanto ressuscitar, isso é o que procuram. Há alguns anos, já se publicou coisas assim, de uns holandeses, acho. Um matemático brasileiro, da Unicamp, encampou a idéia, ciscou, petiscou, assumiur os "resultados"experimentais como verdadeiros, empolgou-se com a foto publicada e escreveu um livro com soluções inéditas das eqs. do eletromagnetismo, tudo dentro dos conformes de suas idéias anti-relativísitcas. Algum tempo depois, lí um livro, o segundo volume das conferências sobre o trabalho de David Bohm, onde ele, um dos conferencistas, questionado, reconheceu o erro do tal experimento, e os etcéteras e tal de direito. Há outros exemplos. A TRG sempre resulta incólume, no fim das contas(nos dois sentidos). E veja que o próprio autor da Relatividade disse inúmeras vezes que não considerava sua teoria como definitiva. Das que inventaram até o momento, ela continua sendo a melhor, superior. Ela até poderá ser revista e complementada. Mas não acredito, de maneira nenhuma que um dia, no passado, eu venha namorar a minha mãe. Credo!, eu filho de mim mesmo?, putz!

Sds,

Victor.
----- Original Message -----
From: Alvaro Augusto - Lunabay
To: ciencialist@yahoogrupos.com.br
Sent: Friday, December 17, 2004 10:02 AM
Subject: Re: [ciencialist] Mais rápido que a luz?


Parece mais uma das célebres confusões entre velocidade de grupo e de
fase... O texto da Agência RBS diz que não houve violação da "casualidade".
O que é isso? Alguém ir trabalhar de terno e gravata na sexta-feira? :-)

[ ]s

Alvaro Augusto



----- Original Message -----
From: "rosevena3" <rosevena3@yahoo.com.br>
To: <ciencialist@yahoogrupos.com.br>
Sent: Friday, December 17, 2004 6:15 AM
Subject: [ciencialist] Mais rápido que a luz?




Pessoal não sei se o assunto já foi tratado aqui se já,
digam me onde.
Caso não peço ajuda para entender
------------------------------------
http://www.fisicaju.com.br/fisica/luz.htm

Experimento americano pode ter superado velocidade da luz

Quinta, 20 de julho de 2000, 11h08min

Um grupo de físicos americanos pode ter descoberto uma
velocidade
superior ao que antes se suponha ser a velocidade máxima do
universo -
a velocidade da luz. Durante gerações, os físicos supuseram
que não
havia nada mais veloz do que o movimento da luz no vazio, uma
velocidade de 300 mil quilômetros por segundo. Mas, em uma
experiência da Universidade de Princeton, Nova Jersey (EUA),
físicos
enviaram um raio de luz de laser através de vapor de césio numa
velocidade tão alta que saiu da câmera antes de terminar de
entrar.
Os cientistas dizem que esta foi a demonstração mais convincente
de
que a velocidade da luz pode ser superada, ao menos sob determinadas
circunstâncias em laboratório.
Os resultados serão publicados na edição de amanhã da
revista
Nature. O feito ainda não tem aplicações práticas imediatas,
mas
experimentos similares têm gerado entusiasmo na comunidade
internacional de físicos teóricos e ópticos. A
experiência de
Princeton põe à prova os limites da teoria da relatividade que
Albert
Einstein descobriu no príncipio do século, segundo a qual a
velocidade das partículas de luz no vazio é a única medida
absoluta
do universo. A velocidade de tudo mais seria relativa ao observador.
Nas circunstâncias cotidianas, um objeto não pode viajar numa
velocidade maior do que a da luz. Em possíveis aplicações
práticas, o
experimento poderia contribuir no desenvolvimento de computadores
mais velozes, que transportariam informações em partículas de
luz.

Agência RBS
Relatividade não foi violada
O experimento de Lijun Wang, A. Kuzmich e A. Dogariu, do NEC
Research Institute, publicado na Nature, 406, 277, de 20 jul 2000,
consiste na propagação de um pulso de laser com 3,7 microns de
extensão por uma cápsula de 6 cm contendo átomos de césio
especialmente excitados ao segundo nível, em uma situação
programada
para que o índice de refração esteja mudando rapidamente. Como
um
pulso de curta duração é necessariamente formado por um grande
número
de ondas com freqüências diferentes, o que se mede é a
interferência
das diferentes freqüências componentes do pulso, que mudam de fase
devido à variação do índice de refração. A fase relativa
muda fazendo
que a parte central do pulso se adiante 1,7% de sua extensão. Como
o
início do pulso inicial ocorre antes do centro do pico de
saída, não
há qualquer violação da casualidade. Como a velocidade de fase
da luz
não muda, só a velocidade de grupo do pulso, não há
qualquer violação
da relatividade. O próprio Lijun Wang, líder da equipe
explica: "Falando precisamente, é a velocidade de transferência
da
informação que é limitada pela velocidade da luz no vácuo."
Toda a
informação necessária sobre o pulso está contida em sua
minúscula
borda frontal. Assim que essa parte do pulso entra na câmara, os
átomos especialmente preparados podem começar a reproduzir
outro
pulso idêntico no outro lado da câmara. (27 jul 2000)

Fonte: Departamento de Astronomia do Instituto de Física da UFRGS
------------------------------------------
PERGUNTO:
Da pra ultrapassar c ou não?
Pois o "Falando precisamente, é a velocidade de transferência
da
informação que é limitada pela velocidade da luz no vácuo."
Me deixou
com a pulga atrás da orelha, junto com a pulga do neutrino.







##### ##### #####

Para saber mais visite
http://www.ciencialist.hpg.ig.com.br


##### ##### ##### #####
Links do Yahoo! Grupos












##### ##### #####

Para saber mais visite
http://www.ciencialist.hpg.ig.com.br


##### ##### ##### #####


Yahoo! Grupos, um serviço oferecido por:







------------------------------------------------------------------------------
Links do Yahoo! Grupos

a.. Para visitar o site do seu grupo na web, acesse:
http://br.groups.yahoo.com/group/ciencialist/

b.. Para sair deste grupo, envie um e-mail para:
ciencialist-unsubscribe@yahoogrupos.com.br

c.. O uso que você faz do Yahoo! Grupos está sujeito aos Termos do Serviço do Yahoo!.



---
Outgoing mail is certified Virus Free.
Checked by AVG anti-virus system (http://www.grisoft.com).
Version: 6.0.817 / Virus Database: 555 - Release Date: 15/12/04

[As partes desta mensagem que não continham texto foram removidas]



SUBJECT: ICR, Institute for CREATION Research, (Física para cristãos?)
FROM: Maria Natália <grasdic@hotmail.com>
TO: ciencialist@yahoogrupos.com.br
DATE: 17/12/2004 10:29


Será?
Isto me veio para às mãos:
http://www.icr.org/pubs/btg-a/btg-174a.htm
Analisemos pois.

bom fds
Maria Natália






SUBJECT: Professores de Física pra malhação, JÁ!
FROM: Maria Natália <grasdic@hotmail.com>
TO: ciencialist@yahoogrupos.com.br
DATE: 17/12/2004 10:31


Acho que chegou a hora de estarmos todos a pé pra curtir malhação.
Já se preparou? Ele tá aí:
http://www.physics2005.org/
E quem não malha não é bom físico
Maria Natália, malhando já






SUBJECT: Ação e ReAção
FROM: marcelo ferrari <emailferrari@yahoo.com.br>
TO: emailferrari@yahoo.com.br
DATE: 17/12/2004 11:04

Se eu jogo uma bola contra a parede, ela vai e volta, ação e reação.
A pergunta é: onde termina a ação e começa a reação?



__________________________________________________
Converse com seus amigos em tempo real com o Yahoo! Messenger
http://br.download.yahoo.com/messenger/

[As partes desta mensagem que não continham texto foram removidas]



SUBJECT: Re: [ciencialist] Re: Tudo é energia
FROM: "Luiz Ferraz Netto" <leobarretos@uol.com.br>
TO: <ciencialist@yahoogrupos.com.br>
DATE: 17/12/2004 11:21

De R. Takata:

[ ... ] quantitativas) entre esses outros aspectos e a energia. Por exemplo, a
famosa formula einsteniana: E = m.c^2. (Claro q. ainda existira' a
discussao de se essa relacao significa identidadade. Por exemplo,
podemos ter uma relacao US$ 1 = R$ 2,70 - o q. nao significa q. dolar
e real sejam identicos, sejam a mesma coisa, [ ... ]

Será melhor escrever assim:

1 US$ (a unidade sempre depois do número de medida) = 2,70 (fator de proporcionalidade, adimensional) x 1 R$

generalizando: US$ = 2,70.R$ , comparando grandezas de mesma espécie (monetárias). A grandeza monetária US$ é 2,70 vezes maior que a grandeza monetária R$. Aliás, 'toda grandeza US' é maior que a correspondente 'grandeza Br', veja-se, por exemplo:

1 " = 2,54 x 1 cm

[e não se escreve: "1 = 2,54 . cm1];

o próprio US está mais alto que o Br, basta ver o mapa mundi, o US está lá em cima e nós aqui em baixo.
Tenho dito e uma moeda de 1 real,

[o 'dito' é um objeto?]

[]'

Léo


SUBJECT: Re: [ciencialist] Tudo é energia
FROM: "Luiz Ferraz Netto" <leobarretos@uol.com.br>
TO: <ciencialist@yahoogrupos.com.br>
DATE: 17/12/2004 11:38

De: "Oraculo"
[ ... ]
Se usarmos um conceito estrito para o termo energia (strictu senso) acho que fica mais fácil analisar a frase "tudo é energia". Primeiro, a capacidade de produzir trabalho é energia. Se produz trabalho, cabe o uso do termo, se não produz, não cabe. Assim, eletricidade é energia, mas uma palestra ou mesmo um elemento químico não são.

Léo: Trabalho é grandeza inerentemente associada à força; sempre será "trabalho de uma força"; logo, "tudo é força"?
Por outro lado (tudo tem lado, até as ilhas, 'um monte de terra cercada de água por todos os lados', como dizia minha fessora de geografia --- e até hoje procuro onde está a água do 'lado de baixo e do lado de cima da ilha") ELETRICIDADE não é nada senão o nome dado a um conjunto de fenômenos (como óptica, acústica etc.). Um Curso de Eletricidade, um Curso de Óptica, "Eletricidade - Volume 3" ... fazem sentido. Mas, não queira tratar a 'eletricidade' como grandeza física e muito menos titula-la de energia; a eletricidade não corre pelos fios, a eletricidade não sai do gerador e vai para a lâmpada, a eletricidade não se gasta ao passar pela lâmpada, a eletricidade não retorna ao gerador, do mesmo modo como as ópticas não saem das lanternas e vão bater nas paredes!

> "Tudo é energia, em geral, é uma afirmação-escapatória, usada esotericametne na falta de evidencias ou argumentos confiáveis..:-)"

'O mero' fato de usar a palavra 'tudo' já nos transporta para as pseudociências.

[]'

o mero Léo,




SUBJECT: Re: [ciencialist] Re: Mais rápido que a luz?
FROM: "Alvaro Augusto - Lunabay" <alvaro@lunabay.com.br>
TO: <ciencialist@yahoogrupos.com.br>
DATE: 17/12/2004 12:18

Não conheço o trabalho de Mr McQueijo em detalhes, mas me parece que ele não
invalida a relatividade especial. O que ele diz é a luz pode ter sido mais
rápida no passado, e que estaria ficando mais "cansada" a cada dia que
passa. Isso não invalidade a hipótese da invariância da luz. Notem, essa
hipótese diz apenas que a velocidade da luz deve ser a mesma em todos os
referenciais, não que deva ser constante no tempo. A velocidade pode mudar
ao longo do tempo, desde que o faça em todos lugares ao mesmo tempo. Agora,
qual mecanismo assegura que isso aconteça em todos os lugares ao mesmo
termpo? Sei lá! Deve ser o mesmo mecanismo que assegura que a velocidade da
luz é um invariante!!!

Aqui desse lado do Atlântico nós também temos muita gente querendo ocupar o
lugar de Einstein, mostrando que ele estava errado. Esse é o verdadeiro
graal da física. Dizem que o graal é a busca pela unificação, mas não é. A
unificação é uma tentativa de mostrar que Einstein estava certo, enquanto
mostrar que Einstein estava errado conferiria o verdaderio prêmio de 100
milhões de dólares, mais a garantia de ter o nome inscrito na eternidade, ou
talvez além dela...

[ ]s

Alvaro Augusto



----- Original Message -----
From: "Maria Natália" <grasdic@hotmail.com>
To: <ciencialist@yahoogrupos.com.br>
Sent: Friday, December 17, 2004 10:22 AM
Subject: [ciencialist] Re: Mais rápido que a luz?




Rosevena:
´
Ah! João Magueijo? Ya meu. Malta, pessoal e galera falar muito dele
aqui.
Tu já leu?
Então toma e uma vez que as mensagens devem estar a meses luz e dá
trabalho:

http://tinyurl.com/6s5my

http://www.meguimaraes.com/imagensepalavras/arquivo/002838.html
http://frontwheeldrive.com/joao_magueijo.html
http://www-ctp.mit.edu/cosmo/y0001/magueijo_abs.html
http://revistagalileu.globo.com/Galileu/0,6993,ECT656750-1724,00.html
http://www.cvil.wustl.edu/~dawei/life/astronomy.htm

Entretanto tua dúvida seguiu para João
Um abraço
Maria Natália


--- Em ciencialist@yahoogrupos.com.br, "rosevena3" <rosevena3@y...>
escreveu
>
> Pessoal não sei se o assunto já foi tratado aqui se já,
> digam me onde.
> Caso não peço ajuda para entender
> ------------------------------------
> http://www.fisicaju.com.br/fisica/luz.htm
>
> Experimento americano pode ter superado velocidade da luz
>
> Quinta, 20 de julho de 2000, 11h08min
>
> Um grupo de físicos americanos pode ter descoberto uma
> velocidade
> superior ao que antes se suponha ser a velocidade máxima do
> universo -
> a velocidade da luz. Durante gerações, os físicos supuseram
> que não
> havia nada mais veloz do que o movimento da luz no vazio, uma
> velocidade de 300 mil quilômetros por segundo. Mas, em uma
> experiência da Universidade de Princeton, Nova Jersey (EUA),
> físicos
> enviaram um raio de luz de laser através de vapor de césio numa
> velocidade tão alta que saiu da câmera antes de terminar de
> entrar.
> Os cientistas dizem que esta foi a demonstração mais convincente
> de
> que a velocidade da luz pode ser superada, ao menos sob
determinadas
> circunstâncias em laboratório.
> Os resultados serão publicados na edição de amanhã da
> revista
> Nature. O feito ainda não tem aplicações práticas imediatas,
> mas
> experimentos similares têm gerado entusiasmo na comunidade
> internacional de físicos teóricos e ópticos. A
> experiência de
> Princeton põe à prova os limites da teoria da relatividade que
> Albert
> Einstein descobriu no príncipio do século, segundo a qual a
> velocidade das partículas de luz no vazio é a única medida
> absoluta
> do universo. A velocidade de tudo mais seria relativa ao
observador.
> Nas circunstâncias cotidianas, um objeto não pode viajar numa
> velocidade maior do que a da luz. Em possíveis aplicações
> práticas, o
> experimento poderia contribuir no desenvolvimento de computadores
> mais velozes, que transportariam informações em partículas de
> luz.
>
> Agência RBS
> Relatividade não foi violada
> O experimento de Lijun Wang, A. Kuzmich e A. Dogariu, do NEC
> Research Institute, publicado na Nature, 406, 277, de 20 jul 2000,
> consiste na propagação de um pulso de laser com 3,7 microns de
> extensão por uma cápsula de 6 cm contendo átomos de césio
> especialmente excitados ao segundo nível, em uma situação
> programada
> para que o índice de refração esteja mudando rapidamente. Como
> um
> pulso de curta duração é necessariamente formado por um grande
> número
> de ondas com freqüências diferentes, o que se mede é a
> interferência
> das diferentes freqüências componentes do pulso, que mudam de fase
> devido à variação do índice de refração. A fase relativa
> muda fazendo
> que a parte central do pulso se adiante 1,7% de sua extensão. Como
> o
> início do pulso inicial ocorre antes do centro do pico de
> saída, não
> há qualquer violação da casualidade. Como a velocidade de fase
> da luz
> não muda, só a velocidade de grupo do pulso, não há
> qualquer violação
> da relatividade. O próprio Lijun Wang, líder da equipe
> explica: "Falando precisamente, é a velocidade de transferência
> da
> informação que é limitada pela velocidade da luz no vácuo."
> Toda a
> informação necessária sobre o pulso está contida em sua
> minúscula
> borda frontal. Assim que essa parte do pulso entra na câmara, os
> átomos especialmente preparados podem começar a reproduzir
> outro
> pulso idêntico no outro lado da câmara. (27 jul 2000)
>
> Fonte: Departamento de Astronomia do Instituto de Física da UFRGS
> ------------------------------------------
> PERGUNTO:
> Da pra ultrapassar c ou não?
> Pois o "Falando precisamente, é a velocidade de transferência
> da
> informação que é limitada pela velocidade da luz no vácuo."
> Me deixou
> com a pulga atrás da orelha, junto com a pulga do neutrino.





##### ##### #####

Para saber mais visite
http://www.ciencialist.hpg.ig.com.br


##### ##### ##### #####
Links do Yahoo! Grupos












SUBJECT: Re: [ciencialist] Ação e ReAção
FROM: "Luiz Ferraz Netto" <leobarretos@uol.com.br>
TO: <ciencialist@yahoogrupos.com.br>, <emailferrari@yahoo.com.br>
DATE: 17/12/2004 12:27

De: "marcelo ferrari"

"Se eu jogo uma bola contra a parede, ela vai e volta, ação e reação.
A pergunta é: onde termina a ação e começa a reação?"

A descrição completa do fenômeno implica na teoria dos choques mecânicos, parcialmente elásticos, com 0 < e < 1 (e=coeficiente de restituição).
Chamemos de ação (A) a força que a bola aplica contra a parede e reação (R) a força que a parede aplica contra a bola. Em cada instante, ambas (A e R) têm a mesma intensidade, mesma direção, sentidos opostos e, obviamente, os pontos de aplicações são distintos, um na parede (o da ação) e outro na bola (o da reação).
No instante zero, onde se inicia o choque, o par é nulo, a pressão no interior da bola é PB, a tensão elástica da câmara é TE, a pressão atmosférica é Pat e a temperatura ambiente (igual à do ar da bola, no início) é TA.
Iniciado o choque a pequena A deforma imperceptivelmente a parede e a pequena R deforma a bola, achatando-a, comprimindo o ar, aumentando PB, aumentando TE, aumentando TA interno ... e diminuindo a velocidade do CM da bola. Quando essa velocidade do CM se anula, a A (logo R) é máxima e começa a fase de recuperação da forma da bola. Vale a pena ver tudo isso em fotos estroboscópicas.
E assim a coisa evolui até novamente A (e R) se anularem (fim do choque), bola adquirir velocidade de afastamento menor que aquela de aproximação, ar no interior da bola mais 'quente', etc. Parte da perda de energia cinética é convertida em som.
Galvanômetros balísticos podem medir o tempo de interação bola-parede e assim nos dá a conhecer quanto tempo dura a existência do par ação/reação.
Não tem sentido a pergunta "onde termina a ação e começa a reação", uma vez que são interações simultâneas; as duas começam e terminam juntas, como se pode constatar pelo relato acima.
Por outro lado, é preciso muito cuidado com a redação científica. A frase inicial ["Se eu jogo uma bola contra a parede, ela vai e volta, ação e reação.], por exemplo, pode dar falsa interpretação ao leitor se ele associar "vai" com "ação" e "volta" com "reação".
Se a bola 'grudasse' na parede, ou simplesmente caísse ao chão (bola mucha jogada na parede) --- (eliminando a 'volta') ---, ainda assim existiriam "ação e reação".
Tais tipos de frases sempre levam à má interpretação, veja essa: Um barquinho bateu num transatlântico e afundou --- mas, se ação e reação sempre são iguais, por que o transatlântico tb não afundou?

[]'
Léo





SUBJECT: Re: [ciencialist] Tudo é energia
FROM: "Oraculo" <oraculo@atibaia.com.br>
TO: <ciencialist@yahoogrupos.com.br>
DATE: 17/12/2004 13:20

Olá Leo

Recebi e registrado..:-) Obrigado pela correção, vou modificar os exemplos para os próximos embates com o "tudo é energia" (sempre recorrente..:-). Realmente, usei eletricidade de forma um tanto equivocada neste caso, um modo leigo de tratar da coitada. Espero que o sentido geral tenha ficado claro, pelo menos..:-)

Um abraço.

Homero

----- Original Message -----
From: Luiz Ferraz Netto
To: ciencialist@yahoogrupos.com.br
Sent: Friday, December 17, 2004 11:38 AM
Subject: Re: [ciencialist] Tudo é energia


De: "Oraculo"
[ ... ]
Se usarmos um conceito estrito para o termo energia (strictu senso) acho que fica mais fácil analisar a frase "tudo é energia". Primeiro, a capacidade de produzir trabalho é energia. Se produz trabalho, cabe o uso do termo, se não produz, não cabe. Assim, eletricidade é energia, mas uma palestra ou mesmo um elemento químico não são.

Léo: Trabalho é grandeza inerentemente associada à força; sempre será "trabalho de uma força"; logo, "tudo é força"?
Por outro lado (tudo tem lado, até as ilhas, 'um monte de terra cercada de água por todos os lados', como dizia minha fessora de geografia --- e até hoje procuro onde está a água do 'lado de baixo e do lado de cima da ilha") ELETRICIDADE não é nada senão o nome dado a um conjunto de fenômenos (como óptica, acústica etc.). Um Curso de Eletricidade, um Curso de Óptica, "Eletricidade - Volume 3" ... fazem sentido. Mas, não queira tratar a 'eletricidade' como grandeza física e muito menos titula-la de energia; a eletricidade não corre pelos fios, a eletricidade não sai do gerador e vai para a lâmpada, a eletricidade não se gasta ao passar pela lâmpada, a eletricidade não retorna ao gerador, do mesmo modo como as ópticas não saem das lanternas e vão bater nas paredes!

> "Tudo é energia, em geral, é uma afirmação-escapatória, usada esotericametne na falta de evidencias ou argumentos confiáveis..:-)"

'O mero' fato de usar a palavra 'tudo' já nos transporta para as pseudociências.

[]'

o mero Léo,




##### ##### #####

Para saber mais visite
http://www.ciencialist.hpg.ig.com.br


##### ##### ##### #####


Yahoo! Grupos, um serviço oferecido por:







------------------------------------------------------------------------------
Links do Yahoo! Grupos

a.. Para visitar o site do seu grupo na web, acesse:
http://br.groups.yahoo.com/group/ciencialist/

b.. Para sair deste grupo, envie um e-mail para:
ciencialist-unsubscribe@yahoogrupos.com.br

c.. O uso que você faz do Yahoo! Grupos está sujeito aos Termos do Serviço do Yahoo!.



[As partes desta mensagem que não continham texto foram removidas]



SUBJECT: Re: Ação e ReAção
FROM: marcelo ferrari <emailferrari@yahoo.com.br>
TO: yhoo/grp <ciencialist@yahoogrupos.com.br>
DATE: 17/12/2004 14:55

"Se eu jogo uma bola contra a parede, ela vai e volta, ação e reação.
A pergunta é: onde termina a ação e começa a reação?"


Luiz, grato pela explicação. Explicou até mais do que eu esperava. Porém, para entender melhor o que estou querendo entender, preciso investigar mais alguns dos pontos que colocou. Se puder continuar me ajudando, agradeço.

LUIZ >>> Chamemos de ação (A) a força que a bola aplica contra a parede e reação (R) a força que a parede aplica contra a bola. Em cada instante, ambas (A e R) têm a mesma intensidade, mesma direção, sentidos opostos.

Então, o que diferencia A de R é apenas o sentido? Ou seja, a ação termina no ponto onde ela muda de sentido, é isto?

LUIZ >>> e, obviamente, os pontos de aplicações são distintos, um na parede (o da ação) e outro na bola (o da reação).

Vc poderia me explicar o que significa "ponto de aplicação"?

LUIZ >>> No instante zero, onde se inicia o choque, o par é nulo.

Interressante, neste instante nulo, o que existe? Existe força? Movimento? Algo?

LUIZ >>> Não tem sentido a pergunta "onde termina a ação e começa a reação", uma vez que são interações simultâneas; as duas começam e terminam juntas, como se pode constatar pelo relato acima.

Entendo, mas se A e R são distintos e subsequentes, então existem um fim em A para que comece R. A pregunta é sobre este fim. Faz sentido?

LUIZ >>> Por outro lado, é preciso muito cuidado com a redação científica. A frase inicial ["Se eu jogo uma bola contra a parede, ela vai e volta, ação e reação.], por exemplo, pode dar falsa interpretação ao leitor se ele associar "vai" com "ação" e "volta" com "reação".

Estou partindo esta associação, pois é uma maneira de materializar e visualizar a lei. Entendo a o que vc quer dizer, mas creio que neste caso, a vizualização ajuda.


Marcelo Ferrari.




---------------------------------
Yahoo! Mail - Agora com 250MB de espaço gratuito. Abra uma conta agora!

[As partes desta mensagem que não continham texto foram removidas]



SUBJECT: Re: Mais rápido que a luz?
FROM: "dfahlb" <dfahlb@yahoo.com>
TO: ciencialist@yahoogrupos.com.br
DATE: 17/12/2004 15:00


--- Em ciencialist@yahoogrupos.com.br, "rosevena3" <rosevena3@y...>
escreveu
>
> Pessoal não sei se o assunto já foi tratado aqui se já,
> digam me onde.
> Caso não peço ajuda para entender
> ------------------------------------
> http://www.fisicaju.com.br/fisica/luz.htm

Olá Rosevena3,

Antes de mais nada, existem teorias em que se supõe que conforme o
Universo fica "mais velho" o valor das constantes poderiam mudar (sei
que soa engraçado "constantes variando no tempo" :o) ). Ainda que
possam parecer estranhas, tais teorias tem sido bastante trabalhadas
mas no entanto são ainda novas e, como toda teoria nova, se
confirmada nos testes iniciais iriam passar por uma série de
desenvolvimentos ainda.

Foi mais ou menos isso (sobre a velocidade da luz variar no tempo)
que Alvaro Augusto - Lunabay disse em
http://br.groups.yahoo.com/group/ciencialist/message/43260 . Não me
lembro agora quem foi o primeiro a propor que as constantes da
Natureza poderiam mudar conforme o Universo fosse evoluindo. De
qualquer forma, os que tenho ouvido falar sobre dizem sempre como o
Alvaro disse "Notem, essa hipótese [da Relatividade de Einstein]diz
apenas que a velocidade da luz deve ser a mesma em todos os
referenciais, não que deva ser constante no tempo."

Entretando me chamou a atenção em alguns pontos do artigo que você
enviou, vamos a ele:

" durante gerações, os físicos supuseram
que não havia nada mais veloz do que o movimento da luz no
vazioDurante gerações, os físicos supuseram que não havia nada mais
veloz do que o movimento da luz NO VAZIO [grifo meu][...]"

Mais para frente diz que " físicos enviaram um raio de luz de laser
através de vapor de césio"

Espere aí!!! É sabido que a velocidade da luz muda quando muda de
meio (refração) e a conjectura inicial era no vazio. Se a velocidade
da luz não é a maior em alguns determinados meios teria a ver com a
interação da luz com a matéria. Uma coisa é a velocidade da luz no
vazio e outra é interagindo com a matéria.

Ou seja, como a luz interage com cargas se você a fizer passar num
meio com muitas cargas, ela terá sua velocidade diminuída. Partículas
que, nesse mesmo meio, interajam pouco, podem ir mais rápido que a
velocidade da luz. E existem tais partículas! Um exemplo são os
neutrinos. Os neutrinos são partículas de massa bem pequena, tão
pequena que um elétron teria a massa de mais de 170 mil neutrinos! O
spin do neutrino é meio (é um férmion, portanto).

Entretanto, a seção de choque do neutrino é bem pequena (interage
pouco com a matéria). Tão pequena que teria-se uma probabilidade bem
alta de eles atravessarem a Terra inteira!!! A luz obviamente não
conseguiria tal feita.

Então, você teria nitidamente, meios opacos à luz e transparentes aos
neutrinos. E outros meios no qual a velocidade da luz seria não-nula
mas menor que as dos neutrinos, tendo isso a ver com a interação
entre a luz com a matéria e os neutrinos com a matéria.

Entretanto, no vácuo, a velocidade da luz é a maior.

Voltando ao artigo que você enviou:

" Relatividade não foi violada "

Os próprios experimentais dizem isso. Experiências outras sobre
velocidades superluminais já foram feitas. Por exemplo:
http://www.nature.com/cgi-taf/DynaPage.taf?
file=/nature/journal/v429/n6987/full/nature02586_fs.html

http://www.nature.com/dynasearch/app/dynasearch.taf

etc.







SUBJECT: Re: Mais rápido que a luz? rosevena3
FROM: "dfahlb" <dfahlb@yahoo.com>
TO: ciencialist@yahoogrupos.com.br
DATE: 17/12/2004 15:03


Re: Mais rápido que a luz?

--- Em ciencialist@y..., "rosevena3" <rosevena3@y...>
escreveu
>
> Pessoal não sei se o assunto já foi tratado aqui se já,
> digam me onde.
> Caso não peço ajuda para entender
> ------------------------------------
> http://www.fisicaju.com.br/fisica/luz.htm

Olá Rosevena3,

Antes de mais nada, existem teorias em que se supõe que conforme o
Universo fica "mais velho" o valor das constantes poderiam mudar (sei
que soa engraçado "constantes variando no tempo" :o) ). Ainda que
possam parecer estranhas, tais teorias tem sido bastante trabalhadas
mas no entanto são ainda novas e, como toda teoria nova, se
confirmada nos testes iniciais iriam passar por uma série de
desenvolvimentos ainda.

Foi mais ou menos isso (sobre a velocidade da luz variar no tempo)
que Alvaro Augusto - Lunabay disse em
http://br.groups.yahoo.com/group/ciencialist/message/43260 . Não me
lembro agora quem foi o primeiro a propor que as constantes da
Natureza poderiam mudar conforme o Universo fosse evoluindo. De
qualquer forma, os que tenho ouvido falar sobre dizem sempre como o
Alvaro disse "Notem, essa hipótese [da Relatividade de Einstein]diz
apenas que a velocidade da luz deve ser a mesma em todos os
referenciais, não que deva ser constante no tempo."

Entretando me chamou a atenção em alguns pontos do artigo que você
enviou, vamos a ele:

" durante gerações, os físicos supuseram
que não havia nada mais veloz do que o movimento da luz no
vazioDurante gerações, os físicos supuseram que não havia nada mais
veloz do que o movimento da luz NO VAZIO [grifo meu][...]"

Mais para frente diz que " físicos enviaram um raio de luz de laser
através de vapor de césio"

Espere aí!!! É sabido que a velocidade da luz muda quando muda de
meio (refração) e a conjectura inicial era no vazio. Se a velocidade
da luz não é a maior em alguns determinados meios teria a ver com a
interação da luz com a matéria. Uma coisa é a velocidade da luz no
vazio e outra é interagindo com a matéria.

Ou seja, como a luz interage com cargas se você a fizer passar num
meio com muitas cargas, ela terá sua velocidade diminuída. Partículas
que, nesse mesmo meio, interajam pouco, podem ir mais rápido que a
velocidade da luz. E existem tais partículas! Um exemplo são os
neutrinos. Os neutrinos são partículas de massa bem pequena, tão
pequena que um elétron teria a massa de mais de 170 mil neutrinos! O
spin do neutrino é meio (é um férmion, portanto).

Entretanto, a seção de choque do neutrino é bem pequena (interage
pouco com a matéria). Tão pequena que teria-se uma probabilidade bem
alta de eles atravessarem a Terra inteira!!! A luz obviamente não
conseguiria tal feita.

Então, você teria nitidamente, meios opacos à luz e transparentes aos
neutrinos. E outros meios no qual a velocidade da luz seria não-nula
mas menor que as dos neutrinos, tendo isso a ver com a interação
entre a luz com a matéria e os neutrinos com a matéria.

Entretanto, no vácuo, a velocidade da luz é a maior.

Voltando ao artigo que você enviou:

" Relatividade não foi violada "

Os próprios experimentais dizem isso. Experiências outras sobre
velocidades superluminais já foram feitas. Por exemplo:
http://www.nature.com/cgi-taf/DynaPage.taf?
file=/nature/journal/v429/n6987/full/nature02586_fs.html

http://www.nature.com/dynasearch/app/dynasearch.taf

etc.

Até







SUBJECT: Re: Mais rápido que a luz? rosevena3
FROM: "dfahlb" <dfahlb@yahoo.com>
TO: ciencialist@yahoogrupos.com.br
DATE: 17/12/2004 15:09


Esqueci-me de dizer sobre os neutrinos em

"Ou seja, como a luz interage com cargas se você a fizer passar num
meio com muitas cargas, ela terá sua velocidade diminuída. Partículas
que, nesse mesmo meio, interajam pouco, podem ir mais rápido que a
velocidade da luz. E existem tais partículas! Um exemplo são os
neutrinos. Os neutrinos são partículas de massa bem pequena, tão
pequena que um elétron teria a massa de mais de 170 mil neutrinos! O
spin do neutrino é meio (é um férmion, portanto)."

que os neutrinos não tem carga. Isso é um dos motivos que ele
interage pouco com a matéria.

até







SUBJECT: Re: Gravidade
FROM: "tipoalgo" <tipoalgo@bol.com.br>
TO: ciencialist@yahoogrupos.com.br
DATE: 17/12/2004 15:12


Olá a Vitor e a todos,

Obrigado pelo seu retorno.

O professor Fran, como é conhecido (acho que é "Fran" de Francisco),
parece seguir o método científico em todas as suas fases, incluindo
modelos práticos e análise de resultados. O que está faltando então
para o Santo Graal aparecer? Ou será apenas mais um caso de engodo
cientificamente apresentado. Ou será estória para inglês ver, dai o
idioma principal do seu sítio ser em inglês.
Assumo o meu despreparo para julgar o trabalho do professor Fran de
Aquino; tempo até que eu tenho, mas também é só; por isso estou
recorrendo aos mestres da lista. Já vi muitos (em sítios diversos)
irem contra os seus estudos, mas ele não responde a nada. Será que
essa foi a forma que ele encontrou para manter a sua teoria intacta?

Abraços

Tipoalgo






SUBJECT: Física quântica causa interpretações diferentes entre cientistas
FROM: "Cyberlander" <mrcyberlander@i12.com>
TO: <Undisclosed-Recipient:;>
DATE: 17/12/2004 15:56

repassando...

Física quântica causa interpretações diferentes entre cientistas
MARCELO GLEISER


Sem a menor dúvida, o mundo dos átomos é muito diferente do nosso.

As três primeiras décadas do século 20 foram marcadas por uma combinação de debates angustiados e idéias geniais propostas por um grupo de cientistas que incluía Werner Heisenberg, Niels Bohr, Erwin Schrödinger, Max Planck, Albert Einstein, entre muitos outros. A angústia vinha da crise pela qual passava a chamada física clássica, que, apesar de tão eficiente na descrição das coisas que ocorrem à nossa volta, era praticamente inútil para explicar o que ocorria com os átomos.

Pode-se dizer que a revolução quântica, o resultado dessas várias idéias geniais, foi imposta contra a vontade dos físicos, uma revolução provocada pelo desenvolvimento de tecnologias e técnicas de laboratório que permitiram a exploração de toda uma nova realidade física, invisível aos nossos olhos. Esse ponto é muito importante: na história das ciências naturais, a maioria das revoluções foi causada pela descoberta de novas tecnologias e instrumentação.

A revolução quântica, devido à sua excentricidade, causa grandes confusões de interpretação, especialmente quando os seus conceitos são usados fora de contexto. Para explicar os resultados obtidos no laboratório, os pioneiros da física quântica criaram toda uma nova linguagem, apropriada ao que ocorre em sistemas de dimensões atômicas e subatômicas.

Por exemplo, o elétron não é descrito como uma partícula de posição bem determinada no espaço ou como uma onda com uma posição indeterminada no espaço, mas como sendo potencialmente partícula e onda: a realidade física do elétron e de todas as outras partículas de matéria e radiação é determinada pelo ato de observar. Quem determina se o elétron é partícula ou onda é o observador, na medida em que ele interage com o elétron. De fato, antes de o elétron ser medido, ou seja, antes da interação entre o observador e o observado, não se pode nem dizer que o elétron existe. No mundo quântico, entidades só existem quando medidas por um observador (ou melhor, por um aparelho).

É fácil ver como a dualidade partícula-onda pode ser distorcida fora de contexto. Por exemplo: "Ah! Então, sem observadores, a realidade não existe. Mais ainda, como a realidade é definida pelo observador por meio do ato de observar, e como o observador carrega consigo a sua própria subjetividade, a essência fundamental da realidade é subjetiva, dependente do observador."

A consequência direta dessa interpretação é pôr o homem no centro do cosmo, ao menos na medida em que somos aqueles que têm consciência do que significa observar: a realidade física passa a ser consequência de nossa existência. Pior ainda: como cada observador define a própria realidade, é impossível termos uma realidade universal. Tudo passa a ser ameno a interpretações, a desconstruções subjetivas do mundo e dos seus significados. A própria ciência se torna vítima do seu sucesso: afinal, se o seu objetivo é descrever a realidade e essa realidade é subjetiva, devem existir tantas ciências quanto há observadores. A ciência se torna inútil.

Outra consequência interessante da má interpretação da física quântica é que ela implica um holismo, uma conexão entre tudo o que existe, entre as nossas mentes e o Universo, desde as suas partículas mais fundamentais até as galáxias mais distantes. Inevitavelmente, esse holismo é visto como uma dimensão espiritual da física, uma redescoberta de ensinamentos antigos, em particular aqueles das religiões orientais.

Mesmo físicos, como Fritjof Capra, caem vítimas dessa tentação. O problema com esses abusos do quantum é usar conceitos aplicáveis a uma realidade que existe em dimensões de bilionésimos de metro a situações do nosso cotidiano, que é completamente removido da realidade quântica. As regras que regem as nossas interpretações do que é um elétron ou de como ele se comporta em um átomo são completamente irrelevantes para explicar como nos relacionamos com o mundo à nossa volta, como o nosso cérebro obtém e registra informação desse mundo ou como decidimos agir em nossas vidas. Elétrons não explicam neurônios ou decisões morais que tomamos no decorrer de nossas vidas.

Para aprender sobre o mundo, é necessário se aproximar dele. A espiritualidade que vejo na ciência está nessa aproximação, no constante processo de desvendar algo de novo sobre a natureza, nesse levantar dos véus. É nessa ressonância que reside o mistério, na agonia da dúvida e no êxtase da descoberta.


" A mais profunda emoção que podemos experimentar é inspirada pelo senso de mistério. Essa é a emoção fundamental que inspira a verdadeira arte e a verdadeira ciência. Quem despreza esse fato e não é mais capaz de se questionar ou de se maravilhar, está mais morto do que vivo, sua visão, comprometida. Foi o senso do mistério, mesmo se misturado com o medo, que gerou a religião.
A existência de algo que não podemos penetrar, a percepção da mais profunda razão e da beleza mais radiante no mundo à nossa volta, que apenas em suas formas mais primitivas são acessíveis às nossas mentes, é esse conhecimento e emoção que constituem a verdadeira religiosidade; nesse sentido e nesse sentido apenas, eu sou profundamente religioso. "

( Albert Einstein )


A ciência tb possui seu sentimento de espiritualidade , só que este reside no mistério a ser revelado e não naquele que deveria permanecer eternamente escondido.

Ao descobrirmos uma nova face da natureza nosso sentimento é similar a aquele que vcs experimentam em sua meditações.


Engana-se quem pensa que a ciência e os cientistas são duros e frios.



[ ]'s

D.C.


CYBERLANDER

Ama a realidade que constróis,
que nem a morte deterá teu voo · ·




[As partes desta mensagem que não continham texto foram removidas]



SUBJECT: Re: Mais rápido que a luz? rosevena3
FROM: "rosevena3" <rosevena3@yahoo.com.br>
TO: ciencialist@yahoogrupos.com.br
DATE: 17/12/2004 16:21


--- Em ciencialist@yahoogrupos.com.br, "dfahlb" <dfahlb@y...> escreveu
>

---------------
Ficou claro como a luz.
obrigado






SUBJECT: Re: [ciencialist] Tudo é energia
FROM: "Amauri Jr" <amaurijunior2@yahoo.com.br>
TO: <ciencialist@yahoogrupos.com.br>
DATE: 17/12/2004 18:57

Tudo é energia quando se movimenta ou quando as células se reproduzem, mas não é só isso, quando os peixes nadam geram energia e quando se tem a fecundação dos animais se gera energia. Tudo que há no universo é de alguma forma gerador de energia, o sol com sua fusão nuclear é um gerador de energia, um planeta fazendo sua rotação....enfim tudo que há movimento gera energia.

Abraços
Amauri
----- Original Message -----
From: brudna
To: ciencialist@yahoogrupos.com.br
Sent: Thursday, December 16, 2004 6:35 PM
Subject: [ciencialist] Tudo é energia



Fico confuso com o argumento ´Tudo é energia´.

Onde estão os erros? Que conceitos falhos pode ter esse argumento?
Como aproveitar essa oportunidade para ensinar?


Até
Luís Brudna






##### ##### #####

Para saber mais visite
http://www.ciencialist.hpg.ig.com.br


##### ##### ##### #####


Yahoo! Grupos, um serviço oferecido por:







------------------------------------------------------------------------------
Links do Yahoo! Grupos

a.. Para visitar o site do seu grupo na web, acesse:
http://br.groups.yahoo.com/group/ciencialist/

b.. Para sair deste grupo, envie um e-mail para:
ciencialist-unsubscribe@yahoogrupos.com.br

c.. O uso que você faz do Yahoo! Grupos está sujeito aos Termos do Serviço do Yahoo!.



[As partes desta mensagem que não continham texto foram removidas]



SUBJECT: Re: [ciencialist] Re: Tudo é energia
FROM: "Amauri Jr" <amaurijunior2@yahoo.com.br>
TO: <ciencialist@yahoogrupos.com.br>
DATE: 17/12/2004 19:17

"Sob essa perspectiva nas verdade nada gera energia. No maximo
transforma energia de uma para outra forma."

[A] Qual é a lei da dinâmica mesmo? Eu sempre aprendi que tudo gera energia, uma bola de futebol tem que ter energia para se movimentar, só que esse energia começa gerada no pé do jogador, no ponto inicial. Não importa o que gera energia e sim tudo se dara na geração dessa energia.....

Amauri
----- Original Message -----
From: rmtakata
To: ciencialist@yahoogrupos.com.br
Sent: Thursday, December 16, 2004 7:00 PM
Subject: [ciencialist] Re: Tudo é energia



--- Em ciencialist@yahoogrupos.com.br, "Amauri Jr"
> Tudo é energia quando se movimenta ou quando as células se
> reproduzem, mas não é só isso, quando os peixes nadam geram energia e
> quando se tem a fecundação dos animais se gera energia. Tudo que há no
> universo é de alguma forma gerador de energia, o sol com sua fusão
> nuclear é um gerador de energia, um planeta fazendo sua
> rotação....enfim tudo que há movimento gera energia.



[]s,

Roberto Takata





##### ##### #####

Para saber mais visite
http://www.ciencialist.hpg.ig.com.br


##### ##### ##### #####


Yahoo! Grupos, um serviço oferecido por:







------------------------------------------------------------------------------
Links do Yahoo! Grupos

a.. Para visitar o site do seu grupo na web, acesse:
http://br.groups.yahoo.com/group/ciencialist/

b.. Para sair deste grupo, envie um e-mail para:
ciencialist-unsubscribe@yahoogrupos.com.br

c.. O uso que você faz do Yahoo! Grupos está sujeito aos Termos do Serviço do Yahoo!.



[As partes desta mensagem que não continham texto foram removidas]



SUBJECT: Um questão de preferência
FROM: José Renato <jrma@terra.com.br>
TO: <ciencialist@yahoogrupos.com.br>
DATE: 17/12/2004 19:18

Qual a opção que vc escolheria para o reajuste do salário mínimo: 290 já em janeiro ou 300 em maio de 2005?

Os números dessa comparação, podem ser verificados nas contas a seguir.

1ª Opção: (290 - 260) x (12 + 1) = 30 x 13 = 390 reais a mais em 2005.

2ª Opção: (300 - 260) x (8 + 1) = 40 x 9 = 360 reais a mais em 2005.

Estamos considerarmos que o próximo SM será corrigido em janeiro de 2006.

Entretanto, se o próximo salário mínimo for reajustado somente em maio de 2006, as contas serão diferentes.

1ª Opção: 30 x (13 + 4) = 30 x 17 = 510

2ª Opção: 40 x (9 + 4) = 40 x 13 = 520.

Verificamos que a melhor opção só poderia ser determinada se soubéssemos em que mês o SM será atualizado em 2006.

Mas a escolha tem de ser definida bem antes disso.
E agora, que opção você prefere?

José Renato M. de Almeida
Salvador - Bahia


[As partes desta mensagem que não continham texto foram removidas]



SUBJECT: Re: [ciencialist] Electrão como um vector
FROM: "Alberto Mesquita Filho" <albmesq@uol.com.br>
TO: <ciencialist@yahoogrupos.com.br>
DATE: 17/12/2004 19:32

----- Original Message -----
From: "Sergio M. M. Taborda"
Sent: Thursday, December 16, 2004 8:59 PM
Subject: [ciencialist] Electrão como um vector

> Sim, Alberto esta é para si.

Olá, Sérgio! Ói nóis aqui traveiz ;-)

Atualmente você anda muito sério. Gostava mais do irreverente (no bom
sentido) "garotão" Sérgio (o SMMT) que conheci no news "pt.ciencia.geral" do
ano 2000. :-)) Ou será que fui eu que mudei?!!! :-)

> Eu andei pensando nas hipoteses que vc usa na sua teoria e não chego a
> nenhuma conclusão do pq vc supor que o eletrão pode ser caracterizado
> por um vector.

Antes de prosseguir acredito que seria interessante fazer uma adenda que
poderá ser útil para o entendimento do que se segue. Como não é impossível
que existam outros leitores interessados no assunto, deixarei o sintetismo
de lado, logo perdoe-me se incorrer em aparentes redundâncias.

Não estou tentando caracterizar o elétron apenas por um vetor, mas
também. Ou seja, a hipótese 1 da minha teoria preconiza que o elétron pode
(1) ser caracterizado *matematicamente* por (2) sua posição e por (3) um
vetor. A hipótese 1 está portanto a retratar três realidades relacionadas ao
elétron, quais sejam:

1) O elétron não é apenas isso, ou seja, essa seria apenas a sua realidade
matemática. As quatro hipóteses servem para caracterizar a realidade
físico-matemática do elétron, e creio que aí temos um conjunto de hipóteses
suficientes para que possamos evoluir em direção a um objetivo e sem
acrescentar mais nada que não decorra da experimentação. Digo então que
existe uma realidade matemática (hipótese 1), uma realidade
físico-matemática (o conjunto das quatro hipóteses) e uma realidade de fato.
As quatro hipóteses caracterizam um conjunto fechado e a experimentação
representa um conjunto aberto de conhecimentos a serem progressivamente
acumulados.

2) Esse elétron ocupa uma posição no espaço e está sendo assumido como um
objeto clássico. A hipótese 1 admite a existência de uma estrutura interna e
a hipótese 3 reforça o argumento falando num possível raio matemático --ou
seja, esse elétron não está sendo pensado como puntiforme e/ou sem volume
físico, mas sim como algo a ocupar efetivamente uma região do espaço. Em
outras palavras, o elétron físico não deve ser pensado apenas como sendo o
ponto P e um vetor w aí localizado, essa seria tão somente uma simplificação
de natureza matemática, como inúmeras outras que caracterizam a física
clássica newtoniana. Matematicamente o elétron pode ser pensado como
puntiforme no mesmo sentido que aquele que adotamos para um planeta quando
estudamos a órbita que o mesmo percorre ao redor do Sol.

3) Além da posição, eu estou assumindo que esse elétron está dotado de
características vetoriais. Isso se traduz matematicamente pela sua
representação vetorial, ou seja, como um vetor w. Esse vetor não é um
campo vetorial, mas sim um vetor único e a acompanhar o elétron. Como esse
elétron é clássico, não vejo óbices teóricos a comprometerem o procedimento
adotado. Do contrário sim, eu teria ou de acrescentar alguma hipótese a
caracterizar uma possível e previsível anomalia não clássica e/ou então
deixar a liberdade para que essa anomalia viesse a ser constatada
experimentalmente, sem a necessidade de acrescentar hipóteses "ad hoc".
Chego a comentar alguma coisa a respeito dessa liberdade no primeiro
parágrafo que se segue à apresentação das hipóteses.

Para os que não conhecem as hipóteses da minha teoria, diria que elas estão
sintetizadas em http://ecientificocultural.com/Eletron/eletron22.htm e os
comentários sobre as mesmas no primeiro link a seguir. Vamos então para os
questionamentos.

> Tenho algumas perguntas em relação a isso. A primiera é exactamente Pq um
> vector ?

Vou tentar responder apelando para uma analogia. Sei que você não gosta
muito de analogias, mas desde que não nos apeguemos em demasia nas
similaridades, eu diria que elas têm um elevado valor didático.

Imagine que você está frente a um viscosímetro de Couette. Para o que nos
interessa, esse aparato pode ser pensado como um cilindro maciço girante e
mergulhado no centro de um dado volume de água que está contida em um
recipiente que pode ser pensado como sendo outro cilindro, agora oco e em
repouso (pode ser pensado como uma panela dessas de cozinha). À medida em
que o cilindro central gira, ele transmite seu movimento para as moléculas
de água. Se alguém se interessar por mais detalhes relacionados a essa
analogia, e supondo que esteja habituado ao Cálculo Avançado, sugiro que
consulte o artigo "Similaridades entre Campos de Velocidade e o Campo
Eletromagnético" que está em
http://ecientificocultural.com/Eletron2/flettner1.htm ; para os demais creio
que o que direi a seguir seja suficiente.

Suponha que o seu mundo físico-experimental, ou seja, aquele passível de
experimentação, restrinja-se à água. Ou seja, você não tem acesso ao
cilindro central e nem mesmo tem condições de garantir que ele está girando.
Você está assistindo e/ou mergulhado num campo de velocidades e/ou de
vorticidades, mas não tem como agir diretamente sobre o agente causal
(fonte) desse campo. Será que você poderá garantir que a fonte do campo é um
objeto que gira? Quero crer que sim, mas teríamos que recorrer a outras
teorias clássicas já bem conhecidas. Se o raio do cilindro externo (ou da
panela) tender a infinito, é possível que você consiga, através da
experimentação, relacionar a velocidade vetorial V num ponto qualquer do
fluido à distância vetorial R desse ponto ao centro do cilindro, através de
uma equação vetorial do tipo V = W x R, em que W é um vetor que pode ser
pensado como relacionado ao giro do cilindro, mas nada me garante que o seja
a não ser uma suposição bem firmada em teorias outras. Matematicamente
estamos frente a um vetor W que se traduz numa equação a satisfazer um
efeito físico, mas essa experimentação, por si só, não nos garante dizer que
W representa a velocidade angular do cilindro. Podemos então dizer, sem medo
de errar, que uma das características desse cilindro é que ele pode ser
representado matematicamente por um vetor W estimável experimentalmente.

Mas vamos complicar um pouco mais o que chamei de "mundo
físico-experimental". Suponha que você não sente ou não tenha como medir os
efeitos da água a não ser através de certos corpúsculos, digamos, algumas
bolinhas girantes. Você está, ainda que não saiba, girando num mundo não
inercial, sem equipamentos para constatar essa não inercialidade, e enxerga
a distância um cilindro aparentemente em repouso e que atrai algumas dessas
bolinhas e repele outras, conforme o giro se mostre no sentido horário ou
anti-horário. Esse seria o chamado efeito Magnus e, em dúvida, sugiro que
consulte a introdução do artigo acima indicado (Aviso aos demais
net-navegantes: Para a compreensão dessa introdução não é necessário o
conhecimento de Cálculo). Se as bolinhas forem todas idênticas e de giros
idênticos, você poderá até mesmo relacionar essa força a uma hipotética e
desconhecida velocidade V do meio e/ou a um hipotético vetor W do cilindro,
descrito acima, se bem que neste caso você não consegue "materializar" os
vetore V e W, a não ser através de suposições e/ou da recorrência a
artifícios teorizadores. Ou seja, através de um efeito físico (atração e
repulsão), podemos suspeitar de uma característica do cilindro que pode ser
associada a uma propriedade física que pode ser descrita matematicamente
através de um vetor.

Mais uma complicação: Vamos supor que você não consiga enxergar e nem
determinar o giro das bolinhas. A única coisa que você consegue constatar
fisicamente é que algumas dessas bolinhas são atraídas pelo cilindro, outras
são repelidas e outras não sofrem nenhum efeito (aquelas que não giram,
ainda que você não saiba disso). Você pode avaliar essa força de atração ou
de repulsão através, por exemplo, de uma mola. Percebe então uma dependência
entre o módulo da força e a distância do cilindro central. De duas uma: 1)
Se você for um físico mais adepto da experimentação, você procurará refinar
seus equipamentos, procurando visualizar cada vez mais os efeitos que não
estão se mostrando com os métodos atuais; 2) Se você for um físico mais
adepto da teorização, promoverá algumas viagens ao mundo do desconhecido,
até encontrar uma possível solução. Nada impede que você persiga os dois
métodos, mas existem casos em que a teorização economiza etapas e até mesmo
dinheiro, conquanto não seja isenta de riscos de interpretação (a teorização
faz previsões de maneira a que se possa "observar" um dado efeito de maneira
indireta, e aí sim, entraria a experimentação).

Resumindo: Ao observarmos um campo a se manifestar por forças, a exemplo do
campo eletromagnético, não é impossível que consigamos caracterizar o agente
dessa força como sendo um objeto (ou uma partícula) dotado de uma
propriedade física (que pode ser ou não um giro) que possa ser descrita
matematicamente por um vetor. A hipótese 1 aceita essa idéia e a analogia
acima foi apresentada apenas para demonstrar que não há nenhum absurdo nessa
conduta, ainda que eu não a tenha respondido de maneira totalmente
satisfatória. Creio que para tal precisaria escrever um livro a narrar como
o meu pensamento evoluiu, desde 1983 até 1992. Já pensei nisso, mas é coisa
que farei se, e somente se, a minha teoria chegar a vingar enquanto eu
estiver vivo e com disposição para tal.

Responderei o restante da sua msg oportunamente. Para meu controle, mantive
abaixo esse restante. Tenho a impressão que algumas dessas dúvidas devem ter
se dissipado em virtude do que expus acima. Em particular, lembro que o
"meu" eletron é *um* vetor, e não um vetor campo, ou um campo vetorial. Logo
creio não fazer sentido pensarmos em co-vetor. Quando muito poderemos
discutir se ele é ou não um vetor axial, mas deixo isso para a próxima
mensagem.

[ ]´s
Alberto
http://ecientificocultural.com/indice.htm
Mas indiferentemente a tudo isso, o neutrino tem massa, o elétron não é
uma carga elétrica coulombiana e a Terra se move. E a história se repetirá.

Abaixo, o restante da msg ainda não respondido:

******** início da citação ************

Aceitando que é um vector, será um vector ou um co-vector ?
Depois vc caracteriza o vector como W(w_x,w_y,w_z) = K w1 , onde w1 é
um vector unitário. Isto significa que podemos reduzir o estudo ao K
que não é um vector. Ora, isto não contraria a propria hipotese ?

Depois, eu suponho que W dependa de x,y,z ou seja que as derivadas
parciais am ordem a estes não sejam nulas. O que por conseguinte
significa que as derivadas de K tb dependem.

Depois, se W é um vector no espaço, ele muda a sua orientação ? Ou
seja, como são as derivadas em ordem ao tempo ?

Outra coisa que não consegui encontrar no seu site é a relação entre o
vector do electrão e a carga do electrão.

Se o electrão A cria um campo de efeitos EA e este é captado por um
electrão B que tem o seu campo EB , qual é o efeito de A em B , ou
seja, o que acontece a B ? Sabemos que cargas opostas se repetem, pelo
que EA aplicado em B deverá cria um força em B que faz o electrão se
afastar F=dp/dt. Como encontrar esta força a partir das definições dos
efeitos electricos e magneticos?

Basicamente, como se relaciona a carga , com os campos de efeitos e
estes dois com as forças repulsão e atracção ?

Outra interrogação é se a teoria so se aplica a electrões e protoes ou
a qq particula com carga , e nesse caso como entram as particulas sem
carga ?

******** final da citação ************



SUBJECT: Re: Ação e Reação - sentido
FROM: marcelo ferrari <emailferrari@yahoo.com.br>
TO: emailferrari@yahoo.com.br
DATE: 18/12/2004 07:16

Recebi várias respostas, de vários lugares diferentes, abstraindo, todas parecem apontar para o mesmo lugar: sentido.

A bolinha está indo assim:

> o > o > o > o > o > o >

ao interagir, ou re-agir com a parede fica assim

< o < o < o < o < o < o <

Se...

Ação = > o > o > o > o > o > o >

e reAção = < o < o < o < o < o < o <

O que muda é apenas o sentido, concordam?

Sendo assim, concluo que uma "determinada" ação temina no limite do seu sentido e a reAção começa no ponto de mudança de sentido.

Faz sentido?

marcelo ferrari



__________________________________________________
Converse com seus amigos em tempo real com o Yahoo! Messenger
http://br.download.yahoo.com/messenger/

[As partes desta mensagem que não continham texto foram removidas]



SUBJECT: Ação e Reação - sistema fechado
FROM: marcelo ferrari <emailferrari@yahoo.com.br>
TO: emailferrari@yahoo.com.br
DATE: 18/12/2004 07:23

Imagine um quarto fechado com espelhos por todos os lados, em diferentes angulaçoes, aleatórias. Um ambiente fechado onde cada parede reflita a luz para outra parede e assim por diante. Fazemos um buraco em algum canto qualquer e colocamos ali a ponta de uma canhão de raio lazer e ligamos o canhão.

Perguntas:

1) O facho de luz que sai da ponta do canhão vai retornar ao canhão?
2) Porque?
3) Tem alguma lei cientifica que descreve isto?


grato.
marcelo ferrari



__________________________________________________
Converse com seus amigos em tempo real com o Yahoo! Messenger
http://br.download.yahoo.com/messenger/

[As partes desta mensagem que não continham texto foram removidas]



SUBJECT: Re: [ciencialist] Electrão como um vector
FROM: "Alberto Mesquita Filho" <albmesq@uol.com.br>
TO: <ciencialist@yahoogrupos.com.br>
DATE: 18/12/2004 07:27

----- Original Message -----
From: "Sergio M. M. Taborda"
Sent: Thursday, December 16, 2004 8:59 PM
Subject: [ciencialist] Electrão como um vector
----- 2a PARTE -----

Continuação:

> Aceitando que é um vector, será um vector ou um co-vector?

Estou falando em um vetor único e a ser imaginado como algo que acompanha
uma partícula clássica, e não um campo vetorial. Conseqüentemente não pode
ser um co-vetor no sentido mais geral que se dá ao termo (o covetor não é um
vetor, mas sim um tipo de função vetorial). Poderíamos, quando muito, pensar
se esse vetor elétron teria propriedades matemáticas próprias aos vetores
axiais, tais como o vetor deslocamento angular. Não obstante, isso não faz
parte das hipóteses da teoria, o que não significa dizer que a teoria
despreze esse caráter, mas sim que deixa essa característica como algo a ser
explorado pela experimentação e/ou pela interpretação das experiências já
realizadas e ainda não devidamente explicadas. Explico melhor: Ao avançarmos
no estudo da teoria, no sentido de verificarmos se as hipóteses se coadunam
com a experimentação, certamente ficaremos frente as mesmas evidências
experimentais que levaram os físicos "modernos" a suspeitarem sobre a ainda
mal explicada, a meu ver, natureza quiral do elétron como, por exemplo, a
experiência de Stern Gerlach [Natureza quiral, para os que não sabem,
significa dizer que a imagem no espelho não se sobrepõe ao objeto, da mesma
maneira que a mão direita não se sobrepõe à mão esquerda]. Resumindo em
miúdos, porque eu deveria me antecipar à experimentação assumindo, por
hipótese, que o elétron é um férmion? Quero crer que eu estaria, de maneira
desnecessária, me antecipando à experimentação.

Por outro lado, se ele for de fato um férmion isso não deverá entrar em
conflito com o núcleo da teoria (hipóteses básicas), mesmo porque eu não
precisei dessa hipótese para justificar, numa primeira instância, a gênese
do campo eletromagnético. Ir além das hipóteses necessárias é um erro comum,
via de regra flagrado nos teorizadores novatos e/ou como um artifício a
superproteger uma teoria de falseamentos. [Perdoem-me os leitores leigos em
teoria quântica mas o restante deste parágrafo irá parecer um hieróglifo
para vocês e quiçá dotado de incorreções, pois apesar de não ser um profundo
conhecedor de teoria quântica, bem ou mal consigo "me virar no pedaço" ;-).]
Seria como postular, de maneira "ad hoc", o recurso da comutação ou da
anticomutação e, ao mesmo tempo, assumir a possibilidade de ignorá-los
quando assim nos convier, apelando agora para o primariamente assumido, e
por outros motivos, princípio da complementaridade de Bohr. Afinal, não é
isso o que se faz quando identificamos um elétron a um elemento de volume do
fluido elétrico? Onde estaria a quiralidade desse hipotético elemento de
volume, supondo-se que não tenha sido anulada pelo "decreto da
complementaridade de Bohr"? Bem, deixemos a quântica com os quânticos.

> Depois vc caracteriza o vector como W(w_x,w_y,w_z) = K w1 , onde w1 é
> um vector unitário. Isto significa que podemos reduzir o estudo ao K
> que não é um vector. Ora, isto não contraria a propria hipotese?

Você não leu devidamente a hipótese, ou se leu não entendeu. Em nenhum
momento eu escrevi W(w_x,w_y,w_z) = K w1, mas sim W = (w_x,w_y,w_z) = K w1.
Da maneira como você escreveu tem-se a impressão que W é uma função de
alguma coisa relacionada à posição, logo explica-se a sua dúvida anterior
(covetor). Não, (w_x, w_y e w_z) são as componentes do vetor W, assim como
(x, y, z) são as componentes do vetor de posição r, ou seja, r = (x, y, z).

[Lembro aos demais que tanto eu quanto o Sérgio estamos escrevendo num
editor de texto que não comporta determinados caracteres, logo estou
procurando interpretar o que ele escreveu utilizando-me dos mesmos símbolos
por ele utilizado. Quem quiser observar a expressão correta sugiro que se
dirija à webpage http://ecientificocultural.com/Eletron/eletron22.htm e
examine a hipótese 1.]

> Depois, eu suponho que W dependa de x,y,z ou seja que as derivadas
> parciais am ordem a estes não sejam nulas. O que por conseguinte
> significa que as derivadas de K tb dependem.

Não, W não é um vetor campo, mas um vetor fixo ao elétron. Seu valor é
independente da posição (x, y, z), ainda que tenha um fator dependente de
como a estrutura interna do elétron se orienta no espaço. Não se trata de
uma função contínua no espaço, logo não podemos derivá-la direcionalmente.
Temporalmente sim, mas esse é um problema que poderá ou não surgir quando
nos aprofundarmos no estudo da teoria. Por enquanto não há porque nos
preocuparmos com derivações temporais do vetor W.

> Depois, se W é um vector no espaço, ele muda a sua orientação?

Sim, sempre que a estrutura interna do elétron se reorientar o espaço. Por
exemplo, o elétron aprisionado a um átomo está com seu vetor W sempre
girando, independentemente do fato do W em si já poder representar um giro
de outro nível (nível estrutural).

> Ou seja, como são as derivadas em ordem ao tempo?

Depende do movimento que está sendo considerado. Mas como disse acima, você
está se antecipando e, como que, pretendendo chegar ao epílogo de um romance
sem se dar ao trabalho de ler os seus capítulos. As minhas hipóteses sequer
falam em tempo! Quando muito assumem que o elétron *emite* alguma coisa para
o espaço (hipótese 2), logo deve existir um fluxo dessa alguma coisa, e o
tempo está implícito nessa suposição.

> Outra coisa que não consegui encontrar no seu site é a relação entre o
> vector do electrão e a carga do electrão.

Nem poderia encontrar!!! Você nunca leu o desfecho de minhas mensagens? Se
não leu, exponho aqui, mas também estará abaixo: "um elétron não é uma carga
elétrica coulombiana". O máximo que você poderá procurar no meu trabalho é
saber qual o artifício que eu utilizo para chegar numa carga elétrica
macroscópica partindo de estruturas que não se assemelham em nada a uma
carga elétrica. Seria como que montar um quebra cabeças retangular
utilizando-se de peças de formatos outros. Obviamente o elétron exerce um
efeito sobre outros elétrons, mas isso não significa dizer que ele seja uma
carga elétrica. A menos que você assuma que ele é uma carga de outro tipo,
mas ainda não senti necessidade de criar uma "carga mesquitiana". Chamo-o
apenas por elétron, e você dê a ele o nome que quiser, desde que não o
confunda com uma carga elétrica com características coulombianas (aquelas
que geram um campo elétrico de Coulomb).

> Se o electrão A cria um campo de efeitos EA e este é captado por um
> electrão B que tem o seu campo EB , qual é o efeito de A em B , ou
> seja, o que acontece a B?

Isso está exposto no capítulo 6, e poderá vir a originar um bom debate, mas
creio que ainda é muito cedo para isso. Afinal, noto pelas perguntas que
você ainda não conseguiu digerir nem a hipótese 1! ;-))

Lembro que cada elétron cria dois campos de efeitos (ou seja, no sentido
físico da palavra efeito), e não apenas um (a rigor seriam três, mas um
desses pode ser deixado de lado por ora). Esses dois campos de efeitos
(elétrico ou magnético) estariam contidos num único campo matemático
(vetorial) "A" (equações 6.5). Lembro que a notação que você escolheu (EA e
EB) não é muito boa pelo fato de poder vir a gerar inúmeras confusões.
Parece-me que você utilizou "EA" (ou "EB") para se referir ao campo de um
suposto Elétron a ocupar um ponto A (ou B). Acontece que "A" é a expressão
geral do campo fundamental da minha teoria (e também é a expressão utilizada
para o vetor potencial magnético, que sob muitos aspectos confunde-se com o
meu vetor "A"); "E" é o campo elétrico e "B" é o campo de indução magnética,
esse últimos da teoria de Maxwell. Sei que não será fácil expressarmos aqui
as equações vetoriais, mas é sempre bom chamar a atenção para essa
dificuldade que não é apenas de natureza editorial (edição de texto).

> Sabemos que cargas opostas se repetem, pelo
> que EA aplicado em B deverá cria um força em B que faz o electrão se
> afastar F=dp/dt. Como encontrar esta força a partir das definições dos
> efeitos electricos e magneticos?
> Basicamente, como se relaciona a carga , com os campos de efeitos e
> estes dois com as forças repulsão e atracção ?

Um elétron não é uma carga elétrica, logo não espere sempre por essa
repulsão. A repulsão surgirá ou não dependendo das orientações espaciais do
elétron. Esse assunto começa a ser discutido no capítulo 5, no que diz
respeito ao efeito elétrico. É importante lembrar que existe também um
efeito magnético, mesmo que o elétron esteja em repouso (sem entrar nas
considerações de um possível giro associado ao vetor W, ou seja, seria um
repouso no sentido translacional). Vê-se aí mais uma diferença entre o
elétron e a carga elétrica, pois uma carga elétrica em repouso translacional
tem um campo de efeitos magnéticos nulo, para um observador também em
repouso. Ou seja, ao integrarmos o campo de efeitos magnéticos de minha
teoria afim de chegarmos no campo de efeitos magnéticos resultante de uma
carga elétrica, chegamos num valor nulo, como deveria ser para que a teoria
se compatibilizasse com a experimentação. Mas a coisa é bem mais complicada
e não convém entrarmos em detalhes no momento. Tudo a seu tempo.

> Outra interrogação é se a teoria so se aplica a electrões e protoes ou
> a qq particula com carga , e nesse caso como entram as particulas sem
> carga?

Em vista do acima exposto, devolvo a pergunta com outras: O que é uma
partícula com carga? O que é uma partícula sem carga? O que é uma partícula
e o que é uma partícula realmente elementar?

A sua pergunta é realmente interessante, mas quando vista sob um prisma mais
abrangente e a relacionar-se à hipótese 2: O que são informações
eletromagnéticas na sua intimidade? Obviamente a minha teoria não responde a
essa pergunta, mas nada impede que possamos evoluir para teorias mais gerais
e mais representacionais, e a incluir também as informações gravitacionais.
Será que não poderíamos evoluir a idéia do campo "A" a ponto de considerar
aí um terceiro (na realidade um quarto) efeito a contemplar a gravitação?
Não espero responder a isso nas próximas mensagens e nem mesmo garanto que
possa dar uma resposta satisfatória, mas chamo a atenção para a
possibilidade de unificarmos o eletromagnetismo e a gravitação por essa via.
Enfim, não posso fazer sozinho, e numa única existência, aquilo que milhares
de físicos não conseguiram fazer por outras vias nos quase quatrocentos anos
que sucederam a Galileu. Por outro lado, não pretendam entender a minha
teoria em todos os seus detalhes num tempo muito menor do que aquele que
gastariam para entender em profundidade o eletromagnetismo de Maxwell.

[ ]´s
Alberto
http://ecientificocultural.com/indice.htm
Mas indiferentemente a tudo isso, o neutrino tem massa, o elétron não é
uma carga elétrica coulombiana e a Terra se move. E a história se repetirá.



SUBJECT: Re: [ciencialist] Electrão como um vector - ERRATA
FROM: "Alberto Mesquita Filho" <albmesq@uol.com.br>
TO: <ciencialist@yahoogrupos.com.br>
DATE: 18/12/2004 11:12

----- Original Message -----
From: "Alberto Mesquita Filho"
Sent: Friday, December 17, 2004 6:32 PM
Subject: Re: [ciencialist] Electrão como um vector
----- 1a PARTE -----

ERRATA:

Na msg http://br.groups.yahoo.com/group/ciencialist/message/43271 ,

ONDE SE LÊ:

V = W x R

LEIA-SE:

V = f(W, R)

ou então:

V = V(W, R).

Na pressa eu escrevi a equação que se verifica numa plataforma girante
quando deveria ter escrito uma equação a relacionar-se com o cilindro
girante. Acima eu estou substituindo por uma equação mais geral, apenas a
caracterizar a dependência da função, e que seria válida para ambos. Em
dúvida consultar os anexos do artigo citado, pois aí a equação de Navier
Stokes é resolvida para as duas situações:
http://ecientificocultural.com/Eletron2/flettner1.htm

[ ]´s
Alberto
http://ecientificocultural.com/indice.htm
Mas indiferentemente a tudo isso, o neutrino tem massa, o elétron não é
uma carga elétrica coulombiana e a Terra se move. E a história se repetirá.



SUBJECT: Re: [ciencialist] Electrão como um vector
FROM: "Alberto Mesquita Filho" <albmesq@uol.com.br>
TO: <ciencialist@yahoogrupos.com.br>
DATE: 18/12/2004 11:14

----- Original Message -----
From: "Sergio M. M. Taborda"
Sent: Thursday, December 16, 2004 8:59 PM
Subject: [ciencialist] Electrão como um vector

----- 2a PARTE -----

Continuação:

> Aceitando que é um vector, será um vector ou um co-vector?

Estou falando em um vetor único e a ser imaginado como algo que acompanha
uma partícula clássica, e não um campo vetorial. Conseqüentemente não pode
ser um co-vetor no sentido mais geral que se dá ao termo (o covetor não é um
vetor, mas sim um tipo de função vetorial). Poderíamos, quando muito, pensar
se esse vetor elétron teria propriedades matemáticas próprias aos vetores
axiais, tais como o vetor deslocamento angular. Não obstante, isso não faz
parte das hipóteses da teoria, o que não significa dizer que a teoria
despreze esse caráter, mas sim que deixa essa característica como algo a ser
explorado pela experimentação e/ou pela interpretação das experiências já
realizadas e ainda não devidamente explicadas. Explico melhor: Ao avançarmos
no estudo da teoria, no sentido de verificarmos se as hipóteses se coadunam
com a experimentação, certamente ficaremos frente as mesmas evidências
experimentais que levaram os físicos "modernos" a suspeitarem sobre a ainda
mal explicada, a meu ver, natureza quiral do elétron como, por exemplo, a
experiência de Stern Gerlach [Natureza quiral, para os que não sabem,
significa dizer que a imagem no espelho não se sobrepõe ao objeto, da mesma
maneira que a mão direita não se sobrepõe à mão esquerda]. Resumindo em
miúdos, porque eu deveria me antecipar à experimentação assumindo, por
hipótese, que o elétron é um férmion? Quero crer que eu estaria, de maneira
desnecessária, me antecipando à experimentação.

Por outro lado, se ele for de fato um férmion isso não deverá entrar em
conflito com o núcleo da teoria (hipóteses básicas), mesmo porque eu não
precisei dessa hipótese para justificar, numa primeira instância, a gênese
do campo eletromagnético. Ir além das hipóteses necessárias é um erro comum,
via de regra flagrado nos teorizadores novatos e/ou como um artifício a
superproteger uma teoria de falseamentos. [Perdoem-me os leitores leigos em
teoria quântica mas o restante deste parágrafo irá parecer um hieróglifo
para vocês e quiçá dotado de incorreções, pois apesar de não ser um profundo
conhecedor de teoria quântica, bem ou mal consigo "me virar no pedaço" ;-).]
Seria como postular, de maneira "ad hoc", o recurso da comutação ou da
anticomutação e, ao mesmo tempo, assumir a possibilidade de ignorá-los
quando assim nos convier, apelando agora para o primariamente assumido, e
por outros motivos, princípio da complementaridade de Bohr. Afinal, não é
isso o que se faz quando identificamos um elétron a um elemento de volume do
fluido elétrico? Onde estaria a quiralidade desse hipotético elemento de
volume, supondo-se que não tenha sido anulada pelo "decreto da
complementaridade de Bohr"? Bem, deixemos a quântica com os quânticos.

> Depois vc caracteriza o vector como W(w_x,w_y,w_z) = K w1 , onde w1 é
> um vector unitário. Isto significa que podemos reduzir o estudo ao K
> que não é um vector. Ora, isto não contraria a propria hipotese?

Você não leu devidamente a hipótese, ou se leu não entendeu. Em nenhum
momento eu escrevi W(w_x,w_y,w_z) = K w1, mas sim W = (w_x,w_y,w_z) = K w1.
Da maneira como você escreveu tem-se a impressão que W é uma função de
alguma coisa relacionada à posição, logo explica-se a sua dúvida anterior
(covetor). Não, (w_x, w_y e w_z) são as componentes do vetor W, assim como
(x, y, z) são as componentes do vetor de posição r, ou seja, r = (x, y, z).

[Lembro aos demais que tanto eu quanto o Sérgio estamos escrevendo num
editor de texto que não comporta determinados caracteres, logo estou
procurando interpretar o que ele escreveu utilizando-me dos mesmos símbolos
por ele utilizado. Quem quiser observar a expressão correta sugiro que se
dirija à webpage http://ecientificocultural.com/Eletron/eletron22.htm e
examine a hipótese 1.]

> Depois, eu suponho que W dependa de x,y,z ou seja que as derivadas
> parciais am ordem a estes não sejam nulas. O que por conseguinte
> significa que as derivadas de K tb dependem.

Não, W não é um vetor campo, mas um vetor fixo ao elétron. Seu valor é
independente da posição (x, y, z), ainda que tenha um fator dependente de
como a estrutura interna do elétron se orienta no espaço. Não se trata de
uma função contínua no espaço, logo não podemos derivá-la direcionalmente.
Temporalmente sim, mas esse é um problema que poderá ou não surgir quando
nos aprofundarmos no estudo da teoria. Por enquanto não há porque nos
preocuparmos com derivações temporais do vetor W.

> Depois, se W é um vector no espaço, ele muda a sua orientação?

Sim, sempre que a estrutura interna do elétron se reorientar o espaço. Por
exemplo, o elétron aprisionado a um átomo está com seu vetor W sempre
girando, independentemente do fato do W em si já poder representar um giro
de outro nível (nível estrutural).

> Ou seja, como são as derivadas em ordem ao tempo?

Depende do movimento que está sendo considerado. Mas como disse acima, você
está se antecipando e, como que, pretendendo chegar ao epílogo de um romance
sem se dar ao trabalho de ler os seus capítulos. As minhas hipóteses sequer
falam em tempo! Quando muito assumem que o elétron *emite* alguma coisa para
o espaço (hipótese 2), logo deve existir um fluxo dessa alguma coisa, e o
tempo está implícito nessa suposição.

> Outra coisa que não consegui encontrar no seu site é a relação entre o
> vector do electrão e a carga do electrão.

Nem poderia encontrar!!! Você nunca leu o desfecho de minhas mensagens? Se
não leu, exponho aqui, mas também estará abaixo: "um elétron não é uma carga
elétrica coulombiana". O máximo que você poderá procurar no meu trabalho é
saber qual o artifício que eu utilizo para chegar numa carga elétrica
macroscópica partindo de estruturas que não se assemelham em nada a uma
carga elétrica. Seria como que montar um quebra cabeças retangular
utilizando-se de peças de formatos outros. Obviamente o elétron exerce um
efeito sobre outros elétrons, mas isso não significa dizer que ele seja uma
carga elétrica. A menos que você assuma que ele é uma carga de outro tipo,
mas ainda não senti necessidade de criar uma "carga mesquitiana". Chamo-o
apenas por elétron, e você dê a ele o nome que quiser, desde que não o
confunda com uma carga elétrica com características coulombianas (aquelas
que geram um campo elétrico de Coulomb).

> Se o electrão A cria um campo de efeitos EA e este é captado por um
> electrão B que tem o seu campo EB , qual é o efeito de A em B , ou
> seja, o que acontece a B?

Isso está exposto no capítulo 6, e poderá vir a originar um bom debate, mas
creio que ainda é muito cedo para isso. Afinal, noto pelas perguntas que
você ainda não conseguiu digerir nem a hipótese 1! ;-))

Lembro que cada elétron cria dois campos de efeitos (ou seja, no sentido
físico da palavra efeito), e não apenas um (a rigor seriam três, mas um
desses pode ser deixado de lado por ora). Esses dois campos de efeitos
(elétrico ou magnético) estariam contidos num único campo matemático
(vetorial) "A" (equações 6.5). Lembro que a notação que você escolheu (EA e
EB) não é muito boa pelo fato de poder vir a gerar inúmeras confusões.
Parece-me que você utilizou "EA" (ou "EB") para se referir ao campo de um
suposto Elétron a ocupar um ponto A (ou B). Acontece que "A" é a expressão
geral do campo fundamental da minha teoria (e também é a expressão utilizada
para o vetor potencial magnético, que sob muitos aspectos confunde-se com o
meu vetor "A"); "E" é o campo elétrico e "B" é o campo de indução magnética,
esse últimos da teoria de Maxwell. Sei que não será fácil expressarmos aqui
as equações vetoriais, mas é sempre bom chamar a atenção para essa
dificuldade que não é apenas de natureza editorial (edição de texto).

> Sabemos que cargas opostas se repetem, pelo
> que EA aplicado em B deverá cria um força em B que faz o electrão se
> afastar F=dp/dt. Como encontrar esta força a partir das definições dos
> efeitos electricos e magneticos?
> Basicamente, como se relaciona a carga , com os campos de efeitos e
> estes dois com as forças repulsão e atracção ?

Um elétron não é uma carga elétrica, logo não espere sempre por essa
repulsão. A repulsão surgirá ou não dependendo das orientações espaciais do
elétron. Esse assunto começa a ser discutido no capítulo 5, no que diz
respeito ao efeito elétrico. É importante lembrar que existe também um
efeito magnético, mesmo que o elétron esteja em repouso (sem entrar nas
considerações de um possível giro associado ao vetor W, ou seja, seria um
repouso no sentido translacional). Vê-se aí mais uma diferença entre o
elétron e a carga elétrica, pois uma carga elétrica em repouso translacional
tem um campo de efeitos magnéticos nulo, para um observador também em
repouso. Ou seja, ao integrarmos o campo de efeitos magnéticos de minha
teoria afim de chegarmos no campo de efeitos magnéticos resultante de uma
carga elétrica, chegamos num valor nulo, como deveria ser para que a teoria
se compatibilizasse com a experimentação. Mas a coisa é bem mais complicada
e não convém entrarmos em detalhes no momento. Tudo a seu tempo.

> Outra interrogação é se a teoria so se aplica a electrões e protoes ou
> a qq particula com carga , e nesse caso como entram as particulas sem
> carga?

Em vista do acima exposto, devolvo a pergunta com outras: O que é uma
partícula com carga? O que é uma partícula sem carga? O que é uma partícula
e o que é uma partícula realmente elementar?

A sua pergunta é realmente interessante, mas quando vista sob um prisma mais
abrangente e a relacionar-se à hipótese 2: O que são informações
eletromagnéticas na sua intimidade? Obviamente a minha teoria não responde a
essa pergunta, mas nada impede que possamos evoluir para teorias mais gerais
e mais representacionais, e a incluir também as informações gravitacionais.
Será que não poderíamos evoluir a idéia do campo "A" a ponto de considerar
aí um terceiro (na realidade um quarto) efeito a contemplar a gravitação?
Não espero responder a isso nas próximas mensagens e nem mesmo garanto que
possa dar uma resposta satisfatória, mas chamo a atenção para a
possibilidade de unificarmos o eletromagnetismo e a gravitação por essa via.
Enfim, não posso fazer sozinho, e numa única existência, aquilo que milhares
de físicos não conseguiram fazer por outras vias nos quase quatrocentos anos
que sucederam a Galileu. Por outro lado, não pretendam entender a minha
teoria em todos os seus detalhes num tempo muito menor do que aquele que
gastariam para entender em profundidade o eletromagnetismo de Maxwell.

[ ]´s
Alberto
http://ecientificocultural.com/indice.htm
Mas indiferentemente a tudo isso, o neutrino tem massa, o elétron não é
uma carga elétrica coulombiana e a Terra se move. E a história se repetirá.



SUBJECT: Re: Ação e Reação - sentido
FROM: Maria Natália <grasdic@hotmail.com>
TO: ciencialist@yahoogrupos.com.br
DATE: 18/12/2004 13:31


Marcelo:
A tua resposta NÃO TEM SENTIDO! Nem lógico dedutiva é!
Não percebeste nada do que os físicos te estiveram aqui a dizer..
Torna a ler as mensagens enviadas pelos físicos.
Está lá escrito a diferença entre o Reacção e a Acção. E outra coisa
os vectores se representam usando ---- e > ou < e nada de bolinhas.
Essa até fez Einstein dar voltas no túmulo. E já leste as páginas de
mestre Leo no feiradeciencias sobre o tema? Era por aí que deverias
ter começado. Tás perdido? Vai aqui o endereço feiradeciencias.co.br
Onde pará a tua matemática? Ficou no tinteiro.
Vais responder pondo os vectores como deve ser pois isto não é
metafísica, percebes? Aliás tu disseste que querias saber física e
eu agora quero saber qual o teu conceito: se evolui ou não? A tua
resposta me diz que nada sabes de física e te recusas a saber. Então
porque perguntaste? Queres fazer um dos teus contos de autocarro?
Vê lá que andaste para aí a chatear meio mundo por todas as listas
onde fazes publicidade.
Estar na C list é para aprender Ciência e vejo que nada aprendeste,
pá. Tás a gozar com a malta?
Paraste no tempo? E o teu vestibular teve matemática e física?
Vá volta atrás e lê as mensagens. Onde falhámos nós? Ou seria que
leste em diagonal e te estás borrifando para as respostas?
Veremos a tua evolução científica no retorno desta mensagem.
Maria Natália


--- Em ciencialist@yahoogrupos.com.br, marcelo ferrari
<emailferrari@y...> escreveu
> Recebi várias respostas, de vários lugares diferentes, abstraindo,
todas parecem apontar para o mesmo lugar: sentido.
>
> A bolinha está indo assim:
>
> > o > o > o > o > o > o >
>
> ao interagir, ou re-agir com a parede fica assim
>
> < o < o < o < o < o < o <
>
> Se...
>
> Ação = > o > o > o > o > o > o >
>
> e reAção = < o < o < o < o < o < o <
>
> O que muda é apenas o sentido, concordam?
>
> Sendo assim, concluo que uma "determinada" ação temina no limite
do seu sentido e a reAção começa no ponto de mudança de sentido.
>
> Faz sentido?
>
> marcelo ferrari
>
>
>
> __________________________________________________
> Converse com seus amigos em tempo real com o Yahoo! Messenger
> http://br.download.yahoo.com/messenger/
>
> [As partes desta mensagem que não continham texto foram removidas]





SUBJECT: Re: Ação e Reação - sistema fechado
FROM: Maria Natália <grasdic@hotmail.com>
TO: ciencialist@yahoogrupos.com.br
DATE: 18/12/2004 13:49


Marcelu:

Se ainda estás com conceito de Acção Reacção incorrecto porque te
metes noutra? Espera resposta dos físicos da lista. Assim te
atropelas. Põr questões é fácil. E respondê-las vai ser-te fácil.
Vou fazer contigo como fazemos com as criancinhas que nos pedem a
papinha toda feita. (Takata temos aqui outro que não quer pensar...)
Vais à tua matemática e na seccção de probabilidades vais ler com
atenção toda a introdução. Recordas e depois tornas a fazer a
pergunta. E põe aqui a resposta. O pessoal vê e te ajuda a completar
e assim sucessivamente. A Ciência resulta de raciocínio e é isso que
queremos: por-te a PENSAR CIENTIFICAMENTE. Se queres filosofia te
enganaste na porta.
E deixa-te de metafísicas...Tu nem física sabes.
E outra coisa: cuidado que levas com o feixe laser (de rubi,não?) no
olho e ficas ceguinho. Olha que a lista é aberta e tem pessoa por aí
que pode tentar fazer esta tua "experiência"
Maria Natália

--- Em ciencialist@yahoogrupos.com.br, marcelo ferrari
<emailferrari@y...> escreveu
> Imagine um quarto fechado com espelhos por todos os lados, em
diferentes angulaçoes, aleatórias. Um ambiente fechado onde cada
parede reflita a luz para outra parede e assim por diante. Fazemos
um buraco em algum canto qualquer e colocamos ali a ponta de uma
canhão de raio lazer e ligamos o canhão.
>
> Perguntas:
>
> 1) O facho de luz que sai da ponta do canhão vai retornar ao
canhão?
> 2) Porque?
> 3) Tem alguma lei cientifica que descreve isto?
>
>
> grato.
> marcelo ferrari
>
>
>
> __________________________________________________
> Converse com seus amigos em tempo real com o Yahoo! Messenger
> http://br.download.yahoo.com/messenger/
>
> [As partes desta mensagem que não continham texto foram removidas]





SUBJECT: Re: Ação e Reação - sistema fechado
FROM: Maria Natália <grasdic@hotmail.com>
TO: ciencialist@yahoogrupos.com.br
DATE: 18/12/2004 14:05


Marcelu:

O que é para ti um sistema fechado?

Maria Natália

--- Em ciencialist@yahoogrupos.com.br, marcelo ferrari
<emailferrari@y...> escreveu
> Imagine um quarto fechado com espelhos por todos os lados, em
diferentes angulaçoes, aleatórias. Um ambiente fechado onde cada
parede reflita a luz para outra parede e assim por diante. Fazemos
um buraco em algum canto qualquer e colocamos ali a ponta de uma
canhão de raio lazer e ligamos o canhão.
>
> Perguntas:
>
> 1) O facho de luz que sai da ponta do canhão vai retornar ao
canhão?
> 2) Porque?
> 3) Tem alguma lei cientifica que descreve isto?
>
>
> grato.
> marcelo ferrari
>
>
>
> __________________________________________________
> Converse com seus amigos em tempo real com o Yahoo! Messenger
> http://br.download.yahoo.com/messenger/
>
> [As partes desta mensagem que não continham texto foram removidas]





SUBJECT: Re: Ação e ReAção
FROM: Maria Natália <grasdic@hotmail.com>
TO: ciencialist@yahoogrupos.com.br
DATE: 18/12/2004 15:13


Leo:
E não seria melhor enviar menino a sua página na feira de ciências?
É que de restrições ele nada percebe.
E depois os vectores ajudariam mesmo. Aliás veja a resposta que ele
lhe enviou. Própria de quem não entendeu mesmo.
Mas o mais grave é que nem nota e parte para outra.
Bom fds
Maria Natália

--- Em ciencialist@yahoogrupos.com.br, "Luiz Ferraz Netto"
<leobarretos@u...> escreveu
> De: "marcelo ferrari"
>
> "Se eu jogo uma bola contra a parede, ela vai e volta, ação e
reação.
> A pergunta é: onde termina a ação e começa a reação?"
>
> A descrição completa do fenômeno implica na teoria dos choques
mecânicos, parcialmente elásticos, com 0 < e < 1 (e=coeficiente de
restituição).
> Chamemos de ação (A) a força que a bola aplica contra a parede e
reação (R) a força que a parede aplica contra a bola. Em cada
instante, ambas (A e R) têm a mesma intensidade, mesma direção,
sentidos opostos e, obviamente, os pontos de aplicações são
distintos, um na parede (o da ação) e outro na bola (o da reação).
> No instante zero, onde se inicia o choque, o par é nulo, a pressão
no interior da bola é PB, a tensão elástica da câmara é TE, a
pressão atmosférica é Pat e a temperatura ambiente (igual à do ar da
bola, no início) é TA.
> Iniciado o choque a pequena A deforma imperceptivelmente a parede
e a pequena R deforma a bola, achatando-a, comprimindo o ar,
aumentando PB, aumentando TE, aumentando TA interno ... e diminuindo
a velocidade do CM da bola. Quando essa velocidade do CM se anula, a
A (logo R) é máxima e começa a fase de recuperação da forma da bola.
Vale a pena ver tudo isso em fotos estroboscópicas.
> E assim a coisa evolui até novamente A (e R) se anularem (fim do
choque), bola adquirir velocidade de afastamento menor que aquela de
aproximação, ar no interior da bola mais 'quente', etc. Parte da
perda de energia cinética é convertida em som.
> Galvanômetros balísticos podem medir o tempo de interação bola-
parede e assim nos dá a conhecer quanto tempo dura a existência do
par ação/reação.
> Não tem sentido a pergunta "onde termina a ação e começa a
reação", uma vez que são interações simultâneas; as duas começam e
terminam juntas, como se pode constatar pelo relato acima.
> Por outro lado, é preciso muito cuidado com a redação científica.
A frase inicial ["Se eu jogo uma bola contra a parede, ela vai e
volta, ação e reação.], por exemplo, pode dar falsa interpretação ao
leitor se ele associar "vai" com "ação" e "volta" com "reação".
> Se a bola 'grudasse' na parede, ou simplesmente caísse ao chão
(bola mucha jogada na parede) --- (eliminando a 'volta') ---, ainda
assim existiriam "ação e reação".
> Tais tipos de frases sempre levam à má interpretação, veja essa:
Um barquinho bateu num transatlântico e afundou --- mas, se ação e
reação sempre são iguais, por que o transatlântico tb não afundou?
>
> []'
> Léo





SUBJECT: Novo estado: protopolímero
FROM: Maria Natália <grasdic@hotmail.com>
TO: ciencialist@yahoogrupos.com.br
DATE: 18/12/2004 18:21


Paul S. Weiss e Gregory McCarty descobriram uma nova estrutura a
que chamaram protopolímero. Esta estrutura cristalina de moléculas
de fenileno foi obtirda sobre filme de cobre a baixa temperatura.Os
protopolímeros formam-se quando od monómeros se junta e ficam
alinhados e interagem som formar ligações químicas. Ora os efeitos
de superfície dos metal tem sido explicados pelas ligações
estabelecidas com o mesmo. Foi a primeira observação de alinhamento
livre de ligações. Esta descoberta pode servir para controlar o
crescimento e arrumação de molécuas permitindo manipulação de nano
estruturas em escala atómica ou molecular. Até podemos pensar em
melhoria de lentes para óculos...
O paper tem o título:"Formation and Manipulation of Protopolymer
Chains," e saiu publicado no Journal of the American Chemical
Society on 15 December 2004.
Entretanto podem saber um pouco mais e de modo ainda acessível em:
http://www.physorg.com/news2272.html
Um abraço
Maria Natália





SUBJECT: Re: Electrão como um vector
FROM: "Sergio M. M. Taborda" <sergiotaborda@yahoo.com.br>
TO: ciencialist@yahoogrupos.com.br
DATE: 18/12/2004 21:54


--- Em ciencialist@yahoogrupos.com.br, "Alberto Mesquita Filho"

> Continuação:
>
> > Aceitando que é um vector, será um vector ou um co-vector?
>
> Estou falando em um vetor único e a ser imaginado como algo que
acompanha
> uma partícula clássica, e não um campo vetorial.


A minha pergunta sobre pq um vector no sentido mais epistemologico. O
que significa atribuir 3 quantidades ao electrão , o que são elas, e
pq apenas 3 ? pq não 6 (matriz antisimetrica)? pq não 9 (matriz) pq
não uma matriz simetrica em vez de um vector ? Quando vc diz Vector
vcs está postulando a natureza matemática , um modelo matemático, para
a representação do electrão, mas não está informando a razão fisica
para essa escolha. por isso perguntei. Eu sei que sendo uma hipotese,
qq coisa é válida, mas esperava que sendo um assunto fisico, tivesse
algumas desculpa fisica para a escolha.

Agora, um vector, tem 3 quantidades e ele TÊM que sre representado num
sistema de ccordenadas , qq que seja. Essas coordenadas que eu chamei
de w_x, w_y e w_x mas que podemos abreviar para w_i que compem o
vector W têm valores diferentes conforme o sistema de coordenadas.


Conseqüentemente não pode
> ser um co-vetor no sentido mais geral que se dá ao termo (o covetor
não é um
> vetor, mas sim um tipo de função vetorial).

Bem, um co-vetor é um objecto que após uma transformação não matem a
mesma estrutura. Ou seja, os valoes w_i dependeriam do referencia
matemático usado para os representar. Coisa que não acontece com um
vector.

Ou seja, eu estou preocupado com a representação matemática do vector
num sistema de coordenadas. Mas tomemos o sistema carteziano
orto-normado , par ser mais simples.

Podemos encontrar as várias w_i fazendo w_i = u_i.W , onde u_i é o
vector da base.


> certamente ficaremos frente as mesmas evidências
> experimentais que levaram os físicos "modernos" a suspeitarem sobre
a ainda
> mal explicada, a meu ver, natureza quiral do elétron como, por
exemplo, a
> experiência de Stern Gerlach


O que vc está dizendo é que 2 electrões não tem que ser iguais. ora,
isso muito interessante pq vai contra a seua hipotese.
Se W é um vector intrinseco do electrão. Para todos os efeitos esse
vector é o electrão, então 2 eletrões têm que ter o mesmo vector.
Ora um vector, 3 quantidades, não permite quiralidade. Para isso vc
precisa no minimo de uma matix anti-simetrica. (em que os termos fora
do traço seriam diferentes obtendo a quiralidade )

> > Depois vc caracteriza o vector como W(w_x,w_y,w_z) = K w1 , onde w1 é
> > um vector unitário. Isto significa que podemos reduzir o estudo ao K
> > que não é um vector. Ora, isto não contraria a propria hipotese?
>
> Você não leu devidamente a hipótese, ou se leu não entendeu. Em nenhum
> momento eu escrevi W(w_x,w_y,w_z) = K w1, mas sim W = (w_x,w_y,w_z)
= K w1.

É a mesma coisa, na notação que usei.

O que vc está dizendo com isso - embora tlv não conscientemente - é
que o vector pode ser representado por uma função escalar e um outro
vector . Este vector obviamente seria um covector e a função escalar é
apenas para efeitos de normalização.

> Da maneira como você escreveu tem-se a impressão que W é uma função de
> alguma coisa relacionada à posição, logo explica-se a sua dúvida
anterior
> (covetor).

Têm-se, mas não é isso que deveria tem. w_x significa apenas a
primeira coordenada do vector. Ma para ser mais concreto w_x = u_x.W
onde . é o produto interno.

Não, (w_x, w_y e w_z) são as componentes do vetor W, assim como
> (x, y, z) são as componentes do vetor de posição r, ou seja, r = (x,
y, z).

Isso é apenas uma questão de notação. É comum em matemática usar a
notação V (a,b,c) para vector V de ccordenadas a ,b ,c. Uma função é
sempre com letra minuscula. seria algo como g(a,b,c), nunca G(a,b,c)
Mas isso é só notação. O que importa é que estou chamando nomes às
ccordenadas do vector no espaço carteziano usando w para coordendada
do vector W e o x,y,z para numerar as coordenadas, mas escrevendo
apenas "as coordenadas w_i" ou "o vector W" vai dar no mesmo.
Passado esse promenor de nomenclatura ...

> > Depois, eu suponho que W dependa de x,y,z ou seja que as derivadas
> > parciais am ordem a estes não sejam nulas. O que por conseguinte
> > significa que as derivadas de K tb dependem.
>
> Não, W não é um vetor campo, mas um vetor fixo ao elétron.

Ele é fixo ao electrão.ok. isso significa que ele "muda de lugar"
quando o elecrão se move, mas que o seu valor , o seus w_i não mudam.
Eu entendi isso. Portanto, d w_i / d x_i = 0 para qq x_i
Portanto div W = 0 , e rot W = 0;


Seu valor é
> independente da posição (x, y, z), ainda que tenha um fator
dependente de
> como a estrutura interna do elétron se orienta no espaço.

O que é a orientação do electrão, se não a orientação do ppr W ?
Portanto, não vejo que W depende de coisa alguma. Ele é mais como uma
constance universal. Todos os electrões tem um W igual, todos os
protoes têm um W igual e assim vai. Da mesma forma que o momento
angular não depende do ponto do espaço onde o centro de rotação fica,
contudo ele é diferenciaável. A Precessão é isso mesmo. O vector
momento angular roda no espaço. Ora se o W é a orientação do electrão
e essa mode mudar sem que o electrão mude de posição , então W pode
musar e por conseguinte suas coordenadas mudam.


> Não se trata de
> uma função contínua no espaço, logo não podemos derivá-la
direcionalmente.

Podemos, mas o resultado será zero.

> > Ou seja, como são as derivadas em ordem ao tempo?
>
> Depende do movimento que está sendo considerado.

Nenhum. O electrão está parado.

As minhas hipóteses sequer
> falam em tempo!

Isso não significa que ele não exista. Vc fala em explorar a sua
teoria , ora é isso que estou fazendo. E para isso preciso de
considerar o tempo e a variação de W com o tempo.

> > Outra coisa que não consegui encontrar no seu site é a relação entre o
> > vector do electrão e a carga do electrão.
>
> Nem poderia encontrar!!! Você nunca leu o desfecho de minhas
mensagens? Se
> não leu, exponho aqui, mas também estará abaixo: "um elétron não é
uma carga
> elétrica coulombiana".

Vc é que sabe, mas para mim interessa-me saber como se relaciona W com
o numero 1.60217646 × 10^-19 C. A chamada carga do electrão.
A menos que vc tenha uma muito boa razão para que esse numero esteja
errado, ou nõa tenha significado , a sua teoria que fala sobre
electrões deve ao mesmos referir esse numero, nem que seja para
explicar o pq dele não fazer sentido. E foi isso que eu não encontrei,
nem a explicar com a teoria se relaciona com e, nem como não se
relaciona.

O máximo que você poderá procurar no meu trabalho é
> saber qual o artifício que eu utilizo para chegar numa carga elétrica
> macroscópica partindo de estruturas que não se assemelham em nada a uma
> carga elétrica.

E vc sabe o que se assemelha a uma carga electrica ? O que seria ?

Chamo-o
> apenas por elétron, e você dê a ele o nome que quiser, desde que não o
> confunda com uma carga elétrica com características coulombianas
(aquelas
> que geram um campo elétrico de Coulomb).

E quais seria elas ?

Bom, eu parto do principio q vc esteja falando do mesmos electrões que
eu. Esse que ha na natureza e toda a gente diz que estão nos atomos e
tal... ora eles tem propriedade simples como , repelirem-se.
Mas sendo que vc fala tb do protão, dois protoes tb se repelem e um
protão e um electrão não se repelem, eles atraem-se.

Então alguem inventou este numero, a que chamamos carga, que tem sinal
e de forma simples dizenmos que as coisas se atraem se o sinal for
diferente e se repelem caso contrario. Isto, a menos que uma delas
seja zero. Caso em que nada acontece. O sinal da força é portanto dado
pelo produto das cargas. Sendo que forças atractivas são negativas
temos que sign(F) = sign(qa)*sign(qb) , onde qa e qb são as cargas de
duas particulas A e B e sing é a função matemática Sinal (que para
quem nao sabe defini-se para qq x tal que é 0 se x =0 , 1 se x > 0 e
-1 se x<0.)

É desta carga que estou a falar, aliás não conheço outra.

> > Se o electrão A cria um campo de efeitos EA e este é captado por um
> > electrão B que tem o seu campo EB , qual é o efeito de A em B , ou
> > seja, o que acontece a B?
>
> Isso está exposto no capítulo 6,

E qual é o link ? É que eu não consegui navegar até lá.

Já agora, qual é o link que relaciona o campo A com W ?

A pergunta é simples. Como se relaciona o W da particula 1 com o
movimento da particula 2 , supondo que este movimento é apenas devido
à existencia do campo A criado por 1.
Em termos newtonianos, como se relaciona a força sobre a particula 2
com W de 1.


> > Outra interrogação é se a teoria so se aplica a electrões e protoes ou
> > a qq particula com carga , e nesse caso como entram as particulas sem
> > carga?
>
> Em vista do acima exposto, devolvo a pergunta com outras: O que é uma
> partícula com carga? O que é uma partícula sem carga?

Carga é uma propriedade de todo o objecto fisico assim como a massa.
As particulas, sendo objectos fisicos , tem carga. Ela pode ser zero,
caso em que se diz que não têm carga, mas de facto isso é uma dobra na
linguagem para significar que a carga é nula e não que não a têm.
Da mesma forma que um corpo parado tem velocidade, mas é nula.

O que é uma partícula
> e o que é uma partícula realmente elementar?

Ninguem falou em particulas elementares. Particulas são objectos de
estudo fisico - sistemas - que para efeitos desse estudo :
1) não nos preocupamos com o que são, ou qual é a sua estrutura, mas
apenas com suas propriedades fisicas como um todo.
2) são suficientes para explicar / resolver um problema fisico em estudo.
3) são a causa dos efeitos em estudo. (quer directamente ou por
interação com outras particulas)
4) suas dimensões são muito menores que as do sistema onde observamos
os efeitos em estudo.


Sérgio Taborda






SUBJECT: Re: Ação e Reação - sistema fechado
FROM: marcelo ferrari <emailferrari@yahoo.com.br>
TO: ciencialist@yahoogrupos.com.br
DATE: 18/12/2004 22:43

Imagine um quarto fechado com espelhos por todos os lados, em diferentes angulaçoes, aleatórias. Um ambiente fechado onde cada parede reflita a luz para outra parede e assim por diante. Fazemos um buraco em algum canto qualquer e colocamos ali a ponta de uma canhão de raio lazer e ligamos o canhão.


Perguntas:

1) O facho de luz que sai da ponta do canhão vai retornar ao canhão?
2) Porque?
3) Tem alguma lei cientifica que descreve isto?


NATÁLIA > Marcelo, o que é para ti um sistema fechado?


Sistema fechado é o oposto de aberto. Um circulo, por exemplo, é um sistema fechado. No meu exemplo, a sala tem que ser fechada, ou seja, tem que ter espelhos por todos os lados "cercada de espelhos".

Sobre as suas outras mensagens, veja, fiz uma pergunta para colher opiniões, e não sermões. É sempre a mesma ladainha? Eu entendo que existe todas as implicaçoes disto, daquilo e os cambau, apenas criei um exemplo analógico para facilitar a analise e a analogia, eu determinei o exemplo, a bola que vai e volta. Não tem nada de filosofia, nada de metafisica, nada de fora do comum, é uma coisa que acontece regularmente e de fácil visualização. Eis o porque do exemplo. Determinei no exemplo que ir é ação e voltar é reação. Determinei para poder fazer a pergunta que queria investigar e colher as opiniões. E vc vem com a ladainha da virgem maria. Poxa!Você nem sabe porque eu fiz a pergunta e já fica julgando quem vai pro céu e por inferno. Natália, poupe-me, poupe-me, poupe-me, poupe-me, por favor.


marcelo ferrari



__________________________________________________
Converse com seus amigos em tempo real com o Yahoo! Messenger
http://br.download.yahoo.com/messenger/

[As partes desta mensagem que não continham texto foram removidas]



SUBJECT: Preço do vinho x base da garrafa
FROM: Franco <dfranco@pop.com.br>
TO: ciencialist@yahoogrupos.com.br
DATE: 19/12/2004 00:44

Um inglês, desconfiado a respeito de que garrafas de vinhos cujos fundos
(base da garrafa) tinham maior curvatura fossem mais caros que aqueles
cuja base era menos curva, resolveu tirar a prova. Foi ao supermercado e
tirou medidas de cerca de 50 garrafas, plotou-as em um gráfico e ainda
estabeleceu uma fórmula relacionando o preço do vinho com a profundidade
do fundo da garrafa. Não sei se um sujeito desses pode ser levado a sério.
O link é este:
http://www.itchysquirrel.com/

A mesma página, traduzida pelo Google:
*http://tinyurl.com/3zjz4

*Franco


[As partes desta mensagem que não continham texto foram removidas]



SUBJECT: Re: Ação e Reação - sistema fechado
FROM: "Sergio M. M. Taborda" <sergiotaborda@yahoo.com.br>
TO: ciencialist@yahoogrupos.com.br
DATE: 19/12/2004 02:09


--- Em ciencialist@yahoogrupos.com.br, marcelo ferrari
<emailferrari@y...> escreveu
> Imagine um quarto fechado com espelhos por todos os lados, em
diferentes angulaçoes, aleatórias. Um ambiente fechado onde cada
parede reflita a luz para outra parede e assim por diante. Fazemos um
buraco em algum canto qualquer e colocamos ali a ponta de uma canhão
de raio lazer e ligamos o canhão.
>
>
> Perguntas:
>
> 1) O facho de luz que sai da ponta do canhão vai retornar ao canhão?

Não sei.

> 2) Porque?

Pq vc não especificou como os espelhos estavam orientados, e não disse
onde o furo era feito

> 3) Tem alguma lei cientifica que descreve isto?

Tem. Chama-se Lei da refleção e é do ramo da Optica.
E diz assim:
O angulo do raio incidente é igual ao angulo do raio refletido, onde
os angulos são tomados em relação à perpendicular À supercie no ponto
de incidencia.

> NATÁLIA > Marcelo, o que é para ti um sistema fechado?
>
>
> Sistema fechado é o oposto de aberto.


Fisicamente não é.
Um sistema aberto é um sistema de onde pode sair e para onde pode
entrar massa e energia.
Um sistema fechado é um sistema de onde pode sair e para pode entrar
energia, mas não massa.
Como vê , não são antonimos.
Um sistema isolado é um sistema de onde não podem sair e para onde não
podem entrar nem energia, nem massa.


>Um circulo, por exemplo, é um sistema fechado.

Não, não é, pq não é um sistema fisico.

> No meu exemplo, a sala tem que ser fechada, ou seja, tem que ter
espelhos por todos os lados "cercada de espelhos".

Quando vc faz um furo ela deixa de ser fechada.


Sergio Taborda





SUBJECT: Malária estudada em química computacional
FROM: Maria Natália <grasdic@hotmail.com>
TO: ciencialist@yahoogrupos.com.br
DATE: 19/12/2004 02:28


Usando o RAPID do Centro do Centro de biologia e química médica
estrutural e Química Computacional na Universidade de Uppsala,Suécia
o grupo do Profesor Alwyn Jones conseguiu trabalhar com uma nova
enzima do parasita da malária, Plasmodium falciparum.Eles
sóprecisaram da sequência do amino ácido nessa enzima. Estes
resultados vão permitir pesquisa em futuros fármaco para tratamento
da doença.
Podem ler mais em: Joounal of Biochemistry, Professor Johan Åqvist
estudante de doutoramento Sinisa Bjelic.
http://info.uu.se/fakta.nsf/sidor/uppsala.university.id5D.html
Maria Natália






SUBJECT: Re: Ação e ReAção
FROM: "Sergio M. M. Taborda" <sergiotaborda@yahoo.com.br>
TO: ciencialist@yahoogrupos.com.br
DATE: 19/12/2004 02:36


--- Em ciencialist@yahoogrupos.com.br, marcelo ferrari
<emailferrari@y...> escreveu
> Até onde vai uma ação?
> Onde termina a ação e começa a reação?

Esses conceitos não se podem usar assim. Não faz sentido perguntar
onde começa e onde abaca uma ou outra.

Existem dois conceitos chamados acção. A Força de Acção e a Acção
propriamente dita. No contexto vc esta falando da primeira.

A terceira lei de newton enuncia o que é isso , desta forma:

Para cada força exercida por um corpo noutro , existe uma outra força
, exercida por este no primeiro, que é igual e oposta a aquela.

Bom, chamemos de Fa>b à primeira e Fb>a À segunda. A Lei de newton diz
que elas tem que se iguais, ou seja, mesmo modulo, e direcção , e
opostas, ou seja, em sentidos inversos. Se a primeira aponta de sul
para norte, a outra apontará de norte para sul. Mas a leis tb diz que
são duas forças , o que significa que ha dois geradores de força e
dois pontos de apliação.

O exemplo classico é uma pessoa andando. Ao andar vc faz força no chao
e o chao faz força no seu pé. é esta segunda que o faz andar e não a
primeira. A força que vc faz é a acção, pq inicia o processo.
A outra é a reacção pq responde ao processo.
Vc coloca o seu pé à sua frente e empurra o cham no sentido para trás.
O cháo responde com uma força no sentido oposto, o que o faz andar
para a frente.

Básicamente os pares acção-reação são a forma da natureza garantir que
nenhuma perturbação ao seu equilibrio ficará sem resposta.


É a Terra que gira em torno do sol ou o sol que gira em torno da terra
?Esta pergunta não tem sentido fisico, pq na realidade ambos giram em
torno de um 3 ponto que não corresponde nem com um, nem com outro.
Como eu sei ? Porque para cada força que o sol exerça na terra , a
Terra exercerá no sol. Isso significa que a terra está constantemente
caindo no sol e o sol na terra. Aliás, todo o sistema solar é como
uma explosão de granada ao contrario, em que as coisas caem umas para
as outras em vez de se afastarem...
Só que, tem um promenor, que newtow demonstrou. Se a força aplicada
num corpo é central, ou seja, ela aponta sempre para o mesmo local,
essa força produz um movimento de rotação em torno desse local.
Esse ponto será o centro de massa do sistema. Que não tem que ser
nenhum dos corpos. No caso do sol , ele está dentro do volume do sol,
mas isso é mera coincidencia. Poderia não estar.
Ora, isto significa que a terra ficará rodando em torno do ponto para
o qual está caindo, obviamente nunca chegando lá e portanto entrando
num ciclo sem sim, que a mantem eternamente rodando em torno desse ponto.

Sergio Taborda






SUBJECT: Re: Ação e Reação - sentido
FROM: "Sergio M. M. Taborda" <sergiotaborda@yahoo.com.br>
TO: ciencialist@yahoogrupos.com.br
DATE: 19/12/2004 02:44


--- Em ciencialist@yahoogrupos.com.br, marcelo ferrari
<emailferrari@y...> escreveu
> Recebi várias respostas, de vários lugares diferentes, abstraindo,
todas parecem apontar para o mesmo lugar: sentido.
>
> A bolinha está indo assim:
>
> > o > o > o > o > o > o >
>
> ao interagir, ou re-agir com a parede fica assim
>
> < o < o < o < o < o < o <
>
> Se...
>
> Ação = > o > o > o > o > o > o >
>
> e reAção = < o < o < o < o < o < o <
>
> O que muda é apenas o sentido, concordam?

Não.
Primeiro Acção e Reacção não são movimentos. São forças.
Segundo, SE A => B implica que A seja verdadeiro o que não acontece.
Ação =/= > o > o > o > o > o > o >
e
Reação =/= > o > o > o > o > o > o >

pelo que nada se conclui daqui.


Sérgio Taborda






SUBJECT: Re: Ação e Reação - sistema fechado
FROM: Maria Natália <grasdic@hotmail.com>
TO: ciencialist@yahoogrupos.com.br
DATE: 19/12/2004 02:48


Marcelo:

1-Sistema fechado para um químico e um físico é aquele em que não há
trocas de matéria com o exterior. E nunca se deve definir pela
negativa ou usando o antónimo. Seria esquisito eu dizer que um
quadrado é o que não é pentágono.Embora saibamos que círculo não é
circunferência.
2-Perguntou ou usou um físico para suas pesquisas?
E se um físico lhe respondeu também leu a sua resposta como físico
apenas. Aliás ao dirigir-se numa sala a um físico certamente que não
queria um poema de Rómulo de Carvalho...
Pode pedir a um físico que lhe dê a sua opinião sobre a moda de Ana
Salazar mas olhe que ele não é perito em moda.
3--Deve evitar as mensagens para louvar quem responde. Não é poesia,
não é cantoria, nem é romance ou conto. É apenas falar de Ciência
usando termos científicos. Pediu Ciência e a teve como o tem
qualquer pessoa que se dirija a esta lista. Ajudamos mas não
resolvemos. É só pode ser isso: Nada sabemos mas ao menos o sabemos.
Era só
Maria Natália, que não está para metafísicas

--- Em ciencialist@yahoogrupos.com.br, marcelo ferrari
<emailferrari@y...> escreveu
> Imagine um quarto fechado com espelhos por todos os lados, em
diferentes angulaçoes, aleatórias. Um ambiente fechado onde cada
parede reflita a luz para outra parede e assim por diante. Fazemos
um buraco em algum canto qualquer e colocamos ali a ponta de uma
canhão de raio lazer e ligamos o canhão.
>





SUBJECT: Re: Ação e Reação - sistema fechado
FROM: Maria Natália <grasdic@hotmail.com>
TO: ciencialist@yahoogrupos.com.br
DATE: 19/12/2004 03:27


Sérgio:
Para menino de 8º ano sistema aberto (e falo de química) é aquele é
em não há troca de matéria com o exterior: portanto nem entra nem
sai matéria. Pode ser sistema fechado aquele em que durante 10 min o
aluno junta X mL de AgNO3 (aq) a Y mL de NaCl (aq) e se forma o
precipitado pois que se considera a perda de água por vaporização
como desprezável.
Quando a energia não transita falo em sistema isolado. E nem será
conveniente falar-se de matéria e velocidade da luz pois em lab de
ensino elementar não se podem atingir tais velocidades.
Pelas respostas sobre a dúvida da Acção Reacção se vê que este era o
nível de resposta necessária para este utente da C.List.
Sem dúvida que as leis da óptica Mas não convem esquecer a lei das
probabilidades pois se trata mesmo disso quando não se dá sequer a
orientação nem o tipo de espelhos em causa.
Antónimos---sistema aberto é o que não está fechado e o fechado é o
que não está aberto.
Onde se fala de sistemas, aberto, fechado e isolado pela primeira
vez a crianças, cientificamente falando, não é na física mas na
química.
Mas certamente viu a palavra metafísica no fim...e depois o Marcelo
é linguista e um linguista esgrime com palavras sem relacionar
muitas vezes os conceitos científicos por trás das palavras. Há
efectivamente muitas concepções alternativas no consulente a que não
liguei pois sabia de sua profissão e não o quis enjoar com mais um
dos meus sermões aos peixinhos (o Peixinho q me desculpe). Ai de mim
se ao fim destees anos ainda andava de carroça.LOL
Um abraço ao físico de plantão
Maria Natália


--- Em ciencialist@yahoogrupos.com.br, "Sergio M. M. Taborda"
<sergiotaborda@y...> escreveu
>
> --- Em ciencialist@yahoogrupos.com.br, marcelo ferrari
> <emailferrari@y...> escreveu
> > Imagine um quarto fechado com espelhos por todos os lados, em
> diferentes angulaçoes, aleatórias. Um ambiente fechado onde cada
> parede reflita a luz para outra parede e assim por diante. Fazemos
um
> buraco em algum canto qualquer e colocamos ali a ponta de uma
canhão
> de raio lazer e ligamos o canhão.
> >
> >
> > Perguntas:
> >
> > 1) O facho de luz que sai da ponta do canhão vai retornar ao
canhão?
>
> Não sei.
>
> > 2) Porque?
>
> Pq vc não especificou como os espelhos estavam orientados, e não
disse
> onde o furo era feito
>
> > 3) Tem alguma lei cientifica que descreve isto?
>
> Tem. Chama-se Lei da refleção e é do ramo da Optica.
> E diz assim:
> O angulo do raio incidente é igual ao angulo do raio refletido,
onde
> os angulos são tomados em relação à perpendicular À supercie no
ponto
> de incidencia.
>
> > NATÁLIA > Marcelo, o que é para ti um sistema fechado?
> >
> >
> > Sistema fechado é o oposto de aberto.
>
>
> Fisicamente não é.
> Um sistema aberto é um sistema de onde pode sair e para onde pode
> entrar massa e energia.
> Um sistema fechado é um sistema de onde pode sair e para pode
entrar
> energia, mas não massa.
> Como vê , não são antonimos.
> Um sistema isolado é um sistema de onde não podem sair e para onde
não
> podem entrar nem energia, nem massa.
>
>
> >Um circulo, por exemplo, é um sistema fechado.
>
> Não, não é, pq não é um sistema fisico.
>
> > No meu exemplo, a sala tem que ser fechada, ou seja, tem que ter
> espelhos por todos os lados "cercada de espelhos".
>
> Quando vc faz um furo ela deixa de ser fechada.
>
>
> Sergio Taborda





SUBJECT: Re: [ciencialist] Re: Electrão como um vector
FROM: "Alberto Mesquita Filho" <albmesq@uol.com.br>
TO: <ciencialist@yahoogrupos.com.br>
DATE: 19/12/2004 07:08

----- Original Message -----
From: "Sergio M. M. Taborda"
Sent: Saturday, December 18, 2004 8:54 PM
Subject: [ciencialist] Re: Electrão como um vector

> > Continuação: [...]

> A minha pergunta sobre pq um vector no sentido mais epistemologico. O
> que significa atribuir 3 quantidades ao electrão , o que são elas, e
> pq apenas 3 ? pq não 6 (matriz antisimetrica)? pq não 9 (matriz) pq
> não uma matriz simetrica em vez de um vector ? Quando vc diz Vector
> vcs está postulando a natureza matemática , um modelo matemático, para
> a representação do electrão, mas não está informando a razão fisica
> para essa escolha. por isso perguntei. Eu sei que sendo uma hipotese,
> qq coisa é válida, mas esperava que sendo um assunto fisico, tivesse
> algumas desculpa fisica para a escolha.

Espero ter respondido a tudo isso na mensagem introdutória
http://br.groups.yahoo.com/group/ciencialist/message/43275
É possível que você não tenha gostado do método analógico utilizado mas, se
foi o caso, eu já esperava por isso. Como o assunto se repete nos demais
questionamentos, e está sendo apresentado de uma maneira um tanto quanto
confusa, vou parar por aqui, pois é possível que eu tenha que voltar a esse
tema mais abaixo.

> Bem, um co-vetor é um objecto que após uma transformação não matem a
> mesma estrutura. Ou seja, os valoes w_i dependeriam do referencia
> matemático usado para os representar. Coisa que não acontece com um
> vector.

Bem, não quero discutir matemática, mas acho que estamos nos referindo a
coisas distintas. Os valores x_i de um vetor de posição r mudam com a
rotação do referencial, e não me parece que o vetor de posição seja um
co-vetor.

De qualquer maneira, a idéia é bastante interessante, mas quando relacionada
a efeitos outros e estudado no capítulo das aberrações relativistas.
Seria o caso de dois observadores, situados em referenciais em movimento (um
em relação ao outro) e visualizando alguma característica física de um
objeto ou fenômeno, destes que se prestam a serem representados por vetores.
Neste caso sugiro que você veja a figura 7 do item 6 do meu artigo "O Espaço
Curvo Euclidiano e a Relatividade Galileana". O item 6 está no URL
http://ecientificocultural.com/Relat/Euclides6.htm , e a página inicial do
artigo em http://ecientificocultural.com/Relat/espaco_curvo01.htm . A figura
7, que pode ser visualizada clicando-se em
http://ecientificocultural.com/Relat/Eucli07.GIF , mostra exatamente como o
vetor w (em vermelho) se mostraria para um observador em repouso e para
outro observador com uma velocidade v = 0,5c.

Para um melhor entendimento da figura talvez fosse necessário ler o item 6
do artigo, mas estou me referindo ao mesmo apenas a título de curiosidade e
a mostrar que essa característica relativística existe na minha teoria,
apesar de não estar explícita em nenhuma das hipóteses. Aliás, uma teoria
terá sido tão melhor axiomatizada quanto mais enxuta for em hipóteses.
Redundância, na montagem de um arcabouço teórico, não combina com
teorização, e espero já ter dito isso com outras palavras.

Essa aberração está a retratar uma virtualidade, se pensarmos em
concordância com os cânones clássicos. Segundo o realismo clássico (e a
física clássica tem muito a ver com esse realismo) um objeto é o que é, a
despeito de como ele se nos aparenta ser. Obviamente, esta aparência pode se
traduzir em um fenômeno físico (por exemplo, na ação de um campo elétrico).
O fenômeno (a ação do campo sobre outros objetos) é real e o campo pode ser
considerado como tal (num contexto relativístico, é bem verdade), mas o
objeto que está produzindo esse campo não é aquele que estamos visualizando
(a aberração) e sim o objeto como ele é na realidade. O artigo citado
ilustra outras aberrações mais simples do que essa (por exemplo, no item 2),
logo não vou me alongar a respeito.

Quando digo que uma estrutura física pode ser descrita matematicamente por
um vetor, há que se destacar que devem existir aberrações dessa estrutura
(ou virtualidades) que também podem ser descritas matematicamente por outros
vetores. A matemática tanto pode representar o real quanto o virtual, e um
físico realista deve saber distinguir, ou pelo menos suspeitar, sobre a qual
desses objetos ele está aplicando a matemática.

Não tenho a pretensão de convencer um utilitarista sobre as vantagens do
realismo, mas posso apontar as diferenças entre os métodos. Dependendo da
metodologia aceita (realista ou utilitarista), até mesmo o vetor de posição
poderia se encaixar no que você chama de co-vetor, mas também não sei se
alguém concordaria com a conceituação que você apresentou para co-vetor.
Enfim, "palavras são palavras, nada mais que palavras".

> > certamente ficaremos frente as mesmas evidências
> > experimentais que levaram os físicos "modernos" a suspeitarem sobre
> > a ainda mal explicada, a meu ver, natureza quiral do elétron como, por
> > exemplo, a experiência de Stern Gerlach

> O que vc está dizendo é que 2 electrões não tem que ser iguais. ...

A experiência de Stern Gerlach não é feita com elétrons, mas sim com átomos.

> ...ora, isso muito interessante pq vai contra a seua hipotese.

Porque haveria de ir? Em qual hipótese você leu que dois elétrons têm que
ser iguais? Eu não fiz esta afirmação e também não disse o contrário. Mas
talvez seja esta uma boa oportunidade para discorrer a respeito.

Em primeiro lugar, e pela hipótese 1, eu diria que existem infinitos
elétrons diferentes, conforme a orientação espacial do elétron. Você poderá
me questionar e dizer que eles seriam idênticos a menos de uma rotação. Mas
essa identidade também não é assumida pela minha teoria, pois eu não fiz
essa hipótese. Se essa identidade de fato existir, deixo a critério da
experimentação, pois não precisei dessa hipótese. Digo mais: deixei essa
hipótese em aberto no núcleo da teoria (núcleo de uma teoria = conjunto das
hipóteses básicas da teoria). Volte a ler a hipótese 1
[ http://ecientificocultural.com/Eletron/eletron22.htm ] e note que lá está
escrito o seguinte: "K é *presumivelmente* constante."

Porque esse cuidado? Afinal, K é ou não é constante?

Na página http://ecientificocultural.com/Eletron/eletron23.htm , em
Comentários sobre H-1, lê-se o seguinte: "Convém deixar um grau de liberdade
na relação que define w, liberdade esta que se mostrará útil em determinadas
circunstâncias." No final do item 7.2.1 (capítulo 7) [vide
http://ecientificocultural.com/Eletron/eletron72.htm#7.2 ] eu volto ao tema
com as seguintes palavras: "Como já afirmei em artigo anterior, o escalar K,
da hipótese 1 (e, portanto, o valor absoluto do vetor w), parece conter
segredos relacionados aos referenciais inerciais, que somente a física
experimental pode decifrar. Seria extremamente interessante verificar se o
seu valor absoluto, uma vez definido, permanece ou não idêntico, qualquer
que seja o referencial inercial considerado. A variabilidade de K seria um
indício fortemente sugestivo a corroborar a intuição de Newton quanto à
existência de um referencial absoluto. É de se esperar, no entanto, a
observação de K aproximadamente constante para v << c."

Esse "é de se esperar" está em acordo com inúmeras experiências já efetuadas
em eletromagnetismo e interpretadas à luz da minha teoria.

> Se W é um vector intrinseco do electrão. Para todos os efeitos esse
> vector é o electrão, então 2 eletrões têm que ter o mesmo vector.

Eu diria de outra maneira: Sempre que K for constante, o módulo do vetor w
será o mesmo. Redundância?!!! Sim, pois K é o módulo do vetor w. ;-)

> Ora um vector, 3 quantidades, não permite quiralidade.

Eu nunca disse o contrário. Você é quem está confundindo o elétron com um
vetor. Eu simplesmente estou representando uma das propriedades do elétron
que interessa para a minha teoria por um vetor. Se este vetor é ou não
axial, isso deve decorrer da observação experimental, e isso é sugerido pela
experiência de Stern Gerlach, dentre outras.

> Para isso vc
> precisa no minimo de uma matix anti-simetrica. (em que os termos fora
> do traço seriam diferentes obtendo a quiralidade )

Se o vetor for axial (e tudo indica que é), fique à vontade para determinar
experimentalmente qual a regra a ser adotada para determinar a quiralidade
*do elétron* (e não do vetor). Para alicerçar a minha teoria eu não precisei
dessa hipótese, mesmo porque achei que não convinha avançar muito em direção
ao representacionalismo. É claro que se pretendermos destrinchar a estrutura
do elétron, teremos que evoluir nessa direção, acrescentando outras
hipóteses, mas para explicar o eletromagnetismo microscósmico creio que
seria correr um risco desnecessário.

> O que vc está dizendo com isso - embora tlv não conscientemente - é
> que o vector pode ser representado por uma função escalar e um outro
> vector . Este vector obviamente seria um covector e a função escalar é
> apenas para efeitos de normalização.

Você está confundindo "vetor" com "campo vetorial". A força, por exemplo, é
um vetor, e como qualquer outro vetor, pode SEMPRE ser representada pelo
produto de um escalar, a que chamamos módulo, por um vetor unitário (ou
versor). O campo elétrico, por outro lado, é um campo que pode ser
representado como o gradiente de uma função escalar. Neste caso sim, podemos
representar o campo elétrico como o produto de uma função escalar (o módulo
do gradiente no ponto considerado) por um versor, com a direção e sentido
desse gradiente. Um versor que tenha a direção e o sentido de um gradiente
pode ser chamado por covetor. Este versor perfura as superfícies
equipotenciais, ou seja, ele "co-tangencia" essas superfícies, e a expressão
co-vetor decorre dessa propriedade.

> Eu entendi isso. Portanto, d w_i / d x_i = 0 para qq x_i
> Portanto div W = 0 , e rot W = 0;

Bem, se a sua matemática permite a derivação de uma função num ponto de
descontinuidade, então não está mais aqui quem falou. Vamos tocar o barco.

> O que é a orientação do electrão, se não a orientação do ppr W ?
> Portanto, não vejo que W depende de coisa alguma. Ele é mais como uma
> constance universal. Todos os electrões tem um W igual, todos os
> protoes têm um W igual e assim vai. Da mesma forma que o momento
> angular não depende do ponto do espaço onde o centro de rotação fica,
> contudo ele é diferenciaável. A Precessão é isso mesmo. O vector
> momento angular roda no espaço. Ora se o W é a orientação do electrão
> e essa mode mudar sem que o electrão mude de posição , então W pode
> musar e por conseguinte suas coordenadas mudam.

Eu diria que você está fazendo uma miscelânea total. Você começa com algumas
dentre as hipóteses ou conseqüências da física moderna (por exemplo, todos
os elétrons seriam iguais). A partir daí você assume que o meu W seria
equivalente a um momento angular que certamente não deve ser clássico, pois
do contrário os elétrons não seriam todos iguais, ainda que o módulo desse
momento fosse, por hipótese, igual. E conclui que o meu W pode ser derivado
temporalmente, coisa que já assumi na msg anterior. Sinceramente, não sei
nem por onde começar minha crítica ao parágrafo, logo deixarei o dito pelo
não dito.

> > Não se trata de uma função contínua no espaço, logo não podemos
> > derivá-la direcionalmente.

> Podemos, mas o resultado será zero.

Creio que não. Seria zero se fosse uma função contínua e de valor constante
na direção da derivação. Mas como disse acima, estou utilizando-me de outra
matemática.

> > > Ou seja, como são as derivadas em ordem ao tempo?

> > Depende do movimento que está sendo considerado.

> Nenhum. O electrão está parado.

Neste caso a resposta é óbvia. A derivada temporal será zero.

> Vc é que sabe, mas para mim interessa-me saber como se relaciona W com
> o numero 1.60217646 × 10^-19 C. A chamada carga do electrão.

Essa suposta carga do elétron foi determinada em campos elétricos
constantes. Eu não falo em números, mas comento sobre o significado
fenomenológico dessa constante a partir do item 4.3. Posteriormente, eu
chego à expressão analítica da lei de Coulomb (item 4.3.1), mas em função do
número N de elétrons que compõem uma carga elétrica (equação 4.19).
Conhecendo-se a carga macroscópica Q, esse número N pode ser estimado por
métodos a apoiarem-se nas leis de Faraday, e com isso a constante da equação
fica determinável. Conhecendo-se a carga Q e o número N, qualquer um ficará
tentado a dividir Q por N na esperança de encontrar a carga do elétron.
Provavelmente chegaremos no número que você apontou, mas isso não significa
dizer que dois elétrons interagem-se através de uma carga deste valor. Por
outro lado, se você entender como cheguei na lei de Coulomb, entenderá
porque esse número é pouco importante para a minha teoria.

> > O máximo que você poderá procurar no meu trabalho é
> > saber qual o artifício que eu utilizo para chegar numa carga elétrica
> > macroscópica partindo de estruturas que não se assemelham em nada a uma
> > carga elétrica.

> E vc sabe o que se assemelha a uma carga electrica ? O que seria?

Assemelha-se aos corpos eletrizados descritos por Coulomb entre 1784 e 1789.

> > Chamo-o apenas por elétron, e você dê a ele o nome que quiser, desde que
> > não o confunda com uma carga elétrica com características coulombianas
> > (aquelas que geram um campo elétrico de Coulomb).

> E quais seria elas?

Respondi logo acima. Em dúvida é só consultar a literatura, mesmo porque a
experiência de Coulomb já foi repetida milhares de vezes.

> Bom, eu parto do principio q vc esteja falando do mesmos electrões que
> eu. Esse que ha na natureza e toda a gente diz que estão nos atomos e
> tal... ora eles tem propriedade simples como , repelirem-se.
> Mas sendo que vc fala tb do protão, dois protoes tb se repelem e um
> protão e um electrão não se repelem, eles atraem-se.

Que mal pergunte, qual foi a experiência que constatou, de maneira
irrefutável, que dois elétrons ou dois prótons se repelem sempre e de
maneira idêntica a uma carga elétrica, ou seja, através de forças
coulombianas?

> > Isso está exposto no capítulo 6,

> E qual é o link ? É que eu não consegui navegar até lá.

Você pode chegar a todos os itens ou capítulos a partir da página
http://ecientificocultural.com/Eletron/eletron0.htm (clique nos links que
estão na tabela intitulada CONTEÚDO). Em particular, o início do capítulo 6
está em http://ecientificocultural.com/Eletron/eletron6.htm e as equações a
que me referi estão na página seguinte, ou seja, em
http://ecientificocultural.com/Eletron/eletron62.htm . Para prosseguir no
capítulo clique no link "Próximo" situado no final da página, ou então volte
para a tabela CONTEÚDO.

> Já agora, qual é o link que relaciona o campo A com W ?

Vide a equação 6.3 em http://ecientificocultural.com/Eletron/eletron6.htm

> A pergunta é simples. Como se relaciona o W da particula 1 com o
> movimento da particula 2 , supondo que este movimento é apenas devido
> à existencia do campo A criado por 1.
> Em termos newtonianos, como se relaciona a força sobre a particula 2
> com W de 1.

As perguntas são simples, mas as resposta não são. Eu diria que o conjunto
dos 8 capítulos (e mais especialmente, a partir do capítulo 4) foram
escritos tendo como finalidade principal apontar um caminho para a resolução
de problemas desse tipo.

> > Em vista do acima exposto, devolvo a pergunta com outras: O que é uma
> > partícula com carga? O que é uma partícula sem carga?

> Carga é [...]

Pô, Sérgio. Eu esqueci que quando faço uma pergunta você sempre responde. No
Brasil nem sempre a gente faz uma pergunta esperando uma resposta. As vezes
a pergunta é colocada apenas no sentido de enfatizar uma afirmação anterior,
ou a demonstrar que o problema é mais complexo do que parece à primeira
vista. Neste caso não era para responder, mesmo porque eu não estou aqui
sabatinando-o. Desculpe-me pelo trabalho que lhe dei, mas não foi
intencional.

[ ]´s
Alberto
http://ecientificocultural.com/indice.htm
Mas indiferentemente a tudo isso, o neutrino tem massa, o elétron não é
uma carga elétrica coulombiana e a Terra se move. E a história se repetirá.



SUBJECT: Re: Ação e Reação - sentido
FROM: marcelo ferrari <emailferrari@yahoo.com.br>
TO: ciencialist@yahoogrupos.com.br
DATE: 19/12/2004 07:39


TABORDA > Acção e Reacção não são movimentos.
Ação =/= > o > o > o > o > o > o >
e Reação =/= > o > o > o > o > o > o >

pelo que nada se conclui daqui.


- - - - - - -

Sérgio, "suponhamos" que:

Ação = > o > o > o > o > o > o >
e reAção = < o < o < o < o < o < o <

pergunta:

1)Onde termina a ação e onde começa a reação?

2)Porque?

3)Qual a ligação entre a ação a reação?



grato.

marcelo ferrari



---------------------------------
Yahoo! Acesso Grátis - Internet rápida e grátis. Instale o discador do Yahoo! agora.

[As partes desta mensagem que não continham texto foram removidas]



SUBJECT: Re: Ação e Reação - sistema fechado
FROM: marcelo ferrari <emailferrari@yahoo.com.br>
TO: ciencialist@yahoogrupos.com.br
DATE: 19/12/2004 08:52

Imagine um quarto fechado com espelhos por todos os lados, em diferentes angulaçoes, aleatórias. Um ambiente fechado onde cada parede reflita a luz para outra parede e assim por diante. Fazemos um buraco em algum canto qualquer e colocamos ali a ponta de uma canhão
de raio lazer e ligamos o canhão.

Perguntas:

1) O feixo de luz que sai da ponta do canhão vai retornar ao canhão? Porque?

TABORDA > Não sei. Pq vc não especificou como os espelhos estavam orientados, e não disse
onde o furo era feito

FERRARI > Vamos supor uma esfera espelhada. O feixo de luz que sai da ponta do canhão vai retornar ao canhão? Porque? Tem alguma lei cientifica que descreve isto?

TABORDA > Tem. Chama-se Lei da refleção e é do ramo da Optica. E diz assim: O angulo do raio incidente é igual ao angulo do raio refletido, onde os angulos são tomados em relação à perpendicular à supercie no ponto de incidencia.

Sérgio, e sobre voltar ao ponto de origem, tem alguma lei cientifica sobre isto.

TABORDA > Um sistema fechado é um sistema de onde pode entrar energia, mas não massa. Um sistema isolado é um sistema de onde não podem sair e para onde não
podem entrar nem energia, nem massa.

No meu exemplo, o feixe não pode sair, pois tem espelhos por todos os lados, o único buraco é por onde ele entrou. Ele é fechado, isolado, ou o que?

grato.

marcelo ferrari


---------------------------------
Yahoo! Acesso Grátis - Internet rápida e grátis. Instale o discador do Yahoo! agora.

[As partes desta mensagem que não continham texto foram removidas]



SUBJECT: Enviando email: www.albert.einstein.nom.br
FROM: "JVictor" <jvoneto@uol.com.br>
TO: <ciencialist@yahoogrupos.com.br>
DATE: 19/12/2004 10:36

Lista,

Eis um site na língua nativa, produzido pelo Físico C.A.Santos, um dos muitos a compreenderem a essência e a importância do trabalho de Einstein. Einstein é mostrado em sua inteireza, os equívocos veiculados pela imprensa a respeito, a respeito de aspectos da pessoa Einstein, são esclarecidos, a verdade dos fatos posta a pratos limpos. Vasta fonte de informações sobre o cientista, vida e obra, é listada, muitas delas resenhadas.
Recomendo.

Victor.

A mensagem está pronta para ser enviada com estes anexos de arquivo ou link:
Atalho para: http://www.albert.einstein.nom.br/

Observação: para se proteger de vírus de computador, os programas de email podem impedir o envio ou recebimento de alguns tipos de anexo de arquivo. Verifique as configurações de email para determinar como os anexos são manipulados.

[As partes desta mensagem que não continham texto foram removidas]



SUBJECT: Re: Ação e Reação - sistema fechado
FROM: "Sergio M. M. Taborda" <sergiotaborda@yahoo.com.br>
TO: ciencialist@yahoogrupos.com.br
DATE: 19/12/2004 12:25


--- Em ciencialist@yahoogrupos.com.br, marcelo ferrari

> FERRARI > Vamos supor uma esfera espelhada. O feixo de luz que sai
da ponta do canhão vai retornar ao canhão? Porque? Tem alguma lei
cientifica que descreve isto?
>
> TABORDA > Tem. Chama-se Lei da refleção e é do ramo da Optica. E diz
assim: O angulo do raio incidente é igual ao angulo do raio refletido,
onde os angulos são tomados em relação à perpendicular à supercie no
ponto de incidencia.
>
> Sérgio, e sobre voltar ao ponto de origem, tem alguma lei cientifica
sobre isto.

Tem, é aquela. Vc tem que a usar para concluir se sai ou não sai. É um
trabalho que ha que fazer, tem que fazer as continhas... ou vc acha
que fisica é só conversa ?

No caso da esfera a resposta é obvia. Assim que o raio entre ele bate
no ponto oposto da esfera e sendo que nesee ponto o raio de incidencia
faz zero graus com a perpendicular , então a resposta é perpendicular
e no sentido contrário, o feixe volta imediatamente ao furo. Seria o
mesmo a sala ser um cubo com as paredes espelhandas e vc abrir o furo
numa delas, o feixo reflete na parede em frente e volta.

> TABORDA > Um sistema fechado é um sistema de onde pode entrar
energia, mas não massa. Um sistema isolado é um sistema de onde não
podem sair e para onde não
> podem entrar nem energia, nem massa.
>
> No meu exemplo, o feixe não pode sair, pois tem espelhos por todos
os lados, o único buraco é por onde ele entrou. Ele é fechado,
isolado, ou o que?

O feixe pode sair sim, se sair pelo furo por onde entrou.
Um feixe é energia e não é considerado massa. A pergunta é: pode
entrar mais alguma coisa por esse furo ? ar , por exemplo?
Se sim, o sistema é aberto. Se não ele é fechado.

Sergio Taborda





SUBJECT: Re: Ação e Reação - sentido
FROM: Maria Natália <grasdic@hotmail.com>
TO: ciencialist@yahoogrupos.com.br
DATE: 19/12/2004 13:04


Marcelo:
1--Que semelhanças há entre a acção e a reacção?
2--Que semelhanças há entre a Acção ea reacção?
Talvez a coisa posta assim na 1ª aproximação ajude a que detectemos
o teu problema. Onde está a tua dificuldade.
3--Vais desenhar um bloco e as forças por vectores assim:
>>>>>>____
>>>>>>>|cor|
>>>>>>>|po |----->
>>>>>>>|___| Estás a ver este boneco' Tenta desenhares como eu fiz
aquela figura para ver se te entendemos. Deves ler outravez a minha
mensagem na outra lista.
[]'
Maria Natália

--- Em ciencialist@yahoogrupos.com.br, marcelo ferrari
<emailferrari@y...> escreveu
>
> TABORDA > Acção e Reacção não são movimentos.
> Ação =/= > o > o > o > o > o > o >
> e Reação =/= > o > o > o > o > o > o >
>
> pelo que nada se conclui daqui.
>
>
> - - - - - - -
>
> Sérgio, "suponhamos" que:
>
> Ação = > o > o > o > o > o > o >
> e reAção = < o < o < o < o < o < o <
>
> pergunta:
>
> 1)Onde termina a ação e onde começa a reação?
>
> 2)Porque?
>
> 3)Qual a ligação entre a ação a reação?
>
>
>
> grato.
>
> marcelo ferrari
>
>
>
> ---------------------------------
> Yahoo! Acesso Grátis - Internet rápida e grátis. Instale o
discador do Yahoo! agora.
>
> [As partes desta mensagem que não continham texto foram removidas]





SUBJECT: Re: Ação e Reação - sentido
FROM: "rayfisica" <rayfisica@yahoo.com.br>
TO: ciencialist@yahoogrupos.com.br
DATE: 19/12/2004 15:38


>

Ó eu aqui, ó!
Blz...
Não sou erudito apenas respondo o que penso saber, ok?

1)Onde termina a ação e onde começa a reação?
R - depende do ponto de referencia, por exemplo, a bola e a parede,
do ponto de vista da parede a bola aplica uma ação sobre a parede
e a
parede aplica uma reação sobre a bola e vice e verso.

2)Por quê?
R - A toda ação corresponde uma reação, com a mesma
intensidade,
mesma direção e sentidos contrários.


3)Qual a ligação entre a ação à reação?
R - A toda ação corresponde uma reação, com a mesma
intensidade,
mesma direção e sentidos contrários.


E ai acertei?






SUBJECT: Re: [ciencialist] Re: Electrão como um vector - CORRIGINDO
FROM: "Alberto Mesquita Filho" <albmesq@uol.com.br>
TO: <ciencialist@yahoogrupos.com.br>
DATE: 19/12/2004 16:08

----- Original Message -----
From: "Alberto Mesquita Filho"
Sent: Sunday, December 19, 2004 6:08 AM
Subject: Re: [ciencialist] Re: Electrão como um vector


A mensagem introdutória, referida na msg acima, saiu com o link errado. O
correto é:

http://br.groups.yahoo.com/group/ciencialist/message/43271

[ ]´s
Alberto
http://ecientificocultural.com/indice.htm
Mas indiferentemente a tudo isso, o neutrino tem massa, o elétron não é
uma carga elétrica coulombiana e a Terra se move. E a história se repetirá.





SUBJECT: Re: Ação e Reação - sentido/Errata
FROM: Maria Natália <grasdic@hotmail.com>
TO: ciencialist@yahoogrupos.com.br
DATE: 19/12/2004 16:20


Ei Marcelo:
houve gralha.
Deve ler-se

Marcelo:
> 1--Que semelhanças há entre a acção e a reacção?
> 2--Que DIFERENÇAS há entre a Acção ea reacção?
> Talvez a coisa posta assim na 1ª aproximação ajude a que
detectemos

--- Em ciencialist@yahoogrupos.com.br, Maria Natália <grasdic@h...>
escreveu
>
> e grátis. Instale o
> discador do Yahoo! agora.
> >
> > [As partes desta mensagem que não continham texto foram
removidas]





SUBJECT: Um meteoro na Indonésia?
FROM: Maria Natália <grasdic@hotmail.com>
TO: ciencialist@yahoogrupos.com.br
DATE: 19/12/2004 16:45


vou lá ver se não é desculpa de Governo:
http://br.news.yahoo.com//041219/5/q5vv.html
É quase "um raio que te parte" LOL
Um abraço
Maria Natália





SUBJECT: Sobre dimensões.
FROM: Leandro Aparecido Côco <apocrifos@ig.com.br>
TO: ciencialist@yahoogrupos.com.br
DATE: 19/12/2004 20:29


Não consigo perceber as outras dimensões além das 4.
Na verdade não consigo ver o tempo como uma dimensão
pelo que entendi as branas criam particulas através da interação com
o espaço-tempo. É isso?

Leandro






SUBJECT: Re: [ciencialist] Um questão de preferência
FROM: "Luiz Ferraz Netto" <leobarretos@uol.com.br>
TO: <ciencialist@yahoogrupos.com.br>
DATE: 19/12/2004 20:38

A minha preferência é que tal assunto seja postado apenas nos sites de socialismo. Não aqui no C-list.
[]'
===========================
Luiz Ferraz Netto [Léo]
leobarretos@uol.com.br
http://www.feiradeciencias.com.br
===========================
-----Mensagem Original-----
De: "José Renato" <jrma@terra.com.br>
Para: <ciencialist@yahoogrupos.com.br>
Enviada em: sexta-feira, 17 de dezembro de 2004 19:18
Assunto: [ciencialist] Um questão de preferência



Qual a opção que vc escolheria para o reajuste do salário mínimo: 290 já em janeiro ou 300 em maio de 2005?

Os números dessa comparação, podem ser verificados nas contas a seguir.

1ª Opção: (290 - 260) x (12 + 1) = 30 x 13 = 390 reais a mais em 2005.

2ª Opção: (300 - 260) x (8 + 1) = 40 x 9 = 360 reais a mais em 2005.

Estamos considerarmos que o próximo SM será corrigido em janeiro de 2006.

Entretanto, se o próximo salário mínimo for reajustado somente em maio de 2006, as contas serão diferentes.

1ª Opção: 30 x (13 + 4) = 30 x 17 = 510

2ª Opção: 40 x (9 + 4) = 40 x 13 = 520.

Verificamos que a melhor opção só poderia ser determinada se soubéssemos em que mês o SM será atualizado em 2006.

Mas a escolha tem de ser definida bem antes disso.
E agora, que opção você prefere?

José Renato M. de Almeida
Salvador - Bahia


[As partes desta mensagem que não continham texto foram removidas]



##### ##### #####

Para saber mais visite
http://www.ciencialist.hpg.ig.com.br


##### ##### ##### #####
Links do Yahoo! Grupos










SUBJECT: Fw: Temperatura de um raio
FROM: "Luiz Ferraz Netto" <leobarretos@uol.com.br>
TO: "ciencialist" <ciencialist@yahoogrupos.com.br>
DATE: 20/12/2004 04:10

Alguém tem um número para eu dar de exemplo? Ou, uma fonte disso?
[]'
===========================
Luiz Ferraz Netto [Léo]
leobarretos@uol.com.br
http://www.feiradeciencias.com.br
===========================
-----Mensagem Original-----
De: Dasdores
Para: leobarretos@uol.com.br
Enviada em: quinta-feira, 16 de dezembro de 2004 16:58
Assunto: Temperatura de um raio



Nas dúvidas experimentais, por gentileza coloque aqui o endereço da página, isso facilita o confronto. Agradeço. Meu nome é LUIZ FERRAZ NETTO, meu apelido é LÉO e moro em BARRETOS; dai vem meu e-mail: leobarretos@uol.com.br.

Nome: Willianderson Freire Vieira
Escolaridade: 7ª Série
E-mail: willy13@walla.com


Olá! Estou fazendo um trabalho e gostaria de saber o seguinte: " Que temperatura atinge um raio durante sua trajetória ao solo " ?

[As partes desta mensagem que não continham texto foram removidas]



SUBJECT: Fw: aerofolio
FROM: "Luiz Ferraz Netto" <leobarretos@uol.com.br>
TO: "ciencialist" <ciencialist@yahoogrupos.com.br>
DATE: 20/12/2004 04:12

Que digo para o Hugo? Sem assustá-lo!
[]'

===========================
Luiz Ferraz Netto [Léo]
leobarretos@uol.com.br
http://www.feiradeciencias.com.br
===========================
-----Mensagem Original-----
De: Hugo Pavan
Para: leobarretos@uol.com.br
Enviada em: quinta-feira, 16 de dezembro de 2004 19:08
Assunto: aerofolio


1. Se eu quisese fazer um avião,como posso saber as dimensões corretas da asa (ter a melhor eficiencia o possivel)? pois com uma asa muito pequena eu não sairia do chão E isto tem alguma coisa a ver com o peso da maquina no total ou não?

[As partes desta mensagem que não continham texto foram removidas]



SUBJECT: Re: [ciencialist] Re: Ação e Reação - sentido
FROM: "Luiz Ferraz Netto" <leobarretos@uol.com.br>
TO: <ciencialist@yahoogrupos.com.br>, <emailferrari@yahoo.com.br>
DATE: 20/12/2004 04:45

Marcelo escreve:
>"Recebi várias respostas, de vários lugares diferentes, abstraindo, todas parecem apontar para o mesmo lugar: sentido."<

Léo: Faz sentido. É o melhor que se pode abstrair dessas várias respostas. O sentido sem dúvida é relativo; para a parede a bola está chegando e ela vai se preparando para entrar em 'reação' --- para a bola é a parede que está se aproximando perigosamente e ela 'vê' nisso um prelúdio para encerrar sua ação. Essa transmissão de informações é inerente ao Universo das coisas. Newton só vislumbrou esse aspécto do universo como um todo, mas não realmente todo, quando terminou a ação dele começou a reação de outro, Huighens, por exemplo.

>"A bolinha está indo assim:
> o > o > o > o > o > o >
ao interagir, ou re-agir com a parede fica assim
< o < o < o < o < o < o <
Se...
Ação = > o > o > o > o > o > o >
e reAção = < o < o < o < o < o < o <
O que muda é apenas o sentido, concordam? "

Léo: Sim, é isso mesmo, o que muda é apenas o sentido. Mudou o sentido de tudo.
Ação é agora algo bem mais profundo, que raia ao infinito da imaginação e a reação que pode ser pensada filosoficamente como re-AÇÃO e uma ação oposto ao efeito da reação; é algo que vem do infinito e vai se tornando finito e no instante zero, onde tudo é possível ou o nada, a reação se inicia e devolve ao universo os efeitos da ação.

>"Sendo assim, concluo que uma "determinada" ação temina no limite do seu sentido e a reAção começa no ponto de mudança de sentido."

Léo: Exatamente, sábias palavras! 'O limite do sentido' ... essa é a pureza do raciocínio que poucos conseguem atingir e, portanto, não conseguem decifrar o enígma da ação e reação de Newton. Realmente é profundo .... a bola vai indo, vai indo, num sentido ... mas, esse sentido tem limite, é o ponto de mudança --- nesse ponto de mudança os sentidos se confundem e no limite do novo sentido nasce a reação. Ela prospera, progride, avança, percorrendo outro sentido ... e agora ela não vê o limite desse novo sentido. Novamente o infinito é o transcendente. Parabéns, poucos conseguem assim, de cara, absorverem a idéia básica da terceira sinfonia de Newton.

>"Faz sentido?"

Léo: E como faz!

marcelo ferrari



__________________________________________________
Converse com seus amigos em tempo real com o Yahoo! Messenger
http://br.download.yahoo.com/messenger/

[As partes desta mensagem que não continham texto foram removidas]



##### ##### #####

Para saber mais visite
http://www.ciencialist.hpg.ig.com.br


##### ##### ##### #####
Links do Yahoo! Grupos










SUBJECT: não há MRU no mundo real!
FROM: "rosevena3" <rosevena3@yahoo.com.br>
TO: ciencialist@yahoogrupos.com.br
DATE: 20/12/2004 05:14


AFIRMAÇÃO:
Não existe movimento retilíneo uniforme no mundo real.
JUSTIFICATIVA:
Qualquer "informação" ou objeto que viaje no universo esta
sujeita a
forças que alteram suas velocidades no tempo. (x,y,z)
COMENTARIO:
Seria essa a razão do paradoxo de Zenão de Eléia, não
acontecer no
mundo real.

Estou certo?






SUBJECT: Re: Fw: Temperatura de um raio
FROM: "rosevena3" <rosevena3@yahoo.com.br>
TO: ciencialist@yahoogrupos.com.br
DATE: 20/12/2004 05:29


--- Em ciencialist@yahoogrupos.com.br, "Luiz Ferraz Netto"
<leobarretos@u...> escreveu
> Alguém tem um número para eu dar de exemplo? Ou, uma fonte disso?
> []'
> ===========================
> Luiz Ferraz Netto [Léo]
> leobarretos@u...
> http://www.feiradeciencias.com.br
> ===========================
> -----Mensagem Original-----
> De: Dasdores
> Para: leobarretos@u...
> Enviada em: quinta-feira, 16 de dezembro de 2004 16:58
> Assunto: Temperatura de um raio
>
>
>
> Nas dúvidas experimentais, por gentileza coloque aqui o endereço da
página, isso facilita o confronto. Agradeço. Meu nome é LUIZ FERRAZ
NETTO, meu apelido é LÉO e moro em BARRETOS; dai vem meu e-mail:
leobarretos@u...
>
> Nome: Willianderson Freire Vieira
> Escolaridade: 7ª Série
> E-mail: willy13@w...
>
>
> Olá! Estou fazendo um trabalho e gostaria de saber o seguinte: "
Que temperatura atinge um raio durante sua trajetória ao solo " ?
>
> [As partes desta mensagem que não continham texto foram removidas]

-------------------------------------------------

http://www.fisica.ufc.br/conviteafisica/cien_ens_arquivos/numero7/p09.
pdf

Léu esse é um bom material, porem indo direto a resposta.
No tubo central que tem diâmetro de 10mm a 25mm a temperatura atinge
em média 30000° C.






SUBJECT: Re: [ciencialist] Uma questão de preferência
FROM: José Renato <jrma@terra.com.br>
TO: <ciencialist@yahoogrupos.com.br>
DATE: 20/12/2004 08:16

Caro Luiz, tenho a impressão que pra vc Socialismo, Política e Economia, não
são ciências, não são conhecimentos metódicos. E Aritmética Elementar vc
ainda considerada ciência?
Abraços
José Renato
.............................................

----- Original Message -----
From: "Luiz Ferraz Netto" <leobarretos@uol.com.br>
To: <ciencialist@yahoogrupos.com.br>
Sent: Sunday, December 19, 2004 7:38 PM
Subject: Re: [ciencialist] Um questão de preferência



A minha preferência é que tal assunto seja postado apenas nos sites de
socialismo. Não aqui no C-list.
[]'
===========================
Luiz Ferraz Netto [Léo]
leobarretos@uol.com.br
http://www.feiradeciencias.com.br
===========================
-----Mensagem Original-----
De: "José Renato" <jrma@terra.com.br>
Para: <ciencialist@yahoogrupos.com.br>
Enviada em: sexta-feira, 17 de dezembro de 2004 19:18
Assunto: [ciencialist] Um questão de preferência



Qual a opção que vc escolheria para o reajuste do salário mínimo: 290 já em
janeiro ou 300 em maio de 2005?

Os números dessa comparação, podem ser verificados nas contas a seguir.

1ª Opção: (290 - 260) x (12 + 1) = 30 x 13 = 390 reais a mais em 2005.

2ª Opção: (300 - 260) x (8 + 1) = 40 x 9 = 360 reais a mais em 2005.

Estamos considerarmos que o próximo SM será corrigido em janeiro de 2006.

Entretanto, se o próximo salário mínimo for reajustado somente em maio de
2006, as contas serão diferentes.

1ª Opção: 30 x (13 + 4) = 30 x 17 = 510

2ª Opção: 40 x (9 + 4) = 40 x 13 = 520.

Verificamos que a melhor opção só poderia ser determinada se soubéssemos em
que mês o SM será atualizado em 2006.

Mas a escolha tem de ser definida bem antes disso.
E agora, que opção você prefere?

José Renato M. de Almeida
Salvador - Bahia


[As partes desta mensagem que não continham texto foram removidas]



##### ##### #####

Para saber mais visite
http://www.ciencialist.hpg.ig.com.br


##### ##### ##### #####
Links do Yahoo! Grupos










##### ##### #####

Para saber mais visite
http://www.ciencialist.hpg.ig.com.br


##### ##### ##### #####
Links do Yahoo! Grupos









Esta mensagem foi verificada pelo E-mail Protegido Terra.
Scan engine: McAfee VirusScan / Atualizado em 15/12/2004 / Versão: 4.4.00 -
Dat 4415
Proteja o seu e-mail Terra: http://www.emailprotegido.terra.com.br/

E-mail classificado pelo Identificador de Spam Inteligente Terra.
Para alterar a categoria classificada, visite
http://www.terra.com.br/centralunificada/emailprotegido/imail/imail.cgi?+_u=jrma&_l=1,1103497280.605656.24545.mongu.terra.com.br,4728,Des15,Des15




SUBJECT: Re: Tudo é energia
FROM: Hélio Ricardo Carvalho <hrc@fis.puc-rio.br>
TO: ciencialist@yahoogrupos.com.br
DATE: 20/12/2004 11:53


Oi Brudna,

--- Em ciencialist@yahoogrupos.com.br, "brudna" <lrb@i...> escreveu
>
> Fico confuso com o argumento ´Tudo é energia´.
>
> Onde estão os erros? Que conceitos falhos pode ter esse
argumento?
> Como aproveitar essa oportunidade para ensinar?

Acho que o caminho para saber onde está o erro e como aproveitar
para ensinar está na história da ciência.

Como surgiu o conceito de energia (antes ou depois do conceito de
matéria).
Energia cinética é função da velocidade da matéria (em relação a
que...). Se lanço algo para cima sua velocidade escalar vai
diminuindo e com ela a sua energia cinética.
É transformada em energia potencial???
Ou o conceito de energia potencial foi criado posteriormente para
construir artificialmente, talvez, uma conservação de energia
mecânica?

Hoje é moda falar que matéria nada mais é que energia concentrada.
Não seria melhor (mais lógico) dizer que energia nada mais é que
matéria em movimento?

TUDO É MATÉRIA !!!
:-)

Hélio






SUBJECT: Re: Ação e Reação - sentido
FROM: Hélio Ricardo Carvalho <hrc@fis.puc-rio.br>
TO: ciencialist@yahoogrupos.com.br
DATE: 20/12/2004 12:50


Gente,

Não adianta tentar ensinar ao Marcelo. Ele parece uma máquina que
faz uma pergunta e só está programada para receber um SIM ou um NÃO
como resposta. Se a máquina recebe um "A PERGUNTA ESTÁ ERRADA" ela
não sabe o que fazer com isto e repete a mesma pergunta.

O que o Léo está tentando fazer na MSG abaixo é "desesperadamente"
reprogramar esta máquina. Acho muito difícil.

Mas para Léo, nada é impossível!
[:-)]

Hélio.


--- Em ciencialist@yahoogrupos.com.br, "Luiz Ferraz Netto"
<leobarretos@u...> escreveu
> Marcelo escreve:
> >"Recebi várias respostas, de vários lugares diferentes,
abstraindo, todas parecem apontar para o mesmo lugar: sentido."<
>
> Léo: Faz sentido. É o melhor que se pode abstrair dessas várias
respostas. O sentido sem dúvida é relativo; para a parede a bola
está chegando e ela vai se preparando para entrar em 'reação' ---
para a bola é a parede que está se aproximando perigosamente e ela
'vê' nisso um prelúdio para encerrar sua ação. Essa transmissão de
informações é inerente ao Universo das coisas. Newton só vislumbrou
esse aspécto do universo como um todo, mas não realmente todo,
quando terminou a ação dele começou a reação de outro, Huighens, por
exemplo.
>
> >"A bolinha está indo assim:
> > o > o > o > o > o > o >
> ao interagir, ou re-agir com a parede fica assim
> < o < o < o < o < o < o <
> Se...
> Ação = > o > o > o > o > o > o >
> e reAção = < o < o < o < o < o < o <
> O que muda é apenas o sentido, concordam? "
>
> Léo: Sim, é isso mesmo, o que muda é apenas o sentido. Mudou o
sentido de tudo.
> Ação é agora algo bem mais profundo, que raia ao infinito da
imaginação e a reação que pode ser pensada filosoficamente como
re-AÇÃO e uma ação oposto ao efeito da reação; é algo que vem do
infinito e vai se tornando finito e no instante zero, onde tudo é
possível ou o nada, a reação se inicia e devolve ao universo os
efeitos da ação.
>
> >"Sendo assim, concluo que uma "determinada" ação temina no limite
do seu sentido e a reAção começa no ponto de mudança de sentido."
>
> Léo: Exatamente, sábias palavras! 'O limite do sentido' ... essa é
a pureza do raciocínio que poucos conseguem atingir e, portanto, não
conseguem decifrar o enígma da ação e reação de Newton. Realmente é
profundo .... a bola vai indo, vai indo, num sentido ... mas, esse
sentido tem limite, é o ponto de mudança --- nesse ponto de mudança
os sentidos se confundem e no limite do novo sentido nasce a reação.
Ela prospera, progride, avança, percorrendo outro sentido ... e
agora ela não vê o limite desse novo sentido. Novamente o infinito é
o transcendente. Parabéns, poucos conseguem assim, de cara,
absorverem a idéia básica da terceira sinfonia de Newton.
>
> >"Faz sentido?"
>
> Léo: E como faz!
>
> marcelo ferrari
>
>
>
> __________________________________________________
> Converse com seus amigos em tempo real com o Yahoo! Messenger
> http://br.download.yahoo.com/messenger/
>
> [As partes desta mensagem que não continham texto foram removidas]
>
>
>
> ##### ##### #####
>
> Para saber mais visite
> http://www.ciencialist.hpg.ig.com.br
>
>
> ##### ##### ##### #####
> Links do Yahoo! Grupos





SUBJECT: Re: Ação e Reação - sistema fechado
FROM: marcelo ferrari <emailferrari@yahoo.com.br>
TO: ciencialist@yahoogrupos.com.br
DATE: 20/12/2004 13:27

FERRARI > Sérgio, e sobre voltar ao ponto de origem, tem alguma lei cientifica sobre isto.

TABORDA > Tem, é aquela. Vc tem que a usar para concluir se sai ou não sai. É um trabalho que ha que fazer, tem que fazer as continhas... ou vc acha que fisica é só conversa ?

Sergio, vc conhece alguma equação que preveja quantas vezes a o feixe vai bater até voltar o ponto de origem?

grato.
marcelo ferrari


---------------------------------
Yahoo! Acesso Grátis - Internet rápida e grátis. Instale o discador do Yahoo! agora.

[As partes desta mensagem que não continham texto foram removidas]



SUBJECT: Re: [ciencialist] Sobre dimensões.
FROM: "Amauri Jr" <amaurijunior2@yahoo.com.br>
TO: <ciencialist@yahoogrupos.com.br>
DATE: 20/12/2004 14:42

oi Leandro

Tem muito físico bom aqui, até um quântico que o caso do JC, que pode te responder literalmente. Estou a estudar e vou falar eu entendi.


O tempo seria algo relacionado como um expresso em movimento, enquanto eu estou aqui a escrever esse e-mail, o "trem" do tempo vai passando e determinando nossas ações. Seria como se tudo tivesse determinado dentro de nosso universo, como se nada pode ser de nossa escolha, esta é a teoria do tempo segundo a Teoria da Relatividade de Einstein.

As dimensões, segundo teoria quântica, são 10 aproximadamente; o que podemos pôr na teoria do tempo quantifico, cada um de nós temos uma cópia em outras dimensões. Você esta numa festa e conhece uma moça bonita, tem varias hipóteses de acontecer de ela ser sua amiga, ela ser sua namorada ou até mesmo te achar sem graça. Cada uma delas seria uma cópia nossa de outra dimensão que entramos a cada decisão nossa, uma decisão provável.

Abraços
Amauri
----- Original Message -----
From: Leandro Aparecido Côco
To: ciencialist@yahoogrupos.com.br
Sent: Sunday, December 19, 2004 8:29 PM
Subject: [ciencialist] Sobre dimensões.



Não consigo perceber as outras dimensões além das 4.
Na verdade não consigo ver o tempo como uma dimensão
pelo que entendi as branas criam particulas através da interação com
o espaço-tempo. É isso?

Leandro






##### ##### #####

Para saber mais visite
http://www.ciencialist.hpg.ig.com.br


##### ##### ##### #####


Yahoo! Grupos, um serviço oferecido por:
PUBLICIDADE




------------------------------------------------------------------------------
Links do Yahoo! Grupos

a.. Para visitar o site do seu grupo na web, acesse:
http://br.groups.yahoo.com/group/ciencialist/

b.. Para sair deste grupo, envie um e-mail para:
ciencialist-unsubscribe@yahoogrupos.com.br

c.. O uso que você faz do Yahoo! Grupos está sujeito aos Termos do Serviço do Yahoo!.



[As partes desta mensagem que não continham texto foram removidas]



SUBJECT: De JVictorUma discussão para abalar o universo - Destaque
FROM: "JVictor" <jvoneto@uol.com.br>
TO: <ciencialist@yahoogrupos.com.br>
DATE: 20/12/2004 17:46

AN - Uma discussão para abalar o universo - Destaque
Joinville - Quinta-feira, 18 de novembro de 2004 - Santa Catarina - Brasil







ANotícia

D
E
S
T
A
Q
U
E










A Notícia
Capa
Opinião
Esporte
Economia
Política
Destaque
Mundo
Polícia
Geral
Fórmula 1
Colunas
Alça de Mira
Moacir Pereira
Informal
Livre Mercado
Canal Aberto
Raul Sartori
João Carlos Vieira
Edenilson Leandro
Moacir Benvenutti
Cadernos
Anexo
AN Informática
AN Economia
AN Veículos
AN Tevê
AN Turismo
AN Planalto Norte
Circulando
Especiais
AN Verde
Eleições 2004
Eleições 2002
Grandes Entrevistas
Cruz e Sousa
Joinville 152 anos
Festival de Dança
Recicle
Meio Ambiente
Ecologia
Anita Garibaldi
AN 80 anos
Serviços
Indicadores
Quadrinhos
AN Agora
AN Pergunta
AN Pesquisa
Como anunciar
Classificados
Assinatura
Mensagem
Loterias
Cinema
Edições 2004
Edições 2003
Edições 2002
Edições 2001
Edições 2000
Edições 1999
Edições 1998
Edições 1997
Info
Índice
Expediente
Institucional
AN Cidade
Capa
Comunidade
Polícia
Geral
Esporte
Calendário
AN Capital
Capa
Opinião
Geral
Última Página
Ricardinho Machado
Calendário
AN Jaraguá
Capa
Opinião
Política
Geral
Esporte
Calendário


Em Destaque - Big bang


Uma discussão
para abalar o universo


Cosmólogo Mário Novello fala dos estudos que tentam explicar o surgimento da vida


Rubens Herbst

Joinville - Não que seja uma unanimidade, mas não há dúvida de que a teoria mais conhecida a respeito do surgimento do universo é o chamado Big Bang. Você sabe, aquela grandiosa explosão, ocorrida bilhões de ano atrás, que teria dado origem a tudo o que existe. É uma idéia intensamente difundida da década de 70 até a metade da de 90, inclusive pelos meios de comunicação, fazendo com que ela figurasse, pelo menos entre a população em geral, como a melhor explicação científica para o início do universo. Mas isso pode mudar.
Nos últimos dez anos, vêm se discutindo os mecanismos físicos que teriam originado o Big Bang. O termo, aliás, surgiu como uma piada, criada pelo físico russo George Gamow por volta de 1949. Mas as pessoas que defendiam a idéia da explosão adotaram a nomenclatura e lhe retiraram a conotação pejorativa. "O Big Bang nada mais é do que o momento de extrema condensação do universo, e não o seu início", afirma Mário Novello, um dos nomes mais importantes da cosmologia brasileira.
Pesquisador do Centro Brasileiro de Pesquisas Físicas (CBPF), doutor em física pela Universidade de Genebra, na Suíça, com pós-doutorado no Departamento de Astrofísica da Universidade de Oxford, na Inglaterra, Novello explica que ao invés de uma explosão, o universo passou por um processo de expansão, tanto é que os cientistas não têm dúvidas de que ele foi muito menor e mais quente no passado.
Segundo o pesquisador, nós vivemos nessa atual fase de expansão, que pode parar e iniciar um processo de colapso ou se expandir indefinidamente. "Isso vai depender das propriedades da matéria existentes no nosso universo", diz Novello, ressaltando que os cientistas estão estudando o que delineou essa expansão. "Não sabemos a origem do universo, isso está muito longe de ser descoberto", garante.
Ainda assim, todas as civilizações criaram os seus mitos cosmogônicos (teorias sobre a criação do mundo). A Bíblia dos cristãos e judeus, por exemplo, conta com o Gênesis para explicar a origem de tudo. Novello nota que, pela primeira vez, a ciência está discutindo um modo de construir uma teoria a esse respeito, de forma racional e utilizando todo o conhecimento científico disponibilizado pela física e a astrofísica. Isso não coloca religão e ciência em rota de colisão, garante. "Ciência e religião podem coexistir no momento em que cada uma deixar de ser arrogante e aceitar o saber diferente que a outra traz", afirma. Na opinião de Novello, a ciência não tem explicação racional para tudo. Já a religião deve aceitar que a criação do universo pode ser descrita racionalmente pelos cosmólogos, mesmo que, aparentemente, haja contradição entre a verdade deles e os mitos cosmogônicos deste ou daquele credo.



--------------------------------------------------------------------



Palestras para
professores e alunos da Udesc

Além de sua formação internacional e de coordenar o grupo de Cosmologia e Gravitação do CBPF, que fundou em 1972, Mário Novello acumula em seu currículo mais de cem artigos científicos em publicações internacionais e a autoria de dois livros, "Cosmos et Contexte" (Editora Masson, 1987) e "O Círculo do Tempo: um Olhar Científico sobre Viagens Não-convencionais no Tempo" (Editora Campus, 1998). Desde 1982, o físico promove a Escola Brasileira de Cosmologia e Gravitação e atualmente coordena o Instituto Brasileiro de Cosmologia e Astrofísica (Icra). Em junho, ele recebeu o título de doutor honoris causa da Universidade Claude Bernard de Lyon, na França. O último físico agraciado com o título foi o russo Andrei Sakharov, prêmio Nobel da Paz em 1975 e o mais famoso dissidente da ex-União Soviética.
Novello esteve esta semana em Joinville para proferir as palestras "Big Bang ou Universo Eterno?" e "Efeitos da Eletrodinâmica não-linear na Cosmologia" aos professores e alunos do Departamento de Física da Udesc. Ele diz que é complicado explicar para o público em geral conceitos que não são do cotidiano e praticamente desconhecidos por quem não é expert. "Isso não é arrogância de cientista, mas uma constatação", ressalta.
Diante dessa dificuldade, Novello lança no final deste mês o livro "Os Jogos da Natureza" (Editora Campus), no qual procura justamente destrinchar alguns desses conceitos para o público em geral. A trama gira em torno da filha de um cosmólogo, cujos sonhos representam todo o conhecimento que os físicos e cosmólogos têm sobre o universo, incluindo buracos negros, criação do universo e viagens no tempo. "Eu trato desses conceitos de uma maneira acessível para o leitor que não tem um acesso maior à ciência", avisa.
Aliás, Novello percebe as pessoas mais interessadas nessas questões, digamos, espaciais. Ele conta que, no ano passado, deu palestras no Rio e em Lyon para uma grande quantidade de pessoas que não eram físicos, e que fizeram perguntas muito interessantes. "Compreender um pouco do universo é compreender muito sobre o papel do homem nele", acredita. (RH)



--------------------------------------------------------------------



Centro criado há 55 anos

O Centro Brasileiro de Pesquisas Físicas (CBPF) foi fundado em 15 de janeiro de 1949, no Rio, por um grupo de cientistas brasileiros e de pessoas interessadas no desenvolvimento científico do País. Em 1971, foi a primeira organização brasileira a atuar na área da pós-graduação em física e também a primeira a receber autorização governamental para a concessão de diplomas de doutor e mestre. A partir de 1976, passou a fazer parte do Conselho Nacional de Desenvolvimento Científico e Tecnológico (CNPq).
Durante a fase "civil", o CBPF destacou-se pela organização do trabalho em física experimental, particularmente pela ativação do Laboratório de Chacaltaya, contribuindo para a reputação da unidade como centro de excelência em nível internacional. Também se destacou, no início dos anos 60, pelo projeto e construção de aceleradores lineares de elétrons. Ainda na área experimental, ganhou renome com a descoberta da formação de radioisótopos na atmosfera e com métodos especiais para o carregamento e revelação de emulsões nucleares. A partir dos anos 70, a entidade iniciou atividades em física da matéria condensada, tendo sido a primeira instituição brasileira a instalar e operar um liqüefator de hélio.
Após a incorporação ao CNPq, o CBPF se destacou na participação em projetos de colaboração internacional. Também lançou a idéia e promoveu as discussões iniciais que levaram à criação do Laboratório Nacional de Luz Síncrotron.
Aninhado sob o guarda-chuva do CBPF, o Instituto Brasileiro de Cosmologia e Astrofísica (Icra) será oficializado no final deste ano. O órgão dará suporte aos cosmólogos e astrofísicos - brasileiros e latino-americanos - para ser um pólo de investigação científica, formação de pesquisadores e difusão da cosmologia. (RH)



--------------------------------------------------------------------



Para entender
melhor o universo


Conceitos, teorias e personagens

Cosmologia
É o ramo da astronomia que estuda a origem, a estrutura em grande escala e a evolução do universo. Os astrônomos constroem modelos de universos imaginários com símbolos matemáticos e comparam as propriedades desses modelos com as do universo conhecido. Dois dos modelos mais conhecidos são o universo do Estado Estacionário, no qual matéria nova é criada continuamente para manter uma densidade constante, e o modelo do Big Bang, no qual o universo se expande a partir de uma única explosão.

Big Bang
Grande explosão inicial que deu origem ao universo e que teria ocorrido entre 10 e 20 bilhões de anos atrás. Esta teoria explica por que as galáxias estão se afastando umas das outras no universo e também prediz a existência da radiação de fundo, detectada posteriormente por Penzias e Wilson em 1964.

Buraco negro
Objeto invisível no espaço exterior, formado quando uma estrela massiva colapsa devido a sua própria gravidade. Um buraco negro possui atração gravitacional tão forte que nem mesmo a luz consegue escapar.

Viagens no tempo
Mesmo garantindo ser difícil especular abertamente sobre as viagens no tempo - "corre-se o risco de provocar um protesto contra o desperdício de dinheiro público em algo tão ridículo", escreveu - o cientista Stephen Hawking afirmou que a base de quase todas as discussões modernas é a teoria da relatividade de Einstein. A possibilidade da deformação do espaço-tempo permitiria ao crononauta (viajante no tempo) encontrar caminhos que interligam diferentes regiões do espaço e do tempo. Esses atalhos são denominados "buracos de minhoca" (tubos de espaço-tempo), que, se existissem, seriam a solução para o limite de velocidade no espaço (uma espaçonave não poderia superar a velocidade da luz). Segundo Hawking, estes atalhos fariam avançar ou retroceder no tempo, sem a necessidade de percorrer grandes distâncias.

Eletrodinâmica
É a parte da física que trata da ação dinâmica das correntes elétricas. A eletrodinâmica quântica é o ramo da física que introduz conceitos da mecânica quântica no campo da eletrodinâmica. Foi tema de trabalho de cientistas, como o dos vencedores do Prêmio Nobel de Física de 1965, o japonês S. Tomonaga e os americanos J. Schwinger e R. Feymar (EUA). Do ponto de vista prático, as fórmulas da eletrodinâmica quântica servem para estudar o comportamento do elétrons e de suas correspondentes antipartículas, os pósitrons.


George Gamow (1904-1968)

Físico russo, naturalizado norte-americano, que em 1948 apresentou a teoria de que o universo poderia ter se originado a partir de um estado de pressão e calor extremos, tendo subsequentemente esfriado. Esta teoria, mais tarde, seria conhecida como a Teoria da Grande Explosão, o conhecido"Big Bang". Gamow também deu importantes contribuições nas áreas da física nuclear e sobre a constituição das estrelas. Juntamente com o físico brasileiro Mário Schömberg, Gamow desenvolveu a teoria do chamado "processo Urca", nome dado em alusão ao cassino da Urca, no Rio. Gamow também escreveu livros de divulgação científica que ajudaram a despertar a curiosidade de muitos atuais pesquisadores em física e astronomia.



--------------------------------------------------------------------



País


Polícia detém
mais de 50 menores


Operação Turismo Seguro inicia na orla da zona Sul

Rio - Um dia após o secretário da Segurança Pública do Rio, Marcelo Itagiba, constatar o óbvio, que polícia precisa fazer mais rondas, abordar suspeitos e ser "mais pró-ativa", cerca de 50 menores foram detidos na orla de Copacabana, na zona Sul, bairro com o maior índice de assalto a turistas. Em Botafogo, agentes da Coordenadoria de Recursos Especiais (Core) da Polícia Civil detiveram outros 16 menores suspeitos de praticar roubos no bairro, e prenderam um acusado de 18 anos, que já tinha ordem de prisão expedida pela Justiça.
No fim da tarde, todos os menores seriam encaminhados para abrigos da Prefeitura - na prática, como não houve flagrante, serão liberados. "Eles entram por uma porta e saem pela outra", disse o delegado adjunto da Delegacia de Proteção à Criança e ao Adolescente (DPCA), Daniel Valença.
O delegado Marcos Reimão, titular da Core, disse que a participação de uma força de elite da secretaria foi necessária devido ao tamanho do grupo. "Dezessete é brabo, é quase arrastão", declarou Reimão. Segundo ele, a ação partiu de denúncias de motoristas que tinham sido assaltados no bairro e a suspeita foi reforçada pelo fato de um dos menores estar com um pastor alemão.
De acordo com o subchefe da Polícia Civil, José Renato Torres, começa amanhã na orla da zona Sul a operação Turismo Seguro, com participação da DPCA e de todas as delegacias da região, entre elas a de Atendimento ao Turista. Os policiais, segundo ele, vão para as ruas com câmeras filmadoras e fotográficas. A DPCA vai tentar conseguir junto à Vara da Infância e da Juventude mandados de busca e apreensão para menores e adolescentes que estiverem cometendo infrações ou em situações suspeitas.
Dois comandantes de batalhões da PM foram exonerados anteontem. O tenente-coronel Rogério Lira da Costa foi substituído no comando do Batalhão de Policiamento em Vias Especiais (BPVE) pelo coronel Roberto de Oliveira Penteado, até então comandante do 21º Batalhão, em São João de Meriti, na Baixada Fluminense. No 27ºBatalhão, em Santa Cruz, saiu o tenente-coronel José Domingos de Sales Filho e entrou o tenente-coronel Elson Haubricks Batista, ex-subcomandante do 19º Batalhão (Copacabana).



--------------------------------------------------------------------



Juiz autoriza
aborto de feto sem cérebro

Goiânia - A Justiça de Goiás autorizou uma mulher grávida de 27 anos a abortar porque o feto está sem cérebro. Foi a quarta vez que o juiz da 1ª Vara Criminal, Jesseir Coelho de Alcântara, autorizou aborto de anencefálico. A decisão foi respaldada em parecer favorável do Ministério Público e laudo médico confirmando a inexistência de massa cefálica no feto e a incapacidade de gestação completa.
O juiz argumentou que a manutenção da gestação poderia causar danos à mãe, além dos riscos de morte do feto poucas horas após o parto ou ainda dentro do útero materno. Alcântara entendeu que, diante das complicações previstas pelo laudo médico, a mãe tem o direito de pedir o aborto. "Não pode a Justiça, na minha limitada visão, deixar de prestigiar a responsável via escolhida pela requerente ao buscar, no Poder Judiciário, a solução para sua pretensão", decidiu o juiz.
A primeira decisão de Alcântara a favor de aborto de feto anencefálico ocorreu em 2001. Naquela ocasião, o magistrado autorizou o aborto de um feto que estava no quinto mês de gestação.
Ele argumentou que três laudos emitidos por médicos diferentes eram unânimes quanto à impossibilidade de sobrevida de feto após o parto. Em julho deste ano, o juiz concedeu direito a aborto a duas mães que enfrentavam o mesmo problema.
Manchetes AN

Das últimas edições de País
17/11 - Rússia volta a comprar carne catarinense
16/11 - Reforma política ressuscita no Congresso
15/11 - BR-101, a obra nunca esteve tão perto
14/11 - A história que ainda não foi contada oficialmente
13/11 - Projeto de lei preocupa produtor de fumo
12/11 - Morte de Arafat ainda é mistério
11/11 - Estado é destaque na região Sul








AN Manchetes

Insira seu e-mail:

e seu nome:











Busca em A Notícia
Para busca exata digite frase entre aspas
10 reg20 reg30 reg50 regreg
Em A NotíciaNa Web
Edições 2004 | 2003 | 2002 | 2001 | 2000 | 1999 | 1998 | 1997








AN Classificados

P u b l i q u e ou C o n s u l t e
VEÍCULOS IMÓVEIS
EMPREGOS INFORMÁTICA
TECNOLOGIA FINANÇAS
MÁQUINAS EDUCAÇÃO
UTILIDADES COMUNICADOS
TURISMO DIVERSOS
Balcão de Negócios
PUBLIQUE ou CONSULTE









Outros Serviços

Locais e de terceiros
TEMPO CÂMBIO
MOEDAS INDICADORES
COMENTÁRIO TELEFONE
Informações Complementares
CLIQUE E CONSULTE













A Notícia: Capa | Opinião | Esporte | Economia | Política | Destaque | Mundo | Polícia | Geral | Fórmula 1 | Fórmula Indy | Colunas: Alça de Mira | Moacir Pereira | Informal | Raul Sartori | Livre Mercado | Canal Aberto | João Carlos Vieira | Edenilson Leandro | Moacir Benvenutti | Cadernos: Anexo | AN Informática | AN Economia | AN Veículos | AN Tevê | AN Turismo | AN Planalto Norte | Circulando | Especiais: AN Verde | Eleições 2004 | Eleições 2002 | Grandes Entrevistas | Cruz e Sousa | Joinville 152 anos | Festival de Dança | Recicle | Meio Ambiente | Ecologia | Anita Garibaldi | AN 80 anos | Serviços: AN Agora | Indicadores | Quadrinhos | AN Pergunta | AN Pesquisa | Como anunciar | Classificados | Assinatura | Mensagem | Loterias | Cinema | Edições 2004 | Edições 2003 | Edições 2002 | Edições 2001 | Edições 2000 | Edições 1999 | Edições 1998 | Edições 1997 | Info: Índice | Expediente | Institucional | AN Cidade: Capa | Comunidade | Polícia | Geral | Esporte | Calendário | AN Capital: Capa | Geral | Opinião | Última Página | Ricardinho Machado | Calendário | AN Jaraguá: Capa | Opinião | Política | Geral | Esporte | Calendário
--------------------------------------------------------------------
Copyright © 2000 A Notícia - Fone: 055-0xx47 431 9000 - Fax: 055-0xx47 431 9100 - Rua Caçador, 112 - CEP 89203-610 - C. Postal: 2 - 89201-972 - Joinville - SC - BRASIL -EXPEDIENTE - Redaçìo:anoticia@an.com.br - Fotografia:foto@an.com.br - Classificados:classificados@an.com.br - Comercial:anuncio@an.com.br - Circulação:assinatura@an.com.br - Web site:internet@an.com.br


Por:Torque Comunicação e Internet


[As partes desta mensagem que não continham texto foram removidas]



SUBJECT: Re: Sobre dimensões.
FROM: Hélio Ricardo Carvalho <hrc@fis.puc-rio.br>
TO: ciencialist@yahoogrupos.com.br
DATE: 20/12/2004 18:54


Leandro,

Antes que Amaury e os quânticos mais esotéricos da lista te deixem
doido, eu vou falar um pouco da minha visão das três dimensões
espaciais e do tempo.

A natureza do espaço é uma e a do tempo é outra.

Matematicamente podemos colocar tudo num quadrivetor.
Podemos fazer muitas coisas com a matemática e são coisas úteis para
facilitar o estudo de certos fenômenos naturais mas não são os
fenômenos em si.
Números imaginários não existem são somente frutos de sua
imaginação.
:-)

Leandro disse:
> pelo que entendi as branas criam particulas através da interação
com
> o espaço-tempo.

Na minha humilde opinião (mensagem nesta lista não é um artigo
científico para publicação em revista internacional, logo posso
expressar minhas próprias opiniões) não existe interação COM o
espaço-tempo e sim interações entre matérias no espaço e no tempo.

Sem qualquer um desses componentes (Matéria, Espaço e Tempo), não há
interação.


Hélio



--- Em ciencialist@yahoogrupos.com.br, "Amauri Jr"
<amaurijunior2@y...> escreveu
> oi Leandro
>
> Tem muito físico bom aqui, até um quântico que o caso do JC, que
pode te responder literalmente. Estou a estudar e vou falar eu
entendi.
>
>
> O tempo seria algo relacionado como um expresso em movimento,
enquanto eu estou aqui a escrever esse e-mail, o "trem" do tempo vai
passando e determinando nossas ações. Seria como se tudo tivesse
determinado dentro de nosso universo, como se nada pode ser de nossa
escolha, esta é a teoria do tempo segundo a Teoria da Relatividade
de Einstein.
>
> As dimensões, segundo teoria quântica, são 10 aproximadamente; o
que podemos pôr na teoria do tempo quantifico, cada um de nós temos
uma cópia em outras dimensões. Você esta numa festa e conhece uma
moça bonita, tem varias hipóteses de acontecer de ela ser sua amiga,
ela ser sua namorada ou até mesmo te achar sem graça. Cada uma delas
seria uma cópia nossa de outra dimensão que entramos a cada decisão
nossa, uma decisão provável.
>
> Abraços
> Amauri






SUBJECT: Re: [ciencialist] Re: Tudo é energia
FROM: "Alberto Mesquita Filho" <albmesq@uol.com.br>
TO: <ciencialist@yahoogrupos.com.br>
DATE: 20/12/2004 19:18

----- Original Message -----
From: "Hélio Ricardo Carvalho"
Sent: Monday, December 20, 2004 10:53 AM
Subject: [ciencialist] Re: Tudo é energia


> TUDO É MATÉRIA !!!

Falou e disse.

[ ]´s
Alberto
http://ecientificocultural.com/indice.htm
Mas indiferentemente a tudo isso, o neutrino tem massa, o elétron não é
uma carga elétrica coulombiana e a Terra se move. E a história se repetirá.



SUBJECT: Re: [ciencialist] Tudo é energia
FROM: José Renato <jrma@terra.com.br>
TO: <ciencialist@yahoogrupos.com.br>
DATE: 21/12/2004 00:04

Inclusive aquilo que chamamos de anti-matéria...
Continuamos atropelados pelos diversos vocábulos e significados incompletos!
Abraços
José Renato
...................................................

From: "Alberto Mesquita Filho" <albmesq@uol.com.br>
To: <ciencialist@yahoogrupos.com.br>
Sent: Monday, December 20, 2004 6:18 PM
Subject: Re: [ciencialist] Re: Tudo é energia


>
> ----- Original Message -----
> From: "Hélio Ricardo Carvalho"
> Sent: Monday, December 20, 2004 10:53 AM
> Subject: [ciencialist] Re: Tudo é energia
>
>
>> TUDO É MATÉRIA !!!
>
> Falou e disse.
>
> [ ]´s
> Alberto
> http://ecientificocultural.com/indice.htm
> Mas indiferentemente a tudo isso, o neutrino tem massa, o elétron não é
> uma carga elétrica coulombiana e a Terra se move. E a história se
> repetirá.



SUBJECT: Re: [ciencialist] Tudo é energia
FROM: "Alberto Mesquita Filho" <albmesq@uol.com.br>
TO: <ciencialist@yahoogrupos.com.br>
DATE: 21/12/2004 02:59

----- Original Message -----
From: "José Renato"
Sent: Monday, December 20, 2004 11:04 PM
Subject: Re: [ciencialist] Tudo é energia

Hélio:
> > > TUDO É MATÉRIA !!!

Alberto:
> > Falou e disse.

José Renato:
> Inclusive aquilo que chamamos de anti-matéria... Continuamos atropelados
> pelos diversos vocábulos e significados incompletos!

Alberto:
Bem lembrado. Eu diria que a "anti-matéria" representa a "materialização"
daquilo que vemos no espelho. Ou seja, "anti-matéria" é tanto "matéria"
quanto aquilo que chamamos matéria. Um férmion (como, por exemplo, o
elétron) tem por imagem no espelho um anti-férmion (no caso assinalado, um
pósitron), enquanto um bóson será sempre idêntico a sua imagem especular,
graças a suas propriedades geométrico-estruturais. Mas essa idéia
incompatibiliza-se com a física "moderna", pelo menos enquanto os físicos
"modernos" continuarem insistindo que não se pode conceber uma estrutura
para partículas elementares como o elétron. É bem verdade que o dia em que
eles fizerem essa concessão, estarão a um passo de retornarem a uma física a
apoiar-se na metodologia clássica ou, em outras palavras, de evoluirem para
uma física "neo-moderna".

[ ]´s
Alberto
http://ecientificocultural.com/indice.htm
Mas indiferentemente a tudo isso, o neutrino tem massa, o elétron não é
uma carga elétrica coulombiana e a Terra se move. E a história se repetirá.



SUBJECT: sabão
FROM: "E m i l i a n o C h e m e l l o" <chemelloe@yahoo.com.br>
TO: <ciencialist@yahoogrupos.com.br>, <quimica-qaw@yahoogrupos.com.br>
DATE: 21/12/2004 07:58

bom-dia pessoal !

alguém responde?

[ ] 's do Emiliano Chemello
emiliano@quimica.net
http://www.quimica.net/emiliano
http://www.ucs.br/ccet/defq/naeq
--------
Prezado Sr. Emiliano,
Estou particulamente interessada em saber se há possibilidade de formação
de sabão em gordura de aves, com a adição de cal virgem(CaO) ou Hidróxido
de Calcio(Ca(OH)2.
Qual a influência na formação do sabão, o teor de acidez desta gordura?

Aguardo seu retorno e agradeço antecipadamente


Adriana Berti Toscan
Nutrição Animal- Aves
Seara Alimentos S.A.
Fone: 48 431 1025
Fax: 48 431 1011
atoscan@seara.com.br




SUBJECT: Novo site inacreditável
FROM: Rodrigo Marques <rodmarq72@yahoo.com.br>
TO: Ceticismo Aberto <ceticismoaberto@yahoogrupos.com.br>, CienciaList <ciencialist@yahoogrupos.com.br>, Sociedade Brasileira de "C�ticos" e Racionalistas <sbcr@yahoogrupos.com.br>, Sociedade da Terra redonda <strbrasil@yahoogrupos.com.br>
DATE: 21/12/2004 08:00

Pessoal vejam só este site:
http://www.vivendodaluz.com/
Que gosto será que a luz tem???


---------------------------------
Yahoo! Acesso Grátis - Internet rápida e grátis. Instale o discador do Yahoo! agora.

[As partes desta mensagem que não continham texto foram removidas]



SUBJECT: Fw: [ciencialist] Sobre dimensões.
FROM: José Renato <jrma@terra.com.br>
TO: <ciencialist@yahoogrupos.com.br>
DATE: 21/12/2004 09:56

Caros Leandro, Amaury e Hélio, depois de aprender que o tempo "passa,
pressiona, alivia, etc." deixamos de perceber, por nós mesmos, o que são os
diversos fenômenos ou conceitos do que chamamos costumeiramente de tempo.
Vejamos:
1. tempo atmosférico ou condições climáticas de uma região;
2. tempo cronométrico ou duração de um movimento medido em comparação ou em
relação a um movimento padrão;
3. tempo biológico ou duração de movimentos das partes, células, órgãos de
um ser vivo, que definem sua longevidade a depender das condições ambientais
a que está imerso (ovo de galinha gera em 21d);
4. tempo psicológico ou percepção mental do "tempo", depende das condições
emocionais a que o ser está solicitado a interagir com o meio-ambiente.

Resumindo com outras palavras, sempre elas a nos confundir ou esclarecer:
1. o tempo atmosférico tem configuração física;
2. o tempo cronológico é um conceito, uma invenção intelectual, não tem
existência fisica, existem apenas os movimentos a serem comparados para
definir uma medida que chamamos de tempo;
3. o tempo biológico é o tempo cronológico traduzindo os movimentos
biológicos;
4. a percepção de duração ou o tempo psicológico é bastante elástico, pode
parecer "passar" muito rápido quando estamos precisando terminar uma tarefa
urgente ou muito lento como as tardes de domingo ou a espera da pessoa
amada...

Abraços

José Renato

Em tempo:
As 10 dimensões (11 com o tempo!) me parecem que não constam da teoria da
física quântica e sim da teoria das cordas ou supercordas, desenvolvida a
partir da década de 60 do séc. XX;

As infinitas probabilidades do existir, exceto uma, são virtuais,
imaginárias ou apenas possíveis de ocorrer, não significa que ocorram ou,
pior ainda, que todas estejam ocorrendo simultaneamente;

Dentro do determinismo físico dos movimentos no universo, temos condições de
interferir conscientemente ou não. Nossos pensamentos, esperança,
disposição, crenças, suportados na vontade e atitudes concretas, podem
configurar uma realidade diferente daquela que fatos diversos tentam se
impor como única realidade possível e permitida pelo que os romanos chamavam
de Destino.
......................................................


From: "Hélio Ricardo Carvalho" <hrc@fis.puc-rio.br>
To: <ciencialist@yahoogrupos.com.br>
Sent: Monday, December 20, 2004 5:54 PM
Subject: [ciencialist] Re: Sobre dimensões.

Leandro, antes que Amaury e os quânticos mais esotéricos da lista te deixem
doido, eu vou falar um pouco da minha visão das três dimensões
espaciais e do tempo.

A natureza do espaço é uma e a do tempo é outra.

Matematicamente podemos colocar tudo num quadrivetor.
Podemos fazer muitas coisas com a matemática e são coisas úteis para
facilitar o estudo de certos fenômenos naturais mas não são os
fenômenos em si.
Números imaginários não existem são somente frutos de sua
imaginação.
:-)
.....................................................
From: "Amauri Jr" <amaurijunior2@yahoo.com.br>
To: <ciencialist@yahoogrupos.com.br>
Sent: Monday, December 20, 2004 1:42 PM
Subject: Re: [ciencialist] Sobre dimensões.

oi Leandro

Tem muito físico bom aqui, até um quântico que o caso do JC, que pode te
responder literalmente. Estou a estudar e vou falar eu entendi.

O tempo seria algo relacionado como um expresso em movimento, enquanto eu
estou aqui a escrever esse e-mail, o "trem" do tempo vai passando e
determinando nossas ações. Seria como se tudo tivesse determinado dentro de
nosso universo, como se nada pode ser de nossa escolha, esta é a teoria do
tempo segundo a Teoria da Relatividade de Einstein.

As dimensões, segundo teoria quântica, são 10 aproximadamente; o que podemos
pôr na teoria do tempo quantifico, cada um de nós temos uma cópia em outras
dimensões. Você esta numa festa e conhece uma moça bonita, tem varias
hipóteses de acontecer de ela ser sua amiga, ela ser sua namorada ou até
mesmo te achar sem graça. Cada uma delas seria uma cópia nossa de outra
dimensão que entramos a cada decisão nossa, uma decisão provável.

Abraços
Amauri
......................................................
Leandro disse:
> pelo que entendi as branas criam particulas através da interação
com
> o espaço-tempo.

Na minha humilde opinião (mensagem nesta lista não é um artigo
científico para publicação em revista internacional, logo posso
expressar minhas próprias opiniões) não existe interação COM o
espaço-tempo e sim interações entre matérias no espaço e no tempo.

Sem qualquer um desses componentes (Matéria, Espaço e Tempo), não há
interação.


Hélio



--- Em ciencialist@yahoogrupos.com.br, "Amauri Jr"
<amaurijunior2@y...> escreveu
> oi Leandro
>
> Tem muito físico bom aqui, até um quântico que o caso do JC, que
pode te responder literalmente. Estou a estudar e vou falar eu
entendi.
>
>
> O tempo seria algo relacionado como um expresso em movimento,
enquanto eu estou aqui a escrever esse e-mail, o "trem" do tempo vai
passando e determinando nossas ações. Seria como se tudo tivesse
determinado dentro de nosso universo, como se nada pode ser de nossa
escolha, esta é a teoria do tempo segundo a Teoria da Relatividade
de Einstein.
>
> As dimensões, segundo teoria quântica, são 10 aproximadamente; o
que podemos pôr na teoria do tempo quantifico, cada um de nós temos
uma cópia em outras dimensões. Você esta numa festa e conhece uma
moça bonita, tem varias hipóteses de acontecer de ela ser sua amiga,
ela ser sua namorada ou até mesmo te achar sem graça. Cada uma delas
seria uma cópia nossa de outra dimensão que entramos a cada decisão
nossa, uma decisão provável.
>
> Abraços
> Amauri



SUBJECT: Re: [ciencialist] sabão
FROM: "Cyberlander" <cybernews@superig.com.br>
TO: <ciencialist@yahoogrupos.com.br>
DATE: 21/12/2004 09:59

consultoria grátis, Emiliano? Ou a Nutrição Animal- Aves Seara Alimentos S.A. vai pagar pela resposta?
[ ]'s
D.C.

----- Original Message -----
From: E m i l i a n o C h e m e l l o
To: ciencialist@yahoogrupos.com.br ; quimica-qaw@yahoogrupos.com.br
Sent: Tuesday, December 21, 2004 6:58 AM
Subject: [ciencialist] sabão



bom-dia pessoal !

alguém responde?

[ ] 's do Emiliano Chemello
emiliano@quimica.net
http://www.quimica.net/emiliano
http://www.ucs.br/ccet/defq/naeq
--------
Prezado Sr. Emiliano,
Estou particulamente interessada em saber se há possibilidade de formação
de sabão em gordura de aves, com a adição de cal virgem(CaO) ou Hidróxido
de Calcio(Ca(OH)2.
Qual a influência na formação do sabão, o teor de acidez desta gordura?

Aguardo seu retorno e agradeço antecipadamente


Adriana Berti Toscan
Nutrição Animal- Aves
Seara Alimentos S.A.
Fone: 48 431 1025
Fax: 48 431 1011
atoscan@seara.com.br




##### ##### #####

Para saber mais visite
http://www.ciencialist.hpg.ig.com.br


##### ##### ##### #####
Links do Yahoo! Grupos










[As partes desta mensagem que não continham texto foram removidas]



SUBJECT: Re: [ciencialist] Inteligência Artificial
FROM: Daniel Camara <danielc@dcc.ufmg.br>
TO: ciencialist@yahoogrupos.com.br
DATE: 21/12/2004 10:28

Oi,

Estes livros sao bons, e realmente muito bons para introducao e para
conhecimento de IA classsica, ou seja IA atraves de regras de
inferência. Agora muitas coisas mais novas voce nao vai encontrar la',
por exemplo, redes neuronais e heuristicas como colonia de formigas e
particle sworming nao sao cobertas com certeza. Infelizmente nao sou a
pessoa mais indicada para te ajudar com as respostas as suas duvidas,
mesmo por que nao sei se a entendi direito :). Mas a forma de buscar
uma solucao esta' intimamente ligada a solucao adotada e normalmente e'
dificil fazer isto de forma generica. Se voce criar um resolvedor
automatico de problemas independente... Bom voce vai ganhar muito
dinheiro, para dizer o minimo :). Isto e' o que muita gente sonha a
muitos anos.

Com relacao a bibliografia existem varios livros de IA distribuidos na
Internet que tem assuntos mais novos.

Alguns que tenho link aqui....
Livros on-line
Teoricos
Machine Learning, Neural and Statistical Classification
(http://www.amsta.leeds.ac.uk/~charles/statlog/)
Information Theory, Inference and Learning Algorithms
(http://wol.ra.phy.cam.ac.uk/mackay/itprnn/p3.html#book)

Reinforcement Learning:An Introduction
(http://www.cs.ualberta.ca/~sutton/book/the-book.html)

Livro mais voltados a implementacao
http://www.markwatson.com/opencontent/javaai_lic.htm

Bibliografia em IA
ftp://ftp.ira.uka.de/pub/bibliography/Ai/index.html
http://citeseer.ist.psu.edu/

Espero que tenha ajudado.

Abracos

Daniel


fadsoftwares2003 wrote:

>
> Olá Pessoal!
>
> Sou novo na lista, estou fazendo um trabalho sobre Métodos de Busca
> em Inteligência Artificial e gostaria de discutir o assunto com
> algumas pessoas, não sei se aqui seria o lugar correto. Mas vamos
> lá...
>
> Minha dúvida era se poderíamos considerar os métodos de busca como
> a "ferramenta" para produzir soluções em IA independente do paradigma
> adotado, Simbólico ou Conexionista. Sei que para os Conexiosnistas
> isso seria uma afirmação difícil de "engolir", mas para os Simbólicos
> algo mais palpável.
>
> Generalizando,a idéia é sempre *buscar* uma solução satisfatória
> dentro de uma variedade de possibilidade de erros, mesmo se tratanto
> de classificar padrões, tratar ruídos, etc. Isso justificaria o ponto
> de vista acima ou estaria errado?
>
> Se alguém concordar, gostaria de saber se indicariam algum trabalho
> publicado que reforce essa idéia.
>
> E por último, no livro do Stuart J. Russell e Peter Norvig,
> Artificial Intelligence, A modern approach, o qual utilizo como base
> do meu trabalho, no capítulo 4, eles classificam o Simulated
> Annealing como um Algoritmo de Melhoria Iterativa, ou seja, um método
> de busca não informada. Correto ou não?
> Seguindo esse raciocínio, todas as metaheurísticas (Algoritmos
> Genéticos, Agentes Formiga, etc) poderiam ser consideradas como
> métodos de busca não informada?
>
> Me falaram que esse livro é desatualizado, assim como o da Elaine
> Rich, Inteligência Artificial. Mas não encontrei nenhum referência
> melhor, sugestões?
>
> []s
>
> Fabrício Alves Diniz
>
>
>
>
>
>
> ##### ##### #####
>
> Para saber mais visite
> http://www.ciencialist.hpg.ig.com.br
>
>
> ##### ##### ##### #####
>
>
> *Yahoo! Grupos, um serviço oferecido por:*
> <http://br.rd.yahoo.com/SIG=12acra6e7/M=264379.5078783.6203979.1588051/D=brclubs/S=2137111528:HM/EXP=1103713564/A=2191897/R=0/SIG=10vqa2grn/*http://br.diversao.yahoo.com/>
>
> <http://br.rd.yahoo.com/SIG=12acra6e7/M=264379.5078783.6203979.1588051/D=brclubs/S=2137111528:HM/EXP=1103713564/A=2191897/R=1/SIG=10vqa2grn/*http://br.diversao.yahoo.com/>
>
>
>
> ------------------------------------------------------------------------
> *Links do Yahoo! Grupos*
>
> * Para visitar o site do seu grupo na web, acesse:
> http://br.groups.yahoo.com/group/ciencialist/
>
> * Para sair deste grupo, envie um e-mail para:
> ciencialist-unsubscribe@yahoogrupos.com.br
> <mailto:ciencialist-unsubscribe@yahoogrupos.com.br?subject=Unsubscribe>
>
> * O uso que você faz do Yahoo! Grupos está sujeito aos Termos do
> Serviço do Yahoo! <http://br.yahoo.com/info/utos.html>.
>
>


SUBJECT: Re: [ciencialist] sabão
FROM: "E m i l i a n o C h e m e l l o" <chemelloe@yahoo.com.br>
TO: <ciencialist@yahoogrupos.com.br>
DATE: 21/12/2004 10:32

Cyber,

Pelo amor a ciência :-)

[ ] 's do Emiliano Chemello
emiliano@quimica.net
http://www.quimica.net/emiliano
http://www.ucs.br/ccet/defq/naeq

" Rien ne se perd, rien ne se crée,
tout se transforme."

Antoine Laurent de Lavoisier (químico francês, 1743 - 1794)

----- Original Message -----
From: Cyberlander
To: ciencialist@yahoogrupos.com.br
Sent: Tuesday, December 21, 2004 9:59 AM
Subject: Re: [ciencialist] sabão


consultoria grátis, Emiliano? Ou a Nutrição Animal- Aves Seara Alimentos S.A. vai pagar pela resposta?
[ ]'s
D.C.

----- Original Message -----
From: E m i l i a n o C h e m e l l o
To: ciencialist@yahoogrupos.com.br ; quimica-qaw@yahoogrupos.com.br
Sent: Tuesday, December 21, 2004 6:58 AM
Subject: [ciencialist] sabão



bom-dia pessoal !

alguém responde?

[ ] 's do Emiliano Chemello
emiliano@quimica.net
http://www.quimica.net/emiliano
http://www.ucs.br/ccet/defq/naeq
--------
Prezado Sr. Emiliano,
Estou particulamente interessada em saber se há possibilidade de formação
de sabão em gordura de aves, com a adição de cal virgem(CaO) ou Hidróxido
de Calcio(Ca(OH)2.
Qual a influência na formação do sabão, o teor de acidez desta gordura?

Aguardo seu retorno e agradeço antecipadamente


Adriana Berti Toscan
Nutrição Animal- Aves
Seara Alimentos S.A.
Fone: 48 431 1025
Fax: 48 431 1011
atoscan@seara.com.br




##### ##### #####

Para saber mais visite
http://www.ciencialist.hpg.ig.com.br


##### ##### ##### #####
Links do Yahoo! Grupos










[As partes desta mensagem que não continham texto foram removidas]



##### ##### #####

Para saber mais visite
http://www.ciencialist.hpg.ig.com.br


##### ##### ##### #####


Yahoo! Grupos, um serviço oferecido por:
PUBLICIDADE




------------------------------------------------------------------------------
Links do Yahoo! Grupos

a.. Para visitar o site do seu grupo na web, acesse:
http://br.groups.yahoo.com/group/ciencialist/

b.. Para sair deste grupo, envie um e-mail para:
ciencialist-unsubscribe@yahoogrupos.com.br

c.. O uso que você faz do Yahoo! Grupos está sujeito aos Termos do Serviço do Yahoo!.



[As partes desta mensagem que não continham texto foram removidas]



SUBJECT: Re: Ação e Reação - sistema fechado
FROM: Maria Natália <grasdic@hotmail.com>
TO: ciencialist@yahoogrupos.com.br
DATE: 21/12/2004 11:19


Marcelo:

Tem e deverás pensar em ir a um livro de probabilidades. Trata-se de
uma questão de acaso ou aleatoriedade.
E estou a resonder-te pois colocaste a questão nauma lista e que até
nesta, por ser local público, se presta a leitura de todos. Se
quisesses resposta individual deverias enviá-la em pvt ao colega a
quem te dirigias.
Uma das chatas da sala
Maria Natália

--- Em ciencialist@yahoogrupos.com.br, marcelo ferrari
<emailferrari@y...> escreveu
> FERRARI > Sérgio, e sobre voltar ao ponto de origem, tem alguma
lei cientifica sobre isto.
>
> TABORDA > Tem, é aquela. Vc tem que a usar para concluir se sai ou
não sai. É um trabalho que ha que fazer, tem que fazer as
continhas... ou vc acha que fisica é só conversa ?
>
> Sergio, vc conhece alguma equação que preveja quantas vezes a o
feixe vai bater até voltar o ponto de origem?
>
> grato.
> marcelo ferrari
>
>
> ---------------------------------
> Yahoo! Acesso Grátis - Internet rápida e grátis. Instale o
discador do Yahoo! agora.
>
> [As partes desta mensagem que não continham texto foram removidas]





SUBJECT: Re: De JVictorUma discussão para abalar o universo - Destaque
FROM: Maria Natália <grasdic@hotmail.com>
TO: ciencialist@yahoogrupos.com.br
DATE: 21/12/2004 11:38


JVictor:
Apenas isto,

"Compreender um pouco do universo é compreender muito sobre o papel
do homem nele",
Esta é uma das razões pelas quais aqui em Portugal a reforma
curricular do nosso ensino tem como referência a Astronomia.
Significa que em TODAS as discplinas a raiz condutora e elo de
ligação é a Posição do Homem no Universo. Vale a pena terem aí um
currículo universal de conteúdos comuns aos varios estados
federados. Sabemos ambos que o que norteia muitas vezes os
curriculos nada têm a ver com a formação da pessoa enquanto cidadão
do Mundo mas para corresponder aos requesitos para um exame que se
chama vestibular e à competição entre universidades. Deveríamos ler
colaboração entre pares e se faria como essa universidade nas suas
sessões de discussão de temas na comunidade.
Obrigada
um abraço
Maria Natália

--- Em ciencialist@yahoogrupos.com.br, "JVictor" <jvoneto@u...>
escreveu
> AN - Uma discussão para abalar o universo - Destaque
> Joinville - Quinta-feira, 18 de novembro de
2004 - Santa Catarina - Brasil
>
>
>
>
>
>
>
> ANotícia
>
> D
> E
> S
> T
> A
> Q
> U
> E
>
>
>
>
>
>
>
>
>
>
> A Notícia
> Capa
> Opinião
> Esporte
> Economia
> Política
> Destaque
> Mundo
> Polícia
> Geral
> Fórmula 1
> Colunas
> Alça de Mira
> Moacir Pereira
> Informal
> Livre Mercado
> Canal Aberto
> Raul Sartori
> João Carlos Vieira
> Edenilson Leandro
> Moacir Benvenutti
> Cadernos
> Anexo
> AN Informática
> AN Economia
> AN Veículos
> AN Tevê
> AN Turismo
> AN Planalto Norte
> Circulando
> Especiais
> AN Verde
> Eleições 2004
> Eleições 2002
> Grandes Entrevistas
> Cruz e Sousa
> Joinville 152 anos
> Festival de Dança
> Recicle
> Meio Ambiente
> Ecologia
> Anita Garibaldi
> AN 80 anos
> Serviços
> Indicadores
> Quadrinhos
> AN Agora
> AN Pergunta
> AN Pesquisa
> Como anunciar
> Classificados
> Assinatura
> Mensagem
> Loterias
> Cinema
> Edições 2004
> Edições 2003
> Edições 2002
> Edições 2001
> Edições 2000
> Edições 1999
> Edições 1998
> Edições 1997
> Info
> Índice
> Expediente
> Institucional
> AN Cidade
> Capa
> Comunidade
> Polícia
> Geral
> Esporte
> Calendário
> AN Capital
> Capa
> Opinião
> Geral
> Última Página
> Ricardinho Machado
> Calendário
> AN Jaraguá
> Capa
> Opinião
> Política
> Geral
> Esporte
> Calendário
>
>
> Em Destaque - Big bang
>
>
> Uma discussão
> para abalar o universo
>
>
> Cosmólogo Mário Novello fala dos estudos que tentam
explicar o surgimento da vida
>
>
> Rubens Herbst
>
> Joinville - Não que seja uma unanimidade, mas não há
dúvida de que a teoria mais conhecida a respeito do surgimento do
universo é o chamado Big Bang. Você sabe, aquela grandiosa explosão,
ocorrida bilhões de ano atrás, que teria dado origem a tudo o que
existe. É uma idéia intensamente difundida da década de 70 até a
metade da de 90, inclusive pelos meios de comunicação, fazendo com
que ela figurasse, pelo menos entre a população em geral, como a
melhor explicação científica para o início do universo. Mas isso
pode mudar.
> Nos últimos dez anos, vêm se discutindo os mecanismos
físicos que teriam originado o Big Bang. O termo, aliás, surgiu como
uma piada, criada pelo físico russo George Gamow por volta de 1949.
Mas as pessoas que defendiam a idéia da explosão adotaram a
nomenclatura e lhe retiraram a conotação pejorativa. "O Big Bang
nada mais é do que o momento de extrema condensação do universo, e
não o seu início", afirma Mário Novello, um dos nomes mais
importantes da cosmologia brasileira.
> Pesquisador do Centro Brasileiro de Pesquisas Físicas
(CBPF), doutor em física pela Universidade de Genebra, na Suíça, com
pós-doutorado no Departamento de Astrofísica da Universidade de
Oxford, na Inglaterra, Novello explica que ao invés de uma explosão,
o universo passou por um processo de expansão, tanto é que os
cientistas não têm dúvidas de que ele foi muito menor e mais quente
no passado.
> Segundo o pesquisador, nós vivemos nessa atual fase de
expansão, que pode parar e iniciar um processo de colapso ou se
expandir indefinidamente. "Isso vai depender das propriedades da
matéria existentes no nosso universo", diz Novello, ressaltando que
os cientistas estão estudando o que delineou essa expansão. "Não
sabemos a origem do universo, isso está muito longe de ser
descoberto", garante.
> Ainda assim, todas as civilizações criaram os seus
mitos cosmogônicos (teorias sobre a criação do mundo). A Bíblia dos
cristãos e judeus, por exemplo, conta com o Gênesis para explicar a
origem de tudo. Novello nota que, pela primeira vez, a ciência está
discutindo um modo de construir uma teoria a esse respeito, de forma
racional e utilizando todo o conhecimento científico disponibilizado
pela física e a astrofísica. Isso não coloca religão e ciência em
rota de colisão, garante. "Ciência e religião podem coexistir no
momento em que cada uma deixar de ser arrogante e aceitar o saber
diferente que a outra traz", afirma. Na opinião de Novello, a
ciência não tem explicação racional para tudo. Já a religião deve
aceitar que a criação do universo pode ser descrita racionalmente
pelos cosmólogos, mesmo que, aparentemente, haja contradição entre a
verdade deles e os mitos cosmogônicos deste ou daquele credo.
>
>
>
> -------------------------------------------------------------------
-
>
>
>
> Palestras para
> professores e alunos da Udesc
>
> Além de sua formação internacional e de coordenar o
grupo de Cosmologia e Gravitação do CBPF, que fundou em 1972, Mário
Novello acumula em seu currículo mais de cem artigos científicos em
publicações internacionais e a autoria de dois livros, "Cosmos et
Contexte" (Editora Masson, 1987) e "O Círculo do Tempo: um Olhar
Científico sobre Viagens Não-convencionais no Tempo" (Editora
Campus, 1998). Desde 1982, o físico promove a Escola Brasileira de
Cosmologia e Gravitação e atualmente coordena o Instituto Brasileiro
de Cosmologia e Astrofísica (Icra). Em junho, ele recebeu o título
de doutor honoris causa da Universidade Claude Bernard de Lyon, na
França. O último físico agraciado com o título foi o russo Andrei
Sakharov, prêmio Nobel da Paz em 1975 e o mais famoso dissidente da
ex-União Soviética.
> Novello esteve esta semana em Joinville para proferir
as palestras "Big Bang ou Universo Eterno?" e "Efeitos da
Eletrodinâmica não-linear na Cosmologia" aos professores e alunos do
Departamento de Física da Udesc. Ele diz que é complicado explicar
para o público em geral conceitos que não são do cotidiano e
praticamente desconhecidos por quem não é expert. "Isso não é
arrogância de cientista, mas uma constatação", ressalta.
> Diante dessa dificuldade, Novello lança no final deste
mês o livro "Os Jogos da Natureza" (Editora Campus), no qual procura
justamente destrinchar alguns desses conceitos para o público em
geral. A trama gira em torno da filha de um cosmólogo, cujos sonhos
representam todo o conhecimento que os físicos e cosmólogos têm
sobre o universo, incluindo buracos negros, criação do universo e
viagens no tempo. "Eu trato desses conceitos de uma maneira
acessível para o leitor que não tem um acesso maior à ciência",
avisa.
> Aliás, Novello percebe as pessoas mais interessadas
nessas questões, digamos, espaciais. Ele conta que, no ano passado,
deu palestras no Rio e em Lyon para uma grande quantidade de pessoas
que não eram físicos, e que fizeram perguntas muito
interessantes. "Compreender um pouco do universo é compreender muito
sobre o papel do homem nele", acredita. (RH)
>
>
>
> -------------------------------------------------------------------
-
>
>
>
> Centro criado há 55 anos
>
> O Centro Brasileiro de Pesquisas Físicas (CBPF) foi
fundado em 15 de janeiro de 1949, no Rio, por um grupo de cientistas
brasileiros e de pessoas interessadas no desenvolvimento científico
do País. Em 1971, foi a primeira organização brasileira a atuar na
área da pós-graduação em física e também a primeira a receber
autorização governamental para a concessão de diplomas de doutor e
mestre. A partir de 1976, passou a fazer parte do Conselho Nacional
de Desenvolvimento Científico e Tecnológico (CNPq).
> Durante a fase "civil", o CBPF destacou-se pela
organização do trabalho em física experimental, particularmente pela
ativação do Laboratório de Chacaltaya, contribuindo para a reputação
da unidade como centro de excelência em nível internacional. Também
se destacou, no início dos anos 60, pelo projeto e construção de
aceleradores lineares de elétrons. Ainda na área experimental,
ganhou renome com a descoberta da formação de radioisótopos na
atmosfera e com métodos especiais para o carregamento e revelação de
emulsões nucleares. A partir dos anos 70, a entidade iniciou
atividades em física da matéria condensada, tendo sido a primeira
instituição brasileira a instalar e operar um liqüefator de hélio.
> Após a incorporação ao CNPq, o CBPF se destacou na
participação em projetos de colaboração internacional. Também lançou
a idéia e promoveu as discussões iniciais que levaram à criação do
Laboratório Nacional de Luz Síncrotron.
> Aninhado sob o guarda-chuva do CBPF, o Instituto
Brasileiro de Cosmologia e Astrofísica (Icra) será oficializado no
final deste ano. O órgão dará suporte aos cosmólogos e astrofísicos -
brasileiros e latino-americanos - para ser um pólo de investigação
científica, formação de pesquisadores e difusão da cosmologia. (RH)
>
>
>
> -------------------------------------------------------------------
-
>
>
>
> Para entender
> melhor o universo
>
>
> Conceitos, teorias e personagens
>
> Cosmologia
> É o ramo da astronomia que estuda a origem, a
estrutura em grande escala e a evolução do universo. Os astrônomos
constroem modelos de universos imaginários com símbolos matemáticos
e comparam as propriedades desses modelos com as do universo
conhecido. Dois dos modelos mais conhecidos são o universo do Estado
Estacionário, no qual matéria nova é criada continuamente para
manter uma densidade constante, e o modelo do Big Bang, no qual o
universo se expande a partir de uma única explosão.
>
> Big Bang
> Grande explosão inicial que deu origem ao universo e
que teria ocorrido entre 10 e 20 bilhões de anos atrás. Esta teoria
explica por que as galáxias estão se afastando umas das outras no
universo e também prediz a existência da radiação de fundo,
detectada posteriormente por Penzias e Wilson em 1964.
>
> Buraco negro
> Objeto invisível no espaço exterior, formado quando
uma estrela massiva colapsa devido a sua própria gravidade. Um
buraco negro possui atração gravitacional tão forte que nem mesmo a
luz consegue escapar.
>
> Viagens no tempo
> Mesmo garantindo ser difícil especular abertamente
sobre as viagens no tempo - "corre-se o risco de provocar um
protesto contra o desperdício de dinheiro público em algo tão
ridículo", escreveu - o cientista Stephen Hawking afirmou que a base
de quase todas as discussões modernas é a teoria da relatividade de
Einstein. A possibilidade da deformação do espaço-tempo permitiria
ao crononauta (viajante no tempo) encontrar caminhos que interligam
diferentes regiões do espaço e do tempo. Esses atalhos são
denominados "buracos de minhoca" (tubos de espaço-tempo), que, se
existissem, seriam a solução para o limite de velocidade no espaço
(uma espaçonave não poderia superar a velocidade da luz). Segundo
Hawking, estes atalhos fariam avançar ou retroceder no tempo, sem a
necessidade de percorrer grandes distâncias.
>
> Eletrodinâmica
> É a parte da física que trata da ação dinâmica das
correntes elétricas. A eletrodinâmica quântica é o ramo da física
que introduz conceitos da mecânica quântica no campo da
eletrodinâmica. Foi tema de trabalho de cientistas, como o dos
vencedores do Prêmio Nobel de Física de 1965, o japonês S. Tomonaga
e os americanos J. Schwinger e R. Feymar (EUA). Do ponto de vista
prático, as fórmulas da eletrodinâmica quântica servem para estudar
o comportamento do elétrons e de suas correspondentes
antipartículas, os pósitrons.
>
>
> George Gamow (1904-1968)
>
> Físico russo, naturalizado norte-americano, que em
1948 apresentou a teoria de que o universo poderia ter se originado
a partir de um estado de pressão e calor extremos, tendo
subsequentemente esfriado. Esta teoria, mais tarde, seria conhecida
como a Teoria da Grande Explosão, o conhecido"Big Bang". Gamow
também deu importantes contribuições nas áreas da física nuclear e
sobre a constituição das estrelas. Juntamente com o físico
brasileiro Mário Schömberg, Gamow desenvolveu a teoria do
chamado "processo Urca", nome dado em alusão ao cassino da Urca, no
Rio. Gamow também escreveu livros de divulgação científica que
ajudaram a despertar a curiosidade de muitos atuais pesquisadores em
física e astronomia.
>
>
>
> -------------------------------------------------------------------
-
>
>
>
> País
>
>
> Polícia detém
> mais de 50 menores
>
>
> Operação Turismo Seguro inicia na orla da zona Sul
>
> Rio - Um dia após o secretário da Segurança Pública do
Rio, Marcelo Itagiba, constatar o óbvio, que polícia precisa fazer
mais rondas, abordar suspeitos e ser "mais pró-ativa", cerca de 50
menores foram detidos na orla de Copacabana, na zona Sul, bairro com
o maior índice de assalto a turistas. Em Botafogo, agentes da
Coordenadoria de Recursos Especiais (Core) da Polícia Civil
detiveram outros 16 menores suspeitos de praticar roubos no bairro,
e prenderam um acusado de 18 anos, que já tinha ordem de prisão
expedida pela Justiça.
> No fim da tarde, todos os menores seriam encaminhados
para abrigos da Prefeitura - na prática, como não houve flagrante,
serão liberados. "Eles entram por uma porta e saem pela outra",
disse o delegado adjunto da Delegacia de Proteção à Criança e ao
Adolescente (DPCA), Daniel Valença.
> O delegado Marcos Reimão, titular da Core, disse que a
participação de uma força de elite da secretaria foi necessária
devido ao tamanho do grupo. "Dezessete é brabo, é quase arrastão",
declarou Reimão. Segundo ele, a ação partiu de denúncias de
motoristas que tinham sido assaltados no bairro e a suspeita foi
reforçada pelo fato de um dos menores estar com um pastor alemão.
> De acordo com o subchefe da Polícia Civil, José Renato
Torres, começa amanhã na orla da zona Sul a operação Turismo Seguro,
com participação da DPCA e de todas as delegacias da região, entre
elas a de Atendimento ao Turista. Os policiais, segundo ele, vão
para as ruas com câmeras filmadoras e fotográficas. A DPCA vai
tentar conseguir junto à Vara da Infância e da Juventude mandados de
busca e apreensão para menores e adolescentes que estiverem
cometendo infrações ou em situações suspeitas.
> Dois comandantes de batalhões da PM foram exonerados
anteontem. O tenente-coronel Rogério Lira da Costa foi substituído
no comando do Batalhão de Policiamento em Vias Especiais (BPVE) pelo
coronel Roberto de Oliveira Penteado, até então comandante do 21º
Batalhão, em São João de Meriti, na Baixada Fluminense. No
27ºBatalhão, em Santa Cruz, saiu o tenente-coronel José Domingos de
Sales Filho e entrou o tenente-coronel Elson Haubricks Batista, ex-
subcomandante do 19º Batalhão (Copacabana).
>
>
>
> -------------------------------------------------------------------
-
>
>
>
> Juiz autoriza
> aborto de feto sem cérebro
>
> Goiânia - A Justiça de Goiás autorizou uma mulher
grávida de 27 anos a abortar porque o feto está sem cérebro. Foi a
quarta vez que o juiz da 1ª Vara Criminal, Jesseir Coelho de
Alcântara, autorizou aborto de anencefálico. A decisão foi
respaldada em parecer favorável do Ministério Público e laudo médico
confirmando a inexistência de massa cefálica no feto e a
incapacidade de gestação completa.
> O juiz argumentou que a manutenção da gestação poderia
causar danos à mãe, além dos riscos de morte do feto poucas horas
após o parto ou ainda dentro do útero materno. Alcântara entendeu
que, diante das complicações previstas pelo laudo médico, a mãe tem
o direito de pedir o aborto. "Não pode a Justiça, na minha limitada
visão, deixar de prestigiar a responsável via escolhida pela
requerente ao buscar, no Poder Judiciário, a solução para sua
pretensão", decidiu o juiz.
> A primeira decisão de Alcântara a favor de aborto de
feto anencefálico ocorreu em 2001. Naquela ocasião, o magistrado
autorizou o aborto de um feto que estava no quinto mês de gestação.
> Ele argumentou que três laudos emitidos por médicos
diferentes eram unânimes quanto à impossibilidade de sobrevida de
feto após o parto. Em julho deste ano, o juiz concedeu direito a
aborto a duas mães que enfrentavam o mesmo problema.
> Manchetes AN
>
> Das últimas edições de País
> 17/11 - Rússia volta a comprar carne
catarinense
> 16/11 - Reforma política ressuscita no
Congresso
> 15/11 - BR-101, a obra nunca esteve tão
perto
> 14/11 - A história que ainda não foi
contada oficialmente
> 13/11 - Projeto de lei preocupa produtor
de fumo
> 12/11 - Morte de Arafat ainda é mistério
> 11/11 - Estado é destaque na região Sul
>
>
>
>
>
>
>
>
> AN Manchetes
>
> Insira seu e-mail:
>
> e seu nome:
>
>
>
>
>
>
>
>
>
>
>
> Busca em A Notícia
> Para busca exata digite frase entre aspas
> 10 reg20 reg30 reg50 regreg
> Em A NotíciaNa Web
> Edições 2004 | 2003 | 2002 | 2001 | 2000 |
1999 | 1998 | 1997
>
>
>
>
>
>
>
>
> AN Classificados
>
> P u b l i q u e ou C o n s u l t e
> VEÍCULOS IMÓVEIS
> EMPREGOS INFORMÁTICA
> TECNOLOGIA FINANÇAS
> MÁQUINAS EDUCAÇÃO
> UTILIDADES COMUNICADOS
> TURISMO DIVERSOS
> Balcão de Negócios
> PUBLIQUE ou CONSULTE
>
>
>
>
>
>
>
>
>
> Outros Serviços
>
> Locais e de terceiros
> TEMPO CÂMBIO
> MOEDAS INDICADORES
> COMENTÁRIO TELEFONE
> Informações Complementares
> CLIQUE E CONSULTE
>
>
>
>
>
>
>
>
>
>
>
>
>
> A Notícia: Capa | Opinião | Esporte | Economia |
Política | Destaque | Mundo | Polícia | Geral | Fórmula 1 | Fórmula
Indy | Colunas: Alça de Mira | Moacir Pereira | Informal | Raul
Sartori | Livre Mercado | Canal Aberto | João Carlos Vieira |
Edenilson Leandro | Moacir Benvenutti | Cadernos: Anexo | AN
Informática | AN Economia | AN Veículos | AN Tevê | AN Turismo | AN
Planalto Norte | Circulando | Especiais: AN Verde | Eleições 2004 |
Eleições 2002 | Grandes Entrevistas | Cruz e Sousa | Joinville 152
anos | Festival de Dança | Recicle | Meio Ambiente | Ecologia |
Anita Garibaldi | AN 80 anos | Serviços: AN Agora | Indicadores |
Quadrinhos | AN Pergunta | AN Pesquisa | Como anunciar |
Classificados | Assinatura | Mensagem | Loterias | Cinema | Edições
2004 | Edições 2003 | Edições 2002 | Edições 2001 | Edições 2000 |
Edições 1999 | Edições 1998 | Edições 1997 | Info: Índice |
Expediente | Institucional | AN Cidade: Capa | Comunidade | Polícia
| Geral | Esporte | Calendário | AN Capital: Capa | Geral | Opinião
| Última Página | Ricardinho Machado | Calendário | AN Jaraguá: Capa
| Opinião | Política | Geral | Esporte | Calendário
> -------------------------------------------------------------------
-
> Copyright © 2000 A Notícia - Fone: 055-0xx47 431 9000 -
Fax: 055-0xx47 431 9100 - Rua Caçador, 112 - CEP 89203-610 - C.
Postal: 2 - 89201-972 - Joinville - SC - BRASIL -EXPEDIENTE -
Redaçìo:anoticia@a... - Fotografia:foto@a... -
Classificados:classificados@a... - Comercial:anuncio@a... -
Circulação:assinatura@a... - Web site:internet@a...
>
>
> Por:Torque Comunicação e Internet
>
>
> [As partes desta mensagem que não continham texto foram removidas]





SUBJECT: Re: Tudo é energia
FROM: Hélio Ricardo Carvalho <hrc@fis.puc-rio.br>
TO: ciencialist@yahoogrupos.com.br
DATE: 21/12/2004 11:55


José Renato,

Já que você falou no assunto e como estou inspirado ultimamente,
vamos lá.
Se um aglomerado de matéria conhecida simplesmente por MATÉRIA
"encontra" um outro aglomerado de matéria chamada de ANTI-MATÉRIA,
os dois se desmancham e o conteúdo material dos dois é emitido
formando o que chamamos campo ou onda (onda DE matéria).

:-)

Não me peça para provar isto ainda pois ainda é pura especulação.
:-)

Hélio



--- Em ciencialist@yahoogrupos.com.br, José Renato <jrma@t...>
escreveu
> Inclusive aquilo que chamamos de anti-matéria...
> Continuamos atropelados pelos diversos vocábulos e significados
incompletos!
> Abraços
> José Renato
> ...................................................
>
> From: "Alberto Mesquita Filho" <albmesq@u...>
> To: <ciencialist@yahoogrupos.com.br>
> Sent: Monday, December 20, 2004 6:18 PM
> Subject: Re: [ciencialist] Re: Tudo é energia
>
>
> >
> > ----- Original Message -----
> > From: "Hélio Ricardo Carvalho"
> > Sent: Monday, December 20, 2004 10:53 AM
> > Subject: [ciencialist] Re: Tudo é energia
> >
> >
> >> TUDO É MATÉRIA !!!
> >
> > Falou e disse.
> >
> > [ ]´s
> > Alberto
> > http://ecientificocultural.com/indice.htm
> > Mas indiferentemente a tudo isso, o neutrino tem massa, o
elétron não é
> > uma carga elétrica coulombiana e a Terra se move. E a história
se
> > repetirá.





SUBJECT: Re: [ciencialist] sabão
FROM: "Cyberlander" <cybernews@superig.com.br>
TO: <ciencialist@yahoogrupos.com.br>
DATE: 21/12/2004 12:51

saquei! ;-) a Adriana deve ser jeitosinha heheheeh
[ ]'s
D.C.
----- Original Message -----
From: E m i l i a n o C h e m e l l o
To: ciencialist@yahoogrupos.com.br
Sent: Tuesday, December 21, 2004 9:32 AM
Subject: Re: [ciencialist] sabão



Cyber,

Pelo amor a ciência :-)

[ ] 's do Emiliano Chemello
emiliano@quimica.net
http://www.quimica.net/emiliano
http://www.ucs.br/ccet/defq/naeq

" Rien ne se perd, rien ne se crée,
tout se transforme."

Antoine Laurent de Lavoisier (químico francês, 1743 - 1794)

----- Original Message -----
From: Cyberlander
To: ciencialist@yahoogrupos.com.br
Sent: Tuesday, December 21, 2004 9:59 AM
Subject: Re: [ciencialist] sabão


consultoria grátis, Emiliano? Ou a Nutrição Animal- Aves Seara Alimentos S.A. vai pagar pela resposta?
[ ]'s
D.C.

----- Original Message -----
From: E m i l i a n o C h e m e l l o
To: ciencialist@yahoogrupos.com.br ; quimica-qaw@yahoogrupos.com.br
Sent: Tuesday, December 21, 2004 6:58 AM
Subject: [ciencialist] sabão



bom-dia pessoal !

alguém responde?

[ ] 's do Emiliano Chemello
emiliano@quimica.net
http://www.quimica.net/emiliano
http://www.ucs.br/ccet/defq/naeq
--------
Prezado Sr. Emiliano,
Estou particulamente interessada em saber se há possibilidade de formação
de sabão em gordura de aves, com a adição de cal virgem(CaO) ou Hidróxido
de Calcio(Ca(OH)2.
Qual a influência na formação do sabão, o teor de acidez desta gordura?

Aguardo seu retorno e agradeço antecipadamente


Adriana Berti Toscan
Nutrição Animal- Aves
Seara Alimentos S.A.
Fone: 48 431 1025
Fax: 48 431 1011
atoscan@seara.com.br




##### ##### #####

Para saber mais visite
http://www.ciencialist.hpg.ig.com.br


##### ##### ##### #####
Links do Yahoo! Grupos










[As partes desta mensagem que não continham texto foram removidas]



##### ##### #####

Para saber mais visite
http://www.ciencialist.hpg.ig.com.br


##### ##### ##### #####


Yahoo! Grupos, um serviço oferecido por:
PUBLICIDADE




------------------------------------------------------------------------------
Links do Yahoo! Grupos

a.. Para visitar o site do seu grupo na web, acesse:
http://br.groups.yahoo.com/group/ciencialist/

b.. Para sair deste grupo, envie um e-mail para:
ciencialist-unsubscribe@yahoogrupos.com.br

c.. O uso que você faz do Yahoo! Grupos está sujeito aos Termos do Serviço do Yahoo!.



[As partes desta mensagem que não continham texto foram removidas]



##### ##### #####

Para saber mais visite
http://www.ciencialist.hpg.ig.com.br


##### ##### ##### #####
Links do Yahoo! Grupos










[As partes desta mensagem que não continham texto foram removidas]



SUBJECT: Re: [ciencialist] Tudo é energia
FROM: José Renato <jrma@terra.com.br>
TO: <ciencialist@yahoogrupos.com.br>
DATE: 21/12/2004 14:55

Né não, Hélio! Quando o que chamamos de partículas interagem com alguma
anti-partícula combinam-se seus opostos liberando ou emitindo fóton.
Abraços
José Renato
....................................

From: "Hélio Ricardo Carvalho" <hrc@fis.puc-rio.br>
To: <ciencialist@yahoogrupos.com.br>
Sent: Tuesday, December 21, 2004 10:55 AM
Subject: [ciencialist] Re: Tudo é energia




José Renato,

Já que você falou no assunto e como estou inspirado ultimamente,
vamos lá.
Se um aglomerado de matéria conhecida simplesmente por MATÉRIA
"encontra" um outro aglomerado de matéria chamada de ANTI-MATÉRIA,
os dois se desmancham e o conteúdo material dos dois é emitido
formando o que chamamos campo ou onda (onda DE matéria).

:-)

Não me peça para provar isto ainda pois ainda é pura especulação.
:-)

Hélio



--- Em ciencialist@yahoogrupos.com.br, José Renato <jrma@t...>
escreveu
> Inclusive aquilo que chamamos de anti-matéria...
> Continuamos atropelados pelos diversos vocábulos e significados
incompletos!
> Abraços
> José Renato
> ...................................................
>
> From: "Alberto Mesquita Filho" <albmesq@u...>
> To: <ciencialist@yahoogrupos.com.br>
> Sent: Monday, December 20, 2004 6:18 PM
> Subject: Re: [ciencialist] Re: Tudo é energia
>
>
> >
> > ----- Original Message -----
> > From: "Hélio Ricardo Carvalho"
> > Sent: Monday, December 20, 2004 10:53 AM
> > Subject: [ciencialist] Re: Tudo é energia
> >
> >
> >> TUDO É MATÉRIA !!!
> >
> > Falou e disse.
> >
> > [ ]´s
> > Alberto
> > http://ecientificocultural.com/indice.htm
> > Mas indiferentemente a tudo isso, o neutrino tem massa, o
elétron não é
> > uma carga elétrica coulombiana e a Terra se move. E a história
se
> > repetirá.





##### ##### #####

Para saber mais visite
http://www.ciencialist.hpg.ig.com.br


##### ##### ##### #####
Links do Yahoo! Grupos









Esta mensagem foi verificada pelo E-mail Protegido Terra.
Scan engine: McAfee VirusScan / Atualizado em 15/12/2004 / Versão: 4.4.00 -
Dat 4415
Proteja o seu e-mail Terra: http://www.emailprotegido.terra.com.br/

E-mail classificado pelo Identificador de Spam Inteligente Terra.
Para alterar a categoria classificada, visite
http://www.terra.com.br/centralunificada/emailprotegido/imail/imail.cgi?+_u=jrma&_l=1,1103637368.84145.9600.lusaca.terra.com.br,4446,Des15,Des15




SUBJECT: Re: [ciencialist] Tudo é energia
FROM: "E m i l i a n o C h e m e l l o" <chemelloe@yahoo.com.br>
TO: <ciencialist@yahoogrupos.com.br>
DATE: 21/12/2004 15:21

Apenas para complementar a discussão:

Não há conservação de massa nesta caso da interação de uma partícula com sua antipartícula, somente de 'carga'. Por exemplo: digamos que um elétron vá ao encontro de um pósitron (anti-partícula do elétron). Estes se 'aniquilam' produzindo um fóton.

Antes da colisão: (-) + (+) = neutro

Depois da colisão: fóton (sem carga) = neutro


[ ] 's do Emiliano Chemello
emiliano@quimica.net
http://www.quimica.net/emiliano
http://www.ucs.br/ccet/defq/naeq

" Rien ne se perd, rien ne se crée,
tout se transforme."

Antoine Laurent de Lavoisier (químico francês, 1743 - 1794)

----- Original Message -----
From: José Renato
To: ciencialist@yahoogrupos.com.br
Sent: Tuesday, December 21, 2004 2:55 PM
Subject: Re: [ciencialist] Tudo é energia


Né não, Hélio! Quando o que chamamos de partículas interagem com alguma
anti-partícula combinam-se seus opostos liberando ou emitindo fóton.
Abraços
José Renato
....................................

From: "Hélio Ricardo Carvalho" <hrc@fis.puc-rio.br>
To: <ciencialist@yahoogrupos.com.br>
Sent: Tuesday, December 21, 2004 10:55 AM
Subject: [ciencialist] Re: Tudo é energia




José Renato,

Já que você falou no assunto e como estou inspirado ultimamente,
vamos lá.
Se um aglomerado de matéria conhecida simplesmente por MATÉRIA
"encontra" um outro aglomerado de matéria chamada de ANTI-MATÉRIA,
os dois se desmancham e o conteúdo material dos dois é emitido
formando o que chamamos campo ou onda (onda DE matéria).

:-)

Não me peça para provar isto ainda pois ainda é pura especulação.
:-)

Hélio



--- Em ciencialist@yahoogrupos.com.br, José Renato <jrma@t...>
escreveu
> Inclusive aquilo que chamamos de anti-matéria...
> Continuamos atropelados pelos diversos vocábulos e significados
incompletos!
> Abraços
> José Renato
> ...................................................
>
> From: "Alberto Mesquita Filho" <albmesq@u...>
> To: <ciencialist@yahoogrupos.com.br>
> Sent: Monday, December 20, 2004 6:18 PM
> Subject: Re: [ciencialist] Re: Tudo é energia
>
>
> >
> > ----- Original Message -----
> > From: "Hélio Ricardo Carvalho"
> > Sent: Monday, December 20, 2004 10:53 AM
> > Subject: [ciencialist] Re: Tudo é energia
> >
> >
> >> TUDO É MATÉRIA !!!
> >
> > Falou e disse.
> >
> > [ ]´s
> > Alberto
> > http://ecientificocultural.com/indice.htm
> > Mas indiferentemente a tudo isso, o neutrino tem massa, o
elétron não é
> > uma carga elétrica coulombiana e a Terra se move. E a história
se
> > repetirá.





##### ##### #####

Para saber mais visite
http://www.ciencialist.hpg.ig.com.br


##### ##### ##### #####
Links do Yahoo! Grupos









Esta mensagem foi verificada pelo E-mail Protegido Terra.
Scan engine: McAfee VirusScan / Atualizado em 15/12/2004 / Versão: 4.4.00 -
Dat 4415
Proteja o seu e-mail Terra: http://www.emailprotegido.terra.com.br/

E-mail classificado pelo Identificador de Spam Inteligente Terra.
Para alterar a categoria classificada, visite
http://www.terra.com.br/centralunificada/emailprotegido/imail/imail.cgi?+_u=jrma&_l=1,1103637368.84145.9600.lusaca.terra.com.br,4446,Des15,Des15




##### ##### #####

Para saber mais visite
http://www.ciencialist.hpg.ig.com.br


##### ##### ##### #####


Yahoo! Grupos, um serviço oferecido por:







------------------------------------------------------------------------------
Links do Yahoo! Grupos

a.. Para visitar o site do seu grupo na web, acesse:
http://br.groups.yahoo.com/group/ciencialist/

b.. Para sair deste grupo, envie um e-mail para:
ciencialist-unsubscribe@yahoogrupos.com.br

c.. O uso que você faz do Yahoo! Grupos está sujeito aos Termos do Serviço do Yahoo!.



[As partes desta mensagem que não continham texto foram removidas]



SUBJECT: Re: [ciencialist] Tudo é energia
FROM: "Alberto Mesquita Filho" <albmesq@uol.com.br>
TO: <ciencialist@yahoogrupos.com.br>
DATE: 21/12/2004 16:14

Hélio e José Renato:

Que mal pergunte: Deixando de lado o significado dos termos "fóton", "campo"
e "onda", que por sinal são bem pouco conhecidos e/ou definidos, haveria
alguma diferença importante entre os dois pensamentos abaixo?

Hélio: Se um aglomerado de matéria conhecida simplesmente por MATÉRIA
"encontra" um outro aglomerado de matéria chamada de ANTI-MATÉRIA, os dois
se desmancham e o conteúdo material dos dois é emitido formando o que
chamamos campo ou onda (onda DE matéria).

José Renato: Né não, Hélio! Quando o que chamamos de partículas interagem
com alguma anti-partícula combinam-se seus opostos liberando ou emitindo
fóton.

Bem, talvez a diferença esteja no "desmanchar o conteúdo dos dois" e no
"combinar seus opostos liberando...", mas ainda assim acho que seria a mesma
coisa analisada sob um prisma diferente.

[ ]´s
Alberto
http://ecientificocultural.com/indice.htm
Mas indiferentemente a tudo isso, o neutrino tem massa, o elétron não é
uma carga elétrica coulombiana e a Terra se move. E a história se repetirá.



SUBJECT: Moderador Brudna
FROM: "Amauri Jr" <amaurijunior2@yahoo.com.br>
TO: <ciencialist@yahoogrupos.com.br>
DATE: 21/12/2004 16:45

Gostaria de pedir ao moderador que esta mensagem esta totalmente fora dos desígnios da lista....afinal aqui o c-list não é bate papo da Uol.....

Obrigado
Amauri
----- Original Message -----
From: E m i l i a n o C h e m e l l o
To: ciencialist@yahoogrupos.com.br
Sent: Tuesday, December 21, 2004 10:32 AM
Subject: Re: [ciencialist] sabão


Cyber,

Pelo amor a ciência :-)

[ ] 's do Emiliano Chemello
emiliano@quimica.net
http://www.quimica.net/emiliano
http://www.ucs.br/ccet/defq/naeq

" Rien ne se perd, rien ne se crée,
tout se transforme."

Antoine Laurent de Lavoisier (químico francês, 1743 - 1794)

----- Original Message -----
From: Cyberlander
To: ciencialist@yahoogrupos.com.br
Sent: Tuesday, December 21, 2004 9:59 AM
Subject: Re: [ciencialist] sabão


consultoria grátis, Emiliano? Ou a Nutrição Animal- Aves Seara Alimentos S.A. vai pagar pela resposta?
[ ]'s
D.C.

----- Original Message -----
From: E m i l i a n o C h e m e l l o
To: ciencialist@yahoogrupos.com.br ; quimica-qaw@yahoogrupos.com.br
Sent: Tuesday, December 21, 2004 6:58 AM
Subject: [ciencialist] sabão



bom-dia pessoal !

alguém responde?

[ ] 's do Emiliano Chemello
emiliano@quimica.net
http://www.quimica.net/emiliano
http://www.ucs.br/ccet/defq/naeq
--------
Prezado Sr. Emiliano,
Estou particulamente interessada em saber se há possibilidade de formação
de sabão em gordura de aves, com a adição de cal virgem(CaO) ou Hidróxido
de Calcio(Ca(OH)2.
Qual a influência na formação do sabão, o teor de acidez desta gordura?

Aguardo seu retorno e agradeço antecipadamente


Adriana Berti Toscan
Nutrição Animal- Aves
Seara Alimentos S.A.
Fone: 48 431 1025
Fax: 48 431 1011
atoscan@seara.com.br




##### ##### #####

Para saber mais visite
http://www.ciencialist.hpg.ig.com.br


##### ##### ##### #####
Links do Yahoo! Grupos










[As partes desta mensagem que não continham texto foram removidas]



##### ##### #####

Para saber mais visite
http://www.ciencialist.hpg.ig.com.br


##### ##### ##### #####


Yahoo! Grupos, um serviço oferecido por:
PUBLICIDADE




------------------------------------------------------------------------------
Links do Yahoo! Grupos

a.. Para visitar o site do seu grupo na web, acesse:
http://br.groups.yahoo.com/group/ciencialist/

b.. Para sair deste grupo, envie um e-mail para:
ciencialist-unsubscribe@yahoogrupos.com.br

c.. O uso que você faz do Yahoo! Grupos está sujeito aos Termos do Serviço do Yahoo!.



[As partes desta mensagem que não continham texto foram removidas]



##### ##### #####

Para saber mais visite
http://www.ciencialist.hpg.ig.com.br


##### ##### ##### #####


Yahoo! Grupos, um serviço oferecido por:

São Paulo Rio de Janeiro Curitiba Porto Alegre Belo Horizonte Brasília




------------------------------------------------------------------------------
Links do Yahoo! Grupos

a.. Para visitar o site do seu grupo na web, acesse:
http://br.groups.yahoo.com/group/ciencialist/

b.. Para sair deste grupo, envie um e-mail para:
ciencialist-unsubscribe@yahoogrupos.com.br

c.. O uso que você faz do Yahoo! Grupos está sujeito aos Termos do Serviço do Yahoo!.



[As partes desta mensagem que não continham texto foram removidas]



SUBJECT: Re: Tudo é energia
FROM: Hélio Ricardo Carvalho <hrc@fis.puc-rio.br>
TO: ciencialist@yahoogrupos.com.br
DATE: 21/12/2004 17:02


--- Em ciencialist@yahoogrupos.com.br, José Renato <jrma@t...>
escreveu
> Né não, Hélio! Quando o que chamamos de partículas interagem com
alguma
> anti-partícula combinam-se seus opostos liberando ou emitindo
fóton.

Mas é justamente isto que eu disse!!
:-)

Vou falar novamente:

Se um aglomerado de matéria conhecida simplesmente por MATÉRIA
"encontra" um outro aglomerado de matéria chamada de ANTI-MATÉRIA,
os dois se desmancham (combinam-se seus opostos[:-)]) e o conteúdo
material dos dois é emitido
formando o que chamamos campo ou onda TAMBÉM CHAMADO DE FÓTON (2
fótons no mínimo pois há conservação de momento).
:-)

Hélio





SUBJECT: Mais um moderador [administrador]
FROM: "brudna" <lrb@iq.ufrgs.br>
TO: ciencialist@yahoogrupos.com.br
DATE: 21/12/2004 17:39


O Takata deu uma sumida, acho que tá de férias.

Promovi o Alberto Mesquita Filho como moderador. Tem privilégios de
aprovação de associados e mensagens moderadas.

Até
Luís Brudna - administrador da Ciencialist
ps: Alberto, aprove todos os cadastros solicitados. Só reprove se for
alguém reincidente fazendo Spam.







SUBJECT: Outro moderador [administrador]
FROM: "brudna" <lrb@iq.ufrgs.br>
TO: ciencialist@yahoogrupos.com.br
DATE: 21/12/2004 17:47


:-) Mais um foi promovido.

Luiz Ferraz Netto agora também tem privilégios de moderação da
Ciencialist.

Não sei se o Leo tem prática com o sistema. Se não tiver, deixe que
os outros moderadores tomam conta. :-)

Acho que agora temos um número adequado de moderadores. :-)


Ateh
Luis Brudna - administrador da Ciencialist





SUBJECT: Re: Moderador Brudna
FROM: "brudna" <lrb@iq.ufrgs.br>
TO: ciencialist@yahoogrupos.com.br
DATE: 21/12/2004 17:49


Como anunciado. Emiliano está sob moderação temporária. :-)

Até
Luís Brudna - administrador

--- Em ciencialist@yahoogrupos.com.br, "Amauri Jr"
<amaurijunior2@y...> escreveu
> Gostaria de pedir ao moderador que esta mensagem esta totalmente
fora dos desígnios da lista....afinal aqui o c-list não é bate papo da
Uol.....
>
> Obrigado
> Amauri
> ----- Original Message -----
> From: E m i l i a n o C h e m e l l o
> To: ciencialist@yahoogrupos.com.br
> Sent: Tuesday, December 21, 2004 10:32 AM
> Subject: Re: [ciencialist] sabão
>
>
> Cyber,
>
> Pelo amor a ciência :-)
>
> [ ] 's do Emiliano Chemello
> emiliano@q...
> http://www.quimica.net/emiliano
> http://www.ucs.br/ccet/defq/naeq
>
> " Rien ne se perd, rien ne se crée,
> tout se transforme."
>
> Antoine Laurent de Lavoisier (químico francês, 1743 - 1794)
>
> ----- Original Message -----
> From: Cyberlander
> To: ciencialist@yahoogrupos.com.br
> Sent: Tuesday, December 21, 2004 9:59 AM
> Subject: Re: [ciencialist] sabão
>
>
> consultoria grátis, Emiliano? Ou a Nutrição Animal- Aves Seara
Alimentos S.A. vai pagar pela resposta?
> [ ]'s
> D.C.
>
> ----- Original Message -----
> From: E m i l i a n o C h e m e l l o
> To: ciencialist@yahoogrupos.com.br ;
quimica-qaw@yahoogrupos.com.br
> Sent: Tuesday, December 21, 2004 6:58 AM
> Subject: [ciencialist] sabão
>
>
>
> bom-dia pessoal !
>
> alguém responde?
>
> [ ] 's do Emiliano Chemello
> emiliano@q...
> http://www.quimica.net/emiliano
> http://www.ucs.br/ccet/defq/naeq
> --------
> Prezado Sr. Emiliano,
> Estou particulamente interessada em saber se há possibilidade
de formação
> de sabão em gordura de aves, com a adição de cal virgem(CaO)
ou Hidróxido
> de Calcio(Ca(OH)2.
> Qual a influência na formação do sabão, o teor de acidez desta
gordura?
>
> Aguardo seu retorno e agradeço antecipadamente
>
>
> Adriana Berti Toscan
> Nutrição Animal- Aves
> Seara Alimentos S.A.
> Fone: 48 431 1025
> Fax: 48 431 1011
> atoscan@s...
>
>
>
>
> ##### ##### #####
>
> Para saber mais visite
> http://www.ciencialist.hpg.ig.com.br
>
>
> ##### ##### ##### #####
> Links do Yahoo! Grupos
>
>
>
>
>
>
>
>
>
>
> [As partes desta mensagem que não continham texto foram removidas]
>
>
>
> ##### ##### #####
>
> Para saber mais visite
> http://www.ciencialist.hpg.ig.com.br
>
>
> ##### ##### ##### #####
>
>
> Yahoo! Grupos, um serviço oferecido por:
> PUBLICIDADE
>
>
>
>
>
------------------------------------------------------------------------------
> Links do Yahoo! Grupos
>
> a.. Para visitar o site do seu grupo na web, acesse:
> http://br.groups.yahoo.com/group/ciencialist/
>
> b.. Para sair deste grupo, envie um e-mail para:
> ciencialist-unsubscribe@yahoogrupos.com.br
>
> c.. O uso que você faz do Yahoo! Grupos está sujeito aos
Termos do Serviço do Yahoo!.
>
>
>
> [As partes desta mensagem que não continham texto foram removidas]
>
>
>
> ##### ##### #####
>
> Para saber mais visite
> http://www.ciencialist.hpg.ig.com.br
>
>
> ##### ##### ##### #####
>
>
> Yahoo! Grupos, um serviço oferecido por:
>
> São Paulo Rio de Janeiro Curitiba Porto Alegre
Belo Horizonte Brasília
>
>
>
>
>
------------------------------------------------------------------------------
> Links do Yahoo! Grupos
>
> a.. Para visitar o site do seu grupo na web, acesse:
> http://br.groups.yahoo.com/group/ciencialist/
>
> b.. Para sair deste grupo, envie um e-mail para:
> ciencialist-unsubscribe@yahoogrupos.com.br
>
> c.. O uso que você faz do Yahoo! Grupos está sujeito aos Termos
do Serviço do Yahoo!.
>
>
>
> [As partes desta mensagem que não continham texto foram removidas]





SUBJECT: Re: [ciencialist] Tudo é energia
FROM: "Alberto Mesquita Filho" <albmesq@uol.com.br>
TO: <ciencialist@yahoogrupos.com.br>
DATE: 21/12/2004 18:17

----- Original Message -----
From: "E m i l i a n o C h e m e l l o"
Sent: Tuesday, December 21, 2004 2:21 PM
Subject: Re: [ciencialist] Tudo é energia

> Apenas para complementar a discussão:
> Não há conservação de massa nesta caso da interação de uma partícula com
> sua antipartícula [...]

E o que é massa, nessa altura do campeonato? Há quem diga que massa é a
medida da inércia, e há quem diga que a massa de objetos macroscópicos
traduz seu conteúdo de matéria. Alguns mais abusados confundem massa com
matéria e assumem que conservação da massa é equivalente a conservação da
matéria.

Muitos assumem que fóton é matéria, apesar de sua massa em repouso ser nula.
Curiosamente, aqueles que dizem que a massa do fóton em repouso é nula,
apóiam-se num postulado que proibe esse repouso (o postulado da constância
absoluta de "c"). Podemos dizer também que no referencial onde ocorreu a
interação partícula/antipartícula, o fóton tem massa relativística, logo a
massa se conserva. Em meio a tudo isso, ficamos com algumas poucas dúvidas,
tais como:

O que é massa?
O que é inércia?
O que é fóton?
O que é partícula?
O que é antipartícula?
O que é matéria?
O que é campo?
O que é onda?

Ops! Havia esquecido do título desta thread. Acrescento então:

O que é energia?
Energia é movimento?
O que é movimento?
O que é energia potencial?
Existe movimento sem matéria?
A matéria se move, ou matéria é movimento?
Quem sou eu?
Quem é você?
Onde estou, meu Deus?
Deus?!!! Quem é Deus?

Como diria o Léo: :-))))))))))))))))))))))))))))))))))))

[ ]´s
Alberto
http://ecientificocultural.com/indice.htm
Mas indiferentemente a tudo isso, o neutrino tem massa, o elétron não é
uma carga elétrica coulombiana e a Terra se move. E a história se repetirá.



SUBJECT: Re: [ciencialist] Outro moderador [administrador]
FROM: "L.E.R.de Carvalho" <lecarvalho@infolink.com.br>
TO: ciencialist@yahoogrupos.com.br
DATE: 21/12/2004 20:01

Se este negócio fosse democrático, haveria eleição pra Moderador.
E eu votaria na Maria Natália.

Ciência é viver perigosamente.

L.E.



At 20:49 21/12/2004, you wrote:
>----- Original Message -----
>From: "brudna"
>Sent: Tuesday, December 21, 2004 4:47 PM
>Subject: [ciencialist] Outro moderador [administrador]
>
> > Não sei se o Leo tem prática com o sistema. Se não tiver, deixe que
> > os outros moderadores tomam conta. :-)
>
>Pra ser sincero, eu também não tenho prática com o sistema, mas eu acabo me
>acertando. Já estou estudando as páginas relacionadas à moderação.
>
>Grato pela confiança e espero que os net-amigos ciencialisteiros desculpem
>por algum deslize que porventura eu possa vir a cometer em virtude de meu
>noviciado como auxiliar de moderador.
>
>Tudo pela Ciencialist. Já dizia o Poeta: "Tudo vale a pena quando a alma não
>é pequena."
>
>[ ]´s
>Alberto
><http://ecientificocultural.com/indice.htm>http://ecientificocultural.com/indice.htm
>
>
>
>##### ##### #####
>
>Para saber mais visite
><http://www.ciencialist.hpg.ig.com.br>http://www.ciencialist.hpg.ig.com.br
>
>
>##### ##### ##### #####
>
>
>Yahoo! Grupos, um serviço oferecido por:
><http://br.rd.yahoo.com/SIG=12acaeja9/M=264379.5078783.6203979.1588051/D=brclubs/S=2137111528:HM/EXP=1103752224/A=2191897/R=0/SIG=10vqa2grn/*http://br.diversao.yahoo.com/>
>[]
>
><http://br.rd.yahoo.com/SIG=12acaeja9/M=264379.5078783.6203979.1588051/D=brclubs/S=2137111528:HM/EXP=1103752224/A=2191897/R=1/SIG=10vqa2grn/*http://br.diversao.yahoo.com/>
>[]
> São Paulo Rio de Janeiro Curitiba Porto Alegre Belo Horizonte Brasília
>
>
>----------
>Links do Yahoo! Grupos
> * Para visitar o site do seu grupo na web, acesse:
> *
> <http://br.groups.yahoo.com/group/ciencialist/>http://br.groups.yahoo.com/group/ciencialist/
>
> *
> * Para sair deste grupo, envie um e-mail para:
> *
> <mailto:ciencialist-unsubscribe@yahoogrupos.com.br?subject=Unsubscribe>ciencialist-unsubscribe@yahoogrupos.com.br
>
> *
> * O uso que você faz do Yahoo! Grupos está sujeito aos
> <http://br.yahoo.com/info/utos.html>Termos do Serviço do Yahoo!.


[As partes desta mensagem que não continham texto foram removidas]



SUBJECT: Re: [ciencialist] Outro moderador [administrador]
FROM: "Alberto Mesquita Filho" <albmesq@uol.com.br>
TO: <ciencialist@yahoogrupos.com.br>
DATE: 21/12/2004 20:49

----- Original Message -----
From: "brudna"
Sent: Tuesday, December 21, 2004 4:47 PM
Subject: [ciencialist] Outro moderador [administrador]

> Não sei se o Leo tem prática com o sistema. Se não tiver, deixe que
> os outros moderadores tomam conta. :-)

Pra ser sincero, eu também não tenho prática com o sistema, mas eu acabo me
acertando. Já estou estudando as páginas relacionadas à moderação.

Grato pela confiança e espero que os net-amigos ciencialisteiros desculpem
por algum deslize que porventura eu possa vir a cometer em virtude de meu
noviciado como auxiliar de moderador.

Tudo pela Ciencialist. Já dizia o Poeta: "Tudo vale a pena quando a alma não
é pequena."

[ ]´s
Alberto
http://ecientificocultural.com/indice.htm



SUBJECT: Re: Outro moderador [administrador]
FROM: Manuel Bulcão <manuelbulcao@uol.com.br>
TO: ciencialist@yahoogrupos.com.br
DATE: 21/12/2004 21:36


Oi,

Gostaria de parabenizar o Brudna pelo seu 'tino' ao escolher o Léo e
o Mesquita para auxiliá-lo na dura missão de moderador. Com efeito,
esses dois veteranos da Ciencialist, por serem moderados por
temperamento, são moderadores natos!

Já o mesmo não se pode dizer do Takata: sujeito irascível, áspero,
destemperado, concupiscente... ainda bem que está sumido.

Para que fique a contento, é só o Brudna me incluir: eu, o Mesquita
e o Léo daríamos um triunvirato perfeito.

Abraços,
Manuel Bulcão





SUBJECT: Nova pesquisa de opinião para ciencialist
FROM: ciencialist@yahoogrupos.com.br
TO: ciencialist@yahoogrupos.com.br
DATE: 21/12/2004 22:08



Deposite seu voto hoje! Verifique a nova pesquisa de opinião do grupo ciencialist:


O emiliano deve ser moderado?

o sim
o não
o nem sim nem não, muito pelo contrário
o quem deve ser moderado é o Manuel Bulcão


Para votar, visite a seguinte web page:

http://br.groups.yahoo.com/group/ciencialist/polls

Nota: Não responda a esta mensagem. Votos de pesquisas não são coletados por e-mail. Para votar, é preciso ir ao site do Yahoo! Grupos listado acima.

Obrigado!










SUBJECT: Re: [ciencialist] Tudo é energia
FROM: José Renato <jrma@terra.com.br>
TO: <ciencialist@yahoogrupos.com.br>
DATE: 21/12/2004 22:38

Pode ser, Alberto... É a maldição de Babel a nos confundir. :- )
[]s
José Renato
...................................................

From: "Alberto Mesquita Filho" <albmesq@uol.com.br>
To: <ciencialist@yahoogrupos.com.br>
Sent: Tuesday, December 21, 2004 3:14 PM
Subject: Re: [ciencialist] Tudo é energia


> Hélio e José Renato:
>
> Que mal pergunte: Deixando de lado o significado dos termos "fóton",
> "campo"
> e "onda", que por sinal são bem pouco conhecidos e/ou definidos, haveria
> alguma diferença importante entre os dois pensamentos abaixo?
>
> Hélio: Se um aglomerado de matéria conhecida simplesmente por MATÉRIA
> "encontra" um outro aglomerado de matéria chamada de ANTI-MATÉRIA, os dois
> se desmancham e o conteúdo material dos dois é emitido formando o que
> chamamos campo ou onda (onda DE matéria).
>
> José Renato: Né não, Hélio! Quando o que chamamos de partículas interagem
> com alguma anti-partícula combinam-se seus opostos liberando ou emitindo
> fóton.
>
> Bem, talvez a diferença esteja no "desmanchar o conteúdo dos dois" e no
> "combinar seus opostos liberando...", mas ainda assim acho que seria a
> mesma
> coisa analisada sob um prisma diferente.
>
> [ ]´s
> Alberto
> http://ecientificocultural.com/indice.htm
> Mas indiferentemente a tudo isso, o neutrino tem massa, o elétron não é
> uma carga elétrica coulombiana e a Terra se move. E a história se
> repetirá.
>
>
>
> ##### ##### #####
>
> Para saber mais visite
> http://www.ciencialist.hpg.ig.com.br
>
>
> ##### ##### ##### #####
> Links do Yahoo! Grupos
>
>
>
>
>
>
>
>
>
> Esta mensagem foi verificada pelo E-mail Protegido Terra.
> Scan engine: McAfee VirusScan / Atualizado em 15/12/2004 / Versão:
> 4.4.00 - Dat 4415
> Proteja o seu e-mail Terra: http://www.emailprotegido.terra.com.br/
>
> E-mail classificado pelo Identificador de Spam Inteligente Terra.
> Para alterar a categoria classificada, visite
> http://www.terra.com.br/centralunificada/emailprotegido/imail/imail.cgi?+_u=jrma&_l=1,1103649726.283603.23845.lusaca.terra.com.br,3999,Des15,Des15
>



SUBJECT: Re: [ciencialist] Tudo é energia
FROM: José Renato <jrma@terra.com.br>
TO: <ciencialist@yahoogrupos.com.br>
DATE: 21/12/2004 22:44

É verdade, Hélio!
Parece-me que foi o termo "se desmancham" que atropelou meu entendimento.
Valeu!
[]s
José Renato
..............................................
----- Original Message -----
From: "Hélio Ricardo Carvalho" <hrc@fis.puc-rio.br>
To: <ciencialist@yahoogrupos.com.br>
Sent: Tuesday, December 21, 2004 4:02 PM
Subject: [ciencialist] Re: Tudo é energia




--- Em ciencialist@yahoogrupos.com.br, José Renato <jrma@t...>
escreveu
> Né não, Hélio! Quando o que chamamos de partículas interagem com
alguma
> anti-partícula combinam-se seus opostos liberando ou emitindo
fóton.

Mas é justamente isto que eu disse!!
:-)

Vou falar novamente:

Se um aglomerado de matéria conhecida simplesmente por MATÉRIA
"encontra" um outro aglomerado de matéria chamada de ANTI-MATÉRIA,
os dois se desmancham (combinam-se seus opostos[:-)]) e o conteúdo
material dos dois é emitido
formando o que chamamos campo ou onda TAMBÉM CHAMADO DE FÓTON (2
fótons no mínimo pois há conservação de momento).
:-)

Hélio





##### ##### #####

Para saber mais visite
http://www.ciencialist.hpg.ig.com.br


##### ##### ##### #####
Links do Yahoo! Grupos









Esta mensagem foi verificada pelo E-mail Protegido Terra.
Scan engine: McAfee VirusScan / Atualizado em 15/12/2004 / Versão: 4.4.00 -
Dat 4415
Proteja o seu e-mail Terra: http://www.emailprotegido.terra.com.br/

E-mail classificado pelo Identificador de Spam Inteligente Terra.
Para alterar a categoria classificada, visite
http://www.terra.com.br/centralunificada/emailprotegido/imail/imail.cgi?+_u=jrma&_l=1,1103655812.611334.5512.niassa.terra.com.br,3600,Des15,Des15




SUBJECT: Nova pesquisa de opinião para ciencialist
FROM: "L.E.R.de Carvalho" <lecarvalho@infolink.com.br>
TO: ciencialist@yahoogrupos.com.br
DATE: 21/12/2004 23:22


>O emiliano deve ser moderado?



POXA...
SERÁ QUE VOCÊS NÃO ENTENDEM QUE O CARA É QUÍMICO ?

DEIXA ELE, PÔ.

L.E.

[As partes desta mensagem que não continham texto foram removidas]



SUBJECT: Re: Outro moderador [administrador]
FROM: Maria Natália <grasdic@hotmail.com>
TO: ciencialist@yahoogrupos.com.br
DATE: 21/12/2004 23:35


Manuel:


Ex injuria jus non oitur.
Justitia est constans et perpetua voluntas jus suum cuique tribuendi.
Amem
O que quer dizer" A Justiça sem olhar amizades, credo, cor ou raças"
É uma espécie de benção
Abraços
Maria Natália
PS impotentia generandi LOL


--- Em ciencialist@yahoogrupos.com.br, Manuel Bulcão
<manuelbulcao@u...> escreveu
>
> Oi,
>
> Gostaria de parabenizar o Brudna pelo seu 'tino' ao escolher o Léo
e
> o Mesquita para auxiliá-lo na dura missão de moderador. Com
efeito,
> esses dois veteranos da Ciencialist, por serem moderados por
> temperamento, são moderadores natos!
>
> Já o mesmo não se pode dizer do Takata: sujeito irascível, áspero,
> destemperado, concupiscente... ainda bem que está sumido.
>
> Para que fique a contento, é só o Brudna me incluir: eu, o
Mesquita
> e o Léo daríamos um triunvirato perfeito.
>
> Abraços,
> Manuel Bulcão





SUBJECT: Re: Nova pesquisa de opinião para ciencialist
FROM: Manuel Bulcão <manuelbulcao@uol.com.br>
TO: ciencialist@yahoogrupos.com.br
DATE: 21/12/2004 23:49


--- Em ciencialist@yahoogrupos.com.br, "L.E.R.de Carvalho"
<lecarvalho@i...> escreveu
>
> >O emiliano deve ser moderado?
>
>
>
> POXA...
> SERÁ QUE VOCÊS NÃO ENTENDEM QUE O CARA É QUÍMICO ?
>
> DEIXA ELE, PÔ.

Mané: Ocorre que o Emiliano criou uma enquete na lista "Conversa de
Botequim", indagando se a Natália deveria ou não permanecer naquela
lista (lista esta moderada pelo Zé Carlos, ora vejam!). Por conta
dessa enquete, a Natália foi expulsa.

Uma pessoa que é expulsa de uma lista de discussão
intitulada "conversa de botequim" ou é um anjo ou então um demônio
que nem bêbado agüenta, é o que se presume. A Natália tem toda a
razão para ter ficado muito ofendida. Sou-lhe solidário.

Amplexos,
Manuel Bulcão





SUBJECT: Nova pesquisa de opinião para ciencialist
FROM: "L.E.R.de Carvalho" <lecarvalho@infolink.com.br>
TO: ciencialist@yahoogrupos.com.br
DATE: 21/12/2004 23:58


>Mané: Ocorre que o Emiliano criou uma enquete na lista "Conversa de
>Botequim", indagando se a Natália deveria ou não permanecer naquela
>lista (lista esta moderada pelo Zé Carlos, ora vejam!). Por conta
>dessa enquete, a Natália foi expulsa.
>
>Uma pessoa que é expulsa de uma lista de discussão
>intitulada "conversa de botequim" ou é um anjo ou então um demônio
>que nem bêbado agüenta, é o que se presume. A Natália tem toda a
>razão para ter ficado muito ofendida. Sou-lhe solidário.
>
>Amplexos,
>Manuel Bulcão



O CARA É QUÍMICO.
QUE OUTRA EXPLICAÇÃO DEVO BUSCAR PARA TE CONVENCER QUE NÃO ADIANTA MEXER
NISSO ?

A NATÁLIA FOI PREMIADA EM PLENO NATAL.
FAZ SENTIDO.
L.E.

[As partes desta mensagem que não continham texto foram removidas]



SUBJECT: Re: Nova pesquisa de opinião para ciencialist
FROM: Manuel Bulcão <manuelbulcao@uol.com.br>
TO: ciencialist@yahoogrupos.com.br
DATE: 22/12/2004 00:19


--- Em ciencialist@yahoogrupos.com.br, "L.E.R.de Carvalho"
<lecarvalho@i...> escreveu

> O CARA É QUÍMICO.
> QUE OUTRA EXPLICAÇÃO DEVO BUSCAR PARA TE CONVENCER QUE NÃO ADIANTA
MEXER NISSO ?

Manuel: MEU AMIGO, BASTA UM FLATO PARA QUE O BROMETO SE TRANSFORME
EM BROMATO, OU VICE-VERSA!

ADEMAIS, NÃO PRECISA GRITAR! :-(

ABRAÇOS!!!
MANUEL BULCÃO





SUBJECT: Re: Nova pesquisa de opinião para ciencialist
FROM: Maria Natália <grasdic@hotmail.com>
TO: ciencialist@yahoogrupos.com.br
DATE: 22/12/2004 00:45


Manuel:

L.E.R* não sabe nem da missa a metade.Mas nem é preciso ir a essa
lista para se saber.
Creio que os moderadores NUNCA deixarão instalar-se aqui o ambiente
dessa lista que falaste. Seria insultuoso para todos nós.
Sei que apesar de alguns contratempos esta lista é muito capaz de
ensinar a outras o que é a democracia.
Quanto às acções...elas ficam com quem as pratica e nas listas se
nota logo quando mensagens são apagadas. Quem escreveu assume.
Quem entra numa lista deve analisar todos os "postes" indo até à
primeira mensagem para saber onde está e se situar.
Parece OT mas não o é. São normas de segurança de circulação na
internet. Assim como num laboratório se deve ter cuidado a
armazenar reagentes...
Eu sou química. Tenho a infelicidade de ser a única mulher que fala
regularmente.E como ensinar ciência é ser-se revolucionário...
Continuarei agitando as águas e não me assustando com as bombas
caindo aqui e ali porque pratico as normas de segurança.
Conseguem ouvir o que NÃO está sendo dito?
Uma abraço com votos de boa jornada ao triunvirato que ajudará esta
lista a ter e manter a LIBERDADE e respeito com que nasceu.
Um abraço
queridos especialistos
Maria Natália
* Te escrevo em pvt pois isso é OT


--- Em ciencialist@yahoogrupos.com.br, Manuel Bulcão
<manuelbulcao@u...> escreveu
>
> --- Em ciencialist@yahoogrupos.com.br, "L.E.R.de Carvalho"
> <lecarvalho@i...> escreveu
> >
> > >O emiliano deve ser moderado?
> >
> >
> >
> > POXA...
> > SERÁ QUE VOCÊS NÃO ENTENDEM QUE O CARA É QUÍMICO ?
> >
> > DEIXA ELE, PÔ.
>
> Mané: Ocorre que o Emiliano criou uma enquete na lista "Conversa
de
> Botequim", indagando se a Natália deveria ou não permanecer
naquela
> lista (lista esta moderada pelo Zé Carlos, ora vejam!). Por conta
> dessa enquete, a Natália foi expulsa.
>
> Uma pessoa que é expulsa de uma lista de discussão
> intitulada "conversa de botequim" ou é um anjo ou então um demônio
> que nem bêbado agüenta, é o que se presume. A Natália tem toda a
> razão para ter ficado muito ofendida. Sou-lhe solidário.
>
> Amplexos,
> Manuel Bulcão





SUBJECT: Re: Outro moderador [administrador]
FROM: Manuel Bulcão <manuelbulcao@uol.com.br>
TO: ciencialist@yahoogrupos.com.br
DATE: 22/12/2004 01:01


--- Em ciencialist@yahoogrupos.com.br, Maria Natália <grasdic@h...>

> Ex injuria jus non oitur. Justitia est constans et perpetua
voluntas jus suum cuique tribuendi. Amem

Manuel: Data venia, eu estava só brincando. O Mesquita, o Leo e o
Takata são pessoas da melhor índole, até seus inimigos reconhecem
isso!

Ita speratur,
Manuel Bulcão





SUBJECT: Re: Outro moderador [administrador]
FROM: "rmtakata" <rmtakata@altavista.net>
TO: ciencialist@yahoogrupos.com.br
DATE: 22/12/2004 01:07


--- Em ciencialist@yahoogrupos.com.br, Manuel Bulcão
> Já o mesmo não se pode dizer do Takata: sujeito irascível,
> áspero, destemperado, concupiscente... ainda bem que está
> sumido.

Porr#, caral&$, destemperado eh o cac&%$!

============
Seguinte, eu sei q. o pedido eh impossivel de ser atendido, mas tentem
nao trazer questoes pessoais suscitadas alhures para lista
(recomendacao q. eh extensivel no sentido oposto: rusgas aqui geradas,
nao deveriam ser transportadas para outros locais).

Nao hah necessidade de se agir hipocritamente - matar-se em outra
lista e se amar nesta (ou vice-versa).
===========

Feliz Natal pra vc tb, Bulcao. Claro para todos da lista tb.

Nao estou propriamente sumido, mas, sim, estou de ferias. Aproveitando
a onda em q. o governo criou um pacote q. pretende desonerar a
industria livreira no Brasil, tentado aumentar a media de livros lidos
per capita ao ano - menos de 2 - uma sugestao de leitura: A ciencia no
cotidiano de Len Fisher, 2002, ed. Jorge Zahar, 203 pp. - nao
confunfir com A ciencia na vida cotidiana, de Jay Ingram, 2004, ed.
Ediouro, 238 pp. Nao q. este ultimo seja ruim, eh leve e divertido,
mas o humor do australiano Fisher parece-me mais refinado - talvez
contaminado pelo estilo da terra da Rainha, onde trabalha. Ou vai ver
q. Mad Max, Priscilla, Crocodilo Dundee e Babe tEm algo a dizer sobre
a comicidade dos conterraneos dos cangurus. Imagino q. o Luiz Eduardo
deva gostar: hah uma belissima discussao sobre a ciencia de se
mergulhar biscoitos e rosquinhas (ei, em ingles nao hah esse duplo
sentido), trabalho pelo qual, alias, Fisher ganhou um IgNobel; tb como
obter um ovo quente (suspeito q. tb nao exista esse duplo sentido em
ingles) perfeito, no penultimo capitulo o autor analisa a ciencia da
arte do gourmet e do gourmand. (O livro nao eh exclusivamente sobre
culinaria ou sobre a ciencia na culinaria, mas dada as afinidades do
autor, esses temas afloram a quase todo o momento.)

[]s,

Roberto Takata







SUBJECT: Re: Nova pesquisa de opinião para ciencialist
FROM: Maria Natália <grasdic@hotmail.com>
TO: ciencialist@yahoogrupos.com.br
DATE: 22/12/2004 01:36


Querido L.E.R:
Importa-se de me explicar EM PRIVADO, qual o significado de:
"A NATÁLIA FOI PREMIADA EM PLENO NATAL.
FAZ SENTIDO."
Pq era mal empregue lá? Pq não se deitam pérolas a porcos?
É que há coisas que podem ser mal interpretadas e estar a dar
orientações para a enquete como se estivesse aberta campanha
eleitoral para o sim ou para o não. Ou outras coisas piores. Só
direi às pessoas da lista que antes de votarem analisem as
mensagens, AQUI na sala, desde dia 24 de Julho de 2004.
Chamo a atenção que para se estar numa lista em pleno o proceeso
melhor é ter caixa de correio yahoo.com.br Porque assim TEMOS SEMPRE
toas as MENSAGENS presentes.
Não se trata de estar aqui a tratar assuntos de outra lista mas
apenas de se AVALIAR com olhos de ver ESTA lista. E depois como
somos de ciência faremos a análise estatística dos resultados usando
um método que distinga os votos das pessoas que nunca escrevem das
que interveem e ainda da totalidade.
Não seria de alterar e escrever ainda na enquete: Natália banida/
Emiliano banido? Proíbido a portugueses? Ou estilo BB, quem acha
deve ser banido este mês?
obrigada
Maria Natália Botelho



> >intitulada "conversa de botequim" ou é um anjo ou então um demônio
> >que nem bêbado agüenta, é o que se presume. A Natália tem toda a
> >razão para ter ficado muito ofendida. Sou-lhe solidário.
> >
> >Amplexos,
> >Manuel Bulcão
>
>
>
> O CARA É QUÍMICO.
> QUE OUTRA EXPLICAÇÃO DEVO BUSCAR PARA TE CONVENCER QUE NÃO ADIANTA
MEXER
> NISSO ?
>
> A NATÁLIA FOI PREMIADA EM PLENO NATAL.
> FAZ SENTIDO.
> L.E.
>
> [As partes desta mensagem que não continham texto foram removidas]





SUBJECT: Comprem, comprem trás as últimas notícias de Física
FROM: Maria Natália <grasdic@hotmail.com>
TO: ciencialist@yahoogrupos.com.br
DATE: 22/12/2004 02:30



1--Um novo reactor para reacções em plasma não trémico--
http://www.physorg.com/news2464.html

2-- Novidades do Universo--Cassinizinha no dia 25 de Dezembro me vai
dar uma prenda! 7 anos ~ de alguns sustos e precalços mas a nossa
cápsula vai cumpri seu destino. Atenção químicos vamos saber coiss
interessantes sobre a atmosfera de Titã
http://www.physorg.com/news2465.html

3--Câmara espectral para Satélite feito pela Coreia com equipamento
de Israel
http://www.physorg.com/news2457.html

4--Novidades em tecnologia de computadores e para melhora
performance.
http://www.physorg.com/news2462.html

4--Uma laptop em LINUX
http://www.physorg.com/news2459.html

Um abraço
Maria Natália






SUBJECT: A Ciência no Cotidiano - Len Fisher
FROM: "E m i l i a n o C h e m e l l o" <chemelloe@yahoo.com.br>
TO: <ciencialist@yahoogrupos.com.br>
DATE: 22/12/2004 07:50


Caro Takata,

Quanto a referência do livro do Fisher, estou preparando uma resenha sobre ele. Estará no novo site que estarei produzindo a partir do ano que vem. Sem dúvida, os assuntos do livro são de ciência, porém bem mais voltados para a física do que para outra ciência 'hard'. Destaque especial para o capítulo "A física do sexo".

[ ] 's do Emiliano Chemello
emiliano@quimica.net
http://www.quimica.net/emiliano
http://www.ucs.br/ccet/defq/naeq

" Rien ne se perd, rien ne se crée,
tout se transforme."

Antoine Laurent de Lavoisier (químico francês, 1743 - 1794)

----- Original Message -----
From: rmtakata
To: ciencialist@yahoogrupos.com.br
Sent: Wednesday, December 22, 2004 1:07 AM
Subject: [ciencialist] Re: Outro moderador [administrador]



--- Em ciencialist@yahoogrupos.com.br, Manuel Bulcão
> Já o mesmo não se pode dizer do Takata: sujeito irascível,
> áspero, destemperado, concupiscente... ainda bem que está
> sumido.

Porr#, caral&$, destemperado eh o cac&%$!

============
Seguinte, eu sei q. o pedido eh impossivel de ser atendido, mas tentem
nao trazer questoes pessoais suscitadas alhures para lista
(recomendacao q. eh extensivel no sentido oposto: rusgas aqui geradas,
nao deveriam ser transportadas para outros locais).

Nao hah necessidade de se agir hipocritamente - matar-se em outra
lista e se amar nesta (ou vice-versa).
===========

Feliz Natal pra vc tb, Bulcao. Claro para todos da lista tb.

Nao estou propriamente sumido, mas, sim, estou de ferias. Aproveitando
a onda em q. o governo criou um pacote q. pretende desonerar a
industria livreira no Brasil, tentado aumentar a media de livros lidos
per capita ao ano - menos de 2 - uma sugestao de leitura: A ciencia no
cotidiano de Len Fisher, 2002, ed. Jorge Zahar, 203 pp. - nao
confunfir com A ciencia na vida cotidiana, de Jay Ingram, 2004, ed.
Ediouro, 238 pp. Nao q. este ultimo seja ruim, eh leve e divertido,
mas o humor do australiano Fisher parece-me mais refinado - talvez
contaminado pelo estilo da terra da Rainha, onde trabalha. Ou vai ver
q. Mad Max, Priscilla, Crocodilo Dundee e Babe tEm algo a dizer sobre
a comicidade dos conterraneos dos cangurus. Imagino q. o Luiz Eduardo
deva gostar: hah uma belissima discussao sobre a ciencia de se
mergulhar biscoitos e rosquinhas (ei, em ingles nao hah esse duplo
sentido), trabalho pelo qual, alias, Fisher ganhou um IgNobel; tb como
obter um ovo quente (suspeito q. tb nao exista esse duplo sentido em
ingles) perfeito, no penultimo capitulo o autor analisa a ciencia da
arte do gourmet e do gourmand. (O livro nao eh exclusivamente sobre
culinaria ou sobre a ciencia na culinaria, mas dada as afinidades do
autor, esses temas afloram a quase todo o momento.)

[]s,

Roberto Takata


[As partes desta mensagem que não continham texto foram removidas]






SUBJECT: Radiação emitida por celular prejudica DNA
FROM: "E m i l i a n o C h e m e l l o" <chemelloe@yahoo.com.br>
TO: <ciencialist@yahoogrupos.com.br>
DATE: 22/12/2004 08:12


Radiação emitida por celular prejudica DNA, diz estudo

Segunda, 20 de dezembro de 2004, 15h57

::: Resumo :::
Ondas de rádio emitidas por telefones celulares prejudicam as células do
corpo e o DNA em condições de laboratório, afirmou um novo estudo financiado
pela União Européia (UE). O chamado estudo Reflexo, conduzido por 12 grupos
de pesquisa em sete países europeus, não chegou a comprovar que os celulares
são um risco à saúde, mas concluiu que é preciso fazer novas pesquisas para
saber se os efeitos prejudiciais se repetem fora do ambiente de laboratório.
A indústria mundial do celular, que movimenta US$ 100 bilhões ao ano, afirma
que não há nenhuma evidência conclusiva sobre efeitos prejudiciais da
radiação eletromagnética. Só este ano, cerca de 650 milhões de aparelhos
celulares devem ser vendidos aos consumidores. Mais de 1,5 bilhão de pessoas
utilizam celulares no mundo.

O projeto de pesquisa, que levou quatro anos e que foi coordenado pelo
grupo alemão Verum, analisou os efeitos da radiação em células humanas e de
animais em laboratório. Depois de serem expostas a campos eletromagnéticos
típicos de celulares, as células mostraram um aumento significativo nos
rompimentos em uma ou nas duas fitas de DNA. Nem sempre o dano pôde ser
reparado pela célula. O DNA leva o material genético de um organismo e suas
diferentes células.
"Houve um dano permanente para as gerações futuras de células", disse o
líder do projeto, Franz Adlkofer. Células que sofrem mutação são
consideradas uma das possíveis causas do câncer. A radiação utilizada no
estudo estava em níveis de taxa de absorção específica (SAR) entre 0,3 e 2
watts por quilo. A maioria dos celulares emite sinais de rádio em níveis de
SAR entre 0,5 e 1 W/kg.

A SAR é a medida da taxa da absorção da radioenergia pelo tecido do
corpo, e o limite recomendado pela Comissão Internacional de Proteção à
Radiação Não-Ionizante é de 2 W/kg. O estudo também analisou outros efeitos
prejudiciais nas células.
Os pesquisadores afirmaram que o estudo não é comprobatório dos riscos à
saúde por causa das condições de laboratório. Mas acrescentaram que "os
efeitos genotóxicos e fenotípicos claramente exigem estudos adicionais ...
em animais ou em voluntários humanos".

Adlkofer recomendou evitar usar o celular quando há uma alternativa
disponível, além da utilização do fone de ouvido sempre que possível. "Não
queremos criar pânico, mas é bom tomar precauções", disse, acrescentando que
as novas pesquisas podem levar mais quatro ou cinco anos.

Os estudos anteriores sobre os efeitos da radiação dos celulares à saúde
mostraram que ele pode existir. Um exemplos é o aquecimento do tecido, que
provocaria dor de cabeça e náusea. Nenhum estudo comprovou, no entanto, que
a radiação cause danos permanentes. Nenhuma das seis maiores indústrias de
celular respondeu imediatamente à divulgação do estudo.

Em um outro anúncio em Hong Kong, uma empresa alemã chamada G-Hanz
apresentou um novo tipo de celular que, segundo afirma, não emite radiação
prejudicial.

Leia esta notícia no original em:
Terra - Notícias - Ciência e Meio Ambiente
http://noticias.terra.com.br/ciencia/interna/0,,OI442466-EI298,00.html

[ ] 's do Emiliano Chemello






SUBJECT: Nova pesquisa de opinião para ciencialist
FROM: "L.E.R.de Carvalho" <lecarvalho@infolink.com.br>
TO: ciencialist@yahoogrupos.com.br
DATE: 22/12/2004 09:57

Tudo isso é tão engraçado e divertido como rinhas de galo.

Tem quem aposte.

Tem quem seja o galo.

Tem gente que apenas acha curioso, ache inevitável ficar olhando, e mesmo
se divirta.

Eu acho isso tudo muito miseravel.

Numa lista sobre ciencia isso é inaceitável.

A Natalia deve agradecer esse presente Natalino porque é também desse
combustível que ela vive. Não que lhe agrade ao paladar, mas dele, de
alguma forma, acaba de nutrindo, num fenomeno polioperativo, com
transferencia de massa, de energia e de movimento.

Não vai ser a Física que vai explicar esse fenômeno.
Quem sabe o Bulcão tenha respostas.

Fora de tudo isso, uma certeza: quem faz isso com a Natalia é muito
escrotinho.
Alguém tem que reprimir.

L.E.

At 01:36 22/12/2004, you wrote:

>Querido L.E.R:
>Importa-se de me explicar EM PRIVADO, qual o significado de:
>"A NATÁLIA FOI PREMIADA EM PLENO NATAL.
>FAZ SENTIDO."
>Pq era mal empregue lá? Pq não se deitam pérolas a porcos?
>É que há coisas que podem ser mal interpretadas e estar a dar
>orientações para a enquete como se estivesse aberta campanha
>eleitoral para o sim ou para o não. Ou outras coisas piores. Só
>direi às pessoas da lista que antes de votarem analisem as
>mensagens, AQUI na sala, desde dia 24 de Julho de 2004.
>Chamo a atenção que para se estar numa lista em pleno o proceeso
>melhor é ter caixa de correio yahoo.com.br Porque assim TEMOS SEMPRE
>toas as MENSAGENS presentes.
> Não se trata de estar aqui a tratar assuntos de outra lista mas
>apenas de se AVALIAR com olhos de ver ESTA lista. E depois como
>somos de ciência faremos a análise estatística dos resultados usando
>um método que distinga os votos das pessoas que nunca escrevem das
>que interveem e ainda da totalidade.
>Não seria de alterar e escrever ainda na enquete: Natália banida/
>Emiliano banido? Proíbido a portugueses? Ou estilo BB, quem acha
>deve ser banido este mês?
>obrigada
>Maria Natália Botelho


[As partes desta mensagem que não continham texto foram removidas]



SUBJECT: Outro moderador [administrador]
FROM: "L.E.R.de Carvalho" <lecarvalho@infolink.com.br>
TO: ciencialist@yahoogrupos.com.br
DATE: 22/12/2004 10:22

Vou olhar.
Thanks.
L.E.



At 01:07 22/12/2004, you wrote:

>--- Em ciencialist@yahoogrupos.com.br, Manuel Bulcão
> > Já o mesmo não se pode dizer do Takata: sujeito irascível,
> > áspero, destemperado, concupiscente... ainda bem que está
> > sumido.
>
>Porr#, caral&$, destemperado eh o cac&%$!
>
>============
>Seguinte, eu sei q. o pedido eh impossivel de ser atendido, mas tentem
>nao trazer questoes pessoais suscitadas alhures para lista
>(recomendacao q. eh extensivel no sentido oposto: rusgas aqui geradas,
>nao deveriam ser transportadas para outros locais).
>
>Nao hah necessidade de se agir hipocritamente - matar-se em outra
>lista e se amar nesta (ou vice-versa).
>===========
>
>Feliz Natal pra vc tb, Bulcao. Claro para todos da lista tb.
>
>Nao estou propriamente sumido, mas, sim, estou de ferias. Aproveitando
>a onda em q. o governo criou um pacote q. pretende desonerar a
>industria livreira no Brasil, tentado aumentar a media de livros lidos
>per capita ao ano - menos de 2 - uma sugestao de leitura: A ciencia no
>cotidiano de Len Fisher, 2002, ed. Jorge Zahar, 203 pp. - nao
>confunfir com A ciencia na vida cotidiana, de Jay Ingram, 2004, ed.
>Ediouro, 238 pp. Nao q. este ultimo seja ruim, eh leve e divertido,
>mas o humor do australiano Fisher parece-me mais refinado - talvez
>contaminado pelo estilo da terra da Rainha, onde trabalha. Ou vai ver
>q. Mad Max, Priscilla, Crocodilo Dundee e Babe tEm algo a dizer sobre
>a comicidade dos conterraneos dos cangurus. Imagino q. o Luiz Eduardo
>deva gostar: hah uma belissima discussao sobre a ciencia de se
>mergulhar biscoitos e rosquinhas (ei, em ingles nao hah esse duplo
>sentido), trabalho pelo qual, alias, Fisher ganhou um IgNobel; tb como
>obter um ovo quente (suspeito q. tb nao exista esse duplo sentido em
>ingles) perfeito, no penultimo capitulo o autor analisa a ciencia da
>arte do gourmet e do gourmand. (O livro nao eh exclusivamente sobre
>culinaria ou sobre a ciencia na culinaria, mas dada as afinidades do
>autor, esses temas afloram a quase todo o momento.)
>
>[]s,
>
>Roberto Takata
>
>
>
>
>
>
>
>##### ##### #####
>
>Para saber mais visite
><http://www.ciencialist.hpg.ig.com.br>http://www.ciencialist.hpg.ig.com.br
>
>
>##### ##### ##### #####
>
>
>Yahoo! Grupos, um serviço oferecido por:
><http://br.rd.yahoo.com/SIG=12anao87c/M=264379.5078783.6203979.1588051/D=brclubs/S=2137111528:HM/EXP=1103771306/A=2191897/R=0/SIG=10vqa2grn/*http://br.diversao.yahoo.com/>
>[]
>
><http://br.rd.yahoo.com/SIG=12anao87c/M=264379.5078783.6203979.1588051/D=brclubs/S=2137111528:HM/EXP=1103771306/A=2191897/R=1/SIG=10vqa2grn/*http://br.diversao.yahoo.com/>
>[]
> São Paulo Rio de Janeiro Curitiba Porto Alegre Belo Horizonte Brasília
>
>
>----------
>Links do Yahoo! Grupos
> * Para visitar o site do seu grupo na web, acesse:
> *
> <http://br.groups.yahoo.com/group/ciencialist/>http://br.groups.yahoo.com/group/ciencialist/
>
> *
> * Para sair deste grupo, envie um e-mail para:
> *
> <mailto:ciencialist-unsubscribe@yahoogrupos.com.br?subject=Unsubscribe>ciencialist-unsubscribe@yahoogrupos.com.br
>
> *
> * O uso que você faz do Yahoo! Grupos está sujeito aos
> <http://br.yahoo.com/info/utos.html>Termos do Serviço do Yahoo!.


[As partes desta mensagem que não continham texto foram removidas]



SUBJECT: Re: Nova pesquisa de opinião para ciencialist
FROM: Manuel Bulcão <manuelbulcao@uol.com.br>
TO: ciencialist@yahoogrupos.com.br
DATE: 22/12/2004 10:27


--- Em ciencialist@yahoogrupos.com.br, "L.E.R.de Carvalho"
<lecarvalho@i...> escreveu
> Tudo isso é tão engraçado e divertido como rinhas de galo.
>
> Tem quem aposte.
>
> Tem quem seja o galo.
>
> Tem gente que apenas acha curioso, ache inevitável ficar olhando,
e mesmo
> se divirta.
>
> Eu acho isso tudo muito miseravel.
>
> Numa lista sobre ciencia isso é inaceitável.

Manuel: ô LER, você é mesmo um idiota.

Quem é você para dar lição de moral, pôrra! Te manca, rapaz!

Você é mesmo uma criaturinha mesquinha. Um pobre diabo.

Agora vai posar de cientista injuriado, logo quem, o Ler!
Abestalhado que tecla suas mensagens em caixa alta!

Rapaz, você não tem senso do ridículo?

Manuel Bulcão






SUBJECT: Nova pesquisa de opinião para ciencialist
FROM: "L.E.R.de Carvalho" <lecarvalho@infolink.com.br>
TO: ciencialist@yahoogrupos.com.br
DATE: 22/12/2004 10:47


>Manuel: ô LER, você é mesmo um idiota.
>Quem é você para dar lição de moral, pôrra! Te manca, rapaz!
>Você é mesmo uma criaturinha mesquinha. Um pobre diabo.
>Agora vai posar de cientista injuriado, logo quem, o Ler!
>Abestalhado que tecla suas mensagens em caixa alta!
>Rapaz, você não tem senso do ridículo?
>Manuel Bulcão



O GALO FALOU.
SUSPENDE O OTORRINOVETERINARIO.

L.E.

[As partes desta mensagem que não continham texto foram removidas]



SUBJECT: Re: Nova pesquisa de opinião para ciencialist
FROM: Manuel Bulcão <manuelbulcao@uol.com.br>
TO: ciencialist@yahoogrupos.com.br
DATE: 22/12/2004 11:10


--- Em ciencialist@yahoogrupos.com.br, "L.E.R.de Carvalho"
<lecarvalho@i...> escreveu
> O GALO FALOU.
> SUSPENDE O OTORRINOVETERINARIO.
>
> L.E.

Manuel: Isso é piada? É pra rir?

Acho que o L.E. está papagaiando o Diogo Mainardi.

Curupaco-papaco,
Manuel Bulcão





SUBJECT: Re: [ciencialist] Nova pesquisa de opinião para ciencialist
FROM: "Amauri Jr" <amaurijunior2@yahoo.com.br>
TO: <ciencialist@yahoogrupos.com.br>
DATE: 22/12/2004 12:07

Oi "Ler" (o q?)


E daí o cara ser químico ou não, só porque o cara é químico se faz o que der na cabeça?? Acorda e te manca, você pelo jeito deve ser algo dele....

Amauri
----- Original Message -----
From: L.E.R.de Carvalho
To: ciencialist@yahoogrupos.com.br
Sent: Tuesday, December 21, 2004 11:22 PM
Subject: [ciencialist] Nova pesquisa de opinião para ciencialist



>O emiliano deve ser moderado?



POXA...
SERÁ QUE VOCÊS NÃO ENTENDEM QUE O CARA É QUÍMICO ?

DEIXA ELE, PÔ.

L.E.

[As partes desta mensagem que não continham texto foram removidas]



##### ##### #####

Para saber mais visite
http://www.ciencialist.hpg.ig.com.br


##### ##### ##### #####


Yahoo! Grupos, um serviço oferecido por:







------------------------------------------------------------------------------
Links do Yahoo! Grupos

a.. Para visitar o site do seu grupo na web, acesse:
http://br.groups.yahoo.com/group/ciencialist/

b.. Para sair deste grupo, envie um e-mail para:
ciencialist-unsubscribe@yahoogrupos.com.br

c.. O uso que você faz do Yahoo! Grupos está sujeito aos Termos do Serviço do Yahoo!.



[As partes desta mensagem que não continham texto foram removidas]



SUBJECT: Para todos uma Lua de Natal
FROM: Maria Natália <grasdic@hotmail.com>
TO: ciencialist@yahoogrupos.com.br
DATE: 22/12/2004 13:30


Mas que tem esta Lua de diferente das outras? Para nós com interesse
relativo. Para quem qer ver bem objectos distantes é mau porque se
torna muito claro. É luar!
Para os namorados está mesmo a dar...
Mas falando desta lua que vai por fios dourados na noite: ela é a
MAIS pequena de 2004. Estará alta no céu...a 406700 km de nós!
Tecnicamente falando a lua só estará a 100% cheia no dia 26 de
Dezembro. Vai nascer cedo nos dias 24 e 25.
Mas porque ela é a MAIS pequena? É que não é exacto que a órtbita da
Lua seja uma circunferência.É uma elipse: E as posições extremas
são: o perigeu a 356000 km da T e o apaogeu a 406000 km. verifiquem
qa diferença de 50000 km. Quando a Lua está próximo do apogeu parece-
nos mais pequena. Mas mesmo uma Lua cheia e pequena vai ser muito
brilhante e vai haver pessoal a fazer imagem. Olhar para ela de
binóculo vai ser de se ficar encandeado...Nos telescópios usamos um
filtro próprio e que atenuando o brilho intenso nos faz ver as um
pouco melhor as diferenças de tonalidade entre os mares e as
crateras
Gozem o belo luar e sede românticos ao menos uma vez no ano para
com vossos parceiros de vida.
Adaptado da Nasa podem ler com fotos e extra links ou até fazer uma
poster da Lua em:
http://science.nasa.gov/headlines/y2004/22dec_christmas.htm?list62687
Eu na noite estarei a pensar em vós com o vosso Natal tórrido com
caipirinhas...
Um abraço
Maria Natália










SUBJECT: Pedido [administrador]
FROM: "brudna" <lrb@iq.ufrgs.br>
TO: ciencialist@yahoogrupos.com.br
DATE: 22/12/2004 13:43


Vamos cuidar para que mensagens nao sejam mal interpretadas. Para
isso acho melhor evitar a agressividade. Ela pode ocorrer entre duas
pessoas que conhecem o ritmo da brincadeira (Takata e Bulcao, por
exemplo), mas vai acabar confundindo quem nao conhece a interacao
entre as duas pessoas, julgando que eh uma briga real.

Uma brincadeira pode acabar resultando em algo ´mais serio´. :-)

Se nao se comportarem o velho do saco vermelho nao vai trazer
presentes.


Ateh
Luis Brudna - administrador






SUBJECT: Re: Tudo é energia
FROM: Hélio Ricardo Carvalho <hrc@fis.puc-rio.br>
TO: ciencialist@yahoogrupos.com.br
DATE: 22/12/2004 13:50


José Renato,

Acho que este assunto pode continuar mais um pouco. Isto vai ajudar
a aumentar a média de mensagens científicas aqui. :-)

Eu escrevi:
>Mas é justamente isto que eu disse!!
> :-)

Aí você escreveu:
> É verdade, Hélio!
> Parece-me que foi o termo "se desmancham" que atropelou meu
entendimento.
> Valeu!

Agora, APARENTEMENTE me contradizendo, eu digo:
Existe uma diferença "sutil" entre "desmanchar" e "combinar
opostos".

Como o Alberto falou: "...seria a mesma coisa analisada sob um
prisma diferente."

É isto. O prisma.

Para quem acha que TUDO É ENERGIA prefere "combinar opostos" como
termo mais científico e pode aceitar o "desmanchar" como uma figura
de linguagem.

Para quem acha que TUDO É MATÉRIA prefere "desmanchar" como termo
mais científico e pode aceitar o "combinar opostos" como uma figura
de linguagem.

Objetivo disto é mostrar que é possível descrever um resultado
experimental considerando que tudo é matéria.
Como disse antes, isto ainda é muito especulativo e ainda não
consigo explicar tudo. Mas não tenho motivos para parar de "brincar"
com isto nas horas vagas. :-)

Hélio






SUBJECT: Re: Pedido [administrador]/Carvão?
FROM: Maria Natália <grasdic@hotmail.com>
TO: ciencialist@yahoogrupos.com.br
DATE: 22/12/2004 14:20



Brudna:

No meu tempo...ai o reumático... LOL
...se dizia que quem se portava mal recebia um pedaço de carvão nos
sapatinhos.
A propósito de CARVÃO:
Sei dos vossos churrascos. Gostava de saber se usam carvão e nesse
caso como o obtêm. Natural ou artificial? Vão desbastando as
florestas e fazendo prédios e estradas? Ou tendes processo mais
ecológico?
A seguir vos contarei como se obtem em Portugal. É um processo
químico muito engenhoso.
Vamos contar histórias científicas de Natal? sai o capuchinho
vermelho entra o homem das barbas.
Um abraço
Maria Natália
PS não ganhaste para o susto...

--- Em ciencialist@yahoogrupos.com.br, "brudna" <lrb@i...> escreveu
>
> Vamos cuidar para que mensagens nao sejam mal interpretadas. Para
> isso acho melhor evitar a agressividade. Ela pode ocorrer entre
duas
> pessoas que conhecem o ritmo da brincadeira (Takata e Bulcao, por
> exemplo), mas vai acabar confundindo quem nao conhece a interacao
> entre as duas pessoas, julgando que eh uma briga real.
>
> Uma brincadeira pode acabar resultando em algo ´mais serio´. :-)
>
> Se nao se comportarem o velho do saco vermelho nao vai trazer
> presentes.
>
>
> Ateh
> Luis Brudna - administrador





SUBJECT: Re: [ciencialist] Re: Pedido [administrador]/Carvão?
FROM: "Alessandro D. R. Fazenda" <alessandro@servidados.com.br>
TO: "[ciencialist]" <ciencialist@yahoogrupos.com.br>
DATE: 22/12/2004 15:40

Ma. Natalia:

Aqui onde eu vivo, no Rio Grande do Sul, na legitima terra do churrasco e onde nascem os melhores churrasqueiros do mundo, só usamos carvão vegetal. No saco de carvão tem umas inscrições tipo "madeira reflorestada" e "ecologico", mas não sei até onde é verdade e onde começa o marketing.
Me disseram que o melhor carvão é o de acácia negra, não sei porque, mas ainda descubro.

sds

Alessandro


----- Original Message -----
From: Maria Natália
To: ciencialist@yahoogrupos.com.br
Sent: Wednesday, December 22, 2004 2:20 PM
Subject: [ciencialist] Re: Pedido [administrador]/Carvão?





Brudna:

No meu tempo...ai o reumático... LOL
...se dizia que quem se portava mal recebia um pedaço de carvão nos
sapatinhos.
A propósito de CARVÃO:
Sei dos vossos churrascos. Gostava de saber se usam carvão e nesse
caso como o obtêm. Natural ou artificial? Vão desbastando as
florestas e fazendo prédios e estradas? Ou tendes processo mais
ecológico?
A seguir vos contarei como se obtem em Portugal. É um processo
químico muito engenhoso.
Vamos contar histórias científicas de Natal? sai o capuchinho
vermelho entra o homem das barbas.
Um abraço
Maria Natália
PS não ganhaste para o susto...

--- Em ciencialist@yahoogrupos.com.br, "brudna" <lrb@i...> escreveu
>
> Vamos cuidar para que mensagens nao sejam mal interpretadas. Para
> isso acho melhor evitar a agressividade. Ela pode ocorrer entre
duas
> pessoas que conhecem o ritmo da brincadeira (Takata e Bulcao, por
> exemplo), mas vai acabar confundindo quem nao conhece a interacao
> entre as duas pessoas, julgando que eh uma briga real.
>
> Uma brincadeira pode acabar resultando em algo ´mais serio´. :-)
>
> Se nao se comportarem o velho do saco vermelho nao vai trazer
> presentes.
>
>
> Ateh
> Luis Brudna - administrador






[As partes desta mensagem que não continham texto foram removidas]



SUBJECT: Galáxias na galáxia...
FROM: José Renato <jrma@terra.com.br>
TO: <ciencialist@yahoogrupos.com.br>
DATE: 22/12/2004 16:49

Afinal, como é que pode se afirmar que foram "descobertas" 30 galáxias nas bordas de uma galáxia maior: a Via Lacta?
Abraços
José Renato

[As partes desta mensagem que não continham texto foram removidas]



SUBJECT: Nova pesquisa de opinião para cien cialist
FROM: "L.E.R.de Carvalho" <lecarvalho@infolink.com.br>
TO: CIENcialist@yahoogrupos.com.br
DATE: 22/12/2004 17:57


>Oi "Ler" (o q?)
>
>
>E daí o cara ser químico ou não, só porque o cara é químico se faz o que
>der na cabeça?? Acorda e te manca, você pelo jeito deve ser algo dele....
>
>Amauri



OH AMAURI...
EU TAVA SÓ TENTANDO DESVIAR O ASSUNTO.
TEM UM PESSOAL AÍ QUE BRIGA POR COISAS QUE NEM IMAGINO QUAIS SEJAM.

RELAX.

L.E.

[As partes desta mensagem que não continham texto foram removidas]



SUBJECT: Votos de feliz natal e próspero an o novo
FROM: "manuelbulcao" <manuelbulcao@uol.com.br> (by way of "L.E.R.de Carvalho" <lecarvalho@infolink.com.br>)
TO: ciencialist@yahoogrupos.com.br
CC: grasdic@hotmail.com,lrb@iq.ufrgs.br,amaurijunior2@yahoo.com.br,
DATE: 22/12/2004 17:57

Vai te foder, infeliz! :-)
Manuel Bulcão



================


Não entendi.
Tá falando comigo, Manuel ?

Primeiro vocês inventam esse Tribunal de botar gente pra fora.

Agora passam a enviar agressão gratuita em privativo.

Realmente... haja ciencia pra entender esse mundo.

Oh Natalia, tu que é Química, me diz...
tu acha que isso aí é culpa de algum álcool ?

L.E.







-
-
-
-
-
-
-




SUBJECT: Re:
FROM: "JVictor" <jvoneto@uol.com.br>
TO: <ciencialist@yahoogrupos.com.br>
DATE: 22/12/2004 18:34

Recebí isto e não entendí. O que se passa?

Victor.


----- Original Message -----
To: <ciencialist@yahoogrupos.com.br>
Cc: <grasdic@hotmail.com>; <lrb@iq.ufrgs.br>; <amaurijunior2@yahoo.com.br>
Sent: Wednesday, December 22, 2004 5:57 PM


> X-eGroups-Remote-IP: 200.187.64.7
> From: "manuelbulcao" <manuelbulcao@uol.com.br>(by way of "L.E.R.de
> Carvalho" <lecarvalho@infolink.com.br>)
> X-Yahoo-Profile: manuelbulcao
> MIME-Version: 1.0
> Mailing-List: list ciencialist@yahoogrupos.com.br; contact
> ciencialist-owner@yahoogrupos.com.br
> Delivered-To: mailing list ciencialist@yahoogrupos.com.br
> Precedence: bulk
> List-Unsubscribe: <mailto:ciencialist-unsubscribe@yahoogrupos.com.br>
> Date: Wed, 22 Dec 2004 17:57:24 -0200
> Subject: [ciencialist]
> =?iso-8859-1?Q?Votos_de_feliz_natal_e_pr=F3spero_an_o_novo?=
> Reply-To: ciencialist@yahoogrupos.com.br
> Content-Type: text/html; charset=ISO-8859-1
> Content-Transfer-Encoding: quoted-printable
>
>
> <html><body>
>
>
>
> <tt>
> Vai te foder, infeliz! :-)<BR>
> Manuel Bulc=E3o<BR>
> <BR>
> <BR>
> <BR>
> =3D=3D=3D=3D=3D=3D=3D=3D=3D=3D=3D=3D=3D=3D=3D=3D<BR>
> <BR>
> <BR>
> N=E3o entendi.<BR>
> T=E1 falando comigo, Manuel ?<BR>
> <BR>
> Primeiro voc=EAs inventam esse Tribunal de botar gente pra fora.<BR>
> <BR>
> Agora passam a enviar agress=E3o gratuita em privativo.<BR>
> <BR>
> Realmente... haja ciencia pra entender esse mundo.<BR>
> <BR>
> Oh Natalia, tu que =E9 Qu=EDmica, me diz...<BR>
> tu acha que isso a=ED =E9 culpa de algum =E1lcool ?<BR>
> <BR>
> L.E.<BR>
> <BR>
> <BR>
> <BR>
> <BR>
> <BR>
> <BR>
> <BR>
> -<BR>
> -<BR>
> -<BR>
> -<BR>
> -<BR>
> -<BR>
> -<BR>
> <BR>
> <BR>
> </tt>
>
> <br><br>
> <tt>
> #####&nbsp;&nbsp; #####&nbsp;&nbsp; #####<BR>
> <BR>
> Para saber mais visite<BR>
> <a
> href=3D"http://www.ciencialist.hpg.ig.com.br">http://www.ciencialist.hpg=
> .ig.com.br</a><BR>
> <BR>
> <BR>
> #####&nbsp;&nbsp; #####&nbsp;&nbsp; #####&nbsp;&nbsp; #####</tt>
> <br><br>
>
> <br>
>
> <!-- |**|begin egp html banner|**| -->
>
> <table border=3D0 cellspacing=3D0 cellpadding=3D2>
> <tr bgcolor=3D#FFFFCC>
> <td align=3Dcenter><font size=3D"-1" color=3D#003399><b>Yahoo! Grupos, um
> s=
> ervi=E7o oferecido por:</b></font></td>
> </tr>
> <tr bgcolor=3D#FFFFFF>
> <td align=3Dcenter width=3D470><table width=3D"300" border=3D"0"
> cellspacin=
> g=3D"0" cellpadding=3D"0" height=3D"250" bgcolor=3D"#000000">
> <tr>
> <td height=3D"200"><a HREF=3D"http://br.rd.yahoo.com/SIG=3D12aciouh5/M=
> =3D264379.5078783.6203979.1588051/D=3Dbrclubs/S=3D2137111528:HM/EXP=3D11038=
> 31862/A=3D2191897/R=3D0/SIG=3D10vqa2grn/*http://br.diversao.yahoo.com/"><im=
> g
> src=3D"http://br.i1.yimg.com/br.yimg.com/i/br/ads4/div1407_lrec_teatro_di=
> versao.gif" width=3D"300" height=3D"200" border=3D"0"></a></td>
> </tr>
> <tr>
> <td height=3D"50">
> <table width=3D"300" border=3D"0" cellspacing=3D"0"
> cellpadding=3D"0"=
> height=3D"50">
> <tr>
> <td width=3D"150" align=3D"center"><a
> HREF=3D"http://br.rd.yahoo.=
> com/SIG=3D12aciouh5/M=3D264379.5078783.6203979.1588051/D=3Dbrclubs/S=3D2137=
> 111528:HM/EXP=3D1103831862/A=3D2191897/R=3D1/SIG=3D10vqa2grn/*http://br.div=
> ersao.yahoo.com/"><img
> src=3D"http://br.i1.yimg.com/br.yimg.com/i/br/ads4/d=
> iv1407_lrec_assinatura_diversao.gif" width=3D"148" height=3D"47"
> border=3D"=
> 0"></a></td>
> <td width=3D"150" align=3D"center" valign=3D"middle">
> <form name=3D"form_diversao" method=3D"get"
> action=3D"http://br=
> .rd.yahoo.com/SIG=3D12aciouh5/M=3D264379.5078783.6203979.1588051/D=3Dbrclub=
> s/S=3D2137111528:HM/EXP=3D1103831862/A=3D2191897/R=3D2/SIG=3D10vqa2grn/*htt=
> p://br.diversao.yahoo.com/" >
> <select name=3D"select" onChange=3D"window.open(this.value)">
> <option value=3D"http://br.rd.yahoo.com/SIG=3D12aciouh5/M=
> =3D264379.5078783.6203979.1588051/D=3Dbrclubs/S=3D2137111528:HM/EXP=3D11038=
> 31862/A=3D2191897/R=3D3/SIG=3D11fubje7g/*http://br.diversao.yahoo.com/sampa=
> /index.html" selected>S&atilde;o=20
> Paulo</option>
> <option value=3D"http://br.rd.yahoo.com/SIG=3D12aciouh5/M=
> =3D264379.5078783.6203979.1588051/D=3Dbrclubs/S=3D2137111528:HM/EXP=3D11038=
> 31862/A=3D2191897/R=3D4/SIG=3D11dcdc4pm/*http://br.diversao.yahoo.com/rio/i=
> ndex.html">Rio=20
> de Janeiro</option>
> <option value=3D"http://br.rd.yahoo.com/SIG=3D12aciouh5/M=
> =3D264379.5078783.6203979.1588051/D=3Dbrclubs/S=3D2137111528:HM/EXP=3D11038=
> 31862/A=3D2191897/R=3D5/SIG=3D11ih9vigm/*http://br.diversao.yahoo.com/curit=
> iba/index.html">Curitiba</option>
> <option value=3D"http://br.rd.yahoo.com/SIG=3D12aciouh5/M=
> =3D264379.5078783.6203979.1588051/D=3Dbrclubs/S=3D2137111528:HM/EXP=3D11038=
> 31862/A=3D2191897/R=3D6/SIG=3D11d0trfh2/*http://br.diversao.yahoo.com/poa/i=
> ndex.html">Porto=20
> Alegre</option>
> <option value=3D"http://br.rd.yahoo.com/SIG=3D12aciouh5/M=
> =3D264379.5078783.6203979.1588051/D=3Dbrclubs/S=3D2137111528:HM/EXP=3D11038=
> 31862/A=3D2191897/R=3D7/SIG=3D11c9hnho2/*http://br.diversao.yahoo.com/bh/in=
> dex.html">Belo=20
> Horizonte</option>
> <option value=3D"http://br.rd.yahoo.com/SIG=3D12aciouh5/M=
> =3D264379.5078783.6203979.1588051/D=3Dbrclubs/S=3D2137111528:HM/EXP=3D11038=
> 31862/A=3D2191897/R=3D8/SIG=3D11iplshqf/*http://br.diversao.yahoo.com/brasi=
> lia/index.html">Bras&iacute;lia</option>
> </select>
> </form>
> </td>
> </tr>
> </table>
> </td>
> </tr>
> </table></td>
> </tr>
> </table>
>
> <!-- |**|end egp html banner|**| -->
>
>
>
> <!-- |**|begin egp html banner|**| -->
>
> <br>
> <tt><hr width=3D"500">
> <b>Links do Yahoo! Grupos</b><br>
> <ul>
> <li>Para visitar o site do seu grupo na web, acesse:<br><a
> href=3D"http://b=
> r.groups.yahoo.com/group/ciencialist/">http://br.groups.yahoo.com/group/cie=
> ncialist/</a><br>&nbsp;
> <li>Para sair deste grupo, envie um e-mail para:<br><a
> href=3D"mailto:cienc=
> ialist-unsubscribe@yahoogrupos.com.br?subject=3DUnsubscribe">ciencialist-un=
> subscribe@yahoogrupos.com.br</a><br>&nbsp;
> <li>O uso que voc=EA faz do Yahoo! Grupos est=E1 sujeito aos <a
> href=3D"htt=
> p://br.yahoo.com/info/utos.html">Termos do Servi=E7o do Yahoo!</a>.
> </ul>
> </tt>
> </br>
>
> <!-- |**|end egp html banner|**| -->
>
>
> </body></html>
>
>
>
>




SUBJECT: Re: [ciencialist] Tudo é energia
FROM: José Renato <jrma@terra.com.br>
TO: <ciencialist@yahoogrupos.com.br>
DATE: 22/12/2004 18:40

Hélio, percebo a matéria como energia-campos "estruturados" em determinada
configuração. Já o espaço, mesmo pleno de energia e campos, considero como
"não estruturado", sem maiores tensões.
Abraços
José Renato
..............................................

----- Original Message -----
From: "Hélio Ricardo Carvalho" <hrc@fis.puc-rio.br>
To: <ciencialist@yahoogrupos.com.br>
Sent: Wednesday, December 22, 2004 12:50 PM
Subject: [ciencialist] Re: Tudo é energia




José Renato,

Acho que este assunto pode continuar mais um pouco. Isto vai ajudar
a aumentar a média de mensagens científicas aqui. :-)

Eu escrevi:
>Mas é justamente isto que eu disse!!
> :-)

Aí você escreveu:
> É verdade, Hélio!
> Parece-me que foi o termo "se desmancham" que atropelou meu
entendimento.
> Valeu!

Agora, APARENTEMENTE me contradizendo, eu digo:
Existe uma diferença "sutil" entre "desmanchar" e "combinar
opostos".

Como o Alberto falou: "...seria a mesma coisa analisada sob um
prisma diferente."

É isto. O prisma.

Para quem acha que TUDO É ENERGIA prefere "combinar opostos" como
termo mais científico e pode aceitar o "desmanchar" como uma figura
de linguagem.

Para quem acha que TUDO É MATÉRIA prefere "desmanchar" como termo
mais científico e pode aceitar o "combinar opostos" como uma figura
de linguagem.

Objetivo disto é mostrar que é possível descrever um resultado
experimental considerando que tudo é matéria.
Como disse antes, isto ainda é muito especulativo e ainda não
consigo explicar tudo. Mas não tenho motivos para parar de "brincar"
com isto nas horas vagas. :-)

Hélio






##### ##### #####

Para saber mais visite
http://www.ciencialist.hpg.ig.com.br


##### ##### ##### #####
Links do Yahoo! Grupos









Esta mensagem foi verificada pelo E-mail Protegido Terra.
Scan engine: McAfee VirusScan / Atualizado em 15/12/2004 / Versão: 4.4.00 -
Dat 4415
Proteja o seu e-mail Terra: http://www.emailprotegido.terra.com.br/

E-mail classificado pelo Identificador de Spam Inteligente Terra.
Para alterar a categoria classificada, visite
http://www.terra.com.br/centralunificada/emailprotegido/imail/imail.cgi?+_u=jrma&_l=1,1103730652.564961.22754.casama.terra.com.br,4153,Des15,Des15




SUBJECT: Re: [ciencialist] Outro moderador [administrador]
FROM: "Prof. JC" <profjc2003@yahoo.com.br>
TO: <ciencialist@yahoogrupos.com.br>
DATE: 22/12/2004 19:21

Ave Léo!

Nós, os mortais comuns, o saudamos, grande Imperador!

Perdoai nossos offtopics assim como perdoamos os nossos credores e não nos
deixei cair em moderação,

Amémdoim!
Prof. JC


----- Original Message -----
From: "brudna" <lrb@iq.ufrgs.br>
To: <ciencialist@yahoogrupos.com.br>
Sent: Tuesday, December 21, 2004 5:47 PM
Subject: [ciencialist] Outro moderador [administrador]




:-) Mais um foi promovido.

Luiz Ferraz Netto agora também tem privilégios de moderação da
Ciencialist.

Não sei se o Leo tem prática com o sistema. Se não tiver, deixe que
os outros moderadores tomam conta. :-)

Acho que agora temos um número adequado de moderadores. :-)


Ateh
Luis Brudna - administrador da Ciencialist





##### ##### #####

Para saber mais visite
http://www.ciencialist.hpg.ig.com.br


##### ##### ##### #####
Links do Yahoo! Grupos












SUBJECT: Re: Tudo é energia
FROM: Hélio Ricardo Carvalho <hrc@fis.puc-rio.br>
TO: ciencialist@yahoogrupos.com.br
DATE: 22/12/2004 20:11




Este teu "energia-campo" seria composto de que? ... :-)

"Espaço sem maiores tensões"???
O que seria tensão no espaço?

Hoje estou perguntador do tipo do "marcelo ferrari", mas é só por
agora.

[ ]'s
Hélio



--- Em ciencialist@yahoogrupos.com.br, José Renato <jrma@t...>
escreveu
> Hélio, percebo a matéria como energia-campos "estruturados" em
determinada
> configuração. Já o espaço, mesmo pleno de energia e campos,
considero como
> "não estruturado", sem maiores tensões.






SUBJECT: Re: [ciencialist] Nova pesquisa de opinião para ciencialist
FROM: "Prof. JC" <profjc2003@yahoo.com.br>
TO: <ciencialist@yahoogrupos.com.br>
DATE: 22/12/2004 20:30

O pior é que ela já foi expulsa de lá três vezes. :)))

Ops, não vem não! Só respondi porque fui CITADO. Reclama aí com quem me
citou, esse caçador de encrencas natalinas. :)

Abraços,
Prof. JC


----- Original Message -----
From: "L.E.R.de Carvalho" <lecarvalho@infolink.com.br>
To: <ciencialist@yahoogrupos.com.br>
Sent: Tuesday, December 21, 2004 11:58 PM
Subject: [ciencialist] Nova pesquisa de opinião para ciencialist




>Mané: Ocorre que o Emiliano criou uma enquete na lista "Conversa de
>Botequim", indagando se a Natália deveria ou não permanecer naquela
>lista (lista esta moderada pelo Zé Carlos, ora vejam!). Por conta
>dessa enquete, a Natália foi expulsa.
>
>Uma pessoa que é expulsa de uma lista de discussão
>intitulada "conversa de botequim" ou é um anjo ou então um demônio
>que nem bêbado agüenta, é o que se presume. A Natália tem toda a
>razão para ter ficado muito ofendida. Sou-lhe solidário.
>
>Amplexos,
>Manuel Bulcão



O CARA É QUÍMICO.
QUE OUTRA EXPLICAÇÃO DEVO BUSCAR PARA TE CONVENCER QUE NÃO ADIANTA MEXER
NISSO ?

A NATÁLIA FOI PREMIADA EM PLENO NATAL.
FAZ SENTIDO.
L.E.

[As partes desta mensagem que não continham texto foram removidas]






SUBJECT: Re: [ciencialist] Re: Nova pesquisa de opinião para ciencialist
FROM: "Prof. JC" <profjc2003@yahoo.com.br>
TO: <ciencialist@yahoogrupos.com.br>
DATE: 22/12/2004 20:39

Pedindo desculpas antecidas ao Leo e ao Brudna...

Natália, se você for falar alguma coisa do Boteco ou de mim eu abro seu
arquivo e todos saberão quem você é realmente. Inclusive publico na net
todas as suas mensagens e passo o link para conferirem pessoalmente o tipo
de democracia e comportamento que você aprecia e pratica. Então fiquemos
assim: da mesma forma como eu tenho sido ético ao levar não para outras
listas os seus podres, principalmente essa, que é a única onde você é
razoavelmente civilizada, tente não plantar mentiras e nem criar futricas
(que é sua especialidade).

Qualquer dúvida ou problema procure usar o PVT. Você tem meu e-mail, não
tem? É o mesmo que você usou para pedir seu ingresso no Boteco mais de dez
vezes entre uma expulsão e outra. :)

Abraços,
Prof. JC


----- Original Message -----
From: "Maria Natália" <grasdic@hotmail.com>
To: <ciencialist@yahoogrupos.com.br>
Sent: Wednesday, December 22, 2004 12:45 AM
Subject: [ciencialist] Re: Nova pesquisa de opinião para ciencialist




Manuel:

L.E.R* não sabe nem da missa a metade.Mas nem é preciso ir a essa
lista para se saber.
Creio que os moderadores NUNCA deixarão instalar-se aqui o ambiente
dessa lista que falaste. Seria insultuoso para todos nós.
Sei que apesar de alguns contratempos esta lista é muito capaz de
ensinar a outras o que é a democracia.
Quanto às acções...elas ficam com quem as pratica e nas listas se
nota logo quando mensagens são apagadas. Quem escreveu assume.
Quem entra numa lista deve analisar todos os "postes" indo até à
primeira mensagem para saber onde está e se situar.
Parece OT mas não o é. São normas de segurança de circulação na
internet. Assim como num laboratório se deve ter cuidado a
armazenar reagentes...
Eu sou química. Tenho a infelicidade de ser a única mulher que fala
regularmente.E como ensinar ciência é ser-se revolucionário...
Continuarei agitando as águas e não me assustando com as bombas
caindo aqui e ali porque pratico as normas de segurança.
Conseguem ouvir o que NÃO está sendo dito?
Uma abraço com votos de boa jornada ao triunvirato que ajudará esta
lista a ter e manter a LIBERDADE e respeito com que nasceu.
Um abraço
queridos especialistos
Maria Natália
* Te escrevo em pvt pois isso é OT


--- Em ciencialist@yahoogrupos.com.br, Manuel Bulcão
<manuelbulcao@u...> escreveu
>
> --- Em ciencialist@yahoogrupos.com.br, "L.E.R.de Carvalho"
> <lecarvalho@i...> escreveu
> >
> > >O emiliano deve ser moderado?
> >
> >
> >
> > POXA...
> > SERÁ QUE VOCÊS NÃO ENTENDEM QUE O CARA É QUÍMICO ?
> >
> > DEIXA ELE, PÔ.
>
> Mané: Ocorre que o Emiliano criou uma enquete na lista "Conversa
de
> Botequim", indagando se a Natália deveria ou não permanecer
naquela
> lista (lista esta moderada pelo Zé Carlos, ora vejam!). Por conta
> dessa enquete, a Natália foi expulsa.
>
> Uma pessoa que é expulsa de uma lista de discussão
> intitulada "conversa de botequim" ou é um anjo ou então um demônio
> que nem bêbado agüenta, é o que se presume. A Natália tem toda a
> razão para ter ficado muito ofendida. Sou-lhe solidário.
>
> Amplexos,
> Manuel Bulcão





##### ##### #####

Para saber mais visite
http://www.ciencialist.hpg.ig.com.br


##### ##### ##### #####
Links do Yahoo! Grupos












SUBJECT: Re: [ciencialist] Re:
FROM: "Alberto Mesquita Filho" <albmesq@uol.com.br>
TO: <ciencialist@yahoogrupos.com.br>
DATE: 22/12/2004 20:53

----- Original Message -----
From: "JVictor"
Sent: Wednesday, December 22, 2004 5:34 PM
Subject: [ciencialist] Re:

> Recebí isto e não entendí. O que se passa?

Briguinhas que provavelmente ocorreram metade em pvt e metade na lista. Além
do que, a msg saiu "hieroglifada" por razões que a própria razão desconhece
(talvez os especialistas em computação saibam), e acho que foi melhor assim.
Se quiser ler o texto sem hieróglifos sugiro que dirija-se à página web, msg
43371. Espero que a briga termine em pizza, ou melhor, em perus de Natal, um
na casa de cada um dos net-amigos.

Solicito a todos que atendam às recomendações do Brudna (msg
http://br.groups.yahoo.com/group/ciencialist/message/43365 ) bem como
reproduzo a advertência do amigo: "Se não se comportarem o velho do saco
vermelho nao vai trazer presentes."

[ ]´s
Alberto
http://ecientificocultural.com/indice.htm



SUBJECT: Re: [ciencialist] Tudo é energia
FROM: José Renato <jrma@terra.com.br>
TO: <ciencialist@yahoogrupos.com.br>
DATE: 23/12/2004 01:24

Hélio, o meu e acredito que também o seu energia-campo é composto de
fenômenos que denominamos de energia e dos que chamamos de campo! :-))
Tensão no espaço são provocadas pelas forças nucleares, forte e fraca,
magnetismo e gravidade. Exemplo: o átomo é um conjunto material vazio, mais
espaço que matéria, porém devido às "tensões" existentes no núcleo,
interação das subpartículas, nos elétrons - emissores e receptores de quanta
e fótons mas que ainda é considerado uma partícula, digamos,
ndivisível(?) - e entre o núcleo e os elétrons parece ter uma consistência
material "sólida" devido a essa estrutura de tensões.
[]s
José Renato
.......................................................

From: "Hélio Ricardo Carvalho" <hrc@fis.puc-rio.br>
To: <ciencialist@yahoogrupos.com.br>
Sent: Wednesday, December 22, 2004 7:11 PM
Subject: [ciencialist] Re: Tudo é energia

Este teu "energia-campo" seria composto de que? ... :-)

"Espaço sem maiores tensões"???
O que seria tensão no espaço?

Hoje estou perguntador do tipo do "marcelo ferrari", mas é só por
agora.

[ ]'s
Hélio



--- Em ciencialist@yahoogrupos.com.br, José Renato <jrma@t...>
escreveu
> Hélio, percebo a matéria como energia-campos "estruturados" em
determinada
> configuração. Já o espaço, mesmo pleno de energia e campos,
considero como
> "não estruturado", sem maiores tensões.






##### ##### #####

Para saber mais visite
http://www.ciencialist.hpg.ig.com.br


##### ##### ##### #####
Links do Yahoo! Grupos









Esta mensagem foi verificada pelo E-mail Protegido Terra.
Scan engine: McAfee VirusScan / Atualizado em 22/12/2004 / Versão: 4.4.00 -
Dat 4416
Proteja o seu e-mail Terra: http://www.emailprotegido.terra.com.br/

E-mail classificado pelo Identificador de Spam Inteligente Terra.
Para alterar a categoria classificada, visite
http://www.terra.com.br/centralunificada/emailprotegido/imail/imail.cgi?+_u=jrma&_l=1,1103753533.708756.20502.cabue.terra.com.br,3444,Des15,Des15




SUBJECT: Re: Tudo é energia
FROM: "Cyberlander" <cybernews@superig.com.br>
TO: ciencialist@yahoogrupos.com.br
DATE: 23/12/2004 01:36



fótons são pacotinhos com massa ou picos de onda?
[ ]'s
D.C.



--- Em ciencialist@yahoogrupos.com.br, José Renato <jrma@t...>
escreveu
> Hélio, o meu e acredito que também o seu energia-campo é composto
de
> fenômenos que denominamos de energia e dos que chamamos de
campo! :-))
> Tensão no espaço são provocadas pelas forças nucleares, forte e
fraca,
> magnetismo e gravidade. Exemplo: o átomo é um conjunto material
vazio, mais
> espaço que matéria, porém devido às "tensões" existentes no
núcleo,
> interação das subpartículas, nos elétrons - emissores e receptores
de quanta
> e fótons mas que ainda é considerado uma partícula, digamos,
> ndivisível(?) - e entre o núcleo e os elétrons parece ter uma
consistência
> material "sólida" devido a essa estrutura de tensões.
> []s
> José Renato
> .......................................................
>
> From: "Hélio Ricardo Carvalho" <hrc@f...>
> To: <ciencialist@yahoogrupos.com.br>
> Sent: Wednesday, December 22, 2004 7:11 PM
> Subject: [ciencialist] Re: Tudo é energia
>
> Este teu "energia-campo" seria composto de que? ... :-)
>
> "Espaço sem maiores tensões"???
> O que seria tensão no espaço?
>
> Hoje estou perguntador do tipo do "marcelo ferrari", mas é só por
> agora.
>
> [ ]'s
> Hélio
>
>
>
> --- Em ciencialist@yahoogrupos.com.br, José Renato <jrma@t...>
> escreveu
> > Hélio, percebo a matéria como energia-campos "estruturados" em
> determinada
> > configuração. Já o espaço, mesmo pleno de energia e campos,
> considero como
> > "não estruturado", sem maiores tensões.
>
>
>
>
>
>
> ##### ##### #####
>
> Para saber mais visite
> http://www.ciencialist.hpg.ig.com.br
>
>
> ##### ##### ##### #####
> Links do Yahoo! Grupos
>
>
>
>
>
>
>
>
>
> Esta mensagem foi verificada pelo E-mail Protegido Terra.
> Scan engine: McAfee VirusScan / Atualizado em 22/12/2004 / Versão:
4.4.00 -
> Dat 4416
> Proteja o seu e-mail Terra: http://www.emailprotegido.terra.com.br/
>
> E-mail classificado pelo Identificador de Spam Inteligente Terra.
> Para alterar a categoria classificada, visite
>
http://www.terra.com.br/centralunificada/emailprotegido/imail/imail.c
gi?
+_u=jrma&_l=1,1103753533.708756.20502.cabue.terra.com.br,3444,Des15,D
es15





SUBJECT: Re: [ciencialist] Re: Nova pesquisa de opinião para ciencialist
FROM: "Cyberlander" <cybernews@superig.com.br>
TO: <ciencialist@yahoogrupos.com.br>
DATE: 23/12/2004 02:02

JC vc por aqui! hehehehe
[ ]'s
D.C.
----- Original Message -----
From: Prof. JC
To: ciencialist@yahoogrupos.com.br
Sent: Wednesday, December 22, 2004 7:39 PM
Subject: Re: [ciencialist] Re: Nova pesquisa de opinião para ciencialist



Pedindo desculpas antecidas ao Leo e ao Brudna...

Natália, se você for falar alguma coisa do Boteco ou de mim eu abro seu
arquivo e todos saberão quem você é realmente. Inclusive publico na net
todas as suas mensagens e passo o link para conferirem pessoalmente o tipo
de democracia e comportamento que você aprecia e pratica. Então fiquemos
assim: da mesma forma como eu tenho sido ético ao levar não para outras
listas os seus podres, principalmente essa, que é a única onde você é
razoavelmente civilizada, tente não plantar mentiras e nem criar futricas
(que é sua especialidade).

Qualquer dúvida ou problema procure usar o PVT. Você tem meu e-mail, não
tem? É o mesmo que você usou para pedir seu ingresso no Boteco mais de dez
vezes entre uma expulsão e outra. :)

Abraços,
Prof. JC


----- Original Message -----
From: "Maria Natália" <grasdic@hotmail.com>
To: <ciencialist@yahoogrupos.com.br>
Sent: Wednesday, December 22, 2004 12:45 AM
Subject: [ciencialist] Re: Nova pesquisa de opinião para ciencialist




Manuel:

L.E.R* não sabe nem da missa a metade.Mas nem é preciso ir a essa
lista para se saber.
Creio que os moderadores NUNCA deixarão instalar-se aqui o ambiente
dessa lista que falaste. Seria insultuoso para todos nós.
Sei que apesar de alguns contratempos esta lista é muito capaz de
ensinar a outras o que é a democracia.
Quanto às acções...elas ficam com quem as pratica e nas listas se
nota logo quando mensagens são apagadas. Quem escreveu assume.
Quem entra numa lista deve analisar todos os "postes" indo até à
primeira mensagem para saber onde está e se situar.
Parece OT mas não o é. São normas de segurança de circulação na
internet. Assim como num laboratório se deve ter cuidado a
armazenar reagentes...
Eu sou química. Tenho a infelicidade de ser a única mulher que fala
regularmente.E como ensinar ciência é ser-se revolucionário...
Continuarei agitando as águas e não me assustando com as bombas
caindo aqui e ali porque pratico as normas de segurança.
Conseguem ouvir o que NÃO está sendo dito?
Uma abraço com votos de boa jornada ao triunvirato que ajudará esta
lista a ter e manter a LIBERDADE e respeito com que nasceu.
Um abraço
queridos especialistos
Maria Natália
* Te escrevo em pvt pois isso é OT


--- Em ciencialist@yahoogrupos.com.br, Manuel Bulcão
<manuelbulcao@u...> escreveu
>
> --- Em ciencialist@yahoogrupos.com.br, "L.E.R.de Carvalho"
> <lecarvalho@i...> escreveu
> >
> > >O emiliano deve ser moderado?
> >
> >
> >
> > POXA...
> > SERÁ QUE VOCÊS NÃO ENTENDEM QUE O CARA É QUÍMICO ?
> >
> > DEIXA ELE, PÔ.
>
> Mané: Ocorre que o Emiliano criou uma enquete na lista "Conversa
de
> Botequim", indagando se a Natália deveria ou não permanecer
naquela
> lista (lista esta moderada pelo Zé Carlos, ora vejam!). Por conta
> dessa enquete, a Natália foi expulsa.
>
> Uma pessoa que é expulsa de uma lista de discussão
> intitulada "conversa de botequim" ou é um anjo ou então um demônio
> que nem bêbado agüenta, é o que se presume. A Natália tem toda a
> razão para ter ficado muito ofendida. Sou-lhe solidário.
>
> Amplexos,
> Manuel Bulcão





##### ##### #####

Para saber mais visite
http://www.ciencialist.hpg.ig.com.br


##### ##### ##### #####
Links do Yahoo! Grupos












##### ##### #####

Para saber mais visite
http://www.ciencialist.hpg.ig.com.br


##### ##### ##### #####
Links do Yahoo! Grupos










[As partes desta mensagem que não continham texto foram removidas]



SUBJECT: Testes de elevada precisão em Relatividade
FROM: Maria Natália <grasdic@hotmail.com>
TO: ciencialist@yahoogrupos.com.br
DATE: 23/12/2004 04:24


Esta mensagem trata de apenas de Relatividade e nela se encontram
uns links bastante interessantes (principalmente o 3):

1--Física Fundamental na ISS (Estação Espacila Internacional)
http://www.exphy.uni-duesseldorf.de/Laemmerzahl/TT/

2--Óptica Quântica
A melhor prova experimental da isotropia de velocidade da luz
(Experiência de Michelsion-Morley) e a independência da velocidade
da luz (testes de Kennedy-Thorndike) foram realizadas no Instituto
de Física Experimental e universidade de Dusseldorf
http://www.exphy.uni-duesseldorf.de/WelcomeInst.html

3--Física Fundamental: testes de elevada precisão em Relatividade
Geral e Especial
http://www.exphy.uni-duesseldorf.de/ResearchInst/WelcomeFP.html

Vale a pena ler no fds porque o tema exige uma leitura calma.
abs
Maria Natália







SUBJECT: Re: Votos de feliz natal e próspero an o novo/Bêbados e álcool
FROM: Maria Natália <grasdic@hotmail.com>
TO: ciencialist@yahoogrupos.com.br
DATE: 23/12/2004 05:27


L.E.R:

1--Temos muitas moléculas de álcool lá para as bandas da nebulosa de
Orion (M42 - nuven moleculare gigante) Desde os anos 60 que os rádio-
astrónomos detectaram riscas de emissão de micro-ondas produzidas
por muitas moléculas existentes no meio interestelar. Por incrível
que possa parecer, até a molécula de álcool etílico já foi detectada
nesta zona e a quantidade é de tal ordem que daria para toda a
população mundial andar permanentemente bêbada durante 5000 anos!

2--Mas também região desta constelação foram encontrados muitos
planetas extra solares tamanho de Júpiter. Se haverá Vida e que tipo
de Vida será FC falar-se agora.
A experiência de Miller em 1952 mostrou que a vida pode ter
resultado naturalmente de reações químicas ordinárias. Então, dada a
abundância no universo das moléculas que estão na base da origem da
vida, é perfeitamente legítimo supôr que a vida possa também ter
despontado fora da Terra, algures no cosmos

3--Falar-se "de que eles já estão entre nós" é utopia.

4--E para finalizar o Cometa C/2004 Q2 (Machholz)está visível no
hemisfério Norte perto de Orion e na constelação de Eridanus. Mas
para o pessoal do hemisfério Sum será melhor irem ao site do ON

http://costeira1.astrodatabase.net/cometa/04q2.htm
http://www.astro.up.pt/nd/astro_news/2004/1210pt.html

E para o HSul, no local de Leo:
http://www.feiradeciencias.com.br/sala24/24_C11.asp
E Rosely Gregio em:
http://rgregio.astrodatabase.net/efemerides_historicas.htm

E foi um dos objectos observados esta noite até às 3 da manhã no
Observatório do Geográfico do Exército. Com 4 alunos de engenharia
física tecnológica e pessoal do CA2000 fizemos fotografia que após
tratamento colocarei aqui. No dia 6 de Janeiro é que teremos uma
hipótese de melhor localização pois ele estará perto das Pleídas e
sendo esta o "enxame aberto dos cábulas" toda a gente o poderá ver
em local escuro.

Fácil é a partir do álcool fazer-se um passeio nos céus.Difícil é um
bêbado alcançar estas coisas.
q.e.d
Maria Natália


--- Em ciencialist@yahoogrupos.com.br, "manuelbulcao"
<manuelbulcao@u...> (by way of "L.E.R.de Carvalho"
<lecarvalho@i...>) escreveu
> Vai te foder, infeliz! :-)
> Manuel Bulcão
>
>
>
> ================
>
>
> Não entendi.
> Tá falando comigo, Manuel ?
>
> Primeiro vocês inventam esse Tribunal de botar gente pra fora.
>
> Agora passam a enviar agressão gratuita em privativo.
>
> Realmente... haja ciencia pra entender esse mundo.
>
> Oh Natalia, tu que é Química, me diz...
> tu acha que isso aí é culpa de algum álcool ?
>
> L.E.
>
>
>
>
>
>
>
> -
> -
> -
> -
> -
> -
> -





SUBJECT: Era Pedido [adminis] Esta Carvão?
FROM: Maria Natália <grasdic@hotmail.com>
TO: ciencialist@yahoogrupos.com.br
DATE: 23/12/2004 06:32


Alessandro:

1--O carvão que se usa aqui é obtido a partir da madeira de
azinheira e sobreiro e é conhecido pelo nome de carvão de azinho ou
de sobro ou mais geralmente carvão de madeira/vegetal (de choça).
Este carvão é obtido pela combustão incompleta de troncos ou achas
secas e que resultam da poda destas árvores do montado.
Aresanalmente ele é obtido empilhando os troncos ou achas em camadas
por tamanhos de modo a se formar uma "meda" (formato parecido ao de
um migloo mas mais largo).Toda a madeira é coberta por terra e barro
deixando-se uma chaminé central. Depois inicia-se a combustão que se
para quando a a cor do fumo fica mais clara. Para parar a combustão
se fecha a chaminé. O calor desemnvolvidoé muito e como a quantidade
ar a circular é pequena estamos perante uma combustão incompleta
como convem. Carvoeiro azelha tapa mal a meda e lá se via tudo em
cinza.
As medas actualmente evoluiram e podem ver mais algumas indicações
em:

http://www.cm-chamusca.pt/circuito/charneca/chouto/
http://tinyurl.com/4nzmu

2-- O churrasco é a tal refeição que decorre ao ar livre e que era
suposto ser feita em colaboração entre todos para a dona de casa ter
uns momentos de descanso...da cozinha...
Acerca do grelhar sobre brasas li um trabalho de uma universidade
dos EUA (?) sobre as substãncias cancerígenas que se podem gerar na
queima e que sob a forma de vapor penetram na carne. O conselho dado
era de que a carne NUNCA fosse a grelhar sem ser bem untada de molho
(estar a macerar num molho), óleo ou azeite que protegeria a carne.
O deitar petróleo ou gasolina para atear as chamas também trás
consigo a má queima de hidrocarbonetos.
Também se põe o problema da carne por causa de loucuras. e quanto à
crua o perigo da toxoplasmose. Prefiro a carne de veado, de javali,
emas ou avestruz.
O estudo foi mesmo sério e não se trata de hoax ou campanha contra
gaúchos...
Se recomenda também que não se use madeira de móveis velhos por
causa dos terpenos presentes em vernizes e tintas ou até pesticidas.
Esta preocupação é recente...Pois na idade média quando se queimavam
na praça pública os indesejáveis qualquer lenha servia. Hoje se usam
processos mais soft mas de resultados que apresentam falhas.
E sobre o nosso carvão é o que tenho a dizer.
Seria interessante uma tese sobre a procura das causas de essa
madeira ser a escolhida para melhor churrasco.
Um abraço
Maria Natália
Porque a cozinha é um laboratório de química

--- Em ciencialist@yahoogrupos.com.br, "Alessandro D. R. Fazenda"
<alessandro@s...> escreveu
> Ma. Natalia:
>
> Aqui onde eu vivo, no Rio Grande do Sul, na legitima terra do
churrasco e onde nascem os melhores churrasqueiros do mundo, só
usamos carvão vegetal. No saco de carvão tem umas inscrições
tipo "madeira reflorestada" e "ecologico", mas não sei até onde é
verdade e onde começa o marketing.
> Me disseram que o melhor carvão é o de acácia negra, não sei
porque, mas ainda descubro.
>
> sds
>
> Alessandro
>
>
> ----- Original Message -----
> From: Maria Natália
> To: ciencialist@yahoogrupos.com.br
> Sent: Wednesday, December 22, 2004 2:20 PM
> Subject: [ciencialist] Re: Pedido [administrador]/Carvão?
>
>
>
>
>
> Brudna:
>
> No meu tempo...ai o reumático... LOL
> ...se dizia que quem se portava mal recebia um pedaço de carvão
nos
> sapatinhos.
> A propósito de CARVÃO:
> Sei dos vossos churrascos. Gostava de saber se usam carvão e nesse
> caso como o obtêm. Natural ou artificial? Vão desbastando as
> florestas e fazendo prédios e estradas? Ou tendes processo mais
> ecológico?
> A seguir vos contarei como se obtem em Portugal. É um processo
> químico muito engenhoso.
> Vamos contar histórias científicas de Natal? sai o capuchinho
> vermelho entra o homem das barbas.
> Um abraço
> Maria Natália
> PS não ganhaste para o susto...
>
> --- Em ciencialist@yahoogrupos.com.br, "brudna" <lrb@i...> escreveu
> >
> > Vamos cuidar para que mensagens nao sejam mal interpretadas.
Para
> > isso acho melhor evitar a agressividade. Ela pode ocorrer entre
> duas
> > pessoas que conhecem o ritmo da brincadeira (Takata e Bulcao, por
> > exemplo), mas vai acabar confundindo quem nao conhece a interacao
> > entre as duas pessoas, julgando que eh uma briga real.
> >
> > Uma brincadeira pode acabar resultando em algo ´mais serio´. :-
)
> >
> > Se nao se comportarem o velho do saco vermelho nao vai trazer
> > presentes.
> >
> >
> > Ateh
> > Luis Brudna - administrador
>
>
>
>
>
>
> [As partes desta mensagem que não continham texto foram removidas]





SUBJECT: Re: Tudo é energia
FROM: Maria Natália <grasdic@hotmail.com>
TO: ciencialist@yahoogrupos.com.br
DATE: 23/12/2004 07:05


DC:

O fotão é partícula que na teoria corpuscular constitui a radiação
(luz). E pico, critas de onde se falamos na teoria ondulatória da
luz. Estas duas teorias se completam e posso encarar a luz como onda
ou corpúsculo consoante a sua interação com o meio (objecto) Ambas
transportam energia. Quer a masa quer a energia estão quantizadas,
os tais pacotes de energia.
Os fotões de maior energia são os de maior frequência e estão neste
caso os da raios UV, radiação X e Raios cósmicos que têm poder
ionizante e penetrante elevados.
Um abarço
Maria Natália

--- Em ciencialist@yahoogrupos.com.br, "Cyberlander"
<cybernews@s...> escreveu
>
>
> fótons são pacotinhos com massa ou picos de onda?
> [ ]'s
> D.C.
>
>
>
> --- Em ciencialist@yahoogrupos.com.br, José Renato <jrma@t...>
> escreveu
> > Hélio, o meu e acredito que também o seu energia-campo é
composto
> de
> > fenômenos que denominamos de energia e dos que chamamos de
> campo! :-))
> > Tensão no espaço são provocadas pelas forças nucleares, forte e
> fraca,
> > magnetismo e gravidade. Exemplo: o átomo é um conjunto material
> vazio, mais
> > espaço que matéria, porém devido às "tensões" existentes no
> núcleo,
> > interação das subpartículas, nos elétrons - emissores e
receptores
> de quanta
> > e fótons mas que ainda é considerado uma partícula, digamos,
> > ndivisível(?) - e entre o núcleo e os elétrons parece ter uma
> consistência
> > material "sólida" devido a essa estrutura de tensões.
> > []s
> > José Renato
> > .......................................................
> >
> > From: "Hélio Ricardo Carvalho" <hrc@f...>
> > To: <ciencialist@yahoogrupos.com.br>
> > Sent: Wednesday, December 22, 2004 7:11 PM
> > Subject: [ciencialist] Re: Tudo é energia
> >
> > Este teu "energia-campo" seria composto de que? ... :-)
> >
> > "Espaço sem maiores tensões"???
> > O que seria tensão no espaço?
> >
> > Hoje estou perguntador do tipo do "marcelo ferrari", mas é só por
> > agora.
> >
> > [ ]'s
> > Hélio
> >
> >
> >
> > --- Em ciencialist@yahoogrupos.com.br, José Renato <jrma@t...>
> > escreveu
> > > Hélio, percebo a matéria como energia-campos "estruturados" em
> > determinada
> > > configuração. Já o espaço, mesmo pleno de energia e campos,
> > considero como
> > > "não estruturado", sem maiores tensões.
> >
> >
> >
> >
> >
> >
> > ##### ##### #####
> >
> > Para saber mais visite
> > http://www.ciencialist.hpg.ig.com.br
> >
> >
> > ##### ##### ##### #####
> > Links do Yahoo! Grupos
> >
> >
> >
> >
> >
> >
> >
> >
> >
> > Esta mensagem foi verificada pelo E-mail Protegido Terra.
> > Scan engine: McAfee VirusScan / Atualizado em 22/12/2004 /
Versão:
> 4.4.00 -
> > Dat 4416
> > Proteja o seu e-mail Terra:
http://www.emailprotegido.terra.com.br/
> >
> > E-mail classificado pelo Identificador de Spam Inteligente Terra.
> > Para alterar a categoria classificada, visite
> >
>
http://www.terra.com.br/centralunificada/emailprotegido/imail/imail.c
> gi?
>
+_u=jrma&_l=1,1103753533.708756.20502.cabue.terra.com.br,3444,Des15,D
> es15





SUBJECT: Re: Galáxias na galáxia...
FROM: "Rodrigo Marques" <rodmarq72@yahoo.com.br>
TO: ciencialist@yahoogrupos.com.br
DATE: 23/12/2004 08:07


Pelo que eu sei existiam duas, a pequena e a grande nuvem de
Magalhães...e parecia que a Via-Láctea estava "engolindo" pelo menos
uma delas, mas não posso dar certeza...
--- Em ciencialist@yahoogrupos.com.br, José Renato <jrma@t...>
escreveu
> Afinal, como é que pode se afirmar que foram "descobertas" 30
galáxias nas bordas de uma galáxia maior: a Via Lacta?
> Abraços
> José Renato
>
> [As partes desta mensagem que não continham texto foram removidas]





SUBJECT: Boas Festas
FROM: "Sandra Rosario" <coffeacruda@hotmail.com>
TO: ciencialist@yahoogrupos.com.br
DATE: 23/12/2004 17:58


Feliz Natal e um maravilhoso 2005 para todos do grupo ciencialist.
Abraços
Sandra Rosario

_________________________________________________________________
MSN Messenger: converse online com seus amigos .
http://messenger.msn.com.br



SUBJECT: Re: [ciencialist] Re: Votos de feliz natal e próspero an o novo/Bêbados e álcool
FROM: "JVictor" <jvoneto@uol.com.br>
TO: <ciencialist@yahoogrupos.com.br>
DATE: 23/12/2004 19:08


Caros amigos do Ciencialist:
Desejo a todos um feliz Natal. Se durante algumas intervenções minhas alguém ficou chateado, ou se sentiu de alguma maneira agredido, peço desculpas; se assim foi, digo que não houve a intenção.
Se alguém não me perdoar, farei queixas a papai Noé.

Feliz Natal a todos, de coração.

Victor.


##### ##### ##### #####



Yahoo! Grupos, um serviço oferecido por:

São Paulo Rio de Janeiro Curitiba Porto Alegre Belo Horizonte Brasília




------------------------------------------------------------------------------
Links do Yahoo! Grupos

a.. Para visitar o site do seu grupo na web, acesse:
http://br.groups.yahoo.com/group/ciencialist/

b.. Para sair deste grupo, envie um e-mail para:
ciencialist-unsubscribe@yahoogrupos.com.br

c.. O uso que você faz do Yahoo! Grupos está sujeito aos Termos do Serviço do Yahoo!.



[As partes desta mensagem que não continham texto foram removidas]



SUBJECT: Fw: Fisica e Plantas Medicinais
FROM: "Luiz Ferraz Netto" <leobarretos@uol.com.br>
TO: "ciencialist" <ciencialist@yahoogrupos.com.br>
DATE: 23/12/2004 19:27

não sei.

[]'
===========================
Luiz Ferraz Netto [Léo]
leobarretos@uol.com.br
http://www.feiradeciencias.com.br
===========================
-----Mensagem Original-----
De: BelPiva
Para: leobarretos@uol.com.br
Enviada em: quinta-feira, 2 de dezembro de 2004 12:29
Assunto: Fisica e Plantas Medicinais


Ola
Gostaria de saber em quais, ou quais conteudos de Fisica
posso associar Plantas Medicinais.

Osmar B Mende

e-mail osmarbmendes@bol.com.br

[As partes desta mensagem que não continham texto foram removidas]



SUBJECT: SUGESTÃO DE LINK
FROM: "Cyberlander" <cybernews@superig.com.br>
TO: <Undisclosed-Recipient:;>
DATE: 23/12/2004 19:32

link para site introdutório à FILOSOFIA DA CIÊNCIA ...


http://www.cfh.ufsc.br/~wfil/aires.htm

[ ]'S
D.C.

[As partes desta mensagem que não continham texto foram removidas]



SUBJECT: Re: [ciencialist] Novo site inacreditável
FROM: "JVictor" <jvoneto@uol.com.br>
TO: <ciencialist@yahoogrupos.com.br>
DATE: 23/12/2004 20:07

Rodrigo,

O pessoal site está de brincadeira, prá zombar dos outros, ou o quê?

Victor.

----- Original Message -----
From: Rodrigo Marques
To: Ceticismo Aberto ; CienciaList ; Sociedade Brasileira de Céticos e Racionalistas ; Sociedade da Terra redonda
Sent: Tuesday, December 21, 2004 8:00 AM
Subject: [ciencialist] Novo site inacreditável


Pessoal vejam só este site:
http://www.vivendodaluz.com/
Que gosto será que a luz tem???


---------------------------------
Yahoo! Acesso Grátis - Internet rápida e grátis. Instale o discador do Yahoo! agora.

[As partes desta mensagem que não continham texto foram removidas]



##### ##### #####

Para saber mais visite
http://www.ciencialist.hpg.ig.com.br


##### ##### ##### #####


Yahoo! Grupos, um serviço oferecido por:
PUBLICIDADE




------------------------------------------------------------------------------
Links do Yahoo! Grupos

a.. Para visitar o site do seu grupo na web, acesse:
http://br.groups.yahoo.com/group/ciencialist/

b.. Para sair deste grupo, envie um e-mail para:
ciencialist-unsubscribe@yahoogrupos.com.br

c.. O uso que você faz do Yahoo! Grupos está sujeito aos Termos do Serviço do Yahoo!.



[As partes desta mensagem que não continham texto foram removidas]



SUBJECT: ciência - O que é epistemologia ?
FROM: "E m i l i a n o C h e m e l l o" <chemelloe@yahoo.com.br>
TO: "gmail" <chemello@gmail.com>, <ciencialist@yahoogrupos.com.br>, <naeq-ucs@yahoogrupos.com.br>, <quimica-qaw@yahoogrupos.com.br>
DATE: 24/12/2004 08:46

O que é Epistemologia ?
http://www.terravista.pt/FerNoronha/2265/index.htm
Sojia, Filosofia na Web

O termo significa "estudo da ciência" (do grego episthme = conhecimento,
ciência, e logoV = estudo, discurso). É usada em dois sentidos: para indicar
o estudo da origem e do valor do conhecimento humano em geral (e neste
sentido é sinônimo de gnosiologia ou crítica); ou para significar o estudo
as ciências (físicas e humanas), dos princípios sobre o qual se fundam, dos
critérios de verificação e de verdade, do valor dos sistemas científicos.
Pode-se dividi-la em dois sentidos básicos:

a) a crítica do conhecimento científico: exame dos princípios, das hipóteses
e das conclusões das diferentes ciências, tendo em vista determinar seu
alcance e seu valor objetivo.
b) a filosofia da ciência (empirismo, racionalismo, etc), e a história do
desenvolvimento científico.


Breve Prospecto Histórico

Desde que Comte negou à filosofia um domínio próprio de objetos e
confiou-lhe como tarefa específica o estudo das ciências, a determinação de
seus objetos e de suas tarefas, a sua divisão e coordenação, a atenção dos
filósofos dirigiu-se sempre mais para a ciência, a qual se tornou, para
muitos, o argumento principal e central de sua análise. Além disso, a
indagação atenta e aprofundada das características e das funções do saber
científico era exigida quer pela orientação positiva da filosofia, quer
pelos enormes desenvolvimentos e pela extraordinária importância que a
ciência havia adquirido durante os últimos dois séculos, período no qual ela
demonstrou ser um saber extremamente fecundo e prático.

Essa instâncias foram o ponto de partida de uma parte da filosofia, chamada
filosofia da ciência, ou epistemologia . Esta se identifica com a crítica
metodológica da ciência, na medida em que essa crítica tende à explicitação
consciente e sistemática do método e das condições de validade dos juízos -
particulares, singulares ou universais - tornados próprios pelos cientistas,
perseguindo assim uma reconstrução racional, convencionalmente designada por
senso empírico-pragmático, do conceito de conhecimento científico.

A epistemologia propõe-se a responder às seguintes questões: o que é
conhecimento científico? Em outras palavras, em que consiste propriamente o
trabalho do cientista? Que faz ele quando faz ciência? Interpreta, descreve,
explica, prevê? Faz apenas conjecturas ou verdadeiras asserções (gerais e
singulares) que espelham fielmente os aspectos dos fatos? E quando o
cientista explica, o que é que ele explica dos fatos: sua função, origem,
gênese, essência, fim? Qual é o status lógico das leis na ciência? São elas
resultados de procedimentos indutivos (e o que quer dizer indução para a
ciência?), ou antes, conjecturas da imaginação científica que deverão
sujeitar-se a provas empíricas? Em que sentido se fala em causalidade nas
ciências empíricas? Quando, então podemos dizer que uma teoria é "melhor" do
que outra? O que queremos dizer quando afirmamos que as ciências empíricas
são objetivas? Qual é o papel da experiência na pesquisa científica?
Podemos observar que na epistemologia existem mais perguntas que respostas.
Tais perguntas brotam da pergunta inicial sobre o que seja o conhecimento
científico.

Essas questões começaram a impor-se à atenção dos filósofos pelo fim do
século XVIII, no momento em que a atitude de confiança otimista e exaltação
cega das ciências foi substituída por um ceticismo e uma crítica aguda nos
confrontos do conhecimento científico. O nascimento e desenvolvimento da
filosofia da ciência deve-se diretamente à tomada de consciência da
problematicidade desse conhecimento. Tal consciência era ainda ausente em
Descartes, Newton, Kant, Comte, e Spencer.

Os primeiros resultados significativos dessa nova disciplina dizem respeito
à matemática e à geometria. Estas não são mais concebidas como ciências
reais, como representações de situações objetivas, mas sim como construções
formais: como sistemas fundados em postulados escolhidos arbitrariamente e
construídos com técnica da dedução lógica das conseqüências que comportam
tais postulados. Assim, por ação dos epistemólogos e outros estudiosos, a
matemática e a geometria tomaram consciência de sua especificidade como
ciência do possível, diferente da física, que ao contrário, é a ciência do
real.

No concernente à física e às ciências experimentais em geral, passa-se de
uma visão estática e mecanicista a uma visão dinâmica, probabilista e
relativista das leis da natureza. Essa mudança foi motivada pelas
descobertas da entropia, da radiatividade, dos quanta , etc.

Conseqüentemente, os conceitos de um espaço, e de um tempo absolutos, como
também os de simultaneidade, perderam todo o valor. A idéia de espaço curvo
toma o lugar da idéia euclidiana de espaço retilíneo. A idéia de relações
necessárias de causalidade é substituída pela idéia de indeterminação.

Nas ciências da natureza, no início do século XIX, ressalta-se uma série de
questões filosóficas relativas ao caráter e à função do conhecimento
experimental. As ciências naturais não aparecem mais no campo do saber como
conhecimento absoluto com pretensões imperialistas e limites próprios. Seu
âmbito é a quantidade. De tal sorte a física ganha um perfil matemático,
relegando a segundo plano as intenções ontológicas e os elementos sensíveis.

Daí a tendência a reduzir o conhecimento experimental a puros dados métricos
e ao esquema relacional desses dados. Tal esforço de quantificação e
matematização da física acentua os traços que a distinguem tanto do
conhecimento comum, quanto do filosófico.

A filosofia da ciência propriamente dita teve um considerável
desenvolvimento em nosso século, dando origem a três movimentos principais:
o neopositivismo, a interpretação metafísica, e o racionalismo científico.

Os defensores mais qualificados dos neopositivistas são Wittgenstein,
Carnap, e Russel. Os neopositivistas dividem as ciências em dois grandes
ramos: as lógico-matemáticas, e as experimentais. As primeiras são
constituídas por proposições analíticas, ou seja, tautológicas; as segundas
são compostas por proposições factuais. As proposições lógicas e
matemáticas, destituídas de conteúdo, não são mais do que regras para a
utilização dos símbolos e a ordenação das proposições. As experimentais ou
factuais são as empiricamente verificáveis: isto acontece se elas são
traduzíveis em proposições de caráter empírico.

Em contraste radical com o neopositivismo coloca-se a concepção metafísica
da ciência. Esta afirma que a ciência envolve uma metafísica e somente nela
encontra seu fundamento último. Conforme esta concepção, o trabalho
científico apresenta-se como descoberta progressiva da realidade, ou como a
auto-manifestação do espírito humano através da pesquisa científica. No
primeiro caso, refere-se a uma concepção metafísica realista; no segundo, a
uma concepção metafísica idealista.

Um dos maiores expoentes do realismo metafísico é Émile Meyerson
(1859-1933), o qual afirma que a ciência "não é positiva e não contém mesmo
dados positivos, no sentido rigoroso que foi dado a este termo por Comte e
seus seguidores, ou seja, dados desprovidos de qualquer ontologia. A
ontologia faz parte da própria ciência e dela não pode ser separada". É o
realismo do senso comum, segundo Meyerson, que se prolonga na ciência sem
solução de continuidade. A ciência, progredindo na direção do senso comum,
cria essências, cujo caráter real não somente não é eliminado, mas é
intensificado.

Já na interpretação metafísica idealista da ciência, sustenta-se que a
subjetividade é um fator importante na pesquisa cientifica. Nesta
interpretação destacam-se as "leis epistemológicas". Sua característica
peculiar é serem dedutíveis unicamente através do estudo de nossos métodos
de observação. Essas leis necessárias, universais, e exatas constituem o
elemento a priori da física, e das outras ciências experimentais.

Segundo outro grande grupo de autores, a ciência é obra da razão humana, uma
espécie de máquina gerada por ela, cujas estruturas e leis internas é
preciso descobrir. Enquanto o interesse da interpretação metafísica
dirigia-se à infra-estrutura ontológica da ciência, e o do neopositivismo a
seus conteúdos como tais, tomados em seu grau máximo de cristalização
objetiva, o esforço do racionalismo científico, por sua vez, tende a
clarificar o sentido do opus rationale que constitui a ciência.

O principal expoente desta interpretação epistemológica é Gaston Bachelard
(1844-1962), para quem a filosofia da ciência contemporânea não pode aceitar
nem a solução realista, nem a idealista. Segundo ele, deve colocar-se num
meio termo entre ambos, no qual sejam retomados e superados: "Um realismo
que se deparou com a dúvida científica não pode mais ser do mesmo teor que o
realismo imediato... um racionalismo que retificou os juízos a priori, como
sucedeu nos novos ramos da geometria, não pode mais ser um racionalismo
fechado" .

Bachelard em sua gnosiologia, põe o binômio experiência-razão na base de
todo o conhecimento humano. Entretanto, não se trata de um condomínio de
potências iguais, pois o elemento teórico é que desempenha o papel
normativo: "O sentido de setor epistemológico parece-nos bastante claro. Ele
vai certamente do racional para o real, e não na ordem inversa, do real do
geral, como professaram todos os filósofos, de Aristóteles a Bacon" .

Posição análoga à de Bachelard é a sustentada por Karl Popper (1902 -) que
também rejeita decididamente o empirismo em nome de uma certa espécie de
racionalismo . O controle das teorias, a corroboração das proposições
científicas, segundo Popper, não é obtida diretamente, como querem os
neopositivistas, recorrendo à verificação experimental, mas sim
indiretamente, através do processo de falsicabilidade . Este critério
estabelece que uma teoria pode ser considerada científica unicamente se
satisfaz a duas condições:

- ser falsificável, ou seja, poder vir a ser desmentida e contradita em
linha de princípio;
- não ter sido ainda provada como falsa de fato.

O critério do estágio científico de uma teoria é a sua "falsicabilidade", ou
"refutabilidade", ou seja a sua controlabilidade. O critério de demarcação
entre teorias empíricas e não empíricas, não é a verificabilidade, mas sim
sua falsicabilidade. Com efeito, uma lei científica jamais poderá ser
inteiramente confirmada, ao passo que pode ser totalmente falsificada.

O lógico na construção da ciência são os problemas, e com eles, as
hipóteses, as conjecturas, e não as observações. Observamos através de um
ponto de vista, sempre sob o estímulo de um problema. Todos os conhecimentos
são respostas a problemas prévios. Adquirimos os conhecimentos que se
prestam para solucionar nossas interrogações, nossos problemas. Por isso, as
teorias científicas não são cúmulos de observações, mas sistemas de
conjecturas arriscadas e temerárias. Antes de tudo, ciência é invenção de
hipóteses ; a experiência desempenha um papel de controle das teorias.

Percebe-se assim que na epistemologia, a razão humana ainda não conseguiu
chegar a uma solução satisfatória e definitiva, com a qual todos possam
concordar. Mesmo na filosofia da ciência, recolocam-se as alternativas
clássicas: idealismo ou realismo? Racionalismo ou positivismo?

Nessa situação poderíamos ser tentados a abandonar o ambiente de pesquisa
filosófica. Esta, entretanto, não é a melhor decisão, pois o homem é dotado
de razão para procurar a razão das coisas, ou seja, para encontrar uma
explicação profunda, geral, exaustiva, uma explicação filosófica. Assim,
sobre todos os aspectos da realidade, e sobre todos os setores do conhecer e
do agir, será preciso continuar a filosofar. E mesmo no futuro serão obtidos
resultados alternativos, como no passado.

Bibliografia
JOLIVET, R. Curso de Filosofia. Agir, S.Paulo: 1986.
KANT, I. Crítica da Razão Pura. Coleção "Os Pensadores". Nova Cultural,
S.Paulo: 1996.
MONDIN, B. Introdução à Filosofia. Paulinas, S.Paulo: 1987.
MORA, J.F. Dicionário de Filosofia. Martins Fontes, S.Paulo: 1994.
REALE, G. História da Filosofia. Vol. I, II, III. Paulus, S.Paulo: 1991.
TOBIAS, J.A. Iniciação à Filosofia. Unoeste, Pres.Prudente: 1986.


[ ] 's do Emiliano Chemello



SUBJECT: Bons votos!
FROM: "israelmg2003" <israel-cordeiro@ig.com.br>
TO: ciencialist@yahoogrupos.com.br
DATE: 24/12/2004 08:58


Um feliz natal e um feliz e próspero 2005 para todos, que em 2005 em
primeiro lugar a PAZ esteja conosco.

Israel





SUBJECT: off topic - teste p/novos moderadores
FROM: "murilo filo" <avalanchedrive@hotmail.com>
TO: ciencialist@yahoogrupos.com.br, forum-ciencia@yahoogrupos.com.br
CC: avalanchedrive@hotmail.com
DATE: 24/12/2004 16:56

Oi, lista...
todos aí?
Há coisas que, mesmo eu, sempre ligado nelas, ainda procuro entender e
descobrir.
Um exemplo? Natal!
Tem que ver com aquela nossa parte que, queiramos ou não, está aí, negada,
esquecida, mas ainda muito presente em nós e/ou em nossas casas. Um
mistério, queiramos ou não!
Somos uma alma que tem um corpo, muito mais que um corpo com uma alma, e a
humanidade brutal, milagrosamente, ainda consegue manter momentos coletivos
que lembram êste fato básico.
Somos alma, viva, refinada, sutíl, forte, inspiradora, criadora, superior,
enorme e... grátis!
Para ser mais maravilhosa ainda, basta sair, estar mais livre, nem que seja
um pouquinho, desta grosseira e incompleta prisão mental, algo, por sua vez,
derivado daquêle corpo que é da alma, pristina e colaborativa.
Que pelo menos como consolo, ou como indício, flua hoje em todos nós outra
coisinha que também não existiria sem nossa alma viva, as emoções, sejam
elas quais forem. Nem vou falar de amôr!
No Natal comemora-se o nascimento de uma das maiores almas, comemoremos
também as nossas; as de cada um, um presente!
Bom Natal para todos! Abração sincero. Murilo SP 24/dez./2004




SUBJECT: RE: [ciencialist] ciência - O que é epistemologia ?
FROM: "murilo filo" <avalanchedrive@hotmail.com>
TO: ciencialist@yahoogrupos.com.br
DATE: 24/12/2004 17:05

Gente, êste texto é uma jóia.
Emiliano: P A R A B E N S!

>From: "E m i l i a n o C h e m e l l o" <chemelloe@yahoo.com.br>
>Reply-To: ciencialist@yahoogrupos.com.br
>To: "gmail"
><chemello@gmail.com>,<ciencialist@yahoogrupos.com.br>,<naeq-ucs@yahoogrupos.com.br>,<quimica-qaw@yahoogrupos.com.br>
>Subject: [ciencialist] ciência - O que é epistemologia ?
>Date: Fri, 24 Dec 2004 08:46:19 -0200
>
>O que é Epistemologia ?
>http://www.terravista.pt/FerNoronha/2265/index.htm
>Sojia, Filosofia na Web
>
>O termo significa "estudo da ciência" (do grego episthme = conhecimento,
>ciência, e logoV = estudo, discurso). É usada em dois sentidos: para
>indicar
>o estudo da origem e do valor do conhecimento humano em geral (e neste
>sentido é sinônimo de gnosiologia ou crítica); ou para significar o estudo
>as ciências (físicas e humanas), dos princípios sobre o qual se fundam, dos
>critérios de verificação e de verdade, do valor dos sistemas científicos.
>Pode-se dividi-la em dois sentidos básicos:
>
>a) a crítica do conhecimento científico: exame dos princípios, das
>hipóteses
>e das conclusões das diferentes ciências, tendo em vista determinar seu
>alcance e seu valor objetivo.
>b) a filosofia da ciência (empirismo, racionalismo, etc), e a história do
>desenvolvimento científico.
>
>
>Breve Prospecto Histórico
>
>Desde que Comte negou à filosofia um domínio próprio de objetos e
>confiou-lhe como tarefa específica o estudo das ciências, a determinação de
>seus objetos e de suas tarefas, a sua divisão e coordenação, a atenção dos
>filósofos dirigiu-se sempre mais para a ciência, a qual se tornou, para
>muitos, o argumento principal e central de sua análise. Além disso, a
>indagação atenta e aprofundada das características e das funções do saber
>científico era exigida quer pela orientação positiva da filosofia, quer
>pelos enormes desenvolvimentos e pela extraordinária importância que a
>ciência havia adquirido durante os últimos dois séculos, período no qual
>ela
>demonstrou ser um saber extremamente fecundo e prático.
>
>Essa instâncias foram o ponto de partida de uma parte da filosofia, chamada
>filosofia da ciência, ou epistemologia . Esta se identifica com a crítica
>metodológica da ciência, na medida em que essa crítica tende à explicitação
>consciente e sistemática do método e das condições de validade dos juízos -
>particulares, singulares ou universais - tornados próprios pelos
>cientistas,
>perseguindo assim uma reconstrução racional, convencionalmente designada
>por
>senso empírico-pragmático, do conceito de conhecimento científico.
>
>A epistemologia propõe-se a responder às seguintes questões: o que é
>conhecimento científico? Em outras palavras, em que consiste propriamente o
>trabalho do cientista? Que faz ele quando faz ciência? Interpreta,
>descreve,
>explica, prevê? Faz apenas conjecturas ou verdadeiras asserções (gerais e
>singulares) que espelham fielmente os aspectos dos fatos? E quando o
>cientista explica, o que é que ele explica dos fatos: sua função, origem,
>gênese, essência, fim? Qual é o status lógico das leis na ciência? São elas
>resultados de procedimentos indutivos (e o que quer dizer indução para a
>ciência?), ou antes, conjecturas da imaginação científica que deverão
>sujeitar-se a provas empíricas? Em que sentido se fala em causalidade nas
>ciências empíricas? Quando, então podemos dizer que uma teoria é "melhor"
>do
>que outra? O que queremos dizer quando afirmamos que as ciências empíricas
>são objetivas? Qual é o papel da experiência na pesquisa científica?
>Podemos observar que na epistemologia existem mais perguntas que respostas.
>Tais perguntas brotam da pergunta inicial sobre o que seja o conhecimento
>científico.
>
>Essas questões começaram a impor-se à atenção dos filósofos pelo fim do
>século XVIII, no momento em que a atitude de confiança otimista e exaltação
>cega das ciências foi substituída por um ceticismo e uma crítica aguda nos
>confrontos do conhecimento científico. O nascimento e desenvolvimento da
>filosofia da ciência deve-se diretamente à tomada de consciência da
>problematicidade desse conhecimento. Tal consciência era ainda ausente em
>Descartes, Newton, Kant, Comte, e Spencer.
>
>Os primeiros resultados significativos dessa nova disciplina dizem respeito
>à matemática e à geometria. Estas não são mais concebidas como ciências
>reais, como representações de situações objetivas, mas sim como construções
>formais: como sistemas fundados em postulados escolhidos arbitrariamente e
>construídos com técnica da dedução lógica das conseqüências que comportam
>tais postulados. Assim, por ação dos epistemólogos e outros estudiosos, a
>matemática e a geometria tomaram consciência de sua especificidade como
>ciência do possível, diferente da física, que ao contrário, é a ciência do
>real.
>
>No concernente à física e às ciências experimentais em geral, passa-se de
>uma visão estática e mecanicista a uma visão dinâmica, probabilista e
>relativista das leis da natureza. Essa mudança foi motivada pelas
>descobertas da entropia, da radiatividade, dos quanta , etc.
>
>Conseqüentemente, os conceitos de um espaço, e de um tempo absolutos, como
>também os de simultaneidade, perderam todo o valor. A idéia de espaço curvo
>toma o lugar da idéia euclidiana de espaço retilíneo. A idéia de relações
>necessárias de causalidade é substituída pela idéia de indeterminação.
>
>Nas ciências da natureza, no início do século XIX, ressalta-se uma série de
>questões filosóficas relativas ao caráter e à função do conhecimento
>experimental. As ciências naturais não aparecem mais no campo do saber como
>conhecimento absoluto com pretensões imperialistas e limites próprios. Seu
>âmbito é a quantidade. De tal sorte a física ganha um perfil matemático,
>relegando a segundo plano as intenções ontológicas e os elementos
>sensíveis.
>
>Daí a tendência a reduzir o conhecimento experimental a puros dados
>métricos
>e ao esquema relacional desses dados. Tal esforço de quantificação e
>matematização da física acentua os traços que a distinguem tanto do
>conhecimento comum, quanto do filosófico.
>
>A filosofia da ciência propriamente dita teve um considerável
>desenvolvimento em nosso século, dando origem a três movimentos principais:
>o neopositivismo, a interpretação metafísica, e o racionalismo científico.
>
>Os defensores mais qualificados dos neopositivistas são Wittgenstein,
>Carnap, e Russel. Os neopositivistas dividem as ciências em dois grandes
>ramos: as lógico-matemáticas, e as experimentais. As primeiras são
>constituídas por proposições analíticas, ou seja, tautológicas; as segundas
>são compostas por proposições factuais. As proposições lógicas e
>matemáticas, destituídas de conteúdo, não são mais do que regras para a
>utilização dos símbolos e a ordenação das proposições. As experimentais ou
>factuais são as empiricamente verificáveis: isto acontece se elas são
>traduzíveis em proposições de caráter empírico.
>
>Em contraste radical com o neopositivismo coloca-se a concepção metafísica
>da ciência. Esta afirma que a ciência envolve uma metafísica e somente nela
>encontra seu fundamento último. Conforme esta concepção, o trabalho
>científico apresenta-se como descoberta progressiva da realidade, ou como a
>auto-manifestação do espírito humano através da pesquisa científica. No
>primeiro caso, refere-se a uma concepção metafísica realista; no segundo, a
>uma concepção metafísica idealista.
>
>Um dos maiores expoentes do realismo metafísico é Émile Meyerson
>(1859-1933), o qual afirma que a ciência "não é positiva e não contém mesmo
>dados positivos, no sentido rigoroso que foi dado a este termo por Comte e
>seus seguidores, ou seja, dados desprovidos de qualquer ontologia. A
>ontologia faz parte da própria ciência e dela não pode ser separada". É o
>realismo do senso comum, segundo Meyerson, que se prolonga na ciência sem
>solução de continuidade. A ciência, progredindo na direção do senso comum,
>cria essências, cujo caráter real não somente não é eliminado, mas é
>intensificado.
>
>Já na interpretação metafísica idealista da ciência, sustenta-se que a
>subjetividade é um fator importante na pesquisa cientifica. Nesta
>interpretação destacam-se as "leis epistemológicas". Sua característica
>peculiar é serem dedutíveis unicamente através do estudo de nossos métodos
>de observação. Essas leis necessárias, universais, e exatas constituem o
>elemento a priori da física, e das outras ciências experimentais.
>
>Segundo outro grande grupo de autores, a ciência é obra da razão humana,
>uma
>espécie de máquina gerada por ela, cujas estruturas e leis internas é
>preciso descobrir. Enquanto o interesse da interpretação metafísica
>dirigia-se à infra-estrutura ontológica da ciência, e o do neopositivismo a
>seus conteúdos como tais, tomados em seu grau máximo de cristalização
>objetiva, o esforço do racionalismo científico, por sua vez, tende a
>clarificar o sentido do opus rationale que constitui a ciência.
>
>O principal expoente desta interpretação epistemológica é Gaston Bachelard
>(1844-1962), para quem a filosofia da ciência contemporânea não pode
>aceitar
>nem a solução realista, nem a idealista. Segundo ele, deve colocar-se num
>meio termo entre ambos, no qual sejam retomados e superados: "Um realismo
>que se deparou com a dúvida científica não pode mais ser do mesmo teor que
>o
>realismo imediato... um racionalismo que retificou os juízos a priori, como
>sucedeu nos novos ramos da geometria, não pode mais ser um racionalismo
>fechado" .
>
>Bachelard em sua gnosiologia, põe o binômio experiência-razão na base de
>todo o conhecimento humano. Entretanto, não se trata de um condomínio de
>potências iguais, pois o elemento teórico é que desempenha o papel
>normativo: "O sentido de setor epistemológico parece-nos bastante claro.
>Ele
>vai certamente do racional para o real, e não na ordem inversa, do real do
>geral, como professaram todos os filósofos, de Aristóteles a Bacon" .
>
>Posição análoga à de Bachelard é a sustentada por Karl Popper (1902 -) que
>também rejeita decididamente o empirismo em nome de uma certa espécie de
>racionalismo . O controle das teorias, a corroboração das proposições
>científicas, segundo Popper, não é obtida diretamente, como querem os
>neopositivistas, recorrendo à verificação experimental, mas sim
>indiretamente, através do processo de falsicabilidade . Este critério
>estabelece que uma teoria pode ser considerada científica unicamente se
>satisfaz a duas condições:
>
>- ser falsificável, ou seja, poder vir a ser desmentida e contradita em
>linha de princípio;
>- não ter sido ainda provada como falsa de fato.
>
>O critério do estágio científico de uma teoria é a sua "falsicabilidade",
>ou
>"refutabilidade", ou seja a sua controlabilidade. O critério de demarcação
>entre teorias empíricas e não empíricas, não é a verificabilidade, mas sim
>sua falsicabilidade. Com efeito, uma lei científica jamais poderá ser
>inteiramente confirmada, ao passo que pode ser totalmente falsificada.
>
>O lógico na construção da ciência são os problemas, e com eles, as
>hipóteses, as conjecturas, e não as observações. Observamos através de um
>ponto de vista, sempre sob o estímulo de um problema. Todos os
>conhecimentos
>são respostas a problemas prévios. Adquirimos os conhecimentos que se
>prestam para solucionar nossas interrogações, nossos problemas. Por isso,
>as
>teorias científicas não são cúmulos de observações, mas sistemas de
>conjecturas arriscadas e temerárias. Antes de tudo, ciência é invenção de
>hipóteses ; a experiência desempenha um papel de controle das teorias.
>
>Percebe-se assim que na epistemologia, a razão humana ainda não conseguiu
>chegar a uma solução satisfatória e definitiva, com a qual todos possam
>concordar. Mesmo na filosofia da ciência, recolocam-se as alternativas
>clássicas: idealismo ou realismo? Racionalismo ou positivismo?
>
>Nessa situação poderíamos ser tentados a abandonar o ambiente de pesquisa
>filosófica. Esta, entretanto, não é a melhor decisão, pois o homem é dotado
>de razão para procurar a razão das coisas, ou seja, para encontrar uma
>explicação profunda, geral, exaustiva, uma explicação filosófica. Assim,
>sobre todos os aspectos da realidade, e sobre todos os setores do conhecer
>e
>do agir, será preciso continuar a filosofar. E mesmo no futuro serão
>obtidos
>resultados alternativos, como no passado.
>
>Bibliografia
>JOLIVET, R. Curso de Filosofia. Agir, S.Paulo: 1986.
>KANT, I. Crítica da Razão Pura. Coleção "Os Pensadores". Nova Cultural,
>S.Paulo: 1996.
>MONDIN, B. Introdução à Filosofia. Paulinas, S.Paulo: 1987.
>MORA, J.F. Dicionário de Filosofia. Martins Fontes, S.Paulo: 1994.
>REALE, G. História da Filosofia. Vol. I, II, III. Paulus, S.Paulo: 1991.
>TOBIAS, J.A. Iniciação à Filosofia. Unoeste, Pres.Prudente: 1986.
>
>
>[ ] 's do Emiliano Chemello
>




SUBJECT: APROVAR -- oanjobranco
FROM: "Luiz Ferraz Netto" <leobarretos@uol.com.br>
TO: "ciencialist" <ciencialist@yahoogrupos.com.br>
DATE: 25/12/2004 07:30

Recebi 'incumbência' de aprovar/desaprovar "O anjo branco", via Yahoo!. Ao tentar fazer isso verifiquei que o bom senso dos demais moderadores já me haviam antecipado, aprovando o novo 'C-listman'.
A ele meus votos de boa permanência e que nos brinde com um pouco, do muito que nos falta, em termos de conhecimento científico.
Minha recomendação --- apresente-se ao grupo.

aquele abraço,
===========================
Luiz Ferraz Netto [Léo]
leobarretos@uol.com.br
http://www.feiradeciencias.com.br
===========================

-----Mensagem Original-----
De: "Aviso do Yahoo!Grupos" <ciencialist-acceptsub-tlzfxFVrhH8BWE18k4wwWd8@yahoogrupos.com.br>
Para: <ciencialist-owner@yahoogrupos.com.br>
Enviada em: sexta-feira, 24 de dezembro de 2004 10:43
Assunto: APROVAR -- oanjobranco <machado_jr@terra.com.br> gostaria de entrar no grupo ciencialist




SUBJECT: Re: [ciencialist] off topic - teste p/novos moderadores
FROM: "Luiz Ferraz Netto" <leobarretos@uol.com.br>
TO: <ciencialist@yahoogrupos.com.br>, <forum-ciencia@yahoogrupos.com.br>
CC: <avalanchedrive@hotmail.com>
DATE: 25/12/2004 09:24

Nesse natal gastei 867,80 R$ em presentes (esse moderador está incluso entre os presenteados --- 'ganhei' uma rotomatic, 10 000 rpm. Esse $$ todo aliviou-me bastante a 'alma'. Assim sendo, e mesmo já fora do prazo, BOM NATAL a todos(as).
aquele abraço,
===========================
Luiz Ferraz Netto [Léo]
leobarretos@uol.com.br
http://www.feiradeciencias.com.br
===========================
-----Mensagem Original-----
De: "murilo filo" <avalanchedrive@hotmail.com>
Para: <ciencialist@yahoogrupos.com.br>; <forum-ciencia@yahoogrupos.com.br>
Cc: <avalanchedrive@hotmail.com>
Enviada em: sexta-feira, 24 de dezembro de 2004 16:56
Assunto: [ciencialist] off topic - teste p/novos moderadores



Oi, lista...
todos aí?
Há coisas que, mesmo eu, sempre ligado nelas, ainda procuro entender e
descobrir.
Um exemplo? Natal!



SUBJECT: Re: Pedido [administrador]
FROM: Manuel Bulcão <manuelbulcao@uol.com.br>
TO: ciencialist@yahoogrupos.com.br
DATE: 25/12/2004 16:08


--- Em ciencialist@yahoogrupos.com.br, "brudna" <lrb@i...> escreveu

>Vamos cuidar para que mensagens nao sejam mal interpretadas. Para
isso acho melhor evitar a agressividade. Ela pode ocorrer entre duas
pessoas que conhecem o ritmo da brincadeira (Takata e Bulcao, por
exemplo), mas vai acabar confundindo quem nao conhece a interacao
entre as duas pessoas, julgando que eh uma briga real.

Manuel: Como disse o L.E.R., essa lista é uma lista de ciência.
Ocorre que ela é antiga, de modo que os seus membros --
principalmente os da velha guarda -- são também amigos. Ora, entre
amigos as brincadeiras são algo corrente, e não há nada de miserável
nisso. A propósito, acho bastante pertinente as críticas que Sartre
fez ao "espírito de seriedade", tanto que gosto muito dos e-mails do
L.E.R, que são muito divertidos, mesmo quando ele dá total vazão à
sua casca-grossice, chegando mesmo a enviar flatos virtuais.

Quando criei a enquete fatídica, é claro que eu estava brincando,
tanto que incluí nas opções um "nem sim nem não, muito pelo
contrário", e também "quem deve ser moderado é o Manuel Bulcão".
Aliás, votei nesta última opção.

Ressalte-se, também, que a pergunta da enquete era "O Emiliano deve
ser moderado?"; logo, não foi sugerido a expulsão do rapaz - foi o
pedido do Amauri ao Brudna para que ele fosse posto em moderação e,
também, o ocorrido no "Boteco do Zé" (onde muitos ciencialisteiros
costumam encher a cara) que me inspiraram a fazer aquela brincadeira.

Posso ter interpretado errado a mensagem do L. E. Carvalho (uma
conseqüência do embotamento etílico? Talvez) quando ele tachou a
brincadeira de miserável. Uma mensagem no mínimo incoveniente,
sobretudo porque, antes, eu o havia remetido em privado um e-mail de
feliz 2005, dizendo-lhe que ele não esquentasse, pois eu estava
apenas tirando um sarro.

De resto, minha participação nessa lista não se restringe a
piadinhas. Não raro, participo de discussões científicas.

De todo modo, peço desculpas, e reitero meus votos de um
incomensuravelmente feliz 2005 a todos.

E por falar em votos de feliz natal, enviei um ao Takata em privado,
mas a mensagem retornou. Isso é uma falta de consideração!:-(

Abraços,
Manuel Bulcão





SUBJECT: Pedido [administrador]
FROM: "L.E.R.de Carvalho" <lecarvalho@infolink.com.br>
TO: ciencialist@yahoogrupos.com.br
DATE: 25/12/2004 16:24

OH, MANÉ...

QUANDO TU BEBE, TU FICA SÉRIO, NÉ ?

BEBUM QUE CHORA NUM É MOLE NAUN.

VAI LÁ.

POR MIM, TA LIMPO.

NUM TEM POBREMA.

MAS PAREM DE FICAR SACANEANDO COM A NATALIA OH PA.

DEVEMOS TER MAIS TOLERANCIA COM OS QUIMICOS E COM AS MULERES.

FELIZ NATAL PRA VOCE.

FICA COM O LEO.

L.E.

At 16:08 25/12/2004, you wrote:


>--- Em ciencialist@yahoogrupos.com.br, "brudna" <lrb@i...> escreveu
>
> >Vamos cuidar para que mensagens nao sejam mal interpretadas. Para
>isso acho melhor evitar a agressividade. Ela pode ocorrer entre duas
>pessoas que conhecem o ritmo da brincadeira (Takata e Bulcao, por
>exemplo), mas vai acabar confundindo quem nao conhece a interacao
>entre as duas pessoas, julgando que eh uma briga real.
>
>Manuel: Como disse o L.E.R., essa lista é uma lista de ciência.
>Ocorre que ela é antiga, de modo que os seus membros --
>principalmente os da velha guarda -- são também amigos. Ora, entre
>amigos as brincadeiras são algo corrente, e não há nada de miserável
>nisso. A propósito, acho bastante pertinente as críticas que Sartre
>fez ao "espírito de seriedade", tanto que gosto muito dos e-mails do
>L.E.R, que são muito divertidos, mesmo quando ele dá total vazão à
>sua casca-grossice, chegando mesmo a enviar flatos virtuais.
>
>Quando criei a enquete fatídica, é claro que eu estava brincando,
>tanto que incluí nas opções um "nem sim nem não, muito pelo
>contrário", e também "quem deve ser moderado é o Manuel Bulcão".
>Aliás, votei nesta última opção.
>
>Ressalte-se, também, que a pergunta da enquete era "O Emiliano deve
>ser moderado?"; logo, não foi sugerido a expulsão do rapaz - foi o
>pedido do Amauri ao Brudna para que ele fosse posto em moderação e,
>também, o ocorrido no "Boteco do Zé" (onde muitos ciencialisteiros
>costumam encher a cara) que me inspiraram a fazer aquela brincadeira.
>
>Posso ter interpretado errado a mensagem do L. E. Carvalho (uma
>conseqüência do embotamento etílico? Talvez) quando ele tachou a
>brincadeira de miserável. Uma mensagem no mínimo incoveniente,
>sobretudo porque, antes, eu o havia remetido em privado um e-mail de
>feliz 2005, dizendo-lhe que ele não esquentasse, pois eu estava
>apenas tirando um sarro.
>
>De resto, minha participação nessa lista não se restringe a
>piadinhas. Não raro, participo de discussões científicas.
>
>De todo modo, peço desculpas, e reitero meus votos de um
>incomensuravelmente feliz 2005 a todos.
>
>E por falar em votos de feliz natal, enviei um ao Takata em privado,
>mas a mensagem retornou. Isso é uma falta de consideração!:-(
>
>Abraços,
>Manuel Bulcão


[As partes desta mensagem que não continham texto foram removidas]



SUBJECT: Neste Natal da Natália
FROM: Maria Natália <grasdic@hotmail.com>
TO: ciencialist@yahoogrupos.com.br
DATE: 25/12/2004 16:41


http://www.dgrh.unicamp.br/natal2004/natal_dgrh_2004.html
Um abraço
Maria Natália





SUBJECT: I.A. sera q vai ter?
FROM: "Amauri Jr" <amaurijunior2@yahoo.com.br>
TO: <ciencialist@yahoogrupos.com.br>, <grasdic@yahoogroups.co.uk>
DATE: 25/12/2004 19:16

Oi Pessoal

Eu li uma vez um projeto de IA com rede neurais, eles fizeram uma CPU, ou HD, que se auto-programava; mas tiveram que desligar, pois com apenas um minuto ele tinha a intelig�ncia de um menino de dez anos. Ser� que podemos um dia ter andr�ides de auto-programa��o? Tipo os exterminadores do filme "Exterminador do Futuro"?

Amauri

[As partes desta mensagem que n�o continham texto foram removidas]



SUBJECT: Re: Fw: Fisica e Plantas Medicinais
FROM: Maria Natália <grasdic@hotmail.com>
TO: ciencialist@yahoogrupos.com.br
DATE: 26/12/2004 00:26


Leo:
Creio que deve ser em fitofísica.
http://www.metabase.net/docs/earth/15094.html
E uma vez que existe a biofísica...
Talvez por este link ele chegue a alguma conclusão
Um abraço
Maria Natália


--- Em ciencialist@yahoogrupos.com.br, "Luiz Ferraz Netto"
<leobarretos@u...> escreveu
> não sei.
>
> []'
> ===========================
> Luiz Ferraz Netto [Léo]
> leobarretos@u...
> http://www.feiradeciencias.com.br
> ===========================
> -----Mensagem Original-----
> De: BelPiva
> Para: leobarretos@u...
> Enviada em: quinta-feira, 2 de dezembro de 2004 12:29
> Assunto: Fisica e Plantas Medicinais
>
>
> Ola
> Gostaria de saber em quais, ou quais conteudos de Fisica
> posso associar Plantas Medicinais.
>
> Osmar B Mende
>
> e-mail osmarbmendes@b...
>
> [As partes desta mensagem que não continham texto foram removidas]





SUBJECT: Alguém vigiando enquanto dormimos
FROM: Maria Natália <grasdic@hotmail.com>
TO: ciencialist@yahoogrupos.com.br
DATE: 26/12/2004 02:00


Mas não falo de bombeiros. Falo de outros homens e que vigiam os céus
Aqueles que estao estudando movimentos de asteróides e que tratam
dos possíveis impactos com a Terra:
http://neo.jpl.nasa.gov/risk/2004mn4.html
Um abraço
Maria Natália





SUBJECT: Re: Sobre dimensões.
FROM: Leandro Aparecido Côco <apocrifos@ig.com.br>
TO: ciencialist@yahoogrupos.com.br
DATE: 26/12/2004 18:34


não existe interação COM o
espaço-tempo e sim interações entre matérias no espaço e no tempo.
Quais matérias com o espaço tempo para criar particulas?

Outra coisa: Como que uma vibração de onda (ou sei lá o que seja) de
uma brana pode criar particulas?



-- Em ciencialist@yahoogrupos.com.br, Hélio Ricardo Carvalho
<hrc@f...> escreveu
>
> Leandro,
>
> Antes que Amaury e os quânticos mais esotéricos da lista te deixem
> doido, eu vou falar um pouco da minha visão das três dimensões
> espaciais e do tempo.
>
> A natureza do espaço é uma e a do tempo é outra.
>
> Matematicamente podemos colocar tudo num quadrivetor.
> Podemos fazer muitas coisas com a matemática e são coisas úteis
para
> facilitar o estudo de certos fenômenos naturais mas não são os
> fenômenos em si.
> Números imaginários não existem são somente frutos de sua
> imaginação.
> :-)
>
> Leandro disse:
> > pelo que entendi as branas criam particulas através da
interação
> com
> > o espaço-tempo.
>
> Na minha humilde opinião (mensagem nesta lista não é um artigo
> científico para publicação em revista internacional, logo posso
> expressar minhas próprias opiniões) não existe interação COM o
> espaço-tempo e sim interações entre matérias no espaço e no tempo.
>
> Sem qualquer um desses componentes (Matéria, Espaço e Tempo), não

> interação.
>
>
> Hélio
>
>
>
> --- Em ciencialist@yahoogrupos.com.br, "Amauri Jr"
> <amaurijunior2@y...> escreveu
> > oi Leandro
> >
> > Tem muito físico bom aqui, até um quântico que o caso do JC, que
> pode te responder literalmente. Estou a estudar e vou falar eu
> entendi.
> >
> >
> > O tempo seria algo relacionado como um expresso em movimento,
> enquanto eu estou aqui a escrever esse e-mail, o "trem" do tempo
vai
> passando e determinando nossas ações. Seria como se tudo tivesse
> determinado dentro de nosso universo, como se nada pode ser de
nossa
> escolha, esta é a teoria do tempo segundo a Teoria da Relatividade
> de Einstein.
> >
> > As dimensões, segundo teoria quântica, são 10 aproximadamente; o
> que podemos pôr na teoria do tempo quantifico, cada um de nós temos
> uma cópia em outras dimensões. Você esta numa festa e conhece uma
> moça bonita, tem varias hipóteses de acontecer de ela ser sua
amiga,
> ela ser sua namorada ou até mesmo te achar sem graça. Cada uma
delas
> seria uma cópia nossa de outra dimensão que entramos a cada decisão
> nossa, uma decisão provável.
> >
> > Abraços
> > Amauri





SUBJECT: Re: Tudo é energia
FROM: Leandro Aparecido Côco <apocrifos@ig.com.br>
TO: ciencialist@yahoogrupos.com.br
DATE: 26/12/2004 19:10


Amiga, qual a frequência do foton na teoria ondulatória?
Como me parece muito inteligente, poderia me dizer uma coisa?
Por que na física quântica não podemos definir a velocidade e a
posição de uma particula simultâneamente sem alterar o resultado um
do outro?


--- Em ciencialist@yahoogrupos.com.br, Maria Natália <grasdic@h...>
escreveu
>
> DC:
>
> O fotão é partícula que na teoria corpuscular constitui a radiação
> (luz). E pico, critas de onde se falamos na teoria ondulatória da
> luz. Estas duas teorias se completam e posso encarar a luz como
onda
> ou corpúsculo consoante a sua interação com o meio (objecto) Ambas
> transportam energia. Quer a masa quer a energia estão quantizadas,
> os tais pacotes de energia.
> Os fotões de maior energia são os de maior frequência e estão neste
> caso os da raios UV, radiação X e Raios cósmicos que têm poder
> ionizante e penetrante elevados.
> Um abarço
> Maria Natália
>
> --- Em ciencialist@yahoogrupos.com.br, "Cyberlander"
> <cybernews@s...> escreveu
> >
> >
> > fótons são pacotinhos com massa ou picos de onda?
> > [ ]'s
> > D.C.
> >
> >
> >
> > --- Em ciencialist@yahoogrupos.com.br, José Renato <jrma@t...>
> > escreveu
> > > Hélio, o meu e acredito que também o seu energia-campo é
> composto
> > de
> > > fenômenos que denominamos de energia e dos que chamamos de
> > campo! :-))
> > > Tensão no espaço são provocadas pelas forças nucleares, forte e
> > fraca,
> > > magnetismo e gravidade. Exemplo: o átomo é um conjunto material
> > vazio, mais
> > > espaço que matéria, porém devido às "tensões" existentes no
> > núcleo,
> > > interação das subpartículas, nos elétrons - emissores e
> receptores
> > de quanta
> > > e fótons mas que ainda é considerado uma partícula, digamos,
> > > ndivisível(?) - e entre o núcleo e os elétrons parece ter uma
> > consistência
> > > material "sólida" devido a essa estrutura de tensões.
> > > []s
> > > José Renato
> > > .......................................................
> > >
> > > From: "Hélio Ricardo Carvalho" <hrc@f...>
> > > To: <ciencialist@yahoogrupos.com.br>
> > > Sent: Wednesday, December 22, 2004 7:11 PM
> > > Subject: [ciencialist] Re: Tudo é energia
> > >
> > > Este teu "energia-campo" seria composto de que? ... :-)
> > >
> > > "Espaço sem maiores tensões"???
> > > O que seria tensão no espaço?
> > >
> > > Hoje estou perguntador do tipo do "marcelo ferrari", mas é só
por
> > > agora.
> > >
> > > [ ]'s
> > > Hélio
> > >
> > >
> > >
> > > --- Em ciencialist@yahoogrupos.com.br, José Renato <jrma@t...>
> > > escreveu
> > > > Hélio, percebo a matéria como energia-campos "estruturados" em
> > > determinada
> > > > configuração. Já o espaço, mesmo pleno de energia e campos,
> > > considero como
> > > > "não estruturado", sem maiores tensões.
> > >
> > >
> > >
> > >
> > >
> > >
> > > ##### ##### #####
> > >
> > > Para saber mais visite
> > > http://www.ciencialist.hpg.ig.com.br
> > >
> > >
> > > ##### ##### ##### #####
> > > Links do Yahoo! Grupos
> > >
> > >
> > >
> > >
> > >
> > >
> > >
> > >
> > >
> > > Esta mensagem foi verificada pelo E-mail Protegido Terra.
> > > Scan engine: McAfee VirusScan / Atualizado em 22/12/2004 /
> Versão:
> > 4.4.00 -
> > > Dat 4416
> > > Proteja o seu e-mail Terra:
> http://www.emailprotegido.terra.com.br/
> > >
> > > E-mail classificado pelo Identificador de Spam Inteligente
Terra.
> > > Para alterar a categoria classificada, visite
> > >
> >
>
http://www.terra.com.br/centralunificada/emailprotegido/imail/imail.c
> > gi?
> >
>
+_u=jrma&_l=1,1103753533.708756.20502.cabue.terra.com.br,3444,Des15,D
> > es15





SUBJECT: Re: [ciencialist] Re: Sobre dimensões.
FROM: "Luiz Ferraz Netto" <leobarretos@uol.com.br>
TO: <ciencialist@yahoogrupos.com.br>
DATE: 27/12/2004 07:03


>"não existe interação COM o
espaço-tempo e sim interações entre matérias no espaço e no tempo.
Quais matérias com o espaço tempo para criar particulas?
Outra coisa: Como que uma vibração de onda (ou sei lá o que seja) de
uma brana pode criar particulas?"

Léo: A falta de entendimento é apenas devido a erro de grafia; não é brana ... é brahma! Com essa correção fica claro que tudo pode ser criado com o abuso na quantidade.

aquele abraço a todos e, aproveitando o embalo brahmanístico, meus votos de excelente ano de 2005 'for all'.

Espero que no dia 25, passado, tenham pendurado algumas maçãs em suas árvores de natal para comemorar o nascimento daquele que deu 'sentido' ao mundo que vivemos. A 'direção' quem deu foi FORD e a relatividade do 'dia em si' foi explicada pelo meu companheiro de 'jogar dados'(*).

aquele abraço,




SUBJECT: Re: [ciencialist] Re: Tudo é energia
FROM: "Luiz Ferraz Netto" <leobarretos@uol.com.br>
TO: <ciencialist@yahoogrupos.com.br>
DATE: 27/12/2004 07:16


>"Amiga, qual a frequência do foton na teoria ondulatória?
Como me parece muito inteligente, poderia me dizer uma coisa?
Por que na física quântica não podemos definir a velocidade e a
posição de uma particula simultâneamente sem alterar o resultado um
do outro?"

O 'tom' dessa mensagem parece-me áspero. Aprecio que releiam suas mensagens antes de 'enviar'.
Assim sendo, respondo em lugar da 'atacada':

A freqüência do fóton da teoria ondulatória é alta; praticamente em todas as páginas de estudo deparamos com a palavra "fóton"; ela apareceu pela primeira vez com Einstein ao fazer o tratamento do efeito fotoelétrico.
Em quântica (o que quer que seja isso) não é a velocidade e a posição de uma partícula que sofrem dos males de uma 'indeterminação' e sim 'a quantidade de movimento' e a coordenada de posição da partícula.

Outra apreciação é que tomem cuidado com o 'restante' da mensagem --- cortem o desnecessário ao entendimento de sua própria postagem --- o lixo é cumulativo!

aquele abraço
daquele que baixou violentamente de status social; de deus para moderador!

Léo


SUBJECT: refrigerante
FROM: "E m i l i a n o C h e m e l l o" <chemelloe@yahoo.com.br>
TO: <ciencialist@yahoogrupos.com.br>
DATE: 27/12/2004 07:59

Por que o refrigerante, quando quente, faz 'mais espuma' quando transferido da garrafa para o copo do que o refrigerante 'mais gelado'? Admite-se como 'quente' uma temperatura de 25 ºC e o 'mais gelado' como sendo a temperatura média de um congelador, que eu chuto ser algo entorno de 5 ºC

[ ] 's do Emiliano Chemello
emiliano@quimica.net
http://www.quimica.net/emiliano
http://www.ucs.br/ccet/defq/naeq

" Rien ne se perd, rien ne se crée,
tout se transforme."

Antoine Laurent de Lavoisier (químico francês, 1743 - 1794)


[As partes desta mensagem que não continham texto foram removidas]



SUBJECT: Re: [ciencialist] Re: Tudo é energia
FROM: "Alvaro Augusto - Electra" <alvaro@electraenergy.com.br>
TO: <ciencialist@yahoogrupos.com.br>
DATE: 27/12/2004 10:28

Dê uma olhada em http://en.wikipedia.org/wiki/Uncertainty_principle

[ ]s

Alvaro Augusto


----- Original Message -----
From: "Leandro Aparecido Côco" <apocrifos@ig.com.br>
To: <ciencialist@yahoogrupos.com.br>
Sent: Sunday, December 26, 2004 7:10 PM
Subject: [ciencialist] Re: Tudo é energia




Amiga, qual a frequência do foton na teoria ondulatória?
Como me parece muito inteligente, poderia me dizer uma coisa?
Por que na física quântica não podemos definir a velocidade e a
posição de uma particula simultâneamente sem alterar o resultado um
do outro?


--- Em ciencialist@yahoogrupos.com.br, Maria Natália <grasdic@h...>
escreveu
>
> DC:
>
> O fotão é partícula que na teoria corpuscular constitui a radiação
> (luz). E pico, critas de onde se falamos na teoria ondulatória da
> luz. Estas duas teorias se completam e posso encarar a luz como
onda
> ou corpúsculo consoante a sua interação com o meio (objecto) Ambas
> transportam energia. Quer a masa quer a energia estão quantizadas,
> os tais pacotes de energia.
> Os fotões de maior energia são os de maior frequência e estão neste
> caso os da raios UV, radiação X e Raios cósmicos que têm poder
> ionizante e penetrante elevados.
> Um abarço
> Maria Natália
>
> --- Em ciencialist@yahoogrupos.com.br, "Cyberlander"
> <cybernews@s...> escreveu
> >
> >
> > fótons são pacotinhos com massa ou picos de onda?
> > [ ]'s
> > D.C.
> >
> >
> >
> > --- Em ciencialist@yahoogrupos.com.br, José Renato <jrma@t...>
> > escreveu
> > > Hélio, o meu e acredito que também o seu energia-campo é
> composto
> > de
> > > fenômenos que denominamos de energia e dos que chamamos de
> > campo! :-))
> > > Tensão no espaço são provocadas pelas forças nucleares, forte e
> > fraca,
> > > magnetismo e gravidade. Exemplo: o átomo é um conjunto material
> > vazio, mais
> > > espaço que matéria, porém devido às "tensões" existentes no
> > núcleo,
> > > interação das subpartículas, nos elétrons - emissores e
> receptores
> > de quanta
> > > e fótons mas que ainda é considerado uma partícula, digamos,
> > > ndivisível(?) - e entre o núcleo e os elétrons parece ter uma
> > consistência
> > > material "sólida" devido a essa estrutura de tensões.
> > > []s
> > > José Renato
> > > .......................................................
> > >
> > > From: "Hélio Ricardo Carvalho" <hrc@f...>
> > > To: <ciencialist@yahoogrupos.com.br>
> > > Sent: Wednesday, December 22, 2004 7:11 PM
> > > Subject: [ciencialist] Re: Tudo é energia
> > >
> > > Este teu "energia-campo" seria composto de que? ... :-)
> > >
> > > "Espaço sem maiores tensões"???
> > > O que seria tensão no espaço?
> > >
> > > Hoje estou perguntador do tipo do "marcelo ferrari", mas é só
por
> > > agora.
> > >
> > > [ ]'s
> > > Hélio
> > >
> > >
> > >
> > > --- Em ciencialist@yahoogrupos.com.br, José Renato <jrma@t...>
> > > escreveu
> > > > Hélio, percebo a matéria como energia-campos "estruturados" em
> > > determinada
> > > > configuração. Já o espaço, mesmo pleno de energia e campos,
> > > considero como
> > > > "não estruturado", sem maiores tensões.
> > >
> > >
> > >
> > >
> > >
> > >
> > > ##### ##### #####
> > >
> > > Para saber mais visite
> > > http://www.ciencialist.hpg.ig.com.br
> > >
> > >
> > > ##### ##### ##### #####
> > > Links do Yahoo! Grupos
> > >
> > >
> > >
> > >
> > >
> > >
> > >
> > >
> > >
> > > Esta mensagem foi verificada pelo E-mail Protegido Terra.
> > > Scan engine: McAfee VirusScan / Atualizado em 22/12/2004 /
> Versão:
> > 4.4.00 -
> > > Dat 4416
> > > Proteja o seu e-mail Terra:
> http://www.emailprotegido.terra.com.br/
> > >
> > > E-mail classificado pelo Identificador de Spam Inteligente
Terra.
> > > Para alterar a categoria classificada, visite
> > >
> >
>
http://www.terra.com.br/centralunificada/emailprotegido/imail/imail.c
> > gi?
> >
>
+_u=jrma&_l=1,1103753533.708756.20502.cabue.terra.com.br,3444,Des15,D
> > es15




SUBJECT: Re: [ciencialist] Re: Tudo é energia
FROM: "Alvaro Augusto - Electra" <alvaro@electraenergy.com.br>
TO: <ciencialist@yahoogrupos.com.br>
DATE: 27/12/2004 10:29

Dê uma olhada em http://en.wikipedia.org/wiki/Uncertainty_principle

[ ]s

Alvaro Augusto


----- Original Message -----
From: "Leandro Aparecido Côco" <apocrifos@ig.com.br>
To: <ciencialist@yahoogrupos.com.br>
Sent: Sunday, December 26, 2004 7:10 PM
Subject: [ciencialist] Re: Tudo é energia




Amiga, qual a frequência do foton na teoria ondulatória?
Como me parece muito inteligente, poderia me dizer uma coisa?
Por que na física quântica não podemos definir a velocidade e a
posição de uma particula simultâneamente sem alterar o resultado um
do outro?


--- Em ciencialist@yahoogrupos.com.br, Maria Natália <grasdic@h...>
escreveu
>
> DC:
>
> O fotão é partícula que na teoria corpuscular constitui a radiação
> (luz). E pico, critas de onde se falamos na teoria ondulatória da
> luz. Estas duas teorias se completam e posso encarar a luz como
onda
> ou corpúsculo consoante a sua interação com o meio (objecto) Ambas
> transportam energia. Quer a masa quer a energia estão quantizadas,
> os tais pacotes de energia.
> Os fotões de maior energia são os de maior frequência e estão neste
> caso os da raios UV, radiação X e Raios cósmicos que têm poder
> ionizante e penetrante elevados.
> Um abarço
> Maria Natália
>
> --- Em ciencialist@yahoogrupos.com.br, "Cyberlander"
> <cybernews@s...> escreveu
> >
> >
> > fótons são pacotinhos com massa ou picos de onda?
> > [ ]'s
> > D.C.
> >
> >
> >
> > --- Em ciencialist@yahoogrupos.com.br, José Renato <jrma@t...>
> > escreveu
> > > Hélio, o meu e acredito que também o seu energia-campo é
> composto
> > de
> > > fenômenos que denominamos de energia e dos que chamamos de
> > campo! :-))
> > > Tensão no espaço são provocadas pelas forças nucleares, forte e
> > fraca,
> > > magnetismo e gravidade. Exemplo: o átomo é um conjunto material
> > vazio, mais
> > > espaço que matéria, porém devido às "tensões" existentes no
> > núcleo,
> > > interação das subpartículas, nos elétrons - emissores e
> receptores
> > de quanta
> > > e fótons mas que ainda é considerado uma partícula, digamos,
> > > ndivisível(?) - e entre o núcleo e os elétrons parece ter uma
> > consistência
> > > material "sólida" devido a essa estrutura de tensões.
> > > []s
> > > José Renato
> > > .......................................................
> > >
> > > From: "Hélio Ricardo Carvalho" <hrc@f...>
> > > To: <ciencialist@yahoogrupos.com.br>
> > > Sent: Wednesday, December 22, 2004 7:11 PM
> > > Subject: [ciencialist] Re: Tudo é energia
> > >
> > > Este teu "energia-campo" seria composto de que? ... :-)
> > >
> > > "Espaço sem maiores tensões"???
> > > O que seria tensão no espaço?
> > >
> > > Hoje estou perguntador do tipo do "marcelo ferrari", mas é só
por
> > > agora.
> > >
> > > [ ]'s
> > > Hélio
> > >
> > >
> > >
> > > --- Em ciencialist@yahoogrupos.com.br, José Renato <jrma@t...>
> > > escreveu
> > > > Hélio, percebo a matéria como energia-campos "estruturados" em
> > > determinada
> > > > configuração. Já o espaço, mesmo pleno de energia e campos,
> > > considero como
> > > > "não estruturado", sem maiores tensões.
> > >
> > >
> > >
> > >
> > >
> > >
> > > ##### ##### #####
> > >
> > > Para saber mais visite
> > > http://www.ciencialist.hpg.ig.com.br
> > >
> > >
> > > ##### ##### ##### #####
> > > Links do Yahoo! Grupos
> > >
> > >
> > >
> > >
> > >
> > >
> > >
> > >
> > >
> > > Esta mensagem foi verificada pelo E-mail Protegido Terra.
> > > Scan engine: McAfee VirusScan / Atualizado em 22/12/2004 /
> Versão:
> > 4.4.00 -
> > > Dat 4416
> > > Proteja o seu e-mail Terra:
> http://www.emailprotegido.terra.com.br/
> > >
> > > E-mail classificado pelo Identificador de Spam Inteligente
Terra.
> > > Para alterar a categoria classificada, visite
> > >
> >
>
http://www.terra.com.br/centralunificada/emailprotegido/imail/imail.c
> > gi?
> >
>
+_u=jrma&_l=1,1103753533.708756.20502.cabue.terra.com.br,3444,Des15,D
> > es15




SUBJECT: Re: [ciencialist] Re: Tudo é energia
FROM: "Alvaro Augusto - Electra" <alvaro@electraenergy.com.br>
TO: <ciencialist@yahoogrupos.com.br>
DATE: 27/12/2004 10:29

A confusão aumentaria se um engenheiro elétrico fosse adicionado à lista. No
meu caso, a definição seria assim: "energia é aquilo de onde tiro o meu
sustento"...

Na verdade, nem tudo é energia. Além da energia, existem as cordas...

[ ]s

Alvaro Augusto


----- Original Message -----
From: "JVictor" <jvoneto@uol.com.br>
To: <ciencialist@yahoogrupos.com.br>
Sent: Friday, December 17, 2004 7:30 AM
Subject: Re: [ciencialist] Re: Tudo é energia



Natália,

Se você colocar em uma mesa redonda 5 gênios da física e da química para
discutirem o que é energia, no sentido mesmo das verdades essenciais por
trás desse intuitivo conceito, você acha que todos darão a mesma resposta,
embora todos concordem com a conservação dela?
Uma boa experiência, com estudantes do nível médio, acho, é dar-lhes uma
tarefa: estudarem o conceito de energia, usando diversas fontes, mas que
eles estudem em separado. Depois, juntá-los numa espécie de seminário e
pedir-lhes explanarem o que cada um entendeu.
Se eu ainda labutasse na área," juro" que faria algo assim.
Quem sabe, com os jovens, vazios de conceitos batidos e dogmas, não
pudéssemos aprender algo mais, a respeito.

Sds,

Victor.

----- Original Message -----
From: Maria Natália
To: ciencialist@yahoogrupos.com.br
Sent: Friday, December 17, 2004 1:24 AM
Subject: [ciencialist] Re: Tudo é energia



Homero:
E o químico dirá que mesmo aquela cadeira parada, em repouso
elativo, tem energia. Energia interna, energia potencial química e
energia das oscilações, ligações, vibrações, rotacões moléculares.
Um astrofísico falará da energia do vácuo e por aí fora.
É como o orientador de estágio pegar num sapato que coloca em cima
de secretária do estagiário dizendo: e agora com base neste sapato
faça a planificação para aula de química nível 11º ano. Difícil será
introduzir em programa existente, mas faz-se.
E aquele sapato tem tudo e até aula de português permite fazer.
A energia assim falada permite tudo o que a creatividade do
professor quiser.
Definição de energia dá tanta raiva como a definição de força. É que
se necessita do conceito bem esclarecido. Só na universidade se
entende tais definições.
Energia é assim uma expressão em verso de poeta.

Energia é tudo:
Está contigo
foge com a bola
Cavalga no som
no grão de trigo
é núcleo desnudo
dá cabo da tola
De físico bombom*

Um abraço
Maria Natália
* físico muita BOM

--- Em ciencialist@yahoogrupos.com.br, "Oraculo" <oraculo@a...>
escreveu
> Olá
>
> O problema é a definição de energia. Em geral, uma confusão com os
diversos sentidos do termo energia causa problemas ou permite frases
de alto impacto como essa "tudo é energia", embora vazia de
significado..:-) Como na famosa "navegar é preciso, viver não é
preciso" onde os diversos sentidos de precisão podem criar diversas
interpretações do poema.
>
> A energia de uma bateria, a energia de um corpo em movimento, a
energia produzida pela usina hidroelétrica, nada tem a ver com a
energia de um espetáculo de teatro, acordar se sentidno "enérgico",
ou a palestra enérgica do professor de filosofia.
>
> Se usarmos um conceito estrito para o termo energia (strictu
senso) acho que fica mais fácil analisar a frase "tudo é energia".
Primeiro, a capacidade de produzir trabalho é energia. Se produz
trabalho, cabe o uso do termo, se não produz, não cabe. Assim,
eletricidade é energia, mas uma palestra ou mesmo um elemento
químico não são.
>
> Mesmo agora que sabemos que E=mc2, isso não significa que podemos
abandonar os conceitos e definições e considerar "tudo" a mesma
coisa..:-) Tudo o que, cara-pálid, eu perguntaria ao autor..:-) O
fato de que é possível convertar algo em outra coisa não torna as
duas uma só.
>
> Acho que o problema da frase "tudo é energia" é mesmo as
decorrencias da mesma: e daí? O que o autor ou quem a apresenta quer
dizer com isso, quais as implicações decorrentes, caso se aceite
como verdade a afirmação? Tudo é energia ENTÃO podemos nos comunicar
telepaticamente? Tudo é energia, ENTÃO a homeopatia, usando o "tudo"
e a energia, funciona? Ou apenas tudo é energia, e sabemos como
relacionar a matéria e a energia em uma equação matematica correta
(e, como tudo que existe neste universo é matéria ou energia, esse é
um saber consideravel..:-)?
>
> Tudo é energia em geral é uma afirmação-escapatória, usada
esotericametne na falta de evidencias ou argumentos confiáveis..:-)
>
> Um abraço.
>
> Homero
>
> ----- Original Message -----
> From: brudna
> To: ciencialist@yahoogrupos.com.br
> Sent: Thursday, December 16, 2004 6:35 PM
> Subject: [ciencialist] Tudo é energia
>
>
>
> Fico confuso com o argumento ´Tudo é energia´.
>
> Onde estão os erros? Que conceitos falhos pode ter esse
argumento?
> Como aproveitar essa oportunidade para ensinar?
>
>
> Até
> Luís Brudna



SUBJECT: Re: [ciencialist] Re: Tudo é energia
FROM: "Alvaro Augusto - Electra" <alvaro@electraenergy.com.br>
TO: <ciencialist@yahoogrupos.com.br>
DATE: 27/12/2004 10:38


----- Original Message -----
From: "Luiz Ferraz Netto" <leobarretos@uol.com.br>
To: <ciencialist@yahoogrupos.com.br>
Sent: Monday, December 27, 2004 7:16 AM
Subject: Re: [ciencialist] Re: Tudo é energia


> O 'tom' dessa mensagem parece-me áspero. Aprecio que releiam suas
mensagens antes de 'enviar'.
> Assim sendo, respondo em lugar da 'atacada':

> A freqüência do fóton da teoria ondulatória é alta; praticamente em todas
as páginas de estudo deparamos com a palavra "fóton"; ela apareceu pela
primeira vez
> com Einstein ao fazer o tratamento do efeito fotoelétrico.

Na verdade, o termo "fóton" não foi usado por Einstein em seus primeiros
artigos. O termo aparece pela primeira vez com Gilbert Lewis, químico
norte-americano, em carta ao editor da Nature, em 1926. Ver
http://www.nobeliefs.com/photon.htm e
http://en.wikipedia.org/wiki/Gilbert_N._Lewis

> Outra apreciação é que tomem cuidado com o 'restante' da mensagem ---
cortem o desnecessário ao entendimento de sua própria postagem --- o lixo é
> cumulativo!

Seria bom também que as pessoas tomasse o cuidado de não cortar TUDO. De vez
em quando é difícil saber do que alguém está falando, ou a quem está
respondendo...

[ ]s

Alvaro Augusto



SUBJECT: Re: [ciencialist] Re: Sobre dimensões.
FROM: "Alvaro Augusto - Electra" <alvaro@electraenergy.com.br>
TO: <ciencialist@yahoogrupos.com.br>
DATE: 27/12/2004 10:43

A vibração de uma brana não cria propriamente partículas. A vibração de uma
brana É a partícula. Por exemplo, uma 1-brana, anteriormente denominada
"corda" contém energia, e essa energia, de acordo com E=mc2, aparece no
mundo quadridimensional sob a forma de massa. Da mesma forma, a interação
entre branas produz partículas, mas são partículas virtuais.

[ ]s

Alvaro Augusto


----- Original Message -----
From: "Leandro Aparecido Côco" <apocrifos@ig.com.br>
To: <ciencialist@yahoogrupos.com.br>
Sent: Sunday, December 26, 2004 6:34 PM
Subject: [ciencialist] Re: Sobre dimensões.




não existe interação COM o
espaço-tempo e sim interações entre matérias no espaço e no tempo.
Quais matérias com o espaço tempo para criar particulas?

Outra coisa: Como que uma vibração de onda (ou sei lá o que seja) de
uma brana pode criar particulas?



-- Em ciencialist@yahoogrupos.com.br, Hélio Ricardo Carvalho
<hrc@f...> escreveu
>
> Leandro,
>
> Antes que Amaury e os quânticos mais esotéricos da lista te deixem
> doido, eu vou falar um pouco da minha visão das três dimensões
> espaciais e do tempo.
>
> A natureza do espaço é uma e a do tempo é outra.
>
> Matematicamente podemos colocar tudo num quadrivetor.
> Podemos fazer muitas coisas com a matemática e são coisas úteis
para
> facilitar o estudo de certos fenômenos naturais mas não são os
> fenômenos em si.
> Números imaginários não existem são somente frutos de sua
> imaginação.
> :-)
>
> Leandro disse:
> > pelo que entendi as branas criam particulas através da
interação
> com
> > o espaço-tempo.
>
> Na minha humilde opinião (mensagem nesta lista não é um artigo
> científico para publicação em revista internacional, logo posso
> expressar minhas próprias opiniões) não existe interação COM o
> espaço-tempo e sim interações entre matérias no espaço e no tempo.
>
> Sem qualquer um desses componentes (Matéria, Espaço e Tempo), não

> interação.
>
>
> Hélio
>



SUBJECT: refrigerante
FROM: "L.E.R.de Carvalho" <lecarvalho@infolink.com.br>
TO: ciencialist@yahoogrupos.com.br
DATE: 27/12/2004 11:49

At 07:59 27/12/2004, you wrote:
> Por que o refrigerante, quando quente, faz 'mais espuma' quando
> transferido da garrafa para o copo do que o refrigerante 'mais gelado'?
> Admite-se como 'quente' uma temperatura de 25 ºC e o 'mais gelado' como
> sendo a temperatura média de um congelador, que eu chuto ser algo entorno
> de 5 ºC
>
>[ ] 's do Emiliano Chemello



Uma geladeira comum tem uma temperatura acima de 5º C.
E o seu congelador tem uma temperatura bem abaixo dessa, talvez em torno de
10ºC.

Só te dou uma dica: os refrigerantes, em geral, são colocados dentro da
garrafa, já resfriados, com a água bem gelada.

(Pega êle, Físico-Química...)

L.e.

[As partes desta mensagem que não continham texto foram removidas]



SUBJECT: Re: [ciencialist] refrigerante
FROM: "E m i l i a n o C h e m e l l o" <chemelloe@yahoo.com.br>
TO: <ciencialist@yahoogrupos.com.br>
DATE: 27/12/2004 12:00

Olá L.E.R,

Dei parâmetros de temperatura para a pergunta não ficar muito vaga. É um fenômeno que venho observando faz tempo e gostaria de saber se minhas idéias estão de acordo com as idéias dos demais participantes da lista.

Não estou me referindo ao 'antes' do refrigerante, mas sim quando o abrimos e despejamos em um copo. A quantidade daquela 'espuma' que se forma varia com a temperatura. Quanto mais gelado o refrigerante estiver, menos espuma irá produzir.

A resposta deve percorrer, creio eu, analisando a gás que faz o refrigerante ter 'bolinhas'.

[ ] 's do Emiliano Chemello
emiliano@quimica.net
http://www.quimica.net/emiliano
http://www.ucs.br/ccet/defq/naeq

" Rien ne se perd, rien ne se crée,
tout se transforme."

Antoine Laurent de Lavoisier (químico francês, 1743 - 1794)

----- Original Message -----
From: L.E.R.de Carvalho
To: ciencialist@yahoogrupos.com.br
Sent: Monday, December 27, 2004 11:49 AM
Subject: [ciencialist] refrigerante


At 07:59 27/12/2004, you wrote:
> Por que o refrigerante, quando quente, faz 'mais espuma' quando
> transferido da garrafa para o copo do que o refrigerante 'mais gelado'?
> Admite-se como 'quente' uma temperatura de 25 ºC e o 'mais gelado' como
> sendo a temperatura média de um congelador, que eu chuto ser algo entorno
> de 5 ºC
>
>[ ] 's do Emiliano Chemello



Uma geladeira comum tem uma temperatura acima de 5º C.
E o seu congelador tem uma temperatura bem abaixo dessa, talvez em torno de
10ºC.

Só te dou uma dica: os refrigerantes, em geral, são colocados dentro da
garrafa, já resfriados, com a água bem gelada.

(Pega êle, Físico-Química...)

L.e.

[As partes desta mensagem que não continham texto foram removidas]



##### ##### #####

Para saber mais visite
http://www.ciencialist.hpg.ig.com.br


##### ##### ##### #####


Yahoo! Grupos, um serviço oferecido por:

São Paulo Rio de Janeiro Curitiba Porto Alegre Belo Horizonte Brasília




------------------------------------------------------------------------------
Links do Yahoo! Grupos

a.. Para visitar o site do seu grupo na web, acesse:
http://br.groups.yahoo.com/group/ciencialist/

b.. Para sair deste grupo, envie um e-mail para:
ciencialist-unsubscribe@yahoogrupos.com.br

c.. O uso que você faz do Yahoo! Grupos está sujeito aos Termos do Serviço do Yahoo!.



[As partes desta mensagem que não continham texto foram removidas]



SUBJECT: refrigerante
FROM: "L.E.R.de Carvalho" <lecarvalho@infolink.com.br>
TO: ciencialist@yahoogrupos.com.br
DATE: 27/12/2004 12:02

At 12:00 27/12/2004, you wrote:
>Olá L.E.R,
>
> Dei parâmetros de temperatura para a pergunta não ficar muito vaga. É
> um fenômeno que venho observando faz tempo e gostaria de saber se minhas
> idéias estão de acordo com as idéias dos demais participantes da lista.
>
> Não estou me referindo ao 'antes' do refrigerante, mas sim quando o
> abrimos e despejamos em um copo. A quantidade daquela 'espuma' que se
> forma varia com a temperatura. Quanto mais gelado o refrigerante estiver,
> menos espuma irá produzir.
>
> A resposta deve percorrer, creio eu, analisando a gás que faz o
> refrigerante ter 'bolinhas'.
>
>[ ] 's do Emiliano Chemello


Eu não disse que o que ocorreu antes, no engarrafamento, vá influir nessa
hora que você pretende olhar.

Eu disse pra você considerar a motivação de refrigerar antes de embalar.

Pensai.

L.E.

[As partes desta mensagem que não continham texto foram removidas]



SUBJECT: Refrigerante e Handbook de Físico-Química
FROM: "L.E.R.de Carvalho" <lecarvalho@infolink.com.br>
TO: ciencialist@yahoogrupos.com.br
DATE: 27/12/2004 12:04

A resposta está em qualquer handbook de Físico-Química.

Sugiro olhar as curvas de solubilidade de dióxido de carbono em água.
E como a curva varia com a temperatura.

Para uma boa correlação, sugiro olhar também as curvas para cloreto de sódio.

L.E.




SUBJECT: Re: Sobre dimensões.
FROM: Hélio Ricardo Carvalho <hrc@fis.puc-rio.br>
TO: ciencialist@yahoogrupos.com.br
DATE: 27/12/2004 13:18


Alvaro,

O que seria "vibração" neste contexto??

Seria uma mudança de posição DE matéria NO espaço de forma periódica
NO tempo????
:-)

Hélio.

PS. para Leandro: Não diga que não avisei.


--- Em ciencialist@yahoogrupos.com.br, "Alvaro Augusto - Electra"
<alvaro@e...> escreveu
> A vibração de uma brana não cria propriamente partículas. A vibração
de uma
> brana É a partícula. Por exemplo, uma 1-brana, anteriormente denominada
> "corda" contém energia, e essa energia, de acordo com E=mc2, aparece no
> mundo quadridimensional sob a forma de massa. Da mesma forma, a
interação
> entre branas produz partículas, mas são partículas virtuais.
>
> [ ]s
>
> Alvaro Augusto
>
>
> ----- Original Message -----
> From: "Leandro Aparecido Côco" <apocrifos@i...>
> To: <ciencialist@yahoogrupos.com.br>
> Sent: Sunday, December 26, 2004 6:34 PM
> Subject: [ciencialist] Re: Sobre dimensões.
>
>
>
>
> não existe interação COM o
> espaço-tempo e sim interações entre matérias no espaço e no tempo.
> Quais matérias com o espaço tempo para criar particulas?
>
> Outra coisa: Como que uma vibração de onda (ou sei lá o que seja) de
> uma brana pode criar particulas?
>
>
>
> -- Em ciencialist@yahoogrupos.com.br, Hélio Ricardo Carvalho
> <hrc@f...> escreveu
> >
> > Leandro,
> >
> > Antes que Amaury e os quânticos mais esotéricos da lista te deixem
> > doido, eu vou falar um pouco da minha visão das três dimensões
> > espaciais e do tempo.
> >
...
...
...





SUBJECT: Proporcao divina
FROM: André da Cruz <decolhp@pop.com.br>
TO: ciencialist@yahoogrupos.com.br
DATE: 27/12/2004 15:07

Ola amigos da ciencialist, procurei nos arquivos do grupo e o assunto da Proporcao
Divina (o numero PHI) foi discutido no ano de 2000 nessa lista.

Atualmente tenho visto discussoes em um nivel muito elevado de conhecimento
cientifico. Gostaria que alguns membros dessa lista abordassem novamente esse tema,
pois ele virou uma noticia recente, visto que eh abordado no best-seller O CODIGO
DA VINCI.

Realmente a proporcao divina, o numero PHI que eh encontrado a partir da serie de
Fibonacci esta presente em muitas proporcoes da natureza, desde ossos e medidas do
nosso corpo, como em flores e muitas outras propriedades da natureza.

Leonardo da Vinci tomou essa -proporcao divina- como uma verdade e em muitos de
seus momentos tentou justifica-la.

Atualmente eh sabido que se trata apenas de uma proporcao logica, que pode ser
encontrado em tudo. Ja vi alguns artigos e sites em ingles que explicam melhor a
matematica dessa questao, deixando o misticismo e o divino de lado.

Como no livro O CODIGO DA VINCI o autor trata essa proporcao como verdadeira,
dedicando 2 paginas a ela (eu comecei a ler o livro ontem) gostaria que os amigos
cientistas dessa lista pudessem explicar melhor (matematica e cientificamente) essa
questao que eh tao simples e nao tem nada de divina ou misteriosa.

Obrigado
abracos e feliz 2005 a todos

Andre da Cruz



SUBJECT: Revista Ciencia Hoje
FROM: André da Cruz <decolhp@pop.com.br>
TO: ciencialist@yahoogrupos.com.br
DATE: 27/12/2004 16:13

Alguem eh assinante da revista CIENCIA HOJE?
http://cienciahoje.uol.com.br/

Em caso positivo, o que acham? Qual a qualidade da revista, textos, artigos e se
recomendam?

Eu vi no site da revista e achei algumas coisas interessantes, mas antes de assinar
queria a opiniao de algum assinante.

E em outro caso, qual outra revista cientifica (de uma linguagem compreensivel aos
nao cientistas-formados) voces recomendariam?

Obrigado a todos
Andre



SUBJECT: Re: [ciencialist] Re: Tudo é energia
FROM: "Prof. JC" <profjc2003@yahoo.com.br>
TO: <ciencialist@yahoogrupos.com.br>
DATE: 27/12/2004 16:18

Me parece que "energia" não é "coisa", mas tão somente um conceito que nos
permite "relacionar coisas" e fazer previsões sobre o comportamento das
"coisas relacionadas".

Feynman dá uma boa definição de energia no capítulo 4, volume 1, de seu "The
Feynman Lectures on Physics".

Abraços,
Prof. JC

----- Original Message -----
From: "Alvaro Augusto - Electra" <alvaro@electraenergy.com.br>
To: <ciencialist@yahoogrupos.com.br>
Sent: Monday, December 27, 2004 10:29 AM
Subject: Re: [ciencialist] Re: Tudo é energia


>
> A confusão aumentaria se um engenheiro elétrico fosse adicionado à lista.
No
> meu caso, a definição seria assim: "energia é aquilo de onde tiro o meu
> sustento"...
>
> Na verdade, nem tudo é energia. Além da energia, existem as cordas...
>
> [ ]s
>
> Alvaro Augusto
>
>
> ----- Original Message -----
> From: "JVictor" <jvoneto@uol.com.br>
> To: <ciencialist@yahoogrupos.com.br>
> Sent: Friday, December 17, 2004 7:30 AM
> Subject: Re: [ciencialist] Re: Tudo é energia
>
>
>
> Natália,
>
> Se você colocar em uma mesa redonda 5 gênios da física e da química para
> discutirem o que é energia, no sentido mesmo das verdades essenciais por
> trás desse intuitivo conceito, você acha que todos darão a mesma resposta,
> embora todos concordem com a conservação dela?
> Uma boa experiência, com estudantes do nível médio, acho, é dar-lhes uma
> tarefa: estudarem o conceito de energia, usando diversas fontes, mas que
> eles estudem em separado. Depois, juntá-los numa espécie de seminário e
> pedir-lhes explanarem o que cada um entendeu.
> Se eu ainda labutasse na área," juro" que faria algo assim.
> Quem sabe, com os jovens, vazios de conceitos batidos e dogmas, não
> pudéssemos aprender algo mais, a respeito.
>
> Sds,
>
> Victor.
>
> ----- Original Message -----
> From: Maria Natália
> To: ciencialist@yahoogrupos.com.br
> Sent: Friday, December 17, 2004 1:24 AM
> Subject: [ciencialist] Re: Tudo é energia
>
>
>
> Homero:
> E o químico dirá que mesmo aquela cadeira parada, em repouso
> elativo, tem energia. Energia interna, energia potencial química e
> energia das oscilações, ligações, vibrações, rotacões moléculares.
> Um astrofísico falará da energia do vácuo e por aí fora.
> É como o orientador de estágio pegar num sapato que coloca em cima
> de secretária do estagiário dizendo: e agora com base neste sapato
> faça a planificação para aula de química nível 11º ano. Difícil será
> introduzir em programa existente, mas faz-se.
> E aquele sapato tem tudo e até aula de português permite fazer.
> A energia assim falada permite tudo o que a creatividade do
> professor quiser.
> Definição de energia dá tanta raiva como a definição de força. É que
> se necessita do conceito bem esclarecido. Só na universidade se
> entende tais definições.
> Energia é assim uma expressão em verso de poeta.
>
> Energia é tudo:
> Está contigo
> foge com a bola
> Cavalga no som
> no grão de trigo
> é núcleo desnudo
> dá cabo da tola
> De físico bombom*
>
> Um abraço
> Maria Natália
> * físico muita BOM
>
> --- Em ciencialist@yahoogrupos.com.br, "Oraculo" <oraculo@a...>
> escreveu
> > Olá
> >
> > O problema é a definição de energia. Em geral, uma confusão com os
> diversos sentidos do termo energia causa problemas ou permite frases
> de alto impacto como essa "tudo é energia", embora vazia de
> significado..:-) Como na famosa "navegar é preciso, viver não é
> preciso" onde os diversos sentidos de precisão podem criar diversas
> interpretações do poema.
> >
> > A energia de uma bateria, a energia de um corpo em movimento, a
> energia produzida pela usina hidroelétrica, nada tem a ver com a
> energia de um espetáculo de teatro, acordar se sentidno "enérgico",
> ou a palestra enérgica do professor de filosofia.
> >
> > Se usarmos um conceito estrito para o termo energia (strictu
> senso) acho que fica mais fácil analisar a frase "tudo é energia".
> Primeiro, a capacidade de produzir trabalho é energia. Se produz
> trabalho, cabe o uso do termo, se não produz, não cabe. Assim,
> eletricidade é energia, mas uma palestra ou mesmo um elemento
> químico não são.
> >
> > Mesmo agora que sabemos que E=mc2, isso não significa que podemos
> abandonar os conceitos e definições e considerar "tudo" a mesma
> coisa..:-) Tudo o que, cara-pálid, eu perguntaria ao autor..:-) O
> fato de que é possível convertar algo em outra coisa não torna as
> duas uma só.
> >
> > Acho que o problema da frase "tudo é energia" é mesmo as
> decorrencias da mesma: e daí? O que o autor ou quem a apresenta quer
> dizer com isso, quais as implicações decorrentes, caso se aceite
> como verdade a afirmação? Tudo é energia ENTÃO podemos nos comunicar
> telepaticamente? Tudo é energia, ENTÃO a homeopatia, usando o "tudo"
> e a energia, funciona? Ou apenas tudo é energia, e sabemos como
> relacionar a matéria e a energia em uma equação matematica correta
> (e, como tudo que existe neste universo é matéria ou energia, esse é
> um saber consideravel..:-)?
> >
> > Tudo é energia em geral é uma afirmação-escapatória, usada
> esotericametne na falta de evidencias ou argumentos confiáveis..:-)
> >
> > Um abraço.
> >
> > Homero
> >
> > ----- Original Message -----
> > From: brudna
> > To: ciencialist@yahoogrupos.com.br
> > Sent: Thursday, December 16, 2004 6:35 PM
> > Subject: [ciencialist] Tudo é energia
> >
> >
> >
> > Fico confuso com o argumento ´Tudo é energia´.
> >
> > Onde estão os erros? Que conceitos falhos pode ter esse
> argumento?
> > Como aproveitar essa oportunidade para ensinar?
> >
> >
> > Até
> > Luís Brudna
>
>
>
> ##### ##### #####
>
> Para saber mais visite
> http://www.ciencialist.hpg.ig.com.br
>
>
> ##### ##### ##### #####
> Links do Yahoo! Grupos
>
>
>
>
>
>
>
>
>




SUBJECT: Re: [ciencialist] Revista Ciencia Hoje
FROM: "E m i l i a n o C h e m e l l o" <chemelloe@yahoo.com.br>
TO: <ciencialist@yahoogrupos.com.br>
DATE: 27/12/2004 16:34

Olá André,

Já assinei a revista ciência hoje. Sem dúvida, antes de dizer se ela é
boa ou não, devo conhecer o leitor. Para não deixar-lhe sem opinião, diria
que ela é muito diferente da Superinteressante e Galileu. Possui outro
enfoque. Uma revista mais 'hard', com resultados de pesquisas aqui no Brasil
e no Mundo, além de matérias com nível de dificuldade bastante grande. Por
não possuir muito $$$, baixo os pdf's disponibilizados pela revista de
'algumas' resportagens. Veja mais em http://cienciahoje.uol.com.br/view/209
. Entre e baixe uma reportagem do seu agrado. Leia e veja se é o que você
procura.

[ ] 's do Emiliano Chemello
emiliano@quimica.net
http://www.quimica.net/emiliano
http://www.ucs.br/ccet/defq/naeq

" Rien ne se perd, rien ne se crée,
tout se transforme."

Antoine Laurent de Lavoisier (químico francês, 1743 - 1794)

----- Original Message -----
From: André da Cruz
To: ciencialist@yahoogrupos.com.br
Sent: Monday, December 27, 2004 4:13 PM
Subject: [ciencialist] Revista Ciencia Hoje


Alguem eh assinante da revista CIENCIA HOJE?
http://cienciahoje.uol.com.br/

Em caso positivo, o que acham? Qual a qualidade da revista, textos, artigos
e se
recomendam?

Eu vi no site da revista e achei algumas coisas interessantes, mas antes de
assinar
queria a opiniao de algum assinante.

E em outro caso, qual outra revista cientifica (de uma linguagem
compreensivel aos
nao cientistas-formados) voces recomendariam?

Obrigado a todos
Andre



##### ##### #####

Para saber mais visite
http://www.ciencialist.hpg.ig.com.br


##### ##### ##### #####


Yahoo! Grupos, um serviço oferecido por:
PUBLICIDADE





Links do Yahoo! Grupos

Para visitar o site do seu grupo na web, acesse:
http://br.groups.yahoo.com/group/ciencialist/

Para sair deste grupo, envie um e-mail para:
ciencialist-unsubscribe@yahoogrupos.com.br

O uso que você faz do Yahoo! Grupos está sujeito aos Termos do Serviço do
Yahoo!.



SUBJECT: Re: Revista Ciencia Hoje
FROM: "bombaspr" <bombaspr@uol.com.br>
TO: ciencialist@yahoogrupos.com.br
DATE: 27/12/2004 19:16


--- Em ciencialist@yahoogrupos.com.br, "E m i l i a n o C h e m e
l l o" <chemelloe@y...> escreveu
> Olá André,
>
> Já assinei a revista ciência hoje. Sem dúvida, antes de dizer
se ela é
> boa ou não, devo conhecer o leitor. Para não deixar-lhe sem
opinião, diria
> que ela é muito diferente da Superinteressante e Galileu. Possui
outro
> enfoque. Uma revista mais 'hard', com resultados de pesquisas aqui
no Brasil
> e no Mundo, além de matérias com nível de dificuldade bastante
grande.
===========================

Na verdade os textos são escritos de forma que um biólogo entenda
uma matéria sobre física, por ex., isto é, embora possam ser
profundos, os assuntos são iniciados com uma introdução
perfeitamente digerível.

Questão de opinião, mas embora leia as duas prefiro a Ciência Hoje
do que a Scientific American (esta as vezes tem textos
ininteligíveis, ao menos para mim...).

Galileu e Superinteressante são revistas para curiosos (uma boa
opção para introduzir os jovens no mundo da ciência, penso).

Inté+
Mauro

P.S.: Aproveitando o assunto dos refrigerantes. Por que a Coca Cola
faz mais espuma se colocada sobre o gelo se comparado a colocarmos
um gelo sobre o refrigerante? (Não deveria ser o calor, e não o
frio, a liberar o gás?






SUBJECT: Re: refrigerante
FROM: Maria Natália <grasdic@hotmail.com>
TO: ciencialist@yahoogrupos.com.br
DATE: 27/12/2004 22:34


L.E.R:

1-E qual será a temperatura ambiente em sala de enchimento de
produtos gaseificados nas fábricas?
2-A soubilidades dos gases em água* normalmente diminui quando a
temperatura aumenta, considerando uma dada pressão que permanece
constante durante o processo. Quando se aquece água num copo ou
recipiente de Pyrex se vê a formação de bolhas de ar junto à parede
de vidro antes de a água entre em ebulição. Da próxima vez que
aquecermos a água em nossos balões de laboratório ou nas máquinas de
café de vidro vejamos se as bolhas de ar dissolvidas não estão a
sair da solução antes desta entrar em ebulição.
E alguém aqui é pescador? Sai com a caninha de pesca para
dar "banho" à minhoca (isco)? Num dia quente de verão, uma pescadora
experiente (e física) escolhe normalmente zonas profundas do lago ou
curso de água pois sabe que os peixes escolhem as zonas fundas
porque a quantidade de oxigénio existente nas zonas frias é maior do
que nas zonas quentes (da superfície). Pois é, peixe sofre e tem
metabolismo sensível a estas variações de concentração de oxigénio
dissolvido: a solubilidade diminui com a temperatura! Além de terem
de fugir de uma BOA pescadora e que até sabe física...
Boas pescas neste fim de ano frio nooso e quente vosso.
Maria Natália, "matadora" de peixes...e não só...
* entendo por refrigerante qualquer bebida que tem por solvente a
agua como vinho, sucos, cerveja...


--- Em ciencialist@yahoogrupos.com.br, "L.E.R.de Carvalho"
<lecarvalho@i...> escreveu
> At 07:59 27/12/2004, you wrote:
> > Por que o refrigerante, quando quente, faz 'mais espuma'
quando
> > transferido da garrafa para o copo do que o refrigerante 'mais
gelado'?
> > Admite-se como 'quente' uma temperatura de 25 ºC e o 'mais
gelado' como
> > sendo a temperatura média de um congelador, que eu chuto ser
algo entorno
> > de 5 ºC
> >
> >[ ] 's do Emiliano Chemello
>
>
>
> Uma geladeira comum tem uma temperatura acima de 5º C.
> E o seu congelador tem uma temperatura bem abaixo dessa, talvez em
torno de
> 10ºC.
>
> Só te dou uma dica: os refrigerantes, em geral, são colocados
dentro da
> garrafa, já resfriados, com a água bem gelada.
>
> (Pega êle, Físico-Química...)
>
> L.e.
>
> [As partes desta mensagem que não continham texto foram removidas]





SUBJECT: Mistérios para os Watson da C-List
FROM: Maria Natália <grasdic@hotmail.com>
TO: ciencialist@yahoogrupos.com.br
DATE: 27/12/2004 22:53


Puxai pelos cordelinhos e rolinhos de...massa cinzenta:
http://www.space.com/scienceastronomy/mysteries_quiz_041227.html
Passai com caipirinha fresca enquanto estou na neve
Maria Natália





SUBJECT: refrigerante y gaseosas
FROM: "L.E.R.de Carvalho" <lecarvalho@infolink.com.br>
TO: ciencialist@yahoogrupos.com.br
DATE: 27/12/2004 23:19


>
>* entendo por refrigerante qualquer bebida que tem por solvente a
>agua como vinho, sucos, cerveja...



No Brasil, refrigerante é aquilo que, em espanhol, é "gaseosas".

Sem gás... creio que já não é mais refrigerante.

L.E.

[As partes desta mensagem que não continham texto foram removidas]



SUBJECT: Re: [ciencialist] Re: refrigerante
FROM: "Emiliano Chemello - Yahoo Grupos" <chemelloe@yahoo.com.br>
TO: <ciencialist@yahoogrupos.com.br>
DATE: 27/12/2004 23:24

Natália,

Gostaria de ouvir/ler sua opinião sobre meu questionamento. Vou reproduzi-lo
abaixo:

################

Por que o refrigerante, quando quente, faz 'mais espuma' quando transferido
da garrafa para o copo do que o refrigerante 'mais gelado'? Admite-se como
'quente' uma temperatura de 25 ºC e o 'mais gelado' como sendo a temperatura
média de um congelador, que eu chuto ser algo entorno de 5 ºC

################

[ ]'s do

Emiliano Chemello
emiliano@quimica.net
http://www.quimica.net/emiliano
http://www.ucs.br/ccet/defq/naeq
[ MSN ] chemelloe@hotmail.com
[ ICQ ] 145060604

"Rien ne se perd, rien ne se crée, tout se transforme"
Lavoisier, químico francês (1743-1794)

----- Original Message -----
From: Maria Natália
To: ciencialist@yahoogrupos.com.br
Sent: Monday, December 27, 2004 10:34 PM
Subject: [ciencialist] Re: refrigerante



L.E.R:

1-E qual será a temperatura ambiente em sala de enchimento de
produtos gaseificados nas fábricas?
2-A soubilidades dos gases em água* normalmente diminui quando a
temperatura aumenta, considerando uma dada pressão que permanece
constante durante o processo. Quando se aquece água num copo ou
recipiente de Pyrex se vê a formação de bolhas de ar junto à parede
de vidro antes de a água entre em ebulição. Da próxima vez que
aquecermos a água em nossos balões de laboratório ou nas máquinas de
café de vidro vejamos se as bolhas de ar dissolvidas não estão a
sair da solução antes desta entrar em ebulição.
E alguém aqui é pescador? Sai com a caninha de pesca para
dar "banho" à minhoca (isco)? Num dia quente de verão, uma pescadora
experiente (e física) escolhe normalmente zonas profundas do lago ou
curso de água pois sabe que os peixes escolhem as zonas fundas
porque a quantidade de oxigénio existente nas zonas frias é maior do
que nas zonas quentes (da superfície). Pois é, peixe sofre e tem
metabolismo sensível a estas variações de concentração de oxigénio
dissolvido: a solubilidade diminui com a temperatura! Além de terem
de fugir de uma BOA pescadora e que até sabe física...
Boas pescas neste fim de ano frio nooso e quente vosso.
Maria Natália, "matadora" de peixes...e não só...
* entendo por refrigerante qualquer bebida que tem por solvente a
agua como vinho, sucos, cerveja...


--- Em ciencialist@yahoogrupos.com.br, "L.E.R.de Carvalho"
<lecarvalho@i...> escreveu
> At 07:59 27/12/2004, you wrote:
> > Por que o refrigerante, quando quente, faz 'mais espuma'
quando
> > transferido da garrafa para o copo do que o refrigerante 'mais
gelado'?
> > Admite-se como 'quente' uma temperatura de 25 ºC e o 'mais
gelado' como
> > sendo a temperatura média de um congelador, que eu chuto ser
algo entorno
> > de 5 ºC
> >
> >[ ] 's do Emiliano Chemello
>
>
>
> Uma geladeira comum tem uma temperatura acima de 5º C.
> E o seu congelador tem uma temperatura bem abaixo dessa, talvez em
torno de
> 10ºC.
>
> Só te dou uma dica: os refrigerantes, em geral, são colocados
dentro da
> garrafa, já resfriados, com a água bem gelada.
>
> (Pega êle, Físico-Química...)
>
> L.e.
>
> [As partes desta mensagem que não continham texto foram removidas]





##### ##### #####

Para saber mais visite
http://www.ciencialist.hpg.ig.com.br


##### ##### ##### #####


Yahoo! Grupos, um serviço oferecido por:








Links do Yahoo! Grupos

Para visitar o site do seu grupo na web, acesse:
http://br.groups.yahoo.com/group/ciencialist/

Para sair deste grupo, envie um e-mail para:
ciencialist-unsubscribe@yahoogrupos.com.br

O uso que você faz do Yahoo! Grupos está sujeito aos Termos do Serviço do
Yahoo!.



SUBJECT: Fw: Relatorio
FROM: "Luiz Ferraz Netto" <leobarretos@uol.com.br>
TO: "ciencialist" <ciencialist@yahoogrupos.com.br>
DATE: 28/12/2004 08:16

Alguém se habilita para esse espertinho?
[]'
===========================
Luiz Ferraz Netto [Léo]
leobarretos@uol.com.br
http://www.feiradeciencias.com.br
===========================
-----Mensagem Original-----
De: "HEWERTON DO NASCIMENTO CAJU" <hcajusp@hotmail.com>
Para: <leobarretos@uol.com.br>
Enviada em: segunda-feira, 27 de dezembro de 2004 20:14
Assunto: Relatorio


| GOSTARIA DE SABER SE VC TEM COMO ME ENVIAR POR E-MAIL RELATORIOS SOBRE:
| 1- DEFORMAÇOES DE MOLAS;
| 2- OSCILACOES DE ONDAS;
| 3- EMPUXO.
| DESDE JA AGRADEÇO A SUA AJUDA. ESTOU PRECISANDO PARA ENTREGAR COMO NOTA PARA
| A FACULDADE. ESTUDO ENGENHARIA DE PRODUÇAO. ATE MAIS!!
| HEWERTON CAJU.
|
| _________________________________________________________________
| MSN Hotmail, o maior webmail do Brasil. http://www.hotmail.com
|


SUBJECT: Fw: Cinemática dúvida
FROM: "Luiz Ferraz Netto" <leobarretos@uol.com.br>
TO: "ciencialist" <ciencialist@yahoogrupos.com.br>
DATE: 28/12/2004 08:18

O buraco da dúvida da menina é tremendamente mais prá baixo ... há chances?
Todo um curso de 'funções'?
[]'
===========================
Luiz Ferraz Netto [Léo]
leobarretos@uol.com.br
http://www.feiradeciencias.com.br
===========================
-----Mensagem Original-----
De: "contatosandra" <contatosandra@uol.com.br>
Para: "leobarretos" <leobarretos@uol.com.br>
Enviada em: segunda-feira, 27 de dezembro de 2004 16:52
Assunto: Cinemática dúvida


Olá professor! Estou prestando vestibular e minha dúvida é a
seguinte:
Na fórmula: s = so + vo.t + (1/2).a.t2, o que explica a
velocidade ser igual a inicial e nao a final p[or exemplo?

No seu site encontrei a seguinte explicação:
21.2. Parâmetro B - é o valor da derivada da função horária
para t = 0, o que o identifica com a velocidade inicial do
movimento: B = vo .
Lembramos: v = ds/dt = B + 2C.t e, para t=0, v = vo = B.

21.3. Parâmetro C - é o valor da derivada segunda da função
horária, para qualquer t, e identifica-se como a metade do
valor da aceleração escalar: C = a/2 .
Lembramos: a= d2s/dt2 = dv/dt = 2C , donde, C = a/2.

e segundo essa explicação, o que entendi é que então o
istante t que acompanha o V0 é diferente do instante t que é
elevado ao quadrado e acompanha a aceleração, matematicamente
tal conclusão é impossível pois temos a seguinte
possibilidade:
s = s1 + v1.(t-t1) + a(t-t1)2/2

Gostaria também de saber porque que o (t-t1) que acompanha a
aceleração é elevado ao quadrado.
Muito obrigada!





__________________________________________________________________________
Acabe com aquelas janelinhas que pulam na sua tela.
AntiPop-up UOL - É grátis!
http://antipopup.uol.com.br/




SUBJECT: Re: Fw: Cinemática dúvida
FROM: Hélio Ricardo Carvalho <hrc@fis.puc-rio.br>
TO: ciencialist@yahoogrupos.com.br
DATE: 28/12/2004 10:30


Léo,

Realmente é todo um curso de funções. Uma explicação algébrica cai na
necessidade da compreensão do significado do que seja uma integral.
Talvez para ela fique melhor uma explicação geométrica no gráfico de
VxT.
Primeiro mostrar num gráfico de M.U. que a área abaixo da linha
horizontal representa o espaço percorrido (S - S0).
Depois ainda num gráfico VxT mas agora de um M.U.V. em que já parta de
um v0, mostrar que podemos achar a área da figura somando duas partes
da área total: Um retângulo abaixo de v0 [v0*(t-t0)] [ou v0*t se t0=0]
e um triângulo acima de v0 [base * altura /2]
= [(t-t0)*(v-v0)/2]
= [(t-t0)*(a*(t-t0))/2]
= (1/2)a(t-t0)^2

Ficando assim:
S-S0 = v0*(t-t0) + (1/2)*a*(t-t0)^2

Se ainda ficar confuso como foi determinada a altura(v-v0) podemos
dizer que a tangente do ângulo oposto a altura é dv/dt=a logo
a=altura/(t-t0)
altura= a*(t-t0).

Hélio



--- Em ciencialist@yahoogrupos.com.br, "Luiz Ferraz Netto"
<leobarretos@u...> escreveu
>...
> -----Mensagem Original-----
>...Estou prestando vestibular e minha dúvida é a
> seguinte:
> Na fórmula: s = so + vo.t + (1/2).a.t2, o que explica a
> velocidade ser igual a inicial e nao a final p[or exemplo?
>
> No seu site encontrei a seguinte explicação:
> 21.2. Parâmetro B - é o valor da derivada da função horária
> para t = 0, o que o identifica com a velocidade inicial do
> movimento: B = vo .
> Lembramos: v = ds/dt = B + 2C.t e, para t=0, v = vo = B.
>
> 21.3. Parâmetro C - é o valor da derivada segunda da função
> horária, para qualquer t, e identifica-se como a metade do
> valor da aceleração escalar: C = a/2 .
> Lembramos: a= d2s/dt2 = dv/dt = 2C , donde, C = a/2.
>
> e segundo essa explicação, o que entendi é que então o
> istante t que acompanha o V0 é diferente do instante t que é
> elevado ao quadrado e acompanha a aceleração, matematicamente
> tal conclusão é impossível pois temos a seguinte
> possibilidade:
> s = s1 + v1.(t-t1) + a(t-t1)2/2
>
> Gostaria também de saber porque que o (t-t1) que acompanha a
> aceleração é elevado ao quadrado.
> Muito obrigada!
>





SUBJECT: Re: [ciencialist] Re: Fw: Cinemática dúvida
FROM: "Luiz Ferraz Netto" <leobarretos@uol.com.br>
TO: <ciencialist@yahoogrupos.com.br>
DATE: 28/12/2004 11:44

Oi Hélio,

a dúvida da garota é 'bem ampla' --- na verdade! Abrange tanto Física como Matemática. Para ela a expressão s = Vo.t + (1/2)a.t^2 não tem significado algum, em ambos os ramos do conhecimento.

Tentei dar a ela um minúsculo 'Vo' para tentar levantar 1/2 pálpebra de seu olho; veja o que lhe escrevi:

+++++++++++++++++

Oi Sandra,

sua dúvida básica está no conhecimento de 'funções', um tópico de matemática.
Uma função relaciona duas grandezas , normalmente indicadas com y e x (em física pode ser s e t, v e t, a e t, f e t, etc.). A finalidade da função é a seguinte: quando vc conhece o valor da grandeza x (que pode ser qqer grandeza, em física), a expressão permite calcular o valor (ou valores) da grandeza y .

Há muitos tipos de funções, eis alguns exemplos:

1) y = 4 é uma função constante e está nos informando que o valor de y é 4, independente de qqer valor assumido por outras grandezas (sejam elas, x, v, g ,h etc ... pois nem mesmo aparece essa letra na expressão "y=4"!).
Na Física, uma equação do tipo s = 4 significa que o móvel permanece na posição 4 metros, a contar da origem dos espaços, indefinidamente, por todo tempo --- tá parado!

2) y = 2.x é uma função linear que nos indica que y tem sempre o dobro do valor de x. Na Física, uma equação do tipo s = 2.t indica que o espaço s varia duas vezes mais rapidamente que o valor de t (t = tempo, s = espaço), a contar de uma origem dos tempos. Assim, para t = 0 (valor que t assume na origem dos tempos) o espaço s , vale s = 2.0 = 0 . Esse valor do espaço quando t = 0 recebe o nome de espaço inicial (So) e indica onde o móvel está ao ser acionado o cronômetro que começa a registrar o tempo t. Para qqer outro valor de t (por exemplo t = 4 s) é só substituir na expressão do movimento (no exemplo, s = 2.t) para obter a posição correspondente do móvel (no caso, s = 2.4 = 8 m).
Na Física, toda expressão (ou lei horária) onde aparece o "t" com expoente 1 (que não se escreve), ou seja, uma função do primeiro grau, traduz um movimento uniforme, ou seja, aquele em que o espaço varia proporcionalmente com o tempo (como é o caso da lei s = 2t). Depois se mostra que, se isso acontece, sua velocidade é constante.

3) y = 2.x + 3.x^2 (é uma função do segundo grau, pois aparece um termo onde o x está elevado ao quadrado).

Esse é um pequeno empurrão prá vc. É bom pegar um manual de matemática e outro de cinemática escalar e estudar junto o desenvolvimento das funções e seus exemplos nas equações horárias.

aquele abraço,
+++++++++++++++++++++++++++++++++++++++

Foi uma tentativa; com reflexo ou não, tentei!

aquele abraço,
===========================
Luiz Ferraz Netto [Léo]
leobarretos@uol.com.br
http://www.feiradeciencias.com.br
===========================
-----Mensagem Original-----
De: "Hélio Ricardo Carvalho" <hrc@fis.puc-rio.br>
Para: <ciencialist@yahoogrupos.com.br>
Enviada em: terça-feira, 28 de dezembro de 2004 10:30
Assunto: [ciencialist] Re: Fw: Cinemática dúvida




Léo,

Realmente é todo um curso de funções. Uma explicação algébrica cai na
necessidade da compreensão do significado do que seja uma integral.
Talvez para ela fique melhor uma explicação geométrica no gráfico de
VxT.
Primeiro mostrar num gráfico de M.U. que a área abaixo da linha
horizontal representa o espaço percorrido (S - S0).
Depois ainda num gráfico VxT mas agora de um M.U.V. em que já parta de
um v0, mostrar que podemos achar a área da figura somando duas partes
da área total: Um retângulo abaixo de v0 [v0*(t-t0)] [ou v0*t se t0=0]
e um triângulo acima de v0 [base * altura /2]
= [(t-t0)*(v-v0)/2]
= [(t-t0)*(a*(t-t0))/2]
= (1/2)a(t-t0)^2

Ficando assim:
S-S0 = v0*(t-t0) + (1/2)*a*(t-t0)^2

Se ainda ficar confuso como foi determinada a altura(v-v0) podemos
dizer que a tangente do ângulo oposto a altura é dv/dt=a logo
a=altura/(t-t0)
altura= a*(t-t0).

Hélio






SUBJECT: Viagem a Portugal de André Facina
FROM: Maria Natália <grasdic@hotmail.com>
TO: ciencialist@yahoogrupos.com.br
DATE: 29/12/2004 01:36


Poderá parecer Of Topic mas não é.
O GRASDIC, Grupo de Astronomia e de Divulgação Cientifica e que está
presente nesta lista através de sua dinamizadora, torna público que
André Luiz Facina foi o escolhido para vir a Portugal permanecer 30
dias em visita cultural e científica.
São os 5 anos deste grupo que se pretende comemorar em 2005 por meio
do início de intercâmbio Portugal Brasil e para o qual pedimos o
apoio de todos.
Queremos que se conheça:
1- Como é o ensino em Portugal: assistindo a aulas teóricas e de
laboratório em duas escolas públicas de ensino secundário (7º ao 12º
anos=11 a 18 anos de idade);
2-Como são as Universidades públicas de Lisboa e Aveiro com livre
trnsito em bibliotecas e departamentos de Física, Química e
Informática;
3- Que se faz nos centros Ciência Viva e museu interactivos de
Ciência, Oceanário,Aquário, Jardim botãnico e Zoológico;
4- Centros de Astrofísica e Geofísica, Observatório e Planetário e
Sociedades de astronomia, Física e de Química;
5- frequência de cursos: livre sobre cosmologia na Faculdade de
Ciências da Universidade de Lisboa e de astronomia observacional com
telescópio ETX90Ec e Konus newtoniano, 2 noites de trabalho de campo
com astrofotógrafos nacionais na Atalaia;
6- Peça de teatro Einstein e Picasso, no Teatro Trindade, uma semana
de aulas livres de Capoeira, Karaté shotokai,ginástica manutenção,
aeróbica e musculação (à escolha)...no Ginásio Clube Português.
Para além das visitas habituais ao monumentos da cidade de Lisboa e
Serras da Arrábida e Sintra e rios Tejo e Sado com seus roazes.
Depois de muito pensar em datas de modo a não prejudicar aulas de um
lado e outro se escolheu mês de Fevereiro.
A escolha sobre este elemento da lista de Ciência e Astronomia do
GRASDIC se deve a ter de ser brasileiro, em início de vida
universitária, maior e não tendo hipóteses de por seu meios
financeiros se deslocar à Europa.
Esperamos que os pais de André sintam o significado e a Alegria que
este acontecimento se reveste para o nosso colega de listas.
Esperemos que o autorizem a vir.
A viagem está marcada e as datas estipuladas.
O que se pede a este jovem apenas? Que seja portadores no seu
regresso ao Brasil do que se faz e e se luta em Portugal para o
avanço da Ciência.
E dentro de 10 meses contamos estar aí com um grupo de alunos e
sermos bem recebidos por vós.
Um abraço especial para o André que deve estar surpreso a estas
horas quem sabe se chorando...
Maria Natália Botelho, pedindo desculpa do OT





SUBJECT: Re: refrigerante y gaseosas
FROM: Maria Natália <grasdic@hotmail.com>
TO: ciencialist@yahoogrupos.com.br
DATE: 29/12/2004 01:48


L.E.R.

E após esta minha mensagem já podes compreender em que projectos
tenho andado metida e quão cheia de trabalho e sobretudo
responsabilidade.
Nem sei quando poderei respirar...mas voltando à gasosa, suco, sumo
e pirolito...
Portanto concluo que refrigerante tem gás e que quando cai em copo
produz espuma e bolhas de gás que é dióxido de carbono, CO2.
Aos outros produtos sem gás chamamos sumos se % de sumo é perto de
90% e néctar se perto de 30 a 45% de sumo natural e claro resto
água e açucar.
O pirolito existia antigamente no tempo do carro de ferro "chora",
antepassado dos eléctricos(estilo S Francisco,pois)
Obrigada
Maria Natália

--- Em ciencialist@yahoogrupos.com.br, "L.E.R.de Carvalho"
<lecarvalho@i...> escreveu
>
> >
> >* entendo por refrigerante qualquer bebida que tem por solvente a
> >agua como vinho, sucos, cerveja...
>
>
>
> No Brasil, refrigerante é aquilo que, em espanhol, é "gaseosas".
>
> Sem gás... creio que já não é mais refrigerante.
>
> L.E.
>
> [As partes desta mensagem que não continham texto foram removidas]





SUBJECT: Re: Fw: Cinemática dúvida
FROM: "rmtakata" <rmtakata@altavista.net>
TO: ciencialist@yahoogrupos.com.br
DATE: 29/12/2004 04:47


--- Em ciencialist@yahoogrupos.com.br, "Luiz Ferraz Netto"
> -----Mensagem Original-----
> De: "contatosandra" <contatosandra@u...>
> Na fórmula: s = so + vo.t + (1/2).a.t2, o que explica a
> velocidade ser igual a inicial e nao a final p[or exemplo?

Poderemos usar a velocidade final, mas aih a formula tem outra cara.

Considerando-se q. o deslocamento deltaS eh dado pela diferenca entre
a posicao final S e a posicao inicial So, eh facil ver q. a posicao
final S depende da posicao inicial So e do deslocamento realizado.

deltaS = S - So (1)

S = So + deltaS (2)

Ok, precisamos entao calcular o deslocamento deltaS realizado. Para
velocidade constante eh facil notar pela definicao de velocidade - o
qto um corpo se desloca em um determinado tempo (a taxa de alteracao
da posicao) q. o deslocamento serah dado pela velocidade multiplicada
pelo intervalo de tempo:

V = deltaS/deltat (3)

deltaS = V.deltat (4)

(Para simplificar consideremos q. o instante inicial to eh igual a 0,
assim deltat serah igual ao instante final t.)

deltaS = V.t (5)

Mas isso para velocidades constantes. No caso estamos considerando
velocidades q. variam uniformente no tempo - movimento uniformente
acelerado. Se o movimento eh acelerado, eh facil ver q. as velocidades
iniciais sao menores do q as velocidades finais pela definicao de
aceleracao:

a = dV/dt (6)

Como estamos considerando uma variacao uniforme, poderemos considerar
um intervalo maior em lugar de uma variacao infinitesimal (a
aceleracao serah igual em qq ponto dentro desse intervalo):

a = deltaV/deltat (7)

Considerando-se to=0

a = deltaV/t (8)

A aceleracao entao serah dada pela diferenca entre a velocidade final
V e a velocidade inicial Vo dividida pelo intervalo de tempo t

a =(V-Vo)/t (9)

Reescrevendo, poderemos obter a velocidade final V em funcao da
velocidade inicial Vo e intervalo t de tempo e a aceleracao a:

V = Vo + a.t (10)

Eh facil ver q. a velocidade final V, em um movimento acelerado serah
maior do q. a velocidade inicial Vo (e no caso de movimento
desacelerado serah menor). Eh tb facil notar q. qq velocidade em um
instante intermediario, serah uma velocidade de intensidade
intermediaria entre a velocidade final e a inicial - e seu valor serah
proporcional ao tempo decorrido desde o inicio.

Ok, para velocidades constantes notamos q. o deslocamento eh obtido
multiplicando-se a velocidade pelo intervalo de tempo:

deltaS = V.t (5)

Mas e no movimento em q a velocidade varia? Basta pegar a velocidade
media desempenhada pelo corpo no intervalo de tempo considerado. E
qual eh essa velocidade media? No caso de movimentos em q. a
velocidade varia uniformemente, poderemos considerar a velocidade
media como a media aritimetica entre a velocidade inicial Vo e a final
V (ou alternativamente, como a velocidade desempenhada pelo corpo na
metade do intervalo de tempo).

Vm = (V + Vo)/2 (11)

Substituindo-se (11) em (5) teremos:

deltaS = Vm.t = [(V + Vo)/2].t (12)

Desenvolvendo e reescrevendo:

deltaS = Vo.t/2 + V.t/2 (13)

E qual a velocidade final? Por (10) sabemos q:

V = Vo + a.t (10)

Substituindo-se (10) em (13), temos:

deltaS = Vo.t/2 + (Vo + a.t)t/2 (14)

Desenvolvendo e reescrevendo:

deltaS = Vo.t/2 + Vo/2 + (a.t).t/2 = Vo.t + (1/2).a.t^2 (15)

Substituindo (15) em (2), temos a equacao horaria do espaco para
movimentos uniformemente acelerados:

S = So + Vo.t + (1/2).a.t^2 (16)

Seguindo esses passos eh possivel se reescrever em funcao da
velocidade final V. E tb eh possivel se perceber de onde surge a
potencia quadratica do tempo a multiplicar a aceleracao a (bem como o
fator meio 1/2).

Posto de outro modo a posicao final S depende da posicao inicial So
(isso eh facil de se perceber, por exemplo, se o objeto nao se movesse
- em relacao ao referencial adotado - a posicao final seria igual 'a
inicial e se a posicao inicial estivesse mais perto do referencial, a
posicao final seria mais perto do referencial; se posicao inicial
fosse mais longe, a posicao final seria mais longe...), depende tb do
deslocamento proporcionado pelo movimento basal (representado pela
velocidade inicial - considere um movimento retilineo uniforme - isto
eh, sem aceleracao e veremos como a posicao final depende da
velocidade q. o objeto desempenha -- se o objeto se desloca
lentamente, pouco se moverah em relacao a posicao inicial; se se
desloca rapidamente, terah se afastado mais depois do mesmo tempo) e
depende ainda do deslocamento proporcionado pela velocidade q. se
acresce a cada instante 'a velocidade inicial (como eh uma taxa de uma
taxa, isto eh, uma taxa da variacao da velocidade q eh uma taxa da
variacao da posicao ou taxa de deslocamento, surge o t^2).

[]s,

Roberto Takata





SUBJECT: Re: [ciencialist] Re: Fw: Cinemática dúvida
FROM: "Luiz Ferraz Netto" <leobarretos@uol.com.br>
TO: <ciencialist@yahoogrupos.com.br>
DATE: 29/12/2004 08:12

Já enviei para a Sandra o novo quadro de informações do Takata sobre o MU e MUV e as obtenções de suas leis horárias, num quadro bem didático.
Cumprimentos,

... e isso vale para excelente 2005 para todos.

aquele abraço,
===========================
Luiz Ferraz Netto [Léo]
leobarretos@uol.com.br
http://www.feiradeciencias.com.br
===========================
-----Mensagem Original-----
De: "rmtakata" <rmtakata@altavista.net>
Para: <ciencialist@yahoogrupos.com.br>
Enviada em: quarta-feira, 29 de dezembro de 2004 04:47
Assunto: [ciencialist] Re: Fw: Cinemática dúvida




--- Em ciencialist@yahoogrupos.com.br, "Luiz Ferraz Netto"
> -----Mensagem Original-----
> De: "contatosandra" <contatosandra@u...>
> Na fórmula: s = so + vo.t + (1/2).a.t2, o que explica a
> velocidade ser igual a inicial e nao a final p[or exemplo?

Poderemos usar a velocidade final, mas aih a formula tem outra cara.

[ ... ]


SUBJECT: Fw: "VENTILADORES E EXAUSTORES"
FROM: "Luiz Ferraz Netto" <leobarretos@uol.com.br>
TO: "ciencialist" <ciencialist@yahoogrupos.com.br>
DATE: 29/12/2004 08:18

Alguém pode dar algum 'luz' (ou 'vento') prá esse consulente?

[]'
===========================
Luiz Ferraz Netto [Léo]
leobarretos@uol.com.br
http://www.feiradeciencias.com.br
===========================
-----Mensagem Original-----
De: "jsp" <jsp@uol.com.br>
Para: "leobarretos" <leobarretos@uol.com.br>
Enviada em: terça-feira, 28 de dezembro de 2004 16:32
Assunto: "VENTILADORES E EXAUSTORES"


PREZADO PROFESSOR,
ESTOU NECESSITANDO DE UMA LITERATURA NÃO
MUITO CIENTIFICA QUE ME PERMITA FAZER
CORRETAMENTE DIMENSIONAMENTOS DE VENTILADORES DE
EXAUSTORES INDUSTRIAIS. TENHO ENCONTRADO MUITA
DIFICULDADE EM ENCONTRAR ESSE TIPO DE LITERATURA
À RESPEITO DA DINÃMICA DA GERAÇÃO DE VENTOS. SOU
ENGENHEIRO AGRONOMO E CERTAMENTE TENHO UM POUCO
DE FACILIDADE DE ESTUDAR ESSE ASSUNTO SE
ENCONTRA UMA LITERATURA BOA. SE O PROFESSOR
SOUBER ONDE ENCONTRAR E ALGUM TÍTULO QUE ME
RECOMENDE FICAREI GRATO. JAMIL 064 99875044
jsp@uol.com.br.

__________________________________________________________________________
Acabe com aquelas janelinhas que pulam na sua tela.
AntiPop-up UOL - É grátis!
http://antipopup.uol.com.br/




SUBJECT: Re: Fw: Cinemática dúvida
FROM: Maria Natália <grasdic@hotmail.com>
TO: ciencialist@yahoogrupos.com.br
DATE: 29/12/2004 08:23


Leo:
Peço desculpa de só agora poder intervir mas preparar viagem para
alguém que vem a Portugal é mesmo de andar cheia de trabalho.
Acerca da dúvida desta moça: me parece esquisito o seguinte: porque
se dá esta expressão com derivadas? è de matéria de vestibular o
cálculo de derivadas e se calhar primitivas?
Mas esta mesma equação nada tem de difícil pois se chega a ela via
experimental com ou sem sensores e via queda livre ou descida de
plano inclinado. Há também o processo matemática a partir de
carrinhos dinâmicos deslizando em planos inclinados (mesas co
calçes) e estuido de pontos deixados em fita (estilo cauda) fixada a
de carrinho e que passa por marcador magnético de segundos. Mas
pronto suponhamos que vocês não usam o trabalho de laboratório para
chegar às leis...
A moça não conhecerá a equação das velocidades de um movimento
uniformementoe variado? Pois é a partir dela que se pode deduzir a
equação dos espaços. É uma dedução analítica e que acho por demais
evidente. As velocidades no M acelerado aumentam linearmete ou seja
o aumento (variação de velocidade) é directamente proporcional aos
tempos gastos em adquiri-las. Dito usando derivadas a derivada da
velocidade é constante. E se chama aceleração. Escrevendo então a
equação se chega à expressão matemática V= V0 + at ( S I) (é uma
equação algébrica). Esta equação pode ser representada gráficamente
e dá uma recta que se V0=o m por segundo passan pela origem. Como
menina deu antes, deve saber todo o Movimento rectilíneo e uniforme
e sabe que existe a velocidade média por exemplo numa viagem de
automóvel RJ---SP pode determinar a velocidade média (aliás é a 1ª
grandeza a ser dada e ainda antes do MU) Se o conceito estiver bem
apreendido* sucede q traçando gráfico de velocidade MUV (opta por A
e depois extrapola pois se trata de eq. algébrica) de movimento com
velocidade inicila. Do MRUniforme sabe o que é a área entre a curva
das velocidades e os eixos lhe dá o espaço percorrido.Em matemática
avançada iriamos por integrais...(Mas será do programa de
matemática? vestibular)Voltando ao grafico onde deve ter uma recta
que fiorma triângulo rectângulo pode começar em recta passando pela
origem V0= =M por segundo quandt t=0 segundos e vê que a lei dos
espaços (a tal área) é a equação que ela apresenta do 2º grau.
Pode também imaginar que o carro vai a uma velocidade média naquele
intervalo de tempo T0 a t f ou t simplesmente. Assim transforma o
triângulo rectângulo num rectãngulo semelhante ou seja com a mesma
área. (Basta ir a meio da hipotenusa e traça uma recta paralela a
eixos dos tempos do gráfico v=f(t). E calcula a área do rectângulo.
(no movi/o uniforme e uma recta paralela ao eixo dos tempos pois a
velocidade é constante. eixo vertical das velocidades) ( e não vou
falar em ordenadas e abcissas) finalmente a equação que resulta
dessa área deve vir expressa como o que o movimento tem de constante
e que é a aceleração. Escreve a eq de definição de aceleração e faz
substituições e surge
x= v . t ^2 (SI). Depois vai para movimento com v0 e assim a recta
já não passa pelo origem (gráfico V=f(t)Corta o eixo das velocidades
em vo velocidade inicial. A área compreendida agora é igual a um
rectãngulo e um triãngulo rectãngulo e etc e tal como atrás de chega
a uma equação de 2º grau mas completa x=x0 + v0 t +1/2a t^2 (SI)
Outra coisa NÃO USA S nem S0 nas equações. ISSO JÀ DESAPARECEU há
mais de 6 anos ou 10 anos e se usa o x e x0 em ves de S minúsculo
por causa das confusões com símbolo de tempo e que é o segundo s.
Não é mania. São normas internacionais.Mas j+á vos tinha falado
nisto. E agora não tenho tempo mas prometo voltar ao assunto. JURO!
Será que está confuso sem desenho? pois está mas eu não tenho editor
nem tempo e tenho ali à porta o Emiliano...
Acerca da menina: noto concepções alternativas terríveis acerca de
velocidade inicial, velocidade média e vejo ainda qe ESTA MOÇA ESTÁ
PENSANDO mais uma vez que a física são fórmulas matémáticas Mas não
a FÍSICA são conceitos apenas! e raciocínio. Cheira-me a que teve
como explicador de física alguém da área da matemática e ainda noto
que esta menina NUNCA FOI A LABORATÓRIO de Física trabalhar SOZINHA
8ou seja ser ela a realizar, medir, alterar, discutir, mudar,
extrapolar e verificar, comentar, criticar, refazer e SUAR.
E é só
Um abraço
Maria Natália
* OS CONCEITOS, o digerir destas grandezas são fundamentais e deve
ser adquirido hands on (a sair pelos dedos). Sim isso dá pena
pessoal a estudar f´´isica como se se tratasse de código de leis.

--- Em ciencialist@yahoogrupos.com.br, "Luiz Ferraz Netto"
<leobarretos@u...> escreveu
> O buraco da dúvida da menina é tremendamente mais prá baixo ... há
chances?
> Todo um curso de 'funções'?
> []'
> ===========================
> Luiz Ferraz Netto [Léo]
> leobarretos@u...
> http://www.feiradeciencias.com.br
> ===========================
> -----Mensagem Original-----
> De: "contatosandra" <contatosandra@u...>
> Para: "leobarretos" <leobarretos@u...>
> Enviada em: segunda-feira, 27 de dezembro de 2004 16:52
> Assunto: Cinemática dúvida
>
>
> Olá professor! Estou prestando vestibular e minha dúvida é a
> seguinte:
> Na fórmula: s = so + vo.t + (1/2).a.t2, o que explica a
> velocidade ser igual a inicial e nao a final p[or exemplo?
>
> No seu site encontrei a seguinte explicação:
> 21.2. Parâmetro B - é o valor da derivada da função horária
> para t = 0, o que o identifica com a velocidade inicial do
> movimento: B = vo .
> Lembramos: v = ds/dt = B + 2C.t e, para t=0, v = vo = B.
>
> 21.3. Parâmetro C - é o valor da derivada segunda da função
> horária, para qualquer t, e identifica-se como a metade do
> valor da aceleração escalar: C = a/2 .
> Lembramos: a= d2s/dt2 = dv/dt = 2C , donde, C = a/2.
>
> e segundo essa explicação, o que entendi é que então o
> istante t que acompanha o V0 é diferente do instante t que é
> elevado ao quadrado e acompanha a aceleração, matematicamente
> tal conclusão é impossível pois temos a seguinte
> possibilidade:
> s = s1 + v1.(t-t1) + a(t-t1)2/2
>
> Gostaria também de saber porque que o (t-t1) que acompanha a
> aceleração é elevado ao quadrado.
> Muito obrigada!
>
>
>
>
>
>
_____________________________________________________________________
_____
> Acabe com aquelas janelinhas que pulam na sua tela.
> AntiPop-up UOL - É grátis!
> http://antipopup.uol.com.br/





SUBJECT: Re: Fw: "VENTILADORES E EXAUSTORES"
FROM: "tipoalgo" <tipoalgo@bol.com.br>
TO: ciencialist@yahoogrupos.com.br
DATE: 29/12/2004 09:18


Olá jsp,

Ai vão dois caminhos que tratam um pouco do assunto.

http://www.ventisilva.com.br/FalandoExaustores.htm
http://myspace.eng.br/eng/ar/vent1.asp

Como dica faça uma pesquisa simples no google
(http://www.google.com/intl/pt-BR/) usando ventiladores exaustores
como palavras chaves e então é só "garimpar" ou então acrescentar
mais alguma palavra chave.

Abraços

Tipoalgo





SUBJECT: Re: [ciencialist] Re: Fw: Cinemática dúvida
FROM: "Luiz Ferraz Netto" <leobarretos@uol.com.br>
TO: <ciencialist@yahoogrupos.com.br>
DATE: 29/12/2004 09:53

Oi Natália,

A maioria dos livros didáticos de Ensino Médio, em Física, usam da "derivada" apenas como um algoritmo para se calcular a velocidade (a partir da lei horária) e a aceleração (a partir da lei de velocidade). Derivada nesses livros não tem significado de limite de nada, é simples algoritmo mesmo (faça que dá certo!) com meia dúzia de regras de derivação.

Se vc pegar qqer aluno do Ensino médio e propor a seguinte questão (como exemplo):

>>> Dada a lei de movimento s = a.cos(w.t+@), com a, w e @ constantes em relação a t, obter a lei de velocidade e aceleração --- sem o uso de derivadas --- pelas definições. <<<

verá que será um desastre total. E não é pelo fato da lei acima ser 'senoidal' não, mesmo se fosse 'algébrica' não teriam a menor idéia de como fazer isso! Basta tentar propor a seguinte lei horária : s = 3.t^3 e pedir lei de velocidade e aceleração, sem derivadas, e teremos novamente uma catástrofe.

Eu faço esse tratamento em:
http://www.feiradeciencias.com.br/sala19/texto32.asp

mas, rarissimamente (para não dizer nenhum!) algum aluno ou professor comenta esse trabalho (ou pede a continuação dele!). Sabe por que? Porque professores e alunos nunca fizeram esse tratamento em sala de aula --- usam da "derivada" e fim de papo!

Na Sala 04 de Cinemática tenho vários experimentos (tb com kit marcador de tempo + fita) para constatação do MU e MUV. Quantas são as escolas que fazem algo disso em sala de aula ou em laboratório?

Ao fim e ao cabo continuarei contribuindo com minha parcela --- até desistir!

[]'
===========================
Luiz Ferraz Netto [Léo]
leobarretos@uol.com.br
http://www.feiradeciencias.com.br
===========================
-----Mensagem Original-----
De: "Maria Natália" <grasdic@hotmail.com>
Para: <ciencialist@yahoogrupos.com.br>
Enviada em: quarta-feira, 29 de dezembro de 2004 08:23
Assunto: [ciencialist] Re: Fw: Cinemática dúvida




Leo:
Peço desculpa de só agora poder intervir mas preparar viagem para
alguém que vem a Portugal é mesmo de andar cheia de trabalho.
Acerca da dúvida desta moça: me parece esquisito o seguinte: porque
se dá esta expressão com derivadas? è de matéria de vestibular o
cálculo de derivadas e se calhar primitivas?
Mas esta mesma equação nada tem de difícil pois se chega a ela via
experimental com ou sem sensores e via queda livre ou descida de
plano inclinado. Há também o processo matemática a partir de
carrinhos dinâmicos deslizando em planos inclinados (mesas co
calçes) e estuido de pontos deixados em fita (estilo cauda) fixada a
de carrinho e que passa por marcador magnético de segundos. Mas
pronto suponhamos que vocês não usam o trabalho de laboratório para
chegar às leis...
A moça não conhecerá a equação das velocidades de um movimento
uniformementoe variado? Pois é a partir dela que se pode deduzir a
equação dos espaços. É uma dedução analítica e que acho por demais
evidente. As velocidades no M acelerado aumentam linearmete ou seja
o aumento (variação de velocidade) é directamente proporcional aos
tempos gastos em adquiri-las. Dito usando derivadas a derivada da
velocidade é constante. E se chama aceleração. Escrevendo então a
equação se chega à expressão matemática V= V0 + at ( S I) (é uma
equação algébrica). Esta equação pode ser representada gráficamente
e dá uma recta que se V0=o m por segundo passan pela origem. Como
menina deu antes, deve saber todo o Movimento rectilíneo e uniforme
e sabe que existe a velocidade média por exemplo numa viagem de
automóvel RJ---SP pode determinar a velocidade média (aliás é a 1ª
grandeza a ser dada e ainda antes do MU) Se o conceito estiver bem
apreendido* sucede q traçando gráfico de velocidade MUV (opta por A
e depois extrapola pois se trata de eq. algébrica) de movimento com
velocidade inicila. Do MRUniforme sabe o que é a área entre a curva
das velocidades e os eixos lhe dá o espaço percorrido.Em matemática
avançada iriamos por integrais...(Mas será do programa de
matemática? vestibular)Voltando ao grafico onde deve ter uma recta
que fiorma triângulo rectângulo pode começar em recta passando pela
origem V0= =M por segundo quandt t=0 segundos e vê que a lei dos
espaços (a tal área) é a equação que ela apresenta do 2º grau.
Pode também imaginar que o carro vai a uma velocidade média naquele
intervalo de tempo T0 a t f ou t simplesmente. Assim transforma o
triângulo rectângulo num rectãngulo semelhante ou seja com a mesma
área. (Basta ir a meio da hipotenusa e traça uma recta paralela a
eixos dos tempos do gráfico v=f(t). E calcula a área do rectângulo.
(no movi/o uniforme e uma recta paralela ao eixo dos tempos pois a
velocidade é constante. eixo vertical das velocidades) ( e não vou
falar em ordenadas e abcissas) finalmente a equação que resulta
dessa área deve vir expressa como o que o movimento tem de constante
e que é a aceleração. Escreve a eq de definição de aceleração e faz
substituições e surge
x= v . t ^2 (SI). Depois vai para movimento com v0 e assim a recta
já não passa pelo origem (gráfico V=f(t)Corta o eixo das velocidades
em vo velocidade inicial. A área compreendida agora é igual a um
rectãngulo e um triãngulo rectãngulo e etc e tal como atrás de chega
a uma equação de 2º grau mas completa x=x0 + v0 t +1/2a t^2 (SI)
Outra coisa NÃO USA S nem S0 nas equações. ISSO JÀ DESAPARECEU há
mais de 6 anos ou 10 anos e se usa o x e x0 em ves de S minúsculo
por causa das confusões com símbolo de tempo e que é o segundo s.
Não é mania. São normas internacionais.Mas j+á vos tinha falado
nisto. E agora não tenho tempo mas prometo voltar ao assunto. JURO!
Será que está confuso sem desenho? pois está mas eu não tenho editor
nem tempo e tenho ali à porta o Emiliano...
Acerca da menina: noto concepções alternativas terríveis acerca de
velocidade inicial, velocidade média e vejo ainda qe ESTA MOÇA ESTÁ
PENSANDO mais uma vez que a física são fórmulas matémáticas Mas não
a FÍSICA são conceitos apenas! e raciocínio. Cheira-me a que teve
como explicador de física alguém da área da matemática e ainda noto
que esta menina NUNCA FOI A LABORATÓRIO de Física trabalhar SOZINHA
8ou seja ser ela a realizar, medir, alterar, discutir, mudar,
extrapolar e verificar, comentar, criticar, refazer e SUAR.
E é só
Um abraço
Maria Natália
* OS CONCEITOS, o digerir destas grandezas são fundamentais e deve
ser adquirido hands on (a sair pelos dedos). Sim isso dá pena
pessoal a estudar f´´isica como se se tratasse de código de leis.

--- Em ciencialist@yahoogrupos.com.br, "Luiz Ferraz Netto"
<leobarretos@u...> escreveu
> O buraco da dúvida da menina é tremendamente mais prá baixo ... há
chances?
> Todo um curso de 'funções'?
> []'
> ===========================
> Luiz Ferraz Netto [Léo]
> leobarretos@u...
> http://www.feiradeciencias.com.br
> ===========================
> -----Mensagem Original-----
> De: "contatosandra" <contatosandra@u...>
> Para: "leobarretos" <leobarretos@u...>
> Enviada em: segunda-feira, 27 de dezembro de 2004 16:52
> Assunto: Cinemática dúvida
>
>
> Olá professor! Estou prestando vestibular e minha dúvida é a
> seguinte:
> Na fórmula: s = so + vo.t + (1/2).a.t2, o que explica a
> velocidade ser igual a inicial e nao a final p[or exemplo?
>
> No seu site encontrei a seguinte explicação:
> 21.2. Parâmetro B - é o valor da derivada da função horária
> para t = 0, o que o identifica com a velocidade inicial do
> movimento: B = vo .
> Lembramos: v = ds/dt = B + 2C.t e, para t=0, v = vo = B.
>
> 21.3. Parâmetro C - é o valor da derivada segunda da função
> horária, para qualquer t, e identifica-se como a metade do
> valor da aceleração escalar: C = a/2 .
> Lembramos: a= d2s/dt2 = dv/dt = 2C , donde, C = a/2.
>
> e segundo essa explicação, o que entendi é que então o
> istante t que acompanha o V0 é diferente do instante t que é
> elevado ao quadrado e acompanha a aceleração, matematicamente
> tal conclusão é impossível pois temos a seguinte
> possibilidade:
> s = s1 + v1.(t-t1) + a(t-t1)2/2
>
> Gostaria também de saber porque que o (t-t1) que acompanha a
> aceleração é elevado ao quadrado.
> Muito obrigada!
>
>
>
>
>
>
_____________________________________________________________________
_____
> Acabe com aquelas janelinhas que pulam na sua tela.
> AntiPop-up UOL - É grátis!
> http://antipopup.uol.com.br/





##### ##### #####

Para saber mais visite
http://www.ciencialist.hpg.ig.com.br


##### ##### ##### #####
Links do Yahoo! Grupos










SUBJECT: Fw: Manometro
FROM: "Luiz Ferraz Netto" <leobarretos@uol.com.br>
TO: "ciencialist" <ciencialist@yahoogrupos.com.br>
DATE: 29/12/2004 09:56

quem quer escrever algo sobre isso?
[]'
===========================
Luiz Ferraz Netto [Léo]
leobarretos@uol.com.br
http://www.feiradeciencias.com.br
===========================
-----Mensagem Original-----
De: <jadriano@ufpa.br>
Para: <leobarretos@uol.com.br>
Enviada em: terça-feira, 28 de dezembro de 2004 18:59


Olá,

Gostaria de saber o que é, para que serve e qual a finalidade em calibrar um
manometro.

grato. Adriano


SUBJECT: Fw: gerador eletrico
FROM: "Luiz Ferraz Netto" <leobarretos@uol.com.br>
TO: "ciencialist" <ciencialist@yahoogrupos.com.br>
DATE: 29/12/2004 10:14

É o pé do galo!

[]'
===========================
Luiz Ferraz Netto [Léo]
leobarretos@uol.com.br
http://www.feiradeciencias.com.br
===========================
-----Mensagem Original-----
De: Cadcam - Taquara
Para: leobarretos@uol.com.br
Enviada em: terça-feira, 28 de dezembro de 2004 08:27
Assunto: gerador eletrico


favor
posso transformar um motor eletrico em um transformador




Desde já obrigado
José

[As partes desta mensagem que não continham texto foram removidas]



SUBJECT: Re: Fw: gerador eletrico
FROM: César A. K. Grossmann <cesarakg@bol.com.br>
TO: ciencialist@yahoogrupos.com.br
DATE: 29/12/2004 11:18


> posso transformar um motor eletrico em um transformador

Resposta curta: depende do motor elétrico.

Resposta um pouco mais longa:

Tanto o motor elétrico quanto o gerador elétrico funcionam com o mesmo
princípio: um condutor girando em um campo magnético. A diferença
básica entre um e outro é que, no caso do motor, a energia é
introduzida pela corrente elétrica, e convertida em trabalho mecânico
no eixo do motor, e no caso do gerador, o trabalho mecânico no eixo do
motor que é convertido em energia elétrica.

Motores síncronos e assíncronos comuns, bem como motores de corrente
contínua podem ser convertidos em geradores, com pouca ou nenhuma
adaptação (caso de motores com imãs ou magnetos permanentes). Motores
em que o campo magnético tem que ser gerado por corrente elétrica
precisam que seja aplicada alguma energia elétrica na armadura dos
pólos magnéticos para que comecem a funcionar como geradores.

Um dos tipos de motores que não pode ser convertido em gerador, que eu
saiba, é o motor universal. Motores universais tem a armadura em
curto-circuito, e isoladas do exterior. É o caso da maioria dos
motores de eletrodomésticos.

Resposta longa e exata: quem puder e tiver tempo, pode elaborar uma
resposta mais longa...

[]s
--
.o. César A. K. Grossmann
..o http://www.LinuxByGrossmann.cjb.net/
ooo





SUBJECT: VENTILADORES E EXAUSTORES
FROM: "murilo filo" <avalanchedrive@hotmail.com>
TO: ciencialist@yahoogrupos.com.br
CC: avalanchedrive@hotmail.com
DATE: 29/12/2004 12:08

Um outro Leo, um italiano, o Da Vinci, disse que com a água e com o ar, Vc
tem que experimentar. (xí, será que, sem querer, eu cheguei à raiz de um
apelido?)
Há muito empirismo nestas questões de trocas térmicas, e do meu palpitário
sai o seguinte:
- é a troca do ar ambiental que vai arrastar, diretamente, a energia térmica
indesejável do ambiente.
- um ventilador *insufla* e causa pressão positiva no ambiente, um exaustor,
é óbvio, retira ar, causa pressão negativa, e, muito importante, dependendo
do caso, causa menos turbulência interna. Já ví casos em que ambos são
combinados... é algo meio complicado e com armadilhas.
- vc deve ter idéia do delta negativo que vc quer impor à superfície ou
atmosfera dêste ambiente (suponho que vc esteja lidando com uma granja), o
volume do ar contido no ambiente, a temperatura inicial e a ideal, e a
facilidade de fluxo ou renovação do ar. (o melhor é estimar valores médios
de temperatura, p/não complicar)
- muito empiricamente, pode ser feita a seguinte estimativa: vc tem um
volume de ar, tem uma temperatura inicial, tem a teórica ideal escolhida
(que, no ambiente, suponho, nunca será inferior à temperatura do ar novo que
entra) e tem o *calor específico do ar*. Se vc é engenheiro, dá p/calcular
quantos kW/h, ou BTUs, ou kilocalorias, vc quer arrastar p/fora, partindo do
volume horário do ar retirado, ou trocado. MUITO EMPIRICAMENTE, vc vai
chegar a um valor conversível em HPs... colocando +10% de perdas e reservas,
vc vai chegar ao tamanho, ou consumo do *motor*, ou dos motores, do
ventilador(es), o que já é alguma coisa para começar. Supondo as perdas dos
hélices, uns 25%, vc terá aproximadamente o valor da energia elétrica a ser
disponibilizada só para esta função. É um balanço energético simples de
troca direta!
- com certeza, há fornecedores de sistemas completos ou parciais, simples e
sofisticados, que possuem boas tabelas, e, aposto, muitas são empíricas.
- recomendo consultar www.nei.com.br ou a própria revista, em papel, onde há
muita coisa.
Já ouví falar de granjas, se é que ainda pode ser êste o nome, que para
trucidarem 200mil aves/dia, numa só instalação (uma maravillha!), possuía a
capacidade instalada de 4000HP, quase tudo p/refrigeração e ventilação. Nada
é de graça!!! Se vc quizer uma entrada de ar em temperatura abaixo daquela
do ambiente externo, com certeza vc irá precisar de um sistema adicional de
dissipação, talvez por fluido refrigerado circulante e vc já estará entrando
no caro mundo das frigorias.
Favor aguardar pelos outros membros da lista! Se não pintar nada melhor,
isto aquí será um começo. abr/M. SP 29/dez/2004

From: "Luiz Ferraz Netto" <leobarretos@uol.com.br>
Reply-To: ciencialist@yahoogrupos.com.br
To: "ciencialist" <ciencialist@yahoogrupos.com.br>
Subject: [ciencialist] Fw: "VENTILADORES E EXAUSTORES"
Date: Wed, 29 Dec




SUBJECT: Cientistas dizem que terremoto pode ter afetado eixo da Terra
FROM: "Erich Weick" <erich_weick@yahoo.com.br>
TO: ciencialist@yahoogrupos.com.br
DATE: 29/12/2004 13:22


Movimento em placa tectônica teria tornado o planeta mais compacto

O terremoto que provocou milhares de mortes na Ásia pode ter acelerado
de maneira permanente a rotação da Terra – encurtando os dias em uma
fração de segundo – e provocado uma oscilação no eixo do planeta,
afirmaram cientistas norte-americanos nesta terça, dia 28.

Richard Gross, geofísico do Laboratório de Jato-Propulsão da Nasa, na
Califórnia, avaliou que uma movimentação de massa em direção ao
interior da Terra durante o terremoto de domingo fez o planeta girar
3 microsegundos (ou um milionésimo de segundo) mais rápido, e se
inclinar 2,5 cm em relação a seu eixo.

Quando uma grande placa tectônica sob o Oceano Índico foi forçada para
baixo de outra "isso teve o efeito de fazer a Terra mais compacta e
girar mais rápido", disse Gross.

O cientista afirmou que mudanças previstas pelo seu modelo
provavelmente são minúsculas demais para serem detectas pela rede de
posicionamento global por satélite que freqüentemente mede a rotação
da Terra, mas disse que os dados podem revelar uma leve oscilação.

Há muito tempo cientistas fazem teorias sobre o fato de que mudanças
na superfície da Terra poderia afetar sua rotação, mas nunca tiveram
meios precisos para provar isso, disse o sismólogo da Caltech, Hiroo
Kanamori.

As informações são da agência Reuters.





SUBJECT: terra
FROM: "E m i l i a n o C h e m e l l o" <chemelloe@yahoo.com.br>
TO: <ciencialist@yahoogrupos.com.br>
DATE: 29/12/2004 14:27

Qual é a composição química da terra (aquela dos vasos de flor) ?

[ ] 's do Emiliano Chemello
emiliano@quimica.net
http://www.quimica.net/emiliano
http://www.ucs.br/ccet/defq/naeq

" Rien ne se perd, rien ne se crée,
tout se transforme."

Antoine Laurent de Lavoisier (químico francês, 1743 - 1794)


[As partes desta mensagem que não continham texto foram removidas]



SUBJECT: VENTILADORES E EXAUSTORES
FROM: "murilo filo" <avalanchedrive@hotmail.com>
TO: ciencialist@yahoogrupos.com.br
CC: avalanchedrive@hotmail.com
DATE: 29/12/2004 14:40


Um outro Leo, um italiano, o Da Vinci, disse que com a água e com o ar, Vc
tem que experimentar. (xí, será que, sem querer, eu cheguei à raiz de um
apelido?)
Há muito empirismo nestas questões de trocas térmicas, e do meu palpitário
sai o seguinte:
- é a troca do ar ambiental que vai arrastar, diretamente, a energia térmica
indesejável do ambiente.
- um ventilador *insufla* e causa pressão positiva no ambiente, um exaustor,
é óbvio, retira ar, causa pressão negativa, e, muito importante, dependendo
do caso, causa menos turbulência interna. Já ví casos em que ambos são
combinados... é algo meio complicado e com armadilhas.
- vc deve ter idéia do delta negativo que vc quer impor à superfície ou
atmosfera dêste ambiente (suponho que vc esteja lidando com uma granja), o
volume do ar contido no ambiente, a temperatura inicial e a ideal, e a
facilidade de fluxo ou renovação do ar. (o melhor é estimar valores médios
de temperatura, p/não complicar)
- muito empiricamente, pode ser feita a seguinte estimativa: vc tem um
volume de ar, tem uma temperatura inicial, tem a teórica ideal escolhida
(que, no ambiente, suponho, nunca será inferior à temperatura do ar novo que
entra) e tem o *calor específico do ar*. Se vc é engenheiro, dá p/calcular
quantos kW/h, ou BTUs, ou kilocalorias, vc quer arrastar p/fora, partindo do
volume horário do ar retirado, ou trocado. MUITO EMPIRICAMENTE, vc vai
chegar a um valor conversível em HPs... colocando +10% de perdas e reservas,
vc vai chegar ao tamanho, ou consumo do *motor*, ou dos motores, do
ventilador(es), o que já é alguma coisa para começar. Supondo as perdas dos
hélices, uns 25%, vc terá aproximadamente o valor da energia elétrica a ser
disponibilizada só para esta função. É um balanço energético simples de
troca direta!
- com certeza, há fornecedores de sistemas completos ou parciais, simples e
sofisticados, que possuem boas tabelas, e, aposto, muitas são empíricas.
- recomendo consultar www.nei.com.br ou a própria revista, em papel, onde há
muita coisa.
Já ouví falar de granjas, se é que ainda pode ser êste o nome, que para
trucidarem 200mil aves/dia, numa só instalação (uma maravillha!), possuía a
capacidade instalada de 4000HP, quase tudo p/refrigeração e ventilação. Nada
é de graça!!! Se vc quizer uma entrada de ar em temperatura abaixo daquela
do ambiente externo, com certeza vc irá precisar de um sistema adicional de
dissipação, talvez por fluido refrigerado circulante e vc já estará entrando
no caro mundo das frigorias.
Favor aguardar pelos outros membros da lista! Se não pintar nada melhor,
isto aquí será um começo. abr/M. SP 29/dez/2004

From: "Luiz Ferraz Netto" <leobarretos@uol.com.br>
Reply-To: ciencialist@yahoogrupos.com.br
To: "ciencialist" <ciencialist@yahoogrupos.com.br>
Subject: [ciencialist] Fw: "VENTILADORES E EXAUSTORES"
Date: Wed, 29 Dec




SUBJECT: Re: [ciencialist] Re: Fw: Cinemática dúvida
FROM: "Prof. JC" <profjc2003@yahoo.com.br>
TO: <ciencialist@yahoogrupos.com.br>
DATE: 29/12/2004 14:52

É mais fácil, depois de

S = So + deltaS

Calcular o deltaS usando a propriedade deltaS = Área do gráfico vXt

Como a área desse gráfico será um trapézio (no caso geral), resulta que

deltaS = Vo.t + (a/2).t^2

E, finalmente

S = So + Vo.t + (a/2).t^2

A vantagem em se usar o cálculo da área é que, no caso mais geral, teremos:

S = So + Integral de V(t)

Onde V(t) é a função da velocidade.

Abraços,
Prof. JC


----- Original Message -----
From: "rmtakata" <rmtakata@altavista.net>
To: <ciencialist@yahoogrupos.com.br>
Sent: Wednesday, December 29, 2004 4:47 AM
Subject: [ciencialist] Re: Fw: Cinemática dúvida




--- Em ciencialist@yahoogrupos.com.br, "Luiz Ferraz Netto"
> -----Mensagem Original-----
> De: "contatosandra" <contatosandra@u...>
> Na fórmula: s = so + vo.t + (1/2).a.t2, o que explica a
> velocidade ser igual a inicial e nao a final p[or exemplo?

Poderemos usar a velocidade final, mas aih a formula tem outra cara.

Considerando-se q. o deslocamento deltaS eh dado pela diferenca entre
a posicao final S e a posicao inicial So, eh facil ver q. a posicao
final S depende da posicao inicial So e do deslocamento realizado.

deltaS = S - So (1)

S = So + deltaS (2)

Ok, precisamos entao calcular o deslocamento deltaS realizado. Para
velocidade constante eh facil notar pela definicao de velocidade - o
qto um corpo se desloca em um determinado tempo (a taxa de alteracao
da posicao) q. o deslocamento serah dado pela velocidade multiplicada
pelo intervalo de tempo:

V = deltaS/deltat (3)

deltaS = V.deltat (4)

(Para simplificar consideremos q. o instante inicial to eh igual a 0,
assim deltat serah igual ao instante final t.)

deltaS = V.t (5)

Mas isso para velocidades constantes. No caso estamos considerando
velocidades q. variam uniformente no tempo - movimento uniformente
acelerado. Se o movimento eh acelerado, eh facil ver q. as velocidades
iniciais sao menores do q as velocidades finais pela definicao de
aceleracao:

a = dV/dt (6)

Como estamos considerando uma variacao uniforme, poderemos considerar
um intervalo maior em lugar de uma variacao infinitesimal (a
aceleracao serah igual em qq ponto dentro desse intervalo):

a = deltaV/deltat (7)

Considerando-se to=0

a = deltaV/t (8)

A aceleracao entao serah dada pela diferenca entre a velocidade final
V e a velocidade inicial Vo dividida pelo intervalo de tempo t

a =(V-Vo)/t (9)

Reescrevendo, poderemos obter a velocidade final V em funcao da
velocidade inicial Vo e intervalo t de tempo e a aceleracao a:

V = Vo + a.t (10)

Eh facil ver q. a velocidade final V, em um movimento acelerado serah
maior do q. a velocidade inicial Vo (e no caso de movimento
desacelerado serah menor). Eh tb facil notar q. qq velocidade em um
instante intermediario, serah uma velocidade de intensidade
intermediaria entre a velocidade final e a inicial - e seu valor serah
proporcional ao tempo decorrido desde o inicio.

Ok, para velocidades constantes notamos q. o deslocamento eh obtido
multiplicando-se a velocidade pelo intervalo de tempo:

deltaS = V.t (5)

Mas e no movimento em q a velocidade varia? Basta pegar a velocidade
media desempenhada pelo corpo no intervalo de tempo considerado. E
qual eh essa velocidade media? No caso de movimentos em q. a
velocidade varia uniformemente, poderemos considerar a velocidade
media como a media aritimetica entre a velocidade inicial Vo e a final
V (ou alternativamente, como a velocidade desempenhada pelo corpo na
metade do intervalo de tempo).

Vm = (V + Vo)/2 (11)

Substituindo-se (11) em (5) teremos:

deltaS = Vm.t = [(V + Vo)/2].t (12)

Desenvolvendo e reescrevendo:

deltaS = Vo.t/2 + V.t/2 (13)

E qual a velocidade final? Por (10) sabemos q:

V = Vo + a.t (10)

Substituindo-se (10) em (13), temos:

deltaS = Vo.t/2 + (Vo + a.t)t/2 (14)

Desenvolvendo e reescrevendo:

deltaS = Vo.t/2 + Vo/2 + (a.t).t/2 = Vo.t + (1/2).a.t^2 (15)

Substituindo (15) em (2), temos a equacao horaria do espaco para
movimentos uniformemente acelerados:

S = So + Vo.t + (1/2).a.t^2 (16)

Seguindo esses passos eh possivel se reescrever em funcao da
velocidade final V. E tb eh possivel se perceber de onde surge a
potencia quadratica do tempo a multiplicar a aceleracao a (bem como o
fator meio 1/2).

Posto de outro modo a posicao final S depende da posicao inicial So
(isso eh facil de se perceber, por exemplo, se o objeto nao se movesse
- em relacao ao referencial adotado - a posicao final seria igual 'a
inicial e se a posicao inicial estivesse mais perto do referencial, a
posicao final seria mais perto do referencial; se posicao inicial
fosse mais longe, a posicao final seria mais longe...), depende tb do
deslocamento proporcionado pelo movimento basal (representado pela
velocidade inicial - considere um movimento retilineo uniforme - isto
eh, sem aceleracao e veremos como a posicao final depende da
velocidade q. o objeto desempenha -- se o objeto se desloca
lentamente, pouco se moverah em relacao a posicao inicial; se se
desloca rapidamente, terah se afastado mais depois do mesmo tempo) e
depende ainda do deslocamento proporcionado pela velocidade q. se
acresce a cada instante 'a velocidade inicial (como eh uma taxa de uma
taxa, isto eh, uma taxa da variacao da velocidade q eh uma taxa da
variacao da posicao ou taxa de deslocamento, surge o t^2).

[]s,

Roberto Takata





##### ##### #####

Para saber mais visite
http://www.ciencialist.hpg.ig.com.br


##### ##### ##### #####
Links do Yahoo! Grupos












SUBJECT: inovacao tecnologica
FROM: "Osvaldo" <machado_jr@terra.com.br>
TO: "Ciencialist" <ciencialist@yahoogrupos.com.br>
DATE: 29/12/2004 16:16

Ola, alguem aqui conhece este site:



http://www.inovacaotecnologica.com.br
<http://www.inovacaotecnologica.com.br/>



é uma das minhas fontes de estudo das novidades em fisica e informatica.



Osvaldo Machado (autodidata)

Cuiaba-MT





[As partes desta mensagem que não continham texto foram removidas]



SUBJECT: Re: [ciencialist] Fw: gerador eletrico
FROM: "Alvaro Augusto - Electra" <alvaro@electraenergy.com.br>
TO: <ciencialist@yahoogrupos.com.br>
DATE: 29/12/2004 23:09

Não precisa fazer muita coisa, pois todo motor elétrico já é um transformador:

a) Se for um motor de indução, já se trata essencialmente de um "transformador cujo secundário gira". Se for um motor de anéis, basta coletar a corrente nos anéis. Se foi um motor de gaiola, bem, será necessário remodelar completamente o rotor;

b) Se for um motor síncrono (improvável), basta alimentar a armadura (estator) com AC e coletar a corrente nos anéis do rotor, que deve ter a excitação deligada. O problema é que o circuito do campo (rotor) não suportará a corrente nominal da armadura, além de ser monofásico, visto que na operação convencional trata-se de um enrolamento DC;

c) Se for um motor DC, a coisa será mais complicada. Será necessário retificar a corrente AC antes de injetá-la na armadura (que, aqui, é o rotor). Dentro das bobinas da armadura, por causa da comutação, a corrente será novamente AC. Depois é só coletar a corrente no enrolamento de campo (nesse caso, o estator). Se o motor for construído para operar em série (motor universal), o enrolamento de campo suportará a corrente nominal da armadura, mas imagino que a potência nominal nunca vá aparecer no campo, mesmo com rotor bloqueado. Além disso, a produção de harmônicos poderá ser considerável, pois o motor DC não é projetado para trabalhar com corrente senoidal;

Assim, todo motor é um transformador, pois a lei de Faraday opera o tempo todo. O problema será com o fluxo de potência dentro da máquina. Mesmo com o rotor a vazio, uma parte da potência será absorvida para manter as perdas rotacionais, diminuindo o rendimento. Além disso, será muito difícil operar um dispositivo desses com potência nominal, pois motores são construídos de forma que a potência flua pelo rotor, não pelo circuito de excitação. Uma maneira de aumentar a potência seria bloquear o rotor, mas isso acarretaria problemas de superaquecimento, etc. Imagino que não há maneira de fazer uim dispositivos desses trabalhar como um transformador real, com rendimento nominal e tudo mais. É um bom exercício para uma prova de máquinas elétricas, nada mais.

[ ]s

Alvaro Augusto

----- Original Message -----
From: Luiz Ferraz Netto
To: ciencialist
Sent: Wednesday, December 29, 2004 10:14 AM
Subject: [ciencialist] Fw: gerador eletrico


É o pé do galo!

[]'
===========================
Luiz Ferraz Netto [Léo]
leobarretos@uol.com.br
http://www.feiradeciencias.com.br
===========================
-----Mensagem Original-----
De: Cadcam - Taquara
Para: leobarretos@uol.com.br
Enviada em: terça-feira, 28 de dezembro de 2004 08:27
Assunto: gerador eletrico


favor
posso transformar um motor eletrico em um transformador




Desde já obrigado
José

[As partes desta mensagem que não continham texto foram removidas]



##### ##### #####

Para saber mais visite
http://www.ciencialist.hpg.ig.com.br


##### ##### ##### #####


Yahoo! Grupos, um serviço oferecido por:
PUBLICIDADE




------------------------------------------------------------------------------
Links do Yahoo! Grupos

a.. Para visitar o site do seu grupo na web, acesse:
http://br.groups.yahoo.com/group/ciencialist/

b.. Para sair deste grupo, envie um e-mail para:
ciencialist-unsubscribe@yahoogrupos.com.br

c.. O uso que você faz do Yahoo! Grupos está sujeito aos Termos do Serviço do Yahoo!.



[As partes desta mensagem que não continham texto foram removidas]



SUBJECT: Primavera
FROM: "Luiz Ferraz Netto" <leobarretos@uol.com.br>
TO: "ciencialist" <ciencialist@yahoogrupos.com.br>
DATE: 30/12/2004 12:25

Qual a data do início da primavera no hemisfério sul?

[]'
===========================
Luiz Ferraz Netto [Léo]
leobarretos@uol.com.br
http://www.feiradeciencias.com.br
===========================


SUBJECT: Re: [ciencialist] VENTILADORES E EXAUSTORES
FROM: "Luiz Ferraz Netto" <leobarretos@uol.com.br>
TO: <ciencialist@yahoogrupos.com.br>
CC: <avalanchedrive@hotmail.com>
DATE: 30/12/2004 13:15

Interessante (!) como pouca gente conhece a NEI. Estou com o exemplar de outubro de 2004 aqui na minha lateral esquerda.
[]'
===========================
Luiz Ferraz Netto [Léo]
leobarretos@uol.com.br
http://www.feiradeciencias.com.br
===========================
-----Mensagem Original-----
De: "murilo filo" <avalanchedrive@hotmail.com>
Para: <ciencialist@yahoogrupos.com.br>
Cc: <avalanchedrive@hotmail.com>
Enviada em: quarta-feira, 29 de dezembro de 2004 12:08
Assunto: [ciencialist] VENTILADORES E EXAUSTORES



Um outro Leo, um italiano, o Da Vinci, disse que com a água e com o ar, Vc
tem que experimentar. (xí, será que, sem querer, eu cheguei à raiz de um
apelido?)
[ ... ]


SUBJECT: RE: [ciencialist] Primavera
FROM: "murilo filo" <avalanchedrive@hotmail.com>
TO: ciencialist@yahoogrupos.com.br
DATE: 30/12/2004 13:43

É no equinócio, +/- em 21 ou 22/set., 3 meses antes do solstício, em dez.
Equinócio vc sabe o que é, não? Fazfavô, Léo!
abr/M. SP 30/dez/04

>From: "Luiz Ferraz Netto" <leobarretos@uol.com.br>
>Reply-To: ciencialist@yahoogrupos.com.br
>To: "ciencialist" <ciencialist@yahoogrupos.com.br>
>Subject: [ciencialist] Primavera
>Date: Thu, 30 Dec 2004 12:25:36 -0200
>
>Qual a data do início da primavera no hemisfério sul?
>
>[]'
> ===========================
> Luiz Ferraz Netto [Léo]
> leobarretos@uol.com.br
> http://www.feiradeciencias.com.br
> ===========================




SUBJECT: on topics
FROM: "murilo filo" <avalanchedrive@hotmail.com>
TO: ciencialist@yahoogrupos.com.br, forum-ciencia@yahoogrupos.com.br
DATE: 30/12/2004 14:20

Oi, gente!
Não desfazendo dos outros e considerando que ninguém sabe tudo, recomendo
também o site abaixo é muito do porreta!

http://hyperphysics.phy-astr.gsu.edu/hbase/hph.html

Boa sorte. :] M.




SUBJECT: Re: [ciencialist] VENTILADORES E EXAUSTORES
FROM: "murilo filo" <avalanchedrive@hotmail.com>
TO: leobarretos@uol.com.br, ciencialist@yahoogrupos.com.br
DATE: 30/12/2004 14:44

Léo, oi.
Eu não sei como é que há gente que consegue VIVER sem uma revista como a
Nei!
Sua utilidade é algo de difícil explicação! Com a PS também!
A propósito, eu não vi chegar p/a lista minha mensagem sob êste título.
Vc moderou o texto? Vc não gostou? Fique à vontade p/malhar! Pelas leis de
termodinâmica (conservação), acho que não falei grandes besteiras!
abr/M. SP 30/dez./2004

>From: "Luiz Ferraz Netto" <leobarretos@uol.com.br>
>Reply-To: "Luiz Ferraz Netto" <leobarretos@uol.com.br>
>To: <ciencialist@yahoogrupos.com.br>
>CC: <avalanchedrive@hotmail.com>
>Subject: Re: [ciencialist] VENTILADORES E EXAUSTORES
>Date: Thu, 30 Dec 2004 13:15:09 -0200
>
>Interessante (!) como pouca gente conhece a NEI. Estou com o exemplar de
>outubro de 2004 aqui na minha lateral esquerda.
>[]'
> ===========================
> Luiz Ferraz Netto [Léo]
> leobarretos@uol.com.br
> http://www.feiradeciencias.com.br
> ===========================
>-----Mensagem Original-----
>De: "murilo filo" <avalanchedrive@hotmail.com>
>Para: <ciencialist@yahoogrupos.com.br>
>Cc: <avalanchedrive@hotmail.com>
>Enviada em: quarta-feira, 29 de dezembro de 2004 12:08
>Assunto: [ciencialist] VENTILADORES E EXAUSTORES
>
>
>
>Um outro Leo, um italiano, o Da Vinci, disse que com a água e com o ar, Vc
>tem que experimentar. (xí, será que, sem querer, eu cheguei à raiz de um
>apelido?)
>[ ... ]




SUBJECT: Re: refrigerante
FROM: Maria Natália <grasdic@hotmail.com>
TO: ciencialist@yahoogrupos.com.br
DATE: 31/12/2004 02:49


Emiliano:

Não me esqueci de ti. Só que estamos a aproveitar as férias para
fazer os Exames Nacionais de Química de acesso à Universidade e
tenho a visita do André a Portugal entre mãos...
O que me parece se passa:
1-- O refrigerante que sai da garrafa a 25ºC,A, e o refrigerante que
sai da garrafa a 5ºC,B, estão em condições idênticas se abriste uma
garrafa bem vedada. Quer isto dizer que as duas garrafas contêm
quantidades de ar e CO2 saturada em vapor de água. A carica vedava
bem. Sem esta condição não há questão que se ponha;
2--O copo e o ambiente nos dois casos está á mesma temperatura e que
é a ambiente.E será 25ºC para não complicar e por ser um país
tropical
3--PV=nRT e n/V= p/RT mas n/V é a concentração:
[gas A]= [gas B] devido a 1-
PA/TA =PB/TB como TA >TB ===> PA>PB o que não é de admirar a pressão
do gas em A é maior do que a pressão do gas em B
4--Entretanto quando se tira a rolha de A e de B a pressão
atmosféria, Pexterior, é igual inferior a PA e a PB. Abre a rolha de
A a 25ºC e vai ter de descer a Pexterior PA-Pexterior=Xna garrafa A
(25ºC) ;
A 5ºC teremos Pexterior-Pexterior=Yna garrafa B como Pexterior é
igual, 3--, PA>PB vem X>Y (8-3>5-3). De acordo com a lei de Henri
e que diz que a [gas]=kP, quanto menor a pressão maior a
concentração, resulta que a concentração em A é > que a concentração
em B daí veres mais gás a sair.
c.q.d.

[]'
Maria Natália

"Tenho a teoria de que os nossos únicos actos originais são os erros"
Billy,Joel



>
> ################
>
> Por que o refrigerante, quando quente, faz 'mais espuma' quando
transferido
> da garrafa para o copo do que o refrigerante 'mais gelado'? Admite-
se como
> 'quente' uma temperatura de 25 ºC e o 'mais gelado' como sendo a
temperatura
> média de um congelador, que eu chuto ser algo entorno de 5 ºC
>
> ################
>
> [ ]'s do
>
> Emiliano Chemello
> emiliano@q...
> http://www.quimica.net/emiliano
> http://www.ucs.br/ccet/defq/naeq
> [ MSN ] chemelloe@h...
> [ ICQ ] 145060604
>
> "Rien ne se perd, rien ne se crée, tout se transforme"
> Lavoisier, químico francês (1743-1794)
>
> ----- Original Message -----
> From: Maria Natália
> To: ciencialist@yahoogrupos.com.br
> Sent: Monday, December 27, 2004 10:34 PM
> Subject: [ciencialist] Re: refrigerante
>
>
>
> L.E.R:
>
> 1-E qual será a temperatura ambiente em sala de enchimento de
> produtos gaseificados nas fábricas?
> 2-A soubilidades dos gases em água* normalmente diminui quando a
> temperatura aumenta, considerando uma dada pressão que permanece
> constante durante o processo. Quando se aquece água num copo ou
> recipiente de Pyrex se vê a formação de bolhas de ar junto à parede
> de vidro antes de a água entre em ebulição. Da próxima vez que
> aquecermos a água em nossos balões de laboratório ou nas máquinas
de
> café de vidro vejamos se as bolhas de ar dissolvidas não estão a
> sair da solução antes desta entrar em ebulição.
> E alguém aqui é pescador? Sai com a caninha de pesca para
> dar "banho" à minhoca (isco)? Num dia quente de verão, uma
pescadora
> experiente (e física) escolhe normalmente zonas profundas do lago
ou
> curso de água pois sabe que os peixes escolhem as zonas fundas
> porque a quantidade de oxigénio existente nas zonas frias é maior
do
> que nas zonas quentes (da superfície). Pois é, peixe sofre e tem
> metabolismo sensível a estas variações de concentração de oxigénio
> dissolvido: a solubilidade diminui com a temperatura! Além de terem
> de fugir de uma BOA pescadora e que até sabe física...
> Boas pescas neste fim de ano frio nooso e quente vosso.
> Maria Natália, "matadora" de peixes...e não só...
> * entendo por refrigerante qualquer bebida que tem por solvente a
> agua como vinho, sucos, cerveja...
>
>
> --- Em ciencialist@yahoogrupos.com.br, "L.E.R.de Carvalho"
> <lecarvalho@i...> escreveu
> > At 07:59 27/12/2004, you wrote:
> > > Por que o refrigerante, quando quente, faz 'mais espuma'
> quando
> > > transferido da garrafa para o copo do que o refrigerante 'mais
> gelado'?
> > > Admite-se como 'quente' uma temperatura de 25 ºC e o 'mais
> gelado' como
> > > sendo a temperatura média de um congelador, que eu chuto ser
> algo entorno
> > > de 5 ºC
> > >
> > >[ ] 's do Emiliano Chemello
> >
> >
> >
> > Uma geladeira comum tem uma temperatura acima de 5º C.
> > E o seu congelador tem uma temperatura bem abaixo dessa, talvez
em
> torno de
> > 10ºC.
> >
> > Só te dou uma dica: os refrigerantes, em geral, são colocados
> dentro da
> > garrafa, já resfriados, com a água bem gelada.
> >
> > (Pega êle, Físico-Química...)
> >
> > L.e.
> >
> > [As partes desta mensagem que não continham texto foram
removidas]
>
>
>
>
>
> ##### ##### #####
>
> Para saber mais visite
> http://www.ciencialist.hpg.ig.com.br
>
>
> ##### ##### ##### #####
>
>
> Yahoo! Grupos, um serviço oferecido por:
>
>
>
>
>
>
>
>
> Links do Yahoo! Grupos
>
> Para visitar o site do seu grupo na web, acesse:
> http://br.groups.yahoo.com/group/ciencialist/
>
> Para sair deste grupo, envie um e-mail para:
> ciencialist-unsubscribe@yahoogrupos.com.br
>
> O uso que você faz do Yahoo! Grupos está sujeito aos Termos do
Serviço do
> Yahoo!.





SUBJECT: Re: Fw: Manometro
FROM: "tipoalgo" <tipoalgo@bol.com.br>
TO: ciencialist@yahoogrupos.com.br
DATE: 31/12/2004 09:14


> Gostaria de saber o que é, para que serve e qual a finalidade em
calibrar um
> manometro.
>
> grato. Adriano

MANÔMETRO : qualquer instrumento usado para medir a pressão. Se bem
calibrado faz esta medição com maior precisão.

Para saber mais, uma consulta simples no google com a palavra
manometro retorna atualmente em português 18.400 páginas.

Abraços

Tipoalgo





SUBJECT: Enviando email: Navier-Stokes-quem se habilita?
FROM: "JVictor" <jvoneto@uol.com.br>
TO: <ciencialist@yahoogrupos.com.br>
DATE: 31/12/2004 15:54

Caros ciencialisteiros,

Eis algo em razão do que estudar. Prêmio: um milhãozinho de verdinhas, fora o ingresso para a imortalidade. Já gastei 1/2 caderno de e 4 preciosos cabelinhos! E nada!
Mas, estudar e pesquisar essa fantástica equação, descobrinho o que ela esconde já é um bom lazer.
Quem sabe não teremos um 2,76milionário(em nossa fraca moeda) aquí no ciencialist. Eis o endereço:

http://www.prandiano.com.br/html/m_nov.htm

Observação: para se proteger de vírus de computador, os programas de email podem impedir o envio ou recebimento de alguns tipos de anexo de arquivo. Verifique as configurações de email para determinar como os anexos são manipulados.
Feliz Ano Novo para todos e
Boa sorte!

JVictor.

[As partes desta mensagem que não continham texto foram removidas]



SUBJECT: Re: Fw: Cinemática dúvida
FROM: "rmtakata" <rmtakata@altavista.net>
TO: ciencialist@yahoogrupos.com.br
DATE: 01/01/2005 05:05


--- Em ciencialist@yahoogrupos.com.br, "Prof. JC"
> É mais fácil, depois de
>
> S = So + deltaS
>
> Calcular o deltaS usando a propriedade deltaS = Área do
> gráfico vXt

Certamente mais facil. Mas serah mais didatico? (Didatico aqui nem tto
no sentido de facil apreensao, mais no de pertinente 'a questao fisica
do problema.)

(Claro q. em havendo tempo habil apresentar os dois modo seria ainda
melhor - afinal para se entender de ciencia tb eh necessario enteder
as representacoes graficas, alem das algebricas.)

[]s,

Roberto Takata





SUBJECT: fastest current speed/event measurement
FROM: "murilo filo" <avalanchedrive@hotmail.com>
TO: ciencialist@yahoogrupos.com.br, forum-ciencia@yahoogrupos.com.br
DATE: 01/01/2005 12:39

( straight on topics, I mean, curtura téquina gerá!
De coração: um belíssimo 2005 p/todos!
A gente está precisando e já passou muito da hora...
Bração! Muliro SP 01/01/2005 )

>From: best bad example <geb353@yahoo.com>
>Reply-To: buddyrays_backporch@yahoogroups.com
>To: buddyrays backporch <buddyrays_backporch@yahoogroups.com>
>Subject: [buddyrays_backporch] Fwd: [Keelynet] fastest current speed/event
>measurement
>Date: Fri, 31 Dec 2004 22:25:56 -0800 (PST)
>
>--- Jerry Decker - KN <jwdecker@keelynet.com>
>wrote:
>
> > From: Jerry Decker - KN <jwdecker@keelynet.com>
> > To: "interact"
> > <interact@listserv.capital-master.com>
> > Subject: [Keelynet] fastest current speed/event
> > measurement
> > Date: Wed, 29 Dec 2004 16:26:11 -0600
> >
> > Hola Folks!
> >
> > An interesting question/comment about the
> > fastest events measurable,
> > might be useful for ZPE studies;
> >
> >
>http://www.sciam.com/askexpert_question.cfm?articleID=0009D3DB-F051-11C1-B05183414B7F0000&catID=3&chanID=sa005
> >
> > What is the fastest event (shortest time
> > duration) that can be measured
> > with today's technology and how is this done?
> >
> > R. Mitchell - Melbourne, Australia
> >
> > Scott Diddams and Tom O'Brian of the Time and
> > Frequency Division of the
> > National Institute of Standards and Technology,
> > explain.
> >
> > Just how fast an event is depends somewhat on
> > your point of view. In
> > nature around us there are various physical
> > events that occur on time
> > scales from the yoctosecond (10-24 second) to
> > the exasecond (1018
> > second). In the time it just took your heart to
> > beat once, the computer
> > on the desk next to you completed about one
> > billion clock cycles,
> > whereas the electron of a hydrogen atom could
> > have circled its proton
> > about 1 quadrillion (1015) times.
> >
> > On the other hand, that very slow heart beat is
> > actually quite fast and
> > fleeting if one considers it relative to the
> > 500 quadrillion (500 x
> > 1015) second lifetime of our universe. Within
> > this tremendous range of
> > time scales, science and technology, which are
> > constantly improving,
> > determine how accurately different events can
> > be measured or inferred.
> >
> > For example, in the late 19th century, the best
> > scientists and
> > technologists struggled to measure time
> > intervals on the order of a
> > hundredth or thousandth of a second. In a well
> > known (and often
> > mythologized) story, photography pioneer
> > Eadweard Muybridge, on
> > commission from Leland Stanford, took several
> > years to develop a system
> > of rapid-sequence photography to conclusively
> > prove that a galloping or
> > trotting horse briefly has all four feet aloft
> > simultaneously--an event
> > too fast for the human eye to follow. Muybridge
> > was able to perfect his
> > system to record events on the scale of about
> > 0.001 second in 1877.
> >
> > But this story also points out a challenge in
> > answering the original
> > question: The answer depends on how one
> > interprets the word "measured."
> > This might sound like a pedantic dodge, but at
> > the National Institute of
> > Standards and Technology (NIST) we spend a lot
> > of time trying to
> > understand and apply the subtleties of
> > measurement.
> >
> > Muybridge's photography was a record of short
> > duration events--possibly
> > the best such record of its time--but was not a
> > measurement of time
> > interval in the strict sense. Both the
> > recording or inference of short
> > duration events and accurate measurements of
> > such events are of
> > interest, so we suggest rephrasing the original
> > question into two new
> > questions: "What are the shortest time
> > durations that can be measured
> > with a particular accuracy?" and "What are the
> > shortest duration events
> > that can be recorded or inferred in
> > experiments?"
> >
> > THE TIME SCALE on which a variety of physical
> > events in nature occur
> > spans about 42 orders of magnitude. This makes
> > the definition of fast
> > somewhat relative.
> >
> > To best answer the former question about
> > measurement with a particular
> > accuracy, let us agree to define measurement as
> > a comparison to a
> > generally accepted standard. By international
> > treaty the standard for
> > the second, the unit of time, is defined as
> > exactly 9,192,631,770 cycles
> > of a particular electron transition in
> > cesium-133 atoms. So a time
> > measurement is a direct or indirect comparison
> > to this defined standard.
> >
> > Currently, the best technical approach to
> > measuring time against this
> > standard is to use laser-cooled cesium atomic
> > fountain frequency
> > standards, known as cesium atomic fountain
> > clocks. The handful of these
> > cesium atomic fountain devices operating around
> > the world are actually
> > frequency standards rather than clocks
> > (timekeeping devices), and they
> > are used to realize the defined cesium standard
> > frequency with
> > exceptional accuracy of about 1 part in 1015.
> >
> > The best reported uncertainty at the time of
> > this writing is about 6 x
> > 10-16 for the NIST-F1 fountain standard.
> > Because 86,400 seconds make up
> > one day, this relative uncertainty means the
> > standard is accurate to
> > about 50 picoseconds (50 x 10-12 second) per
> > day.
> >
> > Put another way, if the frequency standard
> > could be operated
> > indefinitely as a clock it would neither gain
> > nor lose more than a
> > second in 50 million years compared to a
> > perfect clock.
> >
> > Cesium atomic fountain standards are the
> > world's most accurate primary
> > standards of any kind. No other
> > standard--including ones used for
> > length, mass and electrical current--has an
> > accuracy within even a
> > factor of 1,000 of the cesium atomic fountain
> > clock.
> >
> > The atomic fountain standard uncertainty of
> > about 1 x 10-15 might seem
> > to imply that these "clocks" could be used to
> > measure events on the
> > order of femtoseconds (10-15 second), but in
> > fact fountain standards are
> > not generally useful for directly measuring
> > short-duration events.
> >
> > (there are more pages for this article at the
> > website above)
> >
> > --
> > Jerry Decker -
> > http://www.keelynet.com
> > Public Archive
> > http://www.escribe.com/science/keelynet
> > Order out of Chaos - From an Art to a
> > Science
> >
> >
>
>
>
>
>__________________________________
>Do you Yahoo!?
>Jazz up your holiday email with celebrity designs. Learn more.
>http://celebrity.mail.yahoo.com




SUBJECT: Re: Fw: Cinemática dúvida
FROM: "profjc2003" <profjc2003@yahoo.com.br>
TO: ciencialist@yahoogrupos.com.br
DATE: 01/01/2005 18:56


Oi Takata,

Sim, certamente o método da área do gráfico Vxt é mais didático do
que o desenvolvimento puramente algébrico da função S(t). Por meio do
gráfico pode-se, por exemplo, discutir todo o desenvolvimento
histórico do tema, recuperando os próprios experimentos de galileu e
suas rampas. Além disso o método gráfico é abrangente, generalizante
e tem como subproduto o possível desenvolvimento algébrico a título
de "curiosidade". Não bastasse tudo isso esse método (gráfico) também
permite soluções simples para problemas complexos sobre movimentos de
diferentes tipos realizados em várias etapas e de "alta complexidade"
quando resolvidos apenas com o uso das equações cinemáticas.

Resumindo: sim.

Abraços,
Prof. JC



--- Em ciencialist@yahoogrupos.com.br, "rmtakata" <rmtakata@a...>
escreveu
>
> --- Em ciencialist@yahoogrupos.com.br, "Prof. JC"
> > É mais fácil, depois de
> >
> > S = So + deltaS
> >
> > Calcular o deltaS usando a propriedade deltaS = Área do
> > gráfico vXt
>
> Certamente mais facil. Mas serah mais didatico? (Didatico aqui nem
tto
> no sentido de facil apreensao, mais no de pertinente 'a questao
fisica
> do problema.)
>
> (Claro q. em havendo tempo habil apresentar os dois modo seria ainda
> melhor - afinal para se entender de ciencia tb eh necessario enteder
> as representacoes graficas, alem das algebricas.)
>
> []s,
>
> Roberto Takata





SUBJECT: Ano Novo
FROM: "profjc2003" <profjc2003@yahoo.com.br>
TO: ciencialist@yahoogrupos.com.br
DATE: 01/01/2005 18:57


Ai que dor de cabeça.

Abraços,
Prof. JC






SUBJECT: Re: [ciencialist] Enviando email: Navier-Stokes-quem se habilita?
FROM: "Rodrigo Toledo" <rodrigotoledo11@uol.com.br>
TO: <ciencialist@yahoogrupos.com.br>
DATE: 02/01/2005 12:06

Vitor,

Caso queira tentar outras oportunidades de entrar para a imortalidade ( Com
muito dinheiro !! ), existem outros desafios igualmente, digamos,
complicados.

[]'s

RT

http://www.claymath.org/millennium/
----- Original Message -----
From: "JVictor" <jvoneto@uol.com.br>
To: <ciencialist@yahoogrupos.com.br>
Sent: Friday, December 31, 2004 3:54 PM
Subject: [ciencialist] Enviando email: Navier-Stokes-quem se habilita?



Caros ciencialisteiros,

Eis algo em razão do que estudar. Prêmio: um milhãozinho de verdinhas, fora
o ingresso para a imortalidade. Já gastei 1/2 caderno de e 4 preciosos
cabelinhos! E nada!
Mas, estudar e pesquisar essa fantástica equação, descobrinho o que ela
esconde já é um bom lazer.
Quem sabe não teremos um 2,76milionário(em nossa fraca moeda) aquí no
ciencialist. Eis o endereço:

http://www.prandiano.com.br/html/m_nov.htm

Observação: para se proteger de vírus de computador, os programas de email
podem impedir o envio ou recebimento de alguns tipos de anexo de arquivo.
Verifique as configurações de email para determinar como os anexos são
manipulados.
Feliz Ano Novo para todos e
Boa sorte!

JVictor.

[As partes desta mensagem que não continham texto foram removidas]



##### ##### #####

Para saber mais visite
http://www.ciencialist.hpg.ig.com.br


##### ##### ##### #####
Links do Yahoo! Grupos










SUBJECT: Evolucao e desafios - CartaCapital - 02/01/05
FROM: "L.E.R.de Carvalho" <lecarvalho@infolink.com.br>
TO: ciencialist@yahoogrupos.com.br
DATE: 02/01/2005 13:52


>LabConsS - www.ufrj.br/consumo
>
>
>
> EVOLUÇÃO E DESAFIOS
>
>
> O País viu inúmeros avanços nas áreas médica e
>tecnológica, mas as carências sociais ainda são sua
>pior mazela
>
>Em uma livraria, olho rapidamente as manchetes das
>revistas expostas. A julgar pelas capas, é pouco
>provável que tenhamos problemas de saúde deste mês em
>diante. Soluções curativas. Remédios extremamente
>eficientes. Vida prolongada. Corpos e rostos sem um
>defeito. Independentemente da idade, claro. Câncer?
>Uma brincadeira. Paralisia? Pode preparar as pistas.
>Infarto? E daí?
>
>Deixando de lado uma razoável dose de exagero nessas
>manchetes, encho o peito de orgulho pelas proezas
>atingidas no campo da medicina e da saúde. Vinte anos
>atrás, tudo isso não passaria de sonho, e não
>existiria fora da cabeça imaginativa dos produtores de
>filmes de ficção científica. Mas, ainda hoje, para a
>esmagadora maioria dos cidadãos brasileiros, todo o
>acima mencionado não passa de um sonho, de uma ficção
>científica.
>
>Este ano que acaba nos deu boas notícias,
>encorajadoras, e notícias ruins, no mínimo
>preocupantes. Vimos uma senhora que sofreu um derrame
>cerebral voltar a andar com terapia baseada na
>introdução de células-tronco (células primitivas
>capazes de se transformar em praticamente qualquer
>outra célula normal) no cérebro afetado. Emocionante
>perceber o que esses pequenos passos podem significar
>para milhões de pessoas que não conseguem erguer o
>braço, ou mexer a perna. Parafraseando Neil Armstrong,
>um passo gigante para a humanidade.
>
>A técnica das células-tronco não se restringe a
>restaurar a função de células cerebrais. Hoje em dia,
>pesquisadores brasileiros, e em outros países,
>intensificam seus esforços para melhorar o coração
>depois de infarto, os nervos após lesão traumática, a
>pele após queimadura. A lista parece não ter fim.
>
>Vimos a introdução de remédios geniais na prática
>médica. Geniais na sua concepção, e geniais na sua
>eficiência. Drogas que conseguem agir em um ponto
>específico da célula doente, da célula cancerosa,
>dificultando seu desenvolvimento, seu crescimento, e
>até provocando sua morte. Exemplos que estão
>progressivamente sendo utilizados na prática médica,
>apesar de somente em casos muito selecionados, não
>faltam. Glivec, Iressa, são alguns deles.
>
>Ainda nem acabou o ano e ouvimos há poucas semanas o
>anúncio pela GlaxoSmithKline de uma vacina contra o
>câncer. O Brasil está totalmente empenhado na produção
>da vacina contra o HPV, vírus causador de câncer de
>útero. O prof. dr. Ricardo Brentani, presidente da
>Fundação Antonio Prudente, Hospital do Câncer AC
>Camargo, está entusiasmado e declara:
>
>­ Felizmente nossa instituição foi parceira da Merck
>Sharp & Dohme no desenvolvimento de uma vacina contra
>o HPV. Minha esperança é que em 20 anos tenhamos
>prevenido 7% dos tumores humanos.
>
>Vimos, por outro lado, a preocupação crescente dos
>cientistas em não excluir, a priori, tratamentos
>considerados até então não-convencionais, ou
>complementares. Estudos sérios avaliaram desde a
>acupuntura até a homeopatia, passando por terapias de
>Florais de Bach e tratamentos com vitaminas nas mais
>diversas situações clínicas. Vantagens e desvantagens
>de cada abordagem foram dissecadas minuciosamente.
>
>Cientistas conseguiram demonstrar a eficiência
>incontestável de algumas terapias alternativas em
>certas situações, como a massagem para um dos males do
>século, a dor nas costas. Por outro lado, alertaram
>para a ineficiência e até os efeitos nocivos de outras
>terapias, em outras condições. Por exemplo, para
>tratar bronquite e asma a acupuntura não parece ter
>efeitos importantes. Pelo menos não conseguiram
>detectar esses efeitos nos estudos atuais.
>
>Vimos o lançamento de aparelhos ultramodernos,
>ultra-sensíveis, para detectar doenças e tratá-las.
>Não consigo perceber avanço recente maior do que na
>área da radioterapia. É incrível a precisão dos feixes
>de radiação em atingir o alvo, no caso o câncer, e
>poupar o tecido normal adjacente. Aparelhos novos
>conseguem acompanhar o movimento do corpo para seguir
>o alvo predeterminado, segundo a segundo, obedecendo à
>orientação do médico radioterapeuta. Reduziram-se
>muito os efeitos colaterais. O controle do câncer
>assemelha-se às extensas cirurgias. A nanotecnologia
>(aparelhos miniaturas) é uma febre, e os estudos
>multiplicam-se para definir com mais clareza sua
>aplicação.
>
>A plástica está fazendo tamanhos milagres que nem os
>próprios pacientes conseguem acreditar. Tanto faz quem
>foram seus pais, ou a etnia a qual você pertença.
>Escolha o modelo e ficará parecido. Para pessoas
>ansiosas com a imagem, soluções para quase tudo. Sem
>dúvida, o impacto sobre o estado emocional é notável.
>
>Ao lado dessas notícias que mereceram um destaque
>quase obsessivo nas manchetes de capa, outras
>informações com menos, digamos, glamour, não
>conseguiram espaço nem nas páginas finais das
>revistas.
>
>O objetivo de reduzir em 60% a mortalidade infantil no
>mundo (e o Brasil ainda é um grande protagonista dessa
>estatística) não será atingido em 2015, como
>estabelecido nos Objetivos Milenares da ONU. No
>planeta, morrem por ano mais de 11 milhões de crianças
>com idade inferior a 5 anos. A maioria por doenças
>evitáveis. Diarréia, pneumonia, malária. No mesmo
>período, 500 mil mulheres morrem durante a gravidez ou
>o parto. Doença de Chagas e esquistossomose
>(barriga-d’água) continuam afetando milhares de
>brasileiros. E seu controle está cada vez mais
>próximo. A implementação dos programas já em ação
>poderá melhorar ainda mais esse controle. Para 2005 a
>intensificação dessas abordagens poderá elevar o
>impacto na saúde da população, principalmente nas
>áreas rurais.
>
>O Relatório Mundial de Saúde (The World Health Report
>2003) recomendou às autoridades o fortalecimento dos
>sistemas de saúde, centralizando seu foco na atenção
>primária, além de integrar a prevenção das doenças e a
>promoção da saúde em todos os níveis de atendimento.
>
>A saúde do homem sofreu mudanças drásticas nos últimos
>anos. A expectativa de vida, de brasileiros e de não
>brasileiros, bate recorde atrás de recorde.
>Ultrapassou a marca dos 70 anos e logo passará dos 80,
>90, e quem sabe 100 anos. Avanços sem dúvida notáveis.
>Doenças contagiosas foram substituídas por doenças
>crônicas, como problemas cardiovasculares e câncer, e
>por causas externas, como trauma. Essa mudança de foco
>exigiu modificações intensivas no nível estrutural.
>Algumas especialidades assumiram posição de destaque
>nunca antes alcançada. Os serviços de cardiologia de
>muitos hospitais logo se transformaram em
>departamentos e, a seguir, em grandes centros com
>prédios próprios. O mesmo ocorreu com a oncologia e
>suas variadas especialidades. Houve uma explosão na
>tecnologia e na sofisticação, e, conseqüentemente, nos
>custos ­ alertam os especialistas em saúde pública.
>Recomenda-se também a melhora da saúde da população
>por intermédio do envolvimento acadêmico.
>
>
>Muito por fazer.
>Como melhorar o atendimento ao público do SUS
>
>
>A academia médica no Brasil foi alterada sensivelmente
>nas últimas duas décadas. Progressivamente, as escolas
>médicas e os programas de pós-graduação introduziram
>disciplinas de pesquisa epidemiológica de problemas de
>saúde adaptados à realidade da população do País. Mais
>e mais livros têm sido publicados por acadêmicos
>brasileiros, com enfoque nacional. Mais e mais centros
>de treinamento oferecem vagas para médicos
>interessados em se aperfeiçoar no atendimento básico à
>saúde da família e à clínica geral. O impacto na
>melhora da atenção à saúde do brasileiro pode demorar
>a ser notado. Muito há de ser feito em 2005 e além.
>
>O plano Fome Zero tenta corrigir um problema básico de
>saúde: a miséria e a conseqüente desnutrição. Ninguém
>precisa de doutorado para compreender a conexão direta
>entre a desnutrição e a ocorrência de doenças
>potencialmente graves. É um plano interessante, mas
>ainda longe de atingir seus objetivos de forma
>significativa. Em 2005, e além, o governo deverá fazer
>os ajustes finos para que o programa consiga alcançar
>seu alvo.
>
>A violência é problema de segurança pública, mas
>também é problema de saúde. Não podemos esquecer que,
>no ano passado, mais de 40 mil brasileiros foram
>assassinados. Se acreditarmos nos dados oficiais do
>Ministério da Saúde, naquele período, morreram mais
>brasileiros por tiro do que por câncer de pulmão. O
>controle da violência evitará milhares de mortes e
>seqüelas dramáticas.
>
>A malária é outro problema sem solução a curto prazo.
>Áreas extensas do território nacional são infestadas
>por mosquitos portadores dessa doença. O esforço das
>autoridades regionais e federais deverá ser
>intensificado nas várias frentes: pesquisa de vacinas,
>profilaxia, controle do mosquito e tratamento de
>pacientes infectados. A mortalidade por essa infecção
>deve ser reduzida a todo custo. Estudos recentes
>aventam que a vacina contra a malária pode estar ao
>alcance da ciência em prazo razoável.
>
>No mundo todo, um bilhão de pessoas são infectadas por
>parasitas, como os vermes, e o Brasil tem participação
>significativa nesses dados alarmantes. Vermes no
>intestino não são apenas feios. Eles causam problemas
>como deficiências nutricionais, e, em alguns casos,
>podem até ser fatais. Para mudar isso, saneamento
>básico é fundamental.
>
>Um estudo recentemente realizado e publicado por
>pesquisadores na Universidade do Ceará demonstrou
>claramente a possibilidade de praticamente eliminar os
>parasitas dos pacientes. Remédios eficazes, como a
>ivemerctina, podem ser a solução. Além de tratar o
>paciente, eliminam uma potencial fonte de contaminação
>para outras pessoas. As autoridades devem criar
>sistemas integrados, contínuos, para atacar esse
>problema de saúde pública em várias frentes, em 2005 e
>para sempre.
>
>Quanto à Aids, nos últimos anos, houve um declínio da
>incidência de novos casos na maioria dos estados
>brasileiros. Estudo publicado recentemente por
>pesquisadores da Fundação Oswaldo Cruz, no Rio de
>Janeiro, confirmou que, apesar de recursos limitados
>do governo e da desigualdade socioeconômica aberrante
>no Brasil, a introdução do acesso universal à terapia
>antiviral contribuiu para uma redução impressionante
>na mortalidade por Aids, e pode ter ajudado a diminuir
>sua incidência.
>
>Nessa linha de pensamento, o presidente Lula lançou,
>este ano, o louvável Plano Nacional de Eliminação da
>Hanseníase (a lepra) em até dois anos. Um passo sem
>dúvida importante para controlar essa doença
>contagiosa e debilitante. Paralelamente, o ministro da
>Saúde, Humberto Costa, anunciou a formação de uma
>comissão para promover políticas nacionais de
>reabilitação às pessoas afetadas pela hanseníase:
>
>­ A idéia é possibilitar benefícios, como a
>reabilitação por cirurgias plásticas, financiando para
>a capacitação de médicos, enfermeiros e
>fisioterapeutas para que realizem esses procedimentos
>nos incapacitados fisicamente.
>
>Todos os anos, 42 mil brasileiros contraem a
>hanseníase. E o País só perde para a Índia no ranking
>mundial de casos da doença.
>
>A obesidade, mal do século XXI, mata. Nos EUA, em
>alguns estados o número de óbitos por obesidade
>ultrapassou o de câncer. No Brasil, cientistas
>identificaram obesidade crescente na população. Estudo
>publicado por pesquisadores da Universidade Federal do
>Rio de Janeiro confirmou o aumento progressivo do peso
>dos adolescentes, tanto na Região Nordeste quanto no
>Sudeste do País, desde 1975. Atualmente, 17% dos
>adolescentes da Região Sudeste enquadram-se na
>definição internacional de obesidade. E seus efeitos
>maléficos logo alcançarão a saúde pública. Orientação
>e conscientização devem ser política contínua e bem
>estruturada. A prevenção e o tratamento do sobrepeso
>precisam ser prioridades das políticas de saúde nos
>próximos anos.
>
>Um problema que acomete principalmente as crianças é a
>poluição atmosférica. Um estudo realizado com 5.193
>crianças de duas cidades do Rio de Janeiro, e
>publicado recentemente na revista Annals of Allergy
>Asthma and Immunology, mostrou claramente a correlação
>entre os níveis de poluição e a incidência de
>problemas respiratórios. Observou-se um aumento de
>mais de 50% na freqüência de crises de asma nos
>moradores de Duque de Caxias (local com alta
>concentração de poluentes), comparados aos moradores
>de Seropédica (local com baixos níveis de poluição).
>Outro estudo, realizado pelos pesquisadores do
>laboratório de poluição atmosférica da Universidade de
>São Paulo, confirmou os efeitos nocivos da poluição do
>ar na mortalidade infantil. Há aumento de 6% no número
>de óbitos neonatais nas regiões mais poluídas. O
>controle de poluentes de qualquer origem deverá ser
>obrigação mundial. Ainda mais com a entrada em vigor
>do Protocolo de Kyoto.
>
>Mas, de forma geral, provavelmente o maior desafio das
>autoridades de saúde é garantir um apoio ao avanço
>científico na área de saúde, atingir a maioria da
>população com os avanços médicos e tecnológicos
>alcançados, e insistir na prevenção e na saúde
>primária, básica. Muito mais do que dinheiro, em 2005
>precisamos de estratégia, filosofia e enfoque novos.
>
>O acesso da população, principalmente das camadas
>menos privilegiadas da sociedade, a remédios em geral
>é muito limitado. Várias são as causas, como preços
>elevados na fonte, acréscimos excessivos na cadeia de
>distribuição e impostos. Se as autoridades, de todas
>as esferas, não atentarem a essas dificuldades
>enfrentadas no dia-a-dia da maioria dos cidadãos,
>muito mais será gasto para tratar complicações graves
>de doenças crônicas, como diabetes e hipertensão
>arterial. O Brasil iniciou uma ação interessante nessa
>direção: a aquisição da fábrica da GlaxoSmithKline do
>Brasil pelo governo federal. O presidente Lula
>destacou que “pela primeira vez um governo compra uma
>fábrica da iniciativa privada”, ao mesmo tempo
>criticando “o processo inverso de privatização do
>governo anterior”. Lula declarou, na ocasião, que
>estava “recuperando uma fábrica que seria desativada,
>eliminaria empregos e que agora produzirá, em escala
>nacional, os antibióticos mais usados no Brasil”.
>
>Nessa mesma linha o ministro da Saúde, Humberto Costa,
>deixou muito clara sua estratégia para 2005:
>
>­ Vamos ampliar o acesso da população a medicamentos
>fundamentais com base em três opções. A primeira é a
>rede de farmácias populares, e até o fim do ano
>pretendemos inaugurar cem unidades em todo o Brasil. A
>segunda é a criação de um programa para a venda
>subsidiada de medicamentos básicos para hipertensão e
>diabetes, através da rede de farmácias privada.
>Pretendemos baixar os preços em até 50%. E a terceira
>é a redução do ICMS de 2,8 mil medicamentos até 2005.
>
>De acordo com uma pesquisa do IBGE, de 2003, a saúde
>aparece em terceiro lugar no orçamento das famílias
>brasileiras, e os medicamentos representam 61% desses
>gastos para as pessoas de baixa renda. Além disso,
>metade das pessoas que precisam de tratamento não pode
>pagar os remédios de que necessitam.
>
>A partir de 2005, 50 milhões de unidades de
>antibióticos serão produzidas para a rede do Sistema
>Único de Saúde (SUS) e para as Farmácias Populares. Em
>2007, a produção deve quintuplicar em relação à atual.
>Mais de 10 bilhões de unidades de medicamentos devem
>ser produzidas para as principais doenças que mais
>atingem a população brasileira, como hipertensão,
>diabetes, malária e tuberculose. A Fiocruz também
>produzirá mais vacinas, e o Ministério da Saúde
>enfatiza que “a ampliação do acesso da população aos
>medicamentos é uma das prioridades do governo
>federal”.
>
>Em 2004, muito se fez pela saúde, mas ainda há muito
>mais por fazer. Programas de prevenção e detecção
>precoce de doenças sexualmente transmissíveis, como
>Aids, HPV (câncer de colo de útero), a melhoria na
>qualidade do atendimento dos pacientes do SUS,
>diminuição das filas, controle de poluição, de
>infecção hospitalar e, também, de mortes por agentes
>externos, como traumas (tiros, facadas, acidentes de
>trânsito). Para se ter uma idéia do custo com os
>acidentes de trânsito, a cada ano acontecem mais de
>100 mil mortes, com três a quatro vezes esse número de
>feridos. E a maioria absoluta é tratada com dinheiro
>público. São milhões de reais de gastos evitáveis. Sem
>falar nas seqüelas individuais.
>


[As partes desta mensagem que não continham texto foram removidas]



SUBJECT: Re: [ciencialist] Evolucao e desafios - CartaCapital - 02/01/05
FROM: "Amauri Jr" <amaurijunior2@yahoo.com.br>
TO: <ciencialist@yahoogrupos.com.br>
DATE: 02/01/2005 15:18

Minhas analises em baixo com [A]....
----- Original Message -----
From: L.E.R.de Carvalho
To: ciencialist@yahoogrupos.com.br
Sent: Sunday, January 02, 2005 1:52 PM
Subject: [ciencialist] Evolucao e desafios - CartaCapital - 02/01/05



>LabConsS - www.ufrj.br/consumo
>
>
>
> EVOLUÇÃO E DESAFIOS
>
>
> O País viu inúmeros avanços nas áreas médica e
>tecnológica, mas as carências sociais ainda são sua
>pior mazela
>
>Em uma livraria, olho rapidamente as manchetes das
>revistas expostas. A julgar pelas capas, é pouco
>provável que tenhamos problemas de saúde deste mês em
>diante. Soluções curativas. Remédios extremamente
>eficientes. Vida prolongada. Corpos e rostos sem um
>defeito. Independentemente da idade, claro. Câncer?
>Uma brincadeira. Paralisia? Pode preparar as pistas.
>Infarto? E daí?

[A] temos que pagar e muito caro para a "ciencia" nos curar de males que diz a "ciencia" um direito de todos.

>
>Deixando de lado uma razoável dose de exagero nessas
>manchetes, encho o peito de orgulho pelas proezas
>atingidas no campo da medicina e da saúde. Vinte anos
>atrás, tudo isso não passaria de sonho, e não
>existiria fora da cabeça imaginativa dos produtores de
>filmes de ficção científica. Mas, ainda hoje, para a
>esmagadora maioria dos cidadãos brasileiros, todo o
>acima mencionado não passa de um sonho, de uma ficção
>científica

{A] Parece que todo a ciencia é uma ficção cientifica, vamos dizer que o pensamento é um comercio hoje em dia, ficçao ou não, é com certeza sensacionalismo.

>Este ano que acaba nos deu boas notícias,
>encorajadoras, e notícias ruins, no mínimo
>preocupantes. Vimos uma senhora que sofreu um derrame
>cerebral voltar a andar com terapia baseada na
>introdução de células-tronco (células primitivas
>capazes de se transformar em praticamente qualquer
>outra célula normal) no cérebro afetado. Emocionante
>perceber o que esses pequenos passos podem significar
>para milhões de pessoas que não conseguem erguer o
>braço, ou mexer a perna. Parafraseando Neil Armstrong,
>um passo gigante para a humanidade.

{A} Armstrong, quem é ele? Aquele que "supostamente" foi a Lua? Celulas- tronco é um desafio para a humanidade de cura, o desejo de ajudar seu semelhante. Pensamentos inovadores tem estado junto a humanidade a muito tempo, isso pode ser descrito na historia de Tales de Mileto até Einstein




>A técnica das células-tronco não se restringe a
>restaurar a função de células cerebrais. Hoje em dia,
>pesquisadores brasileiros, e em outros países,
>intensificam seus esforços para melhorar o coração
>depois de infarto, os nervos após lesão traumática, a
>pele após queimadura. A lista parece não ter fim.

[A] A demora faz o crescimento dos beneficiados...
>
>Vimos a introdução de remédios geniais na prática
>médica. Geniais na sua concepção, e geniais na sua
>eficiência. Drogas que conseguem agir em um ponto
>específico da célula doente, da célula cancerosa,
>dificultando seu desenvolvimento, seu crescimento, e
>até provocando sua morte. Exemplos que estão
>progressivamente sendo utilizados na prática médica,
>apesar de somente em casos muito selecionados, não
>faltam. Glivec, Iressa, são alguns deles.

[A] Santa ciencia!! Mata e de forma covarde porque usa seus conhecimentos de forma de interesse, mesmo que sabemos, esses conhecimentos são para todos.
>
>Ainda nem acabou o ano e ouvimos há poucas semanas o
>anúncio pela GlaxoSmithKline de uma vacina contra o
>câncer. O Brasil está totalmente empenhado na produção
>da vacina contra o HPV, vírus causador de câncer de
>útero. O prof. dr. Ricardo Brentani, presidente da
>Fundação Antonio Prudente, Hospital do Câncer AC
>Camargo, está entusiasmado e declara:
>
>­ Felizmente nossa instituição foi parceira da Merck
>Sharp & Dohme no desenvolvimento de uma vacina contra
>o HPV. Minha esperança é que em 20 anos tenhamos
>prevenido 7% dos tumores humanos.

[A] Vacina? Como pode ter vacina para nossas proprias celulas? E do HIV vai ter?

>Vimos, por outro lado, a preocupação crescente dos
>cientistas em não excluir, a priori, tratamentos
>considerados até então não-convencionais, ou
>complementares. Estudos sérios avaliaram desde a
>acupuntura até a homeopatia, passando por terapias de
>Florais de Bach e tratamentos com vitaminas nas mais
>diversas situações clínicas. Vantagens e desvantagens
>de cada abordagem foram dissecadas minuciosamente.

[A] Viu? Sera que alguem aqui duvidda?


>Cientistas conseguiram demonstrar a eficiência
>incontestável de algumas terapias alternativas em
>certas situações, como a massagem para um dos males do
>século, a dor nas costas. Por outro lado, alertaram
>para a ineficiência e até os efeitos nocivos de outras
>terapias, em outras condições. Por exemplo, para
>tratar bronquite e asma a acupuntura não parece ter
>efeitos importantes. Pelo menos não conseguiram
>detectar esses efeitos nos estudos atuais.

[A] Remedios sim? Eu tive começo, eu e meus manos, de bronquite depois que meu pai comprou um tartaruga nunca mais...santa crença ne? Mas como explicar??

>Vimos o lançamento de aparelhos ultramodernos,
>ultra-sensíveis, para detectar doenças e tratá-las.
>Não consigo perceber avanço recente maior do que na
>área da radioterapia. É incrível a precisão dos feixes
>de radiação em atingir o alvo, no caso o câncer, e
>poupar o tecido normal adjacente. Aparelhos novos
>conseguem acompanhar o movimento do corpo para seguir
>o alvo predeterminado, segundo a segundo, obedecendo à
>orientação do médico radioterapeuta. Reduziram-se
>muito os efeitos colaterais. O controle do câncer
>assemelha-se às extensas cirurgias. A nanotecnologia
>(aparelhos miniaturas) é uma febre, e os estudos
>multiplicam-se para definir com mais clareza sua
>aplicação.

{A} Pode matar com sua radioatividade, causando cancer.

>
>A plástica está fazendo tamanhos milagres que nem os
>próprios pacientes conseguem acreditar. Tanto faz quem
>foram seus pais, ou a etnia a qual você pertença.
>Escolha o modelo e ficará parecido. Para pessoas
>ansiosas com a imagem, soluções para quase tudo. Sem
>dúvida, o impacto sobre o estado emocional é notável.

[A] Eita faquinha de dois cumes heim? A plastica pode curar a pessoa de queimaduras ou cicatrizes, mas tambem pode ressaltar sua vaidade.

>
>Ao lado dessas notícias que mereceram um destaque
>quase obsessivo nas manchetes de capa, outras
>informações com menos, digamos, glamour, não
>conseguiram espaço nem nas páginas finais das
>revistas.
[A] A ciencia ja tem a imprensa e seus colaborados para dar esse glamour...


>
>O objetivo de reduzir em 60% a mortalidade infantil no
>mundo (e o Brasil ainda é um grande protagonista dessa
>estatística) não será atingido em 2015, como
>estabelecido nos Objetivos Milenares da ONU. No
>planeta, morrem por ano mais de 11 milhões de crianças
>com idade inferior a 5 anos. A maioria por doenças
>evitáveis. Diarréia, pneumonia, malária. No mesmo
>período, 500 mil mulheres morrem durante a gravidez ou
>o parto. Doença de Chagas e esquistossomose
>(barriga-d'água) continuam afetando milhares de
>brasileiros. E seu controle está cada vez mais
>próximo. A implementação dos programas já em ação
>poderá melhorar ainda mais esse controle. Para 2005 a
>intensificação dessas abordagens poderá elevar o
>impacto na saúde da população, principalmente nas
>áreas rurais.

[A] Isso ai, como fica doenças tao mais antiga diante do deusa peerfeita ciencia??


>O Relatório Mundial de Saúde (The World Health Report
>2003) recomendou às autoridades o fortalecimento dos
>sistemas de saúde, centralizando seu foco na atenção
>primária, além de integrar a prevenção das doenças e a
>promoção da saúde em todos os níveis de atendimento.

[A] Esses relatorios tem efeito??

>
>A saúde do homem sofreu mudanças drásticas nos últimos
>anos. A expectativa de vida, de brasileiros e de não
>brasileiros, bate recorde atrás de recorde.
>Ultrapassou a marca dos 70 anos e logo passará dos 80,
>90, e quem sabe 100 anos. Avanços sem dúvida notáveis.
>Doenças contagiosas foram substituídas por doenças
>crônicas, como problemas cardiovasculares e câncer, e
>por causas externas, como trauma. Essa mudança de foco
>exigiu modificações intensivas no nível estrutural.
>Algumas especialidades assumiram posição de destaque
>nunca antes alcançada. Os serviços de cardiologia de
>muitos hospitais logo se transformaram em
>departamentos e, a seguir, em grandes centros com
>prédios próprios. O mesmo ocorreu com a oncologia e
>suas variadas especialidades. Houve uma explosão na
>tecnologia e na sofisticação, e, conseqüentemente, nos
>custos ­ alertam os especialistas em saúde pública.
>Recomenda-se também a melhora da saúde da população
>por intermédio do envolvimento acadêmico.
>

[A] Sem comentarios, pra que vou querer viver até o 100?
>
>Muito por fazer.
>Como melhorar o atendimento ao público do SUS

[A] Infelizmente a ciencia não melhora ela mesma vai melhorar o SUS, a ciencia infelizmente, anda atendendo interesses dos laboratorios e pondo em pratica, o que determina o codigo americano, Bisness in bisness...
>
>
>A academia médica no Brasil foi alterada sensivelmente
>nas últimas duas décadas. Progressivamente, as escolas
>médicas e os programas de pós-graduação introduziram
>disciplinas de pesquisa epidemiológica de problemas de
>saúde adaptados à realidade da população do País. Mais
>e mais livros têm sido publicados por acadêmicos
>brasileiros, com enfoque nacional. Mais e mais centros
>de treinamento oferecem vagas para médicos
>interessados em se aperfeiçoar no atendimento básico à
>saúde da família e à clínica geral. O impacto na
>melhora da atenção à saúde do brasileiro pode demorar
>a ser notado. Muito há de ser feito em 2005 e além.

[A] Infelizmente, tem muito moleque fazendo medicina pro papai e só sai meleca, ou o diagnostico é virose ou é exames a "toa"...


>O plano Fome Zero tenta corrigir um problema básico de
>saúde: a miséria e a conseqüente desnutrição. Ninguém
>precisa de doutorado para compreender a conexão direta
>entre a desnutrição e a ocorrência de doenças
>potencialmente graves. É um plano interessante, mas
>ainda longe de atingir seus objetivos de forma
>significativa. Em 2005, e além, o governo deverá fazer
>os ajustes finos para que o programa consiga alcançar
>seu alvo.
>
>A violência é problema de segurança pública, mas
>também é problema de saúde. Não podemos esquecer que,
>no ano passado, mais de 40 mil brasileiros foram
>assassinados. Se acreditarmos nos dados oficiais do
>Ministério da Saúde, naquele período, morreram mais
>brasileiros por tiro do que por câncer de pulmão. O
>controle da violência evitará milhares de mortes e
>seqüelas dramáticas.
>
>A malária é outro problema sem solução a curto prazo.
>Áreas extensas do território nacional são infestadas
>por mosquitos portadores dessa doença. O esforço das
>autoridades regionais e federais deverá ser
>intensificado nas várias frentes: pesquisa de vacinas,
>profilaxia, controle do mosquito e tratamento de
>pacientes infectados. A mortalidade por essa infecção
>deve ser reduzida a todo custo. Estudos recentes
>aventam que a vacina contra a malária pode estar ao
>alcance da ciência em prazo razoável.
>
>No mundo todo, um bilhão de pessoas são infectadas por
>parasitas, como os vermes, e o Brasil tem participação
>significativa nesses dados alarmantes. Vermes no
>intestino não são apenas feios. Eles causam problemas
>como deficiências nutricionais, e, em alguns casos,
>podem até ser fatais. Para mudar isso, saneamento
>básico é fundamental.
>
>Um estudo recentemente realizado e publicado por
>pesquisadores na Universidade do Ceará demonstrou
>claramente a possibilidade de praticamente eliminar os
>parasitas dos pacientes. Remédios eficazes, como a
>ivemerctina, podem ser a solução. Além de tratar o
>paciente, eliminam uma potencial fonte de contaminação
>para outras pessoas. As autoridades devem criar
>sistemas integrados, contínuos, para atacar esse
>problema de saúde pública em várias frentes, em 2005 e
>para sempre.
>
>Quanto à Aids, nos últimos anos, houve um declínio da
>incidência de novos casos na maioria dos estados
>brasileiros. Estudo publicado recentemente por
>pesquisadores da Fundação Oswaldo Cruz, no Rio de
>Janeiro, confirmou que, apesar de recursos limitados
>do governo e da desigualdade socioeconômica aberrante
>no Brasil, a introdução do acesso universal à terapia
>antiviral contribuiu para uma redução impressionante
>na mortalidade por Aids, e pode ter ajudado a diminuir
>sua incidência.
>
>Nessa linha de pensamento, o presidente Lula lançou,
>este ano, o louvável Plano Nacional de Eliminação da
>Hanseníase (a lepra) em até dois anos. Um passo sem
>dúvida importante para controlar essa doença
>contagiosa e debilitante. Paralelamente, o ministro da
>Saúde, Humberto Costa, anunciou a formação de uma
>comissão para promover políticas nacionais de
>reabilitação às pessoas afetadas pela hanseníase:
>
>­ A idéia é possibilitar benefícios, como a
>reabilitação por cirurgias plásticas, financiando para
>a capacitação de médicos, enfermeiros e
>fisioterapeutas para que realizem esses procedimentos
>nos incapacitados fisicamente.
>
>Todos os anos, 42 mil brasileiros contraem a
>hanseníase. E o País só perde para a Índia no ranking
>mundial de casos da doença.
>
>A obesidade, mal do século XXI, mata. Nos EUA, em
>alguns estados o número de óbitos por obesidade
>ultrapassou o de câncer. No Brasil, cientistas
>identificaram obesidade crescente na população. Estudo
>publicado por pesquisadores da Universidade Federal do
>Rio de Janeiro confirmou o aumento progressivo do peso
>dos adolescentes, tanto na Região Nordeste quanto no
>Sudeste do País, desde 1975. Atualmente, 17% dos
>adolescentes da Região Sudeste enquadram-se na
>definição internacional de obesidade. E seus efeitos
>maléficos logo alcançarão a saúde pública. Orientação
>e conscientização devem ser política contínua e bem
>estruturada. A prevenção e o tratamento do sobrepeso
>precisam ser prioridades das políticas de saúde nos
>próximos anos.
>
>Um problema que acomete principalmente as crianças é a
>poluição atmosférica. Um estudo realizado com 5.193
>crianças de duas cidades do Rio de Janeiro, e
>publicado recentemente na revista Annals of Allergy
>Asthma and Immunology, mostrou claramente a correlação
>entre os níveis de poluição e a incidência de
>problemas respiratórios. Observou-se um aumento de
>mais de 50% na freqüência de crises de asma nos
>moradores de Duque de Caxias (local com alta
>concentração de poluentes), comparados aos moradores
>de Seropédica (local com baixos níveis de poluição).
>Outro estudo, realizado pelos pesquisadores do
>laboratório de poluição atmosférica da Universidade de
>São Paulo, confirmou os efeitos nocivos da poluição do
>ar na mortalidade infantil. Há aumento de 6% no número
>de óbitos neonatais nas regiões mais poluídas. O
>controle de poluentes de qualquer origem deverá ser
>obrigação mundial. Ainda mais com a entrada em vigor
>do Protocolo de Kyoto.
>
>Mas, de forma geral, provavelmente o maior desafio das
>autoridades de saúde é garantir um apoio ao avanço
>científico na área de saúde, atingir a maioria da
>população com os avanços médicos e tecnológicos
>alcançados, e insistir na prevenção e na saúde
>primária, básica. Muito mais do que dinheiro, em 2005
>precisamos de estratégia, filosofia e enfoque novos.
>
>O acesso da população, principalmente das camadas
>menos privilegiadas da sociedade, a remédios em geral
>é muito limitado. Várias são as causas, como preços
>elevados na fonte, acréscimos excessivos na cadeia de
>distribuição e impostos. Se as autoridades, de todas
>as esferas, não atentarem a essas dificuldades
>enfrentadas no dia-a-dia da maioria dos cidadãos,
>muito mais será gasto para tratar complicações graves
>de doenças crônicas, como diabetes e hipertensão
>arterial. O Brasil iniciou uma ação interessante nessa
>direção: a aquisição da fábrica da GlaxoSmithKline do
>Brasil pelo governo federal. O presidente Lula
>destacou que "pela primeira vez um governo compra uma
>fábrica da iniciativa privada", ao mesmo tempo
>criticando "o processo inverso de privatização do
>governo anterior". Lula declarou, na ocasião, que
>estava "recuperando uma fábrica que seria desativada,
>eliminaria empregos e que agora produzirá, em escala
>nacional, os antibióticos mais usados no Brasil".
>
>Nessa mesma linha o ministro da Saúde, Humberto Costa,
>deixou muito clara sua estratégia para 2005:
>
>­ Vamos ampliar o acesso da população a medicamentos
>fundamentais com base em três opções. A primeira é a
>rede de farmácias populares, e até o fim do ano
>pretendemos inaugurar cem unidades em todo o Brasil. A
>segunda é a criação de um programa para a venda
>subsidiada de medicamentos básicos para hipertensão e
>diabetes, através da rede de farmácias privada.
>Pretendemos baixar os preços em até 50%. E a terceira
>é a redução do ICMS de 2,8 mil medicamentos até 2005.
>
>De acordo com uma pesquisa do IBGE, de 2003, a saúde
>aparece em terceiro lugar no orçamento das famílias
>brasileiras, e os medicamentos representam 61% desses
>gastos para as pessoas de baixa renda. Além disso,
>metade das pessoas que precisam de tratamento não pode
>pagar os remédios de que necessitam.
>
>A partir de 2005, 50 milhões de unidades de
>antibióticos serão produzidas para a rede do Sistema
>Único de Saúde (SUS) e para as Farmácias Populares. Em
>2007, a produção deve quintuplicar em relação à atual.
>Mais de 10 bilhões de unidades de medicamentos devem
>ser produzidas para as principais doenças que mais
>atingem a população brasileira, como hipertensão,
>diabetes, malária e tuberculose. A Fiocruz também
>produzirá mais vacinas, e o Ministério da Saúde
>enfatiza que "a ampliação do acesso da população aos
>medicamentos é uma das prioridades do governo
>federal".
>
>Em 2004, muito se fez pela saúde, mas ainda há muito
>mais por fazer. Programas de prevenção e detecção
>precoce de doenças sexualmente transmissíveis, como
>Aids, HPV (câncer de colo de útero), a melhoria na
>qualidade do atendimento dos pacientes do SUS,
>diminuição das filas, controle de poluição, de
>infecção hospitalar e, também, de mortes por agentes
>externos, como traumas (tiros, facadas, acidentes de
>trânsito). Para se ter uma idéia do custo com os
>acidentes de trânsito, a cada ano acontecem mais de
>100 mil mortes, com três a quatro vezes esse número de
>feridos. E a maioria absoluta é tratada com dinheiro
>público. São milhões de reais de gastos evitáveis. Sem
>falar nas seqüelas individuais.
>

Abraços
Amauri

[As partes desta mensagem que não continham texto foram removidas]



##### ##### #####

Para saber mais visite
http://www.ciencialist.hpg.ig.com.br


##### ##### ##### #####



Yahoo! Grupos, um serviço oferecido por:
PUBLICIDADE




------------------------------------------------------------------------------
Links do Yahoo! Grupos

a.. Para visitar o site do seu grupo na web, acesse:
http://br.groups.yahoo.com/group/ciencialist/

b.. Para sair deste grupo, envie um e-mail para:
ciencialist-unsubscribe@yahoogrupos.com.br

c.. O uso que você faz do Yahoo! Grupos está sujeito aos Termos do Serviço do Yahoo!.



[As partes desta mensagem que não continham texto foram removidas]



SUBJECT: Evolucao e desafios - Consumo de Notícias
FROM: "L.E.R.de Carvalho" <lecarvalho@infolink.com.br>
TO: ciencialist@yahoogrupos.com.br
DATE: 02/01/2005 15:52

At 15:18 2/1/2005, you wrote:
>Minhas analises em baixo com [A]....



Amauri:

revistas são objetos de consumo.

consumidores de notícias são iguais consumidores de pão e salsicha.
Uns querem uma boa salsicha e um pão bom.
Os outros também querem coisa boa. Ou seja: notícia bom

O cara que compra notícia quer notícia boa.

Como a revista tem que vender... ela bota na capa as notícias boas.
Noticia ruim vem de gratis, principalmente por internet, não precisa
comprar não.

L.E.


[As partes desta mensagem que não continham texto foram removidas]



SUBJECT: Re: [ciencialist] Evolucao e desafios - Consumo de Notícias
FROM: "Amauri Jr" <amaurijunior2@yahoo.com.br>
TO: <ciencialist@yahoogrupos.com.br>
DATE: 02/01/2005 16:11

L.E.R

"revistas são objetos de consumo."

[A] Não só a revistas como a própria ciência...

"consumidores de notícias são iguais consumidores de pão e salsicha.Uns querem uma boa salsicha e um pão bom.Os outros também querem coisa boa. Ou seja: notícia bom"

[A] Noticia boa era a desvinculação da ciência ao mercado de cosméticos e remédios que duvido serem ineficazes a algumas doenças como o HIV, que sabemos a causa de não haver uma cura. Sustenta um mercado imenso de AZT e outras porcarias que só fazem aumentar o sofrimento, ou o câncer que sustenta o mercado de antibióticos e radioterapias. Criam toda hora desculpas para nós comemos isso ou aquilo, hora faz mal, hora faz bem, como entender? Teorias absurdas e sem utilidade para humanidade para se sustentar, segurar verba, como procurar pelos em baratas, dentes em pernilongos e por ai vai!! Como a ida a Lua...a ciência é refém do capitalismo.

"O cara que compra notícia quer notícia boa."

[A] Lógico, se você compra porcaria ta jogando dinheiro fora.

"Como a revista tem que vender... ela bota na capa as notícias boas.Noticia ruim vem de grátis, principalmente por internet, não precisa comprar não."

[A] Noticias ruins vem de todo lugar, mesmo as revistas ditas boas vem, a internet só veio a acrescentar esse momento Bikmann

Amauri

----- Original Message -----
From: L.E.R.de Carvalho
To: ciencialist@yahoogrupos.com.br
Sent: Sunday, January 02, 2005 3:52 PM
Subject: [ciencialist] Evolucao e desafios - Consumo de Notícias


At 15:18 2/1/2005, you wrote:
>Minhas analises em baixo com [A]....



Amauri:

revistas são objetos de consumo.

consumidores de notícias são iguais consumidores de pão e salsicha.
Uns querem uma boa salsicha e um pão bom.
Os outros também querem coisa boa. Ou seja: notícia bom

O cara que compra notícia quer notícia boa.

Como a revista tem que vender... ela bota na capa as notícias boas.
Noticia ruim vem de gratis, principalmente por internet, não precisa
comprar não.

L.E.


[As partes desta mensagem que não continham texto foram removidas]



##### ##### #####

Para saber mais visite
http://www.ciencialist.hpg.ig.com.br


##### ##### ##### #####


Yahoo! Grupos, um serviço oferecido por:







------------------------------------------------------------------------------
Links do Yahoo! Grupos

a.. Para visitar o site do seu grupo na web, acesse:
http://br.groups.yahoo.com/group/ciencialist/

b.. Para sair deste grupo, envie um e-mail para:
ciencialist-unsubscribe@yahoogrupos.com.br

c.. O uso que você faz do Yahoo! Grupos está sujeito aos Termos do Serviço do Yahoo!.



[As partes desta mensagem que não continham texto foram removidas]



SUBJECT: TV discute ‘Faço o que meu gene manda?’
FROM: "L.E.R.de Carvalho" <lecarvalho@infolink.com.br>
TO: ciencialist@yahoogrupos.com.br
DATE: 02/01/2005 16:13


JC e-mail 2678, de 30 de Dezembro de 2004.

‘Tome Ciência’ na TV discute ‘Faço o que meu gene manda?’ neste fim de semana

O programa está no ar deste segunda-feira passada e será apresentado até 2
de dezembro, domingo

O programa sobre o tema ‘Faço o que meu gene manda?’ aborda assuntos de
grandes atualidade, embora com antecedentes históricos importantes.

A história documentou a necessidade de colonizadores e conquistadores de
justificar, com base na ciência, a escravidão, o genocídio e a
estratificação da sociedade.

Hoje, pesquisadores e leigos perguntam se de fato as grandes mazelas da
sociedade têm origem na genética. O comportamento violento, as tendências
criminosas, o homossexualismo, a inteligência, o abuso de drogas, a
esquizofrenia são herdados?

Ou a educação e a cultura desempenham papéis predominantes? Essa é uma
discussão polêmica e apaixonada, debatida por defensores das duas
principais correntes do pensamento.

Participantes deste debate do ‘Tome Ciência’:

Jorge Moll Neto, neurologista, coordenador da Unidade de Neurociência
Cognitiva e Comportamental da Rede D'Or de Hospitais, co-autor, com o
neurologista Ricardo Oliveira, de um mapeamento das emoções no cérebro;

Suzana Herculano Houzel, neurocientista do Depto. de Anatomia da UFRJ,
autora de livros sobre o funcionamento do cérebro e nossas emoções e
desejos. Sérgio Snith, médico e psicólogo do Depto. de Neurofisiologia da Uerj;
Ricardo Waizbort, doutor em literatura, que trabalha com filosofia da
biologia na Casa de Oswaldo Cruz, da Fiocruz.

Na segunda-feira, dia 3 de janeiro, muda a programação do ‘Tome Ciência’.

O programa ‘Transgênicos: a gente come, lucra e não conhece’ fica no ar de
3 a 11 de janeiro.

Muita gente já consome alimentos transgênicos sem ao menos saber o que são
esses organismos geneticamente modificados. Há quem se preocupe com
conseqüências para a saúde e meio ambiente.

Mas a Embrapa já conseguiu avanços que permitiram significativos
crescimentos na exportação agrícola brasileira.

Pesquisadores e cientistas debatem o problema, que já foi resolvido em
muitos países do primeiro mundo. A desconfiança do público e dos políticos
se baseia na ignorância?

Quem deve bater o martelo sobre o consumo dos transgênicos? Os cientistas?
Os políticos? Saiba como anda a lei de biossegurança que trata do assunto.

Participam deste ‘Tome Ciência’:

Silvio Valle, coordenador do curso de biossegurança e pesquisador da Escola
de Saúde Pública, da Fundação Oswaldo Cruz, a Fiocruz;

Marília Nutti, engenheira de alimentos, pesquisadora da Embrapa -
Agroindústria de Alimentos;

Norma Rumjanek, doutora em Genética de Microorganismos, pesquisadora em
Agrobiologia na Embrapa; e

Heloisa Helena Barboza, professora titular de Direito Civil da Uerj e
professora do curso de mestrado da Universidade.


[As partes desta mensagem que não continham texto foram removidas]



SUBJECT: Os Terremotos
FROM: "Amauri Jr" <amaurijunior2@yahoo.com.br>
TO: <ciencialist@yahoogrupos.com.br>, "Lucimary Vargas" <sangalli@uai.com.br>, <transconhecimento@yahoogrupos.com.br>, "Telma" <amletsemogrjadv@hotmail.com>, <Sexualidadedodef@yahoogrupos.com.br>, <elio_mollo_k@hotmail.com>, "Ce" <celinavictor94@hotmail.com>, Flávia Mendes Sanches <fla1202@yahoo.com.br>, "Reginaldo" <reginaldo.pereira.li@itelefonica.com.br>, Lênia Luz Nogueira <lenialuz@yahoo.com.br>, "Laracna" <laracna@pop.com.br>, "kathleen_mel" <kathleen_mel@uol.com.br>, <GenealBR@yahoogrupos.com.br>, <Euthymia@yahoogrupos.com.br>, <Conversa_de_Botequim@yahoogrupos.com.br>
DATE: 02/01/2005 19:38

O Jornal El mundo prupucou um estudo como ocorre os terremotos, explica��es em flash

http://www.elmundo.es/elmundo/2003/graficos/jun/s2/terremotos.html

[As partes desta mensagem que n�o continham texto foram removidas]



SUBJECT: Fw: calculo
FROM: "Luiz Ferraz Netto" <leobarretos@uol.com.br>
TO: "ciencialist" <ciencialist@yahoogrupos.com.br>
DATE: 02/01/2005 19:54

Vamos tentar entrar no 'âmago' da questão e ajudar o nanoempresário?
agradeço sugestões.
[]'
===========================
Luiz Ferraz Netto [Léo]
leobarretos@uol.com.br
http://www.feiradeciencias.com.br
===========================
-----Mensagem Original-----
De: <nandofff@ligbr.com.br>
Para: <leobarretos@uol.com.br>
Enviada em: quarta-feira, 29 de dezembro de 2004 16:04
Assunto: calculo


Olá meu é Fernando Fortunato Ferreira, moro em salvadora Bahia e estou
precisando de uma ajuda em cálculos. Deixa-me explicar o quero na verdade ou
se o Senhor Podê me ajudar :
Tenho uma fabrica de fundo de quintalde laje pré moldad. E sempre estou
usando a calculadora para fazer uns cálculos para achar a quantidade de
longarinas e blocos que pega em uma certa aréa sempre sobra longarinas, mas
aí tudo bem da para contornar. o ruim é quando a casa começar com uma
largura e termina com outra.... ex: na fente tem 8 mt e no fundo 3mt como
faço para saber a quanto e quantos centimetros troco o tamanho da longarina
se a casa fosse certa eu faria ex: 8 de largura e 5 de comprimento para
achar a quantidade de longarinas no tamanho de 8 mt faço 8 x 22% que da
aproximadamente 17 longarinas de 5 metros deu para entender prof.


preciso muito de sua ajuda sepoder agradeço



SUBJECT: Fw: Pergunta.
FROM: "Luiz Ferraz Netto" <leobarretos@uol.com.br>
TO: "ciencialist" <ciencialist@yahoogrupos.com.br>
DATE: 02/01/2005 19:57

Esse tb precisa de uma resposta acurada para entender que superar a CPFL não é simples (nem barato!).
[]'
===========================
Luiz Ferraz Netto [Léo]
leobarretos@uol.com.br
http://www.feiradeciencias.com.br
===========================
-----Mensagem Original-----
De: LED PAPELARIA
Para: leobarretos@uol.com.br
Enviada em: quinta-feira, 30 de dezembro de 2004 11:00
Assunto: Pergunta.


Professor, Bom dia, eu estou a procura de um projeto de sobre Energia Heolica, para eu ligar na minha casa, pois a conta de luz está muito alta, se o senhor tever eu ficaria muito contente, eu moro na cidade de Registro/SP (Vale do Ribeira) e tem muito vento aqui na região, atenciosamente, Elias Guedes e um Feliz Ano Novo.

[As partes desta mensagem que não continham texto foram removidas]



SUBJECT: Fw: evapora��o de agua...
FROM: "Luiz Ferraz Netto" <leobarretos@uol.com.br>
TO: "ciencialist" <ciencialist@yahoogrupos.com.br>
DATE: 02/01/2005 19:58

Vamos gente, ano novo, dúvidas velhas!
[]'
===========================
Luiz Ferraz Netto [Léo]
leobarretos@uol.com.br
http://www.feiradeciencias.com.br
===========================
-----Mensagem Original-----
De: Clinica
Para: leobarretos@uol.com.br ; daivemmeue-mailleobarretos@uol.com.br
Enviada em: quinta-feira, 30 de dezembro de 2004 12:12
Assunto: evaporação de agua...


Nas dúvidas experimentais, por gentileza coloque aqui o endereço da página, isso facilita o confronto. Agradeço. Meu nome é LUIZ FERRAZ NETTO, meu apelido é LÉO e moro em BARRETOS


Professor


Tenho uma loja de piscinas, e gostaria de saber como faço para calcular a evaporação de agua de uma piscina.....

Moro em Cascavel-Pr.

agradeceria se pudesse me ajudar...

Paulo Cesar Justino
(45)223-4222





[As partes desta mensagem que não continham texto foram removidas]



SUBJECT: Fw: Teoria da Relatividade Especial
FROM: "Luiz Ferraz Netto" <leobarretos@uol.com.br>
TO: "ciencialist" <ciencialist@yahoogrupos.com.br>
DATE: 02/01/2005 20:10

Vamos responder? ... educadamente ...
[]'
===========================
Luiz Ferraz Netto [Léo]
leobarretos@uol.com.br
http://www.feiradeciencias.com.br
===========================
-----Mensagem Original-----
De: Jaime Pacheco Filho
Para: leobarretos@uol.com.br
Enviada em: sexta-feira, 31 de dezembro de 2004 01:52
Assunto: Teoria da Relatividade Especial


Nas dúvidas experimentais, por gentileza coloque aqui o endereço da página, isso facilita o confronto. Agradeço. Meu nome é LUIZ FERRAZ NETTO, meu apelido é LÉO e moro em BARRETOS; dai vem meu e-mail: leobarretos@uol.com.br.

Jaime Pacheco Filho

jaime-pacheco@uol.com.br

Escolaridade: superior (Economia)



Dúvida: Entendo que o Efeito Doppler é uma conseqüência da independência da velocidade de uma onda (sonora ou luminosa) em relação ao movimento do emissor e do receptor no meio de propagação. Então, por que a velocidade da luz (da qual não se conhece o meio de propagação) é especial e não a do som (do qual se sabe o meio de propagação)?

Será que os efeitos previstos pela Teoria da Relatividade Especial como a alteração no ritmo de relógios e o encurtamento de réguas não são apenas aparentes, devidos ao Efeito Doppler?

Se fôssemos todos cegos talvez a Teoria da Relatividade se baseasse na velocidade do som.

Einstein explicou porque os relógios atrasam e as réguas encurtam quando são acelerados?



Obrigado






[As partes desta mensagem que não continham texto foram removidas]



SUBJECT: Fw: maremoto
FROM: "Luiz Ferraz Netto" <leobarretos@uol.com.br>
TO: "ciencialist" <ciencialist@yahoogrupos.com.br>
DATE: 02/01/2005 20:11

que acham?
[]'
===========================
Luiz Ferraz Netto [Léo]
leobarretos@uol.com.br
http://www.feiradeciencias.com.br
===========================
-----Mensagem Original-----
De: armandopentium
Para: leobarretos@uol.com.br
Enviada em: sexta-feira, 31 de dezembro de 2004 10:31
Assunto: maremoto


Olá! bom dia professor
gostaria de saber o que vai acontecer agora com essa auteração de 2cm do eixo da terra devido ao maremoto , será que vai mudar o clima da terra?

armandopentium@bol.com.br

[As partes desta mensagem que não continham texto foram removidas]



SUBJECT: Fw: Obra Prima
FROM: "Luiz Ferraz Netto" <leobarretos@uol.com.br>
TO: "ciencialist" <ciencialist@yahoogrupos.com.br>
DATE: 02/01/2005 20:13

Boa passagem de ano é assim!
[]'
===========================
Luiz Ferraz Netto [Léo]
leobarretos@uol.com.br
http://www.feiradeciencias.com.br
===========================
-----Mensagem Original-----
De: Geraldo Luiz Cartolano
Para: leobarretos@uol.com.br
Enviada em: sexta-feira, 31 de dezembro de 2004 16:44
Assunto: Obra Prima


Caro Léo,

Hoje é o último dia do ano e enquanto a hora da comemoração não chega, quero aqui deixar meus sinceros elogios a sua obra, a muito tempo procuro algo assim na Internet e não encontrava. Fiquei estupefato com a quantidade de experimentos e a qualidade do material, meus mais calorosos parabéns. Esta página será a mais visitada por mim a partir de hoje.

Feliz Ano Novo

Geraldo Luiz Cartolano
Rio de Janeiro
Analista de Suporte - Xerox do Brasil
Entusiasta da Física


[As partes desta mensagem que não continham texto foram removidas]



SUBJECT: Re: [ciencialist] Enviando email: Navier-Stokes-quem se habilita?
FROM: "JVictor" <jvoneto@uol.com.br>
TO: <ciencialist@yahoogrupos.com.br>
DATE: 02/01/2005 21:48

Rodrigo,

Não. Não tenho a pretensão de entrar para a imortalidade. Coisas assim só acontecem com os outros. É como um acidente! Tudo que coloquei no e-mail é mais como ênfase, para exaltar a importância daquela ferramenta formidável, pois o assunto é deveras emocionante. Nem tenho a pretensão de, dentro de 100 anos(no fim de tempo, voltarei ao assunto, verá...), resolver os desafios navier-stokianos que ora se apresentam. Agora, quem não gostaria? Tentar, ao menos, todos podemos. No mínimo, o conhecimento que resulta da ousadia é de uma riqueza!
Nos cursos de graduação, em geral, tudo o que vemos sobre dinâmica dos fluidos chega até a Equação de Bernoulli, e suas aplicações. Pelo menos comigo foi assim, que fiz engenharia. Não sei nos cursos de Física. A N-Stokes é deduzida a partir da continuidade da massa e considerações energéticas. Mas esta famosa equação(Bernoulli), bem como todas as outras, são meros casos particulares da não tão famosa quanto dramática Equação de Navier-Stokes. Esta não só dá conta de todas as equações que estudamos, como esclarece os comportamentos dinâmicos dos fluidos, como vorticidade e outras mazelas. Não explica, mas descreve satisfatoriamente. Você já viu um anel de fumaça se destacando após uma baforada? Experimente explicar a física envolvida naquele fenômeno! A PETROBRÁS tem inúmeros e caros programas de pesquisas a respeito do que a Navier-Stokes pode nos dizer, a respeito do comportamento de líquidos, consideradas as viscosidades e tudo o mais, em movimento. A Navier-Stokes tem um termo que resulta proporcional a v ao quadrado, tornando-a irremediavelmente não-linear. É aí que o bicho pega. Soluções analíticas, que é bom e todos gostam, só para alguns simples casos particulares. Então, uma dica é descobrir com escantear o termo (v.delta)v, v=velocidade do fluido, vetor; delta=operador gradiente; e o ponto representa o produto escalar. Agora, tudo isso sem alterar a estrutura do monstrengo(no sentido de dificuldade ou impossiblidade de sempre se obter soluções analíticas). Pois é.
Mas o assunto é palpitante e seu estudo e compreensão até que não são tão complicado assim. É só compreender o significado físicco de cada identidade vetorial que vai surgindo. Pronto.

Victor

----- Original Message -----
From: Rodrigo Toledo
To: ciencialist@yahoogrupos.com.br
Sent: Sunday, January 02, 2005 12:06 PM
Subject: Re: [ciencialist] Enviando email: Navier-Stokes-quem se habilita?


Vitor,

Caso queira tentar outras oportunidades de entrar para a imortalidade ( Com
muito dinheiro !! ), existem outros desafios igualmente, digamos,
complicados.

[]'s

RT

http://www.claymath.org/millennium/
----- Original Message -----
From: "JVictor" <jvoneto@uol.com.br>
To: <ciencialist@yahoogrupos.com.br>
Sent: Friday, December 31, 2004 3:54 PM
Subject: [ciencialist] Enviando email: Navier-Stokes-quem se habilita?



Caros ciencialisteiros,

Eis algo em razão do que estudar. Prêmio: um milhãozinho de verdinhas, fora
o ingresso para a imortalidade. Já gastei 1/2 caderno de e 4 preciosos
cabelinhos! E nada!
Mas, estudar e pesquisar essa fantástica equação, descobrinho o que ela
esconde já é um bom lazer.
Quem sabe não teremos um 2,76milionário(em nossa fraca moeda) aquí no
ciencialist. Eis o endereço:

http://www.prandiano.com.br/html/m_nov.htm

Observação: para se proteger de vírus de computador, os programas de email
podem impedir o envio ou recebimento de alguns tipos de anexo de arquivo.
Verifique as configurações de email para determinar como os anexos são
manipulados.
Feliz Ano Novo para todos e
Boa sorte!

JVictor.

[As partes desta mensagem que não continham texto foram removidas]



##### ##### #####

Para saber mais visite
http://www.ciencialist.hpg.ig.com.br


##### ##### ##### #####
Links do Yahoo! Grupos










##### ##### #####

Para saber mais visite
http://www.ciencialist.hpg.ig.com.br


##### ##### ##### #####


Yahoo! Grupos, um serviço oferecido por:







------------------------------------------------------------------------------
Links do Yahoo! Grupos

a.. Para visitar o site do seu grupo na web, acesse:
http://br.groups.yahoo.com/group/ciencialist/

b.. Para sair deste grupo, envie um e-mail para:
ciencialist-unsubscribe@yahoogrupos.com.br

c.. O uso que você faz do Yahoo! Grupos está sujeito aos Termos do Serviço do Yahoo!.



[As partes desta mensagem que não continham texto foram removidas]



SUBJECT: Fw: [ciencialist] Enviando email: Navier-Stokes-quem se habilita?
FROM: "JVictor" <jvoneto@uol.com.br>
TO: <ciencialist@yahoogrupos.com.br>
DATE: 02/01/2005 22:48

Errata.

Onde se lê: A N-Stokes é deduzida... Leia-se: a Eq. de Bernoulli é deduzida...

Bem,aproveito para acrescentar também que a Navier-Stokes é uma aplicação direta da segunda lei de Newton, sem tirar nem por. É simples, o procedimento: calcular a variação do momento linear de todas as partículas em uma linda de fluxo do fluído e igualar isto à segunda de Newton: F=(dP/dt), P e F vetores. Daí surge uma equação diferencial envolvendo coisas como velocidade, viscosidade, forças potenciais(inclusive a gravidez) e não potenciais, pressão, e relações vetoriais entre elas. Entre as quais, a mais chata de todas, que aleija o produto final, como disse no e-mail anterior. Curioso é que a derivação é até simples e mais ou menos direta, envolvendo simples integrais de superfícies e de volumes, bem como os indispensáveis teoremas de Gauss e Stokes, relacionados. Mas o resultado é um osso duro de roer, para a maioria dos mortais(graaaande maioria) e dos problemas reais, daqueles que matam, como furacões, maremotos, redemoinhos, vorticidades, em geral.
Um novo enfoque, tipo einsteiniano, certamente poderá reverter isso. Quem se habilita?

Sds,

Victor.


----- Original Message -----
From: JVictor
To: ciencialist@yahoogrupos.com.br
Sent: Sunday, January 02, 2005 9:48 PM
Subject: Re: [ciencialist] Enviando email: Navier-Stokes-quem se habilita?


Rodrigo,

Não. Não tenho a pretensão de entrar para a imortalidade. Coisas assim só acontecem com os outros. É como um acidente! Tudo que coloquei no e-mail é mais como ênfase, para exaltar a importância daquela ferramenta formidável, pois o assunto é deveras emocionante. Nem tenho a pretensão de, dentro de 100 anos(no fim de tempo, voltarei ao assunto, verá...), resolver os desafios navier-stokianos que ora se apresentam. Agora, quem não gostaria? Tentar, ao menos, todos podemos. No mínimo, o conhecimento que resulta da ousadia é de uma riqueza!
Nos cursos de graduação, em geral, tudo o que vemos sobre dinâmica dos fluidos chega até a Equação de Bernoulli, e suas aplicações. Pelo menos comigo foi assim, que fiz engenharia. Não sei nos cursos de Física. A N-Stokes é deduzida a partir da continuidade da massa e considerações energéticas. Mas esta famosa equação(Bernoulli), bem como todas as outras, são meros casos particulares da não tão famosa quanto dramática Equação de Navier-Stokes. Esta não só dá conta de todas as equações que estudamos, como esclarece os comportamentos dinâmicos dos fluidos, como vorticidade e outras mazelas. Não explica, mas descreve satisfatoriamente. Você já viu um anel de fumaça se destacando após uma baforada? Experimente explicar a física envolvida naquele fenômeno! A PETROBRÁS tem inúmeros e caros programas de pesquisas a respeito do que a Navier-Stokes pode nos dizer, a respeito do comportamento de líquidos, consideradas as viscosidades e tudo o mais, em movimento. A Navier-Stokes tem um termo que resulta proporcional a v ao quadrado, tornando-a irremediavelmente não-linear. É aí que o bicho pega. Soluções analíticas, que é bom e todos gostam, só para alguns simples casos particulares. Então, uma dica é descobrir com escantear o termo (v.delta)v, v=velocidade do fluido, vetor; delta=operador gradiente; e o ponto representa o produto escalar. Agora, tudo isso sem alterar a estrutura do monstrengo(no sentido de dificuldade ou impossiblidade de sempre se obter soluções analíticas). Pois é.
Mas o assunto é palpitante e seu estudo e compreensão até que não são tão complicado assim. É só compreender o significado físicco de cada identidade vetorial que vai surgindo. Pronto.

Victor

----- Original Message -----
From: Rodrigo Toledo
To: ciencialist@yahoogrupos.com.br
Sent: Sunday, January 02, 2005 12:06 PM
Subject: Re: [ciencialist] Enviando email: Navier-Stokes-quem se habilita?


Vitor,

Caso queira tentar outras oportunidades de entrar para a imortalidade ( Com
muito dinheiro !! ), existem outros desafios igualmente, digamos,
complicados.

[]'s

RT

http://www.claymath.org/millennium/
----- Original Message -----
From: "JVictor" <jvoneto@uol.com.br>
To: <ciencialist@yahoogrupos.com.br>
Sent: Friday, December 31, 2004 3:54 PM
Subject: [ciencialist] Enviando email: Navier-Stokes-quem se habilita?



Caros ciencialisteiros,

Eis algo em razão do que estudar. Prêmio: um milhãozinho de verdinhas, fora
o ingresso para a imortalidade. Já gastei 1/2 caderno de e 4 preciosos
cabelinhos! E nada!
Mas, estudar e pesquisar essa fantástica equação, descobrinho o que ela
esconde já é um bom lazer.
Quem sabe não teremos um 2,76milionário(em nossa fraca moeda) aquí no
ciencialist. Eis o endereço:

http://www.prandiano.com.br/html/m_nov.htm

Observação: para se proteger de vírus de computador, os programas de email
podem impedir o envio ou recebimento de alguns tipos de anexo de arquivo.
Verifique as configurações de email para determinar como os anexos são
manipulados.
Feliz Ano Novo para todos e
Boa sorte!

JVictor.

[As partes desta mensagem que não continham texto foram removidas]



##### ##### #####

Para saber mais visite
http://www.ciencialist.hpg.ig.com.br


##### ##### ##### #####
Links do Yahoo! Grupos










##### ##### #####

Para saber mais visite
http://www.ciencialist.hpg.ig.com.br


##### ##### ##### #####


Yahoo! Grupos, um serviço oferecido por:







------------------------------------------------------------------------------
Links do Yahoo! Grupos

a.. Para visitar o site do seu grupo na web, acesse:
http://br.groups.yahoo.com/group/ciencialist/

b.. Para sair deste grupo, envie um e-mail para:
ciencialist-unsubscribe@yahoogrupos.com.br

c.. O uso que você faz do Yahoo! Grupos está sujeito aos Termos do Serviço do Yahoo!.



[As partes desta mensagem que não continham texto foram removidas]



##### ##### #####

Para saber mais visite
http://www.ciencialist.hpg.ig.com.br


##### ##### ##### #####


Yahoo! Grupos, um serviço oferecido por:







--------------------------------------------------------------------------------
Links do Yahoo! Grupos

a.. Para visitar o site do seu grupo na web, acesse:
http://br.groups.yahoo.com/group/ciencialist/

b.. Para sair deste grupo, envie um e-mail para:
ciencialist-unsubscribe@yahoogrupos.com.br

c.. O uso que você faz do Yahoo! Grupos está sujeito aos Termos do Serviço do Yahoo!.



[As partes desta mensagem que não continham texto foram removidas]



SUBJECT: Re: [ciencialist] Enviando email: Navier-Stokes-quem se habilita?
FROM: "Rodrigo Toledo" <rodrigotoledo11@uol.com.br>
TO: <ciencialist@yahoogrupos.com.br>
DATE: 02/01/2005 22:50

Caro Vitor,

É verdade: No meu curso de eng.química chegamos até Bernoulli, um pedaçinho
da N-S como vc disse.

Também acho interessante o assunto e, para ser sincero, já gastei alguns
minutos pensando se deveria passar outros minutos ( Q pretensão a minha !!)
tentando resolver....risos...

Achei um site interessante sobre o assunto:

http://www.navier-stokes.net/

Abraço,

RT


----- Original Message -----
From: "JVictor" <jvoneto@uol.com.br>
To: <ciencialist@yahoogrupos.com.br>
Sent: Sunday, January 02, 2005 9:48 PM
Subject: Re: [ciencialist] Enviando email: Navier-Stokes-quem se habilita?



Rodrigo,

Não. Não tenho a pretensão de entrar para a imortalidade. Coisas assim só
acontecem com os outros. É como um acidente! Tudo que coloquei no e-mail é
mais como ênfase, para exaltar a importância daquela ferramenta formidável,
pois o assunto é deveras emocionante. Nem tenho a pretensão de, dentro de
100 anos(no fim de tempo, voltarei ao assunto, verá...), resolver os
desafios navier-stokianos que ora se apresentam. Agora, quem não gostaria?
Tentar, ao menos, todos podemos. No mínimo, o conhecimento que resulta da
ousadia é de uma riqueza!
Nos cursos de graduação, em geral, tudo o que vemos sobre dinâmica dos
fluidos chega até a Equação de Bernoulli, e suas aplicações. Pelo menos
comigo foi assim, que fiz engenharia. Não sei nos cursos de Física. A
N-Stokes é deduzida a partir da continuidade da massa e considerações
energéticas. Mas esta famosa equação(Bernoulli), bem como todas as outras,
são meros casos particulares da não tão famosa quanto dramática Equação de
Navier-Stokes. Esta não só dá conta de todas as equações que estudamos, como
esclarece os comportamentos dinâmicos dos fluidos, como vorticidade e outras
mazelas. Não explica, mas descreve satisfatoriamente. Você já viu um anel de
fumaça se destacando após uma baforada? Experimente explicar a física
envolvida naquele fenômeno! A PETROBRÁS tem inúmeros e caros programas de
pesquisas a respeito do que a Navier-Stokes pode nos dizer, a respeito do
comportamento de líquidos, consideradas as viscosidades e tudo o mais, em
movimento. A Navier-Stokes tem um termo que resulta proporcional a v ao
quadrado, tornando-a irremediavelmente não-linear. É aí que o bicho pega.
Soluções analíticas, que é bom e todos gostam, só para alguns simples casos
particulares. Então, uma dica é descobrir com escantear o termo (v.delta)v,
v=velocidade do fluido, vetor; delta=operador gradiente; e o ponto
representa o produto escalar. Agora, tudo isso sem alterar a estrutura do
monstrengo(no sentido de dificuldade ou impossiblidade de sempre se obter
soluções analíticas). Pois é.
Mas o assunto é palpitante e seu estudo e compreensão até que não são tão
complicado assim. É só compreender o significado físicco de cada identidade
vetorial que vai surgindo. Pronto.

Victor

----- Original Message -----
From: Rodrigo Toledo
To: ciencialist@yahoogrupos.com.br
Sent: Sunday, January 02, 2005 12:06 PM
Subject: Re: [ciencialist] Enviando email: Navier-Stokes-quem se habilita?


Vitor,

Caso queira tentar outras oportunidades de entrar para a imortalidade (
Com
muito dinheiro !! ), existem outros desafios igualmente, digamos,
complicados.

[]'s

RT

http://www.claymath.org/millennium/
----- Original Message -----
From: "JVictor" <jvoneto@uol.com.br>
To: <ciencialist@yahoogrupos.com.br>
Sent: Friday, December 31, 2004 3:54 PM
Subject: [ciencialist] Enviando email: Navier-Stokes-quem se habilita?



Caros ciencialisteiros,

Eis algo em razão do que estudar. Prêmio: um milhãozinho de verdinhas,
fora
o ingresso para a imortalidade. Já gastei 1/2 caderno de e 4 preciosos
cabelinhos! E nada!
Mas, estudar e pesquisar essa fantástica equação, descobrinho o que ela
esconde já é um bom lazer.
Quem sabe não teremos um 2,76milionário(em nossa fraca moeda) aquí no
ciencialist. Eis o endereço:

http://www.prandiano.com.br/html/m_nov.htm

Observação: para se proteger de vírus de computador, os programas de email
podem impedir o envio ou recebimento de alguns tipos de anexo de arquivo.
Verifique as configurações de email para determinar como os anexos são
manipulados.
Feliz Ano Novo para todos e
Boa sorte!

JVictor.

[As partes desta mensagem que não continham texto foram removidas]



##### ##### #####

Para saber mais visite
http://www.ciencialist.hpg.ig.com.br


##### ##### ##### #####
Links do Yahoo! Grupos










##### ##### #####

Para saber mais visite
http://www.ciencialist.hpg.ig.com.br


##### ##### ##### #####


Yahoo! Grupos, um serviço oferecido por:







----------------------------------------------------------------------------
--
Links do Yahoo! Grupos

a.. Para visitar o site do seu grupo na web, acesse:
http://br.groups.yahoo.com/group/ciencialist/

b.. Para sair deste grupo, envie um e-mail para:
ciencialist-unsubscribe@yahoogrupos.com.br

c.. O uso que você faz do Yahoo! Grupos está sujeito aos Termos do
Serviço do Yahoo!.



[As partes desta mensagem que não continham texto foram removidas]



##### ##### #####

Para saber mais visite
http://www.ciencialist.hpg.ig.com.br


##### ##### ##### #####
Links do Yahoo! Grupos










SUBJECT: Re: [ciencialist] Enviando email: Navier-Stokes-quem se habilita?
FROM: "JVictor" <jvoneto@uol.com.br>
TO: <ciencialist@yahoogrupos.com.br>
DATE: 02/01/2005 23:01

Rodrigo,

Acessei o site que você indicou. Fiquei satisfeito, pois:

1 - sentí que o autor viu a beleza e importância da Equação, como eu ví;
2- ele, por conta disso, faz o que eu gostaria de fazer, se tivesse disciplina, paciência e tempo: montar um site com o objetivo único de fazer as pessoas conhecerem e eventualmente desenvolverem o assunto;
3- bem, ao menos vou acompanhar o trabalho do divulgador. Espero compreender um pouco mais, a cada vez. Quando eu aprender muito, eu conto o causo como o causo foi!

Bom achado.

Sds,

Victor.
----- Original Message -----
From: Rodrigo Toledo
To: ciencialist@yahoogrupos.com.br
Sent: Sunday, January 02, 2005 10:50 PM
Subject: Re: [ciencialist] Enviando email: Navier-Stokes-quem se habilita?


Caro Vitor,

É verdade: No meu curso de eng.química chegamos até Bernoulli, um pedaçinho
da N-S como vc disse.

Também acho interessante o assunto e, para ser sincero, já gastei alguns
minutos pensando se deveria passar outros minutos ( Q pretensão a minha !!)
tentando resolver....risos...

Achei um site interessante sobre o assunto:

http://www.navier-stokes.net/

Abraço,

RT


----- Original Message -----
From: "JVictor" <jvoneto@uol.com.br>
To: <ciencialist@yahoogrupos.com.br>
Sent: Sunday, January 02, 2005 9:48 PM
Subject: Re: [ciencialist] Enviando email: Navier-Stokes-quem se habilita?



Rodrigo,

Não. Não tenho a pretensão de entrar para a imortalidade. Coisas assim só
acontecem com os outros. É como um acidente! Tudo que coloquei no e-mail é
mais como ênfase, para exaltar a importância daquela ferramenta formidável,
pois o assunto é deveras emocionante. Nem tenho a pretensão de, dentro de
100 anos(no fim de tempo, voltarei ao assunto, verá...), resolver os
desafios navier-stokianos que ora se apresentam. Agora, quem não gostaria?
Tentar, ao menos, todos podemos. No mínimo, o conhecimento que resulta da
ousadia é de uma riqueza!
Nos cursos de graduação, em geral, tudo o que vemos sobre dinâmica dos
fluidos chega até a Equação de Bernoulli, e suas aplicações. Pelo menos
comigo foi assim, que fiz engenharia. Não sei nos cursos de Física. A
N-Stokes é deduzida a partir da continuidade da massa e considerações
energéticas. Mas esta famosa equação(Bernoulli), bem como todas as outras,
são meros casos particulares da não tão famosa quanto dramática Equação de
Navier-Stokes. Esta não só dá conta de todas as equações que estudamos, como
esclarece os comportamentos dinâmicos dos fluidos, como vorticidade e outras
mazelas. Não explica, mas descreve satisfatoriamente. Você já viu um anel de
fumaça se destacando após uma baforada? Experimente explicar a física
envolvida naquele fenômeno! A PETROBRÁS tem inúmeros e caros programas de
pesquisas a respeito do que a Navier-Stokes pode nos dizer, a respeito do
comportamento de líquidos, consideradas as viscosidades e tudo o mais, em
movimento. A Navier-Stokes tem um termo que resulta proporcional a v ao
quadrado, tornando-a irremediavelmente não-linear. É aí que o bicho pega.
Soluções analíticas, que é bom e todos gostam, só para alguns simples casos
particulares. Então, uma dica é descobrir com escantear o termo (v.delta)v,
v=velocidade do fluido, vetor; delta=operador gradiente; e o ponto
representa o produto escalar. Agora, tudo isso sem alterar a estrutura do
monstrengo(no sentido de dificuldade ou impossiblidade de sempre se obter
soluções analíticas). Pois é.
Mas o assunto é palpitante e seu estudo e compreensão até que não são tão
complicado assim. É só compreender o significado físicco de cada identidade
vetorial que vai surgindo. Pronto.

Victor

----- Original Message -----
From: Rodrigo Toledo
To: ciencialist@yahoogrupos.com.br
Sent: Sunday, January 02, 2005 12:06 PM
Subject: Re: [ciencialist] Enviando email: Navier-Stokes-quem se habilita?


Vitor,

Caso queira tentar outras oportunidades de entrar para a imortalidade (
Com
muito dinheiro !! ), existem outros desafios igualmente, digamos,
complicados.

[]'s

RT

http://www.claymath.org/millennium/
----- Original Message -----
From: "JVictor" <jvoneto@uol.com.br>
To: <ciencialist@yahoogrupos.com.br>
Sent: Friday, December 31, 2004 3:54 PM
Subject: [ciencialist] Enviando email: Navier-Stokes-quem se habilita?



Caros ciencialisteiros,

Eis algo em razão do que estudar. Prêmio: um milhãozinho de verdinhas,
fora
o ingresso para a imortalidade. Já gastei 1/2 caderno de e 4 preciosos
cabelinhos! E nada!
Mas, estudar e pesquisar essa fantástica equação, descobrinho o que ela
esconde já é um bom lazer.
Quem sabe não teremos um 2,76milionário(em nossa fraca moeda) aquí no
ciencialist. Eis o endereço:

http://www.prandiano.com.br/html/m_nov.htm

Observação: para se proteger de vírus de computador, os programas de email
podem impedir o envio ou recebimento de alguns tipos de anexo de arquivo.
Verifique as configurações de email para determinar como os anexos são
manipulados.
Feliz Ano Novo para todos e
Boa sorte!

JVictor.

[As partes desta mensagem que não continham texto foram removidas]



##### ##### #####

Para saber mais visite
http://www.ciencialist.hpg.ig.com.br


##### ##### ##### #####
Links do Yahoo! Grupos










##### ##### #####

Para saber mais visite
http://www.ciencialist.hpg.ig.com.br


##### ##### ##### #####


Yahoo! Grupos, um serviço oferecido por:







----------------------------------------------------------------------------
--
Links do Yahoo! Grupos

a.. Para visitar o site do seu grupo na web, acesse:
http://br.groups.yahoo.com/group/ciencialist/

b.. Para sair deste grupo, envie um e-mail para:
ciencialist-unsubscribe@yahoogrupos.com.br

c.. O uso que você faz do Yahoo! Grupos está sujeito aos Termos do
Serviço do Yahoo!.



[As partes desta mensagem que não continham texto foram removidas]



##### ##### #####

Para saber mais visite
http://www.ciencialist.hpg.ig.com.br


##### ##### ##### #####
Links do Yahoo! Grupos










##### ##### #####

Para saber mais visite
http://www.ciencialist.hpg.ig.com.br


##### ##### ##### #####


Yahoo! Grupos, um serviço oferecido por:
PUBLICIDADE




------------------------------------------------------------------------------
Links do Yahoo! Grupos

a.. Para visitar o site do seu grupo na web, acesse:
http://br.groups.yahoo.com/group/ciencialist/

b.. Para sair deste grupo, envie um e-mail para:
ciencialist-unsubscribe@yahoogrupos.com.br

c.. O uso que você faz do Yahoo! Grupos está sujeito aos Termos do Serviço do Yahoo!.



[As partes desta mensagem que não continham texto foram removidas]



SUBJECT: Re: [ciencialist] Evolucao e desafios - CartaCapital - 02/01/05
FROM: "Oraculo" <oraculo@atibaia.com.br>
TO: <ciencialist@yahoogrupos.com.br>
DATE: 02/01/2005 23:05

Olá Amury

Minhas análises sobre suas análises..:-)

Você parece estar ressentido com o que chama de "ciência" e a trata como entidade independente, crença de cientistas ou algo a ser adorado ou derrubado. Mas nada disso é ciência, nem o resultados de sua aplicação se confundem com ela. Ciência neste contexto (desta lista e do uso padrão do termo..:-) é apenas o conjunto dos conhecimentos obtidos através de um método padronizado, chamado cientifico, e do rigor derivado do mesmo. E, independente do que pense dela, tem elevada confiabilidade..:-)

Não importa se o resultado é passível de ser julgado subjetivametne como "mau" ou mesmo como "bom". Importa apenas que é mais eficiente que outras formas de conhecimento, mais eficaz em suas previsões e mais confiável que outros instrumentos de compreensão do universo já criado por seres humanos. E que será abandonado assim que uma ferramenta mais eficaz seja apresentada..:-) Até lá, é o que temos de melhor.

Podemos analisar filosoficamente, subjetivamente, os resultados, mas não discutir a eficácia. A ciência, o conhecimento assim acumulado, é tão eficiente quando cura doenças, como a erradicação da malária, como quando destroi milhões de vidas, como em Hiroshima. O resultado é passível de julgamento, mas a ação não. Ela é eficaz, nos dois casos. Se precisar salvar alguém ou matar alguém, deve usar a ciência, é mais eficáz sempre..:-)

Não gostar dela, ou como você pretende com as análises, critica-la ou despreza-la, não importa em nada para sua eficácia e confiabilidade. Você vai viver mais que todos os seus antepassados, goste ou não..:-) Sua expectativa de vida é de 75 anos, e, não sei sua idade, mas se já passou dos 40 como eu, é mais que seu avô esperava viver ao nascer (em 1900 a expectativa de vida no Brasil era de 33 anos).

Na verdade, você usou um computador e a Internet, frutos da ciência, para enviar seus pensamentos e ataques a ciência, ao inves de tentar telepatia, rezas ou mandingas (ou qualquer outra forma de comunicação não cientifica) justamente porque é mais eficaz e confiável que qualquer outra..:-)

Você não parece gostar de muitos dos aspectso do uso do conhecimento cientifico. Direito seu. E pode até mesmo abandona-los, todos, e ir viver em uma aldeia remota no Amazonas, sem (quase) nenhum contato com a ciência (os indios são muito receptivos a forasteiros, vão gostar de recebe-lo), e com todo o onus de viver dessa forma (filhos mortos, vida curta, doenças diversas, poucos dentes, etc). Mas está na verdade criticando comportamentos humanos, que são seres falhos como sabe, não a ciência. Esta, "estricto senso", não é boa ou má, cruel ou gentil, apenas eficiente, confiável em seus efeitos.

Você não "acredita" nela. Engraçado, ela não pede que se acredite em nada.:-) Deve duvidar sempre, até que evidencias se mostrem sólidas o bastante para uma conclusão. E é nesse ponto que você escorrega..:-) O pouso na Lua já tem evidencias suficientes para essa conclusão, em que pese as tolices de diversos malucos mundo afora e suas teorias de conspiração. O HIV tem recuado e matado muito menos que no inicio da epidemia, graças a AZT e os atuais coqueteis antivirais. Se ajustes são feitos a toda hora, e são, sobre o que comer ou não, é apenas porque a ciência não pretende ser "A VERDADE" como religiões, mas um conhecimento que cresce, se aprofunda, se ajusta e melhora. Sabemos mais, não menos sobre o que comer ou não comer.

Sim, dá trabalho, é preciso atenção e constante leitura para acompanhar, seria melhor ter uma única resposta para todas as perguntas e pronto, sem risco e sem mudanças. Mas o mundo não funciona assim..:-) Sabemos hoje mais do que ontem sobre como se alimentar, e saberemos mais amanhã, deixando parte do que sabemos hoje para tras. Parte, não tudo:-)

As teorias conspiratórias são atrativas.:-) É como se nós, os que estão "por dentro", soubessemos de "coisas" que o resto do mundo não sabe, fossemos mais "espertos" ou mais inteligentes, que não se deixam enganar. Mas, na maioria das vezes (eu diria em sua totalidade) é apenas o ego tentando ser mais do que é.:-) Mesmo com centenas de sites "desmistificando" o pouso na Lua, o volume de evidencias a favor é gigantesco e nenhum deles explica um ponto fundamental do problema: como o governo americano fraudou os sinais de radio e TV que eram emitidos de todas as naves Apollo tanto durante as viagens, como do solo lunar, captados por todo radio-amador ao redor do globo, sem falar nos radares e antenas dos paises em disputacom os USA? (lembre-se que, mesmo hoje, não se pode fraudar a origem de um sinal de radio..:-)

Enfim, a ciência é eficaz. Embora seu uso, o uso do conhecimento confiável por ela produzido, possa ser discutido e julgado, isso não é a ciência nem com ela se confunde. E, tenha certeza, seu automóvel vai conduzi-lo amanhã, confiavelmente, conforme as leis da fisica descobertas pela ciência, seu computador receberá esta mensagem, os remédios na farmacia impedirão que você morra, as vacinas que tomou evitarão que adoeça (como todos os seus ancestrais adoeceram) e os satelites em órbita, colocados lá pelo conhecimento cientifico, vão transmitir suas chamadas telefonicas até para o Japão, se você assim desejar..:-)

Um abraço.

Homero




----- Original Message -----
From: Amauri Jr
To: ciencialist@yahoogrupos.com.br
Sent: Sunday, January 02, 2005 3:18 PM
Subject: Re: [ciencialist] Evolucao e desafios - CartaCapital - 02/01/05


Minhas analises em baixo com [A]....
----- Original Message -----
From: L.E.R.de Carvalho
To: ciencialist@yahoogrupos.com.br
Sent: Sunday, January 02, 2005 1:52 PM
Subject: [ciencialist] Evolucao e desafios - CartaCapital - 02/01/05



>LabConsS - www.ufrj.br/consumo
>
>
>
> EVOLUÇÃO E DESAFIOS
>
>
> O País viu inúmeros avanços nas áreas médica e
>tecnológica, mas as carências sociais ainda são sua
>pior mazela
>
>Em uma livraria, olho rapidamente as manchetes das
>revistas expostas. A julgar pelas capas, é pouco
>provável que tenhamos problemas de saúde deste mês em
>diante. Soluções curativas. Remédios extremamente
>eficientes. Vida prolongada. Corpos e rostos sem um
>defeito. Independentemente da idade, claro. Câncer?
>Uma brincadeira. Paralisia? Pode preparar as pistas.
>Infarto? E daí?

[A] temos que pagar e muito caro para a "ciencia" nos curar de males que diz a "ciencia" um direito de todos.

>
>Deixando de lado uma razoável dose de exagero nessas
>manchetes, encho o peito de orgulho pelas proezas
>atingidas no campo da medicina e da saúde. Vinte anos
>atrás, tudo isso não passaria de sonho, e não
>existiria fora da cabeça imaginativa dos produtores de
>filmes de ficção científica. Mas, ainda hoje, para a
>esmagadora maioria dos cidadãos brasileiros, todo o
>acima mencionado não passa de um sonho, de uma ficção
>científica

{A] Parece que todo a ciencia é uma ficção cientifica, vamos dizer que o pensamento é um comercio hoje em dia, ficçao ou não, é com certeza sensacionalismo.

>Este ano que acaba nos deu boas notícias,
>encorajadoras, e notícias ruins, no mínimo
>preocupantes. Vimos uma senhora que sofreu um derrame
>cerebral voltar a andar com terapia baseada na
>introdução de células-tronco (células primitivas
>capazes de se transformar em praticamente qualquer
>outra célula normal) no cérebro afetado. Emocionante
>perceber o que esses pequenos passos podem significar
>para milhões de pessoas que não conseguem erguer o
>braço, ou mexer a perna. Parafraseando Neil Armstrong,
>um passo gigante para a humanidade.

{A} Armstrong, quem é ele? Aquele que "supostamente" foi a Lua? Celulas- tronco é um desafio para a humanidade de cura, o desejo de ajudar seu semelhante. Pensamentos inovadores tem estado junto a humanidade a muito tempo, isso pode ser descrito na historia de Tales de Mileto até Einstein




>A técnica das células-tronco não se restringe a
>restaurar a função de células cerebrais. Hoje em dia,
>pesquisadores brasileiros, e em outros países,
>intensificam seus esforços para melhorar o coração
>depois de infarto, os nervos após lesão traumática, a
>pele após queimadura. A lista parece não ter fim.

[A] A demora faz o crescimento dos beneficiados...
>
>Vimos a introdução de remédios geniais na prática
>médica. Geniais na sua concepção, e geniais na sua
>eficiência. Drogas que conseguem agir em um ponto
>específico da célula doente, da célula cancerosa,
>dificultando seu desenvolvimento, seu crescimento, e
>até provocando sua morte. Exemplos que estão
>progressivamente sendo utilizados na prática médica,
>apesar de somente em casos muito selecionados, não
>faltam. Glivec, Iressa, são alguns deles.

[A] Santa ciencia!! Mata e de forma covarde porque usa seus conhecimentos de forma de interesse, mesmo que sabemos, esses conhecimentos são para todos.
>
>Ainda nem acabou o ano e ouvimos há poucas semanas o
>anúncio pela GlaxoSmithKline de uma vacina contra o
>câncer. O Brasil está totalmente empenhado na produção
>da vacina contra o HPV, vírus causador de câncer de
>útero. O prof. dr. Ricardo Brentani, presidente da
>Fundação Antonio Prudente, Hospital do Câncer AC
>Camargo, está entusiasmado e declara:
>
>­ Felizmente nossa instituição foi parceira da Merck
>Sharp & Dohme no desenvolvimento de uma vacina contra
>o HPV. Minha esperança é que em 20 anos tenhamos
>prevenido 7% dos tumores humanos.

[A] Vacina? Como pode ter vacina para nossas proprias celulas? E do HIV vai ter?

>Vimos, por outro lado, a preocupação crescente dos
>cientistas em não excluir, a priori, tratamentos
>considerados até então não-convencionais, ou
>complementares. Estudos sérios avaliaram desde a
>acupuntura até a homeopatia, passando por terapias de
>Florais de Bach e tratamentos com vitaminas nas mais
>diversas situações clínicas. Vantagens e desvantagens
>de cada abordagem foram dissecadas minuciosamente.

[A] Viu? Sera que alguem aqui duvidda?


>Cientistas conseguiram demonstrar a eficiência
>incontestável de algumas terapias alternativas em
>certas situações, como a massagem para um dos males do
>século, a dor nas costas. Por outro lado, alertaram
>para a ineficiência e até os efeitos nocivos de outras
>terapias, em outras condições. Por exemplo, para
>tratar bronquite e asma a acupuntura não parece ter
>efeitos importantes. Pelo menos não conseguiram
>detectar esses efeitos nos estudos atuais.

[A] Remedios sim? Eu tive começo, eu e meus manos, de bronquite depois que meu pai comprou um tartaruga nunca mais...santa crença ne? Mas como explicar??

>Vimos o lançamento de aparelhos ultramodernos,
>ultra-sensíveis, para detectar doenças e tratá-las.
>Não consigo perceber avanço recente maior do que na
>área da radioterapia. É incrível a precisão dos feixes
>de radiação em atingir o alvo, no caso o câncer, e
>poupar o tecido normal adjacente. Aparelhos novos
>conseguem acompanhar o movimento do corpo para seguir
>o alvo predeterminado, segundo a segundo, obedecendo à
>orientação do médico radioterapeuta. Reduziram-se
>muito os efeitos colaterais. O controle do câncer
>assemelha-se às extensas cirurgias. A nanotecnologia
>(aparelhos miniaturas) é uma febre, e os estudos
>multiplicam-se para definir com mais clareza sua
>aplicação.

{A} Pode matar com sua radioatividade, causando cancer.

>
>A plástica está fazendo tamanhos milagres que nem os
>próprios pacientes conseguem acreditar. Tanto faz quem
>foram seus pais, ou a etnia a qual você pertença.
>Escolha o modelo e ficará parecido. Para pessoas
>ansiosas com a imagem, soluções para quase tudo. Sem
>dúvida, o impacto sobre o estado emocional é notável.

[A] Eita faquinha de dois cumes heim? A plastica pode curar a pessoa de queimaduras ou cicatrizes, mas tambem pode ressaltar sua vaidade.

>
>Ao lado dessas notícias que mereceram um destaque
>quase obsessivo nas manchetes de capa, outras
>informações com menos, digamos, glamour, não
>conseguiram espaço nem nas páginas finais das
>revistas.
[A] A ciencia ja tem a imprensa e seus colaborados para dar esse glamour...


>
>O objetivo de reduzir em 60% a mortalidade infantil no
>mundo (e o Brasil ainda é um grande protagonista dessa
>estatística) não será atingido em 2015, como
>estabelecido nos Objetivos Milenares da ONU. No
>planeta, morrem por ano mais de 11 milhões de crianças
>com idade inferior a 5 anos. A maioria por doenças
>evitáveis. Diarréia, pneumonia, malária. No mesmo
>período, 500 mil mulheres morrem durante a gravidez ou
>o parto. Doença de Chagas e esquistossomose
>(barriga-d'água) continuam afetando milhares de
>brasileiros. E seu controle está cada vez mais
>próximo. A implementação dos programas já em ação
>poderá melhorar ainda mais esse controle. Para 2005 a
>intensificação dessas abordagens poderá elevar o
>impacto na saúde da população, principalmente nas
>áreas rurais.

[A] Isso ai, como fica doenças tao mais antiga diante do deusa peerfeita ciencia??


>O Relatório Mundial de Saúde (The World Health Report
>2003) recomendou às autoridades o fortalecimento dos
>sistemas de saúde, centralizando seu foco na atenção
>primária, além de integrar a prevenção das doenças e a
>promoção da saúde em todos os níveis de atendimento.

[A] Esses relatorios tem efeito??

>
>A saúde do homem sofreu mudanças drásticas nos últimos
>anos. A expectativa de vida, de brasileiros e de não
>brasileiros, bate recorde atrás de recorde.
>Ultrapassou a marca dos 70 anos e logo passará dos 80,
>90, e quem sabe 100 anos. Avanços sem dúvida notáveis.
>Doenças contagiosas foram substituídas por doenças
>crônicas, como problemas cardiovasculares e câncer, e
>por causas externas, como trauma. Essa mudança de foco
>exigiu modificações intensivas no nível estrutural.
>Algumas especialidades assumiram posição de destaque
>nunca antes alcançada. Os serviços de cardiologia de
>muitos hospitais logo se transformaram em
>departamentos e, a seguir, em grandes centros com
>prédios próprios. O mesmo ocorreu com a oncologia e
>suas variadas especialidades. Houve uma explosão na
>tecnologia e na sofisticação, e, conseqüentemente, nos
>custos ­ alertam os especialistas em saúde pública.
>Recomenda-se também a melhora da saúde da população
>por intermédio do envolvimento acadêmico.
>

[A] Sem comentarios, pra que vou querer viver até o 100?
>
>Muito por fazer.
>Como melhorar o atendimento ao público do SUS

[A] Infelizmente a ciencia não melhora ela mesma vai melhorar o SUS, a ciencia infelizmente, anda atendendo interesses dos laboratorios e pondo em pratica, o que determina o codigo americano, Bisness in bisness...
>
>
>A academia médica no Brasil foi alterada sensivelmente
>nas últimas duas décadas. Progressivamente, as escolas
>médicas e os programas de pós-graduação introduziram
>disciplinas de pesquisa epidemiológica de problemas de
>saúde adaptados à realidade da população do País. Mais
>e mais livros têm sido publicados por acadêmicos
>brasileiros, com enfoque nacional. Mais e mais centros
>de treinamento oferecem vagas para médicos
>interessados em se aperfeiçoar no atendimento básico à
>saúde da família e à clínica geral. O impacto na
>melhora da atenção à saúde do brasileiro pode demorar
>a ser notado. Muito há de ser feito em 2005 e além.

[A] Infelizmente, tem muito moleque fazendo medicina pro papai e só sai meleca, ou o diagnostico é virose ou é exames a "toa"...


>O plano Fome Zero tenta corrigir um problema básico de
>saúde: a miséria e a conseqüente desnutrição. Ninguém
>precisa de doutorado para compreender a conexão direta
>entre a desnutrição e a ocorrência de doenças
>potencialmente graves. É um plano interessante, mas
>ainda longe de atingir seus objetivos de forma
>significativa. Em 2005, e além, o governo deverá fazer
>os ajustes finos para que o programa consiga alcançar
>seu alvo.
>
>A violência é problema de segurança pública, mas
>também é problema de saúde. Não podemos esquecer que,
>no ano passado, mais de 40 mil brasileiros foram
>assassinados. Se acreditarmos nos dados oficiais do
>Ministério da Saúde, naquele período, morreram mais
>brasileiros por tiro do que por câncer de pulmão. O
>controle da violência evitará milhares de mortes e
>seqüelas dramáticas.
>
>A malária é outro problema sem solução a curto prazo.
>Áreas extensas do território nacional são infestadas
>por mosquitos portadores dessa doença. O esforço das
>autoridades regionais e federais deverá ser
>intensificado nas várias frentes: pesquisa de vacinas,
>profilaxia, controle do mosquito e tratamento de
>pacientes infectados. A mortalidade por essa infecção
>deve ser reduzida a todo custo. Estudos recentes
>aventam que a vacina contra a malária pode estar ao
>alcance da ciência em prazo razoável.
>
>No mundo todo, um bilhão de pessoas são infectadas por
>parasitas, como os vermes, e o Brasil tem participação
>significativa nesses dados alarmantes. Vermes no
>intestino não são apenas feios. Eles causam problemas
>como deficiências nutricionais, e, em alguns casos,
>podem até ser fatais. Para mudar isso, saneamento
>básico é fundamental.
>
>Um estudo recentemente realizado e publicado por
>pesquisadores na Universidade do Ceará demonstrou
>claramente a possibilidade de praticamente eliminar os
>parasitas dos pacientes. Remédios eficazes, como a
>ivemerctina, podem ser a solução. Além de tratar o
>paciente, eliminam uma potencial fonte de contaminação
>para outras pessoas. As autoridades devem criar
>sistemas integrados, contínuos, para atacar esse
>problema de saúde pública em várias frentes, em 2005 e
>para sempre.
>
>Quanto à Aids, nos últimos anos, houve um declínio da
>incidência de novos casos na maioria dos estados
>brasileiros. Estudo publicado recentemente por
>pesquisadores da Fundação Oswaldo Cruz, no Rio de
>Janeiro, confirmou que, apesar de recursos limitados
>do governo e da desigualdade socioeconômica aberrante
>no Brasil, a introdução do acesso universal à terapia
>antiviral contribuiu para uma redução impressionante
>na mortalidade por Aids, e pode ter ajudado a diminuir
>sua incidência.
>
>Nessa linha de pensamento, o presidente Lula lançou,
>este ano, o louvável Plano Nacional de Eliminação da
>Hanseníase (a lepra) em até dois anos. Um passo sem
>dúvida importante para controlar essa doença
>contagiosa e debilitante. Paralelamente, o ministro da
>Saúde, Humberto Costa, anunciou a formação de uma
>comissão para promover políticas nacionais de
>reabilitação às pessoas afetadas pela hanseníase:
>
>­ A idéia é possibilitar benefícios, como a
>reabilitação por cirurgias plásticas, financiando para
>a capacitação de médicos, enfermeiros e
>fisioterapeutas para que realizem esses procedimentos
>nos incapacitados fisicamente.
>
>Todos os anos, 42 mil brasileiros contraem a
>hanseníase. E o País só perde para a Índia no ranking
>mundial de casos da doença.
>
>A obesidade, mal do século XXI, mata. Nos EUA, em
>alguns estados o número de óbitos por obesidade
>ultrapassou o de câncer. No Brasil, cientistas
>identificaram obesidade crescente na população. Estudo
>publicado por pesquisadores da Universidade Federal do
>Rio de Janeiro confirmou o aumento progressivo do peso
>dos adolescentes, tanto na Região Nordeste quanto no
>Sudeste do País, desde 1975. Atualmente, 17% dos
>adolescentes da Região Sudeste enquadram-se na
>definição internacional de obesidade. E seus efeitos
>maléficos logo alcançarão a saúde pública. Orientação
>e conscientização devem ser política contínua e bem
>estruturada. A prevenção e o tratamento do sobrepeso
>precisam ser prioridades das políticas de saúde nos
>próximos anos.
>
>Um problema que acomete principalmente as crianças é a
>poluição atmosférica. Um estudo realizado com 5.193
>crianças de duas cidades do Rio de Janeiro, e
>publicado recentemente na revista Annals of Allergy
>Asthma and Immunology, mostrou claramente a correlação
>entre os níveis de poluição e a incidência de
>problemas respiratórios. Observou-se um aumento de
>mais de 50% na freqüência de crises de asma nos
>moradores de Duque de Caxias (local com alta
>concentração de poluentes), comparados aos moradores
>de Seropédica (local com baixos níveis de poluição).
>Outro estudo, realizado pelos pesquisadores do
>laboratório de poluição atmosférica da Universidade de
>São Paulo, confirmou os efeitos nocivos da poluição do
>ar na mortalidade infantil. Há aumento de 6% no número
>de óbitos neonatais nas regiões mais poluídas. O
>controle de poluentes de qualquer origem deverá ser
>obrigação mundial. Ainda mais com a entrada em vigor
>do Protocolo de Kyoto.
>
>Mas, de forma geral, provavelmente o maior desafio das
>autoridades de saúde é garantir um apoio ao avanço
>científico na área de saúde, atingir a maioria da
>população com os avanços médicos e tecnológicos
>alcançados, e insistir na prevenção e na saúde
>primária, básica. Muito mais do que dinheiro, em 2005
>precisamos de estratégia, filosofia e enfoque novos.
>
>O acesso da população, principalmente das camadas
>menos privilegiadas da sociedade, a remédios em geral
>é muito limitado. Várias são as causas, como preços
>elevados na fonte, acréscimos excessivos na cadeia de
>distribuição e impostos. Se as autoridades, de todas
>as esferas, não atentarem a essas dificuldades
>enfrentadas no dia-a-dia da maioria dos cidadãos,
>muito mais será gasto para tratar complicações graves
>de doenças crônicas, como diabetes e hipertensão
>arterial. O Brasil iniciou uma ação interessante nessa
>direção: a aquisição da fábrica da GlaxoSmithKline do
>Brasil pelo governo federal. O presidente Lula
>destacou que "pela primeira vez um governo compra uma
>fábrica da iniciativa privada", ao mesmo tempo
>criticando "o processo inverso de privatização do
>governo anterior". Lula declarou, na ocasião, que
>estava "recuperando uma fábrica que seria desativada,
>eliminaria empregos e que agora produzirá, em escala
>nacional, os antibióticos mais usados no Brasil".
>
>Nessa mesma linha o ministro da Saúde, Humberto Costa,
>deixou muito clara sua estratégia para 2005:
>
>­ Vamos ampliar o acesso da população a medicamentos
>fundamentais com base em três opções. A primeira é a
>rede de farmácias populares, e até o fim do ano
>pretendemos inaugurar cem unidades em todo o Brasil. A
>segunda é a criação de um programa para a venda
>subsidiada de medicamentos básicos para hipertensão e
>diabetes, através da rede de farmácias privada.
>Pretendemos baixar os preços em até 50%. E a terceira
>é a redução do ICMS de 2,8 mil medicamentos até 2005.
>
>De acordo com uma pesquisa do IBGE, de 2003, a saúde
>aparece em terceiro lugar no orçamento das famílias
>brasileiras, e os medicamentos representam 61% desses
>gastos para as pessoas de baixa renda. Além disso,
>metade das pessoas que precisam de tratamento não pode
>pagar os remédios de que necessitam.
>
>A partir de 2005, 50 milhões de unidades de
>antibióticos serão produzidas para a rede do Sistema
>Único de Saúde (SUS) e para as Farmácias Populares. Em
>2007, a produção deve quintuplicar em relação à atual.
>Mais de 10 bilhões de unidades de medicamentos devem
>ser produzidas para as principais doenças que mais
>atingem a população brasileira, como hipertensão,
>diabetes, malária e tuberculose. A Fiocruz também
>produzirá mais vacinas, e o Ministério da Saúde
>enfatiza que "a ampliação do acesso da população aos
>medicamentos é uma das prioridades do governo
>federal".
>
>Em 2004, muito se fez pela saúde, mas ainda há muito
>mais por fazer. Programas de prevenção e detecção
>precoce de doenças sexualmente transmissíveis, como
>Aids, HPV (câncer de colo de útero), a melhoria na
>qualidade do atendimento dos pacientes do SUS,
>diminuição das filas, controle de poluição, de
>infecção hospitalar e, também, de mortes por agentes
>externos, como traumas (tiros, facadas, acidentes de
>trânsito). Para se ter uma idéia do custo com os
>acidentes de trânsito, a cada ano acontecem mais de
>100 mil mortes, com três a quatro vezes esse número de
>feridos. E a maioria absoluta é tratada com dinheiro
>público. São milhões de reais de gastos evitáveis. Sem
>falar nas seqüelas individuais.
>

Abraços
Amauri

[As partes desta mensagem que não continham texto foram removidas]



##### ##### #####

Para saber mais visite
http://www.ciencialist.hpg.ig.com.br


##### ##### ##### #####



Yahoo! Grupos, um serviço oferecido por:
PUBLICIDADE




------------------------------------------------------------------------------
Links do Yahoo! Grupos

a.. Para visitar o site do seu grupo na web, acesse:
http://br.groups.yahoo.com/group/ciencialist/

b.. Para sair deste grupo, envie um e-mail para:
ciencialist-unsubscribe@yahoogrupos.com.br

c.. O uso que você faz do Yahoo! Grupos está sujeito aos Termos do Serviço do Yahoo!.



[As partes desta mensagem que não continham texto foram removidas]



##### ##### #####

Para saber mais visite
http://www.ciencialist.hpg.ig.com.br


##### ##### ##### #####


Yahoo! Grupos, um serviço oferecido por:







------------------------------------------------------------------------------
Links do Yahoo! Grupos

a.. Para visitar o site do seu grupo na web, acesse:
http://br.groups.yahoo.com/group/ciencialist/

b.. Para sair deste grupo, envie um e-mail para:
ciencialist-unsubscribe@yahoogrupos.com.br

c.. O uso que você faz do Yahoo! Grupos está sujeito aos Termos do Serviço do Yahoo!.



[As partes desta mensagem que não continham texto foram removidas]



SUBJECT: Re: [ciencialist] Evolucao e desafios - CartaCapital - 02/01/05
FROM: "Oraculo" <oraculo@atibaia.com.br>
TO: <ciencialist@yahoogrupos.com.br>
DATE: 02/01/2005 23:10

Olá Amaury

Para completar a mensagem anterior: uma menina inglesa de 15 aos foi a primeira pessoa a sobreviver depois de contrair raiva sem tomar a vacina antirabica. Ela foi submetida a um novo tratamento revolucionário (e científico), com um coquetel de drogas e coma induzido.

Diferente de milagres divinos e sobrenaturais, o que se aprendeu com a nova técnica pode ser usado em outras pessoas doentes e não afetam apenas a pessoa que recebe a graça..:-)

A partir de agora, mesmo desprezando a "ciência", você pode ficar sossegado, se você ou um de seus filhos ou entes queridos contrair raiva, poderá ser curado, cientificamente curado.:-) A não ser que tenha resolvido ir viver em uma aldeia indigena na remota Amazonia...

Um abraço.

Homero

----- Original Message -----
From: Amauri Jr
To: ciencialist@yahoogrupos.com.br
Sent: Sunday, January 02, 2005 3:18 PM
Subject: Re: [ciencialist] Evolucao e desafios - CartaCapital - 02/01/05


Minhas analises em baixo com [A]....
----- Original Message -----
From: L.E.R.de Carvalho
To: ciencialist@yahoogrupos.com.br
Sent: Sunday, January 02, 2005 1:52 PM
Subject: [ciencialist] Evolucao e desafios - CartaCapital - 02/01/05



>LabConsS - www.ufrj.br/consumo
>
>
>
> EVOLUÇÃO E DESAFIOS
>
>
> O País viu inúmeros avanços nas áreas médica e
>tecnológica, mas as carências sociais ainda são sua
>pior mazela
>
>Em uma livraria, olho rapidamente as manchetes das
>revistas expostas. A julgar pelas capas, é pouco
>provável que tenhamos problemas de saúde deste mês em
>diante. Soluções curativas. Remédios extremamente
>eficientes. Vida prolongada. Corpos e rostos sem um
>defeito. Independentemente da idade, claro. Câncer?
>Uma brincadeira. Paralisia? Pode preparar as pistas.
>Infarto? E daí?

[A] temos que pagar e muito caro para a "ciencia" nos curar de males que diz a "ciencia" um direito de todos.

>
>Deixando de lado uma razoável dose de exagero nessas
>manchetes, encho o peito de orgulho pelas proezas
>atingidas no campo da medicina e da saúde. Vinte anos
>atrás, tudo isso não passaria de sonho, e não
>existiria fora da cabeça imaginativa dos produtores de
>filmes de ficção científica. Mas, ainda hoje, para a
>esmagadora maioria dos cidadãos brasileiros, todo o
>acima mencionado não passa de um sonho, de uma ficção
>científica

{A] Parece que todo a ciencia é uma ficção cientifica, vamos dizer que o pensamento é um comercio hoje em dia, ficçao ou não, é com certeza sensacionalismo.

>Este ano que acaba nos deu boas notícias,
>encorajadoras, e notícias ruins, no mínimo
>preocupantes. Vimos uma senhora que sofreu um derrame
>cerebral voltar a andar com terapia baseada na
>introdução de células-tronco (células primitivas
>capazes de se transformar em praticamente qualquer
>outra célula normal) no cérebro afetado. Emocionante
>perceber o que esses pequenos passos podem significar
>para milhões de pessoas que não conseguem erguer o
>braço, ou mexer a perna. Parafraseando Neil Armstrong,
>um passo gigante para a humanidade.

{A} Armstrong, quem é ele? Aquele que "supostamente" foi a Lua? Celulas- tronco é um desafio para a humanidade de cura, o desejo de ajudar seu semelhante. Pensamentos inovadores tem estado junto a humanidade a muito tempo, isso pode ser descrito na historia de Tales de Mileto até Einstein




>A técnica das células-tronco não se restringe a
>restaurar a função de células cerebrais. Hoje em dia,
>pesquisadores brasileiros, e em outros países,
>intensificam seus esforços para melhorar o coração
>depois de infarto, os nervos após lesão traumática, a
>pele após queimadura. A lista parece não ter fim.

[A] A demora faz o crescimento dos beneficiados...
>
>Vimos a introdução de remédios geniais na prática
>médica. Geniais na sua concepção, e geniais na sua
>eficiência. Drogas que conseguem agir em um ponto
>específico da célula doente, da célula cancerosa,
>dificultando seu desenvolvimento, seu crescimento, e
>até provocando sua morte. Exemplos que estão
>progressivamente sendo utilizados na prática médica,
>apesar de somente em casos muito selecionados, não
>faltam. Glivec, Iressa, são alguns deles.

[A] Santa ciencia!! Mata e de forma covarde porque usa seus conhecimentos de forma de interesse, mesmo que sabemos, esses conhecimentos são para todos.
>
>Ainda nem acabou o ano e ouvimos há poucas semanas o
>anúncio pela GlaxoSmithKline de uma vacina contra o
>câncer. O Brasil está totalmente empenhado na produção
>da vacina contra o HPV, vírus causador de câncer de
>útero. O prof. dr. Ricardo Brentani, presidente da
>Fundação Antonio Prudente, Hospital do Câncer AC
>Camargo, está entusiasmado e declara:
>
>­ Felizmente nossa instituição foi parceira da Merck
>Sharp & Dohme no desenvolvimento de uma vacina contra
>o HPV. Minha esperança é que em 20 anos tenhamos
>prevenido 7% dos tumores humanos.

[A] Vacina? Como pode ter vacina para nossas proprias celulas? E do HIV vai ter?

>Vimos, por outro lado, a preocupação crescente dos
>cientistas em não excluir, a priori, tratamentos
>considerados até então não-convencionais, ou
>complementares. Estudos sérios avaliaram desde a
>acupuntura até a homeopatia, passando por terapias de
>Florais de Bach e tratamentos com vitaminas nas mais
>diversas situações clínicas. Vantagens e desvantagens
>de cada abordagem foram dissecadas minuciosamente.

[A] Viu? Sera que alguem aqui duvidda?


>Cientistas conseguiram demonstrar a eficiência
>incontestável de algumas terapias alternativas em
>certas situações, como a massagem para um dos males do
>século, a dor nas costas. Por outro lado, alertaram
>para a ineficiência e até os efeitos nocivos de outras
>terapias, em outras condições. Por exemplo, para
>tratar bronquite e asma a acupuntura não parece ter
>efeitos importantes. Pelo menos não conseguiram
>detectar esses efeitos nos estudos atuais.

[A] Remedios sim? Eu tive começo, eu e meus manos, de bronquite depois que meu pai comprou um tartaruga nunca mais...santa crença ne? Mas como explicar??

>Vimos o lançamento de aparelhos ultramodernos,
>ultra-sensíveis, para detectar doenças e tratá-las.
>Não consigo perceber avanço recente maior do que na
>área da radioterapia. É incrível a precisão dos feixes
>de radiação em atingir o alvo, no caso o câncer, e
>poupar o tecido normal adjacente. Aparelhos novos
>conseguem acompanhar o movimento do corpo para seguir
>o alvo predeterminado, segundo a segundo, obedecendo à
>orientação do médico radioterapeuta. Reduziram-se
>muito os efeitos colaterais. O controle do câncer
>assemelha-se às extensas cirurgias. A nanotecnologia
>(aparelhos miniaturas) é uma febre, e os estudos
>multiplicam-se para definir com mais clareza sua
>aplicação.

{A} Pode matar com sua radioatividade, causando cancer.

>
>A plástica está fazendo tamanhos milagres que nem os
>próprios pacientes conseguem acreditar. Tanto faz quem
>foram seus pais, ou a etnia a qual você pertença.
>Escolha o modelo e ficará parecido. Para pessoas
>ansiosas com a imagem, soluções para quase tudo. Sem
>dúvida, o impacto sobre o estado emocional é notável.

[A] Eita faquinha de dois cumes heim? A plastica pode curar a pessoa de queimaduras ou cicatrizes, mas tambem pode ressaltar sua vaidade.

>
>Ao lado dessas notícias que mereceram um destaque
>quase obsessivo nas manchetes de capa, outras
>informações com menos, digamos, glamour, não
>conseguiram espaço nem nas páginas finais das
>revistas.
[A] A ciencia ja tem a imprensa e seus colaborados para dar esse glamour...


>
>O objetivo de reduzir em 60% a mortalidade infantil no
>mundo (e o Brasil ainda é um grande protagonista dessa
>estatística) não será atingido em 2015, como
>estabelecido nos Objetivos Milenares da ONU. No
>planeta, morrem por ano mais de 11 milhões de crianças
>com idade inferior a 5 anos. A maioria por doenças
>evitáveis. Diarréia, pneumonia, malária. No mesmo
>período, 500 mil mulheres morrem durante a gravidez ou
>o parto. Doença de Chagas e esquistossomose
>(barriga-d'água) continuam afetando milhares de
>brasileiros. E seu controle está cada vez mais
>próximo. A implementação dos programas já em ação
>poderá melhorar ainda mais esse controle. Para 2005 a
>intensificação dessas abordagens poderá elevar o
>impacto na saúde da população, principalmente nas
>áreas rurais.

[A] Isso ai, como fica doenças tao mais antiga diante do deusa peerfeita ciencia??


>O Relatório Mundial de Saúde (The World Health Report
>2003) recomendou às autoridades o fortalecimento dos
>sistemas de saúde, centralizando seu foco na atenção
>primária, além de integrar a prevenção das doenças e a
>promoção da saúde em todos os níveis de atendimento.

[A] Esses relatorios tem efeito??

>
>A saúde do homem sofreu mudanças drásticas nos últimos
>anos. A expectativa de vida, de brasileiros e de não
>brasileiros, bate recorde atrás de recorde.
>Ultrapassou a marca dos 70 anos e logo passará dos 80,
>90, e quem sabe 100 anos. Avanços sem dúvida notáveis.
>Doenças contagiosas foram substituídas por doenças
>crônicas, como problemas cardiovasculares e câncer, e
>por causas externas, como trauma. Essa mudança de foco
>exigiu modificações intensivas no nível estrutural.
>Algumas especialidades assumiram posição de destaque
>nunca antes alcançada. Os serviços de cardiologia de
>muitos hospitais logo se transformaram em
>departamentos e, a seguir, em grandes centros com
>prédios próprios. O mesmo ocorreu com a oncologia e
>suas variadas especialidades. Houve uma explosão na
>tecnologia e na sofisticação, e, conseqüentemente, nos
>custos ­ alertam os especialistas em saúde pública.
>Recomenda-se também a melhora da saúde da população
>por intermédio do envolvimento acadêmico.
>

[A] Sem comentarios, pra que vou querer viver até o 100?
>
>Muito por fazer.
>Como melhorar o atendimento ao público do SUS

[A] Infelizmente a ciencia não melhora ela mesma vai melhorar o SUS, a ciencia infelizmente, anda atendendo interesses dos laboratorios e pondo em pratica, o que determina o codigo americano, Bisness in bisness...
>
>
>A academia médica no Brasil foi alterada sensivelmente
>nas últimas duas décadas. Progressivamente, as escolas
>médicas e os programas de pós-graduação introduziram
>disciplinas de pesquisa epidemiológica de problemas de
>saúde adaptados à realidade da população do País. Mais
>e mais livros têm sido publicados por acadêmicos
>brasileiros, com enfoque nacional. Mais e mais centros
>de treinamento oferecem vagas para médicos
>interessados em se aperfeiçoar no atendimento básico à
>saúde da família e à clínica geral. O impacto na
>melhora da atenção à saúde do brasileiro pode demorar
>a ser notado. Muito há de ser feito em 2005 e além.

[A] Infelizmente, tem muito moleque fazendo medicina pro papai e só sai meleca, ou o diagnostico é virose ou é exames a "toa"...


>O plano Fome Zero tenta corrigir um problema básico de
>saúde: a miséria e a conseqüente desnutrição. Ninguém
>precisa de doutorado para compreender a conexão direta
>entre a desnutrição e a ocorrência de doenças
>potencialmente graves. É um plano interessante, mas
>ainda longe de atingir seus objetivos de forma
>significativa. Em 2005, e além, o governo deverá fazer
>os ajustes finos para que o programa consiga alcançar
>seu alvo.
>
>A violência é problema de segurança pública, mas
>também é problema de saúde. Não podemos esquecer que,
>no ano passado, mais de 40 mil brasileiros foram
>assassinados. Se acreditarmos nos dados oficiais do
>Ministério da Saúde, naquele período, morreram mais
>brasileiros por tiro do que por câncer de pulmão. O
>controle da violência evitará milhares de mortes e
>seqüelas dramáticas.
>
>A malária é outro problema sem solução a curto prazo.
>Áreas extensas do território nacional são infestadas
>por mosquitos portadores dessa doença. O esforço das
>autoridades regionais e federais deverá ser
>intensificado nas várias frentes: pesquisa de vacinas,
>profilaxia, controle do mosquito e tratamento de
>pacientes infectados. A mortalidade por essa infecção
>deve ser reduzida a todo custo. Estudos recentes
>aventam que a vacina contra a malária pode estar ao
>alcance da ciência em prazo razoável.
>
>No mundo todo, um bilhão de pessoas são infectadas por
>parasitas, como os vermes, e o Brasil tem participação
>significativa nesses dados alarmantes. Vermes no
>intestino não são apenas feios. Eles causam problemas
>como deficiências nutricionais, e, em alguns casos,
>podem até ser fatais. Para mudar isso, saneamento
>básico é fundamental.
>
>Um estudo recentemente realizado e publicado por
>pesquisadores na Universidade do Ceará demonstrou
>claramente a possibilidade de praticamente eliminar os
>parasitas dos pacientes. Remédios eficazes, como a
>ivemerctina, podem ser a solução. Além de tratar o
>paciente, eliminam uma potencial fonte de contaminação
>para outras pessoas. As autoridades devem criar
>sistemas integrados, contínuos, para atacar esse
>problema de saúde pública em várias frentes, em 2005 e
>para sempre.
>
>Quanto à Aids, nos últimos anos, houve um declínio da
>incidência de novos casos na maioria dos estados
>brasileiros. Estudo publicado recentemente por
>pesquisadores da Fundação Oswaldo Cruz, no Rio de
>Janeiro, confirmou que, apesar de recursos limitados
>do governo e da desigualdade socioeconômica aberrante
>no Brasil, a introdução do acesso universal à terapia
>antiviral contribuiu para uma redução impressionante
>na mortalidade por Aids, e pode ter ajudado a diminuir
>sua incidência.
>
>Nessa linha de pensamento, o presidente Lula lançou,
>este ano, o louvável Plano Nacional de Eliminação da
>Hanseníase (a lepra) em até dois anos. Um passo sem
>dúvida importante para controlar essa doença
>contagiosa e debilitante. Paralelamente, o ministro da
>Saúde, Humberto Costa, anunciou a formação de uma
>comissão para promover políticas nacionais de
>reabilitação às pessoas afetadas pela hanseníase:
>
>­ A idéia é possibilitar benefícios, como a
>reabilitação por cirurgias plásticas, financiando para
>a capacitação de médicos, enfermeiros e
>fisioterapeutas para que realizem esses procedimentos
>nos incapacitados fisicamente.
>
>Todos os anos, 42 mil brasileiros contraem a
>hanseníase. E o País só perde para a Índia no ranking
>mundial de casos da doença.
>
>A obesidade, mal do século XXI, mata. Nos EUA, em
>alguns estados o número de óbitos por obesidade
>ultrapassou o de câncer. No Brasil, cientistas
>identificaram obesidade crescente na população. Estudo
>publicado por pesquisadores da Universidade Federal do
>Rio de Janeiro confirmou o aumento progressivo do peso
>dos adolescentes, tanto na Região Nordeste quanto no
>Sudeste do País, desde 1975. Atualmente, 17% dos
>adolescentes da Região Sudeste enquadram-se na
>definição internacional de obesidade. E seus efeitos
>maléficos logo alcançarão a saúde pública. Orientação
>e conscientização devem ser política contínua e bem
>estruturada. A prevenção e o tratamento do sobrepeso
>precisam ser prioridades das políticas de saúde nos
>próximos anos.
>
>Um problema que acomete principalmente as crianças é a
>poluição atmosférica. Um estudo realizado com 5.193
>crianças de duas cidades do Rio de Janeiro, e
>publicado recentemente na revista Annals of Allergy
>Asthma and Immunology, mostrou claramente a correlação
>entre os níveis de poluição e a incidência de
>problemas respiratórios. Observou-se um aumento de
>mais de 50% na freqüência de crises de asma nos
>moradores de Duque de Caxias (local com alta
>concentração de poluentes), comparados aos moradores
>de Seropédica (local com baixos níveis de poluição).
>Outro estudo, realizado pelos pesquisadores do
>laboratório de poluição atmosférica da Universidade de
>São Paulo, confirmou os efeitos nocivos da poluição do
>ar na mortalidade infantil. Há aumento de 6% no número
>de óbitos neonatais nas regiões mais poluídas. O
>controle de poluentes de qualquer origem deverá ser
>obrigação mundial. Ainda mais com a entrada em vigor
>do Protocolo de Kyoto.
>
>Mas, de forma geral, provavelmente o maior desafio das
>autoridades de saúde é garantir um apoio ao avanço
>científico na área de saúde, atingir a maioria da
>população com os avanços médicos e tecnológicos
>alcançados, e insistir na prevenção e na saúde
>primária, básica. Muito mais do que dinheiro, em 2005
>precisamos de estratégia, filosofia e enfoque novos.
>
>O acesso da população, principalmente das camadas
>menos privilegiadas da sociedade, a remédios em geral
>é muito limitado. Várias são as causas, como preços
>elevados na fonte, acréscimos excessivos na cadeia de
>distribuição e impostos. Se as autoridades, de todas
>as esferas, não atentarem a essas dificuldades
>enfrentadas no dia-a-dia da maioria dos cidadãos,
>muito mais será gasto para tratar complicações graves
>de doenças crônicas, como diabetes e hipertensão
>arterial. O Brasil iniciou uma ação interessante nessa
>direção: a aquisição da fábrica da GlaxoSmithKline do
>Brasil pelo governo federal. O presidente Lula
>destacou que "pela primeira vez um governo compra uma
>fábrica da iniciativa privada", ao mesmo tempo
>criticando "o processo inverso de privatização do
>governo anterior". Lula declarou, na ocasião, que
>estava "recuperando uma fábrica que seria desativada,
>eliminaria empregos e que agora produzirá, em escala
>nacional, os antibióticos mais usados no Brasil".
>
>Nessa mesma linha o ministro da Saúde, Humberto Costa,
>deixou muito clara sua estratégia para 2005:
>
>­ Vamos ampliar o acesso da população a medicamentos
>fundamentais com base em três opções. A primeira é a
>rede de farmácias populares, e até o fim do ano
>pretendemos inaugurar cem unidades em todo o Brasil. A
>segunda é a criação de um programa para a venda
>subsidiada de medicamentos básicos para hipertensão e
>diabetes, através da rede de farmácias privada.
>Pretendemos baixar os preços em até 50%. E a terceira
>é a redução do ICMS de 2,8 mil medicamentos até 2005.
>
>De acordo com uma pesquisa do IBGE, de 2003, a saúde
>aparece em terceiro lugar no orçamento das famílias
>brasileiras, e os medicamentos representam 61% desses
>gastos para as pessoas de baixa renda. Além disso,
>metade das pessoas que precisam de tratamento não pode
>pagar os remédios de que necessitam.
>
>A partir de 2005, 50 milhões de unidades de
>antibióticos serão produzidas para a rede do Sistema
>Único de Saúde (SUS) e para as Farmácias Populares. Em
>2007, a produção deve quintuplicar em relação à atual.
>Mais de 10 bilhões de unidades de medicamentos devem
>ser produzidas para as principais doenças que mais
>atingem a população brasileira, como hipertensão,
>diabetes, malária e tuberculose. A Fiocruz também
>produzirá mais vacinas, e o Ministério da Saúde
>enfatiza que "a ampliação do acesso da população aos
>medicamentos é uma das prioridades do governo
>federal".
>
>Em 2004, muito se fez pela saúde, mas ainda há muito
>mais por fazer. Programas de prevenção e detecção
>precoce de doenças sexualmente transmissíveis, como
>Aids, HPV (câncer de colo de útero), a melhoria na
>qualidade do atendimento dos pacientes do SUS,
>diminuição das filas, controle de poluição, de
>infecção hospitalar e, também, de mortes por agentes
>externos, como traumas (tiros, facadas, acidentes de
>trânsito). Para se ter uma idéia do custo com os
>acidentes de trânsito, a cada ano acontecem mais de
>100 mil mortes, com três a quatro vezes esse número de
>feridos. E a maioria absoluta é tratada com dinheiro
>público. São milhões de reais de gastos evitáveis. Sem
>falar nas seqüelas individuais.
>

Abraços
Amauri

[As partes desta mensagem que não continham texto foram removidas]



##### ##### #####

Para saber mais visite
http://www.ciencialist.hpg.ig.com.br


##### ##### ##### #####



Yahoo! Grupos, um serviço oferecido por:
PUBLICIDADE




------------------------------------------------------------------------------
Links do Yahoo! Grupos

a.. Para visitar o site do seu grupo na web, acesse:
http://br.groups.yahoo.com/group/ciencialist/

b.. Para sair deste grupo, envie um e-mail para:
ciencialist-unsubscribe@yahoogrupos.com.br

c.. O uso que você faz do Yahoo! Grupos está sujeito aos Termos do Serviço do Yahoo!.



[As partes desta mensagem que não continham texto foram removidas]



##### ##### #####

Para saber mais visite
http://www.ciencialist.hpg.ig.com.br


##### ##### ##### #####


Yahoo! Grupos, um serviço oferecido por:







------------------------------------------------------------------------------
Links do Yahoo! Grupos

a.. Para visitar o site do seu grupo na web, acesse:
http://br.groups.yahoo.com/group/ciencialist/

b.. Para sair deste grupo, envie um e-mail para:
ciencialist-unsubscribe@yahoogrupos.com.br

c.. O uso que você faz do Yahoo! Grupos está sujeito aos Termos do Serviço do Yahoo!.



[As partes desta mensagem que não continham texto foram removidas]



SUBJECT: Re: [ciencialist] Evolucao e desafios - Consumo de Notícias
FROM: "Oraculo" <oraculo@atibaia.com.br>
TO: <ciencialist@yahoogrupos.com.br>
DATE: 03/01/2005 00:50

Olá

Correção, não foi uma inglesa mas uma norte americana:

http://noticias.terra.com.br/ciencia/interna/0,,OI447670-EI298,00.html

Um abraço.

Homero

----- Original Message -----
From: Amauri Jr
To: ciencialist@yahoogrupos.com.br
Sent: Sunday, January 02, 2005 4:11 PM
Subject: Re: [ciencialist] Evolucao e desafios - Consumo de Notícias


L.E.R

"revistas são objetos de consumo."

[A] Não só a revistas como a própria ciência...

"consumidores de notícias são iguais consumidores de pão e salsicha.Uns querem uma boa salsicha e um pão bom.Os outros também querem coisa boa. Ou seja: notícia bom"

[A] Noticia boa era a desvinculação da ciência ao mercado de cosméticos e remédios que duvido serem ineficazes a algumas doenças como o HIV, que sabemos a causa de não haver uma cura. Sustenta um mercado imenso de AZT e outras porcarias que só fazem aumentar o sofrimento, ou o câncer que sustenta o mercado de antibióticos e radioterapias. Criam toda hora desculpas para nós comemos isso ou aquilo, hora faz mal, hora faz bem, como entender? Teorias absurdas e sem utilidade para humanidade para se sustentar, segurar verba, como procurar pelos em baratas, dentes em pernilongos e por ai vai!! Como a ida a Lua...a ciência é refém do capitalismo.

"O cara que compra notícia quer notícia boa."

[A] Lógico, se você compra porcaria ta jogando dinheiro fora.

"Como a revista tem que vender... ela bota na capa as notícias boas.Noticia ruim vem de grátis, principalmente por internet, não precisa comprar não."

[A] Noticias ruins vem de todo lugar, mesmo as revistas ditas boas vem, a internet só veio a acrescentar esse momento Bikmann

Amauri

----- Original Message -----
From: L.E.R.de Carvalho
To: ciencialist@yahoogrupos.com.br
Sent: Sunday, January 02, 2005 3:52 PM
Subject: [ciencialist] Evolucao e desafios - Consumo de Notícias


At 15:18 2/1/2005, you wrote:
>Minhas analises em baixo com [A]....



Amauri:

revistas são objetos de consumo.

consumidores de notícias são iguais consumidores de pão e salsicha.
Uns querem uma boa salsicha e um pão bom.
Os outros também querem coisa boa. Ou seja: notícia bom

O cara que compra notícia quer notícia boa.

Como a revista tem que vender... ela bota na capa as notícias boas.
Noticia ruim vem de gratis, principalmente por internet, não precisa
comprar não.

L.E.


[As partes desta mensagem que não continham texto foram removidas]



##### ##### #####

Para saber mais visite
http://www.ciencialist.hpg.ig.com.br


##### ##### ##### #####


Yahoo! Grupos, um serviço oferecido por:







------------------------------------------------------------------------------
Links do Yahoo! Grupos

a.. Para visitar o site do seu grupo na web, acesse:
http://br.groups.yahoo.com/group/ciencialist/

b.. Para sair deste grupo, envie um e-mail para:
ciencialist-unsubscribe@yahoogrupos.com.br

c.. O uso que você faz do Yahoo! Grupos está sujeito aos Termos do Serviço do Yahoo!.



[As partes desta mensagem que não continham texto foram removidas]



##### ##### #####

Para saber mais visite
http://www.ciencialist.hpg.ig.com.br


##### ##### ##### #####


Yahoo! Grupos, um serviço oferecido por:

São Paulo Rio de Janeiro Curitiba Porto Alegre Belo Horizonte Brasília




------------------------------------------------------------------------------
Links do Yahoo! Grupos

a.. Para visitar o site do seu grupo na web, acesse:
http://br.groups.yahoo.com/group/ciencialist/

b.. Para sair deste grupo, envie um e-mail para:
ciencialist-unsubscribe@yahoogrupos.com.br

c.. O uso que você faz do Yahoo! Grupos está sujeito aos Termos do Serviço do Yahoo!.



[As partes desta mensagem que não continham texto foram removidas]



SUBJECT: Efeito Allais segundo Fran De Aquino
FROM: "marcelomjr" <marcelomjr@yahoo.com.br>
TO: ciencialist@yahoogrupos.com.br
DATE: 03/01/2005 01:48


Solução do Efeito Allais segundo Fran De Aquino

http://users.elo.com.br/~deaquino//Appendix%20D.pdf

Abraço,

Marcelomjr





SUBJECT: A Próxima Guerra
FROM: Rodrigo Marques <rodmarq72@yahoo.com.br>
TO: Ceticismo Aberto <ceticismoaberto@yahoogrupos.com.br>, CienciaList <ciencialist@yahoogrupos.com.br>, Sociedade Brasileira de "C�ticos" e Racionalistas <sbcr@yahoogrupos.com.br>, Sociedade da Terra redonda <strbrasil@yahoogrupos.com.br>
DATE: 03/01/2005 10:23


Pessoal já faz um tempo que eu recebo esse e-mail com essa história a respeito de Roraima que agora repasso para você com a seguinte pergunta: A situação descrita abaixo é verídica pelo menos em parte ou é que nem aquela história falsa de que nos livros escolares americanos o mapa do Brasil já aparecia sem a amazônia e o pantanal? Será que os e-mails das pessoas mostradas no final são reais e se forem elas sabem que estão sendo usados no repasse desta mensagem?


A PRÓXIMA GUERRA

Segue abaixo o relato de uma pessoa conhecida e séria, que passou
recentemente em um concurso público federal e foi trabalhar em Roraima.

Trata-se de um Brasil que a gente não conhece.

As duas semanas em Manaus foram interessantes para conhecer um Brasil um
pouco diferente, mas chegando em Boa Vista (RR) não pude resistir a fazer um
relato das coisas que tenho visto e escutado por aqui.
Conversei com algumas pessoas nesses três dias, desde engenheiros até
pessoas com um mínimo de instrução.
Para começar o mais difícil de encontrar por aqui é roraimense, pra falar a
verdade, acho que a proporção é de um roraimense para cada 10 pessoas é bem
razoável, tem gaúcho, carioca, cearense, amazonense, piauiense, maranhense e
por aí vai. Portanto falta uma identidade com a terra. Aqui não existem
muitos meios de sobrevivência, ou a pessoa é funcionária pública, e aqui
quase todo mundo é, pois em Boa Vista se concentram todos os órgãos federais
e estaduais de Roraima, além da prefeitura é claro. Se não for funcionário
público a pessoa trabalha no comércio local ou recebe ajuda de Programas do
governo. Não existe indústria de qualquer tipo. Pouco mais de 70% do
Território roraimense é demarcado como reserva indígena, portanto restam
apenas 30%, descontando-se os rios e as terras improdutivas que são muitas,
para se cultivar a terra ou para a localização das próprias cidades. (Na
única rodovia que existe em direção ao Brasil (liga Boa Vista a Manaus,
cerca de 800 km) existe um trecho de aproximadamente 200 km reserva indígena
Waimiri Atroari) por onde você só passa entre 6:00 da manhã e 6:00 da tarde,
nas outras 12 horas a rodovia é fechada pelos índios (com autorização da
FUNAI e dos americanos) para que os mesmos não sejam incomodados.

Detalhe:

Você não passa se for brasileiro, o acesso é livre aos americanos, europeus
e japoneses. Desses 70% de território indígena, diria que em 90% dele
ninguém entra sem uma grande burocracia e autorização da FUNAI.

Detalhe:

Americanos entram na hora que quiserem, se você não tem uma autorização da
FUNAI mas tem dos americanos então você pode entrar. A maioria dos índios
fala a língua nativa além do inglês ou francês, mas a maioria não sabe falar
português. Dizem que é comum na entrada de algumas reservas encontrarem-se
hasteadas bandeiras americanas ou inglesas. É comum se encontrar por aqui
americano tipo nerds com cara de quem não quer nada, que veio caçar
borboleta e joaninha e catalogá-las, mas no final das contas pasme, se você
quiser montar um empresa para exportar plantas e frutas típicas como
cupuaçu, açaí camu-camu etc., medicinais, ou componentes naturais para
fabricação de remédios, pode se preparar para pagar "royalties" para
empresas japonesas e americanas que já patentearam a maioria dos produtos
típicos da Amazônia...

Por três vezes repeti a seguinte frase após ouvir tais relatos: É os
americanos vão acabar tomando a Amazônia e em todas elas ouvi a mesma
resposta em palavras diferentes. Vou reproduzir a resposta de uma senhora
simples que vendia suco e água na rodovia próximo de Mucajaí:

"Irão não minha filha, tu não sabe, mas tudo aqui já é deles, eles comandam
tudo, você não entra em lugar nenhum porque eles não deixam. Quando acabar
essa guerra aí eles virão pra cá, e vão fazer o que fizeram no Iraque quando
determinaram uma faixa para os curdos onde iraquiano não entra, aqui vai ser
a mesma coisa".

A dona é bem informada não? O pior é que segundo a ONU o conceito de nação é
um conceito de soberania e as áreas demarcadas têm o nome de nação indígena.
O que pode levar os americanos a alegarem que estarão libertando os povos
indígenas. Fiquei sabendo que os americanos já estão construindo uma grande
base militar na Colômbia, bem próximo da fronteira com o Brasil numa
parceria com o governo colombiano com o pseudo objetivos de combater o
narcotráfico. Por falar em narcotráfico, aqui é rota de distribuição, pois
essa mãe chamada Brasil mantém suas fronteiras abertas e aqui tem Estrada
para as Guianas e Venezuela. Nenhuma bagagem de estrangeiro é fiscalizada,
principalmente se for americano, europeu ou japonês, (isso pode causar um
incidente diplomático)... Dizem que tem muito colombiano traficante virando
venezuelano, pois na Venezuela é muito fácil comprar a cidadania venezuelana
por cerca de 200 dólares.

Pergunto inocentemente às pessoas; porque os americanos querem tanto
proteger os índios. A resposta é absolutamente a mesma, porque as terras
indígenas além das riquezas animais e vegetais, da abundância de água são
extremamente ricas em ouro (encontram-se pepitas que chegam a ser pesadas em
quilos), diamante, outras pedras preciosas, minério e nas reservas norte de
Roraima e Amazonas, ricas em PETRÓLEO.

Parece que as pessoas contam essas coisas como que num grito de Socorro a
alguém que é do sul, como se eu pudesse dizer isso ao presidente ou a alguma
autoridade do sul que vá fazer alguma coisa. É pessoal, saio daqui com a
quase certeza de que em breve o Brasil irá diminuir de tamanho.

Um grande abraço a todos. Será que podemos fazer alguma coisa??? Acho que
sim.

Repasse esse e-mail para que um maior número de brasileiros fique sabendo
desses absurdos.

Mara Silvia Alexandre Costa
Depto de Biologia Cel. Mol. Bioag. Patog.
FMRP - USP

Opinião pessoal:

Gostaria que você, especialmente que recebeu este e-mail, o repasse para o
maior número possível de pessoas. Do meu ponto de vista seria interessante
que o país inteiro ficasse sabendo desta situação através dos telejornais
antes que isso venha a acontecer.
Afinal foi um momento de fraqueza dos Estados Unidos que os europeus
lançaram o Euro, assim poderá se aproveitar esta situação de fraqueza
norte-americana (perdas na guerra do Iraque) para revelar isto ao mundo a
fim de antecipar a próxima guerra.
Conto com sua participação, no envio deste e-mail..

Celso Luiz Borges de Oliveira
Doutorando em Água e Solo
FEAGRI/UNICAMP
Tel: (19) 3233-1840 Celular: (19) 9136-6472

e-mail´s: celso@ufba.br; celso@agr.unicamp.br; celsoborges@gmail.com




__________________________________________________
Converse com seus amigos em tempo real com o Yahoo! Messenger
http://br.download.yahoo.com/messenger/

[As partes desta mensagem que não continham texto foram removidas]



SUBJECT: Novo combustível nuclear: 16 NGF
FROM: José Renato <jrma@terra.com.br>
TO: <ciencialist@yahoogrupos.com.br>
DATE: 03/01/2005 10:41

A TARDE 03/01/2005 - Nacional
Energia
Novo combustível nuclear produz mais energia

AGÊNCIA ESTADO

Rio - Dez pesquisadores brasileiros integraram a equipe de 29 técnicos que desenvolveu em dois anos um novo combustível nuclear que usa menos urânio, aumenta a potência das usinas, produz menos lixo radioativo e é capaz de gerar 30% a mais de energia. O combustível foi desenvolvido por um consórcio entre a Westinghouse, as Indústrias Nucleares do Brasil (INB) e a KNFC, operadora de usinas na Coréia do Sul. O 16 NGF (Next Generation Fuel) começa a ser usado em janeiro na usina Kori 2, uma das 16 da Coréia do Sul. No Brasil, ele vai abastecer Angra 1 a partir de 2008.

O novo combustível começou a ser estudado depois que técnicos da INB apresentaram uma pesquisa no Congresso Brasileiro de Energia, em 2000, com evidências de que, diminuindo-se a distância das varetas no elemento combustível, obtinha-se maior reatividade (o elemento combustível é feito de pequenas pastilhas de urânio empilhadas dentro de varetas de 4,5 metros de altura).

O estudo interessou a Westinghouse, fabricante de usinas, que propôs que ele fosse ampliado e os custos, divididos entre a empresa, a INB e a KNFC. Foram investidos US$ 10,5 milhões. "Duas recargas de Angra 1 com o novo combustível são suficientes para ressarcir o que o Brasil gastou com o estudo", diz o diretor de Produção do Combustível Nuclear da INB, Samuel Fayad.

Os técnicos brasileiros são físicos e engenheiros que estão há menos de cinco anos na INB, em sua maioria. Chefiados pelo engenheiro Roberto Esteves - PhD pela Universidade de Columbia e um dos três gerentes de projeto da pesquisa do novo combustível -, eles passaram dois anos nos EUA, fazendo testes nos laboratórios da Westinghouse. Além de diminuir a distância entre as varetas, eles mudaram a estrutura do elemento combustível.

Esteves explica que, com as modificações, em Angra 1 será possível usar 373 quilos de urânio por elemento combustível em vez dos 411 necessários hoje. Outra vantagem é que o 16 NGF tem vida útil maior, o que reduz as paradas dos reatores para trocas. A usina brasileira, que hoje tem capacidade para abastecer 300 mil resistências, fornecerá energia para 320 mil famílias.

"Esse elemento combustível só pode ser usado em três usinas gêmeas no mundo (Angra 1, Kori 2 e Krisko, na Eslovênia). É um projeto pequeno, mas o projeto nuclear brasileiro também é pequeno e atende às nossas necessidades", disse Fayad. De acordo com ele, não é possível dizer que a redução da quantidade de urânio fará cair o valor do combustível. "A cotação do urânio está em alta no mundo, mas gastaremos menos dinheiro para produzir o elemento combustível."

O acordo inicial previa que Kori 2 e Angra 1 começariam a operar com o 16 NGF ao mesmo tempo. Mas os trabalhos foram atrasados no Brasil porque a Eletronuclear, operadora das usinas, fará a troca do gerador de vapor de Angra 1 em 2007. Só então a usina será abastecida com o novo combustível.


< http://www.atarde.com.br/materia.php3?id_materia=259&ano=2005&mes=01&id_subcanal=19 >

[As partes desta mensagem que não continham texto foram removidas]



SUBJECT: Re: A Próxima Guerra
FROM: Maria Natália <grasdic@hotmail.com>
TO: ciencialist@yahoogrupos.com.br
DATE: 03/01/2005 11:11


Rodrigo:
Te pergunto ou a quem saiba o que é um "doutorado em água e solo"?
Também gostava de saber, pois deve haver pessoal aqui na área que
estuda ou vive perto dessa universidade, quantos anos de estudo terá
o autor do texto.
Existe em vosso país uma ordem dos engenheiros que reconhece
engenharias? Pode qualquer universidade criar curso de engenharia
mas pode não ter saída porque a Ordem não o creditou. Tamos cá
desses cursos mas os alunos são avisados que correm risco de depois
a ordem não os reconhecer. No fundo é como ser médico português ao
chegar a França, por exemplo, tem de ser reconhecido pela respectiva
Ordem desse país.
Quanto à guerra não conheço para poder saber se é verdade e depois
quem mora perto que nos diga algo. Também se estão em férias podem
ir lá passear e conhecer vosso pais. Outra hipótese é pessoal de
geografia fazer mestrado com tese sobre essa região do Brasil. É um
modo de tirar a prova dos nove.
E como para grandes males grandes remédios até à ONU se pode ir.
Também pode ser Hoax ou indivíduo em causa já andar pedrado.
Um abrço
Maria Natália
--- Em ciencialist@yahoogrupos.com.br, Rodrigo Marques
<rodmarq72@y...> escreveu
>
> Pessoal já faz um tempo que eu recebo esse e-mail com essa
história a respeito de Roraima que agora repasso para você com a
seguinte pergunta: A situação descrita abaixo é verídica pelo menos
em parte ou é que nem aquela história falsa de que nos livros
escolares americanos o mapa do Brasil já aparecia sem a amazônia e o
pantanal? Será que os e-mails das pessoas mostradas no final são
reais e se forem elas sabem que estão sendo usados no repasse desta
mensagem?
>
>
> A PRÓXIMA GUERRA
>
> Segue abaixo o relato de uma pessoa conhecida e séria, que passou
> recentemente em um concurso público federal e foi trabalhar em
Roraima.
>
> Trata-se de um Brasil que a gente não conhece.
>
> As duas semanas em Manaus foram interessantes para conhecer um
Brasil um
> pouco diferente, mas chegando em Boa Vista (RR) não pude resistir
a fazer um
> relato das coisas que tenho visto e escutado por aqui.
> Conversei com algumas pessoas nesses três dias, desde engenheiros
até
>





SUBJECT: Re: [ciencialist] A Próxima Guerra
FROM: "Alvaro Augusto - Electra" <alvaro@electraenergy.com.br>
TO: <ciencialist@yahoogrupos.com.br>
DATE: 03/01/2005 11:19

Se os e-mails são verdadeiros? Se as pessoas sabem que seus nomes estão sendo usados? Basta escrever para eles e perguntar!

[ ]s

Alvaro Augusto

----- Original Message -----
From: Rodrigo Marques
To: Ceticismo Aberto ; CienciaList ; Sociedade Brasileira de Céticos e Racionalistas ; Sociedade da Terra redonda
Sent: Monday, January 03, 2005 10:23 AM
Subject: [ciencialist] A Próxima Guerra



Pessoal já faz um tempo que eu recebo esse e-mail com essa história a respeito de Roraima que agora repasso para você com a seguinte pergunta: A situação descrita abaixo é verídica pelo menos em parte ou é que nem aquela história falsa de que nos livros escolares americanos o mapa do Brasil já aparecia sem a amazônia e o pantanal? Será que os e-mails das pessoas mostradas no final são reais e se forem elas sabem que estão sendo usados no repasse desta mensagem?


A PRÓXIMA GUERRA

Segue abaixo o relato de uma pessoa conhecida e séria, que passou
recentemente em um concurso público federal e foi trabalhar em Roraima.

Trata-se de um Brasil que a gente não conhece.

As duas semanas em Manaus foram interessantes para conhecer um Brasil um
pouco diferente, mas chegando em Boa Vista (RR) não pude resistir a fazer um
relato das coisas que tenho visto e escutado por aqui.
Conversei com algumas pessoas nesses três dias, desde engenheiros até
pessoas com um mínimo de instrução.
Para começar o mais difícil de encontrar por aqui é roraimense, pra falar a
verdade, acho que a proporção é de um roraimense para cada 10 pessoas é bem
razoável, tem gaúcho, carioca, cearense, amazonense, piauiense, maranhense e
por aí vai. Portanto falta uma identidade com a terra. Aqui não existem
muitos meios de sobrevivência, ou a pessoa é funcionária pública, e aqui
quase todo mundo é, pois em Boa Vista se concentram todos os órgãos federais
e estaduais de Roraima, além da prefeitura é claro. Se não for funcionário
público a pessoa trabalha no comércio local ou recebe ajuda de Programas do
governo. Não existe indústria de qualquer tipo. Pouco mais de 70% do
Território roraimense é demarcado como reserva indígena, portanto restam
apenas 30%, descontando-se os rios e as terras improdutivas que são muitas,
para se cultivar a terra ou para a localização das próprias cidades. (Na
única rodovia que existe em direção ao Brasil (liga Boa Vista a Manaus,
cerca de 800 km) existe um trecho de aproximadamente 200 km reserva indígena
Waimiri Atroari) por onde você só passa entre 6:00 da manhã e 6:00 da tarde,
nas outras 12 horas a rodovia é fechada pelos índios (com autorização da
FUNAI e dos americanos) para que os mesmos não sejam incomodados.

Detalhe:

Você não passa se for brasileiro, o acesso é livre aos americanos, europeus
e japoneses. Desses 70% de território indígena, diria que em 90% dele
ninguém entra sem uma grande burocracia e autorização da FUNAI.

Detalhe:

Americanos entram na hora que quiserem, se você não tem uma autorização da
FUNAI mas tem dos americanos então você pode entrar. A maioria dos índios
fala a língua nativa além do inglês ou francês, mas a maioria não sabe falar
português. Dizem que é comum na entrada de algumas reservas encontrarem-se
hasteadas bandeiras americanas ou inglesas. É comum se encontrar por aqui
americano tipo nerds com cara de quem não quer nada, que veio caçar
borboleta e joaninha e catalogá-las, mas no final das contas pasme, se você
quiser montar um empresa para exportar plantas e frutas típicas como
cupuaçu, açaí camu-camu etc., medicinais, ou componentes naturais para
fabricação de remédios, pode se preparar para pagar "royalties" para
empresas japonesas e americanas que já patentearam a maioria dos produtos
típicos da Amazônia...

Por três vezes repeti a seguinte frase após ouvir tais relatos: É os
americanos vão acabar tomando a Amazônia e em todas elas ouvi a mesma
resposta em palavras diferentes. Vou reproduzir a resposta de uma senhora
simples que vendia suco e água na rodovia próximo de Mucajaí:

"Irão não minha filha, tu não sabe, mas tudo aqui já é deles, eles comandam
tudo, você não entra em lugar nenhum porque eles não deixam. Quando acabar
essa guerra aí eles virão pra cá, e vão fazer o que fizeram no Iraque quando
determinaram uma faixa para os curdos onde iraquiano não entra, aqui vai ser
a mesma coisa".

A dona é bem informada não? O pior é que segundo a ONU o conceito de nação é
um conceito de soberania e as áreas demarcadas têm o nome de nação indígena.
O que pode levar os americanos a alegarem que estarão libertando os povos
indígenas. Fiquei sabendo que os americanos já estão construindo uma grande
base militar na Colômbia, bem próximo da fronteira com o Brasil numa
parceria com o governo colombiano com o pseudo objetivos de combater o
narcotráfico. Por falar em narcotráfico, aqui é rota de distribuição, pois
essa mãe chamada Brasil mantém suas fronteiras abertas e aqui tem Estrada
para as Guianas e Venezuela. Nenhuma bagagem de estrangeiro é fiscalizada,
principalmente se for americano, europeu ou japonês, (isso pode causar um
incidente diplomático)... Dizem que tem muito colombiano traficante virando
venezuelano, pois na Venezuela é muito fácil comprar a cidadania venezuelana
por cerca de 200 dólares.

Pergunto inocentemente às pessoas; porque os americanos querem tanto
proteger os índios. A resposta é absolutamente a mesma, porque as terras
indígenas além das riquezas animais e vegetais, da abundância de água são
extremamente ricas em ouro (encontram-se pepitas que chegam a ser pesadas em
quilos), diamante, outras pedras preciosas, minério e nas reservas norte de
Roraima e Amazonas, ricas em PETRÓLEO.

Parece que as pessoas contam essas coisas como que num grito de Socorro a
alguém que é do sul, como se eu pudesse dizer isso ao presidente ou a alguma
autoridade do sul que vá fazer alguma coisa. É pessoal, saio daqui com a
quase certeza de que em breve o Brasil irá diminuir de tamanho.

Um grande abraço a todos. Será que podemos fazer alguma coisa??? Acho que
sim.

Repasse esse e-mail para que um maior número de brasileiros fique sabendo
desses absurdos.

Mara Silvia Alexandre Costa
Depto de Biologia Cel. Mol. Bioag. Patog.
FMRP - USP

Opinião pessoal:

Gostaria que você, especialmente que recebeu este e-mail, o repasse para o
maior número possível de pessoas. Do meu ponto de vista seria interessante
que o país inteiro ficasse sabendo desta situação através dos telejornais
antes que isso venha a acontecer.
Afinal foi um momento de fraqueza dos Estados Unidos que os europeus
lançaram o Euro, assim poderá se aproveitar esta situação de fraqueza
norte-americana (perdas na guerra do Iraque) para revelar isto ao mundo a
fim de antecipar a próxima guerra.
Conto com sua participação, no envio deste e-mail..

Celso Luiz Borges de Oliveira
Doutorando em Água e Solo
FEAGRI/UNICAMP
Tel: (19) 3233-1840 Celular: (19) 9136-6472

e-mail´s: celso@ufba.br; celso@agr.unicamp.br; celsoborges@gmail.com




__________________________________________________
Converse com seus amigos em tempo real com o Yahoo! Messenger
http://br.download.yahoo.com/messenger/

[As partes desta mensagem que não continham texto foram removidas]



##### ##### #####

Para saber mais visite
http://www.ciencialist.hpg.ig.com.br


##### ##### ##### #####


Yahoo! Grupos, um serviço oferecido por:







------------------------------------------------------------------------------
Links do Yahoo! Grupos

a.. Para visitar o site do seu grupo na web, acesse:
http://br.groups.yahoo.com/group/ciencialist/

b.. Para sair deste grupo, envie um e-mail para:
ciencialist-unsubscribe@yahoogrupos.com.br

c.. O uso que você faz do Yahoo! Grupos está sujeito aos Termos do Serviço do Yahoo!.



[As partes desta mensagem que não continham texto foram removidas]



SUBJECT: Evolucao e desafios -Homero
FROM: "Amauri Jr" <amaurijunior2@yahoo.com.br>
TO: <ciencialist@yahoogrupos.com.br>
DATE: 03/01/2005 11:45

Olá Amury


[A] Olá Homero






"Minhas análises sobre suas análises..:-)"



[A] Sinceramente gosto de suas analises, fico satisfeito de você me responder...:-)





"Você parece estar ressentido com o que chama de "ciência" e a trata como entidade independente, crença de cientistas ou algo a ser adorado ou derrubado. Mas nada disso é ciência, nem o resultados de sua aplicação se confundem com ela. Ciência neste contexto (desta lista e do uso padrão do termo..:-) é apenas o conjunto dos conhecimentos obtidos através de um método padronizado, chamado cientifico, e do rigor derivado do mesmo. E, independente do que pense dela, tem elevada confiabilidade..:-)"



[A] Amigo, o que penso que infelizmente o homem usa seu conhecimento para algo não muito para o "bem" humano ou "mal", a natureza humana já é corrupta. Tudo que é humano tem falhas, pois nada ainda é perfeito, mas a ciências como algo que vem para beneficiar a humanidade tinha que ser impartidaria e sem nenhuma crença ou fanatismo; como vimos muito na lista. Por isso, nada tenho contra ou a favor da ciência, apenas é uma visão critica sobre...:-)



"Não importa se o resultado é passível de ser julgado subjetivamente como "mau" ou mesmo como "bom". Importa apenas que é mais eficiente que outras formas de conhecimento, mais eficaz em suas previsões e mais confiável que outros instrumentos de compreensão do universo já criado por seres humanos. E que será abandonado assim que uma ferramenta mais eficaz seja apresentada..:-) Até lá, é o que temos de melhor."



[A] O que temos de melhor, como disse, esta nas mãos dos poderosos, portanto "o que temos de melhor" foi e esta sendo mandado pelo governo dos poderosos. Hoje os norte-americanos mandam e desmandam no que "temos de melhor", tanto se é para o bem da humanidade é imoral, mas se beneficia as partes bélicas deles, seguem numa boa...:-)





"Podemos analisar filosoficamente, subjetivamente, os resultados, mas não discutir a eficácia. A ciência, o conhecimento assim acumulado, é tão eficiente quando cura doenças, como a erradicação da malária, como quando destrói milhões de vidas, como em Hiroshima. O resultado é passível de julgamento, mas a ação não. Ela é eficaz, nos dois casos. Se precisar salvar alguém ou matar alguém, deve usar a ciência, é mais eficaz sempre..:-)"



[A] Podemos sim, tanto que você já disse que ciência é um conjunto de conhecimentos; mas muitos conhecimentos, pois o que vimos, é um "Clube da Ciência" que vê apenas um dos vários lados do universo, mas temos vários e é ainda um incógnito pelo homem ainda não abrir a cabeça...:-) Agora, se erradicamos a malaria não sei, creio que você não esta lendo jornal, pois a malaria, a febre-amarela, o HIV, o câncer, então matando e a ciência "tudo que temos de melhor" fica ai procurando pelos em baratas. Hiroshima foi um ato terrorista covarde e deprimente, que deveria fazer um julgamento dos cientistas e dos governantes no tribunal de Nuremberg, por danos morais humanos.





"Não gostar dela, ou como você pretende com as análises, critica-la ou despreza-la, não importa em nada para sua eficácia e confiabilidade. Você vai viver mais que todos os seus antepassados, goste ou não..:-) Sua expectativa de vida é de 75 anos, e, não sei sua idade, mas se já passou dos 40 como eu, é mais que seu avô esperava viver ao nascer (em 1900 a expectativa de vida no Brasil era de 33 anos)."





[A] Tenho apenas 28 anos, sem filhos e com uma deficiência física, o que sei que são superfluidades da senhora sagrada ciência. O que pode me interessar eu viver 40 ou 70 anos se nem ela pode prever?





"Na verdade, você usou um computador e a Internet, frutos da ciência, para enviar seus pensamentos e ataques a ciência, ao inves de tentar telepatia, rezas ou mandingas (ou qualquer outra forma de comunicação não cientifica) justamente porque é mais eficaz e confiável que qualquer outra..:-)"



[A] Alguma coisa boa tinha que sair daí né?...:-) o problema é esse, vocês da ciência só tem um foco do assunto, não ve as múltiplas variedades de estudos...:)





"Você não parece gostar de muitos dos aspectso do uso do conhecimento cientifico. Direito seu. E pode até mesmo abandona-los, todos, e ir viver em uma aldeia remota no Amazonas, sem (quase) nenhum contato com a ciência (os índios são muito receptivos a forasteiros, vão gostar de recebe-lo), e com todo o ônus de viver dessa forma (filhos mortos, vida curta, doenças diversas, poucos dentes, etc). Mas está na verdade criticando comportamentos humanos, que são seres falhos como sabe, não a ciência. Esta, "estricto senso", não é boa ou má, cruel ou gentil, apenas eficiente, confiável em seus efeitos."



[A] Não tenho nada contra a ciência, mas você tem contra os índios...:) o que vejo é ainda crianças mortas e vidas curtas por uma ciência ineficaz, se eu gosto ou não é outra historia, que se eu não gosta-se da ciência não estaria numa lista de ciências...:) e pouco irei a uma tribo, porque não iria acostumar...:)e uma pergunta: a ciência tem vida própria?



"Você não "acredita" nela. Engraçado, ela não pede que se acredite em nada.:-) Deve duvidar sempre, até que evidencias se mostrem sólidas o bastante para uma conclusão. E é nesse ponto que você escorrega..:-) O pouso na Lua já tem evidencias suficientes para essa conclusão, em que pese as tolices de diversos malucos mundo afora e suas teorias de conspiração. O HIV tem recuado e matado muito menos que no inicio da epidemia, graças a AZT e os atuais coquetéis antivirais. Se ajustes são feitos a toda hora, e são, sobre o que comer ou não, é apenas porque a ciência não pretende ser "A VERDADE" como religiões, mas um conhecimento que cresce, se aprofunda, se ajusta e melhora. Sabemos mais, não menos sobre o que comer ou não comer."



[A] Engraçado que pessoas inteligentes não duvidem que os yanques tenham forjado tudo, mas como você mesmo disse, é um direito seu. Eu acredito muito mais que se possa acreditar, a ciência melhorou minha vida muito, tendo esse computador; em minha cadeira de rodas de alumínio (de ferro era um lastima andar), portanto não estou desacreditando a ciência. Agora se quer achar que os coquetéis são eficazes tudo bem, mas que morre mesmo assim, isso morre...:)



"Sim, dá trabalho, é preciso atenção e constante leitura para acompanhar, seria melhor ter uma única resposta para todas as perguntas e pronto, sem risco e sem mudanças. Mas o mundo não funciona assim..:-) Sabemos hoje mais do que ontem sobre como se alimentar, e saberemos mais amanhã, deixando parte do que sabemos hoje para trás. (Parte, não tudo:-)"



[A] Como se o mesmo alimento hoje esta com proteínas e tudo que precisamos e amanha não?





"As teorias conspiratórias são atrativas.:-) É como se nós, os que estão "por dentro", soubéssemos de "coisas" que o resto do mundo não sabe, fossemos mais "espertos" ou mais inteligentes, que não se deixam enganar. Mas, na maioria das vezes (eu diria em sua totalidade) é apenas o ego tentando ser mais do que é.:-) Mesmo com centenas de sites "desmistificando" o pouso na Lua, o volume de evidencias a favor é gigantesco e nenhum deles explica um ponto fundamental do problema: como o governo americano fraudou os sinais de radio e TV que eram emitidos de todas as naves Apollo tanto durante as viagens, como do solo lunar, captados por todo radioamador ao redor do globo, sem falar nos radares e antenas dos paises em disputassem os USA? (lembre-se que, mesmo hoje, não se pode fraudar a origem de um sinal de radio..:-)"



[A] Não acredito nesses doentes mentais que ficam montando sites para se promover...:) eu tiro dos fatos que ocorreram que podem ser completamente forjados, como as transmissões, ou não sabe que eles são donos do que "temos de melhor"...:)



"Enfim, a ciência é eficaz. Embora seu uso, o uso do conhecimento confiável por ela produzido, possa ser discutido e julgado, isso não é a ciência nem com ela se confunde. E, tenha certeza, seu automóvel vai conduzi-lo amanhã, confiavelmente, conforme as leis da fisica descobertas pela ciência, seu computador receberá esta mensagem, os remédios na farmacia impedirão que você morra, as vacinas que tomou evitarão que adoeça (como todos os seus ancestrais adoeceram) e os satelites em órbita, colocados lá pelo conhecimento cientifico, vão transmitir suas chamadas telefonicas até para o Japão, se você assim desejar..:-)"



[A] Reverencia a ciência como um crente reverencia uma reza...:) o meu carro, vai bater conforma a lei da física, as farmácias venderão remédios que viciam, os satélites vão espionar minha casa e tudo graças a deusa infalível ciência...:)



"Para completar a mensagem anterior: uma menina inglesa de 15 aos foi a primeira pessoa a sobreviver depois de contrair raiva sem tomar a vacina antirabica. Ela foi submetida a um novo tratamento revolucionário (e científico), com um coquetel de drogas e coma induzido."



[A] É? Legal! Mas não são os igreses que usam a ciência para fazer guerrinhas com o Bush?





"Diferente de milagres divinos e sobrenaturais, o que se aprendeu com a nova técnica pode ser usado em outras pessoas doentes e não afetam apenas a pessoa que recebe a graça..:-)"



[A] Também acho, devo adverti-lo novamente que não sou a favor no milagre...:)




"A partir de agora, mesmo desprezando a "ciência", você pode ficar sossegado, se você ou um de seus filhos ou entes queridos contrair raiva, poderá ser curado, cientificamente curado.:-) A não ser que tenha resolvido ir viver em uma aldeia indigena na remota Amazonia..."



[A] Sim vão...vão tomar coca-cola e comer hambúrguer feitos cientificamente e se eu tiver sorte, vão se viciar em LSD feito pela ciência, vão tomar energéticos feito pela ciência, vão ter colesterol de produtos que o "temos de melhor" analisa. O mundo é um paraíso, graças a ciência não acha? ...:)



Abraços

Amauri


---- Original Message -----
From: Oraculo
To: ciencialist@yahoogrupos.com.br
Sent: Sunday, January 02, 2005 11:05 PM
Subject: Re: [ciencialist] Evolucao e desafios - CartaCapital - 02/01/05


Olá Amury

Minhas análises sobre suas análises..:-)

Você parece estar ressentido com o que chama de "ciência" e a trata como entidade independente, crença de cientistas ou algo a ser adorado ou derrubado. Mas nada disso é ciência, nem o resultados de sua aplicação se confundem com ela. Ciência neste contexto (desta lista e do uso padrão do termo..:-) é apenas o conjunto dos conhecimentos obtidos através de um método padronizado, chamado cientifico, e do rigor derivado do mesmo. E, independente do que pense dela, tem elevada confiabilidade..:-)

Não importa se o resultado é passível de ser julgado subjetivametne como "mau" ou mesmo como "bom". Importa apenas que é mais eficiente que outras formas de conhecimento, mais eficaz em suas previsões e mais confiável que outros instrumentos de compreensão do universo já criado por seres humanos. E que será abandonado assim que uma ferramenta mais eficaz seja apresentada..:-) Até lá, é o que temos de melhor.

Podemos analisar filosoficamente, subjetivamente, os resultados, mas não discutir a eficácia. A ciência, o conhecimento assim acumulado, é tão eficiente quando cura doenças, como a erradicação da malária, como quando destroi milhões de vidas, como em Hiroshima. O resultado é passível de julgamento, mas a ação não. Ela é eficaz, nos dois casos. Se precisar salvar alguém ou matar alguém, deve usar a ciência, é mais eficáz sempre..:-)

Não gostar dela, ou como você pretende com as análises, critica-la ou despreza-la, não importa em nada para sua eficácia e confiabilidade. Você vai viver mais que todos os seus antepassados, goste ou não..:-) Sua expectativa de vida é de 75 anos, e, não sei sua idade, mas se já passou dos 40 como eu, é mais que seu avô esperava viver ao nascer (em 1900 a expectativa de vida no Brasil era de 33 anos).

Na verdade, você usou um computador e a Internet, frutos da ciência, para enviar seus pensamentos e ataques a ciência, ao inves de tentar telepatia, rezas ou mandingas (ou qualquer outra forma de comunicação não cientifica) justamente porque é mais eficaz e confiável que qualquer outra..:-)

Você não parece gostar de muitos dos aspectso do uso do conhecimento cientifico. Direito seu. E pode até mesmo abandona-los, todos, e ir viver em uma aldeia remota no Amazonas, sem (quase) nenhum contato com a ciência (os indios são muito receptivos a forasteiros, vão gostar de recebe-lo), e com todo o onus de viver dessa forma (filhos mortos, vida curta, doenças diversas, poucos dentes, etc). Mas está na verdade criticando comportamentos humanos, que são seres falhos como sabe, não a ciência. Esta, "estricto senso", não é boa ou má, cruel ou gentil, apenas eficiente, confiável em seus efeitos.

Você não "acredita" nela. Engraçado, ela não pede que se acredite em nada.:-) Deve duvidar sempre, até que evidencias se mostrem sólidas o bastante para uma conclusão. E é nesse ponto que você escorrega..:-) O pouso na Lua já tem evidencias suficientes para essa conclusão, em que pese as tolices de diversos malucos mundo afora e suas teorias de conspiração. O HIV tem recuado e matado muito menos que no inicio da epidemia, graças a AZT e os atuais coqueteis antivirais. Se ajustes são feitos a toda hora, e são, sobre o que comer ou não, é apenas porque a ciência não pretende ser "A VERDADE" como religiões, mas um conhecimento que cresce, se aprofunda, se ajusta e melhora. Sabemos mais, não menos sobre o que comer ou não comer.

Sim, dá trabalho, é preciso atenção e constante leitura para acompanhar, seria melhor ter uma única resposta para todas as perguntas e pronto, sem risco e sem mudanças. Mas o mundo não funciona assim..:-) Sabemos hoje mais do que ontem sobre como se alimentar, e saberemos mais amanhã, deixando parte do que sabemos hoje para tras. Parte, não tudo:-)

As teorias conspiratórias são atrativas.:-) É como se nós, os que estão "por dentro", soubessemos de "coisas" que o resto do mundo não sabe, fossemos mais "espertos" ou mais inteligentes, que não se deixam enganar. Mas, na maioria das vezes (eu diria em sua totalidade) é apenas o ego tentando ser mais do que é.:-) Mesmo com centenas de sites "desmistificando" o pouso na Lua, o volume de evidencias a favor é gigantesco e nenhum deles explica um ponto fundamental do problema: como o governo americano fraudou os sinais de radio e TV que eram emitidos de todas as naves Apollo tanto durante as viagens, como do solo lunar, captados por todo radio-amador ao redor do globo, sem falar nos radares e antenas dos paises em disputacom os USA? (lembre-se que, mesmo hoje, não se pode fraudar a origem de um sinal de radio..:-)

Enfim, a ciência é eficaz. Embora seu uso, o uso do conhecimento confiável por ela produzido, possa ser discutido e julgado, isso não é a ciência nem com ela se confunde. E, tenha certeza, seu automóvel vai conduzi-lo amanhã, confiavelmente, conforme as leis da fisica descobertas pela ciência, seu computador receberá esta mensagem, os remédios na farmacia impedirão que você morra, as vacinas que tomou evitarão que adoeça (como todos os seus ancestrais adoeceram) e os satelites em órbita, colocados lá pelo conhecimento cientifico, vão transmitir suas chamadas telefonicas até para o Japão, se você assim desejar..:-)

Um abraço.

Homero




----- Original Message -----
From: Amauri Jr
To: ciencialist@yahoogrupos.com.br
Sent: Sunday, January 02, 2005 3:18 PM
Subject: Re: [ciencialist] Evolucao e desafios - CartaCapital - 02/01/05


Minhas analises em baixo com [A]....
----- Original Message -----
From: L.E.R.de Carvalho
To: ciencialist@yahoogrupos.com.br
Sent: Sunday, January 02, 2005 1:52 PM
Subject: [ciencialist] Evolucao e desafios - CartaCapital - 02/01/05



>LabConsS - www.ufrj.br/consumo
>
>
>
> EVOLUÇÃO E DESAFIOS
>
>
> O País viu inúmeros avanços nas áreas médica e
>tecnológica, mas as carências sociais ainda são sua
>pior mazela
>
>Em uma livraria, olho rapidamente as manchetes das
>revistas expostas. A julgar pelas capas, é pouco
>provável que tenhamos problemas de saúde deste mês em
>diante. Soluções curativas. Remédios extremamente
>eficientes. Vida prolongada. Corpos e rostos sem um
>defeito. Independentemente da idade, claro. Câncer?
>Uma brincadeira. Paralisia? Pode preparar as pistas.
>Infarto? E daí?

[A] temos que pagar e muito caro para a "ciencia" nos curar de males que diz a "ciencia" um direito de todos.

>
>Deixando de lado uma razoável dose de exagero nessas
>manchetes, encho o peito de orgulho pelas proezas
>atingidas no campo da medicina e da saúde. Vinte anos
>atrás, tudo isso não passaria de sonho, e não
>existiria fora da cabeça imaginativa dos produtores de
>filmes de ficção científica. Mas, ainda hoje, para a
>esmagadora maioria dos cidadãos brasileiros, todo o
>acima mencionado não passa de um sonho, de uma ficção
>científica

{A] Parece que todo a ciencia é uma ficção cientifica, vamos dizer que o pensamento é um comercio hoje em dia, ficçao ou não, é com certeza sensacionalismo.

>Este ano que acaba nos deu boas notícias,
>encorajadoras, e notícias ruins, no mínimo
>preocupantes. Vimos uma senhora que sofreu um derrame
>cerebral voltar a andar com terapia baseada na
>introdução de células-tronco (células primitivas
>capazes de se transformar em praticamente qualquer
>outra célula normal) no cérebro afetado. Emocionante
>perceber o que esses pequenos passos podem significar
>para milhões de pessoas que não conseguem erguer o
>braço, ou mexer a perna. Parafraseando Neil Armstrong,
>um passo gigante para a humanidade.

{A} Armstrong, quem é ele? Aquele que "supostamente" foi a Lua? Celulas- tronco é um desafio para a humanidade de cura, o desejo de ajudar seu semelhante. Pensamentos inovadores tem estado junto a humanidade a muito tempo, isso pode ser descrito na historia de Tales de Mileto até Einstein




>A técnica das células-tronco não se restringe a
>restaurar a função de células cerebrais. Hoje em dia,
>pesquisadores brasileiros, e em outros países,
>intensificam seus esforços para melhorar o coração
>depois de infarto, os nervos após lesão traumática, a
>pele após queimadura. A lista parece não ter fim.

[A] A demora faz o crescimento dos beneficiados...
>
>Vimos a introdução de remédios geniais na prática
>médica. Geniais na sua concepção, e geniais na sua
>eficiência. Drogas que conseguem agir em um ponto
>específico da célula doente, da célula cancerosa,
>dificultando seu desenvolvimento, seu crescimento, e
>até provocando sua morte. Exemplos que estão
>progressivamente sendo utilizados na prática médica,
>apesar de somente em casos muito selecionados, não
>faltam. Glivec, Iressa, são alguns deles.

[A] Santa ciencia!! Mata e de forma covarde porque usa seus conhecimentos de forma de interesse, mesmo que sabemos, esses conhecimentos são para todos.
>
>Ainda nem acabou o ano e ouvimos há poucas semanas o
>anúncio pela GlaxoSmithKline de uma vacina contra o
>câncer. O Brasil está totalmente empenhado na produção
>da vacina contra o HPV, vírus causador de câncer de
>útero. O prof. dr. Ricardo Brentani, presidente da
>Fundação Antonio Prudente, Hospital do Câncer AC
>Camargo, está entusiasmado e declara:
>
>­ Felizmente nossa instituição foi parceira da Merck
>Sharp & Dohme no desenvolvimento de uma vacina contra
>o HPV. Minha esperança é que em 20 anos tenhamos
>prevenido 7% dos tumores humanos.

[A] Vacina? Como pode ter vacina para nossas proprias celulas? E do HIV vai ter?

>Vimos, por outro lado, a preocupação crescente dos
>cientistas em não excluir, a priori, tratamentos
>considerados até então não-convencionais, ou
>complementares. Estudos sérios avaliaram desde a
>acupuntura até a homeopatia, passando por terapias de
>Florais de Bach e tratamentos com vitaminas nas mais
>diversas situações clínicas. Vantagens e desvantagens
>de cada abordagem foram dissecadas minuciosamente.

[A] Viu? Sera que alguem aqui duvidda?


>Cientistas conseguiram demonstrar a eficiência
>incontestável de algumas terapias alternativas em
>certas situações, como a massagem para um dos males do
>século, a dor nas costas. Por outro lado, alertaram
>para a ineficiência e até os efeitos nocivos de outras
>terapias, em outras condições. Por exemplo, para
>tratar bronquite e asma a acupuntura não parece ter
>efeitos importantes. Pelo menos não conseguiram
>detectar esses efeitos nos estudos atuais.

[A] Remedios sim? Eu tive começo, eu e meus manos, de bronquite depois que meu pai comprou um tartaruga nunca mais...santa crença ne? Mas como explicar??

>Vimos o lançamento de aparelhos ultramodernos,
>ultra-sensíveis, para detectar doenças e tratá-las.
>Não consigo perceber avanço recente maior do que na
>área da radioterapia. É incrível a precisão dos feixes
>de radiação em atingir o alvo, no caso o câncer, e
>poupar o tecido normal adjacente. Aparelhos novos
>conseguem acompanhar o movimento do corpo para seguir
>o alvo predeterminado, segundo a segundo, obedecendo à
>orientação do médico radioterapeuta. Reduziram-se
>muito os efeitos colaterais. O controle do câncer
>assemelha-se às extensas cirurgias. A nanotecnologia
>(aparelhos miniaturas) é uma febre, e os estudos
>multiplicam-se para definir com mais clareza sua
>aplicação.

{A} Pode matar com sua radioatividade, causando cancer.

>
>A plástica está fazendo tamanhos milagres que nem os
>próprios pacientes conseguem acreditar. Tanto faz quem
>foram seus pais, ou a etnia a qual você pertença.
>Escolha o modelo e ficará parecido. Para pessoas
>ansiosas com a imagem, soluções para quase tudo. Sem
>dúvida, o impacto sobre o estado emocional é notável.

[A] Eita faquinha de dois cumes heim? A plastica pode curar a pessoa de queimaduras ou cicatrizes, mas tambem pode ressaltar sua vaidade.

>
>Ao lado dessas notícias que mereceram um destaque
>quase obsessivo nas manchetes de capa, outras
>informações com menos, digamos, glamour, não
>conseguiram espaço nem nas páginas finais das
>revistas.
[A] A ciencia ja tem a imprensa e seus colaborados para dar esse glamour...


>
>O objetivo de reduzir em 60% a mortalidade infantil no
>mundo (e o Brasil ainda é um grande protagonista dessa
>estatística) não será atingido em 2015, como
>estabelecido nos Objetivos Milenares da ONU. No
>planeta, morrem por ano mais de 11 milhões de crianças
>com idade inferior a 5 anos. A maioria por doenças
>evitáveis. Diarréia, pneumonia, malária. No mesmo
>período, 500 mil mulheres morrem durante a gravidez ou
>o parto. Doença de Chagas e esquistossomose
>(barriga-d'água) continuam afetando milhares de
>brasileiros. E seu controle está cada vez mais
>próximo. A implementação dos programas já em ação
>poderá melhorar ainda mais esse controle. Para 2005 a
>intensificação dessas abordagens poderá elevar o
>impacto na saúde da população, principalmente nas
>áreas rurais.

[A] Isso ai, como fica doenças tao mais antiga diante do deusa peerfeita ciencia??


>O Relatório Mundial de Saúde (The World Health Report
>2003) recomendou às autoridades o fortalecimento dos
>sistemas de saúde, centralizando seu foco na atenção
>primária, além de integrar a prevenção das doenças e a
>promoção da saúde em todos os níveis de atendimento.

[A] Esses relatorios tem efeito??

>
>A saúde do homem sofreu mudanças drásticas nos últimos
>anos. A expectativa de vida, de brasileiros e de não
>brasileiros, bate recorde atrás de recorde.
>Ultrapassou a marca dos 70 anos e logo passará dos 80,
>90, e quem sabe 100 anos. Avanços sem dúvida notáveis.
>Doenças contagiosas foram substituídas por doenças
>crônicas, como problemas cardiovasculares e câncer, e
>por causas externas, como trauma. Essa mudança de foco
>exigiu modificações intensivas no nível estrutural.
>Algumas especialidades assumiram posição de destaque
>nunca antes alcançada. Os serviços de cardiologia de
>muitos hospitais logo se transformaram em
>departamentos e, a seguir, em grandes centros com
>prédios próprios. O mesmo ocorreu com a oncologia e
>suas variadas especialidades. Houve uma explosão na
>tecnologia e na sofisticação, e, conseqüentemente, nos
>custos ­ alertam os especialistas em saúde pública.
>Recomenda-se também a melhora da saúde da população
>por intermédio do envolvimento acadêmico.
>

[A] Sem comentarios, pra que vou querer viver até o 100?
>
>Muito por fazer.
>Como melhorar o atendimento ao público do SUS

[A] Infelizmente a ciencia não melhora ela mesma vai melhorar o SUS, a ciencia infelizmente, anda atendendo interesses dos laboratorios e pondo em pratica, o que determina o codigo americano, Bisness in bisness...
>
>
>A academia médica no Brasil foi alterada sensivelmente
>nas últimas duas décadas. Progressivamente, as escolas
>médicas e os programas de pós-graduação introduziram
>disciplinas de pesquisa epidemiológica de problemas de
>saúde adaptados à realidade da população do País. Mais
>e mais livros têm sido publicados por acadêmicos
>brasileiros, com enfoque nacional. Mais e mais centros
>de treinamento oferecem vagas para médicos
>interessados em se aperfeiçoar no atendimento básico à
>saúde da família e à clínica geral. O impacto na
>melhora da atenção à saúde do brasileiro pode demorar
>a ser notado. Muito há de ser feito em 2005 e além.

[A] Infelizmente, tem muito moleque fazendo medicina pro papai e só sai meleca, ou o diagnostico é virose ou é exames a "toa"...


>O plano Fome Zero tenta corrigir um problema básico de
>saúde: a miséria e a conseqüente desnutrição. Ninguém
>precisa de doutorado para compreender a conexão direta
>entre a desnutrição e a ocorrência de doenças
>potencialmente graves. É um plano interessante, mas
>ainda longe de atingir seus objetivos de forma
>significativa. Em 2005, e além, o governo deverá fazer
>os ajustes finos para que o programa consiga alcançar
>seu alvo.
>
>A violência é problema de segurança pública, mas
>também é problema de saúde. Não podemos esquecer que,
>no ano passado, mais de 40 mil brasileiros foram
>assassinados. Se acreditarmos nos dados oficiais do
>Ministério da Saúde, naquele período, morreram mais
>brasileiros por tiro do que por câncer de pulmão. O
>controle da violência evitará milhares de mortes e
>seqüelas dramáticas.
>
>A malária é outro problema sem solução a curto prazo.
>Áreas extensas do território nacional são infestadas
>por mosquitos portadores dessa doença. O esforço das
>autoridades regionais e federais deverá ser
>intensificado nas várias frentes: pesquisa de vacinas,
>profilaxia, controle do mosquito e tratamento de
>pacientes infectados. A mortalidade por essa infecção
>deve ser reduzida a todo custo. Estudos recentes
>aventam que a vacina contra a malária pode estar ao
>alcance da ciência em prazo razoável.
>
>No mundo todo, um bilhão de pessoas são infectadas por
>parasitas, como os vermes, e o Brasil tem participação
>significativa nesses dados alarmantes. Vermes no
>intestino não são apenas feios. Eles causam problemas
>como deficiências nutricionais, e, em alguns casos,
>podem até ser fatais. Para mudar isso, saneamento
>básico é fundamental.
>
>Um estudo recentemente realizado e publicado por
>pesquisadores na Universidade do Ceará demonstrou
>claramente a possibilidade de praticamente eliminar os
>parasitas dos pacientes. Remédios eficazes, como a
>ivemerctina, podem ser a solução. Além de tratar o
>paciente, eliminam uma potencial fonte de contaminação
>para outras pessoas. As autoridades devem criar
>sistemas integrados, contínuos, para atacar esse
>problema de saúde pública em várias frentes, em 2005 e
>para sempre.
>
>Quanto à Aids, nos últimos anos, houve um declínio da
>incidência de novos casos na maioria dos estados
>brasileiros. Estudo publicado recentemente por
>pesquisadores da Fundação Oswaldo Cruz, no Rio de
>Janeiro, confirmou que, apesar de recursos limitados
>do governo e da desigualdade socioeconômica aberrante
>no Brasil, a introdução do acesso universal à terapia
>antiviral contribuiu para uma redução impressionante
>na mortalidade por Aids, e pode ter ajudado a diminuir
>sua incidência.
>
>Nessa linha de pensamento, o presidente Lula lançou,
>este ano, o louvável Plano Nacional de Eliminação da
>Hanseníase (a lepra) em até dois anos. Um passo sem
>dúvida importante para controlar essa doença
>contagiosa e debilitante. Paralelamente, o ministro da
>Saúde, Humberto Costa, anunciou a formação de uma
>comissão para promover políticas nacionais de
>reabilitação às pessoas afetadas pela hanseníase:
>
>­ A idéia é possibilitar benefícios, como a
>reabilitação por cirurgias plásticas, financiando para
>a capacitação de médicos, enfermeiros e
>fisioterapeutas para que realizem esses procedimentos
>nos incapacitados fisicamente.
>
>Todos os anos, 42 mil brasileiros contraem a
>hanseníase. E o País só perde para a Índia no ranking
>mundial de casos da doença.
>
>A obesidade, mal do século XXI, mata. Nos EUA, em
>alguns estados o número de óbitos por obesidade
>ultrapassou o de câncer. No Brasil, cientistas
>identificaram obesidade crescente na população. Estudo
>publicado por pesquisadores da Universidade Federal do
>Rio de Janeiro confirmou o aumento progressivo do peso
>dos adolescentes, tanto na Região Nordeste quanto no
>Sudeste do País, desde 1975. Atualmente, 17% dos
>adolescentes da Região Sudeste enquadram-se na
>definição internacional de obesidade. E seus efeitos
>maléficos logo alcançarão a saúde pública. Orientação
>e conscientização devem ser política contínua e bem
>estruturada. A prevenção e o tratamento do sobrepeso
>precisam ser prioridades das políticas de saúde nos
>próximos anos.
>
>Um problema que acomete principalmente as crianças é a
>poluição atmosférica. Um estudo realizado com 5.193
>crianças de duas cidades do Rio de Janeiro, e
>publicado recentemente na revista Annals of Allergy
>Asthma and Immunology, mostrou claramente a correlação
>entre os níveis de poluição e a incidência de
>problemas respiratórios. Observou-se um aumento de
>mais de 50% na freqüência de crises de asma nos
>moradores de Duque de Caxias (local com alta
>concentração de poluentes), comparados aos moradores
>de Seropédica (local com baixos níveis de poluição).
>Outro estudo, realizado pelos pesquisadores do
>laboratório de poluição atmosférica da Universidade de
>São Paulo, confirmou os efeitos nocivos da poluição do
>ar na mortalidade infantil. Há aumento de 6% no número
>de óbitos neonatais nas regiões mais poluídas. O
>controle de poluentes de qualquer origem deverá ser
>obrigação mundial. Ainda mais com a entrada em vigor
>do Protocolo de Kyoto.
>
>Mas, de forma geral, provavelmente o maior desafio das
>autoridades de saúde é garantir um apoio ao avanço
>científico na área de saúde, atingir a maioria da
>população com os avanços médicos e tecnológicos
>alcançados, e insistir na prevenção e na saúde
>primária, básica. Muito mais do que dinheiro, em 2005
>precisamos de estratégia, filosofia e enfoque novos.
>
>O acesso da população, principalmente das camadas
>menos privilegiadas da sociedade, a remédios em geral
>é muito limitado. Várias são as causas, como preços
>elevados na fonte, acréscimos excessivos na cadeia de
>distribuição e impostos. Se as autoridades, de todas
>as esferas, não atentarem a essas dificuldades
>enfrentadas no dia-a-dia da maioria dos cidadãos,
>muito mais será gasto para tratar complicações graves
>de doenças crônicas, como diabetes e hipertensão
>arterial. O Brasil iniciou uma ação interessante nessa
>direção: a aquisição da fábrica da GlaxoSmithKline do
>Brasil pelo governo federal. O presidente Lula
>destacou que "pela primeira vez um governo compra uma
>fábrica da iniciativa privada", ao mesmo tempo
>criticando "o processo inverso de privatização do
>governo anterior". Lula declarou, na ocasião, que
>estava "recuperando uma fábrica que seria desativada,
>eliminaria empregos e que agora produzirá, em escala
>nacional, os antibióticos mais usados no Brasil".
>
>Nessa mesma linha o ministro da Saúde, Humberto Costa,
>deixou muito clara sua estratégia para 2005:
>
>­ Vamos ampliar o acesso da população a medicamentos
>fundamentais com base em três opções. A primeira é a
>rede de farmácias populares, e até o fim do ano
>pretendemos inaugurar cem unidades em todo o Brasil. A
>segunda é a criação de um programa para a venda
>subsidiada de medicamentos básicos para hipertensão e
>diabetes, através da rede de farmácias privada.
>Pretendemos baixar os preços em até 50%. E a terceira
>é a redução do ICMS de 2,8 mil medicamentos até 2005.
>
>De acordo com uma pesquisa do IBGE, de 2003, a saúde
>aparece em terceiro lugar no orçamento das famílias
>brasileiras, e os medicamentos representam 61% desses
>gastos para as pessoas de baixa renda. Além disso,
>metade das pessoas que precisam de tratamento não pode
>pagar os remédios de que necessitam.
>
>A partir de 2005, 50 milhões de unidades de
>antibióticos serão produzidas para a rede do Sistema
>Único de Saúde (SUS) e para as Farmácias Populares. Em
>2007, a produção deve quintuplicar em relação à atual.
>Mais de 10 bilhões de unidades de medicamentos devem
>ser produzidas para as principais doenças que mais
>atingem a população brasileira, como hipertensão,
>diabetes, malária e tuberculose. A Fiocruz também
>produzirá mais vacinas, e o Ministério da Saúde
>enfatiza que "a ampliação do acesso da população aos
>medicamentos é uma das prioridades do governo
>federal".
>
>Em 2004, muito se fez pela saúde, mas ainda há muito
>mais por fazer. Programas de prevenção e detecção
>precoce de doenças sexualmente transmissíveis, como
>Aids, HPV (câncer de colo de útero), a melhoria na
>qualidade do atendimento dos pacientes do SUS,
>diminuição das filas, controle de poluição, de
>infecção hospitalar e, também, de mortes por agentes
>externos, como traumas (tiros, facadas, acidentes de
>trânsito). Para se ter uma idéia do custo com os
>acidentes de trânsito, a cada ano acontecem mais de
>100 mil mortes, com três a quatro vezes esse número de
>feridos. E a maioria absoluta é tratada com dinheiro
>público. São milhões de reais de gastos evitáveis. Sem
>falar nas seqüelas individuais.
>

Abraços
Amauri

[As partes desta mensagem que não continham texto foram removidas]



##### ##### #####

Para saber mais visite
http://www.ciencialist.hpg.ig.com.br


##### ##### ##### #####



Yahoo! Grupos, um serviço oferecido por:
PUBLICIDADE




------------------------------------------------------------------------------
Links do Yahoo! Grupos

a.. Para visitar o site do seu grupo na web, acesse:
http://br.groups.yahoo.com/group/ciencialist/

b.. Para sair deste grupo, envie um e-mail para:
ciencialist-unsubscribe@yahoogrupos.com.br

c.. O uso que você faz do Yahoo! Grupos está sujeito aos Termos do Serviço do Yahoo!.



[As partes desta mensagem que não continham texto foram removidas]



##### ##### #####

Para saber mais visite
http://www.ciencialist.hpg.ig.com.br


##### ##### ##### #####


Yahoo! Grupos, um serviço oferecido por:







------------------------------------------------------------------------------
Links do Yahoo! Grupos

a.. Para visitar o site do seu grupo na web, acesse:
http://br.groups.yahoo.com/group/ciencialist/

b.. Para sair deste grupo, envie um e-mail para:
ciencialist-unsubscribe@yahoogrupos.com.br

c.. O uso que você faz do Yahoo! Grupos está sujeito aos Termos do Serviço do Yahoo!.



[As partes desta mensagem que não continham texto foram removidas]



##### ##### #####

Para saber mais visite
http://www.ciencialist.hpg.ig.com.br


##### ##### ##### #####


Yahoo! Grupos, um serviço oferecido por:







------------------------------------------------------------------------------
Links do Yahoo! Grupos

a.. Para visitar o site do seu grupo na web, acesse:
http://br.groups.yahoo.com/group/ciencialist/

b.. Para sair deste grupo, envie um e-mail para:
ciencialist-unsubscribe@yahoogrupos.com.br

c.. O uso que você faz do Yahoo! Grupos está sujeito aos Termos do Serviço do Yahoo!.



[As partes desta mensagem que não continham texto foram removidas]



SUBJECT: Re: A Próxima Guerra
FROM: "Rodrigo Marques" <rodmarq72@yahoo.com.br>
TO: ciencialist@yahoogrupos.com.br
DATE: 03/01/2005 12:57


Peço desculpas pela minha preguiça, mas pensei que alguém das listas
já tivesse alguma informação preparada a respeito, por isso não
verifiquei pessoalmente...mas já me passaram um link onde eu posso
conferir o assunto...
Valeu gente!!
--- Em ciencialist@yahoogrupos.com.br, "Alvaro Augusto - Electra"
<alvaro@e...> escreveu
> Se os e-mails são verdadeiros? Se as pessoas sabem que seus nomes
estão sendo usados? Basta escrever para eles e perguntar!
>
> [ ]s
>
> Alvaro Augusto






SUBJECT: primeiros socorros
FROM: "E m i l i a n o C h e m e l l o" <chemelloe@yahoo.com.br>
TO: <ciencialist@yahoogrupos.com.br>
DATE: 03/01/2005 13:15

Claro que ir ao médico é o mais recomendado. Mas no caso de primerios
socorros, o que se pode fazer?

[ ] 's do Emiliano Chemello
---
Contato Naeq:
Nome: lia belart
Email: liabelart@uol.com.br
Telefone: Picada de aranha-marrom Lexosceles
Mensagem: Prezados Senhores,

Gostaria de saber, quais os medicamentos para os primeiros socorros, que
podem ser aplicados, no caso de sermos picados por esse tipo de aranha.
O genro de minha irmã, deve ter sido picado por esse tipo de aranha, pois os
sintomas são iguais.
Atenciosamente,
Lia



SUBJECT: evolucao e desafios
FROM: israel-cordeiro <israel-cordeiro@ig.com.br>
TO: ciencialist@yahoogrupos.com.br
DATE: 03/01/2005 13:33

Olá Amigos,

"Posso não concorda com tudo que dizes...", mas no caso do Amauri em que ele
fala sobre o HIV e Ca concordo plenamente. Bem, todos nós sabemos (só que
uns não querem acreditar) que com a ciência alcançando patamares cada vez
mais altos o "impossível" já é uma palavra excluida dos dicionários
cientifícos, a ciência e tecnologia hoje é quase capaz de qualquer coisa,e
um vírus existente a tanto tempo como o HIV, esta até hoje sem cura todos
sabemos o motivo, não é mesmo? É isso ai, DINHEIRO, MONEY, BUFUNFA, GRANA,
FAZ-ME-RIR...
Acredito sinceramente que se os cientistas quisessem realmente criar uma
cura para o HIV e Ca já o teriam feito, o real motivo de não ter feito ainda
é puramente financeiro.

É como neste caso:

Um homem muito rico e já de meia idade que morava numa pequena cidade tinha
uma inflamação que não sarava nunca, já tinha 6 anos que ele fazia um
tratamento semanal com o Dr. 'X', neste período o filho do Dr. 'X' fazia a
faculdade de medicina, seguindo o caminho do pai, o tempo passou o filho do
Dr. 'X' se formou e tornou-se o Dr. 'Y', então num belo dia o Dr. 'X' saiu,
teve que viajar, e deixou o seu filho tomando conta do consultório para ele,
então o homem muito rico que fazia seu tratamento semanalmente foi consultar
com o Dr. 'Y', e aquele jovem médico com toda a empolgação de um recem
formado consultou o homem (rico) e descobriu que o que causava a inflamação
do homen (rico) era uma bactéria que estava ali dentro daquela inflamação,
então ele tirou aquela bacteria, passado alguns dias aquele homem foi la o
agradecer pois a inflamação de 6 anos que nem piorava e nem melhorava tinha
sido curada, o jovem médico ficou muito entusiasmado e logo que seu pai o
Dr. 'X' chegou ele foi logo contar para ele, após ouvir toda a história do
filho, le sorriu como quem debochasse do filho, o filho achando que seu pai
ficaria surpreso e já sem entender nada, perguntou o pai do que ele estava
rindo, então seu pai lhe explicou:
_Sabe aquela bactéria?
_Sim!
_Foi ela quem pagou sua faculdade de medicina.

Analisem, os dois casos são iguais!!!!!!

um abraço

Israel


[As partes desta mensagem que não continham texto foram removidas]



SUBJECT: Re: [ciencialist] Fw: maremoto
FROM: TARCISIO BORGES <tbs97@fisica.ufpr.br>
TO: ciencialist <ciencialist@yahoogrupos.com.br>
DATE: 03/01/2005 14:21

Claro que vai!

O clima da Terra é um sistema dinâmico adaptativo, ou seja, muda todo o
dia, um verão nunca é igual ao outro.

[]s
TARCISIO BORGES
tbs97@fisica.ufpr.br

On Sun, 2 Jan 2005, Luiz Ferraz Netto wrote:
> que acham?
> ===========================
> Luiz Ferraz Netto [Léo]
> -----Mensagem Original-----
> De: armandopentium
>
> Olá! bom dia professor gostaria de saber o que vai acontecer agora com
> essa auteração de 2cm do eixo da terra devido ao maremoto , será que
> vai mudar o clima da terra?



SUBJECT: Fw: 'coloque aqui o seu tema'
FROM: "Luiz Ferraz Netto" <leobarretos@uol.com.br>
TO: "ciencialist" <ciencialist@yahoogrupos.com.br>
DATE: 03/01/2005 17:06

Ciência para todos .........

[]'
===========================
Luiz Ferraz Netto [Léo]
leobarretos@uol.com.br
http://www.feiradeciencias.com.br
===========================
-----Mensagem Original-----
De: Rodrigues, Rafael de Oliveira
Para: leobarretos@uol.com.br
Enviada em: segunda-feira, 3 de janeiro de 2005 12:55
Assunto: 'coloque aqui o seu tema'


Nas dúvidas experimentais, por gentileza coloque aqui o endereço da página, isso facilita o confronto. Agradeço. Meu nome é LUIZ FERRAZ NETTO, meu apelido é LÉO e moro em BARRETOS; dai vem meu e-mail: leobarretos@uol.com.br.

Nas experiências que envolvem feijão e algodão o primeiro só absorve água e não tem nenhuma fonte mineral(exceto as da água). O feijão é um alimento rico em ferro. Quando ele germina e temos uma vagem, ela também é rica em ferro? E de onde ele vem?



--------------------------------------------------------------------------------


No virus found in this incoming message.
Checked by AVG Anti-Virus.
Version: 7.0.298 / Virus Database: 265.6.7 - Release Date: 30/12/2004

----------

No virus found in this outgoing message.
Checked by AVG Anti-Virus.
Version: 7.0.298 / Virus Database: 265.6.7 - Release Date: 30/12/2004


[As partes desta mensagem que não continham texto foram removidas]



SUBJECT: Presente de Natal - Quântica
FROM: "E m i l i a n o C h e m e l l o" <chemelloe@yahoo.com.br>
TO: <ciencialist@yahoogrupos.com.br>
DATE: 03/01/2005 17:08

hohoho... não é o papai noel, mas trago presente para vocês:

Primer on the Quantum Theory of the Atom
Stephen K. Lower - Simon Fraser University
http://www.chem1.com/acad/pdf/quantum.pdf

Este pdf vai especialmente para os net amigos Alberto e Léo. Para o
Alberto pelo fato dele 'adorar' Quântica. Para o Léo pelo fato dele 'adorar'
textos em inglês ou, como ele mesmo classifica: "língua alienígena"

Amplexos,
Emiliano Chemello
chemelloe@yahoo.com.br



SUBJECT: Super pião
FROM: "Kentaro Mori" <kentaro.mori@itelefonica.com.br>
TO: ciencialist@yahoogrupos.com.br
DATE: 03/01/2005 18:34


Gira por +15 minutos:
http://www.quirkle.com/top/index.htm

Que acham?

Prof Léo, sacrilégio: um levitron foi vendido em um leilão on-line
japonês por nada menos que 730 ienes. Capaz de vir de brinde com um
McLanche Feliz daqui a pouco.

Abraços,

Mori





SUBJECT: Re: [ciencialist] Evolucao e desafios -Amaury
FROM: "Oraculo" <oraculo@atibaia.com.br>
TO: <ciencialist@yahoogrupos.com.br>
DATE: 03/01/2005 19:47

Olá Amaury

Fico contente que goste dos debates e de minha participação (nem sempre é assim, as vezes o pessoal se irrita bastante comigo:-)

Mas ainda acho que está confundindo duas coisas, a ciência, abstrata, e o comportamento humano, passível de ser julgado e avaliado subjetivamente.

É claro que é importante discutir a aplicação, direção, prioridade, etc, de pesquisas e do conhecimento humano. Mas, a ciência, é apenas um conjunto de conhecimentos adquirido de certa forma (método cientifico), não seu uso ou resultado. Isso está confundindo a discussão..:-)

A questao é, se decide agir com eficácia, tomar uma decisão que se mostre confiável no resultado, ou escolher uma afirmação ou conclusão que se aproxime o máximo possível da realidade (sem pretensões de ser 100% ou "A VERDADE"), o que escolheria? Seja para curar pessoas, seja para matar inimigos, o que consideraria mais confiável, mais real ou mais eficaz?

Sua resposa, que certamente é igual a minha, coloca a perspectiva correta na discussão..:-) Ciência, abstratamente, é a melhor ferramenta de compreensão deste universo (universo fisico) que dispomos. O que faremos com o conhecimento assim obtido, é outra coisa. Eu, particularmente, o usaria para melhorar a vida de todo mundo (por exemplo, usando o produto desse conhecimento para enviar ajuda as vitimas do tsunami na Asia). Outros a usariam para matar ou criar armas de destruição em massa.

Mas culpar a ciência, um conhecimento abstrato, pelo uso que malucos fazem dela, é irrelevante e impróprio. Me lembra a piada do matador que, perguntado se não tinah remorso, respondeu: Eu não mato ninguem, só faço o furo, quem mata é Deus (no que ele tem razão, de uma perspectiva religiosa do universo..:-)

Pessoas, seres humanos, matam e maltratam. E fazem isso muito antes de termos armas de destruição em massa ou equivalentes. Culpar o conhecimento do átomo pelo uso da bomba atomica seria como condenar a arma no lugar do atirador (imagino que seria um processo muito engraçado de acompanhar..:-)

Veja este trecho seu:

Amaury: Por isso, nada tenho contra ou a favor da ciência, apenas é uma visão critica sobre...:-)

Se fosse assim, nem haveria a discussão entre nós..:-) Mas seu posicionamento inicial era diferente, contrário a ciência, como se fosse ela responsável por seu uso. Por isso fiz minha análise de sua análise..:-)

A confusão fica mais clara neste trecho:

Amaury:O que temos de melhor, como disse, esta nas mãos dos poderosos, portanto "o que temos de melhor" foi e esta sendo mandado pelo governo dos poderosos. Hoje os norte-americanos mandam e desmandam no que "temos de melhor", tanto se é para o bem da humanidade é imoral, mas se beneficia as partes bélicas deles, seguem numa boa...:-)


Neste caso, o termo "melhor" tem duas conotações, que foram misturadas. O que temos de melhor, no meu sentido, é de mais eficaz. Dá resultados. No seu uso, é um julgamento subjetivo, melhor, bom, correto, etc. Não tem ligação com meu argumento. A ciência é "melhor" se deseja uma ação eficaz, ou um conhecimento confiável. O que fará com esse conhecimento, se algo "melhor", do bem, ou algo "pior", do mal, não é relevante para o argumento.

Já a erradicação da malaria foi um erro, ato falho, meu..:-) Pensei em variola e escrevi malária (acho que passava um especial na TV sobre a malária..:-). Mas, mesmo a malária é mais controlada em paises civilizados, com alto uso de conhecimento cientifico, que nos pobres e sem esse uso. Ainda é um bom argumento sobre a eficácia da mesma..:-)

Finalmente, acho que não compreendeu a questao a emissão de radio e a ida à Lua. Não importa o poder dos USA, nào importa seu governo ou o carater (ou falta de) dos seus governantes, mesmo hoje, se emitir uma onda de rádio, quaisquer dois radio amadores podem dizer, precisamente, de onde vem. Isso se chama triangulação, e é o motivo de se gastar tanto tempo e dinheiro em desenvolvimento de aviões invisiveis, já que, se o aparelho voador reflete a onda do radar, pode ser posicionado precisamente, sem sombra de duvida.

Explicando melhor, se um satélite emite sinal de radio, você, com seu radio amador (ou radar avançado, não importa.:-), pode determinar a direção com precisão. Se outro amigo seu radio amador também o fizer, os dois podem combinar os dados e dizer, com exatidão, de onde parte o sinal e exatamente onde está o satelite. Não há forma de fraudar isso, nem de disfarçar ou fingir estar onde não está. Assim, quando as Apollo orbitavam a Terra, foram acompanhadas de perto por todos que desejassem, sem forma de fraudar isso. Quando orbitaram a Lua, foram acompanhadas (com frustação crescente dos russos..:-), por toda parte.

E ao pousar, e enviar dados do solo lunar, idem, sem possibilidade de fraudar essa emisão.

Assim, se deseja discordar do pouso na Lua, tem de explicar como isso foi feito, ou vai esbarrar em algo chamado evidencia acachapante (que colocou fim nas especulações dos russos, chineses e todos os muitos inimigos dos USA durante a guerra fria que adorariam desmascarar a fraude). Alias, quando os russos colocaram o primeiro satélite, Sputinik, em órbita, alguns jornais americanos e alguns membros do governo americano tentaram vender a idéia de fraude, mas o fato que qualquer um com um bom rádio podia confirmar o feito, foi o bastante para convencer as pessoas.:-)

HIV

Lutamos a séculos com o virus da gripe e ainda não podemos mata-lo. Sua taxa de mutação, como costuma acontecer com virus, é brutal, e sua adaptação ciclica impede que seja feita uma vacina para todas as cepas. A simplicidade de determinados virus é a chave para que escapem tão facilmente da cura. Um organismo complexo tem estruturas que se repetem em todas as suas variações. Em algum momento encontramos essa estrutura e uma vacina é criada. Mas, organismos simples, em especial virus, mudam tão completamente, que uma estrutura, uma chave, única, não acontece. Isso torna bem dificil derrota-lo. Além disso, a simplicidade faz com que muitas estruturas sejam também usadas por células do organismo hospedeiro. Assim, algumas das drogas que efetivametne matam os virus da gripe não podem ser aplicadas a seres humanos, pois matam suas células saudáveis também (esse é um dos problemas com drogas anti-cancer também atingir células saudaveis juntamente as doentes).

Talvez as industrias e governos não invistam o suficiente na pesquisa, eu tendo até a concordar com você nisso. Mas isso indica duas coisas: primeiro, que é caro e dificil descobrir a cura, ou ela já teria sido descoberta mesmo com pouco dinheiro. Segundo, indica que temos FALTA de ciência, que seria possivel com mais recuros, e não exesso. Em qualquer dos casos a crítica feita não se aplica. Precisamos, se queremos encontrar a cura ou tratamentos melhores para a AIDS, de mais ciência, um conheimento confiável e eficaz, e não de menos.

Espectativas

Amaury:Não tenho nada contra a ciência, mas você tem contra os índios...:) o que vejo é ainda crianças mortas e vidas curtas por uma ciência ineficaz, se eu gosto ou não é outra historia, que se eu não gosta-se da ciência não estaria numa lista de ciências...:) e pouco irei a uma tribo, porque não iria acostumar...:)e uma pergunta: a ciência tem vida própria?

A ciência não prometeu curar todas as doenças, consertar o mundo, ser 100% eficaz ou definir A VERDADE (para isso temos as religiòes..:-). Ela é apenas uma ferramenta, uma boa e eficaz ferramenta. Talvez esteja esperando demais da ciência e a frustação de não ter todas as respostas o tenha tornado um pouco rancoroso com ela. Entre duas afirmações, entre duas respostas, a obtida de forma cientifica é a mais confiável, só isso. Existe, sim, uma luta de pesquisadores, seres humanos, para usar essa ferramenta de forma a curar doenças, encontrar respostas, criar conforto, resolver problemas, etc. Mas é uma busca, não um final em sí mesmo.

Tenho as mesmas objeções que você às industrias farmaceuticas (inclusive as homeopaticas..:-). Mas sei que existem pesquisadores legitimamente dedicados a encontrar curas, soluções, respostas, apesar das empresas e até mesmo dentro dessas empresas. Gente que tem legitimos interesses humanos, que pesquisa até mesmo as "doenças de paises pobres" como a malária e febre amarela (erradicadas nos paises desenvlvidos).

Bush é uma "anta" fundamentalista. Mas observe que é uma das pessoas mais contra a ciência que poderiamos encontrar. Seu esforço fundamentalista para obrigar sua visão religiosa de mundo a todo o resto do planeta é frontalmente contrária a ciência, tolerante por natureza. Impedir o estudo de células tronco pode impedir avanços que poderiam curar diversas doenças, inclusive paraplegia. Ensinar criacionismo em escolas e banir a evolução (e a base da biologia moderna hoje) impediria até mesmo que bons médicos se formem ou que novas descobertas sejam feitas.

Ainda assim, ele usa os resultados da ciência onde lhe convém, com armas de destruição e brinquedos tecnologicos que poder. Isso torna a ciência culpada? Ou torna o Bush responsável?

Hiroshima é um caso complicado. Nagazaki seria mais claro, como alvo do debate, desnecessário e violento. Mas Hiroshima é dificil. Antes que "caiam de pau", acho um horror a destruição de uma cidade, a morte de civis e qualquer tipo de conflito. Mas é preciso pensar, sempre, sob pena de não ver com clareza. E Hiroshima é dificil de enxergar, através do horror da bomba e da matança.

Mas havia uma guerra em curso, isso é claro. Havia um pais, Japão, que entrou na guerra por vontade própria, para expadir fronteiras e massacrar vizinhos (embora a vontade a considerar fosse apenas dos mandatarios do pais na época). Havia um comportamento cultural onde a morte era sublime, o sacrificio honroso e o imperador deus (você morreria por seu deus). Dentro dessa análise, continuar a guerra com o Japão produziria, por estimativa, 100 mil mortes de americanos e mais de 350 mil mortes japonesas (estimativas derivadas das batalhas já ocorridas, da decisão de cada soldado japones de só parar depois de morto e das dificuldades de desembarque em ilhas - as piores batalhas da segunda guerra forma em ilhas do Japão).

Agora, você (quem me lê) deve decidir (pressionado certamente pelos falcões militares) o que fazer. Imagine que estamos em guerra com a Argentina. Você, presidente, tem uma arma recem criada que pode acabar a guerra, devido ao seu poder inimaginável. Detonar essa arma em um cidade vai matar 45 mil pessoas do outro pais (que começou a guerra). Não detonar vai prolongar a guerra por mais alguns meses, a Argentina jamais se renderá e 100 mil brasileiros morrerão, e mais de 350 mil argentinos. O que você decidiria? O que diria as mães e pais de seu pais, quando perguntassem porque não parou a guerra quando podia e escolheu ver morrer mais 100 mil de seus conterraneos?

Deve se lembrar que nunca uma arma assim havia sido detonada, e não se sabia o horror que poderia causar (os primeiros testes no deserto americano usaram soldados para verificar o efeito e a descontaminação foi feita com vassouras de palha e agua corrente.:-(

Eu não sei o que escolheria e não gostaria de ter de decidir algo assim. Imagino que não se pode dormir mais depois de ser submetido a coisas desse tipo. Por isso considero que Nagasaki é mais claro, uma decisão certamente militar, resultado de uma perda de poder civil em um periodo dificil. Mas Hiroshima é mais complicado.

Vou repetir, acho um horror o bombardeio de civis, na verdade, acho qualquer guerra um horror, desnecessária e absurda. Mas não acho que Hiroshima foi apenas maldade, crueldade de monstros, mas uma decisão calculada, dificil e medonha, da qual eu não saberia o que escolher como "menos pior". E hoje temos os USA arrogantes e com jeito de donos do mundo, e nossa antiapatia por Bush nos faz ver tudo com um filtro de asco. Mas na época a guerra dos USA era contra imperios crueis e mais perigosos que a america jamais será (Hitler, e todo o horror que representa) e nào se pode relativizar o que teria acontecido se a Alemanhã e o Japão ganhassem a guerra.

De todo modo, estamos mais perto da concordancia que da discordância..:-) A ciência, como ferramenta, é eficaz, confiável e razoável. Seu uso, que pode ser para o bem ou para o mal (mesmo que o bem e o mal sejam relativos, pergunte ao Bin Laden e ao Bush..:-), é que é passível de discussão.

Um abraço.

Homero






----- Original Message -----
From: Amauri Jr
To: ciencialist@yahoogrupos.com.br
Sent: Monday, January 03, 2005 11:45 AM
Subject: [ciencialist] Evolucao e desafios -Homero


Olá Amury


[A] Olá Homero






"Minhas análises sobre suas análises..:-)"



[A] Sinceramente gosto de suas analises, fico satisfeito de você me responder...:-)





"Você parece estar ressentido com o que chama de "ciência" e a trata como entidade independente, crença de cientistas ou algo a ser adorado ou derrubado. Mas nada disso é ciência, nem o resultados de sua aplicação se confundem com ela. Ciência neste contexto (desta lista e do uso padrão do termo..:-) é apenas o conjunto dos conhecimentos obtidos através de um método padronizado, chamado cientifico, e do rigor derivado do mesmo. E, independente do que pense dela, tem elevada confiabilidade..:-)"



[A] Amigo, o que penso que infelizmente o homem usa seu conhecimento para algo não muito para o "bem" humano ou "mal", a natureza humana já é corrupta. Tudo que é humano tem falhas, pois nada ainda é perfeito, mas a ciências como algo que vem para beneficiar a humanidade tinha que ser impartidaria e sem nenhuma crença ou fanatismo; como vimos muito na lista. Por isso, nada tenho contra ou a favor da ciência, apenas é uma visão critica sobre...:-)



"Não importa se o resultado é passível de ser julgado subjetivamente como "mau" ou mesmo como "bom". Importa apenas que é mais eficiente que outras formas de conhecimento, mais eficaz em suas previsões e mais confiável que outros instrumentos de compreensão do universo já criado por seres humanos. E que será abandonado assim que uma ferramenta mais eficaz seja apresentada..:-) Até lá, é o que temos de melhor."



[A] O que temos de melhor, como disse, esta nas mãos dos poderosos, portanto "o que temos de melhor" foi e esta sendo mandado pelo governo dos poderosos. Hoje os norte-americanos mandam e desmandam no que "temos de melhor", tanto se é para o bem da humanidade é imoral, mas se beneficia as partes bélicas deles, seguem numa boa...:-)





"Podemos analisar filosoficamente, subjetivamente, os resultados, mas não discutir a eficácia. A ciência, o conhecimento assim acumulado, é tão eficiente quando cura doenças, como a erradicação da malária, como quando destrói milhões de vidas, como em Hiroshima. O resultado é passível de julgamento, mas a ação não. Ela é eficaz, nos dois casos. Se precisar salvar alguém ou matar alguém, deve usar a ciência, é mais eficaz sempre..:-)"



[A] Podemos sim, tanto que você já disse que ciência é um conjunto de conhecimentos; mas muitos conhecimentos, pois o que vimos, é um "Clube da Ciência" que vê apenas um dos vários lados do universo, mas temos vários e é ainda um incógnito pelo homem ainda não abrir a cabeça...:-) Agora, se erradicamos a malaria não sei, creio que você não esta lendo jornal, pois a malaria, a febre-amarela, o HIV, o câncer, então matando e a ciência "tudo que temos de melhor" fica ai procurando pelos em baratas. Hiroshima foi um ato terrorista covarde e deprimente, que deveria fazer um julgamento dos cientistas e dos governantes no tribunal de Nuremberg, por danos morais humanos.





"Não gostar dela, ou como você pretende com as análises, critica-la ou despreza-la, não importa em nada para sua eficácia e confiabilidade. Você vai viver mais que todos os seus antepassados, goste ou não..:-) Sua expectativa de vida é de 75 anos, e, não sei sua idade, mas se já passou dos 40 como eu, é mais que seu avô esperava viver ao nascer (em 1900 a expectativa de vida no Brasil era de 33 anos)."





[A] Tenho apenas 28 anos, sem filhos e com uma deficiência física, o que sei que são superfluidades da senhora sagrada ciência. O que pode me interessar eu viver 40 ou 70 anos se nem ela pode prever?





"Na verdade, você usou um computador e a Internet, frutos da ciência, para enviar seus pensamentos e ataques a ciência, ao inves de tentar telepatia, rezas ou mandingas (ou qualquer outra forma de comunicação não cientifica) justamente porque é mais eficaz e confiável que qualquer outra..:-)"



[A] Alguma coisa boa tinha que sair daí né?...:-) o problema é esse, vocês da ciência só tem um foco do assunto, não ve as múltiplas variedades de estudos...:)





"Você não parece gostar de muitos dos aspectso do uso do conhecimento cientifico. Direito seu. E pode até mesmo abandona-los, todos, e ir viver em uma aldeia remota no Amazonas, sem (quase) nenhum contato com a ciência (os índios são muito receptivos a forasteiros, vão gostar de recebe-lo), e com todo o ônus de viver dessa forma (filhos mortos, vida curta, doenças diversas, poucos dentes, etc). Mas está na verdade criticando comportamentos humanos, que são seres falhos como sabe, não a ciência. Esta, "estricto senso", não é boa ou má, cruel ou gentil, apenas eficiente, confiável em seus efeitos."



[A] Não tenho nada contra a ciência, mas você tem contra os índios...:) o que vejo é ainda crianças mortas e vidas curtas por uma ciência ineficaz, se eu gosto ou não é outra historia, que se eu não gosta-se da ciência não estaria numa lista de ciências...:) e pouco irei a uma tribo, porque não iria acostumar...:)e uma pergunta: a ciência tem vida própria?



"Você não "acredita" nela. Engraçado, ela não pede que se acredite em nada.:-) Deve duvidar sempre, até que evidencias se mostrem sólidas o bastante para uma conclusão. E é nesse ponto que você escorrega..:-) O pouso na Lua já tem evidencias suficientes para essa conclusão, em que pese as tolices de diversos malucos mundo afora e suas teorias de conspiração. O HIV tem recuado e matado muito menos que no inicio da epidemia, graças a AZT e os atuais coquetéis antivirais. Se ajustes são feitos a toda hora, e são, sobre o que comer ou não, é apenas porque a ciência não pretende ser "A VERDADE" como religiões, mas um conhecimento que cresce, se aprofunda, se ajusta e melhora. Sabemos mais, não menos sobre o que comer ou não comer."



[A] Engraçado que pessoas inteligentes não duvidem que os yanques tenham forjado tudo, mas como você mesmo disse, é um direito seu. Eu acredito muito mais que se possa acreditar, a ciência melhorou minha vida muito, tendo esse computador; em minha cadeira de rodas de alumínio (de ferro era um lastima andar), portanto não estou desacreditando a ciência. Agora se quer achar que os coquetéis são eficazes tudo bem, mas que morre mesmo assim, isso morre...:)



"Sim, dá trabalho, é preciso atenção e constante leitura para acompanhar, seria melhor ter uma única resposta para todas as perguntas e pronto, sem risco e sem mudanças. Mas o mundo não funciona assim..:-) Sabemos hoje mais do que ontem sobre como se alimentar, e saberemos mais amanhã, deixando parte do que sabemos hoje para trás. (Parte, não tudo:-)"



[A] Como se o mesmo alimento hoje esta com proteínas e tudo que precisamos e amanha não?





"As teorias conspiratórias são atrativas.:-) É como se nós, os que estão "por dentro", soubéssemos de "coisas" que o resto do mundo não sabe, fossemos mais "espertos" ou mais inteligentes, que não se deixam enganar. Mas, na maioria das vezes (eu diria em sua totalidade) é apenas o ego tentando ser mais do que é.:-) Mesmo com centenas de sites "desmistificando" o pouso na Lua, o volume de evidencias a favor é gigantesco e nenhum deles explica um ponto fundamental do problema: como o governo americano fraudou os sinais de radio e TV que eram emitidos de todas as naves Apollo tanto durante as viagens, como do solo lunar, captados por todo radioamador ao redor do globo, sem falar nos radares e antenas dos paises em disputassem os USA? (lembre-se que, mesmo hoje, não se pode fraudar a origem de um sinal de radio..:-)"



[A] Não acredito nesses doentes mentais que ficam montando sites para se promover...:) eu tiro dos fatos que ocorreram que podem ser completamente forjados, como as transmissões, ou não sabe que eles são donos do que "temos de melhor"...:)



"Enfim, a ciência é eficaz. Embora seu uso, o uso do conhecimento confiável por ela produzido, possa ser discutido e julgado, isso não é a ciência nem com ela se confunde. E, tenha certeza, seu automóvel vai conduzi-lo amanhã, confiavelmente, conforme as leis da fisica descobertas pela ciência, seu computador receberá esta mensagem, os remédios na farmacia impedirão que você morra, as vacinas que tomou evitarão que adoeça (como todos os seus ancestrais adoeceram) e os satelites em órbita, colocados lá pelo conhecimento cientifico, vão transmitir suas chamadas telefonicas até para o Japão, se você assim desejar..:-)"



[A] Reverencia a ciência como um crente reverencia uma reza...:) o meu carro, vai bater conforma a lei da física, as farmácias venderão remédios que viciam, os satélites vão espionar minha casa e tudo graças a deusa infalível ciência...:)



"Para completar a mensagem anterior: uma menina inglesa de 15 aos foi a primeira pessoa a sobreviver depois de contrair raiva sem tomar a vacina antirabica. Ela foi submetida a um novo tratamento revolucionário (e científico), com um coquetel de drogas e coma induzido."



[A] É? Legal! Mas não são os igreses que usam a ciência para fazer guerrinhas com o Bush?





"Diferente de milagres divinos e sobrenaturais, o que se aprendeu com a nova técnica pode ser usado em outras pessoas doentes e não afetam apenas a pessoa que recebe a graça..:-)"



[A] Também acho, devo adverti-lo novamente que não sou a favor no milagre...:)




"A partir de agora, mesmo desprezando a "ciência", você pode ficar sossegado, se você ou um de seus filhos ou entes queridos contrair raiva, poderá ser curado, cientificamente curado.:-) A não ser que tenha resolvido ir viver em uma aldeia indigena na remota Amazonia..."



[A] Sim vão...vão tomar coca-cola e comer hambúrguer feitos cientificamente e se eu tiver sorte, vão se viciar em LSD feito pela ciência, vão tomar energéticos feito pela ciência, vão ter colesterol de produtos que o "temos de melhor" analisa. O mundo é um paraíso, graças a ciência não acha? ...:)



Abraços

Amauri


---- Original Message -----
From: Oraculo
To: ciencialist@yahoogrupos.com.br
Sent: Sunday, January 02, 2005 11:05 PM
Subject: Re: [ciencialist] Evolucao e desafios - CartaCapital - 02/01/05


Olá Amury

Minhas análises sobre suas análises..:-)

Você parece estar ressentido com o que chama de "ciência" e a trata como entidade independente, crença de cientistas ou algo a ser adorado ou derrubado. Mas nada disso é ciência, nem o resultados de sua aplicação se confundem com ela. Ciência neste contexto (desta lista e do uso padrão do termo..:-) é apenas o conjunto dos conhecimentos obtidos através de um método padronizado, chamado cientifico, e do rigor derivado do mesmo. E, independente do que pense dela, tem elevada confiabilidade..:-)

Não importa se o resultado é passível de ser julgado subjetivametne como "mau" ou mesmo como "bom". Importa apenas que é mais eficiente que outras formas de conhecimento, mais eficaz em suas previsões e mais confiável que outros instrumentos de compreensão do universo já criado por seres humanos. E que será abandonado assim que uma ferramenta mais eficaz seja apresentada..:-) Até lá, é o que temos de melhor.

Podemos analisar filosoficamente, subjetivamente, os resultados, mas não discutir a eficácia. A ciência, o conhecimento assim acumulado, é tão eficiente quando cura doenças, como a erradicação da malária, como quando destroi milhões de vidas, como em Hiroshima. O resultado é passível de julgamento, mas a ação não. Ela é eficaz, nos dois casos. Se precisar salvar alguém ou matar alguém, deve usar a ciência, é mais eficáz sempre..:-)

Não gostar dela, ou como você pretende com as análises, critica-la ou despreza-la, não importa em nada para sua eficácia e confiabilidade. Você vai viver mais que todos os seus antepassados, goste ou não..:-) Sua expectativa de vida é de 75 anos, e, não sei sua idade, mas se já passou dos 40 como eu, é mais que seu avô esperava viver ao nascer (em 1900 a expectativa de vida no Brasil era de 33 anos).

Na verdade, você usou um computador e a Internet, frutos da ciência, para enviar seus pensamentos e ataques a ciência, ao inves de tentar telepatia, rezas ou mandingas (ou qualquer outra forma de comunicação não cientifica) justamente porque é mais eficaz e confiável que qualquer outra..:-)

Você não parece gostar de muitos dos aspectso do uso do conhecimento cientifico. Direito seu. E pode até mesmo abandona-los, todos, e ir viver em uma aldeia remota no Amazonas, sem (quase) nenhum contato com a ciência (os indios são muito receptivos a forasteiros, vão gostar de recebe-lo), e com todo o onus de viver dessa forma (filhos mortos, vida curta, doenças diversas, poucos dentes, etc). Mas está na verdade criticando comportamentos humanos, que são seres falhos como sabe, não a ciência. Esta, "estricto senso", não é boa ou má, cruel ou gentil, apenas eficiente, confiável em seus efeitos.

Você não "acredita" nela. Engraçado, ela não pede que se acredite em nada.:-) Deve duvidar sempre, até que evidencias se mostrem sólidas o bastante para uma conclusão. E é nesse ponto que você escorrega..:-) O pouso na Lua já tem evidencias suficientes para essa conclusão, em que pese as tolices de diversos malucos mundo afora e suas teorias de conspiração. O HIV tem recuado e matado muito menos que no inicio da epidemia, graças a AZT e os atuais coqueteis antivirais. Se ajustes são feitos a toda hora, e são, sobre o que comer ou não, é apenas porque a ciência não pretende ser "A VERDADE" como religiões, mas um conhecimento que cresce, se aprofunda, se ajusta e melhora. Sabemos mais, não menos sobre o que comer ou não comer.

Sim, dá trabalho, é preciso atenção e constante leitura para acompanhar, seria melhor ter uma única resposta para todas as perguntas e pronto, sem risco e sem mudanças. Mas o mundo não funciona assim..:-) Sabemos hoje mais do que ontem sobre como se alimentar, e saberemos mais amanhã, deixando parte do que sabemos hoje para tras. Parte, não tudo:-)

As teorias conspiratórias são atrativas.:-) É como se nós, os que estão "por dentro", soubessemos de "coisas" que o resto do mundo não sabe, fossemos mais "espertos" ou mais inteligentes, que não se deixam enganar. Mas, na maioria das vezes (eu diria em sua totalidade) é apenas o ego tentando ser mais do que é.:-) Mesmo com centenas de sites "desmistificando" o pouso na Lua, o volume de evidencias a favor é gigantesco e nenhum deles explica um ponto fundamental do problema: como o governo americano fraudou os sinais de radio e TV que eram emitidos de todas as naves Apollo tanto durante as viagens, como do solo lunar, captados por todo radio-amador ao redor do globo, sem falar nos radares e antenas dos paises em disputacom os USA? (lembre-se que, mesmo hoje, não se pode fraudar a origem de um sinal de radio..:-)

Enfim, a ciência é eficaz. Embora seu uso, o uso do conhecimento confiável por ela produzido, possa ser discutido e julgado, isso não é a ciência nem com ela se confunde. E, tenha certeza, seu automóvel vai conduzi-lo amanhã, confiavelmente, conforme as leis da fisica descobertas pela ciência, seu computador receberá esta mensagem, os remédios na farmacia impedirão que você morra, as vacinas que tomou evitarão que adoeça (como todos os seus ancestrais adoeceram) e os satelites em órbita, colocados lá pelo conhecimento cientifico, vão transmitir suas chamadas telefonicas até para o Japão, se você assim desejar..:-)

Um abraço.

Homero




----- Original Message -----
From: Amauri Jr
To: ciencialist@yahoogrupos.com.br
Sent: Sunday, January 02, 2005 3:18 PM
Subject: Re: [ciencialist] Evolucao e desafios - CartaCapital - 02/01/05


Minhas analises em baixo com [A]....
----- Original Message -----
From: L.E.R.de Carvalho
To: ciencialist@yahoogrupos.com.br
Sent: Sunday, January 02, 2005 1:52 PM
Subject: [ciencialist] Evolucao e desafios - CartaCapital - 02/01/05



>LabConsS - www.ufrj.br/consumo
>
>
>
> EVOLUÇÃO E DESAFIOS
>
>
> O País viu inúmeros avanços nas áreas médica e
>tecnológica, mas as carências sociais ainda são sua
>pior mazela
>
>Em uma livraria, olho rapidamente as manchetes das
>revistas expostas. A julgar pelas capas, é pouco
>provável que tenhamos problemas de saúde deste mês em
>diante. Soluções curativas. Remédios extremamente
>eficientes. Vida prolongada. Corpos e rostos sem um
>defeito. Independentemente da idade, claro. Câncer?
>Uma brincadeira. Paralisia? Pode preparar as pistas.
>Infarto? E daí?

[A] temos que pagar e muito caro para a "ciencia" nos curar de males que diz a "ciencia" um direito de todos.

>
>Deixando de lado uma razoável dose de exagero nessas
>manchetes, encho o peito de orgulho pelas proezas
>atingidas no campo da medicina e da saúde. Vinte anos
>atrás, tudo isso não passaria de sonho, e não
>existiria fora da cabeça imaginativa dos produtores de
>filmes de ficção científica. Mas, ainda hoje, para a
>esmagadora maioria dos cidadãos brasileiros, todo o
>acima mencionado não passa de um sonho, de uma ficção
>científica

{A] Parece que todo a ciencia é uma ficção cientifica, vamos dizer que o pensamento é um comercio hoje em dia, ficçao ou não, é com certeza sensacionalismo.

>Este ano que acaba nos deu boas notícias,
>encorajadoras, e notícias ruins, no mínimo
>preocupantes. Vimos uma senhora que sofreu um derrame
>cerebral voltar a andar com terapia baseada na
>introdução de células-tronco (células primitivas
>capazes de se transformar em praticamente qualquer
>outra célula normal) no cérebro afetado. Emocionante
>perceber o que esses pequenos passos podem significar
>para milhões de pessoas que não conseguem erguer o
>braço, ou mexer a perna. Parafraseando Neil Armstrong,
>um passo gigante para a humanidade.

{A} Armstrong, quem é ele? Aquele que "supostamente" foi a Lua? Celulas- tronco é um desafio para a humanidade de cura, o desejo de ajudar seu semelhante. Pensamentos inovadores tem estado junto a humanidade a muito tempo, isso pode ser descrito na historia de Tales de Mileto até Einstein




>A técnica das células-tronco não se restringe a
>restaurar a função de células cerebrais. Hoje em dia,
>pesquisadores brasileiros, e em outros países,
>intensificam seus esforços para melhorar o coração
>depois de infarto, os nervos após lesão traumática, a
>pele após queimadura. A lista parece não ter fim.

[A] A demora faz o crescimento dos beneficiados...
>
>Vimos a introdução de remédios geniais na prática
>médica. Geniais na sua concepção, e geniais na sua
>eficiência. Drogas que conseguem agir em um ponto
>específico da célula doente, da célula cancerosa,
>dificultando seu desenvolvimento, seu crescimento, e
>até provocando sua morte. Exemplos que estão
>progressivamente sendo utilizados na prática médica,
>apesar de somente em casos muito selecionados, não
>faltam. Glivec, Iressa, são alguns deles.

[A] Santa ciencia!! Mata e de forma covarde porque usa seus conhecimentos de forma de interesse, mesmo que sabemos, esses conhecimentos são para todos.
>
>Ainda nem acabou o ano e ouvimos há poucas semanas o
>anúncio pela GlaxoSmithKline de uma vacina contra o
>câncer. O Brasil está totalmente empenhado na produção
>da vacina contra o HPV, vírus causador de câncer de
>útero. O prof. dr. Ricardo Brentani, presidente da
>Fundação Antonio Prudente, Hospital do Câncer AC
>Camargo, está entusiasmado e declara:
>
>­ Felizmente nossa instituição foi parceira da Merck
>Sharp & Dohme no desenvolvimento de uma vacina contra
>o HPV. Minha esperança é que em 20 anos tenhamos
>prevenido 7% dos tumores humanos.

[A] Vacina? Como pode ter vacina para nossas proprias celulas? E do HIV vai ter?

>Vimos, por outro lado, a preocupação crescente dos
>cientistas em não excluir, a priori, tratamentos
>considerados até então não-convencionais, ou
>complementares. Estudos sérios avaliaram desde a
>acupuntura até a homeopatia, passando por terapias de
>Florais de Bach e tratamentos com vitaminas nas mais
>diversas situações clínicas. Vantagens e desvantagens
>de cada abordagem foram dissecadas minuciosamente.

[A] Viu? Sera que alguem aqui duvidda?


>Cientistas conseguiram demonstrar a eficiência
>incontestável de algumas terapias alternativas em
>certas situações, como a massagem para um dos males do
>século, a dor nas costas. Por outro lado, alertaram
>para a ineficiência e até os efeitos nocivos de outras
>terapias, em outras condições. Por exemplo, para
>tratar bronquite e asma a acupuntura não parece ter
>efeitos importantes. Pelo menos não conseguiram
>detectar esses efeitos nos estudos atuais.

[A] Remedios sim? Eu tive começo, eu e meus manos, de bronquite depois que meu pai comprou um tartaruga nunca mais...santa crença ne? Mas como explicar??

>Vimos o lançamento de aparelhos ultramodernos,
>ultra-sensíveis, para detectar doenças e tratá-las.
>Não consigo perceber avanço recente maior do que na
>área da radioterapia. É incrível a precisão dos feixes
>de radiação em atingir o alvo, no caso o câncer, e
>poupar o tecido normal adjacente. Aparelhos novos
>conseguem acompanhar o movimento do corpo para seguir
>o alvo predeterminado, segundo a segundo, obedecendo à
>orientação do médico radioterapeuta. Reduziram-se
>muito os efeitos colaterais. O controle do câncer
>assemelha-se às extensas cirurgias. A nanotecnologia
>(aparelhos miniaturas) é uma febre, e os estudos
>multiplicam-se para definir com mais clareza sua
>aplicação.

{A} Pode matar com sua radioatividade, causando cancer.

>
>A plástica está fazendo tamanhos milagres que nem os
>próprios pacientes conseguem acreditar. Tanto faz quem
>foram seus pais, ou a etnia a qual você pertença.
>Escolha o modelo e ficará parecido. Para pessoas
>ansiosas com a imagem, soluções para quase tudo. Sem
>dúvida, o impacto sobre o estado emocional é notável.

[A] Eita faquinha de dois cumes heim? A plastica pode curar a pessoa de queimaduras ou cicatrizes, mas tambem pode ressaltar sua vaidade.

>
>Ao lado dessas notícias que mereceram um destaque
>quase obsessivo nas manchetes de capa, outras
>informações com menos, digamos, glamour, não
>conseguiram espaço nem nas páginas finais das
>revistas.
[A] A ciencia ja tem a imprensa e seus colaborados para dar esse glamour...


>
>O objetivo de reduzir em 60% a mortalidade infantil no
>mundo (e o Brasil ainda é um grande protagonista dessa
>estatística) não será atingido em 2015, como
>estabelecido nos Objetivos Milenares da ONU. No
>planeta, morrem por ano mais de 11 milhões de crianças
>com idade inferior a 5 anos. A maioria por doenças
>evitáveis. Diarréia, pneumonia, malária. No mesmo
>período, 500 mil mulheres morrem durante a gravidez ou
>o parto. Doença de Chagas e esquistossomose
>(barriga-d'água) continuam afetando milhares de
>brasileiros. E seu controle está cada vez mais
>próximo. A implementação dos programas já em ação
>poderá melhorar ainda mais esse controle. Para 2005 a
>intensificação dessas abordagens poderá elevar o
>impacto na saúde da população, principalmente nas
>áreas rurais.

[A] Isso ai, como fica doenças tao mais antiga diante do deusa peerfeita ciencia??


>O Relatório Mundial de Saúde (The World Health Report
>2003) recomendou às autoridades o fortalecimento dos
>sistemas de saúde, centralizando seu foco na atenção
>primária, além de integrar a prevenção das doenças e a
>promoção da saúde em todos os níveis de atendimento.

[A] Esses relatorios tem efeito??

>
>A saúde do homem sofreu mudanças drásticas nos últimos
>anos. A expectativa de vida, de brasileiros e de não
>brasileiros, bate recorde atrás de recorde.
>Ultrapassou a marca dos 70 anos e logo passará dos 80,
>90, e quem sabe 100 anos. Avanços sem dúvida notáveis.
>Doenças contagiosas foram substituídas por doenças
>crônicas, como problemas cardiovasculares e câncer, e
>por causas externas, como trauma. Essa mudança de foco
>exigiu modificações intensivas no nível estrutural.
>Algumas especialidades assumiram posição de destaque
>nunca antes alcançada. Os serviços de cardiologia de
>muitos hospitais logo se transformaram em
>departamentos e, a seguir, em grandes centros com
>prédios próprios. O mesmo ocorreu com a oncologia e
>suas variadas especialidades. Houve uma explosão na
>tecnologia e na sofisticação, e, conseqüentemente, nos
>custos ­ alertam os especialistas em saúde pública.
>Recomenda-se também a melhora da saúde da população
>por intermédio do envolvimento acadêmico.
>

[A] Sem comentarios, pra que vou querer viver até o 100?
>
>Muito por fazer.
>Como melhorar o atendimento ao público do SUS

[A] Infelizmente a ciencia não melhora ela mesma vai melhorar o SUS, a ciencia infelizmente, anda atendendo interesses dos laboratorios e pondo em pratica, o que determina o codigo americano, Bisness in bisness...
>
>
>A academia médica no Brasil foi alterada sensivelmente
>nas últimas duas décadas. Progressivamente, as escolas
>médicas e os programas de pós-graduação introduziram
>disciplinas de pesquisa epidemiológica de problemas de
>saúde adaptados à realidade da população do País. Mais
>e mais livros têm sido publicados por acadêmicos
>brasileiros, com enfoque nacional. Mais e mais centros
>de treinamento oferecem vagas para médicos
>interessados em se aperfeiçoar no atendimento básico à
>saúde da família e à clínica geral. O impacto na
>melhora da atenção à saúde do brasileiro pode demorar
>a ser notado. Muito há de ser feito em 2005 e além.

[A] Infelizmente, tem muito moleque fazendo medicina pro papai e só sai meleca, ou o diagnostico é virose ou é exames a "toa"...


>O plano Fome Zero tenta corrigir um problema básico de
>saúde: a miséria e a conseqüente desnutrição. Ninguém
>precisa de doutorado para compreender a conexão direta
>entre a desnutrição e a ocorrência de doenças
>potencialmente graves. É um plano interessante, mas
>ainda longe de atingir seus objetivos de forma
>significativa. Em 2005, e além, o governo deverá fazer
>os ajustes finos para que o programa consiga alcançar
>seu alvo.
>
>A violência é problema de segurança pública, mas
>também é problema de saúde. Não podemos esquecer que,
>no ano passado, mais de 40 mil brasileiros foram
>assassinados. Se acreditarmos nos dados oficiais do
>Ministério da Saúde, naquele período, morreram mais
>brasileiros por tiro do que por câncer de pulmão. O
>controle da violência evitará milhares de mortes e
>seqüelas dramáticas.
>
>A malária é outro problema sem solução a curto prazo.
>Áreas extensas do território nacional são infestadas
>por mosquitos portadores dessa doença. O esforço das
>autoridades regionais e federais deverá ser
>intensificado nas várias frentes: pesquisa de vacinas,
>profilaxia, controle do mosquito e tratamento de
>pacientes infectados. A mortalidade por essa infecção
>deve ser reduzida a todo custo. Estudos recentes
>aventam que a vacina contra a malária pode estar ao
>alcance da ciência em prazo razoável.
>
>No mundo todo, um bilhão de pessoas são infectadas por
>parasitas, como os vermes, e o Brasil tem participação
>significativa nesses dados alarmantes. Vermes no
>intestino não são apenas feios. Eles causam problemas
>como deficiências nutricionais, e, em alguns casos,
>podem até ser fatais. Para mudar isso, saneamento
>básico é fundamental.
>
>Um estudo recentemente realizado e publicado por
>pesquisadores na Universidade do Ceará demonstrou
>claramente a possibilidade de praticamente eliminar os
>parasitas dos pacientes. Remédios eficazes, como a
>ivemerctina, podem ser a solução. Além de tratar o
>paciente, eliminam uma potencial fonte de contaminação
>para outras pessoas. As autoridades devem criar
>sistemas integrados, contínuos, para atacar esse
>problema de saúde pública em várias frentes, em 2005 e
>para sempre.
>
>Quanto à Aids, nos últimos anos, houve um declínio da
>incidência de novos casos na maioria dos estados
>brasileiros. Estudo publicado recentemente por
>pesquisadores da Fundação Oswaldo Cruz, no Rio de
>Janeiro, confirmou que, apesar de recursos limitados
>do governo e da desigualdade socioeconômica aberrante
>no Brasil, a introdução do acesso universal à terapia
>antiviral contribuiu para uma redução impressionante
>na mortalidade por Aids, e pode ter ajudado a diminuir
>sua incidência.
>
>Nessa linha de pensamento, o presidente Lula lançou,
>este ano, o louvável Plano Nacional de Eliminação da
>Hanseníase (a lepra) em até dois anos. Um passo sem
>dúvida importante para controlar essa doença
>contagiosa e debilitante. Paralelamente, o ministro da
>Saúde, Humberto Costa, anunciou a formação de uma
>comissão para promover políticas nacionais de
>reabilitação às pessoas afetadas pela hanseníase:
>
>­ A idéia é possibilitar benefícios, como a
>reabilitação por cirurgias plásticas, financiando para
>a capacitação de médicos, enfermeiros e
>fisioterapeutas para que realizem esses procedimentos
>nos incapacitados fisicamente.
>
>Todos os anos, 42 mil brasileiros contraem a
>hanseníase. E o País só perde para a Índia no ranking
>mundial de casos da doença.
>
>A obesidade, mal do século XXI, mata. Nos EUA, em
>alguns estados o número de óbitos por obesidade
>ultrapassou o de câncer. No Brasil, cientistas
>identificaram obesidade crescente na população. Estudo
>publicado por pesquisadores da Universidade Federal do
>Rio de Janeiro confirmou o aumento progressivo do peso
>dos adolescentes, tanto na Região Nordeste quanto no
>Sudeste do País, desde 1975. Atualmente, 17% dos
>adolescentes da Região Sudeste enquadram-se na
>definição internacional de obesidade. E seus efeitos
>maléficos logo alcançarão a saúde pública. Orientação
>e conscientização devem ser política contínua e bem
>estruturada. A prevenção e o tratamento do sobrepeso
>precisam ser prioridades das políticas de saúde nos
>próximos anos.
>
>Um problema que acomete principalmente as crianças é a
>poluição atmosférica. Um estudo realizado com 5.193
>crianças de duas cidades do Rio de Janeiro, e
>publicado recentemente na revista Annals of Allergy
>Asthma and Immunology, mostrou claramente a correlação
>entre os níveis de poluição e a incidência de
>problemas respiratórios. Observou-se um aumento de
>mais de 50% na freqüência de crises de asma nos
>moradores de Duque de Caxias (local com alta
>concentração de poluentes), comparados aos moradores
>de Seropédica (local com baixos níveis de poluição).
>Outro estudo, realizado pelos pesquisadores do
>laboratório de poluição atmosférica da Universidade de
>São Paulo, confirmou os efeitos nocivos da poluição do
>ar na mortalidade infantil. Há aumento de 6% no número
>de óbitos neonatais nas regiões mais poluídas. O
>controle de poluentes de qualquer origem deverá ser
>obrigação mundial. Ainda mais com a entrada em vigor
>do Protocolo de Kyoto.
>
>Mas, de forma geral, provavelmente o maior desafio das
>autoridades de saúde é garantir um apoio ao avanço
>científico na área de saúde, atingir a maioria da
>população com os avanços médicos e tecnológicos
>alcançados, e insistir na prevenção e na saúde
>primária, básica. Muito mais do que dinheiro, em 2005
>precisamos de estratégia, filosofia e enfoque novos.
>
>O acesso da população, principalmente das camadas
>menos privilegiadas da sociedade, a remédios em geral
>é muito limitado. Várias são as causas, como preços
>elevados na fonte, acréscimos excessivos na cadeia de
>distribuição e impostos. Se as autoridades, de todas
>as esferas, não atentarem a essas dificuldades
>enfrentadas no dia-a-dia da maioria dos cidadãos,
>muito mais será gasto para tratar complicações graves
>de doenças crônicas, como diabetes e hipertensão
>arterial. O Brasil iniciou uma ação interessante nessa
>direção: a aquisição da fábrica da GlaxoSmithKline do
>Brasil pelo governo federal. O presidente Lula
>destacou que "pela primeira vez um governo compra uma
>fábrica da iniciativa privada", ao mesmo tempo
>criticando "o processo inverso de privatização do
>governo anterior". Lula declarou, na ocasião, que
>estava "recuperando uma fábrica que seria desativada,
>eliminaria empregos e que agora produzirá, em escala
>nacional, os antibióticos mais usados no Brasil".
>
>Nessa mesma linha o ministro da Saúde, Humberto Costa,
>deixou muito clara sua estratégia para 2005:
>
>­ Vamos ampliar o acesso da população a medicamentos
>fundamentais com base em três opções. A primeira é a
>rede de farmácias populares, e até o fim do ano
>pretendemos inaugurar cem unidades em todo o Brasil. A
>segunda é a criação de um programa para a venda
>subsidiada de medicamentos básicos para hipertensão e
>diabetes, através da rede de farmácias privada.
>Pretendemos baixar os preços em até 50%. E a terceira
>é a redução do ICMS de 2,8 mil medicamentos até 2005.
>
>De acordo com uma pesquisa do IBGE, de 2003, a saúde
>aparece em terceiro lugar no orçamento das famílias
>brasileiras, e os medicamentos representam 61% desses
>gastos para as pessoas de baixa renda. Além disso,
>metade das pessoas que precisam de tratamento não pode
>pagar os remédios de que necessitam.
>
>A partir de 2005, 50 milhões de unidades de
>antibióticos serão produzidas para a rede do Sistema
>Único de Saúde (SUS) e para as Farmácias Populares. Em
>2007, a produção deve quintuplicar em relação à atual.
>Mais de 10 bilhões de unidades de medicamentos devem
>ser produzidas para as principais doenças que mais
>atingem a população brasileira, como hipertensão,
>diabetes, malária e tuberculose. A Fiocruz também
>produzirá mais vacinas, e o Ministério da Saúde
>enfatiza que "a ampliação do acesso da população aos
>medicamentos é uma das prioridades do governo
>federal".
>
>Em 2004, muito se fez pela saúde, mas ainda há muito
>mais por fazer. Programas de prevenção e detecção
>precoce de doenças sexualmente transmissíveis, como
>Aids, HPV (câncer de colo de útero), a melhoria na
>qualidade do atendimento dos pacientes do SUS,
>diminuição das filas, controle de poluição, de
>infecção hospitalar e, também, de mortes por agentes
>externos, como traumas (tiros, facadas, acidentes de
>trânsito). Para se ter uma idéia do custo com os
>acidentes de trânsito, a cada ano acontecem mais de
>100 mil mortes, com três a quatro vezes esse número de
>feridos. E a maioria absoluta é tratada com dinheiro
>público. São milhões de reais de gastos evitáveis. Sem
>falar nas seqüelas individuais.
>

Abraços
Amauri

[As partes desta mensagem que não continham texto foram removidas]



##### ##### #####

Para saber mais visite
http://www.ciencialist.hpg.ig.com.br


##### ##### ##### #####



Yahoo! Grupos, um serviço oferecido por:
PUBLICIDADE




------------------------------------------------------------------------------
Links do Yahoo! Grupos

a.. Para visitar o site do seu grupo na web, acesse:
http://br.groups.yahoo.com/group/ciencialist/

b.. Para sair deste grupo, envie um e-mail para:
ciencialist-unsubscribe@yahoogrupos.com.br

c.. O uso que você faz do Yahoo! Grupos está sujeito aos Termos do Serviço do Yahoo!.



[As partes desta mensagem que não continham texto foram removidas]



##### ##### #####

Para saber mais visite
http://www.ciencialist.hpg.ig.com.br


##### ##### ##### #####


Yahoo! Grupos, um serviço oferecido por:







------------------------------------------------------------------------------
Links do Yahoo! Grupos

a.. Para visitar o site do seu grupo na web, acesse:
http://br.groups.yahoo.com/group/ciencialist/

b.. Para sair deste grupo, envie um e-mail para:
ciencialist-unsubscribe@yahoogrupos.com.br

c.. O uso que você faz do Yahoo! Grupos está sujeito aos Termos do Serviço do Yahoo!.



[As partes desta mensagem que não continham texto foram removidas]



##### ##### #####

Para saber mais visite
http://www.ciencialist.hpg.ig.com.br


##### ##### ##### #####


Yahoo! Grupos, um serviço oferecido por:







------------------------------------------------------------------------------
Links do Yahoo! Grupos

a.. Para visitar o site do seu grupo na web, acesse:
http://br.groups.yahoo.com/group/ciencialist/

b.. Para sair deste grupo, envie um e-mail para:
ciencialist-unsubscribe@yahoogrupos.com.br

c.. O uso que você faz do Yahoo! Grupos está sujeito aos Termos do Serviço do Yahoo!.



[As partes desta mensagem que não continham texto foram removidas]



##### ##### #####

Para saber mais visite
http://www.ciencialist.hpg.ig.com.br


##### ##### ##### #####


Yahoo! Grupos, um serviço oferecido por:







------------------------------------------------------------------------------
Links do Yahoo! Grupos

a.. Para visitar o site do seu grupo na web, acesse:
http://br.groups.yahoo.com/group/ciencialist/

b.. Para sair deste grupo, envie um e-mail para:
ciencialist-unsubscribe@yahoogrupos.com.br

c.. O uso que você faz do Yahoo! Grupos está sujeito aos Termos do Serviço do Yahoo!.



[As partes desta mensagem que não continham texto foram removidas]



SUBJECT: Re: [ciencialist] Evolucao e desafios -Amaury
FROM: "JVictor" <jvoneto@uol.com.br>
TO: <ciencialist@yahoogrupos.com.br>
DATE: 03/01/2005 20:39

Grande Homero,

Bela, pertinente e oportuna linha de argumentação. É por aí.
Quanto a decidir, no caso de uma guerra com a Argentina, nas condições que você colocou, não tenho dúvidas de que qualquer brasileiro que estivesse com o poder nas mãos, faria a opção igualmente horrorosa: acabar com a guerra, sacrificando alguns para salvar muitos milhares a mais. Seria a única opção racional. Ainda que o batedor do martelo vivesse o resto da vida correndo de psiquiatra em psiquiatra.
Felizmente isto nunca acontecerá conosco. Coisas assim só acontecem com outros países!...
O teu texto é um mais a figurar no meu Favoritos.

Sds,

Victor.

----- Original Message -----
From: Oraculo
To: ciencialist@yahoogrupos.com.br
Sent: Monday, January 03, 2005 7:47 PM
Subject: Re: [ciencialist] Evolucao e desafios -Amaury


Olá Amaury

Fico contente que goste dos debates e de minha participação (nem sempre é assim, as vezes o pessoal se irrita bastante comigo:-)

Mas ainda acho que está confundindo duas coisas, a ciência, abstrata, e o comportamento humano, passível de ser julgado e avaliado subjetivamente.

É claro que é importante discutir a aplicação, direção, prioridade, etc, de pesquisas e do conhecimento humano. Mas, a ciência, é apenas um conjunto de conhecimentos adquirido de certa forma (método cientifico), não seu uso ou resultado. Isso está confundindo a discussão..:-)

A questao é, se decide agir com eficácia, tomar uma decisão que se mostre confiável no resultado, ou escolher uma afirmação ou conclusão que se aproxime o máximo possível da realidade (sem pretensões de ser 100% ou "A VERDADE"), o que escolheria? Seja para curar pessoas, seja para matar inimigos, o que consideraria mais confiável, mais real ou mais eficaz?

Sua resposa, que certamente é igual a minha, coloca a perspectiva correta na discussão..:-) Ciência, abstratamente, é a melhor ferramenta de compreensão deste universo (universo fisico) que dispomos. O que faremos com o conhecimento assim obtido, é outra coisa. Eu, particularmente, o usaria para melhorar a vida de todo mundo (por exemplo, usando o produto desse conhecimento para enviar ajuda as vitimas do tsunami na Asia). Outros a usariam para matar ou criar armas de destruição em massa.

Mas culpar a ciência, um conhecimento abstrato, pelo uso que malucos fazem dela, é irrelevante e impróprio. Me lembra a piada do matador que, perguntado se não tinah remorso, respondeu: Eu não mato ninguem, só faço o furo, quem mata é Deus (no que ele tem razão, de uma perspectiva religiosa do universo..:-)

Pessoas, seres humanos, matam e maltratam. E fazem isso muito antes de termos armas de destruição em massa ou equivalentes. Culpar o conhecimento do átomo pelo uso da bomba atomica seria como condenar a arma no lugar do atirador (imagino que seria um processo muito engraçado de acompanhar..:-)

Veja este trecho seu:

Amaury: Por isso, nada tenho contra ou a favor da ciência, apenas é uma visão critica sobre...:-)

Se fosse assim, nem haveria a discussão entre nós..:-) Mas seu posicionamento inicial era diferente, contrário a ciência, como se fosse ela responsável por seu uso. Por isso fiz minha análise de sua análise..:-)

A confusão fica mais clara neste trecho:

Amaury:O que temos de melhor, como disse, esta nas mãos dos poderosos, portanto "o que temos de melhor" foi e esta sendo mandado pelo governo dos poderosos. Hoje os norte-americanos mandam e desmandam no que "temos de melhor", tanto se é para o bem da humanidade é imoral, mas se beneficia as partes bélicas deles, seguem numa boa...:-)


Neste caso, o termo "melhor" tem duas conotações, que foram misturadas. O que temos de melhor, no meu sentido, é de mais eficaz. Dá resultados. No seu uso, é um julgamento subjetivo, melhor, bom, correto, etc. Não tem ligação com meu argumento. A ciência é "melhor" se deseja uma ação eficaz, ou um conhecimento confiável. O que fará com esse conhecimento, se algo "melhor", do bem, ou algo "pior", do mal, não é relevante para o argumento.

Já a erradicação da malaria foi um erro, ato falho, meu..:-) Pensei em variola e escrevi malária (acho que passava um especial na TV sobre a malária..:-). Mas, mesmo a malária é mais controlada em paises civilizados, com alto uso de conhecimento cientifico, que nos pobres e sem esse uso. Ainda é um bom argumento sobre a eficácia da mesma..:-)

Finalmente, acho que não compreendeu a questao a emissão de radio e a ida à Lua. Não importa o poder dos USA, nào importa seu governo ou o carater (ou falta de) dos seus governantes, mesmo hoje, se emitir uma onda de rádio, quaisquer dois radio amadores podem dizer, precisamente, de onde vem. Isso se chama triangulação, e é o motivo de se gastar tanto tempo e dinheiro em desenvolvimento de aviões invisiveis, já que, se o aparelho voador reflete a onda do radar, pode ser posicionado precisamente, sem sombra de duvida.

Explicando melhor, se um satélite emite sinal de radio, você, com seu radio amador (ou radar avançado, não importa.:-), pode determinar a direção com precisão. Se outro amigo seu radio amador também o fizer, os dois podem combinar os dados e dizer, com exatidão, de onde parte o sinal e exatamente onde está o satelite. Não há forma de fraudar isso, nem de disfarçar ou fingir estar onde não está. Assim, quando as Apollo orbitavam a Terra, foram acompanhadas de perto por todos que desejassem, sem forma de fraudar isso. Quando orbitaram a Lua, foram acompanhadas (com frustação crescente dos russos..:-), por toda parte.

E ao pousar, e enviar dados do solo lunar, idem, sem possibilidade de fraudar essa emisão.

Assim, se deseja discordar do pouso na Lua, tem de explicar como isso foi feito, ou vai esbarrar em algo chamado evidencia acachapante (que colocou fim nas especulações dos russos, chineses e todos os muitos inimigos dos USA durante a guerra fria que adorariam desmascarar a fraude). Alias, quando os russos colocaram o primeiro satélite, Sputinik, em órbita, alguns jornais americanos e alguns membros do governo americano tentaram vender a idéia de fraude, mas o fato que qualquer um com um bom rádio podia confirmar o feito, foi o bastante para convencer as pessoas.:-)

HIV

Lutamos a séculos com o virus da gripe e ainda não podemos mata-lo. Sua taxa de mutação, como costuma acontecer com virus, é brutal, e sua adaptação ciclica impede que seja feita uma vacina para todas as cepas. A simplicidade de determinados virus é a chave para que escapem tão facilmente da cura. Um organismo complexo tem estruturas que se repetem em todas as suas variações. Em algum momento encontramos essa estrutura e uma vacina é criada. Mas, organismos simples, em especial virus, mudam tão completamente, que uma estrutura, uma chave, única, não acontece. Isso torna bem dificil derrota-lo. Além disso, a simplicidade faz com que muitas estruturas sejam também usadas por células do organismo hospedeiro. Assim, algumas das drogas que efetivametne matam os virus da gripe não podem ser aplicadas a seres humanos, pois matam suas células saudáveis também (esse é um dos problemas com drogas anti-cancer também atingir células saudaveis juntamente as doentes).

Talvez as industrias e governos não invistam o suficiente na pesquisa, eu tendo até a concordar com você nisso. Mas isso indica duas coisas: primeiro, que é caro e dificil descobrir a cura, ou ela já teria sido descoberta mesmo com pouco dinheiro. Segundo, indica que temos FALTA de ciência, que seria possivel com mais recuros, e não exesso. Em qualquer dos casos a crítica feita não se aplica. Precisamos, se queremos encontrar a cura ou tratamentos melhores para a AIDS, de mais ciência, um conheimento confiável e eficaz, e não de menos.

Espectativas

Amaury:Não tenho nada contra a ciência, mas você tem contra os índios...:) o que vejo é ainda crianças mortas e vidas curtas por uma ciência ineficaz, se eu gosto ou não é outra historia, que se eu não gosta-se da ciência não estaria numa lista de ciências...:) e pouco irei a uma tribo, porque não iria acostumar...:)e uma pergunta: a ciência tem vida própria?

A ciência não prometeu curar todas as doenças, consertar o mundo, ser 100% eficaz ou definir A VERDADE (para isso temos as religiòes..:-). Ela é apenas uma ferramenta, uma boa e eficaz ferramenta. Talvez esteja esperando demais da ciência e a frustação de não ter todas as respostas o tenha tornado um pouco rancoroso com ela. Entre duas afirmações, entre duas respostas, a obtida de forma cientifica é a mais confiável, só isso. Existe, sim, uma luta de pesquisadores, seres humanos, para usar essa ferramenta de forma a curar doenças, encontrar respostas, criar conforto, resolver problemas, etc. Mas é uma busca, não um final em sí mesmo.

Tenho as mesmas objeções que você às industrias farmaceuticas (inclusive as homeopaticas..:-). Mas sei que existem pesquisadores legitimamente dedicados a encontrar curas, soluções, respostas, apesar das empresas e até mesmo dentro dessas empresas. Gente que tem legitimos interesses humanos, que pesquisa até mesmo as "doenças de paises pobres" como a malária e febre amarela (erradicadas nos paises desenvlvidos).

Bush é uma "anta" fundamentalista. Mas observe que é uma das pessoas mais contra a ciência que poderiamos encontrar. Seu esforço fundamentalista para obrigar sua visão religiosa de mundo a todo o resto do planeta é frontalmente contrária a ciência, tolerante por natureza. Impedir o estudo de células tronco pode impedir avanços que poderiam curar diversas doenças, inclusive paraplegia. Ensinar criacionismo em escolas e banir a evolução (e a base da biologia moderna hoje) impediria até mesmo que bons médicos se formem ou que novas descobertas sejam feitas.

Ainda assim, ele usa os resultados da ciência onde lhe convém, com armas de destruição e brinquedos tecnologicos que poder. Isso torna a ciência culpada? Ou torna o Bush responsável?

Hiroshima é um caso complicado. Nagazaki seria mais claro, como alvo do debate, desnecessário e violento. Mas Hiroshima é dificil. Antes que "caiam de pau", acho um horror a destruição de uma cidade, a morte de civis e qualquer tipo de conflito. Mas é preciso pensar, sempre, sob pena de não ver com clareza. E Hiroshima é dificil de enxergar, através do horror da bomba e da matança.

Mas havia uma guerra em curso, isso é claro. Havia um pais, Japão, que entrou na guerra por vontade própria, para expadir fronteiras e massacrar vizinhos (embora a vontade a considerar fosse apenas dos mandatarios do pais na época). Havia um comportamento cultural onde a morte era sublime, o sacrificio honroso e o imperador deus (você morreria por seu deus). Dentro dessa análise, continuar a guerra com o Japão produziria, por estimativa, 100 mil mortes de americanos e mais de 350 mil mortes japonesas (estimativas derivadas das batalhas já ocorridas, da decisão de cada soldado japones de só parar depois de morto e das dificuldades de desembarque em ilhas - as piores batalhas da segunda guerra forma em ilhas do Japão).

Agora, você (quem me lê) deve decidir (pressionado certamente pelos falcões militares) o que fazer. Imagine que estamos em guerra com a Argentina. Você, presidente, tem uma arma recem criada que pode acabar a guerra, devido ao seu poder inimaginável. Detonar essa arma em um cidade vai matar 45 mil pessoas do outro pais (que começou a guerra). Não detonar vai prolongar a guerra por mais alguns meses, a Argentina jamais se renderá e 100 mil brasileiros morrerão, e mais de 350 mil argentinos. O que você decidiria? O que diria as mães e pais de seu pais, quando perguntassem porque não parou a guerra quando podia e escolheu ver morrer mais 100 mil de seus conterraneos?

Deve se lembrar que nunca uma arma assim havia sido detonada, e não se sabia o horror que poderia causar (os primeiros testes no deserto americano usaram soldados para verificar o efeito e a descontaminação foi feita com vassouras de palha e agua corrente.:-(

Eu não sei o que escolheria e não gostaria de ter de decidir algo assim. Imagino que não se pode dormir mais depois de ser submetido a coisas desse tipo. Por isso considero que Nagasaki é mais claro, uma decisão certamente militar, resultado de uma perda de poder civil em um periodo dificil. Mas Hiroshima é mais complicado.

Vou repetir, acho um horror o bombardeio de civis, na verdade, acho qualquer guerra um horror, desnecessária e absurda. Mas não acho que Hiroshima foi apenas maldade, crueldade de monstros, mas uma decisão calculada, dificil e medonha, da qual eu não saberia o que escolher como "menos pior". E hoje temos os USA arrogantes e com jeito de donos do mundo, e nossa antiapatia por Bush nos faz ver tudo com um filtro de asco. Mas na época a guerra dos USA era contra imperios crueis e mais perigosos que a america jamais será (Hitler, e todo o horror que representa) e nào se pode relativizar o que teria acontecido se a Alemanhã e o Japão ganhassem a guerra.

De todo modo, estamos mais perto da concordancia que da discordância..:-) A ciência, como ferramenta, é eficaz, confiável e razoável. Seu uso, que pode ser para o bem ou para o mal (mesmo que o bem e o mal sejam relativos, pergunte ao Bin Laden e ao Bush..:-), é que é passível de discussão.

Um abraço.

Homero






----- Original Message -----
From: Amauri Jr
To: ciencialist@yahoogrupos.com.br
Sent: Monday, January 03, 2005 11:45 AM
Subject: [ciencialist] Evolucao e desafios -Homero


Olá Amury


[A] Olá Homero






"Minhas análises sobre suas análises..:-)"



[A] Sinceramente gosto de suas analises, fico satisfeito de você me responder...:-)





"Você parece estar ressentido com o que chama de "ciência" e a trata como entidade independente, crença de cientistas ou algo a ser adorado ou derrubado. Mas nada disso é ciência, nem o resultados de sua aplicação se confundem com ela. Ciência neste contexto (desta lista e do uso padrão do termo..:-) é apenas o conjunto dos conhecimentos obtidos através de um método padronizado, chamado cientifico, e do rigor derivado do mesmo. E, independente do que pense dela, tem elevada confiabilidade..:-)"



[A] Amigo, o que penso que infelizmente o homem usa seu conhecimento para algo não muito para o "bem" humano ou "mal", a natureza humana já é corrupta. Tudo que é humano tem falhas, pois nada ainda é perfeito, mas a ciências como algo que vem para beneficiar a humanidade tinha que ser impartidaria e sem nenhuma crença ou fanatismo; como vimos muito na lista. Por isso, nada tenho contra ou a favor da ciência, apenas é uma visão critica sobre...:-)



"Não importa se o resultado é passível de ser julgado subjetivamente como "mau" ou mesmo como "bom". Importa apenas que é mais eficiente que outras formas de conhecimento, mais eficaz em suas previsões e mais confiável que outros instrumentos de compreensão do universo já criado por seres humanos. E que será abandonado assim que uma ferramenta mais eficaz seja apresentada..:-) Até lá, é o que temos de melhor."



[A] O que temos de melhor, como disse, esta nas mãos dos poderosos, portanto "o que temos de melhor" foi e esta sendo mandado pelo governo dos poderosos. Hoje os norte-americanos mandam e desmandam no que "temos de melhor", tanto se é para o bem da humanidade é imoral, mas se beneficia as partes bélicas deles, seguem numa boa...:-)





"Podemos analisar filosoficamente, subjetivamente, os resultados, mas não discutir a eficácia. A ciência, o conhecimento assim acumulado, é tão eficiente quando cura doenças, como a erradicação da malária, como quando destrói milhões de vidas, como em Hiroshima. O resultado é passível de julgamento, mas a ação não. Ela é eficaz, nos dois casos. Se precisar salvar alguém ou matar alguém, deve usar a ciência, é mais eficaz sempre..:-)"



[A] Podemos sim, tanto que você já disse que ciência é um conjunto de conhecimentos; mas muitos conhecimentos, pois o que vimos, é um "Clube da Ciência" que vê apenas um dos vários lados do universo, mas temos vários e é ainda um incógnito pelo homem ainda não abrir a cabeça...:-) Agora, se erradicamos a malaria não sei, creio que você não esta lendo jornal, pois a malaria, a febre-amarela, o HIV, o câncer, então matando e a ciência "tudo que temos de melhor" fica ai procurando pelos em baratas. Hiroshima foi um ato terrorista covarde e deprimente, que deveria fazer um julgamento dos cientistas e dos governantes no tribunal de Nuremberg, por danos morais humanos.





"Não gostar dela, ou como você pretende com as análises, critica-la ou despreza-la, não importa em nada para sua eficácia e confiabilidade. Você vai viver mais que todos os seus antepassados, goste ou não..:-) Sua expectativa de vida é de 75 anos, e, não sei sua idade, mas se já passou dos 40 como eu, é mais que seu avô esperava viver ao nascer (em 1900 a expectativa de vida no Brasil era de 33 anos)."





[A] Tenho apenas 28 anos, sem filhos e com uma deficiência física, o que sei que são superfluidades da senhora sagrada ciência. O que pode me interessar eu viver 40 ou 70 anos se nem ela pode prever?





"Na verdade, você usou um computador e a Internet, frutos da ciência, para enviar seus pensamentos e ataques a ciência, ao inves de tentar telepatia, rezas ou mandingas (ou qualquer outra forma de comunicação não cientifica) justamente porque é mais eficaz e confiável que qualquer outra..:-)"



[A] Alguma coisa boa tinha que sair daí né?...:-) o problema é esse, vocês da ciência só tem um foco do assunto, não ve as múltiplas variedades de estudos...:)





"Você não parece gostar de muitos dos aspectso do uso do conhecimento cientifico. Direito seu. E pode até mesmo abandona-los, todos, e ir viver em uma aldeia remota no Amazonas, sem (quase) nenhum contato com a ciência (os índios são muito receptivos a forasteiros, vão gostar de recebe-lo), e com todo o ônus de viver dessa forma (filhos mortos, vida curta, doenças diversas, poucos dentes, etc). Mas está na verdade criticando comportamentos humanos, que são seres falhos como sabe, não a ciência. Esta, "estricto senso", não é boa ou má, cruel ou gentil, apenas eficiente, confiável em seus efeitos."



[A] Não tenho nada contra a ciência, mas você tem contra os índios...:) o que vejo é ainda crianças mortas e vidas curtas por uma ciência ineficaz, se eu gosto ou não é outra historia, que se eu não gosta-se da ciência não estaria numa lista de ciências...:) e pouco irei a uma tribo, porque não iria acostumar...:)e uma pergunta: a ciência tem vida própria?



"Você não "acredita" nela. Engraçado, ela não pede que se acredite em nada.:-) Deve duvidar sempre, até que evidencias se mostrem sólidas o bastante para uma conclusão. E é nesse ponto que você escorrega..:-) O pouso na Lua já tem evidencias suficientes para essa conclusão, em que pese as tolices de diversos malucos mundo afora e suas teorias de conspiração. O HIV tem recuado e matado muito menos que no inicio da epidemia, graças a AZT e os atuais coquetéis antivirais. Se ajustes são feitos a toda hora, e são, sobre o que comer ou não, é apenas porque a ciência não pretende ser "A VERDADE" como religiões, mas um conhecimento que cresce, se aprofunda, se ajusta e melhora. Sabemos mais, não menos sobre o que comer ou não comer."



[A] Engraçado que pessoas inteligentes não duvidem que os yanques tenham forjado tudo, mas como você mesmo disse, é um direito seu. Eu acredito muito mais que se possa acreditar, a ciência melhorou minha vida muito, tendo esse computador; em minha cadeira de rodas de alumínio (de ferro era um lastima andar), portanto não estou desacreditando a ciência. Agora se quer achar que os coquetéis são eficazes tudo bem, mas que morre mesmo assim, isso morre...:)



"Sim, dá trabalho, é preciso atenção e constante leitura para acompanhar, seria melhor ter uma única resposta para todas as perguntas e pronto, sem risco e sem mudanças. Mas o mundo não funciona assim..:-) Sabemos hoje mais do que ontem sobre como se alimentar, e saberemos mais amanhã, deixando parte do que sabemos hoje para trás. (Parte, não tudo:-)"



[A] Como se o mesmo alimento hoje esta com proteínas e tudo que precisamos e amanha não?





"As teorias conspiratórias são atrativas.:-) É como se nós, os que estão "por dentro", soubéssemos de "coisas" que o resto do mundo não sabe, fossemos mais "espertos" ou mais inteligentes, que não se deixam enganar. Mas, na maioria das vezes (eu diria em sua totalidade) é apenas o ego tentando ser mais do que é.:-) Mesmo com centenas de sites "desmistificando" o pouso na Lua, o volume de evidencias a favor é gigantesco e nenhum deles explica um ponto fundamental do problema: como o governo americano fraudou os sinais de radio e TV que eram emitidos de todas as naves Apollo tanto durante as viagens, como do solo lunar, captados por todo radioamador ao redor do globo, sem falar nos radares e antenas dos paises em disputassem os USA? (lembre-se que, mesmo hoje, não se pode fraudar a origem de um sinal de radio..:-)"



[A] Não acredito nesses doentes mentais que ficam montando sites para se promover...:) eu tiro dos fatos que ocorreram que podem ser completamente forjados, como as transmissões, ou não sabe que eles são donos do que "temos de melhor"...:)



"Enfim, a ciência é eficaz. Embora seu uso, o uso do conhecimento confiável por ela produzido, possa ser discutido e julgado, isso não é a ciência nem com ela se confunde. E, tenha certeza, seu automóvel vai conduzi-lo amanhã, confiavelmente, conforme as leis da fisica descobertas pela ciência, seu computador receberá esta mensagem, os remédios na farmacia impedirão que você morra, as vacinas que tomou evitarão que adoeça (como todos os seus ancestrais adoeceram) e os satelites em órbita, colocados lá pelo conhecimento cientifico, vão transmitir suas chamadas telefonicas até para o Japão, se você assim desejar..:-)"



[A] Reverencia a ciência como um crente reverencia uma reza...:) o meu carro, vai bater conforma a lei da física, as farmácias venderão remédios que viciam, os satélites vão espionar minha casa e tudo graças a deusa infalível ciência...:)



"Para completar a mensagem anterior: uma menina inglesa de 15 aos foi a primeira pessoa a sobreviver depois de contrair raiva sem tomar a vacina antirabica. Ela foi submetida a um novo tratamento revolucionário (e científico), com um coquetel de drogas e coma induzido."



[A] É? Legal! Mas não são os igreses que usam a ciência para fazer guerrinhas com o Bush?





"Diferente de milagres divinos e sobrenaturais, o que se aprendeu com a nova técnica pode ser usado em outras pessoas doentes e não afetam apenas a pessoa que recebe a graça..:-)"



[A] Também acho, devo adverti-lo novamente que não sou a favor no milagre...:)




"A partir de agora, mesmo desprezando a "ciência", você pode ficar sossegado, se você ou um de seus filhos ou entes queridos contrair raiva, poderá ser curado, cientificamente curado.:-) A não ser que tenha resolvido ir viver em uma aldeia indigena na remota Amazonia..."



[A] Sim vão...vão tomar coca-cola e comer hambúrguer feitos cientificamente e se eu tiver sorte, vão se viciar em LSD feito pela ciência, vão tomar energéticos feito pela ciência, vão ter colesterol de produtos que o "temos de melhor" analisa. O mundo é um paraíso, graças a ciência não acha? ...:)



Abraços

Amauri


---- Original Message -----
From: Oraculo
To: ciencialist@yahoogrupos.com.br
Sent: Sunday, January 02, 2005 11:05 PM
Subject: Re: [ciencialist] Evolucao e desafios - CartaCapital - 02/01/05


Olá Amury

Minhas análises sobre suas análises..:-)

Você parece estar ressentido com o que chama de "ciência" e a trata como entidade independente, crença de cientistas ou algo a ser adorado ou derrubado. Mas nada disso é ciência, nem o resultados de sua aplicação se confundem com ela. Ciência neste contexto (desta lista e do uso padrão do termo..:-) é apenas o conjunto dos conhecimentos obtidos através de um método padronizado, chamado cientifico, e do rigor derivado do mesmo. E, independente do que pense dela, tem elevada confiabilidade..:-)

Não importa se o resultado é passível de ser julgado subjetivametne como "mau" ou mesmo como "bom". Importa apenas que é mais eficiente que outras formas de conhecimento, mais eficaz em suas previsões e mais confiável que outros instrumentos de compreensão do universo já criado por seres humanos. E que será abandonado assim que uma ferramenta mais eficaz seja apresentada..:-) Até lá, é o que temos de melhor.

Podemos analisar filosoficamente, subjetivamente, os resultados, mas não discutir a eficácia. A ciência, o conhecimento assim acumulado, é tão eficiente quando cura doenças, como a erradicação da malária, como quando destroi milhões de vidas, como em Hiroshima. O resultado é passível de julgamento, mas a ação não. Ela é eficaz, nos dois casos. Se precisar salvar alguém ou matar alguém, deve usar a ciência, é mais eficáz sempre..:-)

Não gostar dela, ou como você pretende com as análises, critica-la ou despreza-la, não importa em nada para sua eficácia e confiabilidade. Você vai viver mais que todos os seus antepassados, goste ou não..:-) Sua expectativa de vida é de 75 anos, e, não sei sua idade, mas se já passou dos 40 como eu, é mais que seu avô esperava viver ao nascer (em 1900 a expectativa de vida no Brasil era de 33 anos).

Na verdade, você usou um computador e a Internet, frutos da ciência, para enviar seus pensamentos e ataques a ciência, ao inves de tentar telepatia, rezas ou mandingas (ou qualquer outra forma de comunicação não cientifica) justamente porque é mais eficaz e confiável que qualquer outra..:-)

Você não parece gostar de muitos dos aspectso do uso do conhecimento cientifico. Direito seu. E pode até mesmo abandona-los, todos, e ir viver em uma aldeia remota no Amazonas, sem (quase) nenhum contato com a ciência (os indios são muito receptivos a forasteiros, vão gostar de recebe-lo), e com todo o onus de viver dessa forma (filhos mortos, vida curta, doenças diversas, poucos dentes, etc). Mas está na verdade criticando comportamentos humanos, que são seres falhos como sabe, não a ciência. Esta, "estricto senso", não é boa ou má, cruel ou gentil, apenas eficiente, confiável em seus efeitos.

Você não "acredita" nela. Engraçado, ela não pede que se acredite em nada.:-) Deve duvidar sempre, até que evidencias se mostrem sólidas o bastante para uma conclusão. E é nesse ponto que você escorrega..:-) O pouso na Lua já tem evidencias suficientes para essa conclusão, em que pese as tolices de diversos malucos mundo afora e suas teorias de conspiração. O HIV tem recuado e matado muito menos que no inicio da epidemia, graças a AZT e os atuais coqueteis antivirais. Se ajustes são feitos a toda hora, e são, sobre o que comer ou não, é apenas porque a ciência não pretende ser "A VERDADE" como religiões, mas um conhecimento que cresce, se aprofunda, se ajusta e melhora. Sabemos mais, não menos sobre o que comer ou não comer.

Sim, dá trabalho, é preciso atenção e constante leitura para acompanhar, seria melhor ter uma única resposta para todas as perguntas e pronto, sem risco e sem mudanças. Mas o mundo não funciona assim..:-) Sabemos hoje mais do que ontem sobre como se alimentar, e saberemos mais amanhã, deixando parte do que sabemos hoje para tras. Parte, não tudo:-)

As teorias conspiratórias são atrativas.:-) É como se nós, os que estão "por dentro", soubessemos de "coisas" que o resto do mundo não sabe, fossemos mais "espertos" ou mais inteligentes, que não se deixam enganar. Mas, na maioria das vezes (eu diria em sua totalidade) é apenas o ego tentando ser mais do que é.:-) Mesmo com centenas de sites "desmistificando" o pouso na Lua, o volume de evidencias a favor é gigantesco e nenhum deles explica um ponto fundamental do problema: como o governo americano fraudou os sinais de radio e TV que eram emitidos de todas as naves Apollo tanto durante as viagens, como do solo lunar, captados por todo radio-amador ao redor do globo, sem falar nos radares e antenas dos paises em disputacom os USA? (lembre-se que, mesmo hoje, não se pode fraudar a origem de um sinal de radio..:-)

Enfim, a ciência é eficaz. Embora seu uso, o uso do conhecimento confiável por ela produzido, possa ser discutido e julgado, isso não é a ciência nem com ela se confunde. E, tenha certeza, seu automóvel vai conduzi-lo amanhã, confiavelmente, conforme as leis da fisica descobertas pela ciência, seu computador receberá esta mensagem, os remédios na farmacia impedirão que você morra, as vacinas que tomou evitarão que adoeça (como todos os seus ancestrais adoeceram) e os satelites em órbita, colocados lá pelo conhecimento cientifico, vão transmitir suas chamadas telefonicas até para o Japão, se você assim desejar..:-)

Um abraço.

Homero




----- Original Message -----
From: Amauri Jr
To: ciencialist@yahoogrupos.com.br
Sent: Sunday, January 02, 2005 3:18 PM
Subject: Re: [ciencialist] Evolucao e desafios - CartaCapital - 02/01/05


Minhas analises em baixo com [A]....
----- Original Message -----
From: L.E.R.de Carvalho
To: ciencialist@yahoogrupos.com.br
Sent: Sunday, January 02, 2005 1:52 PM
Subject: [ciencialist] Evolucao e desafios - CartaCapital - 02/01/05



>LabConsS - www.ufrj.br/consumo
>
>
>
> EVOLUÇÃO E DESAFIOS
>
>
> O País viu inúmeros avanços nas áreas médica e
>tecnológica, mas as carências sociais ainda são sua
>pior mazela
>
>Em uma livraria, olho rapidamente as manchetes das
>revistas expostas. A julgar pelas capas, é pouco
>provável que tenhamos problemas de saúde deste mês em
>diante. Soluções curativas. Remédios extremamente
>eficientes. Vida prolongada. Corpos e rostos sem um
>defeito. Independentemente da idade, claro. Câncer?
>Uma brincadeira. Paralisia? Pode preparar as pistas.
>Infarto? E daí?

[A] temos que pagar e muito caro para a "ciencia" nos curar de males que diz a "ciencia" um direito de todos.

>
>Deixando de lado uma razoável dose de exagero nessas
>manchetes, encho o peito de orgulho pelas proezas
>atingidas no campo da medicina e da saúde. Vinte anos
>atrás, tudo isso não passaria de sonho, e não
>existiria fora da cabeça imaginativa dos produtores de
>filmes de ficção científica. Mas, ainda hoje, para a
>esmagadora maioria dos cidadãos brasileiros, todo o
>acima mencionado não passa de um sonho, de uma ficção
>científica

{A] Parece que todo a ciencia é uma ficção cientifica, vamos dizer que o pensamento é um comercio hoje em dia, ficçao ou não, é com certeza sensacionalismo.

>Este ano que acaba nos deu boas notícias,
>encorajadoras, e notícias ruins, no mínimo
>preocupantes. Vimos uma senhora que sofreu um derrame
>cerebral voltar a andar com terapia baseada na
>introdução de células-tronco (células primitivas
>capazes de se transformar em praticamente qualquer
>outra célula normal) no cérebro afetado. Emocionante
>perceber o que esses pequenos passos podem significar
>para milhões de pessoas que não conseguem erguer o
>braço, ou mexer a perna. Parafraseando Neil Armstrong,
>um passo gigante para a humanidade.

{A} Armstrong, quem é ele? Aquele que "supostamente" foi a Lua? Celulas- tronco é um desafio para a humanidade de cura, o desejo de ajudar seu semelhante. Pensamentos inovadores tem estado junto a humanidade a muito tempo, isso pode ser descrito na historia de Tales de Mileto até Einstein




>A técnica das células-tronco não se restringe a
>restaurar a função de células cerebrais. Hoje em dia,
>pesquisadores brasileiros, e em outros países,
>intensificam seus esforços para melhorar o coração
>depois de infarto, os nervos após lesão traumática, a
>pele após queimadura. A lista parece não ter fim.

[A] A demora faz o crescimento dos beneficiados...
>
>Vimos a introdução de remédios geniais na prática
>médica. Geniais na sua concepção, e geniais na sua
>eficiência. Drogas que conseguem agir em um ponto
>específico da célula doente, da célula cancerosa,
>dificultando seu desenvolvimento, seu crescimento, e
>até provocando sua morte. Exemplos que estão
>progressivamente sendo utilizados na prática médica,
>apesar de somente em casos muito selecionados, não
>faltam. Glivec, Iressa, são alguns deles.

[A] Santa ciencia!! Mata e de forma covarde porque usa seus conhecimentos de forma de interesse, mesmo que sabemos, esses conhecimentos são para todos.
>
>Ainda nem acabou o ano e ouvimos há poucas semanas o
>anúncio pela GlaxoSmithKline de uma vacina contra o
>câncer. O Brasil está totalmente empenhado na produção
>da vacina contra o HPV, vírus causador de câncer de
>útero. O prof. dr. Ricardo Brentani, presidente da
>Fundação Antonio Prudente, Hospital do Câncer AC
>Camargo, está entusiasmado e declara:
>
>­ Felizmente nossa instituição foi parceira da Merck
>Sharp & Dohme no desenvolvimento de uma vacina contra
>o HPV. Minha esperança é que em 20 anos tenhamos
>prevenido 7% dos tumores humanos.

[A] Vacina? Como pode ter vacina para nossas proprias celulas? E do HIV vai ter?

>Vimos, por outro lado, a preocupação crescente dos
>cientistas em não excluir, a priori, tratamentos
>considerados até então não-convencionais, ou
>complementares. Estudos sérios avaliaram desde a
>acupuntura até a homeopatia, passando por terapias de
>Florais de Bach e tratamentos com vitaminas nas mais
>diversas situações clínicas. Vantagens e desvantagens
>de cada abordagem foram dissecadas minuciosamente.

[A] Viu? Sera que alguem aqui duvidda?


>Cientistas conseguiram demonstrar a eficiência
>incontestável de algumas terapias alternativas em
>certas situações, como a massagem para um dos males do
>século, a dor nas costas. Por outro lado, alertaram
>para a ineficiência e até os efeitos nocivos de outras
>terapias, em outras condições. Por exemplo, para
>tratar bronquite e asma a acupuntura não parece ter
>efeitos importantes. Pelo menos não conseguiram
>detectar esses efeitos nos estudos atuais.

[A] Remedios sim? Eu tive começo, eu e meus manos, de bronquite depois que meu pai comprou um tartaruga nunca mais...santa crença ne? Mas como explicar??

>Vimos o lançamento de aparelhos ultramodernos,
>ultra-sensíveis, para detectar doenças e tratá-las.
>Não consigo perceber avanço recente maior do que na
>área da radioterapia. É incrível a precisão dos feixes
>de radiação em atingir o alvo, no caso o câncer, e
>poupar o tecido normal adjacente. Aparelhos novos
>conseguem acompanhar o movimento do corpo para seguir
>o alvo predeterminado, segundo a segundo, obedecendo à
>orientação do médico radioterapeuta. Reduziram-se
>muito os efeitos colaterais. O controle do câncer
>assemelha-se às extensas cirurgias. A nanotecnologia
>(aparelhos miniaturas) é uma febre, e os estudos
>multiplicam-se para definir com mais clareza sua
>aplicação.

{A} Pode matar com sua radioatividade, causando cancer.

>
>A plástica está fazendo tamanhos milagres que nem os
>próprios pacientes conseguem acreditar. Tanto faz quem
>foram seus pais, ou a etnia a qual você pertença.
>Escolha o modelo e ficará parecido. Para pessoas
>ansiosas com a imagem, soluções para quase tudo. Sem
>dúvida, o impacto sobre o estado emocional é notável.

[A] Eita faquinha de dois cumes heim? A plastica pode curar a pessoa de queimaduras ou cicatrizes, mas tambem pode ressaltar sua vaidade.

>
>Ao lado dessas notícias que mereceram um destaque
>quase obsessivo nas manchetes de capa, outras
>informações com menos, digamos, glamour, não
>conseguiram espaço nem nas páginas finais das
>revistas.
[A] A ciencia ja tem a imprensa e seus colaborados para dar esse glamour...


>
>O objetivo de reduzir em 60% a mortalidade infantil no
>mundo (e o Brasil ainda é um grande protagonista dessa
>estatística) não será atingido em 2015, como
>estabelecido nos Objetivos Milenares da ONU. No
>planeta, morrem por ano mais de 11 milhões de crianças
>com idade inferior a 5 anos. A maioria por doenças
>evitáveis. Diarréia, pneumonia, malária. No mesmo
>período, 500 mil mulheres morrem durante a gravidez ou
>o parto. Doença de Chagas e esquistossomose
>(barriga-d'água) continuam afetando milhares de
>brasileiros. E seu controle está cada vez mais
>próximo. A implementação dos programas já em ação
>poderá melhorar ainda mais esse controle. Para 2005 a
>intensificação dessas abordagens poderá elevar o
>impacto na saúde da população, principalmente nas
>áreas rurais.

[A] Isso ai, como fica doenças tao mais antiga diante do deusa peerfeita ciencia??


>O Relatório Mundial de Saúde (The World Health Report
>2003) recomendou às autoridades o fortalecimento dos
>sistemas de saúde, centralizando seu foco na atenção
>primária, além de integrar a prevenção das doenças e a
>promoção da saúde em todos os níveis de atendimento.

[A] Esses relatorios tem efeito??

>
>A saúde do homem sofreu mudanças drásticas nos últimos
>anos. A expectativa de vida, de brasileiros e de não
>brasileiros, bate recorde atrás de recorde.
>Ultrapassou a marca dos 70 anos e logo passará dos 80,
>90, e quem sabe 100 anos. Avanços sem dúvida notáveis.
>Doenças contagiosas foram substituídas por doenças
>crônicas, como problemas cardiovasculares e câncer, e
>por causas externas, como trauma. Essa mudança de foco
>exigiu modificações intensivas no nível estrutural.
>Algumas especialidades assumiram posição de destaque
>nunca antes alcançada. Os serviços de cardiologia de
>muitos hospitais logo se transformaram em
>departamentos e, a seguir, em grandes centros com
>prédios próprios. O mesmo ocorreu com a oncologia e
>suas variadas especialidades. Houve uma explosão na
>tecnologia e na sofisticação, e, conseqüentemente, nos
>custos ­ alertam os especialistas em saúde pública.
>Recomenda-se também a melhora da saúde da população
>por intermédio do envolvimento acadêmico.
>

[A] Sem comentarios, pra que vou querer viver até o 100?
>
>Muito por fazer.
>Como melhorar o atendimento ao público do SUS

[A] Infelizmente a ciencia não melhora ela mesma vai melhorar o SUS, a ciencia infelizmente, anda atendendo interesses dos laboratorios e pondo em pratica, o que determina o codigo americano, Bisness in bisness...
>
>
>A academia médica no Brasil foi alterada sensivelmente
>nas últimas duas décadas. Progressivamente, as escolas
>médicas e os programas de pós-graduação introduziram
>disciplinas de pesquisa epidemiológica de problemas de
>saúde adaptados à realidade da população do País. Mais
>e mais livros têm sido publicados por acadêmicos
>brasileiros, com enfoque nacional. Mais e mais centros
>de treinamento oferecem vagas para médicos
>interessados em se aperfeiçoar no atendimento básico à
>saúde da família e à clínica geral. O impacto na
>melhora da atenção à saúde do brasileiro pode demorar
>a ser notado. Muito há de ser feito em 2005 e além.

[A] Infelizmente, tem muito moleque fazendo medicina pro papai e só sai meleca, ou o diagnostico é virose ou é exames a "toa"...


>O plano Fome Zero tenta corrigir um problema básico de
>saúde: a miséria e a conseqüente desnutrição. Ninguém
>precisa de doutorado para compreender a conexão direta
>entre a desnutrição e a ocorrência de doenças
>potencialmente graves. É um plano interessante, mas
>ainda longe de atingir seus objetivos de forma
>significativa. Em 2005, e além, o governo deverá fazer
>os ajustes finos para que o programa consiga alcançar
>seu alvo.
>
>A violência é problema de segurança pública, mas
>também é problema de saúde. Não podemos esquecer que,
>no ano passado, mais de 40 mil brasileiros foram
>assassinados. Se acreditarmos nos dados oficiais do
>Ministério da Saúde, naquele período, morreram mais
>brasileiros por tiro do que por câncer de pulmão. O
>controle da violência evitará milhares de mortes e
>seqüelas dramáticas.
>
>A malária é outro problema sem solução a curto prazo.
>Áreas extensas do território nacional são infestadas
>por mosquitos portadores dessa doença. O esforço das
>autoridades regionais e federais deverá ser
>intensificado nas várias frentes: pesquisa de vacinas,
>profilaxia, controle do mosquito e tratamento de
>pacientes infectados. A mortalidade por essa infecção
>deve ser reduzida a todo custo. Estudos recentes
>aventam que a vacina contra a malária pode estar ao
>alcance da ciência em prazo razoável.
>
>No mundo todo, um bilhão de pessoas são infectadas por
>parasitas, como os vermes, e o Brasil tem participação
>significativa nesses dados alarmantes. Vermes no
>intestino não são apenas feios. Eles causam problemas
>como deficiências nutricionais, e, em alguns casos,
>podem até ser fatais. Para mudar isso, saneamento
>básico é fundamental.
>
>Um estudo recentemente realizado e publicado por
>pesquisadores na Universidade do Ceará demonstrou
>claramente a possibilidade de praticamente eliminar os
>parasitas dos pacientes. Remédios eficazes, como a
>ivemerctina, podem ser a solução. Além de tratar o
>paciente, eliminam uma potencial fonte de contaminação
>para outras pessoas. As autoridades devem criar
>sistemas integrados, contínuos, para atacar esse
>problema de saúde pública em várias frentes, em 2005 e
>para sempre.
>
>Quanto à Aids, nos últimos anos, houve um declínio da
>incidência de novos casos na maioria dos estados
>brasileiros. Estudo publicado recentemente por
>pesquisadores da Fundação Oswaldo Cruz, no Rio de
>Janeiro, confirmou que, apesar de recursos limitados
>do governo e da desigualdade socioeconômica aberrante
>no Brasil, a introdução do acesso universal à terapia
>antiviral contribuiu para uma redução impressionante
>na mortalidade por Aids, e pode ter ajudado a diminuir
>sua incidência.
>
>Nessa linha de pensamento, o presidente Lula lançou,
>este ano, o louvável Plano Nacional de Eliminação da
>Hanseníase (a lepra) em até dois anos. Um passo sem
>dúvida importante para controlar essa doença
>contagiosa e debilitante. Paralelamente, o ministro da
>Saúde, Humberto Costa, anunciou a formação de uma
>comissão para promover políticas nacionais de
>reabilitação às pessoas afetadas pela hanseníase:
>
>­ A idéia é possibilitar benefícios, como a
>reabilitação por cirurgias plásticas, financiando para
>a capacitação de médicos, enfermeiros e
>fisioterapeutas para que realizem esses procedimentos
>nos incapacitados fisicamente.
>
>Todos os anos, 42 mil brasileiros contraem a
>hanseníase. E o País só perde para a Índia no ranking
>mundial de casos da doença.
>
>A obesidade, mal do século XXI, mata. Nos EUA, em
>alguns estados o número de óbitos por obesidade
>ultrapassou o de câncer. No Brasil, cientistas
>identificaram obesidade crescente na população. Estudo
>publicado por pesquisadores da Universidade Federal do
>Rio de Janeiro confirmou o aumento progressivo do peso
>dos adolescentes, tanto na Região Nordeste quanto no
>Sudeste do País, desde 1975. Atualmente, 17% dos
>adolescentes da Região Sudeste enquadram-se na
>definição internacional de obesidade. E seus efeitos
>maléficos logo alcançarão a saúde pública. Orientação
>e conscientização devem ser política contínua e bem
>estruturada. A prevenção e o tratamento do sobrepeso
>precisam ser prioridades das políticas de saúde nos
>próximos anos.
>
>Um problema que acomete principalmente as crianças é a
>poluição atmosférica. Um estudo realizado com 5.193
>crianças de duas cidades do Rio de Janeiro, e
>publicado recentemente na revista Annals of Allergy
>Asthma and Immunology, mostrou claramente a correlação
>entre os níveis de poluição e a incidência de
>problemas respiratórios. Observou-se um aumento de
>mais de 50% na freqüência de crises de asma nos
>moradores de Duque de Caxias (local com alta
>concentração de poluentes), comparados aos moradores
>de Seropédica (local com baixos níveis de poluição).
>Outro estudo, realizado pelos pesquisadores do
>laboratório de poluição atmosférica da Universidade de
>São Paulo, confirmou os efeitos nocivos da poluição do
>ar na mortalidade infantil. Há aumento de 6% no número
>de óbitos neonatais nas regiões mais poluídas. O
>controle de poluentes de qualquer origem deverá ser
>obrigação mundial. Ainda mais com a entrada em vigor
>do Protocolo de Kyoto.
>
>Mas, de forma geral, provavelmente o maior desafio das
>autoridades de saúde é garantir um apoio ao avanço
>científico na área de saúde, atingir a maioria da
>população com os avanços médicos e tecnológicos
>alcançados, e insistir na prevenção e na saúde
>primária, básica. Muito mais do que dinheiro, em 2005
>precisamos de estratégia, filosofia e enfoque novos.
>
>O acesso da população, principalmente das camadas
>menos privilegiadas da sociedade, a remédios em geral
>é muito limitado. Várias são as causas, como preços
>elevados na fonte, acréscimos excessivos na cadeia de
>distribuição e impostos. Se as autoridades, de todas
>as esferas, não atentarem a essas dificuldades
>enfrentadas no dia-a-dia da maioria dos cidadãos,
>muito mais será gasto para tratar complicações graves
>de doenças crônicas, como diabetes e hipertensão
>arterial. O Brasil iniciou uma ação interessante nessa
>direção: a aquisição da fábrica da GlaxoSmithKline do
>Brasil pelo governo federal. O presidente Lula
>destacou que "pela primeira vez um governo compra uma
>fábrica da iniciativa privada", ao mesmo tempo
>criticando "o processo inverso de privatização do
>governo anterior". Lula declarou, na ocasião, que
>estava "recuperando uma fábrica que seria desativada,
>eliminaria empregos e que agora produzirá, em escala
>nacional, os antibióticos mais usados no Brasil".
>
>Nessa mesma linha o ministro da Saúde, Humberto Costa,
>deixou muito clara sua estratégia para 2005:
>
>­ Vamos ampliar o acesso da população a medicamentos
>fundamentais com base em três opções. A primeira é a
>rede de farmácias populares, e até o fim do ano
>pretendemos inaugurar cem unidades em todo o Brasil. A
>segunda é a criação de um programa para a venda
>subsidiada de medicamentos básicos para hipertensão e
>diabetes, através da rede de farmácias privada.
>Pretendemos baixar os preços em até 50%. E a terceira
>é a redução do ICMS de 2,8 mil medicamentos até 2005.
>
>De acordo com uma pesquisa do IBGE, de 2003, a saúde
>aparece em terceiro lugar no orçamento das famílias
>brasileiras, e os medicamentos representam 61% desses
>gastos para as pessoas de baixa renda. Além disso,
>metade das pessoas que precisam de tratamento não pode
>pagar os remédios de que necessitam.
>
>A partir de 2005, 50 milhões de unidades de
>antibióticos serão produzidas para a rede do Sistema
>Único de Saúde (SUS) e para as Farmácias Populares. Em
>2007, a produção deve quintuplicar em relação à atual.
>Mais de 10 bilhões de unidades de medicamentos devem
>ser produzidas para as principais doenças que mais
>atingem a população brasileira, como hipertensão,
>diabetes, malária e tuberculose. A Fiocruz também
>produzirá mais vacinas, e o Ministério da Saúde
>enfatiza que "a ampliação do acesso da população aos
>medicamentos é uma das prioridades do governo
>federal".
>
>Em 2004, muito se fez pela saúde, mas ainda há muito
>mais por fazer. Programas de prevenção e detecção
>precoce de doenças sexualmente transmissíveis, como
>Aids, HPV (câncer de colo de útero), a melhoria na
>qualidade do atendimento dos pacientes do SUS,
>diminuição das filas, controle de poluição, de
>infecção hospitalar e, também, de mortes por agentes
>externos, como traumas (tiros, facadas, acidentes de
>trânsito). Para se ter uma idéia do custo com os
>acidentes de trânsito, a cada ano acontecem mais de
>100 mil mortes, com três a quatro vezes esse número de
>feridos. E a maioria absoluta é tratada com dinheiro
>público. São milhões de reais de gastos evitáveis. Sem
>falar nas seqüelas individuais.
>

Abraços
Amauri

[As partes desta mensagem que não continham texto foram removidas]



##### ##### #####

Para saber mais visite
http://www.ciencialist.hpg.ig.com.br


##### ##### ##### #####



Yahoo! Grupos, um serviço oferecido por:
PUBLICIDADE




------------------------------------------------------------------------------
Links do Yahoo! Grupos

a.. Para visitar o site do seu grupo na web, acesse:
http://br.groups.yahoo.com/group/ciencialist/

b.. Para sair deste grupo, envie um e-mail para:
ciencialist-unsubscribe@yahoogrupos.com.br

c.. O uso que você faz do Yahoo! Grupos está sujeito aos Termos do Serviço do Yahoo!.



[As partes desta mensagem que não continham texto foram removidas]



##### ##### #####

Para saber mais visite
http://www.ciencialist.hpg.ig.com.br


##### ##### ##### #####


Yahoo! Grupos, um serviço oferecido por:







------------------------------------------------------------------------------
Links do Yahoo! Grupos

a.. Para visitar o site do seu grupo na web, acesse:
http://br.groups.yahoo.com/group/ciencialist/

b.. Para sair deste grupo, envie um e-mail para:
ciencialist-unsubscribe@yahoogrupos.com.br

c.. O uso que você faz do Yahoo! Grupos está sujeito aos Termos do Serviço do Yahoo!.



[As partes desta mensagem que não continham texto foram removidas]



##### ##### #####

Para saber mais visite
http://www.ciencialist.hpg.ig.com.br


##### ##### ##### #####


Yahoo! Grupos, um serviço oferecido por:







------------------------------------------------------------------------------
Links do Yahoo! Grupos

a.. Para visitar o site do seu grupo na web, acesse:
http://br.groups.yahoo.com/group/ciencialist/

b.. Para sair deste grupo, envie um e-mail para:
ciencialist-unsubscribe@yahoogrupos.com.br

c.. O uso que você faz do Yahoo! Grupos está sujeito aos Termos do Serviço do Yahoo!.



[As partes desta mensagem que não continham texto foram removidas]



##### ##### #####

Para saber mais visite
http://www.ciencialist.hpg.ig.com.br


##### ##### ##### #####


Yahoo! Grupos, um serviço oferecido por:







------------------------------------------------------------------------------
Links do Yahoo! Grupos

a.. Para visitar o site do seu grupo na web, acesse:
http://br.groups.yahoo.com/group/ciencialist/

b.. Para sair deste grupo, envie um e-mail para:
ciencialist-unsubscribe@yahoogrupos.com.br

c.. O uso que você faz do Yahoo! Grupos está sujeito aos Termos do Serviço do Yahoo!.



[As partes desta mensagem que não continham texto foram removidas]



##### ##### #####

Para saber mais visite
http://www.ciencialist.hpg.ig.com.br


##### ##### ##### #####


Yahoo! Grupos, um serviço oferecido por:
PUBLICIDADE




------------------------------------------------------------------------------
Links do Yahoo! Grupos

a.. Para visitar o site do seu grupo na web, acesse:
http://br.groups.yahoo.com/group/ciencialist/

b.. Para sair deste grupo, envie um e-mail para:
ciencialist-unsubscribe@yahoogrupos.com.br

c.. O uso que você faz do Yahoo! Grupos está sujeito aos Termos do Serviço do Yahoo!.



[As partes desta mensagem que não continham texto foram removidas]



SUBJECT: Novo remedio contra AIDS (era Evolucao e desafios )
FROM: "Oraculo" <oraculo@atibaia.com.br>
TO: <ciencialist@yahoogrupos.com.br>
DATE: 03/01/2005 21:42

Falando nisso..:-)

http://noticias.terra.com.br/ciencia/interna/0,,OI448103-EI298,00.html

Alemães desenvolvem novo remédio contra a aids

Cientistas da Universidade de Hamburgo, dirigidos pela professora Ilona Hauber, desenvolveram um novo remédio contra a aids, que, embora não consiga curar a doença, detém seu desenvolvimento ao impedir o crescimento do vírus. O anúncio foi feito hoje pelo Instituto de Virologia e Imunologia da Universidade de Hamburgo, segundo o qual os resultados da pesquisa serão publicados na edição de janeiro do Journal of Clinical Investigation.
Os cientistas conseguiram inibir a ação de uma enzima humana - deoxyhypusin synhtase - da qual dependem os vírus da aids para se reproduzir. Os remédios contra a aids que existem atualmente atacam enzimas do próprio vírus, e o medicamento de Hamburgo será o primeiro a se dirigir contra uma enzima humana.

Com isso, espera-se poder atacar os vírus da aids que se mostraram resistentes aos remédios conhecidos. Atualmente, segundo o instituto, há cerca de 40 milhões de portadores de aids no mundo todo.


[As partes desta mensagem que não continham texto foram removidas]



SUBJECT: Ciência (era Evolucao e desafios)
FROM: "Oraculo" <oraculo@atibaia.com.br>
TO: <ciencialist@yahoogrupos.com.br>
DATE: 03/01/2005 22:45

Olá pessoal

Interessante palestra sobre ciência, medicina alternativa, princípios e afins. Este trecho dá uma idéia:

"The one overarching principle of alternative medicine, however, is that science is unnecessary, closed of mind, knows nothing and wants to know nothing. Science does not recognize the reality of the universe we live in but instead tries to force reality into a set of man-made rules. Science is also highly fallible, as evidenced by the way that scientists can disagree with each other and even change what is believed to be true."

Um abraço.

Homero
______________________________________________
http://www.skeptics.com.au/journal/pb/23_2_quackery.htm


The following is an edited version of a speech given by Peter Bowditch to a dinner meeting of the Australian Skeptics on Saturday, 22 February, 2003.
--------------------------------------------------------------------------------

Good evening, ladies and gentlemen

I am not a doctor. At least, I am not a medical doctor - I have a PhD from a fake American university - but I have been called a doctor. Apparently this is a derogatory term in certain circles.

I was continually harassed late last year by people demanding too know my qualifications to talk about alternative medicine, as apparently you have to use it to talk about it. My answer was:

My qualifications are that I am a scientifically-literate, concerned citizen with a particular interest in medical quackery. I am sick of seeing liars and thieves get away with their lying and thieving. I am sick of hearing stories about families being impoverished in order to pay for useless medical treatments for their sick children. I am sick of hearing about desperate people being robbed of their life savings by charlatans who only care about money. I am sick of hearing that murderers are heroes and heroes are murderers. Or, as George Mallory said about climbing Mount Everest: "Because it's there".

I am going to talk tonight about alternative medicine, but before I start I would like to point out that I don't like the term, because what I will be talking about is neither medicine nor an alternative. The word "alternative" suggests that a choice can be made from a selection of options which have similar outcomes. When you came here tonight you had the alternatives of coming by car, motorbike, pushbike, train, bus, or even walking if you live close enough. If you came by car you had alternatives when you bought it. For example, you can buy cars with four, five, six, eight, ten or twelve cylinders (or a second-hand Mazda RX7 with no cylinders); you can get diesel or petrol engines, automatic or manual gearboxes (or combinations of the two) with different numbers of gears. All these are alternatives which would cause no questions if you said you used them to get here tonight.

Some alternatives are a bit less likely. If you said that you came by boat, that is certainly possible because you can get boats of a certain size up the Lane Cove River to Fullers Bridge, although it is a bit of a walk up the hill from there. If you said that you came by helicopter it's still believable but you might be asked where you parked it. If you said that you didn't park it anywhere because you were winched down from it outside in Help Street and the pilot then flew it away people might be a bit doubtful, but I happen to know the man who runs the CareFlight helicopter service and they have winches on all their machines, so if he said that that was how he got here I would have to believe him.

If someone said, however, that they got to Chatswood in a 747 or by ocean liner or in an FA18 fighter or in a Formula 1 car that they bought from the Ferrari team after Michael Schumacher won his last World Championship you wouldn't even consider that it was possible. Some things simply cannot be true.

The types of alternative medicine that I'm going to talk about tonight aren't vitamins and massage and meditation and raw food diets and driving here in your family car. In fact, I'm not going to talk about medicine at all. I am going to talk about lies and theft and deceit and fraud and quackery. I'm going to talk about people who claim to be able to cure cancer and AIDS and asthma and autism and MS and arthritis and diabetes and any number of other diseases and ailments. I will just use the term "alternative medicine" as a convenient kind of shorthand.

There are some principles of alternative medicine that you might like to keep in mind throughout this talk and the demonstrations of devices and potions. One is that the body is incapable of regulating itself and consequently becomes polluted with many toxins. Some of these toxins are environmental, like mercury in fillings and vaccines, and some are just by-products of normal living. To get rid of these toxins you need to take things to flush certain organs like the liver, or to have chelation. (This is a process where chemicals are introduced into the bloodstream to capture certain molecules and remove them through the urine. It is a legitimate medical procedure in cases of true heavy metal poisoning.) One very common way of getting rid of toxins is the colon cleanse, which your grandmother would have called an enema except that she was too polite to talk about such things.

Another universal principle is that the body contains all that it needs to cure itself and is self-regulating but it just needs some help occasionally. Chiropractors call this "innate intelligence". This principle means that you just have to get everything in balance, preferably by using "natural" methods. Traditional Chinese Medicine adjusts the hot and cold winds that flow through us, and acupuncture aligns the qi in the meridians when it gets a bit out of kilter, but these are just ways of helping the body heal itself.

Another principle is that conventional medicine is just driven by the obscene, massive profits that can be made, with a common example being the $5.4 billion spent on vaccines throughout the world in 2001. This is contrasted with the related principle that the $17 billion spent on herbal supplements in the US alone in that year is evidence that people are voting with their wallets for the altruistic, profitless alternative alternative.

In alternative medicine there is no such thing as mental illness or somatization or anxiety disorders. It is not possible for the mind to make someone sick and there must be an external cause. If you point out to an alt supporter that Multiple Chemical Sensitivity looks a lot like agoraphobia or Obsessive Compulsive Disorder the reply usually goes along the lines of "You are saying that it is not real. You are saying that it is all in the head". Trying to say that things can be very real, very debilitating and still only in the head just attracts more abuse. Lying comfortably beside this principle is the one that says that the mind is a powerful curer of disease and can, by itself, work miracles. When the scientific absurdity of homeopathy is pointed out, for example, the response is often that the wonderful results achieved in anecdote land come from the amazingly powerful placebo effect, where the mind tells you to get better.

Another principle is that clinical trials of alternative treatments are not necessary, because all you have to know is that things work. Personal experience, that which scientists call "anecdotes", is all the evidence needed. A sub-clause of this principle is that there is no money anyway to do research. (Total sales of alternative medicine in the US are about $30 billion per year, which is about 50% greater than the cosmetics industry, about 4 times the sales of movie tickets and about equal to the entire world market for recorded music. In Australia the expenditure is about $2.3 billion, which is almost the same figure as that for our wine exports. Alt supporters like to say that this is four times the sales of OTC drugs, but it isn't really that bad. The Pharmaceutical Benefits Scheme spends about $7 billion, so alternatives only account for about a quarter of the medication spend.)

Some principles are not so dichotomous. One is that the immune system is under constant attack and needs boosting all the time. Another is what George Orwell called "doublethink", which is the ability to hold more than one contradictory idea at a time and believe them all to be true. An example of this is that iridology, reflexology, acupuncture and chiropractic all assume the body to be filled with meridians or channels of information but alts have no problem in believing all of them at once. Another example is the collection of guaranteed cures for cancer which each offer a unique cause and cure. Another principle is the existence of energy fields (sometimes associated with the meridians and channels I just mentioned) which cannot be measured but which can be adjusted with machines and magnets. Allied with energy fields are vibrations, both in the energy fields and in physical objects such as parasites and cells.

A very important principle, of course, is that natural is better. Anything natural will be good and artificial things are bad. Artificial things contains chemicals. This nice bunch of flowers that I have here is an example. I picked these this afternoon from the creek that runs between my house and the local high school. It is a castor oil plant and the seeds contain ricin, which is one of the most toxic substances known to man. The seeds don't have very much poison in them, so each seed can only kill about three people. It's very natural, very available, and very deadly

The one overarching principle of alternative medicine, however, is that science is unnecessary, closed of mind, knows nothing and wants to know nothing. Science does not recognize the reality of the universe we live in but instead tries to force reality into a set of man-made rules. Science is also highly fallible, as evidenced by the way that scientists can disagree with each other and even change what is believed to be true.

Let's get on to some specific examples.

I have here a book called "The Cure for All Diseases", by Hulda Regehr Clark, Ph.D., N.D. The same author has followed this up with books called "The Cure for All Cancers", "The Cure for All Advanced Cancers" and "The Cure for HIV/AIDS". The author has a legitimate PhD degree earned in 1958 for physiology. The "ND" stands for Doctor of Naturopathy, a degree she obtained from a mail-order place called Clayton University. (Americans have difficulty understanding why Australians find this so funny.) Someone once suggested in a public forum that the "ND" stood for "not doctor" and attributed this witticism to me. I was flattered but I had to admit that I hadn't thought of it, although I wish I had. It's been estimated that Clark sold $7 million worth of books in 1999, and she owns the publishing company so she gets to keep more of the money than most authors. The US Federal Trade Commission has recently taken action against Clark, but she has protected herself well. Her son distributes her products in the USA, her brother does Canada, her web site is hosted in Ohio but run by a main in Switzerland. She has another site which says that all of these are independent of her. She employs a PR man and a lawyer to abuse and defame people, but she can throw any of these people overboard at any time. Her son is already claiming that the web site named "Dr Clark Research Association" (the name of one of her businesses) has nothing to do with her and the man who runs it can swing in the wind.

I have spent some time on Clark because she epitomises quackery. She has no science, she gouges money, she runs from any request for evidence and her only responses to criticism are ad hominem attacks and lawsuits.

Here's something from her book that might be of interest to anyone with diabetes.

Diabetes Can be Cured
All diabetics have a common fluke parasite, Eurytrema pancreaticum, the pancreatic fluke of cattle, in their own pancreas. It seems likely that we get it from cattle, repeatedly, by eating their meat or dairy products in a raw state. It is not hard to kill with a zapper but because of its infective stages in our food supply we can immediately be reinfected. Eurytrema will not settle and multiply in our pancreas without the presence of wood alcohol (methanol). Methanol pollution pervades our food supply -- it is found in processed food including bottled water, artificial sweetener, soda pop, baby formula and powdered drinks of all kinds including health food varieties. I presume wood alcohol is used to wash equipment used in manufacturing. If your child has diabetes, use nothing out of a can, package or bottle except regular milk, and no processed foods.

By killing this parasite and removing wood alcohol from the diet, the need for insulin can be cut in half in three weeks (or sooner!).

Be vigilant with your blood sugar checks. The pancreas with its tiny islets that produce insulin recovers very quickly. Even if 90% of them were destroyed, requiring daily insulin shots, half of them can recover or regenerate so insulin is no longer necessary. The insulin shot itself may be polluted with wood alcohol (this is an especially cruel irony -- the treatment itself is worsening the condition). Test it yourself, using the wood alcohol in automotive fluids (windshield washer) or from a paint store, as a test substance. Try different brands of insulin until you find one that is free of methanol.

Artificial sweeteners are polluted with wood alcohol! Instead of helping you cope with your diabetes, they are actually promoting it! Do not use them.

Drugs that stimulate the pancreas to make more insulin may also carry solvent pollution; test them for wood alcohol and switch brands and bottles until you find a pure one. You may not need them much longer, so the extra expense now may soon reward you.
Copyright © 1993 by Hulda Regehr Clark, Ph.D., N.D.

And what about cancer

The Cause
For many years we have all believed that cancer is different from other diseases. We believed that cancer behaves like a fire, in that you can't stop it once it has started. Therefore, you have to cut it out or radiate it to death or chemically destroy every cancerous cell in the body since it can never become normal again. NOTHING COULD BE MORE WRONG! And we have believed that cancers of different types such as leukemia or breast cancer have different causes. Wrong again!

In this book you will see that all cancers are alike. They are all caused by a parasite. A single parasite! It is the human intestinal fluke. And if you kill this parasite, the cancer stops immediately. The tissue becomes normal again. In order to get cancer, you must have this parasite.

How can the human intestinal fluke cause cancer? This parasite typically lives in the intestine where it might do little harm, causing only colitis, Crohn's disease, or irritable bowel syndrome, or perhaps nothing at all. But if it invades a different organ, like the uterus or the kidneys or liver, it does a great deal of harm. If it establishes itself in the liver, it causes cancer! It only establishes itself in the liver in some people. These people have isopropyl alcohol in their bodies.

All cancer patients have both isopropyl alcohol and the intestinal fluke in their livers. The solvent, isopropyl alcohol, is responsible for letting the fluke establish itself in the liver. In order to get cancer, you must have both the parasite and isopropyl alcohol in your body.
Copyright © 1993 by Hulda Regehr Clark, Ph.D., N.D.

Several devices, potions, techniques and alternative ideas were then demonstrated and discussed, including:
a.. Zappers using electricity, sound and magnetism, following the theories of Hulda Clark, Royal Rife and Robert Beck
b.. A water-powered device to increase breast size
c.. A hand-held microscope used to determine a woman's fertility
d.. Applied Kinesiology, demonstrating the difference in strength when someone is holding sugar or aspartame
e.. Perkins Tractors and their descendant, Therapeutic Touch, used to ease the pain of arthritis (the patient said that it hurt more afterwards, but he was told that this was a healing crisis)
f.. Homeopathic vaccines (see more about this here)
g.. A faith healing in the Benny Hinn tradition (This was performed before the main show)
h.. A chiropractic adjustment (this was done on a soft toy - no humans or animals were harmed in the making of this speech)
i.. Could I leave out the anti-vaccinators? Of course not!


There is no alternative medicine. There is medicine that works and there is something else that doesn't. If some folk remedy can be proved to be effective and safe then it will be assimilated into medicine and no longer be an alternative. What makes it work can be identified and isolated, and may even be synthesised so that it can be produced in a reliable fashion (aspirin is an example).. Much of what is called "alternative medicine" has the same relationship to real medicine as pornography has to love making. Actually, porn is more honest because the participants aren't pretending to be doing anything else.

I never blame the people who are sucked in by the quacks and frauds. If you are desperate with a terminal or chronic condition it is only natural to look for answers anywhere. Some medical interventions are unpleasant, and doctors are rushed and harassed at times. Sometimes there is no answer. The average person doesn't understand science and may even distrust it, so they are easy prey for scamsters who abuse science (in both meanings of the word "abuse"). The bottom line is that the frauds steal not only the money of their victims, but their hopes and their lives.

None of this is new. The following words come from a publication called Pseudodoxia Epidemica, written by Sir Thomas Browne in 1646. In between paragraphs ridiculing urine therapy and astrology (both unfortunately still with us), he had this to say about "Saltimbancoes, Quacksalvers, and Charlatans":

For their Impostures are full of cruelty, and worse than any other; deluding not only unto pecuniary defraudations, but the irreparable deceit of death.

Thank you.


[As partes desta mensagem que não continham texto foram removidas]



SUBJECT: Evolucao e desafios -Homero
FROM: "Amauri Jr" <amaurijunior2@yahoo.com.br>
TO: <ciencialist@yahoogrupos.com.br>
DATE: 03/01/2005 23:53

Oi Homero





"Fico contente que goste dos debates e de minha participação (nem sempre é assim, as vezes o pessoal se irrita bastante comigo:-)"



[A] São pessoas que não reconhecem o verdadeiro valor de um bom debate...:)





"Mas ainda acho que está confundindo duas coisas, a ciência, abstrata, e o comportamento humano, passível de ser julgado e avaliado subjetivamente."



[A] As duas coisas são ciência, uma não haveria ser a se a outra não fosse, a ciência em geral é um processo de conhecimentos.



"É claro que é importante discutir a aplicação, direção, prioridade, etc, de pesquisas e do conhecimento humano. Mas, a ciência, é apenas um conjunto de conhecimentos adquirido de certa forma (método cientifico), não seu uso ou resultado. Isso está confundindo a discussão..:-)"



[A] Mas se não iremos difundir seu uso, por que descobrir? :)



"A questão é, se decide agir com eficácia, tomar uma decisão que se mostre confiável no resultado, ou escolher uma afirmação ou conclusão que se aproxime o máximo possível da realidade (sem pretensões de ser 100% ou "A VERDADE"), o que escolheria? Seja para curar pessoas, seja para matar inimigos, o que consideraria mais confiável, mais real ou mais eficaz?"



[A] Eu certamente, escolheria curar as pessoas, salvar os animais, pesquisar muito mais nosso planeta do que ter aquela balela de descobrir vida fora dele. Se não resolvemos problemas do nosso próprio planeta, vamos ter capacidade moral e tecnológica para explorar o espaço Homero?



"Neste caso, o termo "melhor" tem duas conotações, que foram misturadas. O que temos de melhor, no meu sentido, é de mais eficaz. Dá resultados. No seu uso, é um julgamento subjetivo, melhor, bom, correto, etc. Não tem ligação com meu argumento. A ciência é "melhor" se deseja uma ação eficaz, ou um conhecimento confiável. O que fará com esse conhecimento, se algo "melhor", do bem, ou algo "pior", do mal, não é relevante para o argumento."



[A] O ponto é que você disse que o que temos de melhor era a ciência, até concordo contigo, mas temos um atrito; você diz que ela eficaz, eu digo que não, pois não há numa mente humana uma verdade dos fatos...:)



"Já a erradicação da malaria foi um erro, ato falho, meu..:-) Pensei em varíola e escrevi malária (acho que passava um especial na TV sobre a malária..:-). Mas, mesmo a malária é mais controlada em paises civilizados, com alto uso de conhecimento cientifico, que nos pobres e sem esse uso. Ainda é um bom argumento sobre a eficácia da mesma..:-)"



[A] Homero, mesmo a varíola existem focos em alguns paises, ela não esta erradicada; se engana se a malaria, febre-amarela, colega e etc...não tenha em países desenvolvidos, tem, só que não é escancarado como é na América latina...:) duvido se for no Blonks e não encontrar um negro norte-americano doente, duvido...:)



"Finalmente, acho que não compreendeu a questão a emissão de radio e a ida à Lua. Não importa o poder dos USA, não importa seu governo ou o caráter (ou falta de) dos seus governantes, mesmo hoje, se emitir uma onda de rádio, quaisquer dois radio amadores podem dizer, precisamente, de onde vem. Isso se chama triangulação, e é o motivo de se gastar tanto tempo e dinheiro em desenvolvimento de aviões invisíveis, já que, se o aparelho voador reflete a onda do radar, pode ser posicionado precisamente, sem sombra de duvida".



[A] Se foi ou não, foi dinheiro jogado no lixo, sendo que poderia ser empregado na fome imensa na África ou a reconstrução das cidades que eles sempre destruíram...:)





"E ao pousar, e enviar dados do solo lunar, idem, sem possibilidade de fraudar essa emissão."



[A] Eu posso pegar um radio amador e dizer que estou no Japão, como vão saber? Eu se mais ou menos as coordenadas do Japão e posso falar para os leigos, pois é o que aconteceu...:)



"Lutamos a séculos com o virus da gripe e ainda não podemos mata-lo. Sua taxa de mutação, como costuma acontecer com virus, é brutal, e sua adaptação ciclica impede que seja feita uma vacina para todas as cepas. A simplicidade de determinados virus é a chave para que escapem tão facilmente da cura. Um organismo complexo tem estruturas que se repetem em todas as suas variações. Em algum momento encontramos essa estrutura e uma vacina é criada. Mas, organismos simples, em especial virus, mudam tão completamente, que uma estrutura, uma chave, única, não acontece. Isso torna bem dificil derrota-lo. Além disso, a simplicidade faz com que muitas estruturas sejam também usadas por células do organismo hospedeiro. Assim, algumas das drogas que efetivametne matam os virus da gripe não podem ser aplicadas a seres humanos, pois matam suas células saudáveis também (esse é um dos problemas com drogas anti-cancer também atingir células saudaveis juntamente as doentes)."



[A] Por que mexer com isto? Como diz aquele ditado, não se mexe com quem ta quieto, foi isto que ocorreu; uma "besta" em um centro de pesquisa pegou e deixou escapar o HIV e ele contaminou muita gente. A gripe e todas as doenças do tipo viral, ou se preferir, retro-virus ele é mutante porque se vale da célula hospedeira, portanto teríamos que mudar o código genético dos glóbulos brancos para se valer de eficácia. Mas por enquanto, a deusa ciência em sua magnitude, não encontrou um meio de fazer a tal. Portanto caro amigo, fica como jogar ping-pong sem raquete...:)



"A ciência não prometeu curar todas as doenças, consertar o mundo, ser 100% eficaz ou definir A VERDADE (para isso temos as religiòes..:-). Ela é apenas uma ferramenta, uma boa e eficaz ferramenta. Talvez esteja esperando demais da ciência e a frustação de não ter todas as respostas o tenha tornado um pouco rancoroso com ela. Entre duas afirmações, entre duas respostas, a obtida de forma cientifica é a mais confiável, só isso. Existe, sim, uma luta de pesquisadores, seres humanos, para usar essa ferramenta de forma a curar doenças, encontrar respostas, criar conforto, resolver problemas, etc. Mas é uma busca, não um final em sí mesmo."



[A] Ela foi dirigida por Tales de Mileto para descobrir formas de relacionar e pesquisar os fenômenos naturais, que por trás de um trovão ou raio não esta a fúria de algum deus ou coisa parecida. Mas o que vimos hoje é o cientificismo, uma forma derivada de crenças e fundamentalismo que não cabe dentro da ciência e esta atrapalhando algumas pesquisas de cura. Ela não quer ser a VERDADE, mas para alguns, ser cientista é acreditar na filosofia empírica e não é só isso, ela esta vendo em um ângulo só. Eu não sou rancoroso com ela, apenas estou mostrando (por enquanto não) que ela pode ver alem e encontrar o caminho...sobre religiões, creio ser muito pretensioso achar que aquilo ou isso seja mentira, tem-se sim, de investigar.





"Bush é uma "anta" fundamentalista. Mas observe que é uma das pessoas mais contra a ciência que poderíamos encontrar. Seu esforço fundamentalista para obrigar sua visão religiosa de mundo a todo o resto do planeta é frontalmente contrária a ciência, tolerante por natureza. Impedir o estudo de células tronco pode impedir avanços que poderiam curar diversas doenças, inclusive paraplegia. Ensinar criacionismo em escolas e banir a evolução (e a base da biologia moderna hoje) impediria até mesmo que bons médicos se formem ou que novas descobertas sejam feitas."



[A]Bush não é uma "anta" fundamentalista, ele usa a crença popular a seu favor, pois nessas guerrilhas ridículas dele e de seu amiguinho, enquanto suas empresas de petróleo faturam milhões de dólares nessa. Ao proibir, eu apenas vejo ele atender aos interesses da igreja onde foi apoiado, nada mais do que isto e voltamos a era medieval...:)





"Agora, você (quem me lê) deve decidir (pressionado certamente pelos falcões militares) o que fazer. Imagine que estamos em guerra com a Argentina. Você, presidente, tem uma arma recém criada que pode acabar a guerra, devido ao seu poder inimaginável. Detonar essa arma em um cidade vai matar 45 mil pessoas do outro pais (que começou a guerra). Não detonar vai prolongar a guerra por mais alguns meses, a Argentina jamais se renderá e 100 mil brasileiros morrerão, e mais de 350 mil argentinos. O que você decidiria? O que diria as mães e pais de seu pais, quando perguntassem porque não parou a guerra quando podia e escolheu ver morrer mais 100 mil de seus conterrâneos?"



[A] Eu presidente? Uau!!! :)...bom, eu estou numa guerra com a Argentina, aqui no meu ladinho(que arrepio), milhões de tupiniquins vão ou tem perigo de morte...sinceramente destruiria a Casa Rosa...muito eficaz e de tamanha eficiência e ainda punha uma bandeira brasileira na traseira do presidente da la Argentina....:)



"Deve se lembrar que nunca uma arma assim havia sido detonada, e não se sabia o horror que poderia causar (os primeiros testes no deserto americano usaram soldados para verificar o efeito e a descontaminação foi feita com vassouras de palha e água corrente.:-("



[A] Americano é sempre americano muito soldados morreram de câncer ou outro mal, como disse, eles mexem com que esta quieto...:)

"Vou repetir, acho um horror o bombardeio de civis, na verdade, acho qualquer guerra um horror, desnecessária e absurda. Mas não acho que Hiroshima foi apenas maldade, crueldade de monstros, mas uma decisão calculada, difícil e medonha, da qual eu não saberia o que escolher como "menos pior". E hoje temos os USA arrogantes e com jeito de donos do mundo, e nossa antiapatia por Bush nos faz ver tudo com um filtro de asco. Mas na época a guerra dos USA era contra impérios cruéis e mais perigosos que a América jamais será (Hitler, e todo o horror que representa) e não se pode relativizar o que teria acontecido se a Alemanha e o Japão ganhassem a guerra."



[A] A grande Águia ganhou e esta toda cheia de querer mandar no mundo, síndrome de superego...isso é o ponto, ter poder de construir e destruir para se fazer de herói...:)

----- Original Message -----
From: Oraculo
To: ciencialist@yahoogrupos.com.br
Sent: Monday, January 03, 2005 7:47 PM
Subject: Re: [ciencialist] Evolucao e desafios -Amaury


Olá Amaury

Fico contente que goste dos debates e de minha participação (nem sempre é assim, as vezes o pessoal se irrita bastante comigo:-)

Mas ainda acho que está confundindo duas coisas, a ciência, abstrata, e o comportamento humano, passível de ser julgado e avaliado subjetivamente.

É claro que é importante discutir a aplicação, direção, prioridade, etc, de pesquisas e do conhecimento humano. Mas, a ciência, é apenas um conjunto de conhecimentos adquirido de certa forma (método cientifico), não seu uso ou resultado. Isso está confundindo a discussão..:-)

A questao é, se decide agir com eficácia, tomar uma decisão que se mostre confiável no resultado, ou escolher uma afirmação ou conclusão que se aproxime o máximo possível da realidade (sem pretensões de ser 100% ou "A VERDADE"), o que escolheria? Seja para curar pessoas, seja para matar inimigos, o que consideraria mais confiável, mais real ou mais eficaz?

Sua resposa, que certamente é igual a minha, coloca a perspectiva correta na discussão..:-) Ciência, abstratamente, é a melhor ferramenta de compreensão deste universo (universo fisico) que dispomos. O que faremos com o conhecimento assim obtido, é outra coisa. Eu, particularmente, o usaria para melhorar a vida de todo mundo (por exemplo, usando o produto desse conhecimento para enviar ajuda as vitimas do tsunami na Asia). Outros a usariam para matar ou criar armas de destruição em massa.

Mas culpar a ciência, um conhecimento abstrato, pelo uso que malucos fazem dela, é irrelevante e impróprio. Me lembra a piada do matador que, perguntado se não tinah remorso, respondeu: Eu não mato ninguem, só faço o furo, quem mata é Deus (no que ele tem razão, de uma perspectiva religiosa do universo..:-)

Pessoas, seres humanos, matam e maltratam. E fazem isso muito antes de termos armas de destruição em massa ou equivalentes. Culpar o conhecimento do átomo pelo uso da bomba atomica seria como condenar a arma no lugar do atirador (imagino que seria um processo muito engraçado de acompanhar..:-)

Veja este trecho seu:

Amaury: Por isso, nada tenho contra ou a favor da ciência, apenas é uma visão critica sobre...:-)

Se fosse assim, nem haveria a discussão entre nós..:-) Mas seu posicionamento inicial era diferente, contrário a ciência, como se fosse ela responsável por seu uso. Por isso fiz minha análise de sua análise..:-)

A confusão fica mais clara neste trecho:

Amaury:O que temos de melhor, como disse, esta nas mãos dos poderosos, portanto "o que temos de melhor" foi e esta sendo mandado pelo governo dos poderosos. Hoje os norte-americanos mandam e desmandam no que "temos de melhor", tanto se é para o bem da humanidade é imoral, mas se beneficia as partes bélicas deles, seguem numa boa...:-)


Neste caso, o termo "melhor" tem duas conotações, que foram misturadas. O que temos de melhor, no meu sentido, é de mais eficaz. Dá resultados. No seu uso, é um julgamento subjetivo, melhor, bom, correto, etc. Não tem ligação com meu argumento. A ciência é "melhor" se deseja uma ação eficaz, ou um conhecimento confiável. O que fará com esse conhecimento, se algo "melhor", do bem, ou algo "pior", do mal, não é relevante para o argumento.

Já a erradicação da malaria foi um erro, ato falho, meu..:-) Pensei em variola e escrevi malária (acho que passava um especial na TV sobre a malária..:-). Mas, mesmo a malária é mais controlada em paises civilizados, com alto uso de conhecimento cientifico, que nos pobres e sem esse uso. Ainda é um bom argumento sobre a eficácia da mesma..:-)

Finalmente, acho que não compreendeu a questao a emissão de radio e a ida à Lua. Não importa o poder dos USA, nào importa seu governo ou o carater (ou falta de) dos seus governantes, mesmo hoje, se emitir uma onda de rádio, quaisquer dois radio amadores podem dizer, precisamente, de onde vem. Isso se chama triangulação, e é o motivo de se gastar tanto tempo e dinheiro em desenvolvimento de aviões invisiveis, já que, se o aparelho voador reflete a onda do radar, pode ser posicionado precisamente, sem sombra de duvida.

Explicando melhor, se um satélite emite sinal de radio, você, com seu radio amador (ou radar avançado, não importa.:-), pode determinar a direção com precisão. Se outro amigo seu radio amador também o fizer, os dois podem combinar os dados e dizer, com exatidão, de onde parte o sinal e exatamente onde está o satelite. Não há forma de fraudar isso, nem de disfarçar ou fingir estar onde não está. Assim, quando as Apollo orbitavam a Terra, foram acompanhadas de perto por todos que desejassem, sem forma de fraudar isso. Quando orbitaram a Lua, foram acompanhadas (com frustação crescente dos russos..:-), por toda parte.

E ao pousar, e enviar dados do solo lunar, idem, sem possibilidade de fraudar essa emisão.

Assim, se deseja discordar do pouso na Lua, tem de explicar como isso foi feito, ou vai esbarrar em algo chamado evidencia acachapante (que colocou fim nas especulações dos russos, chineses e todos os muitos inimigos dos USA durante a guerra fria que adorariam desmascarar a fraude). Alias, quando os russos colocaram o primeiro satélite, Sputinik, em órbita, alguns jornais americanos e alguns membros do governo americano tentaram vender a idéia de fraude, mas o fato que qualquer um com um bom rádio podia confirmar o feito, foi o bastante para convencer as pessoas.:-)

HIV

Lutamos a séculos com o virus da gripe e ainda não podemos mata-lo. Sua taxa de mutação, como costuma acontecer com virus, é brutal, e sua adaptação ciclica impede que seja feita uma vacina para todas as cepas. A simplicidade de determinados virus é a chave para que escapem tão facilmente da cura. Um organismo complexo tem estruturas que se repetem em todas as suas variações. Em algum momento encontramos essa estrutura e uma vacina é criada. Mas, organismos simples, em especial virus, mudam tão completamente, que uma estrutura, uma chave, única, não acontece. Isso torna bem dificil derrota-lo. Além disso, a simplicidade faz com que muitas estruturas sejam também usadas por células do organismo hospedeiro. Assim, algumas das drogas que efetivametne matam os virus da gripe não podem ser aplicadas a seres humanos, pois matam suas células saudáveis também (esse é um dos problemas com drogas anti-cancer também atingir células saudaveis juntamente as doentes).

Talvez as industrias e governos não invistam o suficiente na pesquisa, eu tendo até a concordar com você nisso. Mas isso indica duas coisas: primeiro, que é caro e dificil descobrir a cura, ou ela já teria sido descoberta mesmo com pouco dinheiro. Segundo, indica que temos FALTA de ciência, que seria possivel com mais recuros, e não exesso. Em qualquer dos casos a crítica feita não se aplica. Precisamos, se queremos encontrar a cura ou tratamentos melhores para a AIDS, de mais ciência, um conheimento confiável e eficaz, e não de menos.

Espectativas

Amaury:Não tenho nada contra a ciência, mas você tem contra os índios...:) o que vejo é ainda crianças mortas e vidas curtas por uma ciência ineficaz, se eu gosto ou não é outra historia, que se eu não gosta-se da ciência não estaria numa lista de ciências...:) e pouco irei a uma tribo, porque não iria acostumar...:)e uma pergunta: a ciência tem vida própria?

A ciência não prometeu curar todas as doenças, consertar o mundo, ser 100% eficaz ou definir A VERDADE (para isso temos as religiòes..:-). Ela é apenas uma ferramenta, uma boa e eficaz ferramenta. Talvez esteja esperando demais da ciência e a frustação de não ter todas as respostas o tenha tornado um pouco rancoroso com ela. Entre duas afirmações, entre duas respostas, a obtida de forma cientifica é a mais confiável, só isso. Existe, sim, uma luta de pesquisadores, seres humanos, para usar essa ferramenta de forma a curar doenças, encontrar respostas, criar conforto, resolver problemas, etc. Mas é uma busca, não um final em sí mesmo.

Tenho as mesmas objeções que você às industrias farmaceuticas (inclusive as homeopaticas..:-). Mas sei que existem pesquisadores legitimamente dedicados a encontrar curas, soluções, respostas, apesar das empresas e até mesmo dentro dessas empresas. Gente que tem legitimos interesses humanos, que pesquisa até mesmo as "doenças de paises pobres" como a malária e febre amarela (erradicadas nos paises desenvlvidos).

Bush é uma "anta" fundamentalista. Mas observe que é uma das pessoas mais contra a ciência que poderiamos encontrar. Seu esforço fundamentalista para obrigar sua visão religiosa de mundo a todo o resto do planeta é frontalmente contrária a ciência, tolerante por natureza. Impedir o estudo de células tronco pode impedir avanços que poderiam curar diversas doenças, inclusive paraplegia. Ensinar criacionismo em escolas e banir a evolução (e a base da biologia moderna hoje) impediria até mesmo que bons médicos se formem ou que novas descobertas sejam feitas.

Ainda assim, ele usa os resultados da ciência onde lhe convém, com armas de destruição e brinquedos tecnologicos que poder. Isso torna a ciência culpada? Ou torna o Bush responsável?

Hiroshima é um caso complicado. Nagazaki seria mais claro, como alvo do debate, desnecessário e violento. Mas Hiroshima é dificil. Antes que "caiam de pau", acho um horror a destruição de uma cidade, a morte de civis e qualquer tipo de conflito. Mas é preciso pensar, sempre, sob pena de não ver com clareza. E Hiroshima é dificil de enxergar, através do horror da bomba e da matança.

Mas havia uma guerra em curso, isso é claro. Havia um pais, Japão, que entrou na guerra por vontade própria, para expadir fronteiras e massacrar vizinhos (embora a vontade a considerar fosse apenas dos mandatarios do pais na época). Havia um comportamento cultural onde a morte era sublime, o sacrificio honroso e o imperador deus (você morreria por seu deus). Dentro dessa análise, continuar a guerra com o Japão produziria, por estimativa, 100 mil mortes de americanos e mais de 350 mil mortes japonesas (estimativas derivadas das batalhas já ocorridas, da decisão de cada soldado japones de só parar depois de morto e das dificuldades de desembarque em ilhas - as piores batalhas da segunda guerra forma em ilhas do Japão).

Agora, você (quem me lê) deve decidir (pressionado certamente pelos falcões militares) o que fazer. Imagine que estamos em guerra com a Argentina. Você, presidente, tem uma arma recem criada que pode acabar a guerra, devido ao seu poder inimaginável. Detonar essa arma em um cidade vai matar 45 mil pessoas do outro pais (que começou a guerra). Não detonar vai prolongar a guerra por mais alguns meses, a Argentina jamais se renderá e 100 mil brasileiros morrerão, e mais de 350 mil argentinos. O que você decidiria? O que diria as mães e pais de seu pais, quando perguntassem porque não parou a guerra quando podia e escolheu ver morrer mais 100 mil de seus conterraneos?

Deve se lembrar que nunca uma arma assim havia sido detonada, e não se sabia o horror que poderia causar (os primeiros testes no deserto americano usaram soldados para verificar o efeito e a descontaminação foi feita com vassouras de palha e agua corrente.:-(

Eu não sei o que escolheria e não gostaria de ter de decidir algo assim. Imagino que não se pode dormir mais depois de ser submetido a coisas desse tipo. Por isso considero que Nagasaki é mais claro, uma decisão certamente militar, resultado de uma perda de poder civil em um periodo dificil. Mas Hiroshima é mais complicado.

Vou repetir, acho um horror o bombardeio de civis, na verdade, acho qualquer guerra um horror, desnecessária e absurda. Mas não acho que Hiroshima foi apenas maldade, crueldade de monstros, mas uma decisão calculada, dificil e medonha, da qual eu não saberia o que escolher como "menos pior". E hoje temos os USA arrogantes e com jeito de donos do mundo, e nossa antiapatia por Bush nos faz ver tudo com um filtro de asco. Mas na época a guerra dos USA era contra imperios crueis e mais perigosos que a america jamais será (Hitler, e todo o horror que representa) e nào se pode relativizar o que teria acontecido se a Alemanhã e o Japão ganhassem a guerra.

De todo modo, estamos mais perto da concordancia que da discordância..:-) A ciência, como ferramenta, é eficaz, confiável e razoável. Seu uso, que pode ser para o bem ou para o mal (mesmo que o bem e o mal sejam relativos, pergunte ao Bin Laden e ao Bush..:-), é que é passível de discussão.

Um abraço.

Homero






----- Original Message -----
From: Amauri Jr
To: ciencialist@yahoogrupos.com.br
Sent: Monday, January 03, 2005 11:45 AM
Subject: [ciencialist] Evolucao e desafios -Homero


Olá Amury


[A] Olá Homero






"Minhas análises sobre suas análises..:-)"



[A] Sinceramente gosto de suas analises, fico satisfeito de você me responder...:-)





"Você parece estar ressentido com o que chama de "ciência" e a trata como entidade independente, crença de cientistas ou algo a ser adorado ou derrubado. Mas nada disso é ciência, nem o resultados de sua aplicação se confundem com ela. Ciência neste contexto (desta lista e do uso padrão do termo..:-) é apenas o conjunto dos conhecimentos obtidos através de um método padronizado, chamado cientifico, e do rigor derivado do mesmo. E, independente do que pense dela, tem elevada confiabilidade..:-)"



[A] Amigo, o que penso que infelizmente o homem usa seu conhecimento para algo não muito para o "bem" humano ou "mal", a natureza humana já é corrupta. Tudo que é humano tem falhas, pois nada ainda é perfeito, mas a ciências como algo que vem para beneficiar a humanidade tinha que ser impartidaria e sem nenhuma crença ou fanatismo; como vimos muito na lista. Por isso, nada tenho contra ou a favor da ciência, apenas é uma visão critica sobre...:-)



"Não importa se o resultado é passível de ser julgado subjetivamente como "mau" ou mesmo como "bom". Importa apenas que é mais eficiente que outras formas de conhecimento, mais eficaz em suas previsões e mais confiável que outros instrumentos de compreensão do universo já criado por seres humanos. E que será abandonado assim que uma ferramenta mais eficaz seja apresentada..:-) Até lá, é o que temos de melhor."



[A] O que temos de melhor, como disse, esta nas mãos dos poderosos, portanto "o que temos de melhor" foi e esta sendo mandado pelo governo dos poderosos. Hoje os norte-americanos mandam e desmandam no que "temos de melhor", tanto se é para o bem da humanidade é imoral, mas se beneficia as partes bélicas deles, seguem numa boa...:-)





"Podemos analisar filosoficamente, subjetivamente, os resultados, mas não discutir a eficácia. A ciência, o conhecimento assim acumulado, é tão eficiente quando cura doenças, como a erradicação da malária, como quando destrói milhões de vidas, como em Hiroshima. O resultado é passível de julgamento, mas a ação não. Ela é eficaz, nos dois casos. Se precisar salvar alguém ou matar alguém, deve usar a ciência, é mais eficaz sempre..:-)"



[A] Podemos sim, tanto que você já disse que ciência é um conjunto de conhecimentos; mas muitos conhecimentos, pois o que vimos, é um "Clube da Ciência" que vê apenas um dos vários lados do universo, mas temos vários e é ainda um incógnito pelo homem ainda não abrir a cabeça...:-) Agora, se erradicamos a malaria não sei, creio que você não esta lendo jornal, pois a malaria, a febre-amarela, o HIV, o câncer, então matando e a ciência "tudo que temos de melhor" fica ai procurando pelos em baratas. Hiroshima foi um ato terrorista covarde e deprimente, que deveria fazer um julgamento dos cientistas e dos governantes no tribunal de Nuremberg, por danos morais humanos.





"Não gostar dela, ou como você pretende com as análises, critica-la ou despreza-la, não importa em nada para sua eficácia e confiabilidade. Você vai viver mais que todos os seus antepassados, goste ou não..:-) Sua expectativa de vida é de 75 anos, e, não sei sua idade, mas se já passou dos 40 como eu, é mais que seu avô esperava viver ao nascer (em 1900 a expectativa de vida no Brasil era de 33 anos)."





[A] Tenho apenas 28 anos, sem filhos e com uma deficiência física, o que sei que são superfluidades da senhora sagrada ciência. O que pode me interessar eu viver 40 ou 70 anos se nem ela pode prever?





"Na verdade, você usou um computador e a Internet, frutos da ciência, para enviar seus pensamentos e ataques a ciência, ao inves de tentar telepatia, rezas ou mandingas (ou qualquer outra forma de comunicação não cientifica) justamente porque é mais eficaz e confiável que qualquer outra..:-)"



[A] Alguma coisa boa tinha que sair daí né?...:-) o problema é esse, vocês da ciência só tem um foco do assunto, não ve as múltiplas variedades de estudos...:)





"Você não parece gostar de muitos dos aspectso do uso do conhecimento cientifico. Direito seu. E pode até mesmo abandona-los, todos, e ir viver em uma aldeia remota no Amazonas, sem (quase) nenhum contato com a ciência (os índios são muito receptivos a forasteiros, vão gostar de recebe-lo), e com todo o ônus de viver dessa forma (filhos mortos, vida curta, doenças diversas, poucos dentes, etc). Mas está na verdade criticando comportamentos humanos, que são seres falhos como sabe, não a ciência. Esta, "estricto senso", não é boa ou má, cruel ou gentil, apenas eficiente, confiável em seus efeitos."



[A] Não tenho nada contra a ciência, mas você tem contra os índios...:) o que vejo é ainda crianças mortas e vidas curtas por uma ciência ineficaz, se eu gosto ou não é outra historia, que se eu não gosta-se da ciência não estaria numa lista de ciências...:) e pouco irei a uma tribo, porque não iria acostumar...:)e uma pergunta: a ciência tem vida própria?



"Você não "acredita" nela. Engraçado, ela não pede que se acredite em nada.:-) Deve duvidar sempre, até que evidencias se mostrem sólidas o bastante para uma conclusão. E é nesse ponto que você escorrega..:-) O pouso na Lua já tem evidencias suficientes para essa conclusão, em que pese as tolices de diversos malucos mundo afora e suas teorias de conspiração. O HIV tem recuado e matado muito menos que no inicio da epidemia, graças a AZT e os atuais coquetéis antivirais. Se ajustes são feitos a toda hora, e são, sobre o que comer ou não, é apenas porque a ciência não pretende ser "A VERDADE" como religiões, mas um conhecimento que cresce, se aprofunda, se ajusta e melhora. Sabemos mais, não menos sobre o que comer ou não comer."



[A] Engraçado que pessoas inteligentes não duvidem que os yanques tenham forjado tudo, mas como você mesmo disse, é um direito seu. Eu acredito muito mais que se possa acreditar, a ciência melhorou minha vida muito, tendo esse computador; em minha cadeira de rodas de alumínio (de ferro era um lastima andar), portanto não estou desacreditando a ciência. Agora se quer achar que os coquetéis são eficazes tudo bem, mas que morre mesmo assim, isso morre...:)



"Sim, dá trabalho, é preciso atenção e constante leitura para acompanhar, seria melhor ter uma única resposta para todas as perguntas e pronto, sem risco e sem mudanças. Mas o mundo não funciona assim..:-) Sabemos hoje mais do que ontem sobre como se alimentar, e saberemos mais amanhã, deixando parte do que sabemos hoje para trás. (Parte, não tudo:-)"



[A] Como se o mesmo alimento hoje esta com proteínas e tudo que precisamos e amanha não?





"As teorias conspiratórias são atrativas.:-) É como se nós, os que estão "por dentro", soubéssemos de "coisas" que o resto do mundo não sabe, fossemos mais "espertos" ou mais inteligentes, que não se deixam enganar. Mas, na maioria das vezes (eu diria em sua totalidade) é apenas o ego tentando ser mais do que é.:-) Mesmo com centenas de sites "desmistificando" o pouso na Lua, o volume de evidencias a favor é gigantesco e nenhum deles explica um ponto fundamental do problema: como o governo americano fraudou os sinais de radio e TV que eram emitidos de todas as naves Apollo tanto durante as viagens, como do solo lunar, captados por todo radioamador ao redor do globo, sem falar nos radares e antenas dos paises em disputassem os USA? (lembre-se que, mesmo hoje, não se pode fraudar a origem de um sinal de radio..:-)"



[A] Não acredito nesses doentes mentais que ficam montando sites para se promover...:) eu tiro dos fatos que ocorreram que podem ser completamente forjados, como as transmissões, ou não sabe que eles são donos do que "temos de melhor"...:)



"Enfim, a ciência é eficaz. Embora seu uso, o uso do conhecimento confiável por ela produzido, possa ser discutido e julgado, isso não é a ciência nem com ela se confunde. E, tenha certeza, seu automóvel vai conduzi-lo amanhã, confiavelmente, conforme as leis da fisica descobertas pela ciência, seu computador receberá esta mensagem, os remédios na farmacia impedirão que você morra, as vacinas que tomou evitarão que adoeça (como todos os seus ancestrais adoeceram) e os satelites em órbita, colocados lá pelo conhecimento cientifico, vão transmitir suas chamadas telefonicas até para o Japão, se você assim desejar..:-)"



[A] Reverencia a ciência como um crente reverencia uma reza...:) o meu carro, vai bater conforma a lei da física, as farmácias venderão remédios que viciam, os satélites vão espionar minha casa e tudo graças a deusa infalível ciência...:)



"Para completar a mensagem anterior: uma menina inglesa de 15 aos foi a primeira pessoa a sobreviver depois de contrair raiva sem tomar a vacina antirabica. Ela foi submetida a um novo tratamento revolucionário (e científico), com um coquetel de drogas e coma induzido."



[A] É? Legal! Mas não são os igreses que usam a ciência para fazer guerrinhas com o Bush?





"Diferente de milagres divinos e sobrenaturais, o que se aprendeu com a nova técnica pode ser usado em outras pessoas doentes e não afetam apenas a pessoa que recebe a graça..:-)"



[A] Também acho, devo adverti-lo novamente que não sou a favor no milagre...:)




"A partir de agora, mesmo desprezando a "ciência", você pode ficar sossegado, se você ou um de seus filhos ou entes queridos contrair raiva, poderá ser curado, cientificamente curado.:-) A não ser que tenha resolvido ir viver em uma aldeia indigena na remota Amazonia..."



[A] Sim vão...vão tomar coca-cola e comer hambúrguer feitos cientificamente e se eu tiver sorte, vão se viciar em LSD feito pela ciência, vão tomar energéticos feito pela ciência, vão ter colesterol de produtos que o "temos de melhor" analisa. O mundo é um paraíso, graças a ciência não acha? ...:)



Abraços

Amauri


---- Original Message -----
From: Oraculo
To: ciencialist@yahoogrupos.com.br
Sent: Sunday, January 02, 2005 11:05 PM
Subject: Re: [ciencialist] Evolucao e desafios - CartaCapital - 02/01/05


Olá Amury

Minhas análises sobre suas análises..:-)

Você parece estar ressentido com o que chama de "ciência" e a trata como entidade independente, crença de cientistas ou algo a ser adorado ou derrubado. Mas nada disso é ciência, nem o resultados de sua aplicação se confundem com ela. Ciência neste contexto (desta lista e do uso padrão do termo..:-) é apenas o conjunto dos conhecimentos obtidos através de um método padronizado, chamado cientifico, e do rigor derivado do mesmo. E, independente do que pense dela, tem elevada confiabilidade..:-)

Não importa se o resultado é passível de ser julgado subjetivametne como "mau" ou mesmo como "bom". Importa apenas que é mais eficiente que outras formas de conhecimento, mais eficaz em suas previsões e mais confiável que outros instrumentos de compreensão do universo já criado por seres humanos. E que será abandonado assim que uma ferramenta mais eficaz seja apresentada..:-) Até lá, é o que temos de melhor.

Podemos analisar filosoficamente, subjetivamente, os resultados, mas não discutir a eficácia. A ciência, o conhecimento assim acumulado, é tão eficiente quando cura doenças, como a erradicação da malária, como quando destroi milhões de vidas, como em Hiroshima. O resultado é passível de julgamento, mas a ação não. Ela é eficaz, nos dois casos. Se precisar salvar alguém ou matar alguém, deve usar a ciência, é mais eficáz sempre..:-)

Não gostar dela, ou como você pretende com as análises, critica-la ou despreza-la, não importa em nada para sua eficácia e confiabilidade. Você vai viver mais que todos os seus antepassados, goste ou não..:-) Sua expectativa de vida é de 75 anos, e, não sei sua idade, mas se já passou dos 40 como eu, é mais que seu avô esperava viver ao nascer (em 1900 a expectativa de vida no Brasil era de 33 anos).

Na verdade, você usou um computador e a Internet, frutos da ciência, para enviar seus pensamentos e ataques a ciência, ao inves de tentar telepatia, rezas ou mandingas (ou qualquer outra forma de comunicação não cientifica) justamente porque é mais eficaz e confiável que qualquer outra..:-)

Você não parece gostar de muitos dos aspectso do uso do conhecimento cientifico. Direito seu. E pode até mesmo abandona-los, todos, e ir viver em uma aldeia remota no Amazonas, sem (quase) nenhum contato com a ciência (os indios são muito receptivos a forasteiros, vão gostar de recebe-lo), e com todo o onus de viver dessa forma (filhos mortos, vida curta, doenças diversas, poucos dentes, etc). Mas está na verdade criticando comportamentos humanos, que são seres falhos como sabe, não a ciência. Esta, "estricto senso", não é boa ou má, cruel ou gentil, apenas eficiente, confiável em seus efeitos.

Você não "acredita" nela. Engraçado, ela não pede que se acredite em nada.:-) Deve duvidar sempre, até que evidencias se mostrem sólidas o bastante para uma conclusão. E é nesse ponto que você escorrega..:-) O pouso na Lua já tem evidencias suficientes para essa conclusão, em que pese as tolices de diversos malucos mundo afora e suas teorias de conspiração. O HIV tem recuado e matado muito menos que no inicio da epidemia, graças a AZT e os atuais coqueteis antivirais. Se ajustes são feitos a toda hora, e são, sobre o que comer ou não, é apenas porque a ciência não pretende ser "A VERDADE" como religiões, mas um conhecimento que cresce, se aprofunda, se ajusta e melhora. Sabemos mais, não menos sobre o que comer ou não comer.

Sim, dá trabalho, é preciso atenção e constante leitura para acompanhar, seria melhor ter uma única resposta para todas as perguntas e pronto, sem risco e sem mudanças. Mas o mundo não funciona assim..:-) Sabemos hoje mais do que ontem sobre como se alimentar, e saberemos mais amanhã, deixando parte do que sabemos hoje para tras. Parte, não tudo:-)

As teorias conspiratórias são atrativas.:-) É como se nós, os que estão "por dentro", soubessemos de "coisas" que o resto do mundo não sabe, fossemos mais "espertos" ou mais inteligentes, que não se deixam enganar. Mas, na maioria das vezes (eu diria em sua totalidade) é apenas o ego tentando ser mais do que é.:-) Mesmo com centenas de sites "desmistificando" o pouso na Lua, o volume de evidencias a favor é gigantesco e nenhum deles explica um ponto fundamental do problema: como o governo americano fraudou os sinais de radio e TV que eram emitidos de todas as naves Apollo tanto durante as viagens, como do solo lunar, captados por todo radio-amador ao redor do globo, sem falar nos radares e antenas dos paises em disputacom os USA? (lembre-se que, mesmo hoje, não se pode fraudar a origem de um sinal de radio..:-)

Enfim, a ciência é eficaz. Embora seu uso, o uso do conhecimento confiável por ela produzido, possa ser discutido e julgado, isso não é a ciência nem com ela se confunde. E, tenha certeza, seu automóvel vai conduzi-lo amanhã, confiavelmente, conforme as leis da fisica descobertas pela ciência, seu computador receberá esta mensagem, os remédios na farmacia impedirão que você morra, as vacinas que tomou evitarão que adoeça (como todos os seus ancestrais adoeceram) e os satelites em órbita, colocados lá pelo conhecimento cientifico, vão transmitir suas chamadas telefonicas até para o Japão, se você assim desejar..:-)

Um abraço.

Homero




----- Original Message -----
From: Amauri Jr
To: ciencialist@yahoogrupos.com.br
Sent: Sunday, January 02, 2005 3:18 PM
Subject: Re: [ciencialist] Evolucao e desafios - CartaCapital - 02/01/05


Minhas analises em baixo com [A]....
----- Original Message -----
From: L.E.R.de Carvalho
To: ciencialist@yahoogrupos.com.br
Sent: Sunday, January 02, 2005 1:52 PM
Subject: [ciencialist] Evolucao e desafios - CartaCapital - 02/01/05



>LabConsS - www.ufrj.br/consumo
>
>
>
> EVOLUÇÃO E DESAFIOS
>
>
> O País viu inúmeros avanços nas áreas médica e
>tecnológica, mas as carências sociais ainda são sua
>pior mazela
>
>Em uma livraria, olho rapidamente as manchetes das
>revistas expostas. A julgar pelas capas, é pouco
>provável que tenhamos problemas de saúde deste mês em
>diante. Soluções curativas. Remédios extremamente
>eficientes. Vida prolongada. Corpos e rostos sem um
>defeito. Independentemente da idade, claro. Câncer?
>Uma brincadeira. Paralisia? Pode preparar as pistas.
>Infarto? E daí?

[A] temos que pagar e muito caro para a "ciencia" nos curar de males que diz a "ciencia" um direito de todos.

>
>Deixando de lado uma razoável dose de exagero nessas
>manchetes, encho o peito de orgulho pelas proezas
>atingidas no campo da medicina e da saúde. Vinte anos
>atrás, tudo isso não passaria de sonho, e não
>existiria fora da cabeça imaginativa dos produtores de
>filmes de ficção científica. Mas, ainda hoje, para a
>esmagadora maioria dos cidadãos brasileiros, todo o
>acima mencionado não passa de um sonho, de uma ficção
>científica

{A] Parece que todo a ciencia é uma ficção cientifica, vamos dizer que o pensamento é um comercio hoje em dia, ficçao ou não, é com certeza sensacionalismo.

>Este ano que acaba nos deu boas notícias,
>encorajadoras, e notícias ruins, no mínimo
>preocupantes. Vimos uma senhora que sofreu um derrame
>cerebral voltar a andar com terapia baseada na
>introdução de células-tronco (células primitivas
>capazes de se transformar em praticamente qualquer
>outra célula normal) no cérebro afetado. Emocionante
>perceber o que esses pequenos passos podem significar
>para milhões de pessoas que não conseguem erguer o
>braço, ou mexer a perna. Parafraseando Neil Armstrong,
>um passo gigante para a humanidade.

{A} Armstrong, quem é ele? Aquele que "supostamente" foi a Lua? Celulas- tronco é um desafio para a humanidade de cura, o desejo de ajudar seu semelhante. Pensamentos inovadores tem estado junto a humanidade a muito tempo, isso pode ser descrito na historia de Tales de Mileto até Einstein




>A técnica das células-tronco não se restringe a
>restaurar a função de células cerebrais. Hoje em dia,
>pesquisadores brasileiros, e em outros países,
>intensificam seus esforços para melhorar o coração
>depois de infarto, os nervos após lesão traumática, a
>pele após queimadura. A lista parece não ter fim.

[A] A demora faz o crescimento dos beneficiados...
>
>Vimos a introdução de remédios geniais na prática
>médica. Geniais na sua concepção, e geniais na sua
>eficiência. Drogas que conseguem agir em um ponto
>específico da célula doente, da célula cancerosa,
>dificultando seu desenvolvimento, seu crescimento, e
>até provocando sua morte. Exemplos que estão
>progressivamente sendo utilizados na prática médica,
>apesar de somente em casos muito selecionados, não
>faltam. Glivec, Iressa, são alguns deles.

[A] Santa ciencia!! Mata e de forma covarde porque usa seus conhecimentos de forma de interesse, mesmo que sabemos, esses conhecimentos são para todos.
>
>Ainda nem acabou o ano e ouvimos há poucas semanas o
>anúncio pela GlaxoSmithKline de uma vacina contra o
>câncer. O Brasil está totalmente empenhado na produção
>da vacina contra o HPV, vírus causador de câncer de
>útero. O prof. dr. Ricardo Brentani, presidente da
>Fundação Antonio Prudente, Hospital do Câncer AC
>Camargo, está entusiasmado e declara:
>
>­ Felizmente nossa instituição foi parceira da Merck
>Sharp & Dohme no desenvolvimento de uma vacina contra
>o HPV. Minha esperança é que em 20 anos tenhamos
>prevenido 7% dos tumores humanos.

[A] Vacina? Como pode ter vacina para nossas proprias celulas? E do HIV vai ter?

>Vimos, por outro lado, a preocupação crescente dos
>cientistas em não excluir, a priori, tratamentos
>considerados até então não-convencionais, ou
>complementares. Estudos sérios avaliaram desde a
>acupuntura até a homeopatia, passando por terapias de
>Florais de Bach e tratamentos com vitaminas nas mais
>diversas situações clínicas. Vantagens e desvantagens
>de cada abordagem foram dissecadas minuciosamente.

[A] Viu? Sera que alguem aqui duvidda?


>Cientistas conseguiram demonstrar a eficiência
>incontestável de algumas terapias alternativas em
>certas situações, como a massagem para um dos males do
>século, a dor nas costas. Por outro lado, alertaram
>para a ineficiência e até os efeitos nocivos de outras
>terapias, em outras condições. Por exemplo, para
>tratar bronquite e asma a acupuntura não parece ter
>efeitos importantes. Pelo menos não conseguiram
>detectar esses efeitos nos estudos atuais.

[A] Remedios sim? Eu tive começo, eu e meus manos, de bronquite depois que meu pai comprou um tartaruga nunca mais...santa crença ne? Mas como explicar??

>Vimos o lançamento de aparelhos ultramodernos,
>ultra-sensíveis, para detectar doenças e tratá-las.
>Não consigo perceber avanço recente maior do que na
>área da radioterapia. É incrível a precisão dos feixes
>de radiação em atingir o alvo, no caso o câncer, e
>poupar o tecido normal adjacente. Aparelhos novos
>conseguem acompanhar o movimento do corpo para seguir
>o alvo predeterminado, segundo a segundo, obedecendo à
>orientação do médico radioterapeuta. Reduziram-se
>muito os efeitos colaterais. O controle do câncer
>assemelha-se às extensas cirurgias. A nanotecnologia
>(aparelhos miniaturas) é uma febre, e os estudos
>multiplicam-se para definir com mais clareza sua
>aplicação.

{A} Pode matar com sua radioatividade, causando cancer.

>
>A plástica está fazendo tamanhos milagres que nem os
>próprios pacientes conseguem acreditar. Tanto faz quem
>foram seus pais, ou a etnia a qual você pertença.
>Escolha o modelo e ficará parecido. Para pessoas
>ansiosas com a imagem, soluções para quase tudo. Sem
>dúvida, o impacto sobre o estado emocional é notável.

[A] Eita faquinha de dois cumes heim? A plastica pode curar a pessoa de queimaduras ou cicatrizes, mas tambem pode ressaltar sua vaidade.

>
>Ao lado dessas notícias que mereceram um destaque
>quase obsessivo nas manchetes de capa, outras
>informações com menos, digamos, glamour, não
>conseguiram espaço nem nas páginas finais das
>revistas.
[A] A ciencia ja tem a imprensa e seus colaborados para dar esse glamour...


>
>O objetivo de reduzir em 60% a mortalidade infantil no
>mundo (e o Brasil ainda é um grande protagonista dessa
>estatística) não será atingido em 2015, como
>estabelecido nos Objetivos Milenares da ONU. No
>planeta, morrem por ano mais de 11 milhões de crianças
>com idade inferior a 5 anos. A maioria por doenças
>evitáveis. Diarréia, pneumonia, malária. No mesmo
>período, 500 mil mulheres morrem durante a gravidez ou
>o parto. Doença de Chagas e esquistossomose
>(barriga-d'água) continuam afetando milhares de
>brasileiros. E seu controle está cada vez mais
>próximo. A implementação dos programas já em ação
>poderá melhorar ainda mais esse controle. Para 2005 a
>intensificação dessas abordagens poderá elevar o
>impacto na saúde da população, principalmente nas
>áreas rurais.

[A] Isso ai, como fica doenças tao mais antiga diante do deusa peerfeita ciencia??


>O Relatório Mundial de Saúde (The World Health Report
>2003) recomendou às autoridades o fortalecimento dos
>sistemas de saúde, centralizando seu foco na atenção
>primária, além de integrar a prevenção das doenças e a
>promoção da saúde em todos os níveis de atendimento.

[A] Esses relatorios tem efeito??

>
>A saúde do homem sofreu mudanças drásticas nos últimos
>anos. A expectativa de vida, de brasileiros e de não
>brasileiros, bate recorde atrás de recorde.
>Ultrapassou a marca dos 70 anos e logo passará dos 80,
>90, e quem sabe 100 anos. Avanços sem dúvida notáveis.
>Doenças contagiosas foram substituídas por doenças
>crônicas, como problemas cardiovasculares e câncer, e
>por causas externas, como trauma. Essa mudança de foco
>exigiu modificações intensivas no nível estrutural.
>Algumas especialidades assumiram posição de destaque
>nunca antes alcançada. Os serviços de cardiologia de
>muitos hospitais logo se transformaram em
>departamentos e, a seguir, em grandes centros com
>prédios próprios. O mesmo ocorreu com a oncologia e
>suas variadas especialidades. Houve uma explosão na
>tecnologia e na sofisticação, e, conseqüentemente, nos
>custos ­ alertam os especialistas em saúde pública.
>Recomenda-se também a melhora da saúde da população
>por intermédio do envolvimento acadêmico.
>

[A] Sem comentarios, pra que vou querer viver até o 100?
>
>Muito por fazer.
>Como melhorar o atendimento ao público do SUS

[A] Infelizmente a ciencia não melhora ela mesma vai melhorar o SUS, a ciencia infelizmente, anda atendendo interesses dos laboratorios e pondo em pratica, o que determina o codigo americano, Bisness in bisness...
>
>
>A academia médica no Brasil foi alterada sensivelmente
>nas últimas duas décadas. Progressivamente, as escolas
>médicas e os programas de pós-graduação introduziram
>disciplinas de pesquisa epidemiológica de problemas de
>saúde adaptados à realidade da população do País. Mais
>e mais livros têm sido publicados por acadêmicos
>brasileiros, com enfoque nacional. Mais e mais centros
>de treinamento oferecem vagas para médicos
>interessados em se aperfeiçoar no atendimento básico à
>saúde da família e à clínica geral. O impacto na
>melhora da atenção à saúde do brasileiro pode demorar
>a ser notado. Muito há de ser feito em 2005 e além.

[A] Infelizmente, tem muito moleque fazendo medicina pro papai e só sai meleca, ou o diagnostico é virose ou é exames a "toa"...


>O plano Fome Zero tenta corrigir um problema básico de
>saúde: a miséria e a conseqüente desnutrição. Ninguém
>precisa de doutorado para compreender a conexão direta
>entre a desnutrição e a ocorrência de doenças
>potencialmente graves. É um plano interessante, mas
>ainda longe de atingir seus objetivos de forma
>significativa. Em 2005, e além, o governo deverá fazer
>os ajustes finos para que o programa consiga alcançar
>seu alvo.
>
>A violência é problema de segurança pública, mas
>também é problema de saúde. Não podemos esquecer que,
>no ano passado, mais de 40 mil brasileiros foram
>assassinados. Se acreditarmos nos dados oficiais do
>Ministério da Saúde, naquele período, morreram mais
>brasileiros por tiro do que por câncer de pulmão. O
>controle da violência evitará milhares de mortes e
>seqüelas dramáticas.
>
>A malária é outro problema sem solução a curto prazo.
>Áreas extensas do território nacional são infestadas
>por mosquitos portadores dessa doença. O esforço das
>autoridades regionais e federais deverá ser
>intensificado nas várias frentes: pesquisa de vacinas,
>profilaxia, controle do mosquito e tratamento de
>pacientes infectados. A mortalidade por essa infecção
>deve ser reduzida a todo custo. Estudos recentes
>aventam que a vacina contra a malária pode estar ao
>alcance da ciência em prazo razoável.
>
>No mundo todo, um bilhão de pessoas são infectadas por
>parasitas, como os vermes, e o Brasil tem participação
>significativa nesses dados alarmantes. Vermes no
>intestino não são apenas feios. Eles causam problemas
>como deficiências nutricionais, e, em alguns casos,
>podem até ser fatais. Para mudar isso, saneamento
>básico é fundamental.
>
>Um estudo recentemente realizado e publicado por
>pesquisadores na Universidade do Ceará demonstrou
>claramente a possibilidade de praticamente eliminar os
>parasitas dos pacientes. Remédios eficazes, como a
>ivemerctina, podem ser a solução. Além de tratar o
>paciente, eliminam uma potencial fonte de contaminação
>para outras pessoas. As autoridades devem criar
>sistemas integrados, contínuos, para atacar esse
>problema de saúde pública em várias frentes, em 2005 e
>para sempre.
>
>Quanto à Aids, nos últimos anos, houve um declínio da
>incidência de novos casos na maioria dos estados
>brasileiros. Estudo publicado recentemente por
>pesquisadores da Fundação Oswaldo Cruz, no Rio de
>Janeiro, confirmou que, apesar de recursos limitados
>do governo e da desigualdade socioeconômica aberrante
>no Brasil, a introdução do acesso universal à terapia
>antiviral contribuiu para uma redução impressionante
>na mortalidade por Aids, e pode ter ajudado a diminuir
>sua incidência.
>
>Nessa linha de pensamento, o presidente Lula lançou,
>este ano, o louvável Plano Nacional de Eliminação da
>Hanseníase (a lepra) em até dois anos. Um passo sem
>dúvida importante para controlar essa doença
>contagiosa e debilitante. Paralelamente, o ministro da
>Saúde, Humberto Costa, anunciou a formação de uma
>comissão para promover políticas nacionais de
>reabilitação às pessoas afetadas pela hanseníase:
>
>­ A idéia é possibilitar benefícios, como a
>reabilitação por cirurgias plásticas, financiando para
>a capacitação de médicos, enfermeiros e
>fisioterapeutas para que realizem esses procedimentos
>nos incapacitados fisicamente.
>
>Todos os anos, 42 mil brasileiros contraem a
>hanseníase. E o País só perde para a Índia no ranking
>mundial de casos da doença.
>
>A obesidade, mal do século XXI, mata. Nos EUA, em
>alguns estados o número de óbitos por obesidade
>ultrapassou o de câncer. No Brasil, cientistas
>identificaram obesidade crescente na população. Estudo
>publicado por pesquisadores da Universidade Federal do
>Rio de Janeiro confirmou o aumento progressivo do peso
>dos adolescentes, tanto na Região Nordeste quanto no
>Sudeste do País, desde 1975. Atualmente, 17% dos
>adolescentes da Região Sudeste enquadram-se na
>definição internacional de obesidade. E seus efeitos
>maléficos logo alcançarão a saúde pública. Orientação
>e conscientização devem ser política contínua e bem
>estruturada. A prevenção e o tratamento do sobrepeso
>precisam ser prioridades das políticas de saúde nos
>próximos anos.
>
>Um problema que acomete principalmente as crianças é a
>poluição atmosférica. Um estudo realizado com 5.193
>crianças de duas cidades do Rio de Janeiro, e
>publicado recentemente na revista Annals of Allergy
>Asthma and Immunology, mostrou claramente a correlação
>entre os níveis de poluição e a incidência de
>problemas respiratórios. Observou-se um aumento de
>mais de 50% na freqüência de crises de asma nos
>moradores de Duque de Caxias (local com alta
>concentração de poluentes), comparados aos moradores
>de Seropédica (local com baixos níveis de poluição).
>Outro estudo, realizado pelos pesquisadores do
>laboratório de poluição atmosférica da Universidade de
>São Paulo, confirmou os efeitos nocivos da poluição do
>ar na mortalidade infantil. Há aumento de 6% no número
>de óbitos neonatais nas regiões mais poluídas. O
>controle de poluentes de qualquer origem deverá ser
>obrigação mundial. Ainda mais com a entrada em vigor
>do Protocolo de Kyoto.
>
>Mas, de forma geral, provavelmente o maior desafio das
>autoridades de saúde é garantir um apoio ao avanço
>científico na área de saúde, atingir a maioria da
>população com os avanços médicos e tecnológicos
>alcançados, e insistir na prevenção e na saúde
>primária, básica. Muito mais do que dinheiro, em 2005
>precisamos de estratégia, filosofia e enfoque novos.
>
>O acesso da população, principalmente das camadas
>menos privilegiadas da sociedade, a remédios em geral
>é muito limitado. Várias são as causas, como preços
>elevados na fonte, acréscimos excessivos na cadeia de
>distribuição e impostos. Se as autoridades, de todas
>as esferas, não atentarem a essas dificuldades
>enfrentadas no dia-a-dia da maioria dos cidadãos,
>muito mais será gasto para tratar complicações graves
>de doenças crônicas, como diabetes e hipertensão
>arterial. O Brasil iniciou uma ação interessante nessa
>direção: a aquisição da fábrica da GlaxoSmithKline do
>Brasil pelo governo federal. O presidente Lula
>destacou que "pela primeira vez um governo compra uma
>fábrica da iniciativa privada", ao mesmo tempo
>criticando "o processo inverso de privatização do
>governo anterior". Lula declarou, na ocasião, que
>estava "recuperando uma fábrica que seria desativada,
>eliminaria empregos e que agora produzirá, em escala
>nacional, os antibióticos mais usados no Brasil".
>
>Nessa mesma linha o ministro da Saúde, Humberto Costa,
>deixou muito clara sua estratégia para 2005:
>
>­ Vamos ampliar o acesso da população a medicamentos
>fundamentais com base em três opções. A primeira é a
>rede de farmácias populares, e até o fim do ano
>pretendemos inaugurar cem unidades em todo o Brasil. A
>segunda é a criação de um programa para a venda
>subsidiada de medicamentos básicos para hipertensão e
>diabetes, através da rede de farmácias privada.
>Pretendemos baixar os preços em até 50%. E a terceira
>é a redução do ICMS de 2,8 mil medicamentos até 2005.
>
>De acordo com uma pesquisa do IBGE, de 2003, a saúde
>aparece em terceiro lugar no orçamento das famílias
>brasileiras, e os medicamentos representam 61% desses
>gastos para as pessoas de baixa renda. Além disso,
>metade das pessoas que precisam de tratamento não pode
>pagar os remédios de que necessitam.
>
>A partir de 2005, 50 milhões de unidades de
>antibióticos serão produzidas para a rede do Sistema
>Único de Saúde (SUS) e para as Farmácias Populares. Em
>2007, a produção deve quintuplicar em relação à atual.
>Mais de 10 bilhões de unidades de medicamentos devem
>ser produzidas para as principais doenças que mais
>atingem a população brasileira, como hipertensão,
>diabetes, malária e tuberculose. A Fiocruz também
>produzirá mais vacinas, e o Ministério da Saúde
>enfatiza que "a ampliação do acesso da população aos
>medicamentos é uma das prioridades do governo
>federal".
>
>Em 2004, muito se fez pela saúde, mas ainda há muito
>mais por fazer. Programas de prevenção e detecção
>precoce de doenças sexualmente transmissíveis, como
>Aids, HPV (câncer de colo de útero), a melhoria na
>qualidade do atendimento dos pacientes do SUS,
>diminuição das filas, controle de poluição, de
>infecção hospitalar e, também, de mortes por agentes
>externos, como traumas (tiros, facadas, acidentes de
>trânsito). Para se ter uma idéia do custo com os
>acidentes de trânsito, a cada ano acontecem mais de
>100 mil mortes, com três a quatro vezes esse número de
>feridos. E a maioria absoluta é tratada com dinheiro
>público. São milhões de reais de gastos evitáveis. Sem
>falar nas seqüelas individuais.
>

Abraços
Amauri

[As partes desta mensagem que não continham texto foram removidas]



##### ##### #####

Para saber mais visite
http://www.ciencialist.hpg.ig.com.br


##### ##### ##### #####



Yahoo! Grupos, um serviço oferecido por:
PUBLICIDADE




------------------------------------------------------------------------------
Links do Yahoo! Grupos

a.. Para visitar o site do seu grupo na web, acesse:
http://br.groups.yahoo.com/group/ciencialist/

b.. Para sair deste grupo, envie um e-mail para:
ciencialist-unsubscribe@yahoogrupos.com.br

c.. O uso que você faz do Yahoo! Grupos está sujeito aos Termos do Serviço do Yahoo!.



[As partes desta mensagem que não continham texto foram removidas]



##### ##### #####

Para saber mais visite
http://www.ciencialist.hpg.ig.com.br


##### ##### ##### #####


Yahoo! Grupos, um serviço oferecido por:







------------------------------------------------------------------------------
Links do Yahoo! Grupos

a.. Para visitar o site do seu grupo na web, acesse:
http://br.groups.yahoo.com/group/ciencialist/

b.. Para sair deste grupo, envie um e-mail para:
ciencialist-unsubscribe@yahoogrupos.com.br

c.. O uso que você faz do Yahoo! Grupos está sujeito aos Termos do Serviço do Yahoo!.



[As partes desta mensagem que não continham texto foram removidas]



##### ##### #####

Para saber mais visite
http://www.ciencialist.hpg.ig.com.br


##### ##### ##### #####


Yahoo! Grupos, um serviço oferecido por:







------------------------------------------------------------------------------
Links do Yahoo! Grupos

a.. Para visitar o site do seu grupo na web, acesse:
http://br.groups.yahoo.com/group/ciencialist/

b.. Para sair deste grupo, envie um e-mail para:
ciencialist-unsubscribe@yahoogrupos.com.br

c.. O uso que você faz do Yahoo! Grupos está sujeito aos Termos do Serviço do Yahoo!.



[As partes desta mensagem que não continham texto foram removidas]



##### ##### #####

Para saber mais visite
http://www.ciencialist.hpg.ig.com.br


##### ##### ##### #####


Yahoo! Grupos, um serviço oferecido por:







------------------------------------------------------------------------------
Links do Yahoo! Grupos

a.. Para visitar o site do seu grupo na web, acesse:
http://br.groups.yahoo.com/group/ciencialist/

b.. Para sair deste grupo, envie um e-mail para:
ciencialist-unsubscribe@yahoogrupos.com.br

c.. O uso que você faz do Yahoo! Grupos está sujeito aos Termos do Serviço do Yahoo!.



[As partes desta mensagem que não continham texto foram removidas]



##### ##### #####

Para saber mais visite
http://www.ciencialist.hpg.ig.com.br


##### ##### ##### #####


Yahoo! Grupos, um serviço oferecido por:
PUBLICIDADE




------------------------------------------------------------------------------
Links do Yahoo! Grupos

a.. Para visitar o site do seu grupo na web, acesse:
http://br.groups.yahoo.com/group/ciencialist/

b.. Para sair deste grupo, envie um e-mail para:
ciencialist-unsubscribe@yahoogrupos.com.br

c.. O uso que você faz do Yahoo! Grupos está sujeito aos Termos do Serviço do Yahoo!.



[As partes desta mensagem que não continham texto foram removidas]



SUBJECT: Re: primeiros socorros
FROM: Maria Natália <grasdic@hotmail.com>
TO: ciencialist@yahoogrupos.com.br
DATE: 04/01/2005 00:00


Emiliano:

Por este andar já menina se foi à vida.
Ora o nome da aranha é Aranha-marrom - Loxosceles.
encontrei este link:
http://www.saude.rj.gov.br/animaispeconhentos/links.html
E é de Rio Grande do Sul.
Entretanto e esperando que nenhum amigo* das aranhas nos
leia...encontrei também processo de por esta peludas longe de casa
em:http://www2.pucpr.br/educacao/pibic/evento/xiisic/CV76.html
As melhoras da amiga
Maria Natália
*cadeias alimentares, nichos ecológicos...


--- Em ciencialist@yahoogrupos.com.br, "E m i l i a n o C h e m e
l l o" <chemelloe@y...> escreveu
> Claro que ir ao médico é o mais recomendado. Mas no caso de
primerios
> socorros, o que se pode fazer?
>
> [ ] 's do Emiliano Chemello
> ---
> Contato Naeq:
> Nome: lia belart
> Email: liabelart@u...
> Telefone: Picada de aranha-marrom Lexosceles
> Mensagem: Prezados Senhores,
>
> Gostaria de saber, quais os medicamentos para os primeiros
socorros, que
> podem ser aplicados, no caso de sermos picados por esse tipo de
aranha.
> O genro de minha irmã, deve ter sido picado por esse tipo de
aranha, pois os
> sintomas são iguais.
> Atenciosamente,
> Lia





SUBJECT: Re: [ciencialist] Evolucao e desafios -Homero
FROM: "Oraculo" <oraculo@atibaia.com.br>
TO: <ciencialist@yahoogrupos.com.br>
DATE: 04/01/2005 00:43

Olá Amaury

Acho que não estou sendo claro..:-) Vejamos esta parte, de novo:

Homero: "A questão é, se decide agir com eficácia, tomar uma decisão que se mostre confiável no resultado, ou escolher uma afirmação ou conclusão que se aproxime o máximo possível da realidade (sem pretensões de ser 100% ou "A VERDADE"), o que escolheria? Seja para curar pessoas, seja para matar inimigos, o que consideraria mais confiável, mais real ou mais eficaz?"

[Amaury] Eu certamente, escolheria curar as pessoas, salvar os animais, pesquisar muito mais nosso planeta do que ter aquela balela de descobrir vida fora dele. Se não resolvemos problemas do nosso próprio planeta, vamos ter capacidade moral e tecnológica para explorar o espaço Homero?

Não é essa a escolha a que me refiro. Eu também escolheria, certamente, fazer o bem (ou minha versão de "bem"..:-). Não é essa escolha a que me refiro. É, uma vez que decidiu fazer o bem e salvar pessoas (ou fazer o mal e mata-las) o que escolheria como meio mais eficaz, o conhecimento produzido cientificamente ou outro? Essa a questão, não é sobre bem e mal, mas sobre eficaz, real ou ineficaz e irreal.

Em termos concretos, digamos que decidiu salvar a vida de uma população em uma tribo distante, o que escolheria? Mais rezas e xamanismo ou vacinas e remédios? Amuletos e simpatrias ou água encanada, clorada e esgoto tratado? Ou, se decidisse matar uma grande quantidade de pessoas, o que escolheria? Usar dinamite, armas, bombas e tecnologia ou mandingas, feitiços e pragas?

Ou seja, depois de decidir fazer o bem ou o mal (aspectos subjetivos e humanos), qual sua escolha quanto aos meios e conhecimentos que permitiriam concretizar sua escolha?

Agora, esta frase eu não compreendi:

[A] O ponto é que você disse que o que temos de melhor era a ciência, até concordo contigo, mas temos um atrito; você diz que ela eficaz, eu digo que não, pois não há numa mente humana uma verdade dos fatos...:)

Como assim? Se ela não é eficaz, ainda deveriamos viver o mesmo que nossos antepassados, ter as mesmas doenças, sofrer as mesmas dores, viver apenas nos mesmos locais, não teriamos robôs em Marte ou satélites e telefones celulares. Se ela não é eficaz, nada do que afirma teria resultado concreto, nenhuma de suas previsões se concretizaria e seria fácil perceber isso. Nem mesmo este dialogo estaria sendo feito, já que a Internet não existiria (nem computadores). E o que significa não existir em uma mente humana uma verdade dos fatos? Desculpe, mas nào fez muito sentido..:-)

Quanto a variola, está enganado. Não existem focos ou doentes há muito tempo, mais de 3 decadas. Eu tenho marca de vacinação contra varíola no braço, mas você, com apenas 28 anos, nào tem (não era mais necessário, já que ela não mais existe..:-) A variola foi erradicada, apenas cepas em laboratorio ainda existem. E várias doenças caminham para o mesmo fim, como a paralizia infantil (que resiste na África, onde lideres religiosos desconfiam das vacinas e lutam contra ela).

E negros do bronx ficam doentes, mas menos que negros africanos e não de malária ou tifo. E recebem tratamento médico melhor que a maioria de nossos antepassados, mesmo os ricos e nobres (o tratamento médico de um brasileiro pobre, no INPS, é muitas vezes melhor que o que dispunha a rainha Vitória da Inglaterra no periodo vitoriano..:-)

Agora uma guinada de 180 graus..:-)

[A] Se foi ou não, foi dinheiro jogado no lixo, sendo que poderia ser empregado na fome imensa na África ou a reconstrução das cidades que eles sempre destruíram...:)

Deixou de lado a afirmação de fraude e substituiu por um julgamento de valor, sobre se deveriam ter aplicado o dinheiro na Africa ou nos paises pobres. Nesse caso, fica dificil discutir, é subjetivo demais. Eu acho importantissimo o conhecimento produzido pelas missões Apollo, tanto quanto as missões à Marte. Nada é mais interessante e importante que as questões estudadas pela astrofisica, como surgiu o universo, os planetas, de onde vem a materia básica da vida, existe vida em outro planeta, etc. Mas tem todo direito de discordar e escolher outras prioridades.

Mas, nada disso muda o fato de que é a ciência, seu conhecimento confiável, que permitiu tanto a ida à Lua, quanto, se fosse possível, as suas escolhas de onde gastar o dinheiro.

[A] Eu posso pegar um radio amador e dizer que estou no Japão, como vão saber? Eu se mais ou menos as coordenadas do Japão e posso falar para os leigos, pois é o que aconteceu...:)

Não, não pode..:-) Se transmitir, e eu receber em dois aparelhos de radio amador, posso dizer exatamente onde está, e, se usar os satélites de GPS, com precisão de 1 ou 2 metros. Isso se chama triangulação e é um fato, não uma opinião ou uma crença. Se transmitir do Japão, eu precisarei de algum tipo de satelite, mas se transmitir de um satélite, qualquer radio amador ao alcance vai resolver e encontrar você..:-)

Vou tentar explicar, mas seria mais fácil com imagens e desenhos.

Pense em um ponto A em São Paulo. E um ponto B em Campinas (não sei se conhece, mas é uma cidade distante aproximadamente 90 Km de SP). Imagine um receptor em cada cidade citada. Agora, de uma caminhonete com um transmissor, você viaja pela estrada. O receptor em SP vai indicar, a principio, que o sinal mais forte vem da direção Norte, em graus precisos. O que se encontra em Campinas, indica que vem de Noroeste, com graus precisos. Pegue um mapa e desenhe uma reta a partir de cada cidade na direção indicada pelos graus. No ponto onde se cruzam, estará você, a caminhonete e seu transmissor.

Conforme viaja, os angulos vão mudando e o cruzamento continuará indicando sua posição, com precisão extrema.

Troque o transmissor na caminhonete por um sinal em uma nave ou satelite, e os receptores em SP e Campinas podem dizer, com precisão, exatamente onde se encontra o satélite, em cada segundo, sem erro ou possibilidade de fraude.

Se fosse possivel fraudar uma emissão, as aeronaves militares simplesmente levariam mecanismos que o fizesse e evitariam ser detectadas pelo radar inimigo..:-)

Enfim, conhecimento confiável, seja qual for o uso que se deseje dar a ele, é produzido com mais eficácia usando os métodos e mecanismos da ciência, seja apra enviar um homem à Lua, seja para curar doenças, seja para explodir cidades. Devemos responsabilizar quem de direito nessa questão, nos mesmos, não a ferramenta usada..:-)

Um abraço.

Homero




----- Original Message -----
From: Amauri Jr
To: ciencialist@yahoogrupos.com.br
Sent: Monday, January 03, 2005 11:53 PM
Subject: [ciencialist] Evolucao e desafios -Homero


Oi Homero





"Fico contente que goste dos debates e de minha participação (nem sempre é assim, as vezes o pessoal se irrita bastante comigo:-)"



[A] São pessoas que não reconhecem o verdadeiro valor de um bom debate...:)





"Mas ainda acho que está confundindo duas coisas, a ciência, abstrata, e o comportamento humano, passível de ser julgado e avaliado subjetivamente."



[A] As duas coisas são ciência, uma não haveria ser a se a outra não fosse, a ciência em geral é um processo de conhecimentos.



"É claro que é importante discutir a aplicação, direção, prioridade, etc, de pesquisas e do conhecimento humano. Mas, a ciência, é apenas um conjunto de conhecimentos adquirido de certa forma (método cientifico), não seu uso ou resultado. Isso está confundindo a discussão..:-)"



[A] Mas se não iremos difundir seu uso, por que descobrir? :)



"A questão é, se decide agir com eficácia, tomar uma decisão que se mostre confiável no resultado, ou escolher uma afirmação ou conclusão que se aproxime o máximo possível da realidade (sem pretensões de ser 100% ou "A VERDADE"), o que escolheria? Seja para curar pessoas, seja para matar inimigos, o que consideraria mais confiável, mais real ou mais eficaz?"



[A] Eu certamente, escolheria curar as pessoas, salvar os animais, pesquisar muito mais nosso planeta do que ter aquela balela de descobrir vida fora dele. Se não resolvemos problemas do nosso próprio planeta, vamos ter capacidade moral e tecnológica para explorar o espaço Homero?



"Neste caso, o termo "melhor" tem duas conotações, que foram misturadas. O que temos de melhor, no meu sentido, é de mais eficaz. Dá resultados. No seu uso, é um julgamento subjetivo, melhor, bom, correto, etc. Não tem ligação com meu argumento. A ciência é "melhor" se deseja uma ação eficaz, ou um conhecimento confiável. O que fará com esse conhecimento, se algo "melhor", do bem, ou algo "pior", do mal, não é relevante para o argumento."



[A] O ponto é que você disse que o que temos de melhor era a ciência, até concordo contigo, mas temos um atrito; você diz que ela eficaz, eu digo que não, pois não há numa mente humana uma verdade dos fatos...:)



"Já a erradicação da malaria foi um erro, ato falho, meu..:-) Pensei em varíola e escrevi malária (acho que passava um especial na TV sobre a malária..:-). Mas, mesmo a malária é mais controlada em paises civilizados, com alto uso de conhecimento cientifico, que nos pobres e sem esse uso. Ainda é um bom argumento sobre a eficácia da mesma..:-)"



[A] Homero, mesmo a varíola existem focos em alguns paises, ela não esta erradicada; se engana se a malaria, febre-amarela, colega e etc...não tenha em países desenvolvidos, tem, só que não é escancarado como é na América latina...:) duvido se for no Blonks e não encontrar um negro norte-americano doente, duvido...:)



"Finalmente, acho que não compreendeu a questão a emissão de radio e a ida à Lua. Não importa o poder dos USA, não importa seu governo ou o caráter (ou falta de) dos seus governantes, mesmo hoje, se emitir uma onda de rádio, quaisquer dois radio amadores podem dizer, precisamente, de onde vem. Isso se chama triangulação, e é o motivo de se gastar tanto tempo e dinheiro em desenvolvimento de aviões invisíveis, já que, se o aparelho voador reflete a onda do radar, pode ser posicionado precisamente, sem sombra de duvida".



[A] Se foi ou não, foi dinheiro jogado no lixo, sendo que poderia ser empregado na fome imensa na África ou a reconstrução das cidades que eles sempre destruíram...:)





"E ao pousar, e enviar dados do solo lunar, idem, sem possibilidade de fraudar essa emissão."



[A] Eu posso pegar um radio amador e dizer que estou no Japão, como vão saber? Eu se mais ou menos as coordenadas do Japão e posso falar para os leigos, pois é o que aconteceu...:)



"Lutamos a séculos com o virus da gripe e ainda não podemos mata-lo. Sua taxa de mutação, como costuma acontecer com virus, é brutal, e sua adaptação ciclica impede que seja feita uma vacina para todas as cepas. A simplicidade de determinados virus é a chave para que escapem tão facilmente da cura. Um organismo complexo tem estruturas que se repetem em todas as suas variações. Em algum momento encontramos essa estrutura e uma vacina é criada. Mas, organismos simples, em especial virus, mudam tão completamente, que uma estrutura, uma chave, única, não acontece. Isso torna bem dificil derrota-lo. Além disso, a simplicidade faz com que muitas estruturas sejam também usadas por células do organismo hospedeiro. Assim, algumas das drogas que efetivametne matam os virus da gripe não podem ser aplicadas a seres humanos, pois matam suas células saudáveis também (esse é um dos problemas com drogas anti-cancer também atingir células saudaveis juntamente as doentes)."



[A] Por que mexer com isto? Como diz aquele ditado, não se mexe com quem ta quieto, foi isto que ocorreu; uma "besta" em um centro de pesquisa pegou e deixou escapar o HIV e ele contaminou muita gente. A gripe e todas as doenças do tipo viral, ou se preferir, retro-virus ele é mutante porque se vale da célula hospedeira, portanto teríamos que mudar o código genético dos glóbulos brancos para se valer de eficácia. Mas por enquanto, a deusa ciência em sua magnitude, não encontrou um meio de fazer a tal. Portanto caro amigo, fica como jogar ping-pong sem raquete...:)



"A ciência não prometeu curar todas as doenças, consertar o mundo, ser 100% eficaz ou definir A VERDADE (para isso temos as religiòes..:-). Ela é apenas uma ferramenta, uma boa e eficaz ferramenta. Talvez esteja esperando demais da ciência e a frustação de não ter todas as respostas o tenha tornado um pouco rancoroso com ela. Entre duas afirmações, entre duas respostas, a obtida de forma cientifica é a mais confiável, só isso. Existe, sim, uma luta de pesquisadores, seres humanos, para usar essa ferramenta de forma a curar doenças, encontrar respostas, criar conforto, resolver problemas, etc. Mas é uma busca, não um final em sí mesmo."



[A] Ela foi dirigida por Tales de Mileto para descobrir formas de relacionar e pesquisar os fenômenos naturais, que por trás de um trovão ou raio não esta a fúria de algum deus ou coisa parecida. Mas o que vimos hoje é o cientificismo, uma forma derivada de crenças e fundamentalismo que não cabe dentro da ciência e esta atrapalhando algumas pesquisas de cura. Ela não quer ser a VERDADE, mas para alguns, ser cientista é acreditar na filosofia empírica e não é só isso, ela esta vendo em um ângulo só. Eu não sou rancoroso com ela, apenas estou mostrando (por enquanto não) que ela pode ver alem e encontrar o caminho...sobre religiões, creio ser muito pretensioso achar que aquilo ou isso seja mentira, tem-se sim, de investigar.





"Bush é uma "anta" fundamentalista. Mas observe que é uma das pessoas mais contra a ciência que poderíamos encontrar. Seu esforço fundamentalista para obrigar sua visão religiosa de mundo a todo o resto do planeta é frontalmente contrária a ciência, tolerante por natureza. Impedir o estudo de células tronco pode impedir avanços que poderiam curar diversas doenças, inclusive paraplegia. Ensinar criacionismo em escolas e banir a evolução (e a base da biologia moderna hoje) impediria até mesmo que bons médicos se formem ou que novas descobertas sejam feitas."



[A]Bush não é uma "anta" fundamentalista, ele usa a crença popular a seu favor, pois nessas guerrilhas ridículas dele e de seu amiguinho, enquanto suas empresas de petróleo faturam milhões de dólares nessa. Ao proibir, eu apenas vejo ele atender aos interesses da igreja onde foi apoiado, nada mais do que isto e voltamos a era medieval...:)





"Agora, você (quem me lê) deve decidir (pressionado certamente pelos falcões militares) o que fazer. Imagine que estamos em guerra com a Argentina. Você, presidente, tem uma arma recém criada que pode acabar a guerra, devido ao seu poder inimaginável. Detonar essa arma em um cidade vai matar 45 mil pessoas do outro pais (que começou a guerra). Não detonar vai prolongar a guerra por mais alguns meses, a Argentina jamais se renderá e 100 mil brasileiros morrerão, e mais de 350 mil argentinos. O que você decidiria? O que diria as mães e pais de seu pais, quando perguntassem porque não parou a guerra quando podia e escolheu ver morrer mais 100 mil de seus conterrâneos?"



[A] Eu presidente? Uau!!! :)...bom, eu estou numa guerra com a Argentina, aqui no meu ladinho(que arrepio), milhões de tupiniquins vão ou tem perigo de morte...sinceramente destruiria a Casa Rosa...muito eficaz e de tamanha eficiência e ainda punha uma bandeira brasileira na traseira do presidente da la Argentina....:)



"Deve se lembrar que nunca uma arma assim havia sido detonada, e não se sabia o horror que poderia causar (os primeiros testes no deserto americano usaram soldados para verificar o efeito e a descontaminação foi feita com vassouras de palha e água corrente.:-("



[A] Americano é sempre americano muito soldados morreram de câncer ou outro mal, como disse, eles mexem com que esta quieto...:)

"Vou repetir, acho um horror o bombardeio de civis, na verdade, acho qualquer guerra um horror, desnecessária e absurda. Mas não acho que Hiroshima foi apenas maldade, crueldade de monstros, mas uma decisão calculada, difícil e medonha, da qual eu não saberia o que escolher como "menos pior". E hoje temos os USA arrogantes e com jeito de donos do mundo, e nossa antiapatia por Bush nos faz ver tudo com um filtro de asco. Mas na época a guerra dos USA era contra impérios cruéis e mais perigosos que a América jamais será (Hitler, e todo o horror que representa) e não se pode relativizar o que teria acontecido se a Alemanha e o Japão ganhassem a guerra."



[A] A grande Águia ganhou e esta toda cheia de querer mandar no mundo, síndrome de superego...isso é o ponto, ter poder de construir e destruir para se fazer de herói...:)

----- Original Message -----
From: Oraculo
To: ciencialist@yahoogrupos.com.br
Sent: Monday, January 03, 2005 7:47 PM
Subject: Re: [ciencialist] Evolucao e desafios -Amaury


Olá Amaury

Fico contente que goste dos debates e de minha participação (nem sempre é assim, as vezes o pessoal se irrita bastante comigo:-)

Mas ainda acho que está confundindo duas coisas, a ciência, abstrata, e o comportamento humano, passível de ser julgado e avaliado subjetivamente.

É claro que é importante discutir a aplicação, direção, prioridade, etc, de pesquisas e do conhecimento humano. Mas, a ciência, é apenas um conjunto de conhecimentos adquirido de certa forma (método cientifico), não seu uso ou resultado. Isso está confundindo a discussão..:-)

A questao é, se decide agir com eficácia, tomar uma decisão que se mostre confiável no resultado, ou escolher uma afirmação ou conclusão que se aproxime o máximo possível da realidade (sem pretensões de ser 100% ou "A VERDADE"), o que escolheria? Seja para curar pessoas, seja para matar inimigos, o que consideraria mais confiável, mais real ou mais eficaz?

Sua resposa, que certamente é igual a minha, coloca a perspectiva correta na discussão..:-) Ciência, abstratamente, é a melhor ferramenta de compreensão deste universo (universo fisico) que dispomos. O que faremos com o conhecimento assim obtido, é outra coisa. Eu, particularmente, o usaria para melhorar a vida de todo mundo (por exemplo, usando o produto desse conhecimento para enviar ajuda as vitimas do tsunami na Asia). Outros a usariam para matar ou criar armas de destruição em massa.

Mas culpar a ciência, um conhecimento abstrato, pelo uso que malucos fazem dela, é irrelevante e impróprio. Me lembra a piada do matador que, perguntado se não tinah remorso, respondeu: Eu não mato ninguem, só faço o furo, quem mata é Deus (no que ele tem razão, de uma perspectiva religiosa do universo..:-)

Pessoas, seres humanos, matam e maltratam. E fazem isso muito antes de termos armas de destruição em massa ou equivalentes. Culpar o conhecimento do átomo pelo uso da bomba atomica seria como condenar a arma no lugar do atirador (imagino que seria um processo muito engraçado de acompanhar..:-)

Veja este trecho seu:

Amaury: Por isso, nada tenho contra ou a favor da ciência, apenas é uma visão critica sobre...:-)

Se fosse assim, nem haveria a discussão entre nós..:-) Mas seu posicionamento inicial era diferente, contrário a ciência, como se fosse ela responsável por seu uso. Por isso fiz minha análise de sua análise..:-)

A confusão fica mais clara neste trecho:

Amaury:O que temos de melhor, como disse, esta nas mãos dos poderosos, portanto "o que temos de melhor" foi e esta sendo mandado pelo governo dos poderosos. Hoje os norte-americanos mandam e desmandam no que "temos de melhor", tanto se é para o bem da humanidade é imoral, mas se beneficia as partes bélicas deles, seguem numa boa...:-)


Neste caso, o termo "melhor" tem duas conotações, que foram misturadas. O que temos de melhor, no meu sentido, é de mais eficaz. Dá resultados. No seu uso, é um julgamento subjetivo, melhor, bom, correto, etc. Não tem ligação com meu argumento. A ciência é "melhor" se deseja uma ação eficaz, ou um conhecimento confiável. O que fará com esse conhecimento, se algo "melhor", do bem, ou algo "pior", do mal, não é relevante para o argumento.

Já a erradicação da malaria foi um erro, ato falho, meu..:-) Pensei em variola e escrevi malária (acho que passava um especial na TV sobre a malária..:-). Mas, mesmo a malária é mais controlada em paises civilizados, com alto uso de conhecimento cientifico, que nos pobres e sem esse uso. Ainda é um bom argumento sobre a eficácia da mesma..:-)

Finalmente, acho que não compreendeu a questao a emissão de radio e a ida à Lua. Não importa o poder dos USA, nào importa seu governo ou o carater (ou falta de) dos seus governantes, mesmo hoje, se emitir uma onda de rádio, quaisquer dois radio amadores podem dizer, precisamente, de onde vem. Isso se chama triangulação, e é o motivo de se gastar tanto tempo e dinheiro em desenvolvimento de aviões invisiveis, já que, se o aparelho voador reflete a onda do radar, pode ser posicionado precisamente, sem sombra de duvida.

Explicando melhor, se um satélite emite sinal de radio, você, com seu radio amador (ou radar avançado, não importa.:-), pode determinar a direção com precisão. Se outro amigo seu radio amador também o fizer, os dois podem combinar os dados e dizer, com exatidão, de onde parte o sinal e exatamente onde está o satelite. Não há forma de fraudar isso, nem de disfarçar ou fingir estar onde não está. Assim, quando as Apollo orbitavam a Terra, foram acompanhadas de perto por todos que desejassem, sem forma de fraudar isso. Quando orbitaram a Lua, foram acompanhadas (com frustação crescente dos russos..:-), por toda parte.

E ao pousar, e enviar dados do solo lunar, idem, sem possibilidade de fraudar essa emisão.

Assim, se deseja discordar do pouso na Lua, tem de explicar como isso foi feito, ou vai esbarrar em algo chamado evidencia acachapante (que colocou fim nas especulações dos russos, chineses e todos os muitos inimigos dos USA durante a guerra fria que adorariam desmascarar a fraude). Alias, quando os russos colocaram o primeiro satélite, Sputinik, em órbita, alguns jornais americanos e alguns membros do governo americano tentaram vender a idéia de fraude, mas o fato que qualquer um com um bom rádio podia confirmar o feito, foi o bastante para convencer as pessoas.:-)

HIV

Lutamos a séculos com o virus da gripe e ainda não podemos mata-lo. Sua taxa de mutação, como costuma acontecer com virus, é brutal, e sua adaptação ciclica impede que seja feita uma vacina para todas as cepas. A simplicidade de determinados virus é a chave para que escapem tão facilmente da cura. Um organismo complexo tem estruturas que se repetem em todas as suas variações. Em algum momento encontramos essa estrutura e uma vacina é criada. Mas, organismos simples, em especial virus, mudam tão completamente, que uma estrutura, uma chave, única, não acontece. Isso torna bem dificil derrota-lo. Além disso, a simplicidade faz com que muitas estruturas sejam também usadas por células do organismo hospedeiro. Assim, algumas das drogas que efetivametne matam os virus da gripe não podem ser aplicadas a seres humanos, pois matam suas células saudáveis também (esse é um dos problemas com drogas anti-cancer também atingir células saudaveis juntamente as doentes).

Talvez as industrias e governos não invistam o suficiente na pesquisa, eu tendo até a concordar com você nisso. Mas isso indica duas coisas: primeiro, que é caro e dificil descobrir a cura, ou ela já teria sido descoberta mesmo com pouco dinheiro. Segundo, indica que temos FALTA de ciência, que seria possivel com mais recuros, e não exesso. Em qualquer dos casos a crítica feita não se aplica. Precisamos, se queremos encontrar a cura ou tratamentos melhores para a AIDS, de mais ciência, um conheimento confiável e eficaz, e não de menos.

Espectativas

Amaury:Não tenho nada contra a ciência, mas você tem contra os índios...:) o que vejo é ainda crianças mortas e vidas curtas por uma ciência ineficaz, se eu gosto ou não é outra historia, que se eu não gosta-se da ciência não estaria numa lista de ciências...:) e pouco irei a uma tribo, porque não iria acostumar...:)e uma pergunta: a ciência tem vida própria?

A ciência não prometeu curar todas as doenças, consertar o mundo, ser 100% eficaz ou definir A VERDADE (para isso temos as religiòes..:-). Ela é apenas uma ferramenta, uma boa e eficaz ferramenta. Talvez esteja esperando demais da ciência e a frustação de não ter todas as respostas o tenha tornado um pouco rancoroso com ela. Entre duas afirmações, entre duas respostas, a obtida de forma cientifica é a mais confiável, só isso. Existe, sim, uma luta de pesquisadores, seres humanos, para usar essa ferramenta de forma a curar doenças, encontrar respostas, criar conforto, resolver problemas, etc. Mas é uma busca, não um final em sí mesmo.

Tenho as mesmas objeções que você às industrias farmaceuticas (inclusive as homeopaticas..:-). Mas sei que existem pesquisadores legitimamente dedicados a encontrar curas, soluções, respostas, apesar das empresas e até mesmo dentro dessas empresas. Gente que tem legitimos interesses humanos, que pesquisa até mesmo as "doenças de paises pobres" como a malária e febre amarela (erradicadas nos paises desenvlvidos).

Bush é uma "anta" fundamentalista. Mas observe que é uma das pessoas mais contra a ciência que poderiamos encontrar. Seu esforço fundamentalista para obrigar sua visão religiosa de mundo a todo o resto do planeta é frontalmente contrária a ciência, tolerante por natureza. Impedir o estudo de células tronco pode impedir avanços que poderiam curar diversas doenças, inclusive paraplegia. Ensinar criacionismo em escolas e banir a evolução (e a base da biologia moderna hoje) impediria até mesmo que bons médicos se formem ou que novas descobertas sejam feitas.

Ainda assim, ele usa os resultados da ciência onde lhe convém, com armas de destruição e brinquedos tecnologicos que poder. Isso torna a ciência culpada? Ou torna o Bush responsável?

Hiroshima é um caso complicado. Nagazaki seria mais claro, como alvo do debate, desnecessário e violento. Mas Hiroshima é dificil. Antes que "caiam de pau", acho um horror a destruição de uma cidade, a morte de civis e qualquer tipo de conflito. Mas é preciso pensar, sempre, sob pena de não ver com clareza. E Hiroshima é dificil de enxergar, através do horror da bomba e da matança.

Mas havia uma guerra em curso, isso é claro. Havia um pais, Japão, que entrou na guerra por vontade própria, para expadir fronteiras e massacrar vizinhos (embora a vontade a considerar fosse apenas dos mandatarios do pais na época). Havia um comportamento cultural onde a morte era sublime, o sacrificio honroso e o imperador deus (você morreria por seu deus). Dentro dessa análise, continuar a guerra com o Japão produziria, por estimativa, 100 mil mortes de americanos e mais de 350 mil mortes japonesas (estimativas derivadas das batalhas já ocorridas, da decisão de cada soldado japones de só parar depois de morto e das dificuldades de desembarque em ilhas - as piores batalhas da segunda guerra forma em ilhas do Japão).

Agora, você (quem me lê) deve decidir (pressionado certamente pelos falcões militares) o que fazer. Imagine que estamos em guerra com a Argentina. Você, presidente, tem uma arma recem criada que pode acabar a guerra, devido ao seu poder inimaginável. Detonar essa arma em um cidade vai matar 45 mil pessoas do outro pais (que começou a guerra). Não detonar vai prolongar a guerra por mais alguns meses, a Argentina jamais se renderá e 100 mil brasileiros morrerão, e mais de 350 mil argentinos. O que você decidiria? O que diria as mães e pais de seu pais, quando perguntassem porque não parou a guerra quando podia e escolheu ver morrer mais 100 mil de seus conterraneos?

Deve se lembrar que nunca uma arma assim havia sido detonada, e não se sabia o horror que poderia causar (os primeiros testes no deserto americano usaram soldados para verificar o efeito e a descontaminação foi feita com vassouras de palha e agua corrente.:-(

Eu não sei o que escolheria e não gostaria de ter de decidir algo assim. Imagino que não se pode dormir mais depois de ser submetido a coisas desse tipo. Por isso considero que Nagasaki é mais claro, uma decisão certamente militar, resultado de uma perda de poder civil em um periodo dificil. Mas Hiroshima é mais complicado.

Vou repetir, acho um horror o bombardeio de civis, na verdade, acho qualquer guerra um horror, desnecessária e absurda. Mas não acho que Hiroshima foi apenas maldade, crueldade de monstros, mas uma decisão calculada, dificil e medonha, da qual eu não saberia o que escolher como "menos pior". E hoje temos os USA arrogantes e com jeito de donos do mundo, e nossa antiapatia por Bush nos faz ver tudo com um filtro de asco. Mas na época a guerra dos USA era contra imperios crueis e mais perigosos que a america jamais será (Hitler, e todo o horror que representa) e nào se pode relativizar o que teria acontecido se a Alemanhã e o Japão ganhassem a guerra.

De todo modo, estamos mais perto da concordancia que da discordância..:-) A ciência, como ferramenta, é eficaz, confiável e razoável. Seu uso, que pode ser para o bem ou para o mal (mesmo que o bem e o mal sejam relativos, pergunte ao Bin Laden e ao Bush..:-), é que é passível de discussão.

Um abraço.

Homero






----- Original Message -----
From: Amauri Jr
To: ciencialist@yahoogrupos.com.br
Sent: Monday, January 03, 2005 11:45 AM
Subject: [ciencialist] Evolucao e desafios -Homero


Olá Amury


[A] Olá Homero






"Minhas análises sobre suas análises..:-)"



[A] Sinceramente gosto de suas analises, fico satisfeito de você me responder...:-)





"Você parece estar ressentido com o que chama de "ciência" e a trata como entidade independente, crença de cientistas ou algo a ser adorado ou derrubado. Mas nada disso é ciência, nem o resultados de sua aplicação se confundem com ela. Ciência neste contexto (desta lista e do uso padrão do termo..:-) é apenas o conjunto dos conhecimentos obtidos através de um método padronizado, chamado cientifico, e do rigor derivado do mesmo. E, independente do que pense dela, tem elevada confiabilidade..:-)"



[A] Amigo, o que penso que infelizmente o homem usa seu conhecimento para algo não muito para o "bem" humano ou "mal", a natureza humana já é corrupta. Tudo que é humano tem falhas, pois nada ainda é perfeito, mas a ciências como algo que vem para beneficiar a humanidade tinha que ser impartidaria e sem nenhuma crença ou fanatismo; como vimos muito na lista. Por isso, nada tenho contra ou a favor da ciência, apenas é uma visão critica sobre...:-)



"Não importa se o resultado é passível de ser julgado subjetivamente como "mau" ou mesmo como "bom". Importa apenas que é mais eficiente que outras formas de conhecimento, mais eficaz em suas previsões e mais confiável que outros instrumentos de compreensão do universo já criado por seres humanos. E que será abandonado assim que uma ferramenta mais eficaz seja apresentada..:-) Até lá, é o que temos de melhor."



[A] O que temos de melhor, como disse, esta nas mãos dos poderosos, portanto "o que temos de melhor" foi e esta sendo mandado pelo governo dos poderosos. Hoje os norte-americanos mandam e desmandam no que "temos de melhor", tanto se é para o bem da humanidade é imoral, mas se beneficia as partes bélicas deles, seguem numa boa...:-)





"Podemos analisar filosoficamente, subjetivamente, os resultados, mas não discutir a eficácia. A ciência, o conhecimento assim acumulado, é tão eficiente quando cura doenças, como a erradicação da malária, como quando destrói milhões de vidas, como em Hiroshima. O resultado é passível de julgamento, mas a ação não. Ela é eficaz, nos dois casos. Se precisar salvar alguém ou matar alguém, deve usar a ciência, é mais eficaz sempre..:-)"



[A] Podemos sim, tanto que você já disse que ciência é um conjunto de conhecimentos; mas muitos conhecimentos, pois o que vimos, é um "Clube da Ciência" que vê apenas um dos vários lados do universo, mas temos vários e é ainda um incógnito pelo homem ainda não abrir a cabeça...:-) Agora, se erradicamos a malaria não sei, creio que você não esta lendo jornal, pois a malaria, a febre-amarela, o HIV, o câncer, então matando e a ciência "tudo que temos de melhor" fica ai procurando pelos em baratas. Hiroshima foi um ato terrorista covarde e deprimente, que deveria fazer um julgamento dos cientistas e dos governantes no tribunal de Nuremberg, por danos morais humanos.





"Não gostar dela, ou como você pretende com as análises, critica-la ou despreza-la, não importa em nada para sua eficácia e confiabilidade. Você vai viver mais que todos os seus antepassados, goste ou não..:-) Sua expectativa de vida é de 75 anos, e, não sei sua idade, mas se já passou dos 40 como eu, é mais que seu avô esperava viver ao nascer (em 1900 a expectativa de vida no Brasil era de 33 anos)."





[A] Tenho apenas 28 anos, sem filhos e com uma deficiência física, o que sei que são superfluidades da senhora sagrada ciência. O que pode me interessar eu viver 40 ou 70 anos se nem ela pode prever?





"Na verdade, você usou um computador e a Internet, frutos da ciência, para enviar seus pensamentos e ataques a ciência, ao inves de tentar telepatia, rezas ou mandingas (ou qualquer outra forma de comunicação não cientifica) justamente porque é mais eficaz e confiável que qualquer outra..:-)"



[A] Alguma coisa boa tinha que sair daí né?...:-) o problema é esse, vocês da ciência só tem um foco do assunto, não ve as múltiplas variedades de estudos...:)





"Você não parece gostar de muitos dos aspectso do uso do conhecimento cientifico. Direito seu. E pode até mesmo abandona-los, todos, e ir viver em uma aldeia remota no Amazonas, sem (quase) nenhum contato com a ciência (os índios são muito receptivos a forasteiros, vão gostar de recebe-lo), e com todo o ônus de viver dessa forma (filhos mortos, vida curta, doenças diversas, poucos dentes, etc). Mas está na verdade criticando comportamentos humanos, que são seres falhos como sabe, não a ciência. Esta, "estricto senso", não é boa ou má, cruel ou gentil, apenas eficiente, confiável em seus efeitos."



[A] Não tenho nada contra a ciência, mas você tem contra os índios...:) o que vejo é ainda crianças mortas e vidas curtas por uma ciência ineficaz, se eu gosto ou não é outra historia, que se eu não gosta-se da ciência não estaria numa lista de ciências...:) e pouco irei a uma tribo, porque não iria acostumar...:)e uma pergunta: a ciência tem vida própria?



"Você não "acredita" nela. Engraçado, ela não pede que se acredite em nada.:-) Deve duvidar sempre, até que evidencias se mostrem sólidas o bastante para uma conclusão. E é nesse ponto que você escorrega..:-) O pouso na Lua já tem evidencias suficientes para essa conclusão, em que pese as tolices de diversos malucos mundo afora e suas teorias de conspiração. O HIV tem recuado e matado muito menos que no inicio da epidemia, graças a AZT e os atuais coquetéis antivirais. Se ajustes são feitos a toda hora, e são, sobre o que comer ou não, é apenas porque a ciência não pretende ser "A VERDADE" como religiões, mas um conhecimento que cresce, se aprofunda, se ajusta e melhora. Sabemos mais, não menos sobre o que comer ou não comer."



[A] Engraçado que pessoas inteligentes não duvidem que os yanques tenham forjado tudo, mas como você mesmo disse, é um direito seu. Eu acredito muito mais que se possa acreditar, a ciência melhorou minha vida muito, tendo esse computador; em minha cadeira de rodas de alumínio (de ferro era um lastima andar), portanto não estou desacreditando a ciência. Agora se quer achar que os coquetéis são eficazes tudo bem, mas que morre mesmo assim, isso morre...:)



"Sim, dá trabalho, é preciso atenção e constante leitura para acompanhar, seria melhor ter uma única resposta para todas as perguntas e pronto, sem risco e sem mudanças. Mas o mundo não funciona assim..:-) Sabemos hoje mais do que ontem sobre como se alimentar, e saberemos mais amanhã, deixando parte do que sabemos hoje para trás. (Parte, não tudo:-)"



[A] Como se o mesmo alimento hoje esta com proteínas e tudo que precisamos e amanha não?





"As teorias conspiratórias são atrativas.:-) É como se nós, os que estão "por dentro", soubéssemos de "coisas" que o resto do mundo não sabe, fossemos mais "espertos" ou mais inteligentes, que não se deixam enganar. Mas, na maioria das vezes (eu diria em sua totalidade) é apenas o ego tentando ser mais do que é.:-) Mesmo com centenas de sites "desmistificando" o pouso na Lua, o volume de evidencias a favor é gigantesco e nenhum deles explica um ponto fundamental do problema: como o governo americano fraudou os sinais de radio e TV que eram emitidos de todas as naves Apollo tanto durante as viagens, como do solo lunar, captados por todo radioamador ao redor do globo, sem falar nos radares e antenas dos paises em disputassem os USA? (lembre-se que, mesmo hoje, não se pode fraudar a origem de um sinal de radio..:-)"



[A] Não acredito nesses doentes mentais que ficam montando sites para se promover...:) eu tiro dos fatos que ocorreram que podem ser completamente forjados, como as transmissões, ou não sabe que eles são donos do que "temos de melhor"...:)



"Enfim, a ciência é eficaz. Embora seu uso, o uso do conhecimento confiável por ela produzido, possa ser discutido e julgado, isso não é a ciência nem com ela se confunde. E, tenha certeza, seu automóvel vai conduzi-lo amanhã, confiavelmente, conforme as leis da fisica descobertas pela ciência, seu computador receberá esta mensagem, os remédios na farmacia impedirão que você morra, as vacinas que tomou evitarão que adoeça (como todos os seus ancestrais adoeceram) e os satelites em órbita, colocados lá pelo conhecimento cientifico, vão transmitir suas chamadas telefonicas até para o Japão, se você assim desejar..:-)"



[A] Reverencia a ciência como um crente reverencia uma reza...:) o meu carro, vai bater conforma a lei da física, as farmácias venderão remédios que viciam, os satélites vão espionar minha casa e tudo graças a deusa infalível ciência...:)



"Para completar a mensagem anterior: uma menina inglesa de 15 aos foi a primeira pessoa a sobreviver depois de contrair raiva sem tomar a vacina antirabica. Ela foi submetida a um novo tratamento revolucionário (e científico), com um coquetel de drogas e coma induzido."



[A] É? Legal! Mas não são os igreses que usam a ciência para fazer guerrinhas com o Bush?





"Diferente de milagres divinos e sobrenaturais, o que se aprendeu com a nova técnica pode ser usado em outras pessoas doentes e não afetam apenas a pessoa que recebe a graça..:-)"



[A] Também acho, devo adverti-lo novamente que não sou a favor no milagre...:)




"A partir de agora, mesmo desprezando a "ciência", você pode ficar sossegado, se você ou um de seus filhos ou entes queridos contrair raiva, poderá ser curado, cientificamente curado.:-) A não ser que tenha resolvido ir viver em uma aldeia indigena na remota Amazonia..."



[A] Sim vão...vão tomar coca-cola e comer hambúrguer feitos cientificamente e se eu tiver sorte, vão se viciar em LSD feito pela ciência, vão tomar energéticos feito pela ciência, vão ter colesterol de produtos que o "temos de melhor" analisa. O mundo é um paraíso, graças a ciência não acha? ...:)



Abraços

Amauri


---- Original Message -----
From: Oraculo
To: ciencialist@yahoogrupos.com.br
Sent: Sunday, January 02, 2005 11:05 PM
Subject: Re: [ciencialist] Evolucao e desafios - CartaCapital - 02/01/05


Olá Amury

Minhas análises sobre suas análises..:-)

Você parece estar ressentido com o que chama de "ciência" e a trata como entidade independente, crença de cientistas ou algo a ser adorado ou derrubado. Mas nada disso é ciência, nem o resultados de sua aplicação se confundem com ela. Ciência neste contexto (desta lista e do uso padrão do termo..:-) é apenas o conjunto dos conhecimentos obtidos através de um método padronizado, chamado cientifico, e do rigor derivado do mesmo. E, independente do que pense dela, tem elevada confiabilidade..:-)

Não importa se o resultado é passível de ser julgado subjetivametne como "mau" ou mesmo como "bom". Importa apenas que é mais eficiente que outras formas de conhecimento, mais eficaz em suas previsões e mais confiável que outros instrumentos de compreensão do universo já criado por seres humanos. E que será abandonado assim que uma ferramenta mais eficaz seja apresentada..:-) Até lá, é o que temos de melhor.

Podemos analisar filosoficamente, subjetivamente, os resultados, mas não discutir a eficácia. A ciência, o conhecimento assim acumulado, é tão eficiente quando cura doenças, como a erradicação da malária, como quando destroi milhões de vidas, como em Hiroshima. O resultado é passível de julgamento, mas a ação não. Ela é eficaz, nos dois casos. Se precisar salvar alguém ou matar alguém, deve usar a ciência, é mais eficáz sempre..:-)

Não gostar dela, ou como você pretende com as análises, critica-la ou despreza-la, não importa em nada para sua eficácia e confiabilidade. Você vai viver mais que todos os seus antepassados, goste ou não..:-) Sua expectativa de vida é de 75 anos, e, não sei sua idade, mas se já passou dos 40 como eu, é mais que seu avô esperava viver ao nascer (em 1900 a expectativa de vida no Brasil era de 33 anos).

Na verdade, você usou um computador e a Internet, frutos da ciência, para enviar seus pensamentos e ataques a ciência, ao inves de tentar telepatia, rezas ou mandingas (ou qualquer outra forma de comunicação não cientifica) justamente porque é mais eficaz e confiável que qualquer outra..:-)

Você não parece gostar de muitos dos aspectso do uso do conhecimento cientifico. Direito seu. E pode até mesmo abandona-los, todos, e ir viver em uma aldeia remota no Amazonas, sem (quase) nenhum contato com a ciência (os indios são muito receptivos a forasteiros, vão gostar de recebe-lo), e com todo o onus de viver dessa forma (filhos mortos, vida curta, doenças diversas, poucos dentes, etc). Mas está na verdade criticando comportamentos humanos, que são seres falhos como sabe, não a ciência. Esta, "estricto senso", não é boa ou má, cruel ou gentil, apenas eficiente, confiável em seus efeitos.

Você não "acredita" nela. Engraçado, ela não pede que se acredite em nada.:-) Deve duvidar sempre, até que evidencias se mostrem sólidas o bastante para uma conclusão. E é nesse ponto que você escorrega..:-) O pouso na Lua já tem evidencias suficientes para essa conclusão, em que pese as tolices de diversos malucos mundo afora e suas teorias de conspiração. O HIV tem recuado e matado muito menos que no inicio da epidemia, graças a AZT e os atuais coqueteis antivirais. Se ajustes são feitos a toda hora, e são, sobre o que comer ou não, é apenas porque a ciência não pretende ser "A VERDADE" como religiões, mas um conhecimento que cresce, se aprofunda, se ajusta e melhora. Sabemos mais, não menos sobre o que comer ou não comer.

Sim, dá trabalho, é preciso atenção e constante leitura para acompanhar, seria melhor ter uma única resposta para todas as perguntas e pronto, sem risco e sem mudanças. Mas o mundo não funciona assim..:-) Sabemos hoje mais do que ontem sobre como se alimentar, e saberemos mais amanhã, deixando parte do que sabemos hoje para tras. Parte, não tudo:-)

As teorias conspiratórias são atrativas.:-) É como se nós, os que estão "por dentro", soubessemos de "coisas" que o resto do mundo não sabe, fossemos mais "espertos" ou mais inteligentes, que não se deixam enganar. Mas, na maioria das vezes (eu diria em sua totalidade) é apenas o ego tentando ser mais do que é.:-) Mesmo com centenas de sites "desmistificando" o pouso na Lua, o volume de evidencias a favor é gigantesco e nenhum deles explica um ponto fundamental do problema: como o governo americano fraudou os sinais de radio e TV que eram emitidos de todas as naves Apollo tanto durante as viagens, como do solo lunar, captados por todo radio-amador ao redor do globo, sem falar nos radares e antenas dos paises em disputacom os USA? (lembre-se que, mesmo hoje, não se pode fraudar a origem de um sinal de radio..:-)

Enfim, a ciência é eficaz. Embora seu uso, o uso do conhecimento confiável por ela produzido, possa ser discutido e julgado, isso não é a ciência nem com ela se confunde. E, tenha certeza, seu automóvel vai conduzi-lo amanhã, confiavelmente, conforme as leis da fisica descobertas pela ciência, seu computador receberá esta mensagem, os remédios na farmacia impedirão que você morra, as vacinas que tomou evitarão que adoeça (como todos os seus ancestrais adoeceram) e os satelites em órbita, colocados lá pelo conhecimento cientifico, vão transmitir suas chamadas telefonicas até para o Japão, se você assim desejar..:-)

Um abraço.

Homero




----- Original Message -----
From: Amauri Jr
To: ciencialist@yahoogrupos.com.br
Sent: Sunday, January 02, 2005 3:18 PM
Subject: Re: [ciencialist] Evolucao e desafios - CartaCapital - 02/01/05


Minhas analises em baixo com [A]....
----- Original Message -----
From: L.E.R.de Carvalho
To: ciencialist@yahoogrupos.com.br
Sent: Sunday, January 02, 2005 1:52 PM
Subject: [ciencialist] Evolucao e desafios - CartaCapital - 02/01/05



>LabConsS - www.ufrj.br/consumo
>
>
>
> EVOLUÇÃO E DESAFIOS
>
>
> O País viu inúmeros avanços nas áreas médica e
>tecnológica, mas as carências sociais ainda são sua
>pior mazela
>
>Em uma livraria, olho rapidamente as manchetes das
>revistas expostas. A julgar pelas capas, é pouco
>provável que tenhamos problemas de saúde deste mês em
>diante. Soluções curativas. Remédios extremamente
>eficientes. Vida prolongada. Corpos e rostos sem um
>defeito. Independentemente da idade, claro. Câncer?
>Uma brincadeira. Paralisia? Pode preparar as pistas.
>Infarto? E daí?

[A] temos que pagar e muito caro para a "ciencia" nos curar de males que diz a "ciencia" um direito de todos.

>
>Deixando de lado uma razoável dose de exagero nessas
>manchetes, encho o peito de orgulho pelas proezas
>atingidas no campo da medicina e da saúde. Vinte anos
>atrás, tudo isso não passaria de sonho, e não
>existiria fora da cabeça imaginativa dos produtores de
>filmes de ficção científica. Mas, ainda hoje, para a
>esmagadora maioria dos cidadãos brasileiros, todo o
>acima mencionado não passa de um sonho, de uma ficção
>científica

{A] Parece que todo a ciencia é uma ficção cientifica, vamos dizer que o pensamento é um comercio hoje em dia, ficçao ou não, é com certeza sensacionalismo.

>Este ano que acaba nos deu boas notícias,
>encorajadoras, e notícias ruins, no mínimo
>preocupantes. Vimos uma senhora que sofreu um derrame
>cerebral voltar a andar com terapia baseada na
>introdução de células-tronco (células primitivas
>capazes de se transformar em praticamente qualquer
>outra célula normal) no cérebro afetado. Emocionante
>perceber o que esses pequenos passos podem significar
>para milhões de pessoas que não conseguem erguer o
>braço, ou mexer a perna. Parafraseando Neil Armstrong,
>um passo gigante para a humanidade.

{A} Armstrong, quem é ele? Aquele que "supostamente" foi a Lua? Celulas- tronco é um desafio para a humanidade de cura, o desejo de ajudar seu semelhante. Pensamentos inovadores tem estado junto a humanidade a muito tempo, isso pode ser descrito na historia de Tales de Mileto até Einstein




>A técnica das células-tronco não se restringe a
>restaurar a função de células cerebrais. Hoje em dia,
>pesquisadores brasileiros, e em outros países,
>intensificam seus esforços para melhorar o coração
>depois de infarto, os nervos após lesão traumática, a
>pele após queimadura. A lista parece não ter fim.

[A] A demora faz o crescimento dos beneficiados...
>
>Vimos a introdução de remédios geniais na prática
>médica. Geniais na sua concepção, e geniais na sua
>eficiência. Drogas que conseguem agir em um ponto
>específico da célula doente, da célula cancerosa,
>dificultando seu desenvolvimento, seu crescimento, e
>até provocando sua morte. Exemplos que estão
>progressivamente sendo utilizados na prática médica,
>apesar de somente em casos muito selecionados, não
>faltam. Glivec, Iressa, são alguns deles.

[A] Santa ciencia!! Mata e de forma covarde porque usa seus conhecimentos de forma de interesse, mesmo que sabemos, esses conhecimentos são para todos.
>
>Ainda nem acabou o ano e ouvimos há poucas semanas o
>anúncio pela GlaxoSmithKline de uma vacina contra o
>câncer. O Brasil está totalmente empenhado na produção
>da vacina contra o HPV, vírus causador de câncer de
>útero. O prof. dr. Ricardo Brentani, presidente da
>Fundação Antonio Prudente, Hospital do Câncer AC
>Camargo, está entusiasmado e declara:
>
>­ Felizmente nossa instituição foi parceira da Merck
>Sharp & Dohme no desenvolvimento de uma vacina contra
>o HPV. Minha esperança é que em 20 anos tenhamos
>prevenido 7% dos tumores humanos.

[A] Vacina? Como pode ter vacina para nossas proprias celulas? E do HIV vai ter?

>Vimos, por outro lado, a preocupação crescente dos
>cientistas em não excluir, a priori, tratamentos
>considerados até então não-convencionais, ou
>complementares. Estudos sérios avaliaram desde a
>acupuntura até a homeopatia, passando por terapias de
>Florais de Bach e tratamentos com vitaminas nas mais
>diversas situações clínicas. Vantagens e desvantagens
>de cada abordagem foram dissecadas minuciosamente.

[A] Viu? Sera que alguem aqui duvidda?


>Cientistas conseguiram demonstrar a eficiência
>incontestável de algumas terapias alternativas em
>certas situações, como a massagem para um dos males do
>século, a dor nas costas. Por outro lado, alertaram
>para a ineficiência e até os efeitos nocivos de outras
>terapias, em outras condições. Por exemplo, para
>tratar bronquite e asma a acupuntura não parece ter
>efeitos importantes. Pelo menos não conseguiram
>detectar esses efeitos nos estudos atuais.

[A] Remedios sim? Eu tive começo, eu e meus manos, de bronquite depois que meu pai comprou um tartaruga nunca mais...santa crença ne? Mas como explicar??

>Vimos o lançamento de aparelhos ultramodernos,
>ultra-sensíveis, para detectar doenças e tratá-las.
>Não consigo perceber avanço recente maior do que na
>área da radioterapia. É incrível a precisão dos feixes
>de radiação em atingir o alvo, no caso o câncer, e
>poupar o tecido normal adjacente. Aparelhos novos
>conseguem acompanhar o movimento do corpo para seguir
>o alvo predeterminado, segundo a segundo, obedecendo à
>orientação do médico radioterapeuta. Reduziram-se
>muito os efeitos colaterais. O controle do câncer
>assemelha-se às extensas cirurgias. A nanotecnologia
>(aparelhos miniaturas) é uma febre, e os estudos
>multiplicam-se para definir com mais clareza sua
>aplicação.

{A} Pode matar com sua radioatividade, causando cancer.

>
>A plástica está fazendo tamanhos milagres que nem os
>próprios pacientes conseguem acreditar. Tanto faz quem
>foram seus pais, ou a etnia a qual você pertença.
>Escolha o modelo e ficará parecido. Para pessoas
>ansiosas com a imagem, soluções para quase tudo. Sem
>dúvida, o impacto sobre o estado emocional é notável.

[A] Eita faquinha de dois cumes heim? A plastica pode curar a pessoa de queimaduras ou cicatrizes, mas tambem pode ressaltar sua vaidade.

>
>Ao lado dessas notícias que mereceram um destaque
>quase obsessivo nas manchetes de capa, outras
>informações com menos, digamos, glamour, não
>conseguiram espaço nem nas páginas finais das
>revistas.
[A] A ciencia ja tem a imprensa e seus colaborados para dar esse glamour...


>
>O objetivo de reduzir em 60% a mortalidade infantil no
>mundo (e o Brasil ainda é um grande protagonista dessa
>estatística) não será atingido em 2015, como
>estabelecido nos Objetivos Milenares da ONU. No
>planeta, morrem por ano mais de 11 milhões de crianças
>com idade inferior a 5 anos. A maioria por doenças
>evitáveis. Diarréia, pneumonia, malária. No mesmo
>período, 500 mil mulheres morrem durante a gravidez ou
>o parto. Doença de Chagas e esquistossomose
>(barriga-d'água) continuam afetando milhares de
>brasileiros. E seu controle está cada vez mais
>próximo. A implementação dos programas já em ação
>poderá melhorar ainda mais esse controle. Para 2005 a
>intensificação dessas abordagens poderá elevar o
>impacto na saúde da população, principalmente nas
>áreas rurais.

[A] Isso ai, como fica doenças tao mais antiga diante do deusa peerfeita ciencia??


>O Relatório Mundial de Saúde (The World Health Report
>2003) recomendou às autoridades o fortalecimento dos
>sistemas de saúde, centralizando seu foco na atenção
>primária, além de integrar a prevenção das doenças e a
>promoção da saúde em todos os níveis de atendimento.

[A] Esses relatorios tem efeito??

>
>A saúde do homem sofreu mudanças drásticas nos últimos
>anos. A expectativa de vida, de brasileiros e de não
>brasileiros, bate recorde atrás de recorde.
>Ultrapassou a marca dos 70 anos e logo passará dos 80,
>90, e quem sabe 100 anos. Avanços sem dúvida notáveis.
>Doenças contagiosas foram substituídas por doenças
>crônicas, como problemas cardiovasculares e câncer, e
>por causas externas, como trauma. Essa mudança de foco
>exigiu modificações intensivas no nível estrutural.
>Algumas especialidades assumiram posição de destaque
>nunca antes alcançada. Os serviços de cardiologia de
>muitos hospitais logo se transformaram em
>departamentos e, a seguir, em grandes centros com
>prédios próprios. O mesmo ocorreu com a oncologia e
>suas variadas especialidades. Houve uma explosão na
>tecnologia e na sofisticação, e, conseqüentemente, nos
>custos ­ alertam os especialistas em saúde pública.
>Recomenda-se também a melhora da saúde da população
>por intermédio do envolvimento acadêmico.
>

[A] Sem comentarios, pra que vou querer viver até o 100?
>
>Muito por fazer.
>Como melhorar o atendimento ao público do SUS

[A] Infelizmente a ciencia não melhora ela mesma vai melhorar o SUS, a ciencia infelizmente, anda atendendo interesses dos laboratorios e pondo em pratica, o que determina o codigo americano, Bisness in bisness...
>
>
>A academia médica no Brasil foi alterada sensivelmente
>nas últimas duas décadas. Progressivamente, as escolas
>médicas e os programas de pós-graduação introduziram
>disciplinas de pesquisa epidemiológica de problemas de
>saúde adaptados à realidade da população do País. Mais
>e mais livros têm sido publicados por acadêmicos
>brasileiros, com enfoque nacional. Mais e mais centros
>de treinamento oferecem vagas para médicos
>interessados em se aperfeiçoar no atendimento básico à
>saúde da família e à clínica geral. O impacto na
>melhora da atenção à saúde do brasileiro pode demorar
>a ser notado. Muito há de ser feito em 2005 e além.

[A] Infelizmente, tem muito moleque fazendo medicina pro papai e só sai meleca, ou o diagnostico é virose ou é exames a "toa"...


>O plano Fome Zero tenta corrigir um problema básico de
>saúde: a miséria e a conseqüente desnutrição. Ninguém
>precisa de doutorado para compreender a conexão direta
>entre a desnutrição e a ocorrência de doenças
>potencialmente graves. É um plano interessante, mas
>ainda longe de atingir seus objetivos de forma
>significativa. Em 2005, e além, o governo deverá fazer
>os ajustes finos para que o programa consiga alcançar
>seu alvo.
>
>A violência é problema de segurança pública, mas
>também é problema de saúde. Não podemos esquecer que,
>no ano passado, mais de 40 mil brasileiros foram
>assassinados. Se acreditarmos nos dados oficiais do
>Ministério da Saúde, naquele período, morreram mais
>brasileiros por tiro do que por câncer de pulmão. O
>controle da violência evitará milhares de mortes e
>seqüelas dramáticas.
>
>A malária é outro problema sem solução a curto prazo.
>Áreas extensas do território nacional são infestadas
>por mosquitos portadores dessa doença. O esforço das
>autoridades regionais e federais deverá ser
>intensificado nas várias frentes: pesquisa de vacinas,
>profilaxia, controle do mosquito e tratamento de
>pacientes infectados. A mortalidade por essa infecção
>deve ser reduzida a todo custo. Estudos recentes
>aventam que a vacina contra a malária pode estar ao
>alcance da ciência em prazo razoável.
>
>No mundo todo, um bilhão de pessoas são infectadas por
>parasitas, como os vermes, e o Brasil tem participação
>significativa nesses dados alarmantes. Vermes no
>intestino não são apenas feios. Eles causam problemas
>como deficiências nutricionais, e, em alguns casos,
>podem até ser fatais. Para mudar isso, saneamento
>básico é fundamental.
>
>Um estudo recentemente realizado e publicado por
>pesquisadores na Universidade do Ceará demonstrou
>claramente a possibilidade de praticamente eliminar os
>parasitas dos pacientes. Remédios eficazes, como a
>ivemerctina, podem ser a solução. Além de tratar o
>paciente, eliminam uma potencial fonte de contaminação
>para outras pessoas. As autoridades devem criar
>sistemas integrados, contínuos, para atacar esse
>problema de saúde pública em várias frentes, em 2005 e
>para sempre.
>
>Quanto à Aids, nos últimos anos, houve um declínio da
>incidência de novos casos na maioria dos estados
>brasileiros. Estudo publicado recentemente por
>pesquisadores da Fundação Oswaldo Cruz, no Rio de
>Janeiro, confirmou que, apesar de recursos limitados
>do governo e da desigualdade socioeconômica aberrante
>no Brasil, a introdução do acesso universal à terapia
>antiviral contribuiu para uma redução impressionante
>na mortalidade por Aids, e pode ter ajudado a diminuir
>sua incidência.
>
>Nessa linha de pensamento, o presidente Lula lançou,
>este ano, o louvável Plano Nacional de Eliminação da
>Hanseníase (a lepra) em até dois anos. Um passo sem
>dúvida importante para controlar essa doença
>contagiosa e debilitante. Paralelamente, o ministro da
>Saúde, Humberto Costa, anunciou a formação de uma
>comissão para promover políticas nacionais de
>reabilitação às pessoas afetadas pela hanseníase:
>
>­ A idéia é possibilitar benefícios, como a
>reabilitação por cirurgias plásticas, financiando para
>a capacitação de médicos, enfermeiros e
>fisioterapeutas para que realizem esses procedimentos
>nos incapacitados fisicamente.
>
>Todos os anos, 42 mil brasileiros contraem a
>hanseníase. E o País só perde para a Índia no ranking
>mundial de casos da doença.
>
>A obesidade, mal do século XXI, mata. Nos EUA, em
>alguns estados o número de óbitos por obesidade
>ultrapassou o de câncer. No Brasil, cientistas
>identificaram obesidade crescente na população. Estudo
>publicado por pesquisadores da Universidade Federal do
>Rio de Janeiro confirmou o aumento progressivo do peso
>dos adolescentes, tanto na Região Nordeste quanto no
>Sudeste do País, desde 1975. Atualmente, 17% dos
>adolescentes da Região Sudeste enquadram-se na
>definição internacional de obesidade. E seus efeitos
>maléficos logo alcançarão a saúde pública. Orientação
>e conscientização devem ser política contínua e bem
>estruturada. A prevenção e o tratamento do sobrepeso
>precisam ser prioridades das políticas de saúde nos
>próximos anos.
>
>Um problema que acomete principalmente as crianças é a
>poluição atmosférica. Um estudo realizado com 5.193
>crianças de duas cidades do Rio de Janeiro, e
>publicado recentemente na revista Annals of Allergy
>Asthma and Immunology, mostrou claramente a correlação
>entre os níveis de poluição e a incidência de
>problemas respiratórios. Observou-se um aumento de
>mais de 50% na freqüência de crises de asma nos
>moradores de Duque de Caxias (local com alta
>concentração de poluentes), comparados aos moradores
>de Seropédica (local com baixos níveis de poluição).
>Outro estudo, realizado pelos pesquisadores do
>laboratório de poluição atmosférica da Universidade de
>São Paulo, confirmou os efeitos nocivos da poluição do
>ar na mortalidade infantil. Há aumento de 6% no número
>de óbitos neonatais nas regiões mais poluídas. O
>controle de poluentes de qualquer origem deverá ser
>obrigação mundial. Ainda mais com a entrada em vigor
>do Protocolo de Kyoto.
>
>Mas, de forma geral, provavelmente o maior desafio das
>autoridades de saúde é garantir um apoio ao avanço
>científico na área de saúde, atingir a maioria da
>população com os avanços médicos e tecnológicos
>alcançados, e insistir na prevenção e na saúde
>primária, básica. Muito mais do que dinheiro, em 2005
>precisamos de estratégia, filosofia e enfoque novos.
>
>O acesso da população, principalmente das camadas
>menos privilegiadas da sociedade, a remédios em geral
>é muito limitado. Várias são as causas, como preços
>elevados na fonte, acréscimos excessivos na cadeia de
>distribuição e impostos. Se as autoridades, de todas
>as esferas, não atentarem a essas dificuldades
>enfrentadas no dia-a-dia da maioria dos cidadãos,
>muito mais será gasto para tratar complicações graves
>de doenças crônicas, como diabetes e hipertensão
>arterial. O Brasil iniciou uma ação interessante nessa
>direção: a aquisição da fábrica da GlaxoSmithKline do
>Brasil pelo governo federal. O presidente Lula
>destacou que "pela primeira vez um governo compra uma
>fábrica da iniciativa privada", ao mesmo tempo
>criticando "o processo inverso de privatização do
>governo anterior". Lula declarou, na ocasião, que
>estava "recuperando uma fábrica que seria desativada,
>eliminaria empregos e que agora produzirá, em escala
>nacional, os antibióticos mais usados no Brasil".
>
>Nessa mesma linha o ministro da Saúde, Humberto Costa,
>deixou muito clara sua estratégia para 2005:
>
>­ Vamos ampliar o acesso da população a medicamentos
>fundamentais com base em três opções. A primeira é a
>rede de farmácias populares, e até o fim do ano
>pretendemos inaugurar cem unidades em todo o Brasil. A
>segunda é a criação de um programa para a venda
>subsidiada de medicamentos básicos para hipertensão e
>diabetes, através da rede de farmácias privada.
>Pretendemos baixar os preços em até 50%. E a terceira
>é a redução do ICMS de 2,8 mil medicamentos até 2005.
>
>De acordo com uma pesquisa do IBGE, de 2003, a saúde
>aparece em terceiro lugar no orçamento das famílias
>brasileiras, e os medicamentos representam 61% desses
>gastos para as pessoas de baixa renda. Além disso,
>metade das pessoas que precisam de tratamento não pode
>pagar os remédios de que necessitam.
>
>A partir de 2005, 50 milhões de unidades de
>antibióticos serão produzidas para a rede do Sistema
>Único de Saúde (SUS) e para as Farmácias Populares. Em
>2007, a produção deve quintuplicar em relação à atual.
>Mais de 10 bilhões de unidades de medicamentos devem
>ser produzidas para as principais doenças que mais
>atingem a população brasileira, como hipertensão,
>diabetes, malária e tuberculose. A Fiocruz também
>produzirá mais vacinas, e o Ministério da Saúde
>enfatiza que "a ampliação do acesso da população aos
>medicamentos é uma das prioridades do governo
>federal".
>
>Em 2004, muito se fez pela saúde, mas ainda há muito
>mais por fazer. Programas de prevenção e detecção
>precoce de doenças sexualmente transmissíveis, como
>Aids, HPV (câncer de colo de útero), a melhoria na
>qualidade do atendimento dos pacientes do SUS,
>diminuição das filas, controle de poluição, de
>infecção hospitalar e, também, de mortes por agentes
>externos, como traumas (tiros, facadas, acidentes de
>trânsito). Para se ter uma idéia do custo com os
>acidentes de trânsito, a cada ano acontecem mais de
>100 mil mortes, com três a quatro vezes esse número de
>feridos. E a maioria absoluta é tratada com dinheiro
>público. São milhões de reais de gastos evitáveis. Sem
>falar nas seqüelas individuais.
>

Abraços
Amauri

[As partes desta mensagem que não continham texto foram removidas]



##### ##### #####

Para saber mais visite
http://www.ciencialist.hpg.ig.com.br


##### ##### ##### #####



Yahoo! Grupos, um serviço oferecido por:
PUBLICIDADE




------------------------------------------------------------------------------
Links do Yahoo! Grupos

a.. Para visitar o site do seu grupo na web, acesse:
http://br.groups.yahoo.com/group/ciencialist/

b.. Para sair deste grupo, envie um e-mail para:
ciencialist-unsubscribe@yahoogrupos.com.br

c.. O uso que você faz do Yahoo! Grupos está sujeito aos Termos do Serviço do Yahoo!.



[As partes desta mensagem que não continham texto foram removidas]



##### ##### #####

Para saber mais visite
http://www.ciencialist.hpg.ig.com.br


##### ##### ##### #####


Yahoo! Grupos, um serviço oferecido por:







------------------------------------------------------------------------------
Links do Yahoo! Grupos

a.. Para visitar o site do seu grupo na web, acesse:
http://br.groups.yahoo.com/group/ciencialist/

b.. Para sair deste grupo, envie um e-mail para:
ciencialist-unsubscribe@yahoogrupos.com.br

c.. O uso que você faz do Yahoo! Grupos está sujeito aos Termos do Serviço do Yahoo!.



[As partes desta mensagem que não continham texto foram removidas]



##### ##### #####

Para saber mais visite
http://www.ciencialist.hpg.ig.com.br


##### ##### ##### #####


Yahoo! Grupos, um serviço oferecido por:







------------------------------------------------------------------------------
Links do Yahoo! Grupos

a.. Para visitar o site do seu grupo na web, acesse:
http://br.groups.yahoo.com/group/ciencialist/

b.. Para sair deste grupo, envie um e-mail para:
ciencialist-unsubscribe@yahoogrupos.com.br

c.. O uso que você faz do Yahoo! Grupos está sujeito aos Termos do Serviço do Yahoo!.



[As partes desta mensagem que não continham texto foram removidas]



##### ##### #####

Para saber mais visite
http://www.ciencialist.hpg.ig.com.br


##### ##### ##### #####


Yahoo! Grupos, um serviço oferecido por:







------------------------------------------------------------------------------
Links do Yahoo! Grupos

a.. Para visitar o site do seu grupo na web, acesse:
http://br.groups.yahoo.com/group/ciencialist/

b.. Para sair deste grupo, envie um e-mail para:
ciencialist-unsubscribe@yahoogrupos.com.br

c.. O uso que você faz do Yahoo! Grupos está sujeito aos Termos do Serviço do Yahoo!.



[As partes desta mensagem que não continham texto foram removidas]



##### ##### #####

Para saber mais visite
http://www.ciencialist.hpg.ig.com.br


##### ##### ##### #####


Yahoo! Grupos, um serviço oferecido por:
PUBLICIDADE




------------------------------------------------------------------------------
Links do Yahoo! Grupos

a.. Para visitar o site do seu grupo na web, acesse:
http://br.groups.yahoo.com/group/ciencialist/

b.. Para sair deste grupo, envie um e-mail para:
ciencialist-unsubscribe@yahoogrupos.com.br

c.. O uso que você faz do Yahoo! Grupos está sujeito aos Termos do Serviço do Yahoo!.



[As partes desta mensagem que não continham texto foram removidas]



##### ##### #####

Para saber mais visite
http://www.ciencialist.hpg.ig.com.br


##### ##### ##### #####


Yahoo! Grupos, um serviço oferecido por:
PUBLICIDADE




------------------------------------------------------------------------------
Links do Yahoo! Grupos

a.. Para visitar o site do seu grupo na web, acesse:
http://br.groups.yahoo.com/group/ciencialist/

b.. Para sair deste grupo, envie um e-mail para:
ciencialist-unsubscribe@yahoogrupos.com.br

c.. O uso que você faz do Yahoo! Grupos está sujeito aos Termos do Serviço do Yahoo!.



[As partes desta mensagem que não continham texto foram removidas]



SUBJECT: Tabela Periodica on line
FROM: "Cyberlander" <cybernews@superig.com.br>
TO: <Undisclosed-Recipient:;>
DATE: 04/01/2005 04:03



Para ajudar nos estudos !!!! Tabela periodica on line....

http://www.ivstandards.com/extras/pertable/


CYBERLANDER

Ama a realidade que constróis,
que nem a morte deterá teu voo · ·




[As partes desta mensagem que não continham texto foram removidas]



SUBJECT: Maluquinhos de Astrofísica: VINTE artigos!!!
FROM: Maria Natália <grasdic@hotmail.com>
TO: ciencialist@yahoogrupos.com.br
DATE: 04/01/2005 04:56


Aqui vai a prenda de reis da Natália:
Mas menino aí no Brasil, não está de férias!? Dá então para ler
estas matérias in. Mas também eu estou aqui para arreliar Leo e como
convém iniciar bem o ano...
Mas não foi você que perguntou:
1--como se pode provar que o espaço é curvo? Nada mais simples:
pegue numa supernova!
2--existem partículas de matéria escura?
3--O calorímetro da PAMELA está a portar-se bem?
4--E o ciclo solar e a dinãmica da estratosfera-troposfera?
5--Nigth Glow o que é?
indice de autores...
etc etc etc
Vos posso fornecer os Abstracts

Energy from the bulk through parametric resonance
Physics Letters B, Volume 605, Issues 1-2, 6 January 2005, Pages 1-8
F.K. Diakonos, E.N. Saridakis and N. Tetradis

Probing the curvature of the Universe from supernova measurement
Physics Letters B, Volume 605, Issues 1-2, 6 January 2005, Pages 9-
14 Bin Wang, Yungui Gong and Ru-Keng Su

Unstable relics as a source of galactic positrons
Physics Letters B, Volume 605, Issues 1-2, 6 January 2005, Pages 15-
25 Charles Picciotto and Maxim Pospelov

Higher-dimensional dilaton black holes with cosmological constant
Physics Letters B, Volume 605, Issues 1-2, 6 January 2005, Pages 185-
189 Chang Jun Gao and Shuang Nan Zhang

Particle dark matter : evidence, candidates and constraints
Physics Reports, Volume 405, Issues 5-6, January 2005, Pages 279-390
Gianfranco Bertone, Dan Hooper and Joseph Silk

Flux limits on ultra high energy neutrinos with AMANDA-B10
Astroparticle Physics, Volume 22, Issues 5-6, January 2005, Pages
339-353 M. Ackermann, J. Ahrens, H. Albrecht, D. Atlee, X. Bai, R.
Bay, M. Bartelt, S.W. Barwick, T. Becka, K.H. Becker, J.K. Becker,
E. Bernardini, D. Bertrand, D.J. Boersma, S. Böser, O. Botner, A.
Bouchta, O. Bouhali, J. Braun, C. Burgess, T. Burgess, T.
Castermans, D. Chirkin, T. Coarasa, B. Collin, J. Conrad, J. Cooley,
D.F. Cowen, A. Davour, C. De Clercq, T. DeYoung, P. Desiati, P.
Ekström, T. Feser, T.K. Gaisser, R. Ganugapati, H. Geenen, L.
Gerhardt, A. Goldschmidt, A. Groß, A. Hallgren, F. Halzen, K.
Hanson, D. Hardtke, R. Hardtke, T. Harenberg, T. Hauschildt, K.
Helbing, M. Hellwig, P. Herquet, G.C. Hill, J. Hodges, D. Hubert, B.
Hughey, P.O. Hulth, K. Hultqvist, S. Hundertmark, J. Jacobsen, K.H.
Kampert, A. Karle, J. Kelley, M. Kestel, L. Köpke, M. Kowalski, M.
Krasberg, K. Kuehn, H. Leich, M. Leuthold, J. Lundberg, J. Madsen,
K. Mandli, P. Marciniewski, H.S. Matis, C.P. McParland, T.
Messarius, Y. Minaeva, P. Miočinović, R. Morse, S. Movit, K.
Münich, R. Nahnhauer! , J.W. Nam, T. Neunhöffer, P. Niessen, D.R.
Nygren, H. Ögelman, Ph. Olbrechts, C. Pérez de los Heros, A.C. Pohl,
R. Porrata, P.B. Price, G.T. Przybylski, K. Rawlins, E. Resconi, W.
Rhode, M. Ribordy, S. Richter, J. Rodríguez Martino, D. Rutledge,
H.G. Sander, K. Schinarakis, S. Schlenstedt, D. Schneider, R.
Schwarz, A. Silvestri, M. Solarz, G.M. Spiczak, C. Spiering, M.
Stamatikos, D. Steele, P. Steffen, R.G. Stokstad, K.H. Sulanke, I.
Taboada, O. Tarasova, L. Thollander, S. Tilav, L.C. Voicu, W.
Wagner, C. Walck, M. Walter, Y.R. Wang, C.H. Wiebusch, R.
Wischnewski, H. Wissing, K. Woschnagg and G. Yodh

Neutrino detection with CLEAN
Astroparticle Physics, Volume 22, Issues 5-6, January 2005, Pages
355-368 D.N. McKinsey and K.J. Coakley

The winter NAO pattern changes in association with solar and
geomagnetic activity
Journal of Atmospheric and Solar-Terrestrial Physics, Volume 67,
Issues 1-2, January 2005, Pages 17-32 J. Bochníček and P. Hejda

Photo-coulomb neutrino process
Astroparticle Physics, Volume 22, Issues 5-6, January 2005, Pages
369-375 Indranath Bhattacharyya

Uncertainties in limits on TeV-gravity from neutrino-induced showers
Astroparticle Physics, Volume 22, Issues 5-6, January 2005, Pages
377-385 Eun-Joo Ahn, Marco Cavaglià and Angela V. Olinto

A Heitler model of extensive air showers
Astroparticle Physics, Volume 22, Issues 5-6, January 2005, Pages
387-397 J. Matthews

Microscopic black hole detection in UHECR: the double bang signature
Astroparticle Physics, Volume 22, Issues 5-6, January 2005, Pages
399-407 V. Cardoso, M.C. Espírito Santo, M. Paulos, M. Pimenta and
B. Tomé

The solar cycle and stratosphere–troposphere dynamical coupling
Journal of Atmospheric and Solar-Terrestrial Physics, Volume 67,
Issues 1-2, January 2005, Pages 71-82 Mark P. Baldwin and Timothy J.
Dunkerton

Simulation studies of neutron shielding, calibration and veto
systems for gaseous dark matter detectors
Astroparticle Physics, Volume 22, Issues 5-6, January 2005, Pages
409-420 P.F. Smith, D. Snowden-Ifft, N.J.T. Smith, R. Luscher and
J.D. Lewin

Reanalysis of GU miniarray data using CORSIKA
Astroparticle Physics, Volume 22, Issues 5-6, January 2005, Pages
421-429 U.D. Goswami, K. Boruah, P.K. Boruah and T. Bezboruah

Technical design and performance of the NEMO 3 detector
Nuclear Instruments and Methods in Physics Research Section A:
Accelerators, Spectrometers, Detectors and Associated Equipment,
Volume 536, Issues 1-2, 1 January 2005, Pages 79-122 R. Arnold, C.
Augier, A.M. Bakalyarov, J. Baker, A. Barabash, Ph. Bernaudin, M.
Bouchel, V. Brudanin, A.J. Caffrey, J. Cailleret, J.E. Campagne, D.
Dassié, V. Egorov, K. Errahmane, A.I. Etienvre, T. Filipova, J.
Forget, A. Guiral, P. Guiral, J.L. Guyonnet, F. Hubert, Ph. Hubert,
B. Humbert, R. Igersheim, P. Imbert, C. Jollet, S. Jullian, I.
Kisel, A. Klimenko, O. Kochetov, V. Kovalenko, D. Lalanne, F.
Laplanche, B. Lavigne, V.I. Lebedev, J. Lebris, F. Leccia, A.
Leconte, I. Linck, C. Longuemare, Ch. Marquet, G. Martin-Chassard,
F. Mauger, I. Nemchenok, I. Nikolic-Audit, H. Ohsumi, S. Pécourt, F.
Piquemal, J.L. Reyss, A. Richard, C.L. Riddle, J. Rypko, X. Sarazin,
L. Simard, F. Scheibling, Yu. Shitov, A. Smolnikov, I. Štekl, C.S.
Sutton, G. Szklarz, V. Timkin, V. Tretyak, V. Umatov, L. Vála, I.
Vanushin, S. Vasiliev, V. Vasilyev, V. Vorobel, Ts. Vylov, J. Wurtz
and S.V. Zhukov

A particle classification system for the PAMELA calorimeter
Astroparticle Physics, Volume 22, Issues 5-6, January 2005, Pages
431-438 R. Bellotti, M. Boezio and F. Volpe

NIGHTGLOW: an instrument to measure the Earth's nighttime
ultraviolet glow—results from the first engineering flight
Astroparticle Physics, Volume 22, Issues 5-6, January 2005, Pages
439-449 Louis M. Barbier, Robert Smith, Scott Murphy, Eric R.
Christian, Rodger Farley, John F. Krizmanic, John W. Mitchell,
Robert E. Streitmatter, Eugene C. Loh and Stephen Stochaj

Geomagnetic storms, Forbush decreases of cosmic rays and total
ozone at northern higher middle latitudes
Journal of Atmospheric and Solar-Terrestrial Physics, Volume 67,
Issues 1-2, January 2005, Pages 119-124 Jan Laštovička and Peter
Križan

Detailed calculation of test-mass charging in the LISA mission
Astroparticle Physics, Volume 22, Issues 5-6, January 2005, Pages
451-469 H.M. Araújo, P. Wass, D. Shaul, G. Rochester and T.J.
Sumner

Author index
Astroparticle Physics, Volume 22, Issues 5-6, January 2005, Pages
471-484

Empirical model of the dynamics in the mesosphere and lower
thermosphere region over the UK, including solar and geomagnetic
activity
Journal of Atmospheric and Solar-Terrestrial Physics, Volume 67,
Issues 1-2, January 2005, Pages 197-209 D. Pancheva, Pl. Mukhtarov,
N.J. Mitchell and H.G. Muller

Solar cosmic rays and changes in the polar middle atmosphere and
ionosphere
Journal of Atmospheric and Solar-Terrestrial Physics, Volume 67,
Issues 1-2, January 2005, Pages 211-218 Adriena Ondrášková and
Alexei A. Krivolutsky

HST/ACS imaging of a Balmer-dominated shock in Kepler's supernova
remnant
Advances in Space Research, In Press, Corrected Proof, Available
online 22 December 2004 Ravi Sankrit, William P. Blair, Tracey
DeLaney, Lawrence Rudnick, Ilana M. Harrus and Jessica A. Ennis

GRAPES-3—a high-density air shower array for studies on the
structure in the cosmic -ray energy spectrum near the knee
Nuclear Instruments and Methods in Physics Research Section A:
Accelerators, Spectrometers, Detectors and Associated Equipment, In
Press, Uncorrected Proof, Available online 22 December 2004 S.K.
Gupta, Y. Aikawa, N.V. Gopalakrishnan, Y. Hayashi, N. Ikeda, N. Ito,
A. Jain, A.V. John, S. Karthikeyan, S. Kawakami, T. Matsuyama, D.K.
Mohanty, P.K. Mohanty, S.D. Morris, T. Nonaka, A. Oshima, B.S. Rao,
K.C. Ravindran, M. Sasano, K. Sivaprasad, B.V. Sreekantan, H.
Tanaka, S.C. Tonwar, K. Viswanathan and T. Yoshikoshi""

As melhoras.
Eu!? Me estou recompondo LOLLLL
Um abraço
Maria Natália





SUBJECT: Re: [ciencialist] Super pião
FROM: "Luiz Ferraz Netto" <leobarretos@uol.com.br>
TO: <ciencialist@yahoogrupos.com.br>
DATE: 04/01/2005 06:38

E eu ainda não consegui fazer meu pião ficar girando nem por 1 minuto!
Pobre do meu levitron!
[]'
===========================
Luiz Ferraz Netto [Léo]
leobarretos@uol.com.br
http://www.feiradeciencias.com.br
===========================
-----Mensagem Original-----
De: "Kentaro Mori" <kentaro.mori@itelefonica.com.br>
Para: <ciencialist@yahoogrupos.com.br>
Enviada em: segunda-feira, 3 de janeiro de 2005 18:34
Assunto: [ciencialist] Super pião




Gira por +15 minutos:
http://www.quirkle.com/top/index.htm

Que acham?

Prof Léo, sacrilégio: um levitron foi vendido em um leilão on-line
japonês por nada menos que 730 ienes. Capaz de vir de brinde com um
McLanche Feliz daqui a pouco.

Abraços,

Mori





##### ##### #####

Para saber mais visite
http://www.ciencialist.hpg.ig.com.br


##### ##### ##### #####
Links do Yahoo! Grupos










--
No virus found in this incoming message.
Checked by AVG Anti-Virus.
Version: 7.0.298 / Virus Database: 265.6.7 - Release Date: 30/12/2004




--
No virus found in this outgoing message.
Checked by AVG Anti-Virus.
Version: 7.0.298 / Virus Database: 265.6.7 - Release Date: 30/12/2004



SUBJECT: Re: [ciencialist] Super pião
FROM: "Cyberlander" <cybernews@superig.com.br>
TO: <ciencialist@yahoogrupos.com.br>
DATE: 04/01/2005 14:04

Oi,
Alguém tem o esquema para a construção de um desses? pode ser de lata?
[ ]'s
D.C.

CYBERLANDER

Ama a realidade que constróis,
que nem a morte deterá teu voo · ·


----- Original Message -----

From: Luiz Ferraz Netto
To: ciencialist@yahoogrupos.com.br
Sent: Tuesday, January 04, 2005 6:38 AM
Subject: Re: [ciencialist] Super pião


E eu ainda não consegui fazer meu pião ficar girando nem por 1 minuto!
Pobre do meu levitron!
[]'
===========================
Luiz Ferraz Netto [Léo]
leobarretos@uol.com.br
http://www.feiradeciencias.com.br
===========================
-----Mensagem Original-----
De: "Kentaro Mori" <kentaro.mori@itelefonica.com.br>
Para: <ciencialist@yahoogrupos.com.br>
Enviada em: segunda-feira, 3 de janeiro de 2005 18:34
Assunto: [ciencialist] Super pião




Gira por +15 minutos:
http://www.quirkle.com/top/index.htm

Que acham?

Prof Léo, sacrilégio: um levitron foi vendido em um leilão on-line
japonês por nada menos que 730 ienes. Capaz de vir de brinde com um
McLanche Feliz daqui a pouco.

Abraços,

Mori





##### ##### #####

Para saber mais visite
http://www.ciencialist.hpg.ig.com.br


##### ##### ##### #####
Links do Yahoo! Grupos










--
No virus found in this incoming message.
Checked by AVG Anti-Virus.
Version: 7.0.298 / Virus Database: 265.6.7 - Release Date: 30/12/2004




--
No virus found in this outgoing message.
Checked by AVG Anti-Virus.
Version: 7.0.298 / Virus Database: 265.6.7 - Release Date: 30/12/2004



##### ##### #####

Para saber mais visite
http://www.ciencialist.hpg.ig.com.br


##### ##### ##### #####


Yahoo! Grupos, um serviço oferecido por:







------------------------------------------------------------------------------
Links do Yahoo! Grupos

a.. Para visitar o site do seu grupo na web, acesse:
http://br.groups.yahoo.com/group/ciencialist/

b.. Para sair deste grupo, envie um e-mail para:
ciencialist-unsubscribe@yahoogrupos.com.br

c.. O uso que você faz do Yahoo! Grupos está sujeito aos Termos do Serviço do Yahoo!.



[As partes desta mensagem que não continham texto foram removidas]



SUBJECT: Re: [ciencialist] Evolucao e desafios -Homero
FROM: "Amauri Jr" <amaurijunior2@yahoo.com.br>
TO: <ciencialist@yahoogrupos.com.br>
DATE: 04/01/2005 14:28

Oi Homero,

Penso eu ter confundido um pouco meu raciocínio, bom vamos lá:

As escolas são uma só, seja ela para o bem ou para o mal, mas sendo capaz do ser humano optar; seja para curar as doenças ou para destruir o ser humano. O ser humano ainda tem muita limitação com o que chamamos de raciocino e o que seria essa coisa que tanto o ser humano se gaba?

O que vimos em toda historia cientifica e filosófica humana, nunca deixou de criar pensamentos ou criou métodos para facilitar o ser humano em geral. Como vimos nesse século XX é um desenvolvimento tecnológico impressionante dentro de impérios e dispustas belicas, guerras frias, mornas e questes, hoje temos microcomputadores por causa da Aple e da Microsoft, pois se não fosse o produto capitalista, seria restrto ao poder.

O mesmo se deu a internet, era um sistema restrito ao governo americano para se comunicar ao mundo de abrigos anti-nucleares, sendo que o capitalismo achou um meio lucrativo. Como vê, o raciocinio humano tem com a ciencia um mal uso, eu me retringiria a dizer que o mundo é feito disso.

Mas tudo não esta perdido, há uma luz no fim do tunel...:)

Voce deve achar que sou um religioso, pois toda vez que eu falo em ciencia voce vem com rezas e mandigas. Só acho que a ciencia é restrita, uma coisa não pode ter haver com a outra, mas não sou um cara fechado. Vejo que a ciencia tem que abrir os olhos para muitas outras possibilidades, saber se aquela "mandiga" ou aquele chá é eficaz ou não para se ter comprovação ou critica.

Não existe verdades absolutas, como não existe dentro da ciencia nenhum foco dessa verdade, mesmo com comprovaçao. O que sinto é um absurdo chamado pretensionismo em querer conhecer outras coisas e não saber nem que o ser humano necessita e não mandar foguetes a lua. O ser humano ainda confunde ciencia com balela...:)

Abraços
Amauri




----- Original Message -----
From: Oraculo
To: ciencialist@yahoogrupos.com.br
Sent: Tuesday, January 04, 2005 12:43 AM
Subject: Re: [ciencialist] Evolucao e desafios -Homero


Olá Amaury

Acho que não estou sendo claro..:-) Vejamos esta parte, de novo:

Homero: "A questão é, se decide agir com eficácia, tomar uma decisão que se mostre confiável no resultado, ou escolher uma afirmação ou conclusão que se aproxime o máximo possível da realidade (sem pretensões de ser 100% ou "A VERDADE"), o que escolheria? Seja para curar pessoas, seja para matar inimigos, o que consideraria mais confiável, mais real ou mais eficaz?"

[Amaury] Eu certamente, escolheria curar as pessoas, salvar os animais, pesquisar muito mais nosso planeta do que ter aquela balela de descobrir vida fora dele. Se não resolvemos problemas do nosso próprio planeta, vamos ter capacidade moral e tecnológica para explorar o espaço Homero?

Não é essa a escolha a que me refiro. Eu também escolheria, certamente, fazer o bem (ou minha versão de "bem"..:-). Não é essa escolha a que me refiro. É, uma vez que decidiu fazer o bem e salvar pessoas (ou fazer o mal e mata-las) o que escolheria como meio mais eficaz, o conhecimento produzido cientificamente ou outro? Essa a questão, não é sobre bem e mal, mas sobre eficaz, real ou ineficaz e irreal.

Em termos concretos, digamos que decidiu salvar a vida de uma população em uma tribo distante, o que escolheria? Mais rezas e xamanismo ou vacinas e remédios? Amuletos e simpatrias ou água encanada, clorada e esgoto tratado? Ou, se decidisse matar uma grande quantidade de pessoas, o que escolheria? Usar dinamite, armas, bombas e tecnologia ou mandingas, feitiços e pragas?

Ou seja, depois de decidir fazer o bem ou o mal (aspectos subjetivos e humanos), qual sua escolha quanto aos meios e conhecimentos que permitiriam concretizar sua escolha?

Agora, esta frase eu não compreendi:

[A] O ponto é que você disse que o que temos de melhor era a ciência, até concordo contigo, mas temos um atrito; você diz que ela eficaz, eu digo que não, pois não há numa mente humana uma verdade dos fatos...:)

Como assim? Se ela não é eficaz, ainda deveriamos viver o mesmo que nossos antepassados, ter as mesmas doenças, sofrer as mesmas dores, viver apenas nos mesmos locais, não teriamos robôs em Marte ou satélites e telefones celulares. Se ela não é eficaz, nada do que afirma teria resultado concreto, nenhuma de suas previsões se concretizaria e seria fácil perceber isso. Nem mesmo este dialogo estaria sendo feito, já que a Internet não existiria (nem computadores). E o que significa não existir em uma mente humana uma verdade dos fatos? Desculpe, mas nào fez muito sentido..:-)

Quanto a variola, está enganado. Não existem focos ou doentes há muito tempo, mais de 3 decadas. Eu tenho marca de vacinação contra varíola no braço, mas você, com apenas 28 anos, nào tem (não era mais necessário, já que ela não mais existe..:-) A variola foi erradicada, apenas cepas em laboratorio ainda existem. E várias doenças caminham para o mesmo fim, como a paralizia infantil (que resiste na África, onde lideres religiosos desconfiam das vacinas e lutam contra ela).

E negros do bronx ficam doentes, mas menos que negros africanos e não de malária ou tifo. E recebem tratamento médico melhor que a maioria de nossos antepassados, mesmo os ricos e nobres (o tratamento médico de um brasileiro pobre, no INPS, é muitas vezes melhor que o que dispunha a rainha Vitória da Inglaterra no periodo vitoriano..:-)

Agora uma guinada de 180 graus..:-)

[A] Se foi ou não, foi dinheiro jogado no lixo, sendo que poderia ser empregado na fome imensa na África ou a reconstrução das cidades que eles sempre destruíram...:)

Deixou de lado a afirmação de fraude e substituiu por um julgamento de valor, sobre se deveriam ter aplicado o dinheiro na Africa ou nos paises pobres. Nesse caso, fica dificil discutir, é subjetivo demais. Eu acho importantissimo o conhecimento produzido pelas missões Apollo, tanto quanto as missões à Marte. Nada é mais interessante e importante que as questões estudadas pela astrofisica, como surgiu o universo, os planetas, de onde vem a materia básica da vida, existe vida em outro planeta, etc. Mas tem todo direito de discordar e escolher outras prioridades.

Mas, nada disso muda o fato de que é a ciência, seu conhecimento confiável, que permitiu tanto a ida à Lua, quanto, se fosse possível, as suas escolhas de onde gastar o dinheiro.

[A] Eu posso pegar um radio amador e dizer que estou no Japão, como vão saber? Eu se mais ou menos as coordenadas do Japão e posso falar para os leigos, pois é o que aconteceu...:)

Não, não pode..:-) Se transmitir, e eu receber em dois aparelhos de radio amador, posso dizer exatamente onde está, e, se usar os satélites de GPS, com precisão de 1 ou 2 metros. Isso se chama triangulação e é um fato, não uma opinião ou uma crença. Se transmitir do Japão, eu precisarei de algum tipo de satelite, mas se transmitir de um satélite, qualquer radio amador ao alcance vai resolver e encontrar você..:-)

Vou tentar explicar, mas seria mais fácil com imagens e desenhos.

Pense em um ponto A em São Paulo. E um ponto B em Campinas (não sei se conhece, mas é uma cidade distante aproximadamente 90 Km de SP). Imagine um receptor em cada cidade citada. Agora, de uma caminhonete com um transmissor, você viaja pela estrada. O receptor em SP vai indicar, a principio, que o sinal mais forte vem da direção Norte, em graus precisos. O que se encontra em Campinas, indica que vem de Noroeste, com graus precisos. Pegue um mapa e desenhe uma reta a partir de cada cidade na direção indicada pelos graus. No ponto onde se cruzam, estará você, a caminhonete e seu transmissor.

Conforme viaja, os angulos vão mudando e o cruzamento continuará indicando sua posição, com precisão extrema.

Troque o transmissor na caminhonete por um sinal em uma nave ou satelite, e os receptores em SP e Campinas podem dizer, com precisão, exatamente onde se encontra o satélite, em cada segundo, sem erro ou possibilidade de fraude.

Se fosse possivel fraudar uma emissão, as aeronaves militares simplesmente levariam mecanismos que o fizesse e evitariam ser detectadas pelo radar inimigo..:-)

Enfim, conhecimento confiável, seja qual for o uso que se deseje dar a ele, é produzido com mais eficácia usando os métodos e mecanismos da ciência, seja apra enviar um homem à Lua, seja para curar doenças, seja para explodir cidades. Devemos responsabilizar quem de direito nessa questão, nos mesmos, não a ferramenta usada..:-)

Um abraço.

Homero




----- Original Message -----
From: Amauri Jr
To: ciencialist@yahoogrupos.com.br
Sent: Monday, January 03, 2005 11:53 PM
Subject: [ciencialist] Evolucao e desafios -Homero


Oi Homero





"Fico contente que goste dos debates e de minha participação (nem sempre é assim, as vezes o pessoal se irrita bastante comigo:-)"



[A] São pessoas que não reconhecem o verdadeiro valor de um bom debate...:)





"Mas ainda acho que está confundindo duas coisas, a ciência, abstrata, e o comportamento humano, passível de ser julgado e avaliado subjetivamente."



[A] As duas coisas são ciência, uma não haveria ser a se a outra não fosse, a ciência em geral é um processo de conhecimentos.



"É claro que é importante discutir a aplicação, direção, prioridade, etc, de pesquisas e do conhecimento humano. Mas, a ciência, é apenas um conjunto de conhecimentos adquirido de certa forma (método cientifico), não seu uso ou resultado. Isso está confundindo a discussão..:-)"



[A] Mas se não iremos difundir seu uso, por que descobrir? :)



"A questão é, se decide agir com eficácia, tomar uma decisão que se mostre confiável no resultado, ou escolher uma afirmação ou conclusão que se aproxime o máximo possível da realidade (sem pretensões de ser 100% ou "A VERDADE"), o que escolheria? Seja para curar pessoas, seja para matar inimigos, o que consideraria mais confiável, mais real ou mais eficaz?"



[A] Eu certamente, escolheria curar as pessoas, salvar os animais, pesquisar muito mais nosso planeta do que ter aquela balela de descobrir vida fora dele. Se não resolvemos problemas do nosso próprio planeta, vamos ter capacidade moral e tecnológica para explorar o espaço Homero?



"Neste caso, o termo "melhor" tem duas conotações, que foram misturadas. O que temos de melhor, no meu sentido, é de mais eficaz. Dá resultados. No seu uso, é um julgamento subjetivo, melhor, bom, correto, etc. Não tem ligação com meu argumento. A ciência é "melhor" se deseja uma ação eficaz, ou um conhecimento confiável. O que fará com esse conhecimento, se algo "melhor", do bem, ou algo "pior", do mal, não é relevante para o argumento."



[A] O ponto é que você disse que o que temos de melhor era a ciência, até concordo contigo, mas temos um atrito; você diz que ela eficaz, eu digo que não, pois não há numa mente humana uma verdade dos fatos...:)



"Já a erradicação da malaria foi um erro, ato falho, meu..:-) Pensei em varíola e escrevi malária (acho que passava um especial na TV sobre a malária..:-). Mas, mesmo a malária é mais controlada em paises civilizados, com alto uso de conhecimento cientifico, que nos pobres e sem esse uso. Ainda é um bom argumento sobre a eficácia da mesma..:-)"



[A] Homero, mesmo a varíola existem focos em alguns paises, ela não esta erradicada; se engana se a malaria, febre-amarela, colega e etc...não tenha em países desenvolvidos, tem, só que não é escancarado como é na América latina...:) duvido se for no Blonks e não encontrar um negro norte-americano doente, duvido...:)



"Finalmente, acho que não compreendeu a questão a emissão de radio e a ida à Lua. Não importa o poder dos USA, não importa seu governo ou o caráter (ou falta de) dos seus governantes, mesmo hoje, se emitir uma onda de rádio, quaisquer dois radio amadores podem dizer, precisamente, de onde vem. Isso se chama triangulação, e é o motivo de se gastar tanto tempo e dinheiro em desenvolvimento de aviões invisíveis, já que, se o aparelho voador reflete a onda do radar, pode ser posicionado precisamente, sem sombra de duvida".



[A] Se foi ou não, foi dinheiro jogado no lixo, sendo que poderia ser empregado na fome imensa na África ou a reconstrução das cidades que eles sempre destruíram...:)





"E ao pousar, e enviar dados do solo lunar, idem, sem possibilidade de fraudar essa emissão."



[A] Eu posso pegar um radio amador e dizer que estou no Japão, como vão saber? Eu se mais ou menos as coordenadas do Japão e posso falar para os leigos, pois é o que aconteceu...:)



"Lutamos a séculos com o virus da gripe e ainda não podemos mata-lo. Sua taxa de mutação, como costuma acontecer com virus, é brutal, e sua adaptação ciclica impede que seja feita uma vacina para todas as cepas. A simplicidade de determinados virus é a chave para que escapem tão facilmente da cura. Um organismo complexo tem estruturas que se repetem em todas as suas variações. Em algum momento encontramos essa estrutura e uma vacina é criada. Mas, organismos simples, em especial virus, mudam tão completamente, que uma estrutura, uma chave, única, não acontece. Isso torna bem dificil derrota-lo. Além disso, a simplicidade faz com que muitas estruturas sejam também usadas por células do organismo hospedeiro. Assim, algumas das drogas que efetivametne matam os virus da gripe não podem ser aplicadas a seres humanos, pois matam suas células saudáveis também (esse é um dos problemas com drogas anti-cancer também atingir células saudaveis juntamente as doentes)."



[A] Por que mexer com isto? Como diz aquele ditado, não se mexe com quem ta quieto, foi isto que ocorreu; uma "besta" em um centro de pesquisa pegou e deixou escapar o HIV e ele contaminou muita gente. A gripe e todas as doenças do tipo viral, ou se preferir, retro-virus ele é mutante porque se vale da célula hospedeira, portanto teríamos que mudar o código genético dos glóbulos brancos para se valer de eficácia. Mas por enquanto, a deusa ciência em sua magnitude, não encontrou um meio de fazer a tal. Portanto caro amigo, fica como jogar ping-pong sem raquete...:)



"A ciência não prometeu curar todas as doenças, consertar o mundo, ser 100% eficaz ou definir A VERDADE (para isso temos as religiòes..:-). Ela é apenas uma ferramenta, uma boa e eficaz ferramenta. Talvez esteja esperando demais da ciência e a frustação de não ter todas as respostas o tenha tornado um pouco rancoroso com ela. Entre duas afirmações, entre duas respostas, a obtida de forma cientifica é a mais confiável, só isso. Existe, sim, uma luta de pesquisadores, seres humanos, para usar essa ferramenta de forma a curar doenças, encontrar respostas, criar conforto, resolver problemas, etc. Mas é uma busca, não um final em sí mesmo."



[A] Ela foi dirigida por Tales de Mileto para descobrir formas de relacionar e pesquisar os fenômenos naturais, que por trás de um trovão ou raio não esta a fúria de algum deus ou coisa parecida. Mas o que vimos hoje é o cientificismo, uma forma derivada de crenças e fundamentalismo que não cabe dentro da ciência e esta atrapalhando algumas pesquisas de cura. Ela não quer ser a VERDADE, mas para alguns, ser cientista é acreditar na filosofia empírica e não é só isso, ela esta vendo em um ângulo só. Eu não sou rancoroso com ela, apenas estou mostrando (por enquanto não) que ela pode ver alem e encontrar o caminho...sobre religiões, creio ser muito pretensioso achar que aquilo ou isso seja mentira, tem-se sim, de investigar.





"Bush é uma "anta" fundamentalista. Mas observe que é uma das pessoas mais contra a ciência que poderíamos encontrar. Seu esforço fundamentalista para obrigar sua visão religiosa de mundo a todo o resto do planeta é frontalmente contrária a ciência, tolerante por natureza. Impedir o estudo de células tronco pode impedir avanços que poderiam curar diversas doenças, inclusive paraplegia. Ensinar criacionismo em escolas e banir a evolução (e a base da biologia moderna hoje) impediria até mesmo que bons médicos se formem ou que novas descobertas sejam feitas."



[A]Bush não é uma "anta" fundamentalista, ele usa a crença popular a seu favor, pois nessas guerrilhas ridículas dele e de seu amiguinho, enquanto suas empresas de petróleo faturam milhões de dólares nessa. Ao proibir, eu apenas vejo ele atender aos interesses da igreja onde foi apoiado, nada mais do que isto e voltamos a era medieval...:)





"Agora, você (quem me lê) deve decidir (pressionado certamente pelos falcões militares) o que fazer. Imagine que estamos em guerra com a Argentina. Você, presidente, tem uma arma recém criada que pode acabar a guerra, devido ao seu poder inimaginável. Detonar essa arma em um cidade vai matar 45 mil pessoas do outro pais (que começou a guerra). Não detonar vai prolongar a guerra por mais alguns meses, a Argentina jamais se renderá e 100 mil brasileiros morrerão, e mais de 350 mil argentinos. O que você decidiria? O que diria as mães e pais de seu pais, quando perguntassem porque não parou a guerra quando podia e escolheu ver morrer mais 100 mil de seus conterrâneos?"



[A] Eu presidente? Uau!!! :)...bom, eu estou numa guerra com a Argentina, aqui no meu ladinho(que arrepio), milhões de tupiniquins vão ou tem perigo de morte...sinceramente destruiria a Casa Rosa...muito eficaz e de tamanha eficiência e ainda punha uma bandeira brasileira na traseira do presidente da la Argentina....:)



"Deve se lembrar que nunca uma arma assim havia sido detonada, e não se sabia o horror que poderia causar (os primeiros testes no deserto americano usaram soldados para verificar o efeito e a descontaminação foi feita com vassouras de palha e água corrente.:-("



[A] Americano é sempre americano muito soldados morreram de câncer ou outro mal, como disse, eles mexem com que esta quieto...:)

"Vou repetir, acho um horror o bombardeio de civis, na verdade, acho qualquer guerra um horror, desnecessária e absurda. Mas não acho que Hiroshima foi apenas maldade, crueldade de monstros, mas uma decisão calculada, difícil e medonha, da qual eu não saberia o que escolher como "menos pior". E hoje temos os USA arrogantes e com jeito de donos do mundo, e nossa antiapatia por Bush nos faz ver tudo com um filtro de asco. Mas na época a guerra dos USA era contra impérios cruéis e mais perigosos que a América jamais será (Hitler, e todo o horror que representa) e não se pode relativizar o que teria acontecido se a Alemanha e o Japão ganhassem a guerra."



[A] A grande Águia ganhou e esta toda cheia de querer mandar no mundo, síndrome de superego...isso é o ponto, ter poder de construir e destruir para se fazer de herói...:)

----- Original Message -----
From: Oraculo
To: ciencialist@yahoogrupos.com.br
Sent: Monday, January 03, 2005 7:47 PM
Subject: Re: [ciencialist] Evolucao e desafios -Amaury


Olá Amaury

Fico contente que goste dos debates e de minha participação (nem sempre é assim, as vezes o pessoal se irrita bastante comigo:-)

Mas ainda acho que está confundindo duas coisas, a ciência, abstrata, e o comportamento humano, passível de ser julgado e avaliado subjetivamente.

É claro que é importante discutir a aplicação, direção, prioridade, etc, de pesquisas e do conhecimento humano. Mas, a ciência, é apenas um conjunto de conhecimentos adquirido de certa forma (método cientifico), não seu uso ou resultado. Isso está confundindo a discussão..:-)

A questao é, se decide agir com eficácia, tomar uma decisão que se mostre confiável no resultado, ou escolher uma afirmação ou conclusão que se aproxime o máximo possível da realidade (sem pretensões de ser 100% ou "A VERDADE"), o que escolheria? Seja para curar pessoas, seja para matar inimigos, o que consideraria mais confiável, mais real ou mais eficaz?

Sua resposa, que certamente é igual a minha, coloca a perspectiva correta na discussão..:-) Ciência, abstratamente, é a melhor ferramenta de compreensão deste universo (universo fisico) que dispomos. O que faremos com o conhecimento assim obtido, é outra coisa. Eu, particularmente, o usaria para melhorar a vida de todo mundo (por exemplo, usando o produto desse conhecimento para enviar ajuda as vitimas do tsunami na Asia). Outros a usariam para matar ou criar armas de destruição em massa.

Mas culpar a ciência, um conhecimento abstrato, pelo uso que malucos fazem dela, é irrelevante e impróprio. Me lembra a piada do matador que, perguntado se não tinah remorso, respondeu: Eu não mato ninguem, só faço o furo, quem mata é Deus (no que ele tem razão, de uma perspectiva religiosa do universo..:-)

Pessoas, seres humanos, matam e maltratam. E fazem isso muito antes de termos armas de destruição em massa ou equivalentes. Culpar o conhecimento do átomo pelo uso da bomba atomica seria como condenar a arma no lugar do atirador (imagino que seria um processo muito engraçado de acompanhar..:-)

Veja este trecho seu:

Amaury: Por isso, nada tenho contra ou a favor da ciência, apenas é uma visão critica sobre...:-)

Se fosse assim, nem haveria a discussão entre nós..:-) Mas seu posicionamento inicial era diferente, contrário a ciência, como se fosse ela responsável por seu uso. Por isso fiz minha análise de sua análise..:-)

A confusão fica mais clara neste trecho:

Amaury:O que temos de melhor, como disse, esta nas mãos dos poderosos, portanto "o que temos de melhor" foi e esta sendo mandado pelo governo dos poderosos. Hoje os norte-americanos mandam e desmandam no que "temos de melhor", tanto se é para o bem da humanidade é imoral, mas se beneficia as partes bélicas deles, seguem numa boa...:-)


Neste caso, o termo "melhor" tem duas conotações, que foram misturadas. O que temos de melhor, no meu sentido, é de mais eficaz. Dá resultados. No seu uso, é um julgamento subjetivo, melhor, bom, correto, etc. Não tem ligação com meu argumento. A ciência é "melhor" se deseja uma ação eficaz, ou um conhecimento confiável. O que fará com esse conhecimento, se algo "melhor", do bem, ou algo "pior", do mal, não é relevante para o argumento.

Já a erradicação da malaria foi um erro, ato falho, meu..:-) Pensei em variola e escrevi malária (acho que passava um especial na TV sobre a malária..:-). Mas, mesmo a malária é mais controlada em paises civilizados, com alto uso de conhecimento cientifico, que nos pobres e sem esse uso. Ainda é um bom argumento sobre a eficácia da mesma..:-)

Finalmente, acho que não compreendeu a questao a emissão de radio e a ida à Lua. Não importa o poder dos USA, nào importa seu governo ou o carater (ou falta de) dos seus governantes, mesmo hoje, se emitir uma onda de rádio, quaisquer dois radio amadores podem dizer, precisamente, de onde vem. Isso se chama triangulação, e é o motivo de se gastar tanto tempo e dinheiro em desenvolvimento de aviões invisiveis, já que, se o aparelho voador reflete a onda do radar, pode ser posicionado precisamente, sem sombra de duvida.

Explicando melhor, se um satélite emite sinal de radio, você, com seu radio amador (ou radar avançado, não importa.:-), pode determinar a direção com precisão. Se outro amigo seu radio amador também o fizer, os dois podem combinar os dados e dizer, com exatidão, de onde parte o sinal e exatamente onde está o satelite. Não há forma de fraudar isso, nem de disfarçar ou fingir estar onde não está. Assim, quando as Apollo orbitavam a Terra, foram acompanhadas de perto por todos que desejassem, sem forma de fraudar isso. Quando orbitaram a Lua, foram acompanhadas (com frustação crescente dos russos..:-), por toda parte.

E ao pousar, e enviar dados do solo lunar, idem, sem possibilidade de fraudar essa emisão.

Assim, se deseja discordar do pouso na Lua, tem de explicar como isso foi feito, ou vai esbarrar em algo chamado evidencia acachapante (que colocou fim nas especulações dos russos, chineses e todos os muitos inimigos dos USA durante a guerra fria que adorariam desmascarar a fraude). Alias, quando os russos colocaram o primeiro satélite, Sputinik, em órbita, alguns jornais americanos e alguns membros do governo americano tentaram vender a idéia de fraude, mas o fato que qualquer um com um bom rádio podia confirmar o feito, foi o bastante para convencer as pessoas.:-)

HIV

Lutamos a séculos com o virus da gripe e ainda não podemos mata-lo. Sua taxa de mutação, como costuma acontecer com virus, é brutal, e sua adaptação ciclica impede que seja feita uma vacina para todas as cepas. A simplicidade de determinados virus é a chave para que escapem tão facilmente da cura. Um organismo complexo tem estruturas que se repetem em todas as suas variações. Em algum momento encontramos essa estrutura e uma vacina é criada. Mas, organismos simples, em especial virus, mudam tão completamente, que uma estrutura, uma chave, única, não acontece. Isso torna bem dificil derrota-lo. Além disso, a simplicidade faz com que muitas estruturas sejam também usadas por células do organismo hospedeiro. Assim, algumas das drogas que efetivametne matam os virus da gripe não podem ser aplicadas a seres humanos, pois matam suas células saudáveis também (esse é um dos problemas com drogas anti-cancer também atingir células saudaveis juntamente as doentes).

Talvez as industrias e governos não invistam o suficiente na pesquisa, eu tendo até a concordar com você nisso. Mas isso indica duas coisas: primeiro, que é caro e dificil descobrir a cura, ou ela já teria sido descoberta mesmo com pouco dinheiro. Segundo, indica que temos FALTA de ciência, que seria possivel com mais recuros, e não exesso. Em qualquer dos casos a crítica feita não se aplica. Precisamos, se queremos encontrar a cura ou tratamentos melhores para a AIDS, de mais ciência, um conheimento confiável e eficaz, e não de menos.

Espectativas

Amaury:Não tenho nada contra a ciência, mas você tem contra os índios...:) o que vejo é ainda crianças mortas e vidas curtas por uma ciência ineficaz, se eu gosto ou não é outra historia, que se eu não gosta-se da ciência não estaria numa lista de ciências...:) e pouco irei a uma tribo, porque não iria acostumar...:)e uma pergunta: a ciência tem vida própria?

A ciência não prometeu curar todas as doenças, consertar o mundo, ser 100% eficaz ou definir A VERDADE (para isso temos as religiòes..:-). Ela é apenas uma ferramenta, uma boa e eficaz ferramenta. Talvez esteja esperando demais da ciência e a frustação de não ter todas as respostas o tenha tornado um pouco rancoroso com ela. Entre duas afirmações, entre duas respostas, a obtida de forma cientifica é a mais confiável, só isso. Existe, sim, uma luta de pesquisadores, seres humanos, para usar essa ferramenta de forma a curar doenças, encontrar respostas, criar conforto, resolver problemas, etc. Mas é uma busca, não um final em sí mesmo.

Tenho as mesmas objeções que você às industrias farmaceuticas (inclusive as homeopaticas..:-). Mas sei que existem pesquisadores legitimamente dedicados a encontrar curas, soluções, respostas, apesar das empresas e até mesmo dentro dessas empresas. Gente que tem legitimos interesses humanos, que pesquisa até mesmo as "doenças de paises pobres" como a malária e febre amarela (erradicadas nos paises desenvlvidos).

Bush é uma "anta" fundamentalista. Mas observe que é uma das pessoas mais contra a ciência que poderiamos encontrar. Seu esforço fundamentalista para obrigar sua visão religiosa de mundo a todo o resto do planeta é frontalmente contrária a ciência, tolerante por natureza. Impedir o estudo de células tronco pode impedir avanços que poderiam curar diversas doenças, inclusive paraplegia. Ensinar criacionismo em escolas e banir a evolução (e a base da biologia moderna hoje) impediria até mesmo que bons médicos se formem ou que novas descobertas sejam feitas.

Ainda assim, ele usa os resultados da ciência onde lhe convém, com armas de destruição e brinquedos tecnologicos que poder. Isso torna a ciência culpada? Ou torna o Bush responsável?

Hiroshima é um caso complicado. Nagazaki seria mais claro, como alvo do debate, desnecessário e violento. Mas Hiroshima é dificil. Antes que "caiam de pau", acho um horror a destruição de uma cidade, a morte de civis e qualquer tipo de conflito. Mas é preciso pensar, sempre, sob pena de não ver com clareza. E Hiroshima é dificil de enxergar, através do horror da bomba e da matança.

Mas havia uma guerra em curso, isso é claro. Havia um pais, Japão, que entrou na guerra por vontade própria, para expadir fronteiras e massacrar vizinhos (embora a vontade a considerar fosse apenas dos mandatarios do pais na época). Havia um comportamento cultural onde a morte era sublime, o sacrificio honroso e o imperador deus (você morreria por seu deus). Dentro dessa análise, continuar a guerra com o Japão produziria, por estimativa, 100 mil mortes de americanos e mais de 350 mil mortes japonesas (estimativas derivadas das batalhas já ocorridas, da decisão de cada soldado japones de só parar depois de morto e das dificuldades de desembarque em ilhas - as piores batalhas da segunda guerra forma em ilhas do Japão).

Agora, você (quem me lê) deve decidir (pressionado certamente pelos falcões militares) o que fazer. Imagine que estamos em guerra com a Argentina. Você, presidente, tem uma arma recem criada que pode acabar a guerra, devido ao seu poder inimaginável. Detonar essa arma em um cidade vai matar 45 mil pessoas do outro pais (que começou a guerra). Não detonar vai prolongar a guerra por mais alguns meses, a Argentina jamais se renderá e 100 mil brasileiros morrerão, e mais de 350 mil argentinos. O que você decidiria? O que diria as mães e pais de seu pais, quando perguntassem porque não parou a guerra quando podia e escolheu ver morrer mais 100 mil de seus conterraneos?

Deve se lembrar que nunca uma arma assim havia sido detonada, e não se sabia o horror que poderia causar (os primeiros testes no deserto americano usaram soldados para verificar o efeito e a descontaminação foi feita com vassouras de palha e agua corrente.:-(

Eu não sei o que escolheria e não gostaria de ter de decidir algo assim. Imagino que não se pode dormir mais depois de ser submetido a coisas desse tipo. Por isso considero que Nagasaki é mais claro, uma decisão certamente militar, resultado de uma perda de poder civil em um periodo dificil. Mas Hiroshima é mais complicado.

Vou repetir, acho um horror o bombardeio de civis, na verdade, acho qualquer guerra um horror, desnecessária e absurda. Mas não acho que Hiroshima foi apenas maldade, crueldade de monstros, mas uma decisão calculada, dificil e medonha, da qual eu não saberia o que escolher como "menos pior". E hoje temos os USA arrogantes e com jeito de donos do mundo, e nossa antiapatia por Bush nos faz ver tudo com um filtro de asco. Mas na época a guerra dos USA era contra imperios crueis e mais perigosos que a america jamais será (Hitler, e todo o horror que representa) e nào se pode relativizar o que teria acontecido se a Alemanhã e o Japão ganhassem a guerra.

De todo modo, estamos mais perto da concordancia que da discordância..:-) A ciência, como ferramenta, é eficaz, confiável e razoável. Seu uso, que pode ser para o bem ou para o mal (mesmo que o bem e o mal sejam relativos, pergunte ao Bin Laden e ao Bush..:-), é que é passível de discussão.

Um abraço.

Homero






----- Original Message -----
From: Amauri Jr
To: ciencialist@yahoogrupos.com.br
Sent: Monday, January 03, 2005 11:45 AM
Subject: [ciencialist] Evolucao e desafios -Homero


Olá Amury


[A] Olá Homero






"Minhas análises sobre suas análises..:-)"



[A] Sinceramente gosto de suas analises, fico satisfeito de você me responder...:-)





"Você parece estar ressentido com o que chama de "ciência" e a trata como entidade independente, crença de cientistas ou algo a ser adorado ou derrubado. Mas nada disso é ciência, nem o resultados de sua aplicação se confundem com ela. Ciência neste contexto (desta lista e do uso padrão do termo..:-) é apenas o conjunto dos conhecimentos obtidos através de um método padronizado, chamado cientifico, e do rigor derivado do mesmo. E, independente do que pense dela, tem elevada confiabilidade..:-)"



[A] Amigo, o que penso que infelizmente o homem usa seu conhecimento para algo não muito para o "bem" humano ou "mal", a natureza humana já é corrupta. Tudo que é humano tem falhas, pois nada ainda é perfeito, mas a ciências como algo que vem para beneficiar a humanidade tinha que ser impartidaria e sem nenhuma crença ou fanatismo; como vimos muito na lista. Por isso, nada tenho contra ou a favor da ciência, apenas é uma visão critica sobre...:-)



"Não importa se o resultado é passível de ser julgado subjetivamente como "mau" ou mesmo como "bom". Importa apenas que é mais eficiente que outras formas de conhecimento, mais eficaz em suas previsões e mais confiável que outros instrumentos de compreensão do universo já criado por seres humanos. E que será abandonado assim que uma ferramenta mais eficaz seja apresentada..:-) Até lá, é o que temos de melhor."



[A] O que temos de melhor, como disse, esta nas mãos dos poderosos, portanto "o que temos de melhor" foi e esta sendo mandado pelo governo dos poderosos. Hoje os norte-americanos mandam e desmandam no que "temos de melhor", tanto se é para o bem da humanidade é imoral, mas se beneficia as partes bélicas deles, seguem numa boa...:-)





"Podemos analisar filosoficamente, subjetivamente, os resultados, mas não discutir a eficácia. A ciência, o conhecimento assim acumulado, é tão eficiente quando cura doenças, como a erradicação da malária, como quando destrói milhões de vidas, como em Hiroshima. O resultado é passível de julgamento, mas a ação não. Ela é eficaz, nos dois casos. Se precisar salvar alguém ou matar alguém, deve usar a ciência, é mais eficaz sempre..:-)"



[A] Podemos sim, tanto que você já disse que ciência é um conjunto de conhecimentos; mas muitos conhecimentos, pois o que vimos, é um "Clube da Ciência" que vê apenas um dos vários lados do universo, mas temos vários e é ainda um incógnito pelo homem ainda não abrir a cabeça...:-) Agora, se erradicamos a malaria não sei, creio que você não esta lendo jornal, pois a malaria, a febre-amarela, o HIV, o câncer, então matando e a ciência "tudo que temos de melhor" fica ai procurando pelos em baratas. Hiroshima foi um ato terrorista covarde e deprimente, que deveria fazer um julgamento dos cientistas e dos governantes no tribunal de Nuremberg, por danos morais humanos.





"Não gostar dela, ou como você pretende com as análises, critica-la ou despreza-la, não importa em nada para sua eficácia e confiabilidade. Você vai viver mais que todos os seus antepassados, goste ou não..:-) Sua expectativa de vida é de 75 anos, e, não sei sua idade, mas se já passou dos 40 como eu, é mais que seu avô esperava viver ao nascer (em 1900 a expectativa de vida no Brasil era de 33 anos)."





[A] Tenho apenas 28 anos, sem filhos e com uma deficiência física, o que sei que são superfluidades da senhora sagrada ciência. O que pode me interessar eu viver 40 ou 70 anos se nem ela pode prever?





"Na verdade, você usou um computador e a Internet, frutos da ciência, para enviar seus pensamentos e ataques a ciência, ao inves de tentar telepatia, rezas ou mandingas (ou qualquer outra forma de comunicação não cientifica) justamente porque é mais eficaz e confiável que qualquer outra..:-)"



[A] Alguma coisa boa tinha que sair daí né?...:-) o problema é esse, vocês da ciência só tem um foco do assunto, não ve as múltiplas variedades de estudos...:)





"Você não parece gostar de muitos dos aspectso do uso do conhecimento cientifico. Direito seu. E pode até mesmo abandona-los, todos, e ir viver em uma aldeia remota no Amazonas, sem (quase) nenhum contato com a ciência (os índios são muito receptivos a forasteiros, vão gostar de recebe-lo), e com todo o ônus de viver dessa forma (filhos mortos, vida curta, doenças diversas, poucos dentes, etc). Mas está na verdade criticando comportamentos humanos, que são seres falhos como sabe, não a ciência. Esta, "estricto senso", não é boa ou má, cruel ou gentil, apenas eficiente, confiável em seus efeitos."



[A] Não tenho nada contra a ciência, mas você tem contra os índios...:) o que vejo é ainda crianças mortas e vidas curtas por uma ciência ineficaz, se eu gosto ou não é outra historia, que se eu não gosta-se da ciência não estaria numa lista de ciências...:) e pouco irei a uma tribo, porque não iria acostumar...:)e uma pergunta: a ciência tem vida própria?



"Você não "acredita" nela. Engraçado, ela não pede que se acredite em nada.:-) Deve duvidar sempre, até que evidencias se mostrem sólidas o bastante para uma conclusão. E é nesse ponto que você escorrega..:-) O pouso na Lua já tem evidencias suficientes para essa conclusão, em que pese as tolices de diversos malucos mundo afora e suas teorias de conspiração. O HIV tem recuado e matado muito menos que no inicio da epidemia, graças a AZT e os atuais coquetéis antivirais. Se ajustes são feitos a toda hora, e são, sobre o que comer ou não, é apenas porque a ciência não pretende ser "A VERDADE" como religiões, mas um conhecimento que cresce, se aprofunda, se ajusta e melhora. Sabemos mais, não menos sobre o que comer ou não comer."



[A] Engraçado que pessoas inteligentes não duvidem que os yanques tenham forjado tudo, mas como você mesmo disse, é um direito seu. Eu acredito muito mais que se possa acreditar, a ciência melhorou minha vida muito, tendo esse computador; em minha cadeira de rodas de alumínio (de ferro era um lastima andar), portanto não estou desacreditando a ciência. Agora se quer achar que os coquetéis são eficazes tudo bem, mas que morre mesmo assim, isso morre...:)



"Sim, dá trabalho, é preciso atenção e constante leitura para acompanhar, seria melhor ter uma única resposta para todas as perguntas e pronto, sem risco e sem mudanças. Mas o mundo não funciona assim..:-) Sabemos hoje mais do que ontem sobre como se alimentar, e saberemos mais amanhã, deixando parte do que sabemos hoje para trás. (Parte, não tudo:-)"



[A] Como se o mesmo alimento hoje esta com proteínas e tudo que precisamos e amanha não?





"As teorias conspiratórias são atrativas.:-) É como se nós, os que estão "por dentro", soubéssemos de "coisas" que o resto do mundo não sabe, fossemos mais "espertos" ou mais inteligentes, que não se deixam enganar. Mas, na maioria das vezes (eu diria em sua totalidade) é apenas o ego tentando ser mais do que é.:-) Mesmo com centenas de sites "desmistificando" o pouso na Lua, o volume de evidencias a favor é gigantesco e nenhum deles explica um ponto fundamental do problema: como o governo americano fraudou os sinais de radio e TV que eram emitidos de todas as naves Apollo tanto durante as viagens, como do solo lunar, captados por todo radioamador ao redor do globo, sem falar nos radares e antenas dos paises em disputassem os USA? (lembre-se que, mesmo hoje, não se pode fraudar a origem de um sinal de radio..:-)"



[A] Não acredito nesses doentes mentais que ficam montando sites para se promover...:) eu tiro dos fatos que ocorreram que podem ser completamente forjados, como as transmissões, ou não sabe que eles são donos do que "temos de melhor"...:)



"Enfim, a ciência é eficaz. Embora seu uso, o uso do conhecimento confiável por ela produzido, possa ser discutido e julgado, isso não é a ciência nem com ela se confunde. E, tenha certeza, seu automóvel vai conduzi-lo amanhã, confiavelmente, conforme as leis da fisica descobertas pela ciência, seu computador receberá esta mensagem, os remédios na farmacia impedirão que você morra, as vacinas que tomou evitarão que adoeça (como todos os seus ancestrais adoeceram) e os satelites em órbita, colocados lá pelo conhecimento cientifico, vão transmitir suas chamadas telefonicas até para o Japão, se você assim desejar..:-)"



[A] Reverencia a ciência como um crente reverencia uma reza...:) o meu carro, vai bater conforma a lei da física, as farmácias venderão remédios que viciam, os satélites vão espionar minha casa e tudo graças a deusa infalível ciência...:)



"Para completar a mensagem anterior: uma menina inglesa de 15 aos foi a primeira pessoa a sobreviver depois de contrair raiva sem tomar a vacina antirabica. Ela foi submetida a um novo tratamento revolucionário (e científico), com um coquetel de drogas e coma induzido."



[A] É? Legal! Mas não são os igreses que usam a ciência para fazer guerrinhas com o Bush?





"Diferente de milagres divinos e sobrenaturais, o que se aprendeu com a nova técnica pode ser usado em outras pessoas doentes e não afetam apenas a pessoa que recebe a graça..:-)"



[A] Também acho, devo adverti-lo novamente que não sou a favor no milagre...:)




"A partir de agora, mesmo desprezando a "ciência", você pode ficar sossegado, se você ou um de seus filhos ou entes queridos contrair raiva, poderá ser curado, cientificamente curado.:-) A não ser que tenha resolvido ir viver em uma aldeia indigena na remota Amazonia..."



[A] Sim vão...vão tomar coca-cola e comer hambúrguer feitos cientificamente e se eu tiver sorte, vão se viciar em LSD feito pela ciência, vão tomar energéticos feito pela ciência, vão ter colesterol de produtos que o "temos de melhor" analisa. O mundo é um paraíso, graças a ciência não acha? ...:)



Abraços

Amauri


---- Original Message -----
From: Oraculo
To: ciencialist@yahoogrupos.com.br
Sent: Sunday, January 02, 2005 11:05 PM
Subject: Re: [ciencialist] Evolucao e desafios - CartaCapital - 02/01/05


Olá Amury

Minhas análises sobre suas análises..:-)

Você parece estar ressentido com o que chama de "ciência" e a trata como entidade independente, crença de cientistas ou algo a ser adorado ou derrubado. Mas nada disso é ciência, nem o resultados de sua aplicação se confundem com ela. Ciência neste contexto (desta lista e do uso padrão do termo..:-) é apenas o conjunto dos conhecimentos obtidos através de um método padronizado, chamado cientifico, e do rigor derivado do mesmo. E, independente do que pense dela, tem elevada confiabilidade..:-)

Não importa se o resultado é passível de ser julgado subjetivametne como "mau" ou mesmo como "bom". Importa apenas que é mais eficiente que outras formas de conhecimento, mais eficaz em suas previsões e mais confiável que outros instrumentos de compreensão do universo já criado por seres humanos. E que será abandonado assim que uma ferramenta mais eficaz seja apresentada..:-) Até lá, é o que temos de melhor.

Podemos analisar filosoficamente, subjetivamente, os resultados, mas não discutir a eficácia. A ciência, o conhecimento assim acumulado, é tão eficiente quando cura doenças, como a erradicação da malária, como quando destroi milhões de vidas, como em Hiroshima. O resultado é passível de julgamento, mas a ação não. Ela é eficaz, nos dois casos. Se precisar salvar alguém ou matar alguém, deve usar a ciência, é mais eficáz sempre..:-)

Não gostar dela, ou como você pretende com as análises, critica-la ou despreza-la, não importa em nada para sua eficácia e confiabilidade. Você vai viver mais que todos os seus antepassados, goste ou não..:-) Sua expectativa de vida é de 75 anos, e, não sei sua idade, mas se já passou dos 40 como eu, é mais que seu avô esperava viver ao nascer (em 1900 a expectativa de vida no Brasil era de 33 anos).

Na verdade, você usou um computador e a Internet, frutos da ciência, para enviar seus pensamentos e ataques a ciência, ao inves de tentar telepatia, rezas ou mandingas (ou qualquer outra forma de comunicação não cientifica) justamente porque é mais eficaz e confiável que qualquer outra..:-)

Você não parece gostar de muitos dos aspectso do uso do conhecimento cientifico. Direito seu. E pode até mesmo abandona-los, todos, e ir viver em uma aldeia remota no Amazonas, sem (quase) nenhum contato com a ciência (os indios são muito receptivos a forasteiros, vão gostar de recebe-lo), e com todo o onus de viver dessa forma (filhos mortos, vida curta, doenças diversas, poucos dentes, etc). Mas está na verdade criticando comportamentos humanos, que são seres falhos como sabe, não a ciência. Esta, "estricto senso", não é boa ou má, cruel ou gentil, apenas eficiente, confiável em seus efeitos.

Você não "acredita" nela. Engraçado, ela não pede que se acredite em nada.:-) Deve duvidar sempre, até que evidencias se mostrem sólidas o bastante para uma conclusão. E é nesse ponto que você escorrega..:-) O pouso na Lua já tem evidencias suficientes para essa conclusão, em que pese as tolices de diversos malucos mundo afora e suas teorias de conspiração. O HIV tem recuado e matado muito menos que no inicio da epidemia, graças a AZT e os atuais coqueteis antivirais. Se ajustes são feitos a toda hora, e são, sobre o que comer ou não, é apenas porque a ciência não pretende ser "A VERDADE" como religiões, mas um conhecimento que cresce, se aprofunda, se ajusta e melhora. Sabemos mais, não menos sobre o que comer ou não comer.

Sim, dá trabalho, é preciso atenção e constante leitura para acompanhar, seria melhor ter uma única resposta para todas as perguntas e pronto, sem risco e sem mudanças. Mas o mundo não funciona assim..:-) Sabemos hoje mais do que ontem sobre como se alimentar, e saberemos mais amanhã, deixando parte do que sabemos hoje para tras. Parte, não tudo:-)

As teorias conspiratórias são atrativas.:-) É como se nós, os que estão "por dentro", soubessemos de "coisas" que o resto do mundo não sabe, fossemos mais "espertos" ou mais inteligentes, que não se deixam enganar. Mas, na maioria das vezes (eu diria em sua totalidade) é apenas o ego tentando ser mais do que é.:-) Mesmo com centenas de sites "desmistificando" o pouso na Lua, o volume de evidencias a favor é gigantesco e nenhum deles explica um ponto fundamental do problema: como o governo americano fraudou os sinais de radio e TV que eram emitidos de todas as naves Apollo tanto durante as viagens, como do solo lunar, captados por todo radio-amador ao redor do globo, sem falar nos radares e antenas dos paises em disputacom os USA? (lembre-se que, mesmo hoje, não se pode fraudar a origem de um sinal de radio..:-)

Enfim, a ciência é eficaz. Embora seu uso, o uso do conhecimento confiável por ela produzido, possa ser discutido e julgado, isso não é a ciência nem com ela se confunde. E, tenha certeza, seu automóvel vai conduzi-lo amanhã, confiavelmente, conforme as leis da fisica descobertas pela ciência, seu computador receberá esta mensagem, os remédios na farmacia impedirão que você morra, as vacinas que tomou evitarão que adoeça (como todos os seus ancestrais adoeceram) e os satelites em órbita, colocados lá pelo conhecimento cientifico, vão transmitir suas chamadas telefonicas até para o Japão, se você assim desejar..:-)

Um abraço.

Homero




----- Original Message -----
From: Amauri Jr
To: ciencialist@yahoogrupos.com.br
Sent: Sunday, January 02, 2005 3:18 PM
Subject: Re: [ciencialist] Evolucao e desafios - CartaCapital - 02/01/05


Minhas analises em baixo com [A]....
----- Original Message -----
From: L.E.R.de Carvalho
To: ciencialist@yahoogrupos.com.br
Sent: Sunday, January 02, 2005 1:52 PM
Subject: [ciencialist] Evolucao e desafios - CartaCapital - 02/01/05



>LabConsS - www.ufrj.br/consumo
>
>
>
> EVOLUÇÃO E DESAFIOS
>
>
> O País viu inúmeros avanços nas áreas médica e
>tecnológica, mas as carências sociais ainda são sua
>pior mazela
>
>Em uma livraria, olho rapidamente as manchetes das
>revistas expostas. A julgar pelas capas, é pouco
>provável que tenhamos problemas de saúde deste mês em
>diante. Soluções curativas. Remédios extremamente
>eficientes. Vida prolongada. Corpos e rostos sem um
>defeito. Independentemente da idade, claro. Câncer?
>Uma brincadeira. Paralisia? Pode preparar as pistas.
>Infarto? E daí?

[A] temos que pagar e muito caro para a "ciencia" nos curar de males que diz a "ciencia" um direito de todos.

>
>Deixando de lado uma razoável dose de exagero nessas
>manchetes, encho o peito de orgulho pelas proezas
>atingidas no campo da medicina e da saúde. Vinte anos
>atrás, tudo isso não passaria de sonho, e não
>existiria fora da cabeça imaginativa dos produtores de
>filmes de ficção científica. Mas, ainda hoje, para a
>esmagadora maioria dos cidadãos brasileiros, todo o
>acima mencionado não passa de um sonho, de uma ficção
>científica

{A] Parece que todo a ciencia é uma ficção cientifica, vamos dizer que o pensamento é um comercio hoje em dia, ficçao ou não, é com certeza sensacionalismo.

>Este ano que acaba nos deu boas notícias,
>encorajadoras, e notícias ruins, no mínimo
>preocupantes. Vimos uma senhora que sofreu um derrame
>cerebral voltar a andar com terapia baseada na
>introdução de células-tronco (células primitivas
>capazes de se transformar em praticamente qualquer
>outra célula normal) no cérebro afetado. Emocionante
>perceber o que esses pequenos passos podem significar
>para milhões de pessoas que não conseguem erguer o
>braço, ou mexer a perna. Parafraseando Neil Armstrong,
>um passo gigante para a humanidade.

{A} Armstrong, quem é ele? Aquele que "supostamente" foi a Lua? Celulas- tronco é um desafio para a humanidade de cura, o desejo de ajudar seu semelhante. Pensamentos inovadores tem estado junto a humanidade a muito tempo, isso pode ser descrito na historia de Tales de Mileto até Einstein




>A técnica das células-tronco não se restringe a
>restaurar a função de células cerebrais. Hoje em dia,
>pesquisadores brasileiros, e em outros países,
>intensificam seus esforços para melhorar o coração
>depois de infarto, os nervos após lesão traumática, a
>pele após queimadura. A lista parece não ter fim.

[A] A demora faz o crescimento dos beneficiados...
>
>Vimos a introdução de remédios geniais na prática
>médica. Geniais na sua concepção, e geniais na sua
>eficiência. Drogas que conseguem agir em um ponto
>específico da célula doente, da célula cancerosa,
>dificultando seu desenvolvimento, seu crescimento, e
>até provocando sua morte. Exemplos que estão
>progressivamente sendo utilizados na prática médica,
>apesar de somente em casos muito selecionados, não
>faltam. Glivec, Iressa, são alguns deles.

[A] Santa ciencia!! Mata e de forma covarde porque usa seus conhecimentos de forma de interesse, mesmo que sabemos, esses conhecimentos são para todos.
>
>Ainda nem acabou o ano e ouvimos há poucas semanas o
>anúncio pela GlaxoSmithKline de uma vacina contra o
>câncer. O Brasil está totalmente empenhado na produção
>da vacina contra o HPV, vírus causador de câncer de
>útero. O prof. dr. Ricardo Brentani, presidente da
>Fundação Antonio Prudente, Hospital do Câncer AC
>Camargo, está entusiasmado e declara:
>
>­ Felizmente nossa instituição foi parceira da Merck
>Sharp & Dohme no desenvolvimento de uma vacina contra
>o HPV. Minha esperança é que em 20 anos tenhamos
>prevenido 7% dos tumores humanos.

[A] Vacina? Como pode ter vacina para nossas proprias celulas? E do HIV vai ter?

>Vimos, por outro lado, a preocupação crescente dos
>cientistas em não excluir, a priori, tratamentos
>considerados até então não-convencionais, ou
>complementares. Estudos sérios avaliaram desde a
>acupuntura até a homeopatia, passando por terapias de
>Florais de Bach e tratamentos com vitaminas nas mais
>diversas situações clínicas. Vantagens e desvantagens
>de cada abordagem foram dissecadas minuciosamente.

[A] Viu? Sera que alguem aqui duvidda?


>Cientistas conseguiram demonstrar a eficiência
>incontestável de algumas terapias alternativas em
>certas situações, como a massagem para um dos males do
>século, a dor nas costas. Por outro lado, alertaram
>para a ineficiência e até os efeitos nocivos de outras
>terapias, em outras condições. Por exemplo, para
>tratar bronquite e asma a acupuntura não parece ter
>efeitos importantes. Pelo menos não conseguiram
>detectar esses efeitos nos estudos atuais.

[A] Remedios sim? Eu tive começo, eu e meus manos, de bronquite depois que meu pai comprou um tartaruga nunca mais...santa crença ne? Mas como explicar??

>Vimos o lançamento de aparelhos ultramodernos,
>ultra-sensíveis, para detectar doenças e tratá-las.
>Não consigo perceber avanço recente maior do que na
>área da radioterapia. É incrível a precisão dos feixes
>de radiação em atingir o alvo, no caso o câncer, e
>poupar o tecido normal adjacente. Aparelhos novos
>conseguem acompanhar o movimento do corpo para seguir
>o alvo predeterminado, segundo a segundo, obedecendo à
>orientação do médico radioterapeuta. Reduziram-se
>muito os efeitos colaterais. O controle do câncer
>assemelha-se às extensas cirurgias. A nanotecnologia
>(aparelhos miniaturas) é uma febre, e os estudos
>multiplicam-se para definir com mais clareza sua
>aplicação.

{A} Pode matar com sua radioatividade, causando cancer.

>
>A plástica está fazendo tamanhos milagres que nem os
>próprios pacientes conseguem acreditar. Tanto faz quem
>foram seus pais, ou a etnia a qual você pertença.
>Escolha o modelo e ficará parecido. Para pessoas
>ansiosas com a imagem, soluções para quase tudo. Sem
>dúvida, o impacto sobre o estado emocional é notável.

[A] Eita faquinha de dois cumes heim? A plastica pode curar a pessoa de queimaduras ou cicatrizes, mas tambem pode ressaltar sua vaidade.

>
>Ao lado dessas notícias que mereceram um destaque
>quase obsessivo nas manchetes de capa, outras
>informações com menos, digamos, glamour, não
>conseguiram espaço nem nas páginas finais das
>revistas.
[A] A ciencia ja tem a imprensa e seus colaborados para dar esse glamour...


>
>O objetivo de reduzir em 60% a mortalidade infantil no
>mundo (e o Brasil ainda é um grande protagonista dessa
>estatística) não será atingido em 2015, como
>estabelecido nos Objetivos Milenares da ONU. No
>planeta, morrem por ano mais de 11 milhões de crianças
>com idade inferior a 5 anos. A maioria por doenças
>evitáveis. Diarréia, pneumonia, malária. No mesmo
>período, 500 mil mulheres morrem durante a gravidez ou
>o parto. Doença de Chagas e esquistossomose
>(barriga-d'água) continuam afetando milhares de
>brasileiros. E seu controle está cada vez mais
>próximo. A implementação dos programas já em ação
>poderá melhorar ainda mais esse controle. Para 2005 a
>intensificação dessas abordagens poderá elevar o
>impacto na saúde da população, principalmente nas
>áreas rurais.

[A] Isso ai, como fica doenças tao mais antiga diante do deusa peerfeita ciencia??


>O Relatório Mundial de Saúde (The World Health Report
>2003) recomendou às autoridades o fortalecimento dos
>sistemas de saúde, centralizando seu foco na atenção
>primária, além de integrar a prevenção das doenças e a
>promoção da saúde em todos os níveis de atendimento.

[A] Esses relatorios tem efeito??

>
>A saúde do homem sofreu mudanças drásticas nos últimos
>anos. A expectativa de vida, de brasileiros e de não
>brasileiros, bate recorde atrás de recorde.
>Ultrapassou a marca dos 70 anos e logo passará dos 80,
>90, e quem sabe 100 anos. Avanços sem dúvida notáveis.
>Doenças contagiosas foram substituídas por doenças
>crônicas, como problemas cardiovasculares e câncer, e
>por causas externas, como trauma. Essa mudança de foco
>exigiu modificações intensivas no nível estrutural.
>Algumas especialidades assumiram posição de destaque
>nunca antes alcançada. Os serviços de cardiologia de
>muitos hospitais logo se transformaram em
>departamentos e, a seguir, em grandes centros com
>prédios próprios. O mesmo ocorreu com a oncologia e
>suas variadas especialidades. Houve uma explosão na
>tecnologia e na sofisticação, e, conseqüentemente, nos
>custos ­ alertam os especialistas em saúde pública.
>Recomenda-se também a melhora da saúde da população
>por intermédio do envolvimento acadêmico.
>

[A] Sem comentarios, pra que vou querer viver até o 100?
>
>Muito por fazer.
>Como melhorar o atendimento ao público do SUS

[A] Infelizmente a ciencia não melhora ela mesma vai melhorar o SUS, a ciencia infelizmente, anda atendendo interesses dos laboratorios e pondo em pratica, o que determina o codigo americano, Bisness in bisness...
>
>
>A academia médica no Brasil foi alterada sensivelmente
>nas últimas duas décadas. Progressivamente, as escolas
>médicas e os programas de pós-graduação introduziram
>disciplinas de pesquisa epidemiológica de problemas de
>saúde adaptados à realidade da população do País. Mais
>e mais livros têm sido publicados por acadêmicos
>brasileiros, com enfoque nacional. Mais e mais centros
>de treinamento oferecem vagas para médicos
>interessados em se aperfeiçoar no atendimento básico à
>saúde da família e à clínica geral. O impacto na
>melhora da atenção à saúde do brasileiro pode demorar
>a ser notado. Muito há de ser feito em 2005 e além.

[A] Infelizmente, tem muito moleque fazendo medicina pro papai e só sai meleca, ou o diagnostico é virose ou é exames a "toa"...


>O plano Fome Zero tenta corrigir um problema básico de
>saúde: a miséria e a conseqüente desnutrição. Ninguém
>precisa de doutorado para compreender a conexão direta
>entre a desnutrição e a ocorrência de doenças
>potencialmente graves. É um plano interessante, mas
>ainda longe de atingir seus objetivos de forma
>significativa. Em 2005, e além, o governo deverá fazer
>os ajustes finos para que o programa consiga alcançar
>seu alvo.
>
>A violência é problema de segurança pública, mas
>também é problema de saúde. Não podemos esquecer que,
>no ano passado, mais de 40 mil brasileiros foram
>assassinados. Se acreditarmos nos dados oficiais do
>Ministério da Saúde, naquele período, morreram mais
>brasileiros por tiro do que por câncer de pulmão. O
>controle da violência evitará milhares de mortes e
>seqüelas dramáticas.
>
>A malária é outro problema sem solução a curto prazo.
>Áreas extensas do território nacional são infestadas
>por mosquitos portadores dessa doença. O esforço das
>autoridades regionais e federais deverá ser
>intensificado nas várias frentes: pesquisa de vacinas,
>profilaxia, controle do mosquito e tratamento de
>pacientes infectados. A mortalidade por essa infecção
>deve ser reduzida a todo custo. Estudos recentes
>aventam que a vacina contra a malária pode estar ao
>alcance da ciência em prazo razoável.
>
>No mundo todo, um bilhão de pessoas são infectadas por
>parasitas, como os vermes, e o Brasil tem participação
>significativa nesses dados alarmantes. Vermes no
>intestino não são apenas feios. Eles causam problemas
>como deficiências nutricionais, e, em alguns casos,
>podem até ser fatais. Para mudar isso, saneamento
>básico é fundamental.
>
>Um estudo recentemente realizado e publicado por
>pesquisadores na Universidade do Ceará demonstrou
>claramente a possibilidade de praticamente eliminar os
>parasitas dos pacientes. Remédios eficazes, como a
>ivemerctina, podem ser a solução. Além de tratar o
>paciente, eliminam uma potencial fonte de contaminação
>para outras pessoas. As autoridades devem criar
>sistemas integrados, contínuos, para atacar esse
>problema de saúde pública em várias frentes, em 2005 e
>para sempre.
>
>Quanto à Aids, nos últimos anos, houve um declínio da
>incidência de novos casos na maioria dos estados
>brasileiros. Estudo publicado recentemente por
>pesquisadores da Fundação Oswaldo Cruz, no Rio de
>Janeiro, confirmou que, apesar de recursos limitados
>do governo e da desigualdade socioeconômica aberrante
>no Brasil, a introdução do acesso universal à terapia
>antiviral contribuiu para uma redução impressionante
>na mortalidade por Aids, e pode ter ajudado a diminuir
>sua incidência.
>
>Nessa linha de pensamento, o presidente Lula lançou,
>este ano, o louvável Plano Nacional de Eliminação da
>Hanseníase (a lepra) em até dois anos. Um passo sem
>dúvida importante para controlar essa doença
>contagiosa e debilitante. Paralelamente, o ministro da
>Saúde, Humberto Costa, anunciou a formação de uma
>comissão para promover políticas nacionais de
>reabilitação às pessoas afetadas pela hanseníase:
>
>­ A idéia é possibilitar benefícios, como a
>reabilitação por cirurgias plásticas, financiando para
>a capacitação de médicos, enfermeiros e
>fisioterapeutas para que realizem esses procedimentos
>nos incapacitados fisicamente.
>
>Todos os anos, 42 mil brasileiros contraem a
>hanseníase. E o País só perde para a Índia no ranking
>mundial de casos da doença.
>
>A obesidade, mal do século XXI, mata. Nos EUA, em
>alguns estados o número de óbitos por obesidade
>ultrapassou o de câncer. No Brasil, cientistas
>identificaram obesidade crescente na população. Estudo
>publicado por pesquisadores da Universidade Federal do
>Rio de Janeiro confirmou o aumento progressivo do peso
>dos adolescentes, tanto na Região Nordeste quanto no
>Sudeste do País, desde 1975. Atualmente, 17% dos
>adolescentes da Região Sudeste enquadram-se na
>definição internacional de obesidade. E seus efeitos
>maléficos logo alcançarão a saúde pública. Orientação
>e conscientização devem ser política contínua e bem
>estruturada. A prevenção e o tratamento do sobrepeso
>precisam ser prioridades das políticas de saúde nos
>próximos anos.
>
>Um problema que acomete principalmente as crianças é a
>poluição atmosférica. Um estudo realizado com 5.193
>crianças de duas cidades do Rio de Janeiro, e
>publicado recentemente na revista Annals of Allergy
>Asthma and Immunology, mostrou claramente a correlação
>entre os níveis de poluição e a incidência de
>problemas respiratórios. Observou-se um aumento de
>mais de 50% na freqüência de crises de asma nos
>moradores de Duque de Caxias (local com alta
>concentração de poluentes), comparados aos moradores
>de Seropédica (local com baixos níveis de poluição).
>Outro estudo, realizado pelos pesquisadores do
>laboratório de poluição atmosférica da Universidade de
>São Paulo, confirmou os efeitos nocivos da poluição do
>ar na mortalidade infantil. Há aumento de 6% no número
>de óbitos neonatais nas regiões mais poluídas. O
>controle de poluentes de qualquer origem deverá ser
>obrigação mundial. Ainda mais com a entrada em vigor
>do Protocolo de Kyoto.
>
>Mas, de forma geral, provavelmente o maior desafio das
>autoridades de saúde é garantir um apoio ao avanço
>científico na área de saúde, atingir a maioria da
>população com os avanços médicos e tecnológicos
>alcançados, e insistir na prevenção e na saúde
>primária, básica. Muito mais do que dinheiro, em 2005
>precisamos de estratégia, filosofia e enfoque novos.
>
>O acesso da população, principalmente das camadas
>menos privilegiadas da sociedade, a remédios em geral
>é muito limitado. Várias são as causas, como preços
>elevados na fonte, acréscimos excessivos na cadeia de
>distribuição e impostos. Se as autoridades, de todas
>as esferas, não atentarem a essas dificuldades
>enfrentadas no dia-a-dia da maioria dos cidadãos,
>muito mais será gasto para tratar complicações graves
>de doenças crônicas, como diabetes e hipertensão
>arterial. O Brasil iniciou uma ação interessante nessa
>direção: a aquisição da fábrica da GlaxoSmithKline do
>Brasil pelo governo federal. O presidente Lula
>destacou que "pela primeira vez um governo compra uma
>fábrica da iniciativa privada", ao mesmo tempo
>criticando "o processo inverso de privatização do
>governo anterior". Lula declarou, na ocasião, que
>estava "recuperando uma fábrica que seria desativada,
>eliminaria empregos e que agora produzirá, em escala
>nacional, os antibióticos mais usados no Brasil".
>
>Nessa mesma linha o ministro da Saúde, Humberto Costa,
>deixou muito clara sua estratégia para 2005:
>
>­ Vamos ampliar o acesso da população a medicamentos
>fundamentais com base em três opções. A primeira é a
>rede de farmácias populares, e até o fim do ano
>pretendemos inaugurar cem unidades em todo o Brasil. A
>segunda é a criação de um programa para a venda
>subsidiada de medicamentos básicos para hipertensão e
>diabetes, através da rede de farmácias privada.
>Pretendemos baixar os preços em até 50%. E a terceira
>é a redução do ICMS de 2,8 mil medicamentos até 2005.
>
>De acordo com uma pesquisa do IBGE, de 2003, a saúde
>aparece em terceiro lugar no orçamento das famílias
>brasileiras, e os medicamentos representam 61% desses
>gastos para as pessoas de baixa renda. Além disso,
>metade das pessoas que precisam de tratamento não pode
>pagar os remédios de que necessitam.
>
>A partir de 2005, 50 milhões de unidades de
>antibióticos serão produzidas para a rede do Sistema
>Único de Saúde (SUS) e para as Farmácias Populares. Em
>2007, a produção deve quintuplicar em relação à atual.
>Mais de 10 bilhões de unidades de medicamentos devem
>ser produzidas para as principais doenças que mais
>atingem a população brasileira, como hipertensão,
>diabetes, malária e tuberculose. A Fiocruz também
>produzirá mais vacinas, e o Ministério da Saúde
>enfatiza que "a ampliação do acesso da população aos
>medicamentos é uma das prioridades do governo
>federal".
>
>Em 2004, muito se fez pela saúde, mas ainda há muito
>mais por fazer. Programas de prevenção e detecção
>precoce de doenças sexualmente transmissíveis, como
>Aids, HPV (câncer de colo de útero), a melhoria na
>qualidade do atendimento dos pacientes do SUS,
>diminuição das filas, controle de poluição, de
>infecção hospitalar e, também, de mortes por agentes
>externos, como traumas (tiros, facadas, acidentes de
>trânsito). Para se ter uma idéia do custo com os
>acidentes de trânsito, a cada ano acontecem mais de
>100 mil mortes, com três a quatro vezes esse número de
>feridos. E a maioria absoluta é tratada com dinheiro
>público. São milhões de reais de gastos evitáveis. Sem
>falar nas seqüelas individuais.
>

Abraços
Amauri

[As partes desta mensagem que não continham texto foram removidas]



##### ##### #####

Para saber mais visite
http://www.ciencialist.hpg.ig.com.br


##### ##### ##### #####



Yahoo! Grupos, um serviço oferecido por:
PUBLICIDADE




------------------------------------------------------------------------------
Links do Yahoo! Grupos

a.. Para visitar o site do seu grupo na web, acesse:
http://br.groups.yahoo.com/group/ciencialist/

b.. Para sair deste grupo, envie um e-mail para:
ciencialist-unsubscribe@yahoogrupos.com.br

c.. O uso que você faz do Yahoo! Grupos está sujeito aos Termos do Serviço do Yahoo!.



[As partes desta mensagem que não continham texto foram removidas]



##### ##### #####

Para saber mais visite
http://www.ciencialist.hpg.ig.com.br


##### ##### ##### #####


Yahoo! Grupos, um serviço oferecido por:







------------------------------------------------------------------------------
Links do Yahoo! Grupos

a.. Para visitar o site do seu grupo na web, acesse:
http://br.groups.yahoo.com/group/ciencialist/

b.. Para sair deste grupo, envie um e-mail para:
ciencialist-unsubscribe@yahoogrupos.com.br

c.. O uso que você faz do Yahoo! Grupos está sujeito aos Termos do Serviço do Yahoo!.



[As partes desta mensagem que não continham texto foram removidas]



##### ##### #####

Para saber mais visite
http://www.ciencialist.hpg.ig.com.br


##### ##### ##### #####


Yahoo! Grupos, um serviço oferecido por:







------------------------------------------------------------------------------
Links do Yahoo! Grupos

a.. Para visitar o site do seu grupo na web, acesse:
http://br.groups.yahoo.com/group/ciencialist/

b.. Para sair deste grupo, envie um e-mail para:
ciencialist-unsubscribe@yahoogrupos.com.br

c.. O uso que você faz do Yahoo! Grupos está sujeito aos Termos do Serviço do Yahoo!.



[As partes desta mensagem que não continham texto foram removidas]



##### ##### #####

Para saber mais visite
http://www.ciencialist.hpg.ig.com.br


##### ##### ##### #####


Yahoo! Grupos, um serviço oferecido por:







------------------------------------------------------------------------------
Links do Yahoo! Grupos

a.. Para visitar o site do seu grupo na web, acesse:
http://br.groups.yahoo.com/group/ciencialist/

b.. Para sair deste grupo, envie um e-mail para:
ciencialist-unsubscribe@yahoogrupos.com.br

c.. O uso que você faz do Yahoo! Grupos está sujeito aos Termos do Serviço do Yahoo!.



[As partes desta mensagem que não continham texto foram removidas]



##### ##### #####

Para saber mais visite
http://www.ciencialist.hpg.ig.com.br


##### ##### ##### #####


Yahoo! Grupos, um serviço oferecido por:
PUBLICIDADE




------------------------------------------------------------------------------
Links do Yahoo! Grupos

a.. Para visitar o site do seu grupo na web, acesse:
http://br.groups.yahoo.com/group/ciencialist/

b.. Para sair deste grupo, envie um e-mail para:
ciencialist-unsubscribe@yahoogrupos.com.br

c.. O uso que você faz do Yahoo! Grupos está sujeito aos Termos do Serviço do Yahoo!.



[As partes desta mensagem que não continham texto foram removidas]



##### ##### #####

Para saber mais visite
http://www.ciencialist.hpg.ig.com.br


##### ##### ##### #####


Yahoo! Grupos, um serviço oferecido por:
PUBLICIDADE




------------------------------------------------------------------------------
Links do Yahoo! Grupos

a.. Para visitar o site do seu grupo na web, acesse:
http://br.groups.yahoo.com/group/ciencialist/

b.. Para sair deste grupo, envie um e-mail para:
ciencialist-unsubscribe@yahoogrupos.com.br

c.. O uso que você faz do Yahoo! Grupos está sujeito aos Termos do Serviço do Yahoo!.



[As partes desta mensagem que não continham texto foram removidas]



##### ##### #####

Para saber mais visite
http://www.ciencialist.hpg.ig.com.br


##### ##### ##### #####


Yahoo! Grupos, um serviço oferecido por:







------------------------------------------------------------------------------
Links do Yahoo! Grupos

a.. Para visitar o site do seu grupo na web, acesse:
http://br.groups.yahoo.com/group/ciencialist/

b.. Para sair deste grupo, envie um e-mail para:
ciencialist-unsubscribe@yahoogrupos.com.br

c.. O uso que você faz do Yahoo! Grupos está sujeito aos Termos do Serviço do Yahoo!.



[As partes desta mensagem que não continham texto foram removidas]



SUBJECT: Algumas questões simples...
FROM: "Paulo Henrique Lerbach Rodrigues" <phatleta@yahoo.com.br>
TO: <ciencialist@yahoogrupos.com.br>
DATE: 04/01/2005 15:10

Dia claro1º O homem foi ou não foi a Lua? As evidências que a oposição usa são realmente provas contrárias? Por exemplo no meu ponto de vista não existe vacuo total na lua, portanto existe matéria, como a diferença de temperatura é muito grande da parte escura pra parte clara da lua então á convexão da matéria quente e fria, dai pode ventar, como aqui na terra, que que vocês acham?

2º Tem um doido muito doido que expõe suas idéias malucas, mas interesantes, no seguinte site http://www.showdalua.com/ , que que vocês acham sobre a teoria que diz que a terra não gira em torno do sol?

3º De onde vem as bolinhas de ar quando a agua está fervendo, caramba, to encabulado com isso...

hehehe

um abraço pra vocês...

PH



[As partes desta mensagem que não continham texto foram removidas]



SUBJECT: Re: [ciencialist] Algumas questões simples...
FROM: "Oraculo" <oraculo@atibaia.com.br>
TO: <ciencialist@yahoogrupos.com.br>
DATE: 04/01/2005 16:12

Olá Paulo

he he he:-) Esse hoax é o mais resistente que e já ví..:-) Volta e meia ele reaparece, com leigos perguntando "mas, seria possível?".

Veja, as evidencias, sérias e científicas, são acachapantes, mas sempre, sempre, é possível inventar maluquices sobre qualquer assunto e este assunto é bem interessante. E deixa quem inventa com a sensação de "saber mais que o resto do mundo"como toda teoria conspiratória. Se ele usasse sua imaginaçào apra escrever um romance de ficção cientifica, seria ótimo (como O Cóigo da Vince..:-), mas ao espalhar o boato como realidade, ele se sente superior, embora nào ganhe tanto dinheiro como o autor do livro..:-)

Entretanto, analisadas com detalhe e cuidado, nenhuma dessas afirmações resiste, embora para algumas delas seja preciso um maior conhecimento científico que a média para compreender a falha. E é por isso que faz tanto sucesso, a média da população não consegue entender os princípios cientificos envolvidos, como as emissões de rádio e sua triangulação, que impedem que se "simule" uma emissão de rádio da Lua sem estar realmente lá..:-)

E também, como é um assunto recorrente, quem conhece a realidade das missões à Lua se cansa de tanto explicar cada bobagem e acaba respondendo com coisas como "que maluquice, que tolice, isso de novo, vão catar sapo, etc" e quem chegou agora no boato fica com a impressão que não existem respostas para as acusações de fraude e que estão apenas fugindo do assunto.

Para evitar essa sensação, de uma olhada no link abaixo, onde foi compilado uma série de respostas corretas às acusações mais comuns sobre a ida à Lua, com detalhes precisos, refutações cientificas e explicações coerentes. Não precisa, claro, acreditar nos autores, mas apenas avaliar as evidencias, argumentos e explicações e tirar sua própria conclusão (o que os que defendem a fraude não esperm que faça..:-). Dá um certo trabalho, porque, como eu disse, alguns dos argumentos e evidencias precisam de algum esforço de compreensão de teorias e conhecimentos cientificos.

Mas, pense, se quiser realmente saber como funciona este computador de onde escreve, os softwares, hardwares, a rede, protocolos e outros elementos, não precisaria de um esforço? Se um maluco lhe dissesse que os engenheiros e cientistas que desenvolvem computadores estão enganados e nada do que dizem é verdade e que ele tem outra teoria sobre o funcionamento dos mesmos, você acreditaria sem fazer esse esforço para compreender a teoria cientifica comumente aceita? Claro que não, não é?..:-)

É o mesmo para qualquer coisa complexa o suficiente para não ser evidente a primeira vsta de um leigo (somos todos leigos em algum assunto.:-). Então, o esforço vale a pena, para não cair em hoaxes e boatos como o da fraude da Lua..:-) E quando alguém falar a você sobre a "fraude da Lua", é você que se sentirá superior, por ter um conhecimento mais amplo, sólido e real que seu pobre e enganado colega..:-)

O link é este:

http://www.badastronomy.com/

Além de desmontar o hoax da Lua (link Fox TV and the Apollo Moon Hoax), o site tem diversos itens interessantes, como erros de concepção astronomicos, enganos comuns sobre astronomia e física, etc. Muito interessante e bem esclarecedor, dá para ficar muito tempo nele aprendendo bastante..:-)

Um abraço e bom divertimento.

Homero

----- Original Message -----
From: Paulo Henrique Lerbach Rodrigues
To: ciencialist@yahoogrupos.com.br
Sent: Tuesday, January 04, 2005 3:10 PM
Subject: [ciencialist] Algumas questões simples...


Dia claro1º O homem foi ou não foi a Lua? As evidências que a oposição usa são realmente provas contrárias? Por exemplo no meu ponto de vista não existe vacuo total na lua, portanto existe matéria, como a diferença de temperatura é muito grande da parte escura pra parte clara da lua então á convexão da matéria quente e fria, dai pode ventar, como aqui na terra, que que vocês acham?

2º Tem um doido muito doido que expõe suas idéias malucas, mas interesantes, no seguinte site http://www.showdalua.com/ , que que vocês acham sobre a teoria que diz que a terra não gira em torno do sol?

3º De onde vem as bolinhas de ar quando a agua está fervendo, caramba, to encabulado com isso...

hehehe

um abraço pra vocês...

PH



[As partes desta mensagem que não continham texto foram removidas]



##### ##### #####

Para saber mais visite
http://www.ciencialist.hpg.ig.com.br


##### ##### ##### #####


Yahoo! Grupos, um serviço oferecido por:

São Paulo Rio de Janeiro Curitiba Porto Alegre Belo Horizonte Brasília




------------------------------------------------------------------------------
Links do Yahoo! Grupos

a.. Para visitar o site do seu grupo na web, acesse:
http://br.groups.yahoo.com/group/ciencialist/

b.. Para sair deste grupo, envie um e-mail para:
ciencialist-unsubscribe@yahoogrupos.com.br

c.. O uso que você faz do Yahoo! Grupos está sujeito aos Termos do Serviço do Yahoo!.



[As partes desta mensagem que não continham texto foram removidas]



SUBJECT: Tornado. Primeira foto no Brasil?
FROM: "Cyberlander" <cybernews@superig.com.br>
TO: <Undisclosed-Recipient:;>
DATE: 04/01/2005 22:24



Santa Catarina foi tamb�m palco de um "primeiro" furac�o ( moderado) brasileiro, ainda este ano.


Mudan�as clim�ticas?


CYBERLANDER

Ama a realidade que constr�is,
que nem a morte deter� teu voo � �




[As partes desta mensagem que n�o continham texto foram removidas]



SUBJECT: Re: [ciencialist] Tornado. Primeira foto no Brasil?
FROM: "Cyberlander" <cybernews@superig.com.br>
TO: <ciencialist@yahoogrupos.com.br>
DATE: 04/01/2005 22:30

agora com o link: http://img.photobucket.com/albums/v291/Cyberlander/abre04012005.jpg

[ ]'s
D.C.


From: Cyberlander
To: Undisclosed-Recipient:;
Sent: Tuesday, January 04, 2005 10:24 PM
Subject: [ciencialist] Tornado. Primeira foto no Brasil?





Santa Catarina foi também palco de um "primeiro" furacão ( moderado) brasileiro, ainda este ano.


Mudanças climáticas?


CYBERLANDER

Ama a realidade que constróis,
que nem a morte deterá teu voo · ·




[As partes desta mensagem que não continham texto foram removidas]



##### ##### #####

Para saber mais visite
http://www.ciencialist.hpg.ig.com.br


##### ##### ##### #####
Links do Yahoo! Grupos










[As partes desta mensagem que não continham texto foram removidas]



SUBJECT: Re: Algumas questões simples...
FROM: Maria Natália <grasdic@hotmail.com>
TO: ciencialist@yahoogrupos.com.br
DATE: 04/01/2005 23:19


Paulo:

Não se trata "do meu ponto de vista" estamos numa lista de ciência e
ou se estudou, se está a estudar o ramo da Ciência de que estamos a
falar ou então perguntamos.
A Lua é os objectos celestes mais bem conhecidos e estudados.
Não vou voltar aqui a falar de o homem ter estado ou não na lua e
porque essa sua dúvida já foi aqui falada pelo menos umas 3 vezes
desde que esta lista existe. Acho que deve ir ao site da
badastronomy. Pelo google chegará lá. Suponho também que não esteve
a pé durante aquela noite de Julho de 1969...e isso dá a sua idade.
Eu por exemplo não me acredito que em Marte andem rovers "aquilo são
tudo larachas" para nos desviar da questão da guerra do Iraque. Tá a
perceber o que lhe quero dizer?
Mas voltando então atrás:Na lua existe equipamento lá deixado pelas
sondas não tripuladas, tem sido explorda por satélites e como deve
saber de sua física existem pontas de prova, censores de raios
laser, micro ondas, infravermelho... e que da terra também recolhem
informações. Para além disso os eclipses da lua servem para estudo
de sua atmosfera por exemplo.E os astrónomos fazem fotografia não só
para embelezar as paredes do Observatório ou de seus gabinetes. Tudo
permite estudar a lua e saber muito.
Sabemos mais da lua do que do interior de nossa terra, veja lá. Na
sua universidade ou na mais próxima de sua escola vá ao departamento
de astrofísica/física/astronomia/geologia e quase de certeza há
alguém a fazer tese com tema Lua.
Quanto a doidos na net eles sempre existirão e só o nossos
conhecimentos em F´sica e Química e geologia nos podem fazer a
selecção do que é Ciência e Pseudociência. Estas suas observações me
fazem lembrar uma lista de anedotas e que se chama "Teorias da
conspiração". Por lá há muito doido e como deve calcular esses
doidos até podem andar à solta porque não matam ainda, embora possam
moer alguns juizos.
Bolinhas de ar---aí uma semana ou duas tem uma mensagem
minha/Emiliano em que se explica isso. Mas para não pensar que estou
aborrecida por causa da sua visão lunática...lhe direi que as
primeiras bolhas são devidas ao ar que se encontra dissolvido na
água (ou solução aquosa aquecida). A solubilidade dos gases diminui
quando a temperatura aumenta logo se estamos a aquecer essa água o
ar se "sente" a mais e sai.E nota a formação de bolhas de ar junto
ás paredes antes da água ferver. À medida que a temperatura aumenta
as moléculas de ar dissolvidas começam a "ferver" tentando sair da
solução antes da ebulição da própria água (sic Páginas 539 e 540 do
Chang)Sobre a ebulição deve ler neste livro as páginas 499 a 506 não
esquecendo o capítulo 5 sobre gases. 5ª Edição desta BIBLÍA do
químico em português. Vale a compra.
E conte sempre com a simpatia dos amigos da C-List. Gostei de suas
dúvidas.Tem garra para ser um cientista. Muita gente tem medo de
perguntar.
Um abraço
Maria Natália
De livro fechado não se sai letrado
Não continuar a aprender é esquecer o que se sabe
A dúvida nos leva ao conhecimento



ciencialist@yahoogrupos.com.br, "Paulo Henrique Lerbach Rodrigues"
<phatleta@y...> escreveu
> Dia claro1º O homem foi ou não foi a Lua? As evidências que a
oposição usa são realmente provas contrárias? Por exemplo no meu
ponto de vista não existe vacuo total na lua, portanto existe
matéria, como a diferença de temperatura é muito grande da parte
escura pra parte clara da lua então á convexão da matéria quente e
fria, dai pode ventar, como aqui na terra, que que vocês acham?
>
> 2º Tem um doido muito doido que expõe suas idéias malucas, mas
interesantes, no seguinte site http://www.showdalua.com/ , que que
vocês acham sobre a teoria que diz que a terra não gira em torno do
sol?
>
> 3º De onde vem as bolinhas de ar quando a agua está fervendo,
caramba, to encabulado com isso...
>
> hehehe
>
> um abraço pra vocês...
>
> PH
>
>
>
> [As partes desta mensagem que não continham texto foram removidas]





SUBJECT: Re: [ciencialist] Algumas questões simples...
FROM: "Oraculo" <oraculo@atibaia.com.br>
TO: <ciencialist@yahoogrupos.com.br>
DATE: 04/01/2005 23:20

Olá Paulo

Mais uma página muito boa para refutar o hoax da Lua:

http://www.clavius.org/

Com detalhes da analise de fotos, tecnologia disponível, as conspirações, etc.

Bem detalhada, passo a passo.:-)

Um abraço.

Homero

----- Original Message -----
From: Paulo Henrique Lerbach Rodrigues
To: ciencialist@yahoogrupos.com.br
Sent: Tuesday, January 04, 2005 3:10 PM
Subject: [ciencialist] Algumas questões simples...


Dia claro1º O homem foi ou não foi a Lua? As evidências que a oposição usa são realmente provas contrárias? Por exemplo no meu ponto de vista não existe vacuo total na lua, portanto existe matéria, como a diferença de temperatura é muito grande da parte escura pra parte clara da lua então á convexão da matéria quente e fria, dai pode ventar, como aqui na terra, que que vocês acham?

2º Tem um doido muito doido que expõe suas idéias malucas, mas interesantes, no seguinte site http://www.showdalua.com/ , que que vocês acham sobre a teoria que diz que a terra não gira em torno do sol?

3º De onde vem as bolinhas de ar quando a agua está fervendo, caramba, to encabulado com isso...

hehehe

um abraço pra vocês...

PH



[As partes desta mensagem que não continham texto foram removidas]



##### ##### #####

Para saber mais visite
http://www.ciencialist.hpg.ig.com.br


##### ##### ##### #####


Yahoo! Grupos, um serviço oferecido por:

São Paulo Rio de Janeiro Curitiba Porto Alegre Belo Horizonte Brasília




------------------------------------------------------------------------------
Links do Yahoo! Grupos

a.. Para visitar o site do seu grupo na web, acesse:
http://br.groups.yahoo.com/group/ciencialist/

b.. Para sair deste grupo, envie um e-mail para:
ciencialist-unsubscribe@yahoogrupos.com.br

c.. O uso que você faz do Yahoo! Grupos está sujeito aos Termos do Serviço do Yahoo!.



[As partes desta mensagem que não continham texto foram removidas]



SUBJECT: Re: [ciencialist] Tornado. Primeira foto no Brasil?
FROM: "Prof. JC" <profjc2003@yahoo.com.br>
TO: <ciencialist@yahoogrupos.com.br>
DATE: 05/01/2005 02:30

Parece a fumaça da turbina de um OVNI. :)))

Abraços,
Prof. JC


----- Original Message -----
From: "Cyberlander" <cybernews@superig.com.br>
To: <ciencialist@yahoogrupos.com.br>
Sent: Tuesday, January 04, 2005 10:30 PM
Subject: Re: [ciencialist] Tornado. Primeira foto no Brasil?



agora com o link:
http://img.photobucket.com/albums/v291/Cyberlander/abre04012005.jpg

[ ]'s
D.C.


From: Cyberlander
To: Undisclosed-Recipient:;
Sent: Tuesday, January 04, 2005 10:24 PM
Subject: [ciencialist] Tornado. Primeira foto no Brasil?





Santa Catarina foi também palco de um "primeiro" furacão (
moderado) brasileiro, ainda este ano.


Mudanças climáticas?


CYBERLANDER

Ama a realidade que constróis,
que nem a morte deterá teu voo · ·




[As partes desta mensagem que não continham texto foram removidas]



##### ##### #####

Para saber mais visite
http://www.ciencialist.hpg.ig.com.br


##### ##### ##### #####
Links do Yahoo! Grupos










[As partes desta mensagem que não continham texto foram removidas]



##### ##### #####

Para saber mais visite
http://www.ciencialist.hpg.ig.com.br


##### ##### ##### #####
Links do Yahoo! Grupos












SUBJECT: À procura do Tesouro de Einstein ou já se esqueceram?
FROM: Maria Natália <grasdic@hotmail.com>
TO: ciencialist@yahoogrupos.com.br
DATE: 05/01/2005 02:37


Professores de Física desta lista não vos vejo a comunicar aqui à
lista os vossos projectos...Sem ideias...em férias? Mas que desculpa
mais esfarrapadas para quem gosta tanto do nosso homem!?
Esta ideia é dos EUA e claro vem especial Leo. Pega-se num
tesouro e se esconde num local. As pistas para lá se chegar são
experiências...e depois é quente frio, morno, quente e...se ganha
conhecimento.
Vale tudo e até copiar.

"Hello Everyone. I hope you had a great holiday with your family
and loved ones. I received this in the mail and wanted to share it
with the group.
*********************************************************************
***********************************

Join middle school students across the US in a search for Einstein's
mysterious treasure. The search is part of the World Year of
Physics 2005 learning adventure, PhysicQuest. Based on a series of
puzzles outlined in a fictional version of Einstein's last will,
students solve the puzzles by completing enlightening experiments.
Each experiment brings students closer to locating the treasure, as
they simultaneously gain insight into the ways that physics helps us
understand our world.

Sign up for this adventure and you will receive a FREE science
detective kit for your classroom. Students who complete the
PhysicsQuest adventure are eligible to win the grand prize of a trip
to the site of the hidden treasure, sponsored by the World Year of
Physics 2005. For more information or to participate, visit the
World Year of Physics 2005 website and click on "teachers."
www.physics2005.org

This porject is limited to the first 10,000 US classrooms."

Sorrir é o melhor,
><((((º>`·.¸¸.·´¯`·.¸¸><((((º>¸. ·´¯`·.¸.¸¸><((((º> ,
.·´¯`·..><((((º>`·.¸¸.·´¯`·..¸><((((º>¸. ·´¯``·.¸.¸¸><((((º> para o
ALF
Um abraço de
Maria Natália








SUBJECT: Re: Tornado. Primeira foto no Brasil?
FROM: "rmtakata" <rmtakata@altavista.net>
TO: ciencialist@yahoogrupos.com.br
DATE: 05/01/2005 04:39


Na verdade ha' fotos da decada de 60 como esta daqui:

http://www2.uol.com.br/cliquemusic/imgfotos/1139.jpg

Ao q. eu sei foi tirada em algum ponto da BR 3.

[]s,

Roberto Takata





SUBJECT: Teste de envio de mensagem!
FROM: "Lomovtov" <lomovtov@yahoo.com.br>
TO: <ciencialist@yahoogrupos.com.br>
CC: <lomovtov@yahoo.com.br>, <marcelfigueredo@yahoo.com.br>
DATE: 05/01/2005 07:23



Isso é apenas um teste....

[As partes desta mensagem que não continham texto foram removidas]



SUBJECT: Re: [ciencialist] Tornado. Primeira foto no Brasil?
FROM: "Alessandro D. R. Fazenda" <alessandro@servidados.com.br>
TO: <ciencialist@yahoogrupos.com.br>
DATE: 05/01/2005 09:15

Olha tche, se eu não me engano, teve umas fotos do tornado que passou uns anos atras aqui em São Chico (São Francisco de Paula - RS). Parecia ser menor que esse de Santa, mas causou bastante estrago na cidade.
Infelizmente isso tem jeito de ser um fenomeno não tão incomum por aqui...

sds

Alessandro

ps: só não vamos confundir tornados com furacões!

----- Original Message -----
From: Cyberlander
To: ciencialist@yahoogrupos.com.br
Sent: Tuesday, January 04, 2005 10:30 PM
Subject: Re: [ciencialist] Tornado. Primeira foto no Brasil?



agora com o link: http://img.photobucket.com/albums/v291/Cyberlander/abre04012005.jpg

[ ]'s
D.C.


From: Cyberlander
To: Undisclosed-Recipient:;
Sent: Tuesday, January 04, 2005 10:24 PM
Subject: [ciencialist] Tornado. Primeira foto no Brasil?





Santa Catarina foi também palco de um "primeiro" furacão ( moderado) brasileiro, ainda este ano.


Mudanças climáticas?


CYBERLANDER

Ama a realidade que constróis,
que nem a morte deterá teu voo · ·





[As partes desta mensagem que não continham texto foram removidas]



SUBJECT: polar e apolar
FROM: "E m i l i a n o C h e m e l l o" <chemelloe@yahoo.com.br>
TO: <ciencialist@yahoogrupos.com.br>
DATE: 05/01/2005 10:11

alguém com paciência para explicar a 'boa' pergunta que o moço fez?

[ ] 's
Emiliano
---
De:"Ronaldo Stadtlander" <rlander@pop.com.br>
Para:Emiliano Chemello
Assunto:Substancias polares e apolares
Mensagem:Ola,
Em visita ao site www.ucs.br referente a material didático, pude ler sobre
pontes
de hidrogenio e soluções com características polares e apolares. Como não
sou da
área química, gostaria de tirar algumas dúvidas caso fosse possivel.

A água é uma substância polar e mistura-se facilmente com outras substâncias
orgânicas também polares tais como alcoois e seus semelhantes. O alcool por
sua vez
se mistura a gasolina embora a água não tenha esta mesma carcterística. o
alcool
tem a capacidade de ser polar e apolar? Como se explica o fato de um
composto ser
polar e apolar ao mesmo tempo? Como se dá a mistura da gasolina com o
alcool?

Desde já agradeço.
Ronaldo Stadtlander.




SUBJECT: aranhas...
FROM: "E m i l i a n o C h e m e l l o" <chemelloe@yahoo.com.br>
TO: <ciencialist@yahoogrupos.com.br>
DATE: 05/01/2005 10:14

Alguém?

[ ] 's do Emiliano Chemello
emiliano@quimica.net
http://www.quimica.net/emiliano
http://www.ucs.br/ccet/defq/naeq


Contato Naeq:
Nome: Camila
Email: camila_alves19@hotmail.com
Telefone: Dúvidas!!
Mensagem: Há alguns dias eu venho pesquisando sobre aranhas, no meu jardim
tem 2 aranhas construtoras de teia, elas são sedentárias estão sempre
paradas, posicionam suas patas dianteiras para cima e suas traseiras para
baixo fica parecendo que só são 4, suas patas são listradas de amarelo, uma
listra grande amarela pode ser identificada em seu abdome. Mas a minha
dúvida é se elas são venenosas. Em certos sites podemos encontrar que as
aranhas clinicamente perigosas não são construtoras de teias e sim errantes,
mas eu queria saber mais sobre essas aranhas do meu jardim, pois não
encontro nada em sites, apenas o que tenho são essas características. Se
puderem me ajudar, estou estudando para fazer faculdade de biologia. Minha
curiosidade sobre assuntos biologicos é grande.
Obrigada!



SUBJECT: Re: [ciencialist] aranhas...
FROM: Erich Weick <erich_weick@yahoo.com.br>
TO: ciencialist@yahoogrupos.com.br
DATE: 05/01/2005 12:15

> posicionam suas patas dianteiras para cima e suas traseiras para
> baixo fica parecendo que só são 4, suas patas são listradas de amarelo, uma
> listra grande amarela pode ser identificada em seu abdome.


Não estou certo, mas parece com o comportamento da aranha armadeira que é muito venenosa.
Espero ter ajudado.

Erich





_______________________________________________________
Yahoo! Acesso Grátis - Instale o discador do Yahoo! agora. http://br.acesso.yahoo.com/ - Internet rápida e grátis


SUBJECT: Re: [ciencialist] Tornado. Primeira foto no Brasil?
FROM: José Renato <jrma@terra.com.br>
TO: <ciencialist@yahoogrupos.com.br>
DATE: 05/01/2005 12:18

O fenômeno é comum e tem ocorrido praticamente todos os anos. Com a mídia
ocupando grande espaço com notícias do desastre na Ásia esse tipo de
ocorrência passa a ser notícia importante incluindo comentários de
especialistas e previsões para atender à demanda excitada, atenta para esse
tipo de notícia no momento atual. Em países temperados é costume acompanhar
as informações do serviço meteorológico, fornecidas pelos meios de
comunicação com muito maior freqüência do que se faz no Brasil, por exemplo.

Abraços

José Renato

ET. A ocorrência do fenômeno mencionado por Takata foi considera de grande
impacto. Atualmente os fenômenos são outros e em geral são logo exportados.
O da década de 60 do século passado está bastante ameno mas ainda ocorre de
vez em quando na novela das oito.
............................................................

From: "Alessandro D. R. Fazenda" <alessandro@servidados.com.br>
To: <ciencialist@yahoogrupos.com.br>
Sent: Wednesday, January 05, 2005 8:15 AM
Subject: Re: [ciencialist] Tornado. Primeira foto no Brasil?



Olha tche, se eu não me engano, teve umas fotos do tornado que passou uns
anos atras aqui em São Chico (São Francisco de Paula - RS). Parecia ser
menor que esse de Santa, mas causou bastante estrago na cidade.
Infelizmente isso tem jeito de ser um fenomeno não tão incomum por aqui...

sds

Alessandro

ps: só não vamos confundir tornados com furacões!

----- Original Message -----
From: Cyberlander
To: ciencialist@yahoogrupos.com.br
Sent: Tuesday, January 04, 2005 10:30 PM
Subject: Re: [ciencialist] Tornado. Primeira foto no Brasil?



agora com o link:
http://img.photobucket.com/albums/v291/Cyberlander/abre04012005.jpg

[ ]'s
D.C.


From: Cyberlander
To: Undisclosed-Recipient:;
Sent: Tuesday, January 04, 2005 10:24 PM
Subject: [ciencialist] Tornado. Primeira foto no Brasil?





Santa Catarina foi também palco de um "primeiro" furacão (
moderado) brasileiro, ainda este ano.


Mudanças climáticas?


CYBERLANDER

Ama a realidade que constróis,
que nem a morte deterá teu voo · ·





[As partes desta mensagem que não continham texto foram removidas]



##### ##### #####

Para saber mais visite
http://www.ciencialist.hpg.ig.com.br


##### ##### ##### #####
Links do Yahoo! Grupos









E-mail classificado pelo Identificador de Spam Inteligente Terra.
Para alterar a categoria classificada, visite
http://www.terra.com.br/centralunificada/emailprotegido/imail/imail.cgi?+_u=jrma&_l=1,1104923681.897350.25025.soluezi.terra.com.br,4844,Des15,Des15

Esta mensagem foi verificada pelo E-mail Protegido Terra.
Scan engine: McAfee VirusScan / Atualizado em 29/12/2004 / Versão: 4.4.00 -
Dat 4417
Proteja o seu e-mail Terra: http://www.emailprotegido.terra.com.br/




SUBJECT: Re: Tornado. Primeira foto no Brasil?
FROM: André Roviralta Dias Baptista <andrediasbaptista@yahoo.com.br>
TO: ciencialist@yahoogrupos.com.br
DATE: 05/01/2005 15:20


--- Em ciencialist@yahoogrupos.com.br, José Renato <jrma@t...>
escreveu
> O fenômeno é comum e tem ocorrido praticamente todos os anos. Com a
mídia ocupando grande espaço com notícias do desastre na Ásia esse
tipo de ocorrência passa a ser notícia importante incluindo
comentários de especialistas e previsões para atender à demanda
excitada, atenta para esse tipo de notícia no momento atual. Em
países temperados é costume acompanhar as informações do serviço
meteorológico, fornecidas pelos meios de comunicação com muito maior
freqüência do que se faz no Brasil, por exemplo.
>

Não é a primeira foto, mas não é um fenomeno comum no Brasil (se
compararmos com o meio-oeste americano, por exemplo). Ja vi fotos de
uma tromba dágua em SC (tornado sobre o mar) e pelos 1 tornado foi
bem documentado em ITU, pelo rastro de destruição característico. Não
entendi a referência à Ásia. Se não estivessemos sob o impacto do
desastre na Ásia penso que os tornados receberiam mais, e não menos,
destaque na mídia, visto tratar-se de um dos fenomenos atmosféricos
mais espetaculares (além de potencialmente catastróficos).

[]´s

André









SUBJECT: Re: [ciencialist] Tornado. Primeira foto no Brasil?
FROM: "Alvaro Augusto - Electra" <alvaro@electraenergy.com.br>
TO: <ciencialist@yahoogrupos.com.br>
DATE: 05/01/2005 16:46

Para quem não estava embaixo do bicho, a intensidade foi mesmo moderada!

[ ]s

Alvaro Augusto


----- Original Message -----
From: "Cyberlander" <cybernews@superig.com.br>
To: <Undisclosed-Recipient:;>
Sent: Tuesday, January 04, 2005 10:24 PM
Subject: [ciencialist] Tornado. Primeira foto no Brasil?





Santa Catarina foi também palco de um "primeiro" furacão ( moderado)
brasileiro, ainda este ano.


Mudanças climáticas?


CYBERLANDER

Ama a realidade que constróis,
que nem a morte deterá teu voo · ·






SUBJECT: Sagan (era Evolucao e desafios - Amaury)
FROM: "Oraculo" <oraculo@atibaia.com.br>
TO: <ciencialist@yahoogrupos.com.br>
DATE: 05/01/2005 19:05

Olá Amaury

Relendo um texto de Carl Sagan, do livro O Mundo Assombrado Pelos Demonios, chamado A Refinada Arte de Detectar Mentiras, eu achei interessante apra nossa discussão sobre a ciência e as imperfeições humanas. De uma lida, acho que vai gostar.

Este é um pequeno trecho:

"Sagan: As explicações pagas de produtos, especialmente se feitas por verdadeiros ou pretensos especialistas, constituem uma saraivada constante de logros. Revelam menosprezo pela inteligência dos clientes. Criam uma corrupção insidiosa das atitudes populares a respeito da objetividade científica. Hoje, existem até comerciais em que cientistas reais, alguns de considerável distinção, atuam como garotos-propaganda para as empresas. Eles nos ensinam que também os cientistas mentem por dinheiro. Como alertou Tom Paine, o fato de nos acostumarmos com mentiras cria o fundamento para muitos outros males. "

Interessante, não?..:-) Ele também separa a ciência, seu método, o pensamento racional, o cetiscismo, da aplicação prática e das falhas humanas (naturais em todos nós).

O texto completo segue abaixo.

Um abraço.

Homero
_____________________________________________________
A Arte Refinada de Detectar Mentiras
Autor: Carl Sagan
Fonte: O Mundo Assombrado pelos Demônios

A compreensão humana não é um exame desinteressado, mas recebe infusões da vontade e dos afetos; disso se originam ciências que podem ser chamadas "ciências conforme a nossa vontade". Pois um homem acredita mais facilmente no que gostaria que fosse verdade. Assim, ele rejeita coisas difíceis pela impaciência de pesquisar; coisas sensatas, porque diminuem a esperança; as coisas mais profundas da natureza, por superstição; a luz da experiência, por arrogância e orgulho; coisas que não são comumente aceitas, por deferência à opinião do vulgo. Em suma, inúmeras são as maneiras, e às vezes imperceptíveis, pelas quais os afetos colorem e contaminam o entendimento.


Francis Bacon, Novum organon (1620)



Meus pais morreram há anos. Eu era muito ligado a eles. Ainda sinto uma saudade terrível. Sei que sempre sentirei. Desejo acreditar que sua essência, suas personalidades, o que eu tanto amava neles, ainda existe - real e verdadeiramente - em algum lugar. Não pediria muito, apenas cinco ou dez minutos por ano, para lhes contar sobre os netos, pô-las ao corrente das últimas novidades, lembrar-lhes que eu os amo. Uma parte minha - por mais infantil que pareça - se pergunta como é que estarão. "Está tudo bem?", desejo perguntar. As últimas palavras que me vi dizendo a meu pai, na hora de sua morte, foram: "Tome cuidado".

Às vezes sonho que estou falando com meus pais, e de repente - ainda imerso na elaboração do sonho - sou tomado pela consciência esmagadora de que eles não morreram de verdade, de que tudo não passou de um erro horrível. Ora, ali estão eles, vivos e bem de saúde, meu pai fazendo piadas inteligentes, minha mãe muito séria me aconselhando a usar uma manta porque está frio. Quando acordo, passo de novo por um processo abreviado de luto. Evidentemente, existe algo dentro de mim que está pronto a acreditar na vida após a morte. E que não está nem um pouco interessado em saber se há alguma evidência séria que confirme tal coisa.

Por isso, não rio da mulher que visita o túmulo do marido e conversa com ele de vez em quando, talvez no aniversário de sua morte. Não é difícil de compreender. E se tenho dificuldades com o status ontológico daquele com que ela está falando, não faz mal. Não é isso que importa. O que importa é que os seres humanos são humanos. Mais de um terço dos adultos norte-americanos acreditam que em algum nível estabeleceram contato com os mortos. O número parece ter dado um pulo de 15% entre 1977 e 1988. Um quarto dos norte-americanos acredita em reencarnação.

Mas isso não significa que estou disposto a aceitar as pretensões de um "médium", que afirma canalizar os espíritos dos seres amados que partiram, quando tenho consciência de que a prática está cheia de fraudes. Sei o quanto desejo acreditar que meus pais só abandonaram os cascos de seus corpos, como insetos ou cobras na muda, e partiram para outro lugar. Compreendo que esses sentimentos poderiam me tornar uma presa fácil até de um trapaceiro pouco inteligente, de pessoas normais que desconhecem suas mentes inconscientes, ou dos que sofrem de uma desordem psiquiátrica dissociativa. Relutantemente, ponho em ação algumas reservas de ceticismo.

Como é, pergunto a mim mesmo, que os canalizadores nunca nos dão informações verificáveis que nos são inacessíveis por outros meios? Por que Alexandre, o Grande, nunca nos informa sobre a localização exata de sua tumba, Fermat sobre o seu último teorema, James Wilkes Booth sobre a conspiração do assassinato de Lincoln, Hermann Goering sobre o incêndio do Reichstag? Por que Sófocles, Demócrito e Aristarco não ditam as suas obras perdidas? Não querem que as gerações futuras conheçam as suas obras-primas?

Se fosse anunciada alguma evidência real de vida após a morte, desejaria muito examiná-la; mas teria de ser uma evidência real científica, e não simples anedota. Em casos como A Face em Marte e os raptos por alienígenas, eu diria que é melhor a verdade dura do que a fantasia consoladora. E, no cômputo final, revela-se freqüentemente que os fatos são mais consoladores que a fantasia.

A premissa fundamental da "canalização", do espiritismo e de outras formas de necromancia é que não morremos quando experimentamos a morte. Não exatamente. Continua a existir alguma parte de nós que pensa, sente e tem memória. Seja o que for - alma ou espírito, nem matéria nem energia, mas alguma outra coisa -, essa parte pode entrar novamente em corpos humanos ou de outros seres, e assim a morte perde grande parte da sua ferroada. E ainda mais: se as afirmações do espírita ou canalizador são verdadeiras, temos uma oportunidade de entrar em contato com os seres amados que morreram.

J. Z. Knight, do estado de Washington, afirma estar em contato com um ser de 35 mil anos chamado Ramtha. Ele fala inglês muito bem, usando a língua, os lábios e as cordas vocais de Knight, com um sotaque que me parece ser hindu. Como a maioria das pessoas sabe como falar, e muitas - de crianças a atores profissionais - têm um repertório de vozes a seu dispor, a hipótese mais simples sugere que é a própria sra. Knight que faz Ramtha falar, e que ela não tem contato com entidades desencarnadas da época plistocena glacial. Se há provas em contrário, gostaria muito de conhecer. Seria consideravelmente mais impressionante se Ramtha pudesse falar por si mesmo, sem a ajuda da boca da sra. Knight. Isso não sendo possível, como podemos testar a afirmação? (A atriz Shirley MacLaine afirma que Ramtha foi seu irmão em Atlântida, mas isso já é outra história.)

Vamos supor que Ramtha pudesse ser interrogado. Poderíamos verificar se ele é quem afirma ser? Como é que ele sabe que viveu há 35 mil anos, mesmo aproximadamente? Que calendário emprega? Quem está tomando nota dos milênios intermediários? Trinta e cinco mil mais ou menos o quê? Como é que eram as coisas há 35 mil anos? Ou Ramtha tem realmente essa idade, e nesse caso vamos descobrir alguma coisa sobre esse período, ou é uma fraude e ele (ou melhor, ela) vai se trair.

Onde é que Ramtha vivia? (Sei que fala inglês com sotaque hindu, mas onde é que falavam assim há 35 mil anos?) Como era o clima? O que Ramtha comia? (Os arqueólogos têm alguma noção do que as pessoas comiam nessa época.) Quais eram as línguas autóctones, e qual era a estrutura social? Com quem mais Ramtha vivia - com a mulher, mulheres, filhos, netos? Qual era o ciclo da vida, a taxa de mortalidade infantil, a expectativa de vida? Eles tinham controle populacional? Que roupas vestiam? Como elas eram fabricadas? Quais os predadores mais perigosos? Os instrumentos e as estratégias da caça e da pesca? Armas? Sexismo endêmico? Xenofobia e etnocentrismo? E, se Ramtha descendia da "elevada civilização" de Atlântida, onde estão os detalhes lingüísticos, tecnológicos, históricos e de outra natureza? Como era a sua escrita? Respondam. Em lugar disso, a única coisa que recebemos são homilias banais.

Para dar outro exemplo, eis um conjunto de informações que não foram canalizadas de um morto antigo, mas de entidades não humanas desconhecidas que fazem círculos nas plantações, assim como foi registrado pelo jornalista Jim Schnabel:

"Estamos muito ansiosos por essa nação pecadora estar espalhando mentiras sobre nós. Não viemos em máquinas, não pousamos na Terra em máquinas [...]. Viemos como o vento. Somos a Força Vital. A Força Vital do solo [...]. Viemos até aqui [...]. Estamos apenas a um sopro de distância [...] a um sopro de distância [...] não estamos a milhões de milhas de distância [...] uma Força Vital que é mais potente que as energias no corpo humano. Mas nós nos reunimos num nível mais elevado de vida [...]. Não precisamos de nome. Vivemos num mundo paralelo ao seu, ao lado do seu [...]. Os muros se romperam. Dois homens surgirão do passado [...] o grande urso [...] o mundo encontrará a paz".


As pessoas dão atenção a essas maravilhas pueris, principalmente porque elas prometem algo parecido com a religião dos velhos tempos, mas sobretudo a vida depois da morte, até a vida eterna.

O versátil cientista britânico J.B.S. Haldane, que foi, entre muitas outras coisas, um dos fundadores da genética populacional, propôs certa vez uma perspectiva muito diferente para algo semelhante à vida eterna. Haldane imaginava um futuro distante em que as estrelas se obscureceram e o espaço foi preenchido em sua maior parte por um gás frio e fino. Ainda assim, se esperarmos bastante tempo, ocorrerão flutuações estatísticas na densidade desse gás. Ao longo de imensos períodos, as flutuações serão o suficiente para reconstituir um Universo parecido com o nosso. Se o Universo é infinitamente antigo, haverá um número infinito dessas reconstituições, apontava Haldane.

Assim, num Universo infinitamente antigo com um número infinito de nascimentos de galáxias, estrelas, planetas e vida, deve reaparecer uma Terra idêntica em que você e todos os seus seres queridos voltarão a se reunir. Serei capaz de rever meus pais e apresentar-lhes os netos que eles não conheceram. E tudo isso não acontecerá apenas uma vez, mas um número infinito de vezes.

Entretanto, de certo modo isso não oferece os consolos da religião. Se nenhum de nós vai lembrar o que aconteceu desta vez, a época que o leitor e eu estamos partilhando, as satisfações da ressurreição do corpo, pelo menos aos meus ouvidos, soam ocas.

Mas nessa reflexão subestimei o que significa infinidade. Na imagem de Haldane, haverá universos, na verdade um número infinito de universos, em que nossas mentes recordarão perfeitamente todas as vidas anteriores. A satisfação está à mão - moderada, no entanto, pela idéia de todos esses outros universos que também passarão a existir (novamente, não uma vez, mas um número infinito de vezes) com tragédias e horrores que superam em muito qualquer coisa que já experimentei desta vez.

Entretanto, o Consolo de Haldane depende do tipo de universo em que vivemos, e talvez de arcanos, como, por exemplo, saber se há bastante matéria para finalmente reverter à expansão do universo, e o caráter das flutuações no vácuo. Ao que parece, aqueles que sentem um profundo desejo de vida após a morte poderiam se dedicar à cosmologia, à gravidade quântica, à física das partículas elementares e à aritmética trans-finita.

Clemente de Alexandria, um dos padres da Igreja primitiva, em suas Exortações aos gregos (escritas em torno do ano 190), rejeitava as crenças pagãs em termos que pareceriam hoje em dia um pouco irônicos:

"Estamos realmente longe de permitir que os homens adultos dêem ouvidos a essas histórias. Mesmo aos nossos filhos, quando eles berram de cortar o coração, como se diz, não temos o hábito de contar histórias fabulosas para acalmá-los".


Em nossa época, temos padrões menos severos. Contamos às crianças histórias sobre Papai Noel, o coelhinho da Páscoa e a fada do dente por razões que achamos emocionalmente sadias, mas depois, antes de crescerem, nós os desiludimos sobre esses mitos. Por que nos desdizemos? Porque o seu bem-estar como adultos depende de eles conhecerem o mundo tal como é. Nós nos preocupamos, e com razão, com os adultos que ainda acreditam em Papai Noel.

Sobre as religiões doutrinárias, escreveu o filósofo David Hume que

"os homens não ousam confessar, nem mesmo a seus corações, as dúvidas que têm a respeito desses assuntos. Eles valorizam a fé implícita; e disfarçam para si mesmos a sua real descrença, por meio das afirmações mais convictas e do fanatismo mais positivo".


Essa descrença tem conseqüências morais profundas, como escreveu o revolucionário americano Tom Paine em The age of reason:

"A descrença não consiste em acreditar, nem em desacreditar; consiste em professar que se crê naquilo que não se crê. É impossível calcular o dano moral, se é que posso chamá-lo assim, que a mentira mental tem causado na sociedade. Quando o homem corrompeu e prostituiu de tal modo a castidade de sua mente, a ponto de empenhar a sua crença profissional em coisas que não acredita, ele está preparado para a execução de qualquer outro crime".


A formulação de T.H. Huxley foi:

"O fundamento da moralidade é [...] renunciar a fingir que se acredita naquilo que não comporta evidências, e a repetir proposições ininteligíveis sobre coisas que estão além das possibilidades do conhecimento".


Clement, Hume, Paine e Huxley estavam todos falando de religião. Mas grande parte do que escreveram tem aplicações mais gerais - por exemplo, para as importunidades disseminadas no pano de fundo de nossa civilização comercial: há um tipo de comercial de aspirina em que atores fingindo ser médicos revelam que o produto do concorrente tem apenas determinada fração do ingrediente analgésico que os médicos mais recomendam - eles não dizem qual é o misterioso ingrediente. Enquanto o seu produto tem uma quantidade drasticamente maior (1,2 a duas vezes mais por comprimido). Por isso, comprem esse produto. Mas por que não tomar dois comprimidos do concorrente? Ou considere-se o caso do analgésico que funciona melhor do que o produto de "potência regular" do concorrente. Por que não tomar o produto de "potência extra" do outro fabricante? E eles certamente não falam nada sobre as mais de mil mortes por ano causadas pelo uso da aspirina nos Estados Unidos ou os aparentes 5 mil casos anuais de disfunção renal provocados pelo uso de acetaminofeno, de que a marca mais vendida é o Tylenol. (Isso, contudo, talvez represente um caso de correlação sem causalidade.) Ou quem se importa em saber quais os cereais que têm mais vitamina, quando podemos tomar uma pílula de vitamina no café da manhã? Da mesma forma, que importa saber que um antiácido contém cálcio, se o cálcio serve para a nutrição e é irrelevante para a gastrite? A cultura comercial está cheia de informações errôneas e subterfúgios semelhantes à custa do consumidor. Não se devem fazer perguntas. Não pensem. Comprem.

As explicações pagas de produtos, especialmente se feitas por verdadeiros ou pretensos especialistas, constituem uma saraivada constante de logros. Revelam menosprezo pela inteligência dos clientes. Criam uma corrupção insidiosa das atitudes populares a respeito da objetividade científica. Hoje, existem até comerciais em que cientistas reais, alguns de considerável distinção, atuam como garotos-propaganda para as empresas. Eles nos ensinam que também os cientistas mentem por dinheiro. Como alertou Tom Paine, o fato de nos acostumarmos com mentiras cria o fundamento para muitos outros males.

Enquanto escrevo, tenho diante de mim o programa da Whole Life Expo, a exposição anual da Nova Era realizada em San Francisco. É comumente visitada por dezenas de milhares de pessoas. Ali especialistas muito questionáveis fazem propaganda de produtos muito questionáveis. Eis algumas das apresentações: "Como proteínas presas no sangue produzem dor e sofrimento". "Cristais, talismãs ou pedras?" (Tenho a minha opinião.) Prossegue: "Assim como um cristal focaliza as ondas sonoras e luminosas para o rádio e a televisão" - o que é um erro insípido de quem não compreende como o rádio e a televisão funcionam -, "ele pode amplificar as vibrações espirituais para o ser humano afinado". Ou mais esta: "O retorno da deusa, um ritual de apresentação". Outra: "Sincronismo, a experiência do reconhecimento". Essa é fornecida pelo "irmão Charles". Ou, na página seguinte: "Você, Saint-Germain e a cura pela chama violeta". E assim continua, com milhares de anúncios sobre as "oportunidades" - percorrendo a gama estreita que vai do dúbio ao espúrio - que se acham à disposição na Whole Life Expo.

Algumas vítimas de câncer, perturbadas, fazem peregrinações às Filipinas, onde "cirurgiões mediúnicos", depois de esconder na palma da mão pedaços de fígado de galinha ou coração de bode, fingem tocar nas entranhas do paciente e retirar o tecido doente, que é então triunfantemente exibido. Certos líderes de democracias ocidentais consultam regularmente astrólogos e místicos antes de tomar decisões de Estado. Sob a pressão pública por resultados, a polícia, às voltas com um assassinato não solucionado ou um corpo desaparecido, consulta "especialistas" de ESP (percepção extra-sensorial) (que nunca adivinham nada além do esperado pelo senso comum, mas a polícia, dizem os ESPs, continua a chamá-los). Anuncia-se a previsão de uma divergência com nações adversárias, e a CIA, estimulada pelo Congresso, gasta dinheiro dos impostos para descobrir se podemos localizar submarinos nas profundezas do oceano concentrando o pensamento neles. Um "médium" - usando pêndulos sobre mapas e varinhas rabdomânticas em aviões - finge descobrir novos depósitos minerais; uma companhia mineira australiana lhe adianta elevada soma de dólares, irrecuperável em caso de fracasso, garantindo-lhe uma participação na exploração do minério em caso de sucesso. Nada é descoberto. Algumas estátuas de Jesus ou murais de Maria ficam manchados de umidade, e milhares de pessoas bondosas se convencem de que testemunharam um milagre.

Todos esses são casos de mentiras provadas ou presumíveis. Acontece um logro, ora de forma inocente, mas com a colaboração dos envolvidos, ora com premeditação cínica. Em geral, a vítima se vê presa de forte emoção - admiração, medo, ganância, dor. A aceitação crédula da mentira talvez nos custe dinheiro; é o que P.T. Barnum apontou, ao afirmar: "Nasce um otário a cada minuto". Mas pode ser muito mais perigoso que isso, e quando os governos e as sociedades perdem a capacidade de pensar criticamente os resultados podem ser catastróficos - por mais que deploremos aqueles que engoliram a mentira.

Na ciência, podemos começar com resultados experimentais, dados, observações, medições, "fatos". Inventamos, se possível, um rico conjunto de explicações plausíveis e sistematicamente confrontamos cada explicação com os fatos. Ao longo de seu treinamento, os cientistas são equipados com um kit de detecção de mentiras. Este é ativado sempre que novas idéias são apresentadas para consideração. Se a nova idéia sobrevive ao exame das ferramentas do kit, nós lhe concedemos aceitação calorosa, ainda que experimental. Se possuímos essa tendência, se não desejamos engolir mentiras mesmo quando são confortadoras, há precauções que podem ser tomadas; existe um método testado pelo consumidor, experimentado e verdadeiro.

O que existe no kit ? Ferramentas para o pensamento cético.

O pensamento cético se resume no meio de construir e compreender um argumento racional e - o que é especialmente importante - de reconhecer um argumento falacioso ou fraudulento. A questão não é se gostamos da conclusão que emerge de uma cadeia de raciocínio, mas se a conclusão deriva da premissa ou do ponto de partida e se essa premissa é verdadeira.

Eis algumas das ferramentas:

. Sempre que possível, deve haver confirmação independente dos "fatos".

. Devemos estimular um debate substantivo sobre as evidências, do qual participarão notórios partidários de todos os pontos de vista.

. Os argumentos de autoridade têm pouca importância - as "autoridades" cometeram erros no passado. Voltarão a cometê-los no futuro. Uma forma melhor de expressar essa idéia é talvez dizer que na ciência não existem autoridades; quando muito, há especialistas.

. Devemos considerar mais de uma hipótese. Se alguma coisa deve ser explicada, é preciso pensar em todas as maneiras diferentes pelas quais poderia ser explicada. Depois devemos pensar nos testes que poderiam servir para invalidar sistematicamente cada uma das alternativas. O que sobreviver, a hipótese que resistir a todas as refutações nessa seleção darwiniana entre as "múltiplas hipóteses eficazes", tem uma chance muito melhor de ser a resposta correta do que se tivéssemos simplesmente adotado a primeira idéia que prendeu nossa imaginação *1.

. Devemos tentar não ficar demasiado ligados a uma hipótese, só por ser a nossa. É apenas uma estação intermediária na busca do conhecimento. Devemos nos perguntar por que a idéia nos agrada. Devemos compará-la imparcialmente com as alternativas. Devemos verificar se é possível encontrar razões para rejeitá-la. Se não, outros o farão.

. Devemos quantificar. Se o que estiver sendo explicado é passível de medição, de ser relacionado a alguma quantidade numérica, seremos muito mais capazes de discriminar entre as hipóteses concorrentes. O que é vago e qualitativo é suscetível de muitas explicações. Há certamente verdades a serem buscadas nas muitas questões qualitativas que somos obrigados a enfrentar, mas encontrá-las é mais desafiador.

. Se há uma cadeia de argumentos, todos os elos na cadeia devem funcionar (inclusive a premissa) - e não apenas a maioria deles.

. A Navalha de Occam. Essa maneira prática e conveniente de proceder nos incita a escolher a mais simples dentre duas hipóteses que explicam os dados com igual eficiência.

. Devemos sempre perguntar se a hipótese pode ser, pelo menos em princípio, falseada. As proposições que não podem ser testadas ou falseadas não valem grande coisa. Considere-se a idéia grandiosa de que o nosso Universo e tudo o que nele existe é apenas uma partícula elementar - um elétron, por exemplo - num Cosmos muito maior. Mas, se nunca obtemos informações de fora de nosso Universo, essa idéia não se torna impossível de ser refutada? Devemos poder verificar as afirmativas. Os céticos inveterados devem ter a oportunidade de seguir o nosso raciocínio, copiar os nossos experimentos e ver se chegam ao mesmo resultado.

A confiança em experimentos cuidadosamente planejados e controlados é de suma importância, como tentei enfatizar antes. Não aprenderemos com a simples contemplação. É tentador ficar satisfeitos com a primeira explicação possível que passa pelas nossas cabeças. Uma é muito melhor que nenhuma. Mas o que acontece se podemos inventar várias? Como decidir entre elas? Não decidimos. Deixamos que a experimentação faça as escolhas para nós. Francis Bacon indicou a razão clássica: "A argumentação não é suficiente para a descoberta de novos trabalhos, pois a sutileza da natureza é muitas vezes maior do que a sutileza dos argumentos".

Os experimentos de controle são essenciais. Por exemplo, se alegam que um novo remédio cura uma doença em 20% dos casos, temos de nos assegurar se uma população de controle, ao tomar um placebo pensando que ingere a nova droga, também não experimenta cura espontânea da doença em 20% das vezes.

As variáveis devem ser separadas. Vamos supor que nos sentimos mareados, e nos dão uma pulseira que pressiona os pontos indicados pela acupuntura e cinqüenta miligramas de meclizina. Descobrimos que o mal-estar desaparece. O que causou o alívio - a pulseira ou a pílula? Só ficaremos sabendo se tomarmos uma sem usar a outra, na próxima vez em que ficarmos mareados. Agora vamos imaginar que não somos tão dedicados à ciência a ponto de querer ficar mareados. Nesse caso, não separamos as variáveis. Tomamos os dois remédios de novo. Conseguimos o resultado prático desejado; aprofundar o conhecimento, poderíamos dizer, não vale o desconforto de atingi-lo.

Freqüentemente o experimento deve ser realizado pelo método "duplo cego", para que aqueles que aguardam uma certa descoberta não fiquem na posição potencialmente comprometedora de avaliar os resultados. Ao testar um novo remédio, por exemplo, queremos que os médicos que determinam os sintomas a serem mitigados não fiquem sabendo a que pacientes foi ministrada a nova droga. O conhecimento poderia influenciar a sua decisão, ainda que inconscientemente. Em vez disso, a lista dos que sentiram alívio dos sintomas pode ser comparada com a dos que tomaram a nova droga, cada uma determinada independentemente. Só então podemos estabelecer a correlação existente. Ou, ao comandar uma identificação policial pelo reconhecimento de fotos ou dos suspeitos enfileirados, o oficial encarregado não deveria saber quem é o principal suspeito, para não influenciar a testemunha consciente ou inconscientemente.

Além de nos ensinar o que fazer na hora de avaliar uma afirmação, qualquer bom kit de detecção de mentiras deve também nos ensinar o que não fazer. Ele nos ajuda a reconhecer as falácias mais comuns e mais perigosas da lógica e da retórica. Muitos bons exemplos podem ser encontrados na religião e na política, porque seus profissionais são freqüentemente obrigados a justificar duas proposições contraditórias. Entre essas falácias estão:

. ad hominem - expressão latina que significa "ao homem", quando atacamos o argumentador e não o argumento (por exemplo: A reverenda dra. Smith é uma conhecida fundamentalista bíblica, por isso não precisamos levar a sério suas objeções à evolução);

. argumento de autoridade (por exemplo: O presidente Richard Nixon deve ser reeleito porque ele tem um plano secreto para pôr fim à guerra no Sudeste da Ásia - mas, como era secreto, o eleitorado não tinha meios de avaliar os méritos do plano; o argumento se reduzia a confiar em Nixon porque ele era o presidente: um erro, como se veio a saber);

. argumento das conseqüências adversas (por exemplo: Deve existir um Deus que confere castigo e recompensa, porque, se não existisse, a sociedade seria muito mais desordenada e perigosa talvez até ingovernável *2. Ou: O réu de um caso de homicídio amplamente divulgado pelos meios de comunicação deve ser julgado culpado; do contrário, será um estímulo para os outros homens matarem as suas mulheres);

. apelo à ignorância - a afirmação de que qualquer coisa que não provou ser falsa deve ser verdade, e vice-versa (por exemplo: Não há evidência convincente de que os UFOs não estejam visitando a Terra; portanto, os UFOs existem - e há vida inteligente em outros lugares no Universo. Ou: Talvez haja setenta quasilhões de outros mundos, mas não se conhece nenhum que tenha o progresso moral da Terra, por isso ainda somos o centro do Universo). Essa impaciência com a ambigüidade pode ser criticada pela expressão: a ausência de evidência não é evidência da ausência;

. alegação especial, freqüentemente para salvar uma proposição em profunda dificuldade teórica (por exemplo: Como um Deus misericordioso pode condenar as gerações futuras a um tormento interminável, só porque, contra as suas ordens, uma mulher induziu um homem a comer uma maçã? Alegação especial: Você não compreende a doutrina sutil do livre-arbítrio. Ou: Como pode haver um Pai, um Filho e um Espírito Santo igualmente divinos na mesma Pessoa? Alegação especial: Você não compreende o mistério da Santíssima Trindade. Ou: Como Deus permitiu que os seguidores do judaísmo, cristianismo e islamismo - cada um comprometido a seu modo com medidas heróicas de bondade e compaixão - tenham perpetrado tanta crueldade durante tanto tempo? Alegação especial: Mais uma vez você não compreende o livre-arbítrio. E, de qualquer modo, os movimentos de Deus são misteriosos);

. petição de princípio, também chamada de supor a resposta (por exemplo: Devemos instituir a pena de morte para desencorajar o crime violento. Mas a taxa de crimes violentos realmente cai quando é imposta a pena de morte? Ou: A bolsa de valores caiu ontem por causa de um ajuste técnico e da realização de lucros por parte dos investidores. Mas há alguma evidência independente do papel causal do "ajuste" e da realização de lucros? Aprendemos realmente alguma coisa com essa pretensa explicação?);

. seleção das observações, também chamada de enumeração das circunstâncias favoráveis, ou, segundo a descrição do filósofo Francis Bacon, contar os acertos e esquecer os fracassos *3 (por exemplo: Um Estado se vangloria do presidente que gerou, mas se cala sobre os seus assassinos que matam em série);

. estatística dos números pequenos - falácia aparentada com a seleção das observações (por exemplo: " Dizem que uma dentre cada cinco pessoas é chinesa. Como é possível? Conheço centenas de pessoas, e nenhuma delas é chinesa. Atenciosamente ". Ou: Tirei três setes seguidos. Hoje à noite não tenho como perder).

. compreensão errônea da natureza da estatística (por exemplo: O presidente Dwight Eisenhower expressando espanto e apreensão ao descobrir que metade de todos os norte-americanos tem inteligência abaixo da média);

. incoerência (por exemplo: Prepare-se prudentemente para enfrentar o pior na luta com um potencial adversário militar, mas ignore parcimoniosamente projeções científicas sobre perigos ambientais, porque elas não são "comprovadas". Ou: Atribua a diminuição da expectativa de vida na antiga União Soviética aos fracassos do comunismo há muitos anos, mas nunca atribua a alta taxa de mortalidade infantil nos Estados Unidos (no momento, a taxa mais alta das principais nações industriais) aos fracassos do capitalismo. Ou: Considere razoável que o Universo continue a existir para sempre no futuro, mas julgue absurda a possibilidade de que ele tenha duração infinita no passado);

. non sequitur - expressão latina que significa "não se segue" (por exemplo: A nossa nação prevalecerá, porque Deus é grande. Mas quase todas as nações querem que isso seja verdade; a formulação alemã era "Gott mit uns"). Com freqüência, os que caem na falácia non sequitur deixaram simplesmente de reconhecer as possibilidades alternativas;

. post hoc, ergo propter hoc - expressão latina que significa "aconteceu após um fato, logo foi por ele causado" (por exemplo, Jaime Cardinal Sin, arcebispo de Manila: " Conheço [...] uma moça de 26 anos que aparenta sessenta porque ela toma a pílula [anticoncepcional] ". Ou: Antes de as mulheres terem o direito de votar, não havia armas nucleares);

. pergunta sem sentido (por exemplo: O que acontece quando uma força irresistível encontra um objeto imóvel? Mas se existe uma força irresistível, não pode haver objetos imóveis, e vice-versa);

. exclusão do meio-termo, ou dicotomia falsa - considerando apenas os dois extremos num continuum de possibilidades intermediárias (por exemplo: Claro, tome o partido dele; meu marido é perfeito; eu estou sempre errada. Ou: Ame o seu país ou odeie-o. Ou: Se você não é parte da solução, é parte do problema);

. curto prazo versus longo prazo - um subconjunto da exclusão do meio-termo, mas tão importante que o separei para lhe dar atenção especial (por exemplo: Não temos dinheiro para financiar programas que alimentem crianças mal nutridas e eduquem garotos em idade pré-escolar. Precisamos urgentemente tratar do crime nas ruas. Ou: Por que explorar o espaço ou fazer pesquisa de ciência básica, quando temos tantas pessoas sem teto?);

. declive escorregadio, relacionado à exclusão do meio-termo (por exemplo: Se permitirmos o aborto nas primeiras semanas da gravidez, será impossível evitar o assassinato de um bebê no final da gravidez. Ou, inversamente: Se o Estado proíbe o aborto até no nono mês, logo estará nos dizendo o que fazer com os nossos corpos no momento da concepção);

. confusão de correlação e causa (por exemplo: Um levantamento mostra que é maior o número de homossexuais entre os que têm curso superior do que entre os que não o possuem; portanto, a educação torna as pessoas homossexuais. Ou: Os terremotos andinos estão correlacionados com as maiores aproximações do planeta Urano; portanto - apesar da ausência de uma correlação desse tipo com respeito ao planeta Júpiter, mais próximo e mais volumoso - o planeta Urano é a causa dos terremotos); *4

. espantalho - caricaturar uma posição para tornar mais fácil o ataque (por exemplo: Os cientistas supõem que os seres vivos simplesmente se reuniram por acaso - uma formulação que ignora propositadamente a idéia darwiniana central, de que a natureza se constrói guardando o que funciona e jogando fora o que não funciona. Ou isso é também uma falácia de curto prazo/longo prazo - os ambientalistas se importam mais com anhingas e corujas pintadas do que com gente);

. evidência suprimida, ou meia verdade (por exemplo: Uma "profecia" espantosamente exata e muito citada do atentado contra o presidente Reagan é apresentada na televisão; mas - detalhe importante - foi gravada antes ou depois do evento? Ou: Esses abusos do governo pedem uma revolução, mesmo que não se possa fazer uma omelete sem quebrar alguns ovos. Sim, mas será uma revolução que causará muito mais mortes do que o regime anterior? O que sugere a experiência de outras revoluções? Todas as revoluções contra regimes opressivos são desejáveis e vantajosas para o povo?);

. palavras equívocas (por exemplo, a separação dos poderes na Constituição norte-americana especifica que os Estados Unidos não podem travar guerra sem uma declaração do Congresso. Por outro lado, os presidentes detêm o controle da política externa e o comando das guerras, que são potencialmente ferramentas poderosas para que sejam reeleitos. Portanto, os presidentes de qualquer partido político podem ficar tentados a arrumar disputas, enquanto desfraldam a bandeira e dão outros nomes às guerras - "ações policiais", "incursões armadas", "ataques de reação protetores", "pacificação", "salvaguarda dos interesses norte-americanos" e uma enorme variedade de "operações", como a "Operação da Causa Justa". Os eufemismos para a guerra são um dos itens de uma ampla categoria de reinvenções da linguagem para fins políticos. Talleyrand disse: "Uma arte importante dos políticos é encontrar novos nomes para instituições que com seus nomes antigos se tornaram odiosas para o público").

Conhecer a existência dessas falácias lógicas e retóricas completa o nosso conjunto de ferramentas. Como todos os instrumentos, o kit de detecção de mentiras pode ser mal empregado, aplicado fora do contexto, ou até usado como uma alternativa mecânica para o pensamento. Mas, aplicado judiciosamente, pode fazer toda a diferença do mundo - ao menos para avaliar os nossos próprios argumentos antes de os apresentarmos aos outros.

A indústria do tabaco norte-americana fatura cerca de 50 bilhões de dólares por ano. Há uma correlação estatística entre o fumo e o câncer, admite a indústria do fumo, mas não existe, dizem, uma relação causal. Uma falácia lógica está sendo cometida, é o que afirmam. O que significa tudo isso? Talvez as pessoas com predisposições hereditárias para contrair câncer tenham predisposições hereditárias para drogas que viciam - assim, poderia haver uma correlação entre o câncer e o fumo, mas aquele não seria causado por este. Podem-se inventar conexões desse tipo, cada vez mais forçadas. Essa é exatamente uma das razões por que a ciência insiste em fazer experimentos de controle.

Vamos supor que se pintassem as costas de um grande número de camundongos com alcatrão de cigarro, e que também se observasse à saúde de um número quase idêntico de camundongos que não foram pintados. Se os primeiros contraem câncer e os segundos não, pode-se ter bastante certeza de que a correlação é causal. Trague a fumaça de tabaco, e a chance de contrair câncer aumenta; não trague, e a taxa permanece no nível básico. O mesmo vale para o enfisema, a bronquite e as doenças cardiovasculares.

Quando, em 1953, se publicou a primeira obra na literatura científica mostrando que as substâncias presentes na fumaça do cigarro, quando espargidas nas costas de roedores, produzem tumores malignos, a reação das seis maiores companhias de tabaco foi começar uma campanha de relações públicas para impugnar a pesquisa, patrocinada pela Fundação Sloan Kettering. Uma reação semelhante à da Du Pont Corporation, quando em 1974 foi publicada a primeira pesquisa mostrando que seu produto Freon ataca a camada protetora de ozônio. Há muitos outros exemplos.

É de se pensar que, antes de denunciar descobertas científicas indesejadas, as principais companhias deveriam empregar os seus consideráveis recursos para verificar a segurança dos produtos que se propõem fabricar. E, se perdessem algo, se cientistas independentes sugerissem um perigo, por que as companhias se oporiam? Prefeririam matar pessoas a perder lucros? Se, nesse mundo incerto, um erro precisa ser cometido, ele não deveria ter o objetivo de proteger os clientes e o público? E, por outro lado, o que esses casos revelam sobre a capacidade de o sistema de livre empresa policiar a si mesmo? Não são exemplos em que a interferência do governo é claramente a favor do interesse público?

Um relatório interno da Brown and Williamson Tobacco Corporation, de 1971, lista como objetivo da companhia "afastar das mentes de milhões a falsa convicção de que fumar cigarros causa câncer de pulmão e outras doenças; uma convicção baseada em pressupostos fanáticos, rumores falaciosos, afirmações sem fundamento e declarações não científicas de oportunistas que buscam notoriedade". Eles se queixam do

ataque incrível, sem precedentes e abominável contra o cigarro, constituindo o maior libelo e a maior difamação já perpetrados contra um produto na história da livre empresa; um libelo criminoso de tão grandes proporções e implicações que é de se perguntar como essa cruzada de calúnias pode se acomodar sob a Constituição pode ser tão desrespeitada e violada [sic].

Essa retórica é apenas um pouco mais inflamada do que a das declarações que a indústria de tabaco emite de tempos em tempos para consumo público.

Há muitas marcas de cigarros que anunciam baixo nível de alcatrão (dez miligramas ou menos por cigarro). Por que isso é uma virtude? Porque é no alcatrão refratário que os hidrocarbonetos aromáticos policíclicos e algumas outras substâncias cancerígenas se concentram. As propagandas que enfatizam baixos teores de alcatrão não são uma admissão tácita das companhias de tabaco de que os cigarros realmente causam câncer?

A Healthy Building lnternational é uma organização lucrativa, que recebe há anos milhões de dólares da indústria do fumo. Ela realiza pesquisas sobre fumo passivo, e presta declarações para as companhias de tabaco. Em 1994, três de seus técnicos reclamaram que altos executivos teriam falsificado dados sobre partículas de cigarro inaláveis no ar. Em todos os casos, os dados inventados ou "corrigidos" faziam a fumaça de cigarro parecer mais segura do que as medições dos técnicos haviam indicado. Os departamentos de pesquisa da companhia ou as firmas do ramo contratadas já descobriram alguma vez que um produto é mais perigoso do que a empresa de tabaco declarou publicamente? Em caso positivo, mantiveram o emprego?

O tabaco vicia; segundo muitos critérios, ainda mais do que a heroína e a cocaína. Havia uma razão para as pessoas "caminharem uma milha por um Camel", como diziam os anúncios da década de 40. Já morreram mais pessoas por causa do fumo do que em toda a Segunda Guerra Mundial. Segundo a Organização Mundial de Saúde, o fumo mata 3 milhões de pessoas por ano em todo o mundo. Esse número vai chegar a 10 milhões de mortes por ano em 2020 em parte devido a uma grande campanha publicitária que pinta o tabagismo como um hábito avançado e elegante para as jovens mulheres do mundo em desenvolvimento. É em parte por causa da falta disseminada de conhecimento sobre a detecção de mentiras, o pensamento crítico e o método científico que a indústria de tabaco consegue ser o fornecedor bem-sucedido dessa mistura de venenos que viciam. A credulidade mata.



Notas:

1. Esse é um problema que afeta os júris. Estudos retrospectivos mostram que alguns jurados tomam a sua decisão muito cedo - talvez durante a argumentação de abertura; depois guardam na memória as provas que parecem sustentar suas impressões iniciais e rejeitam as contrárias. O método das hipóteses eficazes alternativas não está em funcionamento nas suas cabeças.

2. Uma formulação mais cínica feita pelo historiador romano Políbio: "Como as massas são inconstantes, presas de desejos rebeldes, apaixonadas e sem temor pelas conseqüências, é preciso incutir-lhes medo para que se mantenham em ordem. Por isso, os antigos fizeram muito bem ao inventar os deuses e a crença no castigo depois da morte".

3. Meu exemplo favorito é a história que se conta sobre o físico italiano Enrico Fermi, recém-chegado às praias norte-americanas, membro do Projeto Manhattan de armas nucleares, e tendo de se defrontar com chefes-de-esquadra norte-americanos no meio da Segunda Guerra Mundial.

- Fulano de tal é um grande general - disseram-lhe.

- Qual é a definição de um grande general? - perguntou Fermi na sua maneira característica.

- Acho que é um general que ganhou muitas batalhas consecutivas.

- Quantas?

Depois de alguma hesitação, decidiram-se por cinco.

- Quantos dos generais norte-americanos são grandes generais?

Depois de mais alguma hesitação, decidiram-se por uma pequena porcentagem. - Mas imaginem - replicou Fermi - que não exista isso que vocês chamam de grande general, que todos os exércitos tenham forças iguais, e que vencer uma batalha seja uma simples questão de sorte. Nesse caso, a probabilidade de vencer uma batalha é de uma em duas, ou 1/2; duas batalhas, 1/4; três, 1/8; quatro, 1/16; e cinco batalhas consecutivas, 1/32 - o que é mais ou menos 3%. Vocês esperam que uma pequena porcentagem dos generais norte-americanos ganhe cinco batalhas consecutivas - por uma simples questão de sorte. Agora, algum deles já ganhou dez batalhas consecutivas... ?

4. Ou: As crianças que assistem a programas violentos na televisão tendem a ser mais violentas na vida adulta. Mas a TV causou a violência, ou crianças violentas preferem assistir a programas violentos? Muito provavelmente, as duas coisas. Os defensores comerciais da violência na TV argumentam que qualquer um sabe distinguir entre a televisão e a realidade. Mas os programas infantis das manhãs de sábado têm hoje em dia uma média de 25 atos de violência por hora. No mínimo, isso toma as crianças insensíveis à agressão e à crueldade gratuita. E, se podemos implantar falsas lembranças nos cérebros de adultos impressionáveis, o que não estamos implantando em nossos filhos, quando os expomos a uns 100 mil atos de violência antes de terminarem a escola primária?






[As partes desta mensagem que não continham texto foram removidas]



SUBJECT: Fw: Passarinhos incomodando
FROM: "Luiz Ferraz Netto" <leobarretos@uol.com.br>
TO: "ciencialist" <ciencialist@yahoogrupos.com.br>
DATE: 05/01/2005 20:49

Não há uma pasta (creme, seja lá o que for) que esparramado no local espanta as pombas? Já ouvi algo sobre isso!

[]'
===========================
Luiz Ferraz Netto [Léo]
leobarretos@uol.com.br
http://www.feiradeciencias.com.br
===========================
-----Mensagem Original-----
De: Guilherme
Para: leobarretos@uol.com.br
Cc: ma-padua@uol.com.br
Enviada em: quarta-feira, 5 de janeiro de 2005 11:03
Assunto: Passarinhos incomodando


Estou c/ problema , na varanda de minha casa, esta sendo dormitorio de pomba e gostaria de tira-las de la. Tem algum aparelho que transmite ondas que incomodam e elas somem do lugar. Para fazer cachorro parar de latir euja ouvi falar. Grata Cida


--------------------------------------------------------------------------------


No virus found in this incoming message.
Checked by AVG Anti-Virus.
Version: 7.0.300 / Virus Database: 265.6.8 - Release Date: 03/01/2005

----------

No virus found in this outgoing message.
Checked by AVG Anti-Virus.
Version: 7.0.300 / Virus Database: 265.6.8 - Release Date: 03/01/2005


[As partes desta mensagem que não continham texto foram removidas]



SUBJECT: RE: [ciencialist] Fw: Passarinhos incomodando
FROM: "murilo filo" <avalanchedrive@hotmail.com>
TO: ciencialist@yahoogrupos.com.br
DATE: 05/01/2005 22:20

Favor aguardar. Sempre vejo estas coisas anunciadas no Estadão.
No caderno Agrícola de hoje há um indicado p/ratos e morcegos... pombos não!
Achando eu mando, se ninguém mais mandar e se meu msn (uma josta) deixar.
abr/M. SP 05/jan/2005

>From: "Luiz Ferraz Netto" <leobarretos@uol.com.br>
>Reply-To: ciencialist@yahoogrupos.com.br
>To: "ciencialist" <ciencialist@yahoogrupos.com.br>
>Subject: [ciencialist] Fw: Passarinhos incomodando
>Date: Wed, 5 Jan 2005 20:49:21 -0200
>
>Não há uma pasta (creme, seja lá o que for) que esparramado no local
>espanta as pombas? Já ouvi algo sobre isso!
>
>[]'
> ===========================
> Luiz Ferraz Netto [Léo]
> leobarretos@uol.com.br
> http://www.feiradeciencias.com.br
> ===========================
>-----Mensagem Original-----
>De: Guilherme
>Para: leobarretos@uol.com.br
>Cc: ma-padua@uol.com.br
>Enviada em: quarta-feira, 5 de janeiro de 2005 11:03
>Assunto: Passarinhos incomodando
>
>
>Estou c/ problema , na varanda de minha casa, esta sendo dormitorio de
>pomba e gostaria de tira-las de la. Tem algum aparelho que transmite ondas
>que incomodam e elas somem do lugar. Para fazer cachorro parar de latir
>euja ouvi falar. Grata Cida
>
>
>--------------------------------------------------------------------------------
>
>
>No virus found in this incoming message.
>Checked by AVG Anti-Virus.
>Version: 7.0.300 / Virus Database: 265.6.8 - Release Date: 03/01/2005
>
> ----------
>
>No virus found in this outgoing message.
>Checked by AVG Anti-Virus.
>Version: 7.0.300 / Virus Database: 265.6.8 - Release Date: 03/01/2005
>
>
>[As partes desta mensagem que não continham texto foram removidas]
>




SUBJECT: RE: [ciencialist] aranhas...
FROM: "murilo filo" <avalanchedrive@hotmail.com>
TO: ciencialist@yahoogrupos.com.br
DATE: 05/01/2005 22:46

Se as teias são ''aéreas'', muito fortes, grandes e de cor meio dourada, e
se esta aranha, com uns 5cm, quando ameaçada, mudar de cor, trata-se a
aranha lôbo, que é boazinha, bonitinha, decorativa e útil.
Se a teia for no chão, meio embolada e aberta como um funil na boca de um
pequeno túnel, trata-se da ''armadeira'', perigosa e famosa nas Discovery da
vida. Dizem que é bem pior que a tal viúva negra. Boa sorte e faça seu
testamento! :] Murilo SP 05/jan/2005

>From: "E m i l i a n o C h e m e l l o" <chemelloe@yahoo.com.br>
>Reply-To: ciencialist@yahoogrupos.com.br
>To: <ciencialist@yahoogrupos.com.br>
>Subject: [ciencialist] aranhas...
>Date: Wed, 5 Jan 2005 10:14:55 -0200
>
>Alguém?
>
>[ ] 's do Emiliano Chemello
>emiliano@quimica.net
>http://www.quimica.net/emiliano
>http://www.ucs.br/ccet/defq/naeq
>
>
>Contato Naeq:
>Nome: Camila
>Email: camila_alves19@hotmail.com
>Telefone: Dúvidas!!
>Mensagem: Há alguns dias eu venho pesquisando sobre aranhas, no meu jardim
>tem 2 aranhas construtoras de teia, elas são sedentárias estão sempre
>paradas, posicionam suas patas dianteiras para cima e suas traseiras para
>baixo fica parecendo que só são 4, suas patas são listradas de amarelo, uma
>listra grande amarela pode ser identificada em seu abdome. Mas a minha
>dúvida é se elas são venenosas. Em certos sites podemos encontrar que as
>aranhas clinicamente perigosas não são construtoras de teias e sim
>errantes,
>mas eu queria saber mais sobre essas aranhas do meu jardim, pois não
>encontro nada em sites, apenas o que tenho são essas características. Se
>puderem me ajudar, estou estudando para fazer faculdade de biologia. Minha
>curiosidade sobre assuntos biologicos é grande.
>Obrigada!
>




SUBJECT: Fazendo contas sem usar números - Fernando Reinach
FROM: José Renato <jrma@terra.com.br>
TO: <ciencialist@yahoogrupos.com.br>
DATE: 05/01/2005 23:10

[ O ato de pensar está tão associado às palavras que utilizamos para expressar o pensamento que alguns pesquisadores acreditam que sem linguagem não existe pensamento. ]

............................

Jornal da Ciência

JC e-mail 2681, de 05 de Janeiro de 2005.

Fazendo contas sem usar números, artigo de Fernando Reinach

Estudos indicam que mesmo sem dispor de uma linguagem numérica o cérebro humano é capaz de efetuar operações matemáticas

Fernando Reinach (fernando.reinach@estadao.com.br) é biólogo. Artigo publicado em 'O Estado de SP':

Você seria capaz de fazer contas se a língua que você fala não possuísse palavras para designar números?

O ato de pensar está tão associado às palavras que utilizamos para expressar o pensamento que alguns pesquisadores acreditam que sem linguagem não existe pensamento.

Outros discordam e acreditam que mesmo animais desprovidos de linguagem são capazes de pensar.

Como não é possível estudar pessoas criadas na ausência de uma língua, a solução é estudar populações que utilizam línguas extremamente simples.

Os índios piraãs que vivem na Amazônia falam uma língua na qual só existem três palavras para designar números. 'Hói' quer dizer um; 'hoí' quer dizer dois e 'aíbaagi' significa muitos.

Já os mundurucus do sul do Pará utilizam palavras para os números um a cinco além da palavra 'ade' (muitos). Dois estudos recentes avaliaram a capacidade de essas pessoas executarem tarefas que dependem de raciocínios matemáticos.

Imagine: você senta na frente de um índio piraã e pede que ele repita o que você faz. Se você desenhar dois riscos na areia, 100% deles são capazes de desenhar os dois riscos. Se você desenhar cinco riscos, só 50% traçam o número correto de riscos. Mais de seis riscos e quase ninguém desenha o número correto.

Agora os índios mundurucus: você mostra seis sementes e as coloca em um frasco. Depois você mostra que retirou quatro sementes do frasco. Sem mostrar quantas sobraram no frasco original você pede a ele que escolha entre três frascos (contendo respectivamente uma, duas ou três sementes) o frasco com um número de sementes igual às que sobraram no frasco original.

Nesse experimento, se o número total de sementes for menor que cinco, os mundurucus escolhem o frasco correto tão bem quanto um francês adulto, mas, se você aumentar o número de sementes, a taxa de acerto diminui rapidamente.

Nas tarefas em que é necessário fazer a conta e obter um resultado preciso, os mundurucus têm dificuldades quando o número de objetos é maior que cinco.

O interessante é que em operações matemáticas que não dependem de resultados precisos, os índios são tão competentes quanto os franceses.

Se a tarefa envolve examinar rapidamente dois conjuntos de sementes (de até 80 sementes) e decidir qual conjunto contém mais sementes, os índios acertam com a mesma freqüência que os franceses, indicando que para essas tarefas o cérebro não necessita da linguagem para fazer o raciocínio.

Esses estudos indicam que mesmo sem dispor de uma linguagem numérica o cérebro humano é capaz de efetuar operações matemáticas. Uma linguagem numérica sofisticada parece ser necessária para o cérebro executar operações aritméticas precisas.

Isso sugere que a matemática utiliza habilidades quantitativas que estavam presentes no cérebro humano muito antes de o homem desenvolver uma linguagem sofisticada.

Mais informações na revista Science, volume 306, págs. 496 e 499, 2004.

(O Estado de SP, 5/1)

< http://www.jornaldaciencia.org.br/Detalhe.jsp?id=24504 >

[As partes desta mensagem que não continham texto foram removidas]



SUBJECT: Re: Fazendo contas sem usar números - Fernando Reinach
FROM: "rmtakata" <rmtakata@altavista.net>
TO: ciencialist@yahoogrupos.com.br
DATE: 06/01/2005 00:26


--- Em ciencialist@yahoogrupos.com.br, José Renato <jrma@t...> >
> [ O ato de pensar está tão associado às palavras que
> utilizamos para expressar o pensamento que alguns
> pesquisadores acreditam que sem linguagem não existe
> pensamento. ]

Quase tao eloquente qto isso eh a aparente capacidade de diversos
animais de realizar operacoes matematicas como a contagem.

[]s,

Roberto Takata





SUBJECT: FEIRA DE CIÊNCIAS ... O Imperdível ! sobre o levitron
FROM: "Cyberlander" <cybernews@superig.com.br>
TO: <Undisclosed-Recipient:;>
DATE: 06/01/2005 01:56


<URL: http://www.feiradeciencias.com.br/sala13/13_36.asp >


[ ]'s
D.C.


[As partes desta mensagem que não continham texto foram removidas]



SUBJECT: Sobre Pilhas
FROM: marcelo ferrari <emailferrari@yahoo.com.br>
TO: ciencialist@yahoogrupos.com.br
DATE: 06/01/2005 05:36

Se alguém souber e puder me explicar:


1) Por que a pilha recarregavel é recarregavel e a tradicional não é? Qual é a diferença e como funciona esta diferença?


Grato.
Marcelo Ferrari.

__________________________________________________
Converse com seus amigos em tempo real com o Yahoo! Messenger
http://br.download.yahoo.com/messenger/

[As partes desta mensagem que não continham texto foram removidas]



SUBJECT: Re: [ciencialist] Fw: Passarinhos incomodando
FROM: "Luiz Ferraz Netto" <leobarretos@uol.com.br>
TO: <ciencialist@yahoogrupos.com.br>
DATE: 06/01/2005 08:35

Morcegos? Fiquei interessado. No meu rancho há morcegos entre o telhado e o forro ... uma jostra! Que faço para sumir com eles dali? Chamo o Batman? A Batgirl?
Agradeço a informação.
[]'

===========================
Luiz Ferraz Netto [Léo]
leobarretos@uol.com.br
http://www.feiradeciencias.com.br
===========================
-----Mensagem Original-----
De: "murilo filo" <avalanchedrive@hotmail.com>
Para: <ciencialist@yahoogrupos.com.br>
Enviada em: quarta-feira, 5 de janeiro de 2005 22:20
Assunto: RE: [ciencialist] Fw: Passarinhos incomodando



Favor aguardar. Sempre vejo estas coisas anunciadas no Estadão.
No caderno Agrícola de hoje há um indicado p/ratos e morcegos... pombos não!
Achando eu mando, se ninguém mais mandar e se meu msn (uma josta) deixar.
abr/M. SP 05/jan/2005

>From: "Luiz Ferraz Netto" <leobarretos@uol.com.br>
>Reply-To: ciencialist@yahoogrupos.com.br
>To: "ciencialist" <ciencialist@yahoogrupos.com.br>
>Subject: [ciencialist] Fw: Passarinhos incomodando
>Date: Wed, 5 Jan 2005 20:49:21 -0200
>
>Não há uma pasta (creme, seja lá o que for) que esparramado no local
>espanta as pombas? Já ouvi algo sobre isso!
>
>[]'
> ===========================
> Luiz Ferraz Netto [Léo]
> leobarretos@uol.com.br
> http://www.feiradeciencias.com.br
> ===========================
>-----Mensagem Original-----
>De: Guilherme
>Para: leobarretos@uol.com.br
>Cc: ma-padua@uol.com.br
>Enviada em: quarta-feira, 5 de janeiro de 2005 11:03
>Assunto: Passarinhos incomodando
>
>
>Estou c/ problema , na varanda de minha casa, esta sendo dormitorio de
>pomba e gostaria de tira-las de la. Tem algum aparelho que transmite ondas
>que incomodam e elas somem do lugar. Para fazer cachorro parar de latir
>euja ouvi falar. Grata Cida
>
>
>--------------------------------------------------------------------------------
>
>
>No virus found in this incoming message.
>Checked by AVG Anti-Virus.
>Version: 7.0.300 / Virus Database: 265.6.8 - Release Date: 03/01/2005
>
> ----------
>
>No virus found in this outgoing message.
>Checked by AVG Anti-Virus.
>Version: 7.0.300 / Virus Database: 265.6.8 - Release Date: 03/01/2005
>
>
>[As partes desta mensagem que não continham texto foram removidas]
>




##### ##### #####

Para saber mais visite
http://www.ciencialist.hpg.ig.com.br


##### ##### ##### #####
Links do Yahoo! Grupos










--
No virus found in this incoming message.
Checked by AVG Anti-Virus.
Version: 7.0.300 / Virus Database: 265.6.8 - Release Date: 03/01/2005




--
No virus found in this outgoing message.
Checked by AVG Anti-Virus.
Version: 7.0.300 / Virus Database: 265.6.8 - Release Date: 03/01/2005



SUBJECT: Re: Sobre Pilhas
FROM: "tipoalgo" <tipoalgo@bol.com.br>
TO: ciencialist@yahoogrupos.com.br
DATE: 06/01/2005 09:12


Olá Marcelo,

Eu não vou simplificar o que pode ser bem mais instrutivo.

Vá aos seguintes links e você entenderá o motivo.

http://www.feiradeciencias.com.br/sala21/21_12_06.asp
http://myspace.eng.br/eng/bat1.asp
http://www.espacociencia.pe.gov.br/areas/quimica/tipos.php
http://www.abancar.com.br/literaturabaterias1.htm
http://www.coltec.ufmg.br/alunos/210/baterias/estrut3.htm
http://www.sta-eletronica.com.br/reciclagem.htm
http://paginas.fe.up.pt/~cdm/QAE/documentos/PILHAS_II.pdf

Boa leitura!

Tipoalgo

--- Em ciencialist@yahoogrupos.com.br, marcelo ferrari
<emailferrari@y...> escreveu
> Se alguém souber e puder me explicar:
>
>
> 1) Por que a pilha recarregavel é recarregavel e a tradicional não
é? Qual é a diferença e como funciona esta diferença?
>
>
> Grato.
> Marcelo Ferrari.






SUBJECT: Re: Sobre Pilhas
FROM: marcelo ferrari <emailferrari@yahoo.com.br>
TO: ciencialist@yahoogrupos.com.br
DATE: 06/01/2005 09:16

Tipoalgo, muito obrigado pela dica.


marcelo ferrari




tipoalgo <tipoalgo@bol.com.br> wrote:

Olá Marcelo,

Eu não vou simplificar o que pode ser bem mais instrutivo.

Vá aos seguintes links e você entenderá o motivo.

http://www.feiradeciencias.com.br/sala21/21_12_06.asp
http://myspace.eng.br/eng/bat1.asp
http://www.espacociencia.pe.gov.br/areas/quimica/tipos.php
http://www.abancar.com.br/literaturabaterias1.htm
http://www.coltec.ufmg.br/alunos/210/baterias/estrut3.htm
http://www.sta-eletronica.com.br/reciclagem.htm
http://paginas.fe.up.pt/~cdm/QAE/documentos/PILHAS_II.pdf

Boa leitura!

Tipoalgo

--- Em ciencialist@yahoogrupos.com.br, marcelo ferrari
<emailferrari@y...> escreveu
> Se alguém souber e puder me explicar:
>
>
> 1) Por que a pilha recarregavel é recarregavel e a tradicional não
é? Qual é a diferença e como funciona esta diferença?
>
>
> Grato.
> Marcelo Ferrari.






##### ##### #####

Para saber mais visite
http://www.ciencialist.hpg.ig.com.br


##### ##### ##### #####


Yahoo! Grupos, um serviço oferecido por:



















function SearchComboBox() { if (document.form_combo.keyword.value.length==0){ alert("Por favor, digite algo."); return false; }else { document.form_combo.action ="http://br.rd.yahoo.com/SIG=12a69bsle/M=264105.3931087.6562589.1588051/D=brclubs/S=2137111528:HM/EXP=1105096390/A=2361264/R=0/SIG=11uaou2jn/*http://www.bondfaro.com/bondfaro/in/combosearch_in.jsp?sk=11"; } return true;} [input] [input] [input]

---------------------------------
Links do Yahoo! Grupos

Para visitar o site do seu grupo na web, acesse:
http://br.groups.yahoo.com/group/ciencialist/

Para sair deste grupo, envie um e-mail para:
ciencialist-unsubscribe@yahoogrupos.com.br

O uso que você faz do Yahoo! Grupos está sujeito aos Termos do Serviço do Yahoo!.



---------------------------------
Yahoo! Acesso Grátis - Internet rápida e grátis. Instale o discador do Yahoo! agora.

[As partes desta mensagem que não continham texto foram removidas]



SUBJECT: Re: [ciencialist] Sobre Pilhas
FROM: "Luiz Ferraz Netto" <leobarretos@uol.com.br>
TO: <ciencialist@yahoogrupos.com.br>
DATE: 06/01/2005 09:27

Existem reações químicas reversíveis e irreversíveis. As reversíveis são aquelas que participam das pilhas recarregáveis (NiCd, por exemplo) e as irreversíveis nas demais (pilha comum de lanterna).
Será interessante ver as reações parciais que ocorrem numa bateria comum automotiva tanto na fase de carga como de descarga; do mesmo modo é interessante ver o interior e a 'casca' de zinco de uma pilha comum de lanterna para sentir o efeito da irreversibilidade da reação.
Reações irreversíveis são as mais fáceis de serem obtidas (dai a grande proliferação dos tipos de pilhas na fase áurea das pilhas comuns); um exemplo disso é a pilha necessária para 'tocar' um pequeno radinho portátil em situação de emergência.
Eis uma pilha de acampamento: Dois espetos de materiais diferentes (ou um espeto e um garfo) espetados no chão a 5 cm um do outro, e cada um com um fio levado ao rádio; faça xixi no chão entre eles ... e o rádio começa a funcionar! Isso tb serve para funcionar o relógio digital!
Detalhe: para controlar o ganho do radinho ('volume', com diz a galera) basta enfiar mais ou menos o espeto no chão.

[]'
===========================
Luiz Ferraz Netto [Léo]
leobarretos@uol.com.br
http://www.feiradeciencias.com.br
===========================
-----Mensagem Original-----
De: "marcelo ferrari" <emailferrari@yahoo.com.br>
Para: <ciencialist@yahoogrupos.com.br>
Enviada em: quinta-feira, 6 de janeiro de 2005 05:36
Assunto: [ciencialist] Sobre Pilhas



Se alguém souber e puder me explicar:


1) Por que a pilha recarregavel é recarregavel e a tradicional não é? Qual é a diferença e como funciona esta diferença?


Grato.
Marcelo Ferrari.

__________________________________________________
Converse com seus amigos em tempo real com o Yahoo! Messenger
http://br.download.yahoo.com/messenger/

[As partes desta mensagem que não continham texto foram removidas]



##### ##### #####

Para saber mais visite
http://www.ciencialist.hpg.ig.com.br


##### ##### ##### #####
Links do Yahoo! Grupos










--
No virus found in this incoming message.
Checked by AVG Anti-Virus.
Version: 7.0.300 / Virus Database: 265.6.8 - Release Date: 03/01/2005




--
No virus found in this outgoing message.
Checked by AVG Anti-Virus.
Version: 7.0.300 / Virus Database: 265.6.8 - Release Date: 03/01/2005



SUBJECT: Re: polar e apolar
FROM: "tipoalgo" <tipoalgo@bol.com.br>
TO: ciencialist@yahoogrupos.com.br
DATE: 06/01/2005 10:08


Olá Ronaldo,

Hoje resolvi estudar um pouco.

Como ninguém comentou algo sobre a sua pergunta, resolvi pesquisar o
assunto.

Em http://www.cdcc.sc.usp.br/quimica/experimentos/teor.html encontrei
uma boa introdução à sua boa pergunta.

Agora em
http://www.ucs.br/ccet/defq/naeq/material_didatico/textos_interativos_
33.htm a resposta está quase completa.

Acho que é por ai, mas ainda há mais a desvendar.

Abraços

Tipoalgo


--- Em ciencialist@yahoogrupos.com.br, "E m i l i a n o C h e m e
l l o" <chemelloe@y...> escreveu
> alguém com paciência para explicar a 'boa' pergunta que o moço fez?
>
> [ ] 's
> Emiliano
> ---
> De:"Ronaldo Stadtlander" <rlander@p...>
> Para:Emiliano Chemello
> Assunto:Substancias polares e apolares
> Mensagem:Ola,
> Em visita ao site www.ucs.br referente a material didático, pude
ler sobre
> pontes
> de hidrogenio e soluções com características polares e apolares.
Como não
> sou da
> área química, gostaria de tirar algumas dúvidas caso fosse possivel.
>
> A água é uma substância polar e mistura-se facilmente com outras
substâncias
> orgânicas também polares tais como alcoois e seus semelhantes. O
alcool por
> sua vez
> se mistura a gasolina embora a água não tenha esta mesma
carcterística. o
> alcool
> tem a capacidade de ser polar e apolar? Como se explica o fato de um
> composto ser
> polar e apolar ao mesmo tempo? Como se dá a mistura da gasolina com
o
> alcool?
>
> Desde já agradeço.
> Ronaldo Stadtlander.





SUBJECT: Re: polar e apolar
FROM: "tipoalgo" <tipoalgo@bol.com.br>
TO: ciencialist@yahoogrupos.com.br
DATE: 06/01/2005 11:09


Olá a Ronaldo e a todos,

Só depois de enviar a mensagem percebi que o segundo link que citei
(www.ucs.br/ccet/defq/naeq/material_didatico/textos_interativos_33.htm
)é provavelmente o mesmo que você leu e que o autor é o
próprio químico que remeteu as suas dúvidas aqui para lista.

Era melhor eu ter estudado mais o assunto antes de chover no molhado!

Mas de qualquer forma a intenção foi nobre!

Com o devido pedido de desculpas aos moderadores.

Abraços

Tipoalgo





SUBJECT: Re: Sobre Pilhas
FROM: marcelo ferrari <emailferrari@yahoo.com.br>
TO: ciencialist@yahoogrupos.com.br
DATE: 06/01/2005 11:39

Li o que me foi indicado, xixi a parte, ficou faltando entender o seguinte:

> Porque algumas reações são reversiveis e outras não? O que significa reverter uma reação? Como a eletricidade do recarregador reverte a reação na pilha recarregável?



Grato.
Marcelo Ferrari.





Luiz Ferraz Netto <leobarretos@uol.com.br> wrote:Existem reações químicas reversíveis e irreversíveis. As reversíveis são aquelas que participam das pilhas recarregáveis (NiCd, por exemplo) e as irreversíveis nas demais (pilha comum de lanterna).
Será interessante ver as reações parciais que ocorrem numa bateria comum automotiva tanto na fase de carga como de descarga; do mesmo modo é interessante ver o interior e a 'casca' de zinco de uma pilha comum de lanterna para sentir o efeito da irreversibilidade da reação.
Reações irreversíveis são as mais fáceis de serem obtidas (dai a grande proliferação dos tipos de pilhas na fase áurea das pilhas comuns); um exemplo disso é a pilha necessária para 'tocar' um pequeno radinho portátil em situação de emergência.
Eis uma pilha de acampamento: Dois espetos de materiais diferentes (ou um espeto e um garfo) espetados no chão a 5 cm um do outro, e cada um com um fio levado ao rádio; faça xixi no chão entre eles ... e o rádio começa a funcionar! Isso tb serve para funcionar o relógio digital!
Detalhe: para controlar o ganho do radinho ('volume', com diz a galera) basta enfiar mais ou menos o espeto no chão.

[]'
===========================
Luiz Ferraz Netto [Léo]
leobarretos@uol.com.br
http://www.feiradeciencias.com.br
===========================
-----Mensagem Original-----
De: "marcelo ferrari" <emailferrari@yahoo.com.br>
Para: <ciencialist@yahoogrupos.com.br>
Enviada em: quinta-feira, 6 de janeiro de 2005 05:36
Assunto: [ciencialist] Sobre Pilhas



Se alguém souber e puder me explicar:


1) Por que a pilha recarregavel é recarregavel e a tradicional não é? Qual é a diferença e como funciona esta diferença?


Grato.
Marcelo Ferrari.

__________________________________________________
Converse com seus amigos em tempo real com o Yahoo! Messenger
http://br.download.yahoo.com/messenger/

[As partes desta mensagem que não continham texto foram removidas]



##### ##### #####

Para saber mais visite
http://www.ciencialist.hpg.ig.com.br


##### ##### ##### #####
Links do Yahoo! Grupos










--
No virus found in this incoming message.
Checked by AVG Anti-Virus.
Version: 7.0.300 / Virus Database: 265.6.8 - Release Date: 03/01/2005




--
No virus found in this outgoing message.
Checked by AVG Anti-Virus.
Version: 7.0.300 / Virus Database: 265.6.8 - Release Date: 03/01/2005



##### ##### #####

Para saber mais visite
http://www.ciencialist.hpg.ig.com.br


##### ##### ##### #####


Yahoo! Grupos, um serviço oferecido por:PUBLICIDADE


---------------------------------
Links do Yahoo! Grupos

Para visitar o site do seu grupo na web, acesse:
http://br.groups.yahoo.com/group/ciencialist/

Para sair deste grupo, envie um e-mail para:
ciencialist-unsubscribe@yahoogrupos.com.br

O uso que você faz do Yahoo! Grupos está sujeito aos Termos do Serviço do Yahoo!.



__________________________________________________
Converse com seus amigos em tempo real com o Yahoo! Messenger
http://br.download.yahoo.com/messenger/

[As partes desta mensagem que não continham texto foram removidas]



SUBJECT: RE: [ciencialist] aranhas...
FROM: Erich Weick <erich_weick@yahoo.com.br>
TO: ciencialist@yahoogrupos.com.br
DATE: 06/01/2005 12:18

> Dizem que é bem pior que a tal viúva negra. Boa sorte e faça seu
> testamento! :] Murilo SP 05/jan/2005


Realmente é. Um professor de Biologia que eu tive no ensino médio(um dos raros com
pós-graduação) comentou certa vez sobre a "hierarquia" da periculosidade das aranhas onde
enumerou nesta sequência: Armadeira, Tarântula, Viúva Negra e Aranha Marrom como sendo as
mais perigosas encontradas no MUNDO! (Corrijam-me se eu estiver errado...)

A Armadeira é facilmente encontrada no Brasil, pelo menos aqui no sul, é bastante
agressiva e o princípio ativo do seu veneno é muito forte e bastante doloroso, motivo
óbvio para se ter mesmo muito cuidado com ela. Eu próprio já me deparei com uma dentro de
casa... elas são bem ágeis e difíceis de se atingir com a primeira chinelada... rs

Abraços.


Erich






_______________________________________________________
Yahoo! Acesso Grátis - Instale o discador do Yahoo! agora. http://br.acesso.yahoo.com/ - Internet rápida e grátis


SUBJECT: Analise de Carl Sagan por mim...
FROM: "Amauri Jr" <amaurijunior2@yahoo.com.br>
TO: "Leone" <webleones@sapo.pt>, Lênia Luz Nogueira <lenialuz@yahoo.com.br>, "Reginaldo" <reginaldo.pereira.li@itelefonica.com.br>, "Serginho" <serginho_st@hotmail.com>, <Sexualidadedodef@yahoogrupos.com.br>, "Lucimary Vargas" <sangalli@uai.com.br>, "Marcia Mystery" <mare_mania@hotmail.com>, "Carolina Marques" <marquesacc@hotmail.com>, "Laracna" <laracna@pop.com.br>, "kathleen_mel" <kathleen_mel@uol.com.br>, <Conversa_de_Botequim@yahoogrupos.com.br>, <ciencialist@yahoogrupos.com.br>, <acropolis@yahoogrupos.com.br>
DATE: 06/01/2005 16:45



Oi



Eu vou analisar Sagan um pouquinho...:)









�A compreens�o humana n�o � um exame desinteressado, mas recebe infus�es da vontade e dos afetos; disso se originam ci�ncias que podem ser chamadas "ci�ncias conforme a nossa vontade". Pois um homem acredita mais facilmente no que gostaria que fosse verdade. Assim, ele rejeita coisas dif�ceis pela impaci�ncia de pesquisar; coisas sensatas, porque diminuem a esperan�a; as coisas mais profundas da natureza, por supersti��o; a luz da experi�ncia, por arrog�ncia e orgulho; coisas que n�o s�o comumente aceitas, por defer�ncia � opini�o do vulgo. Em suma, in�meras s�o as maneiras, e �s vezes impercept�veis, pelas quais os afetos colorem e contaminam o entendimento.�



Francis Bacon, Novum organon (1620)



[A] Acho que Bacon foi um pouco ou extremamente pretensioso, ainda reverteu os papeis...:) N�o � a cren�a que quer ter raz�o em tudo e sim a ci�ncia, insisto que a ci�ncia tem muito �ngulos...:) Alem que Bacon � um saco de ler, eu tenho e tentei ler, mas � muito chato...:)





�Meus pais morreram h� anos. Eu era muito ligado a eles. Ainda sinto uma saudade terr�vel. Sei que sempre sentirei. Desejo acreditar que sua ess�ncia, suas personalidades, o que eu tanto amava neles, ainda existe - real e verdadeiramente - em algum lugar. N�o pediria muito, apenas cinco ou dez minutos por ano, para lhes contar sobre os netos, p�-las ao corrente das �ltimas novidades, lembrar-lhes que eu os amo. Uma parte minha - por mais infantil que pare�a - se pergunta como � que estar�o. "Est� tudo bem?", desejo perguntar. As �ltimas palavras que me vi dizendo a meu pai, na hora de sua morte, foram: "Tome cuidado".�



[A] Nossa Carl Sagan � cientista ou um pastor? Eu ao ler esse trecho eu ri e lembrei do Edir Macedo quando ele fala, parece que ele diz por tr�s dessas linhas: �A ci�ncia vai salvar a humanidade e vamos tirar o dem�nio chamado religi�o�, um papo muito religioso e cheios de preconceitos; mas como todo norte americano, fascistinha demais...:)



��s vezes sonho que estou falando com meus pais, e de repente - ainda imerso na elabora��o do sonho - sou tomado pela consci�ncia esmagadora de que eles n�o morreram de verdade, de que tudo n�o passou de um erro horr�vel. Ora, ali est�o eles, vivos e bem de sa�de, meu pai fazendo piadas inteligentes, minha m�e muito s�ria me aconselhando a usar uma manta porque est� frio. Quando acordo, passo de novo por um processo abreviado de luto. Evidentemente, existe algo dentro de mim que est� pronto a acreditar na vida ap�s a morte. E que n�o est� nem um pouco interessado em saber se h� alguma evid�ncia s�ria que confirme tal coisa.�



[A] Vamos l�, esse sinal que tanto queremos esta dentro de nosso ser, muitos acreditam que a eternidade vem junto com nossas c�lulas, gen�tica, ou que quando morremos nossos �tomos se espalham e quando d�o formas a outras vidas. Mas ca�mos na real, n�o devemos esquecer, que se a ci�ncia quiser ter m�rito tem que olhar e analisar tudo, Carl Sagan prega tal ci�ncia, ele quer mostrar que n�o existe e cada vez mais escreve no modo psicol�gico que se enrola mais...:)





�Por isso, n�o rio da mulher que visita o t�mulo do marido e conversa com ele de vez em quando, talvez no anivers�rio de sua morte. N�o � dif�cil de compreender. E se tenho dificuldades com o status ontol�gico daquele com que ela est� falando, n�o faz mal. N�o � isso que importa. O que importa � que os seres humanos s�o humanos. Mais de um ter�o dos adultos norte-americanos acreditam que em algum n�vel estabeleceram contato com os mortos. O n�mero parece ter dado um pulo de 15% entre 1977 e 1988. Um quarto dos norte-americanos acredita em reencarna��o.�



[A] No fundo ri sim, a �verdade� para Sagan � sagrada. Neste trecho eu vejo a mulher simplesmente falando com o marido, qual o problema? Nenhuma mulher vai l� criticar o cientista que ta falando com uma baleia...:)





�Mas isso n�o significa que estou disposto a aceitar as pretens�es de um "m�dium", que afirma canalizar os esp�ritos dos seres amados que partiram, quando tenho consci�ncia de que a pr�tica est� cheia de fraudes. Sei o quanto desejo acreditar que meus pais s� abandonaram os cascos de seus corpos, como insetos ou cobras na muda, e partiram para outro lugar. Compreendo que esses sentimentos poderiam me tornar uma presa f�cil at� de um trapaceiro pouco inteligente, de pessoas normais que desconhecem suas mentes inconscientes, ou dos que sofrem de uma desordem psiqui�trica dissociativa. Relutantemente, ponho em a��o algumas reservas de ceticismo.�



[A] Xiiii! L� vai a ci�ncia falar dos m�diuns, j� Jung tentou e falhou porque n�o teve nem argumentos e nem base de pesquisa, nem para come�ar. Agora eu protesto que gente que acredita que o homem chegou a Lua que tudo isso seja obra do inconsciente, sendo que, o inconsciente n�o se manifesta quando esta consciente porque sen�o n�o haveria um inconsciente para bloquear. Portanto, essa teoria � refut�vel...:)





�Como �, pergunto a mim mesmo, que os canalizadores nunca nos d�o informa��es verific�veis que nos s�o inacess�veis por outros meios? Por que Alexandre, o Grande, nunca nos informa sobre a localiza��o exata de sua tumba, Fermat sobre o seu �ltimo teorema, James Wilkes Booth sobre a conspira��o do assassinato de Lincoln, Hermann Goering sobre o inc�ndio do Reichstag? Por que S�focles, Dem�crito e Aristarco n�o ditam as suas obras perdidas? N�o querem que as gera��es futuras conhe�am as suas obras-primas?�



[A] Para que Sagan quer saber isso tudo? Para o ser humano transformar em patrim�nios e poucos poderem ver? Alexandre, o Grande provavelmente esta onde morreu, Fermat n�o pode falar sobre coisas que n�o se pode ser reveladas; coisas que voc�s das ci�ncias ainda ignoram, mas tudo bem, pode ser que um dia seja reveladas. Mas Lincoln sonhou, foi um aviso que ignorou, mesmo que fosse permitido o sujeito a falar o que seria valido saber? Sobre o inc�ndio idem, tem coisas que voc�s indagam sem pretens�o de solucionar e sim s� pelo gosto de criticar. Os fil�sofos e escritores fizeram sua parte e seus trabalhos foram completamente destru�dos, foi o pr�prio fundamentalismo que levou a destrui��o, um raio n�o pode cair duas vezes no mesmo ponto...:)



�Se fosse anunciada alguma evid�ncia real de vida ap�s a morte, desejaria muito examin�-la; mas teria de ser uma evid�ncia real cient�fica, e n�o simples anedota. Em casos como A Face em Marte e os raptos por alien�genas, eu diria que � melhor a verdade dura do que a fantasia consoladora. E, no c�mputo final, revela-se freq�entemente que os fatos s�o mais consoladores que a fantasia.�





[A] Que fantasia? Ah! Fantasias consol�veis iguais da lua, a supera��o humana desafiando o mundo das id�ias...:)





�A premissa fundamental da "canaliza��o", do espiritismo e de outras formas de necromancia � que n�o morremos quando experimentamos a morte. N�o exatamente. Continua a existir alguma parte de n�s que pensa, sente e tem mem�ria. Seja o que for - alma ou esp�rito, nem mat�ria nem energia, mas alguma outra coisa -, essa parte pode entrar novamente em corpos humanos ou de outros seres, e assim a morte perde grande parte da sua ferroada. E ainda mais: se as afirma��es do esp�rita ou canalizador s�o verdadeiras, temos uma oportunidade de entrar em contato com os seres amados que morreram.�



[A] �, mas eles n�o entram no corpo, s� se ocupam da mente humana e nada mais...:) Outra coisa, n�o se pratica necromancia nesses casos, pois n�o revivem o corpo e sim o esp�rito fala por vontade pr�pria...:)



�J. Z. Knight, do estado de Washington, afirma estar em contato com um ser de 35 mil anos chamado Ramtha. Ele fala ingl�s muito bem, usando a l�ngua, os l�bios e as cordas vocais de Knight, com um sotaque que me parece ser hindu. Como a maioria das pessoas sabe como falar, e muitas - de crian�as a atores profissionais - t�m um repert�rio de vozes a seu dispor, a hip�tese mais simples sugere que � a pr�pria sra. Knight que faz Ramtha falar, e que ela n�o tem contato com entidades desencarnadas da �poca plistocena glacial. Se h� provas em contr�rio, gostaria muito de conhecer. Seria consideravelmente mais impressionante se Ramtha pudesse falar por si mesmo, sem a ajuda da boca da sra. Knight. Isso n�o sendo poss�vel, como podemos testar a afirma��o? (A atriz Shirley MacLaine afirma que Ramtha foi seu irm�o em Atl�ntida, mas isso j� � outra hist�ria.)�



[A] N�o sei quem cobra � verdadeiro, pode ser que sim ou n�o, para desmascarar porque n�o pergunta a localiza��o de Atl�ntida...:)



�Vamos supor que Ramtha pudesse ser interrogado. Poder�amos verificar se ele � quem afirma ser? Como � que ele sabe que viveu h� 35 mil anos, mesmo aproximadamente? Que calend�rio emprega? Quem est� tomando nota dos mil�nios intermedi�rios? Trinta e cinco mil mais ou menos o qu�? Como � que eram as coisas h� 35 mil anos? Ou Ramtha tem realmente essa idade, e nesse caso vamos descobrir alguma coisa sobre esse per�odo, ou � uma fraude e ele (ou melhor, ela) vai se trair.�



[A] Em outras dimens�es pode ter outra propor��o, ou pode se contar de outro jeito, pode ser que ele se saia bem, como disse, a localiza��o de Atl�ntida era melhor...:)



�Onde � que Ramtha vivia? (Sei que fala ingl�s com sotaque hindu, mas onde � que falavam assim h� 35 mil anos?) Como era o clima? O que Ramtha comia? (Os arque�logos t�m alguma no��o do que as pessoas comiam nessa �poca.) Quais eram as l�nguas aut�ctones, e qual era a estrutura social? Com quem mais Ramtha vivia - com a mulher, mulheres, filhos, netos? Qual era o ciclo da vida, a taxa de mortalidade infantil, a expectativa de vida? Eles tinham controle populacional? Que roupas vestiam? Como elas eram fabricadas? Quais os predadores mais perigosos? Os instrumentos e as estrat�gias da ca�a e da pesca? Armas? Sexismo end�mico? Xenofobia e etnocentrismo? E, se Ramtha descendia da "elevada civiliza��o" de Atl�ntida, onde est�o os detalhes ling��sticos, tecnol�gicos, hist�ricos e de outra natureza? Como era a sua escrita? Respondam. Em lugar disso, a �nica coisa que recebemos s�o homilias banais.�



[A] Deixa o cara...ele teve 1050 vivencias como vai lembrar? :) Por outro lado, j� disse que a humanidade faz muito mal uso desses artefatos, tanto, que j� logo v�o ao tesouro de Atl�ntida...:)





�Para dar outro exemplo, eis um conjunto de informa��es que n�o foram canalizadas de um morto antigo, mas de entidades n�o humanas desconhecidas que fazem c�rculos nas planta��es, assim como foi registrado pelo jornalista Jim Schnabel:



"Estamos muito ansiosos por essa na��o pecadora estar espalhando mentiras sobre n�s. N�o viemos em m�quinas, n�o pousamos na Terra em m�quinas [...]. Viemos como o vento. Somos a For�a Vital. A For�a Vital do solo [...]. Viemos at� aqui [...]. Estamos apenas a um sopro de dist�ncia [...] a um sopro de dist�ncia [...] n�o estamos a milh�es de milhas de dist�ncia [...] uma For�a Vital que � mais potente que as energias no corpo humano. Mas n�s nos reunimos num n�vel mais elevado de vida [...]. N�o precisamos de nome. Vivemos num mundo paralelo ao seu, ao lado do seu [...]. Os muros se romperam. Dois homens surgir�o do passado [...] o grande urso [...] o mundo encontrar� a paz".



[A] Porra! Isso � entidade de �ndio...:) Mas falando serio, se ler muitas teorias qu�nticas diz que temos outras dimens�es, ou, seria entidades de extraterrestres que se comunicam com m�diuns tamb�m sem estarem mortos, s�o fen�menos desconhecidos...:)



�As pessoas d�o aten��o a essas maravilhas pueris, principalmente porque elas prometem algo parecido com a religi�o dos velhos tempos, mas sobretudo a vida depois da morte, at� a vida eterna.�



[A] U�, cad� a evolu��o de Dawing? Isso vale para o universo inteiro que evolui...:) portanto nada � eterno, tudo muda, o que ainda o ser humano n�o entendeu...:)



�O vers�til cientista brit�nico J.B.S. Haldane, que foi, entre muitas outras coisas, um dos fundadores da gen�tica populacional, prop�s certa vez uma perspectiva muito diferente para algo semelhante � vida eterna. Haldane imaginava um futuro distante em que as estrelas se obscureceram e o espa�o foi preenchido em sua maior parte por um g�s frio e fino. Ainda assim, se esperarmos bastante tempo, ocorrer�o flutua��es estat�sticas na densidade desse g�s. Ao longo de imensos per�odos, as flutua��es ser�o o suficiente para reconstituir um Universo parecido com o nosso. Se o Universo � infinitamente antigo, haver� um n�mero infinito dessas reconstitui��es, apontava Haldane.�



[A] Bidu!! Se for verdade? :)



�Assim, num Universo infinitamente antigo com um n�mero infinito de nascimentos de gal�xias, estrelas, planetas e vida, deve reaparecer uma Terra id�ntica em que voc� e todos os seus seres queridos voltar�o a se reunir. Serei capaz de rever meus pais e apresentar-lhes os netos que eles n�o conheceram. E tudo isso n�o acontecer� apenas uma vez, mas um n�mero infinito de vezes.�



[A] Poxa, vendo por esse prisma, n�s existimos antes e existiremos depois e assim sucessivamente...:) Inclusive vai ter outro Carl Sagan que vai escrever as mesmas besteiras, ele podia incorporar um m�dium e vim discutir comigo essas perolas americanas (pearl american)...:)





�Entretanto, de certo modo isso n�o oferece os consolos da religi�o. Se nenhum de n�s vai lembrar o que aconteceu desta vez, a �poca que o leitor e eu estamos partilhando, as satisfa��es da ressurrei��o do corpo, pelo menos aos meus ouvidos, soam ocas.�



[A] Ocas como sua cabe�a n� Carl...:) O consolo humano vem do �cash� se n�o tiver �cash� ent�o � uma teoria refutada, tanto que Marx � muito mais respeitado do que Sartre, o ser humano quer tudo no imediato, se n�o tiver, vai sofrer sempre...:)



�Mas nessa reflex�o subestimei o que significa infinidade. Na imagem de Haldane, haver� universos, na verdade um n�mero infinito de universos, em que nossas mentes recordar�o perfeitamente todas as vidas anteriores. A satisfa��o est� � m�o - moderada, no entanto, pela id�ia de todos esses outros universos que tamb�m passar�o a existir (novamente, n�o uma vez, mas um n�mero infinito de vezes) com trag�dias e horrores que superam em muito qualquer coisa que j� experimentei desta vez.�



[A] Nossa, isso me parece muito a teoria evolutiva espiritual, a qu�ntica j� em sua infinita sabedoria, proposto tal teoria, isso para ele antigamente era absurdo, para n�s hoje h� uma possibilidade...:)



�Entretanto, o Consolo de Haldane depende do tipo de universo em que vivemos, e talvez de arcanos, como, por exemplo, saber se h� bastante mat�ria para finalmente reverter � expans�o do universo, e o car�ter das flutua��es no v�cuo. Ao que parece, aqueles que sentem um profundo desejo de vida ap�s a morte poderiam se dedicar � cosmologia, � gravidade qu�ntica, � f�sica das part�culas elementares e � aritm�tica trans-finita.�



[A] San Carl Sagan in EUA...:) Em algumas �reas do espiritismo, existem que estuda essas coisas, tanto que � um estudo serio onde tem que haver uma l�gica seria sen�o vai ser rejeitada, como toda �rea de estudo, h� quem concorde ou n�o, mas existem sim quem estude...:)



�Clemente de Alexandria, um dos padres da Igreja primitiva, em suas Exorta��es aos gregos (escritas em torno do ano 190), rejeitava as cren�as pag�s em termos que pareceriam hoje em dia um pouco ir�nicos:



"Estamos realmente longe de permitir que os homens adultos d�em ouvidos a essas hist�rias. Mesmo aos nossos filhos, quando eles berram de cortar o cora��o, como se diz, n�o temos o h�bito de contar hist�rias fabulosas para acalm�-los".




[A] Exorta��o em que??



�Em nossa �poca, temos padr�es menos severos. Contamos �s crian�as hist�rias sobre Papai Noel, o coelhinho da P�scoa e a fada do dente por raz�es que achamos emocionalmente sadias, mas depois, antes de crescerem, n�s os desiludimos sobre esses mitos. Por que nos desdizemos? Porque o seu bem-estar como adultos depende de eles conhecerem o mundo tal como �. N�s nos preocupamos, e com raz�o, com os adultos que ainda acreditam em Papai Noel.�



[A] um adulto que faz isso com uma crian�a tem que levar uma no ouvidor...:) N�s temos medo da verdade, temos medo de que o Papai Noel vire verdade, pois � coisa de crian�a. De vez em quando, temos que ser um pouco, mas falando cientificamente; isso faz o homem desejar a vida, desejar que um dia vai ver o Papai Noel, nossas c�lulas v�o viver muito mais produtivas...:)



�Sobre as religi�es doutrin�rias, escreveu o fil�sofo David Hume que "os homens n�o ousam confessar, nem mesmo a seus cora��es, as d�vidas que t�m a respeito desses assuntos. Eles valorizam a f� impl�cita; e disfar�am para si mesmos a sua real descren�a, por meio das afirma��es mais convictas e do fanatismo mais positivo".



[A] N�o vale para hoje, Hume foi filho de seu tempo e sim, antigamente pode escrever tal palavras; veja, tudo consiste em ter a cren�a, todos cr�em em algo, seja num deus ou numa verdade. N�o acreditam que o homem pisou na lua? Pois ent�o, isso � cren�a que o homem de que algo aconteceu que ele n�o tem possibilidade de comprovar...mesmo teorias infinitas que vimos por ai...:)



�Essa descren�a tem conseq��ncias morais profundas, como escreveu o revolucion�rio americano Tom Paine em The age of reason: "A descren�a n�o consiste em acreditar, nem em desacreditar; consiste em professar que se cr� naquilo que n�o se cr�. � imposs�vel calcular o dano moral, se � que posso cham�-lo assim, que a mentira mental tem causado na sociedade. Quando o homem corrompeu e prostituiu de tal modo a castidade de sua mente, a ponto de empenhar a sua cren�a profissional em coisas que n�o acredita, ele est� preparado para a execu��o de qualquer outro crime".



[A] Crime � americano que quer saber mais do que todo mundo, a cren�a como disse, n�o � apenas a religi�o e sim tudo que os leigos n�o tem o poder da comprova��o...:)



� A formula��o de T.H. Huxley foi: "O fundamento da moralidade � [...] renunciar a fingir que se acredita naquilo que n�o comporta evid�ncias, e a repetir proposi��es inintelig�veis sobre coisas que est�o al�m das possibilidades do conhecimento".

[A] Sim, a banaliza��o � uma coisinha...:) Mas ent�o renunciaremos o Admir�vel Mundo Novo e simplesmente, vamos esquecer veemente as Portas da Percep��o, pois n�o devemos �fingir acreditar naquilo que n�o comporta evidencias�...:)



�Clement, Hume, Paine e Huxley estavam todos falando de religi�o. Mas grande parte do que escreveram tem aplica��es mais gerais - por exemplo, para as importunidades disseminadas no pano de fundo de nossa civiliza��o comercial: h� um tipo de comercial de aspirina em que atores fingindo ser m�dicos revelam que o produto do concorrente tem apenas determinada fra��o do ingrediente analg�sico que os m�dicos mais recomendam - eles n�o dizem qual � o misterioso ingrediente. Enquanto o seu produto tem uma quantidade drasticamente maior (1,2 a duas vezes mais por comprimido). Por isso, comprem esse produto. Mas por que n�o tomar dois comprimidos do concorrente? Ou considere-se o caso do analg�sico que funciona melhor do que o produto de "pot�ncia regular" do concorrente. Por que n�o tomar o produto de "pot�ncia extra" do outro fabricante? E eles certamente n�o falam nada sobre as mais de mil mortes por ano causadas pelo uso da aspirina nos Estados Unidos ou os aparentes 5 mil casos anuais de disfun��o renal provocados pelo uso de acetaminofeno, de que a marca mais vendida � o Tylenol. (Isso, contudo, talvez represente um caso de correla��o sem causalidade.) Ou quem se importa em saber quais os cereais que t�m mais vitamina, quando podemos tomar uma p�lula de vitamina no caf� da manh�? Da mesma forma, que importa saber que um anti�cido cont�m c�lcio, se o c�lcio serve para a nutri��o e � irrelevante para a gastrite? A cultura comercial est� cheia de informa��es err�neas e subterf�gios semelhantes � custa do consumidor. N�o se devem fazer perguntas. N�o pensem. Comprem.�



[A] puxa, realmente isto ocorre, mas tudo � �cash� e tudo acabar� em �cash�...:) Eu fico pensando que todo mundo, se isto � verdade, compre esses rem�dios porque mostram nos comerciais, muitos s�o receitados pelos m�dicos...:) Que ainda s�o receitados...:)



�As explica��es pagas de produtos, especialmente se feitas por verdadeiros ou pretensos especialistas, constituem uma saraivada constante de logros. Revelam menosprezo pela intelig�ncia dos clientes. Criam uma corrup��o insidiosa das atitudes populares a respeito da objetividade cient�fica. Hoje, existem at� comerciais em que cientistas reais, alguns de consider�vel distin��o, atuam como garotos-propaganda para as empresas. Eles nos ensinam que tamb�m os cientistas mentem por dinheiro. Como alertou Tom Paine, o fato de nos acostumarmos com mentiras cria o fundamento para muitos outros males.�



[A] � sim, mas acho que d�s do come�o da renascen�a o objetivo era esse, j� que os que financiavam eram os burgueses...:)



�Enquanto escrevo, tenho diante de mim o programa da Whole Life Expo, a exposi��o anual da Nova Era realizada em San Francisco. � comumente visitada por dezenas de milhares de pessoas. Ali especialistas muito question�veis fazem propaganda de produtos muito question�veis. Eis algumas das apresenta��es: "Como prote�nas presas no sangue produzem dor e sofrimento". "Cristais, talism�s ou pedras?" (Tenho a minha opini�o.) Prossegue: "Assim como um cristal focaliza as ondas sonoras e luminosas para o r�dio e a televis�o" - o que � um erro ins�pido de quem n�o compreende como o r�dio e a televis�o funcionam -, "ele pode amplificar as vibra��es espirituais para o ser humano afinado". Ou mais esta: "O retorno da deusa, um ritual de apresenta��o". Outra: "Sincronismo, a experi�ncia do reconhecimento". Essa � fornecida pelo "irm�o Charles". Ou, na p�gina seguinte: "Voc�, Saint-Germain e a cura pela chama violeta". E assim continua, com milhares de an�ncios sobre as "oportunidades" - percorrendo a gama estreita que vai do d�bio ao esp�rio - que se acham � disposi��o na Whole Life Expo.�



[A] Seria igual: �o homem em 1969 pisa na Lua e Armstrong diz ser um grande passo para humanidade� ou �sonda espacial chega em Marte, a NASA realiza grande feito�, os chamados �New Age� s�o inofensivos perto na NASA, veja, eu n�o vejo diferen�a de um cristal que acende com o funcionamento da TV ou radio do que as descobertas da ci�ncia, sei que ir�o falar que eu sou leigo, mas s� no fato que os EUA querem mostrar ainda que s�o poderosos fico em duvida. S�o iguarias de gente que queria ter a liberdade, ter o pensamento muito mais profundo do que o poder queria que pensassem, hoje devem ser senhores com a bunda presa em suas poltronas reclamando da vida, vendo futebol e tomando cerveja...:) O povo s� quer um consolo...:)



�Algumas v�timas de c�ncer, perturbadas, fazem peregrina��es �s Filipinas, onde "cirurgi�es medi�nicos", depois de esconder na palma da m�o peda�os de f�gado de galinha ou cora��o de bode, fingem tocar nas entranhas do paciente e retirar o tecido doente, que � ent�o triunfantemente exibido. Certos l�deres de democracias ocidentais consultam regularmente astr�logos e m�sticos antes de tomar decis�es de Estado. Sob a press�o p�blica por resultados, a pol�cia, �s voltas com um assassinato n�o solucionado ou um corpo desaparecido, consulta "especialistas" de ESP (percep��o extra-sensorial) (que nunca adivinham nada al�m do esperado pelo senso comum, mas a pol�cia, dizem os ESPs, continua a cham�-los). Anuncia-se a previs�o de uma diverg�ncia com na��es advers�rias, e a CIA, estimulada pelo Congresso, gasta dinheiro dos impostos para descobrir se podemos localizar submarinos nas profundezas do oceano concentrando o pensamento neles. Um "m�dium" - usando p�ndulos sobre mapas e varinhas rabdom�nticas em avi�es - finge descobrir novos dep�sitos minerais; uma companhia mineira australiana lhe adianta elevada soma de d�lares, irrecuper�vel em caso de fracasso, garantindo-lhe uma participa��o na explora��o do min�rio em caso de sucesso. Nada � descoberto. Algumas est�tuas de Jesus ou murais de Maria ficam manchados de umidade, e milhares de pessoas bondosas se convencem de que testemunharam um milagre.�



[A] Que linda � a ci�ncia, primeiro pensa que os cara jogam fora o dinheiro financiando tal coisa, que com certeza ser� empregado na mesma, certo? Eita joguinho de interesse!...:) Eu tenho uma pergunta: o paciente com c�ncer desesperado que vai morrer, desesperado que a mesma ci�ncia que critica que ela foi no cirurgi�o espiritual que a mesma n�o deu resposta, algum cientista j� fez pesquisa sobre ou vai ficar s� na fal�cia? A R�ssia no tempo da URSS gastou milh�es com pesquisa para pesquisar paranormalidade e descobriu muita coisa...como sempre � s� papo da ci�ncia que vejo...:)



�Todos esses s�o casos de mentiras provadas ou presum�veis. Acontece um logro, ora de forma inocente, mas com a colabora��o dos envolvidos, ora com premedita��o c�nica. Em geral, a v�tima se v� presa de forte emo��o - admira��o, medo, gan�ncia, dor. A aceita��o cr�dula da mentira talvez nos custe dinheiro; � o que P.T. Barnum apontou, ao afirmar: "Nasce um ot�rio a cada minuto". Mas pode ser muito mais perigoso que isso, e quando os governos e as sociedades perdem a capacidade de pensar criticamente os resultados podem ser catastr�ficos - por mais que deploremos aqueles que engoliram a mentira.�



[A] Realmente, vamos pensar que milh�es de mentira se formam nesse tempo, em toda a historia humana: mentiras est�o em todo lugar em toda hora, na TV se conta uma a cada minutinho...:) Mas como disse P.T. Barnun afirmou: �Nasce um ot�rio a cada minuto�, os mesmo que acreditam nesta papagaiada da Lua, da mesma �papagaiada� que somos um conjunto de c�lulas apenas, da mesma �papagaiada� que foi necess�rio jogar a bomba at�mica no Jap�o, nas mesmas palavras bonitas que um dia iremos tornar um mundo melhor e mais bonito gra�as a ci�ncia e seus correligion�rios. E n�o venha com essa que a ci�ncia n�o quer ser a verdade que quer, muito mais, quer que o ser humano se curve a ela; sendo uma obriga��o desta curar doen�as e melhorar a humanidade, n�o mandar foguetes a Lua ou pesquisar pelos em baratas...:)



�Na ci�ncia, podemos come�ar com resultados experimentais, dados, observa��es, medi��es, "fatos". Inventamos, se poss�vel, um rico conjunto de explica��es plaus�veis e sistematicamente confrontamos cada explica��o com os fatos. Ao longo de seu treinamento, os cientistas s�o equipados com um kit de detec��o de mentiras. Este � ativado sempre que novas id�ias s�o apresentadas para considera��o. Se a nova id�ia sobrevive ao exame das ferramentas do kit, n�s lhe concedemos aceita��o calorosa, ainda que experimental. Se possu�mos essa tend�ncia, se n�o desejamos engolir mentiras mesmo quando s�o confortadoras, h� precau��es que podem ser tomadas; existe um m�todo testado pelo consumidor, experimentado e verdadeiro.�



[A] Vamos ver a palha�ada...:)



O que existe no kit ? Ferramentas para o pensamento c�tico.

O pensamento c�tico se resume no meio de construir e compreender um argumento racional e - o que � especialmente importante - de reconhecer um argumento falacioso ou fraudulento. A quest�o n�o � se gostamos da conclus�o que emerge de uma cadeia de racioc�nio, mas se a conclus�o deriva da premissa ou do ponto de partida e se essa premissa � verdadeira.

Eis algumas das ferramentas:


. Sempre que poss�vel, deve haver confirma��o independente dos "fatos".



. Devemos estimular um debate substantivo sobre as evid�ncias, do qual participar�o not�rios partid�rios de todos os pontos de vista.



. Os argumentos de autoridade t�m pouca import�ncia - as "autoridades" cometeram erros no passado. Voltar�o a comet�-los no futuro. Uma forma melhor de expressar essa id�ia � talvez dizer que na ci�ncia n�o existem autoridades; quando muito, h� especialistas.



. Devemos considerar mais de uma hip�tese. Se alguma coisa deve ser explicada, � preciso pensar em todas as maneiras diferentes pelas quais poderia ser explicada. Depois devemos pensar nos testes que poderiam servir para invalidar sistematicamente cada uma das alternativas. O que sobreviver, a hip�tese que resistir a todas as refuta��es nessa sele��o darwiniana entre as "m�ltiplas hip�teses eficazes", tem uma chance muito melhor de ser a resposta correta do que se tiv�ssemos simplesmente adotado a primeira id�ia que prendeu nossa imagina��o *1.



. Devemos tentar n�o ficar demasiado ligados a uma hip�tese, s� por ser a nossa. � apenas uma esta��o intermedi�ria na busca do conhecimento. Devemos nos perguntar por que a id�ia nos agrada. Devemos compar�-la imparcialmente com as alternativas. Devemos verificar se � poss�vel encontrar raz�es para rejeit�-la. Se n�o, outros o far�o.

. Devemos quantificar. Se o que estiver sendo explicado � pass�vel de medi��o, de ser relacionado a alguma quantidade num�rica, seremos muito mais capazes de discriminar entre as hip�teses concorrentes. O que � vago e qualitativo � suscet�vel de muitas explica��es. H� certamente verdades a serem buscadas nas muitas quest�es qualitativas que somos obrigados a enfrentar, mas encontr�-las � mais desafiador.

. Se h� uma cadeia de argumentos, todos os elos na cadeia devem funcionar (inclusive a premissa) - e n�o apenas a maioria deles.



. A Navalha de Occam. Essa maneira pr�tica e conveniente de proceder nos incita a escolher a mais simples dentre duas hip�teses que explicam os dados com igual efici�ncia.



. Devemos sempre perguntar se a hip�tese pode ser, pelo menos em princ�pio, falseada. As proposi��es que n�o podem ser testadas ou falseadas n�o valem grande coisa. Considere-se a id�ia grandiosa de que o nosso Universo e tudo o que nele existe � apenas uma part�cula elementar - um el�tron, por exemplo - num Cosmos muito maior. Mas, se nunca obtemos informa��es de fora de nosso Universo, essa id�ia n�o se torna imposs�vel de ser refutada? Devemos poder verificar as afirmativas. Os c�ticos inveterados devem ter a oportunidade de seguir o nosso racioc�nio, copiar os nossos experimentos e ver se chegam ao mesmo resultado.



[A] Algum cientista leu isto?...:)



�A confian�a em experimentos cuidadosamente planejados e controlados � de suma import�ncia, como tentei enfatizar antes. N�o aprenderemos com a simples contempla��o. � tentador ficar satisfeitos com a primeira explica��o poss�vel que passa pelas nossas cabe�as. Uma � muito melhor que nenhuma. Mas o que acontece se podemos inventar v�rias? Como decidir entre elas? N�o decidimos. Deixamos que a experimenta��o fa�a as escolhas para n�s. Francis Bacon indicou a raz�o cl�ssica: "A argumenta��o n�o � suficiente para a descoberta de novos trabalhos, pois a sutileza da natureza � muitas vezes maior do que a sutileza dos argumentos".�



[A] Bacon tinha raz�o, a sutileza da natureza e seus tsinamis � fant�stica...:)Mas o que interessa � a ci�ncia fazer seu papel e nada mais de bla bla bla...mas descobri onde que Hitler tirou as experi�ncias com o judeus ou onde os soldados Americanos forem testado na radioatividade de Bacon, a ci�ncia � fant�stica...:)



�Os experimentos de controle s�o essenciais. Por exemplo, se alegam que um novo rem�dio cura uma doen�a em 20% dos casos, temos de nos assegurar se uma popula��o de controle, ao tomar um placebo pensando que ingere a nova droga, tamb�m n�o experimenta cura espont�nea da doen�a em 20% das vezes.�



[A] Como fazem na �frica...:)



�As vari�veis devem ser separadas. Vamos supor que nos sentimos mareados, e nos d�o uma pulseira que pressiona os pontos indicados pela acupuntura e cinq�enta miligramas de meclizina. Descobrimos que o mal-estar desaparece. O que causou o al�vio - a pulseira ou a p�lula? S� ficaremos sabendo se tomarmos uma sem usar a outra, na pr�xima vez em que ficarmos mareados. Agora vamos imaginar que n�o somos t�o dedicados � ci�ncia a ponto de querer ficar mareados. Nesse caso, n�o separamos as vari�veis. Tomamos os dois rem�dios de novo. Conseguimos o resultado pr�tico desejado; aprofundar o conhecimento poder�amos dizer, n�o vale o desconforto de atingi-lo.�



[A] A ci�ncia me faz rir...:) Veja s�, o cara deseja me convencer que uma ci�ncia oriental de cinco mil anos aproximadamente, n�o tem resultados sendo pesquisada hoje pelas universidades exclusive nas brasileiras...:) Abra a mente e me expliquem, Sagan era serio?...:)



�Freq�entemente o experimento deve ser realizado pelo m�todo "duplo cego", para que aqueles que aguardam uma certa descoberta n�o fiquem na posi��o potencialmente comprometedora de avaliar os resultados. Ao testar um novo rem�dio, por exemplo, queremos que os m�dicos que determinam os sintomas a serem mitigados n�o fiquem sabendo a que pacientes foi ministrada a nova droga. O conhecimento poderia influenciar a sua decis�o, ainda que inconscientemente. Em vez disso, a lista dos que sentiram al�vio dos sintomas pode ser comparada com a dos que tomaram a nova droga, cada uma determinada independentemente. S� ent�o podemos estabelecer a correla��o existente. Ou, ao comandar uma identifica��o policial pelo reconhecimento de fotos ou dos suspeitos enfileirados, o oficial encarregado n�o deveria saber quem � o principal suspeito, para n�o influenciar a testemunha consciente ou inconscientemente.�



[A] Se morrerem que se danem?...:) Se o medico errar deva ser preso, ao identificar testemunha hoje temos DNA...:)



�Al�m de nos ensinar o que fazer na hora de avaliar uma afirma��o, qualquer bom kit de detec��o de mentiras deve tamb�m nos ensinar o que n�o fazer. Ele nos ajuda a reconhecer as fal�cias mais comuns e mais perigosas da l�gica e da ret�rica. Muitos bons exemplos podem ser encontrados na religi�o e na pol�tica, porque seus profissionais s�o freq�entemente obrigados a justificar duas proposi��es contradit�rias. Entre essas fal�cias est�o:

. ad hominem - express�o latina que significa "ao homem", quando atacamos o argumentador e n�o o argumento (por exemplo: A reverenda dra. Smith � uma conhecida fundamentalista b�blica, por isso n�o precisamos levar a s�rio suas obje��es � evolu��o);

. argumento de autoridade (por exemplo: O presidente Richard Nixon deve ser reeleito porque ele tem um plano secreto para p�r fim � guerra no Sudeste da �sia - mas, como era secreto, o eleitorado n�o tinha meios de avaliar os m�ritos do plano; o argumento se reduzia a confiar em Nixon porque ele era o presidente: um erro, como se veio a saber);

. argumento das conseq��ncias adversas (por exemplo: Deve existir um Deus que confere castigo e recompensa, porque, se n�o existisse, a sociedade seria muito mais desordenada e perigosa talvez at� ingovern�vel *2. Ou: O r�u de um caso de homic�dio amplamente divulgado pelos meios de comunica��o deve ser julgado culpado; do contr�rio, ser� um est�mulo para os outros homens matarem as suas mulheres);

. apelo � ignor�ncia - a afirma��o de que qualquer coisa que n�o provou ser falsa deve ser verdade, e vice-versa (por exemplo: N�o h� evid�ncia convincente de que os UFOs n�o estejam visitando a Terra; portanto, os UFOs existem - e h� vida inteligente em outros lugares no Universo. Ou: Talvez haja setenta quasilh�es de outros mundos, mas n�o se conhece nenhum que tenha o progresso moral da Terra, por isso ainda somos o centro do Universo). Essa impaci�ncia com a ambig�idade pode ser criticada pela express�o: a aus�ncia de evid�ncia n�o � evid�ncia da aus�ncia;

. alega��o especial, freq�entemente para salvar uma proposi��o em profunda dificuldade te�rica (por exemplo: Como um Deus misericordioso pode condenar as gera��es futuras a um tormento intermin�vel, s� porque, contra as suas ordens, uma mulher induziu um homem a comer uma ma��? Alega��o especial: Voc� n�o compreende a doutrina sutil do livre-arb�trio. Ou: Como pode haver um Pai, um Filho e um Esp�rito Santo igualmente divinos na mesma Pessoa? Alega��o especial: Voc� n�o compreende o mist�rio da Sant�ssima Trindade. Ou: Como Deus permitiu que os seguidores do juda�smo, cristianismo e islamismo - cada um comprometido a seu modo com medidas her�icas de bondade e compaix�o - tenham perpetrado tanta crueldade durante tanto tempo? Alega��o especial: Mais uma vez voc� n�o compreende o livre-arb�trio. E, de qualquer modo, os movimentos de Deus s�o misteriosos);

. peti��o de princ�pio, tamb�m chamada de supor a resposta (por exemplo: Devemos instituir a pena de morte para desencorajar o crime violento. Mas a taxa de crimes violentos realmente cai quando � imposta a pena de morte? Ou: A bolsa de valores caiu ontem por causa de um ajuste t�cnico e da realiza��o de lucros por parte dos investidores. Mas h� alguma evid�ncia independente do papel causal do "ajuste" e da realiza��o de lucros? Aprendemos realmente alguma coisa com essa pretensa explica��o?);

. sele��o das observa��es, tamb�m chamada de enumera��o das circunst�ncias favor�veis, ou, segundo a descri��o do fil�sofo Francis Bacon, contar os acertos e esquecer os fracassos *3 (por exemplo: Um Estado se vangloria do presidente que gerou, mas se cala sobre os seus assassinos que matam em s�rie);

. estat�stica dos n�meros pequenos - fal�cia aparentada com a sele��o das observa��es (por exemplo: " Dizem que uma dentre cada cinco pessoas � chinesa. Como � poss�vel? Conhe�o centenas de pessoas, e nenhuma delas � chinesa. Atenciosamente ". Ou: Tirei tr�s setes seguidos. Hoje � noite n�o tenho como perder).

. compreens�o err�nea da natureza da estat�stica (por exemplo: O presidente Dwight Eisenhower expressando espanto e apreens�o ao descobrir que metade de todos os norte-americanos tem intelig�ncia abaixo da m�dia);

. incoer�ncia (por exemplo: Prepare-se prudentemente para enfrentar o pior na luta com um potencial advers�rio militar, mas ignore parcimoniosamente proje��es cient�ficas sobre perigos ambientais, porque elas n�o s�o "comprovadas". Ou: Atribua a diminui��o da expectativa de vida na antiga Uni�o Sovi�tica aos fracassos do comunismo h� muitos anos, mas nunca atribua a alta taxa de mortalidade infantil nos Estados Unidos (no momento, a taxa mais alta das principais na��es industriais) aos fracassos do capitalismo. Ou: Considere razo�vel que o Universo continue a existir para sempre no futuro, mas julgue absurda a possibilidade de que ele tenha dura��o infinita no passado);

. non sequitur - express�o latina que significa "n�o se segue" (por exemplo: A nossa na��o prevalecer�, porque Deus � grande. Mas quase todas as na��es querem que isso seja verdade; a formula��o alem� era "Gott mit uns"). Com freq��ncia, os que caem na fal�cia non sequitur deixaram simplesmente de reconhecer as possibilidades alternativas;

. post hoc, ergo propter hoc - express�o latina que significa "aconteceu ap�s um fato, logo foi por ele causado" (por exemplo, Jaime Cardinal Sin, arcebispo de Manila: " Conhe�o [...] uma mo�a de 26 anos que aparenta sessenta porque ela toma a p�lula [anticoncepcional] ". Ou: Antes de as mulheres terem o direito de votar, n�o havia armas nucleares);

. pergunta sem sentido (por exemplo: O que acontece quando uma for�a irresist�vel encontra um objeto im�vel? Mas se existe uma for�a irresist�vel, n�o pode haver objetos im�veis, e vice-versa);

. exclus�o do meio-termo, ou dicotomia falsa - considerando apenas os dois extremos num continuum de possibilidades intermedi�rias (por exemplo: Claro, tome o partido dele; meu marido � perfeito; eu estou sempre errada. Ou: Ame o seu pa�s ou odeie-o. Ou: Se voc� n�o � parte da solu��o, � parte do problema);

. curto prazo versus longo prazo - um subconjunto da exclus�o do meio-termo, mas t�o importante que o separei para lhe dar aten��o especial (por exemplo: N�o temos dinheiro para financiar programas que alimentem crian�as mal nutridas e eduquem garotos em idade pr�-escolar. Precisamos urgentemente tratar do crime nas ruas. Ou: Por que explorar o espa�o ou fazer pesquisa de ci�ncia b�sica, quando temos tantas pessoas sem teto?);

. declive escorregadio, relacionado � exclus�o do meio-termo (por exemplo: Se permitirmos o aborto nas primeiras semanas da gravidez, ser� imposs�vel evitar o assassinato de um beb� no final da gravidez. Ou, inversamente: Se o Estado pro�be o aborto at� no nono m�s, logo estar� nos dizendo o que fazer com os nossos corpos no momento da concep��o);

. confus�o de correla��o e causa (por exemplo: Um levantamento mostra que � maior o n�mero de homossexuais entre os que t�m curso superior do que entre os que n�o o possuem; portanto, a educa��o torna as pessoas homossexuais. Ou: Os terremotos andinos est�o correlacionados com as maiores aproxima��es do planeta Urano; portanto - apesar da aus�ncia de uma correla��o desse tipo com respeito ao planeta J�piter, mais pr�ximo e mais volumoso - o planeta Urano � a causa dos terremotos); *4

. espantalho - caricaturar uma posi��o para tornar mais f�cil o ataque (por exemplo: Os cientistas sup�em que os seres vivos simplesmente se reuniram por acaso - uma formula��o que ignora propositadamente a id�ia darwiniana central, de que a natureza se constr�i guardando o que funciona e jogando fora o que n�o funciona. Ou isso � tamb�m uma fal�cia de curto prazo/longo prazo - os ambientalistas se importam mais com anhingas e corujas pintadas do que com gente);

. evid�ncia suprimida, ou meia verdade (por exemplo: Uma "profecia" espantosamente exata e muito citada do atentado contra o presidente Reagan � apresentada na televis�o; mas - detalhe importante - foi gravada antes ou depois do evento? Ou: Esses abusos do governo pedem uma revolu��o, mesmo que n�o se possa fazer uma omelete sem quebrar alguns ovos. Sim, mas ser� uma revolu��o que causar� muito mais mortes do que o regime anterior? O que sugere a experi�ncia de outras revolu��es? Todas as revolu��es contra regimes opressivos s�o desej�veis e vantajosas para o povo?);

. palavras equ�vocas (por exemplo, a separa��o dos poderes na Constitui��o norte-americana especifica que os Estados Unidos n�o podem travar guerra sem uma declara��o do Congresso. Por outro lado, os presidentes det�m o controle da pol�tica externa e o comando das guerras, que s�o potencialmente ferramentas poderosas para que sejam reeleitos. Portanto, os presidentes de qualquer partido pol�tico podem ficar tentados a arrumar disputas, enquanto desfraldam a bandeira e d�o outros nomes �s guerras - "a��es policiais", "incurs�es armadas", "ataques de rea��o protetores", "pacifica��o", "salvaguarda dos interesses norte-americanos" e uma enorme variedade de "opera��es", como a "Opera��o da Causa Justa". Os eufemismos para a guerra s�o um dos itens de uma ampla categoria de reinven��es da linguagem para fins pol�ticos. Talleyrand disse: "Uma arte importante dos pol�ticos � encontrar novos nomes para institui��es que com seus nomes antigos se tornaram odiosas para o p�blico").�



[A] Sabe o que acontece? O ser humano � um ser muito irracional, tem seus preconceito e ter� no futuro, ele inventa tudo para argumentar coisas in�cuas e absurdas; ai Sagan tem raz�o, porem comete erros grotescos, pois s�o coisas enraizado que o ser humano n�o deixara de uma hora para outra...:) Todas suas afirma��es s�o corretas, s�o explic�veis, mas Sagan n�o � o senhor da verdade absoluta, um dia ele vai ser superado e o que ganha infelizmente � a propaganda, a propaganda que come�a com o nazismo e at� hoje esta em evidencia...:) Pr�ximo...:)





�Conhecer a exist�ncia dessas fal�cias l�gicas e ret�ricas completa o nosso conjunto de ferramentas. Como todos os instrumentos, o kit de detec��o de mentiras pode ser mal empregado, aplicado fora do contexto, ou at� usado como uma alternativa mec�nica para o pensamento. Mas, aplicado judiciosamente, pode fazer toda a diferen�a do mundo - ao menos para avaliar os nossos pr�prios argumentos antes de os apresentarmos aos outros.�



[A] E viva a Am�rica!





�A ind�stria do tabaco norte-americana fatura cerca de 50 bilh�es de d�lares por ano. H� uma correla��o estat�stica entre o fumo e o c�ncer, admite a ind�stria do fumo, mas n�o existe, dizem, uma rela��o causal. Uma fal�cia l�gica est� sendo cometida, � o que afirmam. O que significa tudo isso? Talvez as pessoas com predisposi��es heredit�rias para contrair c�ncer tenham predisposi��es heredit�rias para drogas que viciam - assim, poderia haver uma correla��o entre o c�ncer e o fumo, mas aquele n�o seria causado por este. Podem-se inventar conex�es desse tipo, cada vez mais for�adas. Essa � exatamente uma das raz�es por que a ci�ncia insiste em fazer experimentos de controle.�





[A] hoje sabemos que o tabaco causa c�ncer e o fator drogas pode ser heredit�rio mesmo, mas n�o controlamos e nem a ci�ncia ao menos conseguiu...:)



�Vamos supor que se pintassem as costas de um grande n�mero de camundongos com alcatr�o de cigarro, e que tamb�m se observasse � sa�de de um n�mero quase id�ntico de camundongos que n�o foram pintados. Se os primeiros contraem c�ncer e os segundos n�o, pode-se ter bastante certeza de que a correla��o � causal. Trague a fuma�a de tabaco, e a chance de contrair c�ncer aumenta; n�o trague, e a taxa permanece no n�vel b�sico. O mesmo vale para o enfisema, a bronquite e as doen�as cardiovasculares.�



[A] Esse texto � velho...:)





�Quando, em 1953, se publicou a primeira obra na literatura cient�fica mostrando que as subst�ncias presentes na fuma�a do cigarro, quando espargidas nas costas de roedores, produzem tumores malignos, a rea��o das seis maiores companhias de tabaco foi come�ar uma campanha de rela��es p�blicas para impugnar a pesquisa, patrocinada pela Funda��o Sloan Kettering. Uma rea��o semelhante � da Du Pont Corporation, quando em 1974 foi publicada a primeira pesquisa mostrando que seu produto Freon ataca a camada protetora de oz�nio. H� muitos outros exemplos.�



[A] Foram fechadas?...:)





�� de se pensar que, antes de denunciar descobertas cient�ficas indesejadas, as principais companhias deveriam empregar os seus consider�veis recursos para verificar a seguran�a dos produtos que se prop�em fabricar. E, se perdessem algo, se cientistas independentes sugerissem um perigo, por que as companhias se oporiam? Prefeririam matar pessoas a perder lucros? Se, nesse mundo incerto, um erro precisa ser cometido, ele n�o deveria ter o objetivo de proteger os clientes e o p�blico? E, por outro lado, o que esses casos revelam sobre a capacidade de o sistema de livre empresa policiar a si mesmo? N�o s�o exemplos em que a interfer�ncia do governo � claramente a favor do interesse p�blico?�



[A] E os impostos como ficam?



�Um relat�rio interno da Brown and Williamson Tobacco Corporation, de 1971, lista como objetivo da companhia "afastar das mentes de milh�es a falsa convic��o de que fumar cigarros causa c�ncer de pulm�o e outras doen�as; uma convic��o baseada em pressupostos fan�ticos, rumores falaciosos, afirma��es sem fundamento e declara��es n�o cient�ficas de oportunistas que buscam notoriedade". Eles se queixam do ataque incr�vel, sem precedentes e abomin�vel contra o cigarro, constituindo o maior libelo e a maior difama��o j� perpetrados contra um produto na hist�ria da livre empresa; um libelo criminoso de t�o grandes propor��es e implica��es que � de se perguntar como essa cruzada de cal�nias pode se acomodar sob a Constitui��o pode ser t�o desrespeitada e violada [sic].�



[A] quem esta no governo?...:)





�Essa ret�rica � apenas um pouco mais inflamada do que a das declara��es que a ind�stria de tabaco emite de tempos em tempos para consumo p�blico.�



[A] Como diria Hebe: �Gracinha de menino!�





�H� muitas marcas de cigarros que anunciam baixo n�vel de alcatr�o (dez miligramas ou menos por cigarro). Por que isso � uma virtude? Porque � no alcatr�o refrat�rio que os hidrocarbonetos arom�ticos polic�clicos e algumas outras subst�ncias cancer�genas se concentram. As propagandas que enfatizam baixos teores de alcatr�o n�o s�o uma admiss�o t�cita das companhias de tabaco de que os cigarros realmente causam c�ncer?�



[A] Sim, elas que d�o um ar de fumar � ser her�i...:)







�A Healthy Building lnternational � uma organiza��o lucrativa, que recebe h� anos milh�es de d�lares da ind�stria do fumo. Ela realiza pesquisas sobre fumo passivo, e presta declara��es para as companhias de tabaco. Em 1994, tr�s de seus t�cnicos reclamaram que altos executivos teriam falsificado dados sobre part�culas de cigarro inal�veis no ar. Em todos os casos, os dados inventados ou "corrigidos" faziam a fuma�a de cigarro parecer mais segura do que as medi��es dos t�cnicos haviam indicado. Os departamentos de pesquisa da companhia ou as firmas do ramo contratadas j� descobriram alguma vez que um produto � mais perigoso do que a empresa de tabaco declarou publicamente? Em caso positivo, mantiveram o emprego?�



[A] como em todas as drogas legais...:)





�O tabaco vicia; segundo muitos crit�rios, ainda mais do que a hero�na e a coca�na. Havia uma raz�o para as pessoas "caminharem uma milha por um Camel", como diziam os an�ncios da d�cada de 40. J� morreram mais pessoas por causa do fumo do que em toda a Segunda Guerra Mundial. Segundo a Organiza��o Mundial de Sa�de, o fumo mata 3 milh�es de pessoas por ano em todo o mundo. Esse n�mero vai chegar a 10 milh�es de mortes por ano em 2020 em parte devido a uma grande campanha publicit�ria que pinta o tabagismo como um h�bito avan�ado e elegante para as jovens mulheres do mundo em desenvolvimento. � em parte por causa da falta disseminada de conhecimento sobre a detec��o de mentiras, o pensamento cr�tico e o m�todo cient�fico que a ind�stria de tabaco consegue ser o fornecedor bem-sucedido dessa mistura de venenos que viciam. A credulidade mata.�



[A] Pensamento critico nem mesmo o Sagan teve, o que ocorre que a ci�ncia com seu discursinho tenta por a n�s no rumo certo das palavras bonitas, acreditem que o Sol � um monte de gases, uma grande fus�o nuclear, o mundo acabara por causa dos vulc�es e tudo que ela pode nos dizer. N�o far� nada com o c�ncer, muito menos com a AIDS...vai sempre dizer essas coisas sem ao menos pesquisar, como diria Nietzsche, a ci�ncia � a fuga mas � humana demasiada humana...:)



Obrigado Sagan por esse divertimento...:)





Amauri

[As partes desta mensagem que n�o continham texto foram removidas]



SUBJECT: Re: [ciencialist] Fw: Passarinhos incomodando
FROM: "murilo filo" <avalanchedrive@hotmail.com>
TO: ciencialist@yahoogrupos.com.br
DATE: 06/01/2005 17:13

Não!!!! Não!!!!! Vc chama o Murilão!
+ tarde lhe passo os dados, que estão em casa. abr/M.

>From: "Luiz Ferraz Netto" <leobarretos@uol.com.br>
>Reply-To: ciencialist@yahoogrupos.com.br
>To: <ciencialist@yahoogrupos.com.br>
>Subject: Re: [ciencialist] Fw: Passarinhos incomodando
>Date: Thu, 6 Jan 2005 08:35:53 -0200
>
>Morcegos? Fiquei interessado. No meu rancho há morcegos entre o telhado e o
>forro ... uma jostra! Que faço para sumir com eles dali? Chamo o Batman? A
>Batgirl?
>Agradeço a informação.
>[]'
>
> ===========================
> Luiz Ferraz Netto [Léo]
> leobarretos@uol.com.br
> http://www.feiradeciencias.com.br
> ===========================
>-----Mensagem Original-----
>De: "murilo filo" <avalanchedrive@hotmail.com>
>Para: <ciencialist@yahoogrupos.com.br>
>Enviada em: quarta-feira, 5 de janeiro de 2005 22:20
>Assunto: RE: [ciencialist] Fw: Passarinhos incomodando
>
>
>
>Favor aguardar. Sempre vejo estas coisas anunciadas no Estadão.
>No caderno Agrícola de hoje há um indicado p/ratos e morcegos... pombos
>não!
>Achando eu mando, se ninguém mais mandar e se meu msn (uma josta) deixar.
>abr/M. SP 05/jan/2005
>
> >From: "Luiz Ferraz Netto" <leobarretos@uol.com.br>
> >Reply-To: ciencialist@yahoogrupos.com.br
> >To: "ciencialist" <ciencialist@yahoogrupos.com.br>
> >Subject: [ciencialist] Fw: Passarinhos incomodando
> >Date: Wed, 5 Jan 2005 20:49:21 -0200
> >
> >Não há uma pasta (creme, seja lá o que for) que esparramado no local
> >espanta as pombas? Já ouvi algo sobre isso!
> >
> >[]'
> > ===========================
> > Luiz Ferraz Netto [Léo]
> > leobarretos@uol.com.br
> > http://www.feiradeciencias.com.br
> > ===========================
> >-----Mensagem Original-----
> >De: Guilherme
> >Para: leobarretos@uol.com.br
> >Cc: ma-padua@uol.com.br
> >Enviada em: quarta-feira, 5 de janeiro de 2005 11:03
> >Assunto: Passarinhos incomodando
> >
> >
> >Estou c/ problema , na varanda de minha casa, esta sendo dormitorio de
> >pomba e gostaria de tira-las de la. Tem algum aparelho que transmite
>ondas
> >que incomodam e elas somem do lugar. Para fazer cachorro parar de latir
> >euja ouvi falar. Grata Cida
> >
> >
> >--------------------------------------------------------------------------------
> >
> >
> >No virus found in this incoming message.
> >Checked by AVG Anti-Virus.
> >Version: 7.0.300 / Virus Database: 265.6.8 - Release Date: 03/01/2005
> >
> > ----------
> >
> >No virus found in this outgoing message.
> >Checked by AVG Anti-Virus.
> >Version: 7.0.300 / Virus Database: 265.6.8 - Release Date: 03/01/2005
> >
> >
> >[As partes desta mensagem que não continham texto foram removidas]
> >
>
>
>
>
>##### ##### #####
>
>Para saber mais visite
>http://www.ciencialist.hpg.ig.com.br
>
>
>##### ##### ##### #####
>Links do Yahoo! Grupos
>
>
>
>
>
>
>
>
>
>
>--
>No virus found in this incoming message.
>Checked by AVG Anti-Virus.
>Version: 7.0.300 / Virus Database: 265.6.8 - Release Date: 03/01/2005
>
>
>
>
>--
>No virus found in this outgoing message.
>Checked by AVG Anti-Virus.
>Version: 7.0.300 / Virus Database: 265.6.8 - Release Date: 03/01/2005
>




SUBJECT: FW: Massive space explosion bursts record: in World Science
FROM: "murilo filo" <avalanchedrive@hotmail.com>
TO: ciencialist@yahoogrupos.com.br, forum-ciencia@yahoogrupos.com.br
DATE: 06/01/2005 17:19

( STRAIGHT ON TOPICS!!! ENJOY. M. 06/JAN )

>From: "World Science" <emailnews@world-science.net>
>To: <emailnews@world-science.net>
>Subject: Massive space explosion bursts record: in World Science
>Date: Wed, 5 Jan 2005 20:44:09 -0500
>
>* Massive space explosion bursts record:
>A supermassive black hole has created the most
>powerful explosion on record, according to
>astronomers.
>
>http://www.world-science.net/othernews/050105_spaceblastfrm.htm
>
>* Giant eagles evolved super-big, super-fast:
>Scientists say a lack of predators made the
>extinct New Zealand eagles become so huge,
>they hunted animals much bigger than most
>humans are.
>
>http://www.world-science.net/othernews/050105_eaglefrm.htm
>
>* New language area in brain found:
>The discovery could reveal facts about the
>origins of language, researchers say.
>
>http://www.world-science.net/othernews/050105_brainlangfrm.htm
>
>
>++++++++++++++++++++++
>
>This is the World Science email newsletter.
>To cancel your subscription, please reply to this
>email address with "unsubscribe" in the subject
>line. To change the address to which you would
>like the newsletter sent, please write to this email
>address and request the change.




SUBJECT: Re: [ciencialist] Analise de Carl Sagan por mim...
FROM: "Oraculo" <oraculo@atibaia.com.br>
TO: <ciencialist@yahoogrupos.com.br>
DATE: 06/01/2005 17:36

Olá Amaury

risos..:-) Você vai analisar Sagan e detonar Bacon?..:-) Bem, boa sorte,
gente bastante equipada e capaz tentou o mesmo, com um monte de dados e
informação, e ainda assim, não foi fácil, espero que tenha maior sorte..:-)

Mas, ainda penso que está confundindo as bolas (um bocado). É fácil perceber
que sua irritação contra americanos, os USA, gente de grana, poderosos em
geral, está misturando seu raciocínio. Ou não escreveria coisas como essa:

"Amaury:[A] Para que Sagan quer saber isso tudo? Para o ser humano
transformar em patrimônios e poucos poderem ver? "

Amury, ele não quer saber tudo isso (perguntas que deveriam ser feitas aos
canalizadores, conforme o texto original). Ele pouco se importa com as
respostas..:-) Ele pergunta para decidir se o canalizado é real ou fraude,
engano ou disfunção mental. Qualquer um ao ler o trecho percebe isso, você
deve estar muito irritado para não faze-lo..:-) Por exemplo, se eu estivesse
presente na canalização de um espírito de soldado romano do tempo do
império, gostaria de saber se ele pode falar em latin, ou ao menos
compreender frases em latin. Não me interessa a resposta ou o que ele vai
dizer, mas o fato de que sabe latin..:-) Um espirito milenar que pode se
lembrar de sua vida em Roma imperial, suas andanças pelas ruas de Roma, mas
não reconhece uma única palavra em latin me provoca muita desconfiança..;-)

Antes de concluir que o medium está realmetne recebendo um espírito
(concluir, não acreditar), Sagan quer ter mais dados, mais elementos. Como o
médico que pede ao paciente que diga 33 e não tem O MENOR INTERESSE NO
NÚMERO 33. Ou você pensa que ao ouvir 33 o médico diz "oh, puxa, verdade, é
33?"? Ou que vai transformar essa informação, o 33, em patrimonio?

O texto é claro. O argumento de Sagan é perfeito e eficaz. Pode até tentar
refuta-lo, mas não é contestando o interesse dele nas informações..:-) São
perguntas de teste, padrão em experimentos de pesquisa, não dados para
"roubar" do médium e ficar rico..:-)

Todo o resto de seu texto segue essa linha, diatribes sem muita ligação com
o texto de Sagan. É dificil refutar algo assim e nem vou tentar. Basta ler o
texto..:-)

Eu aconselharia que rele-se o texto de Sagan, com menos raiva, e, ou
compreendesse o que ele defende, ou refutasse com algum argumento razoável.
Sem isso, fica apenas a sensação de mau humor e disputa emocional com o
autor..:-)

Um abraço.

Homero






SUBJECT: Re: Fw: Teoria da Relatividade Especial
FROM: "dfahlb" <dfahlb@yahoo.com>
TO: ciencialist@yahoogrupos.com.br
DATE: 06/01/2005 18:29


--- Em ciencialist@yahoogrupos.com.br, "Luiz Ferraz Netto"
<leobarretos@u...> escreveu
> Vamos responder? ... educadamente ...

Já que ninguém quis respondera msg
http://br.groups.yahoo.com/group/ciencialist/message/43472 , deixe-me
tentar.

> Assunto: Teoria da Relatividade Especial
>
> Jaime Pacheco Filho
>
> jaime-pacheco@u...
>
> Escolaridade: superior (Economia)
>
>
>
> Dúvida: Entendo que o Efeito Doppler é uma conseqüência da
>independência da velocidade de uma onda (sonora ou luminosa) em
>relação ao movimento do emissor e do receptor no meio de propagação.

Na verdade é o contrário: o Efeito Doppler é uma consequência da
DEPENDÊNCIA da velocidade de uma onda em relação ao movimento do
emissor e do receptor no meio de propagação. A


>Então, por que a velocidade da luz (da qual não se conhece o meio de
>propagação) é especial e não a do som (do qual se sabe o meio de
>propagação)?

Não é que não se conhece, é que não existe tal meio. A luz se
movimenta no vácuo, por exemplo, aonde não existe meio material. Na
verdade, o meio material atrapalha a propagação da luz. A luz tem a
velocidade reduzida em meios materiais pois o material acaba por
aborsorvê-la e reemiti-la o que causa retardamento da velocidade (nos
meios opacos, a luz é absorvida pelo material e ou é re-emitida sob a
forma de outra frequência ou é transformada em calor - em movimento
dos átomos do meio).

Agora, por que a velocidade da luz é a mesma para todos os
observadores (independendo de do movimento deles em relação à luz ou
da fonte da luz em relação a eles) e não o som?

Bom, no caso do som, a velocidade não é a mesma pois depende da
velocidade da fonte e do observador em relação ao meio. Como a luz
não dependende de meio para se propagar (afinal se propaga no vácuo)
se a velocidade dela dependesse de algum fator, poderíamos encontrar
o referencial absoluto!!!

Vai um exemplo. Suponha que a velocidade da luz dependesse do
movimento do observador. Agora suponha que um observador, chamemo-lo
de O1, estivesse numa nave se movimentando com movimento uniforme e
retilíneo (velocidade constante, chamemo-la de v) ou seja, não há a
presença de campo gravitacional (ou seja sobre ele não é exercida a
força gravitacional) ou então que a resultante das forças que agem
sobre a nave é zero.

Continuando: como a luz se propaga no vácuo, vamos deixar nossa nave
se movimentando lá, no vácuo. Como o movimento é uniforme e retilínio
se o observador O1 fizesse QUALQUER experiência mecânica (por pêndulo
para oscilar, lançar objetos para cima, etc) ele NÃO VAI conseguir
dizer se está com alguma velocidade ou não, afinal o movimento
retilínio e uniforme é relativo pois depende de um ponto de
referência, ou referencial.

Se você se move, você se move em relação a que? Se não há nada por
perto, você não poderia dizer se está se movimentando com velocidade
retilínea e uniforme ou se não está. Mas, mais que isso.

Na verdade, tal observador se tivesse um ponto de referência, veria
esse ponto de referência se afastar dele e ele poderia afirmar que É
O PONTO DE REFERÊNCIA que se afasta. Outro observador, chamemo-lo de
O2, que estivesse no ponto de referência diria que não, que É A NAVE
que se afasta. Ambos estariam certos!!!! Afinal, o movimento uniforme
e retilíneo é relativo: depende do observador.

Agora, voltando a pergunta: por que a velocidade da luz é a mesma
para todos os observadores (independendo de do movimento deles em
relação à luz ou da fonte da luz em relação a eles)?

Imagine então O2, que está num ponto de referência, vê O1 (que está
na nave) nalgum instante, passar por ele e indo "da esquerda para a
direita". O1 diria discordia dizendo que foi O2 que passou vindo "da
direita para a esquerda". Como disse: ambos estariam certos pois o
movimento é relativo e NENHUMA experiência mecânica poderia dizer que
está num referencial privilegiado (referencial absoluto).

Então, O2, manda um feixe de luz ir de encontro com O1. A luz sai com
velocidade c de O2. Ela, em dado instante, ultrapassa O1. No instante
que a luz passa por O1, ele mede a velocidade dela. O1, então, mede a
velocidade (chamemo-la de w) com que a luz ultrapassa ele. Se ele
medisse e visse que desse w = c - v, ele, O1, saberia de cara que ele
está se movimentando! Ou seja, a luz tem que ultrapassá-lo com
velocidade c.

Essa idéia é comentada no livro "Evolução das Idéias da Física" de
Einstein e Infeld um livro usado no primeiro ano de Física em algumas
faculdades. É um livro de fácil leitura e acessível. Einstein não usa
nenhuma equação e explica ainda assim de maneira clara a Relatividade
no livro.

> Será que os efeitos previstos pela Teoria da
>Relatividade Especial como a alteração no ritmo de relógios e o
>encurtamento de réguas não são apenas aparentes, devidos ao Efeito
>Doppler?

Não, não são. São detectados experimentalmente. Até experiências em
aviões supersônicos (medidos os atrasos dos relógios nele) já foram
feitos.

Inclusive, como o Efeito Doppler é derivado das transformações de
Galileu, ele está incorreto, sobretudo para velocidades altas, ainda
que você possa usá-lo para velocidades baixas porque o erro é
desprezível.

O Efeito Doppler mais geral seria o Efeito Doppler Relativístico.
Inclusive, este último é que deve ser usado mesmo para ondas que não
precisam de meio para se propagar, como as ondas eletromagnéticas (a
luz).

>
> Se fôssemos todos cegos talvez a Teoria da Relatividade
se baseasse na velocidade do som.

Não, não seria. Se fôssemos cegos e conseguíssemos ir desenvolvendo
uma tecnologia cada vez mais precisa, veríamos que as transformações
de Galileu (as da Mecânica Clássica) não são válidas e são apenas uma
aproximação. Que quando estamos em velocidade baixa ela é
aproximadamente exata. Mas o erro dela é melhor percebido a altas
velocidades.

Se fôssemos cegos, talvez descobríssemos a luz pois quando fôssemos
medir a velocidade com que o calor se propaga por radiação é mais
rápido que a velocidade do som e mede c.

>
> Einstein explicou porque os relógios atrasam e as
>réguas encurtam quando são acelerados?

Sim, leia o livro "Evolução das Idéias da Física" de Einstein e
Infeld, é o livro mais fácil de entende a Relatividade para os não-
físicos.

A explicação é toda a que estamos tendo e mais algumas. Primeiro:
evidências experimentais mostraram que a velocidade da luz não
depende nem do movimento da fonte e não do observador em relação a
ela e Einstein foi o primeiro a admitir isso. Depois Einstein supôs
que não apenas as Leis da Mecânica, mas também do Eletromagnetismo
(experiências com luz) não poderiam definir o referencial absoluto,
no que ele sintetizou nos princípios:

1. As Leis da Física são invariantes em todos os sistemas inerciais.
Nenhum sistema inercial preferido existe. (O Princípio da
Relatividade)
2. A velocidade da luz no espaço livre tem o mesmo valor "c" em todos
os sistemas inerciais. (O Princípio da Constância da Velocidade da
Luz).

Aqui "sistemas" e "referenciais" são sinônimos.
O princípio 1 é o Princípio da Relatividade já pensado anteriormente
por Galileu, entretando, as transformações de Galileu, que mostram
que se muda de um sistema para outro, está errada. Os erros são tão
maiores quanto mais próximo se aproxima da velocidade da luz. Claro
que a velocidades baixas, os erros são desprezíveis.

Um livro que também é interessante de se olhar é: Introduction To
Special Relativity. Autor: Robert Resnick.
Usado: no 3.o e 4.o ano de Física (bacharelado e licenciatura,
respectivamente). Assunto: além de explicar vários conceitos da
Relatividade de Einstein, ele dá outras experiências que só a
Relatividade de Einstein explica além de discorrer brevemente sobre
algumas outras teorias rivais à Relatividade de Einstein mas que
falharam e aonde falharam.

Este livro, embora de leitura um pouco mais complexa que o "Evolução
das Idéias da Física" de Einstein e Infeld, vale a pena ser lido e
ainda assim, é de linguagem acessível.

Espero ter ajudado.
Abraços.

>
>
>
> Obrigado
>






SUBJECT: Re: Fw: Passarinhos incomodando
FROM: Maria Natália <grasdic@hotmail.com>
TO: ciencialist@yahoogrupos.com.br
DATE: 06/01/2005 18:38


Leo:
Aí não se faz canja de pombo? Arrozinho de pombo?
E se puser pimenta na varanda? Pombo espirra...LOL. Outra hipótes é
ir à comunidade "Eu odeio pombo" no Orkut. Deve haver.
O Orkut tá giríssimo e permite um trabalho fantástico de análise e
fazer uns contos ou histórias de FC.
Mas que guano de pombo é muito ácido e dá cabo de peitoril de
janela, dá.
Um abraço

--- Em ciencialist@yahoogrupos.com.br, "Luiz Ferraz Netto"
<leobarretos@u...> escreveu
> Não há uma pasta (creme, seja lá o que for) que esparramado no
local espanta as pombas? Já ouvi algo sobre isso!
>
> []'
> ===========================
> Luiz Ferraz Netto [Léo]
> leobarretos@u...
> http://www.feiradeciencias.com.br
> ===========================
> -----Mensagem Original-----
> De: Guilherme
> Para: leobarretos@u...
> Cc: ma-padua@u...
> Enviada em: quarta-feira, 5 de janeiro de 2005 11:03
> Assunto: Passarinhos incomodando
>
>
> Estou c/ problema , na varanda de minha casa, esta sendo
dormitorio de pomba e gostaria de tira-las de la. Tem algum aparelho
que transmite ondas que incomodam e elas somem do lugar. Para fazer
cachorro parar de latir euja ouvi falar. Grata Cida
>
>
> -------------------------------------------------------------------
-------------
>
>
> No virus found in this incoming message.
> Checked by AVG Anti-Virus.
> Version: 7.0.300 / Virus Database: 265.6.8 - Release Date:
03/01/2005
>
> ----------
>
> No virus found in this outgoing message.
> Checked by AVG Anti-Virus.
> Version: 7.0.300 / Virus Database: 265.6.8 - Release Date:
03/01/2005
>
>
> [As partes desta mensagem que não continham texto foram removidas]





SUBJECT: Re: [ciencialist] aranhas...
FROM: "Amauri Jr" <amaurijunior2@yahoo.com.br>
TO: <ciencialist@yahoogrupos.com.br>
DATE: 06/01/2005 19:19

E esses que ficam na parece ou de baixo das pias do banheiro??

Amauri
----- Original Message -----
From: murilo filo
To: ciencialist@yahoogrupos.com.br
Sent: Wednesday, January 05, 2005 10:46 PM
Subject: RE: [ciencialist] aranhas...


Se as teias são ''aéreas'', muito fortes, grandes e de cor meio dourada, e
se esta aranha, com uns 5cm, quando ameaçada, mudar de cor, trata-se a
aranha lôbo, que é boazinha, bonitinha, decorativa e útil.
Se a teia for no chão, meio embolada e aberta como um funil na boca de um
pequeno túnel, trata-se da ''armadeira'', perigosa e famosa nas Discovery da
vida. Dizem que é bem pior que a tal viúva negra. Boa sorte e faça seu
testamento! :] Murilo SP 05/jan/2005

>From: "E m i l i a n o C h e m e l l o" <chemelloe@yahoo.com.br>
>Reply-To: ciencialist@yahoogrupos.com.br
>To: <ciencialist@yahoogrupos.com.br>
>Subject: [ciencialist] aranhas...
>Date: Wed, 5 Jan 2005 10:14:55 -0200
>
>Alguém?
>
>[ ] 's do Emiliano Chemello
>emiliano@quimica.net
>http://www.quimica.net/emiliano
>http://www.ucs.br/ccet/defq/naeq
>
>
>Contato Naeq:
>Nome: Camila
>Email: camila_alves19@hotmail.com
>Telefone: Dúvidas!!
>Mensagem: Há alguns dias eu venho pesquisando sobre aranhas, no meu jardim
>tem 2 aranhas construtoras de teia, elas são sedentárias estão sempre
>paradas, posicionam suas patas dianteiras para cima e suas traseiras para
>baixo fica parecendo que só são 4, suas patas são listradas de amarelo, uma
>listra grande amarela pode ser identificada em seu abdome. Mas a minha
>dúvida é se elas são venenosas. Em certos sites podemos encontrar que as
>aranhas clinicamente perigosas não são construtoras de teias e sim
>errantes,
>mas eu queria saber mais sobre essas aranhas do meu jardim, pois não
>encontro nada em sites, apenas o que tenho são essas características. Se
>puderem me ajudar, estou estudando para fazer faculdade de biologia. Minha
>curiosidade sobre assuntos biologicos é grande.
>Obrigada!
>




##### ##### #####

Para saber mais visite
http://www.ciencialist.hpg.ig.com.br


##### ##### ##### #####


Yahoo! Grupos, um serviço oferecido por:

São Paulo Rio de Janeiro Curitiba Porto Alegre Belo Horizonte Brasília




------------------------------------------------------------------------------
Links do Yahoo! Grupos

a.. Para visitar o site do seu grupo na web, acesse:
http://br.groups.yahoo.com/group/ciencialist/

b.. Para sair deste grupo, envie um e-mail para:
ciencialist-unsubscribe@yahoogrupos.com.br

c.. O uso que você faz do Yahoo! Grupos está sujeito aos Termos do Serviço do Yahoo!.



[As partes desta mensagem que não continham texto foram removidas]



SUBJECT: Passarinhos incomodando
FROM: "L.E.R.de Carvalho" <lecarvalho@infolink.com.br>
TO: ciencialist@yahoogrupos.com.br
DATE: 06/01/2005 20:11

At 18:38 6/1/2005, you wrote:

>Leo:
>Aí não se faz canja de pombo? Arrozinho de pombo?
>E se puser pimenta na varanda? Pombo espirra...LOL. Outra hipótes é
>ir à comunidade "Eu odeio pombo" no Orkut. Deve haver.
>O Orkut tá giríssimo e permite um trabalho fantástico de análise e
>fazer uns contos ou histórias de FC.
>Mas que guano de pombo é muito ácido e dá cabo de peitoril de
>janela, dá.
>Um abraço


NO ORKUT, EU SÓ CONHEÇO DUAS COMUNIDADES.

UMA É...
GENTE QUE ANDA PELA CASA ESCOVANDO OS DENTES.

A OUTRA É...
GENTE QUE FOI EXPULSA DE TUDO QUANTO É LISTA E SE JUNTOU AQUI.


Será que alguma delas serve pra tu ?

Luiz Eduardo

[As partes desta mensagem que não continham texto foram removidas]



SUBJECT: Re: [ciencialist] Passarinhos incomodando - Alerta ao L.E.R. de Carvalho
FROM: "Luiz Ferraz Netto" <leobarretos@uol.com.br>
TO: <ciencialist@yahoogrupos.com.br>
DATE: 06/01/2005 22:16

Ooops L.E.R. de Carvalho,

vc está forçando a barra. Que foi que a Natália lhe fez, nessa mensagem dirigida a mim, para que vc poste um comentário, pesado e rancoroso desses?
Fica esse alerta. Entrarei em contato com Brudna e Mesquita. Nesse meio tempo, para não esticar o problema, solicito que não envie qqer mensagem ao C-list. Nem mesmo responda a essa mensagem de alerta.

[]'
===========================
Luiz Ferraz Netto [Léo]
leobarretos@uol.com.br
http://www.feiradeciencias.com.br
===========================
-----Mensagem Original-----
De: "L.E.R.de Carvalho" <lecarvalho@infolink.com.br>
Para: <ciencialist@yahoogrupos.com.br>
Enviada em: quinta-feira, 6 de janeiro de 2005 20:11
Assunto: [ciencialist] Passarinhos incomodando



At 18:38 6/1/2005, you wrote:

>Leo:
>Aí não se faz canja de pombo? Arrozinho de pombo?
>E se puser pimenta na varanda? Pombo espirra...LOL. Outra hipótes é
>ir à comunidade "Eu odeio pombo" no Orkut. Deve haver.
>O Orkut tá giríssimo e permite um trabalho fantástico de análise e
>fazer uns contos ou histórias de FC.
>Mas que guano de pombo é muito ácido e dá cabo de peitoril de
>janela, dá.
>Um abraço


NO ORKUT, EU SÓ CONHEÇO DUAS COMUNIDADES.

UMA É...
GENTE QUE ANDA PELA CASA ESCOVANDO OS DENTES.

A OUTRA É...
GENTE QUE FOI EXPULSA DE TUDO QUANTO É LISTA E SE JUNTOU AQUI.


Será que alguma delas serve pra tu ?

Luiz Eduardo

[As partes desta mensagem que não continham texto foram removidas]



##### ##### #####

Para saber mais visite
http://www.ciencialist.hpg.ig.com.br


##### ##### ##### #####
Links do Yahoo! Grupos










--
No virus found in this incoming message.
Checked by AVG Anti-Virus.
Version: 7.0.300 / Virus Database: 265.6.8 - Release Date: 03/01/2005




--
No virus found in this outgoing message.
Checked by AVG Anti-Virus.
Version: 7.0.300 / Virus Database: 265.6.8 - Release Date: 03/01/2005



SUBJECT: Re: [ciencialist] Fw: Passarinhos incomodando
FROM: "murilo filo" <avalanchedrive@hotmail.com>
TO: ciencialist@yahoogrupos.com.br
DATE: 06/01/2005 22:20

Léo, aquí vão os dados anti-morcegais:
Brastécnica - Alfenas MG 35 3292 1889
site, q/ainda não ví www.brastecnica.com.brrrrrrrrrr
São equipamenros de ultra-som... alta sacanagem com os coitados!
Pelo anúncio há um tal de ''SG 02'' q/deve ser o capêta!
Sorte! M. SP 06/jan/2005

>From: "Luiz Ferraz Netto" <leobarretos@uol.com.br>
>Reply-To: ciencialist@yahoogrupos.com.br
>To: <ciencialist@yahoogrupos.com.br>
>Subject: Re: [ciencialist] Fw: Passarinhos incomodando
>Date: Thu, 6 Jan 2005 08:35:53 -0200
>
>Morcegos? Fiquei interessado. No meu rancho há morcegos entre o telhado e o
>forro ... uma jostra! Que faço para sumir com eles dali? Chamo o Batman? A
>Batgirl?
>Agradeço a informação.
>[]'
>
> ===========================
> Luiz Ferraz Netto [Léo]
> leobarretos@uol.com.br
> http://www.feiradeciencias.com.br
> ===========================
>-----Mensagem Original-----
>De: "murilo filo" <avalanchedrive@hotmail.com>
>Para: <ciencialist@yahoogrupos.com.br>
>Enviada em: quarta-feira, 5 de janeiro de 2005 22:20
>Assunto: RE: [ciencialist] Fw: Passarinhos incomodando
>
>
>
>Favor aguardar. Sempre vejo estas coisas anunciadas no Estadão.
>No caderno Agrícola de hoje há um indicado p/ratos e morcegos... pombos
>não!
>Achando eu mando, se ninguém mais mandar e se meu msn (uma josta) deixar.
>abr/M. SP 05/jan/2005
>
> >From: "Luiz Ferraz Netto" <leobarretos@uol.com.br>
> >Reply-To: ciencialist@yahoogrupos.com.br
> >To: "ciencialist" <ciencialist@yahoogrupos.com.br>
> >Subject: [ciencialist] Fw: Passarinhos incomodando
> >Date: Wed, 5 Jan 2005 20:49:21 -0200
> >
> >Não há uma pasta (creme, seja lá o que for) que esparramado no local
> >espanta as pombas? Já ouvi algo sobre isso!
> >
> >[]'
> > ===========================
> > Luiz Ferraz Netto [Léo]
> > leobarretos@uol.com.br
> > http://www.feiradeciencias.com.br
> > ===========================
> >-----Mensagem Original-----
> >De: Guilherme
> >Para: leobarretos@uol.com.br
> >Cc: ma-padua@uol.com.br
> >Enviada em: quarta-feira, 5 de janeiro de 2005 11:03
> >Assunto: Passarinhos incomodando
> >
> >
> >Estou c/ problema , na varanda de minha casa, esta sendo dormitorio de
> >pomba e gostaria de tira-las de la. Tem algum aparelho que transmite
>ondas
> >que incomodam e elas somem do lugar. Para fazer cachorro parar de latir
> >euja ouvi falar. Grata Cida
> >
> >
> >--------------------------------------------------------------------------------
> >
> >
> >No virus found in this incoming message.
> >Checked by AVG Anti-Virus.
> >Version: 7.0.300 / Virus Database: 265.6.8 - Release Date: 03/01/2005
> >
> > ----------
> >
> >No virus found in this outgoing message.
> >Checked by AVG Anti-Virus.
> >Version: 7.0.300 / Virus Database: 265.6.8 - Release Date: 03/01/2005
> >
> >
> >[As partes desta mensagem que não continham texto foram removidas]
> >
>
>
>
>
>##### ##### #####
>
>Para saber mais visite
>http://www.ciencialist.hpg.ig.com.br
>
>
>##### ##### ##### #####
>Links do Yahoo! Grupos
>
>
>
>
>
>
>
>
>
>
>--
>No virus found in this incoming message.
>Checked by AVG Anti-Virus.
>Version: 7.0.300 / Virus Database: 265.6.8 - Release Date: 03/01/2005
>
>
>
>
>--
>No virus found in this outgoing message.
>Checked by AVG Anti-Virus.
>Version: 7.0.300 / Virus Database: 265.6.8 - Release Date: 03/01/2005
>




SUBJECT: RE: [ciencialist] Re: Fw: Passarinhos incomodando
FROM: "murilo filo" <avalanchedrive@hotmail.com>
TO: ciencialist@yahoogrupos.com.br
DATE: 06/01/2005 22:24

Mª Nª, oi.
Aquí em São Paulo, dizemos que pombo é o rato de asas...
Não dá nem para chegar perto! abr/M.

>From: Maria Natália <grasdic@hotmail.com>
>Reply-To: ciencialist@yahoogrupos.com.br
>To: ciencialist@yahoogrupos.com.br
>Subject: [ciencialist] Re: Fw: Passarinhos incomodando
>Date: Thu, 06 Jan 2005 20:38:33 -0000
>
>
>Leo:
>Aí não se faz canja de pombo? Arrozinho de pombo?
>E se puser pimenta na varanda? Pombo espirra...LOL. Outra hipótes é
>ir à comunidade "Eu odeio pombo" no Orkut. Deve haver.
>O Orkut tá giríssimo e permite um trabalho fantástico de análise e
>fazer uns contos ou histórias de FC.
>Mas que guano de pombo é muito ácido e dá cabo de peitoril de
>janela, dá.
>Um abraço
>
>--- Em ciencialist@yahoogrupos.com.br, "Luiz Ferraz Netto"
><leobarretos@u...> escreveu
> > Não há uma pasta (creme, seja lá o que for) que esparramado no
>local espanta as pombas? Já ouvi algo sobre isso!
> >
> > []'
> > ===========================
> > Luiz Ferraz Netto [Léo]
> > leobarretos@u...
> > http://www.feiradeciencias.com.br
> > ===========================
> > -----Mensagem Original-----
> > De: Guilherme
> > Para: leobarretos@u...
> > Cc: ma-padua@u...
> > Enviada em: quarta-feira, 5 de janeiro de 2005 11:03
> > Assunto: Passarinhos incomodando
> >
> >
> > Estou c/ problema , na varanda de minha casa, esta sendo
>dormitorio de pomba e gostaria de tira-las de la. Tem algum aparelho
>que transmite ondas que incomodam e elas somem do lugar. Para fazer
>cachorro parar de latir euja ouvi falar. Grata Cida
> >
> >
> > -------------------------------------------------------------------
>-------------
> >
> >
> > No virus found in this incoming message.
> > Checked by AVG Anti-Virus.
> > Version: 7.0.300 / Virus Database: 265.6.8 - Release Date:
>03/01/2005
> >
> > ----------
> >
> > No virus found in this outgoing message.
> > Checked by AVG Anti-Virus.
> > Version: 7.0.300 / Virus Database: 265.6.8 - Release Date:
>03/01/2005
> >
> >
> > [As partes desta mensagem que não continham texto foram removidas]
>
>
>




SUBJECT: Re: [ciencialist] aranhas...
FROM: "murilo filo" <avalanchedrive@hotmail.com>
TO: ciencialist@yahoogrupos.com.br
DATE: 06/01/2005 23:01

Devem ser as papa-moscas, pequenas e rápidas; concorrentes das lagartixas!
Aquí em casa, ninguém mata... inofensivas e ainda comem pernilongos!
Ou será que vc está falando daquelas magrelas de perninhas fininhas e que
têm tremedeiras?
Nestas não tenho a mínima tesão! Não gosto! Sem graça e muito paradonas.
Possuem um veneno forte, mas não conseguem inocular, parece.
Antes, eu também repetia com outros, gozações sobre a ''cultura inútil'',
até eu descobrir as delícias dos produtivos novos enlaces cerebrais...
My name is Discovery... abr/M.

>From: "Amauri Jr" <amaurijunior2@yahoo.com.br>
>Reply-To: ciencialist@yahoogrupos.com.br
>To: <ciencialist@yahoogrupos.com.br>
>Subject: Re: [ciencialist] aranhas...
>Date: Thu, 6 Jan 2005 19:19:30 -0200
>
>E esses que ficam na parece ou de baixo das pias do banheiro??
>
>Amauri
> ----- Original Message -----
> From: murilo filo
> To: ciencialist@yahoogrupos.com.br
> Sent: Wednesday, January 05, 2005 10:46 PM
> Subject: RE: [ciencialist] aranhas...
>
>
> Se as teias são ''aéreas'', muito fortes, grandes e de cor meio dourada,
>e
> se esta aranha, com uns 5cm, quando ameaçada, mudar de cor, trata-se a
> aranha lôbo, que é boazinha, bonitinha, decorativa e útil.
> Se a teia for no chão, meio embolada e aberta como um funil na boca de
>um
> pequeno túnel, trata-se da ''armadeira'', perigosa e famosa nas
>Discovery da
> vida. Dizem que é bem pior que a tal viúva negra. Boa sorte e faça seu
> testamento! :] Murilo SP 05/jan/2005
>
> >From: "E m i l i a n o C h e m e l l o" <chemelloe@yahoo.com.br>
> >Reply-To: ciencialist@yahoogrupos.com.br
> >To: <ciencialist@yahoogrupos.com.br>
> >Subject: [ciencialist] aranhas...
> >Date: Wed, 5 Jan 2005 10:14:55 -0200
> >
> >Alguém?
> >
> >[ ] 's do Emiliano Chemello
> >emiliano@quimica.net
> >http://www.quimica.net/emiliano
> >http://www.ucs.br/ccet/defq/naeq
> >
> >
> >Contato Naeq:
> >Nome: Camila
> >Email: camila_alves19@hotmail.com
> >Telefone: Dúvidas!!
> >Mensagem: Há alguns dias eu venho pesquisando sobre aranhas, no meu
>jardim
> >tem 2 aranhas construtoras de teia, elas são sedentárias estão sempre
> >paradas, posicionam suas patas dianteiras para cima e suas traseiras
>para
> >baixo fica parecendo que só são 4, suas patas são listradas de amarelo,
>uma
> >listra grande amarela pode ser identificada em seu abdome. Mas a minha
> >dúvida é se elas são venenosas. Em certos sites podemos encontrar que
>as
> >aranhas clinicamente perigosas não são construtoras de teias e sim
> >errantes,
> >mas eu queria saber mais sobre essas aranhas do meu jardim, pois não
> >encontro nada em sites, apenas o que tenho são essas características.
>Se
> >puderem me ajudar, estou estudando para fazer faculdade de biologia.
>Minha
> >curiosidade sobre assuntos biologicos é grande.
> >Obrigada!
> >
>
>
>
>
> ##### ##### #####
>
> Para saber mais visite
> http://www.ciencialist.hpg.ig.com.br
>
>
> ##### ##### ##### #####
>
>
> Yahoo! Grupos, um serviço oferecido por:
>
> São Paulo Rio de Janeiro Curitiba Porto Alegre Belo
>Horizonte Brasília
>
>
>
>
>------------------------------------------------------------------------------
> Links do Yahoo! Grupos
>
> a.. Para visitar o site do seu grupo na web, acesse:
> http://br.groups.yahoo.com/group/ciencialist/
>
> b.. Para sair deste grupo, envie um e-mail para:
> ciencialist-unsubscribe@yahoogrupos.com.br
>
> c.. O uso que você faz do Yahoo! Grupos está sujeito aos Termos do
>Serviço do Yahoo!.
>
>
>
>[As partes desta mensagem que não continham texto foram removidas]
>




SUBJECT: Re: [ciencialist] aranhas...
FROM: "Amauri Jr" <amaurijunior2@yahoo.com.br>
TO: <ciencialist@yahoogrupos.com.br>
DATE: 06/01/2005 23:03

Oi Murilo

São umas que fazem teias e são meia marron com a traseira pretinha...

Amauri
----- Original Message -----
From: murilo filo
To: ciencialist@yahoogrupos.com.br
Sent: Thursday, January 06, 2005 11:01 PM
Subject: Re: [ciencialist] aranhas...


Devem ser as papa-moscas, pequenas e rápidas; concorrentes das lagartixas!
Aquí em casa, ninguém mata... inofensivas e ainda comem pernilongos!
Ou será que vc está falando daquelas magrelas de perninhas fininhas e que
têm tremedeiras?
Nestas não tenho a mínima tesão! Não gosto! Sem graça e muito paradonas.
Possuem um veneno forte, mas não conseguem inocular, parece.
Antes, eu também repetia com outros, gozações sobre a ''cultura inútil'',
até eu descobrir as delícias dos produtivos novos enlaces cerebrais...
My name is Discovery... abr/M.

>From: "Amauri Jr" <amaurijunior2@yahoo.com.br>
>Reply-To: ciencialist@yahoogrupos.com.br
>To: <ciencialist@yahoogrupos.com.br>
>Subject: Re: [ciencialist] aranhas...
>Date: Thu, 6 Jan 2005 19:19:30 -0200
>
>E esses que ficam na parece ou de baixo das pias do banheiro??
>
>Amauri
> ----- Original Message -----
> From: murilo filo
> To: ciencialist@yahoogrupos.com.br
> Sent: Wednesday, January 05, 2005 10:46 PM
> Subject: RE: [ciencialist] aranhas...
>
>
> Se as teias são ''aéreas'', muito fortes, grandes e de cor meio dourada,
>e
> se esta aranha, com uns 5cm, quando ameaçada, mudar de cor, trata-se a
> aranha lôbo, que é boazinha, bonitinha, decorativa e útil.
> Se a teia for no chão, meio embolada e aberta como um funil na boca de
>um
> pequeno túnel, trata-se da ''armadeira'', perigosa e famosa nas
>Discovery da
> vida. Dizem que é bem pior que a tal viúva negra. Boa sorte e faça seu
> testamento! :] Murilo SP 05/jan/2005
>
> >From: "E m i l i a n o C h e m e l l o" <chemelloe@yahoo.com.br>
> >Reply-To: ciencialist@yahoogrupos.com.br
> >To: <ciencialist@yahoogrupos.com.br>
> >Subject: [ciencialist] aranhas...
> >Date: Wed, 5 Jan 2005 10:14:55 -0200
> >
> >Alguém?
> >
> >[ ] 's do Emiliano Chemello
> >emiliano@quimica.net
> >http://www.quimica.net/emiliano
> >http://www.ucs.br/ccet/defq/naeq
> >
> >
> >Contato Naeq:
> >Nome: Camila
> >Email: camila_alves19@hotmail.com
> >Telefone: Dúvidas!!
> >Mensagem: Há alguns dias eu venho pesquisando sobre aranhas, no meu
>jardim
> >tem 2 aranhas construtoras de teia, elas são sedentárias estão sempre
> >paradas, posicionam suas patas dianteiras para cima e suas traseiras
>para
> >baixo fica parecendo que só são 4, suas patas são listradas de amarelo,
>uma
> >listra grande amarela pode ser identificada em seu abdome. Mas a minha
> >dúvida é se elas são venenosas. Em certos sites podemos encontrar que
>as
> >aranhas clinicamente perigosas não são construtoras de teias e sim
> >errantes,
> >mas eu queria saber mais sobre essas aranhas do meu jardim, pois não
> >encontro nada em sites, apenas o que tenho são essas características.
>Se
> >puderem me ajudar, estou estudando para fazer faculdade de biologia.
>Minha
> >curiosidade sobre assuntos biologicos é grande.
> >Obrigada!
> >
>
>
>
>
> ##### ##### #####
>
> Para saber mais visite
> http://www.ciencialist.hpg.ig.com.br
>
>
> ##### ##### ##### #####
>
>
> Yahoo! Grupos, um serviço oferecido por:
>
> São Paulo Rio de Janeiro Curitiba Porto Alegre Belo
>Horizonte Brasília
>
>
>
>
>------------------------------------------------------------------------------
> Links do Yahoo! Grupos
>
> a.. Para visitar o site do seu grupo na web, acesse:
> http://br.groups.yahoo.com/group/ciencialist/
>
> b.. Para sair deste grupo, envie um e-mail para:
> ciencialist-unsubscribe@yahoogrupos.com.br
>
> c.. O uso que você faz do Yahoo! Grupos está sujeito aos Termos do
>Serviço do Yahoo!.
>
>
>
>[As partes desta mensagem que não continham texto foram removidas]
>




##### ##### #####

Para saber mais visite
http://www.ciencialist.hpg.ig.com.br


##### ##### ##### #####


Yahoo! Grupos, um serviço oferecido por:
PUBLICIDADE




------------------------------------------------------------------------------
Links do Yahoo! Grupos

a.. Para visitar o site do seu grupo na web, acesse:
http://br.groups.yahoo.com/group/ciencialist/

b.. Para sair deste grupo, envie um e-mail para:
ciencialist-unsubscribe@yahoogrupos.com.br

c.. O uso que você faz do Yahoo! Grupos está sujeito aos Termos do Serviço do Yahoo!.



[As partes desta mensagem que não continham texto foram removidas]



SUBJECT: Re: [ciencialist] Re: Fw: Teoria da Relatividade Especial
FROM: "Prof. JC" <profjc2003@yahoo.com.br>
TO: <ciencialist@yahoogrupos.com.br>
DATE: 06/01/2005 23:04

dfahlb,

A velocidade do som independe da velocidade da fonte ou do receptor e
depende apenas do meio de propagação e da velocidade relativa desse meio em
relação a um dado referencial. Acho que você fez alguma confusão ai... Ou
será que fui eu?

O que depende da velocidade da fonte e do emissor em relação ao meio de
propagação do som é a freqüência (ou, equivalentemente, o comprimento de
onda) percebida pelo receptor.

Abraços,
Prof. JC



----- Original Message -----
From: "dfahlb" <dfahlb@yahoo.com>
To: <ciencialist@yahoogrupos.com.br>
Sent: Thursday, January 06, 2005 6:29 PM
Subject: [ciencialist] Re: Fw: Teoria da Relatividade Especial




--- Em ciencialist@yahoogrupos.com.br, "Luiz Ferraz Netto"
<leobarretos@u...> escreveu
> Vamos responder? ... educadamente ...

Já que ninguém quis respondera msg
http://br.groups.yahoo.com/group/ciencialist/message/43472 , deixe-me
tentar.

> Assunto: Teoria da Relatividade Especial
>
> Jaime Pacheco Filho
>
> jaime-pacheco@u...
>
> Escolaridade: superior (Economia)
>
>
>
> Dúvida: Entendo que o Efeito Doppler é uma conseqüência da
>independência da velocidade de uma onda (sonora ou luminosa) em
>relação ao movimento do emissor e do receptor no meio de propagação.

Na verdade é o contrário: o Efeito Doppler é uma consequência da
DEPENDÊNCIA da velocidade de uma onda em relação ao movimento do
emissor e do receptor no meio de propagação. A


>Então, por que a velocidade da luz (da qual não se conhece o meio de
>propagação) é especial e não a do som (do qual se sabe o meio de
>propagação)?

Não é que não se conhece, é que não existe tal meio. A luz se
movimenta no vácuo, por exemplo, aonde não existe meio material. Na
verdade, o meio material atrapalha a propagação da luz. A luz tem a
velocidade reduzida em meios materiais pois o material acaba por
aborsorvê-la e reemiti-la o que causa retardamento da velocidade (nos
meios opacos, a luz é absorvida pelo material e ou é re-emitida sob a
forma de outra frequência ou é transformada em calor - em movimento
dos átomos do meio).

Agora, por que a velocidade da luz é a mesma para todos os
observadores (independendo de do movimento deles em relação à luz ou
da fonte da luz em relação a eles) e não o som?

Bom, no caso do som, a velocidade não é a mesma pois depende da
velocidade da fonte e do observador em relação ao meio. Como a luz
não dependende de meio para se propagar (afinal se propaga no vácuo)
se a velocidade dela dependesse de algum fator, poderíamos encontrar
o referencial absoluto!!!

Vai um exemplo. Suponha que a velocidade da luz dependesse do
movimento do observador. Agora suponha que um observador, chamemo-lo
de O1, estivesse numa nave se movimentando com movimento uniforme e
retilíneo (velocidade constante, chamemo-la de v) ou seja, não há a
presença de campo gravitacional (ou seja sobre ele não é exercida a
força gravitacional) ou então que a resultante das forças que agem
sobre a nave é zero.

Continuando: como a luz se propaga no vácuo, vamos deixar nossa nave
se movimentando lá, no vácuo. Como o movimento é uniforme e retilínio
se o observador O1 fizesse QUALQUER experiência mecânica (por pêndulo
para oscilar, lançar objetos para cima, etc) ele NÃO VAI conseguir
dizer se está com alguma velocidade ou não, afinal o movimento
retilínio e uniforme é relativo pois depende de um ponto de
referência, ou referencial.

Se você se move, você se move em relação a que? Se não há nada por
perto, você não poderia dizer se está se movimentando com velocidade
retilínea e uniforme ou se não está. Mas, mais que isso.

Na verdade, tal observador se tivesse um ponto de referência, veria
esse ponto de referência se afastar dele e ele poderia afirmar que É
O PONTO DE REFERÊNCIA que se afasta. Outro observador, chamemo-lo de
O2, que estivesse no ponto de referência diria que não, que É A NAVE
que se afasta. Ambos estariam certos!!!! Afinal, o movimento uniforme
e retilíneo é relativo: depende do observador.

Agora, voltando a pergunta: por que a velocidade da luz é a mesma
para todos os observadores (independendo de do movimento deles em
relação à luz ou da fonte da luz em relação a eles)?

Imagine então O2, que está num ponto de referência, vê O1 (que está
na nave) nalgum instante, passar por ele e indo "da esquerda para a
direita". O1 diria discordia dizendo que foi O2 que passou vindo "da
direita para a esquerda". Como disse: ambos estariam certos pois o
movimento é relativo e NENHUMA experiência mecânica poderia dizer que
está num referencial privilegiado (referencial absoluto).

Então, O2, manda um feixe de luz ir de encontro com O1. A luz sai com
velocidade c de O2. Ela, em dado instante, ultrapassa O1. No instante
que a luz passa por O1, ele mede a velocidade dela. O1, então, mede a
velocidade (chamemo-la de w) com que a luz ultrapassa ele. Se ele
medisse e visse que desse w = c - v, ele, O1, saberia de cara que ele
está se movimentando! Ou seja, a luz tem que ultrapassá-lo com
velocidade c.

Essa idéia é comentada no livro "Evolução das Idéias da Física" de
Einstein e Infeld um livro usado no primeiro ano de Física em algumas
faculdades. É um livro de fácil leitura e acessível. Einstein não usa
nenhuma equação e explica ainda assim de maneira clara a Relatividade
no livro.

> Será que os efeitos previstos pela Teoria da
>Relatividade Especial como a alteração no ritmo de relógios e o
>encurtamento de réguas não são apenas aparentes, devidos ao Efeito
>Doppler?

Não, não são. São detectados experimentalmente. Até experiências em
aviões supersônicos (medidos os atrasos dos relógios nele) já foram
feitos.

Inclusive, como o Efeito Doppler é derivado das transformações de
Galileu, ele está incorreto, sobretudo para velocidades altas, ainda
que você possa usá-lo para velocidades baixas porque o erro é
desprezível.

O Efeito Doppler mais geral seria o Efeito Doppler Relativístico.
Inclusive, este último é que deve ser usado mesmo para ondas que não
precisam de meio para se propagar, como as ondas eletromagnéticas (a
luz).

>
> Se fôssemos todos cegos talvez a Teoria da Relatividade
se baseasse na velocidade do som.

Não, não seria. Se fôssemos cegos e conseguíssemos ir desenvolvendo
uma tecnologia cada vez mais precisa, veríamos que as transformações
de Galileu (as da Mecânica Clássica) não são válidas e são apenas uma
aproximação. Que quando estamos em velocidade baixa ela é
aproximadamente exata. Mas o erro dela é melhor percebido a altas
velocidades.

Se fôssemos cegos, talvez descobríssemos a luz pois quando fôssemos
medir a velocidade com que o calor se propaga por radiação é mais
rápido que a velocidade do som e mede c.

>
> Einstein explicou porque os relógios atrasam e as
>réguas encurtam quando são acelerados?

Sim, leia o livro "Evolução das Idéias da Física" de Einstein e
Infeld, é o livro mais fácil de entende a Relatividade para os não-
físicos.

A explicação é toda a que estamos tendo e mais algumas. Primeiro:
evidências experimentais mostraram que a velocidade da luz não
depende nem do movimento da fonte e não do observador em relação a
ela e Einstein foi o primeiro a admitir isso. Depois Einstein supôs
que não apenas as Leis da Mecânica, mas também do Eletromagnetismo
(experiências com luz) não poderiam definir o referencial absoluto,
no que ele sintetizou nos princípios:

1. As Leis da Física são invariantes em todos os sistemas inerciais.
Nenhum sistema inercial preferido existe. (O Princípio da
Relatividade)
2. A velocidade da luz no espaço livre tem o mesmo valor "c" em todos
os sistemas inerciais. (O Princípio da Constância da Velocidade da
Luz).

Aqui "sistemas" e "referenciais" são sinônimos.
O princípio 1 é o Princípio da Relatividade já pensado anteriormente
por Galileu, entretando, as transformações de Galileu, que mostram
que se muda de um sistema para outro, está errada. Os erros são tão
maiores quanto mais próximo se aproxima da velocidade da luz. Claro
que a velocidades baixas, os erros são desprezíveis.

Um livro que também é interessante de se olhar é: Introduction To
Special Relativity. Autor: Robert Resnick.
Usado: no 3.o e 4.o ano de Física (bacharelado e licenciatura,
respectivamente). Assunto: além de explicar vários conceitos da
Relatividade de Einstein, ele dá outras experiências que só a
Relatividade de Einstein explica além de discorrer brevemente sobre
algumas outras teorias rivais à Relatividade de Einstein mas que
falharam e aonde falharam.

Este livro, embora de leitura um pouco mais complexa que o "Evolução
das Idéias da Física" de Einstein e Infeld, vale a pena ser lido e
ainda assim, é de linguagem acessível.

Espero ter ajudado.
Abraços.

>
>
>
> Obrigado
>






##### ##### #####

Para saber mais visite
http://www.ciencialist.hpg.ig.com.br


##### ##### ##### #####
Links do Yahoo! Grupos












SUBJECT: Passarinhos incomodando - Alerta ao L.E.R. de Carvalho
FROM: "L.E.R.de Carvalho" <lecarvalho@infolink.com.br>
TO: ciencialist@yahoogrupos.com.br
CC: leobarretos@uol.com.br,grasdic@hotmail.com
DATE: 06/01/2005 23:18

At 22:16 6/1/2005, you wrote:
>Ooops L.E.R. de Carvalho,
>
>vc está forçando a barra. Que foi que a Natália lhe fez, nessa mensagem
>dirigida a mim, para que vc poste um comentário, pesado e rancoroso desses?
>Fica esse alerta. Entrarei em contato com Brudna e Mesquita. Nesse meio
>tempo, para não esticar o problema, solicito que não envie qqer mensagem
>ao C-list. Nem mesmo responda a essa mensagem de alerta.
>
>[]'
> ===========================
> Luiz Ferraz Netto [Léo]
> leobarretos@uol.com.br
> <http://www.feiradeciencias.com.br>http://www.feiradeciencias.com.br


Ops, digo eu.

Você tem que ver, com ela, se ela se ofendeu.

Eu imagino que ela gostou.

Só se você se ofendeu.
Mas aí já é outro assunto.

Luiz Eduardo

[As partes desta mensagem que não continham texto foram removidas]



SUBJECT: Re: [ciencialist] Analise de Carl Sagan por mim...
FROM: "Amauri Jr" <amaurijunior2@yahoo.com.br>
TO: <ciencialist@yahoogrupos.com.br>
DATE: 07/01/2005 12:29

Olá Homero

"risos..:-) Você vai analisar Sagan e detonar Bacon?..:-) Bem, boa sorte,gente bastante equipada e capaz tentou o mesmo,com um monte de dados e informação, e ainda assim, não foi fácil, espero que tenha maior sorte..:-)"

[A] Quem manda Bacon ficar no meio?...:) Obrigado, mas me diverti o bastante...:)

"Mas, ainda penso que está confundindo as bolas (um bocado). É fácil perceber que sua irritação contra americanos, os USA, gente de grana, poderosos em geral, está misturando seu raciocínio.Ou não escreveria coisas como essa:"

[A] Quem esta confundindo aqui é voce, eu apenas estou analisando os fatos; não estou nem um pouco irritado com os EUA, por mim eles passavam desapercebidos, mas tenho que ver tudo que é coisa deles. Sendo eles poderosos ou não, não me importa, Sagan poderia ser até coriano; mas seria analisado do mesmo jeito...:) Agora os EUA ter grana ou não, isso mostra o quanto a propaganda dar certo né?...:)

"Amury, ele não quer saber tudo isso (perguntas que deveriam ser feitas aos canalizadores, conforme o texto original). Ele pouco se importa com as respostas..:-) Ele pergunta para decidir se o canalizado é real ou fraude, engano ou disfunção mental. Qualquer um ao ler o trecho percebe isso, você deve estar muito irritado para não faze-lo..:-) Por exemplo, se eu estivesse presente na canalização de um espírito de soldado romano do tempo do império, gostaria de saber se ele pode falar em latin, ou ao menos compreender frases em latin. Não me interessa a resposta ou o que ele vai dizer, mas o fato de que sabe latin..:-) Um espirito milenar que pode se lembrar de sua vida em Roma imperial, suas andanças pelas ruas de Roma, mas não reconhece uma única palavra em latin me provoca muita desconfiança..;-)"

[A] Risos...:) Todo analista que refuta uma tese as pessoas pensam que o cara ta irritado, muitos texto de analise de filosofos que refutaram outro mostra isto como: "Kant se irrita com Hume" ou "Voltaire se irrita com Rosseau" e por ai vai...:) Não estou irritado com ele ém si, mas como uma pergunta dessas é feita num texto serio e muito bem elaborado...mas tambem descofio destes espiritos, muito embora outros, falam a lingua onde viveram sem muito problemas...:)

"Antes de concluir que o médium está realmente recebendo um espírito (concluir, não acreditar), Sagan quer ter mais dados, mais elementos. Como o médico que pede ao paciente que diga 33 e não tem O MENOR INTERESSE NO NÚMERO 33. Ou você pensa que ao ouvir 33 o médico diz "oh, puxa, verdade, é 33?"? Ou que vai transformar essa informação, o 33, em patrimônio?"

[A] Sim, isso que doutrinas serias fazem, procuram dados...:)

"O texto é claro. O argumento de Sagan é perfeito e eficaz. Pode até tentar refuta-lo, mas não é contestando o interesse dele nas informações..:-) São perguntas de teste, padrão em experimentos de pesquisa, não dados para "roubar" do médium e ficar rico..:-)"

{A] Eu disse isto?...:)

"u aconselharia que rele-se o texto de agan, com menos raiva, e, ou compreendesse o que ele defende, ou refutasse com algum argumento razoável. Sem isso, fica apenas a sensação de mau humor e disputa emocional com o autor..:-)"

[A] Eu vou e se me der na telha, terminarei meu texto sobre ciencia e espiritualidade; obrigado pela oportunidade de analisar e aprofundar no texto do grande Sagan...:)

Abraços
Amauri

----- Original Message -----
From: Oraculo
To: ciencialist@yahoogrupos.com.br
Sent: Thursday, January 06, 2005 5:36 PM
Subject: Re: [ciencialist] Analise de Carl Sagan por mim...


Olá Amaury

risos..:-) Você vai analisar Sagan e detonar Bacon?..:-) Bem, boa sorte,
gente bastante equipada e capaz tentou o mesmo, com um monte de dados e
informação, e ainda assim, não foi fácil, espero que tenha maior sorte..:-)

Mas, ainda penso que está confundindo as bolas (um bocado). É fácil perceber
que sua irritação contra americanos, os USA, gente de grana, poderosos em
geral, está misturando seu raciocínio. Ou não escreveria coisas como essa:

"Amaury:[A] Para que Sagan quer saber isso tudo? Para o ser humano
transformar em patrimônios e poucos poderem ver? "

Amury, ele não quer saber tudo isso (perguntas que deveriam ser feitas aos
canalizadores, conforme o texto original). Ele pouco se importa com as
respostas..:-) Ele pergunta para decidir se o canalizado é real ou fraude,
engano ou disfunção mental. Qualquer um ao ler o trecho percebe isso, você
deve estar muito irritado para não faze-lo..:-) Por exemplo, se eu estivesse
presente na canalização de um espírito de soldado romano do tempo do
império, gostaria de saber se ele pode falar em latin, ou ao menos
compreender frases em latin. Não me interessa a resposta ou o que ele vai
dizer, mas o fato de que sabe latin..:-) Um espirito milenar que pode se
lembrar de sua vida em Roma imperial, suas andanças pelas ruas de Roma, mas
não reconhece uma única palavra em latin me provoca muita desconfiança..;-)

Antes de concluir que o medium está realmetne recebendo um espírito
(concluir, não acreditar), Sagan quer ter mais dados, mais elementos. Como o
médico que pede ao paciente que diga 33 e não tem O MENOR INTERESSE NO
NÚMERO 33. Ou você pensa que ao ouvir 33 o médico diz "oh, puxa, verdade, é
33?"? Ou que vai transformar essa informação, o 33, em patrimonio?

O texto é claro. O argumento de Sagan é perfeito e eficaz. Pode até tentar
refuta-lo, mas não é contestando o interesse dele nas informações..:-) São
perguntas de teste, padrão em experimentos de pesquisa, não dados para
"roubar" do médium e ficar rico..:-)

Todo o resto de seu texto segue essa linha, diatribes sem muita ligação com
o texto de Sagan. É dificil refutar algo assim e nem vou tentar. Basta ler o
texto..:-)

Eu aconselharia que rele-se o texto de Sagan, com menos raiva, e, ou
compreendesse o que ele defende, ou refutasse com algum argumento razoável.
Sem isso, fica apenas a sensação de mau humor e disputa emocional com o
autor..:-)

Um abraço.

Homero






##### ##### #####

Para saber mais visite
http://www.ciencialist.hpg.ig.com.br


##### ##### ##### #####


Yahoo! Grupos, um serviço oferecido por:
PUBLICIDADE




------------------------------------------------------------------------------
Links do Yahoo! Grupos

a.. Para visitar o site do seu grupo na web, acesse:
http://br.groups.yahoo.com/group/ciencialist/

b.. Para sair deste grupo, envie um e-mail para:
ciencialist-unsubscribe@yahoogrupos.com.br

c.. O uso que você faz do Yahoo! Grupos está sujeito aos Termos do Serviço do Yahoo!.



[As partes desta mensagem que não continham texto foram removidas]



SUBJECT: Re: Passarinhos incomodando
FROM: Maria Natália <grasdic@hotmail.com>
TO: ciencialist@yahoogrupos.com.br
DATE: 07/01/2005 13:27


Luiz Eduardo de Carvalho:

Bem me parecia que havia qualquer pedra em seu sapato.
A frase "Será que alguma delas serve pra tu ?" está incorrecta.
Deveria dizer Será que alguma delas serve para ti?
Agoras vamos aos pombos.
O pombos e a sua existência em cidades é assunto de comunidade e
Cãmara municipal. Por aqui, Europa e devido à tal acidez que falei
do guano estragam monumentos, etc. Que fazemos para não ter pombos a
estragar pintura de carro? Se lhes dá a pílula e com peso conta e
medida pois não se pretende atacar essas aves que tão românticas são
e se prestam a quadros bucólicos com namorados etc. Isto mete
investigação e veterinário. Nos monumentos nacionais usam-se uns
espetos metálicos ou de polímero(de baixo parecem picos estilo
palito) e que não permitem que ele pouse e passe lá anoite ou até
faço ninho.
Se realmente pertence a Sociedade Columbófila ou à sociedade
protectora dos animais devia dizer-mo de outro modo explicando e
ensinado-me o porquê de meu erro. Afinal estamos aqui para aprender.
Com respeito a temas de outras listas e para não ser aqui OT lhe
digo que quando tenho problemas com alguém os discuto pessoalmente
com o indivíduo em questão. Costumo apanhar combóio ou avião e vou
acertar contas com a pessoa.Repare que muito tenho visto em listas
no tocante a comportamentos. Avalio listas e sou moderadora de 3
sabendo portanto que as acções ficam com quem as pratica.
Em pvt envie-me sua morada em SP pois terei gosto em "acertar
contas" com você também. Aliás avisarei todos para nos podemos
reunir e lavarmos a roupa suja de modo digno como professores que
somos.
Maria Natália Botelho
>

> NO ORKUT, EU SÓ CONHEÇO DUAS COMUNIDADES.
>
> UMA É...
> GENTE QUE ANDA PELA CASA ESCOVANDO OS DENTES.
>
> A OUTRA É...
> GENTE QUE FOI EXPULSA DE TUDO QUANTO É LISTA E SE JUNTOU AQUI.
>
>
> Será que alguma delas serve pra tu ?
>
> Luiz Eduardo
>
> [As partes desta mensagem que não continham texto foram removidas]





SUBJECT: Passarinhos não incomodando MAIS
FROM: "L.E.R.de Carvalho" <lecarvalho@infolink.com.br>
TO: ciencialist@yahoogrupos.com.br
DATE: 07/01/2005 13:41

At 13:27 7/1/2005, you wrote:

>Luiz Eduardo de Carvalho:
>
>Bem me parecia que havia qualquer pedra em seu sapato.
>Maria Natália Botelho



Oh, Natalia.
Eu estou até descalço.
E escrevi errado justamente para realçar a sinalização que não estava
falando sério.

Estava apenas brincando com essa sua música, de que te expulsam das listas,
e que haveria uma comunidade dessas no Orkut.

Não era minha intenção lhe incomodar.
Pensei apenas que você pudesse ver isso com humor, e aproveitar
positivamente esse "levantamento de bola".

Se não foi o caso, me desculpe.
Não foi minha intenção causar disturbio.

Procurarei evitar a repetição desse tipo de mal-entendido.
Mas o melhor recurso para evitar isso é, da próxima vez, não falar nada.

Uma pena.

Luiz Eduardo

[As partes desta mensagem que não continham texto foram removidas]



SUBJECT: Ciência do nojo
FROM: "brudna" <lrb@iq.ufrgs.br>
TO: ciencialist@yahoogrupos.com.br
DATE: 07/01/2005 15:10


Entre no site

http://www.bbc.co.uk/science/humanbody/mind/disgust

e responda as perguntas (em inglês). É uma ´pesquisa´ para verificar o
nojo. Dá pra perceber que as respostas são meio óbvias. Mas é
interessante.

Até
Luís Brudna
http://antiparticula.webcindario.com







SUBJECT: Re: Fw: Teoria da Relatividade Especial
FROM: "dfahlb" <dfahlb@yahoo.com>
TO: ciencialist@yahoogrupos.com.br
DATE: 07/01/2005 15:48


--- Em ciencialist@yahoogrupos.com.br, "Prof. JC" <profjc2003@y...>
escreveu
> dfahlb,
>
> A velocidade do som independe da velocidade da fonte ou do receptor
e
> depende apenas do meio de propagação e da velocidade relativa desse
meio em
> relação a um dado referencial. Acho que você fez alguma confusão
ai... Ou
> será que fui eu?

Mas depende. Veja, se depende do meio de propagação, vai depender de
como os observadores (receptores, no caso) estão se movendo em
relação a esse meio.

Pense num jato supersônico: quando ele passa da velocidade do som,
nenhum som chega até ele. É claro que o som precisa de um meio para
se propagar. Mas eu me referia é do movimento dos observadores e
fontes com relação à esse meio ou à fonte. É claro que a fonte se
movimentando no meio mudará a frequência/comprimento de onda para um
observador estacionário ao meio.

>
> O que depende da velocidade da fonte e do emissor em relação ao
meio de
> propagação do som é a freqüência (ou, equivalentemente, o
comprimento de
> onda) percebida pelo receptor.
>
> Abraços,
> Prof. JC

O que você está a discutir agora é o Efeito Doppler. O Efeito Doppler
te explica justamente isso: de como a frequência ou o comprimento de
onda mudam com a velocidade da fonte ou do receptor em relação ao
meio. O Efeito Doppler é derivado das Transformações de Galileu, etc.

O Efeito Doppler serve para explicar o porquê de quando o jato
supersônico (do exemplo acima) ao atravessar a barreira do som, o
piloto e passageiros presentes ouvem um estrondol

Abraços.

>
>






SUBJECT: Fw: d�vida
FROM: "Luiz Ferraz Netto" <leobarretos@uol.com.br>
TO: "ciencialist" <ciencialist@yahoogrupos.com.br>
DATE: 07/01/2005 20:33

Refazendo as perguntas do consulente:

1) Qual a massa de 1 m^3 de GLP? Ao cabo, qual a massa específica do GLP à temperatura ambiente?

2) 13 kg de GLP (contidos no tradicional bujão) transformam-se em 'quantos m^3' a CNTP?

[]'
===========================
Luiz Ferraz Netto [Léo]
leobarretos@uol.com.br
http://www.feiradeciencias.com.br
===========================
-----Mensagem Original-----
De: Nenegrando
Para: leobarretos@uol.com.br
Enviada em: quinta-feira, 6 de janeiro de 2005 16:25
Assunto: dúvida


Olá professor!! Tudo tranquilo neste início de ano!!
Depois das historias com o p.s.i., tenho outra dúvida:
Um m3 (metro cúbico) de glp são quantos kg? Ou, em um botijão de 13 kg tem quantos m3?
Preciso descobrir porque acho que estou sendo passado pra tráz!!
Não consigo achar tabelas de conversão!
Responde se tiver um tempinho, vou continuar pesquisando pra ver se descubro.
um abraço...
obrigado


----------

No virus found in this outgoing message.
Checked by AVG Anti-Virus.
Version: 7.0.300 / Virus Database: 265.6.9 - Release Date: 06/01/2005


[As partes desta mensagem que não continham texto foram removidas]



SUBJECT: Re: [ciencialist] aranhas...
FROM: "murilo filo" <avalanchedrive@hotmail.com>
TO: ciencialist@yahoogrupos.com.br
DATE: 07/01/2005 22:46

Amauri,
oi.
Sua descritiva está por demais vaga... seja + específico:
- tamanho?
- pernas finas?
- teias grandes?
Se forem miudinhas, considere-me fora do caso. Muito comuns!
Vc disse que estavam debaixo da pia e nas paredes... hmmm.... acho bom,
também, vc marcar uma reunião com a faxineira... eu sei, a ecologia
tropical ataca! :]
abr/M.

>From: "Amauri Jr" <amaurijunior2@yahoo.com.br>
>Reply-To: ciencialist@yahoogrupos.com.br
>To: <ciencialist@yahoogrupos.com.br>
>Subject: Re: [ciencialist] aranhas...
>Date: Thu, 6 Jan 2005 23:03:51 -0200
>
>Oi Murilo
>
>São umas que fazem teias e são meia marron com a traseira pretinha...
>
>Amauri
> ----- Original Message -----
> From: murilo filo
> To: ciencialist@yahoogrupos.com.br
> Sent: Thursday, January 06, 2005 11:01 PM
> Subject: Re: [ciencialist] aranhas...
>
>
> Devem ser as papa-moscas, pequenas e rápidas; concorrentes das
>lagartixas!
> Aquí em casa, ninguém mata... inofensivas e ainda comem pernilongos!
> Ou será que vc está falando daquelas magrelas de perninhas fininhas e
>que
> têm tremedeiras?
> Nestas não tenho a mínima tesão! Não gosto! Sem graça e muito paradonas.
> Possuem um veneno forte, mas não conseguem inocular, parece.
> Antes, eu também repetia com outros, gozações sobre a ''cultura
>inútil'',
> até eu descobrir as delícias dos produtivos novos enlaces cerebrais...
> My name is Discovery... abr/M.
>
> >From: "Amauri Jr" <amaurijunior2@yahoo.com.br>
> >Reply-To: ciencialist@yahoogrupos.com.br
> >To: <ciencialist@yahoogrupos.com.br>
> >Subject: Re: [ciencialist] aranhas...
> >Date: Thu, 6 Jan 2005 19:19:30 -0200
> >
> >E esses que ficam na parece ou de baixo das pias do banheiro??
> >
> >Amauri
> > ----- Original Message -----
> > From: murilo filo
> > To: ciencialist@yahoogrupos.com.br
> > Sent: Wednesday, January 05, 2005 10:46 PM
> > Subject: RE: [ciencialist] aranhas...
> >
> >
> > Se as teias são ''aéreas'', muito fortes, grandes e de cor meio
>dourada,
> >e
> > se esta aranha, com uns 5cm, quando ameaçada, mudar de cor, trata-se
>a
> > aranha lôbo, que é boazinha, bonitinha, decorativa e útil.
> > Se a teia for no chão, meio embolada e aberta como um funil na boca
>de
> >um
> > pequeno túnel, trata-se da ''armadeira'', perigosa e famosa nas
> >Discovery da
> > vida. Dizem que é bem pior que a tal viúva negra. Boa sorte e faça
>seu
> > testamento! :] Murilo SP 05/jan/2005
> >
> > >From: "E m i l i a n o C h e m e l l o" <chemelloe@yahoo.com.br>
> > >Reply-To: ciencialist@yahoogrupos.com.br
> > >To: <ciencialist@yahoogrupos.com.br>
> > >Subject: [ciencialist] aranhas...
> > >Date: Wed, 5 Jan 2005 10:14:55 -0200
> > >
> > >Alguém?
> > >
> > >[ ] 's do Emiliano Chemello
> > >emiliano@quimica.net
> > >http://www.quimica.net/emiliano
> > >http://www.ucs.br/ccet/defq/naeq
> > >
> > >
> > >Contato Naeq:
> > >Nome: Camila
> > >Email: camila_alves19@hotmail.com
> > >Telefone: Dúvidas!!
> > >Mensagem: Há alguns dias eu venho pesquisando sobre aranhas, no meu
> >jardim
> > >tem 2 aranhas construtoras de teia, elas são sedentárias estão
>sempre
> > >paradas, posicionam suas patas dianteiras para cima e suas
>traseiras
> >para
> > >baixo fica parecendo que só são 4, suas patas são listradas de
>amarelo,
> >uma
> > >listra grande amarela pode ser identificada em seu abdome. Mas a
>minha
> > >dúvida é se elas são venenosas. Em certos sites podemos encontrar
>que
> >as
> > >aranhas clinicamente perigosas não são construtoras de teias e sim
> > >errantes,
> > >mas eu queria saber mais sobre essas aranhas do meu jardim, pois
>não
> > >encontro nada em sites, apenas o que tenho são essas
>características.
> >Se
> > >puderem me ajudar, estou estudando para fazer faculdade de
>biologia.
> >Minha
> > >curiosidade sobre assuntos biologicos é grande.
> > >Obrigada!
> > >
> >
> >
> >
> >
> > ##### ##### #####
> >
> > Para saber mais visite
> > http://www.ciencialist.hpg.ig.com.br
> >
> >
> > ##### ##### ##### #####
> >
> >
> > Yahoo! Grupos, um serviço oferecido por:
> >
> > São Paulo Rio de Janeiro Curitiba Porto Alegre
>Belo
> >Horizonte Brasília
> >
> >
> >
> >
>
> >------------------------------------------------------------------------------
> > Links do Yahoo! Grupos
> >
> > a.. Para visitar o site do seu grupo na web, acesse:
> > http://br.groups.yahoo.com/group/ciencialist/
> >
> > b.. Para sair deste grupo, envie um e-mail para:
> > ciencialist-unsubscribe@yahoogrupos.com.br
> >
> > c.. O uso que você faz do Yahoo! Grupos está sujeito aos Termos do
> >Serviço do Yahoo!.
> >
> >
> >
> >[As partes desta mensagem que não continham texto foram removidas]
> >
>
>
>
>
> ##### ##### #####
>
> Para saber mais visite
> http://www.ciencialist.hpg.ig.com.br
>
>
> ##### ##### ##### #####
>
>
> Yahoo! Grupos, um serviço oferecido por:
> PUBLICIDADE
>
>
>
>
>------------------------------------------------------------------------------
> Links do Yahoo! Grupos
>
> a.. Para visitar o site do seu grupo na web, acesse:
> http://br.groups.yahoo.com/group/ciencialist/
>
> b.. Para sair deste grupo, envie um e-mail para:
> ciencialist-unsubscribe@yahoogrupos.com.br
>
> c.. O uso que você faz do Yahoo! Grupos está sujeito aos Termos do
>Serviço do Yahoo!.
>
>
>
>[As partes desta mensagem que não continham texto foram removidas]
>




SUBJECT: Re: [ciencialist] aranhas...
FROM: "Amauri Jr" <amaurijunior2@yahoo.com.br>
TO: <ciencialist@yahoogrupos.com.br>
DATE: 07/01/2005 22:48

Oi Murilo

São Miudinhas e tecem muita teia....são meias morronzinhas e ficam no banheiro todo.

Abr
Amauri
----- Original Message -----
From: murilo filo
To: ciencialist@yahoogrupos.com.br
Sent: Friday, January 07, 2005 10:46 PM
Subject: Re: [ciencialist] aranhas...


Amauri,
oi.
Sua descritiva está por demais vaga... seja + específico:
- tamanho?
- pernas finas?
- teias grandes?
Se forem miudinhas, considere-me fora do caso. Muito comuns!
Vc disse que estavam debaixo da pia e nas paredes... hmmm.... acho bom,
também, vc marcar uma reunião com a faxineira... eu sei, a ecologia
tropical ataca! :]
abr/M.

>From: "Amauri Jr" <amaurijunior2@yahoo.com.br>
>Reply-To: ciencialist@yahoogrupos.com.br
>To: <ciencialist@yahoogrupos.com.br>
>Subject: Re: [ciencialist] aranhas...
>Date: Thu, 6 Jan 2005 23:03:51 -0200
>
>Oi Murilo
>
>São umas que fazem teias e são meia marron com a traseira pretinha...
>
>Amauri
> ----- Original Message -----
> From: murilo filo
> To: ciencialist@yahoogrupos.com.br
> Sent: Thursday, January 06, 2005 11:01 PM
> Subject: Re: [ciencialist] aranhas...
>
>
> Devem ser as papa-moscas, pequenas e rápidas; concorrentes das
>lagartixas!
> Aquí em casa, ninguém mata... inofensivas e ainda comem pernilongos!
> Ou será que vc está falando daquelas magrelas de perninhas fininhas e
>que
> têm tremedeiras?
> Nestas não tenho a mínima tesão! Não gosto! Sem graça e muito paradonas.
> Possuem um veneno forte, mas não conseguem inocular, parece.
> Antes, eu também repetia com outros, gozações sobre a ''cultura
>inútil'',
> até eu descobrir as delícias dos produtivos novos enlaces cerebrais...
> My name is Discovery... abr/M.
>
> >From: "Amauri Jr" <amaurijunior2@yahoo.com.br>
> >Reply-To: ciencialist@yahoogrupos.com.br
> >To: <ciencialist@yahoogrupos.com.br>
> >Subject: Re: [ciencialist] aranhas...
> >Date: Thu, 6 Jan 2005 19:19:30 -0200
> >
> >E esses que ficam na parece ou de baixo das pias do banheiro??
> >
> >Amauri
> > ----- Original Message -----
> > From: murilo filo
> > To: ciencialist@yahoogrupos.com.br
> > Sent: Wednesday, January 05, 2005 10:46 PM
> > Subject: RE: [ciencialist] aranhas...
> >
> >
> > Se as teias são ''aéreas'', muito fortes, grandes e de cor meio
>dourada,
> >e
> > se esta aranha, com uns 5cm, quando ameaçada, mudar de cor, trata-se
>a
> > aranha lôbo, que é boazinha, bonitinha, decorativa e útil.
> > Se a teia for no chão, meio embolada e aberta como um funil na boca
>de
> >um
> > pequeno túnel, trata-se da ''armadeira'', perigosa e famosa nas
> >Discovery da
> > vida. Dizem que é bem pior que a tal viúva negra. Boa sorte e faça
>seu
> > testamento! :] Murilo SP 05/jan/2005
> >
> > >From: "E m i l i a n o C h e m e l l o" <chemelloe@yahoo.com.br>
> > >Reply-To: ciencialist@yahoogrupos.com.br
> > >To: <ciencialist@yahoogrupos.com.br>
> > >Subject: [ciencialist] aranhas...
> > >Date: Wed, 5 Jan 2005 10:14:55 -0200
> > >
> > >Alguém?
> > >
> > >[ ] 's do Emiliano Chemello
> > >emiliano@quimica.net
> > >http://www.quimica.net/emiliano
> > >http://www.ucs.br/ccet/defq/naeq
> > >
> > >
> > >Contato Naeq:
> > >Nome: Camila
> > >Email: camila_alves19@hotmail.com
> > >Telefone: Dúvidas!!
> > >Mensagem: Há alguns dias eu venho pesquisando sobre aranhas, no meu
> >jardim
> > >tem 2 aranhas construtoras de teia, elas são sedentárias estão
>sempre
> > >paradas, posicionam suas patas dianteiras para cima e suas
>traseiras
> >para
> > >baixo fica parecendo que só são 4, suas patas são listradas de
>amarelo,
> >uma
> > >listra grande amarela pode ser identificada em seu abdome. Mas a
>minha
> > >dúvida é se elas são venenosas. Em certos sites podemos encontrar
>que
> >as
> > >aranhas clinicamente perigosas não são construtoras de teias e sim
> > >errantes,
> > >mas eu queria saber mais sobre essas aranhas do meu jardim, pois
>não
> > >encontro nada em sites, apenas o que tenho são essas
>características.
> >Se
> > >puderem me ajudar, estou estudando para fazer faculdade de
>biologia.
> >Minha
> > >curiosidade sobre assuntos biologicos é grande.
> > >Obrigada!
> > >
> >
> >
> >
> >
> > ##### ##### #####
> >
> > Para saber mais visite
> > http://www.ciencialist.hpg.ig.com.br
> >
> >
> > ##### ##### ##### #####
> >
> >
> > Yahoo! Grupos, um serviço oferecido por:
> >
> > São Paulo Rio de Janeiro Curitiba Porto Alegre
>Belo
> >Horizonte Brasília
> >
> >
> >
> >
>
> >------------------------------------------------------------------------------
> > Links do Yahoo! Grupos
> >
> > a.. Para visitar o site do seu grupo na web, acesse:
> > http://br.groups.yahoo.com/group/ciencialist/
> >
> > b.. Para sair deste grupo, envie um e-mail para:
> > ciencialist-unsubscribe@yahoogrupos.com.br
> >
> > c.. O uso que você faz do Yahoo! Grupos está sujeito aos Termos do
> >Serviço do Yahoo!.
> >
> >
> >
> >[As partes desta mensagem que não continham texto foram removidas]
> >
>
>
>
>
> ##### ##### #####
>
> Para saber mais visite
> http://www.ciencialist.hpg.ig.com.br
>
>
> ##### ##### ##### #####
>
>
> Yahoo! Grupos, um serviço oferecido por:
> PUBLICIDADE
>
>
>
>
>------------------------------------------------------------------------------
> Links do Yahoo! Grupos
>
> a.. Para visitar o site do seu grupo na web, acesse:
> http://br.groups.yahoo.com/group/ciencialist/
>
> b.. Para sair deste grupo, envie um e-mail para:
> ciencialist-unsubscribe@yahoogrupos.com.br
>
> c.. O uso que você faz do Yahoo! Grupos está sujeito aos Termos do
>Serviço do Yahoo!.
>
>
>
>[As partes desta mensagem que não continham texto foram removidas]
>




##### ##### #####

Para saber mais visite
http://www.ciencialist.hpg.ig.com.br


##### ##### ##### #####


Yahoo! Grupos, um serviço oferecido por:







------------------------------------------------------------------------------
Links do Yahoo! Grupos

a.. Para visitar o site do seu grupo na web, acesse:
http://br.groups.yahoo.com/group/ciencialist/

b.. Para sair deste grupo, envie um e-mail para:
ciencialist-unsubscribe@yahoogrupos.com.br

c.. O uso que você faz do Yahoo! Grupos está sujeito aos Termos do Serviço do Yahoo!.



[As partes desta mensagem que não continham texto foram removidas]



SUBJECT: É o conhecimento perigoso?
FROM: "Amauri Jr" <amaurijunior2@yahoo.com.br>
TO: <Euthymia@yahoogrupos.com.br>, <elio_mollo_k@hotmail.com>, "Lucimary Vargas" <sangalli@uai.com.br>, "Laracna" <laracna@pop.com.br>, "kathleen_mel" <kathleen_mel@uol.com.br>, <Conversa_de_Botequim@yahoogrupos.com.br>, <ciencialist@yahoogrupos.com.br>, <acropolis@yahoogrupos.com.br>
DATE: 07/01/2005 23:25


� o conhecimento perigoso?
Fronteiras entre ci�ncia, tecnologia e �tica
Orlando Tambosi


1. As feridas da modernidade e a antici�ncia


O tra�o mais profundo e mais perturbador de nossa �poca � a dissocia��o de fato e valor, ser e dever ser, ou f�sica e �tica, conhecimento da realidade e atribui��o de sentido � vida. As ci�ncias descrevem e conhecem o mundo tal qual �, mas calam sobre as ang�stias humanas, tornando o homem praticamente um acidente no cosmo. Despertando de seu sonho milenar, como diz o bi�logo Monod, o ser humano agora "sabe que, como um cigano, est� � margem do Universo onde deve viver. Um Universo surdo � sua m�sica, indiferente �s suas esperan�as, como a seus sofrimentos ou a seus crimes". Ele sabe que "est� sozinho na imensid�o indiferente do Universo, de onde emergiu por acaso. N�o mais do que seu destino, seu dever n�o est� escrito em lugar algum" (Monod: 1989, p. 190-8).

Atrav�s da ci�ncia, a modernidade rompeu a "alian�a anim�stica" entre homem e Natureza, calcada exatamente na identifica��o de fato e valor - fundamento da vis�o antropoc�ntrica do mundo. A cosmologia medieval (aristot�lico-crist�) realizava a coincid�ncia plena disso que, para n�s, � dividido: conhecimento da realidade e compreens�o do "sentido" da nossa vida � sua destina��o ou valor � eram uma s� coisa. Por mais de dois mil anos, a metaf�sica (o nome remete, como se sabe, ao conhecimento do transcendente ou do supra-sens�vel) sustentou a separa��o entre mundo terrestre e mundo celeste: embaixo, o reino do ef�mero, do nascer e do perecer; no alto, com suas esferas perfeitas, o reino do divino, do incorrupt�vel, do eterno, do verdadeiro Ser. Os n�veis de realidade exprimem ao mesmo tempo uma hierarquia de valores. A Terra, no centro, � o palco em que se desenrola o drama humano, em vista do qual o pr�prio cosmo foi criado.

A modernidade rompe essa imagem. A revolu��o astron�mica explode esse cosmo finito e fechado, revelando um universo de propor��es ilimitadas. A Terra j� n�o � mais o centro de nada. "� um ponto infinitesimal, uma min�scula ilha perdida num oceano sem praias, onde se contam bilh�es de gal�xias, cada uma delas com centenas de bilh�es de s�is. Explicar essa realidade em fun��o do homem, ou dela extrair um significado para a nossa exist�ncia, � simplesmente imposs�vel" (Colletti: 1989, p. III-IV, e 1996, p.15).

Depois que Cop�rnico arrancou o homem do centro do universo, Darwin obrigou-o a reconhecer que n�o passa de um ser entre outros no reino animal (competindo com as outras esp�cies e, freq�entemente, perdendo a luta para as mais microsc�picas). S�o duas feridas insan�veis que corroem o narcisismo humano, como definiu Freud, e que produzem mal-estar ainda hoje. Da� a hostilidade em rela��o �s ci�ncias e �s tecnologias, comum a algumas vertentes filos�ficas e tend�ncias culturais contempor�neas, particularmente as que se autodenominam "p�s-modernas". A antici�ncia, por sinal, encontra confort�vel abrigo nas ci�ncias sociais e humanidades, minadas pelo relativismo cognitivo e cultural; e, junto com as pseudoci�ncias, conta com generoso espa�o na m�dia.

Muito do que se produz nessas �reas � hostil a conceitos como "realidade", "objetividade", "verdade", fundamentais tanto � ci�ncia quanto ao jornalismo cient�fico. Para a cultura "p�s-moderna", o "real", os "fatos" que as ci�ncias buscam conhecer � e o jornalismo reportar - n�o passam de "constru��es intelectuais". Mero discurso ou "narrativa", a ci�ncia � ideol�gica, isto �, instrumento de domina��o de uma civiliza��o "branca", "euroc�ntrica", "opressora", "machista", "heterossexual" etc. (ver, a respeito, Gross e Levitt: 1998 - livro que inspirou Alan Sokal e Jean Bricmont a escreverem seu Imposturas intelectuais, Rio de Janeiro, Record, 1999, outra consistente den�ncia do relativismo e da falta de rigor nas humanidades).

Sob essa bandeira campeiam os multiculturalismos, o social-construtivismo, o ecofeminismo, os estudos culturais, as leituras de "g�nero", o ressentimento contra as ci�ncias. Privilegiam-se o intuitivo, o m�gico, o m�stico, o irracional, o marginal, abrindo-se as portas da academia para a New Age, as bruxas, o tar�, o ocultismo, a astrologia � temas freq�entes junto a certos comunic�logos, notadamente os de forma��o antropol�gica.

Diante disso, n�o espanta a condena��o, dentro das pr�prias universidades, n�o s� da ci�ncia e da tecnologia, mas tamb�m da racionalidade e da seculariza��o, "desencantadoras do mundo": n�o por acaso, fen�menos produzidos pela modernidade. N�o � o "p�s-modernismo" justamente esse conjunto de atitudes estil�sticas e julgamentos contr�rios ao que se sup�e ser ou ter sido a modernidade (em especial, ao que ela herdou do Iluminismo)? N�o espanta, igualmente, que universidades de prest�gio tragam ao Brasil, �s custas do dinheiro p�blico, soci�logos delirantes como Jean Baudrillard, que, a cada tr�s meses, vem nos advertir que a realidade n�o existe.

Afinal, n�o nos garante essa filosofia de sal�o chamada relativismo cultural que a ci�ncia n�o tem mais direito em afirmar a verdade do que o mito tribal?; ou que a ci�ncia � apenas a mitologia adotada por nossa tribo ocidental moderna? Vale lembrar, a prop�sito, um curioso relato do bi�logo Richard Dawkins, hoje professor da c�tedra de Compreens�o P�blica da Ci�ncia em Oxford, cuja obra deve, necessariamente, figurar numa bibliografia de jornalismo cient�fico. Conta ele que, certa vez, respondendo a uma provoca��o de um colega antrop�logo, colocou-lhe a seguinte quest�o: "Suponha que existe uma tribo que acredita que a Lua � uma caba�a velha lan�ada aos c�us, pendurada fora de alcance um pouco acima do topo das �rvores. Voc� afirma realmente que nossa verdade cient�fica � que afirma que a Lua est� a 382 mil quil�metros afastada e tem um quarto do di�metro da Terra � n�o � mais verdadeira do que a caba�a da tribo?" A resposta do antrop�logo foi direta: "Sim. N�s apenas fomos criados em uma cultura que v� o mundo de um modo cient�fico. Eles foram criados para ver o mundo de outro modo. Nenhum desses modos � mais verdadeiro do que o outro". Conclui Dawkins: "aponte-me um relativista cultural a 10 quil�metros de dist�ncia e lhe mostrarei um hip�crita. Avi�es constru�dos de acordo com princ�pios cient�ficos funcionam. Eles mant�m-se no ar e o levam ao seu destino escolhido. Avi�es constru�dos de acordo com especifica��es tribais ou mitol�gicas, tais como os avi�es de imita��o dos cultos de carregamento nas clareiras das selvas (...), n�o funcionam. Se voc� estiver voando para um congresso internacional de antrop�logos ou de cr�ticos liter�rios, a raz�o pela qual voc� provavelmente chegar� l� (...) � que uma multid�o de engenheiros ocidentais cientificamente treinados realizou os c�lculos corretamente. A ci�ncia ocidental, com base na evid�ncia confi�vel de que a Lua orbita em torno da Terra a uma dist�ncia de 382 mil quil�metros, conseguiu colocar pessoas em sua superf�cie. A ci�ncia tribal, acreditando que a Lua estava um pouco acima do topo das �rvores, nunca chegar� a toc�-la, exceto em sonhos" (Dawkins: 1996, p. 39-40).

Se a antici�ncia, atualmente, procede do circuito Paris-Nova York, de l� se espraiando para outros pa�ses, no s�culo XX foi da Alemanha que partiram os ataques mais fortes e duradouros. Os precedentes s�o long�nquos: nem o grande fil�sofo idealista G. W. F. Hegel (1770-1831) pouparia cr�ticas tanto �s ci�ncias quanto aos cientistas e fil�sofos mais pr�ximos de uma perspectiva cient�fica. Nunca escondeu, por exemplo, sua m�-vontade em rela��o a Newton (1642-1727), o pai da f�sica moderna, e a F. Bacon (1561-1626), fundador do m�todo indutivo moderno e precursor da sistematiza��o dos procedimentos cient�ficos.

2. De Marcuse ao Unabomber

A partir dos anos 40 do s�culo passado, a chamada Escola de Frankfurt � que se encarregaria de fomentar, por tr�s de sua cr�tica ao capitalismo, uma das mais persistentes e influentes cr�ticas � racionalidade cient�fica, com profundas repercuss�es nos movimentos estudantis da Europa da d�cada de 60. "A f�sica � burguesa", "a ci�ncia � o capital": estas inscri��es, nos muros da Paris de 68, resumiam, na verdade, os temas de Adorno, Horkheimer e, principalmente, Herbert Marcuse (1898-1979), o guru dos revoltosos (um dos tr�s grandes "M" da �poca, junto com Marx e Mao).

Para Marcuse, ci�ncia e capitalismo s�o uma s� coisa. Em outras palavras, ci�ncia (conhecimento racional e objetivo) e ideologia (concep��o de mundo) se confundem. Desaparece o valor objetivo do conhecimento cient�fico. A cr�tica da "raz�o instrumental" � ou "raz�o unidimensional", ou "raz�o t�cnica" � encerra, no fundo, uma cr�tica da pr�pria Civiliza��o. Da� o ataque � "sociedade industrial" ou "tecnol�gica", justamente a sociedade moderna, baseada na ci�ncia e na tecnologia.

Apenas os fil�sofos italianos (especialmente Galvano Della Volpe e Lucio Colletti) perceberam esta tr�gica confus�o, denunciando in loco a "Grande Recusa" marcusiana como a retomada de temas irracionalistas e rom�nticos. A "contracultura" gerada neste ambiente cultural, no entanto, fixaria ra�zes e amoldaria mentes; boa parte da gera��o que, nas humanidades, cresceu ouvindo essas melanc�licas diatribes contra a racionalidade cient�fica, a t�cnica, a "ind�stria cultural", etc., hoje as reproduz nas universidades e nas revistas acad�micas, quando n�o nos jornais. Principalmente no Brasil, onde ainda h� saudosos das "barricadas do desejo" de 68 e o prest�gio dos "frankfurtianos" continua inc�lume entre muitos intelectuais.

Por brevidade, considerarei aqui apenas as idiossincrasias anticient�ficas de Marcuse. N�o � necess�rio rastrear muito para se deparar, em sua obra, com inspira��es irracionalistas-rom�nticas. Ali�s, elas percorrem toda a sua teoria: j� num escrito de 1933 (Sobre os fundamentos filos�ficos do conceito de trabalho na ci�ncia econ�mica), sua pol�mica era contra a objetividade, com a "submiss�o" do homem �s coisas. Independentemente das �pocas hist�ricas, o trabalho sempre foi, para ele, "trabalho alienado" (o marxismo marcusiano confundia o que para Marx era distinto: "objetiva��o" e "aliena��o").

Para Marcuse, eliminar a "aliena��o" � eliminar a pr�pria objetividade. Essa "supera��o", portanto, n�o pode ser buscada no trabalho, mas... no jogo. � somente no jogo que o homem "n�o se conforma aos objetos, � sua regularidade". Somente ao colocar-se "acima da objetividade" � que o homem alcan�a a si pr�prio, "numa dimens�o de sua liberdade que � negada no trabalho". Para o fil�sofo alem�o, "um simples lance de bola, por parte de um jogador, representa um triunfo da liberdade humana sobre a objetividade que � infinitamente maior que a mais estrondosa conquista do trabalho t�cnico" (cit. em Tambosi: 1999, p. 150 � s�o do livro tamb�m as cita��es seguintes).

E pense-se no Marcuse de Raz�o e revolu��o (1941), em que afirma que "a raz�o � a verdadeira forma da realidade", onde "todos os antagonismos do sujeito e do objeto s�o integrados". Hegel, afinal, j� dissera que o real � o racional. Mas � no seu livro mais c�lebre � O homem unidimensional (1964) � que Marcuse transformar� sua rejei��o � objetividade num ataque � racionalidade cient�fica. O "dom�nio", agora, estava inscrito na pr�pria tecnologia. A "aliena��o" surge da produ��o industrial. A ci�ncia, mais uma vez, � ideologia.

A �ltima esperan�a, para ele, eram o Lumpenproletariat das metr�poles e as massas pobres do chamado "Terceiro Mundo", cuja oposi��o "� revolucion�ria, ainda que sua consci�ncia n�o o seja". Tudo isto antecipava temas que dominariam os discursos nos anos seguintes. A "Grande Recusa" influenciaria n�o s� a "Nova Esquerda" europ�ia, mas tamb�m o costume e a mentalidade comuns. Ci�ncia e capital eram uma s� coisa: os males que o marxismo havia denunciado no capitalismo eram descarregados por Marcuse (e, diga-se, tamb�m por Adorno e Horkheimer) "nos ombros de Galilei e Bacon". O desastre havia come�ado j� com a revolu��o cient�fica do s�culo XVII.

� a isto que os pensadores italianos chamam de "rea��o idealista" contra as ci�ncias e a t�cnica. Que chega, no caso de Marcuse, � nega��o total do existente: al�m de atacar o capitalismo, ele condenava tamb�m o socialismo por "submeter-se ao aparato tecnol�gico". O fil�sofo sonhava com uma "nova ci�ncia" e uma "nova t�cnica" � sobre as quais n�o forneceu jamais a m�nima indica��o. O problema da teoria marxista cl�ssica, segundo ele, estava em conceber "a transi��o do capitalismo para o socialismo como uma revolu��o pol�tica", isto �, em destruir o "aparato pol�tico", mas n�o o "aparato tecnol�gico"!

No capitalismo avan�ado � prossegue Marcuse � "a racionalidade t�cnica est� personificada, a despeito de seu uso irracional, no aparato produtor. Isto n�o se aplica apenas �s f�bricas mecanizadas, ferramentas e explora��o de recursos, mas tamb�m � maneira de trabalhar como adapta��o ao processo mec�nico do mesmo, conforme programado pela �ger�ncia cient�fica�. Nem a nacionaliza��o nem a socializa��o alteram por si essa personaliza��o f�sica da racionalidade tecnol�gica; pelo contr�rio, esta permanece uma condi��o pr�via para o desenvolvimento socialista das for�as produtivas" (Marcuse: 1989, p. 41).

Estamos no cora��o da "Grande Recusa", uma heran�a ideol�gica que, junto �s filosofias "p�s-modernas", ainda inspira manifesta��es contra a ci�ncia e a tecnologia. O alvo, agora, � o vertiginoso processo de informatiza��o. Lado a lado com a chamada III revolu��o industrial, crescem tamb�m a tecnofobia e a rejei��o das tecnologias. O mal-estar assume novas � e at� violentas � formas. Basta lembrar o caso do Unabomber, nos EUA, que enviava cartas-bombas para cientistas e universidades. Tamb�m nesse pa�s, um intelectual que escreveu livros como Rebeldes contra o futuro e A revolu��o verde ilustra suas confer�ncias quebrando computadores a golpes de martelo.

Recorde-se que o manifesto do terrorista Unabomber, publicado em setembro de 1995 pelo Washington Post, elege como inimiga a "sociedade industrial" (curiosamente, um conceito marcusiano), que ele considera "um desastre para a esp�cie humana" e contra a qual prop�e uma "revolu��o": "a �nica sa�da" � pontifica � "� dispensar o sistema tecnol�gico inteiro". Seu temor s�o as "m�quinas inteligentes", que acabar�o por decidir no lugar da humanidade. "Quando chegar a esse est�gio, as m�quinas estar�o, efetivamente, no controle. As pessoas n�o poder�o simplesmente desligar as m�quinas porque elas estar�o t�o dependentes delas que deslig�-las equivaleria a cometer suic�dio". A estrat�gia? "Promover o estresse social e a instabilidade na sociedade industrial, e desenvolver e difundir uma ideologia que se oponha � tecnologia e ao sistema industrial". O terrorista quer simplesmente "a elimina��o da tecnologia moderna" (Folha de S. Paulo, 20/09/95).

Kirkpatrick Sale, o destruidor de computadores e autoproclamado l�der do "neoludismo" (herdeiro do movimento de desempregados ingleses que, entre 1811 e 1813, quebravam m�quinas em protesto contra a revolu��o industrial), compartilha esse mal-estar em rela��o � sociedade moderna. Para ele, "a civiliza��o � catastr�fica porque destr�i a si mesma e o ambiente natural", e "o uso da ci�ncia e das suas tecnologias � um atentado � Natureza, uma tentativa de criar uma natureza tecnol�gica, de modo que a humanidade possa controlar todas as coisas" (L�Espresso, 11/08/95). Do Unabomber, Sale s� discorda quanto aos "m�todos", porque "a inten��o � boa".

No "para�so" vislumbrado por Sale desaparecem os produtos tecnol�gicos: do computador ao forno de microondas, da videoc�mera ao telefone digital. O autom�vel � demon�aco. Voltemos �s bicicletas, recomenda ele. N�o � �-toa que nos EUA j� exista, entre as associa��es antitecnol�gicas, at� um "Clube do L�pis", que defende a escrita � m�o. A utopia do �ltimo dos luditas, como quase todas as utopias anticient�ficas contempor�neas, � uma volta ao passado.

3. � o conhecimento perigoso?

A julgar pelas vertentes e tend�ncias aqui apontadas, a resposta � positiva. Na verdade, a id�ia de que o conhecimento � perigoso est� arraigada na nossa cultura. J� Ad�o e Eva, segundo a B�blia, foram proibidos de alimentar-se dos frutos da �rvore do Conhecimento. Prometeu foi punido por ter dado o saber ao mundo. Na literatura, o Dr. Frankenstein � a imagem do cientista, pintado como um arrogante desalmado que de tudo � capaz para atingir seus objetivos, quaisquer que sejam as conseq��ncias. No cinema, � o g�nio louco que produz monstros e cat�strofes.

Imoral manipulador da Natureza, o cientista tamb�m foi responsabilizado pela constru��o da bomba at�mica e, agora, � visto com suspeita em virtude da engenharia gen�tica. Jornais e revistas publicam com freq��ncia textos alarmistas que advertem sobre os "perigos" da pesquisa gen�tica (lembre-se a histeria sobre a clonagem), do projeto do genoma humano e dos transg�nicos ("comida Frankenstein"). Nos t�tulos, invariavelmente, a insinua��o de que o cientista "brinca de ser Deus". O horror, por�m, convive com o fasc�nio, j� que se espera da ci�ncia a solu��o para a cura do c�ncer e da Aids, entre outras doen�as.

A an�lise desse problema nos remete, de novo, � separa��o moderna de fatos e valores, ou seja, de ci�ncia e �tica. Como processo de conhecimento racional e objetivo, a ci�ncia n�o � guiada por valores. Ela apenas nos mostra como o mundo �. A ci�ncia descreve, a �tica prescreve; a ci�ncia explica, a �tica avalia. Ci�ncia, portanto, n�o produz �tica. Das proposi��es descritivas n�o � poss�vel deduzir asser��es prescritivas, como bem viu o fil�sofo Hume (1711-1776). A separa��o de fatos e valores � conhecida justamente como Lei de Hume � impede que do "� "derive o "deve", que do "ser" derive o "dever ser".

Em oposi��o a essas tend�ncias filos�ficas e culturais, e considerando o patrim�nio humano j� alcan�ado, podemos afirmar que o conhecimento cient�fico n�o � perigoso. O conhecimento � um bem em si mesmo. Para o ser humano, conhecer � t�o vital quanto alimentar-se, defender-se ou amar. J� a tecnologia, contrariamente, pode ser tanto uma d�diva quanto uma maldi��o. H� processos tecnol�gicos intrinsecamente perversos, como a fabrica��o de instrumentos de tortura, armas bacteriol�gicas, etc. Como resume Bunge, "n�o se trata do mau uso imprevisto de um setor de conhecimento, como seria o mau uso de uma tesoura ou de um f�sforo. A tecnologia da maldade � maldosa" (Bunge: 1980, p. 202).

Quando a pesquisa cient�fica � posta em pr�tica � por exemplo, em experimentos que envolvam seres humanos ou outros animais �, ou quando a ci�ncia � aplicada � tecnologia, problemas �ticos relevantes podem e devem ser levantados. Mas aqui � importante distinguir ci�ncia de tecnologia, pois suas motiva��es s�o diferentes. Em poucas palavras, ci�ncia (b�sica) produz id�ias, teorias; tecnologia produz objetos, bens. Uma visa simplesmente conhecer; outra � voltada para fins pr�ticos.

Conv�m observar que a tecnologia � muito mais antiga que a ci�ncia e possui uma hist�ria pr�pria. Todos os povos produziram tecnologias, mas s� o povo grego criou a ci�ncia de que somos herdeiros. Num belo livro, o historiador da tecnologia George Basalla demonstra que, at� o s�culo XIX, a ci�ncia exerceu pouco impacto sobre a tecnologia. Sem aux�lio da ci�ncia, a tecnologia gerou a agricultura, os artefatos de metais, as conquistas da engenharia chinesa e at� mesmo as catedrais do Renascimento. Essas imponentes constru��es foram erguidas por engenheiros que se baseavam na experi�ncia pr�tica, aprendendo diariamente com os erros, e n�o em teorias cient�ficas. Prevalecia ent�o, como sugere outro autor, "o teorema dos cinco minutos" � se uma estrutura permanecesse de p� por cinco minutos depois de tirados os suportes, presumia-se que se manteria de p� para sempre (cf. Basalla: 1999; e Wolpert: 1996).

A esta altura, imp�e-se indagar quais s�o, afinal, as responsabilidades e obriga��es morais dos cientistas. N�o h� d�vida de que eles possuem deveres distintos das obriga��es dos demais cidad�os. Posto que os cientistas det�m conhecimento especializado sobre como � e como funciona o mundo, e isto nem sempre � acess�vel aos outros, � obriga��o deles tornar p�blicas as implica��es sociais de seu trabalho e suas aplica��es tecnol�gicas" (cf. artigo de Wolpert na revista Nature, 398 (1999), p. 281-82; e Wolpert: 1996, p. 185 e segs.).

Se ci�ncia e �tica, como vimos, s�o distintas, nem por isso o cientista est� isento de deveres �ticos. O bi�logo ingl�s Lewis Wolpert aponta, a prop�sito, um exemplo de comportamento imoral por parte dos cientistas no movimento da eugenia, iniciado na Inglaterra no final do s�culo XIX, estendendo-se depois aos EUA. O movimento, cuja pretens�o era "melhorar as ra�as", envolveu inicialmente nomes ilustres como Galton (criador do pr�prio conceito), Fisher, Haldane, Huxley, Morgan, Davenport, Havelock Ellis e at� o literato Bernard Shaw. N�o demorou que se passasse a considerar heredit�rio n�o s� o talento, mas a pobreza; que se considerasse os negros "biologicamente inferiores" e que algumas "ra�as" possu�am "tend�ncia � debilidade mental".

A Sociedade Eug�nica Americana chegou a promover concursos para "fam�lias geneticamente s�s", qualificando, em seu "catecismo eug�nico", o "plasma germinal humano" como "a coisa mais preciosa do mundo". Para impedir a "contamina��o" dos plasmas, a receita era a esteriliza��o em massa. Estima-se que, entre 1907 e 1928, nove mil pessoas foram submetidas a tal tratamento, sob a gen�rica etiqueta de "debilidade mental". E pense-se no horror nazista: a lei sobre esteriliza��o eug�nica, que Hitler decretou em 1933, foi o primeiro passo para as atrocidades cometidas pelos m�dicos nos campos de concentra��o (ver Wolpert: 1996, p. 194-98). Em rela��o � eugenia, portanto, est� claro que os cientistas n�o assumiram suas obriga��es �ticas.

Diverso foi o comportamento dos pesquisadores envolvidos na constru��o da bomba at�mica, um empreendimento tecnol�gico baseado em conhecimento cient�fico. Aqui podemos perceber claramente como a confus�o entre ci�ncia e tecnologia conduziu a uma vis�o err�nea sobre o papel da ci�ncia. As aplica��es desta n�o s�o, necessariamente, responsabilidade dos cientistas: as decis�es cabem, muito mais, a governantes e pol�ticos. No caso da bomba at�mica, a responsabilidade foi assumida exclusivamente pelo presidente Roosevelt, como demonstra o jornalista norte-americano Richard Rhodes num livro admir�vel, ao qual remeto: The making of the atomic bomb: 1988). Em outras palavras, a decis�o foi pol�tica, n�o cient�fica.

Quem primeiro teve a id�ia de uma poss�vel rea��o em cadeia de n�utrons foi o f�sico h�ngaro Leo Szilard, ent�o residente na Inglaterra. Atrav�s de Einstein, ele comunicou essa possibilidade a Roosevelt, que autorizou a montagem de um gigantesco projeto (secreto), envolvendo cientistas e engenheiros. Antes mesmo do primeiro teste nuclear (15 de julho de 1945), por�m, Szilard demonstrou-se preocupado com uma opera��o sobre a qual, em realidade, os cientistas tinham pouco ou nenhum controle. Chegou a pensar, inclusive, num controle internacional que evitasse o monop�lio norte-americano da bomba. Com a II Guerra chegando ao final, pensava ele, n�o havia raz�es para a utiliza��o dessa arma. Szilard fez ent�o circular uma peti��o, firmada por 66 cientistas que trabalhavam no projeto, a ser enviada ao presidente Truman, sucessor de Roosevelt (morto em maio de 45).

Argumentam os subscritores que "uma na��o que estabelece o precedente de usar as for�as da natureza recentemente desencadeadas com fins destrutivos, poder� ter que assumir tamb�m a responsabilidade de ter aberto as portas a uma era de devasta��o em dimens�es inimagin�veis." Por isso, pediam eles que o presidente usasse suas prerrogativas para impedir que os Estados Unidos recorressem ao emprego de bombas at�micas, salvo no caso de o Jap�o rejeitar as condi��es de rendi��o que lhe fossem impostas, e depois que tais condi��es fossem de amplo dom�nio p�blico (Rhodes: 1988, p. 749 e segs.).

O fato � que a carta jamais chegou �s m�os do presidente. No dia 6 de agosto de 1945, como se sabe, a bomba destruiu Hiroshima. Quanto a Szilard, dedicou-se depois da guerra a divulgar ao p�blico as implica��es do conhecimento cient�fico. Jamais se cansou de ressaltar a necessidade de o p�blico ser informado tanto sobre a ci�ncia quanto sobre suas aplica��es. Cumpriu � risca, portanto, o dever �tico de todo cientista.

Orlando Tambosi

Textos citados


Basalla, George. The history of tecnology. Cambridge, Cambridge University Press, 8� ed., 1999.
Bunge, Mario. Epistemologia. S�o Paulo, T. A. Queiroz, 2�, 1987.
______. La ciencia. Su m�todo e su filosof�a. Buenos Aires, Sudamericana, 3�, 1998.
Colletti, Lucio. Pagine di filosofia e politica. Mil�o, Rizzoli, 1989.
_____. Fine della filosofia e altri saggi. Roma, Ideazione, 1996.
Dawkins, Richard. O rio que sa�a do �den. Rio de Janeiro, Rocco, 1996.
Gross, Paul, e Levitt, Norman. Higher superstition. The academic left and its quarrels with science. Baltimore, John Hopkins University Press, 2� ed., 1998.
Marcuse, Herbert. O homem unidimensional. A ideologia da sociedade industrial. Rio de Janeiro, Zahar, 5� ed., 1979.
_____. Raz�o e revolu��o. Rio de Janeiro, Paz e Terra, 2� ed., 1978.
Monod, Jacques. O acaso e a necessidade. Ensaio sobre a filosofia natural da biologia moderna. Petr�polis, Vozes, 4� ed., 1989.
Rhodes, Richard. The making of atomic bomb. Nova York, Simon & Schuster, 1988.
Tambosi, Orlando. O decl�nio do marxismo e a heran�a hegeliana. Florian�polis, Editora da UFSC, 1999.
Wolpert, Lewis. La natura innaturale della scienza. Bari, Edizioni Dedalo, 1996.


[As partes desta mensagem que n�o continham texto foram removidas]



SUBJECT: A Ciência e o Desporto
FROM: Maria Natália <grasdic@hotmail.com>
TO: ciencialist@yahoogrupos.com.br
DATE: 07/01/2005 23:31


A Direcção do Pavilhão do Conhecimento
– Ciência Viva tem o prazer de convidar para a abertura da exposição
A CIÊNCIA E O DESPORTO, no dia 12 de Janeiro, quarta-feira, às 11h00.

"O desporto moderno é uma combinação de ciência, destreza, dedicação
e esforço total.

Nesta exposição pode testar as suas capacidades e experimentar uma
variedade de actividades desportivas.

Visite, experimente.
Seja um atleta com ciência.

De 21 de Dezembro 2004 a 18 de Setembro 2005

Por favor, confirme a sua presença:Tel. 218 917 100 |
info@pavconhecimento.pt"
um abraço
Maria Natália





SUBJECT: Re: [ciencialist] É o conhecimento perigoso?
FROM: "Oraculo" <oraculo@atibaia.com.br>
TO: <ciencialist@yahoogrupos.com.br>
DATE: 07/01/2005 23:53

Olá Amaury

Maravilhoso este texto, muito bom mesmo. Expõe com clareza muito maior do
que eu jamais poderia, meu ponto de vista, sobre a separação da ciência e da
ética. Expõe a subjetividade humana como responsável final pela aplicação da
ciência, não por sua produção.

A parte que afirma que "A ciência descreve, a ética prescreve; a ciência
explica, a ética avalia." perfeita: precisa e clara, uma bela exposição da
questão.

Entretanto, não sei se percebe que este texto também é frontalmente
contrário à sua posição inicial, contra a ciência. E discorda, de forma
absoluta, de suas críticas a Sagan e seu texto. Não sei se era essa a sua
intenção, mas penso que talvez tenha compreendido meu ponto (e mudado de
idéia..:-).

Apesar da primeira parte levantar com precisão os atuais ataques contra a
ciência (que alguns chamam de cientificismo), ele o faz apenas para
demonstrar o erro dessa posição logo a seguir. Veja estes trechos:

"Apenas os filósofos italianos (especialmente Galvano Della Volpe e Lucio
Colletti) perceberam esta trágica confusão, denunciando in loco a "Grande
Recusa" marcusiana como a retomada de temas irracionalistas e românticos. A
"contracultura" gerada neste ambiente cultural, no entanto, fixaria raízes e
amoldaria mentes; boa parte da geração que, nas humanidades, cresceu ouvindo
essas melancólicas diatribes contra a racionalidade científica, a técnica, a
"indústria cultural", etc., hoje as reproduz nas universidades e nas
revistas acadêmicas, quando não nos jornais. Principalmente no Brasil, onde
ainda há saudosos das "barricadas do desejo" de 68 e o prestígio dos
"frankfurtianos" continua incólume entre muitos intelectuais."

e este:

"Em oposição a essas tendências filosóficas e culturais, e considerando o
patrimônio humano já alcançado, podemos afirmar que o conhecimento
científico não é perigoso. O conhecimento é um bem em si mesmo. Para o ser
humano, conhecer é tão vital quanto alimentar-se, defender-se ou amar. Já a
tecnologia, contrariamente, pode ser tanto uma dádiva quanto uma maldição."

Fica clara a posição do autor sore o assunto, não? O conehcimento é um bem
em sí mesmo, e o conheciemtno científico é diferente de seu uso por seres
humanos, governos ou políticos. Exatametne o que venho tentando demonstrar
em nosso debate..:-)

Um abraço.

Homero



SUBJECT: Re: É o conhecimento perigoso?
FROM: Maria Natália <grasdic@hotmail.com>
TO: ciencialist@yahoogrupos.com.br
DATE: 08/01/2005 00:20


Texto muito bom, este. Gostei e continua.
Vamos fazendo revisões da matéria já dada há uns tempos.
Obrigada
Um abraço
Maria Natália
--- Em ciencialist@yahoogrupos.com.br, "Amauri Jr"
<amaurijunior2@y...> escreveu
>
> É o conhecimento perigoso?
> Fronteiras entre ciência, tecnologia e ética
> Orlando Tambosi
>
>
> 1. As feridas da modernidade e a anticiência
>
>
> O traço mais profundo e mais perturbador de nossa época é a
dissociação de fato e valor, ser e dever ser, ou física e ética,
conhecimento da realidade e atribuição de sentido à vida. As
ciências descrevem e conhecem o mundo tal qual é, mas calam sobre as
angústias humanas, tornando o homem praticamente um acidente no
cosmo. Despertando de seu sonho milenar, como diz o biólogo Monod, o
ser humano agora "sabe que, como um cigano, está à margem do
Universo onde deve viver. Um Universo surdo à sua música,
indiferente às suas esperanças, como a seus sofrimentos ou a seus
crimes". Ele sabe que "está sozinho na imensidão indiferente do
Universo, de onde emergiu por acaso. Não mais do que seu destino,
seu dever não está escrito em lugar algum" (Monod: 1989, p. 190-8).
>
> Através da ciência, a modernidade rompeu a "aliança animística"
entre homem e Natureza, calcada exatamente na identificação de fato
e valor - fundamento da visão antropocêntrica do mundo. A cosmologia
medieval (aristotélico-cristã) realizava a coincidência plena disso
que, para nós, é dividido: conhecimento da realidade e compreensão
do "sentido" da nossa vida – sua destinação ou valor – eram uma só
coisa. Por mais de dois mil anos, a metafísica (o nome remete, como
se sabe, ao conhecimento do transcendente ou do supra-sensível)
sustentou a separação entre mundo terrestre e mundo celeste:
embaixo, o reino do efêmero, do nascer e do perecer; no alto, com
suas esferas perfeitas, o reino do divino, do incorruptível, do
eterno, do verdadeiro Ser. Os níveis de realidade exprimem ao mesmo
tempo uma hierarquia de valores. A Terra, no centro, é o palco em
que se desenrola o drama humano, em vista do qual o próprio cosmo
foi criado.
>
> A modernidade rompe essa imagem. A revolução astronômica explode
esse cosmo finito e fechado, revelando um universo de proporções
ilimitadas. A Terra já não é mais o centro de nada. "É um ponto
infinitesimal, uma minúscula ilha perdida num oceano sem praias,
onde se contam bilhões de galáxias, cada uma delas com centenas de
bilhões de sóis. Explicar essa realidade em função do homem, ou dela
extrair um significado para a nossa existência, é simplesmente
impossível" (Colletti: 1989, p. III-IV, e 1996, p.15).
>
> Depois que Copérnico arrancou o homem do centro do universo,
Darwin obrigou-o a reconhecer que não passa de um ser entre outros
no reino animal (competindo com as outras espécies e,
freqüentemente, perdendo a luta para as mais microscópicas). São
duas feridas insanáveis que corroem o narcisismo humano, como
definiu Freud, e que produzem mal-estar ainda hoje. Daí a
hostilidade em relação às ciências e às tecnologias, comum a algumas
vertentes filosóficas e tendências culturais contemporâneas,
particularmente as que se autodenominam "pós-modernas". A
anticiência, por sinal, encontra confortável abrigo nas ciências
sociais e humanidades, minadas pelo relativismo cognitivo e
cultural; e, junto com as pseudociências, conta com generoso espaço
na mídia.
>
> Muito do que se produz nessas áreas é hostil a conceitos
como "realidade", "objetividade", "verdade", fundamentais tanto à
ciência quanto ao jornalismo científico. Para a cultura "pós-
moderna", o "real", os "fatos" que as ciências buscam conhecer – e o
jornalismo reportar - não passam de "construções intelectuais". Mero
discurso ou "narrativa", a ciência é ideológica, isto é, instrumento
de dominação de uma
civilização "branca", "eurocêntrica", "opressora", "machista", "heter
ossexual" etc. (ver, a respeito, Gross e Levitt: 1998 - livro que
inspirou Alan Sokal e Jean Bricmont a escreverem seu Imposturas
intelectuais, Rio de Janeiro, Record, 1999, outra consistente
denúncia do relativismo e da falta de rigor nas humanidades).
>
> Sob essa bandeira campeiam os multiculturalismos, o social-
construtivismo, o ecofeminismo, os estudos culturais, as leituras
de "gênero", o ressentimento contra as ciências. Privilegiam-se o
intuitivo, o mágico, o místico, o irracional, o marginal, abrindo-se
as portas da academia para a New Age, as bruxas, o tarô, o
ocultismo, a astrologia – temas freqüentes junto a certos
comunicólogos, notadamente os de formação antropológica.
>
> Diante disso, não espanta a condenação, dentro das próprias
universidades, não só da ciência e da tecnologia, mas também da
racionalidade e da secularização, "desencantadoras do mundo": não
por acaso, fenômenos produzidos pela modernidade. Não é o "pós-
modernismo" justamente esse conjunto de atitudes estilísticas e
julgamentos contrários ao que se supõe ser ou ter sido a modernidade
(em especial, ao que ela herdou do Iluminismo)? Não espanta,
igualmente, que universidades de prestígio tragam ao Brasil, às
custas do dinheiro público, sociólogos delirantes como Jean
Baudrillard, que, a cada três meses, vem nos advertir que a
realidade não existe.
>
> Afinal, não nos garante essa filosofia de salão chamada
relativismo cultural que a ciência não tem mais direito em afirmar a
verdade do que o mito tribal?; ou que a ciência é apenas a mitologia
adotada por nossa tribo ocidental moderna? Vale lembrar, a
propósito, um curioso relato do biólogo Richard Dawkins, hoje
professor da cátedra de Compreensão Pública da Ciência em Oxford,
cuja obra deve, necessariamente, figurar numa bibliografia de
jornalismo científico. Conta ele que, certa vez, respondendo a uma
provocação de um colega antropólogo, colocou-lhe a seguinte
questão: "Suponha que existe uma tribo que acredita que a Lua é uma
cabaça velha lançada aos céus, pendurada fora de alcance um pouco
acima do topo das árvores. Você afirma realmente que nossa verdade
científica – que afirma que a Lua está a 382 mil quilômetros
afastada e tem um quarto do diâmetro da Terra – não é mais
verdadeira do que a cabaça da tribo?" A resposta do antropólogo foi
direta: "Sim. Nós apenas fomos criados em uma cultura que vê o mundo
de um modo científico. Eles foram criados para ver o mundo de outro
modo. Nenhum desses modos é mais verdadeiro do que o outro". Conclui
Dawkins: "aponte-me um relativista cultural a 10 quilômetros de
distância e lhe mostrarei um hipócrita. Aviões construídos de acordo
com princípios científicos funcionam. Eles mantêm-se no ar e o levam
ao seu destino escolhido. Aviões construídos de acordo com
especificações tribais ou mitológicas, tais como os aviões de
imitação dos cultos de carregamento nas clareiras das selvas (...),
não funcionam. Se você estiver voando para um congresso
internacional de antropólogos ou de críticos literários, a razão
pela qual você provavelmente chegará lá (...) é que uma multidão de
engenheiros ocidentais cientificamente treinados realizou os
cálculos corretamente. A ciência ocidental, com base na evidência
confiável de que a Lua orbita em torno da Terra a uma distância de
382 mil quilômetros, conseguiu colocar pessoas em sua superfície. A
ciência tribal, acreditando que a Lua estava um pouco acima do topo
das árvores, nunca chegará a tocá-la, exceto em sonhos" (Dawkins:
1996, p. 39-40).
>
> Se a anticiência, atualmente, procede do circuito Paris-Nova York,
de lá se espraiando para outros países, no século XX foi da Alemanha
que partiram os ataques mais fortes e duradouros. Os precedentes são
longínquos: nem o grande filósofo idealista G. W. F. Hegel (1770-
1831) pouparia críticas tanto às ciências quanto aos cientistas e
filósofos mais próximos de uma perspectiva científica. Nunca
escondeu, por exemplo, sua má-vontade em relação a Newton (1642-
1727), o pai da física moderna, e a F. Bacon (1561-1626), fundador
do método indutivo moderno e precursor da sistematização dos
procedimentos científicos.
>
> 2. De Marcuse ao Unabomber
>
> A partir dos anos 40 do século passado, a chamada Escola de
Frankfurt é que se encarregaria de fomentar, por trás de sua crítica
ao capitalismo, uma das mais persistentes e influentes críticas à
racionalidade científica, com profundas repercussões nos movimentos
estudantis da Europa da década de 60. "A física é burguesa", "a
ciência é o capital": estas inscrições, nos muros da Paris de 68,
resumiam, na verdade, os temas de Adorno, Horkheimer e,
principalmente, Herbert Marcuse (1898-1979), o guru dos revoltosos
(um dos três grandes "M" da época, junto com Marx e Mao).
>
> Para Marcuse, ciência e capitalismo são uma só coisa. Em outras
palavras, ciência (conhecimento racional e objetivo) e ideologia
(concepção de mundo) se confundem. Desaparece o valor objetivo do
conhecimento científico. A crítica da "razão instrumental" –
ou "razão unidimensional", ou "razão técnica" – encerra, no fundo,
uma crítica da própria Civilização. Daí o ataque à "sociedade
industrial" ou "tecnológica", justamente a sociedade moderna,
baseada na ciência e na tecnologia.
>
> Apenas os filósofos italianos (especialmente Galvano Della Volpe e
Lucio Colletti) perceberam esta trágica confusão, denunciando in
loco a "Grande Recusa" marcusiana como a retomada de temas
irracionalistas e românticos. A "contracultura" gerada neste
ambiente cultural, no entanto, fixaria raízes e amoldaria mentes;
boa parte da geração que, nas humanidades, cresceu ouvindo essas
melancólicas diatribes contra a racionalidade científica, a técnica,
a "indústria cultural", etc., hoje as reproduz nas universidades e
nas revistas acadêmicas, quando não nos jornais. Principalmente no
Brasil, onde ainda há saudosos das "barricadas do desejo" de 68 e o
prestígio dos "frankfurtianos" continua incólume entre muitos
intelectuais.
>
> Por brevidade, considerarei aqui apenas as idiossincrasias
anticientíficas de Marcuse. Não é necessário rastrear muito para se
deparar, em sua obra, com inspirações irracionalistas-românticas.
Aliás, elas percorrem toda a sua teoria: já num escrito de 1933
(Sobre os fundamentos filosóficos do conceito de trabalho na ciência
econômica), sua polêmica era contra a objetividade, com
a "submissão" do homem às coisas. Independentemente das épocas
históricas, o trabalho sempre foi, para ele, "trabalho alienado" (o
marxismo marcusiano confundia o que para Marx era
distinto: "objetivação" e "alienação").
>
> Para Marcuse, eliminar a "alienação" é eliminar a própria
objetividade. Essa "superação", portanto, não pode ser buscada no
trabalho, mas... no jogo. É somente no jogo que o homem "não se
conforma aos objetos, à sua regularidade". Somente ao colocar-
se "acima da objetividade" é que o homem alcança a si próprio, "numa
dimensão de sua liberdade que é negada no trabalho". Para o filósofo
alemão, "um simples lance de bola, por parte de um jogador,
representa um triunfo da liberdade humana sobre a objetividade que é
infinitamente maior que a mais estrondosa conquista do trabalho
técnico" (cit. em Tambosi: 1999, p. 150 – são do livro também as
citações seguintes).
>
> E pense-se no Marcuse de Razão e revolução (1941), em que afirma
que "a razão é a verdadeira forma da realidade", onde "todos os
antagonismos do sujeito e do objeto são integrados". Hegel, afinal,
já dissera que o real é o racional. Mas é no seu livro mais célebre –
O homem unidimensional (1964) – que Marcuse transformará sua
rejeição à objetividade num ataque à racionalidade científica.
O "domínio", agora, estava inscrito na própria tecnologia.
A "alienação" surge da produção industrial. A ciência, mais uma vez,
é ideologia.
>
> A última esperança, para ele, eram o Lumpenproletariat das
metrópoles e as massas pobres do chamado "Terceiro Mundo", cuja
oposição "é revolucionária, ainda que sua consciência não o seja".
Tudo isto antecipava temas que dominariam os discursos nos anos
seguintes. A "Grande Recusa" influenciaria não só a "Nova Esquerda"
européia, mas também o costume e a mentalidade comuns. Ciência e
capital eram uma só coisa: os males que o marxismo havia denunciado
no capitalismo eram descarregados por Marcuse (e, diga-se, também
por Adorno e Horkheimer) "nos ombros de Galilei e Bacon". O desastre
havia começado já com a revolução científica do século XVII.
>
> É a isto que os pensadores italianos chamam de "reação idealista"
contra as ciências e a técnica. Que chega, no caso de Marcuse, à
negação total do existente: além de atacar o capitalismo, ele
condenava também o socialismo por "submeter-se ao aparato
tecnológico". O filósofo sonhava com uma "nova ciência" e uma "nova
técnica" – sobre as quais não forneceu jamais a mínima indicação. O
problema da teoria marxista clássica, segundo ele, estava em
conceber "a transição do capitalismo para o socialismo como uma
revolução política", isto é, em destruir o "aparato político", mas
não o "aparato tecnológico"!
>
> No capitalismo avançado – prossegue Marcuse – "a racionalidade
técnica está personificada, a despeito de seu uso irracional, no
aparato produtor. Isto não se aplica apenas às fábricas mecanizadas,
ferramentas e exploração de recursos, mas também à maneira de
trabalhar como adaptação ao processo mecânico do mesmo, conforme
programado pela `gerência científica'. Nem a nacionalização nem a
socialização alteram por si essa personalização física da
racionalidade tecnológica; pelo contrário, esta permanece uma
condição prévia para o desenvolvimento socialista das forças
produtivas" (Marcuse: 1989, p. 41).
>
> Estamos no coração da "Grande Recusa", uma herança ideológica que,
junto às filosofias "pós-modernas", ainda inspira manifestações
contra a ciência e a tecnologia. O alvo, agora, é o vertiginoso
processo de informatização. Lado a lado com a chamada III revolução
industrial, crescem também a tecnofobia e a rejeição das
tecnologias. O mal-estar assume novas – e até violentas – formas.
Basta lembrar o caso do Unabomber, nos EUA, que enviava cartas-
bombas para cientistas e universidades. Também nesse país, um
intelectual que escreveu livros como Rebeldes contra o futuro e A
revolução verde ilustra suas conferências quebrando computadores a
golpes de martelo.
>
> Recorde-se que o manifesto do terrorista Unabomber, publicado em
setembro de 1995 pelo Washington Post, elege como inimiga
a "sociedade industrial" (curiosamente, um conceito marcusiano), que
ele considera "um desastre para a espécie humana" e contra a qual
propõe uma "revolução": "a única saída" – pontifica – "é dispensar o
sistema tecnológico inteiro". Seu temor são as "máquinas
inteligentes", que acabarão por decidir no lugar da
humanidade. "Quando chegar a esse estágio, as máquinas estarão,
efetivamente, no controle. As pessoas não poderão simplesmente
desligar as máquinas porque elas estarão tão dependentes delas que
desligá-las equivaleria a cometer suicídio". A estratégia? "Promover
o estresse social e a instabilidade na sociedade industrial, e
desenvolver e difundir uma ideologia que se oponha à tecnologia e ao
sistema industrial". O terrorista quer simplesmente "a eliminação da
tecnologia moderna" (Folha de S. Paulo, 20/09/95).
>
> Kirkpatrick Sale, o destruidor de computadores e autoproclamado
líder do "neoludismo" (herdeiro do movimento de desempregados
ingleses que, entre 1811 e 1813, quebravam máquinas em protesto
contra a revolução industrial), compartilha esse mal-estar em
relação à sociedade moderna. Para ele, "a civilização é catastrófica
porque destrói a si mesma e o ambiente natural", e "o uso da ciência
e das suas tecnologias é um atentado à Natureza, uma tentativa de
criar uma natureza tecnológica, de modo que a humanidade possa
controlar todas as coisas" (L'Espresso, 11/08/95). Do Unabomber,
Sale só discorda quanto aos "métodos", porque "a intenção é boa".
>
> No "paraíso" vislumbrado por Sale desaparecem os produtos
tecnológicos: do computador ao forno de microondas, da videocâmera
ao telefone digital. O automóvel é demoníaco. Voltemos às
bicicletas, recomenda ele. Não é à-toa que nos EUA já exista, entre
as associações antitecnológicas, até um "Clube do Lápis", que
defende a escrita à mão. A utopia do último dos luditas, como quase
todas as utopias anticientíficas contemporâneas, é uma volta ao
passado.
>
> 3. É o conhecimento perigoso?
>
> A julgar pelas vertentes e tendências aqui apontadas, a resposta é
positiva. Na verdade, a idéia de que o conhecimento é perigoso está
arraigada na nossa cultura. Já Adão e Eva, segundo a Bíblia, foram
proibidos de alimentar-se dos frutos da Árvore do Conhecimento.
Prometeu foi punido por ter dado o saber ao mundo. Na literatura, o
Dr. Frankenstein é a imagem do cientista, pintado como um arrogante
desalmado que de tudo é capaz para atingir seus objetivos, quaisquer
que sejam as conseqüências. No cinema, é o gênio louco que produz
monstros e catástrofes.
>
> Imoral manipulador da Natureza, o cientista também foi
responsabilizado pela construção da bomba atômica e, agora, é visto
com suspeita em virtude da engenharia genética. Jornais e revistas
publicam com freqüência textos alarmistas que advertem sobre
os "perigos" da pesquisa genética (lembre-se a histeria sobre a
clonagem), do projeto do genoma humano e dos transgênicos ("comida
Frankenstein"). Nos títulos, invariavelmente, a insinuação de que o
cientista "brinca de ser Deus". O horror, porém, convive com o
fascínio, já que se espera da ciência a solução para a cura do
câncer e da Aids, entre outras doenças.
>
> A análise desse problema nos remete, de novo, à separação moderna
de fatos e valores, ou seja, de ciência e ética. Como processo de
conhecimento racional e objetivo, a ciência não é guiada por
valores. Ela apenas nos mostra como o mundo é. A ciência descreve, a
ética prescreve; a ciência explica, a ética avalia. Ciência,
portanto, não produz ética. Das proposições descritivas não é
possível deduzir asserções prescritivas, como bem viu o filósofo
Hume (1711-1776). A separação de fatos e valores — conhecida
justamente como Lei de Hume — impede que do "é "derive o "deve", que
do "ser" derive o "dever ser".
>
> Em oposição a essas tendências filosóficas e culturais, e
considerando o patrimônio humano já alcançado, podemos afirmar que o
conhecimento científico não é perigoso. O conhecimento é um bem em
si mesmo. Para o ser humano, conhecer é tão vital quanto alimentar-
se, defender-se ou amar. Já a tecnologia, contrariamente, pode ser
tanto uma dádiva quanto uma maldição. Há processos tecnológicos
intrinsecamente perversos, como a fabricação de instrumentos de
tortura, armas bacteriológicas, etc. Como resume Bunge, "não se
trata do mau uso imprevisto de um setor de conhecimento, como seria
o mau uso de uma tesoura ou de um fósforo. A tecnologia da maldade é
maldosa" (Bunge: 1980, p. 202).
>
> Quando a pesquisa científica é posta em prática — por exemplo, em
experimentos que envolvam seres humanos ou outros animais —, ou
quando a ciência é aplicada à tecnologia, problemas éticos
relevantes podem e devem ser levantados. Mas aqui é importante
distinguir ciência de tecnologia, pois suas motivações são
diferentes. Em poucas palavras, ciência (básica) produz idéias,
teorias; tecnologia produz objetos, bens. Uma visa simplesmente
conhecer; outra é voltada para fins práticos.
>
> Convém observar que a tecnologia é muito mais antiga que a ciência
e possui uma história própria. Todos os povos produziram
tecnologias, mas só o povo grego criou a ciência de que somos
herdeiros. Num belo livro, o historiador da tecnologia George
Basalla demonstra que, até o século XIX, a ciência exerceu pouco
impacto sobre a tecnologia. Sem auxílio da ciência, a tecnologia
gerou a agricultura, os artefatos de metais, as conquistas da
engenharia chinesa e até mesmo as catedrais do Renascimento. Essas
imponentes construções foram erguidas por engenheiros que se
baseavam na experiência prática, aprendendo diariamente com os
erros, e não em teorias científicas. Prevalecia então, como sugere
outro autor, "o teorema dos cinco minutos" – se uma estrutura
permanecesse de pé por cinco minutos depois de tirados os suportes,
presumia-se que se manteria de pé para sempre (cf. Basalla: 1999; e
Wolpert: 1996).
>
> A esta altura, impõe-se indagar quais são, afinal, as
responsabilidades e obrigações morais dos cientistas. Não há dúvida
de que eles possuem deveres distintos das obrigações dos demais
cidadãos. Posto que os cientistas detêm conhecimento especializado
sobre como é e como funciona o mundo, e isto nem sempre é acessível
aos outros, é obrigação deles tornar públicas as implicações sociais
de seu trabalho e suas aplicações tecnológicas" (cf. artigo de
Wolpert na revista Nature, 398 (1999), p. 281-82; e Wolpert: 1996,
p. 185 e segs.).
>
> Se ciência e ética, como vimos, são distintas, nem por isso o
cientista está isento de deveres éticos. O biólogo inglês Lewis
Wolpert aponta, a propósito, um exemplo de comportamento imoral por
parte dos cientistas no movimento da eugenia, iniciado na Inglaterra
no final do século XIX, estendendo-se depois aos EUA. O movimento,
cuja pretensão era "melhorar as raças", envolveu inicialmente nomes
ilustres como Galton (criador do próprio conceito), Fisher, Haldane,
Huxley, Morgan, Davenport, Havelock Ellis e até o literato Bernard
Shaw. Não demorou que se passasse a considerar hereditário não só o
talento, mas a pobreza; que se considerasse os
negros "biologicamente inferiores" e que algumas "raças"
possuíam "tendência à debilidade mental".
>
> A Sociedade Eugênica Americana chegou a promover concursos
para "famílias geneticamente sãs", qualificando, em seu "catecismo
eugênico", o "plasma germinal humano" como "a coisa mais preciosa do
mundo". Para impedir a "contaminação" dos plasmas, a receita era a
esterilização em massa. Estima-se que, entre 1907 e 1928, nove mil
pessoas foram submetidas a tal tratamento, sob a genérica etiqueta
de "debilidade mental". E pense-se no horror nazista: a lei sobre
esterilização eugênica, que Hitler decretou em 1933, foi o primeiro
passo para as atrocidades cometidas pelos médicos nos campos de
concentração (ver Wolpert: 1996, p. 194-98). Em relação à eugenia,
portanto, está claro que os cientistas não assumiram suas obrigações
éticas.
>
> Diverso foi o comportamento dos pesquisadores envolvidos na
construção da bomba atômica, um empreendimento tecnológico baseado
em conhecimento científico. Aqui podemos perceber claramente como a
confusão entre ciência e tecnologia conduziu a uma visão errônea
sobre o papel da ciência. As aplicações desta não são,
necessariamente, responsabilidade dos cientistas: as decisões cabem,
muito mais, a governantes e políticos. No caso da bomba atômica, a
responsabilidade foi assumida exclusivamente pelo presidente
Roosevelt, como demonstra o jornalista norte-americano Richard
Rhodes num livro admirável, ao qual remeto: The making of the atomic
bomb: 1988). Em outras palavras, a decisão foi política, não
científica.
>
> Quem primeiro teve a idéia de uma possível reação em cadeia de
nêutrons foi o físico húngaro Leo Szilard, então residente na
Inglaterra. Através de Einstein, ele comunicou essa possibilidade a
Roosevelt, que autorizou a montagem de um gigantesco projeto
(secreto), envolvendo cientistas e engenheiros. Antes mesmo do
primeiro teste nuclear (15 de julho de 1945), porém, Szilard
demonstrou-se preocupado com uma operação sobre a qual, em
realidade, os cientistas tinham pouco ou nenhum controle. Chegou a
pensar, inclusive, num controle internacional que evitasse o
monopólio norte-americano da bomba. Com a II Guerra chegando ao
final, pensava ele, não havia razões para a utilização dessa arma.
Szilard fez então circular uma petição, firmada por 66 cientistas
que trabalhavam no projeto, a ser enviada ao presidente Truman,
sucessor de Roosevelt (morto em maio de 45).
>
> Argumentam os subscritores que "uma nação que estabelece o
precedente de usar as forças da natureza recentemente desencadeadas
com fins destrutivos, poderá ter que assumir também a
responsabilidade de ter aberto as portas a uma era de devastação em
dimensões inimagináveis." Por isso, pediam eles que o presidente
usasse suas prerrogativas para impedir que os Estados Unidos
recorressem ao emprego de bombas atômicas, salvo no caso de o Japão
rejeitar as condições de rendição que lhe fossem impostas, e depois
que tais condições fossem de amplo domínio público (Rhodes: 1988, p.
749 e segs.).
>
> O fato é que a carta jamais chegou às mãos do presidente. No dia 6
de agosto de 1945, como se sabe, a bomba destruiu Hiroshima. Quanto
a Szilard, dedicou-se depois da guerra a divulgar ao público as
implicações do conhecimento científico. Jamais se cansou de
ressaltar a necessidade de o público ser informado tanto sobre a
ciência quanto sobre suas aplicações. Cumpriu à risca, portanto, o
dever ético de todo cientista.
>
> Orlando Tambosi
>
> Textos citados
>
>
> Basalla, George. The history of tecnology. Cambridge, Cambridge
University Press, 8ª ed., 1999.
> Bunge, Mario. Epistemologia. São Paulo, T. A. Queiroz, 2ª, 1987.
> ______. La ciencia. Su método e su filosofía. Buenos Aires,
Sudamericana, 3ª, 1998.
> Colletti, Lucio. Pagine di filosofia e politica. Milão, Rizzoli,
1989.
> _____. Fine della filosofia e altri saggi. Roma, Ideazione, 1996.
> Dawkins, Richard. O rio que saía do Éden. Rio de Janeiro, Rocco,
1996.
> Gross, Paul, e Levitt, Norman. Higher superstition. The academic
left and its quarrels with science. Baltimore, John Hopkins
University Press, 2ª ed., 1998.
> Marcuse, Herbert. O homem unidimensional. A ideologia da sociedade
industrial. Rio de Janeiro, Zahar, 5ª ed., 1979.
> _____. Razão e revolução. Rio de Janeiro, Paz e Terra, 2ª ed.,
1978.
> Monod, Jacques. O acaso e a necessidade. Ensaio sobre a filosofia
natural da biologia moderna. Petrópolis, Vozes, 4ª ed., 1989.
> Rhodes, Richard. The making of atomic bomb. Nova York, Simon &
Schuster, 1988.
> Tambosi, Orlando. O declínio do marxismo e a herança hegeliana.
Florianópolis, Editora da UFSC, 1999.
> Wolpert, Lewis. La natura innaturale della scienza. Bari, Edizioni
Dedalo, 1996.
>
>
> [As partes desta mensagem que não continham texto foram removidas]





SUBJECT: Re: [ciencialist] Re: Fw: Teoria da Relatividade Especial
FROM: "Prof. JC" <profjc2003@yahoo.com.br>
TO: <ciencialist@yahoogrupos.com.br>
DATE: 08/01/2005 01:11

Oi dfahlb,

A velocidade de propagação do som em um dado meio não depende da velocidade
da fonte ou do receptor. Isso é "fato".

Veja bem, dado que exista um meio de propagação, como o ar, por exemplo,
para qualquer velocidade do emissor do som em relação a esse meio (ar) e do
receptor, o som se deslocará sempre com a mesma velocidade medida em relação
ao meio.

Se o meio se desloca então a velocidade de propagação do som nele também é a
mesma em relação a esse meio, mas não será a mesma se medida de outro
referencial. Por exemplo, dentro de um jato a 300 m/s o som desloca-se a
aproximadamente 340 m/s dentro do avião e, para um observador fixo na Terra
com uma velocidade de 300+340 = 640 m/s.

É claro que se a velocidade do som for medida em relação a um referencial
que se mova em relação ao meio, então ela será diferente. É o caso, por
exemplo, de se tomar como referencial um emissor (ou receptor) que esteja se
movendo em relação ao meio.

O que diz a teoria da relatividade restrita é a velocidade da luz medida
dentro do avião ou fora dele será de aproximadamente 300.000 km/s, em ambos
os casos. Isto é, em qualquer referencial ela sempre terá o mesmo valor.

Ajudou?

Abraços,
Prof. JC


----- Original Message -----
From: "dfahlb" <dfahlb@yahoo.com>
To: <ciencialist@yahoogrupos.com.br>
Sent: Friday, January 07, 2005 3:48 PM
Subject: [ciencialist] Re: Fw: Teoria da Relatividade Especial




--- Em ciencialist@yahoogrupos.com.br, "Prof. JC" <profjc2003@y...>
escreveu
> dfahlb,
>
> A velocidade do som independe da velocidade da fonte ou do receptor
e
> depende apenas do meio de propagação e da velocidade relativa desse
meio em
> relação a um dado referencial. Acho que você fez alguma confusão
ai... Ou
> será que fui eu?

Mas depende. Veja, se depende do meio de propagação, vai depender de
como os observadores (receptores, no caso) estão se movendo em
relação a esse meio.

Pense num jato supersônico: quando ele passa da velocidade do som,
nenhum som chega até ele. É claro que o som precisa de um meio para
se propagar. Mas eu me referia é do movimento dos observadores e
fontes com relação à esse meio ou à fonte. É claro que a fonte se
movimentando no meio mudará a frequência/comprimento de onda para um
observador estacionário ao meio.

>
> O que depende da velocidade da fonte e do emissor em relação ao
meio de
> propagação do som é a freqüência (ou, equivalentemente, o
comprimento de
> onda) percebida pelo receptor.
>
> Abraços,
> Prof. JC

O que você está a discutir agora é o Efeito Doppler. O Efeito Doppler
te explica justamente isso: de como a frequência ou o comprimento de
onda mudam com a velocidade da fonte ou do receptor em relação ao
meio. O Efeito Doppler é derivado das Transformações de Galileu, etc.

O Efeito Doppler serve para explicar o porquê de quando o jato
supersônico (do exemplo acima) ao atravessar a barreira do som, o
piloto e passageiros presentes ouvem um estrondol

Abraços.

>
>






##### ##### #####

Para saber mais visite
http://www.ciencialist.hpg.ig.com.br


##### ##### ##### #####
Links do Yahoo! Grupos












SUBJECT: Júpiter de dia, o cometa "verde", a ISS sobre Lisboa e o mergulho em Titá
FROM: Maria Natália <grasdic@hotmail.com>
TO: ciencialist@yahoogrupos.com.br
DATE: 08/01/2005 02:56


Notícias de astronomia

Pois na manhã de dia 5, pelas 8h da manhã vi Júpiter. Tirando a
ocasião de eclipse do Sol, foi a primeira vez que vi aquele pontinho
brilhante, aqui, à esquerda da Lua em fase minguante. A lu me
ensinou o caminho.

Cometa verde está aqui em Portugal muito bonito ali para os lados
das Pleíadas:
Green Comet
http://science.nasa.gov/headlines/y2005/05jan_machholz.htm

A queda da sonda Cassini em Titan:
Parachuting to Titan
http://science.nasa.gov/headlines/y2004/30dec_titan.htm

E a estação espacial a passar sobre Lisboa. Usando os mesmo sites
dados podem programar este acontecimento para a vossa localidade.
Ela brilha muito e se vê muito bem


by Science @ NASA

Help for this report is available on the J-Pass web pages.
http://science.nasa.gov/RealTime/JPass/PassGenerator/help.html


Start Time : 1/07 Latitude : 38.7200012207
End Time : 1/10 Longitude: -9.1300001144
UTC-1 Not Daylight Savings.

>> STATION approx. vis. mag. -1
Date Dur. Lit Dur. Max.
mm/dd Rise Set mm:ss mm:ss Elev.
---------------------------------------------------
1/09 06:10:45pm SW NE 10:19 04:33 79 S
1/10 05:01:59pm SSW ENE 09:40 07:09 23 SE
__-_-___--__--_------
E amanhã vou para a Atalaia das 20h às 2 da madrugada

Um abraço
Maria Natália






SUBJECT: RE: [ciencialist] Re: É o conhecimento perigoso?
FROM: "murilo filo" <avalanchedrive@hotmail.com>
TO: ciencialist@yahoogrupos.com.br
DATE: 08/01/2005 11:53

O conhecimento SEMPRE foi perigoso e seguirá sendo!
O Gilberto Dupas, no Estadão de hoje, trata praticamente do mesmo tema, a
que chama de ''próteses tecnológicas'', de que depende hoje a vida moderna.
Acho também que o perigo tem aumentado.
Poucos possuem a consciência de que um ser humano é autônomo, é um segmento
individual e bêbado de 'querer', 'fazer', 'pensar', alterar, desejar, etc. O
'querer e fazer', altamente racional OU emocional e até louco, causam os
mesmos efeitos no mundo externo e nas pessoas à volta.
Todos nascemos com esta capacidade modificadora individual; a tecnologia
PODE vir a ser a extensão deste braço irresponsável e também a causa da
vulnerabilidade das pessoas. abr/M.

>From: Maria Natália <grasdic@hotmail.com>
>Reply-To: ciencialist@yahoogrupos.com.br
>To: ciencialist@yahoogrupos.com.br
>Subject: [ciencialist] Re: É o conhecimento perigoso?
>Date: Sat, 08 Jan 2005 02:20:38 -0000
>
>
>Texto muito bom, este. Gostei e continua.
> Vamos fazendo revisões da matéria já dada há uns tempos.
>Obrigada
>Um abraço
>Maria Natália
>--- Em ciencialist@yahoogrupos.com.br, "Amauri Jr"
><amaurijunior2@y...> escreveu
> >
> > É o conhecimento perigoso?
> > Fronteiras entre ciência, tecnologia e ética
> > Orlando Tambosi
> >
> >
> > 1. As feridas da modernidade e a anticiência
> >
> >
> > O traço mais profundo e mais perturbador de nossa época é a
>dissociação de fato e valor, ser e dever ser, ou física e ética,
>conhecimento da realidade e atribuição de sentido à vida. As
>ciências descrevem e conhecem o mundo tal qual é, mas calam sobre as
>angústias humanas, tornando o homem praticamente um acidente no
>cosmo. Despertando de seu sonho milenar, como diz o biólogo Monod, o
>ser humano agora "sabe que, como um cigano, está à margem do
>Universo onde deve viver. Um Universo surdo à sua música,
>indiferente às suas esperanças, como a seus sofrimentos ou a seus
>crimes". Ele sabe que "está sozinho na imensidão indiferente do
>Universo, de onde emergiu por acaso. Não mais do que seu destino,
>seu dever não está escrito em lugar algum" (Monod: 1989, p. 190-8).
> >
> > Através da ciência, a modernidade rompeu a "aliança animística"
>entre homem e Natureza, calcada exatamente na identificação de fato
>e valor - fundamento da visão antropocêntrica do mundo. A cosmologia
>medieval (aristotélico-cristã) realizava a coincidência plena disso
>que, para nós, é dividido: conhecimento da realidade e compreensão
>do "sentido" da nossa vida – sua destinação ou valor – eram uma só
>coisa. Por mais de dois mil anos, a metafísica (o nome remete, como
>se sabe, ao conhecimento do transcendente ou do supra-sensível)
>sustentou a separação entre mundo terrestre e mundo celeste:
>embaixo, o reino do efêmero, do nascer e do perecer; no alto, com
>suas esferas perfeitas, o reino do divino, do incorruptível, do
>eterno, do verdadeiro Ser. Os níveis de realidade exprimem ao mesmo
>tempo uma hierarquia de valores. A Terra, no centro, é o palco em
>que se desenrola o drama humano, em vista do qual o próprio cosmo
>foi criado.
> >
> > A modernidade rompe essa imagem. A revolução astronômica explode
>esse cosmo finito e fechado, revelando um universo de proporções
>ilimitadas. A Terra já não é mais o centro de nada. "É um ponto
>infinitesimal, uma minúscula ilha perdida num oceano sem praias,
>onde se contam bilhões de galáxias, cada uma delas com centenas de
>bilhões de sóis. Explicar essa realidade em função do homem, ou dela
>extrair um significado para a nossa existência, é simplesmente
>impossível" (Colletti: 1989, p. III-IV, e 1996, p.15).
> >
> > Depois que Copérnico arrancou o homem do centro do universo,
>Darwin obrigou-o a reconhecer que não passa de um ser entre outros
>no reino animal (competindo com as outras espécies e,
>freqüentemente, perdendo a luta para as mais microscópicas). São
>duas feridas insanáveis que corroem o narcisismo humano, como
>definiu Freud, e que produzem mal-estar ainda hoje. Daí a
>hostilidade em relação às ciências e às tecnologias, comum a algumas
>vertentes filosóficas e tendências culturais contemporâneas,
>particularmente as que se autodenominam "pós-modernas". A
>anticiência, por sinal, encontra confortável abrigo nas ciências
>sociais e humanidades, minadas pelo relativismo cognitivo e
>cultural; e, junto com as pseudociências, conta com generoso espaço
>na mídia.
> >
> > Muito do que se produz nessas áreas é hostil a conceitos
>como "realidade", "objetividade", "verdade", fundamentais tanto à
>ciência quanto ao jornalismo científico. Para a cultura "pós-
>moderna", o "real", os "fatos" que as ciências buscam conhecer – e o
>jornalismo reportar - não passam de "construções intelectuais". Mero
>discurso ou "narrativa", a ciência é ideológica, isto é, instrumento
>de dominação de uma
>civilização "branca", "eurocêntrica", "opressora", "machista", "heter
>ossexual" etc. (ver, a respeito, Gross e Levitt: 1998 - livro que
>inspirou Alan Sokal e Jean Bricmont a escreverem seu Imposturas
>intelectuais, Rio de Janeiro, Record, 1999, outra consistente
>denúncia do relativismo e da falta de rigor nas humanidades).
> >
> > Sob essa bandeira campeiam os multiculturalismos, o social-
>construtivismo, o ecofeminismo, os estudos culturais, as leituras
>de "gênero", o ressentimento contra as ciências. Privilegiam-se o
>intuitivo, o mágico, o místico, o irracional, o marginal, abrindo-se
>as portas da academia para a New Age, as bruxas, o tarô, o
>ocultismo, a astrologia – temas freqüentes junto a certos
>comunicólogos, notadamente os de formação antropológica.
> >
> > Diante disso, não espanta a condenação, dentro das próprias
>universidades, não só da ciência e da tecnologia, mas também da
>racionalidade e da secularização, "desencantadoras do mundo": não
>por acaso, fenômenos produzidos pela modernidade. Não é o "pós-
>modernismo" justamente esse conjunto de atitudes estilísticas e
>julgamentos contrários ao que se supõe ser ou ter sido a modernidade
>(em especial, ao que ela herdou do Iluminismo)? Não espanta,
>igualmente, que universidades de prestígio tragam ao Brasil, às
>custas do dinheiro público, sociólogos delirantes como Jean
>Baudrillard, que, a cada três meses, vem nos advertir que a
>realidade não existe.
> >
> > Afinal, não nos garante essa filosofia de salão chamada
>relativismo cultural que a ciência não tem mais direito em afirmar a
>verdade do que o mito tribal?; ou que a ciência é apenas a mitologia
>adotada por nossa tribo ocidental moderna? Vale lembrar, a
>propósito, um curioso relato do biólogo Richard Dawkins, hoje
>professor da cátedra de Compreensão Pública da Ciência em Oxford,
>cuja obra deve, necessariamente, figurar numa bibliografia de
>jornalismo científico. Conta ele que, certa vez, respondendo a uma
>provocação de um colega antropólogo, colocou-lhe a seguinte
>questão: "Suponha que existe uma tribo que acredita que a Lua é uma
>cabaça velha lançada aos céus, pendurada fora de alcance um pouco
>acima do topo das árvores. Você afirma realmente que nossa verdade
>científica – que afirma que a Lua está a 382 mil quilômetros
>afastada e tem um quarto do diâmetro da Terra – não é mais
>verdadeira do que a cabaça da tribo?" A resposta do antropólogo foi
>direta: "Sim. Nós apenas fomos criados em uma cultura que vê o mundo
>de um modo científico. Eles foram criados para ver o mundo de outro
>modo. Nenhum desses modos é mais verdadeiro do que o outro". Conclui
>Dawkins: "aponte-me um relativista cultural a 10 quilômetros de
>distância e lhe mostrarei um hipócrita. Aviões construídos de acordo
>com princípios científicos funcionam. Eles mantêm-se no ar e o levam
>ao seu destino escolhido. Aviões construídos de acordo com
>especificações tribais ou mitológicas, tais como os aviões de
>imitação dos cultos de carregamento nas clareiras das selvas (...),
>não funcionam. Se você estiver voando para um congresso
>internacional de antropólogos ou de críticos literários, a razão
>pela qual você provavelmente chegará lá (...) é que uma multidão de
>engenheiros ocidentais cientificamente treinados realizou os
>cálculos corretamente. A ciência ocidental, com base na evidência
>confiável de que a Lua orbita em torno da Terra a uma distância de
>382 mil quilômetros, conseguiu colocar pessoas em sua superfície. A
>ciência tribal, acreditando que a Lua estava um pouco acima do topo
>das árvores, nunca chegará a tocá-la, exceto em sonhos" (Dawkins:
>1996, p. 39-40).
> >
> > Se a anticiência, atualmente, procede do circuito Paris-Nova York,
>de lá se espraiando para outros países, no século XX foi da Alemanha
>que partiram os ataques mais fortes e duradouros. Os precedentes são
>longínquos: nem o grande filósofo idealista G. W. F. Hegel (1770-
>1831) pouparia críticas tanto às ciências quanto aos cientistas e
>filósofos mais próximos de uma perspectiva científica. Nunca
>escondeu, por exemplo, sua má-vontade em relação a Newton (1642-
>1727), o pai da física moderna, e a F. Bacon (1561-1626), fundador
>do método indutivo moderno e precursor da sistematização dos
>procedimentos científicos.
> >
> > 2. De Marcuse ao Unabomber
> >
> > A partir dos anos 40 do século passado, a chamada Escola de
>Frankfurt é que se encarregaria de fomentar, por trás de sua crítica
>ao capitalismo, uma das mais persistentes e influentes críticas à
>racionalidade científica, com profundas repercussões nos movimentos
>estudantis da Europa da década de 60. "A física é burguesa", "a
>ciência é o capital": estas inscrições, nos muros da Paris de 68,
>resumiam, na verdade, os temas de Adorno, Horkheimer e,
>principalmente, Herbert Marcuse (1898-1979), o guru dos revoltosos
>(um dos três grandes "M" da época, junto com Marx e Mao).
> >
> > Para Marcuse, ciência e capitalismo são uma só coisa. Em outras
>palavras, ciência (conhecimento racional e objetivo) e ideologia
>(concepção de mundo) se confundem. Desaparece o valor objetivo do
>conhecimento científico. A crítica da "razão instrumental" –
>ou "razão unidimensional", ou "razão técnica" – encerra, no fundo,
>uma crítica da própria Civilização. Daí o ataque à "sociedade
>industrial" ou "tecnológica", justamente a sociedade moderna,
>baseada na ciência e na tecnologia.
> >
> > Apenas os filósofos italianos (especialmente Galvano Della Volpe e
>Lucio Colletti) perceberam esta trágica confusão, denunciando in
>loco a "Grande Recusa" marcusiana como a retomada de temas
>irracionalistas e românticos. A "contracultura" gerada neste
>ambiente cultural, no entanto, fixaria raízes e amoldaria mentes;
>boa parte da geração que, nas humanidades, cresceu ouvindo essas
>melancólicas diatribes contra a racionalidade científica, a técnica,
>a "indústria cultural", etc., hoje as reproduz nas universidades e
>nas revistas acadêmicas, quando não nos jornais. Principalmente no
>Brasil, onde ainda há saudosos das "barricadas do desejo" de 68 e o
>prestígio dos "frankfurtianos" continua incólume entre muitos
>intelectuais.
> >
> > Por brevidade, considerarei aqui apenas as idiossincrasias
>anticientíficas de Marcuse. Não é necessário rastrear muito para se
>deparar, em sua obra, com inspirações irracionalistas-românticas.
>Aliás, elas percorrem toda a sua teoria: já num escrito de 1933
>(Sobre os fundamentos filosóficos do conceito de trabalho na ciência
>econômica), sua polêmica era contra a objetividade, com
>a "submissão" do homem às coisas. Independentemente das épocas
>históricas, o trabalho sempre foi, para ele, "trabalho alienado" (o
>marxismo marcusiano confundia o que para Marx era
>distinto: "objetivação" e "alienação").
> >
> > Para Marcuse, eliminar a "alienação" é eliminar a própria
>objetividade. Essa "superação", portanto, não pode ser buscada no
>trabalho, mas... no jogo. É somente no jogo que o homem "não se
>conforma aos objetos, à sua regularidade". Somente ao colocar-
>se "acima da objetividade" é que o homem alcança a si próprio, "numa
>dimensão de sua liberdade que é negada no trabalho". Para o filósofo
>alemão, "um simples lance de bola, por parte de um jogador,
>representa um triunfo da liberdade humana sobre a objetividade que é
>infinitamente maior que a mais estrondosa conquista do trabalho
>técnico" (cit. em Tambosi: 1999, p. 150 – são do livro também as
>citações seguintes).
> >
> > E pense-se no Marcuse de Razão e revolução (1941), em que afirma
>que "a razão é a verdadeira forma da realidade", onde "todos os
>antagonismos do sujeito e do objeto são integrados". Hegel, afinal,
>já dissera que o real é o racional. Mas é no seu livro mais célebre –
> O homem unidimensional (1964) – que Marcuse transformará sua
>rejeição à objetividade num ataque à racionalidade científica.
>O "domínio", agora, estava inscrito na própria tecnologia.
>A "alienação" surge da produção industrial. A ciência, mais uma vez,
>é ideologia.
> >
> > A última esperança, para ele, eram o Lumpenproletariat das
>metrópoles e as massas pobres do chamado "Terceiro Mundo", cuja
>oposição "é revolucionária, ainda que sua consciência não o seja".
>Tudo isto antecipava temas que dominariam os discursos nos anos
>seguintes. A "Grande Recusa" influenciaria não só a "Nova Esquerda"
>européia, mas também o costume e a mentalidade comuns. Ciência e
>capital eram uma só coisa: os males que o marxismo havia denunciado
>no capitalismo eram descarregados por Marcuse (e, diga-se, também
>por Adorno e Horkheimer) "nos ombros de Galilei e Bacon". O desastre
>havia começado já com a revolução científica do século XVII.
> >
> > É a isto que os pensadores italianos chamam de "reação idealista"
>contra as ciências e a técnica. Que chega, no caso de Marcuse, à
>negação total do existente: além de atacar o capitalismo, ele
>condenava também o socialismo por "submeter-se ao aparato
>tecnológico". O filósofo sonhava com uma "nova ciência" e uma "nova
>técnica" – sobre as quais não forneceu jamais a mínima indicação. O
>problema da teoria marxista clássica, segundo ele, estava em
>conceber "a transição do capitalismo para o socialismo como uma
>revolução política", isto é, em destruir o "aparato político", mas
>não o "aparato tecnológico"!
> >
> > No capitalismo avançado – prossegue Marcuse – "a racionalidade
>técnica está personificada, a despeito de seu uso irracional, no
>aparato produtor. Isto não se aplica apenas às fábricas mecanizadas,
>ferramentas e exploração de recursos, mas também à maneira de
>trabalhar como adaptação ao processo mecânico do mesmo, conforme
>programado pela `gerência científica'. Nem a nacionalização nem a
>socialização alteram por si essa personalização física da
>racionalidade tecnológica; pelo contrário, esta permanece uma
>condição prévia para o desenvolvimento socialista das forças
>produtivas" (Marcuse: 1989, p. 41).
> >
> > Estamos no coração da "Grande Recusa", uma herança ideológica que,
>junto às filosofias "pós-modernas", ainda inspira manifestações
>contra a ciência e a tecnologia. O alvo, agora, é o vertiginoso
>processo de informatização. Lado a lado com a chamada III revolução
>industrial, crescem também a tecnofobia e a rejeição das
>tecnologias. O mal-estar assume novas – e até violentas – formas.
>Basta lembrar o caso do Unabomber, nos EUA, que enviava cartas-
>bombas para cientistas e universidades. Também nesse país, um
>intelectual que escreveu livros como Rebeldes contra o futuro e A
>revolução verde ilustra suas conferências quebrando computadores a
>golpes de martelo.
> >
> > Recorde-se que o manifesto do terrorista Unabomber, publicado em
>setembro de 1995 pelo Washington Post, elege como inimiga
>a "sociedade industrial" (curiosamente, um conceito marcusiano), que
>ele considera "um desastre para a espécie humana" e contra a qual
>propõe uma "revolução": "a única saída" – pontifica – "é dispensar o
>sistema tecnológico inteiro". Seu temor são as "máquinas
>inteligentes", que acabarão por decidir no lugar da
>humanidade. "Quando chegar a esse estágio, as máquinas estarão,
>efetivamente, no controle. As pessoas não poderão simplesmente
>desligar as máquinas porque elas estarão tão dependentes delas que
>desligá-las equivaleria a cometer suicídio". A estratégia? "Promover
>o estresse social e a instabilidade na sociedade industrial, e
>desenvolver e difundir uma ideologia que se oponha à tecnologia e ao
>sistema industrial". O terrorista quer simplesmente "a eliminação da
>tecnologia moderna" (Folha de S. Paulo, 20/09/95).
> >
> > Kirkpatrick Sale, o destruidor de computadores e autoproclamado
>líder do "neoludismo" (herdeiro do movimento de desempregados
>ingleses que, entre 1811 e 1813, quebravam máquinas em protesto
>contra a revolução industrial), compartilha esse mal-estar em
>relação à sociedade moderna. Para ele, "a civilização é catastrófica
>porque destrói a si mesma e o ambiente natural", e "o uso da ciência
>e das suas tecnologias é um atentado à Natureza, uma tentativa de
>criar uma natureza tecnológica, de modo que a humanidade possa
>controlar todas as coisas" (L'Espresso, 11/08/95). Do Unabomber,
>Sale só discorda quanto aos "métodos", porque "a intenção é boa".
> >
> > No "paraíso" vislumbrado por Sale desaparecem os produtos
>tecnológicos: do computador ao forno de microondas, da videocâmera
>ao telefone digital. O automóvel é demoníaco. Voltemos às
>bicicletas, recomenda ele. Não é à-toa que nos EUA já exista, entre
>as associações antitecnológicas, até um "Clube do Lápis", que
>defende a escrita à mão. A utopia do último dos luditas, como quase
>todas as utopias anticientíficas contemporâneas, é uma volta ao
>passado.
> >
> > 3. É o conhecimento perigoso?
> >
> > A julgar pelas vertentes e tendências aqui apontadas, a resposta é
>positiva. Na verdade, a idéia de que o conhecimento é perigoso está
>arraigada na nossa cultura. Já Adão e Eva, segundo a Bíblia, foram
>proibidos de alimentar-se dos frutos da Árvore do Conhecimento.
>Prometeu foi punido por ter dado o saber ao mundo. Na literatura, o
>Dr. Frankenstein é a imagem do cientista, pintado como um arrogante
>desalmado que de tudo é capaz para atingir seus objetivos, quaisquer
>que sejam as conseqüências. No cinema, é o gênio louco que produz
>monstros e catástrofes.
> >
> > Imoral manipulador da Natureza, o cientista também foi
>responsabilizado pela construção da bomba atômica e, agora, é visto
>com suspeita em virtude da engenharia genética. Jornais e revistas
>publicam com freqüência textos alarmistas que advertem sobre
>os "perigos" da pesquisa genética (lembre-se a histeria sobre a
>clonagem), do projeto do genoma humano e dos transgênicos ("comida
>Frankenstein"). Nos títulos, invariavelmente, a insinuação de que o
>cientista "brinca de ser Deus". O horror, porém, convive com o
>fascínio, já que se espera da ciência a solução para a cura do
>câncer e da Aids, entre outras doenças.
> >
> > A análise desse problema nos remete, de novo, à separação moderna
>de fatos e valores, ou seja, de ciência e ética. Como processo de
>conhecimento racional e objetivo, a ciência não é guiada por
>valores. Ela apenas nos mostra como o mundo é. A ciência descreve, a
>ética prescreve; a ciência explica, a ética avalia. Ciência,
>portanto, não produz ética. Das proposições descritivas não é
>possível deduzir asserções prescritivas, como bem viu o filósofo
>Hume (1711-1776). A separação de fatos e valores — conhecida
>justamente como Lei de Hume — impede que do "é "derive o "deve", que
>do "ser" derive o "dever ser".
> >
> > Em oposição a essas tendências filosóficas e culturais, e
>considerando o patrimônio humano já alcançado, podemos afirmar que o
>conhecimento científico não é perigoso. O conhecimento é um bem em
>si mesmo. Para o ser humano, conhecer é tão vital quanto alimentar-
>se, defender-se ou amar. Já a tecnologia, contrariamente, pode ser
>tanto uma dádiva quanto uma maldição. Há processos tecnológicos
>intrinsecamente perversos, como a fabricação de instrumentos de
>tortura, armas bacteriológicas, etc. Como resume Bunge, "não se
>trata do mau uso imprevisto de um setor de conhecimento, como seria
>o mau uso de uma tesoura ou de um fósforo. A tecnologia da maldade é
>maldosa" (Bunge: 1980, p. 202).
> >
> > Quando a pesquisa científica é posta em prática — por exemplo, em
>experimentos que envolvam seres humanos ou outros animais —, ou
>quando a ciência é aplicada à tecnologia, problemas éticos
>relevantes podem e devem ser levantados. Mas aqui é importante
>distinguir ciência de tecnologia, pois suas motivações são
>diferentes. Em poucas palavras, ciência (básica) produz idéias,
>teorias; tecnologia produz objetos, bens. Uma visa simplesmente
>conhecer; outra é voltada para fins práticos.
> >
> > Convém observar que a tecnologia é muito mais antiga que a ciência
>e possui uma história própria. Todos os povos produziram
>tecnologias, mas só o povo grego criou a ciência de que somos
>herdeiros. Num belo livro, o historiador da tecnologia George
>Basalla demonstra que, até o século XIX, a ciência exerceu pouco
>impacto sobre a tecnologia. Sem auxílio da ciência, a tecnologia
>gerou a agricultura, os artefatos de metais, as conquistas da
>engenharia chinesa e até mesmo as catedrais do Renascimento. Essas
>imponentes construções foram erguidas por engenheiros que se
>baseavam na experiência prática, aprendendo diariamente com os
>erros, e não em teorias científicas. Prevalecia então, como sugere
>outro autor, "o teorema dos cinco minutos" – se uma estrutura
>permanecesse de pé por cinco minutos depois de tirados os suportes,
>presumia-se que se manteria de pé para sempre (cf. Basalla: 1999; e
>Wolpert: 1996).
> >
> > A esta altura, impõe-se indagar quais são, afinal, as
>responsabilidades e obrigações morais dos cientistas. Não há dúvida
>de que eles possuem deveres distintos das obrigações dos demais
>cidadãos. Posto que os cientistas detêm conhecimento especializado
>sobre como é e como funciona o mundo, e isto nem sempre é acessível
>aos outros, é obrigação deles tornar públicas as implicações sociais
>de seu trabalho e suas aplicações tecnológicas" (cf. artigo de
>Wolpert na revista Nature, 398 (1999), p. 281-82; e Wolpert: 1996,
>p. 185 e segs.).
> >
> > Se ciência e ética, como vimos, são distintas, nem por isso o
>cientista está isento de deveres éticos. O biólogo inglês Lewis
>Wolpert aponta, a propósito, um exemplo de comportamento imoral por
>parte dos cientistas no movimento da eugenia, iniciado na Inglaterra
>no final do século XIX, estendendo-se depois aos EUA. O movimento,
>cuja pretensão era "melhorar as raças", envolveu inicialmente nomes
>ilustres como Galton (criador do próprio conceito), Fisher, Haldane,
>Huxley, Morgan, Davenport, Havelock Ellis e até o literato Bernard
>Shaw. Não demorou que se passasse a considerar hereditário não só o
>talento, mas a pobreza; que se considerasse os
>negros "biologicamente inferiores" e que algumas "raças"
>possuíam "tendência à debilidade mental".
> >
> > A Sociedade Eugênica Americana chegou a promover concursos
>para "famílias geneticamente sãs", qualificando, em seu "catecismo
>eugênico", o "plasma germinal humano" como "a coisa mais preciosa do
>mundo". Para impedir a "contaminação" dos plasmas, a receita era a
>esterilização em massa. Estima-se que, entre 1907 e 1928, nove mil
>pessoas foram submetidas a tal tratamento, sob a genérica etiqueta
>de "debilidade mental". E pense-se no horror nazista: a lei sobre
>esterilização eugênica, que Hitler decretou em 1933, foi o primeiro
>passo para as atrocidades cometidas pelos médicos nos campos de
>concentração (ver Wolpert: 1996, p. 194-98). Em relação à eugenia,
>portanto, está claro que os cientistas não assumiram suas obrigações
>éticas.
> >
> > Diverso foi o comportamento dos pesquisadores envolvidos na
>construção da bomba atômica, um empreendimento tecnológico baseado
>em conhecimento científico. Aqui podemos perceber claramente como a
>confusão entre ciência e tecnologia conduziu a uma visão errônea
>sobre o papel da ciência. As aplicações desta não são,
>necessariamente, responsabilidade dos cientistas: as decisões cabem,
>muito mais, a governantes e políticos. No caso da bomba atômica, a
>responsabilidade foi assumida exclusivamente pelo presidente
>Roosevelt, como demonstra o jornalista norte-americano Richard
>Rhodes num livro admirável, ao qual remeto: The making of the atomic
>bomb: 1988). Em outras palavras, a decisão foi política, não
>científica.
> >
> > Quem primeiro teve a idéia de uma possível reação em cadeia de
>nêutrons foi o físico húngaro Leo Szilard, então residente na
>Inglaterra. Através de Einstein, ele comunicou essa possibilidade a
>Roosevelt, que autorizou a montagem de um gigantesco projeto
>(secreto), envolvendo cientistas e engenheiros. Antes mesmo do
>primeiro teste nuclear (15 de julho de 1945), porém, Szilard
>demonstrou-se preocupado com uma operação sobre a qual, em
>realidade, os cientistas tinham pouco ou nenhum controle. Chegou a
>pensar, inclusive, num controle internacional que evitasse o
>monopólio norte-americano da bomba. Com a II Guerra chegando ao
>final, pensava ele, não havia razões para a utilização dessa arma.
>Szilard fez então circular uma petição, firmada por 66 cientistas
>que trabalhavam no projeto, a ser enviada ao presidente Truman,
>sucessor de Roosevelt (morto em maio de 45).
> >
> > Argumentam os subscritores que "uma nação que estabelece o
>precedente de usar as forças da natureza recentemente desencadeadas
>com fins destrutivos, poderá ter que assumir também a
>responsabilidade de ter aberto as portas a uma era de devastação em
>dimensões inimagináveis." Por isso, pediam eles que o presidente
>usasse suas prerrogativas para impedir que os Estados Unidos
>recorressem ao emprego de bombas atômicas, salvo no caso de o Japão
>rejeitar as condições de rendição que lhe fossem impostas, e depois
>que tais condições fossem de amplo domínio público (Rhodes: 1988, p.
>749 e segs.).
> >
> > O fato é que a carta jamais chegou às mãos do presidente. No dia 6
>de agosto de 1945, como se sabe, a bomba destruiu Hiroshima. Quanto
>a Szilard, dedicou-se depois da guerra a divulgar ao público as
>implicações do conhecimento científico. Jamais se cansou de
>ressaltar a necessidade de o público ser informado tanto sobre a
>ciência quanto sobre suas aplicações. Cumpriu à risca, portanto, o
>dever ético de todo cientista.
> >
> > Orlando Tambosi
> >
> > Textos citados
> >
> >
> > Basalla, George. The history of tecnology. Cambridge, Cambridge
>University Press, 8ª ed., 1999.
> > Bunge, Mario. Epistemologia. São Paulo, T. A. Queiroz, 2ª, 1987.
> > ______. La ciencia. Su método e su filosofía. Buenos Aires,
>Sudamericana, 3ª, 1998.
> > Colletti, Lucio. Pagine di filosofia e politica. Milão, Rizzoli,
>1989.
> > _____. Fine della filosofia e altri saggi. Roma, Ideazione, 1996.
> > Dawkins, Richard. O rio que saía do Éden. Rio de Janeiro, Rocco,
>1996.
> > Gross, Paul, e Levitt, Norman. Higher superstition. The academic
>left and its quarrels with science. Baltimore, John Hopkins
>University Press, 2ª ed., 1998.
> > Marcuse, Herbert. O homem unidimensional. A ideologia da sociedade
>industrial. Rio de Janeiro, Zahar, 5ª ed., 1979.
> > _____. Razão e revolução. Rio de Janeiro, Paz e Terra, 2ª ed.,
>1978.
> > Monod, Jacques. O acaso e a necessidade. Ensaio sobre a filosofia
>natural da biologia moderna. Petrópolis, Vozes, 4ª ed., 1989.
> > Rhodes, Richard. The making of atomic bomb. Nova York, Simon &
>Schuster, 1988.
> > Tambosi, Orlando. O declínio do marxismo e a herança hegeliana.
>Florianópolis, Editora da UFSC, 1999.
> > Wolpert, Lewis. La natura innaturale della scienza. Bari, Edizioni
>Dedalo, 1996.
> >
> >
> > [As partes desta mensagem que não continham texto foram removidas]
>
>
>




SUBJECT: Re: [ciencialist] aranhas...
FROM: "murilo filo" <avalanchedrive@hotmail.com>
TO: ciencialist@yahoogrupos.com.br
DATE: 08/01/2005 11:58

Amauri Jr!
Vc entende? Eu não posso me interessar por aranhinhas tão comuns.
A não ser que elas sejam filhotes de algum tipo mais emocionante! :]
Sorry! abr/M.

>From: "Amauri Jr" <amaurijunior2@yahoo.com.br>
>Reply-To: ciencialist@yahoogrupos.com.br
>To: <ciencialist@yahoogrupos.com.br>
>Subject: Re: [ciencialist] aranhas...
>Date: Fri, 7 Jan 2005 22:48:07 -0200
>
>Oi Murilo
>
>São Miudinhas e tecem muita teia....são meias morronzinhas e ficam no
>banheiro todo.
>
>Abr
>Amauri
> ----- Original Message -----
> From: murilo filo
> To: ciencialist@yahoogrupos.com.br
> Sent: Friday, January 07, 2005 10:46 PM
> Subject: Re: [ciencialist] aranhas...
>
>
> Amauri,
> oi.
> Sua descritiva está por demais vaga... seja + específico:
> - tamanho?
> - pernas finas?
> - teias grandes?
> Se forem miudinhas, considere-me fora do caso. Muito comuns!
> Vc disse que estavam debaixo da pia e nas paredes... hmmm.... acho
>bom,
> também, vc marcar uma reunião com a faxineira... eu sei, a ecologia
> tropical ataca! :]
> abr/M.
>
> >From: "Amauri Jr" <amaurijunior2@yahoo.com.br>
> >Reply-To: ciencialist@yahoogrupos.com.br
> >To: <ciencialist@yahoogrupos.com.br>
> >Subject: Re: [ciencialist] aranhas...
> >Date: Thu, 6 Jan 2005 23:03:51 -0200
> >
> >Oi Murilo
> >
> >São umas que fazem teias e são meia marron com a traseira pretinha...
> >
> >Amauri
> > ----- Original Message -----
> > From: murilo filo
> > To: ciencialist@yahoogrupos.com.br
> > Sent: Thursday, January 06, 2005 11:01 PM
> > Subject: Re: [ciencialist] aranhas...
> >
> >
> > Devem ser as papa-moscas, pequenas e rápidas; concorrentes das
> >lagartixas!
> > Aquí em casa, ninguém mata... inofensivas e ainda comem pernilongos!
> > Ou será que vc está falando daquelas magrelas de perninhas fininhas
>e
> >que
> > têm tremedeiras?
> > Nestas não tenho a mínima tesão! Não gosto! Sem graça e muito
>paradonas.
> > Possuem um veneno forte, mas não conseguem inocular, parece.
> > Antes, eu também repetia com outros, gozações sobre a ''cultura
> >inútil'',
> > até eu descobrir as delícias dos produtivos novos enlaces
>cerebrais...
> > My name is Discovery... abr/M.
> >
> > >From: "Amauri Jr" <amaurijunior2@yahoo.com.br>
> > >Reply-To: ciencialist@yahoogrupos.com.br
> > >To: <ciencialist@yahoogrupos.com.br>
> > >Subject: Re: [ciencialist] aranhas...
> > >Date: Thu, 6 Jan 2005 19:19:30 -0200
> > >
> > >E esses que ficam na parece ou de baixo das pias do banheiro??
> > >
> > >Amauri
> > > ----- Original Message -----
> > > From: murilo filo
> > > To: ciencialist@yahoogrupos.com.br
> > > Sent: Wednesday, January 05, 2005 10:46 PM
> > > Subject: RE: [ciencialist] aranhas...
> > >
> > >
> > > Se as teias são ''aéreas'', muito fortes, grandes e de cor meio
> >dourada,
> > >e
> > > se esta aranha, com uns 5cm, quando ameaçada, mudar de cor,
>trata-se
> >a
> > > aranha lôbo, que é boazinha, bonitinha, decorativa e útil.
> > > Se a teia for no chão, meio embolada e aberta como um funil na
>boca
> >de
> > >um
> > > pequeno túnel, trata-se da ''armadeira'', perigosa e famosa nas
> > >Discovery da
> > > vida. Dizem que é bem pior que a tal viúva negra. Boa sorte e
>faça
> >seu
> > > testamento! :] Murilo SP 05/jan/2005
> > >
> > > >From: "E m i l i a n o C h e m e l l o"
><chemelloe@yahoo.com.br>
> > > >Reply-To: ciencialist@yahoogrupos.com.br
> > > >To: <ciencialist@yahoogrupos.com.br>
> > > >Subject: [ciencialist] aranhas...
> > > >Date: Wed, 5 Jan 2005 10:14:55 -0200
> > > >
> > > >Alguém?
> > > >
> > > >[ ] 's do Emiliano Chemello
> > > >emiliano@quimica.net
> > > >http://www.quimica.net/emiliano
> > > >http://www.ucs.br/ccet/defq/naeq
> > > >
> > > >
> > > >Contato Naeq:
> > > >Nome: Camila
> > > >Email: camila_alves19@hotmail.com
> > > >Telefone: Dúvidas!!
> > > >Mensagem: Há alguns dias eu venho pesquisando sobre aranhas, no
>meu
> > >jardim
> > > >tem 2 aranhas construtoras de teia, elas são sedentárias estão
> >sempre
> > > >paradas, posicionam suas patas dianteiras para cima e suas
> >traseiras
> > >para
> > > >baixo fica parecendo que só são 4, suas patas são listradas de
> >amarelo,
> > >uma
> > > >listra grande amarela pode ser identificada em seu abdome. Mas
>a
> >minha
> > > >dúvida é se elas são venenosas. Em certos sites podemos
>encontrar
> >que
> > >as
> > > >aranhas clinicamente perigosas não são construtoras de teias e
>sim
> > > >errantes,
> > > >mas eu queria saber mais sobre essas aranhas do meu jardim,
>pois
> >não
> > > >encontro nada em sites, apenas o que tenho são essas
> >características.
> > >Se
> > > >puderem me ajudar, estou estudando para fazer faculdade de
> >biologia.
> > >Minha
> > > >curiosidade sobre assuntos biologicos é grande.
> > > >Obrigada!
> > > >
> > >
> > >
> > >
> > >
> > > ##### ##### #####
> > >
> > > Para saber mais visite
> > > http://www.ciencialist.hpg.ig.com.br
> > >
> > >
> > > ##### ##### ##### #####
> > >
> > >
> > > Yahoo! Grupos, um serviço oferecido por:
> > >
> > > São Paulo Rio de Janeiro Curitiba Porto Alegre
> >Belo
> > >Horizonte Brasília
> > >
> > >
> > >
> > >
> >
> >
> >------------------------------------------------------------------------------
> > > Links do Yahoo! Grupos
> > >
> > > a.. Para visitar o site do seu grupo na web, acesse:
> > > http://br.groups.yahoo.com/group/ciencialist/
> > >
> > > b.. Para sair deste grupo, envie um e-mail para:
> > > ciencialist-unsubscribe@yahoogrupos.com.br
> > >
> > > c.. O uso que você faz do Yahoo! Grupos está sujeito aos
>Termos do
> > >Serviço do Yahoo!.
> > >
> > >
> > >
> > >[As partes desta mensagem que não continham texto foram removidas]
> > >
> >
> >
> >
> >
> > ##### ##### #####
> >
> > Para saber mais visite
> > http://www.ciencialist.hpg.ig.com.br
> >
> >
> > ##### ##### ##### #####
> >
> >
> > Yahoo! Grupos, um serviço oferecido por:
> > PUBLICIDADE
> >
> >
> >
> >
>
> >------------------------------------------------------------------------------
> > Links do Yahoo! Grupos
> >
> > a.. Para visitar o site do seu grupo na web, acesse:
> > http://br.groups.yahoo.com/group/ciencialist/
> >
> > b.. Para sair deste grupo, envie um e-mail para:
> > ciencialist-unsubscribe@yahoogrupos.com.br
> >
> > c.. O uso que você faz do Yahoo! Grupos está sujeito aos Termos do
> >Serviço do Yahoo!.
> >
> >
> >
> >[As partes desta mensagem que não continham texto foram removidas]
> >
>
>
>
>
> ##### ##### #####
>
> Para saber mais visite
> http://www.ciencialist.hpg.ig.com.br
>
>
> ##### ##### ##### #####
>
>
> Yahoo! Grupos, um serviço oferecido por:
>
>
>
>
>
>
>
>------------------------------------------------------------------------------
> Links do Yahoo! Grupos
>
> a.. Para visitar o site do seu grupo na web, acesse:
> http://br.groups.yahoo.com/group/ciencialist/
>
> b.. Para sair deste grupo, envie um e-mail para:
> ciencialist-unsubscribe@yahoogrupos.com.br
>
> c.. O uso que você faz do Yahoo! Grupos está sujeito aos Termos do
>Serviço do Yahoo!.
>
>
>
>[As partes desta mensagem que não continham texto foram removidas]
>




SUBJECT: Maria Nathalia
FROM: "Cyberlander" <cybernews@superig.com.br>
TO: ciencialist@yahoogrupos.com.br
DATE: 08/01/2005 12:22


Oi
Soube pela minha filha que vc me telefonou. Não estou em casa.
Se quiser se comunicar por email escreva para :
cybernews@superig.com.br
neste novo email eu posso ler aqui em Santa Teresa.
{}'s
D.C.





SUBJECT: Relatividade Especial (tentando ser mais claro)
FROM: "dfahlb" <dfahlb@yahoo.com>
TO: ciencialist@yahoogrupos.com.br
DATE: 08/01/2005 16:50


--- Em ciencialist@yahoogrupos.com.br, "Prof. JC" <profjc2003@y...>
escreveu
> Oi dfahlb,

Olá Prof. JC, e Luiz Ferraz Netto.

Luiz Ferraz Netto, segue agora minha resposta à mensagem anterior do
Prof. JC o qual você pode usar para complementar/corrigir a que dei à
primeira vez.

Prof. JC, é realmente salutar o seu preciosismo. Haja vista que temos
que ser exatos na medida do possível, claro. Oxalá esse preciosismo
também significasse interesse em responder a pergunta do Luiz Ferraz
Netto em
http://br.groups.yahoo.com/group/ciencialist/message/43472 ,
haja vista você teria tanto a contribuir mas prefiriu comentar
comentários. Vamos lá:

É claro que concordo que quando um ouvinte se aproxima de uma fonte
sonora estacionária, a frequência do som que ele percebe é maior do
que quando ele está em repouso em relação à mesma fonte.

Se o ouvinte se afasta da fonte estacionária, ele escuta um som mais
grave do que se estvisse parado. Obtemos resultados semelhantes
quando a fonte se movimenta afastando-se de um ouvinte estacionário.
O timbre do apito de uma locomotiva é mais agudo quando ela está se
aproximando do ouvinte e mais grave após a fonte ter passado e estar
se afastando.

Acho que você também concorda. Isso, também é um "fato". Nem eu e nem
você parecemos discordar disso. O exemplo que você deu do jato,
embora certo, não tinha nada a ver com o que eu estava falando. Veja
o que você disse:

> Por exemplo, dentro de um jato a 300 m/s o som desloca-se a
> aproximadamente 340 m/s dentro do avião e, para um observador fixo
> na Terra com uma velocidade de 300+340 = 640 m/s.

Isto é óbvio e se eu quis parecer estar dizendo o oposto o erro foi
meu. Pois não quis. Na verdade, vou dar minha contribuição
preciosista agora: esta adição de velocidades que você uso segue as
transformações de Galileu que está errada. Entretanto, como o efeito
relativístico para esse módulo das velocidades é tão pequeno, ele
pode ser desprezado. Muito embora deve-se sempre usar as
Transformações de Lorentz pois elas quem dão a resposta e, dessa
resposta, é que sabemos se o efeito relativístico é ou não
desprezível.

Usando as Transformações de Lorentz, daria aproximadamente 640,22 m/s.
O resultado que você obteve usando as Transformações de Galileu estão
erradas mas apenas com erro de aproximadamente 0,034% do resultado
correto. É pouco, mas quanto maior as velocidades usadas em questão
este percentual aumenta.

Deixe-me ver se sou claro agora. Eu disse:

"Veja, se [a velocidade do som] depende do meio de propagação, vai
depender de como os observadores (receptores, no caso) estão se
movendo em relação a esse meio."

O que eu quis dizer com isso? Vamos por partes:

"a velocidade do som depende do meio de propagação". Aqui me referia
ao fato de que o som se propaga mais rápido em determinados meios que
outros (por exemplo, num sólido se propaga mais rápido que na água e
na água, é mais rápido que no ar) e nalguns meios sequer se propaga
(exemplo, no vácuo e em meios bastante rarefeitos, o som não se
propaga). Isso porque o som usa a vibração dos átomos e moléculas.
Quanto mais próximas, por exemplo, no sólido, mais rápido ele se
propaga. Uma rede cristalida (sólido) tem seus átomos e moléculas
mais fortemente ligados do que gases, por exemplo.

É claro que nos exemplos mais simples de serem pensados, faz-se
transformações de velocidades, etc, para fonte e receptores no mesmo
meio.

Agora, com respeito às transformações de velocidade que eu me referia
quando eu disse "Veja, se [a velocidade do som] depende do meio de
propagação, vai depender de como os observadores (receptores, no
caso) estão se movendo em relação a esse meio." referia-se ao
seguinte:

Se você está no referencial do meio (parado em relação ao meio) no
qual o som se propaga, você medirá uma velocidade. Agora, suponha que
o meio seja o ar e o som se propague por volta de 300m/s nesse meio.

Agora vamos pegar um outro observador que esteja se afastando da
fonte à esses exatos 300m/s. Ora, temos a velocida no som para este
referencial:

vel = 300 - 300 = 0 m/s. Nas Transformações de Lorentz também daria 0
m/s.

O que eu me referia era a isso. Um observador que esteja a mais de
300 m/s se afastando da fonte sonora não ouvirá som algum desta fonte
sonora. Isto é o "fato" a que me referia.

Sobre o Efeito Doppler para o som e a luz, era o seguinte, claro que
concordando com o que você vem dizendo, e mais claro agora:

"Ao deduzir o efeito Doppler para o som ve-se que a variação da
frequência, para uma dada velocidade 'v', depende de estar a fonte ou
o receptor em movimento com esta velocidade. Esta distinção é
possível para o som em virtude de haver um meio (o ar, por exemplo)
em relação ao qual o movimento ocorre. Por isso, o movimento da fonte
ou do receptor em relação ao ar (no caso) em repouso pode ser
distinguido do outro. Esta distinção entre o movimento da fonte ou do
receptor não pode ser feita para a luz ou para ouras ondas
eletromagnéticas no vácuo. As expressões que deduz-se para o Efeito
Doppler não podem ser corretas para a luz."

Extraído do livro: Física. Autor: Paul A. Tipler. Volume: 2b. Usado:
2o. ano de Bacharelado/Licenciatura em Física. Assunto:
Eletromagnetismo e tópicos especiais em Relatividade e Mecânica
Quântica. Concordo com esta explicação do Tipler e com ela
complemento: na verdade o Efeito Doppler está errado também para o
som! O que o faz parecer funcionar é que como o efeito relativístico
é desprezível, pensava-se ser ele correto e exato.

Se você gosta de pensar em termos de frequência e comprimento de onda
para o som, então lá vai: para um observador que esteja se afastando
da fonte sonora à um valor maior que o da velocidade do som no meio
medirá uma frequência zero pois não medirá som nenhum vindo da fonte
chegando até ele (acho que podemos dizer que "nenhum som = som de
frequência zero" num certo abuso de linguagem, mas acho que pode).

Espero ter sido claro agora para você e para o Economista da pergunta
ao Luiz Ferraz Netto.

Abraços.






SUBJECT: OT: Compra de livro de CTS no Brasil
FROM: Maria Natália <grasdic@hotmail.com>
TO: ciencialist@yahoogrupos.com.br
DATE: 08/01/2005 19:55


Caríssimos:
Alguém me sabe explicar qual o processo que há para efectuar a
compra de um livrobrasileiro sobre CTS? Trata-se de um livro da área
de Educação que trata do Projecto Salters (é área Ciência
Tecnologia e Sociedade). Obrigada
Maria Natália





SUBJECT: Re: Relatividade Especial (tentando ser mais claro)
FROM: "profjc2003" <profjc2003@yahoo.com.br>
TO: ciencialist@yahoogrupos.com.br
DATE: 08/01/2005 20:08


dfahlb

Ok, compadre, se lhe desagrada que eu comente seus comentários então
não os comentarei mais, incluindo a esse último que parece não ter
sido útil a nenhum de nós dois. Assunto encerrado.

Tenha um bom ano e muito trabalho, porque eu estou tendo ambos. :)

Abraços,
Prof. JC



--- Em ciencialist@yahoogrupos.com.br, "dfahlb" <dfahlb@y...>
escreveu
>
> --- Em ciencialist@yahoogrupos.com.br, "Prof. JC"
<profjc2003@y...>
> escreveu
> > Oi dfahlb,
>
> Olá Prof. JC, e Luiz Ferraz Netto.
>
> Luiz Ferraz Netto, segue agora minha resposta à mensagem anterior
do
> Prof. JC o qual você pode usar para complementar/corrigir a que
dei à
> primeira vez.
>
> Prof. JC, é realmente salutar o seu preciosismo. Haja vista que
temos
> que ser exatos na medida do possível, claro. Oxalá esse
preciosismo
> também significasse interesse em responder a pergunta do Luiz
Ferraz
> Netto em
> http://br.groups.yahoo.com/group/ciencialist/message/43472 ,
> haja vista você teria tanto a contribuir mas prefiriu comentar
> comentários. Vamos lá:
>
> É claro que concordo que quando um ouvinte se aproxima de uma
fonte
> sonora estacionária, a frequência do som que ele percebe é maior
do
> que quando ele está em repouso em relação à mesma fonte.
>
> Se o ouvinte se afasta da fonte estacionária, ele escuta um som
mais
> grave do que se estvisse parado. Obtemos resultados semelhantes
> quando a fonte se movimenta afastando-se de um ouvinte
estacionário.
> O timbre do apito de uma locomotiva é mais agudo quando ela está
se
> aproximando do ouvinte e mais grave após a fonte ter passado e
estar
> se afastando.
>
> Acho que você também concorda. Isso, também é um "fato". Nem eu e
nem
> você parecemos discordar disso. O exemplo que você deu do jato,
> embora certo, não tinha nada a ver com o que eu estava falando.
Veja
> o que você disse:
>
> > Por exemplo, dentro de um jato a 300 m/s o som desloca-se a
> > aproximadamente 340 m/s dentro do avião e, para um observador
fixo
> > na Terra com uma velocidade de 300+340 = 640 m/s.
>
> Isto é óbvio e se eu quis parecer estar dizendo o oposto o erro
foi
> meu. Pois não quis. Na verdade, vou dar minha contribuição
> preciosista agora: esta adição de velocidades que você uso segue
as
> transformações de Galileu que está errada. Entretanto, como o
efeito
> relativístico para esse módulo das velocidades é tão pequeno, ele
> pode ser desprezado. Muito embora deve-se sempre usar as
> Transformações de Lorentz pois elas quem dão a resposta e, dessa
> resposta, é que sabemos se o efeito relativístico é ou não
> desprezível.
>
> Usando as Transformações de Lorentz, daria aproximadamente 640,22
m/s.
> O resultado que você obteve usando as Transformações de Galileu
estão
> erradas mas apenas com erro de aproximadamente 0,034% do resultado
> correto. É pouco, mas quanto maior as velocidades usadas em
questão
> este percentual aumenta.
>
> Deixe-me ver se sou claro agora. Eu disse:
>
> "Veja, se [a velocidade do som] depende do meio de propagação, vai
> depender de como os observadores (receptores, no caso) estão se
> movendo em relação a esse meio."
>
> O que eu quis dizer com isso? Vamos por partes:
>
> "a velocidade do som depende do meio de propagação". Aqui me
referia
> ao fato de que o som se propaga mais rápido em determinados meios
que
> outros (por exemplo, num sólido se propaga mais rápido que na água
e
> na água, é mais rápido que no ar) e nalguns meios sequer se
propaga
> (exemplo, no vácuo e em meios bastante rarefeitos, o som não se
> propaga). Isso porque o som usa a vibração dos átomos e moléculas.
> Quanto mais próximas, por exemplo, no sólido, mais rápido ele se
> propaga. Uma rede cristalida (sólido) tem seus átomos e moléculas
> mais fortemente ligados do que gases, por exemplo.
>
> É claro que nos exemplos mais simples de serem pensados, faz-se
> transformações de velocidades, etc, para fonte e receptores no
mesmo
> meio.
>
> Agora, com respeito às transformações de velocidade que eu me
referia
> quando eu disse "Veja, se [a velocidade do som] depende do meio
de
> propagação, vai depender de como os observadores (receptores, no
> caso) estão se movendo em relação a esse meio." referia-se ao
> seguinte:
>
> Se você está no referencial do meio (parado em relação ao meio) no
> qual o som se propaga, você medirá uma velocidade. Agora, suponha
que
> o meio seja o ar e o som se propague por volta de 300m/s nesse
meio.
>
> Agora vamos pegar um outro observador que esteja se afastando da
> fonte à esses exatos 300m/s. Ora, temos a velocida no som para
este
> referencial:
>
> vel = 300 - 300 = 0 m/s. Nas Transformações de Lorentz também
daria 0
> m/s.
>
> O que eu me referia era a isso. Um observador que esteja a mais de
> 300 m/s se afastando da fonte sonora não ouvirá som algum desta
fonte
> sonora. Isto é o "fato" a que me referia.
>
> Sobre o Efeito Doppler para o som e a luz, era o seguinte, claro
que
> concordando com o que você vem dizendo, e mais claro agora:
>
> "Ao deduzir o efeito Doppler para o som ve-se que a variação da
> frequência, para uma dada velocidade 'v', depende de estar a fonte
ou
> o receptor em movimento com esta velocidade. Esta distinção é
> possível para o som em virtude de haver um meio (o ar, por
exemplo)
> em relação ao qual o movimento ocorre. Por isso, o movimento da
fonte
> ou do receptor em relação ao ar (no caso) em repouso pode ser
> distinguido do outro. Esta distinção entre o movimento da fonte ou
do
> receptor não pode ser feita para a luz ou para ouras ondas
> eletromagnéticas no vácuo. As expressões que deduz-se para o
Efeito
> Doppler não podem ser corretas para a luz."
>
> Extraído do livro: Física. Autor: Paul A. Tipler. Volume: 2b.
Usado:
> 2o. ano de Bacharelado/Licenciatura em Física. Assunto:
> Eletromagnetismo e tópicos especiais em Relatividade e Mecânica
> Quântica. Concordo com esta explicação do Tipler e com ela
> complemento: na verdade o Efeito Doppler está errado também para o
> som! O que o faz parecer funcionar é que como o efeito
relativístico
> é desprezível, pensava-se ser ele correto e exato.
>
> Se você gosta de pensar em termos de frequência e comprimento de
onda
> para o som, então lá vai: para um observador que esteja se
afastando
> da fonte sonora à um valor maior que o da velocidade do som no
meio
> medirá uma frequência zero pois não medirá som nenhum vindo da
fonte
> chegando até ele (acho que podemos dizer que "nenhum som = som de
> frequência zero" num certo abuso de linguagem, mas acho que pode).
>
> Espero ter sido claro agora para você e para o Economista da
pergunta
> ao Luiz Ferraz Netto.
>
> Abraços.





SUBJECT: vaga-lume
FROM: "Emiliano Chemello - Yahoo Grupos" <chemelloe@yahoo.com.br>
TO: <Conversa_de_Botequim@yahoogrupos.com.br>, <ciencialist@yahoogrupos.com.br>
DATE: 08/01/2005 21:57

eheheh,

[ ]'s

Emiliano

---
Contato Naeq:
Nome: Lúcia Meira
Telefone: Sobre a Bioluminescência e sobre este site
Mensagem: Sobre a Bioluminescência:
Estou escrevendo um livro de ficção no qual abordo a idéia de um planeta em
que a luz solar não está presente. Gostaria que me tirassem algumas dúvidas.
1 : Acredito que sem a presença da luz solar as espécies
quimiossíntetizantes e bioluminescentes, resultantes da evolução adaptativa,
seriam dominantes na Terra. Concordam? Isso seria possível?
Caso a resposta seja sim, isso afetaria de alguma forma a composição química
do ar e poderia o organismo humano sobreviver a isso?

Sobre o site:
Acho que este gif animado de vaga-lume muito... insuportável!!!
Por favor, pelo bem do conhecimento mundial, retirem-no.

Obrigada.
Aguardarei anciosamente pela resposta.



SUBJECT: RE: [ciencialist] vaga-lume
FROM: "murilo filo" <avalanchedrive@hotmail.com>
TO: ciencialist@yahoogrupos.com.br
DATE: 08/01/2005 22:38

... ai, que preguiça... o;

>From: "Emiliano Chemello - Yahoo Grupos" <chemelloe@yahoo.com.br>
>Reply-To: ciencialist@yahoogrupos.com.br
>To:
><Conversa_de_Botequim@yahoogrupos.com.br>,<ciencialist@yahoogrupos.com.br>
>Subject: [ciencialist] vaga-lume
>Date: Sat, 8 Jan 2005 21:57:47 -0200
>
>eheheh,
>
>[ ]'s
>
>Emiliano
>
>---
>Contato Naeq:
>Nome: Lúcia Meira
>Telefone: Sobre a Bioluminescência e sobre este site
>Mensagem: Sobre a Bioluminescência:
>Estou escrevendo um livro de ficção no qual abordo a idéia de um planeta em
>que a luz solar não está presente. Gostaria que me tirassem algumas
>dúvidas.
>1 : Acredito que sem a presença da luz solar as espécies
>quimiossíntetizantes e bioluminescentes, resultantes da evolução
>adaptativa,
>seriam dominantes na Terra. Concordam? Isso seria possível?
>Caso a resposta seja sim, isso afetaria de alguma forma a composição
>química
>do ar e poderia o organismo humano sobreviver a isso?
>
>Sobre o site:
>Acho que este gif animado de vaga-lume muito... insuportável!!!
>Por favor, pelo bem do conhecimento mundial, retirem-no.
>
>Obrigada.
>Aguardarei anciosamente pela resposta.
>




SUBJECT: mou conto- Fator quântico do conhecimento
FROM: "Amauri Jr" <amaurijunior2@yahoo.com.br>
TO: <elio_mollo_k@hotmail.com>, <Sexualidadedodef@yahoogrupos.com.br>, "Lucimary Vargas" <sangalli@uai.com.br>, Renata  <renatafariak@hotmail.com>, "Renata" <renifoci@hotmail.com>, <portal_brasileiro_da_filosofia@yahoogrupos.com.br>, "Marcia Mystery" <mare_mania@hotmail.com>, "Carolina Marques" <marquesacc@hotmail.com>, <Leitura_e_Literatura@yahoogrupos.com.br>, "Laracna" <laracna@pop.com.br>, "kathleen_mel" <kathleen_mel@uol.com.br>, <Euthymia@yahoogrupos.com.br>, <deficienteeficiente@yahoogrupos.com.br>, "Dengosa" <berta_oliver@hotmail.com>, Cris Passinato - Eu só sei que: EU TÔ FODIDA!!!! Ai, meu Deus, pq eu, ein???? <crispassinato@hotmail.com>, <Conversa_de_Botequim@yahoogrupos.com.br>, <ciencialist@yahoogrupos.com.br>, <acropolis@yahoogrupos.com.br>
DATE: 09/01/2005 00:16

Fator qu�ntico do conhecimento



Amauri Nolasco Sanches Junior









Eu sentado nesta poltrona olhando esse mundo repleto de multiplicidades e inoc�ncias qu�nticas, inoc�ncias que s�o part�culas, ou melhor, �tomos que se dividem em milh�es e podem esta em muito universos ao mesmo tempo. Olho o retrato de minha fiel amada, esse rosto lindo, pode esta em milh�es de lugares diferentes; ter o conhecimento, inato ou n�o, de conter o sonho de ter ou n�o ele em sua mente. O mundo e sua pluralidade, o sonho humano exaustivo de ter a felicidade, isto realmente n�o existe; somos nada vezes nada, pois se n�o temos anatomia de escolher nossos sonhos e desejos o que importa a vida?

Olho na janela e vejo a rua, repleta de gente que contem suas vidas, em suas mentes seus pensamentos e dores; um leque de sentimentos que nossa pluralidade nos presenteia e nossa mente nos tr�s. Aquele velhinho que cansado teve sua vida longas paix�es, distantes desejos, e o que virou? Um velho cansado que a pr�pria sorte lhe p�s diante de sonhos que nunca se realizaram, foram perdidos no v�cuo do tempo; tudo que conhece, tudo que viu, nada ser� passado porque o velho vai espelhar seus �tomos novamente no planeta. Aquilo que ele viu, tudo que viveu em seu intimo nada ficar�, tudo explodira um dia e a escurid�o planara tudo.

Tenho pena daquela crian�a, t�o inocente tr�s consigo um mundo s� seu, vai ficar adolescente e desejar algu�m; talvez se for bobo n�o olhara para ele, se for uma pessoa �descolada� n�o ser� dif�cil. S� que ele com toda certeza, ficara desiludido com o mundo repleto de guerras; gan�ncias m�ltiplas de um animal que apenas sobrevive, por um combust�vel f�ssil. Ele ver� o mundo como ele deseja ver tudo, se apaixonara e por ela chorara, se desesperara quando tiver sozinho, como eu. O mundo dos sonhos humanos tem milhares de portas e que se abrirmos a cada uma delas, haver� erros e acertos, mas tem que haver esses erros. Ele pelo seu rostinho, angelical, deve ser um bom menino; uma forma pat�tica de ser humanista, uma ovelha para um grande rebanho. Coitado! Batera nas portas de muitos cora��es e sentira medo de se fazer de inteligente, sentira em seu ego, uma profunda vontade de ser um ser fraco, pat�tico, mas um ser religiosamente bem feitor; aquele que qualquer um cumprimenta e quando receber esse cumprimento, respondera com um sorriso �idiota� e ser� o menino �padr�o�. Esses seres morrem logo, pois n�o expressam sua vontade, n�o tem nenhuma vontade alias, s�o hologramas de um sistema doente.

Logo atr�s uma garota bonita, atraente aos olhos masculinos que te olham com um desejo insaci�vel de cio, desejos bizarros; de animais que pegam sua f�mea para procriar violentamente, mas as leis humanas n�o permitem que uma mo�a t�o atraente atraia algum homem fora do fluxo padr�o; ora, essa mo�a com seus atributos avantajados e de bem com a vida, mas aposto, aposto mil reais que � frustrada. Pode ter lindos seios e uma bela bunda, mas em seu intimo s� tr�s para si, homem desejando sua carne e nunca seus esp�rito, ora inteligente, ora banal. � engra�ado, muito engra�ado, um ser que desprov�em de seu conhecimento, para ser uma pessoa agrad�vel, que debaixo daqueles seios e aquela bunda rebolativa h� sentimentos fortes e que espera seu pr�ncipe.

Olha l�, um religioso com sua b�blia debaixo do bra�o olhando a bunda da garota, desejando que aquela saia levante; aposto, que ta pensando na cor de sua calcinha, se ela � pequenina...que pensamento moribundos passam em minha cabe�a! Mas se sei, aquele homem onde grita aos quatro ventos que Jesus o salvou, desejando aquela mo�a e olhando-a para o seu deleite. Imagina que ela possa saciar seu desejo, ora de pervertido pecador, ora de homem dedicado e bondoso. Mas seu olhar n�o � de maneira nenhuma, uma maneira gentil de olhar uma mulher; mas nada a mais tenho a acrescentar nesse pensamento obscuro que eu tenho desse pobre homem, olhando uma bunda que nunca ira tocar.

Mas s�o banaliza��es de um desejo animal que � mais que natural, um desejo que o ser humano tem por milh�es de anos e que reproduz seus genes pelo seu planeta d�s de aquele momentos. Eu j� deixei a porta desse pensamento e indo a outro, aquele que o ser humano esta de muitas maneiras punindo por fantasias expostas; aquele pobre homem pode estar por ai em outra dimens�o, ora agarrando-a em seus bra�os, ora tomando um tapa dela. Estou a rir de minhas pr�prias id�ias er�ticas qu�nticas, estou a rir da falsa moral humana, que deixa louco os fracos e deixa os fortes sustentarem seus desejos. Ruim isto? N�o sei, n�o sou padre e nem pastor para lhe falarem dessas morais religiosas, moribundas de um tra�o de humanismo que me causa n�useas; aquele pensamento que todos s�o iguais perante o senhor, mas que senhor? Vejo aquele pobre homem querendo um pouquinho de horas com aquela mo�a e n�o pode, ele quer derramar os seus genes nela, perpetuar sua esp�cie.

Olho de novo a janela, entre os vidros vejo a velhinha a segurar seu ter�o e a outra m�o ocupada por uma velha b�blia; a mesma que o pessoal usa para suas rezas; aquele homem distra�do com a mo�a; sentado lendo sua b�blia, quando a velhinha se aproxima e sorri para ele; pergunta se esta louvando direito...Ai se ela soubesse o que ele olhava distraidamente em seus loucos desejos, talvez aquela b�blia em seu colo era para esconder...o que n�o desejo falar.

Mas olho mais adiante que havia dois homens discutindo sobre vis�es diferentes sobre um pensador, acho que ouvi Marx; aquele pobre homem que quis fazer o mundo um po�o de igualdade e que fizeram de sua teoria uma sadista pelega pol�tica; como sempre, usaram de seu pensamento como se ouve-se uma salva��o para o mundo. O mundo se salvou de Marx e sua filosofia, uma est�pida e desnecess�ria teoria que a grande massa grande gosta; a culpa de minha solid�o, por exemplo, seria daquele homem que pode conseguir aquela mo�a e ter o que deseja; ele tem que dividir ela comigo � sua obriga��o dividi-la, pois sou um fracassado por dentro e n�o consigo chegar nela. Simples? N�o t�o simples, Sartre e seus companheiros com aquele grande pensamento antigo �conhe�a a ti mesmo� elaborou uma teoria existencialista; as portas foram abertas a s�culos por S�crates, o homem que descobrir a si pr�prio, pode ser comparado a um tanque de guerra; pois se te conhecer saber� o que te aflige. Mas o ser humano � lavado ao pensamento da maioria, isso que temos como base o inconsciente coletivo como diria Jung.

Mas aqueles homens nem sabem de Marx e muito menos de Sartre, a exist�ncia repugnante humana com sua moral, com seus pensamentos s�o apenas desculpas para controlar a grande massa. � t�o insignificante tal pensamento que n�o me entranho neles e nem distraio meus mon�logos e minhas premissas. Meus pensamentos est�o em minha vida que h� horas agrad�veis e horas de extremos desafios, mas sei o que me toca por dentro de meu humilde �mago cansado de tantas situa��es que n�o pode controlar ou se se realizar. Somos insignificantes, somos filhos do acaso, como diz a deusa ci�ncia, estou aqui olhando esta janela e me deparando com a mais vasta solid�o causada pelo meus pr�prios erros e pela mais insignific�ncia incompreens�o. Mas n�s, homens das letras, somos na maioria das vezes somos interpretados como arrogantes senhores da verdade; nem sempre queremos passar isto, mas muitas vezes o preconceito e a ignor�ncia da maioria, nos faz assim. Talvez seja preconceito daquele homem que esta segurando a b�blia, talvez n�o, talvez seja uma �tica particular daquele episodio, pois eu vi com minha percep��o que todos que s�o assim seguram sua natureza.

Eu olho de novo para a janela e vejo aquele homem solit�rio que por acaso chegou naquela hora e fez o que qualquer homem faria, ele puxa conversa, abro um pouco a janela e ao longe eu escuto o que falavam. Falavam da pergunta daquele homem que havia naquela beleza toda alguma espiritualidade, alguma for�a para ela se salvar, a resposta foi n�o; ela respeita toda forma de espiritualidade, mas n�o segue nada. Que pergunta para se chegar a uma boa bunda? Sim, talvez aquele homem esteja querendo salvar a pr�pria alma de seus pensamentos pecaminosos, santos somente para pensamentos perversos como o meu que v� na moral um iceberg para afundar o grande Titanic social humana. Caos? Talvez, se todos irem ao encontro da mesma bunda, mas eu um homem cheio de maldade nesse cora��o como eu; n�o mais posso pensar em ter uma beleza dessas, em meu cora��o em trevas. Mas caramba! At� me falaram que h� for�as obscuras que regem o universo tamb�m que n�o s� os neur�nios regem o nosso c�rebro, talvez se eu ir l� e passar na frente daquela deusa ela olhe para mim.

Imagina se eu vou estragar a conversinha med�ocre daquele coitado, aquele que teve por alguns minutos um momento nano-gloriosos de uma pequena vida. Pense que o mundo � feito de part�culas, essas mesmas part�culas que nos fazer viver; somos filhos do universo e de suas estrelas; aquele homem procurando desesperadamente aquela companhia, � um filho de uma estrela que explodiu a milh�es de anos atr�s; e formou part�culas acidas que vieram nos meteoros desgovernados e cometas doid�es. Engra�ado, somos part�culas subat�micas de uma estrela que morreu a muito tempo e eu aqui olhando esses dois conversarem, como se eles fossem importantes, como minha vida mudasse algo com essa situa��o. Sei que estou mais olhando o decote da linda mo�a, mas aqueles seios nunca ir�o ser tocados por mim, ent�o por que olhar? Qual o intuito original que fez n�s sermos seres vivos vivendo nesse planeta?

Aquele homem, por exemplo, ele acredita que quando morrer ser� levado para o para�so, levado pelas suas boas a��es; inclusive, tentando converter aquela pobre alma, (do inferno?), sei que por tr�s daquela suposta convers�o h� um intuito maior. Mas ent�o, o sentido da vida dele � fazer com que o mundo n�o tenha pecado, que ap�s sua morte, ser� julgado suas boas e m�s a��es; para ele n�o interessa e nem deve saber dessas part�culas, mol�culas, �tomos e etc...porque h� nele uma vontade de se inferiorizar profundamente sua alma; ter aquele ter o esp�rito de rebanho, ter pena de si pr�prio. Humanizar um ser ca�tico que s� quer saber de guerras e posi��es? Pat�tico isto, mas segundo meu conceito, h� uma grande necessidade para isto tudo.

O ser humano pensar que � necess�rio para este planeta � um pensamento antiplaneta Terra, pois um planeta com um �bando� de seres que destroem por mero prazer da destrui��o, da predat�ria alienada. Mas tudo isto � de sua natureza moribunda, aquele homem ficar� naquela posi��o e falando e querendo aquilo que sempre desejara; uma boa b�blia e uma boa bunda a olhar; isso mesmo, n�o sejam hip�critas em dizer que esses homens n�o olham essas mo�as e suas boas bundas.

Mas qual n�o olharia? Afinal � de nossa natureza ancestral, mas nos aprofundamos ao homo sapiens e vamos ver que, sua natureza nada tem de proveito, nem mesmo suas fal�cias sobre �paz� e causas humanit�rias que s�o importantes para a sociedade. Mas qual sociedade quer ajuda de m�seros pensamentos moribundos que a s�culos vem causando o c�ncer globalizado? Vamos ver ao longo desses s�culos humanos, onde sangue foi derramado por muitos lideres e nada mudou; por ainda o ser humanas estas entrela�adas com seus pensamentos materiais, nada acrescentam no mundo al�m desse, de seu pensamento e de querer sempre sua Eterna Gloria. O Universo � seu, pensam eles, mas e da�? Ser� que tudo � do homem e do homem voltara?

Puxa! Sempre ter� esses homens e suas b�blias por motivos que fogem de nossa compreens�o, tentando converter essas mo�as e transforma-la em mais uma em seu rebanho, para ele o homem foi criado do p�. Veja s� voc�s, o humano demasiado humano, � apenas resto de estrelas mortas a milh�es de anos que resultou da Terra e seu �cidos que est�o eu suas c�lulas, pois ele pensa ser p� da terra que de alguma maneira n�o �, � o p� c�smico das estrelas. Mas o que seria de nossas vidas sem essas situa��es? O mundo seria insuport�vel!

Sempre vamos ter pessoas malucas com suas cren�as e seus momentos loucos, seus momentos de insanidade, de crer em uma coisa que nunca v�o se saber a verdade. Talvez a for�a existencial fosse as mazelas da vida; pequenez insuport�vel, mas � engra�ado o que se consegue ver na rua, onde as coisas n�o s�o como pensamos ser. Os sonhos, s�o pequenos, s�o um mundo completo que n�s mesmo fazemos; completamos muitos pensamentos que s� mostramos a n�s por termos vergonha ou receio de mostrarmos nossa verdadeira natureza. Criamos mascaras e condi��es para se escondemos de si e da sociedade, ent�o criamos religi�es ou crermos em ideologias, ficamos atr�s de muros ideol�gicos. No fundo, se pensarmos somos iguais ao homem do banco a conversar com aquela mo�a; o que nos importa � conversar com uma bela mo�a e nos iludir com cren�as, assim nossa vida passa sem dor da verdade. Precisamos de imagens pomposas, ideologias bonitas, her�icas, para sustentarmos diante do medo; medo de sermos nada, que na verdade em nosso intimo, somos s�mios querendo um lugar no universo. Nada vezes nada, somos p� de estrelas e vamos virar de novo p� de estrelas.

Somos insignificantes diante de tantas coisas que n�o sabemos porque acreditamos, a lei ser nada, para sabemos nada e ningu�m saber� nada. Na verdade, aquele homem nada sabe, s� esta repetindo o que o pastor disse; aquele � mais um pobre coitado � um filho da ilus�o. N�s somos filhos de muitas ilus�es que n�o suportamos que n�o sejam verdades, a verdade nem sempre � o que queremos.

Mas vejo eles irem cada um no seu lado, cada um vivera sua vida e seus moment�neos sentimentos, sentimentos que est�o ali de cada um em seu momento explodira. Mas estou no escuro novamente, a rua se esvaziou e nada de interessante vejo por l�; fico lendo bulas, qual o fator qu�ntico de uma bula de rem�dio? Bom, ou�o passos subir as escadas e ao abrir a porta da casa e ouvi um �oi� e era minha amada chegando exausta....



Fim


[As partes desta mensagem que n�o continham texto foram removidas]



SUBJECT: Pedra esquisita
FROM: "Kentaro Mori" <kentaro.mori@itelefonica.com.br>
TO: ciencialist@yahoogrupos.com.br
DATE: 09/01/2005 00:19


Vejam:

http://marsrovers.jpl.nasa.gov/gallery/press/opportunity/20040524a.html

Clicando em qualquer uma das imagens, notem à esquerda, próximo da
base do rover, uma pedra de formato alongado. Parece um tronco de
madeira, mas não só isso, um tronco de madeira em formato retangular.
Arca de Noé marciana?

Sem brincadeira, é uma pedra de formato curioso. Algum geólogo na
lista disposto a comentar? Veja que o rover parece ter efetivamente
passado sobre a pedra (perceba os rastros), o que também permite notar
que a pedra é relativamente pequena, menos de um metro de comprimento.
Bonsai? :)

Enfim, agradeceria comentários sobre possíveis processos de formação
da rocha, e especialmente, se alguém conhecer outra imagem marciana
com outra pedra de aparência similar.

Abraços,

Mori





SUBJECT: Re: vaga-lume
FROM: "rmtakata" <rmtakata@altavista.net>
TO: ciencialist@yahoogrupos.com.br
DATE: 09/01/2005 00:46


--- Em ciencialist@yahoogrupos.com.br, "Emiliano Chemello -
> ---
> Contato Naeq:
> Nome: Lúcia Meira
> Estou escrevendo um livro de ficção no qual abordo a idéia
> de um planeta em que a luz solar não está presente.

Terah q. arranjar uma boa desculpa para q. o planeta nao fique
permanentemente congelado. Sem uma fonte de energia, o planeta
rapidamente entraria em equilibrio termico com o universo - alguns
centesimos acima do zero absoluto. Nesse caso as reacoes quimicas
seriam incrivelmente vagarosas.

Sim, a quimiossintese seria uma alternativa. Nao, eh pouco provavel q.
em um ambiente permanentemente no escuro viessem a se desenvolver
seres bioluminescentes. A bioluminescencia pode ser util apenas para
organismos q. enxergam (ou, em alguns casos, q. possuem predadores q.
por sua vez possuem seus proprios predadores q. enxergam; ou q possuem
presas q enxergam e sao atraidas por luz). Em outras palavras, seria
preciso q houvesse primeiro seres com olhos (sensores opticos). Em um
ambiente de total escuridao, eh pouco provavel q. surgissem seres com
olhos.

Na Terra existe bioluminescencia em ambientes escuros porq. os
organismos sao derivados de organismos q. enxergavam jah anteriormente
-- viviam em ambiente iluminados (ainda q. fracamente).

Para esse mundo, soh se o Sol tivesse se apagado um tanto
recentemente. (Mas nao necessariamente seres bioluminescentes seriam
predominantes -- embora nao haja um impeditivo serio para q. isso
viesse a ocorrer.)

O oxigenio atmosferico na Terra eh todo devido 'a atividade
fotossintetica. A luz eh utilizada para catalisar reacoes de fotolise
de moleculas de agua, o hidrogenio eh utilizado para reduzir compostos
de carbono e o oxigenio eh usado como o aceptor final de eletrons. A
quimiossintese (pelo menos as reacoes quimiossinteticas q. ocorrem por
aqui) utiliza outras especies quimicas como aceptores de eletrons
(como o enxofre do acido sulfridico), nao liberando oxigenio. Nesse
caso, a composicao atmosferica da Terra seria muito diferente sem a
fotossintese -- haveria muito mais gas carbonico e quase nenhum
oxigenio (q se combinaria rapidamente com metais e outros compostos).

[]s,

Roberto Takata





SUBJECT: Re: Pedra esquisita
FROM: "rmtakata" <rmtakata@altavista.net>
TO: ciencialist@yahoogrupos.com.br
DATE: 09/01/2005 01:28


Bem, nao sou geologo (havia uma geologa por aqui, mas ela anda sumida,
na lista da SBCR tem o Prof. Arlei q. eh do Instituto de Geociencias
da USP).

Pela foto nao dah pra se ter uma ideia muito boa se eh mesmo uma pedra
longa. Pode ser um ressalto da superficie em perspectiva baixa.

Da minha opiniao leiga parece ser uma regiao de derrame basaltico, com
formacao de basalto colunar (o q. explicaria o ladrilhamento da
superficie).

http://www.bsu.edu/geology/photodis/neu3.htm
http://granolagoth.mearcair.net/albums/album12/53_columnar_basalt.jpg
http://www.kidscosmos.org/kid-stuff/mars-trip-basalt.html

Se nao for isso pode ser o monolito negro. Re re.

[]s,

Roberto Takata

--- Em ciencialist@yahoogrupos.com.br, "Kentaro Mori"
http://marsrovers.jpl.nasa.gov/gallery/press/opportunity/20040524a.html
> Sem brincadeira, é uma pedra de formato curioso. Algum






SUBJECT: neutrino, existir or not be?
FROM: "rayfisica" <rayfisica@yahoo.com.br>
TO: ciencialist@yahoogrupos.com.br
DATE: 09/01/2005 03:48


Gostaria apenas de saber a quantum anda a certeza na existência do
neutrino (pelo ao menos como foi inicialmente e a que propósito
proposto), quem na lista hoje daria uma caixa de fósforo a um
amigo
dizendo:- garantia de conter 300 neutrinos.





SUBJECT: Neurofisiologia
FROM: "neurophys" <jbrasil@unb.br>
TO: ciencialist@yahoogrupos.com.br
DATE: 09/01/2005 08:49


Olá,amigos!

Confiram as seguintes novidades, no Site da Neurofisiologia e
Fisiologia Endócrina:

1.Neuro-News: Antibiótico parece ser terapêutico na Esclerose Lateral
Amiotrófica!

2.Presente de Férias: peça grátis o seu E-Book "Sombras de Mão para
Projetar na Parede"- divirta-se com esta antiga arte que requer
apenas as suas mãos, luz e a sombra na parede!

3.Leonardo ou Mona? Será a Mona Lisa um auto-retrato de Leonardo da
Vinci, como afirma Dan Brown no livro "O Código da Vinci?" Veja a
suposta prova dessa teoria numa animação computadorizada...

O endereço do site é http://neurofisiologia.cjb.net

Boas Férias!

Joaquim





SUBJECT: Re: [ciencialist] É o conhecimento perigoso?
FROM: "Jamil Orlandelli" <orlandel@ig.com.br>
TO: <ciencialist@yahoogrupos.com.br>
DATE: 09/01/2005 09:40

Se gosta do assunto tem um capítulo inteiro do Livro "A Cultura Científica e
os Seus Inimigos" (Gerald Holton) da Gradiva que pode ser lido de graça :)
[Jamil]

http://www.gradiva.pt/livro.asp?L=2100


Holton adverte-nos contra a crescente «rebelião romântica» dos nossos dias,
em que se acusa a ciência de todos os males sociais e se substitui a razão
pelas «vias de conhecimento» da new age. Partindo da análise do pensamento
de Einstein, mostra-nos que a ciência é de facto a expressão criativa da
tradição cultural ocidental e que os maiores avanços científicos resultaram
sempre de grandes saltos intuitivos da imaginação. Abrangente e vigoroso, um
livro obrigatório para todos os que pretendam entender o lugar que a ciência
ocupa no nosso mundo.
----- Original Message -----
From: "Amauri Jr" Sent: Friday, January 07, 2005 10:25 PM
Subject: [ciencialist] É o conhecimento perigoso?

É o conhecimento perigoso?
Fronteiras entre ciência, tecnologia e ética
Orlando Tambosi






SUBJECT: Re: [ciencialist] neutrino, existir or not be?
FROM: "JVictor" <jvoneto@uol.com.br>
TO: <ciencialist@yahoogrupos.com.br>
DATE: 09/01/2005 10:22

Caro Rayfisica,


Minha confiança nas leis da natureza e, sobretudo, nas de conservação, me dão a convicção inabalável de que o neutrino existe mesmo. Sua existência foi comprovada desde a década de 50. Não há dúvidas a respeito.
E como surgiu essa idéia? Antes, algumas informações básicas.
As simetrias da natureza, que nos dizem que coisas no universo podem acontecer ou que não mudam nunca, bem como quais aquelas que não podem acontecer, de maneira expontânea, constituem um ferramental de alto interêsse na física, pois especificam claramente que coisas são invariantes e que coisas não o são. A sua caneta jamais vai sair expontâneamente de sua mesa de trabalho e pular para seu bolso, sem uma ajudinha sua!
Uma matemática genial, lá dos idos de Einstein, E. Noether, percebendo essas sutilezas curiosas no comportamento da natureza, fez sua descrição matemática em forma de um teorema, que norteia os caminhos das pesquisas teóricas em física. Em linhas gerais, é o seguinte: se um sistema físico qualquer sofrer alguma transformação dinâmica e as EQUAÇÕES que descrevem esse sistema físico não mudarem por conta dessa transformação, então isto implica, necessariamente, que, para cada uma dessas transformações, haverá uma quantidade física que permanece como estava, não muda, se conserva. Isto simplesmente significa que essa quantidade não muda com o tempo! Esta descoberta magistralmente genial e simples me permite concluir, sem qualquer resquício de dúvidas, que as coisas que transformam e criam energia do nada simplesmente não podem acontecer. É o caso de motos perpétuos, cujas enganosas possibilidades têm ativado os neurônios de pessoas até brilhantes, mas equivocadas conceitualmente a esse respeito. O Teorema de Noether me diz quais os motivos em vista dos quais certas coisas são impossíveis. Esse Teorema dá conta de todas as leis de conservação da natureza, tais como: conservação da energia, do momentum linear, angular, etc. A concordância com a experiência, essa juíza fria e imparcial, é indiscutível. Simples assim.
Essas leis de conservação são uma decorrência das simetrias que a natureza nos oferta, para melhor descrevê-la, e não o contrário! Tomemos a lei da conservação do momento linear, como exemplo. É fácil verificar que se dois sistemas sólidos, num sistema fechado, antes de uma interação, como uma colisão, tiverem uma quantidade de momentum linear p1 + p2, então, após a interação, essa quantidade de momentum será p3+p4, ou seja, as quantidades individuais mudam, mais o total, a soma, fica como estava. (Os p´s são quantidades vetoriais).
Uma granada, antes de explodir, tem momento linear, digamos, zero. Quando explode, sai estilhaços para todo lado. Para cada pedaço numa direção A, sai outro numa direção -A, tal que a soma final, de todos os momentos seja zero.
Agora, um experimento e uma conclusão, que certamente responderá à sua questão. O nêutron é uma partícula atômica instável. Ela não fica "hibernando" por mais do que 10 mínutos(uma eternidade, no nível atômico). Ela decai, ou seja, se transosfrma, em um proton e um elétron, que, por possuirem carga ELÉTRICA COULOMBIANA, podem ter seus rastros detectados por certos equipamentos de detecção específicos. Nos primeiros estudos, esperava-se que o proton, de massa quase 1900 vezes maior que a do elétron, saisse para um lado com uma certa velocidade, e o elétron saisse para o outro, na mesma direção, com uma velocidade bem maior, tal a que a soma dos mv(v=velocidade, vetorial e m= massa, escalar) antes e depois fosse a mesma. Isto não aconteceu. Verificou-se que o próton sai numa direção e o eletron sai noutra, formando um ângulo menor do que 180 graus com a direção do proton. Nessas circunstâncias, a soma após a transformação é diferente de zero. Faltava a indicação de um rastro que denunciasse a existência de outra quantidade vetorial que, compondo-se com as observadas, desse o resultado esperado!. A conservação do momentum entrou pela perna do pinto e saiu pela do pato? Era o que parecia. Mas, dentre o fisicos de então, havia um mais ousado, chamado Wolfgang Pauli, conhecido pelo seu rigor, sua inteligência e sagacidade mental, respeitadíssimo por todos, inclusive por pessoas como Einstein, Planck, Heisenberg e outros. Uma opinião sua sobre qualquer trabalho de qualquer um deles, era um aval aceito por todos! Também era sarcástico, quanto a teorias marginais e assemelhados. Conta-se que, certa vez, um físico(ou uma editora, não lembro agora) lhe pediu para avaliar um artigo que escrevera, antes que pudesse publicá-lo. Após a leitura e análise, sua resposta para o interessado foi curta, seca e grossa: isto SEQUER é um artigo. Ponto. Era assim, o gênio Pauli.
Assim, no caso acima, do decaimento do neutron, físicos comuns poderiam simplesmente escantear a leis da conservação do momentum e da energia, pois ambas estão envolvidas no processo. Quem poderia sair com uma " loucura" dessas: propor a existência de uma certa partícula, produzida no processo de decaimento do neutron, que, compondo-se com o elétron e o prótron, garantisse as leis da conservação? Tal partícula deveria, ainda: ser indetectável, elétricamente neutra, e muiiiiiiiiiiiiiiito leve, mais ainda do que o elétron.
Propor uma " heresia dessas" não era algo fácil, ou coisa para qualquer um. Pauli, por outro lado, não era qualquer um, via alguns centímetros a mais diante do nariz e não era de jeito nenhum um sujeito fácil. Daí ter tascado essa idéia, que foi salvadora. Do contrário, hoje a física não seria física, mas um caos. A física só existe por causa das simetrias e de suas consequências, as leis da conservação, nas quais Pauli tinha extrema confiança! Salvos pelo gongo, digo, por Pauli.*

Pois é, respondendo a sua questão, a tal partícula salvadora, chamada de neutrino por outro grande físico, Fermi, surgiu assim, pela absoluta confiança nas leis da natureza, leis de conservação, resultado das simetrias existentes. Muitas das teorias de hoje, sobretudo as que dão conta do comportamento de estrelas e outros fogaréus, são devidas à produção de neutrinos nos processos. Há inclusive um ilustre físico brasileiro que elaborou importante teoria, com outro físico russo, envolvendo o neutrino.(Não seria interessante alguém da lista expor o trabalho desse físico, Mário Schenberg, a esse respeito?).
Espero ter sido claro.

Sds,


Victor.


* Um dos melhores livros, talvez o melhor, pela clareza, pela precisão, pela fidelidade ao espírito da teoria, é o seu Theory of Relativity. Só acreditei nisso depois que comprei o livro e comecei a lê-lo. Infelizmente está na língua alienígena, como diz o Prof. Léo. Mas, fazer o quê? Já ví muitas obras de elevadíssimo nível, como o Tolman, Rindler, Synge, C.Moller, que até têm contribuições pessoais ao desenvolvimento. Em minha opinião, o W.Pauli é o melhor. Envolve tanto TRR como a TRG. Quando ele o escreveu, tinha meros 21 anos!. E foi um dos poucos a haverem compreendido, com todas as incidências e profundidade requeridas, a teoria de Einstein. Quem quiser conferir, garanto que será um investimento de grande retorno intelectual. Se não ocorrer assim, podem me denunciar ao PROCOM, por propaganda enganosa!!!!!

Sds, de novo

Victor.


----- Original Message -----
From: rayfisica
To: ciencialist@yahoogrupos.com.br
Sent: Sunday, January 09, 2005 3:48 AM
Subject: [ciencialist] neutrino, existir or not be?



Gostaria apenas de saber a quantum anda a certeza na existência do
neutrino (pelo ao menos como foi inicialmente e a que propósito
proposto), quem na lista hoje daria uma caixa de fósforo a um
amigo
dizendo:- garantia de conter 300 neutrinos.





##### ##### #####

Para saber mais visite
http://www.ciencialist.hpg.ig.com.br


##### ##### ##### #####


Yahoo! Grupos, um serviço oferecido por:







------------------------------------------------------------------------------
Links do Yahoo! Grupos

a.. Para visitar o site do seu grupo na web, acesse:
http://br.groups.yahoo.com/group/ciencialist/

b.. Para sair deste grupo, envie um e-mail para:
ciencialist-unsubscribe@yahoogrupos.com.br

c.. O uso que você faz do Yahoo! Grupos está sujeito aos Termos do Serviço do Yahoo!.



[As partes desta mensagem que não continham texto foram removidas]



SUBJECT: Fw: [sabedoria_antiga] O que é ciência?
FROM: "Amauri Jr" <amaurijunior2@yahoo.com.br>
TO: "Lucimary Vargas" <sangalli@uai.com.br>, <Sexualidadedodef@yahoogrupos.com.br>, "Laracna" <laracna@pop.com.br>, "kathleen_mel" <kathleen_mel@uol.com.br>, <Conversa_de_Botequim@yahoogrupos.com.br>, <ciencialist@yahoogrupos.com.br>, <acropolis@yahoogrupos.com.br>
DATE: 09/01/2005 13:03


----- Original Message -----
From: A ERA DO ESPÍRITO
To: sabedoria_antiga@grupos.com.br
Sent: Saturday, January 08, 2005 10:32 PM
Subject: [sabedoria_antiga] O que é ciência?


O que é ciência?



silvio seno chibeni

Departamento de Filosofia - IFCH - Unicamp

chibeni@unicamp.br - http://www.unicamp.br/~chibeni







Resumo:

Este trabalho apresenta para um público geral algumas das principais concepções de ciência defendidas por filósofos da ciência desde o surgimento da ciência moderna, no século XVII. Procura-se destacar que essas concepções evoluíram na direção de uma melhor adequação ao que de fato se verificou na história da ciência.



Índice:

1. A visão comum de ciência............................................................................. 1

2. Objeções à visão comum da ciência............................................................... 3

3. Popper e o falseacionismo............................................................................. 6

4. Limitações do falseacionismo......................................................................... 8

5. Lakatos: uma visão contemporânea da ciência............................................... 13

Referências..................................................................................................... 16



1. A visão comum de ciência

Constitui crença generalizada que o conhecimento fornecido pela ciência distingue-se por um grau de certeza alto, desfrutando assim de uma posição privilegiada com relação aos demais tipos de conhecimento (o do homem comum, por exemplo). Teorias, métodos, técnicas, produtos, contam com aprovação geral quando considerados científicos. A autoridade da ciência é evocada amplamente. Indústrias, por exemplo, freqüentemente rotulam de "científicos" processos por meio dos quais fabricam seus produtos, bem como os testes aos quais os submetem. Atividades várias de pesquisa nascentes se auto-qualificam "científicas", buscando afirmar-se: ciências sociais, ciência política, ciência agrária, etc.



Essa atitude de veneração frente à ciência deve-se, em grande parte, ao extraordinário sucesso prático alcançado pela física, pela química e pela biologia, principalmente. Assume-se, implícita ou explicitamente, que por detrás desse sucesso existe um "método" especial, uma "receita" que, quando seguida, redunda em conhecimento certo, seguro.



A questão do "método científico" tem constituído uma das principais preocupações dos filósofos, desde que a ciência ingressou em uma nova era (ou nasceu, como preferem alguns), no século 17. Formou-se em torno dela e de outras questões correlacionadas um ramo especial da filosofia, a filosofia da ciência. Investigações pioneiras sobre o "método científico" foram conduzidas por Francis Bacon (1561-1626). Secundadas no século 17 por declarações de eminentes cientistas, como Galileo, Newton, Boyle, e, no século seguinte, pelos Enciclopedistas, suas teses passaram a gozar de ampla aceitação até nossos dias, não tanto entre os filósofos, mas principalmente entre os cientistas, que até hoje muitas vezes afirmam seguir o método baconiano em suas pesquisas. Isso é singular, visto que os estudos recentes em história da ciência vêm revelando que os métodos efetivamente empregados pelos grandes construtores tanto da ciência clássica quanto da moderna têm pouca conexão com as prescrições do filósofo inglês.



De forma simplificada, podemos identificar nas múltiplas variantes dessa visão da atividade científica e da natureza da ciência ¾ a que chamaremos visão comum da ciência ¾ algumas pressuposições centrais:



a) A ciência começa por observações. Bacon propôs que a etapa inicial da investigação científica deveria consistir na elaboração, com base na experiência, de extensos catálogos de observações neutras dos mais variados fenômenos, aos quais chamou "tábuas de coordenações de exemplos" (Novum Organum, II, 10). Como exemplo, elaborou ele mesmo uma lista de instâncias de corpos quentes, visando iniciar o estudo científico do calor. Essa tábua é então complementada por duas outras, igualmente de longa extensão, reunindo "instâncias negativas" (corpos privados de calor) e casos de corpos que possuem uma "disposição" para o calor.



b) As observações são neutras. As referidas observações podem e devem ser feitas sem qualquer antecipação especulativa, sem qualquer diretriz teórica. A mente do cientista deve estar limpa de todas as idéias que adquiriu dos seus educadores, dos teólogos, dos filósofos, dos cientistas; ele não deve ter nada em vista, a não ser a observação pura.



c) Indução. As leis científicas são extraídas do conjunto das observações por um processo supostamente seguro e objetivo, chamado indução, que consiste na obtenção de proposições gerais (como as leis científicas) a partir de proposições particulares (como os relatos observacionais). Servindo-nos de uma ilustração simples, a lei segundo a qual todo papel é combustível seria, segundo a visão que estamos apresentando, obtida de modo seguro de um certo número de observações de pedaços de papel que se queimam. A lei representa, pois, uma generalização da experiência. O processo inverso, de extração de proposições particulares de uma lei geral, assumida como verdadeira, cai no domínio da lógica, sendo um caso de dedução.



Durante a primeira metade do século XX, uma plêiade de eminentes filósofos empreendeu aperfeiçoar aquilo que vimos denominando de concepção comum de ciência, em um sofisticado programa filosófico, conhecido como positivismo lógico. Esse movimento, cujo núcleo original formou-se em torno do chamado Círculo de Viena, na década de 1920, exerceu uma influência marcante sobre a comunidade científica, que perdura até nossos dias, não obstante críticas severas ao positivismo lógico haverem surgido ainda na década de 1930.



2. Objeções à visão comum da ciência

Iniciemos nossa simplificada exposição das objeções à visão comum da ciência examinando brevemente a questão da justificação da indução. Dentro do âmbito restrito de nossa discussão, o processo dedutivo não apresenta maiores dificuldades; podemos assumir que se a verdade de uma proposição estiver assegurada, também o estará a de todas as proposições que dela decorrerem dedutivamente, pelo uso das leis da lógica. Tais leis, no entanto, não asseguram a validade do processo indutivo. Voltando ao nosso exemplo, nenhum conjunto de observações de incineração de pedaços de papel, por maior e mais variado que seja, é suficiente para justificar logicamente a lei segundo a qual todo papel é combustível. Não há contradição formal, lógica, em se afirmar que embora todos os pedaços de papel já examinados tenham se queimado, esta folha não é combustível. Isso pode contrariar o senso-comum, as leis da química e da física, mas não as da lógica.



Eliminada a possibilidade de justificação lógica, resta, segundo os pressupostos empiristas dos próprios defensores dessa concepção, unicamente a justificação empírica. No entanto, os filósofos John Locke e David Hume apontaram, no final do século 17 e início do 18, que a justificação empírica da indução envolve dificuldades insuperáveis.



Essa constatação veio a exercer uma enorme influência na filosofia, estimulando, por um lado, a retomada de doutrinas racionalistas (Kant) e, por outro, a reformulação dos objetivos empiristas, com o reconhecimento de que o ideal original de certeza e infalibilidade do conhecimento geral do mundo exterior não pode ser atingido. Procurou-se, assim, determinar condições nas quais o salto indutivo seja feito da maneira mais segura possível. Entre as condições que têm sido propostas destacaríamos:



d) o número de observações de um dado fenômeno deve ser grande;

e) deve-se variar amplamente as condições em que o fenômeno se produz; e

f) não deve existir nenhuma contra-evidência, i.e., observação que contrarie a lei.



Embora pareçam prima facie razoáveis, um pouco de reflexão e inspeção cuidadosa da história da ciência revelam que tais condições não são nem suficientes para garantir as inferências indutivas, nem necessárias ao estabelecimento de nossas melhores teorias científicas.



Que não são suficientes para assegurar a validade do processo indutivo já está claro de nossas considerações anteriores. Dada uma proposição geral qualquer, não importa quão numerosas e variadas tenham sido as observações que lhe forneceram suporte indutivo, é sempre possível que a próxima observação venha a contrariar as anteriores, falseando a proposição geral. Se apelarmos para o princípio da regularidade da natureza, estaremos na obrigação de justificá-lo. Mas tal princípio evidentemente não é de natureza lógica; e se lhe quisermos dar justificação empírica, caímos de novo no problema da indução.



Além disso, podemos ver que as condições enumeradas também não são necessárias para as mais importantes teorias científicas. Primeiro, quando à condição (d), atentemos para o fato de que alguns dos mais fundamentais experimentos científicos não foram repetidos senão umas poucas vezes, ou mesmo, como é comum, foram realizados apenas uma vez. Muitas das generalizações empíricas nas quais mais certeza depositamos resultaram de uma única observação. Quem, por exemplo, duvidaria que a explosão de bombas atômicas causa a morte de seres humanos após Hiroshima haver sido arrasada?



Quanto à condição (e), notemos que a variação das condições de observação também não tem ocorrido ao longo do desenvolvimento da ciência. Essa exigência é inexeqüível, se interpretada rigorosamente, já que os fatores que em princípio podem influir são em número indefinido. Por exemplo, para verificarmos a lei da queda dos corpos, teríamos que variar não somente a forma e a massa do corpo que cai, e o meio no qual se move, mas também a sua temperatura, a sua cor, a hora do dia na qual o experimento é feito, a estação do ano, o sexo do experimentador, o seu cheiro, etc. Isso faz ver que há sempre pressuposições teóricas guiando a escolha das condições que devem ser controladas ou variadas; são nossos pressupostos teóricos que nos causam riso diante de algumas das condições que acabamos de enumerar. Este ponto será retomado adiante, dada a sua importância.



Finalmente, nem mesmo a condição (f) tem sido respeitada pela ciência. As teorias científicas nascem e se desenvolvem em meio a inúmeras "anomalias" ou contra-exemplos empíricos. A teoria de Copérnico conviveu, até o advento do telescópio, com o contra-exemplo da observação da invariância das dimensões de Vênus ao longo do ano. A mecânica newtoniana atingiu a glória mesmo tendo que aguardar décadas antes que pudesse entrar em acordo com as observações da trajetória da Lua; e nem foi abandonada no século 19 quando não pôde dar conta da órbita de Urano. A hipótese de Prout sobre os pesos atômicos dos elementos químicos esperou quase um século antes que seu conflito com abundantes experiências fosse removido.



Passemos agora às objeções ao princípio (a) da visão comum da ciência: começo da investigação científica por observações.



O comentário que fizemos sobre a variação das condições de observação já indica uma dificuldade: se não tivermos nenhuma diretriz teórica para guiar as observações, estas nunca poderão ser concluídas, já que a rigor teríamos que considerar uma infinidade de fatores. Essa constatação de que, por uma questão de princípio, a investigação científica não pode principiar com observações puras é reforçada pelo testemunho histórico. Os catálogos baconianos são uma ficção, nunca tendo sido elaborados por qualquer cientista. O cientista, quando vai ao laboratório, sempre tem uma idéia, ainda que provisória e reformulável, do que deve ou não ser observado, controlado, variado.



É interessante ainda lembrar que há casos notáveis de descobertas de leis científicas estimuladas por fatores não-empíricos. Um exemplo típico é a idéia ocorrida ao físico francês Louis de Broglie de que a matéria dita "ponderável" (elétrons, átomos, etc.) apresentaria um comportamento ondulatório. Essa idéia, que contribuiu decisivamente para os desenvolvimentos que levaram ao surgimento da mecânica quântica, não se baseava de modo direto em nenhuma evidência empírica disponível na época (1924), mas na consideração estética, de simetria, de que se a luz, tida como de natureza ondulatória, apresentava, em determinadas circunstâncias, um comportamento corpuscular (fato esse, aliás, também constatado depois de haver sido previsto teoricamente por Einstein), então os corpúsculos materiais igualmente deveriam, em certas circunstâncias, comportar-se como ondas.



As objeções que se têm levantado contra o princípio (b), da neutralidade das observações, são demasiadamente complexas para serem tratadas neste texto voltado a um público leigo. De forma simplificada, a análise filosófica e psicológica do processo de percepção fornece evidência de que o conteúdo mental (idéias, conceitos, juízos) formado quando se observa um determinado objeto ou conjunto de objetos varia significativamente de indivíduo para indivíduo, conforme sua bagagem intelectual. Em certo sentido, a apreensão da realidade se faz parcialmente mediante "recortes" próprios de cada observador, determinados por sua experiência prévia, as teorias que aceita, os objetivos que tem em vista. A tarefa de isolar elementos completamente objetivos, ou pelo menos inter-subjetivos, em nossas experiências está envolta em dificuldades maiores do que se supôs nas etapas iniciais do desenvolvimento da filosofia empirista moderna, quando se propunha que o material básico de todo conhecimento era um conjunto de "idéias", "impressões", "conceitos" ou "dados sensoriais" comuns. Parece que em cada ocasião em que a mente interage com algo, esses dados sensoriais já vêm inextricavelmente associados a interpretações, condicionadas pelos fatores apontados.



Tais constatações, porém, não devem conduzir a um subjetivismo completo, incompatível com aquilo que de fato se faz em nosso dia-a-dia e na ciência. Aliás, parte da atividade científica consiste justamente em se buscar uma descrição tão objetiva quanto possível do mundo, e o que está sendo aqui exposto visa apenas a indicar que esse ideal tem que ser buscado por meio de um controle crítico incessante dos fatores subjetivos inelimináveis. Ao contrário do que poderia resultar de uma abordagem estritamente kantiana dessa questão, defendemos que a "grade" intelectual segundo a qual percebemos a realidade não é fixa, determinada de forma totalmente independente de nosso arbítrio, mas pode ser adaptada por esforços deliberados, com a finalidade de se encontrar uma representação das coisas que mais se aproxime daquele ideal, maximizando-se simultaneamente a coerência e o poder explicativo de nosso conjunto de crenças e teorias.



3. Popper e o falseacionismo

Objeções incisivas à concepção comum de ciência, então vestida nas roupagens do positivismo lógico, foram levantadas já em 1934 pelo filósofo austríaco (mais tarde naturalizado britânico) Karl Popper, exatamente quando essa doutrina vivia o seu apogeu. Tais objeções, enfeixadas no livro Logik der Forschung, publicado em Viena naquele ano, foram ignoradas durante quase trinta anos, só recebendo atenção no final da década de 1950, quando os próprios positivistas lógicos já haviam admitido muitas limitações no seu programa original. Em 1959, o livro de Popper foi revisto, ampliado e vertido para o inglês, sob o título The Logic of Scientific Discovery. A partir de então (e, é claro, não somente pela influência desta obra) instalou-se um período de significativos avanços na filosofia da ciência, com o aperfeiçoamento e crítica das teses popperianas, e com o aparecimento de outras concepções de ciência, entre as quais se destacam as de Thomas Kuhn e Imre Lakatos.



A idéia central de Popper é a de substituir o empirismo justificacionista-indutivista da concepção tradicional por um empirismo não-justificacionista e não-indutivista, que ficou conhecido por falseacionismo. Popper rejeita que as teorias científicas sejam construídas por um processo indutivo a partir de uma base empírica neutra, e propõe que elas têm um caráter completamente conjetural. Teorias são criações livres da mente, destinadas a ajustar-se tão bem quanto possível ao conjunto de fenômenos de que tratam. Uma vez proposta, uma teoria deve ser rigorosamente testada por observações e experimentos. Se falhar, deve ser sumariamente eliminada e substituída por outra capaz de passar nos testes em que a anterior falhou, bem como em todos aqueles nos quais tenha passado. Assim, a ciência avança por um processo de tentativa e erro, conjeturas e refutações. "Aprendemos com nossos erros", enfatiza Popper, que traça um paralelo (com restrições importantes) entre a evolução da ciência e a evolução das espécies, segundo a teoria de Darwin-Wallace:



Nosso conhecimento consiste, em cada momento, daquelas hipóteses que mostraram sua (relativa) adaptação, por terem até então sobrevivido em sua luta pela existência, uma luta competitiva que elimina as hipóteses não-adaptadas. (Objective Knowledge, p. 261.)

A cientificidade de uma teoria reside, para Popper, não em sua impossível prova a partir de uma base empírica, mas em sua refutabilidade. Ele argumenta que somente as teorias passíveis de serem falseadas por observações fornecem informação sobre o mundo; as que estejam fora do alcance da refutação empírica não possuem "pontos de contato" com a realidade, e sobre ela nada dizem, mesmo quando na aparência digam, caindo no âmbito da metafísica. Alguns dos exemplos preferidos de Popper de teorias irrefutáveis, e portanto não-científicas, são a astrologia, a psicanálise e o marxismo.



Vejamos agora como a concepção falseacionista posiciona-se diante das características da ciência que constituíram embaraço à concepção indutivista tradicional.



Primeiramente, notemos que a visão falseacionista escapa completamente ao problema da justificação da indução, já que nela não se pretende que as teorias sejam provadas indutivamente. O vínculo empírico das teorias se localiza em sua refutabilidade. E aqui o falseacionismo explora habilmente a assimetria lógica que existe entre os processos de inferência de proposições particulares a partir de proposições gerais e de gerais a partir de particulares: se nenhum conjunto finito de proposições particulares pode levar logicamente uma proposição geral, a falsidade de uma proposição particular acarreta logicamente a falsidade da proposição que representa a sua generalização. Ilustremos o ponto retomando o nosso exemplo da lei segundo a qual todo papel é combustível. Conforme mencionamos, essa lei não pode ser provada logicamente por observações de pedaços de papel que se queimam. Porém se encontrarmos um único pedaço de papel incombustível, concluiremos logicamente que a referida lei é falsa.



Uma segunda vantagem da concepção falseacionista está em não pretender que a investigação científica comece por observações. Discorrendo sobre as relações entre observação e teoria, Popper afirma:



Acredito que a teoria ¾ pelo menos alguma expectativa ou teoria rudimentar ¾ sempre vem primeiro, sempre precede a observação; e que o papel fundamental das observações e testes experimentais é mostrar que algumas de nossas teorias são falsas, estimulando-nos assim a produzir teorias melhores.

Conseguintemente, digo que não partimos de observações, mas sempre de problemas ¾ seja de problemas práticos ou de uma teoria que tenha topado com dificuldades. (Objective Knowledge, p. 258.)

Isso isenta o falseacionismo de várias das objeções filosóficas, notadamente da relativa à necessidade de diretrizes teóricas na condução das observações, e também o colocam em concordância com o processo que efetivamente ocorre ao longo da história da ciência.



Por fim, além do apelo intuitivo do falseacionismo (em nossa vida prática, pelo menos, freqüentemente aprendemos com nossos erros), cabe mencionar que o compromisso com essa posição filosófica força a formulação das teorias de maneira clara e precisa. De fato, não é fácil ver como uma teoria obscura ou imprecisa possa ser submetida a testes rigorosos e, ainda que o seja, poderá ser sempre salva de um veredicto desfavorável por meio de reinterpretações, de manobras semânticas, o que trai sua irrefutabilidade, e portanto o seu caráter não-científico.



4. Limitações do falseacionismo

Embora represente um avanço em relação à concepção comum de ciência, o falseacionismo, tal qual o descrevemos acima, de modo simplificado, padece de várias limitações. Não faríamos justiça plena a Popper atribuindo-lhe essa forma tosca de falseacionismo, não obstante haja evidência textual que poderia ser evocada para essa atribuição, como gostam de notar seus opositores.



Foge ao escopo deste nosso trabalho efetuar uma análise dos muitos matizes do pensamento popperiano, bem como avaliar as críticas que lhe foram feitas. Diremos apenas que mesmo as versões mais sofisticadas do falseacionismo não estão isentas de dificuldades, o que deu lugar ao surgimento de diversas teorias da ciência alternativas. Essas teorias vão desde a metodologia dos programas científicos de pesquisa, de Lakatos, que representa um desdobramento das linhas popperianas, até o auto-denominado "dadaísmo metodológico", de Paul Feyerabend, que nega a existência de qualquer método na ciência. Daremos abaixo uma descrição breve das idéias centrais de Lakatos, não somente por suas virtudes intrínsecas, mas também por servir bem às nossas análises posteriores. Antes, porém, exporemos de forma sucinta algumas das objeções que se têm levantado contra o falseacionismo, e que motivaram o desenvolvimento das concepções lakatosianas.



A dificuldade mais fundamental enfrentada pelo falseacionismo é o chamado "problema de Duhem-Quine". Vimos acima que uma proposição geral como 'Todo papel é combustível' pode ser falseada por uma proposição particular como 'A folha de papel x não é combustível', cuja verdade usualmente se admite apoiar na experiência. No entanto, as teorias reais ou de algum interesse nunca são proposições gerais isoladas, mas conjuntos de tais proposições, e não podem, além disso, ser submetidas a testes empíricos senão quando suplementadas por teorias e hipóteses auxiliares (como as referentes ao funcionamento dos aparelhos eventualmente empregados na observação), proposições acerca das condições iniciais e de contorno, etc. Se então esse complexo de proposições permite inferir uma proposição que conflita com alguma proposição empírica, o máximo que a lógica nos informa é que o conjunto de proposições está refutado, caso se assuma a verdade da proposição empírica. Mas não nos habilita a singularizar como responsável por essa refutação uma das proposições do conjunto, nem mesmo o subconjunto delas que constitui a teoria particular que estamos procurando testar.



Ilustremos a dificuldade considerando uma situação que, segundo a concepção falseacionista, representaria a refutação de uma dada teoria mecânica por observações astronômicas. Para fixar idéias, tomemos essa teoria como sendo a mecânica newtoniana, que consiste de três leis dinâmicas, as conhecidas "leis de Newton", que denotaremos por L1, L2 e L3, e da lei da gravitação universal, que denotaremos por G. Uma eventual refutação dessa teoria por uma proposição empírica, E, implica necessariamente a possibilidade de se deduzir a partir dela uma proposição T logicamente incompatível com E. Em outros termos, diríamos neste caso que a previsão teórica T (a respeito, por exemplo, da trajetória de um dado planeta) foi contrariada pela experiência, expressa através da proposição E, estando assim refutada a teoria mecânica em questão.



O problema está em que o conjunto de leis L1, L2, L3 e G não basta para a dedução de nenhuma proposição do tipo de T. Para tanto, deve ser complementado por várias outras proposições, classificadas em duas categorias principais: De um lado, estão as proposições gerais (A1, A2, A3, ... ) de teorias auxiliares, como por exemplo as de teorias ópticas envolvidas na construção e operação dos telescópios usados na observação do planeta, na correção das aberrações ópticas introduzidas pela atmosfera terrestre, etc. De outro lado, há as proposições particulares (I1, I2, I3, ... ) referentes às chamadas condições iniciais do problema, como sejam as empregadas para especificar as massas e posições iniciais do planeta, da Terra, do Sol e dos demais planetas e satélites. Temos então que é somente o amplo conjunto de proposições L1, L2, L3, G, A1, A2, A3, ... I1, I2, I3, ... que permite inferir uma proposição T imediatamente confrontável com a observação. Se agora encontrarmos que essa proposição T é empiricamente falsa, poderemos concluir somente que a vasta conjunção de proposições que permitiu deduzi-la é falsa; mas a lógica não dá nenhuma indicação de qual (ou quais) proposição que a compõe é falsa; sabemos apenas que pelo menos uma deverá sê-lo, mas não qual. Assim, o conflito de T com a observação não pode ser interpretado como uma refutação da teoria mecânica em análise (e mesmo que pudesse, não saberíamos qual das leis que a compõem é falsa), pois a falha pode estar em qualquer uma das inúmeras proposições subsidiárias A1, A2, A3, ... I1, I2, I3, ... . Conforme se verifica pelo exame cuidadoso das situações reais de teste das teorias científicas, esse conjunto de proposições subsidiárias é em geral bastante extenso.



Quine expressou metaforicamente o problema em foco dizendo que "nossas proposições sobre o mundo externo enfrentam o tribunal da experiência sensível não individualmente, mas corporativamente" ("Two dogmas of Empiricism", seção 5). Recorreu ainda a duas imagens para figurar as relações entre teoria e experiência:



A totalidade de nosso assim chamado conhecimento ... é um tecido feito pelo homem, que toca a experiência somente em suas bordas. Ou, mudando a imagem, a ciência é como um campo de força cujas condições de contorno são a experiência. Um conflito com a experiência na periferia causa reajustes no interior do campo ... A reavaliação de algumas proposições acarreta a reavaliação de outras, devido às interconexões lógicas entre elas ... Mas o campo é de tal modo subdeterminado por suas condições de contorno (a experiência), que há muita liberdade de escolha sobre quais proposições devem ser reavaliadas à luz de qualquer experiência individual contrária. (Ibid., seção 6.)

Conforme vemos, o problema de Duhem-Quine incide sobre os próprios fundamentos da concepção falseacionista de ciência. Sua relevância é acentuada pelo testemunho da história da ciência, que fornece muitos exemplos de conflitos entre previsões teóricas e observações que foram resolvidos não pelo abandono da teoria particular que levou à previsão, mas por ajustes nas teorias subsidiárias requeridas para a efetivação do teste. Mencionamos anteriormente alguns exemplos importantes, que agora relembraremos, junto com mais alguns.



A teoria astronômica de Copérnico conflitava com a observada constância nas dimensões de Vênus e Marte ao longo do ano. O heliocentrismo não foi por isso tido como refutado por todos; muitos preferiram colocar em dúvida a assumida capacidade de nosso sistema visual perceber pequenas variações de tamanho de objetos brilhantes pequenos. O mesmo ocorreu com relação a inúmeras previsões mecânicas empiricamente falsas que os opositores do sistema copernicano deduziram da hipotética rotação da Terra: a produção de ventos fortíssimos na direção oeste; a projeção de todos os corpos soltos sobre a superfície da Terra; o desvio para oeste de corpos em queda livre; a Lua seria deixada para trás pela Terra em seu movimento de translação, etc. Bruno, Galileo, Kepler e outros não viram nessas abundantes conseqüências falsas da teoria heliocêntrica a sua refutação, preferindo atribuí-las às teorias mecânicas subjacentes, muito embora o desenvolvimento de uma nova mecânica, capaz de produzir previsões empíricas corretas a partir da teoria heliocêntrica, devesse ainda aguardar a contribuição de Newton, no final do século 17.



Por sua vez, a mecânica newtoniana dava resultados incorretos para a trajetória da Lua. Isso não foi interpretado como sua refutação; o ajuste empírico da teoria foi alcançado em meados do século 18, por modificações nas técnicas matemáticas envolvidas nos cálculos da trajetória lunar. Caso semelhante se deu com as previsões da teoria newtoniana para a órbita de Urano, incompatível com as observações astronômicas do início do século 19. Desta vez, a refutação da teoria foi evitada pelo questionamento das condições iniciais do problema, introduzindo-se a hipótese de um corpo celeste até então nunca observado, que modificaria as forças gravitacionais que atuam sobre aquele planeta. Esse hipotético corpo foi mais tarde detectado empiricamente, sendo o que hoje se conhece como o planeta Netuno.



Também já aludimos à hipótese que Prout propôs em 1815 acerca dos pesos atômicos dos elementos químicos, que conviveu durante quase cem anos com farta evidência empírica contrária. A discrepância foi atribuída a pressuposições referentes aos processos de purificação química. Aqui também esse redirecionamento da refutação mostrou-se justificado pelos desenvolvimentos científicos de nosso século.



Finalizando esta breve exposição das dificuldades do falseacionismo, temos ainda que mencionar que a ênfase que dá ao processo de falseamento das teorias conduz freqüentemente a uma subestimação do papel das confirmações no desenvolvimento da ciência. (Entendemos aqui 'confirmação' não no sentido da concepção tradicional de ciência, que em geral se confunde com 'prova'; por esse termo significamos apenas a evidência empírica favorável.)



Na versão tosca que lhe demos acima, o falseacionismo não reconhecia a importância das confirmações. Um tanto impiedosamente, poderíamos isolar muitas passagens dos escritos de Popper que parecem apoiar esse ponto de vista, como por exemplo esta prescrição feita à página 266 de seu Objective Knowledge: "Tenha por ambição refutar e substituir suas próprias teorias." Ou ainda estas frases de Conjectures and Refutations: "Observações e experimentos ... funcionam na ciência como testes de nossas conjeturas ou hipóteses, i.e., como tentativas de refutação" (p. 53). "Todo teste genuíno de uma teoria é uma tentativa de falseá-la ou refutá-la" (p. 36).



Não podemos disfarçar nossa estranheza diante de tais afirmações, dado seu contraste com a atitude usual dos cientistas, que vem norteando o desenvolvimento da ciência. Naturalmente, quando considerado em seu conjunto, o pensamento popperiano mostra-se mais refinado. Popper trata mesmo com alguma extensão o assunto da "evidência corroborativa". Não é claro, todavia, que ele tenha feito justiça plena ao papel que a confirmação efetivamente desempenha na ciência. Vejamos, por exemplo, este seu comentário específico sobre a questão: "Evidência confirmatória não deve contar, exceto quando é o resultado de um teste genuíno da teoria, ou seja, quando possa ser apresentada como uma tentativa séria, não obstante mal sucedida, de falsear a teoria." (Conjectures and Refutations, p. 36; o destaque é de Popper.) O desacordo com o que se observa na prática da ciência reside não no reconhecimento de que as "confirmações devem contar somente se são o resultado de predições arriscadas" (ibid., p. 36), mas na insistência em interpretar observações e experimentos como tentativas deliberadas de refutação. Definitivamente, parece não haver exemplos de cientistas que se tenham empenhado ansiosamente na refutação de suas próprias teorias, ou daquelas com as quais simpatizem. E o que vimos acima nos autoriza a concluir que se esse fosse o objetivo precípuo dos cientistas, não lhes faltariam razões para dar como refutadas todas as teorias científicas.



Além disso, há que observar a irrelevância de certas refutações para a ciência. Este ponto foi expresso com clareza por Chalmers em seu livro What Is This Thing Called Science? (pp. 51-2):



É um erro tomar a falseação de conjeturas ousadas e altamente falseáveis como ocasiões de significantes avanços na ciência ... Avanços significantes distinguem-se pela confirmação de conjeturas ousadas ou pela falseação de conjeturas prudentes. Casos do primeiro tipo são informativos, e constituem uma importante contribuição ao conhecimento científico, exatamente porque assinalam a descoberta de algo previamente não-cogitado ou tido como improvável ... As falseações de conjeturas prudentes são informativas porque estabelecem que o que era considerado pacificamente verdadeiro é de fato falso ... Em contraste, pouco se aprende com a falseação de uma conjetura ousada ou da confirmação de uma conjetura prudente. Se uma conjetura ousada é falseada, então tudo o que se aprende é que mais uma idéia maluca mostrou-se errada ... Semelhantemente, a confirmação de hipóteses prudentes ... indica meramente que alguma teoria bem estabelecida e vista como não-problemática foi aplicada com sucesso mais uma vez.



5. Lakatos: uma visão contemporânea da ciência

Do que vimos sobre as limitações das concepções indutivista e falseacionista de ciência, transparece que elas representam as teorias científicas e suas relações com a experiência de modo demasiadamente simples e fragmentário. A inspeção da natureza, gênese e desenvolvimento das teorias científicas reais evidencia que devem ser consideradas como estruturas complexas e dinâmicas, que nascem e se elaboram gradativamente, em um processo de influenciação recíproca com a experiência, bem como com outras teorias. Essa visão da ciência é ainda apoiada por argumentos de ordem filosófica e metodológica.



Se é verdade que as teorias científicas devem apoiar-se na experiência ¾ embora não dos modos descritos pelo indutivismo e pelo falseacionismo ¾, residindo mesmo nela a sua principal razão de ser, não é menos verdade que a busca, condução, classificação e análise dos dados empíricos requer diretrizes teóricas.



Além disso, a própria malha conceitual através da qual formulamos nossas idéias e experiências sensoriais constitui-se ao menos parcialmente pela atuação de nosso intelecto. No caso específico dos conceitos abstratos da ciência, o exame de sua criação e evolução mostra que surgem tipicamente como idéias vagas, só adquirindo significado gradualmente mais preciso na medida em que as teorias em que comparecem se estruturam, embasam e ganham coerência.



Por fim, em contraste com o que propõe a visão indutivista (e talvez também a falseacionista), as teorias científicas não consistem de meros aglomerados de leis gerais. Devem incorporar ainda regras metodológicas que disciplinem a absorção de impactos empíricos desfavoráveis, e norteiem as pesquisas futuras com vistas ao seu aperfeiçoamento.



O filósofo Imre Lakatos sistematizou de maneira interessante as características da ciência que vimos discutindo, introduzindo a noção de programa científico de pesquisa. Iniciaremos nossa breve e simplificada exposição das idéias centrais de Lakatos recorrendo a este parágrafo do citado livro de Chalmers (p. 76):



Um programa de pesquisa lakatosiano é uma estrutura que fornece um guia para futuras pesquisas, tanto de maneira positiva, como negativa. A heurística negativa de um programa envolve a estipulação de que as assunções básicas subjacentes ao programa, que formam o seu núcleo rígido, não devem ser rejeitadas ou modificadas. Esse núcleo rígido é resguardado contra falseações por um cinturão protetor de hipóteses auxiliares, condições iniciais, etc. A heurística positiva constitui-se de prescrições não muito precisas que indicam como o programa deve ser desenvolvido... Os programas de pesquisa são considerados progressivos ou degenerantes, conforme tenham sucesso, ou persistentemente fracassem, em levar à descoberta de novos fenômenos.

O núcleo rígido (hard core) de um programa é aquilo que essencialmente o identifica e caracteriza, constituindo-se de uma ou mais hipóteses teóricas. Eis alguns exemplos. O núcleo rígido da cosmologia aristotélica inclui, entre outras, as hipóteses da finitude e esfericidade do Universo, a impossibilidade do vazio, os movimentos naturais, a incorruptibilidade dos céus. O núcleo da astronomia copernicana consiste das assunções de que a Terra gira sobre si mesma em um dia e em torno do Sol em um ano, e de que os demais planetas também orbitam o Sol. O da mecânica newtoniana é formado pelas três leis dinâmicas e pela lei da gravitação universal; o da teoria especial da relatividade, pelo princípio da relatividade e pela constância da velocidade da luz; o da teoria da evolução de Darwin-Wallace, pelo mecanismo da seleção natural.



Por "uma decisão metodológica de seus protagonistas" (Lakatos 1970, p. 133), o núcleo rígido de um programa de pesquisa é "decretado" não-refutável. Possíveis discrepâncias com os resultados empíricos são eliminadas pela modificação das hipóteses do cinturão protetor. Essa regra é a heurística negativa do programa, e tem a função de limitar, metodologicamente, a incerteza quanto à parte da teoria atingida pelas "falseações". Recomendando-nos direcionar as "refutações" para as hipóteses não-essenciais da teoria, a heurística negativa representa uma regra de tolerância, que visa a dar uma chance para os princípios fundamentais do núcleo mostrarem a sua potencialidade. O testemunho da história da ciência parece de fato corroborar essa regra, como vimos nos exemplos que demos acima. Uma certa dose de obstinação parece ter sido essencial para salvar nossas melhores teorias científicas dos problemas de ajuste empírico que apresentavam quando de sua criação.



Lakatos reconhece, porém, que essa atitude conservadora tem seus limites. Quando o programa como um todo mostra-se sistematicamente incapaz de dar conta de fatos importantes e de levar à predição de novos fenômenos (i.e., torna-se "degenerante"), deve ceder lugar a um programa mais adequado, "progressivo". Como uma questão de fato histórico, nota-se que um programa nunca é abandonado antes que um substituto melhor esteja disponível.



A heurística positiva de um programa é mais vaga e difícil de caracterizar que a heurística negativa. Segundo Lakatos, ela consiste "de um conjunto parcialmente articulado de sugestões ou idéias de como mudar ou desenvolver as 'variantes refutáveis' do programa de pesquisa, de como modificar, sofisticar, o cinturão protetor 'refutável'." (op. cit. p. 135) No caso da astronomia copernicana, por exemplo, a heurística positiva indicava claramente a necessidade do desenvolvimento de uma mecânica adequada à hipótese da Terra móvel, bem como de novos instrumentos de observação astronômica, capazes de detectar as previstas variações no tamanho aparente dos planetas e as fases de Vênus, por exemplo. Assim, o telescópio foi construído algumas décadas após a morte de Copérnico pelo seu ardente defensor, Galileo, que também principiou a criação da nova mecânica. Esta, a seu turno, uma vez formulada por Newton, apontou para um imenso campo aberto, no qual se deveriam buscar uma nova matemática, medidas das dimensões da Terra, aparelhos para a detecção da força gravitacional entre pequenos objetos, etc.



Tentando uma representação gráfica de um programa de pesquisas lakatosiano teríamos mais ou menos o seguinte:











A concepção lakatosiana de ciência envolve um novo critério de demarcação entre ciência e não-ciência. Lembremos que o critério indutivista considerava científicas somente as teorias provadas empiricamente. Tal critério é, como vimos, forte demais: não haveria, segundo ele, nenhuma teoria genuinamente científica, pois todo conhecimento do mundo exterior é falível. Também o critério falseacionista, segundo o qual só são científicas as teorias refutáveis, elimina demais: como nenhuma teoria pode ser rigorosamente falseada, nenhuma poderia classificar-se como científica.



O critério de demarcação proposto por Lakatos, por outro lado, adequadamente situa no campo científico algumas das teorias unanimemente tidas como científicas, como as grandes teorias da física. Esse critério funda-se em duas exigências principais: uma teoria deve, para ser científica, estar imersa em um programa de pesquisa, e este programa deve ser progressivo. Deixemos a Lakatos a palavra (1970, pp. 175-6):



Pode-se compreender muito pouco do desenvolvimento da ciência quando nosso paradigma de uma porção de conhecimento científico é uma teoria isolada, como 'Todo cisne é branco', solta no ar, sem estar imersa em um grande programa de pesquisa. Minha abordagem implica um novo critério de demarcação entre 'ciência madura', que consiste de programas de pesquisa, e 'ciência imatura', que consiste de uma colcha de retalhos de tentativas e erros ...

A ciência madura consiste de programas de pesquisa nos quais são antecipados não apenas fatos novos, mas também novas teorias auxiliares; a ciência madura possui 'poder heurístico', em contraste com os processos banais de tentativa e erro. Lembremos que na heurística positiva de um programa vigoroso há, desde o início, um esboço geral de como construir os cinturões protetores: esse poder heurístico gera a autonomia da ciência teórica.

Essa exigência de crescimento contínuo [progressividade do programa] é minha reconstrução racional da exigência amplamente reconhecida de 'unidade' ou 'beleza' da ciência. Ela põe a descoberto a fraqueza de dois tipos de teorização aparentemente muito diferentes entre si. Primeiro, evidencia a fraqueza de programas que, como o marxismo ou o freudismo, são indubitavelmente 'unificados', e fornecem um plano geral do tipo de teorias auxiliares que irão utilizar para a absorção de anomalias, mas que invariavelmente criam suas teorias na esteira dos fatos, sem ao mesmo tempo anteciparem fatos novos. (Que fatos novos o marxismo previu desde, digamos, 1917?) Em segundo lugar, ela golpeia seqüências remendadas de ajustes 'empíricos' rasteiros e sem imaginação, tão freqüentes, por exemplo, na psicologia social moderna. Tais ajustes podem, com o auxílio das chamadas 'técnicas estatísticas', produzir algumas predições 'novas', podendo mesmo evocar alguns fragmentos irrelevantes de verdade que encerrem. Semelhantes teorizações, todavia, não possuem nenhuma idéia unificadora, nenhum poder heurístico, nenhuma continuidade. Não indicam nenhum programa de pesquisa, e são, no seu todo, inúteis.



Referências

BACON, F. Novum Organum. Trad. A.R. de Andrade. São Paulo, Abril Cultural, 1973.

CHALMERS, A.F. What is this Thing called Science? St. Lucia, University of Queensland Press, 1976.

EINSTEIN, A. Autobiographical notes. Trad. P.A. Schilpp. In: Schilpp 1949, pp. 3-94. 1949a.

LAKATOS, I. Falsification and the methodology of scientific research programmes. In: Lakatos & Musgrave 1970, pp. 91-195.

LAKATOS, I. & MUSGRAVE, A. Criticism and the Growth of Knowledge. Cambridge, Cambridge University Press, 1970.

LOCKE, J. An Essay Concerning Human Understanding. London, Oxford University Press, 1975.

POPPER, K.R. The Logic of Scientific Discovery. 5.ed., revista. London, Hitchison, 1968.

¾¾¾. Conjectures and Refutations. 4.ed., revista. London, Routledge and Kegan Paul, 1972.

¾¾¾. Objective Knowledge. Oxford, Clarendon Press, 1972.

QUINE, W.V.O. Two dogmas of empiricism. In: Quine 1953, pp. 20-46.

¾¾¾. From a Logical Point of View. Cambridge, Mass., 1953.

SCHILPP, P. A. Albert Einstein: Philosopher-Scientist. 3rd. ed. La Salle, Illinois, Open Court, 1949.








Com esta mensagem eletrônica
seguem muitas vibrações de paz e amor
para você
---------




cancelar assinatura - página do grupo


[As partes desta mensagem que não continham texto foram removidas]



SUBJECT: Fw: O GRUPO DE ESTUDOS ESPÍRITAS DA UNICAMP: HISTÓRICO E DIRETRIZES
FROM: "Amauri Jr" <amaurijunior2@yahoo.com.br>
TO: "Lucimary Vargas" <sangalli@uai.com.br>, <transconhecimento@yahoogrupos.com.br>, Flávia Mendes Sanches <fla1202@yahoo.com.br>, "kathleen_mel" <kathleen_mel@uol.com.br>, <Euthymia@yahoogrupos.com.br>, <Conversa_de_Botequim@yahoogrupos.com.br>, <ciencialist@yahoogrupos.com.br>, <acropolis@yahoogrupos.com.br>
DATE: 09/01/2005 14:36


----- Original Message -----
From: A ERA DO ESPÍRITO
To: aeradoespirito@grupos.com.br
Sent: Monday, December 27, 2004 9:40 PM
Subject: [Estudos Espíritas] O GRUPO DE ESTUDOS ESPÍRITAS DA UNICAMP: HISTÓRICO E DIRETRIZES


O GRUPO DE ESTUDOS ESPÍRITAS DA UNICAMP:
HISTÓRICO E DIRETRIZES
Silvio Seno Chibeni


Resumo:
Neste trabalho relata-se a experiência do Grupo de Estudos Espíritas da Unicamp (GEEU), que promove reuniões semanais de estudo no campus universitário desde 1979. Inicialmente, apresenta-se um breve histórico do Grupo. Em seguida, as diretrizes que têm direcionado suas atividades são expostas e comentadas.



I. UM BREVE HISTÓRICO
1. As origens
Ao ingressar na universidade, em 1977, fiquei sabendo da existência Grupo Universitário de Estudos Espíritas (GUEE), que se reunia semanalmente nas dependências da Faculdade de Engenharia de Alimentos. Aberto à participação de todos os interessados, esse grupo era coordenado por alguns alunos bastante dedicados à tarefa. Deixou de existir no final de 1978 ou início do ano seguinte, quando quase todos os seus integrantes concluíram suas atividades na Unicamp.



Pouco depois, muito provavelmente no ano de 1979, alguns alunos espíritas, em sua maioria dos cursos de graduação em física e química, convidaram-me para uma reunião de estudos, na sala IF-15 do Instituto de Física Gleb Wataghin. Éramos não mais do que dez pessoas, nenhuma das quais, com exceção de mim, havia participado do extinto GUEE. Afigura-se-me à percepção presente que foi a primeira reunião do atual grupo de estudos. Consolidando-se gradativamente, e divulgando suas atividades, passou a ser freqüentado também por funcionários e, mais tarde, por alguns docentes da universidade.

2. As reuniões
Visto que a universidade é uma instituição que, por sua natureza, não se dedica a pesquisas espíritas, sempre tivemos o cuidado de solicitar autorização oficial para utilizarmos suas dependências. Essa circunstância influiu na transferência do Grupo, poucas semanas após a referida reunião, para uma das salas de aula do Instituto de Química (IQ). À época, o Instituto tinha como diretor o Professor Aécio Pereira Chagas, que desde então vem colaborando com o Grupo, não apenas quando das renovações anuais do pedido de uso de sala (para o que alguns outros docentes do IQ também têm emprestado seu apoio), mas também participando ativamente das reuniões, em diversas oportunidades.



Nos anos iniciais, havia em geral mais do que uma reunião por semana, até o máximo de quatro, durante um ou dois semestres, com vistas ao desenvolvimento de estudos específicos de diversas obras. Depois, fatores circunstanciais diversos acabaram determinando a concentração das atividades em apenas uma reunião semanal, como ocorre hoje.

No final de 1994 o Grupo transferiu-se para o auditório da Diretoria Geral da Administração (DGA), na qual à época trabalhavam alguns de seus mais assíduos integrantes. Pudemos utilizar esse local até o final de 1996. No presente ano, reinstalamo-nos no IQ.



As reuniões têm duração aproximada de uma hora e meia, e são iniciadas e encerradas com breves momentos de prece. O número de pessoas tem variado muito ao longo dos anos e ao longo dos períodos letivos. Ordinariamente, observa-se acentuado declínio da freqüência nos finais de semestre, época de provas e exames. Houve tempos em que estávamos presentes apenas uma ou duas pessoas. Atualmente, têm comparecido de quinze a trinta pessoas, acima portanto, da média histórica, que deve girar em torno de dez freqüentadores.

3. Algumas dificuldades
Entre os problemas de ordem material enfrentados pelo Grupo, destaca-se a alteração e, por vezes, a precariedade do local de reunião. Ressente-se também da insuficiência e efemeridade dos meios de divulgação.



Mas, acima de tudo, a flutuação do público tem sido o fator que mais negativamente interfere no desenvolvimento de estudos sistemáticos e seqüenciais ao longo dos anos. Essa flutuação tem um componente inevitável o fato de que, por sua natureza, o corpo discente da universidade não é fixo , e outro que se liga ao interesse e dedicação incertos de boa parte daqueles que se aproximam do Grupo. A experiência evidencia que nos agrupamentos espíritas costuma ser reduzida a fração daqueles que perseveram no estudo, que assumem responsabilidades e flexibilizam suas demais atividades em benefício da causa espírita.

4. Atividades adicionais
A finalidade precípua do GEEU é o estudo sistematizado do Espiritismo em suas reuniões semanais. No intento de incrementar a divulgação do Espiritismo, nos anos de 1995 e 1996 o Grupo promoveu três semanas espíritas, que consistiram de ciclos de palestras e feiras do livro espírita. Nas feiras, procuramos dar ênfase às obras de Allan Kardec e outras de reconhecido valor doutrinário. Serviram não somente para divulgar a literatura espírita, mas também para dialogar mais diretamente com pessoas da comunidade universitária interessadas no Espiritismo.



Desde 1995, um membro do Grupo mantém na Internet uma homepage destinada a divulgar o Espiritismo em sua pureza original. Trata-se de uma das primeiras iniciativas do gênero, explorando esse novo canal de divulgação doutrinária, cuja importância é cada dia maior. Voltada prioritariamente à comunidade internacional, essa página é escrita quase que integralmente em inglês, e tem apresentado expressivo e compensador retorno. (Nota de 10/3/97: por razões de força maior, a homepage original, intitulada "Spiritism to the World", encontra-se temporariamente fora do ar.)

II. DIRETRIZES DE ATUAÇÃO
1. Divulgação.
O GEEU sempre julgou indispensável estar aberto à participação de todos os interessados. A cada semestre, renova-se a divulgação das reuniões por toda a universidade, por meio de cartazes, boletins de notícias (quando possível) e, mais recentemente, pela Internet.

2. Fidelidade doutrinária.
O GEEU é um grupo de estudo de Espiritismo. Embora respeitemos as demais abordagens do elemento espiritual, de natureza religiosa, mística, filosófica ou supostamente científica, acreditamos que, assim como ocorre nas academias, deve haver uma especialização de atividades. Pessoas que queiram estudar essas vertentes não-espíritas têm toda a liberdade de formar outros núcleos e participar das instituições que melhor atendam aos seus interesses. O GEEU, porém, tem por finalidade exclusiva manter dentro da universidade um espaço no qual se possa aprender e aprofundar os conhecimentos genuinamente espíritas.



Análises do Espiritismo à luz da moderna filosofia da ciência revelam seguramente que ele constitui uma verdadeira ciência, um programa científico de pesquisa, ou paradigma científico, com corpo teórico, conceitos e metodologia próprios e autônomos, como lucidamente notou o próprio Allan Kardec. No primeiro capítulo de seu livro A Gênese, ele adverte que embora o Espiritismo seja progressivo e aberto como qualquer ciência, seu desenvolvimento deve se processar segundo os recursos heurísticos e teóricos do próprio programa espírita, sem a enxertia apressada de elementos estranhos, venham de onde vierem. Reconhecendo as "verdades práticas" das demais ciências, o Espiritismo estará em harmonia com elas, naquilo que houverem estabelecido de forma estável. Seu papel é complementá-las na investigação da Natureza, examinando o elemento espiritual, de que elas não se ocupam, por sua própria concepção.



Igualmente, pode-se mostrar que as propostas de investigação dos aspectos espirituais da realidade que surgiram depois do Espiritismo com o propósito de suplantá-lo enquanto ciência, tais como a metapsíquica e a parapsicologia, invariavelmente não lograram alcançar esse objetivo, por falhas conceituais, teóricas e metodológicas diversas. É comum que pessoas e grupos do meio universitário interessados no estudo de fenômenos espíritas não percebam adequadamente esse fato, e acreditem que o estudo científico de tais fenômenos deva se desenvolver segundo essas perspectivas paralelas ao Espiritismo. Indivíduos com esse enfoque aparecem, de tempos em tempos, no nosso Grupo, afastando-se alguns, quando verificam que ali estudamos "só" Espiritismo, integrando-se outros, quando se dispõem a aprofundar conosco a questão. (Aliás, esse assunto foi, em diversas ocasiões, detalhadamente examinado por nós, resultando daí alguns textos que se encontram publicados na imprensa espírita. Veja-se, por exemplo, Chibeni 1988 e 1994. Os artigos de Aécio Chagas, Ademir Xavier Jr. e Juvanir Borges de Souza listados no final poderão também ser consultados a esse respeito.)

3. O roteiro de estudos
Embora este ponto não estivesse claro para todos os participantes das reuniões iniciais do Grupo, gradualmente foi-se estabelecendo que os estudos deveriam gravitar em torno do núcleo doutrinário estabelecido por Allan Kardec. Desse modo, alguns meses após sua criação o GEEU já tomava o "O Livro dos Espíritos" como o centro de suas atenções. Não apenas essa obra aborda de maneira segura, embora por vezes sucinta, todos os princípios centrais do Espiritismo, mas igualmente define-lhe os conceitos fundamentais. Além disso, sua organização didática dos assuntos pode ser aproveitada como um excelente roteiro de estudos, que enseja as oportunas complementações, com base nas demais obras de Kardec e da literatura espírita de boa qualidade que surgiu após elas.



Conforme já mencionado, houve épocas em que o GEEU manteve mais de uma reunião semanal: uma para o estudo do "O Livro dos Espíritos", outra para o "O Livro dos Médiuns", outra para o "O Evangelho segundo o Espiritismo" e outra para principiantes. Essa multiplicidade de reuniões especializadas não se pôde sustentar por muito tempo, dada a falta de recursos humanos. Mas por diversos anos conseguimos manter a reunião dedicada ao estudo aprofundado do "O Livro dos Médiuns", que constitui a base experimental da ciência espírita. Posteriormente tentamos, em vista do interesse mais direto das pessoas que então freqüentavam o Grupo, introduzir um roteiro de temas que não obedeciam à ordem estrita do "O Livro dos Espíritos", sempre porém, com base nas obras fundamentais. Esse roteiro mostrou-se útil durante algum tempo. De uns anos para cá, retomamos a seqüência do "O Livro dos Espíritos", ao lado de alguns tópicos especiais, como a história do Espiritismo e questões relativas ao movimento espírita.



Não nos propomos a percorrer tantos capítulos ou itens em tanto tempo, como é comum acontecer em cursos de instituições espíritas mais formalizadas. O que nos interessa é a compreensão satisfatória dos textos básicos, desde a introdução, parágrafo por parágrafo, bem como dos pontos complementares suscitados por seu estudo. Não temos pressa de cumprir programas, mas de aprender. Tem acontecido de o estudo de um único capítulo de "O Livro dos Espíritos" estender-se por quase um semestre.



Dado o caráter flutuante do grosso dos freqüentadores, há uma recorrente necessidade de esclarecer e orientar novos membros, que não raro chegam com um nível de conhecimento doutrinário elementar, ou com distorções de compreensão doutrinária. Isso nos leva a abrir "parênteses" mais ou menos longos no roteiro estabelecido. Se tal circunstância dificulta o aprofundamento e o estudo metódico dos temas, por outro lado é levada em conta em nossa proposta. Não estamos lá para nos encerrar em um círculo de pretenso saber; além de não corresponder à realidade, isso representaria a extinção certa do Grupo. Queremos tornar a idéia espírita disponível a todos os interessados que se acerquem de nós, seja qual for a sua condição doutrinária ou cultural, contanto que movidos pela vontade sincera de aprender. Em meio a um movimento espírita onde vicejam tantas idéias mal fundamentadas e tantas esquisitices, acalentamos a esperança de contribuir para a preservação da doutrina, ainda que de forma muito limitada, divulgando textos fundamentais e ressaltando sua excelência em nossas discussões.

4. Desvinculação institucional.
Não obstante a seriedade com que sempre procurou atuar, o GEEU nunca pretendeu institucionalizar-se.



Não obstante constitua uma verdadeira ciência (e também, pode-se argumentar, uma filosofia e uma religião), o Espiritismo distingue-se das ciências acadêmicas, pela especificidade de seu objeto de estudo e de seus objetivos. Assim, não vemos como apropriada a sua inserção institucional nas academias, pelo menos na presente era da Humanidade (veja-se Chagas 1994).



Quanto às relações do GEEU com o movimento espírita, igualmente não formalizadas, têm sido harmônicas e produtivas. Vários dos participantes do Grupo desenvolvem atividades regulares ou excepcionais junto a instituições espíritas de Campinas e de outras cidades. O Grupo também já teve a oportunidade de convidar pessoas ligadas a essas instituições para apresentar palestras ou seminários, especialmente quando da realização das semanas espíritas. As três feiras de livros promovidas nessas ocasiões contaram com a eficiente e simpática colaboração da distribuidora de livros da USE-Campinas.

5. Integração fraterna.
Temos envidado esforços para que as relações humanas entre os membros do Grupo sejam o mais fraternas possível. Gostaríamos que laços de amizade se juntassem ao interesse comum pelo estudo do Espiritismo, e isso em muitos casos tem-se verificado. O clima fraterno não apenas faz parte essencial de nossa vida moral, incrementando as condições de nossa felicidade, mas também mostra-se altamente favorável para o próprio desenvolvimento dos estudos, contribuindo para a superação dos melindres, da timidez, do isolamento durante as reuniões.

Referências
BORGES DE SOUZA, J. "Pesquisas e métodos", Reformador, abril de 1986, pp. 99-101.

CHAGAS, A. P. "O que é a ciência", Reformador, março de 1984, pp. 80-83 e 93-95. "As provas científicas", Reformador, agosto de 1987, pp. 232-33. "A Ciência confirma o Espiritismo?" Reformador, julho de 1995, pp. 208-11. "O Espiritismo na Academia?" Revista Internacional de Espiritismo, fevereiro de 1994, pp. 20-22 e março de 1994, p. 41-43.

CHIBENI, S.S. "A excelência metodológica do Espiritismo", Reformador, novembro de 1988, pp. 328-33, e dezembro de 1988, pp. 373-78. "O paradigma espírita", Reformador, junho de 1994, pp. 176-80.

XAVIER JR., A. L. "Algumas considerações oportunas sobre a relação Espiritismo-Ciência", Reformador, agosto de 1995, pp. 244-46.





----------------------------------------------------------------------------


Local e dia das reuniões:
Sala IQ-21 do Bloco F do Instituto de Química da Unicamp, toda quinta-feira, das 12 às 13:30 h.



A colaboração de todos na divulgação de nossas atividades é muito importante!



Textos presentemente em estudo no GEEU:

Programação para o primeiro semestre de 1999:
Manteremos o esquema do semestre passado, reservando as primeiras quintas do mês para temas evangélicos. O primeiro tema abordado será a Amizade.



Nos demais dias, continuaremos o estudo de "O Livro dos Espíritos". Decidimos recomeçar a partir do primeiro capítulo da segunda parte. Após a conclusão desse capítulo, provavelmente abriremos espaço para estudar um pouco o tema Mediunidade, com base no "O Livro dos Médiuns" e no texto "Estudo sobre a mediunidade" (disponível nesta homepage, seção "Artigos").



This page is hosted by Geocities. Get your free homepage and e-mail at http://www.geocities.com

Artigo publicado em Reformador, abril de 1986, pp. 102-3.











Esta mensagem eletrônica foi enviada à você
com as melhores vibrações
de paz e amor

----------





cancelar assinatura - página do grupo


[As partes desta mensagem que não continham texto foram removidas]



SUBJECT: Re: neutrino, existir or not be?
FROM: "rayfisica" <rayfisica@yahoo.com.br>
TO: ciencialist@yahoogrupos.com.br
DATE: 09/01/2005 18:22


--- Em ciencialist@yahoogrupos.com.br, "JVictor" <jvoneto@u...>
escreveu
> Caro Rayfisica,
>
>
> Minha confiança nas leis da natureza e, sobretudo, nas de
conservação, me dão a convicção inabalável de que o
neutrino existe
mesmo. Sua existência foi comprovada desde a década de 50.
Não há
dúvidas a respeito.

JVictor

Sua resposta é tão clara e precisa que não cabe
contraditório, alem
de que demonstra seu conhecimento.
E aproveitando para beber na fonte, pergunto que dizer da teoria de
que o neutrino pode violar a conservação da energia apresentada
por
alguns físicos para explicar a defasagem de neutrinos solares?

Outra porque de vez em quando ainda entre os "
respeitáveis" surge a
possibilidade de queda da conservação da energia, por exemplo,
quando
surgiu o mago de Manchester com a coisa, uma das possibilidades
levantada por alguns especialistas seria um pequeno moto-continuo?

Caso não Houvesse a conservação da energia a ciência
física não
existiria? A partir de onde não existiria Newton? Sobraria o
que?






SUBJECT: Satélites da NASA e gorilas
FROM: Maria Natália <grasdic@hotmail.com>
TO: ciencialist@yahoogrupos.com.br
DATE: 09/01/2005 22:05


Que relação?
Determinar se uma dada espécie está em risco de extinção tem muitos
obstáculos: guerra, pobreza, falta de envolvimento governamental,
inacessibilidade e a incerteza dos locais remotos. Ainda se sabe
pouco (ou tardiamente) do estado de espécies, como os gorilas. Para
evitar tal a tecnologia de satélites pode ser aplicada nas áreas
naturais:
http://www.physorg.com/news2613.html

Já convivi com gorilas no Congo e gostei...eram mais humanos que
alguns Homens...
Um abraço
Maria Natália







SUBJECT: GM e hidrogénio e um pouco de Química
FROM: Maria Natália <grasdic@hotmail.com>
TO: ciencialist@yahoogrupos.com.br
DATE: 09/01/2005 22:18


A GM e os laboratório nacional SANDIA assinaram um protocolo para a
construção de método avançado em armazenamento de hidrogénio
recorrendo a hidretos metálicos (HNa,HLi...)
Os hidretos metálicos (bastante reactivos, por sinal) formam-se
quando o hidrogénio reage com as ligas metálicas. Estes compostos
podem absorver hidrogénio em sua estrutura que depois, nas células
de combustível, pode ser liberado para se combinar com o oxigénio e
produzir enrgia eléctrica. Ver mais em:
http://www.physorg.com/news2612.h
Maria Natália





SUBJECT: Ressonância de Shumann
FROM: "Cyberlander" <mrcyberlander@superig.com.br>
TO: <Undisclosed-Recipient:;>
DATE: 09/01/2005 23:13

Ressonância de Shumann

Autor: Leonardo Boff




Não apenas as pessoas mais idosas mas também jovens fazem a experiência de que tudo está se acelerando excessivamente. Ontem foi Carnaval, dentro de pouco será Páscoa, mais um pouco, Natal. Esse sentimento é ilusório ou tem base real?



Pela ressonância Schumann se procura dar uma explicação. O físico alemão W.O. Schumann constatou em 1952 que a Terra é cercada por um campo eletromagnético poderoso que se forma entre o solo e a parte inferior da ionosfera, cerca de 100km acima de nós. Esse campo possui uma ressonância (dai chamar-se ressonância Schumann), mais ou menos constante, da ordem de 7,83 pulsações por segundo.



Funciona como uma espécie de marca-passo, responsável pelo equilíbrio da biosfera, condição comum de todas as formas de vida. Verificou-se também que todos os vertebrados e o nosso cérebro são dotados da mesma frequência de 7,83 hertz.



Empiricamente fez-se a constatação de que não podemos ser saudáveis fora dessa frequência biológica natural. Sempre que os astronautas, em razão das viagens espaciais, ficavam fora da ressonância Schumann, adoeciam. Mas submetidos à ação de um simulador Schumann recuperavam o equilíbrio e a saúde. Por milhares de anos as batidas do coração da Terra tinham essa freqüência de pulsações e a vida se desenrolava em relativo equilíbrio ecológico. Ocorre que a partir dos anos 80, e de forma mais acentuada a partir dos anos 90, a freqüência passou de 7,83para 11 e para 13 hertz.



O coração da Terra disparou. Coincidentemente, desequilíbrios ecológicos se fizeram sentir: perturbações climáticas, maior atividade dos vulcões, crescimento de tensões e conflitos no mundo e aumento geral de comportamentos desviantes nas pessoas, entre outros. Devido à aceleração geral, a jornada de 24 horas, na verdade, é somente de 16 horas. Portanto, a percepção de que tudo está passando rápido demais não é ilusória,mas teria base real nesse transtorno da ressonância Schumann.



Gaia, esse superorganismo vivo que é a Mãe Terra, deverá estar buscando formas de retornar a seu equilíbrio natural. E vaiconsegui-lo, mas não sabemos a que preço, a ser pago pela biosfera e pelos seres humanos. Aqui abre-se o espaço para grupos esotéricos e outros futuristas projetarem cenários, ora dramáticos, com catástrofes terríveis, ora esperançadores, como a irrupção da quarta dimensão, pela qual todos seremos mais intuitivos, mais espirituais e mais sintonizados com o biorritmo da Terra.



Não pretendo reforçar esse tipo de leitura. Apenas enfatizo a tese recorrente entre grandes cosmólogos e biólogos de que a Terra é, efetivamente, um superorganismo vivo, de que Terra e humanidade formamos uma única entidade, como os astronautas testemunham de suas naves espaciais. Nós, seres humanos, somos Terra que sente, pensa, ama e venera. Porque somos isso, possuímos a mesma natureza bioelétrica e estamos envoltos pelas mesmas ondas ressonantes Schumann.



Se queremos que a Terra reencontre seu equilíbrio, devemos começar por nós mesmos: fazer tudo sem estresse, com mais serenidade, com mais harmonia, com mais amor, que é uma energia essencialmente harmonizadora. Para isso importa termos coragem de ser anticultura dominante, que nos obriga a ser cada vez mais competitivos e efetivos. Precisamos respirar juntos com a Terra, para conspirar com ela pela paz.



[ ]'s

D.C.



CYBERLANDER

Ama a realidade que constróis,
que nem a morte deterá teu voo · ·




[As partes desta mensagem que não continham texto foram removidas]



SUBJECT: Re: [ciencialist] Re: neutrino, existir or not be?
FROM: "JVictor" <jvoneto@uol.com.br>
TO: <ciencialist@yahoogrupos.com.br>
DATE: 09/01/2005 23:59


----- Original Message -----
From: rayfisica
To: ciencialist@yahoogrupos.com.br
Sent: Sunday, January 09, 2005 6:22 PM
Subject: [ciencialist] Re: neutrino, existir or not be?



--- Em ciencialist@yahoogrupos.com.br, "JVictor" <jvoneto@u...>
escreveu
> Caro Rayfisica,
>
>
> Minha confiança nas leis da natureza e, sobretudo, nas de
conservação, me dão a convicção inabalável de que o
neutrino existe
mesmo. Sua existência foi comprovada desde a década de 50.
Não há
dúvidas a respeito.

JVictor

Ray: Sua resposta é tão clara e precisa que não cabe
contraditório, alem
de que demonstra seu conhecimento.
E aproveitando para beber na fonte, pergunto que dizer da teoria de
que o neutrino pode violar a conservação da energia apresentada
por
alguns físicos para explicar a defasagem de neutrinos solares?

Victor: sem a menor dúvida garanto que há ainda algum aspecto do processo não compreendido, o pedaço que falta para equilibrar o balanço energético. Vamos ressuscitar Pauli que num instante as coisas se esclarecem. Mas propalar que a conservação da energia foi violada, que um mot-contínuo foi construído, que Einstein errou, que sua teoria é nada, dá ibope, atrai atenções e se fica conhecido. Mais cedo ou mais tarde o culpado pela quebra de simetria será encontrado!

Ray: Outra porque de vez em quando ainda entre os "
respeitáveis" surge a
possibilidade de queda da conservação da energia, por exemplo,
quando
surgiu o mago de Manchester com a coisa, uma das possibilidades
levantada por alguns especialistas seria um pequeno moto-continuo?

Victor: a resposta stá no Teorema de Noether, como comentei. Não há como fugir dele. Outra resposta seria: falta de conhecimento adequado, falta de compreensão das leis existentes, ou compreensão equivocada, ou sei lá o quê. Acho que Freud explica.

Ray: Caso não Houvesse a conservação da energia a ciência
física não
existiria?

Victor: Veja bem, Ray. No texto anterior não mencionei algumas coisas importantes e que são igualmente determinantes nessa questão de conservação. São elas: a muito bem estabelecida, pela observação e pelas medições, hipótese da homogeneidade do tempo e do espaço. A homogeneidade do tempo que dá, como consequência, uma lei de conservação: a da energia mecânica, que é válida tanto para sistemas fechados como para sistemas que se encontram em um campo constante, independente do tempo; a homogeneidade do espaço, que me garante que as propriedades mecânicas de um sistema fechado não mudam sob qualquer translação paralela. Isto dá, como consequência, a conservação de uma quantidade chamada quantidade de momento linear, ou quantidade de movimento linear, conforme já mencionamos; e a isotropia do espaço, que me garante que as propriedades mecânicas não variam qualquer que seja a rotação de todo o sistema, no espaço. Trata-se agora do momentum angular ou momento de impulso, que se conserva(para aprofundamentos a esse respeito, veja Mecânica, de Landau, Mec.Analítica, de Nivaldo A.Lemos ou Introdução à Teoria da Relatividade, de R.Resnick)). Se não houvesse homogneidade do tempo, num instante t1, teríamos F=ma e num instante t1 + dt teríamos F2 = ma(t) + f(t), ou outra relação, f(t) sendo alguma coisa introduzida por essa variação no tempo. Ou seja, a segunda lei de Newton, ou outra qualquer teria um jeito diferente a cada momento!. Se o espaço e o tempo não fossem homogênios, um comprimento ou um intervalo de tempo(para um mesmo evento) seriam diferentes, conforme fossem observados e medidos aqui ou acolá.
E, ainda mais, de acordo com essas hipóteses poderosas, homogeneidade (do tempo e espaço) e isotropia do espaço(que diz que o espaço tem as mesmas propriedades em todas as direções e nos dá a lei da conservação do momento angular), as relações matemáticas entre as medidas feitas por dois observadores inerciais são, obrigatoriamente lineares. Ou seja, as coordenadas espaciais e temporais de um sistema S e as coordenadas de outro sistema K, ambos inerciais, com velocidade relativa v entre eles, são necessariamente lineares. Sem perdão. Qualquer outro tipo de relação entre as coordenadas de um e de outro sistema que não seja linear viola as hipóteses da homogeneidade e isotropia, que sabemos serem corretas. Se assim não fosse, também não haveria leis de conservação ou simetrias. Assim, certamente, respondendo a tua pergunta, a física não existiria, no sentido de que as leis atuais de nada serviriam. Você teria de escrever um livro de física, com novas leis a cada instante, em cada casa ou apartamento.
Não é demais acrescentar que o espaço não privilegia posições no espaço ou direções. Todos têm o mesmo status. E isto se torna mais claro, patente, no espaço tempo quadri-mensional, onde, por exemplo, o momento e a energia se misturam para um compor uma única entidade físico-matemática, que é uma coisa chamada tensor momento-energia, elemento fundamental da equação de Einstein, da RG. No espaço nosso, de meras (e chega!) 3 dimensões, o momento e a energia são entidades separadas, sem ligação direta. A relatividade me garante que a invariância de velocidade da luz aponta para outra simetria da natureza, simetria essa que costurou, inexorávelvelmente, o espaço e o tempo, transformando os dois numa só e única realidade, chamada espaçotempo, como disse seu descobridor, Minkowski.
Falei muito em simetrias da natureza, mas acho que não disse de sua real importância. Vou dizer agora. Quando mais simétrico for um sistema, menos variáveis serão necessárias para descrevê-lo. Imagine uma esfera. Do jeito que você olhar para ela, você verá sempre as mesmas coisas; pode fazer o quiser, olhar de baixo, de cima, rodá-lo, olhar através de um espelho, qualquer coisa. Verá sempre a mesma coisa. Nada muda. A esfera é o o objeto mais simples e mais simétrico que podemos mencionar, com uma característica: através de uma única variável, seu raio, a esfera é completamente descrita. As simetrias de que falei estão, assim, relacionadas ao espaço tempo. Mas há outras quantidades que se conservam, e que não têm relação com o espaçotempo, que ninguém é de ferro. Exemplo: a carga elétrica. Mas isto já é outra história. Há muito, muito o que falar e aprender a respeito.

Ray: A partir de onde não existiria Newton? Sobraria o
que?

Victor: Acho que os argumentos anteriores podem responder a isso, de maneira a não restarem dúvidas. Talvez o grande Newton inventasse um sistema para contornar o caos resultante. Será?

Sds,

Victor.







##### ##### #####

Para saber mais visite
http://www.ciencialist.hpg.ig.com.br


##### ##### ##### #####



Yahoo! Grupos, um serviço oferecido por:
PUBLICIDADE




------------------------------------------------------------------------------
Links do Yahoo! Grupos

a.. Para visitar o site do seu grupo na web, acesse:
http://br.groups.yahoo.com/group/ciencialist/

b.. Para sair deste grupo, envie um e-mail para:
ciencialist-unsubscribe@yahoogrupos.com.br

c.. O uso que você faz do Yahoo! Grupos está sujeito aos Termos do Serviço do Yahoo!.



[As partes desta mensagem que não continham texto foram removidas]



SUBJECT: Fw: [ciencialist] Re: neutrino, existir or not be?
FROM: "JVictor" <jvoneto@uol.com.br>
TO: <ciencialist@yahoogrupos.com.br>
DATE: 10/01/2005 00:08

Ray,

Comento a seguir suas observações.

Victor.


Em ciencialist@yahoogrupos.com.br, "JVictor" <jvoneto@u...>
escreveu
> Caro Rayfisica,
>
>
> Minha confiança nas leis da natureza e, sobretudo, nas de
conservação, me dão a convicção inabalável de que o
neutrino existe
mesmo. Sua existência foi comprovada desde a década de 50.
Não há
dúvidas a respeito.

JVictor

Ray: Sua resposta é tão clara e precisa que não cabe
contraditório, alem
de que demonstra seu conhecimento.
E aproveitando para beber na fonte, pergunto que dizer da teoria de
que o neutrino pode violar a conservação da energia apresentada
por
alguns físicos para explicar a defasagem de neutrinos solares?

Victor: sem a menor dúvida garanto que há ainda algum aspecto do processo não compreendido, o pedaço que falta para equilibrar o balanço energético. Vamos ressuscitar Pauli que num instante as coisas se esclarecem. Mas propalar que a conservação da energia foi violada, que um mot-contínuo foi construído, que Einstein errou, que sua teoria é nada, dá ibope, atrai atenções e se fica conhecido. Mais cedo ou mais tarde o culpado pela quebra de simetria será encontrado!

Ray: Outra porque de vez em quando ainda entre os "
respeitáveis" surge a
possibilidade de queda da conservação da energia, por exemplo,
quando
surgiu o mago de Manchester com a coisa, uma das possibilidades
levantada por alguns especialistas seria um pequeno moto-continuo?

Victor: a resposta stá no Teorema de Noether, como comentei. Não há como fugir dele. Outra resposta seria: falta de conhecimento adequado, falta de compreensão das leis existentes, ou compreensão equivocada, ou sei lá o quê. Acho que Freud explica.

Ray: Caso não Houvesse a conservação da energia a ciência
física não
existiria?

Victor: Veja bem, Ray. No texto anterior não mencionei algumas coisas importantes e que são igualmente determinantes nessa questão de conservação. São elas: a muito bem estabelecida, pela observação e pelas medições, hipótese da homogeneidade do tempo e do espaço. A homogeneidade do tempo que dá, como consequência, uma lei de conservação: a da energia mecânica, que é válida tanto para sistemas fechados como para sistemas que se encontram em um campo constante, independente do tempo; a homogeneidade do espaço, que me garante que as propriedades mecânicas de um sistema fechado não mudam sob qualquer translação paralela. Isto dá, como consequência, a conservação de uma quantidade chamada quantidade de momento linear, ou quantidade de movimento linear, conforme já mencionamos; e a isotropia do espaço, que me garante que as propriedades mecânicas não variam qualquer que seja a rotação de todo o sistema, no espaço. Trata-se agora do momentum angular ou momento de impulso, que se conserva(para aprofundamentos a esse respeito, veja Mecânica, de Landau, Mec.Analítica, de Nivaldo A.Lemos ou Introdução à Teoria da Relatividade, de R.Resnick)). Se não houvesse homogneidade do tempo, num instante t1, teríamos F=ma e num instante t1 + dt teríamos F2 = ma(t) + f(t), ou outra relação, f(t) sendo alguma coisa introduzida por essa variação no tempo. Ou seja, a segunda lei de Newton, ou outra qualquer teria um jeito diferente a cada momento!. Se o espaço e o tempo não fossem homogênios, um comprimento ou um intervalo de tempo(para um mesmo evento) seriam diferentes, conforme fossem observados e medidos aqui ou acolá.
E, ainda mais, de acordo com essas hipóteses poderosas, homogeneidade (do tempo e espaço) e isotropia do espaço(que diz que o espaço tem as mesmas propriedades em todas as direções e nos dá a lei da conservação do momento angular), as relações matemáticas entre as medidas feitas por dois observadores inerciais são, obrigatoriamente lineares. Ou seja, as coordenadas espaciais e temporais de um sistema S e as coordenadas de outro sistema K, ambos inerciais, com velocidade relativa v entre eles, são necessariamente lineares. Sem perdão. Qualquer outro tipo de relação entre as coordenadas de um e de outro sistema que não seja linear viola as hipóteses da homogeneidade e isotropia, que sabemos serem corretas. Se assim não fosse, também não haveria leis de conservação ou simetrias. Assim, certamente, respondendo a tua pergunta, a física não existiria, no sentido de que as leis atuais de nada serviriam. Você teria de escrever um livro de física, com novas leis a cada instante, em cada casa ou apartamento.
Não é demais acrescentar que o espaço não privilegia posições no espaço ou direções. Todos têm o mesmo status. E isto se torna mais claro, patente, no espaço tempo quadri-mensional, onde, por exemplo, o momento e a energia se misturam para compor uma única entidade físico-matemática, que é uma coisa chamada tensor momento-energia, elemento fundamental da equação de Einstein, da RG. No espaço nosso, de meras (e chega!) 3 dimensões, o momento e a energia são entidades separadas, sem ligação direta. A relatividade me garante que a invariância de velocidade da luz aponta para outra simetria da natureza, simetria essa que costurou, inexorávelvelmente, o espaço e o tempo, transformando os dois numa só e única realidade, chamada espaçotempo, como disse seu descobridor, Minkowski.
Falei muito em simetrias da natureza, mas acho que não disse de sua real importância. Vou dizer agora. Quando mais simétrico for um sistema, menos variáveis serão necessárias para descrevê-lo. Imagine uma esfera. Do jeito que você olhar para ela, você verá sempre as mesmas coisas; pode fazer o quiser, olhar de baixo, de cima, rodá-lo, olhar através de um espelho, qualquer coisa. Verá sempre a mesma coisa. Nada muda. A esfera é o o objeto mais simples e mais simétrico que podemos mencionar, com uma característica: através de uma única variável, seu raio, a esfera é completamente descrita. As simetrias de que falei estão, assim, relacionadas ao espaço tempo. Mas há outras quantidades que se conservam, e que não têm relação com o espaçotempo, que ninguém é de ferro. Exemplo: a carga elétrica. Mas isto já é outra história. Há muito, muito o que falar e aprender a respeito.

Ray: A partir de onde não existiria Newton? Sobraria o
que?

Victor: Acho que os argumentos anteriores podem responder a isso, de maneira a não restarem dúvidas. Talvez o grande Newton inventasse um sistema para contornar o caos resultante. Será?

Sds,

Victor.







##### ##### #####

Para saber mais visite
http://www.ciencialist.hpg.ig.com.br


##### ##### ##### #####



Yahoo! Grupos, um serviço oferecido por:
PUBLICIDADE




------------------------------------------------------------------------------
Links do Yahoo! Grupos

a.. Para visitar o site do seu grupo na web, acesse:
http://br.groups.yahoo.com/group/ciencialist/

b.. Para sair deste grupo, envie um e-mail para:
ciencialist-unsubscribe@yahoogrupos.com.br

c.. O uso que você faz do Yahoo! Grupos está sujeito aos Termos do Serviço do Yahoo!.



[As partes desta mensagem que não continham texto foram removidas]



SUBJECT: Re: É o conhecimento perigoso?
FROM: Manuel Bulcão <manuelbulcao@uol.com.br>
TO: ciencialist@yahoogrupos.com.br
DATE: 10/01/2005 00:10


--- Em ciencialist@yahoogrupos.com.br, "Oraculo" <oraculo@a...>
escreveu
> Apesar da primeira parte levantar com precisão os atuais ataques
contra a ciência (que alguns chamam de cientificismo)...

Homero,

Existe ciência e existe cientificismo, consistindo este último
numa "ideologia" com verniz científico, ou seja, num sistema de
crenças pararreligioso (que engloba inclusive uma ética)
pretensamente fundamentado na ciência.

Para o cientificismo, o conhecimento científico não é só indicativo:
é também "imperativo" -- leis do ser (i. e., regularidades do mundo
natural que os cientistas paulatinamente desvendam mediante um
método rigoroso de investigação) são confundidas com "normas", ou
melhor, com leis do dever-ser. Vale dizer, os
cientificistas "valoram" as leis naturais, conferem-lhes uma
dimensão axiológica SEM QUE HAJA UMA RAZÃO PARA ISSO, de modo que
todos os vieses, tudo que não é regular ou que é produto de uma
vontade "arbitrária" deve ser considerado um erro, uma morbidez, uma
mentira, um epifania do MAL.

Outros dogmas do cientificismo: a) Não existe outra gnose (forma de
conhecimento) além da Ciência e Galileu é o seu profeta; b) O método
científico é omnipenetrante, capaz de desvendar todos os mistérios.
[Qualquer semelhança entre essa concepção da Ciência e Iavé ou Alá
talvez não seja mera coincidência.]

São variantes do cientificismo: os positivismos de Condorset e
Comte, o eugenismo de Galton, o social-darwinismo de Spencer & Cia,
o marxismo-leninismo, o nazismo, o "liberismo" (fundamentalismo do
Mercado) de F. Hayek e M. Friedmann, o transumanismo, etc.

Assim como se pode ser católico fervoroso sem ser membro da
integralista TFP, da franco-salazarista Opus Dei ou da facção
radical da Teologia da Libertação, pode-se ser um entusiasta do
pensamento científico sem ser um cientificista.

Por esta razão, um ataque ao cientificismo não é necessariamente uma
investida contra a ciência -- se bem que há filósofos idiotas, como,
por exemplo, o nazista M. Heidegger, que não conseguem discernir
essa diferença.

Abraços,
Manuel Bulcão





SUBJECT: Re: Ressonância de Shumann
FROM: "rmtakata" <rmtakata@altavista.net>
TO: ciencialist@yahoogrupos.com.br
DATE: 10/01/2005 03:24


Foi o Boff mesmo quem escreveu essa bobagem toda?

*Nao* eh verdade que a ressonancia de Schumann esteja aumentando de valor.

"The Schumann resonance frequency observed at this observatory does
not exhibit any unusual change or drift since the start of
observations by the BDSN in 1995."
http://quake.geo.berkeley.edu/ncedc/em.intro.html

A explicacao mais banal para a sensacao de diminuicao de tempo q.
muitas pessoas (mas nem de longe todas) sentem eh simplesmente em
funcao das modificacoes socioculturais. No Brasil e no mundo houve um
intenso processo de urbanizacao da populacao - e a vida nas cidades,
com muito mais pessoas ao redor, eh substancialmente mais agitada do
q. o ritmo rural.

Aumento da eficiencia nos servicos de telecomunicacoes - telefonia,
internet, televisao e outros - permite uma comunicacao muito mais
rapida: se antes levava meses para uma noticia cruzar o Atlantico,
isso eh feito hoje em milesimos de segundo. Hah muito mais dados
circulando no mundo hoje. Isso tb aumenta a percepcao de mudancas - e
aumentaria mesmo q. tais mudancas nao tivesse seu ritmo aumentado. Mas
como essa circulacao de dados tb interfere na taxa de alteracoes, em
muitos casos, efetivamente hah muito mais mudancas em um mesmo
intervalo de tempo (quem jah viu o sobe e desce das acoes em um pregao
eletronico da bolsa percebe isso quase ao vivo - em tempo real).

A mensagem abaixo eh paradoxal. Diz (falsamente) q, se alguem estiver
fora da frequencia de Schumann adoece; diz (tb falsamente) q. a
frequencia de Schumann estah aumentando e conclui de modo obtuso q
devemos diminuir o ritmo! Oras, se temos q. manter o passo com uma
frequencia q. aumenta, temos q. *aumentar* tb o ritmo, do contrario a
defasagem cresce ainda mais.

Nao existe nenhuma ligacao conhecida entre a frequencia de Schumann e
os ritmos biologicos, ou os ritmos naturais (por exemplo, os dias
estao ficando mais *longos* no decorrer dos seculos - pela
desaceleracao da rotacao da Terra pelos atritos de mare's.)

Nao existe uma frequencia unica para a RS, os 7 Hz eh um valor medio
de diversas frequencias.

Nao existe uma frequencia unica em nossos cerebros. Existem quatro
grupos principais de ondas eletricas detectadas em EEG: as deltas (de
1 a 3 Hz), as tetas (de 4 a 7 Hz), as alfas (de 8 a 13 Hz - media de
10 Hz), as betas (de 14 a 3o Hz). Qdo estamos acordados as frequencias
dominantes (de maior amplitude) sao as betas. As ondas alfas sao
detetadas em celulas isoladas ateh de invertebrados (e elas nao estao
ajustadas na frequencia Schumann). (O agrumento das ondas eh
relativamente arbitrario.)

http://dx.doi.org/10.1016/S0167-8760(97)00753-8

[]s,

Roberto Takata

-- Em ciencialist@yahoogrupos.com.br, "Cyberlander"
> Ressonância de Shumann
>
> Autor: Leonardo Boff
>
> Não apenas as pessoas mais idosas mas também jovens fazem a
> experiência de que tudo está se acelerando excessivamente.
> Ontem foi Carnaval, dentro de pouco será Páscoa, mais um
> pouco, Natal. Esse sentimento é ilusório ou tem base real?
>
> Pela ressonância Schumann se procura dar uma explicação. O
> físico alemão W.O. Schumann constatou em 1952 que a Terra é
> cercada por um campo eletromagnético poderoso que se forma
> entre o solo e a parte inferior da ionosfera, cerca de 100km
> acima de nós. Esse campo possui uma ressonância (dai
> chamar-se ressonância Schumann), mais ou menos constante, da
> ordem de 7,83 pulsações por segundo.
>
> Funciona como uma espécie de marca-passo, responsável pelo
> equilíbrio da biosfera, condição comum de todas as formas de
> vida. Verificou-se também que todos os vertebrados e o nosso
> cérebro são dotados da mesma frequência de 7,83 hertz.
>
> Empiricamente fez-se a constatação de que não podemos ser
> saudáveis fora dessa frequência biológica natural. Sempre
> que os astronautas, em razão das viagens espaciais, ficavam
> fora da ressonância Schumann, adoeciam. Mas submetidos à
> ação de um simulador Schumann recuperavam o equilíbrio e a
> saúde. Por milhares de anos as batidas do coração da Terra
> tinham essa freqüência de pulsações e a vida se desenrolava
> em relativo equilíbrio ecológico. Ocorre que a partir dos
> anos 80, e de forma mais acentuada a partir dos anos 90, a
> freqüência passou de 7,83para 11 e para 13 hertz.
>
> O coração da Terra disparou. Coincidentemente,
> desequilíbrios ecológicos se fizeram sentir: perturbações
> climáticas, maior atividade dos vulcões, crescimento de
> tensões e conflitos no mundo e aumento geral de
> comportamentos desviantes nas pessoas, entre outros. Devido
> à aceleração geral, a jornada de 24 horas, na verdade, é
> somente de 16 horas. Portanto, a percepção de que tudo está
> passando rápido demais não é ilusória,mas teria base real
> nesse transtorno da ressonância Schumann.
>
> Gaia, esse superorganismo vivo que é a Mãe Terra, deverá
> estar buscando formas de retornar a seu equilíbrio natural.
> E vaiconsegui-lo, mas não sabemos a que preço, a ser pago
> pela biosfera e pelos seres humanos. Aqui abre-se o espaço
> para grupos esotéricos e outros futuristas projetarem
> cenários, ora dramáticos, com catástrofes terríveis, ora
> esperançadores, como a irrupção da quarta dimensão, pela
> qual todos seremos mais intuitivos, mais espirituais e mais
> sintonizados com o biorritmo da Terra.
>
> Não pretendo reforçar esse tipo de leitura. Apenas
> enfatizo a tese recorrente entre grandes cosmólogos e
> biólogos de que a Terra é, efetivamente, um superorganismo
> vivo, de que Terra e humanidade formamos uma única entidade,
> como os astronautas testemunham de suas naves espaciais.
> Nós, seres humanos, somos Terra que sente, pensa, ama e
> venera. Porque somos isso, possuímos a mesma natureza
> bioelétrica e estamos envoltos pelas mesmas ondas
> ressonantes Schumann.
>
> Se queremos que a Terra reencontre seu equilíbrio, devemos
> começar por nós mesmos: fazer tudo sem estresse, com mais
> serenidade, com mais harmonia, com mais amor, que é uma
> energia essencialmente harmonizadora. Para isso importa
> termos coragem de ser anticultura dominante, que nos obriga
> a ser cada vez mais competitivos e efetivos. Precisamos
> respirar juntos com a Terra, para conspirar com ela pela
> paz.






SUBJECT: Fw: [ciencialist] Re: neutrino, existir or not be?
FROM: "rayfisica" <rayfisica@yahoo.com.br>
TO: ciencialist@yahoogrupos.com.br
DATE: 10/01/2005 08:43


--- Em ciencialist@yahoogrupos.com.br, "JVictor" <jvoneto@u...>
escreveu
> Ray,
>
> Comento a seguir suas observações.
>
> Victor.
>
>

>
>
>
Penso que foi Stephen Hawking que disse, e se não disse perdeu uma
ótima oportunidade de dizer, quando escreveu o UMA BREVE HISTORIA
DO
TEMPO que quem realmente conhece um assunto sabe demonstrar para o
vulgo (algo assim).
Porem aproveitando a oportunidade de aprender, eu pergunto, e as
outras dimensões?
EXE. Dizem que se a terra fosse oca não haveria gravidade no
interior, sendo portanto uma dimensão extra (acho) pelo ao menos
no
tangente a gravidade, portanto poderia ser muito mais fácil ir do
Brasil ao Japão pelo interior, do que viajar a mesma distancia
pelo
exterior (espero ter sido claro) como ficaria a simetria nesse caso?
Desculpe estou com dificuldade em colocar a questão sem parecer
idiota, faça um esforço.





SUBJECT: Re: [ciencialist] Re: Ressonância de Shumann
FROM: "Norberto Kawakami" <kawakami@ig.com.br>
TO: <ciencialist@yahoogrupos.com.br>
DATE: 10/01/2005 09:21


Takata: A explicacao mais banal para a sensacao de diminuicao de tempo q.
muitas pessoas (mas nem de longe todas) sentem eh simplesmente em
funcao das modificacoes socioculturais.

Norberto:
Complementando este comentário, pode-se ler o artigo
http://obeco.planetaclix.pt/rkurz29.htm sobre a aceleração do tempo
cultural...

[]s



SUBJECT: Posto Petrobrás de hidrogênio
FROM: José Renato <jrma@terra.com.br>
TO: <ciencialist@yahoogrupos.com.br>
DATE: 10/01/2005 10:03

AMBIENTE BRASIL 09/01/2005
Petrobrás terá posto de abastecimento para ônibus movido a hidrogênio

O Cenpes - Centro de Pesquisas da Petrobrás começa a construir, nos próximos meses, um posto de abastecimento de combustíveis alternativos. O centro produzirá hidrogênio a partir de gás natural ou de água, por eletrólise, para abastecer o protótipo do ônibus híbrido brasileiro.

O veículo está sendo desenvolvido pela Coordenação de Pós-Graduação de Engenharia da Universidade Federal do Rio de Janeiro (Coppe/UFRJ) em parceria com a indústria nacional. O projeto tem apoio financeiro da Petrobrás e da Finep - Financiadora de Estudos e Projetos, no total de R$ 3 milhões.

O coordenador técnico da Gerência de Gás e Energia do Cenpes, responsável pela concepção do posto, Newton Reis de Moura, disse nesta sexta-feira (7) à Agência Brasil que o projeto está em fase de planejamento e aprovação do design. Para ele, a iniciativa coloca a Petrobrás no mesmo nível das demais empresas de energia, como a Shell.

O posto deve funcionar ainda neste ano ou, no máximo, até o início de 2006, bem antes da conclusão do protótipo do ônibus movido a hidrogênio. A Coppe prevê que o ônibus esteja trafegando na Ilha do Fundão no segundo semestre de 2006.

Segundo a coordenadora de Projetos de Hidrogênio e Célula Combustível do Cenpes, Maria Helena Troise Frank, o desenvolvimento desse primeiro projeto, com tecnologia nacional, tem importância essencial para o Brasil. Para ela, a iniciativa abre possibilidade de mercado atraente para o país, quando o hidrogênio for o combustível adotado no setor automotivo mundial. Ela lembrou que o projeto terá aplicabilidade comercial a médio e longo prazos, daí a importância de ser desenvolvido neste momento.

Para o gerente interino de Gás e Energia do Cenpes, Marco Antonio Haikal Leite, outro fator importante do projeto é a parte tecnológica. "Os ônibus a hidrogênio apresentam uma tecnologia emergente que vários países estão pesquisando e são excelentes do ponto de vista ambiental. Você os alimenta com hidrogênio e, em vez de sair gás carbônico ou outros poluentes na atmosfera, sai apenas água".

Haikal afirmou que isso vai contribuir para melhorar a qualidade de vida das cidades. "A Petrobrás está atenta a isso e procurando sempre melhorar o ponto de vista ambiental". Para Maria Helena, a estatal está buscando soluções tecnologicamente inovadoras que sejam ambientalmente amigáveis, visando um futuro de energia sustentável.

O consultor Newton Reis de Moura disse que o posto de abastecimento terá caráter eco-tecnológico, com preocupação com o meio ambiente e o social. Ali serão testados, além do hidrogênio, outros combustíveis alternativos, entre os quais biodiesel e gás natural, e novas tecnologias de abastecimento para o país. "Esse posto terá um enfoque muito forte na parte de gás natural", acrescentou.

Moura revelou que há planos de fazer uma mistura de hidrogênio com gás natural, para inserir gradualmente o hidrogênio na matriz energética, de modo a abastecer veículos movidos a gás natural, sem necessidade de alterar os motores. Ele lembrou que, além da sustentabilidade, espera-se que o posto seja auto-suficiente em termos de energia, já que terá equipamentos de geração energética, com tecnologias inovadoras e não apenas as disponíveis no mercado. O Cenpes estima que, com a definição do lay-out, o posto leve seis meses para ser construído. (Alana Gandra / Agência Brasil)


< http://www.ambientebrasil.com.br/noticias/index.php3?action=ler&id=17560 >

[As partes desta mensagem que não continham texto foram removidas]



SUBJECT: Re: Ressonância de Shumann
FROM: "brudna" <lrb@iq.ufrgs.br>
TO: ciencialist@yahoogrupos.com.br
DATE: 10/01/2005 12:21


Takata, parabéns pela argumentação. Também duvido que o Boff
escreveu o texto.

Esse texto do Boof(?) me faz lembrar do filme do Super Homem, no
qual ele faz a Terra rodar em sentido contrário para o tempo voltar.
Hahahahaha

Daria até para lapidar o seu texto e deixar pela internet para
´sacar´ quando esse texto do Boff reaparecer. :-)

Até
Luís Brudna

--- Em ciencialist@yahoogrupos.com.br, "rmtakata" <rmtakata@a...> escreveu
>
> Foi o Boff mesmo quem escreveu essa bobagem toda?
>
> *Nao* eh verdade que a ressonancia de Schumann esteja aumentando de
valor.
>
> "The Schumann resonance frequency observed at this observatory does
> not exhibit any unusual change or drift since the start of
> observations by the BDSN in 1995."
> http://quake.geo.berkeley.edu/ncedc/em.intro.html
>
> A explicacao mais banal para a sensacao de diminuicao de tempo q.
> muitas pessoas (mas nem de longe todas) sentem eh simplesmente em
> funcao das modificacoes socioculturais. No Brasil e no mundo houve um
> intenso processo de urbanizacao da populacao - e a vida nas cidades,
> com muito mais pessoas ao redor, eh substancialmente mais agitada do
> q. o ritmo rural.
>
> Aumento da eficiencia nos servicos de telecomunicacoes - telefonia,
> internet, televisao e outros - permite uma comunicacao muito mais
> rapida: se antes levava meses para uma noticia cruzar o Atlantico,
> isso eh feito hoje em milesimos de segundo. Hah muito mais dados
> circulando no mundo hoje. Isso tb aumenta a percepcao de mudancas - e
> aumentaria mesmo q. tais mudancas nao tivesse seu ritmo aumentado. Mas
> como essa circulacao de dados tb interfere na taxa de alteracoes, em
> muitos casos, efetivamente hah muito mais mudancas em um mesmo
> intervalo de tempo (quem jah viu o sobe e desce das acoes em um pregao
> eletronico da bolsa percebe isso quase ao vivo - em tempo real).
>
> A mensagem abaixo eh paradoxal. Diz (falsamente) q, se alguem estiver
> fora da frequencia de Schumann adoece; diz (tb falsamente) q. a
> frequencia de Schumann estah aumentando e conclui de modo obtuso q
> devemos diminuir o ritmo! Oras, se temos q. manter o passo com uma
> frequencia q. aumenta, temos q. *aumentar* tb o ritmo, do contrario a
> defasagem cresce ainda mais.
>
> Nao existe nenhuma ligacao conhecida entre a frequencia de Schumann e
> os ritmos biologicos, ou os ritmos naturais (por exemplo, os dias
> estao ficando mais *longos* no decorrer dos seculos - pela
> desaceleracao da rotacao da Terra pelos atritos de mare's.)
>
> Nao existe uma frequencia unica para a RS, os 7 Hz eh um valor medio
> de diversas frequencias.
>
> Nao existe uma frequencia unica em nossos cerebros. Existem quatro
> grupos principais de ondas eletricas detectadas em EEG: as deltas (de
> 1 a 3 Hz), as tetas (de 4 a 7 Hz), as alfas (de 8 a 13 Hz - media de
> 10 Hz), as betas (de 14 a 3o Hz). Qdo estamos acordados as frequencias
> dominantes (de maior amplitude) sao as betas. As ondas alfas sao
> detetadas em celulas isoladas ateh de invertebrados (e elas nao estao
> ajustadas na frequencia Schumann). (O agrumento das ondas eh
> relativamente arbitrario.)
>
> http://dx.doi.org/10.1016/S0167-8760(97)00753-8
>
> []s,
>
> Roberto Takata
>
> -- Em ciencialist@yahoogrupos.com.br, "Cyberlander"
> > Ressonância de Shumann
> >
> > Autor: Leonardo Boff
> >
> > Não apenas as pessoas mais idosas mas também jovens fazem a
> > experiência de que tudo está se acelerando excessivamente.
> > Ontem foi Carnaval, dentro de pouco será Páscoa, mais um
> > pouco, Natal. Esse sentimento é ilusório ou tem base real?
> >
> > Pela ressonância Schumann se procura dar uma explicação. O
> > físico alemão W.O. Schumann constatou em 1952 que a Terra é
> > cercada por um campo eletromagnético poderoso que se forma
> > entre o solo e a parte inferior da ionosfera, cerca de 100km
> > acima de nós. Esse campo possui uma ressonância (dai
> > chamar-se ressonância Schumann), mais ou menos constante, da
> > ordem de 7,83 pulsações por segundo.
> >
> > Funciona como uma espécie de marca-passo, responsável pelo
> > equilíbrio da biosfera, condição comum de todas as formas de
> > vida. Verificou-se também que todos os vertebrados e o nosso
> > cérebro são dotados da mesma frequência de 7,83 hertz.
> >
> > Empiricamente fez-se a constatação de que não podemos ser
> > saudáveis fora dessa frequência biológica natural. Sempre
> > que os astronautas, em razão das viagens espaciais, ficavam
> > fora da ressonância Schumann, adoeciam. Mas submetidos à
> > ação de um simulador Schumann recuperavam o equilíbrio e a
> > saúde. Por milhares de anos as batidas do coração da Terra
> > tinham essa freqüência de pulsações e a vida se desenrolava
> > em relativo equilíbrio ecológico. Ocorre que a partir dos
> > anos 80, e de forma mais acentuada a partir dos anos 90, a
> > freqüência passou de 7,83para 11 e para 13 hertz.
> >
> > O coração da Terra disparou. Coincidentemente,
> > desequilíbrios ecológicos se fizeram sentir: perturbações
> > climáticas, maior atividade dos vulcões, crescimento de
> > tensões e conflitos no mundo e aumento geral de
> > comportamentos desviantes nas pessoas, entre outros. Devido
> > à aceleração geral, a jornada de 24 horas, na verdade, é
> > somente de 16 horas. Portanto, a percepção de que tudo está
> > passando rápido demais não é ilusória,mas teria base real
> > nesse transtorno da ressonância Schumann.
> >
> > Gaia, esse superorganismo vivo que é a Mãe Terra, deverá
> > estar buscando formas de retornar a seu equilíbrio natural.
> > E vaiconsegui-lo, mas não sabemos a que preço, a ser pago
> > pela biosfera e pelos seres humanos. Aqui abre-se o espaço
> > para grupos esotéricos e outros futuristas projetarem
> > cenários, ora dramáticos, com catástrofes terríveis, ora
> > esperançadores, como a irrupção da quarta dimensão, pela
> > qual todos seremos mais intuitivos, mais espirituais e mais
> > sintonizados com o biorritmo da Terra.
> >
> > Não pretendo reforçar esse tipo de leitura. Apenas
> > enfatizo a tese recorrente entre grandes cosmólogos e
> > biólogos de que a Terra é, efetivamente, um superorganismo
> > vivo, de que Terra e humanidade formamos uma única entidade,
> > como os astronautas testemunham de suas naves espaciais.
> > Nós, seres humanos, somos Terra que sente, pensa, ama e
> > venera. Porque somos isso, possuímos a mesma natureza
> > bioelétrica e estamos envoltos pelas mesmas ondas
> > ressonantes Schumann.
> >
> > Se queremos que a Terra reencontre seu equilíbrio, devemos
> > começar por nós mesmos: fazer tudo sem estresse, com mais
> > serenidade, com mais harmonia, com mais amor, que é uma
> > energia essencialmente harmonizadora. Para isso importa
> > termos coragem de ser anticultura dominante, que nos obriga
> > a ser cada vez mais competitivos e efetivos. Precisamos
> > respirar juntos com a Terra, para conspirar com ela pela
> > paz.





SUBJECT: Células-tronco eliminam sintomas de Parkinson em macacos
FROM: "Amauri Jr" <amaurijunior2@yahoo.com.br>
TO: "Lucimary Vargas" <sangalli@uai.com.br>, <Sexualidadedodef@yahoogrupos.com.br>, "kathleen_mel" <kathleen_mel@uol.com.br>, <deficienteeficiente@yahoogrupos.com.br>, <Conversa_de_Botequim@yahoogrupos.com.br>, <ciencialist@yahoogrupos.com.br>, <autoiniciacaodaconsciencia_plena@yahoogrupos.com.br>, <acropolis@yahoogrupos.com.br>
DATE: 10/01/2005 13:25

05/01/2005 - 12h29
C�lulas-tronco eliminam sintomas de Parkinson em macacos
da Folha Online
da Reuters

C�lulas-tronco retiradas de embri�es de macacos e implantadas no c�rebro conseguiram reverter alguns dos sintomas do mal de Parkinson de s�mios que participaram de um estudo sobre a doen�a.

Pesquisadores japoneses argumentam, em estudo publicado no "Journal of Clinical Investigation", que as c�lulas-tronco podem ser usadas para substituir tecidos danificados por uma s�rie de enfermidades.

Mas eles tamb�m advertem que � necess�rio mais acompanhamento para comprovar a efic�cia do tratamento.

Yasushi Takagi e sua equipe na Universidade de Kyoto obtiveram c�lulas-tronco a partir de embri�es de macacos induziram-nas a transformarem-se em neur�nios. Eles ent�o transplantaram essas c�lulas para o c�rebro de s�mios que desenvolveram uma doen�a semelhante ao mal de Parkinson depois de tratamento qu�mico.

Os sintomas da doen�a foram reduzidos e, quando as cobaias foram mortas, as c�lulas transplantadas tinham crescido em seus c�rebros.

O mal de Parkinson ocorre quando c�lulas importantes do c�rebro que produzem dopamina --qu�mico respons�vel basicamente pelo transporte de mensagens-- se degeneram e morrem. N�o h� cura para a doen�a.

O transplante de c�lulas tem sido a grande esperan�a dos cientistas, e muitos grupos j� tentaram transplantar c�lulas do c�rebro em pacientes, inclusive c�lulas de fetos de porcos.

Quem defende a pesquisa com c�lulas-tronco acredita que seu pr�prio campo de trabalho oferece uma boa oportunidade, j� que essas c�lulas-curinga t�m a habilidade de se transformar em uma s�rie de tecidos do corpo sem causar rea��es imunol�gicas.

Mas quem se op�e � pr�tica acredita ser anti�tico usar embri�es humanos para esse tipo de pesquisa ou tratamento. A legisla��o brasileira atualmente pro�be a cria��o de embri�es para a obten��o de c�lulas-tronco.

William Langston, do Instituto de Parkinson, na Calif�rnia (EUA), acredita que a descoberta seja um marco, mas relatou limita��es em rela��o � sobreviv�ncia das c�lulas criadas pela equipe de cientistas japoneses.

De qualquer modo, o estudo "vai adiantar as pesquisas com o objetivo de validar o uso de c�lulas-tronco para tratar doen�as neurodegenerativas", disse Langston em um coment�rio.

[As partes desta mensagem que n�o continham texto foram removidas]



SUBJECT: Re: [ciencialist] Re: É o conhecimento perigoso? p/ Manuel
FROM: "Oraculo" <oraculo@atibaia.com.br>
TO: <ciencialist@yahoogrupos.com.br>
DATE: 10/01/2005 15:19

Olá Manuel

Obrigado pela detalhada explicação. Realmente, eu estava usando no termo no ajuste que me foi apresentado, como sendo, a admiração pela ciência e sua confiabilidade (do ponto de vista do universo físicco), uma atitude cientificista. Seria uma espécie de cegueira ao "não perceber o relativismo do conhecimento cientifico"..:-)

Mas no enfoque que você me apresentou, é outra coisa. Inclusive já debati nesses exatos termos, acho que na lista Ceticismo Aberto, quando uma discussão sobre ética sem deuses descambou para o "natural é bom, artificial é ruim"..:-) Inclusive com os mesmos exemplos, eugenia, darwinsmo social, etc, (apesar de não tão bem descritos como em sua exposição..:-)

Conceitos como justiça ou gratidão não são naturais, não existem na natureza e entretanto, são excelentes e importantes criações humanas..:-) Confundir natural com correto é algo sempre presente (até nos derivados do movimetno hippie, como alimentos "naturais" e afins..:-)

Outro ponto que você apresenta com clareza é um que costuma me incomodar em praticamente TODA discussão sobre a eficácia da ciência: a acusação de que a ciência pretende "saber tudo sobre tudo e com 100% de certeza". Uma acusação totalmente falsa, mas uma visão da ciência muito difundida entre leigos..:-) É o caso de seu DOGMA B..:-)

Existe uma diferença entre avaliarmos dois saberes em termos de confiabilidade e aproximação com a realidade, e a certeza de 100% que se pretende ser objeto da ciência (e que fica melhor nas religiões..:-). O status da Lua, em termos de universo físico, pode ser mensurado como um satélite rochoso a orbitar a Terra, em perfeito acordo com as leis do movimento e relatividade ou ser uma cabaça iluminada a poucos metros acima da copa das árvores. Mas isso não significa mais nada, além do grau de confiabilidade do status real da Lua..:-)

Assim, no sentido de apropriação do conhecimento cientifico como verdade absoluta (100% de acerto..:-), o cientificismo é realmente apenas uma deformacào da idéia de ciência. Mas é mais comum em leigos que em cientistas, já que a idéia central da ciência é a dúvida não a certeza..:-)

Um abraço.

Homero





----- Original Message -----
From: Manuel Bulcão
To: ciencialist@yahoogrupos.com.br
Sent: Monday, January 10, 2005 12:10 AM
Subject: [ciencialist] Re: É o conhecimento perigoso?



--- Em ciencialist@yahoogrupos.com.br, "Oraculo" <oraculo@a...>
escreveu
> Apesar da primeira parte levantar com precisão os atuais ataques
contra a ciência (que alguns chamam de cientificismo)...

Homero,

Existe ciência e existe cientificismo, consistindo este último
numa "ideologia" com verniz científico, ou seja, num sistema de
crenças pararreligioso (que engloba inclusive uma ética)
pretensamente fundamentado na ciência.

Para o cientificismo, o conhecimento científico não é só indicativo:
é também "imperativo" -- leis do ser (i. e., regularidades do mundo
natural que os cientistas paulatinamente desvendam mediante um
método rigoroso de investigação) são confundidas com "normas", ou
melhor, com leis do dever-ser. Vale dizer, os
cientificistas "valoram" as leis naturais, conferem-lhes uma
dimensão axiológica SEM QUE HAJA UMA RAZÃO PARA ISSO, de modo que
todos os vieses, tudo que não é regular ou que é produto de uma
vontade "arbitrária" deve ser considerado um erro, uma morbidez, uma
mentira, um epifania do MAL.

Outros dogmas do cientificismo: a) Não existe outra gnose (forma de
conhecimento) além da Ciência e Galileu é o seu profeta; b) O método
científico é omnipenetrante, capaz de desvendar todos os mistérios.
[Qualquer semelhança entre essa concepção da Ciência e Iavé ou Alá
talvez não seja mera coincidência.]

São variantes do cientificismo: os positivismos de Condorset e
Comte, o eugenismo de Galton, o social-darwinismo de Spencer & Cia,
o marxismo-leninismo, o nazismo, o "liberismo" (fundamentalismo do
Mercado) de F. Hayek e M. Friedmann, o transumanismo, etc.

Assim como se pode ser católico fervoroso sem ser membro da
integralista TFP, da franco-salazarista Opus Dei ou da facção
radical da Teologia da Libertação, pode-se ser um entusiasta do
pensamento científico sem ser um cientificista.

Por esta razão, um ataque ao cientificismo não é necessariamente uma
investida contra a ciência -- se bem que há filósofos idiotas, como,
por exemplo, o nazista M. Heidegger, que não conseguem discernir
essa diferença.

Abraços,
Manuel Bulcão





##### ##### #####

Para saber mais visite
http://www.ciencialist.hpg.ig.com.br


##### ##### ##### #####


Yahoo! Grupos, um serviço oferecido por:
PUBLICIDADE




------------------------------------------------------------------------------
Links do Yahoo! Grupos

a.. Para visitar o site do seu grupo na web, acesse:
http://br.groups.yahoo.com/group/ciencialist/

b.. Para sair deste grupo, envie um e-mail para:
ciencialist-unsubscribe@yahoogrupos.com.br

c.. O uso que você faz do Yahoo! Grupos está sujeito aos Termos do Serviço do Yahoo!.



[As partes desta mensagem que não continham texto foram removidas]



SUBJECT: Ressonância de Shumann p/ Brudna
FROM: "Oraculo" <oraculo@atibaia.com.br>
TO: <ciencialist@yahoogrupos.com.br>
DATE: 10/01/2005 15:28

Olá Brudna

Se você gostou do Super-Homem que gira a Tera ao contrário e faz o tempo voltar, vai gostar dessa..:-) Em uma discussão, no forum da STR, um participante argumentou que "existe tempo aqui, porque a Terra gira em torno do Sol e por isso temos dias e noites, e marcando o tempo. Se o universo não gira, não há tempo fora daqui"..:-)

É uma pérola da imaginação humana e da capacidade de falar tolices, não?..:-)

Se se interessar pelo resto da argumentação (é de um criacionista) eu te mando o link do debate..:-)

Um abraço.

Homero
----- Original Message -----
From: brudna
To: ciencialist@yahoogrupos.com.br
Sent: Monday, January 10, 2005 12:21 PM
Subject: [ciencialist] Re: Ressonância de Shumann



Takata, parabéns pela argumentação. Também duvido que o Boff
escreveu o texto.

Esse texto do Boof(?) me faz lembrar do filme do Super Homem, no
qual ele faz a Terra rodar em sentido contrário para o tempo voltar.
Hahahahaha

Daria até para lapidar o seu texto e deixar pela internet para
´sacar´ quando esse texto do Boff reaparecer. :-)

Até
Luís Brudna

--- Em ciencialist@yahoogrupos.com.br, "rmtakata" <rmtakata@a...> escreveu
>
> Foi o Boff mesmo quem escreveu essa bobagem toda?
>
> *Nao* eh verdade que a ressonancia de Schumann esteja aumentando de
valor.
>
> "The Schumann resonance frequency observed at this observatory does
> not exhibit any unusual change or drift since the start of
> observations by the BDSN in 1995."
> http://quake.geo.berkeley.edu/ncedc/em.intro.html
>
> A explicacao mais banal para a sensacao de diminuicao de tempo q.
> muitas pessoas (mas nem de longe todas) sentem eh simplesmente em
> funcao das modificacoes socioculturais. No Brasil e no mundo houve um
> intenso processo de urbanizacao da populacao - e a vida nas cidades,
> com muito mais pessoas ao redor, eh substancialmente mais agitada do
> q. o ritmo rural.
>
> Aumento da eficiencia nos servicos de telecomunicacoes - telefonia,
> internet, televisao e outros - permite uma comunicacao muito mais
> rapida: se antes levava meses para uma noticia cruzar o Atlantico,
> isso eh feito hoje em milesimos de segundo. Hah muito mais dados
> circulando no mundo hoje. Isso tb aumenta a percepcao de mudancas - e
> aumentaria mesmo q. tais mudancas nao tivesse seu ritmo aumentado. Mas
> como essa circulacao de dados tb interfere na taxa de alteracoes, em
> muitos casos, efetivamente hah muito mais mudancas em um mesmo
> intervalo de tempo (quem jah viu o sobe e desce das acoes em um pregao
> eletronico da bolsa percebe isso quase ao vivo - em tempo real).
>
> A mensagem abaixo eh paradoxal. Diz (falsamente) q, se alguem estiver
> fora da frequencia de Schumann adoece; diz (tb falsamente) q. a
> frequencia de Schumann estah aumentando e conclui de modo obtuso q
> devemos diminuir o ritmo! Oras, se temos q. manter o passo com uma
> frequencia q. aumenta, temos q. *aumentar* tb o ritmo, do contrario a
> defasagem cresce ainda mais.
>
> Nao existe nenhuma ligacao conhecida entre a frequencia de Schumann e
> os ritmos biologicos, ou os ritmos naturais (por exemplo, os dias
> estao ficando mais *longos* no decorrer dos seculos - pela
> desaceleracao da rotacao da Terra pelos atritos de mare's.)
>
> Nao existe uma frequencia unica para a RS, os 7 Hz eh um valor medio
> de diversas frequencias.
>
> Nao existe uma frequencia unica em nossos cerebros. Existem quatro
> grupos principais de ondas eletricas detectadas em EEG: as deltas (de
> 1 a 3 Hz), as tetas (de 4 a 7 Hz), as alfas (de 8 a 13 Hz - media de
> 10 Hz), as betas (de 14 a 3o Hz). Qdo estamos acordados as frequencias
> dominantes (de maior amplitude) sao as betas. As ondas alfas sao
> detetadas em celulas isoladas ateh de invertebrados (e elas nao estao
> ajustadas na frequencia Schumann). (O agrumento das ondas eh
> relativamente arbitrario.)
>
> http://dx.doi.org/10.1016/S0167-8760(97)00753-8
>
> []s,
>
> Roberto Takata
>
> -- Em ciencialist@yahoogrupos.com.br, "Cyberlander"
> > Ressonância de Shumann
> >
> > Autor: Leonardo Boff
> >
> > Não apenas as pessoas mais idosas mas também jovens fazem a
> > experiência de que tudo está se acelerando excessivamente.
> > Ontem foi Carnaval, dentro de pouco será Páscoa, mais um
> > pouco, Natal. Esse sentimento é ilusório ou tem base real?
> >
> > Pela ressonância Schumann se procura dar uma explicação. O
> > físico alemão W.O. Schumann constatou em 1952 que a Terra é
> > cercada por um campo eletromagnético poderoso que se forma
> > entre o solo e a parte inferior da ionosfera, cerca de 100km
> > acima de nós. Esse campo possui uma ressonância (dai
> > chamar-se ressonância Schumann), mais ou menos constante, da
> > ordem de 7,83 pulsações por segundo.
> >
> > Funciona como uma espécie de marca-passo, responsável pelo
> > equilíbrio da biosfera, condição comum de todas as formas de
> > vida. Verificou-se também que todos os vertebrados e o nosso
> > cérebro são dotados da mesma frequência de 7,83 hertz.
> >
> > Empiricamente fez-se a constatação de que não podemos ser
> > saudáveis fora dessa frequência biológica natural. Sempre
> > que os astronautas, em razão das viagens espaciais, ficavam
> > fora da ressonância Schumann, adoeciam. Mas submetidos à
> > ação de um simulador Schumann recuperavam o equilíbrio e a
> > saúde. Por milhares de anos as batidas do coração da Terra
> > tinham essa freqüência de pulsações e a vida se desenrolava
> > em relativo equilíbrio ecológico. Ocorre que a partir dos
> > anos 80, e de forma mais acentuada a partir dos anos 90, a
> > freqüência passou de 7,83para 11 e para 13 hertz.
> >
> > O coração da Terra disparou. Coincidentemente,
> > desequilíbrios ecológicos se fizeram sentir: perturbações
> > climáticas, maior atividade dos vulcões, crescimento de
> > tensões e conflitos no mundo e aumento geral de
> > comportamentos desviantes nas pessoas, entre outros. Devido
> > à aceleração geral, a jornada de 24 horas, na verdade, é
> > somente de 16 horas. Portanto, a percepção de que tudo está
> > passando rápido demais não é ilusória,mas teria base real
> > nesse transtorno da ressonância Schumann.
> >
> > Gaia, esse superorganismo vivo que é a Mãe Terra, deverá
> > estar buscando formas de retornar a seu equilíbrio natural.
> > E vaiconsegui-lo, mas não sabemos a que preço, a ser pago
> > pela biosfera e pelos seres humanos. Aqui abre-se o espaço
> > para grupos esotéricos e outros futuristas projetarem
> > cenários, ora dramáticos, com catástrofes terríveis, ora
> > esperançadores, como a irrupção da quarta dimensão, pela
> > qual todos seremos mais intuitivos, mais espirituais e mais
> > sintonizados com o biorritmo da Terra.
> >
> > Não pretendo reforçar esse tipo de leitura. Apenas
> > enfatizo a tese recorrente entre grandes cosmólogos e
> > biólogos de que a Terra é, efetivamente, um superorganismo
> > vivo, de que Terra e humanidade formamos uma única entidade,
> > como os astronautas testemunham de suas naves espaciais.
> > Nós, seres humanos, somos Terra que sente, pensa, ama e
> > venera. Porque somos isso, possuímos a mesma natureza
> > bioelétrica e estamos envoltos pelas mesmas ondas
> > ressonantes Schumann.
> >
> > Se queremos que a Terra reencontre seu equilíbrio, devemos
> > começar por nós mesmos: fazer tudo sem estresse, com mais
> > serenidade, com mais harmonia, com mais amor, que é uma
> > energia essencialmente harmonizadora. Para isso importa
> > termos coragem de ser anticultura dominante, que nos obriga
> > a ser cada vez mais competitivos e efetivos. Precisamos
> > respirar juntos com a Terra, para conspirar com ela pela
> > paz.





##### ##### #####

Para saber mais visite
http://www.ciencialist.hpg.ig.com.br


##### ##### ##### #####


Yahoo! Grupos, um serviço oferecido por:







------------------------------------------------------------------------------
Links do Yahoo! Grupos

a.. Para visitar o site do seu grupo na web, acesse:
http://br.groups.yahoo.com/group/ciencialist/

b.. Para sair deste grupo, envie um e-mail para:
ciencialist-unsubscribe@yahoogrupos.com.br

c.. O uso que você faz do Yahoo! Grupos está sujeito aos Termos do Serviço do Yahoo!.



[As partes desta mensagem que não continham texto foram removidas]



SUBJECT: É o conhecimento perigoso? p/ Manuel - 2
FROM: "Oraculo" <oraculo@atibaia.com.br>
TO: <ciencialist@yahoogrupos.com.br>
DATE: 10/01/2005 15:36

Olá Manuel

Relendo minha mensagem anterior, achei que este trecho poderia ser mal interpretado:

"Uma acusação totalmente falsa, mas uma visão da ciência muito difundida entre leigos..:-) É o caso de seu DOGMA B..:-) "

Não significa, claro, que você é um leigo ou que pensa da forma como exposto..:-) A frase deveria ser : "é o que apresenta no dogma B". E você o faz de modo crítico e correto.

Um abraço.

Homero

PS: É preciso cuidado ao expor idéias e frases em textos escrito em listas de discussão, a chance de ser pouco claro e confundir a sentido é sempre grande..:-)

----- Original Message -----
From: Oraculo
To: ciencialist@yahoogrupos.com.br
Sent: Monday, January 10, 2005 3:19 PM
Subject: Re: [ciencialist] Re: É o conhecimento perigoso? p/ Manuel


Olá Manuel

Obrigado pela detalhada explicação. Realmente, eu estava usando no termo no ajuste que me foi apresentado, como sendo, a admiração pela ciência e sua confiabilidade (do ponto de vista do universo físicco), uma atitude cientificista. Seria uma espécie de cegueira ao "não perceber o relativismo do conhecimento cientifico"..:-)

Mas no enfoque que você me apresentou, é outra coisa. Inclusive já debati nesses exatos termos, acho que na lista Ceticismo Aberto, quando uma discussão sobre ética sem deuses descambou para o "natural é bom, artificial é ruim"..:-) Inclusive com os mesmos exemplos, eugenia, darwinsmo social, etc, (apesar de não tão bem descritos como em sua exposição..:-)

Conceitos como justiça ou gratidão não são naturais, não existem na natureza e entretanto, são excelentes e importantes criações humanas..:-) Confundir natural com correto é algo sempre presente (até nos derivados do movimetno hippie, como alimentos "naturais" e afins..:-)

Outro ponto que você apresenta com clareza é um que costuma me incomodar em praticamente TODA discussão sobre a eficácia da ciência: a acusação de que a ciência pretende "saber tudo sobre tudo e com 100% de certeza". Uma acusação totalmente falsa, mas uma visão da ciência muito difundida entre leigos..:-) É o caso de seu DOGMA B..:-)

Existe uma diferença entre avaliarmos dois saberes em termos de confiabilidade e aproximação com a realidade, e a certeza de 100% que se pretende ser objeto da ciência (e que fica melhor nas religiões..:-). O status da Lua, em termos de universo físico, pode ser mensurado como um satélite rochoso a orbitar a Terra, em perfeito acordo com as leis do movimento e relatividade ou ser uma cabaça iluminada a poucos metros acima da copa das árvores. Mas isso não significa mais nada, além do grau de confiabilidade do status real da Lua..:-)

Assim, no sentido de apropriação do conhecimento cientifico como verdade absoluta (100% de acerto..:-), o cientificismo é realmente apenas uma deformacào da idéia de ciência. Mas é mais comum em leigos que em cientistas, já que a idéia central da ciência é a dúvida não a certeza..:-)

Um abraço.

Homero





----- Original Message -----
From: Manuel Bulcão
To: ciencialist@yahoogrupos.com.br
Sent: Monday, January 10, 2005 12:10 AM
Subject: [ciencialist] Re: É o conhecimento perigoso?



--- Em ciencialist@yahoogrupos.com.br, "Oraculo" <oraculo@a...>
escreveu
> Apesar da primeira parte levantar com precisão os atuais ataques
contra a ciência (que alguns chamam de cientificismo)...

Homero,

Existe ciência e existe cientificismo, consistindo este último
numa "ideologia" com verniz científico, ou seja, num sistema de
crenças pararreligioso (que engloba inclusive uma ética)
pretensamente fundamentado na ciência.

Para o cientificismo, o conhecimento científico não é só indicativo:
é também "imperativo" -- leis do ser (i. e., regularidades do mundo
natural que os cientistas paulatinamente desvendam mediante um
método rigoroso de investigação) são confundidas com "normas", ou
melhor, com leis do dever-ser. Vale dizer, os
cientificistas "valoram" as leis naturais, conferem-lhes uma
dimensão axiológica SEM QUE HAJA UMA RAZÃO PARA ISSO, de modo que
todos os vieses, tudo que não é regular ou que é produto de uma
vontade "arbitrária" deve ser considerado um erro, uma morbidez, uma
mentira, um epifania do MAL.

Outros dogmas do cientificismo: a) Não existe outra gnose (forma de
conhecimento) além da Ciência e Galileu é o seu profeta; b) O método
científico é omnipenetrante, capaz de desvendar todos os mistérios.
[Qualquer semelhança entre essa concepção da Ciência e Iavé ou Alá
talvez não seja mera coincidência.]

São variantes do cientificismo: os positivismos de Condorset e
Comte, o eugenismo de Galton, o social-darwinismo de Spencer & Cia,
o marxismo-leninismo, o nazismo, o "liberismo" (fundamentalismo do
Mercado) de F. Hayek e M. Friedmann, o transumanismo, etc.

Assim como se pode ser católico fervoroso sem ser membro da
integralista TFP, da franco-salazarista Opus Dei ou da facção
radical da Teologia da Libertação, pode-se ser um entusiasta do
pensamento científico sem ser um cientificista.

Por esta razão, um ataque ao cientificismo não é necessariamente uma
investida contra a ciência -- se bem que há filósofos idiotas, como,
por exemplo, o nazista M. Heidegger, que não conseguem discernir
essa diferença.

Abraços,
Manuel Bulcão





##### ##### #####

Para saber mais visite
http://www.ciencialist.hpg.ig.com.br


##### ##### ##### #####


Yahoo! Grupos, um serviço oferecido por:
PUBLICIDADE




------------------------------------------------------------------------------
Links do Yahoo! Grupos

a.. Para visitar o site do seu grupo na web, acesse:
http://br.groups.yahoo.com/group/ciencialist/

b.. Para sair deste grupo, envie um e-mail para:
ciencialist-unsubscribe@yahoogrupos.com.br

c.. O uso que você faz do Yahoo! Grupos está sujeito aos Termos do Serviço do Yahoo!.



[As partes desta mensagem que não continham texto foram removidas]



##### ##### #####

Para saber mais visite
http://www.ciencialist.hpg.ig.com.br


##### ##### ##### #####


Yahoo! Grupos, um serviço oferecido por:







------------------------------------------------------------------------------
Links do Yahoo! Grupos

a.. Para visitar o site do seu grupo na web, acesse:
http://br.groups.yahoo.com/group/ciencialist/

b.. Para sair deste grupo, envie um e-mail para:
ciencialist-unsubscribe@yahoogrupos.com.br

c.. O uso que você faz do Yahoo! Grupos está sujeito aos Termos do Serviço do Yahoo!.



[As partes desta mensagem que não continham texto foram removidas]



SUBJECT: : É o conhecimento perigoso? p/Homero
FROM: "Amauri Jr" <amaurijunior2@yahoo.com.br>
TO: <ciencialist@yahoogrupos.com.br>
DATE: 10/01/2005 16:05

[O]Obrigado pela detalhada explicação. Realmente, eu estava usando no termo no ajuste que me foi apresentado, como sendo, a admiração pela ciência e sua confiabilidade (do ponto de vista do universo físicco), uma atitude cientificista. Seria uma espécie de cegueira ao "não perceber o relativismo do conhecimento cientifico"..:-)

[A] O que seria Homero?

[O]Mas no enfoque que você me apresentou, é outra coisa. Inclusive já debati nesses exatos termos, acho que na lista Ceticismo Aberto, quando uma discussão sobre ética sem deuses descambou para o "natural é bom, artificial é ruim"..:-) Inclusive com os mesmos exemplos, eugenia, darwinsmo social, etc, (apesar de não tão bem descritos como em sua exposição..:-)

[A] Mas a ciencia em si não é natural? A inteligencia humana ao serviço do homem?

[O]Conceitos como justiça ou gratidão não são naturais, não existem na natureza e entretanto, são excelentes e importantes criações humanas..:-) Confundir natural com correto é algo sempre presente (até nos derivados do movimetno hippie, como alimentos "naturais" e afins..:-)

[A] Não, segundo os hippies, o natural é levado a elevação do espirito no conhecimento transcendental. Não julgam a ciencia como "bem" ou "mal", mas uma amarra para a evolução intelectual e espiritual

[O]Outro ponto que você apresenta com clareza é um que costuma me incomodar em praticamente TODA discussão sobre a eficácia da ciência: a acusação de que a ciência pretende "saber tudo sobre tudo e com 100% de certeza". Uma acusação totalmente falsa, mas uma visão da ciência muito difundida entre leigos..:-) É o caso de seu DOGMA B..:-)

[A] Acusação que eu posso provar que existe, como muitos charlatões dogmaticos existem da ciencia tambem existe. Vimos por ai, muita gente quer saber e provar que a ciencia é perfeita...:)

{O]Existe uma diferença entre avaliarmos dois saberes em termos de confiabilidade e aproximação com a realidade, e a certeza de 100% que se pretende ser objeto da ciência (e que fica melhor nas religiões..:-). O status da Lua, em termos de universo físico, pode ser mensurado como um satélite rochoso a orbitar a Terra, em perfeito acordo com as leis do movimento e relatividade ou ser uma cabaça iluminada a poucos metros acima da copa das árvores. Mas isso não significa mais nada, além do grau de confiabilidade do status real da Lua..:-)

[A] pq sempre diz esse exemplo da lua? As religiões eram formas de se expecular o que ouve ou o que há...:)

[O]Assim, no sentido de apropriação do conhecimento cientifico como verdade absoluta (100% de acerto..:-), o cientificismo é realmente apenas uma deformacào da idéia de ciência. Mas é mais comum em leigos que em cientistas, já que a idéia central da ciência é a dúvida não a certeza..:-)

[A] Posso provar em A+B que existe cienticismo dentro da ciencia...:)))

Abraços
Amauri








--- Original Message -----
From: Oraculo
To: ciencialist@yahoogrupos.com.br
Sent: Monday, January 10, 2005 3:19 PM
Subject: Re: [ciencialist] Re: É o conhecimento perigoso? p/ Manuel


Olá Manuel

Obrigado pela detalhada explicação. Realmente, eu estava usando no termo no ajuste que me foi apresentado, como sendo, a admiração pela ciência e sua confiabilidade (do ponto de vista do universo físicco), uma atitude cientificista. Seria uma espécie de cegueira ao "não perceber o relativismo do conhecimento cientifico"..:-)

Mas no enfoque que você me apresentou, é outra coisa. Inclusive já debati nesses exatos termos, acho que na lista Ceticismo Aberto, quando uma discussão sobre ética sem deuses descambou para o "natural é bom, artificial é ruim"..:-) Inclusive com os mesmos exemplos, eugenia, darwinsmo social, etc, (apesar de não tão bem descritos como em sua exposição..:-)

Conceitos como justiça ou gratidão não são naturais, não existem na natureza e entretanto, são excelentes e importantes criações humanas..:-) Confundir natural com correto é algo sempre presente (até nos derivados do movimetno hippie, como alimentos "naturais" e afins..:-)

Outro ponto que você apresenta com clareza é um que costuma me incomodar em praticamente TODA discussão sobre a eficácia da ciência: a acusação de que a ciência pretende "saber tudo sobre tudo e com 100% de certeza". Uma acusação totalmente falsa, mas uma visão da ciência muito difundida entre leigos..:-) É o caso de seu DOGMA B..:-)

Existe uma diferença entre avaliarmos dois saberes em termos de confiabilidade e aproximação com a realidade, e a certeza de 100% que se pretende ser objeto da ciência (e que fica melhor nas religiões..:-). O status da Lua, em termos de universo físico, pode ser mensurado como um satélite rochoso a orbitar a Terra, em perfeito acordo com as leis do movimento e relatividade ou ser uma cabaça iluminada a poucos metros acima da copa das árvores. Mas isso não significa mais nada, além do grau de confiabilidade do status real da Lua..:-)

Assim, no sentido de apropriação do conhecimento cientifico como verdade absoluta (100% de acerto..:-), o cientificismo é realmente apenas uma deformacào da idéia de ciência. Mas é mais comum em leigos que em cientistas, já que a idéia central da ciência é a dúvida não a certeza..:-)

Um abraço.

Homero





----- Original Message -----
From: Manuel Bulcão
To: ciencialist@yahoogrupos.com.br
Sent: Monday, January 10, 2005 12:10 AM
Subject: [ciencialist] Re: É o conhecimento perigoso?



--- Em ciencialist@yahoogrupos.com.br, "Oraculo" <oraculo@a...>
escreveu
> Apesar da primeira parte levantar com precisão os atuais ataques
contra a ciência (que alguns chamam de cientificismo)...

Homero,

Existe ciência e existe cientificismo, consistindo este último
numa "ideologia" com verniz científico, ou seja, num sistema de
crenças pararreligioso (que engloba inclusive uma ética)
pretensamente fundamentado na ciência.

Para o cientificismo, o conhecimento científico não é só indicativo:
é também "imperativo" -- leis do ser (i. e., regularidades do mundo
natural que os cientistas paulatinamente desvendam mediante um
método rigoroso de investigação) são confundidas com "normas", ou
melhor, com leis do dever-ser. Vale dizer, os
cientificistas "valoram" as leis naturais, conferem-lhes uma
dimensão axiológica SEM QUE HAJA UMA RAZÃO PARA ISSO, de modo que
todos os vieses, tudo que não é regular ou que é produto de uma
vontade "arbitrária" deve ser considerado um erro, uma morbidez, uma
mentira, um epifania do MAL.

Outros dogmas do cientificismo: a) Não existe outra gnose (forma de
conhecimento) além da Ciência e Galileu é o seu profeta; b) O método
científico é omnipenetrante, capaz de desvendar todos os mistérios.
[Qualquer semelhança entre essa concepção da Ciência e Iavé ou Alá
talvez não seja mera coincidência.]

São variantes do cientificismo: os positivismos de Condorset e
Comte, o eugenismo de Galton, o social-darwinismo de Spencer & Cia,
o marxismo-leninismo, o nazismo, o "liberismo" (fundamentalismo do
Mercado) de F. Hayek e M. Friedmann, o transumanismo, etc.

Assim como se pode ser católico fervoroso sem ser membro da
integralista TFP, da franco-salazarista Opus Dei ou da facção
radical da Teologia da Libertação, pode-se ser um entusiasta do
pensamento científico sem ser um cientificista.

Por esta razão, um ataque ao cientificismo não é necessariamente uma
investida contra a ciência -- se bem que há filósofos idiotas, como,
por exemplo, o nazista M. Heidegger, que não conseguem discernir
essa diferença.

Abraços,
Manuel Bulcão





##### ##### #####

Para saber mais visite
http://www.ciencialist.hpg.ig.com.br


##### ##### ##### #####


Yahoo! Grupos, um serviço oferecido por:
PUBLICIDADE




------------------------------------------------------------------------------
Links do Yahoo! Grupos

a.. Para visitar o site do seu grupo na web, acesse:
http://br.groups.yahoo.com/group/ciencialist/

b.. Para sair deste grupo, envie um e-mail para:
ciencialist-unsubscribe@yahoogrupos.com.br

c.. O uso que você faz do Yahoo! Grupos está sujeito aos Termos do Serviço do Yahoo!.



[As partes desta mensagem que não continham texto foram removidas]



##### ##### #####

Para saber mais visite
http://www.ciencialist.hpg.ig.com.br


##### ##### ##### #####


Yahoo! Grupos, um serviço oferecido por:







------------------------------------------------------------------------------
Links do Yahoo! Grupos

a.. Para visitar o site do seu grupo na web, acesse:
http://br.groups.yahoo.com/group/ciencialist/

b.. Para sair deste grupo, envie um e-mail para:
ciencialist-unsubscribe@yahoogrupos.com.br

c.. O uso que você faz do Yahoo! Grupos está sujeito aos Termos do Serviço do Yahoo!.



[As partes desta mensagem que não continham texto foram removidas]



SUBJECT: : Ressonância de Shumann/Takata
FROM: "Amauri Jr" <amaurijunior2@yahoo.com.br>
TO: <ciencialist@yahoogrupos.com.br>
DATE: 10/01/2005 16:49

Oi Takata


"A explicacao mais banal para a sensacao de diminuicao de tempo q. muitas pessoas (mas nem de longe todas) sentem eh simplesmente em funcao das modificacoes socioculturais. No Brasil e no mundo houve um intenso processo de urbanizacao da populacao - e a vida nas cidades, com muito mais pessoas ao redor, eh substancialmente mais agitada do q. o ritmo rural."

(A) Se fizer uma pesquisa, vai ver que na area rural tambem tem essa percebição...por outro lado, o tempo é formado pela consciencia humana; mesmo tendo a teoria de Einstein e a quantica.

"Aumento da eficiencia nos servicos de telecomunicacoes - telefonia,internet, televisao e outros - permite uma comunicacao muito mais rapida: se antes levava meses para uma noticia cruzar o Atlantico,isso eh feito hoje em milesimos de segundo. Hah muito mais dados circulando no mundo hoje. Isso tb aumenta a percepcao de mudancas - e aumentaria mesmo q. tais mudancas nao tivesse seu ritmo aumentado. Mas como essa circulacao de dados tb interfere na taxa de alteracoes, em muitos casos, efetivamente hah muito mais mudancas em um mesmo intervalo de tempo (quem jah viu o sobe e desce das acoes em um pregaoeletronico da bolsa percebe isso quase ao vivo - em tempo real)."

(A) E dai? Como disse, o tempo é uma percepção humana...:))


"A mensagem abaixo eh paradoxal. Diz (falsamente) q, se alguem estiver fora da frequencia de Schumann adoece; diz (tb falsamente) q. afrequencia de Schumann estah aumentando e conclui de modo obtuso qdevemos diminuir o ritmo! Oras, se temos q. manter o passo com umafrequencia q. aumenta, temos q. *aumentar* tb o ritmo, do contrario adefasagem cresce ainda mais."

(A) Nossa!!! Cade a conclusão empirista?



"Nao existe uma frequencia unica em nossos cerebros. Existem quatro grupos principais de ondas eletricas detectadas em EEG: as deltas (de1 a 3 Hz), as tetas (de 4 a 7 Hz), as alfas (de 8 a 13 Hz - media de10 Hz), as betas (de 14 a 3o Hz). Qdo estamos acordados as frequenciasdominantes (de maior amplitude) sao as betas. As ondas alfas saodetetadas em celulas isoladas ateh de invertebrados (e elas nao estaoajustadas na frequencia Schumann). (O agrumento das ondas eh relativamente arbitrario.)"

(A) Ué, não estamos falando das freguencias da Terra? O que tem o cerebro?

Amauri
























----- Original Message -----
From: rmtakata
To: ciencialist@yahoogrupos.com.br
Sent: Monday, January 10, 2005 3:24 AM
Subject: [ciencialist] Re: Ressonância de Shumann



Foi o Boff mesmo quem escreveu essa bobagem toda?

*Nao* eh verdade que a ressonancia de Schumann esteja aumentando de valor.

"The Schumann resonance frequency observed at this observatory does
not exhibit any unusual change or drift since the start of
observations by the BDSN in 1995."
http://quake.geo.berkeley.edu/ncedc/em.intro.html

A explicacao mais banal para a sensacao de diminuicao de tempo q.
muitas pessoas (mas nem de longe todas) sentem eh simplesmente em
funcao das modificacoes socioculturais. No Brasil e no mundo houve um
intenso processo de urbanizacao da populacao - e a vida nas cidades,
com muito mais pessoas ao redor, eh substancialmente mais agitada do
q. o ritmo rural.

Aumento da eficiencia nos servicos de telecomunicacoes - telefonia,
internet, televisao e outros - permite uma comunicacao muito mais
rapida: se antes levava meses para uma noticia cruzar o Atlantico,
isso eh feito hoje em milesimos de segundo. Hah muito mais dados
circulando no mundo hoje. Isso tb aumenta a percepcao de mudancas - e
aumentaria mesmo q. tais mudancas nao tivesse seu ritmo aumentado. Mas
como essa circulacao de dados tb interfere na taxa de alteracoes, em
muitos casos, efetivamente hah muito mais mudancas em um mesmo
intervalo de tempo (quem jah viu o sobe e desce das acoes em um pregao
eletronico da bolsa percebe isso quase ao vivo - em tempo real).

A mensagem abaixo eh paradoxal. Diz (falsamente) q, se alguem estiver
fora da frequencia de Schumann adoece; diz (tb falsamente) q. a
frequencia de Schumann estah aumentando e conclui de modo obtuso q
devemos diminuir o ritmo! Oras, se temos q. manter o passo com uma
frequencia q. aumenta, temos q. *aumentar* tb o ritmo, do contrario a
defasagem cresce ainda mais.

Nao existe nenhuma ligacao conhecida entre a frequencia de Schumann e
os ritmos biologicos, ou os ritmos naturais (por exemplo, os dias
estao ficando mais *longos* no decorrer dos seculos - pela
desaceleracao da rotacao da Terra pelos atritos de mare's.)

Nao existe uma frequencia unica para a RS, os 7 Hz eh um valor medio
de diversas frequencias.

Nao existe uma frequencia unica em nossos cerebros. Existem quatro
grupos principais de ondas eletricas detectadas em EEG: as deltas (de
1 a 3 Hz), as tetas (de 4 a 7 Hz), as alfas (de 8 a 13 Hz - media de
10 Hz), as betas (de 14 a 3o Hz). Qdo estamos acordados as frequencias
dominantes (de maior amplitude) sao as betas. As ondas alfas sao
detetadas em celulas isoladas ateh de invertebrados (e elas nao estao
ajustadas na frequencia Schumann). (O agrumento das ondas eh
relativamente arbitrario.)

http://dx.doi.org/10.1016/S0167-8760(97)00753-8

[]s,

Roberto Takata

-- Em ciencialist@yahoogrupos.com.br, "Cyberlander"
> Ressonância de Shumann
>
> Autor: Leonardo Boff
>
> Não apenas as pessoas mais idosas mas também jovens fazem a
> experiência de que tudo está se acelerando excessivamente.
> Ontem foi Carnaval, dentro de pouco será Páscoa, mais um
> pouco, Natal. Esse sentimento é ilusório ou tem base real?
>
> Pela ressonância Schumann se procura dar uma explicação. O
> físico alemão W.O. Schumann constatou em 1952 que a Terra é
> cercada por um campo eletromagnético poderoso que se forma
> entre o solo e a parte inferior da ionosfera, cerca de 100km
> acima de nós. Esse campo possui uma ressonância (dai
> chamar-se ressonância Schumann), mais ou menos constante, da
> ordem de 7,83 pulsações por segundo.
>
> Funciona como uma espécie de marca-passo, responsável pelo
> equilíbrio da biosfera, condição comum de todas as formas de
> vida. Verificou-se também que todos os vertebrados e o nosso
> cérebro são dotados da mesma frequência de 7,83 hertz.
>
> Empiricamente fez-se a constatação de que não podemos ser
> saudáveis fora dessa frequência biológica natural. Sempre
> que os astronautas, em razão das viagens espaciais, ficavam
> fora da ressonância Schumann, adoeciam. Mas submetidos à
> ação de um simulador Schumann recuperavam o equilíbrio e a
> saúde. Por milhares de anos as batidas do coração da Terra
> tinham essa freqüência de pulsações e a vida se desenrolava
> em relativo equilíbrio ecológico. Ocorre que a partir dos
> anos 80, e de forma mais acentuada a partir dos anos 90, a
> freqüência passou de 7,83para 11 e para 13 hertz.
>
> O coração da Terra disparou. Coincidentemente,
> desequilíbrios ecológicos se fizeram sentir: perturbações
> climáticas, maior atividade dos vulcões, crescimento de
> tensões e conflitos no mundo e aumento geral de
> comportamentos desviantes nas pessoas, entre outros. Devido
> à aceleração geral, a jornada de 24 horas, na verdade, é
> somente de 16 horas. Portanto, a percepção de que tudo está
> passando rápido demais não é ilusória,mas teria base real
> nesse transtorno da ressonância Schumann.
>
> Gaia, esse superorganismo vivo que é a Mãe Terra, deverá
> estar buscando formas de retornar a seu equilíbrio natural.
> E vaiconsegui-lo, mas não sabemos a que preço, a ser pago
> pela biosfera e pelos seres humanos. Aqui abre-se o espaço
> para grupos esotéricos e outros futuristas projetarem
> cenários, ora dramáticos, com catástrofes terríveis, ora
> esperançadores, como a irrupção da quarta dimensão, pela
> qual todos seremos mais intuitivos, mais espirituais e mais
> sintonizados com o biorritmo da Terra.
>
> Não pretendo reforçar esse tipo de leitura. Apenas
> enfatizo a tese recorrente entre grandes cosmólogos e
> biólogos de que a Terra é, efetivamente, um superorganismo
> vivo, de que Terra e humanidade formamos uma única entidade,
> como os astronautas testemunham de suas naves espaciais.
> Nós, seres humanos, somos Terra que sente, pensa, ama e
> venera. Porque somos isso, possuímos a mesma natureza
> bioelétrica e estamos envoltos pelas mesmas ondas
> ressonantes Schumann.
>
> Se queremos que a Terra reencontre seu equilíbrio, devemos
> começar por nós mesmos: fazer tudo sem estresse, com mais
> serenidade, com mais harmonia, com mais amor, que é uma
> energia essencialmente harmonizadora. Para isso importa
> termos coragem de ser anticultura dominante, que nos obriga
> a ser cada vez mais competitivos e efetivos. Precisamos
> respirar juntos com a Terra, para conspirar com ela pela
> paz.






##### ##### #####

Para saber mais visite
http://www.ciencialist.hpg.ig.com.br


##### ##### ##### #####


Yahoo! Grupos, um serviço oferecido por:







------------------------------------------------------------------------------
Links do Yahoo! Grupos

a.. Para visitar o site do seu grupo na web, acesse:
http://br.groups.yahoo.com/group/ciencialist/

b.. Para sair deste grupo, envie um e-mail para:
ciencialist-unsubscribe@yahoogrupos.com.br

c.. O uso que você faz do Yahoo! Grupos está sujeito aos Termos do Serviço do Yahoo!.



[As partes desta mensagem que não continham texto foram removidas]



SUBJECT: Sal e a viscosidade
FROM: "E m i l i a n o C h e m e l l o" <chemelloe@yahoo.com.br>
TO: <ciencialist@yahoogrupos.com.br>, <quimica-qaw@yahoogrupos.com.br>, <naeq-ucs@yahoogrupos.com.br>
DATE: 10/01/2005 17:37

Alguém ajuda o fábio?

[ ] 's do Emiliano Chemello

---

From: Fábio fabiobiff@aol.com
To: Emiliano Chemello

oi colega, eu estava lendo sobre detergentes no site do NAEQ e em
outros sites também, dizendo que o cloreto de sódio é usando como
espessante, ou seja, a adição do sal aumentaria a viscodade do produto.
Pois bem, eu pensei no seguinte entao: "se eu adicionar mais sal no
detergente da minha casa, ele se tornaria mais viscoso". Entao eu coloquei
um pouco do detergente num recipiente e adicionei sal de cozinha nele.
Porém, em vez dele ficar mais viscoso, ele ficou menos viscoso ainda. Depois
eu pensei em deixar em repouso um pouco achando que tinha que esperar.
Então, eu esperei um tempo e notei que o sistema ficou heterogêneo com 2
fases: uma fase em cima com cor branca (acho q devido ao sal dissolvido) e
outra em baixo transparente (eu nao sei o q era) e, essa sim, permanecia
viscosa.
Eu gostaria que, se você souber, me explicasse o que aconteceu.
Porque o detergente nao ficou mais viscoso com a adição do sal?
O sal a ser usado depende do tipo de tensoativo?

Bom... eu usei o detergente de coco da ypê (nao sei se coco é marca ou só
aroma) e sal de cozinha para "tentar" torná-lo mais viscoso.

valeu colega... brigado.




SUBJECT: Física Quântica e o tempo
FROM: "flasp" <flasp@uol.com.br>
TO: "ciencialist" <ciencialist@yahoogrupos.com.br>
DATE: 10/01/2005 19:11


O Tempo
Razões para rápidos Natais

Aldo Novak

> "O cérebro humano mede o tempo por meio da observação dos
> movimentos.
> Se alguém colocar você dentro de uma sala branca vazia, sem
>nenhuma
> mobília, sem portas ou janelas, sem relógio, você começará
a perder
> a
> noção do tempo. Por alguns dias, sua mente detectará a
passagem do
> tempo sentindo as reações internas do seu corpo, incluindo
os
> batimentos cardíacos,ciclos de sono, fome, sede e pressão
sanguínea.
> Então... quando tempo suficiente houver passado, você
perderá
> completamente a noção das horas,dos dias ou anos.
>
> Estou exagerando para efeito didático, mas em essência é o
que
> ocorreria. Isso acontece porque nossa noção de passagem do
tempo
> deriva
> do movimento dos objetos, pessoas, sinais naturais e da
repetição de
> eventos cíclicos, como o nascer e o pôr do sol. Se alguém
tirar
> estes
> sinais sensoriais da nossa vida, simplesmente perdemos a
noção da
> passagem do tempo.
>
> Compreendido este ponto, há outra coisa que você tem que
> considerar :
> nosso cérebro é extremamente otimizado. Ele evita fazer
duas vezes o
> mesmo trabalho. Um adulto médio tem entre 40 e 60 mil
pensamentos
> por
> dia. Qualquer um de nós ficaria louco se o cérebro tivesse
que
> processar, conscientemente tal quantidade. Por isso, a
maior parte
> destes pensamentos é automatizada e não aparece no índice
de eventos
> do
> dia. Para que não fiquemos loucos, o cérebro faz parecer
que nós não
> vimos, não sentimos e não vivenciamos aqueles pensamentos
> automáticos,
> repetidos, iguais.
>
> Por isso, quando você vive uma experiência pela primeira
vez, ele
> dedica muitos recursos para compreender o que está
acontecendo. É
> quando você se sente mais vivo. Conforme a mesma
experiência vai se
> repetindo, ele vai simplesmente colocando suas reações no
modo
> automático e "apagando" as experiências duplicadas.
>
> Se você entendeu estes dois pontos, já vai compreender
porque parece
> que o tempo acelera, quando ficamos mais velhos e porque os
Natais
> chegam cada vez mais rapidamente.
>
> Quando começamos a dirigir, tudo parece muito complicado, o
câmbio,
> os
> espelhos, os outros veículos... Nossa atenção parece ser
requisitada
> ao
> máximo. Então, um dia dirigimos trocando de marcha, olhando
os
> semáforos,lendo os sinais ou até falando ao celular
(proibido no
> Brasil), ao mesmo tempo. E você usa apenas uma
pequena "área" da
> atenção para isso.
>
> Como acontece? Simples: o cérebro já sabe o que está
escrito nas
> placas(você não lê com os olhos, mas com a imagem anterior,
na
> mente);
> O cérebro já sabe qual marcha trocar (ele simplesmente pega
suas
> experiências passadas e usa, no lugar de repetir realmente a
> experiência). Em outras palavras, você não vivenciou aquela
> experiência, pelo menos para mente.
>
> Aqueles críticos segundos de troca de marcha, leitura de
placa...
> são
> apagados da sua noção de passagem do tempo... Porque estou
> explicando
> isso? Que relação tem isso com a aparente aceleração do
tempo? Tudo.
>
> A primeira vez que isso me ocorreu foi quando passei três
meses nas
> florestas de New Hampshire, Estados Unidos, morando em uma
cabana.
> Era
> tudo tão diferente, as pessoas, a paisagem, a língua, que
eu tinha
> dores de cabeça sempre que viajava em uma estrada, porque
meu
> cérebro
> ficava lendo todas as placas (eu lia mesmo, pois era tudo
novidade,
> para mim). Foram somente três meses, mas ao final do
segundo mês eu
> já
> me sentia como se estivesse há um ano longe do Brasil. Foi
quando
> comecei a pesquisar a razão dessa diferença de percepção.
>
> Bastou eu voltar ao Brasil e o tempo voltou a "acelerar".
Pelo
> menos,
> assim parecia. Veja, quando você começa a repetir algo
exatamente
> igual,a mente apaga a experiência repetida. Conforme
envelhecemos,
> as
> coisas começam a se repetir - as mesmas ruas, pessoas,
problemas,
> desafios, programas de televisão, reclamações... enfim... as
> experiências novas (aquelas que fazem a mente parar e
pensar de
> verdade, fazendo com que seu dia pareça ter sido longo e
cheio de
> novidades), vão diminuindo. Até que tanta coisa se repete
que fica
> difícil dizer o que tivemos de novidade na semana, no ano
ou, para
> algumas pessoas, na década.
>
> Em outras palavras, o que faz o tempo parecer que acelera é
a
> r-o-t-i-n-a.
>
> Não me entenda mal. A rotina é essencial para a vida e
otimiza muita
> coisa, mas a maioria das pessoas ama tanto a rotina que, ao
longo da
> vida, seu diário acaba sendo um livro de um só capítulo,
repetido
> todos
> os anos."



__________________________________________________________________________
Acabe com aquelas janelinhas que pulam na sua tela.
AntiPop-up UOL - É grátis!
http://antipopup.uol.com.br/




SUBJECT: Origem da vida?
FROM: André <riemma@gmail.com>
TO: ciencialist@yahoogrupos.com.br
DATE: 10/01/2005 23:35

Alguem sabe qual a teoria mais aceita atualmente sobre a origem da
vida na Terra?

Obrigado
André


SUBJECT: Re: [ciencialist] Origem da vida?
FROM: "Amauri Jr" <amaurijunior2@yahoo.com.br>
TO: <ciencialist@yahoogrupos.com.br>
DATE: 10/01/2005 23:35

Meteoritos que vieram do espaço e trouxeram acidos nucleucos...:)

Abraços
Amauri
----- Original Message -----
From: André
To: ciencialist@yahoogrupos.com.br
Sent: Monday, January 10, 2005 11:35 PM
Subject: [ciencialist] Origem da vida?


Alguem sabe qual a teoria mais aceita atualmente sobre a origem da
vida na Terra?

Obrigado
André


##### ##### #####

Para saber mais visite
http://www.ciencialist.hpg.ig.com.br


##### ##### ##### #####


Yahoo! Grupos, um serviço oferecido por:
PUBLICIDADE




------------------------------------------------------------------------------
Links do Yahoo! Grupos

a.. Para visitar o site do seu grupo na web, acesse:
http://br.groups.yahoo.com/group/ciencialist/

b.. Para sair deste grupo, envie um e-mail para:
ciencialist-unsubscribe@yahoogrupos.com.br

c.. O uso que você faz do Yahoo! Grupos está sujeito aos Termos do Serviço do Yahoo!.



[As partes desta mensagem que não continham texto foram removidas]



SUBJECT: Re: [ciencialist] Origem da vida?
FROM: "Alvaro Augusto \(E\)" <alvaro@electraenergy.com.br>
TO: <ciencialist@yahoogrupos.com.br>
DATE: 10/01/2005 23:52

Eu sei!!

[ ]s

Alvaro Augusto

----- Original Message -----
From: André
To: ciencialist@yahoogrupos.com.br
Sent: Monday, January 10, 2005 11:35 PM
Subject: [ciencialist] Origem da vida?


Alguem sabe qual a teoria mais aceita atualmente sobre a origem da
vida na Terra?

Obrigado
André


[As partes desta mensagem que não continham texto foram removidas]



SUBJECT: Re: Pedra esquisita
FROM: "Kentaro Mori" <kentaro.mori@itelefonica.com.br>
TO: ciencialist@yahoogrupos.com.br
DATE: 11/01/2005 00:23


Valeu Takata, andei pesquisando por aí e a região parece mesmo um
derrame basáltico, o "tronco" poderia ser apenas, como vc notou, um
ressalto. Encontrei outras imagens de pedras semelhantes, e outras da
mesma sob outros ângulos.

Também achei um bom fórum de geólogos discutindo sobre Marte, depois
vou perguntar lá também:
http://www.markcarey.com/mars/mars-geology/forum.html

Abraço,

Mori

--- Em ciencialist@yahoogrupos.com.br, "rmtakata" <rmtakata@a...> escreveu
>
> Bem, nao sou geologo (havia uma geologa por aqui, mas ela anda sumida,
> na lista da SBCR tem o Prof. Arlei q. eh do Instituto de Geociencias
> da USP).
>
> Pela foto nao dah pra se ter uma ideia muito boa se eh mesmo uma pedra
> longa. Pode ser um ressalto da superficie em perspectiva baixa.
>
> Da minha opiniao leiga parece ser uma regiao de derrame basaltico, com
> formacao de basalto colunar (o q. explicaria o ladrilhamento da
> superficie).
>
> http://www.bsu.edu/geology/photodis/neu3.htm
> http://granolagoth.mearcair.net/albums/album12/53_columnar_basalt.jpg
> http://www.kidscosmos.org/kid-stuff/mars-trip-basalt.html
>
> Se nao for isso pode ser o monolito negro. Re re.
>
> []s,
>
> Roberto Takata
>
> --- Em ciencialist@yahoogrupos.com.br, "Kentaro Mori"
> http://marsrovers.jpl.nasa.gov/gallery/press/opportunity/20040524a.html
> > Sem brincadeira, é uma pedra de formato curioso. Algum





SUBJECT: Re: Ressonância de Shumann
FROM: "Kentaro Mori" <kentaro.mori@itelefonica.com.br>
TO: ciencialist@yahoogrupos.com.br
DATE: 11/01/2005 00:26


Essa é recorrente... além das boas respostas que já deriam, repasso
abaixo o que escrevi em uma lista sobre ufologia, quando discutiram
sobre o mesmo texto.

Depois adapto, plagio um pouco algo das respostas daqui e coloco no
CeticismoAberto também. Por isso, por favor, corrijam qualquer erro da
nota abaixo (a parte de 1 picoTesla e emissões de um computador, por
exemplo).

Abraços,

Mori

===============

É como o mito do Cinturão de Fótons, ou do dia perdido que a NASA
descobriu: são lendas que procuram sustentar crenças com base em
alguma "descoberta científica" mirabolante.

A ressonância Shumann, contudo, é real. O que não é real é a premissa
básica utilizada nessa interpretação Nova Era, a de que o valor em que
se dá essa ressonância estaria mudando.

O Northern California Earthquake Data Center vem fazendo medições
diárias, e declara:

"A freqüência da ressonância Shumann observada neste observatório não
exibiu nenhuma mudança ou deslocamento incomum desde o início das
observações pelo BDSN em 1995"
http://quake.geo.berkeley.edu/ncedc/em.intro.html

Deve-se ter em mente que essas ondas estacionárias são muito tênues.
Pode-se dar uma olhada nesta página onde um sujeito descreve as
dificuldades que encontrou para detectar por si as freqüências de
ressonância Shumann:
http://wavelab.homestead.com/Schumanns.html

A energia mesmo da freqüencia básica, a mais potente delas, está na
ordem de 1 picoTesla, ou aproximadamente 500.000 vezes mais fraca que
a do campo magnético terrestre. O computador a seu lado, ou o monitor
à sua frente, ou um microondas podem gerar algo bem mais potente que
isso. E, apesar de ser tema de grande polêmica, não há prova
científica de que o uso prolongado de telefones celulares a poucos
centímetros do cérebro cause algum transtorno.

Todavia, há especulações de que talvez haja alguma influência dessas
ondas persistentes da ressonãncia Schumann em nossa biologia. Veja:
http://chrono.umin.jp/htm/H400107.htm
http://www.salzburg.gv.at/Cherry_Schumann_Resonances.pdf
Esses trabalhos tentam evidenciar correlações entre a dita cuja e a
vida animal, mas uma relação causal seria ainda mais difícil. Isso
porque a ressonância se relaciona com a ionosfera, que se relaciona
com tempestades solares, que podem vir a afetar a vida animal de
outras formas que não através da ressonância Schumann. Assim, poderia
haver de fato correlação, mas não causação.

Mesmo que houvesse, contudo, o simples fato de que não há mudanças
significativas em tal ressonância contraria a idéia central do mito.

O que sim é comprovado por inúmeras pesquisas é que nossa percepção da
passagem do tempo pode sim se alterar grandemente devido ao stress e
afins. Mas isso acho que todos sabemos. POr exemplo, ler uma mensagem
chata como esta deve dilatar a percepção de tempo dos leitores a um
período que pode parecer uma eternidade.

Abraços,

Mori





SUBJECT: Re: : Ressonância de Shumann/Takata
FROM: "rmtakata" <rmtakata@altavista.net>
TO: ciencialist@yahoogrupos.com.br
DATE: 11/01/2005 00:49


--- Em ciencialist@yahoogrupos.com.br, "Amauri Jr"
> (A) Se fizer uma pesquisa, vai ver que na area rural tambem
> tem essa percebição...por outro lado, o tempo é formado pela
> consciencia humana; mesmo tendo a teoria de Einstein e a
> quantica.

Isso eh o q. se chama pensamento volitivo. Apenas porq. vc quer q.
essa percepcao de tempo mais curto ocorra tb na area rural nao quer
dizer q. de fato ela ocorra.

Eh bastante provavel q. em muitas areas rurais de fato o ritmo tenha
aumentado, mas de novo nada fora do alcance da melhoria dos meios de
comunicacao e integracao a uma comunidade maior e mais dinamica - hah
um processo intensificado de 'urbanizacao' rural, com criacao de
infra-estruturas tipicas de cidades no meio de areas agricolas: a
mecanizacao e a informatizacao das propriedades rurais traz dessas
consequencias - mesmo aspectos mais basicos como a eletrificacao rural
(q. permite a chegada de televisores e eletrodomesticos). Mas isso nem
de longe ocorre em todos os locais.

> (A) E dai? Como disse, o tempo é uma percepção humana...:))

Existe a *percepcao* da passagem do tempo, o q. eh bem diferente de
dizer q. isso *eh* o tempo. De todo modo em muitos casos a percepcao
se dah simplesmente pelo maior numero de informacoes circulando.

> "A mensagem abaixo eh paradoxal. Diz (falsamente) q, se alguem
estiver fora da frequencia de Schumann adoece; diz (tb falsamente) q.
afrequencia de Schumann estah aumentando e conclui de modo obtuso
qdevemos diminuir o ritmo! Oras, se temos q. manter o passo com
umafrequencia q. aumenta, temos q. *aumentar* tb o ritmo, do contrario
adefasagem cresce ainda mais."
>
> (A) Nossa!!! Cade a conclusão empirista?

(Conclusao empirista eh um termo q. vc inventou?)

> (A) Ué, não estamos falando das freguencias da Terra? O que tem o
cerebro?

Nao teve nem o trabalho de ler a mensagem comentada?

"Verificou-se também que todos os vertebrados e o nosso cérebro são
dotados da mesma frequência de 7,83 hertz."
http://br.groups.yahoo.com/group/ciencialist/message/43591

[]s,

Roberto Takata





SUBJECT: Re: Origem da vida?
FROM: "rmtakata" <rmtakata@altavista.net>
TO: ciencialist@yahoogrupos.com.br
DATE: 11/01/2005 01:01


Entre as explicacoes cientificas ha' duas correntes principais.

A da origem autoctone: a vida terrestre teria origem na propria Terra
atraves de uma rede de reacoes quimicas e fisicas aqui ocorridas.

A da origem extraterrena: a vida na Terra teria sido semeada por
cometas, meteoros ou poeira cosmica trazendo esporos de vida surgida
em outro ponto do universo. (Alguns versoes dessa corrente sao
conhecidas como panespermia.)

Embora cientistas como Hoyle, Wickramasinghe e mesmo Crick (este em
uma provocacao mais aberta com a panespermia direta: isto eh, alguma
civilizacao alienigena tendo semeado diretamente o planeta) tenha
defendido versoes da segunda corrente, a primeira parece ter mais
adeptos entre os pesquisadores das origens da vida.

Alguns aceitam q. certos compostos organicos como aminoacidos foram
trazidos por cometas e meteoritos, mas q. o processo de formacao do
primeiro ser vivo tenha se dado em algum ponto da Terra.
Principalmente em funcao da sintese de compostos organicos a partir de
substancias mais simples em condicoes encontradas no espaco e tb a
descoberta de compostos organicos no espaco e no interior de meteoritos.

Recomendo o livro O Quinto Milagre de Paul Davies (1998, Cia das
Letras, 359 pp). Tb o site de astrobilogia da Nasa:

http://astrobiology.arc.nasa.gov/

(Eh dificil falar em uma teoria especifica para a origem da vida,
estao mais para cenarios de origem da vida.)

[]s,

Roberto Takata

--- Em ciencialist@yahoogrupos.com.br, André <riemma@g...>
> Alguem sabe qual a teoria mais aceita atualmente sobre a
> origem da vida na Terra?
>
> Obrigado
> André





SUBJECT: Re: [ciencialist] Origem da vida?
FROM: "Prof. JC" <profjc2003@yahoo.com.br>
TO: <ciencialist@yahoogrupos.com.br>
DATE: 11/01/2005 03:39

Se a explicação mais aceita for aquela que tiver o maior número de crentes,
então é a explicação criacionista, ainda que ninguém saiba dizer o que é
isso.

Abraços,
Prof. JC


----- Original Message -----
From: "Amauri Jr" <amaurijunior2@yahoo.com.br>
To: <ciencialist@yahoogrupos.com.br>
Sent: Monday, January 10, 2005 11:35 PM
Subject: Re: [ciencialist] Origem da vida?



Meteoritos que vieram do espaço e trouxeram acidos nucleucos...:)

Abraços
Amauri
----- Original Message -----
From: André
To: ciencialist@yahoogrupos.com.br
Sent: Monday, January 10, 2005 11:35 PM
Subject: [ciencialist] Origem da vida?


Alguem sabe qual a teoria mais aceita atualmente sobre a origem da
vida na Terra?

Obrigado
André


##### ##### #####

Para saber mais visite
http://www.ciencialist.hpg.ig.com.br


##### ##### ##### #####


Yahoo! Grupos, um serviço oferecido por:
PUBLICIDADE




------------------------------------------------------------------------------
Links do Yahoo! Grupos

a.. Para visitar o site do seu grupo na web, acesse:
http://br.groups.yahoo.com/group/ciencialist/

b.. Para sair deste grupo, envie um e-mail para:
ciencialist-unsubscribe@yahoogrupos.com.br

c.. O uso que você faz do Yahoo! Grupos está sujeito aos Termos do
Serviço do Yahoo!.



[As partes desta mensagem que não continham texto foram removidas]



##### ##### #####

Para saber mais visite
http://www.ciencialist.hpg.ig.com.br


##### ##### ##### #####
Links do Yahoo! Grupos












SUBJECT: Re: [ciencialist] Re: Origem da vida?
FROM: "JVictor" <jvoneto@uol.com.br>
TO: <ciencialist@yahoogrupos.com.br>
DATE: 11/01/2005 07:22

Takata,

É. Mas supondo que tudo isto seja verdade, resta saber como essa vida alienígena surgiu. Ficamos, então, na mesma. Ou seja: ??????!!???
Mas não custa nada continuar a exercitar os neurônios.
O desconsolo descomunal é que já vocês jamais saberão. Assim queremos nós, os deuses, que somos os responsáveis e culpados por toda essa parafernália. Brinquem não, se não a gente, ó!

Secretário Geral do Olimpo.



----- Original Message -----
From: rmtakata
To: ciencialist@yahoogrupos.com.br
Sent: Tuesday, January 11, 2005 1:01 AM
Subject: [ciencialist] Re: Origem da vida?



Entre as explicacoes cientificas ha' duas correntes principais.

A da origem autoctone: a vida terrestre teria origem na propria Terra
atraves de uma rede de reacoes quimicas e fisicas aqui ocorridas.

A da origem extraterrena: a vida na Terra teria sido semeada por
cometas, meteoros ou poeira cosmica trazendo esporos de vida surgida
em outro ponto do universo. (Alguns versoes dessa corrente sao
conhecidas como panespermia.)

Embora cientistas como Hoyle, Wickramasinghe e mesmo Crick (este em
uma provocacao mais aberta com a panespermia direta: isto eh, alguma
civilizacao alienigena tendo semeado diretamente o planeta) tenha
defendido versoes da segunda corrente, a primeira parece ter mais
adeptos entre os pesquisadores das origens da vida.

Alguns aceitam q. certos compostos organicos como aminoacidos foram
trazidos por cometas e meteoritos, mas q. o processo de formacao do
primeiro ser vivo tenha se dado em algum ponto da Terra.
Principalmente em funcao da sintese de compostos organicos a partir de
substancias mais simples em condicoes encontradas no espaco e tb a
descoberta de compostos organicos no espaco e no interior de meteoritos.

Recomendo o livro O Quinto Milagre de Paul Davies (1998, Cia das
Letras, 359 pp). Tb o site de astrobilogia da Nasa:

http://astrobiology.arc.nasa.gov/

(Eh dificil falar em uma teoria especifica para a origem da vida,
estao mais para cenarios de origem da vida.)

[]s,

Roberto Takata

--- Em ciencialist@yahoogrupos.com.br, André <riemma@g...>
> Alguem sabe qual a teoria mais aceita atualmente sobre a
> origem da vida na Terra?
>
> Obrigado
> André





##### ##### #####

Para saber mais visite
http://www.ciencialist.hpg.ig.com.br


##### ##### ##### #####


Yahoo! Grupos, um serviço oferecido por:







------------------------------------------------------------------------------
Links do Yahoo! Grupos

a.. Para visitar o site do seu grupo na web, acesse:
http://br.groups.yahoo.com/group/ciencialist/

b.. Para sair deste grupo, envie um e-mail para:
ciencialist-unsubscribe@yahoogrupos.com.br

c.. O uso que você faz do Yahoo! Grupos está sujeito aos Termos do Serviço do Yahoo!.



[As partes desta mensagem que não continham texto foram removidas]



SUBJECT: Re: [ciencialist] Sal e a viscosidade
FROM: "JVictor" <jvoneto@uol.com.br>
TO: <ciencialist@yahoogrupos.com.br>
DATE: 11/01/2005 07:34

Fábio

Não sei química. Mas acho que o fato de o pesquisador caseiro não haver observado o aumento da viscosidade com a adição de talvez se deva ao processo. Adicionar sal durante a formação do composto, onde as trocas de energias garantem novas configurações moleculares, deve ter um efeito diferente daquele que se obtém ao adicionar sal depois que o produto está pronto, definido e acabado.

Sds,

Victor.
----- Original Message -----
From: E m i l i a n o C h e m e l l o
To: ciencialist@yahoogrupos.com.br ; quimica-qaw@yahoogrupos.com.br ; naeq-ucs@yahoogrupos.com.br
Sent: Monday, January 10, 2005 5:37 PM
Subject: [ciencialist] Sal e a viscosidade


Alguém ajuda o fábio?

[ ] 's do Emiliano Chemello

---

From: Fábio fabiobiff@aol.com
To: Emiliano Chemello

oi colega, eu estava lendo sobre detergentes no site do NAEQ e em
outros sites também, dizendo que o cloreto de sódio é usando como
espessante, ou seja, a adição do sal aumentaria a viscodade do produto.
Pois bem, eu pensei no seguinte entao: "se eu adicionar mais sal no
detergente da minha casa, ele se tornaria mais viscoso". Entao eu coloquei
um pouco do detergente num recipiente e adicionei sal de cozinha nele.
Porém, em vez dele ficar mais viscoso, ele ficou menos viscoso ainda. Depois
eu pensei em deixar em repouso um pouco achando que tinha que esperar.
Então, eu esperei um tempo e notei que o sistema ficou heterogêneo com 2
fases: uma fase em cima com cor branca (acho q devido ao sal dissolvido) e
outra em baixo transparente (eu nao sei o q era) e, essa sim, permanecia
viscosa.
Eu gostaria que, se você souber, me explicasse o que aconteceu.
Porque o detergente nao ficou mais viscoso com a adição do sal?
O sal a ser usado depende do tipo de tensoativo?

Bom... eu usei o detergente de coco da ypê (nao sei se coco é marca ou só
aroma) e sal de cozinha para "tentar" torná-lo mais viscoso.

valeu colega... brigado.




##### ##### #####

Para saber mais visite
http://www.ciencialist.hpg.ig.com.br


##### ##### ##### #####


Yahoo! Grupos, um serviço oferecido por:

São Paulo Rio de Janeiro Curitiba Porto Alegre Belo Horizonte Brasília




------------------------------------------------------------------------------
Links do Yahoo! Grupos

a.. Para visitar o site do seu grupo na web, acesse:
http://br.groups.yahoo.com/group/ciencialist/

b.. Para sair deste grupo, envie um e-mail para:
ciencialist-unsubscribe@yahoogrupos.com.br

c.. O uso que você faz do Yahoo! Grupos está sujeito aos Termos do Serviço do Yahoo!.



[As partes desta mensagem que não continham texto foram removidas]



SUBJECT: Água em fúria: tsunami
FROM: José Renato <jrma@terra.com.br>
TO: <ciencialist@yahoogrupos.com.br>
DATE: 11/01/2005 10:17

AMBIENTE BRASIL

Tsunamis

De origem japonesa - tsunami designa ondas oceânicas de grande altura. Embora sejam erroneamente denominadas de ondas de maré, as tsunamis não são causadas por influência das forças de maré (forças astronômicas de atração do Sol e da Lua).

Tsunamis são ondas de grande energia geradas por abalos sísmicos. Têm sua origem em maremotos, erupções vulcânicas e nos diversos tipos de movimentos das placas do fundo submarino.

Portanto uma boa definição para a tsunami seria uma onda sísmica que se propaga no oceano. Historicamente, é no Oceano Pacífico onde ocorreram a maioria das tsunamis, por ser uma área cercada por atividades vulcânicas e freqüentes abalos sísmicos. Ao norte do Oceano Pacífico, desde o Japão até o Alasca, existe uma faixa de maior incidência de maremotos e erupções vulcânicas que originariam as tsunamis mais freqüentes do nosso planeta.

Talvez a tsunami mais famosa tenha sido a provocada pela explosão vulcânica da Ilha de Krakatoa no Oceano Pacífico em 26 e 27 de agosto de 1883. A tsunami resultante atingiu as ilhas da Indonésia com ondas de até 35 metros de altura.

As tsunamis ao se propagarem no oceano possuem comprimento da ordem de 150 a 200 km de extensão e apenas 1 metro de altura. Portanto, em alto mar elas são quase imperceptíveis. Entretanto, ao se aproximar de zonas costeiras mais rasas, a redução da velocidade, devido ao atrito com fundo do seu comprimento, porém a energia continua a mesma. Conseqüentemente, a altura da onda aumenta bastante em pouco tempo. Neste ponto, ela pode atingir 10, 20 e até 30 metros de altura, em função de sua energia e da distância do epicentro da tsunami.

Na recentemente levantada hipótese sobre o perigo de um maremoto de grandes proporções, ele seria tão catastrófico quanto maior for presumida explosão vulcânica nas Ilhas Canárias, local onde foi detectada importante atividade sísmica no subsolo.

Uma analogia a esse processo seria uma panela de pressão que tem a sua válvula reguladora obstruída enquanto aumenta o calor interno gerado pelo fogo. A pressão interna vai aumentando proporcionalmente ao acúmulo de energia potencial. Este processo tem continuidade até que ocorra uma ruptura em algum ponto da estrutura da panela redundando em uma explosão, ou seja, na liberação instantânea de grande quantidade de energia.

No caso das Ilhas Canárias foi observado um aumento da atividade sísmica/vulcânica no interior da ilha. Como a mesma estava inerte a várias dezenas de anos, o topo do cone vulcânico, que é a própria ilha, se consolidou de tal forma que se extinguiu o respiro ou válvula de alívio da pressão interna do vulcão. Assim, quanto mais sinais ela der de atividade vulcânica no seu interior maior será o risco de haver uma erupção vulcânica de grandes proporções. O tamanho da onda tsunami gerada será proporcional à quantidade de energia transmitida ao mar no momento da erupção.

Por outro lado, uma erupção vulcânica não é um evento comum e, se levarmos em conta outros fatores, veremos que a probabilidade de formação de uma onda tsunami destruidora é pequena.

Outro fator a ser considerado é a distância do litoral brasileiro, especificamente dos estados do Rio Grande do Norte, Ceará, Maranhão, Piauí, Pará e Amapá, em relação à Ilhas Canárias. São aproximadamente 4,500 km/h, o que equivaleria a 8 horas de percurso até chegar ao litoral brasileiro.

Assim, quanto maior for a distância entre a origem (epicentro) e o litoral de impacto, maior será a perda de sua intensidade por espalhamento e mesmo dissipação de sua energia. Outro fator de reflexão é que quanto menor for a profundidade das zonas por onde a onda propaga maior vai ser a redução de sua energia pelo atrito com o fundo submarino.

Se somarmos a probabilidade e os registros históricos de erupções e/ou abalos sísmicos em ilhas do Oceano Atlântico, que são mínimos, veremos que as chances de ocorrer um acidente ambiental de grandes proporções são baixas.

Desta forma, antes do Brasil, Portugal, Norte da África e o arquipélago de Cabo Verde serão as vítimas potenciais devido à proximidade do epicentro da eventual explosão vulcânica, recebendo diretamente o impacto da onda de grande altura.

Por outro lado se existe a probabilidade é preciso ter cuidado de alterar para as possíveis conseqüências do fenômeno. A conjunção de fatores intervenientes pode provocar estragos catastróficos, daí a importância de que a população seja informada e que as autoridades competentes tomem as devidas precauções. Um bom exemplo desse tipo de política de seguranças é o desenvolvimento através de informações de satélite pela Organização Meteorológica Mundial - OMM.

Devido a freqüência da ocorrência de tsunamis no Pacífico, existe uma rede internacional de sismógrafos ao longo do cinturão de fogo que altera para a formação de qualquer onda catastrófica. Como resultado dessa iniciativa nenhuma morte foi contabilizada com a passagem de uma tsunami no Havaí em 1957. Já a tsunami de 1946, com altura inferior à de 1957, causou inúmeras vítimas fatais pela ausência de um sistema de alerta.

Portanto, medidas preventivas são muito menos onerosas e possíveis de serem tomadas do que medidas corretivas, muito mais dolorosas. O medo é gerado pela ignorância, o respeito é gerado pelo conhecimento.

Fonte: Revista ECO 21 - www.eco21.com.br - outubro, 2001.

.....................................................................................................

Água em Fúria

Tsunami é a palavra japonesa pela qual são mundialmente conhecidos os maremotos, uma catástrofe de extraordinária violência que se abate sobre zonas costeiras. A causa mais freqüente para os tsunamis são terremotos no assoalho marítimo; também podem decorrer de erupções vulcânicas submarinas ou de explosões causadas por gases acumulados no subsolo do oceano.

Podem, ainda vir associados com um terremoto terrestre (foi o que ocorreu em Lisboa, no século XVII: trinta minutos após um gigantesco abalo sísmico, ergueu-se no mar uma onda de dez metros de altura, colhendo milhares de pessoas que aviam fugido do interior da cidade para a costa.

A ação do tsunami é rápida e aterradora. Quando ele começa no meio do oceano, forma ondas de cerca de seis metros, qua avançam em alta velocidade. Ao aproximar-se da costa, a velocidade da onda diminui. O processo se assemelha ao de uma parada brusca, que projeta a água para a frente - e aí pode alcançar até 30 metros adiante.



Os maiores, os piores, os últimos:

1707: Japão (após terremoto; 30 mil mortos)

1755: Lisboa, Portugal (após terremotos; total: 60 mil mortos)

1883: Krakatoa, Indonésia (36 mil mortos)

1896: Honchu, Japão (27 mil mortos)

1976: Mindanao, Filipinas (8 mil mortos)

1992: Nicarágua (100 mortos)

1998: Papua-Nova Guiné (3 mil mortos)

2001: Arequipa, Peru (20 mortos)

2004: Países Asiáticos e Africanos (Índia, Indonésia, Sri Lanka, Maldivas, Malásia, Tailândia, Bangladesh e Mianma, na Ásia; e Somália, Tanzânia, Seichelas e Quênia, na costa leste da África - aproximadamente 150 mil mortos)




Fonte: Como cuidar da nossa água. Coleção Entenda e Aprenda. BEI. São Paulo-SP, 2003.



<http://www.ambientebrasil.com.br/composer.php3?base=./agua/salgada/index.html&conteudo=./agua/salgada/artigos/tsunamis.html >


[As partes desta mensagem que não continham texto foram removidas]



SUBJECT: Re: Origem da vida?
FROM: Manuel Bulcão <manuelbulcao@uol.com.br>
TO: ciencialist@yahoogrupos.com.br
DATE: 11/01/2005 10:58


--- Em ciencialist@yahoogrupos.com.br, "JVictor" <jvoneto@u...>
escreveu
> Takata,
>
> É. Mas supondo que tudo isto seja verdade, resta saber como essa
vida alienígena surgiu. Ficamos, então, na mesma.

Manuel: O astrônomo Fred Hoyle, teórico da panspermia, foi também um
defensor da téoria cosmológica do estado estacionário -- de que o
universo é isotrópico, infinito e eterno: não teve começo nem terá
fim.

Ora, se o universo é o mesmo em todas as direções e "sempre foi
assim", então não é absurda a possibilidade de que a vida seja um
fenômeno que o acompanha desde a eternidade pretérita, isto é, que a
vida também sempre existiu.

Podemos comparar o universo ao conjunto dos números reais e a vida,
ao conjunto dos números naturais. O primeiro conjunto contém o
segundo, mas ambos são infinitos (embora um infinito seja menor que
o outro).

Abraços,
Manuel Bulcão





SUBJECT: Teoria das Cordas
FROM: "E m i l i a n o C h e m e l l o" <chemelloe@yahoo.com.br>
TO: <ciencialist@yahoogrupos.com.br>
DATE: 11/01/2005 11:30

Alguma sugestão da turma?

[ ] 's

Emiliano Chemello
---
Roterdan" <roterdan.abreu@bol.com.br>
Para: Emiliano Chemello
Assunto:Curioso sobre as cordas
Mensagem:Senhores, gostaria de um texto simples (sei que não é fácil..) que
abordasse o tema sobre a teoria das cordas. Tenho interesse em conhecer um
pouco mais sobre o assunto, mas quero fazer uma incursão gradativa no tema.

Obrigado,

Roterdan Abreu
Eng. Eletricista
PUC-RJ / 1980




SUBJECT: Re: [ciencialist] Teoria das Cordas
FROM: "Alvaro Augusto \(E\)" <alvaro@electraenergy.com.br>
TO: <ciencialist@yahoogrupos.com.br>
DATE: 11/01/2005 12:27

É mais fácil construir o moto perpétuo do que encontrar um texto simples sobre teoria das cordas...

Há um texto razoavelmente simples na Wikipedia, mas em inglês: http://en.wikipedia.org/wiki/Superstrings .

Algumas referências em português:
- http://members.tripod.com/alkimia/supercordas.htm
- http://fma.if.usp.br/%7Erivelles/superstrings/sc1.html (artigo que saiu na Ciência Hoje);
- http://brownzilians.het.brown.edu/noticias/news_item.2004-12-13.4465445312 (artigo de Marcelo Gleiser).

[ ]s

Alvaro Augusto



----- Original Message -----
From: E m i l i a n o C h e m e l l o
To: ciencialist@yahoogrupos.com.br
Sent: Tuesday, January 11, 2005 11:30 AM
Subject: [ciencialist] Teoria das Cordas


Alguma sugestão da turma?

[ ] 's

Emiliano Chemello
---
Roterdan" <roterdan.abreu@bol.com.br>
Para: Emiliano Chemello
Assunto:Curioso sobre as cordas
Mensagem:Senhores, gostaria de um texto simples (sei que não é fácil..) que
abordasse o tema sobre a teoria das cordas. Tenho interesse em conhecer um
pouco mais sobre o assunto, mas quero fazer uma incursão gradativa no tema.

Obrigado,

Roterdan Abreu
Eng. Eletricista
PUC-RJ / 1980




##### ##### #####

Para saber mais visite
http://www.ciencialist.hpg.ig.com.br


##### ##### ##### #####


Yahoo! Grupos, um serviço oferecido por:
PUBLICIDADE




------------------------------------------------------------------------------
Links do Yahoo! Grupos

a.. Para visitar o site do seu grupo na web, acesse:
http://br.groups.yahoo.com/group/ciencialist/

b.. Para sair deste grupo, envie um e-mail para:
ciencialist-unsubscribe@yahoogrupos.com.br

c.. O uso que você faz do Yahoo! Grupos está sujeito aos Termos do Serviço do Yahoo!.



[As partes desta mensagem que não continham texto foram removidas]



SUBJECT: Re: [ciencialist] Re: : Ressonância de Shumann/Takata
FROM: "Amauri Jr" <amaurijunior2@yahoo.com.br>
TO: <ciencialist@yahoogrupos.com.br>
DATE: 11/01/2005 13:53

--- Em ciencialist@yahoogrupos.com.br, "Amauri Jr"
> (A) Se fizer uma pesquisa, vai ver que na area rural tambem
> tem essa percebição...por outro lado, o tempo é formado pela
> consciencia humana; mesmo tendo a teoria de Einstein e a
> quantica.

Isso eh o q. se chama pensamento volitivo. Apenas porq. vc quer q.
essa percepcao de tempo mais curto ocorra tb na area rural nao quer
dizer q. de fato ela ocorra.

(A) Não quero nada, só quero mostrar que nas areas rurais exitem o tempo relativamente rapido tambem, que defende essa teoria meu caro é cietistas...:)


Eh bastante provavel q. em muitas areas rurais de fato o ritmo tenha
aumentado, mas de novo nada fora do alcance da melhoria dos meios de
comunicacao e integracao a uma comunidade maior e mais dinamica - hah
um processo intensificado de 'urbanizacao' rural, com criacao de
infra-estruturas tipicas de cidades no meio de areas agricolas: a
mecanizacao e a informatizacao das propriedades rurais traz dessas
consequencias - mesmo aspectos mais basicos como a eletrificacao rural
(q. permite a chegada de televisores e eletrodomesticos). Mas isso nem
de longe ocorre em todos os locais.

(A) Uma teoria para toda Terra e não só para a cidade...:)

> (A) E dai? Como disse, o tempo é uma percepção humana...:))

Existe a *percepcao* da passagem do tempo, o q. eh bem diferente de
dizer q. isso *eh* o tempo. De todo modo em muitos casos a percepcao
se dah simplesmente pelo maior numero de informacoes circulando.

(A) Poxa se nem a ciencia tem seu senso de comunhão o que podera nos salvar???


> "A mensagem abaixo eh paradoxal. Diz (falsamente) q, se alguem
estiver fora da frequencia de Schumann adoece; diz (tb falsamente) q.
afrequencia de Schumann estah aumentando e conclui de modo obtuso
qdevemos diminuir o ritmo! Oras, se temos q. manter o passo com
umafrequencia q. aumenta, temos q. *aumentar* tb o ritmo, do contrario
adefasagem cresce ainda mais."
>
> (A) Nossa!!! Cade a conclusão empirista?

(Conclusao empirista eh um termo q. vc inventou?)

(A) Desculpe erro gramatical, era empirica, mas dá pra saber...:)

> (A) Ué, não estamos falando das freguencias da Terra? O que tem o
cerebro?

Nao teve nem o trabalho de ler a mensagem comentada?

(A) Sim, mesmo o porque tambem li o texto e li a teoria...tudo que nela esta escrito esta ocorrendo, como se explica?

[]s
Amauri


"Verificou-se também que todos os vertebrados e o nosso cérebro são
dotados da mesma frequência de 7,83 hertz."
http://br.groups.yahoo.com/group/ciencialist/message/43591

[]s,

Roberto Takata





##### ##### #####

Para saber mais visite
http://www.ciencialist.hpg.ig.com.br


##### ##### ##### #####



Yahoo! Grupos, um serviço oferecido por:









--------------------------------------------------------------------------------

Links do Yahoo! Grupos

a.. Para visitar o site do seu grupo na web, acesse:
http://br.groups.yahoo.com/group/ciencialist/

b.. Para sair deste grupo, envie um e-mail para:
ciencialist-unsubscribe@yahoogrupos.com.br

c.. O uso que você faz do Yahoo! Grupos está sujeito aos Termos do Serviço do Yahoo!.



[As partes desta mensagem que não continham texto foram removidas]



SUBJECT: Re: Teoria das Cordas
FROM: Manuel Bulcão <manuelbulcao@uol.com.br>
TO: ciencialist@yahoogrupos.com.br
DATE: 11/01/2005 14:08


--- Em ciencialist@yahoogrupos.com.br, "E m i l i a n o C h e m e
l l o" <chemelloe@y...> escreveu
> Alguma sugestão da turma?

Manuel: Na edição 20 da Scientific American Brasil (Janeiro de 2004)
há um artigo de fácil compreensão sobre a teoria das
cordas: "Descobrindo a Teoria das Cordas", de Nathan Jacob Beskovits.

[]s
Manuel Bulcão

> ---
> Roterdan" <roterdan.abreu@b...>
> Para: Emiliano Chemello
> Assunto:Curioso sobre as cordas
> Mensagem:Senhores, gostaria de um texto simples (sei que não é
fácil..) que
> abordasse o tema sobre a teoria das cordas. Tenho interesse em
conhecer um
> pouco mais sobre o assunto, mas quero fazer uma incursão gradativa
no tema.
>
> Obrigado,
>
> Roterdan Abreu
> Eng. Eletricista
> PUC-RJ / 1980





SUBJECT: Re: [ciencialist] Re: Teoria das Cordas
FROM: "E m i l i a n o C h e m e l l o" <chemelloe@yahoo.com.br>
TO: <ciencialist@yahoogrupos.com.br>
DATE: 11/01/2005 14:24

Olá Manuel,

Esse aqui?

Descobrindo a Teoria das Cordas
http://www.ucs.br/ccet/defq/naeq/material_didatico/textos_interativos_10.htm

eheheh. O consulente já leu e achou difícil. Mesmo assim, obrigado pela dica.

[ ] 's do Emiliano Chemello

----- Original Message -----
From: Manuel Bulcão
To: ciencialist@yahoogrupos.com.br
Sent: Tuesday, January 11, 2005 2:08 PM
Subject: [ciencialist] Re: Teoria das Cordas



--- Em ciencialist@yahoogrupos.com.br, "E m i l i a n o C h e m e
l l o" <chemelloe@y...> escreveu
> Alguma sugestão da turma?

Manuel: Na edição 20 da Scientific American Brasil (Janeiro de 2004)
há um artigo de fácil compreensão sobre a teoria das
cordas: "Descobrindo a Teoria das Cordas", de Nathan Jacob Beskovits.

[]s
Manuel Bulcão

> ---
> Roterdan" <roterdan.abreu@b...>
> Para: Emiliano Chemello
> Assunto:Curioso sobre as cordas
> Mensagem:Senhores, gostaria de um texto simples (sei que não é
fácil..) que
> abordasse o tema sobre a teoria das cordas. Tenho interesse em
conhecer um
> pouco mais sobre o assunto, mas quero fazer uma incursão gradativa
no tema.
>
> Obrigado,
>
> Roterdan Abreu
> Eng. Eletricista
> PUC-RJ / 1980





[As partes desta mensagem que não continham texto foram removidas]



SUBJECT: Origem da vida? p/ André
FROM: "Oraculo" <oraculo@atibaia.com.br>
TO: <ciencialist@yahoogrupos.com.br>
DATE: 11/01/2005 15:18

Olá Andre

Um interessante texto para dar uma idéia do atual estágia da teoria autoctone está neste link:

http://www.str.com.br/Scientia/vida.htm

Como o servidor deste site está trocando acentos, para visualizar corretamente o texto clique (no menu de seu navegador IE) em Exibir, Codificação, e escolha Europeu Ocidental que os sinais serão corrigidos.

Um abraço.

Homero
----- Original Message -----
From: rmtakata
To: ciencialist@yahoogrupos.com.br
Sent: Tuesday, January 11, 2005 1:01 AM
Subject: [ciencialist] Re: Origem da vida?



Entre as explicacoes cientificas ha' duas correntes principais.

A da origem autoctone: a vida terrestre teria origem na propria Terra
atraves de uma rede de reacoes quimicas e fisicas aqui ocorridas.

A da origem extraterrena: a vida na Terra teria sido semeada por
cometas, meteoros ou poeira cosmica trazendo esporos de vida surgida
em outro ponto do universo. (Alguns versoes dessa corrente sao
conhecidas como panespermia.)

Embora cientistas como Hoyle, Wickramasinghe e mesmo Crick (este em
uma provocacao mais aberta com a panespermia direta: isto eh, alguma
civilizacao alienigena tendo semeado diretamente o planeta) tenha
defendido versoes da segunda corrente, a primeira parece ter mais
adeptos entre os pesquisadores das origens da vida.

Alguns aceitam q. certos compostos organicos como aminoacidos foram
trazidos por cometas e meteoritos, mas q. o processo de formacao do
primeiro ser vivo tenha se dado em algum ponto da Terra.
Principalmente em funcao da sintese de compostos organicos a partir de
substancias mais simples em condicoes encontradas no espaco e tb a
descoberta de compostos organicos no espaco e no interior de meteoritos.

Recomendo o livro O Quinto Milagre de Paul Davies (1998, Cia das
Letras, 359 pp). Tb o site de astrobilogia da Nasa:

http://astrobiology.arc.nasa.gov/

(Eh dificil falar em uma teoria especifica para a origem da vida,
estao mais para cenarios de origem da vida.)

[]s,

Roberto Takata

--- Em ciencialist@yahoogrupos.com.br, André <riemma@g...>
> Alguem sabe qual a teoria mais aceita atualmente sobre a
> origem da vida na Terra?
>
> Obrigado
> André





##### ##### #####

Para saber mais visite
http://www.ciencialist.hpg.ig.com.br


##### ##### ##### #####


Yahoo! Grupos, um serviço oferecido por:







------------------------------------------------------------------------------
Links do Yahoo! Grupos

a.. Para visitar o site do seu grupo na web, acesse:
http://br.groups.yahoo.com/group/ciencialist/

b.. Para sair deste grupo, envie um e-mail para:
ciencialist-unsubscribe@yahoogrupos.com.br

c.. O uso que você faz do Yahoo! Grupos está sujeito aos Termos do Serviço do Yahoo!.



[As partes desta mensagem que não continham texto foram removidas]



SUBJECT: Teoria das Cordas p/ Emiliano
FROM: "Oraculo" <oraculo@atibaia.com.br>
TO: <ciencialist@yahoogrupos.com.br>
DATE: 11/01/2005 15:27

Olá Emiliano

Realmente, um texto simples e fácil de entender sobre qualquer aspecto da astrofísica, cosmologia ou física avançada deve ser mais dificil de encontrar (ou criar) que um moto perpétuo..:-) Mas se o seu consulente entende ingles, (falado ou escrito) ele deve dar uma olhada neste link, que leva à uma sequencia de filmes sobre a teoria, muito, muito bom mesmo.

E relativamente (e põe relativametne nisso..:-) fácil de entender..:-)

The Elegant Universe

http://www.pbs.org/wgbh/nova/elegant/program.html

Eu já assisti diversas vezes, é de exceletne qualidade, excelentes animações, excelente didática, dentro do possível para a teoria das cordas, claro. Se o texto narrado estiver dificil de compreender, é possível ajustar o sistema de exibição para apresentar as legendas do que está sendo dito, o que facilita bastante a compreensão.

De uma olhada você também, acho que vai gostar muito.

Um abraço.

Homero
----- Original Message -----
From: E m i l i a n o C h e m e l l o
To: ciencialist@yahoogrupos.com.br
Sent: Tuesday, January 11, 2005 2:24 PM
Subject: Re: [ciencialist] Re: Teoria das Cordas


Olá Manuel,

Esse aqui?

Descobrindo a Teoria das Cordas
http://www.ucs.br/ccet/defq/naeq/material_didatico/textos_interativos_10.htm

eheheh. O consulente já leu e achou difícil. Mesmo assim, obrigado pela dica.

[ ] 's do Emiliano Chemello

----- Original Message -----
From: Manuel Bulcão
To: ciencialist@yahoogrupos.com.br
Sent: Tuesday, January 11, 2005 2:08 PM
Subject: [ciencialist] Re: Teoria das Cordas



--- Em ciencialist@yahoogrupos.com.br, "E m i l i a n o C h e m e
l l o" <chemelloe@y...> escreveu
> Alguma sugestão da turma?

Manuel: Na edição 20 da Scientific American Brasil (Janeiro de 2004)
há um artigo de fácil compreensão sobre a teoria das
cordas: "Descobrindo a Teoria das Cordas", de Nathan Jacob Beskovits.

[]s
Manuel Bulcão

> ---
> Roterdan" <roterdan.abreu@b...>
> Para: Emiliano Chemello
> Assunto:Curioso sobre as cordas
> Mensagem:Senhores, gostaria de um texto simples (sei que não é
fácil..) que
> abordasse o tema sobre a teoria das cordas. Tenho interesse em
conhecer um
> pouco mais sobre o assunto, mas quero fazer uma incursão gradativa
no tema.
>
> Obrigado,
>
> Roterdan Abreu
> Eng. Eletricista
> PUC-RJ / 1980





[As partes desta mensagem que não continham texto foram removidas]



##### ##### #####

Para saber mais visite
http://www.ciencialist.hpg.ig.com.br


##### ##### ##### #####


Yahoo! Grupos, um serviço oferecido por:

São Paulo Rio de Janeiro Curitiba Porto Alegre Belo Horizonte Brasília




------------------------------------------------------------------------------
Links do Yahoo! Grupos

a.. Para visitar o site do seu grupo na web, acesse:
http://br.groups.yahoo.com/group/ciencialist/

b.. Para sair deste grupo, envie um e-mail para:
ciencialist-unsubscribe@yahoogrupos.com.br

c.. O uso que você faz do Yahoo! Grupos está sujeito aos Termos do Serviço do Yahoo!.



[As partes desta mensagem que não continham texto foram removidas]



SUBJECT: Re: [ciencialist] Teoria das Cordas p/ Emiliano
FROM: "Alvaro Augusto \(E\)" <alvaro@electraenergy.com.br>
TO: <ciencialist@yahoogrupos.com.br>
DATE: 11/01/2005 16:09

Bem, eu me referia a um texto simples que não se limitasse a dizer que as partículas são formadas de cordas que vibram em 10 dimensões (ou 11, dependendo da teoria), e que tudo é muito belo exceto por três ou quatro problemas que ninguém consegue resolver...

O problema em se aprender supercordas é que a teoria é bestialmente complicada. Qualquer pessoa inteligente, estudando sozinha, pode aprender relatividade geral ou mecânica quântica, mas a matemática das supercordas é muito complexa. A lista de conhecimentos matemáticos publicada em http://superstringtheory.com/math/math2.html dá uma idéia do problema.

[ ]s

Alvaro Augusto

----- Original Message -----
From: Oraculo
To: ciencialist@yahoogrupos.com.br
Sent: Tuesday, January 11, 2005 3:27 PM
Subject: [ciencialist] Teoria das Cordas p/ Emiliano


Olá Emiliano

Realmente, um texto simples e fácil de entender sobre qualquer aspecto da astrofísica, cosmologia ou física avançada deve ser mais dificil de encontrar (ou criar) que um moto perpétuo..:-) Mas se o seu consulente entende ingles, (falado ou escrito) ele deve dar uma olhada neste link, que leva à uma sequencia de filmes sobre a teoria, muito, muito bom mesmo.

E relativamente (e põe relativametne nisso..:-) fácil de entender..:-)

The Elegant Universe

http://www.pbs.org/wgbh/nova/elegant/program.html

Eu já assisti diversas vezes, é de exceletne qualidade, excelentes animações, excelente didática, dentro do possível para a teoria das cordas, claro. Se o texto narrado estiver dificil de compreender, é possível ajustar o sistema de exibição para apresentar as legendas do que está sendo dito, o que facilita bastante a compreensão.

De uma olhada você também, acho que vai gostar muito.

Um abraço.

Homero
----- Original Message -----
From: E m i l i a n o C h e m e l l o
To: ciencialist@yahoogrupos.com.br
Sent: Tuesday, January 11, 2005 2:24 PM
Subject: Re: [ciencialist] Re: Teoria das Cordas


Olá Manuel,

Esse aqui?

Descobrindo a Teoria das Cordas
http://www.ucs.br/ccet/defq/naeq/material_didatico/textos_interativos_10.htm

eheheh. O consulente já leu e achou difícil. Mesmo assim, obrigado pela dica.

[ ] 's do Emiliano Chemello

----- Original Message -----
From: Manuel Bulcão
To: ciencialist@yahoogrupos.com.br
Sent: Tuesday, January 11, 2005 2:08 PM
Subject: [ciencialist] Re: Teoria das Cordas



--- Em ciencialist@yahoogrupos.com.br, "E m i l i a n o C h e m e
l l o" <chemelloe@y...> escreveu
> Alguma sugestão da turma?

Manuel: Na edição 20 da Scientific American Brasil (Janeiro de 2004)
há um artigo de fácil compreensão sobre a teoria das
cordas: "Descobrindo a Teoria das Cordas", de Nathan Jacob Beskovits.

[]s
Manuel Bulcão

> ---
> Roterdan" <roterdan.abreu@b...>
> Para: Emiliano Chemello
> Assunto:Curioso sobre as cordas
> Mensagem:Senhores, gostaria de um texto simples (sei que não é
fácil..) que
> abordasse o tema sobre a teoria das cordas. Tenho interesse em
conhecer um
> pouco mais sobre o assunto, mas quero fazer uma incursão gradativa
no tema.
>
> Obrigado,
>
> Roterdan Abreu
> Eng. Eletricista
> PUC-RJ / 1980




[As partes desta mensagem que não continham texto foram removidas]



SUBJECT: Huygens em Titã - nesta sexta, 14/01
FROM: Luis Brudna <luisbrudna@gmail.com>
TO: ciencialist@yahoogrupos.com.br, ctjovem@yahoogrupos.com.br
DATE: 11/01/2005 16:45

Especial 2005
CIÊNCIA NO PLANETÁRIO
Colóquios Eduardo Dorneles Barcelos
-----------------------------------------------------------------------------
--------------------------

Convidamos todos os interessados a participarem dos Colóquios introdutórios
+
Transmissão (webcast) da descida da sonda espacial HUYGENS em Titã, lua de
Saturno. Esta atividade dá início ao projeto Ciência no Planetário 2005, e
acon-
tecerá nesta sexta-feira, dia 13/01/2005, na sala multimeios do Planetário
da
UFRGS (Av. Ipiranga, 2000), em dois horários diferentes, a saber:

Sessão 1 - Acompanhando a descida - 9h00-12h00
Sessão 2 - Revisando os primeiros dados - 18h30-21h00

Além do público em geral, esta atividade é de grande interesse para
professores
do ensino fundamental e médio, bem como os demais interessados no ensino e
divulgação de ciências, em particular, da astronomia.

O projeto Ciência no Planetário é uma promoção do Planetário da UFRGS, das
Pró-Reitorias de Pesquisa (PROPESQ) e de Extensão (PROREXT) da UFRGS e
da Sociedade Astronômica Riograndense (SARG).

Mais Informações sobre o evento:
Fone (51) 3316-5384
www.planetario.ufrgs.br
www.ufrgs.br/exobiologia

===========================================================

"Huygens em Titã"
transmissão ao vivo
Nesta data, a sonda européia HUYGENS (ESA), transportada pela CASSINI (NASA)
descerá em Titã, maior lua
de Saturno, segunda maior de todo o sistema solar e única a possuir uma
atmosfera. Apesar de muito fria (-170
graus Celsius), a diversidade de moléculas orgânicas ali detectadas fazem
desta lua um importante laboratório
para compreendermos a origem da vida na terra e, talvez, até mesmo verificar
as possibilidades de vida (unice-
lular) em outros corpos do sistema solar. Acompanhe este momento histórico
em primeira mão.

sexta-feira, 14 de janeiro de 2005
Sessão 1 - Acompanhando a descida
9h00-12h00
Sessão 2 - Revisando os primeiros dados
18h30-21h00
Entrada Franca

*

Apresentação
Prof. Jorge A Quillfeldt
Depto de Biofísica, IB / UFRGS
(coordenador da disciplina BIO10-012 Exobiologia)

*

Mais informações
www.ufrgs.br/exobiologia

===========================================================

Jorge A Quillfeldt
LPBNC - Biofísica - UFRGS








==================================

Até
Luís Brudna
http://antiparticula.webcindario.com


SUBJECT: Re: [ciencialist] Huygens em Titã - nesta sexta, 14/01
FROM: "Oraculo" <oraculo@atibaia.com.br>
TO: <ciencialist@yahoogrupos.com.br>
DATE: 11/01/2005 17:17

Olá pessoal

Uma descrição da missão da Huygens e do que se espera dela. Muito emocionante, deêm uma lida.:-)

http://www.nasa.gov/mission_pages/cassini/whycassini/30dec_titan.html

Um abraço.

Homero
----- Original Message -----
From: Luis Brudna
To: ciencialist@yahoogrupos.com.br ; ctjovem@yahoogrupos.com.br
Sent: Tuesday, January 11, 2005 4:45 PM
Subject: [ciencialist] Huygens em Titã - nesta sexta, 14/01


Especial 2005
CIÊNCIA NO PLANETÁRIO
Colóquios Eduardo Dorneles Barcelos
-----------------------------------------------------------------------------
--------------------------

Convidamos todos os interessados a participarem dos Colóquios introdutórios
+
Transmissão (webcast) da descida da sonda espacial HUYGENS em Titã, lua de
Saturno. Esta atividade dá início ao projeto Ciência no Planetário 2005, e
acon-
tecerá nesta sexta-feira, dia 13/01/2005, na sala multimeios do Planetário
da
UFRGS (Av. Ipiranga, 2000), em dois horários diferentes, a saber:

Sessão 1 - Acompanhando a descida - 9h00-12h00
Sessão 2 - Revisando os primeiros dados - 18h30-21h00

Além do público em geral, esta atividade é de grande interesse para
professores
do ensino fundamental e médio, bem como os demais interessados no ensino e
divulgação de ciências, em particular, da astronomia.

O projeto Ciência no Planetário é uma promoção do Planetário da UFRGS, das
Pró-Reitorias de Pesquisa (PROPESQ) e de Extensão (PROREXT) da UFRGS e
da Sociedade Astronômica Riograndense (SARG).

Mais Informações sobre o evento:
Fone (51) 3316-5384
www.planetario.ufrgs.br
www.ufrgs.br/exobiologia

===========================================================

"Huygens em Titã"
transmissão ao vivo
Nesta data, a sonda européia HUYGENS (ESA), transportada pela CASSINI (NASA)
descerá em Titã, maior lua
de Saturno, segunda maior de todo o sistema solar e única a possuir uma
atmosfera. Apesar de muito fria (-170
graus Celsius), a diversidade de moléculas orgânicas ali detectadas fazem
desta lua um importante laboratório
para compreendermos a origem da vida na terra e, talvez, até mesmo verificar
as possibilidades de vida (unice-
lular) em outros corpos do sistema solar. Acompanhe este momento histórico
em primeira mão.

sexta-feira, 14 de janeiro de 2005
Sessão 1 - Acompanhando a descida
9h00-12h00
Sessão 2 - Revisando os primeiros dados
18h30-21h00
Entrada Franca

*

Apresentação
Prof. Jorge A Quillfeldt
Depto de Biofísica, IB / UFRGS
(coordenador da disciplina BIO10-012 Exobiologia)

*

Mais informações
www.ufrgs.br/exobiologia

===========================================================

Jorge A Quillfeldt
LPBNC - Biofísica - UFRGS








==================================

Até
Luís Brudna
http://antiparticula.webcindario.com


##### ##### #####

Para saber mais visite
http://www.ciencialist.hpg.ig.com.br


##### ##### ##### #####


Yahoo! Grupos, um serviço oferecido por:







------------------------------------------------------------------------------
Links do Yahoo! Grupos

a.. Para visitar o site do seu grupo na web, acesse:
http://br.groups.yahoo.com/group/ciencialist/

b.. Para sair deste grupo, envie um e-mail para:
ciencialist-unsubscribe@yahoogrupos.com.br

c.. O uso que você faz do Yahoo! Grupos está sujeito aos Termos do Serviço do Yahoo!.



[As partes desta mensagem que não continham texto foram removidas]



SUBJECT: Habilidades para linguagem são compartilhadas com mamiferos
FROM: "Oraculo" <oraculo@atibaia.com.br>
TO: <ciencialist@yahoogrupos.com.br>
DATE: 11/01/2005 17:29

http://noticias.terra.com.br/ciencia/interna/0,,OI452018-EI238,00.html

Estudo mostra que ratos distinguem idiomas

Cientistas demonstraram que alguns mamíferos distinguem, pela entonação e o ritmo, línguas diferentes. Em um estudo publicado neste domingo pelo Journal of Experimental Psychology, um grupo espanhol de especialistas em neurociência cognitiva demonstra que os ratos reconhecem a diferença entre o holandês e o japonês.
Pesquisas anteriores constataram que os humanos adultos e os recém-nascidos, assim como uma espécie de macaco, também contam com esta capacidade. No entanto, esta é a primeira vez que isso é demonstrado em um animal não-primata e parece indicar que existe uma base perceptiva comum a todos os mamíferos, que lhes permite apreciar as diferenças no ritmo e na entonação de línguas diferentes.

Essa habilidade apontaria a existência de "precursores da linguagem" compartilhados pelos mamíferos e adquiridos como resultado do processo evolutivo. "Foi surpreendente descobrir que os ratos percebem certa informação que parece tão importante no desenvolvimento da linguagem nos humanos", apontou Juan Toro, autor principal do estudo, que pertence ao Grupo de Pesquisas de Neurociencia Cognitiva da Universidade de Barcelona.

O estudo mostra "que habilidades que os seres humanos usam para a linguagem são compartilhadas com outros animais e quais são particulares dos seres humanos", explicou. No experimento, alguns ratos foram treinados para que pressionassem uma alavanca quando ouvissem uma frase de cinco segundos em holandês e a não fazê-lo quando a frase era em japonês, e vice-versa.

Posteriormente, os cientistas usaram frases novas nesses dois idiomas e os roedores reconheceram que pertenciam a uma ou outra língua, e reagiram corretamente. Os especialistas escolheram holandês e japonês para o experimento porque pertencem a famílias lingüísticas diferentes e seu ritmo é diferente, segundo o artigo.



[As partes desta mensagem que não continham texto foram removidas]



SUBJECT: Re: : Ressonância de Shumann/Takata
FROM: "rmtakata" <rmtakata@altavista.net>
TO: ciencialist@yahoogrupos.com.br
DATE: 12/01/2005 02:14


--- Em ciencialist@yahoogrupos.com.br, "Amauri Jr"
> exitem o tempo relativamente rapido tambem, que defende essa
> teoria meu caro é cietistas...:)

Se vc puder nomear um trabalho.

> (A) Uma teoria para toda Terra e não só para a cidade...:)

Uma explicacao baseada, por exemplo, no acesso a comunicacao explica
muito melhor as diferencas de ritmos em diversos locais. Qto maior a
rede de comunicacao local e conexao com outros locais, mais intenso o
ritmo.

Conhecemos certos grotoes em q. o ritmo de vida mudou com a chegada da
eletrificacao, da televisao ou simplesmente dos correios. Outras
facilidades da vida moderna tb alteram os ritmos - pense na diferenca
de tempo entre acender o fogao a lenha, pegar a agua do poco,
esquentah-la em uma panela para ter agua quente e simplesmente girar o
registro do chuveiro.

> (A) Poxa se nem a ciencia tem seu senso de comunhão o que
> podera nos salvar???

E q. maior senso de comunhao do q. a comunicacao? E sempre poderemos
contar com o Polegar Vermelho!

> (Conclusao empirista eh um termo q. vc inventou?)
>
> (A) Desculpe erro gramatical, era empirica, mas dá pra
> saber...:)

Na mesma: Conclusao empirica eh um termo q. vc inventou?

> (A) Sim, mesmo o porque tambem li o texto e li a teoria...tudo que
nela esta escrito esta ocorrendo, como se explica?

Eu procurei demonstrar em minha mensagem q. pouca coisa do q. o texto
diz estah ocorrendo. Para comecar *nao* existe um aumento da
frequencia da ressonancia Schumann.

"The Schumann resonance frequency observed at this observatory does
not exhibit any unusual change or drift since the start of
observations by the BDSN in 1995."
http://quake.geo.berkeley.edu/ncedc/em.intro.html

[A frequencia da ressonancia de Schumann observada deste observatorio
*nao* apresenta nenhuma modificacao incomum ou deriva desde o inicio
das observacoes pelo BDSN em 1995. - Grifo meu.]

[]s,

Roberto Takata





SUBJECT: Re: : Ressonância de Shumann/Takata
FROM: "rmtakata" <rmtakata@altavista.net>
TO: ciencialist@yahoogrupos.com.br
DATE: 12/01/2005 03:22


Dois estudos interessantes sobre fatores q. influenciam a percepcao do
tempo:

Assessment of time perception: the effect of aging.
http://www.ncbi.nlm.nih.gov/entrez/query.fcgi?cmd=Retrieve&db=pubmed&dopt=Abstract&list_uids==15147591

Social contagion of time perception
http://dx.doi.org/10.1016/S0022-1031(03)00089-1

E um estudo sobre a influencia de campos magneticos de baixa
frequencia sobre a cognicao humana - ou melhor, a *ausencia* de
influencia.

No influence of 20 and 400 microT, 50 Hz magnetic field exposure on
cognitive function in humans.
http://www.ncbi.nlm.nih.gov/entrez/query.fcgi?cmd=Retrieve&db=pubmed&dopt=Abstract&list_uids==15515037

[]s,

Roberto Takata





SUBJECT: Re: [ciencialist] Re: neutrino, existir or not be?
FROM: "JVictor" <jvoneto@uol.com.br>
TO: <ciencialist@yahoogrupos.com.br>
DATE: 12/01/2005 08:59

Ray,

Verei se no próximo fim de semana posso opinar a respeito. Alguém da lista poderia ajudar?

Sds,

Victor.
----- Original Message -----
From: rayfisica
To: ciencialist@yahoogrupos.com.br
Sent: Monday, January 10, 2005 8:43 AM
Subject: Fw: [ciencialist] Re: neutrino, existir or not be?



--- Em ciencialist@yahoogrupos.com.br, "JVictor" <jvoneto@u...>
escreveu
> Ray,
>
> Comento a seguir suas observações.
>
> Victor.
>
>

>
>
>
Penso que foi Stephen Hawking que disse, e se não disse perdeu uma
ótima oportunidade de dizer, quando escreveu o UMA BREVE HISTORIA
DO
TEMPO que quem realmente conhece um assunto sabe demonstrar para o
vulgo (algo assim).
Porem aproveitando a oportunidade de aprender, eu pergunto, e as
outras dimensões?
EXE. Dizem que se a terra fosse oca não haveria gravidade no
interior, sendo portanto uma dimensão extra (acho) pelo ao menos
no
tangente a gravidade, portanto poderia ser muito mais fácil ir do
Brasil ao Japão pelo interior, do que viajar a mesma distancia
pelo
exterior (espero ter sido claro) como ficaria a simetria nesse caso?
Desculpe estou com dificuldade em colocar a questão sem parecer
idiota, faça um esforço.





##### ##### #####

Para saber mais visite
http://www.ciencialist.hpg.ig.com.br


##### ##### ##### #####


Yahoo! Grupos, um serviço oferecido por:

São Paulo Rio de Janeiro Curitiba Porto Alegre Belo Horizonte Brasília




------------------------------------------------------------------------------
Links do Yahoo! Grupos

a.. Para visitar o site do seu grupo na web, acesse:
http://br.groups.yahoo.com/group/ciencialist/

b.. Para sair deste grupo, envie um e-mail para:
ciencialist-unsubscribe@yahoogrupos.com.br

c.. O uso que você faz do Yahoo! Grupos está sujeito aos Termos do Serviço do Yahoo!.



---
Outgoing mail is certified Virus Free.
Checked by AVG anti-virus system (http://www.grisoft.com).
Version: 6.0.830 / Virus Database: 565 - Release Date: 06/01/05

[As partes desta mensagem que não continham texto foram removidas]



SUBJECT: Por que massa atrai massa?
FROM: "E m i l i a n o C h e m e l l o" <chemelloe@yahoo.com.br>
TO: <ciencialist@yahoogrupos.com.br>
DATE: 12/01/2005 09:00

Em uma pergunta feita numa lista de discussão, foi pedido: "Por que massa atrai massa?". Eu respondi, com uma citação:

"Ninguém sabe porque as coisas são como são".
Richard Feynman

A ciência investiga 'como' as coisas funcionam e não 'porque' funcionam. Alguém concorda? Alguém discorda? Alguém não tá nem ai para isso? :-)

[ ] 's do Emiliano Chemello
emiliano@quimica.net
http://www.quimica.net/emiliano
http://www.ucs.br/ccet/defq/naeq

" Rien ne se perd, rien ne se crée,
tout se transforme."

Antoine Laurent de Lavoisier (químico francês, 1743 - 1794)


[As partes desta mensagem que não continham texto foram removidas]



SUBJECT: Re: Por que massa atrai massa?
FROM: marcelo ferrari <emailferrari@yahoo.com.br>
TO: ciencialist@yahoogrupos.com.br
DATE: 12/01/2005 13:40

Emiliano,

Se "como" é o "porque" da ciencia, basta efetuar uma pequena manobra semantica a pergunta ficará assim: "Como massa atrai massa?".


marcelo ferrari.





E m i l i a n o C h e m e l l o <chemelloe@yahoo.com.br> wrote:
Em uma pergunta feita numa lista de discussão, foi pedido: "Por que massa atrai massa?". Eu respondi, com uma citação:

"Ninguém sabe porque as coisas são como são".
Richard Feynman

A ciência investiga 'como' as coisas funcionam e não 'porque' funcionam. Alguém concorda? Alguém discorda? Alguém não tá nem ai para isso? :-)

[ ] 's do Emiliano Chemello
emiliano@quimica.net
http://www.quimica.net/emiliano
http://www.ucs.br/ccet/defq/naeq

" Rien ne se perd, rien ne se crée,
tout se transforme."

Antoine Laurent de Lavoisier (químico francês, 1743 - 1794)


[As partes desta mensagem que não continham texto foram removidas]



##### ##### #####

Para saber mais visite
http://www.ciencialist.hpg.ig.com.br


##### ##### ##### #####


Yahoo! Grupos, um serviço oferecido por:



















function SearchComboBox() { if (document.form_combo.keyword.value.length==0){ alert("Por favor, digite algo."); return false; }else { document.form_combo.action ="http://br.rd.yahoo.com/SIG=12ap2paan/M=264105.3931087.6562589.1588051/D=brclubs/S=2137111528:HM/EXP=1105613876/A=2361264/R=0/SIG=11uaou2jn/*http://www.bondfaro.com/bondfaro/in/combosearch_in.jsp?sk=11"; } return true;} [input] [input] [input]

---------------------------------
Links do Yahoo! Grupos

Para visitar o site do seu grupo na web, acesse:
http://br.groups.yahoo.com/group/ciencialist/

Para sair deste grupo, envie um e-mail para:
ciencialist-unsubscribe@yahoogrupos.com.br

O uso que você faz do Yahoo! Grupos está sujeito aos Termos do Serviço do Yahoo!.



---------------------------------
Yahoo! Acesso Grátis - Internet rápida e grátis. Instale o discador do Yahoo! agora.

[As partes desta mensagem que não continham texto foram removidas]



SUBJECT: Novo Fórum de discussão - O Estrangeiro
FROM: psicopr <psicopr@yahoo.com.br>
TO: psicopr@yahoo.com.br
DATE: 12/01/2005 14:03

Olá a todos,

O site "O Estrangeiro" (www.oestrangeiro.net ) está abrindo um novo fórum de discussão sobre os assuntos que aborda (filosofia, ciências humanas, Foucault, Deleuze, Guattari, etc..): www.forumoestrangeiro.cjb.net

Para participar, basta inscrever-se dentro do próprio fórum, e participar das discussões e trocas de referências.

Aguardamos sua participação


---------------------------------
Yahoo! Acesso Grátis - Internet rápida e grátis. Instale o discador do Yahoo! agora.

---------------------------------
Yahoo! Acesso Grátis - Internet rápida e grátis. Instale o discador do Yahoo! agora.

[As partes desta mensagem que não continham texto foram removidas]



SUBJECT: Re: [ciencialist] Re: Por que massa atrai massa?
FROM: "Alvaro Augusto \(E\)" <alvaro@electraenergy.com.br>
TO: <ciencialist@yahoogrupos.com.br>
DATE: 12/01/2005 15:35

Resposta religiosa: por que os anjos celestiais garantem que isso aconteça, trabalhando incessantemente...

Resposta New Age: não é a matéria que atrai matéria, mas sim a sua mente que pensa que isso acontece...

Resposta antrópica: por que, caso contrário, não estaríamos aqui para fazer essa pergunta...

Resposta escolástica: por que Deus, em sua sabedoria, assim o quis...

Resposta newtoniana: deixo tais hipóteses a cargo dos meus leitores...

Resposta einsteiniana: não é a matéria que atrai matéria, mas sim a curvatura do espaço-tempo que faz com que a matéria se mova...

Resposta quântica: não é a matéria que atrai matéria, mas sim os grávitons que atuam como intermediários, transportando a força da gravidade...

[ ]s

Alvaro Augusto

----- Original Message -----
From: marcelo ferrari
To: ciencialist@yahoogrupos.com.br
Sent: Wednesday, January 12, 2005 1:40 PM
Subject: [ciencialist] Re: Por que massa atrai massa?


Emiliano,

Se "como" é o "porque" da ciencia, basta efetuar uma pequena manobra semantica a pergunta ficará assim: "Como massa atrai massa?".


marcelo ferrari.





E m i l i a n o C h e m e l l o <chemelloe@yahoo.com.br> wrote:
Em uma pergunta feita numa lista de discussão, foi pedido: "Por que massa atrai massa?". Eu respondi, com uma citação:

"Ninguém sabe porque as coisas são como são".
Richard Feynman

A ciência investiga 'como' as coisas funcionam e não 'porque' funcionam. Alguém concorda? Alguém discorda? Alguém não tá nem ai para isso? :-)

[ ] 's do Emiliano Chemello
emiliano@quimica.net
http://www.quimica.net/emiliano
http://www.ucs.br/ccet/defq/naeq

" Rien ne se perd, rien ne se crée,
tout se transforme."

Antoine Laurent de Lavoisier (químico francês, 1743 - 1794)


[As partes desta mensagem que não continham texto foram removidas]



##### ##### #####

Para saber mais visite
http://www.ciencialist.hpg.ig.com.br


##### ##### ##### #####


Yahoo! Grupos, um serviço oferecido por:



















function SearchComboBox() { if (document.form_combo.keyword.value.length==0){ alert("Por favor, digite algo."); return false; }else { document.form_combo.action ="http://br.rd.yahoo.com/SIG=12ap2paan/M=264105.3931087.6562589.1588051/D=brclubs/S=2137111528:HM/EXP=1105613876/A=2361264/R=0/SIG=11uaou2jn/*http://www.bondfaro.com/bondfaro/in/combosearch_in.jsp?sk=11"; } return true;} [input] [input] [input]

---------------------------------
Links do Yahoo! Grupos

Para visitar o site do seu grupo na web, acesse:
http://br.groups.yahoo.com/group/ciencialist/

Para sair deste grupo, envie um e-mail para:
ciencialist-unsubscribe@yahoogrupos.com.br

O uso que você faz do Yahoo! Grupos está sujeito aos Termos do Serviço do Yahoo!.



---------------------------------
Yahoo! Acesso Grátis - Internet rápida e grátis. Instale o discador do Yahoo! agora.

[As partes desta mensagem que não continham texto foram removidas]



##### ##### #####

Para saber mais visite
http://www.ciencialist.hpg.ig.com.br


##### ##### ##### #####


Yahoo! Grupos, um serviço oferecido por:







------------------------------------------------------------------------------
Links do Yahoo! Grupos

a.. Para visitar o site do seu grupo na web, acesse:
http://br.groups.yahoo.com/group/ciencialist/

b.. Para sair deste grupo, envie um e-mail para:
ciencialist-unsubscribe@yahoogrupos.com.br

c.. O uso que você faz do Yahoo! Grupos está sujeito aos Termos do Serviço do Yahoo!.



[As partes desta mensagem que não continham texto foram removidas]



SUBJECT: Re: [ciencialist] Re: Por que massa atrai massa?
FROM: TARCISIO BORGES <tbs97@fisica.ufpr.br>
TO: ciencialist@yahoogrupos.com.br
DATE: 12/01/2005 16:06

Mas daí fica fácil!!!

Campo:

Duas cargas de massa irão interagir através do campo gravitacional. Esta
interação será atrativa.

Partículas:

Duas cargas de massa irão interagir através de troca de partículas
chamadas grávitons. Esta interação será atrativa.

[]s
TARCISIO BORGES
tbs97@fisica.ufpr.br

On Wed, 12 Jan 2005, marcelo ferrari wrote:
>
> Emiliano,
> Se "como" é o "porque" da ciencia, basta efetuar uma pequena manobra
> semantica a pergunta ficará assim: "Como massa atrai massa?".
>
> marcelo ferrari.



SUBJECT: Muro curvado (placas tectônicas)
FROM: "brudna" <lrb@iq.ufrgs.br>
TO: ciencialist@yahoogrupos.com.br
DATE: 12/01/2005 16:11


Existe uma foto de um muro que ficou curvo devido ao movimento de
placas tectônicas na California.

Está disponívek neste link

http://epod.usra.edu/archive/epodviewer.php3?oid=225984

Tem chance de ser um hoax ou uma má interpretação?

Até
Luís Brudna
ps: esse deslocamento não tem relação direta com o terremoto na Ásia. :-)







SUBJECT: Re: [ciencialist] Re: Por que massa atrai massa?
FROM: "E m i l i a n o C h e m e l l o" <chemelloe@yahoo.com.br>
TO: <ciencialist@yahoogrupos.com.br>
DATE: 12/01/2005 16:33

Alvaro,

Na minha opinião, não compete a ciência descobrir todos os 'porquês' da
natureza. Outro exemplo é a atração e repulsão entre cargas. Cargas de mesmo
sinal se repelem. Já cargas de sinais contrários se atraem. Por que? A
ciência não sabe. Porém, construiu muito - uma consquência disso é a
tecnologia atual - em cima destas observações científicas.

Postei essa pergunta para desmistificar, de uma vez por todas, a idéia
de que a 'Ciência", e por consequência os "Cientistas", sabem tudo. Creio
que Feynman foi muito feliz na frase que estruturou.

[ ] 's do Emiliano Chemello
emiliano@quimica.net
http://www.quimica.net/emiliano
http://www.ucs.br/ccet/defq/naeq

" Rien ne se perd, rien ne se crée,
tout se transforme."

Antoine Laurent de Lavoisier (químico francês, 1743 - 1794)

----- Original Message -----
From: Alvaro Augusto (E)
To: ciencialist@yahoogrupos.com.br
Sent: Wednesday, January 12, 2005 3:35 PM
Subject: Re: [ciencialist] Re: Por que massa atrai massa?


Resposta religiosa: por que os anjos celestiais garantem que isso aconteça,
trabalhando incessantemente...

Resposta New Age: não é a matéria que atrai matéria, mas sim a sua mente que
pensa que isso acontece...

Resposta antrópica: por que, caso contrário, não estaríamos aqui para fazer
essa pergunta...

Resposta escolástica: por que Deus, em sua sabedoria, assim o quis...

Resposta newtoniana: deixo tais hipóteses a cargo dos meus leitores...

Resposta einsteiniana: não é a matéria que atrai matéria, mas sim a
curvatura do espaço-tempo que faz com que a matéria se mova...

Resposta quântica: não é a matéria que atrai matéria, mas sim os grávitons
que atuam como intermediários, transportando a força da gravidade...

[ ]s

Alvaro Augusto

----- Original Message -----
From: marcelo ferrari
To: ciencialist@yahoogrupos.com.br
Sent: Wednesday, January 12, 2005 1:40 PM
Subject: [ciencialist] Re: Por que massa atrai massa?


Emiliano,

Se "como" é o "porque" da ciencia, basta efetuar uma pequena manobra
semantica a pergunta ficará assim: "Como massa atrai massa?".


marcelo ferrari.





E m i l i a n o C h e m e l l o <chemelloe@yahoo.com.br> wrote:
Em uma pergunta feita numa lista de discussão, foi pedido: "Por que massa
atrai massa?". Eu respondi, com uma citação:

"Ninguém sabe porque as coisas são como são".
Richard Feynman

A ciência investiga 'como' as coisas funcionam e não 'porque'
funcionam. Alguém concorda? Alguém discorda? Alguém não tá nem ai para isso?
:-)

[ ] 's do Emiliano Chemello
emiliano@quimica.net
http://www.quimica.net/emiliano
http://www.ucs.br/ccet/defq/naeq

" Rien ne se perd, rien ne se crée,
tout se transforme."

Antoine Laurent de Lavoisier (químico francês, 1743 - 1794)


[As partes desta mensagem que não continham texto foram removidas]



##### ##### #####

Para saber mais visite
http://www.ciencialist.hpg.ig.com.br


##### ##### ##### #####


Yahoo! Grupos, um serviço oferecido por:



















function SearchComboBox() { if
(document.form_combo.keyword.value.length==0){ alert("Por favor, digite
algo."); return false; }else { document.form_combo.action
="http://br.rd.yahoo.com/SIG=12ap2paan/M=264105.3931087.6562589.1588051/D=br
clubs/S=2137111528:HM/EXP=1105613876/A=2361264/R=0/SIG=11uaou2jn/*http://www
.bondfaro.com/bondfaro/in/combosearch_in.jsp?sk=11"; } return true;} [input]
[input] [input]

---------------------------------
Links do Yahoo! Grupos

Para visitar o site do seu grupo na web, acesse:
http://br.groups.yahoo.com/group/ciencialist/

Para sair deste grupo, envie um e-mail para:
ciencialist-unsubscribe@yahoogrupos.com.br

O uso que você faz do Yahoo! Grupos está sujeito aos Termos do Serviço
do Yahoo!.



---------------------------------
Yahoo! Acesso Grátis - Internet rápida e grátis. Instale o discador do
Yahoo! agora.

[As partes desta mensagem que não continham texto foram removidas]



##### ##### #####

Para saber mais visite
http://www.ciencialist.hpg.ig.com.br


##### ##### ##### #####


Yahoo! Grupos, um serviço oferecido por:







----------------------------------------------------------------------------
--
Links do Yahoo! Grupos

a.. Para visitar o site do seu grupo na web, acesse:
http://br.groups.yahoo.com/group/ciencialist/

b.. Para sair deste grupo, envie um e-mail para:
ciencialist-unsubscribe@yahoogrupos.com.br

c.. O uso que você faz do Yahoo! Grupos está sujeito aos Termos do
Serviço do Yahoo!.



[As partes desta mensagem que não continham texto foram removidas]



##### ##### #####

Para saber mais visite
http://www.ciencialist.hpg.ig.com.br


##### ##### ##### #####


Yahoo! Grupos, um serviço oferecido por:








Links do Yahoo! Grupos

Para visitar o site do seu grupo na web, acesse:
http://br.groups.yahoo.com/group/ciencialist/

Para sair deste grupo, envie um e-mail para:
ciencialist-unsubscribe@yahoogrupos.com.br

O uso que você faz do Yahoo! Grupos está sujeito aos Termos do Serviço do
Yahoo!.




SUBJECT: Fw: Sistema de Informação de Vôo
FROM: "Cyberlander" <mrcyberlander@superig.com.br>
TO: <Undisclosed-Recipient:;>
DATE: 12/01/2005 18:44



+---------------------------------------------------------+
| |
| Amigos(as), |
| |
| A infraero esta dispinibilizando na internet o SIV |
| - Sistema de Informação de Vôo que para quem não |
| sabe são as televisões que estão no saguão do |
| aeroporto e informam partidas e chegadas, onde |
| podemos visualizar agora via internet em tempo real |
| o movimento de vôos de vários aeroportos |
| administrados pela infraero. |
| |
| Segue o link: |
| http://www.infraero.gov.br/pls/sivnet/siv |
| |
+---------------------------------------------------------+


[ ]'s
D.C.



CYBERLANDER

Ama a realidade que constróis,
que nem a morte deterá teu voo · ·




[As partes desta mensagem que não continham texto foram removidas]



SUBJECT: Hidratos de Metano
FROM: "Amauri Jr" <amaurijunior2@yahoo.com.br>
TO: <Conversa_de_Botequim@yahoogrupos.com.br>, <ciencialist@yahoogrupos.com.br>, <ciencia.2005@grupos.com.br>, <autoiniciacaodaconsciencia_plena@yahoogrupos.com.br>, <acropolis@yahoogrupos.com.br>
DATE: 12/01/2005 19:48


Hidratos de Metano
O combust�vel do futuro ou uma amea�a � humanidade?

Um gelo que queima. Esta seria a primeira descri��o da combina��o cristalina entre mol�culas de metano e mol�culas de �gua, encontrada em regi�es profundas do oceano. Os hidratos de metano j� s�o considerados, pelos cientistas, a principal fonte de energia para o s�culo XXI. A explora��o pode, entretanto, provocar a libera��o destes gases e causar
o maior efeito estufa j� visto no planeta.



No in�cio deste ano, a Japan National Oil Company come�ou a constru��o da primeira plataforma para extra��o de hidratos de metano do fundo do m�r C�spio. Diversas empresas norte-americanas j� investem milh�es de d�lares na pesquisa para a tecnologia de extra��o de metano do fundo do mar. Embora o material seja abundante, a extra��o n�o � um processo f�cil: esta mistura cristalina � muito inst�vel; v�rios acidentes j� aconteceram, na tentativa de encanar este g�s. O U.S. Geological Survey (USGS) estima que a quantidade de metano hidratado existente somente nas �guas norte-americanas chegam a 600 trilh�es de metros c�bicos de g�s, suficiente para abastecer toda a na��o (EUA) por mais de 2000 anos!

o que s�o os hidratos de metano?
Quando as bact�rias digerem a mat�ria org�nica, no fundo do mar, liberam mol�culas de CH4 (metano).

Estas mol�culas acabam "aprisionadas" por cristais de �gua, formando os hidratos ou, ainda, se combinam com o limo e o barro do fundo do oceano, formando bolhas de g�s entre densas camadas de barro.

Uma estrutura normal de hidrato de metano cont�m 46 mol�culas de �gua e 8 mol�culas de metano. Sua apar�ncia � como o gelo mas, entretanto, � est�vel somente a altas press�es e baixas temperaturas. N�o existe liga��o covalente entre o a �gua e o metano; o hidrato, quando se funde, libera �gua l�quida e g�s metano.



Antes de 1970, ningu�m sabia que hidratos de metano existiam no fundo do mar, embora estes hidratos n�o sejam raros: pelo contr�rio, eles est�o por toda a parte. Estudos recentes indicaram a presen�a de grandes dep�sitos submarinos de hidratos de metano em praticamente todos os oceanos, incluindo o litoral brasileiro!

Mais de 50% de todo o carbono existente no planeta est� no fundo do mar, sob a forma de hidratos de metano. � mais do que todas as reservas de materiais f�sseis, todos os seres vivos e todas as minas de carv�o somadas.

O "gelo de metano", como tamb�m s�o chamados os hidratos deste g�s, formam-se naturalmente nas regi�es profundas, de alta press�o e baixa temperatura, do oceano, mas geralmente fica enterrado sob o sedimento marino. O Golfo do M�xico � um dos poucos lugares one pode ser encontrado exposto no fundo do oceano. Embora tanto a �gua como o metano sejam incolores, muitos hidratos de metano s�o coloridos. Os hidratos encontrados no Golfo do M�xico s�o amarelo, laranja ou vermelhos. Os encontrados nas Bahamas s�o cinza ou azul. As cores devem-se � presen�a de bact�rias, minerais e outros gases que tamb�m s�o incorporados nos hidratos.

Os hidratos de metano se formaram, durante milh�es de anos, pela degrada��o da mat�ria org�nica, pelas bact�rias. Curiosamente, a abund�ncia relativa do Carbono-12 nos hidratos de metano � muito mais acentuada do que em amostras de carbono (org�nico ou mineral) do resto do planeta. Uma das explica��es � que as bact�rias exibem uma seletividade na hora de digerir o material org�nico, tendendo a "sequestrar" mais Carbono-12 do que seus outros is�topos.

Embora muitos considerem estes hidratos uma excelente fonte de energia, existem cientistas extremamente preocupados: a explora��o indevida ou mesmo um acidente natural, como um grande terremoto, pode vir a liberar grande parte deste g�s para a atmosfera. O metano tamb�m � um g�s que causa o efeito estufa; al�m disso, na atmosfera, ele se oxida e gera CO2. A temperatura do planeta, em uma situa��o como esta, iria aumentar drasticamente, as calotas polares iriam derreter e a vida iria se tornar mais dif�cil. Por isso nenhum pa�s, ainda, come�ou com for�a total a explora��o dos hidratos de metano.

Em 1984, cientistas suecos observaram a exist�ncia de "chamin�s" naturais no fundo do oceano. Havia o despreendimento constante de g�s. Somente mais tarde encotraram uma explica��o: o g�s era o metano, provindo de hidratos que estavam sendo "esmagados" pela fric��o entre duas placas tect�nicas. Estas correntes de bolhas s�o chamadas de "cold winds". Algumas vezes, a origem do g�s � a fus�o dos hidratos, quando estes est�o pr�ximos a regi�es, submarinas, com atividade vulc�nica - os chamados "hot winds". Nestes casos, n�o somente �gua e CH4 s�o liberados, mas tamb�m pequenas quantidades de H2S e NH3. A oxida��o destes compostos qu�micos em g�s carb�nico, sulfato e nitrato oferece um meio alternativo de sobreviv�ncia, para muitas esp�cies, mesmo na aus�ncia de oxig�nio. Estranhas criaturas vivem nas proximidades dos hidratos de metano. Entre elas, os tube worms, que formam col�nias gigantescas nas �reas de ocorr�ncia de hot winds.

Criaturas Bizarras
Um ambiente sem luz ou oxig�nio e sob uma enorme press�o. Embora pare�a totalmente in�spito para qualquer um de n�s este � o lar de estranhas criaturas, os Pogonophora, conhecidos como "tube worms", ou seja, "vermes tubulares". Observados, pela primeira vez, em 1900, na Indon�sia, hoje j� se conhecem mais de 80 esp�cies diferentes de Pogonophora, incluindo algumas que chegam a 1,5 metros de comprimento.
O nome - pogonophora - � a palavra grega para "portadores de barba", em alus�o aos pequenos tent�culos observados em muitas das esp�cies de tube worms.


Certas bact�rias desenvolvem uma rela��o simbi�tica com os tube worms, garantindo o seu sustento. As bact�rias vivem em um �rg�o do tube worm chamado trofosomo; de l�, as bact�rias oxidam os sulfetos que saem dos hot winds. A bact�ria obt�m energia atrav�s da oxida��o do enxofre, e utiliza-a para sintetizar mol�culas maiores de carbonos - o alimento dos tube worms.



Voc� sabia?
Uma das explica��es para o fen�meno no "Tri�ngulo das Bermudas", onde v�rios navios foram "sugados" para o fundo do mar, cita os hidratos de metano: uma s�bita libera��o de grande quantidade deste g�s seria suficiente para afundar um navio.
Obviamente, a descoberta destes hidratos ofereceu uma possibilidade sem refer�ncias na obten��o de energia para a humanidade. Al�m disso, a pitoresca biodiversidade encontrada nestas regi�es abriu um campo novo de estudos para os bi�logos - a vida, na aus�ncia de oxig�nio, pode apontar para outros modos de organiza��o de vida interplanet�ria. Por outro lado, revelou-se um perigo de cat�strofe: se estes gases forem liberados a humanidade estar� amea�ada. Um perigo que repousa, em sil�ncio, no fundo dos 7 mares...

� dificil de saber se, realmente, os hidratos de metano venham a se tornar uma fonte de combust�vel. De qualquer forma, os dias do petr�leo abundante est�o contados, e as na��es necessitar�o, em breve, novas fontes de energia. Os hidratos de metano podem ser esta fonte.


Saiba mais:
> Hidratos de gases no Golfo do M�xico
> Hidratos e o Tri�ngulo das Bermudas
> Cientistas da LSU descobrem Vermes Tubulares no Golfo
> Info sobre hidratos no U.S. Geological Survey
> Fotoss�ntes vs. Quimioss�ntese





[As partes desta mensagem que n�o continham texto foram removidas]



SUBJECT: Brilho metálico
FROM: "pedrolazaromoreira" <pedro_lazaro@portugalmail.pt>
TO: ciencialist@yahoogrupos.com.br
DATE: 12/01/2005 19:50


Olá a todos os membros do ciencialist.

venha por este meio pedir ajuda para me ajudarem a explicar com toda
a clareza necessária o porque dos metais apresentarem um brilho
caracteristico.

Abraço.

Pedro Moreira.

Já agora, quando uma solução de peróxido de hidrogénio contém 30
volume, qual é a sua concentração? Como se determina-se?





SUBJECT: Re: [ciencialist] Por que massa atrai massa?
FROM: "Prof. JC" <profjc2003@yahoo.com.br>
TO: <ciencialist@yahoogrupos.com.br>
DATE: 12/01/2005 20:09

Oi Chemello,

Eu respondi exatamente isso em uma lista de discussão, também recentemente.
Coincidências acontecem. :)

Abraços,
Prof. JC


----- Original Message -----
From: "E m i l i a n o C h e m e l l o" <chemelloe@yahoo.com.br>
To: <ciencialist@yahoogrupos.com.br>
Sent: Wednesday, January 12, 2005 9:00 AM
Subject: [ciencialist] Por que massa atrai massa?



Em uma pergunta feita numa lista de discussão, foi pedido: "Por que massa
atrai massa?". Eu respondi, com uma citação:

"Ninguém sabe porque as coisas são como são".
Richard Feynman

A ciência investiga 'como' as coisas funcionam e não 'porque' funcionam.
Alguém concorda? Alguém discorda? Alguém não tá nem ai para isso? :-)

[ ] 's do Emiliano Chemello
emiliano@quimica.net
http://www.quimica.net/emiliano
http://www.ucs.br/ccet/defq/naeq

" Rien ne se perd, rien ne se crée,
tout se transforme."

Antoine Laurent de Lavoisier (químico francês, 1743 - 1794)


[As partes desta mensagem que não continham texto foram removidas]



##### ##### #####

Para saber mais visite
http://www.ciencialist.hpg.ig.com.br


##### ##### ##### #####
Links do Yahoo! Grupos













SUBJECT: Re: Teoria das Cordas
FROM: Maria Natália <grasdic@hotmail.com>
TO: ciencialist@yahoogrupos.com.br
DATE: 12/01/2005 20:59


Manuel:
Só falta o endereço e que é
http://www2.uol.com.br/sciam/
A revista é mesmo muito boa para divulgação e iniciação
Um abraço
Maria Natália


--- Em ciencialist@yahoogrupos.com.br, Manuel Bulcão
<manuelbulcao@u...> escreveu
>
> --- Em ciencialist@yahoogrupos.com.br, "E m i l i a n o C h e m
e
> l l o" <chemelloe@y...> escreveu
> > Alguma sugestão da turma?
>
> Manuel: Na edição 20 da Scientific American Brasil (Janeiro de
2004)
> há um artigo de fácil compreensão sobre a teoria das
> cordas: "Descobrindo a Teoria das Cordas", de Nathan Jacob
Beskovits.
>
> []s
> Manuel Bulcão
>
> > ---
> > Roterdan" <roterdan.abreu@b...>
> > Para: Emiliano Chemello
> > Assunto:Curioso sobre as cordas
> > Mensagem:Senhores, gostaria de um texto simples (sei que não é
> fácil..) que
> > abordasse o tema sobre a teoria das cordas. Tenho interesse em
> conhecer um
> > pouco mais sobre o assunto, mas quero fazer uma incursão
gradativa
> no tema.
> >
> > Obrigado,
> >
> > Roterdan Abreu
> > Eng. Eletricista
> > PUC-RJ / 1980





SUBJECT: Re: [ciencialist] Re: Por que massa atrai massa?
FROM: "Alvaro Augusto \(E\)" <alvaro@electraenergy.com.br>
TO: <ciencialist@yahoogrupos.com.br>
DATE: 12/01/2005 21:32

Caro Emiliano,

Note que as minhas respostas não explicam nada, pois remetem o mistério para outras instâncias.

Eu concordo apenas parcialmente como você. A ciência deseja, sim, explicar o "porquê" das coisas. O problema é que não sabemos fazer isso, e temos que nos contentar com o "como". Mas a busca dos porquês é tão intensa que a teoria das supercordas tem exatamente esse objetivo: obter um sistema físico tão bem montado que não haja parâmetros livres, e que todas as cargas, massas e demais propriedades das partíciulas decorram naturamente, sem "ajustes" (como no Modelo Padrão).

Obs.: eu acabo de verificar quem, na minha mensagem anterior, escrevi errado todos os "porquês"... Devemos nos contentar com os "comos" mesmo...

[ ]s

Alvaro Augusto


----- Original Message -----
From: E m i l i a n o C h e m e l l o
To: ciencialist@yahoogrupos.com.br
Sent: Wednesday, January 12, 2005 4:33 PM
Subject: Re: [ciencialist] Re: Por que massa atrai massa?


Alvaro,

Na minha opinião, não compete a ciência descobrir todos os 'porquês' da
natureza. Outro exemplo é a atração e repulsão entre cargas. Cargas de mesmo
sinal se repelem. Já cargas de sinais contrários se atraem. Por que? A
ciência não sabe. Porém, construiu muito - uma consquência disso é a
tecnologia atual - em cima destas observações científicas.

Postei essa pergunta para desmistificar, de uma vez por todas, a idéia
de que a 'Ciência", e por consequência os "Cientistas", sabem tudo. Creio
que Feynman foi muito feliz na frase que estruturou.

[ ] 's do Emiliano Chemello
emiliano@quimica.net
http://www.quimica.net/emiliano
http://www.ucs.br/ccet/defq/naeq

" Rien ne se perd, rien ne se crée,
tout se transforme."

Antoine Laurent de Lavoisier (químico francês, 1743 - 1794)

----- Original Message -----
From: Alvaro Augusto (E)
To: ciencialist@yahoogrupos.com.br
Sent: Wednesday, January 12, 2005 3:35 PM
Subject: Re: [ciencialist] Re: Por que massa atrai massa?


Resposta religiosa: por que os anjos celestiais garantem que isso aconteça,
trabalhando incessantemente...

Resposta New Age: não é a matéria que atrai matéria, mas sim a sua mente que
pensa que isso acontece...

Resposta antrópica: por que, caso contrário, não estaríamos aqui para fazer
essa pergunta...

Resposta escolástica: por que Deus, em sua sabedoria, assim o quis...

Resposta newtoniana: deixo tais hipóteses a cargo dos meus leitores...

Resposta einsteiniana: não é a matéria que atrai matéria, mas sim a
curvatura do espaço-tempo que faz com que a matéria se mova...

Resposta quântica: não é a matéria que atrai matéria, mas sim os grávitons
que atuam como intermediários, transportando a força da gravidade...

[ ]s

Alvaro Augusto

----- Original Message -----
From: marcelo ferrari
To: ciencialist@yahoogrupos.com.br
Sent: Wednesday, January 12, 2005 1:40 PM
Subject: [ciencialist] Re: Por que massa atrai massa?


Emiliano,

Se "como" é o "porque" da ciencia, basta efetuar uma pequena manobra
semantica a pergunta ficará assim: "Como massa atrai massa?".


marcelo ferrari.





E m i l i a n o C h e m e l l o <chemelloe@yahoo.com.br> wrote:
Em uma pergunta feita numa lista de discussão, foi pedido: "Por que massa
atrai massa?". Eu respondi, com uma citação:

"Ninguém sabe porque as coisas são como são".
Richard Feynman

A ciência investiga 'como' as coisas funcionam e não 'porque'
funcionam. Alguém concorda? Alguém discorda? Alguém não tá nem ai para isso?
:-)

[ ] 's do Emiliano Chemello
emiliano@quimica.net
http://www.quimica.net/emiliano
http://www.ucs.br/ccet/defq/naeq

" Rien ne se perd, rien ne se crée,
tout se transforme."

Antoine Laurent de Lavoisier (químico francês, 1743 - 1794)



[As partes desta mensagem que não continham texto foram removidas]



SUBJECT: A moda das cotas "pegou"
FROM: "Prof. JC" <profjc2003@yahoo.com.br>
TO: <acropolis@yahoogrupos.com.br>, <Conversa_de_Botequim@yahoogrupos.com.br>, <ciencialist@yahoogrupos.com.br>
DATE: 12/01/2005 21:44

Cotas para os cursos de medicina
Lígia Formenti
Preocupado com a concentração de médicos nos grandes centros
urbanos, deixando regiões afastadas sem assistência, o Ministério da Saúde
quer criar um sistema de cotas para os cursos de medicina. A idéia, que será
discutida nos próximos dias com o Ministério da Educação, é reservar um
determinado número de vagas para os moradores dessas regiões mais isoladas.
A sugestão será debatida na definição dos critérios para a abertura de
novos cursos de medicina, suspensa pelo MEC por seis meses. Os dois
ministérios vão estabelecer juntos essas novas regras - a expectativa é de
que sejam divulgadas dentro de um mês. A proposta entrou em estudo diante da
constatação de que cursos de medicina sempre atraem candidatos de todo o
País. "Recentemente, um curso em Roraima foi aberto e 80% das vagas foram
preenchidas com estudantes de outras partes do País", constata o diretor do
Departamento da Educação na Saúde, Ricardo Ceccin. Ele diz que a grande
maioria dos estudantes, depois de concluído o curso, retorna para os grandes
centros. "As cotas seriam um mecanismo importante para quebrar esse ciclo.
Caso contrário, o problema da falta de profissionais dificilmente será
resolvido."






SUBJECT: Re: Água em fúria: tsunami
FROM: Maria Natália <grasdic@hotmail.com>
TO: ciencialist@yahoogrupos.com.br
DATE: 12/01/2005 22:27


José Renato:

Bom artigo. E se permites, né?
E a confusão deve ser da pororoca. Tu sabes o que é no Rio Amazonas,
ou era? Por aqui na Europa é o clima que está mudando e até creio
que teremos inundações em fins de Janeiro e Fevereiro. Ontem houve
abalo de terra 3.8 Richter no Alentejo...Qualquer dia é aqui no Tejo
e na falha de Benavente que há descraça.
Por agora aqui são desde 5ºC negativos, a 5ºC em Lisboa . Vós de fio
dental...a suar as estupinhas.
Boas férias
Maria Natália


--- Em ciencialist@yahoogrupos.com.br, José Renato <jrma@t...>
escreveu
> AMBIENTE BRASIL
>
> Tsunamis
>
> De origem japonesa - tsunami designa ondas oceânicas de grande
altura. Embora sejam erroneamente denominadas de ondas de maré, as
tsunamis não são causadas por influência das forças de maré (forças
astronômicas de atração do Sol e da Lua).
>
> Tsunamis são ondas de grande energia geradas por abalos sísmicos.
Têm sua origem em maremotos, erupções vulcânicas e nos diversos
tipos de movimentos das placas do fundo submarino.
>
> Portanto uma boa definição para a tsunami seria uma onda sísmica
que se propaga no oceano. Historicamente, é no Oceano Pacífico onde
ocorreram a maioria das tsunamis, por ser uma área cercada por
atividades vulcânicas e freqüentes abalos sísmicos. Ao norte do
Oceano Pacífico, desde o Japão até o Alasca, existe uma faixa de
maior incidência de maremotos e erupções vulcânicas que originariam
as tsunamis mais freqüentes do nosso planeta.
>
> Talvez a tsunami mais famosa tenha sido a provocada pela explosão
vulcânica da Ilha de Krakatoa no Oceano Pacífico em 26 e 27 de
agosto de 1883. A tsunami resultante atingiu as ilhas da Indonésia
com ondas de até 35 metros de altura.
>
> As tsunamis ao se propagarem no oceano possuem comprimento da
ordem de 150 a 200 km de extensão e apenas 1 metro de altura.
Portanto, em alto mar elas são quase imperceptíveis. Entretanto, ao
se aproximar de zonas costeiras mais rasas, a redução da velocidade,
devido ao atrito com fundo do seu comprimento, porém a energia
continua a mesma. Conseqüentemente, a altura da onda aumenta
bastante em pouco tempo. Neste ponto, ela pode atingir 10, 20 e até
30 metros de altura, em função de sua energia e da distância do
epicentro da tsunami.
>
> Na recentemente levantada hipótese sobre o perigo de um maremoto
de grandes proporções, ele seria tão catastrófico quanto maior for
presumida explosão vulcânica nas Ilhas Canárias, local onde foi
detectada importante atividade sísmica no subsolo.
>
> Uma analogia a esse processo seria uma panela de pressão que tem a
sua válvula reguladora obstruída enquanto aumenta o calor interno
gerado pelo fogo. A pressão interna vai aumentando proporcionalmente
ao acúmulo de energia potencial. Este processo tem continuidade até
que ocorra uma ruptura em algum ponto da estrutura da panela
redundando em uma explosão, ou seja, na liberação instantânea de
grande quantidade de energia.
>
> No caso das Ilhas Canárias foi observado um aumento da atividade
sísmica/vulcânica no interior da ilha. Como a mesma estava inerte a
várias dezenas de anos, o topo do cone vulcânico, que é a própria
ilha, se consolidou de tal forma que se extinguiu o respiro ou
válvula de alívio da pressão interna do vulcão. Assim, quanto mais
sinais ela der de atividade vulcânica no seu interior maior será o
risco de haver uma erupção vulcânica de grandes proporções. O
tamanho da onda tsunami gerada será proporcional à quantidade de
energia transmitida ao mar no momento da erupção.
>
> Por outro lado, uma erupção vulcânica não é um evento comum e, se
levarmos em conta outros fatores, veremos que a probabilidade de
formação de uma onda tsunami destruidora é pequena.
>
> Outro fator a ser considerado é a distância do litoral brasileiro,
especificamente dos estados do Rio Grande do Norte, Ceará, Maranhão,
Piauí, Pará e Amapá, em relação à Ilhas Canárias. São
aproximadamente 4,500 km/h, o que equivaleria a 8 horas de percurso
até chegar ao litoral brasileiro.
>
> Assim, quanto maior for a distância entre a origem (epicentro) e o
litoral de impacto, maior será a perda de sua intensidade por
espalhamento e mesmo dissipação de sua energia. Outro fator de
reflexão é que quanto menor for a profundidade das zonas por onde a
onda propaga maior vai ser a redução de sua energia pelo atrito com
o fundo submarino.
>
> Se somarmos a probabilidade e os registros históricos de erupções
e/ou abalos sísmicos em ilhas do Oceano Atlântico, que são mínimos,
veremos que as chances de ocorrer um acidente ambiental de grandes
proporções são baixas.
>
> Desta forma, antes do Brasil, Portugal, Norte da África e o
arquipélago de Cabo Verde serão as vítimas potenciais devido à
proximidade do epicentro da eventual explosão vulcânica, recebendo
diretamente o impacto da onda de grande altura.
>
> Por outro lado se existe a probabilidade é preciso ter cuidado de
alterar para as possíveis conseqüências do fenômeno. A conjunção de
fatores intervenientes pode provocar estragos catastróficos, daí a
importância de que a população seja informada e que as autoridades
competentes tomem as devidas precauções. Um bom exemplo desse tipo
de política de seguranças é o desenvolvimento através de informações
de satélite pela Organização Meteorológica Mundial - OMM.
>
> Devido a freqüência da ocorrência de tsunamis no Pacífico, existe
uma rede internacional de sismógrafos ao longo do cinturão de fogo
que altera para a formação de qualquer onda catastrófica. Como
resultado dessa iniciativa nenhuma morte foi contabilizada com a
passagem de uma tsunami no Havaí em 1957. Já a tsunami de 1946, com
altura inferior à de 1957, causou inúmeras vítimas fatais pela
ausência de um sistema de alerta.
>
> Portanto, medidas preventivas são muito menos onerosas e possíveis
de serem tomadas do que medidas corretivas, muito mais dolorosas. O
medo é gerado pela ignorância, o respeito é gerado pelo conhecimento.
>
> Fonte: Revista ECO 21 - www.eco21.com.br - outubro, 2001.
>
> ...................................................................
..................................
>
> Água em Fúria
>
> Tsunami é a palavra japonesa pela qual são mundialmente conhecidos
os maremotos, uma catástrofe de extraordinária violência que se
abate sobre zonas costeiras. A causa mais freqüente para os tsunamis
são terremotos no assoalho marítimo; também podem decorrer de
erupções vulcânicas submarinas ou de explosões causadas por gases
acumulados no subsolo do oceano.
>
> Podem, ainda vir associados com um terremoto terrestre (foi o que
ocorreu em Lisboa, no século XVII: trinta minutos após um gigantesco
abalo sísmico, ergueu-se no mar uma onda de dez metros de altura,
colhendo milhares de pessoas que aviam fugido do interior da cidade
para a costa.
>
> A ação do tsunami é rápida e aterradora. Quando ele começa no meio
do oceano, forma ondas de cerca de seis metros, qua avançam em alta
velocidade. Ao aproximar-se da costa, a velocidade da onda diminui.
O processo se assemelha ao de uma parada brusca, que projeta a água
para a frente - e aí pode alcançar até 30 metros adiante.
>
>
>
> Os maiores, os piores, os últimos:
>
> 1707: Japão (após terremoto; 30 mil mortos)
>
> 1755: Lisboa, Portugal (após terremotos; total: 60 mil mortos)
>
> 1883: Krakatoa, Indonésia (36 mil mortos)
>
> 1896: Honchu, Japão (27 mil mortos)
>
> 1976: Mindanao, Filipinas (8 mil mortos)
>
> 1992: Nicarágua (100 mortos)
>
> 1998: Papua-Nova Guiné (3 mil mortos)
>
> 2001: Arequipa, Peru (20 mortos)
>
> 2004: Países Asiáticos e Africanos (Índia, Indonésia, Sri Lanka,
Maldivas, Malásia, Tailândia, Bangladesh e Mianma, na Ásia; e
Somália, Tanzânia, Seichelas e Quênia, na costa leste da África -
aproximadamente 150 mil mortos)
>
>
>
>
> Fonte: Como cuidar da nossa água. Coleção Entenda e Aprenda. BEI.
São Paulo-SP, 2003.
>
>
>
> <http://www.ambientebrasil.com.br/composer.php3?
base=./agua/salgada/index.html&conteudo=./agua/salgada/artigos/tsunam
is.html >
>
>
> [As partes desta mensagem que não continham texto foram removidas]





SUBJECT: A EVOLUÇÃO DO (PRE)CONCEITO DE DEFICIÊNCIA
FROM: "Amauri Jr" <amaurijunior2@yahoo.com.br>
TO: <Lidenskab@yahoogrupos.com.br>, "Laracna" <laracna@pop.com.br>, "kathleen_mel" <kathleen_mel@uol.com.br>, <Euthymia@yahoogrupos.com.br>, <elio_mollo_k@hotmail.com>, <deficienteeficiente@yahoogrupos.com.br>, <Historia_Antiga@yahoogrupos.com.br>, <acropolis@yahoogrupos.com.br>, <autoiniciacaodaconsciencia_plena@yahoogrupos.com.br>, <CIADEF@yahoogrupos.com.br>, <ciencialist@yahoogrupos.com.br>, <Conversa_de_Botequim@yahoogrupos.com.br>, <grasdic@yahoogroups.co.uk>, "Meiriely" <meiriely@fisiomail.com.br>, "Renata" <renifoci@hotmail.com>, <Sexualidadedodef@yahoogrupos.com.br>, <sementesdefraternidade@yahoogrupos.com.br>, <transconhecimento@yahoogrupos.com.br>, "Lucimary Vargas" <sangalli@uai.com.br>, <voadores@yahoogrupos.com.br>, <cafeconsciencia@yahoogrupos.com.br>
DATE: 12/01/2005 22:28


A EVOLU��O DO (PRE)CONCEITO DE DEFICI�NCIA

--------------------------------------------------------------------------------

Vaneza Cauduro Peranzoni
Soraia Napole�o Freitas
.
O presente artigo procura explicitar a evolu��o hist�rica da conceitua��o da defici�ncia sob a �tica do estigma e da discrimina��o que configuram (e, por vezes, ainda configuram!) as p�ginas de nossa hist�ria, mais precisamente quanto �s oportunidades educacionais (negadas e proporcionadas) e aos servi�os oferecidos � clientela dita especial (hoje chamadas de Portadores de Necessidades Educativas Especiais pela sociedade que, em certa �poca, pregou integra��o mas segregou e, hoje, prega inclus�o mas ainda exclui.

Palavras-Chave - defici�ncia - hist�ria - evolu��o


--------------------------------------------------------------------------------

Como seres humanos, encontramo-nos em constante evolu��o e sede de saber. Deste modo, no contexto educacional, educador e educandos tornam-se sujeitos hist�ricos de busca, de inquieta��es, operacionalizando a supera��o de desafios nos mais diferentes sentidos da constru��o do conhecimento.
Sob essa �tica, a proposta deste artigo objetiva apresentar como transcorreu a evolu��o hist�rica da conceitua��o da defici�ncia, discutida desde os pioneiros at� os dias atuais. Assim, observamos que, por meio da hist�ria, as PNEE v�m sendo consideradas de diferentes maneiras, sempre relacionadas aos valores sociais, morais, filos�ficos, �ticos e religiosos de cada �poca, isto �, historicamente as PNEE foram e ainda s�o consideradas conforme as diversas concep��es de homem e de sociedade nas diferentes culturas.

Para compreendermos a situa��o atual das PNEE, faz-se necess�rio um resgate hist�rico-evolutivo de como essas pessoas foram tratadas. Analisando a literatura existente que versa sobre este tema, fica-nos uma certeza: a quase constante situa��o de segrega��o e exclus�o que perpassou a cria��o e a evolu��o da humanidade e que perpassa at� os dias atuais.

A literatura aponta que, na Roma antiga, muitas crian�as com defici�ncia ou que nasciam com alguma malforma��o eram abandonadas em pequenos cestos nas margens do Rio Tibre. Por outro lado, Roma tamb�m teve no seio de seu comando muitos imperadores que apresentavam algum tipo de defici�ncia, como: Caio J�lio C�sar, �pio Cl�udio, Cl�udio I e Nero. S� que suas defici�ncias eram "escondidas" e ignoradas pelo povo, devido ao poder que estes possu�am em suas m�os para governar. Portanto, sendo detentores do poder, tinham a chance de n�o serem segregados. Al�m desses imperadores, Roma teve muitos outros imperadores com defici�ncias, que s�o: Galba, que apresentava problemas nas m�os e nos p�s; Othon, com deforma��o f�sica nas pernas; e Vit�lio, que possu�a grave les�o nas pernas (BEZ, (s/d) In: Revista Viv�ncia).

De acordo com BEZ (s/d), na antiga Gr�cia, a defici�ncia era totalmente ignorada, n�o possu�a nenhum espa�o, considerando que a beleza e o culto ao corpo e � perfei��o f�sica eram tidos como condi��o sine qua non para a participa��o em sociedade, e uma pessoa com defici�ncia, considerada ent�o feia, malformada, era, por conseq��ncia, praticamente uma ofensa ao povo. Assim, quando nascia uma crian�a que apresentava alguma defici�ncia esta devia passar por um "conselho" que definiria se deveria viver ou morrer. A crian�a destinada � morte era conduzida ao Apothetai, o que significa dep�sito. Mas, como em Roma, na Gr�cia houve muitas personalidades que apresentavam alguma defici�ncia, entre eles: Homero, que era cego; Alexandre, o Grande, que sofria de epilepsia; e Dem�stenes, que sofria de gagueira.

BEZ (s/d) coloca ainda que "no tempo de Arist�teles, havia, em Atenas, 20 mil pessoas portadoras de defici�ncias, que correspondiam a aproximadamente 20% de toda a popula��o, recebendo pens�o do Estado", p. 08. Esse �ndice era considerado alto, por isso, talvez, os romanos e gregos tentavam de todas as formas impedir que crian�as consideradas deficientes permanecessem vivas. Ao mesmo tempo, podemos dizer que estes povos serviam de "f�brica" de defici�ncia, considerando que, com suas guerras constantes, muitos soldados voltavam mutilados, tornando-se, assim, portadores de defici�ncia f�sica.

Ent�o, muito seguramente, podemos colocar que a sociedade n�o s� eliminava as pessoas consideradas diferentes, mas tamb�m as fazia ficar nesta condi��o e perpetuava a quest�o da elimina��o, num movimento c�clico e perene de segrega��o, em analogia ao "darwinismo social" e � "eugenia", considerando que "os portadores de defici�ncia foram vistos como vil�es, portadores de taras heredit�rias, que representavam um perigo para a continuidade da esp�cie" (MENDES s/d, p. 08).

Na Idade Antiga, acreditava-se que o "comportamento diferente" da PNEE era conseq��ncia de for�as sobrenaturais, sugerindo a cren�a em uma origem demon�aca das doen�as e, mais especificamente, da defici�ncia mental. Com o advento da Idade M�dia, intensificou-se a cren�a no sobrenatural. O homem passou a ser submetido a poderes invis�veis, tanto para o bem quanto para o mal; a pr�tica de magia e as rela��es com o dem�nio eram parte do cotidiano. Em conseq��ncia disso, segundo AMIRALIAN (1986), a sociedade agia distintamente com as PNEE, conforme o tipo de excepcionalidade apresentada: os psic�ticos e epil�pticos eram considerados possu�dos pelo dem�nio; alguns estados de transe eram aceitos como possess�o divina, e os cegos eram reverenciados como videntes, profetas e adivinhos.

Em torno do s�culo XVIII, come�am aparecer explica��es naturalistas para o comportamento dos deficientes. Segundo PESSOTTI (1984, p.72), "o desenvolvimento da ci�ncia permite questionar os dogmas religiosos e come�am a surgir estudos mais sistem�ticos na �rea m�dica visando explicar tais comportamentos". Os estudos na �rea da medicina permitiram verificar que muitas defici�ncias eram resultantes de les�es e disfun��es no organismo. Dessa forma, a medicina come�a a ganhar um forte espa�o, e as PNEE passam a ser vistas como objeto e clientela de estudo desta �rea. Isso n�o significou ainda uma redu��o na discrimina��o social de que eram v�timas, mas, sim, um marco no que se refere ao atendimento �s suas necessidades b�sicas de sa�de apenas. Assim, podemos dizer que h� uma continuidade da segrega��o aos deficientes. Com o objetivo de oferecer tratamento m�dico e aliviar a sobrecarga da fam�lia e da sociedade, as PNEE eram mandadas para asilos e hospitais, na companhia de prostitutas, loucos e delinq�entes.

Com a Revolu��o Industrial, o panorama da concep��o de defici�ncia muda um pouco seu foco, considerando que esse per�odo retrata um processo de transforma��es econ�micas e sociais, caracterizadas pela acelera��o do processo produtivo e pela consolida��o da produ��o capitalista, abrindo caminho para o processo de produ��o em s�rie, que exige a escolariza��o em massa de seus trabalhadores. Surge, ent�o, uma nova parcela da popula��o que passou a ser considerada menos eficiente, ou seja, deficiente, aqueles que n�o conseguiam aprender conforme as normas escolares institu�das.

Observa-se que, at� o in�cio do s�culo XIX, a defici�ncia estava associada � incapacidade, � id�ia de inutilidade e depend�ncia, e n�o havia nenhuma preocupa��o com a mudan�a desse quadro. O abandono e a elimina��o das PNEE eram atitudes comuns e n�o eram fundamentadas a preceitos morais e �ticos que regiam as rela��es sociais das diferentes �pocas.

Somente no final do s�culo XIX e no in�cio do s�culo XX � que se criaram institui��es para os deficientes mentais moderados e profundos, com a finalidade de servir de asilo para que eles "incomodassem" o menos poss�vel.

Se observarmos a evolu��o hist�rica da defici�ncia no que se refere ao atendimento educacional, a �rea denominada de Educa��o Especial expandiu-se, no Brasil, com a cria��o de entidades filantr�picas assistenciais e especializadas destinadas � popula��o das classes menos favorecidas. Ao lado dessas institui��es, surgiram cl�nicas e escolas privadas para o atendimento das PNEE das classes mais altas. Em rela��o ao atendimento, notamos um n�mero elevado de profissionais ligados ao modelo m�dico da defici�ncia, no qual esta � vista como um "problema" do indiv�duo e, por isso, o pr�prio deficiente ter� que mudar para se adaptar � sociedade ou ter� que ser mudado por profissionais atrav�s da reabilita��o ou cura.

Esse modelo m�dico da defici�ncia � um dos grandes respons�veis pela resist�ncia da sociedade em aceitar mudar as suas estruturas e atitudes para a inclus�o das PNEE, pois durante anos tratou estas pessoas com fins m�dicos e cl�nicos e n�o pedag�gicos. Na verdade, o que se almeja � o fim pedag�gico, o da inclus�o, em que profissionais trabalhem com as capacidades e habilidades das PNEE. A sociedade deve ser aberta a todos e n�o deve segregar e apresentar barreiras a ningu�m.

A escola seria uma das institui��es que poderia quebrar com muitos tabus, mas, ao contr�rio, � permeada de preconceitos e ju�zos pr�vios sobre os alunos e suas fam�lias. De acordo HELLER (1970, p. 17), "a vida cotidiana � a do homem inteiro; ou seja, o homem participa na vida cotidiana com todos os aspectos de sua individualidade, de sua personalidade". Infelizmente, na maioria das vezes, a individualidade e a personalidade das PNEE n�o s�o respeitadas e nem levadas em conta pela sociedade. Este indiv�duo se torna v�tima de muitos preconceitos. Mas o que vem a ser preconceito?

O preconceito � um fen�meno conhecido h� muito tempo, mas seu objeto e seu conceito t�m variado historicamente, torna-se muito dif�cil defini-lo. Conforme Jahoda e Ackerman (1969) apud CROCHIK (1997, p.29) o

... preconceito � considerado por (...) como um pr�-julgamento que predisp�e o indiv�duo a ter atitudes frente ao objeto em quest�o, e este pr�-julgamento, por sua vez, � determinado pela rela��o entre o indiv�duo e aquilo que a cultura lhe oferece para se expressar e ser expressada por ele.

Mais adiante, os mesmos autores colocam que o preconceito

... ... ... representa uma subcategoria do pr�-conceito, ap�ia-se no pensar estereotipado, sem confundir-se com um ou com outro. Do ponto de vista psicol�gico, o preconceito � (...) uma atitude de hostilidade nas rela��es interpessoais, dirigida contra um grupo inteiro ou contra os indiv�duos pertencentes a ele, e que preenche uma fun��o irracional definida dentro da personalidade.

Em cada �poca, a PNEE, como foi relatado anteriormente, foi v�tima de um tipo de preconceito, conforme os valores e os costumes do pr�prio per�odo.

Os preconceitos, segundo Kant apud CROCHIK (1997), s�o incutidos nos homens, os quais s�o impedidos e se impedem de pensar por si pr�prios. O autor v� a experi�ncia e a raz�o como fundamentais para o conhecimento e o preconceito como seu maior obst�culo. O preconceito se remete � domina��o e, quando necess�rio, � proposta de elimina��o do desconhecido para se manter aquilo que j� � conhecido.
A vida cotidiana e o cotidiano escolar t�m muitos preconceitos, devido a muitos fatores. Sabemos que a vida cotidiana � heterog�nea e hier�rquica, e o homem j� nasce inserido em sua cotidianidade. Com o amadurecimento, ele adquire todas as habilidades para a vida cotidiana da sociedade. Esse amadurecimento come�a sempre por grupos. Mas, muitas vezes, a PNEE � privada deste conv�vio em grupos, sendo segregada, exclu�da da sociedade por causa das suas diferen�as e limita��es.

Segundo HELLER (1970, p. 20), "a vida cotidiana est� no centro do acontecer hist�rico: � a verdadeira "ess�ncia" da subst�ncia social". E o indiv�duo � sempre um ser particular e gen�rico, simultaneamente. N�o se deve esquecer disso no cotidiano escolar.

Mas, infelizmente, o cotidiano escolar � o espa�o onde se concretiza a produ��o do fracasso escolar (e n�o o contr�rio), onde n�o se d� a aten��o necess�ria para aquele ser "diferente". Contudo, devemos romper esta muralha de preconceitos, para construir uma escola comprometida com as PNEE.

Ser capaz de se elevar � esfera do humano-gen�rico, suspendendo a vida cotidiana e suas in�meras reivindica��es, e a� ser capaz de transformar seu pr�prio cotidiano, � essencial se pretendemos ser sujeitos de nossa pr�pria historia. Apresento, aqui, para reflex�o, esta cita��o de COLLARES & MOYS�S (1996, p. 260): "Se, por�m, pretendemos ser agentes efetivos de transforma��o social, sujeitos da hist�ria, fica o desafio de sermos capazes de nos infiltrar na vida cotidiana, quebrar seu sistema de preconceitos e retomar a cotidianidade em outra dire��o."

Portanto, buscamos uma escola e uma sociedade inclusivas, a que todos tenham acesso e onde sejam respeitados os limites de cada um, sendo dado espa�o a todas as pessoas para que elas possam crescer e transformar cada dia mais o seu meio, rompendo com muitas das barreiras que lhes s�o impostas.


--------------------------------------------------------------------------------

BIBLIOGRAFIA

AMIRALIAN, Maria L�cia. Psicologia do Excepcional. v. 3. S�o Paulo: EPU, 1986.

ARIES, Philippe e DUBY, Georges (Dir.) Do ventre materno ao testamento. In:. Hist�ria da vida privada. Do Imp�rio Romano ao ano mil. V. I. S�o Paulo: Companhia das Letras, 1990. p. 23-43.

___. A individualiza��o da crian�a. In: Hist�ria da vida privada. Da Renascen�a ao S�culo das Luzes. v. III. S�o Paulo: Companhia das Letras, 1991. p. 311-329.

BEZ, Volnei Martins. O Ocaso dos mitos. In: Revista Viv�ncia. Funda��o Catarinense de Educa��o Especial. n. 14, p. 5-9. S�o Jos�, SC.

COLLARES, C. & MOYS�S, M. A. Preconceitos no cotidiano escolar: ensino e medicaliza��o. S�o Paulo: Cortez, 1996.

CROCHIK, Jos� Leon. Preconceito: indiv�duo e cultura. S�o Paulo: Robe Editorial, 1997.

GOYOS, Celso (org.) Temas em Educa��o Especial. S�o Carlos, SP: Editora da UFSCar,1996.

HELLER, Agnes. O cotidiano e a hist�ria. Rio de Janeiro: Paz e Terra, 1970.

___. Sociologia de la vida cotidiana. Barcelona. Edicione Pen�nsula, 1991.

JANNUZZI, Gilberta. A luta pela educa��o do deficiente mental no Brasil. Campinas, SP: Editora Autores Associados, 1992.

MANTOAN, Maria Tereza. Compreendendo a defici�ncia mental: novos caminhos educacionais. S�o Paulo: Scipione, 1989.

MAZZOTTA, Marcos J. S. Educa��o Especial no Brasil: Hist�ria e pol�ticas p�blicas. S�o Paulo: Cortez, 1996.

MENDES, Enic�ia Gon�alves. Hist�ria da Educa��o Especial para Portadores de Defici�ncia Mental no Brasil. S/d. Mimeo.

___. Construindo a concep��o de defici�ncia: implica��es no processo de forma��o de educadores. S/d. Mimeo.

PESSOTTI, Isa�as. Defici�ncia Mental: da supersti��o � ci�ncia. S�o Paulo: T. A. Queiroz/EDUSP, 1984.

SASSAKI, Romeu Kazumi. Inclus�o. Rio de Janeiro: WVA, 1997.


[As partes desta mensagem que n�o continham texto foram removidas]



SUBJECT: Re: [ciencialist] Re: Por que massa atrai massa?
FROM: "Prof. JC" <profjc2003@yahoo.com.br>
TO: <ciencialist@yahoogrupos.com.br>
DATE: 12/01/2005 22:52

Marcelo,

"Como" não é o "porque" da ciência. "Como" é o "porquê" da metafísica.

Abraços,
Prof. JC


----- Original Message -----
From: "marcelo ferrari" <emailferrari@yahoo.com.br>
To: <ciencialist@yahoogrupos.com.br>
Sent: Wednesday, January 12, 2005 1:40 PM
Subject: [ciencialist] Re: Por que massa atrai massa?



Emiliano,

Se "como" é o "porque" da ciencia, basta efetuar uma pequena manobra
semantica a pergunta ficará assim: "Como massa atrai massa?".


marcelo ferrari.





E m i l i a n o C h e m e l l o <chemelloe@yahoo.com.br> wrote:
Em uma pergunta feita numa lista de discussão, foi pedido: "Por que massa
atrai massa?". Eu respondi, com uma citação:

"Ninguém sabe porque as coisas são como são".
Richard Feynman

A ciência investiga 'como' as coisas funcionam e não 'porque' funcionam.
Alguém concorda? Alguém discorda? Alguém não tá nem ai para isso? :-)

[ ] 's do Emiliano Chemello
emiliano@quimica.net
http://www.quimica.net/emiliano
http://www.ucs.br/ccet/defq/naeq

" Rien ne se perd, rien ne se crée,
tout se transforme."

Antoine Laurent de Lavoisier (químico francês, 1743 - 1794)


[As partes desta mensagem que não continham texto foram removidas]



##### ##### #####

Para saber mais visite
http://www.ciencialist.hpg.ig.com.br


##### ##### ##### #####


Yahoo! Grupos, um serviço oferecido por:



















function SearchComboBox() { if
(document.form_combo.keyword.value.length==0){ alert("Por favor, digite
algo."); return false; }else { document.form_combo.action
="http://br.rd.yahoo.com/SIG=12ap2paan/M=264105.3931087.6562589.1588051/D=brclubs/S=2137111528:HM/EXP=1105613876/A=2361264/R=0/SIG=11uaou2jn/*http://www.bondfaro.com/bondfaro/in/combosearch_in.jsp?sk=11";
} return true;} [input] [input] [input]

---------------------------------
Links do Yahoo! Grupos

Para visitar o site do seu grupo na web, acesse:
http://br.groups.yahoo.com/group/ciencialist/

Para sair deste grupo, envie um e-mail para:
ciencialist-unsubscribe@yahoogrupos.com.br

O uso que você faz do Yahoo! Grupos está sujeito aos Termos do Serviço do
Yahoo!.



---------------------------------
Yahoo! Acesso Grátis - Internet rápida e grátis. Instale o discador do
Yahoo! agora.

[As partes desta mensagem que não continham texto foram removidas]



##### ##### #####

Para saber mais visite
http://www.ciencialist.hpg.ig.com.br


##### ##### ##### #####
Links do Yahoo! Grupos













SUBJECT: Re: Brilho metálico
FROM: "rmtakata" <rmtakata@altavista.net>
TO: ciencialist@yahoogrupos.com.br
DATE: 13/01/2005 04:43


--- Em ciencialist@yahoogrupos.com.br, "pedrolazaromoreira"
> venha por este meio pedir ajuda para me ajudarem a explicar
> com toda a clareza necessária o porque dos metais
> apresentarem um brilho caracteristico.

Isso eh facil. Basta apenas um pequeno background de fisica quantica
de nivel medio.

Os metais sao caracterizados por possuirem atomos circundados por uma
nuvem de eletrons - seus eletrons mais externos estao apenas
fracamente ligados ao nucleo e com isso podem fluir livremente de um
atomos para outro - ao cabo esses eletrons sao compartilhados por
todos os nucleos, formando a chamada ligacao metalica. Essa fluidez
dos eletrons externos confere as caracteristicas dos metais:
condutibilidade eletrica (por motivos obvios), maleabilidade - os
atomos podem deslizar um em relacao aos outros sem q. ocorra a quebra
das ligacoes (se ele perde contato com um atomo, imediatamente liga-se
ao outro -- alias, em uma amostra metalica, mal se pode falar em
atomos individuais), condutibilidade termica: os eletrons externos
rapidamente distribuem a energia por todo o metal.

Qto ao brilho metalico, ele nada mais eh do q. um brilho semelhante ao
espelho. E o q. o espelho faz? Nada mais, nada menos do q. refletir de
volta (quase) toda luz q. incide sobre ele - soh q. de forma mais ou
menos ordenada - se a luz refletida eh espalhada em todas as direcoes
temos a coloracao branca. Bem, aqui entram os conhecimentos de fisica
quantica: qdo um eletron absorve energia, ele pula para uma orbita
mais externa, ao devolver essa energia ele volta para sua orbita
original. Os eletrons podem absorver apenas luz em certas frequencias
- pois cada frequencia corresponde a uma determinada quantidade de
energia de cada foton que compoe a luz e os eletrons absorvem apenas
certas quantidade de energia ao mudar de orbita (ele nao pode absorver
energias de valores intermediarios). Entao a distribuicao dos eletrons
pelas camadas em torno do nucleo (reveja a parte dos modelos atomicos)
determina que frequencias de luz um determinado material absorve e
reemite - isto eh, reflete. E como a cor depende das propriedades de
reflexao (e transmissao) da luz incidente, essa distribuicao de
eletrons vai determinar a colocaracao caracteristica da substancia, do
composto, do material. Lembre-se de q. falamos q. os eletrons mais
externos distribuem-se quase livremente pelos nucleos dos atomos no
metal. Outra caracteristica quantica: dois eletrons nao podem ocupar
ao mesmo tempo uma mesma configuracao quantica - o q. significa q. nao
podem ter ao mesmo tempo numeros quanticos iguais para camada,
subnivel, orbital e spin (principio da exclusao de Pauli). Porem o
compartilhamento dos eletrons mais externos preencheria todas as
configuracoes possiveis, jah q. sao em quantidades finitas (existem
apenas dois estados possiveis de spin, por exemplo). A presenca de
diversos eletrons faz com q. a camada mais externa se subdivida em
muitos e muitos subniveis bastante proximos entre si. Essa divisao
fina faz com q. diversas frequencias de ondas possam ser absorvidas -
e reemitidas. A estruturacao ordenada dos atomos no metal faz com q. a
absorcao e reemissao da luz se dej de modo relativamente ordenado - o
q. somado a reflexao de praticamente todos os comprimentos de onda (ao
menos na faixa do visivel) garante uma reflexao semelhante ao do
espelho (se polirmos uma superficie metalica termos de fato um bom
espelho), isto eh, um brilho metalico.

[O mercurio, q. eh liquido em cor ambiente, tem uma menor ordenacao em
suas moleculas, o q. faz com q. adquira uma coloracao esbranquicada -
dispersa um pouco a luz refletida.
O ouro e o cobre, em funcao de algumas peculiaridades de sua
distribuicao eletronica (e teremos q. recorrer um pouco a relatividade
einsteniana para explicar o porque), perde parte de sua capacidade de
refletir comprimentos de ondas mais curtos (frequencias maiores),
adquirindo tons amarelados e avermelhados.]

> Já agora, quando uma solução de peróxido de hidrogénio
> contém 30 volume, qual é a sua concentração? Como se
> determina-se?

30 volumes significa q., em volume, H2O2 corresponde a 30% da solucao.
(Usa-se volume, para horror dos puristas tecnicos em metrologia, uma
vez q. % eh adimensional, mas se se indicasse apenas 30% nao se
poderia saber se era 30% em massa, 30% em moleculas ou outra coisa -
cada qual significando uma coisa diferente.)

[]s,

Roberto Takata





SUBJECT: Re: [ciencialist] Re: Brilho metálico
FROM: "Prof. JC" <profjc2003@yahoo.com.br>
TO: <ciencialist@yahoogrupos.com.br>
DATE: 13/01/2005 05:42

Pô Takata, você enrolou o moço. :)

Você disse que a reflexão dos metais é semelhante à dos espelhos, mas os
bons espelhos são justamente feitos de metal (nitrato de prata, por exemplo,
depositado sobre o vidro). Enrolão! :) (brincadeirinha)

Se inglês não for problema aqui também tem uma boa explicação para a
reflexão em metais: http://phya.snu.ac.kr/~sk_eah/sk_eah/whymirror.html

Abraços,
Prof. JC



----- Original Message -----
From: "rmtakata" <rmtakata@altavista.net>
To: <ciencialist@yahoogrupos.com.br>
Sent: Thursday, January 13, 2005 4:43 AM
Subject: [ciencialist] Re: Brilho metálico




--- Em ciencialist@yahoogrupos.com.br, "pedrolazaromoreira"
> venha por este meio pedir ajuda para me ajudarem a explicar
> com toda a clareza necessária o porque dos metais
> apresentarem um brilho caracteristico.

Isso eh facil. Basta apenas um pequeno background de fisica quantica
de nivel medio.

Os metais sao caracterizados por possuirem atomos circundados por uma
nuvem de eletrons - seus eletrons mais externos estao apenas
fracamente ligados ao nucleo e com isso podem fluir livremente de um
atomos para outro - ao cabo esses eletrons sao compartilhados por
todos os nucleos, formando a chamada ligacao metalica. Essa fluidez
dos eletrons externos confere as caracteristicas dos metais:
condutibilidade eletrica (por motivos obvios), maleabilidade - os
atomos podem deslizar um em relacao aos outros sem q. ocorra a quebra
das ligacoes (se ele perde contato com um atomo, imediatamente liga-se
ao outro -- alias, em uma amostra metalica, mal se pode falar em
atomos individuais), condutibilidade termica: os eletrons externos
rapidamente distribuem a energia por todo o metal.

Qto ao brilho metalico, ele nada mais eh do q. um brilho semelhante ao
espelho. E o q. o espelho faz? Nada mais, nada menos do q. refletir de
volta (quase) toda luz q. incide sobre ele - soh q. de forma mais ou
menos ordenada - se a luz refletida eh espalhada em todas as direcoes
temos a coloracao branca. Bem, aqui entram os conhecimentos de fisica
quantica: qdo um eletron absorve energia, ele pula para uma orbita
mais externa, ao devolver essa energia ele volta para sua orbita
original. Os eletrons podem absorver apenas luz em certas frequencias
- pois cada frequencia corresponde a uma determinada quantidade de
energia de cada foton que compoe a luz e os eletrons absorvem apenas
certas quantidade de energia ao mudar de orbita (ele nao pode absorver
energias de valores intermediarios). Entao a distribuicao dos eletrons
pelas camadas em torno do nucleo (reveja a parte dos modelos atomicos)
determina que frequencias de luz um determinado material absorve e
reemite - isto eh, reflete. E como a cor depende das propriedades de
reflexao (e transmissao) da luz incidente, essa distribuicao de
eletrons vai determinar a colocaracao caracteristica da substancia, do
composto, do material. Lembre-se de q. falamos q. os eletrons mais
externos distribuem-se quase livremente pelos nucleos dos atomos no
metal. Outra caracteristica quantica: dois eletrons nao podem ocupar
ao mesmo tempo uma mesma configuracao quantica - o q. significa q. nao
podem ter ao mesmo tempo numeros quanticos iguais para camada,
subnivel, orbital e spin (principio da exclusao de Pauli). Porem o
compartilhamento dos eletrons mais externos preencheria todas as
configuracoes possiveis, jah q. sao em quantidades finitas (existem
apenas dois estados possiveis de spin, por exemplo). A presenca de
diversos eletrons faz com q. a camada mais externa se subdivida em
muitos e muitos subniveis bastante proximos entre si. Essa divisao
fina faz com q. diversas frequencias de ondas possam ser absorvidas -
e reemitidas. A estruturacao ordenada dos atomos no metal faz com q. a
absorcao e reemissao da luz se dej de modo relativamente ordenado - o
q. somado a reflexao de praticamente todos os comprimentos de onda (ao
menos na faixa do visivel) garante uma reflexao semelhante ao do
espelho (se polirmos uma superficie metalica termos de fato um bom
espelho), isto eh, um brilho metalico.

[O mercurio, q. eh liquido em cor ambiente, tem uma menor ordenacao em
suas moleculas, o q. faz com q. adquira uma coloracao esbranquicada -
dispersa um pouco a luz refletida.
O ouro e o cobre, em funcao de algumas peculiaridades de sua
distribuicao eletronica (e teremos q. recorrer um pouco a relatividade
einsteniana para explicar o porque), perde parte de sua capacidade de
refletir comprimentos de ondas mais curtos (frequencias maiores),
adquirindo tons amarelados e avermelhados.]

> Já agora, quando uma solução de peróxido de hidrogénio
> contém 30 volume, qual é a sua concentração? Como se
> determina-se?

30 volumes significa q., em volume, H2O2 corresponde a 30% da solucao.
(Usa-se volume, para horror dos puristas tecnicos em metrologia, uma
vez q. % eh adimensional, mas se se indicasse apenas 30% nao se
poderia saber se era 30% em massa, 30% em moleculas ou outra coisa -
cada qual significando uma coisa diferente.)

[]s,

Roberto Takata





##### ##### #####

Para saber mais visite
http://www.ciencialist.hpg.ig.com.br


##### ##### ##### #####
Links do Yahoo! Grupos













SUBJECT: Re: [ciencialist] Por que massa atrai massa?
FROM: "E m i l i a n o C h e m e l l o" <chemelloe@yahoo.com.br>
TO: <ciencialist@yahoogrupos.com.br>
DATE: 13/01/2005 10:33

Olá JC,

Que bom que pensamos igual. Coincidência mesmo. Este estigma que a
ciência/cientista é dono da verdade tem que acabar. Existem coisas ditas
'fundamentais' dentro da ciência. Um exemplo é a atração as massas. Outra é
a atração/repulsão entre cargas. E antes do Big Bang? Perguntará alguém. Não
sabemos.

Eu elenco algumas palavras que os cientistas devem ter cuidado ao fazer
uso: "é", "sempre", "nunca".

[ ] 's do Emiliano Chemello
emiliano@quimica.net
http://www.quimica.net/emiliano
http://www.ucs.br/ccet/defq/naeq

" Rien ne se perd, rien ne se crée,
tout se transforme."

Antoine Laurent de Lavoisier (químico francês, 1743 - 1794)

----- Original Message -----
From: Prof. JC
To: ciencialist@yahoogrupos.com.br
Sent: Wednesday, January 12, 2005 8:09 PM
Subject: Re: [ciencialist] Por que massa atrai massa?


Oi Chemello,

Eu respondi exatamente isso em uma lista de discussão, também recentemente.
Coincidências acontecem. :)

Abraços,
Prof. JC


----- Original Message -----
From: "E m i l i a n o C h e m e l l o" <chemelloe@yahoo.com.br>
To: <ciencialist@yahoogrupos.com.br>
Sent: Wednesday, January 12, 2005 9:00 AM
Subject: [ciencialist] Por que massa atrai massa?



Em uma pergunta feita numa lista de discussão, foi pedido: "Por que massa
atrai massa?". Eu respondi, com uma citação:

"Ninguém sabe porque as coisas são como são".
Richard Feynman

A ciência investiga 'como' as coisas funcionam e não 'porque' funcionam.
Alguém concorda? Alguém discorda? Alguém não tá nem ai para isso? :-)

[ ] 's do Emiliano Chemello
emiliano@quimica.net
http://www.quimica.net/emiliano
http://www.ucs.br/ccet/defq/naeq

" Rien ne se perd, rien ne se crée,
tout se transforme."

Antoine Laurent de Lavoisier (químico francês, 1743 - 1794)


[As partes desta mensagem que não continham texto foram removidas]



##### ##### #####

Para saber mais visite
http://www.ciencialist.hpg.ig.com.br


##### ##### ##### #####
Links do Yahoo! Grupos













##### ##### #####

Para saber mais visite
http://www.ciencialist.hpg.ig.com.br


##### ##### ##### #####


Yahoo! Grupos, um serviço oferecido por:

São Paulo Rio de Janeiro Curitiba Porto Alegre Belo Horizonte Brasília





Links do Yahoo! Grupos

Para visitar o site do seu grupo na web, acesse:
http://br.groups.yahoo.com/group/ciencialist/

Para sair deste grupo, envie um e-mail para:
ciencialist-unsubscribe@yahoogrupos.com.br

O uso que você faz do Yahoo! Grupos está sujeito aos Termos do Serviço do
Yahoo!.




SUBJECT: Re: [ciencialist] Por que massa atrai massa?
FROM: "Alvaro Augusto \(E\)" <alvaro@electraenergy.com.br>
TO: <ciencialist@yahoogrupos.com.br>
DATE: 13/01/2005 12:23

Cabe notar que o comentário de Feynman reflete a visão que ele tinha do mundo, na tradição iniciada por Dirac, e não é necessariamente compartilhada por todos. Feynman não poderia pensar de outra forma. Caso contrário ele não conseguiria explicar a ninguém a sua interpretação da mecânica quântica, baseada nas integrais de percurso. Se alguém sobreviveu a Schrödinger, a Heinsenberg e a Dirac, corre sério risco de sofrer um colapso ao chegar a Feynman. Para ele, a probabilidade de um elétron passar por uma fenda, por exemplo, é a soma das probabilidades do elétron percorrer todos os caminhos possíveis, inclusive um caminho que passa reto pela fenda, inclusive um caminho que passa pela galáxia de Andrômeda, inclusive um caminho que não passa pela fenda! (na verdade, não se trata da "soma das probabilidades", mas sim da soma das "amplitudes de probabilidades", mas vamos deixar esses detalhes técnicos de lado).

O incrível é que as integrais de Feynman dão resultado corretos, embora sejam muito mais usadas na teoria quântica dos campos do que na teoria quântica não relativística. Mas nós não sabemos porque elas funcionam, daí o argumento de Feynman. Muita gente, contudo, pensa de maneira platônica ("a verdade está lá fora") e não pragmática ("a verdade é o que conseguimos obter). Roger Penrose, por exemplo, é um platônico, enquanto Feynman foi um grande pragmático.

[ ]s

Alvaro Augusto


----- Original Message -----
From: E m i l i a n o C h e m e l l o
To: ciencialist@yahoogrupos.com.br
Sent: Thursday, January 13, 2005 10:33 AM
Subject: Re: [ciencialist] Por que massa atrai massa?


Olá JC,

Que bom que pensamos igual. Coincidência mesmo. Este estigma que a
ciência/cientista é dono da verdade tem que acabar. Existem coisas ditas
'fundamentais' dentro da ciência. Um exemplo é a atração as massas. Outra é
a atração/repulsão entre cargas. E antes do Big Bang? Perguntará alguém. Não
sabemos.

Eu elenco algumas palavras que os cientistas devem ter cuidado ao fazer
uso: "é", "sempre", "nunca".

[ ] 's do Emiliano Chemello
emiliano@quimica.net
http://www.quimica.net/emiliano
http://www.ucs.br/ccet/defq/naeq

" Rien ne se perd, rien ne se crée,
tout se transforme."

Antoine Laurent de Lavoisier (químico francês, 1743 - 1794)

----- Original Message -----
From: Prof. JC
To: ciencialist@yahoogrupos.com.br
Sent: Wednesday, January 12, 2005 8:09 PM
Subject: Re: [ciencialist] Por que massa atrai massa?


Oi Chemello,

Eu respondi exatamente isso em uma lista de discussão, também recentemente.
Coincidências acontecem. :)

Abraços,
Prof. JC


----- Original Message -----
From: "E m i l i a n o C h e m e l l o" <chemelloe@yahoo.com.br>
To: <ciencialist@yahoogrupos.com.br>
Sent: Wednesday, January 12, 2005 9:00 AM
Subject: [ciencialist] Por que massa atrai massa?



Em uma pergunta feita numa lista de discussão, foi pedido: "Por que massa
atrai massa?". Eu respondi, com uma citação:

"Ninguém sabe porque as coisas são como são".
Richard Feynman

A ciência investiga 'como' as coisas funcionam e não 'porque' funcionam.
Alguém concorda? Alguém discorda? Alguém não tá nem ai para isso? :-)

[ ] 's do Emiliano Chemello
emiliano@quimica.net
http://www.quimica.net/emiliano
http://www.ucs.br/ccet/defq/naeq

" Rien ne se perd, rien ne se crée,
tout se transforme."

Antoine Laurent de Lavoisier (químico francês, 1743 - 1794)


[As partes desta mensagem que não continham texto foram removidas]


[As partes desta mensagem que não continham texto foram removidas]



SUBJECT: Re: [ciencialist] Por que massa atrai massa?
FROM: "Eurico Ferreira de Souza Jr." <caodejah@yahoo.com.br>
TO: ciencialist@yahoogrupos.com.br
DATE: 13/01/2005 12:34


[E]> a atração gravitacional é a velocidade da luz, parada.

[]s

Eurico





_\|/_

---------------------------------
Yahoo! Acesso Grátis - Internet rápida e grátis. Instale o discador do Yahoo! agora.

[As partes desta mensagem que não continham texto foram removidas]



SUBJECT: Re: Por que massa atrai massa?
FROM: marcelo ferrari <emailferrari@yahoo.com.br>
TO: ciencialist@yahoogrupos.com.br
DATE: 13/01/2005 12:53

Eurico,

Você está fazendo piada (ironia) ou está falando em algo que vc pensa?










"Eurico Ferreira de Souza Jr." <caodejah@yahoo.com.br> wrote:

[E]> a atração gravitacional é a velocidade da luz, parada.

[]s

Eurico





_\|/_

---------------------------------
Yahoo! Acesso Grátis - Internet rápida e grátis. Instale o discador do Yahoo! agora.

[As partes desta mensagem que não continham texto foram removidas]



##### ##### #####

Para saber mais visite
http://www.ciencialist.hpg.ig.com.br


##### ##### ##### #####


Yahoo! Grupos, um serviço oferecido por:



















function SearchComboBox() { if (document.form_combo.keyword.value.length==0){ alert("Por favor, digite algo."); return false; }else { document.form_combo.action ="http://br.rd.yahoo.com/SIG=12ad2ag8v/M=264105.3931087.6562589.1588051/D=brclubs/S=2137111528:HM/EXP=1105713296/A=2361264/R=0/SIG=11uaou2jn/*http://www.bondfaro.com/bondfaro/in/combosearch_in.jsp?sk=11"; } return true;} [input] [input] [input]

---------------------------------
Links do Yahoo! Grupos

Para visitar o site do seu grupo na web, acesse:
http://br.groups.yahoo.com/group/ciencialist/

Para sair deste grupo, envie um e-mail para:
ciencialist-unsubscribe@yahoogrupos.com.br

O uso que você faz do Yahoo! Grupos está sujeito aos Termos do Serviço do Yahoo!.



---------------------------------
Yahoo! Acesso Grátis - Internet rápida e grátis. Instale o discador do Yahoo! agora.

[As partes desta mensagem que não continham texto foram removidas]



SUBJECT: Re: [ciencialist] Re: Por que massa atrai massa?
FROM: "Eurico Ferreira de Souza Jr." <caodejah@yahoo.com.br>
TO: ciencialist@yahoogrupos.com.br
DATE: 13/01/2005 15:28



marcelo ferrari <emailferrari@yahoo.com.br> wrote:
Eurico,

Você está fazendo piada (ironia) ou está falando em algo que vc pensa?\

[E]> quanticamente, as duas opções são válidas. :D

"e=mc2", lembra? tudo é energia, e essa energia pode estar correndo a 300 mil km/s ou "fazendo peso"










"Eurico Ferreira de Souza Jr." <caodejah@yahoo.com.br> wrote:

[E]> a atração gravitacional é a velocidade da luz, parada.

[]s

Eurico





_\|/_

---------------------------------
Yahoo! Acesso Grátis - Internet rápida e grátis. Instale o discador do Yahoo! agora.

[As partes desta mensagem que não continham texto foram removidas]



##### ##### #####

Para saber mais visite
http://www.ciencialist.hpg.ig.com.br


##### ##### ##### #####


Yahoo! Grupos, um serviço oferecido por:



















function SearchComboBox() { if (document.form_combo.keyword.value.length==0){ alert("Por favor, digite algo."); return false; }else { document.form_combo.action ="http://br.rd.yahoo.com/SIG=12ad2ag8v/M=264105.3931087.6562589.1588051/D=brclubs/S=2137111528:HM/EXP=1105713296/A=2361264/R=0/SIG=11uaou2jn/*http://www.bondfaro.com/bondfaro/in/combosearch_in.jsp?sk=11"; } return true;} [input] [input] [input]

---------------------------------
Links do Yahoo! Grupos

Para visitar o site do seu grupo na web, acesse:
http://br.groups.yahoo.com/group/ciencialist/

Para sair deste grupo, envie um e-mail para:
ciencialist-unsubscribe@yahoogrupos.com.br

O uso que você faz do Yahoo! Grupos está sujeito aos Termos do Serviço do Yahoo!.



---------------------------------
Yahoo! Acesso Grátis - Internet rápida e grátis. Instale o discador do Yahoo! agora.

[As partes desta mensagem que não continham texto foram removidas]



##### ##### #####

Para saber mais visite
http://www.ciencialist.hpg.ig.com.br


##### ##### ##### #####



Yahoo! Grupos, um serviço oferecido por: São Paulo Rio de Janeiro Curitiba Porto Alegre Belo Horizonte Brasília

---------------------------------
Links do Yahoo! Grupos

Para visitar o site do seu grupo na web, acesse:
http://br.groups.yahoo.com/group/ciencialist/

Para sair deste grupo, envie um e-mail para:
ciencialist-unsubscribe@yahoogrupos.com.br

O uso que você faz do Yahoo! Grupos está sujeito aos Termos do Serviço do Yahoo!.




_\|/_
__________________________________________________
Converse com seus amigos em tempo real com o Yahoo! Messenger
http://br.download.yahoo.com/messenger/

[As partes desta mensagem que não continham texto foram removidas]



SUBJECT: Re: [ciencialist] Por que massa atrai massa?
FROM: "Alvaro Augusto \(E\)" <alvaro@electraenergy.com.br>
TO: <ciencialist@yahoogrupos.com.br>
DATE: 13/01/2005 17:11

E por que a luz tem velocidade, afinal? Por que os fótons não conseguem ficar parados no lugar? E será que a velocidade da luz é a mesma coisa que a velocidade dos fótons?

[ ]s

Alvaro Augusto

----- Original Message -----
From: Eurico Ferreira de Souza Jr.
To: ciencialist@yahoogrupos.com.br
Sent: Thursday, January 13, 2005 12:34 PM
Subject: Re: [ciencialist] Por que massa atrai massa?



[E]> a atração gravitacional é a velocidade da luz, parada.

[]s

Eurico





_\|/_

---------------------------------
Yahoo! Acesso Grátis - Internet rápida e grátis. Instale o discador do Yahoo! agora.

[As partes desta mensagem que não continham texto foram removidas]



[As partes desta mensagem que não continham texto foram removidas]



SUBJECT: consciência leve
FROM: "murilo filo" <avalanchedrive@hotmail.com>
TO: ciencialist@yahoogrupos.com.br, forum-ciencia@yahoogrupos.com.br
DATE: 13/01/2005 17:27


Olha só que coisa mais linda! ( revista do MIT )
Onde está o Mesquitão?
E há tanta gente procurando a consciência em cristais e/ou aminoácidos, e
etc! Êste inglês, pelo menos, já ficou desconfiado...
Há também gente que não vê nenhuma novidade nêstes conceitos. ;7
abr/Murilo SP

The Unobserable Mind
A leading British philosipher is skeptical that neurobiology can tell us
anything about self-consciousness. By Roger Scruton.
http://www.technologyreview.com/articles/05/02/issue/review_mind.asp?trk=nl
http://www.technologyreview.com/articles/05/02/issue/review_mind.asp?trk=nl




SUBJECT: Re: Por que massa atrai massa?
FROM: marcelo ferrari <emailferrari@yahoo.com.br>
TO: ciencialist@yahoogrupos.com.br
DATE: 13/01/2005 17:41

Boa pergunta, e a resposta.




"Alvaro Augusto (E)" <alvaro@electraenergy.com.br> wrote: E por que a luz tem velocidade, afinal? Por que os fótons não conseguem ficar parados no lugar? E será que a velocidade da luz é a mesma coisa que a velocidade dos fótons?

[ ]s

Alvaro Augusto

----- Original Message -----
From: Eurico Ferreira de Souza Jr.
To: ciencialist@yahoogrupos.com.br
Sent: Thursday, January 13, 2005 12:34 PM
Subject: Re: [ciencialist] Por que massa atrai massa?



[E]> a atração gravitacional é a velocidade da luz, parada.

[]s

Eurico





_\|/_

---------------------------------
Yahoo! Acesso Grátis - Internet rápida e grátis. Instale o discador do Yahoo! agora.

[As partes desta mensagem que não continham texto foram removidas]



[As partes desta mensagem que não continham texto foram removidas]



##### ##### #####

Para saber mais visite
http://www.ciencialist.hpg.ig.com.br


##### ##### ##### #####


Yahoo! Grupos, um serviço oferecido por:



















function SearchComboBox() { if (document.form_combo.keyword.value.length==0){ alert("Por favor, digite algo."); return false; }else { document.form_combo.action ="http://br.rd.yahoo.com/SIG=12a4fuu0l/M=264105.3931087.6562589.1588051/D=brclubs/S=2137111528:HM/EXP=1105729907/A=2361264/R=0/SIG=11uaou2jn/*http://www.bondfaro.com/bondfaro/in/combosearch_in.jsp?sk=11"; } return true;} [input] [input] [input]

---------------------------------
Links do Yahoo! Grupos

Para visitar o site do seu grupo na web, acesse:
http://br.groups.yahoo.com/group/ciencialist/

Para sair deste grupo, envie um e-mail para:
ciencialist-unsubscribe@yahoogrupos.com.br

O uso que você faz do Yahoo! Grupos está sujeito aos Termos do Serviço do Yahoo!.





---------------------------------
Yahoo! Acesso Grátis - Internet rápida e grátis. Instale o discador do Yahoo! agora.

[As partes desta mensagem que não continham texto foram removidas]



SUBJECT: Re: [ciencialist] Re: Por que massa atrai massa?
FROM: "Eurico Ferreira de Souza Jr." <caodejah@yahoo.com.br>
TO: ciencialist@yahoogrupos.com.br
DATE: 13/01/2005 18:05




"Alvaro Augusto (E)" <alvaro@electraenergy.com.br> wrote: E por que a luz tem velocidade, afinal?

[E]> imagine um receptor de fm (rádio) sintonizado numa determinada frequência. Outras transmissões enviadas em frequência diversa são ignoradas pelo receptor, né? Nosso universo está sintonizado na frequência da "velocidade da luz - matéria", ou ainda, é o que acontece nessa "frequência existencial". Talvez existam energias se manifestando em infinitas outras frequências (velocidades e atributos físicos), inperceptíveis neste universo.

Por que os fótons não conseguem ficar parados no lugar?

[E]> eles podem! ficam "presos" fazendo um elétron subir um subnível na eletrosfera. lembra?

E será que a velocidade da luz é a mesma coisa que a velocidade dos fótons?

[E]> sim. fóton = unidade de luz.

[]s

Eurico



_\|/_

---------------------------------
Yahoo! Acesso Grátis - Internet rápida e grátis. Instale o discador do Yahoo! agora.

[As partes desta mensagem que não continham texto foram removidas]



SUBJECT: Re: [ciencialist] Re: Por que massa atrai massa?
FROM: "Alvaro Augusto \(E\)" <alvaro@electraenergy.com.br>
TO: <ciencialist@yahoogrupos.com.br>
DATE: 13/01/2005 19:36

----- Original Message -----
From: Eurico Ferreira de Souza Jr.
To: ciencialist@yahoogrupos.com.br
Sent: Thursday, January 13, 2005 6:05 PM
Subject: Re: [ciencialist] Re: Por que massa atrai massa?


> "Alvaro Augusto (E)" <alvaro@electraenergy.com.br> wrote: E por que a luz
tem velocidade, afinal?

> [E]> imagine um receptor de fm (rádio) sintonizado numa determinada
frequência. Outras transmissões enviadas em frequência diversa são ignoradas
pelo
> receptor, né? Nosso universo está sintonizado na frequência da "velocidade
da luz - matéria", ou ainda, é o que acontece nessa "frequência
existencial". Talvez
> existam energias se manifestando em infinitas outras frequências
(velocidades e atributos físicos), inperceptíveis neste universo.

Acho que não formulei muito bem a pergunta. Era para ser: "por que a luz
precisa ter uma velocidade?"

> Por que os fótons não conseguem ficar parados no lugar?

> [E]> eles podem! ficam "presos" fazendo um elétron subir um subnível na
eletrosfera. lembra?

Não é bem assim. Antes de serem emitidos, os fótons não existem. Eles não
estão dentro do elétron esperando para escapar.

> E será que a velocidade da luz é a mesma coisa que a velocidade dos
fótons?

> [E]> sim. fóton = unidade de luz.

Eu não sou especialista em QED, mas me lembro de ter lido em algum lugar que
os fótons podem ter várias velocidades, embora a velocidade da luz seja
invariante.

[ ]s

Alvaro Augusto




SUBJECT: Re: [ciencialist] Re: Por que massa atrai massa?
FROM: "Silvio Cordeiro" <scordeiro@terra.com.br>
TO: <ciencialist@yahoogrupos.com.br>
DATE: 13/01/2005 21:53

Signore Marcel:


Seu humor ao dizer que Henri, o touromáquico genoves que em suas pesquisas
no Fermi descobriu que a luz obedece à sinalização de trânsito sideral,
parando nos semáforos para permitir sem maiores problemas a passagem de uma
galaxia grávida, envolta em seu diáfano talar de matéria negra,torna-o uma
pessoa que merece levar 75 chibatatas nas plantas dos pés, todas as sextras
feiras menos no Radamã.Fosse eu Sultão das Arábias ou mesmo um régugo de
qualquer aldeia maori, tudo por puro despeito e inveja por sua eructação
flactuosa.

Que ódio!!!!!

carlos o mais pior dos ruinsmente péssimos..
----- Original Message -----
From: "marcelo ferrari" <emailferrari@yahoo.com.br>
To: <ciencialist@yahoogrupos.com.br>
Sent: Thursday, January 13, 2005 12:53 PM
Subject: [ciencialist] Re: Por que massa atrai massa?



Eurico,

Você está fazendo piada (ironia) ou está falando em algo que vc pensa?










"Eurico Ferreira de Souza Jr." <caodejah@yahoo.com.br> wrote:

[E]> a atração gravitacional é a velocidade da luz, parada.

[]s

Eurico





_\|/_

---------------------------------
Yahoo! Acesso Grátis - Internet rápida e grátis. Instale o discador do
Yahoo! agora.

[As partes desta mensagem que não continham texto foram removidas]



##### ##### #####

Para saber mais visite
http://www.ciencialist.hpg.ig.com.br


##### ##### ##### #####


Yahoo! Grupos, um serviço oferecido por:



















function SearchComboBox() { if
(document.form_combo.keyword.value.length==0){ alert("Por favor, digite
algo."); return false; }else { document.form_combo.action
="http://br.rd.yahoo.com/SIG=12ad2ag8v/M=264105.3931087.6562589.1588051/D=brclubs/S=2137111528:HM/EXP=1105713296/A=2361264/R=0/SIG=11uaou2jn/*http://www.bondfaro.com/bondfaro/in/combosearch_in.jsp?sk=11";
} return true;} [input] [input] [input]

---------------------------------
Links do Yahoo! Grupos

Para visitar o site do seu grupo na web, acesse:
http://br.groups.yahoo.com/group/ciencialist/

Para sair deste grupo, envie um e-mail para:
ciencialist-unsubscribe@yahoogrupos.com.br

O uso que você faz do Yahoo! Grupos está sujeito aos Termos do Serviço do
Yahoo!.



---------------------------------
Yahoo! Acesso Grátis - Internet rápida e grátis. Instale o discador do
Yahoo! agora.

[As partes desta mensagem que não continham texto foram removidas]



##### ##### #####

Para saber mais visite
http://www.ciencialist.hpg.ig.com.br


##### ##### ##### #####
Links do Yahoo! Grupos












SUBJECT: RE: [ciencialist] Re: Brilho metálico
FROM: "murilo filo" <avalanchedrive@hotmail.com>
TO: ciencialist@yahoogrupos.com.br
DATE: 13/01/2005 22:07

BANG!... BANG!... ( MATOU! MATOU BONITO.)

>From: "rmtakata" <rmtakata@altavista.net>
>Reply-To: ciencialist@yahoogrupos.com.br
>To: ciencialist@yahoogrupos.com.br
>Subject: [ciencialist] Re: Brilho metálico
>Date: Thu, 13 Jan 2005 06:43:22 -0000
>
>
>--- Em ciencialist@yahoogrupos.com.br, "pedrolazaromoreira"
> > venha por este meio pedir ajuda para me ajudarem a explicar
> > com toda a clareza necessária o porque dos metais
> > apresentarem um brilho caracteristico.
>
>Isso eh facil. Basta apenas um pequeno background de fisica quantica
>de nivel medio.
>
>Os metais sao caracterizados por possuirem atomos circundados por uma
>nuvem de eletrons - seus eletrons mais externos estao apenas
>fracamente ligados ao nucleo e com isso podem fluir livremente de um
>atomos para outro - ao cabo esses eletrons sao compartilhados por
>todos os nucleos, formando a chamada ligacao metalica. Essa fluidez
>dos eletrons externos confere as caracteristicas dos metais:
>condutibilidade eletrica (por motivos obvios), maleabilidade - os
>atomos podem deslizar um em relacao aos outros sem q. ocorra a quebra
>das ligacoes (se ele perde contato com um atomo, imediatamente liga-se
>ao outro -- alias, em uma amostra metalica, mal se pode falar em
>atomos individuais), condutibilidade termica: os eletrons externos
>rapidamente distribuem a energia por todo o metal.
>
>Qto ao brilho metalico, ele nada mais eh do q. um brilho semelhante ao
>espelho. E o q. o espelho faz? Nada mais, nada menos do q. refletir de
>volta (quase) toda luz q. incide sobre ele - soh q. de forma mais ou
>menos ordenada - se a luz refletida eh espalhada em todas as direcoes
>temos a coloracao branca. Bem, aqui entram os conhecimentos de fisica
>quantica: qdo um eletron absorve energia, ele pula para uma orbita
>mais externa, ao devolver essa energia ele volta para sua orbita
>original. Os eletrons podem absorver apenas luz em certas frequencias
>- pois cada frequencia corresponde a uma determinada quantidade de
>energia de cada foton que compoe a luz e os eletrons absorvem apenas
>certas quantidade de energia ao mudar de orbita (ele nao pode absorver
>energias de valores intermediarios). Entao a distribuicao dos eletrons
>pelas camadas em torno do nucleo (reveja a parte dos modelos atomicos)
>determina que frequencias de luz um determinado material absorve e
>reemite - isto eh, reflete. E como a cor depende das propriedades de
>reflexao (e transmissao) da luz incidente, essa distribuicao de
>eletrons vai determinar a colocaracao caracteristica da substancia, do
>composto, do material. Lembre-se de q. falamos q. os eletrons mais
>externos distribuem-se quase livremente pelos nucleos dos atomos no
>metal. Outra caracteristica quantica: dois eletrons nao podem ocupar
>ao mesmo tempo uma mesma configuracao quantica - o q. significa q. nao
>podem ter ao mesmo tempo numeros quanticos iguais para camada,
>subnivel, orbital e spin (principio da exclusao de Pauli). Porem o
>compartilhamento dos eletrons mais externos preencheria todas as
>configuracoes possiveis, jah q. sao em quantidades finitas (existem
>apenas dois estados possiveis de spin, por exemplo). A presenca de
>diversos eletrons faz com q. a camada mais externa se subdivida em
>muitos e muitos subniveis bastante proximos entre si. Essa divisao
>fina faz com q. diversas frequencias de ondas possam ser absorvidas -
>e reemitidas. A estruturacao ordenada dos atomos no metal faz com q. a
>absorcao e reemissao da luz se dej de modo relativamente ordenado - o
>q. somado a reflexao de praticamente todos os comprimentos de onda (ao
>menos na faixa do visivel) garante uma reflexao semelhante ao do
>espelho (se polirmos uma superficie metalica termos de fato um bom
>espelho), isto eh, um brilho metalico.
>
>[O mercurio, q. eh liquido em cor ambiente, tem uma menor ordenacao em
>suas moleculas, o q. faz com q. adquira uma coloracao esbranquicada -
>dispersa um pouco a luz refletida.
>O ouro e o cobre, em funcao de algumas peculiaridades de sua
>distribuicao eletronica (e teremos q. recorrer um pouco a relatividade
>einsteniana para explicar o porque), perde parte de sua capacidade de
>refletir comprimentos de ondas mais curtos (frequencias maiores),
>adquirindo tons amarelados e avermelhados.]
>
> > Já agora, quando uma solução de peróxido de hidrogénio
> > contém 30 volume, qual é a sua concentração? Como se
> > determina-se?
>
>30 volumes significa q., em volume, H2O2 corresponde a 30% da solucao.
>(Usa-se volume, para horror dos puristas tecnicos em metrologia, uma
>vez q. % eh adimensional, mas se se indicasse apenas 30% nao se
>poderia saber se era 30% em massa, 30% em moleculas ou outra coisa -
>cada qual significando uma coisa diferente.)
>
>[]s,
>
>Roberto Takata
>
>
>




SUBJECT: Re: [ciencialist] Re: Por que massa atrai massa?
FROM: "Silvio Cordeiro" <scordeiro@terra.com.br>
TO: <ciencialist@yahoogrupos.com.br>
DATE: 13/01/2005 22:10

Hitler atraiu as massas, assim, tambem o Sr. Lula vulgo Ignacius II, Moisés
tambem só que tsunames. Papa Doc, Buda, O Marido da Madalena< paulo
coelho.....

este troço de atrações massivas é um enrosco de substância incognita.

carlos, o demiurgo.
----- Original Message -----
From: "marcelo ferrari" <emailferrari@yahoo.com.br>
To: <ciencialist@yahoogrupos.com.br>
Sent: Thursday, January 13, 2005 12:53 PM
Subject: [ciencialist] Re: Por que massa atrai massa?



Eurico,

Você está fazendo piada (ironia) ou está falando em algo que vc pensa?










"Eurico Ferreira de Souza Jr." <caodejah@yahoo.com.br> wrote:

[E]> a atração gravitacional é a velocidade da luz, parada.

[]s

Eurico





_\|/_

---------------------------------
Yahoo! Acesso Grátis - Internet rápida e grátis. Instale o discador do
Yahoo! agora.

[As partes desta mensagem que não continham texto foram removidas]



##### ##### #####

Para saber mais visite
http://www.ciencialist.hpg.ig.com.br


##### ##### ##### #####


Yahoo! Grupos, um serviço oferecido por:



















function SearchComboBox() { if
(document.form_combo.keyword.value.length==0){ alert("Por favor, digite
algo."); return false; }else { document.form_combo.action
="http://br.rd.yahoo.com/SIG=12ad2ag8v/M=264105.3931087.6562589.1588051/D=brclubs/S=2137111528:HM/EXP=1105713296/A=2361264/R=0/SIG=11uaou2jn/*http://www.bondfaro.com/bondfaro/in/combosearch_in.jsp?sk=11";
} return true;} [input] [input] [input]

---------------------------------
Links do Yahoo! Grupos

Para visitar o site do seu grupo na web, acesse:
http://br.groups.yahoo.com/group/ciencialist/

Para sair deste grupo, envie um e-mail para:
ciencialist-unsubscribe@yahoogrupos.com.br

O uso que você faz do Yahoo! Grupos está sujeito aos Termos do Serviço do
Yahoo!.



---------------------------------
Yahoo! Acesso Grátis - Internet rápida e grátis. Instale o discador do
Yahoo! agora.

[As partes desta mensagem que não continham texto foram removidas]



##### ##### #####

Para saber mais visite
http://www.ciencialist.hpg.ig.com.br


##### ##### ##### #####
Links do Yahoo! Grupos












SUBJECT: Re: [ciencialist] Por que massa atrai massa?
FROM: "murilo filo" <avalanchedrive@hotmail.com>
TO: ciencialist@yahoogrupos.com.br
DATE: 13/01/2005 22:22

Vou aproveitar e repetir meu postuladinho:
A gravidade, ou atração entre massas, é a memória coersiva do grande 'não
sei o que' pré big-bang, e é uma prova de que o negativo de um big-bang é
possível, precedendo outro big-bang... Certamente a gravidade será a
resultante de outros fatores energéticos numa larga escala, de moléculas às
partículas sub-atômicas. abr/M.

>From: "Eurico Ferreira de Souza Jr." <caodejah@yahoo.com.br>
>Reply-To: ciencialist@yahoogrupos.com.br
>To: ciencialist@yahoogrupos.com.br
>Subject: Re: [ciencialist] Por que massa atrai massa?
>Date: Thu, 13 Jan 2005 11:34:49 -0300 (ART)
>
>
>[E]> a atração gravitacional é a velocidade da luz, parada.
>
>[]s
>
>Eurico
>
>
>
>
>
>_\|/_
>
>---------------------------------
>Yahoo! Acesso Grátis - Internet rápida e grátis. Instale o discador do
>Yahoo! agora.
>
>[As partes desta mensagem que não continham texto foram removidas]
>




SUBJECT: Re: [ciencialist] Por que massa atrai massa?
FROM: "Alvaro Augusto \(E\)" <alvaro@electraenergy.com.br>
TO: <ciencialist@yahoogrupos.com.br>
DATE: 13/01/2005 23:20

Hum, na verdade é quase isso...

A gravidade foi o jeito que o universo inventou para respeitar um grande princípio de simetria, que é o princípio da relativdade. De acordo com esse princípio, as leis físicas devem ser as mesmas em todos os referenciais, inclusive os não inerciais. Só que, se a gravidade não existisse, a atração sentida por um corpo que acelara não teria contrapartida em referenciais inerciais. A gravidade é o fenômeno que garante essa contrapartida.

Agora, cabe a pergunta: Por que o universo insiste em "respeitar" princípios de simetria? Bem, a melhor resposta que eu conheço é: Sei lá!

[ ]s

Alvaro Augusto

----- Original Message -----
From: murilo filo
To: ciencialist@yahoogrupos.com.br
Sent: Thursday, January 13, 2005 10:22 PM
Subject: Re: [ciencialist] Por que massa atrai massa?


Vou aproveitar e repetir meu postuladinho:
A gravidade, ou atração entre massas, é a memória coersiva do grande 'não
sei o que' pré big-bang, e é uma prova de que o negativo de um big-bang é
possível, precedendo outro big-bang... Certamente a gravidade será a
resultante de outros fatores energéticos numa larga escala, de moléculas às
partículas sub-atômicas. abr/M.

>From: "Eurico Ferreira de Souza Jr." <caodejah@yahoo.com.br>
>Reply-To: ciencialist@yahoogrupos.com.br
>To: ciencialist@yahoogrupos.com.br
>Subject: Re: [ciencialist] Por que massa atrai massa?
>Date: Thu, 13 Jan 2005 11:34:49 -0300 (ART)
>
>
>[E]> a atração gravitacional é a velocidade da luz, parada.
>
>[]s
>
>Eurico
>


[As partes desta mensagem que não continham texto foram removidas]



SUBJECT: Re: [ciencialist] Por que massa atrai massa?
FROM: "Prof. JC" <profjc2003@yahoo.com.br>
TO: <ciencialist@yahoogrupos.com.br>
DATE: 14/01/2005 01:32

Yeh! Aprendi física com os livrinhos do Feynman, por isso sou suspeito a
opiniar, mas acho a visão dele de mundo, principalmente a do mundo físico, a
melhor que conheço.

Enfim... Matéria atrai matéria na razão direta de suas massas e na razão
inversa do quadro da distância que separa seus centros de gravidade... Ainda
que não saibamos o que é matéria, o que é massa, e que o "centro de
gravidade" seja simplesmente um conceito derivado do próprio conceito de
gravidade (mas, se quisermos, também podemos substituir centro de gravidade
por "centro de massa", o que remete a questão para o conceito de massa). :)

E pur si muove...

Abraços,
Prof. JC


----- Original Message -----
From: "Alvaro Augusto (E)" <alvaro@electraenergy.com.br>
To: <ciencialist@yahoogrupos.com.br>
Sent: Thursday, January 13, 2005 12:23 PM
Subject: Re: [ciencialist] Por que massa atrai massa?



Cabe notar que o comentário de Feynman reflete a visão que ele tinha do
mundo, na tradição iniciada por Dirac, e não é necessariamente compartilhada
por todos. Feynman não poderia pensar de outra forma. Caso contrário ele não
conseguiria explicar a ninguém a sua interpretação da mecânica quântica,
baseada nas integrais de percurso. Se alguém sobreviveu a Schrödinger, a
Heinsenberg e a Dirac, corre sério risco de sofrer um colapso ao chegar a
Feynman. Para ele, a probabilidade de um elétron passar por uma fenda, por
exemplo, é a soma das probabilidades do elétron percorrer todos os caminhos
possíveis, inclusive um caminho que passa reto pela fenda, inclusive um
caminho que passa pela galáxia de Andrômeda, inclusive um caminho que não
passa pela fenda! (na verdade, não se trata da "soma das probabilidades",
mas sim da soma das "amplitudes de probabilidades", mas vamos deixar esses
detalhes técnicos de lado).

O incrível é que as integrais de Feynman dão resultado corretos, embora
sejam muito mais usadas na teoria quântica dos campos do que na teoria
quântica não relativística. Mas nós não sabemos porque elas funcionam, daí o
argumento de Feynman. Muita gente, contudo, pensa de maneira platônica ("a
verdade está lá fora") e não pragmática ("a verdade é o que conseguimos
obter). Roger Penrose, por exemplo, é um platônico, enquanto Feynman foi um
grande pragmático.

[ ]s

Alvaro Augusto


----- Original Message -----
From: E m i l i a n o C h e m e l l o
To: ciencialist@yahoogrupos.com.br
Sent: Thursday, January 13, 2005 10:33 AM
Subject: Re: [ciencialist] Por que massa atrai massa?


Olá JC,

Que bom que pensamos igual. Coincidência mesmo. Este estigma que a
ciência/cientista é dono da verdade tem que acabar. Existem coisas ditas
'fundamentais' dentro da ciência. Um exemplo é a atração as massas. Outra
é
a atração/repulsão entre cargas. E antes do Big Bang? Perguntará alguém.
Não
sabemos.

Eu elenco algumas palavras que os cientistas devem ter cuidado ao
fazer
uso: "é", "sempre", "nunca".

[ ] 's do Emiliano Chemello
emiliano@quimica.net
http://www.quimica.net/emiliano
http://www.ucs.br/ccet/defq/naeq

" Rien ne se perd, rien ne se crée,
tout se transforme."

Antoine Laurent de Lavoisier (químico francês, 1743 - 1794)

----- Original Message -----
From: Prof. JC
To: ciencialist@yahoogrupos.com.br
Sent: Wednesday, January 12, 2005 8:09 PM
Subject: Re: [ciencialist] Por que massa atrai massa?


Oi Chemello,

Eu respondi exatamente isso em uma lista de discussão, também
recentemente.
Coincidências acontecem. :)

Abraços,
Prof. JC


----- Original Message -----
From: "E m i l i a n o C h e m e l l o" <chemelloe@yahoo.com.br>
To: <ciencialist@yahoogrupos.com.br>
Sent: Wednesday, January 12, 2005 9:00 AM
Subject: [ciencialist] Por que massa atrai massa?



Em uma pergunta feita numa lista de discussão, foi pedido: "Por que massa
atrai massa?". Eu respondi, com uma citação:

"Ninguém sabe porque as coisas são como são".
Richard Feynman

A ciência investiga 'como' as coisas funcionam e não 'porque'
funcionam.
Alguém concorda? Alguém discorda? Alguém não tá nem ai para isso? :-)

[ ] 's do Emiliano Chemello
emiliano@quimica.net
http://www.quimica.net/emiliano
http://www.ucs.br/ccet/defq/naeq

" Rien ne se perd, rien ne se crée,
tout se transforme."

Antoine Laurent de Lavoisier (químico francês, 1743 - 1794)


[As partes desta mensagem que não continham texto foram removidas]


[As partes desta mensagem que não continham texto foram removidas]



##### ##### #####

Para saber mais visite
http://www.ciencialist.hpg.ig.com.br


##### ##### ##### #####
Links do Yahoo! Grupos













SUBJECT: Re: Brilho metálico
FROM: "rmtakata" <rmtakata@altavista.net>
TO: ciencialist@yahoogrupos.com.br
DATE: 14/01/2005 04:02


--- Em ciencialist@yahoogrupos.com.br, "rmtakata"
> [O mercurio, q. eh liquido em cor ambiente, tem uma menor

Liquido em cor ambiente? Leiam: liquido em *temperatura* ambiente.

[]s,

Roberto Takata





SUBJECT: Re: [ciencialist] Por que massa atrai massa?
FROM: "JVictor" <jvoneto@uol.com.br>
TO: <ciencialist@yahoogrupos.com.br>
DATE: 14/01/2005 07:56



Álvaro: Agora, cabe a pergunta: Por que o universo insiste em "respeitar" princípios de simetria? Bem, a melhor resposta que eu conheço é: Sei lá!

Victor: Tudo leva a crer que o universo faz mais do que "respeitar" os princípios das simetrias. É como se as simetrias fossem inerentes, constitutivas, do universo; pois as leis de conservação, que comandam nossa vidinha e as coisas que fazemos, todas, são o resultado de alguma simetria. As simetrias clarificam as coisas que podem acontecer e "amarram" as que, nem com reza, acontecem! E as simetrias marcham para a simplificação das coisas. Acho que a compulsão de Einstein em busca de princípios simples e gerais talvez se deva a um conhecimento interior profundo dessas nuances exibidas pela natureza.
Agora, depois desse papo, posso dizer com segurança o que são simetrias e quais devam ser as eventuais verdades essenciais por trás delas:

-Sei lá!

Meu, nosso, consolo: ninguém sabe também.

Muito claros teus últimos textos a respeito dessas coisas. Sobre tua impressão sobre o pensamento do grande Feynman, também concordo, acrescentando que ele era um cara pé no chão e operacionalista prá ninguém botar defeito. Não se fazem mais feynman´s como antigamente!

Victor.





[As partes desta mensagem que não continham texto foram removidas]



##### ##### #####

Para saber mais visite
http://www.ciencialist.hpg.ig.com.br


##### ##### ##### #####



Yahoo! Grupos, um serviço oferecido por:







------------------------------------------------------------------------------
Links do Yahoo! Grupos

a.. Para visitar o site do seu grupo na web, acesse:
http://br.groups.yahoo.com/group/ciencialist/

b.. Para sair deste grupo, envie um e-mail para:
ciencialist-unsubscribe@yahoogrupos.com.br

c.. O uso que você faz do Yahoo! Grupos está sujeito aos Termos do Serviço do Yahoo!.



[As partes desta mensagem que não continham texto foram removidas]



SUBJECT: Para aqueles se ligam em Energia
FROM: "Tipoalgo" <tipoalgo@bol.com.br>
TO: ciencialist@yahoogrupos.com.br
DATE: 14/01/2005 11:11


Quem gosta do assunto, uma visita ao site fendel.com.br é uma boa.
O Sr. fendel é um entusiasta daqueles.
O link abaixo mostra um pouco desse entusiamo.

http://www2.ipef.br/pipermail/bioenergia-l/2004-November/001082.html


Abraços

Tipoalgo





SUBJECT: Re: Por que massa atrai massa?
FROM: marcelo ferrari <emailferrari@yahoo.com.br>
TO: ciencialist@yahoogrupos.com.br
DATE: 14/01/2005 11:15

Álvaro >>> Cabe a pergunta: Por que o universo insiste em "respeitar" princípios de simetria?

Álvaro, porque um computador insiste em respeitar o software?

Sim, o computador também respeita a lei da física, mas a minha pergunta é uma analogia e propoe um caminho para a solução da sua. Pense como uma metáfora. Vamos supor que o universo é um computador com um programa básico chamado simetria. Sendo assim: Por que o universo insiste em "respeitar" princípios de simetria?


marcelo ferrari






"Alvaro Augusto (E)" <alvaro@electraenergy.com.br> wrote:
Hum, na verdade é quase isso...

A gravidade foi o jeito que o universo inventou para respeitar um grande princípio de simetria, que é o princípio da relativdade. De acordo com esse princípio, as leis físicas devem ser as mesmas em todos os referenciais, inclusive os não inerciais. Só que, se a gravidade não existisse, a atração sentida por um corpo que acelara não teria contrapartida em referenciais inerciais. A gravidade é o fenômeno que garante essa contrapartida.

Agora, cabe a pergunta: Por que o universo insiste em "respeitar" princípios de simetria? Bem, a melhor resposta que eu conheço é: Sei lá!

[ ]s

Alvaro Augusto

----- Original Message -----
From: murilo filo
To: ciencialist@yahoogrupos.com.br
Sent: Thursday, January 13, 2005 10:22 PM
Subject: Re: [ciencialist] Por que massa atrai massa?


Vou aproveitar e repetir meu postuladinho:
A gravidade, ou atração entre massas, é a memória coersiva do grande 'não
sei o que' pré big-bang, e é uma prova de que o negativo de um big-bang é
possível, precedendo outro big-bang... Certamente a gravidade será a
resultante de outros fatores energéticos numa larga escala, de moléculas às
partículas sub-atômicas. abr/M.

>From: "Eurico Ferreira de Souza Jr." <caodejah@yahoo.com.br>
>Reply-To: ciencialist@yahoogrupos.com.br
>To: ciencialist@yahoogrupos.com.br
>Subject: Re: [ciencialist] Por que massa atrai massa?
>Date: Thu, 13 Jan 2005 11:34:49 -0300 (ART)
>
>
>[E]> a atração gravitacional é a velocidade da luz, parada.
>
>[]s
>
>Eurico
>


[As partes desta mensagem que não continham texto foram removidas]



##### ##### #####

Para saber mais visite
http://www.ciencialist.hpg.ig.com.br


##### ##### ##### #####


Yahoo! Grupos, um serviço oferecido por:



















function SearchComboBox() { if (document.form_combo.keyword.value.length==0){ alert("Por favor, digite algo."); return false; }else { document.form_combo.action ="http://br.rd.yahoo.com/SIG=12aa6hdbq/M=264105.3931087.6562589.1588051/D=brclubs/S=2137111528:HM/EXP=1105752043/A=2361264/R=0/SIG=11uaou2jn/*http://www.bondfaro.com/bondfaro/in/combosearch_in.jsp?sk=11"; } return true;} [input] [input] [input]

---------------------------------
Links do Yahoo! Grupos

Para visitar o site do seu grupo na web, acesse:
http://br.groups.yahoo.com/group/ciencialist/

Para sair deste grupo, envie um e-mail para:
ciencialist-unsubscribe@yahoogrupos.com.br

O uso que você faz do Yahoo! Grupos está sujeito aos Termos do Serviço do Yahoo!.




---------------------------------
Yahoo! Acesso Grátis - Internet rápida e grátis. Instale o discador do Yahoo! agora.

[As partes desta mensagem que não continham texto foram removidas]



SUBJECT: "ófitópic": dica firewall + Fwd: [Livros_Digitais] REVISTA HACKER GRATUITA
FROM: "Eurico Ferreira de Souza Jr." <caodejah@yahoo.com.br>
TO: acropolis@yahoogrupos.com.br
CC: ciencialist@yahoogrupos.com.br
DATE: 14/01/2005 11:29

[E]> Salve,

instalei o sygate personal firewall.
agora eu decido qual programa acessa a internet.
descobri que o MSN messenger (que eu nunca usei) fica tentando fazer sei lá o que na net, e que o ad-aware fica tentando acessar o gator !
o norton fica avisando que o firewall tá baixando pacote suspeito e o ad-aware alerta que o spy-bot tá suspeito também...
eu me pergunto: há limites? :D



papagrupo <papagrupo@yahoo.com.br> wrote:
Para: Livros_Digitais@yahoogrupos.com.br
De: "papagrupo"

Data: Fri, 14 Jan 2005 05:21:32 -0000
Assunto: [Livros_Digitais] REVISTA HACKER GRATUITA



Quem se interessa por temas de segurança na internet pode baixar
gratuitamente o primeiro número da revista HACKER.BR. É só clicar na
capinha, não precisa fazer cadastro. Gostei de duas matérias, uma
que ensina a acessar o e-Mail por telnet e outra com a história dos
hackers de 1960 até hoje.

A revista está no número zero e pode ser baixada gratuitamente:

www.cursodehacker.com.br



_\|/_

---------------------------------
Yahoo! Acesso Grátis - Internet rápida e grátis. Instale o discador do Yahoo! agora.

[As partes desta mensagem que não continham texto foram removidas]



SUBJECT: Re: [ciencialist] Re: Por que massa atrai massa?
FROM: "Eurico Ferreira de Souza Jr." <caodejah@yahoo.com.br>
TO: ciencialist@yahoogrupos.com.br
DATE: 14/01/2005 12:17



"Alvaro Augusto (E)" <alvaro@electraenergy.com.br> wrote:


Acho que não formulei muito bem a pergunta. Era para ser: "por que a luz
precisa ter uma velocidade?"

[E]> pra se manifestar.

(...)

Não é bem assim. Antes de serem emitidos, os fótons não existem. Eles não
estão dentro do elétron esperando para escapar.

[E]> onde estava o tsunami antes do terremoto? um fótom é uma onda no "nada quântico" que é o nosso universo. o elétron balança o "nada" fazendo "ondinhas fotônicas"...

(...)

Eu não sou especialista em QED, mas me lembro de ter lido em algum lugar que
os fótons podem ter várias velocidades, embora a velocidade da luz seja
invariante.

[E]> essa é novidade pra mim...





_\|/_

---------------------------------
Yahoo! Acesso Grátis - Internet rápida e grátis. Instale o discador do Yahoo! agora.

[As partes desta mensagem que não continham texto foram removidas]



SUBJECT: mais rápido que a luz?
FROM: "Eurico Ferreira de Souza Jr." <caodejah@yahoo.com.br>
TO: acropolis@yahoogrupos.com.br
CC: ciencialist@yahoogrupos.com.br
DATE: 14/01/2005 14:02



Jjaime <jjaime@miportal.es> wrote:Para:
De: "Jjaime"
Data: Sun, 19 Dec 2004 02:15:25 -0200
Assunto: Re: RES: [GPS] Fw: [urania-br] Pergunta - Tempo


14 "objetos" que viajariam mais rápido que a luz:

- A interseção das lâminas de um par de tesouras, sendo que tais lâminas
sejam longas o suficiente e estas sejam fechadas a uma velocidade próxima de
"c".

2- A imagem dos movimentos rápidos do facho de um holofote, desde que você
esteja olhando de uma distância grande o suficiente.

3- A sombra do eclipse de um planeta sobre um outro mais distante.

4- Uma barra longa, infinitamente rígida, na qual o empurrão empregado em
uma extremidade, é instântaneamente transmitido a outra extremidade, não
importa o quão distante estejam.

5- A colisão de duas ondas (como as do mar) na qual ondículas transitórias
se formam e parecem se movimentar mais rápido que a luz.

6- Alguns Quasares parecem emitir pequenas quantidades de material
movendo-se a velocidades superiores à luz.

7- A velocidade da fase de um sinal radiofônico viajando através de plasma
parece adquirir velocidade super-luz.

8- Certas luzes de marcação (aquelas vistas no alto de prédios muito altos)
podem ser programadas para piscarem em velocidades aparentemente maiores que
c.

9- Uma longa cauda de cometa, como um holofote, viaja mais rápido que a luz
quando a cabeça do cometa gira em torno do sol.

10- As ondículas que se formam quando as ondas do oceano quebram na praia
podem viajar a velocidades infinitas quando tal onda atinge a praia em
certos ângulos e em toda parte simultaneamente.

11- O elétron que forma o ponto em um ociloscópio pode, a princípio, viajar
a velocidades maiores que a da luz.

12- Em uma situação ideal, na qual a terra pode ser considerada em repouso
(o antigo modelo Geocêntrico aceito antes de Copérnico) implicaria em
velocidades super-luz para Netuno e Plutão.

13- A própria teoria do Big Bang admite que o universo se expandiu a
velocidades super-luz imediatamente após a explosão.

14- Devido à dilatação do tempo a bordo de uma nave que viaja a uma
velocidade próxima a da luz, os tripulantes desta teriam a impressão de
estar viajando a velocidades superiores a c.

Quanto a este último exemplo, é lógico que os observadores externos ainda
veriam a nave em velocidade sub-luz.


Nick Herbert - Faster than Light, Superluminal Loopholes in Physics - Mais
rápido que a luz, Falhas na Teoria do Movimento Super-Luz na Física
(movimento super-luz não está vetado pela relatividade)




_\|/_

---------------------------------
Yahoo! Acesso Grátis - Internet rápida e grátis. Instale o discador do Yahoo! agora.

[As partes desta mensagem que não continham texto foram removidas]



SUBJECT: Re: Por que massa atrai massa?
FROM: marcelo ferrari <emailferrari@yahoo.com.br>
TO: ciencialist@yahoogrupos.com.br
DATE: 14/01/2005 14:17

[E]> Onde estava o tsunami antes do terremoto?


Boa maneira de pensar, Eurico. Onde estava esta minha resposta entes de eu lhe enviar? Então, me parece que o "onde" é a sobra do "quanto", ou vice-versa. O que acha?



marcelo ferrari


---------------------------------
Yahoo! Acesso Grátis - Internet rápida e grátis. Instale o discador do Yahoo! agora.

[As partes desta mensagem que não continham texto foram removidas]



SUBJECT: quimica inorganica
FROM: "Camila" <camilasaki@yahoo.com.br>
TO: ciencialist@yahoogrupos.com.br
DATE: 14/01/2005 15:20


Será que alguem aki conheceria um site para experimentos de quimica
inorganica, de nivel superior, com materiais alternativos?

desde já agradeço

Camila





SUBJECT: Re: [ciencialist] Re: Por que massa atrai massa?
FROM: "Eurico Ferreira de Souza Jr." <caodejah@yahoo.com.br>
TO: ciencialist@yahoogrupos.com.br
DATE: 14/01/2005 15:45



marcelo ferrari <emailferrari@yahoo.com.br> wrote:
[E]> Onde estava o tsunami antes do terremoto?


Boa maneira de pensar, Eurico. Onde estava esta minha resposta entes de eu lhe enviar?

[E]> no futuro. :D

Então, me parece que o "onde" é a sobra do "quanto", ou vice-versa. O que acha?

[E]> pode ser a soma do local e do momento... me lembrei do heráclito... :D





_\|/_

---------------------------------
Yahoo! Acesso Grátis - Internet rápida e grátis. Instale o discador do Yahoo! agora.

[As partes desta mensagem que não continham texto foram removidas]



SUBJECT: Re: Por que massa atrai massa?
FROM: marcelo ferrari <emailferrari@yahoo.com.br>
TO: ciencialist@yahoogrupos.com.br
DATE: 14/01/2005 16:37

[F]> Onde estava esta minha resposta entes de eu lhe enviar?

[E]> no futuro. :D

E onde estava o futuro?


---------------------------------
Yahoo! Acesso Grátis - Internet rápida e grátis. Instale o discador do Yahoo! agora.

[As partes desta mensagem que não continham texto foram removidas]



SUBJECT: Re: [ciencialist] Re: Por que massa atrai massa?
FROM: "Eurico Ferreira de Souza Jr." <caodejah@yahoo.com.br>
TO: ciencialist@yahoogrupos.com.br
DATE: 14/01/2005 17:04



marcelo ferrari <emailferrari@yahoo.com.br> wrote:[F]> Onde estava esta minha resposta entes de eu lhe enviar?

[E]> no futuro. :D

E onde estava o futuro?

[E]> onde agora se chama passado.


_\|/_
__________________________________________________
Converse com seus amigos em tempo real com o Yahoo! Messenger
http://br.download.yahoo.com/messenger/

[As partes desta mensagem que não continham texto foram removidas]



SUBJECT: Re: Por que massa atrai massa?
FROM: marcelo ferrari <emailferrari@yahoo.com.br>
TO: ciencialist@yahoogrupos.com.br
DATE: 14/01/2005 18:01


[F1]> Onde estava esta minha resposta entes de eu lhe enviar?
[E1]> no futuro. :D

[F2]> E onde estava o futuro?
[E2]> Onde agora se chama passado.

[F3]> E onde estava o passado?
[E3]>


__________________________________________________
Converse com seus amigos em tempo real com o Yahoo! Messenger
http://br.download.yahoo.com/messenger/

[As partes desta mensagem que não continham texto foram removidas]



SUBJECT: onde estava o passado...?
FROM: "Eurico Ferreira de Souza Jr." <caodejah@yahoo.com.br>
TO: ciencialist@yahoogrupos.com.br
DATE: 14/01/2005 18:09



marcelo ferrari <emailferrari@yahoo.com.br> wrote:
[F1]> Onde estava esta minha resposta entes de eu lhe enviar?
[E1]> no futuro. :D

[F2]> E onde estava o futuro?
[E2]> Onde agora se chama passado.

[F3]> E onde estava o passado?
[E3]> na verdade, passado e futuro são conceitos abstratos. só existe o agora.




_\|/_
__________________________________________________
Converse com seus amigos em tempo real com o Yahoo! Messenger
http://br.download.yahoo.com/messenger/

[As partes desta mensagem que não continham texto foram removidas]



SUBJECT: Re: onde estava o passado...?
FROM: marcelo ferrari <emailferrari@yahoo.com.br>
TO: ciencialist@yahoogrupos.com.br
DATE: 14/01/2005 19:18

[F1]> Onde estava esta minha resposta entes de eu lhe enviar?
[E1]> no futuro. :D

[F2]> E onde estava o futuro?
[E2]> Onde agora se chama passado.

[F3]> E onde estava o passado?
[E3]> na verdade, passado e futuro são conceitos abstratos. só existe o agora.

[F4]> Quer dizer que antes de eu te enviar esta minha resposta aqui ela já estava enviada, pois já estava no agora? É isto?
[E4]>




---------------------------------
Yahoo! Acesso Grátis - Internet rápida e grátis. Instale o discador do Yahoo! agora.

[As partes desta mensagem que não continham texto foram removidas]



SUBJECT: Re: Por que massa atrai massa?
FROM: marcelo ferrari <emailferrari@yahoo.com.br>
TO: ciencialist@yahoogrupos.com.br
DATE: 14/01/2005 19:20

Silvio, é bom ver seu bom humor de novo.
Forte abraço.

Marcelo.



Silvio Cordeiro <scordeiro@terra.com.br> wrote:
Signore Marcel:


Seu humor ao dizer que Henri, o touromáquico genoves que em suas pesquisas
no Fermi descobriu que a luz obedece à sinalização de trânsito sideral,
parando nos semáforos para permitir sem maiores problemas a passagem de uma
galaxia grávida, envolta em seu diáfano talar de matéria negra,torna-o uma
pessoa que merece levar 75 chibatatas nas plantas dos pés, todas as sextras
feiras menos no Radamã.Fosse eu Sultão das Arábias ou mesmo um régugo de
qualquer aldeia maori, tudo por puro despeito e inveja por sua eructação
flactuosa.

Que ódio!!!!!

carlos o mais pior dos ruinsmente péssimos..
----- Original Message -----
From: "marcelo ferrari" <emailferrari@yahoo.com.br>
To: <ciencialist@yahoogrupos.com.br>
Sent: Thursday, January 13, 2005 12:53 PM
Subject: [ciencialist] Re: Por que massa atrai massa?



Eurico,

Você está fazendo piada (ironia) ou está falando em algo que vc pensa?










"Eurico Ferreira de Souza Jr." <caodejah@yahoo.com.br> wrote:

[E]> a atração gravitacional é a velocidade da luz, parada.

[]s

Eurico





_\|/_

---------------------------------
Yahoo! Acesso Grátis - Internet rápida e grátis. Instale o discador do
Yahoo! agora.

[As partes desta mensagem que não continham texto foram removidas]



##### ##### #####

Para saber mais visite
http://www.ciencialist.hpg.ig.com.br


##### ##### ##### #####


Yahoo! Grupos, um serviço oferecido por:



















function SearchComboBox() { if
(document.form_combo.keyword.value.length==0){ alert("Por favor, digite
algo."); return false; }else { document.form_combo.action
="http://br.rd.yahoo.com/SIG=12ad2ag8v/M=264105.3931087.6562589.1588051/D=brclubs/S=2137111528:HM/EXP=1105713296/A=2361264/R=0/SIG=11uaou2jn/*http://www.bondfaro.com/bondfaro/in/combosearch_in.jsp?sk=11";
} return true;} [input] [input] [input]

---------------------------------
Links do Yahoo! Grupos

Para visitar o site do seu grupo na web, acesse:
http://br.groups.yahoo.com/group/ciencialist/

Para sair deste grupo, envie um e-mail para:
ciencialist-unsubscribe@yahoogrupos.com.br

O uso que você faz do Yahoo! Grupos está sujeito aos Termos do Serviço do
Yahoo!.



---------------------------------
Yahoo! Acesso Grátis - Internet rápida e grátis. Instale o discador do
Yahoo! agora.

[As partes desta mensagem que não continham texto foram removidas]



##### ##### #####

Para saber mais visite
http://www.ciencialist.hpg.ig.com.br


##### ##### ##### #####
Links do Yahoo! Grupos












##### ##### #####

Para saber mais visite
http://www.ciencialist.hpg.ig.com.br


##### ##### ##### #####


Yahoo! Grupos, um serviço oferecido por: São Paulo Rio de Janeiro Curitiba Porto Alegre Belo Horizonte Brasília

---------------------------------
Links do Yahoo! Grupos

Para visitar o site do seu grupo na web, acesse:
http://br.groups.yahoo.com/group/ciencialist/

Para sair deste grupo, envie um e-mail para:
ciencialist-unsubscribe@yahoogrupos.com.br

O uso que você faz do Yahoo! Grupos está sujeito aos Termos do Serviço do Yahoo!.


__________________________________________________
Converse com seus amigos em tempo real com o Yahoo! Messenger
http://br.download.yahoo.com/messenger/

[As partes desta mensagem que não continham texto foram removidas]



SUBJECT: Galera: Felicidade!! Já estamos em Titã!!!!!!!!!!!!!!
FROM: Maria Natália <grasdic@hotmail.com>
TO: ciencialist@yahoogrupos.com.br
DATE: 14/01/2005 19:59


13.34 Surface touchdownThis time may vary by plus or minus 15
minutes depending on how Titan's atmosphere and winds affect
Huygens's parachuting descent. Huygens will hit the surface at a
speed of 5-6 metres per second. Huygens could land on a hard surface
of rock or ice or possibly land on an ethane sea. In either case,
Huygens's Surface Science Package is designed to capture every piece
of information about the surface that can be determined in the three
remaining minutes that Huygens is designed to survive after landing.

Ver pela ordem de importância Mas deve estar tudo saturado

http://spaceflightnow.com/cassini/status.html

http://saturn.jpl.nasa.gov/operations/present-position.cfm
http://www.esa.int/export/esaCP/SEMQOI71Y3E_index_0.html


http://www.esa.int/export/esaCP/
http://www.esa.int/SPECIALS/VideoTalk/SEM5M5XJD1E_0.html
Viva a EUROPA! Viva a ESA!!!
Um abraço
Maria Natália










SUBJECT: 100 brasileiros
FROM: "Esteban Moreno" <estebanmoreno@idhi.org.br>
TO: "Vivens Scientia" <VivensScientia@yahoogroups.com>, <ciencialist@yahoogrupos.com.br>
DATE: 14/01/2005 20:20

Belíssimo trabalho:

http://www.brasil.gov.br/100brasileiros/

O resumo:
A Secretaria de Comunicação de Governo e Gestão Estratégica da Presidência da República (Secom) incorporou o livro "100 Brasileiros" à campanha "O Melhor do Brasil é o Brasileiro" lançada pela Associação Brasileira de Anunciantes (ABA). A obra, coordenada pela Secom e produzida pela Biblioteca Nacional, reúne biografias de personalidades de destaque na história de nosso país em diversas áreas. Ao destacar, na história, 100 personalidades ilustres, é possível reforçar a crença na capacidade do povo brasileiro de vencer e superar grandes desafios. Em síntese, o livro tem dois conceitos centrais. Uma nação não pode ser verdadeiramente forte se não acredita em si mesma, nas suas realizações, nas suas potencialidades e no seu trabalho. E os nossos talentos, homens e mulheres que contribuíram para a vida política e enriqueceram a vida cultural-artística, merecem ser conhecidos.


Esteban.

[As partes desta mensagem que não continham texto foram removidas]



SUBJECT: Re: Por que massa atrai massa?
FROM: "Sergio M. M. Taborda" <sergiotaborda@yahoo.com.br>
TO: ciencialist@yahoogrupos.com.br
DATE: 14/01/2005 21:16


--- Em ciencialist@yahoogrupos.com.br, "Alvaro Augusto \(E\)"
<alvaro@e...> escreveu
> E por que a luz tem velocidade, afinal? Por que os fótons não
conseguem ficar parados no lugar? E será que a velocidade da luz é a
mesma coisa que a velocidade dos fótons?

O que é vento ? são as particulas de ar se movendo. Quando não ha
vento, não ha ar ?
As particulas não desaparecem pq não as vemos (no sentido lato) se
moverem. Um fotão parado não é visivel, luz parada não visivel, tal
como vento parado não é visivel.

O que é a velocidade da luz afinal ? a velocidade de grupo ? de fase ?
Aquilo que chamamos de velocidade da luz, que é a de grupo não tem a
mesma definição que a velocidade de objectos macroscopicos.
A velocidade macorscopica v = s /t = s * (1/t) = s f é uma velocidade
de fase. Mas a velocidade de fase de uma onda não carrega informação.
É essa diferença basilar que diferencia a fisica da luz, da fisica das
outras coisas.
As coisas complicam-se quando sabemos que a luz tem momento , tal como
um objecto macroscopico , mas a definição de momento é algo diferente

para objectos macroscopicos p = m v = m v_fase e sabemos o que é m
(nem que seja por definição)
para a luz p = h lambda = h v_fase / f e não sabemos o que h/f é ,
nem por definição.

Fazemos analogias, e tudo o mais, mas não sabemos pq. Sabemos como,
mas não porquê.

Sérgio Taborda









SUBJECT: Re: [ciencialist] Por que massa atrai massa?
FROM: "murilo filo" <avalanchedrive@hotmail.com>
TO: ciencialist@yahoogrupos.com.br
DATE: 14/01/2005 21:58

Papos assim fazem valer êste grupo. abr/M.

>From: "JVictor" <jvoneto@uol.com.br>
>Reply-To: ciencialist@yahoogrupos.com.br
>To: <ciencialist@yahoogrupos.com.br>
>Subject: Re: [ciencialist] Por que massa atrai massa?
>Date: Fri, 14 Jan 2005 07:56:55 -0200
>
>
>
>Álvaro: Agora, cabe a pergunta: Por que o universo insiste em "respeitar"
>princípios de simetria? Bem, a melhor resposta que eu conheço é: Sei lá!
>
>Victor: Tudo leva a crer que o universo faz mais do que "respeitar" os
>princípios das simetrias. É como se as simetrias fossem inerentes,
>constitutivas, do universo; pois as leis de conservação, que comandam nossa
>vidinha e as coisas que fazemos, todas, são o resultado de alguma simetria.
>As simetrias clarificam as coisas que podem acontecer e "amarram" as que,
>nem com reza, acontecem! E as simetrias marcham para a simplificação das
>coisas. Acho que a compulsão de Einstein em busca de princípios simples e
>gerais talvez se deva a um conhecimento interior profundo dessas nuances
>exibidas pela natureza.
>Agora, depois desse papo, posso dizer com segurança o que são simetrias e
>quais devam ser as eventuais verdades essenciais por trás delas:
>
>-Sei lá!
>
>Meu, nosso, consolo: ninguém sabe também.
>
>Muito claros teus últimos textos a respeito dessas coisas. Sobre tua
>impressão sobre o pensamento do grande Feynman, também concordo,
>acrescentando que ele era um cara pé no chão e operacionalista prá ninguém
>botar defeito. Não se fazem mais feynman´s como antigamente!
>
>Victor.
>
>
>
>
>
> [As partes desta mensagem que não continham texto foram removidas]
>
>
>
> ##### ##### #####
>
> Para saber mais visite
> http://www.ciencialist.hpg.ig.com.br
>
>
> ##### ##### ##### #####
>
>
>
> Yahoo! Grupos, um serviço oferecido por:
>
>
>
>
>
>
>
>------------------------------------------------------------------------------
> Links do Yahoo! Grupos
>
> a.. Para visitar o site do seu grupo na web, acesse:
> http://br.groups.yahoo.com/group/ciencialist/
>
> b.. Para sair deste grupo, envie um e-mail para:
> ciencialist-unsubscribe@yahoogrupos.com.br
>
> c.. O uso que você faz do Yahoo! Grupos está sujeito aos Termos do
>Serviço do Yahoo!.
>
>
>
>[As partes desta mensagem que não continham texto foram removidas]
>




SUBJECT: Re: Por que massa atrai massa?
FROM: marcelo ferrari <emailferrari@yahoo.com.br>
TO: ciencialist@yahoogrupos.com.br
DATE: 15/01/2005 06:22

SERGIO > Quando não há vento, não ha ar? As particulas de luz não desaparecem pq não as vemos (no sentido lato) se moverem.

1) Ar em movimento é vento. Foton em movimento é luz? É esta a analogia?
2) Ar é feito de quimica. Foton é feito de que?
3) O que poe o ar em movimento pra surgir o vento? O que poe os fotons em movimento para surgir a luz?

SERGIO > Um fotão parado não é visivel, luz parada não visivel, tal como vento parado não é visivel.

Porque, um fotom parado não é visivel? O que isto significa?

SERGIO > O que é a velocidade da luz afinal? a velocidade de grupo? de fase?

O vento pode ter várias velocidades, a luz não? Porque?


marcelo ferrari


---------------------------------
Yahoo! Acesso Grátis - Internet rápida e grátis. Instale o discador do Yahoo! agora.

[As partes desta mensagem que não continham texto foram removidas]



SUBJECT: Re: Por que massa atrai massa?
FROM: "rayfisica" <rayfisica@yahoo.com.br>
TO: ciencialist@yahoogrupos.com.br
DATE: 15/01/2005 10:03


--- Em ciencialist@yahoogrupos.com.br, marcelo ferrari
<emailferrari@y...> escreveu


Pergunta inicial

Por que massa atrai massa?

Sendo a gravidade uma característica da massa, a resposta não
seria
porque existe a gravidade.

Parece-me então que a maioria das respostas postadas tratam da
gravidade, o que, por que, relacionado a gravidade, não?

O fóton não tem massa, porem sofre a influencia da gravidade





SUBJECT: Re: Por que massa atrai massa?
FROM: Maria Natália <grasdic@hotmail.com>
TO: ciencialist@yahoogrupos.com.br
DATE: 15/01/2005 13:09


--- Em ciencialist@yahoogrupos.com.br, "rayfisica" <rayfisica@y...>
escreveu
>
> --- Em ciencialist@yahoogrupos.com.br, marcelo ferrari
> <emailferrari@y...> escreveu
>
>
> Pergunta inicial
>
> Por que massa atrai massa?
>
> Sendo a gravidade uma característica da massa, a resposta não
> seria
> porque existe a gravidade.
>
> Parece-me então que a maioria das respostas postadas tratam da
> gravidade, o que, por que, relacionado a gravidade, não?
>
> O fóton não tem massa, porem sofre a influencia da gravidade





SUBJECT: Re: Por que massa atrai massa?
FROM: Maria Natália <grasdic@hotmail.com>
TO: ciencialist@yahoogrupos.com.br
DATE: 15/01/2005 13:53


rayfisica:
A isto se chama ponto de ordem na discussão.
Peguei na questão inicila e a coloquei a dois níveis de 10º ano e de
12º ano.
E houve várias respostas. nível 10ª--As forças existem aos pares foi
logo a 1ª a aparecer. Essas forças já foram medidas
experimentalmente foi outra resposta. Estas afirmações pressupõem
que aluno tem uma progressão em física desde os 13 anos e portanto
ele lida com o conceito de força mais ou menos bem apreendido. É que
o conceito de força é difícil e só a nível das primeiras abstracções
se alcança (16 anos).
Ao nível dos mais adiantados: essa ideia de força e que trás por
trás misconceptions desaparece com a introdução da noção de campo.
Cmapo gravítico, campo de velocidades, etc Como sendo o local onde
existe energia. Portanto uma massa colocada num local que tem
energia (potencial gravítica) por já lá estar outra massa é de fácil
entendimento, aceitação sem grandes bases da matemática mesmo
vectorial (força é grandeza vectorial). Até com campo se "sente" o
surgir (quase simultâneo) das duas forças. Mas a velocidade de
transmissão dá problema e se não tivermos o senso isto vai parar a
dinâmica relativistíca e nunca mais ninguém agarra a criança e uma
pergunta que à partida se destinava a retirar uma dúvida está
gerando grande confusão pois não temos bases. A relatividade não é
aquela deusa que sem a matemática vive.
Mas ainda estamos discutindo nos intervalos de aulas este tema. Aí
em férias os meninos de nota 10 de física deviam discuti-lo na praia
com seus professores. As férias também servem para conversa informal
entre professor e seus melhores alunos sobre temas científicos.
Serve até para vermos até que ponto fomos entendidos.
Como no laboratório se pode medir essa interacção entre duas massas?
É questão para os professores de ensino secundário/médio desta
lista. Mas antes vou ali ao Leo feiraceciencias.com.br
Não ajudei? E só travei? Quem sabe?
Um abraço
Maria Natália
PS. desculpem mensagem em branco anterior mas meu gato Max Planck,
no teclado, quis responder LOLLLLL


--- Em ciencialist@yahoogrupos.com.br, "rayfisica" <rayfisica@y...>
escreveu
>
> --- Em ciencialist@yahoogrupos.com.br, marcelo ferrari
> <emailferrari@y...> escreveu
>
>
> Pergunta inicial
>
> Por que massa atrai massa?
>
> Sendo a gravidade uma característica da massa, a resposta não
> seria
> porque existe a gravidade.
>
> Parece-me então que a maioria das respostas postadas tratam da
> gravidade, o que, por que, relacionado a gravidade, não?
>
> O fóton não tem massa, porem sofre a influencia da gravidade





SUBJECT: Re: Por que massa atrai massa?
FROM: Maria Natália <grasdic@hotmail.com>
TO: ciencialist@yahoogrupos.com.br
DATE: 15/01/2005 14:13


Me esqueci:
Assim como já não se usa o s mas o x em cinemática, estamos falando
actualmente em interacções e não forças.
Maria Natália

--- Em ciencialist@yahoogrupos.com.br, Maria Natália <grasdic@h...>
escreveu
>
> rayfisica:
> A isto se chama ponto de ordem na discussão.
> Peguei na questão inicila e a coloquei a dois níveis de 10º ano e
de
> 12º ano.
> E houve várias respostas. nível 10ª--As forças existem aos pares
foi
> logo a 1ª a aparecer. Essas forças já foram medidas
> experimentalmente foi outra resposta. Estas afirmações pressupõem
> que aluno tem uma progressão em física desde os 13 anos e portanto
> ele lida com o conceito de força mais ou menos bem apreendido. É
que
> o conceito de força é difícil e só a nível das primeiras
abstracções
> se alcança (16 anos).
> Ao nível dos mais adiantados: essa ideia de força e que trás por
> trás misconceptions desaparece com a introdução da noção de campo.
> Cmapo gravítico, campo de velocidades, etc Como sendo o local onde
> existe energia. Portanto uma massa colocada num local que tem
> energia (potencial gravítica) por já lá estar outra massa é de
fácil
> entendimento, aceitação sem grandes bases da matemática mesmo
> vectorial (força é grandeza vectorial). Até com campo se "sente" o
> surgir (quase simultâneo) das duas forças. Mas a velocidade de
> transmissão dá problema e se não tivermos o senso isto vai parar
a
> dinâmica relativistíca e nunca mais ninguém agarra a criança e uma
> pergunta que à partida se destinava a retirar uma dúvida está
> gerando grande confusão pois não temos bases. A relatividade não é
> aquela deusa que sem a matemática vive.
> Mas ainda estamos discutindo nos intervalos de aulas este tema. Aí
> em férias os meninos de nota 10 de física deviam discuti-lo na
praia
> com seus professores. As férias também servem para conversa
informal
> entre professor e seus melhores alunos sobre temas científicos.
> Serve até para vermos até que ponto fomos entendidos.
> Como no laboratório se pode medir essa interacção entre duas
massas?
> É questão para os professores de ensino secundário/médio desta
> lista. Mas antes vou ali ao Leo feiraceciencias.com.br
> Não ajudei? E só travei? Quem sabe?
> Um abraço
> Maria Natália
> PS. desculpem mensagem em branco anterior mas meu gato Max Planck,
> no teclado, quis responder LOLLLLL
>
>
> --- Em ciencialist@yahoogrupos.com.br, "rayfisica"
<rayfisica@y...>
> escreveu
> >
> > --- Em ciencialist@yahoogrupos.com.br, marcelo ferrari
> > <emailferrari@y...> escreveu
> >
> >
> > Pergunta inicial
> >
> > Por que massa atrai massa?
> >
> > Sendo a gravidade uma característica da massa, a resposta não
> > seria
> > porque existe a gravidade.
> >
> > Parece-me então que a maioria das respostas postadas tratam da
> > gravidade, o que, por que, relacionado a gravidade, não?
> >
> > O fóton não tem massa, porem sofre a influencia da gravidade





SUBJECT: Re: [ciencialist] Por que massa atrai massa?
FROM: "Alvaro Augusto \(E\)" <alvaro@electraenergy.com.br>
TO: <ciencialist@yahoogrupos.com.br>
DATE: 15/01/2005 14:52

Caro Victor,

Obrigado pelos seus comentários.

Suas considerações sobre simetrias são importantes. Será interessante se, algum dia, for possível construir uma teoria na qual todas as leis da física decorrem naturalmente da geometria e simetria do universo. Esse era o sonho de Einstein e continua sendo o sonho do pessoal das supercordas.

[ ]s

Alvaro Augusto


----- Original Message -----
From: JVictor
To: ciencialist@yahoogrupos.com.br
Sent: Friday, January 14, 2005 7:56 AM
Subject: Re: [ciencialist] Por que massa atrai massa?




Álvaro: Agora, cabe a pergunta: Por que o universo insiste em "respeitar" princípios de simetria? Bem, a melhor resposta que eu conheço é: Sei lá!

Victor: Tudo leva a crer que o universo faz mais do que "respeitar" os princípios das simetrias. É como se as simetrias fossem inerentes, constitutivas, do universo; pois as leis de conservação, que comandam nossa vidinha e as coisas que fazemos, todas, são o resultado de alguma simetria. As simetrias clarificam as coisas que podem acontecer e "amarram" as que, nem com reza, acontecem! E as simetrias marcham para a simplificação das coisas. Acho que a compulsão de Einstein em busca de princípios simples e gerais talvez se deva a um conhecimento interior profundo dessas nuances exibidas pela natureza.
Agora, depois desse papo, posso dizer com segurança o que são simetrias e quais devam ser as eventuais verdades essenciais por trás delas:

-Sei lá!

Meu, nosso, consolo: ninguém sabe também.

Muito claros teus últimos textos a respeito dessas coisas. Sobre tua impressão sobre o pensamento do grande Feynman, também concordo, acrescentando que ele era um cara pé no chão e operacionalista prá ninguém botar defeito. Não se fazem mais feynman´s como antigamente!

Victor.






[As partes desta mensagem que não continham texto foram removidas]



SUBJECT: Re: [ciencialist] Re: Por que massa atrai massa?
FROM: "Alvaro Augusto \(E\)" <alvaro@electraenergy.com.br>
TO: <ciencialist@yahoogrupos.com.br>
DATE: 15/01/2005 14:54

Caro Ray,

O fóton não tem massa, mas tem energia, e é essa energia que interage com o campo gravitacional.

[ ]s

Alvaro Augusto


----- Original Message -----
From: rayfisica
To: ciencialist@yahoogrupos.com.br
Sent: Saturday, January 15, 2005 10:03 AM
Subject: [ciencialist] Re: Por que massa atrai massa?



--- Em ciencialist@yahoogrupos.com.br, marcelo ferrari
<emailferrari@y...> escreveu


Pergunta inicial

Por que massa atrai massa?

Sendo a gravidade uma característica da massa, a resposta não
seria
porque existe a gravidade.

Parece-me então que a maioria das respostas postadas tratam da
gravidade, o que, por que, relacionado a gravidade, não?

O fóton não tem massa, porem sofre a influencia da gravidade




[As partes desta mensagem que não continham texto foram removidas]



SUBJECT: Re: [ciencialist] Re: Por que massa atrai massa?
FROM: "Alvaro Augusto \(E\)" <alvaro@electraenergy.com.br>
TO: <ciencialist@yahoogrupos.com.br>
DATE: 15/01/2005 15:00

Caro Sérgio,

A comparação entre vento e luz é válida só até certo ponto. É fácil determinar a causa do vento, que são as diferenças de pressão atmosférica. Mas os fótons se movem "empurrados" por quem? Flutuações quânticas do vácuo? Por que o resultado líquido (a luz) se move sempre à mesma velocidade?

Você tem razão. Nós não sabemos o porquê das coisas, e, não sabendo o porquê, não sabemos nada1

[ ]s

Alvaro Augusto
----- Original Message -----
From: Sergio M. M. Taborda
To: ciencialist@yahoogrupos.com.br
Sent: Friday, January 14, 2005 9:16 PM
Subject: [ciencialist] Re: Por que massa atrai massa?



--- Em ciencialist@yahoogrupos.com.br, "Alvaro Augusto \(E\)"
<alvaro@e...> escreveu
> E por que a luz tem velocidade, afinal? Por que os fótons não
conseguem ficar parados no lugar? E será que a velocidade da luz é a
mesma coisa que a velocidade dos fótons?

O que é vento ? são as particulas de ar se movendo. Quando não ha
vento, não ha ar ?
As particulas não desaparecem pq não as vemos (no sentido lato) se
moverem. Um fotão parado não é visivel, luz parada não visivel, tal
como vento parado não é visivel.

O que é a velocidade da luz afinal ? a velocidade de grupo ? de fase ?
Aquilo que chamamos de velocidade da luz, que é a de grupo não tem a
mesma definição que a velocidade de objectos macroscopicos.
A velocidade macorscopica v = s /t = s * (1/t) = s f é uma velocidade
de fase. Mas a velocidade de fase de uma onda não carrega informação.
É essa diferença basilar que diferencia a fisica da luz, da fisica das
outras coisas.
As coisas complicam-se quando sabemos que a luz tem momento , tal como
um objecto macroscopico , mas a definição de momento é algo diferente

para objectos macroscopicos p = m v = m v_fase e sabemos o que é m
(nem que seja por definição)
para a luz p = h lambda = h v_fase / f e não sabemos o que h/f é ,
nem por definição.

Fazemos analogias, e tudo o mais, mas não sabemos pq. Sabemos como,
mas não porquê.

Sérgio Taborda






[As partes desta mensagem que não continham texto foram removidas]



SUBJECT: A igreja católica é a favor da evolução?
FROM: "Amauri Jr" <amaurijunior2@yahoo.com.br>
TO: "Lucimary Vargas" <sangalli@uai.com.br>, "Lenia" <lenia_luz@hotmail.com>, "Laracna" <laracna@pop.com.br>, <Conversa_de_Botequim@yahoogrupos.com.br>, <ciencialist@yahoogrupos.com.br>, <ciencia.2005@grupos.com.br>, <autoiniciacaodaconsciencia_plena@yahoogrupos.com.br>, <acropolis@yahoogrupos.com.br>
DATE: 15/01/2005 23:58

http://www.chamada.com.br/mensagens/artigos/evolucao.shtml


A igreja cat�lica � a favor da evolu��o?

A maioria dos n�o-cat�licos ficou surpresa quando o papa Jo�o Paulo II, num documento enviado � Pontif�cia Academia de Ci�ncias do Vaticano em outubro de 1996, falou a favor da evolu��o. Na verdade, ele estava apenas reiterando a posi��o oficial do catolicismo. Considere os seguintes excertos:

Em sua enc�clica Humani generis [Sobre o G�nero Humano], de 1950, meu predecessor Pio XII j� havia afirmado n�o haver oposi��o entre a evolu��o e a doutrina da f� a respeito do homem... Pio XII enfatizou este ponto essencial: se o corpo humano tem sua origem na mat�ria org�nica pr�-existente, a alma espiritual � imediatamente criada por Deus... O exegeta e o te�logo precisam manter-se informados sobre... as ci�ncias naturais... verdade n�o pode contradizer a verdade...

A teoria da evolu��o... tem sido progressivamente aceita pelos pesquisadores em v�rios campos do conhecimento. A converg�ncia... dos resultados de pesquisas conduzidas independentemente �, em si mesma, um argumento significativo em favor dessa teoria.(1)

Sem d�vida, o fiasco embara�oso do julgamento de Galileu veio � mente do papa quando ele advertiu os te�logos da Igreja a "[se manterem] informados sobre... as ci�ncias naturais..." O papa Urbano VIII amea�ou de tortura um Galileu idoso e muito enfermo se este n�o renunciasse �s alega��es de que a Terra girava em torno do Sol. Ajoelhado diante do Santo Of�cio da Inquisi��o de Roma, temendo pela pr�pria vida, Galileu renunciou � sua "heresia" � mas n�o em seu cora��o. A id�ia, repetidamente afirmada por papas "infal�veis", de que o Sol e todos os corpos celestes giravam em torno da Terra permaneceu como dogma cat�lico oficial at� 1992, quando o Vaticano finalmente admitiu oficialmente que Galileu estava certo.

Para evitar que a ci�ncia continue a fazer de tola a hierarquia "infal�vel" da Igreja, o papa admoestou os te�logos cat�licos a consultarem os cientistas antes de interpretarem as Escrituras. No entanto, Pedro, que os cat�licos insistem ter sido o primeiro papa, declarou que as Escrituras foram inspiradas pelo Esp�rito Santo (2 Pe 1.21). Certamente o Esp�rito Santo n�o precisa da ajuda dos cientistas! Se a B�blia n�o for infal�vel quando fala do que pertence ao campo da ci�ncia, por que confiar nela no que diz respeito a Deus e � salva��o? Edward Daschbach, um sacerdote cat�lico, explica que tomar a B�blia literalmente exigiria admitir que a mulher que se assenta sobre a besta em Apocalipse 17 � a Igreja Cat�lica Romana! Ele escreve:

A Igreja, portanto, n�o aceita... a interpreta��o literal dos primeiros cap�tulos do livro de G�nesis... Quando os que advogam o criacionismo aplicam suas ferramentas fundamentalistas a este �ltimo livro [Apocalipse], a Igreja muitas vezes se torna alvo de veementes ataques.(2)

Protestantes que, como Charles Colson, juntaram for�as com Roma, advogam que o catolicismo concorda com eles sobre a inerr�ncia da B�blia. Pelo contr�rio, o Conc�lio Vaticano II declara: "Da� afirmarmos que a B�blia � livre de erro naquilo que pertence � verdade religiosa revelada para nossa salva��o. N�o � necessariamente livre de erro em outros assuntos (por exemplo, ci�ncias naturais)" [�nfase no original].(3)

Isso n�o � uma quest�o trivial. Se o relato da cria��o em G�nesis n�o � digno de confian�a, o restante da B�blia tamb�m n�o pode ser confi�vel, pois depende desse relato. Al�m disso, prova-se que Cristo n�o era realmente Deus, mas um mero mortal que, tolamente interpretou literalmente a hist�ria de Ad�o e Eva (Mt 19.4-5), e n�o pode, portanto, ser nosso Salvador. O peri�dico The American Atheist [O Ateu Americano] sabe muito bem qual � a quest�o: "Destruam-se Ad�o e Eva e o pecado original, e nos escombros se encontrar�o os restos mortais do Filho de Deus, eliminando-se assim qualquer significado para sua morte."(4)

Em maio de 1982, honrando o centen�rio da morte de Darwin, a Pontif�cia Academia de Ci�ncias do Vaticano publicou a seguinte declara��o: "Grande quantidade de evid�ncias torna a aplica��o do conceito de evolu��o... acima de qualquer discuss�o s�ria".(5) A Nova Enciclop�dia Cat�lica diz:

Especialistas... por mais de cem anos, reuniram as provas necess�rias... a evolu��o est� estabelecida t�o firmemente quanto a ci�ncia � capaz de estabelecer fatos...(6)

Cientistas descartam Darwin!

Nem tanto assim. Um n�mero cada vez maior de cientistas, a maioria deles n�o-crist�os, se op�e � evolu��o. O astr�nomo e matem�tico Sir Fred Hoyle diz: "O mundo cient�fico foi iludido e acabou crendo que a evolu��o fora provada. Nada poderia estar mais longe da verdade".(7) O bi�logo Michael Denton, autor de Evolution: A Theory in Crisis [Evolu��o: Uma Teoria em Crise], diz que a ci�ncia desacreditou t�o completamente o evolucionismo darwiniano que este deveria ser descartado. O professor de matem�tica Wolfgang Smith chama a evolu��o de "um mito metaf�sico... completamente desprovido de aprova��o cient�fica..."(8)

Colin Patterson, paleont�logo-chefe do Museu Brit�nico de Hist�ria Natural, confessou depois de mais de vinte anos envolvido com o movimento evolucionista: "Nada havia que eu realmente conhecesse sobre a evolu��o. � um choque enorme descobrir-se enganado por tanto tempo". Patterson "come�ou a pedir a outros cientistas que lhe apresentassem uma coisa de que tinham certeza sobre a evolu��o." Os bi�logos do Museu Americano de Hist�ria Natural em Nova Iorque ficaram mudos. Diz Patterson:

Experimentei a pergunta com o pessoal da geologia do Museu de Campo de Hist�ria Natural, e a �nica resposta que recebi foi o sil�ncio. Tentei obter resposta dos membros do Semin�rio de Morfologia Evolucionista na Universidade de Chicago, um grupo prestigioso de evolucionistas, e recebi de volta um longo sil�ncio, at� que, por fim, uma pessoa disse: "Eu sei uma coisa � n�o deveria ser ensinada no primeiro e segundo grau."(9)

A despeito disso, no caso Edwards versus Aguillard, 482 U.S. 578 (1978), a Suprema Corte americana decidiu que era inconstitucional que as escolas ensinassem o criacionismo lado a lado com o darwinismo como uma outra teoria de origens. Os evang�licos reclamam com justi�a por ver a evolu��o ensinada como fato nas escolas p�blicas, mas ela tamb�m � ensinada como fato em escolas cat�licas.(10) Na revista The Catholic World Report, Stephen F. Smith escreve: "Na escola arquidiocesana de Washington, fomos ensinados que a teoria da evolu��o de Darwin era t�o verdadeira quanto o evangelho."(11) Michael Behe, bioqu�mico, relembra seus dias em escolas cat�licas:

Fui ensinado... a vida... veio de Deus, e que... a principal explica��o cient�fica de como Ele o fizera era a teoria darwiniana da evolu��o. Eu n�o... via qualquer conflito com o ensino da Igreja.(12)

A evolu��o � matematicamente imposs�vel

Em seu livro The Blind Watchmaker [O Relojoeiro Cego], o zo�logo Richard Dawkins, da Universidade de Oxford, um destacado evolucionista, chama a biologia de "o estudo de coisas complicadas que d�o a apar�ncia de terem sido criadas com algum prop�sito."(13) Sem d�vida! Uma c�lula, a menor unidade viva, chega a ter 100.000 mol�culas, e 10.000 rea��es qu�micas interrelacionadas simult�neas. As c�lulas n�o podem ter surgido por acaso! Dawkins admite que cada c�lula cont�m, no seu n�cleo, um banco de dados digitalmente codificado que � maior... do que a soma de todos os 30 volumes da Enciclop�dia Brit�nica."(14) � imposs�vel sequer imaginar a �nfima probabilidade do acaso criar uma enciclop�dia de 30 volumes! E isso equivale apenas a uma c�lula � e h� trilh�es de c�lulas no corpo humano, milhares de tipos diferentes, operando em relacionamentos incrivelmente complexos e delicadamente equilibrados!

A probabilidade astronomicamente pequena torna a evolu��o matematicamente imposs�vel. Hoyle calculou que a probabilidade da produ��o ocasional apenas das enzimas b�sicas para a produ��o da vida s�o de 1 sobre 1 seguido de 40.000 zeros. Em compara��o, a chance de, por acaso, pegar um �tomo espec�fico em todo o universo seria de apenas 1 sobre 1 seguido de 80 zeros. Mesmo que cada �tomo existente se tornasse outro universo, as chances de pegar um �tomo qualquer em todos esses universos seria de apenas 1 sobre 1 seguido de 160 zeros. Uma chance em 1040.000 s� para produzir as enzimas b�sicas! Mas as enzimas realizam coisas not�veis, e esse fato complica ainda mais o problema da evolu��o com essas chances infinitamente pequenas.

Por que raz�o o sangue s� coagula no ponto de sangramento e n�o dentro das veias e art�rias? E por que p�ra quando cessa o sangramento? Imagine os bilh�es de animais que teriam sangrado at� morrer, ou teriam morrido por uma coagula��o inadequada antes que esse processo incr�vel tivesse sido aperfei�oado por mero acaso! O sistema imunol�gico � ainda mais surpreendente, diz Behe. "A complexidade do sistema garante o insucesso de qualquer explica��o darwiniana..."(15) E assim acontece com centenas de outros sistemas que sustentam a vida. Lembre-se de que esses sistemas precisavam ser operacionais para serem �teis; n�o poderiam ter evolu�do em est�gios.

Em seu excelente livro, publicado em 1996, Darwin�s Black Box [A Caixa Preta de Darwin], Behe documenta a incompreens�vel complexidade da vida em seu n�vel qu�mico celular mais b�sico � uma complexidade inimagin�vel para Darwin. Behe, que afirma que a evolu��o "deveria ser banida",(16) demole a teoria darwiniana oferecendo m�ltiplos exemplos, no n�vel bioqu�mico, de elementos "irredutivelmente complexos" intrincadamente planejados, que nunca poderiam ter evolu�do:

[A evolu��o] n�o pode explicar a origem das complexas estruturas bioqu�micas que sustentam a vida. Sequer tenta explicar... A conclus�o de um plano inteligente flui naturalmente dos pr�prios dados � n�o de livros sagrados nem de cren�as sect�rias.(17)

A evolu��o te�sta contradiz a B�blia

Em apoio ao papa, Donald Devine escreve: "O homem pr�-humano aparentemente existiu por milh�es de anos... Isso n�o � uma refuta��o da B�blia, mas uma confirma��o � pois indica que foi preciso que Deus soprasse nele uma alma antes que o homem pudesse ser homem."(18) Pelo contr�rio! A evolu��o te�sta, que exige ancestrais pr�-humanos para o homem (para os quais nenhuma evid�ncia jamais foi encontrada), n�o contradiz apenas o livro de G�nesis, mas toda a B�blia.

Mois�s afirma que Deus formou Ad�o "do p� da terra", e que depois formou Eva a partir de uma de suas costelas (Gn 2.7, 18-22). Ancestrais pr�-humanos n�o podem ser reconciliados com o relato autenticado por Jesus: "N�o tendes lido que o Criador desde o princ�pio os fez homem e mulher, e que disse: Por esta causa deixar� o homem pai e m�e, e se unir� a sua mulher, tornando-se os dois uma s� carne?" (Mt 19.4-5). Cristo confirma o relato de G�nesis ao cit�-lo em Seu ensino. Paulo tamb�m atesta a veracidade do relato ao declarar que "primeiro foi formado Ad�o, e depois Eva" (1 Tm 2.13-14 � ver tamb�m 1 Co 15.22, 45; Judas 14). Eles n�o eram um par de criaturas pr�-humanas nas quais Deus infundiu almas humanas.

Al�m disso, Paulo afirmou que o pecado entrou no mundo por meio de Ad�o, e pelo pecado a morte (Rm 5.12). Se Ad�o e Eva tivessem tido ancestrais que viveram e morreram por milhares (ou milh�es) de anos de evolu��o at� que Deus os humanizasse, a morte teria operado na terra antes que Ad�o pecasse � uma contradi��o clara do relato de G�nesis, do ensino de Cristo, da prega��o de Paulo e do Evangelho. O cardeal de Nova Iorque, John O�Connor, diz que Ad�o e Eva podem ter sido "animais inferiores".(19)

Evolu��o � uma artimanha sat�nica

A evolu��o, "a mais gorda das vacas sagradas",(20) tem sido uma poderosa ferramenta de Satan�s para convencer milh�es de pessoas de que a B�blia n�o � digna de confian�a. Como afirmou Phillip Johnson, professor de direito em Berkeley: "O �nico prop�sito da hist�ria evolucionista darwiniana �... demonstrar que n�o � necess�ria a exist�ncia pr�via de um ser inteligente...[para] haver a cria��o."(21) Johnson causou um choque no mundo acad�mico em 1991 ao lan�ar seu livro Darwin on Trial [Darwin no Banco nos R�us]. Com a precis�o de um promotor, ele destruiu o darwinismo e acusou os evolucionistas de terem "abandonado o relato verdadeiro e preciso com o qual a ci�ncia estava tradicionalmente compromissada, no seu zelo por extirpar e descartar a religi�o..."(22)

A evolu��o teria preenchido o registro f�ssil de bilh�es de criaturas intermedi�rias, e no entanto nem um sequer desses "elos perdidos" foi encontrado! Imagine a quantidade necess�ria de restos mortais desses milh�es de pequenos incrementos evolutivos ao longo de milh�es de anos para a passagem de guelras a pulm�es, de pernas dianteiras para asas, para produzir est�magos e sistemas digestivos, olhos, rins, c�rebros e sistemas nervosos que se estendessem por todo o corpo, a corrente sangu�nea, o esperma e o �vulo dos mam�feros, o ovo e sua casca para os r�pteis e p�ssaros, etc. A impossibilidade aumenta geometricamente, pois cada um desses sistemas � incrivelmente complexo e n�o poderia evoluir gradativamente, mas precisaria ser funcional para sustentar a vida e ajudar na "sobreviv�ncia" � como seria o caso, por exemplo, do sofisticado sistema de radar dos morcegos.

Quantos milh�es de andorinhas do �rtico morreram afogadas antes que a primeira "aprendesse", por acaso, a navega��o a�rea sobre milhares de quil�metros de oceano? Quantos salm�es se perderam e jamais conseguiram chegar ao riacho em que haviam nascido para desovar antes que essa estranha capacidade fosse desenvolvida? Quantas aranhas morreram de fome antes que o fant�stico mecanismo de cria��o de teias tivesse, por acaso, surgido � e quem teria ensinado as aranhas a usar tal recurso? Quantos ovos de toda esp�cie de ave apodreceram antes que surgisse o instinto de choc�-los? Como foi aprendido e transmitido? H� incont�veis impossibilidades para o acaso.

A preocupa��o atual com as "esp�cies amea�adas" contradiz Darwin. A evolu��o elimina os incapazes. � imposs�vel crer na evolu��o e trabalhar em prol da preserva��o ecol�gica das esp�cies. Como o produto final da evolu��o, o homem deveria, sem miseric�rdia, eliminar todos os rivais na luta pela sobreviv�ncia. As contradi��es s�o intermin�veis.

Em seu �ltimo livro, Reason in the Balance [A Raz�o na Balan�a], Phillip Johnson argumenta que somente a cria��o divina pode explicar a consci�ncia moral do homem. A natureza n�o tem moral. O senso �tico e moral do homem desaprova a evolu��o. Se a evolu��o fosse verdadeira, dever�amos fechar os hospitais, parar a produ��o de rem�dios e permitir que os doentes e os fracos morressem. � imposs�vel reconciliar bondade e compaix�o com a sobreviv�ncia dos mais capazes.

No entanto, o homem � compelido por consci�ncia e compaix�o, prova de que � feito � imagem de um Deus santo e amoroso. Ao rejeitar a massacrante evid�ncia de prop�sito no mundo que o cerca (Rm 1.18-32), e por recusar-se a obedecer �s leis de Deus gravadas em sua consci�ncia (Rm 2.14-15), o homem tornou-se v�tima de seu pr�prio ego e de toda sorte de males. Apesar disso, Deus ama o homem, e em amor e gra�a veio a esta terra pelo nascimento virginal para que, como o Homem perfeito, sem pecado, pudesse morrer em nosso lugar, pagando a penalidade infinita que a Sua pr�pria justi�a exigia pelo pecado. � apenas com base nisso � o pleno pagamento da penalidade do pecado, efetuado por Cristo, e a aceita��o desse pagamento por parte do homem � que este pode se tornar uma nova criatura em Cristo. Vamos permanecer leais a esse Evangelho de Jesus Cristo e � Palavra de Deus que o declara; e vamos lutar com determina��o contra toda tentativa de diluir, perverter ou comprometer a verdade de Deus. (TBC 2/97, traduzido por Carlos Osvaldo Pinto)

Notas:

1.. Papa Jo�o Paulo II, "Mensagem � Pontif�cia Academia de Ci�ncias", L�Osservatore Romano (30 de outubro de 1996), 3.7.
2.. Frei Edward Daschbach, S.V.D., "Catholics and Creationism", Visitor (21 de outubro de 1984), 3.
3.. Vaticano II, Vatican Council II, Divine Revelation (edi��o parafraseada da organiza��o Knights of Columbus), III.I 1e.
4.. The American Atheist (1978), 19, conforme citado em The Christian News (11 de novembro de 1996), 15.
5.. Daschbach, loc. cit.
6.. New Catholic Encyclopedia, vol. 5 (McGraw-Hill, 1967), 689.
7.. George W. Cornell, "Scientist calls Darwin evolution theory absurd", Times-Advocate, 10 de dezembro de 1982, A10.
8.. Wolfgang Smith, Teilhard and the New Religion (Tan Books, 1988), 242.
9.. Thomas E. Woodward, "Doubts About Darwin", Moody Monthly (setembro de 1988), 20.
10.. The Times Picayune (Fl�rida, 25 de outubro de 1996), A-30.
11.. Stephen F. Smith, "Is Darwinism a Religion?", The Catholic World Report (dezembro de 1996), 50.
12.. William Bole, "Of Biochemistry and Belief", Our Sunday Visitor (1 de dezembro de 1996), 6.
13.. Richard Dawkins, The Blind Watchmaker (England: Longman, 1986), 1.
14.. Dawkins, op. cit., 18.
15.. Michael J. Behe, Darwin�s Black Box: The Biochemical Challenge to Evolution (The Free Press, 1996), 139.
16.. Ibid., 186.
17.. Ibid., 192-93. 18. Donald Devine, Human Events (13 de dezembro de 1996), 19.
18.. Los Angeles Times (30 de novembro de 1996), B13.
19.. Doug Bandow, "Fossils and Fallacies", National Review (29 de abril de 1991), 47.
20.. Russell Schoch, "The Evolution of a Creationist", California Monthly (novembro de 1991), 22.
21.. The Catholic World Report (dezembro de 1996), 50.
Chamada da Meia-Noite, janeiro de 1998



[As partes desta mensagem que n�o continham texto foram removidas]



SUBJECT: foton parado?
FROM: "rayfisica" <rayfisica@yahoo.com.br>
TO: ciencialist@yahoogrupos.com.br
DATE: 16/01/2005 10:30


SERGIO > Um fotão parado não é visivel, luz parada não visivel, tal
como vento
parado não é visivel.

É possível no nosso universo que um fóton fique com movimento zero?

É possível algo no nosso universo com movimento zero?

Nosso universo = acontecimento que já aconteceu e foi observado
(ainda que não visto)

grato

O que seria, vento parado?





SUBJECT: Intervalo ...musical para os Amigos!
FROM: Maria Natália <grasdic@hotmail.com>
TO: ciencialist@yahoogrupos.com.br
DATE: 16/01/2005 12:12


http://planetary.org/sounds/huygens_sounds.html
Me digam se gostaram
Um abraço do longínquo...sistema solar
Maria Natália





SUBJECT: Professores de coragem se precisam...Alguém aí?
FROM: Maria Natália <grasdic@hotmail.com>
TO: ciencialist@yahoogrupos.com.br
DATE: 16/01/2005 13:58


Para quem tem massas e está de férias escolares aqui está a
oportunidade de sair do Brasil:

"EDUCATORS WANTED FOR EARTH EXPEDITIONS 2005: PROJECT DRAGONFLY
http://www.earthexpeditions.org/
We're getting the word out that Earth Expeditions from Miami
University's Project Dragonfly and The Cincinnati Zoo & Botanical
Garden is accepting applications now for its 2005 tuition-free
graduate courses that take place overseas and at the Cincinnati Zoo.
Field Expedition courses take both formal and informal educators to
signature Zoo conservation sites in Belize, Thailand, Namibia, Costa
Rica, and Trinidad. Join cheetah conservation efforts in Namibia,
explore neotropical ecology in Costa Rica, or help reintroduce blue-
and-gold macaws in Trinidad. New additions for 2005 are
opportunities to study Buddhism and conservation in Thailand or
manatees in Belize. Zoo Expedition courses take place on Cincinnati
Zoo grounds. Educators and their students investigate primate
behavior, adaptation, and plants. Field Expedition courses have an
application deadline, while Zoo Expedition courses are filled on a
first-come, first-served basis."

Só nos EUA,né?
sds
Maria Natália








SUBJECT: Re: [ciencialist] A igreja católica é a favor da evolução?
FROM: "JVictor" <jvoneto@uol.com.br>
TO: <ciencialist@yahoogrupos.com.br>
DATE: 16/01/2005 14:55


Texto: A Igreja, portanto, não aceita... a interpretação literal dos
primeiros capítulos do livro de Gênesis... Quando os que advogam o
criacionismo aplicam suas ferramentas fundamentalistas a este último livro
[Apocalipse], a Igreja muitas vezes se torna alvo de veementes ataques.(2)

Victor: Não sou muito ligado nesses assuntos. Mas já fui.
E sei que, em relação à interpretação, entre outras, há as variantes:
a simbólica, a literal e a alegórica. A Igreja Católica e sua mais famosa
dissidência, a Protestante, adotam a interpretação literal, onde pregam que
a bíblia está certa, prevalecendo a inerrância da bíblia, como dogma
fundamental. Ou seja: a Bíblia está cem por cento certa, " todas as palavras
foram inspiradas por Deus, todos os livros estão exatamente como foram
originariamente escritos, e estão isentos de erro em cada detalhe." (Uma
análise crítica da Bíblia-C.G.Shalders).
Mas, em minha opinião pessoal-e respeito a de todos os outros- ela não foi
ditada por algum espírito santo coisa nenhuma. A bíblia, segundo
pesquisadores, foi escrita num espaço de tempo estimado entre 1000 a 1.700
anos. Fica imediato concluir que ela não foi escrita por uma só pessoa, ou
um grupo homogêneo de pessoas. Assim, seus autores seguramente eram pessoas
que não se conheciam, viveram em épocas diferentes, eram de culturas
diferentes, formações e costumes diferentes. Segundo ela própria, Zacarias e
Jeremias eram profetas(viviam fazendo medo ao povo...); Davi e Salomão eram
reis(um destes, não lembro qual, teve ao longo de sua vida cerca de 700
concubinas. Devia ser um auê desgraçado para dar conta de tanto " trabalho".
Nos moldes bíblicos, teve ter " pecado" um bocado. Mas no fim da vida,
exausto, sem viagra por perto, arrependeu-se e escreveu muitas coisas
virtuosas, no desejo de se resssantificar... (Tinha muita graça! Até que o
cara não era lá muito besta.); Lucas, era um médico; Mateus, era um cobrador
de impostos do governo de então; Paulo era um cara versado nas leis
religiosas; João, Pedro e Tiago viviam catando os bichinhos do mar, como
profissão; outro, Amós, era lavrador. E assim por diante. Estima-se que se
passaram cerca de cinco gerações até que o contexto estivesse pronto e
acabado. Pesquisas também apontam para o seguinte: os livros do Velho
Testamento existiam na literatura oral do povo judeu. Esdras, um dos
bíblicos, segundo as pesquisas foi quem reuniu esse conhecimento oral,
existentes na memória das pessoas(particularmente, acho isto uma tarefa
descomunal, pois, hoje, um batalhão de repórteres, especialmente treinados,
não daria conta de tantas entrevistas que teriam a fazer, depois juntar
tudo e separar o o contexto atual desse bestseller. Quanto mais um cara
só!). Mas dizem que ele foi à praça pública e disse que aquela era a lei
judaica e teria sido ditada por Deus. Pronto. Daí por diante, prevaleceu
aquela " constituição divina". C.G.Salders, que foi pastor portestante por
meio século, avisa que a "bíblia é um livro humano como outro qualquer".
(No que eu, na minha ignorância, concordo em gênero, número e grau).Contém
ensinamentos bons, e muitos outros que não passam de palha seca; ela se
contradiz em muitos pontos, em outros violenta o bom senso, contradiz a
ciência, e em muitos trechos ainda, é positivamente iníqua. Se fôsse
recebida como livro meramente humano, poder-se-ia tirar dela o que tem de
útil, o que de bom e deixar de lado o que não presta; mas aceitando-a como a
Palavra de Deus, como livro inspirado por Deus, resulta confusão." É é
precisamente dessa confusão, que desnorteia quem raciocina, que surgem
tantas interpretações diferentes, o que gerou o montão de seitas que aí
estão, enganando o povo, de forma irremediável e perene! Cruzes.
Mas, segundo as Igrejas, a interpretação é mesmo literal. Mas, como é
literal, mesmo ante tantos furos? Eu, hein?
Um dia eu lí: "...enquanto a maioria das pessoas vive se preocupando com o
fato de Deus existir ou não, sua curta vida vai se acabando..." Avaliem só o
profundo significado prático desse ensinamento. Quem proferiu semelhante
pérola, foi um gordinho chamado Buda. Cara sabido. Que Deus o tenha!


Sds,

Victor.




----- Original Message -----
From: "Amauri Jr" <amaurijunior2@yahoo.com.br>
To: "Lucimary Vargas" <sangalli@uai.com.br>; "Lenia"
<lenia_luz@hotmail.com>; "Laracna" <laracna@pop.com.br>;
<Conversa_de_Botequim@yahoogrupos.com.br>; <ciencialist@yahoogrupos.com.br>;
<ciencia.2005@grupos.com.br>;
<autoiniciacaodaconsciencia_plena@yahoogrupos.com.br>;
<acropolis@yahoogrupos.com.br>
Sent: Saturday, January 15, 2005 11:58 PM
Subject: [ciencialist] A igreja católica é a favor da evolução?



http://www.chamada.com.br/mensagens/artigos/evolucao.shtml


A igreja católica é a favor da evolução?

A maioria dos não-católicos ficou surpresa quando o papa João Paulo
II, num documento enviado à Pontifícia Academia de Ciências do Vaticano em
outubro de 1996, falou a favor da evolução. Na verdade, ele estava apenas
reiterando a posição oficial do catolicismo. Considere os seguintes
excertos:

Em sua encíclica Humani generis [Sobre o Gênero Humano], de 1950, meu
predecessor Pio XII já havia afirmado não haver oposição entre a evolução e
a doutrina da fé a respeito do homem... Pio XII enfatizou este ponto
essencial: se o corpo humano tem sua origem na matéria orgânica
pré-existente, a alma espiritual é imediatamente criada por Deus... O
exegeta e o teólogo precisam manter-se informados sobre... as ciências
naturais... verdade não pode contradizer a verdade...

A teoria da evolução... tem sido progressivamente aceita pelos
pesquisadores em vários campos do conhecimento. A convergência... dos
resultados de pesquisas conduzidas independentemente é, em si mesma, um
argumento significativo em favor dessa teoria.(1)

Sem dúvida, o fiasco embaraçoso do julgamento de Galileu veio à mente
do papa quando ele advertiu os teólogos da Igreja a "[se manterem]
informados sobre... as ciências naturais..." O papa Urbano VIII ameaçou de
tortura um Galileu idoso e muito enfermo se este não renunciasse às
alegações de que a Terra girava em torno do Sol. Ajoelhado diante do Santo
Ofício da Inquisição de Roma, temendo pela própria vida, Galileu renunciou à
sua "heresia" - mas não em seu coração. A idéia, repetidamente afirmada por
papas "infalíveis", de que o Sol e todos os corpos celestes giravam em torno
da Terra permaneceu como dogma católico oficial até 1992, quando o Vaticano
finalmente admitiu oficialmente que Galileu estava certo.

Para evitar que a ciência continue a fazer de tola a hierarquia
"infalível" da Igreja, o papa admoestou os teólogos católicos a consultarem
os cientistas antes de interpretarem as Escrituras. No entanto, Pedro, que
os católicos insistem ter sido o primeiro papa, declarou que as Escrituras
foram inspiradas pelo Espírito Santo (2 Pe 1.21). Certamente o Espírito
Santo não precisa da ajuda dos cientistas! Se a Bíblia não for infalível
quando fala do que pertence ao campo da ciência, por que confiar nela no que
diz respeito a Deus e à salvação? Edward Daschbach, um sacerdote católico,
explica que tomar a Bíblia literalmente exigiria admitir que a mulher que se
assenta sobre a besta em Apocalipse 17 é a Igreja Católica Romana! Ele
escreve:

A Igreja, portanto, não aceita... a interpretação literal dos
primeiros capítulos do livro de Gênesis... Quando os que advogam o
criacionismo aplicam suas ferramentas fundamentalistas a este último livro
[Apocalipse], a Igreja muitas vezes se torna alvo de veementes ataques.(2)

Protestantes que, como Charles Colson, juntaram forças com Roma,
advogam que o catolicismo concorda com eles sobre a inerrância da Bíblia.
Pelo contrário, o Concílio Vaticano II declara: "Daí afirmarmos que a Bíblia
é livre de erro naquilo que pertence à verdade religiosa revelada para nossa
salvação. Não é necessariamente livre de erro em outros assuntos (por
exemplo, ciências naturais)" [ênfase no original].(3)

Isso não é uma questão trivial. Se o relato da criação em Gênesis não
é digno de confiança, o restante da Bíblia também não pode ser confiável,
pois depende desse relato. Além disso, prova-se que Cristo não era realmente
Deus, mas um mero mortal que, tolamente interpretou literalmente a história
de Adão e Eva (Mt 19.4-5), e não pode, portanto, ser nosso Salvador. O
periódico The American Atheist [O Ateu Americano] sabe muito bem qual é a
questão: "Destruam-se Adão e Eva e o pecado original, e nos escombros se
encontrarão os restos mortais do Filho de Deus, eliminando-se assim qualquer
significado para sua morte."(4)

Em maio de 1982, honrando o centenário da morte de Darwin, a
Pontifícia Academia de Ciências do Vaticano publicou a seguinte declaração:
"Grande quantidade de evidências torna a aplicação do conceito de
evolução... acima de qualquer discussão séria".(5) A Nova Enciclopédia
Católica diz:

Especialistas... por mais de cem anos, reuniram as provas
necessárias... a evolução está estabelecida tão firmemente quanto a ciência
é capaz de estabelecer fatos...(6)

Cientistas descartam Darwin!

Nem tanto assim. Um número cada vez maior de cientistas, a maioria
deles não-cristãos, se opõe à evolução. O astrônomo e matemático Sir Fred
Hoyle diz: "O mundo científico foi iludido e acabou crendo que a evolução
fora provada. Nada poderia estar mais longe da verdade".(7) O biólogo
Michael Denton, autor de Evolution: A Theory in Crisis [Evolução: Uma Teoria
em Crise], diz que a ciência desacreditou tão completamente o evolucionismo
darwiniano que este deveria ser descartado. O professor de matemática
Wolfgang Smith chama a evolução de "um mito metafísico... completamente
desprovido de aprovação científica..."(8)

Colin Patterson, paleontólogo-chefe do Museu Britânico de História
Natural, confessou depois de mais de vinte anos envolvido com o movimento
evolucionista: "Nada havia que eu realmente conhecesse sobre a evolução. É
um choque enorme descobrir-se enganado por tanto tempo". Patterson "começou
a pedir a outros cientistas que lhe apresentassem uma coisa de que tinham
certeza sobre a evolução." Os biólogos do Museu Americano de História
Natural em Nova Iorque ficaram mudos. Diz Patterson:

Experimentei a pergunta com o pessoal da geologia do Museu de Campo de
História Natural, e a única resposta que recebi foi o silêncio. Tentei obter
resposta dos membros do Seminário de Morfologia Evolucionista na
Universidade de Chicago, um grupo prestigioso de evolucionistas, e recebi de
volta um longo silêncio, até que, por fim, uma pessoa disse: "Eu sei uma
coisa - não deveria ser ensinada no primeiro e segundo grau."(9)

A despeito disso, no caso Edwards versus Aguillard, 482 U.S. 578
(1978), a Suprema Corte americana decidiu que era inconstitucional que as
escolas ensinassem o criacionismo lado a lado com o darwinismo como uma
outra teoria de origens. Os evangélicos reclamam com justiça por ver a
evolução ensinada como fato nas escolas públicas, mas ela também é ensinada
como fato em escolas católicas.(10) Na revista The Catholic World Report,
Stephen F. Smith escreve: "Na escola arquidiocesana de Washington, fomos
ensinados que a teoria da evolução de Darwin era tão verdadeira quanto o
evangelho."(11) Michael Behe, bioquímico, relembra seus dias em escolas
católicas:

Fui ensinado... a vida... veio de Deus, e que... a principal
explicação científica de como Ele o fizera era a teoria darwiniana da
evolução. Eu não... via qualquer conflito com o ensino da Igreja.(12)

A evolução é matematicamente impossível

Em seu livro The Blind Watchmaker [O Relojoeiro Cego], o zoólogo
Richard Dawkins, da Universidade de Oxford, um destacado evolucionista,
chama a biologia de "o estudo de coisas complicadas que dão a aparência de
terem sido criadas com algum propósito."(13) Sem dúvida! Uma célula, a menor
unidade viva, chega a ter 100.000 moléculas, e 10.000 reações químicas
interrelacionadas simultâneas. As células não podem ter surgido por acaso!
Dawkins admite que cada célula contém, no seu núcleo, um banco de dados
digitalmente codificado que é maior... do que a soma de todos os 30 volumes
da Enciclopédia Britânica."(14) É impossível sequer imaginar a ínfima
probabilidade do acaso criar uma enciclopédia de 30 volumes! E isso equivale
apenas a uma célula - e há trilhões de células no corpo humano, milhares de
tipos diferentes, operando em relacionamentos incrivelmente complexos e
delicadamente equilibrados!

A probabilidade astronomicamente pequena torna a evolução
matematicamente impossível. Hoyle calculou que a probabilidade da produção
ocasional apenas das enzimas básicas para a produção da vida são de 1 sobre
1 seguido de 40.000 zeros. Em comparação, a chance de, por acaso, pegar um
átomo específico em todo o universo seria de apenas 1 sobre 1 seguido de 80
zeros. Mesmo que cada átomo existente se tornasse outro universo, as chances
de pegar um átomo qualquer em todos esses universos seria de apenas 1 sobre
1 seguido de 160 zeros. Uma chance em 1040.000 só para produzir as enzimas
básicas! Mas as enzimas realizam coisas notáveis, e esse fato complica ainda
mais o problema da evolução com essas chances infinitamente pequenas.

Por que razão o sangue só coagula no ponto de sangramento e não dentro
das veias e artérias? E por que pára quando cessa o sangramento? Imagine os
bilhões de animais que teriam sangrado até morrer, ou teriam morrido por uma
coagulação inadequada antes que esse processo incrível tivesse sido
aperfeiçoado por mero acaso! O sistema imunológico é ainda mais
surpreendente, diz Behe. "A complexidade do sistema garante o insucesso de
qualquer explicação darwiniana..."(15) E assim acontece com centenas de
outros sistemas que sustentam a vida. Lembre-se de que esses sistemas
precisavam ser operacionais para serem úteis; não poderiam ter evoluído em
estágios.

Em seu excelente livro, publicado em 1996, Darwin's Black Box [A Caixa
Preta de Darwin], Behe documenta a incompreensível complexidade da vida em
seu nível químico celular mais básico - uma complexidade inimaginável para
Darwin. Behe, que afirma que a evolução "deveria ser banida",(16) demole a
teoria darwiniana oferecendo múltiplos exemplos, no nível bioquímico, de
elementos "irredutivelmente complexos" intrincadamente planejados, que nunca
poderiam ter evoluído:

[A evolução] não pode explicar a origem das complexas estruturas
bioquímicas que sustentam a vida. Sequer tenta explicar... A conclusão de um
plano inteligente flui naturalmente dos próprios dados - não de livros
sagrados nem de crenças sectárias.(17)

A evolução teísta contradiz a Bíblia

Em apoio ao papa, Donald Devine escreve: "O homem pré-humano
aparentemente existiu por milhões de anos... Isso não é uma refutação da
Bíblia, mas uma confirmação - pois indica que foi preciso que Deus soprasse
nele uma alma antes que o homem pudesse ser homem."(18) Pelo contrário! A
evolução teísta, que exige ancestrais pré-humanos para o homem (para os
quais nenhuma evidência jamais foi encontrada), não contradiz apenas o livro
de Gênesis, mas toda a Bíblia.

Moisés afirma que Deus formou Adão "do pó da terra", e que depois
formou Eva a partir de uma de suas costelas (Gn 2.7, 18-22). Ancestrais
pré-humanos não podem ser reconciliados com o relato autenticado por Jesus:
"Não tendes lido que o Criador desde o princípio os fez homem e mulher, e
que disse: Por esta causa deixará o homem pai e mãe, e se unirá a sua
mulher, tornando-se os dois uma só carne?" (Mt 19.4-5). Cristo confirma o
relato de Gênesis ao citá-lo em Seu ensino. Paulo também atesta a veracidade
do relato ao declarar que "primeiro foi formado Adão, e depois Eva" (1 Tm
2.13-14 - ver também 1 Co 15.22, 45; Judas 14). Eles não eram um par de
criaturas pré-humanas nas quais Deus infundiu almas humanas.

Além disso, Paulo afirmou que o pecado entrou no mundo por meio de
Adão, e pelo pecado a morte (Rm 5.12). Se Adão e Eva tivessem tido
ancestrais que viveram e morreram por milhares (ou milhões) de anos de
evolução até que Deus os humanizasse, a morte teria operado na terra antes
que Adão pecasse - uma contradição clara do relato de Gênesis, do ensino de
Cristo, da pregação de Paulo e do Evangelho. O cardeal de Nova Iorque, John
O'Connor, diz que Adão e Eva podem ter sido "animais inferiores".(19)

Evolução - uma artimanha satânica

A evolução, "a mais gorda das vacas sagradas",(20) tem sido uma
poderosa ferramenta de Satanás para convencer milhões de pessoas de que a
Bíblia não é digna de confiança. Como afirmou Phillip Johnson, professor de
direito em Berkeley: "O único propósito da história evolucionista darwiniana
é... demonstrar que não é necessária a existência prévia de um ser
inteligente...[para] haver a criação."(21) Johnson causou um choque no mundo
acadêmico em 1991 ao lançar seu livro Darwin on Trial [Darwin no Banco nos
Réus]. Com a precisão de um promotor, ele destruiu o darwinismo e acusou os
evolucionistas de terem "abandonado o relato verdadeiro e preciso com o qual
a ciência estava tradicionalmente compromissada, no seu zelo por extirpar e
descartar a religião..."(22)

A evolução teria preenchido o registro fóssil de bilhões de criaturas
intermediárias, e no entanto nem um sequer desses "elos perdidos" foi
encontrado! Imagine a quantidade necessária de restos mortais desses milhões
de pequenos incrementos evolutivos ao longo de milhões de anos para a
passagem de guelras a pulmões, de pernas dianteiras para asas, para produzir
estômagos e sistemas digestivos, olhos, rins, cérebros e sistemas nervosos
que se estendessem por todo o corpo, a corrente sanguínea, o esperma e o
óvulo dos mamíferos, o ovo e sua casca para os répteis e pássaros, etc. A
impossibilidade aumenta geometricamente, pois cada um desses sistemas é
incrivelmente complexo e não poderia evoluir gradativamente, mas precisaria
ser funcional para sustentar a vida e ajudar na "sobrevivência" - como seria
o caso, por exemplo, do sofisticado sistema de radar dos morcegos.

Quantos milhões de andorinhas do Ártico morreram afogadas antes que a
primeira "aprendesse", por acaso, a navegação aérea sobre milhares de
quilômetros de oceano? Quantos salmões se perderam e jamais conseguiram
chegar ao riacho em que haviam nascido para desovar antes que essa estranha
capacidade fosse desenvolvida? Quantas aranhas morreram de fome antes que o
fantástico mecanismo de criação de teias tivesse, por acaso, surgido - e
quem teria ensinado as aranhas a usar tal recurso? Quantos ovos de toda
espécie de ave apodreceram antes que surgisse o instinto de chocá-los? Como
foi aprendido e transmitido? Há incontáveis impossibilidades para o acaso.

A preocupação atual com as "espécies ameaçadas" contradiz Darwin. A
evolução elimina os incapazes. É impossível crer na evolução e trabalhar em
prol da preservação ecológica das espécies. Como o produto final da
evolução, o homem deveria, sem misericórdia, eliminar todos os rivais na
luta pela sobrevivência. As contradições são intermináveis.

Em seu último livro, Reason in the Balance [A Razão na Balança],
Phillip Johnson argumenta que somente a criação divina pode explicar a
consciência moral do homem. A natureza não tem moral. O senso ético e moral
do homem desaprova a evolução. Se a evolução fosse verdadeira, deveríamos
fechar os hospitais, parar a produção de remédios e permitir que os doentes
e os fracos morressem. É impossível reconciliar bondade e compaixão com a
sobrevivência dos mais capazes.

No entanto, o homem é compelido por consciência e compaixão, prova de
que é feito à imagem de um Deus santo e amoroso. Ao rejeitar a massacrante
evidência de propósito no mundo que o cerca (Rm 1.18-32), e por recusar-se a
obedecer às leis de Deus gravadas em sua consciência (Rm 2.14-15), o homem
tornou-se vítima de seu próprio ego e de toda sorte de males. Apesar disso,
Deus ama o homem, e em amor e graça veio a esta terra pelo nascimento
virginal para que, como o Homem perfeito, sem pecado, pudesse morrer em
nosso lugar, pagando a penalidade infinita que a Sua própria justiça exigia
pelo pecado. É apenas com base nisso - o pleno pagamento da penalidade do
pecado, efetuado por Cristo, e a aceitação desse pagamento por parte do
homem - que este pode se tornar uma nova criatura em Cristo. Vamos
permanecer leais a esse Evangelho de Jesus Cristo e à Palavra de Deus que o
declara; e vamos lutar com determinação contra toda tentativa de diluir,
perverter ou comprometer a verdade de Deus. (TBC 2/97, traduzido por Carlos
Osvaldo Pinto)

Notas:

1.. Papa João Paulo II, "Mensagem à Pontifícia Academia de
Ciências", L'Osservatore Romano (30 de outubro de 1996), 3.7.
2.. Frei Edward Daschbach, S.V.D., "Catholics and Creationism",
Visitor (21 de outubro de 1984), 3.
3.. Vaticano II, Vatican Council II, Divine Revelation (edição
parafraseada da organização Knights of Columbus), III.I 1e.
4.. The American Atheist (1978), 19, conforme citado em The
Christian News (11 de novembro de 1996), 15.
5.. Daschbach, loc. cit.
6.. New Catholic Encyclopedia, vol. 5 (McGraw-Hill, 1967), 689.
7.. George W. Cornell, "Scientist calls Darwin evolution theory
absurd", Times-Advocate, 10 de dezembro de 1982, A10.
8.. Wolfgang Smith, Teilhard and the New Religion (Tan Books, 1988),
242.
9.. Thomas E. Woodward, "Doubts About Darwin", Moody Monthly
(setembro de 1988), 20.
10.. The Times Picayune (Flórida, 25 de outubro de 1996), A-30.
11.. Stephen F. Smith, "Is Darwinism a Religion?", The Catholic
World Report (dezembro de 1996), 50.
12.. William Bole, "Of Biochemistry and Belief", Our Sunday Visitor
(1 de dezembro de 1996), 6.
13.. Richard Dawkins, The Blind Watchmaker (England: Longman, 1986),
1.
14.. Dawkins, op. cit., 18.
15.. Michael J. Behe, Darwin's Black Box: The Biochemical Challenge
to Evolution (The Free Press, 1996), 139.
16.. Ibid., 186.
17.. Ibid., 192-93. 18. Donald Devine, Human Events (13 de dezembro
de 1996), 19.
18.. Los Angeles Times (30 de novembro de 1996), B13.
19.. Doug Bandow, "Fossils and Fallacies", National Review (29 de
abril de 1991), 47.
20.. Russell Schoch, "The Evolution of a Creationist", California
Monthly (novembro de 1991), 22.
21.. The Catholic World Report (dezembro de 1996), 50.
Chamada da Meia-Noite, janeiro de 1998



[As partes desta mensagem que não continham texto foram removidas]



##### ##### #####

Para saber mais visite
http://www.ciencialist.hpg.ig.com.br


##### ##### ##### #####
Links do Yahoo! Grupos












SUBJECT: Re: A igreja católica é a favor da evolução?
FROM: Maria Natália <grasdic@hotmail.com>
TO: ciencialist@yahoogrupos.com.br
DATE: 16/01/2005 23:52


JVictor:

Este texto caía que nem ginjas numa lista de filosofia com ministros
do senhor, frades, freiras, monjes, seminaristas e noviças e claro
ateus.
Aqui espero que seja lido por pessoas de Ciência e como opinião
sobre o livro de Ciência com o título Bíblia e que está nos top há
uns anos a esta parte.
Penso o mesmo sobre o autores dos vários volumes.
Em tempos em que era obrigatória a aula de moral e educação
religiosa nunca me foi ensinado criacionismos e tive sempre boas
professores de biologia. Estou com Darwin e felizmente em Portugal
este problema não se levanta nas escolas. O exame final é nacional e
se "não sabes não tens nota para entrar em medicina". Ponto final
parágrafo. Certamente por ser química e ver a vida reduzida a umas
tantas moléculas e à transmissão de um código.
Um abraço
Maria Natália

--- Em ciencialist@yahoogrupos.com.br, "JVictor" <jvoneto@u...>
escreveu
>
> Texto: A Igreja, portanto, não aceita... a interpretação literal
dos
> primeiros capítulos do livro de Gênesis... Quando os que advogam o
> criacionismo aplicam suas ferramentas fundamentalistas a este
último livro
> [Apocalipse], a Igreja muitas vezes se torna alvo de veementes
ataques.(2)
>
> Victor: Não sou muito ligado nesses assuntos. Mas já fui.
> E sei que, em relação à interpretação, entre outras, há as
variantes:
> a simbólica, a literal e a alegórica. A Igreja Católica e sua
mais famosa
> dissidência, a Protestante, adotam a interpretação literal, onde
pregam que
> a bíblia está certa, prevalecendo a inerrância da bíblia, como
dogma
> fundamental. Ou seja: a Bíblia está cem por cento certa, " todas
as palavras
> foram inspiradas por Deus, todos os livros estão exatamente como
foram
> originariamente escritos, e estão isentos de erro em cada
detalhe." (Uma
> análise crítica da Bíblia-C.G.Shalders).
> Mas, em minha opinião pessoal-e respeito a de todos os outros- ela
não foi
> ditada por algum espírito santo coisa nenhuma. A bíblia, segundo
> pesquisadores, foi escrita num espaço de tempo estimado entre 1000
a 1.700
> anos. Fica imediato concluir que ela não foi escrita por uma só
pessoa, ou
> um grupo homogêneo de pessoas. Assim, seus autores seguramente
eram pessoas
> que não se conheciam, viveram em épocas diferentes, eram de
culturas
> diferentes, formações e costumes diferentes. Segundo ela própria,
Zacarias e
> Jeremias eram profetas(viviam fazendo medo ao povo...); Davi e
Salomão eram
> reis(um destes, não lembro qual, teve ao longo de sua vida cerca
de 700
> concubinas. Devia ser um auê desgraçado para dar conta de tanto "
trabalho".
> Nos moldes bíblicos, teve ter " pecado" um bocado. Mas no fim da
vida,
> exausto, sem viagra por perto, arrependeu-se e escreveu muitas
coisas
> virtuosas, no desejo de se resssantificar... (Tinha muita graça!
Até que o
> cara não era lá muito besta.); Lucas, era um médico; Mateus, era
um cobrador
> de impostos do governo de então; Paulo era um cara versado nas
leis
> religiosas; João, Pedro e Tiago viviam catando os bichinhos do
mar, como
> profissão; outro, Amós, era lavrador. E assim por diante. Estima-
se que se
> passaram cerca de cinco gerações até que o contexto estivesse
pronto e
> acabado. Pesquisas também apontam para o seguinte: os livros do
Velho
> Testamento existiam na literatura oral do povo judeu. Esdras, um
dos
> bíblicos, segundo as pesquisas foi quem reuniu esse conhecimento
oral,
> existentes na memória das pessoas(particularmente, acho isto uma
tarefa
> descomunal, pois, hoje, um batalhão de repórteres, especialmente
treinados,
> não daria conta de tantas entrevistas que teriam a fazer, depois
juntar
> tudo e separar o o contexto atual desse bestseller. Quanto mais
um cara
> só!). Mas dizem que ele foi à praça pública e disse que aquela era
a lei
> judaica e teria sido ditada por Deus. Pronto. Daí por diante,
prevaleceu
> aquela " constituição divina". C.G.Salders, que foi pastor
portestante por
> meio século, avisa que a "bíblia é um livro humano como outro
qualquer".
> (No que eu, na minha ignorância, concordo em gênero, número e
grau).Contém
> ensinamentos bons, e muitos outros que não passam de palha seca;
ela se
> contradiz em muitos pontos, em outros violenta o bom senso,
contradiz a
> ciência, e em muitos trechos ainda, é positivamente iníqua. Se
fôsse
> recebida como livro meramente humano, poder-se-ia tirar dela o que
tem de
> útil, o que de bom e deixar de lado o que não presta; mas
aceitando-a como a
> Palavra de Deus, como livro inspirado por Deus, resulta
confusão." É é
> precisamente dessa confusão, que desnorteia quem raciocina, que
surgem
> tantas interpretações diferentes, o que gerou o montão de seitas
que aí
> estão, enganando o povo, de forma irremediável e perene! Cruzes.
> Mas, segundo as Igrejas, a interpretação é mesmo literal. Mas,
como é
> literal, mesmo ante tantos furos? Eu, hein?
> Um dia eu lí: "...enquanto a maioria das pessoas vive se
preocupando com o
> fato de Deus existir ou não, sua curta vida vai se acabando..."
Avaliem só o
> profundo significado prático desse ensinamento. Quem proferiu
semelhante
> pérola, foi um gordinho chamado Buda. Cara sabido. Que Deus o
tenha!
>
>
> Sds,
>
> Victor.
>
>
>
>
> ----- Original Message -----
> From: "Amauri Jr" <amaurijunior2@y...>
> To: "Lucimary Vargas" <sangalli@u...>; "Lenia"
> <lenia_luz@h...>; "Laracna" <laracna@p...>;
> <Conversa_de_Botequim@yahoogrupos.com.br>;
<ciencialist@yahoogrupos.com.br>;
> <ciencia.2005@g...>;
> <autoiniciacaodaconsciencia_plena@yahoogrupos.com.br>;
> <acropolis@yahoogrupos.com.br>
> Sent: Saturday, January 15, 2005 11:58 PM
> Subject: [ciencialist] A igreja católica é a favor da evolução?
>
>
>
> http://www.chamada.com.br/mensagens/artigos/evolucao.shtml
>
>
> A igreja católica é a favor da evolução?
>
> A maioria dos não-católicos ficou surpresa quando o papa
João Paulo
> II, num documento enviado à Pontifícia Academia de Ciências do
Vaticano em
> outubro de 1996, falou a favor da evolução. Na verdade, ele estava
apenas
> reiterando a posição oficial do catolicismo. Considere os
seguintes
> excertos:
>
> Em sua encíclica Humani generis [Sobre o Gênero Humano], de
1950, meu
> predecessor Pio XII já havia afirmado não haver oposição entre a
evolução e
> a doutrina da fé a respeito do homem... Pio XII enfatizou este
ponto
> essencial: se o corpo humano tem sua origem na matéria orgânica
> pré-existente, a alma espiritual é imediatamente criada por
Deus... O
> exegeta e o teólogo precisam manter-se informados sobre... as
ciências
> naturais... verdade não pode contradizer a verdade...
>
> A teoria da evolução... tem sido progressivamente aceita
pelos
> pesquisadores em vários campos do conhecimento. A convergência...
dos
> resultados de pesquisas conduzidas independentemente é, em si
mesma, um
> argumento significativo em favor dessa teoria.(1)
>
> Sem dúvida, o fiasco embaraçoso do julgamento de Galileu
veio à mente
> do papa quando ele advertiu os teólogos da Igreja a "[se manterem]
> informados sobre... as ciências naturais..." O papa Urbano VIII
ameaçou de
> tortura um Galileu idoso e muito enfermo se este não renunciasse
às
> alegações de que a Terra girava em torno do Sol. Ajoelhado diante
do Santo
> Ofício da Inquisição de Roma, temendo pela própria vida, Galileu
renunciou à
> sua "heresia" - mas não em seu coração. A idéia, repetidamente
afirmada por
> papas "infalíveis", de que o Sol e todos os corpos celestes
giravam em torno
> da Terra permaneceu como dogma católico oficial até 1992, quando o
Vaticano
> finalmente admitiu oficialmente que Galileu estava certo.
>
> Para evitar que a ciência continue a fazer de tola a
hierarquia
> "infalível" da Igreja, o papa admoestou os teólogos católicos a
consultarem
> os cientistas antes de interpretarem as Escrituras. No entanto,
Pedro, que
> os católicos insistem ter sido o primeiro papa, declarou que as
Escrituras
> foram inspiradas pelo Espírito Santo (2 Pe 1.21). Certamente o
Espírito
> Santo não precisa da ajuda dos cientistas! Se a Bíblia não for
infalível
> quando fala do que pertence ao campo da ciência, por que confiar
nela no que
> diz respeito a Deus e à salvação? Edward Daschbach, um sacerdote
católico,
> explica que tomar a Bíblia literalmente exigiria admitir que a
mulher que se
> assenta sobre a besta em Apocalipse 17 é a Igreja Católica Romana!
Ele
> escreve:
>
> A Igreja, portanto, não aceita... a interpretação literal
dos
> primeiros capítulos do livro de Gênesis... Quando os que advogam o
> criacionismo aplicam suas ferramentas fundamentalistas a este
último livro
> [Apocalipse], a Igreja muitas vezes se torna alvo de veementes
ataques.(2)
>
> Protestantes que, como Charles Colson, juntaram forças com
Roma,
> advogam que o catolicismo concorda com eles sobre a inerrância da
Bíblia.
> Pelo contrário, o Concílio Vaticano II declara: "Daí afirmarmos
que a Bíblia
> é livre de erro naquilo que pertence à verdade religiosa revelada
para nossa
> salvação. Não é necessariamente livre de erro em outros assuntos
(por
> exemplo, ciências naturais)" [ênfase no original].(3)
>
> Isso não é uma questão trivial. Se o relato da criação em
Gênesis não
> é digno de confiança, o restante da Bíblia também não pode ser
confiável,
> pois depende desse relato. Além disso, prova-se que Cristo não era
realmente
> Deus, mas um mero mortal que, tolamente interpretou literalmente a
história
> de Adão e Eva (Mt 19.4-5), e não pode, portanto, ser nosso
Salvador. O
> periódico The American Atheist [O Ateu Americano] sabe muito bem
qual é a
> questão: "Destruam-se Adão e Eva e o pecado original, e nos
escombros se
> encontrarão os restos mortais do Filho de Deus, eliminando-se
assim qualquer
> significado para sua morte."(4)
>
> Em maio de 1982, honrando o centenário da morte de Darwin, a
> Pontifícia Academia de Ciências do Vaticano publicou a seguinte
declaração:
> "Grande quantidade de evidências torna a aplicação do conceito de
> evolução... acima de qualquer discussão séria".(5) A Nova
Enciclopédia
> Católica diz:
>
> Especialistas... por mais de cem anos, reuniram as provas
> necessárias... a evolução está estabelecida tão firmemente quanto
a ciência
> é capaz de estabelecer fatos...(6)
>
> Cientistas descartam Darwin!
>
> Nem tanto assim. Um número cada vez maior de cientistas, a
maioria
> deles não-cristãos, se opõe à evolução. O astrônomo e matemático
Sir Fred
> Hoyle diz: "O mundo científico foi iludido e acabou crendo que a
evolução
> fora provada. Nada poderia estar mais longe da verdade".(7) O
biólogo
> Michael Denton, autor de Evolution: A Theory in Crisis [Evolução:
Uma Teoria
> em Crise], diz que a ciência desacreditou tão completamente o
evolucionismo
> darwiniano que este deveria ser descartado. O professor de
matemática
> Wolfgang Smith chama a evolução de "um mito metafísico...
completamente
> desprovido de aprovação científica..."(8)
>
> Colin Patterson, paleontólogo-chefe do Museu Britânico de
História
> Natural, confessou depois de mais de vinte anos envolvido com o
movimento
> evolucionista: "Nada havia que eu realmente conhecesse sobre a
evolução. É
> um choque enorme descobrir-se enganado por tanto tempo".
Patterson "começou
> a pedir a outros cientistas que lhe apresentassem uma coisa de que
tinham
> certeza sobre a evolução." Os biólogos do Museu Americano de
História
> Natural em Nova Iorque ficaram mudos. Diz Patterson:
>
> Experimentei a pergunta com o pessoal da geologia do Museu
de Campo de
> História Natural, e a única resposta que recebi foi o silêncio.
Tentei obter
> resposta dos membros do Seminário de Morfologia Evolucionista na
> Universidade de Chicago, um grupo prestigioso de evolucionistas, e
recebi de
> volta um longo silêncio, até que, por fim, uma pessoa disse: "Eu
sei uma
> coisa - não deveria ser ensinada no primeiro e segundo grau."(9)
>
> A despeito disso, no caso Edwards versus Aguillard, 482 U.S.
578
> (1978), a Suprema Corte americana decidiu que era inconstitucional
que as
> escolas ensinassem o criacionismo lado a lado com o darwinismo
como uma
> outra teoria de origens. Os evangélicos reclamam com justiça por
ver a
> evolução ensinada como fato nas escolas públicas, mas ela também é
ensinada
> como fato em escolas católicas.(10) Na revista The Catholic World
Report,
> Stephen F. Smith escreve: "Na escola arquidiocesana de Washington,
fomos
> ensinados que a teoria da evolução de Darwin era tão verdadeira
quanto o
> evangelho."(11) Michael Behe, bioquímico, relembra seus dias em
escolas
> católicas:
>
> Fui ensinado... a vida... veio de Deus, e que... a principal
> explicação científica de como Ele o fizera era a teoria darwiniana
da
> evolução. Eu não... via qualquer conflito com o ensino da Igreja.
(12)
>
> A evolução é matematicamente impossível
>
> Em seu livro The Blind Watchmaker [O Relojoeiro Cego], o
zoólogo
> Richard Dawkins, da Universidade de Oxford, um destacado
evolucionista,
> chama a biologia de "o estudo de coisas complicadas que dão a
aparência de
> terem sido criadas com algum propósito."(13) Sem dúvida! Uma
célula, a menor
> unidade viva, chega a ter 100.000 moléculas, e 10.000 reações
químicas
> interrelacionadas simultâneas. As células não podem ter surgido
por acaso!
> Dawkins admite que cada célula contém, no seu núcleo, um banco de
dados
> digitalmente codificado que é maior... do que a soma de todos os
30 volumes
> da Enciclopédia Britânica."(14) É impossível sequer imaginar a
ínfima
> probabilidade do acaso criar uma enciclopédia de 30 volumes! E
isso equivale
> apenas a uma célula - e há trilhões de células no corpo humano,
milhares de
> tipos diferentes, operando em relacionamentos incrivelmente
complexos e
> delicadamente equilibrados!
>
> A probabilidade astronomicamente pequena torna a evolução
> matematicamente impossível. Hoyle calculou que a probabilidade da
produção
> ocasional apenas das enzimas básicas para a produção da vida são
de 1 sobre
> 1 seguido de 40.000 zeros. Em comparação, a chance de, por acaso,
pegar um
> átomo específico em todo o universo seria de apenas 1 sobre 1
seguido de 80
> zeros. Mesmo que cada átomo existente se tornasse outro universo,
as chances
> de pegar um átomo qualquer em todos esses universos seria de
apenas 1 sobre
> 1 seguido de 160 zeros. Uma chance em 1040.000 só para produzir as
enzimas
> básicas! Mas as enzimas realizam coisas notáveis, e esse fato
complica ainda
> mais o problema da evolução com essas chances infinitamente
pequenas.
>
> Por que razão o sangue só coagula no ponto de sangramento e
não dentro
> das veias e artérias? E por que pára quando cessa o sangramento?
Imagine os
> bilhões de animais que teriam sangrado até morrer, ou teriam
morrido por uma
> coagulação inadequada antes que esse processo incrível tivesse
sido
> aperfeiçoado por mero acaso! O sistema imunológico é ainda mais
> surpreendente, diz Behe. "A complexidade do sistema garante o
insucesso de
> qualquer explicação darwiniana..."(15) E assim acontece com
centenas de
> outros sistemas que sustentam a vida. Lembre-se de que esses
sistemas
> precisavam ser operacionais para serem úteis; não poderiam ter
evoluído em
> estágios.
>
> Em seu excelente livro, publicado em 1996, Darwin's Black
Box [A Caixa
> Preta de Darwin], Behe documenta a incompreensível complexidade da
vida em
> seu nível químico celular mais básico - uma complexidade
inimaginável para
> Darwin. Behe, que afirma que a evolução "deveria ser banida",(16)
demole a
> teoria darwiniana oferecendo múltiplos exemplos, no nível
bioquímico, de
> elementos "irredutivelmente complexos" intrincadamente planejados,
que nunca
> poderiam ter evoluído:
>
> [A evolução] não pode explicar a origem das complexas
estruturas
> bioquímicas que sustentam a vida. Sequer tenta explicar... A
conclusão de um
> plano inteligente flui naturalmente dos próprios dados - não de
livros
> sagrados nem de crenças sectárias.(17)
>
> A evolução teísta contradiz a Bíblia
>
> Em apoio ao papa, Donald Devine escreve: "O homem pré-humano
> aparentemente existiu por milhões de anos... Isso não é uma
refutação da
> Bíblia, mas uma confirmação - pois indica que foi preciso que Deus
soprasse
> nele uma alma antes que o homem pudesse ser homem."(18) Pelo
contrário! A
> evolução teísta, que exige ancestrais pré-humanos para o homem
(para os
> quais nenhuma evidência jamais foi encontrada), não contradiz
apenas o livro
> de Gênesis, mas toda a Bíblia.
>
> Moisés afirma que Deus formou Adão "do pó da terra", e que
depois
> formou Eva a partir de uma de suas costelas (Gn 2.7, 18-22).
Ancestrais
> pré-humanos não podem ser reconciliados com o relato autenticado
por Jesus:
> "Não tendes lido que o Criador desde o princípio os fez homem e
mulher, e
> que disse: Por esta causa deixará o homem pai e mãe, e se unirá a
sua
> mulher, tornando-se os dois uma só carne?" (Mt 19.4-5). Cristo
confirma o
> relato de Gênesis ao citá-lo em Seu ensino. Paulo também atesta a
veracidade
> do relato ao declarar que "primeiro foi formado Adão, e depois
Eva" (1 Tm
> 2.13-14 - ver também 1 Co 15.22, 45; Judas 14). Eles não eram um
par de
> criaturas pré-humanas nas quais Deus infundiu almas humanas.
>
> Além disso, Paulo afirmou que o pecado entrou no mundo por
meio de
> Adão, e pelo pecado a morte (Rm 5.12). Se Adão e Eva tivessem tido
> ancestrais que viveram e morreram por milhares (ou milhões) de
anos de
> evolução até que Deus os humanizasse, a morte teria operado na
terra antes
> que Adão pecasse - uma contradição clara do relato de Gênesis, do
ensino de
> Cristo, da pregação de Paulo e do Evangelho. O cardeal de Nova
Iorque, John
> O'Connor, diz que Adão e Eva podem ter sido "animais inferiores".
(19)
>
> Evolução - uma artimanha satânica
>
> A evolução, "a mais gorda das vacas sagradas",(20) tem sido
uma
> poderosa ferramenta de Satanás para convencer milhões de pessoas
de que a
> Bíblia não é digna de confiança. Como afirmou Phillip Johnson,
professor de
> direito em Berkeley: "O único propósito da história evolucionista
darwiniana
> é... demonstrar que não é necessária a existência prévia de um ser
> inteligente...[para] haver a criação."(21) Johnson causou um
choque no mundo
> acadêmico em 1991 ao lançar seu livro Darwin on Trial [Darwin no
Banco nos
> Réus]. Com a precisão de um promotor, ele destruiu o darwinismo e
acusou os
> evolucionistas de terem "abandonado o relato verdadeiro e preciso
com o qual
> a ciência estava tradicionalmente compromissada, no seu zelo por
extirpar e
> descartar a religião..."(22)
>
> A evolução teria preenchido o registro fóssil de bilhões de
criaturas
> intermediárias, e no entanto nem um sequer desses "elos perdidos"
foi
> encontrado! Imagine a quantidade necessária de restos mortais
desses milhões
> de pequenos incrementos evolutivos ao longo de milhões de anos
para a
> passagem de guelras a pulmões, de pernas dianteiras para asas,
para produzir
> estômagos e sistemas digestivos, olhos, rins, cérebros e sistemas
nervosos
> que se estendessem por todo o corpo, a corrente sanguínea, o
esperma e o
> óvulo dos mamíferos, o ovo e sua casca para os répteis e pássaros,
etc. A
> impossibilidade aumenta geometricamente, pois cada um desses
sistemas é
> incrivelmente complexo e não poderia evoluir gradativamente, mas
precisaria
> ser funcional para sustentar a vida e ajudar na "sobrevivência" -
como seria
> o caso, por exemplo, do sofisticado sistema de radar dos morcegos.
>
> Quantos milhões de andorinhas do Ártico morreram afogadas
antes que a
> primeira "aprendesse", por acaso, a navegação aérea sobre milhares
de
> quilômetros de oceano? Quantos salmões se perderam e jamais
conseguiram
> chegar ao riacho em que haviam nascido para desovar antes que essa
estranha
> capacidade fosse desenvolvida? Quantas aranhas morreram de fome
antes que o
> fantástico mecanismo de criação de teias tivesse, por acaso,
surgido - e
> quem teria ensinado as aranhas a usar tal recurso? Quantos ovos de
toda
> espécie de ave apodreceram antes que surgisse o instinto de chocá-
los? Como
> foi aprendido e transmitido? Há incontáveis impossibilidades para
o acaso.
>
> A preocupação atual com as "espécies ameaçadas" contradiz
Darwin. A
> evolução elimina os incapazes. É impossível crer na evolução e
trabalhar em
> prol da preservação ecológica das espécies. Como o produto final
da
> evolução, o homem deveria, sem misericórdia, eliminar todos os
rivais na
> luta pela sobrevivência. As contradições são intermináveis.
>
> Em seu último livro, Reason in the Balance [A Razão na
Balança],
> Phillip Johnson argumenta que somente a criação divina pode
explicar a
> consciência moral do homem. A natureza não tem moral. O senso
ético e moral
> do homem desaprova a evolução. Se a evolução fosse verdadeira,
deveríamos
> fechar os hospitais, parar a produção de remédios e permitir que
os doentes
> e os fracos morressem. É impossível reconciliar bondade e
compaixão com a
> sobrevivência dos mais capazes.
>
> No entanto, o homem é compelido por consciência e compaixão,
prova de
> que é feito à imagem de um Deus santo e amoroso. Ao rejeitar a
massacrante
> evidência de propósito no mundo que o cerca (Rm 1.18-32), e por
recusar-se a
> obedecer às leis de Deus gravadas em sua consciência (Rm 2.14-15),
o homem
> tornou-se vítima de seu próprio ego e de toda sorte de males.
Apesar disso,
> Deus ama o homem, e em amor e graça veio a esta terra pelo
nascimento
> virginal para que, como o Homem perfeito, sem pecado, pudesse
morrer em
> nosso lugar, pagando a penalidade infinita que a Sua própria
justiça exigia
> pelo pecado. É apenas com base nisso - o pleno pagamento da
penalidade do
> pecado, efetuado por Cristo, e a aceitação desse pagamento por
parte do
> homem - que este pode se tornar uma nova criatura em Cristo. Vamos
> permanecer leais a esse Evangelho de Jesus Cristo e à Palavra de
Deus que o
> declara; e vamos lutar com determinação contra toda tentativa de
diluir,
> perverter ou comprometer a verdade de Deus. (TBC 2/97, traduzido
por Carlos
> Osvaldo Pinto)
>
> Notas:
>
> 1.. Papa João Paulo II, "Mensagem à Pontifícia Academia de
> Ciências", L'Osservatore Romano (30 de outubro de 1996), 3.7.
> 2.. Frei Edward Daschbach, S.V.D., "Catholics and
Creationism",
> Visitor (21 de outubro de 1984), 3.
> 3.. Vaticano II, Vatican Council II, Divine Revelation
(edição
> parafraseada da organização Knights of Columbus), III.I 1e.
> 4.. The American Atheist (1978), 19, conforme citado em
The
> Christian News (11 de novembro de 1996), 15.
> 5.. Daschbach, loc. cit.
> 6.. New Catholic Encyclopedia, vol. 5 (McGraw-Hill, 1967),
689.
> 7.. George W. Cornell, "Scientist calls Darwin evolution
theory
> absurd", Times-Advocate, 10 de dezembro de 1982, A10.
> 8.. Wolfgang Smith, Teilhard and the New Religion (Tan
Books, 1988),
> 242.
> 9.. Thomas E. Woodward, "Doubts About Darwin", Moody
Monthly
> (setembro de 1988), 20.
> 10.. The Times Picayune (Flórida, 25 de outubro de 1996),
A-30.
> 11.. Stephen F. Smith, "Is Darwinism a Religion?", The
Catholic
> World Report (dezembro de 1996), 50.
> 12.. William Bole, "Of Biochemistry and Belief", Our
Sunday Visitor
> (1 de dezembro de 1996), 6.
> 13.. Richard Dawkins, The Blind Watchmaker (England:
Longman, 1986),
> 1.
> 14.. Dawkins, op. cit., 18.
> 15.. Michael J. Behe, Darwin's Black Box: The Biochemical
Challenge
> to Evolution (The Free Press, 1996), 139.
> 16.. Ibid., 186.
> 17.. Ibid., 192-93. 18. Donald Devine, Human Events (13 de
dezembro
> de 1996), 19.
> 18.. Los Angeles Times (30 de novembro de 1996), B13.
> 19.. Doug Bandow, "Fossils and Fallacies", National Review
(29 de
> abril de 1991), 47.
> 20.. Russell Schoch, "The Evolution of a Creationist",
California
> Monthly (novembro de 1991), 22.
> 21.. The Catholic World Report (dezembro de 1996), 50.
> Chamada da Meia-Noite, janeiro de 1998
>
>
>
> [As partes desta mensagem que não continham texto foram removidas]
>
>
>
> ##### ##### #####
>
> Para saber mais visite
> http://www.ciencialist.hpg.ig.com.br
>
>
> ##### ##### ##### #####
> Links do Yahoo! Grupos





SUBJECT: Re: [ciencialist] A igreja católica é a favor da evolução?
FROM: "Oraculo" <oraculo@atibaia.com.br>
TO: <ciencialist@yahoogrupos.com.br>
DATE: 17/01/2005 00:57

Olá Amaury

Como costuma acontecer com textos desse tipo, o autor não compreende muito
bem a evolução, não compreende muito bem a matemática, não compreende muito
bem a quimica e a bioquimica, etc, mas dá um bocado de palpite..:-)

Alegar que a evolução tem pontos que precisam ser explicados, o que é
correto, que tem coisa a serem descobertas, o que é correto, que tem
cientistas e pesquisadores em conflitos em pontos específicos, o que também
é correto, e concluir que ela foi refutada ou está sendo combatida como
teoria geral, é um engano (ou má fé mesmo..:-) bem conhecido.

A Igreja aceita a evolução simplesmente porque seus próceres tem estudado a
fundo as evidencias e não veem como refuta-la como uma teoria geral. Sim,
existem pontos a serem melhor explicados, como em TODA teoria e conhecimento
científico, mas o brutal volume de evidências de diferentes áreas do
conhecimento faz com que seja intelectualmente impossível refuta-la sem
desonestidade (isso se você estudar tanto quanto os cientistas da santa
sé..:-)

Cientistas descartam Darwin!

risos... Tirando a meia duzia de sempre, que já tem sua crença e aproveitam
qualquer disputa pública entre pesquisadores para alegar que a evolução foi
"contestada", e os criacionistas de sempre, a maioria (maioria é
eufemismo..:-) dos pesquisadores de todas as áreas considera a teoria
evolucionária bem embasada e provada, dentro do que se espera de provas de
uma teoria cientifica. Os "descartes" em geral são de pontos específicos,
lacunas, onde colocar seu mistério preferido.

A evolução é matematicamente impossível

Ele não compreende a matemática, e nem leu o resto do livro de Dawkins. Se
lesse, descobriria que não só não é matematicamente impossível, como até
mesmo estatisticamente provável. Mas, ler e pensar, nem sempre são
confortáveis para crenças, assim, melhor pinçar um trecho e basear todo seu
argumetno nele..:-) Se o leitor já pensa como ele, o autor, ótimo, ele não
só vai gostar de ler coisas assim, como jamais lerá o livro de Dawkins.

Mas, se o leitor fizer questão de conhecer a fonte, aprenderá mais e
perceberá a tolice do argumento. Mas esse leitor que vai procurar dados e
pensa por sí, não é o alvo desse tipo de texto..:-)

Confundir acaso e aleatoriedade com a evolução é comum. "Esquecer" do papel
da seleção natural na filtragem das mudanças aleatorioas dos genes
também..:-)

Este trecho é um primor de desinformação e desonestidade argumentativa:

" Por que razão o sangue só coagula no ponto de sangramento e não dentro
das veias e artérias? E por que pára quando cessa o sangramento? Imagine os
bilhões de animais que teriam sangrado até morrer, ou teriam morrido por uma
coagulação inadequada antes que esse processo incrível tivesse sido
aperfeiçoado por mero acaso! "

Se ele fisesse seu trabalho de pesquisador direito, saberia a resposta sem
precisar passar pelo vexame acima..:-) Animais inicialmente NÃO TINHAM
SANGUE. Ou seja, não precisavam de processos de coagulação. Líquidos para
suprir elementos necessários surgiram antes do sangue, e da necessidade de
coagular, já que os primeiros animais estavam em meios liquidos, menos
agressivos nesse ponto. Ao passar para a superfície terrestre, os que podiam
manter sua integridade mais tempo, em ambiente seco, podiam ser mais capazes
de ampliar a descendencia. Milhões de anos, e um novo processo de
coagulação, que impede a perda de sangue, é o resultado.

Mas não é só isso. O autor leu, mas não compreendeu, o exemplo original de
um dos autores que citou, que afirma que a coagulação refuta a evolução. Mas
não pelos motivos que o autor pensa ter compreendido..:-) É até engraçado,
mas ele cita o exemplo certo, mas com motivos errados..;-)

O argumento original, que o autor não compreendeu, é da complexidade
irredutivel. E o sangue seria uma prova disso. O número de elementos e
substancias quimicas e bioquimicas envolvidas no processo de coagulação
faria com que qualquer outra combinação fosse inútil. Assim, a coagulação
teria de ter surgido já complexa, de uma vez, nunca por pequenos passo
evolutivos.

O que, se fosse verdade, refutaria a evolução. Mas nào é..:-)

Realmente, o processo de coagulação é complexo. E, em sua forma final,
altamente eficiente. Mas, é possível surgir um processo simples,
inicialmente menos eficiente, e só depois se tornar um processo complexo. E,
um mecanismo chamado exoadaptação, onde um mecanismo é reutilizado de outra
forma (ler O Polegar do Panda de Jay Gould) também explica a coagulação. No
início, um mecanismo é o fator principal. Com o tempo, um novo mecanismo se
ajusta, e o que era principal, se torna acessório. Com o tempo, analisado
apenas o resultado final, este aparenta ser uma complexidade irredutível.

Se se desse ao trabalho de pesquisar isso (ou lesse o livro de Gould..:-), o
autor não escreveria tanta bobagem.

Desde que foi publicado, A Caixa Preta de Darwin, e as teses de Behe, já
foram refutadas diversas vezes, cada vez com mais elementos e evidências
(isso que dá esconder o sobrenatural dentro de lacunas, ele acaba
despejado..:-). Se algum efeito este tipo de livro causa, é a benéfica
explosão de pesquisas que acabam por refuta-lo e nos ensina mais e mais
sobre os processos que guiaram nosso desenvolvimento: a evolução, os genes
com modificações aleatórias e a seleção natural.

Tem um bocado de coisas para aprender neste universo, e a ciência tem muito
a pesquisar. Mas, a lacuna de um conhecimento, a falta temporária de uma
explicação cientifica, não significa que qualquer outra explicação
sobrenatural esteja autorizada ou seja correta..:-) Não saber (ainda)
exatamente como determinada reação bioquímica se dá a nível celular, não
significa que deus criou uma estátua de barro e assoprou no nariz dela para
dar vida..:-)

Homero



----- Original Message -----
From: "Amauri Jr" <amaurijunior2@yahoo.com.br>
To: "Lucimary Vargas" <sangalli@uai.com.br>; "Lenia"
<lenia_luz@hotmail.com>; "Laracna" <laracna@pop.com.br>;
<Conversa_de_Botequim@yahoogrupos.com.br>; <ciencialist@yahoogrupos.com.br>;
<ciencia.2005@grupos.com.br>;
<autoiniciacaodaconsciencia_plena@yahoogrupos.com.br>;
<acropolis@yahoogrupos.com.br>
Sent: Saturday, January 15, 2005 11:58 PM
Subject: [ciencialist] A igreja católica é a favor da evolução?



http://www.chamada.com.br/mensagens/artigos/evolucao.shtml


A igreja católica é a favor da evolução?

A maioria dos não-católicos ficou surpresa quando o papa João Paulo
II, num documento enviado à Pontifícia Academia de Ciências do Vaticano em
outubro de 1996, falou a favor da evolução. Na verdade, ele estava apenas
reiterando a posição oficial do catolicismo. Considere os seguintes
excertos:

Em sua encíclica Humani generis [Sobre o Gênero Humano], de 1950, meu
predecessor Pio XII já havia afirmado não haver oposição entre a evolução e
a doutrina da fé a respeito do homem... Pio XII enfatizou este ponto
essencial: se o corpo humano tem sua origem na matéria orgânica
pré-existente, a alma espiritual é imediatamente criada por Deus... O
exegeta e o teólogo precisam manter-se informados sobre... as ciências
naturais... verdade não pode contradizer a verdade...

A teoria da evolução... tem sido progressivamente aceita pelos
pesquisadores em vários campos do conhecimento. A convergência... dos
resultados de pesquisas conduzidas independentemente é, em si mesma, um
argumento significativo em favor dessa teoria.(1)

Sem dúvida, o fiasco embaraçoso do julgamento de Galileu veio à mente
do papa quando ele advertiu os teólogos da Igreja a "[se manterem]
informados sobre... as ciências naturais..." O papa Urbano VIII ameaçou de
tortura um Galileu idoso e muito enfermo se este não renunciasse às
alegações de que a Terra girava em torno do Sol. Ajoelhado diante do Santo
Ofício da Inquisição de Roma, temendo pela própria vida, Galileu renunciou à
sua "heresia" - mas não em seu coração. A idéia, repetidamente afirmada por
papas "infalíveis", de que o Sol e todos os corpos celestes giravam em torno
da Terra permaneceu como dogma católico oficial até 1992, quando o Vaticano
finalmente admitiu oficialmente que Galileu estava certo.

Para evitar que a ciência continue a fazer de tola a hierarquia
"infalível" da Igreja, o papa admoestou os teólogos católicos a consultarem
os cientistas antes de interpretarem as Escrituras. No entanto, Pedro, que
os católicos insistem ter sido o primeiro papa, declarou que as Escrituras
foram inspiradas pelo Espírito Santo (2 Pe 1.21). Certamente o Espírito
Santo não precisa da ajuda dos cientistas! Se a Bíblia não for infalível
quando fala do que pertence ao campo da ciência, por que confiar nela no que
diz respeito a Deus e à salvação? Edward Daschbach, um sacerdote católico,
explica que tomar a Bíblia literalmente exigiria admitir que a mulher que se
assenta sobre a besta em Apocalipse 17 é a Igreja Católica Romana! Ele
escreve:

A Igreja, portanto, não aceita... a interpretação literal dos
primeiros capítulos do livro de Gênesis... Quando os que advogam o
criacionismo aplicam suas ferramentas fundamentalistas a este último livro
[Apocalipse], a Igreja muitas vezes se torna alvo de veementes ataques.(2)

Protestantes que, como Charles Colson, juntaram forças com Roma,
advogam que o catolicismo concorda com eles sobre a inerrância da Bíblia.
Pelo contrário, o Concílio Vaticano II declara: "Daí afirmarmos que a Bíblia
é livre de erro naquilo que pertence à verdade religiosa revelada para nossa
salvação. Não é necessariamente livre de erro em outros assuntos (por
exemplo, ciências naturais)" [ênfase no original].(3)

Isso não é uma questão trivial. Se o relato da criação em Gênesis não
é digno de confiança, o restante da Bíblia também não pode ser confiável,
pois depende desse relato. Além disso, prova-se que Cristo não era realmente
Deus, mas um mero mortal que, tolamente interpretou literalmente a história
de Adão e Eva (Mt 19.4-5), e não pode, portanto, ser nosso Salvador. O
periódico The American Atheist [O Ateu Americano] sabe muito bem qual é a
questão: "Destruam-se Adão e Eva e o pecado original, e nos escombros se
encontrarão os restos mortais do Filho de Deus, eliminando-se assim qualquer
significado para sua morte."(4)

Em maio de 1982, honrando o centenário da morte de Darwin, a
Pontifícia Academia de Ciências do Vaticano publicou a seguinte declaração:
"Grande quantidade de evidências torna a aplicação do conceito de
evolução... acima de qualquer discussão séria".(5) A Nova Enciclopédia
Católica diz:

Especialistas... por mais de cem anos, reuniram as provas
necessárias... a evolução está estabelecida tão firmemente quanto a ciência
é capaz de estabelecer fatos...(6)

Cientistas descartam Darwin!

Nem tanto assim. Um número cada vez maior de cientistas, a maioria
deles não-cristãos, se opõe à evolução. O astrônomo e matemático Sir Fred
Hoyle diz: "O mundo científico foi iludido e acabou crendo que a evolução
fora provada. Nada poderia estar mais longe da verdade".(7) O biólogo
Michael Denton, autor de Evolution: A Theory in Crisis [Evolução: Uma Teoria
em Crise], diz que a ciência desacreditou tão completamente o evolucionismo
darwiniano que este deveria ser descartado. O professor de matemática
Wolfgang Smith chama a evolução de "um mito metafísico... completamente
desprovido de aprovação científica..."(8)

Colin Patterson, paleontólogo-chefe do Museu Britânico de História
Natural, confessou depois de mais de vinte anos envolvido com o movimento
evolucionista: "Nada havia que eu realmente conhecesse sobre a evolução. É
um choque enorme descobrir-se enganado por tanto tempo". Patterson "começou
a pedir a outros cientistas que lhe apresentassem uma coisa de que tinham
certeza sobre a evolução." Os biólogos do Museu Americano de História
Natural em Nova Iorque ficaram mudos. Diz Patterson:

Experimentei a pergunta com o pessoal da geologia do Museu de Campo de
História Natural, e a única resposta que recebi foi o silêncio. Tentei obter
resposta dos membros do Seminário de Morfologia Evolucionista na
Universidade de Chicago, um grupo prestigioso de evolucionistas, e recebi de
volta um longo silêncio, até que, por fim, uma pessoa disse: "Eu sei uma
coisa - não deveria ser ensinada no primeiro e segundo grau."(9)

A despeito disso, no caso Edwards versus Aguillard, 482 U.S. 578
(1978), a Suprema Corte americana decidiu que era inconstitucional que as
escolas ensinassem o criacionismo lado a lado com o darwinismo como uma
outra teoria de origens. Os evangélicos reclamam com justiça por ver a
evolução ensinada como fato nas escolas públicas, mas ela também é ensinada
como fato em escolas católicas.(10) Na revista The Catholic World Report,
Stephen F. Smith escreve: "Na escola arquidiocesana de Washington, fomos
ensinados que a teoria da evolução de Darwin era tão verdadeira quanto o
evangelho."(11) Michael Behe, bioquímico, relembra seus dias em escolas
católicas:

Fui ensinado... a vida... veio de Deus, e que... a principal
explicação científica de como Ele o fizera era a teoria darwiniana da
evolução. Eu não... via qualquer conflito com o ensino da Igreja.(12)

A evolução é matematicamente impossível

Em seu livro The Blind Watchmaker [O Relojoeiro Cego], o zoólogo
Richard Dawkins, da Universidade de Oxford, um destacado evolucionista,
chama a biologia de "o estudo de coisas complicadas que dão a aparência de
terem sido criadas com algum propósito."(13) Sem dúvida! Uma célula, a menor
unidade viva, chega a ter 100.000 moléculas, e 10.000 reações químicas
interrelacionadas simultâneas. As células não podem ter surgido por acaso!
Dawkins admite que cada célula contém, no seu núcleo, um banco de dados
digitalmente codificado que é maior... do que a soma de todos os 30 volumes
da Enciclopédia Britânica."(14) É impossível sequer imaginar a ínfima
probabilidade do acaso criar uma enciclopédia de 30 volumes! E isso equivale
apenas a uma célula - e há trilhões de células no corpo humano, milhares de
tipos diferentes, operando em relacionamentos incrivelmente complexos e
delicadamente equilibrados!

A probabilidade astronomicamente pequena torna a evolução
matematicamente impossível. Hoyle calculou que a probabilidade da produção
ocasional apenas das enzimas básicas para a produção da vida são de 1 sobre
1 seguido de 40.000 zeros. Em comparação, a chance de, por acaso, pegar um
átomo específico em todo o universo seria de apenas 1 sobre 1 seguido de 80
zeros. Mesmo que cada átomo existente se tornasse outro universo, as chances
de pegar um átomo qualquer em todos esses universos seria de apenas 1 sobre
1 seguido de 160 zeros. Uma chance em 1040.000 só para produzir as enzimas
básicas! Mas as enzimas realizam coisas notáveis, e esse fato complica ainda
mais o problema da evolução com essas chances infinitamente pequenas.

Por que razão o sangue só coagula no ponto de sangramento e não dentro
das veias e artérias? E por que pára quando cessa o sangramento? Imagine os
bilhões de animais que teriam sangrado até morrer, ou teriam morrido por uma
coagulação inadequada antes que esse processo incrível tivesse sido
aperfeiçoado por mero acaso! O sistema imunológico é ainda mais
surpreendente, diz Behe. "A complexidade do sistema garante o insucesso de
qualquer explicação darwiniana..."(15) E assim acontece com centenas de
outros sistemas que sustentam a vida. Lembre-se de que esses sistemas
precisavam ser operacionais para serem úteis; não poderiam ter evoluído em
estágios.

Em seu excelente livro, publicado em 1996, Darwin's Black Box [A Caixa
Preta de Darwin], Behe documenta a incompreensível complexidade da vida em
seu nível químico celular mais básico - uma complexidade inimaginável para
Darwin. Behe, que afirma que a evolução "deveria ser banida",(16) demole a
teoria darwiniana oferecendo múltiplos exemplos, no nível bioquímico, de
elementos "irredutivelmente complexos" intrincadamente planejados, que nunca
poderiam ter evoluído:

[A evolução] não pode explicar a origem das complexas estruturas
bioquímicas que sustentam a vida. Sequer tenta explicar... A conclusão de um
plano inteligente flui naturalmente dos próprios dados - não de livros
sagrados nem de crenças sectárias.(17)

A evolução teísta contradiz a Bíblia

Em apoio ao papa, Donald Devine escreve: "O homem pré-humano
aparentemente existiu por milhões de anos... Isso não é uma refutação da
Bíblia, mas uma confirmação - pois indica que foi preciso que Deus soprasse
nele uma alma antes que o homem pudesse ser homem."(18) Pelo contrário! A
evolução teísta, que exige ancestrais pré-humanos para o homem (para os
quais nenhuma evidência jamais foi encontrada), não contradiz apenas o livro
de Gênesis, mas toda a Bíblia.

Moisés afirma que Deus formou Adão "do pó da terra", e que depois
formou Eva a partir de uma de suas costelas (Gn 2.7, 18-22). Ancestrais
pré-humanos não podem ser reconciliados com o relato autenticado por Jesus:
"Não tendes lido que o Criador desde o princípio os fez homem e mulher, e
que disse: Por esta causa deixará o homem pai e mãe, e se unirá a sua
mulher, tornando-se os dois uma só carne?" (Mt 19.4-5). Cristo confirma o
relato de Gênesis ao citá-lo em Seu ensino. Paulo também atesta a veracidade
do relato ao declarar que "primeiro foi formado Adão, e depois Eva" (1 Tm
2.13-14 - ver também 1 Co 15.22, 45; Judas 14). Eles não eram um par de
criaturas pré-humanas nas quais Deus infundiu almas humanas.

Além disso, Paulo afirmou que o pecado entrou no mundo por meio de
Adão, e pelo pecado a morte (Rm 5.12). Se Adão e Eva tivessem tido
ancestrais que viveram e morreram por milhares (ou milhões) de anos de
evolução até que Deus os humanizasse, a morte teria operado na terra antes
que Adão pecasse - uma contradição clara do relato de Gênesis, do ensino de
Cristo, da pregação de Paulo e do Evangelho. O cardeal de Nova Iorque, John
O'Connor, diz que Adão e Eva podem ter sido "animais inferiores".(19)

Evolução - uma artimanha satânica

A evolução, "a mais gorda das vacas sagradas",(20) tem sido uma
poderosa ferramenta de Satanás para convencer milhões de pessoas de que a
Bíblia não é digna de confiança. Como afirmou Phillip Johnson, professor de
direito em Berkeley: "O único propósito da história evolucionista darwiniana
é... demonstrar que não é necessária a existência prévia de um ser
inteligente...[para] haver a criação."(21) Johnson causou um choque no mundo
acadêmico em 1991 ao lançar seu livro Darwin on Trial [Darwin no Banco nos
Réus]. Com a precisão de um promotor, ele destruiu o darwinismo e acusou os
evolucionistas de terem "abandonado o relato verdadeiro e preciso com o qual
a ciência estava tradicionalmente compromissada, no seu zelo por extirpar e
descartar a religião..."(22)

A evolução teria preenchido o registro fóssil de bilhões de criaturas
intermediárias, e no entanto nem um sequer desses "elos perdidos" foi
encontrado! Imagine a quantidade necessária de restos mortais desses milhões
de pequenos incrementos evolutivos ao longo de milhões de anos para a
passagem de guelras a pulmões, de pernas dianteiras para asas, para produzir
estômagos e sistemas digestivos, olhos, rins, cérebros e sistemas nervosos
que se estendessem por todo o corpo, a corrente sanguínea, o esperma e o
óvulo dos mamíferos, o ovo e sua casca para os répteis e pássaros, etc. A
impossibilidade aumenta geometricamente, pois cada um desses sistemas é
incrivelmente complexo e não poderia evoluir gradativamente, mas precisaria
ser funcional para sustentar a vida e ajudar na "sobrevivência" - como seria
o caso, por exemplo, do sofisticado sistema de radar dos morcegos.

Quantos milhões de andorinhas do Ártico morreram afogadas antes que a
primeira "aprendesse", por acaso, a navegação aérea sobre milhares de
quilômetros de oceano? Quantos salmões se perderam e jamais conseguiram
chegar ao riacho em que haviam nascido para desovar antes que essa estranha
capacidade fosse desenvolvida? Quantas aranhas morreram de fome antes que o
fantástico mecanismo de criação de teias tivesse, por acaso, surgido - e
quem teria ensinado as aranhas a usar tal recurso? Quantos ovos de toda
espécie de ave apodreceram antes que surgisse o instinto de chocá-los? Como
foi aprendido e transmitido? Há incontáveis impossibilidades para o acaso.

A preocupação atual com as "espécies ameaçadas" contradiz Darwin. A
evolução elimina os incapazes. É impossível crer na evolução e trabalhar em
prol da preservação ecológica das espécies. Como o produto final da
evolução, o homem deveria, sem misericórdia, eliminar todos os rivais na
luta pela sobrevivência. As contradições são intermináveis.

Em seu último livro, Reason in the Balance [A Razão na Balança],
Phillip Johnson argumenta que somente a criação divina pode explicar a
consciência moral do homem. A natureza não tem moral. O senso ético e moral
do homem desaprova a evolução. Se a evolução fosse verdadeira, deveríamos
fechar os hospitais, parar a produção de remédios e permitir que os doentes
e os fracos morressem. É impossível reconciliar bondade e compaixão com a
sobrevivência dos mais capazes.

No entanto, o homem é compelido por consciência e compaixão, prova de
que é feito à imagem de um Deus santo e amoroso. Ao rejeitar a massacrante
evidência de propósito no mundo que o cerca (Rm 1.18-32), e por recusar-se a
obedecer às leis de Deus gravadas em sua consciência (Rm 2.14-15), o homem
tornou-se vítima de seu próprio ego e de toda sorte de males. Apesar disso,
Deus ama o homem, e em amor e graça veio a esta terra pelo nascimento
virginal para que, como o Homem perfeito, sem pecado, pudesse morrer em
nosso lugar, pagando a penalidade infinita que a Sua própria justiça exigia
pelo pecado. É apenas com base nisso - o pleno pagamento da penalidade do
pecado, efetuado por Cristo, e a aceitação desse pagamento por parte do
homem - que este pode se tornar uma nova criatura em Cristo. Vamos
permanecer leais a esse Evangelho de Jesus Cristo e à Palavra de Deus que o
declara; e vamos lutar com determinação contra toda tentativa de diluir,
perverter ou comprometer a verdade de Deus. (TBC 2/97, traduzido por Carlos
Osvaldo Pinto)

Notas:

1.. Papa João Paulo II, "Mensagem à Pontifícia Academia de
Ciências", L'Osservatore Romano (30 de outubro de 1996), 3.7.
2.. Frei Edward Daschbach, S.V.D., "Catholics and Creationism",
Visitor (21 de outubro de 1984), 3.
3.. Vaticano II, Vatican Council II, Divine Revelation (edição
parafraseada da organização Knights of Columbus), III.I 1e.
4.. The American Atheist (1978), 19, conforme citado em The
Christian News (11 de novembro de 1996), 15.
5.. Daschbach, loc. cit.
6.. New Catholic Encyclopedia, vol. 5 (McGraw-Hill, 1967), 689.
7.. George W. Cornell, "Scientist calls Darwin evolution theory
absurd", Times-Advocate, 10 de dezembro de 1982, A10.
8.. Wolfgang Smith, Teilhard and the New Religion (Tan Books, 1988),
242.
9.. Thomas E. Woodward, "Doubts About Darwin", Moody Monthly
(setembro de 1988), 20.
10.. The Times Picayune (Flórida, 25 de outubro de 1996), A-30.
11.. Stephen F. Smith, "Is Darwinism a Religion?", The Catholic
World Report (dezembro de 1996), 50.
12.. William Bole, "Of Biochemistry and Belief", Our Sunday Visitor
(1 de dezembro de 1996), 6.
13.. Richard Dawkins, The Blind Watchmaker (England: Longman, 1986),
1.
14.. Dawkins, op. cit., 18.
15.. Michael J. Behe, Darwin's Black Box: The Biochemical Challenge
to Evolution (The Free Press, 1996), 139.
16.. Ibid., 186.
17.. Ibid., 192-93. 18. Donald Devine, Human Events (13 de dezembro
de 1996), 19.
18.. Los Angeles Times (30 de novembro de 1996), B13.
19.. Doug Bandow, "Fossils and Fallacies", National Review (29 de
abril de 1991), 47.
20.. Russell Schoch, "The Evolution of a Creationist", California
Monthly (novembro de 1991), 22.
21.. The Catholic World Report (dezembro de 1996), 50.
Chamada da Meia-Noite, janeiro de 1998



[As partes desta mensagem que não continham texto foram removidas]



##### ##### #####

Para saber mais visite
http://www.ciencialist.hpg.ig.com.br


##### ##### ##### #####
Links do Yahoo! Grupos












SUBJECT: Re: quimica inorganica
FROM: "rmtakata" <rmtakata@altavista.net>
TO: ciencialist@yahoogrupos.com.br
DATE: 17/01/2005 07:08


--- Em ciencialist@yahoogrupos.com.br, "Camila" <camilasaki@y...>
> Será que alguem aki conheceria um site para experimentos de quimica
> inorganica, de nivel superior, com materiais alternativos?

Isso eh com o Brudna e o Chemello.

[]s,

Roberto Takata





SUBJECT: Re: [ciencialist] Re: quimica inorganica
FROM: "E m i l i a n o C h e m e l l o" <chemelloe@yahoo.com.br>
TO: <ciencialist@yahoogrupos.com.br>
DATE: 17/01/2005 08:30

Esses biólogos... :-)

Camila!

Dê uma olhada neste site:

http://www.chemkeys.com/bra/md/eddns_2/eddns_2.htm

Talvez tenha algo de interessante.

Espero ter ajudado,

Abraços do
Emiliano Chemello

----- Original Message -----
From: rmtakata
To: ciencialist@yahoogrupos.com.br
Sent: Monday, January 17, 2005 7:08 AM
Subject: [ciencialist] Re: quimica inorganica



--- Em ciencialist@yahoogrupos.com.br, "Camila" <camilasaki@y...>
> Será que alguem aki conheceria um site para experimentos de quimica
> inorganica, de nivel superior, com materiais alternativos?

Isso eh com o Brudna e o Chemello.

[]s,

Roberto Takata





##### ##### #####

Para saber mais visite
http://www.ciencialist.hpg.ig.com.br


##### ##### ##### #####


Yahoo! Grupos, um serviço oferecido por:







------------------------------------------------------------------------------
Links do Yahoo! Grupos

a.. Para visitar o site do seu grupo na web, acesse:
http://br.groups.yahoo.com/group/ciencialist/

b.. Para sair deste grupo, envie um e-mail para:
ciencialist-unsubscribe@yahoogrupos.com.br

c.. O uso que você faz do Yahoo! Grupos está sujeito aos Termos do Serviço do Yahoo!.



[As partes desta mensagem que não continham texto foram removidas]



SUBJECT: Fw: Maremotos...
FROM: "Luiz Ferraz Netto" <leobarretos@uol.com.br>
TO: "ciencialist" <ciencialist@yahoogrupos.com.br>
DATE: 17/01/2005 10:08

Estou voltando de férias rancheiras.
Querem sugerir nesses maremotos?
[]'
===========================
Luiz Ferraz Netto [Léo]
leobarretos@uol.com.br
http://www.feiradeciencias.com.br
===========================
-----Mensagem Original-----
De: Fernando Guilherme Neves Gueiros
Para: leobarretos@uol.com.br
Enviada em: domingo, 9 de janeiro de 2005 01:44
Assunto: Maremotos...


Prezado Luiz Ferraz Netto,

Muito se têm dissertado sobre o derretimento polar e suas consequências a nível dos mares.
Pouco, porém, se tem esclarecido a respeito do que ocorrerá em graves consequências o PESO deste derretimento.

Com a desestabilização do PESO "perdido" nas calotas, haverá grande aréas de atritos tanto no fundo do mar, dado ao volume de água aumentado, quanto A PERDA deste volume nas calotas, havendo transferências para outras placas, desinstabilizando-as por esse fenomeno.

A exemplo: Não seria este extremo acidente numa aréa até então não muito desestabilizada, ressalto no Mar Indico, com perdas de vidas para mais de 156 mil.

Outras virão ainda este ano.

Não há aqui o carater de Pitonisa.

Minha QUESTÃO seria: quais estudos há por parte dos geografos e cientista na geologia terrestre, principalmente, nas questões das placas?

Meus respeitos,

EfeGueiros.



--------------------------------------------------------------------------------


Internal Virus Database is out-of-date.
Checked by AVG Anti-Virus.
Version: 7.0.300 / Virus Database: 265.6.9 - Release Date: 06/01/2005

----------

No virus found in this outgoing message.
Checked by AVG Anti-Virus.
Version: 7.0.300 / Virus Database: 265.6.13 - Release Date: 16/01/2005


[As partes desta mensagem que não continham texto foram removidas]



SUBJECT: Fw: Por favor... preciso de ajuda !!
FROM: "Luiz Ferraz Netto" <leobarretos@uol.com.br>
TO: "ciencialist" <ciencialist@yahoogrupos.com.br>
DATE: 17/01/2005 10:11

Esse é Imperdível!

[]'
===========================
Luiz Ferraz Netto [Léo]
leobarretos@uol.com.br
http://www.feiradeciencias.com.br
===========================
-----Mensagem Original-----
De: Marcia Anitta
Para: leobarretos@uol.com.br
Enviada em: domingo, 9 de janeiro de 2005 12:41
Assunto: Por favor... preciso de ajuda !!


Olá! Prof.
Ano passado peguei um hacker aqui em meu pc.
Depois de muito gastos consegui retirá-lo...
Mas ele não é um simples hacker...é uma pessoa muito inteligente e estudiosa.

-Ele agora tem o "domínio" de minha casa com a eletricidade.

Interage comigo pela tv( ligando a luzinha do stéreo e fazendo interferência de formas variadas, vezes riscos e em outras retorce a imagem), como se tivesse conversando comigo, durante noticiário , novelas e em outros programas , rádio( não posso ouvir radio que pedem músicas, pois ele interage tb e não posso ouvir por tempo a mesma rádio), radio -relógio( já o desligou, mudou de estação e perdi a hora de compromissos), e o pior já estragou minha máquina de lavar roupas que era "eletrônica" e agora é manual, pois fez ela trabalhar sozinha e alagou toda minha área de serviço.
A geladeira será a próxima coisa que ele irá estragar, pois para interagir "todo momento" ele liga e a desliga, e está sempre acelerada. E o gelo forma-se rapidamente e tenho que ficar degelando sempre!
O ligar , o clique que ele dá é alto e quando ele acorda já começa, até conseguir me acordar... Quando ele tem insônia é uma "barra", pois não para ! E não me deixa dormir.
Troquei de telefone mais ...sei que ainda escuta o que falamos no telefone, por isso é raro eu usá-lo.
Sei tb que ele é radioamador...ou Dx...sei lá o que ele é; mas não sei "quem" é!
Entende muito desse assunto, isso eu sei.
Minhas lâmpadas não paravam ...queimavam , agora parou um pouco, pois ele "altera" a intensidade da energia daqui de casa.
É só eu acender uma lâmpada, que ele já sabe que estou em casa.
Sabe tudo que faço "se eu usar " a energia.
Estou cansada de tudo isso. Não sei como "falar" para ele parar.
Quanto mais ele faz isso mais irritada eu fico.
Estou estressada, sem saber como conseguir sair dessa!

Por favor tem algo que eu possa fazer?
Como "anular" e conseguir que ele "Não" tenha acesso aqui em casa?
Por favor! Me ajude!

Não posso ficar me expondo por aí, NÃO acreditam e não tenho como provar que essas coisas acontecem. Já tentei e nada consegui.

Só eu e meu filho sabemos que tudo que relatei é "verdade" e acontecem.

Não sou criança tenho, quase 50 anos e estou presa em minha própria casa por um "paranóico!

Agradeço se puder ajudar-me.

Marcia.
__________________________________________________
Converse com seus amigos em tempo real com o Yahoo! Messenger
http://br.download.yahoo.com/messenger/



--------------------------------------------------------------------------------


Internal Virus Database is out-of-date.
Checked by AVG Anti-Virus.
Version: 7.0.300 / Virus Database: 265.6.9 - Release Date: 06/01/2005

----------

No virus found in this outgoing message.
Checked by AVG Anti-Virus.
Version: 7.0.300 / Virus Database: 265.6.13 - Release Date: 16/01/2005


[As partes desta mensagem que não continham texto foram removidas]



SUBJECT: Re: A igreja católica é a favor da evolução?
FROM: "rmtakata" <rmtakata@altavista.net>
TO: ciencialist@yahoogrupos.com.br
DATE: 17/01/2005 10:12


--- Em ciencialist@yahoogrupos.com.br, "Amauri Jr"
> uma coisa de que tinham certeza sobre a evolução."

Certeza eh coisa de religiosos, nao de cientistas.

> As células não podem ter surgido por acaso!

E ninguem disse isso. A selecao natural eh longe, muito longe de ser
casual.

> A probabilidade astronomicamente pequena torna a evolução
> matematicamente impossível.

De forma alguma. Seria impossivel se a selecao natural dependesse
unica e exclusivamente do acaso.

> Por que razão o sangue só coagula no ponto de sangramento e não
> dentro das veias e artérias? E por que pára quando cessa o
> sangramento? Imagine os bilhões de animais que teriam sangrado até
> morrer, ou teriam morrido por uma coagulação inadequada antes que
> esse processo incrível tivesse sido aperfeiçoado por mero acaso!

Por isso q. a selecao natural eh um otimo agente explicativo: ela nao
depende do acaso!

Imagine uma pessoa hemofilica. Sim, se nao for tratada, ela tem
grandes chances de morrer, mas ela nao irah necessariamente morrer. O
sistema de coagulacao nao precisa ter evoluido somente depois do
surgimento do sistema circulatorio. Antes dos organismos apresentarem
sistema circulatorio, eles jah estavam sujeitos a diversos tipos de
ferimentos. Eles jah apresentavam mecanismos de cicatrizacao e
regeneracao. Parte desses processos puderam ser cooptados no
desenvolvimento do sistema de coagulacao.

> complexidade do sistema garante o insucesso de qualquer explicação
> darwiniana..."

Apenas nos sonhos de Behe. Existem organismos com graus variados de
complexidade do sistema imunologico. Eles apresentam como estagios
intermediarios poderiam atuar.

> Pelo contrário! A evolução teísta, que exige ancestrais pré-humanos
> para o homem (para os quais nenhuma evidência jamais foi
> encontrada), não contradiz apenas o livro de Gênesis, mas toda a
> Bíblia.

Se isso eh verdade, azar da Biblia.

> A evolução, "a mais gorda das vacas sagradas",(20) tem sido
> uma poderosa ferramenta de Satanás para convencer milhões de pessoas
> de que a Bíblia não é digna de confiança.

Prove q Satanas existe.

> A evolução teria preenchido o registro fóssil de bilhões de
> criaturas intermediárias, e no entanto nem um sequer desses "elos
> perdidos" foi encontrado!

Os elos perdidos, nao. Mas os elos achados sim. Ichthyostega e
Achantostega apresentam um belo exemplo da transicao de peixes para
tetrapodos. Archaeopteryx e Microraptor apresentam outro belos
exemplos da transicao de teropodos para aves. A transicao de
pelicossauros para mamiferos eh tao branda e suave q. eh preciso
estabelecer uma caracteristica arbitraria - como a articulacao da
mandibula eh constituida - para separar o q. eh mamifero do q. nao eh
mamifero.

> Quantos milhões de andorinhas do Ártico morreram afogadas antes que
> a primeira "aprendesse", por acaso, a navegação aérea sobre milhares
> de quilômetros de oceano?

Ue'? Basta q. a cada geracao as andorinhas fossem capazes de ir um
metro mais alem: ha' um ditado chines q. diz q. mesmo a mais longa
jornada comeca com um simples passo. Se o incremento fosse mesmo de
alguns metros, em 40 milhoes de anos seriam capazes de dar uma volta
completa ao redor do globo. Considerando-se q. Archeopteryx data do
Neo-Jurassico - uns 240 milhoes de anos atras - houve tempo de sobra
para o voo transoceanico das aves. Como o voo transoceanico pode ter
evoluido? De varias formas. Elas poderiam inicialmente utilizar ilhas
no meio do oceano como escala. Ou poderiam simplesmente ter seguido a
lenta separacao dos oceanos: o Atlantico, por exemplo, surgiu por
volta do meio do Cretaceo - uns 100 milhoes de anos - e gradativamente
foi se alargando.

> Quantos salmões se perderam e jamais conseguiram chegar ao riacho em
> que haviam nascido para desovar antes que essa estranha capacidade
> fosse desenvolvida?

Ue' mesmo hoje - com sua capacidade migratoria plenamente desenvolvida
- milhoes de salmoes morrem antes de completar sua jornada. Sacanagem
divina? Ou a mortandade de milhoes de animais inocentes eh cruel
apenas de for por meio da selecao natural, mas passa a ser um
sacrificio beatifico se eh por ordem de Deus? Eh vai ver q. eh, afinal
poderia ser o mesmo Deus q. mandou Abraao sacrificar seu proprio filho
para testar sua feh.

> Quantas aranhas morreram de fome antes que o fantástico mecanismo de
> criação de teias tivesse, por acaso, surgido – e quem teria ensinado
> as aranhas a usar tal recurso?

Quantos milhoes de aranhas morrem mesmo com teias? Na verdade, todas
as aranhas acabam morrendo de um ou de outro jeito - ainda nao se
descobriu nenhuma aranha Highlander. As teias certamente surgiram
depois da seda. As sedas sao utilizadas de diversas formas por aranhas
q. mesmo hoje nao tecem teias - nao consta q. aranhas q. nao tecem
teias morram de fome por falta de teia.

> Quantos ovos de toda espécie de ave apodreceram antes que surgisse o
> instinto de chocá-los?

Provavelmente nenhum, porq. o instinto de chocar os ovos surgiu antes
da primeira ave. Eh muito bem documentado em oviraptores fosseis - um
adulto sentado sobre os ovos. Crocodilos nao se sentam em cima, mas
chocam seus ovos enterrado em um ninho de lama e folhas em
decomposicao. Mas existem muitos animais q. abandonam seus ovos e nao
consta q. apodrecam por falta de incubacao.

> Como foi aprendido e transmitido? Há incontáveis impossibilidades
> para o acaso.

E ha' tb muitas possibilidades para a selecao natural. Como ateh hoje
o comportamento nao eh transmitido por aprendizagem, mas por
transmissao genetica, eh bem possivel q. tenha se dado do mesmo modo
no passado.

> A preocupação atual com as "espécies ameaçadas" contradiz Darwin. A
> evolução elimina os incapazes. É impossível crer na evolução e
> trabalhar em prol da preservação ecológica das espécies.

De forma alguma. Do mesmo modo q. crer q. nao dar comida a alguem faz
com q. essa pessoa morra de fome nao faz com q. seja contraditorio
trabalhar por um mundo mais justo. Saber q. a selecao natural tende a
eliminar individuos menos adaptados a um dado ambienta, nada nos diz
sobre como devemos agir em relacao a preservacao das especies.

> Como o produto final da evolução, o homem deveria, sem misericórdia,
> eliminar todos os rivais na luta pela sobrevivência.

Como a criacao divina suprema o homem deveria, sem misericordia,
eliminar todos os rivais na luta pela feh. Epa, mas isso ele jah faz.
Inclusive outras pessoas. Nao foi Javeh quem ordenou q. se matassem
criancas de uma tribo q. seguia os preceitos por ele ditados?

[]s,

Roberto Takata





SUBJECT: Economia mundial: previsão p/2020
FROM: José Renato <jrma@terra.com.br>
TO: <ciencialist@yahoogrupos.com.br>
DATE: 17/01/2005 10:39

Sinopse da imprensa:
CIA prevê Brasil entre os grandes dentro de 15 anos
07:21 17/01


Da Redação (editorultimosegundo@ig.com.br)


A economia do Brasil será maior do que a maioria dos países da Europa dentro de 15 anos. Ao lado da Rússia, Indonésia e África do Sul, o País será uma das grandes potências do mundo - um patamar abaixo da China e da Índia - os dois grandes poderes emergentes. As informações são do jornal "O Estado de S. Paulo".

Essas afirmações foram feitas no estudo sobre o Panorama Global em 2020, que o Conselho Nacional de Inteligência (setor de análises da CIA) divulgou na semana passada.



Baseado em consultas encomendadas a especialistas e conferências, o trabalho identifica *este momento* como o de maior mudança potencial das formas e da natureza dos alinhamentos internacionais desde a formação da Aliança Ocidental, em 1949.



A CIA ainda afirma que "o papel dos EUA será uma importante variável na forma como o mundo evoluirá, influenciando os caminhos que estados e não-estados seguirão" e reconhece a deterioração da imagem americana.



O estudo ainda prevê que a China terá o segundo PIB do planeta em 2020, atrás dos EUA e que a Índia superará a economia das potências européias.



< http://ultimosegundo.ig.com.br/materias/brasil/1854501-1855000/1854703/1854703_1.xml >


[As partes desta mensagem que não continham texto foram removidas]



SUBJECT: Re: Fw: Maremotos...
FROM: "rmtakata" <rmtakata@altavista.net>
TO: ciencialist@yahoogrupos.com.br
DATE: 17/01/2005 10:48


--- Em ciencialist@yahoogrupos.com.br, "Luiz Ferraz Netto"
> -----Mensagem Original-----
> Muito se têm dissertado sobre o derretimento polar e suas
> consequências a nível dos mares.
> Pouco, porém, se tem esclarecido a respeito do que ocorrerá em
> graves consequências o PESO deste derretimento.

Existe um fenomeno chamado hidrostaticidade. Imagine - ou melhor -
faca esse pequeno experimento em sua casa. Pegue um pequeno bloco de
madeira e coloque o para flutar em uma tina d'agua. Repare q. existe
uma porcao q. fica acima do nivel d'agua. Agora coloque sobre o bloco
um peso pequeno. Veja q. a porcao do bloco q. fica acima da agua
diminui. Retire o peso e veja q. o bloco sobe.

Supoem-se q. isso ocorra tb nas placas tectonicas qdo ocorrem
fenomenos como a glaciacao e degelo. Qdo se formam imensas geleiras
sobre uma parte de uma placa, o peso extra pode fazer com q. a placa
afunde ligeiramente (fenomeno da subsidencia), qdo esse peso extra se
desfaz a placa sobe (elevacao). (Claro q. nem toda subsidencia e
elevacao se dah pela formacao e recuo de geleiras.)

Mas isso afetaria mais diretamente as placas q. em q. estao as calotas
polares hoje - no caso de um eventual derretimento. Sua pergunta eh
sobre o q. ocorreria com outras placas. Bem, o nivel do mar deve subir
algumas dezenas de metros nesse cenario, o q. faria com q. a pressao
sobre as placas aumentasse apenas algumas atmosferas - a cada 10
metros de coluna d'agua, a pressao aumentaria 1 atmosfera. Nao seria
muito significativa para as placas: isso porq. a agua iria se espalhar
por uma superficie muito, muito ampla - basicamente todos os oceanos
sao interconetados. Assim o peso extra da agua do gelo derretido iria
se espalhar por uma superficie muito ampla.

Imagine o seguinte cenario: se a carga de um caminhao de batatas
caisse inteirinho sobre vc - isso *nao* eh uma praga - vc estaria
muito, muito encrencado. Mas imagine agora todas as batatas
distribuidas entre milhares de pessoas: cada qual teria algumas
centenas de gramas de batata nas maos - e cada um aguentaria esse peso
extra sem problemas.

De todo modo, apesar de algumas medicoes indicarem q. estah havendo o
degelo, sua intensidade atual ainda eh muito baixa para ser
considerada como afetando o nivel do mar hj.

O terrivel maremoto q. arrasou paises banhados pelo Indico foi causado
por um tremor de grande magnitude na costa de Sumatra. Esse tremor foi
causado pela atividade tectonica da borda da placa local. Se virmos
uma figura da distribuicao das placas como a seguir:

http://www.iris.edu/seismon/imgs/plates.gif

Perceberemos q. o local em q. ocorre o sismo coincide com a borda de
uma placa - a eurasiana. O q. surpreendeu nao foi a ocorrencia do
tremor, mas sua intensidade. Eles sao raros - ainda bem - mas ocorrem
de tempos de em tempos, qdo grande quantidade de energia eh acumulada
em um ponto devido ao atrito entre as placas.

Os sismos sao gerados mais frequentemente qdo as placas estao uma se
batendo contra a outra - uma indo por baixo da outra ou as duas
deslizando lado a lado. As placas africana e sulamericana encontram-se
no meio do Atlantico Sul, mas estao as duas afastando-se entre si,
entao a probabilidade de geracao de sismos dessa natureza eh menor -
embora nao totalmente impossivel - por isso a probabilidade de
ocorrencia de tsunami na nossa costa ou na costa ocidental africana eh
menor.

[]s,

Roberto Takata





SUBJECT: explique isso se puderes.
FROM: "rayfisica" <rayfisica@yahoo.com.br>
TO: ciencialist@yahoogrupos.com.br
DATE: 17/01/2005 11:05


Penso eu em minha ignorância que graças a Deus o universo
não é tão
simples como querem os inventores do neutrino

http://www.fisica.ufc.br/donafifi/raioscosmicos/raioscosmicos6.htm

Nos últimos anos, a comunidade de caçadores de raios
cósmicos agitou-
se com a descoberta de partículas que chegam à Terra com
energia
quase inacreditável. Algumas, detetadas bem recentemente, chegam
com
energias bilhões de vezes superiores à maior energia
alcançada nos
maiores aceleradores que existem. Dizem que a mais danada tinha
energia 10.000.000 (107) vezes maior que a maior energia conseguida
em aceleradores. Sua velocidade era 99,99999999999999999999% da
velocidade da luz. Com tal velocidade, a partícula pode atravessar
o
universo visível em menos de 3 semanas!






SUBJECT: Re: explique isso se puderes.
FROM: "rmtakata" <rmtakata@altavista.net>
TO: ciencialist@yahoogrupos.com.br
DATE: 17/01/2005 11:16


--- Em ciencialist@yahoogrupos.com.br, "rayfisica" <rayfisica@y...>
> Com tal velocidade, a partícula pode atravessar o universo visível em
> menos de 3 semanas!
http://www.fisica.ufc.br/donafifi/raioscosmicos/raioscosmicos6.htm

Ha' um erro grotesco aqui. O universo visivel tem bilhoes de anos luz
- se a particula viajasse a velocidade da luz levaria uns 10 bilhoes
de anos para atravessar o universo visivel.

Poderemos considerar o universo visivel a olho nu e mesmo assim tem
centenas de milhares de anos-luz - somente a Via Lactea tem esse tamanho.

Para atravessar em meras 3 semanas, a particula teria q. correr a
velocidades *maiores* do q. a luz e nao a uma fracao menor do q. a da
luz: 99,99999999999999999999% c < c.

Então eu posso explicar isso apontando esse erro crasso.

[]s,

Roberto Takata





SUBJECT: Re: [ciencialist] A igreja católica é a favor da evolução?
FROM: José Renato <jrma@terra.com.br>
TO: <ciencialist@yahoogrupos.com.br>
DATE: 17/01/2005 11:26

Homero escreveu: Este trecho é um primor de desinformação e desonestidade
argumentativa:
" Por que razão o sangue só coagula no ponto de sangramento e não dentro
das veias e artérias? E por que pára quando cessa o sangramento? Imagine os
bilhões de animais que teriam sangrado até morrer, ou teriam morrido por uma
coagulação inadequada antes que esse processo incrível tivesse sido
aperfeiçoado por mero acaso! "
....................................................

Caro Homero, o que a grosso modo chamamos de sobrenatural são os fenômenos
que não sabemos explicar, ainda, como ocorre. Ou melhor, não temos nem
idéia, visto que alguns deles mesmo sem as explicações satisfatórias já não
são considerados com tal. Vale ressaltar, também, do uso inadequado do
vocábulo "sobrenatural". Natural é o que ocorre na natureza, no universo.
Tudo o que ocorre "por aqui" é literalmente natural. A confusão é feita com
as idéias de normal, ordinário (natural) versus anormal ou extraordinário
(sobrenatural). Na essência a principal diferença entre criacionismo e
evolucionismo está na nomeação da causa ou agente que a promove: Deus ou
Acaso; Criador ou Interação Evolutiva; Arquiteto ou Probabilidade Quântica;
Pai ou Determinismo Científico... Todos com seus mistérios ainda não
alcançados completamente. Alguns, já bastante desgastados e outros nem
tanto...
[]s
José Renato
...................................................
From: "Oraculo" <oraculo@To: <ciencialist@yahoogrupos.com.br>
Sent: Sunday, January 16, 2005 11:57 PM
Subject: Re: [ciencialist] A igreja católica é a favor da evolução?


> Olá Amaury
>
> Como costuma acontecer com textos desse tipo, o autor não compreende muito
> bem a evolução, não compreende muito bem a matemática, não compreende
> muito
> bem a quimica e a bioquimica, etc, mas dá um bocado de palpite..:-)
>
> Alegar que a evolução tem pontos que precisam ser explicados, o que é
> correto, que tem coisa a serem descobertas, o que é correto, que tem
> cientistas e pesquisadores em conflitos em pontos específicos, o que
> também
> é correto, e concluir que ela foi refutada ou está sendo combatida como
> teoria geral, é um engano (ou má fé mesmo..:-) bem conhecido.
>
> A Igreja aceita a evolução simplesmente porque seus próceres tem estudado
> a
> fundo as evidencias e não veem como refuta-la como uma teoria geral. Sim,
> existem pontos a serem melhor explicados, como em TODA teoria e
> conhecimento
> científico, mas o brutal volume de evidências de diferentes áreas do
> conhecimento faz com que seja intelectualmente impossível refuta-la sem
> desonestidade (isso se você estudar tanto quanto os cientistas da santa
> sé..:-)
>
> Cientistas descartam Darwin!
>
> risos... Tirando a meia duzia de sempre, que já tem sua crença e
> aproveitam
> qualquer disputa pública entre pesquisadores para alegar que a evolução
> foi
> "contestada", e os criacionistas de sempre, a maioria (maioria é
> eufemismo..:-) dos pesquisadores de todas as áreas considera a teoria
> evolucionária bem embasada e provada, dentro do que se espera de provas de
> uma teoria cientifica. Os "descartes" em geral são de pontos específicos,
> lacunas, onde colocar seu mistério preferido.
>
> A evolução é matematicamente impossível
>
> Ele não compreende a matemática, e nem leu o resto do livro de Dawkins. Se
> lesse, descobriria que não só não é matematicamente impossível, como até
> mesmo estatisticamente provável. Mas, ler e pensar, nem sempre são
> confortáveis para crenças, assim, melhor pinçar um trecho e basear todo
> seu
> argumetno nele..:-) Se o leitor já pensa como ele, o autor, ótimo, ele não
> só vai gostar de ler coisas assim, como jamais lerá o livro de Dawkins.
>
> Mas, se o leitor fizer questão de conhecer a fonte, aprenderá mais e
> perceberá a tolice do argumento. Mas esse leitor que vai procurar dados e
> pensa por sí, não é o alvo desse tipo de texto..:-)
>
> Confundir acaso e aleatoriedade com a evolução é comum. "Esquecer" do
> papel
> da seleção natural na filtragem das mudanças aleatorioas dos genes
> também..:-)
>
> Este trecho é um primor de desinformação e desonestidade argumentativa:
>
> " Por que razão o sangue só coagula no ponto de sangramento e não dentro
> das veias e artérias? E por que pára quando cessa o sangramento? Imagine
> os
> bilhões de animais que teriam sangrado até morrer, ou teriam morrido por
> uma
> coagulação inadequada antes que esse processo incrível tivesse sido
> aperfeiçoado por mero acaso! "
>
> Se ele fisesse seu trabalho de pesquisador direito, saberia a resposta sem
> precisar passar pelo vexame acima..:-) Animais inicialmente NÃO TINHAM
> SANGUE. Ou seja, não precisavam de processos de coagulação. Líquidos para
> suprir elementos necessários surgiram antes do sangue, e da necessidade de
> coagular, já que os primeiros animais estavam em meios liquidos, menos
> agressivos nesse ponto. Ao passar para a superfície terrestre, os que
> podiam
> manter sua integridade mais tempo, em ambiente seco, podiam ser mais
> capazes
> de ampliar a descendencia. Milhões de anos, e um novo processo de
> coagulação, que impede a perda de sangue, é o resultado.
>
> Mas não é só isso. O autor leu, mas não compreendeu, o exemplo original de
> um dos autores que citou, que afirma que a coagulação refuta a evolução.
> Mas
> não pelos motivos que o autor pensa ter compreendido..:-) É até engraçado,
> mas ele cita o exemplo certo, mas com motivos errados..;-)
>
> O argumento original, que o autor não compreendeu, é da complexidade
> irredutivel. E o sangue seria uma prova disso. O número de elementos e
> substancias quimicas e bioquimicas envolvidas no processo de coagulação
> faria com que qualquer outra combinação fosse inútil. Assim, a coagulação
> teria de ter surgido já complexa, de uma vez, nunca por pequenos passo
> evolutivos.
>
> O que, se fosse verdade, refutaria a evolução. Mas nào é..:-)
>
> Realmente, o processo de coagulação é complexo. E, em sua forma final,
> altamente eficiente. Mas, é possível surgir um processo simples,
> inicialmente menos eficiente, e só depois se tornar um processo complexo.
> E,
> um mecanismo chamado exoadaptação, onde um mecanismo é reutilizado de
> outra
> forma (ler O Polegar do Panda de Jay Gould) também explica a coagulação.
> No
> início, um mecanismo é o fator principal. Com o tempo, um novo mecanismo
> se
> ajusta, e o que era principal, se torna acessório. Com o tempo, analisado
> apenas o resultado final, este aparenta ser uma complexidade irredutível.
>
> Se se desse ao trabalho de pesquisar isso (ou lesse o livro de Gould..:-),
> o
> autor não escreveria tanta bobagem.
>
> Desde que foi publicado, A Caixa Preta de Darwin, e as teses de Behe, já
> foram refutadas diversas vezes, cada vez com mais elementos e evidências
> (isso que dá esconder o sobrenatural dentro de lacunas, ele acaba
> despejado..:-). Se algum efeito este tipo de livro causa, é a benéfica
> explosão de pesquisas que acabam por refuta-lo e nos ensina mais e mais
> sobre os processos que guiaram nosso desenvolvimento: a evolução, os genes
> com modificações aleatórias e a seleção natural.
>
> Tem um bocado de coisas para aprender neste universo, e a ciência tem
> muito
> a pesquisar. Mas, a lacuna de um conhecimento, a falta temporária de uma
> explicação cientifica, não significa que qualquer outra explicação
> sobrenatural esteja autorizada ou seja correta..:-) Não saber (ainda)
> exatamente como determinada reação bioquímica se dá a nível celular, não
> significa que deus criou uma estátua de barro e assoprou no nariz dela
> para
> dar vida..:-)
>
> Homero
>
>
>
> ----- Original Message -----
> From: "Amauri Jr" <amaurijunior2@yahoo.com.br>
> To: "Lucimary Vargas" <sangalli@uai.com.br>; "Lenia"
> <lenia_luz@hotmail.com>; "Laracna" <laracna@pop.com.br>;
> <Conversa_de_Botequim@yahoogrupos.com.br>;
> <ciencialist@yahoogrupos.com.br>;
> <ciencia.2005@grupos.com.br>;
> <autoiniciacaodaconsciencia_plena@yahoogrupos.com.br>;
> <acropolis@yahoogrupos.com.br>
> Sent: Saturday, January 15, 2005 11:58 PM
> Subject: [ciencialist] A igreja católica é a favor da evolução?
>
>
>
> http://www.chamada.com.br/mensagens/artigos/evolucao.shtml
>
>
> A igreja católica é a favor da evolução?
>
> A maioria dos não-católicos ficou surpresa quando o papa João Paulo
> II, num documento enviado à Pontifícia Academia de Ciências do Vaticano em
> outubro de 1996, falou a favor da evolução. Na verdade, ele estava apenas
> reiterando a posição oficial do catolicismo. Considere os seguintes
> excertos:
>
> Em sua encíclica Humani generis [Sobre o Gênero Humano], de 1950, meu
> predecessor Pio XII já havia afirmado não haver oposição entre a evolução
> e
> a doutrina da fé a respeito do homem... Pio XII enfatizou este ponto
> essencial: se o corpo humano tem sua origem na matéria orgânica
> pré-existente, a alma espiritual é imediatamente criada por Deus... O
> exegeta e o teólogo precisam manter-se informados sobre... as ciências
> naturais... verdade não pode contradizer a verdade...
>
> A teoria da evolução... tem sido progressivamente aceita pelos
> pesquisadores em vários campos do conhecimento. A convergência... dos
> resultados de pesquisas conduzidas independentemente é, em si mesma, um
> argumento significativo em favor dessa teoria.(1)
>
> Sem dúvida, o fiasco embaraçoso do julgamento de Galileu veio à mente
> do papa quando ele advertiu os teólogos da Igreja a "[se manterem]
> informados sobre... as ciências naturais..." O papa Urbano VIII ameaçou de
> tortura um Galileu idoso e muito enfermo se este não renunciasse às
> alegações de que a Terra girava em torno do Sol. Ajoelhado diante do Santo
> Ofício da Inquisição de Roma, temendo pela própria vida, Galileu renunciou
> à
> sua "heresia" - mas não em seu coração. A idéia, repetidamente afirmada
> por
> papas "infalíveis", de que o Sol e todos os corpos celestes giravam em
> torno
> da Terra permaneceu como dogma católico oficial até 1992, quando o
> Vaticano
> finalmente admitiu oficialmente que Galileu estava certo.
>
> Para evitar que a ciência continue a fazer de tola a hierarquia
> "infalível" da Igreja, o papa admoestou os teólogos católicos a
> consultarem
> os cientistas antes de interpretarem as Escrituras. No entanto, Pedro, que
> os católicos insistem ter sido o primeiro papa, declarou que as Escrituras
> foram inspiradas pelo Espírito Santo (2 Pe 1.21). Certamente o Espírito
> Santo não precisa da ajuda dos cientistas! Se a Bíblia não for infalível
> quando fala do que pertence ao campo da ciência, por que confiar nela no
> que
> diz respeito a Deus e à salvação? Edward Daschbach, um sacerdote católico,
> explica que tomar a Bíblia literalmente exigiria admitir que a mulher que
> se
> assenta sobre a besta em Apocalipse 17 é a Igreja Católica Romana! Ele
> escreve:
>
> A Igreja, portanto, não aceita... a interpretação literal dos
> primeiros capítulos do livro de Gênesis... Quando os que advogam o
> criacionismo aplicam suas ferramentas fundamentalistas a este último livro
> [Apocalipse], a Igreja muitas vezes se torna alvo de veementes ataques.(2)
>
> Protestantes que, como Charles Colson, juntaram forças com Roma,
> advogam que o catolicismo concorda com eles sobre a inerrância da Bíblia.
> Pelo contrário, o Concílio Vaticano II declara: "Daí afirmarmos que a
> Bíblia
> é livre de erro naquilo que pertence à verdade religiosa revelada para
> nossa
> salvação. Não é necessariamente livre de erro em outros assuntos (por
> exemplo, ciências naturais)" [ênfase no original].(3)
>
> Isso não é uma questão trivial. Se o relato da criação em Gênesis não
> é digno de confiança, o restante da Bíblia também não pode ser confiável,
> pois depende desse relato. Além disso, prova-se que Cristo não era
> realmente
> Deus, mas um mero mortal que, tolamente interpretou literalmente a
> história
> de Adão e Eva (Mt 19.4-5), e não pode, portanto, ser nosso Salvador. O
> periódico The American Atheist [O Ateu Americano] sabe muito bem qual é a
> questão: "Destruam-se Adão e Eva e o pecado original, e nos escombros se
> encontrarão os restos mortais do Filho de Deus, eliminando-se assim
> qualquer
> significado para sua morte."(4)
>
> Em maio de 1982, honrando o centenário da morte de Darwin, a
> Pontifícia Academia de Ciências do Vaticano publicou a seguinte
> declaração:
> "Grande quantidade de evidências torna a aplicação do conceito de
> evolução... acima de qualquer discussão séria".(5) A Nova Enciclopédia
> Católica diz:
>
> Especialistas... por mais de cem anos, reuniram as provas
> necessárias... a evolução está estabelecida tão firmemente quanto a
> ciência
> é capaz de estabelecer fatos...(6)
>
> Cientistas descartam Darwin!
>
> Nem tanto assim. Um número cada vez maior de cientistas, a maioria
> deles não-cristãos, se opõe à evolução. O astrônomo e matemático Sir Fred
> Hoyle diz: "O mundo científico foi iludido e acabou crendo que a evolução
> fora provada. Nada poderia estar mais longe da verdade".(7) O biólogo
> Michael Denton, autor de Evolution: A Theory in Crisis [Evolução: Uma
> Teoria
> em Crise], diz que a ciência desacreditou tão completamente o
> evolucionismo
> darwiniano que este deveria ser descartado. O professor de matemática
> Wolfgang Smith chama a evolução de "um mito metafísico... completamente
> desprovido de aprovação científica..."(8)
>
> Colin Patterson, paleontólogo-chefe do Museu Britânico de História
> Natural, confessou depois de mais de vinte anos envolvido com o movimento
> evolucionista: "Nada havia que eu realmente conhecesse sobre a evolução. É
> um choque enorme descobrir-se enganado por tanto tempo". Patterson
> "começou
> a pedir a outros cientistas que lhe apresentassem uma coisa de que tinham
> certeza sobre a evolução." Os biólogos do Museu Americano de História
> Natural em Nova Iorque ficaram mudos. Diz Patterson:
>
> Experimentei a pergunta com o pessoal da geologia do Museu de Campo
> de
> História Natural, e a única resposta que recebi foi o silêncio. Tentei
> obter
> resposta dos membros do Seminário de Morfologia Evolucionista na
> Universidade de Chicago, um grupo prestigioso de evolucionistas, e recebi
> de
> volta um longo silêncio, até que, por fim, uma pessoa disse: "Eu sei uma
> coisa - não deveria ser ensinada no primeiro e segundo grau."(9)
>
> A despeito disso, no caso Edwards versus Aguillard, 482 U.S. 578
> (1978), a Suprema Corte americana decidiu que era inconstitucional que as
> escolas ensinassem o criacionismo lado a lado com o darwinismo como uma
> outra teoria de origens. Os evangélicos reclamam com justiça por ver a
> evolução ensinada como fato nas escolas públicas, mas ela também é
> ensinada
> como fato em escolas católicas.(10) Na revista The Catholic World Report,
> Stephen F. Smith escreve: "Na escola arquidiocesana de Washington, fomos
> ensinados que a teoria da evolução de Darwin era tão verdadeira quanto o
> evangelho."(11) Michael Behe, bioquímico, relembra seus dias em escolas
> católicas:
>
> Fui ensinado... a vida... veio de Deus, e que... a principal
> explicação científica de como Ele o fizera era a teoria darwiniana da
> evolução. Eu não... via qualquer conflito com o ensino da Igreja.(12)
>
> A evolução é matematicamente impossível
>
> Em seu livro The Blind Watchmaker [O Relojoeiro Cego], o zoólogo
> Richard Dawkins, da Universidade de Oxford, um destacado evolucionista,
> chama a biologia de "o estudo de coisas complicadas que dão a aparência de
> terem sido criadas com algum propósito."(13) Sem dúvida! Uma célula, a
> menor
> unidade viva, chega a ter 100.000 moléculas, e 10.000 reações químicas
> interrelacionadas simultâneas. As células não podem ter surgido por acaso!
> Dawkins admite que cada célula contém, no seu núcleo, um banco de dados
> digitalmente codificado que é maior... do que a soma de todos os 30
> volumes
> da Enciclopédia Britânica."(14) É impossível sequer imaginar a ínfima
> probabilidade do acaso criar uma enciclopédia de 30 volumes! E isso
> equivale
> apenas a uma célula - e há trilhões de células no corpo humano, milhares
> de
> tipos diferentes, operando em relacionamentos incrivelmente complexos e
> delicadamente equilibrados!
>
> A probabilidade astronomicamente pequena torna a evolução
> matematicamente impossível. Hoyle calculou que a probabilidade da produção
> ocasional apenas das enzimas básicas para a produção da vida são de 1
> sobre
> 1 seguido de 40.000 zeros. Em comparação, a chance de, por acaso, pegar um
> átomo específico em todo o universo seria de apenas 1 sobre 1 seguido de
> 80
> zeros. Mesmo que cada átomo existente se tornasse outro universo, as
> chances
> de pegar um átomo qualquer em todos esses universos seria de apenas 1
> sobre
> 1 seguido de 160 zeros. Uma chance em 1040.000 só para produzir as enzimas
> básicas! Mas as enzimas realizam coisas notáveis, e esse fato complica
> ainda
> mais o problema da evolução com essas chances infinitamente pequenas.
>
> Por que razão o sangue só coagula no ponto de sangramento e não
> dentro
> das veias e artérias? E por que pára quando cessa o sangramento? Imagine
> os
> bilhões de animais que teriam sangrado até morrer, ou teriam morrido por
> uma
> coagulação inadequada antes que esse processo incrível tivesse sido
> aperfeiçoado por mero acaso! O sistema imunológico é ainda mais
> surpreendente, diz Behe. "A complexidade do sistema garante o insucesso de
> qualquer explicação darwiniana..."(15) E assim acontece com centenas de
> outros sistemas que sustentam a vida. Lembre-se de que esses sistemas
> precisavam ser operacionais para serem úteis; não poderiam ter evoluído em
> estágios.
>
> Em seu excelente livro, publicado em 1996, Darwin's Black Box [A
> Caixa
> Preta de Darwin], Behe documenta a incompreensível complexidade da vida em
> seu nível químico celular mais básico - uma complexidade inimaginável para
> Darwin. Behe, que afirma que a evolução "deveria ser banida",(16) demole a
> teoria darwiniana oferecendo múltiplos exemplos, no nível bioquímico, de
> elementos "irredutivelmente complexos" intrincadamente planejados, que
> nunca
> poderiam ter evoluído:
>
> [A evolução] não pode explicar a origem das complexas estruturas
> bioquímicas que sustentam a vida. Sequer tenta explicar... A conclusão de
> um
> plano inteligente flui naturalmente dos próprios dados - não de livros
> sagrados nem de crenças sectárias.(17)
>
> A evolução teísta contradiz a Bíblia
>
> Em apoio ao papa, Donald Devine escreve: "O homem pré-humano
> aparentemente existiu por milhões de anos... Isso não é uma refutação da
> Bíblia, mas uma confirmação - pois indica que foi preciso que Deus
> soprasse
> nele uma alma antes que o homem pudesse ser homem."(18) Pelo contrário! A
> evolução teísta, que exige ancestrais pré-humanos para o homem (para os
> quais nenhuma evidência jamais foi encontrada), não contradiz apenas o
> livro
> de Gênesis, mas toda a Bíblia.
>
> Moisés afirma que Deus formou Adão "do pó da terra", e que depois
> formou Eva a partir de uma de suas costelas (Gn 2.7, 18-22). Ancestrais
> pré-humanos não podem ser reconciliados com o relato autenticado por
> Jesus:
> "Não tendes lido que o Criador desde o princípio os fez homem e mulher, e
> que disse: Por esta causa deixará o homem pai e mãe, e se unirá a sua
> mulher, tornando-se os dois uma só carne?" (Mt 19.4-5). Cristo confirma o
> relato de Gênesis ao citá-lo em Seu ensino. Paulo também atesta a
> veracidade
> do relato ao declarar que "primeiro foi formado Adão, e depois Eva" (1 Tm
> 2.13-14 - ver também 1 Co 15.22, 45; Judas 14). Eles não eram um par de
> criaturas pré-humanas nas quais Deus infundiu almas humanas.
>
> Além disso, Paulo afirmou que o pecado entrou no mundo por meio de
> Adão, e pelo pecado a morte (Rm 5.12). Se Adão e Eva tivessem tido
> ancestrais que viveram e morreram por milhares (ou milhões) de anos de
> evolução até que Deus os humanizasse, a morte teria operado na terra antes
> que Adão pecasse - uma contradição clara do relato de Gênesis, do ensino
> de
> Cristo, da pregação de Paulo e do Evangelho. O cardeal de Nova Iorque,
> John
> O'Connor, diz que Adão e Eva podem ter sido "animais inferiores".(19)
>
> Evolução - uma artimanha satânica
>
> A evolução, "a mais gorda das vacas sagradas",(20) tem sido uma
> poderosa ferramenta de Satanás para convencer milhões de pessoas de que a
> Bíblia não é digna de confiança. Como afirmou Phillip Johnson, professor
> de
> direito em Berkeley: "O único propósito da história evolucionista
> darwiniana
> é... demonstrar que não é necessária a existência prévia de um ser
> inteligente...[para] haver a criação."(21) Johnson causou um choque no
> mundo
> acadêmico em 1991 ao lançar seu livro Darwin on Trial [Darwin no Banco nos
> Réus]. Com a precisão de um promotor, ele destruiu o darwinismo e acusou
> os
> evolucionistas de terem "abandonado o relato verdadeiro e preciso com o
> qual
> a ciência estava tradicionalmente compromissada, no seu zelo por extirpar
> e
> descartar a religião..."(22)
>
> A evolução teria preenchido o registro fóssil de bilhões de criaturas
> intermediárias, e no entanto nem um sequer desses "elos perdidos" foi
> encontrado! Imagine a quantidade necessária de restos mortais desses
> milhões
> de pequenos incrementos evolutivos ao longo de milhões de anos para a
> passagem de guelras a pulmões, de pernas dianteiras para asas, para
> produzir
> estômagos e sistemas digestivos, olhos, rins, cérebros e sistemas nervosos
> que se estendessem por todo o corpo, a corrente sanguínea, o esperma e o
> óvulo dos mamíferos, o ovo e sua casca para os répteis e pássaros, etc. A
> impossibilidade aumenta geometricamente, pois cada um desses sistemas é
> incrivelmente complexo e não poderia evoluir gradativamente, mas
> precisaria
> ser funcional para sustentar a vida e ajudar na "sobrevivência" - como
> seria
> o caso, por exemplo, do sofisticado sistema de radar dos morcegos.
>
> Quantos milhões de andorinhas do Ártico morreram afogadas antes que a
> primeira "aprendesse", por acaso, a navegação aérea sobre milhares de
> quilômetros de oceano? Quantos salmões se perderam e jamais conseguiram
> chegar ao riacho em que haviam nascido para desovar antes que essa
> estranha
> capacidade fosse desenvolvida? Quantas aranhas morreram de fome antes que
> o
> fantástico mecanismo de criação de teias tivesse, por acaso, surgido - e
> quem teria ensinado as aranhas a usar tal recurso? Quantos ovos de toda
> espécie de ave apodreceram antes que surgisse o instinto de chocá-los?
> Como
> foi aprendido e transmitido? Há incontáveis impossibilidades para o acaso.
>
> A preocupação atual com as "espécies ameaçadas" contradiz Darwin. A
> evolução elimina os incapazes. É impossível crer na evolução e trabalhar
> em
> prol da preservação ecológica das espécies. Como o produto final da
> evolução, o homem deveria, sem misericórdia, eliminar todos os rivais na
> luta pela sobrevivência. As contradições são intermináveis.
>
> Em seu último livro, Reason in the Balance [A Razão na Balança],
> Phillip Johnson argumenta que somente a criação divina pode explicar a
> consciência moral do homem. A natureza não tem moral. O senso ético e
> moral
> do homem desaprova a evolução. Se a evolução fosse verdadeira, deveríamos
> fechar os hospitais, parar a produção de remédios e permitir que os
> doentes
> e os fracos morressem. É impossível reconciliar bondade e compaixão com a
> sobrevivência dos mais capazes.
>
> No entanto, o homem é compelido por consciência e compaixão, prova de
> que é feito à imagem de um Deus santo e amoroso. Ao rejeitar a massacrante
> evidência de propósito no mundo que o cerca (Rm 1.18-32), e por recusar-se
> a
> obedecer às leis de Deus gravadas em sua consciência (Rm 2.14-15), o homem
> tornou-se vítima de seu próprio ego e de toda sorte de males. Apesar
> disso,
> Deus ama o homem, e em amor e graça veio a esta terra pelo nascimento
> virginal para que, como o Homem perfeito, sem pecado, pudesse morrer em
> nosso lugar, pagando a penalidade infinita que a Sua própria justiça
> exigia
> pelo pecado. É apenas com base nisso - o pleno pagamento da penalidade do
> pecado, efetuado por Cristo, e a aceitação desse pagamento por parte do
> homem - que este pode se tornar uma nova criatura em Cristo. Vamos
> permanecer leais a esse Evangelho de Jesus Cristo e à Palavra de Deus que
> o
> declara; e vamos lutar com determinação contra toda tentativa de diluir,
> perverter ou comprometer a verdade de Deus. (TBC 2/97, traduzido por
> Carlos
> Osvaldo Pinto)
>
> Notas:
>
> 1.. Papa João Paulo II, "Mensagem à Pontifícia Academia de
> Ciências", L'Osservatore Romano (30 de outubro de 1996), 3.7.
> 2.. Frei Edward Daschbach, S.V.D., "Catholics and Creationism",
> Visitor (21 de outubro de 1984), 3.
> 3.. Vaticano II, Vatican Council II, Divine Revelation (edição
> parafraseada da organização Knights of Columbus), III.I 1e.
> 4.. The American Atheist (1978), 19, conforme citado em The
> Christian News (11 de novembro de 1996), 15.
> 5.. Daschbach, loc. cit.
> 6.. New Catholic Encyclopedia, vol. 5 (McGraw-Hill, 1967), 689.
> 7.. George W. Cornell, "Scientist calls Darwin evolution theory
> absurd", Times-Advocate, 10 de dezembro de 1982, A10.
> 8.. Wolfgang Smith, Teilhard and the New Religion (Tan Books,
> 1988),
> 242.
> 9.. Thomas E. Woodward, "Doubts About Darwin", Moody Monthly
> (setembro de 1988), 20.
> 10.. The Times Picayune (Flórida, 25 de outubro de 1996), A-30.
> 11.. Stephen F. Smith, "Is Darwinism a Religion?", The Catholic
> World Report (dezembro de 1996), 50.
> 12.. William Bole, "Of Biochemistry and Belief", Our Sunday Visitor
> (1 de dezembro de 1996), 6.
> 13.. Richard Dawkins, The Blind Watchmaker (England: Longman,
> 1986),
> 1.
> 14.. Dawkins, op. cit., 18.
> 15.. Michael J. Behe, Darwin's Black Box: The Biochemical Challenge
> to Evolution (The Free Press, 1996), 139.
> 16.. Ibid., 186.
> 17.. Ibid., 192-93. 18. Donald Devine, Human Events (13 de dezembro
> de 1996), 19.
> 18.. Los Angeles Times (30 de novembro de 1996), B13.
> 19.. Doug Bandow, "Fossils and Fallacies", National Review (29 de
> abril de 1991), 47.
> 20.. Russell Schoch, "The Evolution of a Creationist", California
> Monthly (novembro de 1991), 22.
> 21.. The Catholic World Report (dezembro de 1996), 50.
> Chamada da Meia-Noite, janeiro de 1998
>
>
>
> [As partes desta mensagem que não continham texto foram removidas]
>
>
>
> ##### ##### #####
>
> Para saber mais visite
> http://www.ciencialist.hpg.ig.com.br
>
>
> ##### ##### ##### #####
> Links do Yahoo! Grupos
>
>
>
>
>
>
>
>
>
>
>
>
> ##### ##### #####
>
> Para saber mais visite
> http://www.ciencialist.hpg.ig.com.br
>
>
> ##### ##### ##### #####
> Links do Yahoo! Grupos
>
>
>
>
>
>
>
>
>
> E-mail classificado pelo Identificador de Spam Inteligente Terra.
> Para alterar a categoria classificada, visite
> http://www.terra.com.br/centralunificada/emailprotegido/imail/imail.cgi?+_u=jrma&_l=1,1105930671.349283.17234.mueru.terra.com.br,27385,Des15,Des15
>
> Esta mensagem foi verificada pelo E-mail Protegido Terra.
> Scan engine: McAfee VirusScan / Atualizado em 12/01/2005 / Versão:
> 4.4.00 - Dat 4419
> Proteja o seu e-mail Terra: http://www.emailprotegido.terra.com.br/
>



SUBJECT: Re: explique isso se puderes.
FROM: "Daniel Moser" <dmoser@pop.com.br>
TO: ciencialist@yahoogrupos.com.br
DATE: 17/01/2005 11:54


Olá pessoal.
Meu nome é Daniel e sou novo na lista. Espero realizar diversas discussões com vocês.

--- Em ciencialist@yahoogrupos.com.br, "rayfisica" <rayfisica@y...>
> Com tal velocidade, a partícula pode atravessar o universo visível em
> menos de 3 semanas!
http://www.fisica.ufc.br/donafifi/raioscosmicos/raioscosmicos6.htm

Ha' um erro grotesco aqui. O universo visivel tem bilhoes de anos luz
- se a particula viajasse a velocidade da luz levaria uns 10 bilhoes
de anos para atravessar o universo visivel.

Completando a resposta do Takata, devemos observar como está no texto do link
acima, 3 semanas é o tempo que a PARTÍCULA "sentiria" passar se cruzasse o
universo, enquanto que para nós, aqui da Terra, "sentiriamos" passar estes 10
bilhões de anos.

Daniel Moser


SUBJECT: Fw: ESCLARECIMENTO
FROM: "Luiz Ferraz Netto" <leobarretos@uol.com.br>
TO: "ciencialist" <ciencialist@yahoogrupos.com.br>
DATE: 17/01/2005 12:24

Aceito sugestões,

[]'
===========================
Luiz Ferraz Netto [Léo]
leobarretos@uol.com.br
http://www.feiradeciencias.com.br
===========================
-----Mensagem Original-----
De: <gilmar.nogueira@globo.com>
Para: <leobarretos@uol.com.br>
Enviada em: domingo, 9 de janeiro de 2005 22:53
Assunto: ESCLARECIMENTO


UMA TURBINA QUE GERA ELETRICIDADE AO INVÉS DE APRISIONAREM OS ELETROS APRISIONA
O ÁTOMO INTEIRO?

AS TURBINAS APRISIONAM OS ELETROS E OS PROTONS, SE ELA TEM DE DESINTEGRAR
O ÁTOMO PARA ISSO É QUE É A DÚVIDA ?

DE UMA FORMA SIMPLES

AGRADEÇO A RESPOSTA UM ABRAÇO DO GILMAR DE OLÍMPIA S.P



--
Internal Virus Database is out-of-date.
Checked by AVG Anti-Virus.
Version: 7.0.300 / Virus Database: 265.6.9 - Release Date: 06/01/2005




--
No virus found in this outgoing message.
Checked by AVG Anti-Virus.
Version: 7.0.300 / Virus Database: 265.6.13 - Release Date: 16/01/2005



SUBJECT: Fw: Duvida
FROM: "Luiz Ferraz Netto" <leobarretos@uol.com.br>
TO: "ciencialist" <ciencialist@yahoogrupos.com.br>
DATE: 17/01/2005 12:26

Tb aceito sugestões .........
[]'
===========================
Luiz Ferraz Netto [Léo]
leobarretos@uol.com.br
http://www.feiradeciencias.com.br
===========================
-----Mensagem Original-----
De: Rafael Vivas
Para: leobarretos@uol.com.br
Enviada em: segunda-feira, 10 de janeiro de 2005 00:34
Assunto: Duvida


Caro Professor,
Bom dia, estive navegando em seu site. Muitissímo interessante! Tenho uma duvida e ficarei feliz se o senhor puder respnde-la:
Gostaria de saber qual liquido podera ser aquecido por resistência elétrica de forma a gastar a menor energia possível e aquecer o liquido o máximo possivel? Penso que seria um liquido com baixo calor especifico, certo? Os trocadores de calor utilizam esse tipo de liquido?
Desde ja agradeço.
Cordialmente
Rafael Vivas
__________________________________________________
Converse com seus amigos em tempo real com o Yahoo! Messenger
http://br.download.yahoo.com/messenger/



--------------------------------------------------------------------------------


Internal Virus Database is out-of-date.
Checked by AVG Anti-Virus.
Version: 7.0.300 / Virus Database: 265.6.9 - Release Date: 06/01/2005

----------

No virus found in this outgoing message.
Checked by AVG Anti-Virus.
Version: 7.0.300 / Virus Database: 265.6.13 - Release Date: 16/01/2005


[As partes desta mensagem que não continham texto foram removidas]



SUBJECT: cade o neutrino do argonio 40
FROM: "rayfisica" <rayfisica@yahoo.com.br>
TO: ciencialist@yahoogrupos.com.br
DATE: 17/01/2005 12:27


Retirados de apostilas de dona fifi:

Rutherford, então, fez outra hipótese: o núcleo conteria,
além de
prótons, outras partículas com peso semelhante ao dos
prótons, mas,
sem carga elétrica. Segundo o palpite do neozelandês, essa
partícula
neutra seria híbrida, composta da associação íntima de um
próton com
um elétron. O núcleo do carbono, por exemplo, conteria 6
prótons e 6
partículas neutras. A radiação de uma beta aconteceria quando
essa
partícula neutra se rompesse em um próton e um elétron - e,
como o
elétron não fica quieto dentro de um núcleo, seria cuspido
para fora.
O núcleo, nesse processo, teria sua carga aumentada de uma unidade
e
passaria a ser o núcleo de outro elemento.



A radiação beta se constitui de elétrons emitidos pelo
núcleo.
Elétrons? E um núcleo tem elétrons? Não tem elétrons
livres, mas, um
neutron, por razões que não precisamos explicar agora, pode
virar um
próton cuspindo um elétron para fora do núcleo. A carga
elétrica
total, dessa maneira, é mantida. Como o núcleo perde uma carga
negativa (a beta, sendo um elétron, tem carga -1), seu número
atômico
cresce de uma unidade, pois ganhou um próton a mais. E como a beta
é
muito levezinha, o número de massa do núcleo fica o mesmo de
antes.
Veja o exemplo de um núcleo de cobalto-60 virando níquel-60
depois de
emitir uma beta:

----------------------------------------------------------------------
-----------------------------------------------------------

O potássio é o elemento de número 19 na tabela periódica.
Um pequena
fração de 0,0118% do potássio natural é formada pelo
isótopo 40K19,
que tem 19 prótons e 21 neutrons, é claro. Esse isótopo é
radioativo
e vira cálcio-40 emitindo uma partícula beta, segundo a
reação:
40K19 --> 40Ca20 + &#61538;-1
Esse processo não serve para datação pois as rochas costumam
ter
cálcio de tudo que é tipo, primário ou não, e não
dá para saber qual
veio do decaimento do potássio. Entretanto, o potássio-40 tem
outro
jeito curioso de decair. No caso, o núcleo do K-40
simplesmente "engole" um elétron que orbitava descuidadamente por
perto. O elétron engolido compensa a carga de um próton e o
resultado
é como se o núcleo perdesse um próton e ganhasse um neutron.
O peso
atômico não muda mas o núcleo passa a ser de outro elemento,
o
argônio-40. A reação pode ser escrita como:
40K19 + e-1 --> 40Ar18
É claro que esse processo não é radioativo pois nenhuma
partícula é
emitida - apenas o infeliz elétron é deglutido pelo núcleo
----------------------------------------------------------------------
------------------------------------------------------------
Ainda falando em simetria, CADE O NEUTRINO nesse decaimento?
Pois não deveria ser o potássio 40 + neutrino=argônio 40
Ou o argônio 40 tem o núcleo aleijado (falta um neutrino)






SUBJECT: Re: explique isso se puderes.
FROM: "rayfisica" <rayfisica@yahoo.com.br>
TO: ciencialist@yahoogrupos.com.br
DATE: 17/01/2005 12:38


--- Em ciencialist@yahoogrupos.com.br, "Daniel Moser" <dmoser@p...>
escreveu
>
> Olá pessoal.
> Meu nome é Daniel e sou novo na lista. Espero realizar diversas
discussões com vocês.
>
>

Por mim mesmo seja bem vindo, eu tenho aprendido muito aqui, espero
poder contar contigo para continuar aprendendo.

Quanto a minha questão, deixemos claro que apenas colei o texto.

E se erro há não diminui a importância daquelas apostilas

obrigado





SUBJECT: Re: explique isso se puderes.
FROM: "rmtakata" <rmtakata@altavista.net>
TO: ciencialist@yahoogrupos.com.br
DATE: 17/01/2005 12:40


--- Em ciencialist@yahoogrupos.com.br, "Daniel Moser" <dmoser@p...>
> Completando a resposta do Takata, devemos observar como está no texto
> do link acima, 3 semanas é o tempo que a PARTÍCULA "sentiria" passar
> se cruzasse o universo, enquanto que para nós, aqui da Terra,
> "sentiriamos" passar estes 10 bilhões de anos.

Ah! Ok. Entao o erro grotesco foi desta mula aqui q. nao leu o texto
em seu contexto.

[]s,

Roberto Takata





SUBJECT: Re: Fw: ESCLARECIMENTO
FROM: "rmtakata" <rmtakata@altavista.net>
TO: ciencialist@yahoogrupos.com.br
DATE: 17/01/2005 13:23


As turbinas dos geradores na verdade nao aprisionam, o q. elas fazem
eh girar potentes imas. Esses imas ao girar induzem uma corrente em
uma bobina q. envolve o ima.

Se tiver disponivel um multimetro, um pedaco de fio de cobre e um
pequeno ima eh possivel fazer um experimento para observar a induz de
corrente eletrica por campos magneticos variaveis. Ajuste o multimetro
para medir correntes eletricas de baixa amperagem, conete-os 'as
extremidades do fio de cobre. Aproxime o ima do fio de cobre e mova
para frente e para tras ao longo do comprimento do fio. Verifique se o
ponteiro do multimetro/amperimetro se move (se for digital, se a
leitura do mostrador se altera).

Diversas fontes pode fazer a turbina girar. As usina hidreletricas
fazem com q. uma corrente de agua gire a turbina. As termoeletricas
aquecem vapor d'agua pela queima de combustiveis. As termonucleares
aquecem vapor d'agua pela quebra de nucleos de atomos instaveis como o
uranio. As eolicas, usam a energia dos ventos; as geotermicas, a
energia termica q. vEm de partes profundas da Terra. Ha' os q. usam a
batida das ondas ou o sobe e desce das mares. Pequenos geradores podem
funcionar pela forca muscular humana ou de outro animal - como aqueles
usados para fazer acender o farol da bicicleta.

[]s,

Roberto Takata

--- Em ciencialist@yahoogrupos.com.br, "Luiz Ferraz Netto"
> ===========================
> Luiz Ferraz Netto [Léo]
> -----Mensagem Original-----
> De: <gilmar.nogueira@g...>
> UMA TURBINA QUE GERA ELETRICIDADE AO INVÉS DE APRISIONAREM OS
> ELETROS APRISIONA O ÁTOMO INTEIRO?
>
> AS TURBINAS APRISIONAM OS ELETROS E OS PROTONS, SE ELA TEM DE
> DESINTEGRAR O ÁTOMO PARA ISSO É QUE É A DÚVIDA ?
>
> DE UMA FORMA SIMPLES
>
> AGRADEÇO A RESPOSTA UM ABRAÇO DO GILMAR DE OLÍMPIA S.P





SUBJECT: Re: Economia mundial: previsão p/2020
FROM: "rmtakata" <rmtakata@altavista.net>
TO: ciencialist@yahoogrupos.com.br
DATE: 17/01/2005 13:33


--- Em ciencialist@yahoogrupos.com.br, José Renato <jrma@t...>
> Da Redação (editorultimosegundo@i...)
> A economia do Brasil será maior do que a maioria dos países da
> Europa dentro de 15 anos.

Ueh, jah faz decadas q. a economia do Brasil eh maior do q. a da
maioria dos paises da Europa... Perde apenas para as da Italia,
Espanha, Inglaterra e Alemanha.

> Ao lado da Rússia, Indonésia e África do Sul, o País será uma das
> grandes potências do mundo - um patamar abaixo da China e da Índia -

A Africa do Sul tem um grande problema: a Aids. Se ela nao conseguir
controla'-la poderah ter seu futuro economico abortado. A China nao
sei ateh qdo conseguirah evitar o dilema entre o crescimento politico
e a liberdade interna. Indonesia e Russia terao q. enfrentar a questao
do terrorismo (de fundamentalistas islamicos e nacionalistas
chechenos). Todos, incluindo o Brasil, terao q. enfrentar a questao da
desigualdade da distribuicao de riquezas. Farao a licao de casa ateh 2020?

[]s,

Roberto Takata





SUBJECT: Re: Economia mundial: previsão p/2020
FROM: "rmtakata" <rmtakata@altavista.net>
TO: ciencialist@yahoogrupos.com.br
DATE: 17/01/2005 13:41


--- Em ciencialist@yahoogrupos.com.br, "rmtakata" <rmtakata@a...>
> Ueh, jah faz decadas q. a economia do Brasil eh maior do q. a da
> maioria dos paises da Europa... Perde apenas para as da Italia,
> Espanha, Inglaterra e Alemanha.

Upa! Esqueci-me dos franceses.

[]s,

Roberto Takata





SUBJECT: Fw: amplificar energia
FROM: "Luiz Ferraz Netto" <leobarretos@uol.com.br>
TO: "ciencialist" <ciencialist@yahoogrupos.com.br>
DATE: 17/01/2005 13:45

Quem sabe?
[] '
===========================
Luiz Ferraz Netto [Léo]
leobarretos@uol.com.br
http://www.feiradeciencias.com.br
===========================
-----Mensagem Original-----
De: wictor
Para: leobarretos@uol.com.br
Enviada em: terça-feira, 11 de janeiro de 2005 14:50
Assunto: amplificar energia


Ola Sr. Leo Barretos


Haveria alguma forma simples, sem muitas complicacoes de se ter um ganho de energia a partir de uma carga qualquer, ou seja, um lampada de 60 WATS consumir apenas 30 WATS e algum sistema intermediario me dar esses 30 WATS que me faltam, isso se tratando realmente de tentar economizar energia eletrica? E, existindo essa possibilidade, como poderia isso ser feito? Existe algum projeto de custo beneficio ao qual o acesso a ele seja possivel? Ficaria muito grato se alguma maneira o senhor pudesse me ajudar com isso desde que nao o venha a atrapalhar no seu cotidiano que deve ser bem corrido.



Desde ja agradeco e fico no aguardo de uma resposta



Wictor Junior Alves - Florianopolis - S.C.

(48) 266-4097 / 9125-3574


--------------------------------------------------------------------------------


Internal Virus Database is out-of-date.
Checked by AVG Anti-Virus.
Version: 7.0.300 / Virus Database: 265.6.9 - Release Date: 06/01/2005

----------

No virus found in this outgoing message.
Checked by AVG Anti-Virus.
Version: 7.0.300 / Virus Database: 265.6.13 - Release Date: 16/01/2005


[As partes desta mensagem que não continham texto foram removidas]



SUBJECT: Re: Fw: amplificar energia
FROM: "rmtakata" <rmtakata@altavista.net>
TO: ciencialist@yahoogrupos.com.br
DATE: 17/01/2005 13:53


Paineis fotovoltaicos?

[]s,

Roberto Takata

--- Em ciencialist@yahoogrupos.com.br, "Luiz Ferraz Netto"
> Quem sabe?
> ===========================
> Luiz Ferraz Netto [Léo]
> -----Mensagem Original-----
> De: wictor
> Ola Sr. Leo Barretos
> Haveria alguma forma simples, sem muitas complicacoes de se ter
um ganho de energia a partir de uma carga qualquer, ou seja, um
lampada de 60 WATS consumir apenas 30 WATS e algum sistema
intermediario me dar esses 30 WATS que me faltam, isso se tratando
realmente de tentar economizar energia eletrica? E, existindo essa
possibilidade, como poderia isso ser feito? Existe algum projeto de
custo beneficio ao qual o acesso a ele seja possivel? Ficaria muito
grato se alguma maneira o senhor pudesse me ajudar com isso desde que
nao o venha a atrapalhar no seu cotidiano que deve ser bem corrido.





SUBJECT: OFF-TOPIC: Fortran-90
FROM: Paulo Sérgio Dias <psdlistdisc@terra.com.br>
TO: <ciencialist@yahoogrupos.com.br>
DATE: 17/01/2005 14:09

Pessoal:

Um ex-professor entrou em contato comigo,
perguntando se eu tinha conhecimentos
em FORTRAN-90, para fazer algumas
alteracoes em rotinas que ele encontrou
em um site. Estudei FORTRAN ha' uns
10 anos, mas nunca mais mexi nisso.
Nao me sinto em condicoes de aceitar tal tarefa...

Se alguem conhece essa linguagem,
ou se puder indicar alguem, solicito
a gentileza de enviar um e-mail
em private (psdias.sor@terra.com.br),
para eu repassar a esse meu ex-professor
(nao tenho ideia de quanto vale tal servico)

Muito obrigado pela atencao de todos
Paulo

[As partes desta mensagem que não continham texto foram removidas]



SUBJECT: Re: [ciencialist] Fw: Por favor... preciso de ajuda !!
FROM: "E m i l i a n o C h e m e l l o" <chemelloe@yahoo.com.br>
TO: <ciencialist@yahoogrupos.com.br>
DATE: 17/01/2005 15:21

Pai de santo Léo,

Diga para a moça que ela deve desligar a chave geral da casa, comprar
velas (ou lampiões, o que for mais barato) e conviver com o inevitável: se
existir eletricidade, 'ele' lhe perturbará!

Falando sério. Sugiro que a moça vá para um psiquiátra. Me parece um
caso de esquisofrenia. As evidências que levam a esse parecer são as
descrições que ela faz da suposta 'perseguição' e a idade 'meio' avançada
(50 anos).

[ ] 's
Emiliano, que ainda não sabe o significado da palavra 'papibaquígrafo' :-)


----- Original Message -----
From: Luiz Ferraz Netto
To: ciencialist
Sent: Monday, January 17, 2005 10:11 AM
Subject: [ciencialist] Fw: Por favor... preciso de ajuda !!


Esse é Imperdível!

[]'
===========================
Luiz Ferraz Netto [Léo]
leobarretos@uol.com.br
http://www.feiradeciencias.com.br
===========================
-----Mensagem Original-----
De: Marcia Anitta
Para: leobarretos@uol.com.br
Enviada em: domingo, 9 de janeiro de 2005 12:41
Assunto: Por favor... preciso de ajuda !!


Olá! Prof.
Ano passado peguei um hacker aqui em meu pc.
Depois de muito gastos consegui retirá-lo...
Mas ele não é um simples hacker...é uma pessoa muito inteligente e
estudiosa.

-Ele agora tem o "domínio" de minha casa com a eletricidade.

Interage comigo pela tv( ligando a luzinha do stéreo e fazendo interferência
de formas variadas, vezes riscos e em outras retorce a imagem), como se
tivesse conversando comigo, durante noticiário , novelas e em outros
programas , rádio( não posso ouvir radio que pedem músicas, pois ele
interage tb e não posso ouvir por tempo a mesma rádio), radio -relógio( já o
desligou, mudou de estação e perdi a hora de compromissos), e o pior já
estragou minha máquina de lavar roupas que era "eletrônica" e agora é
manual, pois fez ela trabalhar sozinha e alagou toda minha área de serviço.
A geladeira será a próxima coisa que ele irá estragar, pois para interagir
"todo momento" ele liga e a desliga, e está sempre acelerada. E o gelo
forma-se rapidamente e tenho que ficar degelando sempre!
O ligar , o clique que ele dá é alto e quando ele acorda já começa, até
conseguir me acordar... Quando ele tem insônia é uma "barra", pois não para
! E não me deixa dormir.
Troquei de telefone mais ...sei que ainda escuta o que falamos no telefone,
por isso é raro eu usá-lo.
Sei tb que ele é radioamador...ou Dx...sei lá o que ele é; mas não sei
"quem" é!
Entende muito desse assunto, isso eu sei.
Minhas lâmpadas não paravam ...queimavam , agora parou um pouco, pois ele
"altera" a intensidade da energia daqui de casa.
É só eu acender uma lâmpada, que ele já sabe que estou em casa.
Sabe tudo que faço "se eu usar " a energia.
Estou cansada de tudo isso. Não sei como "falar" para ele parar.
Quanto mais ele faz isso mais irritada eu fico.
Estou estressada, sem saber como conseguir sair dessa!

Por favor tem algo que eu possa fazer?
Como "anular" e conseguir que ele "Não" tenha acesso aqui em casa?
Por favor! Me ajude!

Não posso ficar me expondo por aí, NÃO acreditam e não tenho como provar que
essas coisas acontecem. Já tentei e nada consegui.

Só eu e meu filho sabemos que tudo que relatei é "verdade" e acontecem.

Não sou criança tenho, quase 50 anos e estou presa em minha própria casa por
um "paranóico!

Agradeço se puder ajudar-me.

Marcia.
__________________________________________________
Converse com seus amigos em tempo real com o Yahoo! Messenger
http://br.download.yahoo.com/messenger/



----------------------------------------------------------------------------
----


Internal Virus Database is out-of-date.
Checked by AVG Anti-Virus.
Version: 7.0.300 / Virus Database: 265.6.9 - Release Date: 06/01/2005

----------

No virus found in this outgoing message.
Checked by AVG Anti-Virus.
Version: 7.0.300 / Virus Database: 265.6.13 - Release Date: 16/01/2005


[As partes desta mensagem que não continham texto foram removidas]



##### ##### #####

Para saber mais visite
http://www.ciencialist.hpg.ig.com.br


##### ##### ##### #####


Yahoo! Grupos, um serviço oferecido por:








Links do Yahoo! Grupos

Para visitar o site do seu grupo na web, acesse:
http://br.groups.yahoo.com/group/ciencialist/

Para sair deste grupo, envie um e-mail para:
ciencialist-unsubscribe@yahoogrupos.com.br

O uso que você faz do Yahoo! Grupos está sujeito aos Termos do Serviço do
Yahoo!.




SUBJECT: Re: Fw: Por favor... preciso de ajuda !!
FROM: "rmtakata" <rmtakata@altavista.net>
TO: ciencialist@yahoogrupos.com.br
DATE: 17/01/2005 16:32


--- Em ciencialist@yahoogrupos.com.br, "E m i l i a n o C h e m e l
> Emiliano, que ainda não sabe o significado da palavra
> 'papibaquígrafo' :-)

Oras, basta consultar o Novíssimo Diccionário Embromatológico da
Lengua Portogueza de 1745. Aqui o verbete:

"Papibaquígrafo. 1. Diz-se daquele que papibaquigrafa. 2.
Papibaquigráfico. 3. Papibaquigrafologista."

[]s,

Roberto Takata





SUBJECT: Unidades de medida: símbolo e nome da grandeza
FROM: "E m i l i a n o C h e m e l l o" <chemelloe@yahoo.com.br>
TO: <ciencialist@yahoogrupos.com.br>, <forum-ciencia@yahoogrupos.com.br>
DATE: 17/01/2005 16:56

Olá pessoal,

Tá rolando no orkut uma discussão sobre unidades de medida. Resolvi
trazê-la para cá. As questões são as seguinte:

Um dos participantes escreveu: " ... Já vi autores defenderem moles como
no Inglês. Nós brasileiros escrevemos mols. Como quem manda aqui somos nós..
levando em consideração a sonoridade da língua, como vocês acham que devem
ser? "

Além disso, proponho a discussão sobre outro aspecto: Quando utilizamos
o símbolo 'mol', da mesma forma que utilizamos 'g' para se referir a massa ,
não colocamos o símbolo da grandeza no plural. Exemplo: 2 g e *não* 2 gs; 2
mol / L e *não* 2 mols / L Já quando estamos escrevendo o nome da grandeza,
ai sim podemos utilizar o plural: Ex: 2 gramas de cloreto de sódio; 2 mols
de bicarbonato de sódio. Segundo o inmetro, eu estou correto:

http://www.inmetro.gov.br/consumidor/unidLegaisMed.asp

A grande confusão se dá porque o símbolo e a unidade da grandeza são o
mesmo: 'mol'.

Aguardo considerações,

Emiliano Chemello




SUBJECT: PUC RS 2005 - Resolução da prova de química
FROM: "E m i l i a n o C h e m e l l o" <chemelloe@yahoo.com.br>
TO: <quimica-qaw@yahoogrupos.com.br>, <forum-ciencia@yahoogrupos.com.br>, <ciencialist@yahoogrupos.com.br>, <naeq-ucs@yahoogrupos.com.br>, <quimica@grupos.com.br>, <quimica.pe@grupos.com.br>
DATE: 17/01/2005 17:06

Olá Pessoal,

Quem quiser dar uma olhada na resolução da prova de química do
vestibular PUC-RS Verão 2005 pode acessá-la através do seguinte endereço:

www.unificado.com.br/provas/p_pucrs/coment/qui_pucrs051_emiliano.pdf

Aguardo comentários!

[ ] 's do
Emiliano Chemello




SUBJECT: Re: [ciencialist] PUC RS 2005 - Resolução da prova de química
FROM: Italo <itscefet03@yahoo.com.br>
TO: ciencialist@yahoogrupos.com.br
DATE: 17/01/2005 17:35


Emiliano, a PUC-RS deu a Tabela Periódica na Prova?



Ítalo



---------------------------------
Yahoo! Acesso Grátis - Internet rápida e grátis. Instale o discador do Yahoo! agora.

[As partes desta mensagem que não continham texto foram removidas]



SUBJECT: Re: [ciencialist] Re: Fw: amplificar energia
FROM: "Eurico Ferreira de Souza Jr." <caodejah@yahoo.com.br>
TO: ciencialist@yahoogrupos.com.br
DATE: 17/01/2005 19:10

[E]> lâmpada de 220 v ? :D

rmtakata <rmtakata@altavista.net> wrote:
Paineis fotovoltaicos?

[]s,

Roberto Takata

--- Em ciencialist@yahoogrupos.com.br, "Luiz Ferraz Netto"
> Quem sabe?
> ===========================
> Luiz Ferraz Netto [Léo]
> -----Mensagem Original-----
> De: wictor
> Ola Sr. Leo Barretos
> Haveria alguma forma simples, sem muitas complicacoes de se ter
um ganho de energia a partir de uma carga qualquer, ou seja, um
lampada de 60 WATS consumir apenas 30 WATS e algum sistema
intermediario me dar esses 30 WATS que me faltam, isso se tratando
realmente de tentar economizar energia eletrica? E, existindo essa
possibilidade, como poderia isso ser feito? Existe algum projeto de
custo beneficio ao qual o acesso a ele seja possivel? Ficaria muito
grato se alguma maneira o senhor pudesse me ajudar com isso desde que
nao o venha a atrapalhar no seu cotidiano que deve ser bem corrido.





##### ##### #####

Para saber mais visite
http://www.ciencialist.hpg.ig.com.br


##### ##### ##### #####


Yahoo! Grupos, um serviço oferecido por:



















function SearchComboBox() { if (document.form_combo.keyword.value.length==0){ alert("Por favor, digite algo."); return false; }else { document.form_combo.action ="http://br.rd.yahoo.com/SIG=12afjacuo/M=264105.3931087.6562589.1588051/D=brclubs/S=2137111528:HM/EXP=1106063650/A=2361264/R=0/SIG=11uaou2jn/*http://www.bondfaro.com/bondfaro/in/combosearch_in.jsp?sk=11"; } return true;} [input] [input] [input]

---------------------------------
Links do Yahoo! Grupos

Para visitar o site do seu grupo na web, acesse:
http://br.groups.yahoo.com/group/ciencialist/

Para sair deste grupo, envie um e-mail para:
ciencialist-unsubscribe@yahoogrupos.com.br

O uso que você faz do Yahoo! Grupos está sujeito aos Termos do Serviço do Yahoo!.




_\|/_

---------------------------------
Yahoo! Acesso Grátis - Internet rápida e grátis. Instale o discador do Yahoo! agora.

[As partes desta mensagem que não continham texto foram removidas]



SUBJECT: Re: [ciencialist] A igreja católica é a favor da evolução?
FROM: Amauri Jr <amaurijunior2@yahoo.com.br>
TO: ciencialist@yahoogrupos.com.br
DATE: 17/01/2005 19:18

Oi Oraculo


Muita gente sabe que esses autores sabem nada e só propaganda barata da
igreja catolica, mas vimos muitos erros em tal teporia que são muito
grosseiros. Como por exemplo, hoje ja meio desvendado o aparecimento da
vida da Terra, onde temos os que defende que ela de alguma forma
apareceu de um conjunto de coisas e componentes daqui mesmo; outros,
defende que os acidos vieram cristalisados dentro de meteoros que cairam
na Terra quando essa era jovem. Essa eu acredito muito mais, porque nào
seria possivel, a Terra ter todos os componentes necessarios...:)

Outro lado na materia, vimos a ingnorancia que possamos ter com o sangue
se existia ou não nos primordios. Todo aluno que faz uma aula de
biologia basica, sabe que o sangue apareceu com animais já complexos; já
no começo, não haveria o "porque" de se ter um sangue ou coisa parecida.
As criaturas una celulares não podiam ter ou espaço para a tal...:) Mas,
coagulação veio para reparar uma fissura para o animal nào ter todo o
sangue exaurido, por isso não é muito complexo quando pensamos...:) O
cientitas pensam?

Abraços
Amauri






Oraculo wrote:

> Olá Amaury
>
> Como costuma acontecer com textos desse tipo, o autor não compreende muito
> bem a evolução, não compreende muito bem a matemática, não compreende
> muito
> bem a quimica e a bioquimica, etc, mas dá um bocado de palpite..:-)
>
> Alegar que a evolução tem pontos que precisam ser explicados, o que é
> correto, que tem coisa a serem descobertas, o que é correto, que tem
> cientistas e pesquisadores em conflitos em pontos específicos, o que
> também
> é correto, e concluir que ela foi refutada ou está sendo combatida como
> teoria geral, é um engano (ou má fé mesmo..:-) bem conhecido.
>
> A Igreja aceita a evolução simplesmente porque seus próceres tem
> estudado a
> fundo as evidencias e não veem como refuta-la como uma teoria geral. Sim,
> existem pontos a serem melhor explicados, como em TODA teoria e
> conhecimento
> científico, mas o brutal volume de evidências de diferentes áreas do
> conhecimento faz com que seja intelectualmente impossível refuta-la sem
> desonestidade (isso se você estudar tanto quanto os cientistas da santa
> sé..:-)
>
> Cientistas descartam Darwin!
>
> risos... Tirando a meia duzia de sempre, que já tem sua crença e
> aproveitam
> qualquer disputa pública entre pesquisadores para alegar que a
> evolução foi
> "contestada", e os criacionistas de sempre, a maioria (maioria é
> eufemismo..:-) dos pesquisadores de todas as áreas considera a teoria
> evolucionária bem embasada e provada, dentro do que se espera de provas de
> uma teoria cientifica. Os "descartes" em geral são de pontos específicos,
> lacunas, onde colocar seu mistério preferido.
>
> A evolução é matematicamente impossível
>
> Ele não compreende a matemática, e nem leu o resto do livro de Dawkins. Se
> lesse, descobriria que não só não é matematicamente impossível, como até
> mesmo estatisticamente provável. Mas, ler e pensar, nem sempre são
> confortáveis para crenças, assim, melhor pinçar um trecho e basear
> todo seu
> argumetno nele..:-) Se o leitor já pensa como ele, o autor, ótimo, ele não
> só vai gostar de ler coisas assim, como jamais lerá o livro de Dawkins.
>
> Mas, se o leitor fizer questão de conhecer a fonte, aprenderá mais e
> perceberá a tolice do argumento. Mas esse leitor que vai procurar dados e
> pensa por sí, não é o alvo desse tipo de texto..:-)
>
> Confundir acaso e aleatoriedade com a evolução é comum. "Esquecer" do
> papel
> da seleção natural na filtragem das mudanças aleatorioas dos genes
> também..:-)
>
> Este trecho é um primor de desinformação e desonestidade argumentativa:
>
> " Por que razão o sangue só coagula no ponto de sangramento e não dentro
> das veias e artérias? E por que pára quando cessa o sangramento?
> Imagine os
> bilhões de animais que teriam sangrado até morrer, ou teriam morrido
> por uma
> coagulação inadequada antes que esse processo incrível tivesse sido
> aperfeiçoado por mero acaso! "
>
> Se ele fisesse seu trabalho de pesquisador direito, saberia a resposta sem
> precisar passar pelo vexame acima..:-) Animais inicialmente NÃO TINHAM
> SANGUE. Ou seja, não precisavam de processos de coagulação. Líquidos para
> suprir elementos necessários surgiram antes do sangue, e da necessidade de
> coagular, já que os primeiros animais estavam em meios liquidos, menos
> agressivos nesse ponto. Ao passar para a superfície terrestre, os que
> podiam
> manter sua integridade mais tempo, em ambiente seco, podiam ser mais
> capazes
> de ampliar a descendencia. Milhões de anos, e um novo processo de
> coagulação, que impede a perda de sangue, é o resultado.
>
> Mas não é só isso. O autor leu, mas não compreendeu, o exemplo original de
> um dos autores que citou, que afirma que a coagulação refuta a
> evolução. Mas
> não pelos motivos que o autor pensa ter compreendido..:-) É até engraçado,
> mas ele cita o exemplo certo, mas com motivos errados..;-)
>
> O argumento original, que o autor não compreendeu, é da complexidade
> irredutivel. E o sangue seria uma prova disso. O número de elementos e
> substancias quimicas e bioquimicas envolvidas no processo de coagulação
> faria com que qualquer outra combinação fosse inútil. Assim, a coagulação
> teria de ter surgido já complexa, de uma vez, nunca por pequenos passo
> evolutivos.
>
> O que, se fosse verdade, refutaria a evolução. Mas nào é..:-)
>
> Realmente, o processo de coagulação é complexo. E, em sua forma final,
> altamente eficiente. Mas, é possível surgir um processo simples,
> inicialmente menos eficiente, e só depois se tornar um processo
> complexo. E,
> um mecanismo chamado exoadaptação, onde um mecanismo é reutilizado de
> outra
> forma (ler O Polegar do Panda de Jay Gould) também explica a
> coagulação. No
> início, um mecanismo é o fator principal. Com o tempo, um novo
> mecanismo se
> ajusta, e o que era principal, se torna acessório. Com o tempo, analisado
> apenas o resultado final, este aparenta ser uma complexidade irredutível.
>
> Se se desse ao trabalho de pesquisar isso (ou lesse o livro de
> Gould..:-), o
> autor não escreveria tanta bobagem.
>
> Desde que foi publicado, A Caixa Preta de Darwin, e as teses de Behe, já
> foram refutadas diversas vezes, cada vez com mais elementos e evidências
> (isso que dá esconder o sobrenatural dentro de lacunas, ele acaba
> despejado..:-). Se algum efeito este tipo de livro causa, é a benéfica
> explosão de pesquisas que acabam por refuta-lo e nos ensina mais e mais
> sobre os processos que guiaram nosso desenvolvimento: a evolução, os genes
> com modificações aleatórias e a seleção natural.
>
> Tem um bocado de coisas para aprender neste universo, e a ciência tem
> muito
> a pesquisar. Mas, a lacuna de um conhecimento, a falta temporária de uma
> explicação cientifica, não significa que qualquer outra explicação
> sobrenatural esteja autorizada ou seja correta..:-) Não saber (ainda)
> exatamente como determinada reação bioquímica se dá a nível celular, não
> significa que deus criou uma estátua de barro e assoprou no nariz dela
> para
> dar vida..:-)
>
> Homero
>
>
>
> ----- Original Message -----
> From: "Amauri Jr" <amaurijunior2@yahoo.com.br>
> To: "Lucimary Vargas" <sangalli@uai.com.br>; "Lenia"
> <lenia_luz@hotmail.com>; "Laracna" <laracna@pop.com.br>;
> <Conversa_de_Botequim@yahoogrupos.com.br>;
> <ciencialist@yahoogrupos.com.br>;
> <ciencia.2005@grupos.com.br>;
> <autoiniciacaodaconsciencia_plena@yahoogrupos.com.br>;
> <acropolis@yahoogrupos.com.br>
> Sent: Saturday, January 15, 2005 11:58 PM
> Subject: [ciencialist] A igreja católica é a favor da evolução?
>
>
>
> http://www.chamada.com.br/mensagens/artigos/evolucao.shtml
>
>
> A igreja católica é a favor da evolução?
>
> A maioria dos não-católicos ficou surpresa quando o papa João Paulo
> II, num documento enviado à Pontifícia Academia de Ciências do Vaticano em
> outubro de 1996, falou a favor da evolução. Na verdade, ele estava apenas
> reiterando a posição oficial do catolicismo. Considere os seguintes
> excertos:
>
> Em sua encíclica Humani generis [Sobre o Gênero Humano], de
> 1950, meu
> predecessor Pio XII já havia afirmado não haver oposição entre a
> evolução e
> a doutrina da fé a respeito do homem... Pio XII enfatizou este ponto
> essencial: se o corpo humano tem sua origem na matéria orgânica
> pré-existente, a alma espiritual é imediatamente criada por Deus... O
> exegeta e o teólogo precisam manter-se informados sobre... as ciências
> naturais... verdade não pode contradizer a verdade...
>
> A teoria da evolução... tem sido progressivamente aceita pelos
> pesquisadores em vários campos do conhecimento. A convergência... dos
> resultados de pesquisas conduzidas independentemente é, em si mesma, um
> argumento significativo em favor dessa teoria.(1)
>
> Sem dúvida, o fiasco embaraçoso do julgamento de Galileu veio à
> mente
> do papa quando ele advertiu os teólogos da Igreja a "[se manterem]
> informados sobre... as ciências naturais..." O papa Urbano VIII ameaçou de
> tortura um Galileu idoso e muito enfermo se este não renunciasse às
> alegações de que a Terra girava em torno do Sol. Ajoelhado diante do Santo
> Ofício da Inquisição de Roma, temendo pela própria vida, Galileu
> renunciou à
> sua "heresia" - mas não em seu coração. A idéia, repetidamente
> afirmada por
> papas "infalíveis", de que o Sol e todos os corpos celestes giravam em
> torno
> da Terra permaneceu como dogma católico oficial até 1992, quando o
> Vaticano
> finalmente admitiu oficialmente que Galileu estava certo.
>
> Para evitar que a ciência continue a fazer de tola a hierarquia
> "infalível" da Igreja, o papa admoestou os teólogos católicos a
> consultarem
> os cientistas antes de interpretarem as Escrituras. No entanto, Pedro, que
> os católicos insistem ter sido o primeiro papa, declarou que as Escrituras
> foram inspiradas pelo Espírito Santo (2 Pe 1.21). Certamente o Espírito
> Santo não precisa da ajuda dos cientistas! Se a Bíblia não for infalível
> quando fala do que pertence ao campo da ciência, por que confiar nela
> no que
> diz respeito a Deus e à salvação? Edward Daschbach, um sacerdote católico,
> explica que tomar a Bíblia literalmente exigiria admitir que a mulher
> que se
> assenta sobre a besta em Apocalipse 17 é a Igreja Católica Romana! Ele
> escreve:
>
> A Igreja, portanto, não aceita... a interpretação literal dos
> primeiros capítulos do livro de Gênesis... Quando os que advogam o
> criacionismo aplicam suas ferramentas fundamentalistas a este último livro
> [Apocalipse], a Igreja muitas vezes se torna alvo de veementes ataques.(2)
>
> Protestantes que, como Charles Colson, juntaram forças com Roma,
> advogam que o catolicismo concorda com eles sobre a inerrância da Bíblia.
> Pelo contrário, o Concílio Vaticano II declara: "Daí afirmarmos que a
> Bíblia
> é livre de erro naquilo que pertence à verdade religiosa revelada para
> nossa
> salvação. Não é necessariamente livre de erro em outros assuntos (por
> exemplo, ciências naturais)" [ênfase no original].(3)
>
> Isso não é uma questão trivial. Se o relato da criação em
> Gênesis não
> é digno de confiança, o restante da Bíblia também não pode ser confiável,
> pois depende desse relato. Além disso, prova-se que Cristo não era
> realmente
> Deus, mas um mero mortal que, tolamente interpretou literalmente a
> história
> de Adão e Eva (Mt 19.4-5), e não pode, portanto, ser nosso Salvador. O
> periódico The American Atheist [O Ateu Americano] sabe muito bem qual é a
> questão: "Destruam-se Adão e Eva e o pecado original, e nos escombros se
> encontrarão os restos mortais do Filho de Deus, eliminando-se assim
> qualquer
> significado para sua morte."(4)
>
> Em maio de 1982, honrando o centenário da morte de Darwin, a
> Pontifícia Academia de Ciências do Vaticano publicou a seguinte
> declaração:
> "Grande quantidade de evidências torna a aplicação do conceito de
> evolução... acima de qualquer discussão séria".(5) A Nova Enciclopédia
> Católica diz:
>
> Especialistas... por mais de cem anos, reuniram as provas
> necessárias... a evolução está estabelecida tão firmemente quanto a
> ciência
> é capaz de estabelecer fatos...(6)
>
> Cientistas descartam Darwin!
>
> Nem tanto assim. Um número cada vez maior de cientistas, a maioria
> deles não-cristãos, se opõe à evolução. O astrônomo e matemático Sir Fred
> Hoyle diz: "O mundo científico foi iludido e acabou crendo que a evolução
> fora provada. Nada poderia estar mais longe da verdade".(7) O biólogo
> Michael Denton, autor de Evolution: A Theory in Crisis [Evolução: Uma
> Teoria
> em Crise], diz que a ciência desacreditou tão completamente o
> evolucionismo
> darwiniano que este deveria ser descartado. O professor de matemática
> Wolfgang Smith chama a evolução de "um mito metafísico... completamente
> desprovido de aprovação científica..."(8)
>
> Colin Patterson, paleontólogo-chefe do Museu Britânico de História
> Natural, confessou depois de mais de vinte anos envolvido com o movimento
> evolucionista: "Nada havia que eu realmente conhecesse sobre a evolução. É
> um choque enorme descobrir-se enganado por tanto tempo". Patterson
> "começou
> a pedir a outros cientistas que lhe apresentassem uma coisa de que tinham
> certeza sobre a evolução." Os biólogos do Museu Americano de História
> Natural em Nova Iorque ficaram mudos. Diz Patterson:
>
> Experimentei a pergunta com o pessoal da geologia do Museu de
> Campo de
> História Natural, e a única resposta que recebi foi o silêncio. Tentei
> obter
> resposta dos membros do Seminário de Morfologia Evolucionista na
> Universidade de Chicago, um grupo prestigioso de evolucionistas, e
> recebi de
> volta um longo silêncio, até que, por fim, uma pessoa disse: "Eu sei uma
> coisa - não deveria ser ensinada no primeiro e segundo grau."(9)
>
> A despeito disso, no caso Edwards versus Aguillard, 482 U.S. 578
> (1978), a Suprema Corte americana decidiu que era inconstitucional que as
> escolas ensinassem o criacionismo lado a lado com o darwinismo como uma
> outra teoria de origens. Os evangélicos reclamam com justiça por ver a
> evolução ensinada como fato nas escolas públicas, mas ela também é
> ensinada
> como fato em escolas católicas.(10) Na revista The Catholic World Report,
> Stephen F. Smith escreve: "Na escola arquidiocesana de Washington, fomos
> ensinados que a teoria da evolução de Darwin era tão verdadeira quanto o
> evangelho."(11) Michael Behe, bioquímico, relembra seus dias em escolas
> católicas:
>
> Fui ensinado... a vida... veio de Deus, e que... a principal
> explicação científica de como Ele o fizera era a teoria darwiniana da
> evolução. Eu não... via qualquer conflito com o ensino da Igreja.(12)
>
> A evolução é matematicamente impossível
>
> Em seu livro The Blind Watchmaker [O Relojoeiro Cego], o zoólogo
> Richard Dawkins, da Universidade de Oxford, um destacado evolucionista,
> chama a biologia de "o estudo de coisas complicadas que dão a aparência de
> terem sido criadas com algum propósito."(13) Sem dúvida! Uma célula, a
> menor
> unidade viva, chega a ter 100.000 moléculas, e 10.000 reações químicas
> interrelacionadas simultâneas. As células não podem ter surgido por acaso!
> Dawkins admite que cada célula contém, no seu núcleo, um banco de dados
> digitalmente codificado que é maior... do que a soma de todos os 30
> volumes
> da Enciclopédia Britânica."(14) É impossível sequer imaginar a ínfima
> probabilidade do acaso criar uma enciclopédia de 30 volumes! E isso
> equivale
> apenas a uma célula - e há trilhões de células no corpo humano,
> milhares de
> tipos diferentes, operando em relacionamentos incrivelmente complexos e
> delicadamente equilibrados!
>
> A probabilidade astronomicamente pequena torna a evolução
> matematicamente impossível. Hoyle calculou que a probabilidade da produção
> ocasional apenas das enzimas básicas para a produção da vida são de 1
> sobre
> 1 seguido de 40.000 zeros. Em comparação, a chance de, por acaso, pegar um
> átomo específico em todo o universo seria de apenas 1 sobre 1 seguido
> de 80
> zeros. Mesmo que cada átomo existente se tornasse outro universo, as
> chances
> de pegar um átomo qualquer em todos esses universos seria de apenas 1
> sobre
> 1 seguido de 160 zeros. Uma chance em 1040.000 só para produzir as enzimas
> básicas! Mas as enzimas realizam coisas notáveis, e esse fato complica
> ainda
> mais o problema da evolução com essas chances infinitamente pequenas.
>
> Por que razão o sangue só coagula no ponto de sangramento e não
> dentro
> das veias e artérias? E por que pára quando cessa o sangramento?
> Imagine os
> bilhões de animais que teriam sangrado até morrer, ou teriam morrido
> por uma
> coagulação inadequada antes que esse processo incrível tivesse sido
> aperfeiçoado por mero acaso! O sistema imunológico é ainda mais
> surpreendente, diz Behe. "A complexidade do sistema garante o insucesso de
> qualquer explicação darwiniana..."(15) E assim acontece com centenas de
> outros sistemas que sustentam a vida. Lembre-se de que esses sistemas
> precisavam ser operacionais para serem úteis; não poderiam ter evoluído em
> estágios.
>
> Em seu excelente livro, publicado em 1996, Darwin's Black Box [A
> Caixa
> Preta de Darwin], Behe documenta a incompreensível complexidade da vida em
> seu nível químico celular mais básico - uma complexidade inimaginável para
> Darwin. Behe, que afirma que a evolução "deveria ser banida",(16) demole a
> teoria darwiniana oferecendo múltiplos exemplos, no nível bioquímico, de
> elementos "irredutivelmente complexos" intrincadamente planejados, que
> nunca
> poderiam ter evoluído:
>
> [A evolução] não pode explicar a origem das complexas estruturas
> bioquímicas que sustentam a vida. Sequer tenta explicar... A conclusão
> de um
> plano inteligente flui naturalmente dos próprios dados - não de livros
> sagrados nem de crenças sectárias.(17)
>
> A evolução teísta contradiz a Bíblia
>
> Em apoio ao papa, Donald Devine escreve: "O homem pré-humano
> aparentemente existiu por milhões de anos... Isso não é uma refutação da
> Bíblia, mas uma confirmação - pois indica que foi preciso que Deus
> soprasse
> nele uma alma antes que o homem pudesse ser homem."(18) Pelo contrário! A
> evolução teísta, que exige ancestrais pré-humanos para o homem (para os
> quais nenhuma evidência jamais foi encontrada), não contradiz apenas o
> livro
> de Gênesis, mas toda a Bíblia.
>
> Moisés afirma que Deus formou Adão "do pó da terra", e que depois
> formou Eva a partir de uma de suas costelas (Gn 2.7, 18-22). Ancestrais
> pré-humanos não podem ser reconciliados com o relato autenticado por
> Jesus:
> "Não tendes lido que o Criador desde o princípio os fez homem e mulher, e
> que disse: Por esta causa deixará o homem pai e mãe, e se unirá a sua
> mulher, tornando-se os dois uma só carne?" (Mt 19.4-5). Cristo confirma o
> relato de Gênesis ao citá-lo em Seu ensino. Paulo também atesta a
> veracidade
> do relato ao declarar que "primeiro foi formado Adão, e depois Eva" (1 Tm
> 2.13-14 - ver também 1 Co 15.22, 45; Judas 14). Eles não eram um par de
> criaturas pré-humanas nas quais Deus infundiu almas humanas.
>
> Além disso, Paulo afirmou que o pecado entrou no mundo por meio de
> Adão, e pelo pecado a morte (Rm 5.12). Se Adão e Eva tivessem tido
> ancestrais que viveram e morreram por milhares (ou milhões) de anos de
> evolução até que Deus os humanizasse, a morte teria operado na terra antes
> que Adão pecasse - uma contradição clara do relato de Gênesis, do
> ensino de
> Cristo, da pregação de Paulo e do Evangelho. O cardeal de Nova Iorque,
> John
> O'Connor, diz que Adão e Eva podem ter sido "animais inferiores".(19)
>
> Evolução - uma artimanha satânica
>
> A evolução, "a mais gorda das vacas sagradas",(20) tem sido uma
> poderosa ferramenta de Satanás para convencer milhões de pessoas de que a
> Bíblia não é digna de confiança. Como afirmou Phillip Johnson,
> professor de
> direito em Berkeley: "O único propósito da história evolucionista
> darwiniana
> é... demonstrar que não é necessária a existência prévia de um ser
> inteligente...[para] haver a criação."(21) Johnson causou um choque no
> mundo
> acadêmico em 1991 ao lançar seu livro Darwin on Trial [Darwin no Banco nos
> Réus]. Com a precisão de um promotor, ele destruiu o darwinismo e
> acusou os
> evolucionistas de terem "abandonado o relato verdadeiro e preciso com
> o qual
> a ciência estava tradicionalmente compromissada, no seu zelo por
> extirpar e
> descartar a religião..."(22)
>
> A evolução teria preenchido o registro fóssil de bilhões de
> criaturas
> intermediárias, e no entanto nem um sequer desses "elos perdidos" foi
> encontrado! Imagine a quantidade necessária de restos mortais desses
> milhões
> de pequenos incrementos evolutivos ao longo de milhões de anos para a
> passagem de guelras a pulmões, de pernas dianteiras para asas, para
> produzir
> estômagos e sistemas digestivos, olhos, rins, cérebros e sistemas nervosos
> que se estendessem por todo o corpo, a corrente sanguínea, o esperma e o
> óvulo dos mamíferos, o ovo e sua casca para os répteis e pássaros, etc. A
> impossibilidade aumenta geometricamente, pois cada um desses sistemas é
> incrivelmente complexo e não poderia evoluir gradativamente, mas
> precisaria
> ser funcional para sustentar a vida e ajudar na "sobrevivência" - como
> seria
> o caso, por exemplo, do sofisticado sistema de radar dos morcegos.
>
> Quantos milhões de andorinhas do Ártico morreram afogadas antes
> que a
> primeira "aprendesse", por acaso, a navegação aérea sobre milhares de
> quilômetros de oceano? Quantos salmões se perderam e jamais conseguiram
> chegar ao riacho em que haviam nascido para desovar antes que essa
> estranha
> capacidade fosse desenvolvida? Quantas aranhas morreram de fome antes
> que o
> fantástico mecanismo de criação de teias tivesse, por acaso, surgido - e
> quem teria ensinado as aranhas a usar tal recurso? Quantos ovos de toda
> espécie de ave apodreceram antes que surgisse o instinto de chocá-los?
> Como
> foi aprendido e transmitido? Há incontáveis impossibilidades para o acaso.
>
> A preocupação atual com as "espécies ameaçadas" contradiz Darwin. A
> evolução elimina os incapazes. É impossível crer na evolução e
> trabalhar em
> prol da preservação ecológica das espécies. Como o produto final da
> evolução, o homem deveria, sem misericórdia, eliminar todos os rivais na
> luta pela sobrevivência. As contradições são intermináveis.
>
> Em seu último livro, Reason in the Balance [A Razão na Balança],
> Phillip Johnson argumenta que somente a criação divina pode explicar a
> consciência moral do homem. A natureza não tem moral. O senso ético e
> moral
> do homem desaprova a evolução. Se a evolução fosse verdadeira, deveríamos
> fechar os hospitais, parar a produção de remédios e permitir que os
> doentes
> e os fracos morressem. É impossível reconciliar bondade e compaixão com a
> sobrevivência dos mais capazes.
>
> No entanto, o homem é compelido por consciência e compaixão,
> prova de
> que é feito à imagem de um Deus santo e amoroso. Ao rejeitar a massacrante
> evidência de propósito no mundo que o cerca (Rm 1.18-32), e por
> recusar-se a
> obedecer às leis de Deus gravadas em sua consciência (Rm 2.14-15), o homem
> tornou-se vítima de seu próprio ego e de toda sorte de males. Apesar
> disso,
> Deus ama o homem, e em amor e graça veio a esta terra pelo nascimento
> virginal para que, como o Homem perfeito, sem pecado, pudesse morrer em
> nosso lugar, pagando a penalidade infinita que a Sua própria justiça
> exigia
> pelo pecado. É apenas com base nisso - o pleno pagamento da penalidade do
> pecado, efetuado por Cristo, e a aceitação desse pagamento por parte do
> homem - que este pode se tornar uma nova criatura em Cristo. Vamos
> permanecer leais a esse Evangelho de Jesus Cristo e à Palavra de Deus
> que o
> declara; e vamos lutar com determinação contra toda tentativa de diluir,
> perverter ou comprometer a verdade de Deus. (TBC 2/97, traduzido por
> Carlos
> Osvaldo Pinto)
>
> Notas:
>
> 1.. Papa João Paulo II, "Mensagem à Pontifícia Academia de
> Ciências", L'Osservatore Romano (30 de outubro de 1996), 3.7.
> 2.. Frei Edward Daschbach, S.V.D., "Catholics and Creationism",
> Visitor (21 de outubro de 1984), 3.
> 3.. Vaticano II, Vatican Council II, Divine Revelation (edição
> parafraseada da organização Knights of Columbus), III.I 1e.
> 4.. The American Atheist (1978), 19, conforme citado em The
> Christian News (11 de novembro de 1996), 15.
> 5.. Daschbach, loc. cit.
> 6.. New Catholic Encyclopedia, vol. 5 (McGraw-Hill, 1967), 689.
> 7.. George W. Cornell, "Scientist calls Darwin evolution theory
> absurd", Times-Advocate, 10 de dezembro de 1982, A10.
> 8.. Wolfgang Smith, Teilhard and the New Religion (Tan Books,
> 1988),
> 242.
> 9.. Thomas E. Woodward, "Doubts About Darwin", Moody Monthly
> (setembro de 1988), 20.
> 10.. The Times Picayune (Flórida, 25 de outubro de 1996), A-30.
> 11.. Stephen F. Smith, "Is Darwinism a Religion?", The Catholic
> World Report (dezembro de 1996), 50.
> 12.. William Bole, "Of Biochemistry and Belief", Our Sunday
> Visitor
> (1 de dezembro de 1996), 6.
> 13.. Richard Dawkins, The Blind Watchmaker (England: Longman,
> 1986),
> 1.
> 14.. Dawkins, op. cit., 18.
> 15.. Michael J. Behe, Darwin's Black Box: The Biochemical
> Challenge
> to Evolution (The Free Press, 1996), 139.
> 16.. Ibid., 186.
> 17.. Ibid., 192-93. 18. Donald Devine, Human Events (13 de
> dezembro
> de 1996), 19.
> 18.. Los Angeles Times (30 de novembro de 1996), B13.
> 19.. Doug Bandow, "Fossils and Fallacies", National Review (29 de
> abril de 1991), 47.
> 20.. Russell Schoch, "The Evolution of a Creationist", California
> Monthly (novembro de 1991), 22.
> 21.. The Catholic World Report (dezembro de 1996), 50.
> Chamada da Meia-Noite, janeiro de 1998
>
>
>
> [As partes desta mensagem que não continham texto foram removidas]
>
>
>
> ##### ##### #####
>
> Para saber mais visite
> http://www.ciencialist.hpg.ig.com.br
>
>
> ##### ##### ##### #####
> Links do Yahoo! Grupos
>
>
>
>
>
>
>
>
>
>
>
>
> ##### ##### #####
>
> Para saber mais visite
> http://www.ciencialist.hpg.ig.com.br
>
>
> ##### ##### ##### #####
>
>
> *Yahoo! Grupos, um serviço oferecido por:*
> PUBLICIDADE
> <http://br.rd.yahoo.com/SIG=12auikna3/M=264379.5078783.6203979.1588051/D=brclubs/S=2137111528:HM/EXP=1106017054/A=2332652/R=0/id=noscript/SIG=119058f8i/*http://br.download.yahoo.com/messenger/>
>
>
>
> ------------------------------------------------------------------------
> *Links do Yahoo! Grupos*
>
> * Para visitar o site do seu grupo na web, acesse:
> http://br.groups.yahoo.com/group/ciencialist/
>
> * Para sair deste grupo, envie um e-mail para:
> ciencialist-unsubscribe@yahoogrupos.com.br
> <mailto:ciencialist-unsubscribe@yahoogrupos.com.br?subject=Unsubscribe>
>
> * O uso que você faz do Yahoo! Grupos está sujeito aos Termos do
> Serviço do Yahoo! <http://br.yahoo.com/info/utos.html>.
>
>




SUBJECT: RES: [ciencialist] A igreja cat�lica � a favor da evolu��o?
FROM: <roncolato.s@tutopia.com.br>
TO: ciencialist@yahoogrupos.com.br
DATE: 17/01/2005 19:43


Caro Amauri, ol�!

� f�cil de compreender a total confus�o do autor em assuntos da Religi�o
Cat�lica. Ele faz uma salada tremenda em seu texto, misturando livrecos de te�ricos,
com posi��es ordin�rias do Papa e uma bibliografia muit�ssimo suspeita.

A Igreja sempre ensinou uma Verdade Objetiva. Portanto, a quest�o "evolutiva"
� uma quest�o de cunho ontol�gico e n�o pode ser simplesmente discutida num foro
naturalista que hoje domina os meios pseudo-academicos. Nesta quest�o h� uma moda
mundial que tenda restringir e neutralizar cada dia mais a Religi�o Cat�lica, como se
fosse algo irracional. Santo Anselmo Doutor, dizia, que n�o cremos para crer, o que
seria uma tolice, cremos para compreender.

Infalivelmente a Igreja Cat�lica � contra o poligenismo. Ele foi condenado na
Enc�lcia 'Humani Generis' do Papa Pio XII. Assim como � contra dizer que os livros
s�o aleg�ricos, embora sua interpreta��o s� � devida ao Magist�rio extraordin�rio(ex
cathedra) do Papa a quem compete condenar ou homologar as teses.

Por fim, ou autor tenta usar de um sentimentalismo nada cient�fico, que � o
caso mal abordado de Galileu. As teses de Galileu n�o foram condenadas porque a Terra
se "movia". (apesar de n�o estar muito claras as quest�es acerca do movimento
relativo)mas, por ele afirmar em seu 'Il saggiatore' a continuidade da mat�ria de um
corpo na luz. Isso era diretamente contra o Conc�lio de Trento.

Andr�.







SUBJECT: Revista "Diversity and Distributions"/ Biogeografia de Conservação
FROM: Maria Natália <grasdic@hotmail.com>
TO: ciencialist@yahoogrupos.com.br
DATE: 17/01/2005 21:07


Compartilhando com os ambientalistas da C-List:
""A primeira edição deste ano da revista "Diversity and
Distributions", vem
um novo sub-título: uma revista de Biogeografia da Conservação. Este
sub-título substitui o anterior (revista de Invasões Biológicas). A
relevância desta mudança prende-se com o reconhecimento formal da
existência de uma nova área científica que procura aplicar
princípios,
teorias e instrumentos analíticos da Biogeografia para a conservação
da
biodiversidade. Este campo de intervenção tanto pode considerar-se
uma
sub-disciplina da Biologia da Conservação como da Biogeografia. O que
importa é reconhecer que muitos dos desafios que se colocam, hoje em
dia,
no que respeita a conservação da biodiversidade, se processam a
escalas
globais pelo que é necessário estudar muitos destes problemas a
escalas
biogeográficas e tal implica o repensar de alguns paradigmas
analíticos.

Para os mais interessados recomendamos a leitura do artigo que
formaliza o
conceito e revê alguns trabalhos que têm vindo a ser desenvolvidos
nesta
óptica:

Whittaker, R.J., Araújo, M.B., Jepson, P., Ladle, R.J., Watson,
J.E.M.,
and Willis, K.J. 2005. Conservation biogeography: assessment and
prospect.
Diversity and Distributions, 11, 3-23

Pode ser acedido directamente através do blog da ambio:

http://ambio.blogspot.com/

Boas leituras""
_______________________________________________

Um abraço
Maria Natália





SUBJECT: Revistas
FROM: "Rodrigo Toledo" <rodrigotoledo11@uol.com.br>
TO: <ciencialist@yahoogrupos.com.br>
DATE: 17/01/2005 21:28

OLá,

Alguém sabe me informar onde posso comprar exemplares das revistas Nature e
Science ?

Já tentei em diversos lugares e a única coisa que sabem me dizer é que não
comercializam por ser muito caro..

Quem souber...agradeço muito.

Obrigado,

RT



SUBJECT: Re: [ciencialist] PUC RS 2005 - Resolução da prova de química
FROM: Emiliano Chemello <chemelloe@yahoo.com.br>
TO: ciencialist@yahoogrupos.com.br
DATE: 17/01/2005 21:54

--- Italo <itscefet03@yahoo.com.br> escreveu:

Emiliano, a PUC-RS deu a Tabela Periódica na Prova?

Ítalo

---------------------------------

Olá ìtalo

Sim. Deram uma tabela periódica básica. Porém não
coloquei na resolução pois não vem junto com o pdf da
prova. Eles colocam na contra-capa.

Sem tabela fica difícil resolver :-)


[ ]'s do Emiliano Chemello



=====
[ ]'s
---
Emiliano Chemello
chemelloe@yahoo.com.br
www.quimica.net/emiliano
www.ucs.br/ccet/defq/naeq
[ MSN ] chemelloe@hotmail.com
[ ICQ ] 145060604


__________________________________________________
Converse com seus amigos em tempo real com o Yahoo! Messenger
http://br.download.yahoo.com/messenger/


SUBJECT: Re: PUC RS 2005 - Resolução da prova de química
FROM: Maria Natália <grasdic@hotmail.com>
TO: ciencialist@yahoogrupos.com.br
DATE: 17/01/2005 23:36


BOA Emiliano!

Gostava de saber se este teste escrito percorre TODO o programa para
vestibular. Ou me dá endereço de onde está o programa (depende da
Universiddade?)
Agradeço o envio do enunciado pois vai entar em minha investigação.
APENAS o enunciado! A resolução a hei-de ver mas não é disso que
tratarei e em Julho ta entrego em mão, tá?
Manda mais pois gostei
Maria Natália
--- Em ciencialist@yahoogrupos.com.br, "E m i l i a n o C h e m e
l l o" <chemelloe@y...> escreveu
> Olá Pessoal,
>
> Quem quiser dar uma olhada na resolução da prova de química do
> vestibular PUC-RS Verão 2005 pode acessá-la através do seguinte
endereço:
>
>
www.unificado.com.br/provas/p_pucrs/coment/qui_pucrs051_emiliano.pdf
>
> Aguardo comentários!
>
> [ ] 's do
> Emiliano Chemello





SUBJECT: Fw: dúvida sobre galvanizado
FROM: "Luiz Ferraz Netto" <leobarretos@uol.com.br>
TO: "ciencialist" <ciencialist@yahoogrupos.com.br>
DATE: 18/01/2005 07:11

Bom dia cuimicos.

[]'
===========================
Luiz Ferraz Netto [Léo]
leobarretos@uol.com.br
http://www.feiradeciencias.com.br
===========================
-----Mensagem Original-----
De: "raobla" <raobla@ig.com.br>
Para: <leobarretos@uol.com.br>
Enviada em: domingo, 16 de janeiro de 2005 22:05
Assunto: dúvida sobre galvanizado


assunto: galvanizado zincado

Boa Noite!

Gostaria de saber se existe algum produto químico (solução, pó, etc) para a
recuperação de uma peça metálica que sofreu o processo de galvanoplastia
(galvanizado zincado) e que depois de algum tempo apresentou manchinhas e
uma pequena perda de brilho.
Sem mais, agradeço a atenção dispensada e aguardo breve resposta.



--------------------------------------------------------------------------------


No virus found in this incoming message.
Checked by AVG Anti-Virus.
Version: 7.0.300 / Virus Database: 265.6.13 - Release Date: 16/01/2005



--
No virus found in this outgoing message.
Checked by AVG Anti-Virus.
Version: 7.0.300 / Virus Database: 265.6.13 - Release Date: 16/01/2005



SUBJECT: Re: [ciencialist] Re: PUC RS 2005 - Resolução da prova de química
FROM: "E m i l i a n o C h e m e l l o" <chemelloe@yahoo.com.br>
TO: <ciencialist@yahoogrupos.com.br>
DATE: 18/01/2005 07:57

Natália,

Abaixo, o programa da prova de química do vestibular PUC RS 2005

Substâncias químicas

Conceitos básicos:

Propriedade da matéria; substâncias simples e compostas; misturas e
processos de separação; átomos e moléculas; representação dos elementos e
substâncias químicas: símbolos e fórmulas; quantidade de matéria: constante
de Avogadro, mol e massas molares.

Constituição:

- Estrutura do átomo: evolução dos modelos atômicos; núcleo: prótons e
nêutrons; número atômico e número de massa; isótopos; noções de
radioatividade; eletrosfera; elétrons; níveis de energia e distribuição
eletrônica; potencial de ionização; afinidade eletrônica;
eletronegatividade; estado de oxidação.

- Modelos de ligação química: ligação covalente, iônica e metálica;
interações intermoleculares: Van der Waals e pontes de hidrogênio; relação
entre os modelos de ligação e propriedades das substâncias.

Classificação:

- Elementos químicos: princípios de organização e principais classificações
dos elementos na Tabela Periódica.

- Substâncias inorgânicas: características, nomenclatura, principais
representantes e aplicações das funções ácido, base, sal e óxido.

-Substâncias orgânicas: aspectos estruturais envolvendo cadeias carbônicas,
radicais e isomeria; características, nomenclatura, fontes de obtenção,
propriedades dos compostos mais simples e representativos das funções:
hidrocarbonetos alifáticos e aromáticos, substâncias oxigenadas: álcoois,
fenóis, éteres, aldeídos, cetonas, ácidos carboxílicos e derivados;
substâncias nitrogenadas: aminas; substâncias de importância biológica:
glicídios, lipídios, aminoácidos e proteínas.

Estados de agregação

Estudo qualitativo de sólidos, líquidos e gases; mudanças de estado;
caracterização das misturas: suspensões, dispersões coloidais e soluções;
solubilidade e classificação das soluções; expressões de concentração:
concentração comum, título, concentração em quantidade de matéria
(molaridade); propriedades coligativas.


Reações Químicas


Conceitos básicos:

- Reação química e sua representação gráfica: equação química; acerto de
coeficientes e de cargas; cálculos estequiométricos: relações ponderais e
volumétricas.

Energia nas reações químicas:

- Energia termoquímica: princípios de conservação de energia; entalpias de
reação; equação termoquímica; reações exotérmicas e endotérmicas; lei de
Hess; entropia, energia livre e espontaneidade das reações químicas.
- Energia eletroquímica: reações de oxi-redução e potenciais padrão;
obtenção de energia elétrica a partir das reações de oxi-redução: pilhas;
emprego da energia elétrica na produção de reações químicas (eletrólise);
noções de corrosão eletroquímica.

Velocidade das reações químicas - Cinética química:

- Caracterização da velocidade das reações e sua relação com as colisões
efetivas das partículas reagentes; energia de ativação; fatores que
influenciam na velocidade de uma reação química.


Reações reversíveis - Equilíbrios químicos:

- Sistemas em equilíbrio e sua caracterização; representação do equilíbrio
através de suas constantes; fatores que influenciam no equilíbrio químico de
um sistema; princípio de Le Chatelier; equilíbrio em solução envolvendo
ácidos e bases: pH.


Algumas Aplicações da Química


Compostos predominantemente inorgânicos:

- Crosta terrestre, minérios e silicatos; metais e suas ligas, siderurgia e
galvanoplastia; indústria química: fabricação de cloro e hidróxido de sódio,
ácido sulfúrico, amônia e ácido nítrico.


Compostos predominantemente orgânicos:

- Petróleo e seus derivados, polímeros; carvão e seus derivados;
combustíveis; alimentos e aditivos alimentares; compostos empregados na
limpeza.


Química do ambiente:

- A utilização do solo, da água e do ar: principais agentes poluentes.



[ ] 's do Emiliano Chemello




SUBJECT: Fw: informa��o
FROM: "Luiz Ferraz Netto" <leobarretos@uol.com.br>
TO: "ciencialist" <ciencialist@yahoogrupos.com.br>
DATE: 18/01/2005 09:12

?
===========================
Luiz Ferraz Netto [Léo]
leobarretos@uol.com.br
http://www.feiradeciencias.com.br
===========================
-----Mensagem Original-----
De: Santiago Prates Junior
Para: leobarretos@uol.com.br
Enviada em: quarta-feira, 12 de janeiro de 2005 04:32
Assunto: informação


Bom dia...
Estou tentando obter uma informação sobre um problema que passo em casa, e , vistando
seu site, embora não tenha encontrado a resposta, queria saber:
É possivel desentupir um vaso sanitario usando acido sulfurico, pois eu usei soda castica
e nada aconteceu.
Grato

SantiagoPrates junior
santiago-prates@procergs.rs.gov.br
Pr: 41566 - DGA_SDA_VIG
Telefone: 51*92285950
Turno da madrugada









--------------------------------------------------------------------------------


Internal Virus Database is out-of-date.
Checked by AVG Anti-Virus.
Version: 7.0.300 / Virus Database: 265.6.9 - Release Date: 06/01/2005

----------

No virus found in this outgoing message.
Checked by AVG Anti-Virus.
Version: 7.0.300 / Virus Database: 265.6.13 - Release Date: 16/01/2005


[As partes desta mensagem que não continham texto foram removidas]



SUBJECT: Fw: Magnetismo?
FROM: "Luiz Ferraz Netto" <leobarretos@uol.com.br>
TO: "ciencialist" <ciencialist@yahoogrupos.com.br>
DATE: 18/01/2005 09:14

alguém já colocou um ímã 'natural' dentro d'água e após 100 anos observou o resultado final?
[]'
===========================
Luiz Ferraz Netto [Léo]
leobarretos@uol.com.br
http://www.feiradeciencias.com.br
===========================
-----Mensagem Original-----
De: Jefferson S.S.
Para: leobarretos@uol.com.br
Enviada em: quarta-feira, 12 de janeiro de 2005 08:56
Assunto: Magnetismo?


Olá sou o Jefferson de Bragança Paulista Dr. queria esclarecer uma dúvida sobre o magnetismo, o imã natural dentro da água perde o seu magnetimo, ou seja, a água é algúm tipo de isolante sobre o imã ou qualque outro copo que emite o magnetismo?


--------------------------------------------------------------------------------
Natal no MSN Shopping: COMPROU, GANHOU $$! Veja Como!


--------------------------------------------------------------------------------


Internal Virus Database is out-of-date.
Checked by AVG Anti-Virus.
Version: 7.0.300 / Virus Database: 265.6.9 - Release Date: 06/01/2005

----------

No virus found in this outgoing message.
Checked by AVG Anti-Virus.
Version: 7.0.300 / Virus Database: 265.6.13 - Release Date: 16/01/2005


[As partes desta mensagem que não continham texto foram removidas]



SUBJECT: Fw: P�so espec�fico
FROM: "Luiz Ferraz Netto" <leobarretos@uol.com.br>
TO: "ciencialist" <ciencialist@yahoogrupos.com.br>
DATE: 18/01/2005 09:36

Alguém conhece o INCONEL ?
[]'
===========================
Luiz Ferraz Netto [Léo]
leobarretos@uol.com.br
http://www.feiradeciencias.com.br
===========================
-----Mensagem Original-----
De: Paulo Rocha
Para: leobarretos@uol.com.br
Enviada em: quarta-feira, 12 de janeiro de 2005 10:12
Assunto: Pêso específico


Prof. Léo

O sr. Teria como me enviar o pêso específico do material INCONEL ? Teria um tabela de pêso específico?

Muito grato

Paulo Rocha

HTA

55 12 39423422

55 12 39131070





Nas dúvidas experimentais, por gentileza coloque aqui o endereço da página, isso facilita o confronto. Agradeço. Meu nome é LUIZ FERRAZ NETTO, meu apelido é LÉO e moro em BARRETOS; dai vem meu e-mail: leobarretos@uol.com.br.



--------------------------------------------------------------------------------


Internal Virus Database is out-of-date.
Checked by AVG Anti-Virus.
Version: 7.0.300 / Virus Database: 265.6.9 - Release Date: 06/01/2005

----------

No virus found in this outgoing message.
Checked by AVG Anti-Virus.
Version: 7.0.300 / Virus Database: 265.6.13 - Release Date: 16/01/2005


[As partes desta mensagem que não continham texto foram removidas]



SUBJECT: Re: Revistas
FROM: "rmtakata" <rmtakata@altavista.net>
TO: ciencialist@yahoogrupos.com.br
DATE: 18/01/2005 09:54


--- Em ciencialist@yahoogrupos.com.br, "Rodrigo Toledo"
> Alguém sabe me informar onde posso comprar exemplares das revistas
> Nature e Science ?

Vc pode fazer a assinatura diretamente no site deles.

[]s,

Roberto Takata





SUBJECT: Re: Fw: Magnetismo?
FROM: "rmtakata" <rmtakata@altavista.net>
TO: ciencialist@yahoogrupos.com.br
DATE: 18/01/2005 09:56


Bem, rochas do fundo oceanico nao perdem sua orientacao magnetica
mesmo depois de alguns milhoes de anos.

[]s,

Roberto Takata

--- Em ciencialist@yahoogrupos.com.br, "Luiz Ferraz Netto"
> alguém já colocou um ímã 'natural' dentro d'água e após 100 anos
> observou o resultado final?
> Luiz Ferraz Netto [Léo]
> -----Mensagem Original-----
> De: Jefferson S.S.
> Olá sou o Jefferson de Bragança Paulista Dr. queria esclarecer uma
dúvida sobre o magnetismo, o imã natural dentro da água perde o seu
magnetimo, ou seja, a água é algúm tipo de isolante sobre o imã ou
qualque outro copo que emite o magnetismo?





SUBJECT: UFRGS 2005 - Resolução da prova de química
FROM: "E m i l i a n o C h e m e l l o" <chemelloe@yahoo.com.br>
TO: <naeq-ucs@yahoogrupos.com.br>, <quimica-qaw@yahoogrupos.com.br>, <quimica.pe@grupos.com.br>, <quimica@grupos.com.br>, <ciencialist@yahoogrupos.com.br>, <forum-ciencia@yahoogrupos.com.br>
DATE: 18/01/2005 11:36

Caros amigos (e amigas),

Segue abaixo o link para acesso a resolução da prova de química do
vestibular UFRGS 2005.

http://www.unificado.com.br/provas/p_ufrgs/05/qui_ufrgs05_emiliano.pdf

Aguardo comentários!

[ ] 's do Emiliano Chemello




SUBJECT: Re: Eletoimas!
FROM: "Luiz Ferraz Netto" <leobarretos@uol.com.br>
TO: "Ricardo" <ricardoryco@hotmail.com>
CC: "ciencialist" <ciencialist@yahoogrupos.com.br>
DATE: 18/01/2005 12:03

Olá Ricardo,

vamos ao seu texto:
===========================
Luiz Ferraz Netto [Léo]
leobarretos@uol.com.br
http://www.feiradeciencias.com.br
===========================
Atualmente estou mexendo com alguns projetos eletrônicos e o site de vocês foi muito útil.

Andei lendo algo sobe eletroímãs e tenho algumas dúvidas:

1) Por que o núcleo das bobinas de eletroímãs tem que ser de ferro ou ferrite e não de aço?

Léo ==> O que se pretende com um eletroímã é que ele seja "ímã" quando passa a corrente elétrica e 'deixe' de ser ímã quando essa corrente cessa. Para tanto, o núcleo deve ser feito de material de baixa remanência, ou seja, que retenha muito pouco a orientação magnética em seus domínios de Weistrass (cada vez que escrevo esse nome devo consultar a grafia --- hoje não fiz isso!). Assim, o núcleo de bons eletroímãs são feitos de ferro doce laminado ou ferrite. Se fosse feito de aço o eletroímã continuaria 'ímantado' mesmo após a interrupção da corrente.

2) Por que quando vejo algum interior de motor elétrico ou fonte o fio é enrolado em ferriti (aglomerado de chapinhas prensadas, e não um bloco interisso?

Léo ==> Nos motores universais, tanto o núcleo do estator como o do rotor é feito de lâminas de ferro doce. Essas lâminas são esmaltadas (ou oxidadas) para minimizar as correntes de Foucault (criadas pela indução eletromagnética) as quais, em lugar de circulares por toda uma massa de ferro (bloco inteiriço) circulam apenas em estreitas regiões de grande resistência elétrica.

4) No eletroímã que estou construindo preciso usar como fonte uma bateria de automóvel, como poderia usar neste eletroímã uma tensão especifica? Ex: 12v e 1 apere.

Léo ==> A tensão a ser aplicada no eletroímã, quem decide é vc. A corrente que vai circular pelo eletroímã quem decide é ele, o eletroímã. Se a tensão é contínua, basta saber a resistência ôhmica do enrolamento; se for alternada deveremos saber a impedância da bobina.
Se vc quiser construir uma fonte para seu eletroímã, para UMA DETERMINADA CORRENTE, primeiro ligue esse eletroímã numa fonte regulável e verifique SOB QUE TENSÃO o eletroímã é percorrida POR AQUELA DETERMINADA CORRENTE; depois construa uma fonte PARA FORNECER ESSA TENSÃO FIXA.

Aquele abraço,

Léo

----------

No virus found in this outgoing message.
Checked by AVG Anti-Virus.
Version: 7.0.300 / Virus Database: 265.6.13 - Release Date: 16/01/2005


[As partes desta mensagem que não continham texto foram removidas]



SUBJECT: Re: [ciencialist] UFRGS 2005 - Resolução da prova de química
FROM: "Prof. JC" <profjc2003@yahoo.com.br>
TO: <ciencialist@yahoogrupos.com.br>
DATE: 18/01/2005 12:26

Chemello,

Você está fazendo um ótimo trabalho. Principalmente pela iniciativa de
disponibilizar as resoluções. Parabéns.

Abraços,
Prof. JC


----- Original Message -----
From: "E m i l i a n o C h e m e l l o" <chemelloe@yahoo.com.br>
To: <naeq-ucs@yahoogrupos.com.br>; <quimica-qaw@yahoogrupos.com.br>;
<quimica.pe@grupos.com.br>; <quimica@grupos.com.br>;
<ciencialist@yahoogrupos.com.br>; <forum-ciencia@yahoogrupos.com.br>
Sent: Tuesday, January 18, 2005 11:36 AM
Subject: [ciencialist] UFRGS 2005 - Resolução da prova de química


>
> Caros amigos (e amigas),
>
> Segue abaixo o link para acesso a resolução da prova de química do
> vestibular UFRGS 2005.
>
> http://www.unificado.com.br/provas/p_ufrgs/05/qui_ufrgs05_emiliano.pdf
>
> Aguardo comentários!
>
> [ ] 's do Emiliano Chemello
>
>
>
>
> ##### ##### #####
>
> Para saber mais visite
> http://www.ciencialist.hpg.ig.com.br
>
>
> ##### ##### ##### #####
> Links do Yahoo! Grupos
>
>
>
>
>
>
>
>
>





SUBJECT: Re: Martelo Pneumático
FROM: "rmtakata" <rmtakata@altavista.net>
TO: ciencialist@yahoogrupos.com.br
DATE: 18/01/2005 13:31


--- Em ciencialist@yahoogrupos.com.br, "Sandro" <ssilva@p...> escreveu
> Alguém poderia me informar ou citar algum site sobre a possibilidade
> de um prego destes martelos pneumáticos para carpintaria/marcenaria
> passar pelo crânio humano. O impacto (penetração no tecido
> muscular/ósseo) dos pregos seria equivalente aos projéteis de uma
> pistola de mão?

Pneumatic Nailer Injuries: A Report on Washington State 1990-1998
http://www.cdc.gov/elcosh/docs/d0400/d000436/d000436.html

Unusual craniocerebral injury caused by a pneumatic nail gun.
http://www.ncbi.nlm.nih.gov/entrez/query.fcgi?cmd=Retrieve&db=PubMed&list_uids=12376837&dopt=Abstract

Se atravessa madeira, nao eh espantoso q. atravesse o cranio.

[]s,

Roberto Takata





SUBJECT: Você é hands on?
FROM: Rodrigo Marques <rodmarq72@yahoo.com.br>
TO: Ceticismo Aberto <ceticismoaberto@yahoogrupos.com.br>, CienciaList <ciencialist@yahoogrupos.com.br>, Sociedade Brasileira de "C�ticos" e Racionalistas <sbcr@yahoogrupos.com.br>, Sociedade da Terra redonda <strbrasil@yahoogrupos.com.br>
DATE: 18/01/2005 13:56

Gostei muito deste texto.

VOCÊ É HANDS ON?

Max Gehringer (Colunista Revista EXAME)



Vi um anúncio de emprego. A vaga era de gestor de atendimento interno,
nome

que agora se dá à seção de serviços gerais. E a empresa contratante
exigia

que os eventuais interessados possuíssem - sem contar a formação
superior,

liderança, criatividade, energia, ambição, conhecimentos de
informática,

fluência em inglês e não bastasse tudo isso, ainda fossem hands on.

Para o felizardo que conseguisse convencer o entrevistador de que
possuía

mesmo essa variada gama de habilidades, o salário era um assombro: 800

reais.



Ou seja, um pitico.



Não que esse fosse algum exemplo absolutamente fora da realidade.



Pelo contrário, ele é quase o paradigma dos anúncios de emprego atuais.
A

abundância de candidatos está permitindo que as empresas levantem, cada
vez

mais, a altura da barra que o postulante terá de saltar para ser
admitido.



E muitos, de fato, saltam. E se empolgam. E aí vêm as agruras da

superqualificação, que é uma espécie do lado avesso do efeito pitico...

Vamos supor que, após uma duríssima competição com outros candidatos
tão bem

preparados quanto ela, a Fabiana conseguisse ser admitida como gestora
de

atendimento interno.. E um de seus primeiros clientes fosse o seu
Borges,

gerente da contabilidade.



- Fabiana, eu quero três cópias deste relatório.

- In a hurry!

- Saúde.

- Não, isso quer dizer "bem rapidinho". É que eu tenho fluência em
inglês.

Aliás, desculpe perguntar, mas por que a empresa exige fluência em
inglês

se aqui só se fala português?

- E eu sei lá? Dá para você tirar logo as cópias?

- O senhor não prefere que eu digitalize o relatório? Porque eu
tenho

profundos conhecimentos de informática.

- Não, não.. Cópias normais mesmo.

- Certo. Mas eu não poderia deixar de mencionar minha criatividade.
Eu já

comecei a desenvolver um projeto pessoal visando eliminar 30% das
cópias

que tiramos.

- Fabiana, desse jeito não vai dar!

- E eu não sei? Preciso urgentemente de uma auxiliar.

- Como assim?

- É que eu sou líder, e não tenho ninguém para liderar. E considero
isso

um desperdício do meu potencial energético.

- Olha, neste momento, eu só preciso das três có...

- Com certeza. Mas antes vamos discutir meu futuro...

- Futuro? Que futuro?

- É que eu sou ambiciosa. Já faz dois dias que eu estou aqui e ainda
não

aconteceu nada.

- Fabiana, eu estou aqui há 18 anos e também não me aconteceu nada!

- Sei. Mas o senhor é hands on?

- Hã?

- Hands on. Mão na massa.

- Claro que sou!

- Então o senhor mesmo tira as cópias. E agora com licença que eu
vou sair

por aí explorando minhas potencialidades. Foi o que me prometeram
quando eu

fui contratada.



Então, o mercado de trabalho está ficando dividido em duas facções:

1 - Uma, cada vez maior, é a dos que não conseguem boas vagas porque
não têm

as qualificações requeridas.

2 - E o outro grupo, pequeno, mas crescente, é o dos que são admitidos

porque possuem todas as

competências exigidas nos anúncios, mas não poderão usar nem metade
delas,

porque, no fundo, a função não precisava

delas.



Alguém ponderará - com justa razão - que a empresa está de olho no
longo

prazo: sendo portador de tantos talentos, o funcionário poderá ir sendo

preparado para assumir responsabilidades cada vez maiores.



Em uma empresa em que trabalhei, nós caímos nessa armadilha. Admitimos
um

montão de gente superqualificada. E as conversas ficaram de tão alto
nível

que um visitante desavisado que chegasse de repente confundiria nossa

salinha do café com o auditório da Fundação Alfred Nobel.



Pessoas superqualificadas não resolvem simples problemas!



Um dia um grupo de marketing e finanças foi visitar uma de nossas
fábricas e

no meio da estrada, a van da empresa pifou.



Como isso foi antes do advento do milagre do celular, o jeito era
confiar

no especialista, o Cleto, motorista da van. E aí todos descobriram que
o

Cleto falava inglês, tinha noções de informática e possuía energia e

criatividade. Sem mencionar que estava fazendo pós-graduação. Só que
não

sabia nem abrir o capô.



Duas horas depois, quando o pessoal ainda estava tentando destrinchar o

manual do proprietário, passou um sujeito de bicicleta. Para horror de

todos, ele falava "nóis vai" e coisas do gênero. Mas, em 2 minutos,
para

espanto geral, botou a van para funcionar.



Deram-lhe uns trocados, e ele foi embora feliz da vida. Aquele
ciclista

anônimo era o protótipo do funcionário para quem as empresas modernas
torcem

o nariz:

O que é capaz de resolver, mas não de impressionar.

__________________________________________________
Converse com seus amigos em tempo real com o Yahoo! Messenger
http://br.download.yahoo.com/messenger/

[As partes desta mensagem que não continham texto foram removidas]



SUBJECT: Martelo Pneumático
FROM: "Sandro" <ssilva@pmc.curitiba.pr.gov.br>
TO: <ciencialist@yahoogrupos.com.br>
DATE: 18/01/2005 14:18

Olá Pessoal,

Alguém poderia me informar ou citar algum site sobre a possibilidade de um
prego destes martelos pneumáticos para carpintaria/marcenaria passar pelo
crânio humano. O impacto (penetração no tecido muscular/ósseo) dos pregos
seria equivalente aos projéteis de uma pistola de mão?

obrigado por qualquer informação,

Sandro Silva







SUBJECT: Re: [ciencialist] Re: onde estava o passado...?
FROM: "Eurico Ferreira de Souza Jr." <caodejah@yahoo.com.br>
TO: ciencialist@yahoogrupos.com.br
DATE: 18/01/2005 18:32



marcelo ferrari <emailferrari@yahoo.com.br> wrote:(...)

[F4]> Quer dizer que antes de eu te enviar esta minha resposta aqui ela já estava enviada, pois já estava no agora? É isto?
[E4]> não. antes ela não existia.





_\|/_
__________________________________________________
Converse com seus amigos em tempo real com o Yahoo! Messenger
http://br.download.yahoo.com/messenger/

[As partes desta mensagem que não continham texto foram removidas]



SUBJECT: Convite para download de arquivo
FROM: "Luis" <luis.alcides@he.com.br>
TO: ciencialist@yahoogrupos.com.br
DATE: 18/01/2005 20:53


Ilustres colegas, gostaria de convidar a todos para fazer o
download da terceira edição do Periódico Tchê Química, uma revista
eletronica de divulgação científica.

Na edição n. 3 do Periódico Tchê Química temos os seguintes
destaques:
* Dois artigos da UFRN;
* Um artigo da Unversidade de Coimbra, Portugal;
* Uma revisão sobre um modelo atômico postulado por um brasileiro!;
* Entrevista com Benoit B. Mandelbrot, sobre a Teoria dos Fractais,
inventada por ele.

Endereço para download:
http://www.deboni.he.com.br/tq/clics/clic.php3?url=www.deboni.he.com.br/revista3\
p.pdf

Outras opções de download estão disponiveis no site:
http://www.tchequimica.com

Grato pela atenção, Luis A. De Boni.







SUBJECT: ói qui bão prá nóis
FROM: "murilo filo" <avalanchedrive@hotmail.com>
TO: ciencialist@yahoogrupos.com.br, forum-ciencia@yahoogrupos.com.br
DATE: 18/01/2005 22:17


Nem sei bem se é off-topic; só sei que não deveria ser!
Alguém quer comentar o fato de o governo ter retirado o inglês das provas
para o Itamaratí?
Será que não há ninguém impressionado com a possibilidade de virmos a ter
DIPLOMATAS que não possam se comunicar com o resto do MUNDO?
A lamentável má comunicação doméstica, oral e escrita, será globalizada?
Não será o caso de logo termos vestibulares p/engenharia sem matemática +
física e medicina sem biologia?
Vcs não estão vendo que a ignorância vai imperar?
Vcs não estão vendo que estamos para chegar ao ponto em que logo teremos um
homem bomba brasileiro?
Agora aquela pergunta: QUEM E QUANTOS GANHARÃO COM ISTO? SERÁ QUE VALE?
obr/abr Murilo SP 17/jan/2005




SUBJECT:
FROM: "murilo filo" <avalanchedrive@hotmail.com>
TO: ciencialist@yahoogrupos.com.br, forum-ciencia@yahoogrupos.com.br
DATE: 18/01/2005 22:32

Para obter ajuda visite http://help.yahoo.com/help/br/groups/


From: murilo filo <avalanchedrive@hotmail.com>
To: ciencialist@yahoogrupos.com.br, forum-ciencia@yahoogrupos.com.br
Subject: ói qui bão prá nóis
Sent: segunda-feira, 17 de janeiro de 2005 22:21:58
Nem sei bem se é off-topic; só sei que não deveria ser!
Alguém quer comentar o fato de o governo ter retirado o inglês das provas
para o Itamaratí?
Será que não há ninguém impressionado com a possibilidade de virmos a ter
DIPLOMATAS que não possam se comunicar com o resto do MUNDO?
A lamentável má comunicação doméstica, oral e escrita, será globalizada?
Não será o caso de logo termos vestibulares p/engenharia sem matemática +
física e medicina sem biologia?
Vcs não estão vendo que a ignorância vai imperar?
Vcs não estão vendo que estamos para chegar ao ponto em que logo teremos um
homem bomba brasileiro?
Agora aquela pergunta: QUEM E QUANTOS GANHARÃO COM ISTO? SERÁ QUE VALE?
obr/abr Murilo SP 17/jan/2005




SUBJECT: Aproximacao com o sol...
FROM: "Rick" <rickardorios@yahoo.com.br>
TO: "l - Ciencia" <ciencialist@yahoogrupos.com.br>
DATE: 19/01/2005 00:05

Pessoal, eu li em uma questao de um livro que quando um planeta qualquer
está na região do periélio não é verão. Por que não? A proximidade com o sol
nao resultaria em dias mais quentes e consequentemente na estação mais
quente?...

Agradecendo a ajuda

Rick




SUBJECT: Re: ói qui bão prá nóis
FROM: Maria Natália <grasdic@hotmail.com>
TO: ciencialist@yahoogrupos.com.br
DATE: 19/01/2005 05:15


Murilo:

Mas o governo pode ter retirado o inglês apenas este ano porque há
filho de pessoa importante a querer entrar nesse curso só sabendo
francês e se não fosse assim moçinho ficava à porta.
Mas também poderemos pensar que o governo sabe que alunos sabem
imenso inglês* mal terminam o médio...Para quê estar a exigir mais
uma prova?
Por outro lado se pode ser médico fazendo tele medicina...Aplicam-se
umas pontas de prova na barriga do paciente e enquanto este fica a
falar pro computador o Informático, digo médico, vai fumar para o
corredor ver passar enfermeirinha***.
Portanto o rei vai nu.
Podem os ilustres professores universitários que vagueiam por aqui
dirigir documento escrito ao boss da educação perguntando o porquê
de tal medida. Professor por estar a lidar com crianças pode estar
estúpido e agradece ajuda de ministro.
E pensando bem quem precisa do inglês se se fala BB e Orkutiano
cybernético."Mi dá teu niver, Olha namo, naum ias pra k ladi" = sons
de tita.bjus tahi, Nim tou aiiii. é foda."
Bem haja o senhor ministro cabeça iluminada desse governo**
saudações e votos de melhor ensino
Maria Natália
*Não é que 90% dos brasileiros acede à net!? E como tal já sabem
inglês de site pornô
** calma aqui tb há essa de ir para engenharia sem ser obrigatório
física. Pode ter só filosofia, história e latim que entra na mesma.
Aliás ouvi dizer que a filosofia é a mãe (pai?) de todas as ciências
logo quem tem lic em filo sabe tudo.
*** Essa sim tem curso puxado e até estudou biologia.



--- Em ciencialist@yahoogrupos.com.br, "murilo filo"
<avalanchedrive@h...> escreveu
>
> Nem sei bem se é off-topic; só sei que não deveria ser!
> Alguém quer comentar o fato de o governo ter retirado o inglês das
provas
> para o Itamaratí?
> Será que não há ninguém impressionado com a possibilidade de
virmos a ter
> DIPLOMATAS que não possam se comunicar com o resto do MUNDO?
> A lamentável má comunicação doméstica, oral e escrita, será
globalizada?
> Não será o caso de logo termos vestibulares p/engenharia sem
matemática +
> física e medicina sem biologia?
> Vcs não estão vendo que a ignorância vai imperar?
> Vcs não estão vendo que estamos para chegar ao ponto em que logo
teremos um
> homem bomba brasileiro?
> Agora aquela pergunta: QUEM E QUANTOS GANHARÃO COM ISTO? SERÁ QUE
VALE?
> obr/abr Murilo SP 17/jan/2005





SUBJECT: Falando do olho
FROM: Maria Natália <grasdic@hotmail.com>
TO: ciencialist@yahoogrupos.com.br
DATE: 19/01/2005 05:21


Já lá diz meu amigo oftalmologista-chefe no Hospital de Santa Maria
em Lx, a vista não existe existe o olho.
Há uma esperança para pessoas com problemas na retina e que nos
chega da Universidade de Bristol:
REMODELLING THE EYE - LATEST DEVELOPMENTS IN PREVENTING BLINDNESS,
January 18
At the Bristol Eye Hospital, in the South West of England,
researchers
from the University of Bristol have made an exciting discovery that
could result in 'remodelling' cells in the eye. This could have a
major
impact on eye damage that results from disease of the retina.
Full story at http://www.physorg.com/news2725.html"
sds
Maria Natália






SUBJECT: neutrino traveis
FROM: "rayfisica" <rayfisica@yahoo.com.br>
TO: ciencialist@yahoogrupos.com.br
DATE: 19/01/2005 08:08


Senhores, posso até ser um chato, porem minhas questões são
de quem
realmente quer aprender, ainda que de um jeito não passivo, pois
se
assim fora eu iria ser um padre e ai eu tentaria aceitar a
evolução
com a visão bíblica.
Quando gralho, gralho por que penso e se penso, logo existo.
(ninguém
pode tirar isso de mim)

Mas falando de assunto serio:

No dia 23 de Fevereiro de 1987, em um curto intervalo de poucos
minutos, o Kamiokande acusou a chegada de uma grande quantidade de
neutrinos. Para dizer a verdade, essa "grande quantidade" foi de
apenas 12 neutrinos. Mas, para quem está acostumado a contar menos
de
um neutrino por dia, isso é uma tremenda tempestadade. Logo se
descobriu que esses neutrinos vinham de uma supernova que explodira
na galáxia vizinha, a 170.000 anos-luz da Terra. Algumas horas
depois
dessa "chuva" de neutrinos atingir o Kamiokande, a luz da supernova
foi observada em um telescópio no Chile. Portanto, esses
neutrinos,
os primeiros neutrinos extra-galáticos a serem detetados, sairam
da
supernova antes da intensa luz da explosão ser jogada no espaço.
Como interpretar isso, pois:

Tendo o neutrino massa, logo a sua velocidade no vácuo
interestrelar,
por ser menor que a velocidade da luz...

Com a saída dos neutrinos das supernovas o colapso é
instantâneo ou
não?

Os fótons não deveriam ultrapassar os neutrinos no caminho
até a
terra, chegando muito tempo antes, afinal 170000 anos luz?






SUBJECT: Re: Aproximacao com o sol...
FROM: "rmtakata" <rmtakata@altavista.net>
TO: ciencialist@yahoogrupos.com.br
DATE: 19/01/2005 08:08


--- Em ciencialist@yahoogrupos.com.br, "Rick" <rickardorios@y...>
> Pessoal, eu li em uma questao de um livro que quando um planeta
> qualquer está na região do periélio não é verão. Por que não? A
> proximidade com o sol nao resultaria em dias mais quentes e
> consequentemente na estação mais quente?...

No caso da Terra em q. a orbita eh quase circular, praticamente nao
existe diferenca de distancia entre o perielio e o afelio, o q.
importa eh a inclinacao do eixo de rotacao da Terra - tto eh q. qdo eh
verao em um hemisferio eh primavera em outro.

Mais ainda, verao e inverno, embora tradicionalmente se liguem a
temperaturas medias mais altas ou mais baixas (aqui na Terra),
astronomicamente se refere a dias com periodos de luminosidade mais
longos ou mais curtos. Na Terra calha de coincidir q. periodos de luz
mais longos significa dias mais quentes (jah q. a intensidade luminosa
pouco varia ou ateh aumenta).

[]s,

Roberto Takata





SUBJECT: Re: Por que massa atrai massa?
FROM: "Sergio M. M. Taborda" <sergiotaborda@yahoo.com.br>
TO: ciencialist@yahoogrupos.com.br
DATE: 19/01/2005 08:41


--- Em ciencialist@yahoogrupos.com.br, marcelo ferrari
<emailferrari@y...> escreveu
> SERGIO > Quando não há vento, não ha ar? As particulas de luz não
desaparecem pq não as vemos (no sentido lato) se moverem.
>
> 1) Ar em movimento é vento. Foton em movimento é luz? É esta a analogia?

Sim.

> 2) Ar é feito de quimica. Foton é feito de que?

Pq ele tem que ser feito de alguma coisa ?

> 3) O que poe o ar em movimento pra surgir o vento? O que poe os
fotons em movimento para surgir a luz?

--- "Alvaro Augusto escreveu

> A comparação entre vento e luz é válida só até certo ponto. É fácil
determinar a causa do vento, que são as diferenças de pressão
atmosférica. Mas os fótons se movem "empurrados" por quem? Flutuações
quânticas do vácuo?

Os fotões se movem empurrados pelo campo electromagnético.

>Por que o resultado líquido (a luz) se move sempre à mesma velocidade?

Nem sempre se move à mesma velocidade. Por isso que existem fenomenos
como a refleção e a refracção. A velocidade da luz é a mesma para um
dado meio, mas varia de meio para meio. Isto pq, de meio para meio,
variam as propriedades electromanneticas. Pense que cada meio tem uma
resitencia electrica e uma resistencia magnética diferentes , que
interferem com os campos que estão empurrando os fotões ,
enfraqucendo-os. É como um vento que tenta se mover por entre as
folhas das arvores, não tem a mesma velocidade do que se se estiver se
movendo "livre".


> SERGIO > Um fotão parado não é visivel, luz parada não visivel, tal
como vento parado não é visivel.
>
> Porque, um fotom parado não é visivel? O que isto significa?

--- "rayfisica" escreveu

> É possível no nosso universo que um fóton fique com movimento zero?
>
> É possível algo no nosso universo com movimento zero?
>
> Nosso universo = acontecimento que já aconteceu e foi observado
> (ainda que não visto)
>

Quanto a mim, a essencia do entendimento desta questão está
exactamente em responder a estas perguntas.

Primeiro, o universo não é nosso. Ou seja, não temos sobre ele nenhuma
influencia, ele que tem sobre nós.
Segundo, aquilo que vemos do universo é apenas aquilo que PODEMOS ver
do universo, o que nos limita bastante. Aquilo que vemos podem ser
manifestações de acontecimentos que estamos limitados a não poder
observar directamente. Ou seja, existe um universo muito maior que
fenomenos do que aqueles que podemos observar. Alguns desses fenomenos
sabemos que existem porque geram fenomenos observaveis.

Um fotão é uma particula cuja energia é puramente cinética. Isto
significa que ele só tem energia quando em movimento. Ele só transmite
energia quando em movimento. P
A termodinamica mostra que todo e qq sistema que mude de estado é
caracterizado por uma energia interna que varia. Os estados do fotão
são parado e em movimento. Sendo que esse movimento só pode acontecer
a uma certa velocidade , que depende do meio em que o fotão se
desloca. Mas, o fotão sempre viaja à máxima velocidade que lhe é
permitida pelo meio onde viaja - tal como a velocidade máxima de um
carro não depende do seu motor, mas do limite de velocidade máxima
imposto pela estrada onde se desloca. O carro pode acelerar e travar
conforme o limite aumenta e diminui, mas no fim, a sua velocidade vai
ser directamente porporcional ao limite imposto pela estrada e não
pelo motor. Pelo menos até um certo limite.

Imagine-se um laser. Um feixe puro (uma so cor) de luz. Embora nos
filmes vc veja um laser de lado, se deslocando, na realidade isso é
mentira. Todos os lasers são invisiveis quando olhados de lado. A
unica forma de saber a cor da luz é olhando de frente para o laser (o
que é perigoso), ou fazendo-o refletir em algum objecto. É por isso
que vemos a luz do sol, pq estamoa na sua frente, mas não vemos os
raios de sol que vão para a lua , pq passam ao lado da terra. Só os
vemos quando eles refletem na lua.

Assim, entendam que, mesmo um fotão em movimento pode não ser
observável e só vemos fotões quando eles batem nos nosso olhos . Em
geral, observamos luz quando os fotões chegam aos sensores de luz.
Então , é possivel que o fotão fique parado ? Sim. Basta que ele perca
toda a sua energia. E como toda a sua energia é cinética, ele fica
parado quando perde toda a sua energia. Mas quando ele perde toda a
energia e fica parado ele não mais pode vir ter com os nosso olhos,
pelo que não mais o veremos. Ele existe, parado, mas não é observável.
por esse mesmo facto.

Imagine que o espaço está preenhido por estas particulas minusculas de
massa nula que apenas são actuadas por forças electromaneticas
dispostas de certa forma. Todos o espaço está cheio destas particulas
como uma manta cheia de particulas de pó. O pó sozinho não se move a
menos que algo coisa o faça mover-se e não qq coisa, mas uma força em
especial. Não basta que haja vento, ele tem que ter uma orientação
especial para arranacar as particulas de po de onde estão. Uma vez
actuadas pelo vento elas se movem, mas eles não se movem pq querem, e
sim pq estão sendo empurradas. Os fotões não se movem sozinhos,
movem-se pq estão sendo arrastados pelo campo e.m.
Sem campo e.m. eles permanecem nos seus lugares sem se mover, e
portanto, não são observaveis.

> o que seria vento parado ?

Não seria vento.


Porque massa atrai massa ?

Não existe ainda uma resposta para isso. Sabemos como a massa atrai a
massa .. melhor, não sabemos como, sabemos quanto. Sabemos quanto uma
massa A atrai um massa B. Sabemos quantificar a sua atracção, mas não
conhecemos o mecanismo que a causa.
Chamamos Gravidade a esse mecanismo, mas não sabemos o que é.
Qualquer resposta a esta pergunta é mera especulação.

Quanto às simetrias do universo / natureza , de facto desempenham um
papel preponderante em toda a fisica. Isso é simplesmente ignorado nas
escolas. E é levemente referido nas faculdades.
As Leis de Conservação da Energia e do Momento, são o resultado do
facto do espaço (momento) ser simetrico , ou seja, qq ponto é
indestinguivel de qq outro ponto. E qq instance é igual a qq outro.
Básicamente isto significa que o espaço e o tempo, por si, não têm
quaisquer propriedades.
Ora, isso deixa de ser verdade quando alguma coisa torna os pontos do
espaço destinguiveis. Por exemplo, um planeta rodando sobre o seu
eixo. Existem pontos do espaço que são diferentes do outros. São
aqueles que estão no eixo. Mas qq outro ponto diferente desses,
continua sendo simetrico relativamente a esse eixo. Este
"relativamente" é que causa toda a diferença, e toda a fisica.

A presença de campos torna os pontos destinguiveis e portanto quebra a
simetria do espaço e/ou do tempo. Mas mesmo assim existem
sub-simetrias possiveis. Dentro de um cristal os pontos são
destinguveis pq existem pontos de referencia (o centros cristalinos).
mas um ponto a x distancia de um centro cristalino do tipo A é igual a
um outro ponto à mesma distancia de outro centro do mesmo tipo que
está do outro lado do cristal. Isso cria uma sub-simetria.
Qualquer sub-simetria pode ser representada por um Periodo no espaço e
/ou no tempo. À muitos nomes para estes periodos conforme o caso.

A luz também é uma sub-simetria. O comprimento de onda e a frequencia
cria simetria entre pontos distantes do espaço

A gravidade também é uma sub-simetria, mas está é bem mais complexa de
tratar pq não ha nenhum periodo aparente. Ela é uma sub-simetria com
em grau diferente.

Toda a ideia da Teoria das Cordas é entender as coisas baseado em
simetrias. E as cordas são só representações matemáticas da simetria.

A pergunta que se levanta hoje, não é mais se massa atrai massa, mas
pq massa atrai energia.
Eisntein provou que a luz é atraida pela gravidade. Mas provou também
que ela não tem massa. Então pq ela continua sendo atraida. A
relatividade resolve isto dizendo que : A luz não tem massa, mas
quando olhamos para ela, ela parece ter.
Então é como se a gravidade fosse um vortice que fica testando tudo o
que entra no seu alcance e suga impiedosamente tudo o que tiver massa.
A luz passa por perto , pensando que, por não ter massa, irá passa
desapercebida.
Mas do ponto de vista da gravidade , ela vê a luz com uma massa e
portanto puxa-a. A luz diz : Não! , Não!, eu não tenho massa, estás
cometendo um engano! E a gravidade responde: Não mintas, eu estou
VENDO que tens massa. Ao que a luz responde: mas não é minha !
Ao que a gravidade responde: isso é o que dizem todos.

Segundo a relatividade ambos têm razão. Do ponto de vista da gravidade
tudo tem massa, e do ponto de vista da luz ela não tem massa.

Mas luz e fotões são a mesma coisa ? E a gravidade atrai qual ?

Tb segundo Eisntein, luz e fotões não são a mesma coisa. fotões são
particulas, luz é um campo e.m. que actua sobre os fotoes.
Se pensarmos que ha fotões parados em todo o lado, se a gravidade
actuasse sobre eles teria-mos correntes de fotões entrando para os
planetas todo o tempo. Ora, se o fotão se move, ele ganha energia, e
por isso os planetas estariam ganhando energia todo o tempo. O sol
estaria ganhando energia e não perdendo (a menos que expulse mais
fotoes do que aqueles que absorve.) todo o corpo com massa absorveria
fotoes , causando que existisse uma corrente de fotoes vinda do
espaço, mas que não fora enviada por ninguem. O que acabaria sugando
todos os fotoes do universo - supondo que eles não se criam, nem se
destroem. Tanto quanto sabemos isso não existe. Então os fotoes não
são actuados pela gravidade e portanto, a sua massa é nula (pensando
que a gravidade é causada pela existencia de uma massa não nula). Por
outro lado, se os fotoes tivessem massa, eles proprios criariam
gravidade atraindo outros fotoes. O que sabemos irira criar uma rede
de fotoes girando em torno un dos outros (tal como os planetas e suas
luas) e não feixes.

Então e a luz ?
A luz coloca os fotoes em movimento. Esse movimento transporta energia
e momento. Uma vez em movimento, a gravidade não os destingue de um
corpo com massa e por isso os atrai. Mas a gravidade não manipula o
campo e.m. em si mesmo nem actua sobre fotoes parados.
A analogia seria uma estada com carros passando todos no mesmo
sentido. E algures no meio uma esfera gigante. A estrada continua reta.
Agora pensemos que é de noite e que apenas os carros são visiveis pq
são iluminados. Nõs não veriamos a esfera, nem a estrada. As linhas
retas de carros curvariam em algum ponto, mas voltando ao normal logo
depois (como naquelas fotos de cidades à noite onde não se ve~em
carros mas linhas de luz). Acendendo a esfera, veriamos que esse ponto
é a esfera. Os carros afastan-se da esfera, mas a estrada não os deixa
afastar muito, fazendo com que eles retornem à pista e siguam caminho
normalmente. Se a esfera fosse magnetica ela atriria os carros e o
efeito seria ao contrario.

Isto é o efeito de lente gravitacional. A gravidade altera a rota dos
fotoes mas não altera o campo que os empurra. (o campo e.m. pode se
sobrepor ao gravitico sem que um interfira com o outro. São
independentes)


A gravidade é a ultima força, a ultima fronteira. Quando soubemos
dominá-la não haverá mais segredos e saberemos pq massa atrai massa.


Sérgio Taborda






SUBJECT: Re: Unidades de medida: símbolo e nome da grandeza
FROM: "Sergio M. M. Taborda" <sergiotaborda@yahoo.com.br>
TO: ciencialist@yahoogrupos.com.br
DATE: 19/01/2005 09:13


--- Em ciencialist@yahoogrupos.com.br, "E m i l i a n o C h e m e l
l o" <chemelloe@y...> escreveu
> Olá pessoal,
>
> Tá rolando no orkut uma discussão sobre unidades de medida. Resolvi
> trazê-la para cá. As questões são as seguinte:
>
> Um dos participantes escreveu: " ... Já vi autores defenderem
moles como
> no Inglês. Nós brasileiros escrevemos mols. Como quem manda aqui
somos nós..
> levando em consideração a sonoridade da língua, como vocês acham que
devem
> ser? "

O Qual é o plural de uma palavra que termina em l ?

mil = mis
formol = formois.

mol = mois.

Supondo que o nome da unidade , em portugues, é mol - o que não é -
obtemos o bonito plural : mois.
Mas como no nome da unidade é mole, o plural é o bonito moles.
Ah é tão dificil chegar nesta conclusão ...

> Além disso, proponho a discussão sobre outro aspecto: Quando
utilizamos
> o símbolo 'mol', da mesma forma que utilizamos 'g' para se referir a
massa ,
> não colocamos o símbolo da grandeza no plural. Exemplo: 2 g e *não*
2 gs; 2
> mol / L e *não* 2 mols / L Já quando estamos escrevendo o nome da
grandeza,
> ai sim podemos utilizar o plural: Ex: 2 gramas de cloreto de sódio;
2 mols
> de bicarbonato de sódio. Segundo o inmetro, eu estou correto:
>
> http://www.inmetro.gov.br/consumidor/unidLegaisMed.asp
>
> A grande confusão se dá porque o símbolo e a unidade da grandeza
são o
> mesmo: 'mol'.

Não. Isso é o mesmo que dizer que voce e o seu nome são a mesma coisa.
Não, o seu nome é uma representação de vc.
O simbolo é uma representação da unidade de grandeza, que por sua vez
é abstracta.
A solução para o seu problema é um pouco de matemática e portugues
basicos. O que é uma Unidade ? Por definição é algo que não se pode
separar em partes. Por exemplo a unidade matemática é um 1. o 1 é ele
mesmo e não pode ser separado em partes. Podemos encontrar numeros que
representariam a separação do 1 em partes, como 1/2 ou 1/6 , mas esses
numeros são a representação das partes e não as partes em si - pois
elas não existem.
Se eu contar laranjas a minha unidade de medida é laranja (singular)
Eu não posso dividir a minha unidade pq ela não existe. É um conceito
abstracto. Eu posso criar outras abstrações que seriam partes da
laranja - como gomo - mas eu estaria criando novas unidades e não
dividindo a unidade que tenho.
O que é medir ? É atribuir a uma grnadeza um multiplo da unidade de
medida. Um multiplo. Então 2 g significa 2 x grama e não que eu tenho
duas unidades grama (uma mais gorda e outra mais magra).
Eu tenho 3 laranja no meu cesto não é portugues correcto.
Mas: eu tenho 3 lr em meu cesto é. Note que subtituimos o nome
"laranja" pelo simbolo lr. Simbolos não têm plural , da mesma forma
que abreviações não tem plural. Às vezes forçamos isso, como em
"ovnis", mas isso é um neologismo. (que para quem não sabe é um metodo
que cria palavras em uma lingua )
Não faz sentido falar em ONUs , mas em "ovnis" sim. Ovni não é mais so
a abreviação, mas um neologismo que significa : nave espacial -
pertensamente - de outro mundo. Objecto voador não identificado é
apenas uma das acepções.

Mas voltando. Dizemos 2 g e pronunciamos (dois gramas) pq simbolos
não têm plural. Mas ao escrever escrevemos dois gramas, pq a lingua
portuguesa assim o obriga.
E lembrar que a unidade grama é do genero masculino.
Um grama = a quantidade de massa de alguma coisa
Uma grama = uma planta.

Para terminar,
2 mol / L = dois gramas por litro.
2 gs = dois grama segundo

Sérgio Taborda





SUBJECT: Re: Aproximacao com o sol...
FROM: "Sergio M. M. Taborda" <sergiotaborda@yahoo.com.br>
TO: ciencialist@yahoogrupos.com.br
DATE: 19/01/2005 09:32


--- Em ciencialist@yahoogrupos.com.br, "Rick" <rickardorios@y...> escreveu
> Pessoal, eu li em uma questao de um livro que quando um planeta qualquer
> está na região do periélio não é verão. Por que não? A proximidade
com o sol
> nao resultaria em dias mais quentes e consequentemente na estação mais
> quente?...

Não, não resultaria. Esse é o maior Hoax (engano) de todos os tempos e
o pior é que é ensinado nas escolas! Que VERGONHA.

A verdade é que as estações do ano se devem à inclinação do eixo de
rotação do planeta em relação ao sol e ao facto de qualquer que seja a
inclinação, ela ser constante ao logo da orbita.
Se o eixo não fosse inclinado não haveria estações , e o clima no
hemisferio norte seria igual ao so hemisferio sul.

As estações têm a ver com o menor ou maior aquecimento, mas não devido
À proximidade e sim à energia transportada pelos raios. A atmosfera
reflete os raios conforme o angulo em que eles incidem, que por sua
vez, depende da inclinação do eixo. Confome o planeta se move, o
mesmos raios paralelos do sol vão sendo refletidos de forma diferente
e por isso aquecendo de forma diferente.


Sérgio Taborda






SUBJECT: Re: Aproximacao com o sol...
FROM: "rmtakata" <rmtakata@altavista.net>
TO: ciencialist@yahoogrupos.com.br
DATE: 19/01/2005 09:52


--- Em ciencialist@yahoogrupos.com.br, "Sergio M. M. Taborda"
> Não, não resultaria. Esse é o maior Hoax (engano) de todos os tempos

Nao sei se eh o maior engano presente em livros didaticos. Existiram
outros tao ou mais cabeludos - parece q. com o sistema de avaliacao
dos livros a serem comprados pelo sistema publico, tais erros
dimiuiram aqui no Brasil.

Falava-se q. plantas no quarto de domir 'a noite eram perigosas - pois
elas respirariam sem fazer a fotossintese e poderiam sufocar a pessoa
se o quarto nao fosse bem ventilado. (Como se compartilhar o quarto
com outra pessoa significasse perigo de asfixia - claro, nao estou
falando de dormir com maniacos homicidas.)

Alguns diziam q. as fases da Lua se deviam à sombra da Terra
projetada. (Descreviam um eclipse lunar.)

E coisas do gênero. O missão agora é melhorar a qualificação dos
professores as rede pública.

[]s,

Roberto Takata





SUBJECT: Re: [ciencialist] Re: Unidades de medida: símbolo e nome da grandeza
FROM: "E m i l i a n o C h e m e l l o" <chemelloe@yahoo.com.br>
TO: <ciencialist@yahoogrupos.com.br>
DATE: 19/01/2005 10:02

Olá Sérgio,

[Sérgio]
O Qual é o plural de uma palavra que termina em l ?

mil = mis
formol = formois.

mol = mois.

Supondo que o nome da unidade , em portugues, é mol - o que não é -
obtemos o bonito plural : mois.
Mas como no nome da unidade é mole, o plural é o bonito moles.
Ah é tão dificil chegar nesta conclusão ...

[Emiliano]
O inmetro diz que o plural de 'mol' é 'mols'
http://www.inmetro.gov.br/consumidor/unidLegaisMed.asp

Se a gente for se preocupar com tudo... veja a palavra 'átomo' então? :-)

[Sérgio]
Não. Isso é o mesmo que dizer que voce e o seu nome são a mesma coisa. Não,
o seu nome é uma representação de vc. O simbolo é uma representação da
unidade de grandeza, que por sua vez é abstracta. A solução para o seu
problema é um pouco de matemática e portugues basicos. O que é uma Unidade ?
Por definição é algo que não se pode separar em partes. Por exemplo a
unidade matemática é um 1. o 1 é ele mesmo e não pode ser separado em
partes. Podemos encontrar numeros que representariam a separação do 1 em
partes, como 1/2 ou 1/6 , mas esses numeros são a representação das partes e
não as partes em si - pois elas não existem.

Se eu contar laranjas a minha unidade de medida é laranja (singular) Eu não
posso dividir a minha unidade pq ela não existe. É um conceito abstracto.
Eu posso criar outras abstrações que seriam partes da laranja - como gomo -
mas eu estaria criando novas unidades e não dividindo a unidade que tenho.

O que é medir ? É atribuir a uma grnadeza um multiplo da unidade de medida.
Um multiplo. Então 2 g significa 2 x grama e não que eu tenho duas unidades
grama (uma mais gorda e outra mais magra). Eu tenho 3 laranja no meu cesto
não é portugues correcto. Mas: eu tenho 3 lr em meu cesto é. Note que
subtituimos o nome "laranja" pelo simbolo lr. Simbolos não têm plural , da
mesma forma que abreviações não tem plural. Às vezes forçamos isso, como em
"ovnis", mas isso é um neologismo. (que para quem não sabe é um metodo que
cria palavras em uma lingua ) Não faz sentido falar em ONUs , mas em "ovnis"
sim. Ovni não é mais so a abreviação, mas um neologismo que significa : nave
espacial - pertensamente - de outro mundo. Objecto voador não identificado
é penas uma das acepções.

Mas voltando. Dizemos 2 g e pronunciamos (dois gramas) pq simbolos não têm
plural. Mas ao escrever escrevemos dois gramas, pq a lingua portuguesa assim
o obriga. E lembrar que a unidade grama é do genero masculino. Um grama = a
quantidade de massa de alguma coisa Uma grama = uma planta.

Para terminar,
2 mol / L = dois gramas por litro.
2 gs = dois grama segundo

[Emiliano]
Você me interpretou erroneamente. Eu não disse que o nome da unidade e a
unidade eram a mesma coisa. Eu quis dizer que elas eram representadas por
'mol'. Qual é o nome da grandeza que está associada a quantidade de matéria?
*Mol*. Qual é seu símbolo? *Mol*. Isso não acontece somente com o "mol", mas
também com outras unidades, como você pode ver no site que lhe indiquei
(Inmetro).

Outro aspecto sobre os plurais. Se o 'mol x mols' está errado
'portuguesamente' falando, então outras unidades também estão. Ex: pascal x
pascals.

Emiliano Chemello

----- Original Message -----
From: Sergio M. M. Taborda
To: ciencialist@yahoogrupos.com.br
Sent: Wednesday, January 19, 2005 9:13 AM
Subject: [ciencialist] Re: Unidades de medida: símbolo e nome da grandeza



--- Em ciencialist@yahoogrupos.com.br, "E m i l i a n o C h e m e l
l o" <chemelloe@y...> escreveu
> Olá pessoal,
>
> Tá rolando no orkut uma discussão sobre unidades de medida. Resolvi
> trazê-la para cá. As questões são as seguinte:
>
> Um dos participantes escreveu: " ... Já vi autores defenderem
moles como
> no Inglês. Nós brasileiros escrevemos mols. Como quem manda aqui
somos nós..
> levando em consideração a sonoridade da língua, como vocês acham que
devem
> ser? "

O Qual é o plural de uma palavra que termina em l ?

mil = mis
formol = formois.

mol = mois.

Supondo que o nome da unidade , em portugues, é mol - o que não é -
obtemos o bonito plural : mois.
Mas como no nome da unidade é mole, o plural é o bonito moles.
Ah é tão dificil chegar nesta conclusão ...

> Além disso, proponho a discussão sobre outro aspecto: Quando
utilizamos
> o símbolo 'mol', da mesma forma que utilizamos 'g' para se referir a
massa ,
> não colocamos o símbolo da grandeza no plural. Exemplo: 2 g e *não*
2 gs; 2
> mol / L e *não* 2 mols / L Já quando estamos escrevendo o nome da
grandeza,
> ai sim podemos utilizar o plural: Ex: 2 gramas de cloreto de sódio;
2 mols
> de bicarbonato de sódio. Segundo o inmetro, eu estou correto:
>
> http://www.inmetro.gov.br/consumidor/unidLegaisMed.asp
>
> A grande confusão se dá porque o símbolo e a unidade da grandeza
são o
> mesmo: 'mol'.

Não. Isso é o mesmo que dizer que voce e o seu nome são a mesma coisa.
Não, o seu nome é uma representação de vc.
O simbolo é uma representação da unidade de grandeza, que por sua vez
é abstracta.
A solução para o seu problema é um pouco de matemática e portugues
basicos. O que é uma Unidade ? Por definição é algo que não se pode
separar em partes. Por exemplo a unidade matemática é um 1. o 1 é ele
mesmo e não pode ser separado em partes. Podemos encontrar numeros que
representariam a separação do 1 em partes, como 1/2 ou 1/6 , mas esses
numeros são a representação das partes e não as partes em si - pois
elas não existem.
Se eu contar laranjas a minha unidade de medida é laranja (singular)
Eu não posso dividir a minha unidade pq ela não existe. É um conceito
abstracto. Eu posso criar outras abstrações que seriam partes da
laranja - como gomo - mas eu estaria criando novas unidades e não
dividindo a unidade que tenho.
O que é medir ? É atribuir a uma grnadeza um multiplo da unidade de
medida. Um multiplo. Então 2 g significa 2 x grama e não que eu tenho
duas unidades grama (uma mais gorda e outra mais magra).
Eu tenho 3 laranja no meu cesto não é portugues correcto.
Mas: eu tenho 3 lr em meu cesto é. Note que subtituimos o nome
"laranja" pelo simbolo lr. Simbolos não têm plural , da mesma forma
que abreviações não tem plural. Às vezes forçamos isso, como em
"ovnis", mas isso é um neologismo. (que para quem não sabe é um metodo
que cria palavras em uma lingua )
Não faz sentido falar em ONUs , mas em "ovnis" sim. Ovni não é mais so
a abreviação, mas um neologismo que significa : nave espacial -
pertensamente - de outro mundo. Objecto voador não identificado é
apenas uma das acepções.

Mas voltando. Dizemos 2 g e pronunciamos (dois gramas) pq simbolos
não têm plural. Mas ao escrever escrevemos dois gramas, pq a lingua
portuguesa assim o obriga.
E lembrar que a unidade grama é do genero masculino.
Um grama = a quantidade de massa de alguma coisa
Uma grama = uma planta.

Para terminar,
2 mol / L = dois gramas por litro.
2 gs = dois grama segundo

Sérgio Taborda





##### ##### #####

Para saber mais visite
http://www.ciencialist.hpg.ig.com.br


##### ##### ##### #####


Yahoo! Grupos, um serviço oferecido por:
PUBLICIDADE





Links do Yahoo! Grupos

Para visitar o site do seu grupo na web, acesse:
http://br.groups.yahoo.com/group/ciencialist/

Para sair deste grupo, envie um e-mail para:
ciencialist-unsubscribe@yahoogrupos.com.br

O uso que você faz do Yahoo! Grupos está sujeito aos Termos do Serviço do
Yahoo!.




SUBJECT: Re: Unidades de medida: símbolo e nome da grandeza
FROM: "rmtakata" <rmtakata@altavista.net>
TO: ciencialist@yahoogrupos.com.br
DATE: 19/01/2005 10:10


--- Em ciencialist@yahoogrupos.com.br, "Sergio M. M. Taborda"
> O Qual é o plural de uma palavra que termina em l ?
>
> mil = mis
> formol = formois.
>
> mol = mois.
>
> Supondo que o nome da unidade , em portugues, é mol - o que não é -
> obtemos o bonito plural : mois.
> Mas como no nome da unidade é mole, o plural é o bonito moles.
> Ah é tão dificil chegar nesta conclusão ...

Mole talvez em Portugal. No Brasil eh mol. E o plural mols. (Quem
decide aqui essas questoes nao eh a ABL, nem a gramática, mas o
Inmetro - q adapta as convenções internacionais para o sistema
nacional. Em Portugal creio q. seja o IPQ ou acate diretamente as
resoluções do CEN). Assim como se usa gol e o plural eh gols (em
Portugal se fala em golo e golos).

> > A grande confusão se dá porque o símbolo e a unidade da grandeza
> > são o mesmo: 'mol'.
>
> Não. Isso é o mesmo que dizer que voce e o seu nome são a mesma
> coisa.

O q. o Chemmelo diz eh outra coisa. Q. tto o nome da unidade se
escreve mol como o símbolo da unidade se escreve mol. Diferentemente
do grama, q. tem como símbolo a letra g. Para diferenciar o mol nome
por extenso do mol símbolo, usemos nesta mensagem mol* para designar o
último caso. Assim 1 mol e 2 mols (ou em Portugal 1 mole e 2 moles),
mas 1 mol* e 2 mol*, do mesmo modo como 1 grama e 2 gramas e 1g e 2g.
2 mols por litro ou 2 mol*/l - nao se poderia escrever 2 mols/l do
mesmo modo como não se pode escrever 2 quilômetros/h (mas sim 2
quilômetros por hora ou 2 km/h).

No caso de grama e g e quilômetro e km, como são graficamente
distintos, a distinção do uso é mais fácil de se apreender. Mas, como
mol e mol* são graficamente indistintos (exceto se usamos subterfúgios
como este q. usei aqui com a marcação por asterisco ou outro sinal de
diferenciação), a apreensão é dificultada para os estudantes.

[]s,

Roberto Takata





SUBJECT: resenha: "Oxigênio"
FROM: "E m i l i a n o C h e m e l l o" <chemelloe@yahoo.com.br>
TO: <quimica.pe@grupos.com.br>, <quimica@grupos.com.br>, <quimica-qaw@yahoogrupos.com.br>, <forum-ciencia@yahoogrupos.com.br>, <naeq-ucs@yahoogrupos.com.br>, <Conversa_de_Botequim@yahoogrupos.com.br>, <ciencialist@yahoogrupos.com.br>
DATE: 19/01/2005 10:46

Caros amigos,

Segue abaixo o link para vocês baixarem a revista "Tche Química, nº 3" e
verificarem, na página 38, a resenha que fiz sobre o livro "Oxigênio: uma
peça em 2 atos e 20 cenas" de Carl DJERASSI e Roald HOFFMANN. Nem preciso
dizer que recomendo a leitura do livro :-)

http://www.deboni.he.com.br/tq/clics/clic.php3?url=www.deboni.he.com.br/revista3p.pdf


Aquele abraço do

Emiliano Chemello
chemelloe@yahoo.com.br
http://www.quimica.net/emiliano
http://www.ucs.br/ccet/defq/naeq

" Rien ne se perd, rien ne se crée,
tout se transforme."

Antoine Laurent de Lavoisier (químico francês, 1743 - 1794)




SUBJECT: Re: [ciencialist] resenha: "Oxigênio"
FROM: "Luis Alcides HE" <luis.alcides@he.com.br>
TO: <ciencialist@yahoogrupos.com.br>
DATE: 19/01/2005 11:25

Olá, existe uma outra opção para o download desta revista, é um arquivo melhor para imprimir, pois tem 2 páginas por página, porém o arquivo tem 5Mb ao invés dos 1.8Mb do anterior. O link para o arquivo é o seguinte:


http://www.deboni.he.com.br/tq/clics/clic.php3?url=www.deboni.he.com.br/revista32p.pdf

----- Original Message -----
From: E m i l i a n o C h e m e l l o
To: quimica.pe@grupos.com.br ; quimica@grupos.com.br ; quimica-qaw@yahoogrupos.com.br ; forum-ciencia@yahoogrupos.com.br ; naeq-ucs@yahoogrupos.com.br ; Conversa_de_Botequim@yahoogrupos.com.br ; ciencialist@yahoogrupos.com.br
Sent: Wednesday, January 19, 2005 10:46 AM
Subject: [ciencialist] resenha: "Oxigênio"


Caros amigos,

Segue abaixo o link para vocês baixarem a revista "Tche Química, nº 3" e
verificarem, na página 38, a resenha que fiz sobre o livro "Oxigênio: uma
peça em 2 atos e 20 cenas" de Carl DJERASSI e Roald HOFFMANN. Nem preciso
dizer que recomendo a leitura do livro :-)

http://www.deboni.he.com.br/tq/clics/clic.php3?url=www.deboni.he.com.br/revista3p.pdf


Aquele abraço do

Emiliano Chemello
chemelloe@yahoo.com.br
http://www.quimica.net/emiliano
http://www.ucs.br/ccet/defq/naeq

" Rien ne se perd, rien ne se crée,
tout se transforme."

Antoine Laurent de Lavoisier (químico francês, 1743 - 1794)




[As partes desta mensagem que não continham texto foram removidas]



SUBJECT: Fw: ajuda
FROM: "E m i l i a n o C h e m e l l o" <chemelloe@yahoo.com.br>
TO: <ciencialist@yahoogrupos.com.br>
DATE: 19/01/2005 11:51

estou 'meio' sem tempo... eheheh

[ ] 's do Emiliano Chemello
emiliano@quimica.net
http://www.quimica.net/emiliano
http://www.ucs.br/ccet/defq/naeq

" Rien ne se perd, rien ne se crée,
tout se transforme."

Antoine Laurent de Lavoisier (químico francês, 1743 - 1794)

----- Original Message -----
From: Maurício
To: emiliano@quimica.net
Sent: Friday, January 14, 2005 2:48 PM
Subject: ajuda


Boa tarde professor!! se o senhor pudesse me ajudar com uma questão (q na
verdade é de matemática) eu ficaria grato...

" Os átomos de um elemento químico radioativo possuem uma tendência natural
a se desintegrar. Sabendo que chamamos de meia-vida o tempo em que o
elemento leva para desintegrar metade de sua massa e que o antibiótico
Axetil apresenta meia vida de 3 horas, qual é a quantidade aproximada de
antibiótico ainda presente no organismo de uma pessoa que tomou 50mg desse
medicamento, após terem transcorrido 5 horas ?? "

Se puder me mandar os cálculos de resolução eu agradeceria... a resposta eu
sei que é 15,7mg

Obrigado, abraço
Maurício




SUBJECT: Re: resenha: "Oxigênio"
FROM: "rmtakata" <rmtakata@altavista.net>
TO: ciencialist@yahoogrupos.com.br
DATE: 19/01/2005 12:01


--- Em ciencialist@yahoogrupos.com.br, "E m i l i a n o C h e m e l
> Segue abaixo o link para vocês baixarem a revista "Tche Química, nº
> 3" e

Pelo artigo sobre saboes verificamos q. aquele cara q. colocou sal no
detergente fez exatamente o q. a industria faz para *aumentar* a
viscosidade do produto.

http://www.madsci.org/posts/archives/dec99/944247085.Ch.r.html

[]s,

Roberto Takata





SUBJECT: Re: Fw: ajuda
FROM: "rmtakata" <rmtakata@altavista.net>
TO: ciencialist@yahoogrupos.com.br
DATE: 19/01/2005 12:21


O enunciado eh enganoso. Faz uma associacao entre meia-vida de
elementos radioativos e a meia-vida de um composto quimico em um
organismo, como se o composto fosse eliminado pelo mesmo processo.

O calculo eh bastante simples. Sabemos q. a cada 3 horas metade da
massa do composto eh eliminado (metabolizado ou filtrado). Sendo 1 a
massa inicial: 1 - 0h; 0,5 - 3h; 0,25 - 6h; 0,125 - 9h e assim por
diante. Entao a quantidade massa se relaciona com o tempo t decorrido
do seguinte modo:

m = mo/2^(t/3)

A massa inicial mo serah divida por dois tantas vezes qto o tempo t
decorrido for maior do q. a meia-vida do composto (3 horas).

Sendo t = 5 horas, a massa mo serah dividido por dois 5/3 vezes.

m = mo/2^(5/3)

m = 50/2^(5/3) ~ 15,75 mg

Agora uma questao mais dificil: sabendo-se q. a meia-vida de um
produto eh de 2,3 horas e q. a dose inicial administrada em um humano
adulto de 72,3 kg foi de 32mg, de qto em qto tempo devemos administrar
novas doses de 32mg - sabendo q. o produto passa a ter efeitos toxicos
significativos acima de 0,5mg/kg - de modo a maximizar os beneficios
do medicamento durante o periodo de tratamento? (Considere q. a
absorção do medicamento é instantânea - por exemplo, é ministrada por
via endovenosa e q. qto maior a dose, mais rapidamente a doença é
debelada.)

[]s,

Roberto Takata

--- Em ciencialist@yahoogrupos.com.br, "E m i l i a n o C h e m e l
> [ ] 's do Emiliano Chemello
> ----- Original Message -----
> From: Maurício
> " Os átomos de um elemento químico radioativo possuem uma tendência
> natural a se desintegrar. Sabendo que chamamos de meia-vida o tempo
> em que o elemento leva para desintegrar metade de sua massa e que o
> antibiótico Axetil apresenta meia vida de 3 horas, qual é a
> quantidade aproximada de antibiótico ainda presente no organismo de
> uma pessoa que tomou 50mg desse medicamento, após terem transcorrido
> 5 horas ?? "
>
> Se puder me mandar os cálculos de resolução eu agradeceria... a
> resposta eu sei que é 15,7mg
>
> Obrigado, abraço
> Maurício





SUBJECT: Re: Por que massa atrai massa?
FROM: "rayfisica" <rayfisica@yahoo.com.br>
TO: ciencialist@yahoogrupos.com.br
DATE: 19/01/2005 12:49


--- Em ciencialist@yahoogrupos.com.br, "Sergio M. M. Taborda"
<sergiotaborda@y...> escreveu
> >
>
> Sérgio Taborda


Fantástica sua explicação.

Obrigado






SUBJECT: Re: Por que massa atrai massa?
FROM: marcelo ferrari <emailferrari@yahoo.com.br>
TO: ciencialist@yahoogrupos.com.br
DATE: 19/01/2005 13:09

Sergio, sou grato também pelas suas respostas, sempre.
Desta vez foi aristótelico.
Mas em breve, pergunto mais alguma coisa.


[]´s. marcelo ferrari




rayfisica <rayfisica@yahoo.com.br> wrote:
--- Em ciencialist@yahoogrupos.com.br, "Sergio M. M. Taborda"
<sergiotaborda@y...> escreveu
> >
>
> Sérgio Taborda


Fantástica sua explicação.

Obrigado






##### ##### #####

Para saber mais visite
http://www.ciencialist.hpg.ig.com.br


##### ##### ##### #####


Yahoo! Grupos, um serviço oferecido por:



















function SearchComboBox() { if (document.form_combo.keyword.value.length==0){ alert("Por favor, digite algo."); return false; }else { document.form_combo.action ="http://br.rd.yahoo.com/SIG=12a17ivll/M=264105.3931087.6562589.1588051/D=brclubs/S=2137111528:HM/EXP=1106232584/A=2361264/R=0/SIG=11uaou2jn/*http://www.bondfaro.com/bondfaro/in/combosearch_in.jsp?sk=11"; } return true;} [input] [input] [input]

---------------------------------
Links do Yahoo! Grupos

Para visitar o site do seu grupo na web, acesse:
http://br.groups.yahoo.com/group/ciencialist/

Para sair deste grupo, envie um e-mail para:
ciencialist-unsubscribe@yahoogrupos.com.br

O uso que você faz do Yahoo! Grupos está sujeito aos Termos do Serviço do Yahoo!.



__________________________________________________
Converse com seus amigos em tempo real com o Yahoo! Messenger
http://br.download.yahoo.com/messenger/

[As partes desta mensagem que não continham texto foram removidas]



SUBJECT: Gravitação...
FROM: "Rick" <rickardorios@yahoo.com.br>
TO: "l - Ciencia" <ciencialist@yahoogrupos.com.br>
DATE: 19/01/2005 15:01

Pessoal, mais algumas dúvidas astronômicas... :)

1º) Qual a relação entre o peso e a força de atração gravitacional? Como
todas as duas representam um par de acao e reacao entre massas elas nao
deveriam ser a mesma coisa?

2º) Quando o movimento de um corpo em orbita é eliptico, a velocidade
aerolar é constante e a velocidade de translação sofre variações a depender
da posição em que se encontra na orbita. A força centripeta que age neste
corpo é a força de atração gravitacional? Se a resposta for afirmativa,
igualando-se as duas expressôes obtem-se V = raiz (GM/R), o que torna a
velocidade constante. Isso nao deveria contrariar a aceleração que existe na
velocidade de translação? Caso a resposta seja negativa, qual seria a força
centripeta que atua no corpo?

Obrigaduuu!!!

Rick, plagiando o Fábio Junior... hehe :P




SUBJECT: Re: Economia mundial: previsão p/2020
FROM: Manuel Bulcão <manuelbulcao@uol.com.br>
TO: ciencialist@yahoogrupos.com.br
DATE: 19/01/2005 16:01


--- Em ciencialist@yahoogrupos.com.br, José Renato <jrma@t...>
escreveu
> Sinopse da imprensa:
> CIA prevê Brasil entre os grandes dentro de 15 anos
> 07:21 17/01

Manuel: Sobre "previsões", Stephen Hawking escreveu: "Quando
tentamos deduzir o comportamento humano das leis da ciência, ficamos
presos no paradoxo lógico dos sistemas auto-referentes. (...) Se o
que fazemos pudesse ser PREVISTO (...) 0 fato de fazer uma previsão
iria perturbar o sistema e poderia levar a um resultado diferente.
(Buracos Negros, Universos Bebês e Outros Ensaios; Ed. Rocco; pp.
107/110)

Isso significa que, se o futuro já está determinado (e Einstein
sustenta isso), entre as causas que o determinam estão as leis da
matéria -- entre as quais as leis da economia -- como também as
previsões que fazemos tendo por base essas leis: a vontade humana
(o "agir com conhecimento de causa", para usar uma expressão do
velho Engels) também é um fator determinante no universo.

A economia capitalista é um sistema auto-referente. Os capitalistas
fazem seus investimentos (ou decidem entre investir ou entesourar)
tendo por base situações futuras, isto é, "previsões".

O controle da informação sempre foi uma preocupação para todas as
classes ou estamentos dominantes (sacerdotes, aristocratas,
burocratas, etc.). Mas, pelas razões acima, ela é ainda mais
fundamental para as grandes corporações capitalistas.

Abraços,
Manuel Bulcão





SUBJECT: Fw: Consulta 102
FROM: "Silvio" <scordeiro@terra.com.br>
TO: "vanessaloura" <vanessaloura@bol.com.br>, <Conversa_de_Botequim@yahoogrupos.com.br>, <ciencialist@yahoogrupos.com.br>, "Bebeth C. Andrade" <bebethcandrade@yahoo.com.br>, <acropolis@yahoogrupos.com.br>
DATE: 19/01/2005 19:51








Consulta Gratuita ao 102
Vejam só como não somos avisados das coisas que realmente são importantes.

CONSULTA AO 102 A GENTE PAGA R$ 1,20 PELO SERVIÇO. SÓ QUE A TELEFÔNICA NÃO AVISA QUE EXISTE UM SERVIÇO GRATUITO PARA ATENDIMENTO À LISTA. LIGUE 0800.310.102 É O MESMO SERVIÇO, SÓ QUE GRATUITO. (não ligue mais para o 102, que é um roubo!!!)

Se vc já sabia, ótimo! Não custa divulgar para mais gente ficar sabendo.

Serve até mesmo do celular.







[As partes desta mensagem que não continham texto foram removidas]



SUBJECT: Re: ói qui bão prá nóis
FROM: Manuel Bulcão <manuelbulcao@uol.com.br>
TO: ciencialist@yahoogrupos.com.br
DATE: 19/01/2005 19:55


--- Em ciencialist@yahoogrupos.com.br, "murilo filo"
<avalanchedrive@h...> escreveu
>
> Nem sei bem se é off-topic; só sei que não deveria ser!
> Alguém quer comentar o fato de o governo ter retirado o inglês das
provas
> para o Itamaratí?

Manuel: Em verdade vos digo: o inglês não foi retirado, apenas a
prova de inglês passou a ser classificatória, não mais eliminatória.
Durante o curso preparatório posterior, aulas de aperfeiçoamento da
língua serão ministradas.

Mas não estou de acordo com essa medida.

Abraços,
Manuel Bulcão





SUBJECT: Agradecimento aos colegas!
FROM: "Luis" <luis.alcides@he.com.br>
TO: ciencialist@yahoogrupos.com.br
DATE: 19/01/2005 22:48


Gostaria de agradecer a boa receptividade que recebemos dos colegas
no lançamento do Periódico Tchê Química n°3, que em menos de 24h já
teve quase 200 cópias retiradas. Sei que 200 é um número pequeno,
porém as outras edições levaram semanas para chegar apenas na primeira
centena.

Aproveitando esta mensagem, convido aos colegas pesquisadores que
quiserem publicar com a gente que nos enviem seus artigos. A próxima
edição do Periódico Tchê Química será lançada entre julho e agosto
(lembrando que a revista é semestral).

Vou adicionar nesta mensagem os links para se alguém que
eventualmente não leu as edições 1 e 2 possa copiar direto daqui. Na
edição 1 nós tivemos um artigo do professor Wilczek, que acho que é o
que chama mais atenção entre as pessoas que não costumam ler revistas
do tipo científicas, pois ele ganhou o prêmio Nobel de física no ano
passado. E na edição 2, como a gente havia combinado com ele, nós
colocamos a tradução do artigo em português. Na edição 2 a gente teve
a participação de vários professores, ficou uma revista muito boa.

Muito obrigado pela atenção e apoio dos colegas, Luis A. B. De Boni,
Grupo Tchê Química.

Links:

revista 1:
http://www.deboni.he.com.br/tq/clics/clic.php3?url=www.deboni.he.com.br/revistanepreview.pdf


revista 2:
http://www.deboni.he.com.br/tq/clics/clic.php3?url=www.deboni.he.com.br/revista2n.pdf


revista 3:
http://www.deboni.he.com.br/tq/clics/clic.php3?url=www.deboni.he.com.br/revista3p.pdf






SUBJECT: Re: [ciencialist] Re: Fw: ajuda
FROM: "Rodrigo Toledo" <rodrigotoledo11@uol.com.br>
TO: <ciencialist@yahoogrupos.com.br>
DATE: 19/01/2005 22:58

Takata,

Concordo que sua resolução matemática, considerando a resposta já fornecida,
está correta.

Entretanto, a cinética de decomposição de um composto químico não segue
necessariamente uma regra linear. Sabe-se que meia vida de uma reação
química ( pode ser a decomposição de um composto ) depende de vários
fatores, como concentração inicial, ordem da reação, temperatura e
outros...mas concordo tb q a associação com elementos radioativos é
enganosa.

[]'s

RT
----- Original Message -----
From: "rmtakata" <rmtakata@altavista.net>
To: <ciencialist@yahoogrupos.com.br>
Sent: Wednesday, January 19, 2005 12:21 PM
Subject: [ciencialist] Re: Fw: ajuda




O enunciado eh enganoso. Faz uma associacao entre meia-vida de
elementos radioativos e a meia-vida de um composto quimico em um
organismo, como se o composto fosse eliminado pelo mesmo processo.

O calculo eh bastante simples. Sabemos q. a cada 3 horas metade da
massa do composto eh eliminado (metabolizado ou filtrado). Sendo 1 a
massa inicial: 1 - 0h; 0,5 - 3h; 0,25 - 6h; 0,125 - 9h e assim por
diante. Entao a quantidade massa se relaciona com o tempo t decorrido
do seguinte modo:

m = mo/2^(t/3)

A massa inicial mo serah divida por dois tantas vezes qto o tempo t
decorrido for maior do q. a meia-vida do composto (3 horas).

Sendo t = 5 horas, a massa mo serah dividido por dois 5/3 vezes.

m = mo/2^(5/3)

m = 50/2^(5/3) ~ 15,75 mg

Agora uma questao mais dificil: sabendo-se q. a meia-vida de um
produto eh de 2,3 horas e q. a dose inicial administrada em um humano
adulto de 72,3 kg foi de 32mg, de qto em qto tempo devemos administrar
novas doses de 32mg - sabendo q. o produto passa a ter efeitos toxicos
significativos acima de 0,5mg/kg - de modo a maximizar os beneficios
do medicamento durante o periodo de tratamento? (Considere q. a
absorção do medicamento é instantânea - por exemplo, é ministrada por
via endovenosa e q. qto maior a dose, mais rapidamente a doença é
debelada.)

[]s,

Roberto Takata

--- Em ciencialist@yahoogrupos.com.br, "E m i l i a n o C h e m e l
> [ ] 's do Emiliano Chemello
> ----- Original Message -----
> From: Maurício
> " Os átomos de um elemento químico radioativo possuem uma tendência
> natural a se desintegrar. Sabendo que chamamos de meia-vida o tempo
> em que o elemento leva para desintegrar metade de sua massa e que o
> antibiótico Axetil apresenta meia vida de 3 horas, qual é a
> quantidade aproximada de antibiótico ainda presente no organismo de
> uma pessoa que tomou 50mg desse medicamento, após terem transcorrido
> 5 horas ?? "
>
> Se puder me mandar os cálculos de resolução eu agradeceria... a
> resposta eu sei que é 15,7mg
>
> Obrigado, abraço
> Maurício





##### ##### #####

Para saber mais visite
http://www.ciencialist.hpg.ig.com.br


##### ##### ##### #####
Links do Yahoo! Grupos












SUBJECT: Re: Unidades de medida: símbolo e nome da grandeza
FROM: "Sergio M. M. Taborda" <sergiotaborda@yahoo.com.br>
TO: ciencialist@yahoogrupos.com.br
DATE: 19/01/2005 23:02


--- Em ciencialist@yahoogrupos.com.br, "E m i l i a n o C h e m e
l l o" <chemelloe@y...> escreveu
> Olá Sérgio,
>
> [Sérgio]
> O Qual é o plural de uma palavra que termina em l ?
>
> mil = mis
> formol = formois.
>
> mol = mois.
>
> Supondo que o nome da unidade , em portugues, é mol - o que não é -
> obtemos o bonito plural : mois.
> Mas como no nome da unidade é mole, o plural é o bonito moles.
> Ah é tão dificil chegar nesta conclusão ...
>
> [Emiliano]
> O inmetro diz que o plural de 'mol' é 'mols'
> http://www.inmetro.gov.br/consumidor/unidLegaisMed.asp

Tudo bem, em roma sê romano. Mas eu não consigo pronunciar mols, nem
pascals ... soa mal ao ouvido portuguesmente falando.
Eu tb não entendo pq vcs chama ions , etc.. quando portuguesmente
falando o correcto é iões.
Veja, quando eu digo correcto, eu estou me referindo às regras de
formação de palavras, que são aquilo que faz uma lingua.
É incorrecto usar estrangeirismos quando a nossa lingua tem palavras
que os podem substituir facilmente. Ainda por cima no caso dos nomes
das particulas que derivam todas do grego tal como o portugues e
portanto não ha razão logica para recorrer ao ingles. Os ingles
tiveram o seu trabalho adpatando o grego que não pertence à linhagem
do ingles, e agora querem adptar a adpatação ? è no minimo, um
trabalho mal feito. Para não dizer desilegante e porco.
Mas como no brasil ninguem quer nem saber, acho que vai ter que
continuar asssim. E em ultima analise os orgãos de qualidade é que
ditam as regras. Não faz nenhum sentido, mas pronto.

Eu só quero deixar claro que cada lingua tem o seu processo de criar
palavras e esse processo é um algoritmo bem estabelecido. é uma das
poucas coisas em que linguistica se assemelha a uma ciencia. Passar
por cima disso é simplesmente idiota, sobretudo no ambito da
ciencia. É como ter um martelo e pregar com uma chave-de-parafusos
pq ela é electrica. É simplesmente absurdo.

> Se a gente for se preocupar com tudo... veja a palavra 'átomo'
então? :-)

Essa não entendi, qual é o problema com "átomos" ?


Sérgio Taborda





SUBJECT: Re: Unidades de medida: símbolo e nome da grandeza
FROM: "Sergio M. M. Taborda" <sergiotaborda@yahoo.com.br>
TO: ciencialist@yahoogrupos.com.br
DATE: 19/01/2005 23:25


--- Em ciencialist@yahoogrupos.com.br, "rmtakata" <rmtakata@a...>
escreveu

> No caso de grama e g e quilômetro e km, como são graficamente
> distintos, a distinção do uso é mais fácil de se apreender. Mas,
como
> mol e mol* são graficamente indistintos (exceto se usamos
subterfúgios
> como este q. usei aqui com a marcação por asterisco ou outro sinal
de
> diferenciação), a apreensão é dificultada para os estudantes.

Qq estudante que tem dificuldade em entender a diferença deve mudar
de curso. ciencias não é para ele. Se o estudante não consegue
seguir um raciocinio simples como a diferença entre mol - nome e
mol - simbolo , como ele conseguirá alguma vez seguir um raciocinio
cientifico complexo ?

Eu não estou enxergando nem uma situação em que pelo contexto não se
entenda do que se está a falar. Quanto a mim esse problema é
simplesmente ridiculo. O pior é que vcs mesmos se impoem o problema
ao violar as regras de formação de palavras e plurais da vossa
propria lingua...

Sérgio Taborda

Raramente tive que expressar meus calculos em unidades que não as do
SI ( MKS como é chamado tb , por semelhança a CGS , notem que as
maiusculas são abreviações e não simbolos), se bem que usei várias
unidades auxiliares como o litro , o quilograma-força , mas não me
lembro de ter nenhum problema em calcular as mols das soluções. A
regra de três é o mantra dos quimicos e toda a gente sabe que se

x mol - y
a mol - z

então

x = ya/z (mol)

qualquer que seja a unidade de y e z.





...ou será só da minha lingua ? lol





SUBJECT: Re: Aproximacao com o sol...
FROM: Maria Natália <grasdic@hotmail.com>
TO: ciencialist@yahoogrupos.com.br
DATE: 20/01/2005 03:06


Rick:


Estás falando da Terra ou de um planeta que tenha seu eixo de
rotação inclinada no plano de translação da sua estrela central, né?
Pois se eixo de planeta estiver perpendicular ao plano as estações
são iguais e não variam no ano ou se eixo estiver no plano da órbita
haverá sempre uma zona de Inverso e outra de verão eternos (sem
alternância).
Posto este ponto de ordem nas ideias vamos então pensar:
Devido à tal inclinação do eixo da Terra (23,4º) sucede que o sol
está mais inclinado no inverno que no Verão e depois a órbita da
Terra não é assim tão excêntrica. Trocando mais em miudos: Pegas
numa lanterna daquelas de bolso (que se usa qd se fica às escuras)
num quarto à escuras pegas na lanterna ligada e a fazes incidir
perpendicularmente à mesa e que vês? Um disco com a forma de círculo
de luz. Agora inclina o feixe de luz pondo a lanterna oblíqua em
relação à mesa. Que verificas? A superfície iluminda é uma elipse e
parece menos brilhante. Pois é que na mesa a "luz", energia se
espalha agora por maior superfície. Então achas que aquece mais? No
inverno a inclinação dos raios luminosos é maior no hemisfério
Norte 21 ou 22 de Dezembro (varia). Para veres melhor arranjas um
fruto redondo e lhe espete um palito (espeto). Pensa que a mesa é o
plano onde a tua laranja, maçã e o sol estão (este quase ao centro)e
põe a fruta em translação mantendo sempre o palito paralelo nas 4
posições fumdamentais, 2 equinócios e dois solstícios. E se fizeres
isto às escuras com o uma vela a fazer de sol...melhor.
Pensa agora num cometa e na sua órbita à volta do sol. Pois como
este tem órbita com grnade excentricidade...aí já o periélio e
afélio dele são bastante diferentes. É diferente e aí sim a
distãncia contaria.
(no cometa não é só interacção deste estilo há o vento solar qd ele
está próximo). Onde encontras os bonecos que explicam bem isto: num
livro de geografia e qd se fala de clema (geografia física) ou um
bom livro sobre sistema solar ou creio que Leo nosso mestre tem algo
na feiradeciencias.com.br.
Agora entram os físicos com as equações etc e tal e tu vais aguentar
pois já ceitaste o que te expliquei.
Outro facto que certamente ainda não notaste (até porque se calhar
nunca andaste de avião a 13 km de altitude) é que á medida que sobes
na atmosfera (troposfera) a temperatura diminui. Ora se pensassemos
que nos estamos a aproximar do sol teriamos mais calor não era?
Pois ...Isto é só para te dizer que a lógica...tem seus perigos. Nem
tudo o qie parece é. Outros dizem que a lógica é...uma batata.
Um abraço
Maria Natália
--- Em ciencialist@yahoogrupos.com.br, "Rick" <rickardorios@y...>
escreveu
> Pessoal, eu li em uma questao de um livro que quando um planeta
qualquer
> está na região do periélio não é verão. Por que não? A proximidade
com o sol
> nao resultaria em dias mais quentes e consequentemente na estação
mais
> quente?...
>
> Agradecendo a
ajuda
>
> Rick





SUBJECT: Re: Aproximacao com o sol...
FROM: Maria Natália <grasdic@hotmail.com>
TO: ciencialist@yahoogrupos.com.br
DATE: 20/01/2005 03:15



Ei Rick:

Me esqueci mas a massa de atmosfera atravessada pelo feixe de
sol/lanterna é maior qd está mais oblíquo. Logo as poeiras da
atmosfera gastam essa energia ou a difundem e portanto menos chega á
Terra. Mas o mais importante é superfície aquecida com a mesma
fonte.
Maria Natália


--- Em ciencialist@yahoogrupos.com.br, Maria Natália <grasdic@h...>
escreveu
>
> Rick:
>
>
> Estás falando da Terra ou de um planeta que tenha seu eixo de
> rotação inclinada no plano de translação da sua estrela central,
né?
> Pois se eixo de planeta estiver perpendicular ao plano as estações
> são iguais e não variam no ano ou se eixo estiver no plano da
órbita
> haverá sempre uma zona de Inverso e outra de verão eternos (sem
> alternância).
> Posto este ponto de ordem nas ideias vamos então pensar:
> Devido à tal inclinação do eixo da Terra (23,4º) sucede que o sol
> está mais inclinado no inverno que no Verão e depois a órbita da
> Terra não é assim tão excêntrica. Trocando mais em miudos: Pegas
> numa lanterna daquelas de bolso (que se usa qd se fica às escuras)
> num quarto à escuras pegas na lanterna ligada e a fazes incidir
> perpendicularmente à mesa e que vês? Um disco com a forma de
círculo
> de luz. Agora inclina o feixe de luz pondo a lanterna oblíqua em
> relação à mesa. Que verificas? A superfície iluminda é uma elipse
e
> parece menos brilhante. Pois é que na mesa a "luz", energia se
> espalha agora por maior superfície. Então achas que aquece mais?
No
> inverno a inclinação dos raios luminosos é maior no hemisfério
> Norte 21 ou 22 de Dezembro (varia). Para veres melhor arranjas um
> fruto redondo e lhe espete um palito (espeto). Pensa que a mesa é
o
> plano onde a tua laranja, maçã e o sol estão (este quase ao centro)
e
> põe a fruta em translação mantendo sempre o palito paralelo nas 4
> posições fumdamentais, 2 equinócios e dois solstícios. E se
fizeres
> isto às escuras com o uma vela a fazer de sol...melhor.
> Pensa agora num cometa e na sua órbita à volta do sol. Pois como
> este tem órbita com grnade excentricidade...aí já o periélio e
> afélio dele são bastante diferentes. É diferente e aí sim a
> distãncia contaria.
> (no cometa não é só interacção deste estilo há o vento solar qd
ele
> está próximo). Onde encontras os bonecos que explicam bem isto:
num
> livro de geografia e qd se fala de clema (geografia física) ou um
> bom livro sobre sistema solar ou creio que Leo nosso mestre tem
algo
> na feiradeciencias.com.br.
> Agora entram os físicos com as equações etc e tal e tu vais
aguentar
> pois já ceitaste o que te expliquei.
> Outro facto que certamente ainda não notaste (até porque se calhar
> nunca andaste de avião a 13 km de altitude) é que á medida que
sobes
> na atmosfera (troposfera) a temperatura diminui. Ora se
pensassemos
> que nos estamos a aproximar do sol teriamos mais calor não era?
> Pois ...Isto é só para te dizer que a lógica...tem seus perigos.
Nem
> tudo o qie parece é. Outros dizem que a lógica é...uma batata.
> Um abraço
> Maria Natália
> --- Em ciencialist@yahoogrupos.com.br, "Rick" <rickardorios@y...>
> escreveu
> > Pessoal, eu li em uma questao de um livro que quando um planeta
> qualquer
> > está na região do periélio não é verão. Por que não? A
proximidade
> com o sol
> > nao resultaria em dias mais quentes e consequentemente na
estação
> mais
> > quente?...
> >
> > Agradecendo
a
> ajuda
> >
> > Rick





SUBJECT: Fw: Vestibular
FROM: "Luiz Ferraz Netto" <leobarretos@uol.com.br>
TO: "ciencialist" <ciencialist@yahoogrupos.com.br>
DATE: 20/01/2005 06:07

Quem pode resolver esse ligeiro problema da Terezinha?
[]'
===========================
Luiz Ferraz Netto [Léo]
leobarretos@uol.com.br
http://www.feiradeciencias.com.br
===========================
-----Mensagem Original-----
De: "TERESINHA APARECIDA DE CAMARGO" <teresinha.acamargo@sp.senac.br>
Para: <leobarretos@uol.com.br>
Enviada em: terça-feira, 18 de janeiro de 2005 15:31
Assunto: Vestibular


Boa tarde, Professor Léo.
Que bom encontrar o seu site,quem sabe você possa me ajudar.
Meu nome é Teresinha, sou de Botucatu mas,atualmente resido na cidade de São Paulo,preciso muito de sua ajuda,sou péssima em física mas vou prestar o vestubular pela segunda vez,porque a primeira eu não consegui passar,no vestibular vai física com os temas,Mêcanica,Termodinâmica,Ondas,Som e luz,Eletromagnetismo,Interarações,Matéria e Energia.
Não tenho nada parecido para estudar e o vestubular será no dia 30/01 no domingo ás 9:00.
Estou desesperada,porque todos estão botando a maior fé que irei passar.
Muito Obrigada pelo desabafo.
Espero muito sua resposta professor léo sei lá uma dica qualquer coisa.
Boa Tarde!
teka





Teresinha Ap.de Camargo
Senac Lapa Tito
Biblioteca - Atendimento
11 3868-6932 / 6933
teresinha.acamargo@sp.senac.br



--
No virus found in this incoming message.
Checked by AVG Anti-Virus.
Version: 7.0.300 / Virus Database: 265.6.13 - Release Date: 16/01/2005




--
No virus found in this outgoing message.
Checked by AVG Anti-Virus.
Version: 7.0.300 / Virus Database: 265.6.13 - Release Date: 16/01/2005



SUBJECT: Fw: pesquisa
FROM: "Luiz Ferraz Netto" <leobarretos@uol.com.br>
TO: "ciencialist" <ciencialist@yahoogrupos.com.br>
DATE: 20/01/2005 06:08

Para os Cuímicos...........

[]'
===========================
Luiz Ferraz Netto [Léo]
leobarretos@uol.com.br
http://www.feiradeciencias.com.br
===========================
-----Mensagem Original-----
De: bruno dias sales
Para: leobarretos@uol.com.br
Enviada em: quarta-feira, 19 de janeiro de 2005 01:59
Assunto: pesquisa


Gostaria de saber qual metal tem maior potencial de oxidação e qual metal tem maior potencial de redução em um meio eletrolito formado por água e sal ?Como a pressão pode influenciar nesta reação de oxi-redução? Se aumentar a pressão, aumenta a diferença de potencial? Como a concentração atua neste aspecto? E qual a importância da temperatura e do ph levando em consideração o processo de oxi-redução? Obrigado pela atenção! Espero contato!
__________________________________________________
Converse com seus amigos em tempo real com o Yahoo! Messenger
http://br.download.yahoo.com/messenger/



--------------------------------------------------------------------------------


No virus found in this incoming message.
Checked by AVG Anti-Virus.
Version: 7.0.300 / Virus Database: 265.6.13 - Release Date: 16/01/2005

----------

No virus found in this outgoing message.
Checked by AVG Anti-Virus.
Version: 7.0.300 / Virus Database: 265.6.13 - Release Date: 16/01/2005


[As partes desta mensagem que não continham texto foram removidas]



SUBJECT: Re: [ciencialist] Re: Aproximacao com o sol...
FROM: "Luiz Ferraz Netto" <leobarretos@uol.com.br>
TO: <ciencialist@yahoogrupos.com.br>
DATE: 20/01/2005 06:58

Modestas sugestões do Feira de Ciências:

Estações do ano --- nível 1 (5a/8a séries)
http://www.feiradeciencias.com.br/sala19/texto02.asp

Dia e Noite (durações) --- nível 2 (ensino médio)
http://www.feiradeciencias.com.br/sala19/texto71c.asp

Estações do ano --- níveis 1 e 2
Abaixo, cópia do item 15 da página:
http://www.feiradeciencias.com.br/sala19/texto53.asp

"15 - Estações do Ano
Insolação é exposição à radiação solar. Com a atmosfera límpida, a energia radiante incidente na superfície da Terra por unidade de área depende da duração da exposição e da altura do Sol. Em cada local o máximo de a ocorre ao meio-dia.

Chama-se constante solar a potência da radiação solar incidente na alta atmosfera por unidade de área em superfície normal aos raios; é ela:

(CS) ~= 2,0 cal/min.cm2 ~= 1,4 kW/m2

A Terra recebe energia radiante solar no hemisfério iluminado; simultaneamente, em todo o globo, ela emite energia radiante para o espaço cósmico.

Em média, a temperatura em uma região do globo é determinada principalmente por estes ganhos e perdas, mas há outros fatores poderosamente influentes. Por exemplo: ventos dominantes, frentes frias e quentes, corrente do Golfo, 'El Niño'.
Um dos fatores climáticos mais decisivos é a diferença na distribuição de terras e mares entre os dois hemisférios. A terra acumula a energia recebida na insolação até 10 ou 15 cm de profundidade enquanto na água este aquecimento chega a 100 m. Como a camada de terra libera mais facilmente a energia recebida, o hemisfério norte tem os verões muito mais quentes e os invernos muito mais frios.

A alternância de Equinócios e Solstícios decorre da obliqüidade e da eclíptica: o eixo SN da Terra faz ângulo (90o - e ) = 66,55o com o plano da eclíptica. Resultam as quatro estações do ano entre os sucessivos Equinócios e Solstícios:

Primavera N
Outono S 21-Mar a 21-Jun
Verão N
Inverno S 21-Jun a 23-Set
Outono N
Primavera S 23-Set a 21-Dez
Inverno N
Verão S 21-Dez - 21-Mar

Em cada zona da Terra a insolação varia de uma estação para outra. É este o fator preponderante nas condições meteorológicas (temperatura, umidade do ar, regime de chuvas, ventos, correntes marítimas etc). Por sua vez, estes fatores influem na vida vegetal (semeadura, colheita) e na vida animal (migrações, ...).

De uma estação para outra, a variação das condições físicas e biológicas se acentua com o aumento de latitude da zona terrestre considerada."

aquele abraço,

===========================
Luiz Ferraz Netto [Léo]
leobarretos@uol.com.br
http://www.feiradeciencias.com.br
===========================
-----Mensagem Original-----
De: "Maria Natália" <grasdic@hotmail.com>
Para: <ciencialist@yahoogrupos.com.br>
Enviada em: quinta-feira, 20 de janeiro de 2005 03:06
Assunto: [ciencialist] Re: Aproximacao com o sol...




Rick:


Estás falando da Terra ou de um planeta que tenha seu eixo de
rotação inclinada no plano de translação da sua estrela central, né?
Pois se eixo de planeta estiver perpendicular ao plano as estações
são iguais e não variam no ano ou se eixo estiver no plano da órbita
haverá sempre uma zona de Inverso e outra de verão eternos (sem
alternância).
[ ... ]


--
No virus found in this outgoing message.
Checked by AVG Anti-Virus.
Version: 7.0.300 / Virus Database: 265.6.13 - Release Date: 16/01/2005



SUBJECT: Re: Unidades de medida: símbolo e nome da grandeza
FROM: "rmtakata" <rmtakata@altavista.net>
TO: ciencialist@yahoogrupos.com.br
DATE: 20/01/2005 08:30


--- Em ciencialist@yahoogrupos.com.br, "Sergio M. M. Taborda"
> Tudo bem, em roma sê romano. Mas eu não consigo pronunciar mols, nem
> pascals ... soa mal ao ouvido portuguesmente falando.

Eh q. aqui o 'l' no fim das palavras mal se distingue do 'u' -- aqui
mal e mau foneticamente sao indistinguiveis. Assim, se quiser
pronunciar mols 'a moda brazuca, basta pensar em 'mous' (com a tonica
em 'o').

> Eu tb não entendo pq vcs chama ions , etc.. quando portuguesmente
> falando o correcto é iões.

Aqui, aceitam-se palavras terminadas em 'n' - elétron (e não
electrão), fóton (e não fotão), pólen (e não polem), hífen (e não
hífene). Muitas das questões podem ser resumidas às diferenças
fonéticas entre Brasil e Portugal. À moda lusitana certas grafias
brasileiras soariam mal e vice-versa: o 'a' em Portugal soa como um
schwa (como os ingleses pronunciam o 'a' em 'about' - um som
intermediário entre 'a', 'e' e 'o'), aqui o 'a' é quase sempre
pronunciado aberto, assim faz sentido q. em Lusitânia se escreva 'ão'
e os tuipiniquins escrevam 'on' (o som acaba se aproximando - mas se
'ão' é pronunciado à brasileira sai um som bastante diferente; do
mesmo modo q. se um português pronunciar 'on', teremos um outro som).
(Claro tb há difereça na sílaba em q. recai o acento tônico. Mas
novamente é uma questão da estrutura fonética.)

> Veja, quando eu digo correcto, eu estou me referindo às regras de
> formação de palavras, que são aquilo que faz uma lingua.

A língua se faz com muito mais do q. regras. As regras em geral são
derivadas e depois tenta-se impô-la como uma camisa de força. Certo q.
é necessário q. haja regras de modo a garantir um mínimo de unidade.
Mas ela não consegue atuar de modo absoluto. A língua é dinâmica posto
q. ela é um instrumento de comunicação interpessoal - influi a
cultura, o ambiente físico (sim! pense no seguinte - em um lugar
barulhento, a tendência é aumentar a intensidade da voz, isso altera o
modo como pronunciamos as palavras), a história, a conexão com os
vizinhos. (Sem falar nas diferenças entre as pessoas: nível
socioeconômico, idade, sexo e assim por diante.) As alterações
dinâmicas da língua podem ser freadas, mas não contidas. (Se duvida,
leia Camões em sua grafia original. Talvez ele dissesse q. os
portugueses hodiernos desfiguraram a pureza da recém-emergida língua
lusa.)

Não creia q. o português de Portugal é o mais correto (ou correcto).
Tampouco é o menos correto. Cada variante adapta-se às condições
locais. Veja como os angolanos se comunicam, como os timorenses
orientais escrevem o português. Isso ocorre tb com as diferenças entre
o inglês britânico (ou mais precisamente inglês inglês, já q. existem
diferenças entre os países da ilha), o australiano e o americano. Com
o espanhol argentino, mexicano e o da Espanha. Compare o português do
Porto ou de Lisboa e o de Trás-os-Montes. (O português gaúcho,
fluminense e cearense.)

(Se se preocupa com os estrangeirismos. Golo poderia ser substituído
perfeitamente por tento, ponto. Apenas não seria a mesma coisa q. o
goal do futebol...)

[]s,

Roberto Takata





SUBJECT: Re: Unidades de medida: símbolo e nome da grandeza
FROM: "rmtakata" <rmtakata@altavista.net>
TO: ciencialist@yahoogrupos.com.br
DATE: 20/01/2005 08:39


> --- Em ciencialist@yahoogrupos.com.br, "rmtakata" <rmtakata@a...>
> > No caso de grama e g e quilômetro e km, como são graficamente
> > distintos, a distinção do uso é mais fácil de se apreender. Mas,
> > como mol e mol* são graficamente indistintos (exceto se usamos
> > subterfúgios como este q. usei aqui com a marcação por asterisco
> > ou outro sinal de diferenciação), a apreensão é dificultada para
> > os estudantes.

--- Em ciencialist@yahoogrupos.com.br, "Sergio M. M. Taborda"
> Qq estudante que tem dificuldade em entender a diferença deve mudar
> de curso. ciencias não é para ele.

É uma opinião. Embora na minha opinião seja uma opinião radical. O
estudante provavelmente entenderá a diferença se ela for bem
trabalhada - o professor deveria chamar a atenção para isso.
Infelizmente mesmo o professor, muitas vezes, não tem uma consciência
clara sobre esse problema.

> Raramente tive que expressar meus calculos em unidades que não as do
> SI ( MKS como é chamado tb , por semelhança a CGS , notem que as

Na verdade o SI é um desenvolvimento do MKS. Um teórico realmente
raras terá de se defrontar com unidades q. não o SI. Já um prático
terá q. se defrontar com unidades como onças (troy e averdupois),
libras, polegadas, pés - tudo por causa do comércio internacional e os
americanos e ingleses sentem-se muito confortáveis com suas medidas
medievais (ou mais antigas ainda).

> mas não me lembro de ter nenhum problema em calcular as mols das
> soluções.

A questão não é calcular, mas representar a unidade. Muitas pessoas
ainda colocam 's' como plural de símbolos como g, km. 2 gs (ou 2 grms)
ou 30 kms ou 5 hs (aliás, por influência americana com freqüência se
escreve por aqui 7:30 em lugar de 7h30), a tentação é grande, já q.
nas abreviaturas com freqüência se emprega o 's' pluralizante. E do q.
se confunde abreviatura com símbolo surgem os pontos e os esses.

[]s,

Roberto Takata







SUBJECT: Re: [ciencialist] Re: Unidades de medida: símbolo e nome da grandeza
FROM: "E m i l i a n o C h e m e l l o" <chemelloe@yahoo.com.br>
TO: <ciencialist@yahoogrupos.com.br>
DATE: 20/01/2005 08:50

> Se a gente for se preocupar com tudo... veja a palavra 'átomo'
então? :-)

[Sérgio]
Essa não entendi, qual é o problema com "átomos" ?

[Emiliano]
Ops... eu derivei o assunto sem avisar antes. Me referi ao 'sentido de
átomo' e a problemática de sua origem com relação ao que hoje conhecemos
dele.

[ ] 's do Emiliano Chemello
emiliano@quimica.net
http://www.quimica.net/emiliano
http://www.ucs.br/ccet/defq/naeq

" Rien ne se perd, rien ne se crée,
tout se transforme."

Antoine Laurent de Lavoisier (químico francês, 1743 - 1794)

----- Original Message -----
From: Sergio M. M. Taborda
To: ciencialist@yahoogrupos.com.br
Sent: Wednesday, January 19, 2005 11:02 PM
Subject: [ciencialist] Re: Unidades de medida: símbolo e nome da grandeza



--- Em ciencialist@yahoogrupos.com.br, "E m i l i a n o C h e m e
l l o" <chemelloe@y...> escreveu
> Olá Sérgio,
>
> [Sérgio]
> O Qual é o plural de uma palavra que termina em l ?
>
> mil = mis
> formol = formois.
>
> mol = mois.
>
> Supondo que o nome da unidade , em portugues, é mol - o que não é -
> obtemos o bonito plural : mois.
> Mas como no nome da unidade é mole, o plural é o bonito moles.
> Ah é tão dificil chegar nesta conclusão ...
>
> [Emiliano]
> O inmetro diz que o plural de 'mol' é 'mols'
> http://www.inmetro.gov.br/consumidor/unidLegaisMed.asp

Tudo bem, em roma sê romano. Mas eu não consigo pronunciar mols, nem
pascals ... soa mal ao ouvido portuguesmente falando.
Eu tb não entendo pq vcs chama ions , etc.. quando portuguesmente
falando o correcto é iões.
Veja, quando eu digo correcto, eu estou me referindo às regras de
formação de palavras, que são aquilo que faz uma lingua.
É incorrecto usar estrangeirismos quando a nossa lingua tem palavras
que os podem substituir facilmente. Ainda por cima no caso dos nomes
das particulas que derivam todas do grego tal como o portugues e
portanto não ha razão logica para recorrer ao ingles. Os ingles
tiveram o seu trabalho adpatando o grego que não pertence à linhagem
do ingles, e agora querem adptar a adpatação ? è no minimo, um
trabalho mal feito. Para não dizer desilegante e porco.
Mas como no brasil ninguem quer nem saber, acho que vai ter que
continuar asssim. E em ultima analise os orgãos de qualidade é que
ditam as regras. Não faz nenhum sentido, mas pronto.

Eu só quero deixar claro que cada lingua tem o seu processo de criar
palavras e esse processo é um algoritmo bem estabelecido. é uma das
poucas coisas em que linguistica se assemelha a uma ciencia. Passar
por cima disso é simplesmente idiota, sobretudo no ambito da
ciencia. É como ter um martelo e pregar com uma chave-de-parafusos
pq ela é electrica. É simplesmente absurdo.

> Se a gente for se preocupar com tudo... veja a palavra 'átomo'
então? :-)

Essa não entendi, qual é o problema com "átomos" ?


Sérgio Taborda






SUBJECT: Re: [ciencialist] Fw: Consulta 102
FROM: "Alvaro Augusto \(E\)" <alvaro@electraenergy.com.br>
TO: <ciencialist@yahoogrupos.com.br>
DATE: 20/01/2005 10:26

Caro Silvio,

Eu disquei para 0800.310.102 e esse número não está ativo!

[ ]s

Alvaro Augusto
----- Original Message -----
From: Silvio
To: vanessaloura ; Conversa_de_Botequim@yahoogrupos.com.br ; ciencialist@yahoogrupos.com.br ; Bebeth C. Andrade ; acropolis@yahoogrupos.com.br
Sent: Wednesday, January 19, 2005 7:51 PM
Subject: [ciencialist] Fw: Consulta 102









Consulta Gratuita ao 102
Vejam só como não somos avisados das coisas que realmente são importantes.

CONSULTA AO 102 A GENTE PAGA R$ 1,20 PELO SERVIÇO. SÓ QUE A TELEFÔNICA NÃO AVISA QUE EXISTE UM SERVIÇO GRATUITO PARA ATENDIMENTO À LISTA. LIGUE 0800.310.102 É O MESMO SERVIÇO, SÓ QUE GRATUITO. (não ligue mais para o 102, que é um roubo!!!)

Se vc já sabia, ótimo! Não custa divulgar para mais gente ficar sabendo.

Serve até mesmo do celular.





[As partes desta mensagem que não continham texto foram removidas]



SUBJECT: Re: [ciencialist] Gravitação...
FROM: "Luiz Ferraz Netto" <leobarretos@uol.com.br>
TO: <ciencialist@yahoogrupos.com.br>
DATE: 20/01/2005 10:26

Olá Rick,

de verdade em verdade vos digo:

1) Peso é a resultante das forças que agem sobre uma determinada massa mergulhada no campo gravitacional de alguma outra massa e é dependente do referencial adotado.
Não tem sentido a pergunta "Qual o peso da Lua?" sem a indicação do referencial, assim como é deficiente a pergunta "Qual o peso de um Fusca?" sem a indicação do local onde ele se encontra sobre a superfície da Terra (não há Fuscas em outros planetas ou satélites --- não esqueça: de verdade em verdade vos digo!)
Força de atração gravitacional é exclusivamente a força entre duas massas (para ficar dentro dos exemplos).
Em algumas situações 'ideais' o peso de um corpo e a força gravitacional sobre ele se confundem; é o caso de um corpo caindo sobre a superfície da Terra (abstraindo-se a presença do ar e de todos os movimentos da Terra). É tb o caso do peso dos satélites artificiais em órbita. É o caso de se calcular o peso da Lua em relação à Terra, ou em relação ao Sol.
Para o corpo na superfície da Terra, por exemplo, essa igualdade já não é verdadeira devido à presença da força centrífuga que age no corpo no referencial Terra, além da atração gravitacional. Nesse caso, o peso do corpo é dependente da latitude (pois altera a distância dele até o eixo da Terra).
Não entendi o que é que o par ação/reação tem a ver com isso?

2) "A força centrípeta que age neste corpo é a força de atração gravitacional?"
Resposta: Sim, e nesse caso é tb denominada 'força central'.
"Se a resposta for afirmativa, igualando-se as duas expressões obtém-se V = raiz (GM/R), o que torna a velocidade constante. Isso não deveria contrariar a aceleração que existe na velocidade de translação?"
Resposta: Conclusão falsa.
Vejamos: GmM/r^2 = mv^2/r ===> v = sqrt(GM/r) <===
Observe: v é função de r (onde r é o raio de curvatura da curva no instante considerado). Raio de curvatura de uma curva, num dado ponto, é o raio da circunferência osculadora à curva nesse ponto.

[]'
===========================
Luiz Ferraz Netto [Léo]
leobarretos@uol.com.br
http://www.feiradeciencias.com.br
===========================
-----Mensagem Original-----
De: "Rick" <rickardorios@yahoo.com.br>
Para: "l - Ciencia" <ciencialist@yahoogrupos.com.br>
Enviada em: quarta-feira, 19 de janeiro de 2005 15:01
Assunto: [ciencialist] Gravitação...



Pessoal, mais algumas dúvidas astronômicas... :)

1º) Qual a relação entre o peso e a força de atração gravitacional? Como
todas as duas representam um par de acao e reacao entre massas elas nao
deveriam ser a mesma coisa?

2º) Quando o movimento de um corpo em orbita é eliptico, a velocidade
aerolar é constante e a velocidade de translação sofre variações a depender
da posição em que se encontra na orbita. A força centripeta que age neste
corpo é a força de atração gravitacional? Se a resposta for afirmativa,
igualando-se as duas expressôes obtem-se V = raiz (GM/R), o que torna a
velocidade constante. Isso nao deveria contrariar a aceleração que existe na
velocidade de translação? Caso a resposta seja negativa, qual seria a força
centripeta que atua no corpo?

Obrigaduuu!!!

Rick, plagiando o Fábio Junior... hehe :P




##### ##### #####

Para saber mais visite
http://www.ciencialist.hpg.ig.com.br


##### ##### ##### #####
Links do Yahoo! Grupos










--
No virus found in this incoming message.
Checked by AVG Anti-Virus.
Version: 7.0.300 / Virus Database: 265.6.13 - Release Date: 16/01/2005




--
No virus found in this outgoing message.
Checked by AVG Anti-Virus.
Version: 7.0.300 / Virus Database: 265.6.13 - Release Date: 16/01/2005



SUBJECT: Re: A igreja católica é a favor da evolução?
FROM: Manuel Bulcão <manuelbulcao@uol.com.br>
TO: ciencialist@yahoogrupos.com.br
DATE: 20/01/2005 12:16


Oi,

> Quantos salmões se perderam e jamais conseguiram chegar ao riacho
em que haviam nascido para desovar antes que essa estranha
capacidade fosse desenvolvida?

Takata: Ue' mesmo hoje - com sua capacidade migratoria plenamente
desenvolvida - milhoes de salmoes morrem antes de completar sua
jornada. Sacanagem divina? Ou a mortandade de milhoes de animais
inocentes eh cruel apenas de for por meio da selecao natural, mas
passa a ser um sacrificio beatifico se eh por ordem de Deus? Eh vai
ver q. eh, afinal

Manuel: A pior "sacanagem divina" que conheço é a protagonizada
pelas "moscas icnêumones", que na verdade não são moscas, mas um
gênero de vespa que possui mais espécies que todos os vertebrados
juntos.

Os icnêumones injetam seus ovos em outros animais, de preferência
lagartas (larvas de borboleta), afídios e aranhas. Esses hospedeiros
são fábricas de alimentos para as larvas dessas vespas: eles são
comidos vivos por dentro, a começar pelos órgãos não vitais. Embora
paralisadas (ou semiparalisadas, pois é comum observar o grilo ou a
aranha se debatendo enquanto suas entranhas são devoradas), o animal
não se encontra anestesiado, de modo que ele sente dor durante todo
ou a maior parte do processo, que dura dias, às vezes semanas.

Outra sacanagem divina que vi foi há uns cinco anos atrás: uma
mulher pobre conduzindo sozinha e de ônibus (ônibus lotado) seus
dois filhos autistas "severos" à Casa da Esperança, uma instituição
criada por pais e amigos dos autistas. As crianças gritavam,
urravam, "incomodavam". Muita gente insensível (ou brutalizadas pela
ignorância) riam e/ou reclamavam. Jamais me esquecerei do rosto
daquela mãe.

É, a natureza é complexa, mas não é perfeita. E se Deus escreve
certo com linhas tortas, sinceramente, êita caligrafiazinha ruim!

Abraços,
Manuel Bulcão





SUBJECT: Icneumonas (era A igreja católica é a favor da evolução?)
FROM: "Oraculo" <oraculo@atibaia.com.br>
TO: <ciencialist@yahoogrupos.com.br>
DATE: 20/01/2005 12:37

Olá Manuel

A história dos icneumonas pode ser encntrada com detalhes no livro do Jay Gould, Os Dentes da Galinha. É excelente, e está bem baratinho na Submarino..:-)

Um abraço.

Homero

----- Original Message -----
From: Manuel Bulcão
To: ciencialist@yahoogrupos.com.br
Sent: Thursday, January 20, 2005 12:16 PM
Subject: [ciencialist] Re: A igreja católica é a favor da evolução?



Oi,

> Quantos salmões se perderam e jamais conseguiram chegar ao riacho
em que haviam nascido para desovar antes que essa estranha
capacidade fosse desenvolvida?

Takata: Ue' mesmo hoje - com sua capacidade migratoria plenamente
desenvolvida - milhoes de salmoes morrem antes de completar sua
jornada. Sacanagem divina? Ou a mortandade de milhoes de animais
inocentes eh cruel apenas de for por meio da selecao natural, mas
passa a ser um sacrificio beatifico se eh por ordem de Deus? Eh vai
ver q. eh, afinal

Manuel: A pior "sacanagem divina" que conheço é a protagonizada
pelas "moscas icnêumones", que na verdade não são moscas, mas um
gênero de vespa que possui mais espécies que todos os vertebrados
juntos.

Os icnêumones injetam seus ovos em outros animais, de preferência
lagartas (larvas de borboleta), afídios e aranhas. Esses hospedeiros
são fábricas de alimentos para as larvas dessas vespas: eles são
comidos vivos por dentro, a começar pelos órgãos não vitais. Embora
paralisadas (ou semiparalisadas, pois é comum observar o grilo ou a
aranha se debatendo enquanto suas entranhas são devoradas), o animal
não se encontra anestesiado, de modo que ele sente dor durante todo
ou a maior parte do processo, que dura dias, às vezes semanas.

Outra sacanagem divina que vi foi há uns cinco anos atrás: uma
mulher pobre conduzindo sozinha e de ônibus (ônibus lotado) seus
dois filhos autistas "severos" à Casa da Esperança, uma instituição
criada por pais e amigos dos autistas. As crianças gritavam,
urravam, "incomodavam". Muita gente insensível (ou brutalizadas pela
ignorância) riam e/ou reclamavam. Jamais me esquecerei do rosto
daquela mãe.

É, a natureza é complexa, mas não é perfeita. E se Deus escreve
certo com linhas tortas, sinceramente, êita caligrafiazinha ruim!

Abraços,
Manuel Bulcão





##### ##### #####

Para saber mais visite
http://www.ciencialist.hpg.ig.com.br


##### ##### ##### #####


Yahoo! Grupos, um serviço oferecido por:







------------------------------------------------------------------------------
Links do Yahoo! Grupos

a.. Para visitar o site do seu grupo na web, acesse:
http://br.groups.yahoo.com/group/ciencialist/

b.. Para sair deste grupo, envie um e-mail para:
ciencialist-unsubscribe@yahoogrupos.com.br

c.. O uso que você faz do Yahoo! Grupos está sujeito aos Termos do Serviço do Yahoo!.



[As partes desta mensagem que não continham texto foram removidas]



SUBJECT: Re: A igreja católica é a favor da evolução?
FROM: Manuel Bulcão <manuelbulcao@uol.com.br>
TO: ciencialist@yahoogrupos.com.br
DATE: 20/01/2005 12:37


--- Em ciencialist@yahoogrupos.com.br, "JVictor" <jvoneto@u...>
escreveu
> Um dia eu lí: "...enquanto a maioria das pessoas vive se
preocupando com o fato de Deus existir ou não, sua curta vida vai se
acabando..." Avaliem só o profundo significado prático desse
ensinamento. Quem proferiu semelhante pérola, foi um gordinho
chamado Buda. Cara sabido. Que Deus o tenha!

Manuel: uma outra sentença de Buda citada por Gore Vidal em seu
romance "Criação": "Talvez os deuses existam, mas não devemos nos
preocupar com eles, já que eles não se preocupam com a gente" (as
palavras são mais ou menos essas)

Abraços,
Manuel Bulcão





SUBJECT: Os Dentes da Galinha (era A igreja católica é a favor da evolução?)
FROM: "Oraculo" <oraculo@atibaia.com.br>
TO: <ciencialist@yahoogrupos.com.br>
DATE: 20/01/2005 12:39

Olá Manuel

E eu já havia esquecido, mas postei o livro Os Dentes da Galinha nos arquivos da lista, basta entrar e baixar para ler. É um livro curto, rápido de ler e bem instrutivo..:-)

Um abraço.

Homero
----- Original Message -----
From: Manuel Bulcão
To: ciencialist@yahoogrupos.com.br
Sent: Thursday, January 20, 2005 12:16 PM
Subject: [ciencialist] Re: A igreja católica é a favor da evolução?



Oi,

> Quantos salmões se perderam e jamais conseguiram chegar ao riacho
em que haviam nascido para desovar antes que essa estranha
capacidade fosse desenvolvida?

Takata: Ue' mesmo hoje - com sua capacidade migratoria plenamente
desenvolvida - milhoes de salmoes morrem antes de completar sua
jornada. Sacanagem divina? Ou a mortandade de milhoes de animais
inocentes eh cruel apenas de for por meio da selecao natural, mas
passa a ser um sacrificio beatifico se eh por ordem de Deus? Eh vai
ver q. eh, afinal

Manuel: A pior "sacanagem divina" que conheço é a protagonizada
pelas "moscas icnêumones", que na verdade não são moscas, mas um
gênero de vespa que possui mais espécies que todos os vertebrados
juntos.

Os icnêumones injetam seus ovos em outros animais, de preferência
lagartas (larvas de borboleta), afídios e aranhas. Esses hospedeiros
são fábricas de alimentos para as larvas dessas vespas: eles são
comidos vivos por dentro, a começar pelos órgãos não vitais. Embora
paralisadas (ou semiparalisadas, pois é comum observar o grilo ou a
aranha se debatendo enquanto suas entranhas são devoradas), o animal
não se encontra anestesiado, de modo que ele sente dor durante todo
ou a maior parte do processo, que dura dias, às vezes semanas.

Outra sacanagem divina que vi foi há uns cinco anos atrás: uma
mulher pobre conduzindo sozinha e de ônibus (ônibus lotado) seus
dois filhos autistas "severos" à Casa da Esperança, uma instituição
criada por pais e amigos dos autistas. As crianças gritavam,
urravam, "incomodavam". Muita gente insensível (ou brutalizadas pela
ignorância) riam e/ou reclamavam. Jamais me esquecerei do rosto
daquela mãe.

É, a natureza é complexa, mas não é perfeita. E se Deus escreve
certo com linhas tortas, sinceramente, êita caligrafiazinha ruim!

Abraços,
Manuel Bulcão





##### ##### #####

Para saber mais visite
http://www.ciencialist.hpg.ig.com.br


##### ##### ##### #####


Yahoo! Grupos, um serviço oferecido por:







------------------------------------------------------------------------------
Links do Yahoo! Grupos

a.. Para visitar o site do seu grupo na web, acesse:
http://br.groups.yahoo.com/group/ciencialist/

b.. Para sair deste grupo, envie um e-mail para:
ciencialist-unsubscribe@yahoogrupos.com.br

c.. O uso que você faz do Yahoo! Grupos está sujeito aos Termos do Serviço do Yahoo!.



[As partes desta mensagem que não continham texto foram removidas]



SUBJECT: Novo arquivo carregado em ciencialist
FROM: ciencialist@yahoogrupos.com.br
TO: ciencialist@yahoogrupos.com.br
DATE: 20/01/2005 12:41



Olá,

Esta mensagem é uma notificação para informá-lo que um novo arquivo foi adicionado no grupo ciencialist.

Arquivo : /Tabula rasa.doc
Carregado por : oraculo333 <oraculo@atibaia.com.br>
Descrição : Capítulo do livro Tabula Rasa, de Pinker

Você pode acessar o arquivo pela URL:

http://br.groups.yahoo.com/group/ciencialist/files/Tabula%20rasa.doc

Para saber mais sobre compartilhamento de arquivos no grupo, leia:

http://help.yahoo.com/help/br/groups/files

Atenciosamente,

oraculo333 <oraculo@atibaia.com.br>









SUBJECT: Re: Os Dentes da Galinha (era A igreja católica é a favor da evolução?)
FROM: Manuel Bulcão <manuelbulcao@uol.com.br>
TO: ciencialist@yahoogrupos.com.br
DATE: 20/01/2005 12:51


--- Em ciencialist@yahoogrupos.com.br, "Oraculo" <oraculo@a...>
escreveu
> Olá Manuel
>
> E eu já havia esquecido, mas postei o livro Os Dentes da Galinha
nos arquivos da lista, basta entrar e baixar para ler. É um livro
curto, rápido de ler e bem instrutivo..:-)

Oi Homero

Valeu sua iniciativa!

Esse opúsculo "Os Dentes da Galinha" é uma seleção de ensaios de um
livro mais robusto do Jay Gould: "Quando as Galinhas Tiverem
Dentes". O ensaio em questão é "Natureza Amoral".

Abraços,
Manuel Bulcão





SUBJECT: Re: A igreja católica é a favor da evolução?
FROM: "rmtakata" <rmtakata@altavista.net>
TO: ciencialist@yahoogrupos.com.br
DATE: 20/01/2005 13:09


--- Em ciencialist@yahoogrupos.com.br, Manuel Bulcão
> Takata: Ue' mesmo hoje - com sua capacidade migratoria plenamente
> desenvolvida - milhoes de salmoes morrem antes de completar sua
> jornada. Sacanagem divina? Ou a mortandade de milhoes de animais
> inocentes eh cruel apenas de for por meio da selecao natural, mas
> passa a ser um sacrificio beatifico se eh por ordem de Deus? Eh vai
> ver q. eh, afinal
>
> Manuel: A pior "sacanagem divina" que conheço é a protagonizada
> pelas "moscas icnêumones", que na verdade não são moscas, mas um
> gênero de vespa que possui mais espécies que todos os vertebrados
> juntos.

Bom deixar claro, ja' q. certamente ha' pessoas religiosas nesta
lista, q. a minha linha de argumentacao acima nao eh nem no sentido de
concluir q. deus (ou outra divindade) seja cruel, nem q. ele nao
exista. Eh tao somente uma contra-argumentacao a uma linha de
argumentacao q. considero totalmente equivocada.

Respeito os sentimentos religiosos de cada um e nao quero ferir
sensibilidades. Mas nao devemos deixar esses sentimentos distorcerem
os fatos ou as interpretacoes sobre os fatos. (Eh dificil, mas temos
q. agir assim se nao quisermos ser levianos.)

[]s,

Roberto Takata





SUBJECT: Re: A igreja católica é a favor da evolução?
FROM: Manuel Bulcão <manuelbulcao@uol.com.br>
TO: ciencialist@yahoogrupos.com.br
DATE: 20/01/2005 14:22


--- Em ciencialist@yahoogrupos.com.br, "rmtakata" <rmtakata@a...>
escreveu
> Bom deixar claro, ja' q. certamente ha' pessoas religiosas nesta
lista, q. a minha linha de argumentacao acima nao eh nem no sentido
deconcluir q. deus (ou outra divindade) seja cruel, nem q. ele nao
exista.

Manuel: Mas, penso eu, sua linha de argumentação conclui que Deus,
se Ele realmente existe, é um tanto indiferente com relação às
criaturas que forja, tão indiferente como o acaso das mutações e a
seleção natural.

Aliás, essa idéia acerca da indiferença divina passou pela cabeça de
um certo homem, homem este que a mitologia cristã considera o Filho
de Deus: "Senhor, Senhor, por que me abandonaste" (Mateus, XXVI, 46).

Eis o que mais admiro no cristianismo, a permissão de questionar os
atos e os desígnios divinos, patenteada neste e em outros lamentos
de Cristo (Pai, afasta de mim este cálice). alguns antropólogos e
psicólogos, entre eles Erich Fromm, afirmam que o culto do deus-
filho surgiu e foi alimentado por um desapontamento ou ressentimento
com relação ao deus-pai, o que lembra o "conflito de gerações".
Talvez o mesmo se possa dizer com relação ao "ateísmo" da religião
budista.

Em um ensaio que escrevi, há uma trecho que diz: <<Certamente, posso
ainda acreditar no deus "razão-de-ser" de Leibniz, que explica por
que existe algo e não nada. O problema é que, infelizmente, não vejo
como inferir do princípio da razão suficiente uma norma ética
fundamental que nos dê alento. Vale dizer, se "Isso" é Deus, não há
nada que nos garanta que Ele ou "Isso" tenha um interesse por nós,
uma preocupação pela nossa felicidade ou destino. Diante dessa
divindade, sinto-me como aquele polichinelo do poema de
Baudelaire "O Bobo e a Vênus": ele, o bufão triste, prostrado aos
pés da deusa da beleza, clama por piedade; enquanto ela, a Vênus
implacável, apenas "olha para longe, vagamente, com os seus olhos de
mármore.">>

[]s
Manuel Bulcão













SUBJECT: Re: A igreja católica é a favor da evolução?
FROM: Manuel Bulcão <manuelbulcao@uol.com.br>
TO: ciencialist@yahoogrupos.com.br
DATE: 20/01/2005 14:23


--- Em ciencialist@yahoogrupos.com.br, "rmtakata" <rmtakata@a...>
escreveu
> Bom deixar claro, ja' q. certamente ha' pessoas religiosas nesta
lista, q. a minha linha de argumentacao acima nao eh nem no sentido
deconcluir q. deus (ou outra divindade) seja cruel, nem q. ele nao
exista.

Manuel: Mas, penso eu, sua linha de argumentação conclui que Deus,
se Ele realmente existe, é um tanto indiferente com relação às
criaturas que forja, tão indiferente como o acaso das mutações e a
seleção natural.

Aliás, essa idéia acerca da indiferença divina passou pela cabeça de
um certo homem, homem este que a mitologia cristã considera o Filho
de Deus: "Senhor, Senhor, por que me abandonaste" (Mateus, XXVI, 46).

Eis o que mais admiro no cristianismo, a permissão de questionar os
atos e os desígnios divinos, patenteada neste e em outros lamentos
de Cristo (Pai, afasta de mim este cálice). alguns antropólogos e
psicólogos, entre eles Erich Fromm, afirmam que o culto do deus-
filho surgiu e foi alimentado por um desapontamento ou ressentimento
com relação ao deus-pai, o que lembra o "conflito de gerações".
Talvez o mesmo se possa dizer com relação ao "ateísmo" da religião
budista.

Em um ensaio que escrevi, há uma trecho que diz: <<Certamente, posso
ainda acreditar no deus "razão-de-ser" de Leibniz, que explica por
que existe algo e não nada. O problema é que, infelizmente, não vejo
como inferir do princípio da razão suficiente uma norma ética
fundamental que nos dê alento. Vale dizer, se "Isso" é Deus, não há
nada que nos garanta que Ele ou "Isso" tenha um interesse por nós,
uma preocupação pela nossa felicidade ou destino. Diante dessa
divindade, sinto-me como aquele polichinelo do poema de
Baudelaire "O Bobo e a Vênus": ele, o bufão triste, prostrado aos
pés da deusa da beleza, clama por piedade; enquanto ela, a Vênus
implacável, apenas "olha para longe, vagamente, com os seus olhos de
mármore.">>

[]s
Manuel Bulcão





SUBJECT: Re: A igreja católica é a favor da evolução?
FROM: "rmtakata" <rmtakata@altavista.net>
TO: ciencialist@yahoogrupos.com.br
DATE: 20/01/2005 15:15


--- Em ciencialist@yahoogrupos.com.br, Manuel Bulcão
> Manuel: Mas, penso eu, sua linha de argumentação conclui que Deus,
> se Ele realmente existe, é um tanto indiferente com relação às
> criaturas que forja, tão indiferente como o acaso das mutações e a
> seleção natural.

Sim, seria, a se valer da mesma linha torpe q. tentou fazer crer q. a
selecao natural nao poderia operar por seu mecanismo cruel ou
perdulario. Eh na onda: gde coisa, eh o roto falando do rasgado.

O q. nao quer dizer q., sob outro ponto de vista, nao possa existir um
deus talvez piedoso (na minha opiniao). (Nao me perguntem como isso
seria possivel, sou anhostico. Apenas nao ficaria torrando a paciencia
dos religiosos sobre esse ponto - claro, desde q. nao queiram forcar a
barra e dizer bobagens como a presente na pagina citada com a morte
supostamente inutil de salmoes, andorinhas e quetais em etapas
intermediarias.)

[]s,

Roberto Takata





SUBJECT: Re: [ciencialist] Re: Unidades de medida: símbolo e nome da grandeza
FROM: "Silvio" <scordeiro@terra.com.br>
TO: <ciencialist@yahoogrupos.com.br>
DATE: 20/01/2005 18:19

Sergim:
Uma regra gramatical que ocês d´alem mar não conhecem:

FLEXÃO..
Regra 69 - "Todas palavras terminadas em "U" fazem o feminino em "ARA"

Exemplo: bambu - taquara.

-----Mensagem Original-----
De: "Sergio M. M. Taborda" <sergiotaborda@yahoo.com.br>
Para: <ciencialist@yahoogrupos.com.br>
Enviada em: quarta-feira, 19 de janeiro de 2005 23:02
Assunto: [ciencialist] Re: Unidades de medida: símbolo e nome da grandeza




--- Em ciencialist@yahoogrupos.com.br, "E m i l i a n o C h e m e
l l o" <chemelloe@y...> escreveu
> Olá Sérgio,
>
> [Sérgio]
> O Qual é o plural de uma palavra que termina em l ?
>
> mil = mis
> formol = formois.
>
> mol = mois.
>
> Supondo que o nome da unidade , em portugues, é mol - o que não é -
> obtemos o bonito plural : mois.
> Mas como no nome da unidade é mole, o plural é o bonito moles.
> Ah é tão dificil chegar nesta conclusão ...
>
> [Emiliano]
> O inmetro diz que o plural de 'mol' é 'mols'
> http://www.inmetro.gov.br/consumidor/unidLegaisMed.asp

Tudo bem, em roma sê romano. Mas eu não consigo pronunciar mols, nem
pascals ... soa mal ao ouvido portuguesmente falando.
Eu tb não entendo pq vcs chama ions , etc.. quando portuguesmente
falando o correcto é iões.
Veja, quando eu digo correcto, eu estou me referindo às regras de
formação de palavras, que são aquilo que faz uma lingua.
É incorrecto usar estrangeirismos quando a nossa lingua tem palavras
que os podem substituir facilmente. Ainda por cima no caso dos nomes
das particulas que derivam todas do grego tal como o portugues e
portanto não ha razão logica para recorrer ao ingles. Os ingles
tiveram o seu trabalho adpatando o grego que não pertence à linhagem
do ingles, e agora querem adptar a adpatação ? è no minimo, um
trabalho mal feito. Para não dizer desilegante e porco.
Mas como no brasil ninguem quer nem saber, acho que vai ter que
continuar asssim. E em ultima analise os orgãos de qualidade é que
ditam as regras. Não faz nenhum sentido, mas pronto.

Eu só quero deixar claro que cada lingua tem o seu processo de criar
palavras e esse processo é um algoritmo bem estabelecido. é uma das
poucas coisas em que linguistica se assemelha a uma ciencia. Passar
por cima disso é simplesmente idiota, sobretudo no ambito da
ciencia. É como ter um martelo e pregar com uma chave-de-parafusos
pq ela é electrica. É simplesmente absurdo.

> Se a gente for se preocupar com tudo... veja a palavra 'átomo'
então? :-)

Essa não entendi, qual é o problema com "átomos" ?


Sérgio Taborda





##### ##### #####

Para saber mais visite
http://www.ciencialist.hpg.ig.com.br


##### ##### ##### #####
Links do Yahoo! Grupos












SUBJECT: NEUROCIÊNCIA - Dá para sentir?
FROM: "Oraculo" <oraculo@atibaia.com.br>
TO: <ciencialist@yahoogrupos.com.br>
DATE: 20/01/2005 19:11

http://nationalgeographic.abril.uol.com.br/edicoes/0501/recursos/index.html



NEUROCIÊNCIA


Dá para sentir?

Não é difícil enganar o sentido do tato

Diante dos apêndices que saem de nossos ombros, pensamos: esses são meus braços. Aquelas ferramentas com vários dedos na extremidade são minhas mãos. Aliás, estou precisando cortar as unhas. E assim por diante. A percepção que temos da posição de nosso corpo no espaço é chamada de propriocepção.

Os cientistas descobriram que as orientações proprioceptivas podem ser enganadas. Henrik Ehrsson realizou um experimento no qual um voluntário foi posto em uma máquina de ressonância magnética com sua mão direita apoiada na perna, sob uma superfície rígida; uma mão artificial, de borracha, foi colocada sobre a mesma superfície. Um pesquisador usou um pincel para tocar a mão de verdade, que o voluntário não podia ver, ao mesmo tempo que tocava no dedo correspondente da mão de borracha, que estava visível.

Foram necessários apenas 15 segundos para que os voluntários ficassem convencidos, de maneira geral, de que a de borracha era sua verdadeira mão. Eles tentavam esquivar-se quando Ehrsson ameaçava dar um soco na mão de borracha. E ficavam surpresos quando percebiam que não conseguiam mover os dedos de borracha. Embora tivessem consciência do que estava ocorrendo, nenhum esforço de pensamento racional conseguia desfazer a ilusão sensorial.

"Não se trata apenas de raciocinar", diz Ehrsson. "Mas, sim, de sentir. Não é algo que se dissipe com o pensamento."

Ehrsson não é o primeiro a comprovar que a propriocepção pode ser enganada. Foi o primeiro, no entanto, a usar a tomografia para verificar quais são as regiões do cérebro que se mantêm ativas durante o experimento. Segundo ele, a sensação de posse do próprio corpo é controlada pelo córtex pré-motor, uma área do cérebro que integra a visão e os movimento corporais.

É uma abordagem científica e experimental de uma questão que vem sendo discutida há séculos pelos filósofos. Em que medida podemos de fato comprovar que nós e o mundo ao nosso redor são reais? Embora, no século 17, René Descartes tenha afirmado que "Penso, logo existo", isso não prova de maneira cabal que ele não era apenas um cérebro boiando no laboratório de um cientista enlouquecido. No século 18, o filósofo George Berkeley retomou o debate ao propor uma teoria idealista, segundo a qual o mundo real existe apenas em virtude de nossa percepção dele - ou seja, que os objetos materiais existem apenas como concepções, ou idéias, em nossa mente.

Nossa suposição mais forte, porém, é de que o mundo existe mesmo e não é ilusão. Mas não podemos evitar o fato de que muitas de nossas percepções são elaboradas internamente. É como um filme que está sendo realizado, editado e, por vezes, censurado por um diretor, cheio de idiossincrasias, instalado dentro do nosso crânio.

- Por Joel Achenbach


[As partes desta mensagem que não continham texto foram removidas]



SUBJECT: Aminas heterocíclicas e cancro (cancer)/CERVEJOLAS
FROM: Maria Natália <grasdic@hotmail.com>
TO: ciencialist@yahoogrupos.com.br
DATE: 21/01/2005 00:58


Uma notícia boa para os adoradores de...cerveja não alcoolica:
Arimoto-Kobayashi na Universidade de Okayama,Japão, verificou que em
ratos bebedores de cerveja sem álcool a % de cancro era inferior ás
do que bebiam água. Pensa-se que este efeito é devido à existência
de um composto desconhecido e que permitiria não se quebrar a cadeia
das aminas existente nos alimentos cozinhados.
Que o estômago é um tubo de ensaio já dúvidas não temos É uma
questão que já sabe sobre o consumo de aminas ( presentes no vinho)
e ainda nitritos e nitratos presentes no presunto ou comida salgada
e que é mutagénica:
http://www.pharm.okayama-u.ac.jp/lab/genetic/arimotoE.html
Interessa também saber se a cerveja normal tem o mesmo efeito.
Há que fazer um balanço sobre as vantagens e riscos de se beber
moderadamente.
Para saberem mais:
http://www.newscientist.com/channel/being-human/drugs-alcohol
http://pubs.acs.org/journals/jafcau/index.html

Resumo da notícia:
Mystery compound in beer fights cancer
· 19:00 19 January 2005
· Exclusive from New Scientist Print Edition
· Andy Coghlan
Some cancers are caused by heterocyclic amines, DNA-damaging
chemicals found in cooked meat and fish. When Sakae Arimoto-
Kobayashi's team at Okayama University in Japan fed these chemicals
to mice, the DNA damage to their liver, lungs and kidneys was
reduced by up to 85% if the mice drank non-alcoholic beer instead of
water.
Arimoto-Kobayashi thinks as-yet unidentified compounds in lager and
stout prevent the amines binding to and damaging DNA. If these
compounds can be identified, brewers might be able to produce beers
particularly rich in them, or they could be added to foods.
Heavy alcohol consumption is blamed for around 6% of all cancers in
western countries (New Scientist print edition, 18 December 2004),
though moderate consumption reduces the risk of heart disease. Since
the mice drank non-alcoholic beer, the findings do not show whether
moderate consumption of normal beer has any anti-cancer
benefits. "The total benefits and risks of beer with alcohol are
still under consideration," says Arimoto-Kobayashi.
Journal reference: Journal of Agricultural and Food Chemistry (DOI:
10.1021/jf049208k)
Related Articles
· Genetic variation gives a taste for alcohol
· 15 November 2004
· One drink per day boosts breast cancer risk
· 12 November 2002
Weblinks
· Sakae Arimoto-Kobayashi, Okayama University
· Journal of Agricultural and Food Chemistry
· Drugs and Alcohol, New Scientist "
sds
Maria Natália









SUBJECT: Re: Unidades de medida: símbolo e nome da grandeza
FROM: Maria Natália <grasdic@hotmail.com>
TO: ciencialist@yahoogrupos.com.br
DATE: 21/01/2005 01:02


Silvinho:
Ubu ===>> capivara
Obrigadim pela lição
Maria Natália


--- Em ciencialist@yahoogrupos.com.br, "Silvio" <scordeiro@t...>
escreveu
> Sergim:
> Uma regra gramatical que ocês d´alem mar não conhecem:
>
> FLEXÃO..
> Regra 69 - "Todas palavras terminadas em "U" fazem o feminino
em "ARA"
>
> Exemplo: bambu - taquara.
>
> -----Mensagem Original-----
> De: "Sergio M. M. Taborda" <sergiotaborda@y...>
> Para: <ciencialist@yahoogrupos.com.br>
> Enviada em: quarta-feira, 19 de janeiro de 2005 23:02
> Assunto: [ciencialist] Re: Unidades de medida: símbolo e nome da
grandeza
>
>
>
>
> --- Em ciencialist@yahoogrupos.com.br, "E m i l i a n o C h e m
e
> l l o" <chemelloe@y...> escreveu
> > Olá Sérgio,
> >
> > [Sérgio]
> > O Qual é o plural de uma palavra que termina em l ?
> >
> > mil = mis
> > formol = formois.
> >
> > mol = mois.
> >
> > Supondo que o nome da unidade , em portugues, é mol - o que não
é -
> > obtemos o bonito plural : mois.
> > Mas como no nome da unidade é mole, o plural é o bonito moles.
> > Ah é tão dificil chegar nesta conclusão ...
> >
> > [Emiliano]
> > O inmetro diz que o plural de 'mol' é 'mols'
> > http://www.inmetro.gov.br/consumidor/unidLegaisMed.asp
>
> Tudo bem, em roma sê romano. Mas eu não consigo pronunciar mols,
nem
> pascals ... soa mal ao ouvido portuguesmente falando.
> Eu tb não entendo pq vcs chama ions , etc.. quando portuguesmente
> falando o correcto é iões.
> Veja, quando eu digo correcto, eu estou me referindo às regras de
> formação de palavras, que são aquilo que faz uma lingua.
> É incorrecto usar estrangeirismos quando a nossa lingua tem
palavras
> que os podem substituir facilmente. Ainda por cima no caso dos
nomes
> das particulas que derivam todas do grego tal como o portugues e
> portanto não ha razão logica para recorrer ao ingles. Os ingles
> tiveram o seu trabalho adpatando o grego que não pertence à
linhagem
> do ingles, e agora querem adptar a adpatação ? è no minimo, um
> trabalho mal feito. Para não dizer desilegante e porco.
> Mas como no brasil ninguem quer nem saber, acho que vai ter que
> continuar asssim. E em ultima analise os orgãos de qualidade é que
> ditam as regras. Não faz nenhum sentido, mas pronto.
>
> Eu só quero deixar claro que cada lingua tem o seu processo de
criar
> palavras e esse processo é um algoritmo bem estabelecido. é uma das
> poucas coisas em que linguistica se assemelha a uma ciencia. Passar
> por cima disso é simplesmente idiota, sobretudo no ambito da
> ciencia. É como ter um martelo e pregar com uma chave-de-parafusos
> pq ela é electrica. É simplesmente absurdo.
>
> > Se a gente for se preocupar com tudo... veja a palavra 'átomo'
> então? :-)
>
> Essa não entendi, qual é o problema com "átomos" ?
>
>
> Sérgio Taborda
>
>
>
>
>
> ##### ##### #####
>
> Para saber mais visite
> http://www.ciencialist.hpg.ig.com.br
>
>
> ##### ##### ##### #####
> Links do Yahoo! Grupos





SUBJECT: serie harmonica
FROM: "Luiz Ferraz Netto" <leobarretos@uol.com.br>
TO: "ciencialist" <ciencialist@yahoogrupos.com.br>
DATE: 21/01/2005 06:44

Oi pessoal,
Como provar que a série harmônica diverge, sem utilizar o teste da Integral?
[]'
===========================
Luiz Ferraz Netto [Léo]
leobarretos@uol.com.br
http://www.feiradeciencias.com.br
===========================


----------

No virus found in this outgoing message.
Checked by AVG Anti-Virus.
Version: 7.0.300 / Virus Database: 265.6.13 - Release Date: 16/01/2005


[As partes desta mensagem que não continham texto foram removidas]



SUBJECT: Re: Unidades de medida: símbolo e nome da grandeza
FROM: "Sergio M. M. Taborda" <sergiotaborda@yahoo.com.br>
TO: ciencialist@yahoogrupos.com.br
DATE: 21/01/2005 08:04


--- Em ciencialist@yahoogrupos.com.br, "rmtakata"

> > Veja, quando eu digo correcto, eu estou me referindo às regras de
> > formação de palavras, que são aquilo que faz uma lingua.
>
> A língua se faz com muito mais do q. regras. As regras em geral são
> derivadas e depois tenta-se impô-la como uma camisa de força. Certo q.
> é necessário q. haja regras de modo a garantir um mínimo de unidade.
> Mas ela não consegue atuar de modo absoluto. A língua é dinâmica posto
> q. ela é um instrumento de comunicação interpessoal - influi a
> cultura, o ambiente físico (sim! pense no seguinte - em um lugar
> barulhento, a tendência é aumentar a intensidade da voz, isso altera o
> modo como pronunciamos as palavras), a história, a conexão com os
> vizinhos. (Sem falar nas diferenças entre as pessoas: nível
> socioeconômico, idade, sexo e assim por diante.) As alterações
> dinâmicas da língua podem ser freadas, mas não contidas. (Se duvida,
> leia Camões em sua grafia original. Talvez ele dissesse q. os
> portugueses hodiernos desfiguraram a pureza da recém-emergida língua
> lusa.)

Isso é válido para a lingua falada e não para a lingua escrita.
A lingua escrita tende a ser mais complexa (mais cheia de regras) que
a lingua falada.
Em portugues não usamos apostofes- que outrora eram usados - como
ainda fazem os ingleses e os franceses. Camões os usa e toran a sua
escrita ilegivel. Mas se vc os tirar a poesia perde a rima.
A lingua escrita dos ingleses e franceses é mais semelhante À sua
lingua falada. Contudo, em portugues isso não é assim, e por isso
muitos estrageiros se quixam que o portugues é dificil.
As palavras homografas provam que o falado é mais rico e o escrito
mais estruturado.
A linguagem cientifica tende a ser mais clara que a linguagem comum, e
isso torna-a mais perto da lingua escrita do que da falada. Não
interessa se ion soa melhor que ião. Interessa de onde se origina a
palavra e como ela é transportada para o portugues. Embora a origem
seja o grego ion ( se não me engano significa "errante"), ao passar
para o portugues ele ganha um ão. vc não diz con, e sim cão. não diz
mon, mas mão. Então não deveria escrever ion, e sim ião.
Tem que haver coerencia.
Existem linguas mais preocupadas com a coerencia do que outras e
portugues não é uma delas. Não ha um instituto que chame a si cuidar
desses assuntos, como ha na espanha por exemplo (A Real Academia das
Linguas). Aqui os dicionários não são fundados em ciencia (historia,
linguistica , etc..) e sim em palpite. Embora os dicionários
brasileiros digam o mesmo que os dicionarios portugueses , os
brasileiros tendem a ser mais , digamos, superficiais. Preocupados
apenas com o significado da palavra em vez de com a palavra em si,
como um todo. (grafia, fonetica, semantica)

> Não creia q. o português de Portugal é o mais correto (ou correcto).
> Tampouco é o menos correto.

Não estou falando isso. Estou falando que a lingua portuguesa, seja a
falada aqui ou lá , tem regras e essas regras devem ser seguidas.
Se vc me fala que as regras são alteradas no tempo por efeitos de
ambiente e de circusntancia, tudo bem. Mas enqaunto elas valem, elas
valem e devem ser cumpridas. ESPECIALMENTE - e este é que é o ponto -
em se tratando de ciencia , onde as coisas devem ser o mais imparciais
possivel (universais) pelo bem do entendimento comum.

> (Se se preocupa com os estrangeirismos. Golo poderia ser substituído
> perfeitamente por tento, ponto. Apenas não seria a mesma coisa q. o
> goal do futebol...)

Golo é um neologismo, necessária peloa influencia do desposto nos
costumes. Em portugual não se usam palavras ingleses em futebol
excepto "penalti". Golo é um neologismos enquanto penalti é um
estrangeirismo. O pernality (penalidade) virou penalti (palavra da
giria futubolistica) e goal (objectivo) virou golo (palavra da giria)

A giria , é uma sublinguagem e isso não me preocupa. Poderiamos
argumentar que a linguagem cientifica tb é uma giria. A diferença é
que a linguagem cientifica tem obrigação de ser clara enquanto as
outras não têm.

A adptação de palavrasestrangeiras à nossa lingua é a coisa mais feia
que existe, mas dentro de uma giria isso até é desculpável pq não
atinge todo o mundo. Mas a linguagem cientifica, por ser tecnica e por
ser cientifica (impacial e independente) não deve sucumbir a essas
coisas.

Independente da região ou do pais, mãe é mãe e pai é pai, então pq ion
não é ião ou enqüanto não é enquanto. Repara que a regra do trema é
futil pq em protugual dizemos as mesmas palavras sem precisar dele.

E para acabar. Tenho reparado que aqui no brasil existe bastante
poluição visual nas ruas com inumeros cartazes fazendo propaganda de
tudo e mais alguma coisa. O pior, quanto a mim, é que muitos deses
anuncios contém erros grassos de escrita e às vezes de gramática.
Em portugual , podemos ter um indice de analfabetismo de 80% mas as
pessoas preocupam-se em escrever bem coisas que centenas de outras
iram ler. Falta uma preocupação, um repeito pela lingua.
Ningem é perfeito e todos podemos cometer erros ao escrever, mas num
anuncio , num cartaz , que passou por várias pessoas , que várias
pessoas leêm e leram antes de ser colocado e não se encontra o erro
?!!? Isso é , no minimo, estranho.

O mais estranho que vi foi escrito "Comida cazeira". Tudo bem que se
confunda cozido com cosido, mas casa é casa , bolas! comida de casa é
comida caseira. Claro que vc poderá dizer que se baseia na forma da
palavra tal como era usada no seculo 16 , mas não vivemos no seculo
16. Não ha razão para escrever mal.

Pior que isso é que aqui se da muito mais ventilação aos problemas da
lingua como certos programas de certos profesores na TV cultura. Mas
de que serve aquilo tudo se ninguem tem vergonha de escrever cartazes
e panfletos de forma errada!

Tlv pq o meu pai é Tipografo estas coisas sempre foram importantes
para mim. è normal que a pessoa chegue e diga : quero um cartaz com
esta frase , ou quero um planfleto com este texto. E que ele contenha
erros. O que não é normal é que durante o processo de fabricação
ninguem descubra que ha um erro. Bolas, existem dicionários para isso
mesmo. Usem-os.

Quanto ao uso da lingua em ciencia, as coisas estão regradas por
institutos de qualidade e regras. E devemos usá-las. Mas até as regras
podem estar erradas, ou pelo menos em conflito com outras regras
"maiores". Temos pq aqui jornalistas, cuja vida é escrever. O que ha a
fazer num caso como este ?

Sérgio Taborda






SUBJECT: Re: Unidades de medida: símbolo e nome da grandeza
FROM: "Sergio M. M. Taborda" <sergiotaborda@yahoo.com.br>
TO: ciencialist@yahoogrupos.com.br
DATE: 21/01/2005 08:07


--- Em ciencialist@yahoogrupos.com.br, "rmtakata" <rmtakata@a...> escreveu

> > Raramente tive que expressar meus calculos em unidades que não as do
> > SI ( MKS como é chamado tb , por semelhança a CGS , notem que as
>
> Na verdade o SI é um desenvolvimento do MKS. Um teórico realmente
> raras terá de se defrontar com unidades q. não o SI.

Um teorico raramente usa unidades. Por isso que as teorias acabam não
fazendo sentido muitas vezes. Analise dimencional é uma coisa que os
teoricos fazem no fim e que qq pessoa deve fazer no inicio, no meio e
no fim. Os teoricos usam o esquema de fazer todas as constantes igual
a 1. o que significa cria formulas adimensionais.

Já um prático
> terá q. se defrontar com unidades como onças (troy e averdupois),
> libras, polegadas, pés - tudo por causa do comércio internacional e os
> americanos e ingleses sentem-se muito confortáveis com suas medidas
> medievais (ou mais antigas ainda).
>
> > mas não me lembro de ter nenhum problema em calcular as mols das
> > soluções.
>
> A questão não é calcular, mas representar a unidade.

A questão não é o que está escrito, mas sim o que ele representa. O
objectivo foi mostrar que pelo contexto é muito simples saber se deve
usar mols ou mol. "calcular as mols" "x = y a/ z (mol)"

Sérgio Taborda






SUBJECT: Re: Unidades de medida: símbolo e nome da grandeza
FROM: "Sergio M. M. Taborda" <sergiotaborda@yahoo.com.br>
TO: ciencialist@yahoogrupos.com.br
DATE: 21/01/2005 08:21


--- Em ciencialist@yahoogrupos.com.br, "E m i l i a n o C h e m e l
l o" <chemelloe@y...> escreveu
> > Se a gente for se preocupar com tudo... veja a palavra 'átomo'
> então? :-)
>
> [Sérgio]
> Essa não entendi, qual é o problema com "átomos" ?
>
> [Emiliano]
> Ops... eu derivei o assunto sem avisar antes. Me referi ao 'sentido de
> átomo' e a problemática de sua origem com relação ao que hoje conhecemos
> dele.

Mais um hoax escolar. Os gregos chegaram ao conceito de atomo, que
significa Indivisivel. O conceito do indivisivel ainda existe hoje. As
particulas elementares (não compostas, e portanto não divisiveis) é o
conceito de Indivisivel dos gregos. As particulas elementares de hoje,
são o real conceito dos atomos gregos.
O conceito é excelente. O problema é o reconheicmento do conceito na
natureza. Para os gregos o conceito parava nas moleculas. Sabão tinha
um indivisivel diferente que agua, e eles de madeira, e assim por
diante. Eram indivisiveis macroscopicos.
Com a invensão das lents de aumento e do microsopio muitos cairam por
terra. A quimica chegou nos elementos. Os atomos, do ponto de vista
quimico são indivisiveis. Poderiamos ter chamado outra coisa aos
elementos quimicos mais primordiais (elementos ja significa que não
podem ser divididos). Os atomos quimicos são indivisiveis do ponto de
vista quimico, mas não em geral, da mesma forma que a agua é
indivisivel do ponto de vista macroscopico, mas não do quimico.
A fisica então dividio o indivisivel quimico, como a quimico o
indivisivel macroscopico, obtendos os seus ppr indivisiveis (as
particulas elementares). A cada passo uma nova ciencia surge para
destruir os indivisiveis da ciencia anteriror e isso se chama evolução.

O importante nesta historia é que os gregos não se enganaram no
conceito de indivisivel , o conceito é válido até hoje. O que eles se
enganaram foi no uso do conceito em coisas reais. Tal como os quimicos
se enganaram , tal como os fisicos se enganam, etc..
Nenhum fisico lhe dirá que o electrão é divisivel. Contudo ele pode
muito bem ser.
É um erro grasso, um hoax disfarçado , cometido e prepertado nas
escolas ensinar que o atomo , ao contrario do que os cregos disiam, é
divisivel sim. Ora, os gregos nem sequer tinham o conceito de elemento
quimico, muito menos de atomo quimico. O que eles tinham , isso, sim,
era oconceito de que as coisas eram divisiveis em particulas menores,
mas apenas até um certo ponto, a partir do qual não poderiamos dividir
mais e teriamos encontrado particulas indivisiveis, atomos.
Só que a ciencia ainda não chegou nesse ponto. Estamos tentando Às
mais de 5000 anos. (se bem que apenas nos ultimos 10 seculos tentamos
realmente)

Tentar culpar os gregos, ou pintá-los de inocentes ou idiotas é um
desmérito para quem o afirma.

Sérgio Taborda

Os povos atigos eram muito mais inteligentes e tenham muito mais
tecnologia do que imainamos/ sabemos / queremos admitir. Não sei pq
esse fenomeno, mas ele é prepétuado noas escolas todos os dias, como
se os antigos fossem mais burros que os modernos.







SUBJECT: Re: Unidades de medida: símbolo e nome da grandeza
FROM: "Sergio M. M. Taborda" <sergiotaborda@yahoo.com.br>
TO: ciencialist@yahoogrupos.com.br
DATE: 21/01/2005 08:28


--- Em ciencialist@yahoogrupos.com.br, "Silvio" <scordeiro@t...> escreveu
> Sergim:
> Uma regra gramatical que ocês d´alem mar não conhecem:
>
> FLEXÃO..
> Regra 69 - "Todas palavras terminadas em "U" fazem o feminino em "ARA"


cu => cara


Sérgio Taborda





SUBJECT: Re: serie harmonica
FROM: "rmtakata" <rmtakata@altavista.net>
TO: ciencialist@yahoogrupos.com.br
DATE: 21/01/2005 08:30


--- Em ciencialist@yahoogrupos.com.br, "Luiz Ferraz Netto"
> Como provar que a série harmônica diverge, sem utilizar o teste da
> Integral?

Considere a soma dos termos da serie harmonica:

S = 1 + 1/2 + 1/3 + 1/4 + 1/5 + 1/6 + 1/7 + 1/8 + 1/9 + 1/10 + 1/11 +
1/12 + 1/13 + 1/14...

Poderemos agrupar os termos do seguinte modo, a partir de 1/3
(inclusive) reuna os termos na sequencia 2, 4, 8, 16...:

S = 1 + 1/2 + (1/3 + 1/4) + (1/5 + 1/6 + 1/7 + 1/8) + (1/9 + 1/10 +
1/11 + 1/12 + 1/13 + 1/14 + 1/15 + 1/16)...

Cada soma entre parenteses sempre dara' uma soma maior do q. 1/2.

Para provar isso, substitua os termos anteriores de cada parenteses
pelo ultimo termo do mesmo parenteses:

(1/3 + 1/4) > (1/4 + 1/4) = 2*1/4 = 1/2
(1/5 + 1/6 + 1/7 + 1/8) > (1/8 + 1/8 + 1/8 + 1/8) = 4*1/8 = 1/2
(1/9 + 1/10 + 1/11 + 1/12 + 1/13 + 1/14 + 1/15 + 1/16) > 8*1/16 = 1/2

Assim:

S > 1 + 1/2 + 1/2 + 1/2 + 1/2...

Como a soma de 1 com infinitos 1/2 diverge (basta juntar o 1/2 aos pares):

S > 1 + 1 + 1 + 1 + 1...

A soma de infinitos termos da serie harmonica tb diverge.

Essa eh a prova de Oresme, redescoberta independentemente por Bernoulli.

[]s,

Roberto Takata





SUBJECT: Re: Unidades de medida: símbolo e nome da grandeza
FROM: "rmtakata" <rmtakata@altavista.net>
TO: ciencialist@yahoogrupos.com.br
DATE: 21/01/2005 09:09


--- Em ciencialist@yahoogrupos.com.br, "Sergio M. M. Taborda"
> Isso é válido para a lingua falada e não para a lingua escrita.

Para as duas. Realmente a falada eh mais dinamica, mas a escrita acaba
indo a reboque. Nao demora - jah houve propostas serias - e os
pronomes obliquos, q. na fala de Portugal sao atonos: me, te, se, nos,
vos, mas q. no Brasil sao tonicos, poderao perfeitamente ser usados no
inicio de frases. Me faz um favor. (Mario de Andrade propos a escrita
mi, ti, si - para marcar a tonicidade desses pronomes. Outro Andrade,
o Oswald, ironizou:

Dê-me um cigarro
Diz a gramática
Do professor e do aluno
E do mulato sabido
Mas o bom negro e o bom branco
Da Nação Brasileira
Dizem todos os dias
Deixa disso camarada
Me dá um cigarro)

> Em portugues não usamos apostofes- que outrora eram usados - como
> ainda fazem os ingleses e os franceses.

Usamos em expressoes como pingo-d'água, tromba-d'água...

> Não interessa se ion soa melhor que ião. Interessa de onde se
> origina a palavra e como ela é transportada para o portugues.

Interessa e muito. O modo como ela eh transliterada para outras
linguas depende da estrutura fonetica da lingua. Assim os ingleses
escrevem: rationalise, os americanos: rationalize.

> para o portugues ele ganha um ão. vc não diz con, e sim cão. não diz
> mon, mas mão.

Ate' por q. cão vem de canis e mão vem de manus - ambos do latim. Íon
(ou ião em Portugal) vem de ion mesmo - do grego, via inglês ou alemão
ou francês. Sim, escrevemos panteão, mas neon, freon, náilon (ou
nylon), cíclotron... Vc pede coerência e há uma grande coerência: os
tecnicismos conservam a desinência 'on'.

Ou vai dizer q. não é coerente q. sal tenha como plural sais, mas mal
tenha como plural males? Não é corente com uma regra artificial
rígida, digamos, *sempre* palavras oxítona termina em 'l' faz plural
substituindo o 'l' por 'is'. Isso pode funcionar para línguas
artificiais como esperanto, não para línguas naturais com ampla
abrangência geográfica e diversas fontes de influências.

> Não ha um instituto que chame a si cuidar desses assuntos, como ha
> na espanha por exemplo (A Real Academia das Linguas).

Aqui no Brasil, quem cuida disso eh a Academia Brasileira de Letras -
a ABL.

> Preocupados apenas com o significado da palavra em vez de com a
> palavra em si, como um todo. (grafia, fonetica, semantica)

Acho q. vc nunca leu um Houaiss, um Caldas Aulete ou um Antenor
Nascentes...

> Não estou falando isso. Estou falando que a lingua portuguesa, seja
> a falada aqui ou lá , tem regras e essas regras devem ser seguidas.

Só q. as regras do português do Brasil não implica q. ion dá ião ou
photon dá fotão. Isso funciona em Portugal, não aqui. É como um inglês
reclamar q. os EUA as mãos de ruas são contrárias.

> Golo é um neologismo, necessária peloa influencia do desposto nos
> costumes. Em portugual não se usam palavras ingleses em futebol
> excepto "penalti". Golo é um neologismos enquanto penalti é um
> estrangeirismo. O pernality (penalidade) virou penalti (palavra da
> giria futubolistica) e goal (objectivo) virou golo (palavra da
> giria)

Penalidade tb nao tem a mesma expressividade futebolísitca do q.
pênalti. Outros foram substituídos corner, virou escanteio; forward,
atacante; goal-keeper, goleiro. Outros são insubstituíveis: drible,
time e o proprio futebol (q. ludopedio e balipodo nao seriam capazes
de representar a altura). E isso é coisa de língua falada - a língua
escrita pode apenas tentar representar essas novas palavras de acordo
com os modelos da língua.

> que a linguagem cientifica tem obrigação de ser clara enquanto as
> outras não têm.

Fóton e íon são claríssimos.

> Independente da região ou do pais, mãe é mãe e pai é pai, então pq
> ion não é ião ou enqüanto não é enquanto.

Repare q. banheiro por aqui é diferente do q. significa em Portugal
(q. deve chamar de casa de banhos). O banheiro em Portugal é o
salva-vidas por aqui. Bicha se pega em Portugal, por aqui é fila
(bicha é outra coisa no Brasil). Ião em Portugal e íon no Brasil, não
vejo problemas.

> O mais estranho que vi foi escrito "Comida cazeira".

O mais estranho q. eu vi foi: Vende-se *cocô* verde gelado.

[]s,

Roberto Takata





SUBJECT: Re: Unidades de medida: símbolo e nome da grandeza
FROM: "rmtakata" <rmtakata@altavista.net>
TO: ciencialist@yahoogrupos.com.br
DATE: 21/01/2005 09:17


> > A questão não é calcular, mas representar a unidade.

--- Em ciencialist@yahoogrupos.com.br, "Sergio M. M. Taborda"
> A questão não é o que está escrito, mas sim o que ele representa. O
> objectivo foi mostrar que pelo contexto é muito simples saber se deve
> usar mols ou mol. "calcular as mols" "x = y a/ z (mol)"

A questao e' como se escreve. Como mol e mol* escrevem-se do mesmo
jeito para o aluno pode não ser tão fácil saber o q. representa.

Um exemplo rápido:

Gosto de cerveja.

Quer dizer q. eu gosto de cerveja ou está se referindo ao gosto de
cerveja?

Ou como protestava Aurélio usando como exemplo o poema de Bandeira:

Vai por cinqüenta anos
Que lhes dei a norma:
Reduzi sem danos
A formas a forma.

Formas são a aparência ou são o molde?

[]s,

Roberto Takata





SUBJECT: Re: [ciencialist] Re: Unidades de medida: símbolo e nome da grandeza
FROM: "E m i l i a n o C h e m e l l o" <chemelloe@yahoo.com.br>
TO: <ciencialist@yahoogrupos.com.br>
DATE: 21/01/2005 09:45

Taborda,

O 'conceito' ainda existe sim. Porém, é empregado, desde Thonson se não
me engano, de forma equivocada para representar a 'estrutura' do que se
imagina ser os 'átomos' hoje.

Eu não disse que os gregos se enganaram. Nós, ou melhor, os cientistas
da época do descobrimento de entidades ainda mais elementares do que os
átomos (os elétrons, por exemplo), é que erraram ao 'manter' o nome nome
'átomo'. Imagine um extraterrestre - verde, com antenas, tanto faz -
chegando na Terra. Não entenderia porque algo que é divisível tem como nome
'átomo', que significa exatamente o contrário.

Quanto a sua afirmação:

[Sérgio]
Os povos atigos eram muito mais inteligentes e tenham muito mais tecnologia
do que imainamos/ sabemos / queremos admitir. Não sei pq esse fenomeno, mas
ele é prepétuado noas escolas todos os dias, como se os antigos fossem mais
burros que os modernos

[Emiliano]
Muito mais inteligentes? Eu não diria nem mais nem menos inteligentes do
que aquela época. Para demonstrar o que eu penso, vou colocar uma frase
popular:

"Em terra de cego, quem tem um olho é rei".

Por qualquer descoberta, os cientistas eram idolatrados. Hoje, as
descobertas são bem mais elaboradas do que em épocas passadas. Mas variáveis
se somam a cada dia.

[ ] 's do Emiliano Chemello
emiliano@quimica.net
http://www.quimica.net/emiliano
http://www.ucs.br/ccet/defq/naeq

" Rien ne se perd, rien ne se crée,
tout se transforme."

Antoine Laurent de Lavoisier (químico francês, 1743 - 1794)

----- Original Message -----
From: Sergio M. M. Taborda
To: ciencialist@yahoogrupos.com.br
Sent: Friday, January 21, 2005 8:21 AM
Subject: [ciencialist] Re: Unidades de medida: símbolo e nome da grandeza



--- Em ciencialist@yahoogrupos.com.br, "E m i l i a n o C h e m e l
l o" <chemelloe@y...> escreveu
> > Se a gente for se preocupar com tudo... veja a palavra 'átomo'
> então? :-)
>
> [Sérgio]
> Essa não entendi, qual é o problema com "átomos" ?
>
> [Emiliano]
> Ops... eu derivei o assunto sem avisar antes. Me referi ao 'sentido de
> átomo' e a problemática de sua origem com relação ao que hoje conhecemos
> dele.

Mais um hoax escolar. Os gregos chegaram ao conceito de atomo, que
significa Indivisivel. O conceito do indivisivel ainda existe hoje. As
particulas elementares (não compostas, e portanto não divisiveis) é o
conceito de Indivisivel dos gregos. As particulas elementares de hoje,
são o real conceito dos atomos gregos.
O conceito é excelente. O problema é o reconheicmento do conceito na
natureza. Para os gregos o conceito parava nas moleculas. Sabão tinha
um indivisivel diferente que agua, e eles de madeira, e assim por
diante. Eram indivisiveis macroscopicos.
Com a invensão das lents de aumento e do microsopio muitos cairam por
terra. A quimica chegou nos elementos. Os atomos, do ponto de vista
quimico são indivisiveis. Poderiamos ter chamado outra coisa aos
elementos quimicos mais primordiais (elementos ja significa que não
podem ser divididos). Os atomos quimicos são indivisiveis do ponto de
vista quimico, mas não em geral, da mesma forma que a agua é
indivisivel do ponto de vista macroscopico, mas não do quimico.
A fisica então dividio o indivisivel quimico, como a quimico o
indivisivel macroscopico, obtendos os seus ppr indivisiveis (as
particulas elementares). A cada passo uma nova ciencia surge para
destruir os indivisiveis da ciencia anteriror e isso se chama evolução.

O importante nesta historia é que os gregos não se enganaram no
conceito de indivisivel , o conceito é válido até hoje. O que eles se
enganaram foi no uso do conceito em coisas reais. Tal como os quimicos
se enganaram , tal como os fisicos se enganam, etc..
Nenhum fisico lhe dirá que o electrão é divisivel. Contudo ele pode
muito bem ser.
É um erro grasso, um hoax disfarçado , cometido e prepertado nas
escolas ensinar que o atomo , ao contrario do que os cregos disiam, é
divisivel sim. Ora, os gregos nem sequer tinham o conceito de elemento
quimico, muito menos de atomo quimico. O que eles tinham , isso, sim,
era oconceito de que as coisas eram divisiveis em particulas menores,
mas apenas até um certo ponto, a partir do qual não poderiamos dividir
mais e teriamos encontrado particulas indivisiveis, atomos.
Só que a ciencia ainda não chegou nesse ponto. Estamos tentando Às
mais de 5000 anos. (se bem que apenas nos ultimos 10 seculos tentamos
realmente)

Tentar culpar os gregos, ou pintá-los de inocentes ou idiotas é um
desmérito para quem o afirma.

Sérgio Taborda

Os povos atigos eram muito mais inteligentes e tenham muito mais
tecnologia do que imainamos/ sabemos / queremos admitir. Não sei pq
esse fenomeno, mas ele é prepétuado noas escolas todos os dias, como
se os antigos fossem mais burros que os modernos.







##### ##### #####

Para saber mais visite
http://www.ciencialist.hpg.ig.com.br


##### ##### ##### #####


Yahoo! Grupos, um serviço oferecido por:
PUBLICIDADE





Links do Yahoo! Grupos

Para visitar o site do seu grupo na web, acesse:
http://br.groups.yahoo.com/group/ciencialist/

Para sair deste grupo, envie um e-mail para:
ciencialist-unsubscribe@yahoogrupos.com.br

O uso que você faz do Yahoo! Grupos está sujeito aos Termos do Serviço do
Yahoo!.




SUBJECT: Re: [ciencialist] A igreja católica é a favor da evolução?
FROM: José Renato <jrma@terra.com.br>
TO: <ciencialist@yahoogrupos.com.br>
DATE: 21/01/2005 11:05

Olá, Manuel e Takata, ainda estamos tentando conformar o agente
criador-mantenedor - que geralmente englobamos no vocábulo Deus - às nossas
ingênuas fantasias, em vez de procurar descobrir como isso é ou se apresenta
na realidade. Ficamos frustrados quando não conseguimos enquadrar o deus dos
nossos desejos e necessidades ao Deus que vislumbramos na realidade
observada com isenção e lucidez.
[]s
José Renato
.........................................

From: "Manuel Bulcão" <manuelbulcao@To: <ciencialist@yahoogrupos.com.br>
Sent: Thursday, January 20, 2005 11:16 AM
Subject: [ciencialist] Re: A igreja católica é a favor da evolução?

Oi,

> Quantos salmões se perderam e jamais conseguiram chegar ao riacho
em que haviam nascido para desovar antes que essa estranha
capacidade fosse desenvolvida?

Takata: Ue' mesmo hoje - com sua capacidade migratoria plenamente
desenvolvida - milhoes de salmoes morrem antes de completar sua
jornada. Sacanagem divina? Ou a mortandade de milhoes de animais
inocentes eh cruel apenas de for por meio da selecao natural, mas
passa a ser um sacrificio beatifico se eh por ordem de Deus? Eh vai
ver q. eh, afinal

Manuel: A pior "sacanagem divina" que conheço é a protagonizada
pelas "moscas icnêumones", que na verdade não são moscas, mas um
gênero de vespa que possui mais espécies que todos os vertebrados
juntos.

Os icnêumones injetam seus ovos em outros animais, de preferência
lagartas (larvas de borboleta), afídios e aranhas. Esses hospedeiros
são fábricas de alimentos para as larvas dessas vespas: eles são
comidos vivos por dentro, a começar pelos órgãos não vitais. Embora
paralisadas (ou semiparalisadas, pois é comum observar o grilo ou a
aranha se debatendo enquanto suas entranhas são devoradas), o animal
não se encontra anestesiado, de modo que ele sente dor durante todo
ou a maior parte do processo, que dura dias, às vezes semanas.

Outra sacanagem divina que vi foi há uns cinco anos atrás: uma
mulher pobre conduzindo sozinha e de ônibus (ônibus lotado) seus
dois filhos autistas "severos" à Casa da Esperança, uma instituição
criada por pais e amigos dos autistas. As crianças gritavam,
urravam, "incomodavam". Muita gente insensível (ou brutalizadas pela
ignorância) riam e/ou reclamavam. Jamais me esquecerei do rosto
daquela mãe.

É, a natureza é complexa, mas não é perfeita. E se Deus escreve
certo com linhas tortas, sinceramente, êita caligrafiazinha ruim!

Abraços,
Manuel Bulcão





##### ##### #####

Para saber mais visite
http://www.ciencialist.hpg.ig.com.br


##### ##### ##### #####
Links do Yahoo! Grupos









E-mail classificado pelo Identificador de Spam Inteligente Terra.
Para alterar a categoria classificada, visite
http://www.terra.com.br/centralunificada/emailprotegido/imail/imail.cgi?+_u=jrma&_l=1,1106230643.572019.29596.cabue.terra.com.br,4841,Des15,Des15

Esta mensagem foi verificada pelo E-mail Protegido Terra.
Scan engine: McAfee VirusScan / Atualizado em 19/01/2005 / Versão: 4.4.00 -
Dat 4420
Proteja o seu e-mail Terra: http://www.emailprotegido.terra.com.br/




SUBJECT: Re: Unidades de medida: símbolo e nome da grandeza
FROM: Manuel Bulcão <manuelbulcao@uol.com.br>
TO: ciencialist@yahoogrupos.com.br
DATE: 21/01/2005 11:19


--- Em ciencialist@yahoogrupos.com.br, "rmtakata" <rmtakata@a...>
escreveu
>
> --- Em ciencialist@yahoogrupos.com.br, "Sergio M. M. Taborda"
> > Isso é válido para a lingua falada e não para a lingua escrita.
>
> Para as duas. Realmente a falada eh mais dinamica, mas a escrita
acaba indo a reboque.

Manuel: Estou me lembrando agora daquela pavorosa lagartixa
translúcida e notívaga que em Portugal se chama "osga". Aqui no
Brasil há várias espécies desses répteis da família dos geconídeos,
sendo a mais disseminada a Hemidactylus mabovia que, se não me
engano (o Takata que me corrija) não é autóctone, mas originária da
África. Por aqui, essa lagartixa é popularmente conhecida
como "briba", uma corruptela de "víbora". Ora, "víbora" não é
lagarto, mas serpente. Impressionante o que a arraia miúda iletrada
faz com o vernáculo.

Ocorre que uma nova espécie desse lagarto foi descoberta por
biólogos no Centro-Oeste do País, acho que em Tocantins. Não era,
entretanto, desconhecida pelos nativos da região, que a chamava
de... briba.

Pois bem, essa espécie foi batizada com o nome científico "Briba
brasiliana", por enquanto a única espécie do gênero "briba".

É só consultar o google: "briba brasiliana".

[]s
Manuel Bulcão









SUBJECT: Re: Unidades de medida: símbolo e nome da grandeza
FROM: "rmtakata" <rmtakata@altavista.net>
TO: ciencialist@yahoogrupos.com.br
DATE: 21/01/2005 11:56


--- Em ciencialist@yahoogrupos.com.br, Manuel Bulcão
> Manuel: Estou me lembrando agora daquela pavorosa lagartixa
> translúcida e notívaga que em Portugal se chama "osga". Aqui no
> Brasil há várias espécies desses répteis da família dos geconídeos,
> sendo a mais disseminada a Hemidactylus mabovia que, se não me
> engano (o Takata que me corrija) não é autóctone, mas originária da
> África. Por aqui, essa lagartixa é popularmente conhecida
> como "briba", uma corruptela de "víbora". Ora, "víbora" não é
> lagarto, mas serpente. Impressionante o que a arraia miúda iletrada
> faz com o vernáculo.

O Ora pro nobis (verso da Ave Maria - Ora pro nobis pecatorum), virou
o ora-pro-nobis, uma cactacea! A carne dos pobres.

http://www.melissotroficas.com.br/orapronobis/ora_06.asp

Povim sabe latim!

[]s,

Roberto Takata





SUBJECT: Estranho fenômeno paranormal
FROM: Manuel Bulcão <manuelbulcao@uol.com.br>
TO: ciencialist@yahoogrupos.com.br
DATE: 21/01/2005 14:46


Oi,

Assisti num canal por assinatura a um documentário muito
interessante sobre um certo tipo de acontecimento paranormal (ou
talvez sobrenatural) geralmente relatado em mesas de bar e clubes-do-
bolinha e que, penso eu, merece uma séria investigação por parte das
associações de céticos e racionalistas (atenção CSICOP, STR, Daniel
Sottomaior & Cia.)

Trata-se de contatos imediatos em quarto grau (eróticos) que obedece
a certos padrões:

a) As vítimas seduzidas são homens muito feios e/ou desprovidos de
qualquer charme – geralmente gordos, baixinhos, carecas, nerds,
quatro-olhos, barnabés, office-boys, bebuns contumazes, etc.

b) Os agentes sedutores são criaturas de aparência feminina
provavelmente de outras dimensões ou "aliens", já que, segundo o
testemunho dos abduzidos, possuem elas uma beleza angelical "de
outro mundo".

c) O contato é secreto, já que só acontece quando não há testemunhas
ou pessoas conhecidas por perto. Como, por exemplo, pontos de ônibus
às três e meia da madrugada de terça-feira, ermas estradas vicinais
(quando a estranha modelo ou diva de cinema usa como truque
automóveis com pneu furado), elevadores, bares no outro lado da
cidade (ou seja, muito distantes da residência do abduzido) ou então
na própria casa da vítima e nas ocasiões em que esta se encontra
sozinha – neste último caso, as belíssimas mulheres aparecem
disfarçadas de entregadoras de pizza.

d) O relato das vítimas se dá com mais freqüência e desembaraço
quando elas se encontram sob o efeito de algum "soro da verdade",
como uísque, cerveja, caipirinha, rum Montila com coca-cola, etc.

Não importa se essas divas são ET's interessados em nosso DNA,
súcubos ou apenas belas mulheres encantadas com a lábia do caboré.
Seja qual for o caso, há paranormalidade.

Ou seriam esses caborés apenas pobres onanistas mitômanos,
falastrões sequiosos de atenção ou mesmo charlatões? Caso sejam,
precisam ser urgentemente desmascarados, pois suas confabulações,
suas estórias fantasiosas estão criando um clima de descrença geral
da qual não escapa nem mesmo nós, os verdadeiros fodões e come-
quietos.

Abraços,
Manuel Bulcão





SUBJECT: Re: [ciencialist] Re: Unidades de medida: símbolo e nome da grandeza
FROM: "Prof. JC" <profjc2003@yahoo.com.br>
TO: <ciencialist@yahoogrupos.com.br>
DATE: 21/01/2005 15:48

Tatu => Araraquara? :)))


--- Em ciencialist@yahoogrupos.com.br, "Silvio" <scordeiro@t...>
escreveu
> Sergim:
> Uma regra gramatical que ocês d´alem mar não conhecem:
>
> FLEXÃO..
> Regra 69 - "Todas palavras terminadas em "U" fazem o feminino
em "ARA"
>
> Exemplo: bambu - taquara.
>
> -----Mensagem Original-----
> De: "Sergio M. M. Taborda" <sergiotaborda@y...>
> Para: <ciencialist@yahoogrupos.com.br>
> Enviada em: quarta-feira, 19 de janeiro de 2005 23:02
> Assunto: [ciencialist] Re: Unidades de medida: símbolo e nome da
grandeza
>
>
>
>
> --- Em ciencialist@yahoogrupos.com.br, "E m i l i a n o C h e m
e
> l l o" <chemelloe@y...> escreveu
> > Olá Sérgio,
> >
> > [Sérgio]
> > O Qual é o plural de uma palavra que termina em l ?
> >
> > mil = mis
> > formol = formois.
> >
> > mol = mois.
> >
> > Supondo que o nome da unidade , em portugues, é mol - o que não
é -
> > obtemos o bonito plural : mois.
> > Mas como no nome da unidade é mole, o plural é o bonito moles.
> > Ah é tão dificil chegar nesta conclusão ...
> >
> > [Emiliano]
> > O inmetro diz que o plural de 'mol' é 'mols'
> > http://www.inmetro.gov.br/consumidor/unidLegaisMed.asp
>
> Tudo bem, em roma sê romano. Mas eu não consigo pronunciar mols,
nem
> pascals ... soa mal ao ouvido portuguesmente falando.
> Eu tb não entendo pq vcs chama ions , etc.. quando portuguesmente
> falando o correcto é iões.
> Veja, quando eu digo correcto, eu estou me referindo às regras de
> formação de palavras, que são aquilo que faz uma lingua.
> É incorrecto usar estrangeirismos quando a nossa lingua tem
palavras
> que os podem substituir facilmente. Ainda por cima no caso dos
nomes
> das particulas que derivam todas do grego tal como o portugues e
> portanto não ha razão logica para recorrer ao ingles. Os ingles
> tiveram o seu trabalho adpatando o grego que não pertence à
linhagem
> do ingles, e agora querem adptar a adpatação ? è no minimo, um
> trabalho mal feito. Para não dizer desilegante e porco.
> Mas como no brasil ninguem quer nem saber, acho que vai ter que
> continuar asssim. E em ultima analise os orgãos de qualidade é que
> ditam as regras. Não faz nenhum sentido, mas pronto.
>
> Eu só quero deixar claro que cada lingua tem o seu processo de
criar
> palavras e esse processo é um algoritmo bem estabelecido. é uma das
> poucas coisas em que linguistica se assemelha a uma ciencia. Passar
> por cima disso é simplesmente idiota, sobretudo no ambito da
> ciencia. É como ter um martelo e pregar com uma chave-de-parafusos
> pq ela é electrica. É simplesmente absurdo.
>
> > Se a gente for se preocupar com tudo... veja a palavra 'átomo'
> então? :-)
>
> Essa não entendi, qual é o problema com "átomos" ?
>
>
> Sérgio Taborda
>
>
>
>
>
> ##### ##### #####
>
> Para saber mais visite
> http://www.ciencialist.hpg.ig.com.br
>
>
> ##### ##### ##### #####
> Links do Yahoo! Grupos





##### ##### #####

Para saber mais visite
http://www.ciencialist.hpg.ig.com.br


##### ##### ##### #####
Links do Yahoo! Grupos













SUBJECT: fumaça...
FROM: "E m i l i a n o C h e m e l l o" <chemelloe@yahoo.com.br>
TO: <ciencialist@yahoogrupos.com.br>
DATE: 21/01/2005 16:27

Olha a fumaça ai gente!

[ ] 's do Emiliano Chemello
---

I ai cara, blz? TEnho uma pergunta a fazer ... eu fiz uma bomba de fumaça que ia salitre do chile (duplo potassico) + açucar. Eu tive que fundir as duas substancias...
Eu queria saber se a fumaça que é liberada é ou não é toxica?
Se vc puder me ajudar me mande um email: fbcgon@yahoo.com.br
Obrigado...


[As partes desta mensagem que não continham texto foram removidas]



SUBJECT: Meu Blog
FROM: Amauri Jr <amaurijunior2@yahoo.com.br>
TO: ciencialist@yahoogrupos.com.br, Conversa_de_Botequim@yahoogrupos.com.br
DATE: 21/01/2005 21:50

http://cartasaomundo666.blogspot.com/

Amauri



SUBJECT: Dúvida em química
FROM: "Luis" <luis.alcides@he.com.br>
TO: ciencialist@yahoogrupos.com.br
DATE: 21/01/2005 23:46


Olá, alguém por algum acaso se lembra como detectar Cl-, CN-,
[Fe(CN)6]----, [Fe(CN)6]--- e SCN- uns na presença dos outros? Em uma
solução que contém todos eles misturados?

Saudações a todos. Luis.





SUBJECT: Essas, pois, são as inquietudes tolas de mais um bobo.(2)
FROM: "rayfisica" <rayfisica@yahoo.com.br>
TO: ciencialist@yahoogrupos.com.br
DATE: 22/01/2005 07:54


Com certeza o assunto já foi tratado aqui, como sou novo na casa...

Tenho ouvido tanta besteira com o nome de ciência e em nome da
ciência (apesar de minhas limitações) que desconfio de tudo,
por
favor, alguém pode explicar isso para mim de um jeito bem simples.


http://observatorio.ultimosegundo.ig.com.br/artigos.asp?cod=286OFC003


TABUS DA CIÊNCIA
Seria Einstein uma fraude?

Osório Barbosa (*)

Por necessidade de serviço (estou escrevendo um romance sobre a
história do Direito, onde, para melhor contá-la, senti
necessidade de
que o personagem viajasse no tempo), como já tinha ouvido falar e
lido de relance sobre tal possibilidade, resolvi pesquisar na
internet sobre o assunto, mais especificamente sobre a Teoria da
Relatividade.

Para minha surpresa, deparei-me com a entrevista de César Lattes,
intitulada: "Albert Einstein é uma farsa".

Mas quem é César Lattes?

Fiquei sabendo que é um físico brasileiro que esteve prestes a
ganhar, por duas vezes, o Prêmio Nobel de Física, por ter
descoberto
o méson pi, que é uma partícula integrante do núcleo do
átomo. Ela é
apontada como uma das principais responsáveis pela integridade do
núcleo, impedindo-o de desintegrar-se. Se o núcleo do átomo
fosse
formado apenas pelas partículas negativa e positiva, ele se
desintegrava, me explicou mais tarde o professor da Unicamp.

Depois de ler a entrevista de Lattes, fui ler um livro de Ronaldo
Rogério de Freitas Mourão, cujo título é Explicando a
Teoria da
Relatividade (Ediouro, 1997). Nele está dito:

"A medida do desvio da luz das estrelas ao passar próximo ao Sol.

O primeiro valor do desvio da luz, previsto por Einstein, em 1911,
era de 0,875 segundo de arco. (p. 14) Neste intervalo, Einstein, ao
rever sua teoria, duplicou a previsão: a deflexão deveria ser
de 1,75
segundo de arco, e não de 0.875. A razão desta alteração
deve-se ao
fato de não ter sido considerada a curvatura de espaço no
cálculo
anterior, o que só foi possível com a conclusão da teoria da
relatividade geral, em 1915.

Analisando a origem das idéias de Einstein, Feuer faz um paralelo
com
o matemático francês H. Poincaré (1854-1912), que tinha tudo
para ser
o criador da teoria da relatividade, como se pode deduzir da leitura
de seus livros. Todavia, do ponto de vista pessoal, possuía o
condicionamento do francês sempre prudente, comprometido com o
establishment e os colegiados de professores. A grande diferença
entre Poincaré e Einstein seria, segundo Feuer, o não-
comprometimento, que o colocava numa posição de observador
privilegiado. (p. 22)."

O parágrafo acima consta de uma nota de rodapé!

Resolvi, por intermédio de um conhecido (que tem laços com a
Unicamp), tentar localizar o professor Lattes, a fim de ouvir dele se
confirmava a entrevista e que me explicasse as razões de sua
afirmação a respeito do renomado Einstein.

Meu conhecido me passou o telefone do professor Lattes. Ele me
atendeu gentilmente e marcamos uma entrevista. Fui até sua casa em
Campinas. Lá, ele não só confirmou a entrevista, como
acrescentou
outros pontos que não vêm ao caso. Perguntei ao professor
Lattes de
onde ele tirava tanta convicção para fazer aquelas afirmativas
sobre
Einstein (uma vez que um colega de trabalho me disse que poderia ser
inveja, ou o dizer por dizer, sem provas. É claro que não disse
isso
ao físico).

Ele, então, me repassou duas fontes de suas afirmações:

1ª) Cópia de uma ata de 1906 onde Poincaré expôs a
teoria, num
congresso de cientistas, mais precisamente, Sur la dynamique de
l'électron, publicado pelo Circolo Matematico di Palermo, t. 21,
p.
129-176, em 1906. Se a publicação é de 1906, segundo Lattes, o
trabalho de Poincaré é do ano anterior, portanto, de 1905.

Como Einstein apresentou seu trabalho em 1911, ou seja, seis anos
após o de Poincaré (fato que é confirmado por Ronaldo
Rogério),
passei a observar as afirmações de Lattes.

2ª) Cópia do livro História das teorias do éter e da
eletricidade, de
Sir Edmundo Whittakker R.R.S. (Humanities Press, capítulo I, 1973,
Nova York) o qual chega à mesma conclusão de Ronaldo
Rogério. Este
autor cita, abertamente, Lorentz e Poincaré, como os precursores.
Poincaré, por exemplo, em 1900, já apresentava a famosa
fórmula:
E=mc2.

Diz, especificamente, Whittakker:

"Como foi visto, J. J. Thomson em 1881 chegou ao resultado que um
condutor esférico carregado movendo-se em linha reta comporta-se
como
se tivesse uma quantidade de massa adicional (4/3 c2) vezes a energia
de seu campo eletrostático. Em 1900 Poincaré, referindo-se ao
fato
que no éter livre o momento eletromagnético é (1/c2) vezes o
fluxo
Poynting de energia, sugeriu que energia eletromagnética pudesse
possuir densidade de massa igual a (1/c2) vezes a densidade da
energia: o que quer dizer, E= mc2 onde E é energia e m é massa:
e ele
apontou que se isto fosse assim, então um oscilador Hertz, o qual
envia energia eletromagnética preponderantemente em uma
direção,
deveria recuar como os revólveres fazem quando são disparados.
Em
1904 F. Hasenörl (1874-1915) considerou uma caixa oca com paredes
perfeitamente reflexivas preenchida com radiação, e descobriu que
quando estava em movimento, há uma adição aparente à sua
massa, com
valor (8/3c2) vezes a energia possuída pela radiação quando a
caixa
está em repouso: no ano seguinte ele corrigiu isto para (4/3c2)
vezes
a energia possuída pela radiação quando a caixa está em
repouso; quer
dizer, ele concordou com a equação E = 3/4mc2 de J. J. Thomson ao
invés de com a fórmula E= mc2 de Poincaré. Em 1905 A.
Einstein
afirmou que quando um corpo está perdendo energia em forma de
radiação sua massa é diminuída aproximadamente (isto é,
desprezando
quantidades da quarta ordem) por (1/c2) vezes a quantidade de energia
perdida. Ele apontou que não é essencial que a energia perdida
pelo
corpo deveria consistir em radiação, e sugeriu a conclusão
geral, em
concordância com Poincaré, que a massa de um corpo é uma
medida do
seu conteúdo de energia: se a energia muda por E ergs, a massa
muda
no mesmo sentido por (E/c2)gramas. No ano seguinte ele alegou que
esta lei é a condição necessária e suficiente e que a lei da
conservação do movimento do centro de gravidade deveria ser
válida
para sistemas nos quais processos eletromagnéticos bem como
mecânicos
estejam ocorrendo.

Em 1908 G. N. Lewis provou, por meio da teoria da pressão
radiativa,
que um corpo que absorve energia radiante aumenta sua massa de acordo
com a equação

dE = c2dm

e afirmou que a massa de um corpo é uma medida direta de sua
energia
total, de acordo com a equação

E = mc2.

Como vimos, Poincaré havia sugerido esta equação mas não
havia
praticamente dado prova, enquanto Einstein, que também a sugerira,
havia feito prova (a qual, no entanto, foi divulgada somente como
aproximada) para um caso particular: Lewis encarou-a como uma
equação
exata, mas sua prova não era de caráter geral. Lewis, no
entanto,
apontou que se este princípio é aceito, então na equação
de Planck em
1906."

Depois disso, li um artigo do professor Marcelo Gleiser (publicados
no caderno Mais!, da Folha de S. Paulo, de 30/5/04),
intitulado: "Einstein, ícone da ciência".

Também na Folha (6/6/04) li na coluna do jornalista Luís Nassif
que
ele aponta o físico César Lattes como "um quase ganhador " do
Prêmio
Nobel.

As últimas afirmações embaralharam a posição que pretendo
adotar!

Então resolvi trocar e-mails com o professor Gleiser, que
gentilmente
me respondeu. No entanto, apenas contribuiu para aumentar minhas
dúvidas.

Enviei a ele os posicionamentos do professor Lattes e o apoio ao seu
conhecimento por Nassif, concitando-o: se possível, gostaria que o
senhor me explicasse o assunto, uma vez que todos só falam de
Einstein como autor da citada teoria.

Recebi a seguinte resposta:

"caro sr., a discussão sobre a autoria da teoria da relatividade
passa por altos e baixos. Não existe a menor dúvida que foi
Einstein
o autor da teoria. O que sabemos é que Lorenz e Poincaré
estavam já
pensando no assunto e obtiveram algumas das fórmulas que Einstein
re-
obteve. (Ele não sabia delas). Mas o que faltou a Poincaré e
Lorenz
foi a interpretação apropriada da teoria. Poincaré estava
próximo,
mas não existe um trabalho seu que se compare à clareza do de
Einstein. Mais ainda, lembre-se que a teoria da relatividade tem 2
partes. O sr. está citando apenas a especial. A teoria geral, que
levou à redefinição da gravidade como a curvatura do espaço,
é
completamente de autoria única do Einstein. abraço, Marcelo
Gleiser."

Como a teoria é complicada para a maioria dos mortais, não
consegui
entender, confrontando a resposta com o disse Whittakker:

** Se não existe a menor dúvida de que foi Einstein o autor da
teoria, por que a discussão sobre a autoria passa por altos e
baixos?

** Se a famosa fórmula (E= mc2) data de 1900, por que ela é
atribuída
a Einstein em 1911?

** Re-obteve não significa obter algo já obtido?

** Mas o que faltou a Poincaré e Lorenz foi a interpretação
apropriada da teoria; não é contraditório em relação ao
que afirma
Ronaldo Rogério quando diz: "Poincaré (1854-1912), que tinha
tudo
para ser o criador da teoria da relatividade, como pode se deduzir da
leitura de seus livros. Todavia, do ponto de vista pessoal,
possuía o
condicionamento do francês sempre prudente, comprometido com o
establishment e os colegiados de professores."

** Por fim, segundo Lattes, inexiste diferença entre as chamadas
Teoria da Relatividade geral e especial, e que Einstein chegou a
confundir: medida com grandeza.

Sei que é complicado para os que entendem a física explicar a
neófitos conceitos que somente eles entendem. Muitos o fazem,
até,
para manter o hermetismo de seus saberes e assim não dividi-los;
outros, como era o caso do meu professor de Física do cursinho em
Manaus, não sabia mesmo, apenas aprendeu a deduzir umas
fórmulas e
assim ia ensinando aos candidatos das áreas de humanas no
vestibular.

Muitas inquietudes

De qualquer modo, minhas dúvidas continuam sem respostas e, creio,
aqueles que escrevem, especialmente em jornais e revistas, têm de
ensinar aos seus leitores.

Outro ponto que me leva a um pedido de esclarecimento sobre Einstein
é aquele, também trazido por Lattes, segundo o qual:

Einstein, após concitar o presidente americano Roosevelt a
construir
a bomba atômica, não participou dos trabalhos. Segundo Lattes,
porque
não sabia.

O contraditório disso é que depois se disse pacifista.

Numa resenha do livro Einstein – o viajante da relatividade na
América do Sul (revista Scientific American Brasil, março 2004,
p.
96), está dito:

a) "... Einstein, tentando romper com a tradição francesa..."
(isso
me remeteu a Poincaré);

b) "Nessa época a pesquisa científica na Argentina estava muito
à
frente do Brasil e o convite a Einstein era evidência disso. Mas
lá,
como aqui, enfrentou detratores desinformados da relatividade". Esse
é um belo argumento de autoridade (detratores desinformados da
relatividade). Quem não concorda é detrator e desinformado. Mas
seria
esse o caso de Lattes, Ronaldo Rogério e Whittakker?

c) "Einstein voltou ao Brasil na tarde de 4 de maio. As gafes não
foram poucas nem perdoáveis. Na primeira conferência, no Clube
de
Engenharia, atulhado de gente, o calor ficou insuportável e as
janelas foram abertas. A acústica que não era boa ficou pior, o
que,
na avaliação de Einstein, levou a uma exposição
ininteligível". Isso
me recordou do homem que falava javanês.

d) "No Brasil positivista Einstein enfrentou – e obviamente
não
respondeu – críticas ingênuas e desinformadas de
autoridades
intelectuais como o jurista Pontes de Miranda". Por que não
esclarecer a quem não sabe? Quem critica é sempre ingênuo e
desinformado?

e) Diogo Mainardi também resenhou o livro de Alfredo Tiomno
Tolmasquim (revista Veja, nº 1.860, de 30/6/04), trazendo a
informação de que "a teoria da relatividade ganhou sua primeira
confirmação empírica na cidade cearense de Sobral, onde
cientistas
ingleses fotografaram algumas estrelas num eclipse solar". Essas
fotos/provas, dizem, sumiram!

Sobre o ocorrido em Sobral/Ceará/Brasil e a fama de Einstein,
veja-se
o que disseram Lattes e Marcos Cesar Danhoni Neves, ao Jornal da
Unicamp, nº 165, de agosto/2001:

"Danhoni – É essa adesão ao modelo tido como real. Quem fez
a fama do
Einstein foi o Arthur Eddington (astrônomo e físico inglês,
1882-
1944).

Lattes – Esse é um palhaço puro.

Jornal da Unicamp – O Eddington é o inglês...

Danhoni – O Eddington mandou uma equipe para o Brasil em 1919, em
Sobral, e outra acompanhou na Guiné os melhores pontos para se
observar um eclipse. É só durante um eclipse que você vê
as estrelas
de fundo e pode comparar se elas estão no mesmo ponto ou se a luz
delas está defletida ou não. Só que ele pegou as chapas com
desvio
acima ou abaixo do previsto por Einstein e desconsiderou. Fez uma
análise estatística de erros observacionais e experimentais e
aquilo
se encaixava dentro do previsto por Einstein. Isso fez a fama de
Einstein."

Será que ambos são irrelevantes e tolos?

Tudo isso fez aflorar outras dúvidas:

a) Dizem que a ex-esposa de Einstein (Mileva Maric) era quem fazia os
cálculos matemáticos para ele. Após a separação do casal,
nada mais
foi produzido por ele.

b) Era incapaz de dirigir uma bicicleta, ou melhor, dava umas
pedaladas depois de idoso.

c) Sempre se antecipa ou se justifica qualquer possível
questionamento sobre o cientista, tanto assim que na Revista da Folha
nº 627, de 4/7/04, na matéria intitulada "Escreveu, não
leu", p. 9,
colhe-se o seguinte:

"... a dislexia foi diagnosticada pela primeira vez em 1896, pelo
neurologista inglês Pringle Morgan, que a chamou de cegueira
visual."

"Entidade e associações costumam listar famosos supostamente
vítimas
do distúrbio, como o cientista Albert Einstein..."

A revista, nesse caso, foi crítica ao usar o "supostamente", já
que
outras dão como fato certo. Se a dislexia foi descoberta em 1896,
e
tendo Einstein falecido em 1955, teria sido possível que ele tenha
se
submetido ao diagnóstico? Seria interessante se quem sabe pudesse
esclarecer e que não ficasse tudo na base do "é porque é".

Se em 1900, a fórmula já era E= mc2, por que em 1911 E= mc2
passou a
ser diferente? Se Einstein não era fraudador, era, então, um
plagiador? Por que a imprensa endeusa Einstein e ignora quase que
totalmente César Lattes?

Essas, pois, são as inquietudes tolas de mais um bobo.

(*) Procurador da República, mestre em Direito Constitucional e
curioso






SUBJECT: Essas, pois, são as inquietudes tolas de mais um bobo.(2)
FROM: "rayfisica" <rayfisica@yahoo.com.br>
TO: ciencialist@yahoogrupos.com.br
DATE: 22/01/2005 07:59


Com certeza o assunto já foi tratado aqui, como sou novo na casa...

Tenho ouvido tanta besteira com o nome de ciência e em nome da
ciência (apesar de minhas limitações) que desconfio de tudo,
por
favor, alguém pode explicar isso para mim de um jeito bem simples.


http://observatorio.ultimosegundo.ig.com.br/artigos.asp?cod=286OFC003


TABUS DA CIÊNCIA
Seria Einstein uma fraude?

Osório Barbosa (*)

Por necessidade de serviço (estou escrevendo um romance sobre a
história do Direito, onde, para melhor contá-la, senti
necessidade de
que o personagem viajasse no tempo), como já tinha ouvido falar e
lido de relance sobre tal possibilidade, resolvi pesquisar na
internet sobre o assunto, mais especificamente sobre a Teoria da
Relatividade.

Para minha surpresa, deparei-me com a entrevista de César Lattes,
intitulada: "Albert Einstein é uma farsa".

Mas quem é César Lattes?

Fiquei sabendo que é um físico brasileiro que esteve prestes a
ganhar, por duas vezes, o Prêmio Nobel de Física, por ter
descoberto
o méson pi, que é uma partícula integrante do núcleo do
átomo. Ela é
apontada como uma das principais responsáveis pela integridade do
núcleo, impedindo-o de desintegrar-se. Se o núcleo do átomo
fosse
formado apenas pelas partículas negativa e positiva, ele se
desintegrava, me explicou mais tarde o professor da Unicamp.

Depois de ler a entrevista de Lattes, fui ler um livro de Ronaldo
Rogério de Freitas Mourão, cujo título é Explicando a
Teoria da
Relatividade (Ediouro, 1997). Nele está dito:

"A medida do desvio da luz das estrelas ao passar próximo ao Sol.

O primeiro valor do desvio da luz, previsto por Einstein, em 1911,
era de 0,875 segundo de arco. (p. 14) Neste intervalo, Einstein, ao
rever sua teoria, duplicou a previsão: a deflexão deveria ser
de 1,75
segundo de arco, e não de 0.875. A razão desta alteração
deve-se ao
fato de não ter sido considerada a curvatura de espaço no
cálculo
anterior, o que só foi possível com a conclusão da teoria da
relatividade geral, em 1915.

Analisando a origem das idéias de Einstein, Feuer faz um paralelo
com
o matemático francês H. Poincaré (1854-1912), que tinha tudo
para ser
o criador da teoria da relatividade, como se pode deduzir da leitura
de seus livros. Todavia, do ponto de vista pessoal, possuía o
condicionamento do francês sempre prudente, comprometido com o
establishment e os colegiados de professores. A grande diferença
entre Poincaré e Einstein seria, segundo Feuer, o não-
comprometimento, que o colocava numa posição de observador
privilegiado. (p. 22)."

O parágrafo acima consta de uma nota de rodapé!

Resolvi, por intermédio de um conhecido (que tem laços com a
Unicamp), tentar localizar o professor Lattes, a fim de ouvir dele se
confirmava a entrevista e que me explicasse as razões de sua
afirmação a respeito do renomado Einstein.

Meu conhecido me passou o telefone do professor Lattes. Ele me
atendeu gentilmente e marcamos uma entrevista. Fui até sua casa em
Campinas. Lá, ele não só confirmou a entrevista, como
acrescentou
outros pontos que não vêm ao caso. Perguntei ao professor
Lattes de
onde ele tirava tanta convicção para fazer aquelas afirmativas
sobre
Einstein (uma vez que um colega de trabalho me disse que poderia ser
inveja, ou o dizer por dizer, sem provas. É claro que não disse
isso
ao físico).

Ele, então, me repassou duas fontes de suas afirmações:

1ª) Cópia de uma ata de 1906 onde Poincaré expôs a
teoria, num
congresso de cientistas, mais precisamente, Sur la dynamique de
l'électron, publicado pelo Circolo Matematico di Palermo, t. 21,
p.
129-176, em 1906. Se a publicação é de 1906, segundo Lattes, o
trabalho de Poincaré é do ano anterior, portanto, de 1905.

Como Einstein apresentou seu trabalho em 1911, ou seja, seis anos
após o de Poincaré (fato que é confirmado por Ronaldo
Rogério),
passei a observar as afirmações de Lattes.

2ª) Cópia do livro História das teorias do éter e da
eletricidade, de
Sir Edmundo Whittakker R.R.S. (Humanities Press, capítulo I, 1973,
Nova York) o qual chega à mesma conclusão de Ronaldo
Rogério. Este
autor cita, abertamente, Lorentz e Poincaré, como os precursores.
Poincaré, por exemplo, em 1900, já apresentava a famosa
fórmula:
E=mc2.

Diz, especificamente, Whittakker:

"Como foi visto, J. J. Thomson em 1881 chegou ao resultado que um
condutor esférico carregado movendo-se em linha reta comporta-se
como
se tivesse uma quantidade de massa adicional (4/3 c2) vezes a energia
de seu campo eletrostático. Em 1900 Poincaré, referindo-se ao
fato
que no éter livre o momento eletromagnético é (1/c2) vezes o
fluxo
Poynting de energia, sugeriu que energia eletromagnética pudesse
possuir densidade de massa igual a (1/c2) vezes a densidade da
energia: o que quer dizer, E= mc2 onde E é energia e m é massa:
e ele
apontou que se isto fosse assim, então um oscilador Hertz, o qual
envia energia eletromagnética preponderantemente em uma
direção,
deveria recuar como os revólveres fazem quando são disparados.
Em
1904 F. Hasenörl (1874-1915) considerou uma caixa oca com paredes
perfeitamente reflexivas preenchida com radiação, e descobriu que
quando estava em movimento, há uma adição aparente à sua
massa, com
valor (8/3c2) vezes a energia possuída pela radiação quando a
caixa
está em repouso: no ano seguinte ele corrigiu isto para (4/3c2)
vezes
a energia possuída pela radiação quando a caixa está em
repouso; quer
dizer, ele concordou com a equação E = 3/4mc2 de J. J. Thomson ao
invés de com a fórmula E= mc2 de Poincaré. Em 1905 A.
Einstein
afirmou que quando um corpo está perdendo energia em forma de
radiação sua massa é diminuída aproximadamente (isto é,
desprezando
quantidades da quarta ordem) por (1/c2) vezes a quantidade de energia
perdida. Ele apontou que não é essencial que a energia perdida
pelo
corpo deveria consistir em radiação, e sugeriu a conclusão
geral, em
concordância com Poincaré, que a massa de um corpo é uma
medida do
seu conteúdo de energia: se a energia muda por E ergs, a massa
muda
no mesmo sentido por (E/c2)gramas. No ano seguinte ele alegou que
esta lei é a condição necessária e suficiente e que a lei da
conservação do movimento do centro de gravidade deveria ser
válida
para sistemas nos quais processos eletromagnéticos bem como
mecânicos
estejam ocorrendo.

Em 1908 G. N. Lewis provou, por meio da teoria da pressão
radiativa,
que um corpo que absorve energia radiante aumenta sua massa de acordo
com a equação

dE = c2dm

e afirmou que a massa de um corpo é uma medida direta de sua
energia
total, de acordo com a equação

E = mc2.

Como vimos, Poincaré havia sugerido esta equação mas não
havia
praticamente dado prova, enquanto Einstein, que também a sugerira,
havia feito prova (a qual, no entanto, foi divulgada somente como
aproximada) para um caso particular: Lewis encarou-a como uma
equação
exata, mas sua prova não era de caráter geral. Lewis, no
entanto,
apontou que se este princípio é aceito, então na equação
de Planck em
1906."

Depois disso, li um artigo do professor Marcelo Gleiser (publicados
no caderno Mais!, da Folha de S. Paulo, de 30/5/04),
intitulado: "Einstein, ícone da ciência".

Também na Folha (6/6/04) li na coluna do jornalista Luís Nassif
que
ele aponta o físico César Lattes como "um quase ganhador " do
Prêmio
Nobel.

As últimas afirmações embaralharam a posição que pretendo
adotar!

Então resolvi trocar e-mails com o professor Gleiser, que
gentilmente
me respondeu. No entanto, apenas contribuiu para aumentar minhas
dúvidas.

Enviei a ele os posicionamentos do professor Lattes e o apoio ao seu
conhecimento por Nassif, concitando-o: se possível, gostaria que o
senhor me explicasse o assunto, uma vez que todos só falam de
Einstein como autor da citada teoria.

Recebi a seguinte resposta:

"caro sr., a discussão sobre a autoria da teoria da relatividade
passa por altos e baixos. Não existe a menor dúvida que foi
Einstein
o autor da teoria. O que sabemos é que Lorenz e Poincaré
estavam já
pensando no assunto e obtiveram algumas das fórmulas que Einstein
re-
obteve. (Ele não sabia delas). Mas o que faltou a Poincaré e
Lorenz
foi a interpretação apropriada da teoria. Poincaré estava
próximo,
mas não existe um trabalho seu que se compare à clareza do de
Einstein. Mais ainda, lembre-se que a teoria da relatividade tem 2
partes. O sr. está citando apenas a especial. A teoria geral, que
levou à redefinição da gravidade como a curvatura do espaço,
é
completamente de autoria única do Einstein. abraço, Marcelo
Gleiser."

Como a teoria é complicada para a maioria dos mortais, não
consegui
entender, confrontando a resposta com o disse Whittakker:

** Se não existe a menor dúvida de que foi Einstein o autor da
teoria, por que a discussão sobre a autoria passa por altos e
baixos?

** Se a famosa fórmula (E= mc2) data de 1900, por que ela é
atribuída
a Einstein em 1911?

** Re-obteve não significa obter algo já obtido?

** Mas o que faltou a Poincaré e Lorenz foi a interpretação
apropriada da teoria; não é contraditório em relação ao
que afirma
Ronaldo Rogério quando diz: "Poincaré (1854-1912), que tinha
tudo
para ser o criador da teoria da relatividade, como pode se deduzir da
leitura de seus livros. Todavia, do ponto de vista pessoal,
possuía o
condicionamento do francês sempre prudente, comprometido com o
establishment e os colegiados de professores."

** Por fim, segundo Lattes, inexiste diferença entre as chamadas
Teoria da Relatividade geral e especial, e que Einstein chegou a
confundir: medida com grandeza.

Sei que é complicado para os que entendem a física explicar a
neófitos conceitos que somente eles entendem. Muitos o fazem,
até,
para manter o hermetismo de seus saberes e assim não dividi-los;
outros, como era o caso do meu professor de Física do cursinho em
Manaus, não sabia mesmo, apenas aprendeu a deduzir umas
fórmulas e
assim ia ensinando aos candidatos das áreas de humanas no
vestibular.

Muitas inquietudes

De qualquer modo, minhas dúvidas continuam sem respostas e, creio,
aqueles que escrevem, especialmente em jornais e revistas, têm de
ensinar aos seus leitores.

Outro ponto que me leva a um pedido de esclarecimento sobre Einstein
é aquele, também trazido por Lattes, segundo o qual:

Einstein, após concitar o presidente americano Roosevelt a
construir
a bomba atômica, não participou dos trabalhos. Segundo Lattes,
porque
não sabia.

O contraditório disso é que depois se disse pacifista.

Numa resenha do livro Einstein – o viajante da relatividade na
América do Sul (revista Scientific American Brasil, março 2004,
p.
96), está dito:

a) "... Einstein, tentando romper com a tradição francesa..."
(isso
me remeteu a Poincaré);

b) "Nessa época a pesquisa científica na Argentina estava muito
à
frente do Brasil e o convite a Einstein era evidência disso. Mas
lá,
como aqui, enfrentou detratores desinformados da relatividade". Esse
é um belo argumento de autoridade (detratores desinformados da
relatividade). Quem não concorda é detrator e desinformado. Mas
seria
esse o caso de Lattes, Ronaldo Rogério e Whittakker?

c) "Einstein voltou ao Brasil na tarde de 4 de maio. As gafes não
foram poucas nem perdoáveis. Na primeira conferência, no Clube
de
Engenharia, atulhado de gente, o calor ficou insuportável e as
janelas foram abertas. A acústica que não era boa ficou pior, o
que,
na avaliação de Einstein, levou a uma exposição
ininteligível". Isso
me recordou do homem que falava javanês.

d) "No Brasil positivista Einstein enfrentou – e obviamente
não
respondeu – críticas ingênuas e desinformadas de
autoridades
intelectuais como o jurista Pontes de Miranda". Por que não
esclarecer a quem não sabe? Quem critica é sempre ingênuo e
desinformado?

e) Diogo Mainardi também resenhou o livro de Alfredo Tiomno
Tolmasquim (revista Veja, nº 1.860, de 30/6/04), trazendo a
informação de que "a teoria da relatividade ganhou sua primeira
confirmação empírica na cidade cearense de Sobral, onde
cientistas
ingleses fotografaram algumas estrelas num eclipse solar". Essas
fotos/provas, dizem, sumiram!

Sobre o ocorrido em Sobral/Ceará/Brasil e a fama de Einstein,
veja-se
o que disseram Lattes e Marcos Cesar Danhoni Neves, ao Jornal da
Unicamp, nº 165, de agosto/2001:

"Danhoni – É essa adesão ao modelo tido como real. Quem fez
a fama do
Einstein foi o Arthur Eddington (astrônomo e físico inglês,
1882-
1944).

Lattes – Esse é um palhaço puro.

Jornal da Unicamp – O Eddington é o inglês...

Danhoni – O Eddington mandou uma equipe para o Brasil em 1919, em
Sobral, e outra acompanhou na Guiné os melhores pontos para se
observar um eclipse. É só durante um eclipse que você vê
as estrelas
de fundo e pode comparar se elas estão no mesmo ponto ou se a luz
delas está defletida ou não. Só que ele pegou as chapas com
desvio
acima ou abaixo do previsto por Einstein e desconsiderou. Fez uma
análise estatística de erros observacionais e experimentais e
aquilo
se encaixava dentro do previsto por Einstein. Isso fez a fama de
Einstein."

Será que ambos são irrelevantes e tolos?

Tudo isso fez aflorar outras dúvidas:

a) Dizem que a ex-esposa de Einstein (Mileva Maric) era quem fazia os
cálculos matemáticos para ele. Após a separação do casal,
nada mais
foi produzido por ele.

b) Era incapaz de dirigir uma bicicleta, ou melhor, dava umas
pedaladas depois de idoso.

c) Sempre se antecipa ou se justifica qualquer possível
questionamento sobre o cientista, tanto assim que na Revista da Folha
nº 627, de 4/7/04, na matéria intitulada "Escreveu, não
leu", p. 9,
colhe-se o seguinte:

"... a dislexia foi diagnosticada pela primeira vez em 1896, pelo
neurologista inglês Pringle Morgan, que a chamou de cegueira
visual."

"Entidade e associações costumam listar famosos supostamente
vítimas
do distúrbio, como o cientista Albert Einstein..."

A revista, nesse caso, foi crítica ao usar o "supostamente", já
que
outras dão como fato certo. Se a dislexia foi descoberta em 1896,
e
tendo Einstein falecido em 1955, teria sido possível que ele tenha
se
submetido ao diagnóstico? Seria interessante se quem sabe pudesse
esclarecer e que não ficasse tudo na base do "é porque é".

Se em 1900, a fórmula já era E= mc2, por que em 1911 E= mc2
passou a
ser diferente? Se Einstein não era fraudador, era, então, um
plagiador? Por que a imprensa endeusa Einstein e ignora quase que
totalmente César Lattes?

Essas, pois, são as inquietudes tolas de mais um bobo.

(*) Procurador da República, mestre em Direito Constitucional e
curioso






SUBJECT: Essas, pois, são as inquietudes tolas de mais um bobo.(2)
FROM: "rayfisica" <rayfisica@yahoo.com.br>
TO: ciencialist@yahoogrupos.com.br
DATE: 22/01/2005 08:06


Com certeza o assunto já foi tratado aqui, como sou novo na casa...

Tenho ouvido tanta besteira com o nome de ciência e em nome da
ciência (apesar de minhas limitações) que desconfio de tudo,
por
favor, alguém pode explicar isso para mim de um jeito bem simples.


http://observatorio.ultimosegundo.ig.com.br/artigos.asp?cod=286OFC003


TABUS DA CIÊNCIA
Seria Einstein uma fraude?

Osório Barbosa (*)

Por necessidade de serviço (estou escrevendo um romance sobre a
história do Direito, onde, para melhor contá-la, senti
necessidade de
que o personagem viajasse no tempo), como já tinha ouvido falar e
lido de relance sobre tal possibilidade, resolvi pesquisar na
internet sobre o assunto, mais especificamente sobre a Teoria da
Relatividade.

Para minha surpresa, deparei-me com a entrevista de César Lattes,
intitulada: "Albert Einstein é uma farsa".

Mas quem é César Lattes?

Fiquei sabendo que é um físico brasileiro que esteve prestes a
ganhar, por duas vezes, o Prêmio Nobel de Física, por ter
descoberto
o méson pi, que é uma partícula integrante do núcleo do
átomo. Ela é
apontada como uma das principais responsáveis pela integridade do
núcleo, impedindo-o de desintegrar-se. Se o núcleo do átomo
fosse
formado apenas pelas partículas negativa e positiva, ele se
desintegrava, me explicou mais tarde o professor da Unicamp.

Depois de ler a entrevista de Lattes, fui ler um livro de Ronaldo
Rogério de Freitas Mourão, cujo título é Explicando a
Teoria da
Relatividade (Ediouro, 1997). Nele está dito:

"A medida do desvio da luz das estrelas ao passar próximo ao Sol.

O primeiro valor do desvio da luz, previsto por Einstein, em 1911,
era de 0,875 segundo de arco. (p. 14) Neste intervalo, Einstein, ao
rever sua teoria, duplicou a previsão: a deflexão deveria ser
de 1,75
segundo de arco, e não de 0.875. A razão desta alteração
deve-se ao
fato de não ter sido considerada a curvatura de espaço no
cálculo
anterior, o que só foi possível com a conclusão da teoria da
relatividade geral, em 1915.

Analisando a origem das idéias de Einstein, Feuer faz um paralelo
com
o matemático francês H. Poincaré (1854-1912), que tinha tudo
para ser
o criador da teoria da relatividade, como se pode deduzir da leitura
de seus livros. Todavia, do ponto de vista pessoal, possuía o
condicionamento do francês sempre prudente, comprometido com o
establishment e os colegiados de professores. A grande diferença
entre Poincaré e Einstein seria, segundo Feuer, o não-
comprometimento, que o colocava numa posição de observador
privilegiado. (p. 22)."

O parágrafo acima consta de uma nota de rodapé!

Resolvi, por intermédio de um conhecido (que tem laços com a
Unicamp), tentar localizar o professor Lattes, a fim de ouvir dele se
confirmava a entrevista e que me explicasse as razões de sua
afirmação a respeito do renomado Einstein.

Meu conhecido me passou o telefone do professor Lattes. Ele me
atendeu gentilmente e marcamos uma entrevista. Fui até sua casa em
Campinas. Lá, ele não só confirmou a entrevista, como
acrescentou
outros pontos que não vêm ao caso. Perguntei ao professor
Lattes de
onde ele tirava tanta convicção para fazer aquelas afirmativas
sobre
Einstein (uma vez que um colega de trabalho me disse que poderia ser
inveja, ou o dizer por dizer, sem provas. É claro que não disse
isso
ao físico).

Ele, então, me repassou duas fontes de suas afirmações:

1ª) Cópia de uma ata de 1906 onde Poincaré expôs a
teoria, num
congresso de cientistas, mais precisamente, Sur la dynamique de
l'électron, publicado pelo Circolo Matematico di Palermo, t. 21,
p.
129-176, em 1906. Se a publicação é de 1906, segundo Lattes, o
trabalho de Poincaré é do ano anterior, portanto, de 1905.

Como Einstein apresentou seu trabalho em 1911, ou seja, seis anos
após o de Poincaré (fato que é confirmado por Ronaldo
Rogério),
passei a observar as afirmações de Lattes.

2ª) Cópia do livro História das teorias do éter e da
eletricidade, de
Sir Edmundo Whittakker R.R.S. (Humanities Press, capítulo I, 1973,
Nova York) o qual chega à mesma conclusão de Ronaldo
Rogério. Este
autor cita, abertamente, Lorentz e Poincaré, como os precursores.
Poincaré, por exemplo, em 1900, já apresentava a famosa
fórmula:
E=mc2.

Diz, especificamente, Whittakker:

"Como foi visto, J. J. Thomson em 1881 chegou ao resultado que um
condutor esférico carregado movendo-se em linha reta comporta-se
como
se tivesse uma quantidade de massa adicional (4/3 c2) vezes a energia
de seu campo eletrostático. Em 1900 Poincaré, referindo-se ao
fato
que no éter livre o momento eletromagnético é (1/c2) vezes o
fluxo
Poynting de energia, sugeriu que energia eletromagnética pudesse
possuir densidade de massa igual a (1/c2) vezes a densidade da
energia: o que quer dizer, E= mc2 onde E é energia e m é massa:
e ele
apontou que se isto fosse assim, então um oscilador Hertz, o qual
envia energia eletromagnética preponderantemente em uma
direção,
deveria recuar como os revólveres fazem quando são disparados.
Em
1904 F. Hasenörl (1874-1915) considerou uma caixa oca com paredes
perfeitamente reflexivas preenchida com radiação, e descobriu que
quando estava em movimento, há uma adição aparente à sua
massa, com
valor (8/3c2) vezes a energia possuída pela radiação quando a
caixa
está em repouso: no ano seguinte ele corrigiu isto para (4/3c2)
vezes
a energia possuída pela radiação quando a caixa está em
repouso; quer
dizer, ele concordou com a equação E = 3/4mc2 de J. J. Thomson ao
invés de com a fórmula E= mc2 de Poincaré. Em 1905 A.
Einstein
afirmou que quando um corpo está perdendo energia em forma de
radiação sua massa é diminuída aproximadamente (isto é,
desprezando
quantidades da quarta ordem) por (1/c2) vezes a quantidade de energia
perdida. Ele apontou que não é essencial que a energia perdida
pelo
corpo deveria consistir em radiação, e sugeriu a conclusão
geral, em
concordância com Poincaré, que a massa de um corpo é uma
medida do
seu conteúdo de energia: se a energia muda por E ergs, a massa
muda
no mesmo sentido por (E/c2)gramas. No ano seguinte ele alegou que
esta lei é a condição necessária e suficiente e que a lei da
conservação do movimento do centro de gravidade deveria ser
válida
para sistemas nos quais processos eletromagnéticos bem como
mecânicos
estejam ocorrendo.

Em 1908 G. N. Lewis provou, por meio da teoria da pressão
radiativa,
que um corpo que absorve energia radiante aumenta sua massa de acordo
com a equação

dE = c2dm

e afirmou que a massa de um corpo é uma medida direta de sua
energia
total, de acordo com a equação

E = mc2.

Como vimos, Poincaré havia sugerido esta equação mas não
havia
praticamente dado prova, enquanto Einstein, que também a sugerira,
havia feito prova (a qual, no entanto, foi divulgada somente como
aproximada) para um caso particular: Lewis encarou-a como uma
equação
exata, mas sua prova não era de caráter geral. Lewis, no
entanto,
apontou que se este princípio é aceito, então na equação
de Planck em
1906."

Depois disso, li um artigo do professor Marcelo Gleiser (publicados
no caderno Mais!, da Folha de S. Paulo, de 30/5/04),
intitulado: "Einstein, ícone da ciência".

Também na Folha (6/6/04) li na coluna do jornalista Luís Nassif
que
ele aponta o físico César Lattes como "um quase ganhador " do
Prêmio
Nobel.

As últimas afirmações embaralharam a posição que pretendo
adotar!

Então resolvi trocar e-mails com o professor Gleiser, que
gentilmente
me respondeu. No entanto, apenas contribuiu para aumentar minhas
dúvidas.

Enviei a ele os posicionamentos do professor Lattes e o apoio ao seu
conhecimento por Nassif, concitando-o: se possível, gostaria que o
senhor me explicasse o assunto, uma vez que todos só falam de
Einstein como autor da citada teoria.

Recebi a seguinte resposta:

"caro sr., a discussão sobre a autoria da teoria da relatividade
passa por altos e baixos. Não existe a menor dúvida que foi
Einstein
o autor da teoria. O que sabemos é que Lorenz e Poincaré
estavam já
pensando no assunto e obtiveram algumas das fórmulas que Einstein
re-
obteve. (Ele não sabia delas). Mas o que faltou a Poincaré e
Lorenz
foi a interpretação apropriada da teoria. Poincaré estava
próximo,
mas não existe um trabalho seu que se compare à clareza do de
Einstein. Mais ainda, lembre-se que a teoria da relatividade tem 2
partes. O sr. está citando apenas a especial. A teoria geral, que
levou à redefinição da gravidade como a curvatura do espaço,
é
completamente de autoria única do Einstein. abraço, Marcelo
Gleiser."

Como a teoria é complicada para a maioria dos mortais, não
consegui
entender, confrontando a resposta com o disse Whittakker:

** Se não existe a menor dúvida de que foi Einstein o autor da
teoria, por que a discussão sobre a autoria passa por altos e
baixos?

** Se a famosa fórmula (E= mc2) data de 1900, por que ela é
atribuída
a Einstein em 1911?

** Re-obteve não significa obter algo já obtido?

** Mas o que faltou a Poincaré e Lorenz foi a interpretação
apropriada da teoria; não é contraditório em relação ao
que afirma
Ronaldo Rogério quando diz: "Poincaré (1854-1912), que tinha
tudo
para ser o criador da teoria da relatividade, como pode se deduzir da
leitura de seus livros. Todavia, do ponto de vista pessoal,
possuía o
condicionamento do francês sempre prudente, comprometido com o
establishment e os colegiados de professores."

** Por fim, segundo Lattes, inexiste diferença entre as chamadas
Teoria da Relatividade geral e especial, e que Einstein chegou a
confundir: medida com grandeza.

Sei que é complicado para os que entendem a física explicar a
neófitos conceitos que somente eles entendem. Muitos o fazem,
até,
para manter o hermetismo de seus saberes e assim não dividi-los;
outros, como era o caso do meu professor de Física do cursinho em
Manaus, não sabia mesmo, apenas aprendeu a deduzir umas
fórmulas e
assim ia ensinando aos candidatos das áreas de humanas no
vestibular.

Muitas inquietudes

De qualquer modo, minhas dúvidas continuam sem respostas e, creio,
aqueles que escrevem, especialmente em jornais e revistas, têm de
ensinar aos seus leitores.

Outro ponto que me leva a um pedido de esclarecimento sobre Einstein
é aquele, também trazido por Lattes, segundo o qual:

Einstein, após concitar o presidente americano Roosevelt a
construir
a bomba atômica, não participou dos trabalhos. Segundo Lattes,
porque
não sabia.

O contraditório disso é que depois se disse pacifista.

Numa resenha do livro Einstein – o viajante da relatividade na
América do Sul (revista Scientific American Brasil, março 2004,
p.
96), está dito:

a) "... Einstein, tentando romper com a tradição francesa..."
(isso
me remeteu a Poincaré);

b) "Nessa época a pesquisa científica na Argentina estava muito
à
frente do Brasil e o convite a Einstein era evidência disso. Mas
lá,
como aqui, enfrentou detratores desinformados da relatividade". Esse
é um belo argumento de autoridade (detratores desinformados da
relatividade). Quem não concorda é detrator e desinformado. Mas
seria
esse o caso de Lattes, Ronaldo Rogério e Whittakker?

c) "Einstein voltou ao Brasil na tarde de 4 de maio. As gafes não
foram poucas nem perdoáveis. Na primeira conferência, no Clube
de
Engenharia, atulhado de gente, o calor ficou insuportável e as
janelas foram abertas. A acústica que não era boa ficou pior, o
que,
na avaliação de Einstein, levou a uma exposição
ininteligível". Isso
me recordou do homem que falava javanês.

d) "No Brasil positivista Einstein enfrentou – e obviamente
não
respondeu – críticas ingênuas e desinformadas de
autoridades
intelectuais como o jurista Pontes de Miranda". Por que não
esclarecer a quem não sabe? Quem critica é sempre ingênuo e
desinformado?

e) Diogo Mainardi também resenhou o livro de Alfredo Tiomno
Tolmasquim (revista Veja, nº 1.860, de 30/6/04), trazendo a
informação de que "a teoria da relatividade ganhou sua primeira
confirmação empírica na cidade cearense de Sobral, onde
cientistas
ingleses fotografaram algumas estrelas num eclipse solar". Essas
fotos/provas, dizem, sumiram!

Sobre o ocorrido em Sobral/Ceará/Brasil e a fama de Einstein,
veja-se
o que disseram Lattes e Marcos Cesar Danhoni Neves, ao Jornal da
Unicamp, nº 165, de agosto/2001:

"Danhoni – É essa adesão ao modelo tido como real. Quem fez
a fama do
Einstein foi o Arthur Eddington (astrônomo e físico inglês,
1882-
1944).

Lattes – Esse é um palhaço puro.

Jornal da Unicamp – O Eddington é o inglês...

Danhoni – O Eddington mandou uma equipe para o Brasil em 1919, em
Sobral, e outra acompanhou na Guiné os melhores pontos para se
observar um eclipse. É só durante um eclipse que você vê
as estrelas
de fundo e pode comparar se elas estão no mesmo ponto ou se a luz
delas está defletida ou não. Só que ele pegou as chapas com
desvio
acima ou abaixo do previsto por Einstein e desconsiderou. Fez uma
análise estatística de erros observacionais e experimentais e
aquilo
se encaixava dentro do previsto por Einstein. Isso fez a fama de
Einstein."

Será que ambos são irrelevantes e tolos?

Tudo isso fez aflorar outras dúvidas:

a) Dizem que a ex-esposa de Einstein (Mileva Maric) era quem fazia os
cálculos matemáticos para ele. Após a separação do casal,
nada mais
foi produzido por ele.

b) Era incapaz de dirigir uma bicicleta, ou melhor, dava umas
pedaladas depois de idoso.

c) Sempre se antecipa ou se justifica qualquer possível
questionamento sobre o cientista, tanto assim que na Revista da Folha
nº 627, de 4/7/04, na matéria intitulada "Escreveu, não
leu", p. 9,
colhe-se o seguinte:

"... a dislexia foi diagnosticada pela primeira vez em 1896, pelo
neurologista inglês Pringle Morgan, que a chamou de cegueira
visual."

"Entidade e associações costumam listar famosos supostamente
vítimas
do distúrbio, como o cientista Albert Einstein..."

A revista, nesse caso, foi crítica ao usar o "supostamente", já
que
outras dão como fato certo. Se a dislexia foi descoberta em 1896,
e
tendo Einstein falecido em 1955, teria sido possível que ele tenha
se
submetido ao diagnóstico? Seria interessante se quem sabe pudesse
esclarecer e que não ficasse tudo na base do "é porque é".

Se em 1900, a fórmula já era E= mc2, por que em 1911 E= mc2
passou a
ser diferente? Se Einstein não era fraudador, era, então, um
plagiador? Por que a imprensa endeusa Einstein e ignora quase que
totalmente César Lattes?

Essas, pois, são as inquietudes tolas de mais um bobo.

(*) Procurador da República, mestre em Direito Constitucional e
curioso






SUBJECT: por favor me desculpem
FROM: "rayfisica" <rayfisica@yahoo.com.br>
TO: ciencialist@yahoogrupos.com.br
DATE: 22/01/2005 08:25


acho que tem alguma coisa estranha acontecendo no meu brownser, peço
perdão, se possível moderadores retirem duas das mensagens, por favor





SUBJECT: Células-tronco curam diabetes
FROM: José Renato <jrma@terra.com.br>
TO: <ciencialist@yahoogrupos.com.br>
DATE: 22/01/2005 09:58

[ O método consiste em extrair células do osso ilíaco, no quadril, e depois manipulá-las em laboratórios para o implante no pâncreas com um cateter especial, que é introduzido na artéria femoral, via de acesso direto ao "rabo" do pâncreas. Trata-se de um método inédito, livre de riscos de rejeição, sem intermediação prolongada e que pode ser realizado por qualquer médico especialista com ampla experiência em cateterismos, explicou o cardiologista argentino Roberto Fernández Viñas, coordenador da equipe. ]
..................................

A TARDE 22/01/2005 - 22:21 - Internacional

Saúde
Células-tronco curam o diabetes

Médicos argentinos conseguem reverter a deficiência de insulina no sangue com técnica inédita de implante no pâncreas

SONIA AVALOS
AGÊNCIA FRANCE PRESSE

BUENOS AIRES - Milhões de diabéticos poderão esquecer em breve a injeção de insulina, se for confirmado o resultado bem-sucedido do primeiro implante de células-tronco no pâncreas, feito por médicos argentinos que se dedicam à procura da cura para a doença.

Trata-se de um método inédito, livre de riscos de rejeição, sem intermediação prolongada e que pode ser realizado por qualquer médico especialista com ampla experiência em cateterismos, explicou o cardiologista argentino Roberto Fernández Viñas, coordenador da equipe que realizou o procedimento em um paciente diabético dependente de insulina, na Clínica San Nicolás, na cidade de mesmo nome.

Viñas integra a equipe médica de terapia celular do pâncreas com profissionais da Universidade Nacional de Rosário. Ele também participa do serviço de Imunologia do MD Anderson Center de Houston (Estados Unidos).

O método consiste em extrair células do osso ilíaco, no quadril, e depois manipulá-las em laboratórios para o implante no pâncreas com um cateter especial, que é introduzido na artéria femoral, via de acesso direto ao "rabo" do pâncreas.

O Center Cardiovascular Research Foundation, de Washington, já contatou a equipe para saber do procedimento revolucionário, afirmou Fernández Viñas. "Trata-se de uma técnica inédita porque emprega células-tronco não-embrionárias, diferentemente do que se fazia até agora, e o implante é feito por indução, já que escolhemos uma artéria direta e não uma via periférica", disse.

O método "abre um campo de pesquisa enorme" para o tratamento de outras patologias, como a hepatite C. Assim como as células-tronco embrionárias, estas têm a propriedade de atuar como "copiadoras" da informação que encontram no órgão onde são depositadas.

Os pacientes diabéticos sofrem de uma carência no pâncreas de células "beta", encarregadas de produzir a insulina, que regula os níveis de glicose no sangue. A introdução de células "copiadoras" no pâncreas gera a reprodução das células beta, aumentando a capacidade de produção da insulina necessária para equilibrar a glicose do paciente.

PRIMEIRA OPERAÇÃO - No dia 3 de janeiro, a equipe de Fernández Viñas realizou o primeiro implante em um paciente diabético de 42 anos que dependia das injeções de insulina desde os 25. Até agora, os estudos demonstram que o método funciona porque os níveis de glicose se restabelecem sem o auxílio de medicamentos. "De qualquer maneira, é necessário ser prudente e atuar com muita cautela", advertiu o especialista. "Cada paciente é um caso diferente e cada pâncreas pode reagir de uma forma ao estímulo".

O método é resultado da pesquisa que começou em 2003 na Argentina, a partir do implante de células-tronco no coração para reparar tecidos enfartados. A terapia celular no pâncreas é uma técnica "passível de ser reproduzida e copiada, porque pode ser realizada mais de uma vez no mesmo paciente e porque seu método não requer prolongados processos de treinamento do médico que a realiza", disse Fernández Viñas. Também não é necessária a permanência do paciente no hospital: ele pode voltar para casa no dia seguinte da intervenção.

A pesquisa, financiada por uma fundação científica particular, com custo médio de 5 mil pesos por implante (US$ 1.600), continuará até 1º de fevereiro. Em uma segunda etapa, serão selecionados 35 pacientes de 22 a 65 anos entre 500 voluntários, que já se ofereceram para participar do programa.

- Nosso objetivo é conseguir um "fast-track": queremos que seja um tratamento cujos resultados apareçam rapidamente e não se precise esperar anos para reverter a doença - disse Viñas.

A incidência do diabetes vem crescendo por causa da obesidade e da vida sedentária. Na Argentina, estima-se que 2,5% da população sofre da doença, quase um milhão de pessoas, das quais grande parte não tem conhecimento disso.


< http://www.atarde.com.br/materia.php3?id_materia=3031&ano=2005&mes=01&id_subcanal=15 >

[As partes desta mensagem que não continham texto foram removidas]



SUBJECT: Re: [ciencialist] Re: Unidades de medida: símbolo e nome da grandeza
FROM: "Silvio" <scordeiro@terra.com.br>
TO: <ciencialist@yahoogrupos.com.br>
DATE: 22/01/2005 10:15

Tia:

Ubu em tupi é "terra"
Capivara é o rol de crimes listados nos compoutadores da polícia.

silvio, phylólogo.

-----Mensagem Original-----
De: "Maria Natália" <grasdic@hotmail.com>
Para: <ciencialist@yahoogrupos.com.br>
Enviada em: sexta-feira, 21 de janeiro de 2005 01:02
Assunto: [ciencialist] Re: Unidades de medida: símbolo e nome da grandeza




Silvinho:
Ubu ===>> capivara
Obrigadim pela lição
Maria Natália




SUBJECT: Re: [ciencialist] Re: serie harmonica
FROM: "Luiz Ferraz Netto" <leobarretos@uol.com.br>
TO: <ciencialist@yahoogrupos.com.br>
DATE: 22/01/2005 11:15

Brilhante!
[]'
===========================
Luiz Ferraz Netto [Léo]
leobarretos@uol.com.br
http://www.feiradeciencias.com.br
===========================
-----Mensagem Original-----
De: "rmtakata" <rmtakata@altavista.net>
Para: <ciencialist@yahoogrupos.com.br>
Enviada em: sexta-feira, 21 de janeiro de 2005 08:30
Assunto: [ciencialist] Re: serie harmonica




--- Em ciencialist@yahoogrupos.com.br, "Luiz Ferraz Netto"
> Como provar que a série harmônica diverge, sem utilizar o teste da
> Integral?

Considere a soma dos termos da serie harmonica:

S = 1 + 1/2 + 1/3 + 1/4 + 1/5 + 1/6 + 1/7 + 1/8 + 1/9 + 1/10 + 1/11 +
1/12 + 1/13 + 1/14...

Poderemos agrupar os termos do seguinte modo, a partir de 1/3
(inclusive) reuna os termos na sequencia 2, 4, 8, 16...:

S = 1 + 1/2 + (1/3 + 1/4) + (1/5 + 1/6 + 1/7 + 1/8) + (1/9 + 1/10 +
1/11 + 1/12 + 1/13 + 1/14 + 1/15 + 1/16)...

Cada soma entre parenteses sempre dara' uma soma maior do q. 1/2.

Para provar isso, substitua os termos anteriores de cada parenteses
pelo ultimo termo do mesmo parenteses:

(1/3 + 1/4) > (1/4 + 1/4) = 2*1/4 = 1/2
(1/5 + 1/6 + 1/7 + 1/8) > (1/8 + 1/8 + 1/8 + 1/8) = 4*1/8 = 1/2
(1/9 + 1/10 + 1/11 + 1/12 + 1/13 + 1/14 + 1/15 + 1/16) > 8*1/16 = 1/2

Assim:

S > 1 + 1/2 + 1/2 + 1/2 + 1/2...

Como a soma de 1 com infinitos 1/2 diverge (basta juntar o 1/2 aos pares):

S > 1 + 1 + 1 + 1 + 1...

A soma de infinitos termos da serie harmonica tb diverge.

Essa eh a prova de Oresme, redescoberta independentemente por Bernoulli.

[]s,

Roberto Takata





##### ##### #####

Para saber mais visite
http://www.ciencialist.hpg.ig.com.br


##### ##### ##### #####
Links do Yahoo! Grupos










--
No virus found in this incoming message.
Checked by AVG Anti-Virus.
Version: 7.0.300 / Virus Database: 265.6.13 - Release Date: 16/01/2005




--
No virus found in this outgoing message.
Checked by AVG Anti-Virus.
Version: 7.0.300 / Virus Database: 265.6.13 - Release Date: 16/01/2005



SUBJECT: Re: Essas, pois, são as inquietudes tolas de mais um bobo.(2)
FROM: "Sergio M. M. Taborda" <sergiotaborda@yahoo.com.br>
TO: ciencialist@yahoogrupos.com.br
DATE: 22/01/2005 14:57


--- Em ciencialist@yahoogrupos.com.br, "rayfisica" <rayfisica@y...>
escreveu
>
> Com certeza o assunto já foi tratado aqui, como sou novo na casa...
>
> Tenho ouvido tanta besteira com o nome de ciência e em nome da
> ciência (apesar de minhas limitações) que desconfio de tudo,
> por
> favor, alguém pode explicar isso para mim de um jeito bem simples.

Uma teoria cientifica consta dos seguintes passos

1:ideia , insight: alguem em algum ponto tem uma ideia para explicar
alguma coisa. Pode acontecer que pessoas diferentes em lugares
tenham a mesma ideia.
2:desenvolvimento: que pode ser teorico , experimental ou uma
combinação de ambos. Aqui a ideia é esenvolvida para chegar numa
explicação aplicavel a vários casos. A ideia desenvolve-se tb numa
forma universal de aplicação. Essa forma pasa pela criação de
postulados , regras, leis, convensões, etc.. que forma uma estrutura
sempre aplicável. Um particular isso tudo pode traduzir-e em
matemática, mas não é obrigatorio.
3: teste. Ema vez desenvolvida a teoria é apresentada ao publico em
geral e submete-se a teste. O teste so é válido se for feito por
outras pessoas que não as envolvidas nos passos anteriores. Escolhe-
se um caso simples em que a teoria possa ser aplicada ,e aplica-se.
Fazem-se as medições necessárias e tira-se conclusões.
4: aceitação. Este passo não é imediato. Acontece quando o passo
anteiror é repetido diversas vezes e nunca chegamos numa contradição
da teoria.

No desenvolvimento de uma teoria nunca existe apenas uma pessoas,
por isso é uma imbecilidade querer atribuir as teorias a uma so
pessoa. Essa imbecilidade é prepetudada por quem não entende nada do
assunto e precisa de um icone, alguem para por no pedestal, alguem a
quem cobrar responsabilidade, esse tipo de coisas pateticas que a
nossa sociedade faz, mas que no caso da ciencia são futeis.

Desde a epoca em que Maxwell desenvolveu as eq so e.m. que sabemos
que E=mc^2 , mas essa formula é diferente do E=mc^2 de Eisntein. É
isso que as pessoas ingenuas e desinformadas não entendem e não
querem entender. tlv elas não saibam que precisam entender e por
isso que são ingenuas , e não têm meios para entender mesmo que
queiram, por isso que são desinformadas.
Se qq bobo pudesse ser fisico não era preciso estudar muito e fazer
faculdade para o ser. É peciso ter talento, claro, mas é preciso ter
treino tb.

Então vejamos. Do ponto de vista do Electromagnetismo, que é ponto
de vista de Lorentz , Poincaré e outros. E=mc2 significa que :
A energia do campo e.m. é proprocional a uma massa. A energia do
campo e.m é a energia contida nos campos electrico e magnetico.
Pelas mesmas ideias podemos deduzir que o momento linear do campo
e.m seria p = mc , sendo m = E/c2 , ou seja, que a energia do campo
é proporcional ao seu momento (ou seja, que a energia é puramente
cinetica) E = pc.

Estas formulas são válidas para o campo electromagnetico , MAS não
são válidas para qq corpo em geral. É aqui que entra eisntein. Na
teoria de eisntein , dados certos postulados preliminares , conclui-
se que E=mc2 para qq corpo MAS o que significam agoras sa letras?
Esta formula significa que a Energia (total, não paenas a cinetica)
medida por um observador A de um corpo B é propocional à massa que A
mede de B, que por sua vez não é igual à massa que B mede se si
mesmo.
A complexidade de raciocinio para chegar a E=mc2 via eisntein é
muito maior que via poincaré. E as formulas não são equivalentes.
Embora tenham a mesma grafia , se usem as mesmas letras e tudo o
mais, elas não significam a mesma coisa !!!!!

E qq outra discussão sobre se é poincaré ou Eisntein a inventar
E=mc2 é completamente inepta , idota , edionda e intragável.

Eisntein consegiu traçar todo o perfil de uma teoria que explica pq
a formula é aquela enquanto os outros apenas usam de matemática e
ideias ah doc.

Sérgio Taborda





SUBJECT: Re: [ciencialist] Essas, pois, são as inquietudes tolas de mais um bobo.(2)
FROM: "Alvaro Augusto \(E\)" <alvaro@electraenergy.com.br>
TO: <ciencialist@yahoogrupos.com.br>
DATE: 22/01/2005 15:57

Caro Ray,

Esse assunto já apareceu aqui, e mais de uma vez. Trata-se de parte do esforço para "provar que Cesar Lattes é grande tentando provar que Einstein não era".

Bobagem. Pura perda de tempo.

Einstein não andava de bicicleta? Não dirigia um carro? Pulava a cerca e submetia as esposas a uma vida de quase empregada doméstica? E daí? Isso tudo só mostra que Einstein era mais humano do que pensamos, e não menos, além de não afetar em nada as teorias que ele criou.

A relatividade geral não é aceita porque gostamos de Einstein. Ela é aceita porque passa nos testes. Existem outras 40 teorias concorrentes, e a relatividade geral é a única que passa em todos os testes.

[ ]s

Alvaro Augusto


----- Original Message -----
From: rayfisica
To: ciencialist@yahoogrupos.com.br
Sent: Saturday, January 22, 2005 7:54 AM
Subject: [ciencialist] Essas, pois, são as inquietudes tolas de mais um bobo.(2)



Com certeza o assunto já foi tratado aqui, como sou novo na casa...

Tenho ouvido tanta besteira com o nome de ciência e em nome da
ciência (apesar de minhas limitações) que desconfio de tudo,
por
favor, alguém pode explicar isso para mim de um jeito bem simples.


http://observatorio.ultimosegundo.ig.com.br/artigos.asp?cod=286OFC003


TABUS DA CIÊNCIA
Seria Einstein uma fraude?

Osório Barbosa (*)

Por necessidade de serviço (estou escrevendo um romance sobre a
história do Direito, onde, para melhor contá-la, senti
necessidade de
que o personagem viajasse no tempo), como já tinha ouvido falar e
lido de relance sobre tal possibilidade, resolvi pesquisar na
internet sobre o assunto, mais especificamente sobre a Teoria da
Relatividade.

Para minha surpresa, deparei-me com a entrevista de César Lattes,
intitulada: "Albert Einstein é uma farsa".

Mas quem é César Lattes?

Fiquei sabendo que é um físico brasileiro que esteve prestes a
ganhar, por duas vezes, o Prêmio Nobel de Física, por ter
descoberto
o méson pi, que é uma partícula integrante do núcleo do
átomo. Ela é
apontada como uma das principais responsáveis pela integridade do
núcleo, impedindo-o de desintegrar-se. Se o núcleo do átomo
fosse
formado apenas pelas partículas negativa e positiva, ele se
desintegrava, me explicou mais tarde o professor da Unicamp.

Depois de ler a entrevista de Lattes, fui ler um livro de Ronaldo
Rogério de Freitas Mourão, cujo título é Explicando a
Teoria da
Relatividade (Ediouro, 1997). Nele está dito:

"A medida do desvio da luz das estrelas ao passar próximo ao Sol.

O primeiro valor do desvio da luz, previsto por Einstein, em 1911,
era de 0,875 segundo de arco. (p. 14) Neste intervalo, Einstein, ao
rever sua teoria, duplicou a previsão: a deflexão deveria ser
de 1,75
segundo de arco, e não de 0.875. A razão desta alteração
deve-se ao
fato de não ter sido considerada a curvatura de espaço no
cálculo
anterior, o que só foi possível com a conclusão da teoria da
relatividade geral, em 1915.

Analisando a origem das idéias de Einstein, Feuer faz um paralelo
com
o matemático francês H. Poincaré (1854-1912), que tinha tudo
para ser
o criador da teoria da relatividade, como se pode deduzir da leitura
de seus livros. Todavia, do ponto de vista pessoal, possuía o
condicionamento do francês sempre prudente, comprometido com o
establishment e os colegiados de professores. A grande diferença
entre Poincaré e Einstein seria, segundo Feuer, o não-
comprometimento, que o colocava numa posição de observador
privilegiado. (p. 22)."

O parágrafo acima consta de uma nota de rodapé!

Resolvi, por intermédio de um conhecido (que tem laços com a
Unicamp), tentar localizar o professor Lattes, a fim de ouvir dele se
confirmava a entrevista e que me explicasse as razões de sua
afirmação a respeito do renomado Einstein.

Meu conhecido me passou o telefone do professor Lattes. Ele me
atendeu gentilmente e marcamos uma entrevista. Fui até sua casa em
Campinas. Lá, ele não só confirmou a entrevista, como
acrescentou
outros pontos que não vêm ao caso. Perguntei ao professor
Lattes de
onde ele tirava tanta convicção para fazer aquelas afirmativas
sobre
Einstein (uma vez que um colega de trabalho me disse que poderia ser
inveja, ou o dizer por dizer, sem provas. É claro que não disse
isso
ao físico).

Ele, então, me repassou duas fontes de suas afirmações:

1ª) Cópia de uma ata de 1906 onde Poincaré expôs a
teoria, num
congresso de cientistas, mais precisamente, Sur la dynamique de
l'électron, publicado pelo Circolo Matematico di Palermo, t. 21,
p.
129-176, em 1906. Se a publicação é de 1906, segundo Lattes, o
trabalho de Poincaré é do ano anterior, portanto, de 1905.

Como Einstein apresentou seu trabalho em 1911, ou seja, seis anos
após o de Poincaré (fato que é confirmado por Ronaldo
Rogério),
passei a observar as afirmações de Lattes.

2ª) Cópia do livro História das teorias do éter e da
eletricidade, de
Sir Edmundo Whittakker R.R.S. (Humanities Press, capítulo I, 1973,
Nova York) o qual chega à mesma conclusão de Ronaldo
Rogério. Este
autor cita, abertamente, Lorentz e Poincaré, como os precursores.
Poincaré, por exemplo, em 1900, já apresentava a famosa
fórmula:
E=mc2.

Diz, especificamente, Whittakker:

"Como foi visto, J. J. Thomson em 1881 chegou ao resultado que um
condutor esférico carregado movendo-se em linha reta comporta-se
como
se tivesse uma quantidade de massa adicional (4/3 c2) vezes a energia
de seu campo eletrostático. Em 1900 Poincaré, referindo-se ao
fato
que no éter livre o momento eletromagnético é (1/c2) vezes o
fluxo
Poynting de energia, sugeriu que energia eletromagnética pudesse
possuir densidade de massa igual a (1/c2) vezes a densidade da
energia: o que quer dizer, E= mc2 onde E é energia e m é massa:
e ele
apontou que se isto fosse assim, então um oscilador Hertz, o qual
envia energia eletromagnética preponderantemente em uma
direção,
deveria recuar como os revólveres fazem quando são disparados.
Em
1904 F. Hasenörl (1874-1915) considerou uma caixa oca com paredes
perfeitamente reflexivas preenchida com radiação, e descobriu que
quando estava em movimento, há uma adição aparente à sua
massa, com
valor (8/3c2) vezes a energia possuída pela radiação quando a
caixa
está em repouso: no ano seguinte ele corrigiu isto para (4/3c2)
vezes
a energia possuída pela radiação quando a caixa está em
repouso; quer
dizer, ele concordou com a equação E = 3/4mc2 de J. J. Thomson ao
invés de com a fórmula E= mc2 de Poincaré. Em 1905 A.
Einstein
afirmou que quando um corpo está perdendo energia em forma de
radiação sua massa é diminuída aproximadamente (isto é,
desprezando
quantidades da quarta ordem) por (1/c2) vezes a quantidade de energia
perdida. Ele apontou que não é essencial que a energia perdida
pelo
corpo deveria consistir em radiação, e sugeriu a conclusão
geral, em
concordância com Poincaré, que a massa de um corpo é uma
medida do
seu conteúdo de energia: se a energia muda por E ergs, a massa
muda
no mesmo sentido por (E/c2)gramas. No ano seguinte ele alegou que
esta lei é a condição necessária e suficiente e que a lei da
conservação do movimento do centro de gravidade deveria ser
válida
para sistemas nos quais processos eletromagnéticos bem como
mecânicos
estejam ocorrendo.

Em 1908 G. N. Lewis provou, por meio da teoria da pressão
radiativa,
que um corpo que absorve energia radiante aumenta sua massa de acordo
com a equação

dE = c2dm

e afirmou que a massa de um corpo é uma medida direta de sua
energia
total, de acordo com a equação

E = mc2.

Como vimos, Poincaré havia sugerido esta equação mas não
havia
praticamente dado prova, enquanto Einstein, que também a sugerira,
havia feito prova (a qual, no entanto, foi divulgada somente como
aproximada) para um caso particular: Lewis encarou-a como uma
equação
exata, mas sua prova não era de caráter geral. Lewis, no
entanto,
apontou que se este princípio é aceito, então na equação
de Planck em
1906."

Depois disso, li um artigo do professor Marcelo Gleiser (publicados
no caderno Mais!, da Folha de S. Paulo, de 30/5/04),
intitulado: "Einstein, ícone da ciência".

Também na Folha (6/6/04) li na coluna do jornalista Luís Nassif
que
ele aponta o físico César Lattes como "um quase ganhador " do
Prêmio
Nobel.

As últimas afirmações embaralharam a posição que pretendo
adotar!

Então resolvi trocar e-mails com o professor Gleiser, que
gentilmente
me respondeu. No entanto, apenas contribuiu para aumentar minhas
dúvidas.

Enviei a ele os posicionamentos do professor Lattes e o apoio ao seu
conhecimento por Nassif, concitando-o: se possível, gostaria que o
senhor me explicasse o assunto, uma vez que todos só falam de
Einstein como autor da citada teoria.

Recebi a seguinte resposta:

"caro sr., a discussão sobre a autoria da teoria da relatividade
passa por altos e baixos. Não existe a menor dúvida que foi
Einstein
o autor da teoria. O que sabemos é que Lorenz e Poincaré
estavam já
pensando no assunto e obtiveram algumas das fórmulas que Einstein
re-
obteve. (Ele não sabia delas). Mas o que faltou a Poincaré e
Lorenz
foi a interpretação apropriada da teoria. Poincaré estava
próximo,
mas não existe um trabalho seu que se compare à clareza do de
Einstein. Mais ainda, lembre-se que a teoria da relatividade tem 2
partes. O sr. está citando apenas a especial. A teoria geral, que
levou à redefinição da gravidade como a curvatura do espaço,
é
completamente de autoria única do Einstein. abraço, Marcelo
Gleiser."

Como a teoria é complicada para a maioria dos mortais, não
consegui
entender, confrontando a resposta com o disse Whittakker:

** Se não existe a menor dúvida de que foi Einstein o autor da
teoria, por que a discussão sobre a autoria passa por altos e
baixos?

** Se a famosa fórmula (E= mc2) data de 1900, por que ela é
atribuída
a Einstein em 1911?

** Re-obteve não significa obter algo já obtido?

** Mas o que faltou a Poincaré e Lorenz foi a interpretação
apropriada da teoria; não é contraditório em relação ao
que afirma
Ronaldo Rogério quando diz: "Poincaré (1854-1912), que tinha
tudo
para ser o criador da teoria da relatividade, como pode se deduzir da
leitura de seus livros. Todavia, do ponto de vista pessoal,
possuía o
condicionamento do francês sempre prudente, comprometido com o
establishment e os colegiados de professores."

** Por fim, segundo Lattes, inexiste diferença entre as chamadas
Teoria da Relatividade geral e especial, e que Einstein chegou a
confundir: medida com grandeza.

Sei que é complicado para os que entendem a física explicar a
neófitos conceitos que somente eles entendem. Muitos o fazem,
até,
para manter o hermetismo de seus saberes e assim não dividi-los;
outros, como era o caso do meu professor de Física do cursinho em
Manaus, não sabia mesmo, apenas aprendeu a deduzir umas
fórmulas e
assim ia ensinando aos candidatos das áreas de humanas no
vestibular.

Muitas inquietudes

De qualquer modo, minhas dúvidas continuam sem respostas e, creio,
aqueles que escrevem, especialmente em jornais e revistas, têm de
ensinar aos seus leitores.

Outro ponto que me leva a um pedido de esclarecimento sobre Einstein
é aquele, também trazido por Lattes, segundo o qual:

Einstein, após concitar o presidente americano Roosevelt a
construir
a bomba atômica, não participou dos trabalhos. Segundo Lattes,
porque
não sabia.

O contraditório disso é que depois se disse pacifista.

Numa resenha do livro Einstein - o viajante da relatividade na
América do Sul (revista Scientific American Brasil, março 2004,
p.
96), está dito:

a) "... Einstein, tentando romper com a tradição francesa..."
(isso
me remeteu a Poincaré);

b) "Nessa época a pesquisa científica na Argentina estava muito
à
frente do Brasil e o convite a Einstein era evidência disso. Mas
lá,
como aqui, enfrentou detratores desinformados da relatividade". Esse
é um belo argumento de autoridade (detratores desinformados da
relatividade). Quem não concorda é detrator e desinformado. Mas
seria
esse o caso de Lattes, Ronaldo Rogério e Whittakker?

c) "Einstein voltou ao Brasil na tarde de 4 de maio. As gafes não
foram poucas nem perdoáveis. Na primeira conferência, no Clube
de
Engenharia, atulhado de gente, o calor ficou insuportável e as
janelas foram abertas. A acústica que não era boa ficou pior, o
que,
na avaliação de Einstein, levou a uma exposição
ininteligível". Isso
me recordou do homem que falava javanês.

d) "No Brasil positivista Einstein enfrentou - e obviamente
não
respondeu - críticas ingênuas e desinformadas de
autoridades
intelectuais como o jurista Pontes de Miranda". Por que não
esclarecer a quem não sabe? Quem critica é sempre ingênuo e
desinformado?

e) Diogo Mainardi também resenhou o livro de Alfredo Tiomno
Tolmasquim (revista Veja, nº 1.860, de 30/6/04), trazendo a
informação de que "a teoria da relatividade ganhou sua primeira
confirmação empírica na cidade cearense de Sobral, onde
cientistas
ingleses fotografaram algumas estrelas num eclipse solar". Essas
fotos/provas, dizem, sumiram!

Sobre o ocorrido em Sobral/Ceará/Brasil e a fama de Einstein,
veja-se
o que disseram Lattes e Marcos Cesar Danhoni Neves, ao Jornal da
Unicamp, nº 165, de agosto/2001:

"Danhoni - É essa adesão ao modelo tido como real. Quem fez
a fama do
Einstein foi o Arthur Eddington (astrônomo e físico inglês,
1882-
1944).

Lattes - Esse é um palhaço puro.

Jornal da Unicamp - O Eddington é o inglês...

Danhoni - O Eddington mandou uma equipe para o Brasil em 1919, em
Sobral, e outra acompanhou na Guiné os melhores pontos para se
observar um eclipse. É só durante um eclipse que você vê
as estrelas
de fundo e pode comparar se elas estão no mesmo ponto ou se a luz
delas está defletida ou não. Só que ele pegou as chapas com
desvio
acima ou abaixo do previsto por Einstein e desconsiderou. Fez uma
análise estatística de erros observacionais e experimentais e
aquilo
se encaixava dentro do previsto por Einstein. Isso fez a fama de
Einstein."

Será que ambos são irrelevantes e tolos?

Tudo isso fez aflorar outras dúvidas:

a) Dizem que a ex-esposa de Einstein (Mileva Maric) era quem fazia os
cálculos matemáticos para ele. Após a separação do casal,
nada mais
foi produzido por ele.

b) Era incapaz de dirigir uma bicicleta, ou melhor, dava umas
pedaladas depois de idoso.

c) Sempre se antecipa ou se justifica qualquer possível
questionamento sobre o cientista, tanto assim que na Revista da Folha
nº 627, de 4/7/04, na matéria intitulada "Escreveu, não
leu", p. 9,
colhe-se o seguinte:

"... a dislexia foi diagnosticada pela primeira vez em 1896, pelo
neurologista inglês Pringle Morgan, que a chamou de cegueira
visual."

"Entidade e associações costumam listar famosos supostamente
vítimas
do distúrbio, como o cientista Albert Einstein..."

A revista, nesse caso, foi crítica ao usar o "supostamente", já
que
outras dão como fato certo. Se a dislexia foi descoberta em 1896,
e
tendo Einstein falecido em 1955, teria sido possível que ele tenha
se
submetido ao diagnóstico? Seria interessante se quem sabe pudesse
esclarecer e que não ficasse tudo na base do "é porque é".

Se em 1900, a fórmula já era E= mc2, por que em 1911 E= mc2
passou a
ser diferente? Se Einstein não era fraudador, era, então, um
plagiador? Por que a imprensa endeusa Einstein e ignora quase que
totalmente César Lattes?

Essas, pois, são as inquietudes tolas de mais um bobo.

(*) Procurador da República, mestre em Direito Constitucional e
curioso



[As partes desta mensagem que não continham texto foram removidas]



SUBJECT: Re: Unidades de medida: símbolo e nome da grandeza
FROM: Maria Natália <grasdic@hotmail.com>
TO: ciencialist@yahoogrupos.com.br
DATE: 22/01/2005 19:09



Ói, velhinho:




























FIM


--- Em ciencialist@yahoogrupos.com.br, "Silvio" <scordeiro@t...>
escreveu
> Tia:
>
> Ubu em tupi é "terra"
> Capivara é o rol de crimes listados nos compoutadores da polícia.
>
> silvio, phylólogo.
>
>





SUBJECT: Mulheres: Cozinha e na cama, já!!!!!!!!!!
FROM: Maria Natália <grasdic@hotmail.com>
TO: ciencialist@yahoogrupos.com.br
DATE: 22/01/2005 19:13


Esta mensagem tem muito de Ciência embora se destine a agitar as
massas machistas da lista.
http://www.president.harvard.edu/speeches/2005/womensci.html
No comments.
Um abração
da Maria Natália





SUBJECT: Re: Essas, pois, são as inquietudes tolas de mais um bobo.(2)
FROM: Manuel Bulcão <manuelbulcao@uol.com.br>
TO: ciencialist@yahoogrupos.com.br
DATE: 22/01/2005 19:46


--- Em ciencialist@yahoogrupos.com.br, "rayfisica" <rayfisica@y...>
escreveu

> por favor, alguém pode explicar isso para mim de um jeito bem
simples.

> TABUS DA CIÊNCIA
> Seria Einstein uma fraude?

Manuel: Oi,

Vou dar meu pitaco sobre esse assunto tão controvertido:

A RELATIVIDADE GALILEANA: De acordo com esse princípio, as leis da
física clássica, galileico-newtonianas, não variam quando se passa
de uma estrutura referencial estacionária para uma outra em
movimento.


A RELATIVIDADE ESPECIAL SEGUNDO LORENZ: Por meio das "transformações
de Lorenz", o estranho resultado da experiência de Michelson-Morley
é explicado em termos de contrações físicas ou distorções de objetos
que se deslocam "no éter". A versão de Lorenz, portanto, pressupõe o
éter (aliás, foi o modelo matemático que conferiu à hipótese do éter
uma sobrevida), bem como preserva a noção de tempo absoluto da
mecânica de Newton.

O PRINCÍPIO DA RELATIVIDADE ESPECIAL POR POINCARÉ E EINSTEIN:
Poincaré e Einstein observaram "de uma forma independente" que a) as
equações de Newton e Maxwell combinadas não satisfazem a
relatividade galileana; b) esse fato fez com que ambos especulassem
sobre a possibilidade de uma teoria da relatividade "especial"
segundo a qual as equações de Maxwell também permaneceriam
inalteradas quando se passa de um marco referencial estacionário
para um outro em movimento, caso em que a velocidade da luz seria um
limite intransponível; c) explorando essa possibilidade, Poincaré e
Einstein inferiram "de forma independente" várias implicações
físicas do princípio da relatividade especial, como, por exemplo, a
relatividade da simultaneidade.

Ocorre que as regras para compatibilizar o eletromagnetismo com a
relatividade galileana são, por sua vez, "incompatíveis" com a
física de Newton. Trabalho de Sísifo: para tapar o buraco, cava-se
outro.

Logo, de duas uma: ou modifica-se a mecânica tal como era concebida
na época ou então se forja um modelo explicativo em que o princípio
da relatividade é descartado.

Poincaré não era físico, mas um matemático (com todas as virtudes e
os vícios dos matemáticos) e, sendo a matemática a "ciência do
possível" (Bertrand Russell), tendia ele a considerar ambas as
alternativas apenas como "possibilidades lógico-matemáticas". Por
ser matemático e não um cientista empírico, não se empenhou muito em
conferir a suas construções teóricas um poder preditivo ou em
formular testes empíricos para elas, e era isso o que mais o
diferenciava de Albert Einstein. (a respeito disso, vide R. Penrose;
A Mente Nova do Rei; A Relatividade Especial de Einstein e
Poincaré). Parece que, devido a sua maior familiaridade e
proximidade com a "forma" matemática, "Poincaré morreu sem entender
o `conteúdo' físico da relatividade especial" (Einstein, "in" A.
Pais: "The science and life of Albert Einstein").

De resto, Foi Einstein (e não Poincaré) que concluiu que o princípio
da relatividade especial só faz sentido se o espaço e o tempo forem
considerados como aspectos de uma mesma entidade, o "espaço-tempo
quadridimencional". Mas – justiça seja feita – Einstein não teria
chegado a esta conclusão sem a ajuda de um outro grande matemático,
o geômetra Hermann Minkowski, que aliás foi seu professor na
Politécnica de Zurique. (o espaço-tempo da teoria da relatividade
nada mais é que o espaço-tempo de Minkowski).

Bem, como sou um leigo (apesar de petulante), devo ter dito uma ruma
de besteira. Contestem-me, por favor, mas não é preciso humilhar. :-(

Abraços,
Manuel Bulcão





SUBJECT: Re: [ciencialist] Re: Essas, pois, são as inquietudes tolas de mais um bobo.(2)
FROM: "JVictor" <jvoneto@uol.com.br>
TO: <ciencialist@yahoogrupos.com.br>
DATE: 23/01/2005 00:04

Rayfisica/Sérgio,

Pegando uma carona no lúcido, correto e pertinente raciocínio do Taborda, que enfoca muito bem as nuances envolvidas nas teorias de Lorentz e Poincaré de um lado, e a visão de Einstein, de outro, gostaria de tecer alguns comentários a respeito, se me permitem. Quando digo que Einstein viu um palmo diante do nariz a mais que todos os outros de sua época, quero dizer exatamente isso que Taborda destacou, a respeito do significado mais profundo de coisas relativas ao espaço e o tempo, com todas suas peculiaridades, postas a descoberto, pelo menos a nível de uma teoria que descreva consistentemente eventos até então obscuros.
Bom, mas quero comentar brevemente as falácias de alguns grandes homens de ciência, quando dizem que Einstein plagiou Poincaré e Lorentz, pois alegam, como prova, que não há qualquer menção, nos trabalhos de Einstein, ao trabalho de Poincaré e Lorentz nessa área, que não são citados por Einstein em seus paper´s, embora suas teorias(Poincaré e Lorentz), do ponto de vista experimental, conduzam aos mesmíssimos resultados experimentais que a teoria vista segundo este último.
Então, não mencionar os trabalhos anteriores está sendo considerado como uma prova de plágio, pelo menos na visão de alguns, como o Prof. Lattes e outros, como os que fazem o Autodinamics, sociedade criada em torno das teorias de um certo argentino, Dr.Ricardo Carezani, que contesta a TR e propõe outra.
Como diz o taborda, uma teoria, por mais original que seja, não é, em geral, obra de uma mente só. Alguém já questionou, levantou a questão, depois deixou prá lá, outro retomou o fio da meada, e assim por diante. É como se houvesse um inconsciente coletivo que, de certa forma, mantivesse acesa a chama da dúvida e os indícios da necessidade de mudança. Até que, numa dada época, alguém enxerga com clareza todas as incidências das questões e tudo fica claro como um dia de sol.
Assim, a teoria da relatividade não foi, de jeito nenhum, uma excessão. Até onde sei, tudo começou em 1887, com Woldemar Voight(1850-1919), da Universidade de Gottingen, visto hoje em dia como o não celebrado heroi da relatividade especial! A bem da verdade histórica, a concepção de uma velocidade universal de luz, c, e de um tempo relativo, foi concebido e proposto naquela ano, pelo Prof. Voight, que era um físico de primeira linha. Voight, em função disto, postulou a invariância da lei de propagação de ondas luminosas para derivar as transformações quadri-dimensionais do espaço e do tempo, com vistas a entender o deslocamento Doppler dos comprimentos de onda e das frequências associadas. Claramente, isto foi um extraordinário, fenomenal, insight para os tempos de então. Um espanto.
As transformações desenvolvidas por Voight diferem das de Lorentz unicamente por um fator constante(Conforme discutido no paper de Andreas Ernest e Jong-Ping Hsu: " First Proposal of the universal speed of light por Voight in 1887". Neste trabalho, ao qual tive acesso, e lí, os autores descrevem e comentam toda a teoria, sustentada por um belo, simples e preciso, aparato matemático. Vale a pena ler o paper. Poderá ser encontrado em "Lorentz and Poincaré Invariance-100 years of relativity", de Jong-Ping Hsu e Yuan-Zhong Zhang). Infelizmente, como diz Ping, " o trabalho pioneiro de Voight foi condenado a sofrer destino similar ao que recebeu o trabalho científico de Leonardo da Vinci." Acrescenta, ainda: "Tanto um como o outro nasceram cedo demais e então não representaram maiores influências no desenvolvimento científico de seus dias". É como dizer que suas mentes estavam muito à frente das mentes e das concepções de então.
Ora, quanto a isto não há dúvidas. O assunto já zanzava pelos meios científicos, havia publicações como o paper acima, etc.

Contudo, e tão somente contudo, nem Poincaré, nem Einstein e muitos outros, fizeram qualquer referência ao paper de Voight, que tratava dos mesmos assuntos, ainda que em fase seminal, que já haviam sido pensados e publicados! Lorentz, em 1912, Reanalisando o seu trabalho de 1904, " Fenômenos eletromagnéticos num sitema que se move com qualquer velocidade inferior à da luz", ainda fez um ligeira referência ao trabalho de Voight, conforme transcrevo no fim deste texto, citação II. Mas só o fez em 1912, 7 aninhos após o ano miraculoso de 1905, como dizem os mais apaixonados.

E agora a questão: O fato de Einstein não mencionar Poincaré, faz dele um plagiador sem vergonha, como dizem alguns? E Poincaré não plagiou Voight? Pois Poincaré não citou, em momento algum, uma vírgula daquele trabalho prioneiro! Lorentz ainda chegou a trocar cartas com Voight, e discutiram seus trabalhos por essa via. Mas, mesmo assim, nos trabalhos de Lorentz, em seus primeiros paper´s, antes de 1904(pelo menos não achei, mas procurei e estou procurando), não há referências ao bem dito paper de Voight! Lorentz também é um plagiador, por isso? Se Einstein é, os dois outros também o são...Ora! Nessa linha de argumentos, quem seria o plagiador a respeito da invenção do cálculo: Newton ou Leibnitz?
Ao que tudo indica, parece que não era mesmo costume da época ficar mencionando trabalhos de outros, quando os seus eram de lavra própria(até mesmo modernamente, cientistas como Landau não fazem referências a outros autores. Ele só cita alguém, quando reconhece que o que o alguém diz não é considerado trivial. Ele não apresenta uma lista enorme de livros que teria lido ou pesquisado. Há autores que exibem uma lista tão grande de livros para justificar seu trabalho que dá para ficar com dúvidas a respeito de se ele, numa vida, teria tempo de ler tudo!, ou se aquilo tudo é para encher linguiça).
Os trabalhos ora comentados foram feitos de maneira independente. Não tenho dúvidas. Nem um pouquinho que seja. E mais: para mim é irrelevante se Einstein conhecia ou não o trabalho de Poincaré(que, a bem da verdade, é gigantesco). Einstein conferiu à TRR uma significação física absolutamente diferente e original, embora os postulados sejam os mesmos. Só que Einstein extraiu destes o que o outro nem sonhou! Pessoas com o nível intelectual de um Voight, Lorentz, Poincaré e Einstein, entre outros, como Moller, Tolman, Landau, não precisam plagiar quem ou que quer que seja! Ou seja, basta aprofundar um pouco, ir atrás da verdade dos fatos, que as coisas vão se esclarecendo e pondo a descoberto esses pretensos críticos que, na verdade, são ignorantes e irresponsáveis, pois tentam formar opiniões de maneira igualmente irresponsável. Apesar de alardearem seus cursos de pós, pós do pós, que acaba resultando no pó amarelado e tóxico que dissseminam. Só para completar, E=mc2 foi estabelecida pela primeira vez por Voight, antes de Poincaré. Poincaré chegou a postular a mesma coisa, mas sem qualquer prova. Mas o significado, que foi uma decorrência dos postulados de Voight, é aquele corretamente discutido por Sérgio Taborda. Nem mais nem menos.
A propósito, o artigo que deu causa ao assunto deste e-mail, foi escrito por um advogado, sem formação alguma em física ou em história da ciência. Apenas à base do ouví dizer, que lí assim, que um famoso físico brasileiro disse, que um físico escritor ficou em dúvida(ficou em dúvida em razão de não conhecer a verdade dos fatos, apesar de ser aclamado no Brasil e terra de Bush como divulgador da ciência). Não que o autor não tenha capacidade de fazer um trabalho mais próximo da realidade. Claro que tem. Ele sabe ler, e só precisa disso e mais uma consultoria técnica junto a mais de um bom físico ou matemático. Só isso! Mas, não fez assim. Portanto, o que ele fez "sequer é um artigo", como diria W.Pauli.
Existem outras teorias alternativas da Relatividade,todas conduzindo basicamente aos mesmo resultados. Mas nenhuma é tão simples e objetiva quanto a TRR, de Einstein. O livro acima citado analisa algumas, até interessantes. Quando houver oportunidade, comentarei.
Lembrem-se de que nem sempre o que é publicado na internet é verdadeiro. Há muita coisa boa. Mas há também muita porcaria. A propósito, não é demais transcrever(já o fiz antes, neste Ciencialist) duas coisinhas ditas pelos próprios interessados, Poincaré e Lorentz, que teriam sido "roubados" em suas teorias por Einstein:

I - Poincaré, sobre Einstein, em Arquitetos de Idéias-A história das Grandes Teorias da Humanidade, de Ernest R. Trattner:

"Einstein, escreveu Poincaré, é um dos espíritos mais originais que tenho conhecido. A despeito de seus verdes anos, já ocupa uma posição
distinta entre os grandes sábios desta época. O que nos causa maravilha neste moço é, sobretudo, a facilidade com que ele se adapta às novas concepções,
extraindo dela todas as conclusões possíveis. Não se apega a princípios clássicos, mas, ao encarar um problema de física, toma em consideração
todas as conjecturas possíveis. Em seu espírito isto se transforma numa antecipação de novos fenômenos, que poderão ser verificados um dia pela
experiência real. O futuro dará provas cada vez mais abundantes dos méritos de Alberto Einstein, e a universidade que lograr atraí-lo aos seu seio há de
tornar-se famosa por sua ligação com o jovem mestre".


II - Lorentz, em 1912, reanalisando o seu trabalho de 1904," Fenômenos eletromagnéticos num sistema que se move com qualquer velocidade inferior à da luz".

Lorentz, após tentar obter certas equações de transformação, que deixariam invariantes certas quantidades eletromagnéticas(usando, naturalmente, a teoria do éter), escreveu a seguinte nota(página 19 do Volume do O princípio da Relatividade(H.A.Lorentz, A.Einstein e H.Minkowski, 4a. Edição, Fundação Calouste Gulbenkian):

"Deve notar-se que, neste trabalho, eu não cheguei a atingir totalmente as equações de transofrmação da teoria da relatividade de Einstein. Nem a equação (7) nem as fórmulas (8) têm a forma dada por Einstein e, em consequência disso, não cheguei a fazer desaparecer o termo (-wu^linha_x/c2) na primeira equação (9). pelo que não conseguí dar às fórmulas (9) uma forma rigorosamente válida para um sistema em repouso. Dependem desta circunstância os embaraços em que esbarram muitas das considerações ulteriores deste trabalho.
Pertence a Einstein o mérito de ter sido o primeiro a enunciar o princípio da relatividade como uma lei geral, rígida e exacta.

Acrescento a isto a observação de que Voight já no ano de 1887(Gottinger Nachrichten, pág. 41), num trabalho <<Uber das Dopplersche Pirnzip>>(Descrito no livro acima comentado, do Jong-Ping) aplicou as equaçôes da forma " del phi - (1/c2) parcial^2 phi/parcial t^2 = 0 "
uma transformação que é equivalente à contida nas equações (4) e(5) do meu trabalho".

Estas foram coisas proferidas e escritas por Poincaré e Lorentz, sobre o cara acusado de plagiá-los!. Tirem suas conclusões.


Sds,

Victor.


----- Original Message -----
From: Sergio M. M. Taborda
To: ciencialist@yahoogrupos.com.br
Sent: Saturday, January 22, 2005 2:57 PM
Subject: [ciencialist] Re: Essas, pois, são as inquietudes tolas de mais um bobo.(2)



--- Em ciencialist@yahoogrupos.com.br, "rayfisica" <rayfisica@y...>
escreveu
>
> Com certeza o assunto já foi tratado aqui, como sou novo na casa...
>
> Tenho ouvido tanta besteira com o nome de ciência e em nome da
> ciência (apesar de minhas limitações) que desconfio de tudo,
> por
> favor, alguém pode explicar isso para mim de um jeito bem simples.

Uma teoria cientifica consta dos seguintes passos

1:ideia , insight: alguem em algum ponto tem uma ideia para explicar
alguma coisa. Pode acontecer que pessoas diferentes em lugares
tenham a mesma ideia.
2:desenvolvimento: que pode ser teorico , experimental ou uma
combinação de ambos. Aqui a ideia é esenvolvida para chegar numa
explicação aplicavel a vários casos. A ideia desenvolve-se tb numa
forma universal de aplicação. Essa forma pasa pela criação de
postulados , regras, leis, convensões, etc.. que forma uma estrutura
sempre aplicável. Um particular isso tudo pode traduzir-e em
matemática, mas não é obrigatorio.
3: teste. Ema vez desenvolvida a teoria é apresentada ao publico em
geral e submete-se a teste. O teste so é válido se for feito por
outras pessoas que não as envolvidas nos passos anteriores. Escolhe-
se um caso simples em que a teoria possa ser aplicada ,e aplica-se.
Fazem-se as medições necessárias e tira-se conclusões.
4: aceitação. Este passo não é imediato. Acontece quando o passo
anteiror é repetido diversas vezes e nunca chegamos numa contradição
da teoria.

No desenvolvimento de uma teoria nunca existe apenas uma pessoas,
por isso é uma imbecilidade querer atribuir as teorias a uma so
pessoa. Essa imbecilidade é prepetudada por quem não entende nada do
assunto e precisa de um icone, alguem para por no pedestal, alguem a
quem cobrar responsabilidade, esse tipo de coisas pateticas que a
nossa sociedade faz, mas que no caso da ciencia são futeis.

Desde a epoca em que Maxwell desenvolveu as eq so e.m. que sabemos
que E=mc^2 , mas essa formula é diferente do E=mc^2 de Eisntein. É
isso que as pessoas ingenuas e desinformadas não entendem e não
querem entender. tlv elas não saibam que precisam entender e por
isso que são ingenuas , e não têm meios para entender mesmo que
queiram, por isso que são desinformadas.
Se qq bobo pudesse ser fisico não era preciso estudar muito e fazer
faculdade para o ser. É peciso ter talento, claro, mas é preciso ter
treino tb.

Então vejamos. Do ponto de vista do Electromagnetismo, que é ponto
de vista de Lorentz , Poincaré e outros. E=mc2 significa que :
A energia do campo e.m. é proprocional a uma massa. A energia do
campo e.m é a energia contida nos campos electrico e magnetico.
Pelas mesmas ideias podemos deduzir que o momento linear do campo
e.m seria p = mc , sendo m = E/c2 , ou seja, que a energia do campo
é proporcional ao seu momento (ou seja, que a energia é puramente
cinetica) E = pc.

Estas formulas são válidas para o campo electromagnetico , MAS não
são válidas para qq corpo em geral. É aqui que entra eisntein. Na
teoria de eisntein , dados certos postulados preliminares , conclui-
se que E=mc2 para qq corpo MAS o que significam agoras sa letras?
Esta formula significa que a Energia (total, não paenas a cinetica)
medida por um observador A de um corpo B é propocional à massa que A
mede de B, que por sua vez não é igual à massa que B mede se si
mesmo.
A complexidade de raciocinio para chegar a E=mc2 via eisntein é
muito maior que via poincaré. E as formulas não são equivalentes.
Embora tenham a mesma grafia , se usem as mesmas letras e tudo o
mais, elas não significam a mesma coisa !!!!!

E qq outra discussão sobre se é poincaré ou Eisntein a inventar
E=mc2 é completamente inepta , idota , edionda e intragável.

Eisntein consegiu traçar todo o perfil de uma teoria que explica pq
a formula é aquela enquanto os outros apenas usam de matemática e
ideias ah doc.

Sérgio Taborda





##### ##### #####

Para saber mais visite
http://www.ciencialist.hpg.ig.com.br


##### ##### ##### #####


Yahoo! Grupos, um serviço oferecido por:







------------------------------------------------------------------------------
Links do Yahoo! Grupos

a.. Para visitar o site do seu grupo na web, acesse:
http://br.groups.yahoo.com/group/ciencialist/

b.. Para sair deste grupo, envie um e-mail para:
ciencialist-unsubscribe@yahoogrupos.com.br

c.. O uso que você faz do Yahoo! Grupos está sujeito aos Termos do Serviço do Yahoo!.



[As partes desta mensagem que não continham texto foram removidas]



SUBJECT: Re: [ciencialist] César Lattes
FROM: "JVictor" <jvoneto@uol.com.br>
TO: <ciencialist@yahoogrupos.com.br>
DATE: 23/01/2005 00:23


----- Original Message -----
From: "Alberto Mesquita Filho" <albmesq@uol.com.br>
To: <ciencialist@yahoogrupos.com.br>
Sent: Sunday, July 18, 2004 2:51 PM
Subject: Re: [ciencialist] César Lattes


----- Original Message -----
From: "JVictor"
Sent: Sunday, July 18, 2004 10:51 AM
Subject: Re: [ciencialist] César Lattes


Mesquita: Creio que você está valorizando um bate-papo que, ao que tudo
indica, teria
sido efetuado num ambiente totalmente descontraído.

Victor: Não, Alberto. Não estou valorizando um mero bate-papa
descontraído, infelizmente.
Tais pronunciamentos vêm de muito tempo, estão bem registrados.

Mesquita: Não me consta que em
nenhuma de suas obras sérias --que você afirmou taxativamente não ter
nenhum valor ("não há trabalho de física que ele tenha escrito, que tenha
utilidade")-- ele tivesse cuspido no prato em que comeu.

Victor: Você poderia citar alguma, algum trabalho de porte onde ele defenda
com
bases sólidas seus pensamentos? Certamente, eu leria. Mas não tenho a
mínima dúvida de
que ele tomou a trilha errada, com tais disparates, nos aspectos que ora
discuto. Que ele é
de uma importância extrema no meio científico, isto está fora de questão.

Mesquita: Quanto ao Hawking, o Lattes está comentando um livro que fez
sucesso
unicamente graças ao poder da mídia. Sequer traduz o significado dos
trabalhos efetuados
pelo próprio Hawking. Não concordo com a maioria do que o Taborda disse, mas
em meio a
seu texto ele assinalou uma grande verdade: esse livro não tem nada além de
finalidades puramente lucrativas.

Victor: Ora, mas Sérgio está brincando. Quer só provocar, prá ver os
"gladiadores" se arranjam na arena.
Só pode ser.
Agora, realmente, o Uma Breve História do Tempo, foi um livro escrito numa
época em que não havia tanta profusão
de obras de divulgação. E, certamente, transformou-se num BestSeller. Todo
mundo comprou. Mas, creio, pouca gente conseguiu ler. E tenho a consciência
de que as pessoas o compraram assim, em tão grande escala, em vista do
inusitado:
um sujeito, já mundialmente famoso como cosmologista, portando uma
deficência física daquela espécie, escrever algo
difícil como é o assunto de que trata, num nível tal, que pretendia atingir
o leigo. Evidentemente, que tal objetivo não foi conseguido, o de todos
lerem, gostarem, compreenderem, essas coisas. Mas a intenção, ainda que em
segundo plano foi essa. Em primeiro plano, pode até sido para angariar
fundos, para dar suportes aos tratamentos e cuidados que precisava ter, em
face se seu mal. Mas, o que importa isso? Qual o problema com isso? O que
importa é que o livro não tinha o objetivo de ser tratado científico,
dirigido a experts, os muito dotados e versados naquelas coisas. Então, um
cara, numa condição "favorabilíisima" como a dele, descer ao nível que
desceu, num linguajar simples e até humorado, mesmo numa tradução
considerada de ruim prá la, ousar uma ventura do tipo, merece, só por isso
algum respeito, mínimo que seja. Lí o livro, como algo de divulgação, com
informações razoáveis acerca do cosmo. Honestamente, até não não ví qualquer
motivação para tanto rangir de dentes, para tanto rancor. Também, conta-se
alí a história pessoal dele, pois a muita gente interessa, talvez não a
alguns experts que acham uma babaquice. Mas o problema aí não mais é de
Hawking. Não vejo como crucificar um cara desses, por causa disso. Também,
não é uma obra para alguém pesquisar e definir a capacidade e/ou
personalidade do autor, para que se possa assacar contra ele, não da maneira
como aquí se tem visto. Não o idolatro, nem a ele ou a outro qualquer. Por
exemplo, tenho pela Teoria da Relatividade o maior respeito, por uma razão:
quanto mais a estudo, seja a restrita, seja a Geral, quanto faço contas,
mais me convenço de que está correta e que há, em sua estrutura matemática,
alguns aspectos ainda não compreendidos. Pelo seu Autor, o Einstein, e por
uma natural extensão, também tenho uma extrema admiração e respeito. Mas,
não a ponto de cegar, de idolatrar. Se surgir outra teoria que a substitua,
que a complemente, qual o problema? Até o próprio foi useiro e vezeiro em
dizer que tudo aquilo poderia até ser provisório, que aquilo tudo não era
uma verdade absoluta, intocável, como você mesmo sabe. Só que, até o
momento, ciscaram muito e fizeram muito trovão e nadinha de chuva. (!!???).
Então...
Além disso, Einstein não tem a estatura científica que tem só pela Teoria da
Relatividade, que foi apenas uma das facetas de seu gênio criativo, apenas
um detalhe, gigantesco é verdade, mas um detalhe. A obra dele é bem mais
extensa. Bem mais abrangente.

Victor: > Paciência, rapaz. Isso já é idolatria, e idolatria
> é o que mais se condena nesta lista. Como deve ser, em qualquer outro
> lugar. Sabia que é pecado mortal ser idólatra, vai pro inferno, aviso.

Mesquita: Idolatria de quem e por quem? Quanto ao inferno, se a praga
dirige-se a mim,
diria que não tenho medo, pois sou amigo do Léo. :-)

Victor: Com as perguntas acima, você já respondeu e eu entendí. Enganei-me.
Desocupe o Léo, pois você não vai pro inferno!

Mesquita: Mas idolatria por quem? Quanto ao Cesar Lattes não fui eu quem o
indicou
para o posto de um dos melhores físicos brasileiros de todos os tempos e
muito menos para dar o nome à Plataforma do CNPq. Já disse que dentre os
três "ídolos" da física brasileira (foi você quem os citou) sou mais
Schenberg e não abro. Mas nem por isso eu fico idolatrando o Schenberg. Será
que você não está "projetando" a sua possível idolatria pelo Hawking ou
pelo Einstein?

Victor: Será? Mas acho que não. Tentei responder a isso mais acima.
Quanto a Schenberg, estou com você e não abro. Embora, infelizmente, não
conheça
a obra científica dele, apenas o que falam dela. Mas lí, por duas vezes, o
seu Pequeno Grande Opúsculo,
"Pensando a Física", Editora Nova Stella, 3a. Edição.


Mesquita: Eu, particularmente, tenho uma "profunda admiração" por
quatro pensadores já falecidos: Newton, Popper, Einstein e Schenberg. Mas
isso não significa que eu deva me subjugar a suas idéias. Se puder
aproveitar algo de bom dentre o que nos legaram, ótimo. Do contrário dou
asas à imaginação e parto para minhas críticas às IDÉIAS deles, mas não a
eles como pessoas e/ou como cientistas. Pensem o que quiserem a respeito de
minhas pretensões, mas já critiquei os quatro "ídolos" que mais admiro. E
apesar disso continuo admirando-os.

Victor: Ora, esse é o próprio Alberto Mesquita. E está certo!

Victor: Mas, sinceramente, Alberto, tenho a certeza de
> você é prá lá de razoável, moço inteligente, culto, filósofo, físico
> amador e idealista, já puder ver, lendo alguns de seus trabalhos. Não
> tenho a menor dúvida de que você ainda tem jeito, tem concerto... É só uma
> questão de tempo!...

Mesquita: Grato por tudo, especialmente pelo "moço" e pela expectativa. :-).
Para os
que não sabem tenho 61 anos bem vividos e espero viver outros tantos, mas
daí a dizer que eu tenha conserto? ;-)

Victor: Mesquita, você sabe que tal colocação é para descontrair, foi um
brincadeira.
Não leve a mal.


Victor: > Aproveito, agora, o
> ensejo para confessar, de público, e é um direito que me assiste, que a
> impressão que você passa, a mim pelo mesmo, que não me considero tão
> prendado intelectual e mentalmente como você e outros notáveis da lista,
> que você, ao invés de "brincar" com os físicos modernos, como disse acima,
> você na verdade debocha e espinafra(segundo Chemello) tais profissionais,
> pertencentes ao sistema ou não, que trabalham e vivem a mecânica quântica
> e a relatividade no dia a dia, ano após ano.

Mesquita: De fato e sob esse aspecto sou reu confesso. Lembro que o termo
espinafrar
fui eu que utilizei pela primeira vez, não o Chemello. Ele me perguntou o
que isso significava e eu disse que seria debochar, ou algo do gênero. Até
brinquei, dizendo que como se tratava de física quântica, o mais correto
seria SPINafrar. E que mal há nisso? Por acaso os profissionais quânticos
são idolatras? Se sim seria bom avisarem, pois não costumo espinafrar
comportamentos religiosos, por mais absurdos que sejam. Se não, eles que se
defendam com idéias, pois em ciência quem não tem competência não se
estabelece. Sempre que "espinafro" a física quântica dou a entender porquê o
faço. Não obstante, sempre que espinafraram as minhas teorias, eu jamais
consegui sequer desconfiar onde foi que acharam que eu estava errado.

Victor: Posturas suas. Respeito e não comento.

Mesquita: Parece-me que você é muito novo na Ciencialist para ficar julgando
o
comportamento de quem está aqui há mais de cinco anos.

Victor: Tempo de permanência em qualquer atividade, não imuniza quem quer
que seja,
não o protege ou o torna inatacável. Você poderia ter 50 anos de
Ciencialist. Isto não lhe confere "autoridade" nem acrescenta atributos que
o tornem diferente de qualquer outro em qualquer nível que
Isto não importaria,
não o tornaria nem mais nem menos
Por outro lado, se os
profissionais quânticos ignoram o que eu penso, então não vejo de que
maneira posso tê-los ofendido. Que defendam suas idéias, oras. Do contrário,
"que enfiem a viola no saco". Será que, na atualidade, o único que entende
de física quântica aqui é o biólogo Takata? Onde estão os quânticos que no
passado me enfrentavam, sem conseguir convencer a ninguém?

> As suas adjetivações a respeito
> das TR e da MC, confesso novamente(menos na primeira e bastante com
> respeito à última), mexem comigo. E sei também que acontece com outros,
> que não se pronunciam ou para evitar melindres ou em razão de acharem
> descabidas as suas conotações. Isso sim, é uma falta de respeito
> generalizada, segundo meu falho achômetro.

Melindre é próprio de quem não tem argumentos. Defendam suas idéias,
derrubem meus argumentos (tenho mais de 500 mensagens aqui na Ciencialist
não respondidas a altura) ou então, como já disse acima, "enfiem a viola no
saco". Afinal, estamos numa lista de discussão. Se eu digo o que quero e
vocês tão somente ficam melindrados, então sinto muito, mas vocês merecem
mais do que tão somente serem espinafrados.

> Espero que não se zangue(devo estar errado, como disse),
> pode ser apenas impressão minha, mas... "Papagaios cultos", só porque os
> que estudam, vivenciam, ensinam ou pesquisam essas disciplinas têm
> opiniões, bastante embasadas-no caso da grande maioria-, divergentes das
> suas?

O termo "papagaio culto" que emprego não tem nada a ver com minha crítica à
física moderna, mas sim ao ensino brasileiro. Relaciona-se ao ensino de
"receita de bolo", em detrimento do despertar da criatividade. Até mesmo o
Feynman, quando esteve no Brasil, chegou a comentar algo do gênero. Qualquer
pedagogo sabe sobre o que estou falando, e recentemente o Léo também chegou
a comentar algo a respeito.

> [...] Essa é uma postura
> kantiana, que considero, no mínimo, de razoável prá cima. Kant também
> dizia que o bem dotado se basta a si próprio, assim ele não se metia.

Perfeito, mas Kant também defendia o despertar da criatividade nos jovens
estudantes. Dava a entender que a finalidade da escola seria promover a
maioridade científica. Pois é exatamente isso o que tenho defendido. Que me
importa o que Newton, Einstein, Hawking ou Lattes afirmaram? Se não
concordar com seus argumentos tenho mais que pichar as suas idéias (as
idéias, bem entendido). Agora, se os defensores dessas idéias se ofendem
com isso, e se mostram incapazes de defendê-las, tenho mais é que "tirar o
sarro" deles, e/ou "espinafrá-los", e é o que tenho feito, doa a quem doer.

> - Sei que você está fazendo um pós-graduação em História da Ciência. Lí
> tal informação por aqui mesmo.

Apenas como aluno especial. Interessei-me por uma disciplina e me dispus a
cursá-la, somente isso.

> A minha curiosidade básica é simples. Já sei que Newton hesitava em
> publicar suas descobertas. Só o fazendo após insistências de amigos
> dedicados. E agora o que está descrito acima sugere mais uma mania sua, de
> se manter no anonimato, de não aparecer, quando outros fatos sugerem que
> não era bem isso. Será que já haviam inventado o "charminho" e Newton
> adorava isso?

Creio que não. Os tempos eram outros. Dizem que devemos os Principia à
insistência do Halley, pois Newton estava se recusando a publicá-lo. No
escritório de Newton encontrou-se centenas de textos não publicados. Alguns
foram publicados após a sua morte, outros são peças de museu. Muita gente
critica a religiosidade de Newton, mas poucos sabem que para permanecer na
sua cátedra ele era obrigado, por contrato, a estudar religião, e até mesmo
a se transformar, após oito anos, numa espécie de sacerdote. Nesta época,
quase no mundo todo, a universidade estava nas mãos dos religiosos, e tinha
que se tomar muito cuidado com o que fosse publicado.

> Quando me sobrar tempo, penso também iniciar um pós em História de
> Ciência, ao menos.

Faz muito bem. "Não tenho a menor dúvida que você ainda tem jeito, tem
conserto." ,-) Quiçá um curso de história de ciência possa ser útil para
descobrirmos em que ponto da história a física atolou de vez. Há muito tempo
li num livro (creio que de um astrônomo ou talvez do Gribbin -- se alguém
leu essa passagem e puder me indicá-la ficaria bastante agradecido) que em
algum lugar da história os físicos cometeram um erro muito grave, e isto
acabou gerando o que está aí. O difícil é localizar o erro, pois não tenho
dúvidas que tanto Einstein quanto Bohr simplesmente tentaram pisar no
acelerador, com o que o veículo acabou atolando de vez. Creio que o erro
está na luz ondulatória e na idéia de fluidos elétricos, e isso é bem
anterior a Maxwell.

[ ]´s
Alberto
http://ecientificocultural.com/indice.htm
Mas indiferentemente a tudo isso, o neutrino tem massa, o elétron não é
uma carga elétrica coulombiana e a Terra se move. E a história se repetirá.



##### ##### #####
Para saber mais visite
http://www.ciencialist.hpg.ig.com.br
Para sair da lista envie um mail para
ciencialist-unsubscribe@yahoogroups.com
##### ##### ##### ##### #####
Links do Yahoo! Grupos










SUBJECT: Re: [ciencialist] César Lattes
FROM: "JVictor" <jvoneto@uol.com.br>
TO: <ciencialist@yahoogrupos.com.br>
DATE: 23/01/2005 08:07

Desconsiderem. Esta mensagem já foi postado´há um século. É que eu estava revendo algumas e a reenviei num descuido.

Victor.
----- Original Message -----
From: JVictor
To: ciencialist@yahoogrupos.com.br
Sent: Sunday, January 23, 2005 12:23 AM
Subject: Re: [ciencialist] César Lattes



----- Original Message -----
From: "Alberto Mesquita Filho" <albmesq@uol.com.br>
To: <ciencialist@yahoogrupos.com.br>
Sent: Sunday, July 18, 2004 2:51 PM
Subject: Re: [ciencialist] César Lattes


----- Original Message -----
From: "JVictor"
Sent: Sunday, July 18, 2004 10:51 AM
Subject: Re: [ciencialist] César Lattes


Mesquita: Creio que você está valorizando um bate-papo que, ao que tudo
indica, teria
sido efetuado num ambiente totalmente descontraído.

Victor: Não, Alberto. Não estou valorizando um mero bate-papa
descontraído, infelizmente.
Tais pronunciamentos vêm de muito tempo, estão bem registrados.

Mesquita: Não me consta que em
nenhuma de suas obras sérias --que você afirmou taxativamente não ter
nenhum valor ("não há trabalho de física que ele tenha escrito, que tenha
utilidade")-- ele tivesse cuspido no prato em que comeu.

Victor: Você poderia citar alguma, algum trabalho de porte onde ele defenda
com
bases sólidas seus pensamentos? Certamente, eu leria. Mas não tenho a
mínima dúvida de
que ele tomou a trilha errada, com tais disparates, nos aspectos que ora
discuto. Que ele é
de uma importância extrema no meio científico, isto está fora de questão.

Mesquita: Quanto ao Hawking, o Lattes está comentando um livro que fez
sucesso
unicamente graças ao poder da mídia. Sequer traduz o significado dos
trabalhos efetuados
pelo próprio Hawking. Não concordo com a maioria do que o Taborda disse, mas
em meio a
seu texto ele assinalou uma grande verdade: esse livro não tem nada além de
finalidades puramente lucrativas.

Victor: Ora, mas Sérgio está brincando. Quer só provocar, prá ver os
"gladiadores" se arranjam na arena.
Só pode ser.
Agora, realmente, o Uma Breve História do Tempo, foi um livro escrito numa
época em que não havia tanta profusão
de obras de divulgação. E, certamente, transformou-se num BestSeller. Todo
mundo comprou. Mas, creio, pouca gente conseguiu ler. E tenho a consciência
de que as pessoas o compraram assim, em tão grande escala, em vista do
inusitado:
um sujeito, já mundialmente famoso como cosmologista, portando uma
deficência física daquela espécie, escrever algo
difícil como é o assunto de que trata, num nível tal, que pretendia atingir
o leigo. Evidentemente, que tal objetivo não foi conseguido, o de todos
lerem, gostarem, compreenderem, essas coisas. Mas a intenção, ainda que em
segundo plano foi essa. Em primeiro plano, pode até sido para angariar
fundos, para dar suportes aos tratamentos e cuidados que precisava ter, em
face se seu mal. Mas, o que importa isso? Qual o problema com isso? O que
importa é que o livro não tinha o objetivo de ser tratado científico,
dirigido a experts, os muito dotados e versados naquelas coisas. Então, um
cara, numa condição "favorabilíisima" como a dele, descer ao nível que
desceu, num linguajar simples e até humorado, mesmo numa tradução
considerada de ruim prá la, ousar uma ventura do tipo, merece, só por isso
algum respeito, mínimo que seja. Lí o livro, como algo de divulgação, com
informações razoáveis acerca do cosmo. Honestamente, até não não ví qualquer
motivação para tanto rangir de dentes, para tanto rancor. Também, conta-se
alí a história pessoal dele, pois a muita gente interessa, talvez não a
alguns experts que acham uma babaquice. Mas o problema aí não mais é de
Hawking. Não vejo como crucificar um cara desses, por causa disso. Também,
não é uma obra para alguém pesquisar e definir a capacidade e/ou
personalidade do autor, para que se possa assacar contra ele, não da maneira
como aquí se tem visto. Não o idolatro, nem a ele ou a outro qualquer. Por
exemplo, tenho pela Teoria da Relatividade o maior respeito, por uma razão:
quanto mais a estudo, seja a restrita, seja a Geral, quanto faço contas,
mais me convenço de que está correta e que há, em sua estrutura matemática,
alguns aspectos ainda não compreendidos. Pelo seu Autor, o Einstein, e por
uma natural extensão, também tenho uma extrema admiração e respeito. Mas,
não a ponto de cegar, de idolatrar. Se surgir outra teoria que a substitua,
que a complemente, qual o problema? Até o próprio foi useiro e vezeiro em
dizer que tudo aquilo poderia até ser provisório, que aquilo tudo não era
uma verdade absoluta, intocável, como você mesmo sabe. Só que, até o
momento, ciscaram muito e fizeram muito trovão e nadinha de chuva. (!!???).
Então...
Além disso, Einstein não tem a estatura científica que tem só pela Teoria da
Relatividade, que foi apenas uma das facetas de seu gênio criativo, apenas
um detalhe, gigantesco é verdade, mas um detalhe. A obra dele é bem mais
extensa. Bem mais abrangente.

Victor: > Paciência, rapaz. Isso já é idolatria, e idolatria
> é o que mais se condena nesta lista. Como deve ser, em qualquer outro
> lugar. Sabia que é pecado mortal ser idólatra, vai pro inferno, aviso.

Mesquita: Idolatria de quem e por quem? Quanto ao inferno, se a praga
dirige-se a mim,
diria que não tenho medo, pois sou amigo do Léo. :-)

Victor: Com as perguntas acima, você já respondeu e eu entendí. Enganei-me.
Desocupe o Léo, pois você não vai pro inferno!

Mesquita: Mas idolatria por quem? Quanto ao Cesar Lattes não fui eu quem o
indicou
para o posto de um dos melhores físicos brasileiros de todos os tempos e
muito menos para dar o nome à Plataforma do CNPq. Já disse que dentre os
três "ídolos" da física brasileira (foi você quem os citou) sou mais
Schenberg e não abro. Mas nem por isso eu fico idolatrando o Schenberg. Será
que você não está "projetando" a sua possível idolatria pelo Hawking ou
pelo Einstein?

Victor: Será? Mas acho que não. Tentei responder a isso mais acima.
Quanto a Schenberg, estou com você e não abro. Embora, infelizmente, não
conheça
a obra científica dele, apenas o que falam dela. Mas lí, por duas vezes, o
seu Pequeno Grande Opúsculo,
"Pensando a Física", Editora Nova Stella, 3a. Edição.


Mesquita: Eu, particularmente, tenho uma "profunda admiração" por
quatro pensadores já falecidos: Newton, Popper, Einstein e Schenberg. Mas
isso não significa que eu deva me subjugar a suas idéias. Se puder
aproveitar algo de bom dentre o que nos legaram, ótimo. Do contrário dou
asas à imaginação e parto para minhas críticas às IDÉIAS deles, mas não a
eles como pessoas e/ou como cientistas. Pensem o que quiserem a respeito de
minhas pretensões, mas já critiquei os quatro "ídolos" que mais admiro. E
apesar disso continuo admirando-os.

Victor: Ora, esse é o próprio Alberto Mesquita. E está certo!

Victor: Mas, sinceramente, Alberto, tenho a certeza de
> você é prá lá de razoável, moço inteligente, culto, filósofo, físico
> amador e idealista, já puder ver, lendo alguns de seus trabalhos. Não
> tenho a menor dúvida de que você ainda tem jeito, tem concerto... É só uma
> questão de tempo!...

Mesquita: Grato por tudo, especialmente pelo "moço" e pela expectativa. :-).
Para os
que não sabem tenho 61 anos bem vividos e espero viver outros tantos, mas
daí a dizer que eu tenha conserto? ;-)

Victor: Mesquita, você sabe que tal colocação é para descontrair, foi um
brincadeira.
Não leve a mal.


Victor: > Aproveito, agora, o
> ensejo para confessar, de público, e é um direito que me assiste, que a
> impressão que você passa, a mim pelo mesmo, que não me considero tão
> prendado intelectual e mentalmente como você e outros notáveis da lista,
> que você, ao invés de "brincar" com os físicos modernos, como disse acima,
> você na verdade debocha e espinafra(segundo Chemello) tais profissionais,
> pertencentes ao sistema ou não, que trabalham e vivem a mecânica quântica
> e a relatividade no dia a dia, ano após ano.

Mesquita: De fato e sob esse aspecto sou reu confesso. Lembro que o termo
espinafrar
fui eu que utilizei pela primeira vez, não o Chemello. Ele me perguntou o
que isso significava e eu disse que seria debochar, ou algo do gênero. Até
brinquei, dizendo que como se tratava de física quântica, o mais correto
seria SPINafrar. E que mal há nisso? Por acaso os profissionais quânticos
são idolatras? Se sim seria bom avisarem, pois não costumo espinafrar
comportamentos religiosos, por mais absurdos que sejam. Se não, eles que se
defendam com idéias, pois em ciência quem não tem competência não se
estabelece. Sempre que "espinafro" a física quântica dou a entender porquê o
faço. Não obstante, sempre que espinafraram as minhas teorias, eu jamais
consegui sequer desconfiar onde foi que acharam que eu estava errado.

Victor: Posturas suas. Respeito e não comento.

Mesquita: Parece-me que você é muito novo na Ciencialist para ficar julgando
o
comportamento de quem está aqui há mais de cinco anos.

Victor: Tempo de permanência em qualquer atividade, não imuniza quem quer
que seja,
não o protege ou o torna inatacável. Você poderia ter 50 anos de
Ciencialist. Isto não lhe confere "autoridade" nem acrescenta atributos que
o tornem diferente de qualquer outro em qualquer nível que
Isto não importaria,
não o tornaria nem mais nem menos
Por outro lado, se os
profissionais quânticos ignoram o que eu penso, então não vejo de que
maneira posso tê-los ofendido. Que defendam suas idéias, oras. Do contrário,
"que enfiem a viola no saco". Será que, na atualidade, o único que entende
de física quântica aqui é o biólogo Takata? Onde estão os quânticos que no
passado me enfrentavam, sem conseguir convencer a ninguém?

> As suas adjetivações a respeito
> das TR e da MC, confesso novamente(menos na primeira e bastante com
> respeito à última), mexem comigo. E sei também que acontece com outros,
> que não se pronunciam ou para evitar melindres ou em razão de acharem
> descabidas as suas conotações. Isso sim, é uma falta de respeito
> generalizada, segundo meu falho achômetro.

Melindre é próprio de quem não tem argumentos. Defendam suas idéias,
derrubem meus argumentos (tenho mais de 500 mensagens aqui na Ciencialist
não respondidas a altura) ou então, como já disse acima, "enfiem a viola no
saco". Afinal, estamos numa lista de discussão. Se eu digo o que quero e
vocês tão somente ficam melindrados, então sinto muito, mas vocês merecem
mais do que tão somente serem espinafrados.

> Espero que não se zangue(devo estar errado, como disse),
> pode ser apenas impressão minha, mas... "Papagaios cultos", só porque os
> que estudam, vivenciam, ensinam ou pesquisam essas disciplinas têm
> opiniões, bastante embasadas-no caso da grande maioria-, divergentes das
> suas?

O termo "papagaio culto" que emprego não tem nada a ver com minha crítica à
física moderna, mas sim ao ensino brasileiro. Relaciona-se ao ensino de
"receita de bolo", em detrimento do despertar da criatividade. Até mesmo o
Feynman, quando esteve no Brasil, chegou a comentar algo do gênero. Qualquer
pedagogo sabe sobre o que estou falando, e recentemente o Léo também chegou
a comentar algo a respeito.

> [...] Essa é uma postura
> kantiana, que considero, no mínimo, de razoável prá cima. Kant também
> dizia que o bem dotado se basta a si próprio, assim ele não se metia.

Perfeito, mas Kant também defendia o despertar da criatividade nos jovens
estudantes. Dava a entender que a finalidade da escola seria promover a
maioridade científica. Pois é exatamente isso o que tenho defendido. Que me
importa o que Newton, Einstein, Hawking ou Lattes afirmaram? Se não
concordar com seus argumentos tenho mais que pichar as suas idéias (as
idéias, bem entendido). Agora, se os defensores dessas idéias se ofendem
com isso, e se mostram incapazes de defendê-las, tenho mais é que "tirar o
sarro" deles, e/ou "espinafrá-los", e é o que tenho feito, doa a quem doer.

> - Sei que você está fazendo um pós-graduação em História da Ciência. Lí
> tal informação por aqui mesmo.

Apenas como aluno especial. Interessei-me por uma disciplina e me dispus a
cursá-la, somente isso.

> A minha curiosidade básica é simples. Já sei que Newton hesitava em
> publicar suas descobertas. Só o fazendo após insistências de amigos
> dedicados. E agora o que está descrito acima sugere mais uma mania sua, de
> se manter no anonimato, de não aparecer, quando outros fatos sugerem que
> não era bem isso. Será que já haviam inventado o "charminho" e Newton
> adorava isso?

Creio que não. Os tempos eram outros. Dizem que devemos os Principia à
insistência do Halley, pois Newton estava se recusando a publicá-lo. No
escritório de Newton encontrou-se centenas de textos não publicados. Alguns
foram publicados após a sua morte, outros são peças de museu. Muita gente
critica a religiosidade de Newton, mas poucos sabem que para permanecer na
sua cátedra ele era obrigado, por contrato, a estudar religião, e até mesmo
a se transformar, após oito anos, numa espécie de sacerdote. Nesta época,
quase no mundo todo, a universidade estava nas mãos dos religiosos, e tinha
que se tomar muito cuidado com o que fosse publicado.

> Quando me sobrar tempo, penso também iniciar um pós em História de
> Ciência, ao menos.

Faz muito bem. "Não tenho a menor dúvida que você ainda tem jeito, tem
conserto." ,-) Quiçá um curso de história de ciência possa ser útil para
descobrirmos em que ponto da história a física atolou de vez. Há muito tempo
li num livro (creio que de um astrônomo ou talvez do Gribbin -- se alguém
leu essa passagem e puder me indicá-la ficaria bastante agradecido) que em
algum lugar da história os físicos cometeram um erro muito grave, e isto
acabou gerando o que está aí. O difícil é localizar o erro, pois não tenho
dúvidas que tanto Einstein quanto Bohr simplesmente tentaram pisar no
acelerador, com o que o veículo acabou atolando de vez. Creio que o erro
está na luz ondulatória e na idéia de fluidos elétricos, e isso é bem
anterior a Maxwell.

[ ]´s
Alberto
http://ecientificocultural.com/indice.htm
Mas indiferentemente a tudo isso, o neutrino tem massa, o elétron não é
uma carga elétrica coulombiana e a Terra se move. E a história se repetirá.



##### ##### #####
Para saber mais visite
http://www.ciencialist.hpg.ig.com.br
Para sair da lista envie um mail para
ciencialist-unsubscribe@yahoogroups.com
##### ##### ##### ##### #####
Links do Yahoo! Grupos










##### ##### #####

Para saber mais visite
http://www.ciencialist.hpg.ig.com.br


##### ##### ##### #####


Yahoo! Grupos, um serviço oferecido por:







------------------------------------------------------------------------------
Links do Yahoo! Grupos

a.. Para visitar o site do seu grupo na web, acesse:
http://br.groups.yahoo.com/group/ciencialist/

b.. Para sair deste grupo, envie um e-mail para:
ciencialist-unsubscribe@yahoogrupos.com.br

c.. O uso que você faz do Yahoo! Grupos está sujeito aos Termos do Serviço do Yahoo!.



[As partes desta mensagem que não continham texto foram removidas]



SUBJECT: Re: [ciencialist] Re: Essas, pois, são as inquietudes tolas de mais um bobo.(2)
FROM: "JVictor" <jvoneto@uol.com.br>
TO: <ciencialist@yahoogrupos.com.br>
DATE: 23/01/2005 08:47

Lista,

Adendo:

Uma observação sobre a citação II, de Lorentz.
Reparem que ele, "a vítima", diz textualmente: ...eu não cheguei a atingir totalmente as equações de transformação da teoria da relatividade de Einstein(ênfase minha).
Ele não diz: "eu não cheguei a atingir totalmente as equações de transformação de Poincaré"(e minhas também, pois Poincaré apenas analisou e extendeu o trabalho de Lorentz-segundo " verdadeiro autor"- dando-lhe roupagem condizente com seu próprio pensamento, complementando-o adequadamente).!!!!!!

A citação I, a respeito das impressões de Poincaré, que no final, refere-se a Einstein como " jovem mestre" é, por si só, esclarecedora e taxativa.

Mas alguns dos detratores alegam que Poincaré assim o fez por ser ele, Poincaré, um gentleman, extremamente educado, não querer polêmicas(ué!, mas ele foi extremamente polêmico... leiam A Ciência e Hipótese e vejam o pau entre B.Russel e Poincaré, e entre este e e o matemático Couturat, sobre a diferença entre seus pensamentos a respeito de coisas da geometria e da lógica matemática; em O Valor da Ciência há outros exemplos de que ele não deixava por menos). Acham que o jovem Einstein, desconhecido, pobre, barrigudo e feio,iria ser poupado, sobretudo se houvesse lhe roubado algo mais importante e mais profundo do que simples conceitos geométrico, pelo que puxou as orelhas de um cara da dimensão intelectual de B.Russel?

Mas coisas do tipo acima ninguém cita. Ninguém, que digo, os "outros, invejosos,despeitados e não muito bem intencionados".

Só isso.

Sds,

Victor.

----- Original Message -----
From: JVictor
To: ciencialist@yahoogrupos.com.br
Sent: Sunday, January 23, 2005 12:04 AM
Subject: Re: [ciencialist] Re: Essas, pois, são as inquietudes tolas de mais um bobo.(2)


Rayfisica/Sérgio,

Pegando uma carona no lúcido, correto e pertinente raciocínio do Taborda, que enfoca muito bem as nuances envolvidas nas teorias de Lorentz e Poincaré de um lado, e a visão de Einstein, de outro, gostaria de tecer alguns comentários a respeito, se me permitem. Quando digo que Einstein viu um palmo diante do nariz a mais que todos os outros de sua época, quero dizer exatamente isso que Taborda destacou, a respeito do significado mais profundo de coisas relativas ao espaço e o tempo, com todas suas peculiaridades, postas a descoberto, pelo menos a nível de uma teoria que descreva consistentemente eventos até então obscuros.
Bom, mas quero comentar brevemente as falácias de alguns grandes homens de ciência, quando dizem que Einstein plagiou Poincaré e Lorentz, pois alegam, como prova, que não há qualquer menção, nos trabalhos de Einstein, ao trabalho de Poincaré e Lorentz nessa área, que não são citados por Einstein em seus paper´s, embora suas teorias(Poincaré e Lorentz), do ponto de vista experimental, conduzam aos mesmíssimos resultados experimentais que a teoria vista segundo este último.
Então, não mencionar os trabalhos anteriores está sendo considerado como uma prova de plágio, pelo menos na visão de alguns, como o Prof. Lattes e outros, como os que fazem o Autodinamics, sociedade criada em torno das teorias de um certo argentino, Dr.Ricardo Carezani, que contesta a TR e propõe outra.
Como diz o taborda, uma teoria, por mais original que seja, não é, em geral, obra de uma mente só. Alguém já questionou, levantou a questão, depois deixou prá lá, outro retomou o fio da meada, e assim por diante. É como se houvesse um inconsciente coletivo que, de certa forma, mantivesse acesa a chama da dúvida e os indícios da necessidade de mudança. Até que, numa dada época, alguém enxerga com clareza todas as incidências das questões e tudo fica claro como um dia de sol.
Assim, a teoria da relatividade não foi, de jeito nenhum, uma excessão. Até onde sei, tudo começou em 1887, com Woldemar Voight(1850-1919), da Universidade de Gottingen, visto hoje em dia como o não celebrado heroi da relatividade especial! A bem da verdade histórica, a concepção de uma velocidade universal de luz, c, e de um tempo relativo, foi concebido e proposto naquela ano, pelo Prof. Voight, que era um físico de primeira linha. Voight, em função disto, postulou a invariância da lei de propagação de ondas luminosas para derivar as transformações quadri-dimensionais do espaço e do tempo, com vistas a entender o deslocamento Doppler dos comprimentos de onda e das frequências associadas. Claramente, isto foi um extraordinário, fenomenal, insight para os tempos de então. Um espanto.
As transformações desenvolvidas por Voight diferem das de Lorentz unicamente por um fator constante(Conforme discutido no paper de Andreas Ernest e Jong-Ping Hsu: " First Proposal of the universal speed of light por Voight in 1887". Neste trabalho, ao qual tive acesso, e lí, os autores descrevem e comentam toda a teoria, sustentada por um belo, simples e preciso, aparato matemático. Vale a pena ler o paper. Poderá ser encontrado em "Lorentz and Poincaré Invariance-100 years of relativity", de Jong-Ping Hsu e Yuan-Zhong Zhang). Infelizmente, como diz Ping, " o trabalho pioneiro de Voight foi condenado a sofrer destino similar ao que recebeu o trabalho científico de Leonardo da Vinci." Acrescenta, ainda: "Tanto um como o outro nasceram cedo demais e então não representaram maiores influências no desenvolvimento científico de seus dias". É como dizer que suas mentes estavam muito à frente das mentes e das concepções de então.
Ora, quanto a isto não há dúvidas. O assunto já zanzava pelos meios científicos, havia publicações como o paper acima, etc.

Contudo, e tão somente contudo, nem Poincaré, nem Einstein e muitos outros, fizeram qualquer referência ao paper de Voight, que tratava dos mesmos assuntos, ainda que em fase seminal, que já haviam sido pensados e publicados! Lorentz, em 1912, Reanalisando o seu trabalho de 1904, " Fenômenos eletromagnéticos num sitema que se move com qualquer velocidade inferior à da luz", ainda fez um ligeira referência ao trabalho de Voight, conforme transcrevo no fim deste texto, citação II. Mas só o fez em 1912, 7 aninhos após o ano miraculoso de 1905, como dizem os mais apaixonados.

E agora a questão: O fato de Einstein não mencionar Poincaré, faz dele um plagiador sem vergonha, como dizem alguns? E Poincaré não plagiou Voight? Pois Poincaré não citou, em momento algum, uma vírgula daquele trabalho prioneiro! Lorentz ainda chegou a trocar cartas com Voight, e discutiram seus trabalhos por essa via. Mas, mesmo assim, nos trabalhos de Lorentz, em seus primeiros paper´s, antes de 1904(pelo menos não achei, mas procurei e estou procurando), não há referências ao bem dito paper de Voight! Lorentz também é um plagiador, por isso? Se Einstein é, os dois outros também o são...Ora! Nessa linha de argumentos, quem seria o plagiador a respeito da invenção do cálculo: Newton ou Leibnitz?
Ao que tudo indica, parece que não era mesmo costume da época ficar mencionando trabalhos de outros, quando os seus eram de lavra própria(até mesmo modernamente, cientistas como Landau não fazem referências a outros autores. Ele só cita alguém, quando reconhece que o que o alguém diz não é considerado trivial. Ele não apresenta uma lista enorme de livros que teria lido ou pesquisado. Há autores que exibem uma lista tão grande de livros para justificar seu trabalho que dá para ficar com dúvidas a respeito de se ele, numa vida, teria tempo de ler tudo!, ou se aquilo tudo é para encher linguiça).
Os trabalhos ora comentados foram feitos de maneira independente. Não tenho dúvidas. Nem um pouquinho que seja. E mais: para mim é irrelevante se Einstein conhecia ou não o trabalho de Poincaré(que, a bem da verdade, é gigantesco). Einstein conferiu à TRR uma significação física absolutamente diferente e original, embora os postulados sejam os mesmos. Só que Einstein extraiu destes o que o outro nem sonhou! Pessoas com o nível intelectual de um Voight, Lorentz, Poincaré e Einstein, entre outros, como Moller, Tolman, Landau, não precisam plagiar quem ou que quer que seja! Ou seja, basta aprofundar um pouco, ir atrás da verdade dos fatos, que as coisas vão se esclarecendo e pondo a descoberto esses pretensos críticos que, na verdade, são ignorantes e irresponsáveis, pois tentam formar opiniões de maneira igualmente irresponsável. Apesar de alardearem seus cursos de pós, pós do pós, que acaba resultando no pó amarelado e tóxico que dissseminam. Só para completar, E=mc2 foi estabelecida pela primeira vez por Voight, antes de Poincaré. Poincaré chegou a postular a mesma coisa, mas sem qualquer prova. Mas o significado, que foi uma decorrência dos postulados de Voight, é aquele corretamente discutido por Sérgio Taborda. Nem mais nem menos.
A propósito, o artigo que deu causa ao assunto deste e-mail, foi escrito por um advogado, sem formação alguma em física ou em história da ciência. Apenas à base do ouví dizer, que lí assim, que um famoso físico brasileiro disse, que um físico escritor ficou em dúvida(ficou em dúvida em razão de não conhecer a verdade dos fatos, apesar de ser aclamado no Brasil e terra de Bush como divulgador da ciência). Não que o autor não tenha capacidade de fazer um trabalho mais próximo da realidade. Claro que tem. Ele sabe ler, e só precisa disso e mais uma consultoria técnica junto a mais de um bom físico ou matemático. Só isso! Mas, não fez assim. Portanto, o que ele fez "sequer é um artigo", como diria W.Pauli.
Existem outras teorias alternativas da Relatividade,todas conduzindo basicamente aos mesmo resultados. Mas nenhuma é tão simples e objetiva quanto a TRR, de Einstein. O livro acima citado analisa algumas, até interessantes. Quando houver oportunidade, comentarei.
Lembrem-se de que nem sempre o que é publicado na internet é verdadeiro. Há muita coisa boa. Mas há também muita porcaria. A propósito, não é demais transcrever(já o fiz antes, neste Ciencialist) duas coisinhas ditas pelos próprios interessados, Poincaré e Lorentz, que teriam sido "roubados" em suas teorias por Einstein:

I - Poincaré, sobre Einstein, em Arquitetos de Idéias-A história das Grandes Teorias da Humanidade, de Ernest R. Trattner:

"Einstein, escreveu Poincaré, é um dos espíritos mais originais que tenho conhecido. A despeito de seus verdes anos, já ocupa uma posição
distinta entre os grandes sábios desta época. O que nos causa maravilha neste moço é, sobretudo, a facilidade com que ele se adapta às novas concepções,
extraindo dela todas as conclusões possíveis. Não se apega a princípios clássicos, mas, ao encarar um problema de física, toma em consideração
todas as conjecturas possíveis. Em seu espírito isto se transforma numa antecipação de novos fenômenos, que poderão ser verificados um dia pela
experiência real. O futuro dará provas cada vez mais abundantes dos méritos de Alberto Einstein, e a universidade que lograr atraí-lo aos seu seio há de
tornar-se famosa por sua ligação com o jovem mestre".


II - Lorentz, em 1912, reanalisando o seu trabalho de 1904," Fenômenos eletromagnéticos num sistema que se move com qualquer velocidade inferior à da luz".

Lorentz, após tentar obter certas equações de transformação, que deixariam invariantes certas quantidades eletromagnéticas(usando, naturalmente, a teoria do éter), escreveu a seguinte nota(página 19 do Volume do O princípio da Relatividade(H.A.Lorentz, A.Einstein e H.Minkowski, 4a. Edição, Fundação Calouste Gulbenkian):

"Deve notar-se que, neste trabalho, eu não cheguei a atingir totalmente as equações de transofrmação da teoria da relatividade de Einstein. Nem a equação (7) nem as fórmulas (8) têm a forma dada por Einstein e, em consequência disso, não cheguei a fazer desaparecer o termo (-wu^linha_x/c2) na primeira equação (9). pelo que não conseguí dar às fórmulas (9) uma forma rigorosamente válida para um sistema em repouso. Dependem desta circunstância os embaraços em que esbarram muitas das considerações ulteriores deste trabalho.
Pertence a Einstein o mérito de ter sido o primeiro a enunciar o princípio da relatividade como uma lei geral, rígida e exacta.

Acrescento a isto a observação de que Voight já no ano de 1887(Gottinger Nachrichten, pág. 41), num trabalho <<Uber das Dopplersche Pirnzip>>(Descrito no livro acima comentado, do Jong-Ping) aplicou as equaçôes da forma " del phi - (1/c2) parcial^2 phi/parcial t^2 = 0 "
uma transformação que é equivalente à contida nas equações (4) e(5) do meu trabalho".

Estas foram coisas proferidas e escritas por Poincaré e Lorentz, sobre o cara acusado de plagiá-los!. Tirem suas conclusões.


Sds,

Victor.


----- Original Message -----
From: Sergio M. M. Taborda
To: ciencialist@yahoogrupos.com.br
Sent: Saturday, January 22, 2005 2:57 PM
Subject: [ciencialist] Re: Essas, pois, são as inquietudes tolas de mais um bobo.(2)



--- Em ciencialist@yahoogrupos.com.br, "rayfisica" <rayfisica@y...>
escreveu
>
> Com certeza o assunto já foi tratado aqui, como sou novo na casa...
>
> Tenho ouvido tanta besteira com o nome de ciência e em nome da
> ciência (apesar de minhas limitações) que desconfio de tudo,
> por
> favor, alguém pode explicar isso para mim de um jeito bem simples.

Uma teoria cientifica consta dos seguintes passos

1:ideia , insight: alguem em algum ponto tem uma ideia para explicar
alguma coisa. Pode acontecer que pessoas diferentes em lugares
tenham a mesma ideia.
2:desenvolvimento: que pode ser teorico , experimental ou uma
combinação de ambos. Aqui a ideia é esenvolvida para chegar numa
explicação aplicavel a vários casos. A ideia desenvolve-se tb numa
forma universal de aplicação. Essa forma pasa pela criação de
postulados , regras, leis, convensões, etc.. que forma uma estrutura
sempre aplicável. Um particular isso tudo pode traduzir-e em
matemática, mas não é obrigatorio.
3: teste. Ema vez desenvolvida a teoria é apresentada ao publico em
geral e submete-se a teste. O teste so é válido se for feito por
outras pessoas que não as envolvidas nos passos anteriores. Escolhe-
se um caso simples em que a teoria possa ser aplicada ,e aplica-se.
Fazem-se as medições necessárias e tira-se conclusões.
4: aceitação. Este passo não é imediato. Acontece quando o passo
anteiror é repetido diversas vezes e nunca chegamos numa contradição
da teoria.

No desenvolvimento de uma teoria nunca existe apenas uma pessoas,
por isso é uma imbecilidade querer atribuir as teorias a uma so
pessoa. Essa imbecilidade é prepetudada por quem não entende nada do
assunto e precisa de um icone, alguem para por no pedestal, alguem a
quem cobrar responsabilidade, esse tipo de coisas pateticas que a
nossa sociedade faz, mas que no caso da ciencia são futeis.

Desde a epoca em que Maxwell desenvolveu as eq so e.m. que sabemos
que E=mc^2 , mas essa formula é diferente do E=mc^2 de Eisntein. É
isso que as pessoas ingenuas e desinformadas não entendem e não
querem entender. tlv elas não saibam que precisam entender e por
isso que são ingenuas , e não têm meios para entender mesmo que
queiram, por isso que são desinformadas.
Se qq bobo pudesse ser fisico não era preciso estudar muito e fazer
faculdade para o ser. É peciso ter talento, claro, mas é preciso ter
treino tb.

Então vejamos. Do ponto de vista do Electromagnetismo, que é ponto
de vista de Lorentz , Poincaré e outros. E=mc2 significa que :
A energia do campo e.m. é proprocional a uma massa. A energia do
campo e.m é a energia contida nos campos electrico e magnetico.
Pelas mesmas ideias podemos deduzir que o momento linear do campo
e.m seria p = mc , sendo m = E/c2 , ou seja, que a energia do campo
é proporcional ao seu momento (ou seja, que a energia é puramente
cinetica) E = pc.

Estas formulas são válidas para o campo electromagnetico , MAS não
são válidas para qq corpo em geral. É aqui que entra eisntein. Na
teoria de eisntein , dados certos postulados preliminares , conclui-
se que E=mc2 para qq corpo MAS o que significam agoras sa letras?
Esta formula significa que a Energia (total, não paenas a cinetica)
medida por um observador A de um corpo B é propocional à massa que A
mede de B, que por sua vez não é igual à massa que B mede se si
mesmo.
A complexidade de raciocinio para chegar a E=mc2 via eisntein é
muito maior que via poincaré. E as formulas não são equivalentes.
Embora tenham a mesma grafia , se usem as mesmas letras e tudo o
mais, elas não significam a mesma coisa !!!!!

E qq outra discussão sobre se é poincaré ou Eisntein a inventar
E=mc2 é completamente inepta , idota , edionda e intragável.

Eisntein consegiu traçar todo o perfil de uma teoria que explica pq
a formula é aquela enquanto os outros apenas usam de matemática e
ideias ah doc.

Sérgio Taborda





##### ##### #####

Para saber mais visite
http://www.ciencialist.hpg.ig.com.br


##### ##### ##### #####


Yahoo! Grupos, um serviço oferecido por:







------------------------------------------------------------------------------
Links do Yahoo! Grupos

a.. Para visitar o site do seu grupo na web, acesse:
http://br.groups.yahoo.com/group/ciencialist/

b.. Para sair deste grupo, envie um e-mail para:
ciencialist-unsubscribe@yahoogrupos.com.br

c.. O uso que você faz do Yahoo! Grupos está sujeito aos Termos do Serviço do Yahoo!.



[As partes desta mensagem que não continham texto foram removidas]



##### ##### #####

Para saber mais visite
http://www.ciencialist.hpg.ig.com.br


##### ##### ##### #####


Yahoo! Grupos, um serviço oferecido por:
PUBLICIDADE




------------------------------------------------------------------------------
Links do Yahoo! Grupos

a.. Para visitar o site do seu grupo na web, acesse:
http://br.groups.yahoo.com/group/ciencialist/

b.. Para sair deste grupo, envie um e-mail para:
ciencialist-unsubscribe@yahoogrupos.com.br

c.. O uso que você faz do Yahoo! Grupos está sujeito aos Termos do Serviço do Yahoo!.



[As partes desta mensagem que não continham texto foram removidas]



SUBJECT: Selo Comemorativo - Imperdivel !
FROM: "Luiz Ferraz Netto" <leobarretos@uol.com.br>
TO: <ciencialist@yahoogrupos.com.br>
CC: <fisica-mg@grupos.com.br>, "Marcelo Moura" <mmmoura.bh@terra.com.br>, <eletronicabr@yahoogrupos.com.br>, <fisica@grupos.com.br>, <galena_brasil@yahoogrupos.com.br>, <Conversa_de_Botequim@yahoogrupos.com.br>, <sbcr@yahoogrupos.com.br>, <strbrasil@yahoogrupos.com.br>
DATE: 23/01/2005 08:57

Selo ou não selo, eis a questão.

Bem pessoal, Feira de Ciências, O Imperdível! já se aproxima, aceleradamente, da casa dos 10 milhões de consultas/vistas/pageview (10 000 000) ... e o contador de acessos só tem 7 dígitos.
A partir de então o contador registrará 0 000 000 e começara tudo de novo ... e, um consulente novato (entrando pela primeira vez no Imperdível) vai observar "coitado! tanto trabalho e nem uma boa quantidade de visitas ele tem!" --- e meu ego põe uma capa, abre o guarda-chuva e se despede de mim, por não saber nadar em minhas lágrimas (putz, ficou bonito pacas!).

Bem, foi sugestão do amigo Marcelo Moura, o astrônomo que administra a Sala 24 do Feira de Ciências, que se faça um selo comemorativo dos 10 MILHÕES pois, com ego ou não, atingir essa meta num site de Educação e Divulgação Científica, no Brasil, é algo pelo menos digno de ... um SELO!

Assim sendo, e para presentear a posteridade, estou abrindo só para aos amigos o Concurso do Selo Comemorativo do Imperdível. Um "gif" (animado ou não) para ser colocado ao lado, ou abaixo do contador de acessos, na coluna da esquerda, nas páginas INDEX das 25 Salas do site. Como prêmio posso oferecer (a escolher): um livro de Física (pode ter certeza que será algo diferenciado --- alguém tem, por exemplo, o livro para 'ensino médio' escrito por Eduard Branly --- sim, o inventor do coesor Branly ---.), um gerador eletromagnético didático ou um gerador eletrostático tipo Van de Graaff (esse só poderá ser entregue a partir de julho, pois estou fazendo esse projeto para fornecer kits desse gerador via WEB). Se interessar posso oferecer também uma máquina Kirlian, [que já tem incorporada um fantasma (fantasma residente)] --- acompanha 10 ml de ectoplasma --- e fornece fotos de auras em 4 dimensões :-).

Alguém sugeriu colocar esse Concurso no próprio site do FEIRA; será que 'cai bem'?

Agradeço aos amigos que, direta ou indiretamente, participaram desse sucesso do site. Eu, particularmente, não esperava tal acolhida; o primeiro milhão foi realmente comemorado com festinha regada a vinho aqui em casa --- tal foi meu 'espanto'.

Aquele abraço,

===========================
Luiz Ferraz Netto [Léo]
leobarretos@uol.com.br
http://www.feiradeciencias.com.br
===========================
----------

Internal Virus Database is out-of-date.
Checked by AVG Anti-Virus.
Version: 7.0.300 / Virus Database: 265.6.13 - Release Date: 16/01/2005


[As partes desta mensagem que não continham texto foram removidas]



SUBJECT: Re: [ciencialist] Re: Essas, pois, são as inquietudes tolas de mais um bobo.(2)
FROM: "JVictor" <jvoneto@uol.com.br>
TO: <ciencialist@yahoogrupos.com.br>
DATE: 23/01/2005 11:53

----- Original Message -----
From: Manuel Bulcão
To: ciencialist@yahoogrupos.com.br
Sent: Saturday, January 22, 2005 7:46 PM
Subject: [ciencialist] Re: Essas, pois, são as inquietudes tolas de mais um bobo.(2)

Grande Manoel,



--- Em ciencialist@yahoogrupos.com.br, "rayfisica" <rayfisica@y...>
escreveu

> por favor, alguém pode explicar isso para mim de um jeito bem
simples.

> TABUS DA CIÊNCIA
> Seria Einstein uma fraude?

Manuel: Vou dar meu pitaco sobre esse assunto tão controvertido:

Victor: Se você reparar direitinho verá que o "assunto controvertido" assim o é em razão de "forçadas de barra"... Foi uima "dissidência mal orquestrada", que só Freud explica (talvez nem ele..), que engendrou e alimentou essas tais controvérsias. Uma análise nem tão profunda e um pouco de profissionalismo honesto num instante desvenda o "mistério" e põe a limpo as verdades essenciais a respeito. Na realidade, não há controvérsias, pois há farta documentação que restabelece a verdade dos fatos, sem quaisquer dúvidas. É a mesma coisa a respeito de certos paradoxos, criados em razão de não se entender o assunto que lhe deu causa. Ou seja, sem querer ser grosseiro e deselengante: pura ignorância e incapacidade mental e intelectual de ver um pouco mais adiante!... É o caso, entre outros, do falacioso paradoxo dos gêmeos. Não é paradoxo coisa nenhuma.
Você até que pitacou direitinho, dentro da coerência e honestidade intelectual que lhe são peculiares.

É isso aí.

Sds,

Victor.




[As partes desta mensagem que não continham texto foram removidas]



SUBJECT: Re: [ciencialist] Mulheres: Cozinha e na cama, já!!!!!!!!!!
FROM: "JVictor" <jvoneto@uol.com.br>
TO: <ciencialist@yahoogrupos.com.br>
DATE: 23/01/2005 12:11

Natália,


É evidente que esse Summers tem algum problema de identidade sexual. Há um parafuso frouxo nessa área da mente. Ele precisa valorizar o seu lado sexual, e que deus me livre de arriscar qual seja. Como os assim assim da vida, que valorizam os grandes personagens, suspeitos de também serem meninos alegres. Pois isso pode servir de âncora para a manutenção e aceitação da classe, mais esta do que aquela. Há até um movimento, Orgulho Gay(embora até hoje eu não tenha conseguido entender de quê é esse tal orgulho. Também não consigo entender o Apocalipse, no sentido de que qual mente louca pode escrever tanta coisa ridícula) para marcar suas presenças neste mundinho. Não tenho nada contra o que quer os outros façam e usem, claro. Cada um é cada um. Respeito is good e todos gostam. Agora, aproveitar-se de uma pretensa autoridade científica e de um cargo para pronunciar tamanha besteira é algo que transcende as raias do suportável e do racional. É claro que é mais uma falácia. As mulheres têm capacidade idêntica sim, apenas os preconceitos e as oportunidades, são muitos os primeiros e pouquinhos as segundas.
Por outro lado, cozinha nem tanto. Mas cama, isso lá, sim, deveria ser muito tanto!...
Estou com vocês e não abro, nem que a tampa voe.

Sds,

Victor.

----- Original Message -----
From: Maria Natália
To: ciencialist@yahoogrupos.com.br
Sent: Saturday, January 22, 2005 7:13 PM
Subject: [ciencialist] Mulheres: Cozinha e na cama, já!!!!!!!!!!



Esta mensagem tem muito de Ciência embora se destine a agitar as
massas machistas da lista.
http://www.president.harvard.edu/speeches/2005/womensci.html
No comments.
Um abração
da Maria Natália





##### ##### #####

Para saber mais visite
http://www.ciencialist.hpg.ig.com.br


##### ##### ##### #####


Yahoo! Grupos, um serviço oferecido por:
PUBLICIDADE




------------------------------------------------------------------------------
Links do Yahoo! Grupos

a.. Para visitar o site do seu grupo na web, acesse:
http://br.groups.yahoo.com/group/ciencialist/

b.. Para sair deste grupo, envie um e-mail para:
ciencialist-unsubscribe@yahoogrupos.com.br

c.. O uso que você faz do Yahoo! Grupos está sujeito aos Termos do Serviço do Yahoo!.



[As partes desta mensagem que não continham texto foram removidas]



SUBJECT: Re: [ciencialist] "ófitópic": dica firewall + Fwd: [Livros_Digitais] REVISTA HACKER GRATUITA
FROM: "JVictor" <jvoneto@uol.com.br>
TO: <ciencialist@yahoogrupos.com.br>
DATE: 23/01/2005 14:39

Uso o Norton,mas por umas duas vezes vírus chegou e ficou!. Não sei se é a versão que tenho, ou se o tal não está bem configurado. Só sei que o cara é covarde. Fica de cócoras debaixo da mesa. Quando o vírus aparece, ele sai correndo e deixa a tampa voar!...

Sds,

Victor.
----- Original Message -----
From: Eurico Ferreira de Souza Jr.
To: acropolis@yahoogrupos.com.br
Cc: ciencialist@yahoogrupos.com.br
Sent: Friday, January 14, 2005 11:29 AM
Subject: [ciencialist] "ófitópic": dica firewall + Fwd: [Livros_Digitais] REVISTA HACKER GRATUITA


[E]> Salve,

instalei o sygate personal firewall.
agora eu decido qual programa acessa a internet.
descobri que o MSN messenger (que eu nunca usei) fica tentando fazer sei lá o que na net, e que o ad-aware fica tentando acessar o gator !
o norton fica avisando que o firewall tá baixando pacote suspeito e o ad-aware alerta que o spy-bot tá suspeito também...
eu me pergunto: há limites? :D



papagrupo <papagrupo@yahoo.com.br> wrote:
Para: Livros_Digitais@yahoogrupos.com.br
De: "papagrupo"

Data: Fri, 14 Jan 2005 05:21:32 -0000
Assunto: [Livros_Digitais] REVISTA HACKER GRATUITA



Quem se interessa por temas de segurança na internet pode baixar
gratuitamente o primeiro número da revista HACKER.BR. É só clicar na
capinha, não precisa fazer cadastro. Gostei de duas matérias, uma
que ensina a acessar o e-Mail por telnet e outra com a história dos
hackers de 1960 até hoje.

A revista está no número zero e pode ser baixada gratuitamente:

www.cursodehacker.com.br



_\|/_

---------------------------------
Yahoo! Acesso Grátis - Internet rápida e grátis. Instale o discador do Yahoo! agora.

[As partes desta mensagem que não continham texto foram removidas]



##### ##### #####

Para saber mais visite
http://www.ciencialist.hpg.ig.com.br


##### ##### ##### #####


Yahoo! Grupos, um serviço oferecido por:







------------------------------------------------------------------------------
Links do Yahoo! Grupos

a.. Para visitar o site do seu grupo na web, acesse:
http://br.groups.yahoo.com/group/ciencialist/

b.. Para sair deste grupo, envie um e-mail para:
ciencialist-unsubscribe@yahoogrupos.com.br

c.. O uso que você faz do Yahoo! Grupos está sujeito aos Termos do Serviço do Yahoo!.



[As partes desta mensagem que não continham texto foram removidas]



SUBJECT: O Feira e o Professor Luis Brudna
FROM: "brudna" <lrb@iq.ufrgs.br>
TO: ciencialist@yahoogrupos.com.br
DATE: 23/01/2005 16:22


Estou dando umas aulas de ´Recursos de informatica para quimicos´
na Unijui (univ particular de Ijui, cidade onde nasci). Divido o
trabalho com um professor da informatica, ele dah a parte de
Windows, Word e Excel e eu fico com a parte de busca de informacoes
cientificas na internet e softs de quimica.

A turma tem bem poucos conhecimentos em informatica. Alguns nunca
usaram um computador. :(

O interessante dessas aulas eh que percebi como eh dificil achar
informacoes cientificas em portugues.

Deixei os alunos pesquisando livremente na internet, e o curioso
eh que muitos estavam visitando o site ´Feira de ciencias´. :) Enchi
o peito de orgulho e disse "Eu conheco o cara que faz esse site, ele
eh MUITO legal!". Uma aluna disse "Nao tem um site assim para a
quimica?!". Fiquei sem resposta. Nao conheco nenhum. Indiquei o site
do Emiliano, mas nao eh exatamente o estilo desejado.
Ela acabou copiando algumas coisas que estavam no ´Feira´, como
aquela expericia da Cromatografia em Papel.

Alias, o site do Emiliano foi muito util para eu propor o trabalho
que vale nota. A ideia foi tentar encontrar o maximo de informacoes
sobre um assunto e escrever um trabalho no Word. Escolhi muitos
assuntos que estavam no site do Emiliano. :)

Claro que nao perdi a oportunidade de falar do Humor na ciencia.
Heheh. Nao cheguei a indicar a Ciencialist pq eles nao sabem usar
direito o mail e uma lista com muitas mensagens ia acabar dando
confusao.

Em fevereiro comeco a dar aulas de Fisico Quimica II e FQ
Experimental. Vou ter que montar experimentos, junto com outros
professores, sobre FQ. Quem sabe nao sai um mini-site com alguns
experimentos. :)

Ateh
Luis Brudna
ps: pena que ainda nao tenho contrato fixo. :(



--- Em ciencialist@yahoogrupos.com.br, "Luiz Ferraz Netto"
<leobarretos@u...> escreveu
> Selo ou não selo, eis a questão.
>
> Bem pessoal, Feira de Ciências, O Imperdível! já se aproxima,
aceleradamente, da casa dos 10 milhões de consultas/vistas/pageview
(10 000 000) ... e o contador de acessos só tem 7 dígitos.
> A partir de então o contador registrará 0 000 000 e começara tudo
de novo ... e, um consulente novato (entrando pela primeira vez no
Imperdível) vai observar "coitado! tanto trabalho e nem uma boa
quantidade de visitas ele tem!" --- e meu ego põe uma capa, abre o
guarda-chuva e se despede de mim, por não saber nadar em minhas
lágrimas (putz, ficou bonito pacas!).
>
> Bem, foi sugestão do amigo Marcelo Moura, o astrônomo que
administra a Sala 24 do Feira de Ciências, que se faça um selo
comemorativo dos 10 MILHÕES pois, com ego ou não, atingir essa meta
num site de Educação e Divulgação Científica, no Brasil, é algo pelo
menos digno de ... um SELO!
>
> Assim sendo, e para presentear a posteridade, estou abrindo só
para aos amigos o Concurso do Selo Comemorativo do Imperdível.
Um "gif" (animado ou não) para ser colocado ao lado, ou abaixo do
contador de acessos, na coluna da esquerda, nas páginas INDEX das 25
Salas do site. Como prêmio posso oferecer (a escolher): um livro de
Física (pode ter certeza que será algo diferenciado --- alguém tem,
por exemplo, o livro para 'ensino médio' escrito por Eduard Branly --
- sim, o inventor do coesor Branly ---.), um gerador eletromagnético
didático ou um gerador eletrostático tipo Van de Graaff (esse só
poderá ser entregue a partir de julho, pois estou fazendo esse
projeto para fornecer kits desse gerador via WEB). Se interessar
posso oferecer também uma máquina Kirlian, [que já tem incorporada
um fantasma (fantasma residente)] --- acompanha 10 ml de ectoplasma -
-- e fornece fotos de auras em 4 dimensões :-).
>
> Alguém sugeriu colocar esse Concurso no próprio site do FEIRA;
será que 'cai bem'?
>
> Agradeço aos amigos que, direta ou indiretamente, participaram
desse sucesso do site. Eu, particularmente, não esperava tal
acolhida; o primeiro milhão foi realmente comemorado com festinha
regada a vinho aqui em casa --- tal foi meu 'espanto'.
>
> Aquele abraço,
>
> ===========================
> Luiz Ferraz Netto [Léo]
> leobarretos@u...
> http://www.feiradeciencias.com.br
> ===========================






SUBJECT: They burn heretics, don't they?
FROM: "marcelomjr" <marcelomjr@yahoo.com.br>
TO: ciencialist@yahoogrupos.com.br
DATE: 23/01/2005 16:40


They burn heretics, don't they?

Extract from New Scientist, 6 April 1996

http://www.newtonphysics.on.ca/faq/heretics-14.html





SUBJECT: Flat Universe - and - Common sense
FROM: "marcelomjr" <marcelomjr@yahoo.com.br>
TO: ciencialist@yahoogrupos.com.br
DATE: 23/01/2005 17:16


Series #4Flat Universe - and - Common sense
-----------------------
Question - (4-A)
Can you explain in words, what is the nature of the changes that have
been introduced by Einstein, as a consequence of the theory of
General Relativity?
A - Einstein introduced the Theory of Relativity because he
(...)

http://www.newtonphysics.on.ca/faq/flat-universe.html





SUBJECT: E = mc2
FROM: "marcelomjr" <marcelomjr@yahoo.com.br>
TO: ciencialist@yahoogrupos.com.br
DATE: 23/01/2005 17:48


Albert Einstein: The Incorrigible Plagiarist

http://home.comcast.net/~xtxinc/AEIPBook.htm

---------------------------------------------

E = mc2

Excerpts from Chapter Five

Contrary to popular myth, Einstein did not usher in the atomic age,
in fact, he found the idea of atomic energy to be silly, 217 nor was
Einstein the first to state the mass-energy equivalence, or E =
mc2.218 Myths such as Einstein's supposed discoveries are not
uncommon. Newton did not discover gravity, nor did he offer a viable
explanation for it, nor did he believe that matter attracted other
matter. . . . It appears that the physics community and the media
invented a comic book figure, "Einstein", with "E = mc2" stenciled
across his chest. . .

In anticipation of Thomson, De Pretto and the Einsteins, S. Tolver
Preston formulated atomic energy, the atomic bomb and
superconductivity back in the 1870's, based on the formula E = mc2,
where celeritas, "c", signifies the speed of light. Pursuing Le
Sage's theory, Preston believed that mass could be attenuated into
aether, thereby releasing a tremendous store of energy; since aether
particles move at light speed--a limiting velocity, the energy store
is equal to mass times the square of the speed of light. Albert
Einstein never even came close to such insights. . . .

Maxwell's equations implicitly contain the formula E = mc2. Simon
Newcomb pioneered the concept of relativistic energy in 1889.224
Preston, J.J. Thompson,225 Poincare,226 Olinto De Pretto,227 Fritz
Hasenohrl,228 [etc. etc. etc.] each effectively (Albert Einstein,
himself, did not expressly state it in 1905), or directly, presented
the formula E = mc2, before 1905, and Max Planck229 refined the
concept in 1906 - 1908, including Newton's230, Bessel's231 and
Eotvos'232 implications that inertial mass and gravitational mass are
equivalent -- before Albert Einstein.

Alexander Bain expressly stated in 1870 that,

"matter, force, and inertia, are three names for substantially the
same fact"

and,

"force and matter are not two things, but one thing"

and,

"force, inertia, momentum, matter, are all but one fact".239






SUBJECT: Re: [ciencialist] O Feira e o Professor Luis Brudna
FROM: "JVictor" <jvoneto@uol.com.br>
TO: <ciencialist@yahoogrupos.com.br>
DATE: 23/01/2005 17:50

Olá, Prof.JC,

Até o momento, a que conclusões chegou, a respeito das "más relações" entre a física e os estudantes? Fiquei curioso, aliás, interessado em saber.

Sds,

Victor.


----- Original Message -----
From: Prof. JC
To: ciencialist@yahoogrupos.com.br
Sent: Sunday, January 23, 2005 5:59 PM
Subject: Re: [ciencialist] O Feira e o Professor Luis Brudna


Brudna,

Faiz favoire, me conta melhor essa história (pode ser em PVT mesmo). Também
estou trabalhando com capacitação de professores para o uso de computadores
e Internet.

Aliás, Léo, parabéns de novo (já lhe dei os parabéns lá no boteco, mas não
sei se você anda bebendo ultimamente, he he). 10^7 é um número cabalístico!

Ah, e se servir de (des)consolo, mais ou menos seis meses atrás eu criei uma
comunidade no Orkut com o objetivo de "entender melhor a relação dos
adolescentes com a física", ou, mais especificamente, as "más relações". A
comunidade chama-se "Eu odeio Física" e, pasmem, já está na casa dos 5.000
associados. :)))

Abraços,
Prof. JC


----- Original Message -----
From: "brudna" <lrb@iq.ufrgs.br>
To: <ciencialist@yahoogrupos.com.br>
Sent: Sunday, January 23, 2005 4:22 PM
Subject: [ciencialist] O Feira e o Professor Luis Brudna




Estou dando umas aulas de ´Recursos de informatica para quimicos´
na Unijui (univ particular de Ijui, cidade onde nasci). Divido o
trabalho com um professor da informatica, ele dah a parte de
Windows, Word e Excel e eu fico com a parte de busca de informacoes
cientificas na internet e softs de quimica.

A turma tem bem poucos conhecimentos em informatica. Alguns nunca
usaram um computador. :(

O interessante dessas aulas eh que percebi como eh dificil achar
informacoes cientificas em portugues.

Deixei os alunos pesquisando livremente na internet, e o curioso
eh que muitos estavam visitando o site ´Feira de ciencias´. :) Enchi
o peito de orgulho e disse "Eu conheco o cara que faz esse site, ele
eh MUITO legal!". Uma aluna disse "Nao tem um site assim para a
quimica?!". Fiquei sem resposta. Nao conheco nenhum. Indiquei o site
do Emiliano, mas nao eh exatamente o estilo desejado.
Ela acabou copiando algumas coisas que estavam no ´Feira´, como
aquela expericia da Cromatografia em Papel.

Alias, o site do Emiliano foi muito util para eu propor o trabalho
que vale nota. A ideia foi tentar encontrar o maximo de informacoes
sobre um assunto e escrever um trabalho no Word. Escolhi muitos
assuntos que estavam no site do Emiliano. :)

Claro que nao perdi a oportunidade de falar do Humor na ciencia.
Heheh. Nao cheguei a indicar a Ciencialist pq eles nao sabem usar
direito o mail e uma lista com muitas mensagens ia acabar dando
confusao.

Em fevereiro comeco a dar aulas de Fisico Quimica II e FQ
Experimental. Vou ter que montar experimentos, junto com outros
professores, sobre FQ. Quem sabe nao sai um mini-site com alguns
experimentos. :)

Ateh
Luis Brudna
ps: pena que ainda nao tenho contrato fixo. :(



--- Em ciencialist@yahoogrupos.com.br, "Luiz Ferraz Netto"
<leobarretos@u...> escreveu
> Selo ou não selo, eis a questão.
>
> Bem pessoal, Feira de Ciências, O Imperdível! já se aproxima,
aceleradamente, da casa dos 10 milhões de consultas/vistas/pageview
(10 000 000) ... e o contador de acessos só tem 7 dígitos.
> A partir de então o contador registrará 0 000 000 e começara tudo
de novo ... e, um consulente novato (entrando pela primeira vez no
Imperdível) vai observar "coitado! tanto trabalho e nem uma boa
quantidade de visitas ele tem!" --- e meu ego põe uma capa, abre o
guarda-chuva e se despede de mim, por não saber nadar em minhas
lágrimas (putz, ficou bonito pacas!).
>
> Bem, foi sugestão do amigo Marcelo Moura, o astrônomo que
administra a Sala 24 do Feira de Ciências, que se faça um selo
comemorativo dos 10 MILHÕES pois, com ego ou não, atingir essa meta
num site de Educação e Divulgação Científica, no Brasil, é algo pelo
menos digno de ... um SELO!
>
> Assim sendo, e para presentear a posteridade, estou abrindo só
para aos amigos o Concurso do Selo Comemorativo do Imperdível.
Um "gif" (animado ou não) para ser colocado ao lado, ou abaixo do
contador de acessos, na coluna da esquerda, nas páginas INDEX das 25
Salas do site. Como prêmio posso oferecer (a escolher): um livro de
Física (pode ter certeza que será algo diferenciado --- alguém tem,
por exemplo, o livro para 'ensino médio' escrito por Eduard Branly --
- sim, o inventor do coesor Branly ---.), um gerador eletromagnético
didático ou um gerador eletrostático tipo Van de Graaff (esse só
poderá ser entregue a partir de julho, pois estou fazendo esse
projeto para fornecer kits desse gerador via WEB). Se interessar
posso oferecer também uma máquina Kirlian, [que já tem incorporada
um fantasma (fantasma residente)] --- acompanha 10 ml de ectoplasma -
-- e fornece fotos de auras em 4 dimensões :-).
>
> Alguém sugeriu colocar esse Concurso no próprio site do FEIRA;
será que 'cai bem'?
>
> Agradeço aos amigos que, direta ou indiretamente, participaram
desse sucesso do site. Eu, particularmente, não esperava tal
acolhida; o primeiro milhão foi realmente comemorado com festinha
regada a vinho aqui em casa --- tal foi meu 'espanto'.
>
> Aquele abraço,
>
> ===========================
> Luiz Ferraz Netto [Léo]
> leobarretos@u...
> http://www.feiradeciencias.com.br
> ===========================






##### ##### #####

Para saber mais visite
http://www.ciencialist.hpg.ig.com.br


##### ##### ##### #####
Links do Yahoo! Grupos













##### ##### #####

Para saber mais visite
http://www.ciencialist.hpg.ig.com.br


##### ##### ##### #####


Yahoo! Grupos, um serviço oferecido por:

São Paulo Rio de Janeiro Curitiba Porto Alegre Belo Horizonte Brasília




------------------------------------------------------------------------------
Links do Yahoo! Grupos

a.. Para visitar o site do seu grupo na web, acesse:
http://br.groups.yahoo.com/group/ciencialist/

b.. Para sair deste grupo, envie um e-mail para:
ciencialist-unsubscribe@yahoogrupos.com.br

c.. O uso que você faz do Yahoo! Grupos está sujeito aos Termos do Serviço do Yahoo!.



[As partes desta mensagem que não continham texto foram removidas]



SUBJECT: Re: César Lattes
FROM: "rayfisica" <rayfisica@yahoo.com.br>
TO: ciencialist@yahoogrupos.com.br
DATE: 23/01/2005 17:51


--- Em ciencialist@yahoogrupos.com.br, "JVictor" <jvoneto@u...>
escreveu
> Desconsiderem. Esta mensagem já foi postado´há um século. É que eu
estava revendo algumas e a reenviei num descuido.
>
> Victor.
>

Freud explica...
Talvez seja o gato da Natália que finalmente conseguiu escapar dela
Até diz um poeta já morto que o diabo dá os toques no teclado

Mas eu particularmente gostei muito.






SUBJECT: Re: [ciencialist] O Feira e o Professor Luis Brudna
FROM: "Prof. JC" <profjc2003@yahoo.com.br>
TO: <ciencialist@yahoogrupos.com.br>
DATE: 23/01/2005 17:59

Brudna,

Faiz favoire, me conta melhor essa história (pode ser em PVT mesmo). Também
estou trabalhando com capacitação de professores para o uso de computadores
e Internet.

Aliás, Léo, parabéns de novo (já lhe dei os parabéns lá no boteco, mas não
sei se você anda bebendo ultimamente, he he). 10^7 é um número cabalístico!

Ah, e se servir de (des)consolo, mais ou menos seis meses atrás eu criei uma
comunidade no Orkut com o objetivo de "entender melhor a relação dos
adolescentes com a física", ou, mais especificamente, as "más relações". A
comunidade chama-se "Eu odeio Física" e, pasmem, já está na casa dos 5.000
associados. :)))

Abraços,
Prof. JC


----- Original Message -----
From: "brudna" <lrb@iq.ufrgs.br>
To: <ciencialist@yahoogrupos.com.br>
Sent: Sunday, January 23, 2005 4:22 PM
Subject: [ciencialist] O Feira e o Professor Luis Brudna




Estou dando umas aulas de ´Recursos de informatica para quimicos´
na Unijui (univ particular de Ijui, cidade onde nasci). Divido o
trabalho com um professor da informatica, ele dah a parte de
Windows, Word e Excel e eu fico com a parte de busca de informacoes
cientificas na internet e softs de quimica.

A turma tem bem poucos conhecimentos em informatica. Alguns nunca
usaram um computador. :(

O interessante dessas aulas eh que percebi como eh dificil achar
informacoes cientificas em portugues.

Deixei os alunos pesquisando livremente na internet, e o curioso
eh que muitos estavam visitando o site ´Feira de ciencias´. :) Enchi
o peito de orgulho e disse "Eu conheco o cara que faz esse site, ele
eh MUITO legal!". Uma aluna disse "Nao tem um site assim para a
quimica?!". Fiquei sem resposta. Nao conheco nenhum. Indiquei o site
do Emiliano, mas nao eh exatamente o estilo desejado.
Ela acabou copiando algumas coisas que estavam no ´Feira´, como
aquela expericia da Cromatografia em Papel.

Alias, o site do Emiliano foi muito util para eu propor o trabalho
que vale nota. A ideia foi tentar encontrar o maximo de informacoes
sobre um assunto e escrever um trabalho no Word. Escolhi muitos
assuntos que estavam no site do Emiliano. :)

Claro que nao perdi a oportunidade de falar do Humor na ciencia.
Heheh. Nao cheguei a indicar a Ciencialist pq eles nao sabem usar
direito o mail e uma lista com muitas mensagens ia acabar dando
confusao.

Em fevereiro comeco a dar aulas de Fisico Quimica II e FQ
Experimental. Vou ter que montar experimentos, junto com outros
professores, sobre FQ. Quem sabe nao sai um mini-site com alguns
experimentos. :)

Ateh
Luis Brudna
ps: pena que ainda nao tenho contrato fixo. :(



--- Em ciencialist@yahoogrupos.com.br, "Luiz Ferraz Netto"
<leobarretos@u...> escreveu
> Selo ou não selo, eis a questão.
>
> Bem pessoal, Feira de Ciências, O Imperdível! já se aproxima,
aceleradamente, da casa dos 10 milhões de consultas/vistas/pageview
(10 000 000) ... e o contador de acessos só tem 7 dígitos.
> A partir de então o contador registrará 0 000 000 e começara tudo
de novo ... e, um consulente novato (entrando pela primeira vez no
Imperdível) vai observar "coitado! tanto trabalho e nem uma boa
quantidade de visitas ele tem!" --- e meu ego põe uma capa, abre o
guarda-chuva e se despede de mim, por não saber nadar em minhas
lágrimas (putz, ficou bonito pacas!).
>
> Bem, foi sugestão do amigo Marcelo Moura, o astrônomo que
administra a Sala 24 do Feira de Ciências, que se faça um selo
comemorativo dos 10 MILHÕES pois, com ego ou não, atingir essa meta
num site de Educação e Divulgação Científica, no Brasil, é algo pelo
menos digno de ... um SELO!
>
> Assim sendo, e para presentear a posteridade, estou abrindo só
para aos amigos o Concurso do Selo Comemorativo do Imperdível.
Um "gif" (animado ou não) para ser colocado ao lado, ou abaixo do
contador de acessos, na coluna da esquerda, nas páginas INDEX das 25
Salas do site. Como prêmio posso oferecer (a escolher): um livro de
Física (pode ter certeza que será algo diferenciado --- alguém tem,
por exemplo, o livro para 'ensino médio' escrito por Eduard Branly --
- sim, o inventor do coesor Branly ---.), um gerador eletromagnético
didático ou um gerador eletrostático tipo Van de Graaff (esse só
poderá ser entregue a partir de julho, pois estou fazendo esse
projeto para fornecer kits desse gerador via WEB). Se interessar
posso oferecer também uma máquina Kirlian, [que já tem incorporada
um fantasma (fantasma residente)] --- acompanha 10 ml de ectoplasma -
-- e fornece fotos de auras em 4 dimensões :-).
>
> Alguém sugeriu colocar esse Concurso no próprio site do FEIRA;
será que 'cai bem'?
>
> Agradeço aos amigos que, direta ou indiretamente, participaram
desse sucesso do site. Eu, particularmente, não esperava tal
acolhida; o primeiro milhão foi realmente comemorado com festinha
regada a vinho aqui em casa --- tal foi meu 'espanto'.
>
> Aquele abraço,
>
> ===========================
> Luiz Ferraz Netto [Léo]
> leobarretos@u...
> http://www.feiradeciencias.com.br
> ===========================






##### ##### #####

Para saber mais visite
http://www.ciencialist.hpg.ig.com.br


##### ##### ##### #####
Links do Yahoo! Grupos













SUBJECT: Re: [ciencialist] Mulheres: Cozinha e na cama, já!!!!!!!!!!
FROM: "Alvaro Augusto \(E\)" <alvaro@electraenergy.com.br>
TO: <ciencialist@yahoogrupos.com.br>
DATE: 23/01/2005 18:28

Em relação a mulheres ocupando altos cargos em universidades, empresas ou qualquer coisa parecida, fico com as considerações de Scott Adams, criador do personagem Dilbert: "Não importa que haja uma mulher lá. O que importa é que não é você!"

De qualquer forma, os homens estão apavorados. Nas sociedades tribais havia uma clara divisão de trabalho: homens eram caçadores, mulheres eram agricultoras; homens traziam comida para casa, mulheres preparavam a comida e cuidavam da casa; homens tratavam dos interesses tribais; mulhreres tratavam dos interesses do clã. E, embora os homens e mulheres de hoje em dia não sejam em nada diferentes dos homens e mulhreres do neolítico, há uma pequena diferença: tecnologia. Devido à tecnologia as mulheres podem, se assim o quiserem, evitar ter um filho por ano durante a idade fértil. Devido à tecnologia, as mulheres podem fazer tudo o que os homens fazem, inclusive dirigir máquinas pesadas (direção hidráulica!), pilotar aviões, dirigir empresas, lecionar, etc. As mulheres podem, em resumo, colocar comida na mesa, tornando-se caçadoras, e não mais agricultoras. E não há nada mais apavorante, para um homem, do que ver uma mulher caçando!

[ ]s

Alvaro Augusto


----- Original Message -----
From: JVictor
To: ciencialist@yahoogrupos.com.br
Sent: Sunday, January 23, 2005 12:11 PM
Subject: Re: [ciencialist] Mulheres: Cozinha e na cama, já!!!!!!!!!!


Natália,


É evidente que esse Summers tem algum problema de identidade sexual. Há um parafuso frouxo nessa área da mente. Ele precisa valorizar o seu lado sexual, e que deus me livre de arriscar qual seja. Como os assim assim da vida, que valorizam os grandes personagens, suspeitos de também serem meninos alegres. Pois isso pode servir de âncora para a manutenção e aceitação da classe, mais esta do que aquela. Há até um movimento, Orgulho Gay(embora até hoje eu não tenha conseguido entender de quê é esse tal orgulho. Também não consigo entender o Apocalipse, no sentido de que qual mente louca pode escrever tanta coisa ridícula) para marcar suas presenças neste mundinho. Não tenho nada contra o que quer os outros façam e usem, claro. Cada um é cada um. Respeito is good e todos gostam. Agora, aproveitar-se de uma pretensa autoridade científica e de um cargo para pronunciar tamanha besteira é algo que transcende as raias do suportável e do racional. É claro que é mais uma falácia. As mulheres têm capacidade idêntica sim, apenas os preconceitos e as oportunidades, são muitos os primeiros e pouquinhos as segundas.
Por outro lado, cozinha nem tanto. Mas cama, isso lá, sim, deveria ser muito tanto!...
Estou com vocês e não abro, nem que a tampa voe.

Sds,

Victor.

----- Original Message -----
From: Maria Natália
To: ciencialist@yahoogrupos.com.br
Sent: Saturday, January 22, 2005 7:13 PM
Subject: [ciencialist] Mulheres: Cozinha e na cama, já!!!!!!!!!!



Esta mensagem tem muito de Ciência embora se destine a agitar as
massas machistas da lista.
http://www.president.harvard.edu/speeches/2005/womensci.html
No comments.
Um abração
da Maria Natália





##### ##### #####

Para saber mais visite
http://www.ciencialist.hpg.ig.com.br


##### ##### ##### #####


Yahoo! Grupos, um serviço oferecido por:
PUBLICIDADE




------------------------------------------------------------------------------
Links do Yahoo! Grupos

a.. Para visitar o site do seu grupo na web, acesse:
http://br.groups.yahoo.com/group/ciencialist/

b.. Para sair deste grupo, envie um e-mail para:
ciencialist-unsubscribe@yahoogrupos.com.br

c.. O uso que você faz do Yahoo! Grupos está sujeito aos Termos do Serviço do Yahoo!.



[As partes desta mensagem que não continham texto foram removidas]



##### ##### #####

Para saber mais visite
http://www.ciencialist.hpg.ig.com.br


##### ##### ##### #####


Yahoo! Grupos, um serviço oferecido por:
PUBLICIDADE




------------------------------------------------------------------------------
Links do Yahoo! Grupos

a.. Para visitar o site do seu grupo na web, acesse:
http://br.groups.yahoo.com/group/ciencialist/

b.. Para sair deste grupo, envie um e-mail para:
ciencialist-unsubscribe@yahoogrupos.com.br

c.. O uso que você faz do Yahoo! Grupos está sujeito aos Termos do Serviço do Yahoo!.



[As partes desta mensagem que não continham texto foram removidas]



SUBJECT: orguloso do saber?
FROM: "rayfisica" <rayfisica@yahoo.com.br>
TO: ciencialist@yahoogrupos.com.br
DATE: 23/01/2005 19:11


Orgulhoso do próprio saber!
Não é uma agressão e nem direcionado a ser humano algum,
estou apenas
colhendo informações para aumentar o meu próprio saber.
Os cientistas de hoje tem razão de ficarem orgulhosos do saber que
possuem ou que possam possuir?
Tomando por base as ultimas comunicações vejo a distancia entre os
cientistas e o vulgo, parece-me que é impossível à pessoa
comum
entender patavina do que falam os doutos.
Não seria transformar a ciência em religião como fez a
igreja que ate
mandou pessoas para a fogueira em nome da crença na criação
conforme
o descrito na bíblia e agora vem com uma nova historia como se
não
tivesse mudado de idéia.
Ou dentro do próprio seio da ciência onde pessoas eram
arrogantes por
possuir conhecimentos que em sua época era o ápice e que hoje
não
passa de falácias, se cometermos os mesmos erros não poderemos
passar
por tolos arrogantes no futuro.
As teorias que hoje são o nosso orgulho estão suficientemente
esclarecidas para que tenhamos a certeza que não cairão por
terra
como a éter universal, por exemplo?
Eu que sou muito mais ignorante do que até eu acredito, não
consegui
engolir o neutrino, pois eu penso que as demonstrações ditas
empíricas do neutrino não passa das mesmas ocorrências que
deram a
duvida que originou a invenção do neutrino, e o resultado não
foi de
modo algum satisfatório como por exemplo a queda de corpos com
massa
diferente realizado por Galileu.
E plagiando alguém aqui da lista quem investiria o seu dinheiro
num
processo que demonstra apenas um terço do esperado pela teoria,
que
não os mesmos crentes da igreja, que repito queimaram seus
irmãos,
pela sua certeza.
Conversando um dia com um professor sobre uma duvida, após algum
tempo ele virou para mim, talvez até para se livrar de mim e
disse,
eu já estou muito bitolado (eu particularmente não quero ser um
bitolado).
Com certeza não sou douto, porem com mais certeza não sou um
sacerdote bitolado.
Penso que há um enorme castelo construído sobre um alicerce
aparentemente frágil, fico preocupado se o alicerce cair o que
sustentara o castelo, e é lógico que quem habita o castelo esta
tranqüilo, tranqüilo.
Esse é apenas um exemplo com certeza os integrantes da lista com
mais
saber que eu serão capaz de encontrar outros furos nesse alicerce.
Se é que há os furos.






SUBJECT: Re: [ciencialist] O Feira e o Professor Luis Brudna
FROM: "Emiliano Chemello - Yahoo Grupos" <chemelloe@yahoo.com.br>
TO: <ciencialist@yahoogrupos.com.br>
DATE: 23/01/2005 22:36

Olá Brudna,

Até que enfim meus textos serviram para alguma coisa! :-) Novidades este
ano! Site novo, escritores (além de mim) novos, tudo novo! Muita química
para 2005. Aguardem! :-)

Falando em site, hoje encontrei com um professor amigo meu e ele me
disse que eu estou famoso ai na UFRGS. Só que ninguém me conhece
pessoalmente. Essa tecnologias ... :-)

Aquele abraço do net-amigo,

Emiliano Chemello

----- Original Message -----
From: brudna
To: ciencialist@yahoogrupos.com.br
Sent: Sunday, January 23, 2005 4:22 PM
Subject: [ciencialist] O Feira e o Professor Luis Brudna



Estou dando umas aulas de ´Recursos de informatica para quimicos´
na Unijui (univ particular de Ijui, cidade onde nasci). Divido o
trabalho com um professor da informatica, ele dah a parte de
Windows, Word e Excel e eu fico com a parte de busca de informacoes
cientificas na internet e softs de quimica.

A turma tem bem poucos conhecimentos em informatica. Alguns nunca
usaram um computador. :(

O interessante dessas aulas eh que percebi como eh dificil achar
informacoes cientificas em portugues.

Deixei os alunos pesquisando livremente na internet, e o curioso
eh que muitos estavam visitando o site ´Feira de ciencias´. :) Enchi
o peito de orgulho e disse "Eu conheco o cara que faz esse site, ele
eh MUITO legal!". Uma aluna disse "Nao tem um site assim para a
quimica?!". Fiquei sem resposta. Nao conheco nenhum. Indiquei o site
do Emiliano, mas nao eh exatamente o estilo desejado.
Ela acabou copiando algumas coisas que estavam no ´Feira´, como
aquela expericia da Cromatografia em Papel.

Alias, o site do Emiliano foi muito util para eu propor o trabalho
que vale nota. A ideia foi tentar encontrar o maximo de informacoes
sobre um assunto e escrever um trabalho no Word. Escolhi muitos
assuntos que estavam no site do Emiliano. :)

Claro que nao perdi a oportunidade de falar do Humor na ciencia.
Heheh. Nao cheguei a indicar a Ciencialist pq eles nao sabem usar
direito o mail e uma lista com muitas mensagens ia acabar dando
confusao.

Em fevereiro comeco a dar aulas de Fisico Quimica II e FQ
Experimental. Vou ter que montar experimentos, junto com outros
professores, sobre FQ. Quem sabe nao sai um mini-site com alguns
experimentos. :)

Ateh
Luis Brudna
ps: pena que ainda nao tenho contrato fixo. :(



--- Em ciencialist@yahoogrupos.com.br, "Luiz Ferraz Netto"
<leobarretos@u...> escreveu
> Selo ou não selo, eis a questão.
>
> Bem pessoal, Feira de Ciências, O Imperdível! já se aproxima,
aceleradamente, da casa dos 10 milhões de consultas/vistas/pageview
(10 000 000) ... e o contador de acessos só tem 7 dígitos.
> A partir de então o contador registrará 0 000 000 e começara tudo
de novo ... e, um consulente novato (entrando pela primeira vez no
Imperdível) vai observar "coitado! tanto trabalho e nem uma boa
quantidade de visitas ele tem!" --- e meu ego põe uma capa, abre o
guarda-chuva e se despede de mim, por não saber nadar em minhas
lágrimas (putz, ficou bonito pacas!).
>
> Bem, foi sugestão do amigo Marcelo Moura, o astrônomo que
administra a Sala 24 do Feira de Ciências, que se faça um selo
comemorativo dos 10 MILHÕES pois, com ego ou não, atingir essa meta
num site de Educação e Divulgação Científica, no Brasil, é algo pelo
menos digno de ... um SELO!
>
> Assim sendo, e para presentear a posteridade, estou abrindo só
para aos amigos o Concurso do Selo Comemorativo do Imperdível.
Um "gif" (animado ou não) para ser colocado ao lado, ou abaixo do
contador de acessos, na coluna da esquerda, nas páginas INDEX das 25
Salas do site. Como prêmio posso oferecer (a escolher): um livro de
Física (pode ter certeza que será algo diferenciado --- alguém tem,
por exemplo, o livro para 'ensino médio' escrito por Eduard Branly --
- sim, o inventor do coesor Branly ---.), um gerador eletromagnético
didático ou um gerador eletrostático tipo Van de Graaff (esse só
poderá ser entregue a partir de julho, pois estou fazendo esse
projeto para fornecer kits desse gerador via WEB). Se interessar
posso oferecer também uma máquina Kirlian, [que já tem incorporada
um fantasma (fantasma residente)] --- acompanha 10 ml de ectoplasma -
-- e fornece fotos de auras em 4 dimensões :-).
>
> Alguém sugeriu colocar esse Concurso no próprio site do FEIRA;
será que 'cai bem'?
>
> Agradeço aos amigos que, direta ou indiretamente, participaram
desse sucesso do site. Eu, particularmente, não esperava tal
acolhida; o primeiro milhão foi realmente comemorado com festinha
regada a vinho aqui em casa --- tal foi meu 'espanto'.
>
> Aquele abraço,
>
> ===========================
> Luiz Ferraz Netto [Léo]
> leobarretos@u...
> http://www.feiradeciencias.com.br
> ===========================






##### ##### #####

Para saber mais visite
http://www.ciencialist.hpg.ig.com.br


##### ##### ##### #####


Yahoo! Grupos, um serviço oferecido por:

São Paulo Rio de Janeiro Curitiba Porto Alegre Belo Horizonte Brasília





Links do Yahoo! Grupos

Para visitar o site do seu grupo na web, acesse:
http://br.groups.yahoo.com/group/ciencialist/

Para sair deste grupo, envie um e-mail para:
ciencialist-unsubscribe@yahoogrupos.com.br

O uso que você faz do Yahoo! Grupos está sujeito aos Termos do Serviço do
Yahoo!.




SUBJECT: Re: [ciencialist] Gravitação... (prof. leo)
FROM: "Rick" <rickardorios@yahoo.com.br>
TO: "l - Ciencia" <ciencialist@yahoogrupos.com.br>
DATE: 23/01/2005 22:49

Professor léo, agradeço suas explicações. Mais tem algumas coisas que ainda
não entendi. O senhor afirmou que peso é a resultante das forças que agem
sobre uma determinada massa mergulhada no campo gravitacional. De quais
forças o peso resulta? pensei que a unica força presente fosse a de atração
gravitacional, que, obviamente, resultaria no peso. '

O par ação e reação que falei tem haver com as forças trocadas entre as
massas atrativas. Por exemplo, a terra atrai a lua e a lua, reagindo, atrai
a terra. Muito embora, acabei de perceber, talvez esteja errado em afirmar
que isto é um conceito de ação reação, pois, mesmo obedecendo as
caracteristicas vetorias, elas diferem quanto a intensidade... acho que me
expressei mal no ultimo e-mail, malz :(

Eu não entendi o porquê da igualdade nao ser verdadeira na superficie da
terra. Como a força centrifuga age nesta situação?

Amplexos estrelares...
Rick
----- Original Message -----
From: Luiz Ferraz Netto
To: ciencialist@yahoogrupos.com.br
Sent: Thursday, January 20, 2005 9:26 AM
Subject: Re: [ciencialist] Gravitação...


Olá Rick,

de verdade em verdade vos digo:

1) Peso é a resultante das forças que agem sobre uma determinada massa
mergulhada no campo gravitacional de alguma outra massa e é dependente do
referencial adotado.
Não tem sentido a pergunta "Qual o peso da Lua?" sem a indicação do
referencial, assim como é deficiente a pergunta "Qual o peso de um Fusca?"
sem a indicação do local onde ele se encontra sobre a superfície da Terra
(não há Fuscas em outros planetas ou satélites --- não esqueça: de verdade
em verdade vos digo!)
Força de atração gravitacional é exclusivamente a força entre duas
massas (para ficar dentro dos exemplos).
Em algumas situações 'ideais' o peso de um corpo e a força gravitacional
sobre ele se confundem; é o caso de um corpo caindo sobre a superfície da
Terra (abstraindo-se a presença do ar e de todos os movimentos da Terra). É
tb o caso do peso dos satélites artificiais em órbita. É o caso de se
calcular o peso da Lua em relação à Terra, ou em relação ao Sol.
Para o corpo na superfície da Terra, por exemplo, essa igualdade já
não é verdadeira devido à presença da força centrífuga que age no corpo no
referencial Terra, além da atração gravitacional. Nesse caso, o peso do
corpo é dependente da latitude (pois altera a distância dele até o eixo da
Terra).
Não entendi o que é que o par ação/reação tem a ver com isso?

2) "A força centrípeta que age neste corpo é a força de atração
gravitacional?"
Resposta: Sim, e nesse caso é tb denominada 'força central'.
"Se a resposta for afirmativa, igualando-se as duas expressões
obtém-se V = raiz (GM/R), o que torna a velocidade constante. Isso não
deveria contrariar a aceleração que existe na velocidade de translação?"
Resposta: Conclusão falsa.
Vejamos: GmM/r^2 = mv^2/r ===> v = sqrt(GM/r) <===
Observe: v é função de r (onde r é o raio de curvatura da curva no
instante considerado). Raio de curvatura de uma curva, num dado ponto, é o
raio da circunferência osculadora à curva nesse ponto.

[]'
===========================
Luiz Ferraz Netto [Léo]
leobarretos@uol.com.br
http://www.feiradeciencias.com.br
===========================
-----Mensagem Original-----
De: "Rick" <rickardorios@yahoo.com.br>
Para: "l - Ciencia" <ciencialist@yahoogrupos.com.br>
Enviada em: quarta-feira, 19 de janeiro de 2005 15:01
Assunto: [ciencialist] Gravitação...



Pessoal, mais algumas dúvidas astronômicas... :)

1º) Qual a relação entre o peso e a força de atração gravitacional? Como
todas as duas representam um par de acao e reacao entre massas elas nao
deveriam ser a mesma coisa?

2º) Quando o movimento de um corpo em orbita é eliptico, a velocidade
aerolar é constante e a velocidade de translação sofre variações a
depender
da posição em que se encontra na orbita. A força centripeta que age neste
corpo é a força de atração gravitacional? Se a resposta for afirmativa,
igualando-se as duas expressôes obtem-se V = raiz (GM/R), o que torna a
velocidade constante. Isso nao deveria contrariar a aceleração que existe
na
velocidade de translação? Caso a resposta seja negativa, qual seria a
força
centripeta que atua no corpo?


Obrigaduuu!!!

Rick, plagiando o Fábio Junior... hehe :P




##### ##### #####

Para saber mais visite
http://www.ciencialist.hpg.ig.com.br


##### ##### ##### #####
Links do Yahoo! Grupos










--
No virus found in this incoming message.
Checked by AVG Anti-Virus.
Version: 7.0.300 / Virus Database: 265.6.13 - Release Date: 16/01/2005




--
No virus found in this outgoing message.
Checked by AVG Anti-Virus.
Version: 7.0.300 / Virus Database: 265.6.13 - Release Date: 16/01/2005



##### ##### #####

Para saber mais visite
http://www.ciencialist.hpg.ig.com.br


##### ##### ##### #####


Yahoo! Grupos, um serviço oferecido por:
PUBLICIDADE




----------------------------------------------------------------------------
--
Links do Yahoo! Grupos

a.. Para visitar o site do seu grupo na web, acesse:
http://br.groups.yahoo.com/group/ciencialist/

b.. Para sair deste grupo, envie um e-mail para:
ciencialist-unsubscribe@yahoogrupos.com.br

c.. O uso que você faz do Yahoo! Grupos está sujeito aos Termos do
Serviço do Yahoo!.





SUBJECT: RE: [ciencialist] Selo Comemorativo - Imperdivel !
FROM: "murilo filo" <avalanchedrive@hotmail.com>
TO: ciencialist@yahoogrupos.com.br
DATE: 23/01/2005 23:21

Léo, oi.
Parabens mesmo! Esta é uma boa marca mundial: 10 milhões!
Quanto ao selo, tenho uma idéia bem porretinha! Sou 1/2 publicitário, entre
outras coisas.
Posso lhe passar a descritiva do job e vc fazer, ou mandar fazer, a arte.
Que tal?
Aceita? Mando em aberto? Não precisa prêmio! abr/M. SP 23/jan

>From: "Luiz Ferraz Netto" <leobarretos@uol.com.br>
>Reply-To: ciencialist@yahoogrupos.com.br
>To: <ciencialist@yahoogrupos.com.br>
>CC: <fisica-mg@grupos.com.br>,"Marcelo Moura"
><mmmoura.bh@terra.com.br>,<eletronicabr@yahoogrupos.com.br>,
><fisica@grupos.com.br>,<galena_brasil@yahoogrupos.com.br>,<Conversa_de_Botequim@yahoogrupos.com.br>,
><sbcr@yahoogrupos.com.br>,<strbrasil@yahoogrupos.com.br>
>Subject: [ciencialist] Selo Comemorativo - Imperdivel !
>Date: Sun, 23 Jan 2005 08:57:46 -0200
>
>Selo ou não selo, eis a questão.
>
>Bem pessoal, Feira de Ciências, O Imperdível! já se aproxima,
>aceleradamente, da casa dos 10 milhões de consultas/vistas/pageview (10 000
>000) ... e o contador de acessos só tem 7 dígitos.
>A partir de então o contador registrará 0 000 000 e começara tudo de novo
>... e, um consulente novato (entrando pela primeira vez no Imperdível) vai
>observar "coitado! tanto trabalho e nem uma boa quantidade de visitas ele
>tem!" --- e meu ego põe uma capa, abre o guarda-chuva e se despede de mim,
>por não saber nadar em minhas lágrimas (putz, ficou bonito pacas!).
>
>Bem, foi sugestão do amigo Marcelo Moura, o astrônomo que administra a Sala
>24 do Feira de Ciências, que se faça um selo comemorativo dos 10 MILHÕES
>pois, com ego ou não, atingir essa meta num site de Educação e Divulgação
>Científica, no Brasil, é algo pelo menos digno de ... um SELO!
>
>Assim sendo, e para presentear a posteridade, estou abrindo só para aos
>amigos o Concurso do Selo Comemorativo do Imperdível. Um "gif" (animado ou
>não) para ser colocado ao lado, ou abaixo do contador de acessos, na coluna
>da esquerda, nas páginas INDEX das 25 Salas do site. Como prêmio posso
>oferecer (a escolher): um livro de Física (pode ter certeza que será algo
>diferenciado --- alguém tem, por exemplo, o livro para 'ensino médio'
>escrito por Eduard Branly --- sim, o inventor do coesor Branly ---.), um
>gerador eletromagnético didático ou um gerador eletrostático tipo Van de
>Graaff (esse só poderá ser entregue a partir de julho, pois estou fazendo
>esse projeto para fornecer kits desse gerador via WEB). Se interessar posso
>oferecer também uma máquina Kirlian, [que já tem incorporada um fantasma
>(fantasma residente)] --- acompanha 10 ml de ectoplasma --- e fornece fotos
>de auras em 4 dimensões :-).
>
>Alguém sugeriu colocar esse Concurso no próprio site do FEIRA; será que
>'cai bem'?
>
>Agradeço aos amigos que, direta ou indiretamente, participaram desse
>sucesso do site. Eu, particularmente, não esperava tal acolhida; o primeiro
>milhão foi realmente comemorado com festinha regada a vinho aqui em casa
>--- tal foi meu 'espanto'.
>
>Aquele abraço,
>
> ===========================
> Luiz Ferraz Netto [Léo]
> leobarretos@uol.com.br
> http://www.feiradeciencias.com.br
> ===========================
> ----------
>
>Internal Virus Database is out-of-date.
>Checked by AVG Anti-Virus.
>Version: 7.0.300 / Virus Database: 265.6.13 - Release Date: 16/01/2005
>
>
>[As partes desta mensagem que não continham texto foram removidas]
>




SUBJECT: Re: [ciencialist] Mulheres: Cozinha e na cama, já!!!!!!!!!!
FROM: "murilo filo" <avalanchedrive@hotmail.com>
TO: ciencialist@yahoogrupos.com.br
DATE: 23/01/2005 23:41

Nada mais natural do que ver uma dona moderna caçando...
Pior mesmo é vc ser a caça dela! Mulher marvada dá muito menos mole do que
homem!
É kréu, sem compaixão... e tudo bem dentro das regras e sempre no 'pessoal'!
:[
E se vc tiver medo, a coisa fica muito pior! ( e ela ainda pode chorar!)
M.

>From: "Alvaro Augusto (E)" <alvaro@electraenergy.com.br>
>Reply-To: ciencialist@yahoogrupos.com.br
>To: <ciencialist@yahoogrupos.com.br>
>Subject: Re: [ciencialist] Mulheres: Cozinha e na cama, já!!!!!!!!!!
>Date: Sun, 23 Jan 2005 18:28:07 -0200
>
>Em relação a mulheres ocupando altos cargos em universidades, empresas ou
>qualquer coisa parecida, fico com as considerações de Scott Adams, criador
>do personagem Dilbert: "Não importa que haja uma mulher lá. O que importa é
>que não é você!"
>
>De qualquer forma, os homens estão apavorados. Nas sociedades tribais havia
>uma clara divisão de trabalho: homens eram caçadores, mulheres eram
>agricultoras; homens traziam comida para casa, mulheres preparavam a comida
>e cuidavam da casa; homens tratavam dos interesses tribais; mulhreres
>tratavam dos interesses do clã. E, embora os homens e mulheres de hoje em
>dia não sejam em nada diferentes dos homens e mulhreres do neolítico, há
>uma pequena diferença: tecnologia. Devido à tecnologia as mulheres podem,
>se assim o quiserem, evitar ter um filho por ano durante a idade fértil.
>Devido à tecnologia, as mulheres podem fazer tudo o que os homens fazem,
>inclusive dirigir máquinas pesadas (direção hidráulica!), pilotar aviões,
>dirigir empresas, lecionar, etc. As mulheres podem, em resumo, colocar
>comida na mesa, tornando-se caçadoras, e não mais agricultoras. E não há
>nada mais apavorante, para um homem, do que ver uma mulher caçando!
>
>[ ]s
>
>Alvaro Augusto
>
>
> ----- Original Message -----
> From: JVictor
> To: ciencialist@yahoogrupos.com.br
> Sent: Sunday, January 23, 2005 12:11 PM
> Subject: Re: [ciencialist] Mulheres: Cozinha e na cama, já!!!!!!!!!!
>
>
> Natália,
>
>
> É evidente que esse Summers tem algum problema de identidade sexual. Há
>um parafuso frouxo nessa área da mente. Ele precisa valorizar o seu lado
>sexual, e que deus me livre de arriscar qual seja. Como os assim assim da
>vida, que valorizam os grandes personagens, suspeitos de também serem
>meninos alegres. Pois isso pode servir de âncora para a manutenção e
>aceitação da classe, mais esta do que aquela. Há até um movimento, Orgulho
>Gay(embora até hoje eu não tenha conseguido entender de quê é esse tal
>orgulho. Também não consigo entender o Apocalipse, no sentido de que qual
>mente louca pode escrever tanta coisa ridícula) para marcar suas presenças
>neste mundinho. Não tenho nada contra o que quer os outros façam e usem,
>claro. Cada um é cada um. Respeito is good e todos gostam. Agora,
>aproveitar-se de uma pretensa autoridade científica e de um cargo para
>pronunciar tamanha besteira é algo que transcende as raias do suportável e
>do racional. É claro que é mais uma falácia. As mulheres têm capacidade
>idêntica sim, apenas os preconceitos e as oportunidades, são muitos os
>primeiros e pouquinhos as segundas.
> Por outro lado, cozinha nem tanto. Mas cama, isso lá, sim, deveria ser
>muito tanto!...
> Estou com vocês e não abro, nem que a tampa voe.
>
> Sds,
>
> Victor.
>
> ----- Original Message -----
> From: Maria Natália
> To: ciencialist@yahoogrupos.com.br
> Sent: Saturday, January 22, 2005 7:13 PM
> Subject: [ciencialist] Mulheres: Cozinha e na cama, já!!!!!!!!!!
>
>
>
> Esta mensagem tem muito de Ciência embora se destine a agitar as
> massas machistas da lista.
> http://www.president.harvard.edu/speeches/2005/womensci.html
> No comments.
> Um abração
> da Maria Natália
>
>
>
>
>
> ##### ##### #####
>
> Para saber mais visite
> http://www.ciencialist.hpg.ig.com.br
>
>
> ##### ##### ##### #####
>
>
> Yahoo! Grupos, um serviço oferecido por:
> PUBLICIDADE
>
>
>
>
>
>------------------------------------------------------------------------------
> Links do Yahoo! Grupos
>
> a.. Para visitar o site do seu grupo na web, acesse:
> http://br.groups.yahoo.com/group/ciencialist/
>
> b.. Para sair deste grupo, envie um e-mail para:
> ciencialist-unsubscribe@yahoogrupos.com.br
>
> c.. O uso que você faz do Yahoo! Grupos está sujeito aos Termos do
>Serviço do Yahoo!.
>
>
>
> [As partes desta mensagem que não continham texto foram removidas]
>
>
>
> ##### ##### #####
>
> Para saber mais visite
> http://www.ciencialist.hpg.ig.com.br
>
>
> ##### ##### ##### #####
>
>
> Yahoo! Grupos, um serviço oferecido por:
> PUBLICIDADE
>
>
>
>
>------------------------------------------------------------------------------
> Links do Yahoo! Grupos
>
> a.. Para visitar o site do seu grupo na web, acesse:
> http://br.groups.yahoo.com/group/ciencialist/
>
> b.. Para sair deste grupo, envie um e-mail para:
> ciencialist-unsubscribe@yahoogrupos.com.br
>
> c.. O uso que você faz do Yahoo! Grupos está sujeito aos Termos do
>Serviço do Yahoo!.
>
>
>
>[As partes desta mensagem que não continham texto foram removidas]
>




SUBJECT: Re: Essas, pois, são as inquietudes tolas de mais um bobo.(2)
FROM: Manuel Bulcão <manuelbulcao@uol.com.br>
TO: ciencialist@yahoogrupos.com.br
DATE: 23/01/2005 23:47


Salve Victor e demais!

VICTOR: Grande Manoel,

MANUEL, RETRIBUINDO A GENTILEZA:Grande? Talvez, mas certamente um
liliputiano frente à Vossa Enormidade!:-)

VICTOR: Se você reparar direitinho verá que o "assunto
controvertido" assim o é em razão de "forçadas de barra"...

MANUEL: Talvez em relação à Teoria da Relatividade Restrita haja
mesmo alguma lenha (não apenas folhas verdes, que produzem muito
mais fumaça do que fogo) capaz de sustentar as labaredas de uma boa
controvérsia.

Afinal, muitos membros ilustres do establishment científico referem-
se à TRR como "Teoria da Relatividade Especial de Einstein e
Poincaré" (p. ex., Roger Penrose, cf. "A Mente Nova do Rei", Ed.
Campus, quarta reimpressão, pp. 212/224), do mesmo modo que muitos
biólogos (inclusive o grande mestre uspiano Roberto Takata) referem-
se à Teoria da evolução por descendência com modificação
como "Teoria de Darwin-Wallace".

Mas não é à toa ou por conta apenas da ordem alfabética -- nem por
obra de uma confraria de ateístas sanguinários que detém o controle
total da mídia por meios sub-reptícios que só o Olavo de Carvalho é
capaz de desmascarar -- que Darwin aparece em primeiro plano, quase
a obliterar por completo Wallace. E a diferença não está apenas na
forma de explanar as idéias, no talento ou estilo literário. Aliás,
o "opus magnus" de Darwin não é um livro agradável de se ler. Nele,
a Grande Idéia se perde em meio a uma profusão de minudências
empíricas. Em termos de chatice, é comparável ao "Em busca do Tempo
Perdido", de Proust.

Darwin se sobressai "também" porque sua formulação – que destaca o
papel do acaso – é capaz de explicar fenômenos que escaparam à
compreensão de Wallace. Como, por exemplo, a "consciência" humana, o
senso moral (que amiúde contradiz a bio-`lógica') característico do
Homo sapiens. Para explicar este fenômeno, Wallace lançou mão de
um "skyhook" (Deus, vaga resposta para qualquer pergunta), enquanto
que a versão de Darwin prescinde disso -- embora, para alguns (entre
os quais me incluo), abre um enorme buraco no peito.

Pois bem, para encurtar a conversa (fiquei com sono), Einstein está
para Darwin assim como Poincaré está para Wallace. "Mutatis
mutandis".

VICTOR: Você até que pitacou direitinho, dentro da coerência e
honestidade intelectual que lhe são peculiares.

MANUEL: Ufa! Sua avaliação não chega a ser um elogio, mas é melhor
que o silêncio altaneiro do especialista. Obrigado :-)

Grande abraço,
Manuel Bulcão.






SUBJECT: RE: [ciencialist] orguloso do saber?
FROM: "murilo filo" <avalanchedrive@hotmail.com>
TO: ciencialist@yahoogrupos.com.br
DATE: 23/01/2005 23:50

Quanto mais se sobe na montanha, mais se avistam terras estranhas por
conhecer...
Falando-se em conhecimento, de repente fiquei preocupado com a qualidade do
inglês falado pelo comandante companheiro do aerolula... será que poderemos
ficar tranquilos... carregando tanta gente importante pelo mundo afora?? Xí!
Sei não! abr.M.

>From: "rayfisica" <rayfisica@yahoo.com.br>
>Reply-To: ciencialist@yahoogrupos.com.br
>To: ciencialist@yahoogrupos.com.br
>Subject: [ciencialist] orguloso do saber?
>Date: Sun, 23 Jan 2005 21:11:06 -0000
>
>
>Orgulhoso do próprio saber!
>Não é uma agressão e nem direcionado a ser humano algum,
>estou apenas
>colhendo informações para aumentar o meu próprio saber.
>Os cientistas de hoje tem razão de ficarem orgulhosos do saber que
>possuem ou que possam possuir?
>Tomando por base as ultimas comunicações vejo a distancia entre os
>cientistas e o vulgo, parece-me que é impossível à pessoa
>comum
>entender patavina do que falam os doutos.
>Não seria transformar a ciência em religião como fez a
>igreja que ate
>mandou pessoas para a fogueira em nome da crença na criação
>conforme
>o descrito na bíblia e agora vem com uma nova historia como se
>não
>tivesse mudado de idéia.
>Ou dentro do próprio seio da ciência onde pessoas eram
>arrogantes por
>possuir conhecimentos que em sua época era o ápice e que hoje
>não
>passa de falácias, se cometermos os mesmos erros não poderemos
>passar
>por tolos arrogantes no futuro.
>As teorias que hoje são o nosso orgulho estão suficientemente
>esclarecidas para que tenhamos a certeza que não cairão por
>terra
>como a éter universal, por exemplo?
>Eu que sou muito mais ignorante do que até eu acredito, não
>consegui
>engolir o neutrino, pois eu penso que as demonstrações ditas
>empíricas do neutrino não passa das mesmas ocorrências que
>deram a
>duvida que originou a invenção do neutrino, e o resultado não
>foi de
>modo algum satisfatório como por exemplo a queda de corpos com
>massa
>diferente realizado por Galileu.
>E plagiando alguém aqui da lista quem investiria o seu dinheiro
>num
>processo que demonstra apenas um terço do esperado pela teoria,
>que
>não os mesmos crentes da igreja, que repito queimaram seus
>irmãos,
>pela sua certeza.
>Conversando um dia com um professor sobre uma duvida, após algum
>tempo ele virou para mim, talvez até para se livrar de mim e
>disse,
>eu já estou muito bitolado (eu particularmente não quero ser um
>bitolado).
>Com certeza não sou douto, porem com mais certeza não sou um
>sacerdote bitolado.
>Penso que há um enorme castelo construído sobre um alicerce
>aparentemente frágil, fico preocupado se o alicerce cair o que
>sustentara o castelo, e é lógico que quem habita o castelo esta
>tranqüilo, tranqüilo.
>Esse é apenas um exemplo com certeza os integrantes da lista com
>mais
>saber que eu serão capaz de encontrar outros furos nesse alicerce.
>Se é que há os furos.
>
>
>
>




SUBJECT: D�vida de qu�mica: carbonyl = carbonila?
FROM: Phoenix Myth <phoenixclever@yahoo.com.br>
TO: ciencialist@yahoogrupos.com.br
DATE: 24/01/2005 00:03

Ola pessoal,

Esta i uma dzvida que os qummicos podem me
responder... O ligante monodentado (CO) de um complexo
organometalico recebe o nome em ingljs de carbonyl. A
tradugco correta para o portugujs i carbonila ou
carbonilo?

Se alguem puder me mandar uma resposta com as
resferjncias seria stimo.

Obrigado,




__________________________________
Do you Yahoo!?
All your favorites on one personal page  Try My Yahoo!
http://my.yahoo.com


SUBJECT: Re: [ciencialist] Dúvida de química: carbonyl = carbonila?
FROM: "Rodrigo Toledo" <rodrigotoledo11@uol.com.br>
TO: <ciencialist@yahoogrupos.com.br>
DATE: 24/01/2005 00:40

carbonila.

RT
----- Original Message -----
From: "Phoenix Myth" <phoenixclever@yahoo.com.br>
To: <ciencialist@yahoogrupos.com.br>
Sent: Monday, January 24, 2005 12:03 AM
Subject: [ciencialist] Dúvida de química: carbonyl = carbonila?



Olá pessoal,

Esta é uma dúvida que os químicos podem me
responder... O ligante monodentado (CO) de um complexo
organometálico recebe o nome em inglês de carbonyl. A
tradução correta para o português é carbonila ou
carbonilo?

Se alguem puder me mandar uma resposta com as
resferências seria ótimo.

Obrigado,




__________________________________
Do you Yahoo!?
All your favorites on one personal page - Try My Yahoo!
http://my.yahoo.com


##### ##### #####

Para saber mais visite
http://www.ciencialist.hpg.ig.com.br


##### ##### ##### #####
Links do Yahoo! Grupos












SUBJECT: Re: [ciencialist] orguloso do saber?
FROM: "Alvaro Augusto \(E\)" <alvaro@electraenergy.com.br>
TO: <ciencialist@yahoogrupos.com.br>
DATE: 24/01/2005 00:50

Caro Ray,

Esse distanciamento não é só culpa dos cientistas. O problema é que a ciência ficou mais complicada. Na época em que só existia a mecânica newtoniana, qualquer estudante de segundo grau podia dominar bastante do assunto, desde que se esforçasse. A relatividade especial tornou as coisas mais complicadas, mas não muito. A relatividade geral , contudo, exige muito mais do leitor, por causa do cálculo tensorial. Ainda assim, um estudante de graduação que seja razoavelmente versado em cálculo e mecânica newtoniana pode vir a entender bastante do assunto. Mas relatividade geral é coisa da década de 1910. Depois disso, especialmente com a criação da teoria quântica dos campos, a coisa se complicou muitíssimo. Não vejo, por exemplo, como um estudante médio de graduação possa vir a entender uma coisa como teoria das supercordas, pelo menos no estágio atual de desenvolvimento da teoria.

De qualquer forma, existe atualmente uma quantidade de livros de divulgação muito maior do que a que existiu em qualquer era anterior. Para quem lê inglês, uma quantidade enorme e crescente de material está disponível na internet, por exemplo, na Wikipedia: http://www.wikipedia.org

Quanto ao perigo do castelo cair, ele existe. Alguns até torçem por isso!

[ ]s

Alvaro Augusto

----- Original Message -----
From: rayfisica
To: ciencialist@yahoogrupos.com.br
Sent: Sunday, January 23, 2005 7:11 PM
Subject: [ciencialist] orguloso do saber?



Orgulhoso do próprio saber!
Não é uma agressão e nem direcionado a ser humano algum,
estou apenas
colhendo informações para aumentar o meu próprio saber.
Os cientistas de hoje tem razão de ficarem orgulhosos do saber que
possuem ou que possam possuir?
Tomando por base as ultimas comunicações vejo a distancia entre os
cientistas e o vulgo, parece-me que é impossível à pessoa
comum
entender patavina do que falam os doutos.
Não seria transformar a ciência em religião como fez a
igreja que ate
mandou pessoas para a fogueira em nome da crença na criação
conforme
o descrito na bíblia e agora vem com uma nova historia como se
não
tivesse mudado de idéia.
Ou dentro do próprio seio da ciência onde pessoas eram
arrogantes por
possuir conhecimentos que em sua época era o ápice e que hoje
não
passa de falácias, se cometermos os mesmos erros não poderemos
passar
por tolos arrogantes no futuro.
As teorias que hoje são o nosso orgulho estão suficientemente
esclarecidas para que tenhamos a certeza que não cairão por
terra
como a éter universal, por exemplo?
Eu que sou muito mais ignorante do que até eu acredito, não
consegui
engolir o neutrino, pois eu penso que as demonstrações ditas
empíricas do neutrino não passa das mesmas ocorrências que
deram a
duvida que originou a invenção do neutrino, e o resultado não
foi de
modo algum satisfatório como por exemplo a queda de corpos com
massa
diferente realizado por Galileu.
E plagiando alguém aqui da lista quem investiria o seu dinheiro
num
processo que demonstra apenas um terço do esperado pela teoria,
que
não os mesmos crentes da igreja, que repito queimaram seus
irmãos,
pela sua certeza.
Conversando um dia com um professor sobre uma duvida, após algum
tempo ele virou para mim, talvez até para se livrar de mim e
disse,
eu já estou muito bitolado (eu particularmente não quero ser um
bitolado).
Com certeza não sou douto, porem com mais certeza não sou um
sacerdote bitolado.
Penso que há um enorme castelo construído sobre um alicerce
aparentemente frágil, fico preocupado se o alicerce cair o que
sustentara o castelo, e é lógico que quem habita o castelo esta
tranqüilo, tranqüilo.
Esse é apenas um exemplo com certeza os integrantes da lista com
mais
saber que eu serão capaz de encontrar outros furos nesse alicerce.
Se é que há os furos.





[As partes desta mensagem que não continham texto foram removidas]



SUBJECT: Re: orguloso do saber?
FROM: Manuel Bulcão <manuelbulcao@uol.com.br>
TO: ciencialist@yahoogrupos.com.br
DATE: 24/01/2005 02:27


--- Em ciencialist@yahoogrupos.com.br, "murilo filo"
<avalanchedrive@h...> escreveu
> Quanto mais se sobe na montanha, mais se avistam terras estranhas
por conhecer... Falando-se em conhecimento, de repente fiquei
preocupado com a qualidade do inglês falado pelo comandante
companheiro do aerolula...

Manuel: Conheço várias pessoas capazes de falar abobrinhas
FLUENTEMENTE em vários idiomas, principalmente em inglês. Não
conseguem falar nada além de abobrinhas porque, afinal, aprender uma
língua e falá-la FLUENTEMENTE exige muita concentração e tempo. Para
tanto, se não for um desses que vivem meio tempo cá e meio tempo no
exterior (filho de diplomata, rebento de burguês, etc.), terão que
secrificar o tempo necessário para aprender várias outras coisas,
como filosofia, história, etc.

Conheci um que era craque em alemão, tão craque que quase não tinha
sotaque. Para tanto, consumiu uma década do seu tempo de vida
estudando esta língua. Entretanto, até então não tinha lido nada de
Goethe, Thomas Mann ou Günter Grass. Nem nunca tinha ouvido falar em
Hegel, Scheling, Heidegger ou sobre a Escola de Frankfurt. Em suma,
o carinha não passava de um oligofrênico poliglota (além do alemão e
do português, também arranhava o inglês, o espanhol e o francês).

[]s
Manuel Bulcão







SUBJECT: Fw: pesquisa sobre eletroquimica
FROM: "Luiz Ferraz Netto" <leobarretos@uol.com.br>
TO: "ciencialist" <ciencialist@yahoogrupos.com.br>
DATE: 24/01/2005 04:51

Químicos podem responder (direto e conciso)?
aquele abraço,
===========================
Luiz Ferraz Netto [Léo]
leobarretos@uol.com.br
http://www.feiradeciencias.com.br
===========================
-----Mensagem Original-----
De: bucapeta
Para: leobarretos
Enviada em: domingo, 23 de janeiro de 2005 23:27
Assunto: pesquisa sobre eletroquimica


Leo, gostaria de saber como a pressão, a concentração mol/L , temperatura e pH influência numa reação de oxi-redução para o aumento da diferença de potencial? Tem como criar outra tabela? Obrigado pela atenção! espero contato!


--------------------------------------------------------------------------------


Internal Virus Database is out-of-date.
Checked by AVG Anti-Virus.
Version: 7.0.300 / Virus Database: 265.6.13 - Release Date: 16/01/2005

----------

Internal Virus Database is out-of-date.
Checked by AVG Anti-Virus.
Version: 7.0.300 / Virus Database: 265.6.13 - Release Date: 16/01/2005


[As partes desta mensagem que não continham texto foram removidas]



SUBJECT: Re: [ciencialist] orguloso do saber?
FROM: "Luiz Ferraz Netto" <leobarretos@uol.com.br>
TO: <ciencialist@yahoogrupos.com.br>
DATE: 24/01/2005 11:02

De: "murilo filo"

> Quanto mais se sobe na montanha, mais se avistam terras estranhas por
conhecer...

Não gostei dessa citação acima porque não encerra qqer "medida" do conhecimento já existente ou dos novos 'por conhecer'.

Prefiro (no plano): Quanto mais se avança no 'raio' de seu círculo do saber, a área desconhecida avança com o quadrado desse raio.

Prefiro (no espaço): Quanto mais se avança no 'raio' de sua esfera do saber, o volume desconhecido avança com o cubo desse raio.

Prefiro (no buteco): 'Raios'! Essa eu não sabia ... aprendi mais uma! Salta outra dose.

[]'
Léo




--
Internal Virus Database is out-of-date.
Checked by AVG Anti-Virus.
Version: 7.0.300 / Virus Database: 265.6.13 - Release Date: 16/01/2005



SUBJECT: Re: [ciencialist] Re: Essas, pois, são as inquietudes tolas de mais um bobo.(2)
FROM: "Luiz Ferraz Netto" <leobarretos@uol.com.br>
TO: <ciencialist@yahoogrupos.com.br>
DATE: 24/01/2005 11:10

De: "Manuel Bulcão"
>>>> Salve Victor e demais!
VICTOR: Grande Manoel,

MANUEL, RETRIBUINDO A GENTILEZA:Grande? Talvez, mas certamente um
liliputiano frente à Vossa Enormidade!:-) <<<<

Isso é notável aqui no Clist. Prá que jogar confetes se podemos jogar discos de concreto ou tampas de bueiro!

De verdade em verdade vos digo: continuem assim.

[]'
deus




--
Internal Virus Database is out-of-date.
Checked by AVG Anti-Virus.
Version: 7.0.300 / Virus Database: 265.6.13 - Release Date: 16/01/2005



SUBJECT: Re: [ciencialist] Mulheres: Cozinha e na cama, já!!!!!!!!!!
FROM: ÿffffcdtalo Rocha <imrochaguedes@yahoo.com.br>
TO: ciencialist@yahoogrupos.com.br
DATE: 24/01/2005 15:03

Creio que este é um fórum de discussão de temas científicos,
principalmente, e acho que seria saudável se utilizássemos algo
chamado ceticismo na consideração de qualquer assunto. O cientista,
como ser humano, não é nem pode ser um autômato coletor de dados e
tirador de conclusões. Temos nossas crenças pessoais, sem dúvida, mas
a atitude científica perante o mundo preconiza (ou deveria preconizar)
uma certa cessação de nossos vieses ideológicos quando tratamos de
assuntos científicos. Não há dúvida que somos herdeiros de milênios de
cultura machista e opressora da mulher e negadora de suas capacidades
intelectuais, entretanto pergunto-me e pergunto-lhes, correndo o risco
de ser chamado de chauvinista, se seríamos capazes de por de lado
nossas crenças pessoais se estudos comprovadamente sérios (e não sei
se este é o caso) mostrassem inequivocamente uma menor capacidade feminina
para as ciências e para a matemática? Será que uma celeuma
semelhante seria causada se dissessem que os homens anglo-saxões protestantes é que têm uma tal
incapacidade? Não creio que se chegue a uma razoável apreciação de
assuntos como esse com acusações fáceis de machista, feminista,
chauvinista ou o que seja na velha tradição inquisitorial que parece
que herdamos dos ibéricos, gritando e queimando bruxas ao invés de nos
debruçarmos sobre as fontes primárias, os artigos científicos que apresentaram tal e tal proposição. É óbvio para qualquer um com um mínimo conhecimento de biologia que tais tendências intelectuais não são fruto apenas da genética mas tambbém do meio (cultural, econômico, ideológico) altamente variável. Por outro lado, a partir de afirmações como as que foram feitas aqui e em outros foruns supostamente devotados à discussão de temas científicos, tenho visto tanta intolerância e dogmatismo e aparente desconhecimento do método científico (não por parte de todos, claro) como em sites de fundamentalismo religioso. Às pessoas que se dispõem a discutir Ciência o reconhecimento que as crenças pessoais devem se adequar aos dados científicos concretos, sérios, exaustivamente comprovados e não o contrário devia ser patente.
Cordialmente, Ítalo Moraes Rocha Guedes.

__________________________________________________
Converse com seus amigos em tempo real com o Yahoo! Messenger
http://br.download.yahoo.com/messenger/

[As partes desta mensagem que não continham texto foram removidas]



SUBJECT: Re: [ciencialist] orguloso do saber?
FROM: "Oraculo" <oraculo@atibaia.com.br>
TO: <ciencialist@yahoogrupos.com.br>
DATE: 24/01/2005 15:05

Olá rayfisica

Vou dar uns palpites em seu questionamento, porque me parece que alguns conceitos erroneos estão atrapalhando, ok?..:-)

"ray: Os cientistas de hoje tem razão de ficarem orgulhosos do saber que
possuem ou que possam possuir?"

Bem, defina orgulho.:-) Conceitos subjetivos como esse sempre provocam problemas. Não há nada de mal em ficar orgulhoso, digamos, de seu filho, mas ser orgulhoso tem conotação pejorativa. Assim, que uso do termo está dando na pergunta?

Se você fosse um pesquisador, cientista, e descobrisse uma droga eficaz e segura contra a malária, que evitaria a morte e o sofrimento de milhões pelo mundo, principalmente crianças, ficaria orgulhoso de seu trabalho? Seria um orgulho "ruim"?

Por outro lado, pessoas são seres humanos, e mesmo cientistas tem as caracteristicas, sejam virtudes ou defeitos, humanos. Alguns podem ser orgulhosos, no mau sentido, outros generosos, outros cruéis, outros solidários, etc. Nada disso tem ligação com o resultado de seu trabalho que é confiável, cientifico, ou não.

Assim, a resposta a sua pergunta, depende do sentido que pretende dar a ela..;-)

"ray: Tomando por base as ultimas comunicações vejo a distancia entre os
cientistas e o vulgo, parece-me que é impossível à pessoa
comum entender patavina do que falam os doutos.
Não seria transformar a ciência em religião..."

Primeiro, como bem detalhou o Alberto, a ciência se torna mais complexa e nada há que se possa fazer para evitar isso. E a solução não é diminuir o conhecimento cientifico, mas aumentar o nível de capacidade do cidadão comum. Não é preciso que cada um conheça fisica ou biologia a fundo, mas que aprenda a se utilizar do pensamento crítico e compreenda o método cientifico e sua função de rigor (aumentar a confiabilidade de um resultado). Isso seria suficiente para a maioria acompanhar sem problemas o avanço da ciência.

Em segundo, a ciência não corre o risco de virar religião por sua própria natureza, sujeita ao ajuste, a refutação e ao teste. Mostrar que um conhecimento cientifíco, por mais sólido e aceito que seja, precisa de ajustes ou revisão (e faze-lo de forma consistente e metódica) em geral dá ao autor, não uma fogueira, mas um premio Nobel..:-)

Claro que, sendo humanos, sempre haverá quem esteja procurando por uma religião, uma certeza, e acabe tomando a ciência como tal. Mas é um engano humano, não cientifico.

"ray: Ou dentro do próprio seio da ciência onde pessoas eram
arrogantes por possuir conhecimentos que em sua época era o ápice e que hoje
não passa de falácias, se cometermos os mesmos erros não poderemos
passar por tolos arrogantes no futuro."

Pessoas são arrogantes. Sejam cientistas, sejam bispos católicos, sejam aiatolas ou ateus. E pessoas são não-arrogantes, sejam o que forem. Não é algo que dependa da ciência ou de seu conhecimento, mas das pessoas que os portam.

Outro engano comum, o conhecimento científico, mesmo o antigo e o que foi revisto, não são falácias, termo que tem significado preciso, mas simplesmente um conhecimento possível com dados existentes. Todo cientista sabe que seu trabalho pode ser contestado a qualquer momento. Mas, quanto mais ele é contestado e mais ele resite, maior a chance de ser correto ou de estar mas proximo da realidade.

A Terra continuará redonda, achatada nos polos, não importa quanto tempo passe e concluir isso hoje não é arriscar a ser tolo e arrogante no futuro..:-) Mas, no tempo antigo, quando o que se podia analisar para decidir sobre a forma da Terra era o que se via e sentia, concluir que ela era chata estava correto. Era chata para todos os efeitos práticos e testáveis da época. Não era uma falácia, e não eram tolos os que concluiram assim, nem estúpidos. Tiraram a melhor conclusão possível com os dados disponíveis para eles..;-)

"ray: As teorias que hoje são o nosso orgulho estão suficientemente
esclarecidas para que tenhamos a certeza que não cairão por
terra como a éter universal, por exemplo?"

Idem ao parágrafo acima..:-)

"ray: Eu que sou muito mais ignorante do que até eu acredito, não
consegui engolir o neutrino, pois eu penso que as demonstrações ditas
empíricas do neutrino não passa das mesmas ocorrências que
deram a duvida que originou a invenção do neutrino, e o resultado não
foi de modo algum satisfatório como por exemplo a queda de corpos com
massa diferente realizado por Galileu.
E plagiando alguém aqui da lista quem investiria o seu dinheiro
num processo que demonstra apenas um terço do esperado pela teoria,
que não os mesmos crentes da igreja, que repito queimaram seus irmãos,
pela sua certeza."

risos... :-) Ninguém investiria. Nem os autores dos estudos e experimentos que descobriram o neutrino..:-) Não, ainda. E esse ainda é que está confundindo você. A ciência não prega verdades, mas apresenta conclusões. Que, claro, podem estar incorretas e devem ser exaustivamente testadas, até o limite. Apenas passando e resistindo a todas essas tentativas, é que será conhecimento científico.

Neutrinos são uma possibilidade, boa possibilidade, e explicam diversos aspectos. Engula você ou não, elas fazem isso..:-) E o plagiado sou eu, que sempre uso este exemplo..:-)

Mas, falta muito para qualquer pessoa colocar dinheiro nisso (embora seja melhor que colocar dinheiro em uma rede de comunicações baseada em telepatia ou poderes mediúnicos..:-) Nem os que estudam neutrinos fariam isso..;-)

"ray: Penso que há um enorme castelo construído sobre um alicerce
aparentemente frágil, fico preocupado se o alicerce cair o que
sustentara o castelo, e é lógico que quem habita o castelo esta
tranqüilo, tranqüilo."

Precisa detalhar melhor seus exemplos..:-) O que chama de castelo de cartas, expressão que indica fragilidade? O conjunto dos conhecimentos produzidos pelo método científico? As vacinas, que eliminaram doenças da face da Terra, as leis da gravitação e relatividade, quer permitem que satelites de comunicação façam seu celular funcionar, e colocar robos em Marte e sondas em Titã, os principios cientificos que permitem essa nossa conversa em uma rede de alcance mundial, etc?

Ou está se referindo a determinadas questões da fisica avançada, onde a teoria está no estado da arte e precisa de muito mais ainda para definir com clareza o que é real e o que não é (como novos aceleradores de particulas, e coisa assim)?

Se está falando disso, da fisica avançada, bem, boa parte dos físicos que estudam isso concorda com você..:-) Há muito por fazer e muitas perguntas sem resposta. Boa parte das teorias, como as supercordas, é um castelo de cartas (coisa que mesmo seus autores sabem e concordam..:-), e vai levar tempo para construir bases sólidas para ela.

Mas se fala do conjunto do conhecimento, está errado. E pode perceber isso simplesmentre olhando para os lados e comparando a confiabilidade do atual conhecimento com o antigo..:-)

Um abraço.

Homero





















----- Original Message -----
From: rayfisica
To: ciencialist@yahoogrupos.com.br
Sent: Sunday, January 23, 2005 7:11 PM
Subject: [ciencialist] orguloso do saber?



Orgulhoso do próprio saber!
Não é uma agressão e nem direcionado a ser humano algum,
estou apenas
colhendo informações para aumentar o meu próprio saber.
Os cientistas de hoje tem razão de ficarem orgulhosos do saber que
possuem ou que possam possuir?
Tomando por base as ultimas comunicações vejo a distancia entre os
cientistas e o vulgo, parece-me que é impossível à pessoa
comum
entender patavina do que falam os doutos.
Não seria transformar a ciência em religião como fez a
igreja que ate
mandou pessoas para a fogueira em nome da crença na criação
conforme
o descrito na bíblia e agora vem com uma nova historia como se
não
tivesse mudado de idéia.
Ou dentro do próprio seio da ciência onde pessoas eram
arrogantes por
possuir conhecimentos que em sua época era o ápice e que hoje
não
passa de falácias, se cometermos os mesmos erros não poderemos
passar
por tolos arrogantes no futuro.
As teorias que hoje são o nosso orgulho estão suficientemente
esclarecidas para que tenhamos a certeza que não cairão por
terra
como a éter universal, por exemplo?
Eu que sou muito mais ignorante do que até eu acredito, não
consegui
engolir o neutrino, pois eu penso que as demonstrações ditas
empíricas do neutrino não passa das mesmas ocorrências que
deram a
duvida que originou a invenção do neutrino, e o resultado não
foi de
modo algum satisfatório como por exemplo a queda de corpos com
massa
diferente realizado por Galileu.
E plagiando alguém aqui da lista quem investiria o seu dinheiro
num
processo que demonstra apenas um terço do esperado pela teoria,
que
não os mesmos crentes da igreja, que repito queimaram seus
irmãos,
pela sua certeza.
Conversando um dia com um professor sobre uma duvida, após algum
tempo ele virou para mim, talvez até para se livrar de mim e
disse,
eu já estou muito bitolado (eu particularmente não quero ser um
bitolado).
Com certeza não sou douto, porem com mais certeza não sou um
sacerdote bitolado.
Penso que há um enorme castelo construído sobre um alicerce
aparentemente frágil, fico preocupado se o alicerce cair o que
sustentara o castelo, e é lógico que quem habita o castelo esta
tranqüilo, tranqüilo.
Esse é apenas um exemplo com certeza os integrantes da lista com
mais
saber que eu serão capaz de encontrar outros furos nesse alicerce.
Se é que há os furos.






##### ##### #####

Para saber mais visite
http://www.ciencialist.hpg.ig.com.br


##### ##### ##### #####


Yahoo! Grupos, um serviço oferecido por:







------------------------------------------------------------------------------
Links do Yahoo! Grupos

a.. Para visitar o site do seu grupo na web, acesse:
http://br.groups.yahoo.com/group/ciencialist/

b.. Para sair deste grupo, envie um e-mail para:
ciencialist-unsubscribe@yahoogrupos.com.br

c.. O uso que você faz do Yahoo! Grupos está sujeito aos Termos do Serviço do Yahoo!.



[As partes desta mensagem que não continham texto foram removidas]



SUBJECT: Re: orguloso do saber?
FROM: "rayfisica" <rayfisica@yahoo.com.br>
TO: ciencialist@yahoogrupos.com.br
DATE: 24/01/2005 17:01


--- Em ciencialist@yahoogrupos.com.br, "Alvaro Augusto \(E\)"
<alvaro@e...> escreveu
> Caro Ray,
>
>
> Quanto ao perigo do castelo cair, ele existe. Alguns até
torçem por
isso!
>
> [ ]s
>
> Alvaro Augusto
>
>
Perdão mas, questionar, testar e tentar derrubar o pensamento
vigente
não é o que fizeram todos os grandes sábios da humanidade,
incluindo
ai Mister Eisten.







SUBJECT: Re: [ciencialist] Re: orguloso do saber?
FROM: "Oraculo" <oraculo@atibaia.com.br>
TO: <ciencialist@yahoogrupos.com.br>
DATE: 24/01/2005 17:04

Olá Ray

"ray: Perdão mas, questionar, testar e tentar derrubar o pensamento vigente não é o que fizeram todos os grandes sábios da humanidade, incluindo ai Mister Eisten."

Se Einstein, que redefiniu a física moderna, modificando todo conhecimento sobre ela, desbancando o grande Newton e revolucionando o conhecimento, não estava questionando, testando e derrubando o pensamento vigente, quem estaria?????

Ray, se der uma olhada nos premios Nobels, recentes ou mais antigos, e analisar o trabalho de quem os recebeu, boa parte do que verá será exatamente isso, questionamento, teste e derrubada de conhecimento vigente..:-)

E mesmo antes, Galileu questiona a posição da Terra no sistema solar e no universo, pensamento vigente, Darwin questiona a criação separada e independente de todos os seres vivos, pensamento vigente, Pasteur questiona fluidos maleficos, castigos divinos e demonios como causa das doenças, alegando que micro-organismos são responsáveis, questionando o pensamento vigente, Newton questiona a causa do movimento de planetas e de toda matéria que seria simples vontade da divindade, e estabelece leis físicas naturais para esse movimento, contra o pensamento vigente à época, e assim vai.

E Einstein, nem seria preciso responder, já que sua teoria da relatividade questiona até hoje o senso comum do leigo, o pensamento vigente, que tempo é absoluto..:-)

Ray, parece que há um engano em sua posição, que talvez derive de algum tipo de conceito sobre a ciência que não corresponde a realidade da mesma. Cientistas fazem de tudo para refutar um conhecimento vigente, o castelo de cartas. Alguns pelo prazer de derrubar, outros pelo reconhecimento público mas muitos porque sabem que, de toda forma, sempre ganharão, sempre aprenderão algo: se derrubar o castelo, novos dados surgirão e novo castelo, mais sólido, será erguido. Se não derrubar, o castelo de cartas ganhará mais cimento e concreto, se tornará mais forte, mais real e mais confiável.

Homero


----- Original Message -----
From: rayfisica
To: ciencialist@yahoogrupos.com.br
Sent: Monday, January 24, 2005 5:01 PM
Subject: [ciencialist] Re: orguloso do saber?



--- Em ciencialist@yahoogrupos.com.br, "Alvaro Augusto \(E\)"
<alvaro@e...> escreveu
> Caro Ray,
>
>
> Quanto ao perigo do castelo cair, ele existe. Alguns até
torçem por
isso!
>
> [ ]s
>
> Alvaro Augusto
>
>
Perdão mas, questionar, testar e tentar derrubar o pensamento
vigente
não é o que fizeram todos os grandes sábios da humanidade,
incluindo
ai Mister Eisten.







##### ##### #####

Para saber mais visite
http://www.ciencialist.hpg.ig.com.br


##### ##### ##### #####


Yahoo! Grupos, um serviço oferecido por:
PUBLICIDADE




------------------------------------------------------------------------------
Links do Yahoo! Grupos

a.. Para visitar o site do seu grupo na web, acesse:
http://br.groups.yahoo.com/group/ciencialist/

b.. Para sair deste grupo, envie um e-mail para:
ciencialist-unsubscribe@yahoogrupos.com.br

c.. O uso que você faz do Yahoo! Grupos está sujeito aos Termos do Serviço do Yahoo!.



[As partes desta mensagem que não continham texto foram removidas]



SUBJECT: Re: orguloso do saber?
FROM: "Sergio M. M. Taborda" <sergiotaborda@terra.com.br>
TO: ciencialist@yahoogrupos.com.br
DATE: 24/01/2005 17:36


--- Em ciencialist@yahoogrupos.com.br, "Oraculo" <oraculo@a...> escreveu


> "ray: Tomando por base as ultimas comunicações vejo a distancia
entre os
> cientistas e o vulgo, parece-me que é impossível à pessoa
> comum entender patavina do que falam os doutos.
> Não seria transformar a ciência em religião..."
>
> Primeiro, como bem detalhou o Alberto, a ciência se torna mais
complexa e nada há que se possa fazer para evitar isso.

Ha sim. A ciencia de hoje virou uma palaçada onde os palhaços se
disfarçam de matemática para que não vejamos suas caras tristes.
É o colosal uso da matemática que atrapalha a compreensão do leigo e
tolda a compreensão do especialista. O especialista não se sabe
expressar sem ser matemáticamente, e o leigo não entende a não ser que
tudo seja explicado so com palavras. Estes mundos antagonicos podem
ser aproximados se colocarmos a matemática e os termos tecnicos no seu
lugar dando um espaço diferente À divulgação cientifica cujo objectivo
deve ser elucidar o leigo e não lançar teorias escabrosas , mal
acabadas e não testadas para o confundir ainda mais.

Por outro lado, a publicação cientifica está minada. toda a gente sabe
isso. A questão é como retomar dai. Eu acho que se deveria fazer como
antigamente onde os estudiosos publicavam livros com suas descobertas.
Já que é ai que vemos o que vale a teoria com um todo. Um paper sobre
1/1000 da teoria não prova nada.

> E a solução não é diminuir o conhecimento cientifico, mas aumentar o
nível de capacidade do cidadão comum.

E diminuir a altivez do especialista.

Não é preciso que cada um conheça fisica ou biologia a fundo, mas que
aprenda a se utilizar do pensamento crítico e compreenda o método
cientifico e sua função de rigor (aumentar a confiabilidade de um
resultado).

Não é possivel saber o que é o metodo quando ninguem consegue chegar
numa definição. muito menos usar uma coisa que não se sabe definir.


>Isso seria suficiente para a maioria acompanhar sem problemas o
avanço da ciência.

Não. Não seria. Para acompanhar é preciso entender, e para entender é
peciso estar a par dos termos e preocupações dos especialistas.

> Em segundo, a ciência não corre o risco de virar religião por sua
própria natureza, sujeita ao ajuste, a refutação e ao teste.

O conhecimento não se pode tornar religião, mas pode tornar-se
idolatrado, tal como uma estrela de cinema. Acho que desse ponto de
vista o Ray tem razão. A ciencia está no limite da idolatria e isto é
insustentável. Idolatram-se coisas que nem sequer se entendem bem (tal
como na religião) apenas para manter a face e pertencer ao mainstream.
(= receber dinheiro por fazer ciencia)

Sergio Taborda





SUBJECT: Re: [ciencialist] Re: orguloso do saber?
FROM: "Oraculo" <oraculo@atibaia.com.br>
TO: <ciencialist@yahoogrupos.com.br>
DATE: 24/01/2005 17:47

Olá

Bem, todos tem direito a ter sua opinião, mesmo que equivocada..:-) Respeito a sua.

Mas continuo com a minha, o ray não tem razão. Está confundindo ciência com o uso que é feito dela por seres humanos.

E penso que conhecer o método não é tão dificil, nem penso que falte definição. Você não aceita e tem restrições, o que nào significa que esteja certo e pronto, e o resto do mundo errado. O método, qualquer que seja sua concepção, envolve rigor e confiabilidade de resultados. Se o leigo puder perceber isso e comparar com o conhecimento produzido sem ele (achismos e crendices, revelações ou pseudo-ciencias como a astrologia), poderá compreender melhor porque aceitamos determinadas alegações e não outras, mesmo que nào conheçamos a fundo a matéria.

Pensamento crítico é mais interessante ainda, já que estimula a dúvida e o questionamento. Que podem ser aplicados na escolha do que levar em conta ao decidir. E é isso que precisa o leigo, e não compreender fisica quantica para decidir se a teoria das super cordas deve ou não ser pesquisada (quem tem de decidir e saber sobre isso são os cientistas envolvidos).

Você tem mágoa com a ciência, aquela que virou uma "palhaçada". Direito seu. Mas muitos cientistas, que também merecem respeito, pensam diferente. Fico com eles..:-) Escolha minha. Minha opinião.

O respeito que aos resultados que a ciência produziu (resultados, e não a própria ciência) é merecido. Melhorou a vida de muita gente e continua ajudando e sendo confiável. Já o conhecimento cientifico merece apenas ser analisado, sempre, em busca de mais confiabilidade. Não "respeito", que impeça de ser testado e refutado, se possível, mas análise crítica.

A crítica a politica dentro da ciência, a influencia de poderse e interesses, a defeitos humanos e sociais é perfeita e importante. Corrigir é imprescindível. Mas misturar essa necessidade com ataques de mágoa contra o conhecimento como um todo, não faz sentido.

Esta é minha opinião.

Homero





----- Original Message -----
From: Sergio M. M. Taborda
To: ciencialist@yahoogrupos.com.br
Sent: Monday, January 24, 2005 5:36 PM
Subject: [ciencialist] Re: orguloso do saber?



--- Em ciencialist@yahoogrupos.com.br, "Oraculo" <oraculo@a...> escreveu


> "ray: Tomando por base as ultimas comunicações vejo a distancia
entre os
> cientistas e o vulgo, parece-me que é impossível à pessoa
> comum entender patavina do que falam os doutos.
> Não seria transformar a ciência em religião..."
>
> Primeiro, como bem detalhou o Alberto, a ciência se torna mais
complexa e nada há que se possa fazer para evitar isso.

Ha sim. A ciencia de hoje virou uma palaçada onde os palhaços se
disfarçam de matemática para que não vejamos suas caras tristes.
É o colosal uso da matemática que atrapalha a compreensão do leigo e
tolda a compreensão do especialista. O especialista não se sabe
expressar sem ser matemáticamente, e o leigo não entende a não ser que
tudo seja explicado so com palavras. Estes mundos antagonicos podem
ser aproximados se colocarmos a matemática e os termos tecnicos no seu
lugar dando um espaço diferente À divulgação cientifica cujo objectivo
deve ser elucidar o leigo e não lançar teorias escabrosas , mal
acabadas e não testadas para o confundir ainda mais.

Por outro lado, a publicação cientifica está minada. toda a gente sabe
isso. A questão é como retomar dai. Eu acho que se deveria fazer como
antigamente onde os estudiosos publicavam livros com suas descobertas.
Já que é ai que vemos o que vale a teoria com um todo. Um paper sobre
1/1000 da teoria não prova nada.

> E a solução não é diminuir o conhecimento cientifico, mas aumentar o
nível de capacidade do cidadão comum.

E diminuir a altivez do especialista.

Não é preciso que cada um conheça fisica ou biologia a fundo, mas que
aprenda a se utilizar do pensamento crítico e compreenda o método
cientifico e sua função de rigor (aumentar a confiabilidade de um
resultado).

Não é possivel saber o que é o metodo quando ninguem consegue chegar
numa definição. muito menos usar uma coisa que não se sabe definir.


>Isso seria suficiente para a maioria acompanhar sem problemas o
avanço da ciência.

Não. Não seria. Para acompanhar é preciso entender, e para entender é
peciso estar a par dos termos e preocupações dos especialistas.

> Em segundo, a ciência não corre o risco de virar religião por sua
própria natureza, sujeita ao ajuste, a refutação e ao teste.

O conhecimento não se pode tornar religião, mas pode tornar-se
idolatrado, tal como uma estrela de cinema. Acho que desse ponto de
vista o Ray tem razão. A ciencia está no limite da idolatria e isto é
insustentável. Idolatram-se coisas que nem sequer se entendem bem (tal
como na religião) apenas para manter a face e pertencer ao mainstream.
(= receber dinheiro por fazer ciencia)

Sergio Taborda





##### ##### #####

Para saber mais visite
http://www.ciencialist.hpg.ig.com.br


##### ##### ##### #####


Yahoo! Grupos, um serviço oferecido por:







------------------------------------------------------------------------------
Links do Yahoo! Grupos

a.. Para visitar o site do seu grupo na web, acesse:
http://br.groups.yahoo.com/group/ciencialist/

b.. Para sair deste grupo, envie um e-mail para:
ciencialist-unsubscribe@yahoogrupos.com.br

c.. O uso que você faz do Yahoo! Grupos está sujeito aos Termos do Serviço do Yahoo!.



[As partes desta mensagem que não continham texto foram removidas]



SUBJECT: Re: orguloso do saber?
FROM: "rayfisica" <rayfisica@yahoo.com.br>
TO: ciencialist@yahoogrupos.com.br
DATE: 24/01/2005 17:53


--- Em ciencialist@yahoogrupos.com.br, "Oraculo" <oraculo@a...>
escreveu
> Olá rayfisica
>
> Vou dar uns palpites em seu questionamento, porque me parece que
alguns conceitos erroneos estão atrapalhando, ok?..:-)
>
>

Agradeço sua resposta que como sempre demonstra sabedoria e sabia
prudência.

É lógico que até eu com minhas limitações sei reconhecer
o bom da
ciência principalmente quando faço uso do, por exemplo,
computador.
Porem com certeza o senhor concordara comigo que algumas pessoas da
ciência agem exatamente como senhores absolutos da verdade
absoluta,
ou não? É a esses que questiono.
E todos nós sabemos que grande parte das conquistas da ciência
ocorreu por motivos outros que não os 99,99% de esforço, não?
Minha critica é dirigida para aqueles que por galgar um degrau a
mais
na subida da montanha que amplia os horizontes parecem ignorar
exatamente que os horizontes ampliam se, critico os que por estar do
lado da aparente verdade tratam os que estão do lado da aparente
mentira como loucos dementes, dignos da fogueira, exatamente como
fizeram os membros da instituição já referida anteriormente.
Os tolos, senhor, não seguem os aparentes tolos, mas sim os
aparentes
sábios ainda que tolos, o problema é que o aparente sábio
por ser
seguido por tolos pensa que realmente é sábio, ai já viu
né?
E estar do lado da maioria da um status.
E com certeza o senhor sabe que para produzir algo em ciência
temos
que ser como disse Albert Eisten, MONO-MANIACO e isso limita muito
nossa capacidade de julgamento em outras áreas ou não?
Critico veementemente o deus senhor de todo conhecimento que provoca
muito sofrimento no pobre coitado do gênio, como fizeram outrora
os
pais destes aos verdadeiros "profetas", hahaha, Galileu,
Darwin etc.
Quanto ao neutrino, claro que não importa a minha crença, pois
sei
que nada sou porem gostaria de lembrar o quanto foi gasto com sua
procura,em dinheiro e tempo e se ele não existir ... quanto as
respostas que ele dá, será que realmente mudara alguma coisa na
nossa
vida real, eu afirmo com certeza que nada mudara, porem os criadores
do neutrino falam como se o mundo fosse acabar, parecem não
entender
que o que acabara é o castelo.
Já disse alguém que quem tem teto de vidro não joga pedra no
telhado
alheio, só por causa do desnível.
Peço escusas por minhas limitações e por favor atenham se ao
espírito
de minha comunicação e não as perolas






SUBJECT: Re: [ciencialist] Re: orguloso do saber?
FROM: "Oraculo" <oraculo@atibaia.com.br>
TO: <ciencialist@yahoogrupos.com.br>
DATE: 24/01/2005 17:56

Olá ray

"ray: Porem com certeza o senhor concordara comigo que algumas pessoas da
ciência agem exatamente como senhores absolutos da verdade absoluta, ou não? "

Concordo e direi mais: muitas pessoas, tanto fora como dentro da ciência fazem isso..:-) Dirigentes de futebol, bispos, religiosos em geral, navegadores, pilotos de aviào, ateus e evangelicos, etc, tomam suas proprias crenças e certezas como verdades absolutas. É um defeito humano.

Justamente por saber disso (ou melhor, por aprender a duras penas) que todo um processo de rigor e seleção foi criado para filtrar esses defeitos humanos, bem como engano e erros de avaliação.

Quando um cientista publica um estudo (ou se deseja publicar em uma revista cientifica) é preciso que submeta seus dados, procedimentos e resutlados a outros cientisatas, que devem conferir e se possivel repetir tudo, diversas vezes. Assim, uma eventual falha engano ou erro de avaliação, até devido a certeza humana na verdade absoluta deste estudo, pode ser filtrado e encontrado. Mas quando o resultado chega ao publico e aos resto de nós, ele foi duramente testado. Compare isso aos palpites e previsões de um astrologo na virada de cada ano..:-)

As vezes falha, claro, mas mais acerta que falha e é o melhor que temos..:-)

Pessoas pensam ter a verdade absoluta, um cientista que preza esse nome nunca. A dúvida é o fundamento do cientista e da ciência, não a certeza. Embora procure pela verdade, ele não espera que o que encontre seja absoluto, ok?

Critica-se pessoas, personalidades (muitos cientistas, como seres humanos, foram péssimos e mesmo arrogantes, mas seriam arrogantes mesmo que tivessem escolhido ser padres, motoristas de onibus ou jogadores de futebol..:-), nào a ciência ou seu produto como arrogantes ou outra crítica subjetiva.

Homero

----- Original Message -----
From: rayfisica
To: ciencialist@yahoogrupos.com.br
Sent: Monday, January 24, 2005 5:53 PM
Subject: [ciencialist] Re: orguloso do saber?



--- Em ciencialist@yahoogrupos.com.br, "Oraculo" <oraculo@a...>
escreveu
> Olá rayfisica
>
> Vou dar uns palpites em seu questionamento, porque me parece que
alguns conceitos erroneos estão atrapalhando, ok?..:-)
>
>

Agradeço sua resposta que como sempre demonstra sabedoria e sabia
prudência.

É lógico que até eu com minhas limitações sei reconhecer
o bom da
ciência principalmente quando faço uso do, por exemplo,
computador.
Porem com certeza o senhor concordara comigo que algumas pessoas da
ciência agem exatamente como senhores absolutos da verdade
absoluta,
ou não? É a esses que questiono.
E todos nós sabemos que grande parte das conquistas da ciência
ocorreu por motivos outros que não os 99,99% de esforço, não?
Minha critica é dirigida para aqueles que por galgar um degrau a
mais
na subida da montanha que amplia os horizontes parecem ignorar
exatamente que os horizontes ampliam se, critico os que por estar do
lado da aparente verdade tratam os que estão do lado da aparente
mentira como loucos dementes, dignos da fogueira, exatamente como
fizeram os membros da instituição já referida anteriormente.
Os tolos, senhor, não seguem os aparentes tolos, mas sim os
aparentes
sábios ainda que tolos, o problema é que o aparente sábio
por ser
seguido por tolos pensa que realmente é sábio, ai já viu
né?
E estar do lado da maioria da um status.
E com certeza o senhor sabe que para produzir algo em ciência
temos
que ser como disse Albert Eisten, MONO-MANIACO e isso limita muito
nossa capacidade de julgamento em outras áreas ou não?
Critico veementemente o deus senhor de todo conhecimento que provoca
muito sofrimento no pobre coitado do gênio, como fizeram outrora
os
pais destes aos verdadeiros "profetas", hahaha, Galileu,
Darwin etc.
Quanto ao neutrino, claro que não importa a minha crença, pois
sei
que nada sou porem gostaria de lembrar o quanto foi gasto com sua
procura,em dinheiro e tempo e se ele não existir ... quanto as
respostas que ele dá, será que realmente mudara alguma coisa na
nossa
vida real, eu afirmo com certeza que nada mudara, porem os criadores
do neutrino falam como se o mundo fosse acabar, parecem não
entender
que o que acabara é o castelo.
Já disse alguém que quem tem teto de vidro não joga pedra no
telhado
alheio, só por causa do desnível.
Peço escusas por minhas limitações e por favor atenham se ao
espírito
de minha comunicação e não as perolas






##### ##### #####

Para saber mais visite
http://www.ciencialist.hpg.ig.com.br


##### ##### ##### #####


Yahoo! Grupos, um serviço oferecido por:







------------------------------------------------------------------------------
Links do Yahoo! Grupos

a.. Para visitar o site do seu grupo na web, acesse:
http://br.groups.yahoo.com/group/ciencialist/

b.. Para sair deste grupo, envie um e-mail para:
ciencialist-unsubscribe@yahoogrupos.com.br

c.. O uso que você faz do Yahoo! Grupos está sujeito aos Termos do Serviço do Yahoo!.



[As partes desta mensagem que não continham texto foram removidas]



SUBJECT: Re: [ciencialist] Re: orguloso do saber?
FROM: "Oraculo" <oraculo@atibaia.com.br>
TO: <ciencialist@yahoogrupos.com.br>
DATE: 24/01/2005 17:58

Olá ray

risos..:-) Está explicado..:-) Eu achei mesmo estranha a colocação, deveria ter pensado nisso, na falta do sinal de interrogação..:-)

Um abraço.

Homero

----- Original Message -----
From: rayfisica
To: ciencialist@yahoogrupos.com.br
Sent: Monday, January 24, 2005 6:01 PM
Subject: [ciencialist] Re: orguloso do saber?



--- Em ciencialist@yahoogrupos.com.br, "Oraculo" <oraculo@a...>
escreveu
> Olá Ray
>
> "ray: Perdão mas, questionar, testar e tentar derrubar o pensamento
vigente não é o que fizeram todos os grandes sábios da humanidade,
incluindo ai Mister Eisten."
>
> Se Einstein, que redefiniu a física moderna, modificando todo
conhecimento sobre ela, desbancando o grande Newton e revolucionando
o conhecimento, não estava questionando, testando e derrubando o
pensamento vigente, quem estaria?????
>


é que de vez em quando eu esqueço pra que serve o ???
desculpe.





##### ##### #####

Para saber mais visite
http://www.ciencialist.hpg.ig.com.br


##### ##### ##### #####


Yahoo! Grupos, um serviço oferecido por:
PUBLICIDADE




------------------------------------------------------------------------------
Links do Yahoo! Grupos

a.. Para visitar o site do seu grupo na web, acesse:
http://br.groups.yahoo.com/group/ciencialist/

b.. Para sair deste grupo, envie um e-mail para:
ciencialist-unsubscribe@yahoogrupos.com.br

c.. O uso que você faz do Yahoo! Grupos está sujeito aos Termos do Serviço do Yahoo!.



[As partes desta mensagem que não continham texto foram removidas]



SUBJECT: Re: orguloso do saber?
FROM: "rayfisica" <rayfisica@yahoo.com.br>
TO: ciencialist@yahoogrupos.com.br
DATE: 24/01/2005 18:01


--- Em ciencialist@yahoogrupos.com.br, "Oraculo" <oraculo@a...>
escreveu
> Olá Ray
>
> "ray: Perdão mas, questionar, testar e tentar derrubar o pensamento
vigente não é o que fizeram todos os grandes sábios da humanidade,
incluindo ai Mister Eisten."
>
> Se Einstein, que redefiniu a física moderna, modificando todo
conhecimento sobre ela, desbancando o grande Newton e revolucionando
o conhecimento, não estava questionando, testando e derrubando o
pensamento vigente, quem estaria?????
>


é que de vez em quando eu esqueço pra que serve o ???
desculpe.





SUBJECT: Re: [ciencialist] Re: orguloso do saber?
FROM: "Oraculo" <oraculo@atibaia.com.br>
TO: <ciencialist@yahoogrupos.com.br>
DATE: 24/01/2005 18:03

Olá ray

"ray:Quanto ao neutrino, claro que não importa a minha crença, pois
sei que nada sou porem gostaria de lembrar o quanto foi gasto com sua
procura,em dinheiro e tempo e se ele não existir ... "

Se descobrirmos que ele não existe, todo esse dinehro terá sido muito bem gasto..:-) Erros ensinam mais que acertos. Dezenas de experimentos que confirmam um teoria não valem muito, já sabiamos disso. Um único que contrarie, ensinará muito mais.

Em ciência, e principalmente em fisica, determinar a existencia ou realidade de algo é bem dificil. E o avanço se faz eliminando hipoteses e teorias, possibilidades e alegações. Descobrir, de forma confiável, que não existem neutrinos (na forma proposta, claro, negativas genericas são impossíveis de serem provadas..:-), nos ensinará muito, reduzira o leque de investigaçòes, indicará novos caminhos, etc.

Um abraço.

Homero






----- Original Message -----
From: rayfisica
To: ciencialist@yahoogrupos.com.br
Sent: Monday, January 24, 2005 5:53 PM
Subject: [ciencialist] Re: orguloso do saber?



--- Em ciencialist@yahoogrupos.com.br, "Oraculo" <oraculo@a...>
escreveu
> Olá rayfisica
>
> Vou dar uns palpites em seu questionamento, porque me parece que
alguns conceitos erroneos estão atrapalhando, ok?..:-)
>
>

Agradeço sua resposta que como sempre demonstra sabedoria e sabia
prudência.

É lógico que até eu com minhas limitações sei reconhecer
o bom da
ciência principalmente quando faço uso do, por exemplo,
computador.
Porem com certeza o senhor concordara comigo que algumas pessoas da
ciência agem exatamente como senhores absolutos da verdade
absoluta,
ou não? É a esses que questiono.
E todos nós sabemos que grande parte das conquistas da ciência
ocorreu por motivos outros que não os 99,99% de esforço, não?
Minha critica é dirigida para aqueles que por galgar um degrau a
mais
na subida da montanha que amplia os horizontes parecem ignorar
exatamente que os horizontes ampliam se, critico os que por estar do
lado da aparente verdade tratam os que estão do lado da aparente
mentira como loucos dementes, dignos da fogueira, exatamente como
fizeram os membros da instituição já referida anteriormente.
Os tolos, senhor, não seguem os aparentes tolos, mas sim os
aparentes
sábios ainda que tolos, o problema é que o aparente sábio
por ser
seguido por tolos pensa que realmente é sábio, ai já viu
né?
E estar do lado da maioria da um status.
E com certeza o senhor sabe que para produzir algo em ciência
temos
que ser como disse Albert Eisten, MONO-MANIACO e isso limita muito
nossa capacidade de julgamento em outras áreas ou não?
Critico veementemente o deus senhor de todo conhecimento que provoca
muito sofrimento no pobre coitado do gênio, como fizeram outrora
os
pais destes aos verdadeiros "profetas", hahaha, Galileu,
Darwin etc.
Quanto ao neutrino, claro que não importa a minha crença, pois
sei
que nada sou porem gostaria de lembrar o quanto foi gasto com sua
procura,em dinheiro e tempo e se ele não existir ... quanto as
respostas que ele dá, será que realmente mudara alguma coisa na
nossa
vida real, eu afirmo com certeza que nada mudara, porem os criadores
do neutrino falam como se o mundo fosse acabar, parecem não
entender
que o que acabara é o castelo.
Já disse alguém que quem tem teto de vidro não joga pedra no
telhado
alheio, só por causa do desnível.
Peço escusas por minhas limitações e por favor atenham se ao
espírito
de minha comunicação e não as perolas






##### ##### #####

Para saber mais visite
http://www.ciencialist.hpg.ig.com.br


##### ##### ##### #####


Yahoo! Grupos, um serviço oferecido por:







------------------------------------------------------------------------------
Links do Yahoo! Grupos

a.. Para visitar o site do seu grupo na web, acesse:
http://br.groups.yahoo.com/group/ciencialist/

b.. Para sair deste grupo, envie um e-mail para:
ciencialist-unsubscribe@yahoogrupos.com.br

c.. O uso que você faz do Yahoo! Grupos está sujeito aos Termos do Serviço do Yahoo!.



[As partes desta mensagem que não continham texto foram removidas]



SUBJECT: D�vida de matem�tica.
FROM: "Luiz Ferraz Netto" <leobarretos@uol.com.br>
TO: "ciencialist" <ciencialist@yahoogrupos.com.br>
DATE: 24/01/2005 20:32

Querem me dar uma mãozinha nessa? Recebi de um consulente ... e hoje o número de mensagens extrapolou!

"Gostaria de pedir a sua ajuda para resolver uma equação que à algum tempo estou tentando e não obtenho sucesso.
A equação é a seguinte:
a * cos(teta) - b * raiz ( 1 - cos^2(teta) ) = c
onde a, b, c são constantes e a incógnita é "teta".

[]'

===========================
Luiz Ferraz Netto [Léo]
leobarretos@uol.com.br
http://www.feiradeciencias.com.br
===========================
----------

Internal Virus Database is out-of-date.
Checked by AVG Anti-Virus.
Version: 7.0.300 / Virus Database: 265.6.13 - Release Date: 16/01/2005


[As partes desta mensagem que não continham texto foram removidas]



SUBJECT: Fw: 'coloque aqui o seu tema'
FROM: "Luiz Ferraz Netto" <leobarretos@uol.com.br>
TO:
CC: "ciencialist" <ciencialist@yahoogrupos.com.br>
DATE: 24/01/2005 20:50

Minha dúvida é:
1- O resultado do ensaio de resistência mecânica veio 12,55 daN/cm , a quanto equivale esta força? Como posso explicar numa linguagem que qualquer pessoa compreenda?

Léo: Primeiro lembremos que 'da' é prefixo 'deca' referindo-se ao multiplo 10;
1 dam = 10 m (leia-se: 1 decâmetro corresponde a 10 metros).
Desse modo, 1 daN = 10 N (leia-se: 1 decanewton corresponde a 10 newtons).
Por outro lado, lembremos que existem várias unidades técnicas, diferentes daquelas exigidas pelo Sistema Internacional de Unidades (S.I.U.). Uma delas é o quilograma-força (kgf) que é 'praticamente' equivalente a 10 newtons ( 1 kgf ~= 10 N).
A unidade de força "newton" é 'pequena' e o 'povo' não tem noção prática dela, mas tem muita noção prática do "quilograma-força", apesar de não ter noção da sua existência. O 'povo' diz "vou comprar um quilo de batata" como se estivesse dizendo alguma coisa lógica e correta. O dito não tem qqer significado físico. Na 'batatolina', comprar "um quilo de batata" significaria comprar "1000 batatas", uma vez que 'quilo' (k) é apenas um prefixo grego que significa 1000. O correto seria dizer "vou comprar um quilograma de batatas". Todavia, ainda há um erro nessa idéia, uma vez que a idéia do 'povo' , ao comprar batatas, é sobre "peso" e não sobre "massa". Desse modo, o 'mais correto' seria dizer "vou comprar um quilograma-força de batatas (1 kgf)". Agora sim, chegamos ao nervo da questão.
Como 1 kgf é a idéia prática mais popular (apesar de não saberem disso!), mas não é a oficial (do S.I.U.), pois a unidade oficial de força é o newton (símbolo N), então a 'jogada' é escrever "1 daN" em substituição ao "1 kgf".

Sua leitura no ensaio (12,55 daN/cm) também é 'um tanto fajuta' [ :-) ], pois mistura a parte oficial (N), com outra parte não oficial que é o centímetro (cm). A unidade deveria ser, pelo menos, daN/m (decanewton por metro). Outra coisa que não entendi (dessa sua leitura no ensaio) é a grandeza envolvida: força por unidade de comprimento? Teste de que tipo de resistência mecânica?

2- Como interpretar (entender) "Segundo ao quadrado"(s²)?

Léo: Segundo ao quadrado não é unidade de nenhuma grandeza física; essa notação não aparece isoladamente, aparece apenas em unidades derivadas, tais como a velocidade, aceleração, etc.
Normalmente, quando numa grandeza derivada aparece o tempo no denominador, a idéia é traduzir uma "rapidez", assim:
m/s --- é uma velocidade e traduz a 'rapidez' com que se vence uma certa distância;
m/s/s --- é uma aceleração e traduz a 'rapidez' com que a velocidade aumenta. Em lugar de m/s/s pode-se escrever m/s^2 ou ainda (mais 'oficialmente'), m.s^(-2); leia-se, metro vezes segundo elevado a menos dois.
Uma aceleração de 10 m/s^2 significa que a velocidade aumenta de 10 m/s a cada segundo que passa, ou seja, no primeiro segundo a velocidade é de 10 m/s, no segundo segundo é de 20 m/s, no terceiro segundo a velocidade é de 30 m/s ...

aquele abraço,
Léo


Léo, Bom dia!

Muito obrigada por ter respondido.
Sua explicação foi de grande valia, me ajudou muito.

Um abraço,
Silmara.

----------

Internal Virus Database is out-of-date.
Checked by AVG Anti-Virus.
Version: 7.0.300 / Virus Database: 265.6.13 - Release Date: 16/01/2005


[As partes desta mensagem que não continham texto foram removidas]



SUBJECT: Re: orguloso do saber?
FROM: "rayfisica" <rayfisica@yahoo.com.br>
TO: ciencialist@yahoogrupos.com.br
DATE: 24/01/2005 21:43


--- Em ciencialist@yahoogrupos.com.br, "Oraculo" <oraculo@a...>
escreveu
> Olá ray
>
> risos..:-) Está explicado..:-) Eu achei mesmo estranha a colocação,
deveria ter pensado nisso, na falta do sinal de interrogação..:-)
>
> Um abraço.
>
> Homero
>
Tudo bem senhor fico até feliz de produzir risos em alguém tão nobre,
sinto mesmo é fazê-lo perder o seu tempo com a falta de um ponto de
interrogação, alias houve um tempo que eles não existiam e nem por
isso as pessoas deixavam de se entender, lógico que com um pouco de
boa vontade e respeito, novamente desculpe me.





SUBJECT: Re: orguloso do saber?
FROM: "rayfisica" <rayfisica@yahoo.com.br>
TO: ciencialist@yahoogrupos.com.br
DATE: 24/01/2005 22:09


--- Em ciencialist@yahoogrupos.com.br, "Oraculo" <oraculo@a...>
escreveu
> Olá ray
>
> "ray:Quanto ao neutrino, claro que não importa a minha crença, pois
> sei que nada sou porem gostaria de lembrar o quanto foi gasto com
sua
> procura,em dinheiro e tempo e se ele não existir ... "
>
> Se descobrirmos que ele não existe, todo esse dinehro terá sido
muito bem gasto..:-) Erros ensinam mais que acertos. Dezenas de
experimentos que confirmam um teoria não valem muito, já sabiamos
disso. Um único que contrarie, ensinará muito mais.
>
> Em ciência, e principalmente em fisica, determinar a existencia ou
realidade de algo é bem dificil. E o avanço se faz eliminando
hipoteses e teorias, possibilidades e alegações. Descobrir, de forma
confiável, que não existem neutrinos (na forma proposta, claro,
negativas genericas são impossíveis de serem provadas..:-), nos
ensinará muito, reduzira o leque de investigaçòes, indicará novos
caminhos, etc.
>
> Um abraço.
>
> Homero
>
>
>
>
>Prometo que da minha parte é o ultimo comunicado a respeito do
assunto, pois que sinto que estou cansando.
Alguém da lista poderia encontrar uma forma de provar num
experimento, o que aconteceria num tanque com 600 toneladas de
tetracloroetileno onde os neutrinos não pudessem penetrar.
Não estaria faltando um experimento com a ausência do neutrino?
Pois com certeza sem o neutrino não haveria a transformação do cloro
37 em argônio 37, nem em um milhão de anos, e logicamente as
supernovas não colapsariam.
Qual é o papel do neutrino na natureza,???, não é a mais inútil das
partículas? (Apesar de ter gente que diz representar 90% do universo)
A essa afirmação peço um pouco de compreensão com o alcance mais
amplo do significado, pois que levaria muito tempo para demonstrar o
que estou tentando dizer.






SUBJECT: Re: [ciencialist] orguloso do saber?
FROM: "murilo filo" <avalanchedrive@hotmail.com>
TO: ciencialist@yahoogrupos.com.br
DATE: 24/01/2005 22:23

Raios!
Pois eu fico com a minha ( que não é minha ) definição.
Ela não quantifica, mas qualifica.
Parabens por ver vc atuando na esfera do saber! Achei 'pssionante!
Sái mais uma branca... zuzubeim... ;/

>From: "Luiz Ferraz Netto" <leobarretos@uol.com.br>
>Reply-To: ciencialist@yahoogrupos.com.br
>To: <ciencialist@yahoogrupos.com.br>
>Subject: Re: [ciencialist] orguloso do saber?
>Date: Mon, 24 Jan 2005 11:02:58 -0200
>
>De: "murilo filo"
>
> > Quanto mais se sobe na montanha, mais se avistam terras estranhas por
>conhecer...
>
>Não gostei dessa citação acima porque não encerra qqer "medida" do
>conhecimento já existente ou dos novos 'por conhecer'.
>
>Prefiro (no plano): Quanto mais se avança no 'raio' de seu círculo do
>saber, a área desconhecida avança com o quadrado desse raio.
>
>Prefiro (no espaço): Quanto mais se avança no 'raio' de sua esfera do
>saber, o volume desconhecido avança com o cubo desse raio.
>
>Prefiro (no buteco): 'Raios'! Essa eu não sabia ... aprendi mais uma! Salta
>outra dose.
>
>[]'
>Léo
>
>
>
>
>--
>Internal Virus Database is out-of-date.
>Checked by AVG Anti-Virus.
>Version: 7.0.300 / Virus Database: 265.6.13 - Release Date: 16/01/2005
>




SUBJECT: RE: [ciencialist] Re: orguloso do saber?
FROM: "murilo filo" <avalanchedrive@hotmail.com>
TO: ciencialist@yahoogrupos.com.br
DATE: 24/01/2005 22:49

Eu entendo do que vc está falando. Os energúmenos estão cada vez mais
disseminados!
Dá até nojo esta coisa de filho de burguês e de diplomata, não dá? É tudo
viado!
O problema é que o outro lado, ou outros lados, são só feras! Ou vc não sabe
disto?
Vai mandar gente bundinha p/falar com êles e sem entender as nuances da
língua em que estão tratando! Está achando que dá p/quebrar o galho com a
cintura brazuca?
As situações não são muito diferentes daquela de um piloto no exterior, pela
fonia, em aproximação e/ou em emergências, ou seja, SOB PRESSÃO!
O tempo vai dizer, e não será nada diferente daquilo que estamos vendo se
armar.
abr/M. SP 24/jan/2005

>From: Manuel Bulcão <manuelbulcao@uol.com.br>
>Reply-To: ciencialist@yahoogrupos.com.br
>To: ciencialist@yahoogrupos.com.br
>Subject: [ciencialist] Re: orguloso do saber?
>Date: Mon, 24 Jan 2005 04:27:00 -0000
>
>
>--- Em ciencialist@yahoogrupos.com.br, "murilo filo"
><avalanchedrive@h...> escreveu
> > Quanto mais se sobe na montanha, mais se avistam terras estranhas
>por conhecer... Falando-se em conhecimento, de repente fiquei
>preocupado com a qualidade do inglês falado pelo comandante
>companheiro do aerolula...
>
>Manuel: Conheço várias pessoas capazes de falar abobrinhas
>FLUENTEMENTE em vários idiomas, principalmente em inglês. Não
>conseguem falar nada além de abobrinhas porque, afinal, aprender uma
>língua e falá-la FLUENTEMENTE exige muita concentração e tempo. Para
>tanto, se não for um desses que vivem meio tempo cá e meio tempo no
>exterior (filho de diplomata, rebento de burguês, etc.), terão que
>secrificar o tempo necessário para aprender várias outras coisas,
>como filosofia, história, etc.
>
>Conheci um que era craque em alemão, tão craque que quase não tinha
>sotaque. Para tanto, consumiu uma década do seu tempo de vida
>estudando esta língua. Entretanto, até então não tinha lido nada de
>Goethe, Thomas Mann ou Günter Grass. Nem nunca tinha ouvido falar em
>Hegel, Scheling, Heidegger ou sobre a Escola de Frankfurt. Em suma,
>o carinha não passava de um oligofrênico poliglota (além do alemão e
>do português, também arranhava o inglês, o espanhol e o francês).
>
>[]s
>Manuel Bulcão
>
>
>
>
>




SUBJECT: Mensagem com o meu endereço
FROM: "Jose Colucci Jr. \(Home\)" <j.colucci@rcn.com>
TO: <ciencialist@yahoogrupos.com.br>
DATE: 25/01/2005 01:19

Esta lista (e várias outras) pode ter recebido uma ou mais mensagens
contendo vírus com o meu endereço de e-mail no remetente. Posso garantir
que não sairam do meu computador. Como tenho um único endereço de e-mail
há vários anos, uso todo o tipo de proteção contra vírus e hacking. A
rede da minha casa está protegida por um firewall e tenho um software
que monitora toda a atividade de e-mail, me alertando no caso de envio
não autorizado. A empresa onde trabalho conta com proteção semelhante,
ou melhor.

Parece ser o caso de um vírus tipo Klez, que envia e-mails com o
endereço falso a partir da lista de endereços do Outlook. Alguém tinha o
meu nome em sua lista. Não há muito que eu possa fazer a respeito, a não
ser alertar a todos contra o perigo de abrir "attachments" e aconselhar
o uso de antivirus e de toda a proteção possível. As dicas abaixo são do
Elton Carvalho, que também me alertou para o problema:


-------------------------------------------------------
O computador se torna bem mais vulnerável a esse tipo de ataque quando
se faz uso do software Microsoft Outlook, vítima predileta dos
criadores desse tipo de software mal intencionado.

Sugiro que seja feita uma vertificação contra vírus em seu computador,
com a atualização de seu software antivirus (normalmente feita de
maneira gratuita pela internet). Caso não possua um software
antivírus, sugiro que este seja instalado com a máxima urgência. Um
software antvírus gratuito e de boa qualidade pode ser encontrado em
http://free.grisoft.com/freeweb.php/doc/2/ .

É improtante também manter seu sistema atualizado, visitando
quinzenalmente o site www.windowsupdate.com e fazendo todas as
atualizações necessárias de segurança e mantendo sempre a veresão mais
recente dos softwares Microsoft Outlook e Microsoft Internet Explorer.

Entretanto, sugiro veementemente que se evite o uso do Microsoft
Outlook como gerenciador de e-mails, tendo em vista que a maioria dos
vírus e worms mais recentes o têm como seu alvo principal. Utilizar um
outro sotware para a mesma função, como o Mozilla Thunderbird
(http://www.mozilla.org/products/thunderbird/ ) é altamente
recomendado.

Para uma segurançamais completa e maior controle do seu trafego de
internet, sugiro também a substituição do browser Internet Explroer,
também vítima de diversos programas mal intencionados, pelo Mozilla
FireFox, bem mais seguro e com desenvolvimento mais rápido, além de
possuir dezenas de recursos adicionais. O FireFox pode ser encontrado
em (http://www.mozilla.org/products/firefox/ ) .




SUBJECT: Novo tópico, mesmo assunto?
FROM: "rayfisica" <rayfisica@yahoo.com.br>
TO: ciencialist@yahoogrupos.com.br
DATE: 25/01/2005 07:48


Gostaria da ajuda dos amigos da lista para resolver essa questão:

No dia 23 de Fevereiro de 1987, em um curto intervalo de poucos
minutos, o Kamiokande acusou a chegada de uma grande quantidade de
neutrinos. Para dizer a verdade, essa "grande quantidade" foi de
apenas 12 neutrinos. Mas, para quem está acostumado a contar menos
de
um neutrino por dia, isso é uma tremenda tempestadade. Logo se
descobriu que esses neutrinos vinham de uma supernova que explodira
na galáxia vizinha, a 170.000 anos-luz da Terra. Algumas horas
depois
dessa "chuva" de neutrinos atingir o Kamiokande, a luz da supernova
foi observada em um telescópio no Chile. Portanto, esses
neutrinos,
os primeiros neutrinos extra-galáticos a serem detetados, sairam
da
supernova antes da intensa luz da explosão ser jogada no espaço.
Diz se que a supernova esta a 170000 anos luz, como desde já a
algum
tempo sabemos que a terra não é um lugar privilegiado no
universo
esses neutrinos hipoteticamente vindos da estrela varreram o
espaço
em todas as direções à mesma quantidade média por unidade de
espaço.
Tomando por base o tanque de tetracloroetileno do professor Reymond
Davis Jr. que continha 600 tns. Dando a cada lt um kg o tanque teria
600m3, portanto cada face com aproximadamente 72m2 no caso de um cubo
o que é tomado apenas para facilitar o meu raciocínio.
12 neutrinos foram detectados numa área de aprox. 72m2 em poucos
minutos,quando o normal é menos que 1 neutrino por dia nessa mesma
área, para facilitar vamos tomar que os 12 neutrinos tenham
atingido
o equipamento em uma hora, na média a cada 5 minutos 1 neutrino
atingiu literalmente o detector, portanto 288 vezes mais que o normal.
Acontece que a estrela encontra se a 170 000 anos luz da terra, sendo
esse o raio da esfera hipotética varrida pelos neutrinos oriundos
da
estrelas cuja superfície é dada por (4piR2), acontece que
apenas um
terço dos neutrinos foram detectados, portanto no mínimo 36
passaram
pelos 72m2 do tanque ou seja ½ neutrino por m2, isso a 170000 anos
luz de distancia enquanto que o sol a 8,5 minutos-luz a média é
de
0.013888 neutrinos a cada 24 horas por m2, imagine na esfera a 8,5
min-luz de distancia dessa estrela a quantidade de neutrino por metro
quadrado de área, não seria o suficiente para originar uma
reação em
cadeia que consumiria toda a galáxia? (compreensão com minhas
limitações)
Pergunto ainda se num universo com bilhões de galáxias contendo
bilhões de estrelas tendo incontáveis estrelas colapsando
porque
apenas uma vez o detector coletou neutrinos estra-estelar aja visto a
improbabilidade destes neutrinos não chegarem a terra?
Não é um absurdo os números que encontraremos se realizarmos
alguns
cálculos, que eu confesso incapaz de realizar.
Não estaria o conhecimento cegando a inteligência, como disse o
poeta.
São apenas questões de um quase idiota, agradeço caso
alguém possa me
ajudar com respostas concretas e lógicas, afinal estou nessa lista
para aprender ciência e não religião, grato.






SUBJECT: Re: [ciencialist] Novo tópico, mesmo assunto?
FROM: "JVictor" <jvoneto@uol.com.br>
TO: <ciencialist@yahoogrupos.com.br>
DATE: 25/01/2005 07:48

Ray,

Abaixo um site(veja seus desdobramentos) que " fuxicam" sobre esse camarada misterioso e vero.

http://www.fisica.ufc.br/donafifi/neutrino/neutrino6.htm

Sds,

Victor

----- Original Message -----
From: rayfisica
To: ciencialist@yahoogrupos.com.br
Sent: Tuesday, January 25, 2005 7:48 AM
Subject: [ciencialist] Novo tópico, mesmo assunto?



Gostaria da ajuda dos amigos da lista para resolver essa questão:

No dia 23 de Fevereiro de 1987, em um curto intervalo de poucos
minutos, o Kamiokande acusou a chegada de uma grande quantidade de
neutrinos. Para dizer a verdade, essa "grande quantidade" foi de
apenas 12 neutrinos. Mas, para quem está acostumado a contar menos
de
um neutrino por dia, isso é uma tremenda tempestadade. Logo se
descobriu que esses neutrinos vinham de uma supernova que explodira
na galáxia vizinha, a 170.000 anos-luz da Terra. Algumas horas
depois
dessa "chuva" de neutrinos atingir o Kamiokande, a luz da supernova
foi observada em um telescópio no Chile. Portanto, esses
neutrinos,
os primeiros neutrinos extra-galáticos a serem detetados, sairam
da
supernova antes da intensa luz da explosão ser jogada no espaço.
Diz se que a supernova esta a 170000 anos luz, como desde já a
algum
tempo sabemos que a terra não é um lugar privilegiado no
universo
esses neutrinos hipoteticamente vindos da estrela varreram o
espaço
em todas as direções à mesma quantidade média por unidade de
espaço.
Tomando por base o tanque de tetracloroetileno do professor Reymond
Davis Jr. que continha 600 tns. Dando a cada lt um kg o tanque teria
600m3, portanto cada face com aproximadamente 72m2 no caso de um cubo
o que é tomado apenas para facilitar o meu raciocínio.
12 neutrinos foram detectados numa área de aprox. 72m2 em poucos
minutos,quando o normal é menos que 1 neutrino por dia nessa mesma
área, para facilitar vamos tomar que os 12 neutrinos tenham
atingido
o equipamento em uma hora, na média a cada 5 minutos 1 neutrino
atingiu literalmente o detector, portanto 288 vezes mais que o normal.
Acontece que a estrela encontra se a 170 000 anos luz da terra, sendo
esse o raio da esfera hipotética varrida pelos neutrinos oriundos
da
estrelas cuja superfície é dada por (4piR2), acontece que
apenas um
terço dos neutrinos foram detectados, portanto no mínimo 36
passaram
pelos 72m2 do tanque ou seja ½ neutrino por m2, isso a 170000 anos
luz de distancia enquanto que o sol a 8,5 minutos-luz a média é
de
0.013888 neutrinos a cada 24 horas por m2, imagine na esfera a 8,5
min-luz de distancia dessa estrela a quantidade de neutrino por metro
quadrado de área, não seria o suficiente para originar uma
reação em
cadeia que consumiria toda a galáxia? (compreensão com minhas
limitações)
Pergunto ainda se num universo com bilhões de galáxias contendo
bilhões de estrelas tendo incontáveis estrelas colapsando
porque
apenas uma vez o detector coletou neutrinos estra-estelar aja visto a
improbabilidade destes neutrinos não chegarem a terra?
Não é um absurdo os números que encontraremos se realizarmos
alguns
cálculos, que eu confesso incapaz de realizar.
Não estaria o conhecimento cegando a inteligência, como disse o
poeta.
São apenas questões de um quase idiota, agradeço caso
alguém possa me
ajudar com respostas concretas e lógicas, afinal estou nessa lista
para aprender ciência e não religião, grato.






##### ##### #####

Para saber mais visite
http://www.ciencialist.hpg.ig.com.br


##### ##### ##### #####


Yahoo! Grupos, um serviço oferecido por:
PUBLICIDADE




------------------------------------------------------------------------------
Links do Yahoo! Grupos

a.. Para visitar o site do seu grupo na web, acesse:
http://br.groups.yahoo.com/group/ciencialist/

b.. Para sair deste grupo, envie um e-mail para:
ciencialist-unsubscribe@yahoogrupos.com.br

c.. O uso que você faz do Yahoo! Grupos está sujeito aos Termos do Serviço do Yahoo!.



[As partes desta mensagem que não continham texto foram removidas]



SUBJECT: Re: Mulheres: Cozinha e na cama, já!!!!!!!!!!
FROM: "Rodrigo Marques" <rodmarq72@yahoo.com.br>
TO: ciencialist@yahoogrupos.com.br
DATE: 25/01/2005 08:03



--- Em ciencialist@yahoogrupos.com.br, "Alvaro Augusto \(E\)"
<alvaro@e...> escreveu

"E, embora os homens e mulheres de hoje em dia não sejam em nada
diferentes dos homens e mulhreres do neolítico, há uma pequena
diferença: tecnologia. Devido à tecnologia as mulheres podem, se
assim o quiserem, evitar ter um filho por ano durante a idade
fértil. Devido à tecnologia, as mulheres podem fazer tudo o que os
homens fazem, inclusive dirigir máquinas pesadas (direção
hidráulica!), pilotar aviões, dirigir empresas, lecionar, etc. As
mulheres podem, em resumo, colocar comida na mesa, tornando-se
caçadoras, e não mais agricultoras. "

Você esqueceu de dizer que grande parte dessa tecnologia (senão toda
ela) que facilita a vida das mulheres dando-lhes conforto e fazendo
com que elas possam competir em pé de igualdade com os homens foi
criada pelos próprios homens.
Veja o link:
http://inforum.insite.com.br/jroliveira/857780.html





SUBJECT: Re: [ciencialist] Re: orguloso do saber?
FROM: "Sergio M. M. Taborda" <sergiotaborda@terra.com.br>
TO: ciencialist@yahoogrupos.com.br
DATE: 25/01/2005 10:52

Oraculo wrote:

>Olá
>
>Bem, todos tem direito a ter sua opinião, mesmo que equivocada..:-) Respeito a sua.
>
>
>Mas continuo com a minha, o ray não tem razão. Está confundindo ciência com o uso que é feito dela por seres humanos.
>
>

Não existe essa destinção. A ciencia é aquilo que fazemos com ela, pois
ela não existe sem nós.

>E penso que conhecer o método não é tão dificil, nem penso que falte definição. Você não aceita e tem restrições, o que nào significa que esteja certo e pronto, e o resto do mundo errado. O método, qualquer que seja sua concepção, envolve rigor e confiabilidade de resultados.
>

O que adianta isso se lhe negarem todos os fundos para fazer a
experiencia ? Vc acha que as experiencias são baratas ? Em que mundo vc
vive ? No mundo real a ciencia paga-se , e custa dinheiro e só investe
nela quem tem interesse , e ninguem tem interesse em ser difamado por
patrocinar uma expriencia de alguem que é considerado um loco pelos seus
congeneres.

>Se o leigo puder perceber isso e comparar com o conhecimento produzido sem ele (achismos e crendices, revelações ou pseudo-ciencias como a astrologia), poderá compreender melhor porque aceitamos determinadas alegações e não outras, mesmo que nào conheçamos a fundo a matéria.
>
>
Estamos fartos de saber essa resa sr padre. O assunto aqui é o leigo não
consegue entender ciencia naquilo que ela diz. Por exemplo, o leigo não
consegue entender o que é o Big Bang , o que é a Teoria dos Grafos, o
que é a teoria da evolução das especies moderna, não sabe o que Marte
tem a ver com Jupiter . Não sabe o que é a Teora dos Muitos mundos, ou
se sequer se vale a pena sabê-la. Não sabe muitas vezes a Lei da
Acçao-Reacção, A Lei do Movimento Perpétuo, nem a Lei da Equivalencia.
Muito menos o primeiro postulado da teoria da relatividade, o que ele
tem a ver com efeito de Doppler, o que é o efeito de Doppler . Muitas
vezes o leigo nem consegue aplicar a 2ª Lei de Newton lei como deve ser
, quanto mais a 2ª lei da termodinamica, ou a 3ª. É claro que ele pode
ler, ele pode entender o método e não acreditar em supertições mais
isso não o fará entender fisica, nem nenhuma ciencia em particular. No
máximo terá uma boa ideia daquilo que sabemos, do ponto de vista
enciclopédico, mas dificilmente saberá encaixar as partes para formar
novos raciocinios que não tenha sido expostos, ou que sejam errados à
partida.
O pensamento critico, a liberdade de pensamento, a imparcialidade , as
regras , o metodo ... são tudo ferramentas que não funcionam sozinhas.

>Pensamento crítico é mais interessante ainda, já que estimula a dúvida e o questionamento. Que podem ser aplicados na escolha do que levar em conta ao decidir.
>
Ao dicidir o quê ? Se Eisntein plagiou Poincaré ? Se o Big Bang existiu
? Se existe vida noutros planetas ? Se devemos suportar o projecto SETI
? SE devemos comprar um livro do Hawkings ? Se devemos dar ouvidos a
quem nunca leu um paper verdadeiro ? Se os professores da faculdade
sabem mais que os alunos ? Se é preciso entrar numa faculdade para fazer
ciencia ? Se vale a pena vir colocar as duvidas na cienclist pois a
resposta pode ser mais idiota que a pergunta ? Onde pára o poder de
decisão que a ciencia oferece ?

> E é isso que precisa o leigo, e não compreender fisica quantica para decidir se a teoria das super cordas deve ou não ser pesquisada (quem tem de decidir e saber sobre isso são os cientistas envolvidos).
>
>
Ja foi tempo em que ele não queria entender essas coisas. A media faz
tanta pressão que o leigo sentese idiota por não entender essas teorias
avançadas e recorre às fontes da propria media para se informar. O
problema é que essas fontes são ainda mais leigas que ele , e ele acaba
recorrendo a listas com esta. O leigo precisa, primeiro de tudo
entender, que não ha mal no mundo se ele não entender fisica quantica ,
ou TRG.

>Você tem mágoa com a ciência, aquela que virou uma "palhaçada". Direito seu. Mas muitos cientistas, que também merecem respeito, pensam diferente. Fico com eles..:-) Escolha minha. Minha opinião.
>
>
Pena é que sua opinião não tenha nada a ver com o assunto. O assunto são
as pessoas, o cooperativismo cientifico, as facadas que cada cientista
tem que dar no outro se quiser publicar seu paper e receber fundos para
mais um ano de trabalho. A impossibilidade de trabalhar com teorias
alternativas so porque elas são alternativas. Em resumo, tudo o que
acontece no mundo real - que concerteza não acontece no seu mundo de
fantasia - e que nada tem a ver com metodo, teste, entendimento e sim
com inveja, cobiça e ego.
Pelas suas palavras deduzo que vc não deve trabalhar num centro onde se
faça ciencia, pois se trabalhasse teria sentido esses efeitos e pensaria
duas vezes antes de descrever o seu mundo perfeito.

>A crítica a politica dentro da ciência, a influencia de poderse e interesses, a defeitos humanos e sociais é perfeita e importante. Corrigir é imprescindível. Mas misturar essa necessidade com ataques de mágoa contra o conhecimento como um todo, não faz sentido.
>
>
Mas ninguem atacou o conhecimento como um todo. Vc deve ter entendido
algo alguma coisa fora do contexto que não estáva escrito. É exactamente
da "politica dentro da ciência, a influencia de poderse e interesses, a
defeitos humanos e sociais" que estamos falando. O que não faz sentido é
vc vir com essa conversa de metodo e teste e blablabla que não tem nada
a ver com o assunto.
A prespectiva do Ray é muito lucida e verdadeira, doa a quem doer.

Sérgio Taborda




--
No virus found in this outgoing message.
Checked by AVG Anti-Virus.
Version: 7.0.300 / Virus Database: 265.7.2 - Release Date: 21-01-2005



SUBJECT: Para quem quer se indignar!!!
FROM: ÿffffcdtalo Rocha <imrochaguedes@yahoo.com.br>
TO: Lista <ciencialist@yahoogrupos.com.br>
DATE: 25/01/2005 11:02

Saiu dia 24/01 no Jornal da Ciência da SBPC (Sociedade Brasileira para o Progresso da Ciência):
SBPC se solidariza com Lúcio Flávio Pinto, vítima de brutal agressão em Belém Mensagem do presidente da SBPC, Ennio Candotti, afirma que ‘a prepotência e a brutalidade são a marca dos tiranetes’ que Lúcia Flávio sempre combateu. Veja abaixo notícia do ‘Diário do Parᒠa respeito

Eis a íntegra da mensagem:

‘Meu caro Lúcio Flávio, conselheiro da SBPC, amigo,

Foi com grande indignação que recebi a noticia da covarde agressão que você sofreu. Das ameaças de morte, pusilânimes. A prepotência e a brutalidade são a marca dos tiranetes que você sempre combateu. Que nós combatemos. Solidários.

A SBPC está com você. Em terras amigas e também nas inimigas.

Belém, e não só Belém, vive um tempo em que o estado de violência ainda persiste, nos subterrâneos e fora deles. A punição tarda. A sociedade reclama.

Contamos com a informação, o conhecimento e a educação para derrubar a violência, prepotente e assassina, e promover o estado de justiça e liberdade. Em Belém e no Brasil.

Oferecemos nossas melhores forças para que você possa continuar a escrever, a denunciar as injustiças, a pensar. O seu exemplo agora é de todos. É dele que nos orgulhamos.

É isso que os seus agressores mais temem.

Com um forte abraço,
Ennio Candotti’

Leia, a seguir, a reportagem do ‘Diário do Pará’, publicada nesta segunda-feira a respeito da agressão sofrida por Lúcio Flávio Pinto:

Jornalista vai ao ministro da Justiça

Nome de executivo das ORM corre o país pela agressão e ameaça de morte a Lúcio Flávio Pinto

O jornalista Lúcio Flávio Pinto vai pedir uma audiência com o ministro da Justiça, Márcio Thomaz Bastos, para denunciar o espancamento de que foi vítima na última sexta-feira.

Ele foi agredido em público e ameaçado de morte pelo empresário Ronaldo Maiorana, diretor-corporativo das Organizações Romulo Maiorana e por quatro seguranças (sargentos da Polícia Militar), no restaurante do Parque da Residência.

O jornalista afirma que o inquérito policial sobre o caso já está instaurado e que vai entrar na Justiça por conta da ameaça de morte, além de preparar uma ação de indenização.

As Organizações Romulo Maiorana editam o jornal O Liberal e retransmitem a programação da TV Globo.

A agressão sofrida pelo jornalista Lúcio Flávio Pinto, na última sexta-feira, no restaurante do Parque da Residência, ainda repercute nos meios de comunicação de todo o Brasil.

Lúcio foi espancado brutalmente pelo empresário e proprietário das Organizações Romulo Maiorana e quatro policiais militares que faziam sua segurança particular.

Em entrevista ao jornalista Maurício Araújo, do site http://www.amazonia.org, Lúcio afirmou que vivia ‘uma sensação de impotência e tristeza. Pois eles são donos do Pará. Na minha terra sou submetido a uma lei de cangaço’.

Você pode contar como foi o conflito no restaurante?

LFP - Estávamos almoçando no restaurante do Parque da Residência, como fazemos habitualmente, eu e um grupo de amigos, umas oito pessoas, salvo engano. Passada meia hora no restaurante, chegou o Ronaldo com um amigo. Sentou numa mesa exatamente atrás de mim, de modo que ficou nas minhas costas - eu não podia vê-lo. Após um intervalo de tempo de cerca de vinte minutos senti um murro na face direita. Neste momento, ele já estava acompanhado de seus seguranças particulares, que são dois soldados da Polícia Militar, um deles conhecido como Saddam. Após o murro ele me deu uma gravata, arrebentando todos os botões da minha camisa. Na seqüência veio um empurrão, caí sobre cadeiras, indo parar no chão. Ronaldo gritava que ia me matar, senão me matasse ali naquele momento seria em outro lugar, mas que eu nunca mais iria escrever sobre a família dele! Após a tentativa de apartar a briga, o André (NR: o economista e comerciante André Carrapatoso Coelho, que acompanhava Lúcio no almoço) passou a
ser agredido por um dos seguranças também. Enquanto Ronaldo gritava, um dos seguranças me chutava também. Algumas pessoas tentavam segurá-lo. Finalmente eu consegui me levantar. Disse que não tinha sentido o que ele estava fazendo. Os próprios seguranças pararam de bater e foram segurá-lo. Em seguida retiraram-se do restaurante, que estava lotado, tinha aproximadamente 80, talvez 100 pessoas.

Você então se dirigiu à polícia?

LFP - Do restaurante mesmo eu chamei uma patrulha da PM. A patrulha chegou 20 minutos depois. Fomos pra Seccional Urbana de São Brás. Registramos a ocorrência, eu e o André, e depois fui ao IML fazer exame de corpo de delito, onde foram constatadas as seguintes agressões: hematomas no rosto, escoriações nos braços, nas pernas e nas costas.

Quais providências foram tomadas?

LFP - Já está em andamento o inquérito policial. Vou entrar na Justiça devido à ameaça de morte e entrar com uma ação de indenização. Ao mesmo tempo, pretendo responsabilizar a família Maiorana por qualquer coisa que venha a me acontecer. Pretendo escrever uma carta ao jornal O Estado de São Paulo onde trabalhei por 17 anos. Vou pedir uma audiência à OAB e vou pedir uma audiência com o ministro da justiça, pois no Pará não há garantia para o trabalho da imprensa.

Esta é a primeira vez que você é ameaçado de morte?

LFP - Não é a primeira vez que me ameaçam de morte. O marido da Rosângela (NR: Rosângela Maiorana Kzan, irmã mais velha de Ronaldo e diretora administrativa do grupo ORM) já me agrediu - e ele é faixa-preta de caratê - e ameaçou de morte. Na época, eu estava fazendo uma série de reportagens sobre as dissidências internas no grupo ORM. A Rosângela entrou com cinco ações sucessivas contra mim, com base na Lei de Imprensa, isto foi em setembro, outubro de 1992, há mais de dez anos atrás. Quatro ações penais e uma ação cível para proibir que eu falasse dela, ou seja, tentou estabelecer uma espécie de censura prévia. Detalhe: ela nunca mandou cartas ao meu jornal, nunca contestou as reportagens. Registrei queixa, foi instaurado o inquérito. E apesar de ele ter sido réu confesso, o Ministério Público não fez a denúncia e eu tive que entrar com ação privada. Mesmo assim, o processo não foi adiante e acabou sendo arquivado por prescrição.

Como você se sente? Acha possível que a ameaça seja cumprida?

LFP - Não sei, quanto a isso estou tomando as precauções legais, o que penso é que essa ameaça existe, de fato. E a simples ameaça mostra que eles se colocam acima do bem e do mal, para fazer isso - uma ameaça de morte - na frente de pessoas da sociedade, com dois seguranças armados que são da Polícia Militar ao lado! É como estar à mercê de um poder sem limites. Minha sensação é de impotência e tristeza. Pois eles se consideram donos do Pará. Na minha terra sou submetido a uma lei de cangaço. Ele não disse nada sobre o meu artigo, nem uma palavra. Bastou não gostar para julgar que tinha o direito de agredir e ameaçar de morte um jornalista.

Repúdio online

No blog do jornalista e colunista, Ricardo Noblat (noblat.blig.ig.com.br/) a informação sob o título ‘Diretor de jornal espanca jornalista em Belém’ vários comentários de internautas a respeito do ato classificado com selvageria.

Veja alguns:

- ‘Acho fundamental que este blig publique fatos escabrosos como este narrado pelo jornalista de Belém, vítima de agressor seguro da impunidade. Essas oligarquias regionais ignoram qualquer evolução possível em nossa (‘delas’) sociedade. Mais sintomático é que essa figura é membro do Partido Liberal... Seja no Pará ou qualquer outro Estado brasileiro, é preciso mostrar que os reis estão nus. Essa é a função do bom jornalismo. Esclarecer a sociedade. Obrigado’.

- ‘Este cidadão que agrediu o jornalista deveria estar é preso, pois lugar de animais é atrás das grades. Quem ameaça também tem medo, pois se não tivesse medo, não sairia por aí se garantindo nos seus cães de guarda. Animais iguais a ele. Ricardo Noblat, continue escrevendo o bom jornalismo, porque se o povo não estiver a par dos acontecimentos, estes canalhas estragam o Brasil’.

- ‘O jornalista, com o seu sacrifício, jogou a luz sobre mais um afiliado da rede Globo, que sempre está do lado do mal. Que coisa, já repararam que a Globo sempre está contra a corrente?’

- ‘Andei pelas bandas de Belém nos anos 70, pouco depois que o comerciante potiguar, radicado na cidade, Romulo Maiorana adquiriu o jornal O Liberal. Na época trabalhava nos Diários Associados e estava justamente fazendo uma matéria sobre o envolvimento do comerciante Romulo Maiorana com o contrabando, haja vista que seu crescimento econômico repentino induzia a essa premissa. Graças aos generais de plantão O Liberal cresceu e sufocou os demais jornais paraenses, inclusive A Província do Pará, os Diários Associados, Maiorana, a semelhança de ACM na Bahia, conseguiu transferir a programação da Rede Globo para a sua TV Liberal e a partir daí se transformou no imperador das comunicações do Pará. Paparicado pela elite paraense, Romulo sempre utilizou o expediente que seu filho consagrou agora publicamente com a agressão ao Lúcio Flávio. Só que Romulo pai o fazia sorrateiramente. Valeu a pena postar a notícia da agressão para que os blogueiros, do eixo Rio/São Paulo, sempre alheios ao que
acontece por esse Brasil a fora tenham conhecimento que o buraco para jornalistas de fora do eixo é mais embaixo’.

- ‘Espero que o jornalista continue o seu trabalho, com as forças redobradas! E que a ‘grande mídia’ preste mais atenção, pois este tipo de atitude, que é no fundo uma coerção, ocorrendo quase que diariamente sobre os jornalistas por este País a fora! La Passionaria em seu discurso de despedida da fantástica Brigada Internacional: ‘...Vocês podem sentir-se orgulhosos. Vocês são a História. São a lenda. São o exemplo heróico da solidariedade e da universalidade da democracia. Mais ainda, soldados do mais alto ideal de redenção humana...’

- ‘O Diretor das Organizações Romulo Maiorana, Ronaldo Maiorana que vem a ser o Presidente do Diretório Regional do PL, estava cercado de PM’s que fazem a sua segurança. Foi um ato covarde, está registrado em filme amador e quem quiser saber de maiores detalhes é só acessar a página do Diário do Pará (http://www.diariodopara.com.br). A família Maiorana tem história no Pará...’

- ‘Sou leitor do jornal do Lúcio e acredito que seja a fonte mais fidedigna de informações que conheço em meu Estado. Lamento que quem fala a verdade mereça castigo.’


__________________________________________________
Converse com seus amigos em tempo real com o Yahoo! Messenger
http://br.download.yahoo.com/messenger/

[As partes desta mensagem que não continham texto foram removidas]



SUBJECT: Re: [ciencialist] Re: orguloso do saber?
FROM: ÿffffcdtalo Rocha <imrochaguedes@yahoo.com.br>
TO: ciencialist@yahoogrupos.com.br
DATE: 25/01/2005 11:40

E o que é que sugere o senhor Sergio "todo indignação" Taborda? Sim, porque o que entendi foi a mesma coisa que o outro senhor a quem vc respondeu, de forma tão estúpida, entendeu. Tenho minhas dúvidas se o leigo realmente faz um esforço tão grande em entender a ciência, e já que não entende como pode criticá-la de uma forma tão "lúcida"? Concordo plenamente em que o entendimento real da prática científica só pode ser alcançado pelo entendimento real do método científico. Se quer uma boa definição, para o leigo, do que é o método científico pode tentar o livro, escrito para leigos, A Escalada da Ciência, de Brian Silver.
Ítalo Moraes Rocha Guedes


---------------------------------
Yahoo! Acesso Grátis - Internet rápida e grátis. Instale o discador do Yahoo! agora.

[As partes desta mensagem que não continham texto foram removidas]



SUBJECT: Re: [ciencialist] Dúvida de matemática.
FROM: Italo <itscefet03@yahoo.com.br>
TO: ciencialist@yahoogrupos.com.br
DATE: 25/01/2005 12:36

Acho que está correta, como usei o equaltional editor não dá para mandar para o mail do grupo coloquei em:
http://geocities.yahoo.com.br/itscefet03/eq.doc


Ah! E eu coloquei resolvendo-sea (o a ficou junto desculpe) equação (o método foi o famoso Báskara :-)

[ ]'s

Ítalo



Luiz Ferraz Netto <leobarretos@uol.com.br> wrote:
Querem me dar uma mãozinha nessa? Recebi de um consulente ... e hoje o número de mensagens extrapolou!

"Gostaria de pedir a sua ajuda para resolver uma equação que à algum tempo estou tentando e não obtenho sucesso.
A equação é a seguinte:
a * cos(teta) - b * raiz ( 1 - cos^2(teta) ) = c
onde a, b, c são constantes e a incógnita é "teta".

[]'

===========================
Luiz Ferraz Netto [Léo]
leobarretos@uol.com.br
http://www.feiradeciencias.com.br
===========================
----------

Internal Virus Database is out-of-date.
Checked by AVG Anti-Virus.
Version: 7.0.300 / Virus Database: 265.6.13 - Release Date: 16/01/2005


[As partes desta mensagem que não continham texto foram removidas]



##### ##### #####

Para saber mais visite
http://www.ciencialist.hpg.ig.com.br


##### ##### ##### #####


Yahoo! Grupos, um serviço oferecido por:PUBLICIDADE


---------------------------------
Links do Yahoo! Grupos

Para visitar o site do seu grupo na web, acesse:
http://br.groups.yahoo.com/group/ciencialist/

Para sair deste grupo, envie um e-mail para:
ciencialist-unsubscribe@yahoogrupos.com.br

O uso que você faz do Yahoo! Grupos está sujeito aos Termos do Serviço do Yahoo!.


__________________________________________________
Converse com seus amigos em tempo real com o Yahoo! Messenger
http://br.download.yahoo.com/messenger/

[As partes desta mensagem que não continham texto foram removidas]



SUBJECT: Re: [ciencialist] Re: orguloso do saber?
FROM: "Sergio M. M. Taborda" <sergiotaborda@terra.com.br>
TO: ciencialist@yahoogrupos.com.br
DATE: 25/01/2005 14:15

ÿffffcdtalo Rocha wrote:

> E o que é que sugere o senhor Sergio "todo indignação" Taborda?

O sr Sergio n\ao sugere porra nenhuma. J]a foi tempo em que gastava meu
latim con sugestões. Cada um que se desenrrasque. Cá estarei para
responder a duvidas especificas, mas não tenho mais paciencia para
duvidas básicas que aparecem todos os meses aqui. Mais valia criar um
FAQ para não temos que repetir sempre a mesma conversa.

> Sim, porque o que entendi foi a mesma coisa que o outro senhor a quem
> vc respondeu, de forma tão estúpida, entendeu.

E daí ? Vc entendeu ? Otimo. E qual é sua profissão mesmo ?

> Tenho minhas dúvidas se o leigo realmente faz um esforço tão grande em
> entender a ciência, e já que não entende como pode criticá-la de uma
> forma tão "lúcida"? Concordo plenamente em que o entendimento real da
> prática científica só pode ser alcançado pelo entendimento real do
> método científico.

E o que o método cientifico tem a ver com a politiquice entre os
cientistas e entre estes e as instituições que os contratam , e o que
tudo isso tem a ver com a negação da publicação por peer-reviwers ?
Queiram por favor responder a esta pergunta.

Sérgio Taborda

----------

No virus found in this outgoing message.
Checked by AVG Anti-Virus.
Version: 7.0.300 / Virus Database: 265.7.2 - Release Date: 21-01-2005


[As partes desta mensagem que não continham texto foram removidas]



SUBJECT: Re: [ciencialist] Re: orguloso do saber?
FROM: "Oraculo" <oraculo@atibaia.com.br>
TO: <ciencialist@yahoogrupos.com.br>
DATE: 25/01/2005 14:31

Olá Taborda

Descupe, mas estamos nos desviando dos argumentos para posturas politicas. Veja este trecho de sua reposta:

"Taborda: O que adianta isso se lhe negarem todos os fundos para fazer a
experiencia ? Vc acha que as experiencias são baratas ? Em que mundo vc
vive ? No mundo real a ciencia paga-se , e custa dinheiro e só investe
nela quem tem interesse , e ninguem tem interesse em ser difamado por
patrocinar uma expriencia de alguem que é considerado um loco pelos seus
congeneres."

Eu digo que seria bom, para melhorar a compreensão da ciência, que se torna mais complexa a cada dia, que as pessoas conhecessem o método científico, a forma como a ciência testa com rigor seus procedimentos e resultados. Você diz que é bobagem que nem mesmo existe um consenso e como se poderia saber sobre algo que ninguém sabe o que é. Eu respondo que penso diferente, que o método é preciso, conhecido, e daria boa idéia da confiabilidade de resultados, caso o leigo também se aproximasse dele.

E você responde assim, "o que adianta isso (existir o método, conhece-lo), se lhe negarem fundos de pesquisa? Experiencias não são baratas etc".

"Oras" Taborda, eu concordo com você..:-) É uma critica perfeitamente válida e devemos lutar e criticar esse problema. Mas, o que tem isso a ver com a discussão inicial? Sua crítica, com a qual concordo, implica na existencia, validade e um mínimo consenso sobre o método cientifico.:-)

Mais uma vez, não estou discutindo as deficiências dos seres humanos, vaidades e problemas politicos, poderes e interferencia, estou debatendo a abstração chamada ciência.

Sim, ciência existe porque nós existimos, mas os conhecimentos que ela produz independem disso. Átomos se fundiam no Sol, mesmo antes de exirtirem seres humanos apra comprewender como isso ocorre, DNA ainda guardava informação e codificava proteinas, etc. Ciência depende de nós, mas pode ser analisada a partir de uma perspectiva exterior, quanto a confiabilidade de seus resultados.

E uma pesquisa que "não foi feita porque seus patrocinadores consideraram a premissa louca" e outra que "foi feita porque seus patrocinadores consideraram a premissa válida", não tem nada com isso. A discussão era, tanto a pesquisa que foi feita, por não ser louca, quanto a que não foi feita, por ser louca, apresentariam resultados confiáveis se usassem o rigor cientifico.

Críticas sobre o uso desse conhecimento são válidas, e eu as partilho com você. Mas, e daí?

Homero


----- Original Message -----
From: Sergio M. M. Taborda
To: ciencialist@yahoogrupos.com.br
Sent: Tuesday, January 25, 2005 10:52 AM
Subject: Re: [ciencialist] Re: orguloso do saber?


Oraculo wrote:

>Olá
>
>Bem, todos tem direito a ter sua opinião, mesmo que equivocada..:-) Respeito a sua.
>
>
>Mas continuo com a minha, o ray não tem razão. Está confundindo ciência com o uso que é feito dela por seres humanos.
>
>

Não existe essa destinção. A ciencia é aquilo que fazemos com ela, pois
ela não existe sem nós.

>E penso que conhecer o método não é tão dificil, nem penso que falte definição. Você não aceita e tem restrições, o que nào significa que esteja certo e pronto, e o resto do mundo errado. O método, qualquer que seja sua concepção, envolve rigor e confiabilidade de resultados.
>

O que adianta isso se lhe negarem todos os fundos para fazer a
experiencia ? Vc acha que as experiencias são baratas ? Em que mundo vc
vive ? No mundo real a ciencia paga-se , e custa dinheiro e só investe
nela quem tem interesse , e ninguem tem interesse em ser difamado por
patrocinar uma expriencia de alguem que é considerado um loco pelos seus
congeneres.

>Se o leigo puder perceber isso e comparar com o conhecimento produzido sem ele (achismos e crendices, revelações ou pseudo-ciencias como a astrologia), poderá compreender melhor porque aceitamos determinadas alegações e não outras, mesmo que nào conheçamos a fundo a matéria.
>
>
Estamos fartos de saber essa resa sr padre. O assunto aqui é o leigo não
consegue entender ciencia naquilo que ela diz. Por exemplo, o leigo não
consegue entender o que é o Big Bang , o que é a Teoria dos Grafos, o
que é a teoria da evolução das especies moderna, não sabe o que Marte
tem a ver com Jupiter . Não sabe o que é a Teora dos Muitos mundos, ou
se sequer se vale a pena sabê-la. Não sabe muitas vezes a Lei da
Acçao-Reacção, A Lei do Movimento Perpétuo, nem a Lei da Equivalencia.
Muito menos o primeiro postulado da teoria da relatividade, o que ele
tem a ver com efeito de Doppler, o que é o efeito de Doppler . Muitas
vezes o leigo nem consegue aplicar a 2ª Lei de Newton lei como deve ser
, quanto mais a 2ª lei da termodinamica, ou a 3ª. É claro que ele pode
ler, ele pode entender o método e não acreditar em supertições mais
isso não o fará entender fisica, nem nenhuma ciencia em particular. No
máximo terá uma boa ideia daquilo que sabemos, do ponto de vista
enciclopédico, mas dificilmente saberá encaixar as partes para formar
novos raciocinios que não tenha sido expostos, ou que sejam errados à
partida.
O pensamento critico, a liberdade de pensamento, a imparcialidade , as
regras , o metodo ... são tudo ferramentas que não funcionam sozinhas.

>Pensamento crítico é mais interessante ainda, já que estimula a dúvida e o questionamento. Que podem ser aplicados na escolha do que levar em conta ao decidir.
>
Ao dicidir o quê ? Se Eisntein plagiou Poincaré ? Se o Big Bang existiu
? Se existe vida noutros planetas ? Se devemos suportar o projecto SETI
? SE devemos comprar um livro do Hawkings ? Se devemos dar ouvidos a
quem nunca leu um paper verdadeiro ? Se os professores da faculdade
sabem mais que os alunos ? Se é preciso entrar numa faculdade para fazer
ciencia ? Se vale a pena vir colocar as duvidas na cienclist pois a
resposta pode ser mais idiota que a pergunta ? Onde pára o poder de
decisão que a ciencia oferece ?

> E é isso que precisa o leigo, e não compreender fisica quantica para decidir se a teoria das super cordas deve ou não ser pesquisada (quem tem de decidir e saber sobre isso são os cientistas envolvidos).
>
>
Ja foi tempo em que ele não queria entender essas coisas. A media faz
tanta pressão que o leigo sentese idiota por não entender essas teorias
avançadas e recorre às fontes da propria media para se informar. O
problema é que essas fontes são ainda mais leigas que ele , e ele acaba
recorrendo a listas com esta. O leigo precisa, primeiro de tudo
entender, que não ha mal no mundo se ele não entender fisica quantica ,
ou TRG.

>Você tem mágoa com a ciência, aquela que virou uma "palhaçada". Direito seu. Mas muitos cientistas, que também merecem respeito, pensam diferente. Fico com eles..:-) Escolha minha. Minha opinião.
>
>
Pena é que sua opinião não tenha nada a ver com o assunto. O assunto são
as pessoas, o cooperativismo cientifico, as facadas que cada cientista
tem que dar no outro se quiser publicar seu paper e receber fundos para
mais um ano de trabalho. A impossibilidade de trabalhar com teorias
alternativas so porque elas são alternativas. Em resumo, tudo o que
acontece no mundo real - que concerteza não acontece no seu mundo de
fantasia - e que nada tem a ver com metodo, teste, entendimento e sim
com inveja, cobiça e ego.
Pelas suas palavras deduzo que vc não deve trabalhar num centro onde se
faça ciencia, pois se trabalhasse teria sentido esses efeitos e pensaria
duas vezes antes de descrever o seu mundo perfeito.

>A crítica a politica dentro da ciência, a influencia de poderse e interesses, a defeitos humanos e sociais é perfeita e importante. Corrigir é imprescindível. Mas misturar essa necessidade com ataques de mágoa contra o conhecimento como um todo, não faz sentido.
>
>
Mas ninguem atacou o conhecimento como um todo. Vc deve ter entendido
algo alguma coisa fora do contexto que não estáva escrito. É exactamente
da "politica dentro da ciência, a influencia de poderse e interesses, a
defeitos humanos e sociais" que estamos falando. O que não faz sentido é
vc vir com essa conversa de metodo e teste e blablabla que não tem nada
a ver com o assunto.
A prespectiva do Ray é muito lucida e verdadeira, doa a quem doer.

Sérgio Taborda




--
No virus found in this outgoing message.
Checked by AVG Anti-Virus.
Version: 7.0.300 / Virus Database: 265.7.2 - Release Date: 21-01-2005



##### ##### #####

Para saber mais visite
http://www.ciencialist.hpg.ig.com.br


##### ##### ##### #####


Yahoo! Grupos, um serviço oferecido por:







------------------------------------------------------------------------------
Links do Yahoo! Grupos

a.. Para visitar o site do seu grupo na web, acesse:
http://br.groups.yahoo.com/group/ciencialist/

b.. Para sair deste grupo, envie um e-mail para:
ciencialist-unsubscribe@yahoogrupos.com.br

c.. O uso que você faz do Yahoo! Grupos está sujeito aos Termos do Serviço do Yahoo!.



[As partes desta mensagem que não continham texto foram removidas]



SUBJECT: Re: [ciencialist] Re: orguloso do saber?
FROM: "Oraculo" <oraculo@atibaia.com.br>
TO: <ciencialist@yahoogrupos.com.br>
DATE: 25/01/2005 14:46

Olá Taborda


"Taborda: E o que o método cientifico tem a ver com a politiquice entre os
cientistas e entre estes e as instituições que os contratam , e o que
tudo isso tem a ver com a negação da publicação por peer-reviwers ?
Queiram por favor responder a esta pergunta."


Eu respondo com prazer: NADA..:-) Rigorosamente nada. E exatamente por isso, nada tem a ver com a discussão inicial, como tornar a ciência mais comprensível e proxima ao leigo, que não pretende nem estudar fisica avançada, nem ser contratado para pesquisas, nem ser afetado pela politiquice, nem publicar peer-rewies, apenas compreender melhor a tal da ciência.

A revolta contra a politicagem no meio cientifico é justa e válida. Mas não tem nada a ver com a discussão.

Compreender o método cientifico ajuda a compreender a confiabilidade de um resultado. Ajuda a compreender porque se confia nesses resultados mais que, digamos, casos anedóticos. Ajuda até mesmo a comprender que uma "politicagem" distorce essa confiabilidade e permite que o cidadão de mas importancia a controles e rigor nos mecanismos não cientificos (sobre os quas ele tem alcance), como no caso dos medicamentos e drogas.

Essa era a discussão, como melhorar a compreensão do leigo sobre ciência. E isso não se resolve com a furia contra tudo e todos nem tentando ensinar fisica avançada ou biologia molecular para todas as pessoas e cidadãos.

Um abraço.

Homero




----- Original Message -----
From: Sergio M. M. Taborda
To: ciencialist@yahoogrupos.com.br
Sent: Tuesday, January 25, 2005 2:15 PM
Subject: Re: [ciencialist] Re: orguloso do saber?


ÿffffcdtalo Rocha wrote:

> E o que é que sugere o senhor Sergio "todo indignação" Taborda?

O sr Sergio n\ao sugere porra nenhuma. J]a foi tempo em que gastava meu
latim con sugestões. Cada um que se desenrrasque. Cá estarei para
responder a duvidas especificas, mas não tenho mais paciencia para
duvidas básicas que aparecem todos os meses aqui. Mais valia criar um
FAQ para não temos que repetir sempre a mesma conversa.

> Sim, porque o que entendi foi a mesma coisa que o outro senhor a quem
> vc respondeu, de forma tão estúpida, entendeu.

E daí ? Vc entendeu ? Otimo. E qual é sua profissão mesmo ?

> Tenho minhas dúvidas se o leigo realmente faz um esforço tão grande em
> entender a ciência, e já que não entende como pode criticá-la de uma
> forma tão "lúcida"? Concordo plenamente em que o entendimento real da
> prática científica só pode ser alcançado pelo entendimento real do
> método científico.

E o que o método cientifico tem a ver com a politiquice entre os
cientistas e entre estes e as instituições que os contratam , e o que
tudo isso tem a ver com a negação da publicação por peer-reviwers ?
Queiram por favor responder a esta pergunta.

Sérgio Taborda

----------

No virus found in this outgoing message.
Checked by AVG Anti-Virus.
Version: 7.0.300 / Virus Database: 265.7.2 - Release Date: 21-01-2005


[As partes desta mensagem que não continham texto foram removidas]



##### ##### #####

Para saber mais visite
http://www.ciencialist.hpg.ig.com.br


##### ##### ##### #####


Yahoo! Grupos, um serviço oferecido por:







------------------------------------------------------------------------------
Links do Yahoo! Grupos

a.. Para visitar o site do seu grupo na web, acesse:
http://br.groups.yahoo.com/group/ciencialist/

b.. Para sair deste grupo, envie um e-mail para:
ciencialist-unsubscribe@yahoogrupos.com.br

c.. O uso que você faz do Yahoo! Grupos está sujeito aos Termos do Serviço do Yahoo!.



[As partes desta mensagem que não continham texto foram removidas]



SUBJECT: Missão Cassini-Huygens supera a ficção científica
FROM: "Oraculo" <oraculo@atibaia.com.br>
TO: <ciencialist@yahoogrupos.com.br>
DATE: 25/01/2005 15:29

Missão Cassini-Huygens supera a ficção científica
Gasto com a ciência deveria prevalecer sobre as despesas militares

Lawrence M. Krauss*
Especial para o NYTimes

Uma pequena sonda encalhada em um mundo muito distante e hostil opera por duas preciosas horas em uma temperatura de cerca de -180 °C. Desesperadamente, ela transmite informações à sua nave-mãe, antes que esta desapareça abaixo do horizonte, deixando a pequena exploradora sozinha sobre o solo esponjoso do seu novo lar alienígena, perdendo vagarosamente energia e condenada a jazer eternamente em uma lua congelada a 1,2 bilhão de quilômetros da Terra.

Eu poderia ser acusado de fazer antropomorfização, mas o destino da pequena sonda Cassini-Huygens, assentada nas proximidades de uma planície coberta de metano e de hidrocarbonetos congelados na maior lua de Saturno, Titã, capturou a minha imaginação mais do que qualquer tarefa que possa estar sendo desempenhada neste momento pelos astronautas da Estação Espacial Internacional.

O que realmente me tocou foi o céu alaranjado. Ele demonstrou com impressionante clareza que as maravilhas da ficção científica com as quais sonhei quando era criança estão sendo reveladas pelas nossas sondas espaciais não-tripuladas. E isso de uma maneira que é ao mesmo tempo mais encantadora e informativa do que qualquer novidade que possa advir do investimento de todas as verbas da Nasa na ida de mais alguns astronautas à Lua e, algum dia, a Marte.

Admito que já havia sido cativado por imagens da Internet como aquelas transmitidas pelos veículos exploradores marcianos. Imagens que lembram, de forma suspeitosa, um pôr do sol enevoado em Los Angeles. Mas, até agora, os mundos que chegaram de maneira impressionante ao meu computador estavam mais próximos daquilo que eu poderia vislumbrar ao explorar um deserto terrestre do que daqueles locais exóticos que capturaram a minha imaginação quando, na infância, lia histórias de ficção científica, ou das ilustrações de artistas que procuraram mostrar superfícies planetárias imaginárias.

Quando visitei o site da sonda Cassini-Huygens, os pedregulhos escuros compostos de gelo sujo de hidrocarbonetos na superfície de Titã surgiram em meio ao brilho alaranjado de uma atmosfera diferente de tudo o que eu já havia visto.

Lembrei-me das velhas histórias de ficção científica. Na Internet encontrei um exemplo recente daquilo que eu costumava saborear. Um conto premiado, "Slow Life" ("Vida Lenta"), de Michael Swanwick, a respeito de exploradores humanos que procuravam vida em Titã.

"As pessoas falavam muito sobre a 'atmosfera turva alaranjada' de Titã, mas os nossos olhos se ajustaram. Após aumentar o ganho do visor do capacete, as montanhas de gelo se tornaram fascinantes! As correntes de metano escavaram runas crípticas nas montanhas. Então, no solo repleto de tolina (substância marrom-avermelhada formada de compostos orgânicos complexos), o branco se transformou em uma profusão de laranjas, vermelhos e amarelos".

Na verdade, o gelo é mais sujo e a superfície mais escura. Mas a paisagem de Titã é assustadoramente similar àquela imaginada de forma tão intensa por Swanwick. Exceto pelo fato de que a verdade é ainda mais estranha e encantadora que a sua ficção.

Soube por meio de uma entrevista à imprensa concedida na última sexta-feira (21/01) pela equipe científica da sonda Cassini-Huygens que existem evidências de que há vulcões ativos na superfície de Titã, com base no argônio 40 presente na sua atmosfera. Mas esses vulcões não expelem lava derretida. Em vez disso, os vulcões titanianos soltam colunas de água e amônia, assim como aqueles que fiz com meus kits de química quando era criança.

Há de fato nuvens, metano e chuvas de hidrocarbonetos, mas a realidade de uma atmosfera turbulenta de ventos de metano me foi passada de uma maneira impossível de ser descrita por qualquer texto. Devido a uma brilhante perspicácia, a sonda Huygens é dotada de um microfone. À medida que a sonda descia através das nuvens, a partir de uma altitude de 160 quilômetros, foi possível ouvir, bem como ver, a superfície que se aproximava, conforme a nave enviava uma série de fotos. Ficar sentado em frente ao meu computador, no meio da madrugada, ouvindo rajadas de ventos alienígenas em uma lua remota de Saturno foi uma experiência ao mesmo tempo estranha e comovente.

Considero-me uma pessoa de sorte por viver em uma época na qual os seres humanos são capazes de ver tal mundo alienígena verdadeiro dotado de charcos de metano e de um céu de cores novas. Foi isso, provavelmente, o que me atraiu a princípio para a ciência. Embora a literatura tenha o poder de nos retirar do tédio da existência cotidiana, a ciência em sua melhor forma possui a capacidade de nos transportar até mundos totalmente diferentes, tanto de forma literal quanto metafórica. Ela pode nos levar até onde as nossas imaginações talvez nunca pudessem viajar.

Em duas curtas horas, uma pequena sonda não tripulada mudou a minha experiência direta do sistema solar de uma maneira que eu jamais imaginara. Agora fico ansiando por mais episódios do gênero. Talvez eu testemunhe outras sondas que mergulhem em mares alienígenas distantes sob as superfícies de luas congeladas. Quem sabe uma delas envie para a Terra as evidências de vida alienígena existente ou extinta.

Porém, falando de forma realista, o futuro provavelmente estará repleto de retrocessos e carências, com bilhões de dólares alocados para a proteção de populações que nós colocamos em risco, ou para a construção de caros sistemas de defesas antimíssil contra ameaças inexistentes. Só nos é possível torcer para que tenha restado alguma imaginação no governo para permitir que continuemos a financiar essas missões. Missões que fazem a ciência que realmente modifica a forma como pensamos a respeito do nosso lugar no universo.

Ir destemidamente até locais nunca dantes visitados, de uma maneira só possível às espaçonaves não tripuladas, custará comparativamente tão pouco que tal iniciativa não deverá interferir na atual prioridade de permitir que os astronautas experimentem novas aventuras na Lua.

Quanto a isso, é significativo que a sonda Huygens seja um produto da Agência Espacial Européia, trabalhando em conjunto com a Nasa e com o Laboratório de Jato-Propulsão. Isso não só demonstra que a Europa é atualmente uma liderança no campo da exploração espacial, mas revela também que, no que diz respeito aos grandes projetos humanos, como a exploração do nosso universo ou do espaço e do tempo em escalas fundamentais, nós podemos e precisamos trabalhar juntos em uma escala global.

Esse é um dos benefícios colaterais da aventura científica. No entanto, ainda mais do que isso, o universo continua a nos surpreender. Em última instância ele é bem mais interessante do que qualquer coisa que os escritores de ficção científica ou os artistas possam imaginar. A vida pode imitar a arte, mas no final das contas ela a transcende. E é por isso que algumas vezes precisamos nos voltar para o próprio universo em busca de inspiração.

*Lawrence M. Krauss é diretor do Centro de Educação e Pesquisa em Cosmologia e Astrofísica da Universidade Case Western Reserve. Seu livro mais recente foi "Atom".

http://noticias.uol.com.br/midiaglobal/nytimes/2005/01/25/ult574u4994.jhtm



[As partes desta mensagem que não continham texto foram removidas]



SUBJECT: Re: orguloso do saber?
FROM: "rayfisica" <rayfisica@yahoo.com.br>
TO: ciencialist@yahoogrupos.com.br
DATE: 25/01/2005 16:30


--- Em ciencialist@yahoogrupos.com.br, ÿffffcdtalo Rocha
<imrochaguedes@y...> escreveu
Tenho minhas dúvidas se o leigo realmente faz um esforço
tão grande
em entender a ciência, e já que não entende como pode
criticá-la de
uma forma tão "lúcida"?


Solicito permissão para entrar nessa, ainda que eu seja um
néscio,
talvez professor o leigo e representando esse que vos fala, faça
um
real esforço para tentar entender a ciência, o problema é
que o lugar
que os senhores nos reservaram é muito acanhado, é o mesmo que
o
pessoal daquela instituição já referida fez.
Mas nos os leigos estamos um pouquinho tristes pois quando os
senhores brigaram com ela nos vos demos apoio e agora os senhores
querem nos tratar exatamente como ela, mas o pior é que a dita
instituição apenas leva nossa alma pro inferno depois da morte
(hahahaha) e parece me que os senhores estão nos levando prum sei

o que, que vai pra lugar nenhum em vida.
Ajudem-nos, não alijando como cães sarnentos que talvez
possamos
surpreendê-los e talvez a nossa critica seja justa por ter a ver
conosco mesmo, lembre se você já foi um de nos e nos poderemos
ser um
de vocês.+-
Ah, e se o seu conhecimento for baseado em falácias, tu és o
maior
néscio de todos nos.



Concordo plenamente em que o entendimento real da prática
científica
só pode ser alcançado pelo entendimento real do método
científico. Se
quer uma boa definição, para o leigo, do que é o método
científico
pode tentar o livro, escrito para leigos, A Escalada da Ciência,
de
Brian Silver.
&#61656; Ítalo Moraes Rocha Guedes

Obrigadíssimo pela dica senhor, de verdade.
Desculpen nos se somos desconfiados, mas estamos fazendo exatamente o
que os senhores nos ensinaram outrora.






SUBJECT: Re: Novo tópico, mesmo assunto?
FROM: "rayfisica" <rayfisica@yahoo.com.br>
TO: ciencialist@yahoogrupos.com.br
DATE: 25/01/2005 16:41


--- Em ciencialist@yahoogrupos.com.br, "JVictor" <jvoneto@u...>
escreveu
> Ray,
>
> Abaixo um site(veja seus desdobramentos) que " fuxicam" sobre esse
camarada misterioso e vero.
>
> http://www.fisica.ufc.br/donafifi/neutrino/neutrino6.htm
>
> Sds,
>
> Victor


Como sempre as suas dicas e mensagens são muito úteis, eu já conheço
a famosa dama de sobral e o seu jeitinho de ensinar e vou até
confessar que sou um pouco apaixonado por ela.
Muito obrigado pelas dicas e ensinamentos que o senhor tem me dado.
Desculpe se as vezes pareço um pouco arrogante, mas eu é que não vou
perder a chance de aprender apenas por um pequeno detalhe, vou
arrancar o que eu puder em ensinamento.
Estou pesquisando a wikipedia, valeu em.






SUBJECT: aranha "papa-mosca"
FROM: "E m i l i a n o C h e m e l l o" <chemelloe@yahoo.com.br>
TO: <ciencialist@yahoogrupos.com.br>
DATE: 25/01/2005 17:30

mais aranhas... Takata! É contigo! :-)

[ ] 's do Emiliano Chemello
emiliano@quimica.net
http://www.quimica.net/emiliano
http://www.ucs.br/ccet/defq/naeq

--

Contato Naeq:
Nome: cl ber
Email: CLEBER-MENDES@UOL.COM.BR
Telefone: arranhas
Mensagem: queria saber se as papa-mosca tem veneno e se mortal
Contato Naeq:
Nome: cl ber
Email: CLEBER-MENDES@UOL.COM.BR
Telefone: arranhas
Mensagem: queria saber se as papa-mosca tem veneno e se mortal





SUBJECT: Re: [ciencialist] Re: Novo tópico, mesmo assunto?
FROM: JVictor <jvoneto@uol.com.br>
TO: ciencialist@yahoogrupos.com.br
DATE: 25/01/2005 20:12



rayfisica wrote:

>
> --- Em ciencialist@yahoogrupos.com.br, "JVictor" <jvoneto@u...>
> escreveu
> > Ray,
> >
> > Abaixo um site(veja seus desdobramentos) que " fuxicam" sobre esse
> camarada misterioso e vero.
> >
> > http://www.fisica.ufc.br/donafifi/neutrino/neutrino6.htm
> >
> > Sds,
> >
> > Victor
>
>
> Ray: eu já conheço
> a famosa dama de sobral e o seu jeitinho de ensinar e vou até
> confessar que sou um pouco apaixonado por ela.

Victor: Espalha não, mas aquí só prá nós: eu também.

Sds,

Victor.

>
>
>
>
>
>
> ##### ##### #####
>
> Para saber mais visite
> http://www.ciencialist.hpg.ig.com.br
>
>
> ##### ##### ##### #####
>
>
> *Yahoo! Grupos, um serviço oferecido por:*
> <http://br.rd.yahoo.com/SIG=12ar327tb/M=264379.5078783.6203979.1588051/D=brclubs/S=2137111528:HM/EXP=1106764890/A=2191897/R=0/SIG=10vqa2grn/*http://br.diversao.yahoo.com/>
>
> <http://br.rd.yahoo.com/SIG=12ar327tb/M=264379.5078783.6203979.1588051/D=brclubs/S=2137111528:HM/EXP=1106764890/A=2191897/R=1/SIG=10vqa2grn/*http://br.diversao.yahoo.com/>
>
>
>
> ------------------------------------------------------------------------
> *Links do Yahoo! Grupos*
>
> * Para visitar o site do seu grupo na web, acesse:
> http://br.groups.yahoo.com/group/ciencialist/
>
> * Para sair deste grupo, envie um e-mail para:
> ciencialist-unsubscribe@yahoogrupos.com.br
> <mailto:ciencialist-unsubscribe@yahoogrupos.com.br?subject=Unsubscribe>
>
> * O uso que você faz do Yahoo! Grupos está sujeito aos Termos do
> Serviço do Yahoo! <http://br.yahoo.com/info/utos.html>.
>
>



SUBJECT: Era uma vez um telescópio....chamada Hubble
FROM: Maria Natália <grasdic@hotmail.com>
TO: ciencialist@yahoogrupos.com.br
DATE: 26/01/2005 01:03


Afinal Hubble vai morrer pois não haverá dinheiro:
http://space.com/scienceastronomy/hubble_reaction_050121.html
Um abraço
Maria Natália







SUBJECT: Re: aranha "papa-mosca"
FROM: Maria Natália <grasdic@hotmail.com>
TO: ciencialist@yahoogrupos.com.br
DATE: 26/01/2005 03:53


Mas que aranha é essa?
A de cruz? Ou aquela que vive nas vidraças das janelas esperando sem
teia que as moscas batam nos vidros transparentes?
Se se refere a estas não são venenosas
Mas diga o nosso expert.
Maria Natália


--- Em ciencialist@yahoogrupos.com.br, "E m i l i a n o C h e m e l
l o" <chemelloe@y...> escreveu
> mais aranhas... Takata! É contigo! :-)
>
> [ ] 's do Emiliano Chemello
> emiliano@q...
> http://www.quimica.net/emiliano
> http://www.ucs.br/ccet/defq/naeq
>
> --
>
> Contato Naeq:
> Nome: cl ber
> Email: CLEBER-MENDES@U...
> Telefone: arranhas
> Mensagem: queria saber se as papa-mosca tem veneno e se mortal
> Contato Naeq:
> Nome: cl ber
> Email: CLEBER-MENDES@U...
> Telefone: arranhas
> Mensagem: queria saber se as papa-mosca tem veneno e se mortal





SUBJECT: Ainda acerca de mulheres na Ciência
FROM: Maria Natália <grasdic@hotmail.com>
TO: ciencialist@yahoogrupos.com.br
DATE: 26/01/2005 03:55


Desculpe Leo mas vai em inglês pois trata-se de Harvard e da posição
do NSTA:
"Harvard University President Lawrence Summers set off a firestorm
last week when he commented on why fewer women succeed in the fields
of science and math. He made remarks at an academic conference on
women and minorities in the science and engineering workforce, which
prompted one female participant in the audience to walk out. The
comments drew retort from Harvard faculty, its student body, and
beyond. More than 120 professors in Harvard's Faculty of Arts and
Sciences sent Dr. Summers a rebuttal, the National Organization for
Women called for his immediate dismissal, and a media frenzy has
ensued since the Boston Globe initially reported the story.
Dr. Summers has apologized for his comments. In a letter posted on the
Harvard University's web site, he said "I deeply regret the impact of
my comments and apologize for not having weighed them more carefully.
Despite reports to the contrary, I did not say, and I do not believe,
that girls are intellectually less able than boys, or that women lack
the ability to succeed at the highest levels of science." To view Dr.
Summer's letter, go to
http://www.president.harvard.edu/speeches/2005/womensci.html.
NSTA believes strongly that girls can and do succeed in science and
should be supported fully in this endeavor. To spread this message,
NSTA President Anne Tweed, along with astronaut Sally Ride and a
science student from Harvard, conducted a live interview on ABC's Top
Priority news program on Friday, January 21, and has given comments to
various other media outlets. To read an article in The New York Times
about Dr. Summer's apology, go to
http://www.nytimes.com/2005/01/20/education/20harvard.html (Free
registration is required). To view the NSTA position statement on
Gender Equity, go to http://www.nsta.org/positionstatement&psid=37."
sds
Maria Natália






SUBJECT: Re: Para quem quer se indignar!!!
FROM: Maria Natália <grasdic@hotmail.com>
TO: ciencialist@yahoogrupos.com.br
DATE: 26/01/2005 04:01



òi pessoal.
isto está mau aí...
Será que tenho de levar segurança ou vocês vão-me ajudar?
É preciso coragem para ir ao Brasil!
Maria Natália


--- Em ciencialist@yahoogrupos.com.br, ÿffffcdtalo Rocha
<imrochaguedes@y...> escreveu
> Saiu dia 24/01 no Jornal da Ciência da SBPC (Sociedade Brasileira
para o Progresso da Ciência):
> SBPC se solidariza com Lúcio Flávio Pinto, vítima de brutal agressão
em Belém Mensagem do presidente da SBPC, Ennio Candotti, afirma que
`a prepotência e a brutalidade são a marca dos tiranetes' que Lúcia
Flávio sempre combateu. Veja abaixo notícia do `Diário do Pará' a respeito
>
> Eis a íntegra da mensagem:
>
> `Meu caro Lúcio Flávio, conselheiro da SBPC, amigo,
>
> Foi com grande indignação que recebi a noticia da covarde agressão
que você sofreu. Das ameaças de morte, pusilânimes. A prepotência e a
brutalidade são a marca dos tiranetes que você sempre combateu. Que
nós combatemos. Solidários.
>
> A SBPC está com você. Em terras amigas e também nas inimigas.
>
> Belém, e não só Belém, vive um tempo em que o estado de violência
ainda persiste, nos subterrâneos e fora deles. A punição tarda. A
sociedade reclamPessoal,
Isto está mal por aí...
Será que tenho de levar segurança ó vocês me bastais?
Maria Natáliaa.
>
> Contamos com a informação, o conhecimento e a educação para derrubar
a violência, prepotente e assassina, e promover o estado de justiça e
liberdade. Em Belém e no Brasil.
>
> Oferecemos nossas melhores forças para que você possa continuar a
escrever, a denunciar as injustiças, a pensar. O seu exemplo agora é
de todos. É dele que nos orgulhamos.
>
> É isso que os seus agressores mais temem.
>
> Com um forte abraço,
> Ennio Candotti'
>
> Leia, a seguir, a reportagem do `Diário do Pará', publicada nesta
segunda-feira a respeito da agressão sofrida por Lúcio Flávio Pinto:
>
> Jornalista vai ao ministro da Justiça
>
> Nome de executivo das ORM corre o país pela agressão e ameaça de
morte a Lúcio Flávio Pinto
>
> O jornalista Lúcio Flávio Pinto vai pedir uma audiência com o
ministro da Justiça, Márcio Thomaz Bastos, para denunciar o
espancamento de que foi vítima na última sexta-feira.
>
> Ele foi agredido em público e ameaçado de morte pelo empresário
Ronaldo Maiorana, diretor-corporativo das Organizações Romulo Maiorana
e por quatro seguranças (sargentos da Polícia Militar), no restaurante
do Parque da Residência.
>
> O jornalista afirma que o inquérito policial sobre o caso já está
instaurado e que vai entrar na Justiça por conta da ameaça de morte,
além de preparar uma ação de indenização.
>
> As Organizações Romulo Maiorana editam o jornal O Liberal e
retransmitem a programação da TV Globo.
>
> A agressão sofrida pelo jornalista Lúcio Flávio Pinto, na última
sexta-feira, no restaurante do Parque da Residência, ainda repercute
nos meios de comunicação de todo o Brasil.
>
> Lúcio foi espancado brutalmente pelo empresário e proprietário das
Organizações Romulo Maiorana e quatro policiais militares que faziam
sua segurança particular.
>
> Em entrevista ao jornalista Maurício Araújo, do site
http://www.amazonia.org, Lúcio afirmou que vivia `uma sensação de
impotência e tristeza. Pois eles são donos do Pará. Na minha terra sou
submetido a uma lei de cangaço'.
>
> Você pode contar como foi o conflito no restaurante?
>
> LFP - Estávamos almoçando no restaurante do Parque da Residência,
como fazemos habitualmente, eu e um grupo de amigos, umas oito
pessoas, salvo engano. Passada meia hora no restaurante, chegou o
Ronaldo com um amigo. Sentou numa mesa exatamente atrás de mim, de
modo que ficou nas minhas costas - eu não podia vê-lo. Após um
intervalo de tempo de cerca de vinte minutos senti um murro na face
direita. Neste momento, ele já estava acompanhado de seus seguranças
particulares, que são dois soldados da Polícia Militar, um deles
conhecido como Saddam. Após o murro ele me deu uma gravata,
arrebentando todos os botões da minha camisa. Na seqüência veio um
empurrão, caí sobre cadeiras, indo parar no chão. Ronaldo gritava que
ia me matar, senão me matasse ali naquele momento seria em outro
lugar, mas que eu nunca mais iria escrever sobre a família dele! Após
a tentativa de apartar a briga, o André (NR: o economista e
comerciante André Carrapatoso Coelho, que acompanhava Lúcio no almoço)
passou a
> ser agredido por um dos seguranças também. Enquanto Ronaldo
gritava, um dos seguranças me chutava também. Algumas pessoas tentavam
segurá-lo. Finalmente eu consegui me levantar. Disse que não tinha
sentido o que ele estava fazendo. Os próprios seguranças pararam de
bater e foram segurá-lo. Em seguida retiraram-se do restaurante, que
estava lotado, tinha aproximadamente 80, talvez 100 pessoas.
>
> Você então se dirigiu à polícia?
>
> LFP - Do restaurante mesmo eu chamei uma patrulha da PM. A patrulha
chegou 20 minutos depois. Fomos pra Seccional Urbana de São Brás.
Registramos a ocorrência, eu e o André, e depois fui ao IML fazer
exame de corpo de delito, onde foram constatadas as seguintes
agressões: hematomas no rosto, escoriações nos braços, nas pernas e
nas costas.
>
> Quais providências foram tomadas?
>
> LFP - Já está em andamento o inquérito policial. Vou entrar na
Justiça devido à ameaça de morte e entrar com uma ação de indenização.
Ao mesmo tempo, pretendo responsabilizar a família Maiorana por
qualquer coisa que venha a me acontecer. Pretendo escrever uma carta
ao jornal O Estado de São Paulo onde trabalhei por 17 anos. Vou pedir
uma audiência à OAB e vou pedir uma audiência com o ministro da
justiça, pois no Pará não há garantia para o trabalho da imprensa.
>
> Esta é a primeira vez que você é ameaçado de morte?
>
> LFP - Não é a primeira vez que me ameaçam de morte. O marido da
Rosângela (NR: Rosângela Maiorana Kzan, irmã mais velha de Ronaldo e
diretora administrativa do grupo ORM) já me agrediu - e ele é
faixa-preta de caratê - e ameaçou de morte. Na época, eu estava
fazendo uma série de reportagens sobre as dissidências internas no
grupo ORM. A Rosângela entrou com cinco ações sucessivas contra mim,
com base na Lei de Imprensa, isto foi em setembro, outubro de 1992, há
mais de dez anos atrás. Quatro ações penais e uma ação cível para
proibir que eu falasse dela, ou seja, tentou estabelecer uma espécie
de censura prévia. Detalhe: ela nunca mandou cartas ao meu jornal,
nunca contestou as reportagens. Registrei queixa, foi instaurado o
inquérito. E apesar de ele ter sido réu confesso, o Ministério Público
não fez a denúncia e eu tive que entrar com ação privada. Mesmo assim,
o processo não foi adiante e acabou sendo arquivado por prescrição.
>
> Como você se sente? Acha possível que a ameaça seja cumprida?
>
> LFP - Não sei, quanto a isso estou tomando as precauções legais, o
que penso é que essa ameaça existe, de fato. E a simples ameaça mostra
que eles se colocam acima do bem e do mal, para fazer isso - uma
ameaça de morte - na frente de pessoas da sociedade, com dois
seguranças armados que são da Polícia Militar ao lado! É como estar à
mercê de um poder sem limites. Minha sensação é de impotência e
tristeza. Pois eles se consideram donos do Pará. Na minha terra sou
submetido a uma lei de cangaço. Ele não disse nada sobre o meu artigo,
nem uma palavra. Bastou não gostar para julgar que tinha o direito de
agredir e ameaçar de morte um jornalista.
>
> Repúdio online
>
> No blog do jornalista e colunista, Ricardo Noblat
(noblat.blig.ig.com.br/) a informação sob o título `Diretor de jornal
espanca jornalista em Belém' vários comentários de internautas a
respeito do ato classificado com selvageria.
>
> Veja alguns:
>
> - `Acho fundamental que este blig publique fatos escabrosos como
este narrado pelo jornalista de Belém, vítima de agressor seguro da
impunidade. Essas oligarquias regionais ignoram qualquer evolução
possível em nossa (`delas') sociedade. Mais sintomático é que essa
figura é membro do Partido Liberal... Seja no Pará ou qualquer outro
Estado brasileiro, é preciso mostrar que os reis estão nus. Essa é a
função do bom jornalismo. Esclarecer a sociedade. Obrigado'.
>
> - `Este cidadão que agrediu o jornalista deveria estar é preso, pois
lugar de animais é atrás das grades. Quem ameaça também tem medo, pois
se não tivesse medo, não sairia por aí se garantindo nos seus cães de
guarda. Animais iguais a ele. Ricardo Noblat, continue escrevendo o
bom jornalismo, porque se o povo não estiver a par dos acontecimentos,
estes canalhas estragam o Brasil'.
>
> - `O jornalista, com o seu sacrifício, jogou a luz sobre mais um
afiliado da rede Globo, que sempre está do lado do mal. Que coisa, já
repararam que a Globo sempre está contra a corrente?'
>
> - `Andei pelas bandas de Belém nos anos 70, pouco depois que o
comerciante potiguar, radicado na cidade, Romulo Maiorana adquiriu o
jornal O Liberal. Na época trabalhava nos Diários Associados e estava
justamente fazendo uma matéria sobre o envolvimento do comerciante
Romulo Maiorana com o contrabando, haja vista que seu crescimento
econômico repentino induzia a essa premissa. Graças aos generais de
plantão O Liberal cresceu e sufocou os demais jornais paraenses,
inclusive A Província do Pará, os Diários Associados, Maiorana, a
semelhança de ACM na Bahia, conseguiu transferir a programação da Rede
Globo para a sua TV Liberal e a partir daí se transformou no imperador
das comunicações do Pará. Paparicado pela elite paraense, Romulo
sempre utilizou o expediente que seu filho consagrou agora
publicamente com a agressão ao Lúcio Flávio. Só que Romulo pai o fazia
sorrateiramente. Valeu a pena postar a notícia da agressão para que os
blogueiros, do eixo Rio/São Paulo, sempre alheios ao que
> acontece por esse Brasil a fora tenham conhecimento que o buraco
para jornalistas de fora do eixo é mais embaixo'.
>
> - `Espero que o jornalista continue o seu trabalho, com as forças
redobradas! E que a `grande mídia' preste mais atenção, pois este tipo
de atitude, que é no fundo uma coerção, ocorrendo quase que
diariamente sobre os jornalistas por este País a fora! La Passionaria
em seu discurso de despedida da fantástica Brigada Internacional:
`...Vocês podem sentir-se orgulhosos. Vocês são a História. São a
lenda. São o exemplo heróico da solidariedade e da universalidade da
democracia. Mais ainda, soldados do mais alto ideal de redenção humana...'
>
> - `O Diretor das Organizações Romulo Maiorana, Ronaldo Maiorana que
vem a ser o Presidente do Diretório Regional do PL, estava cercado de
PM's que fazem a sua segurança. Foi um ato covarde, está registrado em
filme amador e quem quiser saber de maiores detalhes é só acessar a
página do Diário do Pará (http://www.diariodopara.com.br). A família
Maiorana tem história no Pará...'
>
> - `Sou leitor do jornal do Lúcio e acredito que seja a fonte mais
fidedigna de informações que conheço em meu Estado. Lamento que quem
fala a verdade mereça castigo.'
>
>
> __________________________________________________
> Converse com seus amigos em tempo real com o Yahoo! Messenger
> http://br.download.yahoo.com/messenger/
>
> [As partes desta mensagem que não continham texto foram removidas]





SUBJECT: Re: Mensagem com o meu endereço
FROM: Maria Natália <grasdic@hotmail.com>
TO: ciencialist@yahoogrupos.com.br
DATE: 26/01/2005 04:06



José:
Foi um horror e quem me valeu foi precisamente meu filho informático.
me põs no Mozilla. Tá muito melhor.
Tempos dificeis estes.
Para que servem os filhos? LOLLLLL
Um abraço
Maria Natália


--- Em ciencialist@yahoogrupos.com.br, "Jose Colucci Jr. \(Home\)"
<j.colucci@r...> escreveu
> Esta lista (e várias outras) pode ter recebido uma ou mais mensagens
> contendo vírus com o meu endereço de e-mail no remetente. Posso garantir
> que não sairam do meu computador. Como tenho um único endereço de e-mail
> há vários anos, uso todo o tipo de proteção contra vírus e hacking. A
> rede da minha casa está protegida por um firewall e tenho um software
> que monitora toda a atividade de e-mail, me alertando no caso de envio
> não autorizado. A empresa onde trabalho conta com proteção semelhante,
> ou melhor.
>
> Parece ser o caso de um vírus tipo Klez, que envia e-mails com o
> endereço falso a partir da lista de endereços do Outlook. Alguém tinha o
> meu nome em sua lista. Não há muito que eu possa fazer a respeito, a não
> ser alertar a todos contra o perigo de abrir "attachments" e aconselhar
> o uso de antivirus e de toda a proteção possível. As dicas abaixo são do
> Elton Carvalho, que também me alertou para o problema:
>
>
> -------------------------------------------------------
> O computador se torna bem mais vulnerável a esse tipo de ataque quando
> se faz uso do software Microsoft Outlook, vítima predileta dos
> criadores desse tipo de software mal intencionado.
>
> Sugiro que seja feita uma vertificação contra vírus em seu computador,
> com a atualização de seu software antivirus (normalmente feita de
> maneira gratuita pela internet). Caso não possua um software
> antivírus, sugiro que este seja instalado com a máxima urgência. Um
> software antvírus gratuito e de boa qualidade pode ser encontrado em
> http://free.grisoft.com/freeweb.php/doc/2/ .
>
> É improtante também manter seu sistema atualizado, visitando
> quinzenalmente o site www.windowsupdate.com e fazendo todas as
> atualizações necessárias de segurança e mantendo sempre a veresão mais
> recente dos softwares Microsoft Outlook e Microsoft Internet Explorer.
>
> Entretanto, sugiro veementemente que se evite o uso do Microsoft
> Outlook como gerenciador de e-mails, tendo em vista que a maioria dos
> vírus e worms mais recentes o têm como seu alvo principal. Utilizar um
> outro sotware para a mesma função, como o Mozilla Thunderbird
> (http://www.mozilla.org/products/thunderbird/ ) é altamente
> recomendado.
>
> Para uma segurançamais completa e maior controle do seu trafego de
> internet, sugiro também a substituição do browser Internet Explroer,
> também vítima de diversos programas mal intencionados, pelo Mozilla
> FireFox, bem mais seguro e com desenvolvimento mais rápido, além de
> possuir dezenas de recursos adicionais. O FireFox pode ser encontrado
> em (http://www.mozilla.org/products/firefox/ ) .





SUBJECT: Re: Mulheres: Cozinha e na cama, já!!!!!!!!!!
FROM: Maria Natália <grasdic@hotmail.com>
TO: ciencialist@yahoogrupos.com.br
DATE: 26/01/2005 04:15


As letras dessa página são horrorosas em tamanho e daquilo que já li
num dos comentários achei que o fulano, Celso salvo erro, deve ter
problemas com a mãezinha dele.
O documento seguiu para ser analisado no mestrado sobre mulheres e
ainda para aulas de educação sexual.
Como ando cheiiiinha de trabalho só poderei ler no Carnaval quando
estiver em isolamento no Alentejo a trabalhar com o telescóipio.
Realmente não gramo Carnaval.
Obrigada pelos endereços
Maria Natália

--- Em ciencialist@yahoogrupos.com.br, "Rodrigo Marques"
<rodmarq72@y...> escreveu
>
>
> --- Em ciencialist@yahoogrupos.com.br, "Alvaro Augusto \(E\)"
> <alvaro@e...> escreveu
>
> "E, embora os homens e mulheres de hoje em dia não sejam em nada
> diferentes dos homens e mulhreres do neolítico, há uma pequena
> diferença: tecnologia. Devido à tecnologia as mulheres podem, se
> assim o quiserem, evitar ter um filho por ano durante a idade
> fértil. Devido à tecnologia, as mulheres podem fazer tudo o que os
> homens fazem, inclusive dirigir máquinas pesadas (direção
> hidráulica!), pilotar aviões, dirigir empresas, lecionar, etc. As
> mulheres podem, em resumo, colocar comida na mesa, tornando-se
> caçadoras, e não mais agricultoras. "
>
> Você esqueceu de dizer que grande parte dessa tecnologia (senão toda
> ela) que facilita a vida das mulheres dando-lhes conforto e fazendo
> com que elas possam competir em pé de igualdade com os homens foi
> criada pelos próprios homens.
> Veja o link:
> http://inforum.insite.com.br/jroliveira/857780.html





SUBJECT: Re: Mulheres: Cozinha e na cama, já!!!!!!!!!!
FROM: "rmtakata" <rmtakata@altavista.net>
TO: ciencialist@yahoogrupos.com.br
DATE: 26/01/2005 08:33


--- Em ciencialist@yahoogrupos.com.br, "JVictor" <jvoneto@u...>
> É evidente que esse Summers tem algum problema de identidade sexual.

Bem, pra mim nao eh tao evidente assim.

Houve uma tempestada em uma piscina. O reitor de Harvard foi
certamente infeliz, mas o q. ele quer dizer eh perfeitamente
compreensivel e de certo modo defensavel.

Ele, ao q. parece, jamais quis dizer q. as mulheres devem ser vetadas
de praticar ciencia. E nem q. mulheres nao possam ser grandes
cientistas. Temos exemplos de grandes mulheres de Ciencia - eu
clamaria pela Marie Curie como o exemplo mais bem acabado (mas temos
muitas outras).

Mas inegavelmente ha' mais homens em ciencia do q. mulheres - mesmo
nesta lista q nao eh propriamente uma atividade cientifica, mas aqui
se pretende discutir ciencias.

Sim, sim, sem duvida fatores como o machismo afetam enormemente a
participacao feminina em diversos campos, *inclusive* na ciencia. (Ou
seria *sobretudo* na ciencia?)

Mas ele tem razao ao dizer q. meninas sistematicamente tiram menores
notas em matematica e ciencias do q. meninos em testes padronizados -
eqto elas se saem, de modo geral, melhores em testes de expressao
linguistica. Pode-se clamar q. isso seria uma repeticao da infeliz da
tese de The Bell Curve, seria se examinassemos apenas isso, sem isolar
as condicoes de criacao. Porem, as diferencas persistem - claro, com
boa atenuacao - se isolarmos o fator de criacao. Outros testes
cognitivos indicam uma diferenca no funcionamento e na propria
estrutura cerebral.

Sim, as condicoes mudaram desde o tempo das cavernas. Mas
aparentemente os tempos remotos deixaram sua marca evolutiva na
diferenciacao neurologica entre homens e mulheres. Quer dizer q.
inerentemente as mulheres sao intelectualmente mais fracas do q. os
homens? Nao. A serem verdadeiras as diferencas acima e ser verdade q.
elas refletem acoes diferenciais de genes ligadas ao sexo, as mulheres
seriam, em media, melhores escritoras do q. os homens (e ficamos tb
com a pergunta: por q. ha' mais homens escritores do q. mulheres
escritoras - ao menos entre os mais editados e premiados? - aqui deve
operar mais fortemente o fator historico-ambiental. E justamente
retrucariam: oras, se no mundo intelectual literario, o fator
ambiental deve pesar mais, por q. seria diferente no mundo intelectual
cientifico? Uma resposta possivel: nao eh essencialmente diferente.
Como admitido acima, o fator ambiental certamente tem um peso enorme
tb no mundo cientifico, apenas q. aparentemente tb pode haver
diferencas geneticas funcionando contra as mulheres.)

Reparem o uso das expressoes no condicional. Esses 'ses' significam o
q (se e' verdade q. as mulheres tEm desempenho pior do q. os homens em
ciencia, se e' verdade q. essa diferenca e' genetica...)? Significam
q. e' necessario um estudo mais aprofundado. Mas tb significam q. tais
hipoteses nao sao tao disparatadas.

Mesmo q. houvesse uma implicacao normativa - e nao ha' nenhuma q. se
derive unica e exclusivamente disso - do tipo: mulheres nao devem
fazer ciencia, nao haveriamos de rejeitar a possibilidade de
veracidade disso. Muitas pessoas ainda rejeitam a evolucao biologica
com base em argumentos supostamente morais: diminuiria a condicao
sagrada do humano.

Porem, nao ha' essa implicacao normativa, nem na teoria da evolucao
biologica, nem na hipotese genetica da diferenca da participacao entre
homens e mulheres nas ciencias. O proprio reitor de Harvard se diz
defensor de medidas q. retirem as dificuldades ambientais - q. ele
admite q. existem - do acesso das mulheres ao mundo cientifico.
Retirando-se essas barreiras, as mulheres sao capazes de aumentar
ainda mais sua contribuicao ao conhecimento cientifico - a despeito de
eventuais diferencas cognitivas inatas (afinal, mesmo mulheres
possuindo uma menor habilidade media na percepcao espacial do q. os
homens, elas sao melhores motoristas do q. os homens - o valor a ser
pago para o seguro eh menor: mulheres se envolvem menos em acidentes
do q. os homens - isso porq. outras caracteristicas ao volante q.
diferenciam os sexos fazem a compensacao: homens sao bem menos
cuidadosos e mais agressivos na direcao; o mesmo pode se dar em
ciencia. As mulheres possuindo uma tendencia maior do q. os homens em
ver o todo - de modo sintetico e nao analitico -, por exemplo, podem
dar grandes contribuicoes 'a ciencia a seu modo.)

[]s,

Roberto Takata





SUBJECT: Firefox, mistério para ignorantes, mistério
FROM: JVictor <jvoneto@uol.com.br>
TO: "ciencialist@yahoogrupos.com.br" <ciencialist@yahoogrupos.com.br>
DATE: 26/01/2005 10:53

Help, Firefox

Instalalei o Firefox em duas máquinas em meu escritório. Tudo bem, estão
funcionando. Na terceira máquina, aconteceu algo inusitado, pelo me nos
para mim. Ao iniciar o Setup, caixa de mensagem:
Bem Vindo à instalação do Netscape 7.0! Mas o logo da caixa continuou
sendo o do Firefox!
Fui ao site do Firefox, e baixei novamente, tanto o tal quanto o
Thunderbird 1.0. Tentei instalá-los.
Qual o quê! Mesmas estranhezas acima citadas, para ambos!
Entrei num pesquisador para ver se havia o Netscape entalado em algum
lugar, atrapalhando. Nada.
Bem, meu nível de conhecimento informático é tal que: help! Alguém
atinaria para o mister?
Também, consultei um dos técnicos de software que nos assessora, e a
resposta foi: para mim isso é novo, não sei, vou verificar. Pelo menos a
ignorância não é só minha. Grande consolo!
Desde já agradeço.

Sds,

Victor.




SUBJECT: Re: orguloso do saber?
FROM: Manuel Bulcão <manuelbulcao@uol.com.br>
TO: ciencialist@yahoogrupos.com.br
DATE: 26/01/2005 12:04


--- Em ciencialist@yahoogrupos.com.br, "murilo filo"
<avalanchedrive@h...> escreveu

> Dá até nojo esta coisa de filho de burguês e de diplomata, não dá?
É tudo viado!

Manuel: Discordo. Não tenho preconceitos deste tipo. Também não acho
que quem não sabe falar fluentemente e com floreados o "ingrês" seja
gente bundinha. Apenas acho que se está supervalorizando -- i.e.,
valorizando mais que o devido -- a cultura "formal", que, sem
dúvida, é importante.

Sinceramente, ao Rui Barbosa (beletrista insosso que levou o Banco
do Brasil à falência) prefiro o Chico Mendes com um bom intérprete
do lado.

Ademais, a prova de inglês não foi eliminada do concurso para o
Itamarati, apenas deixou de ser eliminatória. No curso preparatório
posterior, o noviço é treinado nesta língua.

[]s
Manuel Bulcão






SUBJECT: Re: [ciencialist] Re: orguloso do saber?
FROM: ÿffffcdtalo Rocha <imrochaguedes@yahoo.com.br>
TO: ciencialist@yahoogrupos.com.br
DATE: 26/01/2005 12:16

Meu caro senhor,
Alguém que escreve da forma que o senhor faz deve ser ou estar profundamente revoltado com o que quer que seja, pelo que se depreende de suas mensagens, talvez pelo fato de não conseguir financiamento de projetos ou que publiquem trabalhos seus. Creio que isto é razoavelmente comum para qualquer um que trabalha com pesquisa, entretanto não vejo porque destratar as pessoas da forma extremamente vulgar como o senhor faz. Respondendo à sua pergunta, sou engenheiro agrônomo com mestrado em Fitotecnia (Produção Vegetal) pela Universidade Federal de Viçosa, Minas Gerais, e estou agora na fase final de um doutorado em Ciência do Solo na mesma instituição, por quê? Imagino que pensou que falei sem conhecimento de causa, sem conhecer a guerra de egos envolvida na carreira acadêmica e na frustração que isso causa, no descaso em relação ao financiamento de pesquisas, na falta de compreensão do público e dos governos quanto à importância da pesquisa científica e tudo o mais. Parece-me que talvez
seja essa a razão de sua revolta ou talvez eu esteja enganado, na verdade não me importo muito. Se for isto, também eu tenho as mesmas opiniões e é uma das razões por ter entrado em um fórum como este, mas de forma alguma pretendo dar patadas como o senhor tão competentemente faz.
Ítalo Moraes Rocha Guedes.


---------------------------------
Yahoo! Acesso Grátis - Internet rápida e grátis. Instale o discador do Yahoo! agora.

[As partes desta mensagem que não continham texto foram removidas]



SUBJECT: Re: [ciencialist] Re: Mensagem com o meu endereço
FROM: "Luiz Ferraz Netto" <leobarretos@uol.com.br>
TO: <ciencialist@yahoogrupos.com.br>
DATE: 26/01/2005 12:29

Maria Natália

>>José:
[ ... ] Para que servem os filhos? <<

Para que serve um bebê?
(Faraday)

Léo


--
Internal Virus Database is out-of-date.
Checked by AVG Anti-Virus.
Version: 7.0.300 / Virus Database: 265.6.13 - Release Date: 16/01/2005



SUBJECT: Re: [ciencialist] Re: Mulheres: Cozinha e na cama, já!!!!!!!!!!
FROM: "Luiz Ferraz Netto" <leobarretos@uol.com.br>
TO: <ciencialist@yahoogrupos.com.br>
DATE: 26/01/2005 12:31

"rmtakata"
[ ... ]
>> Ele, ao q. parece, jamais quis dizer q. as mulheres devem ser vetadas
de praticar ciencia. E nem q. mulheres nao possam ser grandes
cientistas. Temos exemplos de grandes mulheres de Ciencia - eu
clamaria pela Marie Curie como o exemplo mais bem acabado (mas temos
muitas outras). <<<

Cite mais 3!
[]'
Léo




--
Internal Virus Database is out-of-date.
Checked by AVG Anti-Virus.
Version: 7.0.300 / Virus Database: 265.6.13 - Release Date: 16/01/2005



SUBJECT: Re: [ciencialist] Re: orguloso do saber?
FROM: ÿffffcdtalo Rocha <imrochaguedes@yahoo.com.br>
TO: ciencialist@yahoogrupos.com.br
DATE: 26/01/2005 12:32

Meu caro senhor "Rayfisica", entendo perfeitamente sua indignação (educada) quanto ao que escrevi em relação ao esforço dos leigos em entender a ciência. Embora realmente ache que seja uma minoria os que realmente fazem um esforço para compreender os métodos e resultados da prática científica, quero crer que isto é resultado mais da péssima situação educacional de nosso país do que por má-vontade pura e simples. Provavelmente escrevi aquilo contaminado pela ignorância com que o outro senhor (Taborda) respondeu a alguém que tentava justamente entrar na discussão. Tenho profundo interesse na divulgação da ciência e do pensamento humanista, razão pela qual participo deste fórum e de outros dedicados principalmente a isto. O que não suporto é vulgaridade nem falta de respeito, porque imagino que em um fórum destes tratamos com pessoas civilizadas. Além do livro que lhe indiquei (o nome completo do autor é Brian L. Silver, um físico-químico inglês, o livro custa em torno de R$ 60,00 e é
facilmente comprado pela internet) há um excelente artigo do falecido Stephen Jay Gould publicado na Science em 2000, se não me engano no mês de maio, e que rebate de forma brilhante as críticas geralmente feitas à ciência e ao método científico.
Cordialmente, Ítalo Moraes Rocha Guedes.


---------------------------------
Yahoo! Acesso Grátis - Internet rápida e grátis. Instale o discador do Yahoo! agora.

[As partes desta mensagem que não continham texto foram removidas]



SUBJECT: Earth’s magnetic field is becoming generally weaker at an astonishing rate
FROM: "marcelomjr" <marcelomjr@yahoo.com.br>
TO: ciencialist@yahoogrupos.com.br
DATE: 26/01/2005 12:42


http://www.physorg.com/news2792.html

Focus on our magnetic planet


January 24, 2005

(...)

The Earth's magnetic field is becoming generally weaker at an
astonishing rate. When a French-Danish team compared Ørsted's results
for 2000 with those from an American satellite, Magsat, 20 years
earlier, the decline in the field's strength suggested that it might
disappear completely in a thousand years or so. The experts wonder if
our planet is preparing to swap its north and south magnetic poles
around, as it has often done before during the Earth's long history.
(...)





SUBJECT: Re: Mulheres: Cozinha e na cama, já!!!!!!!!!!
FROM: "rmtakata" <rmtakata@altavista.net>
TO: ciencialist@yahoogrupos.com.br
DATE: 26/01/2005 12:58


--- Em ciencialist@yahoogrupos.com.br, "Luiz Ferraz Netto"
> Cite mais 3!

Rosalyn Franklin, Lynn Margulis, Caroline Herschell, Rosalyn Sussman
Yalow...

http://www.factmonster.com/spot/whmbios2.html

[]s,

Roberto Takata





SUBJECT: Re: Mensagem com o meu endereço
FROM: "rmtakata" <rmtakata@altavista.net>
TO: ciencialist@yahoogrupos.com.br
DATE: 26/01/2005 13:00


--- Em ciencialist@yahoogrupos.com.br, "Luiz Ferraz Netto"
> Maria Natália
>
> >>José:
> [ ... ] Para que servem os filhos? <<
>
> Para que serve um bebê?
> (Faraday)

Para que serve um ser humano?

[]s,

Roberto Takata





SUBJECT: Re: [ciencialist] Re: Mulheres: Cozinha e na cama, já!!!!!!!!!!
FROM: "Alvaro Augusto \(E\)" <alvaro@electraenergy.com.br>
TO: <ciencialist@yahoogrupos.com.br>
DATE: 26/01/2005 13:12

Maria Mayer, Lise Meitner, Jane Goodall, Shiung Wu, Jill Tarter, Sofia Kovalevski, Hipácia de Alexandria...

e até algumas em uma área que tentou ser ciência, como Anna Freud e Melanie Klein.

[ ]s

Alvaro Augusto

----- Original Message -----
From: Luiz Ferraz Netto
To: ciencialist@yahoogrupos.com.br
Sent: Wednesday, January 26, 2005 12:31 PM
Subject: Re: [ciencialist] Re: Mulheres: Cozinha e na cama, já!!!!!!!!!!


"rmtakata"
[ ... ]
>> Ele, ao q. parece, jamais quis dizer q. as mulheres devem ser vetadas
de praticar ciencia. E nem q. mulheres nao possam ser grandes
cientistas. Temos exemplos de grandes mulheres de Ciencia - eu
clamaria pela Marie Curie como o exemplo mais bem acabado (mas temos
muitas outras). <<<

Cite mais 3!
[]'
Léo




--
Internal Virus Database is out-of-date.
Checked by AVG Anti-Virus.
Version: 7.0.300 / Virus Database: 265.6.13 - Release Date: 16/01/2005



##### ##### #####

Para saber mais visite
http://www.ciencialist.hpg.ig.com.br


##### ##### ##### #####


Yahoo! Grupos, um serviço oferecido por:







------------------------------------------------------------------------------
Links do Yahoo! Grupos

a.. Para visitar o site do seu grupo na web, acesse:
http://br.groups.yahoo.com/group/ciencialist/

b.. Para sair deste grupo, envie um e-mail para:
ciencialist-unsubscribe@yahoogrupos.com.br

c.. O uso que você faz do Yahoo! Grupos está sujeito aos Termos do Serviço do Yahoo!.



[As partes desta mensagem que não continham texto foram removidas]



SUBJECT: Re: [ciencialist] Re: Mensagem com o meu endereço
FROM: "Alvaro Augusto \(E\)" <alvaro@electraenergy.com.br>
TO: <ciencialist@yahoogrupos.com.br>
DATE: 26/01/2005 13:24

Resposta biológica: para fazer outros seres humanos;

Resposta religiosa: para manifestar a grandeza de Deus;

Resposta metafísica: para fazer essa pergunta;

Resposta socrática: defina homem!

Resposta do chimpanzé: para que possamos ter alguém de quem que rir (afinal, não existe nada mais engraçado que um macaco pelado que anda ereto!)

[ ]s

Alvaro Augusto


----- Original Message -----
From: rmtakata
To: ciencialist@yahoogrupos.com.br
Sent: Wednesday, January 26, 2005 1:00 PM
Subject: [ciencialist] Re: Mensagem com o meu endereço



--- Em ciencialist@yahoogrupos.com.br, "Luiz Ferraz Netto"
> Maria Natália
>
> >>José:
> [ ... ] Para que servem os filhos? <<
>
> Para que serve um bebê?
> (Faraday)

Para que serve um ser humano?

[]s,

Roberto Takata






[As partes desta mensagem que não continham texto foram removidas]



SUBJECT: Re: [ciencialist] Re: Mulheres: Cozinha e na cama, já!!!!!!!!!!
FROM: ÿffffcdtalo Rocha <imrochaguedes@yahoo.com.br>
TO: ciencialist@yahoogrupos.com.br
DATE: 26/01/2005 13:41

Não esquecer, no Brasil, a Johanna Dobereiner, agrônoma, responsável pela descoberta de bactérias fixadoras de nitrogênio em gramíneas e uma das responsáveis pelo fato de o Brasil ter o mais econômico plantio de soja do mundo.

rmtakata <rmtakata@altavista.net> wrote:

--- Em ciencialist@yahoogrupos.com.br, "Luiz Ferraz Netto"
> Cite mais 3!

Rosalyn Franklin, Lynn Margulis, Caroline Herschell, Rosalyn Sussman
Yalow...

http://www.factmonster.com/spot/whmbios2.html

[]s,

Roberto Takata





##### ##### #####

Para saber mais visite
http://www.ciencialist.hpg.ig.com.br


##### ##### ##### #####
Links do Yahoo! Grupos










__________________________________________________
Converse com seus amigos em tempo real com o Yahoo! Messenger
http://br.download.yahoo.com/messenger/

[As partes desta mensagem que não continham texto foram removidas]



SUBJECT: O Itamaraty e o inglês, por Renato Janine
FROM: "manuelbulcao" <manuelbulcao@uol.com.br>
TO: "ciencialist" <ciencialist@yahoogrupos.com.br>
DATE: 26/01/2005 14:55

O Itamaraty e o inglês
Por Renato Janine 22/01/2005 às 01:27



A língua inglesa continua sendo exigido e contando pontos no
exame do Itamaraty, só que não é mais eliminatório. Isto é,
deixou de ser o grande critério, numa das fases, para admitir
ou não o candidato. Mas continua contando pontos


Por Renato Janine - Filósofo e professor na USP. Autor do
livro "A sociedade contra o social – o alto custo da vida
pública no Brasil" (Companhia das Letras)


“Era só o que faltava”, assim reagiu Miriam Leitão, no
programa Bom dia Brasil de quinta-feira, 13 de janeiro, à
notícia de que a língua inglesa deixou de ser eliminatória no
concurso de ingresso para a carreira diplomática brasileira.
Um pouco antes, o ex-ministro das Relações Exteriores do
governo Fernando Henrique, o embaixador Luis Felipe Lampreia,
dizia: “o inglês é para a diplomacia como a matemática para a
engenharia”. Exageros, para dizer o mínimo.


O fato é: o Brasil tem pouco mais de mil diplomatas, enquanto
o México, país de menor população e PIB, conta com três mil.
Três vezes mais! Precisamos hoje de um grande contingente de
diplomatas altamente qualificados. E dispomos de uma das
melhores escolas de formação de diplomatas do mundo, que é o
Instituto Rio Branco.


Só que, de três mil ou mais candidatos que se têm apresentado
a cada concurso, às vezes o Rio Branco não consegue preencher
nem mesmo as cinqüenta vagas disponíveis. Isso é sinal, sem
dúvida, da má qualidade de nosso ensino. Mas, como o
Itamaraty não pode sozinho alterar tudo isso, o que cabe é
discutir se as exigências para a admissão não estavam sendo
excessivas.


Vejamos: o que se pede na prova de inglês é, disse-me o
embaixador João Almino, ex-diretor do Instituto, “pós-
proficiência”. Isto é, mais do que a proficiência. A
proficiency em inglês é medida por dois exames principais, o
ELTS e o TOEFL, um britânico e outro norte-americano, que são
muito exigentes. Quem já tentou uma bolsa para país de língua
inglesa sabe que muitos são reprovados. E é justo que assim
seja, porque para estudar fora o mínimo é que você saiba a
língua.


Lembro um exemplo clássico de exames para o Itamaraty, que
era (no espanhol) traduzir uma frase que incluía o “presunto
blanco”. Quem traduzisse por “presunto branco” estava
fora. “Blanco” é, como diz a etimologia, o alvo. “Presunto” é
presumido. “Presunto blanco” seria o alvo suposto, presumido.
Será correto reprovar alguém num vestibular com uma pegadinha
destas?


Mas será que um candidato a diplomata precisa ter mais do que
a proficiência? Vejam, não estou falando do diplomata já
formado, que precisa ter fluência total na língua mais
importante do mundo atual. Mas do candidato, que vai estudar
no Itamaraty com ótimos professores. Para entrar, não há
dúvida de que ele tem de saber ler inglês perfeitamente, e
falar e escrever bem. Para sair, precisa também saber
entender o inglês falado, pronunciá-lo muito bem e escrever
sem erro algum. Mas isso pode ser acrescentado durante o
curso, que é muito bom.


Por isso, eu estranho a reação dos jornalistas e mesmo de
alguns professores de cursos de relações internacionais, que –
pelo que disseram – pareciam estar comentando uma decisão de
só termos diplomatas que não soubessem nada de inglês! Na
verdade, o inglês continua sendo exigido e contando pontos no
exame do Itamaraty, só que não é mais eliminatório. Isto é,
deixou de ser o grande critério, numa das fases, para admitir
ou não o candidato. Mas continua contando pontos. Continua
valendo.


Portanto, só posso entender que Miriam Leitão, vários
jornalistas e mesmo professores não entenderam o que está
acontecendo.


A frase do ex-chanceler, embaixador Lampreia, representa
outra espécie de equívoco. O inglês não é, nem pode ser,
comparado à matemática! Esta ciência constitui a base de
praticamente todo pensamento científico. Sem ela, ninguém
entenderá nada de física, química, biologia. Sem ela, é
impossível ser engenheiro. Mas isso porque todo o
conhecimento que essas áreas transmitem é de base matemática.


Não é o caso do inglês. Essa língua é importantíssima. Sem
ela, é difícil hoje alguém se atualizar nas ciências. Mas
isso não quer dizer que a base do conhecimento científico,
qualquer um deles, seja o inglês. Tal idioma é apenas o
veículo no qual as comunicações se fazem. Podia ser o latim,
o tailandês, o suaili.


Em outras palavras, um engenheiro não será engenheiro sem a
matemática. A formação do engenheiro está, toda ela, baseada
na matemática. Mas a formação do diplomata está baseada em
muita coisa além do inglês.


Por exemplo, hoje o Brasil precisa de diplomatas que conheçam
patentes. Um doutor em patentes é essencial para representar
o País. Necessita também de secretários e embaixadores que
conheçam bem nossa sociedade. Precisa de pessoas bem
capacitadas em defesa e em segurança nacional.


Ou seja, nada justifica que o inglês seja o fator decisivo
para termos o melhor diplomata possível! Para as coisas
ficarem muito claras: o inglês é uma ferramenta essencial,
mas não é a base do conhecimento do diplomata – tal como a
matemática é o alicerce da ciência necessária ao engenheiro.
O inglês é fundamental para várias coisas. Primeira, para a
pessoa se atualizar. Para tanto, o inglês suficiente é o da
proficiência. Não é nem necessário pronunciar ou escutar
direito: é preciso ler. O diplomata, porém, precisa de mais
do que isso. Terá de falar, ouvir e escrever bem. Só que
essas capacidades ele pode aprimorar durante o curso.


Na verdade, o que está em jogo é outra coisa. Quando alguém
reduz a competência do diplomata a um núcleo duro que seria o
domínio da língua inglesa, ele está transformando o meio em
fim, a ferramenta em fundamento, o instrumento em essência. O
diplomata precisa, antes de mais nada, defender os interesses
e anseios do País. Para isso, ele precisa ter uma base sólida
em ciências humanas e sociais e, dependendo da área para a
qual se dirija, em economia, direito, até mesmo engenharia e
ciências físicas e biológicas. A diplomacia é o exemplo
acabado de uma competência interdisciplinar. Por isso, não há
sentido em sinalizar, para a sociedade brasileira, que
o “sine qua non” da diplomacia é o conhecimento de um idioma
estrangeiro.


Penso, em última análise, que está em jogo aqui uma
ideologia, que confunde a capacidade de se expressar nos
fóruns mundiais (de ter o que dizer) com o domínio de um
idioma (de saber como falar). Precisamos dizer. Para isso, é
bom saber falar (o inglês), mas antes disso precisamos ter o
que dizer: ter conteúdos.


Será coincidência que, no mesmo dia em que assisti na TV às
críticas às mudanças no Itamaraty, também vi nela um
economista brasileiro “do banco J. P. Morgan” pregando o
aumento de juros na reunião da semana que vem do Copom, e
mais tarde uma âncora de jornal noturno ridicularizando a
Argentina, porque nossos vizinhos só estão dispostos a pagar
o que podem pagar, de uma dívida externa que subiu
astronomicamente devido à ganância dos bancos internacionais?


Muita gente parece acreditar que só é bom, no Brasil, o que
estiver nos moldes impostos pelo Primeiro Mundo – que só é
bom o que é a “lição de casa”, concebida pelo capital
internacional. Não precisamos ser xiitas para tentar pensar
com a própria cabeça.

__________________________________________________________________________
Acabe com aquelas janelinhas que pulam na sua tela.
AntiPop-up UOL - É grátis!
http://antipopup.uol.com.br/




SUBJECT: Re: [ciencialist] Firefox, mistério para ignorantes, mistério
FROM: "Oraculo" <oraculo@atibaia.com.br>
TO: <ciencialist@yahoogrupos.com.br>
DATE: 26/01/2005 14:56

Olá Victor

Esse problema pode ser causado por arquivos "fantasmas" em sua máquina. Não, não aqueles de verdade, ectoplasmas de outro plano espiritual (se fossem, você poderia reinvidicar o premio de um milhão de dolares que o Randi dá a quem provar o sobrenatural..:-), mas os do tipo mais comum mesmo.

Ao instalar um programa em sua máquina, alguns arquivos de instalação, chaves de registro do Windows, e muitas outras coisas estranhas, podem permanecer no sistema, mesmo muito tempo depois que o próprio programa foi desinstalado.

Assim, se sua máquina em algum momento instalou o Netscape, chaves e registros, e mesmo arquivos, podem ter ficado. Se o Firefox usar arquivos de instalação com o mesmo nome, ou mesmo se utilizar do mesmo mecanismo de instalação (eles tem a mesma origem, o Netscape e o Firefox), isso pode ocorrer.

Um exemplo, o Nestcape anotou a mensagem de boas vindas de instalação em uma chave de registro, e o Firefox usa a mesma chave. Ao instalar, o Firefox lê a chave e apresenta a mensagem antiga.

Ou o Firefox usa um arquivo, que contém a mensagem e a tela de boas vindas, com o mesmo nome do antigo Netscape. Como ao instalar, em geral não se sobrescreve arquivos que já existem, ele pode ter usado o do Netscape no lugar de seu novo arquivo (muito comum com DLLs).

Mas se o Firefox instalou normalmente, não deve se preocupar muito, nada de muito ruim vai acontecer com seu sistema..:-)

Homero


----- Original Message -----
From: JVictor
To: ciencialist@yahoogrupos.com.br
Sent: Wednesday, January 26, 2005 10:53 AM
Subject: [ciencialist] Firefox, mistério para ignorantes, mistério


Help, Firefox

Instalalei o Firefox em duas máquinas em meu escritório. Tudo bem, estão
funcionando. Na terceira máquina, aconteceu algo inusitado, pelo me nos
para mim. Ao iniciar o Setup, caixa de mensagem:
Bem Vindo à instalação do Netscape 7.0! Mas o logo da caixa continuou
sendo o do Firefox!
Fui ao site do Firefox, e baixei novamente, tanto o tal quanto o
Thunderbird 1.0. Tentei instalá-los.
Qual o quê! Mesmas estranhezas acima citadas, para ambos!
Entrei num pesquisador para ver se havia o Netscape entalado em algum
lugar, atrapalhando. Nada.
Bem, meu nível de conhecimento informático é tal que: help! Alguém
atinaria para o mister?
Também, consultei um dos técnicos de software que nos assessora, e a
resposta foi: para mim isso é novo, não sei, vou verificar. Pelo menos a
ignorância não é só minha. Grande consolo!
Desde já agradeço.

Sds,

Victor.




##### ##### #####

Para saber mais visite
http://www.ciencialist.hpg.ig.com.br


##### ##### ##### #####


Yahoo! Grupos, um serviço oferecido por:
PUBLICIDADE




------------------------------------------------------------------------------
Links do Yahoo! Grupos

a.. Para visitar o site do seu grupo na web, acesse:
http://br.groups.yahoo.com/group/ciencialist/

b.. Para sair deste grupo, envie um e-mail para:
ciencialist-unsubscribe@yahoogrupos.com.br

c.. O uso que você faz do Yahoo! Grupos está sujeito aos Termos do Serviço do Yahoo!.



[As partes desta mensagem que não continham texto foram removidas]



SUBJECT: Texto de Gould p Italo (era orguloso do saber?)
FROM: "Oraculo" <oraculo@atibaia.com.br>
TO: <ciencialist@yahoogrupos.com.br>
DATE: 26/01/2005 14:59

Olá Italo

Por acaso tem mais dados sobre o artigo do Jay Gould? Gosto muito dos livros dele e da forma como escreve (embora tenha discordancias e prefira os argumentos de Dawkins..:-) e gostaria de encontrar o texto que citou.

Poderia informar algo mais, como o titulo ou a edição mais precisamente?

Um abraço.

Homero

----- Original Message -----
From: ÿffffcdtalo Rocha
To: ciencialist@yahoogrupos.com.br
Sent: Wednesday, January 26, 2005 12:32 PM
Subject: Re: [ciencialist] Re: orguloso do saber?


Meu caro senhor "Rayfisica", entendo perfeitamente sua indignação (educada) quanto ao que escrevi em relação ao esforço dos leigos em entender a ciência. Embora realmente ache que seja uma minoria os que realmente fazem um esforço para compreender os métodos e resultados da prática científica, quero crer que isto é resultado mais da péssima situação educacional de nosso país do que por má-vontade pura e simples. Provavelmente escrevi aquilo contaminado pela ignorância com que o outro senhor (Taborda) respondeu a alguém que tentava justamente entrar na discussão. Tenho profundo interesse na divulgação da ciência e do pensamento humanista, razão pela qual participo deste fórum e de outros dedicados principalmente a isto. O que não suporto é vulgaridade nem falta de respeito, porque imagino que em um fórum destes tratamos com pessoas civilizadas. Além do livro que lhe indiquei (o nome completo do autor é Brian L. Silver, um físico-químico inglês, o livro custa em torno de R$ 60,00 e é
facilmente comprado pela internet) há um excelente artigo do falecido Stephen Jay Gould publicado na Science em 2000, se não me engano no mês de maio, e que rebate de forma brilhante as críticas geralmente feitas à ciência e ao método científico.
Cordialmente, Ítalo Moraes Rocha Guedes.


---------------------------------
Yahoo! Acesso Grátis - Internet rápida e grátis. Instale o discador do Yahoo! agora.

[As partes desta mensagem que não continham texto foram removidas]



##### ##### #####

Para saber mais visite
http://www.ciencialist.hpg.ig.com.br


##### ##### ##### #####


Yahoo! Grupos, um serviço oferecido por:

São Paulo Rio de Janeiro Curitiba Porto Alegre Belo Horizonte Brasília




------------------------------------------------------------------------------
Links do Yahoo! Grupos

a.. Para visitar o site do seu grupo na web, acesse:
http://br.groups.yahoo.com/group/ciencialist/

b.. Para sair deste grupo, envie um e-mail para:
ciencialist-unsubscribe@yahoogrupos.com.br

c.. O uso que você faz do Yahoo! Grupos está sujeito aos Termos do Serviço do Yahoo!.



[As partes desta mensagem que não continham texto foram removidas]



SUBJECT: Re: O Itamaraty e o inglês, por Renato Janine
FROM: "rmtakata" <rmtakata@altavista.net>
TO: ciencialist@yahoogrupos.com.br
DATE: 26/01/2005 15:12


Poderiam reapresentar o tema de modo que seja cientificamente tratavel?

(Creio q. basta uma modificacao cosmetica. Por exemplo, sob o ponto de
vista geopolitico.)

[]s,

Roberto Takata





SUBJECT: Re: [ciencialist] Re: Mulheres: Cozinha e na cama, já!!!!!!!!!!
FROM: "Oraculo" <oraculo@atibaia.com.br>
TO: <ciencialist@yahoogrupos.com.br>
DATE: 26/01/2005 15:15

Olá

Essa (nova) polemica lembra o escandalo feito por "politicamente corretos" com o estudo sobre o estupro.

Os pesquisadores desse estudo concluiram que este era uma estratégia eficaz e funcional, na natureza, para certos grupos de mamiferos, já que aparentavam ser uma forma eficiente de produzir descendentes. Não a principal, nem a melhor, mas ligeiramente eficaz.

Os "politicamente corretos" e feministas radicais escutaram "válido" (com sentido subjetivo para o termo), no lugar de "eficaz", e crucificaram os pesquisadores e o estudo, afirmando que estavam tentando justificar o estupro, e desculpar os que o cometiam como uma forma de compulsão irressistível. Tolice completa.

Leões machos, ao tomar um grupo de leoas de outro, matam os filhotes pequenso, apra fazer com que as fêmeas entrem no cio e tenham novos filhotes, agora com seu patrimonio genético. Saber disso, estudar esse comportamento em mamiferos, significa aprovar que seres humanos matem os filhos de suas segundas esposas??? Rídiculo, não?..:-)

Se a causa da menor participação feminina na ciência, em especial nas abstratas fisica e matematica, for genética, compreender isso é a melhor forma de corrigir e estimular uma maior participação e não varrer para debaixo do tapete.

Se descobrirmos que homens tem uma, genérica, maior dificuldade com comunicação e expressão de idéias, reforçar essas habilidades, com estudos dirigidos, melhorará inclusive a transmição do conhecimento que produzirem. E, claro, é o mesmo com as diferenças (e não deficiências) genéticas em mulheres.

Nada, nada mesmo, na conclusão de que existem diferenças cognitivas entre sexos levaria a conclusão que isso deriva de inferioridade ou discriminação, ou que é motivo para impedir o estudo ou o ingresso de mulheres nas universidades e nas ciências. Seria o mesmo que afirmar que as eventuais dificuldades de linguagem e comunicação na média masculina deveriam impedir que estes estudassem linguas, história ou escrevessem livros. Uma tolice evidente.

Um abraço.

Homero (que tem uma irmã, engenheira mecânica, matematicamente brilhante e uma filha intelectualmente privilegiada..:-)



----- Original Message -----
From: rmtakata
To: ciencialist@yahoogrupos.com.br
Sent: Wednesday, January 26, 2005 8:33 AM
Subject: [ciencialist] Re: Mulheres: Cozinha e na cama, já!!!!!!!!!!



--- Em ciencialist@yahoogrupos.com.br, "JVictor" <jvoneto@u...>
> É evidente que esse Summers tem algum problema de identidade sexual.

Bem, pra mim nao eh tao evidente assim.

Houve uma tempestada em uma piscina. O reitor de Harvard foi
certamente infeliz, mas o q. ele quer dizer eh perfeitamente
compreensivel e de certo modo defensavel.

Ele, ao q. parece, jamais quis dizer q. as mulheres devem ser vetadas
de praticar ciencia. E nem q. mulheres nao possam ser grandes
cientistas. Temos exemplos de grandes mulheres de Ciencia - eu
clamaria pela Marie Curie como o exemplo mais bem acabado (mas temos
muitas outras).

Mas inegavelmente ha' mais homens em ciencia do q. mulheres - mesmo
nesta lista q nao eh propriamente uma atividade cientifica, mas aqui
se pretende discutir ciencias.

Sim, sim, sem duvida fatores como o machismo afetam enormemente a
participacao feminina em diversos campos, *inclusive* na ciencia. (Ou
seria *sobretudo* na ciencia?)

Mas ele tem razao ao dizer q. meninas sistematicamente tiram menores
notas em matematica e ciencias do q. meninos em testes padronizados -
eqto elas se saem, de modo geral, melhores em testes de expressao
linguistica. Pode-se clamar q. isso seria uma repeticao da infeliz da
tese de The Bell Curve, seria se examinassemos apenas isso, sem isolar
as condicoes de criacao. Porem, as diferencas persistem - claro, com
boa atenuacao - se isolarmos o fator de criacao. Outros testes
cognitivos indicam uma diferenca no funcionamento e na propria
estrutura cerebral.

Sim, as condicoes mudaram desde o tempo das cavernas. Mas
aparentemente os tempos remotos deixaram sua marca evolutiva na
diferenciacao neurologica entre homens e mulheres. Quer dizer q.
inerentemente as mulheres sao intelectualmente mais fracas do q. os
homens? Nao. A serem verdadeiras as diferencas acima e ser verdade q.
elas refletem acoes diferenciais de genes ligadas ao sexo, as mulheres
seriam, em media, melhores escritoras do q. os homens (e ficamos tb
com a pergunta: por q. ha' mais homens escritores do q. mulheres
escritoras - ao menos entre os mais editados e premiados? - aqui deve
operar mais fortemente o fator historico-ambiental. E justamente
retrucariam: oras, se no mundo intelectual literario, o fator
ambiental deve pesar mais, por q. seria diferente no mundo intelectual
cientifico? Uma resposta possivel: nao eh essencialmente diferente.
Como admitido acima, o fator ambiental certamente tem um peso enorme
tb no mundo cientifico, apenas q. aparentemente tb pode haver
diferencas geneticas funcionando contra as mulheres.)

Reparem o uso das expressoes no condicional. Esses 'ses' significam o
q (se e' verdade q. as mulheres tEm desempenho pior do q. os homens em
ciencia, se e' verdade q. essa diferenca e' genetica...)? Significam
q. e' necessario um estudo mais aprofundado. Mas tb significam q. tais
hipoteses nao sao tao disparatadas.

Mesmo q. houvesse uma implicacao normativa - e nao ha' nenhuma q. se
derive unica e exclusivamente disso - do tipo: mulheres nao devem
fazer ciencia, nao haveriamos de rejeitar a possibilidade de
veracidade disso. Muitas pessoas ainda rejeitam a evolucao biologica
com base em argumentos supostamente morais: diminuiria a condicao
sagrada do humano.

Porem, nao ha' essa implicacao normativa, nem na teoria da evolucao
biologica, nem na hipotese genetica da diferenca da participacao entre
homens e mulheres nas ciencias. O proprio reitor de Harvard se diz
defensor de medidas q. retirem as dificuldades ambientais - q. ele
admite q. existem - do acesso das mulheres ao mundo cientifico.
Retirando-se essas barreiras, as mulheres sao capazes de aumentar
ainda mais sua contribuicao ao conhecimento cientifico - a despeito de
eventuais diferencas cognitivas inatas (afinal, mesmo mulheres
possuindo uma menor habilidade media na percepcao espacial do q. os
homens, elas sao melhores motoristas do q. os homens - o valor a ser
pago para o seguro eh menor: mulheres se envolvem menos em acidentes
do q. os homens - isso porq. outras caracteristicas ao volante q.
diferenciam os sexos fazem a compensacao: homens sao bem menos
cuidadosos e mais agressivos na direcao; o mesmo pode se dar em
ciencia. As mulheres possuindo uma tendencia maior do q. os homens em
ver o todo - de modo sintetico e nao analitico -, por exemplo, podem
dar grandes contribuicoes 'a ciencia a seu modo.)

[]s,

Roberto Takata





##### ##### #####

Para saber mais visite
http://www.ciencialist.hpg.ig.com.br


##### ##### ##### #####


Yahoo! Grupos, um serviço oferecido por:







------------------------------------------------------------------------------
Links do Yahoo! Grupos

a.. Para visitar o site do seu grupo na web, acesse:
http://br.groups.yahoo.com/group/ciencialist/

b.. Para sair deste grupo, envie um e-mail para:
ciencialist-unsubscribe@yahoogrupos.com.br

c.. O uso que você faz do Yahoo! Grupos está sujeito aos Termos do Serviço do Yahoo!.



[As partes desta mensagem que não continham texto foram removidas]



SUBJECT: Re: [ciencialist] Re: Mensagem com o meu endereço
FROM: "Luiz Ferraz Netto" <leobarretos@uol.com.br>
TO: <ciencialist@yahoogrupos.com.br>
DATE: 26/01/2005 16:20

> [ ... ] Para que servem os filhos? <<
>
> Para que serve um bebê?
> (Faraday)

>> Para que serve um ser humano?

Essa é fácil!
Para ter filhos! É simplesmente a manutenção da espécie; a perpetuação do plasma. Nas horas vagas, guerreiam um com outros (para impedir que 'os outros' proliferem).

[]'
Léo


--
Internal Virus Database is out-of-date.
Checked by AVG Anti-Virus.
Version: 7.0.300 / Virus Database: 265.6.13 - Release Date: 16/01/2005



SUBJECT: Re: [ciencialist] Re: Mulheres: Cozinha e na cama, já!!!!!!!!!!
FROM: "Luiz Ferraz Netto" <leobarretos@uol.com.br>
TO: <ciencialist@yahoogrupos.com.br>
DATE: 26/01/2005 16:23

Ei pessoal,

vcs estão citando apenas 'cantoras de roquinrol'? Perguntei por 3 exemplos nivelados com M. Curie.
Freud e Klein --- não tem algo a ver com as cuecas? :-)

[]'Léo

===========================
Luiz Ferraz Netto [Léo]
leobarretos@uol.com.br
http://www.feiradeciencias.com.br
===========================
-----Mensagem Original-----
De: "Alvaro Augusto (E)" <alvaro@electraenergy.com.br>
Para: <ciencialist@yahoogrupos.com.br>
Enviada em: quarta-feira, 26 de janeiro de 2005 13:12
Assunto: Re: [ciencialist] Re: Mulheres: Cozinha e na cama, já!!!!!!!!!!



Maria Mayer, Lise Meitner, Jane Goodall, Shiung Wu, Jill Tarter, Sofia Kovalevski, Hipácia de Alexandria...

e até algumas em uma área que tentou ser ciência, como Anna Freud e Melanie Klein.

[ ]s

Alvaro Augusto

----- Original Message -----
From: Luiz Ferraz Netto
To: ciencialist@yahoogrupos.com.br
Sent: Wednesday, January 26, 2005 12:31 PM
Subject: Re: [ciencialist] Re: Mulheres: Cozinha e na cama, já!!!!!!!!!!


"rmtakata"
[ ... ]
>> Ele, ao q. parece, jamais quis dizer q. as mulheres devem ser vetadas
de praticar ciencia. E nem q. mulheres nao possam ser grandes
cientistas. Temos exemplos de grandes mulheres de Ciencia - eu
clamaria pela Marie Curie como o exemplo mais bem acabado (mas temos
muitas outras). <<<

Cite mais 3!
[]'
Léo







--
Internal Virus Database is out-of-date.
Checked by AVG Anti-Virus.
Version: 7.0.300 / Virus Database: 265.6.13 - Release Date: 16/01/2005



SUBJECT: Re: [ciencialist] Re: Mulheres: Cozinha e na cama, já!!!!!!!!!!
FROM: "Luiz Ferraz Netto" <leobarretos@uol.com.br>
TO: <ciencialist@yahoogrupos.com.br>
DATE: 26/01/2005 16:25

Tá bom, tá bom,

... vamos comer grama. :-)

>Não esquecer, no Brasil, a Johanna Dobereiner, agrônoma, responsável pela descoberta de bactérias fixadoras de nitrogênio em gramíneas e uma das responsáveis pelo fato de o Brasil ter o mais econômico plantio de soja do mundo.

>>Rosalyn Franklin, Lynn Margulis, Caroline Herschell, Rosalyn Sussman
Yalow...



--
Internal Virus Database is out-of-date.
Checked by AVG Anti-Virus.
Version: 7.0.300 / Virus Database: 265.6.13 - Release Date: 16/01/2005



SUBJECT: Re: Mensagem com o meu endereço
FROM: "rmtakata" <rmtakata@altavista.net>
TO: ciencialist@yahoogrupos.com.br
DATE: 26/01/2005 16:32


--- Em ciencialist@yahoogrupos.com.br, "Luiz Ferraz Netto"
> >> Para que serve um ser humano?
>
> Essa é fácil!
> Para ter filhos!

Nessa linha Dawkins diria algo como: seres humanos sao um modo q. os
genes humanos encontraram para propagar genes humanos.

Bem, entao o q. dizer dos q. nao pretendem se reproduzir? (Estereis,
vasectomizados, abstemios convictos - todos sabem q. padres catolicos
e freiras cristas nao fazem sexo -, homossexuais e quetais?)

[]s,

Roberto Takata





SUBJECT: Re: [ciencialist] Re: Mulheres: Cozinha e na cama, já!!!!!!!!!!
FROM: "Luiz Ferraz Netto" <leobarretos@uol.com.br>
TO: <ciencialist@yahoogrupos.com.br>
DATE: 26/01/2005 16:37


>Homero
(que tem uma irmã, engenheira mecânica, matematicamente brilhante e uma filha intelectualmente privilegiada..:-)<

Se ela (a irmã) for solteira, viuva, divorciada, separada etc., aceito as duas (irmã/filha) para tomarem conta de mim. :-)
A frase deixou-me em dúvida sobre de quem é a 'filha'; é sua ou de sua irmã?
Ah! Cientificamente falando.

[]'
Léo



--
Internal Virus Database is out-of-date.
Checked by AVG Anti-Virus.
Version: 7.0.300 / Virus Database: 265.6.13 - Release Date: 16/01/2005



SUBJECT: Re: Mulheres: Cozinha e na cama, já!!!!!!!!!!
FROM: "rmtakata" <rmtakata@altavista.net>
TO: ciencialist@yahoogrupos.com.br
DATE: 26/01/2005 16:40


--- Em ciencialist@yahoogrupos.com.br, "Luiz Ferraz Netto"
> Perguntei por 3 exemplos nivelados com M. Curie.

Niveladas com Marie Currie seria o mesmo q. pedir pra listar qq humano
nivelado com Leonardo da Vinci. Eu listei grandes mulheres de ciencia
- soberbas mulheres de ciencia sao poucas, assim como soberbos homens
de ciencia tb sao poucos.

Claro q. ha' mais homens de ciencia de alto nivel do q. mulheres de
ciencia - mas isso em boa parte eh em funcao das ciencias serem um
clube do Bolinhas (por um ou outro motivo).

[]s,

Roberto Takata





SUBJECT: Re: [ciencialist] Re: Mulheres: Cozinha e na cama, já!!!!!!!!!!
FROM: "Alvaro Augusto \(E\)" <alvaro@electraenergy.com.br>
TO: <ciencialist@yahoogrupos.com.br>
DATE: 26/01/2005 16:46

Bem, não sou profundo conhecedor do assunto, mas me lembro que as gramíneas incluem a cana-de-açúcar, um produto que tem certa importância na economia brasileira...

[ ]s

Alvaro Augusto

----- Original Message -----
From: Luiz Ferraz Netto
To: ciencialist@yahoogrupos.com.br
Sent: Wednesday, January 26, 2005 4:25 PM
Subject: Re: [ciencialist] Re: Mulheres: Cozinha e na cama, já!!!!!!!!!!


Tá bom, tá bom,

... vamos comer grama. :-)

>Não esquecer, no Brasil, a Johanna Dobereiner, agrônoma, responsável pela descoberta de bactérias fixadoras de nitrogênio em gramíneas e uma das responsáveis pelo fato de o Brasil ter o mais econômico plantio de soja do mundo.

>>Rosalyn Franklin, Lynn Margulis, Caroline Herschell, Rosalyn Sussman
Yalow...





[As partes desta mensagem que não continham texto foram removidas]



SUBJECT: Re: [ciencialist] Re: Mulheres: Cozinha e na cama, já!!!!!!!!!!
FROM: "Oraculo" <oraculo@atibaia.com.br>
TO: <ciencialist@yahoogrupos.com.br>
DATE: 26/01/2005 16:49

Olá Leo

He, he, he..:-) Cientificamente falando, é minha filha, mas muito nova ainda para cuidar de marmanjos (pais são sempre cegos para filhas que crescem..:-) E a irmã, infelizmente, é casada (infelizmente no bom sentido..:-)

Um abraço.

Homero


----- Original Message -----
From: Luiz Ferraz Netto
To: ciencialist@yahoogrupos.com.br
Sent: Wednesday, January 26, 2005 4:37 PM
Subject: Re: [ciencialist] Re: Mulheres: Cozinha e na cama, já!!!!!!!!!!



>Homero
(que tem uma irmã, engenheira mecânica, matematicamente brilhante e uma filha intelectualmente privilegiada..:-)<

Se ela (a irmã) for solteira, viuva, divorciada, separada etc., aceito as duas (irmã/filha) para tomarem conta de mim. :-)
A frase deixou-me em dúvida sobre de quem é a 'filha'; é sua ou de sua irmã?
Ah! Cientificamente falando.

[]'
Léo



--
Internal Virus Database is out-of-date.
Checked by AVG Anti-Virus.
Version: 7.0.300 / Virus Database: 265.6.13 - Release Date: 16/01/2005



##### ##### #####

Para saber mais visite
http://www.ciencialist.hpg.ig.com.br


##### ##### ##### #####


Yahoo! Grupos, um serviço oferecido por:

São Paulo Rio de Janeiro Curitiba Porto Alegre Belo Horizonte Brasília




------------------------------------------------------------------------------
Links do Yahoo! Grupos

a.. Para visitar o site do seu grupo na web, acesse:
http://br.groups.yahoo.com/group/ciencialist/

b.. Para sair deste grupo, envie um e-mail para:
ciencialist-unsubscribe@yahoogrupos.com.br

c.. O uso que você faz do Yahoo! Grupos está sujeito aos Termos do Serviço do Yahoo!.



[As partes desta mensagem que não continham texto foram removidas]



SUBJECT: Re: [ciencialist] Re: Mensagem com o meu endereço
FROM: "Luiz Ferraz Netto" <leobarretos@uol.com.br>
TO: <ciencialist@yahoogrupos.com.br>
DATE: 26/01/2005 16:56

Takata: > >> Para que serve um ser humano?
>
Léo: > Essa é fácil! > Para ter filhos!

Roberto Takata: >>Nessa linha Dawkins diria algo como: seres humanos sao um modo q. os
genes humanos encontraram para propagar genes humanos.

Bem, entao o q. dizer dos q. nao pretendem se reproduzir? (Estereis,
vasectomizados, abstemios convictos - todos sabem q. padres catolicos
e freiras cristas nao fazem sexo -, homossexuais e quetais?)<<

Eu diria que é muita sorte da 'evolução da espécie' que os citados não queiram/possam se reproduzir!

[]'
Léo




--
Internal Virus Database is out-of-date.
Checked by AVG Anti-Virus.
Version: 7.0.300 / Virus Database: 265.6.13 - Release Date: 16/01/2005



SUBJECT: Re: [ciencialist] Re: Mulheres: Cozinha e na cama, já!!!!!!!!!!
FROM: "E m i l i a n o C h e m e l l o" <chemelloe@yahoo.com.br>
TO: <ciencialist@yahoogrupos.com.br>
DATE: 26/01/2005 16:56

PessoALL,

Recomendo a leitura do livro "A ciência é Masculina? É sim Senhora!" do meu amigo Áttico Chassot. Caso haja interesse da lista (Brudna, é contigo), posso convidar o autor para uma discussão sobre o assunto.

Mais detalhes: http://www.if.ufrgs.br/cafe-sci/chassot.doc

[ ] 's do Emiliano Chemello
emiliano@quimica.net
http://www.quimica.net/emiliano
http://www.ucs.br/ccet/defq/naeq

" Rien ne se perd, rien ne se crée,
tout se transforme."

Antoine Laurent de Lavoisier (químico francês, 1743 - 1794)

----- Original Message -----
From: rmtakata
To: ciencialist@yahoogrupos.com.br
Sent: Wednesday, January 26, 2005 4:40 PM
Subject: [ciencialist] Re: Mulheres: Cozinha e na cama, já!!!!!!!!!!



--- Em ciencialist@yahoogrupos.com.br, "Luiz Ferraz Netto"
> Perguntei por 3 exemplos nivelados com M. Curie.

Niveladas com Marie Currie seria o mesmo q. pedir pra listar qq humano
nivelado com Leonardo da Vinci. Eu listei grandes mulheres de ciencia
- soberbas mulheres de ciencia sao poucas, assim como soberbos homens
de ciencia tb sao poucos.

Claro q. ha' mais homens de ciencia de alto nivel do q. mulheres de
ciencia - mas isso em boa parte eh em funcao das ciencias serem um
clube do Bolinhas (por um ou outro motivo).

[]s,

Roberto Takata





##### ##### #####

Para saber mais visite
http://www.ciencialist.hpg.ig.com.br


##### ##### ##### #####


Yahoo! Grupos, um serviço oferecido por:







------------------------------------------------------------------------------
Links do Yahoo! Grupos

a.. Para visitar o site do seu grupo na web, acesse:
http://br.groups.yahoo.com/group/ciencialist/

b.. Para sair deste grupo, envie um e-mail para:
ciencialist-unsubscribe@yahoogrupos.com.br

c.. O uso que você faz do Yahoo! Grupos está sujeito aos Termos do Serviço do Yahoo!.



[As partes desta mensagem que não continham texto foram removidas]



SUBJECT: Re: Mulheres: Cozinha e na cama, já!!!!!!!!!!
FROM: "rmtakata" <rmtakata@altavista.net>
TO: ciencialist@yahoogrupos.com.br
DATE: 26/01/2005 16:57


--- Em ciencialist@yahoogrupos.com.br, "rmtakata" <rmtakata@a...>
> Niveladas com Marie Currie seria o mesmo q. pedir pra listar qq
> humano nivelado com Leonardo da Vinci.

Mas listemos as mulheres q. ganharam o Nobel em uma area cientifica:

Irene Joliot-Curie 1935 - Quimica
Gerty Theresa Cori 1947 - Medicina
Maria Goeppert-Mayer 1963 - Fisica
Dorothy Crowfoot Hodgkin 1964 - Quimica
Rosalyn Sussman Yalow 1977 - Medicina (ja' mencionada anteriormente)
Barbara McClintock 1983 - Medicina
Rita Levi-Montalcini 1986 - Medicina
Gertrude Elion 1988 - Medicina
Christiane Nusslein-Volhard 1995 - Medicina
Linda B. Buck 2004 - Medicina

(Claro q. em qq listagem sempre cometemos injusticas. Mas nao poderia
ter deixado de mencionar Irene Curie [filha dos Curie - talento tb eh
hereditario] e Gerty Cori.)

[]s,

Roberto Takata





SUBJECT: Re: orguloso do saber?
FROM: "Sergio M. M. Taborda" <sergiotaborda@terra.com.br>
TO: ciencialist@yahoogrupos.com.br
DATE: 26/01/2005 16:59



A minha aspereza - ou como perferem chamar alguns: mal educação - ao
comentar o texto do Oráculo, no assunto em epigrafe , deve-se ao facto
de eu não encontrar no texto do ray nenhuma mensão ao método
cientifico, à estutura teoria da ciencia, à metafisica da ciencia, ou
ha ciencia em si. Apenas encontro mensões a cientistas, pessoas que
fazem, ou querem fazer ciencia, e pssoas que entendem ou querem
entender ciencia. Portanto o assunto, quanto a mim são as pessoas- os
conluios, arrogancias, egos e auto-embromações, tipicas do ser humano
- e não a ciencia em si. O ppr titulo é impeditivo que se trate da
ciencia, pois a ciencia não é uma pessoa, não tem orgulho.

O foco era o orgulho das pessoas que fazem ciencia: os cientistas e
onde isso interfere com o bom funcionamento da ciencia.

cito: "Os cientistas de hoje tem razão de ficarem orgulhosos do saber
que possuem ou que possam possuir?"

o ray estende-se pela impossibilidade de uma pessoa normal, um leigo,
entender um especialista.

"Tomando por base as ultimas comunicações vejo a distancia entre os
cientistas e o vulgo, parece-me que é impossível à pessoa
comum entender patavina do que falam os doutos."

Aqui eu entendi a palavra doutos com sentido prejurativo de
arrogancia. E foi isso que eu comentei , que os cientistas sim são
arrogantes no que toca À defesa das suas teorias, e o mainsteam é
arrogante até ao ponto em que organiza a intervensão dos especialistas
de tal forma a escluir todos os que não seguem as suas ideias. E mais
do que isso, os especialistas negam-se a explicar em palavras simples
as suas teorias , que pensam eles ser tão boas, ao comum dos mortais.

O ray continua comparando a ciencia com a religião , mas do ponto de
vista das pessoa e não dos métodos. Ele compara realmente os
cientistas com os religiosos idolatras que enviavam para a fogueira
quem não concordava com eles ou não idolatrava os mesmos idolos. Pondo
em causa, enfim, a hipotetica solidez dos conhecimentos que temos
hoje, já que são baseados em esquemas de medo, e imposição de autoridade.

Depois , ray lança vários exemplos em que levanta a questão de que os
especialistas são obrigados a seguir a mainstream, a bitola, a regra,
o acordo comum, ou estão fadados à extinção. E pior que isso, os
cientistas nessas consições sentem que isso é normal, e que se não
fosse assim, algo estaria errado.

"Conversando um dia com um professor sobre uma duvida, após algum
tempo ele virou para mim, talvez até para se livrar de mim e
disse, eu já estou muito bitolado (eu particularmente não quero ser
um bitolado)."


" Com certeza não sou douto, porem com mais certeza não sou um
sacerdote bitolado."

Com esta frase, reforça-se mais uma vez o foco nas pessoas e não na
ciencia como um campo do saber, ou no seu metodo de trabalho. São as
pessoas que estão bitoladas e por isso a sua produção cientifica é
mediocre.

Depois, o Ray finaliza com mais uma supeita de que as teorias que são
aceites hoje, que os cientistas bitolados são obrigados a seguir,
podem ser tão verdadeiras como a teoria do ether ou do calorico de ha
seculos atrás. A diferença é que seculos atráz havia liberdade de
escolher, hoje não ha.

" Penso que há um enorme castelo construído sobre um alicerce
aparentemente frágil, fico preocupado se o alicerce cair o que
sustentara o castelo, e é lógico que quem habita o castelo esta
tranqüilo, tranqüilo."

E agora eu pergunto ao Oraculo e ao Italo, o que este texto tem a
haver com o método cientifico e toda a panóplia de construções
intelectuais em que a ciencia, como um campo do saber , é baseada ? E
pq vcs insitem em que esse é o tema da discussão ?


Sérgio Taborda





SUBJECT: Re: [ciencialist] Firefox, mistério para ignoran tes, mistério
FROM: "Itabajara Vaz - UFRGS" <ita@cbiot.ufrgs.br>
TO: ciencialist@yahoogrupos.com.br
DATE: 26/01/2005 17:07

ola,
da uma olhada neste grupo de discussao




ita


On 26 Jan 2005 at 14:56, Oraculo wrote:

>
> Olá Victor
>
> Esse problema pode ser causado por arquivos "fantasmas" em sua máquina. Não, não aqueles de verdade, ectoplasmas de outro plano espiritual (se fossem, você poderia reinvidicar o premio de um milhão de dolares que o Randi dá a quem provar o sobrenatural..:-), mas os do tipo mais comum mesmo.
>
> Ao instalar um programa em sua máquina, alguns arquivos de instalação, chaves de registro do Windows, e muitas outras coisas estranhas, podem permanecer no sistema, mesmo muito tempo depois que o próprio programa foi desinstalado.
>
> Assim, se sua máquina em algum momento instalou o Netscape, chaves e registros, e mesmo arquivos, podem ter ficado. Se o Firefox usar arquivos de instalação com o mesmo nome, ou mesmo se utilizar do mesmo mecanismo de instalação (eles tem a mesma origem, o Netscape e o Firefox), isso pode ocorrer.
>
> Um exemplo, o Nestcape anotou a mensagem de boas vindas de instalação em uma chave de registro, e o Firefox usa a mesma chave. Ao instalar, o Firefox lê a chave e apresenta a mensagem antiga.
>
> Ou o Firefox usa um arquivo, que contém a mensagem e a tela de boas vindas, com o mesmo nome do antigo Netscape. Como ao instalar, em geral não se sobrescreve arquivos que já existem, ele pode ter usado o do Netscape no lugar de seu novo arquivo (muito comum com DLLs).
>
> Mas se o Firefox instalou normalmente, não deve se preocupar muito, nada de muito ruim vai acontecer com seu sistema..:-)
>
> Homero
>
>
> ----- Original Message -----
> From: JVictor
> To: ciencialist@yahoogrupos.com.br
> Sent: Wednesday, January 26, 2005 10:53 AM
> Subject: [ciencialist] Firefox, mistério para ignorantes, mistério
>
>
> Help, Firefox
>
> Instalalei o Firefox em duas máquinas em meu escritório. Tudo bem, estão
> funcionando. Na terceira máquina, aconteceu algo inusitado, pelo me nos
> para mim. Ao iniciar o Setup, caixa de mensagem:
> Bem Vindo à instalação do Netscape 7.0! Mas o logo da caixa continuou
> sendo o do Firefox!
> Fui ao site do Firefox, e baixei novamente, tanto o tal quanto o
> Thunderbird 1.0. Tentei instalá-los.
> Qual o quê! Mesmas estranhezas acima citadas, para ambos!
> Entrei num pesquisador para ver se havia o Netscape entalado em algum
> lugar, atrapalhando. Nada.
> Bem, meu nível de conhecimento informático é tal que: help! Alguém
> atinaria para o mister?
> Também, consultei um dos técnicos de software que nos assessora, e a
> resposta foi: para mim isso é novo, não sei, vou verificar. Pelo menos a
> ignorância não é só minha. Grande consolo!
> Desde já agradeço.
>
> Sds,
>
> Victor.
>
>
>
>
> ##### ##### #####
>
> Para saber mais visite
> http://www.ciencialist.hpg.ig.com.br
>
>
> ##### ##### ##### #####
>
>
> Yahoo! Grupos, um serviço oferecido por:
> PUBLICIDADE
>
>
>
>
> ------------------------------------------------------------------------------
> Links do Yahoo! Grupos
>
> a.. Para visitar o site do seu grupo na web, acesse:
> http://br.groups.yahoo.com/group/ciencialist/
>
> b.. Para sair deste grupo, envie um e-mail para:
> ciencialist-unsubscribe@yahoogrupos.com.br
>
> c.. O uso que você faz do Yahoo! Grupos está sujeito aos Termos do Serviço do Yahoo!.
>
>
>
> [As partes desta mensagem que não continham texto foram removidas]
>
>
>
> ##### ##### #####
>
> Para saber mais visite
> http://www.ciencialist.hpg.ig.com.br
>
>
> ##### ##### ##### #####
> Links do Yahoo! Grupos
>
>
>
>
>
>
>
>




[As partes desta mensagem que não continham texto foram removidas]



SUBJECT: Re: Mulheres: Cozinha e na cama, já!!!!!!!!!!
FROM: "rmtakata" <rmtakata@altavista.net>
TO: ciencialist@yahoogrupos.com.br
DATE: 26/01/2005 17:20


--- Em ciencialist@yahoogrupos.com.br, "Alvaro Augusto \(E\)"
> Bem, não sou profundo conhecedor do assunto, mas me lembro que as
> gramíneas incluem a cana-de-açúcar, um produto que tem certa
> importância na economia brasileira...

Alem do milho, trigo, arroz, painço, aveia e - o q. deve trazer o
interesse do Leo - cevada! (Ainda q. a maior parte da cerveja seja
feita de trigo e milho mesmo.)

As gramíneas são a base de alimentação humana: seja diretamente (o q.
inclui derivados como o pao), seja indiretamente (como ração para
gado: capim e farelo de milho).

[]s,

Roberto Takata





SUBJECT: Re: [ciencialist] Firefox, mistério para i CORRECAO
FROM: "Itabajara Vaz - UFRGS" <ita@cbiot.ufrgs.br>
TO: ciencialist@yahoogrupos.com.br
DATE: 26/01/2005 17:27

ola,
da uma olhada neste grupo de discussao...
(a outra mensagem estava errada)

Para visitar o site do seu grupo na web, acesse:
http://br.groups.yahoo.com/group/MozillaBR-Users/



ita




On 26 Jan 2005 at 16:07, Itabajara Vaz - UFRGS wrote:

>
> ola,
> da uma olhada neste grupo de discussao
>
>
>
>
> ita
>
>
> On 26 Jan 2005 at 14:56, Oraculo wrote:
>
> >
> > Olá Victor
> >
> > Esse problema pode ser causado por arquivos "fantasmas" em sua máquina. Não, não aqueles de verdade, ectoplasmas de outro plano espiritual (se fossem, você poderia reinvidicar o premio de um milhão de dolares que o Randi dá a quem provar o sobrenatural..:-), mas os do tipo mais comum mesmo.
> >
> > Ao instalar um programa em sua máquina, alguns arquivos de instalação, chaves de registro do Windows, e muitas outras coisas estranhas, podem permanecer no sistema, mesmo muito tempo depois que o próprio programa foi desinstalado.
> >
> > Assim, se sua máquina em algum momento instalou o Netscape, chaves e registros, e mesmo arquivos, podem ter ficado. Se o Firefox usar arquivos de instalação com o mesmo nome, ou mesmo se utilizar do mesmo mecanismo de instalação (eles tem a mesma origem, o Netscape e o Firefox), isso pode ocorrer.
> >
> > Um exemplo, o Nestcape anotou a mensagem de boas vindas de instalação em uma chave de registro, e o Firefox usa a mesma chave. Ao instalar, o Firefox lê a chave e apresenta a mensagem antiga.
> >
> > Ou o Firefox usa um arquivo, que contém a mensagem e a tela de boas vindas, com o mesmo nome do antigo Netscape. Como ao instalar, em geral não se sobrescreve arquivos que já existem, ele pode ter usado o do Netscape no lugar de seu novo arquivo (muito comum com DLLs).
> >
> > Mas se o Firefox instalou normalmente, não deve se preocupar muito, nada de muito ruim vai acontecer com seu sistema..:-)
> >
> > Homero
> >
> >
> > ----- Original Message -----
> > From: JVictor
> > To: ciencialist@yahoogrupos.com.br
> > Sent: Wednesday, January 26, 2005 10:53 AM
> > Subject: [ciencialist] Firefox, mistério para ignorantes, mistério
> >
> >
> > Help, Firefox
> >
> > Instalalei o Firefox em duas máquinas em meu escritório. Tudo bem, estão
> > funcionando. Na terceira máquina, aconteceu algo inusitado, pelo me nos
> > para mim. Ao iniciar o Setup, caixa de mensagem:
> > Bem Vindo à instalação do Netscape 7.0! Mas o logo da caixa continuou
> > sendo o do Firefox!
> > Fui ao site do Firefox, e baixei novamente, tanto o tal quanto o
> > Thunderbird 1.0. Tentei instalá-los.
> > Qual o quê! Mesmas estranhezas acima citadas, para ambos!
> > Entrei num pesquisador para ver se havia o Netscape entalado em algum
> > lugar, atrapalhando. Nada.
> > Bem, meu nível de conhecimento informático é tal que: help! Alguém
> > atinaria para o mister?
> > Também, consultei um dos técnicos de software que nos assessora, e a
> > resposta foi: para mim isso é novo, não sei, vou verificar. Pelo menos a
> > ignorância não é só minha. Grande consolo!
> > Desde já agradeço.
> >
> > Sds,
> >
> > Victor.
> >
> >
> >
> >
> > ##### ##### #####
> >
> > Para saber mais visite
> > http://www.ciencialist.hpg.ig.com.br
> >
> >
> > ##### ##### ##### #####
> >
> >
> > Yahoo! Grupos, um serviço oferecido por:
> > PUBLICIDADE
> >
> >
> >
> >
> > ------------------------------------------------------------------------------
> > Links do Yahoo! Grupos
> >
> > a.. Para visitar o site do seu grupo na web, acesse:
> > http://br.groups.yahoo.com/group/ciencialist/
> >
> > b.. Para sair deste grupo, envie um e-mail para:
> > ciencialist-unsubscribe@yahoogrupos.com.br
> >
> > c.. O uso que você faz do Yahoo! Grupos está sujeito aos Termos do Serviço do Yahoo!.
> >
> >
> >
> > [As partes desta mensagem que não continham texto foram removidas]
> >
> >
> >
> > ##### ##### #####
> >
> > Para saber mais visite
> > http://www.ciencialist.hpg.ig.com.br
> >
> >
> > ##### ##### ##### #####
> > Links do Yahoo! Grupos
> >
> >
> >
> >
> >
> >
> >
> >
>
>
>
>
> [As partes desta mensagem que não continham texto foram removidas]
>
>
>
> ##### ##### #####
>
> Para saber mais visite
> http://www.ciencialist.hpg.ig.com.br
>
>
> ##### ##### ##### #####
> Links do Yahoo! Grupos
>
>
>
>
>
>
>
>




[As partes desta mensagem que não continham texto foram removidas]



SUBJECT: Re: Mensagem com o meu endereço
FROM: "rmtakata" <rmtakata@altavista.net>
TO: ciencialist@yahoogrupos.com.br
DATE: 26/01/2005 17:30


--- Em ciencialist@yahoogrupos.com.br, "Luiz Ferraz Netto"
> Bem, entao o q. dizer dos q. nao pretendem se reproduzir? (Estereis,
> vasectomizados, abstemios convictos - todos sabem q. padres
> catolicos e freiras cristas nao fazem sexo -, homossexuais e
> quetais?)<<
>
> Eu diria que é muita sorte da 'evolução da espécie' que os citados
> não queiram/possam se reproduzir!

Bem, para a sorte da evolucao das especies, eu estou no meio do grupo
citado: nao pretendo ter filhos (se os tiver serao adotivos).

Mas para a infelicidade da sociedade humana caras como Alan Turing nao
deixaram descendentes (ele se inclui entre os homossexuais na lista
acima, assim como Oscar Wilde, Zelia Duncan, Gertrude Stein e, dizem,
Santos Dumont e Leonardo da Vinci).

Entre os padres temos Antonio Vieira.

Felizmente a filiacao intelectual nao necessita de filiacao sanguinea
- infelizmente, esta ultima facilita enormemente a primeira.

[]s,

Roberto Takata





SUBJECT: Re: orguloso do saber?
FROM: "rayfisica" <rayfisica@yahoo.com.br>
TO: ciencialist@yahoogrupos.com.br
DATE: 26/01/2005 17:33


--- Em ciencialist@yahoogrupos.com.br, "Sergio M. M. Taborda"
<sergiotaborda@t...> escreveu
>
>
> - e não a ciencia em si. O ppr titulo é impeditivo que se
trate da
> ciencia, pois a ciencia não é uma pessoa, não tem orgulho.
>
> O foco era o orgulho das pessoas que fazem ciencia: os cientistas e
> onde isso interfere com o bom funcionamento da ciencia.
>
> cito: "Os cientistas de hoje tem razão de ficarem orgulhosos do
saber
>


Exatamente isso.
Parece-me, senhores que estabelecemos um grande fosso entre nos...
E agora não conseguimos nem nos entender, não critico a
ciência, não
de modo algum eu a amo...
Penso que a ciência é a única coisa que pode dar algum
significado as
nossas vidas enquanto seres pensantes, pois ou fazemos amor e
ciência
ou então nos destruiremos fazendo guerra e religião, eu em...

ah, eu tambem mudei para o firefox e bird(alguma coisa)





SUBJECT: Interplanetary "Day After Tomorrow?"
FROM: "marcelomjr" <marcelomjr@yahoo.com.br>
TO: ciencialist@yahoogrupos.com.br
DATE: 26/01/2005 17:35


http://www.enterprisemission.com/_articles/05-14-
2004_Interplanetary_Part_1/Interplanetary_1.htm

Interplanetary "Day After Tomorrow?"

Part 1

An Enterprise Mission Hyperdimensional Report

Richard C. Hoagland
David Wilcock

Copyright © 2004

The entire solar system - not just our one small planet -- is
currently undergoing profound, never-before-seen physical changes.
This paper will address and scientifically document a wide variety of
significant examples, drawing from a host of published mainstream
sources.

(...)





SUBJECT: Re: aranha "papa-mosca"
FROM: "rmtakata" <rmtakata@altavista.net>
TO: ciencialist@yahoogrupos.com.br
DATE: 26/01/2005 18:01


Tem veneno (como todas as aranhas) e eh mortal... para as presas delas
- como as moscas.

Em humanos eh pouco efetivo. No Brasil, devemos nos preocupar (em
termos de saude) apenas com a aranha-marrom, a armadeira e a
viuva-negra (ou o nome q. tenham na regiao).

http://www.butantan.gov.br/hvb_ut.htm

[]s,

Roberto Takata

--- Em ciencialist@yahoogrupos.com.br, "E m i l i a n o C h e m e l
> mais aranhas... Takata! É contigo! :-)
>
> [ ] 's do Emiliano Chemello

> Contato Naeq:
> Email: CLEBER-MENDES@U...
> Telefone: arranhas
> Mensagem: queria saber se as papa-mosca tem veneno e se mortal





SUBJECT: Re: Mulheres: Cozinha e na cama, já!!!!!!!!!!
FROM: "rmtakata" <rmtakata@altavista.net>
TO: ciencialist@yahoogrupos.com.br
DATE: 26/01/2005 18:34


Não li, mas parece ser interessante:

4000 years of women in science
http://www.astr.ua.edu/4000WS/4000WS.html

http://www.astr.ua.edu/4000WS/summary.shtml

[]s,

Roberto Takata





SUBJECT: Re: [ciencialist] Re: Mulheres: Cozinha e na cama, já!!!!!!!!!!
FROM: "Alvaro Augusto \(E\)" <alvaro@electraenergy.com.br>
TO: <ciencialist@yahoogrupos.com.br>
DATE: 26/01/2005 19:29

Biografia ultraresumida de algumas dessas "roqueiras":

- Maria Mayer - Nobel em 1963 por trabalhos sobre estrutura dos núcleos atômicos;

- Lise Meitner - prêmio Enrico Fermi de 1966 por trabalhos sobre fissão nuclear;

- Jane Goodall - primatologista, ganhadora de 23 doutorados honorários e mais de 50 condecorações por trabalhos sobre chimpanzés, incluindo "Dame of the British Empire";

- Shiung Wu - trabalhou no projeto Manhattan; conhecida como a "primeira dama da física", ou "Marie Curie chinesa"; Nobel de física em 1978;

- Sofia Kovalevsky - matemática famosa por ter ganho o prêmio Bordin da Academia Francesa de Ciências, em 1888, com um trabalho sobre rotações de um corpo sólido assimétrico em torno de um ponto fixo (pião de Kovalevsky);

Se, ao fim da vida, eu tiver conseguido realizar 10% do que cada uma dessas mulheres conseguiu, poderei me dar por satisfeito.

[ ]s

Alvaro Augusto



----- Original Message -----
From: Luiz Ferraz Netto
To: ciencialist@yahoogrupos.com.br
Sent: Wednesday, January 26, 2005 4:23 PM
Subject: Re: [ciencialist] Re: Mulheres: Cozinha e na cama, já!!!!!!!!!!


Ei pessoal,

vcs estão citando apenas 'cantoras de roquinrol'? Perguntei por 3 exemplos nivelados com M. Curie.
Freud e Klein --- não tem algo a ver com as cuecas? :-)

[]'Léo

===========================
Luiz Ferraz Netto [Léo]
leobarretos@uol.com.br
http://www.feiradeciencias.com.br
===========================
-----Mensagem Original-----
De: "Alvaro Augusto (E)" <alvaro@electraenergy.com.br>
Para: <ciencialist@yahoogrupos.com.br>
Enviada em: quarta-feira, 26 de janeiro de 2005 13:12
Assunto: Re: [ciencialist] Re: Mulheres: Cozinha e na cama, já!!!!!!!!!!



Maria Mayer, Lise Meitner, Jane Goodall, Shiung Wu, Jill Tarter, Sofia Kovalevski, Hipácia de Alexandria...

e até algumas em uma área que tentou ser ciência, como Anna Freud e Melanie Klein.

[ ]s

Alvaro Augusto

----- Original Message -----
From: Luiz Ferraz Netto
To: ciencialist@yahoogrupos.com.br
Sent: Wednesday, January 26, 2005 12:31 PM
Subject: Re: [ciencialist] Re: Mulheres: Cozinha e na cama, já!!!!!!!!!!


"rmtakata"
[ ... ]
>> Ele, ao q. parece, jamais quis dizer q. as mulheres devem ser vetadas
de praticar ciencia. E nem q. mulheres nao possam ser grandes
cientistas. Temos exemplos de grandes mulheres de Ciencia - eu
clamaria pela Marie Curie como o exemplo mais bem acabado (mas temos
muitas outras). <<<

Cite mais 3!
[]'
Léo







--
Internal Virus Database is out-of-date.
Checked by AVG Anti-Virus.
Version: 7.0.300 / Virus Database: 265.6.13 - Release Date: 16/01/2005



##### ##### #####

Para saber mais visite
http://www.ciencialist.hpg.ig.com.br


##### ##### ##### #####


Yahoo! Grupos, um serviço oferecido por:








Links do Yahoo! Grupos

Para visitar o site do seu grupo na web, acesse:
http://br.groups.yahoo.com/group/ciencialist/

Para sair deste grupo, envie um e-mail para:
ciencialist-unsubscribe@yahoogrupos.com.br

O uso que você faz do Yahoo! Grupos está sujeito aos Termos do Serviço do Yahoo!.

[As partes desta mensagem que não continham texto foram removidas]



SUBJECT: Re: [ciencialist] Re: Mulheres: Cozinha e na cama, já!!!!!!!!!!
FROM: JVictor <jvoneto@uol.com.br>
TO: ciencialist@yahoogrupos.com.br
DATE: 26/01/2005 19:35


E E.Noether.

Sds,

Victor.


rmtakata escreveu:

>
> --- Em ciencialist@yahoogrupos.com.br, "Luiz Ferraz Netto"
> > Cite mais 3!
>
> Rosalyn Franklin, Lynn Margulis, Caroline Herschell, Rosalyn Sussman
> Yalow...
>
> http://www.factmonster.com/spot/whmbios2.html
>
> []s,
>
> Roberto Takata
>
>
>
>
>
> ##### ##### #####
>
> Para saber mais visite
> http://www.ciencialist.hpg.ig.com.br
>
>
> ##### ##### ##### #####
>
>
> *Yahoo! Grupos, um serviço oferecido por:*
>
> *
> <http://br.rd.yahoo.com/SIG=12af23din/M=264105.3931087.6562589.1588051/D=brclubs/S=2137111528:HM/EXP=1106837925/A=2361264/R=6/SIG=10v4acpp0/*http://br.shopping.yahoo.com/>*
>
>
>
> ------------------------------------------------------------------------
> *Links do Yahoo! Grupos*
>
> * Para visitar o site do seu grupo na web, acesse:
> http://br.groups.yahoo.com/group/ciencialist/
>
> * Para sair deste grupo, envie um e-mail para:
> ciencialist-unsubscribe@yahoogrupos.com.br
> <mailto:ciencialist-unsubscribe@yahoogrupos.com.br?subject=Unsubscribe>
>
> * O uso que você faz do Yahoo! Grupos está sujeito aos Termos do
> Serviço do Yahoo! <http://br.yahoo.com/info/utos.html>.
>
>




SUBJECT: Re: [ciencialist] Firefox, mistério para ignorantes, mistério
FROM: JVictor <jvoneto@uol.com.br>
TO: ciencialist@yahoogrupos.com.br
DATE: 26/01/2005 19:42


Olá, Homero,


Grato pelo retorno. Acabei não indo em frente com a instalação. Mas
amanhã farei isso. Faz sentido o que você informou.

Sds,

Victor.






Oraculo escreveu:

> Olá Victor
>
> Esse problema pode ser causado por arquivos "fantasmas" em sua
> máquina. Não, não aqueles de verdade, ectoplasmas de outro plano
> espiritual (se fossem, você poderia reinvidicar o premio de um milhão
> de dolares que o Randi dá a quem provar o sobrenatural..:-), mas os do
> tipo mais comum mesmo.
>
> Ao instalar um programa em sua máquina, alguns arquivos de instalação,
> chaves de registro do Windows, e muitas outras coisas estranhas, podem
> permanecer no sistema, mesmo muito tempo depois que o próprio programa
> foi desinstalado.
>
> Assim, se sua máquina em algum momento instalou o Netscape, chaves e
> registros, e mesmo arquivos, podem ter ficado. Se o Firefox usar
> arquivos de instalação com o mesmo nome, ou mesmo se utilizar do mesmo
> mecanismo de instalação (eles tem a mesma origem, o Netscape e o
> Firefox), isso pode ocorrer.
>
> Um exemplo, o Nestcape anotou a mensagem de boas vindas de instalação
> em uma chave de registro, e o Firefox usa a mesma chave. Ao instalar,
> o Firefox lê a chave e apresenta a mensagem antiga.
>
> Ou o Firefox usa um arquivo, que contém a mensagem e a tela de boas
> vindas, com o mesmo nome do antigo Netscape. Como ao instalar, em
> geral não se sobrescreve arquivos que já existem, ele pode ter usado o
> do Netscape no lugar de seu novo arquivo (muito comum com DLLs).
>
> Mas se o Firefox instalou normalmente, não deve se preocupar muito,
> nada de muito ruim vai acontecer com seu sistema..:-)
>
> Homero
>
>
> ----- Original Message -----
> From: JVictor
> To: ciencialist@yahoogrupos.com.br
> Sent: Wednesday, January 26, 2005 10:53 AM
> Subject: [ciencialist] Firefox, mistério para ignorantes, mistério
>
>
> Help, Firefox
>
> Instalalei o Firefox em duas máquinas em meu escritório. Tudo bem,
> estão
> funcionando. Na terceira máquina, aconteceu algo inusitado, pelo me nos
> para mim. Ao iniciar o Setup, caixa de mensagem:
> Bem Vindo à instalação do Netscape 7.0! Mas o logo da caixa continuou
> sendo o do Firefox!
> Fui ao site do Firefox, e baixei novamente, tanto o tal quanto o
> Thunderbird 1.0. Tentei instalá-los.
> Qual o quê! Mesmas estranhezas acima citadas, para ambos!
> Entrei num pesquisador para ver se havia o Netscape entalado em algum
> lugar, atrapalhando. Nada.
> Bem, meu nível de conhecimento informático é tal que: help! Alguém
> atinaria para o mister?
> Também, consultei um dos técnicos de software que nos assessora, e a
> resposta foi: para mim isso é novo, não sei, vou verificar. Pelo
> menos a
> ignorância não é só minha. Grande consolo!
> Desde já agradeço.
>
> Sds,
>
> Victor.
>
>
>
>
> ##### ##### #####
>
> Para saber mais visite
> http://www.ciencialist.hpg.ig.com.br
>
>
> ##### ##### ##### #####
>
>
> Yahoo! Grupos, um serviço oferecido por:
> PUBLICIDADE
>
>
>
>
> ------------------------------------------------------------------------------
> Links do Yahoo! Grupos
>
> a.. Para visitar o site do seu grupo na web, acesse:
> http://br.groups.yahoo.com/group/ciencialist/
>
> b.. Para sair deste grupo, envie um e-mail para:
> ciencialist-unsubscribe@yahoogrupos.com.br
>
> c.. O uso que você faz do Yahoo! Grupos está sujeito aos Termos do
> Serviço do Yahoo!.
>
>
>
> [As partes desta mensagem que não continham texto foram removidas]
>
>
>
> ##### ##### #####
>
> Para saber mais visite
> http://www.ciencialist.hpg.ig.com.br
>
>
> ##### ##### ##### #####
>
>
> *Yahoo! Grupos, um serviço oferecido por:*
> <http://br.rd.yahoo.com/SIG=12a174ak0/M=264379.5078783.6203979.1588051/D=brclubs/S=2137111528:HM/EXP=1106844808/A=2191897/R=0/SIG=10vqa2grn/*http://br.diversao.yahoo.com/>
>
> <http://br.rd.yahoo.com/SIG=12a174ak0/M=264379.5078783.6203979.1588051/D=brclubs/S=2137111528:HM/EXP=1106844808/A=2191897/R=1/SIG=10vqa2grn/*http://br.diversao.yahoo.com/>
>
>
>
> ------------------------------------------------------------------------
> *Links do Yahoo! Grupos*
>
> * Para visitar o site do seu grupo na web, acesse:
> http://br.groups.yahoo.com/group/ciencialist/
>
> * Para sair deste grupo, envie um e-mail para:
> ciencialist-unsubscribe@yahoogrupos.com.br
> <mailto:ciencialist-unsubscribe@yahoogrupos.com.br?subject=Unsubscribe>
>
> * O uso que você faz do Yahoo! Grupos está sujeito aos Termos do
> Serviço do Yahoo! <http://br.yahoo.com/info/utos.html>.
>
>




SUBJECT: Re: [ciencialist] Re: Mensagem com o meu endereço
FROM: JVictor <jvoneto@uol.com.br>
TO: ciencialist@yahoogrupos.com.br
DATE: 26/01/2005 19:50

rmtakata escreveu:

>
> --- Em ciencialist@yahoogrupos.com.br, "Luiz Ferraz Netto"
> > >> Para que serve um ser humano?
> >
> > Essa é fácil!
> > Para ter filhos!
>
> Nessa linha Dawkins diria algo como: seres humanos sao um modo q. os
> genes humanos encontraram para propagar genes humanos.
>
> Bem, entao o q. dizer dos q. nao pretendem se reproduzir? (Estereis,
> vasectomizados, abstemios convictos - todos sabem q. padres catolicos
> e freiras cristas nao fazem sexo -, homossexuais e quetais?)
>
> Victor: Bela sinuca de bico.
>
>
>
>
> ##### ##### #####
>
> Para saber mais visite
> http://www.ciencialist.hpg.ig.com.br
>
>
> ##### ##### ##### #####
>
>
> *Yahoo! Grupos, um serviço oferecido por:*
> PUBLICIDADE
> <http://br.rd.yahoo.com/SIG=12aobc40i/M=264379.5078783.6203979.1588051/D=brclubs/S=2137111528:HM/EXP=1106850791/A=2332652/R=0/id=noscript/SIG=119058f8i/*http://br.download.yahoo.com/messenger/>
>
>
>
> ------------------------------------------------------------------------
> *Links do Yahoo! Grupos*
>
> * Para visitar o site do seu grupo na web, acesse:
> http://br.groups.yahoo.com/group/ciencialist/
>
> * Para sair deste grupo, envie um e-mail para:
> ciencialist-unsubscribe@yahoogrupos.com.br
> <mailto:ciencialist-unsubscribe@yahoogrupos.com.br?subject=Unsubscribe>
>
> * O uso que você faz do Yahoo! Grupos está sujeito aos Termos do
> Serviço do Yahoo! <http://br.yahoo.com/info/utos.html>.
>
>




SUBJECT: Re: orguloso do saber?
FROM: "oraculo333" <oraculo@atibaia.com.br>
TO: ciencialist@yahoogrupos.com.br
DATE: 26/01/2005 23:57


Olá

"Tergiversação - ato ou efeito de tergiversar - rodeio, subterfúgio,
evasiva." Gosto dessa palavra..:-)

Mas voltando ao assunto, será um prazer responder o que tem tudo
isso ver com métido cientifico, Taborda..:-)

O ray, em que pese a interpretação alternativa sua, postou um texto
comentando a distancia entre os leigos e a ciência, sua sempre
crescente complexidade, e perguntou o que se poderia fazer a
respeito.

Minha ponderação, e de alguns outros da lista, foi que a ciência é
complexa por sua natureza, e tende sempre a aumentar essa
complexidade. Que, embora não seja pssível (nem recomendável)
diminuir isso, é possível melhorar a proximidade e a compreensão do
leigo de outras formas, que não ensinar fisica avançada ou biologia
molecular a cada um deles.

E uma forma seria o conhecimento de aspectos da ciência que a fazem
única, e confiável, como seu método, o pensamento crítico, a razão e
a lógica, etc.

Todo o ponderado na messagem abaixo, é razoável, mas dificilmente
foi o problema ou a causa do desentendimento. Como apresentado na
outra mensagem, não estavamos discutindo as pessoas que fazem
ciência, ou a crítica à erros e defeitos de sua aplicação politica
ou prática, mas debatendo soluções para a distancia entre pessoas e
o conhecimento cientifico.

É possivel notar isso simplesmente relendo a linha de mensagens com
essa thread, desde a primeira postada pelo ray.

Por último, é possível discordar de argumentos e opiniões, sem ser
áspero ou mal-educado (eu não chamaria assim, no máximo seco.:-)ao
faze-lo. Discordar de alguém não significa que este alguém é
estúpido.

E é possível argumentar que alguns cientistas são arrogantes sem
necessáriamente discordar que a maior proximidade com aspectos da
ciência beneficiaria o entendimento desta pelo público. Uma coisa
não invalida a outra, e tanto eu como os outros repetimos diversas
vezes que concordamos com esta crítica. Mas ainda insistimos que não
estavamos dizendo nenhuma bobagem..:-)

Sequencia lógica (simplificada..:-):

Ray - há uma distancia entre ciência, sua compreensão, e o leigo, o
que fazer?

Homero e outros - Sobre a complexidade pouco, mas aproximar o leigo
do funcionamento básico da ciência, em especial seu método dito
cientifico, pode ajudar.

Taborda - Tolice, nem existe consenso sobre o que é o método, nem
mesmo se existe um, que dira ensinar ao leigo.

Homero e outros - Discordo, existe um método, pode ser compreendido
e ensinado, e tem resultado em uma maior confiabilidade nos
resultados.

Taborda - E de que adianta, se não há verbas, esses *&^%^%$ que são
quem manda são uns *&*&^%$% e pensa que se faz ciência sem
dinheiro? Esses *&^%^%$^ etc.

Bem, a falta de verbas, a politicagem, o custo de fazer ciência,
etc, não são refutações para a afirmação de existencia do método
cientifico ou da necessidade de compreende-lo para melhorar a
compreensão da ciência. Alias, é uma concordancia, já que receber
verbas de pesquisa, quando possível, resultará na aplicação do
método para garantir o rigor nos resultados..:-)

Enfim, seria uma discussão proveitosa, com diversos enfoques, se não
fosse cerceada na base, com a proibição de argumentar sobre aspectos
que o Taborda não considera legítimos..:-)

Um abraço.

Homero





--- Em ciencialist@yahoogrupos.com.br, "Sergio M. M. Taborda"
<sergiotaborda@t...> escreveu
>
>
> A minha aspereza - ou como perferem chamar alguns: mal educação -
ao
> comentar o texto do Oráculo, no assunto em epigrafe , deve-se ao
facto
> de eu não encontrar no texto do ray nenhuma mensão ao método
> cientifico, à estutura teoria da ciencia, à metafisica da ciencia,
ou
> ha ciencia em si. Apenas encontro mensões a cientistas, pessoas que
> fazem, ou querem fazer ciencia, e pssoas que entendem ou querem
> entender ciencia. Portanto o assunto, quanto a mim são as pessoas-
os
> conluios, arrogancias, egos e auto-embromações, tipicas do ser
humano
> - e não a ciencia em si. O ppr titulo é impeditivo que se trate da
> ciencia, pois a ciencia não é uma pessoa, não tem orgulho.
>
> O foco era o orgulho das pessoas que fazem ciencia: os cientistas e
> onde isso interfere com o bom funcionamento da ciencia.
>
> cito: "Os cientistas de hoje tem razão de ficarem orgulhosos do
saber
> que possuem ou que possam possuir?"
>
> o ray estende-se pela impossibilidade de uma pessoa normal, um
leigo,
> entender um especialista.
>
> "Tomando por base as ultimas comunicações vejo a distancia entre
os
> cientistas e o vulgo, parece-me que é impossível à pessoa
> comum entender patavina do que falam os doutos."
>
> Aqui eu entendi a palavra doutos com sentido prejurativo de
> arrogancia. E foi isso que eu comentei , que os cientistas sim são
> arrogantes no que toca À defesa das suas teorias, e o mainsteam é
> arrogante até ao ponto em que organiza a intervensão dos
especialistas
> de tal forma a escluir todos os que não seguem as suas ideias. E
mais
> do que isso, os especialistas negam-se a explicar em palavras
simples
> as suas teorias , que pensam eles ser tão boas, ao comum dos
mortais.
>
> O ray continua comparando a ciencia com a religião , mas do ponto
de
> vista das pessoa e não dos métodos. Ele compara realmente os
> cientistas com os religiosos idolatras que enviavam para a fogueira
> quem não concordava com eles ou não idolatrava os mesmos idolos.
Pondo
> em causa, enfim, a hipotetica solidez dos conhecimentos que temos
> hoje, já que são baseados em esquemas de medo, e imposição de
autoridade.
>
> Depois , ray lança vários exemplos em que levanta a questão de que
os
> especialistas são obrigados a seguir a mainstream, a bitola, a
regra,
> o acordo comum, ou estão fadados à extinção. E pior que isso, os
> cientistas nessas consições sentem que isso é normal, e que se não
> fosse assim, algo estaria errado.
>
> "Conversando um dia com um professor sobre uma duvida, após algum
> tempo ele virou para mim, talvez até para se livrar de mim e
> disse, eu já estou muito bitolado (eu particularmente não quero
ser
> um bitolado)."
>
>
> " Com certeza não sou douto, porem com mais certeza não sou um
> sacerdote bitolado."
>
> Com esta frase, reforça-se mais uma vez o foco nas pessoas e não na
> ciencia como um campo do saber, ou no seu metodo de trabalho. São
as
> pessoas que estão bitoladas e por isso a sua produção cientifica é
> mediocre.
>
> Depois, o Ray finaliza com mais uma supeita de que as teorias que
são
> aceites hoje, que os cientistas bitolados são obrigados a seguir,
> podem ser tão verdadeiras como a teoria do ether ou do calorico de
ha
> seculos atrás. A diferença é que seculos atráz havia liberdade de
> escolher, hoje não ha.
>
> " Penso que há um enorme castelo construído sobre um alicerce
> aparentemente frágil, fico preocupado se o alicerce cair o que
> sustentara o castelo, e é lógico que quem habita o castelo esta
> tranqüilo, tranqüilo."
>
> E agora eu pergunto ao Oraculo e ao Italo, o que este texto tem a
> haver com o método cientifico e toda a panóplia de construções
> intelectuais em que a ciencia, como um campo do saber , é
baseada ? E
> pq vcs insitem em que esse é o tema da discussão ?
>
>
> Sérgio Taborda





SUBJECT: Re: Mulheres: Cozinha e na cama, já!!!!!!!!!!/Takata
FROM: Maria Natália <grasdic@hotmail.com>
TO: ciencialist@yahoogrupos.com.br
DATE: 27/01/2005 00:42


Takata:
E eu, ratona, e que trouxe aqui a polémica para a lista estou
observando. ih ih ih.
Sabes porque as astrónomas em lista de 300 pessoas são só duas?
É fácil quando não há divisão de trabalho em casa por exemplo. Sei de
meninas que nem vão a conferências depois das 20h porque tem maridinho
em casa e olha que será uma a duas vezes no ano ou uma vez por mês. No
caso da astronomia lua nova só há uma por mês.
E teria madame Curie quem lhe lavassa as fraldas em casa? Será que
Pierre Curie punha a roupa a secar? Enfim eu nem falo de inferioridade
física/mental/intelectual. Por exemplo ter hipótese de fazer
doutoramento em Harvard e ter de recusar porque se tem filho de 8
meses e sem dinheiro para ter babá? O que é? Triste.
Apesar de os homens fazerem robots para a cozinha ainda não têm
hipóteses de os comprar para equipar sua casa...sem o recurso ao
vencimento(ordenado/emprego) das mulheres. Oh pescadinha de rabo na boca.
Este problema se coloca muito mais em Ci~encia porque a Mulher de
Ciência precisa de passar mais de 59% de sua investigação em
Laboratório ou Trabalho de Campo.
Aqui nesta parte das mulheres apesar de teoricamente terem toda a
igualdade...Era esse aspecto que queria ressalvar e chamada de atenção
para os men da lista.E claro observando e concluindo.
Aliás este tema está já muito bem tratado em teses de mestrado e de
doutoramento.
Pensai no modo como estais a educar vossos filhos (rapazes e
raparigas) Pois o futuro se se diz depende doa escola que se tem
também tem a ver com a educação que os pais derem em casa: O
EXEMPLO!!! Não o blá blá.
Tá dito.
Um abraço aos querdos machões da lista
Maria Natália

--- Em ciencialist@yahoogrupos.com.br, "rmtakata" <rmtakata@a...> escreveu
>
> --- Em ciencialist@yahoogrupos.com.br, "rmtakata" <rmtakata@a...>
> > Niveladas com Marie Currie seria o mesmo q. pedir pra listar qq
> > humano nivelado com Leonardo da Vinci.
>
> Mas listemos as mulheres q. ganharam o Nobel em uma area cientifica:
>
> Irene Joliot-Curie 1935 - Quimica
> Gerty Theresa Cori 1947 - Medicina
> Maria Goeppert-Mayer 1963 - Fisica
> Dorothy Crowfoot Hodgkin 1964 - Quimica
> Rosalyn Sussman Yalow 1977 - Medicina (ja' mencionada anteriormente)
> Barbara McClintock 1983 - Medicina
> Rita Levi-Montalcini 1986 - Medicina
> Gertrude Elion 1988 - Medicina
> Christiane Nusslein-Volhard 1995 - Medicina
> Linda B. Buck 2004 - Medicina
>
> (Claro q. em qq listagem sempre cometemos injusticas. Mas nao poderia
> ter deixado de mencionar Irene Curie [filha dos Curie - talento tb eh
> hereditario] e Gerty Cori.)
>
> []s,
>
> Roberto Takata





SUBJECT: As aranhas é que a sabem toda
FROM: Maria Natália <grasdic@hotmail.com>
TO: ciencialist@yahoogrupos.com.br
DATE: 27/01/2005 00:53


Especial para os amantes das...aranhas e aranhiços:

As aranhas escolhem alimentação saudável é o que ressalta do estudo
publicado na revista Science de dia 8 de Janeiro. Lá é relatada uma
experiência com três espécies de aranhas e que provou que elas tinham
o instinto para equilibrar a alimentação.
Foi David Mayntz da Universidade de Oxford que colocou os aracnídeos
em regime de excesso de gorduras e de proteinas. Logo que as retirou
deste "castigo" elas optaram por nutrientes que equilibravam a dieta.
Será dos ditos 8 olhos? Vêem melhor? LOL
Um abraço
da teia
Maria Natália
PS isto já sem falar naquela que come o macho depois da noite de
núpcias. Para poder ser investigadora em Ciência.





SUBJECT: Re: orguloso do saber?/visão de professor
FROM: Maria Natália <grasdic@hotmail.com>
TO: ciencialist@yahoogrupos.com.br
DATE: 27/01/2005 01:31


Desculpa estar a responder aqui pois isto se refere a várias mensagens
postadas aqui sobre o tema. Vou apanhar aqui o combóio e dizer a
propósito:
"Tolice, nem existe consenso sobre o que é o método, nem
mesmo se existe um, que dira ensinar ao leigo."

Ensinar o leigo---Ora o modo de se "ensinar" ciência não é pelo blá
blá mas pelo hands on desde os 5/6 anos de idade. Ou seja, fazendo a
criança praticar e dando nós o exemplo pela nossa actuação. A Ciência
é um modo de estar em sociedade. E todo o Homem devia ter aceso
através das ESCOLAS PÚBLICAS a esse amadurecimento.
Quanto ao "método" ele não existe pelo menos há 8 anos ou mais em Pt.
As mais renitentes a deixar de ensinar "o método científico", lamento,
mas foram os colegas professores de biologia, geologia e afins. Mas
hoje já ninguém fala disso sob pena de ser lançado à fogueira.
Será através da alteração dos curricula nas escolas que se fará a
convivência pacífica entre cientistas e leigos e não cientistas.
Altera-se, aplica-se, avalia-se a qualidade e daqui a 15/16 anos
vai-se ver o resultado.
Apenas isto e que todos sabeis mas que como Brasil é babel de culturas
e coleginhos nenhum de vós tem a força suficiente para implementar.
Como aqui é mais pequenino se vai indo de mansinho e daqui a 8 anos se
verá que adultos teremos. Tanto? Pois as reformas em educação não dão
resultados de um ano para o outro como os políticos gostariam e ao
sabor do calendário eleitoral deles.
Desculpa Homero
um abraço
Maria Natália

--- Em ciencialist@yahoogrupos.com.br, "oraculo333" <oraculo@a...>
escreveu
>
> Olá
>
> "Tergiversação - ato ou efeito de tergiversar - rodeio, subterfúgio,
> evasiva." Gosto dessa palavra..:-)
>
> Mas voltando ao assunto, será um prazer responder o que tem tudo
> isso ver com métido cientifico, Taborda..:-)
>
> O ray, em que pese a interpretação alternativa sua, postou um texto
> comentando a distancia entre os leigos e a ciência, sua sempre
> crescente complexidade, e perguntou o que se poderia fazer a
> respeito.
>
> Minha ponderação, e de alguns outros da lista, foi que a ciência é
> complexa por sua natureza, e tende sempre a aumentar essa
> complexidade. Que, embora não seja pssível (nem recomendável)
> diminuir isso, é possível melhorar a proximidade e a compreensão do
> leigo de outras formas, que não ensinar fisica avançada ou biologia
> molecular a cada um deles.
>

> E uma forma seria o conhecimento de aspectos da ciência que a fazem
> única, e confiável, como seu método, o pensamento crítico, a razão e
> a lógica, etc.
>
> Todo o ponderado na messagem abaixo, é razoável, mas dificilmente
> foi o problema ou a causa do desentendimento. Como apresentado na
> outra mensagem, não estavamos discutindo as pessoas que fazem
> ciência, ou a crítica à erros e defeitos de sua aplicação politica
> ou prática, mas debatendo soluções para a distancia entre pessoas e
> o conhecimento cientifico.
>
> É possivel notar isso simplesmente relendo a linha de mensagens com
> essa thread, desde a primeira postada pelo ray.
>
> Por último, é possível discordar de argumentos e opiniões, sem ser
> áspero ou mal-educado (eu não chamaria assim, no máximo seco.:-)ao
> faze-lo. Discordar de alguém não significa que este alguém é
> estúpido.
>
> E é possível argumentar que alguns cientistas são arrogantes sem
> necessáriamente discordar que a maior proximidade com aspectos da
> ciência beneficiaria o entendimento desta pelo público. Uma coisa
> não invalida a outra, e tanto eu como os outros repetimos diversas
> vezes que concordamos com esta crítica. Mas ainda insistimos que não
> estavamos dizendo nenhuma bobagem..:-)
>
> Sequencia lógica (simplificada..:-):
>
> Ray - há uma distancia entre ciência, sua compreensão, e o leigo, o
> que fazer?
>
> Homero e outros - Sobre a complexidade pouco, mas aproximar o leigo
> do funcionamento básico da ciência, em especial seu método dito
> cientifico, pode ajudar.
>
> Taborda - Tolice, nem existe consenso sobre o que é o método, nem
> mesmo se existe um, que dira ensinar ao leigo.
>
> Homero e outros - Discordo, existe um método, pode ser compreendido
> e ensinado, e tem resultado em uma maior confiabilidade nos
> resultados.
>
> Taborda - E de que adianta, se não há verbas, esses *&^%^%$ que são
> quem manda são uns *&*&^%$% e pensa que se faz ciência sem
> dinheiro? Esses *&^%^%$^ etc.
>
> Bem, a falta de verbas, a politicagem, o custo de fazer ciência,
> etc, não são refutações para a afirmação de existencia do método
> cientifico ou da necessidade de compreende-lo para melhorar a
> compreensão da ciência. Alias, é uma concordancia, já que receber
> verbas de pesquisa, quando possível, resultará na aplicação do
> método para garantir o rigor nos resultados..:-)
>
> Enfim, seria uma discussão proveitosa, com diversos enfoques, se não
> fosse cerceada na base, com a proibição de argumentar sobre aspectos
> que o Taborda não considera legítimos..:-)
>
> Um abraço.
>
> Homero
>
>
>
>
>
> --- Em ciencialist@yahoogrupos.com.br, "Sergio M. M. Taborda"
> <sergiotaborda@t...> escreveu
> >
> >
> > A minha aspereza - ou como perferem chamar alguns: mal educação -
> ao
> > comentar o texto do Oráculo, no assunto em epigrafe , deve-se ao
> facto
> > de eu não encontrar no texto do ray nenhuma mensão ao método
> > cientifico, à estutura teoria da ciencia, à metafisica da ciencia,
> ou
> > ha ciencia em si. Apenas encontro mensões a cientistas, pessoas que
> > fazem, ou querem fazer ciencia, e pssoas que entendem ou querem
> > entender ciencia. Portanto o assunto, quanto a mim são as pessoas-
> os
> > conluios, arrogancias, egos e auto-embromações, tipicas do ser
> humano
> > - e não a ciencia em si. O ppr titulo é impeditivo que se trate da
> > ciencia, pois a ciencia não é uma pessoa, não tem orgulho.
> >
> > O foco era o orgulho das pessoas que fazem ciencia: os cientistas e
> > onde isso interfere com o bom funcionamento da ciencia.
> >
> > cito: "Os cientistas de hoje tem razão de ficarem orgulhosos do
> saber
> > que possuem ou que possam possuir?"
> >
> > o ray estende-se pela impossibilidade de uma pessoa normal, um
> leigo,
> > entender um especialista.
> >
> > "Tomando por base as ultimas comunicações vejo a distancia entre
> os
> > cientistas e o vulgo, parece-me que é impossível à pessoa
> > comum entender patavina do que falam os doutos."
> >
> > Aqui eu entendi a palavra doutos com sentido prejurativo de
> > arrogancia. E foi isso que eu comentei , que os cientistas sim são
> > arrogantes no que toca À defesa das suas teorias, e o mainsteam é
> > arrogante até ao ponto em que organiza a intervensão dos
> especialistas
> > de tal forma a escluir todos os que não seguem as suas ideias. E
> mais
> > do que isso, os especialistas negam-se a explicar em palavras
> simples
> > as suas teorias , que pensam eles ser tão boas, ao comum dos
> mortais.
> >
> > O ray continua comparando a ciencia com a religião , mas do ponto
> de
> > vista das pessoa e não dos métodos. Ele compara realmente os
> > cientistas com os religiosos idolatras que enviavam para a fogueira
> > quem não concordava com eles ou não idolatrava os mesmos idolos.
> Pondo
> > em causa, enfim, a hipotetica solidez dos conhecimentos que temos
> > hoje, já que são baseados em esquemas de medo, e imposição de
> autoridade.
> >
> > Depois , ray lança vários exemplos em que levanta a questão de que
> os
> > especialistas são obrigados a seguir a mainstream, a bitola, a
> regra,
> > o acordo comum, ou estão fadados à extinção. E pior que isso, os
> > cientistas nessas consições sentem que isso é normal, e que se não
> > fosse assim, algo estaria errado.
> >
> > "Conversando um dia com um professor sobre uma duvida, após algum
> > tempo ele virou para mim, talvez até para se livrar de mim e
> > disse, eu já estou muito bitolado (eu particularmente não quero
> ser
> > um bitolado)."
> >
> >
> > " Com certeza não sou douto, porem com mais certeza não sou um
> > sacerdote bitolado."
> >
> > Com esta frase, reforça-se mais uma vez o foco nas pessoas e não na
> > ciencia como um campo do saber, ou no seu metodo de trabalho. São
> as
> > pessoas que estão bitoladas e por isso a sua produção cientifica é
> > mediocre.
> >
> > Depois, o Ray finaliza com mais uma supeita de que as teorias que
> são
> > aceites hoje, que os cientistas bitolados são obrigados a seguir,
> > podem ser tão verdadeiras como a teoria do ether ou do calorico de
> ha
> > seculos atrás. A diferença é que seculos atráz havia liberdade de
> > escolher, hoje não ha.
> >
> > " Penso que há um enorme castelo construído sobre um alicerce
> > aparentemente frágil, fico preocupado se o alicerce cair o que
> > sustentara o castelo, e é lógico que quem habita o castelo esta
> > tranqüilo, tranqüilo."
> >
> > E agora eu pergunto ao Oraculo e ao Italo, o que este texto tem a
> > haver com o método cientifico e toda a panóplia de construções
> > intelectuais em que a ciencia, como um campo do saber , é
> baseada ? E
> > pq vcs insitem em que esse é o tema da discussão ?
> >
> >
> > Sérgio Taborda





SUBJECT: Emprego de professor em França
FROM: Maria Natália <grasdic@hotmail.com>
TO: ciencialist@yahoogrupos.com.br
DATE: 27/01/2005 02:26


Recrutement d'un Professeur a l'Universite de Montpellier II

Un poste de Professeur en section CNU 34, est susceptible d'etre vacant a
la rentree prochaine
sur l'Universite Montpellier II, avec affectation sur l' UMR 5024 - GRAAL
(Groupe de Recherche
en Astronomie et Astrophysique du Languedoc).
Ce poste devrait etre publie au JO pour la prochaine campagne de
recrutement (printemps 2005).

Des informations sur ce poste (profils enseignement et recherche) sont
disponibles
a partir du site WEB du GRAAL (http://www.isteem.univ-montp2.fr/GRAAL/)
sur la page :
http://www.isteem.univ-montp2.fr/GRAAL/posteprof.html

Pour tout contact :
Bertrand PLEZ plez@graal.univ-montp2.fr tel: 04 67 14 48 91






SUBJECT: Re: orguloso do saber?
FROM: "rayfisica" <rayfisica@yahoo.com.br>
TO: ciencialist@yahoogrupos.com.br
DATE: 27/01/2005 05:37


--- Em ciencialist@yahoogrupos.com.br, "oraculo333" <oraculo@a...>
escreveu
>
> Olá
>
Sequencia lógica (simplificada..:-):

------------------------------------------------------------
Senhor Homero
Claro que não tenho condições de entrar num embate técnico com o
senhor, pois que como demonstra o arranjo realizado por você, eu não
sei sequer expressar meus pensamentos.
Não sei se quer utilizar a pontuação, porem ainda assim eu solicitei o
seu auxilio no que fui prontamente atendido e creia aprendi muito, por
isso eu gostaria de poder contar com o senhor e também com o senhor
Taborda para aprender um pouquinho para que eu não cometa tantos erros.
Porem senhor foi exatamente a distancia entre os doutos e nós os
comuns que me fez questioná-los.






SUBJECT: Re: [ciencialist] Re: Mensagem com o meu endereço
FROM: "Luiz Ferraz Netto" <leobarretos@uol.com.br>
TO: <ciencialist@yahoogrupos.com.br>
DATE: 27/01/2005 08:49

É, ... isso serve prá ocê vê a minha ignorância sobre a evolução da espécie, ou melhor, do conhecimento sobre a minha própria espécie.

[]'
===========================
Luiz Ferraz Netto [Léo]
leobarretos@uol.com.br
http://www.feiradeciencias.com.br
===========================
-----Mensagem Original-----
De: "rmtakata" <rmtakata@altavista.net>
Para: <ciencialist@yahoogrupos.com.br>
Enviada em: quarta-feira, 26 de janeiro de 2005 17:30
Assunto: [ciencialist] Re: Mensagem com o meu endereço




--- Em ciencialist@yahoogrupos.com.br, "Luiz Ferraz Netto"
> Bem, entao o q. dizer dos q. nao pretendem se reproduzir? (Estereis,
> vasectomizados, abstemios convictos - todos sabem q. padres
> catolicos e freiras cristas nao fazem sexo -, homossexuais e
> quetais?)<<
>
> Eu diria que é muita sorte da 'evolução da espécie' que os citados
> não queiram/possam se reproduzir!

Bem, para a sorte da evolucao das especies, eu estou no meio do grupo
citado: nao pretendo ter filhos (se os tiver serao adotivos).

Mas para a infelicidade da sociedade humana caras como Alan Turing nao
deixaram descendentes (ele se inclui entre os homossexuais na lista
acima, assim como Oscar Wilde, Zelia Duncan, Gertrude Stein e, dizem,
Santos Dumont e Leonardo da Vinci).

Entre os padres temos Antonio Vieira.

Felizmente a filiacao intelectual nao necessita de filiacao sanguinea
- infelizmente, esta ultima facilita enormemente a primeira.

[]s,

Roberto Takata




--
Internal Virus Database is out-of-date.
Checked by AVG Anti-Virus.
Version: 7.0.300 / Virus Database: 265.6.13 - Release Date: 16/01/2005



SUBJECT: Re: [ciencialist] Re: Mulheres: Cozinha e na cama, já!!!!!!!!!!/Takata
FROM: JVictor <jvoneto@uol.com.br>
TO: ciencialist@yahoogrupos.com.br
DATE: 27/01/2005 09:12

Maria Natália escreveu:

> Natália,

Apesar de a msg dirirgir-se a Takata, não resisto ao impulso de dar uma
opinião.

>
> Natália: Enfim eu nem falo de inferioridade
> física/mental/intelectual.

Victor: E tem razão. Se alguma diferença há, ela não deve ser procurada
no aspecto físico/mental/intelectual, genético, ou o que seja, da
constituição do cérebro. Essa discutível diferença de produção
intelectual, em qualquer área do conhecimento, está nos preconceitos
masculinos contra a mulher, ao longo de diversas gerações, pela maioria
das civilizações, cada uma caracterizada pela "constituição civil" das
respectivas sociedades, em menor ou maior grau, e "competentemente"
incorporados por elas ao longo de milhares de anos. A elas, nós, os
machões(estou, hoje, com um bocado de prestações atrasadas....)
atribuímos grandes deveres e obrigações, em todas as áreas, bem como
castigos ou sanções, pelos eventuais não cumprimento e observação dos
tais, sempre cumpridos rigorosamente. Em nome de quê?: religião,
pretensa superioridade dos masculinos, "reconhecimento" de
características femininas incompatíveis com as dos homens, pecaminosas e
impuras, a assim por diante. Assim, a elas, nada, a não ser satisfazer e
procriar e vice-versa. Isto tudo acabou sendo uma primeira realidade na
psiquê desses seres humanos maravilhosos, que são as mulheres.
Incorporadas "tais normas" ao inconsciente coletivo feminino, o
comportamento compulsoriamente "natural" feminino vem sofrendo poucas
alterações, ao longo da evolução humana e social. No Ocidente, que
sempre foi mais liberal com elas, o avanço tem sido lento, mas
progressivo, como podemos ver. No Oriente, ainda há comportamentos e
atitudes para além de medievais, conforme todos sabem. Os recuos têm
sido sido de ínfímos a quase nada, ante o pouco, ao menos, que poderia ser.
No ocidente, basta ver histórias recentes do papel da mulher na
sociedade. Por conta disso tudo, dos preconceitos, dos deveres e
obrigações atribuídos à mulher, é que ela "parece" exibir desempenho
inferior ao nosso, machões descarados(os que o são, claro). Crie-se uma
sociedade onde haja uma equalização entre tais deveres e obrigações para
ambos os sexos, e eu aposto que os desempenhos serão também
equalizados(descontandos, claro, os trabalhos maternais; assim mesmo,
apenas os referentes a parir e amamentar. O resto, igual para ambos.). E
olhe, olhe, se não ficaremos atrás. O fato de haver menos mulheres em
destaque no âmbito da ciência, em particular, deve-se única e
exclusivamente a isso.
Uma pergunta: e o homem hoje, em geral, aceita, meeeesmo, que a mulher
lhe seja par a par, seja no trabalho ou em casa, como cara-metade?
Viva a mulher. Meu reino por um bocado delas!

Sds,

Victor.

>
> --- Em ciencialist@yahoogrupos.com.br, "rmtakata" <rmtakata@a...> escreveu
> >
> > --- Em ciencialist@yahoogrupos.com.br, "rmtakata" <rmtakata@a...>
> > > Niveladas com Marie Currie seria o mesmo q. pedir pra listar qq
> > > humano nivelado com Leonardo da Vinci.
> >
> > Mas listemos as mulheres q. ganharam o Nobel em uma area cientifica:
> >
> > Irene Joliot-Curie 1935 - Quimica
> > Gerty Theresa Cori 1947 - Medicina
> > Maria Goeppert-Mayer 1963 - Fisica
> > Dorothy Crowfoot Hodgkin 1964 - Quimica
> > Rosalyn Sussman Yalow 1977 - Medicina (ja' mencionada anteriormente)
> > Barbara McClintock 1983 - Medicina
> > Rita Levi-Montalcini 1986 - Medicina
> > Gertrude Elion 1988 - Medicina
> > Christiane Nusslein-Volhard 1995 - Medicina
> > Linda B. Buck 2004 - Medicina
> >
> > (Claro q. em qq listagem sempre cometemos injusticas. Mas nao poderia
> > ter deixado de mencionar Irene Curie [filha dos Curie - talento tb eh
> > hereditario] e Gerty Cori.)
> >
> > []s,
> >
> > Roberto Takata
>
>
>
>
>
> ##### ##### #####
>
> Para saber mais visite
> http://www.ciencialist.hpg.ig.com.br
>
>
> ##### ##### ##### #####
>
>
> *Yahoo! Grupos, um serviço oferecido por:*
>
> *
> <http://br.rd.yahoo.com/SIG=12aseb0ds/M=264105.3931087.6562589.1588051/D=brclubs/S=2137111528:HM/EXP=1106880195/A=2361264/R=6/SIG=10v4acpp0/*http://br.shopping.yahoo.com/>*
>
>
>
> ------------------------------------------------------------------------
> *Links do Yahoo! Grupos*
>
> * Para visitar o site do seu grupo na web, acesse:
> http://br.groups.yahoo.com/group/ciencialist/
>
> * Para sair deste grupo, envie um e-mail para:
> ciencialist-unsubscribe@yahoogrupos.com.br
> <mailto:ciencialist-unsubscribe@yahoogrupos.com.br?subject=Unsubscribe>
>
> * O uso que você faz do Yahoo! Grupos está sujeito aos Termos do
> Serviço do Yahoo! <http://br.yahoo.com/info/utos.html>.
>
>




SUBJECT: Re: Mulheres: Cozinha e na cama, já!!!!!!!!!!/Takata
FROM: "rmtakata" <rmtakata@altavista.net>
TO: ciencialist@yahoogrupos.com.br
DATE: 27/01/2005 09:47


> > Natália: Enfim eu nem falo de inferioridade
> > física/mental/intelectual.

--- Em ciencialist@yahoogrupos.com.br, JVictor <jvoneto@u...> escreveu
> Victor: E tem razão. Se alguma diferença há, ela não deve ser
> procurada no aspecto físico/mental/intelectual, genético, ou o que
> seja, da constituição do cérebro.

Se deve ou nao, nao sei, mas essa diferenca pode ser procurada tb
nesses aspectos. Aparentemente encontraram muitas diferencas ai' (a
presenca de um X no lugar de um Y faz uma diferenca enorme - ainda
bem, acho q. eu nao ficaria bem com duas mamas enormes, embora uma
Naomi Campbell com barba por fazer nao seja tampouco uma visao idilica).

Mas essas diferencas nao significam de modo algum inferioridade e isso
eh preciso deixar muito claro. Significa isto sim uma diversidade. E
viva 'as diferencas!

Tampouco o fato dessas diferencas existirem significa q. seja as
unicas causas da exclusao das mulheres (ou dos homens) em uma dada area.

> No Ocidente, que sempre foi mais liberal com elas, o avanço tem sido
> lento, mas progressivo, como podemos ver. No Oriente, ainda há
> comportamentos e atitudes para além de medievais, conforme todos

O Ocidente nao foi sempre mais liberal em relacao 'as mulheres, nao.
Em verdade, em verdade, a tradicao grega colocava a mulher em uma
posicao mais inferior do q. ela tinha em sociedades do Oriente Proximo
(da Asia Menor). Outro ponto, a tradicao crista tb colocava as
mulheres em posicao inferior - entre outras coisas pela culpa do
pecado de Eva - em contraste com as religioes calcadas na figura da
mulher como simbolo da fertilidade. Junte a tradicao machista grega
com a tradicao machista crista e teremos a sociedade ocidental machista.

O espaco feminino foi conquistado a ferro e a fogo no Ocidente. O
*sempre* se refere a coisa de nao mais do q. 70 anos - e mais
efetivamente de 40 anos para ca'.

Qto ao Oriente, q. Oriente? O Oriente Medio, a Asia Central, o Sudeste
Asiatico, o Sul da Asia, o Norte da Ásia, o Extremo Oriente?

[]s,

Roberto Takata





SUBJECT: a midia e o saber p/ Sr. Taborda.
FROM: "rayfisica" <rayfisica@yahoo.com.br>
TO: ciencialist@yahoogrupos.com.br
DATE: 27/01/2005 12:03


Taborda--Ja foi tempo em que ele não queria entender essas coisas.
A
media faz
tanta pressão que o leigo sentese idiota por não entender essas
teorias
avançadas e recorre às fontes da propria media para se
informar. O
problema é que essas fontes são ainda mais leigas que ele , e
ele acaba
recorrendo a listas com esta. O leigo precisa, primeiro de tudo
entender, que não ha mal no mundo se ele não entender fisica
quantica ,
ou TRG.
--------------------------------------------------------------
27/01/2005 - 10h43
Universo começou com nuvens de matéria escura, dizem cientistas
SALVADOR NOGUEIRA
da Folha de S.Paulo

http://www1.folha.uol.com.br/folha/ciencia/ult306u12882.shtml

Eles foram criados no princípio dos tempos. Ainda estão por
aí,
vagando pelo Universo, atravessando corpos como se eles não
existissem
e somente em raras ocasiões interagindo com a matéria normal.
Quem
você vai chamar? Com certeza, não os Caça-Fantasmas. Mas
tente Ben
Moore, do Instituto de Física Teórica da Universidade de
Zurique, Suíça.

Usando um poderoso supercomputador para simular nada menos que a
evolução do Universo quase desde o Big Bang (começando uns 20
milhões
de anos depois da explosão que teria dado origem ao cosmos), ele
chegou à conclusão de que os primeiros objetos formados foram
nuvens
com o tamanho do Sistema Solar e a massa da Terra, feitos de
partículas que ninguém consegue ver.

Esses chamados "halos" ainda existem por aí e varrem o planeta (e
tudo
que há nele, inclusive nós) a cada poucos milhares de anos. E
há um
bocado deles lá fora. Só nos arredores da Via Láctea, a
galáxia na
qual o Sistema Solar está, haveria cerca de 1 milhão de
bilhões de
objetos como esses, se os cálculos de Moore estiverem corretos.
É 10
mil vezes mais do que o número de estrelas na mesma região.

"Eles são mais como nuvens de partículas, mais concentradas na
região
do centro", disse o cientista à Folha. Seu estudo, publicado na
edição
de hoje da revista científica britânica "Nature", ajuda a
explicar
onde está toda a matéria do Universo.

Minoria

O mistério intriga os cientistas há décadas. Ao somarem
todas as
estrelas e galáxias que podem ver, eles descobriram que essa
matéria
"comum" não responde nem por 5% de toda a massa que deveria haver
para
explicar alguns efeitos gravitacionais observados, como, por exemplo,
o ritmo de rotação de galáxias (que é mais rápido do que
deveria ser
se as estrelas e o gás fossem tudo o que há por lá).

À massa faltante os cientistas deram o auspicioso nome de
"matéria
escura", e ninguém até hoje sabe o que ela é de fato. Os
grupo de Ben
Moore aposta que ela é composta por partículas chamadas de
neutralinos, os primos ricos dos neutrinos.

"Bem, neutrinos e neutralinos são bem parecidos, ambos têm
massa e
ambos interagem bem fracamente com os átomos normais. Portanto, os
dois passam direto por nosso corpo em enormes quantidades a cada
segundo", diz Moore. "A principal diferença é a sua massa --a do
neutralino é bem maior."

Um problema para a hipótese defendida por Moore, que hoje é a
principal candidata a explicar a matéria escura, é o fato de
que,
diferente dos neutrinos, nunca ninguém conseguiu ver um
neutralino. E
a própria idéia vem da teoria de que existe uma "supersimetria"
entre
partículas, algo que também não foi confirmado.

Mas não o será por muito tempo, segundo Moore. "O Grande
Colisor de
Hádrons [LHC, na sigla em inglês], no Cern [principal
laboratório de
física de partículas na Europa], irá detectar evidências
da
supersimetria e dos neutralinos, se eles estiverem lá!"

Segundo ele, a tal supersimetria, embora não-comprovada, é uma
idéia
consolidada. "Não é tão controversa --grande parte da
motivação para
gastar os bilhões de dólares que o LHC vai custar é
demonstrar a
existência da supersimetria. Os cientistas em geral não gastam
esse
monte de dinheiro sem uma razão muito boa."

Segundo o grupo, também será possível detectar uma nuvem de
neutralinos no espaço, caso ela exista mesmo --ela emitiria raios
gama, o tipo de radiação eletromagnética mais energético do
Universo,
num padrão bem distinto, reconhecível.
------------------------------
Acho que isso demonstra (aos gritos)o que o senhor disse, eu não to
conseguindo si quer aceitar a idéia de entender o NEUTRINO agora
recebo a noticia do NEUTRALINO, o que é isso?





SUBJECT: OT - dicas do Google
FROM: "E m i l i a n o C h e m e l l o" <chemelloe@yahoo.com.br>
TO: <Conversa_de_Botequim@yahoogrupos.com.br>, <ciencialist@yahoogrupos.com.br>, <naeq-ucs@yahoogrupos.com.br>, <quimica-qaw@yahoogrupos.com.br>, <quimica@grupos.com.br>
DATE: 27/01/2005 12:11

Use o Google do jeito certo
http://pcworld.uol.com.br/AdPortalV3/adCmsDocumentoShow.aspx?documento=8205577&Area=435000

[ ] 's do Emiliano Chemello
emiliano@quimica.net
http://www.quimica.net/emiliano
http://www.ucs.br/ccet/defq/naeq

" Rien ne se perd, rien ne se crée,
tout se transforme."

Antoine Laurent de Lavoisier (químico francês, 1743 - 1794)




SUBJECT: orguloso do saber? p/ Rayfisica
FROM: "Oraculo" <oraculo@atibaia.com.br>
TO: <ciencialist@yahoogrupos.com.br>
DATE: 27/01/2005 14:11

Olá ray

Não se preocupe (e não me chame de senhor..:-), será sempre um prazer conversar e debater com você..:-)

Essa distancia, que existe, muitas vezes é causada pela própria personalidade das pessoas envolvidas, sejam cientistas ou não. Boa parte das pessoas é arrogante e convencida, e nem precisam saber muito para ser assim..:-)

Mas, muitos cientistas e divulgadores de ciência são ótimas pessoas, bons para explicar e ensinar, como Sagan, Feyman, Dawkins, Gould e muitos mais.

Se alguém se interessa por ciência, vai encontrar sempre quem o ajude a compreender mais e mais (inclusive o pessoa desta lista..:-)

Um abraço.

Homero

----- Original Message -----
From: rayfisica
To: ciencialist@yahoogrupos.com.br
Sent: Thursday, January 27, 2005 5:37 AM
Subject: [ciencialist] Re: orguloso do saber?



--- Em ciencialist@yahoogrupos.com.br, "oraculo333" <oraculo@a...>
escreveu
>
> Olá
>
Sequencia lógica (simplificada..:-):

------------------------------------------------------------
Senhor Homero
Claro que não tenho condições de entrar num embate técnico com o
senhor, pois que como demonstra o arranjo realizado por você, eu não
sei sequer expressar meus pensamentos.
Não sei se quer utilizar a pontuação, porem ainda assim eu solicitei o
seu auxilio no que fui prontamente atendido e creia aprendi muito, por
isso eu gostaria de poder contar com o senhor e também com o senhor
Taborda para aprender um pouquinho para que eu não cometa tantos erros.
Porem senhor foi exatamente a distancia entre os doutos e nós os
comuns que me fez questioná-los.






##### ##### #####

Para saber mais visite
http://www.ciencialist.hpg.ig.com.br


##### ##### ##### #####


Yahoo! Grupos, um serviço oferecido por:







------------------------------------------------------------------------------
Links do Yahoo! Grupos

a.. Para visitar o site do seu grupo na web, acesse:
http://br.groups.yahoo.com/group/ciencialist/

b.. Para sair deste grupo, envie um e-mail para:
ciencialist-unsubscribe@yahoogrupos.com.br

c.. O uso que você faz do Yahoo! Grupos está sujeito aos Termos do Serviço do Yahoo!.



[As partes desta mensagem que não continham texto foram removidas]



SUBJECT: Re: [ciencialist] a midia e o saber p/ Sr. Taborda.
FROM: "Oraculo" <oraculo@atibaia.com.br>
TO: <ciencialist@yahoogrupos.com.br>
DATE: 27/01/2005 14:21

Olá Taborda

Lendo esse trecho do seu texto que o ray postou, não resisti em responder que, penso exatamente igual..:-) O que é interessante, já que aparentemente estamos discutidno em "oposição"..:-)

Este em especial:

"O leigo precisa, primeiro de tudo entender, que não ha mal no mundo se ele não entender fisica quantica , ou TRG."

Exato..:-) Venho tentando dizer isso desde a primeira mensagem. Veja, nela eu digo que para aproximar o leigo da ciência, não é preciso ensinar fisica avançada ou biologia molecular para ele. Não faz mal não saber sobre nada disso, pelo menos não saber profundamente.

Mas, saber como e de que forma esse tipo de conhecimento foi gerado, pode diminuir a desconfiança natural de leigo sobre algo que não compreende. Se este pensar que cada uma dessas teorias é fruto de viagens de imaginação, bebedeira de cientistas, ou sonho de noite mal dormida de pesquisadores, vai dar o mesmo valor que qualquer outra afirmação maluca que se houve na mídia (a "terrivel" midia malvada..:-)

Entretanto, se comprender o esforço, as regras de segurança e o rigor envolvidos na produção de conhecimento cientifico (esqueçamos por um instante as falhas de politicagem..:-), se compreender que teoria não é algo que alguém sobnhou depois de beber muito, se puder entender como a acrescimo de dados vai fortalecendo (ou destruindo) um conhecimento cientifico, estará mais perto da ciência, e nem achará mal se não compreender fisica quantica, ou TRG.

Era isso, exatamente, que eu tentava colocar ao rayfisica...:-)

Um abraço.

Homero




----- Original Message -----
From: rayfisica
To: ciencialist@yahoogrupos.com.br
Sent: Thursday, January 27, 2005 12:03 PM
Subject: [ciencialist] a midia e o saber p/ Sr. Taborda.



Taborda--Ja foi tempo em que ele não queria entender essas coisas.
A
media faz
tanta pressão que o leigo sentese idiota por não entender essas
teorias
avançadas e recorre às fontes da propria media para se
informar. O
problema é que essas fontes são ainda mais leigas que ele , e
ele acaba
recorrendo a listas com esta. O leigo precisa, primeiro de tudo
entender, que não ha mal no mundo se ele não entender fisica
quantica ,
ou TRG.
--------------------------------------------------------------
27/01/2005 - 10h43
Universo começou com nuvens de matéria escura, dizem cientistas
SALVADOR NOGUEIRA
da Folha de S.Paulo

http://www1.folha.uol.com.br/folha/ciencia/ult306u12882.shtml

Eles foram criados no princípio dos tempos. Ainda estão por
aí,
vagando pelo Universo, atravessando corpos como se eles não
existissem
e somente em raras ocasiões interagindo com a matéria normal.
Quem
você vai chamar? Com certeza, não os Caça-Fantasmas. Mas
tente Ben
Moore, do Instituto de Física Teórica da Universidade de
Zurique, Suíça.

Usando um poderoso supercomputador para simular nada menos que a
evolução do Universo quase desde o Big Bang (começando uns 20
milhões
de anos depois da explosão que teria dado origem ao cosmos), ele
chegou à conclusão de que os primeiros objetos formados foram
nuvens
com o tamanho do Sistema Solar e a massa da Terra, feitos de
partículas que ninguém consegue ver.

Esses chamados "halos" ainda existem por aí e varrem o planeta (e
tudo
que há nele, inclusive nós) a cada poucos milhares de anos. E
há um
bocado deles lá fora. Só nos arredores da Via Láctea, a
galáxia na
qual o Sistema Solar está, haveria cerca de 1 milhão de
bilhões de
objetos como esses, se os cálculos de Moore estiverem corretos.
É 10
mil vezes mais do que o número de estrelas na mesma região.

"Eles são mais como nuvens de partículas, mais concentradas na
região
do centro", disse o cientista à Folha. Seu estudo, publicado na
edição
de hoje da revista científica britânica "Nature", ajuda a
explicar
onde está toda a matéria do Universo.

Minoria

O mistério intriga os cientistas há décadas. Ao somarem
todas as
estrelas e galáxias que podem ver, eles descobriram que essa
matéria
"comum" não responde nem por 5% de toda a massa que deveria haver
para
explicar alguns efeitos gravitacionais observados, como, por exemplo,
o ritmo de rotação de galáxias (que é mais rápido do que
deveria ser
se as estrelas e o gás fossem tudo o que há por lá).

À massa faltante os cientistas deram o auspicioso nome de
"matéria
escura", e ninguém até hoje sabe o que ela é de fato. Os
grupo de Ben
Moore aposta que ela é composta por partículas chamadas de
neutralinos, os primos ricos dos neutrinos.

"Bem, neutrinos e neutralinos são bem parecidos, ambos têm
massa e
ambos interagem bem fracamente com os átomos normais. Portanto, os
dois passam direto por nosso corpo em enormes quantidades a cada
segundo", diz Moore. "A principal diferença é a sua massa --a do
neutralino é bem maior."

Um problema para a hipótese defendida por Moore, que hoje é a
principal candidata a explicar a matéria escura, é o fato de
que,
diferente dos neutrinos, nunca ninguém conseguiu ver um
neutralino. E
a própria idéia vem da teoria de que existe uma "supersimetria"
entre
partículas, algo que também não foi confirmado.

Mas não o será por muito tempo, segundo Moore. "O Grande
Colisor de
Hádrons [LHC, na sigla em inglês], no Cern [principal
laboratório de
física de partículas na Europa], irá detectar evidências
da
supersimetria e dos neutralinos, se eles estiverem lá!"

Segundo ele, a tal supersimetria, embora não-comprovada, é uma
idéia
consolidada. "Não é tão controversa --grande parte da
motivação para
gastar os bilhões de dólares que o LHC vai custar é
demonstrar a
existência da supersimetria. Os cientistas em geral não gastam
esse
monte de dinheiro sem uma razão muito boa."

Segundo o grupo, também será possível detectar uma nuvem de
neutralinos no espaço, caso ela exista mesmo --ela emitiria raios
gama, o tipo de radiação eletromagnética mais energético do
Universo,
num padrão bem distinto, reconhecível.
------------------------------
Acho que isso demonstra (aos gritos)o que o senhor disse, eu não to
conseguindo si quer aceitar a idéia de entender o NEUTRINO agora
recebo a noticia do NEUTRALINO, o que é isso?





##### ##### #####

Para saber mais visite
http://www.ciencialist.hpg.ig.com.br


##### ##### ##### #####


Yahoo! Grupos, um serviço oferecido por:
PUBLICIDADE




------------------------------------------------------------------------------
Links do Yahoo! Grupos

a.. Para visitar o site do seu grupo na web, acesse:
http://br.groups.yahoo.com/group/ciencialist/

b.. Para sair deste grupo, envie um e-mail para:
ciencialist-unsubscribe@yahoogrupos.com.br

c.. O uso que você faz do Yahoo! Grupos está sujeito aos Termos do Serviço do Yahoo!.



[As partes desta mensagem que não continham texto foram removidas]



SUBJECT: conferir calculo
FROM: "Luiz Ferraz Netto" <leobarretos@uol.com.br>
TO: <ciencialist@yahoogrupos.com.br>
DATE: 27/01/2005 14:40

Um consulente fez a seguinte pergunta:

> Gostaria de saber se tem como definir a força em newtons necessária para lançar um corpo, tendo como dado só a altura máxima atingida por este corpo.

E esbocei a seguinte resposta:

o cálculo, numa situação ideal, não deve ser tão difícil. Todavia, já posso adiantar que outros dados serão necessários; vejamos:

h - altura a ser alcançada (supondo efeito zero para a resistência do ar --- caso de uma bolinha de aço, por exemplo) --- Dado da questão
g = aceleração da gravidade --- deve ser conhecido
m = massa da bolinha --- deve ser conhecida
T = tempo de lançamento (quanto tempo a bolinha fica na mão durante o arremesso vertical --- ou no estilingue, ou na catapulta, etc.) --- deve ser conhecido
Fm = força média durante o arremesso.

Cálculo da velocidade inicial vertical (velocidade do arremesso) :
mgh =(1/2)m.v^2
v^2 = 2gh
v = sqrt(2gh) ------ leia: v igual raíz quadrada de (2gh)

Cálculo da força média:
Fm.T = m.v (teorema do impulso: o impulso mede a variação da quantidade de movimento)
Fm = m.v/T

Fm = m.sqrt(2gh)/T

Se a bolinha de aço tem massa de 0,1 kg, deve alcançar a altura de 20 m e o arremesso durou 1 segundo, a força média valerá:

Fm = 0,1 .sqrt(2.10.20)/1 = 0,1.20 = 2,0 N
++++++++++++++++++++++++++++++++++
Eis meu problema: 2,0 N é uma intensidade muito pequena --- isso furou minha imaginação --- bateu o gongo de que algo está errado! Onde errei?
Sei, sei, ao nascer, mas ... na resposta da questão?

aquele abraço,

===========================
Luiz Ferraz Netto [Léo]
leobarretos@uol.com.br
http://www.feiradeciencias.com.br
===========================


--
Internal Virus Database is out-of-date.
Checked by AVG Anti-Virus.
Version: 7.0.300 / Virus Database: 265.6.13 - Release Date: 16/01/2005



SUBJECT: Re: [ciencialist] Texto de Gould p Italo (era orguloso do saber?)
FROM: ÿffffcdtalo Rocha <imrochaguedes@yahoo.com.br>
TO: ciencialist@yahoogrupos.com.br
DATE: 27/01/2005 14:46

Caro Homero,
Também tenho uma certa preferência pela visão de Dawkins, principalmente no que diz respeito ao papel da evolução na formação da consciência e comportamento humanos. Stephen Jay Gould, apesar da controvérsia de algumas de suas idéias, é um excelente escritor e incomparável defensor do método científico. O artigo em questão chama-se "Deconstructing the 'Science Wars' by reconstructing an old mold" e foi publicado na Science de 14 de janeiro de 2000, 287:253-261.
Cordialmente, Ítalo Moraes Rocha Guedes.

Oraculo <oraculo@atibaia.com.br> wrote:

Olá Italo

Por acaso tem mais dados sobre o artigo do Jay Gould? Gosto muito dos livros dele e da forma como escreve (embora tenha discordancias e prefira os argumentos de Dawkins..:-) e gostaria de encontrar o texto que citou.

Poderia informar algo mais, como o titulo ou a edição mais precisamente?

Um abraço.

Homero

----- Original Message -----
From: ÿffffcdtalo Rocha
To: ciencialist@yahoogrupos.com.br
Sent: Wednesday, January 26, 2005 12:32 PM
Subject: Re: [ciencialist] Re: orguloso do saber?


Meu caro senhor "Rayfisica", entendo perfeitamente sua indignação (educada) quanto ao que escrevi em relação ao esforço dos leigos em entender a ciência. Embora realmente ache que seja uma minoria os que realmente fazem um esforço para compreender os métodos e resultados da prática científica, quero crer que isto é resultado mais da péssima situação educacional de nosso país do que por má-vontade pura e simples. Provavelmente escrevi aquilo contaminado pela ignorância com que o outro senhor (Taborda) respondeu a alguém que tentava justamente entrar na discussão. Tenho profundo interesse na divulgação da ciência e do pensamento humanista, razão pela qual participo deste fórum e de outros dedicados principalmente a isto. O que não suporto é vulgaridade nem falta de respeito, porque imagino que em um fórum destes tratamos com pessoas civilizadas. Além do livro que lhe indiquei (o nome completo do autor é Brian L. Silver, um físico-químico inglês, o livro custa em torno de R$ 60,00 e é
facilmente comprado pela internet) há um excelente artigo do falecido Stephen Jay Gould publicado na Science em 2000, se não me engano no mês de maio, e que rebate de forma brilhante as críticas geralmente feitas à ciência e ao método científico.
Cordialmente, Ítalo Moraes Rocha Guedes.


---------------------------------
Yahoo! Acesso Grátis - Internet rápida e grátis. Instale o discador do Yahoo! agora.

[As partes desta mensagem que não continham texto foram removidas]



##### ##### #####

Para saber mais visite
http://www.ciencialist.hpg.ig.com.br


##### ##### ##### #####


Yahoo! Grupos, um serviço oferecido por:

São Paulo Rio de Janeiro Curitiba Porto Alegre Belo Horizonte Brasília




------------------------------------------------------------------------------
Links do Yahoo! Grupos

a.. Para visitar o site do seu grupo na web, acesse:
http://br.groups.yahoo.com/group/ciencialist/

b.. Para sair deste grupo, envie um e-mail para:
ciencialist-unsubscribe@yahoogrupos.com.br

c.. O uso que você faz do Yahoo! Grupos está sujeito aos Termos do Serviço do Yahoo!.



[As partes desta mensagem que não continham texto foram removidas]



##### ##### #####

Para saber mais visite
http://www.ciencialist.hpg.ig.com.br


##### ##### ##### #####
Links do Yahoo! Grupos










---------------------------------
Yahoo! Acesso Grátis - Internet rápida e grátis. Instale o discador do Yahoo! agora.

[As partes desta mensagem que não continham texto foram removidas]



SUBJECT: Texto de Gould p Italo - agradecimento
FROM: "Oraculo" <oraculo@atibaia.com.br>
TO: <ciencialist@yahoogrupos.com.br>
DATE: 27/01/2005 15:01

Olá Italo

Grato pela informação. Já encontrei o artigo e é excelente. Se alguém quiser conferir, ele pode ser encontrado neste link:

http://www.stephenjaygould.org/library/gould_science-wars.html

Em especial, o Rayfisica pode gostar bastante do assunto e da forma como Gould o apresenta.

Um abraço.

Homero


----- Original Message -----
From: ÿffffcdtalo Rocha
To: ciencialist@yahoogrupos.com.br
Sent: Thursday, January 27, 2005 2:46 PM
Subject: Re: [ciencialist] Texto de Gould p Italo (era orguloso do saber?)


Caro Homero,
Também tenho uma certa preferência pela visão de Dawkins, principalmente no que diz respeito ao papel da evolução na formação da consciência e comportamento humanos. Stephen Jay Gould, apesar da controvérsia de algumas de suas idéias, é um excelente escritor e incomparável defensor do método científico. O artigo em questão chama-se "Deconstructing the 'Science Wars' by reconstructing an old mold" e foi publicado na Science de 14 de janeiro de 2000, 287:253-261.
Cordialmente, Ítalo Moraes Rocha Guedes.

Oraculo <oraculo@atibaia.com.br> wrote:

Olá Italo

Por acaso tem mais dados sobre o artigo do Jay Gould? Gosto muito dos livros dele e da forma como escreve (embora tenha discordancias e prefira os argumentos de Dawkins..:-) e gostaria de encontrar o texto que citou.

Poderia informar algo mais, como o titulo ou a edição mais precisamente?

Um abraço.

Homero

----- Original Message -----
From: ÿffffcdtalo Rocha
To: ciencialist@yahoogrupos.com.br
Sent: Wednesday, January 26, 2005 12:32 PM
Subject: Re: [ciencialist] Re: orguloso do saber?


Meu caro senhor "Rayfisica", entendo perfeitamente sua indignação (educada) quanto ao que escrevi em relação ao esforço dos leigos em entender a ciência. Embora realmente ache que seja uma minoria os que realmente fazem um esforço para compreender os métodos e resultados da prática científica, quero crer que isto é resultado mais da péssima situação educacional de nosso país do que por má-vontade pura e simples. Provavelmente escrevi aquilo contaminado pela ignorância com que o outro senhor (Taborda) respondeu a alguém que tentava justamente entrar na discussão. Tenho profundo interesse na divulgação da ciência e do pensamento humanista, razão pela qual participo deste fórum e de outros dedicados principalmente a isto. O que não suporto é vulgaridade nem falta de respeito, porque imagino que em um fórum destes tratamos com pessoas civilizadas. Além do livro que lhe indiquei (o nome completo do autor é Brian L. Silver, um físico-químico inglês, o livro custa em torno de R$ 60,00 e é
facilmente comprado pela internet) há um excelente artigo do falecido Stephen Jay Gould publicado na Science em 2000, se não me engano no mês de maio, e que rebate de forma brilhante as críticas geralmente feitas à ciência e ao método científico.
Cordialmente, Ítalo Moraes Rocha Guedes.


---------------------------------
Yahoo! Acesso Grátis - Internet rápida e grátis. Instale o discador do Yahoo! agora.

[As partes desta mensagem que não continham texto foram removidas]



##### ##### #####

Para saber mais visite
http://www.ciencialist.hpg.ig.com.br


##### ##### ##### #####


Yahoo! Grupos, um serviço oferecido por:

São Paulo Rio de Janeiro Curitiba Porto Alegre Belo Horizonte Brasília




------------------------------------------------------------------------------
Links do Yahoo! Grupos

a.. Para visitar o site do seu grupo na web, acesse:
http://br.groups.yahoo.com/group/ciencialist/

b.. Para sair deste grupo, envie um e-mail para:
ciencialist-unsubscribe@yahoogrupos.com.br

c.. O uso que você faz do Yahoo! Grupos está sujeito aos Termos do Serviço do Yahoo!.



[As partes desta mensagem que não continham texto foram removidas]



##### ##### #####

Para saber mais visite
http://www.ciencialist.hpg.ig.com.br


##### ##### ##### #####
Links do Yahoo! Grupos










---------------------------------
Yahoo! Acesso Grátis - Internet rápida e grátis. Instale o discador do Yahoo! agora.

[As partes desta mensagem que não continham texto foram removidas]



##### ##### #####

Para saber mais visite
http://www.ciencialist.hpg.ig.com.br


##### ##### ##### #####


Yahoo! Grupos, um serviço oferecido por:







------------------------------------------------------------------------------
Links do Yahoo! Grupos

a.. Para visitar o site do seu grupo na web, acesse:
http://br.groups.yahoo.com/group/ciencialist/

b.. Para sair deste grupo, envie um e-mail para:
ciencialist-unsubscribe@yahoogrupos.com.br

c.. O uso que você faz do Yahoo! Grupos está sujeito aos Termos do Serviço do Yahoo!.



[As partes desta mensagem que não continham texto foram removidas]



SUBJECT: Re: [ciencialist] Re: orguloso do saber?
FROM: ÿffffcdtalo Rocha <imrochaguedes@yahoo.com.br>
TO: ciencialist@yahoogrupos.com.br
DATE: 27/01/2005 15:11

Caro Sergio Taborda,
Entendo o que quis dizer o Ray e você, e concordo em parte. Primeiro, acho que deve haver cuidado com generalizações do tipo "o cientista é ou parece ser arrogante", embora não negue que não devem ser poucos os que agem desta forma, realmente existem alguns que ostentam um título de doutor como um título de nobreza que inclusive confere infalibilidade a tudo que dizem ou pensam. Não nego que muito do dogmatismo tão criticado nas religiões também existe na ciência, o próprio divulgador Brian L. Silver, de quem já falei, admite que muito, mas não tudo, do que se acredita na ciência repousa mais sobre uma base de fé do que em observações realmente, creio que principalmente no campo da física teórica, mas não tenho autoridade para defender esta idéia. Sei que para o leigo a ciência moderna deve parecer tão esotérica quanto as instruções rosa-cruz, eu mesmo só sou (ou tento ser) um especialista em minha área restrita e pouco mais além disto. Entretanto, contra a afirmação de que a base do
castelo da ciência pode ser frágil e estaríamos ingenuamente despreocupados sobre as mesmas, apresento o fato, certamente conhecido por ambos, de a ciência, ao contrário de outors ramos do conhecimento humano, fazer previsões do comportamento do mundo de uma forma não casual e não poucas vezes de maneira notavelmente exata, e é exatamente por matematizar o mundo que isto é possível, e sei que o que digo não é novo, mas não deixa de ter conseqüências profundas. Entendo que provavelmente o nó da questão está na linguagem, e acho que todos os desentendimentos humanos residem aí, e não é uma questão fácil. Creio que a dificuldade de divulgar acessivelmente a ciência, no Brasil, se deve ao desinteresse dos cientistas e, aqui concordarei, com um certo descaso em relação aos que não são da área, talvez reflexo ainda de nossa cultura de desprezo e preconceito oculto, não sei bem. É algo a se discutir.
Cordialmente, Ítalo Moraes Rocha Guedes.

"Sergio M. M. Taborda" <sergiotaborda@terra.com.br> wrote:



A minha aspereza - ou como perferem chamar alguns: mal educação - ao
comentar o texto do Oráculo, no assunto em epigrafe , deve-se ao facto
de eu não encontrar no texto do ray nenhuma mensão ao método
cientifico, à estutura teoria da ciencia, à metafisica da ciencia, ou
ha ciencia em si. Apenas encontro mensões a cientistas, pessoas que
fazem, ou querem fazer ciencia, e pssoas que entendem ou querem
entender ciencia. Portanto o assunto, quanto a mim são as pessoas- os
conluios, arrogancias, egos e auto-embromações, tipicas do ser humano
- e não a ciencia em si. O ppr titulo é impeditivo que se trate da
ciencia, pois a ciencia não é uma pessoa, não tem orgulho.

O foco era o orgulho das pessoas que fazem ciencia: os cientistas e
onde isso interfere com o bom funcionamento da ciencia.

cito: "Os cientistas de hoje tem razão de ficarem orgulhosos do saber
que possuem ou que possam possuir?"

o ray estende-se pela impossibilidade de uma pessoa normal, um leigo,
entender um especialista.

"Tomando por base as ultimas comunicações vejo a distancia entre os
cientistas e o vulgo, parece-me que é impossível à pessoa
comum entender patavina do que falam os doutos."

Aqui eu entendi a palavra doutos com sentido prejurativo de
arrogancia. E foi isso que eu comentei , que os cientistas sim são
arrogantes no que toca À defesa das suas teorias, e o mainsteam é
arrogante até ao ponto em que organiza a intervensão dos especialistas
de tal forma a escluir todos os que não seguem as suas ideias. E mais
do que isso, os especialistas negam-se a explicar em palavras simples
as suas teorias , que pensam eles ser tão boas, ao comum dos mortais.

O ray continua comparando a ciencia com a religião , mas do ponto de
vista das pessoa e não dos métodos. Ele compara realmente os
cientistas com os religiosos idolatras que enviavam para a fogueira
quem não concordava com eles ou não idolatrava os mesmos idolos. Pondo
em causa, enfim, a hipotetica solidez dos conhecimentos que temos
hoje, já que são baseados em esquemas de medo, e imposição de autoridade.

Depois , ray lança vários exemplos em que levanta a questão de que os
especialistas são obrigados a seguir a mainstream, a bitola, a regra,
o acordo comum, ou estão fadados à extinção. E pior que isso, os
cientistas nessas consições sentem que isso é normal, e que se não
fosse assim, algo estaria errado.

"Conversando um dia com um professor sobre uma duvida, após algum
tempo ele virou para mim, talvez até para se livrar de mim e
disse, eu já estou muito bitolado (eu particularmente não quero ser
um bitolado)."


" Com certeza não sou douto, porem com mais certeza não sou um
sacerdote bitolado."

Com esta frase, reforça-se mais uma vez o foco nas pessoas e não na
ciencia como um campo do saber, ou no seu metodo de trabalho. São as
pessoas que estão bitoladas e por isso a sua produção cientifica é
mediocre.

Depois, o Ray finaliza com mais uma supeita de que as teorias que são
aceites hoje, que os cientistas bitolados são obrigados a seguir,
podem ser tão verdadeiras como a teoria do ether ou do calorico de ha
seculos atrás. A diferença é que seculos atráz havia liberdade de
escolher, hoje não ha.

" Penso que há um enorme castelo construído sobre um alicerce
aparentemente frágil, fico preocupado se o alicerce cair o que
sustentara o castelo, e é lógico que quem habita o castelo esta
tranqüilo, tranqüilo."

E agora eu pergunto ao Oraculo e ao Italo, o que este texto tem a
haver com o método cientifico e toda a panóplia de construções
intelectuais em que a ciencia, como um campo do saber , é baseada ? E
pq vcs insitem em que esse é o tema da discussão ?


Sérgio Taborda





##### ##### #####

Para saber mais visite
http://www.ciencialist.hpg.ig.com.br


##### ##### ##### #####
Links do Yahoo! Grupos









__________________________________________________
Converse com seus amigos em tempo real com o Yahoo! Messenger
http://br.download.yahoo.com/messenger/

[As partes desta mensagem que não continham texto foram removidas]



SUBJECT: Re: [ciencialist] Re: Mulheres: Cozinha e na cama, já!!!!!!!!!!
FROM: ÿffffcdtalo Rocha <imrochaguedes@yahoo.com.br>
TO: ciencialist@yahoogrupos.com.br
DATE: 27/01/2005 15:18

Meu caro, talvez não saiba, mas também são gramíneas a cana-de-açúcar, o milho e o arroz, este último, parece-me, bastante comido por aí.
Ítalo Moraes Rocha Guedes.

Luiz Ferraz Netto <leobarretos@uol.com.br> wrote:

Tá bom, tá bom,

... vamos comer grama. :-)

>Não esquecer, no Brasil, a Johanna Dobereiner, agrônoma, responsável pela descoberta de bactérias fixadoras de nitrogênio em gramíneas e uma das responsáveis pelo fato de o Brasil ter o mais econômico plantio de soja do mundo.

>>Rosalyn Franklin, Lynn Margulis, Caroline Herschell, Rosalyn Sussman
Yalow...



--
Internal Virus Database is out-of-date.
Checked by AVG Anti-Virus.
Version: 7.0.300 / Virus Database: 265.6.13 - Release Date: 16/01/2005



##### ##### #####

Para saber mais visite
http://www.ciencialist.hpg.ig.com.br


##### ##### ##### #####
Links do Yahoo! Grupos









__________________________________________________
Converse com seus amigos em tempo real com o Yahoo! Messenger
http://br.download.yahoo.com/messenger/

[As partes desta mensagem que não continham texto foram removidas]



SUBJECT: Re: [ciencialist] Re: Mulheres: Cozinha e na cama, já!!!!!!!!!!
FROM: ÿffffcdtalo Rocha <imrochaguedes@yahoo.com.br>
TO: ciencialist@yahoogrupos.com.br
DATE: 27/01/2005 15:21

Perfeitamente!
Ítalo M. R. Guedes.

rmtakata <rmtakata@altavista.net> wrote:


--- Em ciencialist@yahoogrupos.com.br, "Alvaro Augusto \(E\)"
> Bem, não sou profundo conhecedor do assunto, mas me lembro que as
> gramíneas incluem a cana-de-açúcar, um produto que tem certa
> importância na economia brasileira...

Alem do milho, trigo, arroz, painço, aveia e - o q. deve trazer o
interesse do Leo - cevada! (Ainda q. a maior parte da cerveja seja
feita de trigo e milho mesmo.)

As gramíneas são a base de alimentação humana: seja diretamente (o q.
inclui derivados como o pao), seja indiretamente (como ração para
gado: capim e farelo de milho).

[]s,

Roberto Takata





##### ##### #####

Para saber mais visite
http://www.ciencialist.hpg.ig.com.br


##### ##### ##### #####
Links do Yahoo! Grupos









__________________________________________________
Converse com seus amigos em tempo real com o Yahoo! Messenger
http://br.download.yahoo.com/messenger/

[As partes desta mensagem que não continham texto foram removidas]



SUBJECT: Re: [ciencialist] a midia e o saber p/ Sr. Taborda.
FROM: "Luiz Ferraz Netto" <leobarretos@uol.com.br>
TO: <ciencialist@yahoogrupos.com.br>
DATE: 27/01/2005 15:30

Homero diz:

>Olá Taborda, lendo esse trecho do seu texto que o ray postou, não resisti em responder que, penso exatamente igual..:-) O que é interessante, já que aparentemente estamos discutidno em "oposição"..:-)
Este em especial: "O leigo precisa, primeiro de tudo entender, que não ha mal no mundo se ele não entender fisica quantica , ou TRG."<

Bolas! Finalmente chegaram na minha praia! Esse é o maior trabalho que tenho não só na divulgação mas, principalmente, nas Feiras e palestras de que participo.

Nas Feiras há todo tipo de público e, portanto, o percentual de quem não sabe nada de Ciência encosta nos 99%; mesmo os ditos 'técnicos' (que são os que mais perguntam) têm falhas conceituais profundas sobre sua própria profissão.
Um técnico em motores não sabe o que venha a ser a interação corrente/campo, para ele é o campo do indutor que repele campo do rotor e o faz rodar --- e como mostrar prá ele que campo não repele campo? Por isso, por exemplo, acrescento entre as demonstrações um simples fio entre os pólos de um ímã em U e faço passar corrente pelo fio --- um 'cartaz' ao lado apresenta um visual que me ajuda nos 'explicamentos', mas é uma dificuldade.
Alunos nem fazem perguntas, eles não conseguem associar nada do que aprenderam em aula com o material e experimento que está sobre a mesa!
Se vc colocar dois espelhos planos fazendo ângulo e uma vela entre eles, com 6 imagens, com certeza vão ficar procurando onde está o cadavér e podem até perguntar se aquilo é para economizar velas.
Jamais irei apresentar algo de Quântica ou TRG numa Feira (mesmo porque até já coloquei aqui no Clist, quem me indicava um experimento --- nível ensino médio --- de Física Quântica para colocar no site; até hoje o site não tem nada experimental de FQ.
O meu modelo mecânico para estudo da relatividade restrita ( http://www.feiradeciencias.com.br/sala23/23_R00.asp)
deu margem a dois ou três e-mails, ou seja, nenhum em comparação com o restante do site.

Que adianta, numa Feira, aparecer um cartaz sobre "Big Bang", a Expansão do Universo e o célebre disparate do balão com bolinha de caneta para 'mostrar' a expansão. Nesse experimento os pontos das próprias bolinhas de tinta feitas com caneta 'se separam' e, se isso acontecesse realmente, suas orelhas iriam se afastando uma da outra (pois tudo se expande!). Esse experimento deveria, pelo menos ter pedacinhos de fita adesiva nos lugares das bolinhas de tinta para mostrar que, durante a expansão, as dimensões das fitas não são alteradas.

Leigo precisa ter um primeiro contato com a Ciência de modo a entender que existe algo muito facil, porém escondido, para ser percebido. Nem a inicial 'observação' passa pela cabeça dele!

A experiência 'mais chocante' que já faço nas palestras para leigos (não imp[orta a presente de alunos e professores municipais ou do 'estado') é a 'descoberta' do CG, centro de gravidade, de uma vassoura (que arrumo na hora --- peço a um dos presentes que procure e me traga uma vassoura ou rodo ali pelas dependências). Depois de explicar o que é CG, sua importância em vários projetos (super simples, claro, tal como um martelo, um giz etc.), coloco a vassoura apoiada entre meus dois dedos indicadores e vou aproximando os dedos --- é um sucesso total! A partir disso explico e encontro o centro de percursão da vassoura e discuto porque o giz ao cair quebra em 2, 3 ou 4 pedaços (e porque quebra exatamente a 45 graus se é torcido!).

Cada palestra é uma aventura ... curto isso!

aquele abraço,

Léo


--
Internal Virus Database is out-of-date.
Checked by AVG Anti-Virus.
Version: 7.0.300 / Virus Database: 265.6.13 - Release Date: 16/01/2005



SUBJECT: Re: [ciencialist] Texto de Gould p Italo (era orguloso do saber?)
FROM: "Luiz Ferraz Netto" <leobarretos@uol.com.br>
TO: <ciencialist@yahoogrupos.com.br>
DATE: 27/01/2005 15:33

"ÿffffcdtalo Rocha"

tem jeito de vc melhorar esse nick? Veja como ele nos chega "ÿffffcdtalo Rocha".

agradecemos,

Léo


--
Internal Virus Database is out-of-date.
Checked by AVG Anti-Virus.
Version: 7.0.300 / Virus Database: 265.6.13 - Release Date: 16/01/2005



SUBJECT: Re: [ciencialist] Re: Mulheres: Cozinha e na cama, já!!!!!!!!!!
FROM: "Luiz Ferraz Netto" <leobarretos@uol.com.br>
TO: <ciencialist@yahoogrupos.com.br>
DATE: 27/01/2005 15:35

"Pai, perdoa-os, eles não sabem o que fazem."
[]'
jc
-----Mensagem Original-----
De: "ÿffffcdtalo Rocha"

Meu caro, talvez não saiba, mas também são gramíneas a cana-de-açúcar, o milho e o arroz, este último, parece-me, bastante comido por aí.



--
Internal Virus Database is out-of-date.
Checked by AVG Anti-Virus.
Version: 7.0.300 / Virus Database: 265.6.13 - Release Date: 16/01/2005



SUBJECT: Quem opina?
FROM: "Luiz Ferraz Netto" <leobarretos@uol.com.br>
TO: <ciencialist@yahoogrupos.com.br>
DATE: 27/01/2005 15:41

http://pesn.com/2004/06/30/6900029PerendevPowerMagneticMotor/
[]'
===========================
Luiz Ferraz Netto [Léo]
leobarretos@uol.com.br
http://www.feiradeciencias.com.br
===========================


--
Internal Virus Database is out-of-date.
Checked by AVG Anti-Virus.
Version: 7.0.300 / Virus Database: 265.6.13 - Release Date: 16/01/2005



SUBJECT: Re: [ciencialist] Texto de Gould p Italo (era orguloso do saber?)
FROM: ÿffffcdtalo Rocha <imrochaguedes@yahoo.com.br>
TO: ciencialist@yahoogrupos.com.br
DATE: 27/01/2005 15:44

Sinceramente, não sei por que meu nick aparece assim, imagino que seja por eu ter acentuado o I de Ítalo, nem sei também como posso mudá-lo. De toda forma, chamo-me Ítalo Moraes Rocha Guedes, desculpe o incoveniente.

Luiz Ferraz Netto <leobarretos@uol.com.br> wrote:
"ÿffffcdtalo Rocha"

tem jeito de vc melhorar esse nick? Veja como ele nos chega "ÿffffcdtalo Rocha".

agradecemos,

Léo


--
Internal Virus Database is out-of-date.
Checked by AVG Anti-Virus.
Version: 7.0.300 / Virus Database: 265.6.13 - Release Date: 16/01/2005



##### ##### #####

Para saber mais visite
http://www.ciencialist.hpg.ig.com.br


##### ##### ##### #####
Links do Yahoo! Grupos










__________________________________________________
Converse com seus amigos em tempo real com o Yahoo! Messenger
http://br.download.yahoo.com/messenger/

[As partes desta mensagem que não continham texto foram removidas]



SUBJECT: Re: a midia e o saber p/ Sr. Taborda.
FROM: "rmtakata" <rmtakata@altavista.net>
TO: ciencialist@yahoogrupos.com.br
DATE: 27/01/2005 16:00


--- Em ciencialist@yahoogrupos.com.br, "Luiz Ferraz Netto"
> porque até já coloquei aqui no Clist, quem me indicava um
> experimento --- nível ensino médio --- de Física Quântica para
> colocar no site; até hoje o site não tem nada experimental de FQ.

Efeito fotoeletrico?
http://hyperphysics.phy-astr.gsu.edu/hbase/mod1.html

Davisson-Germer
http://hyperphysics.phy-astr.gsu.edu/hbase/davger.html

Franck-Hertz
http://hyperphysics.phy-astr.gsu.edu/hbase/FrHz.html

Stern-Gerlach
http://hyperphysics.phy-astr.gsu.edu/hbase/spin.html#c5

Espalhamento de Crompton
http://hyperphysics.phy-astr.gsu.edu/hbase/quantum/comptint.html

(Acho q. alguns pelo menos podem ser adaptados para o EM.)

[]s,

Roberto Takata





SUBJECT: Re: [ciencialist] a midia e o saber p/ Sr. Taborda.
FROM: "Jamil Orlandelli" <orlandel@ig.com.br>
TO: <ciencialist@yahoogrupos.com.br>
DATE: 27/01/2005 20:20

"O leigo precisa, primeiro de tudo entender, que não ha mal no mundo se ele
não entender fisica quantica , ou TRG."

Olá,
Também concordo com isso aí. Suponha que eu seja pessoa dedicada à
Literatura. Possuo outra forma de conhecimento que não é o científico, mas
tenho interesse em compreender também a ciência porque acho que qualquer
atividade que uma pessoa atue e requeira pensamento talvez seja uma pessoa a
quem a ciência diz alguma coisa. Mas vou estudar física ? biologia, TR ,MQ,
etc.? . Nâo acredito que desta forma, como leigo , vou compreender a ciência
(existem muitos especialistas em ciência que não compreendem a ciência,)
Como leigo vou procurar estudar a história da ciência, e o seu método. É
claro que a história da ciência não vai substituir a ciência, mas vai dar o
esqueleto do desenvolvimento da ciência. Vai lançar uma ponte entre a
Literatura que domino e a ciência, estabelecendo uma unidade na minha
formação.
Ou seja, vou tentar compreender a ciência como cultura descobrindo-a dentro
da minha própria cultura, ou da Arte. Acho que este é um caminho bom para os
leigos.

[Jamil] (que é licenciado em física mas adora literatura)

----- Original Message -----
From: "Oraculo" <oraculo@atibaia.com.br>
To: <ciencialist@yahoogrupos.com.br>
Sent: Thursday, January 27, 2005 1:21 PM
Subject: Re: [ciencialist] a midia e o saber p/ Sr. Taborda.



Olá Taborda

Lendo esse trecho do seu texto que o ray postou, não resisti em responder
que, penso exatamente igual..:-) O que é interessante, já que aparentemente
estamos discutidno em "oposição"..:-)

Este em especial:

"O leigo precisa, primeiro de tudo entender, que não ha mal no mundo se ele
não entender fisica quantica , ou TRG."

Exato..:-) Venho tentando dizer isso desde a primeira mensagem. Veja, nela
eu digo que para aproximar o leigo da ciência, não é preciso ensinar fisica
avançada ou biologia molecular para ele. Não faz mal não saber sobre nada
disso, pelo menos não saber profundamente.

Mas, saber como e de que forma esse tipo de conhecimento foi gerado, pode
diminuir a desconfiança natural de leigo sobre algo que não compreende. Se
este pensar que cada uma dessas teorias é fruto de viagens de imaginação,
bebedeira de cientistas, ou sonho de noite mal dormida de pesquisadores, vai
dar o mesmo valor que qualquer outra afirmação maluca que se houve na mídia
(a "terrivel" midia malvada..:-)

Entretanto, se comprender o esforço, as regras de segurança e o rigor
envolvidos na produção de conhecimento cientifico (esqueçamos por um
instante as falhas de politicagem..:-), se compreender que teoria não é algo
que alguém sobnhou depois de beber muito, se puder entender como a acrescimo
de dados vai fortalecendo (ou destruindo) um conhecimento cientifico, estará
mais perto da ciência, e nem achará mal se não compreender fisica quantica,
ou TRG.

Era isso, exatamente, que eu tentava colocar ao rayfisica...:-)

Um abraço.

Homero




----- Original Message -----
From: rayfisica
To: ciencialist@yahoogrupos.com.br
Sent: Thursday, January 27, 2005 12:03 PM
Subject: [ciencialist] a midia e o saber p/ Sr. Taborda.



Taborda--Ja foi tempo em que ele não queria entender essas coisas.
A
media faz
tanta pressão que o leigo sentese idiota por não entender essas
teorias
avançadas e recorre às fontes da propria media para se
informar. O
problema é que essas fontes são ainda mais leigas que ele , e
ele acaba
recorrendo a listas com esta. O leigo precisa, primeiro de tudo
entender, que não ha mal no mundo se ele não entender fisica
quantica ,
ou TRG.
--------------------------------------------------------------
27/01/2005 - 10h43
Universo começou com nuvens de matéria escura, dizem cientistas
SALVADOR NOGUEIRA
da Folha de S.Paulo

http://www1.folha.uol.com.br/folha/ciencia/ult306u12882.shtml

Eles foram criados no princípio dos tempos. Ainda estão por
aí,
vagando pelo Universo, atravessando corpos como se eles não
existissem
e somente em raras ocasiões interagindo com a matéria normal.
Quem
você vai chamar? Com certeza, não os Caça-Fantasmas. Mas
tente Ben
Moore, do Instituto de Física Teórica da Universidade de
Zurique, Suíça.

Usando um poderoso supercomputador para simular nada menos que a
evolução do Universo quase desde o Big Bang (começando uns 20
milhões
de anos depois da explosão que teria dado origem ao cosmos), ele
chegou à conclusão de que os primeiros objetos formados foram
nuvens
com o tamanho do Sistema Solar e a massa da Terra, feitos de
partículas que ninguém consegue ver.

Esses chamados "halos" ainda existem por aí e varrem o planeta (e
tudo
que há nele, inclusive nós) a cada poucos milhares de anos. E
há um
bocado deles lá fora. Só nos arredores da Via Láctea, a
galáxia na
qual o Sistema Solar está, haveria cerca de 1 milhão de
bilhões de
objetos como esses, se os cálculos de Moore estiverem corretos.
É 10
mil vezes mais do que o número de estrelas na mesma região.

"Eles são mais como nuvens de partículas, mais concentradas na
região
do centro", disse o cientista à Folha. Seu estudo, publicado na
edição
de hoje da revista científica britânica "Nature", ajuda a
explicar
onde está toda a matéria do Universo.

Minoria

O mistério intriga os cientistas há décadas. Ao somarem
todas as
estrelas e galáxias que podem ver, eles descobriram que essa
matéria
"comum" não responde nem por 5% de toda a massa que deveria haver
para
explicar alguns efeitos gravitacionais observados, como, por exemplo,
o ritmo de rotação de galáxias (que é mais rápido do que
deveria ser
se as estrelas e o gás fossem tudo o que há por lá).

À massa faltante os cientistas deram o auspicioso nome de
"matéria
escura", e ninguém até hoje sabe o que ela é de fato. Os
grupo de Ben
Moore aposta que ela é composta por partículas chamadas de
neutralinos, os primos ricos dos neutrinos.

"Bem, neutrinos e neutralinos são bem parecidos, ambos têm
massa e
ambos interagem bem fracamente com os átomos normais. Portanto, os
dois passam direto por nosso corpo em enormes quantidades a cada
segundo", diz Moore. "A principal diferença é a sua massa --a do
neutralino é bem maior."

Um problema para a hipótese defendida por Moore, que hoje é a
principal candidata a explicar a matéria escura, é o fato de
que,
diferente dos neutrinos, nunca ninguém conseguiu ver um
neutralino. E
a própria idéia vem da teoria de que existe uma "supersimetria"
entre
partículas, algo que também não foi confirmado.

Mas não o será por muito tempo, segundo Moore. "O Grande
Colisor de
Hádrons [LHC, na sigla em inglês], no Cern [principal
laboratório de
física de partículas na Europa], irá detectar evidências
da
supersimetria e dos neutralinos, se eles estiverem lá!"

Segundo ele, a tal supersimetria, embora não-comprovada, é uma
idéia
consolidada. "Não é tão controversa --grande parte da
motivação para
gastar os bilhões de dólares que o LHC vai custar é
demonstrar a
existência da supersimetria. Os cientistas em geral não gastam
esse
monte de dinheiro sem uma razão muito boa."

Segundo o grupo, também será possível detectar uma nuvem de
neutralinos no espaço, caso ela exista mesmo --ela emitiria raios
gama, o tipo de radiação eletromagnética mais energético do
Universo,
num padrão bem distinto, reconhecível.
------------------------------
Acho que isso demonstra (aos gritos)o que o senhor disse, eu não to
conseguindo si quer aceitar a idéia de entender o NEUTRINO agora
recebo a noticia do NEUTRALINO, o que é isso?





##### ##### #####

Para saber mais visite
http://www.ciencialist.hpg.ig.com.br


##### ##### ##### #####


Yahoo! Grupos, um serviço oferecido por:
PUBLICIDADE




------------------------------------------------------------------------------
Links do Yahoo! Grupos

a.. Para visitar o site do seu grupo na web, acesse:
http://br.groups.yahoo.com/group/ciencialist/

b.. Para sair deste grupo, envie um e-mail para:
ciencialist-unsubscribe@yahoogrupos.com.br

c.. O uso que você faz do Yahoo! Grupos está sujeito aos Termos do
Serviço do Yahoo!.



[As partes desta mensagem que não continham texto foram removidas]



##### ##### #####

Para saber mais visite
http://www.ciencialist.hpg.ig.com.br


##### ##### ##### #####
Links do Yahoo! Grupos













SUBJECT: Re: Quem opina? por favor
FROM: "rayfisica" <rayfisica@yahoo.com.br>
TO: ciencialist@yahoogrupos.com.br
DATE: 27/01/2005 22:32


--- Em ciencialist@yahoogrupos.com.br, "Luiz Ferraz Netto"
<leobarretos@u...> escreveu
> http://pesn.com/2004/06/30/6900029PerendevPowerMagneticMotor/
> []'
> ===========================
> Luiz Ferraz Netto [Léo]
> leobarretos@u...
> http://www.feiradeciencias.com.br
> ===========================
>
Eu vi o vídeo e então vou direto ao ponto:
É um P.M.M ?
Supondo se remotamente que fosse, o que aconteceria?





SUBJECT: Portugal estará em São Paulo até 13 de Março!!!
FROM: Maria Natália <grasdic@hotmail.com>
TO: ciencialist@yahoogrupos.com.br
DATE: 27/01/2005 22:49


Do SUPERNOVAS - BOLETIM BRASILEIRO DE ASTRONOMIA -
http://www.supernovas.cjb.net

""27 de Janeiro de 2005 - Edicao No. 291
CONHECA PARTE DO ACERVO DO MUSEU DE FISICA E DO OBSERVATORIO
ASTRONOMICO DA UNIVERSIDADE DE COIMBRA
Ate' 13 de marco, na Pinacoteca do Estado de SP. Na Europa do seculo
18, as demonstracoes de fisica experimental se faziam nao so' nas
Universidades, mas tambem em clubes e sociedades, salas alugadas
pelos chamados fisicos demonstradores - que viajavam com sua colecao
de instrumentos - e nas residencias. Nao por acaso, essas
experiencias ganhavam ares cenicos e o termo teatro era
frequentemente associado a eventos do tipo. Havia o Teatro das
Experiencias, o Teatro das Maquinas, o Teatro da Phisica Experimental
e o Teatro de Poleni, entre outros. "Ja' em meados do seculo 17 a
pratica experimental, como meio de descoberta e de validacao do
conhecimento, comecara a criar raizes firmes", diz Ermelinda Antunes,
pesquisadora do Depto. de Fisica da Universidade de Coimbra
(Portugal). Com esse forte componente de entretenimento na fisica da
epoca, a Pinacoteca do Estado de SP tornou-se o lugar ideal para a
exposicao "Laboratorio do mundo - ideias e saberes do seculo XVIII",
que vai ate' o dia 13 de marco e da qual Ermelinda e' a curadora.
Trata-se de uma reuniao de 212 pecas: instrumentos cientificos e
livros dos seculos 18 e 19 - cerca de 110 deles pertencentes ao Museu
de Fisica e ao Observatorio Astronomico da Universidade de Coimbra -,
mapas, quadros, gravuras e pinturas do acervo da Biblioteca Nacional
do RJ. O evento integra as comemoracoes dos 450 anos de SP e resulta
de uma parceria entre o Gabinete das Relacoes Culturais
Internacionais do Ministerio da Cultura de Portugal e a Pinacoteca. A
mostra e' o centro de varias atividades sobre historia do seculo 18.
Outro resultado dessa cooperacao podera' ser visto a partir do dia 25
de janeiro, quando sera' aberta a exposicao "Cartografia de uma
historia", em conjunto com o Museu Paulista da USP, sobre os mapas
relativos ao territorio da Capitania de SP. De todos os
eventos, "Laboratorio do mundo" e' o que mais expressa a mudanca de
paradigmas e a adocao de novas ideias numa epoca em que se comecava
muito lentamente a abandonar as antigas teorias sobre o mundo
natural, baseadas em Aristoteles. "A valorizacao da experiencia tinha
sido defendida por Francis Bacon na sua obra Novum Organum, publicada
em 1620, em que ele afirmava que 'o progresso so' poderia advir de
uma uniao proxima e estrita das faculdades racionais e experimentais,
que ate' ali nunca se uniram'", conta Ermelinda. No seculo 18 os
fenomenos naturais passaram a ser vistos como um misto de materia e
forcas e a ser descritos em linguagem matematica. Leia a reportagem
em: http://www.revistapesquisa.fapesp.br/show.php?
lang=pt&id=revistas1.fapesp1..20050103.200501107..SEC2_2. (Neldson
Marcolin, Agencia Fapesp)
Ed: CE""

Estão avisados. Mexam-se!!!
Maria Natália





SUBJECT: Ciência começa aos 6 anos!!!!
FROM: Maria Natália <grasdic@hotmail.com>
TO: ciencialist@yahoogrupos.com.br
DATE: 27/01/2005 23:21


Férias com a Ciência - Carnaval 2005

Caros amigos,


O @rrobinha desafia as crianças com idades entre os 6 e os 12 anos a
passar as férias do Carnaval com a Ciência, participando em
fantásticas aventuras: com muita ciência vem quebrar a Maldição dos
Piratas, navegando de módulo em módulo até chegar à nossa arca do
tesouro. Será que os dinossáurios têm alguma coisa a ver com a
matemática? E construir um relógio de água e um calendário que dura
toda a vida, será possível? Mascarar o @rrobinha e a construção de uma
página electrónica são outros desafios que te esperam, este Carnaval,
no Pavilhão do Conhecimento.

Datas / Preço:
7, 8 e 9 de Fevereiro - Preço: 100 Euros (Sócios: 90 Euros)

Preço por dia:
40 Euros (sócios: 35 Euros)

Horário:
Das 9h00 às 18h00 (terça-feira, dia 8, das 10h00 às 19h00)

Idade:
dos 6 aos 12 anos

Marcações:
Tel. 218 917 100
Email: info@pavconhecimento.pt

PROGRAMA
9.00 - 9.30
Recepção das crianças
9.30 - 10.00
Briefing / lanche (trazem de casa)
10.00 - 12.00
Actividades
12.00 - 13.00
Almoço (fornecido pelo Pavilhão)
13.00 - 16.00
Actividades
16.00 - 16.30
Lanche (fornecido pelo Pavilhão)
16.30 - 18.00
Actividades
18.00
Entrega aos familiares

Para mais informações:
Pavilhão do Conhecimento - Ciência Viva
Tel. 218 917 100
Email: info@pavconhecimento.pt

E de pequenino se torce o pepino!
Melhor que isto!? Só levar saco cama e dormir no fundo do Oceano
Pacífico rodeado de tubarões...
Quem já dormiu?
Um abraço
Maria Natália
PS Eu no fds? Vou subir a Serra da Arrábida. Há por aí algum português
para acompanhar?






SUBJECT: Sexo até aos 80 anos* e notícias sobre investigação em sexualidade
FROM: Maria Natália <grasdic@hotmail.com>
TO: ciencialist@yahoogrupos.com.br
DATE: 28/01/2005 00:31


E como ainda não vi aqui ninguém falar em Sexual Science, cá vão dois
assuntos sérios:
"Sex pheromone spray boosts senior romance
· 19:00 26 January 2005
· New Scientist Print Edition
· Andy Coghlan
A mystery chemical that young women deploy as a sex attractant
pheromone seems to work for post-menopausal women too.
Joan Friebely of Harvard University, US, and Susan Rako, a private
physician in Newton, Massachusetts, US, have studied 44
post-menopausal women. Half added Athena Pheromone 10:13, originally
isolated from a woman's armpit sweat, to their perfume while half
added a dummy compound. Neither the women nor the researchers knew who
was in each group until the results were in.
In diaries kept by the women for six weeks, 41% of pheromone users
reported more petting, kissing and affection with partners compared
with 14% receiving the placebo. Overall, 68% of pheromone users
reported increases in at least one of four "intimate socio-sexual
behaviours" such as formal dates and sex, as against 41% on the placebo.
But the pheromone's discoverer, biologist Winnifred Cutler, is keeping
its identity secret until patents have been granted to Cutler's Athena
Institute for Wo
men's Wellness Research in Chester Springs, Pennsylvania, US. "It's
still a mystery substance being applied to individuals at unknown
concentrations," says George Preti of the Monell Chemical Senses
Centre in Philadelphia, Pennsylvania.
Friebely and Rako say they have no financial interest in the product.
Journal reference: The Journal of Sex Research (vol 41, p 372)
Related Articles
· Pheromones soothe stressed out dogs
· 26 April 2004
· Colour vision ended human pheromone use
· 16 June 2003
· Pheromone triples women's sexual success
· 20 March 2002
Weblinks
· Harvard University
· Athena Institute
· Journal of Sex Research""
Foi especial biólogos...
*-*-*-*-*-*
E COMO POR AQUI PODE ANDAR ALGUÉM QUE SEJA INVESTIGADOR NA ÁREA (eu só
tenho pós graduação em Educação sexual em contexto escolar):
Aqui vão os tópicos que se pede para a revista e que vai sair em
Fevereiro:
""Topics could include (but are not limited to):
• types and geography of public sex
• spaces of courtship
• impact of surroundings on sexual health practices
• representations of sexuality in public spaces
• role of place in the discourse of romance
• spatial distribution of sexual practice
• state manipulation of spatial systems and sexuality
• locations of sexual economies – including analyses of "red-light
districts" and
"adult entertainment districts"
• environmental influence on sociocultural rituals
• physical spaces of sexual cultures
• globalization and sexuality
• sex and community activism
We welcome questions about submissions. Please contact Special Issue
Editors: Mindy
Thompson Fullilove or Moriah McSharry McGrath, c/o Community Research
Group, 1051
Riverside Drive, Unit 29, New York, NY 10032,
212.740.7292,mm2348@columbia.edu.
To be considered for this special issue, manuscripts must be received
by February 1, 2004. Early
submission is encouraged. An accompanying letter should include a
statement that the
manuscript has not been published and is not currently under
consideration elsewhere; letters
accompanying empirical papers should describe the ethical review
process employed by authors.
Four paper copies of the manuscript must be submitted, prepared
according to the Publication
Manual of the American Psychological Association (5th ed.).
(Over)
The Journal of Sex Research (JSR) is a scholarly journal devoted to
the publication of articles relevant to the variety of disciplines
involved in the scientific study of sexuality. JSR is published
quarterly by the Society for the Scientific Study of Sexuality. JSR is
designed to stimulate research and to promote an interdisciplinary
understanding of the diverse topics in contemporary sexual science.""
www.sexscience.org

Mais sobre este tema como Encontros/ Conferências:
http://www.sstarnet.org/upcomingMeetings.cfm
Um tópico tabu? Ora bolas para os tabus...
Maria Natália






SUBJECT: Re: Mulheres: Cozinha e na cama, já!!!!!!!!!!/Takata
FROM: Maria Natália <grasdic@hotmail.com>
TO: ciencialist@yahoogrupos.com.br
DATE: 28/01/2005 00:42


A Natália Barbuda agradece LOLLLLLL
Li uma coisa gira por aqui:
Ah a tradição cristã colocava, COLOCAVA!? E já não coloca? rs rs
Lê a epístola da missa do casamento. Mas isto já é Acropolis
Um abração
Maria Natália

--- Em ciencialist@yahoogrupos.com.br, "rmtakata" <rmtakata@a...> escreveu
>
> > > Natália: Enfim eu nem falo de inferioridade
> > > física/mental/intelectual.
>
> --- Em ciencialist@yahoogrupos.com.br, JVictor <jvoneto@u...> escreveu
> > Victor: E tem razão. Se alguma diferença há, ela não deve ser
> > procurada no aspecto físico/mental/intelectual, genético, ou o que
> > seja, da constituição do cérebro.
>
> Se deve ou nao, nao sei, mas essa diferenca pode ser procurada tb
> nesses aspectos. Aparentemente encontraram muitas diferencas ai' (a
> presenca de um X no lugar de um Y faz uma diferenca enorme - ainda
> bem, acho q. eu nao ficaria bem com duas mamas enormes, embora uma
> Naomi Campbell com barba por fazer nao seja tampouco uma visao idilica).
>
> Mas essas diferencas nao significam de modo algum inferioridade e isso
> eh preciso deixar muito claro. Significa isto sim uma diversidade. E
> viva 'as diferencas!
>
> Tampouco o fato dessas diferencas existirem significa q. seja as
> unicas causas da exclusao das mulheres (ou dos homens) em uma dada area.
>
> > No Ocidente, que sempre foi mais liberal com elas, o avanço tem sido
> > lento, mas progressivo, como podemos ver. No Oriente, ainda há
> > comportamentos e atitudes para além de medievais, conforme todos
>
> O Ocidente nao foi sempre mais liberal em relacao 'as mulheres, nao.
> Em verdade, em verdade, a tradicao grega colocava a mulher em uma
> posicao mais inferior do q. ela tinha em sociedades do Oriente Proximo
> (da Asia Menor). Outro ponto, a tradicao crista tb colocava as
> mulheres em posicao inferior - entre outras coisas pela culpa do
> pecado de Eva - em contraste com as religioes calcadas na figura da
> mulher como simbolo da fertilidade. Junte a tradicao machista grega
> com a tradicao machista crista e teremos a sociedade ocidental machista.
>
> O espaco feminino foi conquistado a ferro e a fogo no Ocidente. O
> *sempre* se refere a coisa de nao mais do q. 70 anos - e mais
> efetivamente de 40 anos para ca'.
>
> Qto ao Oriente, q. Oriente? O Oriente Medio, a Asia Central, o Sudeste
> Asiatico, o Sul da Asia, o Norte da Ásia, o Extremo Oriente?
>
> []s,
>
> Roberto Takata





SUBJECT: Aumento de temperatura
FROM: "E m i l i a n o C h e m e l l o" <chemelloe@yahoo.com.br>
TO: <ciencialist@yahoogrupos.com.br>
DATE: 28/01/2005 08:06

Temperatura da Terra pode subir até 11ºC

Quarta, 26 de janeiro de 2005, 18h54

As temperaturas na Terra podem subir até 11ºC, quase o dobro do que se
previa, segundo uma das projeções mais abrangentes já realizadas sobre o
clima, pela Universidade de Oxford, na Grã-Bretanha. Os cientistas por trás
da pesquisa, chamada Climateprediction.net ("Previsões sobre o clima", em
tradução livre), dizem que não existe um nível seguro de emissões de gás
carbônico (CO2).

O estudo, que foi publicado na revista científica Nature, usou
computadores pessoais de todo o mundo para produzir dados: em vez de usar um
supercomputador para rodar modelos climáticos, o projeto pedia que usuários
comuns de PCs baixassem um software que funciona quando o computador não
está sendo usado. Mais de 95 mil pessoas se registraram, em mais de 150
países. Somados, esses computadores rodaram mais de 60 mil simulações do
clima no futuro.

Diferenças

Cada PC roda uma simulação ligeiramente diferente da outra, e cada uma
examina o que acontece com o clima mundial quando os níveis de gás carbônico
na atmosfera são duas vezes maior que os níveis pré-industriais - o que,
segundo os cientistas, pode ocorrer em meados deste século.

O que variou mais entre as simulações foi exatamente a natureza do
processo físico, como por exemplo o das correntes de ar dentro de nuvens
tropicais, que rege o transporte do calor em torno do planeta.
Dessa forma, nenhuma simulação produziria exatamente os mesmos resultados.
De maneira geral, o projeto produz um retrato da possível abrangência de
resultados, considerando-se os conhecimentos científicos que se tem hoje. O
menor aumento de temperatura que o estudo prevê é de 2ºC, podendo chegar a
11ºC.

"Urgência"

A variação real vai depender da velocidade com que se duplicará a
quantidade de CO2. Mas grandes aumentos só devem ocorrer dentro de pelo
menos um século.

"Acredito que esses resultados sugerem que é talvez mais urgente do que
se pensa a necessidade de fazermos algo em relação às mudanças climáticas",
disse à BBC o cientista David Stainforth, um dos autores do
Climateprediction.net.

"Entretanto, com o que sabemos hoje, não podemos definir qual é o nível
seguro de gás carbônico na atmosfera". Na segunda-feira, a Força-Tarefa
Internacional para Mudanças Climáticas, anunciou que uma concentração
atmosférica de gás carbônico de mais de 400 ppm (partes por milhão) seria
"perigosa". Atualmente, esse índice está em torno dos 378 ppm, subindo cerca
de 2 ppm por ano.

Os pesquisadores por trás do Climateprediction.net acreditam que, por
ser distribuído em computadores individuais, o projeto pode ajudar a
informar as pessoas sobre o assunto - o que pode, para eles, acarretar em
mudanças políticas.

O uso de PCs para pesquisas científicas não é novidade. Projetos como o
Seti (Search for Extra-Terrestrial Intelligence ou "Busca por Inteligência
Extra-Terrestre", em tradução livre) contam com milhões de pessoas que
baixaram seu software, permitindo aos cientistas analisar dados da
observação de galáxias distantes.

BBC Brasil

Leia esta notícia no original em:
Terra - Notícias
http://noticias.terra.com.br/ciencia/interna/0,,OI461325-EI238,00.html


[ ] 's do Emiliano Chemello
emiliano@quimica.net
http://www.quimica.net/emiliano
http://www.ucs.br/ccet/defq/naeq

" Rien ne se perd, rien ne se crée,
tout se transforme."

Antoine Laurent de Lavoisier (químico francês, 1743 - 1794)




SUBJECT: Re: orguloso do saber?
FROM: "Sergio M. M. Taborda" <sergiotaborda@terra.com.br>
TO: ciencialist@yahoogrupos.com.br
DATE: 28/01/2005 08:24


--- Em ciencialist@yahoogrupos.com.br, ÿffffcdtalo Rocha
<imrochaguedes@y...> escreveu

Em primeiro lugar queria deixar o aviso a si, mas a todos em geral ,
para não fazerem top-posting (que é responder acima do texto ,
deixando o texto todo em baixo, tal como fez na mensagem a que respondo).

> Caro Sergio Taborda,
> Entretanto, contra a afirmação de que a base do
> castelo da ciência pode ser frágil e estaríamos ingenuamente
despreocupados sobre as mesmas, apresento o fato, certamente conhecido
por ambos, de a ciência, ao contrário de outors ramos do conhecimento
humano, fazer previsões do comportamento do mundo de uma forma não
casual e não poucas vezes de maneira notavelmente exata, e é
exatamente por matematizar o mundo que isto é possível, e sei que o
que digo não é novo, mas não deixa de ter conseqüências profundas.


Essas consequencias já foram mais profundas. Hoje, isso de que fala é
um dos dogmas da ciencia. Se não está em forma matemática, não tem valor.
Isto é especialemnte verdadeiro em fisica, onde a matemática é rainha,
mas já se está vendo em outros campos também.
Ora, a boa fisica não é feita com equações. A matemática não é uma
linguagem rica o suficiente para abarcar todas as veriantes de uma
teoria, e muitas vezes o que vemos são resultados particulares das
teorias.
Por exemplo, a teoria de Maxwell afirma , matemáticamente, que a luz é
o resultado de um capo electromagnético. Mas isso só é verdade
,segundo as contas, se não existir carga nem corrente no espaço
considerado. (ou melhor, gradiente de carga e rotacional de corrente)
Ora, num atomo ha carga em todo o lado e o gradiente dela não é nulo,
contudo consideramos que são fotoes que se libertam e absorvem para
que os electrões se movam das suas orbitas. Isso , não é difedigno À
matemática da coisa. Claro, que entretanto a matemática das eq de
maxwell deixa de ser válida para esse caso - que forma facil de fugir
do problema.

Ainda que a ciencia seja baseada em experiencias, as experiencias são
baseadas em conceitos subjectivos, politicos e economicos que fogem à
esfera de influencia da ciencia. A energia atomica só foi estudada por
motivos bélicos , primeiro, e motivos economicos , depois. A corrida
ao espaço é principalmente por motivos politicos e militares: toda a
gente sabe que a corrida à lua era uma disputa politica entre
americanos e russos. Hoje o dominio das tecnologias de satelites é uma
necessidade economica e politica pq ninguem quer depender dos serviços
de outrem. O sistema GPS ameriano é controlado pelos militares e não é
por acaso que a agencia aeroepacial europeia quer fazer o seu ppr GPs
, desta vez controlado por civis.
A tecnologia quimica é a maior vendida do mundo. Medicamentos ,
plasticos , alimentos , quanse tudo passa pela industria quimica em
algum ponto. A industria farmaceutica não está querendo curar doenças,
mas sim criar paliativos. A aspirina não cura a dor de cabeça, ela
remove a dor, mas não a origem da dor. Uma industria que ganha milhões
cria cada vez mais placebos já que as pessoas se sentem bem
simplesmente por tomar medicamentos, não importa quais. Existe muita
gente que não se sente bem se for ao medico e ele não receitar
medicamentos.
A ciencia , pela ciencia, como faziam Newton e outro antes dele, e
outros até ao principio do seculo 20 não existe mais.
A ciencia hoje é mais um fabrica de dinheiro. É um emprendimento
financeiro. E cmo todos, ele não pode dar prejuizo. Por isso, que
experiencias são cada vez mais atrazadas e quem manda quer ter cada
vez mais certezas que vai valer a pena invertir naquela experiencia.

O exemplo do colisor do CERN é um bom exemplo. O CERN estava na
falencia quando de repente se começou a falar num monte de particulas
novas que so poderiam ser vistas se tivessemos equipamentos melhores.
O que significa invertir dinheiro. Uma otima forma de sair da
falencia. Todos os cientistas e asperiantes fazem questão de basear
suas teorias naquilo que o colisor irá mostrar, como se houvesse
certeza da existencia das coisas antes de as ver. Isso não é ingenuo.
Isso é uma forma de tornar o colisor necewssário aos olhos de quem paga.

Fase dos tipo "vamos encontrar, se estive lá" são tipicas dos tempos
de hoje. Uma falsa modestia, uma forma de tapar o sol com a peneira.

Existem coisas muito mais importantes no mundo do que a procaria do
neutrino , do quark, ou de desse montde de lixo que ha por ai. Tuda
essa investigação faria sentido se ela podesse ser usada a nosso
favor, de alguma forma. Mas, os principais objectivos são toldados à
priori : não podemos viajar pelas estrelas pq a TR não deixa, não
podemos fazer computadores infinitamente pequenos pq a quimica não
deixa, não podemos resolver o problema da energia no mundo, pq ninguem
quer investir em educação de governos para eles entendam que é muio
mais facil do que se pensa economizar energia e conseguir fontes
alternativas de energia. Luxos como os celulares e a internet ,
exemplos da tecnologia de ponta, que temos hoje, apenas existem por
empresas particulares os usam para lucrar.

Cencia não é apenas uma questão para os senhores de bata branca que
escrevem em quadros negros com giz coisas que ninguem mais entende.
Ciencia é acima de tudo um estorvo na sociedade de hoje. Ou se vive
com ela, ou se vive sem ela. E cada vez mais se quer viver sem ela.
Queremos os luxos, mas não queremos pagar por eles. Esse é o mote dos
tempos que vivemos. E queremos ciencia, gratis. Isso seria muito bom,
mas não é possivel.

Então, cada cientista é uma maquina de dinheiro e os melhores , rendem
mais. E quem estabelece quem são os melhores ? são aqueles que mais
são vistos. Já ouviram falar em protecção de interesses ?
Uma universidade contrata um Hawkings da vida para professor
catedrático. O que esse professor rende À universidade ? Concerteza a
universidade vive de alunos, e alunos só veêm se houver professores,
mas não uns quaisquer, professores com prestigio. A universidade quer
professores com prestigio, e os professores querem prestigio para
poder entrar nas universidades , ganhando bem para o resto da vida.
E como se ganha prestigio ? Hoje em dia, o prestigio é vitual.
ÉMede-se pela quantidade de sitações , livros e aprencias na midia.
Não mais pela quantidade de papers, ou pel QUALIDADE dos papers.
Vcs acham que um peer-reviewer iria boicotar um paper do Hawkings ?
Ah! era um tiro no pé! Mas tirando isso, o sistema de peer-review é
uma máfia. É um sistema falido que vicia a propria ciencia, mas que é
necessário apenas do ponto de vista economico.
Em condições puramente cientificas, cada um publicaria os papers que
quisesse, que seriam lidos por quem interessado. As pessoas filtram o
que interessa e o que não interessa rápidamente sem precisar de
peer-review. Ai chegariamos num ponto em é necessário fazer
experiencias para que o assunto continue, e seria nesse ponto que
alguem tomaria essa incumbencia, criando novos papers que entraria no
circuito. Mas, o que vemos , é que é preciso pagar para ler um paper.
E quem recebe esse dinheiro e que fim ele toma ?
Assistimos na ciencia, o que se assitia no software e é chegada a
altura de clamar por ciencia livre , sem essas amarrações idiotas.
Um paper é melhor pq tenho que pagar por ele ? Os paper livres que
existem na net são inuteis ? Ora, todos sabemos que não é bem assim.

Tudo é realmente uma questão de orgulho. E quanto vale o orgulho dos
cientistas de hoje ? Vale o custo os seus papers, das suas
experiencias, o volume de vendas dos seus livros e palestras.

A ciencia, é um conhecimento universal, que deveria ser compartilhado
universalmente.

Sérgio Taborda







SUBJECT: Louva-a-deus no teto
FROM: "Rodrigo Marques" <rodmarq72@yahoo.com.br>
TO: ciencialist@yahoogrupos.com.br
DATE: 28/01/2005 08:48


Como nesta lista parece ter pessoas que entendem de biologia eu
aproveito para perguntar sobre a visão dos insetos, pois outro dia
na hospedagem onde passo a semana tinha um louva-a-deus (mantis) no
teto do corredor, e o bichinho apesar de estar a uns 3 metros de
altura me encarava diretamente virando a cabeça para onde eu estava
e acompanhando meu movimento, mostrando que me percebia claramente
apesar dos seus pequenos olhos. Essa criatura consegue enxergar
longe? Vê em cores?

As fêmeas do louva-a-deus também devem ser cientistas por comerem
(sentido real) seus maridos depois de serem comidas (sentido
figurado) por eles, assim os caras não prejudicam a sua produção
científica.(hehehehe)

Abraços.





SUBJECT: Re: Louva-a-deus no teto
FROM: "rmtakata" <rmtakata@altavista.net>
TO: ciencialist@yahoogrupos.com.br
DATE: 28/01/2005 09:50


--- Em ciencialist@yahoogrupos.com.br, "Rodrigo Marques"
> e acompanhando meu movimento, mostrando que me percebia claramente
> apesar dos seus pequenos olhos.

Louva-a-deus, pequenos olhos?

Nao se deixe enganar, os olhos do louva-a-deus sao relativamente
grandes - aquele pontinho preto é apenas fruto da incidencia da luz
(na verdade da absorcao - eh o ponto de q. um certo angulo de visao,
mais absorve luz).

http://home1.gte.net/cpq1cvvg/PrayingMantisBadEye.htm

Os louva-a-deus como a maioria dos insetos (na verdade, artropodos),
possui alguns olhos simples e um par de grandes olhos compostos. Os
olhos simples nao formam imagem tto qto se sabe - diferenciam apenas o
claro do escuro. Ja' os olhos compostos sao capazes de formar imagens.

http://users.rcn.com/jkimball.ma.ultranet/BiologyPages/C/CompoundEye.html

Side-to-side head movements to obtain motion depth cues: A short
review of research on the praying mantis
http://dx.doi.org/10.1016/S0376-6357(98)00007-2

Functional binocular vision is not dependent on visual experience in
the praying mantis.
http://www.ncbi.nlm.nih.gov/entrez/query.fcgi?cmd=Retrieve&db=pubmed&dopt=Abstract&list_uids==1504028

Spectral Sensitivity Studies on the Visual System of the Praying
Mantis, /Tenodera sinensis/
http://www.jgp.org/cgi/reprint/57/1/93.pdf

Responses to computer-generated visual stimuli by the male praying
mantis, Sphodromantis lineola (Burmeister)
http://dx.doi.org/10.1006/anbe.2001.1955

> Essa criatura consegue enxergar longe? Vê em cores?

Pra essas duas perguntas, nao encontrei uma resposta direta.

[]s,

Roberto Takata





SUBJECT: comente isso Sr. Homero.
FROM: "rayfisica" <rayfisica@yahoo.com.br>
TO: ciencialist@yahoogrupos.com.br
DATE: 28/01/2005 10:07


Porem espero ensinamentos de todos.

Responde o senhor:
------------------------------
Este em especial:

"O leigo precisa, primeiro de tudo entender, que não ha mal no
mundo
se ele não
entender fisica quantica , ou TRG."

Exato..:-) Venho tentando dizer isso desde a primeira mensagem. Veja,
nela eu
digo que para aproximar o leigo da ciência, não é preciso
ensinar fisica
avançada ou biologia molecular para ele. Não faz mal não
saber sobre
nada disso,
pelo menos não saber profundamente.



Na pagina enviada pelo professor Leo, temos:

http://pesn.com/2004/06/30/6900029PerendevPowerMagneticMotor/
For centuries, inventors have been claiming to come up with magnetic
motor designs that use nothing more than the power of permanent
magnets for the motive force; and for the same amount of time,
mainstream science has responded that this is impossible. "It has
been proven mathematically that no combination of permanent magnets in
any arrangement will generate power."
History tells us that what has been proven in many people's back yards
and garages does not always coincide with mathematics of the day.
Refusing to be daunted by what he considers to be petty dogmas of
academic science, inventor Michael J. Brady of Johannesburg not only
claims to have produced such a device, but reports that his company,
Perendev Power Developments Pty (Ltd) is now in process of
manufacturing it on a large scale for markets in Europe, Russia, and
Australia


Não estaria esses inventores contribuindo com a ciência ainda
que
realizando um trabalho que do ponto de vista do douto é pura
idiotice
e perda de tempo?
É lógico que se o "idiota" estiver certo aquele castelo
cairá, porem
não saira daí uma ciência mais próxima do mundo real?
Pois nenhum douto realizara esse trabalho, então não deveriam
estar
esses mesmos doutos felizes por esses "idiotas" dispostos a
dar suas
caras aos tapas, exatamente pelo bem da ciência, pois a cada erro
destes, prova se que o conhecimento atual é verdadeiro, ou atacam a
esses os doutos exatamente por saber da fragilidade do castelo?
E aquela conversa de que esses estão enganando o povo... (é
conversa
pra boiada dormir), pois ao atacá-los os doutos apenas transformam
os
em dom Quixote e ai já viu né.
Espero que os senhores possam alcançar o sentido maior de minha
mensagem, pois que para mim é muito difícil transformar o
pensamento
em palavras.
E por favor emita um comentário a respeito do possível invento
da
matéria, eu estou ansioso por isso e com certeza guardarei seu
comentário com muito carinho(hahaha)
Como disse o poeta: vai cair, vai cair...
Respeitosamente.





SUBJECT: Um mundo de telemóveis.
FROM: "Hugo Santos" <urano@netvisao.pt>
TO: <ciencialist@yahoogrupos.com.br>
DATE: 28/01/2005 10:33

Neste link <http://www.gizmo.com.au/go/3644/> pode-se ler um artigo que
fala de um estudo efectuado sobre as razões que levam as pessoas a usar
telemóveis. Segundo este, há três tipos de utilizador de telemóvel: os que
os usam para negócios e trabalho; os que os usam mais pelo estilo e imagem
social; e os que os usam só pela sensação de segurança adicional.

Achei piada ao ponto relativo aos homens serem os que ligam mais ao estilo
comparativamente com as mulheres que preferem a segurança. Dados os
preconceitos existentes, imaginaria a coisa ao contrário (como o mundo
muda...).



Hugo Santos



P.S.: Já agora, segundo este artigo, só uso o telemóvel por uma questão de
segurança, uma vez que sou completamente minimalista na minha forma de ver
os telemóveis (e eu concordo). ;)



[As partes desta mensagem que não continham texto foram removidas]



SUBJECT: Re: orguloso do saber?
FROM: "rayfisica" <rayfisica@yahoo.com.br>
TO: ciencialist@yahoogrupos.com.br
DATE: 28/01/2005 10:45


--- Em ciencialist@yahoogrupos.com.br, "Sergio M. M. Taborda"
<sergiotaborda@t...> escreveu
>

Essas consequencias já foram mais profundas. Hoje, isso de que
fala é
um dos dogmas da ciencia. Se não está em forma matemática,
não tem valor.
Isto é especialemnte verdadeiro em fisica, onde a matemática
é rainha,
mas já se está vendo em outros campos também.
Ora, a boa fisica não é feita com equações. A matemática
não é uma
linguagem rica o suficiente para abarcar todas as veriantes de uma
teoria, e muitas vezes o que vemos são resultados particulares das
teorias.
Por exemplo, a teoria de Maxwell afirma , matemáticamente, que a
luz é
o resultado de um capo electromagnético. Mas isso só é
verdade
,segundo as contas, se não existir carga nem corrente no espaço
considerado. (ou melhor, gradiente de carga e rotacional de corrente)
Ora, num atomo ha carga em todo o lado e o gradiente dela não é
nulo,
contudo consideramos que são fotoes que se libertam e absorvem para
que os electrões se movam das suas orbitas. Isso , não é
difedigno À
matemática da coisa. Claro, que entretanto a matemática das eq
de
maxwell deixa de ser válida para esse caso - que forma facil de
fugir
do problema.




>>>>>Parece me que isso não é novo pois eu li em algum lugar
que ao
não conseguir resolver satisfatoriamente o paradoxo de Zenão
com a
matemática disseram "esse assunto pertence a física e
não a matemática".



A ciencia , pela ciencia, como faziam Newton e outro antes dele, e
outros até ao principio do seculo 20 não existe mais.
A ciencia hoje é mais um fabrica de dinheiro. É um emprendimento
financeiro. E cmo todos, ele não pode dar prejuizo. Por isso, que
experiencias são cada vez mais atrazadas e quem manda quer ter cada
vez mais certezas que vai valer a pena invertir naquela experiencia.



>>>penso eu que ainda por cima o coitado periga ganhar um ignobel,
alias não dá pra duvidar de um premio desses, não seria uma
forma de
controle.




A ciencia, é um conhecimento universal, que deveria ser
compartilhado
universalmente.

Sérgio Taborda

>>>Sem comentario





SUBJECT: Re: [ciencialist] OT - dicas do Google
FROM: Chicao Valadares <chicaovaladares@yahoo.com.br>
TO: ciencialist@yahoogrupos.com.br
DATE: 28/01/2005 12:37

tem um livro da o´really chaMado google hacks, que tem
isso e muito mais dicas...


--- E m i l i a n o C h e m e l l o
<chemelloe@yahoo.com.br> escreveu:

---------------------------------
Use o Google do jeito certo
http://pcworld.uol.com.br/AdPortalV3/adCmsDocumentoShow.aspx?documento=8205577&Area=435000

[ ] 's do Emiliano Chemello
emiliano@quimica.net
http://www.quimica.net/emiliano
http://www.ucs.br/ccet/defq/naeq

" Rien ne se perd, rien ne se crée,
tout se transforme."

Antoine Laurent de Lavoisier (químico francês,
1743 - 1794)




##### ##### #####

Para saber mais visite
http://www.ciencialist.hpg.ig.com.br


##### ##### ##### #####


Yahoo! Grupos, um serviço oferecido por:

S&atilde;o Paulo
Rio de Janeiro
Curitiba Porto
Alegre Belo Horizonte
Bras&iacute;lia


---------------------------------
Links do Yahoo! Grupos

Para visitar o site do seu grupo na web, acesse:
http://br.groups.yahoo.com/group/ciencialist/

Para sair deste grupo, envie um e-mail para:
ciencialist-unsubscribe@yahoogrupos.com.br

O uso que você faz do Yahoo! Grupos está sujeito
aos Termos do Serviço do Yahoo!.


=====
"O Binômio de Newton é tão belo como a Vênus de Milo.
O que há é pouca gente para dar por isso... "
Fernando Pessoa - Poesias de Alvaro Campos

_________________________________________________________________
As informações existentes nessa mensagem e no(s) arquivo(s) anexado(s)
são
para uso restrito, sendo seu sigilo protegido por lei. Caso não seja
destinatário, saiba que leitura, divulgação ou cópia são proibidas.
Favor
apagar as informações e notificar o remetente. O uso impróprio será
tratado
conforme as normas da empresa e a legislação em vigor. Agradecemos sua
colaboração.


The information mentioned in this message and in the archives attached
are
of restricted use, and its privacy is protected by law. If you are not
the
addressee, be aware that reading, disclosure or copy are forbidden.
Please
delete this information and notify the sender. Inappropriate use will
be
tracted according to company's rules and valid laws. Thank you for your
cooperation.





_______________________________________________________
Yahoo! Acesso Grátis - Instale o discador do Yahoo! agora. http://br.acesso.yahoo.com/ - Internet rápida e grátis


SUBJECT: Re: [ciencialist] Re: orguloso do saber?
FROM: ÿffffcdtalo Rocha <imrochaguedes@yahoo.com.br>
TO: ciencialist@yahoogrupos.com.br
DATE: 28/01/2005 13:12

Caro Sérgio Taborda, o que vc diz em sua mensagem realmente é, infelizmente, o que acontece em boa parte dos casos, a ciência como outras manifestações culturais, principalmente da civilização ocidental, tem se submetido sem questionar à visão míope da sociedade consumista. Não apenas na física, mas principalmente nas ciências biológicas, com o boom da biotecnologia, genômica, proteômica, em que antes de publicar qualquer coisa o pesquisador parece fundar uma empresa e patentear o que quer que tenha encontrado, por mais ridículo que possa parecer, patenteiam-se genes, o que chega a ser trágico. Acredito que tem completa razão na maior parte do que diz, no entanto, quanto ao que vc chama de "máfia do peer-review" e como fazer para acabá-la tenho algumas dúvidas: atuo como revisor de uma publicação em minha área e os papers que me chegam não vêm com os nomes dos autores, sei que isto é uma norma dó periódico, claro que os editores têm estes nomes e eles é que decidem quanto à
publicação do paper, porém pareceu-me que vc criticava os revisores, desculpe se entendi mal, de toda forma sua sugestão creio que criaria um caos no meio científico, além do mais não creio que todos os editores são tão corruptíveis nem que haja tantas "autoridades" científicas tão "publicáveis" quanto o Stephen Hawking para questionar toda a instituição do peer-review.
Ítalo Moraes Rocha Guedes.

__________________________________________________
Converse com seus amigos em tempo real com o Yahoo! Messenger
http://br.download.yahoo.com/messenger/

[As partes desta mensagem que não continham texto foram removidas]



SUBJECT: Re: orguloso do saber?
FROM: "Sergio M. M. Taborda" <sergiotaborda@terra.com.br>
TO: ciencialist@yahoogrupos.com.br
DATE: 28/01/2005 15:33


--- Em ciencialist@yahoogrupos.com.br, ÿffffcdtalo Rocha
<imrochaguedes@y...> escreveu

>Acredito que tem completa razão na maior parte do que diz, no
entanto, quanto ao que vc chama de "máfia do peer-review" e como fazer
para acabá-la tenho algumas dúvidas: atuo como revisor de uma
publicação em minha área e os papers que me chegam não vêm com os
nomes dos autores, sei que isto é uma norma dó periódico, claro que os
editores têm estes nomes e eles é que decidem quanto à
publicação do paper, porém pareceu-me que vc criticava os revisores,
desculpe se entendi mal,

Eles decidem a publicação baseados na _sua_ revisão. Contudo, eles so
enviam os trabalhos para revisão se precisarem. Como vc disse, eles
sabem quem é o autor, e isso pode muitas vezes bastar (o celebre caso
Eisntein-Bose). Agora, por outro lado, se vc dicidir que o trabalho
não merece atenção eles não o publicaram. Basta que vc faça um critica
nos seguintes termos: "Segundo a teoria X o que fulano diz não pode
ser possivel" , tal como : "Segundo a termodinamica, um motor como
este não pode ser possivel, pois violaria a 1 e 2 leis da
termodinaica". Vc não tem que provar que realmente viola, vc so tem
que dizer e lastrear isso com boa argumentação.
Os editores não sabem o suficiente para duvidar de si, ou eles mesmos
seriam peer-reviwers, pelo que eles aceitam o que vcs diz, e não
publicam o trabalho.
Por outro lado, vc pode não ter entendido o trabalho e mesmo depois de
consultar com o autor, vc pode continuar não entendendo, pelo que o
trabalho é vetado. Ele pode ser enviado a outro peer, claro. Mas não a
muitos mais, pois a revista não tem tempo para esses testes.
Vc pode não perceber o trabalho por muitos motivos: pq está escrito
numa linguagem matemática que vc não entende, pq é baseado em teorias
de que vc nunca ouviu falar, ou simplesmente pq o autor escreve mal.

Existem muitos motivos para um trabalho ser vetado que nada têm a ver
com a qualidade da ciencia neles, mas apenas com burucracia.A final, o
objectivo é a publicação, e toda a publicação obdece a um molde.

>de toda forma sua sugestão creio que criaria um caos no meio científico,

Não. Antigamente não existiam peer-reviwers e os trabalhos eram
publicados e a ciencia avançava. Mesmos os trabalhos
nada-a-ver-com-nada era publicados. Claro que estes eram rapidamente
esquecidos, enquanto outros como os trabalhos de Newton, Copernico,
Kepler, Faraday , entre outros, eram aproveitados.
A ppr comunidade cientifica tem capacidade de filtrar o trigo do joio
sem que isso caia nos ombros de uma so pessoa.
O facto é que a ciencia não precisa de peer-review, são as revistas ,
a editoras, e principalemnte, os editores, que precisam: pois eles
podem perceber de edição, mas não de ciencia. (ou seja, são leigos e
portanto a ciencia é controlada por leigos que não entendem o que está
em causa) mas não querem perder dinheiro , ou reputação (que equivale
a dinheiro)
O sistema de peer-review é necesário apenas para a protecção dos
interesses das revistas , não da ciencia. Já pensou se a nature
publica algo fajuto ? ela deixa de ser comprada. Isso já aconteceu,
mas sempre foi abafado. E cada vez mais os artigos são eclisiásticos
para que ninguem os passa por em duvida em tempo util.

Tudo isso não é ciencia real, é burucracia. E a ciencia real perde-se
em burucracias desse tipo.

Quanto do que o Hawkings escreve nos seus livros está publicado na
nature, ou outras ?


Sérgio Taborda









SUBJECT: Re: orguloso do saber?
FROM: Manuel Bulcão <manuelbulcao@uol.com.br>
TO: ciencialist@yahoogrupos.com.br
DATE: 28/01/2005 19:11


Salve, Sérgio!

Sérgio: A ciencia, pela ciencia, como faziam Newton e outro antes
dele, e outros até ao principio do seculo 20 não existe mais. A
ciencia hoje é mais um fabrica de dinheiro. É um emprendimento
financeiro.

Manuel: Há uma grande verdade nesta sua asserção. De fato, com o
advento da revolução científico-tecnológica -- que se deu com a
invenção do motor a explosão, da máquina-ferramenta, da correia de
transmissão e da gerência científica -- a ciência se converteu
num "fator de produção" ou, para usar o jarguão marxista, num novo
componente das "forças produtivas". A própria produção do
conhecimento científico vem se transformando num setor da indústria
cada vez mais autônomo conhecido como P&D: Pesquisa e
desenvolvimento.

Ora, sendo uma produção voltada primacialmente para o lucro como
qualquer empreendimento capitalista - que privilegia o valor-de-
troca do produto (no caso, o valor-de-troca do saber) em detrimento
do seu valor de uso -, relegando assim a um segundo plano a
satisfação do nosso instinto epistemofílico, é normal que se
valorize mais o "poder" de predição do que a "estética" da
explicação. Aliás, como o conhecimento atrelado à economia
capitalista implica "poder" mundano cuja unidade de medida é o
dólar, para alguns mais comprometidos com o sistema é uma benção a
inevitável dificuldade da linguagem técnica da ciência; pois, assim,
restringe-se o saber e, em conseqüência, monopoliza-se a riqueza que
resulta deste conhecimento.

Talvez seja por isso que os maiores divulgadores da ciência ou
ciencistas preocupados em divulgar a ciência -- Carl Sagan, Stephen
Gould, Dawkins, Paul Davies, Lynn Margulis, Penrose, Jacques Monod,
Galbraith (no que diz respeito à ciência econômica), etc. -- têm uma
formação liberal (no sentido americano do termo) ou mesmo de
esquerda. Isto é, são críticos do status quo.

Abraços,
Manuel Bulcão





SUBJECT: Prova científica?
FROM: "Oraculo" <oraculo@atibaia.com.br>
TO: <ciencialist@yahoogrupos.com.br>
DATE: 29/01/2005 05:14

http://www.evoluindo.biociencia.org/prova.htm

Prova científica?

Douglas Theobald, PhD, 2003.

O que se entende por evidência científica e prova científica? Na verdade, a ciência não pode estabelecer "verdade" ou "fato" no sentido em que um enunciado científico pode ser inquestionável. Todos os enunciados científicos e conceitos estão abertos à reavaliação conforme novos dados são adquiridos e novas tecnologias emergem. Uma "prova", então, só existe no reino da matemática e da lógica. Quer dizer, nós freqüentemente ouvimos mencionar "prova" em um contexto científico, e existe um sentido que denota "fortemente suportado por meios científicos". Mesmo que alguém ouça "prova" em situações como estas, é um uso descuidado e inexato do termo. Conseqüentemente, exceto em relação à matemática, esta é a última vez que você vai ler o termo "prova" neste artigo.


Senso comum não é ciência

Apesar da ciência formalmente não estabelecer uma verdade absoluta, ela pode fornecer impressionantes evidências a favor de certas idéias. Freqüentemente estas idéias não são óbvias, e normalmente elas confrontam com o senso comum. O senso comum nos diz que a Terra é chata, que o Sol realmente nasce e se põe, que a superfície da Terra não está girando a mais de 1000 milhas por hora, que as bolas de boliche caem mais depressa que bolas de gude, que partículas não se curvam na extremidade como ondas ao redor de uma doca flutuante, que os continentes não se movem, e que os objetos mais pesados que o ar não podem sustentar vôo a menos que batam asas. Entretanto, a ciência tem sido utilizada para demonstrar que todas essas idéias do senso comum estão erradas.


A ciência fornece evidência para o que não é observável

A função primária da ciência é demonstrar a existência de fenômenos que não podem ser observados diretamente. A ciência não precisa nos mostrar coisas que não podemos ver com nossos olhos. A observação direta não é somente desnecessária para a ciência; a observação direta é, de fato, normalmente impossível para coisas que realmente importam. Por exemplo, as descobertas mais importantes da ciência só podem ser inferidas via observação indireta, incluindo coisas como átomos, elétrons, vírus, bactérias, germes, ondas de rádio, raios-X, luz ultravioleta, energia, entropia, entalpia, fusão solar, genes, enzimas, e a dupla hélice do DNA. A forma arredondada da Terra não foi observada diretamente por humanos até 1961, já o conceito contraintuitivo tem sido considerado um fato científico há mais de 2000 anos. A hipótese de Copérnico de que a Terra órbita o Sol tem sido reconhecida desde os tempos de Galileu, embora ninguém tivesse observado o processo até hoje, e a despeito do fato de que a observação direta indica o exato oposto. Todas estas inferências "invisíveis" foram elucidadas utilizando-se o método científico. Quando o termo "evidência" é usado neste artigo, é estritamente no contexto deste método científico.


O método científico: mais que mera experimentação

O que é o método científico? Esta é uma pergunta complexa e contenciosa, e o campo de investigação conhecido como filosofia da ciência está comprometido em iluminar a natureza do método científico. Provavelmente o filósofo da ciência mais influente do século XX foi Sir Karl Popper. Outros notáveis são Thomas Kuhn, Imre Lakatos, Paul Feyerabend, Paul Kitcher, A. F. Chalmers, Wesley Salmon e Bas C. van Fraassen. Este não é o lugar para nos aprofundarmos na explicação das várias filosofias representadas por estes pensadores; para mais informação, eu sugiro a vocês seus trabalhos e a discussão apresentada por John Wilkins em sua FAQ em Evolução e Filosofia. Pessoalmente eu emprego uma visão experimentalista e comparativa Bayesiana do método científico (Salmon, 1990; Mayo, 1996), e será através desta que eu apresentarei as evidências para origem comum.

Agora vamos responder a questão "O que é o método científico?" - muito simplificadamente (e um pouco infantil), o método científico é um programa para pesquisa que compreende quatro passos principais. Na prática estes passos seguem mais uma ordem lógica que cronológica:

1 - fazer observações;
2 - formar uma hipótese testável para explicar as observações;
3 - deduzir predições a partir das hipóteses;
4 - buscar confirmações das predições; se as predições contradizem a observação empírica, volta-se ao passo 2.

Uma vez que os cientistas estão constantemente fazendo novas observações e testando através daquelas observações, os quatro passos são normalmente praticados ao mesmo tempo. Novas observações, embora não preditas, podem ser explicáveis retrospectivamente pela hipótese. Novas informações, especialmente detalhes de alguns processos previamente não compreendidos, podem impor novos limites à hipótese original. Então, novas informações, combinadas com uma hipótese antiga, freqüentemente leva a novas predições que podem ser testadas mais tarde.

O exame do método científico revela que a ciência envolve muito mais que empirismo crédulo. Pesquisas que envolvem somente observações, repetições e medidas não são suficientes para contar como ciência. Estas 3 técnicas são meramente parte do processo de fazer observações (passo 1, acima). Astrólogos, wiccas, alquimistas e xamãs observam, repetem e medem - mas eles não fazem ciência. O que distingue a ciência é claramente o modo como as observações são interpretadas, testadas e utilizadas.


A hipótese testável

A característica que define a ciência é o conceito da hipótese testável. Uma hipótese testável deve fazer predições que podem ser validadas por observadores independentes. Por "testável" queremos dizer que as predições devem incluir exemplos do que deve e o que não deve ser observado se a hipótese for verdadeira. Uma hipótese que pode explicar todas as observações possíveis e os dados não são testáveis, não é científica. Uma boa hipótese científica deve rejeitar algumas possibilidades concebíveis, pelo menos em princípio. Além disso, uma explicação científica precisa fazer predições de risco - as predições deveriam ser necessárias se a teoria estiver correta, e poucas outras teorias deveriam fazer as mesmas predições necessárias. Estes requisitos científicos são a essência da falseabilidade e corroboração de Popper.

Por exemplo, a hipótese solipsista de que o universo todo é na verdade uma invenção de sua imaginação não é uma hipótese científica. O solipsismo não faz predições específicas ou de resco, simplesmente prediz que as coisas são "como elas são". Nenhuma observação possível poderia conflitar com o solipsismo, uma vez que todas as observações sempre podem ser explicadas como simplesmente outra criação de nossa imaginação. Muitos outros exemplos extremos podem ser pensados, como a hipótese de que o universo surgiu subitamente no todo, cinco minutos atrás, com todas as nossas memórias de eventos anteriores intactas. Em geral, criacionismo e "projeto inteligente" (ID) falham cientificamente por estas mesmas razões, uma vez que ambos podem facilmente explicar todas as observações biológicas possíveis, uma vez que não há predições arriscadas nem específicas.

Em contraste, a teoria científica da gravitação universal de Newton prediz que a força entre duas massas deve ser inversamente proporcional ao quadrado da distância entre elas (também conhecida como "lei do quadrado inverso"). A princípio, nós podemos tomar as medidas que indicam que a força é realmente inversamente proporcional ao cubo da distância. Tal observação seria inconsistente com a predição da teoria da gravitação universal de Newton, e desta forma, a teoria é falseável. Anti-evolucionistas, como os criacionistas "científicos", são especialmente fãs de Karl Popper e seu critério de falseabilidade, e são bem conhecidos por afirmar que a teoria evolutiva não é científica porque não pode ser falseada. Neste artigo, estas acusações são desmascaradas. Cada uma das evidências apontadas para a origem comum contém uma seção mostrando exemplos de potenciais refutações, isto é, exemplos de observações que são previstos como não observáveis caso a teoria esteja correta.


Graus de testabilidade: hipóteses, teorias, fatos

A testabilidade não é um conceito este-ou-aquele; algumas hipóteses são mais testáveis que outras. Ao contrário do que alguns anti-evolucionistas argumentam, nem todas as hipóteses são igualmente válicas como "interpretações" científicas da evidência. Algumas hipóteses tem mais sussesso em termos de método científico. Baseado no método científico, uma hipótese que simplesmente e elegantemente explica os fatos observados, que prediz muitos fenômenos previamente não observados, e que resiste a muitas refutações potenciais é considerada uma hipótese válida e útil. Da perspectiva Bayesiana e de acordo com as medidas de corroboração de Popper, a melhor hipótese disponível é aquela que explica a maioria dos fatos com poucas suposições. É aquela que faz a maioria das predições que são confirmadas, e aquela que é mais aberta a testes e refutações.

Na prática científica, uma hipótese superior e bem suportada será considerada uma teoria. Uma teoria é aquela que resistiu ao teste do tempo e a coleção de novos dados tende a torna-la um fato científico. Um exemplo é a antes mencionada noção heliocêntrica do sistema solar. Antigamente era uma mera hipótese. Embora seja ainda uma teoria formalmente bem suportada, validada por muitas linhas independentes de evidências, atualmente é amplamente reconhecida como um fato científico. Ninguém jamais observou diretamente um elétron, uma fusão estelar, ondas de rádio, entropia, ou a Terra girando em torno do Sol, no entanto estes são todos fatos científicos. Como Stephen J. Gould dizia, um fato científico não é uma "certeza absoluta", mas simplesmente uma teoria que tem sido "confirmada em tal grau que seria perverso reter o consentimento provisório".


Testar envolve uma totalidade de evidências e estatísticas

A validade de uma hipótese não permanece ou cai baseada em apenas algumas confirmações ou contradições, mas na totalidade da evidência. Freqüentemente os dados que inicialmente poderiam ser vistos como inconsistentes com a teoria, na realidade levarão a novas e importantes predições. A história da física de Newton é um exemplo claro. O movimento anormal de Urano foi inicialmente considerado uma refutação potencial da nova teoria de Newton. Entretanto, pelo argumento da existência de um planeta não visto, a anomalia foi explicada dentro do paradigma de Newton. Em geral, uma explicação para um comportamento anômalo deve ser considerado "ad hoc" a menos que seja verificado independentemente. Apresentar a posição de um novo e não visto planeta pode ser considerado um escape se não houver nenhuma maneira independente de detectar se um novo planeta realmente existe. Todavia, quando a tecnologia avançou o suficiente para testar a nova predição, o planeta não visto foi encontrado e chamado de Netuno.

A lição que se deve aprender é que explicações alternativas para "anomalias" devem ser tratadas como qualquer outra hipótese: devem ser pesadas, testadas e eliminadas ou confirmadas. Mas uma hipótese não pode ser considerada refutada até que seus testes produzam muitas linhas de evidências positivas indicando que a hipótese é realmente inconsistente com os dados empíricos.

Um ponto relacionado e crucial é que as teorias científicas modernas são probabilísticas. Isto significa que todos os testes de predições científicas são conduzidos em um sistema estatístico. A probabilidade e a estatística impregnam as teorias científicas modernas, incluindo a termodinâmica (mecânica estatística), geologia, mecânica quântica, genética e medicina. A matemática da probabilidade é uma disciplina que muitas pessoas acham, eu diria, desagradável. Entretanto, um bom conhecimento em estatística é absolutamente essencial para julgar a relação entre os dados observados e as predições de uma teoria.

Referências:

Chalmers, A. F. (1982) What is this thing called Science? Queensland, Australia, University of Queensland Press.

Stephen J. Gould (1981) "Evolution as Fact and Theory." Discover. May issue.

Kuhn, T. (1970) The Structure of Scientific Revolutions.

Lakatos, I. (1974) "Falsification and the Methodology of Scientific Research Progammes." Criticism and the Growth of Knowledge. I. Lakatos and A. Musgrave. Cambridge, Cambridge University Press: 91-196.

Mayo, D. (1996) Error and the Growth of Experimental Knowledge. Chicago, University of Chicago Press.

Popper, K. R. (1968) The Logic of Scientific Discovery. London, Hutchinson.

Salmon, W. (1990) "Rationality and Objectivity in Science, or Tom Kuhn meets Tom Bayes." Scientific Theories. C. W. Savage. Minneapolis, University of Minnesota Press. 14.

von Fraassen, B. C. (1980) The Scientific Image. Oxford, Clarendon Press.


[As partes desta mensagem que não continham texto foram removidas]



SUBJECT: Re: [ciencialist] Prova científica?
FROM: "Jamil Orlandelli" <orlandel@ig.com.br>
TO: <ciencialist@yahoogrupos.com.br>
DATE: 29/01/2005 08:28

Olá
Comentário de leigo com relação ao "senso comum não é ciência." As idéias
de ordem, causa e acaso, têm aplicação na ciência, me parece até que são
centrais à ciência, e no entanto são ideias do senso comum. São mais velhas
que a ciência . São as idéias que nos ajudam a governar a vida.

[Jamil]
----- Original Message -----
From: "Oraculo" <oraculo@atibaia.com.br>
To: <ciencialist@yahoogrupos.com.br>
Sent: Saturday, January 29, 2005 4:14 AM
Subject: [ciencialist] Prova científica?



http://www.evoluindo.biociencia.org/prova.htm

Prova científica?

Douglas Theobald, PhD, 2003.

O que se entende por evidência científica e prova científica? Na verdade, a
ciência não pode estabelecer "verdade" ou "fato" no sentido em que um
enunciado científico pode ser inquestionável. Todos os enunciados
científicos e conceitos estão abertos à reavaliação conforme novos dados são
adquiridos e novas tecnologias emergem. Uma "prova", então, só existe no
reino da matemática e da lógica. Quer dizer, nós freqüentemente ouvimos
mencionar "prova" em um contexto científico, e existe um sentido que denota
"fortemente suportado por meios científicos". Mesmo que alguém ouça "prova"
em situações como estas, é um uso descuidado e inexato do termo.
Conseqüentemente, exceto em relação à matemática, esta é a última vez que
você vai ler o termo "prova" neste artigo.


Senso comum não é ciência

Apesar da ciência formalmente não estabelecer uma verdade absoluta, ela pode
fornecer impressionantes evidências a favor de certas idéias. Freqüentemente
estas idéias não são óbvias, e normalmente elas confrontam com o senso
comum. O senso comum nos diz que a Terra é chata, que o Sol realmente nasce
e se põe, que a superfície da Terra não está girando a mais de 1000 milhas
por hora, que as bolas de boliche caem mais depressa que bolas de gude, que
partículas não se curvam na extremidade como ondas ao redor de uma doca
flutuante, que os continentes não se movem, e que os objetos mais pesados
que o ar não podem sustentar vôo a menos que batam asas. Entretanto, a
ciência tem sido utilizada para demonstrar que todas essas idéias do senso
comum estão erradas.


A ciência fornece evidência para o que não é observável

A função primária da ciência é demonstrar a existência de fenômenos que não
podem ser observados diretamente. A ciência não precisa nos mostrar coisas
que não podemos ver com nossos olhos. A observação direta não é somente
desnecessária para a ciência; a observação direta é, de fato, normalmente
impossível para coisas que realmente importam. Por exemplo, as descobertas
mais importantes da ciência só podem ser inferidas via observação indireta,
incluindo coisas como átomos, elétrons, vírus, bactérias, germes, ondas de
rádio, raios-X, luz ultravioleta, energia, entropia, entalpia, fusão solar,
genes, enzimas, e a dupla hélice do DNA. A forma arredondada da Terra não
foi observada diretamente por humanos até 1961, já o conceito
contraintuitivo tem sido considerado um fato científico há mais de 2000
anos. A hipótese de Copérnico de que a Terra órbita o Sol tem sido
reconhecida desde os tempos de Galileu, embora ninguém tivesse observado o
processo até hoje, e a despeito do fato de que a observação direta indica o
exato oposto. Todas estas inferências "invisíveis" foram elucidadas
utilizando-se o método científico. Quando o termo "evidência" é usado neste
artigo, é estritamente no contexto deste método científico.


O método científico: mais que mera experimentação

O que é o método científico? Esta é uma pergunta complexa e contenciosa, e o
campo de investigação conhecido como filosofia da ciência está comprometido
em iluminar a natureza do método científico. Provavelmente o filósofo da
ciência mais influente do século XX foi Sir Karl Popper. Outros notáveis são
Thomas Kuhn, Imre Lakatos, Paul Feyerabend, Paul Kitcher, A. F. Chalmers,
Wesley Salmon e Bas C. van Fraassen. Este não é o lugar para nos
aprofundarmos na explicação das várias filosofias representadas por estes
pensadores; para mais informação, eu sugiro a vocês seus trabalhos e a
discussão apresentada por John Wilkins em sua FAQ em Evolução e Filosofia.
Pessoalmente eu emprego uma visão experimentalista e comparativa Bayesiana
do método científico (Salmon, 1990; Mayo, 1996), e será através desta que eu
apresentarei as evidências para origem comum.

Agora vamos responder a questão "O que é o método científico?" - muito
simplificadamente (e um pouco infantil), o método científico é um programa
para pesquisa que compreende quatro passos principais. Na prática estes
passos seguem mais uma ordem lógica que cronológica:

1 - fazer observações;
2 - formar uma hipótese testável para explicar as observações;
3 - deduzir predições a partir das hipóteses;
4 - buscar confirmações das predições; se as predições contradizem a
observação empírica, volta-se ao passo 2.

Uma vez que os cientistas estão constantemente fazendo novas observações e
testando através daquelas observações, os quatro passos são normalmente
praticados ao mesmo tempo. Novas observações, embora não preditas, podem ser
explicáveis retrospectivamente pela hipótese. Novas informações,
especialmente detalhes de alguns processos previamente não compreendidos,
podem impor novos limites à hipótese original. Então, novas informações,
combinadas com uma hipótese antiga, freqüentemente leva a novas predições
que podem ser testadas mais tarde.

O exame do método científico revela que a ciência envolve muito mais que
empirismo crédulo. Pesquisas que envolvem somente observações, repetições e
medidas não são suficientes para contar como ciência. Estas 3 técnicas são
meramente parte do processo de fazer observações (passo 1, acima).
Astrólogos, wiccas, alquimistas e xamãs observam, repetem e medem - mas eles
não fazem ciência. O que distingue a ciência é claramente o modo como as
observações são interpretadas, testadas e utilizadas.


A hipótese testável

A característica que define a ciência é o conceito da hipótese testável. Uma
hipótese testável deve fazer predições que podem ser validadas por
observadores independentes. Por "testável" queremos dizer que as predições
devem incluir exemplos do que deve e o que não deve ser observado se a
hipótese for verdadeira. Uma hipótese que pode explicar todas as observações
possíveis e os dados não são testáveis, não é científica. Uma boa hipótese
científica deve rejeitar algumas possibilidades concebíveis, pelo menos em
princípio. Além disso, uma explicação científica precisa fazer predições de
risco - as predições deveriam ser necessárias se a teoria estiver correta, e
poucas outras teorias deveriam fazer as mesmas predições necessárias. Estes
requisitos científicos são a essência da falseabilidade e corroboração de
Popper.

Por exemplo, a hipótese solipsista de que o universo todo é na verdade uma
invenção de sua imaginação não é uma hipótese científica. O solipsismo não
faz predições específicas ou de resco, simplesmente prediz que as coisas são
"como elas são". Nenhuma observação possível poderia conflitar com o
solipsismo, uma vez que todas as observações sempre podem ser explicadas
como simplesmente outra criação de nossa imaginação. Muitos outros exemplos
extremos podem ser pensados, como a hipótese de que o universo surgiu
subitamente no todo, cinco minutos atrás, com todas as nossas memórias de
eventos anteriores intactas. Em geral, criacionismo e "projeto inteligente"
(ID) falham cientificamente por estas mesmas razões, uma vez que ambos podem
facilmente explicar todas as observações biológicas possíveis, uma vez que
não há predições arriscadas nem específicas.

Em contraste, a teoria científica da gravitação universal de Newton prediz
que a força entre duas massas deve ser inversamente proporcional ao quadrado
da distância entre elas (também conhecida como "lei do quadrado inverso"). A
princípio, nós podemos tomar as medidas que indicam que a força é realmente
inversamente proporcional ao cubo da distância. Tal observação seria
inconsistente com a predição da teoria da gravitação universal de Newton, e
desta forma, a teoria é falseável. Anti-evolucionistas, como os
criacionistas "científicos", são especialmente fãs de Karl Popper e seu
critério de falseabilidade, e são bem conhecidos por afirmar que a teoria
evolutiva não é científica porque não pode ser falseada. Neste artigo, estas
acusações são desmascaradas. Cada uma das evidências apontadas para a origem
comum contém uma seção mostrando exemplos de potenciais refutações, isto é,
exemplos de observações que são previstos como não observáveis caso a teoria
esteja correta.


Graus de testabilidade: hipóteses, teorias, fatos

A testabilidade não é um conceito este-ou-aquele; algumas hipóteses são mais
testáveis que outras. Ao contrário do que alguns anti-evolucionistas
argumentam, nem todas as hipóteses são igualmente válicas como
"interpretações" científicas da evidência. Algumas hipóteses tem mais
sussesso em termos de método científico. Baseado no método científico, uma
hipótese que simplesmente e elegantemente explica os fatos observados, que
prediz muitos fenômenos previamente não observados, e que resiste a muitas
refutações potenciais é considerada uma hipótese válida e útil. Da
perspectiva Bayesiana e de acordo com as medidas de corroboração de Popper,
a melhor hipótese disponível é aquela que explica a maioria dos fatos com
poucas suposições. É aquela que faz a maioria das predições que são
confirmadas, e aquela que é mais aberta a testes e refutações.

Na prática científica, uma hipótese superior e bem suportada será
considerada uma teoria. Uma teoria é aquela que resistiu ao teste do tempo e
a coleção de novos dados tende a torna-la um fato científico. Um exemplo é a
antes mencionada noção heliocêntrica do sistema solar. Antigamente era uma
mera hipótese. Embora seja ainda uma teoria formalmente bem suportada,
validada por muitas linhas independentes de evidências, atualmente é
amplamente reconhecida como um fato científico. Ninguém jamais observou
diretamente um elétron, uma fusão estelar, ondas de rádio, entropia, ou a
Terra girando em torno do Sol, no entanto estes são todos fatos científicos.
Como Stephen J. Gould dizia, um fato científico não é uma "certeza
absoluta", mas simplesmente uma teoria que tem sido "confirmada em tal grau
que seria perverso reter o consentimento provisório".


Testar envolve uma totalidade de evidências e estatísticas

A validade de uma hipótese não permanece ou cai baseada em apenas algumas
confirmações ou contradições, mas na totalidade da evidência. Freqüentemente
os dados que inicialmente poderiam ser vistos como inconsistentes com a
teoria, na realidade levarão a novas e importantes predições. A história da
física de Newton é um exemplo claro. O movimento anormal de Urano foi
inicialmente considerado uma refutação potencial da nova teoria de Newton.
Entretanto, pelo argumento da existência de um planeta não visto, a anomalia
foi explicada dentro do paradigma de Newton. Em geral, uma explicação para
um comportamento anômalo deve ser considerado "ad hoc" a menos que seja
verificado independentemente. Apresentar a posição de um novo e não visto
planeta pode ser considerado um escape se não houver nenhuma maneira
independente de detectar se um novo planeta realmente existe. Todavia,
quando a tecnologia avançou o suficiente para testar a nova predição, o
planeta não visto foi encontrado e chamado de Netuno.

A lição que se deve aprender é que explicações alternativas para "anomalias"
devem ser tratadas como qualquer outra hipótese: devem ser pesadas, testadas
e eliminadas ou confirmadas. Mas uma hipótese não pode ser considerada
refutada até que seus testes produzam muitas linhas de evidências positivas
indicando que a hipótese é realmente inconsistente com os dados empíricos.

Um ponto relacionado e crucial é que as teorias científicas modernas são
probabilísticas. Isto significa que todos os testes de predições científicas
são conduzidos em um sistema estatístico. A probabilidade e a estatística
impregnam as teorias científicas modernas, incluindo a termodinâmica
(mecânica estatística), geologia, mecânica quântica, genética e medicina. A
matemática da probabilidade é uma disciplina que muitas pessoas acham, eu
diria, desagradável. Entretanto, um bom conhecimento em estatística é
absolutamente essencial para julgar a relação entre os dados observados e as
predições de uma teoria.

Referências:

Chalmers, A. F. (1982) What is this thing called Science? Queensland,
Australia, University of Queensland Press.

Stephen J. Gould (1981) "Evolution as Fact and Theory." Discover. May issue.

Kuhn, T. (1970) The Structure of Scientific Revolutions.

Lakatos, I. (1974) "Falsification and the Methodology of Scientific Research
Progammes." Criticism and the Growth of Knowledge. I. Lakatos and A.
Musgrave. Cambridge, Cambridge University Press: 91-196.

Mayo, D. (1996) Error and the Growth of Experimental Knowledge. Chicago,
University of Chicago Press.

Popper, K. R. (1968) The Logic of Scientific Discovery. London, Hutchinson.

Salmon, W. (1990) "Rationality and Objectivity in Science, or Tom Kuhn meets
Tom Bayes." Scientific Theories. C. W. Savage. Minneapolis, University of
Minnesota Press. 14.

von Fraassen, B. C. (1980) The Scientific Image. Oxford, Clarendon Press.


[As partes desta mensagem que não continham texto foram removidas]



##### ##### #####

Para saber mais visite
http://www.ciencialist.hpg.ig.com.br


##### ##### ##### #####
Links do Yahoo! Grupos













SUBJECT: Re: [ciencialist] Prova científica?
FROM: "Luiz Ferraz Netto" <leobarretos@uol.com.br>
TO: <ciencialist@yahoogrupos.com.br>
DATE: 29/01/2005 12:24

Oi Ja1000,

Não, idéias não podem ser confundidas com senso comum --- podem ser associadas a 'bom senso', se tanto!
Constitui 'idéia' a imagem mental de uma "coisa", representação dessa coisa na mente de um individuo. E mais, se for só individual, não tem valor científico. Exemplo: Sua idéia das cores coincide com a de um colega? Sua idéia de como funciona uma válvula termiônica é o senso comum sobre funcionamento de válvulas?
Uma forma mais avançada é a 'Concepção', ainda idéia, ou conjunto de idéias mais ou menos vago. É a forma embrionária de um conceito. Exemplo: Julio Verne expôs concepções fantásticas.
Que diz o senso comum sobre o olfato e paladar? Que idéias já foram representadas na mente para essa coisa de olfato e paladar. Entre os sentidos humanos, o problema da visão está bem compreendido devido ao aperfeiçoamento da óptica. Mas não existem ainda "ciências do olfato e do paladar (gosto)" em seus significados rigorosos. Sob o ponto de vista das idéias (ou de estágios), encontram-se estagnados no mais primitivo, onde ainda não se conhece nem mesmo como descrever os fenômenos. Portanto a transformação em ciência é problema de um futuro remoto.

aquele abraço,
===========================
Luiz Ferraz Netto [Léo]
leobarretos@uol.com.br
http://www.feiradeciencias.com.br
===========================

-----Mensagem Original-----
De: "Jamil Orlandelli" <orlandel@ig.com.br>
Para: <ciencialist@yahoogrupos.com.br>
Enviada em: sábado, 29 de janeiro de 2005 08:28
Assunto: Re: [ciencialist] Prova científica?



Olá
Comentário de leigo com relação ao "senso comum não é ciência." As idéias
de ordem, causa e acaso, têm aplicação na ciência, me parece até que são
centrais à ciência, e no entanto são ideias do senso comum. São mais velhas
que a ciência . São as idéias que nos ajudam a governar a vida.

[Jamil]
----- Original Message -----
From: "Oraculo" <oraculo@atibaia.com.br>
To: <ciencialist@yahoogrupos.com.br>
Sent: Saturday, January 29, 2005 4:14 AM
Subject: [ciencialist] Prova científica?



http://www.evoluindo.biociencia.org/prova.htm

Prova científica?

Douglas Theobald, PhD, 2003.




--
Internal Virus Database is out-of-date.
Checked by AVG Anti-Virus.
Version: 7.0.300 / Virus Database: 265.6.13 - Release Date: 16/01/2005



SUBJECT: o vidro de maionese e o café
FROM: "murilo filo" <avalanchedrive@hotmail.com>
TO: ciencialist@yahoogrupos.com.br, forum-ciencia@yahoogrupos.com.br
DATE: 29/01/2005 16:29

Se alguém tiver tempo, vale traduzir.
Muito legal! Espero que gostem, se é que já não conheciam. abr/M.

''When things in your life seem almost too much to handle, when 24 hours
in a day are not enough, remember the mayonnaise jar and the coffee....

A professor stood before his philosophy class and had some items in
front of him. When the class began, wordlessly, he picked up a very
large and empty mayonnaise jar and proceeded to fill it with golf balls.
He then asked the students if the jar was full.

They agreed that it was.

The professor then picked up a box of pebbles and poured them into the
jar. He shook the jar lightly. The pebbles rolled into the open areas
between the golf balls. He then asked the students again if the jar was
full.

They agreed it was.

The professor next picked up a box of sand and poured it into the jar.
Of course, the sand filled up everything else. He asked once more if the
jar was full.

The students responded with an unanimous "yes."

The professor then produced two cups of coffee from under the table and
poured the entire contents into the jar, effectively filling the empty
space between the sand. The students laughed.

"Now," said the professor, as the laughter subsided, "I want you to
recognize that this jar represents your life. The golf balls are the
important things - God, family, your children, your health, your
friends, and your favorite passions-things that if everything else was
lost and only they remained, your life would still be full. The pebbles
are the other things that matter like your job, your house, and your
car. The sand is everything else-the small stuff.

"If you put the sand into the jar first," he continued, "there is no
room for the pebbles or the golf balls. The same goes for life. If you
spend all your time and energy on the small stuff, you will never have
room for the things that are important to you.

Pay attention to the things that are critical to your happiness. Play
with your children. Take time to get medical checkups. Take your partner
out to dinner. Play another 18. There will always be time to clean the
house and fix the disposal." Take care of the golf balls first, the
things that really matter. Set your priorities. The rest is just sand."

One of the students raised her hand and inquired what the coffee
represented.

The professor smiled. "I'm glad you asked. It just goes to show you that
no matter how full your life may seem, there's always room for a couple
of cups of coffee with a friend."



Message Archive:

http://www.escribe.com/science/nuenergy/index.html




SUBJECT: Cosmic oddity casts doubt on theory of universe
FROM: "marcelomjr" <marcelomjr@yahoo.com.br>
TO: ciencialist@yahoogrupos.com.br
DATE: 29/01/2005 17:02


http://www.theglobeandmail.com/servlet/ArticleNews/TPStory/LAC/2005012
9/BANG29/TPScience
TODAY'S PAPER

Cosmic oddity casts doubt on theory of universe


By DAN FALK
Saturday, January 29, 2005 - Page F8


A new analysis of the "echo" of the Big Bang has left cosmologists
scratching their heads and could throw a monkey wrench into efforts
to understand how the universe began.

U.S. and European scientists analyzed the distribution of "hot"
and "cold" regions -- areas that are putting out greater or less
amounts of energy than the average -- of the cosmic microwave
background radiation (the so-called echo). What they found was
unexpected: an apparent correlation between those hot and cold spots
and the orientation and motion of our solar system.

"All of this is mysterious," says Glenn Starkman, a Canadian
physicist based at Case Western Reserve University in Cleveland and
one of the authors of a recent paper in Physical Review Letters that
outlined the finding. "And the strange thing is, the more you delve
into it, the more mysteries you find."

The study, by Case Western scientists and the European Centre for
Nuclear Research in Geneva, is based on data from the WMAP satellite,
the NASA spacecraft that began mapping the cosmic microwave
background (CMB) radiation in fine detail in 2001.

The observed correlation is troubling on several fronts.

First of all, there is no reason to believe that the finding reflects
any physical connection between our local astronomical neighbourhood
and the universe at large.

As Dr. Starkman puts it: "None of us believe that the universe knows
about the solar system, or that the solar system knows about the
universe."

Far more plausible, he says, is that something within our solar
system is producing or absorbing microwaves. That means that anyone
doing cosmology would have to take into account such "local"
contamination.

(The correlation involves the largest-scale fluctuations of the CMB
radiation. If some of those fluctuations are a local rather than a
cosmological phenomenon, it would mean that the truly cosmological
large-scale fluctuations are even less intense than previously
thought.)

There is, however, another possibility: The patterns seen by Dr.
Starkman and his colleagues might simply be a fluke -- an accidental
alignment between the solar system and patterns in the CMB radiation.

If the correlation is real, however, it could cast doubt on the
popular "inflation" model of the early universe. That model, which
builds on the well-established Big Bang theory, says the universe
underwent a period of incredibly rapid, exponential growth in the
first split-second of its existence.

One of its predictions is that the universe should be nearly
perfectly "smooth," that the CMB fluctuations should be equally
intense at all scales.

An analogy with a musical instrument can be helpful: If you hit a
drum, you hear many tones at the same time -- a primary tone as well
as many overtones, or "harmonics." The inflation model predicts that
all the overtones in the CMB should be equally intense, but
instead "we're missing the bass," Dr. Starkman says. "And what bass
there is seems to be not generated by the universe, but by something
local."

Other physicists are responding with caution to the finding.

"There is no way to judge the real significance of such a result,"
says Charles Bennett of NASA's Goddard Space Flight Center in
Greenbelt, Md., the leader of the WMAP team.

It all depends on how we perceive "chance," and how we evaluate
probabilities, Dr. Bennett says. The alignments seen in the CMB may
seem unlikely, he says, but that doesn't necessarily mean that they
require new physics to explain them.

He points out that "improbable things happen frequently because there
are lots of opportunities for them to occur." In other words, he
says, the newly discovered CMB correlations are most likely the
product of chance.

Dan Falk is a science journalist based in Toronto.





SUBJECT: WHY SCIENCE IS FAILING
FROM: "marcelomjr" <marcelomjr@yahoo.com.br>
TO: ciencialist@yahoogrupos.com.br
DATE: 29/01/2005 17:05


http://www.softcom.net/users/greebo/whysci.htm

WHY SCIENCE IS FAILING
(Website Addition of March 15, 2003)

Right from the very beginning in science,
authority tends to override independent judgement.
Halton Arp

MAIN MENU - CRITICAL THINKING

This part of this website was of low priority and has been put off
until now. In the meantime, I purchased Halton Arp's book, Seeing
Red: Redshifts, Cosmology, and Academic Science. I was delighted when
I read chapter ten in Professor Arp's book because it is a far better
presentation of what I had in mind than anything I would have
written. It is twenty-one pages of excellent writing and extremely
informative content. My advice is to read that book which is
published by

Apeiron
4405, rue St-Dominique
Montreal, Quebec
H2W 2B2
Canada
http://redshift.vif.com

So this presentation will be a brief and inferior version which is
taken from my perspective and is different in many ways. I have been
an engineer for a large corporation for most of my working life, and
Professor Arp has been dealing with the Academic part of our society.

The Way Science is Supposed to Work

(...)

http://www.softcom.net/users/greebo/whysci.htm





SUBJECT: Habilidade para matemática não muda vida de "pequeno gênio"
FROM: "Kentaro Mori" <kentaro.mori@itelefonica.com.br>
TO: ciencialist@yahoogrupos.com.br
DATE: 29/01/2005 17:15


http://www1.folha.uol.com.br/fsp/cotidian/ff2901200501.htm

Habilidade para matemática não muda vida de "pequeno gênio"

ANTÔNIO GOIS
ENVIADO ESPECIAL A SÃO JOÃO D'ALIANÇA

Há cinco anos, o raro talento de Pedro Dão dos Santos para fazer
contas matemáticas complexas de cabeça o levou a programas de
televisão e o transformou numa pequena celebridade do município de São
João D'Aliança, no interior de Goiás.
Aos dez anos, ele vendia picolés num posto de gasolina da cidade e
respondia a desafios matemáticos em troca de dinheiro. Fazia de cabeça
-e acertava na imensa maioria das vezes- contas como a raiz quadrada
de 56.169 ou a raiz cúbica de 9.261. Quando a resposta não dava um
número inteiro, até as vírgulas costumava acertar.
Na semana passada, a Folha voltou a São João D'Aliança e perguntou a
Pedro o que havia mudado em sua vida. "Não mudou muita coisa, não.
Acho que eu continuo na mesma", respondeu.
De fato, pouca coisa mudou na vida de um garoto que, pelo singular
potencial para a matemática, teve seu talento subaproveitado. Com 14
anos (completa 15 no mês que vem), seu local de trabalho continua
sendo o mesmo posto de gasolina. Agora, no entanto, ele ganha um
salário mínimo trabalhando na lanchonete, distribuindo comandas para
os clientes que lá entram.

Computador
A fama instantânea trouxe alguns benefícios imediatos. Após aparecer
em 2000 numa reportagem da Folha, ele foi a principal atração de um
"Domingo Legal", programa de televisão do apresentador Gugu Liberato.
Lá, ganhou um computador, máquina para fazer estampas em camisetas,
eletrodomésticos, promessas de envio de cestas básicas e até um
carrinho de sorvetes de uma empresa para que continuasse, mesmo tendo
apenas 10 anos, a vender picolé no posto.
O computador teve pouca utilidade no início e o pai de Pedro, José
Serafim dos Santos, trocou-o por uma bezerra. As cestas básicas,
segundo a mãe, Antônia dos Santos, nunca chegaram. Os demais presentes
tiveram alguma utilidade no início, mas em pouco alteraram a rotina de
Pedro.
Hoje, seu desempenho em matemática continua muito acima da média dos
colegas de escola. A Folha repetiu as mesmas perguntas que fez em 2000
e obteve, novamente, as respostas corretas. De cabeça, ele respondeu
que a raiz quadrada de 56.169 é 237 e que a raiz cúbica de 9.261 é 21.
Esse talento, no entanto, não faz muita diferença em sua função atual.
Às vezes, outros funcionários pedem que ele calcule o total da conta
de um cliente. "É mais rápido do que somar na calculadora", diz Carla
Suelen, também funcionária da lanchonete.
Trabalhar na lanchonete do posto, mesmo autorizado pelo juizado de
menores, pode parecer absurdo para quem poderia estar aproveitando o
tempo para desenvolver ainda mais uma habilidade incomum. No entanto,
dada a condição financeira da família, essa situação é saudada pela mãe.
"Se ele não trabalhasse, tudo estaria muito pior. Graças a Deus, ele
conseguiu esse emprego", diz Antônia, mãe ainda de outros 12 filhos e
duas netas. A renda média da família é de R$ 1.000 e o maior salário
fixo é justamente o de Pedro: R$ 260.
Numa família muito pobre de agricultores no interior de Goiás, o fato
de um dos filhos ter completado o ensino fundamental e ainda alimentar
sonhos de cursar engenharia numa universidade pode ser considerado uma
vitória. No caso de Pedro, porém, há a certeza de que, devidamente
apoiado, ele poderia ir mais longe.

Talentos
Histórias como a de Pedro podem parecer excêntricas ou improváveis de
acontecer em famílias muito pobres. Mas há iniciativas no Brasil que
provam que casos como esse são mais comuns do que se imagina e podem
ter outro final.
São exemplos como o do projeto Ismart, que seleciona meninos
talentosos carentes no Rio e em São Paulo e dá a eles condições que
não teriam em escolas públicas de desenvolver sua potencialidade. Há
também experiências bem-sucedidas na rede pública.
A probabilidade de uma criança desenvolver uma habilidade acima da
média é de 3% a 5%, independentemente da classe social, afirma Zenita
Guenther, diretora-técnica do Centro para o Desenvolvimento do Talento
de Lavras e coordenadora do curso de pós-graduação em educação
especial para talentosos e bem dotados da Universidade Federal de Lavras.





SUBJECT: Olimpíadas Nacionais de Astronomia (em Portugal)
FROM: Maria Natália <grasdic@hotmail.com>
TO: ciencialist@yahoogrupos.com.br
DATE: 29/01/2005 17:58


Olimpíadas da Astronomia
Estão abertas as inscrições até dia 15 de Fevereiro
www.sp-astronomia.pt/actividades/olimpiadas/info.html
As 1ª ONA destinam-se a alunos do ensino diurno e nocturno de todo o
território nacional que estejam inscritos no ensino secundário em
2004/2005.
Conclui-se daqui que se destinam a alunos de 10º, 11º e 12º anos
As datas são: a 1ª eliminatória local será 4 de Março de 2005, a
regional a 20 de Abril de 2005. A teórica final ocorrerá a 27 de Maio
e a parte prática pelas 21h
Na realidade não são apenas estas olimpíadas que estão a decorrer e já
percebeis porque tenho andado com imenso trabalho: temos as de Física,
as de Química, o Robótica 2005 em Coimbra e o Ano Internacional de
Física. Nem só de Astronomia vive a Ciência
Um abraço
Maria Natália






SUBJECT: Prémio Crafoord 2005
FROM: Maria Natália <grasdic@hotmail.com>
TO: ciencialist@yahoogrupos.com.br
DATE: 29/01/2005 18:02


O Prémio Crafoord 2005 da Academia Sueca das Ciências e que é
atribuído nas áreas de investigação não cobertas pelo Nobel foi
atribuido a 3 cosmólogos: James Gunn, James Peebles e Martin Rees
(conhecem este?). Ver mais em:
http://physicsweb.org/article/news/9/1/16
Abraço da Maria Natália e que vai partir para os -5ºC








SUBJECT: Biologia dos monstros
FROM: "brudna" <lrb@iq.ufrgs.br>
TO: ciencialist@yahoogrupos.com.br
DATE: 29/01/2005 18:26


O texto

http://fathom.lib.uchicago.edu/2/21701757/

traz várias explicações sobre a impossibilidade de existência de
monstros de filmes de horror (B-movies).





Mais em
http://antiparticula.webcindario.com

Até
Luís Brudna





SUBJECT: Book classico: "Journeys Out of the Body"
FROM: "marcelomjr" <marcelomjr@yahoo.com.br>
TO: ciencialist@yahoogrupos.com.br
DATE: 30/01/2005 03:18


Minha sugestão pessoal aos estudiosos das Neurociencias, Psicologia,
Psiquiatria, Sono, Sonhos, Fases (REM, Não-REM, hipnagogia,
hinopompia), ondas cerebrais, sincronização hemisférica, estados
alterados etc, etc, e outras estudos afins e a quem mais possa
interessar:

O famoso book "Journeys Out of the Body" de Robert A. Monroe está
disponivel online [na lista de "links" do grupo TMIForum]. No final
do livro tem um Epilogo de Stuart W. Twemlow, M.D., que, por sinal,
não existe na tradução brasileira. Interessante para todos os que
preferem ler este classico na lingua original.

Journeys Out of the Body
Robert Monroe's first book in .pdf format
http://users.telenet.be/sterf/texts/other/journeys_out_of_the_body-
monroe.pdf

Para mais informaçoes visite os seguintes endereços:

http://groups.yahoo.com/group/GatewayJournal/links

http://groups.yahoo.com/group/TMIForum/


Um forte abraço,

Marcelo Moreira Jr.





SUBJECT: Re: Prova científica?
FROM: "rayfisica" <rayfisica@yahoo.com.br>
TO: ciencialist@yahoogrupos.com.br
DATE: 30/01/2005 10:09


--- Em ciencialist@yahoogrupos.com.br, "Oraculo" <oraculo@a...>
escreveu
> http://www.evoluindo.biociencia.org/prova.htm
>
> Prova científica?
>
--------------------------------------------------
---------------------
Ponto de vista do leigo:
Tudo muito bem tudo muito bom, mas perfect world ou mundo real?
>>>>>>>>>>>>>
Pois temos aqui a defesa de um ponto de vista com argumentos para
justificar a crença, com certeza o contraditório tem outro
tanto, cadê
o ponto de vista contrario exposto pelo defensor?
pois conforme o citado pelo autor os quatro passos deve ser dado
inclusive pelo autor da teoria, porem o que vejo inclusive nesta lista
é apenas a defesa de uma teoria e nunca a contradição pelo
"defensor",
a isso chama se método cientifico?
>>>>>>>>>>>>>>>>>.
Quantas vezes na historia o método utilizou se do trabalho dos
xamãs,
astrólogos e alquimistas e reivindicou a paternidade?





SUBJECT: RE: [ciencialist] Biologia dos monstros
FROM: "Sandra Rosario" <coffeacruda@hotmail.com>
TO: ciencialist@yahoogrupos.com.br
DATE: 30/01/2005 11:12

Olá

Com relação as plantas transgênicas, gostaria de saber se estas perdem
alguma propriedade, quando germinam, a favor da mudança genética que
sofreram?
Desde já agradeço,
Sandra

_________________________________________________________________
MSN Messenger: converse online com seus amigos .
http://messenger.msn.com.br



SUBJECT: desculpe - plantas transgênicas
FROM: "Sandra Rosario" <coffeacruda@hotmail.com>
TO: ciencialist@yahoogrupos.com.br
DATE: 30/01/2005 11:13

Olá

Com relação as plantas transgênicas, gostaria de saber se estas perdem
alguma propriedade, quando germinam, a favor da mudança genética que
sofreram?
Desde já agradeço,
Sandra

_________________________________________________________________
MSN Messenger: converse online com seus amigos .
http://messenger.msn.com.br



SUBJECT: Re: Prova científica?
FROM: Manuel Bulcão <manuelbulcao@uol.com.br>
TO: ciencialist@yahoogrupos.com.br
DATE: 30/01/2005 12:46


Oi,

> Uma "prova", então, só existe no reino da matemática e da lógica.

Manuel: Em ciência, em vez de "provar", usa-se o termo "corroborar".
Quer isso dizer que as hipóteses científicas não são verificadas
(provadas) pelos fatos: são "validadas", "apoiadas", "reforçadas",
isto é, "corroboradas".

Mas, penso eu, a corroboração é uma prova "relativa". Ao contrário
de Popper, creio que hípóteses científicas que passam sucessivas
vezes por tentativas de falsificação são "prováveis" no sentido de
que "verossímil".

> Senso comum não é ciência

Manuel: Embora a ciência se baseie em muitas noções do senso comum,
como a idéia de que existem padrões e regularidades na natureza; de
que há uma realidade objetiva independente da consciência; de que o
efeito sucede a causa, etc. Por outro lado, o senso comum, ao longo
do tempo e mesmo que tardiamente, incorpora o conhecimento
científico sob a forma de "bom senso" como bem disse o Léo. Hoje, o
senso comum do homem "civilizado" diz que a Terra é redonda e que
gira em torno do Sol; que o Sol é uma estrela entre outras; que os
corpos materiais são constituídos de átomos, etc. Tais idéias estão
tão estabelecidas que muito pouca gente as contesta: viraram senso
comum.

> O exame do método científico revela que a ciência envolve muito
mais que empirismo crédulo.

Manuel: Sim, a ciência envolve muito mais que empirismo crédulo, mas
envolve muito mais empirismo e indução do que Popper supunha. Para
esse filósofo, que seguia a linha hipotético-dedutivista, por mais
que um enunciado científico seja corroborado pela observação, seu
grau de probabilidade permanece inauterado, dada a quantidade
infinita de observações da mesma natureza que se pode fazer no
futuro. Reduzindo ao absurdo: se, desde que a Terra existe, o Sol
tem surgido no horizonte até hoje, não há "razão", isto é, não é
racionalmente justificado afirmar com base nisso que ele surgirá
amanhã ou que, muito provavelmente, tal fenômeno ocorrerá. Não
há "razão"? Penso que há uma razão "vital" para se crer nisso e que,
para raciocinar (não importa se por indução, por abdução ou
hipotético-dedutivamente)e fazer ciência, é preciso estar vivo.

>Pesquisas que envolvem somente observações, repetições e medidas
não são suficientes para contar como ciência.

Manuel: Não obstante, geram "conhecimento" empírico que já não é
senso comum. Ademais, as ciências novas, incipientes, ainda não
axiomatizadas, consistem basicamente em observações, repetições e
medidas, como o são (ou por muito tempo foram) a etologia, a
arqueologia, a filologia, etc. Por que já não qualificá-las como
ciência?

Acho que a conceitualização da ciência é tão difícil como a
conceitualização da vida. Afinal, os vírus e retrovírus são seres
vivos? E os príons (moléculas proteicas auto-replicáveis)? Há quem
sustente que as idéias -- memes -- são um tipo novo de vida
cujo "caldo nutritivo" é o cérebro humano. Um conhecido meu, o
Adamastor, é estéril. Está ele vivo ou não passa de um zumbi?

Quando algo deixa de ser inanimado e passa a ser vivo? Do mesmo
modo, quando um conhecimento empírico válido que já não é senso
comum pode ser classificado como ciência? Apenas quando compreende
hipóteses que fazem previsões capazes de serem falseadas? POR QUÊ?

Em vez de ficar estabelecendo critérios rígidos, até mesmo
artificiosos, para demarcar a ciência, muito mais razoável
seria "fluidificar" o conceito de modo que não fiquem de fora
disciplinas tais como a filologia, a arqueologia e até mesmo a
matemática e a lógica. Isto é, estabelecer vários tipos de ciência
(conhecimentos fundados na pesquisa empírica e/ou no raciocínio
lógico-matemático).

> Uma hipótese que pode explicar todas as observações possíveis e os
dados não são testáveis, não é científica.

Manuel: Então, penso eu (posso está enganado), a 2a. Lei da
Termodinâmica está em maus lençóis, pelo menos na formulação que
diz: "todo sistema isolado evolui no sentido da desordem ou da
máxima entropia."

Primeiro porque um sistema isolado "perfeito" (que não troca com o
ambiente nenhuma matéria e nada de energia) que não seja o universo
como um todo não é factível. No máximo, o que se pode observar são
sistemas fechados tão fechadinhos "a ponto de" (prestes ou próximo
a) serem isolados, o que não é o caso da garrafa térmica vagabunda
que comprei antes de ontem.

Segundo, o universo como um todo (o "dado" sistema isolado) não pode
ser testado.

Terceiro, se um subsistema aberto ou fechado deste sistema isolado
apresenta entropia crescente, isso corrobora o Princípio Universal
da Degradação da Energia"; se um outro, pelo contrário, torna-se ao
longo do tempo cada vez mais ordenado, isso se dá à expensas de um
aumento universal da entropia ou desordem, o que também confirma a
teoria. Ou seja, cara eu ganho, coroa você perde.

A generalização indutiva compõe aquela parte do senso comum
denominado "bom senso", apesar de não ter sido gerado pela ciência.
Ora, a ciência baseia-se em muitas "boas" noções do senso comum,
como as que eu citei acima. Por que não incorporar também
esta, "mutatis mudandis"?

Talvez eu tenha dito algumas grandes besteiras, mas por favor,
Sérgio Taborda, se for apontá-las, não me humilhe, plis! :-)

Abraços,
Manuel Bulcão





SUBJECT: Re: Prova científica? (correção)
FROM: Manuel Bulcão <manuelbulcao@uol.com.br>
TO: ciencialist@yahoogrupos.com.br
DATE: 30/01/2005 12:48


Oi,

> Uma "prova", então, só existe no reino da matemática e da lógica.

Manuel: Em ciência, em vez de "provar", usa-se o termo "corroborar".
Quer isso dizer que as hipóteses científicas não são verificadas
(provadas) pelos fatos: são "validadas", "apoiadas", "reforçadas",
isto é, "corroboradas".

Mas, penso eu, a corroboração é uma prova "relativa". Ao contrário
de Popper, creio que hípóteses científicas que passam sucessivas
vezes por tentativas de falsificação são "prováveis" no sentido de
que "verossímil".

> Senso comum não é ciência

Manuel: Embora a ciência se baseie em muitas noções do senso comum,
como a idéia de que existem padrões e regularidades na natureza; de
que há uma realidade objetiva independente da consciência; de que o
efeito sucede a causa, etc. Por outro lado, o senso comum, ao longo
do tempo e mesmo que tardiamente, incorpora o conhecimento
científico sob a forma de "bom senso" como bem disse o Léo. Hoje, o
senso comum do homem "civilizado" diz que a Terra é redonda e que
gira em torno do Sol; que o Sol é uma estrela entre outras; que os
corpos materiais são constituídos de átomos, etc. Tais idéias estão
tão estabelecidas que muito pouca gente as contesta: viraram senso
comum.

> O exame do método científico revela que a ciência envolve muito
mais que empirismo crédulo.

Manuel: Sim, a ciência envolve muito mais que empirismo crédulo, mas
envolve muito mais empirismo e indução do que Popper supunha. Para
esse filósofo, que seguia a linha hipotético-dedutivista, por mais
que um enunciado científico seja corroborado pela observação, seu
grau de probabilidade permanece inauterado, dada a quantidade
infinita de observações da mesma natureza que se pode fazer no
futuro. Reduzindo ao absurdo: se, desde que a Terra existe, o Sol
tem surgido no horizonte até hoje, não há "razão", isto é, não é
racionalmente justificado afirmar com base nisso que ele surgirá
amanhã ou que, muito provavelmente, tal fenômeno ocorrerá. Não
há "razão"? Penso que há uma razão "vital" para se crer nisso e que,
para raciocinar (não importa se por indução, por abdução ou
hipotético-dedutivamente)e fazer ciência, é preciso estar vivo.

A generalização indutiva compõe aquela parte do senso comum
denominado "bom senso", apesar de não ter sido gerado pela ciência.
Ora, a ciência baseia-se em muitas "boas" noções do senso comum,
como as que eu citei acima. Por que não incorporar também
esta, "mutatis mudandis"?

>Pesquisas que envolvem somente observações, repetições e medidas
não são suficientes para contar como ciência.

Manuel: Não obstante, geram "conhecimento" empírico que já não é
senso comum. Ademais, as ciências novas, incipientes, ainda não
axiomatizadas, consistem basicamente em observações, repetições e
medidas, como o são (ou por muito tempo foram) a etologia, a
arqueologia, a filologia, etc. Por que já não qualificá-las como
ciência?

Acho que a conceitualização da ciência é tão difícil como a
conceitualização da vida. Afinal, os vírus e retrovírus são seres
vivos? E os príons (moléculas proteicas auto-replicáveis)? Há quem
sustente que as idéias -- memes -- são um tipo novo de vida
cujo "caldo nutritivo" é o cérebro humano. Um conhecido meu, o
Adamastor, é estéril. Está ele vivo ou não passa de um zumbi?

Quando algo deixa de ser inanimado e passa a ser vivo? Do mesmo
modo, quando um conhecimento empírico válido que já não é senso
comum pode ser classificado como ciência? Apenas quando compreende
hipóteses que fazem previsões capazes de serem falseadas? POR QUÊ?

Em vez de ficar estabelecendo critérios rígidos, até mesmo
artificiosos, para demarcar a ciência, muito mais razoável
seria "fluidificar" o conceito de modo que não fiquem de fora
disciplinas tais como a filologia, a arqueologia e até mesmo a
matemática e a lógica. Isto é, estabelecer vários tipos de ciência
(conhecimentos fundados na pesquisa empírica e/ou no raciocínio
lógico-matemático).

> Uma hipótese que pode explicar todas as observações possíveis e os
dados não são testáveis, não é científica.

Manuel: Então, penso eu (posso está enganado), a 2a. Lei da
Termodinâmica está em maus lençóis, pelo menos na formulação que
diz: "todo sistema isolado evolui no sentido da desordem ou da
máxima entropia."

Primeiro porque um sistema isolado "perfeito" (que não troca com o
ambiente nenhuma matéria e nada de energia) que não seja o universo
como um todo não é factível. No máximo, o que se pode observar são
sistemas fechados tão fechadinhos "a ponto de" (prestes ou próximo
a) serem isolados, o que não é o caso da garrafa térmica vagabunda
que comprei antes de ontem.

Segundo, o universo como um todo (o "dado" sistema isolado) não pode
ser testado.

Terceiro, se um subsistema aberto ou fechado deste sistema isolado
apresenta entropia crescente, isso corrobora o Princípio Universal
da Degradação da Energia"; se um outro, pelo contrário, torna-se ao
longo do tempo cada vez mais ordenado, isso se dá à expensas de um
aumento universal da entropia ou desordem, o que também confirma a
teoria. Ou seja, cara eu ganho, coroa você perde.

Talvez eu tenha dito algumas grandes besteiras, mas por favor,
Sérgio Taborda, se for apontá-las, não me humilhe, plis! :-)

Abraços,
Manuel Bulcão





SUBJECT: Re: [ciencialist] Re: Prova científica?
FROM: "Oraculo" <oraculo@atibaia.com.br>
TO: <ciencialist@yahoogrupos.com.br>
DATE: 30/01/2005 23:13

Olá ray

Bem, quando a primeira parte da sua pergunta, não sei como responder, já que não entendi nada..:-)

Mas quanto a pergunta final, sobre quantas vezes o método usou xamãs e conhecimento misitico para produzir conhecimento confiável, é fácil de responder: nunca..:-) Nenhuma vez..:-)

Na verdade, sempre filtrou e eliminou o esterico e o sobrenatural para determinar o eficaz e confiável, com, por exemplo, na astrologia e a alquimia. Foi exatamente por deixar de lado os esoterismos e misticismos dessas afirmações, que pode produzir a astronomia e a química..:-)

Um abraço.

Homero

----- Original Message -----
From: rayfisica
To: ciencialist@yahoogrupos.com.br
Sent: Sunday, January 30, 2005 10:09 AM
Subject: [ciencialist] Re: Prova científica?



--- Em ciencialist@yahoogrupos.com.br, "Oraculo" <oraculo@a...>
escreveu
> http://www.evoluindo.biociencia.org/prova.htm
>
> Prova científica?
>
--------------------------------------------------
---------------------
Ponto de vista do leigo:
Tudo muito bem tudo muito bom, mas perfect world ou mundo real?
>>>>>>>>>>>>>
Pois temos aqui a defesa de um ponto de vista com argumentos para
justificar a crença, com certeza o contraditório tem outro
tanto, cadê
o ponto de vista contrario exposto pelo defensor?
pois conforme o citado pelo autor os quatro passos deve ser dado
inclusive pelo autor da teoria, porem o que vejo inclusive nesta lista
é apenas a defesa de uma teoria e nunca a contradição pelo
"defensor",
a isso chama se método cientifico?
>>>>>>>>>>>>>>>>>.
Quantas vezes na historia o método utilizou se do trabalho dos
xamãs,
astrólogos e alquimistas e reivindicou a paternidade?





##### ##### #####

Para saber mais visite
http://www.ciencialist.hpg.ig.com.br


##### ##### ##### #####


Yahoo! Grupos, um serviço oferecido por:

São Paulo Rio de Janeiro Curitiba Porto Alegre Belo Horizonte Brasília




------------------------------------------------------------------------------
Links do Yahoo! Grupos

a.. Para visitar o site do seu grupo na web, acesse:
http://br.groups.yahoo.com/group/ciencialist/

b.. Para sair deste grupo, envie um e-mail para:
ciencialist-unsubscribe@yahoogrupos.com.br

c.. O uso que você faz do Yahoo! Grupos está sujeito aos Termos do Serviço do Yahoo!.



[As partes desta mensagem que não continham texto foram removidas]



SUBJECT: Chimps' Sense of Justice Found Similar to Humans'
FROM: "Oraculo" <oraculo@atibaia.com.br>
TO: <ciencialist@yahoogrupos.com.br>
DATE: 30/01/2005 23:56

http://www.scientificamerican.com/article.cfm?chanID=sa003&articleID=000C1A68-BA52-11F6-BA5283414B7F0000

Chimps' Sense of Justice Found Similar to Humans'

Inequities big and small can lead people to believe that life is indeed not fair. But how humans respond to unfair situations depends on the social circumstances: inequality among friends and family, for instance, is less disturbing than it is among strangers. The results of a new study indicate that the same is true for chimpanzees, a finding that sheds light on how our sense of fairness evolved.
In the fall of 2003 Sarah Brosnan and Frans de Waal of the Yerkes National Primate Research Center in Atlanta determined that capuchin monkeys don't like being subjected to treatment they deem unjust. In the new work, the researchers tested the reactions of pairs of chimpanzees to exchanges of food that varied in quality. The animals received either a grape, which they coveted, or a less appealing cucumber, and they could see what their partner obtained. In pairs of chimps that had lived together since birth, the individual given the cucumber was less likely to react negatively to the situation than was the short-changed member of a pair that did not know each other as well. Indeed, chimps in the short-term social groups refused to work after their partner received a better reward for the same job. "Human decisions tend to be emotional and vary depending on the other people involved," Brosnan says. "Our findings in chimpanzees implies this variability in response is adaptive and emphasizes there is not one best response for any given situation, but rather it depends on the social environment at the time."


Further experiments to investigate reactions to unfair situations are ongoing at the center in the hopes of understanding why we humans make the decisions we do. "Identifying a sense of fairness in two closely related nonhuman primate species implies it could have a long evolutionary history," Brosnan remarks. The findings will be published in the February 7 edition of the Proceedings of The Royal Society B: Biological Sciences. --Sarah Graham




[As partes desta mensagem que não continham texto foram removidas]



SUBJECT: Imaginary traumas are as terrifying as the real thing
FROM: "Oraculo" <oraculo@atibaia.com.br>
TO: <ciencialist@yahoogrupos.com.br>
DATE: 31/01/2005 00:03

http://www.scientificamerican.com/article.cfm?chanID=sa006&colID=13&articleID=00008C3F-3EFD-11E7-BB5883414B7F0000

Abducted!
Imaginary traumas are as terrifying as the real thing
By Michael Shermer


In the wee hours of the morning on August 8, 1983, while I was traveling along a lonely rural highway approaching Haigler, Neb., a large craft with bright lights overtook me and forced me to the side of the road. Alien beings exited the craft and abducted me for 90 minutes, after which time I found myself back on the road with no memory of what transpired inside the ship. I can prove that this happened because I recounted it to a film crew shortly afterward.
When alien abductees recount to me their stories, I do not deny that they had a real experience. But thanks to recent research by Harvard University psychologists Richard J. McNally and Susan A. Clancy, we now know that some fantasies are indistinguishable from reality, and they can be just as traumatic. In a 2004 paper in Psychological Science entitled "Psychophysiological Responding during Script-Driven Imagery in People Reporting Abduction by Space Aliens," McNally, Clancy and their colleagues report the results of a study of claimed abductees. The researchers measured heart rate, skin conductance and electromyographic responses in a muscle that lifted the eyebrow--called the left lateral (outer) frontalis--of the study participants as they relived their experiences through script-driven imagery. "Relative to control participants," the authors concluded, "abductees exhibited greater psychophysiological reactivity to abduction and stressful scripts than to positive and neutral scripts." In fact, the abductees' responses were comparable to those of post-traumatic stress disorder (PTSD) patients who had listened to scripts of their actual traumatic experiences.

The abduction study was initiated as a control in a larger investigation of memories of sexual abuse. In his book Remembering Trauma (Harvard University Press, 2003), McNally tracks the history of the recovered memory movement of the 1990s, in which some people, while attempting to recover lost memories of childhood sexual molestation (usually through hypnosis and guided imagery), instead created false memories of abuse that never happened. "The fact that people who believe they have been abducted by space aliens respond like PTSD patients to audiotaped scripts describing their alleged abductions," McNally explains, "underscores the power of belief to drive a physiology consistent with actual traumatic experience." The vividness of a traumatic memory cannot be taken as evidence of its authenticity.


The most likely explanation for alien abductions is sleep paralysis and hypnopompic (on awakening) hallucinations. Temporary paralysis is often accompanied by visual and auditory hallucinations and sexual fantasies, all of which are interpreted within the context of pop culture's fascination with UFOs and aliens. McNally found that abductees "were much more prone to exhibit false recall and false recognition in the lab than were control subjects," and they scored significantly higher than normal on a questionnaire measuring "absorption," a trait related to fantasy proneness that also predicts false recall.
My abduction experience was triggered by sleep deprivation and physical exhaustion. I had just ridden a bicycle 83 straight hours and 1,259 miles in the opening days of the 3,100-mile nonstop transcontinental Race Across America. I was sleepily weaving down the road when my support motor home flashed its high beams and pulled alongside, and my crew entreated me to take a sleep break. At that moment a distant memory of the 1960s television series The Invaders was inculcated into my waking dream. In the series, alien beings were taking over the earth by replicating actual people but, inexplicably, retained a stiff little finger. Suddenly the members of my support team were transmogrified into aliens. I stared intensely at their fingers and grilled them on both technical and personal matters.

After my 90-minute sleep break, the experience represented nothing more than a bizarre hallucination, which I recounted to ABC's Wide World of Sports television crew filming the race. But at the time the experience was real, and that's the point. The human capacity for self-delusion is boundless, and the effects of belief are overpowering. Thanks to science we have learned to tell the difference between fantasy and reality.


--------------------------------------------------------------------------------

Michael Shermer is publisher of Skeptic (www.skeptic.com) and author of The Science of Good and Evil.


[As partes desta mensagem que não continham texto foram removidas]



SUBJECT: Re: desculpe - plantas transgênicas
FROM: "rmtakata" <rmtakata@altavista.net>
TO: ciencialist@yahoogrupos.com.br
DATE: 31/01/2005 08:20


--- Em ciencialist@yahoogrupos.com.br, "Sandra Rosario"
> Com relação as plantas transgênicas, gostaria de saber se estas
> perdem alguma propriedade, quando germinam, a favor da mudança
> genética que sofreram?

Depende. Depende de qual gene foi inserido e onde ele se enfiou (epa!)
no meio do genoma da planta. Cada caso eh um caso.

Se o transgene se meteu bem no meio de um gene funcional da planta,
provavelmente ele fara' com q. esse gene nativo perca a funcao: se a
funcao for vital, dificilmente a planta ira' vingar. Se esse gene for
um q. esta' envolvido, por exemplo, na sintese de certas vitaminas, a
planta podera' perder qualidade nutricional.

Poderia haver tb uma reacao cruzada entre o produto do transgene e
algum gene nativo. Por exemplo, uma proteina transgenica atuar sobre
um composto quimico, cuja sintese eh controlada por genes nativos, e
produzir um outro composto, de propriedades toxicas ou antidigestivas.
Aqui nao seria bem perder uma qualidade, mas ganhar uma nao muito
desejavel.

O transgene poderia se enfiar em uma regiao q. nao codifica nenhuma
proteina, nem controla o funcionamento de outros genes. E nesse caso,
eh bem possivel q. nao provoque nenhuma alteracao no funcionamento
normal da planta (ou outro organismo) - claro, desde q. seu produto
nao reaja com outros componentes do organismo.

Por isso, para cada planta (ou outro organismo) geneticamente
modificada - seja por transgenia ou outro metodo - eh preciso fazer um
estudo e verificar se houve alguma outra alteracao alem da que se
pretendia introduzir e q. efeitos essas mudancas tEm - na qualidade
(em especial, em termos de valor economico, mas tb social) do
organismo modificado, na saude humana, no meio ambiente.

[]s,

Roberto Takata





SUBJECT: Re: Prova científica? (correção)
FROM: "Esteban Lopez Moreno" <estebanmoreno@idhi.org.br>
TO: ciencialist@yahoogrupos.com.br
DATE: 31/01/2005 09:43



Sugiro a leitura do artigo contido em:
http://www.entropylaw.com/thermoevolution1.html

abraço,
Esteban.



>
> Talvez eu tenha dito algumas grandes besteiras, mas por favor,
> Sérgio Taborda, se for apontá-las, não me humilhe, plis! :-)
>
> Abraços,
> Manuel Bulcão





SUBJECT: Re: Prova científica?
FROM: "rayfisica" <rayfisica@yahoo.com.br>
TO: ciencialist@yahoogrupos.com.br
DATE: 31/01/2005 09:45


--- Em ciencialist@yahoogrupos.com.br, "Oraculo" <oraculo@a...> escreveu
> Olá ray
>
> Bem, quando a primeira parte da sua pergunta, não sei como
responder, já que não entendi nada..:-)
>
> Mas quanto a pergunta final, sobre quantas vezes o método usou xamãs
e conhecimento misitico para produzir conhecimento confiável, é fácil
de responder: nunca..:-) Nenhuma vez..:-)
>
> Na verdade, sempre filtrou e eliminou o esterico e o sobrenatural
para determinar o eficaz e confiável, com, por exemplo, na astrologia
e a alquimia. Foi exatamente por deixar de lado os esoterismos e
misticismos dessas afirmações, que pode produzir a astronomia e a
química..:-)
>
> Um abraço.
>
> Homero
> >>>>>>>>>>>>>>>>>>>>>

Tomei conhecimento de que para facilitar o trabalho os cientistas de
nossos dias tem se embreado na Amazônia em busca do que puder ser
transformado naquela ciência bem detalhada por senhor Taborda, e para
adiantar a procura eles subornam (enganam) os xamãs para subtrair o
conhecimento deles no sentido de ganhar tempo, não é disso que estamos
falando?
Será que isso nunca aconteceu antes, Egípcios, Incas etc.?







SUBJECT: Re: [ciencialist] Re: Prova científica?
FROM: "Esteban Moreno" <estebanmoreno@idhi.org.br>
TO: <ciencialist@yahoogrupos.com.br>
DATE: 31/01/2005 10:55

E se não formos embotados pelo preconceito ;- ) muitas ervas utilizadas pelo xamãs confirmaram em seus princípios ativos o poder de cura, a alquimia foi e tem sido profundamente estudada pela sua herança psicológica em um tempo que prevalecia o dogmatismo, além de algumas de suas variantes já serem amplamente reconhecidas em conselhos internacionais de medicina e orgãos oficiais de governos, e a astrologia vai muito bem obrigado.
Abraços de retorno aos bufões da ciência,
Esteban.


Alguém escreveu:
Mas quanto a pergunta final, sobre quantas vezes o método usou xamãs e conhecimento misitico para produzir conhecimento confiável, é fácil de responder: nunca..:-) Nenhuma vez..:-)

Na verdade, sempre filtrou e eliminou o esterico e o sobrenatural para determinar o eficaz e confiável, com, por exemplo, na astrologia e a alquimia. Foi exatamente por deixar de lado os esoterismos e misticismos dessas afirmações, que pode produzir a astronomia e a química..:-)

@yahoogrupos.com.br

a.. O uso que você faz do Yahoo! Grupos está sujeito aos Termos do Serviço do Yahoo!.



[As partes desta mensagem que não continham texto foram removidas]



SUBJECT: Re: [ciencialist] Re: Prova científica?
FROM: "Oraculo" <oraculo@atibaia.com.br>
TO: <ciencialist@yahoogrupos.com.br>
DATE: 31/01/2005 14:40

Olá ray

Bem, isto é um tanto diferente da acusação inicial..:-) Usar plantas como base de desenvolvimento de novas drogas é algo bem conhecido. O fato de consultar as pessoas que vivem em uma determinada região, como fonte de informação sobre essas plantas (que podem ser xamã, mas também podem ser apenas pessoas comuns é perfeitamente compreensível.

E, sim, claro que existem pessoas que roubam plantas e conhecimentos sobre elas de povos nativos. E que traficam fósssies, aves raras, animais selvagens, peixes raros, etc. São ladrões, como quaisquer outros.

Mas, o fato de que existe gente assim nada tem a ver com a eficácia e confiabilidade de um conhecimento, científico ou não.

Veja, um pesquisador que estuda o uso de uma planta amazonica, pode realmente se beneficiar com o conhecimento de que xamãs do local a usam para controlar febre. Mas, ao analisar a planta, ele vai separar o principio ativo, descobrir seus efeitos no organismo, sintetizar a molécula responsável, analisar efeitos colaterais, controlar esses efeitos, definir a dosagem correta (com mais eficácia e menor dano), etc. E certamente vai eliminar o que for desnecessário, como a dança do xamã em volta do paciente, o canto mágico, a aplicação de outras plnatas inócuas, etc. Pode até mesmo descobrir que a planta em questão não tem nenhum efeito contra a febre.

Se ele faz isso com a intenção de criar novas drogas que ajudem as pessoas, inclusive as que são tratadas pelo xamã, e com o respeito aos direitos do pais em questão sobre os resultados, ou se ele rouba e leva para sua empresa farmacologica, isso nada tem a ver com ciência e métodos. Aliás, um xamã que roubasse as plantas de um outro xamã de outra tribo, também estaria sendo desonesto (e isso acontece as vezes) sem que nada disso interfira na eficácia ou confiabilidade dos métodos de cura envolvidos.

Que foi a pergunta inicial: quantas vezes a ciência teria se apropriado da eficácia de um método não cientifico. Para essa pergunta, a resposta continua a mesma, nunca..;-)

Nào devemos deixar que ressentimento e politica nublem a visão dos conceitos e termos envolvidos. Para o bem ou para o mal, usar a ciência e método cientifico é mais eficaz. Seja para salvar vidas, seja para destruir vidas, seja para construir, seja para destruir. Se pretende criar novas drogas para o bem da humanidade ou se pretende roubar conhecimento e ficar rico, fará isso de forma mais eficaz e confiável se usar a ciência..:-)

Um abraço.

Homero
----- Original Message -----
From: rayfisica
To: ciencialist@yahoogrupos.com.br
Sent: Monday, January 31, 2005 9:45 AM
Subject: [ciencialist] Re: Prova científica?



--- Em ciencialist@yahoogrupos.com.br, "Oraculo" <oraculo@a...> escreveu
> Olá ray
>
> Bem, quando a primeira parte da sua pergunta, não sei como
responder, já que não entendi nada..:-)
>
> Mas quanto a pergunta final, sobre quantas vezes o método usou xamãs
e conhecimento misitico para produzir conhecimento confiável, é fácil
de responder: nunca..:-) Nenhuma vez..:-)
>
> Na verdade, sempre filtrou e eliminou o esterico e o sobrenatural
para determinar o eficaz e confiável, com, por exemplo, na astrologia
e a alquimia. Foi exatamente por deixar de lado os esoterismos e
misticismos dessas afirmações, que pode produzir a astronomia e a
química..:-)
>
> Um abraço.
>
> Homero
> >>>>>>>>>>>>>>>>>>>>>

Tomei conhecimento de que para facilitar o trabalho os cientistas de
nossos dias tem se embreado na Amazônia em busca do que puder ser
transformado naquela ciência bem detalhada por senhor Taborda, e para
adiantar a procura eles subornam (enganam) os xamãs para subtrair o
conhecimento deles no sentido de ganhar tempo, não é disso que estamos
falando?
Será que isso nunca aconteceu antes, Egípcios, Incas etc.?







##### ##### #####

Para saber mais visite
http://www.ciencialist.hpg.ig.com.br


##### ##### ##### #####


Yahoo! Grupos, um serviço oferecido por:
PUBLICIDADE




------------------------------------------------------------------------------
Links do Yahoo! Grupos

a.. Para visitar o site do seu grupo na web, acesse:
http://br.groups.yahoo.com/group/ciencialist/

b.. Para sair deste grupo, envie um e-mail para:
ciencialist-unsubscribe@yahoogrupos.com.br

c.. O uso que você faz do Yahoo! Grupos está sujeito aos Termos do Serviço do Yahoo!.



[As partes desta mensagem que não continham texto foram removidas]



SUBJECT: Prova científica? p/ Esteban
FROM: "Oraculo" <oraculo@atibaia.com.br>
TO: <ciencialist@yahoogrupos.com.br>
DATE: 31/01/2005 15:07

Olá Esteban

risos..:-) Bem, abraços aos bufões das pseudo-ciencias..:-)

Se não formos embotados pelo preconceito e pela atração as próprias crenças, temos de reconhecer que não é possível que todas as afirmações sem evidencias sejam reais ao mesmo tempo (já que muitas delas são contraditórias). E reconhecer que temos de descobrir um meio de separar reais de irreais. Esse método, chamanos de cientifico..:-)

A astrologia vai bem, com certeza, dando muito dinheiro a quem interessa, mas continua a mesma a 3 mil anos, sem acrescentar nada ao conhecimento humano, sem se modificar ou ajustar (ofiuco, o 13º signo, devido a precessão dos equinócios, é solenemente ignorado pelos astrólogos..:-), e continua sem nenhuma evidencia de ser real ou de poder fazer o que alega poder fazer..:-)

Sim, também a homeopatia foi aceita (em termos) por algumas organizações médicas. Não por ter demonstrado eficácia, ou estudo de comprovação, pelo contrário, mas simplesmente por não ter demonstrado nenhum efeito, o que a torna inócua e sem perigo.

Outra questão interessante, a acupuntura, tem tido algum sucesso em estudos que afirmam que pode produzir efeito na produção de prostaglandinas e diminuir a sensações dolorosas.

Mas, agulhas que ao serem inseridas disparam reação do organismo que interfere na sintese de prostaglandinas, NÃO É MAIS acupuntura..:-) A definição, o conceito de acupuntura é especifico: o reequilibrio de energias não detectáveis, que se movem por meridianos bem definidos no corpo humano, pela inserção de agulhas em pontos específicos desses meridianos e que permitem curar todos os tipos de doenças. Isso é acupuntura, não agulhas e prostaglandinas..:-)

Se não muda os fluxos de energias nào detectáveis, se não é colocada em pontos específicos de meridianos energéticos, se não cura todas as doenças, nào é acupuntura. É, no máximo, um novo efeito na sintese de prostaglandinas descoberto, que envolve a inserção de agulhas em qualquer parte do corpo (ou o uso de beliscões para isso..:-)

Quanto as ervas, sim, muitas mostraram seu poder de cura e seu princípio ativo. Mas a maioria delas precisou ser separada de elementos e principios ativos daninhos, ser controlada com outras drogas, ter sua dosagem mínima determinada (alguns pacietnes dos xamãs morrem de overdose antes da cura), etc. E muitas também não mostraram princípio algum e foram abandonadas. Pegar simplesmente toda farmacopeia de uma tribo e usa-la é perigoso e, muitas vezes, inócuo.

Para ter uma idéia do que estou falando, procure e analise a expectativa de vida de tribos com e sem acesso a medicamentos ciêntíficos. E a taxa de mortalidade de crianças de tribos com e sem acesso a cuidados e medicamentos alopáticos. Vai ter uma enorme (e desagradável, pelos seus padrões de crença) surpresa quanto a esses dados..:-)

Se xamãs e curandeiros fossem mais eficazes que drogas e conhecimento cientifico, isso deveria ser diferente, nào? Não estou menosprezando esse conhecimento tribal, por favor. Com o que tinham disponível, com o conhecimento de que eram capazes, eles fizeram o máximo e devem ser admirados por isso. Mas esquecer que podemos viver mais de 80 anos, enquanto que nas condições de vida de tribos isoladas, essa espectativa é de menos de 40 anos, é um tanto temerário.

A alquimia nos deixou a quimica. E foi, como disse antes, importante em sua época. Mas tenho certeza que os alquimistas do passado adorariam ter o conhecimento de quimica de hoje, e tenho certeza que analisariam seus dados novamente, abandonariam os incorretos, e abraçariam os corretos, sem dor ou pena, já que eram cientistas tentando encontrar o real e não crentes de uma única fé..:-) Qualquer alquimista ou estudioso do assunto (como Newton) adoraria entrar em um laboratorio moderno, poder contar com os equipamentosw disponíveis, com o conhecimento sobre eletrons, valencia, elementos, átomos, etc. Mesmo que para isso tivessem de deixar para traz suas crenças e conceitos incorretos.

Respeitar os que vieram antes, seus esforços e sua inteligencia é importante, mas manter a mente no passado é tolice. E esse respeito pelos homens e seus intelectos não pode impedir o reconhecimento de seus erros e mesmo de suas tolices.

Afinal, se você pode manter este debate, e participar desta lista de discussões sobre ciência, em um ambiente virtual, e usar seu computador para tanto, é graças a esses que chama de "bufões da ciência" e não aos xamãs e alquimistas..:-) Tente mandar uma mensagem à lista usando um conhecimento xamanico ou construir e usar um chip de silicio usando alquimia..;-) Vai dar bastante trabalho..:-)

Um abraço.

Homero




----- Original Message -----
From: Esteban Moreno
To: ciencialist@yahoogrupos.com.br
Sent: Monday, January 31, 2005 10:55 AM
Subject: Re: [ciencialist] Re: Prova científica?


E se não formos embotados pelo preconceito ;- ) muitas ervas utilizadas pelo xamãs confirmaram em seus princípios ativos o poder de cura, a alquimia foi e tem sido profundamente estudada pela sua herança psicológica em um tempo que prevalecia o dogmatismo, além de algumas de suas variantes já serem amplamente reconhecidas em conselhos internacionais de medicina e orgãos oficiais de governos, e a astrologia vai muito bem obrigado.
Abraços de retorno aos bufões da ciência,
Esteban.


Alguém escreveu:
Mas quanto a pergunta final, sobre quantas vezes o método usou xamãs e conhecimento misitico para produzir conhecimento confiável, é fácil de responder: nunca..:-) Nenhuma vez..:-)

Na verdade, sempre filtrou e eliminou o esterico e o sobrenatural para determinar o eficaz e confiável, com, por exemplo, na astrologia e a alquimia. Foi exatamente por deixar de lado os esoterismos e misticismos dessas afirmações, que pode produzir a astronomia e a química..:-)

@yahoogrupos.com.br

a.. O uso que você faz do Yahoo! Grupos está sujeito aos Termos do Serviço do Yahoo!.



[As partes desta mensagem que não continham texto foram removidas]



##### ##### #####

Para saber mais visite
http://www.ciencialist.hpg.ig.com.br


##### ##### ##### #####


Yahoo! Grupos, um serviço oferecido por:







------------------------------------------------------------------------------
Links do Yahoo! Grupos

a.. Para visitar o site do seu grupo na web, acesse:
http://br.groups.yahoo.com/group/ciencialist/

b.. Para sair deste grupo, envie um e-mail para:
ciencialist-unsubscribe@yahoogrupos.com.br

c.. O uso que você faz do Yahoo! Grupos está sujeito aos Termos do Serviço do Yahoo!.



[As partes desta mensagem que não continham texto foram removidas]



SUBJECT: criacionismo x evolução
FROM: "E m i l i a n o C h e m e l l o" <chemelloe@yahoo.com.br>
TO: <Conversa_de_Botequim@yahoogrupos.com.br>, <ciencialist@yahoogrupos.com.br>
DATE: 31/01/2005 16:04

JC e-mail 2698, de 31 de Janeiro de 2005.
Assalto ao coração da biologia, artigo de José Mariano Amabis

O criacionismo e o design inteligente não têm o status de ciência

José Mariano Amabis é professor do Departamento de Biologia do Instituto de
Biociências USP e autor de livros didáticos. Artigo publicado na 'Folha de
SP':

A edição de 25 de janeiro da revista inglesa "New Scientist" veicula uma
notícia intitulada "Vitória da evolução na justiça". Ela se refere à
proibição judicial de o governo do condado de Cobb, no Estado da Geórgia
(EUA), obrigar os livros de biologia a trazer uma tarja com os dizeres:
"Evolução é uma teoria, não um fato".

A medida foi considerada pela corte como propaganda religiosa, o que é
ilegal em escolas que recebem financiamento público.

Esse é apenas um exemplo das contínuas tentativas realizadas por alguns
grupos religiosos para solapar o ensino da evolução nas escolas americanas.

No Brasil, o movimento criacionista e sua corrente-irmã, o design
inteligente, apoiados por políticos oportunistas locais, arvoram-se em
incluir suas idéias no currículo escolar de ciências biológicas, em
detrimento do ensino da evolução.

Considerar as idéias criacionistas e do chamado design inteligente como
teorias científicas e colocá-las em pé de igualdade com o evolucionismo é
deturpar o significado dos termos "teoria" e "ciência".

No contexto científico, teoria refere-se a uma explicação abrangente e bem
consolidada de algum aspecto do mundo natural, que pode incorporar fatos,
leis, inferências e hipóteses passíveis de teste.

Ciência, por sua vez, pode ser definida como um processo que tenta encontrar
explicações para os fenômenos naturais por meio de inferências lógicas
baseadas em observações empíricas.

O criacionismo e o design inteligente não têm status de ciência, pois não
geram hipóteses que possam ser testadas e não se pautam por inferências
lógicas com base em observações empíricas do mundo natural.

O criacionismo se baseia em dogmas relatados no livro do Gênesis. O chamado
design inteligente se preocupa em encontrar falhas nos testes das hipóteses
geradas com base nos princípios darwinistas, sem apresentar teorias próprias
ou hipóteses que possam ser submetidas a testes científicos.

Sua principal plataforma é que a ciência ainda não tem explicações
definitivas para a origem da vida e para uma reconstituição minuciosa, passo
a passo, de como, a partir de organismos simples, surgiram formas mais
complexas de vida.

Para os defensores da idéia de design inteligente, o que é ainda um mistério
hoje continuará misterioso para sempre e melhor do que procurar explicações
com base no método científico é invocar forças sobrenaturais.

O evolucionismo, por outro lado, parte do princípio de que não há verdades
inquestionáveis e que sempre existe a possibilidade de uma explicação
considerada verdadeira estar errada.

As idéias atualmente aceitas pela ciência são aquelas que, depois de
testadas exaustivamente, não foram refutadas. Mesmo assim, as explicações
científicas nunca são consideradas verdades absolutas; elas são aceitas
enquanto não existirem motivos para se duvidar de sua veracidade, isto é,
enquanto não forem refutadas pelos testes.

A teoria da evolução biológica vem resistindo a todos os testes a que tem
sido submetida, sendo a única explicação racional e coerente para o conjunto
de fatos sobre a vida em nosso planeta.

O evolucionismo é o tema unificador de todos os campos das Ciências
Biológicas: como disse o célebre geneticista Theodosius Dobzhansky
(1900-1975), "nada faz sentido em biologia a não ser sob a luz da evolução".
(Folha de SP, 30/1)


[ ] 's do Emiliano Chemello
emiliano@quimica.net
http://www.quimica.net/emiliano
http://www.ucs.br/ccet/defq/naeq

" Rien ne se perd, rien ne se crée,
tout se transforme."

Antoine Laurent de Lavoisier (químico francês, 1743 - 1794)




SUBJECT: Quadrinhos e ciência
FROM: "E m i l i a n o C h e m e l l o" <chemelloe@yahoo.com.br>
TO: <ciencialist@yahoogrupos.com.br>, <forum-ciencia@yahoogrupos.com.br>
DATE: 31/01/2005 16:10

JC e-mail 2698, de 31 de Janeiro de 2005.
Livro analisa relação entre as histórias em quadrinhos e a ciência

'Ciência e Quadrinhos' é uma grande pedida para as pessoas interessadas em
analisar como os roteiristas de HQ utilizam a ciência e a técnica em suas
histórias

O livro é a transcrição do primeiro capítulo da dissertação de mestrado de
Gian Danton (pseudônimo do professor universitário Ivan Carlo Andrade de
Oliveira), defendida na Universidade Metodista de SP, em 1997.

Em seu trabalho, Gian livro mostra como no começo, as histórias em
quadrinhos têm uma relação de maravilhamento com a ciência.

É uma fase ingênua, em que os cientistas são mostrados de forma romântica,
como solitários benfeitores da humanidade ou solitários malucos, prontos
para usar as suas descobertas para escravizar a humanidade.

Num segundo momento, os cientistas são mostrados como fazendo parte de
projetos governamentais, muitas vezes de natureza militar. Finalmente, em um
momento mais recente, os quadrinistas passam a fazer uma avaliação crítica
da ciência, divulgando novos paradigmas e denunciando aspectos ideológicos
do fazer científico.

A idéia para o livro surgiu a partir da grande procura por obras do gênero.
'Muitos professores, interessados em usar as histórias em quadrinhos em suas
aulas de ciências me mandavam e-mails perguntando se minha dissertação de
mestrado já havia sido publicada. Percebi que havia a falta de um livro que
analisasse como os quadrinhos podem refletir sobre a realidade científica e
como isso pode ser feito sem perder o caráter de diversão dos gibis', afirma
o autor.

Gian Danton é autor de livros voltados ao jornalismo e à cultura pop, além
de organizador da coletânea Agulha Hipodérmica - o poder e os efeitos dos
meios de comunicação de massa.

Em 1999 recebeu o Prêmio Especial Nova-SBAF e os troféus Ângelo Agostini e
HQ Mix por uma de suas histórias, ilustrada por Antônio Éder e José Aguiar,
publicada na revista Manticore Especial (editora Monalisa).

É colunista do Mundo cultural (http://www.mundocultural.com) e colabora
freqüentemente com diversos fanzines e publicações alternativas literárias e
de quadrinhos. Atualmente é professor universitário em Macapá, Amapá.

Ciência e Quadrinhos faz parte da coleção Quiosque, da editora Marca de
Fantasia, e pode ser adquirido através do sítio da editora:
marcadefantasia.sites.uol.com.br.


[ ] 's do Emiliano Chemello
emiliano@quimica.net
http://www.quimica.net/emiliano
http://www.ucs.br/ccet/defq/naeq

" Rien ne se perd, rien ne se crée,
tout se transforme."

Antoine Laurent de Lavoisier (químico francês, 1743 - 1794)




SUBJECT: ninguem é inocente.
FROM: "rayfisica" <rayfisica@yahoo.com.br>
TO: ciencialist@yahoogrupos.com.br
DATE: 31/01/2005 17:14


Isso é manipulação do método, ou é método mesmo?
Pelo menos o ibope da uma margem de erro de 5% pra mais ou pra menos.
>>>>>>>>>>>>>>>>>
Shroud of Turin: Old as Jesus?
By THE NEW YORK TIMES

Published: January 27, 2005


he Shroud of Turin is much older than the medieval date that modern
science has affixed to it and could be old enough to have been the
burial wrapping of Jesus, a new analysis concludes.
Since 1988, most scientists have confidently concluded that it was the
work of a medieval artist, because carbon dating had placed the
production of the fabric between 1260 and 1390.
In an article this month in the journal Thermochimica Acta, Dr.
Raymond N. Rogers, a chemist retired from Los Alamos National
Laboratory, said the carbon dating test was valid but that the piece
tested was about the size of a postage stamp and came from a portion
that had been patched.
"We're darned sure that part of the cloth was not original Shroud of
Turin cloth," he said, adding that threads from the main part of the
shroud were pure linen, which is spun from flax.
The threads in the patched portion contained cotton as well and had
been dyed to match.

SUBJECT: Re: [ciencialist] ninguem é inocente.
FROM: "Oraculo" <oraculo@atibaia.com.br>
TO: <ciencialist@yahoogrupos.com.br>
DATE: 31/01/2005 17:41

Olá ray

Na verdade, isso é uma manipulação religiosa de afirmações sem base e que são mantidas assim por motivos obvios..:-)

O estudo de uma peça antropológica, estudo científico, exige um enorme volume de dados, de comparações, de estudos em diferentes matérias, para permitir conclusão segura ou minimamente confiável. Dado o tempo, as condições, a pouca informação, os registros imprecisos de nossa história, um artefato antigo exige cautela e esforço apra ser corretametne avalidado, quase sempre por diferentes pesqusiadores de diferentes áreas do conhecimento. Apenas se pasar por todos eles com algum sucesso é que as conclusòes a respeito tem alguma confiabilidade.

Assim, ao estudar um artefato, digamos um tecido ou ceramica antigas, os pesquisadores devem contar com uma ampla gama de ciências, que incluem a datação por carbono, mas não apenas a datação por carbono.

Mas o Santo Sudário se mantém escondido e protegido pela igreja, que nunca assumiu a realidade do mesmo, afirmando que é apenas um simbolo religioso, e não o real manto que cobriu jesus. E impede qualquer tentativa de análise mais profunda.

O primeiro estudo, ainda não refutado com sucesso, contou com pequenos fiapos de linha. Seu autor afirma que foram captados de trechos originais do sudario e não de trechos colocados como restauro. Sua análise do novo estudo pode ser encontrada na Web e deve ser lida antes de qualquer conclusão.

De todo modo, há uma forma de terminar com a questão ou no mínimo ampliar de forma concreta as bases para conclusão: a permissão do vaticano para o estudo completo e laboratorial do sudario. Cientistas tomam todo cuidado com artefatos importantes e frágeis, e jamais estragariam o sudario de toda forma (aliás, a igreja quando precisa restaurar e proteger seus tesouros, sempre chama cientistas e especialistas para isso).

Ter entre 1300 e 3000 anos é vago o suficiente para permitir qualquer conclusão, se for apenas o que se tem como base. Se tem 1300, não foi usado por jesus. Se tem 3000 ou 2500, também não. Sim, claro, muitos alegarão que, tendo 2500, pode ter sido usado, já que nada impede que o tecido tenha sido guardado e usado posteriormente. Mas, esse argumento deve ser usado também para o caso do sudário ter 2005 anos (data exata..:-), já que nada impede que um falsificador da idade média usasse um tecido antigo para a fraude.

Precisariamos do sudário e de muitos cientistas para uma conclusão confiável. Mas...

Mas, fiapos de pano continuarão a ser tudo que teremos para análise..:-) Isso pode ser chamado de manipulação, não do método cientifico, mas da esperança das pessoas que crêem.

Um abraço.

Homero

----- Original Message -----
From: rayfisica
To: ciencialist@yahoogrupos.com.br
Sent: Monday, January 31, 2005 5:14 PM
Subject: [ciencialist] ninguem é inocente.



Isso é manipulação do método, ou é método mesmo?
Pelo menos o ibope da uma margem de erro de 5% pra mais ou pra menos.
>>>>>>>>>>>>>>>>>
Shroud of Turin: Old as Jesus?
By THE NEW YORK TIMES

Published: January 27, 2005


he Shroud of Turin is much older than the medieval date that modern
science has affixed to it and could be old enough to have been the
burial wrapping of Jesus, a new analysis concludes.
Since 1988, most scientists have confidently concluded that it was the
work of a medieval artist, because carbon dating had placed the
production of the fabric between 1260 and 1390.
In an article this month in the journal Thermochimica Acta, Dr.
Raymond N. Rogers, a chemist retired from Los Alamos National
Laboratory, said the carbon dating test was valid but that the piece
tested was about the size of a postage stamp and came from a portion
that had been patched.
"We're darned sure that part of the cloth was not original Shroud of
Turin cloth," he said, adding that threads from the main part of the
shroud were pure linen, which is spun from flax.
The threads in the patched portion contained cotton as well and had
been dyed to match.
From other tests, he estimated that the shroud was between 1,300 and
3,000 years old.





##### ##### #####

Para saber mais visite
http://www.ciencialist.hpg.ig.com.br


##### ##### ##### #####


Yahoo! Grupos, um serviço oferecido por:

São Paulo Rio de Janeiro Curitiba Porto Alegre Belo Horizonte Brasília




------------------------------------------------------------------------------
Links do Yahoo! Grupos

a.. Para visitar o site do seu grupo na web, acesse:
http://br.groups.yahoo.com/group/ciencialist/

b.. Para sair deste grupo, envie um e-mail para:
ciencialist-unsubscribe@yahoogrupos.com.br

c.. O uso que você faz do Yahoo! Grupos está sujeito aos Termos do Serviço do Yahoo!.



[As partes desta mensagem que não continham texto foram removidas]



SUBJECT: Venus flytrap para Emiliano
FROM: Maria Natália <grasdic@hotmail.com>
TO: ciencialist@yahoogrupos.com.br
DATE: 31/01/2005 18:05


Não é bem uma aranha mas anda lá pelos predadores bem
sucedidos...segundo investigador de Harvard

"How a venus flytrap traps its victims

A VENUS flytrap snaps shut faster than you can blink. And now we know
why. Whereas our sluggish movements are the result of muscles
contracting, the plant snaps shut in the way that a torn tennis ball
flips inside out.
When Lakshminarayanan Mahadevan of Harvard University and his
colleagues used a high-speed camera to film the leaves closing, they
noticed that the curvature of the leaves flipped from convex to
concave as the trap closes. The transformation takes just one-tenth of
a second. They reasoned that the leaves were snapping from one stable
shape to another - a movement that can occur much faster than muscle
contraction.Although they do not yet know what happens at the cellular
level, they have devised equations that describe the mechanical action
(Nature, vol 433, p 421).

SUBJECT: Sobre Aspartame
FROM: "murilo filo" <avalanchedrive@hotmail.com>
TO: ciencialist@yahoogrupos.com.br, forum-ciencia@yahoogrupos.com.br
DATE: 31/01/2005 19:00

Isto é o 'crime da mala' e já deve ter circulado por aquí.
Será que o resultado químico é êste mesmo?
Eu nunca teria esperado isto da monsanto... M.

>
>>>Subject: FW: Sobre Aspartame
>>>Date: Thu, 27 Jan 2005 17:37:56 -0200
>>>
>>> > Subject: Aspartame - Conheça a realidade
>>> >
>>> >
>>> > ASPARTAME
>>> >
>>> > CONFERÊNCIA MUNDIAL DO MEIO AMBIENTE E FUNDAÇÃO ESCLEROSE MÚLTIPLA EM
>>>ROTA DE COLISÃO COM A MONSANTO
>>> >
>>> > (Artigo escrito pela Dra. Mancy Marckle)
>>> >
>>> >
>>> > Passei alguns dias falando na CONFERÊNCIA MUNDIAL DE MEIO AMBIENTE a
>>>respeito do ASPARTAME, conhecido como Nutrasweet, Equal e Spoonful. Eles
>>>anunciaram que existia uma epidemia de esclerose múltipla e lúpus
>>>sistêmico e não entendiam que toxina estava fazendo com que essas doenças
>>>assolassem os Estados Unidos tão rapidamente. Eu expliquei que eu estava
>>>lá para falar exatamente sobre este assunto.
>>> >
>>> > Quando a temperatura do ASPARTAME excede 30º C, o álcool contido no
>>>ASPARTAME se converte em formaldeído e daí para ácido fórmico, que
>>>provoca acidose metabólica (o ácido fórmico é o veneno das formigas).
>>> >
>>> > A toxidade do metanol imita a esclerose múltipla e as pessoas recebem
>>>diagnóstico errado de esclerose múltipla. A esclerose múltipla não se
>>>constitui em sentença de morte, mas a toxidade do metanol sim. No caso do
>>>lúpus sistêmico, estamos percebendo que é quase tão grave quanto a
>>>esclerose múltipla especialmente em usuários de DIET COKE e DIET PEPSI.
>>> >
>>> > Não é para menos, considerada a toxidade do metanol ... (As vítimas
>>>geralmente bebem de 3 a 4 latas desses refrigerantes por dia, ou mais).
>>>[Sou eu!!! Se alguém souber alguma coisa que comprove a veracidade desta
>>>mensagem, avise-me. Quanto ao meu depoimento, posso dizer que não
>>>apresento os sintomas relatados.] Nos casos de lúpus sistêmico causado
>>>pelo ASPARTAME, a vítima geralmente não sabe que o ASPARTAME é causa de
>>>sua doença e continua com seu uso, agravando o lúpus a um grau tão
>>>intenso que algumas vezes ameaça a vida. Quando interrompemos o uso do
>>>ASPARTAME, as pessoas que tinham lúpus ficam assintomáticas.
>>> >
>>> > Infelizmente, não podemos reverter essa doença. Por outro lado, nos
>>>casos diagnosticados como esclerose múltipla (quando, na realidade, a
>>>doença é devida à toxidade do metanol) a maioria dos sintomas
>>>desaparece. Nós temos visto casos em que a visão retornou e mesmo a
>>>audição for recuperada. Isso também se aplica aos casos de tinittus
>>>auricularis (zumbido no ouvido).
>>> >
>>> > Em uma conferência eu disse: - Se você está usando ASPARTAME
>>>(Nutrasweet, Equal e Spoonful, etc.) e sofre de sintomas como
>>>fibromialgia, espasmos, dores, formigamentos nas pernas, câimbras,
>>>vertigem, tontura, dor de cabeça, zumbido no ouvido, dores articulares,
>>>depressão, ataques de ansiedade, fala atrapalhada, visão borrada ou perda
>>>de memória - VOCÊ PROVAVELMENTE TEM A DOENÇA DO ASPARTAME! As pessoas
>>>começaram a pular durante a palestra dizendo: - Eu tenho isto, é
>>>reversível?
>>> >
>>> > É impressionante. Em uma palestra assistida pelo embaixador de Uganda,
>>>ele nos contou que a indústria de açúcar deles está adicionando ASPARTAME
>>>ao açúcar! Ele contou que o filho de um dos líderes da indústria não
>>>conseguia andar - em parte pelo uso do produto! Estamos com um sério
>>>problema.
>>> >
>>> > Um estranho veio até o Dr. Espisto (um de meus palestrantes) e >
>>>perguntou
>>>por que tantas pessoas estavam tendo esclerose múltipla (MS). Durante a
>>>visita a um hospital, uma enfermeira disse que seis amigos dela que eram
>>>viciados em diet coke, tinham sido diagnosticados com MS. Isso é mais do
>>>que coincidência.
>>> >
>>> > Há um tempo atrás houve audiências no congresso dos EUA incluindo o
>>>ASPARTAME em 100 produtos diferentes. Nada foi feito. Os lobbies da droga
>>>e da indústria química tem bolsos muito profundos. Agora existem mais de
>>>5.000 produtos contaminados com esse produto químico e a patente
>>>expirou. Na época da primeira audiência, as pessoas estavam ficando
>>>cegas. O metanol no ASPARTAME se converte em formaldeído na retina do
>>>olho. Formaldeído é do mesmo grupo das drogas como cianeto e arsênico:
>>>venenos mortais! Infelizmente, leva muito tempo para matar, mas está
>>>matando as pessoas e causando todos os tipos de problemas> neurológicos.
>>> >
>>> > O ASPARTAME muda a química do cérebro. É a causa de diversos tipos de
>>>ataque. Essa droga muda os níveis de dopamina no cérebro. Imagine o que
>>>acontece com os pacientes que sofrem de Doença de Parkinson? Também
>>>causa mal formações fetais.
>>> >
>>> > Não existe nenhuma razão para se utilizar esse produto. NÃO É UM
>>>PRODUTO DIETÉTICO! Os anais do Congresso dizem: ele faz você desejar
>>>carboidratos e, em conseqüência, engordar. Dr. Roberts viu que quando
>>>interrompeu o uso de ASPARTAME a perda de peso foi de 9,5 Kg por pessoa.
>>> >
>>> > O formaldeído se armazena nas cédulas adiposas, principalmente nos
>>>quadris e coxas. O ASPARTAME é especialmente mortal para os diabéticos.
>>>Todos os médicos sabem o que o metanol causaria num diabético. Muitos
>>>médicos acreditam que seus pacientes têm retinopatia, quando de fato, o
>>>mal é causado pelo ASPARTAME.
>>> >
>>> > O ASPARTAME mantém o açúcar sangüíneo fora de controle, fazendo com
>>>que
>>>muitos pacientes entrem em coma. Infelizmente, muitos morreram. Pessoas
>>>nos contaram na Conferência do Colégio Americano de Medicina que tinham
>>>parentes que mudaram de sacarina para o ASPARTAME e agora eventualmente
>>>entram em coma. Seus médicos não conseguem controlar os níveis de
>>>glicemia.
>>> >
>>> > Os pacientes têm perda de memória pelo fato de que o ácido aspártico
>>>e
>>>a fenilalanina são neurotóxicos sem os outros aminoácidos encontrados
>>>nas proteínas. Eles atravessam a barreira hemato-encefálica e causam
>>>deterioração nos neurônios. Dr. Russel Blaylock, neurocirurgião, diz: Os
>>>ingredientes estimulam os neurônios até a morte causando dano cerebral
>>>em vários níveis. Dr. Blaylock escreveu um livro intitulado:
>>>"Excitotoxinas: O Gosto que Mata". (Health Press, 1-800-643-2665).
>>> >
>>> > O Dr. H. J. Roberts, especialista diabético e perito mundial em
>>>envenenamento pelo ASPARTAME, escreveu um livro intitulado: "DEFESA
>>>CONTRA A DOENÇA DE ALZHEIMER" (1-800-814-8900). Dr. Roberts mostra como o
>>>envenenamento pelo ASPARTAME está relacionado à doença de Alzheimer. E
>>>realmente está. Mulheres de 30 anos estão sendo internadas com Alzheimer.
>>>Dr.Blaylock e Dr. Roberts estão escrevendo uma carta-posição com alguns
>>>casos relatados e vão colocá-la na Internet. De acordo com a conferência
>>>do Colégio Americano de Medicina, nós estamos falando de uma praga de
>>>doenças neurológicas causada por esse veneno mortal. Dr.Roberts
>>>descobriu o que aconteceu quando o ASPARTAME foi vendido pela primeira
>>>vez. Ele disse que seus pacientes diabéticos apresentaram perda de
>>>memória, confusão e severa perda de visão.
>>> >
>>> > Na Conferência do Colégio Americano de Medicina, os médicos admitiram
>>>que não sabiam. Eles estavam procurando saber por que ataques tinham
>>>aumentado tanto (a fenilalanina do ASPARTAME diminui o limiar para a
>>>convulsão e depleta a serotonina, o que causa psicose maníaco
>>>depressiva, ataque de pânico, fúria e violência).
>>> >
>>> > Antes da Conferência, eu recebi um fax da Noruega, pedindo um possível
>>>antídoto para esse veneno, porque a pessoa estava sentindo esses tantos
>>>problemas no seu País. Esse veneno, agora, está disponível em muitos
>>>países.
>>> >
>>> > Felizmente, tivemos embaixadores e palestrantes na Conferência que se
>>>engajaram nesta luta. PEDIMOS QUE VOCÊ SE ENGAJE TAMBÉM.
>>> >
>>> > Imprima este artigo e avise todas as pessoas que você conhece.
>>> >
>>> > Tire tudo o que contém aspartame do armário.
>>> >
>>> > Envie para nós sua história.
>>> >
>>> >
>>> > Eu asseguro que a Monsanto, criadora do ASPARTAME, sabe como ele é
>>>mortal. Ela financia a Associação Americana de Diabetes, a Associação
>>>Americana de Dietética, o Congresso e a Conferência do Colégio Americano
>>>de Medicina. O New York Times, em 15 de Novembro de 1996, publicou
>>>artigo a respeito de como a Associação Americana de Dietética recebe
>>>dinheiro da indústria alimentícia para endossar seus produtos. Por isso,
>>>eles não podem criticar ou falar a respeito de sua ligação com a
>>>MONSANTO.>
>>> >
>>> > A que ponto chega isso? Às raias da perversidade. Dissemos a uma mãe
>>>cujo filho estava usando NutraSweet que ela deveria interromper o uso do
>>>produto. A criança estava tendo convulsões diárias. A mãe telefonou para
>>>o médico, que telefonou para a Associação. E a Associação disse ao
>>>médico para não interromper o uso de NutraSweet. Estamos ainda tentando
>>>convencer a mãe que o ASPARTAME está causando as convulsões. Toda vez que
>>>interrompemos o uso do ASPARTAME, as convulsões cessam. Se o bebê morrer,
>>>sabemos de quem é a culpa e contra quem lutamos.
>>> >
>>> > Existem 92 sintomas documentados de ASPARTAME, do coma à morte. A
>>>maioria deles é neurológica, porque o ASPARTAME destrói o sistema
>>>nervoso.
>>> >
>>> > A doença do ASPARTAME é parcialmente a causa da síndrome "Tempestade
>>> > No
>>>Deserto". A queimação na língua e os outros sintomas discutidos em mais
>>>de 60 casos podem estar diretamente relacionados ao consumo de produtos
>>>contendo ASPARTAME. Milhares de latas de bebidas diet foram enviadas para
>>>as tropas do "Tempestade no Deserto". ( Lembre- se que o calor pode
>>>liberar o metanol do ASPARTAME a 30º C.) As bebidas dietéticas foram
>>>expostas ao sol de 45ºC no deserto árabe, por semanas. Homens e mulheres
>>>de serviço bebiam isso o dia todo. Todos os sintomas deles eram
>>>semelhantes ao envenenamento por ASPARTAME.
>>> >
>>> > Dr. Roberts diz que o consumo do ASPARTAME na época da concepção pode
>>>causar defeitos no feto. A FENILALANINA se concentra na placenta
>>>causando retardo mental, de acordo com o Dr. Louis Elsas, Professor de
>>>Genética Pediátrica na Universidade de Emory. Em testes de laboratório,
>>>animais desenvolvem tumores cerebrais (afenilalanina tem um subproduto o
>>>DXP, agente causador de tumores cerebrais).
>>> >
>>> > Quando o Dr. Espisto estava falando, um neurocirurgião da platéia >
>>>disse:
>>>- Encontra-se, de fato, um teor elevado de ASPARTAME nos tumores
>>>cerebrais removidos.
>>> >
>>> > A STÉVIA, um adoçante natural, NÃO É UM ADITIVO, e ajuda no >
>>>metabolismo
>>>do açúcar: seria ideal para os diabéticos e foi aprovada, agora, como
>>>suplemento dietético pelo FDA. Durante anos, o FDA adiou essa aprovação
>>>por causa da sua lealdade à MONSANTO.
>>> >
>>> > (O texto acima foi corrigido por Nivaldo Alves Soares e Josealdo >
>>>Tonholo
>>>- Depto. Química/CCEN - Universidade Federal de Alagoas) - Campus A. C.
>>>Simões - Tabuleiro do Martins 57.072-970 - Maceió - AL - Brasil -
>>>Fone/fax: (082) 214-1389)
>>> >
>>> > Nota: Em 05/07/99 a UPI Farm News noticiou que a Mosanto está
>>>colocando
>>>à venda seu braço de ingredientes alimentares, entre as quais a
>>>NutreSweet. Teriam essas vendas ligações com o artigo acima?
>>> >
>>> >
>>> >
>>> > MONSANTO VENDE NUTRASWEET - UPI Farm News - 05/07/99
>>> >
>>> > "A Monsanto está vendendo seus negócios no ramo de ingredientes
>>>alimentares, inclusive o braço de adoçantes NutraSweet. A receita destas
>>>vendas vai auxiliar a empresa química e gigante da pesquisa agrícola -
>>>com sede em St. Louis - a saldar os débitos da compra desmesurada de
>>>biotecnologia, entre as quais a de empresas produtoras de semente.
>>> >
>>> > A Monsanto necessitava das produtoras de sementes como meio para >
>>>escoar
>>>seu material genético "high-tech" para os agricultores. Mas a companhia
>>>está vendendo empresas para pagar seus débitos resultantes dos
>>>investimentos em biotecnologia desde outubro quando fracassaram os seus
>>>planos para fusão com a American Home Products (com sede em New Jersey).
>>> >
>>> > Os negócios na área de ingredientes alimentares incluem a NutraSweet,
>>> > o
>>>adoçante artificial usado em refrigerantes, além de outros produtos à
>>>base de goma. O NutraSweet também é vendido como adoçante de mesa sob a
>>>marca de Equal."
>>> >
>>> > Sugerimos que cada pessoa que receba este artigo o divulgue para o >
>>>maior
>>>número possível de pessoas e entidades. Precisamos combater os autores de
>>>mais esse crime contra a saúde humana, isto é, a Monsanto e as várias
>>>entidades mencionadas neste artigo.
>>> >
>>> >
>>> >
>>> >
>>>------------------------------------------------------------------------
>>> >
>>> >
>>> > Internal Virus Database is out-of-date.
>>> > Checked by AVG Anti-Virus.
>>> > Version: 7.0.289 / Virus Database: 265.5.4 - Release Date: 15/12/04
>>> >
>>> >
>>> >
>>> > No virus found in this outgoing message.
>>> > Checked by AVG Anti-Virus.
>>> > Version: 7.0.289 / Virus Database: 265.6.13 - Release Date: 16/01/05
>>> >
>>> > | | | | | Caixa de Entrada
>>> > É melhor com a borboleta.
>>> > Início | Meu MSN | Hotmail | Busca | Compras |
>>>Grupos & bate-papo | Messenger Grátis Comentários | Ajuda
>>> > © 2004 Microsoft Corporation. Todos os direitos reservados. TERMOS DE
>>>USO Declaração de Privacidade POLÍTICA ANTI-SPAM DA MICROSOFT
>>> >
>>> >
>>
>>_________________________________________________________________
>>MSN Messenger: converse online com seus amigos .
>>http://messenger.msn.com.br
>>
>




SUBJECT: Re: Sobre Aspartame
FROM: "rmtakata" <rmtakata@altavista.net>
TO: ciencialist@yahoogrupos.com.br
DATE: 31/01/2005 19:20


Para detalhes vc pode pesquisar no historico da lista (use a
palavra-chave aspartame).

Mas basicamente: em uma laranja tem muito mais metanol do q. em um
vidro de adocante dietetico.

(O problema principal eh para os fenilcetonuricos. De resto, teria q.
tomar mais de 60 latinhas de refri dietetico ou light por dia para
aumentar significativamente o risco 'a saude.)

Antes de acreditar em emails do genero, consulte sitios como:

http://www.snopes.com/

Ou

http://www.quatrocantos.com/lendas/lendas_urbanas.htm

Se nao acreditar mais no email, delete-o sem repassar. Se continuar
acreditando, guarde-o para si e nao repasse.

[]s,

Roberto Takata





SUBJECT: Fw: Importante.
FROM: "Rodrigo Toledo" <rodrigotoledo11@uol.com.br>
TO: <ciencialist@yahoogrupos.com.br>
DATE: 31/01/2005 20:50

Desculpem-me pelo off-topic, mas alguém já ouviu falar disso ?

Obrigado,

Rodrigo

----------------------------------------------------


Dêem uma olhada nesse texto... É atribuído a uma Professora da USP. Eu pesquisei no site da USP e essa Professora realmente existe... ainda vou encaminhar esse texto para ela para saber se se trata de uma invenção, um spam, ou de algo que ela realmente escreveu, algo verdadeiro e sério (e se for verdade, é sério MESMO!). Um grande amigo meu que se formou em medicina e agora está indo servir o Exército, como médico, em Boa Vista, disse que essa história realmente tem muito de verdade pois ele já escutou falar muito a respeito disso dentro do próprio Exército! Ele me enviou, inclusive, os dois sites abaixo para quem quiser maiores informações a respeito:

http://www.technet.com.br/~cmdo1bda/ (detalhe para o link "EXÉRCITO REPUDIA DENÚNCIAS PUBLICADAS NO SITE DO JORNAL THE NEW YORK TIMES")

http://www.technet.com.br/~cmdo1bda/uiramuta.html (interessante história sobre o embargo judicial na construção de uma base do Exército na fronteira com a Venezuela)

Abraço a todos!
Thiago Francisco Amaral Moreira de Assis
Analista de Planejamento e Controle
UNIBANCO - Acompanhamento de Gestão Estratégica
Tel.: +55 (11) 3097-5044

"A liberdade mais sagrada é a liberdade de pensar."
(C. B. González Pecotche - Autor da Logosofia)


-----Mensagem original-----
De: Carlos Eduardo Albanez Pimenta
Enviada em: sexta-feira, 7 de janeiro de 2005 14:52
Assunto: A PRÓXIMA GUERRA


Leiam e repassem!

++++++++++++++++++
A PRÓXIMA GUERRA
A pedidos, segue abaixo o relato de uma pessoa conhecida e séria, que passou recentemente em um concurso público federal e foi trabalhar em Roraima.

Trata-se de um Brasil que a gente não conhece.
As duas semanas em Manaus foram interessantes para conhecer um Brasil um pouco diferente, mas chegando em Boa Vista
(RR) não pude
resistir a fazer um relato das coisas que tenho visto e escutado por aqui.
Conversei com algumas pessoas nesses três dias, desde engenheiros até pessoas com um mínimo de instrução.
Para começar o mais difícil de encontrar por aqui é roraimense, pra falar a verdade, acho que a proporção é de um roraimense para cada 10 pessoas é bem razoável, tem gaúcho, carioca, cearense, amazonense, piauiense, maranhense e por aí vai. Portanto falta uma identidade com a terra. Aqui não existem muitos meios de sobrevivência, ou a pessoa é funcionária pública, e aqui qua se todo mundo é, pois em Boa Vista se concentram todos os órgãos federais e estaduais de Roraima, além da prefeitura é claro. Se não for funcionário público a pessoa trabalha no comércio local ou recebe ajuda de Programas do governo. Não existe indústria de qualquer tipo. Pouco mais de 70% do Território roraimense é demarcado como reserva indígena, portanto restam apenas 30%, descontando-se os rios e as terras improdutivas que são muitas, para se cultivar a terra ou para a localização das próprias cidades. Na única rodovia que existe em direção ao Brasil (liga Boa Vista a Manaus, cerca de 800 km) existe um trecho de aproximadamente 200 km reserva indígena Waimiri Atroari) por onde você só passa entre 6:00 da manhã e 6:00 da tarde, nas outras 12 horas a rodovia é fechada pelos índios (com autorização da FUNAI e dos americanos) para que os mesmos não sejam incomodados.

Detalhe:

Você não passa se for brasileiro, o ace sso é livre aos americanos, europeus e japoneses. Desses 70% de território indígena, diria que em 90% dele ninguém entra sem uma grande burocracia e autorização da FUNAI.

Detalhe:
Americanos entram na hora que quiserem, se você não tem uma autorização da FUNAI mas tem dos americanos então você pode entrar. A maioria dos índios fala a língua nativa além do inglês ou francês, mas a maioria não sabe falar português. Dizem que é comum na entrada de algumas reservas encontrarem-se hasteadas bandeiras americanas ou inglesas. É comum se encontrar por aqui americano tipo nerds com cara de quem não quer nada, que veio caçar borboleta e joaninha e catalogá-las, mas no final das contas pasme, se você quiser montar um empresa para exportar plantas e frutas típicas como cupuaçu, açaí camu-camu etc., medicinais, ou componentes naturais para fabricação de remédios, pode se preparar para pagar "royalties" para empresas japonesas e americanas que já patentearam a maioria dos produtos típicos da Amazônia...
Por três vezes repeti a seguinte frase após ouvir tais relatos: É os americanos vão acabar tomando a Amazônia e em todas elas ouvi a mesma resposta em palavras diferentes. Vou reproduzir a resposta de uma senhora simples que vendia suco e água na rodovia próximo de Mucajaí:

"Irão não minha filha, tu não sabe, mas tudo aqui já é deles, eles comandam tudo, você não entra em lugar nenhum porque eles não deixam.
Quando acabar essa guerra aí eles virão pra cá, e vão fazer o que fizeram no Iraque quando determinaram uma faixa para os curdos onde iraquiano não entra, aqui vai ser a mesma coisa".

A dona é bem informada não? O pior é que segundo a ONU o conceito de nação é um conceito de soberania e as áreas demarcadas têm o nome de nação indígena. O que pode levar os americanos a alegarem que estarão libertando os povos indígenas. Fiqu ei sabendo que os americanos já estão construindo uma grande base militar na Colômbia, bem próximo da fronteira com o Brasil numa parceria com o governo colombiano com o pseudo objetivos de combater o narcotráfico. Por falar em narcotráfico, aqui é rota de distribuição, pois essa mãe chamada Brasil mantém suas fronteiras abertas e aqui tem Estrada para as Guianas e Venezuela. Nenhuma bagagem de estrangeiro é fiscalizada, principalmente se for americano, europeu ou japonês, (isso pode causar um incidente diplomático)... Dizem que tem muito colombiano traficante virando venezuelano, pois na Venezuela é muito fácil comprar a cidadania venezuelana por cerca de 200 dólares.
Pergunto inocentemente às pessoas; porque os americanos querem tanto proteger os índios. A resposta é absolutamente a mesma, porque as terras indígenas além das riquezas animais e vegetais, da abundância de água são extremamente ricas em ouro (encontram-se pepitas que chegam a ser pesadas em quilos), diamante, outras pedras preciosas, minério e nas reservas norte de Roraima e Amazonas, ricas em PETRÓLEO.

Parece que as pessoas contam essas coisas como que num grito de Socorro a alguém que é do sul, como se eu pudesse dizer isso ao presidente ou a alguma autoridade do sul que vá fazer alguma coisa. É pessoal, saio daqui com a quase certeza de que em breve o Brasil irá diminuir de tamanho.
Um grande abraço a todos. Será que podemos fazer alguma coisa???
Acho que sim.

Repasse esse e-mail para que um maior número de brasileiros fique sabendo desses absurdos.

Mara Silvia Alexandre Costa
Depto de Biologia Cel. Mol. Bioag. Patog.
FMRP - USP

Opinião pessoal:
Gostaria que você, especialmente que recebeu este e-mail, o repasse para o maior número possível de pessoas. Do meu ponto de vista seria interessante que o p aís inteiro ficasse sabendo desta situação através dos telejornais antes que isso venha a acontecer.
Afinal foi um momento de fraqueza dos Estados Unidos que os europeus lançaram o Euro, assim poderá se aproveitar esta situação de fraqueza norte-americana (perdas na guerra do Iraque) para revelar isto ao mundo a fim de antecipar a próxima guerra.

Celso Luiz Borges de Oliveira
Doutorando em Água e Solo
FEAGRI/UNICAMP
Tel: (19) 3233-1840 Celular: (19) 9136-6472
e-mail´s: celso@ufba.br; celso@agr.unicamp.br; celsoborges@gmail.com
__________________________________________________
Converse com seus amigos em tempo real com o Yahoo! Messenger
http://br.download.yahoo.com/messenger/


[As partes desta mensagem que não continham texto foram removidas]



SUBJECT: RE: [ciencialist] Re: Sobre Aspartame
FROM: "murilo filo" <avalanchedrive@hotmail.com>
TO: ciencialist@yahoogrupos.com.br
DATE: 31/01/2005 22:40

Obrigado. O Jr também me disse algo parecido.
É evidente que não faz nenhum mal para a maioria.
A questão que vejo, por minha *própria experiência*, são os efeitos
alérgicos da coisa.
Estou convencido de que pode ser um treim incontrolável e acumulativo.
E sobre os fenilcetonúricos... qual é o lance?
( nota: não sou nem um pouco hipocondríaco! ) abr/M.

>From: "rmtakata" <rmtakata@altavista.net>
>Reply-To: ciencialist@yahoogrupos.com.br
>To: ciencialist@yahoogrupos.com.br
>Subject: [ciencialist] Re: Sobre Aspartame
>Date: Mon, 31 Jan 2005 21:20:39 -0000
>
>
>Para detalhes vc pode pesquisar no historico da lista (use a
>palavra-chave aspartame).
>
>Mas basicamente: em uma laranja tem muito mais metanol do q. em um
>vidro de adocante dietetico.
>
>(O problema principal eh para os fenilcetonuricos. De resto, teria q.
>tomar mais de 60 latinhas de refri dietetico ou light por dia para
>aumentar significativamente o risco 'a saude.)
>
>Antes de acreditar em emails do genero, consulte sitios como:
>
>http://www.snopes.com/
>
>Ou
>
>http://www.quatrocantos.com/lendas/lendas_urbanas.htm
>
>Se nao acreditar mais no email, delete-o sem repassar. Se continuar
>acreditando, guarde-o para si e nao repasse.
>
>[]s,
>
>Roberto Takata
>
>
>




SUBJECT: RE: [ciencialist] Fw: Importante.
FROM: "murilo filo" <avalanchedrive@hotmail.com>
TO: ciencialist@yahoogrupos.com.br
DATE: 31/01/2005 22:45

Êsse treco está rodando pela internet há + de dois anos.
Ví pelo menos umas quatro vêzes, cada uma com um autor diferente.
abr/M.

>From: "Rodrigo Toledo" <rodrigotoledo11@uol.com.br>
>Reply-To: ciencialist@yahoogrupos.com.br
>To: <ciencialist@yahoogrupos.com.br>
>Subject: [ciencialist] Fw: Importante.
>Date: Mon, 31 Jan 2005 20:50:01 -0200
>
>Desculpem-me pelo off-topic, mas alguém já ouviu falar disso ?
>
>Obrigado,
>
>Rodrigo
>
>----------------------------------------------------
>
>
>Dêem uma olhada nesse texto... É atribuído a uma Professora da USP. Eu
>pesquisei no site da USP e essa Professora realmente existe... ainda vou
>encaminhar esse texto para ela para saber se se trata de uma invenção, um
>spam, ou de algo que ela realmente escreveu, algo verdadeiro e sério (e se
>for verdade, é sério MESMO!). Um grande amigo meu que se formou em medicina
>e agora está indo servir o Exército, como médico, em Boa Vista, disse que
>essa história realmente tem muito de verdade pois ele já escutou falar
>muito a respeito disso dentro do próprio Exército! Ele me enviou,
>inclusive, os dois sites abaixo para quem quiser maiores informações a
>respeito:
>
>http://www.technet.com.br/~cmdo1bda/ (detalhe para o link "EXÉRCITO REPUDIA
>DENÚNCIAS PUBLICADAS NO SITE DO JORNAL THE NEW YORK TIMES")
>
>http://www.technet.com.br/~cmdo1bda/uiramuta.html (interessante história
>sobre o embargo judicial na construção de uma base do Exército na fronteira
>com a Venezuela)
>
>Abraço a todos!
>Thiago Francisco Amaral Moreira de Assis
>Analista de Planejamento e Controle
>UNIBANCO - Acompanhamento de Gestão Estratégica
>Tel.: +55 (11) 3097-5044
>
>"A liberdade mais sagrada é a liberdade de pensar."
>(C. B. González Pecotche - Autor da Logosofia)
>
>
>-----Mensagem original-----
>De: Carlos Eduardo Albanez Pimenta
>Enviada em: sexta-feira, 7 de janeiro de 2005 14:52
>Assunto: A PRÓXIMA GUERRA
>
>
>Leiam e repassem!
>
>++++++++++++++++++
>A PRÓXIMA GUERRA
>A pedidos, segue abaixo o relato de uma pessoa conhecida e séria, que
>passou recentemente em um concurso público federal e foi trabalhar em
>Roraima.
>
>Trata-se de um Brasil que a gente não conhece.
>As duas semanas em Manaus foram interessantes para conhecer um Brasil um
>pouco diferente, mas chegando em Boa Vista
>(RR) não pude
>resistir a fazer um relato das coisas que tenho visto e escutado por aqui.
>Conversei com algumas pessoas nesses três dias, desde engenheiros até
>pessoas com um mínimo de instrução.
>Para começar o mais difícil de encontrar por aqui é roraimense, pra falar a
>verdade, acho que a proporção é de um roraimense para cada 10 pessoas é bem
>razoável, tem gaúcho, carioca, cearense, amazonense, piauiense, maranhense
>e por aí vai. Portanto falta uma identidade com a terra. Aqui não existem
>muitos meios de sobrevivência, ou a pessoa é funcionária pública, e aqui
>qua se todo mundo é, pois em Boa Vista se concentram todos os órgãos
>federais e estaduais de Roraima, além da prefeitura é claro. Se não for
>funcionário público a pessoa trabalha no comércio local ou recebe ajuda de
>Programas do governo. Não existe indústria de qualquer tipo. Pouco mais de
>70% do Território roraimense é demarcado como reserva indígena, portanto
>restam apenas 30%, descontando-se os rios e as terras improdutivas que são
>muitas, para se cultivar a terra ou para a localização das próprias
>cidades. Na única rodovia que existe em direção ao Brasil (liga Boa Vista a
>Manaus, cerca de 800 km) existe um trecho de aproximadamente 200 km reserva
>indígena Waimiri Atroari) por onde você só passa entre 6:00 da manhã e 6:00
>da tarde, nas outras 12 horas a rodovia é fechada pelos índios (com
>autorização da FUNAI e dos americanos) para que os mesmos não sejam
>incomodados.
>
>Detalhe:
>
>Você não passa se for brasileiro, o ace sso é livre aos americanos,
>europeus e japoneses. Desses 70% de território indígena, diria que em 90%
>dele ninguém entra sem uma grande burocracia e autorização da FUNAI.
>
>Detalhe:
>Americanos entram na hora que quiserem, se você não tem uma autorização da
>FUNAI mas tem dos americanos então você pode entrar. A maioria dos índios
>fala a língua nativa além do inglês ou francês, mas a maioria não sabe
>falar português. Dizem que é comum na entrada de algumas reservas
>encontrarem-se hasteadas bandeiras americanas ou inglesas. É comum se
>encontrar por aqui americano tipo nerds com cara de quem não quer nada, que
>veio caçar borboleta e joaninha e catalogá-las, mas no final das contas
>pasme, se você quiser montar um empresa para exportar plantas e frutas
>típicas como cupuaçu, açaí camu-camu etc., medicinais, ou componentes
>naturais para fabricação de remédios, pode se preparar para pagar
>"royalties" para empresas japonesas e americanas que já patentearam a
>maioria dos produtos típicos da Amazônia...
>Por três vezes repeti a seguinte frase após ouvir tais relatos: É os
>americanos vão acabar tomando a Amazônia e em todas elas ouvi a mesma
>resposta em palavras diferentes. Vou reproduzir a resposta de uma senhora
>simples que vendia suco e água na rodovia próximo de Mucajaí:
>
>"Irão não minha filha, tu não sabe, mas tudo aqui já é deles, eles comandam
>tudo, você não entra em lugar nenhum porque eles não deixam.
>Quando acabar essa guerra aí eles virão pra cá, e vão fazer o que fizeram
>no Iraque quando determinaram uma faixa para os curdos onde iraquiano não
>entra, aqui vai ser a mesma coisa".
>
>A dona é bem informada não? O pior é que segundo a ONU o conceito de nação
>é um conceito de soberania e as áreas demarcadas têm o nome de nação
>indígena. O que pode levar os americanos a alegarem que estarão libertando
>os povos indígenas. Fiqu ei sabendo que os americanos já estão construindo
>uma grande base militar na Colômbia, bem próximo da fronteira com o Brasil
>numa parceria com o governo colombiano com o pseudo objetivos de combater o
>narcotráfico. Por falar em narcotráfico, aqui é rota de distribuição, pois
>essa mãe chamada Brasil mantém suas fronteiras abertas e aqui tem Estrada
>para as Guianas e Venezuela. Nenhuma bagagem de estrangeiro é fiscalizada,
>principalmente se for americano, europeu ou japonês, (isso pode causar um
>incidente diplomático)... Dizem que tem muito colombiano traficante virando
>venezuelano, pois na Venezuela é muito fácil comprar a cidadania
>venezuelana por cerca de 200 dólares.
>Pergunto inocentemente às pessoas; porque os americanos querem tanto
>proteger os índios. A resposta é absolutamente a mesma, porque as terras
>indígenas além das riquezas animais e vegetais, da abundância de água são
>extremamente ricas em ouro (encontram-se pepitas que chegam a ser pesadas
>em quilos), diamante, outras pedras preciosas, minério e nas reservas norte
>de Roraima e Amazonas, ricas em PETRÓLEO.
>
>Parece que as pessoas contam essas coisas como que num grito de Socorro a
>alguém que é do sul, como se eu pudesse dizer isso ao presidente ou a
>alguma autoridade do sul que vá fazer alguma coisa. É pessoal, saio daqui
>com a quase certeza de que em breve o Brasil irá diminuir de tamanho.
>Um grande abraço a todos. Será que podemos fazer alguma coisa???
>Acho que sim.
>
>Repasse esse e-mail para que um maior número de brasileiros fique sabendo
>desses absurdos.
>
>Mara Silvia Alexandre Costa
>Depto de Biologia Cel. Mol. Bioag. Patog.
>FMRP - USP
>
>Opinião pessoal:
>Gostaria que você, especialmente que recebeu este e-mail, o repasse para o
>maior número possível de pessoas. Do meu ponto de vista seria interessante
>que o p aís inteiro ficasse sabendo desta situação através dos telejornais
>antes que isso venha a acontecer.
>Afinal foi um momento de fraqueza dos Estados Unidos que os europeus
>lançaram o Euro, assim poderá se aproveitar esta situação de fraqueza
>norte-americana (perdas na guerra do Iraque) para revelar isto ao mundo a
>fim de antecipar a próxima guerra.
>
>Celso Luiz Borges de Oliveira
>Doutorando em Água e Solo
>FEAGRI/UNICAMP
>Tel: (19) 3233-1840 Celular: (19) 9136-6472
>e-mail´s: celso@ufba.br; celso@agr.unicamp.br; celsoborges@gmail.com
>__________________________________________________
>Converse com seus amigos em tempo real com o Yahoo! Messenger
>http://br.download.yahoo.com/messenger/
>
>
>[As partes desta mensagem que não continham texto foram removidas]
>




SUBJECT: Re: Sobre Aspartame
FROM: "rmtakata" <rmtakata@altavista.net>
TO: ciencialist@yahoogrupos.com.br
DATE: 01/02/2005 08:51


--- Em ciencialist@yahoogrupos.com.br, "murilo filo"
> A questão que vejo, por minha *própria experiência*, são os efeitos
> alérgicos da coisa.

Bem, em principio quase qq coisa pode dar alergia a uma determinada
faixa de pessoas. Em relacao a isso, o q. se pode fazer eh alertar
para q. as pessoas sensiveis evitem o produto.

> E sobre os fenilcetonúricos... qual é o lance?

Eles nao metabolizam normalmente o aminoacido fenilalanina (Phe).
Aquele teste do pezinho, entre outras coisas, serve para detetar quem
apresenta esse problema. Essas pessoas devem ter uma dieta rigidamente
controlada. Por lei, os produtos alimenticios q. normalmente nao
possuem o aminoacido devem apresentar um alerta qdo eh acrescido de um
componente q. tenha a Phe.

http://www.fenilcetonuria.com.br/fenilcetonuria.html
http://www.anvisa.gov.br/faqdinamica/asp/usuario.asp?usersecoes=28&userassunto=42

[]s,

Roberto Takata





SUBJECT: Re: [ciencialist] Sobre Aspartame
FROM: pablo@unidavi.edu.br
TO: ciencialist@yahoogrupos.com.br
DATE: 01/02/2005 09:51

Hoax antigo!!! Pura besteira.


Citando murilo filo <avalanchedrive@hotmail.com>:

>
> Isto é o 'crime da mala' e já deve ter circulado por aquí.
> Será que o resultado químico é êste mesmo?
> Eu nunca teria esperado isto da monsanto... M.
>
> >
> >>>Subject: FW: Sobre Aspartame
> >>>Date: Thu, 27 Jan 2005 17:37:56 -0200
> >>>
> >>> > Subject: Aspartame - Conheça a realidade
> >>> >
> >>> >
> >>> > ASPARTAME
> >>> >
> >>> > CONFERÊNCIA MUNDIAL DO MEIO AMBIENTE E FUNDAÇÃO ESCLEROSE MÚLTIPLA EM
> >>>ROTA DE COLISÃO COM A MONSANTO
> >>> >
> >>> > (Artigo escrito pela Dra. Mancy Marckle)
> >>> >
> >>> >
> >>> > Passei alguns dias falando na CONFERÊNCIA MUNDIAL DE MEIO AMBIENTE a
> >>>respeito do ASPARTAME, conhecido como Nutrasweet, Equal e Spoonful. Eles
> >>>anunciaram que existia uma epidemia de esclerose múltipla e lúpus
> >>>sistêmico e não entendiam que toxina estava fazendo com que essas doenças
>
> >>>assolassem os Estados Unidos tão rapidamente. Eu expliquei que eu estava
> >>>lá para falar exatamente sobre este assunto.
> >>> >
> >>> > Quando a temperatura do ASPARTAME excede 30º C, o álcool contido no
> >>>ASPARTAME se converte em formaldeído e daí para ácido fórmico, que
> >>>provoca acidose metabólica (o ácido fórmico é o veneno das formigas).
> >>> >
> >>> > A toxidade do metanol imita a esclerose múltipla e as pessoas recebem
> >>>diagnóstico errado de esclerose múltipla. A esclerose múltipla não se
> >>>constitui em sentença de morte, mas a toxidade do metanol sim. No caso do
>
> >>>lúpus sistêmico, estamos percebendo que é quase tão grave quanto a
> >>>esclerose múltipla especialmente em usuários de DIET COKE e DIET PEPSI.
> >>> >
> >>> > Não é para menos, considerada a toxidade do metanol ... (As vítimas
> >>>geralmente bebem de 3 a 4 latas desses refrigerantes por dia, ou mais).
> >>>[Sou eu!!! Se alguém souber alguma coisa que comprove a veracidade desta
> >>>mensagem, avise-me. Quanto ao meu depoimento, posso dizer que não
> >>>apresento os sintomas relatados.] Nos casos de lúpus sistêmico causado
> >>>pelo ASPARTAME, a vítima geralmente não sabe que o ASPARTAME é causa de
> >>>sua doença e continua com seu uso, agravando o lúpus a um grau tão
> >>>intenso que algumas vezes ameaça a vida. Quando interrompemos o uso do
> >>>ASPARTAME, as pessoas que tinham lúpus ficam assintomáticas.
> >>> >
> >>> > Infelizmente, não podemos reverter essa doença. Por outro lado, nos
> >>>casos diagnosticados como esclerose múltipla (quando, na realidade, a
> >>>doença é devida à toxidade do metanol) a maioria dos sintomas
> >>>desaparece. Nós temos visto casos em que a visão retornou e mesmo a
> >>>audição for recuperada. Isso também se aplica aos casos de tinittus
> >>>auricularis (zumbido no ouvido).
> >>> >
> >>> > Em uma conferência eu disse: - Se você está usando ASPARTAME
> >>>(Nutrasweet, Equal e Spoonful, etc.) e sofre de sintomas como
> >>>fibromialgia, espasmos, dores, formigamentos nas pernas, câimbras,
> >>>vertigem, tontura, dor de cabeça, zumbido no ouvido, dores articulares,
> >>>depressão, ataques de ansiedade, fala atrapalhada, visão borrada ou perda
>
> >>>de memória - VOCÊ PROVAVELMENTE TEM A DOENÇA DO ASPARTAME! As pessoas
> >>>começaram a pular durante a palestra dizendo: - Eu tenho isto, é
> >>>reversível?
> >>> >
> >>> > É impressionante. Em uma palestra assistida pelo embaixador de Uganda,
> >>>ele nos contou que a indústria de açúcar deles está adicionando ASPARTAME
>
> >>>ao açúcar! Ele contou que o filho de um dos líderes da indústria não
> >>>conseguia andar - em parte pelo uso do produto! Estamos com um sério
> >>>problema.
> >>> >
> >>> > Um estranho veio até o Dr. Espisto (um de meus palestrantes) e >
> >>>perguntou
> >>>por que tantas pessoas estavam tendo esclerose múltipla (MS). Durante a
> >>>visita a um hospital, uma enfermeira disse que seis amigos dela que eram
>
> >>>viciados em diet coke, tinham sido diagnosticados com MS. Isso é mais do
> >>>que coincidência.
> >>> >
> >>> > Há um tempo atrás houve audiências no congresso dos EUA incluindo o
> >>>ASPARTAME em 100 produtos diferentes. Nada foi feito. Os lobbies da droga
>
> >>>e da indústria química tem bolsos muito profundos. Agora existem mais de
>
> >>>5.000 produtos contaminados com esse produto químico e a patente
> >>>expirou. Na época da primeira audiência, as pessoas estavam ficando
> >>>cegas. O metanol no ASPARTAME se converte em formaldeído na retina do
> >>>olho. Formaldeído é do mesmo grupo das drogas como cianeto e arsênico:
> >>>venenos mortais! Infelizmente, leva muito tempo para matar, mas está
> >>>matando as pessoas e causando todos os tipos de problemas> neurológicos.
> >>> >
> >>> > O ASPARTAME muda a química do cérebro. É a causa de diversos tipos de
> >>>ataque. Essa droga muda os níveis de dopamina no cérebro. Imagine o que
> >>>acontece com os pacientes que sofrem de Doença de Parkinson? Também
> >>>causa mal formações fetais.
> >>> >
> >>> > Não existe nenhuma razão para se utilizar esse produto. NÃO É UM
> >>>PRODUTO DIETÉTICO! Os anais do Congresso dizem: ele faz você desejar
> >>>carboidratos e, em conseqüência, engordar. Dr. Roberts viu que quando
> >>>interrompeu o uso de ASPARTAME a perda de peso foi de 9,5 Kg por pessoa.
> >>> >
> >>> > O formaldeído se armazena nas cédulas adiposas, principalmente nos
> >>>quadris e coxas. O ASPARTAME é especialmente mortal para os diabéticos.
> >>>Todos os médicos sabem o que o metanol causaria num diabético. Muitos
> >>>médicos acreditam que seus pacientes têm retinopatia, quando de fato, o
> >>>mal é causado pelo ASPARTAME.
> >>> >
> >>> > O ASPARTAME mantém o açúcar sangüíneo fora de controle, fazendo com
> >>>que
> >>>muitos pacientes entrem em coma. Infelizmente, muitos morreram. Pessoas
> >>>nos contaram na Conferência do Colégio Americano de Medicina que tinham
> >>>parentes que mudaram de sacarina para o ASPARTAME e agora eventualmente
> >>>entram em coma. Seus médicos não conseguem controlar os níveis de
> >>>glicemia.
> >>> >
> >>> > Os pacientes têm perda de memória pelo fato de que o ácido aspártico
> >>>e
> >>>a fenilalanina são neurotóxicos sem os outros aminoácidos encontrados
> >>>nas proteínas. Eles atravessam a barreira hemato-encefálica e causam
> >>>deterioração nos neurônios. Dr. Russel Blaylock, neurocirurgião, diz: Os
> >>>ingredientes estimulam os neurônios até a morte causando dano cerebral
> >>>em vários níveis. Dr. Blaylock escreveu um livro intitulado:
> >>>"Excitotoxinas: O Gosto que Mata". (Health Press, 1-800-643-2665).
> >>> >
> >>> > O Dr. H. J. Roberts, especialista diabético e perito mundial em
> >>>envenenamento pelo ASPARTAME, escreveu um livro intitulado: "DEFESA
> >>>CONTRA A DOENÇA DE ALZHEIMER" (1-800-814-8900). Dr. Roberts mostra como o
>
> >>>envenenamento pelo ASPARTAME está relacionado à doença de Alzheimer. E
> >>>realmente está. Mulheres de 30 anos estão sendo internadas com Alzheimer.
>
> >>>Dr.Blaylock e Dr. Roberts estão escrevendo uma carta-posição com alguns
> >>>casos relatados e vão colocá-la na Internet. De acordo com a conferência
>
> >>>do Colégio Americano de Medicina, nós estamos falando de uma praga de
> >>>doenças neurológicas causada por esse veneno mortal. Dr.Roberts
> >>>descobriu o que aconteceu quando o ASPARTAME foi vendido pela primeira
> >>>vez. Ele disse que seus pacientes diabéticos apresentaram perda de
> >>>memória, confusão e severa perda de visão.
> >>> >
> >>> > Na Conferência do Colégio Americano de Medicina, os médicos admitiram
> >>>que não sabiam. Eles estavam procurando saber por que ataques tinham
> >>>aumentado tanto (a fenilalanina do ASPARTAME diminui o limiar para a
> >>>convulsão e depleta a serotonina, o que causa psicose maníaco
> >>>depressiva, ataque de pânico, fúria e violência).
> >>> >
> >>> > Antes da Conferência, eu recebi um fax da Noruega, pedindo um possível
> >>>antídoto para esse veneno, porque a pessoa estava sentindo esses tantos
> >>>problemas no seu País. Esse veneno, agora, está disponível em muitos
> >>>países.
> >>> >
> >>> > Felizmente, tivemos embaixadores e palestrantes na Conferência que se
> >>>engajaram nesta luta. PEDIMOS QUE VOCÊ SE ENGAJE TAMBÉM.
> >>> >
> >>> > Imprima este artigo e avise todas as pessoas que você conhece.
> >>> >
> >>> > Tire tudo o que contém aspartame do armário.
> >>> >
> >>> > Envie para nós sua história.
> >>> >
> >>> >
> >>> > Eu asseguro que a Monsanto, criadora do ASPARTAME, sabe como ele é
> >>>mortal. Ela financia a Associação Americana de Diabetes, a Associação
> >>>Americana de Dietética, o Congresso e a Conferência do Colégio Americano
>
> >>>de Medicina. O New York Times, em 15 de Novembro de 1996, publicou
> >>>artigo a respeito de como a Associação Americana de Dietética recebe
> >>>dinheiro da indústria alimentícia para endossar seus produtos. Por isso,
> >>>eles não podem criticar ou falar a respeito de sua ligação com a
> >>>MONSANTO.>
> >>> >
> >>> > A que ponto chega isso? Às raias da perversidade. Dissemos a uma mãe
> >>>cujo filho estava usando NutraSweet que ela deveria interromper o uso do
> >>>produto. A criança estava tendo convulsões diárias. A mãe telefonou para
> >>>o médico, que telefonou para a Associação. E a Associação disse ao
> >>>médico para não interromper o uso de NutraSweet. Estamos ainda tentando
> >>>convencer a mãe que o ASPARTAME está causando as convulsões. Toda vez que
>
> >>>interrompemos o uso do ASPARTAME, as convulsões cessam. Se o bebê morrer,
>
> >>>sabemos de quem é a culpa e contra quem lutamos.
> >>> >
> >>> > Existem 92 sintomas documentados de ASPARTAME, do coma à morte. A
> >>>maioria deles é neurológica, porque o ASPARTAME destrói o sistema
> >>>nervoso.
> >>> >
> >>> > A doença do ASPARTAME é parcialmente a causa da síndrome "Tempestade
> >>> > No
> >>>Deserto". A queimação na língua e os outros sintomas discutidos em mais
> >>>de 60 casos podem estar diretamente relacionados ao consumo de produtos
> >>>contendo ASPARTAME. Milhares de latas de bebidas diet foram enviadas para
>
> >>>as tropas do "Tempestade no Deserto". ( Lembre- se que o calor pode
> >>>liberar o metanol do ASPARTAME a 30º C.) As bebidas dietéticas foram
> >>>expostas ao sol de 45ºC no deserto árabe, por semanas. Homens e mulheres
> >>>de serviço bebiam isso o dia todo. Todos os sintomas deles eram
> >>>semelhantes ao envenenamento por ASPARTAME.
> >>> >
> >>> > Dr. Roberts diz que o consumo do ASPARTAME na época da concepção pode
> >>>causar defeitos no feto. A FENILALANINA se concentra na placenta
> >>>causando retardo mental, de acordo com o Dr. Louis Elsas, Professor de
> >>>Genética Pediátrica na Universidade de Emory. Em testes de laboratório,
> >>>animais desenvolvem tumores cerebrais (afenilalanina tem um subproduto o
> >>>DXP, agente causador de tumores cerebrais).
> >>> >
> >>> > Quando o Dr. Espisto estava falando, um neurocirurgião da platéia >
> >>>disse:
> >>>- Encontra-se, de fato, um teor elevado de ASPARTAME nos tumores
> >>>cerebrais removidos.
> >>> >
> >>> > A STÉVIA, um adoçante natural, NÃO É UM ADITIVO, e ajuda no >
> >>>metabolismo
> >>>do açúcar: seria ideal para os diabéticos e foi aprovada, agora, como
> >>>suplemento dietético pelo FDA. Durante anos, o FDA adiou essa aprovação
> >>>por causa da sua lealdade à MONSANTO.
> >>> >
> >>> > (O texto acima foi corrigido por Nivaldo Alves Soares e Josealdo >
> >>>Tonholo
> >>>- Depto. Química/CCEN - Universidade Federal de Alagoas) - Campus A. C.
> >>>Simões - Tabuleiro do Martins 57.072-970 - Maceió - AL - Brasil -
> >>>Fone/fax: (082) 214-1389)
> >>> >
> >>> > Nota: Em 05/07/99 a UPI Farm News noticiou que a Mosanto está
> >>>colocando
> >>>à venda seu braço de ingredientes alimentares, entre as quais a
> >>>NutreSweet. Teriam essas vendas ligações com o artigo acima?
> >>> >
> >>> >
> >>> >
> >>> > MONSANTO VENDE NUTRASWEET - UPI Farm News - 05/07/99
> >>> >
> >>> > "A Monsanto está vendendo seus negócios no ramo de ingredientes
> >>>alimentares, inclusive o braço de adoçantes NutraSweet. A receita destas
> >>>vendas vai auxiliar a empresa química e gigante da pesquisa agrícola -
> >>>com sede em St. Louis - a saldar os débitos da compra desmesurada de
> >>>biotecnologia, entre as quais a de empresas produtoras de semente.
> >>> >
> >>> > A Monsanto necessitava das produtoras de sementes como meio para >
> >>>escoar
> >>>seu material genético "high-tech" para os agricultores. Mas a companhia
> >>>está vendendo empresas para pagar seus débitos resultantes dos
> >>>investimentos em biotecnologia desde outubro quando fracassaram os seus
> >>>planos para fusão com a American Home Products (com sede em New Jersey).
> >>> >
> >>> > Os negócios na área de ingredientes alimentares incluem a NutraSweet,
>
> >>> > o
> >>>adoçante artificial usado em refrigerantes, além de outros produtos à
> >>>base de goma. O NutraSweet também é vendido como adoçante de mesa sob a
> >>>marca de Equal."
> >>> >
> >>> > Sugerimos que cada pessoa que receba este artigo o divulgue para o >
> >>>maior
> >>>número possível de pessoas e entidades. Precisamos combater os autores de
>
> >>>mais esse crime contra a saúde humana, isto é, a Monsanto e as várias
> >>>entidades mencionadas neste artigo.
> >>> >
> >>> >
> >>> >
> >>> >
> >>>------------------------------------------------------------------------
> >>> >
> >>> >
> >>> > Internal Virus Database is out-of-date.
> >>> > Checked by AVG Anti-Virus.
> >>> > Version: 7.0.289 / Virus Database: 265.5.4 - Release Date: 15/12/04
> >>> >
> >>> >
> >>> >
> >>> > No virus found in this outgoing message.
> >>> > Checked by AVG Anti-Virus.
> >>> > Version: 7.0.289 / Virus Database: 265.6.13 - Release Date: 16/01/05
> >>> >
> >>> > | | | | | Caixa de Entrada
> >>> > É melhor com a borboleta.
> >>> > Início | Meu MSN | Hotmail | Busca | Compras |
> >>>Grupos & bate-papo | Messenger Grátis Comentários | Ajuda
> >>> > © 2004 Microsoft Corporation. Todos os direitos reservados. TERMOS DE
> >>>USO Declaração de Privacidade POLÍTICA ANTI-SPAM DA MICROSOFT
> >>> >
> >>> >
> >>
> >>_________________________________________________________________
> >>MSN Messenger: converse online com seus amigos .
> >>http://messenger.msn.com.br
> >>
> >
>
>
>
>
> ##### ##### #####
>
> Para saber mais visite
> http://www.ciencialist.hpg.ig.com.br
>
>
> ##### ##### ##### #####
> Links do Yahoo! Grupos
>
>
>
>
>
>
>
>




-------------------------------------------------
This mail sent through IMP: http://horde.org/imp/



SUBJECT: C14
FROM: "harapadalsha2000" <sesa@br.inter.net>
TO: ciencialist@yahoogrupos.com.br
DATE: 01/02/2005 10:12


Olá pessoal

Gostaria de uma ajuda

Como funciona e como é a lógica de datação pelo Carbono14?


Grato


Sérgio





SUBJECT: Re: [ciencialist] C14
FROM: "E m i l i a n o C h e m e l l o" <chemelloe@yahoo.com.br>
TO: <ciencialist@yahoogrupos.com.br>
DATE: 01/02/2005 10:24

Olá Sérgio,

Uma rápida pesquisa no Google com a expressão "Carbono 14" reportou os seguintes resultados interessantes:

http://noticias.terra.com.br/ciencia/interna/0,,OI109680-EI1426,00.html
http://www.ucs.br/ccet/defq/naeq/material_didatico/textos_interativos_29.htm
http://www.fisica.ufc.br/donafifi/datacao/datacao3.htm

Qualquer dúvida, mande para a lista.

---
[ ] 's do Emiliano Chemello
emiliano@quimica.net
http://www.quimica.net/emiliano
http://www.ucs.br/ccet/defq/naeq

" Rien ne se perd, rien ne se crée,
tout se transforme."

Antoine Laurent de Lavoisier (químico francês, 1743 - 1794)

----- Original Message -----
From: harapadalsha2000
To: ciencialist@yahoogrupos.com.br
Sent: Tuesday, February 01, 2005 10:12 AM
Subject: [ciencialist] C14



Olá pessoal

Gostaria de uma ajuda

Como funciona e como é a lógica de datação pelo Carbono14?


Grato


Sérgio





##### ##### #####

Para saber mais visite
http://www.ciencialist.hpg.ig.com.br


##### ##### ##### #####


Yahoo! Grupos, um serviço oferecido por:







------------------------------------------------------------------------------
Links do Yahoo! Grupos

a.. Para visitar o site do seu grupo na web, acesse:
http://br.groups.yahoo.com/group/ciencialist/

b.. Para sair deste grupo, envie um e-mail para:
ciencialist-unsubscribe@yahoogrupos.com.br

c.. O uso que você faz do Yahoo! Grupos está sujeito aos Termos do Serviço do Yahoo!.



[As partes desta mensagem que não continham texto foram removidas]



SUBJECT: Biblioteca de Alexandria
FROM: "Esteban Moreno" <estebanmoreno@idhi.org.br>
TO: <ciencialist@yahoogrupos.com.br>
DATE: 01/02/2005 11:43


http://www.fisica.uel.br/c&t/2002202.html
Biblioteca de Alexandria

Carlos Roberto Apploni

A versão moderna da Biblioteca de Alexandria, embora já esteja aberta ao público desde 23 de abril deste ano, dia Internacional do Livro, foi inaugurada oficialmente dia 16 de outubro, com as presenças dos Presidentes do Egito e da França, entre outros Chefes de Estado. Localizada no mesmo lugar da antiga biblioteca, tem treze andares, foi projetada para receber até 7 milhões de documentos e abriu já com 400 mil obras, uma rica coleção de manuscritos antigos e um arquivo de 10 bilhões de páginas da Internet, compiladas desde 1996. Nela também funciona uma escola nacional de ciências da informação, um centro de conferências, um planetário e um museu. A biblioteca é o resultado de um projeto desenvolvido durante vinte anos com o apoio da UNESCO. Custou US$ 100 milhões de dólares ao Egito, além de doações do Iraque, da Arábia Saudita, dos Emirados Árabes e da UNESCO, chegando a um total de US$ 200 milhões de dólares. Longe do ideal de abarcar todo o tipo de conhecimento, sem quaisquer tipo de restrição, característica da concepção da antiga biblioteca fundada pelo faraó grego Ptolomeu I (que foi general de Alexandre o Grande) e da cultura helenística em que estava baseada, a nova biblioteca privilegia os estudos do mundo árabe e mediterrâneo. No entanto, é necessário ressaltar que o Presidente do Egito, Hosni Mubarak, está projetando o mundo árabe no cenário internacional através de sua espiritualidade, de sua civilização e não através da violência e da guerra, um feito marcante dentro do contexto dos dias de hoje. Outro fato inédito é que o Parlamento egípcio decidiu, já no ano passado, que a biblioteca terá administração independente do governo.

Mitos & História

A confusão sobre o destino final da antiga biblioteca é tal que dois importantes jornais de circulação nacional, ao anunciarem a inauguração da nova biblioteca, divulgaram duas versões diferentes, e erradas, sobre o assunto. Um escreveu que a biblioteca foi destruída por uma série de incêndios no século II. Outro veiculou que foi queimada no século IV. Assim, aproveito para chamar a atenção dos leitores para o ótimo livro de Luciano Canfora, "A Biblioteca Desaparecida - histórias da biblioteca de Alexandria" (195 páginas, Companhia das Letras, 1996). Neste livro o famoso filólogo e historiador italiano disseca toda a história da antiga biblioteca, desde a sua fundação no terceiro século antes de Cristo, acabando com mitos como o de sua destruição (total ou parcial) pelo fogo quando da tomada de Alexandria pelo imperador romano Júlio César em 48 A.C., ou de que no ano de 640 o general árabe Amr ibn al-As, quando de outra tomada da cidade, entrou gritando fanaticamente "onde estão os livros?" Não foi bem assim, mas, pouco tempo depois da tomada de Alexandria e por ordem deste general árabe (vide em especial o triste capítulo XVI do referido livro), realmente os cerca de 700 mil manuscritos da biblioteca foram distribuídos para aquecer os da ordem de 4.000 banhos públicos de Alexandria, sendo necessário uns seis meses para queimar todo o material. Foram poupados apenas os livros de Aristóteles, cujas traduções para o árabe deram enorme impulso à Ciência Islâmica e, através desta, ao Ocidente, na parte da Europa ocupada pelos árabes durante a Idade Média, com fundamentais repercussões para o Renascimento. Já pensou o que teria ocorrido se aquela fantástica biblioteca não tivesse sido destruída?

Frases

A longa história da antiga biblioteca de Alexandria e sua destruição, assim como a maneira como a moderna biblioteca aparece, tem como pano de fundo a questão da intolerância. Os períodos em que a intolerância inexistiu ou em que houve uma certa flexibilização, assim como aqueles em que a intolerância dominou. Intolerância do coração e / ou da mente. Não deixa de ser sintomático as entrelinhas de um trecho da mensagem do Prêmio Nobel da Literatura de 1988, o egípcio Naguib Mahfouz, lida durante a inauguração da biblioteca: "a nova biblioteca representa uma nobre mensagem de paz, que é mais importante do que a da antiga biblioteca". Discordo. A antiga biblioteca representa a paz mais do que qualquer obra do homem moderno, dado o enorme arco de tempo em que a antiga cidade de Alexandria foi um pólo político, econômico, cultural e científico. Além disso a antiga biblioteca tinha como base o princípio de respeito a todas as crenças e valorização do conhecimento de todas as culturas, que foi a marca do imenso império formado por Alexandre o Grande e que permaneceu depois de sua morte, em especial, no Egito governado pela dinastia Ptolomáica. O mundo atual está longe, embora muito necessitado, de resgatar importantes saberes e valores da antiguidade. Uma outra frase vem calhar: "Abolimos as superstições nascidas do coração para adotarmos as superstições nascidas da mente" (Laurens Van der Post, escritor sul-africano, 1906-1996).


[As partes desta mensagem que não continham texto foram removidas]



SUBJECT: Re: [ciencialist] C14
FROM: Luis Brudna <luisbrudna@gmail.com>
TO: ciencialist@yahoogrupos.com.br
DATE: 01/02/2005 11:53

No link do Terra que o Emiliano passou o texto fala em Fosseis e
Paleontologos.

Nao sei qual a exata definicao de ´fosseis´, mas acho que nao se
utiliza metodo de C14 para isso. Pq fosseis tem troca de material
organico por rocha. Ou estou enganado?

Alem disso acho que o melhor seria arqueologo no lugar de paleontologo.

Ateh
Luis Brudna

On Tue, 1 Feb 2005 10:24:31 -0200, E m i l i a n o C h e m e l l o
<chemelloe@yahoo.com.br> wrote:
>
> Olá Sérgio,
>
> Uma rápida pesquisa no Google com a expressão "Carbono 14" reportou os seguintes resultados interessantes:
>
> http://noticias.terra.com.br/ciencia/interna/0,,OI109680-EI1426,00.html
> http://www.ucs.br/ccet/defq/naeq/material_didatico/textos_interativos_29.htm
> http://www.fisica.ufc.br/donafifi/datacao/datacao3.htm
>
> Qualquer dúvida, mande para a lista.
>
> ---
> [ ] 's do Emiliano Chemello
> emiliano@quimica.net
> http://www.quimica.net/emiliano
> http://www.ucs.br/ccet/defq/naeq


SUBJECT: Re: [ciencialist] C14
FROM: "E m i l i a n o C h e m e l l o" <chemelloe@yahoo.com.br>
TO: <ciencialist@yahoogrupos.com.br>
DATE: 01/02/2005 12:21

Olá Brudna,


O que é um fóssil
Fósseis (do latim fossilis) são os restos materiais de antigos organismos ou
as manifestações da sua actividade, que ficaram mais ou menos bem
conservados nas rochas ou em outros fósseis
fonte: http://fossil.uc.pt/pags/fossil.dwt

Paleontologia:
ciência que trata dos fósseis (animais e vegetais).

Arqueologia
ciência que tem por objecto o estudo dos vestígios das antigas civilizações;
ciência da antiguidade.


[ ] 's do Emiliano Chemello
emiliano@quimica.net
http://www.quimica.net/emiliano
http://www.ucs.br/ccet/defq/naeq

" Rien ne se perd, rien ne se crée,
tout se transforme."

Antoine Laurent de Lavoisier (químico francês, 1743 - 1794)

----- Original Message -----
From: Luis Brudna
To: ciencialist@yahoogrupos.com.br
Sent: Tuesday, February 01, 2005 11:53 AM
Subject: Re: [ciencialist] C14


No link do Terra que o Emiliano passou o texto fala em Fosseis e
Paleontologos.

Nao sei qual a exata definicao de ´fosseis´, mas acho que nao se
utiliza metodo de C14 para isso. Pq fosseis tem troca de material
organico por rocha. Ou estou enganado?

Alem disso acho que o melhor seria arqueologo no lugar de paleontologo.

Ateh
Luis Brudna

On Tue, 1 Feb 2005 10:24:31 -0200, E m i l i a n o C h e m e l l o
<chemelloe@yahoo.com.br> wrote:
>
> Olá Sérgio,
>
> Uma rápida pesquisa no Google com a expressão "Carbono 14" reportou os
seguintes resultados interessantes:
>
> http://noticias.terra.com.br/ciencia/interna/0,,OI109680-EI1426,00.html
>
http://www.ucs.br/ccet/defq/naeq/material_didatico/textos_interativos_29.htm
> http://www.fisica.ufc.br/donafifi/datacao/datacao3.htm
>
> Qualquer dúvida, mande para a lista.
>
> ---
> [ ] 's do Emiliano Chemello
> emiliano@quimica.net
> http://www.quimica.net/emiliano
> http://www.ucs.br/ccet/defq/naeq


##### ##### #####

Para saber mais visite
http://www.ciencialist.hpg.ig.com.br


##### ##### ##### #####


Yahoo! Grupos, um serviço oferecido por:
PUBLICIDADE





Links do Yahoo! Grupos

Para visitar o site do seu grupo na web, acesse:
http://br.groups.yahoo.com/group/ciencialist/

Para sair deste grupo, envie um e-mail para:
ciencialist-unsubscribe@yahoogrupos.com.br

O uso que você faz do Yahoo! Grupos está sujeito aos Termos do Serviço do
Yahoo!.




SUBJECT: O Manifesto de Porto Alegre
FROM: "Esteban Moreno" <estebanmoreno@idhi.org.br>
TO: <ciencialist@yahoogrupos.com.br>
DATE: 01/02/2005 14:55


Manifesto de Porto Alegre

Desde o primeiro Fórum Social Mundial realizado em Porto Alegre, em janeiro
de 2001, o fenômeno dos fóruns sociais se estendeu a todos os continentes,
inclusive nos níveis nacional e local. O Fórum favoreceu a emergência de um
espaço público planetário da cidadania e de suas lutas, assim como a
elaboração de propostas de políticas alternativas à tirania da globalização
neoliberal impulsionada pelos mercados financeiros e as transnacionais,
cujo braço armado é o poder imperial dos Estados Unidos.

Por sua diversidade, assim como pela solidariedade entre os atores e os
movimentos sociais que o compõe, o movimento altermundista se transformou
em uma força que já é levada muito em conta em todo o planeta. Entre as
inumeráveis propostas que têm saído dos fóruns, um grande número delas
conta sem dúvida com um amplo apoio junto aos movimentos sociais. Nós,
signatários do Manifesto de Porto Alegre, que nos exprimimos a título
estritamente pessoal, sem pretender, de modo algum, falar em nome do Fórum,
identificamos doze destas propostas que, em conjunto, dão sentido à
construção de outro mundo possível. Se fossem aplicadas, permitiriam que a
cidadania começasse por fim a reapropriar-se de seu futuro.

Submetemos estes pontos fundamentais à apreciação dos atores e movimentos
sociais de todos os países. São eles que, em todos os níveis - mundial,
continental, nacional e local - poderão levar adiante os combates
necessários para que se transformem em realidade. Nós não temos nenhuma
ilusão sobre a real vontade dos governos e das instituições internacionais
em aplicar espontaneamente estas propostas.



A) Outro Mundo Possível deve respeitar o direito à vida de todos os seres
humanos, mediante novas regras econômicas. Para tanto, é necessário:


1) Anular a dívida pública dos países do Sul, que já foi paga várias vezes
e que constitui, para os Estados credores, os estabelecimentos financeiros
e as instituições financeiras internacionais, a melhor maneira de submeter
a maior parte da humanidade à sua tutela e mantê-la na miséria;

2) Aplicar taxas internacionais às transações financeiras (especialmente a
Taxa Tobin às transações especulativas de divisas); aos investimentos
diretos no estrangeiro, aos lucros consolidados das transnacionais, à venda
de armas e às atividades que emitem de forma substantiva gases que produzem
o efeito estufa;

3) Desmantelar progressivamente todas as formas de paraísos fiscais,
jurídicos e bancários, que nada mais são do que refúgios do crime
organizado, da corrupção e de todos os tipos de tráficos, fraudes e evasões
fiscais, operações delituosas de grandes empresas e inclusive de governos;

4) Cada habitante do planeta deve ter direito a um emprego, à proteção
social e à aposentadoria, respeitando a igualdade entre homens e mulheres,
sendo este um imperativo de políticas públicas nacionais e internacionais;

5) Promover todas as formas de comércio justo, rechaçando as regras de
livre comércio da Organização Mundial do Comércio (OMC) e colocando em
execução mecanismos que permitam, nos processos de produção de bens e
serviços, dirigir-se progressivamente a um nivelamento por alto das normas
sociais (tal como estão consignadas nas convenções da Organização
Internacional do Trabalho, OIT) e ambientais. Excluir totalmente a
educação, a saúde, os serviços sociais e a cultura do terreno de aplicação
do Acordo Geral Sobre o Comércio e os Serviços (AGCS) da OMC. A convenção
sobre a diversidade cultural, que atualmente está sendo negociada na
Unesco, deve fazer prevalecer explicitamente o direito à cultura sobre o
direito ao comércio;

6) Garantir o direito à soberania e segurança alimentar de cada país,
mediante a promoção da agricultura campesina. Isso pressupõe a eliminação
total dos subsídios à exportação dos produtos agrícolas, em primeiro lugar
por parte dos Estados Unidos e da União Européia. Da mesma maneira, cada
país ou conjunto de países deve poder decidir soberanamente sobre a
proibição da produção e importação de organismos geneticamente modificados
destinados à alimentação;

7) Proibir todo tipo de patenteamento do conhecimento e dos seres vivos
(tanto humanos, como animais e vegetais) do mesmo modo que toda a
privatização de bens comuns da humanidade, em particular a água;



B) Outro Mundo Possível deve encorajar a vida em comum em paz e com
justiça, para toda a humanidade. Para tanto, é necessário:


8) Lutar, em primeiro lugar, por diferentes políticas públicas contra todas
as formas de discriminação (sexismo, xenofobia, anti-semitismo e racismo).
Reconhecer plenamente os direitos políticos, culturais e ambientais
(incluindo o domínio de recursos naturais), dos povos indígenas;

9) Tomar medidas urgentes para colocar um fim à destruição do meio ambiente
e à ameaça de mudanças climáticas graves devido ao efeito estufa
resultante, em primeiro lugar, da proliferação do transporte individual e
do uso excessivo de energias não renováveis. Começar a implementar outro
modelo de desenvolvimento fundado na sobriedade energética e no controle
democrático dos recursos naturais, em particular a água potável, em uma
escala planetária;

10) Exigir o desmantelamento das bases militares estrangeiras e de suas
tropas em todos os países, salvo quando estejam sob mandato expresso da
Organização das Nações Unidas (ONU);



C) Outro Mundo Possível deve promover a democracia desde o plano local até
o global. Para tanto, é necessário:


11) Garantir o direito à informação e o direito de informar dos cidadãos
mediante legislações que: a) ponham fim à concentração de veículos em
grupos de comunicação gigantes; b) garantam a autonomia dos jornalistas
diante dos acionistas; c) favoreçam a imprensa sem fins lucrativos, em
particular a dos meios alternativos e comunitários. O respeito destes
direitos implica contra-poderes cidadãos, em particular na forma de
observatórios nacionais e internacionais de meios de comunicação;

12) Reformar e democratizar em profundidade as organizações internacionais,
entre elas a ONU, fazendo prevalecer nelas os direitos humanos, econômicos,
sociais e culturais, em concordância com a Declaração Universal dos
Direitos Humanos. Isso implica a incorporação do Banco Mundial, do Fundo
Monetário Internacional e da Organização Mundial do Comércio ao sistema das
Nações Unidas. Caso persistam as violações do direito internacional por
parte dos Estados Unidos, transferir a sede da ONU de Nova York para outro
país, preferencialmente do Sul.



Porto Alegre, 29 de janeiro de 2005



Aminata Traoré, Adolfo Pérez Esquivel, Eduardo Galeano, José Saramago,
François Houtart, Boaventura de Sousa Santos, Armand Mattelart, Roberto
Savio, Riccardo Petrella, Ignacio Ramonet, Bernard Cassen, Samir Amin,
Atilio Boron, Samuel Ruiz Garcia, Tariq Ali, Frei Betto, Emir Sader, Walden
Bello, Immanuel Wallerstein





[As partes desta mensagem que não continham texto foram removidas]



SUBJECT: Re: [ciencialist] O Manifesto de Porto Alegre
FROM: "Alvaro Augusto \(E\)" <alvaro@electraenergy.com.br>
TO: <ciencialist@yahoogrupos.com.br>
DATE: 01/02/2005 14:59

Concordo com todos esses itens. Depois que eles forem implementados, sugiro também o seguinte:

a) Revogar a lei da gravitação universal, pois facilitaria enormemente o transporte de mercadorias e a construção civil;
b) Revogar a Lei de Joule, pois diminuiria drasticamente as perdas ôhmicas em linhas de transmissão;
c) Revogar a lei da oferta e da procura, pois qualquer dispositivo que permite o estabelecimento de preços é desumano;
d) Fazer com que a Jennifer Lopez venha morar aqui do meu lado (se não der tempo, tudo bem...)

[ ]s

Alvaro Augusto

----- Original Message -----
From: Esteban Moreno
To: ciencialist@yahoogrupos.com.br
Sent: Tuesday, February 01, 2005 2:55 PM
Subject: [ciencialist] O Manifesto de Porto Alegre



Manifesto de Porto Alegre

Desde o primeiro Fórum Social Mundial realizado em Porto Alegre, em janeiro
de 2001, o fenômeno dos fóruns sociais se estendeu a todos os continentes,
inclusive nos níveis nacional e local. O Fórum favoreceu a emergência de um
espaço público planetário da cidadania e de suas lutas, assim como a
elaboração de propostas de políticas alternativas à tirania da globalização
neoliberal impulsionada pelos mercados financeiros e as transnacionais,
cujo braço armado é o poder imperial dos Estados Unidos.

Por sua diversidade, assim como pela solidariedade entre os atores e os
movimentos sociais que o compõe, o movimento altermundista se transformou
em uma força que já é levada muito em conta em todo o planeta. Entre as
inumeráveis propostas que têm saído dos fóruns, um grande número delas
conta sem dúvida com um amplo apoio junto aos movimentos sociais. Nós,
signatários do Manifesto de Porto Alegre, que nos exprimimos a título
estritamente pessoal, sem pretender, de modo algum, falar em nome do Fórum,
identificamos doze destas propostas que, em conjunto, dão sentido à
construção de outro mundo possível. Se fossem aplicadas, permitiriam que a
cidadania começasse por fim a reapropriar-se de seu futuro.

Submetemos estes pontos fundamentais à apreciação dos atores e movimentos
sociais de todos os países. São eles que, em todos os níveis - mundial,
continental, nacional e local - poderão levar adiante os combates
necessários para que se transformem em realidade. Nós não temos nenhuma
ilusão sobre a real vontade dos governos e das instituições internacionais
em aplicar espontaneamente estas propostas.



A) Outro Mundo Possível deve respeitar o direito à vida de todos os seres
humanos, mediante novas regras econômicas. Para tanto, é necessário:


1) Anular a dívida pública dos países do Sul, que já foi paga várias vezes
e que constitui, para os Estados credores, os estabelecimentos financeiros
e as instituições financeiras internacionais, a melhor maneira de submeter
a maior parte da humanidade à sua tutela e mantê-la na miséria;

2) Aplicar taxas internacionais às transações financeiras (especialmente a
Taxa Tobin às transações especulativas de divisas); aos investimentos
diretos no estrangeiro, aos lucros consolidados das transnacionais, à venda
de armas e às atividades que emitem de forma substantiva gases que produzem
o efeito estufa;

3) Desmantelar progressivamente todas as formas de paraísos fiscais,
jurídicos e bancários, que nada mais são do que refúgios do crime
organizado, da corrupção e de todos os tipos de tráficos, fraudes e evasões
fiscais, operações delituosas de grandes empresas e inclusive de governos;

4) Cada habitante do planeta deve ter direito a um emprego, à proteção
social e à aposentadoria, respeitando a igualdade entre homens e mulheres,
sendo este um imperativo de políticas públicas nacionais e internacionais;

5) Promover todas as formas de comércio justo, rechaçando as regras de
livre comércio da Organização Mundial do Comércio (OMC) e colocando em
execução mecanismos que permitam, nos processos de produção de bens e
serviços, dirigir-se progressivamente a um nivelamento por alto das normas
sociais (tal como estão consignadas nas convenções da Organização
Internacional do Trabalho, OIT) e ambientais. Excluir totalmente a
educação, a saúde, os serviços sociais e a cultura do terreno de aplicação
do Acordo Geral Sobre o Comércio e os Serviços (AGCS) da OMC. A convenção
sobre a diversidade cultural, que atualmente está sendo negociada na
Unesco, deve fazer prevalecer explicitamente o direito à cultura sobre o
direito ao comércio;

6) Garantir o direito à soberania e segurança alimentar de cada país,
mediante a promoção da agricultura campesina. Isso pressupõe a eliminação
total dos subsídios à exportação dos produtos agrícolas, em primeiro lugar
por parte dos Estados Unidos e da União Européia. Da mesma maneira, cada
país ou conjunto de países deve poder decidir soberanamente sobre a
proibição da produção e importação de organismos geneticamente modificados
destinados à alimentação;

7) Proibir todo tipo de patenteamento do conhecimento e dos seres vivos
(tanto humanos, como animais e vegetais) do mesmo modo que toda a
privatização de bens comuns da humanidade, em particular a água;



B) Outro Mundo Possível deve encorajar a vida em comum em paz e com
justiça, para toda a humanidade. Para tanto, é necessário:


8) Lutar, em primeiro lugar, por diferentes políticas públicas contra todas
as formas de discriminação (sexismo, xenofobia, anti-semitismo e racismo).
Reconhecer plenamente os direitos políticos, culturais e ambientais
(incluindo o domínio de recursos naturais), dos povos indígenas;

9) Tomar medidas urgentes para colocar um fim à destruição do meio ambiente
e à ameaça de mudanças climáticas graves devido ao efeito estufa
resultante, em primeiro lugar, da proliferação do transporte individual e
do uso excessivo de energias não renováveis. Começar a implementar outro
modelo de desenvolvimento fundado na sobriedade energética e no controle
democrático dos recursos naturais, em particular a água potável, em uma
escala planetária;

10) Exigir o desmantelamento das bases militares estrangeiras e de suas
tropas em todos os países, salvo quando estejam sob mandato expresso da
Organização das Nações Unidas (ONU);



C) Outro Mundo Possível deve promover a democracia desde o plano local até
o global. Para tanto, é necessário:


11) Garantir o direito à informação e o direito de informar dos cidadãos
mediante legislações que: a) ponham fim à concentração de veículos em
grupos de comunicação gigantes; b) garantam a autonomia dos jornalistas
diante dos acionistas; c) favoreçam a imprensa sem fins lucrativos, em
particular a dos meios alternativos e comunitários. O respeito destes
direitos implica contra-poderes cidadãos, em particular na forma de
observatórios nacionais e internacionais de meios de comunicação;

12) Reformar e democratizar em profundidade as organizações internacionais,
entre elas a ONU, fazendo prevalecer nelas os direitos humanos, econômicos,
sociais e culturais, em concordância com a Declaração Universal dos
Direitos Humanos. Isso implica a incorporação do Banco Mundial, do Fundo
Monetário Internacional e da Organização Mundial do Comércio ao sistema das
Nações Unidas. Caso persistam as violações do direito internacional por
parte dos Estados Unidos, transferir a sede da ONU de Nova York para outro
país, preferencialmente do Sul.



Porto Alegre, 29 de janeiro de 2005



Aminata Traoré, Adolfo Pérez Esquivel, Eduardo Galeano, José Saramago,
François Houtart, Boaventura de Sousa Santos, Armand Mattelart, Roberto
Savio, Riccardo Petrella, Ignacio Ramonet, Bernard Cassen, Samir Amin,
Atilio Boron, Samuel Ruiz Garcia, Tariq Ali, Frei Betto, Emir Sader, Walden
Bello, Immanuel Wallerstein





[As partes desta mensagem que não continham texto foram removidas]



##### ##### #####

Para saber mais visite
http://www.ciencialist.hpg.ig.com.br


##### ##### ##### #####


Yahoo! Grupos, um serviço oferecido por:







------------------------------------------------------------------------------
Links do Yahoo! Grupos

a.. Para visitar o site do seu grupo na web, acesse:
http://br.groups.yahoo.com/group/ciencialist/

b.. Para sair deste grupo, envie um e-mail para:
ciencialist-unsubscribe@yahoogrupos.com.br

c.. O uso que você faz do Yahoo! Grupos está sujeito aos Termos do Serviço do Yahoo!.



[As partes desta mensagem que não continham texto foram removidas]



SUBJECT: Re: [ciencialist] C14
FROM: "Oraculo" <oraculo@atibaia.com.br>
TO: <ciencialist@yahoogrupos.com.br>
DATE: 01/02/2005 14:59

Olá Emiliano e Brudna

Mas o Brudna tem razão, o fóssil troca seu material organico por minerais e se torna um "molde em pedra" do osso original (ou uma imagem do animal completa, inclusive algumas partes moles e até penas). Essa caracteristica não é muito lembrada, principalmente por criacionistas, que tendem a alegar que o mundo deveria estar "cheio de fóssies intermediarios" se a evolução estivesse correta.

Isso é devido a erronea impressão que os ossos de todos os animais que morrem ficam preservados e enterrados. Mas, devido a raridade do mecanismo de fossilização (morrer em local propício, com imediata sobreposição de camadas de minerais - soterramento rápido, condições climáticas, pluviais, tipo de solo, condições de pressão, etc, etc, etc), milhões de ossos desaparecem para cada um que fossiliza.

Sobre fosseis, fossilização e processos relacionados:

http://geocities.yahoo.com.br/guiluck/processos.html
http://www.scsc.k12.ar.us/TuttS/fossil_formation.htm
http://www.ucmp.berkeley.edu/fosrec/Learning.html


E um link sober a datação de fósseis:

http://www.agiweb.org/news/evolution/datingfossilrecord.html

Um abraço.

Homero

http://www.agiweb.org/news/evolution/fossilrecord.html




----- Original Message -----
From: E m i l i a n o C h e m e l l o
To: ciencialist@yahoogrupos.com.br
Sent: Tuesday, February 01, 2005 12:21 PM
Subject: Re: [ciencialist] C14


Olá Brudna,


O que é um fóssil
Fósseis (do latim fossilis) são os restos materiais de antigos organismos ou
as manifestações da sua actividade, que ficaram mais ou menos bem
conservados nas rochas ou em outros fósseis
fonte: http://fossil.uc.pt/pags/fossil.dwt

Paleontologia:
ciência que trata dos fósseis (animais e vegetais).

Arqueologia
ciência que tem por objecto o estudo dos vestígios das antigas civilizações;
ciência da antiguidade.


[ ] 's do Emiliano Chemello
emiliano@quimica.net
http://www.quimica.net/emiliano
http://www.ucs.br/ccet/defq/naeq

" Rien ne se perd, rien ne se crée,
tout se transforme."

Antoine Laurent de Lavoisier (químico francês, 1743 - 1794)

----- Original Message -----
From: Luis Brudna
To: ciencialist@yahoogrupos.com.br
Sent: Tuesday, February 01, 2005 11:53 AM
Subject: Re: [ciencialist] C14


No link do Terra que o Emiliano passou o texto fala em Fosseis e
Paleontologos.

Nao sei qual a exata definicao de ´fosseis´, mas acho que nao se
utiliza metodo de C14 para isso. Pq fosseis tem troca de material
organico por rocha. Ou estou enganado?

Alem disso acho que o melhor seria arqueologo no lugar de paleontologo.

Ateh
Luis Brudna

On Tue, 1 Feb 2005 10:24:31 -0200, E m i l i a n o C h e m e l l o
<chemelloe@yahoo.com.br> wrote:
>
> Olá Sérgio,
>
> Uma rápida pesquisa no Google com a expressão "Carbono 14" reportou os
seguintes resultados interessantes:
>
> http://noticias.terra.com.br/ciencia/interna/0,,OI109680-EI1426,00.html
>
http://www.ucs.br/ccet/defq/naeq/material_didatico/textos_interativos_29.htm
> http://www.fisica.ufc.br/donafifi/datacao/datacao3.htm
>
> Qualquer dúvida, mande para a lista.
>
> ---
> [ ] 's do Emiliano Chemello
> emiliano@quimica.net
> http://www.quimica.net/emiliano
> http://www.ucs.br/ccet/defq/naeq


##### ##### #####

Para saber mais visite
http://www.ciencialist.hpg.ig.com.br


##### ##### ##### #####


Yahoo! Grupos, um serviço oferecido por:
PUBLICIDADE





Links do Yahoo! Grupos

Para visitar o site do seu grupo na web, acesse:
http://br.groups.yahoo.com/group/ciencialist/

Para sair deste grupo, envie um e-mail para:
ciencialist-unsubscribe@yahoogrupos.com.br

O uso que você faz do Yahoo! Grupos está sujeito aos Termos do Serviço do
Yahoo!.




##### ##### #####

Para saber mais visite
http://www.ciencialist.hpg.ig.com.br


##### ##### ##### #####


Yahoo! Grupos, um serviço oferecido por:
PUBLICIDADE




------------------------------------------------------------------------------
Links do Yahoo! Grupos

a.. Para visitar o site do seu grupo na web, acesse:
http://br.groups.yahoo.com/group/ciencialist/

b.. Para sair deste grupo, envie um e-mail para:
ciencialist-unsubscribe@yahoogrupos.com.br

c.. O uso que você faz do Yahoo! Grupos está sujeito aos Termos do Serviço do Yahoo!.



[As partes desta mensagem que não continham texto foram removidas]



SUBJECT: O Manifesto de Porto Alegre p/ Alvaro
FROM: "Oraculo" <oraculo@atibaia.com.br>
TO: <ciencialist@yahoogrupos.com.br>
DATE: 01/02/2005 15:19

Olá Alvaro

Acho pertinetes suas sugestões, mas acredito que tenha faltado o paragrafo primeiro do artigo d):

d) Fazer com que a Jennifer Lopez venha morar aqui do meu lado (se não der tempo, tudo bem...)

Parag. 1º - Que a Malu Mader se divorcie do Toni Beloto fique irremediavelmente apaixonada por mim (Homero)

Revogam-se as disposições em contrário.

Um abraço.

Homero

PS: Desculpas antecipadas, Brudna, pelo enorme Off Topic sem noção, mas escapou..:-) Efeitos do Forum de Porto Alegre..:-)
----- Original Message -----
From: Alvaro Augusto (E)
To: ciencialist@yahoogrupos.com.br
Sent: Tuesday, February 01, 2005 2:59 PM
Subject: Re: [ciencialist] O Manifesto de Porto Alegre


Concordo com todos esses itens. Depois que eles forem implementados, sugiro também o seguinte:

a) Revogar a lei da gravitação universal, pois facilitaria enormemente o transporte de mercadorias e a construção civil;
b) Revogar a Lei de Joule, pois diminuiria drasticamente as perdas ôhmicas em linhas de transmissão;
c) Revogar a lei da oferta e da procura, pois qualquer dispositivo que permite o estabelecimento de preços é desumano;
d) Fazer com que a Jennifer Lopez venha morar aqui do meu lado (se não der tempo, tudo bem...)

[ ]s

Alvaro Augusto

----- Original Message -----
From: Esteban Moreno
To: ciencialist@yahoogrupos.com.br
Sent: Tuesday, February 01, 2005 2:55 PM
Subject: [ciencialist] O Manifesto de Porto Alegre



Manifesto de Porto Alegre

Desde o primeiro Fórum Social Mundial realizado em Porto Alegre, em janeiro
de 2001, o fenômeno dos fóruns sociais se estendeu a todos os continentes,
inclusive nos níveis nacional e local. O Fórum favoreceu a emergência de um
espaço público planetário da cidadania e de suas lutas, assim como a
elaboração de propostas de políticas alternativas à tirania da globalização
neoliberal impulsionada pelos mercados financeiros e as transnacionais,
cujo braço armado é o poder imperial dos Estados Unidos.

Por sua diversidade, assim como pela solidariedade entre os atores e os
movimentos sociais que o compõe, o movimento altermundista se transformou
em uma força que já é levada muito em conta em todo o planeta. Entre as
inumeráveis propostas que têm saído dos fóruns, um grande número delas
conta sem dúvida com um amplo apoio junto aos movimentos sociais. Nós,
signatários do Manifesto de Porto Alegre, que nos exprimimos a título
estritamente pessoal, sem pretender, de modo algum, falar em nome do Fórum,
identificamos doze destas propostas que, em conjunto, dão sentido à
construção de outro mundo possível. Se fossem aplicadas, permitiriam que a
cidadania começasse por fim a reapropriar-se de seu futuro.

Submetemos estes pontos fundamentais à apreciação dos atores e movimentos
sociais de todos os países. São eles que, em todos os níveis - mundial,
continental, nacional e local - poderão levar adiante os combates
necessários para que se transformem em realidade. Nós não temos nenhuma
ilusão sobre a real vontade dos governos e das instituições internacionais
em aplicar espontaneamente estas propostas.



A) Outro Mundo Possível deve respeitar o direito à vida de todos os seres
humanos, mediante novas regras econômicas. Para tanto, é necessário:


1) Anular a dívida pública dos países do Sul, que já foi paga várias vezes
e que constitui, para os Estados credores, os estabelecimentos financeiros
e as instituições financeiras internacionais, a melhor maneira de submeter
a maior parte da humanidade à sua tutela e mantê-la na miséria;

2) Aplicar taxas internacionais às transações financeiras (especialmente a
Taxa Tobin às transações especulativas de divisas); aos investimentos
diretos no estrangeiro, aos lucros consolidados das transnacionais, à venda
de armas e às atividades que emitem de forma substantiva gases que produzem
o efeito estufa;

3) Desmantelar progressivamente todas as formas de paraísos fiscais,
jurídicos e bancários, que nada mais são do que refúgios do crime
organizado, da corrupção e de todos os tipos de tráficos, fraudes e evasões
fiscais, operações delituosas de grandes empresas e inclusive de governos;

4) Cada habitante do planeta deve ter direito a um emprego, à proteção
social e à aposentadoria, respeitando a igualdade entre homens e mulheres,
sendo este um imperativo de políticas públicas nacionais e internacionais;

5) Promover todas as formas de comércio justo, rechaçando as regras de
livre comércio da Organização Mundial do Comércio (OMC) e colocando em
execução mecanismos que permitam, nos processos de produção de bens e
serviços, dirigir-se progressivamente a um nivelamento por alto das normas
sociais (tal como estão consignadas nas convenções da Organização
Internacional do Trabalho, OIT) e ambientais. Excluir totalmente a
educação, a saúde, os serviços sociais e a cultura do terreno de aplicação
do Acordo Geral Sobre o Comércio e os Serviços (AGCS) da OMC. A convenção
sobre a diversidade cultural, que atualmente está sendo negociada na
Unesco, deve fazer prevalecer explicitamente o direito à cultura sobre o
direito ao comércio;

6) Garantir o direito à soberania e segurança alimentar de cada país,
mediante a promoção da agricultura campesina. Isso pressupõe a eliminação
total dos subsídios à exportação dos produtos agrícolas, em primeiro lugar
por parte dos Estados Unidos e da União Européia. Da mesma maneira, cada
país ou conjunto de países deve poder decidir soberanamente sobre a
proibição da produção e importação de organismos geneticamente modificados
destinados à alimentação;

7) Proibir todo tipo de patenteamento do conhecimento e dos seres vivos
(tanto humanos, como animais e vegetais) do mesmo modo que toda a
privatização de bens comuns da humanidade, em particular a água;



B) Outro Mundo Possível deve encorajar a vida em comum em paz e com
justiça, para toda a humanidade. Para tanto, é necessário:


8) Lutar, em primeiro lugar, por diferentes políticas públicas contra todas
as formas de discriminação (sexismo, xenofobia, anti-semitismo e racismo).
Reconhecer plenamente os direitos políticos, culturais e ambientais
(incluindo o domínio de recursos naturais), dos povos indígenas;

9) Tomar medidas urgentes para colocar um fim à destruição do meio ambiente
e à ameaça de mudanças climáticas graves devido ao efeito estufa
resultante, em primeiro lugar, da proliferação do transporte individual e
do uso excessivo de energias não renováveis. Começar a implementar outro
modelo de desenvolvimento fundado na sobriedade energética e no controle
democrático dos recursos naturais, em particular a água potável, em uma
escala planetária;

10) Exigir o desmantelamento das bases militares estrangeiras e de suas
tropas em todos os países, salvo quando estejam sob mandato expresso da
Organização das Nações Unidas (ONU);



C) Outro Mundo Possível deve promover a democracia desde o plano local até
o global. Para tanto, é necessário:


11) Garantir o direito à informação e o direito de informar dos cidadãos
mediante legislações que: a) ponham fim à concentração de veículos em
grupos de comunicação gigantes; b) garantam a autonomia dos jornalistas
diante dos acionistas; c) favoreçam a imprensa sem fins lucrativos, em
particular a dos meios alternativos e comunitários. O respeito destes
direitos implica contra-poderes cidadãos, em particular na forma de
observatórios nacionais e internacionais de meios de comunicação;

12) Reformar e democratizar em profundidade as organizações internacionais,
entre elas a ONU, fazendo prevalecer nelas os direitos humanos, econômicos,
sociais e culturais, em concordância com a Declaração Universal dos
Direitos Humanos. Isso implica a incorporação do Banco Mundial, do Fundo
Monetário Internacional e da Organização Mundial do Comércio ao sistema das
Nações Unidas. Caso persistam as violações do direito internacional por
parte dos Estados Unidos, transferir a sede da ONU de Nova York para outro
país, preferencialmente do Sul.



Porto Alegre, 29 de janeiro de 2005



Aminata Traoré, Adolfo Pérez Esquivel, Eduardo Galeano, José Saramago,
François Houtart, Boaventura de Sousa Santos, Armand Mattelart, Roberto
Savio, Riccardo Petrella, Ignacio Ramonet, Bernard Cassen, Samir Amin,
Atilio Boron, Samuel Ruiz Garcia, Tariq Ali, Frei Betto, Emir Sader, Walden
Bello, Immanuel Wallerstein





[As partes desta mensagem que não continham texto foram removidas]



##### ##### #####

Para saber mais visite
http://www.ciencialist.hpg.ig.com.br


##### ##### ##### #####


Yahoo! Grupos, um serviço oferecido por:







------------------------------------------------------------------------------
Links do Yahoo! Grupos

a.. Para visitar o site do seu grupo na web, acesse:
http://br.groups.yahoo.com/group/ciencialist/

b.. Para sair deste grupo, envie um e-mail para:
ciencialist-unsubscribe@yahoogrupos.com.br

c.. O uso que você faz do Yahoo! Grupos está sujeito aos Termos do Serviço do Yahoo!.



[As partes desta mensagem que não continham texto foram removidas]



##### ##### #####

Para saber mais visite
http://www.ciencialist.hpg.ig.com.br


##### ##### ##### #####


Yahoo! Grupos, um serviço oferecido por:
PUBLICIDADE




------------------------------------------------------------------------------
Links do Yahoo! Grupos

a.. Para visitar o site do seu grupo na web, acesse:
http://br.groups.yahoo.com/group/ciencialist/

b.. Para sair deste grupo, envie um e-mail para:
ciencialist-unsubscribe@yahoogrupos.com.br

c.. O uso que você faz do Yahoo! Grupos está sujeito aos Termos do Serviço do Yahoo!.



[As partes desta mensagem que não continham texto foram removidas]



SUBJECT: Tens Ciencia disso ?
FROM: "L.E.R.de Carvalho" <lecarvalho@infolink.com.br>
TO: ciencialist@yahoogrupos.com.br
DATE: 01/02/2005 15:36

At 15:19 1/2/2005, you wrote:
>Olá Alvaro
>Parag. 1º - Que a Malu Mader se divorcie do Toni Beloto fique
>irremediavelmente apaixonada por mim (Homero)
>Homero



E aquele monte de filho que ela tem ?

Vão entrar nesse romance que você está escrevendo no imaginário ?

Tu tem CIENCIA disso ?

L.E.

[As partes desta mensagem que não continham texto foram removidas]



SUBJECT: O Zodíaco dos cientistas
FROM: "Jamil Orlandelli" <orlandel@ig.com.br>
TO: <sbcr@yahoogrupos.com.br>
CC: <ciencialist@yahoogrupos.com.br>
DATE: 01/02/2005 20:03

http://revistagalileu.globo.com/Galileu/0,6993,ECT900784-1941-1,00.html

[As partes desta mensagem que não continham texto foram removidas]



SUBJECT: Re: C14
FROM: "rmtakata" <rmtakata@altavista.net>
TO: ciencialist@yahoogrupos.com.br
DATE: 01/02/2005 20:05


--- Em ciencialist@yahoogrupos.com.br, "Oraculo" <oraculo@a...>
> Olá Emiliano e Brudna
>
> Mas o Brudna tem razão, o fóssil troca seu material organico por
> minerais e se torna um "molde em pedra" do osso original (ou uma
> imagem do animal completa, inclusive algumas partes moles e até

Tem outro ponto - mesmo q. uma parte do fossil fosse preservada com
material original, a datacao por meio carbono-14 eh limitada a algo em
torno de 100.000 anos (nao estou certo, mas ha' algumas tecnicas q.
aumentam a sensibilidade para ateh 300.000 anos). Isso porq. o tempo
de meia-vida do carbono-14 eh relativamente pequeno, uns 3.000 anos. A
medida em q. passa o tempo, resta pouco do C-14 original para ser
detetado.

Fosseis e subfosseis relativamente recentes podem ser datados por C-14.

[]s,

Roberto Takata





SUBJECT: RE: [ciencialist] Re: Sobre Aspartame
FROM: "murilo filo" <avalanchedrive@hotmail.com>
TO: ciencialist@yahoogrupos.com.br
DATE: 01/02/2005 22:23

Obrigado, + 1X.
Por mim, e por causa da alergia, não recomendo a ninguém colocar uma
porcaria tão artificial prá dentro do organismo, só p/sentir um docinho na
boca!
É confiança demais na ciência comercial!
Takata, vc me escreveu em off e eu lhe respondí... por duas vêzes a mensagem
voltou.
Qual é o seu i1/2 bom?
abr/M.

>From: "rmtakata" <rmtakata@altavista.net>
>Reply-To: ciencialist@yahoogrupos.com.br
>To: ciencialist@yahoogrupos.com.br
>Subject: [ciencialist] Re: Sobre Aspartame
>Date: Tue, 01 Feb 2005 10:51:48 -0000
>
>
>--- Em ciencialist@yahoogrupos.com.br, "murilo filo"
> > A questão que vejo, por minha *própria experiência*, são os efeitos
> > alérgicos da coisa.
>
>Bem, em principio quase qq coisa pode dar alergia a uma determinada
>faixa de pessoas. Em relacao a isso, o q. se pode fazer eh alertar
>para q. as pessoas sensiveis evitem o produto.
>
> > E sobre os fenilcetonúricos... qual é o lance?
>
>Eles nao metabolizam normalmente o aminoacido fenilalanina (Phe).
>Aquele teste do pezinho, entre outras coisas, serve para detetar quem
>apresenta esse problema. Essas pessoas devem ter uma dieta rigidamente
>controlada. Por lei, os produtos alimenticios q. normalmente nao
>possuem o aminoacido devem apresentar um alerta qdo eh acrescido de um
>componente q. tenha a Phe.
>
>http://www.fenilcetonuria.com.br/fenilcetonuria.html
>http://www.anvisa.gov.br/faqdinamica/asp/usuario.asp?usersecoes=28&userassunto=42
>
>[]s,
>
>Roberto Takata
>
>
>




SUBJECT: Re: O Manifesto de Porto Alegre
FROM: Manuel Bulcão <manuelbulcao@uol.com.br>
TO: ciencialist@yahoogrupos.com.br
DATE: 02/02/2005 01:22


Oi Álvaro,

Álvaro Augusto escreveu: Concordo com todos esses itens. Depois que
eles forem implementados, sugiro também o seguinte:

a) Revogar a lei da gravitação universal...; b) Revogar a Lei de
Joule...; c) Revogar a lei da oferta e da procura...

Manuel: Ô Álvaro, tu sabias que a humanidade tem cerca de 150
milênios e que, até uns dez mil anos atrás, as pessoas produziam e
consumiam sem a intermediação do mercado (mercadorias, moeda, lei da
oferta e da procura...) ou mesmo de qualquer coisa parecida com
isso?

O mercado -- e, portanto, a lei da oferta e da procura -- é um
fenômeno histórico recentíssimo que começou a se formar no
crepúsculo do neolítico, muito tempo depois do surgimento da
agricultura, da pecuária e mesmo da metalurgia. Antes disso, o que
existia era um escambozinho aqui e outro acolá "entre tribos" (não
entre indivíduos de uma mesma comunidade tribal), algo mais parecido
com troca de presentes do que com atividade mercantil.

O que quero dizer é que a lei da oferta e da procura, se por um lado
é algo mais "duro" que uma norma jurídica, por outro lado está longe
de ser uma lei "natural" férrea do tipo que não se pode transcender,
como é o caso da lei da gravitação. Até porque o mercado é uma
instituição dinâmica que evoluiu e continua evoluindo: a moeda, que
surgiu como um simples meio de troca, após algum tempo converteu-se
também num instrumento contábil e, depois disso, na primeira forma
de "capital"; o juro, que numa economia mercantil simples pré-
capitalista consiste na recompensa pela renúncia ao consumo
presente, no mercado capitalista desenvolvido é algo bem distinto: o
preço da mercadoria-moeda.

E quanto a lei da oferta e da procura, ela opera de um jeito numa
economia livre-concorrencial formada por uma miríade de pequenos
produtores e comerciantes, e de uma maneira um tanto diferente no
capitalismo oligopolista moderno, em que um ou poucos sujeitos
econômicos detêm oitenta por cento ou mais da oferta de um
determinado produto e, por isso, via administração e controle da sua
quantidade, têm amplo poder de estabelecer seu preço. Em tal
situação, o jogo da oferta e da procura é um jogo de cartas
marcadas. (Claro que, relativamente ao "mercado da força-de-
trabalho", esses oligopólios continuam a defender a lei "impessoal",
o jogo "honesto" e, para tanto, a desregulamentação.)

Essa evolução do mercado não é "natural" no sentido de "totalmente
indenpendente da vontade dos homens", pelo contrário, trata-se de um
desenvolvimento que foi e continua sendo amplamente dirigido pelo
Estado. Diga-se a propósito que o mercado pseudo-natural (automático
e supostamente impessoal) da economia neoclássica não passa de uma
criação do Estado liberal. Tal Estado "mínimo" tende a ser "máximo"
no que diz respeito às suas instituições policiais e militares,
pois, muito freqüentemente, é necessário impor à populaça ignara --
por meio de uma mão despótica nada invisível -- as leis "naturais"
da economia mercantil-capitalista, como o fez Pinochet no Chile com
total endosso de Friedrich A. Von Hayek.

Assim como a água da chuva corre para o rio e o rio corre pro mar,
num mercado capitalista desregulamentado o dinheiro
vai "naturalmente" para quem tem dinheiro. É de se desconfiar, pois,
que por trás dessa lei "natural" existe uma norma "moral": "dai mais
a quem tem mais".

A economia, quanto mais se converte em ciência "natural" (pretenso
conhecimento objetivo livre de juízos de valor), mais se parece com
uma ideologia descarada. Paradoxo!

Abraços,
Manuel Bulcão






SUBJECT: sabão, sabonetes... etc e tal.
FROM: "E m i l i a n o C h e m e l l o" <chemelloe@yahoo.com.br>
TO: <ciencialist@yahoogrupos.com.br>
DATE: 02/02/2005 09:29

Fora a dica de não utilizá-los, há alguma dica para a moça? :-)

[ ] 's do Emiliano Chemello
emiliano@quimica.net
http://www.quimica.net/emiliano
http://www.ucs.br/ccet/defq/naeq
---
Contato Naeq:
Nome: Tamara
Email: tamara@flynet.com.br
Assunto: sabonete
Mensagem: Boa tarde
parabens pelo artigo "saboes", pude esclarecer muitas duvidas, mas gostaria
ainda de uma informação. O que pode ser usado p/ que o sabão ou sabonete nao
se desmanche tão facil?
Grata pela atenção




SUBJECT: Re: [ciencialist] Louva-a-deus no teto
FROM: "Eurico Ferreira de Souza Jr." <caodejah@yahoo.com.br>
TO: ciencialist@yahoogrupos.com.br
DATE: 02/02/2005 17:31

[E]> uma vez um louva-a-deus encarou meu gato (tá... ele ainda era meio filhote...) abrindo os "braços" e fazendo aquela pose de "karatê kid".
devemos lembrar que o louva-a-deus é um predador...

[]s

Eurico


_\|/_
__________________________________________________
Converse com seus amigos em tempo real com o Yahoo! Messenger
http://br.download.yahoo.com/messenger/

[As partes desta mensagem que não continham texto foram removidas]



SUBJECT: Re: [ciencialist] Re: O Manifesto de Porto Alegre
FROM: "Alvaro Augusto \(E\)" <alvaro@electraenergy.com.br>
TO: <ciencialist@yahoogrupos.com.br>
DATE: 02/02/2005 19:52

Caro Manuel,

Talvez eu tenha dado a entender que coloco a lei da oferta e da procura no lugar de uma lei natural. Não era a minha intenção. Embora haja algum substrato físico nas leis da economia (por exemplo, a lei da oferta e procura pode ser decorrência de um princípio de ação mínima), estas estão longe de ter a precisão matemática das leis da física. De qualquer forma, a única maneira de revogar a lei da oferta e procura seria acabando com o mercado, e isso está longe de acontecer. Alvin Toffler garante que o nosso conceito de mercado será totalmente diferente em uma sociedade da terceira onda, mas deverá passar muito tempo até todos os países estejam nesta onda...

[ ]s

Alvaro Augusto

----- Original Message -----
From: Manuel Bulcão
To: ciencialist@yahoogrupos.com.br
Sent: Wednesday, February 02, 2005 1:22 AM
Subject: [ciencialist] Re: O Manifesto de Porto Alegre



Oi Álvaro,

Álvaro Augusto escreveu: Concordo com todos esses itens. Depois que
eles forem implementados, sugiro também o seguinte:

a) Revogar a lei da gravitação universal...; b) Revogar a Lei de
Joule...; c) Revogar a lei da oferta e da procura...

Manuel: Ô Álvaro, tu sabias que a humanidade tem cerca de 150
milênios e que, até uns dez mil anos atrás, as pessoas produziam e
consumiam sem a intermediação do mercado (mercadorias, moeda, lei da
oferta e da procura...) ou mesmo de qualquer coisa parecida com
isso?

O mercado -- e, portanto, a lei da oferta e da procura -- é um
fenômeno histórico recentíssimo que começou a se formar no
crepúsculo do neolítico, muito tempo depois do surgimento da
agricultura, da pecuária e mesmo da metalurgia. Antes disso, o que
existia era um escambozinho aqui e outro acolá "entre tribos" (não
entre indivíduos de uma mesma comunidade tribal), algo mais parecido
com troca de presentes do que com atividade mercantil.

O que quero dizer é que a lei da oferta e da procura, se por um lado
é algo mais "duro" que uma norma jurídica, por outro lado está longe
de ser uma lei "natural" férrea do tipo que não se pode transcender,
como é o caso da lei da gravitação. Até porque o mercado é uma
instituição dinâmica que evoluiu e continua evoluindo: a moeda, que
surgiu como um simples meio de troca, após algum tempo converteu-se
também num instrumento contábil e, depois disso, na primeira forma
de "capital"; o juro, que numa economia mercantil simples pré-
capitalista consiste na recompensa pela renúncia ao consumo
presente, no mercado capitalista desenvolvido é algo bem distinto: o
preço da mercadoria-moeda.

E quanto a lei da oferta e da procura, ela opera de um jeito numa
economia livre-concorrencial formada por uma miríade de pequenos
produtores e comerciantes, e de uma maneira um tanto diferente no
capitalismo oligopolista moderno, em que um ou poucos sujeitos
econômicos detêm oitenta por cento ou mais da oferta de um
determinado produto e, por isso, via administração e controle da sua
quantidade, têm amplo poder de estabelecer seu preço. Em tal
situação, o jogo da oferta e da procura é um jogo de cartas
marcadas. (Claro que, relativamente ao "mercado da força-de-
trabalho", esses oligopólios continuam a defender a lei "impessoal",
o jogo "honesto" e, para tanto, a desregulamentação.)

Essa evolução do mercado não é "natural" no sentido de "totalmente
indenpendente da vontade dos homens", pelo contrário, trata-se de um
desenvolvimento que foi e continua sendo amplamente dirigido pelo
Estado. Diga-se a propósito que o mercado pseudo-natural (automático
e supostamente impessoal) da economia neoclássica não passa de uma
criação do Estado liberal. Tal Estado "mínimo" tende a ser "máximo"
no que diz respeito às suas instituições policiais e militares,
pois, muito freqüentemente, é necessário impor à populaça ignara --
por meio de uma mão despótica nada invisível -- as leis "naturais"
da economia mercantil-capitalista, como o fez Pinochet no Chile com
total endosso de Friedrich A. Von Hayek.

Assim como a água da chuva corre para o rio e o rio corre pro mar,
num mercado capitalista desregulamentado o dinheiro
vai "naturalmente" para quem tem dinheiro. É de se desconfiar, pois,
que por trás dessa lei "natural" existe uma norma "moral": "dai mais
a quem tem mais".

A economia, quanto mais se converte em ciência "natural" (pretenso
conhecimento objetivo livre de juízos de valor), mais se parece com
uma ideologia descarada. Paradoxo!

Abraços,
Manuel Bulcão






##### ##### #####

Para saber mais visite
http://www.ciencialist.hpg.ig.com.br


##### ##### ##### #####


Yahoo! Grupos, um serviço oferecido por:







------------------------------------------------------------------------------
Links do Yahoo! Grupos

a.. Para visitar o site do seu grupo na web, acesse:
http://br.groups.yahoo.com/group/ciencialist/

b.. Para sair deste grupo, envie um e-mail para:
ciencialist-unsubscribe@yahoogrupos.com.br

c.. O uso que você faz do Yahoo! Grupos está sujeito aos Termos do Serviço do Yahoo!.



[As partes desta mensagem que não continham texto foram removidas]



SUBJECT: virus e ciencialist
FROM: "rayfisica" <rayfisica@yahoo.com.br>
TO: ciencialist@yahoogrupos.com.br
DATE: 02/02/2005 21:38


Gostaria de comunicar aos integrantes da lista que recebi três e-
mails com vírus nas confirmações de recebimento de mensagens
que eu
enviei para a lista.
Com certeza essas pessoas não sabem que estão disseminando
vírus e
por eficiência dos 12 programas que eu mantenho no pc não sofri
nenhum dano, porem eu realizo um esforço grande para manter meu pc
livre dessa praga e também para não ser veiculo dessa
desgraça, me
sinto no direito de pedir o mesmo ao net-amigos já que essas
pessoas
entraram para a pasta de remetentes bloqueados.
Nesse link é possível um pequeno curso de como se livrar das
pragas e
como adquirir programas para tanto, e no site tem informações num
fórum onde o pessoal é muito prestativo, boa sorte:

http://www.baboo.com.br/absolutenm/anmviewer.asp?a=12651&z=24

Ps o ultimo vírus veio de @objetivo.goiania.com.br






SUBJECT: Re: [ciencialist] virus e ciencialist
FROM: "Alvaro Augusto \(E\)" <alvaro@electraenergy.com.br>
TO: <ciencialist@yahoogrupos.com.br>
DATE: 02/02/2005 21:51

Só três??

Os vírus modernos não precisam infectar um computador para enviar e-mails a partir dele. Esses virus pesquisam a rede em busca de e-mails correlacionados e enviam mensagens, a partir de um computador infectado, como sendo originadas de um endereço qualquer. A pessoa pode ter seu computador 100% livre de pragas, e ainda assim ter mensagens enviadas em seu nome a partir de um computador do outro lado da galáxia, digo, Terra...

[ ]s

Alvaro Augusto


----- Original Message -----
From: rayfisica
To: ciencialist@yahoogrupos.com.br
Sent: Wednesday, February 02, 2005 9:38 PM
Subject: [ciencialist] virus e ciencialist



Gostaria de comunicar aos integrantes da lista que recebi três e-
mails com vírus nas confirmações de recebimento de mensagens
que eu
enviei para a lista.
Com certeza essas pessoas não sabem que estão disseminando
vírus e
por eficiência dos 12 programas que eu mantenho no pc não sofri
nenhum dano, porem eu realizo um esforço grande para manter meu pc
livre dessa praga e também para não ser veiculo dessa
desgraça, me
sinto no direito de pedir o mesmo ao net-amigos já que essas
pessoas
entraram para a pasta de remetentes bloqueados.
Nesse link é possível um pequeno curso de como se livrar das
pragas e
como adquirir programas para tanto, e no site tem informações num
fórum onde o pessoal é muito prestativo, boa sorte:

http://www.baboo.com.br/absolutenm/anmviewer.asp?a=12651&z=24

Ps o ultimo vírus veio de @objetivo.goiania.com.br






##### ##### #####

Para saber mais visite
http://www.ciencialist.hpg.ig.com.br


##### ##### ##### #####


Yahoo! Grupos, um serviço oferecido por:
PUBLICIDADE




------------------------------------------------------------------------------
Links do Yahoo! Grupos

a.. Para visitar o site do seu grupo na web, acesse:
http://br.groups.yahoo.com/group/ciencialist/

b.. Para sair deste grupo, envie um e-mail para:
ciencialist-unsubscribe@yahoogrupos.com.br

c.. O uso que você faz do Yahoo! Grupos está sujeito aos Termos do Serviço do Yahoo!.



[As partes desta mensagem que não continham texto foram removidas]



SUBJECT: Re: [ciencialist] O Zodíaco dos cientistas
FROM: "Alvaro Augusto \(E\)" <alvaro@electraenergy.com.br>
TO: <ciencialist@yahoogrupos.com.br>
DATE: 02/02/2005 22:01

Esse argumento não vale contra a astrologia. Os astrólogos sabem disso muito bem e criaram duas explicações:

1. O que interessa não são as posições das constelações, e sim as posições de certos "setores celestiais". Esses setores, que correspondem aos signos, são os verdadeiros influenciadores da vida na Terra, e coincidiam com as posições das constelações quando a astrologia foi criada;

2. Tudo bem, as constelações mudaram de posição e a astrologia também precisa mudar. Para isso criamos a "astrologia sideral", que parte das posições atuais das constelações zodiacais.

E, como diz o autor no fim do texto, "acredite quem quiser"...

[ ]s

Alvaro Augusto

----- Original Message -----
From: Jamil Orlandelli
To: sbcr@yahoogrupos.com.br
Cc: ciencialist@yahoogrupos.com.br
Sent: Tuesday, February 01, 2005 8:03 PM
Subject: [ciencialist] O Zodíaco dos cientistas


http://revistagalileu.globo.com/Galileu/0,6993,ECT900784-1941-1,00.html

[As partes desta mensagem que não continham texto foram removidas]



##### ##### #####

Para saber mais visite
http://www.ciencialist.hpg.ig.com.br


##### ##### ##### #####


Yahoo! Grupos, um serviço oferecido por:







------------------------------------------------------------------------------
Links do Yahoo! Grupos

a.. Para visitar o site do seu grupo na web, acesse:
http://br.groups.yahoo.com/group/ciencialist/

b.. Para sair deste grupo, envie um e-mail para:
ciencialist-unsubscribe@yahoogrupos.com.br

c.. O uso que você faz do Yahoo! Grupos está sujeito aos Termos do Serviço do Yahoo!.



[As partes desta mensagem que não continham texto foram removidas]



SUBJECT: Re: C14
FROM: Maria Natália <grasdic@hotmail.com>
TO: ciencialist@yahoogrupos.com.br
DATE: 02/02/2005 23:48


Sérgio:

Nas minhas férias grandes há 2 anos estive a fazer datação por
radiocarbono em ossadas humanas que se suponham ser romanas.
Foi no nosso reactor nuclear,do Instituto Tecnológico e nuclear, em
Sacavém. Foi um curso para professores e do que lá aprendi aqui vai
resumido. Com os nomes podes fazer pesquisa mais avançada no google ou
dirigires-te a um dos investigadores da bibliografia que te vou
enviar. É o que te posso dar pois estou com imenso trabalho e devo
partir para trabalho de campo para a semana...

""O grupo "Património Cultural e Ciências" está essencialmente
vocacionado para a investigação cientifica e conservação do património
cultural Português, através da aplicação de métodos nucleares de
análise, geoquímica, química e mineralogia.

A sua actividade científica inclui a aplicação do método de análise
por activação neutrónica baseado no reactor Português de investigação,
visando o estudo da origem e tecnologia de produção de artefactos
cerâmicos, bem como o estudo da alteração dos materiais envolvidos na
construção do património nacional edificado.

Dois métodos de datação (14C e luminescência) estão disponíveis para
as comunidades científicas arqueológica/geológica, de modo a
estabelecer cronologias e resolver problemas relacionados com a idade
dos artefactos e dos depósitos.

O grupo, além de serviços, está também envolvido em investigação
fundamental, usando todos os meios disponíveis no ITN.

Laboratório de Datação por Radiocarbono
No laboratório de Datação por Radiocarbono determina-se a concentração
de 14C por Cintilação Líquida. Este método envolve a transformação em
benzeno do carbono existente nas amostras.

Usei uma linha de vácuo para síntese de benzeno (43KB) e dá um
trabalhão retirar a amostra do osso...e claro com a segurança por nós
praticada pois além dos objectos em si e que são demasiado frageis
temos o benzeno e que como deves saber é cacerígeno...
No laboratório de Datação por Radiocarbono existe o seguinte equipamento:
· Contadores de Cintilação Líquida:
o Tri-carb 4530 - Packard
o Tri-carb 2770 TR/SL - Packard


BIBLIOGRAFIA sobre o tema e que se insere na componente "Património
Cultural e Ciências"
2003
· M.I. DIAS, M.I. PRUDÊNCIO, M.A. SEQUEIRA BRAGA, M.A. GOUVEIA, C.A.
ALVES, A.C. VALERA, Provenance and Technology of Pre-Historic Pottery
form Fornos de Algodres (Portugal): the Fraga da Pena archaeological
site. British Archaeological Reports, International Series 1011
(2002), p.253-263.
· M. NASRAOUI, J.C. WAERENBORGH, M.I. PRUDÊNCIO, E. BILAL, Typology of
the granitic stones of the cathedral of Évora (Portugal): a combined
contribution of geochemistry and 57Fe Mössbauer spectroscopy, J. Cult.
Heritage 3 (2002) 127-132.
· M. I. PRUDÊNCIO, M. A. SEQUEIRA BRAGA, H. PAQUET, J. C.
WAERENBORGH, L.C.J.PEREIRA, M. A. GOUVEIA, Clay mineral assemblages in
weathered basalt profiles from central and southern Portugal: climatic
significance, Catena 49 (2002) 77.
· J.C. WAERENBORGH, J. FIGUEIRAS, A. MATEUS, M. GONÇALVES, 57Fe
Mössbauer spectroscopy study of the correlation between the Fe3+
content and the magnetic properties of natural Cr spinels, Eur. J.
Min. 14 (2002) 437-446.
· J. FIGUEIRAS, A. MATEUS, M. GONÇALVES, J.C. WAERENBORGH, P.
FONSECA, Geodynamical evolution of the South Variscan Iberian Suture
as recorded by mineral transformations, Geodin. Acta 15 (2002) 45-61.
· J.M.P. CABRAL, S.M. FONSECA, M.A. GOUVEIA, Caracterização química
das produções de ânforas do Vale do Tejo: III- Quinta do Rouxinol
(continuação), Revista Portuguesa de Arqueologia 5, nº2 (2002) 325–338.
· J.C. WAERENBORGH, J. FIGUEIRAS, A. MATEUS, M. GONÇALVES, Nature and
mechanism of ilmenite alteration: a Mössbauer and X-ray diffraction
study of oxidized ilmenite from the Beja-Acebuches Ophiolite Complex
(SE Portugal), Min. Mag. 66 (2002) 421–430.
· D. RICHTER, B. SCHROEDER, W.J. RINK, P.J. Julig, P.J., Long, D.G.F.
& Schwarcz, H.P., The Middle to Upper Palaeolithic transition in the
Levant and new TL dates for a Late Mousterian assemblage from Jerf
al-Ajla Cave near Palmyra, Syria. Paléorient 27/ (2002) 29-46.
· M. F. ARAÚJO, J.-M. JOUANNEAU, P. VALÉRIO, T. BARBOSA, A. GOUVEIA,
O. WEBER, A. OLIVEIRA, A. RODRIGUES, J.M.A.DIAS, Geochemical Tracers
of Northern Portuguese Estuarine Sediments on the Shelf. Progress in
Oceanography 52 (2002) 277-297. Accepted for publication or in press
· A.E. MARKS, K. MONIGAL, D. RICHTER, W.J. RINK, H.P.SCHWARCZ,
Preliminary results on the excavation of the Lower Palaeolithic site
of Brecha das Lascas, Almonda, Portugal, European Prehistory.
· W.J. RINK, D. RICHTER, N. MERCIER, H. VALLADAS, A.E. MARKS,
H.P.SCHWARCZ, D. KAUFMAN, Age of the Middle Palaeolithic Site of Rosh
Ein Mor, Central Negev, Israel, Journal of Archaeological Science.
· W.J. RINK, L.A.SCHEPARTZ, S. MILLER-ANTONIO, W. HUANG, Y. HOU, D.
BAKKEN, D. RICHTER , H.L.JONES, Electron Spin Resonance (ESR) Dating
of Tooth Enamel at Panxian Dadong Cave, Guizhou, China, British
Archaeology Reports, International Series.
· A. MONGE SOARES, A duna de Magoito Revisitada, Revista Portuguesa
de Arqueologia.
· A. MONGE SOARES, A. C. SOUSA, Cronologia absoluta para os concheiros
de S. Julião, Publicação Monográfica "Concheiros de S. Julião"
· L. C. ALVES, M.F. ARAÚJO, A. MONGE SOARES, Estudo de um torques
proveniente de noroeste peninsular – aplicação de métodos
instrumentais de análise química não-destrutivos, O Arqueólogo Português.
· M.I. DIAS, M.I. PRUDÊNCIO, A.C. VALERA, M. LAGO & M.A. GOUVEIA,
Composition, technology and functional features of Chalcolithic
pottery from Perdigões, Reguengos de Monsaraz (Portugal).
Geoarchaeological and Bioarchaeological Studies, 2002 (in press)
Apresentadas em Conferências
Aceites para publicação ou no prelo
· D. RICHTER, Thermoluminescence dating of heated flint. Method and
Application. Proceedings of the VIII International Flint Symposium,
Bochum.
· J. RAPOSO, C. FABIÃO, A. GUERRA, J. BUGALHÃO, A.L. DUARTE, A.
SABROSA, M.I. DIAS, M.I. PRUDÊNCIO & M.A. GOUVEIA, OREsT Project: late
Roman pottery productions from the Low Tejo. Proceedings of the 1st
International Conference on Late Roman Coarse Wares, Cooking Wares and
Amphorae in the Mediterranean: archaeology and archaeometry,
Barcelona, Espanha, 2002 (in press).
Outras publicações
· M.A. GOUVEIA, M.I. DIAS, M.F. ARAÚJO, M.I. PRUDÊNCIO, M.C. FREITAS,
, P. VALÉRIO, A.P. MARQUES, "Valorizar os Resíduos Monitorizar o
Ambiente - Monitorização de Elementos Traço na envolvente à CTRSU de
S. João da Talha (1999-2002)", Ed. Valorsul - Valorização e tratamento
de resíduos sólidos da área metropolitana de Lisboa (Norte), S.A.(2002).
Comunicações
· J. C. WAERENBORGH, M. I. PRUDÊNCIO, M. A. GASPAR, M. I. DIAS,
Production technology of late roman grey ceramics from Braga (North
Portugal), 33rd International Symposium on Archaeometry Amsterdam, The
Netherlands, April 2002, poster.
· D. RICHTER, "Luminescence dating of small samples of heated silex.
New ways in dating archaeological sites by employing the orange-red
emission and a SAR protocol."8th Colloquium on Luminescence and ESR
Dating Seminar, Sächsische Akademie der Wisenschaften,
Freiberg/Helbigsdorf, Alemanha, 20th October 2002, oral.
· M.I. DIAS, M.I. PRUDÊNCIO, A.C. VALERA, M. LAGO & M.A. GOUVEIA,
Composition, technology and functional features of Chalcolithic
pottery from Perdigões, Reguengos de Monsaraz (Portugal). 33rd
International Symposium on Archaeometry, Amsterdão, Holanda, 2002,
poster.
· M. I. DIAS & G. O. CARDOSO, A Database for technology and provenance
studies of ceramics an example for Portuguese archaeological sites.
33rd International Symposium on Archaeometry, Amsterdão, Holanda,
2002, poster.
· J. RAPOSO, C. FABIÃO, A. GUERRA, J. BUGALHÃO, A.L. DUARTE, A.
SABROSA, M.I. DIAS, M.I. PRUDÊNCIO & M.A. GOUVEIA, OREsT Project: late
Roman pottery productions from the Low Tejo. 1st
InternationalConference on Late Roman Coarse Wares, Cooking Wares and
Amphorae in the Mediterranean: archaeology and archaeometry,
Barcelona, Espanha, 2002, poster.
· J. RAPOSO, C. FABIÃO, A. GUERRA, J. BUGALHÃO, A.L. DUARTE, A.
SABROSA, M.I. DIAS, M.I. PRUDÊNCIO & M.A. GOUVEIA, OREsT Project:
archaeological and archaeometric research in the Low Tejo. 1st
International Conference on Late Roman Coarse Wares, Cooking Wares and
Amphorae in the Mediterranean: archaeology and archaeometry,
Barcelona, Espanha, 2002, poster.
· M.I. DIAS, M.I. PRUDÊNCIO, CARVALHO, A., ALMEIDA, J., "Provenance
studies of marble statuary from Q. Longas, Elvas", Conference "La
villa romana de la Quinta das Longas. Un hallazgo excepcional".Museo
Nacional de Arte Romano, Merida, Spain, oral.
· M.I. DIAS, M.I. PRUDÊNCIO, CARVALHO, A., ALMEIDA, J., "Arqueometria
de Cerâmicas da Villa Romana da Quinta das Longas: Composição Química
vs Proveniência.Estudo preliminar". Conference "Materials circulation
in the Roman period", Museo Nacional de Arqueologia, Cáceres, Spain,
oral.


E creio que tens aqui pano para mangas e te diverte a ler isto no teu
Carnaval.
Um abraço
Maria Natália

--- Em ciencialist@yahoogrupos.com.br, "harapadalsha2000" <sesa@b...>
escreveu
>
> Olá pessoal
>
> Gostaria de uma ajuda
>
> Como funciona e como é a lógica de datação pelo Carbono14?
>
>
> Grato
>
>
> Sérgio





SUBJECT: Re: O Manifesto de Porto Alegre
FROM: "rmtakata" <rmtakata@altavista.net>
TO: ciencialist@yahoogrupos.com.br
DATE: 03/02/2005 11:02


--- Em ciencialist@yahoogrupos.com.br, "Alvaro Augusto \(E\)"
> Alvin Toffler garante que o nosso conceito de mercado será
> totalmente diferente em uma sociedade da terceira onda, mas deverá
> passar muito tempo até todos os países estejam nesta onda...

Sem querer sem indelicado com o sofrimentos das vitimas dos paises do
sudeste e sul da Asia: essa terceira onda vira' na forma de um maremoto?

[]s,

Roberto Takata





SUBJECT: Re: Ciência Maluca
FROM: "rmtakata" <rmtakata@altavista.net>
TO: ciencialist@yahoogrupos.com.br
DATE: 03/02/2005 11:46


--- Em ciencialist@yahoogrupos.com.br, "Esteban Moreno"
> REVISTA SUPER INTERESSANTE - Publicado na Edição 209 - 01/2005
> PQP
> Pesquisadores russos ficaram falando palavrão para uma jarra d'água.
> Tudo em nome da ciência: a tese deles era que o líquido seria afetado
> de alguma maneira. Para checar, regaram sementes com a água "maldita".
> Os cientistas garantem que 48% delas germinaram, contra 93% das que
> receberam água normal.

Deve ser bafo de vodca...

[]s,

Roberto Takata





SUBJECT: Ciência Maluca
FROM: "Esteban Moreno" <estebanmoreno@idhi.org.br>
TO: <ciencialist@yahoogrupos.com.br>
DATE: 03/02/2005 11:57


REVISTA SUPER INTERESSANTE - Publicado na Edição 209 - 01/2005

CIÊNCIA MALUCA
Por Alexandre Versignassi

Mulher gosta de macho
Cientistas americanos e austríacos estudaram o comportamento dos homens e concluíram: os que mais chamam a atenção feminina nos bares são os que têm comportamento de "macho dominante" na frente dos amigos - davam socos de brincadeira, falavam alto, se esparramavam no sofá... Os mais polidos passavam imagem de submissos. E não pegavam ninguém.
Eu bebo sim
Se sua reputação não está lá essas coisas, evite aparecer num daqueles camarotes de Carnaval. Após árduo estudo, psicólogos da Universidade Southern Illinois, nos Estados Unidos, concluíram: quem usa camisetas com estampa de bebida alcoólica passa a imagem de ser "menos honesto, menos responsável, menos pontual e menos sensato". Ah: e "mais propenso a beber". Não diga...

Matança Online
Se você acha videogames de ação violentos, não viu nada. O rancheiro texano John Lockwood criou um sistema para caçar via internet: você usa o mouse para controlar uma câmera e um rifle instalados no rancho dele. Aí é só clicar para matar, de verdade, os veados e antílopes que estiverem por ali.

PQP
Pesquisadores russos ficaram falando palavrão para uma jarra d'água. Tudo em nome da ciência: a tese deles era que o líquido seria afetado de alguma maneira. Para checar, regaram sementes com a água "maldita". Os cientistas garantem que 48% delas germinaram, contra 93% das que receberam água normal. E também juram que falar palavrão causa impotência. Só não ficou claro se a conclusão foi baseada no desempenho sexual obtido pelos próprios após passar o dia xingando a jarra.

Barata Robocop
Inseticida é coisa do passado. Cientistas europeus inventaram um mini-robô que atrai baratas imitando o comportamento delas. Pesquisadores acreditam que em cinco anos as cucarachas cibernéticas estarão no mercado para substituir dedetizações. Tomara que elas não se voltem contra os humanos.


[As partes desta mensagem que não continham texto foram removidas]



SUBJECT: Re: [ciencialist] Ciência Maluca
FROM: "E m i l i a n o C h e m e l l o" <chemelloe@yahoo.com.br>
TO: <ciencialist@yahoogrupos.com.br>
DATE: 03/02/2005 11:57


A revista superinteressante está cada vez MENOS interessante.

---
[ ] 's do Emiliano Chemello
emiliano@quimica.net
http://www.quimica.net/emiliano
http://www.ucs.br/ccet/defq/naeq

" Rien ne se perd, rien ne se crée,
tout se transforme."

Antoine Laurent de Lavoisier (químico francês, 1743 - 1794)

----- Original Message -----
From: Esteban Moreno
To: ciencialist@yahoogrupos.com.br
Sent: Thursday, February 03, 2005 11:57 AM
Subject: [ciencialist] Ciência Maluca



REVISTA SUPER INTERESSANTE - Publicado na Edição 209 - 01/2005

CIÊNCIA MALUCA
Por Alexandre Versignassi

Mulher gosta de macho
Cientistas americanos e austríacos estudaram o comportamento dos homens e concluíram: os que mais chamam a atenção feminina nos bares são os que têm comportamento de "macho dominante" na frente dos amigos - davam socos de brincadeira, falavam alto, se esparramavam no sofá... Os mais polidos passavam imagem de submissos. E não pegavam ninguém.
Eu bebo sim
Se sua reputação não está lá essas coisas, evite aparecer num daqueles camarotes de Carnaval. Após árduo estudo, psicólogos da Universidade Southern Illinois, nos Estados Unidos, concluíram: quem usa camisetas com estampa de bebida alcoólica passa a imagem de ser "menos honesto, menos responsável, menos pontual e menos sensato". Ah: e "mais propenso a beber". Não diga...

Matança Online
Se você acha videogames de ação violentos, não viu nada. O rancheiro texano John Lockwood criou um sistema para caçar via internet: você usa o mouse para controlar uma câmera e um rifle instalados no rancho dele. Aí é só clicar para matar, de verdade, os veados e antílopes que estiverem por ali.

PQP
Pesquisadores russos ficaram falando palavrão para uma jarra d'água. Tudo em nome da ciência: a tese deles era que o líquido seria afetado de alguma maneira. Para checar, regaram sementes com a água "maldita". Os cientistas garantem que 48% delas germinaram, contra 93% das que receberam água normal. E também juram que falar palavrão causa impotência. Só não ficou claro se a conclusão foi baseada no desempenho sexual obtido pelos próprios após passar o dia xingando a jarra.

Barata Robocop
Inseticida é coisa do passado. Cientistas europeus inventaram um mini-robô que atrai baratas imitando o comportamento delas. Pesquisadores acreditam que em cinco anos as cucarachas cibernéticas estarão no mercado para substituir dedetizações. Tomara que elas não se voltem contra os humanos.


[As partes desta mensagem que não continham texto foram removidas]


[As partes desta mensagem que não continham texto foram removidas]



SUBJECT: Democracia de microondas - John Hemingway
FROM: José Renato <jrma@terra.com.br>
TO: <ciencialist@yahoogrupos.com.br>
DATE: 03/02/2005 12:44

DEMOCRACIA DE MICROONDAS

John Hemingway

A Administração Bush é certamente culpada de ter restringido a pesquisa científica e sua difusão. A preocupante teoria do aquecimento global foi banida dos websites do governo e os financiamentos de programas interessados no estudo das propriedades milagrosas das células-tronco foram reduzidos (para acalmar os que são contrários ao aborto).

No Iraque, o Exército americano, com o apoio da Marinha e dos serviços de inteligência, está pesquisando armas dotadas de fio cortante eletromagnético. A "exibição" de democracia dos neoconservadores tornou-se campo de experiência de plataformas de microondas que estão agora saturando a destroçada cidade de Fallujah com a combinação de frequências ULF, VLF, UHF e EHF. O processo conhecido como "super combinação de rádio-frequências" foi inicialmente desenvolvido pela Marinha e responsável, segundo William Thomas em seu artigo no http://globalresearch.ca/articles/THO501A.html, por uma série de inexplicáveis aparecimentos de baleias na Califórnia que estavam na verdade desorientadas pela poderosa comunicação de frequências usadas nos submarinos da Marinha.

O Exército espera conseguir, inundando a cidade (Fallujah) com essas ondas, um efeito sedativo sobre os homens que sobraram e a população potencialmente rebelde. A julgar pelo número de Marines que continuam a morrer, parece que isso não está funcionando a contento. Além disso, muitos soldados americanos têm conseguido tirar férias depois de uma super exposição a essas armas. O microondas pode temporariamente incapacitar uma pessoa, desorientando-a enquanto ela está na área saturada.

Pesquisadores russos chamaram esses efeitos adversos à saúde de "doença do microondas", relatando que "seus primeiros sinais são baixa pressão sanguínea e baixa pulsação. Mais tarde, as manifestações mais comuns são excitação crônica do sistema nervoso simpático (stress) e pressão sanguínea elevada. Esta fase também inclui frequentemente dor de cabeça, tonteira, dor nos olhos, insônia, irritabilidade, ansiedade, dor no estômago, tensão nervosa, incapacidade de concentração, perda de cabelo.mais um aumento da incidência de apendicite, catarata, problemas de reprodução e câncer. Os sintomas crônicos são eventualmente substituídos por crises da supra-renal e isquemia (bloqueio das artérias coronárias e ataque do coração)".

Para melhorar as comunicações entre as dispersas tropas americanas, os militares começaram usando o que é chamado "Ratt rigs" e "Singars", redes móveis de comunicação. O resultado final foi que, não somente Fallujah, mas a maior parte do Iraque virou zona livre de microondas. E é óbvio que para os soldados americanos não há literalmente lugar onde possam esconder-se. Boa parte do ar que respiram está saturada com D.U., o sujo isótopo radioativo Urânio 238. A América continua a revestir muitas de suas bombas e balas com uma substância para torná-las mais duras e mais penetrantes. Na parte de baixo, fora o U-238 meia vida que tem mais de quatro bilhões de anos, está o D.U. que causa câncer e as microondas que tendem, os pesquisadores acreditam, a desenvolver células cancerosas incipientes.

Mas não é só isso, em setembro do último ano Humvees equipados com armas de raios elétromagnéticos chamados "Sheriffs" foram enviados ao Iraque. Esses pesados jeeps, agora alvo favorito dos insurgentes iraquianos, receberam uma nova arma desenvolvida nos anos 90 que podem alvejar qualquer um numa linha de tiro de até 95 Ghz microondas com um raio eletromagnético projetado para penetrar até uma certa camada da carne humana. Basicamente, isso faz a vítima sentir-se como se estivesse pegando fogo, uma intensa sensação de queimadura que, entretanto, desaparece tão logo o raio elétrico é direcionado para outro lugar. Um porta-voz do Exército declarou que não houve nenhum efeito prejudicial do raio eletromagnético e que foi melhor "queimar" e "humilhar" uma pessoa do que matá-la. Certamente outro exemplo brilhante de como o novo Superhomem pensa.

< http://www.diretodaredacao.com/ >



[As partes desta mensagem que não continham texto foram removidas]



SUBJECT: Re: [ciencialist] Democracia de microondas - John Hemingway
FROM: "Eurico Ferreira de Souza Jr." <caodejah@yahoo.com.br>
TO: ciencialist@yahoogrupos.com.br
DATE: 03/02/2005 13:27

[E]> uma "gaiola de faraday" seria escudo contra essa arma?

José Renato <jrma@terra.com.br> wrote:DEMOCRACIA DE MICROONDAS

John Hemingway

A Administração Bush é certamente culpada de ter restringido a pesquisa científica e sua difusão. A preocupante teoria do aquecimento global foi banida dos websites do governo e os financiamentos de programas interessados no estudo das propriedades milagrosas das células-tronco foram reduzidos (para acalmar os que são contrários ao aborto).

No Iraque, o Exército americano, com o apoio da Marinha e dos serviços de inteligência, está pesquisando armas dotadas de fio cortante eletromagnético. A "exibição" de democracia dos neoconservadores tornou-se campo de experiência de plataformas de microondas que estão agora saturando a destroçada cidade de Fallujah com a combinação de frequências ULF, VLF, UHF e EHF. O processo conhecido como "super combinação de rádio-frequências" foi inicialmente desenvolvido pela Marinha e responsável, segundo William Thomas em seu artigo no http://globalresearch.ca/articles/THO501A.html, por uma série de inexplicáveis aparecimentos de baleias na Califórnia que estavam na verdade desorientadas pela poderosa comunicação de frequências usadas nos submarinos da Marinha.

O Exército espera conseguir, inundando a cidade (Fallujah) com essas ondas, um efeito sedativo sobre os homens que sobraram e a população potencialmente rebelde. A julgar pelo número de Marines que continuam a morrer, parece que isso não está funcionando a contento. Além disso, muitos soldados americanos têm conseguido tirar férias depois de uma super exposição a essas armas. O microondas pode temporariamente incapacitar uma pessoa, desorientando-a enquanto ela está na área saturada.

Pesquisadores russos chamaram esses efeitos adversos à saúde de "doença do microondas", relatando que "seus primeiros sinais são baixa pressão sanguínea e baixa pulsação. Mais tarde, as manifestações mais comuns são excitação crônica do sistema nervoso simpático (stress) e pressão sanguínea elevada. Esta fase também inclui frequentemente dor de cabeça, tonteira, dor nos olhos, insônia, irritabilidade, ansiedade, dor no estômago, tensão nervosa, incapacidade de concentração, perda de cabelo.mais um aumento da incidência de apendicite, catarata, problemas de reprodução e câncer. Os sintomas crônicos são eventualmente substituídos por crises da supra-renal e isquemia (bloqueio das artérias coronárias e ataque do coração)".

Para melhorar as comunicações entre as dispersas tropas americanas, os militares começaram usando o que é chamado "Ratt rigs" e "Singars", redes móveis de comunicação. O resultado final foi que, não somente Fallujah, mas a maior parte do Iraque virou zona livre de microondas. E é óbvio que para os soldados americanos não há literalmente lugar onde possam esconder-se. Boa parte do ar que respiram está saturada com D.U., o sujo isótopo radioativo Urânio 238. A América continua a revestir muitas de suas bombas e balas com uma substância para torná-las mais duras e mais penetrantes. Na parte de baixo, fora o U-238 meia vida que tem mais de quatro bilhões de anos, está o D.U. que causa câncer e as microondas que tendem, os pesquisadores acreditam, a desenvolver células cancerosas incipientes.

Mas não é só isso, em setembro do último ano Humvees equipados com armas de raios elétromagnéticos chamados "Sheriffs" foram enviados ao Iraque. Esses pesados jeeps, agora alvo favorito dos insurgentes iraquianos, receberam uma nova arma desenvolvida nos anos 90 que podem alvejar qualquer um numa linha de tiro de até 95 Ghz microondas com um raio eletromagnético projetado para penetrar até uma certa camada da carne humana. Basicamente, isso faz a vítima sentir-se como se estivesse pegando fogo, uma intensa sensação de queimadura que, entretanto, desaparece tão logo o raio elétrico é direcionado para outro lugar. Um porta-voz do Exército declarou que não houve nenhum efeito prejudicial do raio eletromagnético e que foi melhor "queimar" e "humilhar" uma pessoa do que matá-la. Certamente outro exemplo brilhante de como o novo Superhomem pensa.

< http://www.diretodaredacao.com/ >



[As partes desta mensagem que não continham texto foram removidas]



##### ##### #####

Para saber mais visite
http://www.ciencialist.hpg.ig.com.br


##### ##### ##### #####


Yahoo! Grupos, um serviço oferecido por:



















function SearchComboBox() { if (document.form_combo.keyword.value.length==0){ alert("Por favor, digite algo."); return false; }else { document.form_combo.action ="http://br.rd.yahoo.com/SIG=12astrhmj/M=264105.3931087.6562589.1588051/D=brclubs/S=2137111528:HM/EXP=1107528291/A=2361264/R=0/SIG=11uaou2jn/*http://www.bondfaro.com/bondfaro/in/combosearch_in.jsp?sk=11"; } return true;} [input] [input] [input]

---------------------------------
Links do Yahoo! Grupos

Para visitar o site do seu grupo na web, acesse:
http://br.groups.yahoo.com/group/ciencialist/

Para sair deste grupo, envie um e-mail para:
ciencialist-unsubscribe@yahoogrupos.com.br

O uso que você faz do Yahoo! Grupos está sujeito aos Termos do Serviço do Yahoo!.




_\|/_

---------------------------------
Yahoo! Acesso Grátis - Internet rápida e grátis. Instale o discador do Yahoo! agora.

[As partes desta mensagem que não continham texto foram removidas]



SUBJECT: Sobre a Vida
FROM: "Esteban Moreno" <estebanmoreno@idhi.org.br>
TO: <ciencialist@yahoogrupos.com.br>
DATE: 03/02/2005 13:31

Direto de pelotas, um esnaio sobre a vida.
http://www.comp.ufla.br/~monserrat/isc/Complexidade_caos_autoorganizacao.htm
l
Esteban.



SUBJECT: Cartão musical
FROM: "Kentaro Mori" <kentaro.mori@itelefonica.com.br>
TO: ciencialist@yahoogrupos.com.br
DATE: 03/02/2005 14:14


Muito bom: um mini-gramofone
http://www.wfmu.org/MACrec/skipsa.html

Não deixem de conferir o vídeo.

Mori





SUBJECT: Fw: geradores electricos de centrais nucleares
FROM: "Luiz Ferraz Netto" <leobarretos@uol.com.br>
TO: "ciencialist" <ciencialist@yahoogrupos.com.br>
DATE: 03/02/2005 17:07

Oi Roberto,

veja uns links para esse nosso amigo d'além-mar.
Agradeço,
===========================
Luiz Ferraz Netto [Léo]
leobarretos@uol.com.br
http://www.feiradeciencias.com.br
===========================
-----Mensagem Original-----
De: "Ricardo Ribeiro" <kornrivas@hotmail.com>
Para: <leobarretos@uol.com.br>
Enviada em: terça-feira, 1 de fevereiro de 2005 14:26
Assunto: geradores electricos de centrais nucleares


| Oi Professor. Eu sou um aluno portugues do ensino superior e tou c umas
| pequenas duvidas quanto a um trabalho que tenho de realizar...A minha duvida
| reside na transformacao de energia nuclear em energia eletrica, ou seja, a
| nivel do funcionamento do gerador electrico de uma central nuclear. Sera que
| o professor me podia ajudar? muito agradecido, Ricardo Ribeiro
|
| _________________________________________________________________
| MSN Busca: fácil, rápido, direto ao ponto. http://search.msn.com.br
|
|
|
| --
| Internal Virus Database is out-of-date.
| Checked by AVG Anti-Virus.
| Version: 7.0.300 / Virus Database: 265.6.13 - Release Date: 16/01/2005
|
|


--
Internal Virus Database is out-of-date.
Checked by AVG Anti-Virus.
Version: 7.0.300 / Virus Database: 265.6.13 - Release Date: 16/01/2005



SUBJECT: Zodiaco
FROM: "Esteban Moreno" <estebanmoreno@idhi.org.br>
TO: <ciencialist@yahoogrupos.com.br>
DATE: 03/02/2005 17:43

Segue mais um esclarecimento...
Esteban.


O Zodíaco usado pelos astrólogos é o Zodíaco do Signos ou "Zodíaco Tropical". Definido pelo ponto do equinócio de outono (projeção estelar do Sol no momento do equinócio de Outono no HS), e a divisão em 12 partes iguais de 30 graus do Equador Celeste (projeção do equador da Terra no fundo estelar). Cada uma destas partes corresponde a um signo astrológico. O primeiro signo é Áries, e os outros se sucedem em sentido horário, visto desde o hemisfério sul, segundo a ordem: Áries, Touro, Gêmeos, Câncer, Leão, Virgem, Libra, Escorpião, Sagitário, Capricórnio, Aquário e Peixes.

Assim, os "signos zodiacais" estão relacionados fisicamente com a Terra e o Sol (determinando o ponto de equinócio e as Estações da Natureza), com os planetas (determinando a amplitude da faixa zodiacal) e com a Lua (determinando a divisão por 12) e NADA tem a ver com o fundo estelar que só serve como referencia.

Os "Signos" são os canais e campos de manifestação da vida no planeta Terra, e tem uma expressão espacial segundo foi descrito (a forma mais fácil de definir-los é como as três modalidades ou dimensões dos quatro elementos: Fogo, Terra, Água e Ar). A amplitude dos signos pode ir alem da amplitude de 16 graus que define tradicionalmente a faixa zodiacal, definindo gomos que partem da Terra, tem por fronteiras a divisão em 12 do equador celeste e tem por fundo o infinito...

A definição das áreas das constelações realizadas por astrônomos (UIA-1929) correspondem a critérios estéticos e circunstanciais que nada tem a ver com Astrologia. A partir de diferentes desenhos de constelações dos antigos, eles escolheram as que melhor entenderam. Na atualidade esta divisão das constelações não é usada para nada, a não ser como referencia histórica especialmente da arte e da mitologia grega.

Devido ao movimento de precessão do eixo da terra, a projeção estelar do ponto de equinócio tem um movimento retrogrado na eclíptica. A própria eclíptica tem projeção estelar diferente, passando por diferentes constelações com o passar do tempo.

E para finalizar, convido aos astrônomos apavorados com a Astrologia, a serem menos pretensiosos e se informarem melhor para evitar falarem tantas besteiras!

Intentei mandar um e-mail para a revista Galileu, mas não encontrei o caminho.

Héctor Othón





[As partes desta mensagem que não continham texto foram removidas]



SUBJECT: Re: Zodiaco
FROM: "rmtakata" <rmtakata@altavista.net>
TO: ciencialist@yahoogrupos.com.br
DATE: 03/02/2005 18:15


--- Em ciencialist@yahoogrupos.com.br, "Esteban Moreno"
> Segue mais um esclarecimento...
> Esteban.
>
> E para finalizar, convido aos astrônomos apavorados com a
> Astrologia, a serem menos pretensiosos e se informarem melhor para
> evitar falarem tantas besteiras!

Verdade. Tb convidaria os astromantes a estudarem um pouco de
economia, medicina, geologia, psicologia e quejandos para fazerem seus
chutes. Assim ficaria melhor, se bem q., no caso, poderiamos jogar
fora as cartas natais e estudar elementos q. apresentem reais
correlacoes com o fato a ser previsto.

(Uma coisa no entanto eh interessante. Embora *hoje* se diga q., por
exemplo, a astromancia se baseia em setores - e os nomes sao apenas
uma curiosidade historica - as qualidades regidas pelos setores sao
eminentemente ligadas aos nomes dos formatos arbitrarios das
constelacoes q. estavam nos respectivos setores 'a milhares de anos:
por exemplo, ter um aspecto leonino significa ter um carater corajoso
e algo feroz, q. eh exatamente a ideia q. o leao traz, dito, o rei dos
animais; ter um aspecto de libra significa ser equilibrado, q. eh o q.
uma balanca representa - haja vista a balanca da justica; ser de
gemeos eh ter uma dualidade e assim por diante. Por uma incrivel
coincidencia, as caracteristicas influenciadas ou descritas por
determinados setores: ou configuracoes celestes em determinados
setores, seriam exatamente as mesmas das formas arbitrarias de
agrupamento de estrelas q. ocupavam tais setores no momento historico
em q. a astromancia tomou forma. Temos um caso aqui de uma construcao
argumentativa a posteriori. Eu diria q. se a astromancia tivesse se
baseado na divisao setorial por meio de constelacoes tupis, a
significacao das mesmissimas configuracoes celestes seriam totalmente
diferentes - e ainda assim milhoes de pessoas enxergariam espantosas
revelacoes.)

[]s,

Roberto Takata





SUBJECT: Re: Zodiaco
FROM: Maria Natália <grasdic@hotmail.com>
TO: ciencialist@yahoogrupos.com.br
DATE: 03/02/2005 20:23


Estebam:
O zodíaco dos astrólogos é o de há 2000 anos e não tem em conta os
movimentos do eixo da Terra. O zodíaco tem 13 constelações: a seguir
ao capricónio tem o Ofiuco...São 13 os tais signos. E agora que me
dizeis acerca do Ofiuco e que de acordo com o zodíaco actual é o meu
signo: poderei amanhã comprar lotaria para me sair prémio?
Desculpe, por acaso ouvi dizer que os astrónomos e astrofísicos falam
besteira? Ou entendi mal?
Mas vou parar pois aqui já sabem de minha posição acerca da astrologia
e não quero aquecer mais os ânimos.
Maria Natália


--- Em ciencialist@yahoogrupos.com.br, "Esteban Moreno"
<estebanmoreno@i...> escreveu
> Segue mais um esclarecimento...
> Esteban.
>
>
> O Zodíaco usado pelos astrólogos é o Zodíaco do Signos ou "Zodíaco
Tropical". Definido pelo ponto do equinócio de outono (projeção
estelar do Sol no momento do equinócio de Outono no HS), e a divisão
em 12 partes iguais de 30 graus do Equador Celeste (projeção do
equador da Terra no fundo estelar). Cada uma destas partes corresponde
a um signo astrológico. O primeiro signo é Áries, e os outros se
sucedem em sentido horário, visto desde o hemisfério sul, segundo a
ordem: Áries, Touro, Gêmeos, Câncer, Leão, Virgem, Libra, Escorpião,
Sagitário, Capricórnio, Aquário e Peixes.
>
> Assim, os "signos zodiacais" estão relacionados fisicamente com a
Terra e o Sol (determinando o ponto de equinócio e as Estações da
Natureza), com os planetas (determinando a amplitude da faixa
zodiacal) e com a Lua (determinando a divisão por 12) e NADA tem a ver
com o fundo estelar que só serve como referencia.
>
> Os "Signos" são os canais e campos de manifestação da vida no
planeta Terra, e tem uma expressão espacial segundo foi descrito (a
forma mais fácil de definir-los é como as três modalidades ou
dimensões dos quatro elementos: Fogo, Terra, Água e Ar). A amplitude
dos signos pode ir alem da amplitude de 16 graus que define
tradicionalmente a faixa zodiacal, definindo gomos que partem da
Terra, tem por fronteiras a divisão em 12 do equador celeste e tem por
fundo o infinito...
>
> A definição das áreas das constelações realizadas por astrônomos
(UIA-1929) correspondem a critérios estéticos e circunstanciais que
nada tem a ver com Astrologia. A partir de diferentes desenhos de
constelações dos antigos, eles escolheram as que melhor entenderam. Na
atualidade esta divisão das constelações não é usada para nada, a não
ser como referencia histórica especialmente da arte e da mitologia grega.
>
> Devido ao movimento de precessão do eixo da terra, a projeção
estelar do ponto de equinócio tem um movimento retrogrado na
eclíptica. A própria eclíptica tem projeção estelar diferente,
passando por diferentes constelações com o passar do tempo.
>
> E para finalizar, convido aos astrônomos apavorados com a
Astrologia, a serem menos pretensiosos e se informarem melhor para
evitar falarem tantas besteiras!
>
> Intentei mandar um e-mail para a revista Galileu, mas não encontrei
o caminho.
>
> Héctor Othón
>
>
>
>
>
> [As partes desta mensagem que não continham texto foram removidas]





SUBJECT: Re: [ciencialist] Re: Zodiaco
FROM: "Sergio M. M. Taborda" <sergiotaborda@terra.com.br>
TO: ciencialist@yahoogrupos.com.br
DATE: 03/02/2005 20:56

rmtakata wrote:

>
> (Uma coisa no entanto eh interessante. Embora *hoje* se diga q., por
> exemplo, a astromancia se baseia em setores - e os nomes sao apenas
> uma curiosidade historica - as qualidades regidas pelos setores sao
> eminentemente ligadas aos nomes dos formatos arbitrarios das
> constelacoes q. estavam nos respectivos setores 'a milhares de anos:

Essa é a sua opinião. Se vc olhar o ceu , sem saber as contelações -
como eu - você facilmente consegue agrupas a estrelas da forma canónica,
que vc não conseguirá será atribuir o mesmo simbolo. Por exemplo as
Ursas Maior e Menor , não se parecem em nada com ursos e mais com
frigideiras.
O facto das estrelas terem sido agrupadas como foram não espanta, o que
espanta é a sua conecção com simbolos.
Vc acha que a ligação é meramente arbitrária, mas eu podria descorrer
aqui, por não o é.

> por exemplo, ter um aspecto leonino significa ter um carater corajoso
> e algo feroz,

Não. Leão simboliza a coragem (coração+agem=acção do coração), mas não a
ferocidade.
O Leonino é bom por natureza, como um pai. Um pai não é bravo pq é
feroz, mas pq não é respeitado.
É algo bem diferente da ferocidade atribuida aos animais leão.

> q. eh exatamente a ideia q. o leao traz

Ora, mas é a ideia do 5º signo que tem como menomonica o leão, ou o
leão que empretou as suas caracteritsticas ao signo ?
É a primeira e não a segundo, como vc supoe.
Qualquer coisa poderia ser atruida à constelação de leão ,pq um leão ?
Ora, afinal aquilo é so um triagulo com uma ponta

> ter um aspecto de libra significa ser equilibrado,

Não. Significa indiciso. Que não sabe tomar parte, que não sabe decidir
entre as opções.

> q. eh o q.
> uma balanca representa

Não. A balança representa o acto de ponderar. Ela sempre começa
desiquilibrada e cabe a nós equilibrar os pratos para saber quando pesa
algo,
Mas para fazer isso vc precisa de um padrão, uma unidade de medida, e é
isso que os Librinianos não têm, e por isso não chegam a lado nenhum.
A justiça é representada por uma balança, mas não é libra o signo da
justiça pq se assim fosse o juiz nunca saberia decidir nada.

> - haja vista a balanca da justica;

Não são comparáveis.

> ser de
> gemeos eh ter uma dualidade e assim por diante.

Não é uma dualidade, é uma ambiguidade. Da mesma forma que balança não
sabe dicidir por uma opção,
gemeos não sabe dicidir por uma personaldiade.

> Por uma incrivel
> coincidencia, as caracteristicas influenciadas ou descritas por
> determinados setores: ou configuracoes celestes em determinados
> setores, seriam exatamente as mesmas das formas arbitrarias de
> agrupamento de estrelas q. ocupavam tais setores no momento historico
> em q. a astromancia tomou forma.

Compare com a astrologia Indu (a atrologia original onde se baseia a
ocidental actual). Nele os mesmos signos, com as mesms
exactas posições não têm nada a ver com as estrelas no sentido de que
não ha correspondencia entre os nomes das constrelçaões
e dos signos. Tb é verdade que aquilo que é atribuido a cada singo não
exactamente igual ao que a astrologia ocidental atriui, mas
não é assim tão diferente que se possa considerar a diferença. É mais
uma diferença filosofica
´É verdade que ha uma relação historica entre os nomes das constelações
e o significa dos signos q originalmente as enquadravam, mas
isso apenas na astrologia ocidental que é baseada na adptação pelos
gregos da astrologia indu.
Os nomes, as figuras, as hisotiras da mitologia, seus personagens e toda
a riquesa da mitologia grega é apenas uma grande menmonica para
a astrologia grega recem criada (/roubada/adptada) da indu.

> Temos um caso aqui de uma construcao
> argumentativa a posteriori.

Não, não temos. Vc acha que sim pq não conhece um pouco da historia da
astrologia.

E para não o deixar com a vontade eis como se forma o significado do
zodiaco:


O Pai sol (masculino=+) e a Mae lua (femenino =-) (os luminosos, luz
=vida) têm dois filhos Mercurio+ (gemeos) e Mercurio- (virgem).
Mercurio filho do sol e da lua é a comunicação entre os dois. OS pais
comunicam um com o outro atrave dos filhos, e assim eles aprendem a
conhecer o mundo. Mas esse conhecimento demasiado mental leva-os a não
aproveitarem tantas outras coisas do mundo onde os pais vivem. E todo o
pai quer que os filhos conhecam o seu mundo. Dois outros filhos (mais
novos) são venus+ (libra)e venus-(touro). Venus é a possibilidade de
olhar o mundo. (é por isso que libra é indiciso, pq se perde demasiado
com a fachada, a estética e esquece o resto). Repare que o signo de
touro é femenino, o que evidentemente não tem nada a ver com o seu
argumento pois a constelação não encaixa com o significado. A unica
coisa que podramos encaixar é a teimosia.
Os primogenitos são mentalmente activos e esquecem do mundo, os irmaos
são observadores do mundo e não das ideias. Os irmaos mais caçulas
marte-(escorpiao) e marte+ (aries) são aqueles que utilizam do mundo.
Que vão e pegam nas coisas. A sua actividade é de acção para com o mundo
e não mais mental ou contemplativa. Os irmaos mais novos que estes
jupiter+(sagistário) e jupiter-(peixe) passam além do material e
retornam à contemplação, mas desta vez do ponto de vista abstracto e não
mais do ponto de vista fisico, como libra e touro. Por isso que estes
signos são ligados à amaizade e à religião, conceitos que o mundo fisico
não pode entender. Estes são tb os primeiro singos da segunda metado do
zodiaco, que tecnicamente seria a noite, ou seja, o mundo psiquico,
enquanto a primeira metade, representa o mundo fisico, material, o dia.
Or proximo irmãoes são saturno+ (capricornio) e saturno-(aquário) que
vão além de tudo isto e desenvolvem meta-capacidades. Capacidades que
que representam a reflexão da coisa sobre si mesma. Como a meta-fisica é
a reflecção da fisica sobre si mesma. Esta capacidade os torna opostos
aos pais e querem estar um passo à frente dos pais, tornando-se criticos
e desafiadores das regras. Enquanto os outros são uma relção com o
mundo, estes observam as relações entre os irmaos. Pois sendo os mais
novos são os que podem observar a todos os outros toda a sua vida.

Embora se considere que o calendário astrologico comece com aries, a
formação de sinificado não começa por ele.
Voltando ao assunto, os primeiros signos são os ligados ao sol e à lua,
Poderiamos considerar que o leão é tã magnanimo com a selva como o pai
com a sua familia e desse ponto de vista seria um bom simbolo a atribuir
ao signo, até pq as estrelas nesses signo podem parecer um leão, com boa
vontade de quem olha. Mas e a lua, que simbolo podemos lhe atribuir ?
Não mais do que aquele que podemos obter olhando as estrelas. Afinal, o
que um carangueijo tem a ver com a lua ?

OS simbolos são meramente circusntaciais , são simbolos, menmonicas,
nada mais. Mas os significados, que é o que interessa, são baseados numa
logica solida, embora complexa para explicar por uma so prespecitva.

Sérgio Taborda









--
No virus found in this outgoing message.
Checked by AVG Anti-Virus.
Version: 7.0.300 / Virus Database: 265.8.5 - Release Date: 03-02-2005



SUBJECT: Re: [ciencialist] Re: Zodiaco
FROM: "Sergio M. M. Taborda" <sergiotaborda@terra.com.br>
TO: ciencialist@yahoogrupos.com.br
DATE: 03/02/2005 21:08

Maria Natália wrote:

>
> Estebam:
> O zodíaco dos astrólogos é o de há 2000 anos e não tem em conta os
> movimentos do eixo da Terra. O zodíaco tem 13 constelações: a seguir
> ao capricónio tem o Ofiuco...São 13 os tais signos. E agora que me
> dizeis acerca do Ofiuco e que de acordo com o zodíaco actual é o meu
> signo: poderei amanhã comprar lotaria para me sair prémio?

Eu ja respondi a essa pergunta. E a resposta é : Esse argumento faria
sentido SE a astrologia fosse baseada nas constelações.
Como não é , insistir nesse argumento é pura besteira (=coisas que os
bestas fazem)
Jà lhe expliquei pq so ha 12 signos, o que já foi aqui explicado por
outros. É uma razão matemática para que seja assim. É baseada no
conceito de Periodo Sinódico.
Ha uma logica , séria, matemática, fisica, _astronomica_ para o numero
12. Só que aos astronomos - com problemas de falta de atenção -
inventaram esse argumento sobre Ofucio.
fazer o quê ? Mas se vc acredita nesses mentirosos, não é diferente de
quem acredita em astrologos fajutos.

> Desculpe, por acaso ouvi dizer que os astrónomos e astrofísicos falam
> besteira? Ou entendi mal?

Ouvio bem. Eles dizem mesmo. E ha muitas mais do que a de Ofucio.


Sérgio Taborda



--
No virus found in this outgoing message.
Checked by AVG Anti-Virus.
Version: 7.0.300 / Virus Database: 265.8.5 - Release Date: 03-02-2005



SUBJECT: Plural das unidades
FROM: "Emiliano Chemello - Yahoo Grupos" <chemelloe@yahoo.com.br>
TO: <ciencialist@yahoogrupos.com.br>, <quimica-qaw@yahoogrupos.com.br>, <quimica@grupos.com.br>, <naeq-ucs@yahoogrupos.com.br>
DATE: 03/02/2005 21:11

Olá amigos,

Gostaria de confirmar uma suspeita de erro em uma questão de vestibular.
Não é o tipo de erro que promove a anulação da questão. Apenas um detalhe
que percebi ocorrer seguidamente em provas de química.

Abaixo, seguem as alternativas da questão:

a) 0,087 mols
b) 0,181 mols
c) 1,005 mols
d) 1,504 mols
e) 0,035 mols

Percebam que as unidades estão no plural. Discordo dessa escrita. Pela
regra, como os números são menores que '2', não devemos utilizar o plural
na unidade de medida. O uso do plural se daria com número maiores ou iguais
a 2. Por exemplo: 3,003 mols.

Estou correto?

"1 mol de [ ]'s " do

Emiliano Chemello
emiliano@quimica.net
http://www.quimica.net/emiliano
http://www.ucs.br/ccet/defq/naeq
[ MSN ] chemelloe@hotmail.com
[ ICQ ] 145060604

"Rien ne se perd, rien ne se crée, tout se transforme"
Lavoisier, químico francês (1743-1794)




SUBJECT: Mols e Moles
FROM: gisele campos <giscampos@yahoo.com.br>
TO: ciencialist@yahoogrupos.com.br
DATE: 03/02/2005 22:14

Olá amigos ,

Sou Gisele Campos , professora de química ,e estou iniciando meus contatos agora .
Realmente esta questão foi uma gafe como 0,500 moles , como já vi em um livro didático ?
E quando os alunos pensam que 0,5 é menor que 0,500 ( e insistem por causa dos zeros á direita!!.) É moles (rs) ou melhor é mole ???
Gisele

"...Carpe Dien ...."




Emiliano Chemello - Yahoo Grupos <chemelloe@yahoo.com.br> wrote:
Olá amigos,

Gostaria de confirmar uma suspeita de erro em uma questão de vestibular.
Não é o tipo de erro que promove a anulação da questão. Apenas um detalhe
que percebi ocorrer seguidamente em provas de química.

Abaixo, seguem as alternativas da questão:

a) 0,087 mols
b) 0,181 mols
c) 1,005 mols
d) 1,504 mols
e) 0,035 mols

Percebam que as unidades estão no plural. Discordo dessa escrita. Pela
regra, como os números são menores que '2', não devemos utilizar o plural
na unidade de medida. O uso do plural se daria com número maiores ou iguais
a 2. Por exemplo: 3,003 mols.

Estou correto?

"1 mol de [ ]'s " do

Emiliano Chemello
emiliano@quimica.net
http://www.quimica.net/emiliano
http://www.ucs.br/ccet/defq/naeq
[ MSN ] chemelloe@hotmail.com
[ ICQ ] 145060604

"Rien ne se perd, rien ne se crée, tout se transforme"
Lavoisier, químico francês (1743-1794)




##### ##### #####

Para saber mais visite
http://www.ciencialist.hpg.ig.com.br


##### ##### ##### #####


Yahoo! Grupos, um serviço oferecido por:



















function SearchComboBox() { if (document.form_combo.keyword.value.length==0){ alert("Por favor, digite algo."); return false; }else { document.form_combo.action ="http://br.rd.yahoo.com/SIG=12ar944bj/M=264105.3931087.6562589.1588051/D=brclubs/S=2137111528:HM/EXP=1107558804/A=2361264/R=0/SIG=11uaou2jn/*http://www.bondfaro.com/bondfaro/in/combosearch_in.jsp?sk=11"; } return true;} [input] [input] [input]

---------------------------------
Links do Yahoo! Grupos

Para visitar o site do seu grupo na web, acesse:
http://br.groups.yahoo.com/group/ciencialist/

Para sair deste grupo, envie um e-mail para:
ciencialist-unsubscribe@yahoogrupos.com.br

O uso que você faz do Yahoo! Grupos está sujeito aos Termos do Serviço do Yahoo!.




---------------------------------
Yahoo! Acesso Grátis - Internet rápida e grátis. Instale o discador do Yahoo! agora.

[As partes desta mensagem que não continham texto foram removidas]



SUBJECT: Re: Zodiaco
FROM: gisele campos <giscampos@yahoo.com.br>
TO: ciencialist@yahoogrupos.com.br
DATE: 03/02/2005 22:23

Olá ....

Não entendo bulhufas de astrologia,mas fazem dois anos que fiz meu mapa astral , ou esta coisa funciona , ou tem coincidência demais aí ....
Como pode ??
Bem , a descrição da perda de um pai e a substituição de um padrasto com características x ,y e z , é no mínimo estranho......
E a tal da sinastria então , parece brincadeira ......

Gisele




"Sergio M. M. Taborda" <sergiotaborda@terra.com.br> wrote:Maria Natália wrote:

>
> Estebam:
> O zodíaco dos astrólogos é o de há 2000 anos e não tem em conta os
> movimentos do eixo da Terra. O zodíaco tem 13 constelações: a seguir
> ao capricónio tem o Ofiuco...São 13 os tais signos. E agora que me
> dizeis acerca do Ofiuco e que de acordo com o zodíaco actual é o meu
> signo: poderei amanhã comprar lotaria para me sair prémio?

Eu ja respondi a essa pergunta. E a resposta é : Esse argumento faria
sentido SE a astrologia fosse baseada nas constelações.
Como não é , insistir nesse argumento é pura besteira (=coisas que os
bestas fazem)
Jà lhe expliquei pq so ha 12 signos, o que já foi aqui explicado por
outros. É uma razão matemática para que seja assim. É baseada no
conceito de Periodo Sinódico.
Ha uma logica , séria, matemática, fisica, _astronomica_ para o numero
12. Só que aos astronomos - com problemas de falta de atenção -
inventaram esse argumento sobre Ofucio.
fazer o quê ? Mas se vc acredita nesses mentirosos, não é diferente de
quem acredita em astrologos fajutos.

> Desculpe, por acaso ouvi dizer que os astrónomos e astrofísicos falam
> besteira? Ou entendi mal?

Ouvio bem. Eles dizem mesmo. E ha muitas mais do que a de Ofucio.


Sérgio Taborda



--
No virus found in this outgoing message.
Checked by AVG Anti-Virus.
Version: 7.0.300 / Virus Database: 265.8.5 - Release Date: 03-02-2005



##### ##### #####

Para saber mais visite
http://www.ciencialist.hpg.ig.com.br


##### ##### ##### #####


Yahoo! Grupos, um serviço oferecido por:



















function SearchComboBox() { if (document.form_combo.keyword.value.length==0){ alert("Por favor, digite algo."); return false; }else { document.form_combo.action ="http://br.rd.yahoo.com/SIG=12avidbm8/M=264105.3931087.6562589.1588051/D=brclubs/S=2137111528:HM/EXP=1107558491/A=2361264/R=0/SIG=11uaou2jn/*http://www.bondfaro.com/bondfaro/in/combosearch_in.jsp?sk=11"; } return true;} [input] [input] [input]

---------------------------------
Links do Yahoo! Grupos

Para visitar o site do seu grupo na web, acesse:
http://br.groups.yahoo.com/group/ciencialist/

Para sair deste grupo, envie um e-mail para:
ciencialist-unsubscribe@yahoogrupos.com.br

O uso que você faz do Yahoo! Grupos está sujeito aos Termos do Serviço do Yahoo!.




---------------------------------
Yahoo! Acesso Grátis - Internet rápida e grátis. Instale o discador do Yahoo! agora.

[As partes desta mensagem que não continham texto foram removidas]



SUBJECT: Adrenalina !!
FROM: gisele campos <giscampos@yahoo.com.br>
TO: ciencialist@yahoogrupos.com.br
DATE: 03/02/2005 22:37




Estive lendo uma revista esta semana e me deparei com um artigo falando sobre o Stress.Citam que no momento de medo , susto , etc, ocorre a liberação de adrenalina e cortisol no sangue (até áí tudo bem) , e também a diminuição de açúcar . Eu não sei se estou enganada , mas me esclareçam , na verdade, não ocorre o aumento do teor de açúcar ???

" Carpe Dien "



---------------------------------
Yahoo! Acesso Grátis - Internet rápida e grátis. Instale o discador do Yahoo! agora.

[As partes desta mensagem que não continham texto foram removidas]



SUBJECT: RE: [ciencialist] Plural das unidades
FROM: "Hugo Santos" <urano@netvisao.pt>
TO: <ciencialist@yahoogrupos.com.br>
DATE: 03/02/2005 23:26

Que seu saiba, pelo menos assim me foi dito, está convencionado (é óbvio que
isto é tudo uma questão de convenções) que as unidades não têm plural, seja
em que caso for.
Portanto, tanto é 0,5 mol como é 50 mol. Tal como se diz também 0,5 grama e
50 grama.
No entanto, as pessoas teimam em dizer erradamente as unidades no plural, o
que poderá fazer sentido em termos gramaticais, mas errado no sentido
científico.

Abraços,

Hugo Santos

> -----Original Message-----
> From: Emiliano Chemello - Yahoo Grupos [mailto:chemelloe@yahoo.com.br]
> Sent: quinta-feira, 3 de Fevereiro de 2005 23:12
> To: ciencialist@yahoogrupos.com.br; quimica-qaw@yahoogrupos.com.br;
> quimica@grupos.com.br; naeq-ucs@yahoogrupos.com.br
> Subject: [ciencialist] Plural das unidades
>
>
> Olá amigos,
>
> Gostaria de confirmar uma suspeita de erro em uma questão de
vestibular.
> Não é o tipo de erro que promove a anulação da questão. Apenas um detalhe
> que percebi ocorrer seguidamente em provas de química.
>
> Abaixo, seguem as alternativas da questão:
>
> a) 0,087 mols
> b) 0,181 mols
> c) 1,005 mols
> d) 1,504 mols
> e) 0,035 mols
>
> Percebam que as unidades estão no plural. Discordo dessa escrita. Pela
> regra, como os números são menores que '2', não devemos utilizar o plural
> na unidade de medida. O uso do plural se daria com número maiores ou
iguais
> a 2. Por exemplo: 3,003 mols.
>
> Estou correto?
>
> "1 mol de [ ]'s " do
>
> Emiliano Chemello
> emiliano@quimica.net
> http://www.quimica.net/emiliano
> http://www.ucs.br/ccet/defq/naeq
> [ MSN ] chemelloe@hotmail.com
> [ ICQ ] 145060604
>
> "Rien ne se perd, rien ne se crée, tout se transforme"
> Lavoisier, químico francês (1743-1794)



SUBJECT: Re: [ciencialist] Plural das unidades
FROM: "Luiz Ferraz Netto" <leobarretos@uol.com.br>
TO: <ciencialist@yahoogrupos.com.br>, <quimica-qaw@yahoogrupos.com.br>, <quimica@grupos.com.br>, <naeq-ucs@yahoogrupos.com.br>
DATE: 03/02/2005 23:49

10 CLS
===========================
Luiz Ferraz Netto [Léo]
leobarretos@uol.com.br
http://www.feiradeciencias.com.br
===========================
"Emiliano Chemello -

>Gostaria de confirmar uma suspeita de erro em uma questão de vestibular.
Não é o tipo de erro que promove a anulação da questão. Apenas um detalhe
que percebi ocorrer seguidamente em provas de química.

Abaixo, seguem as alternativas da questão:

a) 0,087 mols
b) 0,181 mols
c) 1,005 mols
d) 1,504 mols
e) 0,035 mols

Percebam que as unidades estão no plural. Discordo dessa escrita. Pela
regra, como os números são menores que '2', não devemos utilizar o plural
na unidade de medida. O uso do plural se daria com número maiores ou iguais
a 2. Por exemplo: 3,003 mols.

Estou correto? <

Léo: dir-se-ia que as alternativas dessa questão apresentam o 'nome da unidade' (e não o símbolo da unidade) "escritos por extenso". Assim, é válido o uso do plural; como exemplo:
1,5 newtons (unidade por extenso) ou 1,5N (símbolo da unidade) e não 1,5Ns (plural de símbolos não existem).

Agora, como se lê esses "1,5N" é outro mistério da língua matemática. Vejamos, 'escolha':
(a) um vírgula cinco ene
(b) um vírgula cinco newtons
(c) um newton e meio
(d) um e meio newtons
(e) um e meio ene
(f) um newton e cinco décimos
(g) um newton inteiro e cinco décimos de newtons
(h) um ...tempo... cinco newtons (...tempo... é a pausa na vírgula -- respiração --- é o sinal que impede as mulheres de morrerem asfixiadas quando falam)
(i) devemos escrever "1, 5N" (depois de vírgula não se deve dar um espaço?) e ler "um ... cincoene" (o S.I.U. diz que o símbolo da unidade deve vir 'depois do valor numérico' e sem qqer espaço --- 1,5N e não 1,5 N ---; sem exceção, portanto 1,5R$ e nunca 1,5 R$ ou, pior ainda, R$ 1,5 , com a unidade monetária precedendo o valor numérico).

E então?

[]'
20 GoTo 10





--
Internal Virus Database is out-of-date.
Checked by AVG Anti-Virus.
Version: 7.0.300 / Virus Database: 265.6.13 - Release Date: 16/01/2005



SUBJECT: Re: Plural das unidades
FROM: Manuel Bulcão <manuelbulcao@uol.com.br>
TO: ciencialist@yahoogrupos.com.br
DATE: 04/02/2005 01:31


--- Em ciencialist@yahoogrupos.com.br, "Emiliano Chemello - Yahoo
Grupos" <chemelloe@y...> escreveu
> Estou correto?

"Os números quebrados só podem levar o substantivo, que porventura
os acompanhe, para o plural se forem de dois em diante. Caso
contrário, fica no singular: 1,6 milhão, 0,7 metro, 1,8 grau, 0,9
porcentual. Isto é gramática 100%". (Gabriel Perissé, filólogo)

www.webwritersbrasil.com.br/dica_grama.asp

[]s
Manuel Bulcão





SUBJECT: Re: Plural das unidades
FROM: Manuel Bulcão <manuelbulcao@uol.com.br>
TO: ciencialist@yahoogrupos.com.br
DATE: 04/02/2005 01:53


--- Em ciencialist@yahoogrupos.com.br, "Hugo Santos" <urano@n...>
escreveu
> Portanto, tanto é 0,5 mol como é 50 mol. Tal como se diz também
0,5 grama e 50 grama.

Se não me engano, depende de como a unidade de medida está expressa:

50 gramas - 50g
10 quilômetros - 10km
8 horas - 8h
3 horas e quarenta minutos - 3h40 ou 3h40m

Abraços,
Manuel Bulcão





SUBJECT: Re: Zodiaco
FROM: Maria Natália <grasdic@hotmail.com>
TO: ciencialist@yahoogrupos.com.br
DATE: 04/02/2005 02:05


Sérgio:

Agradeço.
A astrologia se usa aqui para fazer estatísitica a brincar. Pega-se
num jornal e se vê qual o signo que diz " vais cair do escadote". Só o
professor sabe qual o jornal escolhido e todos respondem ao
questionário. Depois os que eram do signo da previsão "má" sofrem
comparação.Mas todos preenchem o inquérito. As perguntas também
permitem tirar conclusões sobre supertições, crenças e mitos. Quanto
mais turmas mais científico será (amostra maior)
Ao Ofiuco também se chama Serpentário, me lembrei agora.
Na escola costumamos apenas nos referirmos às constelações mais
visíveis em cidades com poluição luminosa. Ora se se souber que aqui
onde vivo da Ursa Menor apenas se vê a Polar e mesmo meia "safada" nem
capricónio nos interessa para meninos de 11 e 12 anos.
Quando os alunos fazem a "fita" do Zodíaco na parede da sala de aula,
cúbica ou paralelipipédica, se representam todas para se perceber essa
do "sol está na constelação..." sendo o Sol um aluno que anda junto às
paredes simulando o movimento aparente e a Terra os restantes alunos
num foco. De resto as constelações só começa a ter muito valor quando
queremos ver cometa ou asteróide
Não gostamos quando, a meio da noite, nos aparecem adultos a perguntar
se pelo facto de Vénus estar em Virgem eles vão ter mais sorte no
Totoloto.
Daí as alergias com a astrologia.
E este ano tive até um pai engenheiro a perguntar-me se achava que o
professor X iria faltar muito...O home estava a confundir astrónomo
com astrólogo. Aceitamos bem melhor sermos gastrólogos e gastrónomos.
Mas a astrologia dá para muita brincadeira. É que este pessoal que nos
chega à mão aos 11 anos já tem pelo menos 3 anos de laboratórios e
trabalhos de campo. Vem com a mania de apanhar a alma na ponta de um
bisturi. Vamos ver o que esta geração dará na universidade dentro de 4
anos. Pois se trata da reforma do ensino em curso.
Entretanto já lera o artigo do Galileu noutra lista, o CdA

Abraço
Maria Natália
PS Pois como agora estou a dar aulas longe de casa o tempo vai
falhando para acompanhar diariamente a lista...

--- Em ciencialist@yahoogrupos.com.br, "Sergio M. M. Taborda"
<sergiotaborda@t...> escreveu
> Maria Natália wrote:
>
> >
> > Estebam:
> > O zodíaco dos astrólogos é o de há 2000 anos e não tem em conta os
> > movimentos do eixo da Terra. O zodíaco tem 13 constelações: a seguir
> > ao capricónio tem o Ofiuco...São 13 os tais signos. E agora que me
> > dizeis acerca do Ofiuco e que de acordo com o zodíaco actual é o meu
> > signo: poderei amanhã comprar lotaria para me sair prémio?
>
> Eu ja respondi a essa pergunta. E a resposta é : Esse argumento faria
> sentido SE a astrologia fosse baseada nas constelações.
> Como não é , insistir nesse argumento é pura besteira (=coisas que os
> bestas fazem)
> Jà lhe expliquei pq so ha 12 signos, o que já foi aqui explicado por
> outros. É uma razão matemática para que seja assim. É baseada no
> conceito de Periodo Sinódico.
> Ha uma logica , séria, matemática, fisica, _astronomica_ para o numero
> 12. Só que aos astronomos - com problemas de falta de atenção -
> inventaram esse argumento sobre Ofucio.
> fazer o quê ? Mas se vc acredita nesses mentirosos, não é diferente de
> quem acredita em astrologos fajutos.
>
> > Desculpe, por acaso ouvi dizer que os astrónomos e astrofísicos falam
> > besteira? Ou entendi mal?
>
> Ouvio bem. Eles dizem mesmo. E ha muitas mais do que a de Ofucio.
>
>
> Sérgio Taborda
>
>
>
> --
> No virus found in this outgoing message.
> Checked by AVG Anti-Virus.
> Version: 7.0.300 / Virus Database: 265.8.5 - Release Date: 03-02-2005





SUBJECT: Re: [ciencialist] Plural das unidades
FROM: "Luiz Ferraz Netto" <leobarretos@uol.com.br>
TO: <ciencialist@yahoogrupos.com.br>
DATE: 04/02/2005 02:15

De: "Hugo Santos"

>Que seu saiba, pelo menos assim me foi dito, está convencionado (é óbvio que
isto é tudo uma questão de convenções) que as unidades não têm plural, seja
em que caso for.
Portanto, tanto é 0,5 mol como é 50 mol. Tal como se diz também 0,5 grama e
50 grama.
No entanto, as pessoas teimam em dizer erradamente as unidades no plural, o
que poderá fazer sentido em termos gramaticais, mas errado no sentido
científico.<

Léo: 0,5 mol, unidade escrita por extenso, por tremendo 'erro' do S.I. coincide com 0,5 mol (citação da unidade);
50 mol (citação da unidade), está correto; 50 mol (unidade por extenso, está errado --- deveria escrever 50 moles ou 50 mols.
Com a unidade de massa já não acontece isso porque o símbolo "g" é diferente do extenso "grama" [se bem que é outra cag__a do S.I. pois essa unidade, "não é a unidade" e sim o "kg" (que é um múltiplo do grama)]:
0,5 grama (extenso) e 0,5 g (símbolo)
50 gramas (extenso) e 50 g (símbolo) ---- o extenso tem flexão.

[]'
Léo


--
Internal Virus Database is out-of-date.
Checked by AVG Anti-Virus.
Version: 7.0.300 / Virus Database: 265.6.13 - Release Date: 16/01/2005



SUBJECT: Re: Mols e Moles e outras irregularidades**
FROM: Maria Natália <grasdic@hotmail.com>
TO: ciencialist@yahoogrupos.com.br
DATE: 04/02/2005 02:49


Gisela e Emiliano:
Aqui essa gafe das mols daria para estragar vida profissional da
comissão de exame de química...E haveria paizinho advogado que
tentaria impugnar esse exame.
Por aqui as unidades não têm plural quando estamos a escrevê-las no
problema e sendo símbolo n= 30 mol ou se pode escrever 30 moles. Mas a
escrever por extenso no enunciado será quinhentOS gramas e não
quinhentAs gramas ou quinhentas grama. É como o Kg e o kg (e tudo o
que é múltiplo quilo...) pois K é símbolo do kelvin. São os celsius em
vez do centígrados também outra batalha contra televiseiros.
Outra coisa interessante é as constantes de equílibrio K índice ainda
terem unidades para alguns. Esquecermo-nos que quando ensinamos
meninos até 11º ano não se devem por reacções químicas que não sejam
de 1ª ordem nos problems de equíbrio. Depois é a cacafonia de se
escrever as concentrações molares no equílíbrio com um e em índice.
Ora se é K índice c (de concentração molar) já se diz que as
concentrações são de equílibrio! Se estamos a falar de outra situação
se usa não o K mas o quociente de reacção e que pode ter para meninos
o símbolo Q ou como antigamente o "qui" (aquele símbolo que parece
quase um X maiusculo).
Também usar módulo de vector como sendo igual a norma desse vector é
disparate em física. Para o físico e que muito valor dá às unidades
escrever (norma do vector velocidade): ||v||= 3 m /s está mal; se deve
escrever: ||v||= 3 ou então
|v|= 3 m/s (em qualquer dos casos falta a seta de vector por cima do
v). Assim como escrever vector v =3 m/s porque vector não é uma
grandeza que se exprima só por um único valor. (sentido...)
Há ainda a mania de se ensinar que pH = -log da [...] e nos
esquecermos das condições em que deduzimos esta expressão
simplificada. Se trata de actividades logo deveremos sempre
salvaguardar estudo posterior em que menino irá perceber /ampliar o
conceito.Deve ser pH aproximadamente igual a menos log de... E dar o
pH sem se fazer referência á temperatura da solução tb é incorrecto e
ainda mais agora com o caminho dos planetas de sistema solar já a ser
desbravado.
Outra: quando se escreve a fórmula química do sulfato de cobre (II)
pentahidratado atenção ao ponto antes da molécula da água. Esse
pontinho não deve ficar no local de ponto final mas mais acima. Não
escreve Cu(SO4).5H2O. Põe o ponto for a da linha que une as "bases"
daos símbolos. Nem usa vírgula ou sinal +.
Nunca se devem colocar as equações químicas sem os componentes dos
sistemas não virem afectados dos símbolos de estado: HCl (aq)+ Zn(s)--->
e não HCl + Zn--->
Não é ser picuinhs mas ter rigor ao trabalhar com uma ciência que tem
linguagem própria e depois os testes escritos são nacionais e se
começam a fazer com um ano de antecedência. Já estamos a fazer os de
Junho/Julho de 2006.
Há muita coisa a ser ensinada assim á pressão e falada sem sequer nos
ouvirmos. Só que magister dixit e passou pra gíria.
As revistas que nos chama a atenção para estas pequenas coisas e que
deveriam estar na biblioteca de qualquer universidade deveriam ser a
ALAMBIQUE em espanhol ou a Chemical Education. Elas trazem destes
lembretes para químico apressado.
Quem se lembra de mais faltas de rigor?
Abraços
Maria Natália
** Elas não matam mas moem


--- Em ciencialist@yahoogrupos.com.br, gisele campos <giscampos@y...>
escreveu
> Olá amigos ,
>
> Sou Gisele Campos , professora de química ,e estou iniciando meus
contatos agora .
> Realmente esta questão foi uma gafe como 0,500 moles , como já vi
em um livro didático ?
> E quando os alunos pensam que 0,5 é menor que 0,500 ( e insistem
por causa dos zeros á direita!!.) É moles (rs) ou melhor é mole ???
> Gisele
>
> "...Carpe Dien ...."
>
>
>
>
> Emiliano Chemello - Yahoo Grupos <chemelloe@y...> wrote:
> Olá amigos,
>
> Gostaria de confirmar uma suspeita de erro em uma questão de
vestibular.
> Não é o tipo de erro que promove a anulação da questão. Apenas um
detalhe
> que percebi ocorrer seguidamente em provas de química.
>
> Abaixo, seguem as alternativas da questão:
>
> a) 0,087 mols
> b) 0,181 mols
> c) 1,005 mols
> d) 1,504 mols
> e) 0,035 mols
>
> Percebam que as unidades estão no plural. Discordo dessa
escrita. Pela
> regra, como os números são menores que '2', não devemos utilizar o
plural
> na unidade de medida. O uso do plural se daria com número maiores ou
iguais
> a 2. Por exemplo: 3,003 mols.
>
> Estou correto?
>
> "1 mol de [ ]'s " do
>
> Emiliano Chemello
> emiliano@q...
> http://www.quimica.net/emiliano
> http://www.ucs.br/ccet/defq/naeq
> [ MSN ] chemelloe@h...
> [ ICQ ] 145060604
>
> "Rien ne se perd, rien ne se crée, tout se transforme"
> Lavoisier, químico francês (1743-1794)
>
>
>
>
> ##### ##### #####
>
> Para saber mais visite
> http://www.ciencialist.hpg.ig.com.br
>
>
> ##### ##### ##### #####
>
>
> Yahoo! Grupos, um serviço oferecido por:
>
>
>
>
>
>
>
>
>
>
>
>
>
>
>
>
>
>
>
> function SearchComboBox() {
if (document.form_combo.keyword.value.length==0){ alert("Por favor,
digite algo."); return false; }else { document.form_combo.action
="http://br.rd.yahoo.com/SIG=12ar944bj/M=264105.3931087.6562589.1588051/D=brclubs/S=2137111528:HM/EXP=1107558804/A=2361264/R=0/SIG=11uaou2jn/*http://www.bondfaro.com/bondfaro/in/combosearch_in.jsp?sk=11";
} return true;} [input] [input] [input]
>
> ---------------------------------
> Links do Yahoo! Grupos
>
> Para visitar o site do seu grupo na web, acesse:
> http://br.groups.yahoo.com/group/ciencialist/
>
> Para sair deste grupo, envie um e-mail para:
> ciencialist-unsubscribe@yahoogrupos.com.br
>
> O uso que você faz do Yahoo! Grupos está sujeito aos Termos do
Serviço do Yahoo!.
>
>
>
>
> ---------------------------------
> Yahoo! Acesso Grátis - Internet rápida e grátis. Instale o discador
do Yahoo! agora.
>
> [As partes desta mensagem que não continham texto foram removidas]





SUBJECT: espumas de pororoca
FROM: "E m i l i a n o C h e m e l l o" <chemelloe@yahoo.com.br>
TO: <ciencialist@yahoogrupos.com.br>, <naeq-ucs@yahoogrupos.com.br>, <quimica-qaw@yahoogrupos.com.br>, <quimica@grupos.com.br>
DATE: 04/02/2005 08:20

eheheh... nem fiz doutorado e sou chamado de doutor. Fora a brincadeira,
achei interessante a pergunta do consulente. Alguma contribuição?

[ ] 's do Emiliano Chemello
emiliano@quimica.net
http://www.quimica.net/emiliano
http://www.ucs.br/ccet/defq/naeq

---

Prezado Dr Emiliano:

Estou inconformado com a formação de tanta espuma em nossos rio poluídos.
Por gentileza, na sua opinião o que causa a formação dessa espuma?
Existe algum estudo a respeito?
Veja no site http://www.webcentral.com.br/pirapora/ onde estão publicados as
fotos do que aconteceu em 2003 na cidade religiosa de Pirapora. Obs: mesmo
que as janelas apresentem um (x), favor clicar sobre elas que as fotos
aparecem.
Gostaria de ter a honra de discutir este assunto. Muito obrigado.
Att. Massao eng. civil - voluntário sócioambiental
Presid. da CT-RHN do Comdema- Cons. de defesa do meio ambiente de Jundiaí




SUBJECT: Re: Zodiaco
FROM: "rmtakata" <rmtakata@altavista.net>
TO: ciencialist@yahoogrupos.com.br
DATE: 04/02/2005 08:44


--- Em ciencialist@yahoogrupos.com.br, "Sergio M. M. Taborda"
> Essa é a sua opinião. Se vc olhar o ceu , sem saber as contelações -

Sim, minha opiniao. Bem, de certa forma nao se parece, mas eh o q.
pareceu aos olhos dos antigos. (Eu consigo ver mais ou menos bem a
forma de um escorpiao na constelacao de Escorpiao - sim, pode ser
sugestionamento, mas enfim. Se compararmos o agrupamento feito pelos
babilonicos - ou antes deles - com o dos amerindios notaremos uma
grande diferenca.)

> Vc acha que a ligação é meramente arbitrária, mas eu podria
> descorrer aqui, por não o é.

Nao duvido nem um pouco disso. Os astromantes no minimo falariam de
sincronicidade.

> Não. Leão simboliza a coragem (coração+agem=acção do coração), mas
> não a ferocidade.

Ferocidade nao quer dizer crueldade. Um dos manuais astromantes diz: o
leonino eh corajoso, justo e digno. Mas claro q. apresenta tb aspectos
negativos - por curiosidade sao aproximadamente antagonicos aos
aspectos positivos, de modo q. praticamente qq pessoa possa se
encaixar na descricao generica: orgulhoso, autoritario, megalomano.

> Ora, mas é a ideia do 5º signo que tem como menomonica o leão, ou o
> leão que empretou as suas caracteritsticas ao signo ?
> É a primeira e não a segundo, como vc supoe.
> Qualquer coisa poderia ser atruida à constelação de leão ,pq um
> leão?
> Ora, afinal aquilo é so um triagulo com uma ponta

Eles enxergaram um leao. Sim, poderiam ter enxergado outra coisa, mas
se assim o fizesse, teriam atribuido ao setor outra coisa. (Na verdade
eh um trapezoide deformado com um gancho no lugar da cabeca - segundo
a forma q. eles ligaram.)

> > ter um aspecto de libra significa ser equilibrado,
>
> Não. Significa indiciso. Que não sabe tomar parte, que não sabe
> decidir entre as opções.

Isso significa o aspecto negativo. No aspecto positivo significa
ponderação. Contraditorio? Sem duvida. Isso eh muito bom para a
astromancia, jah q. pode encaixar tto os librianos equilibrados, qto
os indecisos.

> Não. A balança representa o acto de ponderar. Ela sempre começa
> desiquilibrada e cabe a nós equilibrar os pratos para saber quando

Entao tah...

> Compare com a astrologia Indu (a atrologia original onde se baseia a
> ocidental actual). Nele os mesmos signos, com as mesms
> exactas posições não têm nada a ver com as estrelas no sentido de
> que não ha correspondencia entre os nomes das constrelçaões

Lamento, mas a astromancia hindu nao eh uma astromancia independente
da astromancia ocidental. Bebem de uma fonte comum e houve intensa
comunicacao. Tto eh q. dados astronomicos hindus - junto com a
matematica - vieram para o ocidente e vice-versa.

> > Temos um caso aqui de uma construcao
> > argumentativa a posteriori.
>
> Não, não temos. Vc acha que sim pq não conhece um pouco da historia
> da astrologia.

Sua exposicao de defesa nao eh convincente. A mesma coisa acontece com
os planetas. O planeta vermelho acabou sendo ligado 'a guerra e essas
coisas - assim Marte - e antes, seus equivalentes babilonios, egipcios
e demais - acabou por reger as relacoes masculinas, de forca e coisas
assim. Como Venus - ou seus equivalentes babilonicos - foi
identificada com uma divindade feminina, ganhou suposta regencia sobre
atributos femininos. E assim por diante. Trata-se de uma construcao
argumentativa a posteriori dizer q. o q. importa sao os setores e os
nomes dos setores sao apenas um detalhe historico (ou sao efeitos e
nao causas das qualidades atribuidas a cada setor e outros elementos
de configuracao na carta natal).

Mais recentemente, em termos historicos, vimos acontecer isso com
Plutao: como eh o nome do deus romando dos mortos, ganhou regencia
sobre aspectos relacionados a isso: forças ocultas e quetais.

[]s,

Roberto Takata





SUBJECT: Plural das unidades (esclarecimento)
FROM: "E m i l i a n o C h e m e l l o" <chemelloe@yahoo.com.br>
TO: <naeq-ucs@yahoogrupos.com.br>, <ciencialist@yahoogrupos.com.br>, <quimica-qaw@yahoogrupos.com.br>, <quimica@grupos.com.br>
CC: Áttico Chassot <achassot@portoweb.com.br>
DATE: 04/02/2005 09:22

Olá Pessoal

Sim. O tema é polêmico. Farei alguns balizamentos iniciais para
fundamentarmos nossa discussão.

***** O que é mol? *****

O mol é a quantidade de matéria de um sistema contendo tantas entidades
elementares quantos átomos existem em 0,012 quilograma de carbono 12. Quando
se utiliza o mol, as entidades elementares devem ser especificadas, podendo
ser átomos, moléculas, íons, elétrons, assim como outras partículas, ou
agrupamentos especificados em tais partículas.

Veja mais em: http://www.inmetro.gov.br/infotec/publicacoes/Si.pdf

***** Estamos falando do símbolo ou da unidade? *****

Talvez este seja o detalhe que promove a confusão quando discutimos como
devemos escrever o 'mol'. Se estivermos fazendo referência ao *símbolo* da
unidade de medida 'mol', então este não será escrito no plural, mas sempre
no singular da mesma forma que outras unidades, como por exemplo:

"metro" é o nome da unidade utilizada para mensurar a grandeza
"comprimento". Seu símbolo é "m"

3 m e *não* 3 ms
Se escrevermos o nome da unidade, teremos
"três metros "

---

"newton" é o nome da unidade utilizada par mensurar a grandeza "força". Seu
símbolo é "N"

4 N e *não* 4 Ns
Se escrevermos o nome da unidade, teremos
"quatro newtons"

Perceba que quando estamos escrevendo o nome da unidade, ai sim há a
possibilidade de utilizarmos tanto o singular como o plural. A minha questão
é justamente quando utilizamos um ou outro. Veja um exemplo ortodoxo:

1 laranja
2 laranjas
3 laranjas

1 mol
2 mols
3 mols

Se tivermos 1 laranja e mais meia laranja, não podemos dizer 1,5
laranjas, certo?

Veja mais em: http://www.inmetro.gov.br/consumidor/unidLegaisMed.asp

*** Nome da unidade e seu símbolo ****
O nome da unidade que mensurar a quantidade de matéria de um sistema é o
'mol'. O simbolo da unidade 'mol' é 'mol'. Quando estamos fazendo referência
a 'unidade', esta admite plural. Quando estamos nos referindo ao 'símbolo',
este não admite plural.
******

Esta é minha dúvida: podemos encarar o mol como laranjas e colocar a
*escrita da unidade* no plural quando temos um número maior ou igual a 2?

[ ] 's do Emiliano Chemello
emiliano@quimica.net
http://www.quimica.net/emiliano
http://www.ucs.br/ccet/defq/naeq

" Rien ne se perd, rien ne se crée,
tout se transforme."

Antoine Laurent de Lavoisier (químico francês, 1743 - 1794)




SUBJECT: Re: [ciencialist] Re: Plural das unidades
FROM: "E m i l i a n o C h e m e l l o" <chemelloe@yahoo.com.br>
TO: <ciencialist@yahoogrupos.com.br>, <naeq-ucs@yahoogrupos.com.br>, <quimica-qaw@yahoogrupos.com.br>, <quimica@grupos.com.br>
CC: Áttico Chassot <achassot@portoweb.com.br>
DATE: 04/02/2005 09:27

Olá Manuel,

[Manuel citou]
"Os números quebrados só podem levar o substantivo, que porventura os
acompanhe, para o plural se forem de dois em diante. Caso contrário, fica no
singular: 1,6 milhão, 0,7 metro, 1,8 grau, 0,9 porcentual. Isto é gramática
100%". (Gabriel Perissé, filólogo)

www.webwritersbrasil.com.br/dica_grama.asp

[Emiliano]
Você matou a charada (crime doloso, eheheh). É exatamente isso que eu
pensava. Obrigado

[ ] 's do Emiliano Chemello
emiliano@quimica.net
www.quimica.net/emiliano

----- Original Message -----
From: Manuel Bulcão
To: ciencialist@yahoogrupos.com.br
Sent: Friday, February 04, 2005 1:31 AM
Subject: [ciencialist] Re: Plural das unidades


--- Em ciencialist@yahoogrupos.com.br, "Emiliano Chemello - Yahoo
Grupos" <chemelloe@y...> escreveu
> Estou correto?

"Os números quebrados só podem levar o substantivo, que porventura
os acompanhe, para o plural se forem de dois em diante. Caso
contrário, fica no singular: 1,6 milhão, 0,7 metro, 1,8 grau, 0,9
porcentual. Isto é gramática 100%". (Gabriel Perissé, filólogo)

www.webwritersbrasil.com.br/dica_grama.asp

[]s
Manuel Bulcão




SUBJECT: Re: espumas de pororoca
FROM: "rmtakata" <rmtakata@altavista.net>
TO: ciencialist@yahoogrupos.com.br
DATE: 04/02/2005 09:29


Pororoca ou Pirapora?

A espuma eh causada pela presenca de saponaceos e detergentes diluidos
na agua e pela materia organica (afinal eh esgoto domestico na maior
parte) - com o turbilhonamento causado pela queda da agua em barragens
e cachoeiras formam-se as bolhas. Em epoca de estiagem o problema se
agrava com a evaporacao da agua e concentracao dos produtos.

[]s,

Roberto Takata

--- Em ciencialist@yahoogrupos.com.br, "E m i l i a n o C h e m e l
> Prezado Dr Emiliano:
> Por gentileza, na sua opinião o que causa a formação dessa espuma?






SUBJECT: Re: Zodiaco
FROM: "rmtakata" <rmtakata@altavista.net>
TO: ciencialist@yahoogrupos.com.br
DATE: 04/02/2005 09:48


--- Em ciencialist@yahoogrupos.com.br, gisele campos <giscampos@y...>
> Como pode ??

Leitura fria, por exemplo.

[]s,

Roberto Takata





SUBJECT: Re: [ciencialist] Re: Zodiaco
FROM: "Esteban Moreno" <estebanmoreno@idhi.org.br>
TO: <ciencialist@yahoogrupos.com.br>
DATE: 04/02/2005 10:18

Cuidado, Gisele, podes ser escomungada deste fórum com este tipo de depoimento!
Benza Deus! ;-)
Esteban.


Olá ....

Não entendo bulhufas de astrologia,mas fazem dois anos que fiz meu mapa astral , ou esta coisa funciona , ou tem coincidência demais aí ....
Como pode ??
Bem , a descrição da perda de um pai e a substituição de um padrasto com características x ,y e z , é no mínimo estranho......
E a tal da sinastria então , parece brincadeira ......

Gisele




"Sergio M. M. Taborda" <sergiotaborda@terra.com.br> wrote:Maria Natália wrote:

>
> Estebam:
> O zodíaco dos astrólogos é o de há 2000 anos e não tem em conta os
> movimentos do eixo da Terra. O zodíaco tem 13 constelações: a seguir
> ao capricónio tem o Ofiuco...São 13 os tais signos. E agora que me
> dizeis acerca do Ofiuco e que de acordo com o zodíaco actual é o meu
> signo: poderei amanhã comprar lotaria para me sair prémio?

Eu ja respondi a essa pergunta. E a resposta é : Esse argumento faria
sentido SE a astrologia fosse baseada nas constelações.
Como não é , insistir nesse argumento é pura besteira (=coisas que os
bestas fazem)
Jà lhe expliquei pq so ha 12 signos, o que já foi aqui explicado por
outros. É uma razão matemática para que seja assim. É baseada no
conceito de Periodo Sinódico.
Ha uma logica , séria, matemática, fisica, _astronomica_ para o numero
12. Só que aos astronomos - com problemas de falta de atenção -
inventaram esse argumento sobre Ofucio.
fazer o quê ? Mas se vc acredita nesses mentirosos, não é diferente de
quem acredita em astrologos fajutos.

> Desculpe, por acaso ouvi dizer que os astrónomos e astrofísicos falam
> besteira? Ou entendi mal?

Ouvio bem. Eles dizem mesmo. E ha muitas mais do que a de Ofucio.


Sérgio Taborda



--
No virus found in this outgoing message.
Checked by AVG Anti-Virus.
Version: 7.0.300 / Virus Database: 265.8.5 - Release Date: 03-02-2005



##### ##### #####

Para saber mais visite
http://www.ciencialist.hpg.ig.com.br


##### ##### ##### #####


Yahoo! Grupos, um serviço oferecido por:



















function SearchComboBox() { if (document.form_combo.keyword.value.length==0){ alert("Por favor, digite algo."); return false; }else { document.form_combo.action ="http://br.rd.yahoo.com/SIG=12avidbm8/M=264105.3931087.6562589.1588051/D=brclubs/S=2137111528:HM/EXP=1107558491/A=2361264/R=0/SIG=11uaou2jn/*http://www.bondfaro.com/bondfaro/in/combosearch_in.jsp?sk=11"; } return true;} [input] [input] [input]

---------------------------------
Links do Yahoo! Grupos

Para visitar o site do seu grupo na web, acesse:
http://br.groups.yahoo.com/group/ciencialist/

Para sair deste grupo, envie um e-mail para:
ciencialist-unsubscribe@yahoogrupos.com.br

O uso que você faz do Yahoo! Grupos está sujeito aos Termos do Serviço do Yahoo!.




---------------------------------
Yahoo! Acesso Grátis - Internet rápida e grátis. Instale o discador do Yahoo! agora.

[As partes desta mensagem que não continham texto foram removidas]



##### ##### #####

Para saber mais visite
http://www.ciencialist.hpg.ig.com.br


##### ##### ##### #####


Yahoo! Grupos, um serviço oferecido por:

São Paulo Rio de Janeiro Curitiba Porto Alegre Belo Horizonte Brasília




------------------------------------------------------------------------------
Links do Yahoo! Grupos

a.. Para visitar o site do seu grupo na web, acesse:
http://br.groups.yahoo.com/group/ciencialist/

b.. Para sair deste grupo, envie um e-mail para:
ciencialist-unsubscribe@yahoogrupos.com.br

c.. O uso que você faz do Yahoo! Grupos está sujeito aos Termos do Serviço do Yahoo!.



[As partes desta mensagem que não continham texto foram removidas]



SUBJECT: Re: Zodiaco
FROM: "rmtakata" <rmtakata@altavista.net>
TO: ciencialist@yahoogrupos.com.br
DATE: 04/02/2005 10:50


--- Em ciencialist@yahoogrupos.com.br, "Esteban Moreno"
> E no caso da sinastria, como saberia sobre o marido?

Tb poderia ser por leitura fria. Ha' varias pequenas tecnicas q.
funcionam para causar boa impressao nas pessoas - nao apenas no
inocentes bobocas, mas mesmo em pessoas muito bem dotadas
intelectualmente.

Somos humanos e impressionaveis. O gde porem da astromancia eh q. qdo
devidamente testada, ela nao parece funcionar como se alega funcionar.

Se pegarmos descricoes de caracteristicas de personalidade baseadas
tao somente nas cartas natais das pessoas (sem o contato direto entre
o astromante e a pessoa analisada) e dermos a outra pessoa q. teria
uma caracteristica supostamente diferente, essa outra pessoa dara' um
bom indice de identificacao com a descricao.

Um pequeno macete eh o uso de caracteristicas gerais - como humanos
apresentamos grandes semelhancas gerais: a maioria dos humanos
apresentam boa indole, e de vez em qdo saem do serio. Dizer algo como:
"vc eh uma pessoa q. preza a honestidade, mas `as vezes perde a
paciencia" tem altas chances de ser identificado como uma descricao
correta. Outro eh o uso de caracteristicas contraditorias: "vc eh uma
pessoa honesta... 'as vezes cede 'a tentacao". Acerta-se entre os
honestos, os nao tao honestos e a grande parte das pessoas q.
apresentam caracteristicas contraditorias.

[]s,

Roberto Takata





SUBJECT: Re: [ciencialist] Re: Zodiaco
FROM: "Silvio" <scordeiro@terra.com.br>
TO: <ciencialist@yahoogrupos.com.br>
DATE: 04/02/2005 10:56

Gisele:

surpreso pelo surrealista papo sobre astrologia no ciencialist, faço-lhe,
data venia, uma pergunta:

Você morou em París??? Será a mesma???

um harmônico fim de semana,

silvio.

xxxxxxxxxxxxxx
"gisele campos" wrote:

Olá ....

Não entendo bulhufas de astrologia,mas fazem dois anos que fiz meu mapa
astral , ou esta coisa funciona , ou tem coincidência demais aí ....
Como pode ??
Bem , a descrição da perda de um pai e a substituição de um padrasto com
características x ,y e z , é no mínimo estranho......
E a tal da sinastria então , parece brincadeira ......

Gisele




"Sergio M. M. Taborda" <sergiotaborda@terra.com.br> wrote:Maria Natália
wrote:

>
> Estebam:
> O zodíaco dos astrólogos é o de há 2000 anos e não tem em conta os
> movimentos do eixo da Terra. O zodíaco tem 13 constelações: a seguir
> ao capricónio tem o Ofiuco...São 13 os tais signos. E agora que me
> dizeis acerca do Ofiuco e que de acordo com o zodíaco actual é o meu
> signo: poderei amanhã comprar lotaria para me sair prémio?

Eu ja respondi a essa pergunta. E a resposta é : Esse argumento faria
sentido SE a astrologia fosse baseada nas constelações.
Como não é , insistir nesse argumento é pura besteira (=coisas que os
bestas fazem)
Jà lhe expliquei pq so ha 12 signos, o que já foi aqui explicado por
outros. É uma razão matemática para que seja assim. É baseada no
conceito de Periodo Sinódico.
Ha uma logica , séria, matemática, fisica, _astronomica_ para o numero
12. Só que aos astronomos - com problemas de falta de atenção -
inventaram esse argumento sobre Ofucio.
fazer o quê ? Mas se vc acredita nesses mentirosos, não é diferente de
quem acredita em astrologos fajutos.

> Desculpe, por acaso ouvi dizer que os astrónomos e astrofísicos falam
> besteira? Ou entendi mal?

Ouvio bem. Eles dizem mesmo. E ha muitas mais do que a de Ofucio.


Sérgio Taborda



--
No virus found in this outgoing message.
Checked by AVG Anti-Virus.
Version: 7.0.300 / Virus Database: 265.8.5 - Release Date: 03-02-2005



##### ##### #####

Para saber mais visite
http://www.ciencialist.hpg.ig.com.br


##### ##### ##### #####


Yahoo! Grupos, um serviço oferecido por:



















function SearchComboBox() { if
(document.form_combo.keyword.value.length==0){ alert("Por favor, digite
algo."); return false; }else { document.form_combo.action
="http://br.rd.yahoo.com/SIG=12avidbm8/M=264105.3931087.6562589.1588051/D=brclubs/S=2137111528:HM/EXP=1107558491/A=2361264/R=0/SIG=11uaou2jn/*http://www.bondfaro.com/bondfaro/in/combosearch_in.jsp?sk=11";
} return true;} [input] [input] [input]

---------------------------------
Links do Yahoo! Grupos

Para visitar o site do seu grupo na web, acesse:
http://br.groups.yahoo.com/group/ciencialist/

Para sair deste grupo, envie um e-mail para:
ciencialist-unsubscribe@yahoogrupos.com.br

O uso que você faz do Yahoo! Grupos está sujeito aos Termos do Serviço do
Yahoo!.




---------------------------------
Yahoo! Acesso Grátis - Internet rápida e grátis. Instale o discador do
Yahoo! agora.

[As partes desta mensagem que não continham texto foram removidas]



##### ##### #####

Para saber mais visite
http://www.ciencialist.hpg.ig.com.br


##### ##### ##### #####
Links do Yahoo! Grupos












SUBJECT: Desgelo na Antártida
FROM: José Renato <jrma@terra.com.br>
TO: <ciencialist@yahoogrupos.com.br>
DATE: 04/02/2005 11:06

No popular: Quando o ser humano maltrata a natureza pode esperar o revide. E os EUA, que lançam 36% do total das emissões de dióxido de carbono na atmosfera, se negam a assinar o Protocolo de Kyoto.
[]s
José Renato
.....................................................

[ O derretimento da densa camada de gelo que cobre a parte ocidental da Antártida pode elevar em quase 4,9 metros o nível dos oceanos, segundo os prognósticos de uma equipe de cientistas britânicos. [ ... ]
Na mesma conferência, cientistas israelenses previram também o rápido desaparecimento dos recifes de corais conforme for aumentando a acidez dos mares pela absorção do dióxido de carbono da atmosfera. ]
............................................

Notícias Terra
Ciência e Meio Ambiente
Quarta, 2 de fevereiro de 2005, 09h53
Degelo da Antártida pode elevar nível dos oceanos

Os especialistas, do British Antartic Survey (BAS), sediado em Cambridge (Reino Unido), descobriram que estas massas de gelo, que até agora eram consideradas estáveis, podem começar a desintegrar-se, segundo foi explicado em uma conferência internacional sobre o clima que acontece desde terça-feira na cidade inglesa de Exeter.

A equipe do BAS, que mediu a densidade da camada de gelo, chegou à conclusão de que, anualmente, 250 metros cúbicos de água solidificada estão se soltando e caindo no mar, o que já está elevando o nível dos mares em 0,2 milímetro por ano.

O professor Chris Rapley, diretor do British Antartic Survey, declarou aos especialistas reunidos em Exeter que esta descoberta tinha alertado a comunidade científica sobre o perigo que o fenômeno representa para todo o planeta.

Há quatro anos, no último relatório do grupo intergovernamental das Nações Unidas sobre mudança climática, foram desprezados os sinais de alarme neste sentido.

No relatório da ONU, a Antártida era apresentada como um "um gigante adormecido em termos de mudança climática". "Mas eu diria que o gigante despertou. Estamos muito preocupados", afirmou Rapley aos seus colegas.

O cientista britânico acrescentou que hoje não é possível compartilhar do otimismo de estudos anteriores, segundo os quais não havia perigo de a camada de gelo da Antártida se desintegrar antes de 2.100.

O eventual colapso destas camadas de gelo constituiria um desastre de proporções gigantescas, já que inundaria enormes áreas costeiras de países em desenvolvimento e desenvolvidos.

A conferência de Exeter é parte dos esforços do governo do Reino Unido para chamar a atenção do mundo para a questão da mudança climática, enquanto preside durante este ano o G-8 (os países mais ricos e a Rússia).

O primeiro-ministro britânico, Tony Blair, pediu aos cientistas reunidos na cidade britânica que tentem determinar a partir de que momento a mudança climática ao qual assistimos começará a ter conseqüências catastróficas tanto para as sociedades do planeta como para os ecossistemas.

Ontem, foram apresentados vários relatórios na reunião, entre eles um que falava do possível impacto da mudança climática na corrente do Golfo. No entanto, o que causou mais alarme foi exatamente o relacionado ao desgelo da Antártida.

Este continente encontra-se coberto por uma espessa camada de gelo, que é muito estável em sua parte oriental devido a um maciço rochoso mais elevado sobre o qual repousa.

Os cientistas começaram a preocupar-se com o derretimento do gelo polar há mais de 25 anos, quando descobriram que as rochas que lhe servem de base estão em boa parte muito abaixo do nível do mar.

Alguns especialistas chegaram então à conclusão de que, em determinadas circunstâncias, a água do oceano poderia ir escavando essa massa gelada, que terminaria se desintegrando.

Sobre as conclusões relativamente tranqüilizadoras a respeito do relatório do Painel Intergovernamental sobre a Mudança Climática de quatro anos atrás, os cientistas britânicos detectaram o rápido desgelo no mar de Amundsen a partir de três zonas, Thwaites, a ilha dos Pinheiros e uma outra não mencionada. "Nossa descoberta reabre o debate" sobre a ameaça que supõe o desgelo da Antártida, afirmou o diretor do BAS.

Na mesma conferência, cientistas israelenses previram também o rápido desaparecimento dos recifes de corais conforme for aumentando a acidez dos mares pela absorção do dióxido de carbono da atmosfera.

Os oceanos absorvem aproximadamente 48% das emissões de CO2 lançadas pelo homem, o que retarda o efeito estufa, mas ao mesmo tempo aumenta a acidez das águas oceânicas, e isto representa um perigo para os corais e outros organismos.

EFE

Agência Efe - Todos os direitos reservados. É proibido todo tipo de reprodução sem autorização escrita da Agência Efe S/A.

< http://noticias.terra.com.br/ciencia/interna/0,,OI465195-EI238,00.html >


[As partes desta mensagem que não continham texto foram removidas]



SUBJECT: Re: [ciencialist] Re: Zodiaco
FROM: "Esteban Moreno" <estebanmoreno@idhi.org.br>
TO: <ciencialist@yahoogrupos.com.br>
DATE: 04/02/2005 11:32

E no caso da sinastria, como saberia sobre o marido?
E.

----- Original Message -----
From: rmtakata
To: ciencialist@yahoogrupos.com.br
Sent: Friday, February 04, 2005 8:48 AM
Subject: [ciencialist] Re: Zodiaco



--- Em ciencialist@yahoogrupos.com.br, gisele campos <giscampos@y...>
> Como pode ??

Leitura fria, por exemplo.

[]s,

Roberto Takata





##### ##### #####

Para saber mais visite
http://www.ciencialist.hpg.ig.com.br


##### ##### ##### #####


Yahoo! Grupos, um serviço oferecido por:







------------------------------------------------------------------------------
Links do Yahoo! Grupos

a.. Para visitar o site do seu grupo na web, acesse:
http://br.groups.yahoo.com/group/ciencialist/

b.. Para sair deste grupo, envie um e-mail para:
ciencialist-unsubscribe@yahoogrupos.com.br

c.. O uso que você faz do Yahoo! Grupos está sujeito aos Termos do Serviço do Yahoo!.



[As partes desta mensagem que não continham texto foram removidas]



SUBJECT: Re: [ciencialist] Re: Zodiaco
FROM: "Esteban Moreno" <estebanmoreno@idhi.org.br>
TO: <ciencialist@yahoogrupos.com.br>
DATE: 04/02/2005 12:26


Takata:
> E no caso da sinastria, como saberia sobre o marido?
Tb poderia ser por leitura fria. Ha' varias pequenas tecnicas q.
funcionam para causar boa impressao nas pessoas - nao apenas no
inocentes bobocas, mas mesmo em pessoas muito bem dotadas
intelectualmente(...).

E:
É possível. Mas não lhe parece ao menos um pouco suspeito que tantas pessoas
declarem-se imensamente surpreendidas com a precisão das descrições
astrológicas? Um bom parâmetro é a quantidade do mercado que vem se
ampliando extensivamente, incluindo grandes empresas e corporações. Há
estatísticas que mostram isso. A sensação que vocês colocam é que todas as
pessoas são tolas e facilmente influenciaveis, o que pode em maioria ser
verdade, mas tem um limite, não é sempre assim. Não lhes parecem existir um
muro em frente 'as vossas narinas que lhes impede perceber algo?
Naturalmente os céticos não são "influenciáveis", como supõe, e tem um farto
arsenal de exemplos históricos para demonstrar por quantas desventuras
inúteis passou o ser humano ao devotar-se com crendices tolas. Boa parte
parece não servir mesmo, mas isso não lhes permite tanta arrogância para
julgar algo que simplesmente demonstram amplamente não entender e não querer
compreender, repetindo sempre os mesmos contra-argumentos exaustivamente.
Além disso e surpreendentemente, negam-se a uma simples experimentação do
mérito, como faria qualquer cientista de fato.

Mas mesmo que o fizessem, é provável que assumissem uma mesma postura
arrogante e incondicional do "Bussunda" ao ser interpelado pelo astrólogo
Pedro Tornagui, quando previu sem conhece-lo que teria grande talento para
comunicação e poderia ser um ótimo jornalista. Este, pansudamente cético,
disse que jamais pensou em trabalhar em comunicação, apesar de ter feito um
ano e meio de jornalismo e ser um dos maiores humoristas do Brasil. Ou o que
os vossos representantes da terra arredondada (seria mais cientificamente
correto do que redonda) fizeram com o astrólogo Alexey em seu suposto
interesse por uma experimentação, foi algo de covarde e infantil. Talvez
queiram saber sobre isso depois. Aos poucos não me espanto mais, devo estar
contaminado com alguma dessas crendices primitivas. ;-)

Takata:
Um pequeno macete eh o uso de caracteristicas gerais - como humanos
apresentamos grandes semelhancas gerais: a maioria dos humanos
apresentam boa indole, e de vez em qdo saem do serio. (...)

E:
É acreditar demais em tolices. Acorda!!!

Um abraço,
Esteban.







SUBJECT: Re: Zodiaco
FROM: "rmtakata" <rmtakata@altavista.net>
TO: ciencialist@yahoogrupos.com.br
DATE: 04/02/2005 14:45


--- Em ciencialist@yahoogrupos.com.br, "Esteban Moreno"
> É possível. Mas não lhe parece ao menos um pouco suspeito que tantas
> pessoas declarem-se imensamente surpreendidas com a precisão das
> descrições astrológicas?

Entao, em principio pareceria 'suspeito'. Mas como dito, qdo
devidamente testada a astromancia nao parece ter um bom poder de predicao.

> Um bom parâmetro é a quantidade do mercado que vem se
> ampliando extensivamente, incluindo grandes empresas e corporações.

Esse na verdade eh um pessimo parametro. Se for por isso deveremos
creditar confianca na homeopatia, passes espirituais (sem falar na
cirurgia mediunica), toda sorte de mandingas...

[]s,

Roberto Takata





SUBJECT: Re: [ciencialist] Re: Zodiaco
FROM: "Oraculo" <oraculo@atibaia.com.br>
TO: <ciencialist@yahoogrupos.com.br>
DATE: 04/02/2005 14:51

Olá Takata

E tem mais. Se os "setores" tivessem sido analisados pelos sumérios em outra época, digamos 2 mil anos antes, as constelações seriam outras e os valores também. Ou seja, a base desse suposto "conhecimento" continuaria sendo chute e palpite sem relação com a realidade.

E toda a explicação do astrólogo, enviada para a revista Galileu, não muda nada nessa origem, nem informa como esse suposto conhecimento foi (ou poderia ser) validado. Por exemplo, essa "explicação":

"A amplitude dos signos pode ir alem da amplitude de 16 graus que define tradicionalmente a faixa zodiacal, definindo gomos que partem da Terra, tem por fronteiras a divisão em 12 do equador celeste e tem por fundo o infinito."

Bem, e de onde saiu essa informação? O que garante sua validade? Por que a amplitude é de 16 graus e não 20 ou 5 graus? Por que gomos? Por que 12? Sim, eram 12 na época dos criadores do zodiaco, mas, se estes tivessem criado sua explicação dos céus mil anos antes, talvez fossem 11 ou 13 ou 15..:-) Teriamos hoje 15 "setores" e "gomos" e não apenas 12...:-)

De toda forma, a explicação do astrologo não deveria produzir mais que uma única indagação: E daí?..:-) O que isso tudo importa, o que isso tudo informa sobre a validade de um conhecimento absolutamente sem evidencias como a astrologia?

E a afirmação final de que astronomos tem medo da astrologia é a mesma fuga usada com relação a homeopatia (a medicina tem medo da homeopatia), poderes paranormais (a ciência tradicional tem medo da parapsicologia), etc. Em todos os casos, não é porque esses supostos conhecimentos não conseguem demonstrar eficácia ou confiabilidade que não são aceitos, mas porque "alguém tem medo deles"..:-) E duvidar de afirmações sem evindencias não é arrogancia, mas cuidado justificado.

Homero


----- Original Message -----
From: rmtakata
To: ciencialist@yahoogrupos.com.br
Sent: Thursday, February 03, 2005 6:15 PM
Subject: [ciencialist] Re: Zodiaco



--- Em ciencialist@yahoogrupos.com.br, "Esteban Moreno"
> Segue mais um esclarecimento...
> Esteban.
>
> E para finalizar, convido aos astrônomos apavorados com a
> Astrologia, a serem menos pretensiosos e se informarem melhor para
> evitar falarem tantas besteiras!

Verdade. Tb convidaria os astromantes a estudarem um pouco de
economia, medicina, geologia, psicologia e quejandos para fazerem seus
chutes. Assim ficaria melhor, se bem q., no caso, poderiamos jogar
fora as cartas natais e estudar elementos q. apresentem reais
correlacoes com o fato a ser previsto.

(Uma coisa no entanto eh interessante. Embora *hoje* se diga q., por
exemplo, a astromancia se baseia em setores - e os nomes sao apenas
uma curiosidade historica - as qualidades regidas pelos setores sao
eminentemente ligadas aos nomes dos formatos arbitrarios das
constelacoes q. estavam nos respectivos setores 'a milhares de anos:
por exemplo, ter um aspecto leonino significa ter um carater corajoso
e algo feroz, q. eh exatamente a ideia q. o leao traz, dito, o rei dos
animais; ter um aspecto de libra significa ser equilibrado, q. eh o q.
uma balanca representa - haja vista a balanca da justica; ser de
gemeos eh ter uma dualidade e assim por diante. Por uma incrivel
coincidencia, as caracteristicas influenciadas ou descritas por
determinados setores: ou configuracoes celestes em determinados
setores, seriam exatamente as mesmas das formas arbitrarias de
agrupamento de estrelas q. ocupavam tais setores no momento historico
em q. a astromancia tomou forma. Temos um caso aqui de uma construcao
argumentativa a posteriori. Eu diria q. se a astromancia tivesse se
baseado na divisao setorial por meio de constelacoes tupis, a
significacao das mesmissimas configuracoes celestes seriam totalmente
diferentes - e ainda assim milhoes de pessoas enxergariam espantosas
revelacoes.)

[]s,

Roberto Takata





##### ##### #####

Para saber mais visite
http://www.ciencialist.hpg.ig.com.br


##### ##### ##### #####


Yahoo! Grupos, um serviço oferecido por:

São Paulo Rio de Janeiro Curitiba Porto Alegre Belo Horizonte Brasília




------------------------------------------------------------------------------
Links do Yahoo! Grupos

a.. Para visitar o site do seu grupo na web, acesse:
http://br.groups.yahoo.com/group/ciencialist/

b.. Para sair deste grupo, envie um e-mail para:
ciencialist-unsubscribe@yahoogrupos.com.br

c.. O uso que você faz do Yahoo! Grupos está sujeito aos Termos do Serviço do Yahoo!.



[As partes desta mensagem que não continham texto foram removidas]



SUBJECT: Re: [ciencialist] espumas de pororoca
FROM: "Luiz Ferraz Netto" <leobarretos@uol.com.br>
TO: <ciencialist@yahoogrupos.com.br>, <naeq-ucs@yahoogrupos.com.br>, <quimica-qaw@yahoogrupos.com.br>, <quimica@grupos.com.br>
DATE: 04/02/2005 14:58



Emiliano,
>eheheh... nem fiz doutorado e sou chamado de doutor. Fora a brincadeira,
achei interessante a pergunta do consulente. Alguma contribuição?<

Advogado, promotor, juiz, delegado e toda essa 'nata' tb são chamados de 'doutor' e jamais passaram dos 5 anos de estudos do bacharelado do curso de direito; nunca defenderam uma tese e nem sabem o que é isso (só faltava alguém imaginar que Direito é algo científico!) --- o que eles sabem bem é a cada 15 ou 17 palavras falar a 'marca' "paradígma".
Já ouviu um desses doutores falar mais de 30 segundos sem usar dessa 'marca'. Paradígma, para eles, deve ser algo equivalente à vírgula na língua pátria. :-)

aquele abraço,
novato no assunto




--
No virus found in this outgoing message.
Checked by AVG Anti-Virus.
Version: 7.0.300 / Virus Database: 265.8.5 - Release Date: 03/02/2005



SUBJECT: Re: [ciencialist] Re: Zodiaco
FROM: "Oraculo" <oraculo@atibaia.com.br>
TO: <ciencialist@yahoogrupos.com.br>
DATE: 04/02/2005 15:01

Olá Taborda

A respsta é sempre a mesma: prove isso tudo. Demonstre que leoninos são bons pais, que tem relação com seu signo, etc.

Sem isso (e continuamos sem isso..:-), é apenas mais afirmações sem base e sem evidencias. A ligação continua meramente arbitrária, as explicações posteriores, que tentam ajustar as incongruencias, sempre aumentando e nada mais.

O caso dos setores é evidente. Se a astrologia não existisse, mas fosse inventada (inventada é um termo preciso..:-) hoje, teriamos 13 signos, e 13 setores. Como poderia alguém que estivesse criando a astrologia hoje determinar que um dos setores não deveria ser utilizado? Como ele teria acesso a essa informação, se existem 13 constelações a considerar ?

Como o astrologo nos informou, valem os setores criados pelos assirios, com base em seu céu visivel na época. Se criada hoje, a astrologia seria de 13 signos e totalmente diferente em seus significados e interpretações e os mapas astrais diferentes nos resultados. Como pode isso ser uma fonte de conhecimento confiável ou mesmo real?

Conhecimento que determinava que o Sol e a Lua eram planetas e agiam como tal, inclusive girando em torno da Terra (a Lua até que gira, embora não como os planetas, mas o Sol..:-). Isso tem de ser "ajustado" pelos modernos astrologos ou deve ser tomado literalmente? Ou alegoricamente, embora para os criadores da astrologia fosse uma verdade literal?

A cada temporada, novos astrologos e defensores da astrologia retornam. A cada temporada, os mesmos argumentos e a mesma falta de evidencias. Cansativo..:-)

Sempre mais do mesmo. Apenas prove o que afirma, e nenhuma discussão seria necessária.

Homero
----- Original Message -----
From: Sergio M. M. Taborda
To: ciencialist@yahoogrupos.com.br
Sent: Thursday, February 03, 2005 8:56 PM
Subject: Re: [ciencialist] Re: Zodiaco


rmtakata wrote:

>
> (Uma coisa no entanto eh interessante. Embora *hoje* se diga q., por
> exemplo, a astromancia se baseia em setores - e os nomes sao apenas
> uma curiosidade historica - as qualidades regidas pelos setores sao
> eminentemente ligadas aos nomes dos formatos arbitrarios das
> constelacoes q. estavam nos respectivos setores 'a milhares de anos:

Essa é a sua opinião. Se vc olhar o ceu , sem saber as contelações -
como eu - você facilmente consegue agrupas a estrelas da forma canónica,
que vc não conseguirá será atribuir o mesmo simbolo. Por exemplo as
Ursas Maior e Menor , não se parecem em nada com ursos e mais com
frigideiras.
O facto das estrelas terem sido agrupadas como foram não espanta, o que
espanta é a sua conecção com simbolos.
Vc acha que a ligação é meramente arbitrária, mas eu podria descorrer
aqui, por não o é.

> por exemplo, ter um aspecto leonino significa ter um carater corajoso
> e algo feroz,

Não. Leão simboliza a coragem (coração+agem=acção do coração), mas não a
ferocidade.
O Leonino é bom por natureza, como um pai. Um pai não é bravo pq é
feroz, mas pq não é respeitado.
É algo bem diferente da ferocidade atribuida aos animais leão.

> q. eh exatamente a ideia q. o leao traz

Ora, mas é a ideia do 5º signo que tem como menomonica o leão, ou o
leão que empretou as suas caracteritsticas ao signo ?
É a primeira e não a segundo, como vc supoe.
Qualquer coisa poderia ser atruida à constelação de leão ,pq um leão ?
Ora, afinal aquilo é so um triagulo com uma ponta

> ter um aspecto de libra significa ser equilibrado,

Não. Significa indiciso. Que não sabe tomar parte, que não sabe decidir
entre as opções.

> q. eh o q.
> uma balanca representa

Não. A balança representa o acto de ponderar. Ela sempre começa
desiquilibrada e cabe a nós equilibrar os pratos para saber quando pesa
algo,
Mas para fazer isso vc precisa de um padrão, uma unidade de medida, e é
isso que os Librinianos não têm, e por isso não chegam a lado nenhum.
A justiça é representada por uma balança, mas não é libra o signo da
justiça pq se assim fosse o juiz nunca saberia decidir nada.

> - haja vista a balanca da justica;

Não são comparáveis.

> ser de
> gemeos eh ter uma dualidade e assim por diante.

Não é uma dualidade, é uma ambiguidade. Da mesma forma que balança não
sabe dicidir por uma opção,
gemeos não sabe dicidir por uma personaldiade.

> Por uma incrivel
> coincidencia, as caracteristicas influenciadas ou descritas por
> determinados setores: ou configuracoes celestes em determinados
> setores, seriam exatamente as mesmas das formas arbitrarias de
> agrupamento de estrelas q. ocupavam tais setores no momento historico
> em q. a astromancia tomou forma.

Compare com a astrologia Indu (a atrologia original onde se baseia a
ocidental actual). Nele os mesmos signos, com as mesms
exactas posições não têm nada a ver com as estrelas no sentido de que
não ha correspondencia entre os nomes das constrelçaões
e dos signos. Tb é verdade que aquilo que é atribuido a cada singo não
exactamente igual ao que a astrologia ocidental atriui, mas
não é assim tão diferente que se possa considerar a diferença. É mais
uma diferença filosofica
´É verdade que ha uma relação historica entre os nomes das constelações
e o significa dos signos q originalmente as enquadravam, mas
isso apenas na astrologia ocidental que é baseada na adptação pelos
gregos da astrologia indu.
Os nomes, as figuras, as hisotiras da mitologia, seus personagens e toda
a riquesa da mitologia grega é apenas uma grande menmonica para
a astrologia grega recem criada (/roubada/adptada) da indu.

> Temos um caso aqui de uma construcao
> argumentativa a posteriori.

Não, não temos. Vc acha que sim pq não conhece um pouco da historia da
astrologia.

E para não o deixar com a vontade eis como se forma o significado do
zodiaco:


O Pai sol (masculino=+) e a Mae lua (femenino =-) (os luminosos, luz
=vida) têm dois filhos Mercurio+ (gemeos) e Mercurio- (virgem).
Mercurio filho do sol e da lua é a comunicação entre os dois. OS pais
comunicam um com o outro atrave dos filhos, e assim eles aprendem a
conhecer o mundo. Mas esse conhecimento demasiado mental leva-os a não
aproveitarem tantas outras coisas do mundo onde os pais vivem. E todo o
pai quer que os filhos conhecam o seu mundo. Dois outros filhos (mais
novos) são venus+ (libra)e venus-(touro). Venus é a possibilidade de
olhar o mundo. (é por isso que libra é indiciso, pq se perde demasiado
com a fachada, a estética e esquece o resto). Repare que o signo de
touro é femenino, o que evidentemente não tem nada a ver com o seu
argumento pois a constelação não encaixa com o significado. A unica
coisa que podramos encaixar é a teimosia.
Os primogenitos são mentalmente activos e esquecem do mundo, os irmaos
são observadores do mundo e não das ideias. Os irmaos mais caçulas
marte-(escorpiao) e marte+ (aries) são aqueles que utilizam do mundo.
Que vão e pegam nas coisas. A sua actividade é de acção para com o mundo
e não mais mental ou contemplativa. Os irmaos mais novos que estes
jupiter+(sagistário) e jupiter-(peixe) passam além do material e
retornam à contemplação, mas desta vez do ponto de vista abstracto e não
mais do ponto de vista fisico, como libra e touro. Por isso que estes
signos são ligados à amaizade e à religião, conceitos que o mundo fisico
não pode entender. Estes são tb os primeiro singos da segunda metado do
zodiaco, que tecnicamente seria a noite, ou seja, o mundo psiquico,
enquanto a primeira metade, representa o mundo fisico, material, o dia.
Or proximo irmãoes são saturno+ (capricornio) e saturno-(aquário) que
vão além de tudo isto e desenvolvem meta-capacidades. Capacidades que
que representam a reflexão da coisa sobre si mesma. Como a meta-fisica é
a reflecção da fisica sobre si mesma. Esta capacidade os torna opostos
aos pais e querem estar um passo à frente dos pais, tornando-se criticos
e desafiadores das regras. Enquanto os outros são uma relção com o
mundo, estes observam as relações entre os irmaos. Pois sendo os mais
novos são os que podem observar a todos os outros toda a sua vida.

Embora se considere que o calendário astrologico comece com aries, a
formação de sinificado não começa por ele.
Voltando ao assunto, os primeiros signos são os ligados ao sol e à lua,
Poderiamos considerar que o leão é tã magnanimo com a selva como o pai
com a sua familia e desse ponto de vista seria um bom simbolo a atribuir
ao signo, até pq as estrelas nesses signo podem parecer um leão, com boa
vontade de quem olha. Mas e a lua, que simbolo podemos lhe atribuir ?
Não mais do que aquele que podemos obter olhando as estrelas. Afinal, o
que um carangueijo tem a ver com a lua ?

OS simbolos são meramente circusntaciais , são simbolos, menmonicas,
nada mais. Mas os significados, que é o que interessa, são baseados numa
logica solida, embora complexa para explicar por uma so prespecitva.

Sérgio Taborda









--
No virus found in this outgoing message.
Checked by AVG Anti-Virus.
Version: 7.0.300 / Virus Database: 265.8.5 - Release Date: 03-02-2005



##### ##### #####

Para saber mais visite
http://www.ciencialist.hpg.ig.com.br


##### ##### ##### #####


Yahoo! Grupos, um serviço oferecido por:
PUBLICIDADE




------------------------------------------------------------------------------
Links do Yahoo! Grupos

a.. Para visitar o site do seu grupo na web, acesse:
http://br.groups.yahoo.com/group/ciencialist/

b.. Para sair deste grupo, envie um e-mail para:
ciencialist-unsubscribe@yahoogrupos.com.br

c.. O uso que você faz do Yahoo! Grupos está sujeito aos Termos do Serviço do Yahoo!.



[As partes desta mensagem que não continham texto foram removidas]



SUBJECT: Re: [ciencialist] Re: Zodiaco
FROM: "Oraculo" <oraculo@atibaia.com.br>
TO: <ciencialist@yahoogrupos.com.br>
DATE: 04/02/2005 15:05

Olá

Esse argumento faz sentido porque a astrologia FOI baseada nas constelações. Os que a criaram usaram as constelaçòes para basear sua criação. Nada sobre setores, inventados a posteriori para explicar incongruencias. Os criadores nào sabiam nada sobre precessão ou sobre estrelas como sois e o universo como setores. Sabiam empiricametne que as constelações se cucediam e isso permitia marcar o tempo e determinar estaçòes do ano.

A decorrencia foi um sistema de advinhaçào apra o futuro (já que podiam advinhar as estaçòes e marca-las) que redundou em um enorme arcabouço de conhecimento incorreto.

Hoje os setores sào o que importa, já que fica dificil manter a base original da astrologia, mas não foi isso que baseou sua criação.

Se existissem outras constelaçòes no tempo em que foi criada a astrologia, seria uma astrologia diferente. O rsto é uma (mais uma) tentativa de validar o improvável com muita informação mas pouca evidência.

Homero
----- Original Message -----
From: Sergio M. M. Taborda
To: ciencialist@yahoogrupos.com.br
Sent: Thursday, February 03, 2005 9:08 PM
Subject: Re: [ciencialist] Re: Zodiaco


Maria Natália wrote:

>
> Estebam:
> O zodíaco dos astrólogos é o de há 2000 anos e não tem em conta os
> movimentos do eixo da Terra. O zodíaco tem 13 constelações: a seguir
> ao capricónio tem o Ofiuco...São 13 os tais signos. E agora que me
> dizeis acerca do Ofiuco e que de acordo com o zodíaco actual é o meu
> signo: poderei amanhã comprar lotaria para me sair prémio?

Eu ja respondi a essa pergunta. E a resposta é : Esse argumento faria
sentido SE a astrologia fosse baseada nas constelações.
Como não é , insistir nesse argumento é pura besteira (=coisas que os
bestas fazem)
Jà lhe expliquei pq so ha 12 signos, o que já foi aqui explicado por
outros. É uma razão matemática para que seja assim. É baseada no
conceito de Periodo Sinódico.
Ha uma logica , séria, matemática, fisica, _astronomica_ para o numero
12. Só que aos astronomos - com problemas de falta de atenção -
inventaram esse argumento sobre Ofucio.
fazer o quê ? Mas se vc acredita nesses mentirosos, não é diferente de
quem acredita em astrologos fajutos.

> Desculpe, por acaso ouvi dizer que os astrónomos e astrofísicos falam
> besteira? Ou entendi mal?

Ouvio bem. Eles dizem mesmo. E ha muitas mais do que a de Ofucio.


Sérgio Taborda



--
No virus found in this outgoing message.
Checked by AVG Anti-Virus.
Version: 7.0.300 / Virus Database: 265.8.5 - Release Date: 03-02-2005



##### ##### #####

Para saber mais visite
http://www.ciencialist.hpg.ig.com.br


##### ##### ##### #####


Yahoo! Grupos, um serviço oferecido por:







------------------------------------------------------------------------------
Links do Yahoo! Grupos

a.. Para visitar o site do seu grupo na web, acesse:
http://br.groups.yahoo.com/group/ciencialist/

b.. Para sair deste grupo, envie um e-mail para:
ciencialist-unsubscribe@yahoogrupos.com.br

c.. O uso que você faz do Yahoo! Grupos está sujeito aos Termos do Serviço do Yahoo!.



[As partes desta mensagem que não continham texto foram removidas]



SUBJECT: Fw: dinamometro e temometro cutaneo
FROM: "Luiz Ferraz Netto" <leobarretos@uol.com.br>
TO: "ciencialist" <ciencialist@yahoogrupos.com.br>
DATE: 04/02/2005 15:09

Alguém manja isso?

[]'
===========================
Luiz Ferraz Netto [Léo]
leobarretos@uol.com.br
http://www.feiradeciencias.com.br
===========================
-----Mensagem Original-----
De: Vasco Senna-Fernandes
Para: Luiz Ferraz Netto
Enviada em: sexta-feira, 4 de fevereiro de 2005 10:29
Assunto: Re: dinamometro e temometro cutaneo


olã amigo,

Bom dia. Eu pensa que o seu homepage fosse de divulgaçao comercial tambem. Ja que, geralmente, quando congressos ou feiras sao divulgados, segue -se tambem a divulgaçao de produtos relacionados.

Eu gostaria de saber aonde eu poderia comprar um dinamometro para medir força muscular e um termometro cutaneo para estudar a perfusao sanguinea de nivel nacional para finalidade de pesquisa medica.


Tchau


Vasco


-----Mensagem Original-----
De: Vasco Senna-Fernandes
Para: leobarretos@uol.com.br
Enviada em: quinta-feira, 3 de fevereiro de 2005 14:10
Assunto: dinamometro e temometro cutaneo


Olã,


Gostaria saber os preços de dinamometros e termometro cutaneo para finalidades de trabalho de pesquisa cientifica

Tchau

----------

No virus found in this outgoing message.
Checked by AVG Anti-Virus.
Version: 7.0.300 / Virus Database: 265.8.5 - Release Date: 03/02/2005


[As partes desta mensagem que não continham texto foram removidas]



SUBJECT: Re: [ciencialist] Re: Zodiaco
FROM: "Oraculo" <oraculo@atibaia.com.br>
TO: <ciencialist@yahoogrupos.com.br>
DATE: 04/02/2005 15:13

Olá Gisele

risos..:-) Não se preocupe, não será escomungada de modo nenhum, até porque descrentes e céticos malvados não tem escomunhão entre suas armas e predicados..:-)

Mas podemos tentar explicar o seu espantoso mapa astral com bons argumentos lógicos e você decide se deseja refuta-los, aceita-los ou ignora-los, ok?..:-)

Nós (os malvados céticos..:-) apenas evitamos escolher uma explicação para um fenomeno sem antes pensar bastante sobre ele, de todos os pontos de vista e a partir de todas as explicações possíveis. Por exemplo, coincidencias (matemática estatistica e numeros grandes), pareidolia, psicologia cognitiva, percepção seletiva, leitura fria, e até mesmo fraude pura e simples.

Se quiser (e não se sentir ofendida com isso), é possível fazer várias suposições sobre o incidente mapa astral, sem precisar da hipotese "astros permitem ler o futuro"..:-)

Talvez fosse interessante começar por aqui:

http://www.projetoockham.org/pseudo_astrologia_1.html

Um abraço.

Homero


----- Original Message -----
From: Esteban Moreno
To: ciencialist@yahoogrupos.com.br
Sent: Friday, February 04, 2005 10:18 AM
Subject: Re: [ciencialist] Re: Zodiaco


Cuidado, Gisele, podes ser escomungada deste fórum com este tipo de depoimento!
Benza Deus! ;-)
Esteban.


Olá ....

Não entendo bulhufas de astrologia,mas fazem dois anos que fiz meu mapa astral , ou esta coisa funciona , ou tem coincidência demais aí ....
Como pode ??
Bem , a descrição da perda de um pai e a substituição de um padrasto com características x ,y e z , é no mínimo estranho......
E a tal da sinastria então , parece brincadeira ......

Gisele




"Sergio M. M. Taborda" <sergiotaborda@terra.com.br> wrote:Maria Natália wrote:

>
> Estebam:
> O zodíaco dos astrólogos é o de há 2000 anos e não tem em conta os
> movimentos do eixo da Terra. O zodíaco tem 13 constelações: a seguir
> ao capricónio tem o Ofiuco...São 13 os tais signos. E agora que me
> dizeis acerca do Ofiuco e que de acordo com o zodíaco actual é o meu
> signo: poderei amanhã comprar lotaria para me sair prémio?

Eu ja respondi a essa pergunta. E a resposta é : Esse argumento faria
sentido SE a astrologia fosse baseada nas constelações.
Como não é , insistir nesse argumento é pura besteira (=coisas que os
bestas fazem)
Jà lhe expliquei pq so ha 12 signos, o que já foi aqui explicado por
outros. É uma razão matemática para que seja assim. É baseada no
conceito de Periodo Sinódico.
Ha uma logica , séria, matemática, fisica, _astronomica_ para o numero
12. Só que aos astronomos - com problemas de falta de atenção -
inventaram esse argumento sobre Ofucio.
fazer o quê ? Mas se vc acredita nesses mentirosos, não é diferente de
quem acredita em astrologos fajutos.

> Desculpe, por acaso ouvi dizer que os astrónomos e astrofísicos falam
> besteira? Ou entendi mal?

Ouvio bem. Eles dizem mesmo. E ha muitas mais do que a de Ofucio.


Sérgio Taborda



--
No virus found in this outgoing message.
Checked by AVG Anti-Virus.
Version: 7.0.300 / Virus Database: 265.8.5 - Release Date: 03-02-2005



##### ##### #####

Para saber mais visite
http://www.ciencialist.hpg.ig.com.br


##### ##### ##### #####


Yahoo! Grupos, um serviço oferecido por:



















function SearchComboBox() { if (document.form_combo.keyword.value.length==0){ alert("Por favor, digite algo."); return false; }else { document.form_combo.action ="http://br.rd.yahoo.com/SIG=12avidbm8/M=264105.3931087.6562589.1588051/D=brclubs/S=2137111528:HM/EXP=1107558491/A=2361264/R=0/SIG=11uaou2jn/*http://www.bondfaro.com/bondfaro/in/combosearch_in.jsp?sk=11"; } return true;} [input] [input] [input]

---------------------------------
Links do Yahoo! Grupos

Para visitar o site do seu grupo na web, acesse:
http://br.groups.yahoo.com/group/ciencialist/

Para sair deste grupo, envie um e-mail para:
ciencialist-unsubscribe@yahoogrupos.com.br

O uso que você faz do Yahoo! Grupos está sujeito aos Termos do Serviço do Yahoo!.




---------------------------------
Yahoo! Acesso Grátis - Internet rápida e grátis. Instale o discador do Yahoo! agora.

[As partes desta mensagem que não continham texto foram removidas]



##### ##### #####

Para saber mais visite
http://www.ciencialist.hpg.ig.com.br


##### ##### ##### #####


Yahoo! Grupos, um serviço oferecido por:

São Paulo Rio de Janeiro Curitiba Porto Alegre Belo Horizonte Brasília




------------------------------------------------------------------------------
Links do Yahoo! Grupos

a.. Para visitar o site do seu grupo na web, acesse:
http://br.groups.yahoo.com/group/ciencialist/

b.. Para sair deste grupo, envie um e-mail para:
ciencialist-unsubscribe@yahoogrupos.com.br

c.. O uso que você faz do Yahoo! Grupos está sujeito aos Termos do Serviço do Yahoo!.



[As partes desta mensagem que não continham texto foram removidas]



##### ##### #####

Para saber mais visite
http://www.ciencialist.hpg.ig.com.br


##### ##### ##### #####


Yahoo! Grupos, um serviço oferecido por:

São Paulo Rio de Janeiro Curitiba Porto Alegre Belo Horizonte Brasília




------------------------------------------------------------------------------
Links do Yahoo! Grupos

a.. Para visitar o site do seu grupo na web, acesse:
http://br.groups.yahoo.com/group/ciencialist/

b.. Para sair deste grupo, envie um e-mail para:
ciencialist-unsubscribe@yahoogrupos.com.br

c.. O uso que você faz do Yahoo! Grupos está sujeito aos Termos do Serviço do Yahoo!.



[As partes desta mensagem que não continham texto foram removidas]



SUBJECT: Re: [ciencialist] Re: Zodiaco
FROM: "Oraculo" <oraculo@atibaia.com.br>
TO: <ciencialist@yahoogrupos.com.br>
DATE: 04/02/2005 15:27

Olá Esteban

Bem, não parece suspeito que, mesmo com tantas pessoas se mostrando espantadas com o acerto de astrologos (e de cartomantes, mediuns, telepatas, ETs telepatas, espiritos, entidades de santeria, etc, etc, etc), quando colocadas em um teste com o rigor cientifico, elas falhem miseravelmente?

Não parece suspeito que, se pedir diversos mapas astrais a astrologos competentes (em sua profissão, claro), e distribui-los aleatoriamente entre os participantes, mesmo assim a maioria vai declarar que a taxa de acerto é "espantosa"?

Não parece suspeito que, mesmo mapas astrais criados por mágicos ou psicologos, com o uso de tecnicas de leitura fria e psicologia cognitiva, resultem no mesmo espanto? Isso não diz nada a você sobre a forma como nossa mente trabalha?

Não se está discutindo na realidade o acerto, mas a origem do acerto, se é devido ao estudo de setores do céu, ou da aplicação de conhecimento psicologico (e as vezes até fraudulento) da mente humana.

Volta e meia poderes sobrenaturais são desmascarados, com seus proponentes usando desde espionagem até tecnologia avançada para saber mais sobre seus alvos. Mas, antes de descobertos, cada paciente tem a nítida impressão de ser espantoso o conehcimento que o vidente tem sobre ele.

Então, antes de decidir por uma explicação sobrenatural (e ler o futuro ou passado na posição dos astros ou setores é bem sobrenatural..:-) é preciso se certificar de que não é este o caso. Ou que a leitura fria, a sensibilidade natural de seres humanos com outros seres humanos, os enganos cognitivos de nossa mente, etc, não estão envolvidos.

Se um astrologo pode realmente ser preciso em seu mapa astral a ponto de espantar o consulente, deveria ser capaz de fazer isso em condições controladas. E eles nào são. Por exemplo, analises psicologicas de pacientes, lidas por outros psicologos, pode permitir que estes indiquem a quem foi aplicado. Engenharia reversa..:-)

Mas, mapas astrais de pacientes, analisados por outros astrolgoos, não permitem saber a quem foram aplciados, não mais que o simples chute ou sorte. Isso indica que o espanto é mais causado pela percepção seletiva (contes os acertos ignore os erros) que pela real capacidade do mapa de dizer algo sobre o paciente.

E esta enganado, diversos estudos foram efetuados sobre a astrologia, sempre com resultads negativos. E uma hora cansa..:-) Mas mesmo agora um novo está sendo criado, e se quiser participar, posso colocar você em contato (estão precisando de astrolgos..:-) E poderá acompanhar o desenrolar dos resultados.

Homero







----- Original Message -----
From: Esteban Moreno
To: ciencialist@yahoogrupos.com.br
Sent: Friday, February 04, 2005 12:26 PM
Subject: Re: [ciencialist] Re: Zodiaco



Takata:
> E no caso da sinastria, como saberia sobre o marido?
Tb poderia ser por leitura fria. Ha' varias pequenas tecnicas q.
funcionam para causar boa impressao nas pessoas - nao apenas no
inocentes bobocas, mas mesmo em pessoas muito bem dotadas
intelectualmente(...).

E:
É possível. Mas não lhe parece ao menos um pouco suspeito que tantas pessoas
declarem-se imensamente surpreendidas com a precisão das descrições
astrológicas? Um bom parâmetro é a quantidade do mercado que vem se
ampliando extensivamente, incluindo grandes empresas e corporações. Há
estatísticas que mostram isso. A sensação que vocês colocam é que todas as
pessoas são tolas e facilmente influenciaveis, o que pode em maioria ser
verdade, mas tem um limite, não é sempre assim. Não lhes parecem existir um
muro em frente 'as vossas narinas que lhes impede perceber algo?
Naturalmente os céticos não são "influenciáveis", como supõe, e tem um farto
arsenal de exemplos históricos para demonstrar por quantas desventuras
inúteis passou o ser humano ao devotar-se com crendices tolas. Boa parte
parece não servir mesmo, mas isso não lhes permite tanta arrogância para
julgar algo que simplesmente demonstram amplamente não entender e não querer
compreender, repetindo sempre os mesmos contra-argumentos exaustivamente.
Além disso e surpreendentemente, negam-se a uma simples experimentação do
mérito, como faria qualquer cientista de fato.

Mas mesmo que o fizessem, é provável que assumissem uma mesma postura
arrogante e incondicional do "Bussunda" ao ser interpelado pelo astrólogo
Pedro Tornagui, quando previu sem conhece-lo que teria grande talento para
comunicação e poderia ser um ótimo jornalista. Este, pansudamente cético,
disse que jamais pensou em trabalhar em comunicação, apesar de ter feito um
ano e meio de jornalismo e ser um dos maiores humoristas do Brasil. Ou o que
os vossos representantes da terra arredondada (seria mais cientificamente
correto do que redonda) fizeram com o astrólogo Alexey em seu suposto
interesse por uma experimentação, foi algo de covarde e infantil. Talvez
queiram saber sobre isso depois. Aos poucos não me espanto mais, devo estar
contaminado com alguma dessas crendices primitivas. ;-)

Takata:
Um pequeno macete eh o uso de caracteristicas gerais - como humanos
apresentamos grandes semelhancas gerais: a maioria dos humanos
apresentam boa indole, e de vez em qdo saem do serio. (...)

E:
É acreditar demais em tolices. Acorda!!!

Um abraço,
Esteban.







##### ##### #####

Para saber mais visite
http://www.ciencialist.hpg.ig.com.br


##### ##### ##### #####


Yahoo! Grupos, um serviço oferecido por:







------------------------------------------------------------------------------
Links do Yahoo! Grupos

a.. Para visitar o site do seu grupo na web, acesse:
http://br.groups.yahoo.com/group/ciencialist/

b.. Para sair deste grupo, envie um e-mail para:
ciencialist-unsubscribe@yahoogrupos.com.br

c.. O uso que você faz do Yahoo! Grupos está sujeito aos Termos do Serviço do Yahoo!.



[As partes desta mensagem que não continham texto foram removidas]



SUBJECT: Fw: Biologia
FROM: "Luiz Ferraz Netto" <leobarretos@uol.com.br>
TO: "ciencialist" <ciencialist@yahoogrupos.com.br>
DATE: 04/02/2005 15:28

Tema profundo para o Roberto.

[]'
===========================
Luiz Ferraz Netto [Léo]
leobarretos@uol.com.br
http://www.feiradeciencias.com.br
===========================
-----Mensagem Original-----
De: wanessa-saraiva
Para: leobarretos
Enviada em: quarta-feira, 2 de fevereiro de 2005 13:59
Assunto: Biologia


leo eu estou cursando o 1º ano do ensino medio, e a professora passou um trabalho sobre "o que é biologia?" voce poderia mim responder essa pergunta? O QUE É BIOLOGIA?


--------------------------------------------------------------------------------


Internal Virus Database is out-of-date.
Checked by AVG Anti-Virus.
Version: 7.0.300 / Virus Database: 265.6.13 - Release Date: 16/01/2005

----------

No virus found in this outgoing message.
Checked by AVG Anti-Virus.
Version: 7.0.300 / Virus Database: 265.8.5 - Release Date: 03/02/2005


[As partes desta mensagem que não continham texto foram removidas]



SUBJECT: Re: [ciencialist] Re: Zodiaco
FROM: "Oraculo" <oraculo@atibaia.com.br>
TO: <ciencialist@yahoogrupos.com.br>
DATE: 04/02/2005 15:33

Olá

E não esquecer a colocação de maças para duendes, que tem aumentado sobremaneira a produtividade das empresas que o adotaram..:-)

Se a taxa de aumento do mercado é um parametro apra a realidade ou confiabilidade de uma alegação (ainda mais extraordinária), quaase todo tipo de tolice poderia ser real, já que tem grande penetração no mercado..:-)

E viva Walter Mercado, o grande astrologo, cartomante, paranormal, que tem programas na TV e um enorme mercado (sem trocadilhos) cativo, que lhe rende muitos milhares de doláres..:-)

Fala sério, milhares de coisas flutuam na aceitação do mercado, modas e novidades, algumas duram muito, outras pouco, mas sempre com "excelente" aceitaçào popular. Pulseiras sabona, correntes para ficar rico (essas ainda em franca assenção na internet..:-), colchões com imãs que tudo curam, igrejas evangelicas e seus copos de aguá milagrosa colocados sobre a TV, etc.

Levar a aceitação do mercado em conta ao analisar a veracidade de uma proposição é, para dizer pouco, bastante temerário..:-)

Homero




----- Original Message -----
From: rmtakata
To: ciencialist@yahoogrupos.com.br
Sent: Friday, February 04, 2005 2:45 PM
Subject: [ciencialist] Re: Zodiaco



--- Em ciencialist@yahoogrupos.com.br, "Esteban Moreno"
> É possível. Mas não lhe parece ao menos um pouco suspeito que tantas
> pessoas declarem-se imensamente surpreendidas com a precisão das
> descrições astrológicas?

Entao, em principio pareceria 'suspeito'. Mas como dito, qdo
devidamente testada a astromancia nao parece ter um bom poder de predicao.

> Um bom parâmetro é a quantidade do mercado que vem se
> ampliando extensivamente, incluindo grandes empresas e corporações.

Esse na verdade eh um pessimo parametro. Se for por isso deveremos
creditar confianca na homeopatia, passes espirituais (sem falar na
cirurgia mediunica), toda sorte de mandingas...

[]s,

Roberto Takata





##### ##### #####

Para saber mais visite
http://www.ciencialist.hpg.ig.com.br


##### ##### ##### #####


Yahoo! Grupos, um serviço oferecido por:







------------------------------------------------------------------------------
Links do Yahoo! Grupos

a.. Para visitar o site do seu grupo na web, acesse:
http://br.groups.yahoo.com/group/ciencialist/

b.. Para sair deste grupo, envie um e-mail para:
ciencialist-unsubscribe@yahoogrupos.com.br

c.. O uso que você faz do Yahoo! Grupos está sujeito aos Termos do Serviço do Yahoo!.



[As partes desta mensagem que não continham texto foram removidas]



SUBJECT: SKN20/02
FROM: "Luiz Ferraz Netto" <leobarretos@uol.com.br>
TO: <eletronicabr@yahoogrupos.com.br>, "ciencialist" <ciencialist@yahoogrupos.com.br>, Fábio Henrique Cicuto Gonçalves <fabio.henrique.gonca@terra.com.br>, <radiotecnico@yahoogrupos.com.br>
DATE: 04/02/2005 15:36

Oi boa gente,

preciso de 4 diodos retificadores SKN20/02 (ou apenas 2 para 200V, 40A) pois os mesmos no meu carregador de baterias pifaram (devido a um maledeto 'curto'). Aqui em Barretos não tem disso.
Se alguém puder enviar-me entre em contato em PVT para combinarmos as 'transferências'.

Agradeço,
===========================
Luiz Ferraz Netto [Léo]
leobarretos@uol.com.br
http://www.feiradeciencias.com.br
===========================


--
No virus found in this outgoing message.
Checked by AVG Anti-Virus.
Version: 7.0.300 / Virus Database: 265.8.5 - Release Date: 03/02/2005



SUBJECT: Re: Fw: Biologia
FROM: "Emiliano Chemello" <chemelloe@yahoo.com.br>
TO: ciencialist@yahoogrupos.com.br
DATE: 04/02/2005 15:48


> leo eu estou cursando o 1º ano do ensino medio, e a professora
passou um trabalho sobre "o que é biologia?" voce poderia mim
responder essa pergunta? O QUE É BIOLOGIA?
>

What is Biology
http://www.cogsci.princeton.edu/cgi-bin/webwn2.0?
stage=2&word=biology&posnumber=1&searchtypenumber=2&senses=&showglosse
s=1

[ ] 's do Emiliano Chemello





SUBJECT: Re: [ciencialist] Re: Zodiaco
FROM: "Esteban Moreno" <estebanmoreno@idhi.org.br>
TO: <ciencialist@yahoogrupos.com.br>
DATE: 04/02/2005 16:07

Sr. Oracle
A cada temporada, novos astrologos e defensores da astrologia retornam. A
cada temporada, os mesmos argumentos e a mesma falta de evidencias.
Cansativo..:-)


E: Mas são exatamente os mesmos defensores, deve ser o cansanso mesmo...
Um abraço,
Esteban.



SUBJECT: Re: [ciencialist] Re: Zodiaco
FROM: "Esteban Moreno" <estebanmoreno@idhi.org.br>
TO: <ciencialist@yahoogrupos.com.br>
DATE: 04/02/2005 16:24


Mr Takata wrote:
> descrições astrológicas?
Entao, em principio pareceria 'suspeito'. Mas como dito, qdo
devidamente testada a astromancia nao parece ter um bom poder de predicao.

E:
Devidamente testada por quem? Pelos santos céticos imparcialistas da
ciência?
Depende de quem ou como se pesquisa. Há uma série de artefatos e falácias do
que julgam como pesquisas comprobatórias. Não vou enumera-las, mas talvez
ajude a lembrar que a pesquisa da universidade de brasília resultou em dados
bastante promissores. Que péssimo! Uma das justificativas dos céticos acerca
do porquê os pacientes sentem-se bem descritos pelo astrólogo é a chamada
confluência (auto-atribuição é um nome pobre para isso). Abaixo segue um
artigo científico que "desmistifica" (perdoe-me..) este recurso:

Self-attribution, sun-sign traits, and the alleged role of favourableness as
a moderator variable:
long-term e.ect or artefact? Edgar Wunder* Gesellschaft fu¨r Anomalistik,
Postfach 1202, 69200 Sandhausen, Germany Personality and Individual
Differences 35 (2003) 1783-1789 www.elsevier.com/locate/paid

Posso envia-lo em anexo.

Takata:
> Um bom parâmetro é a quantidade do mercado que vem se
> ampliando extensivamente, incluindo grandes empresas e corporações.
Esse na verdade eh um pessimo parametro. Se for por isso deveremos
creditar confianca na homeopatia, passes espirituais (sem falar na
cirurgia mediunica), toda sorte de mandingas...

E:
Talvez devesse dar algum crédito mesmo. Já estão acreditando em
acupuntura... Mas foi testada, nê, botaram uns eletrodozinhos e viram que
tinham razão aqueles outros velhinhos há mais de 3000 anos atrás...
Acordaaaaa!!!!


Infelizmente só retorno daqui a 10 dias.
Um carnaval cheio de boas mandingas para vocês.
Esteban.



SUBJECT: Re: [ciencialist] Re: Zodiaco
FROM: "Esteban Moreno" <estebanmoreno@idhi.org.br>
TO: <ciencialist@yahoogrupos.com.br>
DATE: 04/02/2005 16:35

Oracle
Olá Gisele risos..:-) Não se preocupe, não será escomungada de modo nenhum,
até porque descrentes e céticos malvados não tem escomunhão entre suas armas
e predicados..:-)

E:
É verdade, Gisele, não será excomungada, já passamos por isso, naturalmente
estava brincando. Mas de alguma forma poderá ser sempre lembrada com uma das
pobres vitimas das falácias auto-indutivas dos astrólogos. Uma vítima
inocente, mal preparada, incapaz de perceber que foi iludida por um a
análise
fria e arranjada, para que você se sentisse bem com um série de dizeres
inconsistentes e vazios, mas que pareciam falar de você. Tantas milhares e
milhões de
pobres vítimas deste mundo de Tsnunamis astrais, e não para de crescer...
Mas você pode se regenerar, minha cara!! Creia na ciência, experimente,
digo, não experimente, porque experimentar algo infundado é próprio dos
tolos. Esqueça estas falácias astrománticas, a ciência vos libertará!!


Oracle:
Talvez fosse interessante começar por aqui:
http://www.projetoockham.org/pseudo_astrologia_1.html

E:
É mesmo um bom ponto de partida o projeto ockham. Depois destrincho para
você.


Abraços caranavalescos,
Esteban.






SUBJECT: Zodiaco e Acupuntura..:-)
FROM: "Oraculo" <oraculo@atibaia.com.br>
TO: <ciencialist@yahoogrupos.com.br>
DATE: 04/02/2005 16:49

Olá Esteban

Há uma sequencia de enganos em suas colocações, talvez derivada de um compromentimento com a crença inicial e uma certa aversão a ciência..:-). Métodos usados na pesquisa cientifica não servem apenas para impedir que alegações reais, mas incomodas, sejam demonstradas. Servem para filtar alegações sem base na realidade.

Assim, acusar os que pesqusiasm com ironias do tipo "santos céticos imparcialistas" é bobagem e uma falacia "ad hominem". A não ser que credite tudo a Grande Conspiração Mundial Da Ciência Para Esconder A Verdade. E nesse caso, nada do que eu, ou qualquer outro diga, fará a menor diferença, certo?..:-)

Este trecho é um problema:

Talvez devesse dar algum crédito mesmo. Já estão acreditando em
acupuntura... Mas foi testada, nê, botaram uns eletrodozinhos e viram que
tinham razão aqueles outros velhinhos há mais de 3000 anos atrás...
Acordaaaaa!!!!

Isso já foi explicado diversas vezes, mas vai de novo..:-) E, não precisa acreditar em mim, basta apenas pensar sobre o que argumento e ver se é possível refutar, ok?

Acupuntura é a cura de todas as doenças pela manipulação de meridianos de energia não detectável e mística através da implantação de agulhas em pontos determinados desses meridianos. Isso é acupuntura e a base dessa afirmação é que a causa de todas as doenças é o desequilibrio de energias não detectáveis.

Recentemente, detectou-se algum efeito na sintese de prostaglandinas ao se aplicar agulhas na pele de pacientes. Como as prostaglandinas estão envolvidas nas sensações dolorosas, isso pode causar algum efeito do tipo anestésico leve pelas agulhas.

Mas, isso NÃO É MAIS ACUPUNTURA. Isso é outra coisa, uma nova descoberta sobre efeitos de agulhas em sintese de prostaglandinas. Nào exige que os meridianos sejam identificados, nào exige que pontos especificos sejam encontrados, e pode ser substituida por beliscões, em seu efeito nas prostaglandinas.

Distorcer um conceito, como a definição de acupuntura, para caber em descobertas recentes e cientificas, é um procedimento padrão em pseudociencias, também usado pela astrologia. Mas é evidente que, se não se baseia na causa energética das doenças, se não exige o reequilíbrio dessas energias, se independe de meridianos por onde essa enegia corre, não é, claro, acupuntura.

O que a acupuntura alega, e que nào é reconhecido pela ciência por não ter se mostrado eficaz e nào ter mecanismo de ação reconhecivel, é que doenças são causadas por desequilibrios de energia e que podem ser curadas reequilibrando essas energias com agulhas em pontos específicos e bem determinados da pele. Ela não alega que espetar agulhas (ou causar sensações dolorosas de algum tipo) na pele das pessoas, interfere na sintese de prostaglandinas, podendo servir como anestésico leve.

Veja, nào deve acreditar em mim, deve apenas analisar o argumento. Se puder demonstrar que está incorreto, ótimo. Se não puder, então ele é valido e deve se manter até que algo mude (por exemplo, descobrir que energias nào detectáveis causam tuberculose).

Aqueles velhinhos de 3000 anos continuam enganados quanto a suas alegações. Continuam enganados ao afirmarem que doenças são causadas por desequilibrios energéticos. Continuam enganados ao afirmar que a energia nào detectável corre por linhas determinadas chamadas meridianos. Continuam enganados ao afirmar que podem curar doenças reequilibrando essas energias. Continuam enganados ao afirmar que o reequilibrio se dá com agulhas em pontos determinados da pele. E nào estão enganados sobre a sintese de prostaglandinas, simplesmente porque nada sabiam sobre sintese de prostaglandinas ou sobre qualquer outro sistema metabolico do corpo humano.

Homero



----- Original Message -----
From: Esteban Moreno
To: ciencialist@yahoogrupos.com.br
Sent: Friday, February 04, 2005 4:24 PM
Subject: Re: [ciencialist] Re: Zodiaco



Mr Takata wrote:
> descrições astrológicas?
Entao, em principio pareceria 'suspeito'. Mas como dito, qdo
devidamente testada a astromancia nao parece ter um bom poder de predicao.

E:
Devidamente testada por quem? Pelos santos céticos imparcialistas da
ciência?
Depende de quem ou como se pesquisa. Há uma série de artefatos e falácias do
que julgam como pesquisas comprobatórias. Não vou enumera-las, mas talvez
ajude a lembrar que a pesquisa da universidade de brasília resultou em dados
bastante promissores. Que péssimo! Uma das justificativas dos céticos acerca
do porquê os pacientes sentem-se bem descritos pelo astrólogo é a chamada
confluência (auto-atribuição é um nome pobre para isso). Abaixo segue um
artigo científico que "desmistifica" (perdoe-me..) este recurso:

Self-attribution, sun-sign traits, and the alleged role of favourableness as
a moderator variable:
long-term e.ect or artefact? Edgar Wunder* Gesellschaft fu¨r Anomalistik,
Postfach 1202, 69200 Sandhausen, Germany Personality and Individual
Differences 35 (2003) 1783-1789 www.elsevier.com/locate/paid

Posso envia-lo em anexo.

Takata:
> Um bom parâmetro é a quantidade do mercado que vem se
> ampliando extensivamente, incluindo grandes empresas e corporações.
Esse na verdade eh um pessimo parametro. Se for por isso deveremos
creditar confianca na homeopatia, passes espirituais (sem falar na
cirurgia mediunica), toda sorte de mandingas...

E:
Talvez devesse dar algum crédito mesmo. Já estão acreditando em
acupuntura... Mas foi testada, nê, botaram uns eletrodozinhos e viram que
tinham razão aqueles outros velhinhos há mais de 3000 anos atrás...
Acordaaaaa!!!!


Infelizmente só retorno daqui a 10 dias.
Um carnaval cheio de boas mandingas para vocês.
Esteban.



##### ##### #####

Para saber mais visite
http://www.ciencialist.hpg.ig.com.br


##### ##### ##### #####


Yahoo! Grupos, um serviço oferecido por:







------------------------------------------------------------------------------
Links do Yahoo! Grupos

a.. Para visitar o site do seu grupo na web, acesse:
http://br.groups.yahoo.com/group/ciencialist/

b.. Para sair deste grupo, envie um e-mail para:
ciencialist-unsubscribe@yahoogrupos.com.br

c.. O uso que você faz do Yahoo! Grupos está sujeito aos Termos do Serviço do Yahoo!.



[As partes desta mensagem que não continham texto foram removidas]



SUBJECT: Re: [ciencialist] Re: Zodiaco
FROM: "Oraculo" <oraculo@atibaia.com.br>
TO: <ciencialist@yahoogrupos.com.br>
DATE: 04/02/2005 16:51

Olá Esteban

risos..:-) Com certeza..:-) Se pelo menos a astrologia acrescentasse algum conhecimento verificável a seu conjunto de afirmações, ficaria mais interessante, não?..:-)

Homero

PS: Oraculo é só um nick de brincadeira, muito antigo, pode me chamar de Homero..:-)

----- Original Message -----
From: Esteban Moreno
To: ciencialist@yahoogrupos.com.br
Sent: Friday, February 04, 2005 4:07 PM
Subject: Re: [ciencialist] Re: Zodiaco


Sr. Oracle
A cada temporada, novos astrologos e defensores da astrologia retornam. A
cada temporada, os mesmos argumentos e a mesma falta de evidencias.
Cansativo..:-)


E: Mas são exatamente os mesmos defensores, deve ser o cansanso mesmo...
Um abraço,
Esteban.



##### ##### #####

Para saber mais visite
http://www.ciencialist.hpg.ig.com.br


##### ##### ##### #####


Yahoo! Grupos, um serviço oferecido por:

São Paulo Rio de Janeiro Curitiba Porto Alegre Belo Horizonte Brasília




------------------------------------------------------------------------------
Links do Yahoo! Grupos

a.. Para visitar o site do seu grupo na web, acesse:
http://br.groups.yahoo.com/group/ciencialist/

b.. Para sair deste grupo, envie um e-mail para:
ciencialist-unsubscribe@yahoogrupos.com.br

c.. O uso que você faz do Yahoo! Grupos está sujeito aos Termos do Serviço do Yahoo!.



[As partes desta mensagem que não continham texto foram removidas]



SUBJECT: P/ Gisele (era Zodiaco)
FROM: "Oraculo" <oraculo@atibaia.com.br>
TO: <ciencialist@yahoogrupos.com.br>
DATE: 04/02/2005 17:12

Olá Esteban

Bem, tenho que concordar com a ultima parte, a ciência nos libertará..:-) Mas nem todo mundo gosta da liberdade, então, nem sempre é satisfatório..:-)

E claro que se deve experimentar, mas apenas experimentar é pouco. E, como sabemos a bastante tempo, experiências enganam nosso sentidos (vide as ilusões de ótica e a frase ver para crer..:-). Por isso, depois de muito experimentar e muito falhar em analisar essas experiencias, surgiu um sistema de controle, de rigor experimental, chamado de método cientifico, que usa diversas ferramentas, como raciocinio lógico, pensamento crítico, controles de repetição, etc (esse etc é enorme..:-) para separar os enganos e filtar as falhas de percepção humanas.

É claro também que mesmo esse rigor não é perfeito e as vezes falha. Mas não falha como as outras opções que temos (nem de longe..:-) como revelaçvão, crenças. livros sagrados, conehcimento ancestral, etc.

Assim, se eu experimento algo, por exemplo, um fenomeno espantoso ou quase inexplicável, tenho duas escolhas. Posso, entre diversas explicações sem evidencias para este tipo de fenomeno, todas de algum modo conflitantes entre sí, escolher qualquer uma, ou posso determinar um método para avaliar e aplicar a todas elas, de forma a ter elementos que me permitam escolher com mais chance de estar escolhendo a explicação real. Não se pede que acredite em anda, apenas que use algum mecanismo de rigor na escolha.

Ao fruto do conhecimento obtido dessa forma, se deu o nome de ciência. Que não explica tudo, nem alega faze-lo, mas alega que o que explica, explica com mais confiabilidade e eficácia que outras formas de faze-lo..:-)

Nós sabemos que é possivel criar algo como um mapa astral com grande dose de acerto sem usar astrologia, apenas com os processos naturais da mente humana e conhecimento específico sobre ela. Assim, ao analisar algo espantoso como um mapa astral que parece acertado, devemos levar isso em conta, não?

Apenas isso, levar em conta que é possível que o resultado não seja causado pelas forças que a astrologia alega existirem. Nada mais..:-) Uma dúvida, ainda que pequena, sobre uma afirmação..:-)

Se preferir, pode tentar entender mais como nossa mente (minha e sua inclusive) funciona. Coletar dados e argumentos. Experimetnar novamente, mas com outro enfoque, talvez informando ao astrologo uma data de nascimento incorreta, apenas para comparar. Ou informar a data e hora de uma amiga e depois mostrar, sem que ela saiba (duplo cego) o resultado. Diga que é seu, e veja se ela a reconhece no mapa. Ou peça outro mapa a outro astrologo e procure as discrepancias (e não acertos) entre os dois..:-)

Só estou sugerindo que, antes de comprar de forma definitiva a explicação sobrenatural, experimente mais um pouco..:-) E o faça da mesma forma que os cientistas fazem, com mais dados, com elementos de ajuste, com experimentos de controle, etc.

É só o que se espera do método, que ajude a separar uma nova droga que cura uma doença, de uma nova droga que simplesmente tem um efeito de placebo, onde os pacientes pensam que se curam da doença.

Mais uma vez, não creia na ciência, nunca. Faça como os cientistas, duvide sempre, mesmo quando quem afirma é outro cientista famoso. Repita o experimento do cientista famoso, mesmo que ele tenha um Nobel e mesmo que seja o maior especialista no assunto, antes de aceitar o estudo. Refaça, em diferentes laboratorios, tudo de novo. Mude parametros, procure falhas. Só aceite os resultados e conclusões depois de enorme esforço em provar que ele está incorreto. E apenas se não conseguir, pode reconhecer a maior probabilidade (e não a certeza) de estar correto..:-)

Você é quimica. Não acredite no mol, refaça os experimentos que levaram ao conceito de mol (ou leia a literatura sobre o esforço cientifico para cria-lo..:-). A cada vez que repete com seus alunos os experimentos de quimica em classe, está novamente duvidando da ciência. E a cada vez que os experimentos dão o mesmo resultado, está reafirmando a confiabilidade do mesmo.

Estará fazendo ciência, e não acreditando em nada..:-)

Um abraço.

Homero



----- Original Message -----
From: Esteban Moreno
To: ciencialist@yahoogrupos.com.br
Sent: Friday, February 04, 2005 4:35 PM
Subject: Re: [ciencialist] Re: Zodiaco


Oracle
Olá Gisele risos..:-) Não se preocupe, não será escomungada de modo nenhum,
até porque descrentes e céticos malvados não tem escomunhão entre suas armas
e predicados..:-)

E:
É verdade, Gisele, não será excomungada, já passamos por isso, naturalmente
estava brincando. Mas de alguma forma poderá ser sempre lembrada com uma das
pobres vitimas das falácias auto-indutivas dos astrólogos. Uma vítima
inocente, mal preparada, incapaz de perceber que foi iludida por um a
análise
fria e arranjada, para que você se sentisse bem com um série de dizeres
inconsistentes e vazios, mas que pareciam falar de você. Tantas milhares e
milhões de
pobres vítimas deste mundo de Tsnunamis astrais, e não para de crescer...
Mas você pode se regenerar, minha cara!! Creia na ciência, experimente,
digo, não experimente, porque experimentar algo infundado é próprio dos
tolos. Esqueça estas falácias astrománticas, a ciência vos libertará!!


Oracle:
Talvez fosse interessante começar por aqui:
http://www.projetoockham.org/pseudo_astrologia_1.html

E:
É mesmo um bom ponto de partida o projeto ockham. Depois destrincho para
você.


Abraços caranavalescos,
Esteban.






##### ##### #####

Para saber mais visite
http://www.ciencialist.hpg.ig.com.br


##### ##### ##### #####


Yahoo! Grupos, um serviço oferecido por:

São Paulo Rio de Janeiro Curitiba Porto Alegre Belo Horizonte Brasília




------------------------------------------------------------------------------
Links do Yahoo! Grupos

a.. Para visitar o site do seu grupo na web, acesse:
http://br.groups.yahoo.com/group/ciencialist/

b.. Para sair deste grupo, envie um e-mail para:
ciencialist-unsubscribe@yahoogrupos.com.br

c.. O uso que você faz do Yahoo! Grupos está sujeito aos Termos do Serviço do Yahoo!.



[As partes desta mensagem que não continham texto foram removidas]



SUBJECT: P/ Gisele - correçao (era Zodiaco)
FROM: "Oraculo" <oraculo@atibaia.com.br>
TO: <ciencialist@yahoogrupos.com.br>
DATE: 04/02/2005 17:16

Olá Gisele

Ops..:-) Comecei o testo anterior com Olá Esteban, mas era para você. Como estava respondendo a mensagem do Esteban, acabei errando na saudação..:-) Desculpe, e leia a mensagem anterior como se fosse para você..:-)

Um abraço.

Homero



----- Original Message -----
From: Oraculo
To: ciencialist@yahoogrupos.com.br
Sent: Friday, February 04, 2005 5:12 PM
Subject: [ciencialist] P/ Gisele (era Zodiaco)


Olá Esteban

Bem, tenho que concordar com a ultima parte, a ciência nos libertará..:-) Mas nem todo mundo gosta da liberdade, então, nem sempre é satisfatório..:-)

E claro que se deve experimentar, mas apenas experimentar é pouco. E, como sabemos a bastante tempo, experiências enganam nosso sentidos (vide as ilusões de ótica e a frase ver para crer..:-). Por isso, depois de muito experimentar e muito falhar em analisar essas experiencias, surgiu um sistema de controle, de rigor experimental, chamado de método cientifico, que usa diversas ferramentas, como raciocinio lógico, pensamento crítico, controles de repetição, etc (esse etc é enorme..:-) para separar os enganos e filtar as falhas de percepção humanas.

É claro também que mesmo esse rigor não é perfeito e as vezes falha. Mas não falha como as outras opções que temos (nem de longe..:-) como revelaçvão, crenças. livros sagrados, conehcimento ancestral, etc.

Assim, se eu experimento algo, por exemplo, um fenomeno espantoso ou quase inexplicável, tenho duas escolhas. Posso, entre diversas explicações sem evidencias para este tipo de fenomeno, todas de algum modo conflitantes entre sí, escolher qualquer uma, ou posso determinar um método para avaliar e aplicar a todas elas, de forma a ter elementos que me permitam escolher com mais chance de estar escolhendo a explicação real. Não se pede que acredite em anda, apenas que use algum mecanismo de rigor na escolha.

Ao fruto do conhecimento obtido dessa forma, se deu o nome de ciência. Que não explica tudo, nem alega faze-lo, mas alega que o que explica, explica com mais confiabilidade e eficácia que outras formas de faze-lo..:-)

Nós sabemos que é possivel criar algo como um mapa astral com grande dose de acerto sem usar astrologia, apenas com os processos naturais da mente humana e conhecimento específico sobre ela. Assim, ao analisar algo espantoso como um mapa astral que parece acertado, devemos levar isso em conta, não?

Apenas isso, levar em conta que é possível que o resultado não seja causado pelas forças que a astrologia alega existirem. Nada mais..:-) Uma dúvida, ainda que pequena, sobre uma afirmação..:-)

Se preferir, pode tentar entender mais como nossa mente (minha e sua inclusive) funciona. Coletar dados e argumentos. Experimetnar novamente, mas com outro enfoque, talvez informando ao astrologo uma data de nascimento incorreta, apenas para comparar. Ou informar a data e hora de uma amiga e depois mostrar, sem que ela saiba (duplo cego) o resultado. Diga que é seu, e veja se ela a reconhece no mapa. Ou peça outro mapa a outro astrologo e procure as discrepancias (e não acertos) entre os dois..:-)

Só estou sugerindo que, antes de comprar de forma definitiva a explicação sobrenatural, experimente mais um pouco..:-) E o faça da mesma forma que os cientistas fazem, com mais dados, com elementos de ajuste, com experimentos de controle, etc.

É só o que se espera do método, que ajude a separar uma nova droga que cura uma doença, de uma nova droga que simplesmente tem um efeito de placebo, onde os pacientes pensam que se curam da doença.

Mais uma vez, não creia na ciência, nunca. Faça como os cientistas, duvide sempre, mesmo quando quem afirma é outro cientista famoso. Repita o experimento do cientista famoso, mesmo que ele tenha um Nobel e mesmo que seja o maior especialista no assunto, antes de aceitar o estudo. Refaça, em diferentes laboratorios, tudo de novo. Mude parametros, procure falhas. Só aceite os resultados e conclusões depois de enorme esforço em provar que ele está incorreto. E apenas se não conseguir, pode reconhecer a maior probabilidade (e não a certeza) de estar correto..:-)

Você é quimica. Não acredite no mol, refaça os experimentos que levaram ao conceito de mol (ou leia a literatura sobre o esforço cientifico para cria-lo..:-). A cada vez que repete com seus alunos os experimentos de quimica em classe, está novamente duvidando da ciência. E a cada vez que os experimentos dão o mesmo resultado, está reafirmando a confiabilidade do mesmo.

Estará fazendo ciência, e não acreditando em nada..:-)

Um abraço.

Homero



----- Original Message -----
From: Esteban Moreno
To: ciencialist@yahoogrupos.com.br
Sent: Friday, February 04, 2005 4:35 PM
Subject: Re: [ciencialist] Re: Zodiaco


Oracle
Olá Gisele risos..:-) Não se preocupe, não será escomungada de modo nenhum,
até porque descrentes e céticos malvados não tem escomunhão entre suas armas
e predicados..:-)

E:
É verdade, Gisele, não será excomungada, já passamos por isso, naturalmente
estava brincando. Mas de alguma forma poderá ser sempre lembrada com uma das
pobres vitimas das falácias auto-indutivas dos astrólogos. Uma vítima
inocente, mal preparada, incapaz de perceber que foi iludida por um a
análise
fria e arranjada, para que você se sentisse bem com um série de dizeres
inconsistentes e vazios, mas que pareciam falar de você. Tantas milhares e
milhões de
pobres vítimas deste mundo de Tsnunamis astrais, e não para de crescer...
Mas você pode se regenerar, minha cara!! Creia na ciência, experimente,
digo, não experimente, porque experimentar algo infundado é próprio dos
tolos. Esqueça estas falácias astrománticas, a ciência vos libertará!!


Oracle:
Talvez fosse interessante começar por aqui:
http://www.projetoockham.org/pseudo_astrologia_1.html

E:
É mesmo um bom ponto de partida o projeto ockham. Depois destrincho para
você.


Abraços caranavalescos,
Esteban.






##### ##### #####

Para saber mais visite
http://www.ciencialist.hpg.ig.com.br


##### ##### ##### #####


Yahoo! Grupos, um serviço oferecido por:

São Paulo Rio de Janeiro Curitiba Porto Alegre Belo Horizonte Brasília




------------------------------------------------------------------------------
Links do Yahoo! Grupos

a.. Para visitar o site do seu grupo na web, acesse:
http://br.groups.yahoo.com/group/ciencialist/

b.. Para sair deste grupo, envie um e-mail para:
ciencialist-unsubscribe@yahoogrupos.com.br

c.. O uso que você faz do Yahoo! Grupos está sujeito aos Termos do Serviço do Yahoo!.



[As partes desta mensagem que não continham texto foram removidas]



##### ##### #####

Para saber mais visite
http://www.ciencialist.hpg.ig.com.br


##### ##### ##### #####


Yahoo! Grupos, um serviço oferecido por:







------------------------------------------------------------------------------
Links do Yahoo! Grupos

a.. Para visitar o site do seu grupo na web, acesse:
http://br.groups.yahoo.com/group/ciencialist/

b.. Para sair deste grupo, envie um e-mail para:
ciencialist-unsubscribe@yahoogrupos.com.br

c.. O uso que você faz do Yahoo! Grupos está sujeito aos Termos do Serviço do Yahoo!.



[As partes desta mensagem que não continham texto foram removidas]



SUBJECT: Transplante de célula pode curar diabetes
FROM: "Oraculo" <oraculo@atibaia.com.br>
TO: <ciencialist@yahoogrupos.com.br>
DATE: 04/02/2005 17:59

Olá

Não é preciso "acreditar" na ciência envolvida na descoberta abaixo, mas, se um filho ou ente querido tiver diabetes do tipo 1, seria interessante considerar que a confiabilidade e a probabilidade desse conheciemnto ser real é bem alta (embora nunca 100%) e provavelmente maior que usar acupuntura ou homeopatia para curar a doença ..:-)

Homero

http://www.bbc.co.uk/portuguese/ciencia/story/2005/02/050204_diabetesbg.shtml

Transplante de célula pode curar diabetes, dizem médicos


Médicos japoneses anunciaram nesta sexta-feira ter transplantado com sucesso células produtoras de insulina de uma mãe para sua filha portadora de diabetes.
Uma equipe do Hospital da Universidade de Kyoto removeu células da mãe saudável e as transplantou para sua filha, de 27 anos.

O experimento pioneiro, dizem os médicos, pode se tornar uma forma eficaz de tratar a diabetes do tipo 1.

O transplante pode solucionar o problema da transferência de estruturas microscópicas conhecidas como "ilhotas pancreáticas".

Essas células, encontradas no pâncreas, produzem a insulina, que regula os níveis de açúcar no sangue. A diabetes do tipo 1 destrói essas células e impossibilita o corpo de produzir insulina.

Assas células já foram extraídas de órgãos de doadores mortos. No entanto, as ilhotas costumam ser danificadas após serem armazenadas em locais frios ou após sofrerem a ação de toxinas presentes no sangue após a morte.

As pessoas que sofrem dessa doença costumam ter de aumentar o nível de insulina em seus organismos de forma artificial. Se o nível de insulina dos portadores desse mal não for controlado, eles podem entrar em coma e morrer.

Esperança

O transplante pode representar uma esperança para esses pacientes pois daria a eles novas ilhotas pancreáticas capazes de produzir insulina.

O médico James Shapiro, da Universidade de Alberta, no Canadá, foi um dos cirurgiões que comandou o experimento no Japão.

De acordo com ele, as células começaram a produzir insulina minutos após terem sido transplantadas para a paciente.

Antes da cirurgia, ela havia entrado em coma mais de uma vez devido a baixos níveis de açúcar em seu sangue.

Após o transplante, o nível de açúcar em seu sangue voltou ao nível normal.


[As partes desta mensagem que não continham texto foram removidas]



SUBJECT: RE: [ciencialist] Re: Zodiaco
FROM: "murilo filo" <avalanchedrive@hotmail.com>
TO: ciencialist@yahoogrupos.com.br
DATE: 04/02/2005 19:30

Pois eu tenho uma previsão para alguns de vcs.
( e nada tenho a ganhar com isto! )
Alguns felizardos, e privilegiados, terão, algum dia, a oportunidade de
conhecer a um astrólogo ( a ) SÉRIO, COMPETENTE E C O N S C I E N T E...
quando então ficarão bem desconfiados a respeito de uma arte tão antiga
quanto a humanidade. Ficarão bem encafifados e também verão o quanto a coisa
é diferente daquilo que sai nos jornais.
Todo aquêle cascão da má informação e da superstição irá cair fora.
Está nos astros... aguardem e/ou procurem. Alguns ficarão menores, mas de pé
no chão (e ainda assim seguirão sem qualquer explicação cartesiana, tipo
batatolina!).
A gente não pode gastar a vida só duvidando da inteligência dos outros, por
todo o tempo, pô!
Sinceramente, M. SP 04/fev

>From: Maria Natália <grasdic@hotmail.com>
>Reply-To: ciencialist@yahoogrupos.com.br
>To: ciencialist@yahoogrupos.com.br
>Subject: [ciencialist] Re: Zodiaco
>Date: Fri, 04 Feb 2005 04:05:36 -0000
>
>
>Sérgio:
>
>Agradeço.
>A astrologia se usa aqui para fazer estatísitica a brincar. Pega-se
>num jornal e se vê qual o signo que diz " vais cair do escadote". Só o
>professor sabe qual o jornal escolhido e todos respondem ao
>questionário. Depois os que eram do signo da previsão "má" sofrem
>comparação.Mas todos preenchem o inquérito. As perguntas também
>permitem tirar conclusões sobre supertições, crenças e mitos. Quanto
>mais turmas mais científico será (amostra maior)
>Ao Ofiuco também se chama Serpentário, me lembrei agora.
>Na escola costumamos apenas nos referirmos às constelações mais
>visíveis em cidades com poluição luminosa. Ora se se souber que aqui
>onde vivo da Ursa Menor apenas se vê a Polar e mesmo meia "safada" nem
>capricónio nos interessa para meninos de 11 e 12 anos.
>Quando os alunos fazem a "fita" do Zodíaco na parede da sala de aula,
>cúbica ou paralelipipédica, se representam todas para se perceber essa
>do "sol está na constelação..." sendo o Sol um aluno que anda junto às
>paredes simulando o movimento aparente e a Terra os restantes alunos
>num foco. De resto as constelações só começa a ter muito valor quando
>queremos ver cometa ou asteróide
>Não gostamos quando, a meio da noite, nos aparecem adultos a perguntar
>se pelo facto de Vénus estar em Virgem eles vão ter mais sorte no
>Totoloto.
>Daí as alergias com a astrologia.
>E este ano tive até um pai engenheiro a perguntar-me se achava que o
>professor X iria faltar muito...O home estava a confundir astrónomo
>com astrólogo. Aceitamos bem melhor sermos gastrólogos e gastrónomos.
>Mas a astrologia dá para muita brincadeira. É que este pessoal que nos
>chega à mão aos 11 anos já tem pelo menos 3 anos de laboratórios e
>trabalhos de campo. Vem com a mania de apanhar a alma na ponta de um
>bisturi. Vamos ver o que esta geração dará na universidade dentro de 4
>anos. Pois se trata da reforma do ensino em curso.
>Entretanto já lera o artigo do Galileu noutra lista, o CdA
>
>Abraço
>Maria Natália
>PS Pois como agora estou a dar aulas longe de casa o tempo vai
>falhando para acompanhar diariamente a lista...
>
>--- Em ciencialist@yahoogrupos.com.br, "Sergio M. M. Taborda"
><sergiotaborda@t...> escreveu
> > Maria Natália wrote:
> >
> > >
> > > Estebam:
> > > O zodíaco dos astrólogos é o de há 2000 anos e não tem em conta os
> > > movimentos do eixo da Terra. O zodíaco tem 13 constelações: a seguir
> > > ao capricónio tem o Ofiuco...São 13 os tais signos. E agora que me
> > > dizeis acerca do Ofiuco e que de acordo com o zodíaco actual é o meu
> > > signo: poderei amanhã comprar lotaria para me sair prémio?
> >
> > Eu ja respondi a essa pergunta. E a resposta é : Esse argumento faria
> > sentido SE a astrologia fosse baseada nas constelações.
> > Como não é , insistir nesse argumento é pura besteira (=coisas que os
> > bestas fazem)
> > Jà lhe expliquei pq so ha 12 signos, o que já foi aqui explicado por
> > outros. É uma razão matemática para que seja assim. É baseada no
> > conceito de Periodo Sinódico.
> > Ha uma logica , séria, matemática, fisica, _astronomica_ para o numero
> > 12. Só que aos astronomos - com problemas de falta de atenção -
> > inventaram esse argumento sobre Ofucio.
> > fazer o quê ? Mas se vc acredita nesses mentirosos, não é diferente de
> > quem acredita em astrologos fajutos.
> >
> > > Desculpe, por acaso ouvi dizer que os astrónomos e astrofísicos falam
> > > besteira? Ou entendi mal?
> >
> > Ouvio bem. Eles dizem mesmo. E ha muitas mais do que a de Ofucio.
> >
> >
> > Sérgio Taborda
> >
> >
> >
> > --
> > No virus found in this outgoing message.
> > Checked by AVG Anti-Virus.
> > Version: 7.0.300 / Virus Database: 265.8.5 - Release Date: 03-02-2005
>
>
>




SUBJECT: RE: [ciencialist] SKN20/02
FROM: "murilo filo" <avalanchedrive@hotmail.com>
TO: ciencialist@yahoogrupos.com.br
DATE: 04/02/2005 19:46

Não sou boa gente, mas aquí vai.
O SKN 20/02 está fora de linha e seu carregador, talvez um bom Kita, deve
ser velho prá burro.
Êstes diodos, com cordoalha ou rabicho, onde está o catodo, foram
substituidos pelos SKN 21/02, que, se não me engano, mudou p/SKN 25, não
estou certo.
Na rua Santa Ifigênia há revendedores da Semikron e, com certeza, também aí
pelo interior.
Se achar, o A1B 26, ou 25, da aegis também servirá.
A corrente total do aparelho é de 40A DC, *máximos*, com certeza.
Agora só na 4ª ou 5ª f., se vc ainda precisar, posso dar uma mãozinha.
Na firma, estou sem internet, uma josta. :[ abr/M.

>From: "Luiz Ferraz Netto" <leobarretos@uol.com.br>
>Reply-To: ciencialist@yahoogrupos.com.br
>To: <eletronicabr@yahoogrupos.com.br>,"ciencialist"
><ciencialist@yahoogrupos.com.br>,Fábio Henrique Cicuto Gonçalves
><fabio.henrique.gonca@terra.com.br>,<radiotecnico@yahoogrupos.com.br>
>Subject: [ciencialist] SKN20/02
>Date: Fri, 4 Feb 2005 15:36:16 -0200
>
>Oi boa gente,
>
>preciso de 4 diodos retificadores SKN20/02 (ou apenas 2 para 200V, 40A)
>pois os mesmos no meu carregador de baterias pifaram (devido a um maledeto
>'curto'). Aqui em Barretos não tem disso.
>Se alguém puder enviar-me entre em contato em PVT para combinarmos as
>'transferências'.
>
>Agradeço,
> ===========================
> Luiz Ferraz Netto [Léo]
> leobarretos@uol.com.br
> http://www.feiradeciencias.com.br
> ===========================
>
>
>--
>No virus found in this outgoing message.
>Checked by AVG Anti-Virus.
>Version: 7.0.300 / Virus Database: 265.8.5 - Release Date: 03/02/2005
>




SUBJECT: Re: [ciencialist] Re: Plural das unidades
FROM: JVictor <jvoneto@uol.com.br>
TO: ciencialist@yahoogrupos.com.br
DATE: 04/02/2005 22:06

Manuel Bulcão escreveu:

>
> --- Em ciencialist@yahoogrupos.com.br, "Hugo Santos" <urano@n...>
> escreveu
> > Portanto, tanto é 0,5 mol como é 50 mol. Tal como se diz também
> 0,5 grama e 50 grama.
>
> Manoel: Se não me engano, depende de como a unidade de medida está
> expressa:
>
> 50 gramas - 50g
> 10 quilômetros - 10km
> 8 horas - 8h
> 3 horas e quarenta minutos - 3h40 ou 3h40m


Victor: Mais uma vez você está certo. As unidades físicas nunca levam o
"s" de plural, exceto quando escritas por extenso, como nos exemplos acima.

> (Há um bom manual de Sisstemas de Unidades Físicas, de: Herbert
> F.Pinto, Editora Científica)

> Sds,


Victor.

>
>
>
>
> ##### ##### #####
>
> Para saber mais visite
> http://www.ciencialist.hpg.ig.com.br
>
>
> ##### ##### ##### #####
>
>
> *Yahoo! Grupos, um serviço oferecido por:*
>
> *
> <http://br.rd.yahoo.com/SIG=12afqtlp3/M=264105.3931087.6562589.1588051/D=brclubs/S=2137111528:HM/EXP=1107575593/A=2361264/R=6/SIG=10v4acpp0/*http://br.shopping.yahoo.com/>*
>
>
>
> ------------------------------------------------------------------------
> *Links do Yahoo! Grupos*
>
> * Para visitar o site do seu grupo na web, acesse:
> http://br.groups.yahoo.com/group/ciencialist/
>
> * Para sair deste grupo, envie um e-mail para:
> ciencialist-unsubscribe@yahoogrupos.com.br
> <mailto:ciencialist-unsubscribe@yahoogrupos.com.br?subject=Unsubscribe>
>
> * O uso que você faz do Yahoo! Grupos está sujeito aos Termos do
> Serviço do Yahoo! <http://br.yahoo.com/info/utos.html>.
>
>
>
>
> __________ Informação do NOD32 1.990 (20050202) __________
>
> Esta mensagem foi verificada pelo NOD32 Sistema Antivírus
> http://www.nod32.com.br




SUBJECT: RE: [ciencialist] SKN20/02
FROM: "murilo filo" <avalanchedrive@hotmail.com>
TO: ciencialist@yahoogrupos.com.br
DATE: 04/02/2005 22:17

Perdão... cometí um engano muito básico aao especificar o diodo aegis
equivalente.
Seu código deve ser A1A 25 ou 26. O A1B possui o catodo na base (rôsca M6).
Sorry! M.

>From: "murilo filo" <avalanchedrive@hotmail.com>
>Reply-To: ciencialist@yahoogrupos.com.br
>To: ciencialist@yahoogrupos.com.br
>Subject: RE: [ciencialist] SKN20/02
>Date: Fri, 04 Feb 2005 19:46:41 -0200
>
>Não sou boa gente, mas aquí vai.
>O SKN 20/02 está fora de linha e seu carregador, talvez um bom Kita, deve
>ser velho prá burro.
>Êstes diodos, com cordoalha ou rabicho, onde está o catodo, foram
>substituidos pelos SKN 21/02, que, se não me engano, mudou p/SKN 25, não
>estou certo.
>Na rua Santa Ifigênia há revendedores da Semikron e, com certeza, também aí
>pelo interior.
>Se achar, o A1B 26, ou 25, da aegis também servirá.
>A corrente total do aparelho é de 40A DC, *máximos*, com certeza.
>Agora só na 4ª ou 5ª f., se vc ainda precisar, posso dar uma mãozinha.
>Na firma, estou sem internet, uma josta. :[ abr/M.
>
> >From: "Luiz Ferraz Netto" <leobarretos@uol.com.br>
> >Reply-To: ciencialist@yahoogrupos.com.br
> >To: <eletronicabr@yahoogrupos.com.br>,"ciencialist"
> ><ciencialist@yahoogrupos.com.br>,Fábio Henrique Cicuto Gonçalves
> ><fabio.henrique.gonca@terra.com.br>,<radiotecnico@yahoogrupos.com.br>
> >Subject: [ciencialist] SKN20/02
> >Date: Fri, 4 Feb 2005 15:36:16 -0200
> >
> >Oi boa gente,
> >
> >preciso de 4 diodos retificadores SKN20/02 (ou apenas 2 para 200V, 40A)
> >pois os mesmos no meu carregador de baterias pifaram (devido a um
>maledeto
> >'curto'). Aqui em Barretos não tem disso.
> >Se alguém puder enviar-me entre em contato em PVT para combinarmos as
> >'transferências'.
> >
> >Agradeço,
> > ===========================
> > Luiz Ferraz Netto [Léo]
> > leobarretos@uol.com.br
> > http://www.feiradeciencias.com.br
> > ===========================
> >
> >
> >--
> >No virus found in this outgoing message.
> >Checked by AVG Anti-Virus.
> >Version: 7.0.300 / Virus Database: 265.8.5 - Release Date: 03/02/2005
> >
>
>




SUBJECT: Re: [ciencialist] Re: Zodiaco
FROM: "Sergio M. M. Taborda" <sergiotaborda@terra.com.br>
TO: ciencialist@yahoogrupos.com.br
DATE: 04/02/2005 22:41

rmtakata wrote:

>
> > Vc acha que a ligação é meramente arbitrária, mas eu podria
> > descorrer aqui, por não o é.
>
> Nao duvido nem um pouco disso. Os astromantes no minimo falariam de
> sincronicidade.
>
N\ao me faça rir com esses tipo de argumentos. Sejamos serios.

> > Não. Leão simboliza a coragem (coração+agem=acção do coração), mas
> > não a ferocidade.
>
> Ferocidade nao quer dizer crueldade. Um dos manuais astromantes diz: o
> leonino eh corajoso, justo e digno. Mas claro q. apresenta tb aspectos
> negativos - por curiosidade sao aproximadamente antagonicos aos
> aspectos positivos, de modo q. praticamente qq pessoa possa se
> encaixar na descricao generica: orgulhoso, autoritario, megalomano.


Vc não disse ferocidade, pq ? PQ o manual não diz lá ferocidade.
E ninguem identificou ferocidade com crueldade, isso são ideias suas.
Os simbolos são em muito boa parte adjectivos e é preciso ter cuidado
para não usar qq um.
Orgulho, autoritarismo e megalomania nada têm a ver com ferocidade. São
tudo coisas do coração.
O Leão sente mais do que parece, mas ao contrario da cancer, ele esconde
isso. Tal como um pai,
que tem que ser autoritário para manter o respeito mas que ama os seus
filhos.

Quanto a bons e maus atributos, a primeira coisa que vc aprende com a
astrologia é que nenhum atributo é apenas bom ou mau.
Não é curiosidade, não é coincidencia, é a prorpia essencia da astrologia.
Orgulho , autoridade e magalomania podem ser coisas boas.

(A palavra Aspecto tem um significado tecnico em astrologia, por isso
que mudei para atributo)

> > Ora, mas é a ideia do 5º signo que tem como menomonica o leão, ou o
> > leão que empretou as suas caracteritsticas ao signo ?
> > É a primeira e não a segundo, como vc supoe.
> > Qualquer coisa poderia ser atruida à constelação de leão ,pq um
> > leão?
> > Ora, afinal aquilo é so um triagulo com uma ponta
>
> Eles enxergaram um leao. Sim, poderiam ter enxergado outra coisa, mas
> se assim o fizesse, teriam atribuido ao setor outra coisa.

Isso é o que vc diz. Em que se baseia para afirmar isso. Qual é o dado
cientifico, historico, sei lá - se
é que ha algum dado - que aponte na direcção que diz ?
O efeito é ao contrario, eles teriam escolhido outro simbolo se o
significado fosse diferente. E um significado por ter muitos simbolos.
Vc não escolhe o verde para avançar e o vermelho para parar pq está
relacionando as corres com acções.
Vc relacion as cores com o significado depois dele ser atribuido. E vc
sabe muito bem pq o vermelho é para parar.
Foi escolhido essa cor para esse significado por motivos logicos: os
nossos olhos respondem melhor ao vermelho que ao verde.

Analizando de fora, a correlação entre aparecer vermelho e vc parar é
muito elevada. Matemáticamente falando existe uma relação, um corelação.
O que significa que ha uma relação causa-efeito. Mas todos sabemos que
essa correlação causa efeito não é vinculada, que não é uma lei da
natureza.
A astrologia baseia-se na mesma coisa. Correlações de causa-efeito, sem
dizer nada sobre os vinculos fisicos. Qq coisa que vc ler em contrario
disto é
pura 'astrologia' de 4º classe.

>
>
> > > ter um aspecto de libra significa ser equilibrado,
> >
> > Não. Significa indiciso. Que não sabe tomar parte, que não sabe
> > decidir entre as opções.
>
> Isso significa o aspecto negativo. No aspecto positivo significa
> ponderação. Contraditorio? Sem duvida.

Como vc ainda nao entendeu que uma so coisa pode ser boa e má, vc acha
contradittorio.
Como vc não sabe que essa é a regra astrologia e não mera coincidencia,
vc acha que é oportuno a todos os "astromantes" (alias , esse termo é
bem ofensivo)
que assim seja. É tão oportuno como para a todos os fisicos, que a
gravidade seja atractiva. (pense nisso, ela é atractiva por convensão)


> Isso eh muito bom para a
> astromancia, jah q. pode encaixar tto os librianos equilibrados, qto
> os indecisos.

O facto é que eles são as duas coisas. Em algumas areas serão
equilibrados e outras indecisos.
Uma so pessoa será as duas coisas. Ao contrario do que vc afirma que uma
pessoas será uma coisa e outra , será outra.
Não, cada libraniano será as duas coisas. O mapa, revelará em quais
areas é equilibrado e em quais não é.
E não estamos falando de equilibrio mental.

> > Compare com a astrologia Indu (a atrologia original onde se baseia a
> > ocidental actual). Nele os mesmos signos, com as mesms
> > exactas posições não têm nada a ver com as estrelas no sentido de
> > que não ha correspondencia entre os nomes das constrelçaões
>
> Lamento, mas a astromancia hindu nao eh uma astromancia independente
> da astromancia ocidental.

That's the point! O ponto é esse mesmo. Elas não o são, mas assim mesmo
a indu atriui simbolos
diferentes aos mesmos sigificados. O significa que o significando vem
antes, e o simbolo é apenas uma menmonica.

> Bebem de uma fonte comum e houve intensa
> comunicacao. Tto eh q. dados astronomicos hindus - junto com a
> matematica - vieram para o ocidente e vice-versa


Obrigado. É exactamente isso. Então como vc esplica que os simbolos
sejam outros ?
Se a sua teoria fosse verdaeira , os simbolso seriam os mesmos pois as
estrelas são as mesmas.
Pense nisso, tlv vc entenda que a astrologia nada tem a ver com as
estrelas.

> > > Temos um caso aqui de uma construcao
> > > argumentativa a posteriori.
> >
> > Não, não temos. Vc acha que sim pq não conhece um pouco da historia
> > da astrologia.
>
> Sua exposicao de defesa nao eh convincente.

Como se a sua fosse ... lol.. estou so demontanto seus argumentos (hei,
estou dizendo q vc tem argumentos!...).

> A mesma coisa acontece com
> os planetas. O planeta vermelho acabou sendo ligado 'a guerra e essas
> coisas - assim Marte - e antes, seus equivalentes babilonios, egipcios
> e demais - acabou por reger as relacoes masculinas, de forca e coisas
> assim.

Tudo bem. Então pq os outros planetas não são referidos como o planeta
<cor>?

Pq Jupiter não é o planeta laranja, ou Venus o planeta Roxo ?

Pela mesma razão que apontei antes. Dado que o significado é ligado ao
sange, e marte aparece aos nossos olhos,
vermelho, é natural que a cor vire uma menmnonica. Se fosse a cor a
gerar o significado, o que dizer dos outros planetas ?

> Como Venus - ou seus equivalentes babilonicos - foi
> identificada com uma divindade feminina, ganhou suposta regencia sobre
> atributos femininos. E assim por diante.

Depende. Outro nome para Venus é Luficer. Que todos sabem com o que se
relaciona.
(Sabemos ? Quando leu lucifer ter ter pensando no demonio , que é
simbolizado pelo vermelho.
Mas Lucifer não é nada disso. É a Estrela da Manhã e da Tarde. É dai que
vem a ligação com a beleza.
E por isso o mito de Venus-Afrodite é ligado com a beleza. Vc já viu o
fenomeno ? Etão sabe como é bonito e confortante.

> Trata-se de uma construcao
> argumentativa a posteriori dizer q. o q. importa sao os setores e os
> nomes dos setores sao apenas um detalhe historico (ou sao efeitos e
> nao causas das qualidades atribuidas a cada setor e outros elementos
> de configuracao na carta natal).

Você parece estar muito convencido disso, ignorando toda amatemática
envolvida na astrologia, a
forma como se constroi o circulo zodiacal, as regras que estão sobre ele.
Tudo o que não tem nada a ver com os nomes, e que vc não vai ler nos
horosocopos do jornal.
Vc prefere fixar-se nos nomes do que pensar um pouco como cientista e
humano e pensar
nas regras. Esse é um argumento ... parco - para dizer o menos.
É como vc negar a mecanica quantica pq não gosta do nome quantum.
As regras não foram criadas agora , por isso o argumento do argumento à
posteriori não vale. O sinodico da lua cai 12 vezes no periodo do sol
- faça as contas - , várias astrologias diferentes (como a ocidental e a
indu) tem os mesmos significados mas com simbolos diferentes. Os quais
não dependem de nenhumas estrelas.
A astrologia é baseada em 3 astros, o sol, a lua e a terra. (As eras
astrologicas não são nada mais que a precessão da orbita da terra em
torno do sol. ) e não nas estrelas.
Escrito nas estrelas é apenas uma expressão poetica que não corresponde
com a realidade da teoria. Exactamente como a frase de Eisntein "Deus
não joga aos dados" não corresponde com o que ele disse, mas toda a
gente acha que sim.
Pessoas desinformadas aceitam o mito, aceita a frase de Eisntein tal
como o seu argumento , ou os da Natália, pq não se dão ao trabalho de
procurar as fontes fidedignas.
E que eu acho mais curioso é a critica à astrologia ser baseada nos
mitos levantados em cima dela , em vez de nela mesma. Seria como
criticar Eisntein por ter dito aquela frase, que ele nunca disse. Ele
nunca usou a palavra "deus" - mas vc so entenderá pq se souber sobre a
vida de Eisntein e não do mito da vida dele.
A discussão sobre astrologia pelos seus criticos comete os mesmos erros.
E vc dá ouvidos a quem critica Eisntein por ter dito "Deus não joga aos
Dados" ? Então pq vc dá ouvidos a quem critica o mito e não a historia
real?

O pior são os que se dizem ceticos que cometem estes erros
atropeladamente todos os dias em relação a tudo o que eles não conhecem.
Eles seguem o mito, e não o separam da coisa que é mesmo.
Que tipo de cepticos são que acreditam que o mito exprime a realidade ?
Vc está entrando no mesmo jogo.



--
No virus found in this outgoing message.
Checked by AVG Anti-Virus.
Version: 7.0.300 / Virus Database: 265.8.5 - Release Date: 03-02-2005



SUBJECT: RE: [ciencialist] Zodiaco e Acupuntura..:-)
FROM: "murilo filo" <avalanchedrive@hotmail.com>
TO: ciencialist@yahoogrupos.com.br
DATE: 04/02/2005 23:06

Oraculo, oi.
Vc está parecendo o Padre Quevedo fazendo ginástica anti-espírito!
Conseguí achar um negócio interessante sobre a homeopatia, que também foi
''atacada'' nesta discussão!
FAVOR irem a www.arenales.com.br e lá encontrarão muita informação,
onde especialmente destaco muita ciência e homeopatia para ANIMAIS. Um lance
psicossomático, certamente.
A arte de ''sugestionar'' animais é maravilhosa, não? :]]] abr/M.

>From: "Oraculo" <oraculo@atibaia.com.br>
>Reply-To: ciencialist@yahoogrupos.com.br
>To: <ciencialist@yahoogrupos.com.br>
>Subject: [ciencialist] Zodiaco e Acupuntura..:-)
>Date: Fri, 4 Feb 2005 16:49:32 -0200
>
>Olá Esteban
>
>Há uma sequencia de enganos em suas colocações, talvez derivada de um
>compromentimento com a crença inicial e uma certa aversão a ciência..:-).
>Métodos usados na pesquisa cientifica não servem apenas para impedir que
>alegações reais, mas incomodas, sejam demonstradas. Servem para filtar
>alegações sem base na realidade.
>
>Assim, acusar os que pesqusiasm com ironias do tipo "santos céticos
>imparcialistas" é bobagem e uma falacia "ad hominem". A não ser que credite
>tudo a Grande Conspiração Mundial Da Ciência Para Esconder A Verdade. E
>nesse caso, nada do que eu, ou qualquer outro diga, fará a menor diferença,
>certo?..:-)
>
>Este trecho é um problema:
>
>Talvez devesse dar algum crédito mesmo. Já estão acreditando em
>acupuntura... Mas foi testada, nê, botaram uns eletrodozinhos e viram que
>tinham razão aqueles outros velhinhos há mais de 3000 anos atrás...
>Acordaaaaa!!!!
>
>Isso já foi explicado diversas vezes, mas vai de novo..:-) E, não precisa
>acreditar em mim, basta apenas pensar sobre o que argumento e ver se é
>possível refutar, ok?
>
>Acupuntura é a cura de todas as doenças pela manipulação de meridianos de
>energia não detectável e mística através da implantação de agulhas em
>pontos determinados desses meridianos. Isso é acupuntura e a base dessa
>afirmação é que a causa de todas as doenças é o desequilibrio de energias
>não detectáveis.
>
>Recentemente, detectou-se algum efeito na sintese de prostaglandinas ao se
>aplicar agulhas na pele de pacientes. Como as prostaglandinas estão
>envolvidas nas sensações dolorosas, isso pode causar algum efeito do tipo
>anestésico leve pelas agulhas.
>
>Mas, isso NÃO É MAIS ACUPUNTURA. Isso é outra coisa, uma nova descoberta
>sobre efeitos de agulhas em sintese de prostaglandinas. Nào exige que os
>meridianos sejam identificados, nào exige que pontos especificos sejam
>encontrados, e pode ser substituida por beliscões, em seu efeito nas
>prostaglandinas.
>
>Distorcer um conceito, como a definição de acupuntura, para caber em
>descobertas recentes e cientificas, é um procedimento padrão em
>pseudociencias, também usado pela astrologia. Mas é evidente que, se não se
>baseia na causa energética das doenças, se não exige o reequilíbrio dessas
>energias, se independe de meridianos por onde essa enegia corre, não é,
>claro, acupuntura.
>
>O que a acupuntura alega, e que nào é reconhecido pela ciência por não ter
>se mostrado eficaz e nào ter mecanismo de ação reconhecivel, é que doenças
>são causadas por desequilibrios de energia e que podem ser curadas
>reequilibrando essas energias com agulhas em pontos específicos e bem
>determinados da pele. Ela não alega que espetar agulhas (ou causar
>sensações dolorosas de algum tipo) na pele das pessoas, interfere na
>sintese de prostaglandinas, podendo servir como anestésico leve.
>
>Veja, nào deve acreditar em mim, deve apenas analisar o argumento. Se puder
>demonstrar que está incorreto, ótimo. Se não puder, então ele é valido e
>deve se manter até que algo mude (por exemplo, descobrir que energias nào
>detectáveis causam tuberculose).
>
>Aqueles velhinhos de 3000 anos continuam enganados quanto a suas alegações.
>Continuam enganados ao afirmarem que doenças são causadas por
>desequilibrios energéticos. Continuam enganados ao afirmar que a energia
>nào detectável corre por linhas determinadas chamadas meridianos. Continuam
>enganados ao afirmar que podem curar doenças reequilibrando essas energias.
>Continuam enganados ao afirmar que o reequilibrio se dá com agulhas em
>pontos determinados da pele. E nào estão enganados sobre a sintese de
>prostaglandinas, simplesmente porque nada sabiam sobre sintese de
>prostaglandinas ou sobre qualquer outro sistema metabolico do corpo humano.
>
>Homero
>
>
>
> ----- Original Message -----
> From: Esteban Moreno
> To: ciencialist@yahoogrupos.com.br
> Sent: Friday, February 04, 2005 4:24 PM
> Subject: Re: [ciencialist] Re: Zodiaco
>
>
>
> Mr Takata wrote:
> > descrições astrológicas?
> Entao, em principio pareceria 'suspeito'. Mas como dito, qdo
> devidamente testada a astromancia nao parece ter um bom poder de
>predicao.
>
> E:
> Devidamente testada por quem? Pelos santos céticos imparcialistas da
> ciência?
> Depende de quem ou como se pesquisa. Há uma série de artefatos e
>falácias do
> que julgam como pesquisas comprobatórias. Não vou enumera-las, mas
>talvez
> ajude a lembrar que a pesquisa da universidade de brasília resultou em
>dados
> bastante promissores. Que péssimo! Uma das justificativas dos céticos
>acerca
> do porquê os pacientes sentem-se bem descritos pelo astrólogo é a
>chamada
> confluência (auto-atribuição é um nome pobre para isso). Abaixo segue um
> artigo científico que "desmistifica" (perdoe-me..) este recurso:
>
> Self-attribution, sun-sign traits, and the alleged role of
>favourableness as
> a moderator variable:
> long-term e.ect or artefact? Edgar Wunder* Gesellschaft fu¨r
>Anomalistik,
> Postfach 1202, 69200 Sandhausen, Germany Personality and Individual
> Differences 35 (2003) 1783-1789 www.elsevier.com/locate/paid
>
> Posso envia-lo em anexo.
>
> Takata:
> > Um bom parâmetro é a quantidade do mercado que vem se
> > ampliando extensivamente, incluindo grandes empresas e corporações.
> Esse na verdade eh um pessimo parametro. Se for por isso deveremos
> creditar confianca na homeopatia, passes espirituais (sem falar na
> cirurgia mediunica), toda sorte de mandingas...
>
> E:
> Talvez devesse dar algum crédito mesmo. Já estão acreditando em
> acupuntura... Mas foi testada, nê, botaram uns eletrodozinhos e viram
>que
> tinham razão aqueles outros velhinhos há mais de 3000 anos atrás...
> Acordaaaaa!!!!
>
>
> Infelizmente só retorno daqui a 10 dias.
> Um carnaval cheio de boas mandingas para vocês.
> Esteban.
>
>
>
> ##### ##### #####
>
> Para saber mais visite
> http://www.ciencialist.hpg.ig.com.br
>
>
> ##### ##### ##### #####
>
>
> Yahoo! Grupos, um serviço oferecido por:
>
>
>
>
>
>
>
>------------------------------------------------------------------------------
> Links do Yahoo! Grupos
>
> a.. Para visitar o site do seu grupo na web, acesse:
> http://br.groups.yahoo.com/group/ciencialist/
>
> b.. Para sair deste grupo, envie um e-mail para:
> ciencialist-unsubscribe@yahoogrupos.com.br
>
> c.. O uso que você faz do Yahoo! Grupos está sujeito aos Termos do
>Serviço do Yahoo!.
>
>
>
>[As partes desta mensagem que não continham texto foram removidas]
>




SUBJECT: Re: [ciencialist] Re: Zodiaco
FROM: "Sergio M. M. Taborda" <sergiotaborda@terra.com.br>
TO: ciencialist@yahoogrupos.com.br
DATE: 04/02/2005 23:31

Oraculo wrote:

> Olá Taborda
>
> A respsta é sempre a mesma: prove isso tudo. Demonstre que leoninos
> são bons pais, que tem relação com seu signo, etc.

Eu não disse que os leoninos são bons pais. Não me obrigue a provar o
que eu não afirmei , nem é verdade.

>
>
> Sem isso (e continuamos sem isso..:-),

E vai continuar sem isso, pq vc quer que eu prove mentiras.

> O caso dos setores é evidente. Se a astrologia não existisse, mas
> fosse inventada (inventada é um termo preciso..:-)


Acaso vc não saiba, a astrologia foi inventada sim. Tal como a fisica, a
quimica, a electronia , a politica a economia .. . alias como qq
actividade intelectual do ser humano.

> hoje, teriamos 13 signos, e 13 setores.


Prove que sim. Em que se baseia para dizer isso ?
Ah , ja sei, aquela asneira sobre as constelações. Otimo. Divida o ceu
em 13 partes, quantos graus tem cada uma ?
Vc esqueceu que 13 é um numero primo ? Acaso o que lhe ensinaram na
escola não serve para nada ? Vc não sabe que não se pode divir um numero
por um numero primo e obter um numero inteiro, a menos que esse numero
seja um multiplo dele ? Ah ! afinal vc diz-se muito seguidor do
pensamento logico, mas falha nestes pequeno promenores. Que raios vc
pensa afinal ?

Vc esquece o elementar. A astrologia indu não usa graus. Foram os
gregos que intruziram toda a matematica da coisa.
Se originalmente tivessem escolhido um signo para cada constelação
teriam escolhido 13. Ou vc acha que a 13º constelação aprecer do dia
para a noite ? Acha que eles não a viram ?
Se assim fosse o circulo teria hoje 520 graus (13x4) e não 360.
Mas não, escolheram 12. que os gregos multiplicaram por 4. O pq do 4 é
outra historia.

Se a sua teoria sobre as constalções fosse certa, explique pq eles
ignoraram a 13º.

Tlv isto ajude, vc sabia que o periodo sinodico da lua (29 dias e alguma
coisa) cai 12, e não 13, vezes no do sol

365 / 29 = 12

O que sobra é a precessão do zodiaco , e por isso que o zodiaco
astrologico não é mais sincronizado com o zodiaco celeste.
Matemática, meu caro, não é magia. É Matemática. E lembre-se que eles
fizeram o calculo sem computadores e sem medições de periodos siderais
apenas pelo bom e velho metodo que aprendemos na escola primária.. .bom,
pelo menos eu aprendi.

Vc imagina que a astrologia foi inventada como ? Baseada em quê ?
Palhaçada ?
Prove-se que faz sentido logico-amtematico-astronomico escolher o numero
13 (que é um numero primo) e darei o braço a torcer.

> Como poderia alguém que estivesse criando a astrologia hoje determinar
> que um dos setores não deveria ser utilizado? Como ele teria acesso a
> essa informação, se existem 13 constelações a considerar ?

Se vc soubesse mais de astronoia básica, saberia explicar pq, tal como
eu fiz acima.

>
> Como o astrologo nos informou, valem os setores criados pelos
> assirios, com base em seu céu visivel na época.

E como eles os criaram ? Por magia ? Vamos, lá., vc - o ceptico -
acreditando em magia ?

> Se criada hoje, a astrologia seria de 13 signos e totalmente diferente
> em seus significados e interpretações e os mapas astrais diferentes
> nos resultados.

Vc continua repetindo isso mas sem explicar pq . Qual a razão , a regra,
que faria isso acontecer ? Pq 13 ?

> Como pode isso ser uma fonte de conhecimento confiável ou mesmo real?

A sua ou a minha ?

> Conhecimento que determinava que o Sol e a Lua eram planetas e agiam
> como tal, inclusive girando em torno da Terra (a Lua até que gira,
> embora não como os planetas, mas o Sol..:-).

Se vc soubesse mais de fisica, saberia que o sol sim gira em torno da
terra , tal como a lua, tal tudo o resto, para quem está na terra. Para
o sistema do observador solidário com a terra, tudo o resto anda À volta
dela. É por falta deste conhecimento que pouca gente entende
Relatividade (Galiena mesmo, para não falar da outra). A Primeira regra
é que o que vc observa depende do referencial que esolhe.
A astrologia, não nega que o seu referencial é a Terra. E como tal, é
correcto, observatorialmente e até fisicamente, dizer que é o sol que se
move em torno da terra.

Toda a discussão sobre se o sol era o centro ou não era uma discussão
astrologia. Tlv vc não soubesse disto, e pensasse que era uma discussão
fisica, ou filosofica... .
Mas claro, vc não acredita nisso.

Planeta é uma palavra grega , femenina , que significa Estrela Errante.
Qq fisico sabe que aquilo que não se move, não contem informação. É no
movimento, na mudança das coisas, que está a informação. Então é nos
Planetas, Estrelas que se movem, e não nas Estrelas fixas que deveriamos
concentrar o estudo. É por isso que em astrologia chamamos a lua e o sol
de planetas. Não é para chatear os astronomos. É para ser fiel ao
significado das palavras. Pq eles são estrelas que se movem (são astros
luminosos)
Mas pq Astrologia e não Planetalogia ? Pq o que a astrologia realmente
estuda não são os planetas em si, suas posições etc... isso é apenas uma
ferramenta. O que interessa são os arranjos que eles fazem uns com os
outros no ceu. A forma como eles se agrupam. A forma como as estrelas
errantes de dispoem relativamente umas às outras.Aquilo que hoje
chamamos Aspectos. Mas como se chamava a disposição de estrelas umas em
relação às outras no tempo dos gregos?
O mesmo que hoje :Constelação. Que em grego se diz, Astron e dai
Astron+Logia = Astrologia = Estudo/Catalogo das Constelações , dos
arranjos das estrelas.
Mas entenda de que estrelas estamos falando aqui!
Estamos falando das estrelas que se movem, as Planetas e não das
Estrelas fixas. UAU! Agora vc ficou confuso , sim ? Otimo.

A astrologia é o estudo das estrelas e das constelações, mas não
"estrela" e "constelaçao" no sentido vulgar que vc dá a essas palavras
hoje , e sim, a que era dada à milhares de anos atrás. Eles não faziam
destinção ente Estrela e Planeta como os astornomos. Tudo eram estrelas,
mas umas moviam-se e outras não. (Tudo isto do ponto de vista de quem
olha da Terra) . As que não se movem não contém informação e server
apenas como referencia visual do referencial asbtracto que estamos
usando. As que se movem, as Planetas, forma configurações diversas no
ceu, e são essas configurações que estavam interessados em estudar.
Esses arranjos, essas constelações de estrelas moveis, e não as
constelações de estrelas fixas que não têm nenhum importancia para o
fenomeno. (Um referencial, nunca tem)

Deu para entender pq a astrologia é aquilo que é e pq os argumentos como
os seus e os do takata são ocos de sentido logico ? Pq eles são mais
bestas do que os argumentos dos astrologos em si ?
Se não deu é pura má fá da sua parte. Acho que o texto é bem explicativo.

> Sempre mais do mesmo. Apenas prove o que afirma, e nenhuma discussão
> seria necessária.

Acabei de o fazer.
Claro que vc não está convencido, pq vc escolhe não entender, mas não
pode negar que a logia está ali. A historia, a metamática, a fisica e
até a semântica, confirmam.
Que mais provas vc quer ?

Sérgio Taborda


--
No virus found in this outgoing message.
Checked by AVG Anti-Virus.
Version: 7.0.300 / Virus Database: 265.8.5 - Release Date: 03-02-2005



SUBJECT: Re: [ciencialist] Re: Zodiaco
FROM: "Sergio M. M. Taborda" <sergiotaborda@terra.com.br>
TO: ciencialist@yahoogrupos.com.br
DATE: 04/02/2005 23:55

Oraculo wrote:

>
> "A amplitude dos signos pode ir alem da amplitude de 16 graus que
> define tradicionalmente a faixa zodiacal, definindo gomos que partem
> da Terra, tem por fronteiras a divisão em 12 do equador celeste e tem
> por fundo o infinito."
>
> Bem, e de onde saiu essa informação? O que garante sua validade?

A matemática. (Parece que vc a esqueceu toda) Em particular geometria.
Como se calcula a posição astrologia de um objecto ? Calcula-se a
posição que os astronomos chama de Ascenção Direita que é a coordenada
Teta no sistema esferico de referencia, ou seja, vai de 0 a 360 graus.
Divide-se essaposição por 30 , e obtemos o signo. O planeta está nesse
signo, e o resto da divisao será a posição dele nesse signo.
Ora, pq o signo vai além dos 16 graus ? Pq esses 16 graus dizem respeito
à coordenada fi do sistema esferico que so vai de -90 a 90 graus, e
sendo teta e fi, independentes , não importa qual o fi do objecto e
apenas o teta. Por isso , mesmo que o objecto estivesse fora da faixa
dos 16 graus , ele continuaria no mesmo signo. Por isso , se fala em
gomos, pois essa é a forma que vc obtem de um esfera dividida em partes
tomando teta como separador periodio.
Bolas , é geometria. É tão simples!

> Por que a amplitude é de 16 graus e não 20 ou 5 graus?

Pergunte as astronomos , foram eles que inventaram essa. As astrologos
interessa um pepino o valor da Declinação.

> Por que gomos?

Aprenda geometria e coma mais laranjas.

> Por que 12?

Aprenda a ler. São 12 os signos, lembra-se ?

> Sim, eram 12 na época dos criadores do zodiaco, mas, se estes tivessem
> criado sua explicação dos céus mil anos antes, talvez fossem 11 ou 13
> ou 15..:-) Teriamos hoje 15 "setores" e "gomos" e não apenas 12...:-)

Claro, as estrelas desaparcem assim tão rápido.... Uma cosntelação
inteira apareceu de repente... vc quer que eu acedite que no tempo dos
assirios, sumerios, gregos, arabes, etc... só existiam 12 cosntelações e
hoje existem 13 ???? É isso que vc quer que acreditemos ? Que uma
constelação nasce em menos de 5000 anos Isto contando sumerios, pq os
arabes foi à menos de 1000? Vc está dizendo que o ceu que os arabes viam
era diferente do que vemos hoje ? que a 13º cosntelação apareceu do
nada em menos de 1000 anos ?
O que vc diz ainda é mais ridiculo do que o os astrologos dizem.

>
> De toda forma, a explicação do astrologo não deveria produzir mais que
> uma única indagação: E daí?..:-) O que isso tudo importa, o que isso
> tudo informa sobre a validade de um conhecimento absolutamente sem
> evidencias como a astrologia?
>
> E a afirmação final de que astronomos tem medo da astrologia

Os astronomos não têm medo, eles têm terror. E são completos idiotas por
isso.
são idiotas, pq baseiam os seus contra-argumentos em crendices, mitos e
palermices, em vez de na astrologia em si mesma.
A primeira regra para a critica, é conhecer o objecto a criticar. E os
astronomos, e os ceticos em geral, nem isso conhecem.
Afinal é pedir de mais que se conheça matemática, astronomia, semantica
e que ainda se decorem uma serie de algoritmos e regras e em cima disso
um catalogo extenso de significados das inumeras combinações possiveis
de aspectos. É demasiado para a cabecinha dos astronomos de hoje (que só
sabem as duas primeira, e até duvido que saibam da primeira) e dos
cepticos, que afinal nunca dão provas de perceber realmente de coisa
nenhuma , pois o bom ceptico é aquele que duvida até das suas prorpias
crenças ,e se não as tem, sempre duvida se não as tem realmente.

Não sei como concegue dormir todas as noites com essa sua falta de
execicio do metodo cientifico, que vc presa tanto e apregoa a 7 ventos
que todos deveriam entender, até os leigos. Parece-me que vc não
entendeu ainda a primeira regra. Imparcialidade. Vc ja decidiu o
resultado antes de conduzir o estudo. ( eo mais divertido é que o fez
baseado em informações falsas.... lá se vai o metodo)

A astrologia pode ser toda mentira. Tal como a historia do StarWars ,
mas para poder falar sobre starWars é preciso conhecer a historia.
A historia pode ser ficiticia, mas seria um erro afirmar que o Luke se
casou com a Leia, que o Vader era o demonio, sei lá. Isso falta à
verdade da historia, mesmo sendo ela ficticia. Usar argumentos que se
baseiam na distorção das coisas não funciona, mesmo que a historia
subjacente seja ficticia.


Sérgio Taborda






--
No virus found in this outgoing message.
Checked by AVG Anti-Virus.
Version: 7.0.300 / Virus Database: 265.8.5 - Release Date: 03-02-2005



SUBJECT: Re: [ciencialist] Re: Zodiaco
FROM: "Sergio M. M. Taborda" <sergiotaborda@terra.com.br>
TO: ciencialist@yahoogrupos.com.br
DATE: 04/02/2005 23:59

Oraculo wrote:

>
> Nós (os malvados céticos..:-) apenas evitamos escolher uma explicação
> para um fenomeno sem antes pensar bastante sobre ele


Como vc pode pensar sobre algo que nunca viu ou testemunhou ? Fenomeno é
algo que se pode observar. Como vc pensa sobre algo que não observou
nunca ? e que até diz não ser observável.
Veja, o nivel do seu argumento é o mesmo de quem acredita em deus.
Também pensa nele, sem nunca o ter visto.
Os cepticos são bem mais crentes que os religiosos.

Sérgio



--
No virus found in this outgoing message.
Checked by AVG Anti-Virus.
Version: 7.0.300 / Virus Database: 265.8.5 - Release Date: 03-02-2005



SUBJECT: Re: [ciencialist] Re: Zodiaco
FROM: "Sergio M. M. Taborda" <sergiotaborda@terra.com.br>
TO: ciencialist@yahoogrupos.com.br
DATE: 05/02/2005 00:05

Esteban Moreno wrote:

> Mas mesmo que o fizessem, é provável que assumissem uma mesma postura
> arrogante e incondicional do "Bussunda" ao ser interpelado pelo astrólogo
> Pedro Tornagui, quando previu sem conhece-lo que teria grande talento para
> comunicação e poderia ser um ótimo jornalista. Este, pansudamente cético,
> disse que jamais pensou em trabalhar em comunicação, apesar de ter
> feito um
> ano e meio de jornalismo e ser um dos maiores humoristas do Brasil. Ou
> o que
> os vossos representantes da terra arredondada (seria mais cientificamente
> correto do que redonda) fizeram com o astrólogo Alexey em seu suposto
> interesse por uma experimentação, foi algo de covarde e infantil. Talvez
> queiram saber sobre isso depois.


Eu quero saber agora. O que que foi que eles fizeram ?
( a parte do coverdae e infantil não me surpreende, mas estou curioso
pelos detalhes)

E concordo consigo. Qualquer cientista de facto ao menos experimentaria
uma vez
E qualquer BOM cisntista experimentaria mais do que uma vez.
E por isso que nunca considerarei os cepticos ajudantes da ciencia.
São uma mera inquisição dos tempos modernos. Ajudam-se a si mesmos.

Sergio Taborda


--
No virus found in this outgoing message.
Checked by AVG Anti-Virus.
Version: 7.0.300 / Virus Database: 265.8.5 - Release Date: 03-02-2005



SUBJECT: Re: [ciencialist] Re: Zodiaco
FROM: "Sergio M. M. Taborda" <sergiotaborda@terra.com.br>
TO: ciencialist@yahoogrupos.com.br
DATE: 05/02/2005 00:13

Oraculo wrote:

> Olá
>
> Esse argumento faz sentido porque a astrologia FOI baseada nas
> constelações. Os que a criaram usaram as constelaçòes para basear sua
> criação. Nada sobre setores, inventados a posteriori para explicar
> incongruencias. Os criadores nào sabiam nada sobre precessão ou sobre
> estrelas como sois e o universo como setores. Sabiam empiricametne que
> as constelações se cucediam e isso permitia marcar o tempo e
> determinar estaçòes do ano.

A cada mensagem , vc demosntra mais que não sabe nada de fisica,
astronomia, ou sequer tem bom senso.
O estudo astrologico dada de à mais ou menos 5000 anos. Os quais, os
astrologos passaram compilando posições de todos os planetas visiveis.
Vc não acha que 5000 anos é suficiente para perceber a precessão ?
BOLAS!! A propria teoria das eras astrologicas é baseada nisso
Como eles poderiam basear a teoria das eras em algo que não sabiam ?

>
> Hoje os setores sào o que importa, já que fica dificil manter a base
> original da astrologia, mas não foi isso que baseou sua criação.

Prove isso logo de uma vez e terminemos com esta conversa da treta, ou
cale-se para sempre. Ler erros cientificos uns atrás dos outros dá cabo
da minha paciencia.


Sergio Taborda


--
No virus found in this outgoing message.
Checked by AVG Anti-Virus.
Version: 7.0.300 / Virus Database: 265.8.5 - Release Date: 03-02-2005



SUBJECT: Re: Zodiaco e Acupuntura..:-)
FROM: Manuel Bulcão <manuelbulcao@uol.com.br>
TO: ciencialist@yahoogrupos.com.br
DATE: 05/02/2005 06:18


Salve, Homero!

Homero escreveu: Recentemente, detectou-se algum efeito na sintese
de prostaglandinas ao se aplicar agulhas na pele de pacientes. Como
as prostaglandinas estão envolvidas nas sensações dolorosas, isso
pode causar algum efeito do tipo anestésico leve pelas agulhas.

Manuel: Talvez o efeito das agulhas seja um pouco mais potente que a
de um analgésico leve. Parece que a técnica da "acupuntura" (as
aspas são para lhe agradar :-)) é eficaz como tratamento
complementar de dores "crônicas", como a fibromialgia e até mesmo
dores oncológicas.

Homero: Mas, isso NÃO É MAIS ACUPUNTURA. (...) Não exige que os
meridianos sejam identificados, nào exige que pontos especificos
sejam encontrados, e pode ser substituída por beliscões, em seu
efeito nas prostaglandinas.

Manuel: Foi questionando e testando a técnica da "acupuntura" que a
medicina ocidental chegou à conclusão de que, "realmente", a
estimulação de certos pontos do corpo produz como resposta -- via
secreção de determinados hormônios e neurotransmissores -- analgesia
e sedação. A "acupuntura" PREVÊ esse efeito sensível, previsão esta
que foi confirmada pelos testes empíricos. Significa dizer que a
práxis científico-experimental corroborou a "acupuntura" em alguma
medida.

Além disso, parece que, consoante esses testes, e malgrado a
explicação mágica da medicina sino-nipônica arcaica, muitos pontos
corporais que correspondem aos "meridianos" não são arbitrários.

Quanto ao desequlíbrio de "energia" que causa a doença, bem, pelo
menos no que diz respeito à supressão ou amenização dos sintomas
decorrente das agulhadas, isso se deve a processos "eletroquímicos"
(portanto, "energéticos") que começa com um "choque". Não é a
energia que a turma do new age & Cia. sonha, mas é energia, sem
dúvida.

Acho que o problema está mais na forma da explicação do que no poder
de predição (não quero com isso dizer que o poder de predição de
todas as fórmulas da "acupuntura" sejam supimpas, frise-se). Mas,
será que a explicação dos fenômenos naturais EM LINGUAGEM NÃO-
MATEMÁTICA é unívoca? Há quem sustente que qualquer lei física pode
ser enunciada VALIDAMENTE até mesmo na forma de uma lenda ou conto
de fadas em que as forças da natureza figuram como sujeitos
intencionais. Por exemplo, numa dessas estórias da Carochinha, os
glúons seriam tutores liberais que só intervêm duramente quando os
tutelados (quarks) estão passando dos limites (liberdade
assintótica).

No que tange ao termo designativo dessa técnica terapêutica,
pergunto: há uma palavra melhor para nomeá-la que não
seja "acupuntura"? Será que os fatos acima mencionados não a
legitimam hoje em dia? ô, Homero, meu amigo, você que foi tão firme
ao afirmar "ISSO NÃO É MAIS ACUPUNTURA', apresente uma alternativa!

A ciência deve tanto ao conhecimento que a precedeu (ao conhecimento
empírico rasteiro e à metafísica) que tomou para si muitos das suas
imagens, entre as quais a palavra "átomo": termo cuja etimologia o
torna impróprio para designar qualquer coisa que seja composta,
divisível, quebrável, redutível ou desintegrável -- como o é
um "átomo" de urânio.

Então, por que não preservar também o termo "acupuntura"?

Confetes e serpentinas,
Manuel Bulcão





SUBJECT: Re: [ciencialist] Re: Zodiaco e Acupuntura..:-)
FROM: "Luiz Ferraz Netto" <leobarretos@uol.com.br>
TO: <ciencialist@yahoogrupos.com.br>
DATE: 05/02/2005 09:54

"Manuel
> Acho que o problema está mais na forma da explicação do que no poder
de predição (não quero com isso dizer que o poder de predição de
todas as fórmulas da "acupuntura" sejam supimpas, frise-se). Mas,
será que a explicação dos fenômenos naturais EM LINGUAGEM NÃO-
MATEMÁTICA é unívoca? Há quem sustente que qualquer lei física pode
ser enunciada VALIDAMENTE até mesmo na forma de uma lenda ou conto
de fadas em que as forças da natureza figuram como sujeitos
intencionais. Por exemplo, numa dessas estórias da Carochinha, os
glúons seriam tutores liberais que só intervêm duramente quando os
tutelados (quarks) estão passando dos limites (liberdade
assintótica).<

A apresentação e constatação do fenômeno em linguagem não matemática é UNIVOCA; passou um ímã por perto de uma bobina teremos uma f.e.m. e uma d.d.p. entre os terminais dessa. A linguagem matemática para isso *não é unívoca*; E = - d(fi)/dt pode ser obtido por, pelo menos, dois caminhos distintos (um pelo cálculo do trabalho e outro pela lei de Faraday).

a p'ropria segunda lei de Newton pode ser apresentada de outras formas.

[]'
Léo

>Então, por que não preservar também o termo "acupuntura"?<

Léo: por que não "abunduntura"? :-)




--
No virus found in this outgoing message.
Checked by AVG Anti-Virus.
Version: 7.0.300 / Virus Database: 265.8.5 - Release Date: 03/02/2005



SUBJECT: Tração nas quatro rodas...
FROM: "Rick" <rickardorios@yahoo.com.br>
TO: "l - Ciencia" <ciencialist@yahoogrupos.com.br>
DATE: 05/02/2005 09:55

Físicos da lista, peço vossas ajudas em duas questões...

1 - O que quer dizer tração nas quatro rodas de um automóvel? Qual o seu
efeito (beneficio) para o carro? Como a tração atua no movimento do carro?

Thanks for all..
Rick




SUBJECT: Re: Zodiaco e Acupuntura..:-)
FROM: Manuel Bulcão <manuelbulcao@uol.com.br>
TO: ciencialist@yahoogrupos.com.br
DATE: 05/02/2005 13:52


Diga lá, Léo!

Também decidiu não viajar no carnaval? :-)

Léo: A apresentação e constatação do fenômeno em linguagem não
matemática é UNIVOCA; passou um ímã por perto de uma bobina teremos
uma f.e.m. e uma d.d.p. entre os terminais dessa.

Manuel: E se, em vez de "pólo positivo" e "pólo negativo",
utilizássemos os conceitos "esse menino" e "essa bichinha"
ou "romeu" e "julieta"?

Léo: A linguagem matemática para isso *não é unívoca*; E = - d
(fi)/dt pode ser obtido por, pelo menos, dois caminhos distintos
(um pelo cálculo do trabalho e outro pela lei de Faraday).

Manuel: É, você tem razão. Alguns princípios da física quântica
podem ser enunciados tanto nos termos da "mecânica das matrizes"
(Werner Heisenberg) quanto por meio de equações diferenciais (Erwin
Shrödinger).

Até mesmo a linguagem matemática compreende vários dialetos.

Manuel: Então, por que não preservar também o termo "acupuntura"?<

Léo: por que não "abunduntura"? :-)

Manuel: Esse termo é excelente para designar aquele ramo da
acupuntura em que as agulhas são aplicadas nos meridianos da bunda.
Se existe uma acupuntura "auricular", por que não especular acerca
de uma acupuntura "gluteocular"? :-)

Confetes e serpentinas,
Manuel Bulcão





SUBJECT: Re: [ciencialist] Re: Zodiaco
FROM: "Sergio M. M. Taborda" <sergiotaborda@terra.com.br>
TO: ciencialist@yahoogrupos.com.br
DATE: 05/02/2005 14:51

Maria Natália wrote:

>
> Sérgio:
>
> Agradeço.
> A astrologia se usa aqui para fazer estatísitica a brincar. Pega-se
> num jornal e se vê qual o signo que diz " vais cair do escadote". Só o
> professor sabe qual o jornal escolhido e todos respondem ao
> questionário. Depois os que eram do signo da previsão "má" sofrem
> comparação.Mas todos preenchem o inquérito. As perguntas também
> permitem tirar conclusões sobre supertições, crenças e mitos. Quanto
> mais turmas mais científico será (amostra maior)

Mas que palhaçada é esta ? É assim que se ensina ciencia ? Estou
escandalizado que o dinheiro dos contribuintes seja gasto com esta
palhaçada.
Palhaçada pq não ha outro nome, é um circo de falta de cientificidade. E
depois ainda tem coragem de dizer que aumentando a amostragem aumenta a
cientificidade !!
Estou enojado e revoltado com isto que contou.
Primeiro, a sua experiencia apenas pode demonstrar que a informação do
jornal é errada. Mas o METODO o famoso, querido, acompanhado, idolatravo
METODO CIENTIFICO MANDA, ele não pede, ele MANDA que a toda a informação
do fenomeno seja obtida de forma imparcial e, independente. E esta
experiencia é baseada em informação reconhecidamente falsa, pelos ppr
astrologos.
Esta experiencia não tem qq valor cientifico, pq viola a primeira rega
de um estudo cientifica, a obtenção de dados de forma imparcial e
independente.
Vc verificou quem escreveu o enunciado do jornal ? vc verificou com
outras informações providas por outras fontes ? O cruzamento de
informação é a primeira coisa a fazer. Pq vc não ensina a fazer isso
primeiro ?
Só depois vc pode escrever questionarios e fazer experiencias. Qualquer
resultado que apareça de uma experiencia assim é lixo.
É por estas e por outras que alguem canta que descobriu a fusão a frio e
depois é mentira. Ninguem mais está preparado para destingir ciencia de
palhaçada. E a culpa é das escolas e professores que passam o tempo no
circo.

> Quando os alunos fazem a "fita" do Zodíaco na parede da sala de aula,
> cúbica ou paralelipipédica,

Que perda de tempo. Ha tantas coisas mais importantes para ensinar ...

> se representam todas para se perceber essa
> do "sol está na constelação..."

O sol não pode estar numa constelação. Onde vc ouviu tamanha idiotice ?
(os astrologos dizem "sol no signo" , ou "sol na casa" ,nunca na
cosntelação)

>
> Não gostamos quando, a meio da noite, nos aparecem adultos a perguntar
> se pelo facto de Vénus estar em Virgem eles vão ter mais sorte no
> Totoloto.

A culpa é toda vossa porque cultivam a mistica da astrologia sem
entender a astrologia de facto.

> Daí as alergias com a astrologia.

Quem tem alergia fica longe. Mas os astrónomos ADORAM começar as suas
palestras com exemplos de astrologia, que eles não entendem.
Argumentos idiotas como o de Ofucio é de uma imbecilidade tão grande de
retira qq respeito cientifico pelo astronomo.
Cientistas verdadeiros não fazem conversa com o que não conhecem. É
baixo , muito baixo.
Mas pronto, eu tb nunca achei que astronomos fossem cientistas. E quanto
mais o tempo passa, mas eu sei que tenho razão.

> E este ano tive até um pai engenheiro a perguntar-me se achava que o
> professor X iria faltar muito...O home estava a confundir astrónomo
> com astrólogo.

Não seria com X-Men ? O que tem o professor faltar muito com astrologia ?
Se o professor é um bundão que falta muito - o que é normal, eu tive
muitos desses - é normal que o pai se preocupe.
Eu me preocuparia.

> Mas a astrologia dá para muita brincadeira.

Exactamente pq estão todos num circo, em vez de numa aula de ciencias.
Seria como fazer paidas sobre sexo, para ensinar o sistema de reprodução
humano.

> É que este pessoal que nos
> chega à mão aos 11 anos já tem pelo menos 3 anos de laboratórios e
> trabalhos de campo. Vem com a mania de apanhar a alma na ponta de um
> bisturi. Vamos ver o que esta geração dará na universidade dentro de 4
> anos.

Dará merda, como todas as que vem chegando lá. E o ensino seundário está
cada vez pior e a culpa
é dos professores e as suas estratégias pedagogicas ridiculas.

Sérgio Taborda



--
No virus found in this outgoing message.
Checked by AVG Anti-Virus.
Version: 7.0.300 / Virus Database: 265.8.5 - Release Date: 03-02-2005



SUBJECT: Re: Fw: Biologia
FROM: "rmtakata" <rmtakata@altavista.net>
TO: ciencialist@yahoogrupos.com.br
DATE: 05/02/2005 15:55


A pergunta eh muito, muito simples. Jah a resposta eh muito, muito
complexa. Os biologos podem no maximo dar a resposta padrao: Biologia
eh o estudo cientifico do fenomeno da vida (ou uma variacao
desenvolvida disso).

Hah diferentes aspectos mais sutis q. podem ser explorados (por
exemplo, se a Biologia estuda a vida, o que eh vida?).

Continua carecendo (muito) desenvolvimento, mas tem um esboco da
resposta a essa pergunta em:

http://www.geocities.com/biomabrasil/

(A resposta O que eh Biologia nesse site vai mais na linha: O que um
biologo faz?)

[]s,

Roberto Takata


--- Em ciencialist@yahoogrupos.com.br, "Luiz Ferraz Netto"
> -----Mensagem Original-----
> leo eu estou cursando o 1º ano do ensino medio, e a
> professora passou um trabalho sobre "o que é biologia?" voce
> poderia mim responder essa pergunta? O QUE É BIOLOGIA?






SUBJECT: Re: Zodiaco
FROM: "rmtakata" <rmtakata@altavista.net>
TO: ciencialist@yahoogrupos.com.br
DATE: 05/02/2005 16:17


--- Em ciencialist@yahoogrupos.com.br, "Esteban Moreno"
> Devidamente testada por quem?

Por quem eh uma pergunta cuja resposta nao tem tta importancia assim.
O q. realmente importa eh *como* foi testada. Experimentos duplos
cegos, isolamento de variaveis, reprodutibilidade e parametros afins
eh q. contam na hora de um teste devido.

Por esse processo q. se desenvolveu o conhecimento q. nos permitiu
criar computadores, mandar homens a lua, criar novos medicamentos,
manipular parte da natureza para o beneficio (de uma parcela) da
humanidade - espera-se gerar tb conhecimento de modo a se realizar a
manipulacao da natureza de modo cada vez mais seguro.

> Talvez devesse dar algum crédito mesmo. Já estão acreditando

De forma alguma. Os logicos classificam isso como um tipo de falacia:
apelo 'a galeria ou algo assim. Do contrario teriamos q admitir q. o
estelionato deve ser algo bom para milhoes de pessoas se submeterem a
isso anualmente. O estelionato no sentido estrito movimenta muito mais
dinheiro do q. a astromancia.

Para q. o argumento 'muitas pessoas usam, entao deve ser bom' seja
viavel, seria preciso q. duas premissas fossem verdadeiras: (1) as
pessoas sao estritamente racionais; (2) as pessoas tEm acesso a
informacoes de qualidade. Os psicologos jah demonstraram muito bem q.
a primeira premissa eh falsa: somos capazes de raciocinio, mas
eminentemente nos movemos por motivos emocionais. A segunda premissa
tb eh falsa como pesquisas de opiniao demonstram (boa parte das
pessoas nos EUA e no Brasil acreditam no criacionismo, sao analfabetas
em geografia, conhecimentos basicos de matematica, linguas, ciencias
em geral) - particularmente no Brasil podemos verificar o desempenho
das pessoas em testes padronizados.

Do q. (1) e (2) sao falsos e na verdade a *maioria* das pessoas agem
movidas em grande parte por emocoes (os publicitarios e mercadologos
conhecem isso muito bem - incentivando a compra por impulso apertando
botoes neurais corretos) e a *maioria* das pessoas nao tEm acesso a
boa informacao. Nao eh surpreendente entao q. em muitos casos a
maioria tome uma decisao incorreta: isso explica a eleicao do Collor,
do Maluf, do Pitta... Com base em informacao errada houve um
linchamento moral dos donos da Escola Base (falsamente acusados de
molestarem seus alunos).

Obviamente q. nao se pode concluir pelo contrario: se a maioria pensa
de um jeito, entao eh certo q. esse jeito seja errado.

Entao o q. fazemos? Testes e verificacoes caso a caso. Ao se fazer
isso vimos q. a homeopatia, a acupuntura, a astromancia, os passes
espiritas e coisas assim nao funcionam do modo alegado. O fato da
homeopatia e a acupuntura terem sancao oficial no Brasil pouco
significa - nao eh uma pratica aprovada nos EUA, por exemplo (claro q.
tampouco se diz q porque nao eh aprovado nos esteites entao nao eh bom
- o q. importa aqui sao os resultados) - em termos de validade real.

Claro, eh direito de cada pessoa acreditar no q. quiser. Se mesmo
depois de ter acesso 'a informacoes sobre o desempenho dos astromantes
em suas previsoes, alguem quiser continuar acreditando na astromancia,
eh direito dela continuar a se consultar com astromantes.

[]s,

Roberto Takata





SUBJECT: O cético que encontrou um bom astrologo..:-) (era Zodiaco)
FROM: "Oraculo" <oraculo@atibaia.com.br>
TO: <ciencialist@yahoogrupos.com.br>
DATE: 05/02/2005 19:01

Olá Murilo

Infelizmente, essa previsão vai falhar.:-)

E o motivo é simples, mesmo que um cético ou cientista se encaixe no caso relatado por você, encontrar um astrologo que faça seu mapa astral com enorme relevância de dados, ainda assim ele saberá que é apenas um caso anedotico (não estatistico) e que isso não representa evidencia confiável.

Veja, não é porque conheço o efeito da percepção seletiva, da pareidolia, do famoso "conte os acertos e ignore os erros" que estou imune a eles..:-) Sei perfeitamente que posso ser enganado por minha mente ou por um bom astrologo (ou qualquer outra forma de alegação sem evidencias) tanto quanto sou enganado por mágicos de palco (que repetem exatamente os mesmos fenomenos sobrenaturais)..

Assim, mesmo que eu encontre um astrologo que faça meu mapa astral, de confiança, competente, e que eu me veja refletido no mapa, ainda assim vou manter o ceticismo natural e pedir uma avaliação de outra pessoa, outra analise, uma significancai estatistca no acerto do astrolog, etc, etc, etc. Eu posso me enganar (e minha mente tanto quanto eu..:-) como qualquer pessoa.

E tem de perceber que a classificação "competente", "consciente", "sério", etc, é um julgamento de valor. Como escolher o astrologo que se encaixe? Como decidir se é competente, ou honesto? Como, engim, filtrar os dados e analisar essa questão? Bem, se posso sugerir algo, devemos, de novo, usar o sistema rigoroso da ciência apra faze-lo..:-) Ou teremos, de novo, de acreditar na palavra de quem nos recomenda o astrologo. Se você decidir analisar o trabalho de um astrologo, apra decidir se ele se encaixa nos termos que propos, o que usará como ferramenta de análise?

É isso que parece dificil de explicar e de entender: não é por falta de experiencia pessoal que cientistas e céticos não acreditam, é apenas por saber que é necessário mais que isso para uma conclusão confiável.

Se eu tomar uma erva, receitada por um curandeiro, e minha doença se curar, ainda assim eu vou manter o ceticismo a respeito do poder real de cura da mesma. E estarei apenas sendo cuidadoso com a conclusão, já que para determinar a eficácia de uma droga ou medicamente são necessários testes rigorosos, duplo cego, estudos longos e muita reprodutibilidade e análise. E mesmo você deve concordar com isso, e se recursar a tomar drogas ou ervas (que podem matar) sem essas precauções, não importa o testemunho de quem quer que seja que tenha se curado ao toma-la, não?

Parece que quem crê em algo pensa que céticos tem falta de experiencias pessoais..:-) Que, como você previu, se encontrarem um "bom astrologo" mudarão de opinião e passarão a acreditar. Não é assim que funciona.:-) Ciência é mais que relatos anedoticos, mesmo os pessoais, mesmo as experiencias que eu experimento. É um conhecimento que só se tornou confiável quando passou a ser produzido dentro do rigor necessário.

Um abraço.

Homero

----- Original Message -----
From: murilo filo
To: ciencialist@yahoogrupos.com.br
Sent: Friday, February 04, 2005 7:30 PM
Subject: RE: [ciencialist] Re: Zodiaco


Pois eu tenho uma previsão para alguns de vcs.
( e nada tenho a ganhar com isto! )
Alguns felizardos, e privilegiados, terão, algum dia, a oportunidade de
conhecer a um astrólogo ( a ) SÉRIO, COMPETENTE E C O N S C I E N T E...
quando então ficarão bem desconfiados a respeito de uma arte tão antiga
quanto a humanidade. Ficarão bem encafifados e também verão o quanto a coisa
é diferente daquilo que sai nos jornais.
Todo aquêle cascão da má informação e da superstição irá cair fora.
Está nos astros... aguardem e/ou procurem. Alguns ficarão menores, mas de pé
no chão (e ainda assim seguirão sem qualquer explicação cartesiana, tipo
batatolina!).
A gente não pode gastar a vida só duvidando da inteligência dos outros, por
todo o tempo, pô!
Sinceramente, M. SP 04/fev

>From: Maria Natália <grasdic@hotmail.com>
>Reply-To: ciencialist@yahoogrupos.com.br
>To: ciencialist@yahoogrupos.com.br
>Subject: [ciencialist] Re: Zodiaco
>Date: Fri, 04 Feb 2005 04:05:36 -0000
>
>
>Sérgio:
>
>Agradeço.
>A astrologia se usa aqui para fazer estatísitica a brincar. Pega-se
>num jornal e se vê qual o signo que diz " vais cair do escadote". Só o
>professor sabe qual o jornal escolhido e todos respondem ao
>questionário. Depois os que eram do signo da previsão "má" sofrem
>comparação.Mas todos preenchem o inquérito. As perguntas também
>permitem tirar conclusões sobre supertições, crenças e mitos. Quanto
>mais turmas mais científico será (amostra maior)
>Ao Ofiuco também se chama Serpentário, me lembrei agora.
>Na escola costumamos apenas nos referirmos às constelações mais
>visíveis em cidades com poluição luminosa. Ora se se souber que aqui
>onde vivo da Ursa Menor apenas se vê a Polar e mesmo meia "safada" nem
>capricónio nos interessa para meninos de 11 e 12 anos.
>Quando os alunos fazem a "fita" do Zodíaco na parede da sala de aula,
>cúbica ou paralelipipédica, se representam todas para se perceber essa
>do "sol está na constelação..." sendo o Sol um aluno que anda junto às
>paredes simulando o movimento aparente e a Terra os restantes alunos
>num foco. De resto as constelações só começa a ter muito valor quando
>queremos ver cometa ou asteróide
>Não gostamos quando, a meio da noite, nos aparecem adultos a perguntar
>se pelo facto de Vénus estar em Virgem eles vão ter mais sorte no
>Totoloto.
>Daí as alergias com a astrologia.
>E este ano tive até um pai engenheiro a perguntar-me se achava que o
>professor X iria faltar muito...O home estava a confundir astrónomo
>com astrólogo. Aceitamos bem melhor sermos gastrólogos e gastrónomos.
>Mas a astrologia dá para muita brincadeira. É que este pessoal que nos
>chega à mão aos 11 anos já tem pelo menos 3 anos de laboratórios e
>trabalhos de campo. Vem com a mania de apanhar a alma na ponta de um
>bisturi. Vamos ver o que esta geração dará na universidade dentro de 4
>anos. Pois se trata da reforma do ensino em curso.
>Entretanto já lera o artigo do Galileu noutra lista, o CdA
>
>Abraço
>Maria Natália
>PS Pois como agora estou a dar aulas longe de casa o tempo vai
>falhando para acompanhar diariamente a lista...
>
>--- Em ciencialist@yahoogrupos.com.br, "Sergio M. M. Taborda"
><sergiotaborda@t...> escreveu
> > Maria Natália wrote:
> >
> > >
> > > Estebam:
> > > O zodíaco dos astrólogos é o de há 2000 anos e não tem em conta os
> > > movimentos do eixo da Terra. O zodíaco tem 13 constelações: a seguir
> > > ao capricónio tem o Ofiuco...São 13 os tais signos. E agora que me
> > > dizeis acerca do Ofiuco e que de acordo com o zodíaco actual é o meu
> > > signo: poderei amanhã comprar lotaria para me sair prémio?
> >
> > Eu ja respondi a essa pergunta. E a resposta é : Esse argumento faria
> > sentido SE a astrologia fosse baseada nas constelações.
> > Como não é , insistir nesse argumento é pura besteira (=coisas que os
> > bestas fazem)
> > Jà lhe expliquei pq so ha 12 signos, o que já foi aqui explicado por
> > outros. É uma razão matemática para que seja assim. É baseada no
> > conceito de Periodo Sinódico.
> > Ha uma logica , séria, matemática, fisica, _astronomica_ para o numero
> > 12. Só que aos astronomos - com problemas de falta de atenção -
> > inventaram esse argumento sobre Ofucio.
> > fazer o quê ? Mas se vc acredita nesses mentirosos, não é diferente de
> > quem acredita em astrologos fajutos.
> >
> > > Desculpe, por acaso ouvi dizer que os astrónomos e astrofísicos falam
> > > besteira? Ou entendi mal?
> >
> > Ouvio bem. Eles dizem mesmo. E ha muitas mais do que a de Ofucio.
> >
> >
> > Sérgio Taborda
> >
> >
> >
> > --
> > No virus found in this outgoing message.
> > Checked by AVG Anti-Virus.
> > Version: 7.0.300 / Virus Database: 265.8.5 - Release Date: 03-02-2005
>
>
>




##### ##### #####

Para saber mais visite
http://www.ciencialist.hpg.ig.com.br


##### ##### ##### #####


Yahoo! Grupos, um serviço oferecido por:
PUBLICIDADE




------------------------------------------------------------------------------
Links do Yahoo! Grupos

a.. Para visitar o site do seu grupo na web, acesse:
http://br.groups.yahoo.com/group/ciencialist/

b.. Para sair deste grupo, envie um e-mail para:
ciencialist-unsubscribe@yahoogrupos.com.br

c.. O uso que você faz do Yahoo! Grupos está sujeito aos Termos do Serviço do Yahoo!.



[As partes desta mensagem que não continham texto foram removidas]



SUBJECT: Re: [ciencialist] Re: Zodiaco
FROM: "Oraculo" <oraculo@atibaia.com.br>
TO: <ciencialist@yahoogrupos.com.br>
DATE: 05/02/2005 19:24

Olá Taborda

Sua resposta resvala para coisas do tipo "tudo é tudo, nada é nada, mas as vezes, nada é tudo e tudo é nada..." Bom para Paulo Coelho, mas irrelevante para a discussão ou eficácia da astrologia.

Calculos matemáticos complexos nada significam sozinhos. Calculadoras e planilhas fazem milhões e podem nada significar. Interpretar esses cálculos ou seus resultados, tentando encontrar correspondencia entre eles e o mundo real, entre o mapa astral e a vida de uma pessoa, sua personalidade, isso é que é a astrologia e isso é que não tem base nem evidencia a sustenta-la.

Se toda qualidade é ao mesmo tempo boa e má, se toda pessoa pode ser uma hora uma coisa e outra outra coisa, então, para que sereve a astrologia??? Se sua resposta fizesse sentido, um mapa astral feito para mim, serviria para qualquer pessoa. Todos teriamos todas as qualidades, de uma forma ou de outra: a qualidade ou seu oposto, uma hora seriamos assim, outra assado.

Desmontando os argumetnos do Takata? Nem de longe..:-)

Se o leonino é as vezes autoritario e as vezes não é, então todos somos leoninos, já que todos somos ou não somos autoritários..:-) Que escolha temos? Não se pode ser nem autoritario nem não autoritário, assim como não se pode estar grávida e "também" não estar gravida.

Pessoas fazem mapas astrais para saber o que as diferencia de outras pessoas, o que as torna únicas, baseadas na data e hora em que nasceram. Isso é tolice. Mas poderia ser tolice e ainda assim ser real, mas também não há evidencias disso. Mapas astrais de excelentes astrologos são conflitantes entre sí. Sim, são as variaveis que são muitas. Mas, qualquer coisa em que as variaveis sejam tantas que as respostas nunca coincidam, são inúteis por defrinição.

O cara demorou para decidir? Então, o mapa acertou, afirmou ser indeciso. Mas, se o mapa diz o contrário, que ele é ponderado, também acertou..:-) Hilário.

Esse o argumento bem defendido pelo Takata e que você contornou com jogos de palavras. Não importa em nada o que o mapa afirme, já que qualquer caracteristica descrita pode ser aplicada a qualquer comportamento ou justificar qualquer coincidencia com o paciente.

Marte ser vermelho torna as pessoas de um jeito e não de outro??? Se o ferro no planeta não existisse, regidos por Marte teriam outra personalidade?? Fala sério..:-)

Tem razão em uma coisa, as regras não foram criadas agora. Foram criadas a 3000 anos por pessoas que nada sabiam sobre fisica, leis da gravitação, planetas, e muitas outras coisas (inclusive, sobre psicologia e processos mentais neurologicos). São regras arbitrárias, destinadas a dar ordem a uma aparente desordem no mundo. Não derivam de conhecimento confiável, mas de lendas e mitos, aspectos da cultura onde surgiram. Apresentar essa longevidade como virtude é temerário. Nada indica que qualquer relação real entre as posições dos planetas (sejam eles o Sol a Lua e a Terra ou o conjunto dos demais), setores do zodiaco, ou conjunto de estrelas (que os criadores não sabiam o que eram) influencie a personalidade de qualquer pessoa.

Acreditar nisso é apenas questão de fé (e muita fé), não uma conclusão baseada em evidencias.

Você parece, as vezes, dizer que a astrologia é como uma forma de psicologia, para a auto-compreensão e que por isso independe de dados e ligações fisicas reais. Mas é só você, talvez o único astrologo do mundo, que diz algo assim. E mesmo como auto-analise, auto-descoberta, a astrologia tem pouco a apresentar como evidencia de eficácia.

Homero


----- Original Message -----
From: Sergio M. M. Taborda
To: ciencialist@yahoogrupos.com.br
Sent: Friday, February 04, 2005 10:41 PM
Subject: Re: [ciencialist] Re: Zodiaco


rmtakata wrote:

>
> > Vc acha que a ligação é meramente arbitrária, mas eu podria
> > descorrer aqui, por não o é.
>
> Nao duvido nem um pouco disso. Os astromantes no minimo falariam de
> sincronicidade.
>
N\ao me faça rir com esses tipo de argumentos. Sejamos serios.

> > Não. Leão simboliza a coragem (coração+agem=acção do coração), mas
> > não a ferocidade.
>
> Ferocidade nao quer dizer crueldade. Um dos manuais astromantes diz: o
> leonino eh corajoso, justo e digno. Mas claro q. apresenta tb aspectos
> negativos - por curiosidade sao aproximadamente antagonicos aos
> aspectos positivos, de modo q. praticamente qq pessoa possa se
> encaixar na descricao generica: orgulhoso, autoritario, megalomano.


Vc não disse ferocidade, pq ? PQ o manual não diz lá ferocidade.
E ninguem identificou ferocidade com crueldade, isso são ideias suas.
Os simbolos são em muito boa parte adjectivos e é preciso ter cuidado
para não usar qq um.
Orgulho, autoritarismo e megalomania nada têm a ver com ferocidade. São
tudo coisas do coração.
O Leão sente mais do que parece, mas ao contrario da cancer, ele esconde
isso. Tal como um pai,
que tem que ser autoritário para manter o respeito mas que ama os seus
filhos.

Quanto a bons e maus atributos, a primeira coisa que vc aprende com a
astrologia é que nenhum atributo é apenas bom ou mau.
Não é curiosidade, não é coincidencia, é a prorpia essencia da astrologia.
Orgulho , autoridade e magalomania podem ser coisas boas.

(A palavra Aspecto tem um significado tecnico em astrologia, por isso
que mudei para atributo)

> > Ora, mas é a ideia do 5º signo que tem como menomonica o leão, ou o
> > leão que empretou as suas caracteritsticas ao signo ?
> > É a primeira e não a segundo, como vc supoe.
> > Qualquer coisa poderia ser atruida à constelação de leão ,pq um
> > leão?
> > Ora, afinal aquilo é so um triagulo com uma ponta
>
> Eles enxergaram um leao. Sim, poderiam ter enxergado outra coisa, mas
> se assim o fizesse, teriam atribuido ao setor outra coisa.

Isso é o que vc diz. Em que se baseia para afirmar isso. Qual é o dado
cientifico, historico, sei lá - se
é que ha algum dado - que aponte na direcção que diz ?
O efeito é ao contrario, eles teriam escolhido outro simbolo se o
significado fosse diferente. E um significado por ter muitos simbolos.
Vc não escolhe o verde para avançar e o vermelho para parar pq está
relacionando as corres com acções.
Vc relacion as cores com o significado depois dele ser atribuido. E vc
sabe muito bem pq o vermelho é para parar.
Foi escolhido essa cor para esse significado por motivos logicos: os
nossos olhos respondem melhor ao vermelho que ao verde.

Analizando de fora, a correlação entre aparecer vermelho e vc parar é
muito elevada. Matemáticamente falando existe uma relação, um corelação.
O que significa que ha uma relação causa-efeito. Mas todos sabemos que
essa correlação causa efeito não é vinculada, que não é uma lei da
natureza.
A astrologia baseia-se na mesma coisa. Correlações de causa-efeito, sem
dizer nada sobre os vinculos fisicos. Qq coisa que vc ler em contrario
disto é
pura 'astrologia' de 4º classe.

>
>
> > > ter um aspecto de libra significa ser equilibrado,
> >
> > Não. Significa indiciso. Que não sabe tomar parte, que não sabe
> > decidir entre as opções.
>
> Isso significa o aspecto negativo. No aspecto positivo significa
> ponderação. Contraditorio? Sem duvida.

Como vc ainda nao entendeu que uma so coisa pode ser boa e má, vc acha
contradittorio.
Como vc não sabe que essa é a regra astrologia e não mera coincidencia,
vc acha que é oportuno a todos os "astromantes" (alias , esse termo é
bem ofensivo)
que assim seja. É tão oportuno como para a todos os fisicos, que a
gravidade seja atractiva. (pense nisso, ela é atractiva por convensão)


> Isso eh muito bom para a
> astromancia, jah q. pode encaixar tto os librianos equilibrados, qto
> os indecisos.

O facto é que eles são as duas coisas. Em algumas areas serão
equilibrados e outras indecisos.
Uma so pessoa será as duas coisas. Ao contrario do que vc afirma que uma
pessoas será uma coisa e outra , será outra.
Não, cada libraniano será as duas coisas. O mapa, revelará em quais
areas é equilibrado e em quais não é.
E não estamos falando de equilibrio mental.

> > Compare com a astrologia Indu (a atrologia original onde se baseia a
> > ocidental actual). Nele os mesmos signos, com as mesms
> > exactas posições não têm nada a ver com as estrelas no sentido de
> > que não ha correspondencia entre os nomes das constrelçaões
>
> Lamento, mas a astromancia hindu nao eh uma astromancia independente
> da astromancia ocidental.

That's the point! O ponto é esse mesmo. Elas não o são, mas assim mesmo
a indu atriui simbolos
diferentes aos mesmos sigificados. O significa que o significando vem
antes, e o simbolo é apenas uma menmonica.

> Bebem de uma fonte comum e houve intensa
> comunicacao. Tto eh q. dados astronomicos hindus - junto com a
> matematica - vieram para o ocidente e vice-versa


Obrigado. É exactamente isso. Então como vc esplica que os simbolos
sejam outros ?
Se a sua teoria fosse verdaeira , os simbolso seriam os mesmos pois as
estrelas são as mesmas.
Pense nisso, tlv vc entenda que a astrologia nada tem a ver com as
estrelas.

> > > Temos um caso aqui de uma construcao
> > > argumentativa a posteriori.
> >
> > Não, não temos. Vc acha que sim pq não conhece um pouco da historia
> > da astrologia.
>
> Sua exposicao de defesa nao eh convincente.

Como se a sua fosse ... lol.. estou so demontanto seus argumentos (hei,
estou dizendo q vc tem argumentos!...).

> A mesma coisa acontece com
> os planetas. O planeta vermelho acabou sendo ligado 'a guerra e essas
> coisas - assim Marte - e antes, seus equivalentes babilonios, egipcios
> e demais - acabou por reger as relacoes masculinas, de forca e coisas
> assim.

Tudo bem. Então pq os outros planetas não são referidos como o planeta
<cor>?

Pq Jupiter não é o planeta laranja, ou Venus o planeta Roxo ?

Pela mesma razão que apontei antes. Dado que o significado é ligado ao
sange, e marte aparece aos nossos olhos,
vermelho, é natural que a cor vire uma menmnonica. Se fosse a cor a
gerar o significado, o que dizer dos outros planetas ?

> Como Venus - ou seus equivalentes babilonicos - foi
> identificada com uma divindade feminina, ganhou suposta regencia sobre
> atributos femininos. E assim por diante.

Depende. Outro nome para Venus é Luficer. Que todos sabem com o que se
relaciona.
(Sabemos ? Quando leu lucifer ter ter pensando no demonio , que é
simbolizado pelo vermelho.
Mas Lucifer não é nada disso. É a Estrela da Manhã e da Tarde. É dai que
vem a ligação com a beleza.
E por isso o mito de Venus-Afrodite é ligado com a beleza. Vc já viu o
fenomeno ? Etão sabe como é bonito e confortante.

> Trata-se de uma construcao
> argumentativa a posteriori dizer q. o q. importa sao os setores e os
> nomes dos setores sao apenas um detalhe historico (ou sao efeitos e
> nao causas das qualidades atribuidas a cada setor e outros elementos
> de configuracao na carta natal).

Você parece estar muito convencido disso, ignorando toda amatemática
envolvida na astrologia, a
forma como se constroi o circulo zodiacal, as regras que estão sobre ele.
Tudo o que não tem nada a ver com os nomes, e que vc não vai ler nos
horosocopos do jornal.
Vc prefere fixar-se nos nomes do que pensar um pouco como cientista e
humano e pensar
nas regras. Esse é um argumento ... parco - para dizer o menos.
É como vc negar a mecanica quantica pq não gosta do nome quantum.
As regras não foram criadas agora , por isso o argumento do argumento à
posteriori não vale. O sinodico da lua cai 12 vezes no periodo do sol
- faça as contas - , várias astrologias diferentes (como a ocidental e a
indu) tem os mesmos significados mas com simbolos diferentes. Os quais
não dependem de nenhumas estrelas.
A astrologia é baseada em 3 astros, o sol, a lua e a terra. (As eras
astrologicas não são nada mais que a precessão da orbita da terra em
torno do sol. ) e não nas estrelas.
Escrito nas estrelas é apenas uma expressão poetica que não corresponde
com a realidade da teoria. Exactamente como a frase de Eisntein "Deus
não joga aos dados" não corresponde com o que ele disse, mas toda a
gente acha que sim.
Pessoas desinformadas aceitam o mito, aceita a frase de Eisntein tal
como o seu argumento , ou os da Natália, pq não se dão ao trabalho de
procurar as fontes fidedignas.
E que eu acho mais curioso é a critica à astrologia ser baseada nos
mitos levantados em cima dela , em vez de nela mesma. Seria como
criticar Eisntein por ter dito aquela frase, que ele nunca disse. Ele
nunca usou a palavra "deus" - mas vc so entenderá pq se souber sobre a
vida de Eisntein e não do mito da vida dele.
A discussão sobre astrologia pelos seus criticos comete os mesmos erros.
E vc dá ouvidos a quem critica Eisntein por ter dito "Deus não joga aos
Dados" ? Então pq vc dá ouvidos a quem critica o mito e não a historia
real?

O pior são os que se dizem ceticos que cometem estes erros
atropeladamente todos os dias em relação a tudo o que eles não conhecem.
Eles seguem o mito, e não o separam da coisa que é mesmo.
Que tipo de cepticos são que acreditam que o mito exprime a realidade ?
Vc está entrando no mesmo jogo.



--
No virus found in this outgoing message.
Checked by AVG Anti-Virus.
Version: 7.0.300 / Virus Database: 265.8.5 - Release Date: 03-02-2005



##### ##### #####

Para saber mais visite
http://www.ciencialist.hpg.ig.com.br


##### ##### ##### #####


Yahoo! Grupos, um serviço oferecido por:







------------------------------------------------------------------------------
Links do Yahoo! Grupos

a.. Para visitar o site do seu grupo na web, acesse:
http://br.groups.yahoo.com/group/ciencialist/

b.. Para sair deste grupo, envie um e-mail para:
ciencialist-unsubscribe@yahoogrupos.com.br

c.. O uso que você faz do Yahoo! Grupos está sujeito aos Termos do Serviço do Yahoo!.



[As partes desta mensagem que não continham texto foram removidas]



SUBJECT: Zodiaco e Acupuntura e Homeopatia..:-)
FROM: "Oraculo" <oraculo@atibaia.com.br>
TO: <ciencialist@yahoogrupos.com.br>
DATE: 05/02/2005 19:39

Olá Murilo

risos..:-) Bem, mesmo me chamando de padre Quevedo (o que me deixou assustado..:-), não refutou meus argumentos sobre a acupuntura..:-) Considero isso uma boa coisa, já que estes parecem ser eficazes em convence-lo (ou pelo menos a fazer você pensar sobre eles..:-)

Mas a homeopatia segue o mesmo raciocinio. Ela tem definição precisa, ou deixa de ser a homeopatia. Veja, seu criador, Hahnemann, nada sabia sobre micro-organimos, infecção, sistema imunologico, etc. Sua criação curava doenças a partir de determinados principios. Ou se aceita os principios, ou se recusa o termo homeopatia. Eu acho que foi um avanço na época, já que a medicina usava sanguessugas, purgantes e coisas assim para tratar. Parar com esses "tratamentos" e esperar a doença se curar em geral era mais benéfico ao paciente..:-)

Ele dizia, por exemplo, que doenças cronicas eram causadas por um tipo de espírito maligno, ou misma, a psora (como em psoriase). Devo levar essa afirmação também, junto ao resto das alegações da homeopatia? Ou posso filtrar, com alguma ferramenta racional, o que é correto de que é incorreto? E, se posso fazer isso, que ferramenta deve usar? E o que fazer se a ferramenta derrubar todas as afirmações, e não apenas as mais estranhas?

Influenciar animais é possivel. Na verdade, para a maior parte dos mamiferos (principalmente os animais gregarios) a atençao e o cuidado pessoal traz benficios no bem estar. São animais, mas não são estupidos ou desprovidos de mente ou de processos mentais (filhotes de macacos abandonados pelas mães, preferem mães substitutas feitas de pelo e algodão, mas sem alimentos, do que mães substitutas feitas de arame, mas que poderiam alimenta-los). Tenho um amigo que, depois de tentar bastante o tratamento de seus cavalos e gado com homeopatia (principalmente para a perigosa constipação) desistiu por absoluta falta de efeitos. Esse relato, unico e anedotico, valida a tese que a homeopatia não funciona? Claro que não..:-) Tanto quanto um relato contrário, de efeito positivo, não valida a afirmação de que a homeopatia é real.

Apenas façam com que a homeopatia passe em estudos controlados, e minha conclusão sobre sua eficácia muda..:-)

Mas, enquanto ela falhar, e enquanto seu mecanismo de ação for inexistente (ou francamente incorreto), fico com a conclusão cientifica de que é um placebo, na melhor das hipoteses..:-)

Um abraço

Homero
----- Original Message -----
From: murilo filo
To: ciencialist@yahoogrupos.com.br
Sent: Friday, February 04, 2005 11:06 PM
Subject: RE: [ciencialist] Zodiaco e Acupuntura..:-)


Oraculo, oi.
Vc está parecendo o Padre Quevedo fazendo ginástica anti-espírito!
Conseguí achar um negócio interessante sobre a homeopatia, que também foi
''atacada'' nesta discussão!
FAVOR irem a www.arenales.com.br e lá encontrarão muita informação,
onde especialmente destaco muita ciência e homeopatia para ANIMAIS. Um lance
psicossomático, certamente.
A arte de ''sugestionar'' animais é maravilhosa, não? :]]] abr/M.

>From: "Oraculo" <oraculo@atibaia.com.br>
>Reply-To: ciencialist@yahoogrupos.com.br
>To: <ciencialist@yahoogrupos.com.br>
>Subject: [ciencialist] Zodiaco e Acupuntura..:-)
>Date: Fri, 4 Feb 2005 16:49:32 -0200
>
>Olá Esteban
>
>Há uma sequencia de enganos em suas colocações, talvez derivada de um
>compromentimento com a crença inicial e uma certa aversão a ciência..:-).
>Métodos usados na pesquisa cientifica não servem apenas para impedir que
>alegações reais, mas incomodas, sejam demonstradas. Servem para filtar
>alegações sem base na realidade.
>
>Assim, acusar os que pesqusiasm com ironias do tipo "santos céticos
>imparcialistas" é bobagem e uma falacia "ad hominem". A não ser que credite
>tudo a Grande Conspiração Mundial Da Ciência Para Esconder A Verdade. E
>nesse caso, nada do que eu, ou qualquer outro diga, fará a menor diferença,
>certo?..:-)
>
>Este trecho é um problema:
>
>Talvez devesse dar algum crédito mesmo. Já estão acreditando em
>acupuntura... Mas foi testada, nê, botaram uns eletrodozinhos e viram que
>tinham razão aqueles outros velhinhos há mais de 3000 anos atrás...
>Acordaaaaa!!!!
>
>Isso já foi explicado diversas vezes, mas vai de novo..:-) E, não precisa
>acreditar em mim, basta apenas pensar sobre o que argumento e ver se é
>possível refutar, ok?
>
>Acupuntura é a cura de todas as doenças pela manipulação de meridianos de
>energia não detectável e mística através da implantação de agulhas em
>pontos determinados desses meridianos. Isso é acupuntura e a base dessa
>afirmação é que a causa de todas as doenças é o desequilibrio de energias
>não detectáveis.
>
>Recentemente, detectou-se algum efeito na sintese de prostaglandinas ao se
>aplicar agulhas na pele de pacientes. Como as prostaglandinas estão
>envolvidas nas sensações dolorosas, isso pode causar algum efeito do tipo
>anestésico leve pelas agulhas.
>
>Mas, isso NÃO É MAIS ACUPUNTURA. Isso é outra coisa, uma nova descoberta
>sobre efeitos de agulhas em sintese de prostaglandinas. Nào exige que os
>meridianos sejam identificados, nào exige que pontos especificos sejam
>encontrados, e pode ser substituida por beliscões, em seu efeito nas
>prostaglandinas.
>
>Distorcer um conceito, como a definição de acupuntura, para caber em
>descobertas recentes e cientificas, é um procedimento padrão em
>pseudociencias, também usado pela astrologia. Mas é evidente que, se não se
>baseia na causa energética das doenças, se não exige o reequilíbrio dessas
>energias, se independe de meridianos por onde essa enegia corre, não é,
>claro, acupuntura.
>
>O que a acupuntura alega, e que nào é reconhecido pela ciência por não ter
>se mostrado eficaz e nào ter mecanismo de ação reconhecivel, é que doenças
>são causadas por desequilibrios de energia e que podem ser curadas
>reequilibrando essas energias com agulhas em pontos específicos e bem
>determinados da pele. Ela não alega que espetar agulhas (ou causar
>sensações dolorosas de algum tipo) na pele das pessoas, interfere na
>sintese de prostaglandinas, podendo servir como anestésico leve.
>
>Veja, nào deve acreditar em mim, deve apenas analisar o argumento. Se puder
>demonstrar que está incorreto, ótimo. Se não puder, então ele é valido e
>deve se manter até que algo mude (por exemplo, descobrir que energias nào
>detectáveis causam tuberculose).
>
>Aqueles velhinhos de 3000 anos continuam enganados quanto a suas alegações.
>Continuam enganados ao afirmarem que doenças são causadas por
>desequilibrios energéticos. Continuam enganados ao afirmar que a energia
>nào detectável corre por linhas determinadas chamadas meridianos. Continuam
>enganados ao afirmar que podem curar doenças reequilibrando essas energias.
>Continuam enganados ao afirmar que o reequilibrio se dá com agulhas em
>pontos determinados da pele. E nào estão enganados sobre a sintese de
>prostaglandinas, simplesmente porque nada sabiam sobre sintese de
>prostaglandinas ou sobre qualquer outro sistema metabolico do corpo humano.
>
>Homero
>
>
>
> ----- Original Message -----
> From: Esteban Moreno
> To: ciencialist@yahoogrupos.com.br
> Sent: Friday, February 04, 2005 4:24 PM
> Subject: Re: [ciencialist] Re: Zodiaco
>
>
>
> Mr Takata wrote:
> > descrições astrológicas?
> Entao, em principio pareceria 'suspeito'. Mas como dito, qdo
> devidamente testada a astromancia nao parece ter um bom poder de
>predicao.
>
> E:
> Devidamente testada por quem? Pelos santos céticos imparcialistas da
> ciência?
> Depende de quem ou como se pesquisa. Há uma série de artefatos e
>falácias do
> que julgam como pesquisas comprobatórias. Não vou enumera-las, mas
>talvez
> ajude a lembrar que a pesquisa da universidade de brasília resultou em
>dados
> bastante promissores. Que péssimo! Uma das justificativas dos céticos
>acerca
> do porquê os pacientes sentem-se bem descritos pelo astrólogo é a
>chamada
> confluência (auto-atribuição é um nome pobre para isso). Abaixo segue um
> artigo científico que "desmistifica" (perdoe-me..) este recurso:
>
> Self-attribution, sun-sign traits, and the alleged role of
>favourableness as
> a moderator variable:
> long-term e.ect or artefact? Edgar Wunder* Gesellschaft fu¨r
>Anomalistik,
> Postfach 1202, 69200 Sandhausen, Germany Personality and Individual
> Differences 35 (2003) 1783-1789 www.elsevier.com/locate/paid
>
> Posso envia-lo em anexo.
>
> Takata:
> > Um bom parâmetro é a quantidade do mercado que vem se
> > ampliando extensivamente, incluindo grandes empresas e corporações.
> Esse na verdade eh um pessimo parametro. Se for por isso deveremos
> creditar confianca na homeopatia, passes espirituais (sem falar na
> cirurgia mediunica), toda sorte de mandingas...
>
> E:
> Talvez devesse dar algum crédito mesmo. Já estão acreditando em
> acupuntura... Mas foi testada, nê, botaram uns eletrodozinhos e viram
>que
> tinham razão aqueles outros velhinhos há mais de 3000 anos atrás...
> Acordaaaaa!!!!
>
>
> Infelizmente só retorno daqui a 10 dias.
> Um carnaval cheio de boas mandingas para vocês.
> Esteban.
>
>
>
> ##### ##### #####
>
> Para saber mais visite
> http://www.ciencialist.hpg.ig.com.br
>
>
> ##### ##### ##### #####
>
>
> Yahoo! Grupos, um serviço oferecido por:
>
>
>
>
>
>
>
>------------------------------------------------------------------------------
> Links do Yahoo! Grupos
>
> a.. Para visitar o site do seu grupo na web, acesse:
> http://br.groups.yahoo.com/group/ciencialist/
>
> b.. Para sair deste grupo, envie um e-mail para:
> ciencialist-unsubscribe@yahoogrupos.com.br
>
> c.. O uso que você faz do Yahoo! Grupos está sujeito aos Termos do
>Serviço do Yahoo!.
>
>
>
>[As partes desta mensagem que não continham texto foram removidas]
>




##### ##### #####

Para saber mais visite
http://www.ciencialist.hpg.ig.com.br


##### ##### ##### #####


Yahoo! Grupos, um serviço oferecido por:







------------------------------------------------------------------------------
Links do Yahoo! Grupos

a.. Para visitar o site do seu grupo na web, acesse:
http://br.groups.yahoo.com/group/ciencialist/

b.. Para sair deste grupo, envie um e-mail para:
ciencialist-unsubscribe@yahoogrupos.com.br

c.. O uso que você faz do Yahoo! Grupos está sujeito aos Termos do Serviço do Yahoo!.



[As partes desta mensagem que não continham texto foram removidas]



SUBJECT: Energia "Verde" Já
FROM: "Tipoalgo" <tipoalgo@bol.com.br>
TO: ciencialist@yahoogrupos.com.br
DATE: 05/02/2005 19:40


O Sr. Fendel está correto?

Quais as ressalvas?

Alguém gostaria de comentar?

Agradecimentos antecipados!

Tipoalgo

---Extraído da lista Bioenergia-l, (http://jatoba.esalq.usp.br/cgi-
bin/mailman/listinfo/bioenergia-l)


-----Mensagem original-----
De: bioenergia-l-bounces em jatoba.esalq.usp.br
[mailto:bioenergia-l-bounces em jatoba.esalq.usp.br] Em nome de Fendel
Enviada em: terça-feira, 21 de dezembro de 2004 11:46
Para: ...
Assunto: [Bioenergia-l] RES: Personagem principal


Meu caro UVX

Na extraordinária notícia abaixo, sobre o imenso potencial alcoólico
mundial, tenho 2 comentários a fazer:
1- O deputado Gabeira e seus pares deveriam ser informados sobre a
besteira que é o hidrogênio, pois sua produção e armazenamento requer
4 vezes mais energia do que a então obtida. Nunca produzir hidrogênio
vai ser barato, limpo e eficiente. NUNCA o homem vai conseguir imitar
a fantástica natureza com sua espetacular, graciosa e ignorada
fotossíntese.
2- O mercado de carbono atrapalha a re-evolução das milenares e
desprezadas bioenergias, claro, pois ao invés de investirem na
produção ou compra de álcool, óleos vegetais e demais biomassas, esta
besta esmola aumenta a sobrevida dos porcos combustíveis fósseis, com
o conseqüente aumento de carbono no ar, limitando o uso das
bioenergias atmosfera limpantes. Se o consenso reza que temos que
diminuir o carbono atmosférico, então é uma hipocrisia aumentar ainda
mais a emissão de carbono fóssil, laureado por miseráveis títulos de
seqüestro de carbono. Para o balanço negativo de carbono, é melhor
não deixar os vegetais apodrecerem no campo ou na mata.
Para a efetiva limpeza do ar, a biomassa crescida tem que ser
utilizada para poder dar lugar a sucessivos novos vegetais, sendo seu
carbono fixado no solo e em objetos como: camas de mogno, calcinhas
de algodão, lençóis de linho, livros de papel, estruturas de
eucalipto, produtos bioplásticos, etc;
conforme explicado a seguir:

Coloque um vaso com terra sobre uma balança ensolarada.
Insira uma semente de girassol nesta terra.
Regue regularmente e observe.
Note que a cada dia o peso e a massa vão aumentando exponencialmente
conforme aumenta a matéria verde.
Conclusão:
Parte da água evapora, e outra parte se dissocia em hidrogênio e
oxigênio através da inimitável fotossíntese.
Acontece que os vegetais não se constituem apenas de H e de O.
Quase a metade de sua massa é carbono.
E de onde vem este carbono?
Este carbono vem do gás carbônico dissolvido no ar e assimilado pela
mágica e maravilhosa citada fotossíntese.
Assim, a comida dos vegetais é basicamente o CO2 atmosférico e a
bebida é o H2O pluviométrico, resultando que qualquer planta é
constituída principalmente de carbono, oxigênio e hidrogênio.
E como se não bastasse, ainda exalam oxigênio e perfumados óleos
essenciais.
Continuando a pesquisa, note que a assimilação de CO2 passa por um
máximo e então se reduz, até parar na fase adulta.
Após a fase madura, começa automaticamente o apodrecimento, se as
sementes, folhas e caules não forem colhidos e utilizados.
E nesta fase de apodrecimento, diminue a massa do conjunto, pois
ocorre o lançamento principalmente de CH4 (metano) e CO2 à atmosfera
através da sua digestão aeróbica e anaeróbica pelos microorganismos.
Portanto, nada melhor do que plantar e utilizar intensamente qualquer
vegetal, em forma de óleos, álcool, biogases, carvão vegetal,
madeira, papel, tecido, resíduos, bioquímica, biofertilizantes, etc.

Gratos Bioabraços pelo incentivo.
Eng. Thomas Renatus Fendel
www.fendel.com.br

"A queima das bioenergias devolve menos carbono ao ar do que o
absorvido
pelas plantas, resultando no "efeito geladeira", oposto do "efeito
estufa"
originado pelo uso dos porcos e agonizantes combustíveis fósseis". -
Fendel



-----Mensagem original-----
De: UVX
Enviada em: segunda-feira, 13 de dezembro de 2004 17:33
Para: Fendel
Assunto: Personagem principal


Meu caro

Logo, logo poderems ver estampado em jornais,
revistas, etc...
uma matéria parecida com esta, mas sendo o óleo
vegetal o personagem principal.
Abraços
UVX


ENERGIA: ÁLCOOL BRASILEIRO PODE GANHAR O MUNDO


As perspectivas são inebriantes para o álcool
brasileiro usado como combustível, pela provável
abertura de gigantescos mercados na China e nos países
mais industrializados e ricos da Ásia, Europa e
América do Norte. O Brasil é o maior produtor de
etanol, com 14,750 bilhões de litros no ano passado,
38% do total mundial. Este ano deve exportar 2,2
bilhões de litros, o triplo do que foi exportado em
2003. Usando a cana-de-açúcar como matéria-prima, tem
o mais baixo custo de produção. A demanda mundial por
etanol equivaleria a 7,5 vezes a produção brasileira
se todo o mundo adicionasse 10% desse carburante à
gasolina, estimou Plínio Nastari, diretor da Datagro,
empresa especializada em informações sobre o setor de
cana-de-açúcar, em um seminário realizado há poucos
dias em Brasília.

A mistura é uma tendência geral, impulsionada pelo
combate à poluição urbana, à alta do preço do petróleo
e a entrada em vigor, em fevereiro, do Protocolo de
Kyoto sobre mudança climática. A combustão com etanol
reduz os gases que aquecem a atmosfera, como o dióxido
de carbono, liberados sobretudo pela queima de
derivados de petróleo, gás e carvão. O Brasil é
considerado o único fornecedor capaz de atender no
curto prazo um grande aumento da demanda. Mas, a
euforia nos investimentos na agroindústria da
cana-de-açúcar, que já atrai muitos capitais
estrangeiros, tem como contrapartida os temores
ambientalistas de que o país se transforme em um
imenso canavial e, também, dos importadores, que não
querem ficar reféns do fornecimento brasileiro.

Segundo previsões, para sustentar o aumento interno e
externo de demanda, a área cultivada de cana-de-açúcar
deveria aumentar 75% em 10 anos, invadindo o cerrado,
a savana de florestas ralas que ocupa uma vasta zona
central do país. Trata-se de um ecossistema pouco
protegido e atropelado pelo avanço da agricultura nas
últimas três décadas, especialmente pela soja. O café,
o algodão e, recentemente, a cana-de-açúcar ocupam boa
parte de suas terras. Não importa o tipo de cultivo, o
problema é a "ocupação desordenada" das monoculturas,
esse processo de "terra arrasada", disse à IPS Mario
Barroso, gerente do não-governamental Programa do
Cerrado de Conservação Internacional. Existem
instrumentos legais para uma ocupação sustentável do
solo, como o Código Florestal, que exige a preservação
de 20% da mata nas propriedades.

Se forem cumpridas suas regras e respeitadas as áreas
de conservação, a mata nas margens dos rios e em
declives, haveria uma proteção razoável, avaliou o
ambientalista. Não existe esse risco com a
cana-de-açúcar, garantiu Antonio de Pádua Rodrigues,
diretor da União da Agroindústria Canavieira de São
Paulo (Única), que reúne as maiores empresas do setor.
A entidade estima que a produção nacional de álcool,
que em 2003 atingiu 14,750 bilhões de litros, deverá
crescer outros 10 bilhões de litros, dos quais apenas
30% seriam destinados à exportação. Isso exigirá
ampliar em 2,5 milhões de hectares a área cultivada,
estimou Pádua. Mas, não será necessário avançar muito
no cerrado, pois 60% dessa expansão se dará no Estado
de São Paulo, explicou. Além disso, as terras a serem
cultivadas já estão desmatadas e foram abandonadas
pela pecuária que, ao adotar melhores técnicas de
produção, reduziu drasticamente sua área de pastagem
ressaltou.

A produtividade canavieira também aumentou muito, o
que exige menos terras, especialmente em São Paulo,
que concentra 60% da produção nacional de açúcar e
álcool. Em 1980, eram obtidos 3.500 mil litros de
álcool por hectare, hoje se produz o dobro, disse
Pádua à IPS. O deputado Fernando Gabeira,
ambientalista, admitiu que os grandes produtores (pelo
menos "em fazendas que visitei em São Paulo")
incorporaram os cuidados ambientais por exigências
legais, de responsabilidade social e, inclusive, por
vantagens produtivas. Nos canaviais são intercaladas
áreas plantadas com outros alimentos, explicou.
Algumas culturas ajudam a fertilizar a terra, ao fixar
o nitrogênio. Seria contraditório ampliar a produção
de cana-de-açúcar para contribuir para contribuir para
a solução de um problema global, como a mudança
climática, prejudicando o meio ambiente local, afirmou
Gabeira, defendendo, entretanto, a prática de estudos
sérios de impacto ambiental e o forte controle para
evitar destruição da biodiversidade. Sua esperança é
que o processo seja transitório, enquanto se tornam
viáveis outras alternativas energéticas, como o
hidrogênio.

O mercado internacional do álcool combustível não terá
uma expansão explosiva, "está em construção e esse é
um processo lento", disse Pádua. Do que o Brasil
exporta, apenas 40% se destina para fins combustíveis,
a maior parte ainda é utilizada para uso industrial,
explicou. Os possíveis grandes importadores querem
garantias de abastecimento, hesitam em assinar
contratos para o fornecimento brasileiro por longos
períodos enquanto não aparecem outros fornecedores, um
mercado livre e outros mecanismos de segurança e
estabilização de preços, ressaltou Pádua. E os
investimentos acompanham a expansão do mercado
consumidor. Para construir esse mercado mundial que
interessa ao Brasil será necessário que outros países
produtores de açúcar, como Austrália, Colômbia,
Guatemala, Índia, México e Tailândia, ampliem e
diversifiquem sua oferta, concluiu.

Fonte: Envolverde





SUBJECT: RE: [ciencialist] O cético que encontrou um bom astrologo..:-) (era Zodiaco)
FROM: "murilo filo" <avalanchedrive@hotmail.com>
TO: ciencialist@yahoogrupos.com.br
DATE: 05/02/2005 19:44

Oi, Oraculo.
Estava me referindo à recepção e medição de resultados, e sempre usando êste
estado mental comum que nos suporta por êste mundão velho sem porteira.
A inteligência normalmente treinada e uma estrutura psicológica estável
conseguem razoável cognição entre o que é furado e o tal ''epa, aquí tem
alguma coisa...''
A astrologia é um outro mundo onde se encontram todas aquelas mumunhas
normais e comuns aos outros ramos, inclusive os sacanas e aquêles que
prometem o impossível.
Aprecio muito a intuição.
Previsões? Acho meio difícil, mas já ví. Encaro esta coisa mais como uma
tábua das marés, sempre conforme a pessoa e um cacetão de variáveis,
inclusive as mais prosáicas. Tratam-se de tendências, informações de prá
onde e como correrá, ou corria, a maré. Uma ferramenta a mais.
Quando a indicação de uma 'maré' é muito forte, a coisa é quase inescapável.
Não pense que espero receber de qualquer outra coisa externa um refresco
p/meus pepinos! Estamos todos aquí só para aprender, e o êrro ensina mais do
que o acêrto.
Tudo que ainda não aconteceu pode ser mudado e qualquer tendência poderá ser
contrariada pela vontade, pela ação e até pela ignorância humanas. abr.
M. sp 05/fev


>From: "Oraculo" <oraculo@atibaia.com.br>
>Reply-To: ciencialist@yahoogrupos.com.br
>To: <ciencialist@yahoogrupos.com.br>
>Subject: [ciencialist] O cético que encontrou um bom astrologo..:-) (era
>Zodiaco)
>Date: Sat, 5 Feb 2005 19:01:03 -0200
>
>Olá Murilo
>
>Infelizmente, essa previsão vai falhar.:-)
>
>E o motivo é simples, mesmo que um cético ou cientista se encaixe no caso
>relatado por você, encontrar um astrologo que faça seu mapa astral com
>enorme relevância de dados, ainda assim ele saberá que é apenas um caso
>anedotico (não estatistico) e que isso não representa evidencia confiável.
>
>Veja, não é porque conheço o efeito da percepção seletiva, da pareidolia,
>do famoso "conte os acertos e ignore os erros" que estou imune a eles..:-)
>Sei perfeitamente que posso ser enganado por minha mente ou por um bom
>astrologo (ou qualquer outra forma de alegação sem evidencias) tanto quanto
>sou enganado por mágicos de palco (que repetem exatamente os mesmos
>fenomenos sobrenaturais)..
>
>Assim, mesmo que eu encontre um astrologo que faça meu mapa astral, de
>confiança, competente, e que eu me veja refletido no mapa, ainda assim vou
>manter o ceticismo natural e pedir uma avaliação de outra pessoa, outra
>analise, uma significancai estatistca no acerto do astrolog, etc, etc, etc.
>Eu posso me enganar (e minha mente tanto quanto eu..:-) como qualquer
>pessoa.
>
>E tem de perceber que a classificação "competente", "consciente", "sério",
>etc, é um julgamento de valor. Como escolher o astrologo que se encaixe?
>Como decidir se é competente, ou honesto? Como, engim, filtrar os dados e
>analisar essa questão? Bem, se posso sugerir algo, devemos, de novo, usar o
>sistema rigoroso da ciência apra faze-lo..:-) Ou teremos, de novo, de
>acreditar na palavra de quem nos recomenda o astrologo. Se você decidir
>analisar o trabalho de um astrologo, apra decidir se ele se encaixa nos
>termos que propos, o que usará como ferramenta de análise?
>
>É isso que parece dificil de explicar e de entender: não é por falta de
>experiencia pessoal que cientistas e céticos não acreditam, é apenas por
>saber que é necessário mais que isso para uma conclusão confiável.
>
>Se eu tomar uma erva, receitada por um curandeiro, e minha doença se curar,
>ainda assim eu vou manter o ceticismo a respeito do poder real de cura da
>mesma. E estarei apenas sendo cuidadoso com a conclusão, já que para
>determinar a eficácia de uma droga ou medicamente são necessários testes
>rigorosos, duplo cego, estudos longos e muita reprodutibilidade e análise.
>E mesmo você deve concordar com isso, e se recursar a tomar drogas ou ervas
>(que podem matar) sem essas precauções, não importa o testemunho de quem
>quer que seja que tenha se curado ao toma-la, não?
>
>Parece que quem crê em algo pensa que céticos tem falta de experiencias
>pessoais..:-) Que, como você previu, se encontrarem um "bom astrologo"
>mudarão de opinião e passarão a acreditar. Não é assim que funciona.:-)
>Ciência é mais que relatos anedoticos, mesmo os pessoais, mesmo as
>experiencias que eu experimento. É um conhecimento que só se tornou
>confiável quando passou a ser produzido dentro do rigor necessário.
>
>Um abraço.
>
>Homero
>
> ----- Original Message -----
> From: murilo filo
> To: ciencialist@yahoogrupos.com.br
> Sent: Friday, February 04, 2005 7:30 PM
> Subject: RE: [ciencialist] Re: Zodiaco
>
>
> Pois eu tenho uma previsão para alguns de vcs.
> ( e nada tenho a ganhar com isto! )
> Alguns felizardos, e privilegiados, terão, algum dia, a oportunidade de
> conhecer a um astrólogo ( a ) SÉRIO, COMPETENTE E C O N S C I E N T
>E...
> quando então ficarão bem desconfiados a respeito de uma arte tão antiga
> quanto a humanidade. Ficarão bem encafifados e também verão o quanto a
>coisa
> é diferente daquilo que sai nos jornais.
> Todo aquêle cascão da má informação e da superstição irá cair fora.
> Está nos astros... aguardem e/ou procurem. Alguns ficarão menores, mas
>de pé
> no chão (e ainda assim seguirão sem qualquer explicação cartesiana, tipo
> batatolina!).
> A gente não pode gastar a vida só duvidando da inteligência dos outros,
>por
> todo o tempo, pô!
> Sinceramente, M. SP 04/fev
>
> >From: Maria Natália <grasdic@hotmail.com>
> >Reply-To: ciencialist@yahoogrupos.com.br
> >To: ciencialist@yahoogrupos.com.br
> >Subject: [ciencialist] Re: Zodiaco
> >Date: Fri, 04 Feb 2005 04:05:36 -0000
> >
> >
> >Sérgio:
> >
> >Agradeço.
> >A astrologia se usa aqui para fazer estatísitica a brincar. Pega-se
> >num jornal e se vê qual o signo que diz " vais cair do escadote". Só o
> >professor sabe qual o jornal escolhido e todos respondem ao
> >questionário. Depois os que eram do signo da previsão "má" sofrem
> >comparação.Mas todos preenchem o inquérito. As perguntas também
> >permitem tirar conclusões sobre supertições, crenças e mitos. Quanto
> >mais turmas mais científico será (amostra maior)
> >Ao Ofiuco também se chama Serpentário, me lembrei agora.
> >Na escola costumamos apenas nos referirmos às constelações mais
> >visíveis em cidades com poluição luminosa. Ora se se souber que aqui
> >onde vivo da Ursa Menor apenas se vê a Polar e mesmo meia "safada" nem
> >capricónio nos interessa para meninos de 11 e 12 anos.
> >Quando os alunos fazem a "fita" do Zodíaco na parede da sala de aula,
> >cúbica ou paralelipipédica, se representam todas para se perceber essa
> >do "sol está na constelação..." sendo o Sol um aluno que anda junto às
> >paredes simulando o movimento aparente e a Terra os restantes alunos
> >num foco. De resto as constelações só começa a ter muito valor quando
> >queremos ver cometa ou asteróide
> >Não gostamos quando, a meio da noite, nos aparecem adultos a perguntar
> >se pelo facto de Vénus estar em Virgem eles vão ter mais sorte no
> >Totoloto.
> >Daí as alergias com a astrologia.
> >E este ano tive até um pai engenheiro a perguntar-me se achava que o
> >professor X iria faltar muito...O home estava a confundir astrónomo
> >com astrólogo. Aceitamos bem melhor sermos gastrólogos e gastrónomos.
> >Mas a astrologia dá para muita brincadeira. É que este pessoal que nos
> >chega à mão aos 11 anos já tem pelo menos 3 anos de laboratórios e
> >trabalhos de campo. Vem com a mania de apanhar a alma na ponta de um
> >bisturi. Vamos ver o que esta geração dará na universidade dentro de 4
> >anos. Pois se trata da reforma do ensino em curso.
> >Entretanto já lera o artigo do Galileu noutra lista, o CdA
> >
> >Abraço
> >Maria Natália
> >PS Pois como agora estou a dar aulas longe de casa o tempo vai
> >falhando para acompanhar diariamente a lista...
> >
> >--- Em ciencialist@yahoogrupos.com.br, "Sergio M. M. Taborda"
> ><sergiotaborda@t...> escreveu
> > > Maria Natália wrote:
> > >
> > > >
> > > > Estebam:
> > > > O zodíaco dos astrólogos é o de há 2000 anos e não tem em conta os
> > > > movimentos do eixo da Terra. O zodíaco tem 13 constelações: a
>seguir
> > > > ao capricónio tem o Ofiuco...São 13 os tais signos. E agora que me
> > > > dizeis acerca do Ofiuco e que de acordo com o zodíaco actual é o
>meu
> > > > signo: poderei amanhã comprar lotaria para me sair prémio?
> > >
> > > Eu ja respondi a essa pergunta. E a resposta é : Esse argumento
>faria
> > > sentido SE a astrologia fosse baseada nas constelações.
> > > Como não é , insistir nesse argumento é pura besteira (=coisas que
>os
> > > bestas fazem)
> > > Jà lhe expliquei pq so ha 12 signos, o que já foi aqui explicado por
> > > outros. É uma razão matemática para que seja assim. É baseada no
> > > conceito de Periodo Sinódico.
> > > Ha uma logica , séria, matemática, fisica, _astronomica_ para o
>numero
> > > 12. Só que aos astronomos - com problemas de falta de atenção -
> > > inventaram esse argumento sobre Ofucio.
> > > fazer o quê ? Mas se vc acredita nesses mentirosos, não é diferente
>de
> > > quem acredita em astrologos fajutos.
> > >
> > > > Desculpe, por acaso ouvi dizer que os astrónomos e astrofísicos
>falam
> > > > besteira? Ou entendi mal?
> > >
> > > Ouvio bem. Eles dizem mesmo. E ha muitas mais do que a de Ofucio.
> > >
> > >
> > > Sérgio Taborda
> > >
> > >
> > >
> > > --
> > > No virus found in this outgoing message.
> > > Checked by AVG Anti-Virus.
> > > Version: 7.0.300 / Virus Database: 265.8.5 - Release Date:
>03-02-2005
> >
> >
> >
>
>
>
>
> ##### ##### #####
>
> Para saber mais visite
> http://www.ciencialist.hpg.ig.com.br
>
>
> ##### ##### ##### #####
>
>
> Yahoo! Grupos, um serviço oferecido por:
> PUBLICIDADE
>
>
>
>
>------------------------------------------------------------------------------
> Links do Yahoo! Grupos
>
> a.. Para visitar o site do seu grupo na web, acesse:
> http://br.groups.yahoo.com/group/ciencialist/
>
> b.. Para sair deste grupo, envie um e-mail para:
> ciencialist-unsubscribe@yahoogrupos.com.br
>
> c.. O uso que você faz do Yahoo! Grupos está sujeito aos Termos do
>Serviço do Yahoo!.
>
>
>
>[As partes desta mensagem que não continham texto foram removidas]
>




SUBJECT: RE: [ciencialist] Zodiaco e Acupuntura e Homeopatia..:-)
FROM: "murilo filo" <avalanchedrive@hotmail.com>
TO: ciencialist@yahoogrupos.com.br
DATE: 05/02/2005 20:02

Oraculo.
Conheço algo de homeopatia só como experiência pessoal...
P/constipação há o famoso ''46'' do Almeida Prado, que é ''pá-pum!''
Para animais já lí o suficiente para saber que a HP não é algo p/ser
descartado cegamente.
Perdão pela comparação com o Quevedo! Achei que vc se esforça demais, faz
muita ginástica, para negar, destruir e ridicularizar trecos que não
deveriam ter importância para você e que estão fora do seu mundo.
Mas fica tranquilo, eu entendo, sei que vc gosta de provocar, polemizar e
argumentar. Eu também gosto, mas às vêzes tenho muita preguiça...
abr/M.

>From: "Oraculo" <oraculo@atibaia.com.br>
>Reply-To: ciencialist@yahoogrupos.com.br
>To: <ciencialist@yahoogrupos.com.br>
>Subject: [ciencialist] Zodiaco e Acupuntura e Homeopatia..:-)
>Date: Sat, 5 Feb 2005 19:39:24 -0200
>
>Olá Murilo
>
>risos..:-) Bem, mesmo me chamando de padre Quevedo (o que me deixou
>assustado..:-), não refutou meus argumentos sobre a acupuntura..:-)
>Considero isso uma boa coisa, já que estes parecem ser eficazes em
>convence-lo (ou pelo menos a fazer você pensar sobre eles..:-)
>
>Mas a homeopatia segue o mesmo raciocinio. Ela tem definição precisa, ou
>deixa de ser a homeopatia. Veja, seu criador, Hahnemann, nada sabia sobre
>micro-organimos, infecção, sistema imunologico, etc. Sua criação curava
>doenças a partir de determinados principios. Ou se aceita os principios, ou
>se recusa o termo homeopatia. Eu acho que foi um avanço na época, já que a
>medicina usava sanguessugas, purgantes e coisas assim para tratar. Parar
>com esses "tratamentos" e esperar a doença se curar em geral era mais
>benéfico ao paciente..:-)
>
>Ele dizia, por exemplo, que doenças cronicas eram causadas por um tipo de
>espírito maligno, ou misma, a psora (como em psoriase). Devo levar essa
>afirmação também, junto ao resto das alegações da homeopatia? Ou posso
>filtrar, com alguma ferramenta racional, o que é correto de que é
>incorreto? E, se posso fazer isso, que ferramenta deve usar? E o que fazer
>se a ferramenta derrubar todas as afirmações, e não apenas as mais
>estranhas?
>
>Influenciar animais é possivel. Na verdade, para a maior parte dos
>mamiferos (principalmente os animais gregarios) a atençao e o cuidado
>pessoal traz benficios no bem estar. São animais, mas não são estupidos ou
>desprovidos de mente ou de processos mentais (filhotes de macacos
>abandonados pelas mães, preferem mães substitutas feitas de pelo e algodão,
>mas sem alimentos, do que mães substitutas feitas de arame, mas que
>poderiam alimenta-los). Tenho um amigo que, depois de tentar bastante o
>tratamento de seus cavalos e gado com homeopatia (principalmente para a
>perigosa constipação) desistiu por absoluta falta de efeitos. Esse relato,
>unico e anedotico, valida a tese que a homeopatia não funciona? Claro que
>não..:-) Tanto quanto um relato contrário, de efeito positivo, não valida a
>afirmação de que a homeopatia é real.
>
>Apenas façam com que a homeopatia passe em estudos controlados, e minha
>conclusão sobre sua eficácia muda..:-)
>
>Mas, enquanto ela falhar, e enquanto seu mecanismo de ação for inexistente
>(ou francamente incorreto), fico com a conclusão cientifica de que é um
>placebo, na melhor das hipoteses..:-)
>
>Um abraço
>
>Homero
> ----- Original Message -----
> From: murilo filo
> To: ciencialist@yahoogrupos.com.br
> Sent: Friday, February 04, 2005 11:06 PM
> Subject: RE: [ciencialist] Zodiaco e Acupuntura..:-)
>
>
> Oraculo, oi.
> Vc está parecendo o Padre Quevedo fazendo ginástica anti-espírito!
> Conseguí achar um negócio interessante sobre a homeopatia, que também
>foi
> ''atacada'' nesta discussão!
> FAVOR irem a www.arenales.com.br e lá encontrarão muita
>informação,
> onde especialmente destaco muita ciência e homeopatia para ANIMAIS. Um
>lance
> psicossomático, certamente.
> A arte de ''sugestionar'' animais é maravilhosa, não? :]]] abr/M.
>
> >From: "Oraculo" <oraculo@atibaia.com.br>
> >Reply-To: ciencialist@yahoogrupos.com.br
> >To: <ciencialist@yahoogrupos.com.br>
> >Subject: [ciencialist] Zodiaco e Acupuntura..:-)
> >Date: Fri, 4 Feb 2005 16:49:32 -0200
> >
> >Olá Esteban
> >
> >Há uma sequencia de enganos em suas colocações, talvez derivada de um
> >compromentimento com a crença inicial e uma certa aversão a
>ciência..:-).
> >Métodos usados na pesquisa cientifica não servem apenas para impedir
>que
> >alegações reais, mas incomodas, sejam demonstradas. Servem para filtar
> >alegações sem base na realidade.
> >
> >Assim, acusar os que pesqusiasm com ironias do tipo "santos céticos
> >imparcialistas" é bobagem e uma falacia "ad hominem". A não ser que
>credite
> >tudo a Grande Conspiração Mundial Da Ciência Para Esconder A Verdade. E
> >nesse caso, nada do que eu, ou qualquer outro diga, fará a menor
>diferença,
> >certo?..:-)
> >
> >Este trecho é um problema:
> >
> >Talvez devesse dar algum crédito mesmo. Já estão acreditando em
> >acupuntura... Mas foi testada, nê, botaram uns eletrodozinhos e viram
>que
> >tinham razão aqueles outros velhinhos há mais de 3000 anos atrás...
> >Acordaaaaa!!!!
> >
> >Isso já foi explicado diversas vezes, mas vai de novo..:-) E, não
>precisa
> >acreditar em mim, basta apenas pensar sobre o que argumento e ver se é
> >possível refutar, ok?
> >
> >Acupuntura é a cura de todas as doenças pela manipulação de meridianos
>de
> >energia não detectável e mística através da implantação de agulhas em
> >pontos determinados desses meridianos. Isso é acupuntura e a base dessa
> >afirmação é que a causa de todas as doenças é o desequilibrio de
>energias
> >não detectáveis.
> >
> >Recentemente, detectou-se algum efeito na sintese de prostaglandinas ao
>se
> >aplicar agulhas na pele de pacientes. Como as prostaglandinas estão
> >envolvidas nas sensações dolorosas, isso pode causar algum efeito do
>tipo
> >anestésico leve pelas agulhas.
> >
> >Mas, isso NÃO É MAIS ACUPUNTURA. Isso é outra coisa, uma nova
>descoberta
> >sobre efeitos de agulhas em sintese de prostaglandinas. Nào exige que
>os
> >meridianos sejam identificados, nào exige que pontos especificos sejam
> >encontrados, e pode ser substituida por beliscões, em seu efeito nas
> >prostaglandinas.
> >
> >Distorcer um conceito, como a definição de acupuntura, para caber em
> >descobertas recentes e cientificas, é um procedimento padrão em
> >pseudociencias, também usado pela astrologia. Mas é evidente que, se
>não se
> >baseia na causa energética das doenças, se não exige o reequilíbrio
>dessas
> >energias, se independe de meridianos por onde essa enegia corre, não é,
> >claro, acupuntura.
> >
> >O que a acupuntura alega, e que nào é reconhecido pela ciência por não
>ter
> >se mostrado eficaz e nào ter mecanismo de ação reconhecivel, é que
>doenças
> >são causadas por desequilibrios de energia e que podem ser curadas
> >reequilibrando essas energias com agulhas em pontos específicos e bem
> >determinados da pele. Ela não alega que espetar agulhas (ou causar
> >sensações dolorosas de algum tipo) na pele das pessoas, interfere na
> >sintese de prostaglandinas, podendo servir como anestésico leve.
> >
> >Veja, nào deve acreditar em mim, deve apenas analisar o argumento. Se
>puder
> >demonstrar que está incorreto, ótimo. Se não puder, então ele é valido
>e
> >deve se manter até que algo mude (por exemplo, descobrir que energias
>nào
> >detectáveis causam tuberculose).
> >
> >Aqueles velhinhos de 3000 anos continuam enganados quanto a suas
>alegações.
> >Continuam enganados ao afirmarem que doenças são causadas por
> >desequilibrios energéticos. Continuam enganados ao afirmar que a
>energia
> >nào detectável corre por linhas determinadas chamadas meridianos.
>Continuam
> >enganados ao afirmar que podem curar doenças reequilibrando essas
>energias.
> >Continuam enganados ao afirmar que o reequilibrio se dá com agulhas em
> >pontos determinados da pele. E nào estão enganados sobre a sintese de
> >prostaglandinas, simplesmente porque nada sabiam sobre sintese de
> >prostaglandinas ou sobre qualquer outro sistema metabolico do corpo
>humano.
> >
> >Homero
> >
> >
> >
> > ----- Original Message -----
> > From: Esteban Moreno
> > To: ciencialist@yahoogrupos.com.br
> > Sent: Friday, February 04, 2005 4:24 PM
> > Subject: Re: [ciencialist] Re: Zodiaco
> >
> >
> >
> > Mr Takata wrote:
> > > descrições astrológicas?
> > Entao, em principio pareceria 'suspeito'. Mas como dito, qdo
> > devidamente testada a astromancia nao parece ter um bom poder de
> >predicao.
> >
> > E:
> > Devidamente testada por quem? Pelos santos céticos imparcialistas da
> > ciência?
> > Depende de quem ou como se pesquisa. Há uma série de artefatos e
> >falácias do
> > que julgam como pesquisas comprobatórias. Não vou enumera-las, mas
> >talvez
> > ajude a lembrar que a pesquisa da universidade de brasília resultou
>em
> >dados
> > bastante promissores. Que péssimo! Uma das justificativas dos
>céticos
> >acerca
> > do porquê os pacientes sentem-se bem descritos pelo astrólogo é a
> >chamada
> > confluência (auto-atribuição é um nome pobre para isso). Abaixo
>segue um
> > artigo científico que "desmistifica" (perdoe-me..) este recurso:
> >
> > Self-attribution, sun-sign traits, and the alleged role of
> >favourableness as
> > a moderator variable:
> > long-term e.ect or artefact? Edgar Wunder* Gesellschaft fu¨r
> >Anomalistik,
> > Postfach 1202, 69200 Sandhausen, Germany Personality and Individual
> > Differences 35 (2003) 1783-1789 www.elsevier.com/locate/paid
> >
> > Posso envia-lo em anexo.
> >
> > Takata:
> > > Um bom parâmetro é a quantidade do mercado que vem se
> > > ampliando extensivamente, incluindo grandes empresas e
>corporações.
> > Esse na verdade eh um pessimo parametro. Se for por isso deveremos
> > creditar confianca na homeopatia, passes espirituais (sem falar na
> > cirurgia mediunica), toda sorte de mandingas...
> >
> > E:
> > Talvez devesse dar algum crédito mesmo. Já estão acreditando em
> > acupuntura... Mas foi testada, nê, botaram uns eletrodozinhos e
>viram
> >que
> > tinham razão aqueles outros velhinhos há mais de 3000 anos atrás...
> > Acordaaaaa!!!!
> >
> >
> > Infelizmente só retorno daqui a 10 dias.
> > Um carnaval cheio de boas mandingas para vocês.
> > Esteban.
> >
> >
> >
> > ##### ##### #####
> >
> > Para saber mais visite
> > http://www.ciencialist.hpg.ig.com.br
> >
> >
> > ##### ##### ##### #####
> >
> >
> > Yahoo! Grupos, um serviço oferecido por:
> >
> >
> >
> >
> >
> >
> >
>
> >------------------------------------------------------------------------------
> > Links do Yahoo! Grupos
> >
> > a.. Para visitar o site do seu grupo na web, acesse:
> > http://br.groups.yahoo.com/group/ciencialist/
> >
> > b.. Para sair deste grupo, envie um e-mail para:
> > ciencialist-unsubscribe@yahoogrupos.com.br
> >
> > c.. O uso que você faz do Yahoo! Grupos está sujeito aos Termos do
> >Serviço do Yahoo!.
> >
> >
> >
> >[As partes desta mensagem que não continham texto foram removidas]
> >
>
>
>
>
> ##### ##### #####
>
> Para saber mais visite
> http://www.ciencialist.hpg.ig.com.br
>
>
> ##### ##### ##### #####
>
>
> Yahoo! Grupos, um serviço oferecido por:
>
>
>
>
>
>
>
>------------------------------------------------------------------------------
> Links do Yahoo! Grupos
>
> a.. Para visitar o site do seu grupo na web, acesse:
> http://br.groups.yahoo.com/group/ciencialist/
>
> b.. Para sair deste grupo, envie um e-mail para:
> ciencialist-unsubscribe@yahoogrupos.com.br
>
> c.. O uso que você faz do Yahoo! Grupos está sujeito aos Termos do
>Serviço do Yahoo!.
>
>
>
>[As partes desta mensagem que não continham texto foram removidas]
>




SUBJECT: Re: [ciencialist] Re: Zodiaco
FROM: "Oraculo" <oraculo@atibaia.com.br>
TO: <ciencialist@yahoogrupos.com.br>
DATE: 05/02/2005 20:11

Olá Taborda

Bem, eu não acredito em magia, mas os criadores da astrologia acreditavam..:-) E poderiam perfeitamente ter criado a astrologia da magia, de revelações misticas, conversas com deuses, cogumelos magicos, mitos ancestrais, etc. Faz pouca diferença..:-)

E a matematica do zodiaco pode ser, e deve ser, precisa, mas isso em nada melhora sua eficacia funcional, conforme esta alega ter. Determinar setores do céu, acreditando que estes influenciam a vida na Terra, precisamente, nada indica que essa influencia era real.

Indica no máximo que eles eram melhores matemáticos que eu..:-) Mas isso eu já sabia (e a lista também..:-)

Taborda, se esconder atrás da matematica da astrologia, não vai embasar a alegação de eficácia ou realidade da mesma. Dividiram 12 setores do céu, seja porque o número 6 era sagrado, ou seja porque decidiram encaixar cada setor exatamente em uma constelação é irrelevante como defesa da astrologia. E, sim, poderiam ter a crença de que 13 era um número sagradao e incluido um setor a mais, mesmo sendo apenas 12 constelações na época, já que o que importa era o credo, não a realidade do mesmo.

Usei 13, poderia ter usado 24 ou 6 ou 2 ou 35. Usei 13 porque alguns astrologos (certamente tolos e incapazes..:-) tem ajustado seus zodiacos para encaixar ofiuco e criar mapas astrais que incluam a precessão. O significado é que os criadores da astrologia poderiam ter usado o que fosse necessário para encaixar mitos e crenças em um conhecimento astrologico. Como lembrou o Takata, astrologia tupi-guarani ou asteca é diferente e tem matematica e interpretações diferentes.

"Taborda: Se vc soubesse mais de fisica, saberia que o sol sim gira em torno da
terra , tal como a lua, tal tudo o resto, para quem está na terra. Para
o sistema do observador solidário com a terra, tudo o resto anda À volta
dela. É por falta deste conhecimento que pouca gente entende
Relatividade (Galiena mesmo, para não falar da outra). A Primeira regra
é que o que vc observa depende do referencial que esolhe.
A astrologia, não nega que o seu referencial é a Terra. E como tal, é
correcto, observatorialmente e até fisicamente, dizer que é o sol que se
move em torno da terra."

Bem, sei disso, pelo menos a parte que faz sentido..:-) Sim, a percepção do movimento solar é que se move em torno da Terra. Dai a afirmar que tanto faz e que é fisicamente correto dizer que este gira em torna da Tera, me parece exagero, não?

Também sei que, mesmo observação a olho nú, os planetas se movem de forma diferente, com trajetorias celestes confusas, inexplicáveis sem o conhecimento de que todos, inclusive a Terra, giram em torno do Sol. Foi isso, inclusive, que embasou boa parte da teoria heliocentrica, não? As esferas dentro de esferas, cada vez mais complexas, para tentar explicar os movimentos retrogrados de alguns planetas, tornaram dificil defender o geocentrismo (viu, também tenho alguma cultura .:-)

Se isso foi uma defesa do conhecimento superior dos criadores da astrologia, foi fraco.

"Taborda:A astrologia é o estudo das estrelas e das constelações, mas não
"estrela" e "constelaçao" no sentido vulgar que vc dá a essas palavras
hoje , e sim, a que era dada à milhares de anos atrás. Eles não faziam
destinção ente Estrela e Planeta como os astornomos. Tudo eram estrelas,
mas umas moviam-se e outras não. (Tudo isto do ponto de vista de quem
olha da Terra) . As que não se movem não contém informação e server
apenas como referencia visual do referencial asbtracto que estamos
usando. As que se movem, as Planetas, forma configurações diversas no
ceu, e são essas configurações que estavam interessados em estudar.
Esses arranjos, essas constelações de estrelas moveis, e não as
constelações de estrelas fixas que não têm nenhum importancia para o
fenomeno. (Um referencial, nunca tem)

Deu para entender pq a astrologia é aquilo que é e pq os argumentos como
os seus e os do takata são ocos de sentido logico ? Pq eles são mais
bestas do que os argumentos dos astrologos em si ?
Se não deu é pura má fá da sua parte. Acho que o texto é bem explicativo."

Sim, explica que uma crença forte não pode ser refutada com tolices como argumentos e evidencias..:-) Se não devo dar o sentido atual de estrela e constelações, fica pior ainda. Devo usar um sentido de milhares de anos atrás, sentido que não faz sentido, já que eles ignoravam muito do que é real nesses objetos do universo. E continua com problema, como podem, seja que conceito use, conter informação, essas criações irreais da imaginação humana, principalmente informação sobre o universo fisico e real, que depende dos conceitos atuais e dados sobre a realidade destes?

Eles decidiram que as fixas não tem importancia, e que as que se movem, informam sobre o mundo e sobre a personalidade das pessoas. Ótimo..:-) E provam ou mostram evidencias dessa alegação como? Quem decide? Como testar? Depois que decidiram isso, alguém mais decidiu alguma coisa, ou o conhecimento se mantém estático a 3000 anos? Se decidissem de qualquer outra forma, teriamos que acreditar e pronto??? De onde tiram essas alegações, da matematica e pronto???

Se existe um argumento besta, esse ganharia o título de mais besta..:-)

Homero: > Sempre mais do mesmo. Apenas prove o que afirma, e nenhuma discussão
> seria necessária.

Taborda: Acabei de o fazer.
Claro que vc não está convencido, pq vc escolhe não entender, mas não
pode negar que a logia está ali. A historia, a metamática, a fisica e
até a semântica, confirmam.
Que mais provas vc quer ?

O que você provou??? Que sabe mais sobre a matematica usada que eu??? he he he.. Tem de ser, você é astrologo e eu não..:-) Isso é uma alegação correta, mas e daí??? Não há logica, ou melhor, a lógica aqui apresentada por você não é evidencia, é circular. A astrologia pode conter informação sobre o mundo real e sobre as pessoas porque os que a criaram usavam matematica e liam informação real nos planetas, mas não com o conceito atual, mas com o dos tempos antigos. E daí??? O que isso prova??? Isso é uma afirmação, não uma evidencia.

Prova seria ligar, com bases confiáveis, resultados de previsões astrologicas e dados reais. Isso seria uma prova. Mas, jogos de palavras como "tudo pode ser positivo ou negativo, depende". não é nada disso. Ninguém que lesse esta mensagem diria, bem, agora está provado que calculos astrologicos podem determinar a personalidade de uma pessoa com base na data e hora de seu nasciemento, informação que está contida nas posições de planetas no céu.

Desculpe, mas não há nenhuma prova ou evidencia disso em nada nos argumentos.

Homero






----- Original Message -----
From: Sergio M. M. Taborda
To: ciencialist@yahoogrupos.com.br
Sent: Friday, February 04, 2005 11:31 PM
Subject: Re: [ciencialist] Re: Zodiaco


Oraculo wrote:

> Olá Taborda
>
> A respsta é sempre a mesma: prove isso tudo. Demonstre que leoninos
> são bons pais, que tem relação com seu signo, etc.

Eu não disse que os leoninos são bons pais. Não me obrigue a provar o
que eu não afirmei , nem é verdade.

>
>
> Sem isso (e continuamos sem isso..:-),

E vai continuar sem isso, pq vc quer que eu prove mentiras.

> O caso dos setores é evidente. Se a astrologia não existisse, mas
> fosse inventada (inventada é um termo preciso..:-)


Acaso vc não saiba, a astrologia foi inventada sim. Tal como a fisica, a
quimica, a electronia , a politica a economia .. . alias como qq
actividade intelectual do ser humano.

> hoje, teriamos 13 signos, e 13 setores.


Prove que sim. Em que se baseia para dizer isso ?
Ah , ja sei, aquela asneira sobre as constelações. Otimo. Divida o ceu
em 13 partes, quantos graus tem cada uma ?
Vc esqueceu que 13 é um numero primo ? Acaso o que lhe ensinaram na
escola não serve para nada ? Vc não sabe que não se pode divir um numero
por um numero primo e obter um numero inteiro, a menos que esse numero
seja um multiplo dele ? Ah ! afinal vc diz-se muito seguidor do
pensamento logico, mas falha nestes pequeno promenores. Que raios vc
pensa afinal ?

Vc esquece o elementar. A astrologia indu não usa graus. Foram os
gregos que intruziram toda a matematica da coisa.
Se originalmente tivessem escolhido um signo para cada constelação
teriam escolhido 13. Ou vc acha que a 13º constelação aprecer do dia
para a noite ? Acha que eles não a viram ?
Se assim fosse o circulo teria hoje 520 graus (13x4) e não 360.
Mas não, escolheram 12. que os gregos multiplicaram por 4. O pq do 4 é
outra historia.

Se a sua teoria sobre as constalções fosse certa, explique pq eles
ignoraram a 13º.

Tlv isto ajude, vc sabia que o periodo sinodico da lua (29 dias e alguma
coisa) cai 12, e não 13, vezes no do sol

365 / 29 = 12

O que sobra é a precessão do zodiaco , e por isso que o zodiaco
astrologico não é mais sincronizado com o zodiaco celeste.
Matemática, meu caro, não é magia. É Matemática. E lembre-se que eles
fizeram o calculo sem computadores e sem medições de periodos siderais
apenas pelo bom e velho metodo que aprendemos na escola primária.. .bom,
pelo menos eu aprendi.

Vc imagina que a astrologia foi inventada como ? Baseada em quê ?
Palhaçada ?
Prove-se que faz sentido logico-amtematico-astronomico escolher o numero
13 (que é um numero primo) e darei o braço a torcer.

> Como poderia alguém que estivesse criando a astrologia hoje determinar
> que um dos setores não deveria ser utilizado? Como ele teria acesso a
> essa informação, se existem 13 constelações a considerar ?

Se vc soubesse mais de astronoia básica, saberia explicar pq, tal como
eu fiz acima.

>
> Como o astrologo nos informou, valem os setores criados pelos
> assirios, com base em seu céu visivel na época.

E como eles os criaram ? Por magia ? Vamos, lá., vc - o ceptico -
acreditando em magia ?

> Se criada hoje, a astrologia seria de 13 signos e totalmente diferente
> em seus significados e interpretações e os mapas astrais diferentes
> nos resultados.

Vc continua repetindo isso mas sem explicar pq . Qual a razão , a regra,
que faria isso acontecer ? Pq 13 ?

> Como pode isso ser uma fonte de conhecimento confiável ou mesmo real?

A sua ou a minha ?

> Conhecimento que determinava que o Sol e a Lua eram planetas e agiam
> como tal, inclusive girando em torno da Terra (a Lua até que gira,
> embora não como os planetas, mas o Sol..:-).

Se vc soubesse mais de fisica, saberia que o sol sim gira em torno da
terra , tal como a lua, tal tudo o resto, para quem está na terra. Para
o sistema do observador solidário com a terra, tudo o resto anda À volta
dela. É por falta deste conhecimento que pouca gente entende
Relatividade (Galiena mesmo, para não falar da outra). A Primeira regra
é que o que vc observa depende do referencial que esolhe.
A astrologia, não nega que o seu referencial é a Terra. E como tal, é
correcto, observatorialmente e até fisicamente, dizer que é o sol que se
move em torno da terra.

Toda a discussão sobre se o sol era o centro ou não era uma discussão
astrologia. Tlv vc não soubesse disto, e pensasse que era uma discussão
fisica, ou filosofica... .
Mas claro, vc não acredita nisso.

Planeta é uma palavra grega , femenina , que significa Estrela Errante.
Qq fisico sabe que aquilo que não se move, não contem informação. É no
movimento, na mudança das coisas, que está a informação. Então é nos
Planetas, Estrelas que se movem, e não nas Estrelas fixas que deveriamos
concentrar o estudo. É por isso que em astrologia chamamos a lua e o sol
de planetas. Não é para chatear os astronomos. É para ser fiel ao
significado das palavras. Pq eles são estrelas que se movem (são astros
luminosos)
Mas pq Astrologia e não Planetalogia ? Pq o que a astrologia realmente
estuda não são os planetas em si, suas posições etc... isso é apenas uma
ferramenta. O que interessa são os arranjos que eles fazem uns com os
outros no ceu. A forma como eles se agrupam. A forma como as estrelas
errantes de dispoem relativamente umas às outras.Aquilo que hoje
chamamos Aspectos. Mas como se chamava a disposição de estrelas umas em
relação às outras no tempo dos gregos?
O mesmo que hoje :Constelação. Que em grego se diz, Astron e dai
Astron+Logia = Astrologia = Estudo/Catalogo das Constelações , dos
arranjos das estrelas.
Mas entenda de que estrelas estamos falando aqui!
Estamos falando das estrelas que se movem, as Planetas e não das
Estrelas fixas. UAU! Agora vc ficou confuso , sim ? Otimo.

A astrologia é o estudo das estrelas e das constelações, mas não
"estrela" e "constelaçao" no sentido vulgar que vc dá a essas palavras
hoje , e sim, a que era dada à milhares de anos atrás. Eles não faziam
destinção ente Estrela e Planeta como os astornomos. Tudo eram estrelas,
mas umas moviam-se e outras não. (Tudo isto do ponto de vista de quem
olha da Terra) . As que não se movem não contém informação e server
apenas como referencia visual do referencial asbtracto que estamos
usando. As que se movem, as Planetas, forma configurações diversas no
ceu, e são essas configurações que estavam interessados em estudar.
Esses arranjos, essas constelações de estrelas moveis, e não as
constelações de estrelas fixas que não têm nenhum importancia para o
fenomeno. (Um referencial, nunca tem)

Deu para entender pq a astrologia é aquilo que é e pq os argumentos como
os seus e os do takata são ocos de sentido logico ? Pq eles são mais
bestas do que os argumentos dos astrologos em si ?
Se não deu é pura má fá da sua parte. Acho que o texto é bem explicativo.

> Sempre mais do mesmo. Apenas prove o que afirma, e nenhuma discussão
> seria necessária.

Acabei de o fazer.
Claro que vc não está convencido, pq vc escolhe não entender, mas não
pode negar que a logia está ali. A historia, a metamática, a fisica e
até a semântica, confirmam.
Que mais provas vc quer ?

Sérgio Taborda


--
No virus found in this outgoing message.
Checked by AVG Anti-Virus.
Version: 7.0.300 / Virus Database: 265.8.5 - Release Date: 03-02-2005



##### ##### #####

Para saber mais visite
http://www.ciencialist.hpg.ig.com.br


##### ##### ##### #####


Yahoo! Grupos, um serviço oferecido por:

São Paulo Rio de Janeiro Curitiba Porto Alegre Belo Horizonte Brasília




------------------------------------------------------------------------------
Links do Yahoo! Grupos

a.. Para visitar o site do seu grupo na web, acesse:
http://br.groups.yahoo.com/group/ciencialist/

b.. Para sair deste grupo, envie um e-mail para:
ciencialist-unsubscribe@yahoogrupos.com.br

c.. O uso que você faz do Yahoo! Grupos está sujeito aos Termos do Serviço do Yahoo!.



[As partes desta mensagem que não continham texto foram removidas]



SUBJECT: Re: Zodiaco e Acupuntura e Homeopatia..:-)
FROM: "Pablo Albino Pereira" <pablo@unidavi.edu.br>
TO: ciencialist@yahoogrupos.com.br
DATE: 05/02/2005 20:58


Amigos, só para esclarecer. O famoso complexo homeopático almeida
prado 46, não é um medicamento homeopático. Até há alguuns anos ele
continha fenolftaleína (em doses n homeopáticas), sendo que após a
probição desta substância (fármaco???) foi colocado na formulação 5 mg
de picosulfato de sódio por comprimido, que é um laxante de contato. O
mesmo fármaco é encontrado no Guttalax, com cada comprimido contendo
2,5 mg!!! Então, n é a toa que um comprimidinho do 46 seja tão
efetivo. Na verdade, tão efetivo que contraria a lógica homeopática,
simplesmente por ser um medicamento alopático disfarçado (a fórmula
contém algumas diluições decimais homeopaticamente irracionais).

Todo profissional homeopata (farmacêutico, médico, dentista, m.
veterinário, etc) condena os AP.

Isso só pra esclarecer que o exemplo do 46, utilizado pelo Murillo, n
é adequado. N quero com isso afirmar que homeopatia não funciona, pois
sabe-se que funciona. Apenas, n se sabe como, pois todas as hipotéses
colcadas são, ainda, cientificamente insondáveis...mas esta é outra
história.
[]'s Pablo

--- Em ciencialist@yahoogrupos.com.br, "murilo filo"
<avalanchedrive@h...> escreveu
> Oraculo.
> Conheço algo de homeopatia só como experiência pessoal...
> P/constipação há o famoso ''46'' do Almeida Prado, que é ''pá-pum!''
> Para animais já lí o suficiente para saber que a HP não é algo p/ser
> descartado cegamente.
> Perdão pela comparação com o Quevedo! Achei que vc se esforça
demais, faz
> muita ginástica, para negar, destruir e ridicularizar trecos que não
> deveriam ter importância para você e que estão fora do seu mundo.
> Mas fica tranquilo, eu entendo, sei que vc gosta de provocar,
polemizar e
> argumentar. Eu também gosto, mas às vêzes tenho muita preguiça...
> abr/M.
>
> >From: "Oraculo" <oraculo@a...>
> >Reply-To: ciencialist@yahoogrupos.com.br
> >To: <ciencialist@yahoogrupos.com.br>
> >Subject: [ciencialist] Zodiaco e Acupuntura e Homeopatia..:-)
> >Date: Sat, 5 Feb 2005 19:39:24 -0200
> >
> >Olá Murilo
> >
> >risos..:-) Bem, mesmo me chamando de padre Quevedo (o que me deixou
> >assustado..:-), não refutou meus argumentos sobre a acupuntura..:-)
> >Considero isso uma boa coisa, já que estes parecem ser eficazes em
> >convence-lo (ou pelo menos a fazer você pensar sobre eles..:-)
> >
> >Mas a homeopatia segue o mesmo raciocinio. Ela tem definição
precisa, ou
> >deixa de ser a homeopatia. Veja, seu criador, Hahnemann, nada
sabia sobre
> >micro-organimos, infecção, sistema imunologico, etc. Sua criação
curava
> >doenças a partir de determinados principios. Ou se aceita os
principios, ou
> >se recusa o termo homeopatia. Eu acho que foi um avanço na época,
já que a
> >medicina usava sanguessugas, purgantes e coisas assim para tratar.
Parar
> >com esses "tratamentos" e esperar a doença se curar em geral era mais
> >benéfico ao paciente..:-)
> >
> >Ele dizia, por exemplo, que doenças cronicas eram causadas por um
tipo de
> >espírito maligno, ou misma, a psora (como em psoriase). Devo levar
essa
> >afirmação também, junto ao resto das alegações da homeopatia? Ou posso
> >filtrar, com alguma ferramenta racional, o que é correto de que é
> >incorreto? E, se posso fazer isso, que ferramenta deve usar? E o
que fazer
> >se a ferramenta derrubar todas as afirmações, e não apenas as mais
> >estranhas?
> >
> >Influenciar animais é possivel. Na verdade, para a maior parte dos
> >mamiferos (principalmente os animais gregarios) a atençao e o cuidado
> >pessoal traz benficios no bem estar. São animais, mas não são
estupidos ou
> >desprovidos de mente ou de processos mentais (filhotes de macacos
> >abandonados pelas mães, preferem mães substitutas feitas de pelo e
algodão,
> >mas sem alimentos, do que mães substitutas feitas de arame, mas que
> >poderiam alimenta-los). Tenho um amigo que, depois de tentar
bastante o
> >tratamento de seus cavalos e gado com homeopatia (principalmente
para a
> >perigosa constipação) desistiu por absoluta falta de efeitos. Esse
relato,
> >unico e anedotico, valida a tese que a homeopatia não funciona?
Claro que
> >não..:-) Tanto quanto um relato contrário, de efeito positivo, não
valida a
> >afirmação de que a homeopatia é real.
> >
> >Apenas façam com que a homeopatia passe em estudos controlados, e
minha
> >conclusão sobre sua eficácia muda..:-)
> >
> >Mas, enquanto ela falhar, e enquanto seu mecanismo de ação for
inexistente
> >(ou francamente incorreto), fico com a conclusão cientifica de que
é um
> >placebo, na melhor das hipoteses..:-)
> >
> >Um abraço
> >
> >Homero
> > ----- Original Message -----
> > From: murilo filo
> > To: ciencialist@yahoogrupos.com.br
> > Sent: Friday, February 04, 2005 11:06 PM
> > Subject: RE: [ciencialist] Zodiaco e Acupuntura..:-)
> >
> >
> > Oraculo, oi.
> > Vc está parecendo o Padre Quevedo fazendo ginástica anti-espírito!
> > Conseguí achar um negócio interessante sobre a homeopatia, que
também
> >foi
> > ''atacada'' nesta discussão!
> > FAVOR irem a www.arenales.com.br e lá encontrarão muita
> >informação,
> > onde especialmente destaco muita ciência e homeopatia para
ANIMAIS. Um
> >lance
> > psicossomático, certamente.
> > A arte de ''sugestionar'' animais é maravilhosa, não? :]]] abr/M.
> >
> > >From: "Oraculo" <oraculo@a...>
> > >Reply-To: ciencialist@yahoogrupos.com.br
> > >To: <ciencialist@yahoogrupos.com.br>
> > >Subject: [ciencialist] Zodiaco e Acupuntura..:-)
> > >Date: Fri, 4 Feb 2005 16:49:32 -0200
> > >
> > >Olá Esteban
> > >
> > >Há uma sequencia de enganos em suas colocações, talvez derivada
de um
> > >compromentimento com a crença inicial e uma certa aversão a
> >ciência..:-).
> > >Métodos usados na pesquisa cientifica não servem apenas para
impedir
> >que
> > >alegações reais, mas incomodas, sejam demonstradas. Servem para
filtar
> > >alegações sem base na realidade.
> > >
> > >Assim, acusar os que pesqusiasm com ironias do tipo "santos céticos
> > >imparcialistas" é bobagem e uma falacia "ad hominem". A não ser
que
> >credite
> > >tudo a Grande Conspiração Mundial Da Ciência Para Esconder A
Verdade. E
> > >nesse caso, nada do que eu, ou qualquer outro diga, fará a menor
> >diferença,
> > >certo?..:-)
> > >
> > >Este trecho é um problema:
> > >
> > >Talvez devesse dar algum crédito mesmo. Já estão acreditando em
> > >acupuntura... Mas foi testada, nê, botaram uns eletrodozinhos e
viram
> >que
> > >tinham razão aqueles outros velhinhos há mais de 3000 anos atrás...
> > >Acordaaaaa!!!!
> > >
> > >Isso já foi explicado diversas vezes, mas vai de novo..:-) E, não
> >precisa
> > >acreditar em mim, basta apenas pensar sobre o que argumento e
ver se é
> > >possível refutar, ok?
> > >
> > >Acupuntura é a cura de todas as doenças pela manipulação de
meridianos
> >de
> > >energia não detectável e mística através da implantação de
agulhas em
> > >pontos determinados desses meridianos. Isso é acupuntura e a
base dessa
> > >afirmação é que a causa de todas as doenças é o desequilibrio de
> >energias
> > >não detectáveis.
> > >
> > >Recentemente, detectou-se algum efeito na sintese de
prostaglandinas ao
> >se
> > >aplicar agulhas na pele de pacientes. Como as prostaglandinas estão
> > >envolvidas nas sensações dolorosas, isso pode causar algum
efeito do
> >tipo
> > >anestésico leve pelas agulhas.
> > >
> > >Mas, isso NÃO É MAIS ACUPUNTURA. Isso é outra coisa, uma nova
> >descoberta
> > >sobre efeitos de agulhas em sintese de prostaglandinas. Nào
exige que
> >os
> > >meridianos sejam identificados, nào exige que pontos
especificos sejam
> > >encontrados, e pode ser substituida por beliscões, em seu
efeito nas
> > >prostaglandinas.
> > >
> > >Distorcer um conceito, como a definição de acupuntura, para
caber em
> > >descobertas recentes e cientificas, é um procedimento padrão em
> > >pseudociencias, também usado pela astrologia. Mas é evidente
que, se
> >não se
> > >baseia na causa energética das doenças, se não exige o
reequilíbrio
> >dessas
> > >energias, se independe de meridianos por onde essa enegia
corre, não é,
> > >claro, acupuntura.
> > >
> > >O que a acupuntura alega, e que nào é reconhecido pela ciência
por não
> >ter
> > >se mostrado eficaz e nào ter mecanismo de ação reconhecivel, é que
> >doenças
> > >são causadas por desequilibrios de energia e que podem ser curadas
> > >reequilibrando essas energias com agulhas em pontos específicos
e bem
> > >determinados da pele. Ela não alega que espetar agulhas (ou causar
> > >sensações dolorosas de algum tipo) na pele das pessoas,
interfere na
> > >sintese de prostaglandinas, podendo servir como anestésico leve.
> > >
> > >Veja, nào deve acreditar em mim, deve apenas analisar o
argumento. Se
> >puder
> > >demonstrar que está incorreto, ótimo. Se não puder, então ele é
valido
> >e
> > >deve se manter até que algo mude (por exemplo, descobrir que
energias
> >nào
> > >detectáveis causam tuberculose).
> > >
> > >Aqueles velhinhos de 3000 anos continuam enganados quanto a suas
> >alegações.
> > >Continuam enganados ao afirmarem que doenças são causadas por
> > >desequilibrios energéticos. Continuam enganados ao afirmar que a
> >energia
> > >nào detectável corre por linhas determinadas chamadas meridianos.
> >Continuam
> > >enganados ao afirmar que podem curar doenças reequilibrando essas
> >energias.
> > >Continuam enganados ao afirmar que o reequilibrio se dá com
agulhas em
> > >pontos determinados da pele. E nào estão enganados sobre a
sintese de
> > >prostaglandinas, simplesmente porque nada sabiam sobre sintese de
> > >prostaglandinas ou sobre qualquer outro sistema metabolico do
corpo
> >humano.
> > >
> > >Homero
> > >
> > >
> > >
> > > ----- Original Message -----
> > > From: Esteban Moreno
> > > To: ciencialist@yahoogrupos.com.br
> > > Sent: Friday, February 04, 2005 4:24 PM
> > > Subject: Re: [ciencialist] Re: Zodiaco
> > >
> > >
> > >
> > > Mr Takata wrote:
> > > > descrições astrológicas?
> > > Entao, em principio pareceria 'suspeito'. Mas como dito, qdo
> > > devidamente testada a astromancia nao parece ter um bom poder de
> > >predicao.
> > >
> > > E:
> > > Devidamente testada por quem? Pelos santos céticos
imparcialistas da
> > > ciência?
> > > Depende de quem ou como se pesquisa. Há uma série de artefatos e
> > >falácias do
> > > que julgam como pesquisas comprobatórias. Não vou
enumera-las, mas
> > >talvez
> > > ajude a lembrar que a pesquisa da universidade de brasília
resultou
> >em
> > >dados
> > > bastante promissores. Que péssimo! Uma das justificativas dos
> >céticos
> > >acerca
> > > do porquê os pacientes sentem-se bem descritos pelo
astrólogo é a
> > >chamada
> > > confluência (auto-atribuição é um nome pobre para isso). Abaixo
> >segue um
> > > artigo científico que "desmistifica" (perdoe-me..) este recurso:
> > >
> > > Self-attribution, sun-sign traits, and the alleged role of
> > >favourableness as
> > > a moderator variable:
> > > long-term e.ect or artefact? Edgar Wunder* Gesellschaft fu¨r
> > >Anomalistik,
> > > Postfach 1202, 69200 Sandhausen, Germany Personality and
Individual
> > > Differences 35 (2003) 1783-1789 www.elsevier.com/locate/paid
> > >
> > > Posso envia-lo em anexo.
> > >
> > > Takata:
> > > > Um bom parâmetro é a quantidade do mercado que vem se
> > > > ampliando extensivamente, incluindo grandes empresas e
> >corporações.
> > > Esse na verdade eh um pessimo parametro. Se for por isso
deveremos
> > > creditar confianca na homeopatia, passes espirituais (sem
falar na
> > > cirurgia mediunica), toda sorte de mandingas...
> > >
> > > E:
> > > Talvez devesse dar algum crédito mesmo. Já estão acreditando em
> > > acupuntura... Mas foi testada, nê, botaram uns eletrodozinhos e
> >viram
> > >que
> > > tinham razão aqueles outros velhinhos há mais de 3000 anos
atrás...
> > > Acordaaaaa!!!!
> > >
> > >
> > > Infelizmente só retorno daqui a 10 dias.
> > > Um carnaval cheio de boas mandingas para vocês.
> > > Esteban.
> > >
> > >
> > >
> > > ##### ##### #####
> > >
> > > Para saber mais visite
> > > http://www.ciencialist.hpg.ig.com.br
> > >
> > >
> > > ##### ##### ##### #####
> > >
> > >
> > > Yahoo! Grupos, um serviço oferecido por:
> > >
> > >
> > >
> > >
> > >
> > >
> > >
> >
> >
>------------------------------------------------------------------------------
> > > Links do Yahoo! Grupos
> > >
> > > a.. Para visitar o site do seu grupo na web, acesse:
> > > http://br.groups.yahoo.com/group/ciencialist/
> > >
> > > b.. Para sair deste grupo, envie um e-mail para:
> > > ciencialist-unsubscribe@yahoogrupos.com.br
> > >
> > > c.. O uso que você faz do Yahoo! Grupos está sujeito aos
Termos do
> > >Serviço do Yahoo!.
> > >
> > >
> > >
> > >[As partes desta mensagem que não continham texto foram removidas]
> > >
> >
> >
> >
> >
> > ##### ##### #####
> >
> > Para saber mais visite
> > http://www.ciencialist.hpg.ig.com.br
> >
> >
> > ##### ##### ##### #####
> >
> >
> > Yahoo! Grupos, um serviço oferecido por:
> >
> >
> >
> >
> >
> >
> >
>
>------------------------------------------------------------------------------
> > Links do Yahoo! Grupos
> >
> > a.. Para visitar o site do seu grupo na web, acesse:
> > http://br.groups.yahoo.com/group/ciencialist/
> >
> > b.. Para sair deste grupo, envie um e-mail para:
> > ciencialist-unsubscribe@yahoogrupos.com.br
> >
> > c.. O uso que você faz do Yahoo! Grupos está sujeito aos
Termos do
> >Serviço do Yahoo!.
> >
> >
> >
> >[As partes desta mensagem que não continham texto foram removidas]
> >





SUBJECT: Questões biológicas
FROM: Antonio Ferrão Neto <a_ferrao@terra.com.br>
TO: <ciencialist@yahoogrupos.com.br>
DATE: 05/02/2005 21:24

Caros colegas da lista...

Eu gostaria de saber dos biólogos, nutricionistas e agrônomos:
1 - como são determinadas as calorias dos alimentos, como se reduz o teor
calórico deles, como se reduz o açúcar deles e também as gorduras (saturadas
e totais).
2 - como são produzidas as sementes das frutas que não produzem sementes?
3 - sabe-se que a raça influencia fortemente no comportamento dos cães.
Assim, por exemplo, um cão labrador irá mergulhar espontâneamente numa
piscina, enquanto que cães de algumas outras raças sentirão até medo de se
aproximarem da água. Além de sermos em parte um produto do meio e termos o
nosso comportamento controlado pela razão, existe essa influência da raça
sobre o nosso comportamento (dos seres humanos)?

[]'s
Antonio Ferrão Neto
--
No virus found in this outgoing message.
Checked by AVG Anti-Virus.
Version: 7.0.300 / Virus Database: 265.8.5 - Release Date: 03/02/2005



SUBJECT: P/ Manoel (era Zodiaco e Acupuntura..:-)
FROM: "Oraculo" <oraculo@atibaia.com.br>
TO: <ciencialist@yahoogrupos.com.br>
DATE: 05/02/2005 22:29

Salve Manoel

risos..:-) Realmente, talvez minha forma de colocar a resposta tenha ficado diferente de minha intenção original e muito " intensa"..:-) Vou tentar abrandar a frase.

Ao afirmar, com maiusculas, que nesse caso não era mais acupuntura, estava respondendo a uma questão. E a questão é que define a intensidade da resposta. Quando se afirma que a ciência tem validado a acupuntura, quem o faz não está usando o termo no sentido que você colocou. Não é uma validação parcial, nem uma definição precisa e técnica de um procedimento, inserir agulhas na pela, acupuntura.

Está, na verdade, usando o termo da forma como os antigos chineses o entendem, com a definição que apresentei. Inclusive as "energias" misticas e não detectaveis, tão a gosto da nova era..:-)

Por isso minha enfase. Efeitos de anestesia provocados por interferencia na sintese de prostaglandinas e energias misticas causadores de doenças em caso de desequilibrio de meridianos são, claro, coisas bem diferentes.

E embora seja possível sim, analisar afirmações cientificas, encaixa-las em determinadas construtos da imaginação humana, isso nem de longe significa que os autores dessas criações estavam realmetne pensando nisso. Por exemplo, o Mori (Kentaro) tem um interessante texto onde analisa a história da Branca de Neve como uma descrição de ETs e tecnologia de ponta - http://www.str.com.br/ca/ezequiel_bruxa.htm. E mesmo que o enfoque seja interessante e divertido, além de bem ajustado, nada indica que o autor da Branca de Neve pensava em ETs ao cria-lo..:-)

Também é possível encaixar descobertas atuais da ciência em alegações e histórias do passado, mas é apenas curiosidade, não relação real. É um procedimento padrão, encontrar "sabedoria ancestral" depois que descobertas ciêntíficas surgem (nunca antes..:-). O mesmo com profecias, fica fácil saber o que querem dizer, se o fato já aconteceu e se temos a liberdade de "ajustar" os relatos..:-)

O conhecimento ancestral que baseou a acunpuntura é bem diferente do atual e suas premissas, a grande maioria, incorreta e irreal. Descobrir que algumas das ações, como inserir agulhas, tem efeitos reais, não valida o conjunto de conehcimento em que se baseia. Essa era a intenção do meu texto. Talvez ele ficasse melhor com uma reforma:

Isso não é mais acupuntura, no sentido dado originalmente por seus criadores. Aplicar agulhas e aplacar dores cronicas é uma legítima descoberta cientifica e atual, devendo a acupuntura original apenas a idéia da técnica envolvida..:-)

E, claro, reconheço, e sempre reconheci, o grande valor dos antigos e sua contribuição a nossos tempos. Todo cientista reconhece isso (e nem sou cientista..:-), como demonstra bem a frase famosa de Newton: Se vi tão longe, foi por me apoiar nos ombros de gigantes. Mas o respeito pelos antigos não se confunde com reverencia ou dogma a respeito de seus conhecimentos. Que eram parcos (e com os quais fizeram coisas de muito valor, mas também bastante equivocadas) e mais propensos a erros e enganos.

Se for possível separar do termo acupuntura seu significado não científico, se for possível reconhecer sua ação perfeitamente natural e material, seria interesssante manter o termo. Mas eu temo que isso acabe confundindo quem faz uso dele, permitindo que , recorrentemente, a alegação de que a "ciência tem validado a acupuntura, então os antigos chineses tinham razão e devem existir energias misticas e meridianos que causam as doenças", venha a tona e provoque este mesmo debate..:-)

Um abraço.

Homero


----- Original Message -----
From: Manuel Bulcão
To: ciencialist@yahoogrupos.com.br
Sent: Saturday, February 05, 2005 6:18 AM
Subject: [ciencialist] Re: Zodiaco e Acupuntura..:-)



Salve, Homero!

Homero escreveu: Recentemente, detectou-se algum efeito na sintese
de prostaglandinas ao se aplicar agulhas na pele de pacientes. Como
as prostaglandinas estão envolvidas nas sensações dolorosas, isso
pode causar algum efeito do tipo anestésico leve pelas agulhas.

Manuel: Talvez o efeito das agulhas seja um pouco mais potente que a
de um analgésico leve. Parece que a técnica da "acupuntura" (as
aspas são para lhe agradar :-)) é eficaz como tratamento
complementar de dores "crônicas", como a fibromialgia e até mesmo
dores oncológicas.

Homero: Mas, isso NÃO É MAIS ACUPUNTURA. (...) Não exige que os
meridianos sejam identificados, nào exige que pontos especificos
sejam encontrados, e pode ser substituída por beliscões, em seu
efeito nas prostaglandinas.

Manuel: Foi questionando e testando a técnica da "acupuntura" que a
medicina ocidental chegou à conclusão de que, "realmente", a
estimulação de certos pontos do corpo produz como resposta -- via
secreção de determinados hormônios e neurotransmissores -- analgesia
e sedação. A "acupuntura" PREVÊ esse efeito sensível, previsão esta
que foi confirmada pelos testes empíricos. Significa dizer que a
práxis científico-experimental corroborou a "acupuntura" em alguma
medida.

Além disso, parece que, consoante esses testes, e malgrado a
explicação mágica da medicina sino-nipônica arcaica, muitos pontos
corporais que correspondem aos "meridianos" não são arbitrários.

Quanto ao desequlíbrio de "energia" que causa a doença, bem, pelo
menos no que diz respeito à supressão ou amenização dos sintomas
decorrente das agulhadas, isso se deve a processos "eletroquímicos"
(portanto, "energéticos") que começa com um "choque". Não é a
energia que a turma do new age & Cia. sonha, mas é energia, sem
dúvida.

Acho que o problema está mais na forma da explicação do que no poder
de predição (não quero com isso dizer que o poder de predição de
todas as fórmulas da "acupuntura" sejam supimpas, frise-se). Mas,
será que a explicação dos fenômenos naturais EM LINGUAGEM NÃO-
MATEMÁTICA é unívoca? Há quem sustente que qualquer lei física pode
ser enunciada VALIDAMENTE até mesmo na forma de uma lenda ou conto
de fadas em que as forças da natureza figuram como sujeitos
intencionais. Por exemplo, numa dessas estórias da Carochinha, os
glúons seriam tutores liberais que só intervêm duramente quando os
tutelados (quarks) estão passando dos limites (liberdade
assintótica).

No que tange ao termo designativo dessa técnica terapêutica,
pergunto: há uma palavra melhor para nomeá-la que não
seja "acupuntura"? Será que os fatos acima mencionados não a
legitimam hoje em dia? ô, Homero, meu amigo, você que foi tão firme
ao afirmar "ISSO NÃO É MAIS ACUPUNTURA', apresente uma alternativa!

A ciência deve tanto ao conhecimento que a precedeu (ao conhecimento
empírico rasteiro e à metafísica) que tomou para si muitos das suas
imagens, entre as quais a palavra "átomo": termo cuja etimologia o
torna impróprio para designar qualquer coisa que seja composta,
divisível, quebrável, redutível ou desintegrável -- como o é
um "átomo" de urânio.

Então, por que não preservar também o termo "acupuntura"?

Confetes e serpentinas,
Manuel Bulcão





##### ##### #####

Para saber mais visite
http://www.ciencialist.hpg.ig.com.br


##### ##### ##### #####


Yahoo! Grupos, um serviço oferecido por:
PUBLICIDADE




------------------------------------------------------------------------------
Links do Yahoo! Grupos

a.. Para visitar o site do seu grupo na web, acesse:
http://br.groups.yahoo.com/group/ciencialist/

b.. Para sair deste grupo, envie um e-mail para:
ciencialist-unsubscribe@yahoogrupos.com.br

c.. O uso que você faz do Yahoo! Grupos está sujeito aos Termos do Serviço do Yahoo!.



[As partes desta mensagem que não continham texto foram removidas]



SUBJECT: Calorias nos carboidratos
FROM: "L.E.R.de Carvalho" <lecarvalho@infolink.com.br>
TO: ciencialist@yahoogrupos.com.br
DATE: 05/02/2005 22:30

At 21:24 5/2/2005, you wrote:
>Caros colegas da lista...
>
>Eu gostaria de saber dos biólogos, nutricionistas e agrônomos:
>1 - como são determinadas as calorias dos alimentos, como se reduz o teor
>calórico deles, como se reduz o açúcar deles e também as gorduras
>(saturadas e totais).
>Antonio Ferrão Neto


Olha só...
mas veja bem...

Quantas kcal tem um grama de carboidrato ?

Todo mundo não diz que são 4 kcal ?

Bem...
no arroz, não é 4, e sim 4,12
No trigo é 3,78 kcal se for farinha integral.
Se for farinha com 70% de extração (e 30% de casca jogada fora), o mix dos
carboidratos presentes já são diferentes, e fornecerão... 4,12 kcal por grama.

Do suco de limão ou da lima, os carboidratos presentes te darão 2,48 kcal
por grama.

Os da soja dão 4,07.

A gordura do trigo não fornece 9 kcal por grama, mas 8,37.

Esses são os desvios, a variação, na quantidade de energia em cada
carboidrato, quando se varia o alimento.
Com as proteinas e lipideos é o mesmo.

Imagina então o quanto não varia o teor de carboidrato de um caju pra outro.
E o teor de gordura de um abacate pra outro.

Quem come por número é computador.

Então imagina ainda os erros laboratoriais.
Olha o rótulo do Danoninho, onde diziam que o teor de proteínas era 3,476%...
quando a gente sabe que isso não é um dado exato, pois no laboratório não
se analisa quanto tem de proteína, mas quanto tem de nitrogenio, e depois
se assume que todo o nitrogenio presente é proteico (o que não é verdade),
e que pra cada grama de Nitrogenio deve ter 6,25 gramas de proteínas, o que
é também uma aproximação grosseira.

e por aí vai...

L.E.

[As partes desta mensagem que não continham texto foram removidas]



SUBJECT: Re: [ciencialist] Re: Zodiaco
FROM: "Oraculo" <oraculo@atibaia.com.br>
TO: <ciencialist@yahoogrupos.com.br>
DATE: 05/02/2005 22:57

Olá Taborda

"Homero : Bem, e de onde saiu essa informação? O que garante sua validade?
Taborda: A matemática. "

Desculpe, mas a matematica nada garante, a não ser a precisão dos cálculos efetuados. Nada sobre interpretar esses dados ou sobre sua determinação de ser 12 ou 16. Como as diversas geometrias possiveis, todas bem precisas e exatas, com diversas quantidades de dimensões. Nenhuma delas garante ser a real apenas por ser matematicamente coerente ou precisa.

Podemos criar cálculos precisos que resultem em qualquer conjunto de resultados disponíveis, mas apenas a coerencia desses resultados não garante sua realidade.

Claro que sei que é possível calcular com precisão a posição astronomica de um objeto celeste. E claro que sei que a matemática envolvida na astrologia pode ser exata e precisa em seus calculos. O que não sei, e que não se explica, é porque esses calculos e não outros ou porque determiandos cálculos deveriam informar algo sobre mim, sobre o futuro ou sobre o mundo. Essa é a questão, não se é possivel fazer cálculos precisos com a matemática.

Homero:> Por que gomos?

Taborda: Aprenda geometria e coma mais laranjas

risos..:-) Essa foi boa, uma ironia e um jogo de palavras..:-) Mea culpa, vou reformular.:-) Por que gomos, no sentido de setores determinados, e não, digamos circulos? Sim, circulos deixam áreas de fora, mas e daí? Por que não deixar áreas de fora? Ou porque no sentido de rotação e não em outro, digamos, perpendicular a linha do zodiaco? Sim, sei que na linha fica mais legal, tem mais setores e constelações, mas, sendo uma escolha aleatória em sua base (ninguém diz como e porque os criadores escolheram, a não ser que os cálculos davam certo no final, embora existam muitos cálculos que dão certo e apenas um tenha sido escolhido), tanto faz como se dispõe sua ordem.

Por favor, não sou tolo a ponto de pensar que os criadores da astrologia apenas olharam para o céu, jogaram alguns dados e escolheram sua base. Mas o que estou dizendo é que essa escolha, seja como for, foi tão aleatória quanto isso. Basear-se em mitos, números mágicos, relações de medidas e constelações no céu é tão aleatório e sem embasamento quanto jogar dados.

"Taborda:Claro, as estrelas desaparcem assim tão rápido.... Uma cosntelação
inteira apareceu de repente... vc quer que eu acedite que no tempo dos
assirios, sumerios, gregos, arabes, etc... só existiam 12 cosntelações e
hoje existem 13 ???? É isso que vc quer que acreditemos ? Que uma
constelação nasce em menos de 5000 anos Isto contando sumerios, pq os
arabes foi à menos de 1000? Vc está dizendo que o ceu que os arabes viam
era diferente do que vemos hoje ? que a 13º cosntelação apareceu do
nada em menos de 1000 anos ?
O que vc diz ainda é mais ridiculo do que o os astrologos dizem."

Não, não é mais ridiculo (embora essa concordancia com o ridiculo do que os astrologos dizem me espante e alegre..:-), porque não uso essas afirmações para embasar nenhum conhecimento ou capacidade de ler informações sobre o universo e sobre seres humanos nelas..:-) Reconheço sua inadequação para qualquer propósito parecido e estou apenas apresentando possibilidades tão estapafurdias quanto as que embasam a astrologia.


"Taborda:Os astronomos não têm medo, eles têm terror. E são completos idiotas por
isso.

risos...:-) Acho que isso define a questão..:-) Todos os astronomos, do mundo inteiro, tem terror a astrologia, um medo patologico e apenas Taborda sabe a verdade. Ele apenas tem pena por não conseguir explicar algo tão evidente a todos os astronomos, e faze-los compreender. E dizem que os astronomos são arrogantes..:-)

Não posso, nem devo, discutir certezas tão certas e firmes. Conhecimento é duvida e você não tem nenhuma, o que impede qualquer argumento de ser eficaz ou mesmo analisado.

"Taborda: são idiotas, pq baseiam os seus contra-argumentos em crendices, mitos e
palermices, em vez de na astrologia em si mesma."

Bem, eles não tem escolha, já que baseiam seus argumentos nos astrológos que existem e no que eles dizem. Infelizmente, não conhecem o único astrólogo que tem a visão correta da astrologia, você Taborda. É uma pena..:-) Embora a maioria dos astrólogos dissidentes discordem de você e afirmem serem eles os detentores da única astrologia correta e real (o que significa que pensam que você está errado, esses tolos astrólogos).

Mais uma vez, cálculos corretos de matemática não validam a base da alegação astrológica, que podem interpretar esses dados de forma a ler informações sobre o universo. Sim, matemáticos podem estudar a astrologia e encontrar cálculos precisos nela, consitentes e coerentes. Mas isso nada muda nem prova nada. Prova apenas que astrologos podem fazer bons cálculos matematicos.

Taborda: Não sei como concegue dormir todas as noites com essa sua falta de
execicio do metodo cientifico, que vc presa tanto e apregoa a 7 ventos
que todos deveriam entender, até os leigos. Parece-me que vc não
entendeu ainda a primeira regra. Imparcialidade. Vc ja decidiu o
resultado antes de conduzir o estudo. ( eo mais divertido é que o fez
baseado em informações falsas.... lá se vai o metodo)

Fico comovido com sua preocupação com meu sono, mas garanto que é de boa qualidade e bem restaurador..:-) Além disso, minha imparcialidade está bem administrada, já admiti que, com alguma evidencia confiável, eu mudo de conclusão a respeito da astrologia, homeropatia e até acupuntura, até mesmo se elas não puderem indicar um mecanismo de ação conhecido.

Mas, minha imparcialidade não deve ser tão extrema que, sem nenhuma evidencia de nada, ainda assim eu mantenha a astrologia como possibilidade ou probabilidade alta ou mesmo média. Não é isso que imparcialidade significa.

Homero




----- Original Message -----
From: Sergio M. M. Taborda
To: ciencialist@yahoogrupos.com.br
Sent: Friday, February 04, 2005 11:55 PM
Subject: Re: [ciencialist] Re: Zodiaco


Oraculo wrote:

>
> "A amplitude dos signos pode ir alem da amplitude de 16 graus que
> define tradicionalmente a faixa zodiacal, definindo gomos que partem
> da Terra, tem por fronteiras a divisão em 12 do equador celeste e tem
> por fundo o infinito."
>
> Bem, e de onde saiu essa informação? O que garante sua validade?

A matemática. (Parece que vc a esqueceu toda) Em particular geometria.
Como se calcula a posição astrologia de um objecto ? Calcula-se a
posição que os astronomos chama de Ascenção Direita que é a coordenada
Teta no sistema esferico de referencia, ou seja, vai de 0 a 360 graus.
Divide-se essaposição por 30 , e obtemos o signo. O planeta está nesse
signo, e o resto da divisao será a posição dele nesse signo.
Ora, pq o signo vai além dos 16 graus ? Pq esses 16 graus dizem respeito
à coordenada fi do sistema esferico que so vai de -90 a 90 graus, e
sendo teta e fi, independentes , não importa qual o fi do objecto e
apenas o teta. Por isso , mesmo que o objecto estivesse fora da faixa
dos 16 graus , ele continuaria no mesmo signo. Por isso , se fala em
gomos, pois essa é a forma que vc obtem de um esfera dividida em partes
tomando teta como separador periodio.
Bolas , é geometria. É tão simples!

> Por que a amplitude é de 16 graus e não 20 ou 5 graus?

Pergunte as astronomos , foram eles que inventaram essa. As astrologos
interessa um pepino o valor da Declinação.

> Por que gomos?

Aprenda geometria e coma mais laranjas.

> Por que 12?

Aprenda a ler. São 12 os signos, lembra-se ?

> Sim, eram 12 na época dos criadores do zodiaco, mas, se estes tivessem
> criado sua explicação dos céus mil anos antes, talvez fossem 11 ou 13
> ou 15..:-) Teriamos hoje 15 "setores" e "gomos" e não apenas 12...:-)

Claro, as estrelas desaparcem assim tão rápido.... Uma cosntelação
inteira apareceu de repente... vc quer que eu acedite que no tempo dos
assirios, sumerios, gregos, arabes, etc... só existiam 12 cosntelações e
hoje existem 13 ???? É isso que vc quer que acreditemos ? Que uma
constelação nasce em menos de 5000 anos Isto contando sumerios, pq os
arabes foi à menos de 1000? Vc está dizendo que o ceu que os arabes viam
era diferente do que vemos hoje ? que a 13º cosntelação apareceu do
nada em menos de 1000 anos ?
O que vc diz ainda é mais ridiculo do que o os astrologos dizem.

>
> De toda forma, a explicação do astrologo não deveria produzir mais que
> uma única indagação: E daí?..:-) O que isso tudo importa, o que isso
> tudo informa sobre a validade de um conhecimento absolutamente sem
> evidencias como a astrologia?
>
> E a afirmação final de que astronomos tem medo da astrologia

Os astronomos não têm medo, eles têm terror. E são completos idiotas por
isso.
são idiotas, pq baseiam os seus contra-argumentos em crendices, mitos e
palermices, em vez de na astrologia em si mesma.
A primeira regra para a critica, é conhecer o objecto a criticar. E os
astronomos, e os ceticos em geral, nem isso conhecem.
Afinal é pedir de mais que se conheça matemática, astronomia, semantica
e que ainda se decorem uma serie de algoritmos e regras e em cima disso
um catalogo extenso de significados das inumeras combinações possiveis
de aspectos. É demasiado para a cabecinha dos astronomos de hoje (que só
sabem as duas primeira, e até duvido que saibam da primeira) e dos
cepticos, que afinal nunca dão provas de perceber realmente de coisa
nenhuma , pois o bom ceptico é aquele que duvida até das suas prorpias
crenças ,e se não as tem, sempre duvida se não as tem realmente.

Não sei como concegue dormir todas as noites com essa sua falta de
execicio do metodo cientifico, que vc presa tanto e apregoa a 7 ventos
que todos deveriam entender, até os leigos. Parece-me que vc não
entendeu ainda a primeira regra. Imparcialidade. Vc ja decidiu o
resultado antes de conduzir o estudo. ( eo mais divertido é que o fez
baseado em informações falsas.... lá se vai o metodo)

A astrologia pode ser toda mentira. Tal como a historia do StarWars ,
mas para poder falar sobre starWars é preciso conhecer a historia.
A historia pode ser ficiticia, mas seria um erro afirmar que o Luke se
casou com a Leia, que o Vader era o demonio, sei lá. Isso falta à
verdade da historia, mesmo sendo ela ficticia. Usar argumentos que se
baseiam na distorção das coisas não funciona, mesmo que a historia
subjacente seja ficticia.


Sérgio Taborda






--
No virus found in this outgoing message.
Checked by AVG Anti-Virus.
Version: 7.0.300 / Virus Database: 265.8.5 - Release Date: 03-02-2005



##### ##### #####

Para saber mais visite
http://www.ciencialist.hpg.ig.com.br


##### ##### ##### #####


Yahoo! Grupos, um serviço oferecido por:







------------------------------------------------------------------------------
Links do Yahoo! Grupos

a.. Para visitar o site do seu grupo na web, acesse:
http://br.groups.yahoo.com/group/ciencialist/

b.. Para sair deste grupo, envie um e-mail para:
ciencialist-unsubscribe@yahoogrupos.com.br

c.. O uso que você faz do Yahoo! Grupos está sujeito aos Termos do Serviço do Yahoo!.



[As partes desta mensagem que não continham texto foram removidas]



SUBJECT: Re: [ciencialist] Re: Zodiaco
FROM: "Oraculo" <oraculo@atibaia.com.br>
TO: <ciencialist@yahoogrupos.com.br>
DATE: 05/02/2005 23:11

Olá Taborda

Essa falácia é bem conhecida, e lida com os diversos sentidos do termo "ver", observar ou testemunhar. É claro que se eu, ou qualquer pessoa, só pudesse escolher uma explicação, se antes tivesse observado por si mesma e com seus próprios sentidos, ninguém escolheria nada nunca..:-) Ou, como no passado, escolheriam diversas respostas, nenhuma com grandes chances de ser correta ou ter efeito real.

As diversas explicações sobre as diversas alegações e fenomenos deste universo são numerosas o suficiente para garantir isso.

Justametne por isso, um conjunto de ferramentas e métodos foi sendo aperfeiçoado para permitir que, mesmo sem observação direta, mesmo sem ser um especialista em tudo, ainda assim fosse possível escolher com algum grau de confiabilidade. Para fenomenos cotidianos, ferramentas simples, para fenomenos mais importantes ou sérios, ferramentas mais complexas.

Se eu adoecer, não precisarei estudar medicina por 6 anos para entender o processo de minha doença e decidir que medicamento devo tomar. Um conjunto de pesquisadores e de estudos deve garantir que, mesmo que não seja 100% seguro ou confiável, chegue perto disso. Ou melhor, seja mais seguro e confiável que qualquer outro método de escolha. Se isso estiver correto, se for um método confiável, eu posso prever que a espectativa de vida da população que usar o sistema, vai aumentar de modo significativo. E essa é uma previsão que tem se mostrado correta, o que ajuda a validar a alegação..:-)

Posso aplicar essa análise a qualquer situação onde uma escolha seja necessária, onde decidir o que é real, e afeta o universo fisico, e o que não é, para que seja mais confiável e eficaz.

Portanto, posso pensar sim, sobre a astrologia, sem necessáriamente ser um especialista em cálculos astrologicos. Posso usar o atual conhecimento de fisica, astrofisica, biologia, psicologia, neurologia, processos mentais, etc, para pensar sobre as alegações da astrologia: que a análise de posições de planetas celestes, efetuada por cálculos matemáticos, pode reveler aspectos de minha personaldiade, minha vida, e meu futuro.

Se algo não é, como diz, observável, sua explicação sobre ele, ou qualquer outra explicação, não tem nenhuma evidencia de ser mais ou menos real. Se é observável, se, por exemplo, mapas astrais de diferentes astrologos podem resultar em dados confiáveis, se astrolgoos podem determinar a quem pertence cada mapa astral, usando a engenharia reversa dos calculos, então não seria necessária esta discussão.

Chamar céticos e cientistas de crentes invertidos é uma fuga padrão de todas as religiões (já ouvi isso de católicos, evengélicos, mórmons, defensores de ETs, do Pé Grande, e demais crenças..:-)

Homero



----- Original Message -----
From: Sergio M. M. Taborda
To: ciencialist@yahoogrupos.com.br
Sent: Friday, February 04, 2005 11:59 PM
Subject: Re: [ciencialist] Re: Zodiaco


Oraculo wrote:

>
> Nós (os malvados céticos..:-) apenas evitamos escolher uma explicação
> para um fenomeno sem antes pensar bastante sobre ele


Como vc pode pensar sobre algo que nunca viu ou testemunhou ? Fenomeno é
algo que se pode observar. Como vc pensa sobre algo que não observou
nunca ? e que até diz não ser observável.
Veja, o nivel do seu argumento é o mesmo de quem acredita em deus.
Também pensa nele, sem nunca o ter visto.
Os cepticos são bem mais crentes que os religiosos.

Sérgio



--
No virus found in this outgoing message.
Checked by AVG Anti-Virus.
Version: 7.0.300 / Virus Database: 265.8.5 - Release Date: 03-02-2005



##### ##### #####

Para saber mais visite
http://www.ciencialist.hpg.ig.com.br


##### ##### ##### #####


Yahoo! Grupos, um serviço oferecido por:
PUBLICIDADE




------------------------------------------------------------------------------
Links do Yahoo! Grupos

a.. Para visitar o site do seu grupo na web, acesse:
http://br.groups.yahoo.com/group/ciencialist/

b.. Para sair deste grupo, envie um e-mail para:
ciencialist-unsubscribe@yahoogrupos.com.br

c.. O uso que você faz do Yahoo! Grupos está sujeito aos Termos do Serviço do Yahoo!.



[As partes desta mensagem que não continham texto foram removidas]



SUBJECT: Re: [ciencialist] Re: Zodiaco
FROM: "Oraculo" <oraculo@atibaia.com.br>
TO: <ciencialist@yahoogrupos.com.br>
DATE: 05/02/2005 23:21

Olá Taborda

Bem, também fico com minha paciência em baixa, mas parece que tenho mais que você..:-) O que não chega a ser uma virtude, não?

> Hoje os setores sào o que importa, já que fica dificil manter a base
> original da astrologia, mas não foi isso que baseou sua criação.

Prove isso logo de uma vez e terminemos com esta conversa da treta, ou
cale-se para sempre. Ler erros cientificos uns atrás dos outros dá cabo
da minha paciencia.


Exato..:-) Não posso provar. Exatamente como você, não posso provar nada disso, são só palpites, que demonstram que se poderia ter uma astrologia diferente hoje, se as crenças e mitos da época fossem outros.

Poderia inclusive trocar o desafio, prove que os criadores da astrologia se basearam em dados confirmaveis ou verificaveis, que estudaram tanto os mapas astrais quanto as personalidades, psicologia e processos mentais de seus pacientes e que foi isso que redundou no conhecimento chamado de astrologia. Ou prove que ela funciona, demonstrando que, se receber mapas astrais de diversos astrologos, poderá determinar de que paciente se trata, usando apenas as datas e hora de nascimento.

Se puder, também esta conversa acaba ..:-) E ainda ganhará proeminencia mundial, desbancando os astronomos idiotas e medrosos, que tem terror da astrologia..:-)

Taborda: O estudo astrologico dada de à mais ou menos 5000 anos.

Bem, que dados temos sobre esse periodo de "mais ou menos 5000 anos", que permitem tanta certeza a você? Periodo de pouca informação, dificil de estudar, com um enorme espaço de tempo entre nós e eles (temos mais dados sobre épocas recentes, e mesmo assim pouco sabemos sobre sua cultura e procedimentos), como baseia sua fé na astrologia com tão pouco assim?

E, de novo, o que tem calcular órbitas, posições, precisão matematica, com conhecer a personalidade de uma pessoa??? Essa pergunta nunca é respondida!! Não importa como alega ser um leonino, como a posição de um planeta define essa personalidade????

Homero




----- Original Message -----
From: Sergio M. M. Taborda
To: ciencialist@yahoogrupos.com.br
Sent: Saturday, February 05, 2005 12:13 AM
Subject: Re: [ciencialist] Re: Zodiaco


Oraculo wrote:

> Olá
>
> Esse argumento faz sentido porque a astrologia FOI baseada nas
> constelações. Os que a criaram usaram as constelaçòes para basear sua
> criação. Nada sobre setores, inventados a posteriori para explicar
> incongruencias. Os criadores nào sabiam nada sobre precessão ou sobre
> estrelas como sois e o universo como setores. Sabiam empiricametne que
> as constelações se cucediam e isso permitia marcar o tempo e
> determinar estaçòes do ano.

A cada mensagem , vc demosntra mais que não sabe nada de fisica,
astronomia, ou sequer tem bom senso.
O estudo astrologico dada de à mais ou menos 5000 anos. Os quais, os
astrologos passaram compilando posições de todos os planetas visiveis.
Vc não acha que 5000 anos é suficiente para perceber a precessão ?
BOLAS!! A propria teoria das eras astrologicas é baseada nisso
Como eles poderiam basear a teoria das eras em algo que não sabiam ?

>
> Hoje os setores sào o que importa, já que fica dificil manter a base
> original da astrologia, mas não foi isso que baseou sua criação.

Prove isso logo de uma vez e terminemos com esta conversa da treta, ou
cale-se para sempre. Ler erros cientificos uns atrás dos outros dá cabo
da minha paciencia.


Sergio Taborda


--
No virus found in this outgoing message.
Checked by AVG Anti-Virus.
Version: 7.0.300 / Virus Database: 265.8.5 - Release Date: 03-02-2005



##### ##### #####

Para saber mais visite
http://www.ciencialist.hpg.ig.com.br


##### ##### ##### #####


Yahoo! Grupos, um serviço oferecido por:







------------------------------------------------------------------------------
Links do Yahoo! Grupos

a.. Para visitar o site do seu grupo na web, acesse:
http://br.groups.yahoo.com/group/ciencialist/

b.. Para sair deste grupo, envie um e-mail para:
ciencialist-unsubscribe@yahoogrupos.com.br

c.. O uso que você faz do Yahoo! Grupos está sujeito aos Termos do Serviço do Yahoo!.



[As partes desta mensagem que não continham texto foram removidas]



SUBJECT: Re: [ciencialist] Re: Zodiaco
FROM: "Oraculo" <oraculo@atibaia.com.br>
TO: <ciencialist@yahoogrupos.com.br>
DATE: 05/02/2005 23:28

Olá Taborda

Taborda: E esta
experiencia é baseada em informação reconhecidamente falsa, pelos ppr
astrologos."

Ora, o astrologo que cria os horoscopos, mesmo os de hornais, discordam veementemente de você. Segundo eles, são boas e confiáveis previsões e conselhos.

Certamente devem existir astrologos que discordam disso, mas como decidimos em quais confiar, quais dizem a verdade ou estão certos, se nenhum deles pode apresentar estudo confiãvel de eficácia?

Taborda:Que perda de tempo. Ha tantas coisas mais importantes para ensinar ..."

Concordo plenamente e assino em baixo..:-)

Taborda:Mas pronto, eu tb nunca achei que astronomos fossem cientistas. E quanto
mais o tempo passa, mas eu sei que tenho razão.

Acho que a opinião é reciproca, astronomos também não consideram astrologos cientistas. E, como os resultados de conhecimentos astronomicos são mais evidentes e confiáveis que os resultantes do conhecimento astrológico (satélites, robos em Marte, sondas em Titã, conhecimento sobre o universo, galaxias e estrelas), acho que vou me manter do lado dos astronomos até que alguma evidencia venha se colocar do lado da astrologia..:-)

Homero



----- Original Message -----
From: Sergio M. M. Taborda
To: ciencialist@yahoogrupos.com.br
Sent: Saturday, February 05, 2005 2:51 PM
Subject: Re: [ciencialist] Re: Zodiaco


Maria Natália wrote:

>
> Sérgio:
>
> Agradeço.
> A astrologia se usa aqui para fazer estatísitica a brincar. Pega-se
> num jornal e se vê qual o signo que diz " vais cair do escadote". Só o
> professor sabe qual o jornal escolhido e todos respondem ao
> questionário. Depois os que eram do signo da previsão "má" sofrem
> comparação.Mas todos preenchem o inquérito. As perguntas também
> permitem tirar conclusões sobre supertições, crenças e mitos. Quanto
> mais turmas mais científico será (amostra maior)

Mas que palhaçada é esta ? É assim que se ensina ciencia ? Estou
escandalizado que o dinheiro dos contribuintes seja gasto com esta
palhaçada.
Palhaçada pq não ha outro nome, é um circo de falta de cientificidade. E
depois ainda tem coragem de dizer que aumentando a amostragem aumenta a
cientificidade !!
Estou enojado e revoltado com isto que contou.
Primeiro, a sua experiencia apenas pode demonstrar que a informação do
jornal é errada. Mas o METODO o famoso, querido, acompanhado, idolatravo
METODO CIENTIFICO MANDA, ele não pede, ele MANDA que a toda a informação
do fenomeno seja obtida de forma imparcial e, independente. E esta
experiencia é baseada em informação reconhecidamente falsa, pelos ppr
astrologos.
Esta experiencia não tem qq valor cientifico, pq viola a primeira rega
de um estudo cientifica, a obtenção de dados de forma imparcial e
independente.
Vc verificou quem escreveu o enunciado do jornal ? vc verificou com
outras informações providas por outras fontes ? O cruzamento de
informação é a primeira coisa a fazer. Pq vc não ensina a fazer isso
primeiro ?
Só depois vc pode escrever questionarios e fazer experiencias. Qualquer
resultado que apareça de uma experiencia assim é lixo.
É por estas e por outras que alguem canta que descobriu a fusão a frio e
depois é mentira. Ninguem mais está preparado para destingir ciencia de
palhaçada. E a culpa é das escolas e professores que passam o tempo no
circo.

> Quando os alunos fazem a "fita" do Zodíaco na parede da sala de aula,
> cúbica ou paralelipipédica,

Que perda de tempo. Ha tantas coisas mais importantes para ensinar ...

> se representam todas para se perceber essa
> do "sol está na constelação..."

O sol não pode estar numa constelação. Onde vc ouviu tamanha idiotice ?
(os astrologos dizem "sol no signo" , ou "sol na casa" ,nunca na
cosntelação)

>
> Não gostamos quando, a meio da noite, nos aparecem adultos a perguntar
> se pelo facto de Vénus estar em Virgem eles vão ter mais sorte no
> Totoloto.

A culpa é toda vossa porque cultivam a mistica da astrologia sem
entender a astrologia de facto.

> Daí as alergias com a astrologia.

Quem tem alergia fica longe. Mas os astrónomos ADORAM começar as suas
palestras com exemplos de astrologia, que eles não entendem.
Argumentos idiotas como o de Ofucio é de uma imbecilidade tão grande de
retira qq respeito cientifico pelo astronomo.
Cientistas verdadeiros não fazem conversa com o que não conhecem. É
baixo , muito baixo.
Mas pronto, eu tb nunca achei que astronomos fossem cientistas. E quanto
mais o tempo passa, mas eu sei que tenho razão.

> E este ano tive até um pai engenheiro a perguntar-me se achava que o
> professor X iria faltar muito...O home estava a confundir astrónomo
> com astrólogo.

Não seria com X-Men ? O que tem o professor faltar muito com astrologia ?
Se o professor é um bundão que falta muito - o que é normal, eu tive
muitos desses - é normal que o pai se preocupe.
Eu me preocuparia.

> Mas a astrologia dá para muita brincadeira.

Exactamente pq estão todos num circo, em vez de numa aula de ciencias.
Seria como fazer paidas sobre sexo, para ensinar o sistema de reprodução
humano.

> É que este pessoal que nos
> chega à mão aos 11 anos já tem pelo menos 3 anos de laboratórios e
> trabalhos de campo. Vem com a mania de apanhar a alma na ponta de um
> bisturi. Vamos ver o que esta geração dará na universidade dentro de 4
> anos.

Dará merda, como todas as que vem chegando lá. E o ensino seundário está
cada vez pior e a culpa
é dos professores e as suas estratégias pedagogicas ridiculas.

Sérgio Taborda



--
No virus found in this outgoing message.
Checked by AVG Anti-Virus.
Version: 7.0.300 / Virus Database: 265.8.5 - Release Date: 03-02-2005



##### ##### #####

Para saber mais visite
http://www.ciencialist.hpg.ig.com.br


##### ##### ##### #####


Yahoo! Grupos, um serviço oferecido por:







------------------------------------------------------------------------------
Links do Yahoo! Grupos

a.. Para visitar o site do seu grupo na web, acesse:
http://br.groups.yahoo.com/group/ciencialist/

b.. Para sair deste grupo, envie um e-mail para:
ciencialist-unsubscribe@yahoogrupos.com.br

c.. O uso que você faz do Yahoo! Grupos está sujeito aos Termos do Serviço do Yahoo!.



[As partes desta mensagem que não continham texto foram removidas]



SUBJECT: Re: [ciencialist] Re: Zodiaco
FROM: "Sergio M. M. Taborda" <sergiotaborda@terra.com.br>
TO: ciencialist@yahoogrupos.com.br
DATE: 05/02/2005 23:46

Oraculo wrote:

> Olá Taborda
>
> Sua resposta resvala para coisas do tipo "tudo é tudo, nada é nada,
> mas as vezes, nada é tudo e tudo é nada..." Bom para Paulo Coelho,
> mas irrelevante para a discussão ou eficácia da astrologia.

Não entendi o que vc quiz dizer com isto.

>
> Calculos matemáticos complexos nada significam sozinhos.

Complexos ? 365 / 29 é complexo ?

> Se toda qualidade é ao mesmo tempo boa e má, se toda pessoa pode ser
> uma hora uma coisa e outra outra coisa,

Viu ? Isto é tipico da argumentação não cientifica. Pegam num argumento
, extrapolam-o além do seu significado, e reaplicam-o como se fosse
válido. Vc está pensando da mesma forma que pensa um crente num deus qq.
Auto-convencendo-se de que os argumentos que ele cria, são os desejos
desse deus
É verdade q todos os atributos são bons e maus. Mas ninguem disse que a
pessoa é uma hora uma coisa e outra ora outra. Isso vc fabricou na sua
mente como um sinonimo , mas que não é.
Ai vc chega num resultado logico que vc acha que contraria o argumento
inciial, mas não o faz pq uma das permissas é inválida.

> então, para que sereve a astrologia???

Não estamos discutindo para que serve, e sim, o que é. Em que bases
teoricas se apoia, q filosofia segue e o que tem a ver com astronomia.

> Se sua resposta fizesse sentido, um mapa astral feito para mim,
> serviria para qualquer pessoa.

Não. Se os SEUS argumentos fizessem sentido, VC poderia até chegar nessa
conclusão. Mas essa seria a SUA conclusão baseada nos SEUS argumentos,
que não são os meus , nem os de ninguem mais. E portanto, não são os dos
astrologos, e portanto qq conclusão a que VC chegue não tem nada a ver
com astrologia.

> Desmontando os argumetnos do Takata? Nem de longe..:-)

Eu já conheci várias pessoas como vc. Vcs tem um problema grave de
leitura. Ou não entendem o que leem , ou o distorcem. É por isso que vc
não entendeu a desmontagem e não entende nada do que eu escrevo, em geral.

>
> Se o leonino é as vezes autoritario e as vezes não é, (..)


SE , a palavra inportante dessa frase é SE. SE fosse .Mas não é. Então
tudo o que vc diz a seguir não se aplica.

>
> Pessoas fazem mapas astrais para saber o que as diferencia de outras
> pessoas, o que as torna únicas, baseadas na data e hora em que nasceram.

Vc pode especular a razão à sua vontade. A questão aqui não é quem se
vai consultar com o astrologo, ou o astrologo, mas a astrologia. A
astrologia é uma teoria, como outra qualquer.
E o que está acontecendo aqui é que se está julgando a teoria errada.
Vcs continuam, e insistem , em falar sobre algo que não existe, sobre
uma astrologia que vcs proprios criaram que nada tem a ver com a
verdadeira astrologia. E por isso que eu não me canço de replicar. Pois
considero que a vossa acção não é cientifica e é hipocrita, já que por
um lado defendem a ciencia a todo o custo, e por outro não sabem fazer
ciencia. Vc acusa quem consulta o astrologo, por ser enganado por um
processo qualquer de leitura fria. Mas vc mesmo comete os mesmos erros.
Vc quer acreditar demais que a astrologia não funciona, sem nunca ter
experimentado, sem sequer saber o que é , no que se baseia, q regras
segue, tanto quando vc acha que o consulente quer acreditar no astrologo.
Ao contrario das artes adiinhatorias a astrologia não funciona por
leitura fria, pois o contacto entre o astrologo e o consulente não é
requerido.
A historia de Fernando Pessoa, o poeta portugues ilustra bem isto. Ele
era astrologo e fazia um mapa astral para cada um dos seus heterónimos.
Era com isso que ele conseguia manter a personalidade de cada um
independente da dos outro. Mas , essas pessoas não existem .
Outro ponto é que a astrologia vai muito alem das pessoas. Ela pode ser
usada para estudar paises, ou todo o planeta com um todo. Como se
aplicar leitura fria a um planeta ? ou a um pais ?
São estes promenores que escapam - tão convinientemente - das vosses
considerações.

> Isso é tolice. Mas poderia ser tolice e ainda assim ser real, mas
> também não há evidencias disso. Mapas astrais de excelentes astrologos
> são conflitantes entre sí. Sim, são as variaveis que são muitas. Mas,
> qualquer coisa em que as variaveis sejam tantas que as respostas nunca
> coincidam, são inúteis por defrinição.

Exacto, tal como a Metereologia. Mas mesmo assim vc continua lendo a
previsão do tempo, mesmo assim essa informação é oficial, A metereologia
tem muitas mais variáveis que a astrologia, mas vc a aceita. Tlv vc
pense que ela é baseada na ciencia. Tlv, mas suas previsões são tão boas
quanto olhar o ceu todos os dias.

>
> O cara demorou para decidir? Então, o mapa acertou, afirmou ser
> indeciso. Mas, se o mapa diz o contrário, que ele é ponderado, também
> acertou..:-) Hilário.
>
> Esse o argumento bem defendido pelo Takata e que você contornou com
> jogos de palavras.

Não, não foi esse o argumento em discussão com o Takata. Vc está
distorcendo o que foi escrito. Mas pode sempre citar aqui a frase que
corraborar o que vc está dizendo.
A discussão era sobre a relação entre as constelações, seus nomes e
significados e os atributos que são de cada um dos signos.

> Não importa em nada o que o mapa afirme, já que qualquer
> caracteristica descrita pode ser aplicada a qualquer comportamento ou
> justificar qualquer coincidencia com o paciente.

Não, não pode. Mas como vc acredita piamente nisso, o que eu posso fazer
contra a sua crença ? Vc acha que não, quem sou eu para lhe ensinar que
não é assim.
O seu problema aqui, é que nenhum de vcs se debruçou o minimo sobre o
assunto, para chamar astrologia, ao mesmo que os astrologos chamam
astrologia. Vcs chamam astrologia a ideias que vcs pegaram e juntaram e
nunca consultar os reais manuais de astrologia.
O Takata citou o manual, e como pudemos ver, o atributo ferocidade não é
atribuido a leão. É tão simples como isso. Mas, claro, vc sempre vê
mais coisas das que realmente estão lá.

>
> Marte ser vermelho torna as pessoas de um jeito e não de outro??? Se o
> ferro no planeta não existisse, regidos por Marte teriam outra
> personalidade?? Fala sério..:-)

Fala sério vc. Essa questão é mais uma palhaçada, pq nunca ninguem disse
que o vermelho de marte torna o que quer que seja , no que quer que seja.

>
> Tem razão em uma coisa, as regras não foram criadas agora. Foram
> criadas a 3000 anos por pessoas que nada sabiam sobre fisica, leis da
> gravitação, planetas, e muitas outras coisas (inclusive, sobre
> psicologia e processos mentais neurologicos).

Isso é o que vc acha. Mas vc pode provar isso ? Vc pode provar que eles
não sabiam ? Claro que não pode.
Os planetas são observáveis a olho nu, nas circunstancias metereologicas
convinientes. Não é dificil contar seus periodos, e criar mapas com
isso. A astrologia não precisa da fisica, nem da gravitação, nem de
coisa nenhuma , alem de pontos luminosos que se movem no ceu com
periodos regulares. Tudo o resto é matemática e intrepretação usando
regras defenidas à priori.
Tlv eles não soubesses nada sobre esses assuntos, mas não repcisavam. E
a prova é que vários povos, em vários pontos do mundo faziam a mesma
coisa. Desde os maias aos chineses, passando pelos egipsios , gregos ,
sumerios, etc...

> São regras arbitrárias, destinadas a dar ordem a uma aparente desordem
> no mundo.

As regras do futebol também são arbitrárias, mas toda a gente as pode
conhecer e usar a qq momento.
O problema aqui, é exactamente esse, vc não sabe quais são as regras e
já quer depreciar o jogo.
Eu apenas lhe estou ensinando as regras verdadeiras do jogo.

> Nada indica que qualquer relação real entre as posições dos planetas
> (sejam eles o Sol a Lua e a Terra ou o conjunto dos demais), setores
> do zodiaco, ou conjunto de estrelas (que os criadores não sabiam o que
> eram) influencie a personalidade de qualquer pessoa.

Vc diz isso pq já fez estudos precisos sobre o asumo, presumo.

>
> Acreditar nisso é apenas questão de fé (e muita fé), não uma conclusão
> baseada em evidencias.

E precisa ser ? Só para si, e outros como vc.
Astrologia não é ciencia, não precisa ser baseada em evidencias.

>
> Você parece, as vezes, dizer que a astrologia é como uma forma de
> psicologia, para a auto-compreensão e que por isso independe de dados
> e ligações fisicas reais. Mas é só você, talvez o único astrologo do
> mundo, que diz algo assim. E mesmo como auto-analise, auto-descoberta,
> a astrologia tem pouco a apresentar como evidencia de eficácia.

1) Eu não sou astrologo
2)Não sou o unico que afirma isso.
3) Astrologia não é uma forma de psicologia, pois isso implica não pode
ser auto-aplicada, e so pode ser aplica a pessoas. A astrologia pode ser
auto-aplicada. Além disso a astrologia pode ser aplicada a qq evento, e
não apenas à vida das pessoas.
4) Vc acha que não tem nada a oferecer, pq vc nunca a estudou. Não
conheço ninguem que a tenha estudado e tenha saido pensando o mesmo que
pensava antes.
5) Muitas coisas na vida dos seres humanos não apresentam eficácia e
mesmo asssim as fazemos. Isso, não é argumento. Como por exemplo, eu
tentar fazer-vos entender o vosso erro cientifico e falta de rigor em
seguir o metodo cientifico no que toca às vossas opiniões em relação à
astrologia.. entre outras coisas...

Sergio Taborda




--
No virus found in this outgoing message.
Checked by AVG Anti-Virus.
Version: 7.0.300 / Virus Database: 265.8.5 - Release Date: 03-02-2005



SUBJECT: Re: Zodiaco
FROM: "rmtakata" <rmtakata@altavista.net>
TO: ciencialist@yahoogrupos.com.br
DATE: 05/02/2005 23:52


--- Em ciencialist@yahoogrupos.com.br, "Sergio M. M. Taborda"
> N\ao me fac,a rir com esses tipo de argumentos. Sejamos
> serios.

Estou sendo tao serio qto o tema: astromancia, permite ser.

> Vc na~o disse ferocidade, pq ?

Eu falei algo feroz. Deve conhecer o sentido da palavra 'algo'. Uma de
suas acepcoes - como no caso usado - eh 'um pouco', 'de um certo
modo'. Eu nao disse 'feroz' simplesmente. Usei a expressao com a
devida modificacao.

Nao obstante isso, alguns manuais falam sim de ferocidade - "Co'lera -
O Lea~o, no auge de sua raiva, deixa-se vencer por exploso~es de
temperamento inimagina'veis."
http://www.kairell.donagh.nom.br/zodiaco.htm#leo
(Certo, nao eh um manual, mas uma demonstracao de como isso estah sim
presente em pelo menos alguns seguidores da astromancia.)

> Orgulho, autoritarismo e megalomania nada te^m a ver com
> ferocidade. Sa~o tudo coisas do corac,a~o.

Ferocidade tb sao coisas do 'coracao' - isto eh, eh um aspecto
emocional negativo exacerbado.

> (A palavra Aspecto tem um significado tecnico em astrologia,
> por isso que mudei para atributo)

Ok.

> Isso e' o que vc diz. Em que se baseia para afirmar isso.

Eu me baseio no caso dos planetas descobertos tardiamente. Na epoca da
edificacao da astromancia apenas sete 'planetas' (q. incluia a Lua e o
Sol) eram conhecidos - Marte, Venus, Jupiter, Mercurio, Saturno (q.
foram usados para batizar os dias da semana) - exceto a Terra. Como
dito, qdo outros planetas foram descobertos, os nomes atribuidos aos
novos planetas - Netuno, Urano e Plutao - influenciou as
caracteristicas (ou atributos) q supostamente regeriam.

> Como vc ainda nao entendeu que uma so coisa pode ser boa e
> ma', vc acha contradittorio.

Nao eh o fato de ser bom e mau, mas o fato de ser por exemplo:
generoso x egoista; ativo x passivo; ligado x desencanado e assim por
diante.

> O facto e' que eles sa~o as duas coisas. Em algumas areas
> sera~o equilibrados e outras indecisos.

Bem, de todo modo emulando (supostamente) uma balanca.

> That's the point! O ponto e' esse mesmo. Elas na~o o sa~o, mas
> assim mesmo a indu atriui simbolos diferentes aos mesmos
> sigificados. O significa que o significando vem
> antes, e o simbolo e' apenas uma menmonica.

Not so. Houve uma mistura, mais ou menos como a influencia da
mitologia grega sobre a romana - eles eram parecidos por serem criadas
da mesma fonte, mas adaptacoes foram feitas: assim Marte,
originalmente um deus da agricultura, ao ser identificado com Ares,
passou a encarnar tb - e principalmente - o guerreiro. O significado
nao veio antes, mas sim foi adaptado com a mistura. Coisa similar
ocorreu com o intercambio entre as astromancias hindu e ocidental.

> Obrigado. E' exactamente isso. Enta~o como vc esplica que os
> simbolos sejam outros ?

Outros, mas nao sao fundamentalmente diferentes. Um exemplo de
astromancia mais diferente, com outros simbolos e outros significados
eh a astromancia chinesa.

> Se a sua teoria fosse verdaeira , os simbolso seriam os
> mesmos pois as estrelas sa~o as mesmas.

Isso apenas se nao houvesse intercambio significativo entre os
sistemas astromanticos. Acima dei o exemplo da transformacao de Marte
em direcao a Ares - e Marte cada vez mais adquirindo o formato de
Ares. Porem, Marte ainda carrega sua historia pregressa como
agricultor (como Roma passou de uma sociedade agricola para um Imperio
guerreiro).

Simbolos nao sao estaticos. (Para outro exemplo, veja a origem dos
algarismo indoarabicos.)

> Como se a sua fosse ... lol.. estou so demontanto seus
> argumentos (hei, estou dizendo q vc tem argumentos!...).

Pois eh, estah.

> Tudo bem. Enta~o pq os outros planetas na~o sa~o referidos como
> o planeta <cor>?

Porq. nao hah necessidade de sistematizacao. Assim como
definitivamente um Pereira deve ter tido um ancestral remoto q. ou
cultivava um pomar de peras ou vivia em uma vila em q. havia muitas
peras ou algo assim eqto um Gago nao precisa ter tido um ancestral q.
cultivava plantas com problema de diccao. Ou nem todos os elementos
quimicos homenageiam seus descobridores ou um cientista famoso, por
mais q Curio e Polonio sejam homenagens a Marie e Pierre Curie. Ou
cloro tenha seu nome baseado na cor, mas helio, nao.

Mas todos apresentam uma inspiracao em uma caracteristica divina.
Mercurio com seu ciclo ligeiro parecia o mensageiro dos deuses.
Jupiter com seu andar lento e majestoso, o principal deus do panteao.
Outros podem ter sido batizados de um modo um pouco mais arbitrarios
ainda.

> Depende. Outro nome para Venus e' Luficer. Que todos sabem
> com o que se relaciona.
> (Sabemos ? Quando leu lucifer ter ter pensando no demonio ,

Eu nao. Jah havia lido sobre isso. Deve ter notado q. gosto de ler
sobre a origem dos nomes de certas coisas.

> Voce^ parece estar muito convencido disso, ignorando toda
> amatema'tica envolvida na astrologia, a

A matematica util envolvida na astromancia resume-se 'a previsao da
posicao dos planetas e outros corpos. Essa parte util encontra-se
incorporada na astronomia.

Boa parte dela, porem, foi desenvolvida depois - algumas motivada por
questoes astromanticas.

> E' como vc negar a mecanica quantica pq na~o gosta do nome
> quantum.

Ueh, quem disse q. eu nao gosto dos nomes? Estou tao somente dizendo
q. os atributos conferidos aos segmentos astromanticos celestes foram
influenciados fortemente pelos nomes dados a esses setores. E os nomes
dados a esses setores foram determinados pelas formas com q. estrelas
foram imaginativamente ligadas entre si.

[]s,

Roberto Takata





SUBJECT: Re: [ciencialist] Questões biológicas
FROM: "Oraculo" <oraculo@atibaia.com.br>
TO: <ciencialist@yahoogrupos.com.br>
DATE: 05/02/2005 23:53

Olá Antonio

"3 - sabe-se que a raça influencia fortemente no comportamento dos cães.
Assim, por exemplo, um cão labrador irá mergulhar espontâneamente numa
piscina, enquanto que cães de algumas outras raças sentirão até medo de se
aproximarem da água. Além de sermos em parte um produto do meio e termos o
nosso comportamento controlado pela razão, existe essa influência da raça
sobre o nosso comportamento (dos seres humanos)?"

Se trocar raça por genes, a resposta parece ser positiva. Embora tenha se acreditado (muito por preconceito humano a respeito de sí mesmo) que seres humanos tinhaa pouquíssimos instintos (como sugar leite, chorar de fome e prender os dedos em qualquer coisa quando bebês, por exemplo), novos estudos e teorias sugerem que a natureza humana tem muito de geneticamente influenciada. Ainda que, claro, isso não descarte de forma alguma a influencia da cultura e da sociedade.

Um livro muito bom sobre o assunto (e bastante polemico, claro..:-) é Tabula Rasa, de Pinker. Eu coloquei um capitulo digitalizado na rede, se quiser dar uma olhada, acho que pode interessar:

http://www33.brinkster.com/fatorx/tabula rasa.doc

O link tem um espaço entre tabula e rasa, talvez tenha de colar o endereço acima para baixar o arquivo.


Um abraço.

Homero


----- Original Message -----
From: Antonio Ferrão Neto
To: ciencialist@yahoogrupos.com.br
Sent: Saturday, February 05, 2005 9:24 PM
Subject: [ciencialist] Questões biológicas


Caros colegas da lista...

Eu gostaria de saber dos biólogos, nutricionistas e agrônomos:
1 - como são determinadas as calorias dos alimentos, como se reduz o teor
calórico deles, como se reduz o açúcar deles e também as gorduras (saturadas
e totais).
2 - como são produzidas as sementes das frutas que não produzem sementes?
3 - sabe-se que a raça influencia fortemente no comportamento dos cães.
Assim, por exemplo, um cão labrador irá mergulhar espontâneamente numa
piscina, enquanto que cães de algumas outras raças sentirão até medo de se
aproximarem da água. Além de sermos em parte um produto do meio e termos o
nosso comportamento controlado pela razão, existe essa influência da raça
sobre o nosso comportamento (dos seres humanos)?

[]'s
Antonio Ferrão Neto
--
No virus found in this outgoing message.
Checked by AVG Anti-Virus.
Version: 7.0.300 / Virus Database: 265.8.5 - Release Date: 03/02/2005



##### ##### #####

Para saber mais visite
http://www.ciencialist.hpg.ig.com.br


##### ##### ##### #####


Yahoo! Grupos, um serviço oferecido por:

São Paulo Rio de Janeiro Curitiba Porto Alegre Belo Horizonte Brasília




------------------------------------------------------------------------------
Links do Yahoo! Grupos

a.. Para visitar o site do seu grupo na web, acesse:
http://br.groups.yahoo.com/group/ciencialist/

b.. Para sair deste grupo, envie um e-mail para:
ciencialist-unsubscribe@yahoogrupos.com.br

c.. O uso que você faz do Yahoo! Grupos está sujeito aos Termos do Serviço do Yahoo!.



[As partes desta mensagem que não continham texto foram removidas]



SUBJECT: Re: [ciencialist] Re: Zodiaco e Acupuntura e Homeopatia..:-)
FROM: JVictor <jvoneto@uol.com.br>
TO: ciencialist@yahoogrupos.com.br
DATE: 05/02/2005 23:57

Para Murilo,

>
>
>
> Murilo:> Perdão pela comparação com o Quevedo! Achei que vc se esforça
> demais, faz
> > muita ginástica, para negar, destruir e ridicularizar trecos que não
> > deveriam ter importância para você e que estão fora do seu mundo.
> > Mas fica tranquilo, eu entendo, sei que vc gosta de provocar,
> polemizar e
> > argumentar.


Victor: Não vejo nos textos de Homero qualquer " ginástica, para negar,
destruir e ridicularizar trecos que não
deveriam ter importância para você e que estão fora do seu mundo".
O que ele efetivamente faz, e o faz de maneira séria e competente, é
argumentar à luz do conhecimento científico, apontando implicita e
lucidamente que coincidências como as que são descritas na astrologia
não correspondem a leis da natureza, como as que que são descritas na
física, por exemplo. Defender que tenho tais e tais características, que
sou assim, em vista de haver nascido em tal dia, a tal hora e no ano
tal, é, no mínimo, ter muita coragem ou muita convicção. De quê, eu não sei!
Aproveitarei o ensejo para falar sobre as razões que me levam a não dar
crédito a coisas do tipo. Não quero entrar no mérito daqueles que pensam
diferente. Trata-se de minha opinião pessoal.
Todas as estruturas místicas e/ou religiosas e filosóficas foram
criadas, ao longo da evolução humana, em vista de duas coisas:
ignorância e medo da morte. Medo do desconhecido, enfim. Nem mais nem
menos que isso. Mostrem-me evidências experimentais(ou uma estrutura
matemática coerente), de que eu sou assim e assado em razão de haver
nascido especificamente em tal dia, tal hora, tal mês e ano e que os
milhôes de pessoas que nasceram no mesmo instante que eu tenham as
mesmas características comportamentais, perante si e a sociedade.
Conversei com astrólogos que se dizem aptos a elaborarem mapas astrais,
relatando, inclusive, eventos passados, presentes e, pasmem, futuros! É
a ignorância e o medo da morte(esta mais que aquela) que acirra e
dissemina as crendices, não só em astrologia como em espiritismo, em
todas as suas variantes. Muitos médiuns e/ou videntes dão a receita de
como ser feliz, de como arranjar marido, de como enriquecer, de como
ganhar na loteria. No que muita gente acredita, o que faz os tais
ganharem notoriedade e dinheiro, em geral; havendo exceções, claro. No
caso da astrologia, evidentemente há casos incríveis, de estranhas
coincidências. Mas isto funciona para todos, com aquela "sina" de ter
sua existência associada aos astros? Para as coincidências, há
explicações naturais. O desconhecimento leva a crer que os astros, as
constelações, que só têm aquela ou esta forma em vista de nossa posição,
como observadores, sejam responsáveis pelas tais coincidências. Outros
grupos associam a isso o karma individual e/ou coletivo(já há, por
exemplo, estudiosos da área buscando explicações, nessa linha, para a
ocorrência do tusnami!) Afinal, espíritos e coisas espirituais, só
aparecem e/ou acontecem para quem acredita neles. Assim, também, assumir
um conhecimento ancestral tido como verdade, como é o caso da
astrologia, e incorporá-lo como segunda realidade(por processos
jungianos, como insconsciente coletivo, arquétipos) faz a mente
raciocinar em tais termos e engendrar os padrões associados. É assim que
funciona. É a mente a responsável. Não o pé ou Marte, por girar e rodar,
e onde esteve ou está! É ela, a mente, que cria " tais realidades".
Deveria ser nela 95% das investigações cienbtíficase investimentos
governamentais e particulares. É a única coisa que temos e que somos. O
resto é acessório, que pode funcionar ou não!.
Aliás, que os experts discorram mais apropiadamente sobre o seguinte: as
predições inerentes as esses ramos do "conhecimento" levam as paradoxos
como: existirá destino? E o livre arbítrio? Essas coisinhas que
espíritas, teólogos, e outros estudiosos, não conseguem explicar e ficam
dando voltas em círculos. Mas não reconhecem isso, o que é mais
estranho. E compreensível, também...
Devo deixar claro, também, que a ignorância a que me refiro não tem
significado pejorativo, que ninguém se ofenda. No sentido em que
emprego, tem a haver com estruturas de pensamento, a quadros de
referência, com é fácil de inferir. Não está relacionado, portanto, o
termo, ao fato de A ou B ser ou culto ou analfabeto; formação cultural,
religiosa, costumes da sociedade, preconceitos adquiridos em vista disso
tudo, ao longo de seu crescimento, entre outros atributos adquiridos,
respondem pela estrutura de pensamento acima referida. Como na
religião, onde cada facção tem razão e a outra não, fica patente também
que, para os astrólogos ou outros grupos, por igual forma, a maneira
como eu penso seja fruto de ignorância. Que não é desconhecimento
(total) do assunto astrologia, nem de outras coisas, aviso. Sei disso,
pois já estive lá, já cultivei, num passado mais ou menos remoto,
coisas do tipo, com a razão falando mais alto. Neste "meu mundo", como
no de Homero, as coisas são mais pé no chão. Mais simples e rasteira.
Que os astrólogos, espíritas, místicos, religiosos em geral, sigam seu
caminho em paz, com suas crenças e convicções. E que todos respeitem
essa ou aquela tendência, ainda que não concorde.
Sim, Sr. Lavoisier!

Sds,


Victor.






> > >From: "Oraculo" <oraculo@a...>
> > >Reply-To: ciencialist@yahoogrupos.com.br
> > >To: <ciencialist@yahoogrupos.com.br>
> > >Subject: [ciencialist] Zodiaco e Acupuntura e Homeopatia..:-)
> > >Date: Sat, 5 Feb 2005 19:39:24 -0200
> > >
> > >Olá Murilo
> > >
> > >risos..:-) Bem, mesmo me chamando de padre Quevedo (o que me deixou
> > >assustado..:-), não refutou meus argumentos sobre a acupuntura..:-)
> > >Considero isso uma boa coisa, já que estes parecem ser eficazes em
> > >convence-lo (ou pelo menos a fazer você pensar sobre eles..:-)
> > >
> > >Mas a homeopatia segue o mesmo raciocinio. Ela tem definição
> precisa, ou
> > >deixa de ser a homeopatia. Veja, seu criador, Hahnemann, nada
> sabia sobre
> > >micro-organimos, infecção, sistema imunologico, etc. Sua criação
> curava
> > >doenças a partir de determinados principios. Ou se aceita os
> principios, ou
> > >se recusa o termo homeopatia. Eu acho que foi um avanço na época,
> já que a
> > >medicina usava sanguessugas, purgantes e coisas assim para tratar.
> Parar
> > >com esses "tratamentos" e esperar a doença se curar em geral era mais
> > >benéfico ao paciente..:-)
> > >
> > >Ele dizia, por exemplo, que doenças cronicas eram causadas por um
> tipo de
> > >espírito maligno, ou misma, a psora (como em psoriase). Devo levar
> essa
> > >afirmação também, junto ao resto das alegações da homeopatia? Ou posso
> > >filtrar, com alguma ferramenta racional, o que é correto de que é
> > >incorreto? E, se posso fazer isso, que ferramenta deve usar? E o
> que fazer
> > >se a ferramenta derrubar todas as afirmações, e não apenas as mais
> > >estranhas?
> > >
> > >Influenciar animais é possivel. Na verdade, para a maior parte dos
> > >mamiferos (principalmente os animais gregarios) a atençao e o cuidado
> > >pessoal traz benficios no bem estar. São animais, mas não são
> estupidos ou
> > >desprovidos de mente ou de processos mentais (filhotes de macacos
> > >abandonados pelas mães, preferem mães substitutas feitas de pelo e
> algodão,
> > >mas sem alimentos, do que mães substitutas feitas de arame, mas que
> > >poderiam alimenta-los). Tenho um amigo que, depois de tentar
> bastante o
> > >tratamento de seus cavalos e gado com homeopatia (principalmente
> para a
> > >perigosa constipação) desistiu por absoluta falta de efeitos. Esse
> relato,
> > >unico e anedotico, valida a tese que a homeopatia não funciona?
> Claro que
> > >não..:-) Tanto quanto um relato contrário, de efeito positivo, não
> valida a
> > >afirmação de que a homeopatia é real.
> > >
> > >Apenas façam com que a homeopatia passe em estudos controlados, e
> minha
> > >conclusão sobre sua eficácia muda..:-)
> > >
> > >Mas, enquanto ela falhar, e enquanto seu mecanismo de ação for
> inexistente
> > >(ou francamente incorreto), fico com a conclusão cientifica de que
> é um
> > >placebo, na melhor das hipoteses..:-)
> > >
> > >Um abraço
> > >
> > >Homero
> > > ----- Original Message -----
> > > From: murilo filo
> > > To: ciencialist@yahoogrupos.com.br
> > > Sent: Friday, February 04, 2005 11:06 PM
> > > Subject: RE: [ciencialist] Zodiaco e Acupuntura..:-)
> > >
> > >
> > > Oraculo, oi.
> > > Vc está parecendo o Padre Quevedo fazendo ginástica anti-espírito!
> > > Conseguí achar um negócio interessante sobre a homeopatia, que
> também
> > >foi
> > > ''atacada'' nesta discussão!
> > > FAVOR irem a www.arenales.com.br e lá encontrarão muita
> > >informação,
> > > onde especialmente destaco muita ciência e homeopatia para
> ANIMAIS. Um
> > >lance
> > > psicossomático, certamente.
> > > A arte de ''sugestionar'' animais é maravilhosa, não? :]]] abr/M.
> > >
> > > >From: "Oraculo" <oraculo@a...>
> > > >Reply-To: ciencialist@yahoogrupos.com.br
> > > >To: <ciencialist@yahoogrupos.com.br>
> > > >Subject: [ciencialist] Zodiaco e Acupuntura..:-)
> > > >Date: Fri, 4 Feb 2005 16:49:32 -0200
> > > >
> > > >Olá Esteban
> > > >
> > > >Há uma sequencia de enganos em suas colocações, talvez derivada
> de um
> > > >compromentimento com a crença inicial e uma certa aversão a
> > >ciência..:-).
> > > >Métodos usados na pesquisa cientifica não servem apenas para
> impedir
> > >que
> > > >alegações reais, mas incomodas, sejam demonstradas. Servem para
> filtar
> > > >alegações sem base na realidade.
> > > >
> > > >Assim, acusar os que pesqusiasm com ironias do tipo "santos céticos
> > > >imparcialistas" é bobagem e uma falacia "ad hominem". A não ser
> que
> > >credite
> > > >tudo a Grande Conspiração Mundial Da Ciência Para Esconder A
> Verdade. E
> > > >nesse caso, nada do que eu, ou qualquer outro diga, fará a menor
> > >diferença,
> > > >certo?..:-)
> > > >
> > > >Este trecho é um problema:
> > > >
> > > >Talvez devesse dar algum crédito mesmo. Já estão acreditando em
> > > >acupuntura... Mas foi testada, nê, botaram uns eletrodozinhos e
> viram
> > >que
> > > >tinham razão aqueles outros velhinhos há mais de 3000 anos atrás...
> > > >Acordaaaaa!!!!
> > > >
> > > >Isso já foi explicado diversas vezes, mas vai de novo..:-) E, não
> > >precisa
> > > >acreditar em mim, basta apenas pensar sobre o que argumento e
> ver se é
> > > >possível refutar, ok?
> > > >
> > > >Acupuntura é a cura de todas as doenças pela manipulação de
> meridianos
> > >de
> > > >energia não detectável e mística através da implantação de
> agulhas em
> > > >pontos determinados desses meridianos. Isso é acupuntura e a
> base dessa
> > > >afirmação é que a causa de todas as doenças é o desequilibrio de
> > >energias
> > > >não detectáveis.
> > > >
> > > >Recentemente, detectou-se algum efeito na sintese de
> prostaglandinas ao
> > >se
> > > >aplicar agulhas na pele de pacientes. Como as prostaglandinas estão
> > > >envolvidas nas sensações dolorosas, isso pode causar algum
> efeito do
> > >tipo
> > > >anestésico leve pelas agulhas.
> > > >
> > > >Mas, isso NÃO É MAIS ACUPUNTURA. Isso é outra coisa, uma nova
> > >descoberta
> > > >sobre efeitos de agulhas em sintese de prostaglandinas. Nào
> exige que
> > >os
> > > >meridianos sejam identificados, nào exige que pontos
> especificos sejam
> > > >encontrados, e pode ser substituida por beliscões, em seu
> efeito nas
> > > >prostaglandinas.
> > > >
> > > >Distorcer um conceito, como a definição de acupuntura, para
> caber em
> > > >descobertas recentes e cientificas, é um procedimento padrão em
> > > >pseudociencias, também usado pela astrologia. Mas é evidente
> que, se
> > >não se
> > > >baseia na causa energética das doenças, se não exige o
> reequilíbrio
> > >dessas
> > > >energias, se independe de meridianos por onde essa enegia
> corre, não é,
> > > >claro, acupuntura.
> > > >
> > > >O que a acupuntura alega, e que nào é reconhecido pela ciência
> por não
> > >ter
> > > >se mostrado eficaz e nào ter mecanismo de ação reconhecivel, é que
> > >doenças
> > > >são causadas por desequilibrios de energia e que podem ser curadas
> > > >reequilibrando essas energias com agulhas em pontos específicos
> e bem
> > > >determinados da pele. Ela não alega que espetar agulhas (ou causar
> > > >sensações dolorosas de algum tipo) na pele das pessoas,
> interfere na
> > > >sintese de prostaglandinas, podendo servir como anestésico leve.
> > > >
> > > >Veja, nào deve acreditar em mim, deve apenas analisar o
> argumento. Se
> > >puder
> > > >demonstrar que está incorreto, ótimo. Se não puder, então ele é
> valido
> > >e
> > > >deve se manter até que algo mude (por exemplo, descobrir que
> energias
> > >nào
> > > >detectáveis causam tuberculose).
> > > >
> > > >Aqueles velhinhos de 3000 anos continuam enganados quanto a suas
> > >alegações.
> > > >Continuam enganados ao afirmarem que doenças são causadas por
> > > >desequilibrios energéticos. Continuam enganados ao afirmar que a
> > >energia
> > > >nào detectável corre por linhas determinadas chamadas meridianos.
> > >Continuam
> > > >enganados ao afirmar que podem curar doenças reequilibrando essas
> > >energias.
> > > >Continuam enganados ao afirmar que o reequilibrio se dá com
> agulhas em
> > > >pontos determinados da pele. E nào estão enganados sobre a
> sintese de
> > > >prostaglandinas, simplesmente porque nada sabiam sobre sintese de
> > > >prostaglandinas ou sobre qualquer outro sistema metabolico do
> corpo
> > >humano.
> > > >
> > > >Homero
> > > >
> > > >
> > > >
> > > > ----- Original Message -----
> > > > From: Esteban Moreno
> > > > To: ciencialist@yahoogrupos.com.br
> > > > Sent: Friday, February 04, 2005 4:24 PM
> > > > Subject: Re: [ciencialist] Re: Zodiaco
> > > >
> > > >
> > > >
> > > > Mr Takata wrote:
> > > > > descrições astrológicas?
> > > > Entao, em principio pareceria 'suspeito'. Mas como dito, qdo
> > > > devidamente testada a astromancia nao parece ter um bom poder de
> > > >predicao.
> > > >
> > > > E:
> > > > Devidamente testada por quem? Pelos santos céticos
> imparcialistas da
> > > > ciência?
> > > > Depende de quem ou como se pesquisa. Há uma série de artefatos e
> > > >falácias do
> > > > que julgam como pesquisas comprobatórias. Não vou
> enumera-las, mas
> > > >talvez
> > > > ajude a lembrar que a pesquisa da universidade de brasília
> resultou
> > >em
> > > >dados
> > > > bastante promissores. Que péssimo! Uma das justificativas dos
> > >céticos
> > > >acerca
> > > > do porquê os pacientes sentem-se bem descritos pelo
> astrólogo é a
> > > >chamada
> > > > confluência (auto-atribuição é um nome pobre para isso). Abaixo
> > >segue um
> > > > artigo científico que "desmistifica" (perdoe-me..) este recurso:
> > > >
> > > > Self-attribution, sun-sign traits, and the alleged role of
> > > >favourableness as
> > > > a moderator variable:
> > > > long-term e.ect or artefact? Edgar Wunder* Gesellschaft fu¨r
> > > >Anomalistik,
> > > > Postfach 1202, 69200 Sandhausen, Germany Personality and
> Individual
> > > > Differences 35 (2003) 1783-1789 www.elsevier.com/locate/paid
> > > >
> > > > Posso envia-lo em anexo.
> > > >
> > > > Takata:
> > > > > Um bom parâmetro é a quantidade do mercado que vem se
> > > > > ampliando extensivamente, incluindo grandes empresas e
> > >corporações.
> > > > Esse na verdade eh um pessimo parametro. Se for por isso
> deveremos
> > > > creditar confianca na homeopatia, passes espirituais (sem
> falar na
> > > > cirurgia mediunica), toda sorte de mandingas...
> > > >
> > > > E:
> > > > Talvez devesse dar algum crédito mesmo. Já estão acreditando em
> > > > acupuntura... Mas foi testada, nê, botaram uns eletrodozinhos e
> > >viram
> > > >que
> > > > tinham razão aqueles outros velhinhos há mais de 3000 anos
> atrás...
> > > > Acordaaaaa!!!!
> > > >
> > > >
> > > > Infelizmente só retorno daqui a 10 dias.
> > > > Um carnaval cheio de boas mandingas para vocês.
> > > > Esteban.
> > > >
> > > >
> > > >
> > > > ##### ##### #####
> > > >
> > > > Para saber mais visite
> > > > http://www.ciencialist.hpg.ig.com.br
> > > >
> > > >
> > > > ##### ##### ##### #####
> > > >
> > > >
> > > > Yahoo! Grupos, um serviço oferecido por:
> > > >
> > > >
> > > >
> > > >
> > > >
> > > >
> > > >
> > >
> > >
> >------------------------------------------------------------------------------
> > > > Links do Yahoo! Grupos
> > > >
> > > > a.. Para visitar o site do seu grupo na web, acesse:
> > > > http://br.groups.yahoo.com/group/ciencialist/
> > > >
> > > > b.. Para sair deste grupo, envie um e-mail para:
> > > > ciencialist-unsubscribe@yahoogrupos.com.br
> > > >
> > > > c.. O uso que você faz do Yahoo! Grupos está sujeito aos
> Termos do
> > > >Serviço do Yahoo!.
> > > >
> > > >
> > > >
> > > >[As partes desta mensagem que não continham texto foram removidas]
> > > >
> > >
> > >
> > >
> > >
> > > ##### ##### #####
> > >
> > > Para saber mais visite
> > > http://www.ciencialist.hpg.ig.com.br
> > >
> > >
> > > ##### ##### ##### #####
> > >
> > >
> > > Yahoo! Grupos, um serviço oferecido por:
> > >
> > >
> > >
> > >
> > >
> > >
> > >
> >
> >------------------------------------------------------------------------------
> > > Links do Yahoo! Grupos
> > >
> > > a.. Para visitar o site do seu grupo na web, acesse:
> > > http://br.groups.yahoo.com/group/ciencialist/
> > >
> > > b.. Para sair deste grupo, envie um e-mail para:
> > > ciencialist-unsubscribe@yahoogrupos.com.br
> > >
> > > c.. O uso que você faz do Yahoo! Grupos está sujeito aos
> Termos do
> > >Serviço do Yahoo!.
> > >
> > >
> > >
> > >[As partes desta mensagem que não continham texto foram removidas]
> > >
>
>
>
>
>
> ##### ##### #####
>
> Para saber mais visite
> http://www.ciencialist.hpg.ig.com.br
>
>
> ##### ##### ##### #####
>
>
> *Yahoo! Grupos, um serviço oferecido por:*
>
> *
> <http://br.rd.yahoo.com/SIG=12a6dvtqh/M=264105.3931087.6562589.1588051/D=brclubs/S=2137111528:HM/EXP=1107730748/A=2361264/R=6/SIG=10v4acpp0/*http://br.shopping.yahoo.com/>*
>
>
>
> ------------------------------------------------------------------------
> *Links do Yahoo! Grupos*
>
> * Para visitar o site do seu grupo na web, acesse:
> http://br.groups.yahoo.com/group/ciencialist/
>
> * Para sair deste grupo, envie um e-mail para:
> ciencialist-unsubscribe@yahoogrupos.com.br
> <mailto:ciencialist-unsubscribe@yahoogrupos.com.br?subject=Unsubscribe>
>
> * O uso que você faz do Yahoo! Grupos está sujeito aos Termos do
> Serviço do Yahoo! <http://br.yahoo.com/info/utos.html>.
>
>
>
>
> __________ Informação do NOD32 1.990 (20050202) __________
>
> Esta mensagem foi verificada pelo NOD32 Sistema Antivírus
> http://www.nod32.com.br




SUBJECT: Re: Questões biológicas
FROM: "rmtakata" <rmtakata@altavista.net>
TO: ciencialist@yahoogrupos.com.br
DATE: 06/02/2005 00:06


--- Em ciencialist@yahoogrupos.com.br, Antonio Ferrão Neto
> 1 - como são determinadas as calorias dos alimentos, como se
> reduz o teor calórico deles, como se reduz o açúcar deles e
> também as gorduras (saturadas e totais).

Usa-se uma bomba calorimetrica. Uma quantidade determinada de alimento
eh incinerada em uma camara e o aquecimento da agua q. envolve a
camara eh determinada pela diferenca da temperatura. Desconta-se a
energia fornecida para queimar a comida e se atribui a diferenca ao
teor energetico do alimento.

> 2 - como são produzidas as sementes das frutas que não
> produzem sementes?

Propagacao vegetativa. Existem diversos meios - cultivo de meristema
(tecido indiferenciado no apice caulinar e radicular das plantas),
estaquia, alporquia...

> Além de sermos em parte um produto do meio e termos o
> nosso comportamento controlado pela razão, existe essa
> influência da raça sobre o nosso comportamento (dos seres
> humanos)?

Nao existe uma diferenciacao em raca de sentido biologico em humanos
tto qto se sabe - o q. se chama de raca tem sentido social e se baseia
apenas em uns poucos marcadores externos (cor de pele, formato do
nariz e do cabelo e soh). Entre os caes ocorre um intenso cruzamento
seletivo de modo q. existe uma pureza da raca. No caso de humanos, hah
uma miscigenacao relativamente intensa - diferencas geneticas dentro
de cada populacao costuma ser maior do q. diferencas geneticas entre
as populacoes (considerando-se a media e o desvio padrao).

Mas tto qto se sabe hah uma forte influencia genetica no comportamento
humano, influencia q. pode ser herdada geneticamente - certos tracos
de comportamento apresentam maior concordancia entre gemeos
univitelinos criados separados do q entre pessoas sem vinculacao
genetica maior criadas em uma mesma familia. Claro, existem diversos
outros tracos em q. irmaos de criacao sao mais parecidos entre si do
q, entre irmaos gemeos criados separados.

[]s,

Roberto Takata





SUBJECT: Re: [ciencialist] Re: Zodiaco
FROM: "Sergio M. M. Taborda" <sergiotaborda@terra.com.br>
TO: ciencialist@yahoogrupos.com.br
DATE: 06/02/2005 00:22

Oraculo wrote:

>
> E a matematica do zodiaco pode ser, e deve ser, precisa, mas isso em
> nada melhora sua eficacia funcional, conforme esta alega ter.

Ninguem está discutindo isso. O ponto do assunto é o sistema
astrologico, se ele se baseia ou não nas constelações que portam os nome
dos signos. E para estabelecer esse sistema, e explicar pq eles não se
vinculam com as constelações é preciso matemática, pois é com base nela
que o sistema é criado, ao contrário do q vc pensa.

> Determinar setores do céu, acreditando que estes influenciam a vida na
> Terra, precisamente, nada indica que essa influencia era real.

Vê, se vc soubesse o minimo, não dizia essas asneiras. Os signos não
influenciam em coisa nenhuma.

>
> Taborda, se esconder atrás da matematica da astrologia, não vai
> embasar a alegação de eficácia ou realidade da mesma.

Mais uma vez , vc distorce o que leu. A explica matemática serve para
responder ao argumento de que a astrologia se baseia nas constelações.
Sendo que a matemática mostra, sem margem para duvidas que apenas o
numero 12 poderia ter sido escolhido, acho que é mais do que evidente
que nada tem a ver com constelações. Se vc acha essa explicação
atrofiante da sua arguemtação, o suficiente para vc mudar de assunto, o
problema não é meu. Ninguem está discutindo a eficácia da astrologia.

> Dividiram 12 setores do céu, seja porque o número 6 era sagrado, ou
> seja porque decidiram encaixar cada setor exatamente em uma
> constelação é irrelevante como defesa da astrologia.

Pois é. Mas ninguem está defendendo a astrologia. Eu estou atacando a
vossa incultura. A vossa falta de conhecimentos tecnicos de astrologia (
e de fisica, matemática e astronomia) .
Principalmente estou atacando argumentos imbecis, fajutos e hipocritas
que são levantandos aqui periodicamente.

> E, sim, poderiam ter a crença de que 13 era um número sagradao e
> incluido um setor a mais, mesmo sendo apenas 12 constelações na época,
> já que o que importa era o credo, não a realidade do mesmo.

Acho que vc não percebeu. Não eram 12 constealações na época. Era muitas
mais. Mas o signos são doze, independentemente do numero de constelações.

>
> Usei 13, poderia ter usado 24 ou 6 ou 2 ou 35. Usei 13 porque alguns
> astrologos (certamente tolos e incapazes..:-) tem ajustado seus
> zodiacos para encaixar ofiuco e criar mapas astrais que incluam a
> precessão.

Ha maus proficionais em todas as profissões. Também ha fisicos tentando
encaixar monopolos magneticos e particulas que não podem ser observadas
, nas suas explicações se fenomenos que ninguem viu.
Isso significa que a Fsicia está condenada ? que é mentira ? Ora... que
argumento fraco.

> O significado é que os criadores da astrologia poderiam ter usado o
> que fosse necessário para encaixar mitos e crenças em um conhecimento
> astrologico. Como lembrou o Takata, astrologia tupi-guarani ou asteca
> é diferente e tem matematica e interpretações diferentes.

Ninguem nega isso. Ha muitas astrologias. O que foi mensionado, é que
astrologias que tem a mesma linagem, mesmo asism ,têm simbolos
diferentes, e não dependem das constelações.

>
> "Taborda: Se vc soubesse mais de fisica, saberia que o sol sim gira em
> torno da
> terra , tal como a lua, tal tudo o resto, para quem está na terra. Para
> o sistema do observador solidário com a terra, tudo o resto anda À volta
> dela. É por falta deste conhecimento que pouca gente entende
> Relatividade (Galiena mesmo, para não falar da outra). A Primeira regra
> é que o que vc observa depende do referencial que esolhe.
> A astrologia, não nega que o seu referencial é a Terra. E como tal, é
> correcto, observatorialmente e até fisicamente, dizer que é o sol que se
> move em torno da terra."
>
> Bem, sei disso, pelo menos a parte que faz sentido..:-) Sim, a
> percepção do movimento solar é que se move em torno da Terra. Dai a
> afirmar que tanto faz e que é fisicamente correto dizer que este gira
> em torna da Tera, me parece exagero, não?

Não. É tão fisicamente correcto afirmar que a terra gira em torno do
sol, como que o sol em torno da terra. É isso que a Relatividade nos
ensina.
Se vc quer ser completamente corrrecto, nenhum dos dois gira em torno do
outro, pois ambos giram em torno do centro de massa do sistema.

>
> Também sei que, mesmo observação a olho nú, os planetas se movem de
> forma diferente, com trajetorias celestes confusas, inexplicáveis sem
> o conhecimento de que todos, inclusive a Terra, giram em torno do Sol.

Mas não é objectivo da astrologia explicar o movimento dos planetas. A
astrologia apenas cataloga as suas posições e usa essa informação.
A astrologia não é dinamica, é cinemática.

>
> Se isso foi uma defesa do conhecimento superior dos criadores da
> astrologia, foi fraco.

Não, não foi. Foi meramente uma explicação fisica, cientifica,
necessária ao assunto.

>
> Deu para entender pq a astrologia é aquilo que é e pq os argumentos como
> os seus e os do takata são ocos de sentido logico ? Pq eles são mais
> bestas do que os argumentos dos astrologos em si ?
> Se não deu é pura má fá da sua parte. Acho que o texto é bem explicativo."
>
> Sim, explica que uma crença forte não pode ser refutada com tolices
> como argumentos e evidencias..:-)

Sim. So que a crença é a sua. O facto de eu saber o que é astrologia,
não significa que eu acredite nela.

> Se não devo dar o sentido atual de estrela e constelações, fica pior
> ainda. Devo usar um sentido de milhares de anos atrás, sentido que não
> faz sentido, já que eles ignoravam muito do que é real nesses objetos
> do universo.

Cara, eles tinham a sua lingua e inventaram as palavras. Nós apenas as
usamos as mesmas palavras com significado diferente. E qq linguista lhe
pode dizer que isso é normal.
Agora, vc quer intrepretar as ideias deles com os significados de hoje.
Isso é no minimo injusto, mas vc sabe muito bem que é errado. Vc daria
um pessio arqueologo pensando assim.

> E continua com problema, como podem, seja que conceito use, conter
> informação, essas criações irreais da imaginação humana,
> principalmente informação sobre o universo fisico e real, que depende
> dos conceitos atuais e dados sobre a realidade destes?

Vc que acha que dependem. Não dependem de facto.

> Eles decidiram que as fixas não tem importancia, e que as que se
> movem, informam sobre o mundo e sobre a personalidade das pessoas.
> Ótimo..:-) E provam ou mostram evidencias dessa alegação como? Quem
> decide? Como testar? Depois que decidiram isso, alguém mais decidiu
> alguma coisa, ou o conhecimento se mantém estático a 3000 anos?

Sim, se manteve.

> Se decidissem de qualquer outra forma, teriamos que acreditar e pronto???

Vc não tem que acreditar, isto não é uma religião. Vc pode testar e ver
por vc mesmo os resultados.

> De onde tiram essas alegações, da matematica e pronto???

Não. Da filosofia. Como eles fizeram isso, ninguem sabe.

>
> Homero: > Sempre mais do mesmo. Apenas prove o que afirma, e nenhuma
> discussão
> > seria necessária.
>
> Taborda: Acabei de o fazer.
> Claro que vc não está convencido, pq vc escolhe não entender, mas não
> pode negar que a logia está ali. A historia, a metamática, a fisica e
> até a semântica, confirmam.
> Que mais provas vc quer ?
>
> O que você provou???

Provei que :
1) O numero 12 não é aleatorio. É baseado nas conjunções Sol-Lua que
existem num ano solar.
2) Que sendo , a separação do ceu devida aos periodos do sol e da lua,
essa separação não tem nada a ver com constelações.
3) Que as configurações das constelações das estrelas fixas pode ser
comum a várias civilizações, sem que os simbolos que elas representam
sejam os mesmos.
4) Que os simbolos ortogados aos signos não são os mesmos em todas as
astrologias da linha indu- que é a que estamos discutindo- , mas que
isso não impede que os signos tenham as mesmas caracteristias, o que
significa que primeiro se definem as caracteristicas e depois de escolhe
um simbolo menmeumonico para elas. Isso responde tb ao argumento sobre
marte.


> Que sabe mais sobre a matematica usada que eu???

De facto sei.

> he he he.. Tem de ser, você é astrologo e eu não..:-)


Nenhum de nós é. Mas eu preocupei-me em saber o que é astrologia e vc não.

> Isso é uma alegação correta, mas e daí???

Dai que vc está errado. Seus argumentos são infundados pois são fundados
em informações falsas.

> Não há logica, ou melhor, a lógica aqui apresentada por você não é
> evidencia, é circular.

Seus argumentos tb o são. E sendo que estou abatendo-os, os meus têm que
os rodear.

> A astrologia pode conter informação sobre o mundo real e sobre as
> pessoas porque os que a criaram usavam matematica e liam informação
> real nos planetas, mas não com o conceito atual, mas com o dos tempos
> antigos. E daí???

Vc entendeu tudo errado. Dai , nada.

> O que isso prova??? Isso é uma afirmação, não uma evidencia.

O que vc disse não prova nada. Mas tb, não tem nenhuma relação com o
que eu disse.

>
> Prova seria ligar, com bases confiáveis, resultados de previsões
> astrologicas e dados reais.

Quando vc me responder como converter em dados reais os sentimentos, eu
lhe direm como ligar esses dados com a astrologia.

> Isso seria uma prova. Mas, jogos de palavras como "tudo pode ser
> positivo ou negativo, depende". não é nada disso. Ninguém que lesse
> esta mensagem diria, bem, agora está provado que calculos astrologicos
> podem determinar a personalidade de uma pessoa com base na data e hora
> de seu nasciemento, informação que está contida nas posições de
> planetas no céu.

Nao é na posição, é na posição relativa. É bem diferente.

>
> Desculpe, mas não há nenhuma prova ou evidencia disso em nada nos
> argumentos.

Não era suposto haver. Eu não estava falando disso. Se era disso que
estava à espera que eu falasse, lamento desapontá-lo.


E por favor, não faça top-posting.

Sergio Taborda


--
No virus found in this outgoing message.
Checked by AVG Anti-Virus.
Version: 7.0.300 / Virus Database: 265.8.5 - Release Date: 03-02-2005



SUBJECT: Re: [ciencialist] Re: Zodiaco
FROM: "Oraculo" <oraculo@atibaia.com.br>
TO: <ciencialist@yahoogrupos.com.br>
DATE: 06/02/2005 00:56

Olá Taborda

"Taborda: Não entendi o que vc quiz dizer com isto"

Quiz dizer que respostas vagas e de duplo sentido, como "qualidades as vezes são negativas e as vezes positivas" se ajustam a qualquer coisa e alegação, não sendo base para um argumento ou demonstraçao de evidencia.

Taborda: Complexos ? 365 / 29 é complexo ?"

risos..::-) Isso é só acessório, não principal e nada modifica o argumento. Se preferir, use "calculos simples" no lugar de complexos e ainda vai ser um argumento: calculos matematicos simples nada significam sozinhos.

O que significa que efetuar contas e calculos e eles estarem corretos, nada diz sobre a interpretação dos mesmos ou sobre de onde vem a informação sobre minha personalidade.

Homero: Se toda qualidade é ao mesmo tempo boa e má, se toda pessoa pode ser
uma hora uma coisa e outra outra coisa,

Taborda: Viu ? Isto é tipico da argumentação não cientifica. Pegam num argumento
, extrapolam-o além do seu significado, e reaplicam-o como se fosse
válido. Vc está pensando da mesma forma que pensa um crente num deus qq.
Auto-convencendo-se de que os argumentos que ele cria, são os desejos
desse deus
É verdade q todos os atributos são bons e maus. Mas ninguem disse que a
pessoa é uma hora uma coisa e outra ora outra. Isso vc fabricou na sua
mente como um sinonimo , mas que não é.
Ai vc chega num resultado logico que vc acha que contraria o argumento
inciial, mas não o faz pq uma das permissas é inválida.

Não estou extrapolando nada, estou tentando contornar as vias tortuosas de um argumento sem base. Se essa interpretação não lhe agrada, explique a sua. Mas, insisto, se um atributo é aplicavel a seu oposto tanto quanto a seu sentido original, ele vai servir para qualquer pessoa. Não fabriquei nada, usei seu texto e seu exemplo. Se as premissas são inválidas, são suas premissas que usei.

Determinar que alguém é indeciso ou ponderado dá na mesma e serve a qualquer mapa astral, e o paciente vai encontrar relevancia de todo modo.

Homero: então, para que sereve a astrologia???

Taborda: Não estamos discutindo para que serve, e sim, o que é. Em que bases
teoricas se apoia, q filosofia segue e o que tem a ver com astronomia.

Estamos sim, porque se admitir que não serve para nada, finda a discusão. Todo este debate é simplesmente porque você defende que ela serve para analisar e compreender a personalidade de uma pessoa, o mundo em que vivemos e mesmo o futuro que nos espera. Se não serve para isso, a discussão acabou.

Taborda: Vc pode especular a razão à sua vontade. A questão aqui não é quem se
vai consultar com o astrologo, ou o astrologo, mas a astrologia. A
astrologia é uma teoria, como outra qualquer.

De forma alguma! Teorias fazem previsões e determinam efeitos. Se as previsões se confirma e os efeitos são mensuráveis, ela tem maior probabilidade de ser real. A astrologia faz previsões, mas estas não se confirmam, e seus efeitos não podem ser validados. Ela é muito diferente da maioria das teorias e mais parecida com uma crença esoterica.

Taborda: E o que está acontecendo aqui é que se está julgando a teoria errada.
Vcs continuam, e insistem , em falar sobre algo que não existe, sobre
uma astrologia que vcs proprios criaram que nada tem a ver com a
verdadeira astrologia.

Não, usamos, como todos os que concluiram pela irrealidade e ineficácia da astrologia, as afirmações e alegações de astrologos, embora não se possa usar TODOS os astrologos do mundo. E a "verdadeira" astrologia soa exatamente como a "verdadeira" religião, qualquer delas, que usam o mesmo argumento: não sabemos do que falamos e só existe uma verdadeira .. etc.

Taborda: E por isso que eu não me canço de replicar.

Sim, me lembro que já tivemos essa mesma discussão antes..:-) E não se cansa de replicar, mas não explica a diferença de sua verdadeira astrologia das outras incorretas, não explica como um calculo matemático de posicionamento pode determinar a personalidade de uma pessoas, não explica como a data e hora de nascimento pode influir na personalidade dessa pessoa, em sua sorte, suas caracteristicas fisicas, seu futuro, nada.

Apenas afirma, repetidamente, que não sabemos do que estamos falando, que a verdadeira astrologia é verdadeira e que funciona, embora não exista evidencias disso. Nem mesmo sugeriu um experimento que permitisse testar essa afirmação.

Por isso não cansamos de refutar e afirmar que a astrologia não tem evidencias de ser real ou eficaz no que alega poder fazer.

Tabroda: Vc quer acreditar demais que a astrologia não funciona, sem nunca ter
experimentado, sem sequer saber o que é , no que se baseia, q regras
segue, tanto quando vc acha que o consulente quer acreditar no astrologo.
Ao contrario das artes adiinhatorias a astrologia não funciona por
leitura fria, pois o contacto entre o astrologo e o consulente não é
requerido."

Disse antes e vou repetir, experimentar a astrologia não vai me dar base para concluir que funciona ou não. Assim como ir a um show de mágica não vai me permitir dizer se o artista tem poderes reais ou usa truques. Relatos pessoais e experiencias individuais nada indicam sobre a realidade de uma alegação. Você deve saber disso, pois foi exatametne o que afirmou sobre o experimento da Maria Natalia.

Se a astrologia não funciona por leitura fria, se ela realmente indica com precisão aspectos da personalidade e do futuro, caracteristicas e eventos de um paciente, seria possivel validar essa afirmação, simplesmente fazendo com que astrologos identificassem mapas astrais de diversos pacientes, em testes de duplo cego. Mas ela não consegue fazer isso.

Esses estudos e essas evidencias é que embassam minha conclusão, não uma consulta com um astrologo, por mais capaz que ele seja (eu também estou sujeioto a pareidolia, contar acertos e ignorar erros, ter uma visão pessoal de mim mesmo, etc).

Taborda: A historia de Fernando Pessoa, o poeta portugues ilustra bem isto. Ele
era astrologo e fazia um mapa astral para cada um dos seus heterónimos.
Era com isso que ele conseguia manter a personalidade de cada um
independente da dos outro. Mas , essas pessoas não existem .
Outro ponto é que a astrologia vai muito alem das pessoas. Ela pode ser
usada para estudar paises, ou todo o planeta com um todo. Como se
aplicar leitura fria a um planeta ? ou a um pais ?
São estes promenores que escapam - tão convinientemente - das vosses
considerações.

Voltamos a alegação (meio disfarçada) de ser a astrologia uma forma de auto-conhecimento. Se pode ser aplicada a heteronomios, pode ser apenas um conjunto de afirmações imaginárias, mas que permitiriam a pessoa aumentar seu conhecimento de sí mesma. Deve ser por isso que a maioria dos astrologos não concorda com você..:-) Para eles, é mais que isso, é conhecimento confiável, a ponto de ser possível determinar que uma pessoa não existe apenas pelo mapa astral dela (ou determinar que, se nascesse nessa data e hora, seria com essas caracteristicas).

Mas, se aplica o mapa astral a um país, e se ele é auto-conhecimento, nada vai ajudar ao pais a se compreender. Ajudaria a pessoa que o lê a compreender a sí mesma?

E como validar essa alegação, de que o mapa astral de um pais representa mesmo o pais e suas caracteristicas? E onde determina a data e hora do nascimento de um pais, se muitos deles, apenas surgem com o tempo, sendo suas datas de criação bastante arbitrárias? (problema parecido com cesarianas marcadas para determinar a personalidade de uma criança.:-)

Taborda:Exacto, tal como a Metereologia. Mas mesmo assim vc continua lendo a
previsão do tempo, mesmo assim essa informação é oficial, A metereologia
tem muitas mais variáveis que a astrologia, mas vc a aceita. Tlv vc
pense que ela é baseada na ciencia. Tlv, mas suas previsões são tão boas
quanto olhar o ceu todos os dias.

Errado. A metereologia, em que pese a brincaderia padrão sobre seus desacertos, tem aumentado sua eficácia em muitos níveis, dando boa margem de confiabilidade à suas previsões, principalmente com relação a furacões e tempestades. Evita milhares de mortes por ano e milhões em prejuizos. Comparar a astrologia com a metereologia é bastante incorreto. E ainda assim, não "acredito nela", no sentido de crença. Nem tomo como único parametro de escolha. Sempre, como toda alegação mesmo com evidencias, é preciso considerar a probabilidade de estar correta.

E mesmo em seus primordios, onde ainda se estudava as variaveis que influenciam o tempo, a metereologia nunca afirmou ser precisa ou confiável. Ela sabia que com o enorme numero de variaveis, era dificil produzir previsões corretas e confiaveis, e não alegava de forma alguma faze-lo, nem pedia que se acreditasse nela. Como todo conhecimento cientifico, conhecia suas fraquezas e lidava apenas com probabilidades, bem baixas no inicio.

E, fala sério, olhar para o céu ou usar supercomputadores para analisar milhares de variáveis que permitem até 5 dias de previsão confiável (lembre-se, nada de 100% para a ciência..:-) comparado com olhar para o céu e dizer se leoninos são assim ou assado, se alguém terá sorte ou se terá dificuldades na vida, é muito, mas muito diferente.

Taborda: O Takata citou o manual, e como pudemos ver, o atributo ferocidade não é
atribuido a leão. É tão simples como isso. Mas, claro, vc sempre vê
mais coisas das que realmente estão lá."

Fuga da resposta. Qualquer que seja o atributo, importa mesmo é porque tem de ser assim, e como mensurar astros pode determinar essa afirmação. O que embassa essa conclusão, sobre ser assim ou assado. Pouco importa o que o manual diz, importa é que não tem base para dizer o que quer que seja. Evitou responder a questão principal, apegando-se ao acessório, se tal caracteristica estava ou não no manual.

Não importa o que realmente está lá, importa se pode apresentar evidencias que deveria estar lá. Que, qualquer atributo que o manual determine, tem confiabilidade e relação com a realidade, a personalidade de um leonino. Isso é que importa, e que evitou responder.

Homero: Marte ser vermelho torna as pessoas de um jeito e não de outro??? Se o
ferro no planeta não existisse, regidos por Marte teriam outra
personalidade?? Fala sério..:-)

Taborda: Fala sério vc. Essa questão é mais uma palhaçada, pq nunca ninguem disse
que o vermelho de marte torna o que quer que seja , no que quer que seja.

Mais uma vez, pouco importa. Fosse Marte diferente, não existisse Marte ou fosse verde, seriam os influenciados por Marte diferentes? Essa a questão proposta. O que a posição, cor ou altura, existencia, etc, de Marte tem a ver com a personaldiade de alguém, como se dá essa influencia e de onde vem a base para afirmar qualquer coisa nesse sentido.

Taborda: Isso é o que vc acha. Mas vc pode provar isso ? Vc pode provar que eles
não sabiam ? Claro que não pode.

Depende do que considera provas e evidencias, embora acreditando em astrologia, sua exigencia por evidencias não deve ser alta. Evidencias de que não sabiam um bocado de coisas pode ser inferida pelo que produziram como cultura. Nada de aço, nada de ferro, nada de concreto, nada de remedios, nada de antibioticos, nada de substancias quimicas, explosivos, uma explicação astronomica bastante equivocada sobre os astros, etc.

A não ser que Atlantida esteja envolvida, essas evidencias deveriam ser o bastante para meu argumento.

Taborda: Os planetas são observáveis a olho nu, nas circunstancias metereologicas
convinientes. Não é dificil contar seus periodos, e criar mapas com
isso. A astrologia não precisa da fisica, nem da gravitação, nem de
coisa nenhuma , alem de pontos luminosos que se movem no ceu com
periodos regulares. Tudo o resto é matemática e intrepretação usando
regras defenidas à priori.

A primeira parte é astronomia, não astrologia. A segunda é o cerne do debate: que "regras a priori" e por que essas regras e não outras. De onde sairam essas regras de interpretação? O que dá sustentação a elas? Quem decidiu que determinada hora e posição resultavam em determinada caracteristica e determinada personalidade?

Agora é fácil ver a diferença entre matematica e a interpretação dos resultados dessa matematica. São preciso regras "a priori" para a validação da hipotese astrologica, e nada temos sobre sua confiabilidade, nem maneira de verificar seu acerto. Aceitamos essas regras, ou não.

Taborda: E precisa ser ? Só para si, e outros como vc.
Astrologia não é ciencia, não precisa ser baseada em evidencias.

Nisso concordamos totalmente. E, se pensa assim, por que tanta discussão, se a afirmação inicial era exatamente essa, não é ciência? E se não é ciência, como separa ou decide pela validade ou não entre a astrologia e todas as outras formas divinatorias que também não são ciência: Por que astrolgogia e não cartomancia? Ou leitura da iris, da borra de café, etc? Apenas porque ela usa matematica e as outras não?

Ou por que não o I-Xing, tecnica divinatória ancestral, com profunda relação com as forças do universo? Por que a astrologia, que não é ciência?

Taborda: 4) Vc acha que não tem nada a oferecer, pq vc nunca a estudou. Não
conheço ninguem que a tenha estudado e tenha saido pensando o mesmo que
pensava antes.

Ouvi isso antes, de diversos religiosos, tentando me convencer a estudar seus livros sagrados, sua teologia ou suas crenças. O mais recente foi um mórmom que insitia que se eu lesse atentametne o Livro De Mormom, isso modificaria minha vida (o livro de mormom foi escrito por um jovem chamado Joseph que, com a ajuda de duas pedras magicas colocadas dentro de um chapeu, traduziu magicamente o livro escrito em dialeto desconhecido, com a verdadeira palavra de deus, inclusive demonstrando a apostasia da igreja catolica romana).

Não creio que seja um argumento ou evidencia a demonstrar a realidade da astrologia.

Taborda: 5) Muitas coisas na vida dos seres humanos não apresentam eficácia e
mesmo asssim as fazemos. Isso, não é argumento.

Concordo. Mas, saber que não apresenta eficácia faz toda diferença, na maioria das vezes. E fazemos coisas estranhas, como separar essas coisas estranhas da astrologia? Está afirmando que apesar de não ter eficácia, devemos aplicar a astrologia? E bater na madeira 3 vezes para espantar o azar, que também não tem eficácia, devemos também fazer isso? Ou não passar em baixo de escadas? Evitar gatos pretos? Não estou fazendo gozação, estou perguntando por que, se sabe que não tem eficãcia, insiste em que devemos continuar aplicando o sistema.

Homero


----- Original Message -----
From: Sergio M. M. Taborda
To: ciencialist@yahoogrupos.com.br
Sent: Saturday, February 05, 2005 11:46 PM
Subject: Re: [ciencialist] Re: Zodiaco


Oraculo wrote:

> Olá Taborda
>
> Sua resposta resvala para coisas do tipo "tudo é tudo, nada é nada,
> mas as vezes, nada é tudo e tudo é nada..." Bom para Paulo Coelho,
> mas irrelevante para a discussão ou eficácia da astrologia.

Não entendi o que vc quiz dizer com isto.

>
> Calculos matemáticos complexos nada significam sozinhos.

Complexos ? 365 / 29 é complexo ?

> Se toda qualidade é ao mesmo tempo boa e má, se toda pessoa pode ser
> uma hora uma coisa e outra outra coisa,

Viu ? Isto é tipico da argumentação não cientifica. Pegam num argumento
, extrapolam-o além do seu significado, e reaplicam-o como se fosse
válido. Vc está pensando da mesma forma que pensa um crente num deus qq.
Auto-convencendo-se de que os argumentos que ele cria, são os desejos
desse deus
É verdade q todos os atributos são bons e maus. Mas ninguem disse que a
pessoa é uma hora uma coisa e outra ora outra. Isso vc fabricou na sua
mente como um sinonimo , mas que não é.
Ai vc chega num resultado logico que vc acha que contraria o argumento
inciial, mas não o faz pq uma das permissas é inválida.

> então, para que sereve a astrologia???

Não estamos discutindo para que serve, e sim, o que é. Em que bases
teoricas se apoia, q filosofia segue e o que tem a ver com astronomia.

> Se sua resposta fizesse sentido, um mapa astral feito para mim,
> serviria para qualquer pessoa.

Não. Se os SEUS argumentos fizessem sentido, VC poderia até chegar nessa
conclusão. Mas essa seria a SUA conclusão baseada nos SEUS argumentos,
que não são os meus , nem os de ninguem mais. E portanto, não são os dos
astrologos, e portanto qq conclusão a que VC chegue não tem nada a ver
com astrologia.

> Desmontando os argumetnos do Takata? Nem de longe..:-)

Eu já conheci várias pessoas como vc. Vcs tem um problema grave de
leitura. Ou não entendem o que leem , ou o distorcem. É por isso que vc
não entendeu a desmontagem e não entende nada do que eu escrevo, em geral.

>
> Se o leonino é as vezes autoritario e as vezes não é, (..)


SE , a palavra inportante dessa frase é SE. SE fosse .Mas não é. Então
tudo o que vc diz a seguir não se aplica.

>
> Pessoas fazem mapas astrais para saber o que as diferencia de outras
> pessoas, o que as torna únicas, baseadas na data e hora em que nasceram.

Vc pode especular a razão à sua vontade. A questão aqui não é quem se
vai consultar com o astrologo, ou o astrologo, mas a astrologia. A
astrologia é uma teoria, como outra qualquer.
E o que está acontecendo aqui é que se está julgando a teoria errada.
Vcs continuam, e insistem , em falar sobre algo que não existe, sobre
uma astrologia que vcs proprios criaram que nada tem a ver com a
verdadeira astrologia. E por isso que eu não me canço de replicar. Pois
considero que a vossa acção não é cientifica e é hipocrita, já que por
um lado defendem a ciencia a todo o custo, e por outro não sabem fazer
ciencia. Vc acusa quem consulta o astrologo, por ser enganado por um
processo qualquer de leitura fria. Mas vc mesmo comete os mesmos erros.
Vc quer acreditar demais que a astrologia não funciona, sem nunca ter
experimentado, sem sequer saber o que é , no que se baseia, q regras
segue, tanto quando vc acha que o consulente quer acreditar no astrologo.
Ao contrario das artes adiinhatorias a astrologia não funciona por
leitura fria, pois o contacto entre o astrologo e o consulente não é
requerido.
A historia de Fernando Pessoa, o poeta portugues ilustra bem isto. Ele
era astrologo e fazia um mapa astral para cada um dos seus heterónimos.
Era com isso que ele conseguia manter a personalidade de cada um
independente da dos outro. Mas , essas pessoas não existem .
Outro ponto é que a astrologia vai muito alem das pessoas. Ela pode ser
usada para estudar paises, ou todo o planeta com um todo. Como se
aplicar leitura fria a um planeta ? ou a um pais ?
São estes promenores que escapam - tão convinientemente - das vosses
considerações.

> Isso é tolice. Mas poderia ser tolice e ainda assim ser real, mas
> também não há evidencias disso. Mapas astrais de excelentes astrologos
> são conflitantes entre sí. Sim, são as variaveis que são muitas. Mas,
> qualquer coisa em que as variaveis sejam tantas que as respostas nunca
> coincidam, são inúteis por defrinição.

Exacto, tal como a Metereologia. Mas mesmo assim vc continua lendo a
previsão do tempo, mesmo assim essa informação é oficial, A metereologia
tem muitas mais variáveis que a astrologia, mas vc a aceita. Tlv vc
pense que ela é baseada na ciencia. Tlv, mas suas previsões são tão boas
quanto olhar o ceu todos os dias.

>
> O cara demorou para decidir? Então, o mapa acertou, afirmou ser
> indeciso. Mas, se o mapa diz o contrário, que ele é ponderado, também
> acertou..:-) Hilário.
>
> Esse o argumento bem defendido pelo Takata e que você contornou com
> jogos de palavras.

Não, não foi esse o argumento em discussão com o Takata. Vc está
distorcendo o que foi escrito. Mas pode sempre citar aqui a frase que
corraborar o que vc está dizendo.
A discussão era sobre a relação entre as constelações, seus nomes e
significados e os atributos que são de cada um dos signos.

> Não importa em nada o que o mapa afirme, já que qualquer
> caracteristica descrita pode ser aplicada a qualquer comportamento ou
> justificar qualquer coincidencia com o paciente.

Não, não pode. Mas como vc acredita piamente nisso, o que eu posso fazer
contra a sua crença ? Vc acha que não, quem sou eu para lhe ensinar que
não é assim.
O seu problema aqui, é que nenhum de vcs se debruçou o minimo sobre o
assunto, para chamar astrologia, ao mesmo que os astrologos chamam
astrologia. Vcs chamam astrologia a ideias que vcs pegaram e juntaram e
nunca consultar os reais manuais de astrologia.
O Takata citou o manual, e como pudemos ver, o atributo ferocidade não é
atribuido a leão. É tão simples como isso. Mas, claro, vc sempre vê
mais coisas das que realmente estão lá.

>
> Marte ser vermelho torna as pessoas de um jeito e não de outro??? Se o
> ferro no planeta não existisse, regidos por Marte teriam outra
> personalidade?? Fala sério..:-)

Fala sério vc. Essa questão é mais uma palhaçada, pq nunca ninguem disse
que o vermelho de marte torna o que quer que seja , no que quer que seja.

>
> Tem razão em uma coisa, as regras não foram criadas agora. Foram
> criadas a 3000 anos por pessoas que nada sabiam sobre fisica, leis da
> gravitação, planetas, e muitas outras coisas (inclusive, sobre
> psicologia e processos mentais neurologicos).

Isso é o que vc acha. Mas vc pode provar isso ? Vc pode provar que eles
não sabiam ? Claro que não pode.
Os planetas são observáveis a olho nu, nas circunstancias metereologicas
convinientes. Não é dificil contar seus periodos, e criar mapas com
isso. A astrologia não precisa da fisica, nem da gravitação, nem de
coisa nenhuma , alem de pontos luminosos que se movem no ceu com
periodos regulares. Tudo o resto é matemática e intrepretação usando
regras defenidas à priori.
Tlv eles não soubesses nada sobre esses assuntos, mas não repcisavam. E
a prova é que vários povos, em vários pontos do mundo faziam a mesma
coisa. Desde os maias aos chineses, passando pelos egipsios , gregos ,
sumerios, etc...

> São regras arbitrárias, destinadas a dar ordem a uma aparente desordem
> no mundo.

As regras do futebol também são arbitrárias, mas toda a gente as pode
conhecer e usar a qq momento.
O problema aqui, é exactamente esse, vc não sabe quais são as regras e
já quer depreciar o jogo.
Eu apenas lhe estou ensinando as regras verdadeiras do jogo.

> Nada indica que qualquer relação real entre as posições dos planetas
> (sejam eles o Sol a Lua e a Terra ou o conjunto dos demais), setores
> do zodiaco, ou conjunto de estrelas (que os criadores não sabiam o que
> eram) influencie a personalidade de qualquer pessoa.

Vc diz isso pq já fez estudos precisos sobre o asumo, presumo.

>
> Acreditar nisso é apenas questão de fé (e muita fé), não uma conclusão
> baseada em evidencias.

E precisa ser ? Só para si, e outros como vc.
Astrologia não é ciencia, não precisa ser baseada em evidencias.

>
> Você parece, as vezes, dizer que a astrologia é como uma forma de
> psicologia, para a auto-compreensão e que por isso independe de dados
> e ligações fisicas reais. Mas é só você, talvez o único astrologo do
> mundo, que diz algo assim. E mesmo como auto-analise, auto-descoberta,
> a astrologia tem pouco a apresentar como evidencia de eficácia.

1) Eu não sou astrologo
2)Não sou o unico que afirma isso.
3) Astrologia não é uma forma de psicologia, pois isso implica não pode
ser auto-aplicada, e so pode ser aplica a pessoas. A astrologia pode ser
auto-aplicada. Além disso a astrologia pode ser aplicada a qq evento, e
não apenas à vida das pessoas.
4) Vc acha que não tem nada a oferecer, pq vc nunca a estudou. Não
conheço ninguem que a tenha estudado e tenha saido pensando o mesmo que
pensava antes.
5) Muitas coisas na vida dos seres humanos não apresentam eficácia e
mesmo asssim as fazemos. Isso, não é argumento. Como por exemplo, eu
tentar fazer-vos entender o vosso erro cientifico e falta de rigor em
seguir o metodo cientifico no que toca às vossas opiniões em relação à
astrologia.. entre outras coisas...

Sergio Taborda




--
No virus found in this outgoing message.
Checked by AVG Anti-Virus.
Version: 7.0.300 / Virus Database: 265.8.5 - Release Date: 03-02-2005



##### ##### #####

Para saber mais visite
http://www.ciencialist.hpg.ig.com.br


##### ##### ##### #####


Yahoo! Grupos, um serviço oferecido por:

São Paulo Rio de Janeiro Curitiba Porto Alegre Belo Horizonte Brasília




------------------------------------------------------------------------------
Links do Yahoo! Grupos

a.. Para visitar o site do seu grupo na web, acesse:
http://br.groups.yahoo.com/group/ciencialist/

b.. Para sair deste grupo, envie um e-mail para:
ciencialist-unsubscribe@yahoogrupos.com.br

c.. O uso que você faz do Yahoo! Grupos está sujeito aos Termos do Serviço do Yahoo!.



[As partes desta mensagem que não continham texto foram removidas]



SUBJECT: Re: [ciencialist] Re: Zodiaco
FROM: "Oraculo" <oraculo@atibaia.com.br>
TO: <ciencialist@yahoogrupos.com.br>
DATE: 06/02/2005 01:21

Olá Taborda

Taborda: E para estabelecer esse sistema, e explicar pq eles não se
vinculam com as constelações é preciso matemática, pois é com base nela
que o sistema é criado, ao contrário do q vc pensa."

Não, o que eu penso é que, da forma como se criou a astrologia, com bases de conhecimento equivocadas quantoa causa e feito, ela não funciona como alega. Se minha matematica está incorreta, isso em nada modifica a conclusão sobre a astrologia.

Mesmo assim, a agrande maioria dos astrologos usa o sistema de astros, liga suas interpretações a constelações, faz previsões baseadas em posições de planetas e diversos outros tipos de analise astrolgogica. Apenas porque você tem a única e verdadeira astrologia, não é por isso que os argumentos contrários a ela estão incorretos.

Taborda: Ninguem está discutindo a eficácia da astrologia.

Bem, se isso fosse verdade, foi um enorme desperdicio de tempo e esforço..:-)

Veja, se você, mesmo afirmando que somos tolos enganados e que nada sabemos sobre astrologia, conclui que ela não funciona como alega (e que data e hora nãp permitem saber como a personalidade de alguém será), a discussão acaba aqui..:-) Vou até reconhecer minha ignorancia a respeito dos calculos matematicos que geram os mapas astrais, e concordar com você, a astrologioa não tem eficácia nenhuma, da forma como ela alega ter..:-)

Mas, desde o começo, estamos, me aprece, debatendo a eficaia sim, da astrologia. Não parece razoável que tenah se dado a este trabalho todo apenas apra demonstrar que as bases da astrologia são diferentes, embora igualmente incorretas e irreais.

Homero: Determinar setores do céu, acreditando que estes influenciam a vida na
Terra, precisamente, nada indica que essa influencia era real.

Taborda: Vê, se vc soubesse o minimo, não dizia essas asneiras. Os signos não
influenciam em coisa nenhuma.

Saco Taborda (desculpe..:-), não falei em signos, falei setores, o mesmo termo que usou no inicio do debate apra se referir ao sistema astrologico. Se os planetas determinam a influencia e os setores são usados para determinar sua posição, ou se são os proprios setores que o fazem não importa..:-) Importam as regras "a priori" definidas e que determinam quem será o que de acordo com sua data e hora de nascimento..:-)

Não são asneiras, são argumentos. Evitar responder xinagndo o coitado do argumento não ajuda em nada..:-)

Taborda: Pois é. Mas ninguem está defendendo a astrologia. Eu estou atacando a
vossa incultura. A vossa falta de conhecimentos tecnicos de astrologia (
e de fisica, matemática e astronomia) .
Principalmente estou atacando argumentos imbecis, fajutos e hipocritas
que são levantandos aqui periodicamente.

Bem, se não está defendendo a astrologia, a lista ficará chocada com o engano..:-) E o ataque (reconhecido) ad hominem não vai resolver a questão. Ficaria mais elegante e sincero se dissesse claramente que sou imbecil, fajuto, hipócrita e inculto, ao invés de desancar meus argumentos..:-) Argumentos devem ser refutados, não ofendidos.:-)

Taborda: Não. É tão fisicamente correcto afirmar que a terra gira em torno do
sol, como que o sol em torno da terra. É isso que a Relatividade nos
ensina.
Se vc quer ser completamente corrrecto, nenhum dos dois gira em torno do
outro, pois ambos giram em torno do centro de massa do sistema.

Aceito o segundo argumento, mas não o primeiro. Se fossem equivalentes, não haveria o problema do movimento retrogrado. Se a Terra estivesse no centro de massa (ou próxima dele, como o Sol está) e todos os outros astros girassem em torno dela, não haveria movimento retrogrado aparente dos planetas. Portanto, não é correto equiparar as duas afirmações, e nem elas são equivalentes.

Tomemos outro exemplo, o carrossel do parque infantil. Se minha filha está no carrossel, com um sorvete na mão, seria diferente se o carrosssel girasse e a Terra estivesse parada ou se a Terra girasse em torna do carrossel, e este parado? A relatividade se aplicaria e os dois modos seriam indistinguiveis?

Bem, se minha filha soltar o sorvete, e o carrossel estiver girando e a Terra parada, este vai voar longe, apra fora do carrossel. Mas, se a Terra estiver girando e o carrossel parado, o sorvete cairá no chão, ao lado de minha filha e ela saberá a diferença.

O problema da relatividade e da equivalencia é mais propriamente exemplificado, me parece (sou cuidadoso com certezas, sempre se pode falar uma bobagem sem perceber..:-), como elevador em queda. Se minha filha estivesse dentro de um elevador e este subitametne perdesse a sensação de peso, não seria possível dizer se o cabno foi cortado, ou se a Terra havia sumido (e sua atração gravitacional com ela). Nesse caso, tanto faz se é o elevador que cai ou se a Terra some.

Mas, o que tudo isso tem a ver com saber minha data e hora de nascimento e conhecer minha personalidade???

Taborda: Sim. So que a crença é a sua. O facto de eu saber o que é astrologia,
não significa que eu acredite nela.

risos... Essa me pegou desprevenido..:-) Realmente, não sei como perdemos tanto tempo se nós dois não acreditamos na astrologia..:-)

Homero

[As partes desta mensagem que não continham texto foram removidas]



SUBJECT: E-Books de Fisiologia em CD
FROM: "neurophys" <jbrasil@unb.br>
TO: ciencialist@yahoogrupos.com.br
DATE: 06/02/2005 08:48


Aproveite a promoção de férias do Site da Neurofisiologia e Fisiologia
Endócrina: os E-Books didáticos em CD-ROM estão com 50% de desconto
até 28/02/2005!
Esses E-Books são ideais para apresentação com Data-Show, além de
conterem, de forma condensada, todos os tópicos dos cursos dessas
disciplinas a nível de graduação universitária. O material é de
autoria do Prof. J. P. Brasil-Neto, da Universidade de Brasília.
Para aproveitar a oferta, é necessário acessar o site da
Neurofisiologia e Fisiologia Endócrina, clicar no link da Loja
Online,e fazer o pedido normalmente; ao receber a confirmação do seu
pedido, você ganhará o desconto!
O site da Neurofisiologia e Fisiologia Endócrina pode ser acessado em
http://neurofisiologia.cjb.net

Bom Carnaval e Boas Férias!





SUBJECT: Re: [ciencialist] Zodiaco
FROM: JVictor <jvoneto@uol.com.br>
TO: ciencialist@yahoogrupos.com.br
DATE: 06/02/2005 14:37

Homero,

Vale a pena ler. Não conheço ou autor, mas está interessante.


*/...o que está embaixo é como o que está em cima.../** - Hermes
Trimegisto*

*"A astrologia, do modo que é praticada atualmente (seja na sua
forma tradicional ou psicológica), não é de qualquer relevância
na compreensão de nós mesmos ou de nosso lugar no cosmo. Seus
defensores modernos não são capazes de explicar qual o
fundamento das associações astrológicas com as questões
terrenas, não têm qualquer explicação plausível para suas
alegações e não contribuíram com nenhum conhecimento de valor
para qualquer campo das ciências sociais. Além disso, a
astrologia não tem os recursos teóricos/conceituais para
resolver adequadamente seus próprios problemas internos ou
anomalias externas, ou para se decidir entre alegações ou
sistemas astrológicos conflitantes." --I.W. Kelly, **/Modern
Astrology: A critique/**, p. 931.*

*"O fato de ser incrível não deveria ser motivo para se
descartar a possibilidade de que uma busca suficientemente longa
revelasse um grão dourado de verdade na superstição
astrológica." -- Johannes Kepler*

*If you want to know who controls your soul
don't look to Mars or Venus;
Look instead to the President
for the danger's in his penis.
*/*[Se quer saber quem controla sua alma
não olhe para Marte ou Vênus;
Em vez disso, olhe para o presidente
pois o perigo está em seu pênis.]*/*
[autor desconhecido para mim]*

A astrologia, em sua forma tradicional, é um método de adivinhação
<http://brazil.skepdic.com/adivinhacao.html> baseado na teoria de que as
posições e movimentos dos corpos celestes (estrelas, planetas, sol e
lua), no momento do nascimento, influenciam profundamente a vida da
pessoa. Na sua forma psicológica, a astrologia é um tipo de terapia da
Nova Era, usada para a auto-compreensão e a análise da personalidade.
(Este verbete se refere à astrologia tradicional. Consulte astroterapia
<http://brazil.skepdic.com/astroterapia.html> para uma discussão da
astrologia psicológica.)

A forma mais tradicional é a Astrologia de Signos Solares, tipo
encontrado em numerosos jornais diários que publicam horóscopos. O
horóscopo é um prognóstico astrológico. O termo também é usado para
descrever um mapa do zodíaco no momento do nascimento da pessoa. O
zodíaco se divide em doze zonas celestes, cada qual recebendo o nome de
uma constelação que originalmente coincidia com a zona (Touro, Leão,
etc.). Todas as trajetórias aparentes do sol, da lua e dos principais
planetas se encaixam dentro do zodíaco. Devido ao movimento da
precessão, os pontos de equinócio e solstício se moveram para o oeste
cerca de 30 graus nos últimos 2.000 anos. Assim, as constelações
zodiacais que receberam seus nomes na antiguidade não correspondem mais
aos segmentos do zodíaco representados por seus signos. Em resumo, se
você tivesse nascido na mesma hora do mesmo dia do ano há 2.000 anos,
teria nascido sob um signo diferente.

A astrologia ocidental tradicional pode ser dividida em tropical e
sideral. (Astrólogos de tradições não-ocidentais utilizam sistemas
diferentes.) O ano tropical, ou solar, é medido em relação ao sol e
corresponde ao tempo (365 dias, 5 horas, 48 minutos e 46 segundos) entre
equinócios vernais sucessivos. O ano sideral é o tempo (365 dias, 6
horas, 9 minutos e 9,5 segundos) necessário para que a Terra complete
uma órbita ao redor do sol relativa às estrelas. O ano sideral é mais
longo que o tropical devido à precessão dos equinócios, ou seja, o lento
deslocamento para o oeste dos pontos equinociais ao longo do plano da
eclíptica, a uma taxa de 50,27 segundos de arco por ano, resultante da
precessão do eixo de rotação da Terra. A astrologia sideral utiliza como
base a constelação real na qual o sol se localiza no momento do
nascimento. A tropical usa um setor de 30 graus do zodíaco como base.
Esta é a forma mais popular e baseia suas leituras na época do ano,
geralmente ignorando as posições relativas do sol e das constelações
entre si. A astrologia sideral é usada por uma minoria de astrólogos e
baseia suas leituras nas constelações próximas do sol no momento do
nascimento.

Uma das defesas comuns a favor da astrologia é o argumento falacioso da
popularidade e tradição: bilhões de pessoas no mundo acreditam na
astrologia e ela tem sobrevivido por milhares de anos. Essas afirmações
são verdadeiras, mas irrelevantes como prova da "veracidade" da
astrologia. Os antigos caldeus e assírios se envolveram com a
adivinhação astrológica há cerca 3.000 anos. Em torno de 450 AEC, os
babilônios tinham desenvolvido o zodíaco de 12 signos, mas foram os
gregos --do tempo de Alexandre, o grande, até a conquista pelos
romanos-- que forneceram a maioria dos elementos fundamentais da
astrologia moderna.

A disseminação das práticas astrológicas foi contida pela ascensão do
cristianismo, que enfatizava a intervenção divina e o livre arbítrio. Na
renascença, a astrologia recuperou popularidade, em parte devido ao
ressurgimento do interesse pela ciência e pela astronomia. Teólogos
cristãos, no entanto, combateram a astrologia e, em 1585, o Papa Sisto V
a condenou. Na mesma época, os trabalhos de Kepler e outros
enfraqueceram os princípios astrológicos.

*A astrologia é testável?*

Um segundo argumento a favor da astrologia é o de que ela é testável e
que há indícios de que os dados apóiem a hipótese de uma conexão causal
entre os corpos celestes e os eventos humanos. Por exemplo, de acordo
com o assim chamado Efeito Marte
<http://brazil.skepdic.com/efmarte.html>, grandes atletas são natos, não
feitos. Essa afirmação é baseada numa análise estatística das datas de
nascimento de grandes atletas e a posição de Marte quando nascem. Diz-se
que a correlação é maior do que o que se esperaria pelo acaso. Outros
discordam e alegam que os indícios não mostram uma correlação que não
seria esperada pelo acaso. No entanto, mesmo se houvesse uma correlação
significativa entre a posição de Marte na data do nascimento e o fato da
pessoa se tornar um atleta excepcional, isso não implicaria ou mesmo
indicaria existir uma conexão causal entre a posição de um planeta e o
tipo de atividade em que a pessoa se daria bem na Terra. A correlação
entre x e y não é uma condição suficiente para a crença racional de que
x causa y. Mesmo uma correlação estatisticamente significativa entre x e
y não é uma condição suficiente para uma crença racional numa correlação
causal, muito menos para a crença de que x causa y. Correlação não prova
causalidade.

Embora não prove, a correlação é extremamente atraente para os
defensores da astrologia. Por exemplo: "Entre 3.458 soldados, Júpiter é
encontrado 703 vezes, nascendo ou culminando quando eles nascem. As leis
das probabilidades prevêem que deveriam ser 572. As chances disso
acontecer: uma em um milhão" (Gauquelin). Estou disposto a assumir que
todos os dados estatísticos que mostram uma correlação significativa
entre diversos planetas nascendo, se pondo, culminando, ou o que quer
que se possa vê-los fazer, é acurada. No entanto, seria mais
surpreendente se entre todos os bilhões e bilhões de movimentos celestes
concebíveis não houvesse uma grande parte que pudesse ser
significativamente correlacionada com dezenas de eventos em massa ou
traços de personalidade individuais.

Por exemplo, os defensores da astrologia gostam de observar que 'a
duração do ciclo menstrual da mulher corresponde às fases da lua' e que
'os campos gravitacionais do sol e da lua são fortes o bastante para
causar a subida ou a descida das marés na Terra.' Se a lua pode afetar
as marés, então pode certamente afetar uma pessoa. Mas o que existe de
análogo às marés numa pessoa? Somos lembrados de que os seres humanos
começam a vida num mar amniótico e de que o corpo humano é 70% água! Se
as ostras abrem e fecham as conchas de acordo com as marés, as quais
fluem de acordo com as forças eletromagnéticas e gravitacionais do sol e
da lua, e se os seres humanos estão cheios de água, não é então óbvio
que devam ser influenciados pela lua também? Pode ser óbvio, mas os
indícios vindos dos estudos da lua
<http://brazil.skepdic.com/luacheia.html> não confirmam isso.

Os astrólogos dão ênfase à importância das posições do sol, da lua, dos
planetas, etc. no instante do nascimento. Mas por que as condições
iniciais seriam mais importantes para a personalidade e as
características de uma pessoa que todas as condições subseqüentes? Por
que seria escolhido como o momento crucial o nascimento, e não a
concepção? Por que outras condições iniciais como a saúde da mãe, as
condições do local do parto, fórceps, luzes fortes, sala escura, banco
traseiro de um automóvel, etc., não seriam mais importantes do que se
Marte está ascendendo, descendendo, culminando ou fulminando? Por que o
planeta Terra, muito mais próximo de nós no nascimento, não seria
considerado uma influência importante no que somos e no que nos tornamos?

Além do Sol e da Lua, e de algum cometa ou asteróide passando
ocasionalmente, a maioria dos objetos planetários está demasiado
distante de nós. Qualquer influência que pudessem ter sobre o nosso
planeta seria encoberta pelas do Sol e da Lua. É muito mais provável que
a Terra, e as pessoas e coisas com as quais se tem contato direto, sejam
fatores de influência mais importantes nas nossas vidas que distantes
corpos celestes. Além disso, se for verdade que podemos determinar
efeitos específicos a partir de condições específicas do local do
nascimento, então podemos controlar essas condições de forma a trazer
resultados benéficos. Por outro lado, mesmo se for verdade que a posição
das estrelas e planetas seja mais importante para a vida da pessoa do
que um nascimento difícil e passado sob condições horrendas, não há nada
que possamos fazer em relação à posição das estrelas, e há um limite no
controle que podemos ter sobre o momento do nascimento de uma pessoa.
(Ainda bem que não vou ser um astrólogo na era dos bebês de proveta.
Como eu iria saber quando o meu cliente 'nasceu'? O processo do
nascimento não é instantâneo. Não existe um momento único no qual a
pessoa nasce. O fato de que algum funcionário escreva em algum lugar um
horário do nascimento é irrelevante. Será que escolhem o momento em que
a bolsa se rompe? O momento em que ocorre a primeira dilatação? Quando o
primeiro fio de cabelo ou unha do pé aparece? Quando a última unha do pé
ou fio de cabelo ultrapassa o último milímetro da vagina ou da
superfície da barriga? Quando se corta o cordão umbilical? Quando se faz
a primeira respiração? Ou o momento em que o médico ou a enfermeira
olham para um relógio de parede ou de pulso [sem dúvida magicamente
livre da possibilidade de imprecisão] para anotar o momento do nascimento?)

Ninguém diria que, para que se compreendesse o efeito da lua sobre as
marés ou sobre as batatas, fosse preciso entender as condições inicial
da singularidade que precedeu o Big Bang, ou a posição das estrelas no
momento em que as batatas foram colhidas. Para que se saiba a respeito
da maré baixa de amanhã, não é preciso saber onde estava a lua quando o
primeiro oceano ou rio se formou, ou se os oceanos vieram primeiro e a
lua depois, ou vice-versa. Condições iniciais são menos importantes que
as presentes para que se compreendam efeitos atuais sobre rios e
plantas. Se isso vale para marés e plantas, por que não valeria para as
pessoas?

*correlação não é causalidade*

Esse fascínio pelas correlações também é encontrado no raciocínio dos
que tentam transformar todo sítio megalítico antigo em algum tipo de
observatório astronômico. Os defensores da astrologia deveriam observar
o que Aubrey Burl escreveu a respeito desse tipo de raciocínio.

… são grandes as probabilidades de que ocorra fortuitamente uma
boa linha de visão celeste em quase qualquer círculo. Examine-se
um sítio como Grey Croft, em Cumberland,... 27,1 x 24,4 m de
diâmetro, com doze pedras e uma extra, parecem haver tantas
linhas e tantos alvos possíveis que seria improvável não
descobrir nada (Burl, 50).

Além disso, embora seja verdade que as chances de se passar 20 vezes
seguidas num jogo de dados sejam inconcebivelmente baixas, isso já
aconteceu. Havendo jogos suficientes, o inconcebível se torna freqüente.
Em resumo, o que parece desafiar as "leis" da estatística pode não estar
realmente o fazendo, quando se examina com mais cuidado.

Para concluir, há aqueles que defendem a astrologia argumentando com o
quanto os horóscopos profissionais acertam. Um colega, professor de
história da Universidade da Califórnia em Davis, com título de Ph.D,
pratica a astrologia. Naturalmente, possui tecnologia e tem um programa
de computador para ajudá-lo a fazer as leituras. Conhece todos os
argumentos contra a astrologia e até admite que logicamente ela não
deveria funcionar. Mas funciona, acredita ele. Esse conceito de
'funciona' é curioso. O que será que significa?

Basicamente, dizer que a astrologia /funciona/ quer dizer que há muitos
consumidores satisfeitos. /Não/ significa que seja precisa na predição
do comportamento humano ou de eventos num grau significativamente maior
que o da pura sorte. O principal apoio a esse argumento aparece na forma
de relatos e testemunhos <http://brazil.skepdic.com/testemunhos.html>.
Há muitos clientes satisfeitos que acreditam que os horóscopos os
descrevem com precisão, e que os astrólogos lhes deram bons conselhos.
Esse tipo de indício não comprova a astrologia tão bem quanto demonstra
os efeitos da leitura a frio
<http://brazil.skepdic.com/leiturafria.html>, o efeito Forer
<http://brazil.skepdic.com/forer.html>, e a predisposição para a
confirmação <http://brazil.skepdic.com/confirma.html>. Bons astrólogos
dão bons conselhos, mas isso não valida a astrologia. Vários estudos
mostraram que as pessoas usam o pensamento seletivo para fazer com que
qualquer carta astrológica que apresentarem a elas se encaixe em suas
idéias preconcebidas sobre si mesmas. Muitas das afirmações que são
feitas a respeito de signos e personalidades são vagas, e se encaixariam
em muitas pessoas de diferentes signos. Até mesmo astrólogos
profissionais, a maioria dos quais despreza a Astrologia de Signos
Solares, não é capaz de escolher uma leitura de horóscopo correta num
índice de acertos maior que o esperado pelo acaso. Mesmo assim, a
astrologia continua mantendo a popularidade, apesar de não haver um
mínimo de comprovação científica a seu favor. Mesmo a primeira dama dos
Estados Unidos, Nancy Reagan, e seu marido Ronald, quando era o líder do
mundo livre, consultaram um astrólogo
<http://www.parascope.com/articles/0497/reagan01.htm>, o que me leva a
concluir que /*os astrólogos têm mais influência que as estrelas*/*/./*

Será possível que eu seja quem sou devido à posição dos planetas,
estrelas, luas, cometas, asteróides, quasares, buracos negros, etc., no
momento do meu nascimento? Sim, é possível. Será que eu tenho alguma
razão para achar que isso é mais provável que o oposto, ou seja, de que
essas questões sejam insignificantes e irrelevantes para o meu
'destino'? Não. Não consigo encontrar sequer uma única boa razão para
acreditar em nada disso. Mas eu sou taurino, e todos sabemos o quão
teimoso eu devo ser.


Sds,

Victor.







Oraculo escreveu:

> Olá Taborda
>
> Taborda: E para estabelecer esse sistema, e explicar pq eles não se
> vinculam com as constelações é preciso matemática, pois é com base nela
> que o sistema é criado, ao contrário do q vc pensa."
>
> Não, o que eu penso é que, da forma como se criou a astrologia, com
> bases de conhecimento equivocadas quantoa causa e feito, ela não
> funciona como alega. Se minha matematica está incorreta, isso em nada
> modifica a conclusão sobre a astrologia.
>
> Mesmo assim, a agrande maioria dos astrologos usa o sistema de astros,
> liga suas interpretações a constelações, faz previsões baseadas em
> posições de planetas e diversos outros tipos de analise astrolgogica.
> Apenas porque você tem a única e verdadeira astrologia, não é por isso
> que os argumentos contrários a ela estão incorretos.
>
> Taborda: Ninguem está discutindo a eficácia da astrologia.
>
> Bem, se isso fosse verdade, foi um enorme desperdicio de tempo e
> esforço..:-)
>
> Veja, se você, mesmo afirmando que somos tolos enganados e que nada
> sabemos sobre astrologia, conclui que ela não funciona como alega (e
> que data e hora nãp permitem saber como a personalidade de alguém
> será), a discussão acaba aqui..:-) Vou até reconhecer minha ignorancia
> a respeito dos calculos matematicos que geram os mapas astrais, e
> concordar com você, a astrologioa não tem eficácia nenhuma, da forma
> como ela alega ter..:-)
>
> Mas, desde o começo, estamos, me aprece, debatendo a eficaia sim, da
> astrologia. Não parece razoável que tenah se dado a este trabalho todo
> apenas apra demonstrar que as bases da astrologia são diferentes,
> embora igualmente incorretas e irreais.
>
> Homero: Determinar setores do céu, acreditando que estes influenciam a
> vida na
> Terra, precisamente, nada indica que essa influencia era real.
>
> Taborda: Vê, se vc soubesse o minimo, não dizia essas asneiras. Os
> signos não
> influenciam em coisa nenhuma.
>
> Saco Taborda (desculpe..:-), não falei em signos, falei setores, o
> mesmo termo que usou no inicio do debate apra se referir ao sistema
> astrologico. Se os planetas determinam a influencia e os setores são
> usados para determinar sua posição, ou se são os proprios setores que
> o fazem não importa..:-) Importam as regras "a priori" definidas e que
> determinam quem será o que de acordo com sua data e hora de
> nascimento..:-)
>
> Não são asneiras, são argumentos. Evitar responder xinagndo o coitado
> do argumento não ajuda em nada..:-)
>
> Taborda: Pois é. Mas ninguem está defendendo a astrologia. Eu estou
> atacando a
> vossa incultura. A vossa falta de conhecimentos tecnicos de astrologia (
> e de fisica, matemática e astronomia) .
> Principalmente estou atacando argumentos imbecis, fajutos e hipocritas
> que são levantandos aqui periodicamente.
>
> Bem, se não está defendendo a astrologia, a lista ficará chocada com o
> engano..:-) E o ataque (reconhecido) ad hominem não vai resolver a
> questão. Ficaria mais elegante e sincero se dissesse claramente que
> sou imbecil, fajuto, hipócrita e inculto, ao invés de desancar meus
> argumentos..:-) Argumentos devem ser refutados, não ofendidos.:-)
>
> Taborda: Não. É tão fisicamente correcto afirmar que a terra gira em
> torno do
> sol, como que o sol em torno da terra. É isso que a Relatividade nos
> ensina.
> Se vc quer ser completamente corrrecto, nenhum dos dois gira em torno do
> outro, pois ambos giram em torno do centro de massa do sistema.
>
> Aceito o segundo argumento, mas não o primeiro. Se fossem
> equivalentes, não haveria o problema do movimento retrogrado. Se a
> Terra estivesse no centro de massa (ou próxima dele, como o Sol está)
> e todos os outros astros girassem em torno dela, não haveria movimento
> retrogrado aparente dos planetas. Portanto, não é correto equiparar as
> duas afirmações, e nem elas são equivalentes.
>
> Tomemos outro exemplo, o carrossel do parque infantil. Se minha filha
> está no carrossel, com um sorvete na mão, seria diferente se o
> carrosssel girasse e a Terra estivesse parada ou se a Terra girasse em
> torna do carrossel, e este parado? A relatividade se aplicaria e os
> dois modos seriam indistinguiveis?
>
> Bem, se minha filha soltar o sorvete, e o carrossel estiver girando e
> a Terra parada, este vai voar longe, apra fora do carrossel. Mas, se a
> Terra estiver girando e o carrossel parado, o sorvete cairá no chão,
> ao lado de minha filha e ela saberá a diferença.
>
> O problema da relatividade e da equivalencia é mais propriamente
> exemplificado, me parece (sou cuidadoso com certezas, sempre se pode
> falar uma bobagem sem perceber..:-), como elevador em queda. Se minha
> filha estivesse dentro de um elevador e este subitametne perdesse a
> sensação de peso, não seria possível dizer se o cabno foi cortado, ou
> se a Terra havia sumido (e sua atração gravitacional com ela). Nesse
> caso, tanto faz se é o elevador que cai ou se a Terra some.
>
> Mas, o que tudo isso tem a ver com saber minha data e hora de
> nascimento e conhecer minha personalidade???
>
> Taborda: Sim. So que a crença é a sua. O facto de eu saber o que é
> astrologia,
> não significa que eu acredite nela.
>
> risos... Essa me pegou desprevenido..:-) Realmente, não sei como
> perdemos tanto tempo se nós dois não acreditamos na astrologia..:-)
>
> Homero
>
> [As partes desta mensagem que não continham texto foram removidas]
>
>
>
> ##### ##### #####
>
> Para saber mais visite
> http://www.ciencialist.hpg.ig.com.br
>
>
> ##### ##### ##### #####
>
>
> *Yahoo! Grupos, um serviço oferecido por:*
> <http://br.rd.yahoo.com/SIG=12ajkclst/M=264379.5078783.6203979.1588051/D=brclubs/S=2137111528:HM/EXP=1107746528/A=2191897/R=0/SIG=10vqa2grn/*http://br.diversao.yahoo.com/>
>
> <http://br.rd.yahoo.com/SIG=12ajkclst/M=264379.5078783.6203979.1588051/D=brclubs/S=2137111528:HM/EXP=1107746528/A=2191897/R=1/SIG=10vqa2grn/*http://br.diversao.yahoo.com/>
>
>
>
> ------------------------------------------------------------------------
> *Links do Yahoo! Grupos*
>
> * Para visitar o site do seu grupo na web, acesse:
> http://br.groups.yahoo.com/group/ciencialist/
> * Para sair deste grupo, envie um e-mail para:
> ciencialist-unsubscribe@yahoogrupos.com.br
> <mailto:ciencialist-unsubscribe@yahoogrupos.com.br?subject=Unsubscribe>
> * O uso que você faz do Yahoo! Grupos está sujeito aos Termos do
> Serviço do Yahoo! <http://br.yahoo.com/info/utos.html>.
>
>
>
>
> __________ Informação do NOD32 1.990 (20050202) __________
>
> Esta mensagem foi verificada pelo NOD32 Sistema Antivírus
> http://www.nod32.com.br




SUBJECT: Magnetismo
FROM: Lidiane Pedersoli <lidipicnic@yahoo.com>
TO: ciencialist@yahoogrupos.com.br
DATE: 06/02/2005 15:08

O que ocasiona o distanciamento entre as linhas de indu��o de um im� natural? Antecipadamente agrade�o, Lidiane Santos.


---------------------------------
Do you Yahoo!?
Yahoo! Search presents - Jib Jab's 'Second Term'

[As partes desta mensagem que n�o continham texto foram removidas]



SUBJECT: Re: [ciencialist] Energia "Verde" J�
FROM: "Dino" <nevillex@ig.com.br>
TO: <ciencialist@yahoogrupos.com.br>
DATE: 06/02/2005 15:28

Ol� !

>>From: Tipoalgo Subject: [ciencialist] Energia "Verde" J�
>>1- O deputado Gabeira e seus pares deveriam ser informados
sobre a
>>besteira que � o hidrog�nio, pois sua produ��o e
armazenamento requer
>>4 vezes mais energia do que a ent�o obtida. Nunca produzir
hidrog�nio
>>vai ser barato, limpo e eficiente. NUNCA o homem vai
conseguir imitar
>>a fant�stica natureza com sua espetacular, graciosa e
ignorada fotoss�ntese.

>>O Sr. Fendel est� correto? Quais as ressalvas?
-----------------------------

B�sicamente o sr. Fendel est� correto. A resalva que fa�o se
refere ao par�grafo acima.
Primeiramente, NUNCA significa tempo demais ... acho a
afirma��o exagerada.
Por outro lado, h� que contextualisar a quest�o a nossas
condi��es particulares.

Em termos absolutos, o custo do hidrog�no � alto se formos
levar em conta o investimento espec�fico para sua produ��o,
armazenamento, distribui��o e uso comercial. Entretanto, h�
outros aspectos e benef�cios indiretos, incluindo quest�es
estrat�gicas, que devem ser levados em conta.
O c�lculo de custo dos bens sociais � complexo e geralmente
os estr�tegos economicos usam valores absurdamente
simplificados para definirem a��es administrativas, cujos
resultados s�o normalmente frustrantes.

Exemplificando, pensemos no custo atual do barril de
petr�leo importado que � de 40 dolares em m�dia. Esse valor
� usado como paradigma para definir o custo de outros
produtos, como o alcool nacional.

Mas seria esse um valor adequado ?
Acredito que n�o, pois se considerarmos que o fonecimento
dos d�lares para a compra de petr�leo pela Petrobr�s, que �
feito pelo Banco Central a partir de emprestimos
internacionais de longo prazo, vamos supor, com juros m�dios
de 10 % a.a, linearmente em 20 anos teremos 200 % de juros
ou seja, para a na��o o custo total chegaria a 120 dolares
por barril. O custo do barril equivalente de alcool nacional
anda ao redor de 25 a 30 dolares, e esse pre�o foi usado
para o desmantelamento da produ��o nacional de alcool na
�poca em que o barril de petr�leo custava (aparentemente) 20
a 23 dolares.

No caso do hidrog�nio produzido por eletr�lise de �gua o
maior problema est� na armazenagem dos produtos.
O oxig�nio pode ser liquefeito a baixas temperatura,
ocupando volumes reduzidos, entretanto, o hidrog�nio n�o se
liquefaz e deve ser comprimido a 200 ou mais atm,
necessitando de cilindros resistentes para armazenagem e
transporte. O custo dos equipamentos de produ��o �
relativamente baixo, por�m os de armazenagem � alto.
O custo energ�tico da compress�o dos gases tambem � elevado.
Por�m, considerando apenas o caso da usina hidroel�trica de
Itaipu, que no momento despeja quase 20 % da vas�o do rio
livremente por uma de suas comportas, ou seja, quase 1
gigawats vai para o ralo sem aproveitamento. Ora, se essa
energia (e das centenas de outra hidroel�tricas que tamb�m
despejam energia inaproveitada) pudesse ser convertida em
hidrog�niom, n�o importa a efici�ncia do processo -
qualquer que fosse, resultaria em lucro.

O assunto � complexo, mas merece um estudo adequado, pois a
enorme quantidade de energia atualmente desperdi�ada indica
que o assunto deveria ter sido considerado h� decadas.

Abra�os / villem


[As partes desta mensagem que n�o continham texto foram removidas]



SUBJECT: Fw: �gua
FROM: "Luiz Ferraz Netto" <leobarretos@uol.com.br>
TO: "ciencialist" <ciencialist@yahoogrupos.com.br>
DATE: 06/02/2005 16:54

Olha a idade!
[]'
===========================
Luiz Ferraz Netto [Léo]
leobarretos@uol.com.br
http://www.feiradeciencias.com.br
===========================
-----Mensagem Original-----
De: haroldo hirano
Para: leobarretos@uol.com.br
Enviada em: domingo, 7 de janeiro de 2001 12:20
Assunto: água


"por que a água é molhada?"
Débora ayumi hirano
8 anos segunda série
o e-mail é da mamãe
mirihirano@hotmail.com
Nas dúvidas experimentais, por gentileza coloque aqui o endereço da página, isso facilita o confronto. Agradeço. Meu nome é LUIZ FERRAZ NETTO, meu apelido é LÉO e moro em BARRETOS; dai vem meu e-mail: leobarretos@uol.com.br.


--------------------------------------------------------------------------------


No virus found in this incoming message.
Checked by AVG Anti-Virus.
Version: 7.0.300 / Virus Database: 265.8.5 - Release Date: 03/02/2005

----------

No virus found in this outgoing message.
Checked by AVG Anti-Virus.
Version: 7.0.300 / Virus Database: 265.8.5 - Release Date: 03/02/2005


[As partes desta mensagem que não continham texto foram removidas]



SUBJECT: Re: [ciencialist] Magnetismo
FROM: "Luiz Ferraz Netto" <leobarretos@uol.com.br>
TO: <ciencialist@yahoogrupos.com.br>
DATE: 06/02/2005 17:10

Olá Lidiane,
"O que ocasiona o distanciamento entre as linhas de indução de um imã natural? "

pergunta interessante.

As linhas de indução são utilizadas para a visualização ('materialização') da indução magnética (B) [campo magnético no interior do meio envolvente]. No vácuo, seriam as linhas de força do campo magnético H. Tem-se B = u.H, com u sendo a permeabilidade magnética do meio.
Quanto mais intenso o campo mais próximas estarão as linhas numa dada região pré-fixada. No tal meio, o campo de indução é resultante do campo magnetizante e da magnetização do meio (devido aos seus domínios magnéticos).
Assim, se vc tiver um ímã em forma de U e colocar uma armadura de ferro fechando o U (essa armadura passa a ser o meio substituto do ar que lá existia), todas as linhas se concentram e passam pelo interior da armadura (devido à magnetização desta) --- praticamente não haverá linha alguma fora do ímã fechado pela armadura.

[]'
===========================
Luiz Ferraz Netto [Léo]
leobarretos@uol.com.br
http://www.feiradeciencias.com.br
===========================
-----Mensagem Original-----
De: "Lidiane Pedersoli" <lidipicnic@yahoo.com>
Para: <ciencialist@yahoogrupos.com.br>
Enviada em: domingo, 6 de fevereiro de 2005 15:08
Assunto: [ciencialist] Magnetismo



O que ocasiona o distanciamento entre as linhas de indução de um imã natural? Antecipadamente agradeço, Lidiane Santos.


---------------------------------
Do you Yahoo!?
Yahoo! Search presents - Jib Jab's 'Second Term'

[As partes desta mensagem que não continham texto foram removidas]



##### ##### #####

Para saber mais visite
http://www.ciencialist.hpg.ig.com.br


##### ##### ##### #####
Links do Yahoo! Grupos










--
No virus found in this incoming message.
Checked by AVG Anti-Virus.
Version: 7.0.300 / Virus Database: 265.8.5 - Release Date: 03/02/2005




--
No virus found in this outgoing message.
Checked by AVG Anti-Virus.
Version: 7.0.300 / Virus Database: 265.8.5 - Release Date: 03/02/2005



SUBJECT: Primeira chamada para revista / Call for papers
FROM: "Luis" <luis.alcides@he.com.br>
TO: ciencialist@yahoogrupos.com.br
DATE: 06/02/2005 18:42


O Grupo Tchê Química convida os Professores e Pesquisadores que
quiserem publicar seus artigos na quarta edição do Periódico Tchê
Química para enviar seus artigos para avaliação até 29 de Junho de 2005.

Os trabalhos enviados para avaliação devem obedecer as regras abaixo:

INSTRUÇÕES PARA PUBLICAÇÃO
INSTRUCTIONS FOR PUBLICATIONS

1. O "Periódico Tchê Química" publica artigos inéditos em todos os
campos da química e áreas afins; artigos de revisão bibliográfica de
assuntos específicos; comunicações científicas e resenhas de livros.

1. "Periódico Tchê Química" publishes original work in all areas of
chemistry; review papers on specific subjects; short communications
and book reviews.



2. Artigos originais incluem um resumo com preferência à língua
inglesa, sob título de "Abstract", para expor, de modo claro e
compreensível o conteúdo do trabalho, sem generalidades e detalhes de
metodologia.

2. Original papers should include an abstract with preference written
in English, under the title "Abstract", explaining clearly the
contents of the article. This abstract should not contain general and
experimental details.



3. Os trabalhos devem ser submetidos para o endereço eletrônico e
tchequimica@tchequimica.com ou tchequimica@tchequimica.tk, em formato
Sun Microsystems Inc. OpenOffice.org® (preferencial) ou MS-Word for
Windows®. O cabeçalho é constituído pelo título do trabalho, nome(s)
do(s) autor(es) e respectivas instituições. As tabelas e figuras devem
ser inseridas no corpo do texto, em espaços adequados e perfeitamente
visíveis. A nomenclatura deve obedecer às normas da IUPAC.

3. Manuscript should be submetted to tchequimica@tchequimica.com or
tchequimica@tchequimica.tk, in Sun Microsystems Inc. OpenOffice.org®
(preference) or MS-Word for Windows®. Manuscriptheadings must include
title, author name(s) and instution (s) where the work was done.
Tables and figures must be sent in the text body appropriate and
visible. Nomenclature should conform to IUPAC rules.



4. Citações bibliográficas no texto devem ser fietas por nome e ano
entre colchetes. Alista de referências bibliográficas, ao fim do
trabalho, deve ser organizada por ordem alfabética seqüencialmente
numerada, seguindo ao seguinte modelo:
1. CAVALCANTE, A. A. M., Rübensam, G., Picada, J. N., da Silva, E. G.,
Moreira, J. C. F., Henriques, J. A. P., Mutagenicity, antioxidant
potential and antimutagenic activitity against hydrogen peroxide of
cashew (Anacardium occidentale) apple juice and Cajuina, Envaronmental
and Molecular Mutagenesis 41:5 (2003) 360-369;

4. Bibliographic citations in the txt must be done by author name and
year between brackets. The bibliographic lists, at the endo of the
text, must be organized by alphabetical order and sequential
numeration, obeyng the following model: 1. CAVALCANTE, A. A. M.,
Rübensam, G., Picada, J. N., da Silva, E. G., Moreira, J. C. F.,
Henriques, J. A. P., Mutagenicity, antioxidant potential and
antimutagenic activitity against hydrogen peroxide of cashew
(Anacardium occidentale) apple juice and Cajuina, Envaronmental and
Molecular Mutagenesis 41:5 (2003) 360-369;



5. O Conselho Editorial poderá fazer alterações no texto do trabalho,
para adequá-lo a estas instruções.

5. The Editorial Board may perform modifications in the text to make
adequate to these rules.


Dúvidas? Escreva para tchequimica@tchequimica.com, no campo assunto
coloque o título "duvida sobre publicação". Por favor não envie
arquivos anexados nesta mensagem. Ou escreva para o nosso Yahoo Group
em http://br.groups.yahoo.com/group/tchequimica.


Gratos pela atenção, Grupo Tchê Química. www.tchequimica.com





SUBJECT: p/Pablo - Zodiaco e Acupuntura e Homeopatia..:-)
FROM: "murilo filo" <avalanchedrive@hotmail.com>
TO: ciencialist@yahoogrupos.com.br
DATE: 06/02/2005 19:49

Pablo,
obrigado, gostei.
Esta informação foi a 1ª que obtive contra a AP, e já saquei q/vc tem razão
no '46'.
Existe um outro para sinusite '3', que é quase um milagre.
Pergunta: vc tem algo a dizer do '18'? Estou fazendo uma experiência...
abr/M.

>From: "Pablo Albino Pereira" <pablo@unidavi.edu.br>
>Reply-To: ciencialist@yahoogrupos.com.br
>To: ciencialist@yahoogrupos.com.br
>Subject: [ciencialist] Re: Zodiaco e Acupuntura e Homeopatia..:-)
>Date: Sat, 05 Feb 2005 22:58:36 -0000
>
>
>Amigos, só para esclarecer. O famoso complexo homeopático almeida
>prado 46, não é um medicamento homeopático. Até há alguuns anos ele
>continha fenolftaleína (em doses n homeopáticas), sendo que após a
>probição desta substância (fármaco???) foi colocado na formulação 5 mg
>de picosulfato de sódio por comprimido, que é um laxante de contato. O
>mesmo fármaco é encontrado no Guttalax, com cada comprimido contendo
>2,5 mg!!! Então, n é a toa que um comprimidinho do 46 seja tão
>efetivo. Na verdade, tão efetivo que contraria a lógica homeopática,
>simplesmente por ser um medicamento alopático disfarçado (a fórmula
>contém algumas diluições decimais homeopaticamente irracionais).
>
>Todo profissional homeopata (farmacêutico, médico, dentista, m.
>veterinário, etc) condena os AP.
>
>Isso só pra esclarecer que o exemplo do 46, utilizado pelo Murillo, n
>é adequado. N quero com isso afirmar que homeopatia não funciona, pois
>sabe-se que funciona. Apenas, n se sabe como, pois todas as hipotéses
>colcadas são, ainda, cientificamente insondáveis...mas esta é outra
>história.
> []'s Pablo
>
>--- Em ciencialist@yahoogrupos.com.br, "murilo filo"
><avalanchedrive@h...> escreveu
> > Oraculo.
> > Conheço algo de homeopatia só como experiência pessoal...
> > P/constipação há o famoso ''46'' do Almeida Prado, que é ''pá-pum!''
> > Para animais já lí o suficiente para saber que a HP não é algo p/ser
> > descartado cegamente.
> > Perdão pela comparação com o Quevedo! Achei que vc se esforça
>demais, faz
> > muita ginástica, para negar, destruir e ridicularizar trecos que não
> > deveriam ter importância para você e que estão fora do seu mundo.
> > Mas fica tranquilo, eu entendo, sei que vc gosta de provocar,
>polemizar e
> > argumentar. Eu também gosto, mas às vêzes tenho muita preguiça...
> > abr/M.
> >
> > >From: "Oraculo" <oraculo@a...>
> > >Reply-To: ciencialist@yahoogrupos.com.br
> > >To: <ciencialist@yahoogrupos.com.br>
> > >Subject: [ciencialist] Zodiaco e Acupuntura e Homeopatia..:-)
> > >Date: Sat, 5 Feb 2005 19:39:24 -0200
> > >
> > >Olá Murilo
> > >
> > >risos..:-) Bem, mesmo me chamando de padre Quevedo (o que me deixou
> > >assustado..:-), não refutou meus argumentos sobre a acupuntura..:-)
> > >Considero isso uma boa coisa, já que estes parecem ser eficazes em
> > >convence-lo (ou pelo menos a fazer você pensar sobre eles..:-)
> > >
> > >Mas a homeopatia segue o mesmo raciocinio. Ela tem definição
>precisa, ou
> > >deixa de ser a homeopatia. Veja, seu criador, Hahnemann, nada
>sabia sobre
> > >micro-organimos, infecção, sistema imunologico, etc. Sua criação
>curava
> > >doenças a partir de determinados principios. Ou se aceita os
>principios, ou
> > >se recusa o termo homeopatia. Eu acho que foi um avanço na época,
>já que a
> > >medicina usava sanguessugas, purgantes e coisas assim para tratar.
>Parar
> > >com esses "tratamentos" e esperar a doença se curar em geral era mais
> > >benéfico ao paciente..:-)
> > >
> > >Ele dizia, por exemplo, que doenças cronicas eram causadas por um
>tipo de
> > >espírito maligno, ou misma, a psora (como em psoriase). Devo levar
>essa
> > >afirmação também, junto ao resto das alegações da homeopatia? Ou posso
> > >filtrar, com alguma ferramenta racional, o que é correto de que é
> > >incorreto? E, se posso fazer isso, que ferramenta deve usar? E o
>que fazer
> > >se a ferramenta derrubar todas as afirmações, e não apenas as mais
> > >estranhas?
> > >
> > >Influenciar animais é possivel. Na verdade, para a maior parte dos
> > >mamiferos (principalmente os animais gregarios) a atençao e o cuidado
> > >pessoal traz benficios no bem estar. São animais, mas não são
>estupidos ou
> > >desprovidos de mente ou de processos mentais (filhotes de macacos
> > >abandonados pelas mães, preferem mães substitutas feitas de pelo e
>algodão,
> > >mas sem alimentos, do que mães substitutas feitas de arame, mas que
> > >poderiam alimenta-los). Tenho um amigo que, depois de tentar
>bastante o
> > >tratamento de seus cavalos e gado com homeopatia (principalmente
>para a
> > >perigosa constipação) desistiu por absoluta falta de efeitos. Esse
>relato,
> > >unico e anedotico, valida a tese que a homeopatia não funciona?
>Claro que
> > >não..:-) Tanto quanto um relato contrário, de efeito positivo, não
>valida a
> > >afirmação de que a homeopatia é real.
> > >
> > >Apenas façam com que a homeopatia passe em estudos controlados, e
>minha
> > >conclusão sobre sua eficácia muda..:-)
> > >
> > >Mas, enquanto ela falhar, e enquanto seu mecanismo de ação for
>inexistente
> > >(ou francamente incorreto), fico com a conclusão cientifica de que
>é um
> > >placebo, na melhor das hipoteses..:-)
> > >
> > >Um abraço
> > >
> > >Homero
> > > ----- Original Message -----
> > > From: murilo filo
> > > To: ciencialist@yahoogrupos.com.br
> > > Sent: Friday, February 04, 2005 11:06 PM
> > > Subject: RE: [ciencialist] Zodiaco e Acupuntura..:-)
> > >
> > >
> > > Oraculo, oi.
> > > Vc está parecendo o Padre Quevedo fazendo ginástica anti-espírito!
> > > Conseguí achar um negócio interessante sobre a homeopatia, que
>também
> > >foi
> > > ''atacada'' nesta discussão!
> > > FAVOR irem a www.arenales.com.br e lá encontrarão muita
> > >informação,
> > > onde especialmente destaco muita ciência e homeopatia para
>ANIMAIS. Um
> > >lance
> > > psicossomático, certamente.
> > > A arte de ''sugestionar'' animais é maravilhosa, não? :]]] abr/M.
> > >
> > > >From: "Oraculo" <oraculo@a...>
> > > >Reply-To: ciencialist@yahoogrupos.com.br
> > > >To: <ciencialist@yahoogrupos.com.br>
> > > >Subject: [ciencialist] Zodiaco e Acupuntura..:-)
> > > >Date: Fri, 4 Feb 2005 16:49:32 -0200
> > > >
> > > >Olá Esteban
> > > >
> > > >Há uma sequencia de enganos em suas colocações, talvez derivada
>de um
> > > >compromentimento com a crença inicial e uma certa aversão a
> > >ciência..:-).
> > > >Métodos usados na pesquisa cientifica não servem apenas para
>impedir
> > >que
> > > >alegações reais, mas incomodas, sejam demonstradas. Servem para
>filtar
> > > >alegações sem base na realidade.
> > > >
> > > >Assim, acusar os que pesqusiasm com ironias do tipo "santos céticos
> > > >imparcialistas" é bobagem e uma falacia "ad hominem". A não ser
>que
> > >credite
> > > >tudo a Grande Conspiração Mundial Da Ciência Para Esconder A
>Verdade. E
> > > >nesse caso, nada do que eu, ou qualquer outro diga, fará a menor
> > >diferença,
> > > >certo?..:-)
> > > >
> > > >Este trecho é um problema:
> > > >
> > > >Talvez devesse dar algum crédito mesmo. Já estão acreditando em
> > > >acupuntura... Mas foi testada, nê, botaram uns eletrodozinhos e
>viram
> > >que
> > > >tinham razão aqueles outros velhinhos há mais de 3000 anos atrás...
> > > >Acordaaaaa!!!!
> > > >
> > > >Isso já foi explicado diversas vezes, mas vai de novo..:-) E, não
> > >precisa
> > > >acreditar em mim, basta apenas pensar sobre o que argumento e
>ver se é
> > > >possível refutar, ok?
> > > >
> > > >Acupuntura é a cura de todas as doenças pela manipulação de
>meridianos
> > >de
> > > >energia não detectável e mística através da implantação de
>agulhas em
> > > >pontos determinados desses meridianos. Isso é acupuntura e a
>base dessa
> > > >afirmação é que a causa de todas as doenças é o desequilibrio de
> > >energias
> > > >não detectáveis.
> > > >
> > > >Recentemente, detectou-se algum efeito na sintese de
>prostaglandinas ao
> > >se
> > > >aplicar agulhas na pele de pacientes. Como as prostaglandinas estão
> > > >envolvidas nas sensações dolorosas, isso pode causar algum
>efeito do
> > >tipo
> > > >anestésico leve pelas agulhas.
> > > >
> > > >Mas, isso NÃO É MAIS ACUPUNTURA. Isso é outra coisa, uma nova
> > >descoberta
> > > >sobre efeitos de agulhas em sintese de prostaglandinas. Nào
>exige que
> > >os
> > > >meridianos sejam identificados, nào exige que pontos
>especificos sejam
> > > >encontrados, e pode ser substituida por beliscões, em seu
>efeito nas
> > > >prostaglandinas.
> > > >
> > > >Distorcer um conceito, como a definição de acupuntura, para
>caber em
> > > >descobertas recentes e cientificas, é um procedimento padrão em
> > > >pseudociencias, também usado pela astrologia. Mas é evidente
>que, se
> > >não se
> > > >baseia na causa energética das doenças, se não exige o
>reequilíbrio
> > >dessas
> > > >energias, se independe de meridianos por onde essa enegia
>corre, não é,
> > > >claro, acupuntura.
> > > >
> > > >O que a acupuntura alega, e que nào é reconhecido pela ciência
>por não
> > >ter
> > > >se mostrado eficaz e nào ter mecanismo de ação reconhecivel, é que
> > >doenças
> > > >são causadas por desequilibrios de energia e que podem ser curadas
> > > >reequilibrando essas energias com agulhas em pontos específicos
>e bem
> > > >determinados da pele. Ela não alega que espetar agulhas (ou causar
> > > >sensações dolorosas de algum tipo) na pele das pessoas,
>interfere na
> > > >sintese de prostaglandinas, podendo servir como anestésico leve.
> > > >
> > > >Veja, nào deve acreditar em mim, deve apenas analisar o
>argumento. Se
> > >puder
> > > >demonstrar que está incorreto, ótimo. Se não puder, então ele é
>valido
> > >e
> > > >deve se manter até que algo mude (por exemplo, descobrir que
>energias
> > >nào
> > > >detectáveis causam tuberculose).
> > > >
> > > >Aqueles velhinhos de 3000 anos continuam enganados quanto a suas
> > >alegações.
> > > >Continuam enganados ao afirmarem que doenças são causadas por
> > > >desequilibrios energéticos. Continuam enganados ao afirmar que a
> > >energia
> > > >nào detectável corre por linhas determinadas chamadas meridianos.
> > >Continuam
> > > >enganados ao afirmar que podem curar doenças reequilibrando essas
> > >energias.
> > > >Continuam enganados ao afirmar que o reequilibrio se dá com
>agulhas em
> > > >pontos determinados da pele. E nào estão enganados sobre a
>sintese de
> > > >prostaglandinas, simplesmente porque nada sabiam sobre sintese de
> > > >prostaglandinas ou sobre qualquer outro sistema metabolico do
>corpo
> > >humano.
> > > >
> > > >Homero
> > > >
> > > >
> > > >
> > > > ----- Original Message -----
> > > > From: Esteban Moreno
> > > > To: ciencialist@yahoogrupos.com.br
> > > > Sent: Friday, February 04, 2005 4:24 PM
> > > > Subject: Re: [ciencialist] Re: Zodiaco
> > > >
> > > >
> > > >
> > > > Mr Takata wrote:
> > > > > descrições astrológicas?
> > > > Entao, em principio pareceria 'suspeito'. Mas como dito, qdo
> > > > devidamente testada a astromancia nao parece ter um bom poder de
> > > >predicao.
> > > >
> > > > E:
> > > > Devidamente testada por quem? Pelos santos céticos
>imparcialistas da
> > > > ciência?
> > > > Depende de quem ou como se pesquisa. Há uma série de artefatos e
> > > >falácias do
> > > > que julgam como pesquisas comprobatórias. Não vou
>enumera-las, mas
> > > >talvez
> > > > ajude a lembrar que a pesquisa da universidade de brasília
>resultou
> > >em
> > > >dados
> > > > bastante promissores. Que péssimo! Uma das justificativas dos
> > >céticos
> > > >acerca
> > > > do porquê os pacientes sentem-se bem descritos pelo
>astrólogo é a
> > > >chamada
> > > > confluência (auto-atribuição é um nome pobre para isso). Abaixo
> > >segue um
> > > > artigo científico que "desmistifica" (perdoe-me..) este recurso:
> > > >
> > > > Self-attribution, sun-sign traits, and the alleged role of
> > > >favourableness as
> > > > a moderator variable:
> > > > long-term e.ect or artefact? Edgar Wunder* Gesellschaft fu¨r
> > > >Anomalistik,
> > > > Postfach 1202, 69200 Sandhausen, Germany Personality and
>Individual
> > > > Differences 35 (2003) 1783-1789 www.elsevier.com/locate/paid
> > > >
> > > > Posso envia-lo em anexo.
> > > >
> > > > Takata:
> > > > > Um bom parâmetro é a quantidade do mercado que vem se
> > > > > ampliando extensivamente, incluindo grandes empresas e
> > >corporações.
> > > > Esse na verdade eh um pessimo parametro. Se for por isso
>deveremos
> > > > creditar confianca na homeopatia, passes espirituais (sem
>falar na
> > > > cirurgia mediunica), toda sorte de mandingas...
> > > >
> > > > E:
> > > > Talvez devesse dar algum crédito mesmo. Já estão acreditando em
> > > > acupuntura... Mas foi testada, nê, botaram uns eletrodozinhos e
> > >viram
> > > >que
> > > > tinham razão aqueles outros velhinhos há mais de 3000 anos
>atrás...
> > > > Acordaaaaa!!!!
> > > >
> > > >
> > > > Infelizmente só retorno daqui a 10 dias.
> > > > Um carnaval cheio de boas mandingas para vocês.
> > > > Esteban.
> > > >
> > > >
> > > >
> > > > ##### ##### #####
> > > >
> > > > Para saber mais visite
> > > > http://www.ciencialist.hpg.ig.com.br
> > > >
> > > >
> > > > ##### ##### ##### #####
> > > >
> > > >
> > > > Yahoo! Grupos, um serviço oferecido por:
> > > >
> > > >
> > > >
> > > >
> > > >
> > > >
> > > >
> > >
> > >
> >------------------------------------------------------------------------------
> > > > Links do Yahoo! Grupos
> > > >
> > > > a.. Para visitar o site do seu grupo na web, acesse:
> > > > http://br.groups.yahoo.com/group/ciencialist/
> > > >
> > > > b.. Para sair deste grupo, envie um e-mail para:
> > > > ciencialist-unsubscribe@yahoogrupos.com.br
> > > >
> > > > c.. O uso que você faz do Yahoo! Grupos está sujeito aos
>Termos do
> > > >Serviço do Yahoo!.
> > > >
> > > >
> > > >
> > > >[As partes desta mensagem que não continham texto foram removidas]
> > > >
> > >
> > >
> > >
> > >
> > > ##### ##### #####
> > >
> > > Para saber mais visite
> > > http://www.ciencialist.hpg.ig.com.br
> > >
> > >
> > > ##### ##### ##### #####
> > >
> > >
> > > Yahoo! Grupos, um serviço oferecido por:
> > >
> > >
> > >
> > >
> > >
> > >
> > >
> >
> >------------------------------------------------------------------------------
> > > Links do Yahoo! Grupos
> > >
> > > a.. Para visitar o site do seu grupo na web, acesse:
> > > http://br.groups.yahoo.com/group/ciencialist/
> > >
> > > b.. Para sair deste grupo, envie um e-mail para:
> > > ciencialist-unsubscribe@yahoogrupos.com.br
> > >
> > > c.. O uso que você faz do Yahoo! Grupos está sujeito aos
>Termos do
> > >Serviço do Yahoo!.
> > >
> > >
> > >
> > >[As partes desta mensagem que não continham texto foram removidas]
> > >
>
>
>




SUBJECT: p/Victor - Zodíaco
FROM: "murilo filo" <avalanchedrive@hotmail.com>
TO: ciencialist@yahoogrupos.com.br
DATE: 06/02/2005 20:32

Victor, oi.
Vc também teve um trabalhão e já ví que estudou a coisa a fundo.
P/mim, nada substituirá evidências coletadas pessoalmente.
Já obtive sinastria anual 100% errada de quem já havia acertado 100%.
Isto não invalidou a experiência prévia e nem a posterior. Experiência não é
crença.
O êrro, o engano e a ignorância está em toda parte. A coisa pode ser um
pouco mais complicada do que pensar que quem segue, ou ouve, um recurso como
a astrologia, é burro e só está procurando uma anestesia, uma alienação ou
qualquer submersão supersticiosa e refrescante. Não é tão fácil e nem tão
simples. TODO O MUNDO TEM SEUS PROBLEMAS P/RESOLVER, INCLUSIVE OS
ASTRÓLOGOS, VOCÊ E EU.
É lícito e bom procurarmos soluções e eu detesto mascar chicles muito
mastigados.
Gosto de conferir.
Minha visão está na msg abaixo, emitida p/o Oraculo, ontem.
abr/M.

''Oi, Oraculo.
Estava me referindo à recepção e medição de resultados, e sempre usando êste
estado mental comum que nos suporta por êste mundão velho sem porteira.
A inteligência normalmente treinada e uma estrutura psicológica estável
conseguem razoável cognição entre o que é furado e o tal ''epa, aquí tem
alguma coisa...''
A astrologia é um outro mundo onde se encontram todas aquelas mumunhas
normais e comuns aos outros ramos, inclusive os sacanas e aquêles que
prometem o impossível.
Aprecio muito a intuição.
Previsões? Acho meio difícil, mas já ví. Encaro esta coisa mais como uma
tábua das marés, sempre conforme a pessoa e um cacetão de variáveis,
inclusive as mais prosáicas. Tratam-se de tendências, informações de prá
onde e como correrá, ou corria, a maré. Uma ferramenta a mais.
Quando a indicação de uma 'maré' é muito forte, a coisa é quase inescapável.
Não pense que espero receber de qualquer outra coisa externa um refresco
p/meus pepinos! Estamos todos aquí só para aprender, e o êrro ensina mais do
que o acêrto.
Tudo que ainda não aconteceu pode ser mudado e qualquer tendência poderá ser
contrariada pela vontade, pela ação e até pela ignorância humanas. abr.
M. sp 05/fev''




SUBJECT: Re: [ciencialist] Zodiaco
FROM: "Sergio M. M. Taborda" <sergiotaborda@terra.com.br>
TO: ciencialist@yahoogrupos.com.br
DATE: 06/02/2005 21:42

JVictor wrote:

>Homero,
>
>Vale a pena ler. Não conheço ou autor, mas está interessante.
>
>(...)
>
>A astrologia, em sua forma tradicional, é um método de adivinhação
><http://brazil.skepdic.com/adivinhacao.html> baseado na teoria de que as
>posições e movimentos dos corpos celestes (estrelas, planetas, sol e
>lua), no momento do nascimento, influenciam profundamente a vida da
>pessoa.
>
Não. Isso não é correecto. A astrologia baseia a sua teoria em que todas
as reprocuções podem ser
conhecidas pelos arranjos celestes no momento em que evento ocorre. Não
é um vinculo.
No caso do nascimento, momento em que começa a vida para o mundo, as
astros desse momento
permiter ler as reprocussões dessa vida. A astrologia não se limita a
nascimentos, e isso é esquecido pelos criticos.

>A forma mais tradicional é a Astrologia de Signos Solares, tipo
>encontrado em numerosos jornais diários que publicam horóscopos. O
>horóscopo é um prognóstico astrológico. O termo também é usado para
>descrever um mapa do zodíaco no momento do nascimento da pessoa.
>
Isso tb não é certo. O termo correcto é Mapa Natal.
Horoscopo é um mapa de horas apenas válido num dado intervalo de tempo.

> O
>zodíaco se divide em doze zonas celestes, cada qual recebendo o nome de
>uma constelação que originalmente coincidia com a zona (Touro, Leão,
>etc.). Todas as trajetórias aparentes do sol, da lua e dos principais
>planetas se encaixam dentro do zodíaco. Devido ao movimento da
>precessão, os pontos de equinócio e solstício se moveram para o oeste
>cerca de 30 graus nos últimos 2.000 anos. Assim, as constelações
>zodiacais que receberam seus nomes na antiguidade não correspondem mais
>aos segmentos do zodíaco representados por seus signos. Em resumo, se
>você tivesse nascido na mesma hora do mesmo dia do ano há 2.000 anos,
>teria nascido sob um signo diferente.
>
>
Isso chama-se precessão. É um fenomeno fisico perfeitamente entendido e
que a propria astrologia encaixa nos seus esudo.
Conhecido como Era Astrologica. Provávelmente já ouviu falar da Era de
Aquário.
As Eras são ditadas pela precessão do ponto Vernal, que se move para
trás no zodiaco. As eras não são importantes para a astrologia natal,
pois uma vida é muito menor que uma era mas são importantes para a
astrologia mundial , a que estuda paises e o mundo como um todo.

>Uma das defesas comuns a favor da astrologia é o argumento falacioso da
>popularidade e tradição: bilhões de pessoas no mundo acreditam na
>astrologia e ela tem sobrevivido por milhares de anos.Essas afirmações
>são verdadeiras, mas irrelevantes como prova da "veracidade" da
>astrologia.
>
Irrelevantes para a prova, mas relevantes para a suspeita de que ha algo
mais do que simples mitologia, leitura a frio, ou outro qq truque.

>Os antigos caldeus e assírios se envolveram com a
>adivinhação astrológica há cerca 3.000 anos. Em torno de 450 AEC, os
>babilônios tinham desenvolvido o zodíaco de 12 signos, mas foram os
>gregos --do tempo de Alexandre, o grande, até a conquista pelos
>romanos-- que forneceram a maioria dos elementos fundamentais da
>astrologia moderna.
>
>
Foram eles que a matematizaram , baseados na numerologia.
Os arabes, depois deles, desenvolveram a verdadeira astrologia
ocidental, estendendo
o modelo matemático dos gregos. Digamos que os gregos aportaram a
cinematica, enquanto
os arabes aportaram a dinamica.

>A disseminação das práticas astrológicas foi contida pela ascensão do
>cristianismo, que enfatizava a intervenção divina e o livre arbítrio. Na
>renascença, a astrologia recuperou popularidade, em parte devido ao
>ressurgimento do interesse pela ciência e pela astronomia. Teólogos
>cristãos, no entanto, combateram a astrologia e, em 1585, o Papa Sisto V
>a condenou. Na mesma época, os trabalhos de Kepler e outros
>enfraqueceram os princípios astrológicos.
>
>
LOL ... Kepler aportou o algoritm final, que os os arabe nunca puderam,
e ninguem pode até Kepler.
A discussão sobre o referencial a escolher - o sol , ou a terra - toldou
a solução do problema durante milenios.
A questão era principalmente filosofica, religiosa , até. E foi o
espirito renascentista de Kepler, de ordem matemática
E conseguiu aportar o algoritmo que faltava, que era o de poder calcular
a posição dos planetas matemáticamente.
Isso não enfraqueceu a astrologia, a fortaleceu. A retirou da
necessidade de observar o ceu, e a tornou mais tecnica.
Por esse abandono de olhar os ceus, que a astronomia nasceu. Pois agora,
os ceus não era mais olhados para saber as posições.
A observação era agora um divertimento, a exploração de um outro mundo.

> No entanto, mesmo se houvesse uma correlação
>significativa entre a posição de Marte na data do nascimento e o fato da
>pessoa se tornar um atleta excepcional, isso não implicaria ou mesmo
>indicaria existir uma conexão causal entre a posição de um planeta e o
>tipo de atividade em que a pessoa se daria bem na Terra. A correlação
>entre x e y não é uma condição suficiente para a crença racional de que
>x causa y.
>
É sim. O que não é suficiente é para provar que existe um vinculo fisico
entre a causa e o efeito. Ou seja, uma lei universal.
Os fisicos vivem a vida procurando correlações, e apenas algumas
sobrevivem como leis.
Mas a astrologia não afimar que existem leis, apenas correlações.

>Por exemplo, os defensores da astrologia gostam de observar que 'a
>duração do ciclo menstrual da mulher corresponde às fases da lua'
>
O que, non the less , é verdade. Não improta pq são iguais, mas são.
O facto de haver essa correlação é o que chega e sobra que que se possam
fazer previsões de uma baseada na outra.
Não precida haver vinculo fisico, para que haja vinculo matemático, e a
astrologia é simplesmente matemática.

> e que
>'os campos gravitacionais do sol e da lua são fortes o bastante para
>causar a subida ou a descida das marés na Terra.'
>
Isso é conversa para boi dormir. A astrologia não está preocupada no
como funciona. Aliás, como nenhuma arte está. São os seus criticos que
estão preocupados com isso.
A astrologia foi criada á milhares de anos atrás, quando não se sabia o
que era a força gravitacional. Por isso, como ela poderia ser invocada
para explicar alguma coisa ?
Agora, hoje em dia, alguns astrologos sentem-se compelidos a responder
às perguntas idiotas dos cepticos , numa tentativa de mostrar que ha
algo mais aqui do que acaso.
Eles inventam então uma série de hipoteses- como qq cientista faria -
pegando mão da pouca ciencia que conhecem. Força, marés, energias e
campos são expressões da fisica que os astrologos são obrigados a usar
pelos cepticos , mas que não têm qualquer significado , ou sequer são
usados durante a aconstrução de um mapa astral, ou a sua intrperetação.
Tudo isso é portanto uma teoria à margem do que a astrologia realmente
é, e perde tempo com isso quem quer. Atacar isso, não leva a lado nenhum.

>Mas o que existe deanálogo às marés numa pessoa?
>
Se soubesses responder a isso, a astrologia seria uma ciencia.

>Os astrólogos dão ênfase à importância das posições do sol, da lua, dos
>planetas, etc. no instante do nascimento. Mas por que as condições
>iniciais seriam mais importantes para a personalidade e as
>características de uma pessoa que todas as condições subseqüentes?
>
Não o são. Se estudar astrologia conhecerá não so a astrologia natal,
mas tb a contrução de Transitos e Progressões
Todas estas se baseiam em posições futuras dos planetas. E a astrologia
geral, invoca as posições futuras sempre que ha necessidade.
Por exemplo, se vc queiser saber se um casamento será frutifero, vc
analisa o mapa do evento do casamento (data, hora local) com os mapas
dos noivos para aquele dia , comparados com os seus mapas natais. Vc
pode fazer isto, meses, anos, quando quiser, ANTES que o evento
aconteça. Vc pode fazer isto pq o casamento é algo que vc controla o
acontecimento, mas um nascimento é dificil controlar.
O mapa natal é apenas uma impressão digital, vc não pode ser livrar
dela, e o identifica em qq ponto do espaço e em qq época. Por isso que
ele é estático. Mas ee não é tudo o que ha a dizer.

>Por
>que seria escolhido como o momento crucial o nascimento, e não a
>concepção?
>
Pq o evento em estudo é o começo da vida exposta ao mundo. É nesse
momento, que vc sai da barriga da mae, que o mundo é cosnciente de si
como uma pessoa, como um menbro a mais da humanidade.
É a partir dai que a sua vida começa , não no sentido biologico, mas no
sentido humano, social, externo, no sentido que ela começa para o mundo.

>Por que outras condições iniciais como a saúde da mãe, as
>condições do local do parto, fórceps, luzes fortes, sala escura, banco
>traseiro de um automóvel, etc., não seriam mais importantes do que se
>Marte está ascendendo, descendendo, culminando ou fulminando?
>
Não do ponto de vista astrologico. Tal como do ponto de vista fisico ,
não importa se o carro é branco ou preto, importa a sua velocidade , a
sua massa , a sua posição inicial .
Da mesma forma que a fisica recolhe a informação a astrologia escolhe a
sua. Especular pq escolhe essa em vez de outra é futil.

>Por que o
>planeta Terra, muito mais próximo de nós no nascimento, não seria
>considerado uma influência importante no que somos e no que nos tornamos?
>
>
Bom, a astrologia horária - aquela que faz Horoscopos - usa apenas a
hora, o que é apenas uma caracteristica da rotação da propria terra.

>Além do Sol e da Lua, e de algum cometa ou asteróide passando
>ocasionalmente, a maioria dos objetos planetários está demasiado
>distante de nós.
>
E dai ? Pq teriam que estar proximos ?

> Qualquer influência que pudessem ter sobre o nosso
>planeta seria encoberta pelas do Sol e da Lua.
>
Qual influencia ? de que tipo ? pq a do sol e da lua as encobriria ?
Isto são tudo afirmações sem substentação. Para a astrologia não importa
o quão longe podem estar os corpos.
O sistema referencial da astrologia é esferico e apenas uma das
coordenadas interessa. E não é a distancia.

> É muito mais provável que
>a Terra, e as pessoas e coisas com as quais se tem contato direto, sejam
>fatores de influência mais importantes nas nossas vidas que distantes
>corpos celestes.
>
Tudo bem. Mas isso não seria astrologia. Seria, sociologia, psicologia,
ou outra qq coisa, mas não astrologia.
Mas, a astrologia tb estuda isso pela sinastria.

>Além disso, se for verdade que podemos determinar
>efeitos específicos a partir de condições específicas do local do
>nascimento, então podemos controlar essas condições de forma a trazer
>resultados benéficos.
>
Podemos. Isso já foi tentado. Mas não é eficaz pois não mandamos na
natureza.
E mesmo que o médico diga que a pessoa vai nascer daqui a 7 horas, isso
pode não acontecer tal e qual.
Afinal a previsão médica não é tão boa assim.
No fim existem bastante margem para que o plano não dê certo.

> Por outro lado, mesmo se for verdade que a posição
>das estrelas e planetas seja mais importante para a vida da pessoa do
>que um nascimento difícil e passado sob condições horrendas, não há nada
>que possamos fazer em relação à posição das estrelas, e há um limite no
>controle que podemos ter sobre o momento do nascimento de uma pessoa.
>
>
Mas sendo que o que interessa não é a posição dos planetas, mas a sua
posição relativa , é mais facil do que parece.
Por exemplo o sol tem 12 conjunções com a lua num ano.

>(Ainda bem que não vou ser um astrólogo na era dos bebês de proveta.
>Como eu iria saber quando o meu cliente 'nasceu'?
>
E não nasce na proveta ! Duh!

>Ninguém diria que, para que se compreendesse o efeito da lua sobre as
>marés ou sobre as batatas, fosse preciso entender as condições inicial
>da singularidade que precedeu o Big Bang, ou a posição das estrelas no
>momento em que as batatas foram colhidas.
>
Bom, um dos primeiro usos da astrologia era esse mesmo. Saber qual a
melhor epoca para o plantio e para a colheira dos produtos agriculas.

>Condições iniciais são menos importantes que
>as presentes para que se compreendam efeitos atuais sobre rios e
>plantas. Se isso vale para marés e plantas, por que não valeria para as
>pessoas?
>
>
Vc diz isso pq não é fisico. Para um fisico, as condições iniciais são
tão importantes quanto as regras que transformam essas condições nas
actuais.
E sendo que o movimento planetário é regido pela fisica e não pela
biologia. as condições iniciais são importantes sim.

>*correlação não é causalidade*
>
>
Pois não. Mas essas discusão é irrelevante pq a astrologia não prega a
causalidade.

>
>Para concluir, há aqueles que defendem a astrologia argumentando com o
>quanto os horóscopos profissionais acertam. Um colega, professor de
>história da Universidade da Califórnia em Davis, com título de Ph.D,
>pratica a astrologia. Naturalmente, possui tecnologia e tem um programa
>de computador para ajudá-lo a fazer as leituras.
>
O computador não sabe fazer leituras pois isso requer inteligencia que
ele não tem.
Ele apenas faz as contas e o graficos e mostra o signfiicado de cada
aspecto. Mas é o astrologo que tem que fazer a intrepretação
pois não é uma coisa simples de programar.

> Conhece todos os
>argumentos contra a astrologia e até admite que logicamente ela não
>deveria funcionar. Mas funciona, acredita ele. Esse conceito de
>'funciona' é curioso. O que será que significa?
>
>
Experimente e verá o que significa. É tão simples quanto isso.

>Não. Não consigo encontrar sequer uma única boa razão para
>acreditar em nada disso.
>
O seu problema é achar que é uma questão de crença.

Sergio Taborda


--
No virus found in this outgoing message.
Checked by AVG Anti-Virus.
Version: 7.0.300 / Virus Database: 265.8.5 - Release Date: 03-02-2005



SUBJECT: RE: [ciencialist] Zodiaco
FROM: "Hugo Santos" <urano@netvisao.pt>
TO: <ciencialist@yahoogrupos.com.br>
DATE: 06/02/2005 22:35

Sérgio (e outros pro-astrologia),

Porque é que os astrólogos só ganham dinheiro a fazerem consultas para as
outras pessoas? Não conseguem determinar o seu próprio futuro? Não conseguem
analisar as probabilidades de serem bem sucedidos num negócio? Porque é que
todo o dinheiro que têm costuma ser aquele "retirado" aos seus clientes? No
Top500 da Forbes duvido que conste um único astrólogo de profissão. Seria
isso provável de acontecer no caso de realmente haver qualquer capacidade de
previsão? Mas mesmo que assumamos que os astrólogos (todos) não estão
interessados em ficarem ricos (o que não parece ser nada o caso), podemos
colocar em causa o seu grau de felicidade, o seu grau de sucesso. Não tenho
conhecimento de que as pessoas que praticam astrologia ou que a ela recorrem
sejam SIGNIFICATIVAMENTE mais "felizes" ou "bem sucedidas" do que todas as
outras que não são minimamente crentes, ou que acreditam noutros tipos de
crenças (como as religiões, por exemplo).

Por outro lado, ao dizer que pode prever com alguma precisão as condições
que poderão influenciar decisões futuras, está nesse momento a influenciar
as decisões actuais. Como tal, nesse momento, o futuro já foi alterado. Ou
seja, usar a astrologia para qualquer "cálculo" futuro é irrelevante, porque
nesse momento já se alterou o momento actual e, consequentemente, o futuro.
Como muito bem disse o Sérgio, as condições iniciais são bem importantes e
cada momento presente é uma condição inicial para qualquer momento futuro.
Se a previsão afecta o momento presente, o futuro também é afectado e a
previsão deixa de fazer sentido. Ou seja, a astrologia baseia-se numa
situação paradoxal que, a meu ver, a desacredita imediatamente.

Ora, usar a astrologia para cálculos passados não serve para nada. O passado
já todos nós sabemos o que foi. Ninguém quer fazer "previsões" sobre o
passado. Se o presente é o que é, se o passado foi o que foi e se o futuro
não pode ser previsto porque a previsão o afecta, então a astrologia não
serve para nada.

Devo, no entanto, reconhecer um uso para a astrologia. É um aspecto
importante da esperança de certas pessoas. Basicamente é só mais uma fonte
de esperança. É um pouco como o efeito placebo que foi inúmeras vezes
encontrado em investigações científicas relativas ao efeito de medicamentos
ou situações experimentais. O sentir que têm mais controlo sobre o seu
futuro do que aquele que realmente têm, o "saber" que as coisas vão correr
bem, mesmo que isso possa ser verdade ou não, pode realmente influenciar a
forma como essa pessoa encara esse acontecimento futuro e é esse prévio
"conhecimento" que pode influenciar o decorrer desse acontecimento. É sabido
que uma pessoa que faz as coisas com um sentido positivista, deverá ter uma
probabilidade maior de ser bem sucedida do que uma que o faça de forma
negativista. Não desistir antes de as coisas começarem costuma ser um bom
ponto de partida para se ter sucesso.

Desta forma, coloco a astrologia no mesmo ponto que a religião. Simplesmente
servem para colmatar lacunas na psicologia das pessoas. Quem precisar dessa
"fé" que a use, mas que tenha a noção de que é uma coisa pessoal e que pode
não ter nada a ver com as necessidades de apoio psicológico doutro
indivíduo. Desta forma, tentar convencer os outros de que a nossa crença,
seja ela qual for, é melhor do que as outras, é perfeitamente absurdo e
ABUSIVAMENTE enganadora (o mesmo se aplica a cientistas).

Um abraço,

Hugo Santos

P.S.: Já agora, aproveito para fazer um reparo. A quantidade de erros
gramaticais e ortográficos encontrados nos mails desta lista é simplesmente
assustador. Alguns erros passam e são perfeitamente aceitáveis, mas por
vezes chego mesmo a perguntar-me se se está a escrever em português. Por
favor caros colegas, tenham um pouco mais de cuidado com a forma como
escrevem. Assassinar a língua não é uma coisa muito nobre de se fazer. Como
já outrora ouvi, se não têm tempo para usar um corrector ortográfico, então
não têm tempo para andar para aqui a escrever.




SUBJECT: Re: Fw: água
FROM: "rmtakata" <rmtakata@altavista.net>
TO: ciencialist@yahoogrupos.com.br
DATE: 07/02/2005 00:54


Por causa de duas caracteristicas da agua: uma eh q. a agua 'gruda'
nos objetos - nossa pele, no papel, no tapete... no q. a agua nao
gruda, ela nao eh capaz de molhar: certos tipos de plasticos, panelas
com teflon... outra caracteristica importante eh q. as partes q.
compoem a agua mantem-se unidas entre si.

Se forca q. faz a agua grudar em algo eh muito mais forte do q. a
forca q. faz as parte da agua permanecerem juntas, a agua nao pode
molhar. Se a forca q. mantem as partes da agua juntas eh mais forte, a
agua tb nao eh capaz de molhar.

[]s,

Roberto Takata


--- Em ciencialist@yahoogrupos.com.br, "Luiz Ferraz Netto"
> Olha a idade!
> []'
> ===========================
> Luiz Ferraz Netto [Léo]
> leobarretos@u...
> http://www.feiradeciencias.com.br
> ===========================
> -----Mensagem Original-----
> De: haroldo hirano
> Para: leobarretos@u...
> Enviada em: domingo, 7 de janeiro de 2001 12:20
> Assunto: água
>
>
> "por que a água é molhada?"
> Débora ayumi hirano
> 8 anos segunda série
> o e-mail é da mamãe
> mirihirano@h...
> Nas dúvidas experimentais, por gentileza coloque aqui o endereço da
página, isso facilita o confronto. Agradeço. Meu nome é LUIZ FERRAZ
NETTO, meu apelido é LÉO e moro em BARRETOS; dai vem meu e-mail:
leobarretos@u...
>
>
>
--------------------------------------------------------------------------------
>
>
> No virus found in this incoming message.
> Checked by AVG Anti-Virus.
> Version: 7.0.300 / Virus Database: 265.8.5 - Release Date: 03/02/2005
>
> ----------
>
> No virus found in this outgoing message.
> Checked by AVG Anti-Virus.
> Version: 7.0.300 / Virus Database: 265.8.5 - Release Date: 03/02/2005
>
>
> [As partes desta mensagem que não continham texto foram removidas]





SUBJECT: Re: Zodiaco
FROM: "rmtakata" <rmtakata@altavista.net>
TO: ciencialist@yahoogrupos.com.br
DATE: 07/02/2005 02:16


Como de costume este topico foi muito alem da extensao razoavel q. sua
importancia real propiciaria.

Algumas anotacoes interessantes - q. resume basicamente o ponto de
vista q. adoto do desenvolvimento do zodiaco - podem ser lidas em:

http://geology.wcupa.edu/mgagne/ess362/notes/zodiac.pdf

Ver tb:

http://encarta.msn.com/encyclopedia_761553134/Constellation_(astronomy).html

[]s,

Roberto Takata





SUBJECT: RE: [ciencialist] Zodiaco
FROM: "Felipe Alvisi Galastro Perez" <felipealvisi@uol.com.br>
TO: "ciencialist" <ciencialist@yahoogrupos.com.br>
DATE: 07/02/2005 09:04

Soh pitacos de quem vem avidamente acompanhando a discussão:

[Sérgio (e outros pro-astrologia),
>
> Porque é que os astrólogos só ganham dinheiro a fazerem consultas para as
> outras pessoas? Não conseguem determinar o seu próprio futuro? Não conseguem
> analisar as probabilidades de serem bem sucedidos num negócio? Porque é que
> todo o dinheiro que têm costuma ser aquele "retirado" aos seus clientes? No
> Top500 da Forbes duvido que conste um único astrólogo de profissão. Seria
> isso provável de acontecer no caso de realmente haver qualquer capacidade de
> previsão?]

Caro Hugo,
penso que toda a discussão que vem sendo travada nessa thread pode ser explicada pela idéia destas suas indagações. Todo o "temor" ou pré conceito em relação à astrologia provém da estereotipada idéia pós moderna de que además à explicação científica para tudo, tudo tem que ter também uma explicação prática... O que quero dizer é que a exigência não é mais tão somente por expllicações astronômicas para as interpretações astrológicas, pois exige-se também uma explicação para erros cometidos a a menudo na TV por charlatães, para Valter Mercados da vida, para "por que os astrólogos não são todos ricos e felizes?"... Isso afasta muuuiito a astrologia de um âmbito em que pode ser discutido com cientificidade.
O que o Sérgio Taborda vem fazendo, e tenho que admitir que concordo e avalizo inteiramente (se é q isso vale algo!), é transportar o assunto para um módulo em que se possa ver ciência por trás de algo obscuro, ininteligível, por que não. Exatamente pelos pequenos detalhes cotidianos que compartilhamos numa rede velada com amigos, tv, pais, pessoas que intrinsecamente desvalorizam todo o tempo algo de que não têm o menor conhecimento, como pq os astrólogos não são ricos e felizes?, hehe, ou pq a mão diná (se é q é astrologa, nem sei) errou no gugU do ano passado, é que temos a visão distorcida. E mesmo depois de todo o conhecimento agregado após a discussão de 4a série sobre pq a mãe diná errou, transportamos o estereótipo para o mundo dos "intelectuais".
Nem de longe sou um defensor da astrologia, sou um defensor do ceticismo racional, sem apelo ao intangível para justificar tudo com argumentação repetitiva. Como já foi dito aí pra trás, duvidemos de nossas dúvidas. Há muitas centenas de anos se desenvolve uma "logia" e se perdura até hj é pq alguma validade tem. Amanhã, podemos descobrir que estávamos errados todo o tempo e que existe toda sorte de "energias", "magias" e outras coisas q hj recriminamos.
Mudando um pouco o rumo, mesmo para aqueles que discordam da charlatanisse da astrologia, é inegável que a parametrização do comportamento humano provido por ela é muito contundente.. Bem como a matemática envolvida. Realmente, ninguém aqui vai explicar pq um leonino é um leonino por causa da posição de astros na hora do seu nascimento. Sei que o que querem os "puristas" é que algum dos "não afirmo nada pq a astrologia é um conhecimento muito grande para descartá-lo por pré-conceitos" dê a resposta para essa pergunta para que possam categoricamente refutá-la com alguma contra argumentação do estilo leitura-fria, ou estatística barata, mas convenhamos, tão tolo quanto esta argumentação seria alguém se arriscar com a resposta. A astrologia não é uma ciência exata, e uma resposta linear seria negar o mistério que perdura por trás dela. Tudo que se vem discutindo, é por uma tentativa cartesiana de explicar algo desconhecido. Agora, se pudermos abrir os olhos e discutir a matemática e astronomia por trás da astrologia, que é cartesiana, aí sim chegaremos a algum improvement geral de conhecimento. Se todas as vezes que uma explicação física for refutada com uma retórica retomada de estereóticos infantis acerca de mapas astrais e bobalhões q vão à tv para anunciar idiotices, bem... será mais do mesmo.

[]'s
FAGP


---------- Início da mensagem original -----------
De: "Hugo Santos" urano@netvisao.pt
Para: ciencialist@yahoogrupos.com.br
Cc:
Data: Mon, 7 Feb 2005 00:35:43 -0000
Assunto: RE: [ciencialist] Zodiaco
>
> Sérgio (e outros pro-astrologia),
>
> Porque é que os astrólogos só ganham dinheiro a fazerem consultas para as
> outras pessoas? Não conseguem determinar o seu próprio futuro? Não conseguem
> analisar as probabilidades de serem bem sucedidos num negócio? Porque é que
> todo o dinheiro que têm costuma ser aquele "retirado" aos seus clientes? No
> Top500 da Forbes duvido que conste um único astrólogo de profissão. Seria
> isso provável de acontecer no caso de realmente haver qualquer capacidade de
> previsão? Mas mesmo que assumamos que os astrólogos (todos) não estão
> interessados em ficarem ricos (o que não parece ser nada o caso), podemos
> colocar em causa o seu grau de felicidade, o seu grau de sucesso. Não tenho
> conhecimento de que as pessoas que praticam astrologia ou que a ela recorrem
> sejam SIGNIFICATIVAMENTE mais "felizes" ou "bem sucedidas" do que todas as
> outras que não são minimamente crentes, ou que acreditam noutros tipos de
> crenças (como as religiões, por exemplo).
>
> Por outro lado, ao dizer que pode prever com alguma precisão as condições
> que poderão influenciar decisões futuras, está nesse momento a influenciar
> as decisões actuais. Como tal, nesse momento, o futuro já foi alterado. Ou
> seja, usar a astrologia para qualquer "cálculo" futuro é irrelevante, porque
> nesse momento já se alterou o momento actual e, consequentemente, o futuro.
> Como muito bem disse o Sérgio, as condições iniciais são bem importantes e
> cada momento presente é uma condição inicial para qualquer momento futuro.
> Se a previsão afecta o momento presente, o futuro também é afectado e a
> previsão deixa de fazer sentido. Ou seja, a astrologia baseia-se numa
> situação paradoxal que, a meu ver, a desacredita imediatamente.
>
> Ora, usar a astrologia para cálculos passados não serve para nada. O passado
> já todos nós sabemos o que foi. Ninguém quer fazer "previsões" sobre o
> passado. Se o presente é o que é, se o passado foi o que foi e se o futuro
> não pode ser previsto porque a previsão o afecta, então a astrologia não
> serve para nada.
>
> Devo, no entanto, reconhecer um uso para a astrologia. É um aspecto
> importante da esperança de certas pessoas. Basicamente é só mais uma fonte
> de esperança. É um pouco como o efeito placebo que foi inúmeras vezes
> encontrado em investigações científicas relativas ao efeito de medicamentos
> ou situações experimentais. O sentir que têm mais controlo sobre o seu
> futuro do que aquele que realmente têm, o "saber" que as coisas vão correr
> bem, mesmo que isso possa ser verdade ou não, pode realmente influenciar a
> forma como essa pessoa encara esse acontecimento futuro e é esse prévio
> "conhecimento" que pode influenciar o decorrer desse acontecimento. É sabido
> que uma pessoa que faz as coisas com um sentido positivista, deverá ter uma
> probabilidade maior de ser bem sucedida do que uma que o faça de forma
> negativista. Não desistir antes de as coisas começarem costuma ser um bom
> ponto de partida para se ter sucesso.
>
> Desta forma, coloco a astrologia no mesmo ponto que a religião. Simplesmente
> servem para colmatar lacunas na psicologia das pessoas. Quem precisar dessa
> "fé" que a use, mas que tenha a noção de que é uma coisa pessoal e que pode
> não ter nada a ver com as necessidades de apoio psicológico doutro
> indivíduo. Desta forma, tentar convencer os outros de que a nossa crença,
> seja ela qual for, é melhor do que as outras, é perfeitamente absurdo e
> ABUSIVAMENTE enganadora (o mesmo se aplica a cientistas).
>
> Um abraço,
>
> Hugo Santos
>
> P.S.: Já agora, aproveito para fazer um reparo. A quantidade de erros
> gramaticais e ortográficos encontrados nos mails desta lista é simplesmente
> assustador. Alguns erros passam e são perfeitamente aceitáveis, mas por
> vezes chego mesmo a perguntar-me se se está a escrever em português. Por
> favor caros colegas, tenham um pouco mais de cuidado com a forma como
> escrevem. Assassinar a língua não é uma coisa muito nobre de se fazer. Como
> já outrora ouvi, se não têm tempo para usar um corrector ortográfico, então
> não têm tempo para andar para aqui a escrever.
>
>
>
>
> ##### ##### #####
>
> Para saber mais visite
> http://www.ciencialist.hpg.ig.com.br
>
>
> ##### ##### ##### #####
> Links do Yahoo! Grupos
>
>
>
>
>
>
>
>
>

__________________________________________________________________________
Acabe com aquelas janelinhas que pulam na sua tela.
AntiPop-up UOL - É grátis!
http://antipopup.uol.com.br/



[As partes desta mensagem que não continham texto foram removidas]



SUBJECT: RE: [ciencialist] Zodiaco
FROM: "Hugo Santos" <urano@netvisao.pt>
TO: <ciencialist@yahoogrupos.com.br>
DATE: 07/02/2005 09:41

Felipe,

Admito realmente que o argumento do não aproveitamento do suposto
conhecimento extra que se pode obter da astrologia, não é propriamente
forte. Há inúmeras maneiras de lhe escapar sem, no entanto, o negar. E já
agora, escapar a um argumento, não é uma forma de contra argumentação. Para
isso teria de se apresentar um argumento válido e contrário que invalide o
anterior. Até agora não vi um único desses a favor da astrologia. Parece
que, tal como os religiosos, os astrólogos estão constantemente a
resguardar-se por detrás do desconhecimento que temos na área. Muito
sinceramente, não acho que o facto de não se compreender algo, possa dar
"carta branca" para que se "invente" seja lá o que for e depois argumentar
que não se pode dizer que é charlatanice porque não se sabe como funciona.
Para provar que algo funciona É PRECISO explicar COMO funciona, ou pelo
menos mostrar INDUBITAVELMENTE casos e situações em que esse algo realmente
funcionou. Como é que se pode alguma vez provar que foi uma previsão
astrológica que acertou e não o simples acaso?

Bem, mas nem vale a pena entrar por este âmbito. Na realidade, como já
disse, o argumento da riqueza ou felicidade não é grande coisa. Até mesmo o
que digo em cima é facilmente desfeito com "argumentos escapatórios". No
entanto, ainda não vi ninguém que desfizesse o argumento da previsão
presente afectar o decurso do futuro. Ainda gostava de ver realmente alguma
"ginástica" mental e argumentativa para tentarem escapar a esse! Não é por
terem contra argumentado o meu primeiro parágrafo que todo o meu mail fica
desfeito.

Hugo Santos




SUBJECT: RE: [ciencialist] Zodiaco
FROM: "Felipe Alvisi Galastro Perez" <felipealvisi@uol.com.br>
TO: "ciencialist" <ciencialist@yahoogrupos.com.br>
DATE: 07/02/2005 11:05

e é precisamente por isso que a discussão chega a um limiar estacionário, onde toda a argumentação é fogo cruzado de nada em direção a menos ainda.. :)
[]'s


---------- Início da mensagem original -----------
De: "Hugo Santos" urano@netvisao.pt
Para: ciencialist@yahoogrupos.com.br
Cc:
Data: Mon, 7 Feb 2005 11:41:19 -0000
Assunto: RE: [ciencialist] Zodiaco
>
> Felipe,
>
> Admito realmente que o argumento do não aproveitamento do suposto
> conhecimento extra que se pode obter da astrologia, não é propriamente
> forte. Há inúmeras maneiras de lhe escapar sem, no entanto, o negar. E já
> agora, escapar a um argumento, não é uma forma de contra argumentação. Para
> isso teria de se apresentar um argumento válido e contrário que invalide o
> anterior. Até agora não vi um único desses a favor da astrologia. Parece
> que, tal como os religiosos, os astrólogos estão constantemente a
> resguardar-se por detrás do desconhecimento que temos na área. Muito
> sinceramente, não acho que o facto de não se compreender algo, possa dar
> "carta branca" para que se "invente" seja lá o que for e depois argumentar
> que não se pode dizer que é charlatanice porque não se sabe como funciona.
> Para provar que algo funciona É PRECISO explicar COMO funciona, ou pelo
> menos mostrar INDUBITAVELMENTE casos e situações em que esse algo realmente
> funcionou. Como é que se pode alguma vez provar que foi uma previsão
> astrológica que acertou e não o simples acaso?
>
> Bem, mas nem vale a pena entrar por este âmbito. Na realidade, como já
> disse, o argumento da riqueza ou felicidade não é grande coisa. Até mesmo o
> que digo em cima é facilmente desfeito com "argumentos escapatórios". No
> entanto, ainda não vi ninguém que desfizesse o argumento da previsão
> presente afectar o decurso do futuro. Ainda gostava de ver realmente alguma
> "ginástica" mental e argumentativa para tentarem escapar a esse! Não é por
> terem contra argumentado o meu primeiro parágrafo que todo o meu mail fica
> desfeito.
>
> Hugo Santos
>
>
>
>
> ##### ##### #####
>
> Para saber mais visite
> http://www.ciencialist.hpg.ig.com.br
>
>
> ##### ##### ##### #####
> Links do Yahoo! Grupos
>
>
>
>
>
>
>
>
>

__________________________________________________________________________
Acabe com aquelas janelinhas que pulam na sua tela.
AntiPop-up UOL - É grátis!
http://antipopup.uol.com.br/



[As partes desta mensagem que não continham texto foram removidas]



SUBJECT: Re: Energia "Verde" Já
FROM: "Tipoalgo" <tipoalgo@bol.com.br>
TO: ciencialist@yahoogrupos.com.br
DATE: 07/02/2005 11:50


[Dino] escreveu
...

O assunto é complexo, mas merece um estudo adequado, pois a
enorme quantidade de energia atualmente desperdiçada indica
que o assunto deveria ter sido considerado há decadas.

..."


[Tipoalgo] desabafou

O problema é exatamente este, quem se habilita a debater, opinar, se
expor. Admiro o Sr. Fendel por isso, parece ser um dos poucos a ter
esta consciência. Onde andam os mestre da lista? No carnaval parece
que não. Se existisse mais Fendéis por ai, duvido que a PRF teria
apreendido o seu carro experimental de óleo vegetal.

Trouxe este assunto aqui pra lista porque estou indignado com esta
nossa passividade. Muitos aqui são formadores de opinião e devem se
comprometer com o futuro, se sentir responsável por várias mazelas
que agridem o bom senso. Vamos nos importar mais com a realidade. De
que adianta uma ciência avançada se os políticos, lobistas e
burocratas impedem sua utilização.

Poucos se importam!

Acho que estou precisando ir pular no bloco dos desesperados!

"A vida continua, mesmo com a minha `índio`ssincrasia."

Tipoalgo; sem abraços mesmo!








SUBJECT: FIELD RESONANCE PROPULSION CONCEPT (NASA)
FROM: "marcelomjr" <marcelomjr@yahoo.com.br>
TO: ciencialist@yahoogrupos.com.br
DATE: 07/02/2005 16:22


http://www.keelynet.com/energy/holt1.htm

JSC-16073
(NASA-TM-80961)
FIELD RESONANCE PROPULSION CONCEPT (NASA)
N80-19184 13 p HC A02/MP A01 CSCL 21C
G3/20 14761 - August 1979
NASA - National Aeronautics and Space Administration
Lyndon B. Johnson Space Center
FOREWORD
The speculative "propulsion" concept described in this paper was
presented at a special session of the 15th Joint AIAA/SAE/ASME
Propulsion Conference (June 18-20, 1979), "Propulsion Concepts for
Galactic Spacecraft". The concept was developed as the result of
private, unofficial research. NASA is not involved in UFO research.
However, the research which may be stimulated by this paper could
result in the verification of essential elements of this concept and
in feasibility studies concerning the development of a new generation
of NASA spacecraft. - Alan C. Holt
ABSTRACT
A new propulsion concept has been developed based on a proposed
resonance between coherent, pulsed electromagnetic wave forms and
gravitational wave forms (or space-time metrics). Using this concept,
a spacecraft "propulsion" system potentially capable of galactic and
inter-galactic travel without prohibitive "travel times" has been
designed. The "propulsion" system utilizes recent research associated
with magnetic field line merging, hydromagnetic wave effects, free-
electron lasers, laser generation of megagauss fields, and special
structural and containment metals. Research required to determine
potential, field resonance characteristics and to evaluate various
aspects of the spacecraft "propulsion" design is described.

FIELD RESONANCE "PROPULSION" CONCEPT
ASSUMPTIONS
The field resonance "propulsion" concept has been developed utilizing
(...)

http://www.keelynet.com/energy/holt1.htm





SUBJECT: BREAKTHROUGH PROPULSION PHYSICS
FROM: "marcelomjr" <marcelomjr@yahoo.com.br>
TO: ciencialist@yahoogrupos.com.br
DATE: 07/02/2005 16:26


http://www.earthtech.org/publications/AIAA9921431.pdf

AIAA-99-2143
SPACE TESTING OF ELECTROMAGNETICALLY SENSITIVE MATERIALS FOR
BREAKTHROUGH PROPULSION PHYSICS
Alan C. Holt, Senior Member
NASA Johnson Space Center*, Houston, TX
Eric W. Davis, Ph.D.
National Institute for Discovery Science, Las Vegas, NV
Hal Puthoff, Ph.D.
Institute for Advanced Studies, Austin, TX
Recently it has been experimentally demonstrated that non-local
(instantaneous) communication between two beams
of light (spin direction) can randomly occur. The effect is described
as a consequence of the physics of quantum
mechanics. Other research has experimentally demonstrated that the
zero point radiation/fields (ZPF), which pervade
space-time, can effectively be shielded so as to force two parallel
plates together (Casimir effect). The NASA
Marshall Space Flight Center is investigating the possibility that a
spinning superconductor can cause a reduction in
the weight of nearby objects. Recent ultrahigh-intensity, peta/eta-
watt tabletop lasers have achieved extreme electric
and magnetic fields, pressure, temperature and space-time curvature
that can only be found close to a black hole
horizon. Based on these research and experimental efforts, some
examples of space experiment and
materials/technology testing approaches have been examined to
determine the feasibility and potential benefits of
using the International Space Station (ISS) to address breakthrough
propulsion physics and technology challenges.
The ISS's microgravity environment, combined with the access to an
extreme vacuum and plasma environment, offers
some unique advantages for the testing of various
electromagnetically "sensitive" materials, and associated physical
interactions. The use of this space laboratory could enable much more
rapid progress in the identification and
optimization of anomalous and highly nonlinear effects. A modular
Breakthrough Propulsion Physics testbed could
be developed using one or two mid-deck locker equivalent volumes in a
pressurized Express Rack, which would
enable the operation or testing of various experiments and devices as
part of a long duration, breakthrough physics
and technology testing program. Similarly, an Express Pallet adapter
site could be used as a breakthrough propulsion
physics testbed for those experiments or technology demonstrations
which require direct access to the space
environment and larger operating volumes. Breakthrough propulsion
physics experiments could also be integrated
into (or use extra space in or on) micro-/nano- technology (MNT)
experiments which are already under development.
The International Space Station's research capabilities provide
important opportunities for achieving early
breakthroughs in physics understanding, and the associated
materials/technology interactions, needed to accelerate
advances in space vehicle/platform maneuvering and transport.
Introduction
A number of recent advances in theoretical and
experimental physics and materials development are
providing insight into (...)





SUBJECT: Re: [ciencialist] Re: Zodiaco
FROM: "Sergio M. M. Taborda" <sergiotaborda@terra.com.br>
TO: ciencialist@yahoogrupos.com.br
DATE: 07/02/2005 17:06

Oraculo wrote:

>
>
> Taborda: Ninguem está discutindo a eficácia da astrologia.
>
> Bem, se isso fosse verdade, foi um enorme desperdicio de tempo e
> esforço..:-)
>
> Veja, se você, mesmo afirmando que somos tolos enganados e que nada
> sabemos sobre astrologia, conclui que ela não funciona como alega (e
> que data e hora nãp permitem saber como a personalidade de alguém
> será), a discussão acaba aqui..:-)

Vc veio-se intormeter na conversa que tinha a ver com os fundamentos da
astrologia, querendo referir-se à eficácia da astrologia, entáo o erro
foi seu.
Eu não afirmo nem nego nada pois não tenho provas , nem sequer
evidencias, nem para um lado, nem para o outro, sobre a eficácia da
astrologia. Tenho apenas a minha experiencia pessoal.
Como eu não sei nada sobre leitura fria, duvido que eu possa estar
fazendo isso quando intrepreto um mapa
Eu so digo, que antes de vc poder criticar se funciona ou não, precisa
entender como funcionaria em teoria, para depois testar se funciona na
prática. Só assim se pode levar a cabo experiencias que tenham como
resultado coisas palpaveis e não coisas que um diz que si e o outro diz
que não. Isso é que é ser cientifico, e não o que vc faz que é criticar
sem saber do que está a falar.

Sérgio Taborda


--
No virus found in this outgoing message.
Checked by AVG Anti-Virus.
Version: 7.0.300 / Virus Database: 265.8.6 - Release Date: 07-02-2005



SUBJECT: Re: [ciencialist] p/Victor - Zodíaco
FROM: JVictor <jvoneto@uol.com.br>
TO: "ciencialist@yahoogrupos.com.br" <ciencialist@yahoogrupos.com.br>
DATE: 08/02/2005 00:06





Murilo,


murilo filo escreveu:

> >Experiência não é crença.

Victor: Evidentemente você refere-se a "experiência" pessoal, que é
vivenciada única e exclusivamente por aquela pessoa e por mais ninguém.
Isso não é experiência, no sentido em que se emprega essa palavra no
jargão científico. As sensações e/ou visões são pessoais, com
interpretações pessoais a nível de sua própria mente. Outras pessoas
podem "sentir" interpretações diferentes, a depender de seu quadro de
referências, daquilo em que acredita! São coisas interiores, não
reproduzíveis na hora em que se deseja. Sem exceção, ouso afirmar:
pessoas com certo nível de carisma, que tenham tido alguma dessas
"experiências" costumam ser ouvidas pelo círculo de pessoas que lhe têm
como lider, seja do que for, e tendem a seguir seus ensinamentos, suas
pregações. A história está cheia de coisas assim. Sem exceção, o nível
de crenças, a estrutura social, o desespero, as necessidades, entre
outras coisas, ensejam fortemente a que uma promessa de salvação e a
esperança embutida no fato de que há uma luz no fim do túnel, exacerbada
por esses "gurus", conduz ao que já sabemos. Nem sempre todos os membros
de um grupo sabem mais do que ler, arrastado, a bíblia ou a cartilha do
guru ou da seita. Muitos podem ser pessoas cultas, que estudaram, podem
saber muito de ciência, podendo até mexer bem com coisas como
matemática, física, biologia, história, etc. A questão não está aí. Ela
está, e sempre esteve, no " modus operandi" do processo da formação
mental dessas pessoas, vinculado fortemente à geografia do lugar, à
cultura e estrutura da sociedade e da família. Essas circunstâncias
acabam moldando o nosso comportamento, perante nós mesmos e as
solicitações da vida. Acredito que o móvel de nosso comportamento vem
da mente, de nosso inconsciente e isto é determinante. Não tenho porque
acreditar em outra coisa. Pensando em certos comportamentos dos animais
ditos irracionais, descobrimos rapidinho que há coisas que eles fazem,
que devem independer da vontade(até em razão de eles não a terem), tais
como: alimentar-se, defender-se, aplacar a sede, expelir resíduos do
organismo, acasalamento, buscar a vivência em grupos. Ora, mas com nós
outros, ditos racionais, acontece a mesma coisa. Somos compelidos a
buscar alimentos, água, banheiro, etc. Essas coisas independem de eu
querer ou não. São compulsórias. Mas há outras ações só comuns aos
animais ditos racionais, ou seja nós, que não há nos irracionais. São
certos componentes do comportamento humano que se moldam conforme o
quadro de referências montado de acordo com os fatores acima listados. E
que determinam o nosso próprio jeito de ser, em todas as suas matizes.
Uma vaca criada aquí, no meu quintal, vai ter o mesmo comportamento da
mesma vaca, se criada em alguma região da China, África ou Iraque. Já
com o ser humano, tudo sugere que o comportamento seria outro, pois os
valores agregados seriam diferentes, alguns dos quais nos repugna, por
serem monstruosos e medievais para quem nasceu e se criou por cá. E
onde está a diferença? No corpo não está, é patente, isso. Todos os
humanos têm estrutura orgânica e funções semelhantes. O único
diferencial parece ser a mente. Esta sim, é que é o nó da questão. É
aquela velha história da consciência, que me permite saber que não sei,
por exemplo. Tomar "tento" de minha própria existência, em todas as suas
nuances. E é a única diferença, em termos mentais, entre nós e os
irracionais; mas que diferença!
Psicólogos e psiquiatras de todas as eras sempre se ocuparam disso,
desse comportamento mental. Aventaram muitas hipóteses, das quais muitas
fazem sentido, por isso que ficaram, sendo usadas como parâmetros de
estudo. Dentre estas, estão as que regem o comportamento humano. Joseph
Nutrin, sistematizando descobertas de pessoas como Jung, Adler, Freud e
outros, aventou a forte hipótese de que o inconsciente é regido por um
princípio vital, que é o tal nó da questão a que me referí acima. Seria
esse princípio vital o responsável pela maneira como nos conduzimos.
Alguns estudiosos o apelidaram com nomes como Impulso do Desenvolvimento
Vital, Impulso do Desenvolvimento Global, Impulso de Atualização de sí
próprio, para designarem essa energia que alimenta e conduz nosso
comportamentol. Por exemplo, seria essa a energia que me permite viver,
com todas as minhas mazelas ou não, sem que eu precise saber como. Como
uma vaca. Já uma rocha, não apresenta quaisquer das manisfestações dessa
energia, desse impulso vital.
Buscando algo que relacione esses fatos, comuns aos seres vivos(não sei
se com vegetais ocorre o mesmo, em outro nível. Não pesquisei a
respeito, ainda) encontrei algumas respostas satisfatórias-para mim,
claro-, que tentarei descrever pormenorizadamente, pois isto reflete
mais ou menos o que penso a respeito, como esbocei no e-mail anterior,
de maneira bem genérica. Dentro de tal sistematização, identificou-se
que O Impulso do desenvolvimento Vital ou Global(IDV/G) pode ser
caracterizado por atos que se situam em três níveis comportamentais,
tais como: psicossocial, psicoespiritual e psicobiológico. Com tal
organização, fica simples raciocinar e explicar certos comportamentos.
Em cada um desses níveis, esse IDV, se expande e mostra a que veio! O
terceiro nível acima responde por atos como beber, comer, ir ao
banheiro; o primeiro responde pelos atos: conversar, comunicar, expor
pensamentos(como eu e você, aquí) já coisas como meditar, perguntar,
questionar, conjeturar, concordar e discordar, são expressões do IDV,
sob a segunda modalidade. Como somos nosso próprio laboratório de
pesquisas, não fica difícil concluir que essa é a trilha e ser seguida,
e que deve estar correta. Tudo isto se configura como forças vitais,
interiores. Que têm os seguintes apelidos: necessidades básicas,
tendências fundamentais ou instintos. Entre estes, sagrou-se
"instintos", por ser mais simples de dizer e estar mais na boca de
todos. Tais impulsos inconscientes determinam, segundo os mestres da
área, como somos hoje.
Dentro da mesma linha de sistematização, para cada um dos níveis acima,
identificaram-se impulsos associados, de extrema e vital importância em
nossa formação, ao longo da vida. São eles: Impulsos de:
auto-afirmação e de sociabilidade, determinantes do nível psciossocial;
impulso sexual e de autoconservação, associados ao nível psicobiológico;
e, finalmente, dois impulsos ligados ao nível psicoespiritual, que são:
impulso de autotranscedência e impulso de cogitação do sentido da
existência. Não foram identificados, ainda, estes últimos "
ingredientes" em animais irracionais! Cada um desses impulsos determina
nossas posturas em cada um dos níveis. Acredito que todos nascemos com
esses impulsos na estaca zero, que vão se desenvolvendo em função das
condições em que nos formamos. Tendo em vista esta linha de raciocínio,
permita-me comentar, à guisa de exemplo, cada um dos tais. No nível
psicobiológico, são manifestações do Impulso de auto-conservação:
necessidade de se defender, de comer, de beber, de brincar, de recrear,
buscar lazer, fazer aeróbica ou outra forma de exercício físico. Se este
impulso não é bom formado, se fatores externos interferem, a pessoa
deixar de ter prazer por sua própria vida, é desleixado, coisas do
tipo(já pensou um cirurgião com tal débito, te operando?); e o sexual,
ao qual Freud chamou de libido, que nos move em direção ao "pecado" mais
divino da existência!.... Este impulso é responsável por um
comportamento ético nessa área, sem angústias, construtivo, produtivo,
saudável e prazeirozo, ou o contrário disso tudo. . Pessoas que tiveram
esse impulso desenvolvido de forma inadequada, em vista dos mesmos
motivos já mencionados, durante sua formação, tendem a ter um
comportamento esquesito, até patológico às vezes. Quem não está
resolvido sexualmente, sofre e faz os outros comerem o pão que o diabo
amassou. Dá uma olhada á tua volta, na história, e veja se não é
possivel encontrar manifestações de um impulso sexual "troncho", em
algumas pessoas. Não conheço nenhuma exceção a isso, pelo menos em
termos do que já lí a respeito. O impulso de sociabilidade é aquele que
me faz gregário, sociável; é aquele impulso que faz que pessoas
escrevam, e-meiem e telefonem para outras, reunam-se, procurem-se,
formem grupos. Nesse mesmo nível psicossocial há um outro impulso, o de
auto-afirmação que dirige o homem por trilhas que incomodam outros, que
estão na linha de tiro, sobretudo se são subalternos... Um psicólogo
chamado Alfred Adler, que também correu de Freud, como Jung, "resvalou
na tentação de reduzir todo o comportamento humano, não apenas o
comportamento profissional, a expressões diferentes do impulso de
auto-afirmação", apontou um estudioso brasileiro do comportamento
profissional e pessoal do ser humano(J.Mohana, se não me falha a
memória). É o impulso de auto-afirmação que nos compele a nos impor,
que nos empurra para ações que nos ponham no foco, em destaque, em
qualquer das áreas em que atuemos. Quem não quer ser reconhecido por
algo que faça, por algo que pregue? Ninguém quer ser ignorado. Cada um
quer ser o centro das atenções, em menor ou maior grau. No fundo, no
fundo, por mais ético que seja, por mais desprendido que pretenda ser,
esse desejo inconsciente está lá, quietinho, nos empurrando para o
mister. Que, inclusive, pode até ser patológico, e ainda causar prazer e
satisfação. É um dos tais que leva muita gente ao divã. Agora, para não
me alongar mais, há os dois impulsos presentes no nível psicoespiritual:
o de cogitação do sentido da existência e o de autotranscendência. O de
cogitação do sentido da existência exprime o que o próprio nome quer
dizer. Quem não se questiona ou se questionou com coisas como: diabos, o
que estou fazendo aquí, o que é a vida, etc.? Qual a razão, existirá uma
razão? Por aí. O de autotranscendência é o mais comum e catalizador.
Está ligado ao desconhecido, ao aspecto religioso das pessoas. É, por
isso, chamado também de Impulso religioso, devido á compulsão das
pessoas por questões místicas, religiosas, desconhecidas, enfim. É o
impulso responsável pela busca de um ser supremo, de uma unidade, da
completude. Cada um de um de nós tem um ou outro mais desenvolvido,
predominante.
Agora, assumindo, como eu, a plausibilidade desses entes a nível
inconsciente, fica fácil entender que o comportamento do ser humano está
na dependência de como esses impulsos se desenvolveram. É por isso que
ninguém é igual nessa história de comportamento, pois isto depende do
nível final desses entes, nível esse que se estabelece em função das
referências durante a evolução e a formação da mente. Imagine que
alguém tenham nascido numa família e numa sociedade altamente
religiosa, e cuja cultura seja sacrificar humanos ou vacas para aplacar
a ira de seus deuses. Esse ser terá assumido isso como uma verdade
incontestável, e, se se intelectualizar, em geral, poderá defender e
escrever suas crenças, buscando impô-las. E aí de quem não seguir as
linhas traçadas. Exemplos? (Frequente algumas igrejas, alguns cultos, de
diferentes filosofias, como eu já fiz, à guisa de ouvir para confirmar,
e observe). Além, é claro, de enfiar o punhal na garganta do escolhido,
acreditando firmemente que os deuses gostarão e aquilo será uma honra
para o sacrificado. Se tiver uma formação numa família degenerada
sexualmente, poderá, em função disto, comportar-se semelhante. E por aí
em diante. Exemplos para os demais impulsos não faltam. Basta refletir
um pouco. ´
É o medo da morte, como descreví no e-mail anterior, associado à
ignorância(no sentido em que emprego esta palavra, que sugere um
embotamento da consciência; algo como um jogador de xadrez que não
consegue ver uma jogada salvadora ou preparatória para o desfecho final,
e outro vê sem esforço; ou como a solução de um problema matemático,
cuja resposta estar a depender de um pequeno e simples detalhe, que não
foi visto; e que outro viu, pelo que a solução veio rápida e simples,
também. É nesse sentido que emprego o termo ingorância: embotamento da
consciência, por motivos diversos e não explicáveis por nós),
sobretudo, que parecem exacerbar em maior ou menor grau esse tal impulso
de autotranscendência.
Enfim, acredito, em face do descreví, que nosso comportamento não é
ditado pelo que os deuses dizem ou pela posição desta ou daquela
estrrela, constelação ou planeta. A coisa está mais perto, dentro de nós
mesmos.
Em suma, por que são inconscientes, os impulsos definem nosso "esqueleto
psicológico" moldam nossa maneira de ser, inexoravelmente. Ninguém, ou
melhor, a grande maioria, aje por má fé, quando prega ou defende
tal ou qual sistema de conhecimento. As coisas podem ser mudadas,
alteradas? Claro. É só trabalhar esses tais. Dinheiro, divã e
profissional competente.
Enfim, grande Murilo, estas algumas das razões pelas quais,
particularmente, não acredito em astrologia, biblia, religiões ou
semelhantes. Embora não descarte que certas coincidências são um osso
duro de roer e incomodem. Mas, deste meu ponto de vista, nada que se
relacione, ou que tenha se relacionado, com posições de astros no cosmo.
O sistema que Buda ensinou, de buscar o conhecimento dentro de você
próprio é, ao meu ver, correto. Infelizmente, exige muita disciplina e
desprendimento(que, em mim, são bem pouquinhos) e também inteligência
em certa dose, que em mim também falta um poucão. Então fico por aquí,
em minha ignorância de não conseguir enxergar nadinha do que os
astrólogos e místicos dizem ser tão claro e lógico. E persigo aquilo que
me parece mais consentâneo com o que vai dentro de nossa mente que,
repito, é a única coisa que verdadeiramente somos. O resto é acessório e
tudo se "pela" do medo da morte, mesmo a nível inconsciente. E o
culpado? O instinto ou impulso de autoconservação! Pura e simples.

> Murilo: O êrro, o engano e a ignorância está em toda parte. A coisa
> pode ser um
> pouco mais complicada do que pensar que quem segue, ou ouve, um
> recurso como
> a astrologia, é burro e só está procurando uma anestesia, uma
> alienação ou
> qualquer submersão supersticiosa e refrescante.

Victor: É verdade, em relação à tua primeira frase. No que se refere ao
resto da sentença, creio, esclarecí nos comentários anteriores. E quem
"pensar que quem segue, ou ouve, um recurso como a astrologia, é burro e
só está procurando uma anestesia, uma alienação ou qualquer submersão
supersticiosa e refrescante" está equivocado, e deve tomar " biotônico
fontoura", para ajustar o juízo.
E assim caminha a humanidade.

Sds,

Victor.

>
>
> <http://br.rd.yahoo.com/SIG=12aukg7h4/M=264379.5078783.6203979.1588051/D=brclubs/S=2137111528:HM/EXP=1107815712/A=2191897/R=0/SIG=10vqa2grn/*http://br.diversao.yahoo.com/>
>
> <http://br.rd.yahoo.com/SIG=12aukg7h4/M=264379.5078783.6203979.1588051/D=brclubs/S=2137111528:HM/EXP=1107815712/A=2191897/R=1/SIG=10vqa2grn/*http://br.diversao.yahoo.com/>
>
>
>
> ------------------------------------------------------------------------
> *Links do Yahoo! Grupos*
>
> * Para visitar o site do seu grupo na web, acesse:
> http://br.groups.yahoo.com/group/ciencialist/
>
> * Para sair deste grupo, envie um e-mail para:
> ciencialist-unsubscribe@yahoogrupos.com.br
> <mailto:ciencialist-unsubscribe@yahoogrupos.com.br?subject=Unsubscribe>
>
> * O uso que você faz do Yahoo! Grupos está sujeito aos Termos do
> Serviço do Yahoo! <http://br.yahoo.com/info/utos.html>.
>
>
>
>
> __________ Informação do NOD32 1.990 (20050202) __________
>
> Esta mensagem foi verificada pelo NOD32 Sistema Antivírus
> http://www.nod32.com.br







SUBJECT: P/ Felipe - Zodiaco
FROM: "Oraculo" <oraculo@atibaia.com.br>
TO: <ciencialist@yahoogrupos.com.br>
DATE: 08/02/2005 01:30

Olá Felipe


Felipe:Há muitas centenas de anos se desenvolve uma "logia" e se perdura até hj é pq alguma validade tem."

Suas ponderações são interessantes e muitas delas pertinentes. Mas a expressão acima incorre no erro básico de avaliação nas alegações extraordinárias, que acabam sendo a base da sustentação de coisas como astrologia. Se desenvolve uma lógica própria e perdura até hoje NADA SIGNIFICA quanto a sua validade..:-)

Um erro ou engano muito antigo não se torna verdadeiro devido ao tempo passado, nem ao número de pessoas que acreditam nele (vide as religiões..:-).

A forma mais confiável de saber se uma alegação é real, é testa-la com ferramentas desenvolvidas exatamente para isso, para verificar a correpondência com a realidade. A melhor e mais eficaz delas é o método cientifico, pelo menos para aspectos do universo físico.

"Felipe: A astrologia não é uma ciência exata, e uma resposta linear seria negar o mistério que perdura por trás dela. "

A astrologia não é ciência, nem exata nem inexata, já que não se encaixa no conceito do termo. E negar o místério é exatamente o que se pretende ao tentar descobrir como qualquer coisa age, funciona ou se manifesta..:-) Defender qualquer alegação com a assertiva de que o mistério é parte integrante e indissoluvel da mesma, a retira definitivamente do alcance da compreensão e do exame cientifico. É o mesmo que os mistérios religiosos (de qualquer religião) que estão fora, por definição, de análise ou debate: se crê ou não e pronto.

A questão aqui é simples. O conjunto de alegações e ações conhecido como astrologia (na forma que se desejar, seja a do Taborda, seja a do Walter Mercado), pode ou não determinar aspectos da personalidade, caráter e comportamento de seres humanos, e determinar eventos futuros diversos. Se pode, isso pode ser testado e analisado, de forma a comprovar a realidade da alegação.

É só isso, nada mais..:-) Mesmo eliminando os evidentes charlatões, e aspectos mais folcloricos como horoscopos de jornais, ainda é a mesma base que sustenta a astrologia (e os astrologos de jornais dificilmente concordariam com sua exclusão, usando os mesmos argumentos sobre o misterio e inexatidão de seu conhecimento..:-) e a mesma falta de evidencias que permitam uma conclusão positiva quanto a realidade deste fenomeno.

Um abraço.

Homero

----- Original Message -----
From: Felipe Alvisi Galastro Perez
To: ciencialist
Sent: Monday, February 07, 2005 9:04 AM
Subject: RE: [ciencialist] Zodiaco


Soh pitacos de quem vem avidamente acompanhando a discussão:

[Sérgio (e outros pro-astrologia),
>
> Porque é que os astrólogos só ganham dinheiro a fazerem consultas para as
> outras pessoas? Não conseguem determinar o seu próprio futuro? Não conseguem
> analisar as probabilidades de serem bem sucedidos num negócio? Porque é que
> todo o dinheiro que têm costuma ser aquele "retirado" aos seus clientes? No
> Top500 da Forbes duvido que conste um único astrólogo de profissão. Seria
> isso provável de acontecer no caso de realmente haver qualquer capacidade de
> previsão?]

Caro Hugo,
penso que toda a discussão que vem sendo travada nessa thread pode ser explicada pela idéia destas suas indagações. Todo o "temor" ou pré conceito em relação à astrologia provém da estereotipada idéia pós moderna de que además à explicação científica para tudo, tudo tem que ter também uma explicação prática... O que quero dizer é que a exigência não é mais tão somente por expllicações astronômicas para as interpretações astrológicas, pois exige-se também uma explicação para erros cometidos a a menudo na TV por charlatães, para Valter Mercados da vida, para "por que os astrólogos não são todos ricos e felizes?"... Isso afasta muuuiito a astrologia de um âmbito em que pode ser discutido com cientificidade.
O que o Sérgio Taborda vem fazendo, e tenho que admitir que concordo e avalizo inteiramente (se é q isso vale algo!), é transportar o assunto para um módulo em que se possa ver ciência por trás de algo obscuro, ininteligível, por que não. Exatamente pelos pequenos detalhes cotidianos que compartilhamos numa rede velada com amigos, tv, pais, pessoas que intrinsecamente desvalorizam todo o tempo algo de que não têm o menor conhecimento, como pq os astrólogos não são ricos e felizes?, hehe, ou pq a mão diná (se é q é astrologa, nem sei) errou no gugU do ano passado, é que temos a visão distorcida. E mesmo depois de todo o conhecimento agregado após a discussão de 4a série sobre pq a mãe diná errou, transportamos o estereótipo para o mundo dos "intelectuais".
Nem de longe sou um defensor da astrologia, sou um defensor do ceticismo racional, sem apelo ao intangível para justificar tudo com argumentação repetitiva. Como já foi dito aí pra trás, duvidemos de nossas dúvidas. Há muitas centenas de anos se desenvolve uma "logia" e se perdura até hj é pq alguma validade tem. Amanhã, podemos descobrir que estávamos errados todo o tempo e que existe toda sorte de "energias", "magias" e outras coisas q hj recriminamos.
Mudando um pouco o rumo, mesmo para aqueles que discordam da charlatanisse da astrologia, é inegável que a parametrização do comportamento humano provido por ela é muito contundente.. Bem como a matemática envolvida. Realmente, ninguém aqui vai explicar pq um leonino é um leonino por causa da posição de astros na hora do seu nascimento. Sei que o que querem os "puristas" é que algum dos "não afirmo nada pq a astrologia é um conhecimento muito grande para descartá-lo por pré-conceitos" dê a resposta para essa pergunta para que possam categoricamente refutá-la com alguma contra argumentação do estilo leitura-fria, ou estatística barata, mas convenhamos, tão tolo quanto esta argumentação seria alguém se arriscar com a resposta. A astrologia não é uma ciência exata, e uma resposta linear seria negar o mistério que perdura por trás dela. Tudo que se vem discutindo, é por uma tentativa cartesiana de explicar algo desconhecido. Agora, se pudermos abrir os olhos e discutir a matemática e astronomia por trás da astrologia, que é cartesiana, aí sim chegaremos a algum improvement geral de conhecimento. Se todas as vezes que uma explicação física for refutada com uma retórica retomada de estereóticos infantis acerca de mapas astrais e bobalhões q vão à tv para anunciar idiotices, bem... será mais do mesmo.

[]'s
FAGP


---------- Início da mensagem original -----------
De: "Hugo Santos" urano@netvisao.pt
Para: ciencialist@yahoogrupos.com.br
Cc:
Data: Mon, 7 Feb 2005 00:35:43 -0000
Assunto: RE: [ciencialist] Zodiaco
>
> Sérgio (e outros pro-astrologia),
>
> Porque é que os astrólogos só ganham dinheiro a fazerem consultas para as
> outras pessoas? Não conseguem determinar o seu próprio futuro? Não conseguem
> analisar as probabilidades de serem bem sucedidos num negócio? Porque é que
> todo o dinheiro que têm costuma ser aquele "retirado" aos seus clientes? No
> Top500 da Forbes duvido que conste um único astrólogo de profissão. Seria
> isso provável de acontecer no caso de realmente haver qualquer capacidade de
> previsão? Mas mesmo que assumamos que os astrólogos (todos) não estão
> interessados em ficarem ricos (o que não parece ser nada o caso), podemos
> colocar em causa o seu grau de felicidade, o seu grau de sucesso. Não tenho
> conhecimento de que as pessoas que praticam astrologia ou que a ela recorrem
> sejam SIGNIFICATIVAMENTE mais "felizes" ou "bem sucedidas" do que todas as
> outras que não são minimamente crentes, ou que acreditam noutros tipos de
> crenças (como as religiões, por exemplo).
>
> Por outro lado, ao dizer que pode prever com alguma precisão as condições
> que poderão influenciar decisões futuras, está nesse momento a influenciar
> as decisões actuais. Como tal, nesse momento, o futuro já foi alterado. Ou
> seja, usar a astrologia para qualquer "cálculo" futuro é irrelevante, porque
> nesse momento já se alterou o momento actual e, consequentemente, o futuro.
> Como muito bem disse o Sérgio, as condições iniciais são bem importantes e
> cada momento presente é uma condição inicial para qualquer momento futuro.
> Se a previsão afecta o momento presente, o futuro também é afectado e a
> previsão deixa de fazer sentido. Ou seja, a astrologia baseia-se numa
> situação paradoxal que, a meu ver, a desacredita imediatamente.
>
> Ora, usar a astrologia para cálculos passados não serve para nada. O passado
> já todos nós sabemos o que foi. Ninguém quer fazer "previsões" sobre o
> passado. Se o presente é o que é, se o passado foi o que foi e se o futuro
> não pode ser previsto porque a previsão o afecta, então a astrologia não
> serve para nada.
>
> Devo, no entanto, reconhecer um uso para a astrologia. É um aspecto
> importante da esperança de certas pessoas. Basicamente é só mais uma fonte
> de esperança. É um pouco como o efeito placebo que foi inúmeras vezes
> encontrado em investigações científicas relativas ao efeito de medicamentos
> ou situações experimentais. O sentir que têm mais controlo sobre o seu
> futuro do que aquele que realmente têm, o "saber" que as coisas vão correr
> bem, mesmo que isso possa ser verdade ou não, pode realmente influenciar a
> forma como essa pessoa encara esse acontecimento futuro e é esse prévio
> "conhecimento" que pode influenciar o decorrer desse acontecimento. É sabido
> que uma pessoa que faz as coisas com um sentido positivista, deverá ter uma
> probabilidade maior de ser bem sucedida do que uma que o faça de forma
> negativista. Não desistir antes de as coisas começarem costuma ser um bom
> ponto de partida para se ter sucesso.
>
> Desta forma, coloco a astrologia no mesmo ponto que a religião. Simplesmente
> servem para colmatar lacunas na psicologia das pessoas. Quem precisar dessa
> "fé" que a use, mas que tenha a noção de que é uma coisa pessoal e que pode
> não ter nada a ver com as necessidades de apoio psicológico doutro
> indivíduo. Desta forma, tentar convencer os outros de que a nossa crença,
> seja ela qual for, é melhor do que as outras, é perfeitamente absurdo e
> ABUSIVAMENTE enganadora (o mesmo se aplica a cientistas).
>
> Um abraço,
>
> Hugo Santos
>
> P.S.: Já agora, aproveito para fazer um reparo. A quantidade de erros
> gramaticais e ortográficos encontrados nos mails desta lista é simplesmente
> assustador. Alguns erros passam e são perfeitamente aceitáveis, mas por
> vezes chego mesmo a perguntar-me se se está a escrever em português. Por
> favor caros colegas, tenham um pouco mais de cuidado com a forma como
> escrevem. Assassinar a língua não é uma coisa muito nobre de se fazer. Como
> já outrora ouvi, se não têm tempo para usar um corrector ortográfico, então
> não têm tempo para andar para aqui a escrever.
>
>
>
>
> ##### ##### #####
>
> Para saber mais visite
> http://www.ciencialist.hpg.ig.com.br
>
>
> ##### ##### ##### #####
> Links do Yahoo! Grupos
>
>
>
>
>
>
>
>
>

__________________________________________________________________________
Acabe com aquelas janelinhas que pulam na sua tela.
AntiPop-up UOL - É grátis!
http://antipopup.uol.com.br/



[As partes desta mensagem que não continham texto foram removidas]



##### ##### #####

Para saber mais visite
http://www.ciencialist.hpg.ig.com.br


##### ##### ##### #####


Yahoo! Grupos, um serviço oferecido por:
PUBLICIDADE




------------------------------------------------------------------------------
Links do Yahoo! Grupos

a.. Para visitar o site do seu grupo na web, acesse:
http://br.groups.yahoo.com/group/ciencialist/

b.. Para sair deste grupo, envie um e-mail para:
ciencialist-unsubscribe@yahoogrupos.com.br

c.. O uso que você faz do Yahoo! Grupos está sujeito aos Termos do Serviço do Yahoo!.



[As partes desta mensagem que não continham texto foram removidas]



SUBJECT: Re: [ciencialist] Re: Zodiaco
FROM: "Oraculo" <oraculo@atibaia.com.br>
TO: <ciencialist@yahoogrupos.com.br>
DATE: 08/02/2005 01:51

Olá Taborda

Bem, primeiro não posso ser considerado intrometido apenas por dar palpites em uma lista de discussões onde se entra exatamente para dar e receber palpites..:-) Não é justo dizer que eu vim me intrometer na conversa..:-)

Segundo, a conversa sobre a astrologia e seus métodos diz especificamente sobre sua eficácia, ou seria uma conversa inútil, desde o começo. Você não entrou na conversa dizendo "pessoal, claro que a astrologia não tem evidencias de funcionar, mas vocês estão enganados sobre a aplicação de sua matematica, ou sobre quais os astros são considerados nos cálculos" ou algo parecido. Toda discussão era sobre como a astrologia, da forma como foi proposta, tinha cálculos que resultavam em afirmações coerentes e precisas (como a sua refutação do uso das caracteristicas dos leoninos pelo Takata).

Terceiro, experiencia pessoal dificilmente validaria uma evidência cientifica sobre qualquer coisa, astrologia ou não.

Quarto, claro que tenho de saber antes de criticar ou analisar, mas não tenho, evidentemente, de saber tudo, mas o que for pertinente. No caso da astrologia devo saber que, em sintese, ela usa posições e relações de corpos celestes para conhecer aspectos da personalidade, carater, e eventos futuros de seres humanos.

Não importa se me enganei nos astros, se são apenas os planetas, o Sol e a Lua ou todas as constelações, isso é irrelevante para a análise, já que todos eles partilham da mesma falta de um mecanismo de ação, de evidencia e de resultados verificáveis.

Também não importa como são feitos os calculos do mapa astral, se não há evidencias de que exista uma forma de relacionar os resultados desses calculos com a personalidade de qualquer pessoa.

Então, no que interessa, eu sei do que estou falando, mesmo que não conheça os calculos que você usa na construção de um mapa astral (que diferem dos usados por outros astrólogos, mas eles, claro, são uns idiotas e estão evidentemente enganados..:-)

Para terminar, não saber nada sobre leitura fria não impede de usar a técnica de leitura fria, que é derivada da natural habilidade humana de "ler" sinais, informação, reações e qualquer forma de comunicação em diversos níveis em outros seres humanos. Essa é uma habilidade que pode ser refinada e incrementada mesmo que seu possuidor nada saiba sobre ela. É o que chamariamos de uma pessoa " intuitiva", com grande habilidade de se comunicar com outros, lendo informação em niveis quase subconscientes.

Talvez você seja uma pessoa muito habilidosa nesse sentido, apenas isso. E, não seria interessante, para dar maior base a sua refutação de que não é leitura fria que ocorre no mapa astral, aprender mais sobre essa habilidade, sobre outras capacidades da mente humana? Não é parte do sistema cientifico que se ajuste os experimentos a novos dados e novas informações, para aumentar a confiabilidade dos mesmos? Se existe a suspeita ou mesmo a alegação de que leitura fria poderia interferir nos resultados do mapa, não seria lógico e recomendável aprender sobre isso e verificar se, com mecanismos que controlassem essa variável, os mapas ainda seriam precisos?

Homero







----- Original Message -----
From: Sergio M. M. Taborda
To: ciencialist@yahoogrupos.com.br
Sent: Monday, February 07, 2005 5:06 PM
Subject: Re: [ciencialist] Re: Zodiaco


Oraculo wrote:

>
>
> Taborda: Ninguem está discutindo a eficácia da astrologia.
>
> Bem, se isso fosse verdade, foi um enorme desperdicio de tempo e
> esforço..:-)
>
> Veja, se você, mesmo afirmando que somos tolos enganados e que nada
> sabemos sobre astrologia, conclui que ela não funciona como alega (e
> que data e hora nãp permitem saber como a personalidade de alguém
> será), a discussão acaba aqui..:-)

Vc veio-se intormeter na conversa que tinha a ver com os fundamentos da
astrologia, querendo referir-se à eficácia da astrologia, entáo o erro
foi seu.
Eu não afirmo nem nego nada pois não tenho provas , nem sequer
evidencias, nem para um lado, nem para o outro, sobre a eficácia da
astrologia. Tenho apenas a minha experiencia pessoal.
Como eu não sei nada sobre leitura fria, duvido que eu possa estar
fazendo isso quando intrepreto um mapa
Eu so digo, que antes de vc poder criticar se funciona ou não, precisa
entender como funcionaria em teoria, para depois testar se funciona na
prática. Só assim se pode levar a cabo experiencias que tenham como
resultado coisas palpaveis e não coisas que um diz que si e o outro diz
que não. Isso é que é ser cientifico, e não o que vc faz que é criticar
sem saber do que está a falar.

Sérgio Taborda


--
No virus found in this outgoing message.
Checked by AVG Anti-Virus.
Version: 7.0.300 / Virus Database: 265.8.6 - Release Date: 07-02-2005



##### ##### #####

Para saber mais visite
http://www.ciencialist.hpg.ig.com.br


##### ##### ##### #####


Yahoo! Grupos, um serviço oferecido por:







------------------------------------------------------------------------------
Links do Yahoo! Grupos

a.. Para visitar o site do seu grupo na web, acesse:
http://br.groups.yahoo.com/group/ciencialist/

b.. Para sair deste grupo, envie um e-mail para:
ciencialist-unsubscribe@yahoogrupos.com.br

c.. O uso que você faz do Yahoo! Grupos está sujeito aos Termos do Serviço do Yahoo!.



[As partes desta mensagem que não continham texto foram removidas]



SUBJECT: [Fwd: [feyntrad] [dfma-gen] Núcleo de Física em Portugal (fwd)]
FROM: JVictor <jvoneto@uol.com.br>
TO: "ciencialist@yahoogrupos.com.br" <ciencialist@yahoogrupos.com.br>
DATE: 08/02/2005 08:26

Prof. Léo e demais interessados. Projeto interessante.

Sds,

Victor.





Talvez possamos estabelecer algum contato...

---------- Forwarded message ----------
Date: Mon, 7 Feb 2005 12:44:49 -0200 (BRST)
From: Everton Zanella Alvarenga <everton@fma.if.usp.br>
To: Lista da Física Matemática <dfma-gen@fma.if.usp.br>,
"Lista soc-gen (alunos do IFUSP)" <soc-gen@socrates.if.usp.br>,
Grupo dos Takions <takion@yahoogroups.com>
Subject: [dfma-gen] Núcleo de Física em Portugal

Bom tarde.

Por favor, divulguem para os seu alunos e colegas de estudos

http://nfist.ist.utl.pt/index.html

http://fsr2.nfist.ist.utl.pt/

Abraços,

Prof. Léo e demais interessados. Projeto interessante.

Sds,

Victor.



Everton.

Learn from the mistakes of others. You won't live long enough to make
them all yourself.

********************************************************
Universidade de São Paulo, Instituto de Física (IFUSP)
Departamento de Física Matemática

Everton Zanella Alvarenga everton@fma.if.usp.br
WWW: http://fma.if.usp.br/~everton <http://fma.if.usp.br/%7Eeverton>


[As partes desta mensagem que não continham texto foram removidas]



SUBJECT: Fw: Duvidas sobre tensores
FROM: "Luiz Ferraz Netto" <leobarretos@uol.com.br>
TO: "ciencialist" <ciencialist@yahoogrupos.com.br>
DATE: 08/02/2005 08:53

Aceita-se sugestão.
[]'
===========================
Luiz Ferraz Netto [Léo]
leobarretos@uol.com.br
http://www.feiradeciencias.com.br
===========================
-----Mensagem Original-----
De: "Cláudia Adam Ramos" <claudinhadam@yahoo.com.br>
Para: <leobarretos@uol.com.br>
Enviada em: domingo, 6 de fevereiro de 2005 21:32
Assunto: Duvidas sobre tensores


Estive visitando o seu site, Feira de Ciência, e
imagino qu você possa tirar uma duvida minha...sou
acadêmica do curso de matemática e gostaria de
entender o sentido físico do divergente de um campo
tensorial. Não consigo imaginar fisicamente como
funciona o div de uma matriz de tensoes. Se você puder
tirar esta duvida minha ficarei grata,

Desde já agradeço.


Cláudia A. Ramos

__________________________________________________
Converse com seus amigos em tempo real com o Yahoo! Messenger
http://br.download.yahoo.com/messenger/


--
No virus found in this incoming message.
Checked by AVG Anti-Virus.
Version: 7.0.300 / Virus Database: 265.8.5 - Release Date: 03/02/2005




--
No virus found in this outgoing message.
Checked by AVG Anti-Virus.
Version: 7.0.300 / Virus Database: 265.8.6 - Release Date: 07/02/2005



SUBJECT: Extraterrestres: Cientistas Vêem Alta Probabilidade
FROM: "marcelomjr" <marcelomjr@yahoo.com.br>
TO: ciencialist@yahoogrupos.com.br
DATE: 08/02/2005 09:41


http://www.burn.com.br/modules.php?name=News&file=article&sid=288

Visitantes Extraterrestres: Cientistas Vêem Alta Probabilidade
Postado em Domingo, 16 de janeiro de 2005 @ 15:44:08 BRST por josef

Por Leonard David
SPACE.com, 14 de janeiro de 2005

Há décadas atrás o físico Enrico Fermi ponderou sobre a questão de
civilizações extraterrestres com colegas teoristas em um almoço,
gerando a famosa tirada: "Onde eles estão?". Aquela pergunta depois
se tornou central a debates sobre a contagem do censo cosmológico de
outros povos estelares e possíveis visitantes extraterrestres (ET)
vindos de longe.

O pensamento de Fermi no tópico foi depois chamado de "paradoxo de
Fermi". É um conto bem circulado dos anos 50 quando o cientista
abordou o assunto em discussões com colegas em Los Alamos, Novo
México. Pensamentos relativos à probabilidade de planetas parecidos
com a Terra, o surgimento de civilizações altamente avançadas "lá
fora", e viagem interstelar -- estes permanecem a base para tentar
tentado responder ao paradoxo de Fermi mesmo hoje.

Agora um time de cientistas americanos nota que recentes descobertas
astrofísicas sugerem que nós deveríamos nos encontrar no meio de uma
ou mais civilizações extraterrestres. Além disso, eles argumentarm
que é um engano rejeitar todos os relatos de OVNI já que alguma
evidência para as visitas extraterrestres teoricamente-preditas
poderia ser encontradas justamente neles.

Os pesquisadores fazem sua proposta na edição de janeiro/fevereiro de
2005 do Journal of the British Interplanetary Society (JBIS).


Situação curiosa

Pegue qualquer revista boa sobre ciência e você vai ver as mais
recentes idéias de coçar a cabeça sobre a teoria de supercordas,
buracos de minhocas ou o alongamento do próprio espaço-tempo.
Enquanto isso, a detecção de planetas extrasolares está à beira de se
tornar comum.

"Nós estamos na situação curiosa hoje de que nossas melhores teorias
físicas e astrofísicas modernas predizem que nós deveríamos estar
experimentando visitação extraterrestre, contudo qualquer evidência
possível de tal escondida no fenômeno OVNI é ridicularizada dentro de
nossa comunidade científica", defende o astrofísico Bernard Haisch.

Haisch junto com os físicos James Deardorff, Bruce Maccabee e Harold
Puthoff apresentam seu caso no artigo do JBIS: "Inflation-Theory
Implications for Extraterrestrial Visitation".

Os cientistas apontam para duas descobertas chave feitas por
astrônomos australianos e relatadas no ano passado de que há
uma "zona galáctica habitável" em nossa Galáxia Via Láctea. E mais
importante, que a própria estrela da Terra, o Sol, é relativamente
jovem em comparação à estrela comum nesta zona -- por até um bilhão
de anos.

Então, os investigadores explicam em seu artigo no JBIS que uma
civilização alien comum estaria muito mais avançada e teria
descoberto a Terra há muito tempo. Adicionalmente, outro trabalho de
pesquisa na suposição que está por trás do Big Bang -- conhecida como
a teoria inflacionárias -- eleva o prospecto, eles sugerem, que nosso
mundo está imreso em uma civilização extraterrestre muito maior.


Distâncias de ponto-a-ponto

Assumindo física avançada por um bilhão de anos, viajar zunindo pela
galáxia não seria possível?

Até mesmo hoje a teoria de supercordas supõe outras dimensões... que
poderiam ser Universos habitáveis adjacentes ao nosso próprio, os
investigadores especulam. Poderia ser até mesmo possível contornar o
limite de velocidade da luz entrando e saindo destas dimensões.

"O que nós fizemos é um tanto como uma grande inovação", Contou
Haisch para SPACE.com. "Nós reunimos várias descobertas e assuntos
teóricos recentes que coletivamente apontam à probabilidade forte de
que nós devemos nos encontrar no meio de uma ou mais civilizações
extraterrestres gigantescas", disse ele.

Haisch disse que possibilidades de dimensões de supercordas e buracos
de minhoca e dilatação do espaço-tempo respondem à objeção "não se
pode ir daqui para lá" comumente levantada em vista das distâncias
interestelares de ponto-a-ponto envolvidas. Além disso, modelos de
difusão predizem que até mesmo uma única civilização poderia se
espalhar pela Galáxia em uma fração minúscula da idade da Galáxia -
até mesmo a velocidades inferiores à da luz, ele disse.


Assinatura ET nos dados

Pode a comunidade científica considerar qualquer evidência vinda de
avistamentos misteriosos de coisas estranhas pelo público?

Em grande medida, a comunidade científica tem aparentemente visto a
visitação ET como longe de ser algo a ser cogitado seriamente. Por
quê?

"O descarte tem várias causas, todas reforçando umas às outras",
Haisch respondeu. "A maioria das observações provavelmente são
enganos, ilusões e fraudes. Eu vi pessoas serem confundidas por Vênus
ou mesmo Sírio quando estão brilhando cores baixo no céu, com as
condições certas. Tendo sido desanimados por isto, a maioria dos
cientistas nunca se importaram em olhar além, e assim estão
simplesmente alegremente ignorantes de que pode haver mais nisto",
ele disse.

Deardorff, o principal autor do artigo do JBIS, aponta em uma
declaração à imprensa: "Seria necessário um pouco de humildade para a
comunidade científica suspender seu julgamento e tomar pelo menos
alguns dos relatos de alta qualidade a sério o suficiente para
investigá-los... mas eu espero que nós possamos fazer isso."

De acordo com Haisch, há uma motivação não apenas para a tolerância
científica do assunto OVNI, mas uma predição científica forte de que
deveria haver alguma assinatura ET genuína nos dados.

"Isto potencialmente muda a relação do fenômeno OVNI com a ciência de
um modo significante. Retira o preconceito 'não inventado aqui',
mostrando que um 'sim' para a visitação ET é exatamente o lado a que
nossas teorias físicas e astrofísicas atuais apontariam como a
situação mais provável", Haisch concluiu.

Fonte: Space.com
Tradução: Equipe BURN

Para saber sobre o Padaroxo de Fermi, visite:
http://www.str.com.br/ca/pfermi.htm






SUBJECT: Re: [ciencialist] Extraterrestres: Cientistas Vêem Alta Probabilidade
FROM: "Alvaro Augusto \(E\)" <alvaro@electraenergy.com.br>
TO: <ciencialist@yahoogrupos.com.br>
DATE: 08/02/2005 16:30

Acho que a melhor resposta ao paradoxo de Fermi foi dada por David Bowie:

"There's a starman waiting in the sky
He'd like to come and meet us
But he thinks he'd blow our minds"

Dados os últimos acontecimentos mundiais, tenho que concordar com ele...

[ ]s

Alvaro Augusto



----- Original Message -----
From: marcelomjr
To: ciencialist@yahoogrupos.com.br
Sent: Tuesday, February 08, 2005 9:41 AM
Subject: [ciencialist] Extraterrestres: Cientistas Vêem Alta Probabilidade



http://www.burn.com.br/modules.php?name=News&file=article&sid=288

Visitantes Extraterrestres: Cientistas Vêem Alta Probabilidade
Postado em Domingo, 16 de janeiro de 2005 @ 15:44:08 BRST por josef

Por Leonard David
SPACE.com, 14 de janeiro de 2005

Há décadas atrás o físico Enrico Fermi ponderou sobre a questão de
civilizações extraterrestres com colegas teoristas em um almoço,
gerando a famosa tirada: "Onde eles estão?". Aquela pergunta depois
se tornou central a debates sobre a contagem do censo cosmológico de
outros povos estelares e possíveis visitantes extraterrestres (ET)
vindos de longe.

O pensamento de Fermi no tópico foi depois chamado de "paradoxo de
Fermi". É um conto bem circulado dos anos 50 quando o cientista
abordou o assunto em discussões com colegas em Los Alamos, Novo
México. Pensamentos relativos à probabilidade de planetas parecidos
com a Terra, o surgimento de civilizações altamente avançadas "lá
fora", e viagem interstelar -- estes permanecem a base para tentar
tentado responder ao paradoxo de Fermi mesmo hoje.

Agora um time de cientistas americanos nota que recentes descobertas
astrofísicas sugerem que nós deveríamos nos encontrar no meio de uma
ou mais civilizações extraterrestres. Além disso, eles argumentarm
que é um engano rejeitar todos os relatos de OVNI já que alguma
evidência para as visitas extraterrestres teoricamente-preditas
poderia ser encontradas justamente neles.

Os pesquisadores fazem sua proposta na edição de janeiro/fevereiro de
2005 do Journal of the British Interplanetary Society (JBIS).


Situação curiosa

Pegue qualquer revista boa sobre ciência e você vai ver as mais
recentes idéias de coçar a cabeça sobre a teoria de supercordas,
buracos de minhocas ou o alongamento do próprio espaço-tempo.
Enquanto isso, a detecção de planetas extrasolares está à beira de se
tornar comum.

"Nós estamos na situação curiosa hoje de que nossas melhores teorias
físicas e astrofísicas modernas predizem que nós deveríamos estar
experimentando visitação extraterrestre, contudo qualquer evidência
possível de tal escondida no fenômeno OVNI é ridicularizada dentro de
nossa comunidade científica", defende o astrofísico Bernard Haisch.

Haisch junto com os físicos James Deardorff, Bruce Maccabee e Harold
Puthoff apresentam seu caso no artigo do JBIS: "Inflation-Theory
Implications for Extraterrestrial Visitation".

Os cientistas apontam para duas descobertas chave feitas por
astrônomos australianos e relatadas no ano passado de que há
uma "zona galáctica habitável" em nossa Galáxia Via Láctea. E mais
importante, que a própria estrela da Terra, o Sol, é relativamente
jovem em comparação à estrela comum nesta zona -- por até um bilhão
de anos.

Então, os investigadores explicam em seu artigo no JBIS que uma
civilização alien comum estaria muito mais avançada e teria
descoberto a Terra há muito tempo. Adicionalmente, outro trabalho de
pesquisa na suposição que está por trás do Big Bang -- conhecida como
a teoria inflacionárias -- eleva o prospecto, eles sugerem, que nosso
mundo está imreso em uma civilização extraterrestre muito maior.


Distâncias de ponto-a-ponto

Assumindo física avançada por um bilhão de anos, viajar zunindo pela
galáxia não seria possível?

Até mesmo hoje a teoria de supercordas supõe outras dimensões... que
poderiam ser Universos habitáveis adjacentes ao nosso próprio, os
investigadores especulam. Poderia ser até mesmo possível contornar o
limite de velocidade da luz entrando e saindo destas dimensões.

"O que nós fizemos é um tanto como uma grande inovação", Contou
Haisch para SPACE.com. "Nós reunimos várias descobertas e assuntos
teóricos recentes que coletivamente apontam à probabilidade forte de
que nós devemos nos encontrar no meio de uma ou mais civilizações
extraterrestres gigantescas", disse ele.

Haisch disse que possibilidades de dimensões de supercordas e buracos
de minhoca e dilatação do espaço-tempo respondem à objeção "não se
pode ir daqui para lá" comumente levantada em vista das distâncias
interestelares de ponto-a-ponto envolvidas. Além disso, modelos de
difusão predizem que até mesmo uma única civilização poderia se
espalhar pela Galáxia em uma fração minúscula da idade da Galáxia -
até mesmo a velocidades inferiores à da luz, ele disse.


Assinatura ET nos dados

Pode a comunidade científica considerar qualquer evidência vinda de
avistamentos misteriosos de coisas estranhas pelo público?

Em grande medida, a comunidade científica tem aparentemente visto a
visitação ET como longe de ser algo a ser cogitado seriamente. Por
quê?

"O descarte tem várias causas, todas reforçando umas às outras",
Haisch respondeu. "A maioria das observações provavelmente são
enganos, ilusões e fraudes. Eu vi pessoas serem confundidas por Vênus
ou mesmo Sírio quando estão brilhando cores baixo no céu, com as
condições certas. Tendo sido desanimados por isto, a maioria dos
cientistas nunca se importaram em olhar além, e assim estão
simplesmente alegremente ignorantes de que pode haver mais nisto",
ele disse.

Deardorff, o principal autor do artigo do JBIS, aponta em uma
declaração à imprensa: "Seria necessário um pouco de humildade para a
comunidade científica suspender seu julgamento e tomar pelo menos
alguns dos relatos de alta qualidade a sério o suficiente para
investigá-los... mas eu espero que nós possamos fazer isso."

De acordo com Haisch, há uma motivação não apenas para a tolerância
científica do assunto OVNI, mas uma predição científica forte de que
deveria haver alguma assinatura ET genuína nos dados.

"Isto potencialmente muda a relação do fenômeno OVNI com a ciência de
um modo significante. Retira o preconceito 'não inventado aqui',
mostrando que um 'sim' para a visitação ET é exatamente o lado a que
nossas teorias físicas e astrofísicas atuais apontariam como a
situação mais provável", Haisch concluiu.

Fonte: Space.com
Tradução: Equipe BURN

Para saber sobre o Padaroxo de Fermi, visite:
http://www.str.com.br/ca/pfermi.htm





[As partes desta mensagem que não continham texto foram removidas]



SUBJECT: Re: [ciencialist] Re: Zodiaco
FROM: "Sergio M. M. Taborda" <sergiotaborda@terra.com.br>
TO: ciencialist@yahoogrupos.com.br
DATE: 08/02/2005 17:31

rmtakata wrote:

>
> --- Em ciencialist@yahoogrupos.com.br, "Sergio M. M. Taborda"
> > N\ao me fac,a rir com esses tipo de argumentos. Sejamos
> > serios.
>
> Estou sendo tao serio qto o tema: astromancia, permite ser.
>
> > Vc na~o disse ferocidade, pq ?
>
> Eu falei algo feroz. Deve conhecer o sentido da palavra 'algo'. Uma de
> suas acepcoes - como no caso usado - eh 'um pouco', 'de um certo
> modo'. Eu nao disse 'feroz' simplesmente. Usei a expressao com a
> devida modificacao.
>
> Nao obstante isso, alguns manuais falam sim de ferocidade - "Co'lera -
> O Lea~o, no auge de sua raiva, deixa-se vencer por exploso~es de
> temperamento inimagina'veis."
> http://www.kairell.donagh.nom.br/zodiaco.htm#leo
> (Certo, nao eh um manual, mas uma demonstracao de como isso estah sim
> presente em pelo menos alguns seguidores da astromancia.)

Vc está identificando colera, e outros adjectivos com Ferocidade.
Pronto, é lá consigo.
Mas esse adjectivo nunca é usado em astrologia para se referir ao signo
de leão.
É só isso que estou dizendo.

>
> > Orgulho, autoritarismo e megalomania nada te^m a ver com
> > ferocidade. Sa~o tudo coisas do corac,a~o.
>
> Ferocidade tb sao coisas do 'coracao' - isto eh, eh um aspecto
> emocional negativo exacerbado.

Eu não entendo assim. Ferocidade é uma qualidade irracional, animal,
caracteristica dos animais Selvagens como o leao, ou o tigre.
Mas nada tem a ver com o Leao, ou com sentimentos. É um acto reflexo e
não um acto pensado ou sentido.

> > Isso e' o que vc diz. Em que se baseia para afirmar isso.
>
> Eu me baseio no caso dos planetas descobertos tardiamente. Na epoca da
> edificacao da astromancia apenas sete 'planetas' (q. incluia a Lua e o
> Sol) eram conhecidos - Marte, Venus, Jupiter, Mercurio, Saturno (q.
> foram usados para batizar os dias da semana) - exceto a Terra. Como
> dito, qdo outros planetas foram descobertos, os nomes atribuidos aos
> novos planetas - Netuno, Urano e Plutao - influenciou as
> caracteristicas (ou atributos) q supostamente regeriam.

Bom, tlv vc me quiera mostrar qual a razão para a escolha desses nomes,
quem os escolheu, e baseados em quê.
Ou seja, até que ponto esses nomes são aleatórios.
Já lhe expliquei no outro email como os 7 planetas se destribuiam pelos
signos. Note que apenas 7 são necessários , e que mais do que esses
formariam uma nova "camada" de destribuição
Imagine uma roda com 12 casas e cada uma com os planetas da forma como
expliquei. Agora, adicione mais um, Onde esse +1 iria calhar ?
Seguindo o mesmo padrão de destribuição ele vai alhar exatamente no
mesmo local que Saturno (Aquario e Capricornio) Mas lembre-se que
saturno neste dois signos não se compatibiliza das mesma forma, já que
um so singos é + e o outro é -. A combinação signo, planetas, não é a
mesma. Urano e Saturno regem os mesmos signos, Aquario(+) e
Capricornio(-).
A introdução dos planetas transsaturnianos é obra da astrologia moderna,
desenvolvida desde 1960 +- e que embora , se baseando na astrologia
grega-arabe, é completamente nova , pois tem menos de 100 anos. Os indus
não usam plutão nos seus mapas sobre o argumento que plutão ainda não
deu a volta ao zodiaco e que portanto não podemos saber como ele
influencia as coisas.
Ora, a astrologia não precisa disso, ja que teoricamente a influencia é
sempre a mesma. Portanto, basta saber onde ele estava no passado e saber
o que aconteceu nesse tempo para reconhecer sua influencia.
Contudo, a astrologia Natal, não precisa dos planteas transaturnianos,
já que eles são mutto lentos e influenciam gerações inteiras e não
apenas pessoas singulares. Eles são, contudo, muito importantes em
astrologia mundial que so usa os planetas lentos , de marte para cima.
O que eu queria explicar é que seguindo o mesmo algoritmo que destribui
os 7 planetas pelos signos, podemos destribuir quaisquer uns que nos
aparecam. Seja qual for o nome do planeta a seguir a saturno ele teria
que ser identificado com Arquario e Capricornio. O outro a seguir, teria
que ser identificado com Sagitário e Peixes, e o outro com o Aries e
Escorpião. O proximo, será identificado com Libra e Touro e assim por
diante. Não é aleatorio.
A primeira camada , a dos 7, é chamada de 1º Harmonica, e a outro, de 2ª
Harmonica. Usando uma analogia das notas mosicais , tal como Pitagoras
fez (Ele escolher 7 notas pq havia 7 planetas em astrologia). Quantos
aos significados , são a intersecção dos significados dos signos que
regem e o planeta da primeira harmonica. Desta forma o planeta a seguir
a saturno, tem que revelar as caracteristicas de Saturno, Aquario e
Capicornio, mas num nivel superior de abstracção. Saturno, o deus do
tempo e da mudança (aquario) , mas tb do deslumbramento (capircornio) dá
lugar o Urano , tb deus da mudança, mas de ua mudança não natural como a
se saturno. Uma mudança pela revolução e não pela simples evolução.
O seguinte, tem que caber as caractetisticas de Jupiter, socialidade e
amizade num nivel superior dão origem à religião (peixes) e ao
desprendimento do material (sagitário). Coisas que Jupter já faz, mas de
uma forma menos introspectiva. Plutão, relança as energias de Marte para
a acção e a força, mas não mais no plano material de marte, mas no plano
metafisico. As energia de marte usadas para a força , para a acção, são
agora usadas para a generação. A força activa de Marte de fazer coisas,
é agora a força de plutão de refazer coisas.
Os nomes podem até ser completamente aleatórios - o seu estudo dirá-,
mas os seus significados não o são, pois eles são derivados dos
significados que já tinhamos antes.

>
> > That's the point! O ponto e' esse mesmo. Elas na~o o sa~o, mas
> > assim mesmo a indu atriui simbolos diferentes aos mesmos
> > sigificados. O significa que o significando vem
> > antes, e o simbolo e' apenas uma menmonica.
>
> Not so. Houve uma mistura, mais ou menos como a influencia da
> mitologia grega sobre a romana -

Mas quem estava falando sobre isso ? Estavamos falando de Gregos e
Indus. Não mude de assunto.
Os romanos simplesmente fizeram copy paste e alteraram o que lhes
aproveu por motivos politicos, como era seu costume.

>
> > Obrigado. E' exactamente isso. Enta~o como vc esplica que os
> > simbolos sejam outros ?
>
> Outros, mas nao sao fundamentalmente diferentes.

Embora, isso, outros. Que é o que interessa. Pois o seu argumento é
fundamentalmente que os simbolos derão origem ao significados.
Ou seja, que o simbolo de um leão deu origem o significado de majestade
etc... de um leão, e o de balança ou de equilibrio, etc...
Ora, se isso fosse verdade, simbolos diferentes deveriam dar origem a
significados diferentes. O que não acontece.
Os significados permanecem, e são os simbolos que mudam, conforme a
cultura de quem intrepreta os simbolos.

> Um exemplo de
> astromancia mais diferente, com outros simbolos e outros significados
> eh a astromancia chinesa.

Mas essa é baseada em regras completamente diferentes. Para começar tem
5 elementos, enquanto a astrologia ocidental tem 4.
Pode mostrar-se como o significado dos 4 elementos dão origem ao 12
signos. Entenda que o numero 4 foi introduzido pelos gregos.
A Elementariedade (signos de ar, fogo, terra, agua) foi intruduzida por
eles. Se vc for usar a mesma logica com os 5 elementos da astrologia
chinesa, obviamente o resultado não é o mesmo.
Os sistemas chines e indu são diferentes. Apenas o conceito de olhar o
ceu para entender a terra é o mesmo. O como isso se faz é diferente. E
uma não anula a outra.

>
> > Se a sua teoria fosse verdaeira , os simbolso seriam os
> > mesmos pois as estrelas sa~o as mesmas.
>
> Isso apenas se nao houvesse intercambio significativo entre os
> sistemas astromanticos. Acima dei o exemplo da transformacao de Marte
> em direcao a Ares.

Mas isso se deu por motivos politcos e não astrologicos. Aries, o signo,
sempre compreendeu a agricultura e a guerra, desde dos sumerios.
Não seriam os gregos ou os romanos a mudar isso. Chamem-lhe Ares , ou
Marte, a conversa é a mesma.
Os nomes, são apenas necessários para resumir os conceitos. E não são
tão importantes quanto isso, pois são culturais. Mas o conceito, esse
sim é sempre o mesmo inter-culturas, e isso é que interessa. Em cada
lingua da terra, enegia diz-se de uma forma diferente, mas o conceito
fisico por detraz dessa palavra é sempre o mesmo para todos.

>
>
> Simbolos nao sao estaticos. (Para outro exemplo, veja a origem dos
> algarismo indoarabicos.)

Exacto, mas significados são. E é isso que, ultimamente importa para a
astrologia.
Hoje em dia vc pode fazer astrologia sem simbolos, porque ha outras
formas de recordar os significados, mas antigamente não havia e os
simbolos eram essenciais para decorar os significados e passá-los as
gerações seguintes. .

>
> > Tudo bem. Enta~o pq os outros planetas na~o sa~o referidos como
> > o planeta <cor>?
>
> Porq. nao hah necessidade de sistematizacao.

Então se não ha, a ligação de marte com o vermelho é mera excepção à
regra., já que os outros não são ligados com as cores dos planetas (com
a cor da luz do sol refletida neles)
.Sendo que é uma excepção, qq contrariação dela não contraria a regra.

>
> > Voce^ parece estar muito convencido disso, ignorando toda
> > amatema'tica envolvida na astrologia, a
>
> A matematica util envolvida na astromancia resume-se 'a previsao da
> posicao dos planetas e outros corpos. Essa parte util encontra-se
> incorporada na astronomia.

Não. Passa pela definição do ponto Vernal e dos signos.

>
> Boa parte dela, porem, foi desenvolvida depois - algumas motivada por
> questoes astromanticas.

Não. O ponto Vernal e os signos fazem parte da astrologia desde o
principio.

>
> > E' como vc negar a mecanica quantica pq na~o gosta do nome
> > quantum.
>
> Ueh, quem disse q. eu nao gosto dos nomes? Estou tao somente dizendo
> q. os atributos conferidos aos segmentos astromanticos celestes foram
> influenciados fortemente pelos nomes dados a esses setores.

E eu estou dizendo exactamente o contrário. Apenos que haja boas provas
de algum dos lados, não sairemos daqui.
Mas eu tentei mostrar que é mais natural, usando o que sabemos de
historia, que os significados permaneçam e os simbolos mudem.
Um exemplo classico é o da imagem romana de uma senhora segurando um
bebe, que representava se não me engano Afrodite e seu filho, mas que
passou a representar Maria e Jesus, quando o imperador mandou que todo o
imperio se convertesse ao cristianismo. É a mesma imagem, o mesmo
significado, uma mae e seu filho. Mas que antes era simbolo de afrodite,
e agora de maria.
Não so o simbolo mudou , como o significado intrometeu-se nos
significados religiosos do cristianismo tornando a personagem de Maria,
mais importante que a de jesus. Ao ponto de vc ter o culto a Maria.
Mas o significado do simbolo, que é a representação do laço entre mae e
filho, o nascimento , a fertilidade , e no fundo o papel da mulher num
mundo governado por deuses homems, é a mesma. E é isso que interessa.

> E os nomes
> dados a esses setores foram determinados pelas formas com q. estrelas
> foram imaginativamente ligadas entre si.

Os nomes das constelações sim, os nomes dos signos pelos nomes das
cosntelações, mas não os significados dos signos.
A ordem é Significado -> representação nas estrelas -> nome. Exemplo:
lealdade , magnificencia, e governo -> aniaml leão -> cosntelação de leão.
Vc está covencido que é : Representação nas estrelas, -> nome ->
significado.
Mas minha pergunta é : Pq eles escolheram uma representação em
particular, quando - como ja concordámos- ha tantas possiveis ?
Concerteza tem que haver um motivo para isso. Que motivo vc apresenta ?
Simples acaso ? Estética ?

Sérgio Taborda


--
No virus found in this outgoing message.
Checked by AVG Anti-Virus.
Version: 7.0.300 / Virus Database: 265.8.6 - Release Date: 07-02-2005



SUBJECT: Re: [ciencialist] Re: Zodiaco
FROM: "Sergio M. M. Taborda" <sergiotaborda@terra.com.br>
TO: ciencialist@yahoogrupos.com.br
DATE: 08/02/2005 18:08

Oraculo wrote:

> Olá Taborda
>
> "Taborda: Não entendi o que vc quiz dizer com isto"
>
> Quiz dizer que respostas vagas e de duplo sentido, como "qualidades as
> vezes são negativas e as vezes positivas" se ajustam a qualquer coisa
> e alegação, não sendo base para um argumento ou demonstraçao de evidencia.

Mas eu não proferi a frase que vc coloca entre aspas, por isso não
entendo de onde vem essa conversa e o que ela tem a haver com o que eu
disse.

> Taborda: Complexos ? 365 / 29 é complexo ?"
>
>
>
> O que significa que efetuar contas e calculos e eles estarem corretos,
> nada diz sobre a interpretação dos mesmos ou sobre de onde vem a
> informação sobre minha personalidade.

Diz sim. Mas como vc já esqueceu tudo o que aprendeu na escola vc não
enxerga o pq.
Vc deveria saber que os povos antigos contavam o tempo pelo sol e pela
lua. Normamente nas aulas de historias eles dizem que por um ou por
outro, mas se vc verificar, vc vai ver que sempre é pelos dois. Não me
lembro de nenhum povo da antiguidade que não medisse o tempo pelo sol e
a lua. A forma mais natural de criar intervalos de tempo seria então
pela coincidencia do sol e da lua, ou seja, pelas suas conjunções. E foi
assim que eles seraram o ceu em 12 partes e o ano em 12 meses. Usando o
sol como um marcador e a lua como o divisor.
Eu acho esta explicação o sufcinetente legitima, astonomicamente,
fisicamente , matemáticamente e historicamente para a aceitar. Se vcs
acham que a razão se deve às 12 constelações, a pergunta é então, pq não
13 ? 20 ? 50 ? ja que o agrupamento de estrelas é meramente
convencionado. Como estava escrito no texto que o Takata enviou, a
cosntelação de balança eram aas pinças da de escorpião. Dizendo que as
constelações podem ser arranajadas como nos aprover e baeados em regras
precedentes.
Mas , claro, como sempre, se vc aportar evidencias suficientes que
explicem, como logicamente, a relaçao é baseada no numero de
cosntelações e não no numero de conjunções sol-lua, eu posso conciderar
minha posição. Mas até agora vc so apresnetou argumento sem sentido.

>
> Homero: Se toda qualidade é ao mesmo tempo boa e má, se toda pessoa
> pode ser
> uma hora uma coisa e outra outra coisa,
>
> Taborda: Viu ? Isto é tipico da argumentação não cientifica. Pegam num
> argumento
> , extrapolam-o além do seu significado, e reaplicam-o como se fosse
> válido. Vc está pensando da mesma forma que pensa um crente num deus qq.
> Auto-convencendo-se de que os argumentos que ele cria, são os desejos
> desse deus
> É verdade q todos os atributos são bons e maus. Mas ninguem disse que a
> pessoa é uma hora uma coisa e outra ora outra. Isso vc fabricou na sua
> mente como um sinonimo , mas que não é.
> Ai vc chega num resultado logico que vc acha que contraria o argumento
> inciial, mas não o faz pq uma das permissas é inválida.
>
> Não estou extrapolando nada

Está. Tlv não se dê conta, mas está sim. Vc está afirmando que a
astrologia afirma coisa, que ela não afirma. Isso é extrapolação.

> Mas, insisto, se um atributo é aplicavel a seu oposto tanto quanto a
> seu sentido original, ele vai servir para qualquer pessoa.

O problema é que eu não afirmei isso que vc está dizendo. E vc continua
falando disso como se fosse disso que eu estou a falar. Não. Vc está
viciado nesse argumento e nem sequer dá ao trabalho de ler com atenção o
que eu escrevi.
O que eu disse é que todo e qualquer atributo é dual. Isso é muito
simples. Toda a gente entende isso. Por exemplo, inteligencia pode ser
algo muito bom, pois faz evoluir a pessoa em qq meio ambiente,mas pode
ser mau, como nos mostra o maquiavelismo. Preseverança é bom até que se
transforma em teimosia. Iniciativa é bom até que vc começa muitas coisas
e não acabe nenhuma.
Ponderanção é bom, até que se torna duvida. E assim vai. Um atributo tem
uma serie de graus e a partir de certo grau ele se torna
contraproducente. O que vc vê de errado nesta afirmação ?

>
> Homero: então, para que sereve a astrologia???
>
> Taborda: Não estamos discutindo para que serve, e sim, o que é. Em que
> bases
> teoricas se apoia, q filosofia segue e o que tem a ver com astronomia.
>
> Estamos sim, porque se admitir que não serve para nada, finda a discusão.

Mas como eu posso admitir isso se eu não tenho provas ? Tb não posso
adimitr o contrario pela mesma razão.
Até que existam provas em algum sentido, para mim permanece a duvida e
não vou adimitr coisa nenhuma.
Se é isso que quer, pode tirar o cavalinho da chuva.

>
>
> Taborda: Vc pode especular a razão à sua vontade. A questão aqui não é
> quem se
> vai consultar com o astrologo, ou o astrologo, mas a astrologia. A
> astrologia é uma teoria, como outra qualquer.
>
> De forma alguma! Teorias fazem previsões e determinam efeitos.

A meteorologia faz previsões. Alguem fica perguntado pq eu quero saber o
tempo de amanhã , ou se o meteorologo é um fajuto enganador ?
Não. Então pq o fazem com os astrologos e quem os consulta?
A meteorologia é tão eficaz como a adivinhação , mas existem pessoas
dedicando-se a isso, criando um monte de artefactos em roda isso e
ganhando dinheiro.
Vc não se indigna com isso, pq se indigna com a astrologia que vc nem
sabe do que trata ? O seu esforço é contraproducente. Pq vc não se
procupa com os medicos fajutos que violam crianças, esquecem
instrumentos dentro dos pacientes, ou cobram os olhos da cara por uma
consulta de 5 minutos ? Ou com os jornalistas que so contam mentiras e
trabalham para os politicos para o controle do pais ?
Pq vc não se procupa com eleições idiotas e CARAS - usando o seu
dinheiro - para a presidencia de assembleias que nada fazem ? Pq vc não
se procupa com professores fajutos, mal preparados e que desonram todo o
sistema educativo com os seus comportamentos imbecis que nada aportam à
educação dos jovens ? Pq vc não se preocupa com coisas mais importantes
e que consomem o seu dinheiro a rodos, em vez de se preocupar com coisas
que nem sequer se dá ao trabalho de compreender e que nem sequer gastam
o seu dinheiro?
Já que vc é tão critico, pq não critica coisas mais uteis do que a
astrologia, sendo que vc nem acha que ela funciona ? Critique coisas que
vc acha que fucnionem e andam a funcionar mal!

E eu já pedi para não fazer top-posting. (Isso é outra coisa que vc
deveria ser mais critico, pq está violando as regras da boa conduta na
internet.Ou vc tb acha que isso não funciona?)

Sérgio Taborda


--
No virus found in this outgoing message.
Checked by AVG Anti-Virus.
Version: 7.0.300 / Virus Database: 265.8.6 - Release Date: 07-02-2005



SUBJECT: Re: [ciencialist] Re: Zodiaco
FROM: "Sergio M. M. Taborda" <sergiotaborda@terra.com.br>
TO: ciencialist@yahoogrupos.com.br
DATE: 08/02/2005 18:21

Oraculo wrote:

>
> Então, no que interessa, eu sei do que estou falando, mesmo que não
> conheça os calculos que você usa na construção de um mapa astral (que
> diferem dos usados por outros astrólogos, mas eles, claro, são uns
> idiotas e estão evidentemente enganados..:-)

Mas o pronto é exactamente esse, vc não sabe do que está falando. Se
sabe, vc não o demonstrou em mensagem nenhuma que eu tenha lido.

>
> Para terminar, não saber nada sobre leitura fria não impede de usar a
> técnica de leitura fria, que é derivada da natural habilidade humana
> de "ler" sinais, informação, reações e qualquer forma de comunicação
> em diversos níveis em outros seres humanos. Essa é uma habilidade que
> pode ser refinada e incrementada mesmo que seu possuidor nada saiba
> sobre ela. É o que chamariamos de uma pessoa " intuitiva", com grande
> habilidade de se comunicar com outros, lendo informação em niveis
> quase subconscientes.
>
> Talvez você seja uma pessoa muito habilidosa nesse sentido, apenas
> isso. E, não seria interessante, para dar maior base a sua refutação
> de que não é leitura fria que ocorre no mapa astral, aprender mais
> sobre essa habilidade, sobre outras capacidades da mente humana?

A leitura fria baseia-se na ideia de que ha pelo menos duas pessoas
conversando. Em astrologia não ha duas pessoas conversando. Por isso ela
não pode ser usada.
Eu posso fazer um mapa de qq pessoas que eu saiba os dados de nascimento
sem nunca a ter visto. Posso inferir o que vai acontecer na vida desta
pessoas. Posso inferir como esta pessoa se relacionaria com outra de
quem eu sei os dados de nascimento. Tudo isto, sem conhecer as pessoas
em momento algum. As pessoas podem já nem existir, ou ainda não existir.
No dia que uma prima minha nasceu eu fiz o seu mapa. E inferir dele como
seria a sua relação basica com os pais e o mundo. Como eu posso fazer
leitura fria com um bebe que nem fala nem se exprime de forma nenhuma ?
Se a intrepretação se adqua aos facto eu não posso culpar a leitura
fria. Culpe-se outra coisa, mas não a leitura fria.
O mesmo argumento pode ser usado pelo Tarot , as runa, etc... por quase
todas as formas de adivinhação, que realmente se podem chamar assim,
pois não carecem da presença de mais ninguem que o usuaário da tecnica.
Embora isso seja muito mais dificil para o tarot e as runas por exemplo,
é muito simples para a astrologia.
Aquilo que os ilusionistas fazem em espetaculos é leitura fria, sim e
dai ? O que isso tem a ver com astrologia ?
Vc vai dizer que a maioria dos astrologos fazem leitura fria. Mas como
se mede a leitura fria, como eu sei que ele esta fazendo leitura fria ?
e pior , como eu provo ? Se vc sabe responder a isso, pq vc não prova
que todos os astrologos fazem leitura fria e acaba com o problema?
Faça-se uma licença para astrologos que a recebe quem poder passar no
teste de leitura fria - seja lá o que isso for , supondo que existe tal
teste e ele é 100% infalivel.

Sérgio Taborda


--
No virus found in this outgoing message.
Checked by AVG Anti-Virus.
Version: 7.0.300 / Virus Database: 265.8.6 - Release Date: 07-02-2005



SUBJECT: Re: [ciencialist] p/Victor - Zodíaco
FROM: "murilo filo" <avalanchedrive@hotmail.com>
TO: ciencialist@yahoogrupos.com.br
DATE: 08/02/2005 18:27


Victor, hello.
Caramba, vc mandou um ensaio bem pessoal, gostei!
Seu carnaval não deve ter sido muito diferente do meu! Oisquindô!!
Concordo totalmente: ''A coisa está mais perto, dentro de nós
mesmos.'' É isto mesmo e cada um por sí, assim como a humanidade, também por
sí.
Cada um é o responsável a partir do pequeno universo que carrega por dentro,
e posso até assegurar de que não existe a tutela do Pai. Nascemos já
bastante aparelhados, fortes e irresponsáveis. As verdades humanas são
passageiras, mas ainda assim temos de busca-las, de preferência de dentro
para fora, mais pela experiência, ou conhecimento, e menos pelo ''saber''
das fórmulas feitas, sejam científicas ou não.
Como nós, nada no universo está, ou estará, acabado... tudo é provisório por
natureza, porque a evolução tem que seguir...
BTW, a ''astrologia e deuses'' não têm nicas que ver com misticismo.
obr/abr M. 08/fev/2005

>From: JVictor <jvoneto@uol.com.br>
>Reply-To: ciencialist@yahoogrupos.com.br
>To: "ciencialist@yahoogrupos.com.br" <ciencialist@yahoogrupos.com.br>
>Subject: Re: [ciencialist] p/Victor - Zodíaco
>Date: Tue, 08 Feb 2005 00:06:58 -0200

>Murilo,
>
>
>murilo filo escreveu:
>
> > >Experiência não é crença.
>
> Victor: Evidentemente você refere-se a "experiência" pessoal, que é
>vivenciada única e exclusivamente por aquela pessoa e por mais ninguém.
>Isso não é experiência, no sentido em que se emprega essa palavra no
>jargão científico. As sensações e/ou visões são pessoais, com
>interpretações pessoais a nível de sua própria mente. Outras pessoas
>podem "sentir" interpretações diferentes, a depender de seu quadro de
>referências, daquilo em que acredita! São coisas interiores, não
>reproduzíveis na hora em que se deseja. ...




SUBJECT: Re: [ciencialist] Re: Zodiaco
FROM: "Sergio M. M. Taborda" <sergiotaborda@terra.com.br>
TO: ciencialist@yahoogrupos.com.br
DATE: 08/02/2005 18:37

Oraculo wrote:

> Olá Taborda
>
> Bem, também fico com minha paciência em baixa, mas parece que tenho
> mais que você..:-) O que não chega a ser uma virtude, não?
>
> > Hoje os setores sào o que importa, já que fica dificil manter a base
> > original da astrologia, mas não foi isso que baseou sua criação.
>
> Prove isso logo de uma vez e terminemos com esta conversa da treta, ou
> cale-se para sempre. Ler erros cientificos uns atrás dos outros dá cabo
> da minha paciencia.
>
>
> Exato..:-) Não posso provar. Exatamente como você, não posso provar
> nada disso,

Mas eu posso. E já o fiz mais do que uma vez. A prova é muito simples. A
criação dos sectores baseia-se apenas na posição das conjunções do sol e
da lua.
Se vc não aceita isso como prova é outra historia, mas que é a prova, é.

>
>
> Taborda: O estudo astrologico dada de à mais ou menos 5000 anos.
>
> Bem, que dados temos sobre esse periodo de "mais ou menos 5000 anos",
> que permitem tanta certeza a você?

Dados historicos. Sumerios, Assirios , Indus , Gregos, Arabes dão uma
margem desde à 5000 até os dias de hoje, já que o imperio arabe apenas
caiu por volta de 1400 DC.
Todos eles usavam a mesma astrologia. Incrementada com tecnicas melhores
ao longo dos anos.
Acho que isso é bastante para poder dizer que a astrologia tem 5000 anos
de existencia , mais ou menos. E que, ao longo desse tempo , as estrelas
no ceu foram basicamente sempre as mesmas, pelo que não cabe o argumento
de Ofucio apareceu de repente no ceu. Se não apareceu de repente, já
estava lá quando os primeiros povos criaram o sistema astrologico.
Duvida disto ?
Então, sendo que é seu argumento - e de outro - de que eles o criaram
baseado no numero de constelações no ceu, demosntre pq eles escolheram
um sistema de 12 sectores em vez de 13. Ou pq não agruparam as estrelas
de outras formas para poder formar um outro qq numero artibtrario de
sectores. Basicamente, seu trabalho é demosntrar pq o numero de
sectores não é igual ao numero de constelações SE - conforme vc , e
outros afrirmam - deve haver um sector por constelação na faixa do zodiaco.

>
>
> E, de novo, o que tem calcular órbitas, posições, precisão matematica,
> com conhecer a personalidade de uma pessoa??? Essa pergunta nunca é
> respondida!!

Então eu vou-lhe responder. O que tem a haver é muito simples: astrologia.
Agora va estudar um pouco de astrologia para saber como tem a ver e porque.

> Não importa como alega ser um leonino, como a posição de um planeta
> define essa personalidade????

Da mesma forma que o meu pc pesa 3,5kg. Por convensão.
Satisfeito ?

Sérgio Taborda


--
No virus found in this outgoing message.
Checked by AVG Anti-Virus.
Version: 7.0.300 / Virus Database: 265.8.6 - Release Date: 07-02-2005



SUBJECT: Re: [ciencialist] Fw: Duvidas sobre tensores
FROM: "Sergio M. M. Taborda" <sergiotaborda@terra.com.br>
TO: ciencialist@yahoogrupos.com.br
DATE: 08/02/2005 18:50

Luiz Ferraz Netto wrote:

> Aceita-se sugestão.
> []'
> ===========================
> Luiz Ferraz Netto [Léo]
> leobarretos@uol.com.br
> http://www.feiradeciencias.com.br
> ===========================
> -----Mensagem Original-----
> De: "Cláudia Adam Ramos" <claudinhadam@yahoo.com.br>
> Para: <leobarretos@uol.com.br>
> Enviada em: domingo, 6 de fevereiro de 2005 21:32
> Assunto: Duvidas sobre tensores
>
>
> Estive visitando o seu site, Feira de Ciência, e
> imagino qu você possa tirar uma duvida minha...sou
> acadêmica do curso de matemática e gostaria de
> entender o sentido físico do divergente de um campo
> tensorial. Não consigo imaginar fisicamente como
> funciona o div de uma matriz de tensoes. Se você puder
> tirar esta duvida minha ficarei grata,

A divergencia não é um campo, é um numero. O campo, representa alguma
coisa. Os vários diferencais de um campo, div, rot, grad , representa
como esse campo varia no espaço.
A divergencia de um campo num ponto é a o limite do fluxo de campo,
quando a superficie que rodeia o ponto em questão tende para zero.
A divergencia "mede" então o fluxo do campo em cada ponto do espaço.

Sergio Taborda


--
No virus found in this outgoing message.
Checked by AVG Anti-Virus.
Version: 7.0.300 / Virus Database: 265.8.6 - Release Date: 07-02-2005



SUBJECT: Re: [ciencialist] Fw: Duvidas sobre tensores
FROM: "Luiz Ferraz Netto" <leobarretos@uol.com.br>
TO: <ciencialist@yahoogrupos.com.br>
DATE: 08/02/2005 21:36

Enviado por Léo:
> Estive visitando o seu site, Feira de Ciência, e
> imagino qu você possa tirar uma duvida minha...sou
> acadêmica do curso de matemática e gostaria de
> entender o sentido físico do divergente de um campo
> tensorial. Não consigo imaginar fisicamente como
> funciona o div de uma matriz de tensoes. Se você puder
> tirar esta duvida minha ficarei grata,

Sérgio escreve:

A divergencia não é um campo, é um numero. O campo, representa alguma
coisa. Os vários diferencais de um campo, div, rot, grad , representa
como esse campo varia no espaço.
A divergencia de um campo num ponto é a o limite do fluxo de campo,
quando a superficie que rodeia o ponto em questão tende para zero.
A divergencia "mede" então o fluxo do campo em cada ponto do espaço.

Léo:
Oi Sérgio, agradeço a sugestão.

Fiz (como resposta para a moça) uma ligeira introdução sobre a dificuldade de se interpretar certas estruturas usadas na matemática, quando aplica à física.
A seguir usei todo seu texto, com ligeira alteração na frase; questão de rigor:

sua : "A divergência de um campo num ponto é a o limite do fluxo de campo,
quando a superfície que rodeia o ponto em questão tende para zero."

minha: A divergência de um campo num ponto é a o limite do fluxo de campo,
quando a área da superfície que rodeia o ponto em questão tende para zero.

aquele abraço,
++++++++++++++++++++++++


--
No virus found in this outgoing message.
Checked by AVG Anti-Virus.
Version: 7.0.300 / Virus Database: 265.8.6 - Release Date: 07/02/2005



SUBJECT: Re: [ciencialist] Fw: Duvidas sobre tensores
FROM: "Alvaro Augusto \(E\)" <alvaro@electraenergy.com.br>
TO: <ciencialist@yahoogrupos.com.br>
DATE: 08/02/2005 22:24

----- Original Message -----
From: Sergio M. M. Taborda
To: ciencialist@yahoogrupos.com.br
Sent: Tuesday, February 08, 2005 6:50 PM
Subject: Re: [ciencialist] Fw: Duvidas sobre tensores


Luiz Ferraz Netto wrote:

>> Aceita-se sugestão.
>> []'
>> ===========================
>> Luiz Ferraz Netto [Léo]
>> leobarretos@uol.com.br
>> http://www.feiradeciencias.com.br
>> ===========================
>> -----Mensagem Original-----
>> De: "Cláudia Adam Ramos" <claudinhadam@yahoo.com.br>
>> Para: <leobarretos@uol.com.br>
>> Enviada em: domingo, 6 de fevereiro de 2005 21:32
>> Assunto: Duvidas sobre tensores
>>
>>
>> Estive visitando o seu site, Feira de Ciência, e
>> imagino qu você possa tirar uma duvida minha...sou
>> acadêmica do curso de matemática e gostaria de
>> entender o sentido físico do divergente de um campo
>> tensorial. Não consigo imaginar fisicamente como
>> funciona o div de uma matriz de tensoes. Se você puder
>> tirar esta duvida minha ficarei grata,

>A divergencia não é um campo, é um numero. O campo, representa alguma
>coisa. Os vários diferencais de um campo, div, rot, grad , representa
>como esse campo varia no espaço.
>A divergencia de um campo num ponto é a o limite do fluxo de campo,
>quando a superficie que rodeia o ponto em questão tende para zero.
>A divergencia "mede" então o fluxo do campo em cada ponto do espaço.

>Sergio Taborda

Na verdade, a divergência é um campo, mas um campo escalar.

[ ]s

Alvaro Augusto



SUBJECT: Astrologia e Metereologia e top-posting (era : Zodiaco)
FROM: "Oraculo" <oraculo@atibaia.com.br>
TO: <ciencialist@yahoogrupos.com.br>
DATE: 08/02/2005 22:25

Olá Taborda

Taborda: E eu já pedi para não fazer top-posting. (Isso é outra coisa que vc
deveria ser mais critico, pq está violando as regras da boa conduta na
internet.Ou vc tb acha que isso não funciona?)"

Sinto, mas não faço top-posting, nunca. Se algum trecho pareceu assim me desculpe, mas foi apenas impressão. Todos os meus comentários se seguem ao trecho relacionado. Sendo que tomo o cuidado de sempre apresenta-lo depois do trecho em questão, exatamente como fiz acima.

E eu me preocupo e critico médicos que violam crianças (ou não médicos, tanto faz) e com diversas das proposições que fez (embora discorde de algumas, principalmente sobre eleições inúteis..:-), mas isso não me impede de também criticar a astrologia.

E a metereologia, como já discutido, é bem diferente da astrologia, já que tem a cada vez mais se aperfeiçoado e atingido um bom grau de exatidão, embora para periodos curtos de tempo (até 8 dias). Mas é cada vez melhor em prever tempestades, furacões, etc, colocando pessoas a salvo em caso de perigo (o sistema de alerta de tornados americanos é um bom exemplo). E a compreensão dos mecanismos envolvidos é cada dia maior.

E, o principal, a metereologia sempre soube de suas deficiências e de suas limitações, só fazendo previsões com cuidado e sempre sob resalves, esperando mais dados e maior conehcimento a cada avanço cientifico (como supercomputadores que permitem mais calculos e mais precisas previsões).

Comparada a astrologia, é uma enorme diferença.

Homero


----- Original Message -----
From: Sergio M. M. Taborda
To: ciencialist@yahoogrupos.com.br
Sent: Tuesday, February 08, 2005 6:08 PM
Subject: Re: [ciencialist] Re: Zodiaco


Oraculo wrote:

> Olá Taborda
>
> "Taborda: Não entendi o que vc quiz dizer com isto"
>
> Quiz dizer que respostas vagas e de duplo sentido, como "qualidades as
> vezes são negativas e as vezes positivas" se ajustam a qualquer coisa
> e alegação, não sendo base para um argumento ou demonstraçao de evidencia.

Mas eu não proferi a frase que vc coloca entre aspas, por isso não
entendo de onde vem essa conversa e o que ela tem a haver com o que eu
disse.

> Taborda: Complexos ? 365 / 29 é complexo ?"
>
>
>
> O que significa que efetuar contas e calculos e eles estarem corretos,
> nada diz sobre a interpretação dos mesmos ou sobre de onde vem a
> informação sobre minha personalidade.

Diz sim. Mas como vc já esqueceu tudo o que aprendeu na escola vc não
enxerga o pq.
Vc deveria saber que os povos antigos contavam o tempo pelo sol e pela
lua. Normamente nas aulas de historias eles dizem que por um ou por
outro, mas se vc verificar, vc vai ver que sempre é pelos dois. Não me
lembro de nenhum povo da antiguidade que não medisse o tempo pelo sol e
a lua. A forma mais natural de criar intervalos de tempo seria então
pela coincidencia do sol e da lua, ou seja, pelas suas conjunções. E foi
assim que eles seraram o ceu em 12 partes e o ano em 12 meses. Usando o
sol como um marcador e a lua como o divisor.
Eu acho esta explicação o sufcinetente legitima, astonomicamente,
fisicamente , matemáticamente e historicamente para a aceitar. Se vcs
acham que a razão se deve às 12 constelações, a pergunta é então, pq não
13 ? 20 ? 50 ? ja que o agrupamento de estrelas é meramente
convencionado. Como estava escrito no texto que o Takata enviou, a
cosntelação de balança eram aas pinças da de escorpião. Dizendo que as
constelações podem ser arranajadas como nos aprover e baeados em regras
precedentes.
Mas , claro, como sempre, se vc aportar evidencias suficientes que
explicem, como logicamente, a relaçao é baseada no numero de
cosntelações e não no numero de conjunções sol-lua, eu posso conciderar
minha posição. Mas até agora vc so apresnetou argumento sem sentido.

>
> Homero: Se toda qualidade é ao mesmo tempo boa e má, se toda pessoa
> pode ser
> uma hora uma coisa e outra outra coisa,
>
> Taborda: Viu ? Isto é tipico da argumentação não cientifica. Pegam num
> argumento
> , extrapolam-o além do seu significado, e reaplicam-o como se fosse
> válido. Vc está pensando da mesma forma que pensa um crente num deus qq.
> Auto-convencendo-se de que os argumentos que ele cria, são os desejos
> desse deus
> É verdade q todos os atributos são bons e maus. Mas ninguem disse que a
> pessoa é uma hora uma coisa e outra ora outra. Isso vc fabricou na sua
> mente como um sinonimo , mas que não é.
> Ai vc chega num resultado logico que vc acha que contraria o argumento
> inciial, mas não o faz pq uma das permissas é inválida.
>
> Não estou extrapolando nada

Está. Tlv não se dê conta, mas está sim. Vc está afirmando que a
astrologia afirma coisa, que ela não afirma. Isso é extrapolação.

> Mas, insisto, se um atributo é aplicavel a seu oposto tanto quanto a
> seu sentido original, ele vai servir para qualquer pessoa.

O problema é que eu não afirmei isso que vc está dizendo. E vc continua
falando disso como se fosse disso que eu estou a falar. Não. Vc está
viciado nesse argumento e nem sequer dá ao trabalho de ler com atenção o
que eu escrevi.
O que eu disse é que todo e qualquer atributo é dual. Isso é muito
simples. Toda a gente entende isso. Por exemplo, inteligencia pode ser
algo muito bom, pois faz evoluir a pessoa em qq meio ambiente,mas pode
ser mau, como nos mostra o maquiavelismo. Preseverança é bom até que se
transforma em teimosia. Iniciativa é bom até que vc começa muitas coisas
e não acabe nenhuma.
Ponderanção é bom, até que se torna duvida. E assim vai. Um atributo tem
uma serie de graus e a partir de certo grau ele se torna
contraproducente. O que vc vê de errado nesta afirmação ?

>
> Homero: então, para que sereve a astrologia???
>
> Taborda: Não estamos discutindo para que serve, e sim, o que é. Em que
> bases
> teoricas se apoia, q filosofia segue e o que tem a ver com astronomia.
>
> Estamos sim, porque se admitir que não serve para nada, finda a discusão.

Mas como eu posso admitir isso se eu não tenho provas ? Tb não posso
adimitr o contrario pela mesma razão.
Até que existam provas em algum sentido, para mim permanece a duvida e
não vou adimitr coisa nenhuma.
Se é isso que quer, pode tirar o cavalinho da chuva.

>
>
> Taborda: Vc pode especular a razão à sua vontade. A questão aqui não é
> quem se
> vai consultar com o astrologo, ou o astrologo, mas a astrologia. A
> astrologia é uma teoria, como outra qualquer.
>
> De forma alguma! Teorias fazem previsões e determinam efeitos.

A meteorologia faz previsões. Alguem fica perguntado pq eu quero saber o
tempo de amanhã , ou se o meteorologo é um fajuto enganador ?
Não. Então pq o fazem com os astrologos e quem os consulta?
A meteorologia é tão eficaz como a adivinhação , mas existem pessoas
dedicando-se a isso, criando um monte de artefactos em roda isso e
ganhando dinheiro.
Vc não se indigna com isso, pq se indigna com a astrologia que vc nem
sabe do que trata ? O seu esforço é contraproducente. Pq vc não se
procupa com os medicos fajutos que violam crianças, esquecem
instrumentos dentro dos pacientes, ou cobram os olhos da cara por uma
consulta de 5 minutos ? Ou com os jornalistas que so contam mentiras e
trabalham para os politicos para o controle do pais ?
Pq vc não se procupa com eleições idiotas e CARAS - usando o seu
dinheiro - para a presidencia de assembleias que nada fazem ? Pq vc não
se procupa com professores fajutos, mal preparados e que desonram todo o
sistema educativo com os seus comportamentos imbecis que nada aportam à
educação dos jovens ? Pq vc não se preocupa com coisas mais importantes
e que consomem o seu dinheiro a rodos, em vez de se preocupar com coisas
que nem sequer se dá ao trabalho de compreender e que nem sequer gastam
o seu dinheiro?
Já que vc é tão critico, pq não critica coisas mais uteis do que a
astrologia, sendo que vc nem acha que ela funciona ? Critique coisas que
vc acha que fucnionem e andam a funcionar mal!

E eu já pedi para não fazer top-posting. (Isso é outra coisa que vc
deveria ser mais critico, pq está violando as regras da boa conduta na
internet.Ou vc tb acha que isso não funciona?)

Sérgio Taborda


--
No virus found in this outgoing message.
Checked by AVG Anti-Virus.
Version: 7.0.300 / Virus Database: 265.8.6 - Release Date: 07-02-2005



##### ##### #####

Para saber mais visite
http://www.ciencialist.hpg.ig.com.br


##### ##### ##### #####


Yahoo! Grupos, um serviço oferecido por:
PUBLICIDADE




------------------------------------------------------------------------------
Links do Yahoo! Grupos

a.. Para visitar o site do seu grupo na web, acesse:
http://br.groups.yahoo.com/group/ciencialist/

b.. Para sair deste grupo, envie um e-mail para:
ciencialist-unsubscribe@yahoogrupos.com.br

c.. O uso que você faz do Yahoo! Grupos está sujeito aos Termos do Serviço do Yahoo!.



[As partes desta mensagem que não continham texto foram removidas]



SUBJECT: Re: [ciencialist] Re: Zodiaco
FROM: "Oraculo" <oraculo@atibaia.com.br>
TO: <ciencialist@yahoogrupos.com.br>
DATE: 08/02/2005 22:38

Olá Taborda

"Taborda:Aquilo que os ilusionistas fazem em espetaculos é leitura fria, sim e
dai ? O que isso tem a ver com astrologia ?
Vc vai dizer que a maioria dos astrologos fazem leitura fria. Mas como
se mede a leitura fria, como eu sei que ele esta fazendo leitura fria ?
e pior , como eu provo ? Se vc sabe responder a isso, pq vc não prova
que todos os astrologos fazem leitura fria e acaba com o problema?
Faça-se uma licença para astrologos que a recebe quem poder passar no
teste de leitura fria - seja lá o que isso for , supondo que existe tal
teste e ele é 100% infalivel."

Os ilusionistas, como astrólogos, usam diversos mecanismos e ferramentas, não apenas a leitura fria. Generalizações e a natural tendencia da mente humana de contar acertos e ignorar erros também é um ponto importante nas duas artes..:-) Por isso não diria que a maioria dos astrólogos faz leitura fria, embora os que fazem consultas presenciais possam usar a técnica (mesmo sem saber).

Para provar a leitura fria seria preciso acompanhar a consulta e desmontar a linha de apresentação de dados. Isso poderia demonstrar que a técnica estava sendo usada. Mas para isso seria recomendavel que um especialista, provavelmente um psicologo com conhecimento na técnica, o fizesse. Para uma exceletne demonstração dessa técina, ver o programa do John Edwards (arghh..:-) no canal People and Arts, que usa a carencia da plateia para fingir conversar com mortos e espiritos diversos..:-)

E 100% infalivel é uma ótima fuga, principalmente depois de ler dezenas de vezes que a ciência nunca usa essa porcentagem em nada..:-) Assim, se não for 100%, se pode dizer que a astrologia é uma "dúvida razoável" e manter a crença..:-)

Faça um teste, crie um mapa astral com dados falsos e apresente a um cliente como se verdadeiro fosse (não pode ser na presença do mesmo, já que sinais imperceptiveis poderiam indicar a ele sua pouca confiança no mapa, tem de ser em duplo cego) e veja o que ele acha da taxa de acertos. Tenho certeza de que ele irá achar grande semelhança, apesar de a data ser totalmetne incorreta..:-)

A questão básica permanece, cálculos de posição de astros podem informar aspectos da personalidade de uma pessoa, com base em sua data de nascimento. Isso, se fosse real, poderia ser demonstrado e verificado. Não pode, e isso invalida a alegação astrologica. É bem simples, na verdade.

Homero

PS: Como sempre, meu comentário vem depois do trecho relativo ao mesmo, sem top-posting, conforme a boa norma da etiqueta internética..:-)



----- Original Message -----
From: Sergio M. M. Taborda
To: ciencialist@yahoogrupos.com.br
Sent: Tuesday, February 08, 2005 6:21 PM
Subject: Re: [ciencialist] Re: Zodiaco


Oraculo wrote:

>
> Então, no que interessa, eu sei do que estou falando, mesmo que não
> conheça os calculos que você usa na construção de um mapa astral (que
> diferem dos usados por outros astrólogos, mas eles, claro, são uns
> idiotas e estão evidentemente enganados..:-)

Mas o pronto é exactamente esse, vc não sabe do que está falando. Se
sabe, vc não o demonstrou em mensagem nenhuma que eu tenha lido.

>
> Para terminar, não saber nada sobre leitura fria não impede de usar a
> técnica de leitura fria, que é derivada da natural habilidade humana
> de "ler" sinais, informação, reações e qualquer forma de comunicação
> em diversos níveis em outros seres humanos. Essa é uma habilidade que
> pode ser refinada e incrementada mesmo que seu possuidor nada saiba
> sobre ela. É o que chamariamos de uma pessoa " intuitiva", com grande
> habilidade de se comunicar com outros, lendo informação em niveis
> quase subconscientes.
>
> Talvez você seja uma pessoa muito habilidosa nesse sentido, apenas
> isso. E, não seria interessante, para dar maior base a sua refutação
> de que não é leitura fria que ocorre no mapa astral, aprender mais
> sobre essa habilidade, sobre outras capacidades da mente humana?

A leitura fria baseia-se na ideia de que ha pelo menos duas pessoas
conversando. Em astrologia não ha duas pessoas conversando. Por isso ela
não pode ser usada.
Eu posso fazer um mapa de qq pessoas que eu saiba os dados de nascimento
sem nunca a ter visto. Posso inferir o que vai acontecer na vida desta
pessoas. Posso inferir como esta pessoa se relacionaria com outra de
quem eu sei os dados de nascimento. Tudo isto, sem conhecer as pessoas
em momento algum. As pessoas podem já nem existir, ou ainda não existir.
No dia que uma prima minha nasceu eu fiz o seu mapa. E inferir dele como
seria a sua relação basica com os pais e o mundo. Como eu posso fazer
leitura fria com um bebe que nem fala nem se exprime de forma nenhuma ?
Se a intrepretação se adqua aos facto eu não posso culpar a leitura
fria. Culpe-se outra coisa, mas não a leitura fria.
O mesmo argumento pode ser usado pelo Tarot , as runa, etc... por quase
todas as formas de adivinhação, que realmente se podem chamar assim,
pois não carecem da presença de mais ninguem que o usuaário da tecnica.
Embora isso seja muito mais dificil para o tarot e as runas por exemplo,
é muito simples para a astrologia.
Aquilo que os ilusionistas fazem em espetaculos é leitura fria, sim e
dai ? O que isso tem a ver com astrologia ?
Vc vai dizer que a maioria dos astrologos fazem leitura fria. Mas como
se mede a leitura fria, como eu sei que ele esta fazendo leitura fria ?
e pior , como eu provo ? Se vc sabe responder a isso, pq vc não prova
que todos os astrologos fazem leitura fria e acaba com o problema?
Faça-se uma licença para astrologos que a recebe quem poder passar no
teste de leitura fria - seja lá o que isso for , supondo que existe tal
teste e ele é 100% infalivel.

Sérgio Taborda


--
No virus found in this outgoing message.
Checked by AVG Anti-Virus.
Version: 7.0.300 / Virus Database: 265.8.6 - Release Date: 07-02-2005



##### ##### #####

Para saber mais visite
http://www.ciencialist.hpg.ig.com.br


##### ##### ##### #####


Yahoo! Grupos, um serviço oferecido por:

São Paulo Rio de Janeiro Curitiba Porto Alegre Belo Horizonte Brasília




------------------------------------------------------------------------------
Links do Yahoo! Grupos

a.. Para visitar o site do seu grupo na web, acesse:
http://br.groups.yahoo.com/group/ciencialist/

b.. Para sair deste grupo, envie um e-mail para:
ciencialist-unsubscribe@yahoogrupos.com.br

c.. O uso que você faz do Yahoo! Grupos está sujeito aos Termos do Serviço do Yahoo!.



[As partes desta mensagem que não continham texto foram removidas]



SUBJECT: Re: [ciencialist] Fw: Duvidas sobre tensores
FROM: JVictor <jvoneto@uol.com.br>
TO: ciencialist@yahoogrupos.com.br
DATE: 08/02/2005 23:38

Claúdia escreveu:

>
> ===========================
> -----Mensagem Original-----
> De: "Cláudia Adam Ramos" <claudinhadam@yahoo.com.br>
> Para: <leobarretos@uol.com.br>
> Enviada em: domingo, 6 de fevereiro de 2005 21:32
> Assunto: Duvidas sobre tensores
>
>
> Cláudia: Estive visitando o seu site, Feira de Ciência, e
> imagino qu você possa tirar uma duvida minha...sou
> acadêmica do curso de matemática e gostaria de
> entender o sentido físico do divergente de um campo
> tensorial. Não consigo imaginar fisicamente como
> funciona o div de uma matriz de tensoes. Se você puder
> tirar esta duvida minha ficarei grata,

Victor:
Como você fala em campo tensorial, especificamente, deixe-me falar um
pouco sobre isso, não que eu ache que você não sabe ou não entendeu
adequadamente. É que me ajuda a discernir melhor sobre a questão que,
aliás, é uma boa questão. Um campo tensorial é essencialmente uma função
tensorial da posição, função esta que associa um tensor com cada ponto
do espaço. Agora, o importante é compreender que os campos tensoriais
podem ser caracterizados como de ordem zero, de ordem 1 e ordem maior
que um.
1)Um campo tensorial de ordem zero é, simplesmente, um campo escalar,
que você já conhece. Por exemplo, a distribuição de temperaturas em um
corpo ou em uma região do espaço configura-se como um campo escalar. A
temperatura é, então, uma função de coordenadas cartersianas, contínua,
derivável e local. Em cada ponto essa função pode associar um único
número, uma medida do valor da temperatura, tal como T(0,2,4) = 70graus
centígrados, e este único número é suficiente para uma compreensão
clara do que vai pela região, naquele ponto identificado pelas corrdenas
(0,2,4). Um campo tensorial de ordem zero é chamado, "carinhosamente",
de campo escalar.
2) Em um campo tensorial de ordem 1 é preciso associar a cada ponto do
campo três números, três parâmetros, para que as propriedades do ponto
sejam perfeitamente entendidas e caracterizadas. Estes três números são,
então as componentes de uma entidade matemática a que se chamou vetor.
Exemplos, mais conhecidos do que nós próprios: velocidade e
deslocamento de um corpo. Todas essas grandezas só são perfeitamente
caracterizadas, definidas, mediante três parâmetros. No caso do vetor
deslocamento d(x), x coordenadas cartesianas 3-dimensionais, esse moço
carece, para sua identificação completa, de: um módulo, que me dá
dimensão de seu "tamanho", de uma direção, que me diz para onde se vai e
de um sentido, que me diz se vou naquele sentido, ou no sentido
contrário. Esse campo tensorial de ordem um é chamado de campo vetorial!
3) Em campo tensorial de 2 ordem, a função associa a cada ponto do
espaço não um ou três parâmetros, como nos campos escalare ou vetorial,
mas a bagatela de nove parâmetros!. Se o campo for de ordem três, o
ponto espacial é definido por 27 parâmetros, e assim por diante.

Então, para compreender o significado físico do divergente de um campo
tensorial, basta entender o significado físico do divergente de um campo
vetorial!

Não há diferença, a nãoser pelo número de componentes da entidade. Num
campo vetorial, o divergente em um ponto é, simplesmente, a densidade do
fluxo da grandeza em estudo, naquele ponto. Em um campo tensorial,
genérico, de ordem qualquer, é a mesma coisa. Mas sempre pense no campo
tensorial de ordem 1, que você já conhece; o resto virá por acréscimo. O
que é válido para ele, o é para os demais. Isso é que é democracia!
Há muitas situações, de nosso conhecimento assim mais imediato, onde
convivem, numa boa, sem brigas, campos tensorias de ordem zero, de
ordem 1 e de ordem 2. Por exemplo: um fluido escoando. Temos nele, no
fluido em escoamento: o campo escalar de pressões P(x)-pressões são
escalares-, o campo vetorial da velocidade(pois o fluido está escoando!)
v(x) e um outro campo de ordem maior, que é o campo das tensões no
fluído, cuja designação é tensor das tensões mesmo. Esse campo é um
campo tensorial de ordem 2, com nove componentes. Aliás, esse tensor das
tensões, é de uma importância extrema e fundamental no estudo dos
fluídos, com aplicações, inclusive, na RG. A sua compreensão facilita
enormemente a compreensão de muita coisa no estudo de física.
Há um monstrengo na TRG, chamado Tensor momento-energia, elemento
fundamental e dominante na famosa equação de Einstein da Gravitação,
bastante complexo, ao qual se pode chegar, de maneira bastante simples
e sem muita dor, usando a matriz do tensor tensão. Há outros caminhos,
mas este é o que seguí, sem perder de vista em nenhum momento os
significados físicos inerentes a todo o processo. Há muitas outros
caminhos. Mas corrí deles. Que ninguém é de ferro.Pode até ser besta.
Mas isso é outra coisa!...

Não perca a oportunidade de dominar esse camarada, o cálculo tensorial,
como se domina um namorado apaixonado: com afagos e carinhos!.

Referí-me ao divergente como uma densidade de fluxo. Mas não sei se
alguns dos que lerem esta mensagem vão saber o que diabos é isso. Por
isso, vou traduzir o que é mesmo um divergente, de maneira bem simples e
direta. O divergente é o resultado(um número, um escalar) de uma
operação matemática, que não vem ao caso agora. Para compreender o que
significa, usemos o seguinte exemplo: um recipiente por onde pode
entrar, sair algo, ou nenhum nem outro; um fluído, como água, por
exemplo, para simplificar. Se, num dado intervalo de tempo, entra água
dizemos, por convenção, que o divergente é negativo:o fluído "diverge"
para dentro; se saí água, dizemos que o divergente é positivo: o fluido
"diverge" para fora; é imediato que, se não entra ou não sai nada, o
divergente é zero. As definições matemáticas do processo são simples e
você as conhece. É só isso. Após a compreensão da coisa física, vem o
rigor da sistematiazação, que fica para depois.

Por falar em divergente, aproveito o ensejo para dizer dizer que se
trata de uma quantidade de extrema importância em física. Por resultar
num número, é um escalar! E é invariante, qualquer que seja o sistema de
referência que você use para calculá-lo!. A "agonia" dos físicos, ao
longo da evolução da física, foi, precisamente, a busca de grandezas
invariantes. Bem como ferramentas matemáticas cujos cálculos resultassem
nas mesmas quantidades em qualquer sistema de referência.
Pois bem, essa ferramenta matemática existe, não é difícil, é poderosa e
absolutamente geral, e é usada para descrever todos os fenômenos
físicos. E qual é?
Exatamente o que você está estudando agora: o cálculo tensorial! É um
cálculo de invariantes, essencialmente, e isto faz uma tremenda diferença.
O que você deve saber para aprender e compreender toda importância e
incidências da ferramenta: cálculo diferencial e integral( um básico
mais que básico...), diferenciação parcial de várias variáveis; é
indispensável tembém um conhecimento e uma compreensão de cálculo
vetorial(onde se aprende coisas fundamentais como divergente,
rotacional, gradiente, teoremas tais como de Green, Helmoltz e Gauss) e
álgebra linear. Não escanteie Álgebra Linear! No estudo dos tensores,
tudo isso é largamente usado. Portanto...
Sabe qual foi o primeiro cientista a aplicar esse, inicialmente
"nebuloso", formalismo a questões concretas de física?
Ele mesmo: Einstein,que viu naquela matemática então desconhecida pela
grande maioria de físicos e até matemáticos, o caminho das pedras para
estruturar sua TRG; inclusive dando uma vigorosa contribuição ao
cálculo tensorial, o que deu a esse formalismo um baita impulso, pela
simplificação produzida, e uma juda incomensurável aos preguiçosos, como
eu, que se enrola quando aparecem somatórios cavernosos e mais
somatórios igualmente chatos. Putz!

Bem, há muita coisa a falar sobre essas coisas, mas, como já fugí muito
ao assunto central, que era a resposta à tua pergunta, vou parar por
aquí. Não sei se conseguí que você elimine a dúvida. Mas se perdurar,
pode mandar outra dúvida prá gente discutir e aprender também.
En passant:
Observação final, sem ser para meter medo. Mais à frente você aprenderá
um conceito vital em cálculo tensorial: trata-se do conceito importante
de derivada covariante. E você aprenderá que a maneira correta de
calcular o divergente de um campo tensorial é contraindo a derivada
covariante desse campo. Mas esquente não, pois não morde!

Sds,

Victor.







> A



> __________________________________________________
> Converse com seus amigos em tempo real com o Yahoo! Messenger
> http://br.download.yahoo.com/messenger/
>
>
> --
> No virus found in this incoming message.
> Checked by AVG Anti-Virus.
> Version: 7.0.300 / Virus Database: 265.8.5 - Release Date: 03/02/2005
>
>
>
>
> --
> No virus found in this outgoing message.
> Checked by AVG Anti-Virus.
> Version: 7.0.300 / Virus Database: 265.8.6 - Release Date: 07/02/2005
>
>
>
> ##### ##### #####
>
> Para saber mais visite
> http://www.ciencialist.hpg.ig.com.br
>
>
> ##### ##### ##### #####
>
>
> *Yahoo! Grupos, um serviço oferecido por:*
> PUBLICIDADE
> <http://br.rd.yahoo.com/SIG=12a8bj8dd/M=264379.5078783.6203979.1588051/D=brclubs/S=2137111528:HM/EXP=1107946561/A=2332652/R=0/id=noscript/SIG=119058f8i/*http://br.download.yahoo.com/messenger/>
>
>
>
> ------------------------------------------------------------------------
> *Links do Yahoo! Grupos*
>
> * Para visitar o site do seu grupo na web, acesse:
> http://br.groups.yahoo.com/group/ciencialist/
>
> * Para sair deste grupo, envie um e-mail para:
> ciencialist-unsubscribe@yahoogrupos.com.br
> <mailto:ciencialist-unsubscribe@yahoogrupos.com.br?subject=Unsubscribe>
>
> * O uso que você faz do Yahoo! Grupos está sujeito aos Termos do
> Serviço do Yahoo! <http://br.yahoo.com/info/utos.html>.
>
>
>
>
> __________ Informação do NOD32 1.990 (20050202) __________
>
> Esta mensagem foi verificada pelo NOD32 Sistema Antivírus
> http://www.nod32.com.br





SUBJECT: Sobre batimentos em ondas sonoras
FROM: Franco <dfranco@pop.com.br>
TO: Ciencialist <ciencialist@yahoogrupos.com.br>
DATE: 08/02/2005 23:46

Gente, o princípio para afinar, digamos, uma corda de violão, faz-se
comparando a freqüência dessa corda com a emitida por um diapasão, por
exemplo. Quando notam-se batimentos, então, sei que as freqüências estão
bem próximas. Mas ainda tenho uma dúvida. À medida que vamos "esticando"
a corda (fazendo-a tocar numa freqüênca mais alta), o intervalo entre um
batimento e outro vai diminuindo até desaparecer.
Logo, a partir de que ponto deve-se "afinar" a corda do violão - supondo
aqui que estamos "esticando" a corda: antes de os batimentos serem
percebidos (quando o intervalo entre um e outro é longo), ou a partir do
ponto em que tal intervalo entre um batimento e outro seja tão curto que
eles desaparecem?

Franco.




SUBJECT: Re: [ciencialist] Fw: Duvidas sobre tensores
FROM: "Luiz Ferraz Netto" <leobarretos@uol.com.br>
TO: <ciencialist@yahoogrupos.com.br>
DATE: 09/02/2005 06:02

JVictor nos ensina:
>Como você fala em campo tensorial, especificamente, deixe-me falar um
pouco sobre isso, não que eu ache que você não sabe ou não entendeu
adequadamente. É que me ajuda a discernir melhor sobre a questão que,
aliás, é uma boa questão. Um campo tensorial é essencialmente uma função
tensorial da posição, função esta que associa um tensor com cada ponto
do espaço. Agora, o importante é compreender que os campos tensoriais
podem ser caracterizados como de ordem zero, de ordem 1 e ordem maior
que um.
1)Um campo tensorial de ordem zero é, simplesmente, um campo escalar,
que você já conhece. Por exemplo, a distribuição de temperaturas em um
corpo ou em uma região do espaço configura-se como um campo escalar. A
temperatura é, então, uma função de coordenadas cartersianas, contínua,
derivável e local. Em cada ponto essa função pode associar um único
número, uma medida do valor da temperatura, tal como T(0,2,4) = 70graus
centígrados, e este único número é suficiente para uma compreensão
clara do que vai pela região, naquele ponto identificado pelas corrdenas
(0,2,4). Um campo tensorial de ordem zero é chamado, "carinhosamente",
de campo escalar.
2) Em um campo tensorial de ordem 1 é preciso associar a cada ponto do
campo três números, três parâmetros, para que as propriedades do ponto
sejam perfeitamente entendidas e caracterizadas. Estes três números são,
então as componentes de uma entidade matemática a que se chamou vetor.
Exemplos, mais conhecidos do que nós próprios: velocidade e
deslocamento de um corpo. Todas essas grandezas só são perfeitamente
caracterizadas, definidas, mediante três parâmetros. No caso do vetor
deslocamento d(x), x coordenadas cartesianas 3-dimensionais, esse moço
carece, para sua identificação completa, de: um módulo, que me dá
dimensão de seu "tamanho", de uma direção, que me diz para onde se vai e
de um sentido, que me diz se vou naquele sentido, ou no sentido
contrário. Esse campo tensorial de ordem um é chamado de campo vetorial!
3) Em campo tensorial de 2 ordem, a função associa a cada ponto do
espaço não um ou três parâmetros, como nos campos escalare ou vetorial,
mas a bagatela de nove parâmetros!. Se o campo for de ordem três, o
ponto espacial é definido por 27 parâmetros, e assim por diante.

Então, para compreender o significado físico do divergente de um campo
tensorial, basta entender o significado físico do divergente de um campo
vetorial!

Não há diferença, a nãoser pelo número de componentes da entidade. Num
campo vetorial, o divergente em um ponto é, simplesmente, a densidade do
fluxo da grandeza em estudo, naquele ponto. Em um campo tensorial,
genérico, de ordem qualquer, é a mesma coisa. Mas sempre pense no campo
tensorial de ordem 1, que você já conhece; o resto virá por acréscimo. O
que é válido para ele, o é para os demais. Isso é que é democracia!
Há muitas situações, de nosso conhecimento assim mais imediato, onde
convivem, numa boa, sem brigas, campos tensorias de ordem zero, de
ordem 1 e de ordem 2. Por exemplo: um fluido escoando. Temos nele, no
fluido em escoamento: o campo escalar de pressões P(x)-pressões são
escalares-, o campo vetorial da velocidade(pois o fluido está escoando!)
v(x) e um outro campo de ordem maior, que é o campo das tensões no
fluído, cuja designação é tensor das tensões mesmo. Esse campo é um
campo tensorial de ordem 2, com nove componentes. Aliás, esse tensor das
tensões, é de uma importância extrema e fundamental no estudo dos
fluídos, com aplicações, inclusive, na RG. A sua compreensão facilita
enormemente a compreensão de muita coisa no estudo de física.
Há um monstrengo na TRG, chamado Tensor momento-energia, elemento
fundamental e dominante na famosa equação de Einstein da Gravitação,
bastante complexo, ao qual se pode chegar, de maneira bastante simples
e sem muita dor, usando a matriz do tensor tensão. Há outros caminhos,
mas este é o que seguí, sem perder de vista em nenhum momento os
significados físicos inerentes a todo o processo. Há muitas outros
caminhos. Mas corrí deles. Que ninguém é de ferro.Pode até ser besta.
Mas isso é outra coisa!...

Não perca a oportunidade de dominar esse camarada, o cálculo tensorial,
como se domina um namorado apaixonado: com afagos e carinhos!.

Referí-me ao divergente como uma densidade de fluxo. Mas não sei se
alguns dos que lerem esta mensagem vão saber o que diabos é isso. Por
isso, vou traduzir o que é mesmo um divergente, de maneira bem simples e
direta. O divergente é o resultado(um número, um escalar) de uma
operação matemática, que não vem ao caso agora. Para compreender o que
significa, usemos o seguinte exemplo: um recipiente por onde pode
entrar, sair algo, ou nenhum nem outro; um fluído, como água, por
exemplo, para simplificar. Se, num dado intervalo de tempo, entra água
dizemos, por convenção, que o divergente é negativo:o fluído "diverge"
para dentro; se saí água, dizemos que o divergente é positivo: o fluido
"diverge" para fora; é imediato que, se não entra ou não sai nada, o
divergente é zero. As definições matemáticas do processo são simples e
você as conhece. É só isso. Após a compreensão da coisa física, vem o
rigor da sistematiazação, que fica para depois.

Por falar em divergente, aproveito o ensejo para dizer dizer que se
trata de uma quantidade de extrema importância em física. Por resultar
num número, é um escalar! E é invariante, qualquer que seja o sistema de
referência que você use para calculá-lo!. A "agonia" dos físicos, ao
longo da evolução da física, foi, precisamente, a busca de grandezas
invariantes. Bem como ferramentas matemáticas cujos cálculos resultassem
nas mesmas quantidades em qualquer sistema de referência.
Pois bem, essa ferramenta matemática existe, não é difícil, é poderosa e
absolutamente geral, e é usada para descrever todos os fenômenos
físicos. E qual é?
Exatamente o que você está estudando agora: o cálculo tensorial! É um
cálculo de invariantes, essencialmente, e isto faz uma tremenda diferença.
O que você deve saber para aprender e compreender toda importância e
incidências da ferramenta: cálculo diferencial e integral( um básico
mais que básico...), diferenciação parcial de várias variáveis; é
indispensável tembém um conhecimento e uma compreensão de cálculo
vetorial(onde se aprende coisas fundamentais como divergente,
rotacional, gradiente, teoremas tais como de Green, Helmoltz e Gauss) e
álgebra linear. Não escanteie Álgebra Linear! No estudo dos tensores,
tudo isso é largamente usado. Portanto...
Sabe qual foi o primeiro cientista a aplicar esse, inicialmente
"nebuloso", formalismo a questões concretas de física?
Ele mesmo: Einstein,que viu naquela matemática então desconhecida pela
grande maioria de físicos e até matemáticos, o caminho das pedras para
estruturar sua TRG; inclusive dando uma vigorosa contribuição ao
cálculo tensorial, o que deu a esse formalismo um baita impulso, pela
simplificação produzida, e uma juda incomensurável aos preguiçosos, como
eu, que se enrola quando aparecem somatórios cavernosos e mais
somatórios igualmente chatos. Putz!

Bem, há muita coisa a falar sobre essas coisas, mas, como já fugí muito
ao assunto central, que era a resposta à tua pergunta, vou parar por
aquí. Não sei se conseguí que você elimine a dúvida. Mas se perdurar,
pode mandar outra dúvida prá gente discutir e aprender também.
En passant:
Observação final, sem ser para meter medo. Mais à frente você aprenderá
um conceito vital em cálculo tensorial: trata-se do conceito importante
de derivada covariante. E você aprenderá que a maneira correta de
calcular o divergente de um campo tensorial é contraindo a derivada
covariante desse campo. Mas esquente não, pois não morde!>

Léo:
Nem me atrevi a 'contar' a mensagem no ponto desejado, tal a generalidade do tema. Só retorno para mais um exemplo no campo tensorial de 2 ordem, além da hidrodinâmica, que é o das torções em sistemas discretos.
Esse exemplo eu uso em palestras para um tensor das tensões aplicados a um giz (sim, esse pedacinho tronco-cônico de, se não me falha a moringa, de carbonato de cálcio -- CaCO3 -- um sistema amorfo e prensado; um sistema discreto). Se vc segura o giz pelos extremos (uma mão em cada 'metade') e torce o giz, ele quebra 'exatamente a 45 graus'. Só mesmo fazendo isso para se acreditar nessa proeza da matemática tensorial. O tensor das tensões é representado por uma matriz (9) e apenas os elementos da diagonal principal são não nulos; o que justifica a 'quebra' em ângulo de 45 graus.
Ao se fazer essa quebra do giz, pela torção, repare com muito cuidado que, de um cantinho da quebra desprende-se um pedacinho de giz, coisa pequena com área ao redor dos 2 mm^2, chamado 'cúspide' --- e esse fenômeno ainda não consegui a devida explicação.

Aproveito o embalo dessa maravilhosa manhã (05:50 h) para falar da 'queda do giz' (e aproveitar para uma cutucada nos cuímicos:-)). Quando um giz cai de cerca dos 2 m de altura, ele quebra. Pode quebrar em 2, 3 ou 4 partes; o mais comum é quebrar em 3 partes, devido a maior probabilidade de tocar o chão por uma de suas extremidades.
Para justificar essas quebras em 2, 3 ou 4 partes basta analisar as forças de interação e as forças de inércia, que nele agem, por ocasião do choque com o piso, com destaque especial para os centros de massa (CM) e de percussão (CP).
Mas, como na maioria das vezes o giz quebra em 3 partes, os cuímicos explicam isso pela fórmula CaCO3 ---- CACO 3 --- 3 cacos!

depois dessa........

aquele abraço,

Léo


--
No virus found in this outgoing message.
Checked by AVG Anti-Virus.
Version: 7.0.300 / Virus Database: 265.8.6 - Release Date: 07/02/2005



SUBJECT: Re: [ciencialist] Sobre batimentos em ondas sonoras
FROM: "Luiz Ferraz Netto" <leobarretos@uol.com.br>
TO: <ciencialist@yahoogrupos.com.br>
DATE: 09/02/2005 06:30

Oi Franco,

vc tem/consegue osciloscópio, 2 canais?
[]'
===========================
Luiz Ferraz Netto [Léo]
leobarretos@uol.com.br
http://www.feiradeciencias.com.br
===========================
-----Mensagem Original-----
De: "Franco" <dfranco@pop.com.br>
Para: "Ciencialist" <ciencialist@yahoogrupos.com.br>
Enviada em: terça-feira, 8 de fevereiro de 2005 23:46
Assunto: [ciencialist] Sobre batimentos em ondas sonoras



Gente, o princípio para afinar, digamos, uma corda de violão, faz-se
comparando a freqüência dessa corda com a emitida por um diapasão, por
exemplo. Quando notam-se batimentos, então, sei que as freqüências estão
bem próximas. Mas ainda tenho uma dúvida. À medida que vamos "esticando"
a corda (fazendo-a tocar numa freqüênca mais alta), o intervalo entre um
batimento e outro vai diminuindo até desaparecer.
Logo, a partir de que ponto deve-se "afinar" a corda do violão - supondo
aqui que estamos "esticando" a corda: antes de os batimentos serem
percebidos (quando o intervalo entre um e outro é longo), ou a partir do
ponto em que tal intervalo entre um batimento e outro seja tão curto que
eles desaparecem?

Franco.




##### ##### #####

Para saber mais visite
http://www.ciencialist.hpg.ig.com.br


##### ##### ##### #####
Links do Yahoo! Grupos










--
No virus found in this incoming message.
Checked by AVG Anti-Virus.
Version: 7.0.300 / Virus Database: 265.8.6 - Release Date: 07/02/2005




--
No virus found in this outgoing message.
Checked by AVG Anti-Virus.
Version: 7.0.300 / Virus Database: 265.8.6 - Release Date: 07/02/2005



SUBJECT: Re: [ciencialist] Astrologia e Metereologia e top-posting (era : Zodiaco)
FROM: "Luiz Ferraz Netto" <leobarretos@uol.com.br>
TO: <ciencialist@yahoogrupos.com.br>
DATE: 09/02/2005 06:43

Bolas! O que é Metereologia?

[]'
Léo


--
No virus found in this outgoing message.
Checked by AVG Anti-Virus.
Version: 7.0.300 / Virus Database: 265.8.6 - Release Date: 07/02/2005



SUBJECT: Re: Zodiaco
FROM: "rmtakata" <rmtakata@altavista.net>
TO: ciencialist@yahoogrupos.com.br
DATE: 09/02/2005 09:24


ANCIENT EGYPTIAN ASTRONOMY
http://www.nmc.edu/science-math/Astronomy/AST109%20Class%20Page/Egypt.pdf

[]s,

Roberto Takata





SUBJECT: Re: Zodiaco
FROM: "rmtakata" <rmtakata@altavista.net>
TO: ciencialist@yahoogrupos.com.br
DATE: 09/02/2005 11:23


The zodiac in history
http://astro.wcupa.edu/mgagne/ess362/notes/zodiac.ppt

[]s,

Roberto Takata






SUBJECT: Re: [ciencialist] Astrologia e Metereologia e top-posting (era : Zodiaco)
FROM: "Alvaro Augusto \(E\)" <alvaro@electraenergy.com.br>
TO: <ciencialist@yahoogrupos.com.br>
DATE: 09/02/2005 13:52

Deve ser aquilo que os meteorologistas fazem quando estão de ressaca...

Bem, o próprio termo "meteorologia" é incorreto. Meteorologia deveria ser o estudo dos meteoros.... Previsão do tempo deveria receber outro nome, tal como "tempoprevisologia"...

[ ]s

Alvaro Augusto

----- Original Message -----
From: Luiz Ferraz Netto
To: ciencialist@yahoogrupos.com.br
Sent: Wednesday, February 09, 2005 6:43 AM
Subject: Re: [ciencialist] Astrologia e Metereologia e top-posting (era : Zodiaco)


Bolas! O que é Metereologia?

[]'
Léo


--
No virus found in this outgoing message.
Checked by AVG Anti-Virus.
Version: 7.0.300 / Virus Database: 265.8.6 - Release Date: 07/02/2005



##### ##### #####

Para saber mais visite
http://www.ciencialist.hpg.ig.com.br


##### ##### ##### #####


Yahoo! Grupos, um serviço oferecido por:

São Paulo Rio de Janeiro Curitiba Porto Alegre Belo Horizonte Brasília




------------------------------------------------------------------------------
Links do Yahoo! Grupos

a.. Para visitar o site do seu grupo na web, acesse:
http://br.groups.yahoo.com/group/ciencialist/

b.. Para sair deste grupo, envie um e-mail para:
ciencialist-unsubscribe@yahoogrupos.com.br

c.. O uso que você faz do Yahoo! Grupos está sujeito aos Termos do Serviço do Yahoo!.



[As partes desta mensagem que não continham texto foram removidas]



SUBJECT: Re: Astrologia e Metereologia e top-posting (era : Zodiaco)
FROM: "rmtakata" <rmtakata@altavista.net>
TO: ciencialist@yahoogrupos.com.br
DATE: 09/02/2005 14:32


--- Em ciencialist@yahoogrupos.com.br, "Alvaro Augusto \(E\)"
> Bem, o próprio termo "meteorologia" é incorreto. Meteorologia
> deveria ser o estudo dos meteoros.... Previsão do tempo deveria
> receber outro nome, tal como "tempoprevisologia"...

Nem tao incorreto. Em certo sentido a meteorologia atual eh sim
estudos de meteoros. So' q. meteoros em uma acepcao um tto em desuso:
fenomenos q. se apresentam no ceu, no ar.

A meteorologia embora conhecida mais pela previsao do tempo, vai um
pouco mais alem. Faz previsoes de clima (importante para o sistema
energetico nacional - embora o ONS nem sempre seja competente para
usar as informacoes sobre a media pluviometrica, incidencias de raios
e quetais), cria modelos de troca de energia entre a terra, oceanos e
a atmosfera; monitora queimadas, faz relatorios de balanco hidrico,
previsao de eclosao de doencas relacionadas ao clima (ar seco, dengue,
poluicao), indices de UV...

[]s,

Roberto Takata





SUBJECT: Meteorologia corrigida..:-) (era Astrologia e Mete(o)rologia e top-posting
FROM: "Oraculo" <oraculo@atibaia.com.br>
TO: <ciencialist@yahoogrupos.com.br>
DATE: 09/02/2005 14:40

Olá Luiz

risos...:-) Deve ser o que resulta de quem se mete a escrever "de ouvido" como eu e as pressas (o pior ]e que meu vacilo ficará eternizado no s títuos desta thread na lista..:-) Falha minha, mas note que não estou só, o Alvaro diz que mesmo meteorologia está incorreto..:-)

Então, leia-se "tempoprevisologia" onde escrevi metereologia e tudo ficará bem..:-)

Um abraço.

Homero

----- Original Message -----
From: Alvaro Augusto (E)
To: ciencialist@yahoogrupos.com.br
Sent: Wednesday, February 09, 2005 1:52 PM
Subject: Re: [ciencialist] Astrologia e Metereologia e top-posting (era : Zodiaco)


Deve ser aquilo que os meteorologistas fazem quando estão de ressaca...

Bem, o próprio termo "meteorologia" é incorreto. Meteorologia deveria ser o estudo dos meteoros.... Previsão do tempo deveria receber outro nome, tal como "tempoprevisologia"...

[ ]s

Alvaro Augusto

----- Original Message -----
From: Luiz Ferraz Netto
To: ciencialist@yahoogrupos.com.br
Sent: Wednesday, February 09, 2005 6:43 AM
Subject: Re: [ciencialist] Astrologia e Metereologia e top-posting (era : Zodiaco)


Bolas! O que é Metereologia?

[]'
Léo


--
No virus found in this outgoing message.
Checked by AVG Anti-Virus.
Version: 7.0.300 / Virus Database: 265.8.6 - Release Date: 07/02/2005



##### ##### #####

Para saber mais visite
http://www.ciencialist.hpg.ig.com.br


##### ##### ##### #####


Yahoo! Grupos, um serviço oferecido por:

São Paulo Rio de Janeiro Curitiba Porto Alegre Belo Horizonte Brasília




------------------------------------------------------------------------------
Links do Yahoo! Grupos

a.. Para visitar o site do seu grupo na web, acesse:
http://br.groups.yahoo.com/group/ciencialist/

b.. Para sair deste grupo, envie um e-mail para:
ciencialist-unsubscribe@yahoogrupos.com.br

c.. O uso que você faz do Yahoo! Grupos está sujeito aos Termos do Serviço do Yahoo!.



[As partes desta mensagem que não continham texto foram removidas]



##### ##### #####

Para saber mais visite
http://www.ciencialist.hpg.ig.com.br


##### ##### ##### #####


Yahoo! Grupos, um serviço oferecido por:
PUBLICIDADE




------------------------------------------------------------------------------
Links do Yahoo! Grupos

a.. Para visitar o site do seu grupo na web, acesse:
http://br.groups.yahoo.com/group/ciencialist/

b.. Para sair deste grupo, envie um e-mail para:
ciencialist-unsubscribe@yahoogrupos.com.br

c.. O uso que você faz do Yahoo! Grupos está sujeito aos Termos do Serviço do Yahoo!.



[As partes desta mensagem que não continham texto foram removidas]



SUBJECT: Media pluviometrica
FROM: "Luiz Ferraz Netto" <leobarretos@uol.com.br>
TO: <ciencialist@yahoogrupos.com.br>
DATE: 09/02/2005 15:29

Ois,

vou aproveitar essa thread do 'meteorologia' e a dissertação do Roberto para falar dessa 'medida pluviométrica'.

O jornal acusa: ....a chuvarada de 1 hora alcançou "100 mm", alagando tudo .... etc.

Entendo que esses "100 mm" corresponde a uma caixa de base 1 m^2 e água alcançando a altura de 100 mm (10 cm = 0,1 m), ou seja, um volume de água de 0,1 m^3.

1 m^3 de água são 1000 litros; 0,1 m^3 são 100 litros (meio tambor de água)
Se essa chuva durou 1 hora (3600 segundos) isso significa que em cada m^2 de terreno caiu 100 litros em 3600 s, ou seja, com boa vontade, 0,03 litros por segundo (1,6 litros por minuto por metro quadrado).

Bolas, pela minha intuição, isso não é lá 'aquela chuvarada' a ponto de invocarmos "Por Tutatis". Estou errado nas continhas?

[]'
===========================
Luiz Ferraz Netto [Léo]
leobarretos@uol.com.br
http://www.feiradeciencias.com.br
===========================
-----Mensagem Original-----
De: "rmtakata" <rmtakata@altavista.net>
Para: <ciencialist@yahoogrupos.com.br>
Enviada em: quarta-feira, 9 de fevereiro de 2005 14:32
Assunto: [ciencialist] Re: Astrologia e Metereologia e top-posting (era : Zodiaco)




--- Em ciencialist@yahoogrupos.com.br, "Alvaro Augusto \(E\)"
> Bem, o próprio termo "meteorologia" é incorreto. Meteorologia
> deveria ser o estudo dos meteoros.... Previsão do tempo deveria
> receber outro nome, tal como "tempoprevisologia"...

Nem tao incorreto. Em certo sentido a meteorologia atual eh sim
estudos de meteoros. So' q. meteoros em uma acepcao um tto em desuso:
fenomenos q. se apresentam no ceu, no ar.

A meteorologia embora conhecida mais pela previsao do tempo, vai um
pouco mais alem. Faz previsoes de clima (importante para o sistema
energetico nacional - embora o ONS nem sempre seja competente para
usar as informacoes sobre a media pluviometrica, incidencias de raios
e quetais), cria modelos de troca de energia entre a terra, oceanos e
a atmosfera; monitora queimadas, faz relatorios de balanco hidrico,
previsao de eclosao de doencas relacionadas ao clima (ar seco, dengue,
poluicao), indices de UV...

[]s,

Roberto Takata





##### ##### #####

Para saber mais visite
http://www.ciencialist.hpg.ig.com.br


##### ##### ##### #####
Links do Yahoo! Grupos










--
No virus found in this incoming message.
Checked by AVG Anti-Virus.
Version: 7.0.300 / Virus Database: 265.8.6 - Release Date: 07/02/2005




--
No virus found in this outgoing message.
Checked by AVG Anti-Virus.
Version: 7.0.300 / Virus Database: 265.8.6 - Release Date: 07/02/2005



SUBJECT: que tal, convence?
FROM: "rayfisica" <rayfisica@yahoo.com.br>
TO: ciencialist@yahoogrupos.com.br
DATE: 09/02/2005 15:32


Surfando por aqui e ali encontrei isso, penso ser porque o Sr.
Clistene sofreu algumas pressões, mas de qualquer maneira serve
como
material de estudo

Algumas equações não aparecerão portanto aqui o link

http://www.cefetsp.br/edu/sinergia/7p6c.html

SOBRE A AMPLIFICAÇÃO DA ENERGIA

Clístenes Xavier de França
Professor da Área de Projeto e Representações Gráficas para
Edificações
do CEFET-PB/UNED - Cajazeiras


A energia do universo é constante? É possível criar energia
do nada? Essas e outras questões são tratadas neste artigo que
discute a conservação da energia no processo descrito em artigo,
do
mesmo autor, sob o título "É possível amplificar a energia?".
Palavras-chave: energia; eletromagnetismo; trabalho.
Is the energy of the universe constant? Is it possible to create
energy from nothing? This and other matters are treated in this
article which discusses the conservation of the energy in the process
described in the article, by the same author, under the title "Is it
possible to increase energy?
Key words: energy; electromagnetism; work.

INTRODUÇÃO
Para melhor compreensão das questões ora discutidas é
indispensável a
leitura do artigo anterior "É possível amplificar a
energia?"[1],
onde é descrito um processo no qual a energia parece não se
conservar. Agora é retomada a discussão sobre a amplificação
da
energia enfocando a conservação. Finalmente é feita uma
análise da
afirmação de Rudolf Julius Emmanuel Clausius (1822-1888) [2], que
descobriu a segunda lei da termodinâmica em 1850, e ao estudar o
trabalho de Nicolas Leonard Sadi Carnot (1796-1832), em 1865,
afirmou: "A energia do universo é constante...".

UMA ANALOGIA
Para facilitar a análise da conservação da energia no processo
descrito no artigo "É possível amplificar a energia?" [1] vamos
fazer
uma analogia com a conservação da energia no campo gravitacional:
Na sacada do décimo andar de um edifício existe um objeto. Um
indivíduo (agente externo) executa o trabalho de efetuar um
pequeno
deslocamento até que ele caia. Ao cair, o corpo fica sujeito à
ação
do campo gravitacional, que transforma sua energia potencial em
energia cinética, até que ele se choque com a superfície
terrestre. A
energia cinética do objeto, que é máxima no instante
imediatamente
anterior ao choque, é toda convertida em calor e em outras formas
de
energia.
Estamos considerando que esse processo se inicia com o corpo já na
altura relativa ao décimo andar do edifício. Está faltando,
portanto,
computar uma parcela da energia: exatamente aquela relativa à
energia
externa necessária para elevá-lo até aquela altura. No final
do
processo, essa é a energia que é transformada em calor e em
outras
formas após o choque do corpo com a superfície da Terra.
Nenhuma
energia potencial fica no objeto porque, na queda, ele não pode ir
além da superfície terrestre.
É interessante observar que a energia adquirida pelo corpo, ao
cair,
independe totalmente da grandeza do trabalho realizado pelo
indivíduo
do décimo andar. Isso ocorre porque o campo gravitacional é
conservativo e, nesse caso, tanto a energia potencial quanto a
cinética dependem apenas da altura em que o objeto se encontrava
em
relação à superfície terrestre.
O processo descrito no artigo [1], no subtítulo "Um amplificador
de
energia", se inicia, como no exemplo acima descrito, numa posição
em
que um pequeno trabalho realizado faz desencadear uma energia muito
maior.
Na posição inicial, com o sistema desligado, os átomos do
gás, no
interior da válvula, são neutros ou não têm um movimento
preferencial
por não estarem sujeitos à ação de um campo elétrico.
Essa posição é
equivalente, no processo acima, ao objeto situado no décimo andar
do
edifício. Ao ser ligado, a energia da fonte, fluindo através de
um
circuito apropriado, causa a ionização do gás residual no
interior da
válvula, criando elétrons livres e íons positivos e dotando
essas
partículas de alguma energia cinética necessária para que
elas se
choquem com os coletores da válvula. Essa ação é semelhante
ao
trabalho executado pelo indivíduo do décimo andar dando um
empurrãozinho no objeto até que ele caia.
Após chocarem-se com os coletores, as cargas elétricas adquirem
energia cinética até chegarem à superfície externa da
armadura
interna do capacitor, indo compor a carga dele. Essa energia
cinética
é fornecida pela propriedade inerente aos condutores isolados que
estabelece que o excesso de cargas elétricas tem que fluir,
obrigatoriamente, para a superfície externa. Isso é
equivalente, na
analogia acima, à ação do campo gravitacional acelerando o
objeto e
transformando sua energia potencial em energia cinética. No
capacitor, a energia é novamente transformada em energia potencial
e
aí fica acumulada para ser convertida em trabalho.
A parcela que falta ser computada, no processo, é relativa à
energia
acumulada no campo elétrico que é equivalente à energia que
um agente
externo utilizaria para transportar a carga de uma até a outra
armadura do capacitor. Tal parcela não foi computada porque a
carga
em questão é proveniente do interior; isso resulta numa energia
excedente que fica disponível no campo elétrico do capacitor.
Analogamente, o campo elétrico é também conservativo. Assim,
a
energia adquirida pela carga no deslocamento até a armadura
interna
do capacitor é totalmente independente da grandeza do trabalho
realizado pela fonte interna.

TRÊS ETAPAS
Na discussão que se segue a palavra condutor é equivalente ao
vocábulo capacitor, uma vez que um condutor isolado equipara-se a
um
capacitor onde uma das armaduras foi colocada no infinito.
A partir de agora, vamos utilizar os termos processo conservativo
para distinguir o processo do qual resulta uma parcela de energia que
altera a energia do campo elétrico e processo não-conservativo
quando
o resultado da energia, nele envolvida, não causa nenhuma
alteração
na energia acumulada no campo elétrico do capacitor. Por exemplo:
na
analogia que fizemos acima, o trabalho realizado pelo indivíduo no
décimo andar, embora possa ser calculado como o produto da
força
utilizada pela grandeza do deslocamento no nível do décimo
andar, do
ponto de visto do campo gravitacional é nulo, porque não altera
a
grandeza da energia potencial do objeto; é, portanto, um processo
não-
conservativo.
Designaremos de subprocesso a um processo que está inserido em um
outro mais amplo, de modo semelhante à definição matemática
de
conjunto e subconjunto.
Para identificar os subprocessos conservativos e não-conservativos
envolvidos no processo, podemos dividi-lo em três etapas.
Na primeira, a energia do gerador (Wg) é utilizada apenas para
dissociar as cargas elétricas dos átomos gerando elétrons
livres e
íons positivos (Ei energia de ionização) e para dotar essas
partículas de energia cinética (Ec) suficiente a fim de se
chocarem
com os coletores da válvula. É, desse modo, um subprocesso
não-
conservativo. Logo,
(1)
Na segunda etapa, a energia relativa ao deslocamento (Ed) das cargas
para a superfície pode ser considerada como "quantizada". A
energia
potencial de uma carga, no interior de um condutor oco isolado,
independe da posição da carga em relação à superfície e
depende
unicamente de sua grandeza e dos parâmetros geométricos do
condutor.
Isso significa que, de qualquer ponto do interior que a carga
elétrica for deslocada, chegará à superfície com a mesma
energia
potencial (q2/2C). Assim, qualquer elétron que chegue à
superfície
transporta consigo a energia (E = e2/2C), onde e é a carga do
elétron
e C é a capacidade elétrica do condutor. Essa é também a
energia que
um agente externo utilizaria para transportar um elétron de uma a
outra armadura do capacitor. Essa etapa é um processo
conservativo.
Sendo (Ee) a energia adquirida pelo campo elétrico, tem-se que:
(2)
Finalmente, na terceira etapa, constatamos que a energia da carga
não
é dissipada quando chega ao capacitor, como acontece na analogia
que
fizemos com o campo gravitacional, onde a energia cinética do
objeto
que cai, ao se chocar com a superfície da Terra, é convertida
em
calor e em outras formas. A energia da carga é acumulada no campo
elétrico do capacitor (Ee) e pode ser utilizada na produção de
trabalho (W). Assim:
(3)

É CONSTANTE A ENERGIA DO UNIVERSO?
Para analisar esta questão, vamos começar vendo o que diz
Sonntag [3]
no que se refere a um moto-perpétuo e à criação da energia:
(...) a segunda lei da termodinâmica tem sido enunciada como a
impossibilidade da construção de um moto-perpétuo de segunda
espécie.
Um moto-perpétuo de primeira espécie criaria trabalho do nada
ou
criaria massa e energia, violando, portanto, a primeira lei. Um moto-
perpétuo de segunda espécie violaria a segunda lei [funcionaria
em
ciclo sem consumir energia externa], e um moto-perpétuo de
terceira
espécie não teria atrito e assim operaria indefinidamente
porém não
produziria trabalho.
Na analogia que fizemos acima todas as formas de energia se
equivalem. A energia potencial é equivalente à energia
cinética na
queda. A energia externa fornecida ao objeto para elevá-lo ao
décimo
andar é equivalente à que é dissipada quando transformada em
calor,
ao chocar-se com a superfície da Terra. Para que o corpo retorne
à
posição anterior, uma outra quantidade de energia externa,
equivalente àquela que foi transformada no choque, tem que ser
fornecida, a fim de que ele seja reconduzido ao décimo andar do
edifício e possa de lá cair novamente.
No campo eletromagnético, as equações (1) (2) e (3) acima
mostram
quantidades de energia que se equivalem (Fig.1). No entanto, é
correta a afirmação de que a energia necessária para reconduzir
as
cargas elétricas de volta ao interior do condutor isolado, para
serem
novamente dissociadas e aceleradas, tem que ser igual à que ficou
acumulada no campo elétrico?
Suponha que no interior do condutor isolado esteja funcionando uma
fonte de energia. Uma bateria, por exemplo. Após algum tempo de
uso,
a bateria se descarrega. Ela é, então, substituída por uma
outra
carregada e pode ser recarregada, a partir da energia do campo
elétrico, num circuito especialmente construído para esta
finalidade.
Vimos no artigo anterior [1] que a energia acumulada no campo
elétrico é muito maior do que a desprendida pela fonte interna
e
seria suficiente para carregar muitas baterias e não apenas uma.
Após efetuada a carga, a bateria seria novamente utilizada no
interior do capacitor. No deslocamento da fonte, nenhuma reação
é
fornecida pelo campo elétrico, porque as armaduras do capacitor
são
muito próximas e o campo elétrico pode ser considerado
inteiramente
confinado entre elas. As cargas elétricas no interior da bateria
não
são percebidas pelo campo. É como se, na analogia que fizemos
com o
campo gravitacional, pudéssemos retornar o objeto para o décimo
andar, de modo que seu deslocamento não seja percebido pelo campo
gravitacional da Terra. Assim, a energia utilizada no deslocamento da
fonte é também independente da energia do campo elétrico. As
baterias
podem ser alternadas e o excedente da energia pode ser utilizado para
produzir trabalho.

Figura 1: Subprocessos conservativos e não-conservativos.
Observando a figura 1, onde estão indicados os subprocessos
envolvidos, vemos que após a carga da fonte, no campo elétrico,
temos
apenas o deslocamento dela que é um subprocesso
não-conservativo. A
fonte, quando submetida ao processo de recarga, ligada ao capacitor,
participa de um processo conservativo, porque acumula energia
decrementando a energia do campo e, quando colocada no interior do
condutor isolado, participa de um subprocesso não-conservativo,
porque, como foi visto, a partir do interior a energia do campo
não
depende da grandeza do trabalho realizado pela fonte.
Considerando as etapas do processo independentemente, a energia se
conserva em cada uma delas. Entretanto, considerando o processo como
um todo, inclusive a recarga e o deslocamento da fonte, a energia
não
se conserva por causa da falta de simetria da fonte, que participa de
um subprocesso não-conservativo quando carrega o capacitor e de um
subprocesso conservativo quando é, por ele, carregada.
A questão agora é a seguinte: sendo a energia cedida pelo
gerador bem
menor do que a energia relativa ao deslocamento das cargas para a
superfície e havendo a possibilidade de quando a fonte, no
interior,
descarregar-se, ser recarregada a partir da energia do campo
elétrico, não estamos, nesse caso, criando trabalho ou energia
do
nada? Estamos negando a afirmação de Clausius? Ou seja, estamos
criando um moto-perpétuo de primeira espécie? E de segunda?
Um moto-perpétuo de terceira espécie, não. Apesar de tudo as
baterias
têm um desgaste e após certo tempo de uso ficam inutilizadas.
Entretanto, qual seria o rendimento de um sistema como este?

CONCLUSÃO
Parece-me possível obter grandes quantidades de energia a partir
das
propriedades elétricas dos condutores ocos isolados, como um
cilindro
ou uma esfera. Essa energia seria totalmente fria, limpa, barata.
Seria um processo muito mais simples do que retirar energia
utilizável da fusão atômica onde estão envolvidas
altíssimas
temperaturas e não deixaria, como na fissão, resíduos
perigosos,
difíceis de serem descartados.

REFERÊNCIAS BIBLIOGRÁFICAS
[1] FRANÇA, Clístenes Xavier de. É possível amplificar a
energia?
Sinergia, São Paulo, CEFET-SP, volume 03, no 2, jul./dez. 2002, p.
147-153.
[2] PAIS, Abraham. Sutil é o senhor... A ciência e a Vida de
Albert
Einstein. Tradução de Fernando Parente e Viriato Esteves.
Revisão da
tradução César Benjamim. Rio de Janeiro: Nova Fronteira, 1995.
p. 68.
[3] SONNTAG, Richard E., BORGNAKKE, Claus., WYLEN, Gordon J. Van.
Fundamentos da termodinâmica. São Paulo: Edgard Blücher,
1998. p. 149.


Para contato com o autor: clistenesxf@bol.com.br







SUBJECT: Re: [ciencialist] Meteorologia corrigida..:-) (era Astrologia e Mete(o)rologia e top-posting
FROM: "Luiz Ferraz Netto" <leobarretos@uol.com.br>
TO: <ciencialist@yahoogrupos.com.br>
DATE: 09/02/2005 15:40

Homero escreve:
Assunto: [ciencialist] Meteorologia corrigida..:-) (era Astrologia e Mete(o)rologia e top-posting


>Olá Luiz

risos...:-) Deve ser o que resulta de quem se mete a escrever "de ouvido" como eu e as pressas (o pior ]e que meu vacilo ficará eternizado no s títuos desta thread na lista..:-) Falha minha, mas note que não estou só, o Alvaro diz que mesmo meteorologia está incorreto..:-)
Então, leia-se "tempoprevisologia" onde escrevi metereologia e tudo ficará bem..:-)
Um abraço.<

Léo (acho que isso é um 'em baixo'-posting; sei que em alienígena top é 'em cima' (por causa do top-less, muito apreciado pelos bezerros); mas não sei como é 'em baixo');

Homero, apenas uma correção no seu texto, para ficar de acordo com a atual nomenclatura científica dos adoradores da biologia: ninguém escreve "de ouvido" e sim "de orelha"; ouvido caiu de moda ... agora é orelha.

Incrível, quanta sabedoria nessas 'otoridades' que modificam as palavras na biologia, as regras no português, os fatos nos livros históricos etc.
[]'
Léo





--
No virus found in this outgoing message.
Checked by AVG Anti-Virus.
Version: 7.0.300 / Virus Database: 265.8.6 - Release Date: 07/02/2005



SUBJECT: Re: [ciencialist] Sobre batimentos em ondas sonoras
FROM: Franco <dfranco@pop.com.br>
TO: ciencialist@yahoogrupos.com.br
DATE: 09/02/2005 15:45

Poxa, Leo, não tenho aqui, mas já devo ter visto alguma demonstração de
interferência de ondas através do osciloscópio, na faculdade.

Segundo minha mensagem anterior, eu penso que à medida em que intervalo
entre um batimento e outro torna-se menor, significa que as freqüências
das duas ondas estão se igualando. Só gostaria de confirmar isso, pois
não foi o que notei aqui.

Fiquei mais de 5 anos sem tocar qualquer instrumento de cordas. Antes,
quando estudava música, afinava meus instrumentos com um 'diapasão',
esqueci o nome específico, mas este parecia-se com um apito e lhe dava a
freqüência da nota Lá. A partir daí você afinava o restante das cordas.
Na época, eu não tinha noção do que eram batimentos. Dessa vez, resolvi
pegar pela primeira, depois de 5 anos, um violão, baixei na internet um
programinha muito legal que fornece a freqüência de algumas notas
musicais e fui tentar afiná-lo, de acordo com os meus conhecimentos
sobre batimentos.

Acontece que "estranhei" a afinação do mesmo usando o que observei
acima (batimentos muito próximos um do outro) - acho difícil lhe
explicar isso. Não sei, mas também estou levando em conta o tempo que
fiquei sem estudar música, não tenho mais 'ouvido' para isso, além do
que já devo ter perdido uns 10% da minha capacidade auditiva nesta
Paulicéia barulhenta :0).

Franco.

Luiz Ferraz Netto escreveu:

> Oi Franco,
>
> vc tem/consegue osciloscópio, 2 canais?
> []'
>
>



SUBJECT: Re: [ciencialist] que tal, convence?
FROM: "Luiz Ferraz Netto" <leobarretos@uol.com.br>
TO: <ciencialist@yahoogrupos.com.br>
DATE: 09/02/2005 15:51

Alguém quer que eu "não" faça top-posting disso? Prefere que eu repita todo o texto postado e depois escreva 'lá em baixo' : "Putz! quanta bobagem escrita! O cidadão nada entende de mecânica e muito menos de eletrônica!"

aquele abraço,

Léo
=========
e vou cortar tudo logo abaixo..........
=========


Surfando por aqui e ali encontrei isso, penso ser porque o Sr.
Clistene sofreu algumas pressões, mas de qualquer maneira serve
como
material de estudo

Algumas equações não aparecerão portanto aqui o link

http://www.cefetsp.br/edu/sinergia/7p6c.html

SOBRE A AMPLIFICAÇÃO DA ENERGIA

Clístenes Xavier de França
Professor da Área de Projeto e Representações Gráficas para
Edificações
do CEFET-PB/UNED - Cajazeiras



--
No virus found in this outgoing message.
Checked by AVG Anti-Virus.
Version: 7.0.300 / Virus Database: 265.8.6 - Release Date: 07/02/2005



SUBJECT: Re: Media pluviometrica
FROM: "rmtakata" <rmtakata@altavista.net>
TO: ciencialist@yahoogrupos.com.br
DATE: 09/02/2005 15:58


--- Em ciencialist@yahoogrupos.com.br, "Luiz Ferraz Netto"
> vontade, 0,03 litros por segundo (1,6 litros por minuto por metro
> quadrado).

Bem, muitas vezes isso cai em um pancada - chuva de 10 a 15 minutos.
Mas digamos q. tenha peneirado durante uma hora. Se o terreno nao
estah impermeabilizado, entao essa agua percola o solo (uma parte
escorre pela superficie), umedencendo. Se os dias de chuva se seguem,
ainda q. peneirando fino, o solo pode se saturar - e se for em uma
regiao de encosta, pode ocorrer deslizamentos.

Mas em uma cidade como Sao Paulo, o solo estah altamente
impermeabilizado por concreto, asfalto e essas tralhas. A agua escorre
e vai ateh o ponto local mais baixo. Ali acumula-se toda a agua caida
em centenas ou quilometros de metros quadrados. Como a vazao nao da'
conta: corregos, rios e esgotos entupidos com o lixo, assoreados com a
perda de cobertura vegetal e degradacoes do tipo - como resultado
tem-se alagamentos.

Para fins de comparacao, na regiao amazonica, o indice de pluviosidade
eh de cerca de > 3.000 mm (hah regioes em q. cai muito mais agua -
quase 10.000 mm) por ano. Na cidade de Sao Paulo, fica por conta de
1.500 mm ao ano. Em uma hora (ou menos), pode cair a chuva
correspondente a 10% do ano todo. Ou entao poderemos imaginar o q. eh
chapinhar em uma camada de 10 cm de agua pra tudo qto eh lado.
(Poderemos tb pensar na quantidade de energia envolvida - toda essa
agua foi evaporada de algum lugar...)

[]s,

Roberto Takata







SUBJECT: Re: Meteorologia corrigida..:-) (era Astrologia e Mete(o)rologia e top-posting
FROM: "rmtakata" <rmtakata@altavista.net>
TO: ciencialist@yahoogrupos.com.br
DATE: 09/02/2005 16:01


--- Em ciencialist@yahoogrupos.com.br, "Luiz Ferraz Netto"
> Incrível, quanta sabedoria nessas 'otoridades' que modificam as
> palavras na biologia, as regras no português, os fatos nos livros
> históricos etc.

Ei, nao maldigam os biologos. Isso eh coisa dos medicos e sua Nomina
Anatomica. Como nao tem muita novidade descoberta todos os dias em
termos de anatomia, para justificar a edicao de mais livros, soh mesmo
mudando rotula para patela e depois para rotula de novo; amigdala para
tonsila; ouvido para orelha; aparelho disgestivo para digestorio;
trompa de Falopio para tuba uterina e essas alteracoes cosmeticas.

[]s,

Roberto Takata





SUBJECT: Re: [ciencialist] Sobre batimentos em ondas sonoras
FROM: "Luiz Ferraz Netto" <leobarretos@uol.com.br>
TO: <ciencialist@yahoogrupos.com.br>
DATE: 09/02/2005 16:06

Oi Franco,

sim entendi seu texto anterior. Apenas perguntei sobre o osciloscópio porque percebi que vc queria "ver" como a coisa funciona e, para isso, nada melhor que um osciloscópio traço duplo.

Ainda no mesmo espírito, ver para crer, pergunto: vc tem algum software para gerar o gráfico de funções? Eu uso o "Equation Grapher - Math.exe". De graça e é uma beleza.

Se baixar o dito cujo de algum terreiro (pode ser perguntando ao Sto Google), coloque lá as funções y1=2.sen(10.x) e y2 = 2.sen(11.x). E observe ais gráficos e suas superposições (interferência) --- ai vc enxerga o batimento.
Depois vá diminuindo a diferença |f2 - f1| de |11 - 10| para |11 - 10,5|, |11 - 10,7| .... e enxergando o efeito do "bit zero".

Com o V.F.O. de um transmissor (na fase de carregar a antena) e o receptor do rádio amador vc tb pode 'escutar' isso; já escutou.

O detetor de metais faz algo parecido; o meu aqui em casa é supimpa.

Depois, bem depois, de uma olhada na lei das cordas de Mersene, ou de Lagrange, na experiência de Melde.

aquele abraço,
===========================
Luiz Ferraz Netto [Léo]
leobarretos@uol.com.br
http://www.feiradeciencias.com.br
===========================
-----Mensagem Original-----
De: "Franco" <dfranco@pop.com.br>
Para: <ciencialist@yahoogrupos.com.br>
Enviada em: quarta-feira, 9 de fevereiro de 2005 15:45
Assunto: Re: [ciencialist] Sobre batimentos em ondas sonoras



Poxa, Leo, não tenho aqui, mas já devo ter visto alguma demonstração de
interferência de ondas através do osciloscópio, na faculdade.

Segundo minha mensagem anterior, eu penso que à medida em que intervalo
entre um batimento e outro torna-se menor, significa que as freqüências
das duas ondas estão se igualando. Só gostaria de confirmar isso, pois
não foi o que notei aqui.

Fiquei mais de 5 anos sem tocar qualquer instrumento de cordas. Antes,
quando estudava música, afinava meus instrumentos com um 'diapasão',
esqueci o nome específico, mas este parecia-se com um apito e lhe dava a
freqüência da nota Lá. A partir daí você afinava o restante das cordas.
Na época, eu não tinha noção do que eram batimentos. Dessa vez, resolvi
pegar pela primeira, depois de 5 anos, um violão, baixei na internet um
programinha muito legal que fornece a freqüência de algumas notas
musicais e fui tentar afiná-lo, de acordo com os meus conhecimentos
sobre batimentos.

Acontece que "estranhei" a afinação do mesmo usando o que observei
acima (batimentos muito próximos um do outro) - acho difícil lhe
explicar isso. Não sei, mas também estou levando em conta o tempo que
fiquei sem estudar música, não tenho mais 'ouvido' para isso, além do
que já devo ter perdido uns 10% da minha capacidade auditiva nesta
Paulicéia barulhenta :0).

Franco.

Luiz Ferraz Netto escreveu:

> Oi Franco,
>
> vc tem/consegue osciloscópio, 2 canais?
> []'
>
>



##### ##### #####

Para saber mais visite
http://www.ciencialist.hpg.ig.com.br


##### ##### ##### #####
Links do Yahoo! Grupos










--
No virus found in this incoming message.
Checked by AVG Anti-Virus.
Version: 7.0.300 / Virus Database: 265.8.6 - Release Date: 07/02/2005




--
No virus found in this outgoing message.
Checked by AVG Anti-Virus.
Version: 7.0.300 / Virus Database: 265.8.6 - Release Date: 07/02/2005



SUBJECT: Meteorologia corrigida..:-) (era Astrologia e Mete(o)rologia e top-posting
FROM: "Oraculo" <oraculo@atibaia.com.br>
TO: <ciencialist@yahoogrupos.com.br>
DATE: 09/02/2005 16:06

Olá Leo

"Leo:(acho que isso é um 'em baixo'-posting; sei que em alienígena top é 'em cima' (por causa do top-less, muito apreciado pelos bezerros); mas não sei como é 'em baixo');"

Talvez seja "bottom-posting", embora o termo não tenha muito uso, já que é o recomendado pela netiqueta (outro neologismo, etiqueta da internet..:-) "Top-posting" é que incomoda alguns usuários na leitura.

E não se toca mais "de ouvido", mas apenas de orelha??? Tô ficando velho..:-)

Um abraço.

Homero




----- Original Message -----
From: Luiz Ferraz Netto
To: ciencialist@yahoogrupos.com.br
Sent: Wednesday, February 09, 2005 3:40 PM
Subject: Re: [ciencialist] Meteorologia corrigida..:-) (era Astrologia e Mete(o)rologia e top-posting


Homero escreve:
Assunto: [ciencialist] Meteorologia corrigida..:-) (era Astrologia e Mete(o)rologia e top-posting


>Olá Luiz

risos...:-) Deve ser o que resulta de quem se mete a escrever "de ouvido" como eu e as pressas (o pior ]e que meu vacilo ficará eternizado no s títuos desta thread na lista..:-) Falha minha, mas note que não estou só, o Alvaro diz que mesmo meteorologia está incorreto..:-)
Então, leia-se "tempoprevisologia" onde escrevi metereologia e tudo ficará bem..:-)
Um abraço.<

Léo (acho que isso é um 'em baixo'-posting; sei que em alienígena top é 'em cima' (por causa do top-less, muito apreciado pelos bezerros); mas não sei como é 'em baixo');

Homero, apenas uma correção no seu texto, para ficar de acordo com a atual nomenclatura científica dos adoradores da biologia: ninguém escreve "de ouvido" e sim "de orelha"; ouvido caiu de moda ... agora é orelha.

Incrível, quanta sabedoria nessas 'otoridades' que modificam as palavras na biologia, as regras no português, os fatos nos livros históricos etc.
[]'
Léo





--
No virus found in this outgoing message.
Checked by AVG Anti-Virus.
Version: 7.0.300 / Virus Database: 265.8.6 - Release Date: 07/02/2005



##### ##### #####

Para saber mais visite
http://www.ciencialist.hpg.ig.com.br


##### ##### ##### #####


Yahoo! Grupos, um serviço oferecido por:

São Paulo Rio de Janeiro Curitiba Porto Alegre Belo Horizonte Brasília




------------------------------------------------------------------------------
Links do Yahoo! Grupos

a.. Para visitar o site do seu grupo na web, acesse:
http://br.groups.yahoo.com/group/ciencialist/

b.. Para sair deste grupo, envie um e-mail para:
ciencialist-unsubscribe@yahoogrupos.com.br

c.. O uso que você faz do Yahoo! Grupos está sujeito aos Termos do Serviço do Yahoo!.



[As partes desta mensagem que não continham texto foram removidas]



SUBJECT: Re: Sobre batimentos em ondas sonoras
FROM: João R. Bonomo <joao_bonomo@yahoo.com.br>
TO: ciencialist@yahoogrupos.com.br
DATE: 09/02/2005 16:09


Olá Franco,

Imagine que a corda do violão está frouxa ( percebe-se de ouvido que
o tom da corda é mais grave do que o tom do diapasão ). A medida que
a corda vai sendo esticada, aparece uma freqüência de batimento que
inicialmente é alta ( intervalos curtos ) e gradativamente vai se
tornando baixa ( intervalos longos ) até ficar inaudível. Neste ponto
podemos considerar que as frequências da corda e do diapasão são
iguais pois estão produzindo uma diferença de frequência (
batimento ) tão baixa que não pode ser ouvida.
Se você continuar a esticar a corda vai novamente começar a ouvir um
batimento, agora iniciando com uma frequência baixa ( intervalos
longos ) e aumentando gradativamente a frequência ( intervalos
ficando cada vez mais curtos ) até tornar-se inaudível novamente.
Agora percebe-se de ouvido que o tom emitido pela corda é mais agudo
que o tom emitido pelo diapasão.
Com alguma prática fica fácil efetuar esta operação de afinação.

Espero ter ajudado.

[]`s
JRB



--- Em ciencialist@yahoogrupos.com.br, "Luiz Ferraz Netto"
<leobarretos@u...> escreveu
> Oi Franco,
>
> vc tem/consegue osciloscópio, 2 canais?
> []'
> ===========================
> Luiz Ferraz Netto [Léo]
> leobarretos@u...
> http://www.feiradeciencias.com.br
> ===========================
> -----Mensagem Original-----
> De: "Franco" <dfranco@p...>
> Para: "Ciencialist" <ciencialist@yahoogrupos.com.br>
> Enviada em: terça-feira, 8 de fevereiro de 2005 23:46
> Assunto: [ciencialist] Sobre batimentos em ondas sonoras
>
>
>
> Gente, o princípio para afinar, digamos, uma corda de violão, faz-
se
> comparando a freqüência dessa corda com a emitida por um diapasão,
por
> exemplo. Quando notam-se batimentos, então, sei que as freqüências
estão
> bem próximas. Mas ainda tenho uma dúvida. À medida que
vamos "esticando"
> a corda (fazendo-a tocar numa freqüênca mais alta), o intervalo
entre um
> batimento e outro vai diminuindo até desaparecer.
> Logo, a partir de que ponto deve-se "afinar" a corda do violão -
supondo
> aqui que estamos "esticando" a corda: antes de os batimentos serem
> percebidos (quando o intervalo entre um e outro é longo), ou a
partir do
> ponto em que tal intervalo entre um batimento e outro seja tão
curto que
> eles desaparecem?
>
> Franco.
>
>
>
>
> ##### ##### #####
>
> Para saber mais visite
> http://www.ciencialist.hpg.ig.com.br
>
>
> ##### ##### ##### #####
> Links do Yahoo! Grupos
>
>
>
>
>
>
>
>
>
>
> --
> No virus found in this incoming message.
> Checked by AVG Anti-Virus.
> Version: 7.0.300 / Virus Database: 265.8.6 - Release Date:
07/02/2005
>
>
>
>
> --
> No virus found in this outgoing message.
> Checked by AVG Anti-Virus.
> Version: 7.0.300 / Virus Database: 265.8.6 - Release Date:
07/02/2005





SUBJECT: Tonsila (era Meteorologia corrigida..:-) ( que era Astrologia e Mete(o)rologia e top-posting
FROM: "Oraculo" <oraculo@atibaia.com.br>
TO: <ciencialist@yahoogrupos.com.br>
DATE: 09/02/2005 16:09

Olá Takata

Takata: amigdala para tonsila;

Taí, um belo nome para uma filha..:-) Tonsila de Orleans e Bragança, fica muito nobre e elegante..:-)

Homero

PS: Acho que esta vai ganhar o premio de maior off-topic do ano, me desculpa Brudna..:-)

----- Original Message -----
From: rmtakata
To: ciencialist@yahoogrupos.com.br
Sent: Wednesday, February 09, 2005 4:01 PM
Subject: [ciencialist] Re: Meteorologia corrigida..:-) (era Astrologia e Mete(o)rologia e top-posting



--- Em ciencialist@yahoogrupos.com.br, "Luiz Ferraz Netto"
> Incrível, quanta sabedoria nessas 'otoridades' que modificam as
> palavras na biologia, as regras no português, os fatos nos livros
> históricos etc.

Ei, nao maldigam os biologos. Isso eh coisa dos medicos e sua Nomina
Anatomica. Como nao tem muita novidade descoberta todos os dias em
termos de anatomia, para justificar a edicao de mais livros, soh mesmo
mudando rotula para patela e depois para rotula de novo; amigdala para
tonsila; ouvido para orelha; aparelho disgestivo para digestorio;
trompa de Falopio para tuba uterina e essas alteracoes cosmeticas.

[]s,

Roberto Takata





##### ##### #####

Para saber mais visite
http://www.ciencialist.hpg.ig.com.br


##### ##### ##### #####


Yahoo! Grupos, um serviço oferecido por:







------------------------------------------------------------------------------
Links do Yahoo! Grupos

a.. Para visitar o site do seu grupo na web, acesse:
http://br.groups.yahoo.com/group/ciencialist/

b.. Para sair deste grupo, envie um e-mail para:
ciencialist-unsubscribe@yahoogrupos.com.br

c.. O uso que você faz do Yahoo! Grupos está sujeito aos Termos do Serviço do Yahoo!.



[As partes desta mensagem que não continham texto foram removidas]



SUBJECT: pirilampos...
FROM: "E m i l i a n o C h e m e l l o" <chemelloe@yahoo.com.br>
TO: <ciencialist@yahoogrupos.com.br>
DATE: 09/02/2005 16:15

alguém contou ? :-)


[ ] 's do Emiliano Chemello
emiliano@quimica.net
http://www.quimica.net/emiliano
http://www.ucs.br/ccet/defq/naeq
---
Contato Naeq:
Nome: Judith Izabel Azevedo de Paula
Email: judith_izabel@yahoo.com.br
Telefone: Pirilampo
Mensagem: Senhor responsável:

gostaria de saber quantas cores de pirilampos existem no Brasil.
Antecipadamente, agrade o a aten o a mim dispensada.

Judith Izabel Azevedo de Paula




SUBJECT: Few in Brazil accept scientific view of human evolution
FROM: "L.E.R.de Carvalho" <lecarvalho@infolink.com.br>
TO: ciencialist@yahoogrupos.com.br
DATE: 09/02/2005 16:31


http://www.scidev.net/News/index.cfm?fuseaction=readNews&itemid=1885&language=1

Few in Brazil accept scientific view of human evolution
*

Luisa Massarani
28 January 2005
Source: SciDev.Net

[RIO DE JANEIRO] The theory that humans have evolved over millions of years
independent of any 'divine' influence is not widely accepted in Brazil,
according a survey by the country's main public opinion analyst, IBOPE.

Scientists believe modern humans are the result of an evolutionary process
that involves random mutations in our genetic material, some of which
confer advantages and are therefore perpetuated down generations.

Only nine per cent of the Brazilians interviewed said they accept that
scientific consensus. More than half (54 per cent) said humans developed
over millions of years, but that a God planned and controlled the process.
And almost one-third (31 per cent) of interviewees believe a God created
humans 10,000 years ago, and that we have not changed since then.

The survey was commissioned by Época magazine, one of Brazil's top three
general magazines, to stimulate debate about a controversial decision made
last year by the governor of Rio de Janeiro, Rosinha Mateus, to introduce
teaching of creationism in schools.

Creationists believe in the literal interpretation of the Bible's account
of creation of the universe and life on earth.

A significant number (89 per cent) of those interviewed by IBOPE said
creationism should be taught in schools, with 75 per cent of the
interviewees going further, saying creationism should replace evolutionary
theory. Many of these respondents were those who stated that humans have
changed over time, but that a God controls the process rather than it being
random, as scientists contend.

Ildeu de Castro Moreira, head of science communication at Brazil's Ministry
of Science and Technology, claims that some of the survey's questions were
confusing, leading to a bias in the answers.

For example, one question asked whether respondents thought humans had been
developing over millions of years, but did not explain clearly what
'developing' meant.

Moreira also says that complexity of the issue means that the results of
the survey cannot be simply explained as a "war between religious and
scientific beliefs". One factor at play, he says, is the poor quality of
science teaching in Brazil.

He says the government and civil society organisations should focus
education efforts on young people, who have open minds.

Ennio Candotti, president of the Brazilian Society for the Advancement of
Science, agrees that the survey highlights the poor quality of science
education, adding that a widespread creationist viewpoint is not limited to
Brazil, but is also a feature of other countries including the United States.

"It is task of the schools to distinguish different fields, keeping areas
such as physics, mathematics and biology separate from moral principles and
cultural values, yet teaching them together in a harmonic way", he says.

Candotti suggests creationism could be taught within the framework of
theories that seek to explain the formation and the evolution of the
universe and the world.

The survey showed education level had only a slight influence on people's
beliefs. Only ten per cent of those with a university degree believed in
evolution unguided by a God, whereas among those without a university
education, the figure was six per cent.

The article published by Época provoked protests from creationists, who
said the survey's results were reported in a way that was biased in favour
of evolution. The Brazilian Creationist Society collected on its website
comments made in response to the Época article by representatives of the
main creationist organisations.

According to one of the comments, by Christiano P. da Silva Neto, president
of the Brazilian Association for Research on Creation, teaching creationism
in schools does not exclude evolutionary theory: "Considering that neither
theory of the origin of life can be proved scientifically, both should be
taught in schools to allow students to consider the arguments for each and
decide what makes sense for them and which they will consider as the true
explanation of our origins."

The IBOPE survey, held on 9-15 December, included 2,002 people, in a
representative sample of the entire Brazilian population over 16 years old.
According to IBOPE, the error of margin in the survey's results in plus or
minus 2.2 per cent.

<http://revistaepoca.globo.com/Epoca/0,6993,EPT884203-1664-1,00.html>E no
princípio era o que mesmo? (Story published by Época, in Portuguese)

<http://revistaepoca.globo.com/Epoca/0%2C6993%2CEPT731549-1664-1%2C00.html>Rosinha
contra Darwin (In Portuguese)


[As partes desta mensagem que não continham texto foram removidas]



SUBJECT: Re: Tonsila (era Meteorologia corrigida..:-) ( que era Astrologia e Mete(o)rolog
FROM: "rmtakata" <rmtakata@altavista.net>
TO: ciencialist@yahoogrupos.com.br
DATE: 09/02/2005 16:58


--- Em ciencialist@yahoogrupos.com.br, "Oraculo" <oraculo@a...>
> Taí, um belo nome para uma filha..:-) Tonsila de Orleans e Bragança,
> fica muito nobre e elegante..:-)

Se tem Rainha Padme Amidala, por q. nao Tonsila Bragança?

http://www.starwars.com/databank/character/amidala/index.html

[]s,

Roberto Takata





SUBJECT: Re: Sobre batimentos em ondas sonoras
FROM: João R. Bonomo <joao_bonomo@yahoo.com.br>
TO: ciencialist@yahoogrupos.com.br
DATE: 09/02/2005 17:00


--- Em ciencialist@yahoogrupos.com.br, Franco <dfranco@p...> escreveu
>
> Segundo minha mensagem anterior, eu penso que à medida em que
intervalo
> entre um batimento e outro torna-se menor, significa que as
freqüências
> das duas ondas estão se igualando. Só gostaria de confirmar isso,
pois
> não foi o que notei aqui.
>
Olá Franco,

É exatamente ao contrário: "à medida em que os intervalos tornam-se
MAIORES, significa que as freqüências estão se igualando".

[]`s
JRB





SUBJECT: Vestibular UCS Verão 2005
FROM: "E m i l i a n o C h e m e l l o" <chemelloe@yahoo.com.br>
TO: <ciencialist@yahoogrupos.com.br>, <quimica-qaw@yahoogrupos.com.br>, <quimica.pe@grupos.com.br>, <tchequimica@yahoogrupos.com.br>, <naeq-ucs@yahoogrupos.com.br>, <quimica@grupos.com.br>
DATE: 09/02/2005 17:38

Caros Amigos,

Está na minha página a resolução da prova de química do concurso vestibular UCS Verão 2005.

Para acessar o arquivo, utilize o seguinte link:
http://www.quimica.net/emiliano/vestibular/ucs/ucs2005_verao_resolucao.pdf

Aguardo considerações.

[ ] 's do Emiliano Chemello
emiliano@quimica.net
http://www.quimica.net/emiliano
http://www.ucs.br/ccet/defq/naeq

" Rien ne se perd, rien ne se crée,
tout se transforme."

Antoine Laurent de Lavoisier (químico francês, 1743 - 1794)


[As partes desta mensagem que não continham texto foram removidas]



SUBJECT: Fw: Teste da gasolina
FROM: "Luiz Ferraz Netto" <leobarretos@uol.com.br>
TO: "ciencialist" <ciencialist@yahoogrupos.com.br>
DATE: 09/02/2005 19:31

Olá Químicos,

podem dar uma mãozinho para esse menino da 8a série?

agradeço,
===========================
Luiz Ferraz Netto [Léo]
leobarretos@uol.com.br
http://www.feiradeciencias.com.br
===========================

-----Mensagem Original-----
De: Eduardo Leão
Para: leobarretos@uol.com.br ; daivemmeue-mailleobarretos@uol.com.br
Enviada em: quarta-feira, 9 de fevereiro de 2005 16:00
Assunto: Teste da gasolina

Me chamo Eduardo (8a. série). Gostaria de fazer uma experiência na feira de ciências de minha escola na qual eu pudesse reproduzir um teste de qualidade da gasolina.
Qual tipo de reação química poderia utilizar?
Quais substâncias químicas (reagentes) reagem com a gasolina possibilitando sua identificação de qualidade (presença de outro solvente não permitido por lei ou muito teor de alcool? Pensei em um teste semelhante aos Kits de teste de piscina para ver a qualidade da água.
Será possível?

Agradeço atenção,
Eduardo




--
No virus found in this outgoing message.
Checked by AVG Anti-Virus.
Version: 7.0.300 / Virus Database: 265.8.6 - Release Date: 07/02/2005



SUBJECT: Re: [ciencialist] Re: Zodiaco
FROM: "Sergio M. M. Taborda" <sergiotaborda@terra.com.br>
TO: ciencialist@yahoogrupos.com.br
DATE: 09/02/2005 21:43

Oraculo wrote:

> Olá Taborda
>
> "Taborda:Aquilo que os ilusionistas fazem em espetaculos é leitura
> fria, sim e
> dai ? O que isso tem a ver com astrologia ?
> Vc vai dizer que a maioria dos astrologos fazem leitura fria. Mas como
> se mede a leitura fria, como eu sei que ele esta fazendo leitura fria ?
> e pior , como eu provo ? Se vc sabe responder a isso, pq vc não prova
> que todos os astrologos fazem leitura fria e acaba com o problema?
> Faça-se uma licença para astrologos que a recebe quem poder passar no
> teste de leitura fria - seja lá o que isso for , supondo que existe tal
> teste e ele é 100% infalivel."
>
> Os ilusionistas, como astrólogos, usam diversos mecanismos e
> ferramentas, não apenas a leitura fria. Generalizações e a natural
> tendencia da mente humana de contar acertos e ignorar erros também é
> um ponto importante nas duas artes..:-) Por isso não diria que a
> maioria dos astrólogos faz leitura fria, embora os que fazem consultas
> presenciais possam usar a técnica (mesmo sem saber).

Espere ai. Vc está dizendo que a maioria dos astrologos não usa leitura
fria. Então o que a maioria dos astrologos usa ?

>
> Faça um teste, crie um mapa astral com dados falsos e apresente a um
> cliente como se verdadeiro fosse (não pode ser na presença do mesmo,
> já que sinais imperceptiveis poderiam indicar a ele sua pouca
> confiança no mapa, tem de ser em duplo cego) e veja o que ele acha da
> taxa de acertos.

Faça isso com um relatorio médico, apresente-o ao paciente e veja o que
ele acha. Faça um relatorio psiquiátrico e apresente-o ao loco
analizado e veja o que ele acha.
Convenhamos, que essa não é a forma de testar se o relatorio médico é
correcto. Da mesma forma, não é a forma de saber se o mapa foi bem
intrepretado.

> Tenho certeza de que ele irá achar grande semelhança, apesar de a data
> ser totalmetne incorreta..:-)

Da mesma que um paciente de qq medico. (desde que a mentira não seja
demasiado evidente)

> A questão básica permanece, cálculos de posição de astros podem
> informar aspectos da personalidade de uma pessoa, com base em sua data
> de nascimento. Isso, se fosse real, poderia ser demonstrado e
> verificado. Não pode, e isso invalida a alegação astrologica.

Mas pq vc diz que não pode ? O que impossibilita esse estudo ?


Sérgio Taborda


--
No virus found in this outgoing message.
Checked by AVG Anti-Virus.
Version: 7.0.300 / Virus Database: 265.8.6 - Release Date: 07-02-2005



SUBJECT: Re: [ciencialist] Astrologia e Metereologia e top-posting (era : Zodiaco)
FROM: "Sergio M. M. Taborda" <sergiotaborda@terra.com.br>
TO: ciencialist@yahoogrupos.com.br
DATE: 09/02/2005 21:52

Oraculo wrote:

> Olá Taborda
>
> Taborda: E eu já pedi para não fazer top-posting. (Isso é outra coisa
> que vc
> deveria ser mais critico, pq está violando as regras da boa conduta na
> internet.Ou vc tb acha que isso não funciona?)"
>
> Sinto, mas não faço top-posting, nunca. Se algum trecho pareceu assim
> me desculpe, mas foi apenas impressão. Todos os meus comentários se
> seguem ao trecho relacionado. Sendo que tomo o cuidado de sempre
> apresenta-lo depois do trecho em questão, exatamente como fiz acima.
>
> E eu me preocupo e critico médicos que violam crianças (ou não
> médicos, tanto faz) e com diversas das proposições que fez (embora
> discorde de algumas, principalmente sobre eleições inúteis..:-), mas
> isso não me impede de também criticar a astrologia.
>
> E a metereologia, como já discutido, é bem diferente da astrologia, já
> que tem a cada vez mais se aperfeiçoado e atingido um bom grau de
> exatidão, embora para periodos curtos de tempo (até 8 dias). Mas é
> cada vez melhor em prever tempestades, furacões, etc, colocando
> pessoas a salvo em caso de perigo (o sistema de alerta de tornados
> americanos é um bom exemplo).

Acho que o pessoal que sofreu com o Tsunami não concorda consigo. Da
mesma forma que o pessoal que sobre com furações os anos passados.
Os meteorologistas não sabem prever este tipo de fenomeno. E nem
poderiam dado o entendimento do clima como um todo. Tem evoluido, sim,
mas o que tem evoluido ? O modelo explicativo.
Apenas o modelo explicativo. Depois que acontece, sabemos explicar, mas
antes não. Certos fenomos atmosfericos eram até desconhecidos até ha bem
pouco tempo, não fosse o estudo sério motivado pelos estudo de
avistamentos OVNI não conheceriamos o Raios Globulares e outros
fenomenos interessantes.
Onde á fumo ha fogo. Não me verá defender a astrologia como uam
ciencia, pq não é . Da mesma forma que não verá defender a psicologia
como uma. Ha muito de arte.
Mas até onde ue posso ver o mito não é explicação suficiente. Nenhum
mito nasce do nada, onde á fumo ha fogo. Falta apenas saber qual é o
fogo. E nãop chegaremos lá se os criticos se esquivam do método
cientifico. Se os criticos , supostamente amantes da ciencia não tiram
as duvidas , quem vai tirar ?

Sértgio Taborda

P.S Quanto ao top-posting, eu continuo receendo seus emails com todo o
texto depois do fim da sua assinatura. Coisa que não acontece com os
outros membros, por isso não é problema do meu leitor, por isso
desconfiuo que seja do seu, já que vc está dizendo que apaga todo o
texto depois da sua assinatura.



--
No virus found in this outgoing message.
Checked by AVG Anti-Virus.
Version: 7.0.300 / Virus Database: 265.8.6 - Release Date: 07-02-2005



SUBJECT: Re: [ciencialist] Astrologia e Metereologia e top-posting (era : Zodiaco)
FROM: "Alvaro Augusto \(E\)" <alvaro@electraenergy.com.br>
TO: <ciencialist@yahoogrupos.com.br>
DATE: 10/02/2005 00:30

----- Original Message -----
From: Sergio M. M. Taborda
To: ciencialist@yahoogrupos.com.br
Sent: Wednesday, February 09, 2005 9:52 PM
Subject: Re: [ciencialist] Astrologia e Metereologia e top-posting (era :
Zodiaco)


Oraculo wrote:

> > Olá Taborda
>> >
>> Taborda: E eu já pedi para não fazer top-posting. (Isso é outra coisa
>> que vc
>> deveria ser mais critico, pq está violando as regras da boa conduta na
>> internet.Ou vc tb acha que isso não funciona?)"
>>
>> Sinto, mas não faço top-posting, nunca. Se algum trecho pareceu assim
>> me desculpe, mas foi apenas impressão. Todos os meus comentários se
>> seguem ao trecho relacionado. Sendo que tomo o cuidado de sempre
>> apresenta-lo depois do trecho em questão, exatamente como fiz acima.
>>
>> E eu me preocupo e critico médicos que violam crianças (ou não
>> médicos, tanto faz) e com diversas das proposições que fez (embora
>> discorde de algumas, principalmente sobre eleições inúteis..:-), mas
>> isso não me impede de também criticar a astrologia.
>>
>> E a metereologia, como já discutido, é bem diferente da astrologia, já
>> que tem a cada vez mais se aperfeiçoado e atingido um bom grau de
>> exatidão, embora para periodos curtos de tempo (até 8 dias). Mas é
>> cada vez melhor em prever tempestades, furacões, etc, colocando
>> pessoas a salvo em caso de perigo (o sistema de alerta de tornados
>> americanos é um bom exemplo).
>
>Acho que o pessoal que sofreu com o Tsunami não concorda consigo. Da
>mesma forma que o pessoal que sobre com furações os anos passados.
>Os meteorologistas não sabem prever este tipo de fenomeno. E nem
>poderiam dado o entendimento do clima como um todo. Tem evoluido, sim,
>mas o que tem evoluido ? O modelo explicativo.
>Apenas o modelo explicativo. Depois que acontece, sabemos explicar, mas
>antes não. Certos fenomos atmosfericos eram até desconhecidos até ha bem
>pouco tempo, não fosse o estudo sério motivado pelos estudo de
>avistamentos OVNI não conheceriamos o Raios Globulares e outros
>fenomenos interessantes.
>Onde á fumo ha fogo. Não me verá defender a astrologia como uam
>ciencia, pq não é . Da mesma forma que não verá defender a psicologia
>como uma. Ha muito de arte.
>Mas até onde ue posso ver o mito não é explicação suficiente. Nenhum
>mito nasce do nada, onde á fumo ha fogo. Falta apenas saber qual é o
>fogo. E nãop chegaremos lá se os criticos se esquivam do método
>cientifico. Se os criticos , supostamente amantes da ciencia não tiram
>as duvidas , quem vai tirar ?
>
>Sértgio Taborda
>
>P.S Quanto ao top-posting, eu continuo receendo seus emails com todo o
>texto depois do fim da sua assinatura. Coisa que não acontece com os
>outros membros, por isso não é problema do meu leitor, por isso
>desconfiuo que seja do seu, já que vc está dizendo que apaga todo o
>texto depois da sua assinatura.

Um tsunami dificilmente seria previsto por meteorologistas, pois, tendo sido
provocado por um abalo sísmico, está mais no campo da sismologia...

Quando ao "top posting", francamente, acho isso um saco. Trata-se de coisa
dos primórdios da internet, quando todos as mensagens eram em modo texto. O
meu leitor de e-mails, por exemplo, não coloca essas porcarias de ">"
automaticamente. Então, tenho que abrir a mensagem em um editor de textos,
colocar todos os ">" e depois colar na resposta. Tudo isso para não ferir
uma regra que algum belezinha criou!

Que problema há em ler as contribuições na ordem inversa, afinal?

[ ]s

Alvaro Augusto
um "top poster", doravante



SUBJECT: Regressei do...Zodíaco, isto é...
FROM: Maria Natália <grasdic@hotmail.com>
TO: ciencialist@yahoogrupos.com.br
DATE: 10/02/2005 02:08


Quando parti para trabalho no céu e no fundo do mar andava zodíaco no
ar por aqui. Regresso com...PUF! zodíaco.
No zodíaco apenas me preocupei com Gémeos porque me permitiram
encontrar Saturno. Andei mais com o Caçador seus Cães e a Lebre sempre
a escapar, o Touro, as Híades e as meninas da família Pleiades
habituais na farra nocturna. A Ursa Maior está este mês na sua posição
óptima para estudo.
Mas a "estrela" da noite foi sem dúvida o cometa Machholz (C/2004
Q2). Para vos fazer raivinhas direi que se tornou um objecto
circumpolar e que vai passar por "cima da Estrela polar" lá para 25 de
Fevereiro. Se o local for escuro se vê um borrão esbranquiçado como se
fosse enxame globular. Mas até um binóculo de tatro serve para se
observar que não é como o desenharam na escola
Ah! Antes que me esqueça: está na altura ideal para se ver a Luz
Zodiacal (e que é causada pela reflexão da luz solar no pó dos
meteoros no plano do sistema solar). Basta numa noite sem lua, 1 ou 2
h depois do por do sol e olhar ao longo da eclíptica no oeste perto do
horizonte. Foi lindo e fiz fotos interessantes.
Ainda para este mês para além do cometa teremos a 27 Jupíter* perto da
Lua.
Quanto ao mar estava impecavelmente gelado mas muito transparente
permitindo ver até as moreias "à lareira" em suas casas e com muito
respeitinho.
Temperatura do ar em média 8 a 11 ºC
Foi um carnaval fantástico pois já há 3 anos que não fazia férias de
mais de 2 dias fora de Lisboa.
No meu carnaval dancei com as melhores estrelas. E vos desamparei um
pouco. Prometo massacrar-vos a partir de sábado.
Se quiserem tentar algo parecido ide a skymaps.com, peçam autorização
ao Kym Thalassoudis e façam download do mapa do hemisfério Sul. Fujam
das grandes cidades para verem algum céu enquanto ele existe.
Continuação de boas férias da Natália
Maria Natália
* Júpiter estava mas quem está in é o Saturno mais Titã






SUBJECT: O Sedna e a astrologia
FROM: Manuel Bulcão <manuelbulcao@uol.com.br>
TO: ciencialist@yahoogrupos.com.br
DATE: 10/02/2005 02:23


Oi,

Gostaria de saber o que os astrólogos têm a dizer a respeito desse
novo planeta do Sistema Solar recentemente descoberto, o Sedna.

[]s
Manuel Bulcão





SUBJECT: Teste da gasolina
FROM: "L.E.R.de Carvalho" <lecarvalho@infolink.com.br>
TO: ciencialist@yahoogrupos.com.br
DATE: 10/02/2005 03:02


>Enviada em: quarta-feira, 9 de fevereiro de 2005 16:00
>Assunto: Teste da gasolina
>
>Me chamo Eduardo (8a. série). Gostaria de fazer uma experiência na feira
>de ciências de minha escola na qual eu pudesse reproduzir um teste de
>qualidade da gasolina.
>Qual tipo de reação química poderia utilizar?
>Quais substâncias químicas (reagentes) reagem com a gasolina
>possibilitando sua identificação de qualidade (presença de outro solvente
>não permitido por lei ou muito teor de alcool? Pensei em um teste
>semelhante aos Kits de teste de piscina para ver a qualidade da água.
>Será possível?
>Agradeço atenção,
>Eduardo


São muitas as respostas.

Uma delas é para medir o teor de álcool adicionado na gasolina.
Isso é feito nos Postos, acho, quando o consumidor solicita.
Basta ele ir lá e olhar.

Outra delas é para medir, me parece, o teor de água no alcool combustível.

No caso da ESSO, ela adiciona um marcador, uma substancia secreta, e ela
vai lá, nos Postos, periodicamente, pra coletar amostras e analisar, no
próprio Posto, se essa substancia está presente na gasolina, nos teores que
ela adicionou.
Com isso ela protege o consumidor indiretamente.
Primeiro, ela trata de proteger a si própria, avaliando se o Posto andou
comprando gasolina de outra marca, gasolina fraudada etc.

No caso da Shell deve ser semelhante, tanto assim que ela propagandeia o
termo "DNA da Shell". Suponho que seja uma substancia "marcador", similar
ao caso da ESSO.

Outra possibilidade é a adição de solventes na gasolina.
Aí é feito porque o solvente é mais barato.
Mas principalmente, suponho, porque o solvente não está onerado pelos
impostos incidentes em cima da gasolina.

Umas frandes prejudicam o rendimento do motor.
Outras afetam o funcionamento.

L.E.

[As partes desta mensagem que não continham texto foram removidas]



SUBJECT: Re: [ciencialist] Teste da gasolina
FROM: "Luiz Ferraz Netto" <leobarretos@uol.com.br>
TO: <ciencialist@yahoogrupos.com.br>
DATE: 10/02/2005 03:45

Com essa já são duas colaborações enviadas ao Eduardo da 8a série.
agradeço,
[]'
===========================
Luiz Ferraz Netto [Léo]
leobarretos@uol.com.br
http://www.feiradeciencias.com.br
===========================
-----Mensagem Original-----
De: "L.E.R.de Carvalho" <lecarvalho@infolink.com.br>
Para: <ciencialist@yahoogrupos.com.br>
Enviada em: quinta-feira, 10 de fevereiro de 2005 03:02
Assunto: [ciencialist] Teste da gasolina




>Enviada em: quarta-feira, 9 de fevereiro de 2005 16:00
>Assunto: Teste da gasolina
>
>Me chamo Eduardo (8a. série). Gostaria de fazer uma experiência na feira
>de ciências de minha escola na qual eu pudesse reproduzir um teste de
>qualidade da gasolina.
>Qual tipo de reação química poderia utilizar?
>Quais substâncias químicas (reagentes) reagem com a gasolina
>possibilitando sua identificação de qualidade (presença de outro solvente
>não permitido por lei ou muito teor de alcool? Pensei em um teste
>semelhante aos Kits de teste de piscina para ver a qualidade da água.
>Será possível?
>Agradeço atenção,
>Eduardo


São muitas as respostas.

Uma delas é para medir o teor de álcool adicionado na gasolina.
Isso é feito nos Postos, acho, quando o consumidor solicita.
Basta ele ir lá e olhar.

Outra delas é para medir, me parece, o teor de água no alcool combustível.

No caso da ESSO, ela adiciona um marcador, uma substancia secreta, e ela
vai lá, nos Postos, periodicamente, pra coletar amostras e analisar, no
próprio Posto, se essa substancia está presente na gasolina, nos teores que
ela adicionou.
Com isso ela protege o consumidor indiretamente.
Primeiro, ela trata de proteger a si própria, avaliando se o Posto andou
comprando gasolina de outra marca, gasolina fraudada etc.

No caso da Shell deve ser semelhante, tanto assim que ela propagandeia o
termo "DNA da Shell". Suponho que seja uma substancia "marcador", similar
ao caso da ESSO.

Outra possibilidade é a adição de solventes na gasolina.
Aí é feito porque o solvente é mais barato.
Mas principalmente, suponho, porque o solvente não está onerado pelos
impostos incidentes em cima da gasolina.

Umas frandes prejudicam o rendimento do motor.
Outras afetam o funcionamento.

L.E.

[As partes desta mensagem que não continham texto foram removidas]



##### ##### #####

Para saber mais visite
http://www.ciencialist.hpg.ig.com.br


##### ##### ##### #####
Links do Yahoo! Grupos










--
No virus found in this incoming message.
Checked by AVG Anti-Virus.
Version: 7.0.300 / Virus Database: 265.8.6 - Release Date: 07/02/2005




--
No virus found in this outgoing message.
Checked by AVG Anti-Virus.
Version: 7.0.300 / Virus Database: 265.8.6 - Release Date: 07/02/2005



SUBJECT: Re: [ciencialist] Sobre batimentos em ondas sonoras
FROM: Franco <dfranco@pop.com.br>
TO: ciencialist@yahoogrupos.com.br
DATE: 10/02/2005 03:51

Ôpa! Grato pela orientação! Já vou providenciar tudo por aqui.
Abraços.


Luiz Ferraz Netto escreveu:

> Oi Franco,
>
> sim entendi seu texto anterior. Apenas perguntei sobre o osciloscópio
> porque percebi que vc queria "ver" como a coisa funciona e, para isso,
> nada melhor que um osciloscópio traço duplo.
>
> Ainda no mesmo espírito, ver para crer, pergunto: vc tem algum
> software para gerar o gráfico de funções? Eu uso o "Equation Grapher -
> Math.exe". De graça e é uma beleza.
>
> Se baixar o dito cujo de algum terreiro (pode ser perguntando ao Sto
> Google), coloque lá as funções y1=2.sen(10.x) e y2 = 2.sen(11.x). E
> observe ais gráficos e suas superposições (interferência) --- ai vc
> enxerga o batimento.
> Depois vá diminuindo a diferença |f2 - f1| de |11 - 10| para |11 -
> 10,5|, |11 - 10,7| .... e enxergando o efeito do "bit zero".
>
> Com o V.F.O. de um transmissor (na fase de carregar a antena) e o
> receptor do rádio amador vc tb pode 'escutar' isso; já escutou.
>
> O detetor de metais faz algo parecido; o meu aqui em casa é supimpa.
>
> Depois, bem depois, de uma olhada na lei das cordas de Mersene, ou de
> Lagrange, na experiência de Melde.
>
> aquele abraço,
> ===========================
> Luiz Ferraz Netto [Léo]
> leobarretos@uol.com.br
> http://www.feiradeciencias.com.br
> ===========================
>




SUBJECT: Re: [ciencialist] Re: Sobre batimentos em ondas sonoras
FROM: Franco <dfranco@pop.com.br>
TO: ciencialist@yahoogrupos.com.br
DATE: 10/02/2005 03:57

Vou comparar o fenômeno a partir de gráficos, observando com mais
atenção. Qualquer dúvida, recorro a vocês.

Franco.

João R. Bonomo escreveu:

>
> --- Em ciencialist@yahoogrupos.com.br, Franco <dfranco@p...> escreveu
> >
> > Segundo minha mensagem anterior, eu penso que à medida em que
> intervalo
> > entre um batimento e outro torna-se menor, significa que as
> freqüências
> > das duas ondas estão se igualando. Só gostaria de confirmar isso,
> pois
> > não foi o que notei aqui.
> >
> Olá Franco,
>
> É exatamente ao contrário: "à medida em que os intervalos tornam-se
> MAIORES, significa que as freqüências estão se igualando".
>
> []`s
> JRB
>
>
>




SUBJECT: Fw: SOLICITA��O
FROM: "Luiz Ferraz Netto" <leobarretos@uol.com.br>
TO: "ciencialist" <ciencialist@yahoogrupos.com.br>
DATE: 10/02/2005 04:56

Quem indica um 'sustituto' para esse capacitor?
[]'
===========================
Luiz Ferraz Netto [Léo]
leobarretos@uol.com.br
http://www.feiradeciencias.com.br
===========================
-----Mensagem Original-----
De: Paulo Almeida
Para: leobarretos@uol.com.br
Enviada em: quarta-feira, 9 de fevereiro de 2005 15:29
Assunto: SOLICITAÇÃO


Cara professor,



Vi o seu nome em uma materia de Feira de Ciências e decidi solicitar o seu auxílio.



Há uns três anos comprei uma lareira importada, aquelas com um vidro na frente e que o calor do fogo aquece uma estrutura toda de ferro da própria lareira. Este calor e canalizado para uma caixa onde há dentro uma pequena turbina que suga este ar quente e o envia para uma tubulação na residência.

O motor que gira a turbina é pequeno. Funciona com corrente 240v e tem um capacitor de partida. Este capacitor está queimado (rachou) e não consigo outro pois o motor também é importado.



No capacitador de partida está escrito: ARCOTRONICS CE 3,15

1.27.6CC2.MKP 3,15 uF+-5%

420V - 10000h/Class 8 470V - 3000h/Class C



Resido em Porto Alegre - RS e não consegui até agora um técnico que me auxilie na resolução deste problema. Gostaria de saber se há algum fabricante que tenha algum capacitor semelhante ou alguém que possa me auxiliar.



Antecipadamente agradeço pela sua atenção.



Paulo Almeida





--
No virus found in this outgoing message.
Checked by AVG Anti-Virus.
Version: 7.0.300 / Virus Database: 265.8.6 - Release Date: 7/2/2005




--------------------------------------------------------------------------------


No virus found in this incoming message.
Checked by AVG Anti-Virus.
Version: 7.0.300 / Virus Database: 265.8.6 - Release Date: 07/02/2005

----------

No virus found in this outgoing message.
Checked by AVG Anti-Virus.
Version: 7.0.300 / Virus Database: 265.8.6 - Release Date: 07/02/2005


[As partes desta mensagem que não continham texto foram removidas]



SUBJECT: Re: Fw: SOLICITAÇÃO
FROM: "Tipoalgo" <tipoalgo@bol.com.br>
TO: ciencialist@yahoogrupos.com.br
DATE: 10/02/2005 07:45


Olá Paulo,

Tive um problema semelhante.

Fui numa loja que vende componentes eletrônicos e comprei um
capacitor de poliester com valores bem próximos, no seu caso 3
microfarads e tensão superior a 450 volts. Os capacitores de
poliester são bem mais baratos que os vendidos em lojas
especializadas em equipamentos elétricos.

Se não encontrar um único capacitor com a capacitância de 3 uF/450V
faça associações de vários. Ex: três de 1 uf/450V ligados em
paralelo, dois de 6uF/250V ligados em série, e por ai vai.

O motor que uso é de pequeno porte, a partida e o funcionamento
ficaram normais.

Se o enrolamento estiver bom, acho que irá funcionar sem problemas.

Sucesso!

Tipoalgo



--- Em ciencialist@yahoogrupos.com.br, "Luiz Ferraz Netto"
<leobarretos@u...> escreveu
> Quem indica um 'sustituto' para esse capacitor?
> []'
> ===========================
> Luiz Ferraz Netto [Léo]
> leobarretos@u...
> http://www.feiradeciencias.com.br
> ===========================
> -----Mensagem Original-----
> De: Paulo Almeida
> Para: leobarretos@u...
> Enviada em: quarta-feira, 9 de fevereiro de 2005 15:29
> Assunto: SOLICITAÇÃO
>
>
> Cara professor,
>
>
>
> Vi o seu nome em uma materia de Feira de Ciências e decidi
solicitar o seu auxílio.
>
>
>
> Há uns três anos comprei uma lareira importada, aquelas com um
vidro na frente e que o calor do fogo aquece uma estrutura toda de
ferro da própria lareira. Este calor e canalizado para uma caixa onde
há dentro uma pequena turbina que suga este ar quente e o envia para
uma tubulação na residência.
>
> O motor que gira a turbina é pequeno. Funciona com corrente 240v e
tem um capacitor de partida. Este capacitor está queimado (rachou) e
não consigo outro pois o motor também é importado.
>
>
>
> No capacitador de partida está escrito: ARCOTRONICS CE 3,15
>
> 1.27.6CC2.MKP 3,15 uF+-5%
>
> 420V - 10000h/Class 8 470V - 3000h/Class C
>
>
>
> Resido em Porto Alegre - RS e não consegui até agora um técnico que
me auxilie na resolução deste problema. Gostaria de saber se há algum
fabricante que tenha algum capacitor semelhante ou alguém que possa
me auxiliar.
>
>
>
> Antecipadamente agradeço pela sua atenção.
>
>
>
> Paulo Almeida
>
>
>
>
>
> --
> No virus found in this outgoing message.
> Checked by AVG Anti-Virus.
> Version: 7.0.300 / Virus Database: 265.8.6 - Release Date: 7/2/2005
>
>
>
>
> --------------------------------------------------------------------
------------
>
>
> No virus found in this incoming message.
> Checked by AVG Anti-Virus.
> Version: 7.0.300 / Virus Database: 265.8.6 - Release Date:
07/02/2005
>
> ----------
>
> No virus found in this outgoing message.
> Checked by AVG Anti-Virus.
> Version: 7.0.300 / Virus Database: 265.8.6 - Release Date:
07/02/2005
>
>
> [As partes desta mensagem que não continham texto foram removidas]





SUBJECT: Fw: O QUE � INSTRUMENTA��O E CONTROLE DE PROCESSO
FROM: "Luiz Ferraz Netto" <leobarretos@uol.com.br>
TO: "ciencialist" <ciencialist@yahoogrupos.com.br>
DATE: 10/02/2005 10:07

Alguém mais 'moderno' que eu pode definir isso no contexto da atualidade?
[]'
===========================
Luiz Ferraz Netto [Léo]
leobarretos@uol.com.br
http://www.feiradeciencias.com.br
===========================
-----Mensagem Original-----
De: Santa Casa
Para: leobarretos@uol.com.br
Enviada em: quarta-feira, 9 de fevereiro de 2005 12:13
Assunto: O QUE É INSTRUMENTAÇÃO E CONTROLE DE PROCESSO



Nas dúvidas experimentais, por gentileza coloque aqui o endereço da página, isso facilita o confronto. Agradeço. Meu nome é LUIZ FERRAZ NETTO, meu apelido é LÉO e moro em BARRETOS; dai vem meu e-mail: leobarretos@uol.com.br.


BOM DIA ..LEO

SOU ALUNO DE MEGATRONICA E ESTOU EM BUSCA DE DEFINICOES CONCRETA A RESPEITO DO TEMA:

1- O QUE INSTRUMENTAÇÃO?
2- O QUE É CONTROLE DE PROCESSO

AGRADEÇO SUA AJUDA.

OBRIGADO
CLEBER


--------------------------------------------------------------------------------


No virus found in this incoming message.
Checked by AVG Anti-Virus.
Version: 7.0.300 / Virus Database: 265.8.6 - Release Date: 07/02/2005

----------

No virus found in this outgoing message.
Checked by AVG Anti-Virus.
Version: 7.0.300 / Virus Database: 265.8.6 - Release Date: 07/02/2005


[As partes desta mensagem que não continham texto foram removidas]



SUBJECT: Re: [ciencialist] O Sedna e a astrologia
FROM: "Alvaro Augusto \(E\)" <alvaro@electraenergy.com.br>
TO: <ciencialist@yahoogrupos.com.br>
DATE: 10/02/2005 10:24

Caro Manuel,

Antes de mais nada, deixe-me alertar que os astrólogos não "engolem" argumentos desse tipo. Eles conseguiram sobreviver à descoberta de Urano, de Netuno e de Plutão. Com toda certeza sobreviverão à descoberta de uma rocha perdida nos confins do sistema solar. Ao contrário dos astrônomos, que usam métodos perturbativos para fazer seus cálculos, os astrólogos parecem usar um método "anti-perturbativo". Afinal, eles não se perturbam por nada, hehe. O que eles têm a dizer sobre Sedna está expresso, por exemplo, em http://www.espaco-do-ceu.com.br/SEDNA.htm, do qual retirei o seguinte trecho:

"Neste caso como ocorre com Varuna, Ixion e Quaoar, a grande maioria dos astrólogos não terá interesse no estudo do significado de Sedna. Contudo, alguns poucos irão acompanhar sua movimentação, formularão hipóteses e o utilizarão em suas interpretações. Isto já ocorreu com alguns asteróides, como Ceres e Palas por exemplo (os maiores do cinturão localizado entre os planetas Marte e Júpiter), descobertos no início do século XIX que a partir do século XX despertaram o interesse de uns poucos astrólogos e dentre estes alguns até consideraram que a eles deve ser dada a regência dos signos de Virgem e Libra respectivamente (tradicionalmente regidos por Mercúrio e Vênus). De qualquer forma, a utilização de um novo corpo celeste por um astrólogo demanda tempo, devido à necessidade de pesquisa através de acompanhamento do seu movimento, formulação de hipótese e verificação prática. "

O raciocínio anti-perturbativo é expresso mais claramente em http://www.jornaldaorla.com.br/coluna8/1391.shtml, onde o autor afirma:

"É importante salientar que quando a humanidade detecta a existência de um novo planeta há, paralelamente, uma mudança na consciência da humanidade. Esse fenômeno aconteceu muito claramente quando foram descobertos Urano (1781), Netuno (1846) e Plutão (1930), o que abordaremos em futuras colunas."

Dito de outra forma, se os fatos não concordam com a teoria, mudem-se os fatos!

[ ]s

Alvaro Augusto
www.lunabay.com.br/alvaro.php
alvaro@lunabay.com.br


----- Original Message -----
From: Manuel Bulcão
To: ciencialist@yahoogrupos.com.br
Sent: Thursday, February 10, 2005 2:23 AM
Subject: [ciencialist] O Sedna e a astrologia



Oi,

Gostaria de saber o que os astrólogos têm a dizer a respeito desse
novo planeta do Sistema Solar recentemente descoberto, o Sedna.

[]s
Manuel Bulcão



[As partes desta mensagem que não continham texto foram removidas]



SUBJECT: Fw: Maquina de benjamin franklin
FROM: "Luiz Ferraz Netto" <leobarretos@uol.com.br>
TO: "ciencialist" <ciencialist@yahoogrupos.com.br>
DATE: 10/02/2005 10:29

Que máquina será essa? Será a pipa? ou o papagaio? ou a cartola?
[]'
===========================
Luiz Ferraz Netto [Léo]
leobarretos@uol.com.br
http://www.feiradeciencias.com.br
===========================
-----Mensagem Original-----
De: Antonio Dourado
Para: leobarretos@uol.com.br
Enviada em: terça-feira, 8 de fevereiro de 2005 13:36
Assunto: Maquina de benjamin franklin


Olá professor, tudo bem?
Preciso de uma ajuda, e pesquisando na internet achei a pessoa certa p/ soluciona-la, o senhor...

Quero saber como funciona e como se monta a maquina que Benjamin Franklin inventou p/ detectar a chegada de nuvens eletricamente caregadas(Cumulus Nimbus).

Grato pela sua sua compreenção...
__________________________________________________
Converse com seus amigos em tempo real com o Yahoo! Messenger
http://br.download.yahoo.com/messenger/



--------------------------------------------------------------------------------


No virus found in this incoming message.
Checked by AVG Anti-Virus.
Version: 7.0.300 / Virus Database: 265.8.6 - Release Date: 07/02/2005

----------

No virus found in this outgoing message.
Checked by AVG Anti-Virus.
Version: 7.0.300 / Virus Database: 265.8.6 - Release Date: 07/02/2005


[As partes desta mensagem que não continham texto foram removidas]



SUBJECT: Máquina de fazer gente - Ana Beatriz
FROM: José Renato <jrma@terra.com.br>
TO: <ciencialist@yahoogrupos.com.br>
DATE: 10/02/2005 10:32

Publicado em 10.02.2005

DIRETO DA REDAÇÃO

MÁQUINA DE FAZER GENTE

ANA BEATRIZ

No livro Admirável Mundo Novo, de Aldous Huxley, as pessoas de um império são criadas em laboratório e pré-condicionadas para certos talentos e profissões, através da clonagem e modificação de DNA. Há cinco classes sociais, listadas em ordem de habilidade e beleza física: Alfa, Beta, Gama, Delta e Epsilon. Nos Estados Unidos, o império da vida real, acontece também um certo condicionamento; porém, o processo é inciado na infância e não precisa de manobras genéticas.

Os americanos começam a cantar quando aprendem a falar. Os pais, assim que notam um certo jeito da criança com o microfone a cantar para um público de Barbies e Barneys, passam a inscrevê-las em tudo quanto é tipo de aula e concurso. Foi assim com Britney Spears, Justin Timberlake e Christina Aguilera. Quando tinham dez anos, os três já contavam com uma longa carreira e foram convidados para apresentarem juntos um programa televisivo da Disney, o Mickey Mouse Club. Após anos de treino, viraram estrelas da música pop, ainda na adolescência. Nesse país, não adianta só saber cantar, ou só saber dançar, ou simplesmente possuir um corpo bonito. Tem que ser bom em tudo, ou então ter um talento extraordinário em alguma das categorias e trabalhar duro para melhorar nas outras. Se a pessoa não entra na roda-viva cedo, sofre uma bruta desvantagem no show biz.

Os alunos nas escolas dos Estados Unidos também passam por um longo condicionamento, esse mais organizado e segmentado. As crianças começam logo a serem monitoradas e separadas por nível de excelência acadêmica. Tomemos como exemplo um garoto chamado John. O pequeno John, já sabendo adicionar e subtrair números rapidamente na terceira série, é classificado como "gifted" (ou "talentoso"), nesse caso em matemática. Então a escola, com a autorização dos pais, o coloca numa sala de aula com crianças que parecem possuir o mesmo nível intelectual. Quando John chega à sétima série, a universidade Yale oferece a ele e aos colegas "gifted" a chance de já prestar o SAT, o vestibular americano. John vai muito bem na parte de matemática, mas não na de literatura. Por não atingir um número de pontos satisfatório, John ainda não tem vaga garantida na Yale - mas não é apenas o vestibular que conta para admissão nas grandes universidades. As pessoas com aptidão em diferentes áreas e envolvidas em atividades extracurriculares, como esportes e serviço comunitário, têm mais chance de entrarem para uma Yale ou até de ganharem bolsa de estudos.

Com o passar dos anos, John melhora na área de humanas e é considerado o melhor na área de exatas. Entra para a aula de "Advanced Placement" em Cálculo e Física, onde já ganha créditos para a faculdade. Um nível abaixo de "Advanced Placement" estão as aulas "Honors", e é lá que John se encaixa nas matérias de História Americana e Literatura Inglesa. Porém, nunca senta-se numa sala de aula de nível comum, não tem muitos amigos, não pratica esportes e não faz parte do clube de senadores mirins. Tendo entrado em depressão por causa da pressão que sofre para conformar-se ao estereótipo de CDF polivalente, e da competição nesse meio, John acaba indo muito mal no vestibular. Não consegue bolsa e os pais não tem dinheiro para pagar uma universidade de prestígio. Ainda mais deprimido, John mete-se com drogas. Rouba pelas drogas e acaba na cadeia.

Pode parecer exagero, mas casos como esse acontecem - e até piores. Jovens desequilibrados podem acabar se suicidando ou matando outras pessoas. Vão da classe Alfa, nos termos de Aldous Huxley, à triste realidade da classe Epsilon. Afinal de contas, é difícil um ex-presidiário arranjar um emprego decente ou poder continuar de onde parou no processo de condicionamento.


< http://www.diretodaredacao.com/ >




[As partes desta mensagem que não continham texto foram removidas]



SUBJECT: memética
FROM: "E m i l i a n o C h e m e l l o" <chemelloe@yahoo.com.br>
TO: <ciencialist@yahoogrupos.com.br>
DATE: 10/02/2005 11:07

Olha só o que eu encontrei na net:

http://www.genismo.com/memetica.htm

[ ] 's do Emiliano Chemello
emiliano@quimica.net
http://www.quimica.net/emiliano
http://www.ucs.br/ccet/defq/naeq

" Rien ne se perd, rien ne se crée,
tout se transforme."

Antoine Laurent de Lavoisier (químico francês, 1743 - 1794)


[As partes desta mensagem que não continham texto foram removidas]



SUBJECT: Re: [ciencialist] memética
FROM: "Luiz Ferraz Netto" <leobarretos@uol.com.br>
TO: <ciencialist@yahoogrupos.com.br>
DATE: 10/02/2005 11:34



Emiliano Chemello

Olha só o que eu encontrei na net:

http://www.genismo.com/memetica.htm


Léo: Falando do Jocax, link acima, quem tem notícia do João do Nônio?

[]'


--
No virus found in this outgoing message.
Checked by AVG Anti-Virus.
Version: 7.0.300 / Virus Database: 265.8.6 - Release Date: 07/02/2005



SUBJECT: Re: Fw: Teste da gasolina
FROM: "rmtakata" <rmtakata@altavista.net>
TO: ciencialist@yahoogrupos.com.br
DATE: 10/02/2005 11:52


Alguns testes muito simples.

O teste do copo plastico. A gasolina dissolve o poliestireno do copo
plastico. Se houver alcool ou agua na mistura, a taxa de reacao sera'
mais lenta: a amostra demorara' mais para corroer o copo. Claro, q. se
o batismo for por meio de solventes a fraude pode ser mais dificil de
ser detetada - embora alguns solventes dissolvam melhor o poliestireno.

Outro eh o teste da decantacao. Coloca-se a amostra em uma proveta
graduada e espera o alcool se separar da mistura. Mede-se a quantidade
de alcool (a fase mais clara).

Pode-se medir a densidade tb. A gasolina tem uma densidade, o alcool
outra. Dependendo do teor de alcool, a densidade da mistura serah maior.

Sao todos testes fisicos de execucao relativamente facil.

Aproveitando, a Secretaria da Fazenda do Estado de Sao Paulo, divulga
lista de postos de gasolina flagrados comercializando combustiveis
adulterados. A lista se encontra em:

http://www.fazenda.sp.gov.br/noticias/combustiveis.asp

[]s,

Roberto Takata

--- Em ciencialist@yahoogrupos.com.br, "Luiz Ferraz Netto"
> Olá Químicos,
>
> podem dar uma mãozinho para esse menino da 8a série?
>
> agradeço,
> ===========================
> Luiz Ferraz Netto [Léo]

> -----Mensagem Original-----
> De: Eduardo Leão
> Para: leobarretos@u... ; daivemmeue-mailleobarretos@u...
> Enviada em: quarta-feira, 9 de fevereiro de 2005 16:00
> Assunto: Teste da gasolina
>
> Me chamo Eduardo (8a. série). Gostaria de fazer uma experiência na
> feira de ciências de minha escola na qual eu pudesse reproduzir um
> teste de qualidade da gasolina.
> Qual tipo de reação química poderia utilizar?
> Quais substâncias químicas (reagentes) reagem com a gasolina
> possibilitando sua identificação de qualidade (presença de outro
> solvente não permitido por lei ou muito teor de alcool? Pensei em um
> teste semelhante aos Kits de teste de piscina para ver a qualidade
> da água.
> Será possível?
>
> Agradeço atenção,
> Eduardo





SUBJECT: Re: que tal, convence?
FROM: Hélio Ricardo Carvalho <hrc@fis.puc-rio.br>
TO: ciencialist@yahoogrupos.com.br
DATE: 10/02/2005 12:42


Rayfisica,

Em dezembro Rosevena3 nos trouxe o artigo anterior deste cara.
Abaixo é o texto da pagina:
http://www.cefetsp.br/edu/sinergia/5p12c.html

Eu respondi um pouco em:
http://br.groups.yahoo.com/group/ciencialist/message/43079
http://br.groups.yahoo.com/group/ciencialist/message/43139
http://br.groups.yahoo.com/group/ciencialist/message/43165

Fique devendo a ele alguns cálculos para uma geometria puramente
esférica. Cheguei a começar isto mais agora não estou encontrando.

Mas vou acrescentar alguns detalhes agora:
Colocar cargas no interior de um condutor oco através de uma correia
(isolante) é muito diferente do que "coloca-las" por uma bateria.
Por definição o compromisso da bateria é com a ddp e não com a
carga. Ela vai a qualquer custo possível tentar manter a ddp
constante. Mas ddp é diferença entre dois pontos. No primeiro caso
do artigo acima é entre o centro e a placa externa.
Quando a esfera de transferência se aproxima da esfera central a
capacitância do sistema vai mudando (aumentando) a a carga na esfera
central também aumenta (tudo em nome de manter a ddp constante). Na
geometria do artigo fica quase impossível quantificar este aumento.
Mas numa geometria totalmente esférica teriamos:
"Esfera do gerador, capacitor esférico envolvendo e uma membrana
metálica esférica cujo raio variasse desde o da esfera do gerador
até a interna do capacitor."

A capacitância equivalente seria a série destes capacitores:

1/Ceq = 1/Cex +1/Cem + 1/Cmi

onde Cex é o capacitor externo;
Cem é o capacitor entre a placa mais interna de Cex e a
membrana;
Cmi é o capacitor entre a membrana e a esfera interna.

Um capacitor esférico pode ser calculado como:
C=4*pi*epsom0 a*b/(b-a)
onde "b" é o raio maior e "a" o menor.

Para que esta msg não fique muito longa vou fazer como antes:
separar.

Aqui termina a primeira parte.
Depois do almoço eu continuo.


[ ]'s
Hélio










SUBJECT: Re: Astrologia e Metereologia e top-posting (era : Zodiaco)
FROM: "Sergio M. M. Taborda" <sergiotaborda@terra.com.br>
TO: ciencialist@yahoogrupos.com.br
DATE: 10/02/2005 12:42


--- Em ciencialist@yahoogrupos.com.br, "Alvaro Augusto \(E\)"
<alvaro@e...> escreveu


> Um tsunami dificilmente seria previsto por meteorologistas, pois,
tendo sido
> provocado por um abalo sísmico, está mais no campo da sismologia...

Vai dar oa mesmo, em questão de ciencias da terra , todas as previsões
são tão boas quanto chutes, já que não ha tempo para fazer nada em
resposta.

> Quando ao "top posting", francamente, acho isso um saco. Trata-se de
coisa
> dos primórdios da internet, quando todos as mensagens eram em modo
texto. O
> meu leitor de e-mails, por exemplo, não coloca essas porcarias de ">"
> automaticamente. Então, tenho que abrir a mensagem em um editor de
textos,
> colocar todos os ">" e depois colar na resposta. Tudo isso para não
ferir
> uma regra que algum belezinha criou!

Mude de cliente de email. O thunderbird é muito bom e gratis.

> Que problema há em ler as contribuições na ordem inversa, afinal?

Não, o problema não é esse. Top-posting é responder à mensagem,
deixando toda a mensagem a que se responde no fim. O problema está em
reenviar um texto que nunca será lido (pq já foi lido antes). Isso
ocupa espaço nas caixas de correio sem nenhuma necessidade. O mesmo é
válido para under-posting, que foi o que vc usou na sua mensagem.
(deixar todo a mesnagem original no topo e responder no fim , sendo
que a resposta versa apenas o ultimo paragrafo)


Sérgio Taborda





SUBJECT: Re: [ciencialist] Fw: Maquina de benjamin franklin
FROM: Luis Brudna <luisbrudna@gmail.com>
TO: ciencialist@yahoogrupos.com.br
DATE: 10/02/2005 12:49

Já vi isso em um documentário. Procurando no Google...

http://www.ph.unimelb.edu.au/staffresources/lecdem/el11.htm
http://tinyurl.com/3wbt2
http://sln.fi.edu/franklin/bells.html
http://www.edgerton.org/kidscorner/franklinsbells.html

Até
Luís Brudna


On Thu, 10 Feb 2005 10:29:24 -0200, Luiz Ferraz Netto
<leobarretos@uol.com.br> wrote:
>
> Que máquina será essa? Será a pipa? ou o papagaio? ou a cartola?
> []'
> ===========================
> Luiz Ferraz Netto [Léo]
> leobarretos@uol.com.br
> http://www.feiradeciencias.com.br
> ===========================
> -----Mensagem Original-----
> De: Antonio Dourado
> Para: leobarretos@uol.com.br
> Enviada em: terça-feira, 8 de fevereiro de 2005 13:36
> Assunto: Maquina de benjamin franklin
>
> Olá professor, tudo bem?
> Preciso de uma ajuda, e pesquisando na internet achei a pessoa certa p/ soluciona-la, o senhor...
>
> Quero saber como funciona e como se monta a maquina que Benjamin Franklin inventou p/ detectar a chegada de nuvens eletricamente caregadas(Cumulus Nimbus).
>
> Grato pela sua sua compreenção...


SUBJECT: Re: [ciencialist] Re: Fw: Teste da gasolina
FROM: Luis Brudna <luisbrudna@gmail.com>
TO: ciencialist@yahoogrupos.com.br
DATE: 10/02/2005 13:51

Adicionando um pouco de água o treco funciona.

http://www.cdcc.sc.usp.br/quimica/experimentos/teor.html

Até
Luís Brudna


On Thu, 10 Feb 2005 13:52:56 -0000, rmtakata <rmtakata@altavista.net> wrote:
> Outro eh o teste da decantacao. Coloca-se a amostra em uma proveta
> graduada e espera o alcool se separar da mistura. Mede-se a quantidade
> de alcool (a fase mais clara).
> Roberto Takata


SUBJECT: Re: Fw: Teste da gasolina
FROM: "rmtakata" <rmtakata@altavista.net>
TO: ciencialist@yahoogrupos.com.br
DATE: 10/02/2005 14:40


--- Em ciencialist@yahoogrupos.com.br, Luis Brudna <luisbrudna@g...>
> Adicionando um pouco de água o treco funciona.
> http://www.cdcc.sc.usp.br/quimica/experimentos/teor.html

Verdade, precisa colocar agua.

Um teste mais complicado eh medir o poder calorifico da amostra.
Queimar uma quantidade conhecida e verificar o qto se aquece uma
massa definida de agua.

[]s,

Roberto Takata





SUBJECT: Re: que tal, convence?
FROM: Hélio Ricardo Carvalho <hrc@fis.puc-rio.br>
TO: ciencialist@yahoogrupos.com.br
DATE: 10/02/2005 15:14


CONTINUANDO ...

>
> A capacitância equivalente seria a série destes capacitores:
>
> 1/Ceq = 1/Cex +1/Cem + 1/Cmi
>
> onde Cex é o capacitor externo;
> Cem é o capacitor entre a placa mais interna de Cex e a
> membrana;
> Cmi é o capacitor entre a membrana e a esfera interna.
>
> Um capacitor esférico pode ser calculado como:
> C=4*pi*epsom0 a*b/(b-a)
> onde "b" é o raio maior e "a" o menor.

Vamos dar nomes aos raios:
r1 --> raio da esfera interna,
r2 --> raio da membrana (no início vamos considera-la muito fina),
r3 --> raio da placa mais interna do capacitor Cex.
r4 --> raio da placa externa de Cex.

Só r2 é variável.

Substituindo temos:

1/Ceq=(1/4*pi*epson0)*[(r4-r3)/(r3*r4) + (r3-r2)/(r2*r3) +
(r2-r1)/(r2*r1)]

Note que estou considerando todas as placas com espessura
desprezível.

1/Ceq = (1/4*pi*epson0)*[1/r3 - 1/r4 + 1/r2 - 1/r3 + 1/r1 -1/r2]
1/Ceq = (1/4*pi*epson0)*[1/r1 -1/r4]
1/Ceq = (1/4*pi*epson0)*[(r4-r1)/(r4*r1)]

Ou seja, se todas as placas forem sem espessura a capacitância vista
pela bateria não vai se alterar com o movimento da membrana (nem a
carga de cada placa) será sempre:
Ceq = 4*pi*epson0*(r4*r1)/(r4-r1).

Logo para uma análise mais realista devemos levar em conta as
espessuras.

sendo:
d2 --> espessura da membrana,
d3 --> espessura da placa mais interna do capacitor Cex.
d4 --> espessura da placa externa de Cex.

Os r's acima continuam sendo os raios das superfície mais interna de
cada placa (exceto r1 que é o raio da esfera interna logo externo a
ela).
Ex.

temos:
1/Ceq=(1/4*pi*epson0)*[(r4-r3+d3)/((r3+d3)*r4) +
(r3-r2+d2)/((r2+d2)*r3) + (r2-r1)/(r2*r1)]

1/Ceq = (1/4*pi*epson0)*[1/(r3+d3) - 1/r4 + 1/(r2+d2) - 1/r3 + 1/r1
-1/r2]
1/Ceq = (1/4*pi*epson0)*[1/r1 - 1/r4 - d3/(r3*(r3+d3)) -
d2/(r2*(r2+d2)) ]
como só r2 é variável, podemos simplificar assim:

1/Ceq = K - (1/4*pi*epson0)*[ d2/(r2*(r2+d2)) ]

onde K é constante.
obs.: note o sinal "menos" depois do "K".

Com isto na mão podemos ver o que acontece com a capacitância
equivalente quando movimentamos r2.
Podemos também ter uma idéia qualitativa para a geometria mista do
artigo.

Levando em conta a expressão acima, quando r2 diminui (aproximando
da esfera interna) a capacitância aumenta (aumenta o termo que é
retirado de K então diminui o 1/Ceq logo aumenta Ceq).

Para manter a ddp constante (V=Q/C) a carga também vai aumentando.

Note que ainda não houve toque.
O que representa este "Q"???

Este "Q" será em cada instante:

Q = V*Ceq = V / { K - (1/4*pi*epson0)*[ d2/(r2*(r2+d2)) ] }

Isto será o módulo da carga negativa na esfera interna (na
transformação de geometria mantive a polaridade da bateria no artigo
original).
Mas também será:

o módulo da carga positiva na superfície mais interna da membrana,

o módulo da carga negativa na superfície mais externa da membrana,

o módulo da carga positiva na superfície mais interna da placa
interna de Cex,

o módulo da carga negativa na superfície mais externa da placa
interna de Cex,

o módulo da carga positiva na superfície mais interna da placa
externa de Cex,

Pode parar por aqui:

Obs.: a superfície mais externa da placa externa de Cex não terá
cargas se a geometria for puramente esférica,

CONTINUA ...


Hélio




















SUBJECT: Re: [ciencialist] Astrologia e Metereologia e top-posting (era : Zodiaco)
FROM: "Oraculo" <oraculo@atibaia.com.br>
TO: <ciencialist@yahoogrupos.com.br>
DATE: 10/02/2005 15:17

Olá Taborda

Taborda: Acho que o pessoal que sofreu com o Tsunami não concorda consigo."

Bem, um tsunami, derivado de um terremoto, que aconteceu devido a movimentação da falha submarina na região, dificilmente seria prevista por meteorologistas ou outra área da ciência, pelo menos no atual estágio do conhecimento humano (embora no futuro talvez seja possível..:-)

Taborda: Da mesma forma que o pessoal que sobre com furações os anos passados."

O pessoal que sofreu com furacões sabia de antemão, pelo menos nos USA, que eles estavam vindo. Pode dizer que são inevitáveis, que sua força ultrapassa a capacidade humana de enfrenta-los ou evita-los, mas não pode dizer que foi por falta de avisos. A maioria dos que não sofreram com eles tiveram tempo de proteger casas, sair de cidades e procurar abrigo. Isso, claro, graças a meteorologia, satelites, radares e boa compreensão do modo de funcionamento, origem, e principalmetne movimentação dos furacões.

Assim, o tsunami não tem muito a ver com nosso debate, embvora os furacões e as vidas salvas graças a meteorologia sejam argumentos a meu favor..:-)

Mesmo assim, sistemas de alerta contra tsunamis (claro, depois do terremoto) podem salvar diversas vidas e os sinais deste tipo de ocorrencia pdoem ser encontrados com a dose de antecipação necessária.

E quanto a astrologia, você a defende como mecanismo hábil e eficaz para conhecer a personalidade, eventos futuros e para determinar datas e horários propícios para ações ou reações. Não defende ser "ciência"? Não importa, o uso que faz dela é suficiente para, se fosse real, encaixa-la na definição de ciência. Se fosse real.

Taborda:Mas até onde ue posso ver o mito não é explicação suficiente. Nenhum
mito nasce do nada, onde á fumo ha fogo."

Muitos mitos nascem do nada, muitos de má compreensão da natureza, outros da imaginação humana, e de outras fontes. Se onde a fumaça há fogo fosse para ser levado a ferro e fogo, deveriamos procurar por entidades mágicas no monte Olimpo (talvez encontrassemos Zeus), por valquirias em cavalos voadores, por duendes e gnomos e outras fumaças. Embora baseado em muitos mitos ancestrais, toda trilogia dos aneis de Tolkien é imaginária e sua enorme coerencia não significa que devem existir elfos e hobbits.

A astrologia não nasceu do nada, mas da compreensão parcial da natureza ciclica das estações, da necessidade de marcar tempo e colheitas de nossos antepassados, da procura humana por respostas, da necessidade que temos de explicar, do controle e poder que "saber mais" que seus conterraneos dá ao possuidor do conhecimento, e de diversas outras origens.

Identificar as origens que se basearam na realidade, como as estações e o movimento aparente de constelações e planetas, e as irreais, frutos da imaginação e da cultura mitologica de sua época, faz toda a diferença.

E me desculpe com relação ao texto depois da mensagem, o completo, mas isso não é top-posting, embora realmente possa incomodar depois de um certo volume de mensagens. Vou tomar o cuidado de eliminar o texto para evitar o acumulo. Top-posting seria a colocação do texto a ser respondido em seguida a resposta. Eu sempre colo o texto a ser respondido antes de minha resposta.

Homero


[As partes desta mensagem que não continham texto foram removidas]



SUBJECT: Re: que tal, convence?
FROM: Hélio Ricardo Carvalho <hrc@fis.puc-rio.br>
TO: ciencialist@yahoogrupos.com.br
DATE: 10/02/2005 16:47



...
...
> Este "Q" será em cada instante:
>
> Q = V*Ceq = V / { K - (1/4*pi*epson0)*[ d2/(r2*(r2+d2)) ] }
>
> Isto será o módulo da carga negativa na esfera interna (na
> transformação de geometria mantive a polaridade da bateria no
artigo
> original).
> Mas também será:
>
> o módulo da carga positiva na superfície mais interna da membrana,
>
> o módulo da carga negativa na superfície mais externa da membrana,
>
> o módulo da carga positiva na superfície mais interna da placa
> interna de Cex,
>
> o módulo da carga negativa na superfície mais externa da placa
> interna de Cex,
>
> o módulo da carga positiva na superfície mais interna da placa
> externa de Cex,
>
> Pode parar por aqui:
>
> Obs.: a superfície mais externa da placa externa de Cex não terá
> cargas se a geometria for puramente esférica,




CONTINUANDO

Vamos agora analisar um caminho inteiro de r2 desde quase (r3-d2)
até r1.
Isto é, a membrana estava quase encostada na placa mais interna de
Cex e foi até encostar na esfera interna.

Q(inicial) = V / { K - (1/4*pi*epson0)*[ d2/(r2*(r2+d2)) ] } = V /
{ K - (1/4*pi*epson0)*[ d2/((r3-d2)*r3 ) ] }

Só para lembrar: Este será o "Q" de cada superfície (incluindo a
esfera interna) neste instante.

Quando a membrana estiver QUASE encostando na esfera interna
teremos:

Q = V / { K - (1/4*pi*epson0)*[ d2/(r2*(r2+d2)) ] } = V / { K -
(1/4*pi*epson0)*[ d2/(r1*(r1+d2)) ] }

como r1*(r1+d2) é menor que (r3-d2)*r3 --> Q será maior que
Q(inicial)
obs.: [r1 < r3-d2] e [r1+d2 < r3].

Ao encostar o "-Q" da esfera interna anula o "+Q" da superfície
interna da membrana e só sobra o "-Q" da superfície externa da
membrana.

ATENÇÃO:
A rigor NÃO houve transferência de carga da esfera interna para a
membrana pois ela já estava lá antes.


Na próxima vez falarei da força, do trabalho e da energia necessária
para levar a membrana por este caminho.

CONTINUA ... :-)



Hélio






SUBJECT: Re: [ciencialist] Astrologia e Metereologia e top-posting (era : Zodiaco)
FROM: "cumullosnimbos" <cumullosnimbos@bol.com.br>
TO: "ciencialist" <ciencialist@yahoogrupos.com.br>
DATE: 10/02/2005 17:16

"(...) Bem, o próprio termo "meteorologia" é incorreto. Meteorologia deveria ser o estudo dos meteoros.... Previsão do tempo deveria receber outro nome, tal como "tempoprevisologia"...(...)"

Olá,

O termo Meteorologia se refere ao "Estudo dos fenômenos atmosféricos" sejam eles, a precipitação, humidade, neve, granizo e os sistemas que induzem a geração destes, tais como: furacões, tornados, Cb's, linhas de instabilidade, supercélulas. Nesta definição simples e direta, não vejo erro algum, visto que, "meteoro" também se refere à um fenômeno atmosférico qualquer.
Se bem entendi a colocação anterior; me refiro ao termo ("tempoprevisologia") acho um enorme equívoco, para não ser mais deselegante...comparar a Meteorologia com a Astrologia! Não pretendo me extender em explicações sobre a eficiência das previsões do tempo, sobretudo da importância delas para os sistemas de alerta da defesa civil. Uma comparação esdrúxula como esta não merece ser retrucada, eu penso que a pessoa que a fez deveria se informar melhor.

---
Daniel.

__________________________________________________________________________
Acabe com aquelas janelinhas que pulam na sua tela.
AntiPop-up UOL - É grátis!
http://antipopup.uol.com.br/




[As partes desta mensagem que não continham texto foram removidas]



SUBJECT: Re: que tal, convence?
FROM: "rayfisica" <rayfisica@yahoo.com.br>
TO: ciencialist@yahoogrupos.com.br
DATE: 10/02/2005 17:41


--- Em ciencialist@yahoogrupos.com.br, Hélio Ricardo Carvalho
<hrc@f...> escreveu
>
>

>
>
> Hélio
>>>>>>>>>>>>>>>>>>>>>>>>>>>>>>>>>>>>>>>>
Obrigado senhor Helio.
Eu estou imprimindo todas as suas mensagens sobre o assunto e não
paro até entender tudo, muitíssimo obrigado por responder com
tanto
interesse, e com equações que até um leigo possa entender, isso
é
ciencia






SUBJECT: Re: O Sedna e a astrologia
FROM: Maria Natália <grasdic@hotmail.com>
TO: ciencialist@yahoogrupos.com.br
DATE: 10/02/2005 20:22


Ai Manel...

Que te calha a Páscoa à 5ª feira...
Pois mas o Sedna "não é um novo planeta do SS" é apenas um asteróide
capturado e portanto não está na lista dos influenciadores do destino
humano. É um pedrinha no sapato do astrónomo. Astrólogo já se
desembaraçou como viste*.
saudações
Maria Natália, tsunami
* fiz de advogada do diabo (com Bulcão...nada mau)

--- Em ciencialist@yahoogrupos.com.br, Manuel Bulcão
<manuelbulcao@u...> escreveu
>
> Oi,
>
> Gostaria de saber o que os astrólogos têm a dizer a respeito desse
> novo planeta do Sistema Solar recentemente descoberto, o Sedna.
>
> []s
> Manuel Bulcão





SUBJECT: Morreu Ernst Mayer , ó bióilogos da lista
FROM: Maria Natália <grasdic@hotmail.com>
TO: ciencialist@yahoogrupos.com.br
DATE: 10/02/2005 20:31


Morreu Ernst Mayr, o Cientista Que Uniu a Teoria da Selecção Natural de
Darwin com a Genética de Mendel
Por CLARA BARATA
Publico, Sábado, 05 de Fevereiro de 2005

Morreu o "Darwin do Século XX", como era chamado o biólogo Ernst Mayr.
Aos 100 anos, completados em Julho passado, Mayr foi um dos cientistas
mais importantes do século XX. Foi uma figura central para a moderna
síntese do neo-darwinismo, ou seja, a amálgama das ciências da
genética, sistemática,paleontologia e ecologia que permitiram
conciliar a teoria da evolução através da selecção natural de Darwin
com as leis da hereditariedade, que o monge checo Gregor Mendel
descobriu, cultivando pacientemente ervilhas no seu jardim.
Mayr nasceu a 5 de Julho de 1904, em Kempten, na Alemanha, herdeiro de
uma longa tradição de médicos, mas tomou-se de amores pela zoologia. A
primeira grande oportunidade surgiu em 1928, quando foi encarregue da
missão de esclarecer a relação entre os muitos espécimes de
aves-do-paraíso dos museus europeus: foi enviado para a Nova Guiné, e
depois para as ilhas Solomão, numa missão "cujas dificuldades são
impossíveis de conceber hoje, quando os exploradores e os seus
assistentes de campo não correm grandes riscos de serem apanhados em
embuscadas por nativos", como escreveu Jared Diamond no prefácio do
livro que Mayr publicou em 2002, aos 98 anos ("What Evolution Is", ou
o que é a evolução, numa tradução literal). Sobreviveu à malária, à
febre de dengue, à desinteria, à morte por afogamento quando a canoa
em que seguia se virou.
Várias vezes foi dado como morto.

Essas viagens de aventureiro permitiram-lhe contactar em primeira mão
com algo que Charles Darwin nunca tinha conseguido definir muito bem -
a evolução das espécies.
Nos anos que passou a bordo do "Beagle", Darwin coleccionou espécimes,
mas foi durante as décadas que se seguiram, em que nunca arredou pé de
Inglaterra, que o naturalista desenvolveu a teoria da selecção natural
como o mecanismo da evolução das espécies. Mas o modo de funcionamento
desse mecanismo permaneceu um pouco nebuloso; Mayr concluiu que é a
partir do isolamento de determinadas populações que surgem novas
espécies. "Foi o pioneiro da definição actualmente aceite de espécie
biológica: uma população de indivíduos que conseguem procriar entre
si, mas não com outros grupos", escreve o comunicado da Universidade
de Harvard, que anunciou a sua morte.

Uma autobiografia?
No início dos anos 30, Ernst Mayr emigrou para os EUA, onde foi
conservador do Museu de História Natural de Nova Iorque, primeiro, e
do de Zoologia da Universidade de Harvard, depois, onde terminou a sua
carreira.Ainda agora tinha um gabinete lá.

Em 2003, numa entrevista à revista "The Scientist", explicou que
sentia a tentação da biografia, embora modestamente: "Nos últimos
tempos, tenho andado a escrever uma autobiografia científica, mas
começo a pensar que estou velho de mais para isso."

Nos seus 80 anos de carreira, Mayr esteve no centro das acesas
polémicas que abalaram o campo dos biologia evolutiva, com o
nascimento da sociobiologia, e da psicologia evolutiva. Escreveu mais
de 700 artigos e 20 livros, mas foi em 1942 que publicou o primeiro
livro que se tornou um marco, "Systematics and the Origin of Species",
onde propunha que a teoria da selecção natural de Darwin podia
explicar toda a evolução, incluindo por que é que os genes evoluem, ao
nível molecular. A teoria do equilíbrio pontuado, desenvolvida por
Stephen Jay Gould e Niles Eldridge, foi beber ao livro "Animal Species
and Evolution", que Mayr publicou em 1963.
Num artigo saído na "Nature" a 21 de Junho de 2004, quando a revista o
homenageou pelo seu 100º aniversário, Mayr fez uma revisão da história
da teoria da evolução no século XX: "Tendo alcançado a rara idade de
100 anos, encontro-me numa posição única: sou o último sobrevivente da
idade de ouro da síntese da evolução", escreveu.
Lamentou só não ter mais tempo: "A biologia da evolução é uma zona de
fronteira infindável e ainda há muito para descobrir. Só tenho pena de
não ir estar presente para apreciar as descobertas futuras."
Um abraço
Maria Natália


______________________________________________





SUBJECT: Re: Máquina de fazer gente - Ana Beatriz
FROM: Maria Natália <grasdic@hotmail.com>
TO: ciencialist@yahoogrupos.com.br
DATE: 10/02/2005 21:11


JR:

Como professora e mesmo não havendo em Portugal tais gifts do
ministério da educação (mas só de papás ricos*) tenho verificado
coisas esquisitas e parecidas. Presentemente tenho uma turma de 11º
ano de 24 alunos (16 a 18 anos) e que para laboratórios e práticas
está dividida ao meio. Verifico que com os alunos tenho um óptimo
contacto e eles compreendem e gostam da inovação que pratico. Ora 4
pais do 2º turno entraram em stress e paranóia total de tal modo que
por volta do fim de 1º período (2 meses de aulas) acharam que eu não
ia dar o programa e os meninos não conseguiriam fazer o vestibular cá
da praça. Foi incrível: eu e os alunos numa nice...e fui a 1ª
professora a terminar a química e que iniciou a física!!! Ficaram
banzados e ainda mais quando viram que os filhos sabiam a matéria e
respondendo a questões mais abertas tinham boas classificações!!!
Estes pais quase me iam matando os alunos do 2º turno! Digo quase
porque se gerou instabilidade e as notas e aulas deste turno foram
sempre de menino de "costas quentes" o que veio a dar notas inferiores
ao 1º turno. Com a mesma professora...
Eu vi com estes olhos que a terra há-de comer meninos de 5 e 7 anos em
conferências na Funddação Calouste Gulbenkian sobre Mecânica Quântica
pelo Paulo Crawford!!!!! Juro que vi meninos em conferências sobre
buracos negros com 5 anos!!! Nunca pensei que criança fosse vítima de
tais maus tratos! HORROR! Que nos fizeram as criançinhas, ó criadores
de génios!?
Os paizinhos devem ter juizinho e ver se, na sua ânsia de se
realizarem, através dos filhos, não estão a criar monstros. Deixai o
trabalho de professores aos profissionais pois os professores também
não se vão meter nos escritórios ou empregos**. Os tempos são outros.
Deixai as crianças serem crianças e crescer fazendo erros. Como é bom
errar, saber que se errou e aprender a lidar com o erro...Qualquer dia
estamos "a lixar" os nossos filhos, como por exemplo, novamente ao
impor-lhes noivo/a logo à nascença. Não se riam pois como contacto
pais há mais de 20 anos tenho assistido a muita estupidez. E dói tanto
ver crianças de 18/19 anos maltratadas. Me dá vontade de dar um
enxerto de porrada em tais pais...Ah regressei ás aulas de karaté, pois.
Deixai vir a mim as criancinhas puras que eu lhes incutirei amor pela
ciência...
Amem
vossos filhos e netos
Um abraço
Maria Natália
PS Conhecem a ensino/educação no Japão?
*meninos que já vão à ópera aos 3 anos...
** os professores também são pais (não eunucos)


--- Em ciencialist@yahoogrupos.com.br, José Renato <jrma@t...> escreveu
> Publicado em 10.02.2005
>
> DIRETO DA REDAÇÃO
>
> MÁQUINA DE FAZER GENTE
>
> ANA BEATRIZ
>
> No livro Admirável Mundo Novo, de Aldous Huxley, as pessoas de um
império são criadas em laboratório e pré-condicionadas para certos
talentos e profissões, através da clonagem e modificação de DNA. Há
cinco classes sociais, listadas em ordem de habilidade e beleza
física: Alfa, Beta, Gama, Delta e Epsilon. Nos Estados Unidos, o
império da vida real, acontece também um certo condicionamento; porém,
o processo é inciado na infância e não precisa de manobras genéticas.
>
>





SUBJECT: Megatronica ? Ou Mecatronica ?
FROM: "L.E.R.de Carvalho" <lecarvalho@infolink.com.br>
TO: ciencialist@yahoogrupos.com.br
DATE: 11/02/2005 02:48

At 10:07 10/2/2005, you wrote:
>Alguém mais 'moderno' que eu pode definir isso no contexto da atualidade?
>[]'
> ===========================
> Luiz Ferraz Netto [Léo]
> leobarretos@uol.com.br
> <http://www.feiradeciencias.com.br>http://www.feiradeciencias.com.br
> ===========================
>-----Mensagem Original-----
>De: Santa Casa
>Para: leobarretos@uol.com.br
>Enviada em: quarta-feira, 9 de fevereiro de 2005 12:13
>Assunto: O QUE É INSTRUMENTAÇÃO E CONTROLE DE PROCESSO
>
>
>
>Nas dúvidas experimentais, por gentileza coloque aqui o endereço da
>página, isso facilita o confronto. Agradeço. Meu nome é LUIZ FERRAZ NETTO,
>meu apelido é LÉO e moro em BARRETOS; dai vem meu e-mail:
>leobarretos@uol.com.br.
>
>
>BOM DIA ..LEO
>
>SOU ALUNO DE MEGATRONICA E ESTOU EM BUSCA DE DEFINICOES CONCRETA A
>RESPEITO DO TEMA:
>
>1- O QUE INSTRUMENTAÇÃO?
>2- O QUE É CONTROLE DE PROCESSO
>
>AGRADEÇO SUA AJUDA.
>
>OBRIGADO
>CLEBER
>
>
>--------------------------------------------------------------------------------
>
>
>No virus found in this incoming message.
>Checked by AVG Anti-Virus.
>Version: 7.0.300 / Virus Database: 265.8.6 - Release Date: 07/02/2005
>
> ----------
>
>No virus found in this outgoing message.
>Checked by AVG Anti-Virus.
>Version: 7.0.300 / Virus Database: 265.8.6 - Release Date: 07/02/2005
>
>
>[As partes desta mensagem que não continham texto foram removidas]


[As partes desta mensagem que não continham texto foram removidas]



SUBJECT: Re: Morreu Ernst Mayer , ó bióilogos da lista
FROM: "rmtakata" <rmtakata@altavista.net>
TO: ciencialist@yahoogrupos.com.br
DATE: 11/02/2005 06:44


--- Em ciencialist@yahoogrupos.com.br, Maria Natália <grasdic@h...>
> Morreu Ernst Mayr, o Cientista Que Uniu a Teoria da Selecção Natural
> de Darwin com a Genética de Mendel
> Por CLARA BARATA
> Publico, Sábado, 05 de Fevereiro de 2005


A perda eh lamentavel.

(Uma pequena correcao. Ernst Mayr eh um dos grandes da ciencia - eh,
no presente, ainda q. tenha ocorrido seu passamento; porem ele eh *um*
dos que uniu a teoria da selecao natural de Darwin e Wallace com a
Genetica mendeliana. Na formalizacao matematica destacam-se Fisher,
Haldane e Wright. Na fixacao dos conceitos dentro da pratica
biologica, Mayr se faz notar ao lado de Dobzhansky e Simpson. Dignos
de mencao tb Huxley e Stebbins. Foi um grande trabalho conjunto e nao
um insight genial de um unico cientista.)

[]s,

Roberto Takata





SUBJECT: Re: [ciencialist] Megatronica ? Ou Mecatronica ?
FROM: "Luiz Ferraz Netto" <leobarretos@uol.com.br>
TO: <ciencialist@yahoogrupos.com.br>
DATE: 11/02/2005 07:11

Não consegui achar onde está a sugestão nessa mensagem!
[]'
===========================
Luiz Ferraz Netto [Léo]
leobarretos@uol.com.br
http://www.feiradeciencias.com.br
===========================
-----Mensagem Original-----
De: "L.E.R.de Carvalho" <lecarvalho@infolink.com.br>
Para: <ciencialist@yahoogrupos.com.br>
Enviada em: sexta-feira, 11 de fevereiro de 2005 02:48
Assunto: [ciencialist] Megatronica ? Ou Mecatronica ?



At 10:07 10/2/2005, you wrote:
>Alguém mais 'moderno' que eu pode definir isso no contexto da atualidade?
>[]'
> ===========================
> Luiz Ferraz Netto [Léo]
> leobarretos@uol.com.br
> <http://www.feiradeciencias.com.br>http://www.feiradeciencias.com.br
> ===========================
>-----Mensagem Original-----
>De: Santa Casa
>Para: leobarretos@uol.com.br
>Enviada em: quarta-feira, 9 de fevereiro de 2005 12:13
>Assunto: O QUE É INSTRUMENTAÇÃO E CONTROLE DE PROCESSO
>
>
>
>Nas dúvidas experimentais, por gentileza coloque aqui o endereço da
>página, isso facilita o confronto. Agradeço. Meu nome é LUIZ FERRAZ NETTO,
>meu apelido é LÉO e moro em BARRETOS; dai vem meu e-mail:
>leobarretos@uol.com.br.
>
>
>BOM DIA ..LEO
>
>SOU ALUNO DE MEGATRONICA E ESTOU EM BUSCA DE DEFINICOES CONCRETA A
>RESPEITO DO TEMA:
>
>1- O QUE INSTRUMENTAÇÃO?
>2- O QUE É CONTROLE DE PROCESSO
>
>AGRADEÇO SUA AJUDA.
>
>OBRIGADO
>CLEBER
>
>
>--------------------------------------------------------------------------------
>
>
>No virus found in this incoming message.
>Checked by AVG Anti-Virus.
>Version: 7.0.300 / Virus Database: 265.8.6 - Release Date: 07/02/2005
>
> ----------
>
>No virus found in this outgoing message.
>Checked by AVG Anti-Virus.
>Version: 7.0.300 / Virus Database: 265.8.6 - Release Date: 07/02/2005
>
>
>[As partes desta mensagem que não continham texto foram removidas]


[As partes desta mensagem que não continham texto foram removidas]



##### ##### #####

Para saber mais visite
http://www.ciencialist.hpg.ig.com.br


##### ##### ##### #####
Links do Yahoo! Grupos










--
No virus found in this incoming message.
Checked by AVG Anti-Virus.
Version: 7.0.300 / Virus Database: 265.8.6 - Release Date: 07/02/2005




--
No virus found in this outgoing message.
Checked by AVG Anti-Virus.
Version: 7.0.300 / Virus Database: 265.8.6 - Release Date: 07/02/2005



SUBJECT: Re: Astrologia e Metereologia e top-posting (era : Zodiaco)
FROM: "Sergio M. M. Taborda" <sergiotaborda@terra.com.br>
TO: ciencialist@yahoogrupos.com.br
DATE: 11/02/2005 08:48


--- Em ciencialist@yahoogrupos.com.br, "cumullosnimbos"
<cumullosnimbos@b...> escreveu
>
> Se bem entendi a colocação anterior; me refiro ao termo
("tempoprevisologia") acho um enorme equívoco, para não ser mais
deselegante...comparar a Meteorologia com a Astrologia! Não pretendo
me extender em explicações sobre a eficiência das previsões do tempo,
sobretudo da importância delas para os sistemas de alerta da defesa
civil. Uma comparação esdrúxula como esta não merece ser retrucada, eu
penso que a pessoa que a fez deveria se informar melhor.

Informar melhor... em um curso de fisica aprende-se muita coisa. E uma
delas é termodinamica. A termodinamica é a base dos modelos do clima.
Os modelos de clima são tão complexos que são precisos
supercomputadores (hoje em dia, programação destribuida) que fazer os
calculos. Uma vez feitos os calculos tem validade de 8 a 15 dias,
sendo que o erro estimado aumenta com o prazo de validade (por isso
que o prazo é tão curto pois se aumentado, o erro supera a previsao -
o que fisicamente não tem significado). O calculo demora pelo menos 2
dias. O que nos deixa com 6 a 13 dias de validade.
Outra coisa que se aprende em fisica é que os modelos não são
infaliveis, sobretudo os modelos ah doc, como os da meteorologia.
A metereologia como uma ciencia baseada em modelos é um fracasso.
Desde dos anos 50 que se vem tentando chegar num modelo e não estamos
nem perto. Isto não sou eu que o digo, são os proprios professores de
metereologia.
A metereologia que funciona é bem mais simples e é baseada em dados
empiricos e não em modelos. (ou seja, é baseada na catalogação de
resultados correctos e não em previsão com modelos). Por exemplo, a
chegada de uma frente fria , é conhecida pela diminuição da
temperatura e traz consigo chuva forte, uma frente quente , traz
subida de temperatura e chuva leve. O que se pode explicar
termodinamicamente com uma alteração da pressão. Mas a medição da
pressão, por si só, não é suficiente para prever a chegada da frente.
Este tipo de conhecimento é apenas baseado na observação e inferencia
de regras do clima, mas não em termodinamica ou em modelos do clima.
Baseado em observação eu sei, quado saio de casa, muito melhor que
tempo vai fazer do que o relatório do tempo no dia anterior - que não
acerta uma. A metereologia é importante , por exemplo, para a
agricultura, mas se houvesse realmente um conhecimento de causa, e um
previsão factual não existiriam desastres agriculas, em que uma geada,
uma chuva ou o calor destroem toda a colheita.

Então, dado o fraco grau de previsão da meteorologia, que precisa de
observação directa (os satelites são só isso e não uma avanço da
metereologia )para tirar alguma conclusão, e que quando se poe a
prever o erro é maior que a previsão, deixando o grau de previsão
igual ao grau de acerto de um chute, não resta duvida que é bem
comparável com a astrologia, que segundos os seus criticos tem um
poder de previsão igualmente escasso.

Mas vc é livre de demonstrar que a meteorologia tem um grau de
previsão maior do que aquele que estou referindo.

Quanto ao sistema de alerta civil ele é apenas uma resposta e não uma
previsão. Quando a defesa civil é alertada de um sismo é pq o
sismografo apresentou um sinal, o que só acontece DEPOIS que o sismo
começa. O sismo pode vir e ir tão rápido e tão forte quanto a natureza
quiser ,e nesses casos não ha nada a fazer. A única benece do sistema
de aviso é quando o sismo principal, é precedido de pequenos outros
sismos e ai dá tempo de avisar - o que raramente acontece. Na erupção
de vulcões o sismografo é mais valioso, já que os movimentos da terra
não se devem ao mesmo tipo de fenomeno e os abalos representam
mudanças de estado do vulcão, que podem informa da sua actividade.
Contudo, não é possivel prever quando, ou se, o vulcão vai entrar em
erupção. Ha boas chances que ele entre em erupção se certos niveis
forem atingidos, mas mesmo ai, ha um erro a considerar. O vulcaõ pode
muito bem erucionar de uma hora para outra sem aviso. Portanto, não
se iludam que os sistema de protecção civil estão prevendo seja o que
for. Eles estão apenas respondendo a coisas que já aconteceram. O
truque é apenas decifrar sinais que nos dão a informação do
acontecimento, a tempo de responder. E isso depende muito de onde
acontece o fenomeno.
No caso dos tsunamis e terremotos, tudo depende do epicentro, que pode
ser longe ou perto. E o que é perto de uns é longe de outros.
Quanto a furacões, não se conhece nenhuma informação ou modelo que
possa prever quando e onde eles se vão formar. O máximo que podemos
fazer é manter o olho nas nuvens com os satélites e ver quando se
formam, mas mais uma vez será uma resposta a um acontecimento e não
uma previsão.


Sérgio Taborda










SUBJECT: Fw: Ricardo Juliao
FROM: "Luiz Ferraz Netto" <leobarretos@uol.com.br>
TO: "ciencialist" <ciencialist@yahoogrupos.com.br>
DATE: 11/02/2005 09:33

Nossa! Hoje a caixa postal tá que tá ...
[]'
===========================
Luiz Ferraz Netto [Léo]
leobarretos@uol.com.br
http://www.feiradeciencias.com.br
===========================
-----Mensagem Original-----
De: Ricardo Julião
Para: Luiz Ferraz Netto
Enviada em: quinta-feira, 10 de fevereiro de 2005 11:15


olá primeiro gostaria q me dissece algo sobre os materiais q são repelidos pelos imãs.

e também como faço para medir com precisao o peso de algo.


--------------------------------------------------------------------------------


No virus found in this incoming message.
Checked by AVG Anti-Virus.
Version: 7.0.300 / Virus Database: 265.8.7 - Release Date: 10/02/2005

----------

No virus found in this outgoing message.
Checked by AVG Anti-Virus.
Version: 7.0.300 / Virus Database: 265.8.7 - Release Date: 10/02/2005


[As partes desta mensagem que não continham texto foram removidas]



SUBJECT: Fw: Efeito Hublle
FROM: "Luiz Ferraz Netto" <leobarretos@uol.com.br>
TO: "ciencialist" <ciencialist@yahoogrupos.com.br>
DATE: 11/02/2005 09:34

que tá .....

[]'
===========================
Luiz Ferraz Netto [Léo]
leobarretos@uol.com.br
http://www.feiradeciencias.com.br
===========================
-----Mensagem Original-----
De: "netovilla" <netovilla@ig.com.br>
Para: <leobarretos@uol.com.br>
Enviada em: quinta-feira, 10 de fevereiro de 2005 11:24
Assunto: Efeito Hublle


Bom dia!! Gostaria de ter disponivel algum material sobre o efeito acima.
Grato.



--------------------------------------------------------------------------------


No virus found in this incoming message.
Checked by AVG Anti-Virus.
Version: 7.0.300 / Virus Database: 265.8.7 - Release Date: 10/02/2005



--
No virus found in this outgoing message.
Checked by AVG Anti-Virus.
Version: 7.0.300 / Virus Database: 265.8.7 - Release Date: 10/02/2005



SUBJECT: Re: Astrologia e Metereologia e top-posting (era : Zodiaco)
FROM: "rmtakata" <rmtakata@altavista.net>
TO: ciencialist@yahoogrupos.com.br
DATE: 11/02/2005 09:43


--- Em ciencialist@yahoogrupos.com.br, "Sergio M. M. Taborda"
> Uma vez feitos os calculos tem validade de 8 a 15 dias,
> sendo que o erro estimado aumenta com o prazo de validade

No dia em q. astromancia conseguir um grau de acerto da meteorologia
eu passo a acreditar nela (astromancia). A meteorologia consegue taxas
de 95% de acerto para dois dias. (A astromancia consegue taxas de 100%
de acerto para dois dias... depois do ocorrido.)

Ateh lah, em uma lista como esta, ela vale mais como curiosidade
historica, estudo das crencas humanas e quetais.

[]s,

Roberto Takata





SUBJECT: Fw: Esclarecimento
FROM: "Luiz Ferraz Netto" <leobarretos@uol.com.br>
TO: "ciencialist" <ciencialist@yahoogrupos.com.br>
DATE: 11/02/2005 09:57

Já preparei o texto básico da resposta --- mas se alguém tiver sugestão, irei incluir.
[]'
===========================
Luiz Ferraz Netto [Léo]
leobarretos@uol.com.br
http://www.feiradeciencias.com.br
===========================
-----Mensagem Original-----
De: eao.chimoio
Para: leobarretos@uol.com.br
Enviada em: sexta-feira, 11 de fevereiro de 2005 07:44
Assunto: Esclarecimento


Saudações!

Chamo-se Luís Daissone Salém, Moçambicano e professor médio de Matemática e Física, formado na ex-Faculdade de Educação na Universidade Eduardo Mondlane em Maputo.

Quando estava a navegar procurando dicas relativas a pressão atmosférica, encontrei muitos experimentos seus, onde um deles continha a nota aue se segue:

"Nota importante:
Nunca represente as pressões, em suas ilustrações, mediante 'setas'. Pressão não é grandeza vetorial e sim grandeza escalar. Não existe pressão 'para baixo' ou 'para cima'. Evite fazer gestos com as mãos para caracterizar que 'pressão aperta alguma coisa'. Pressão atmosférica, por exemplo, 'não aperta' ninguém!
Para desabafar, permitam-me dizer que já estou cansado de ver livros de biologia com desenhos de setas, contra as paredes das células, para indicar as pressões em seus pontos. Será que ainda há professores que fazem isso em sala de aula? "

Aguardamos seus comentários: leobarretos@uol.com.br



A minha dúvida é a seguinte:

Como fazer entender o aluno sem o uso de gestos e vectores para monstrar os efeitos da pressão atmosférica?

De facto na fórmula de Pressão temos duas grandezas importantes ; Força e área, onde força é vectorial e área é escalar.





Aguardo pelo comentário mais generalizado para que a minha dúvida fique dissipada.





Um abraço de Luís.











--------------------------------------------------------------------------------


No virus found in this incoming message.
Checked by AVG Anti-Virus.
Version: 7.0.300 / Virus Database: 265.8.7 - Release Date: 10/02/2005

----------

No virus found in this outgoing message.
Checked by AVG Anti-Virus.
Version: 7.0.300 / Virus Database: 265.8.7 - Release Date: 10/02/2005


[As partes desta mensagem que não continham texto foram removidas]



SUBJECT: Fw: furadeira radial
FROM: "Luiz Ferraz Netto" <leobarretos@uol.com.br>
TO: "ciencialist" <ciencialist@yahoogrupos.com.br>
DATE: 11/02/2005 09:57

O que é furadeira radial?
[]'
===========================
Luiz Ferraz Netto [Léo]
leobarretos@uol.com.br
http://www.feiradeciencias.com.br
===========================
-----Mensagem Original-----
De: SN Ltda.
Para: daivemmeue-mailleobarretos@uol.com.br ; leobarretos@uol.com.br
Enviada em: sexta-feira, 11 de fevereiro de 2005 07:46
Assunto: furadeira radial



Nas dúvidas experimentais, por gentileza coloque aqui o endereço da página, isso facilita o confronto. Agradeço. Meu nome é LUIZ FERRAZ NETTO, meu apelido é LÉO e moro em BARRETOS

BOM DIA,

VOCE TEM INFORMAÇOES SOBRE O QUE E FURADEIRA RADIAL?
E QUE ESTOU FAZENDO UM TRABALHO P/ ESCOLA E PRECISO SABER TUDO SOBRE FURADEIRA RADIAL.

HELIO SOUTO.




____________________________________________________
IncrediMail - O mundo do correio eletrônico finalmente desenvolveu-se - Clique aqui


--------------------------------------------------------------------------------


No virus found in this incoming message.
Checked by AVG Anti-Virus.
Version: 7.0.300 / Virus Database: 265.8.7 - Release Date: 10/02/2005

----------

No virus found in this outgoing message.
Checked by AVG Anti-Virus.
Version: 7.0.300 / Virus Database: 265.8.7 - Release Date: 10/02/2005


[As partes desta mensagem que não continham texto foram removidas]



SUBJECT: Foucault / Luz ondas ou corpusculos.
FROM: Hélio Ricardo Carvalho <hrc@fis.puc-rio.br>
TO: ciencialist@yahoogrupos.com.br
DATE: 11/02/2005 10:20


Tenho uma dúvida.

Procurando na internet sobre a vida de Foucault encontrei esta
pérola:

"...Em outras experiências, [Foucault] demonstrou que a luz é
menos veloz na água do que no ar. Tal resultado dava apoio à teoria
ondulatória da luz, que predizia esse comportamento."

http://geocities.yahoo.com.br/saladefisica9/biografias/foucault.htm

Eu já tinha visto isto também em livros de Física (ex.: Serway,
Física 3, pg. 307).

Bem, o som, justamente por ser um "fenômeno ondulatório" é mais
rápido na água que no ar. O fato da luz ter comportamento oposto
deveria significar o fim da teoria ondulatória da luz e não a sua
consagração.

Então por que esta experiência descartou a luz corpuscular??????


Hélio.







SUBJECT: Simetrias
FROM: JVictor <jvoneto@uol.com.br>
TO: "ciencialist@yahoogrupos.com.br" <ciencialist@yahoogrupos.com.br>
DATE: 11/02/2005 10:31

Há algum tempo andei ensaiando algumas informações sobre simetrias.
Hoje, encontrei um site que oferece coisas práticas sobre o assunto, com
desenhos ilustrativos, operações, e outras "bondades". Vale a pena
conferir.


http://www.seara.ufc.br/especiais/fisica/simetria/simetria1.htm

Sds,

Victor.





SUBJECT: Uma mente brilhante - o filme
FROM: "E m i l i a n o C h e m e l l o" <chemelloe@yahoo.com.br>
TO: <ciencialist@yahoogrupos.com.br>, <forum-ciencia@yahoogrupos.com.br>
DATE: 11/02/2005 11:00

Olá amigos,

Estou escrevendo um artigo no qual vou citar uma passagem do cinema. O
filme é "Uma mente brilhante", o qual conta uma história do matemático John
Nash e suas descobertas que influenciaram diversas áreas do conhecimento (eu
recomendo).

No filme, há uma cena muito interessante. O matemático, sentado em uma
mesa de bar, procura freneticamente uma idéia brilhante para guiar sua tese
de, se não me engano, doutorado. Com um monte de artigos, anotações,
calculadoras e outras coisas, ele estudava enquanto seus colegas jogavam
bilhar e bebiam cerveja. Nisso entram três garotas, duas menos bonitas e
uma, em especial, belíssima. É neste momento do filme que John Nash tem o
insight sobre o tema de sua tese.

A minha pergunta é simples: Esta cena aconteceu realmente? Há alguma
bibliografia de Nash que diga que a cena aconteceu realmente?

Aguardo contribuições.

[ ] 's do Emiliano Chemello
emiliano@quimica.net
http://www.quimica.net/emiliano
http://www.ucs.br/ccet/defq/naeq

" Rien ne se perd, rien ne se crée,
tout se transforme."

Antoine Laurent de Lavoisier (químico francês, 1743 - 1794)




SUBJECT: Re: [ciencialist] Uma mente brilhante - o filme
FROM: "Alvaro Augusto \(E\)" <alvaro@electraenergy.com.br>
TO: <ciencialist@yahoogrupos.com.br>
DATE: 11/02/2005 11:39

Caro Emiliano,

Provavelmente não aconteceu. Isso não quer dizer que a cena não seja válida, até mesmo porque, tirando Jennifer Connely, aquela cena é uma das poucas que vale a pena no filme (o resto é alucinação hollywoodiana!).

Para tirar a dúvida, seria interessante uma consulta ao livro que deu origem ao filme.

[ ]s

Alvaro Augusto


----- Original Message -----
From: E m i l i a n o C h e m e l l o
To: ciencialist@yahoogrupos.com.br ; forum-ciencia@yahoogrupos.com.br
Sent: Friday, February 11, 2005 11:00 AM
Subject: [ciencialist] Uma mente brilhante - o filme


Olá amigos,

Estou escrevendo um artigo no qual vou citar uma passagem do cinema. O
filme é "Uma mente brilhante", o qual conta uma história do matemático John
Nash e suas descobertas que influenciaram diversas áreas do conhecimento (eu
recomendo).

No filme, há uma cena muito interessante. O matemático, sentado em uma
mesa de bar, procura freneticamente uma idéia brilhante para guiar sua tese
de, se não me engano, doutorado. Com um monte de artigos, anotações,
calculadoras e outras coisas, ele estudava enquanto seus colegas jogavam
bilhar e bebiam cerveja. Nisso entram três garotas, duas menos bonitas e
uma, em especial, belíssima. É neste momento do filme que John Nash tem o
insight sobre o tema de sua tese.

A minha pergunta é simples: Esta cena aconteceu realmente? Há alguma
bibliografia de Nash que diga que a cena aconteceu realmente?

Aguardo contribuições.

[ ] 's do Emiliano Chemello
emiliano@quimica.net
http://www.quimica.net/emiliano
http://www.ucs.br/ccet/defq/naeq

" Rien ne se perd, rien ne se crée,
tout se transforme."

Antoine Laurent de Lavoisier (químico francês, 1743 - 1794)




[As partes desta mensagem que não continham texto foram removidas]



SUBJECT: Re: [ciencialist] Foucault / Luz ondas ou corpusculos.
FROM: JVictor <jvoneto@uol.com.br>
TO: ciencialist@yahoogrupos.com.br
DATE: 11/02/2005 11:47

Hélio Ricardo Carvalho escreveu:

>
> Tenho uma dúvida.
>
> Procurando na internet sobre a vida de Foucault encontrei esta
> pérola:
>
> "...Em outras experiências, [Foucault] demonstrou que a luz é
> menos veloz na água do que no ar. Tal resultado dava apoio à teoria
> ondulatória da luz, que predizia esse comportamento."
>
> http://geocities.yahoo.com.br/saladefisica9/biografias/foucault.htm
>
Victor: Não há contradição. A natureza dos tipos de ondas são
diferentes. As ondas sonoras agem sobre a matéria, comunicando sua
energia, molécula a molécula, que vibram, e fazem as seguintes
"tremerem" na mesma frequência, coisa que acontece tão mais rápido
quanto mais rígido seja o meio. É por isso não existe corpo
perfeitamento rígido, pois as propagações poderiam atingir velocidas
infinitas. Mas Einstein disse: non, non, non, a esse desastre! As
luminosas, não. Elas são de origem eletromagnética e o e-mail é que age
sobre elas, dizendo "stop, peraí, cara, por aquí, é mais devagar." Até
os modos de propagação são diferentes. Uma anda "atravessada", como se
fosse de bandinha, a outra, na "direção do nariz".
Quer dizer, o meio, em vista de suas características, atenua as OEM. E a
luz, sendo uma tal, não escapa. O comportamento do meio perante as duas
é como a justiça brasileira: depende da cara e do status. Mais este do
que aquela..

Aproveito o ensejo para contar uma engraçada "molecagem" que o físico
J.Perrin fez usando um giroscópio, um genial e extraordinário invento
de Foulcault, indispensável na navegação marítima e aérea, sendo um
equipamento cujo funcionamento e efeitos, ainda hoje, me deixam perplexo
e admirado! Conta-se que Perrin instalou dentro de uma mala comum, de
viagem, um desses deuses da mecânica, pondo-o a funcionar. E foi para a
estação de trem. No caminho, contratou um chapeado para conduzir, pela
mão, "sua bagagem", e ficou atrás, observando. Enquanto o pobre
carregador andava em linha reta, tudo bem. Mas, ao tentar virar numa
esquina, em direção à estação, a maleta "recusou-se" a virar com ele,
dando-lhe aqueles safanões que a gente já viu, brincando com um
protótipo, feitos até mesmo com uma roda de bicleta. Conta-se que o
pobre homem arregalou os olhos, jogou a maleta no chão, e se mandou,
num baita pernas prá que te quero, e gritando: é o demônio, é o demônio,
socorro (ou coisa assim). Perrin deve ter rido a valer, com a reação e
o susto de seu contratado. Gênio também é gente como a gente.

Sds,

Victor.



SUBJECT: Re:[ciencialist] Re: Astrologia e Metereologia e top-posting (era : Zodiaco)
FROM: "cumullosnimbos" <cumullosnimbos@bol.com.br>
TO: "ciencialist" <ciencialist@yahoogrupos.com.br>
DATE: 11/02/2005 12:14

Em ciencialist@yahoogrupos.com.br, “Sérgio Taborda”- escreveu:

Fragmentos do texto:

“A metereologia como uma ciencia baseada em modelos é um fracasso.”

“Então, dado o fraco grau de previsão da meteorologia, que precisa de
observação directa (os satelites são só isso e não uma avanço da
metereologia )para tirar alguma conclusão, e que quando se poe a
prever o erro é maior que a previsão, deixando o grau de previsão
igual ao grau de acerto de um chute, não resta duvida que é bem
comparável com a astrologia, que segundos os seus criticos tem um
poder de previsão igualmente escasso.”

“A metereologia é importante , por exemplo, para a
agricultura, mas se houvesse realmente um conhecimento de causa, e um
previsão factual não existiriam desastres agriculas, em que uma geada,
uma chuva ou o calor destroem toda a colheita.”

“Quanto ao sistema de alerta civil ele é apenas uma resposta e não uma
previsão.”

“Quanto a furacões, não se conhece nenhuma informação ou modelo que
possa prever quando e onde eles se vão formar.”
-------------

Tenho a impressão, que este debate não vai chegar em lugar algum de qualquer forma defendo mais uma vez a posição da Meteorologia. Eu sempre ouvi dizer que os Físicos de um modo geral julgam a Meteorologia com ciência de segundo escalão e estou confirmando isto neste tópico de discussão. Mas só por curiosidade, este sentimento é geral?


A modelagem atmosférica do tempo não é um fracasso, isto é fato! Como já foi relatado por outros membros da lista, hoje em dia se consegue uma previsão com um grau de acerto considerável para até 7 dias, logicamente, a precisão dos resultados e decem com o avanço do tempo. O campo de modelagem atmosférica que ainda não consegue bons resultados, é o de previsão climática, fazer hoje uma previsão para sete, dois anos é ilusão não vai corresponder a realidade.
A previsão a partir da observação direta é usada concomitantemente os resultados da previsão numérica, e desta forma corroborar o resultados e aumentar o grau da previsão do tempo. A Meteorologia para realizar as suas previsões parte de princípios físicos de interação termodinâmica e dinâmica dos fluidos geofísicos do sistema “Terra-Oceano-Atmosfera” e a Astrologia correlaciona a posição dos astros com vida a de um ser humano, não se baseia em método científico para que se possam estabelecer estas correlações, então fazer uma comparação como esta é uma agressão muito grande, a Meteorologia ainda vai avançar bastante, e a Astrologia algum dia vai conseguir algum resultado realístico?.
O SIMEPAR, realiza no período mais crítico da ocorrência de geada um sistema especial de alerta que é muito respeitado por quem o solicita, é consultado principalmente pelos plantadores de café deste estado e obtém excelentes resultados.
Nos EUA onde o sistema de previsão do tempo é extremamente bem equipado e dispõe de uma densa rede de estações meteorológicas de superfície, para vocês terem uma idéia, eles dispõem até de redes de observação em mesoescala (A mesoescala atmosférica varia entre 2km a 2000km segundo a classificação de Orlanski, 1975), ou seja, eles dispõem de um sistema de alerta muito poderoso, e são capazes de prever com uma exatidão absurda a ocorrência de um tornado, e desta forma são emitidos alertas que permitem a salvaguarda de milhares de pessoas que ao contrário do Brasil acreditam piamente nas previsões. Graças ao sistemas de previsão de Santa Catarina durante a ocorrência do divulgado Furacão Catarina, graças a um alerta emitido a defesa civil foi capaz de realizar a evacuação da área atingida e, desta forma, não houve perdas humanas. Um furacão não pode ser previsto por um modelo numérico? Lógico que pode, os furacões de forma generalista são ondas de Rossby que se desprendem de um “duto de ondas” conhecido como corrente de jato, que ao encontrarem condições ambientais favoráveis, principalmente a temperatura da superfície do mar um pouco elevada podem se intensificar e originar um furacão. Este duto de ondas se encontram na região de máximo gradiente térmico da massas de ar, ou seja, ao longo da banda de nebulosidade de um sistema frontal e como estes sistemas são perfeitamente identificáveis em modelos também os furacões também são.

Saudações Meteorlógicas,
Daniel.

__________________________________________________________________________
Acabe com aquelas janelinhas que pulam na sua tela.
AntiPop-up UOL - É grátis!
http://antipopup.uol.com.br/




[As partes desta mensagem que não continham texto foram removidas]



SUBJECT: Re: Foucault / Luz ondas ou corpusculos.
FROM: "Sergio M. M. Taborda" <sergiotaborda@terra.com.br>
TO: ciencialist@yahoogrupos.com.br
DATE: 11/02/2005 12:52


--- Em ciencialist@yahoogrupos.com.br, Hélio Ricardo Carvalho
<hrc@f...> escreveu


> Bem, o som, justamente por ser um "fenômeno ondulatório" é mais
> rápido na água que no ar. O fato da luz ter comportamento oposto
> deveria significar o fim da teoria ondulatória da luz e não a sua
> consagração.
>
> Então por que esta experiência descartou a luz corpuscular??????

Quando os textos se referem a isso como "o fim da teoria corpuscular"
deve-se sempre ler : "o fim da teoria corpuscular de Newtow".
Newton, baseado na ideia dos corpusculos derivou o resultado que dizia
que a velocudade da luz na agua deveria ser maior que a do ar.
Como isso não ocorre, a ideia de newton foi invalidade, e com ela toda
a teoria de newton. Digamos que foi uma pequena contadição que era
tudo o que os apoiantes da teoria precisavam para mudar para a teoria
ondulatória.

Na realidade , nenhuma das duas tem a ver com o facto real, mas a
teoria ondulatoria preve uma velocidade menor na água (ou em qq outro
meio que não o vácuo).
A velocidade da luz depende de abssorções e emissões em cadeia que não
existem no vácuo - por definição de vácuo - e que nos materiais
atrazam a propagação da luz.

A luz e o som são muito diferentes, pq no caso do som, a vibração é
transportada ao longo do material, pelo movimento do ppr material
(contrações e dilatações do ppr material). No cado da luz, a
propagação é a cosntance absorção e emissão de luz.
Isto casa muito bem com a hipotese de Huygens (cada ponto da frende de
onda é ele mesmo uma fonte de luz) e por isso a descrição ondulatoria
casa tão bem com a experiencia. A teoria ondulatoria explica tb a
famaosa experiencia da dupla fenda de Young, mas falha para explicar
todo o tipo de fenomenos quanticos: o efeito foto-electrico e a não
catastrofe do violeta. Coisas que apenas podem ser explicadas (como
eisntein demonstrou) se ignorarmos as equações de Maxwell e portanto,
a otica e a teoria ondulatoria. (ou seja, usando a fisica quantica)

A teoria ondulatoria - toda a otica - é uma teoria macroscopica, ou
seja, ela vale quando o fenomeno é observado ao longo de grandes
periodos de tempo e/ou ao longo de longos intervalos espaciais. MAS,
não é válida ao nivel mais fundamental.

Normalmente diz-se que a luz é uma fenomeno ondulatorio, e é assim que
nos ensinam a pensar na escola, mas isso não é realmente verdade. Só é
verdade em determinadas circunstancias, que são aquelas que usamos
quando nos ensinam otica. Para as outras, e em geral, a luz é uma
fenomeno corpuscular.

Sérgio Taborda


P.S. Quando eisntein pensou o problema ele criou o fotão como sendo a
PARTICULA de luz. Hoje chamamos fotão , indestintivamente à particua e
ao quantum de energia. Embora a teoria dos quanta fosse precedente,
Eisntein chegou à energia do fotão sem usar a teoria dos quantum, ou
seja, usando pura logica newtoniana (a que supostamente caiui com a
experiencia de Foucault). Mas hoje, toda a gente resiste a afirmar que
a luz é um efeito corpuscular, mesmo embora o efeito foto-electrico
demonstre isso, assim como outras coisas. Eisntein ganhou o prémio
nobel pelo seu trabalho em efeito fotoelectrico exactamente porque sem
a teoria corpuscular não é possivel explicar o fenomeno. Contudo,
Einstein é conhecido pela sua teoria da relatividade, que deriva das
equações de maxwell, que os estudos do corpo negro provaram não serem
válida em geral (o que significa que a relatividade não é válida em
geral, e tanto quanto sabemos , apenas o é para efeitos
macroscopicos.) Mas Dirac, usando ambas as teorias, chegou na teoria
da anti-matéria, que tanto quanto sabemos é válida. Ou seja, uma
teoria errada + uma teoria certa = uma nova teoria certa. É a
maravilha da historia da ciencia.







SUBJECT: Re: Astrologia e Metereologia e top-posting (era : Zodiaco)
FROM: "Sergio M. M. Taborda" <sergiotaborda@terra.com.br>
TO: ciencialist@yahoogrupos.com.br
DATE: 11/02/2005 12:54


--- Em ciencialist@yahoogrupos.com.br, "rmtakata" <rmtakata@a...> escreveu
>
> --- Em ciencialist@yahoogrupos.com.br, "Sergio M. M. Taborda"
> > Uma vez feitos os calculos tem validade de 8 a 15 dias,
> > sendo que o erro estimado aumenta com o prazo de validade
>
> No dia em q. astromancia conseguir um grau de acerto da meteorologia
> eu passo a acreditar nela (astromancia). A meteorologia consegue taxas
> de 95% de acerto para dois dias.

Ha dados para sustentar essa afirmação ou vc está chutando numeros ?
Se ha dados, por favor apresente as fontes.


Sérgio Taborda





SUBJECT: Re:[ciencialist] Re: Astrologia e Metereologia e top-posting (era : Zodiaco)
FROM: "Sergio M. M. Taborda" <sergiotaborda@terra.com.br>
TO: ciencialist@yahoogrupos.com.br
DATE: 11/02/2005 13:09


--- Em ciencialist@yahoogrupos.com.br, "cumullosnimbos"
<cumullosnimbos@b...> escreveu
> Em ciencialist@yahoogrupos.com.br, "Sérgio Taborda"- escreveu:
>
> Fragmentos do texto:
>
> "A metereologia como uma ciencia baseada em modelos é um fracasso."
>
> "Então, dado o fraco grau de previsão da meteorologia, que precisa de
> observação directa (os satelites são só isso e não uma avanço da
> metereologia )para tirar alguma conclusão, e que quando se poe a
> prever o erro é maior que a previsão, deixando o grau de previsão
> igual ao grau de acerto de um chute, não resta duvida que é bem
> comparável com a astrologia, que segundos os seus criticos tem um
> poder de previsão igualmente escasso."
>
> "A metereologia é importante , por exemplo, para a
> agricultura, mas se houvesse realmente um conhecimento de causa, e um
> previsão factual não existiriam desastres agriculas, em que uma geada,
> uma chuva ou o calor destroem toda a colheita."
>
> "Quanto ao sistema de alerta civil ele é apenas uma resposta e não uma
> previsão."
>
> "Quanto a furacões, não se conhece nenhuma informação ou modelo que
> possa prever quando e onde eles se vão formar."
> -------------
>
> Tenho a impressão, que este debate não vai chegar em lugar algum de
qualquer forma defendo mais uma vez a posição da Meteorologia. Eu
sempre ouvi dizer que os Físicos de um modo geral julgam a
Meteorologia com ciência de segundo escalão e estou confirmando isto
neste tópico de discussão. Mas só por curiosidade, este sentimento é
geral?
>
>
> A modelagem atmosférica do tempo não é um fracasso, isto é fato!
Como já foi relatado por outros membros da lista, hoje em dia se
consegue uma previsão com um grau de acerto considerável para até 7
dias, logicamente, a precisão dos resultados e decem com o avanço do
tempo. O campo de modelagem atmosférica que ainda não consegue bons
resultados, é o de previsão climática, fazer hoje uma previsão para
sete, dois anos é ilusão não vai corresponder a realidade.

Portanto, o que eu disse , não é nenhuma mentira.

> A previsão a partir da observação direta é usada concomitantemente
os resultados da previsão numérica, e desta forma corroborar o
resultados e aumentar o grau da previsão do tempo. A Meteorologia para
realizar as suas previsões parte de princípios físicos de interação
termodinâmica e dinâmica dos fluidos geofísicos do sistema
"Terra-Oceano-Atmosfera" e a Astrologia correlaciona a posição dos
astros com vida a de um ser humano, não se baseia em método científico
para que se possam estabelecer estas correlações,

Poias não. Mas sendo que a Astrologia não é uma ciencia, tudo bem que
ela não faça isso. A metereologia, que se afirma uma ciencia, TEM
OBRIGAÇÂO de fazer isso. Portanto, o que vc disse nada mais é do que o
obvio.

> O SIMEPAR, realiza no período mais crítico da ocorrência de geada um
sistema especial de alerta que é muito respeitado por quem o solicita,
é consultado principalmente pelos plantadores de café deste estado e
obtém excelentes resultados.

Otimo. Então diga-me, quando e onde vão ocurrer as proximas 5 geadas
graves.

> Nos EUA onde o sistema de previsão do tempo é extremamente bem
equipado e dispõe de uma densa rede de estações meteorológicas (...)
eles dispõem de um sistema de alerta muito poderoso, e são capazes de
prever com uma exatidão absurda a ocorrência de um tornado,

Defina "exactidão absurda" , tlv mostrando alguns numeros para a gente
ter uma ideia. O erro é de quantos quilometros e quantas horas ?


>e desta forma são emitidos alertas que permitem a salvaguarda de
>milhares de pessoas

Se o furacão se forma sobre a jamaica obivamente ele vai demorar a
chegar ao texas. Esse tipo de previsão não é muito espantosa. Estamos
a falar de previsão , e isso significa SABER ANTES, ou seja, se o
furacao se forma sobre a jamaica, quanto tempo antes os jamaicanos
saberiam disso ?

Por outro lado, se o sistema é tão bom, onde e quando será o próximo
furacao ?

> Graças ao sistemas de previsão de Santa Catarina durante a
>ocorrência do divulgado Furacão Catarina, graças a um alerta emitido
>a defesa civil foi capaz de realizar a evacuação da área atingida e,
>desta forma, não houve perdas humanas.

Onde se formou o furacao e qual a distancia à area evacuada. Qual era
a velocidade media do furação ?

Um furacão não pode ser previsto por um modelo numérico? Lógico que
pode, os furacões de forma generalista são ondas de Rossby que se
desprendem de um "duto de ondas" (...)

O que vc disse é a explicação para o COMO se forma um furacao, mas não
demosntra que haja um modelo numerico para ele, nem que os seus
resultados sejam bons. Queira reformular a sua apresentação. Se ha um
modelo numerico, qual é (o algoritmo deve ter um nome) e qual o seu
grau de erro em horas e quilometros.


>(...)Este duto de ondas se encontram na região de máximo gradiente
>térmico da massas de ar, ou seja, ao longo da banda de nebulosidade
>de um sistema frontal e como estes sistemas são perfeitamente
>identificáveis em modelos também os furacões também são.

Identificável é diferente de previsivel. Identificação presupõe que o
objectos já existe. Previsão presupoe que o objecto ainda não existe.
Previsão é saber se a frente vai degenerar num furacão ou não. Depois
que degenera já não é previsão, é constatação de um facto.
E é capacidade de prever , e não a capacidade de identificar, de que
estamos falando.

Sérgio Taborda






SUBJECT: Re: [ciencialist] Megatronica ? Ou Mecatronica ?
FROM: "L.E.R.de Carvalho" <lecarvalho@infolink.com.br>
TO: ciencialist@yahoogrupos.com.br
DATE: 11/02/2005 13:55

At 07:11 11/2/2005, you wrote:
>Não consegui achar onde está a sugestão nessa mensagem!
>[]'



A sugestão era prestar atenção na veracidade da pergunta.
Um estudante de Mecatrônica...
escreveria Megatrônica ?

L.E.










> ===========================
> Luiz Ferraz Netto [Léo]
> leobarretos@uol.com.br
> <http://www.feiradeciencias.com.br>http://www.feiradeciencias.com.br
> ===========================
>-----Mensagem Original-----
>De: "L.E.R.de Carvalho" <lecarvalho@infolink.com.br>
>Para: <ciencialist@yahoogrupos.com.br>
>Enviada em: sexta-feira, 11 de fevereiro de 2005 02:48
>Assunto: [ciencialist] Megatronica ? Ou Mecatronica ?
>
>
>
>At 10:07 10/2/2005, you wrote:
> >Alguém mais 'moderno' que eu pode definir isso no contexto da atualidade?
> >[]'
> > ===========================
> > Luiz Ferraz Netto [Léo]
> > leobarretos@uol.com.br
> >
> <<http://www.feiradeciencias.com.br>http://www.feiradeciencias.com.br>http://www.feiradeciencias.com.br
> > ===========================
> >-----Mensagem Original-----
> >De: Santa Casa
> >Para: leobarretos@uol.com.br
> >Enviada em: quarta-feira, 9 de fevereiro de 2005 12:13
> >Assunto: O QUE É INSTRUMENTAÇÃO E CONTROLE DE PROCESSO
> >
> >
> >
> >Nas dúvidas experimentais, por gentileza coloque aqui o endereço da
> >página, isso facilita o confronto. Agradeço. Meu nome é LUIZ FERRAZ NETTO,
> >meu apelido é LÉO e moro em BARRETOS; dai vem meu e-mail:
> >leobarretos@uol.com.br.
> >
> >
> >BOM DIA ..LEO
> >
> >SOU ALUNO DE MEGATRONICA E ESTOU EM BUSCA DE DEFINICOES CONCRETA A
> >RESPEITO DO TEMA:
> >
> >1- O QUE INSTRUMENTAÇÃO?
> >2- O QUE É CONTROLE DE PROCESSO
> >
> >AGRADEÇO SUA AJUDA.
> >
> >OBRIGADO
> >CLEBER
> >
> >
> >-------------------------------------------------------------------------
> -------
> >
> >
> >No virus found in this incoming message.
> >Checked by AVG Anti-Virus.
> >Version: 7.0.300 / Virus Database: 265.8.6 - Release Date: 07/02/2005
> >
> > ----------
> >
> >No virus found in this outgoing message.
> >Checked by AVG Anti-Virus.
> >Version: 7.0.300 / Virus Database: 265.8.6 - Release Date: 07/02/2005
> >
> >
> >[As partes desta mensagem que não continham texto foram removidas]
>
>
>[As partes desta mensagem que não continham texto foram removidas]
>
>
>
>##### ##### #####
>
>Para saber mais visite
><http://www.ciencialist.hpg.ig.com.br>http://www.ciencialist.hpg.ig.com.br
>
>
>##### ##### ##### #####
>Links do Yahoo! Grupos
>
>
>
>
>
>
>
>
>
>
>--
>No virus found in this incoming message.
>Checked by AVG Anti-Virus.
>Version: 7.0.300 / Virus Database: 265.8.6 - Release Date: 07/02/2005
>
>
>
>
>--
>No virus found in this outgoing message.
>Checked by AVG Anti-Virus.
>Version: 7.0.300 / Virus Database: 265.8.6 - Release Date: 07/02/2005
>
>
>
>##### ##### #####
>
>Para saber mais visite
><http://www.ciencialist.hpg.ig.com.br>http://www.ciencialist.hpg.ig.com.br
>
>
>##### ##### ##### #####
>
>
>Yahoo! Grupos, um serviço oferecido por:
><http://br.rd.yahoo.com/SIG=12ai6uq9v/M=264379.5078783.6203979.1588051/D=brclubs/S=2137111528:HM/EXP=1108199907/A=2191897/R=0/SIG=10vqa2grn/*http://br.diversao.yahoo.com/>
>[]
>
><http://br.rd.yahoo.com/SIG=12ai6uq9v/M=264379.5078783.6203979.1588051/D=brclubs/S=2137111528:HM/EXP=1108199907/A=2191897/R=1/SIG=10vqa2grn/*http://br.diversao.yahoo.com/>
>[]
> São Paulo Rio de Janeiro Curitiba Porto Alegre Belo Horizonte Brasília
>
>
>----------
>Links do Yahoo! Grupos
> * Para visitar o site do seu grupo na web, acesse:
> *
> <http://br.groups.yahoo.com/group/ciencialist/>http://br.groups.yahoo.com/group/ciencialist/
>
> *
> * Para sair deste grupo, envie um e-mail para:
> *
> <mailto:ciencialist-unsubscribe@yahoogrupos.com.br?subject=Unsubscribe>ciencialist-unsubscribe@yahoogrupos.com.br
>
> *
> * O uso que você faz do Yahoo! Grupos está sujeito aos
> <http://br.yahoo.com/info/utos.html>Termos do Serviço do Yahoo!.


[As partes desta mensagem que não continham texto foram removidas]



SUBJECT: Re: Astrologia e Metereologia e top-posting (era : Zodiaco)
FROM: "rmtakata" <rmtakata@altavista.net>
TO: ciencialist@yahoogrupos.com.br
DATE: 11/02/2005 14:10


--- Em ciencialist@yahoogrupos.com.br, "Sergio M. M. Taborda"
> Ha dados para sustentar essa afirmação ou vc está chutando numeros ?
> Se ha dados, por favor apresente as fontes.

Hah dados.

Nesta pagina a taxa eh de 80%:

http://www.weather2000.com/hdd_0304_verif.html

(Em amarelo os acertos)

Vc pode fazer isso pegando as previsoes publicadas nos jornais e
comparando.

Em 1994, a taxa de acerto para 48 horas era de 70%:
Are Forecasters Better Today? Probably. Deseret News, April 3, 1994,
p.B-2

Vc tb pode pedir dados para o pessoal do CPTec:

"C&T – Qual é hoje a percentagem de confiabilidade na previsão
meteorológica no Brasil?

Nobre – A confiabilidade da previsão meteorológica depende muito do
parâmetro a que nós estamos nos referindo. Temperatura, pressão
atmosférica, tem uma alta confiabilidade, por exemplo, as taxas de
acerto da previsão de 24 horas são acima de 90%, por exemplo, a
previsão da temperatura mínima, da temperatura máxima para o dia
seguinte 24 horas, ou mesmo para 48 horas, índices muito altos, acima
de 85%. E essa previsão tem qualidade até o sexto dia. Quando estamos
falando do parâmetro meteorológico chuva os índices não são tão bons,
isso mundialmente. É muito mais difícil prever chuva, porque chuva
não é uma coisa que acontece continuamente; temperatura, pressão,
existe um contínuo, você não pode ter aqui 25 graus e cinco
quilômetros depois 15 graus. Mas se pode ter muita chuva aqui e
nenhuma chuva a 15 quilômetros. Então, chuva tem um aspecto muito
mais descontínuo e muito mais difícil de prever essa descontinuidade.
Desta forma, para chuva os índices de acerto são da ordem de 80% para
as primeiras 24 horas, e a previsão de chuva no Brasil é boa até três
a quatro dias."
http://www.radiobras.gov.br/ct/falaciencia/2002/falaciencia_130902.htm

[]s,

Roberto Takata





SUBJECT: Re: Simetrias
FROM: "rayfisica" <rayfisica@yahoo.com.br>
TO: ciencialist@yahoogrupos.com.br
DATE: 11/02/2005 14:35


--- Em ciencialist@yahoogrupos.com.br, JVictor <jvoneto@u...> escreveu
> Há algum tempo andei ensaiando algumas informações sobre simetrias.
> Hoje, encontrei um site que oferece coisas práticas sobre o
assunto, com
> desenhos ilustrativos, operações, e outras "bondades". Vale a pena
> conferir.
>
>
> http://www.seara.ufc.br/especiais/fisica/simetria/simetria1.htm
>
> Sds,
>
> Victor.
>>>>>>>>>>>>>>>>>>>>>>>>>>>>>>>>>>>>>>>>>>>>>>.
E eu que pensei que o amigo havia esquecido, valeu...





SUBJECT: Re: [ciencialist] Uma mente brilhante - o filme
FROM: "Luiz Ferraz Netto" <leobarretos@uol.com.br>
TO: <ciencialist@yahoogrupos.com.br>, <forum-ciencia@yahoogrupos.com.br>
DATE: 11/02/2005 14:35

> Com um monte de artigos, anotações,
calculadoras e outras coisas, ele estudava enquanto seus colegas jogavam
bilhar e bebiam cerveja. Nisso entram três garotas, duas menos bonitas e
uma, em especial, belíssima. É neste momento do filme que John Nash tem o
insight sobre o tema de sua tese.

A minha pergunta é simples: Esta cena aconteceu realmente? Há alguma
bibliografia de Nash que diga que a cena aconteceu realmente?

Aguardo contribuições. Emiliano Chemello<

Ao fim e ao cabo, o que vc quer é o telefone da garota 'especial' ... com certeza não vai aparecer na bibliografia!
[]'
Léo



--
No virus found in this outgoing message.
Checked by AVG Anti-Virus.
Version: 7.0.300 / Virus Database: 265.8.7 - Release Date: 10/02/2005



SUBJECT: Re: [ciencialist] Foucault / Luz ondas ou corpusculos.
FROM: "Alberto Mesquita Filho" <albmesq@uol.com.br>
TO: <ciencialist@yahoogrupos.com.br>
DATE: 11/02/2005 15:09

----- Original Message -----
From: "Hélio Ricardo Carvalho"
Sent: Friday, February 11, 2005 9:20 AM
Subject: [ciencialist] Foucault / Luz ondas ou corpusculos.

> "...Em outras experiências, [Foucault] demonstrou que a luz é
> menos veloz na água do que no ar. Tal resultado dava apoio à teoria
> ondulatória da luz, que predizia esse comportamento."
> [...]
> Então por que esta experiência descartou a luz corpuscular??????

Olá Hélio

Com efeito, essa história está muito, mas MUITO mal contada. Há até mesmo
quem afirme que essa experiência vai contra a teoria corpuscular da luz de
Newton, o que retrataria uma profunda ingenuidade a respeito do que seria o
falseamento de uma teoria. Obviamente, Newton não sabia tudo sobre a luz, e
cometeu algumas imprudências em seus *palpites* conclusivos mas, por outro
lado, e como grande teorizador que era, não chegou a se utilizar dessas
"hipóteses" na sua teoria. Muitos dentre os "físicos modernos" gostam de
repetir essas bobagens dando a elas uma ênfase inusitada, como se essas
idéias fossem fundamentais para alicerçarem a óptica newtoniana. Creio que
esses palpiteiros nunca leram Newton e nem mesmo Maxwell, pois do contrário
não ficariam por aí iludindo os jovens estudantes com idéias disparatadas
como essas (aliás não é só na Óptica que se flagra essa ignorância sobre o
que seria a genuína física newtoniana).

Comento alguma coisa a esse respeito em
http://ecientificocultural.com/ECC2/artigos/polar02.htm

[ ]´s
Alberto,
retornando após um longo período de férias
http://ecientificocultural.com/indice.htm
Mas indiferentemente a tudo isso, o neutrino tem massa, o elétron não é
uma carga elétrica coulombiana e a Terra se move. E a história se repetirá.



SUBJECT: Astrologia e Meteorologia
FROM: "Oraculo" <oraculo@atibaia.com.br>
TO: <ciencialist@yahoogrupos.com.br>
DATE: 11/02/2005 15:27

Olá

A comparação da astrologia com a meteorologia foi muito infeliz e será difícil sair disso sem descambar para absurdos lógicos.

Em que pese a natural dificuldade de lidar com um enorme número de variáveis, a meteorologia vem aumentando seu conjunto de conehcimentos, e grau de confiabilidade constantemente. Suas previsões, que eram motivo de piada décadas atrás, se tornaram bem mais precisas, e mesmo o Taborda deve consultar um site sobre o tempo antes de viajar de férias..:-)

O uso de computadores mais potentes aumenta a precisão das respostas, o que indica que o mecanismo de ação está razoavelmetne correto (eu diria mais que razoável, mas vamos manter os limites bem baixos..:-). Sua capacidade de determinar efeitos e fazer previsões é o suficiente para prevenir agricultores de geadas, temporais, nevascas ou secas com grande vantagem sobre a alternativa (não consultar a meteorologia). Claro, não 100% das vezes, mas 80 a 90% das vezes já é impressionante e resulta em menor prejuizo para quem interessa, os agricultores.

Assim, com um mecanismo de ação relativamente bem comprendido, com previsões que se tornam a cada dia mais precisas, como fenomenos relacionados com clareza (zonas de pressão, massas de ar, transferencias de calor e clima), tudo na meteorologia indica uma forma de ciência e um conhecimento confiável, ainda que não 100% (que não é o que se espera da ciência, afinal).

A astrologia, por outro lado, não tem mecanismo de ação conhecido ou mesmo sugerido. Mesmo o Taborda reconehce que não só não se sabe como os criadores da astrologia fizeram as relações entre personalidade e posiçòes de astros, como provavelmetne nunca se saiba. O que se afirmou há 3 mil anos, ainda deve ser considerado nas analises. Sem acumulo recente ou meios de comprovar ou testar.

Imagine que a astrologia pudesse acertar, com taxas de 80%, em suas previsões de personalidade. Ou melhor, já que personalidade é algo sutil e elusivo, acertar momentos bons e ruins para investir, se tratar, comprar ou vender. Isso seria facilmente demonstrado e confirmado, já que bastaria um mapa estatistico dos consulentes e seus sucessos..:-)

Também não possui demonstrações de eficácia, a não ser as mesmas que são apresentadas por diversas alegações sobrenaturais - muita gente acredita, está ai a milhares de anos, eu vi com meus próprios olhos e outras do mesmo tipo. Não passa em nenhum estudo de eficácia, não apresenta elementos testáveis ou quando apresenta eles falham, e é preciso acreditar nas relações entre o mapa e a personalidade, já que não se pode veridficar ou examninar o mecanismo de ação ou a precisão do acerto (se houve erro, é devido ao ascendente, descendente, lateral ou aquele asteroide não computado, não foi falha da astrologia..:-).

Finalmente, a meteorologia sempre reconheceu seus limites, e nunca afirma saber mais do que está a seu alcance ou prever com mais acerto que o verificável. Melhora a cada dia e mesmo assim mantém suas previsões dentro de limites claros e específicos. E é um conhecimento partilhavel, ou seja, o que um pesquisador descobre, pode ser usado por outro para melhorar seus dados e resultados. A astrologia, as diversas e inúmeras formas de astrologia, boa parte delas baseada no mesmo conhecimento ancestral, não só não reconhece limites claros como nem mesmo tenta se adequar a suas próprias afirmações. O que afirma um astrologo nem sempre é o que outro diz. Se alguns usam ofiuco, outros o descartam, se alguns levam Plutão em conta, outros dizem que é irrelevante, etc. Embora cada um deles diga que a SUA astrologia é a correta e os outros estão enganados.

Por qualquer angulo, a meteorologia é uma ciência, ainda que imprecisa em seus resultados, mas precisa em seu conhecimento. E a astrologia é imprecisa em seus resultados e ignorante de seus mecanismos de ação. Irreal comparar as duas.

Homero



[As partes desta mensagem que não continham texto foram removidas]



SUBJECT: Re: Uma mente brilhante - o filme
FROM: marcelo ferrari <emailferrari@yahoo.com.br>
TO: ciencialist@yahoogrupos.com.br
DATE: 11/02/2005 16:17

Emiliano, aproveitando que vc está escrevendo este artigo, me diga: vc tem ai a transcrição desta cena do filme. Ou seja, vc tem escrito o raciocinio dele no bar. Se tiver, pode postar.


grato.
marcelo ferrari



E m i l i a n o C h e m e l l o <chemelloe@yahoo.com.br> wrote:Olá amigos,

Estou escrevendo um artigo no qual vou citar uma passagem do cinema. O
filme é "Uma mente brilhante", o qual conta uma história do matemático John
Nash e suas descobertas que influenciaram diversas áreas do conhecimento (eu
recomendo).

No filme, há uma cena muito interessante. O matemático, sentado em uma
mesa de bar, procura freneticamente uma idéia brilhante para guiar sua tese
de, se não me engano, doutorado. Com um monte de artigos, anotações,
calculadoras e outras coisas, ele estudava enquanto seus colegas jogavam
bilhar e bebiam cerveja. Nisso entram três garotas, duas menos bonitas e
uma, em especial, belíssima. É neste momento do filme que John Nash tem o
insight sobre o tema de sua tese.

A minha pergunta é simples: Esta cena aconteceu realmente? Há alguma
bibliografia de Nash que diga que a cena aconteceu realmente?

Aguardo contribuições.

[ ] 's do Emiliano Chemello
emiliano@quimica.net
http://www.quimica.net/emiliano
http://www.ucs.br/ccet/defq/naeq

" Rien ne se perd, rien ne se crée,
tout se transforme."

Antoine Laurent de Lavoisier (químico francês, 1743 - 1794)




##### ##### #####

Para saber mais visite
http://www.ciencialist.hpg.ig.com.br


##### ##### ##### #####


Yahoo! Grupos, um serviço oferecido por:



















function SearchComboBox() { if (document.form_combo.keyword.value.length==0){ alert("Por favor, digite algo."); return false; }else { document.form_combo.action ="http://br.rd.yahoo.com/SIG=12afmp5ao/M=264105.3931087.6562589.1588051/D=brclubs/S=2137111528:HM/EXP=1108213009/A=2361264/R=0/SIG=11uaou2jn/*http://www.bondfaro.com/bondfaro/in/combosearch_in.jsp?sk=11"; } return true;} [input] [input] [input]

---------------------------------
Links do Yahoo! Grupos

Para visitar o site do seu grupo na web, acesse:
http://br.groups.yahoo.com/group/ciencialist/

Para sair deste grupo, envie um e-mail para:
ciencialist-unsubscribe@yahoogrupos.com.br

O uso que você faz do Yahoo! Grupos está sujeito aos Termos do Serviço do Yahoo!.



__________________________________________________
Converse com seus amigos em tempo real com o Yahoo! Messenger
http://br.download.yahoo.com/messenger/

[As partes desta mensagem que não continham texto foram removidas]



SUBJECT: Re: Foucault / Luz ondas ou corpusculos.
FROM: Hélio Ricardo Carvalho <hrc@fis.puc-rio.br>
TO: ciencialist@yahoogrupos.com.br
DATE: 11/02/2005 18:05


--- Em ciencialist@yahoogrupos.com.br, "Alberto Mesquita Filho"
<albmesq@u...> escreveu
> ...
> quem afirme que essa experiência vai contra a teoria corpuscular
da luz de
> Newton, o que retrataria uma profunda ingenuidade a respeito do
que seria o
> falseamento de uma teoria.

Newton, para mim, foi o maior físico de todos os tempos. Mas não era
Deus. Deve ter cometido muitos "erros".
Coloquei erros entre aspas pois estamos analisando a obra de Newton
hoje com recursos impossíveis na época dele. Logo alguns dos "erros"
dele são aceitáveis.

Acho que em meados dos séculos XVIII e XIX era moda derrubar Newton
então qualquer mínimo detalhe era o suficiente.

Veja no texto do Sergio:

>"Quando os textos se referem a isso como "o fim da teoria
>corpuscular" deve-se sempre ler : "o fim da teoria corpuscular de
>Newtow". Newton, baseado na ideia dos corpusculos derivou o
>resultado que dizia que a velocudade da luz na agua deveria ser
>maior que a do ar. Como isso não ocorre, a ideia de newton foi
>invalidade, e com ela toda a teoria de newton. Digamos que foi uma
>pequena contadição que era tudo o que os apoiantes da teoria
>precisavam para mudar para a teoria ondulatória."

Para Victor:
Você disse:
>"Não há contradição. A natureza dos tipos de ondas são diferentes.
>As ondas sonoras agem sobre a matéria, comunicando sua energia,
>molécula a molécula, que vibram, e fazem as seguintes "tremerem" na
>mesma frequência, coisa que acontece tão mais rápido quanto mais
>rígido seja o meio. É por isso não existe corpo perfeitamento
>rígido, pois as propagações poderiam atingir velocidas infinitas.
>...
> As luminosas, não. Elas são de origem eletromagnética e o e-mail é
>que age sobre elas, dizendo "stop, peraí, cara, por aquí, é mais
>devagar." Até os modos de propagação são diferentes. Uma anda
>"atravessada", como se fosse de bandinha, a outra, na "direção do
>nariz". Quer dizer, o meio, em vista de suas características,
>atenua as OEM. E a luz, sendo uma tal, não escapa."

Se possível traduza isto para uma linguagem mais técnica e
microscópica.

Hoje sabemos (eu acho) que índice de refração, permissividade
elétrica do meio e permeabilidade magnética do meio são grandezas
que expressam efeitos macroscópicos que derivam da absorsão e depois
de um pequeno tempo, reemisão de alguma coisa pelos átomos
(elétrons).
Isto sim é um bom argumento para o fim da teoria ondulatória.

[ ]'s
Hélio






SUBJECT: Re: Foucault / Luz ondas ou corpusculos.
FROM: Hélio Ricardo Carvalho <hrc@fis.puc-rio.br>
TO: ciencialist@yahoogrupos.com.br
DATE: 11/02/2005 18:19


Só para complementar a msg anterior.

Acho que na época de FOUCAULT a teoria ondulatória da luz era ONDAS
LONGITUDINAIS NO ETHER. Logo de mesmo tipo que as sonoras.

Quem conhecer bem história da ciência me corrija.

Se isto é verdade, refaço a pergunta original:

Por que esta experiência descartou a luz corpuscular??????

[ ]'s
Hélio

Hélio Ricardo Carvalho escreveu:
> ...
> Para Victor:
> Você disse:
> >"Não há contradição. A natureza dos tipos de ondas são
diferentes.
> >As ondas sonoras agem sobre a matéria, comunicando sua energia,
> >molécula a molécula, que vibram, e fazem as seguintes "tremerem"
na
> >mesma frequência, coisa que acontece tão mais rápido quanto mais
> >rígido seja o meio. É por isso não existe corpo perfeitamento
> >rígido, pois as propagações poderiam atingir velocidas infinitas.
> >...
> > As luminosas, não. Elas são de origem eletromagnética e o e-mail
é
> >que age sobre elas, dizendo "stop, peraí, cara, por aquí, é mais
> >devagar." Até os modos de propagação são diferentes. Uma anda
> >"atravessada", como se fosse de bandinha, a outra, na "direção do
> >nariz". Quer dizer, o meio, em vista de suas características,
> >atenua as OEM. E a luz, sendo uma tal, não escapa."
>
> Se possível traduza isto para uma linguagem mais técnica e
> microscópica.
>
> Hoje sabemos (eu acho) que índice de refração, permissividade
> elétrica do meio e permeabilidade magnética do meio são grandezas
> que expressam efeitos macroscópicos que derivam da absorsão e
> depois
> de um pequeno tempo, reemisão de alguma coisa pelos átomos
> (elétrons).
> Isto sim é um bom argumento para o fim da teoria ondulatória.
>
> [ ]'s
> Hélio





SUBJECT: Re: [ciencialist] Re: Morreu Ernst Mayer , ó bióilogos da lista
FROM: Expedito Barroso <expebarroso@yahoo.com.br>
TO: ciencialist@yahoogrupos.com.br
DATE: 11/02/2005 18:25

Tal como Darwin, q muito aprendeu com Cuvier, Lyell, Buffon, Erasmus, dentre outros, e teve grande disciplina libertária, criativa e curiosa d alguém a quem chamamos de cientista, e formulou sua (nossa) teoria da selção natutral,sintetizando brilhantemente anos de experiências, o biólogo Ernst Mayr extendeu mais para a frente as nebulosas conceituais referente à Evolução orgânica. Seu conceito biológico de espécie é uma daquelas sínteses geniais, que modificaram o conhecimento sobre vida.
rmtakata <rmtakata@altavista.net> wrote:

--- Em ciencialist@yahoogrupos.com.br, Maria Natália
> Morreu Ernst Mayr, o Cientista Que Uniu a Teoria da Selecção Natural
> de Darwin com a Genética de Mendel
> Por CLARA BARATA
> Publico, Sábado, 05 de Fevereiro de 2005


A perda eh lamentavel.

(Uma pequena correcao. Ernst Mayr eh um dos grandes da ciencia - eh,
no presente, ainda q. tenha ocorrido seu passamento; porem ele eh *um*
dos que uniu a teoria da selecao natural de Darwin e Wallace com a
Genetica mendeliana. Na formalizacao matematica destacam-se Fisher,
Haldane e Wright. Na fixacao dos conceitos dentro da pratica
biologica, Mayr se faz notar ao lado de Dobzhansky e Simpson. Dignos
de mencao tb Huxley e Stebbins. Foi um grande trabalho conjunto e nao
um insight genial de um unico cientista.)

[]s,

Roberto Takata





##### ##### #####

Para saber mais visite
http://www.ciencialist.hpg.ig.com.br


##### ##### ##### #####
Links do Yahoo! Grupos











---------------------------------
Yahoo! Acesso Grátis - Internet rápida e grátis. Instale o discador do Yahoo! agora.

[As partes desta mensagem que não continham texto foram removidas]



SUBJECT: Revista macroCOSMO (número 15)
FROM: Luis Brudna <luisbrudna@gmail.com>
TO: ciencialist@yahoogrupos.com.br
DATE: 11/02/2005 18:37

Repassando...

Destaques da edição de fevereiro da Revista macroCOSMO:

Capa: Mecanismos de emissão das ondas de rádio;
Exploração do Espaço: Titã, admirável mundo novo;
Constelações Zodiacais: Câncer

Nesta edição estamos estreamos duas novas seções: a primeira no
comando de Sérgio Caixeta, traz as ultimas noticias da astronomia no
Brasil e no mundo, enquanto que Edgar Smaniotto publica sua resenha
sobre o livro "O Doutor Benignus".

Não seção Astrodúvidas, conheça as câmeras fotográfica Zenit e
entenda a escuridão da noite. Continue também navegando pelo mundo
cybernetico da astrofotografia.

Participe do Primeiro Censo Brasileiro da Astronomia! Ajude-nos a
traçar um mapa da astronomia em nosso país:
http://www.revistamacrocosmo.com/censo.htm

____________________
A revista macroCOSMO é a primeira revista eletrônica brasileira,
destinada exclusivamente para a difusão da astronomia. Com a carência
de publicações nacionais sobre o tema, a macroCOSMO é direcionada
tanto para crianças, como jovens e adultos que se interessam em
aprender um pouco mais sobre essa fascinante ciência.

A revista é mensal e distribuída gratuitamente pela internet, na
versão PDF, através do endereço:

http://www.revistamacrocosmo.com

ou direto
http://www.revistamacrocosmo.com/2005fevereiro/macrocosmo15.pdf


Participe também das nossas comunidades no Yahoo e Orkut:
http://br.groups.yahoo.com/group/macroCOSMO/?yguid=116094424
http://www.orkut.com/Community.aspx?cmm=179572

Hemerson Brandão
Diretor Editor Chefe
Revista macroCOSMO

-------------------------



Até
Luís Brudna


SUBJECT: Re: que tal, convence?
FROM: Hélio Ricardo Carvalho <hrc@fis.puc-rio.br>
TO: ciencialist@yahoogrupos.com.br
DATE: 11/02/2005 18:48


Eu escrevi:
>Na próxima vez falarei da força, do trabalho e da energia
necessária para levar a membrana por este caminho.

CONTINUANDO

O que acontece se deixarmos livre a membrana em qualquer ponto do
percurso?

Se sua espessura for diferente de zero ela não ficará parada. Mas
para onde ela irá?

Por falta de tempo vou responder isto usando o conceito de energia.

A energia armazenada num capacitor é:

U = (1/2)*Q*V

Usando a definição de capacitância temos:

C = Q/V

Então

U = (1/2)*C*V^2

Como vimos antes se a membrana se aproxima do centro (r2 diminui) a
capacitância Ceq aumenta.
Logo a energia acumulada (U) aumenta.

No caso contrário (se afastando do centro), U diminui.

F= -div U
Se você está no 2 grau, ainda não entende a expressão acima. Mas já
conhece uma que sai desta:

Trab = F d

Trabalho é o produto da força pelo deslocamento.
[conhecida nos pré-vestibulares intensivos como: "Trabalho é foda".]

Ela vale em certas condições, não é uma expressão geral.

A força que naturalmente aparecerá na membrana será para afasta-la
do centro.
Para que eu possa levar a membrana até r2=r1 terei que fornecer
energia para o sistema (do arroz e feijão que eu tinha comido antes)
(trabalho positivo). Ao fazer isto, a capacitância aumenta e a carga
também (V= Q/C = constante).

Anteriormente falei que quando a membrana toca a esfera interna as
cargas da superfície da esfera anulam as da superfície interna da
membrana. Não há nenhuma mudança brusca neste momento pois um pouco
antes do toque a capacitância já era próxima do valor durante o
toque.

E quando ela se separa novamente?
Aí eu parei pois quero explicar mantendo o nível até aqui. Para isto
tenho que estudar mais um pouco.
Vou dar uma olhada no livro do Jackson de Eletromagnetismo e depois
te falo porque.

Mas o que acontece é:

A situação volta a ser exatamente como era um pouco antes de se
tocar.

Isto tem relação com o fato de que um GVDG não funciona se o seu
centro estiver conectado com uma bateria colocando uma ddp fixa em
relação ao terra.

E se eu apoiar a membrana deixando ela ir com velocidade constante
até ao máximo (r2=r3-d2) toda energia que eu tinha dado para o
sistema será devolvida para minha mão.

Vamos ver se semana que vem eu já tenho uma resposta melhor!

[ ]'s
Hélio





SUBJECT: RE: [ciencialist] Re: Máquina de fazer gente - Ana Beatriz
FROM: "murilo filo" <avalanchedrive@hotmail.com>
TO: ciencialist@yahoogrupos.com.br
DATE: 11/02/2005 19:47

Mª Nª, oi.
''Porrada''... karatê????
(existe porrada aí?)
Vc está se transformando numa mulher indigesta, Mª Nª!!! :]
Ainda bem que, nos separa, o Atlântico! abr/M.

>From: Maria Natália <grasdic@hotmail.com>
>Reply-To: ciencialist@yahoogrupos.com.br
>To: ciencialist@yahoogrupos.com.br
>Subject: [ciencialist] Re: Máquina de fazer gente - Ana Beatriz
>Date: Thu, 10 Feb 2005 23:11:37 -0000
>
>
>JR:
>
>Como professora e mesmo não havendo em Portugal tais gifts do
>ministério da educação (mas só de papás ricos*) tenho verificado
>coisas esquisitas e parecidas. Presentemente tenho uma turma de 11º
>ano de 24 alunos (16 a 18 anos) e que para laboratórios e práticas
>está dividida ao meio. Verifico que com os alunos tenho um óptimo
>contacto e eles compreendem e gostam da inovação que pratico. Ora 4
>pais do 2º turno entraram em stress e paranóia total de tal modo que
>por volta do fim de 1º período (2 meses de aulas) acharam que eu não
>ia dar o programa e os meninos não conseguiriam fazer o vestibular cá
>da praça. Foi incrível: eu e os alunos numa nice...e fui a 1ª
>professora a terminar a química e que iniciou a física!!! Ficaram
>banzados e ainda mais quando viram que os filhos sabiam a matéria e
>respondendo a questões mais abertas tinham boas classificações!!!
>Estes pais quase me iam matando os alunos do 2º turno! Digo quase
>porque se gerou instabilidade e as notas e aulas deste turno foram
>sempre de menino de "costas quentes" o que veio a dar notas inferiores
>ao 1º turno. Com a mesma professora...
>Eu vi com estes olhos que a terra há-de comer meninos de 5 e 7 anos em
>conferências na Funddação Calouste Gulbenkian sobre Mecânica Quântica
>pelo Paulo Crawford!!!!! Juro que vi meninos em conferências sobre
>buracos negros com 5 anos!!! Nunca pensei que criança fosse vítima de
>tais maus tratos! HORROR! Que nos fizeram as criançinhas, ó criadores
>de génios!?
>Os paizinhos devem ter juizinho e ver se, na sua ânsia de se
>realizarem, através dos filhos, não estão a criar monstros. Deixai o
>trabalho de professores aos profissionais pois os professores também
>não se vão meter nos escritórios ou empregos**. Os tempos são outros.
>Deixai as crianças serem crianças e crescer fazendo erros. Como é bom
>errar, saber que se errou e aprender a lidar com o erro...Qualquer dia
>estamos "a lixar" os nossos filhos, como por exemplo, novamente ao
>impor-lhes noivo/a logo à nascença. Não se riam pois como contacto
>pais há mais de 20 anos tenho assistido a muita estupidez. E dói tanto
>ver crianças de 18/19 anos maltratadas. Me dá vontade de dar um
>enxerto de porrada em tais pais...Ah regressei ás aulas de karaté, pois.
>Deixai vir a mim as criancinhas puras que eu lhes incutirei amor pela
>ciência...
>Amem
>vossos filhos e netos
>Um abraço
>Maria Natália
>PS Conhecem a ensino/educação no Japão?
>*meninos que já vão à ópera aos 3 anos...
>** os professores também são pais (não eunucos)
>
>
>--- Em ciencialist@yahoogrupos.com.br, José Renato <jrma@t...> escreveu
> > Publicado em 10.02.2005
> >
> > DIRETO DA REDAÇÃO
> >
> > MÁQUINA DE FAZER GENTE
> >
> > ANA BEATRIZ
> >
> > No livro Admirável Mundo Novo, de Aldous Huxley, as pessoas de um
>império são criadas em laboratório e pré-condicionadas para certos
>talentos e profissões, através da clonagem e modificação de DNA. Há
>cinco classes sociais, listadas em ordem de habilidade e beleza
>física: Alfa, Beta, Gama, Delta e Epsilon. Nos Estados Unidos, o
>império da vida real, acontece também um certo condicionamento; porém,
>o processo é inciado na infância e não precisa de manobras genéticas.
> >
> >
>
>
>




SUBJECT: Ainda há tempo
FROM: José Renato <jrma@terra.com.br>
TO: <ciencialist@yahoogrupos.com.br>
DATE: 11/02/2005 20:25

Notícias Terra
Quinta, 10 de fevereiro de 2005, 14h45

ONG diz que restam de 10 a 20 anos para salvar a Terra

A comunidade internacional tem de 10 a 20 anos para tentar evitar uma mudança climática devastadora na Terra, advertiu hoje a Rede Ação Clima (RAC), integrada por 340 ONGs de proteção ao meio ambiente. A RAC fez o alerta em Paris durante a abertura de uma série de atos por conta da entrada em vigor do Protocolo de Kioto no dia 16 de fevereiro.
O acordo internacional, não assinado pelos Estados Unidos, prevê a redução de 5,2% das emissões de gases que provocam o efeito estufa pelos países industrializados até 2012. "Para impedir o aumento de dois graus da temperatura média mundial até 2100 e estabelecer uma margem de segurança, é preciso estabilizar a concentração de CO2 em 400 ppm (partes por milhão) frente às 379 ppm atuais. Para alcançar esse objetivo, temos apenas de 10 a 20 anos", afirmou Laetitia Marez, porta-voz da RAC.

Segundo a RAC, as emissões mundiais de gases que provocam o efeito estufa devem alcançar seu patamar mais elevado em 2020, no máximo e, a partir daí, cair pela metade até 2050 para evitar efeitos devastadores para o planeta.

Segundo os especialistas, reunidos na semana passada em Exeter (Reino Unido), um aumento de um grau nas temperaturas terá um impacto destruidor nos corais e dois graus provocarão o derretimento da Groenlândia, o que faria o nível do mar subir consideravelmente.

A curto prazo, são os países industrializados que devem fazer os maiores esforços para adotar novas medidas restritivas a partir de 2012. A RAC considera necessária a redução de 30% das emissões de CO2 até 2020 e de 75% até 2050.

Para a ONG, países emergentes, como Arábia Saudita e Coréia do Sul, deveriam reduzir suas emissões de gases a partir de 2013. Grandes países como China e Índia deveriam, com a ajuda financeira e tecnológica dos países do Norte, devem se comprometer desde já com a produção industrial e energética limpa, segundo a RAC.

EFE

Agência Efe - Todos os direitos reservados. É proibido todo tipo de reprodução sem autorização escrita da Agência Efe S/A.

< http://noticias.terra.com.br/ciencia/interna/0,,OI469817-EI299,00.html >


[As partes desta mensagem que não continham texto foram removidas]



SUBJECT: Re: [ciencialist] Re: Foucault / Luz ondas ou corpusculos.
FROM: "Alberto Mesquita Filho" <albmesq@uol.com.br>
TO: <ciencialist@yahoogrupos.com.br>
DATE: 11/02/2005 20:33

----- Original Message -----
From: "Hélio Ricardo Carvalho"
Sent: Friday, February 11, 2005 5:05 PM
Subject: [ciencialist] Re: Foucault / Luz ondas ou corpusculos.

> Newton, para mim, foi o maior físico de todos os tempos. Mas não era Deus.
> Deve ter cometido muitos "erros". Coloquei erros entre aspas pois estamos
> analisando a obra de Newton hoje com recursos impossíveis na época dele.

Concordo plenamente. Acho apenas que ele não era um teorizador tão ingênuo
quanto determinados físicos "modernos" querem nos fazer acreditar, quiçá por
ignorância (quero crer que seja este o caso), quiçá por má-fé, quiçá devido
ao desespero ao verificarem que a "física moderna" não consegue nos levar a
lugar algum.

> Acho que em meados dos séculos XVIII e XIX era moda derrubar Newton
> então qualquer mínimo detalhe era o suficiente.

A moda passou [hoje a moda é derrubar Einstein :-) --> essa é para mexer com
o Victor ;-)], mas os argumentos se proliferaram em razão direta com
insucessos de natureza heurístico-epistemológica hoje em dia flagrados na
física moderna.

> Veja no texto do Sergio:

> >"Quando os textos se referem a isso como "o fim da teoria
> >corpuscular" deve-se sempre ler : "o fim da teoria corpuscular de
> >Newtow". Newton, baseado na ideia dos corpusculos derivou o
> >resultado que dizia que a velocudade da luz na agua deveria ser
> >maior que a do ar. Como isso não ocorre, a ideia de newton foi
> >invalidade, e com ela toda a teoria de newton.

Aí está o erro, com grande freqüência cometido pelos físicos "modernos" e
comentado pelo Sérgio. Essa idéia, se é que Newton realmente pensava sempre
assim (a obra de Newton é muito grande e com freqüência ele se contradiz,
quando comparamos textos escritos em épocas diversas), provavelmente foi
invalidada, mas isso em nada abala a teoria corpuscular da luz de Newton.
Uma teoria se derruba pelo falseamento de suas hipóteses básicas (e/ou de
predições a apoiarem-se nessas hipóteses básicas), mas não pela não
verificação de uma especulação mal fundamentada, ainda que esta especulação
tenha sido comentada no transcorrer do capítulo conclusivo da teoria.

> Digamos que foi uma
>pequena contadição que era tudo o que os apoiantes da teoria
>precisavam para mudar para a teoria ondulatória."

Isso está parecendo mais política universitária e de muito mau gosto. Enfim,
os universitários provém da mesma fonte social que os políticos de carreira,
logo não é impossível que alguns gostem de politicagem científica.

> Para Victor:

Bem, o restante da mensagem creio que é para o Victor responder, logo vou
parar por aqui.

[ ]´s
Alberto
http://ecientificocultural.com/indice.htm
Mas indiferentemente a tudo isso, o neutrino tem massa, o elétron não é
uma carga elétrica coulombiana e a Terra se move. E a história se repetirá.



SUBJECT: Re: [ciencialist] Re: Foucault / Luz ondas ou corpusculos.
FROM: "Alberto Mesquita Filho" <albmesq@uol.com.br>
TO: <ciencialist@yahoogrupos.com.br>
DATE: 11/02/2005 21:13

----- Original Message -----
From: "Hélio Ricardo Carvalho"
Sent: Friday, February 11, 2005 5:19 PM
Subject: [ciencialist] Re: Foucault / Luz ondas ou corpusculos.

> Só para complementar a msg anterior.
> Acho que na época de FOUCAULT a teoria ondulatória da luz era ONDAS
> LONGITUDINAIS NO ETHER. Logo de mesmo tipo que as sonoras.
> Quem conhecer bem história da ciência me corrija.

A "história" via de regra é contada pelos "vencedores" e a verdadeira
história costuma levar séculos para vir a tona e entrar no domínio acadêmico
e, assim mesmo, quando passível de ser recuperada. Concordo plenamente
contigo ao dizer que a experiência de Foucault fala muito mais a favor de
uma luz corpuscular do que uma luz mecânico-ondulatória. O Victor fala em
tipos diferentes de ondas, mas lembro que na época em que a experiência foi
feita somente existiam ondas mecânicas, a menos que alguém quisesse
ressuscitar o "espírito da matéria" newtoniano. Enfim, entre esse "espírito
da matéria" e as ondas não mecânicas (eletromagnéticas, gravitacionais
etc.), eu sou mais adepto do "espírito da matéria", ainda que utilize uma
nomenclatura mais apropriada aos dias atuais (informação eletromagnética,
gravitacional etc.). A meu ver, luz não é campo. Luz é luz e campo é campo.
Luz é matéria e o campo é imaterial, se bem que tenha sua origem na matéria
e sua ação em outra matéria.

>Se isto é verdade, refaço a pergunta original:
> Por que esta experiência descartou a luz corpuscular??????

Eu diria que não descartou.

Com respeito à luz ser "corpuscular" ou "ondulatória", vale a pena ler o
seguinte trecho, copiado de Einstein e Infeld (A Evolução da Física):

"A história da busca de uma teoria da luz não está de modo algum concluída.
O veredicto do século XIX não foi final e definitivo. Todo o problema de
decidir entre corpúsculos e ondas ainda existe para a Física moderna, desta
vez de uma forma muito mais profunda e intrincada. Aceitemos a derrota da
teoria corpuscular da luz até reconhecermos a natureza problemática da
vitória da teoria ondulatória."

Este texto, escrito há mais de 60 anos, ainda é válido em toda a sua
plenitude.

[ ]´s
Alberto
http://ecientificocultural.com/indice.htm
Mas indiferentemente a tudo isso, o neutrino tem massa, o elétron não é
uma carga elétrica coulombiana e a Terra se move. E a história se repetirá.



SUBJECT: Re: [ciencialist] Re: Foucault / Luz ondas ou corpusculos.
FROM: JVictor <jvoneto@uol.com.br>
TO: ciencialist@yahoogrupos.com.br
DATE: 11/02/2005 21:32


>
>
> > Para Victor:
>
> Bem, o restante da mensagem creio que é para o Victor responder, logo vou
> parar por aqui.'

> >"Não há contradição. A natureza dos tipos de ondas são
> diferentes.
> > >As ondas sonoras agem sobre a matéria, comunicando sua energia,
> > >molécula a molécula, que vibram, e fazem as seguintes "tremerem"
> na
> > >mesma frequência, coisa que acontece tão mais rápido quanto mais
> > >rígido seja o meio. É por isso não existe corpo perfeitamento
> > >rígido, pois as propagações poderiam atingir velocidas infinitas.
> > >...
> > > As luminosas, não. Elas são de origem eletromagnética e o e-mail
> é
> > >que age sobre elas, dizendo "stop, peraí, cara, por aquí, é mais
> > >devagar." Até os modos de propagação são diferentes. Uma anda
> > >"atravessada", como se fosse de bandinha, a outra, na "direção do
> > >nariz". Quer dizer, o meio, em vista de suas características,
> > >atenua as OEM. E a luz, sendo uma tal, não escapa."
> >
> >Helio: Se possível traduza isto para uma linguagem mais técnica e
> > microscópica.

Ok, Ok.
Antes, deixe-me fazer uma correção no texto acima. Onde se lê: ...e o
e-mail é que age...Leia-se: e o meio é que age...
Trocando os miúdos acima em graudos, abaixo.
As ondas sonoras e ondas eletromagnéticas, têm, em si, algo em comum: a
velocidade de cada uma é a mesma em um mesmo meio, para todas as
frequências; mas as velocidades se alteram para cada meio, a luz da
diminuindo(em relação ao seu valor no vácuo) e a do som podendo
aumentar, em relaçao ao seu valor no ar, sendo o valor desta última
tanto maior quanto quanto mais rígido for o novo meio. Num meio rígido e
opaco a luz nem se propaga, enquanto que com uma sonora isto não
acontoce. As ondas sonoras realacionam-se, pois, com o meio material,
de diversas maneiras. No ar, a zero graus centígrados a velocidade é de
331 m/s. No granito, por exemplo, essa velocidade chega a 6000m/s. Já
num meio como a borracha, por exemplo, ela cai para cerca de 54 m/s!
Quer dizer, sua velocidade num meio fica na dependência de como são os
arranjos moleculares desse meio. As ondas sonoras são longitudinais,
enquanto as luminosas são transversais. O meio influe de maneira
diferente sobre a luz e sobre o som. Pode aumentar a velocidade deste e,
sempre, diminuir a da luz. A razão é que as ondas eletromagnéticas têm
uma natureza diferente das ondas sonoras. Aquelas prescindem de um meio
para se propagarem; as sonoras, sem um meio material, simplesmente não
existem, pois nem chegam a se formar. Não se propagam no vácuo,
enquanto a luz acha uma " beleza" correr nele. Num meio como ar, uma
régua vibrando, por exemplo, comunica às moléculas do meio essa energia
mecânica de vibração, estas transmitem o que recebem e, num processo de
compressão e rarefação, as informações vibratórias se propagam.
Fenômenos como refração, reflexão, difração, interferência,batimentos,
mesma velocidade para todas as frequências, são comuns a todas as ondas.
Cada frequência identifica, de modo indubitável, uma dada "faceta" de
uma onda. Numa onda luminosa é uma cor; numa onda sonora, é um tom.
Enquanto se propaga, uma perturbação eletromagnética age como onda;
quando é absorvida, age como se fosse partícula. Com as ondas sonoras,
não. Isto também se deve à natureza de uma e de outra.
Seria a natureza da luz e, por extensão, as OEM, corpuscular, como
ensinava o grande Newton? De acordo com ele, a luz seria constituída de
pequenas partículas que as fontes luminosas emitem, e se propagam no
espaço com enorme velocidade, em lihna reta, espaço a dentro. Num modelo
corpuscular, o conhecido fenômeno da reflexão conduz a informações
iguais às obtidas com um modêlo ondulatório: velocidade de propagação da
onda refletida, absorção de uma parte da luz incidente, verificação de
que o ângulo de incidência é igual ao de reflexão; Newton também
conseguia explicar satisfatoriamente coisas como pressão da luz, a
variação com o inverso do quadrado da intensidade luminosa, e por aí
entre outras coisas. No que tange ao conhecido fenômeno da refração, o
que se observa, experimentalmente, com um modelo corpuscular?
Velocidade da luz no meio de maior índice, maior que a velocidade da luz
no vácuo; explicação: as partículas de luz, ao colidirem com a
superfície da água ficam submetidas a uma atração, uma força normal à
superfície no ponto de contato, obrigando as dita cujas aproximarem-se
da normal e esta seria a causa da refração, da mudança de direção ao
sair de um meio e entrar em outro, diferente. Como resultado desta
atração, as partículas sofreriam uma pequena aceleração na superfície,
senda esta a causa de sua maior velocidade no meio mais denso. Isto é o
que se obtém com um modelo corpuscular, para o fenômeno refração. Newton
explicava também as cores do arco íris dizendo que cada cor
corresponderia ao uma partícula diferente. As partículas de um dado
grupo impressionaria a retina de maneira diferente, produzindo sensações
diferentes. Na época, um outro gigante, mas um gigante menor,
C.Huyghens, físico, discordava do modelo de Newton. Então propôs um
modelo ondulatório, que produzia, para alguns fenônemos,resultados
discordantes, como, por exemplo, no caso da refração. A querela entre os
seguidores de Newton e os de Huyghens, nestas questões, chegou, sem uma
definição, até o comecinho da década de 1860, quando um sujeito
baixinho, meio vesgo, respeitadíssimo no meio científico de então, muito
sério, extremamente inteligente e criativo, aperfeiçoando um método que
Fizeau usara para medir a velocidade da luz anos antes, usou-o para
medí-la na água, encontrando um valor de cerca de 2,23*10^8m/s, contra
os 3,0*10^8 m/s no ar! Ora, este resultado era totalmente contrário ao
que predizia a teoria corpuscular, que garantia uma velocidade maior do
que os 3*10^8 m/s no vácuo!. A teoria corpuscular, logo em um dos
aspectos cruciais do comportamento da luz, velocidade em um meio mais
denso, deixou de " bater" com a experiência. Assim, Foulcault, com suas
medições, ofereceu um veredito incontestável, que conduziu ao abandono
da teoria corpuscular. Mas esta não estava de todo errada, nem está,
apesar das complicações introduzidas, por exemplo, para explicar as
diferentes cores, forças de atração, que são uns ad hoc´s não
pecaminosos, eu acho, como mostrou nosso amigo Einstein, há 100 anos.
Os físicos modernos hoje, usam tanto um como outro, em suas teorias. E a
natureza tem teimado em não mostrar sequer uma evidenciazinha do tamanho
de mísero quark down, que indique que são coisas separadas, sem relação
entre sí, ainda que esta seja esquesita, do tipo disjuntiva. Ou uma ou
outra. Ou dá ou desce. São aspectos diferentes de uma mesma realidade,
que não sabemos qual é. Nem saberemos. Só que é e continuará sendo.
Em suma, a experiência de Foucault foi decisiva no abandono da teoria
corpuscular, na época. Antes de Foulcault era extremamente difícil saber
realmente qual das duas era "a mais correta". Newton não errou, Huyghens
não errou, apenas não estavam preparados para as "cacetadas" postas a
descoberto após 1905. Nem havia os recursos técnicos e científicos que
surgiriam séculos à frente. Aqueles dois eram gênios, Newton à frente de
todos, mas tudo tem seus limites. Portanto...
Viva todos eles, pois!
Espero ter sido um pouco mais técnico e sério, desta vez, embora não
tenha resistido a dizer algumas brincadeiras, mas sem deixar de ser
sério, claro, claro.
Se cometí algum engano, favor corrigir. Estou aquí para aprender e
aprimorar.

Sds,

Victor.




> [ ]´s
> Alberto
> http://ecientificocultural.com/indice.htm
> Mas indiferentemente a tudo isso, o neutrino tem massa, o elétron não é
> uma carga elétrica coulombiana e a Terra se move. E a história se
> repetirá.
>
>
>
> ##### ##### #####
>
> Para saber mais visite
> http://www.ciencialist.hpg.ig.com.br
>
>
> ##### ##### ##### #####
>
>
> *Yahoo! Grupos, um serviço oferecido por:*
> PUBLICIDADE
> <http://br.rd.yahoo.com/SIG=12a19rk78/M=264379.5078783.6203979.1588051/D=brclubs/S=2137111528:HM/EXP=1108244113/A=2332652/R=0/id=noscript/SIG=119058f8i/*http://br.download.yahoo.com/messenger/>
>
>
>
> ------------------------------------------------------------------------
> *Links do Yahoo! Grupos*
>
> * Para visitar o site do seu grupo na web, acesse:
> http://br.groups.yahoo.com/group/ciencialist/
>
> * Para sair deste grupo, envie um e-mail para:
> ciencialist-unsubscribe@yahoogrupos.com.br
> <mailto:ciencialist-unsubscribe@yahoogrupos.com.br?subject=Unsubscribe>
>
> * O uso que você faz do Yahoo! Grupos está sujeito aos Termos do
> Serviço do Yahoo! <http://br.yahoo.com/info/utos.html>.
>
>
>
>
> __________ Informação do NOD32 1.996 (20050210) __________
>
> Esta mensagem foi verificada pelo NOD32 Sistema Antivírus
> http://www.nod32.com.br




SUBJECT: Re: [ciencialist] Re: Foucault / Luz ondas ou corpusculos.
FROM: "murilo filo" <avalanchedrive@hotmail.com>
TO: ciencialist@yahoogrupos.com.br
DATE: 11/02/2005 22:47

Oi,
só uma rapidinha.
Há uns 45 anos, um tio meu, fotógrafo altamente profissional e de 2ª
geração, contou-me que quando êle queria tirar uma foto CONTRA A LUZ DO SOL,
ÊLE USAVA O FLASH. E o flash evitava a indesejada sensibilização do filme e
a perturbação na lente. O flash, mais forte, ''empurrava'' a luz solar...
coisa linda!
Nunca mais ouví alguém comentar sobre isto, que aprendí nos meus 15 anos.
Para mim, leigo, isto é uma prova de que a luz é *matéria*, queiram ou não!
Alguém comenta? abr/Murilo SP 11/fev

>From: "Alberto Mesquita Filho" <albmesq@uol.com.br>
>Reply-To: ciencialist@yahoogrupos.com.br
>To: <ciencialist@yahoogrupos.com.br>
>Subject: Re: [ciencialist] Re: Foucault / Luz ondas ou corpusculos.
>Date: Fri, 11 Feb 2005 20:13:02 -0300
>
>----- Original Message -----
>From: "Hélio Ricardo Carvalho"
>Sent: Friday, February 11, 2005 5:19 PM
>Subject: [ciencialist] Re: Foucault / Luz ondas ou corpusculos.
>
> > Só para complementar a msg anterior.
> > Acho que na época de FOUCAULT a teoria ondulatória da luz era ONDAS
> > LONGITUDINAIS NO ETHER. Logo de mesmo tipo que as sonoras.
> > Quem conhecer bem história da ciência me corrija.
>
>A "história" via de regra é contada pelos "vencedores" e a verdadeira
>história costuma levar séculos para vir a tona e entrar no domínio
>acadêmico
>e, assim mesmo, quando passível de ser recuperada. Concordo plenamente
>contigo ao dizer que a experiência de Foucault fala muito mais a favor de
>uma luz corpuscular do que uma luz mecânico-ondulatória. O Victor fala em
>tipos diferentes de ondas, mas lembro que na época em que a experiência foi
>feita somente existiam ondas mecânicas, a menos que alguém quisesse
>ressuscitar o "espírito da matéria" newtoniano. Enfim, entre esse "espírito
>da matéria" e as ondas não mecânicas (eletromagnéticas, gravitacionais
>etc.), eu sou mais adepto do "espírito da matéria", ainda que utilize uma
>nomenclatura mais apropriada aos dias atuais (informação eletromagnética,
>gravitacional etc.). A meu ver, luz não é campo. Luz é luz e campo é campo.
>Luz é matéria e o campo é imaterial, se bem que tenha sua origem na matéria
>e sua ação em outra matéria.
>
> >Se isto é verdade, refaço a pergunta original:
> > Por que esta experiência descartou a luz corpuscular??????
>
>Eu diria que não descartou.
>
>Com respeito à luz ser "corpuscular" ou "ondulatória", vale a pena ler o
>seguinte trecho, copiado de Einstein e Infeld (A Evolução da Física):
>
>"A história da busca de uma teoria da luz não está de modo algum concluída.
>O veredicto do século XIX não foi final e definitivo. Todo o problema de
>decidir entre corpúsculos e ondas ainda existe para a Física moderna, desta
>vez de uma forma muito mais profunda e intrincada. Aceitemos a derrota da
>teoria corpuscular da luz até reconhecermos a natureza problemática da
>vitória da teoria ondulatória."
>
>Este texto, escrito há mais de 60 anos, ainda é válido em toda a sua
>plenitude.
>
>[ ]´s
>Alberto
>http://ecientificocultural.com/indice.htm
>Mas indiferentemente a tudo isso, o neutrino tem massa, o elétron não é
>uma carga elétrica coulombiana e a Terra se move. E a história se repetirá.
>




SUBJECT: Re: [ciencialist] Re: Foucault / Luz ondas ou corpusculos.
FROM: JVictor <jvoneto@uol.com.br>
TO: ciencialist@yahoogrupos.com.br
DATE: 11/02/2005 22:58

Alberto Mesquita Filho escreveu:

> ----- Original Message -----
> From: "Hélio Ricardo Carvalho"
> Sent: Friday, February 11, 2005 5:19 PM
> Subject: [ciencialist] Re: Foucault / Luz ondas ou corpusculos.
>
> > Só para complementar a msg anterior.
> > Acho que na época de FOUCAULT a teoria ondulatória da luz era ONDAS
> > LONGITUDINAIS NO ETHER. Logo de mesmo tipo que as sonoras.
> > Quem conhecer bem história da ciência me corrija.

Victor: Não encontrei, ainda, alguma coisa que esclareça o que pensava
Foulcault. Mas suspeito que seu(Alberto) raciocício esteja correto e faz
sentido, pois o éter era uma realidade como meio mecânico para uma
propagação ondulatória, fosse ela qual fosse. Mas, por outro lado,
parece-me irrelevante saber ou não, qual era o modo de propagação. O que
se mediu foi a velocidade da luz num meio mais denso que o ar, e isto
foi determinante para o lento abandono da teoria corpuscular, como disse
na mensagem anterior, mas sem o " lento",

>
> Alberto para Hélio: A Concordo plenamente
> contigo ao dizer que a experiência de Foucault fala muito mais a favor de
> uma luz corpuscular do que uma luz mecânico-ondulatória. O Victor fala em
> tipos diferentes de ondas, mas lembro que na época em que a
> experiência foi
> feita somente existiam ondas mecânicas, a menos que alguém quisesse
> ressuscitar o "espírito da matéria" newtoniano.

Victor: É evidente, e a experiência confirma, que a natureza das OEM é
absolutamente diferente da natureza de uma onda sonora, ou de outra onda
qualquer, mecânica. Acontece que, primeiro, na época, as OEM ainda não
eram bem conhecidas. Sabia-se que se propagavam como ondas. E toda onda
de vergonha na cara tinha que se propagar num meio elástico. Como estava
difícil encontrar um meio para acomodar essa nova realidade, inventaram
um, chamado éter, meio absolutamente anti-cisalhante, perfeito,
imponderável, indetectável, intocável, que penetrava tudo(epa!), sem dó
nem piedade. Tudo era igual para ele. Pior do que democracia de verdade;
segundo, a idéia de que um meio mecânico era algo absolutamente
imprescindível, cuja inexistência impossibilitaria que qualquer onda se
propagasse, por mais milagrosa que fosse, estava de tal maneira
impregnada nas mentes daquelas pessoas, que outro caminho não havia,
senão aventar a hipótese de algo como o éter, com as características
ideais acima descritas, que já nasceu já com defeitos graves, não
detectados, até Poincaré, que identificou entre outras "perebas", que a
terceira lei de Newton é violada dentro dessa teoria! (Tinha influência
sobre os corpos em movimento dentro dele, mas não aceitava o contrário.
Que covardia!) Está lá, em Ciência e Hipótese. Mesmo assim, foram
tocando o barco prá frente até que, finalmente, ele, Einstein, de
mansinho, e rompesse de maneira definitiva com as amarras.

> Alberto: A meu ver, luz não é campo. Luz é luz e campo é campo.
> Luz é matéria e o campo é imaterial, se bem que tenha sua origem na
> matéria
> e sua ação em outra matéria.
>
Victor: Luz não é matéria. Tem matéria associada, mas isso é uma coisa
diferente. Matéria não se propaga à velocidade da luz, " e a luz se
propaga "à velocidade da luz"... Luz é energia, com momentum e tudo, mas
não é matéria, dessas que caem no seu olho, coça e dói...

> >Se isto é verdade, refaço a pergunta original:
> > Por que esta experiência descartou a luz corpuscular??????
>
> Alberto: Eu diria que não descartou.
>
Victor: Eu diria que deu uma sacudida dos diabos nas convicções de
então. Em verdade a teoria ondulatória tomou um impulso magistral, em
detrimento da corpuscular, que foi sendo deixada de lado. Eu diria que
descartou, pelo menos até a época do baixinho Einstein, que a reviveu
magistralmente com sua teoria sobre o efeito fotoelétrico, efeito
bastante conhecido desde épocas anteriores, mas sem explicação
adequada, que fizesse sentido. Então, num rasgo de coragem e, mais uma
vez, contra o pensamento vigente, Einstein ressuscitou o modêlo
corpuscular. Mas, entre Foulcault e Einstein, este modelo deixou de ser
usado.

> Alberto: Com respeito à luz ser "corpuscular" ou "ondulatória", vale a
> pena ler o
> seguinte trecho, copiado de Einstein e Infeld (A Evolução da Física):
>
> "A história da busca de uma teoria da luz não está de modo algum
> concluída.
> O veredicto do século XIX não foi final e definitivo. Todo o problema de
> decidir entre corpúsculos e ondas ainda existe para a Física moderna,
> desta
> vez de uma forma muito mais profunda e intrincada. Aceitemos a derrota da
> teoria corpuscular da luz até reconhecermos a natureza problemática da
> vitória da teoria ondulatória."
> Este texto, escrito há mais de 60 anos, ainda é válido em toda a sua
> plenitude.
>
> Victor: É verdade. E é óbvio também que Einstein só poderia pensar do
> jeito acima. Nunca de outro. Para Einstein, a sua forte e genial
> intuição de que "O veredicto do século XIX não foi final e
> definitivo", efetivamente,levou-o a pensar assim; e foi precisamente
> por isto que ousou ousar! Até hoje, e por mais um tempão, a sentença
> "Aceitemos a derrota da
> teoria corpuscular da luz até reconhecermos a natureza problemática da
> vitória da teoria ondulatória" continuará na ordem do dia. A física
> moderna, como você sabe e "adora", lida hoje muito bem com isso, até
> de maneira conformada, sem stress. Olhando direitinho, não seria mesmo
> muito mais complicado - acho que teríamos que mudar toda a física- se
> a teoria ondulatória fosse absoluta, a ponto de eliminar por completo
> o uso da teoria corpuscular, para explicar certos aspectos, hoje só
> explicado com o auxílio da teoria corpuscular? Seria o começo de uma
> teoria do tudo, no que não acredito, pelos absurdos e inconsistências
> que algo assim encerraria. Bem, para você seria lindo: Adeus à FQ, ao
> menos do jeito que essa teoria é hoje.

Sds,

Victor.

>
>
> ##### ##### ##### #####
>
>
> *Yahoo! Grupos, um serviço oferecido por:*
> PUBLICIDADE
> <http://br.rd.yahoo.com/SIG=12ae47dmo/M=264379.5078783.6203979.1588051/D=brclubs/S=2137111528:HM/EXP=1108246514/A=2332652/R=0/id=noscript/SIG=119058f8i/*http://br.download.yahoo.com/messenger/>
>
>
>
> ------------------------------------------------------------------------
> *Links do Yahoo! Grupos*
>
> * Para visitar o site do seu grupo na web, acesse:
> http://br.groups.yahoo.com/group/ciencialist/
>
> * Para sair deste grupo, envie um e-mail para:
> ciencialist-unsubscribe@yahoogrupos.com.br
> <mailto:ciencialist-unsubscribe@yahoogrupos.com.br?subject=Unsubscribe>
>
> * O uso que você faz do Yahoo! Grupos está sujeito aos Termos do
> Serviço do Yahoo! <http://br.yahoo.com/info/utos.html>.
>
>
>
>
> __________ Informação do NOD32 1.996 (20050210) __________
>
> Esta mensagem foi verificada pelo NOD32 Sistema Antivírus
> http://www.nod32.com.br




SUBJECT: on topics
FROM: "murilo filo" <avalanchedrive@hotmail.com>
TO: ciencialist@yahoogrupos.com.br, forum-ciencia@yahoogrupos.com.br
DATE: 11/02/2005 23:36

Oi,tenho certeza de que êste link será de interesse de alguns.
Já ví outros, mas êste me parece mais refinado e menos paranóico.
abr/Murilo SP 11/fev
>
>http://www.suppressedscience.net/
>No virus found in this outgoing message.
>Checked by AVG Anti-Virus.
>Version: 7.0.296 / Virus Database: 265.8.6 - Release Date: 7/2/2005




SUBJECT: Fw: Antigravity; New solar cell; Nuclear batteries; Ultracapacitors; Cold Fusion
FROM: "murilo filo" <avalanchedrive@hotmail.com>
TO: ciencialist@yahoogrupos.com.br, forum-ciencia@yahoogrupos.com.br
DATE: 11/02/2005 23:40

Os ítens 6 e 7 deverão ser de interesse especial, suponho!
( ultra-capacitores e super-baterias... )
Vamos acompanhar! abr/M. SP 11/fev

>----- Original Message -----
>From: Integrity Research Institute, Thomas Valone
>To: Integrity Research Institute, Thomas Valone
>Sent: Wednesday, February 09, 2005 3:15 PM
>Subject: Antigravity; New solar cell; Nuclear batteries; Ultracapacitors;
>Cold Fusion
>
>
> Future Energy eNews IntegrityResearchInstitute.org
>Feb. 9, 2005
>
>
>
>1) New Solar Cell - Flexible material generates electricity from infrared
>heat with high 30% efficiency
>2) Glaciers Melting at Alarming Rate - Alaska, Alps, Andes all report
>dramatic global warming effects
>3) Cheap Oil is Gone - Oxford Institute for Energy Studies predicts
>sustained high prices
>4) Cold Fusion Gets Activated - Unanimous decision that DOE should fund
>worthy proposals - IRI calls for Senate Hearing
>5) Antigravity Report from European Space Agency - Good summary of theories
>in 20-page report (arxiv link here) though not exhaustive in its scope
>(e.g. electrogravitics overlooked). Nature magazine review of report below.
>6) Super Charged Ultracapacitors - Storage time and energy density rival
>batteries for electric cars
>7) Daintiest Dynamos - Nuclear betavoltaic batteries are the longest
>lasting and "thousands of times stronger than lithium batteries" - IEEE
>Spectrum
>
>--------------------------------------------------------------------------------
>
>
> 1) Canadian Researcher Invents New Solar Cell
>
> Astrid Poei, Science - Reuters, Thu Jan 13, 2005
>http://story.news.yahoo.com/news?tmpl=story&u=/nm/20050113/sc_nm/energy_canada_solar_dc
>
> TORONTO (Reuters) - It may only be a matter of time before we will
>be using our shirts to charge our cellphones.
>
> Researchers at the University of Toronto have invented a flexible
>plastic solar cell that is said to be five times more efficient than
>current methods in converting energy from the sun into electrical energy.
>
>
> Team leader Ted Sargent, a professor of electrical and computer
>engineering at the university, said the cell harnesses infrared light from
>the sun and can form a flexible film on the surface of cloth, paper or
>other materials.
>
>
> And the film can turn 30 percent of the sun's power into usable
>electrical energy -- a far better performance than the 6 percent gleaned
>from the best plastic solar cells now in use.
>
>
> "The fact that these materials harness the sun's energy using
>flexible materials potentially could allow you to weave the plastics into
>fibers, sort of the way we have synthetic fibers already, and to weave
>those into clothing and make something that's a wearable solar cell,"
>Sargent said from Boston, where he is working until the summer.
>
>
> "That's sort of portable electricity."
>
>
> Sargent said the coating could be woven into a shirt or sweater and
>used to charge an item like a cellphone.
>
>
> "We expect that our cellphones or our e-mail can go anywhere with
>us, but we don't have that expectation of a continuous supply of power. The
>best that we have is batteries, which run out," he said.
>
>
> "So if we could have a wireless source of power like how the sun
>would provide, this would be pretty exciting."
>
>
> Research about the new cell was published in the Sunday online
>edition of the scientific journal Nature Materials, and Sargent said he was
>now looking for investors who could turn the invention into a commercially
>viable product.
>
>
> Terry White, president of the Solar Energy Society of Canada said
>solar cells on these lines could transform the industry.
>
>
> "If they make (solar cells) both less expensive and the potential
>applications more varied, then it's a major breakthrough," he said.
>
>
> Sargent said the technology could be available to the average
>consumer within five to ten years. But it already has Wall Street venture
>capitalists interested.
>
>
> "The technology really caught my eye both in the scientific
>literature and the business prospects," said Josh Wolfe, managing partner
>at Lux Capital in Manhattan, a venture capital firm that makes an estimated
>$1- to $2-million U.S. investment per project in early stage
>nanotechnology.
>
>
> "So the concept of having rollable newspaper displays or other
>things that could power your laptop or portable devices or create new
>products that are best left to the creativity of the engineers, to me, it
>represents a pretty sea-change."
>
>
> So what happens if the sun doesn't shine?
>
>
> "There is obviously no power in the visible (light)," said Sargent.
>"But in the infrared, it's not completely zero power. It doesn't have to be
>as hot as the sun, but everything that's warm, gives off some heat. Even
>people and animals give off heat. So there actually is some power remaining
>in the infrared even when it appears to us to be dark outside."
>
>
>
>
>--------------------------------------------------------------------------------
>
>2) Alpinists' Ice-Dreamy Mountains Melting Away
> By Katy Human, The Denver Post, 12 January 2005
>http://www.denverpost.com/Stories/0,1413,36%257E53%257E2648220,00.html?search=filter
>
> Where there was once cold, hard ice, there is now dirty slush and
>crumbling rock.
>
> From the peaks and slopes of many of the world's most challenging
>mountains, ice and snow are dripping away, reshaping the century-old sport
>of alpinism and disquieting longtime mountain climbers.
>
> "Among alpinists who have been climbing for 20, 30 years, there is
>this sense of urgency that these climbs are going away," said John
>Bicknell, a guide, co-director of the Colorado Mountain School in Estes
>Park and a former geologist. "For me, it'll be an immense loss. It's where
>I've spent most of my life. It's the terrain I most love."
>
> Around the world, high-altitude regions are warming and melting.
>Kilimanjaro's glaciers have all but disappeared. Glacier National Park's
>are melting so fast that federal computer models predict they'll be gone by
>2030.
>
> Mark Dyurgerov, a University of Colorado glacier expert and former
>alpinist, calculated that the volume of the world's glaciers has dropped by
>about 10 percent in the past four decades. The decline is even faster in
>some places, he said, including the popular climbing meccas of Alaska, the
>Andes and the Alps.
>
> Regardless of whether people, natural cycles or both are to blame, the
>effects are clear to climbers and guides. They're watching more rocks
>tumble down cliffs, throwing away useless old books and maps, and picking
>their way through miles of crumbly rock only to find climbs too dangerous
>to attempt.
>
> "There are routes you cannot do anymore," said Jose Garcia, a
>Venezuelan who lives and works in Boulder and climbs around the world.
>
> Three years ago, Garcia ventured into the ranges surrounding Piramide,
>a 19,000-foot-plus mountain in Peru. Avalanches constantly rumbled down the
>peak, loosened by warm temperatures and the changing structure of snow and
>ice hugging the mountain. "That used to be a very challenging, very
>interesting mountain," Garcia said. "You cannot climb it anymore. You can
>expect to die."
>
> As glaciers draping the slopes of high mountains retreat, the ice
>moves, Garcia explained. Crevasses yawn wider and deeper, giant cliffs of
>ice called séracs break away, and melting ice or permafrost loosens
>boulders, which tumble down slopes.
>
> "Books are now obsolete," Garcia said. "Maps also."
>
> When considering a climb, he checks the Internet for new route
>descriptions and pictures, or he contacts colleagues who have been there
>recently.
>
> Although mountain climbers are often labeled as risk-takers, most say
>the new risks do not make climbing more fun.
>
> "Climbers seldom look for dangerous routes," said Gary Neptune, owner
>of Neptune Mountaineering in Boulder and a lifetime alpinist. "Challenging,
>yes, but minimizing danger, that's part of the game."
>
> Climbing in Africa several years ago, he and his colleagues searched
>for a glacier - the known access route to a peak in the Rwenzori Mountains.
>It had disappeared sometime in the past 30 years - the age of the
>photographs in his guidebook, Neptune said.
>
> He and his colleagues abandoned the climb. "It's all getting less
>predictable or more extreme," Neptune said.
>
> In the Alps, paths to some peaks have morphed from smooth glacial
>hikes into dangerous scrambles up rock- strewn slopes. Grosses Wiesbachhorn
>- the pitch in Austria where alpinists first used ice pitons in the 1920s -
>hasn't been icy in years, Neptune said.
>
> A few decades ago, Boulder climber and guide Bob Culp loved to
>practice ice climbing at the foot of a glacier coming off France's Mont
>Blanc. He took his son there a few years ago, he said, but the trail to the
>glacier was closed. The two took another route to the glacier's edge.
>
> "While we were standing there looking, a baseball-sized rock came
>tumbling down and hit me in the hip," Culp said. "I wasn't hurt, but I
>thought, 'This is not a place we want to be right now."'
>
>
>
>--------------------------------------------------------------------------------
>
>
>3) With Geopolitics, Cheap Oil Recedes Into Past
>
>By JAD MOUAWAD New York Times, January 3, 2005
>http://www.nytimes.com/2005/01/03/business/03oil.html?ex=1107072526&ei=1&en=abeb977dee80b732
>
>
>IT was a year that people in the oil markets are unlikely to forget - a
>year that prices set records, forecasts lost touch with reality, and almost
>everything that could go wrong, did. It was also a year that politics
>returned to the oil market.
>
>And the trend is likely to continue this year. While oil prices have
>declined since October, many of the issues that have vexed the oil industry
>in 2004 are expected to recur. Cheap oil increasingly looks like a thing of
>the past.
>
>Through the 1990's, prices were stable, supplies were secure and there was
>plenty of extra capacity to keep energy costs low and world growth buzzing.
>At an average of $20 a barrel, oil was viewed as just another commodity.
>
>But then came ethnic and labor troubles in Nigeria; chaos and protests in
>Venezuela before President Hugo Chávez won a referendum allowing him to
>stay in power; hardball energy politics in Russia; and the continuing
>insurgency in Iraq.
>
>While supplies of oil to the world markets were rarely interrupted, the
>uncertainties created by these events raised crude oil prices in New York
>by two-thirds this year, to a high of more than $55 a barrel in October.
>And as energy costs surged, many analysts, traders and politicians woke up
>to the reality that oil was different from cocoa or coffee.
>
>"Oil is a political commodity," said Robert Mabro, president of the Oxford
>Institute for Energy Studies, one of the world's foremost energy experts.
>"Geopolitics is the most fundamental issue if you're looking at oil
>markets. People seem to have forgotten that since the 1980's."
>
>Of course, this is not the first time that oil and politics have mixed.
>
>Decades ago, militant governments in Iran and Libya, for example,
>nationalized their oil sectors, forcing American and European companies out
>and taking charge of their natural resources. Then came the oil embargo and
>the price shocks of 1973-74 and 1978-81, with long lines for gasoline and
>steep rises in inflation.
>
>But for the most part, politics had dropped off the energy map since then.
>In the 1980's, energy experts largely discounted a war between two of the
>Persian Gulf's top oil producers, Iran and Iraq, because Saudi Arabia and
>some other OPEC nations could simply crank up their production to make up
>for losses.
>
>Even the invasion of Kuwait by Iraq in the summer of 1990, and the
>subsequent embargo on their oil exports, roiled energy markets for only a
>few weeks.
>
>But in recent years, the oil industry has undergone a fundamental change.
>While demand has steadily increased each year, the industry's exploration
>efforts have not kept pace in new discoveries.
>
>Now that worldwide production is running at full speed to meet increased
>demand, there is no cushion left in the system to weather a potential blow
>to producers like Iraq, Venezuela, Iran, Russia or Nigeria.
>
>So, once again for oil markets, politics matters.
>
>For instance, said Amy Myers Jaffe, the associate director of Rice
>University's energy program, Saudi Arabia's oil industry is no longer seen
>as being impenetrable to terrorist attacks; tensions in the Persian Gulf
>could swell over Iran's nuclear program; Nigerian factions may erupt in
>violence; and the fighting in Iraq goes on.
>
>"All kinds of things can affect this market," Ms. Jaffe said, "especially
>when you're in a razor-thin situation. The only thing that could
>dramatically alter the outlook is a major economic recession."
>
>The heightened geopolitical risk has translated into higher prices,
>something analysts call a "risk premium." Crude oil prices have averaged
>$30 a barrel since 2000, but last year crude oil in New York climbed to an
>average of $41 a barrel. While energy prices are high, adjusted for
>inflation they are below the level in March 1981, when crude oil approached
>$70 a barrel in today's dollars. Still, analysts do not expect prices to
>fall anytime soon.
>
>High world prices since mid-2002 have helped sustain the economic recovery
>of Russia, which is raising output, according to the Energy Information
>Agency of the Department of Energy.
>
>The former Soviet Union, of which Russia is by far the biggest country, is
>the world's largest producer, the agency says, followed by Saudi Arabia and
>the United States. The biggest consumers are the United States, which
>imports over half its needs; China; Japan; and the former Soviet Union,
>which uses about a third as much as it produces. Leo Drollas, chief
>economist for the Center for Global Energy Studies in London, expects oil
>prices to be higher in 2005, on average, than they have been this year. The
>institute was founded in 1990 by Sheik Ahmed Zaki Yamani, the former Saudi
>oil minister.
>
>Even oil companies, which are usually extremely conservative about their
>price outlook, are coming around to that realization. Lord Browne, the
>chief executive of BP, now sees a new bottom of $30 a barrel for the next
>few years.
>
>"There is something fundamental holding prices up, whether that's at $45,
>$40 or $35 a barrel," Mr. Mabro of the Oxford Institute said. "And politics
>won't improve things. Except if you believe a miracle is going to happen in
>Iraq."
>
>
>
>--------------------------------------------------------------------------------
>
>
>4) US Review Rekindles Cold Fusion Debate
>
>Geoff Brumfiel, Nature, December 2, 2004
>http://www.nature.com/news/2004/041129//full/041129-11.html
>
>Energy panel split over whether experiments produced power.
>
>Claims of cold fusion are intriguing, but not convincing. That is the
>conclusion of an 18-member scientific panel tasked with reviewing research
>in the area.
>
>The findings, which were released on 1 December by the US Department of
>Energy, rekindle a 15-year-old debate over whether nuclear fusion can occur
>at room temperature.
>
>According to the report, the panel was "split approximately evenly" on the
>question of whether cold experiments were actually producing power in the
>form of heat. But members agreed that there is not enough evidence to prove
>that cold fusion has occurred, and they complained that much of the
>published work was poorly documented.
>
>The review is a positive step for the field of cold fusion, according to
>David Nagel at George Washington University in Washington DC, who
>co-authored the summary of cold-fusion work that the panel reviewed. "Most
>scientists think that cold fusion is laughable, but when the dust settled,
>the researchers reviewing our work were evenly split," he says.
>
> "Most scientists think that cold fusion is laughable,
>but when the dust settled, the researchers reviewing our work were evenly
>split."
>
> David Nagel
> cold fusion researcher at George Washington University
>in Washington DC
>
>
>
>Others remain sceptical, however. "It is astonishing that the panel didn't
>find cold fusion convincing after almost 15 years of additional research,"
>says Bob Park, a professor of physics at the University of Maryland,
>College Park, and author of Voodoo Science, a book about junk science. Park
>says that although the quality of research has improved, no one should buy
>into cold fusion just yet.
>
>Hot stuff
>
>Fusion commonly occurs in stars like the Sun, where hydrogen atoms meld
>together to form helium and release huge amounts of energy in the process.
>Scientists have long believed that fusion has the potential to be an
>enormous source of power here on Earth. However, no one has yet been able
>to control fusion reactions because they only occur at temperatures and
>pressures similar to those found in stars.
>
>Or so scientists thought until 1989, when Stanley Pons and Martin
>Fleischmann of the University of Utah claimed to have created a new kind of
>fusion inside a small canister of water. Pons and Fleischmann claimed that
>when they ran an electrical current between two palladium plates separated
>by water containing deuterium, a heavy isotope of hydrogen, it created a
>small but measurable fusion reaction.
>
>
>
> "It is astonishing that the panel didn't find cold
>fusion convincing after almost fifteen years of additional research."
>
> Bob Park
> Professor of physics at the University of Maryland,
>Baltimore
>
>
>
>In a highly publicized press conference in Utah, the scientists claimed
>that this 'cold fusion' had the potential to revolutionize the world's
>energy production.
>
>Pons and Fleischmann's claims were quickly debunked by other scientists,
>who pointed out numerous experimental errors in the measurements. But the
>idea of cold fusion lives on in movies and science fiction, and among a
>small cadre of researchers.
>
>Those researchers finally caught the ear of the US energy secretary,
>Spencer Abraham, who commissioned the review in August 2003 from the
>department's science directorate.
>
>Although the reviewers remained sceptical, they were nearly unanimous in
>their opinion that the energy department should fund well-thought-out
>proposals for cold fusion. Nagel says that he expects many in the long
>neglected field to submit research plans in the coming months. "I will be
>among them," he adds.
>
>Resources
>
> Executive Summary of DOE Cold Fusion Nuclear Reactions Report - DOE
>Office of Science
>http://www.science.doe.gov/Sub/Newsroom/News_Releases/DOE-SC/2004/low_energy/index.htm
>
> US Navy's Space and Naval Warfare Systems Center in San Diego two-volume
>Cold Fusion report. "Thermal and nuclear aspects of the Pd/D2O system: a
>decade of research at Navy laboratories" - Dr. Scott Chubb was one of the
>main authors, with Introduction by Dr. Frank Gordon - Volume I, 3.5 Meg ~
>121 pages in PDF format
>http://www.spawar.navy.mil/sti/publications/pubs/tr/1862/tr1862-vol1.pdf
>
>"US Gives Cold Fusion a Second Look After Fifteen Years" - New York Times -
>2004
>http://query.nytimes.com/mem/tnt.html?tntget=2004/03/25/science/25FUSI.html&tntemail1
>
> "DOE Warms to Cold Fusion" - Physics Today - 2004
>http://www.physicstoday.org/vol-57/iss-4/p27.html
>
> "Cold Fusion Isn't Dead, It's Just Withering from Scientific Neglect"
>Sharon Begley, Wall Street Journal, Science Journal, Sept. 5, 2003
>http://online.wsj.com/article/0,,SB106270936017252700,00.html
> "Reasonable Doubt Doesn't Stop Progress"
> New Scientist Vol 177 Issue 2388 - 29 March 2003, page 36
> www.newscientist.com/news
> Additional web sites for information on cold fusion:
>http://world.std.com/~mica/cft.html
>http://www.lenr-canr.org
>http://www.newenergytimes.com
>
>
>--------------------------------------------------------------------------------
>
>5) Antigravity Has Feet of Clay
>Philip Ball , Nature, 26 January 2005;
>http://www.nature.com/news/2005/050124/full/050124-8.html
>
>Space agency report is a downer for gravity-control researchers.
>Interstellar spacecraft powered by warp drives are still the stuff of
>science fiction.
>
>Could astronauts take a leaf out of H. G. Wells's book The First Men in the
>Moon, and use spacecraft propelled by antigravity devices? Some see the
>idea as science fiction, but major space agencies take it seriously.
>
>In 2001, the European Space Agency (ESA) commissioned two scientists to
>evaluate schemes for gravity control. They have concluded that, even if
>such control were possible, the benefits for lifting spacecraft out of the
>Earth's gravitational field would probably not be worth the effort.
>
>But scientists working on such propulsion schemes dispute the report. "I
>regard the conclusion, even if correct, as uninteresting and, frankly,
>irrelevant", says James Woodward of California State University at
>Fullerton, who has worked for NASA on gravity-control propulsion.
>
>NASA ran a research programme on speculative propulsion methods, called
>Breakthrough Propulsion Physics, from 1996 until its funding was cut in
>2003. The project's founder and former manager, Marc Millis of NASA's Glenn
>Research Center in Cleveland, Ohio, says that the ESA report corrects some
>misconceptions in the field of gravity control. But he thinks its scope is
>too limited to rule out future research in the area.
>
>"The risk of this paper is that the casual reader will more broadly
>interpret the negative findings to apply to all inquiries into
>gravitational or inertial manipulation," says Millis.
>
>The report is not meant to kill off all such ideas, says one of its
>authors, cosmologist Orfeu Bertolami of Lisbon's Technical University in
>Portugal. "Our recommendation to ESA was to keep a critical eye on them,"
>he says. But, he adds, "this should be a low-intensity activity. Our
>estimates show that conventional ideas [for propulsion] are much more
>effective."
>
>Down to Earth
>
>Wells's fantasy hinges on the invention of a substance that shields any
>object placed above it from the Earth's gravity. But can such a material
>really exist? Antigravity seems to violate the law of conservation of
>energy, which prohibits perpetual motion. Place a wheel half over such a
>gravity shield and the shielded segment will rise, causing the wheel to
>rotate forever without a power source.
>
>"Conventional ideas for propulsion are much more effective" says Orfeu
>Bertolami, author of ESA report on antigravity. What's more, gravity cannot
>be screened out in the same way as light or sound: Einstein's general
>theory of relativity explains that gravity results from the way mass
>distorts space-time itself.
>
>But relativity is not the last word on the subject. "Gravity does not fit
>into the standard model of particle physics," says Clovis de Matos,
>technical officer in charge of the ESA study. "And we do not understand the
>gravitational interaction at the quantum level."
>
>De Matos explains that ESA commissioned the survey of gravity control
>partly to establish whether a quantum theory of gravity might expose
>loopholes in our current understanding that space technology could exploit.
>
>Bertolami and his co-author, Martin Tajmar of the space technology company
>ARC Seibersdorf in Austria, looked at proposals for assisting spacecraft
>launch by weakening gravity. They were not impressed. "None of the
>proposals seemed convincing and detailed enough," says Bertolami.
>"Experimentally and theoretically they do not seem to meet a standard we
>could qualify as scientific."
>
>Floating ideas
>
>All the same, the researchers did feel that some ideas for modifying
>gravity are worth exploring. For example, as they are reaching the edge of
>the Solar System, NASA's Pioneer spacecraft are deviating from their
>expected trajectories. This has led some scientists to suggest that the
>current theory of gravity is incomplete.
>
>There have also been suggestions that magnetic effects in materials whose
>behaviour is dominated by quantum effects, such as superconductors, might
>induce a kind of artificial gravity. NASA scientists have studied claims by
>Russian physicist Eugene Podkletnov that a spinning superconductor can act
>as a gravity shield, reducing the weight of an object placed above it by
>about 2%.
>
>Independent scientists have been unable to reproduce this and similar
>claims, says Tajmar. He and Bertolami conclude that there are currently no
>good grounds for taking such effects seriously. All the same, they don't
>rule out the possibility of gravitational anomalies in quantum materials.
>
>Other options involve the gravitational and inertial masses of objects.
>Gravitational mass determines the force of gravity experienced by the
>object; inertial mass determines how much force is needed to set it in
>motion. General relativity says that the two definitions are identical, but
>some theories of quantum gravity suggest that they differ.
>
>Tajmar and Bertolami looked at schemes to alter one kind of mass, leaving
>the other unchanged. They found that reducing the inertial mass has no
>effect on the amount of fuel needed to launch a spacecraft. And altering
>the gravitational mass alone, by gravity shielding for example, doesn't
>help unless the shielding is almost total.
>
>Reference
>
>Tajmar, M. & Bertolami, O. "Hypothetical Gravity Control and Possible
>Influence on Space Propulsion" Report Preprint at
>http://xxx.arxiv.org/abs/physics/0412176 (2005).
>
>
>
>--------------------------------------------------------------------------------
>
> 6) Super Charged
>By Glenn Zorpette, IEEE Spectrum Online, January, 2005
>http://www.spectrum.ieee.org/WEBONLY/publicfeature/jan05/0105wcap.html#f3
>
>A tiny South Korean company is out to make capacitors powerful enough to
>propel the next generation of hybrid-electric cars
>
>Let's say it's 2010, and you're boiling off midlife ennui or burnishing
>your golden years in time-honored fashion: by zooming around in a
>high-performance road machine. The car accelerates powerfully, and yet it
>moves quietly and nimbly, slaloming through curves like a go-cart. Best of
>all, it sips gas like a connoisseur enjoying 40-year-old Armagnac. Would
>you believe you owe these rejuvenating, guilt-free thrills to a bunch of
>capacitors?
>
>Not just any capacitors, of course. To understand what's going on under the
>hood of this car, you'll need to leave behind the Lilliputian world of the
>picofarad and the microfarad and enter the realm of the kilofarad. It is a
>place where NessCap Co., in Yongin, South Korea, holds sway.
>
>NessCap is one of about 10 makers of ultracapacitors, devices that can
>store so much charge that they are beginning to blur the functional
>distinction between the capacitor and the battery. And according to some
>experts, nobody does it better than NessCap, which offers a unit rated at
>an impressive 5000 farads at 2.7 volts in a package a little bigger than a
>half-liter soda bottle. NessCap's capacitors "perform as well as or better
>than any others we've ever tested, in terms of energy and power density,"
>says Marshall Miller, a research engineer at the University of California
>at Davis, where he specializes in testing advanced capacitors and other
>devices.
>
>Ultracapacitors made by NessCap and others are just now starting to show up
>in products ranging from toys to experimental buses, basically as
>alternatives to batteries. The worldwide market isn't large; it was just US
>$38 million in 2002, the most recent year for which figures are available,
>according to the research firm Frost & Sullivan, in San Antonio. But
>NessCap and the handful of other makers of the largest ultracapacitors all
>have their sights set on the automotive market, which could do for their
>business what the iPod did for sales of MP3 songs. Frost & Sullivan, at
>least, is a believer; the company optimistically predicts total 2007
>revenues for ultracapacitors of $355 million.
>
>On paper, anyway, the idea is not far-fetched. In comparison with
>batteries, ultracapacitors can put out much more power for a given weight,
>can be charged in seconds rather than hours, and can function at more
>extreme temperatures. They're also more efficient, and they last much
>longer-in tests at the Idaho National Engineering and Environmental
>Laboratory, in Idaho Falls, upwards of 500 000 charge-discharge cycles have
>been recorded. Automotive traction batteries, for comparison, have much
>shorter lifetimes, particularly if they are discharged deeply.
>
>Pondering the relative strengths of capacitors and batteries, Joel
>Schindall, associate director of the Laboratory for Electromagnetic and
>Electronic Systems at the Massachusetts Institute of Technology, in
>Cambridge, says: "In all ways other than energy density, an electric field
>is superior to chemistry for storing energy regeneratively, because it is
>completely reversible" and therefore intrinsically efficient and durable.
>Part of Schindall's research focuses on advanced materials that could be
>used as electrodes in future ultracapacitors.
>
>Ultracapacitors are now establishing themselves in niches demanding a power
>source that can recharge quickly, be sealed into a system that has to last
>for years, or put out prodigious amounts of power in short bursts.
>Tokyo-based Ricoh Co. is using them in copier machines to store the energy
>needed to warm up the machines quickly, minimizing time spent in the
>energy-wasting standby mode. Makers of high-end car stereo amplifiers are
>using ultracapacitors to deliver the surges of power demanded by musical
>crescendos, without straining the vehicle's battery.
>
>Another use is in solar tiles; a new twist in landscape architecture,
>they're used to guide pedestrians at night, by storing solar-generated
>electricity during the day and using it to power a small light-emitting
>diode panel after dark [see photo, "Bright Idea"]. Sealed into a walkway,
>wall, or staircase, these clear, rugged tiles have to last for a decade or
>more, working without fail night after night, withstanding subfreezing and
>sweltering temperatures alike-criteria only ultracapacitors can fulfill.
>
>And then there are cars. The hybrid-electric vehicle, in its various forms,
>is poised for an increasing share of the automotive market in several parts
>of the world, including the United States. And ultracapacitors have already
>found their way into hybrids, albeit in a minor role: hardly noticed among
>the Toyota Prius's many celebrated technical breakthroughs is the fact that
>it uses ultracapacitors, from Panasonic, to power an electric-hydraulic
>pump in the mechanical braking system.
>
>It's just the start of what some experts say ultra-capacitors will do for
>hybrids. For example, with their lightning-fast charge and discharge
>capability, ultracapacitors could handle the power surges needed for
>accelerating, allowing engineers to use a smaller battery pack in the
>vehicle (and eventually, perhaps, no battery pack at all). Shielded from
>high-current pulses, the batteries would last longer, too.
>
>There are other intriguing possibilities, such as using the devices to give
>more or less ordinary cars "stop-and-go" operation, in which the gasoline
>engine is extinguished at stops and started instantly when the brake pedal
>is released. Ultracapacitors and a powerful starter motor would instantly
>jolt the engine back to life. Such vehicles would also make use of
>regenerative braking, converting into electricity the kinetic energy
>otherwise thrown off as heat in the brakes and storing that electricity in
>the ultracapacitors.
>
> SO WHAT WILL IT TAKE FOR ULTRACAPACITORS to find a home under the hood?
>First, they've got to be a lot cheaper. Today, at roughly $9500 per
>kilowatthour, ultracapacitors are too expensive by a factor of five, at
>least, for cost-conscious carmakers. Second, automotive engineers would
>like to see the devices store more energy (as opposed to power) per unit
>weight, which would let the devices take over more of the energy-storage
>burden from batteries in future vehicles.
>
>If NessCap and its competitors can achieve those goals and crack this
>market, the long-term future looks good. No one knows when, or even if, the
>fuel-cell car will become a mass-market reality-the estimates range from 10
>to 30 years. But if it does happen, it's likely that ultracapacitors will
>be a big part of the reason. Fuel cells, by themselves, deliver power too
>sluggishly to briskly accelerate a full-size car. They must be mated to a
>faster-acting energy-storage device, and for this coupling, ultracapacitors
>are superior in many respects to batteries.
>
>"Capacitors and fuel cells are made for each other," insists Andrew Burke,
>a specialist on ultracapacitors and a research engineer at the University
>of California, Davis. Honda, for example, used only ultracapacitors to
>supplement the fuel cell in its experimental FCX-V3 and FCX-V4 vehicles,
>several of which have been leased in California and Japan [see
>illustration, "Fueling Around"]. For these vehicles, Honda used its own
>ultracapacitors.
>
>At first glance, at least, NessCap may seem an unlikely candidate to get
>ultracapacitors into a production car. NessCap's three main
>competitors-Maxwell Technologies in San Diego; Epcos in Munich, Germany;
>and Panasonic in Osaka, Japan-all have either deep-pocketed parent
>companies or revenue from other product lines with which to support their
>ultracapacitor development. (Panasonic ultracapacitors are manufactured by
>Matsushita Electronic Components Co., in Kadoma City, Japan.)
>
>But what NessCap lacks in resources, it makes up in resourcefulness and
>determination. The company was founded in 2001 by Sun-wook Kim, a Korean
>entrepreneur and former research director at the Daewoo Group. Although Kim
>has a few other ventures, including a new organic-LED display factory in
>Singapore, NessCap is basically a stand-alone enterprise that will either
>succeed or fail on the strength of its ultracapacitors and on its
>executives' ingenuity in promoting them.
>
>Certainly, the company is efficient: all of NessCap's 65 employees work in
>a boxy, yellowish, blue-trimmed building in a gritty suburb outside the
>Korean industrial city of Suwon. It houses NessCap's factory, offices, and
>R&D laboratories and its quality-control, testing, and shipping and
>receiving departments, as well as a subsidiary consumer-electronics spinoff
>and a warehouse. And though it's a small company, NessCap makes all its own
>electrodes for its capacitors. Among the company's closest competitors,
>only Panasonic can also make that claim, says NessCap's chairman, Inho Kim
>(who is not related to Sun-wook Kim).
>
>This distinction is important, he says, because he expects electrode
>refinements to be the main source of future improvements in ultracapacitor
>performance-greater energy storage, for example-and decreases in cost.
>Electrode technology, Inho Kim estimates, determines "70 or 80 percent" of
>the capacitor's performance. "If you own the electrode-manufacturing
>technology, you can basically do anything," he argues.
>
>
>--------------------------------------------------------------------------------
>ULTRACAPACITOR DEVELOPMENT PROJECT
>GOAL: Cut the cost of ultracapacitors are superior to batteries in many
>respects and will almost certainly be used increasingly in hybrid-electric
>and fuel-cell cars
>ORGANIZATION: NessCap Co.
>CENTER OF ACTIVITY: NessCap's facility in Yongin, South Korea
>NUMBER OF PEOPLE ON THE PROJECT: About 15
>BUDGET: Approximately US $2 million
>
>
>--------------------------------------------------------------------------------
>
>TO GET AN IDEA of where these improvements will come from, you've got to
>understand what separates an ultracapacitor from an ordinary capacitor
>(other than a whole lot of farads). First, consider the classic
>parallel-plate capacitor, a sandwich of two conductive plates separated by
>an insulator, or dialectric. When the plates are connected to the positive
>and negative terminals of a battery, opposite charges separate from each
>other and accumulate on the plates. Driven by the battery's voltage, an
>electric field permeates the dielectric. Associated with that field is a
>voltage that opposes the battery's voltage.
>
>The field holds the accumulated, opposing charges apart; in doing so, it
>stores energy. So, unlike a battery, which stores energy in chemical form,
>a capacitor stores energy in an electric field; there are no moving parts
>and no chemical changes of state. To use a capacitor's energy, you just let
>its accumulated charges flow through a circuit, driven by the voltage
>associated with the field.
>
>Capacitance is simply a measure of how much charge a capacitor can store
>for a given voltage. In mathematical terms, the capacitance equals the
>charge on the plates divided by the voltage difference between them. The
>charge, however, is proportional to the area of the plates; larger plates
>can hold more charge. And the voltage is related to the distance between
>the two plates; less separation allows more charge to accumulate for a
>given voltage. So to wring the most capacitance from a device, you want
>plates, or electrodes, that have a large area, and you want to separate
>those plates by a very small distance.
>
>In the early 1960s, at the once mighty research laboratories of Standard
>Oil of Ohio (Sohio), researchers discovered that two pieces of activated
>carbon immersed in a liquid electrolyte formed an amazingly good capacitor,
>owing mainly to the fact that the activated carbon's myriad microscopic
>nodules had enormous surface area. Sohio licensed the technology to NEC
>Corp., Tokyo, in 1971, but it was Panasonic that pushed the concept hardest
>in the 1980s, followed by various projects sponsored by the U.S. Department
>of Energy in the 1990s.
>
>Since Sohio's initial experiments 40 years ago, the basic concept has not
>changed much. Coat two metal-foil electrodes with activated carbon and put
>a paper separator between them. Immerse the whole thing in a liquid
>electrolyte.
>
>Attach wires from the terminals of a battery to the two metal foils, and
>electrons immediately start accumulating in the carbon coated on the foil
>attached to the battery's negative terminal. Those electrons, in turn,
>attract positive ions from the electrolyte into the pores of the carbon on
>that foil. In the other electrode, meanwhile, positive charges accumulate,
>attracting negative ions from the electrolyte into the pores of the carbon.
>Both kinds of ions migrate freely through the paper separator that prevents
>the electrodes from touching each other and conducting current.
>
>Notice that this so-called capacitor is actually a pair of capacitors in
>series with each other. At each electrode, there is a separation of
>charges-electrons and positive ions at the negative electrode, and positive
>charges and negative ions at the positive electrode. So at each electrode
>there are two layers of charge, which is why ultracapacitors are also known
>as electric double-layer capacitors.
>
>The activated carbon's huge surface area comes from the great porosity of
>its microscopic nodules. It enables the positive and the negative ions
>migrating through the electrolyte to find plenty of nooks and crannies to
>occupy as they insinuate themselves as closely as possible into the
>oppositely charged carriers inside the carbon. Basically, as an electrode
>material, the activated carbon provides exactly the characteristics you
>want for high capacitance: vast surface area and the opportunity for the
>oppositely charged carriers to get atomically close to each other.
>
>The surface area of the carbon varies, but 1500 square meters per gram is
>not unusual. So for typical electrodes weighing 250 grams, the total area
>would be 375 000 square meters-or roughly 50 soccer fields.
>
> THE TRICK, OF COURSE, is getting that carbon onto the metal foil as
>uniformly and efficiently as possible. It is the first step in NessCap's
>manufacturing process-and the first topic of discussion on a tour of the
>company's small but spotless factory. All manufacturing at NessCap goes on
>in a series of three brightly lit rooms, whose Kelly green floors are all
>marked with yellow lines to show visitors where to walk.
>
>In big, shiny, stainless steel mixers-think Cuisinarts on
>steroids-technicians mix several types of activated carbon with water and
>with binding agents that cause the carbon-powder particles to stick to each
>other and to the long strips of aluminum foil electrodes. The resulting
>slurry gets coated onto one side of the aluminum, dried in a kiln, and then
>coated onto the other side. After more drying, the coated strips are run
>through a hot press to increase the density of their carbon layers and give
>those layers a uniform thickness.
>
>In the next room, machines scratch the carbon off the aluminum precisely
>and at regular intervals to make places where electrical leads are
>attached. Then the same machine winds together two long strips of the
>carbon-coated metal-one will be the anode, the other the cathode-with a
>strip of paper in between. "No other such machine exists in the world,"
>says Inho Kim proudly.
>
>In the third room, the wound electrode-separator assemblies are dried in a
>kiln and inserted in aluminum cases that are filled with electrolyte and
>welded shut. The finished capacitors are tested in a room across the hall;
>every single capacitor is tested before leaving the factory.
>
>Upstairs, NessCap's R&D department occupies a couple of rooms that take up
>about the same total area as a decent restaurant kitchen. As in an old-time
>apothecary, glass cabinets filled with bottles of powders and reagents line
>the walls.
>
>Not surprisingly, ultracapacitor researchers are mainly interested in two
>things: electrolytes and carbon. In virtually all high-performance
>ultracapacitors, the electrolyte is acetonitrile. It's great stuff, in the
>one way that really matters: it has terrifically low ionic resistance,
>roughly 15 ohm-centimeters, and that means high power density. But when
>acetonitrile burns, it can release cyanide, a fact that makes automakers
>unhappy. "Everybody's looking for a replacement for acetonitrile," says
>Burke at UC Davis. Several organic compounds, notably propylene carbonate,
>show promise, but none at the moment has ionic resistance as low as
>acetonitrile. (Honda used propylene carbonate in its own ultracapacitors,
>in the FCX fuel-cell cars.)
>
>Still, it is the carbon challenge that most consumes ultracapacitor
>researchers now, because it is the key to the two main goals: getting costs
>down and improving the energy (as opposed to power) density. In a typical
>ultracapacitor, the electrode materials-the carbons, essentially-account
>for more than half the cost of the device, Sun-wook Kim says.
>
>During a tour of the laboratory, NessCap's R&D director, Young-ho Kim,
>casually mentions that he's in the midst of running tests on no fewer than
>10 mixtures of activated carbons, looking for a combination of low cost,
>high performance, and durability that has so far eluded ultracapacitor
>makers.
>
>It all comes down to pores, he explains, drawing little circles on a piece
>of paper. You want pores that are all about 20 to 30 angstroms in diameter.
>Pores that are smaller than that aren't big enough to allow the ions to
>move in and out freely, which hurts performance. Lots of big pores, on the
>other hand, mean that the overall surface area is less than it should be,
>which also limits performance.
>
>Ultracapacitor makers are working with two main types of carbon,
>phenyl-resin based and pitch based. Phenyl-resin carbons perform better and
>are the standard now. But the attraction of pitch-based carbons, which are
>derived from coke and are used in asphalt, is their low cost-about
>one-fifth to one-tenth that of phenyl-resin carbons.
>
>The problem, Young-ho Kim says, is that it's harder to control the
>pore-size distributions in the pitch-based carbons, so they wind up with
>poorer characteristics. Their capacitance is usually about 30 percent less
>than that of the phenyl-resin-carbon devices, he explains. That means that
>30 percent more material must be used, which, of course, detracts from the
>cost savings and makes the finished devices larger. Still, Sun-wook Kim is
>confident that work on the pitch-based carbons will be a key factor in
>reducing the overall cost per farad of the devices.
>
>In the next breath, though, he dismisses the conventional wisdom that the
>carbons have to get down to $10 a kilogram to make ultracapacitors
>cost-competitive, from about $100 today (for the phenyl-based carbons). He
>insists that getting costs down will depend as much on manufacturing as on
>carbon prices. He points out that NessCap is now changing its manufacturing
>process to put its largest capacitors in cylindrical rather than
>rectangular cans. The simple shape change allows the electrode assembly to
>be wound more quickly, which in turn shaves more than 20 percent off the
>cost of making the capacitors, Inho Kim estimates.
>
> WHILE NESSCAP AND ITS COMPETITORS FOCUS on getting the cost of the
>carbons down, a few other researchers are investigating exotic, pricey
>forms of carbon that could eliminate the one clear drawback of
>ultracapacitors-low energy density-and let them mount a serious challenge
>to batteries. Commercially available ultracapacitors generally can be
>counted on to store about 3 or 4 watthours per kilogram, Burke says. That's
>a far cry from the 60 or 70 Wh/kg typical of nickel-metal hydride batteries
>or the 110 to 130 Wh/kg delivered by lithium-ion batteries.
>
>An ultracapacitor with batterylike energy density would be almost
>irresistible to automakers, to say nothing of countless other
>manufacturers, says John M. Miller, a retired Ford Motor Co. researcher.
>With high enough energy density, ultracapacitors could reduce or even
>eliminate the need for traction batteries in a hybrid car. "It's a pivotal
>time for energy-storage systems," he concludes.
>
>Tantalizing claims have surfaced of exotic carbon-based technologies that
>could boost the energy density of ultracapacitors 10- or even 100-fold-well
>into the realm of advanced batteries. But so far, these claims have not
>held up to independent scrutiny, say both Burke and Marshall Miller at UC
>Davis. An independent Japanese researcher, Michio Okamura, claims to have
>developed a carbon-based electrode material that he calls nanogate, which
>is nonporous and can deliver energy densities well above 50 Wh/kg. But
>solid, independent verification of his claims is not yet available,
>according to Burke.
>
>At MIT's electromagnetic laboratory, Schindall and lab director John
>Kassakian, with Ph.D. student Riccardo Signorelli, are leading a project to
>investigate the use of carbon nano-tubes, the latest miracle material, in
>electrodes. They are creating materials in which the nanotubes grow out
>perpendicularly from a substrate, like hair on a piece of scalp. The
>nanotubes would become electrically charged, just as the activated carbon
>does, so they would attract oppositely charged ions in the electrolyte. The
>nanotubes would also be spaced so as to hold these ions, much as a sea
>anemone grips small sea creatures in its tentacles. The advantage is that
>this arrangement can in theory trap many more ions than even the pores of
>activated carbon-enough perhaps to raise the energy density of an
>ultracapacitor 100-fold, Schindall estimates.
>
>So far, he and Signorelli have demonstrated technology that can grow the
>right kind of nanotubes and space them appropriately. By next summer, they
>hope to grow a patch of electrode big enough to test in an electrolyte, in
>order to assess its capacitance characteristics. If it works as well as
>their studies suggest, and if it can be easily manufactured-two big ifs-the
>dream of a near-ideal energy storage device will be that much closer to
>realization. "Suddenly, electrical energy storage turns on its
>head-potentially," Schindall says.
>
> MEANWHILE, FOR NESSCAP and its competitors, the game is basically this:
>find enough niche markets to stay afloat until technology advances make
>ultracapacitors even more attractive and automotive markets develop. And
>NessCap isn't waiting for the niche markets to come to it. Last year, the
>company started its own consumer-electronics firm, Infinity Inc., which is
>selling everything from crank-powered radios to solar tiles, all outfitted
>with NessCap capacitors.
>
>NessCap is also working with several other companies on niche automotive
>applications. A well-known courier company, for example, is about to start
>using NessCap's ultracapacitors in 200 of its delivery vans. As they go
>about dropping off packages in densely populated areas, these vans must
>stop and restart their engines as many as 200 times a day. The short
>distances between stops means that the starter batteries can't recharge
>sufficiently and soon wear out. But the short distances are not a problem
>for ultracapacitors, which recharge in seconds and can easily store enough
>energy to fire up the engine. So the delivery-van system couples
>ultracapacitors for short-term energy storage with lead-acid batteries for
>longer-term storage.
>
>Looking beyond these niche applications, Inho Kim has high hopes for
>"micro-hybrids," which would have a 12-V battery, as in a conventional car.
>Micro-hybrids are basically a very mild form of mild hybrid, propelled
>mainly by a gasoline engine but with a beefed-up electric starter motor fed
>by a small rack of ultracapacitors. The capacitors and motor provide the
>stop-and-go operation described above; the car could also make use of
>regenerative braking.
>
>The guilt-free ultracapacitor-based roadster is probably more than a couple
>of years away. But a conventional car with a more reliable starter system,
>or even a micro-hybrid with an ultracapacitor boost, could be in your
>immediate future. If so, the revolution in energy storage will be well
>under way.
>
>TO PROBE FURTHER
>
>Kilofarad International, a trade group formed to promote the ultracapacitor
>industry, is an affiliate of the Electronic Components, Assemblies, and
>Materials Association. Its Web site is at http://www.kilofarad.org/.
>
>Andrew Burke of the University of California, Davis, has written numerous
>technical articles on ultracapacitors. Several are available online,
>including a survey from 2000:
>http://repositories.cdlib.org/cgi/viewcontent.cgi?article=1050&context=itsdavis.
>
>Menahem Anderman, president of the consulting firm Advanced Automotive
>Batteries, plans to release a report on ultracapacitors for automotive uses
>in February. You can order the US $7200 report at
>http://www.advancedautobat.com/Ultracapacitor/index.html.
>
>
>--------------------------------------------------------------------------------
>
>7) The Daintiest Dynamos
>By Amit LaL & James Blanchard, IEEE Spectrum, September, 2004
>http://www.spectrum.ieee.org/WEBONLY/publicfeature/sep04/0904nuct1.html
>
>By harvesting energy from radioactive specks, nuclear microbatteries could
>power tomorrow's microelectromechanical marvels-and maybe your cellphone,
>too
>
>For several decades, electronic circuitry has been shrinking at a famously
>dizzying pace. Too bad the batteries that typically power those circuits
>have not managed to get much smaller at all.
>
>In today's wrist-worn GPS receivers, matchbox-size digital cameras, and
>pocketable personal computers, batteries are a significant portion of the
>volume. And yet they don't provide nearly enough energy, conking out
>seemingly at the worst possible moment.
>
>The reason is simple: batteries are still little cans of chemicals. They
>function in essentially the same way they did two centuries ago, when the
>Italian physicist Alessandro Volta sandwiched zinc and silver disks to
>create the first chemical battery, which he used to make a frog's leg kick.
>
>Now, with technologists busily ushering in a new age of miniaturization
>based on microelectromechanical systems (MEMS), batteries have arrived at a
>critical juncture. MEMS are finding applications in everything from the
>sensors in cars that trigger air bags to injectable drug delivery systems
>to environmental monitoring devices. Many of these systems ideally have to
>work for long periods, and it is not always easy to replace or recharge
>their batteries. So to let these miniature machines really hit their
>stride, we'll need smaller, longer-lasting power sources.
>
>For several years our research groups at Cornell University and the
>University of Wisconsin-Madison have been working on a way around this
>power-source roadblock: harvesting the incredible amount of energy released
>naturally by tiny bits of radioactive material.
>
>The microscale generators we are developing are not nuclear reactors in
>miniature, and they don't involve fission or fusion reactions. All energy
>comes from high-energy particles spontaneously emitted by radioactive
>elements. These devices, which we call nuclear microbatteries, use thin
>radioactive films that pack in energy at densities thousands of times
>greater than those of lithium-ion batteries [see table, "Energy Content"].
>
>A speck of a radioisotope like nickel-63 or tritium, for example, contains
>enough energy to power a MEMS device for decades, and to do it safely. The
>particles these isotopes emit, unlike more energetic particles released by
>other radioactive materials, are blocked by the layer of dead skin that
>covers our bodies. They penetrate no more than 25 micrometers in most
>solids or liquids, so in a battery they could safely be contained by a
>simple plastic package [see sidebar, "Not All Radioisotopes Are Equal."]
>
>Our current prototypes are still relatively big, but like the first
>transistors they will get smaller, going from macro- to microscale devices.
>And if the initial applications powering MEMS devices go well, along with
>the proper packaging and safety considerations, lucrative uses in handheld
>devices could be next. The small nuclear batteries may not be able to
>provide enough electric current for a cellphone or a PDA, but our
>experiments so far suggest that several of these nuclear units could be
>used to trickle charges into the conventional chemical rechargeable
>batteries used in handheld devices. Depending on the power consumption of
>these devices, this trickle charging could enable batteries to go for
>months between recharges, rather than days, or possibly even to avoid
>recharges altogether.
>
>"IT IS A STAGGERINGLY SMALL WORLD THAT IS BELOW," said physicist Richard P.
>Feynman in his famous 1959 talk to the American Physical Society, when he
>envisioned that physical laws allowed for the fabrication of micro- and
>nanomachines and that one day we would be able to write the entire
>Encyclopaedia Britannica on the head of a pin.
>
>Feynman's vision has finally begun to materialize, thanks to ever more
>sophisticated microelectronics. Micro- and nanoscale machines are poised to
>become a multibillion-dollar market as they are incorporated in all kinds
>of electronic devices. Among the revolutionary applications in development
>are ultradense memories capable of storing hundreds of gigabytes in a
>fingernail-size device, micromirrors for enhanced displays and optical
>communications equipment, and highly selective RF filters to reduce
>cellphone size and improve the quality of calls.
>
>But, again, at very small scales, chemical batteries can't provide enough
>juice to power these micromachines. As you reduce the size of such a
>battery, the amount of stored energy goes down exponentially. Reduce each
>side of a cubic battery by a factor of 10 and you reduce the volume-and
>therefore the energy you can store-by a factor of 1000. In fact,
>researchers developing sensors the size of a grain of sand had to attach
>them to batteries they couldn't make smaller than a shirt button.
>
>IN THE QUEST TO BOOST MICROSCALE POWER GENERATION, several groups have
>turned their efforts to well-known energy sources, namely hydrogen and
>hydrocarbon fuels such as propane, methane, gasoline, and diesel. Some
>groups are developing microfuel cells that, like their macroscale
>counterparts, consume hydrogen to produce electricity. Others are
>developing on-chip combustion engines, which actually burn a fuel like
>gasoline to drive a minuscule electric generator.
>
>There are three major challenges for these approaches. One is that these
>fuels have relatively low energy densities, only about five to 10 times
>that of the best lithium-ion batteries. Another is the need to keep
>replenishing the fuel and eliminating byproducts. Finally, the packaging to
>contain the liquid fuel makes it difficult to significantly scale down
>these tiny fuel cells and generators.
>
>The nuclear microbatteries we are developing won't require refueling or
>recharging and will last as long as the half-life of the radioactive
>source, at which point the power output will decrease by a factor of two.
>And even though their efficiency in converting nuclear to electrical energy
>isn't high-about 4 percent for one of our prototypes-the extremely high
>energy density of the radioactive materials makes it possible for these
>microbatteries to produce relatively significant amounts of power.
>
>For example, with 10 milligrams of polonium-210 (contained in about 1 cubic
>millimeter), a nuclear microbattery could produce 50 milliwatts of electric
>power for more than four months (the half-life of polonium-210 is 138
>days). With that level of power, it would be possible to run a simple
>microprocessor and a handful of sensors for all those months.
>
>And the conversion efficiency won't be stuck at 4 percent forever.
>Beginning this past July we started working to boost the efficiency to 20
>percent, as part of a new Defense Advanced Research Projects Agency program
>called Radio Isotope Micro-power Sources.
>
>Space agencies such as NASA in the United States have long recognized the
>extraordinary potential of radioactive materials for generating
>electricity. NASA has been using radioisotope thermoelectric generators, or
>RTGs, since the 1960s in dozens of missions, like Voyager and, more
>recently, the Cassini probe, now in orbit around Saturn. Space probes like
>these travel too far away from the sun to power themselves with
>photovoltaic arrays.
>
>RTGs convert heat into electricity through a process known as the Seebeck
>effect: when you heat one end of a metal bar, electrons in this region will
>have more thermal energy and flow to the other end, producing a voltage
>across the bar. Most of NASA's washing-machine-size RTGs use plutonium-238,
>whose high-energy radiation can produce enormous heat.
>
>But as it turns out, RTGs don't scale down well. At the diminutive
>dimensions of MEMS devices, the ratio between an object's surface and its
>volume gets very high. This relatively large surface makes it difficult to
>sufficiently reduce heat losses and maintain the temperatures necessary for
>RTGs to work. So we had to find other ways of converting nuclear into
>electric energy.
>
>ONE OF THE MICROBATTERIES WE DEVELOPED early last year directly converted
>the high-energy particles emitted by a radioactive source into an electric
>current. The device consisted of a small quantity of nickel-63 placed near
>an ordinary silicon p-n junction-a diode, basically. As the nickel-63
>decayed, it emitted beta particles, which are high-energy electrons that
>spontaneously fly out of the radioisotope's unstable nucleus. The emitted
>beta particles ionized the diode's atoms, creating paired electrons and
>holes that are separated at the vicinity of the p-n interface. These
>separated electrons and holes streamed away from the junction, producing
>the current.
>
>Nickel-63 is ideal for this application because its emitted beta particles
>travel a maximum of 21 µm in silicon before disintegrating; if the
>particles were more energetic, they would travel longer distances, thus
>escaping the battery. The device we built was capable of producing about 3
>nanowatts with 0.1 millicurie of nickel-63, a small amount of power but
>enough for applications such as nanoelectronic memories and the simple
>processors for environmental and battlefield sensors that some groups are
>currently developing.
>
>The new types of microbatteries we are working on now can generate
>substantially more power. These units produce electricity indirectly, like
>minute generators. Radiation from the sample is converted first to
>mechanical energy and then to oscillating pulses of electric energy. Even
>though the energy has to go through the intermediate, mechanical phase, the
>batteries are no less efficient; they tap a significant fraction of the
>kinetic energy of the emitted particles for conversion into mechanical
>energy. By releasing this energy in brief pulses, they provide much more
>instantaneous power than the direct-conversion approach.
>
>For these batteries, which we call radioactive piezoelectric generators,
>the radioactive source is a 4-square-millimeter thin film of nickel-63 [see
>illustration, "Power From Within"]. On top of it, we cantilever a small
>rectangular piece of silicon, its free end able to move up and down. As the
>electrons fly from the radioactive source, they travel across the air gap
>and hit the cantilever, charging it negatively. The source, which is
>positively charged, then attracts the cantilever, bending it down.
>
>A piece of piezoelectric material bonded to the top of the silicon
>cantilever bends along with it. The mechanical stress of the bend
>unbalances the charge distribution inside the piezoelectric crystal
>structure, producing a voltage in electrodes attached to the top and bottom
>of the crystal.
>
>After a brief period-whose length depends on the shape and material of the
>cantilever and the initial size of the gap-the cantilever comes close
>enough to the source to discharge the accumulated electrons by direct
>contact. The discharge can also take place through tunneling or gas
>breakdown. At that moment, electrons flow back to the source, and the
>electrostatic attractive force vanishes. The cantilever then springs back
>and oscillates like a diving board after a diver jumps, and the recurring
>mechanical deformation of the piezoelectric plate produces a series of
>electric pulses.
>
>The charge-discharge cycle of the cantilever repeats continuously, and the
>resulting electric pulses can be rectified and smoothed to provide
>direct-current electricity. Using this cantilever-based power source, we
>recently built a self-powered light sensor [see photo, "It's Got the
>Power"]. The device contains a simple processor connected to a photodiode
>that detects light variations.
>
>
>
>--------------------------------------------------------------------------------
>
>Nuclear batteries can pack in energy at densities THOUSANDS OF TIMES
>greater than those of lithium-ion batteries
>
>--------------------------------------------------------------------------------
>
>Also using the cantilever system, we developed a pressure sensor that works
>by "sensing" the gas molecules in the gap between the cantilever and the
>source. The higher the ambient pressure, the more gas molecules in the gap.
>As a result, it is more difficult for electrons to reach and charge the
>cantilever. Hence, by tracking changes in the cantilever's charging time,
>the sensor even detects millipascal variations in a low-pressure
>environment like a vacuum chamber.
>
>To get the measurements at a distance, we made the cantilever work as an
>antenna and emit radio signals, which we could receive meters away-in this
>application the little machine was "radio active" in more ways than one.
>The cantilever, built from a material with a high dielectric constant, had
>metal electrodes on its top and bottom. An electric field formed inside the
>dielectric as the bottom electrode charged. When it discharged, a charge
>imbalance appeared in the electrodes, making the electric field propagate
>along the dielectric material. The cantilever thus acted like an antenna
>that periodically emitted RF pulses, the interval between pulses varying
>accordingly to the pressure.
>
>What we'd like to do now is add a few transistors and other electronic
>components to this system so that it can not only send simple pulses but
>also modulate signals to carry information. That way, we could make
>MEMS-based sensors that could communicate with each other wirelessly
>without requiring complex, energy-demanding communications circuitry.
>
>NUCLEAR MICROBATTERIES MAY ULTIMATELY CHANGE the way we power many
>electronic devices. The prevalent power source paradigm is to have all
>components in a device's circuitry drain energy from a single battery.
>Here's another idea: give each component-sensor, actuator,
>microprocessor-its own nuclear microbattery. In such a scheme, even if a
>main battery is still necessary for more power-hungry components, it could
>be considerably smaller, and the multiple nuclear microbatteries could run
>a device for months or years, rather than days or hours.
>
>One example is the RF filters in cellphones, which now take up a lot of
>space in handsets. Researchers are developing MEMS-based RF filters with
>better frequency selectivity that could improve the quality of calls and
>make cellphones smaller. These MEMS filters, however, may require
>relatively high dc voltages, and getting these from the main battery would
>require complicated electronics. Instead, a nuclear microbattery designed
>to generate the required voltage-in the range of 10 to 100 volts-could
>power the filter directly and more efficiently.
>
>Another application might be to forgo the electrical conversion altogether
>and simply use the mechanical energy. For example, researchers could use
>the motion of a cantilever-based system to drive MEMS engines, pumps, and
>other mechanical devices. A self-powered actuator could be used, for
>instance, to move the legs of a microscopic robot. The actuator's
>motion-and the robot's tiny steps-would be adjusted according to the
>charge-discharge period of the cantilever and could vary from hundreds of
>times every second to once per hour, or even once per day.
>
>THE FUTURE OF NUCLEAR MICROBATTERIES depends on several factors, such as
>safety, efficiency, and cost. If we keep the amount of radioactive material
>in the devices small, they emit so little radiation that they can be safe
>with only simple packaging. At the same time, we have to find ways of
>increasing the amount of energy that nuclear microbatteries can produce,
>especially as the conversion efficiency begins approaching our targeted 20
>percent. One possibility for improving the cantilever-based system would be
>to scale up the number of cantilevers by placing several of them
>horizontally, side by side. In fact, we are already developing an array
>about the size of a postage stamp containing a million cantilevers. These
>arrays could then be stacked to achieve even greater integration.
>
>Another major challenge is to have inexpensive radioisotope power supplies
>that can be easily integrated into electronic devices. For example, in our
>experimental systems we have been using 1 millicurie of nickel-63, which
>costs about US $25-too much for use in a mass-produced device. A
>potentially cheaper alternative would be tritium, which some nuclear
>reactors produce in huge quantities as a byproduct. There's no reason that
>the amount of tritium needed for a microbattery couldn't cost just a few
>cents.
>
>Once these challenges are overcome, a promising use for nuclear
>microbatteries would be in handheld devices like cellphones and PDAs. As
>mentioned above, the nuclear units could trickle charge into conventional
>batteries. Our one-cantilever system generated pulses with a peak power of
>100 milliwatts; with many more cantilevers, and by using the energy of
>pulses over periods of hours, a nuclear battery would be able to inject a
>significant amount of current into the handheld's battery.
>
>How much that current could increase the device's operation time depends on
>many factors. For a cellphone used for hours every day or for a
>power-hungry PDA, the nuclear energy boost won't help much. But for a
>cellphone used two or three times a day for a few minutes, it could mean
>the difference between recharging the phone every week or so and recharging
>it once a month. And for a simple PDA used mainly for checking schedules
>and phone numbers, the energy boost might keep the batteries perpetually
>charged for as long as the nuclear material lasts.
>
>Nuclear microbatteries won't replace chemical batteries. But they're going
>to power a whole new range of gadgetry, from nanorobots to wireless
>sensors. Feynman's "staggeringly small world" awaits.
>
>FOR MORE INFORMATION (appended by IRI)
>
>See Nuclear Solutions website for details of the "betavoltaic battery"
>invented by Dr. Paul Brown. Brown's patents include "Layered Metal Foil
>Semiconductor Power Device" #6,118,204, "Isotopic Semiconductor Batteries"
>#6,236,812, "Apparatus for Direct Conversion of Radioactive Decay Energy to
>Electrical Energy" #4,835,433 at www.uspto.gov .
>
>Other nuclear beta particle battery patents include:
>
> a.. Lucent Technologies' "Self-Powered Device" #5,642,014
> b.. Ohmart's "Radio Electric Generator" #2,696,564, "Radioactive Battery
>with Chemically Dissimilar Electrodes" #3,019,358 and "Method and Apparatus
>for Converting Ionic Energy into Electrical Energy" #3,142,254
> c.. Linder's "Method and Means for Generating and/or Controlling
>Electrical Energy" #2,548,225
>All of the above are included in IRI's newest report - "Nuclear Batteries:
>A Portable Energy Source"
>
>
>--------------------------------------------------------------------------------
>
>
>
>- Become a member of www.IntegrityResearchInstitute.org during our 2005
>Membership Drive and receive two (2) free bonus products, besides the
>framable certificate, newsletters, annual report, calendar and catalog. See
>our website for more details. Join today!
>
>
>
>--------------------------------------------------------------------------------
>
>
>No virus found in this incoming message.
>Checked by AVG Anti-Virus.
>Version: 7.0.296 / Virus Database: 265.8.6 - Release Date: 7/2/2005
>
>No virus found in this outgoing message.
>Checked by AVG Anti-Virus.
>Version: 7.0.296 / Virus Database: 265.8.6 - Release Date: 7/2/2005




SUBJECT: Re: [ciencialist] Re: Uma mente brilhante - o filme
FROM: Marcos Ludwig <reinada@gmail.com>
TO: ciencialist@yahoogrupos.com.br
DATE: 12/02/2005 02:20

> Emiliano, aproveitando que vc está escrevendo este artigo, me diga: vc tem ai a transcrição desta cena do filme. Ou seja, vc tem escrito o raciocinio dele no bar. Se tiver, pode postar.

achei.

"Nash: If we all go for the blonde and block each other, not a single
one of us is going to get her. So then we go for her friends, but they
will all give us the cold shoulder because no on likes to be second
choice. But what if none of us goes for the blonde? We won't get in
each other's way and we won't insult the other girls. It's the only
way to win. It's the only way we all get laid."

tem a outra cena que antecede a esta, em que ele leva o tapa da garota
do bar que ele tenta pular as "atividades platônicas que antecedem a
cópula":

"Nash: I don't exactly know what I am required to say in order for you
to have intercourse with me. But could we assume that I said all that.
I mean essentially we are talking about fluid exchange right? So could
we go just straight to the sex. "

extraído daqui: http://www.imdb.com/title/tt0268978/quotes

[]!
--
marcos ludwig [ ICQ# 5967606 ] [ marcosludwig@gmail.com ]
rei nada - apologias: [ www.sentinelas.org/reinada ]


SUBJECT: Re: Uma mente brilhante - o filme
FROM: marcelo ferrari <emailferrari@yahoo.com.br>
TO: ciencialist@yahoogrupos.com.br
DATE: 12/02/2005 07:58

Valeu Marcos!

Se entendi bem, o raciocinio dele é o seguinte: Duas ou mais pessoas podem safisazer o mesmo desejo (transar), mas não com o mesmo objeto de desejo (a loira), pois um acabará bloqueando o outro. Assim, para que todos possam satisfazer seus desejos, devem se conscentrar no desejo em si e não no objeto de desejo.

Outra interpretação é a seguinte: Se duas ou mais pessoas tiverem como primeira escolha a primeira opção, ninguém sairá ganhando, pois um acabará bloqueando o outro. Se duas ou mais pessoas tiverem como segunda escolha a segunda opção, serão descartados e ninguém sairá ganhando também, pois a segunda opção nao pode saber que é segunda opção. Porém, se duas ou mais pessoas tiverem como primeira escolha a segunda opção, todos saem ganhando, pois ninguém bloqueia ninguém e a segunda opçao não fica sabendo que é segunda opção.


Quem concorda? Discorda? Tem outra interpretação?


marcelo ferrari
















Marcos Ludwig <reinada@gmail.com> wrote:> Emiliano, aproveitando que vc está escrevendo este artigo, me diga: vc tem ai a transcrição desta cena do filme. Ou seja, vc tem escrito o raciocinio dele no bar. Se tiver, pode postar.

achei.

"Nash: If we all go for the blonde and block each other, not a single
one of us is going to get her. So then we go for her friends, but they
will all give us the cold shoulder because no on likes to be second
choice. But what if none of us goes for the blonde? We won't get in
each other's way and we won't insult the other girls. It's the only
way to win. It's the only way we all get laid."

tem a outra cena que antecede a esta, em que ele leva o tapa da garota
do bar que ele tenta pular as "atividades platônicas que antecedem a
cópula":

"Nash: I don't exactly know what I am required to say in order for you
to have intercourse with me. But could we assume that I said all that.
I mean essentially we are talking about fluid exchange right? So could
we go just straight to the sex. "

extraído daqui: http://www.imdb.com/title/tt0268978/quotes

[]!
--
marcos ludwig [ ICQ# 5967606 ] [ marcosludwig@gmail.com ]
rei nada - apologias: [ www.sentinelas.org/reinada ]


##### ##### #####

Para saber mais visite
http://www.ciencialist.hpg.ig.com.br


##### ##### ##### #####


Yahoo! Grupos, um serviço oferecido por: São Paulo Rio de Janeiro Curitiba Porto Alegre Belo Horizonte Brasília

---------------------------------
Links do Yahoo! Grupos

Para visitar o site do seu grupo na web, acesse:
http://br.groups.yahoo.com/group/ciencialist/

Para sair deste grupo, envie um e-mail para:
ciencialist-unsubscribe@yahoogrupos.com.br

O uso que você faz do Yahoo! Grupos está sujeito aos Termos do Serviço do Yahoo!.



__________________________________________________
Converse com seus amigos em tempo real com o Yahoo! Messenger
http://br.download.yahoo.com/messenger/

[As partes desta mensagem que não continham texto foram removidas]



SUBJECT: Re: [ciencialist] Re: Uma mente brilhante - o filme
FROM: JVictor <jvoneto@uol.com.br>
TO: ciencialist@yahoogrupos.com.br
DATE: 12/02/2005 08:28

marcelo ferrari escreveu:

> Valeu Marcos!
>
> Se entendi bem, o raciocinio dele é o seguinte: Duas ou mais pessoas
> podem safisazer o mesmo desejo (transar), mas não com o mesmo objeto
> de desejo (a loira), pois um acabará bloqueando o outro. Assim, para
> que todos possam satisfazer seus desejos, devem se conscentrar no
> desejo em si e não no objeto de desejo.
>
> Outra interpretação é a seguinte: Se duas ou mais pessoas tiverem como
> primeira escolha a primeira opção, ninguém sairá ganhando, pois um
> acabará bloqueando o outro. Se duas ou mais pessoas tiverem como
> segunda escolha a segunda opção, serão descartados e ninguém sairá
> ganhando também, pois a segunda opção nao pode saber que é segunda
> opção. Porém, se duas ou mais pessoas tiverem como primeira escolha a
> segunda opção, todos saem ganhando, pois ninguém bloqueia ninguém e a
> segunda opçao não fica sabendo que é segunda opção.
>
>
> Quem concorda? Discorda? Tem outra interpretação?
>
Victor: É por aí, Marcelo. Há uma história antiga, que começou com uma
sentença de Salomão, segundo os inventores da bíblia, que, hoje, é
contada assim, de maneira adaptada. Uma mãe vivia querendo saber como
resolver o problema de duas irmãs brigonas, que reclamavam de qualquer
coisa da outra. Um dia, deu-lhes um pedaço de bolo, para lanche. Mas
surgiu um problema. Uma dizia que se a outra o cortasse, o pedaço maior
seria da que cortou, e vice-versa. Um problema! A mãe, então, teve uma
idéia, brilhante, e que resume o pensamento de Nash. Decretou: uma de
vocês corta o bolo, e a outra escolhe seu pedaço! Veja aí: aquela que
vai cortar o bolo tem que se esmerar para produzir dois pedaços iguais!
Sem outra opção. Parece uma idéia simples, mas é de um significado e de
uma profundidade digna de um Newton, ou uma versão superior... Outro
exemplo que especifica de maneira adequada o mesmo significado é o
seguinte(este eu lí em um dos livros de Clemente Nóbrega, um físico
nuclear que virou escritor e consultor empresarial- autor do bestseller
Em busca da Empresa Quântica): você está num shopping. Ao sair com seu
carro, você bate noutro e o amassa e o arranha feio! Você, ético que é,
honesto, se dispõe a procurar o dono ou deixar um cartão seu pregado no
carro, dispondo-se a ressarcí-lo dos prejuízos. Mas há um detalhe. O
carro é um carrão, um Mercedes, importado, ricão. O conserto deverá ser
uma nota. Então o " batedor" pode ser tentado a optar por outra
solução. Já que o cara do carrão deve ser rico, as despesas do reparo
não o incomodarão. Então você olha para um lado, para o outro, não vê
viva alma e, simplesmente, se manda. Com a consciência tranquíla, pois
acha que ficou tudo bem numa boa, ninguém se machucou e o prejuízo não
incomodará o proprietário. Esta é uma opção que, acho, a maioria das
pessoas ficariam tentadas a usar(eu, não, eu não, tô fora!). Só que, a
consequência será a seguinte: o seguro será acionado e arcará com as
despesas, ou parte delas. Mas o seguro terá de repassar esse preju, que
ele não besta. . E o faz no ano seguinte, distribuindo-o com todos os
segurados! Eis a questão. Eu e você iremos pagar, em benefício de um. O
benefício de um implicou numa distribuição negativa para a comunidade.
Há outros exemplos "inocentes" e magistrais, que você encontra num
livro sobre a teoria dos jogos, do Nóbrega. Um ganhou, em detrimento de
outros, que perderam pelo ganho desse um. Essa é a essência. Tem que ser
bom para todos, todos ganham e ficam satisfeitos. Aquele "foi bom, meu
bem?" deveria ser sempre vice-versa. A teoria de Nash deplora a
unilateralidade.

Sds,

Victor.

>
>
>
>
>
>
>
>
>
>
>
>
>
>
>
>
>
>
> Marcos Ludwig <reinada@gmail.com> wrote:> Emiliano, aproveitando que
> vc está escrevendo este artigo, me diga: vc tem ai a transcrição desta
> cena do filme. Ou seja, vc tem escrito o raciocinio dele no bar. Se
> tiver, pode postar.
>
> achei.
>
> "Nash: If we all go for the blonde and block each other, not a single
> one of us is going to get her. So then we go for her friends, but they
> will all give us the cold shoulder because no on likes to be second
> choice. But what if none of us goes for the blonde? We won't get in
> each other's way and we won't insult the other girls. It's the only
> way to win. It's the only way we all get laid."
>
> tem a outra cena que antecede a esta, em que ele leva o tapa da garota
> do bar que ele tenta pular as "atividades platônicas que antecedem a
> cópula":
>
> "Nash: I don't exactly know what I am required to say in order for you
> to have intercourse with me. But could we assume that I said all that.
> I mean essentially we are talking about fluid exchange right? So could
> we go just straight to the sex. "
>
> extraído daqui: http://www.imdb.com/title/tt0268978/quotes
>
> []!
> --
> marcos ludwig [ ICQ# 5967606 ] [ marcosludwig@gmail.com ]
> rei nada - apologias: [ www.sentinelas.org/reinada ]
>
>
> ##### ##### #####
>
> Para saber mais visite
> http://www.ciencialist.hpg.ig.com.br
>
>
> ##### ##### ##### #####
>
>
> Yahoo! Grupos, um serviço oferecido por: São Paulo Rio de Janeiro
> Curitiba Porto Alegre Belo Horizonte Brasília
>
> ---------------------------------
> Links do Yahoo! Grupos
>
> Para visitar o site do seu grupo na web, acesse:
> http://br.groups.yahoo.com/group/ciencialist/
>
> Para sair deste grupo, envie um e-mail para:
> ciencialist-unsubscribe@yahoogrupos.com.br
>
> O uso que você faz do Yahoo! Grupos está sujeito aos Termos do
> Serviço do Yahoo!.
>
>
>
> __________________________________________________
> Converse com seus amigos em tempo real com o Yahoo! Messenger
> http://br.download.yahoo.com/messenger/
>
> [As partes desta mensagem que não continham texto foram removidas]
>
>
>
> ##### ##### #####
>
> Para saber mais visite
> http://www.ciencialist.hpg.ig.com.br
>
>
> ##### ##### ##### #####
>
>
> *Yahoo! Grupos, um serviço oferecido por:*
>
> *
> <http://br.rd.yahoo.com/SIG=12ao2upt3/M=264105.3931087.6562589.1588051/D=brclubs/S=2137111528:HM/EXP=1108288732/A=2361264/R=6/SIG=10v4acpp0/*http://br.shopping.yahoo.com/>*
>
>
>
> ------------------------------------------------------------------------
> *Links do Yahoo! Grupos*
>
> * Para visitar o site do seu grupo na web, acesse:
> http://br.groups.yahoo.com/group/ciencialist/
>
> * Para sair deste grupo, envie um e-mail para:
> ciencialist-unsubscribe@yahoogrupos.com.br
> <mailto:ciencialist-unsubscribe@yahoogrupos.com.br?subject=Unsubscribe>
>
> * O uso que você faz do Yahoo! Grupos está sujeito aos Termos do
> Serviço do Yahoo! <http://br.yahoo.com/info/utos.html>.
>
>
>
>
> __________ Informação do NOD32 1.997 (20050211) __________
>
> Esta mensagem foi verificada pelo NOD32 Sistema Antivírus
> http://www.nod32.com.br




SUBJECT: RE: [ciencialist] Re: Uma mente brilhante - o filme
FROM: "murilo filo" <avalanchedrive@hotmail.com>
TO: ciencialist@yahoogrupos.com.br
DATE: 12/02/2005 11:02

Muito boa esta descritiva!
Para os pragmáticos vão valer muito os atos conscientes, e para os menos
conscientes valerá mais a lei da diversidade.
Há mulher que adora careca, há gente que procura os fedidos, os muito
magros, os muito gordos, os broncos... ou pela semelhança ou pela
oposição... e ficam todos com seus meios de satisfação. abr/M.

>From: marcelo ferrari <emailferrari@yahoo.com.br>
>Reply-To: ciencialist@yahoogrupos.com.br
>To: ciencialist@yahoogrupos.com.br
>Subject: [ciencialist] Re: Uma mente brilhante - o filme
>Date: Sat, 12 Feb 2005 06:58:49 -0300 (ART)
>
>Valeu Marcos!
>
>Se entendi bem, o raciocinio dele é o seguinte: Duas ou mais pessoas podem
>safisazer o mesmo desejo (transar), mas não com o mesmo objeto de desejo (a
>loira), pois um acabará bloqueando o outro. Assim, para que todos possam
>satisfazer seus desejos, devem se conscentrar no desejo em si e não no
>objeto de desejo.
>
>Outra interpretação é a seguinte: Se duas ou mais pessoas tiverem como
>primeira escolha a primeira opção, ninguém sairá ganhando, pois um acabará
>bloqueando o outro. Se duas ou mais pessoas tiverem como segunda escolha a
>segunda opção, serão descartados e ninguém sairá ganhando também, pois a
>segunda opção nao pode saber que é segunda opção. Porém, se duas ou mais
>pessoas tiverem como primeira escolha a segunda opção, todos saem ganhando,
>pois ninguém bloqueia ninguém e a segunda opçao não fica sabendo que é
>segunda opção.
>
>
>Quem concorda? Discorda? Tem outra interpretação?
>
>
>marcelo ferrari
>
>
>
>
>
>
>
>
>
>
>
>
>
>
>
>
>Marcos Ludwig <reinada@gmail.com> wrote:> Emiliano, aproveitando que vc
>está escrevendo este artigo, me diga: vc tem ai a transcrição desta cena do
>filme. Ou seja, vc tem escrito o raciocinio dele no bar. Se tiver, pode
>postar.
>
>achei.
>
>"Nash: If we all go for the blonde and block each other, not a single
>one of us is going to get her. So then we go for her friends, but they
>will all give us the cold shoulder because no on likes to be second
>choice. But what if none of us goes for the blonde? We won't get in
>each other's way and we won't insult the other girls. It's the only
>way to win. It's the only way we all get laid."
>
>tem a outra cena que antecede a esta, em que ele leva o tapa da garota
>do bar que ele tenta pular as "atividades platônicas que antecedem a
>cópula":
>
>"Nash: I don't exactly know what I am required to say in order for you
>to have intercourse with me. But could we assume that I said all that.
>I mean essentially we are talking about fluid exchange right? So could
>we go just straight to the sex. "
>
>extraído daqui: http://www.imdb.com/title/tt0268978/quotes
>
>[]!
>--
>marcos ludwig [ ICQ# 5967606 ] [ marcosludwig@gmail.com ]
>rei nada - apologias: [ www.sentinelas.org/reinada ]
>
>
>##### ##### #####
>
>Para saber mais visite
>http://www.ciencialist.hpg.ig.com.br
>
>
>##### ##### ##### #####
>
>
>Yahoo! Grupos, um serviço oferecido por: São Paulo Rio de Janeiro Curitiba
>Porto Alegre Belo Horizonte Brasília
>
>---------------------------------
>Links do Yahoo! Grupos
>
> Para visitar o site do seu grupo na web, acesse:
>http://br.groups.yahoo.com/group/ciencialist/
>
> Para sair deste grupo, envie um e-mail para:
>ciencialist-unsubscribe@yahoogrupos.com.br
>
> O uso que você faz do Yahoo! Grupos está sujeito aos Termos do Serviço
>do Yahoo!.
>
>
>
>__________________________________________________
>Converse com seus amigos em tempo real com o Yahoo! Messenger
>http://br.download.yahoo.com/messenger/
>
>[As partes desta mensagem que não continham texto foram removidas]
>




SUBJECT: Uma mente brilhante - The Edukators
FROM: "L.E.R.de Carvalho" <lecarvalho@infolink.com.br>
TO: ciencialist@yahoogrupos.com.br
DATE: 12/02/2005 17:24


>Victor: É por aí, Marcelo. Há uma história antiga, que começou com uma
>sentença de Salomão, segundo os inventores da bíblia, que, hoje, é
>contada assim, de maneira adaptada. Uma mãe vivia querendo saber como
>resolver o problema de duas irmãs brigonas, que reclamavam de qualquer
>coisa da outra. Um dia, deu-lhes um pedaço de bolo, para lanche. Mas
>surgiu um problema. Uma dizia que se a outra o cortasse, o pedaço maior
>seria da que cortou, e vice-versa. Um problema! A mãe, então, teve uma
>idéia, brilhante, e que resume o pensamento de Nash. Decretou: uma de
>vocês corta o bolo, e a outra escolhe seu pedaço! Veja aí: aquela que
>vai cortar o bolo tem que se esmerar para produzir dois pedaços iguais!
>Sem outra opção. Parece uma idéia simples, mas é de um significado e de
>uma profundidade digna de um Newton, ou uma versão superior... Outro
>exemplo que especifica de maneira adequada o mesmo significado é o
>seguinte(este eu lí em um dos livros de Clemente Nóbrega, um físico
>nuclear que virou escritor e consultor empresarial- autor do bestseller
>Em busca da Empresa Quântica): você está num shopping. Ao sair com seu
>carro, você bate noutro e o amassa e o arranha feio! Você, ético que é,
>honesto, se dispõe a procurar o dono ou deixar um cartão seu pregado no
>carro, dispondo-se a ressarcí-lo dos prejuízos. Mas há um detalhe. O
>carro é um carrão, um Mercedes, importado, ricão. O conserto deverá ser
>uma nota. Então o " batedor" pode ser tentado a optar por outra
>solução. Já que o cara do carrão deve ser rico, as despesas do reparo
>não o incomodarão. Então você olha para um lado, para o outro, não vê
>viva alma e, simplesmente, se manda. Com a consciência tranquíla, pois
>acha que ficou tudo bem numa boa, ninguém se machucou e o prejuízo não
>incomodará o proprietário. Esta é uma opção que, acho, a maioria das
>pessoas ficariam tentadas a usar(eu, não, eu não, tô fora!). Só que, a
>consequência será a seguinte: o seguro será acionado e arcará com as
>despesas, ou parte delas. Mas o seguro terá de repassar esse preju, que
>ele não besta. . E o faz no ano seguinte, distribuindo-o com todos os
>segurados! Eis a questão. Eu e você iremos pagar, em benefício de um. O
>benefício de um implicou numa distribuição negativa para a comunidade.
>Há outros exemplos "inocentes" e magistrais, que você encontra num
>livro sobre a teoria dos jogos, do Nóbrega. Um ganhou, em detrimento de
>outros, que perderam pelo ganho desse um. Essa é a essência. Tem que ser
>bom para todos, todos ganham e ficam satisfeitos. Aquele "foi bom, meu
>bem?" deveria ser sempre vice-versa. A teoria de Nash deplora a
>unilateralidade.
>
>Sds,
>
>Victor.




E´ Victor,

mas deveria ser proibido colocar na rua um carro de 600 mil reais,
criando-se um risco de um monte de gente pobre, professor, sem querer,
bater nesse carro e depois ter que pagar, quando mal se tem dinheiro pra
manter um carro de 6 mil reais.

sugiro trocar de filme e assistir: The Edukators.
é alemão.
está em cartaz.

L.E.

[As partes desta mensagem que não continham texto foram removidas]



SUBJECT: Uma mente brilhante - os carecas
FROM: "L.E.R.de Carvalho" <lecarvalho@infolink.com.br>
TO: ciencialist@yahoogrupos.com.br
DATE: 12/02/2005 17:26

At 11:02 12/2/2005, you wrote:
>Muito boa esta descritiva!
>Para os pragmáticos vão valer muito os atos conscientes, e para os menos
>conscientes valerá mais a lei da diversidade.
>Há mulher que adora careca, há gente que procura os fedidos, os muito
>magros, os muito gordos, os broncos... ou pela semelhança ou pela
>oposição... e ficam todos com seus meios de satisfação. abr/M.



Também não Murilo.
Hoje vocês não estão em um dia bom.

Nenhuma mulher adora carecas, magros ou fedidos.

Apenas elas por estarem gordinhas ou enrugadas ou algo assim, e sabendo que
pra arrumar um cara rico, que disso elas não abrem mão, elas terão que
abrir mão de alguma coisa, então elas fazem diferentes opções, uma abrindo
mão de um cara com cabelo, outra abrindo mão do cara ser elegante, ou
magro, ou intelectual ou...

Uma boa ciencia começa pela exata observação e descrição do fenômeno ?

L.E.

[As partes desta mensagem que não continham texto foram removidas]



SUBJECT: RE: [ciencialist] Uma mente brilhante - os carecas
FROM: "murilo filo" <avalanchedrive@hotmail.com>
TO: ciencialist@yahoogrupos.com.br
DATE: 12/02/2005 18:49

Que isso, LER...
Se a mina 'desiste' do cabeludo pelo careca, ela está consciente e tendo uma
'decisão' mais prática.
Vc tem razão, uma boa ciência começa pela boa observação e descrição...
Se vc é cabeludo, é melhor minha mulher não lhe conhecer. (:(
abr/M. SP 12/fev

>From: "L.E.R.de Carvalho" <lecarvalho@infolink.com.br>
>Reply-To: ciencialist@yahoogrupos.com.br
>To: ciencialist@yahoogrupos.com.br
>Subject: [ciencialist] Uma mente brilhante - os carecas
>Date: Sat, 12 Feb 2005 17:26:43 -0200
>
>At 11:02 12/2/2005, you wrote:
> >Muito boa esta descritiva!
> >Para os pragmáticos vão valer muito os atos conscientes, e para os menos
> >conscientes valerá mais a lei da diversidade.
> >Há mulher que adora careca, há gente que procura os fedidos, os muito
> >magros, os muito gordos, os broncos... ou pela semelhança ou pela
> >oposição... e ficam todos com seus meios de satisfação. abr/M.
>
>
>
>Também não Murilo.
>Hoje vocês não estão em um dia bom.
>
>Nenhuma mulher adora carecas, magros ou fedidos.
>
>Apenas elas por estarem gordinhas ou enrugadas ou algo assim, e sabendo que
>pra arrumar um cara rico, que disso elas não abrem mão, elas terão que
>abrir mão de alguma coisa, então elas fazem diferentes opções, uma abrindo
>mão de um cara com cabelo, outra abrindo mão do cara ser elegante, ou
>magro, ou intelectual ou...
>
>Uma boa ciencia começa pela exata observação e descrição do fenômeno ?
>
>L.E.
>
>[As partes desta mensagem que não continham texto foram removidas]
>




SUBJECT: Re: [ciencialist] Uma mente brilhante - os carecas
FROM: "Alvaro Augusto \(E\)" <alvaro@electraenergy.com.br>
TO: <ciencialist@yahoogrupos.com.br>
DATE: 12/02/2005 19:24

Caro L.E.,

Não se esqueça que a frase tem um sentido oculto. Quem a formulou não se referia necessariamente a carecas na cabeça (de cima)...

[ ]s

Alvaro Augusto

----- Original Message -----
From: L.E.R.de Carvalho
To: ciencialist@yahoogrupos.com.br
Sent: Saturday, February 12, 2005 5:26 PM
Subject: [ciencialist] Uma mente brilhante - os carecas


At 11:02 12/2/2005, you wrote:
>Muito boa esta descritiva!
>Para os pragmáticos vão valer muito os atos conscientes, e para os menos
>conscientes valerá mais a lei da diversidade.
>Há mulher que adora careca, há gente que procura os fedidos, os muito
>magros, os muito gordos, os broncos... ou pela semelhança ou pela
>oposição... e ficam todos com seus meios de satisfação. abr/M.



Também não Murilo.
Hoje vocês não estão em um dia bom.

Nenhuma mulher adora carecas, magros ou fedidos.

Apenas elas por estarem gordinhas ou enrugadas ou algo assim, e sabendo que
pra arrumar um cara rico, que disso elas não abrem mão, elas terão que
abrir mão de alguma coisa, então elas fazem diferentes opções, uma abrindo
mão de um cara com cabelo, outra abrindo mão do cara ser elegante, ou
magro, ou intelectual ou...

Uma boa ciencia começa pela exata observação e descrição do fenômeno ?

L.E.

[As partes desta mensagem que não continham texto foram removidas]



[As partes desta mensagem que não continham texto foram removidas]



SUBJECT: THERE IS NO MATTER AS SUCH! (Max Planck)
FROM: "marcelomjr" <marcelomjr@yahoo.com.br>
TO: ciencialist@yahoogrupos.com.br
DATE: 12/02/2005 21:48


http://www.worlditc.org/h_07_spiri_093_095.htm

Considerations Regarding the Ether

History has shown time and again that important scientific
discoveries generally happen only when one steps outside his regular
discipline and looks at something from a fresh point of view. Then
what should have been obvious all along comes into focus.
Capt. Edgar D. Mitchell
American Astronaut

(...)

Einstein's illustrious contemporary, Nobel prize winner Max Planck,
during a lecture in Florence, Italy, once made a truly remarkable
statement which describes the problem facing the physicist today:

As a man who has devoted his whole life to the most clear-headed
science, to the study of matter, I can tell you as the result of my
research about the atoms, this much:

THERE IS NO MATTER AS SUCH!

All matter originates and exists only because by virtue of a force
which brings the particles of an atom to vibration and hold this most
minute solar system of the atom together ... We must assume behind
this force the existence of a conscious and intelligent mind. This
mind is the matrix of all matter. *

(...)





SUBJECT: [RN] DIA DE DARWIN
FROM: "rmtakata" <rmtakata@altavista.net>
TO: ciencialist@yahoogrupos.com.br
DATE: 12/02/2005 22:56


O ESTADO DE SAO PAULO, 11-02-2005

Karine Rodrigues

Rio – Hoje é dia de celebrar a Teoria da Evolução das Espécies e seu
autor, Charles Darwin, que nasceu há 196 anos. A data ganhou status
especial há dez anos, com o lançamento do Dia de Darwin – uma reação à
disseminação do movimento criacionista, que se contrapõe aos
fundamentos do cientista inglês ao explicar a origem do universo e do
homem com base na interpretação da Bíblia.
Pela primeira vez, o Dia de Darwin, iniciativa de Robert Stephens, da
Stanford University, será celebrado no Brasil. E a escolha do Rio não
é mero acaso, mas algo 'providencial', segundo a bióloga Isabel
Landim, organizadora do evento. 'Vamos comemorar a personalidade de
Darwin, mas, principalmente, afirmar o valor da Teoria da Evolução,
que vem sendo desaqualificada pelos criacionistas, cada vez mais
atuantes no mundo. Até no Brasil, país laico e liberal. No Rio,
chegou-se ao absurdo de ensinar essa teoria nas escolas públicas.'
A proposta de ensiná-la na rede estadual surgiu em 2000, quando o
então governador, Anthony Garotinho, presbiteriano, sancionou uma lei
instituindo o ensino religioso confessional (com aulas separadas por
credo). A atual governadora e mulher dele, Rosinha Matheus, deu
continuidade ao projeto e, no ano passado, fez concurso para contratar
500 professores de religião – 342 católicos, 132 evangélicos e 26 de
outros credos. 'Estão misturando ciência com religião', critica Isabel.
A participação brasileira na comemoração será uma palestra, às 21
horas, na Livraria da Travessa de Ipanema, do biólogo Mário de Pinna,
diretor-científico do Museu de Zoologia da Universidade de São Paulo.
'Ele vai falar sobre a importância da Teoria da Evolução. Precisamos
nos mobilizar para mostrar à sociedade que o que os criacionistas
divulgam é totalmente infundado', diz Isabel.
'Criamos uma vitrine temática e vamos oferecer um coquetel, afinal,
trata-se de um aniversário', conta o dono da livraria, Rui Campos. A
data será comemorada em mais 11 países, entre eles os Estados Unidos,
onde surgiu o criacionismo.

--------
(Bem, o criacionismo nao surgiu exatamente nos EUA. O q. firmou lah
foi o fundamentalismo cristao, q. brande a bandeira do criacionismo.)

Roberto Takata





SUBJECT: Re: [ciencialist] Uma mente brilhante - os carecas
FROM: JVictor <jvoneto@uol.com.br>
TO: ciencialist@yahoogrupos.com.br
DATE: 12/02/2005 23:29

>
> L.E.R

Você me retornou um e-mail, sobre o carro da cara rico, que o pobre
arranha e quer se mandar. Para os outros pagarem. Não sei o que houve,
mas, numa zevitada, perdí aquele teu e-mail. Mas falava em salário de
professor, essas coisas. Pois, para você ficar mais tranquílo, quero
dizer que prof. de física merece até poema, como a Balada do Professor
de Física, abaixo. É uma grande homenagem. Depois dessa, ganhar bem com
a profissão é até irrelevante!!!....

Balada do Professor de Física
Adaptação livre do original de D. F. Kirwan, publicado em *The Physics
Teacher** 24*, 8 (1986).

Avogadro tem um número
Coulomb uma força tem
Newton tem algumas leis
E vários anéis também.

Kirchhoff tem umas regras
Legendre, uma transformação
Balmer tem a sua série
E Ewald, uma construção.

Huyghens tem suas ondinhas
Schroedinger, uma equação
Rydberg, uma constante
Lagrange, a menor ação.

Foucault pendurou um pêndulo
Uma rede tem Bravais
Gauss tem uma superfície
Fermi, um nível que dá pé.

Fermat tem o seu princípio
Poynting tem o seu vetor
Bose tem uma estatística
Van Der Graaff, um gerador.

Brown tem seu movimento
Geiger tem seu contador
Cavendish, uma balança
E Helmoltz, um ressoador.

Clebsch-Gordon tem coeficientes
E Doppler, um deslocamento
Dirac tem suas matrizes
E Raman, um espalhamento.

Stark tem o seu efeito
Fourier, um teorema
Young tem o seu módulo
E Gell-Mann, o seu esquema.

Rayleigh tem o seu critério
Poiseuille, um fluxo legal
Gibbs tem sua energia
E Cornu, uma espiral.

Zener tem o seu diodo
Bohr tem um raio, pois não?
Brewster arrumou um ângulo
Fitzgerald, uma contração.

Brillouin tem sua zona
Quem tem um ciclo é Carnot
Michelson, um interferômetro
E um teste que falhou.

Atwood tem uma máquina
Josephson, uma junção
Planck lançou uma hipótese
E Green fez uma função.

Rutherford criou um átomo
Tesla fez uma bobina
Fresnel construiu uma lente
E Zeeman, uma estrutura fina.

Maxwell tem o seu demônio
Thomas tem uma precessão
Heisenberg, uma incerteza
E Boltzmann, uma distribuição.

Ising achou o seu modelo
Debye, uma temperatura
Einstein, uma teoria geral
Que ainda hoje perdura.

* * Pauli foi excluído por princípio.

E tu, pobre professor
Onde é fica teu nome?
Será no giz que apagas
Ou em teu salário de fome?

A aula de cada dia
Fama nunca te trará
E do muito que ensinares
Muito pouco restará.

Mas, se não vais deixar marca
Nem leis à posteridade
Que ao menos os teus alunos
Lembrem de ti com saudade.

Sds

Victor.



> [As partes desta mensagem que não continham texto foram removidas]
>
>
>
> ##### ##### #####
>
> Para saber mais visite
> http://www.ciencialist.hpg.ig.com.br
>
>
> ##### ##### ##### #####
>
>
> *Yahoo! Grupos, um serviço oferecido por:*
>
> *
> <http://br.rd.yahoo.com/SIG=12ann8kk0/M=264105.3931087.6562589.1588051/D=brclubs/S=2137111528:HM/EXP=1108322862/A=2361264/R=6/SIG=10v4acpp0/*http://br.shopping.yahoo.com/>*
>
>
>
> ------------------------------------------------------------------------
> *Links do Yahoo! Grupos*
>
> * Para visitar o site do seu grupo na web, acesse:
> http://br.groups.yahoo.com/group/ciencialist/
>
> * Para sair deste grupo, envie um e-mail para:
> ciencialist-unsubscribe@yahoogrupos.com.br
> <mailto:ciencialist-unsubscribe@yahoogrupos.com.br?subject=Unsubscribe>
>
> * O uso que você faz do Yahoo! Grupos está sujeito aos Termos do
> Serviço do Yahoo! <http://br.yahoo.com/info/utos.html>.
>
>
>
>
> __________ Informação do NOD32 1.998 (20050212) __________
>
> Esta mensagem foi verificada pelo NOD32 Sistema Antivírus
> http://www.nod32.com.br




SUBJECT: Pra biólogo. Barcode
FROM: Maria Natália <grasdic@hotmail.com>
TO: ciencialist@yahoogrupos.com.br
DATE: 13/02/2005 02:54


Lançamento oficial do "Barcode" em Londres


Cientistas querem criar catálogo gigante com todas as espécies do planeta


Uma equipa internacional de cientistas lançou hoje oficialmente em
Londres o
"Barcode", projecto que pretende criar um catálogo gigante da vida,
com todas as
espécies do planeta, através da transformação da sua informação
genética em
códigos de barras.

Os cientistas vão vincular os segmentos de ADN das espécies a fotografias,
descrições e todo o tipo de informação sobre cada animal e planta.

A iniciativa arranca com três projectos: um relativo a aves (dez mil
espécies),
outro a peixes (15 mil espécies marinhas e oito mil de água doce) e outro
projecto vai rotular geneticamente as oito mil espécies de plantas na
Costa
Rica.

"Actualmente conhecemos 1,7 milhões de espécies mas suspeitamos que
existam
entre dez a 30 milhões no planeta", disse hoje Richard Lane, director
de Ciência
do Museu de História Natural de Londres, na conferência internacional
para a
Codificação da Vida.

"Descobrimos que é possível termos uma pequena sequência de ADN que
caracterize
todas as formas de vida do planeta", acrescentou.

Dan Janzen, professor da Universidade de Pensilvânia, explicou que o
"barcode"
"é o que liga o volume de informação que os taxonomistas, especialistas em
história natural e biólogos têm vindo a acumular nos últimos 200 anos".

Os códigos de barras de ADN poderão facilitar a identificação de
espécies no
terreno, especialmente em casos onde os métodos tradicionais não são
eficientes,
e torná-la mais credível.

A iniciativa é de um Consórcio que reúne museus, jardins zoológicos,
herbários,
agências governamentais e outras organizações que estudam a biodiversidade
mundial. A estrutura (CBOL - Consortium for the Barcode of Life )
espera vir a
descobrir novas espécies e melhorar o conhecimento sobre a evolução e
ecologia,
bases para melhores políticas de gestão ambiental.

Desde a sua primeira reunião, em Maio de 2004, o consórcio conta com 45
organizações de 22 países em seis continentes

http://ultimahora.publico.pt/shownews.asp?id=1215428


E para quando uns biólogos a se interessar por astronomia? LOLLLLLLL
Um abraço
Maria Natália numa de Zodiacal





SUBJECT: Re: Uma mente brilhante - os carecas
FROM: Maria Natália <grasdic@hotmail.com>
TO: ciencialist@yahoogrupos.com.br
DATE: 13/02/2005 03:03


JVictor:
E conheces a balada do professor de Química aqui para mim, Emiliano e
brudna?
Se não há temos de fazer uma mas bem EXPLOSIVA. Vai capachinho de
careca e tudo pelos ares.
Boa
Um abraço
Maria Natália
--- Em ciencialist@yahoogrupos.com.br, JVictor <jvoneto@u...> escreveu
> >
> > L.E.R
>
> Você me retornou um e-mail, sobre o carro da cara rico, que o pobre
> arranha e quer se mandar. Para os outros pagarem. Não sei o que houve,
> mas, numa zevitada, perdí aquele teu e-mail. Mas falava em salário de
> professor, essas coisas. Pois, para você ficar mais tranquílo, quero
> dizer que prof. de física merece até poema, como a Balada do Professor
> de Física, abaixo. É uma grande homenagem. Depois dessa, ganhar bem
com
> a profissão é até irrelevante!!!....
>
> Balada do Professor de Física
> Adaptação livre do original de D. F. Kirwan, publicado em *The Physics
> Teacher** 24*, 8 (1986).
>
> Avogadro tem um número
> Coulomb uma força tem
> Newton tem algumas leis
> E vários anéis também.
>
> Kirchhoff tem umas regras
> Legendre, uma transformação
> Balmer tem a sua série
>





SUBJECT: As rosas de S.Valentim = Dia dos namorados desta lista
FROM: Maria Natália <grasdic@hotmail.com>
TO: ciencialist@yahoogrupos.com.br
DATE: 13/02/2005 03:09


Pois desculpai Leo , amigo mas vai sair língua de hunos.
Q"uerias oferecer umas rosinhas à namorada? Pois não sei e esta
notícia me chegou via Universidade Católica, polo do Porto de
Biotecnologia. Direi apenas "Até as rosas meu senhor?" Se lembram de
rainha Santa Isabel falando com o D. Diniz?

""Valentine's Day Sweatshops
By Russell Mokhiber and Robert Weissman

Before you buy your sweetie those roses for Valentine's Day, pause for a
moment to consider where they come from, and at what cost -- and what
can be done to give a bit more joy not just to the flowers' recipients,
but their producers.

Cut flowers are a highly globalized industry. The majority of cut
flowers sold in the United States are imported, especially from Colombia
and Ecuador. Kenya and Tanzania are the key overseas supplier for Europe.

Here's how the industry looks from the multinational corporate
perspective: "In just a 24-hour period, each stem is cut, packed and
loaded onto a temperature controlled UPS aircraft heading to Miami.
There, they clear customs and are distributed to florists and consumers
across the country. Eighty-seven percent of all cut flower imports
arrive in Miami." UPS reports that it imported more than 14.8 million
stems of cut flowers into the United States last year from South
American countries such as Colombia and Ecuador.

But on the ground in Colombia and Ecuador, things don't look so smoothly
efficient and trouble free.

Olga Tutillo is secretary general of Rosas del Ecuador, a flower workers
union in Ecuador. She has worked at flower plantations for 22 years. She
is 38 years old and has five children.

Tutillo explains how hard the work is for Ecuador's roughly 100,000
flower workers, about 70 percent of whom are women -- the faces behind
Cupid. The International Labor Organization estimates about 20 percent
of the workforce consists of children.

The workers generally earn the national minimum wage, $145 per month.
They work especially long hours in advance of Valentine's Day and other
flower-giving holidays in the United States. They experience major
occupational risks. Back pain is common among those who must stand or
lean all day. Repetitive motion injuries are common. Rose pickers are
frequently cut by thorns.

"There are also problems caused by pesticide fumigation," she explains.
"Fumigation happens every day, either to prevent the plants from getting
different diseases or to deal with it when they do get those diseases.
Some of these chemicals are highly toxic."

Flower workers who try to organize to improve their working conditions
face severe repression.

"It is extremely difficult to unionize in Ecuador," says Tutillo. "The
companies are organized among themselves and they have a list on the
Internet of the people who have tried to unionize or have unionized. If
someone tries to create a union, the company threatens to fire them and
says they won't be able to find another job. These are the famous
blacklists."

Thanks to firings, blacklisting and other tactics -- like increasing use
of contract workers instead of full-fledged employees -- the
unionization rate in Ecuador is depressingly low. Among 300 flower
companies in Ecuador, reports Tutillo, "only four have unions -- the
other attempts to unionize have been repressed."

The story is much the same in Colombia, says Ricardo Zamudio, president
of Cactus, a Colombian organization that conducts research on issues
related to the flower industry.

Workers are trying to organize despite the repression they face. In
Colombian a recent important development has been independent
unionization at one flower company owned by Dole, which altogether
controls 20 percent of Colombia's flower exports. The International
Labor Rights Fund (ILRF) is running a letter-writing campaign to urge
Dole Fresh Flowers and the Colombian-based firm Splendor Flowers to
respect workers' right to unionize <www.laborrights.org>.

Unfortunately, as long as the repression remains intense, consumers have
much more freedom to demand flower justice than do the flower workers.

In Europe, a flower certification program has taken hold that tells
consumers whether flowers were grown on farms or plantations that
respect minimal environmental and labor conditions. According to the
International Labor Organization, a substantial portion of flowers grown
in Kenya, Tanzania and Zimbabwe receive certification under the Flower
Label Program. The flower certification program is no panacea, but it
does help modestly improve environmental and working conditions, and it
gives workers more space to organize.

The program has had much less impact in South America, in considerable
part because the Flower Label Program hasn't taken hold in the United
States, where most Colombian and Ecuadorian flowers are shipped.

Just like with sweatshops, consumer pressure can make a significant
difference in the lives of the flower workers. But the opportunity is in
some ways greater, because of the concentration among both flower
producers and sellers. ILRF is leading the way, trying to galvanize
consumer pressure to force Dole and large cut flower sellers --
Albertson's, Safeway, Costco and Wal-Mart, among others -- to pressure
flower suppliers to respect workers' rights to organize, protect
employees' health and safety, and pay overtime wages.

So go ahead and give that rose for Valentine's Day. But be careful of
the thorns -- and to avoid sticking it to the flower workers, support
the ILRF campaign.


Russell Mokhiber is editor of the Washington, D.C.-based Corporate Crime
Reporter, <http://www.corporatecrimereporter.com>. Robert Weissman is
editor of the Washington, D.C.-based Multinational Monitor,
<http://www.multinationalmonitor.org>. Mokhiber and Weissman are
co-authors of On the Rampage: Corporate Predators and the Destruction of
Democracy (Monroe, Maine: Common Courage Press).

(c) Russell Mokhiber and Robert Weissman""

Olhai os espinhos, senhores
Maria Natália








SUBJECT: Re: Uma mente brilhante - os carecas
FROM: Maria Natália <grasdic@hotmail.com>
TO: ciencialist@yahoogrupos.com.br
DATE: 13/02/2005 03:17


Alvaro:
Cabeça de cima???? Ai...que já me deu LOLLLLLLL
Mas só me resta acrescentar: É dos carecas que elas gostam. E sabem
porquê?
Pois é simples. Existe uma regra quase universal que diz que um dos
sinais de abastança para além de se criar barriga é o facto de se não
ter cabelo, seja, na cabeça de cima. Logo mulher vai à procura de
estabilidade monetária. É Darwin e preservação da espécie. Ah pois sei
que toda a lei tem excepção.
E a doença (?) que vulgarmente é conhecida por ficar careca se chama
alopécia. Quem a tem deve ter cuidados como se tratasse da cara e usar
até creme por causa do sol. Não brinco. É mau para qualquer mulher
perder sua fonte de rendimentos.
Mas há carecas nesta lista que são uma simpatia...
Um abraço
Maria Natália
PS Falei porque alguém falou em mulher...

--- Em ciencialist@yahoogrupos.com.br, "Alvaro Augusto \(E\)"
<alvaro@e...> escreveu
> Caro L.E.,
>
> Não se esqueça que a frase tem um sentido oculto. Quem a formulou
não se referia necessariamente a carecas na cabeça (de cima)...
>
> [ ]s
>
> Alvaro Augusto
>
> ----- Original Message -----
> From: L.E.R.de Carvalho
> To: ciencialist@yahoogrupos.com.br
> Sent: Saturday, February 12, 2005 5:26 PM
> Subject: [ciencialist] Uma mente brilhante - os carecas
>
>
> At 11:02 12/2/2005, you wrote:
> >Muito boa esta descritiva!
> >Para os pragmáticos vão valer muito os atos conscientes, e para
os menos
> >conscientes valerá mais a lei da diversidade.
> >Há mulher que adora careca, há gente que procura os fedidos, os muito
> >magros, os muito gordos, os broncos... ou pela semelhança ou pela
> nsagem que não continham texto foram removidas]





SUBJECT: Nossa vida e o mar
FROM: José Renato <jrma@terra.com.br>
TO: <ciencialist@yahoogrupos.com.br>
DATE: 13/02/2005 12:48

A VIDA VEIO DO MAR

Quando chega o verão, nós, humanos, nos sentimos atraídos pelo mar.
Multidões se reúnem nas praias buscando um contato com as ondas do mar, que nos proporcionam prazer e descanso. Porém, o caminhar do ser humano deixa sua trilha fatal nas areias da praia.

Milhões de sacolas de nylon e plásticos de todo o tipo são largados na costa e o vento e as marés se encarregam de arrastá-los para o mar.

Uma bolsa de nylon pode navegar várias dezenas de anos sem se degradar.

As tartarugas marinhas confundem-nas com as medusas e as comem, sufocando-se na tentativa de engoli-las. Milhares de golfinhos também se confundem e morrem afogados.

Eles não têm capacidade para reconhecer os lixos dos humanos, simplesmente, se confundem, até porque, "tudo o que flutua no mar se come".

A tampa plástica de uma garrafa, de maior consistência do que a sacola plástica, pode permanecer inalterada, navegando nas águas do mar por mais de um século.

O Dr. James Ludwing, que estava estudando a vida do albatroz na ilha de Midway, no Pacífico, a muitas milhas dos centros povoados, fez uma descoberta espantosa. Quando começou a recolher o conteúdo do estômago de oito filhotes de albatrozes mortos, encontrou: 42 tampinhas plásticas de garrafa, 18 acendedores e restos flutuantes que, em sua maioria, eram pequenos pedaços de plástico.

Esses filhotes haviam sido alimentados por seus pais que não conseguiram fazer a distinção dos desperdícios no momento de escolher o alimento.

A próxima vez em que Você for à sua praia preferida, talvez encontre na areia lixo que outra pessoa ali deixou.

Não foi lixo deixado por Você, porém, é SUA PRAIA, é o SEU MAR, é o SEU MUNDO e Você deve fazer algo por eles.

Muitos pais jogam com seus filhos o jogo de: "vamos ver quem consegue juntar a maior quantidade de plásticos?" como forma de uma inesquecível lição de ecologia.

Outros, em silêncio, recolhem um plástico abandonado e levam- no para suas casas, com restos do mar. Você os verá passarem sorridentes, sabendo que salvaram um golfinho.

"Não se pode defender o que não se ama e não se pode amar o que não se conhece".

Ajude-nos a divulgar essa mensagem.




[As partes desta mensagem que não continham texto foram removidas]



SUBJECT: Ainda há tempo
FROM: José Renato <jrma@terra.com.br>
TO: <ciencialist@yahoogrupos.com.br>
DATE: 13/02/2005 13:00

O alerta dado pelos ambientalistas de 340 ONGs da Rede Ação Clima - RAC, em Paris, durante a abertura de uma série de atos por conta da entrada em vigor do Protocolo de Kioto no dia 16 de fevereiro, fez-me lembrar do artigo que escrevi durante a reunião anual da SBPC, em Salvador, há dois anos, que colo abaixo para lembrança e divulgação.
Abraços
José Renato
....................................................

A PONTE

José Renato M. de Almeida*


Em uma região longínqua, havia uma ponte ligando as duas margens de um rio.

Com o passar dos anos algumas pessoas começaram a percebê-la e, imediatamente, a questionar sobre ela. Inicialmente, essas questões vinham sob um sorriso compreensivo, tolerante ou uma boa risada de deboche. Ora - diziam os mais apressados - ela está aí porque está! Porque sempre esteve!

E o que antes nem era percebido passou a ser a curiosidade e o assunto de muitas conversas, elucubrações e motivo de muitos 'causos'. Quem construiu essa bela ponte? De que ela é feita? Para que foi construída se só agora há habitantes nas margens do rio?



Os mais antigos do lugar diziam que tinham ouvido seus avós contarem que, há muito tempo, chegou um exército numeroso de homens com cabeças resplandecentes, alguns empunhando espadas com o brilho de sol, escalando torres e escadas que alcançavam os céus. Foram eles que escavaram e cravaram grandes tubulões na terra. Durante dia e noite, construíram a ponte em meio a estranhos rugidos e estrondos de trovões.



Outros diziam que ela fora trazida dos céus já pronta e instalada ali mesmo, por seres alados.

Eram várias as lendas e histórias para explicar seu aparecimento, sua construção ou instalação. Lembramos mais algumas delas. Uns diziam que foram as intempéries climáticas, agindo nas rochas, que haviam moldado a ponte naquele formato, por mero acaso. Outros aventavam que uma enorme besta de carga havia sacudido a cangalha do lombo, que, atirada longe, foi se fincar justamente daquele jeito sobre o rio. E muitas outras explicações.



A APM - Assembléia dos Pescadores a Molinete, proclamava aos seus seguidores que a ponte havia sido construída por um antigo senhor muito bondoso, para que eles pudessem pescar com todo o conforto de cima da ponte, apreciando a brisa suave e o sol cálido.

Os da CPT - Congregação dos Pescadores a Tarrafa, afirmavam que ela fora construída para que eles pudessem pescar desfrutando a sombra e a água fresca sob a ponte.

Muitos não entravam nessas questões metafísicas ou utilitárias de menor relevância, mas arranjavam um jeito de cobrar pedágio e licenças daqueles que a atravessavam ou a usavam para pescar.



Todas essas controvérsias, porém, não bastava aos mais curiosos. Um grupo deles reuniu-se para procurar responder as principais questões ainda pendentes. Depois de pesquisar externamente as superfícies e as formas da ponte, decidiram investigar sua estrutura interna. Houve algum receio e até protestos dos mais precavidos. Alguns acreditavam até que, se a ponte fosse perscrutada - a fim de tirar amostras para análise dos materiais internos de que era constituída - ela poderia desabar, caso atingisse pontos críticos de sustentação. Convenhamos que isso seria uma grande catástrofe para os habitantes da região, já acostumados a andar pra lá e pra cá sobre ela. Mas eles não se intimidaram. Após muitas discussões decidiram remover, com equipamentos especiais projetados com esse objetivo, amostra do material de seu interior. Tiveram sucesso no projeto e divulgaram, com grande euforia, o resultado das análises a todos os habitantes.



Foi uma época de sucesso e vitória do conhecimento. Enfim, as respostas estavam dadas! Os componentes usados na edificação da ponte, aqueles que compunham sua estrutura não eram mais nenhum mistério.



Depois disso, fizeram vários corpos de prova na tentativa de obter compostos que proporcionassem resultados práticos semelhantes em resistência, flexibilidade e harmonia de formas. Muitos fracassos foram ocultados e as experiências de sucesso passaram a ser registradas e vendidas, proporcionando grandes somas aos seus 'descobridores'.



Mesmo assim, ainda havia alguns pesquisadores insatisfeitos com todas aquelas maravilhosas descobertas. Esses, silenciosamente, passavam dias e dias pensando, refletindo e conjeturando na tentativa de redesenhar o projeto original da ponte. Investiam em intensas pesquisas com ultra-som, raios-X, ressonância magnética, cálculos complicadíssimos e muita, muita imaginação.



A grande maioria dos habitantes achava perda de tempo essas tentativas de resgatar o projeto original ou pelo menos tentar elaborar um esboço dele. Afinal, diziam, a ponte já estava ali, totalmente pronta e utilizável. Para que então se preocupar com isso? Esses malucos! Será que não têm nada que fazer de mais útil!



O que de fato preocupa a nós outros é que poucos estão empenhados em preservá-la. Poucos se dão conta de sua importância para a própria vida e à vida dos demais habitantes. Acreditamos que, se nada de concreto for acordado e algo de efetivo realizado, breve a ponte estará deteriorada, suja, contaminada e intransitável. A hora de agir já tarda.







Nota: Este artigo foi inspirado nas idéias do filósofo Spinoza (1569-1630) de que Deus é igual à natureza - Deus sive natura. Afirma também, que a felicidade máxima do homem é o conhecimento de Deus e este é precisamente o fim ao qual a filosofia se propõe. Considera que Deus é a única substância existente. Como substância, Deus é a cadeia ou processo causal, a subjacente condição de todas as coisas, a lei e a estrutura do mundo. Esse concreto universo de modos e coisas está para Deus assim como uma ponte está para o seu projeto, sua estrutura, e as leis da matemática e da mecânica segundo as quais ela é construída; são estas as bases de sustentação, a condição subjacente, a substância da ponte; sem elas, ela cairia. (Tratado político-teológico)

As idéias não têm dono, mas têm autoria.



* José Renato M. de Almeida é engenheiro, aprendiz de filosofia e teologia.



SSA, 1° de maio de 2003

Rev. 14.06.2004



[As partes desta mensagem que não continham texto foram removidas]



SUBJECT: Re: [ciencialist] Re: Uma mente brilhante - os carecas
FROM: "Alvaro Augusto \(E\)" <alvaro@electraenergy.com.br>
TO: <ciencialist@yahoogrupos.com.br>
DATE: 13/02/2005 13:19

Bem, meu barbeiro me garante que não vou ficar careca, pois meu cabelo "nasce reto". Coisa de índio. Já da barriga ninguém está livre, mas ainda tenho esperanças de me livrar de uma leve adiposidade abdominal que começou depois dos 30...

Isso quer dizer que vou morrer pobre?

[ ]s

Alvaro Augusto

----- Original Message -----
From: Maria Natália
To: ciencialist@yahoogrupos.com.br
Sent: Sunday, February 13, 2005 2:17 AM
Subject: [ciencialist] Re: Uma mente brilhante - os carecas



Alvaro:
Cabeça de cima???? Ai...que já me deu LOLLLLLLL
Mas só me resta acrescentar: É dos carecas que elas gostam. E sabem
porquê?
Pois é simples. Existe uma regra quase universal que diz que um dos
sinais de abastança para além de se criar barriga é o facto de se não
ter cabelo, seja, na cabeça de cima. Logo mulher vai à procura de
estabilidade monetária. É Darwin e preservação da espécie. Ah pois sei
que toda a lei tem excepção.
E a doença (?) que vulgarmente é conhecida por ficar careca se chama
alopécia. Quem a tem deve ter cuidados como se tratasse da cara e usar
até creme por causa do sol. Não brinco. É mau para qualquer mulher
perder sua fonte de rendimentos.
Mas há carecas nesta lista que são uma simpatia...
Um abraço
Maria Natália
PS Falei porque alguém falou em mulher...

--- Em ciencialist@yahoogrupos.com.br, "Alvaro Augusto \(E\)"
<alvaro@e...> escreveu
> Caro L.E.,
>
> Não se esqueça que a frase tem um sentido oculto. Quem a formulou
não se referia necessariamente a carecas na cabeça (de cima)...
>
> [ ]s
>
> Alvaro Augusto
>
> ----- Original Message -----
> From: L.E.R.de Carvalho
> To: ciencialist@yahoogrupos.com.br
> Sent: Saturday, February 12, 2005 5:26 PM
> Subject: [ciencialist] Uma mente brilhante - os carecas
>
>
> At 11:02 12/2/2005, you wrote:
> >Muito boa esta descritiva!
> >Para os pragmáticos vão valer muito os atos conscientes, e para
os menos
> >conscientes valerá mais a lei da diversidade.
> >Há mulher que adora careca, há gente que procura os fedidos, os muito
> >magros, os muito gordos, os broncos... ou pela semelhança ou pela
> nsagem que não continham texto foram removidas]




[As partes desta mensagem que não continham texto foram removidas]



SUBJECT: Re: [ciencialist] Ainda há tempo
FROM: "Alvaro Augusto \(E\)" <alvaro@electraenergy.com.br>
TO: <ciencialist@yahoogrupos.com.br>
DATE: 13/02/2005 13:23

Isso é exagero. A Terra já passou por coisa muita mais perturbadora que os gases emitidos pela raça humana. Nem mesmo meteroros gigantes conseguiram acabar com a vida por aqui. Assim, podemos dizer, com razoável grau de certeza, que a vida ainda estará presente na Terra no próximo bilhão de anos.

Mas nós não estaremos...

[ ]s

Alvaro Augusto

----- Original Message -----
From: José Renato
To: ciencialist@yahoogrupos.com.br
Sent: Friday, February 11, 2005 7:25 PM
Subject: [ciencialist] Ainda há tempo


Notícias Terra
Quinta, 10 de fevereiro de 2005, 14h45

ONG diz que restam de 10 a 20 anos para salvar a Terra

A comunidade internacional tem de 10 a 20 anos para tentar evitar uma mudança climática devastadora na Terra, advertiu hoje a Rede Ação Clima (RAC), integrada por 340 ONGs de proteção ao meio ambiente. A RAC fez o alerta em Paris durante a abertura de uma série de atos por conta da entrada em vigor do Protocolo de Kioto no dia 16 de fevereiro.
O acordo internacional, não assinado pelos Estados Unidos, prevê a redução de 5,2% das emissões de gases que provocam o efeito estufa pelos países industrializados até 2012. "Para impedir o aumento de dois graus da temperatura média mundial até 2100 e estabelecer uma margem de segurança, é preciso estabilizar a concentração de CO2 em 400 ppm (partes por milhão) frente às 379 ppm atuais. Para alcançar esse objetivo, temos apenas de 10 a 20 anos", afirmou Laetitia Marez, porta-voz da RAC.

Segundo a RAC, as emissões mundiais de gases que provocam o efeito estufa devem alcançar seu patamar mais elevado em 2020, no máximo e, a partir daí, cair pela metade até 2050 para evitar efeitos devastadores para o planeta.

Segundo os especialistas, reunidos na semana passada em Exeter (Reino Unido), um aumento de um grau nas temperaturas terá um impacto destruidor nos corais e dois graus provocarão o derretimento da Groenlândia, o que faria o nível do mar subir consideravelmente.

A curto prazo, são os países industrializados que devem fazer os maiores esforços para adotar novas medidas restritivas a partir de 2012. A RAC considera necessária a redução de 30% das emissões de CO2 até 2020 e de 75% até 2050.

Para a ONG, países emergentes, como Arábia Saudita e Coréia do Sul, deveriam reduzir suas emissões de gases a partir de 2013. Grandes países como China e Índia deveriam, com a ajuda financeira e tecnológica dos países do Norte, devem se comprometer desde já com a produção industrial e energética limpa, segundo a RAC.

EFE

Agência Efe - Todos os direitos reservados. É proibido todo tipo de reprodução sem autorização escrita da Agência Efe S/A.

< http://noticias.terra.com.br/ciencia/interna/0,,OI469817-EI299,00.html >


[As partes desta mensagem que não continham texto foram removidas]



[As partes desta mensagem que não continham texto foram removidas]



SUBJECT: RE: [ciencialist] Re: Uma mente brilhante - os carecas
FROM: "murilo filo" <avalanchedrive@hotmail.com>
TO: ciencialist@yahoogrupos.com.br
DATE: 13/02/2005 14:51

Vcs não sabem o que padece um careca ''conhecido'' meu...
todas cabeças saudáveis, bem saudáveis e... sem o tal l'argent!
Caso triste, o dêste cara... já que desoptou da solução gay!
( cabelo está até sobrando, o problema é o da sua distribuição )
abr/M.

>From: Maria Natália <grasdic@hotmail.com>
>Reply-To: ciencialist@yahoogrupos.com.br
>To: ciencialist@yahoogrupos.com.br
>Subject: [ciencialist] Re: Uma mente brilhante - os carecas
>Date: Sun, 13 Feb 2005 05:17:34 -0000
>
>
>
>Alvaro:
>Cabeça de cima???? Ai...que já me deu LOLLLLLLL
>Mas só me resta acrescentar: É dos carecas que elas gostam. E sabem
>porquê?
>Pois é simples. Existe uma regra quase universal que diz que um dos
>sinais de abastança para além de se criar barriga é o facto de se não
>ter cabelo, seja, na cabeça de cima. Logo mulher vai à procura de
>estabilidade monetária. É Darwin e preservação da espécie. Ah pois sei
>que toda a lei tem excepção.
>E a doença (?) que vulgarmente é conhecida por ficar careca se chama
>alopécia. Quem a tem deve ter cuidados como se tratasse da cara e usar
>até creme por causa do sol. Não brinco. É mau para qualquer mulher
>perder sua fonte de rendimentos.
>Mas há carecas nesta lista que são uma simpatia...
>Um abraço
>Maria Natália
>PS Falei porque alguém falou em mulher...
>
>--- Em ciencialist@yahoogrupos.com.br, "Alvaro Augusto \(E\)"
><alvaro@e...> escreveu
> > Caro L.E.,
> >
> > Não se esqueça que a frase tem um sentido oculto. Quem a formulou
>não se referia necessariamente a carecas na cabeça (de cima)...
> >
> > [ ]s
> >
> > Alvaro Augusto
> >
> > ----- Original Message -----
> > From: L.E.R.de Carvalho
> > To: ciencialist@yahoogrupos.com.br
> > Sent: Saturday, February 12, 2005 5:26 PM
> > Subject: [ciencialist] Uma mente brilhante - os carecas
> >
> >
> > At 11:02 12/2/2005, you wrote:
> > >Muito boa esta descritiva!
> > >Para os pragmáticos vão valer muito os atos conscientes, e para
>os menos
> > >conscientes valerá mais a lei da diversidade.
> > >Há mulher que adora careca, há gente que procura os fedidos, os muito
> > >magros, os muito gordos, os broncos... ou pela semelhança ou pela
> > nsagem que não continham texto foram removidas]
>
>
>
>
>
>##### ##### #####
>
>Para saber mais visite
>http://www.ciencialist.hpg.ig.com.br
>
>
>##### ##### ##### #####
>Links do Yahoo! Grupos
>
>
>
>
>
>
>
>




SUBJECT: Re: que tal, convence?
FROM: "rayfisica" <rayfisica@yahoo.com.br>
TO: ciencialist@yahoogrupos.com.br
DATE: 13/02/2005 15:27


--- Em ciencialist@yahoogrupos.com.br, Hélio Ricardo Carvalho
<hrc@f...> escreveu
>
> Eu escrevi:
> >Na próxima vez falarei da força, do trabalho e da energia
> necessária para levar a membrana por este caminho.
>
> CONTINUANDO

> Mas o que acontece é:
>
> A situação volta a ser exatamente como era um pouco antes de se
> tocar.
>
> Isto tem relação com o fato de que um GVDG não funciona se o
seu
> centro estiver conectado com uma bateria colocando uma ddp fixa em
> relação ao terra.
>
> E se eu apoiar a membrana deixando ela ir com velocidade constante
> até ao máximo (r2=r3-d2) toda energia que eu tinha dado para
o
> sistema será devolvida para minha mão.
>
>
> [ ]'s
> Hélio
>>>>>>>>>>>>>>>>>>>>>>>>>>>>>>>>>>>>

Eu construí um mine gerador de van der graaf que aprendi no site
do
prof. Leo nessa pagina:

http://www.feiradeciencias.com.br/sala11/11_49.asp

Daí eu utilizei a cúpula que nada mais é que uma lata de
refrigerante
que mantendo isolada em um ambiente seco o suficiente, ligada a uma
bateria não houve o carregamento da cúpula.
Apenas que conforme verificado por um multímetro a latinha
(cúpula)
apenas adquiriu o potencial do pólo ao qual estava ligado,
exatamente
igual a se eu houvesse ligado o pólo referido ao exterior da lata.
Usando o gerador (gvdg) para carregar a latinha não tem
diferença se
contatar dentro ou fora da latinha.
Ai eu pensei: vou utilizar duas latinhas uma no pólo positivo e
outra
no pólo negativo, (contatando por dentro lógico) se funcionar
vou
carregar uma cúpula com carga positiva e a outra com carga
negativa.
Não funcionou, as latinhas comportam se como se fossem os pólos
da
bateria, independentemente se contata por dentro ou fora das latinhas.
Portanto eu conclui (apesar de isso só ter valor para mim) devido
ao
método, que não é possível carregar um gerador de van der
graaf
usando uma bateria






SUBJECT: Re: que tal, convence?
FROM: "rayfisica" <rayfisica@yahoo.com.br>
TO: ciencialist@yahoogrupos.com.br
DATE: 13/02/2005 15:38


Desculpem-me apenas quero deixar claro que o mini gerador que o prof.
Leo ensina no link realmente funciona quando o ambiente é seco o
suficiente, exatamente como o da faculdade, apenas que eu não sei o
potencial que ele atinge devido ao fato que meu multímetro só vai até
1000V.





SUBJECT: Dois relógios e a relatividade?
FROM: "rayfisica" <rayfisica@yahoo.com.br>
TO: ciencialist@yahoogrupos.com.br
DATE: 13/02/2005 16:23


Alguém pode me dizer algo sobre uma experiência realizada com
dois
relógios muito precisos onde um foi colocado em um avião e o
outro
ficou em terra, e segundo dizem comprovou a relatividade.
Não estou questionando a relatividade.
Porem ainda que os relógios funcionassem a diversa velocidade,
qualquer outro equipamento no avião estaria sofrendo o efeito da
relatividade o mesmo acontecendo na terra, e qualquer tentativa, de
igualar os sistemas, provocaria um ajuste.(por exemplo aterrizar o
avião ou acelerar o relógio da terra)
Pergunto lhes, como foi realizado essa experiência, uma câmara
fotográfica japonesa ultrahipermegabeta,que estando na terra
fotografou os dois relógios ao mesmo tempo (visse verso) ou essa
é
uma daquelas experiência onde vale o que disse quem realizou.






SUBJECT: Fw: ci�ncias
FROM: "Luiz Ferraz Netto" <leobarretos@uol.com.br>
TO: "ciencialist" <ciencialist@yahoogrupos.com.br>
DATE: 13/02/2005 16:27

Sugestões?
[]'
===========================
Luiz Ferraz Netto [Léo]
leobarretos@uol.com.br
http://www.feiradeciencias.com.br
===========================
-----Mensagem Original-----
De: Lula Klautau
Para: leobarretos@uol.com.br
Enviada em: sábado, 12 de fevereiro de 2005 10:04
Assunto: ciências


Quero saber algumas ciências que estuda o passado



--------------------------------------------------------------------------------


No virus found in this incoming message.
Checked by AVG Anti-Virus.
Version: 7.0.300 / Virus Database: 265.8.7 - Release Date: 10/02/2005

----------

No virus found in this outgoing message.
Checked by AVG Anti-Virus.
Version: 7.0.300 / Virus Database: 265.8.7 - Release Date: 10/02/2005


[As partes desta mensagem que não continham texto foram removidas]



SUBJECT: Fw: como calculo a velocidade de um motor que est� girando em um banco dinamom�trico. Trabalho na Fiat MG
FROM: "Luiz Ferraz Netto" <leobarretos@uol.com.br>
TO: "ciencialist" <ciencialist@yahoogrupos.com.br>
DATE: 13/02/2005 16:41

Sugestões ... principalmente dos adoradores de carros FIAT!
[]'
PS: só uso voquisvagen.
===========================
Luiz Ferraz Netto [Léo]
leobarretos@uol.com.br
http://www.feiradeciencias.com.br
===========================
-----Mensagem Original-----
De: PIERRE Marcelo da Silva
Para: leobarretos@uol.com.br
Enviada em: sábado, 12 de fevereiro de 2005 09:20
Assunto: como calculo a velocidade de um motor que está girando em um banco dinamométrico. Trabalho na Fiat MG


Nas dúvidas experimentais, por gentileza coloque aqui o endereço da página, isso facilita o confronto. Agradeço. Meu nome é LUIZ FERRAZ NETTO, meu apelido é LÉO e moro em BARRETOS; dai vem meu e-mail: leobarretos@uol.com.br.

Olá, Dr. Léo Barreto, trabalho na Fiat Automóveis, em uma sala de testes dotada de banco dinamométrico no final da linha de produção e estou entrando com um tipo de teste que faz-se fundamental eu conhecer o que corresponde em volocidade a realização da rodagem deste motor há um total de 200 horas de provas.

como faço este cálculo. O Sr. pode além de citar as f´órmulas, pode me citar um exemplo de cálculo?

Favor responder no meu e-mail pessoal. mspierrot@uol.com.br

Dados: Os motores são testados em dinamometro corrente foucoult shenck.

Antecipadamente, já lhe agradeço.



Pierre

Cabine Prova Especial
FIAT-GM POWERTRAIN
Fone: (031) 2123-5152 / 5284
Av. do Contorno da Fiat, 3455
Distr. Ind. Paulo Camilo Pena - 32530-490 - Betim - MG - Brasil
mspierrot@uol.com.br
P Antes de imprimir pense em sua responsabilidade e compromisso com o MEIO AMBIENTE.




--------------------------------------------------------------------------------


No virus found in this incoming message.
Checked by AVG Anti-Virus.
Version: 7.0.300 / Virus Database: 265.8.7 - Release Date: 10/02/2005

----------

No virus found in this outgoing message.
Checked by AVG Anti-Virus.
Version: 7.0.300 / Virus Database: 265.8.7 - Release Date: 10/02/2005


[As partes desta mensagem que não continham texto foram removidas]



SUBJECT: Re: [ciencialist] Re: que tal, convence?
FROM: "Luiz Ferraz Netto" <leobarretos@uol.com.br>
TO: <ciencialist@yahoogrupos.com.br>
DATE: 13/02/2005 17:08

Rayfisica escreve:
> Eu construí um mine gerador de van der graaf que aprendi no site do prof. Leo nessa pagina:

http://www.feiradeciencias.com.br/sala11/11_49.asp

Daí eu utilizei a cúpula que nada mais é que uma lata de refrigerante que mantendo isolada em um ambiente seco o suficiente, ligada a uma bateria não houve o carregamento da cúpula.
Apenas que conforme verificado por um multímetro a latinha(cúpula) apenas adquiriu o potencial do pólo ao qual estava ligado, exatamente igual a se eu houvesse ligado o pólo referido ao exterior da lata.
Usando o gerador (gvdg) para carregar a latinha não tem diferença se contatar dentro ou fora da latinha.
Ai eu pensei: vou utilizar duas latinhas uma no pólo positivo e outra no pólo negativo, (contatando por dentro lógico) se funcionar vou carregar uma cúpula com carga positiva e a outra com carga negativa.
Não funcionou, as latinhas comportam se como se fossem os pólos da bateria, independentemente se contata por dentro ou fora das latinhas.
Portanto eu conclui (apesar de isso só ter valor para mim) devido ao método, que não é possível carregar um gerador de van der graaf usando uma bateria.<

Léo
Não peguei o início dessa thread e, assim, estou pegando o bonde bem na curva ... não sei o que se passou na reta!

Não há modo de se 'carregar' o VDG por processo estático, ligando-o internamente ou externamente a pilhas ou baterias; quando muito se leva o potencial elétrico da cúpula ao potencial elétrico da pilha ou bateria.
O único modo é através do trabalho externo de se levar cargas discretas até o interior da cúpula e tocar na cúpula. É um tantinho de carga por vez.

Vc deve ter percebido a discrepância entre as quantidades de carga com as quais se trabalha em eletrostática (uC) e em eletrodinâmica (C).
Cargas em equilíbrio eletrostático em um condutor e cargas em movimento num condutor são fenômenos de mesma espécie, porém, de enorme discrepância quantitativa.
Quer coisa mais corriqueira que um fio de cobre conduzindo miserável corrente de 1A? Qqer fontezinha permite bem mais que isso. Uma bateria de carro, ao girar a chave de partida, pode enviar ao motor de arranque correntes da ordem dos 300A.
Agora, calcule quanto de carga passa pela seção do fio de cobre (aquele que conduz 1A) em 1 minuto. A seguir, imagine o que aconteceria se a cúpula de um VDG (raio de 1m) tivesse essa quantidade de carga.
Corrente contínua não pode escoar para dentro de um VDG, a não ser em pacotinhos isoados do meio ambiente, e com uma minúscula carga em cada um deles! O Peletron é um bom exemplo.

[]'

Léo







##### ##### #####

Para saber mais visite
http://www.ciencialist.hpg.ig.com.br


##### ##### ##### #####
Links do Yahoo! Grupos










--
No virus found in this incoming message.
Checked by AVG Anti-Virus.
Version: 7.0.300 / Virus Database: 265.8.7 - Release Date: 10/02/2005




--
No virus found in this outgoing message.
Checked by AVG Anti-Virus.
Version: 7.0.300 / Virus Database: 265.8.7 - Release Date: 10/02/2005



SUBJECT: Re: [ciencialist] Ainda há tempo
FROM: José Renato <jrma@terra.com.br>
TO: <ciencialist@yahoogrupos.com.br>
DATE: 13/02/2005 17:17

Álvaro, vc tem razão. A Terra permanecerá...
A humanidade é que muito provavelmente não sobreviverá. Lembre-se que nos
períodos de extinção, provocados por motivos diversos, a vida quase se
extinguiu.
[]s
JR
..............................................

From: "Alvaro Augusto (E)" <alvaro@To: <ciencialist@yahoogrupos.com.br>
Sent: Sunday, February 13, 2005 12:23 PM
Subject: Re: [ciencialist] Ainda há tempo



Isso é exagero. A Terra já passou por coisa muita mais perturbadora que os
gases emitidos pela raça humana. Nem mesmo meteroros gigantes conseguiram
acabar com a vida por aqui. Assim, podemos dizer, com razoável grau de
certeza, que a vida ainda estará presente na Terra no próximo bilhão de
anos.

Mas nós não estaremos...

[ ]s

Alvaro Augusto

----- Original Message -----
From: José Renato
To: ciencialist@yahoogrupos.com.br
Sent: Friday, February 11, 2005 7:25 PM
Subject: [ciencialist] Ainda há tempo


Notícias Terra
Quinta, 10 de fevereiro de 2005, 14h45

ONG diz que restam de 10 a 20 anos para salvar a Terra

A comunidade internacional tem de 10 a 20 anos para tentar evitar uma
mudança climática devastadora na Terra, advertiu hoje a Rede Ação Clima
(RAC), integrada por 340 ONGs de proteção ao meio ambiente. A RAC fez o
alerta em Paris durante a abertura de uma série de atos por conta da entrada
em vigor do Protocolo de Kioto no dia 16 de fevereiro.
O acordo internacional, não assinado pelos Estados Unidos, prevê a redução
de 5,2% das emissões de gases que provocam o efeito estufa pelos países
industrializados até 2012. "Para impedir o aumento de dois graus da
temperatura média mundial até 2100 e estabelecer uma margem de segurança, é
preciso estabilizar a concentração de CO2 em 400 ppm (partes por milhão)
frente às 379 ppm atuais. Para alcançar esse objetivo, temos apenas de 10 a
20 anos", afirmou Laetitia Marez, porta-voz da RAC.

Segundo a RAC, as emissões mundiais de gases que provocam o efeito estufa
devem alcançar seu patamar mais elevado em 2020, no máximo e, a partir daí,
cair pela metade até 2050 para evitar efeitos devastadores para o planeta.

Segundo os especialistas, reunidos na semana passada em Exeter (Reino
Unido), um aumento de um grau nas temperaturas terá um impacto destruidor
nos corais e dois graus provocarão o derretimento da Groenlândia, o que
faria o nível do mar subir consideravelmente.

A curto prazo, são os países industrializados que devem fazer os maiores
esforços para adotar novas medidas restritivas a partir de 2012. A RAC
considera necessária a redução de 30% das emissões de CO2 até 2020 e de 75%
até 2050.

Para a ONG, países emergentes, como Arábia Saudita e Coréia do Sul,
deveriam reduzir suas emissões de gases a partir de 2013. Grandes países
como China e Índia deveriam, com a ajuda financeira e tecnológica dos países
do Norte, devem se comprometer desde já com a produção industrial e
energética limpa, segundo a RAC.

EFE

Agência Efe - Todos os direitos reservados. É proibido todo tipo de
reprodução sem autorização escrita da Agência Efe S/A.

< http://noticias.terra.com.br/ciencia/interna/0,,OI469817-EI299,00.html >


[As partes desta mensagem que não continham texto foram removidas]



[As partes desta mensagem que não continham texto foram removidas]



##### ##### #####

Para saber mais visite
http://www.ciencialist.hpg.ig.com.br


##### ##### ##### #####
Links do Yahoo! Grupos









E-mail classificado pelo Identificador de Spam Inteligente Terra.
Para alterar a categoria classificada, visite
http://www.terra.com.br/centralunificada/emailprotegido/imail/imail.cgi?+_u=jrma&_l=1,1108308218.361478.11496.chipata.terra.com.br,6339,Des15,Des15

Esta mensagem foi verificada pelo E-mail Protegido Terra.
Scan engine: McAfee VirusScan / Atualizado em 09/02/2005 / Versão: 4.4.00 -
Dat 4427
Proteja o seu e-mail Terra: http://www.emailprotegido.terra.com.br/




SUBJECT: Re: Uma mente brilhante - os carecas
FROM: Maria Natália <grasdic@hotmail.com>
TO: ciencialist@yahoogrupos.com.br
DATE: 13/02/2005 18:04


Alvaro:
Mas rico em conhecimento e deixando umas tantas de nós tristes(bem eu
vou morrer primeiro). Basta andar aqui com o Leo, o Brudna , o Takata
e demais engenhocas para se estar em constante enriquecimento
intelectual. Muitas de nós põem o dinheiro em 4ª opção...Gostamos de
Homens e não de homenzinhos berdes.
Espero que não te babes e percas a tal barriga fazendo a malhação da
praxe mas sob vigilância médica! Devagar se volta à linha. E gostaria
ainda antes de morrer de dançar contigo um corridinho e um samba.
A tal barriga de que te falei é criada devido ao sedentarismo pois
muitos de nós faz do aparelho digestivo tubo de ensaio e nem sequer
pensa em fazer sair essas calorias a mais. As comidas pobres em fibra
fazem os ventres opados. As misturas...
E agora vem o L.E.R que sabe disto a potes ou o Emiliano sobre a
química do quilo.
Abraço
Maria Natália

--- Em ciencialist@yahoogrupos.com.br, "Alvaro Augusto \(E\)"
<alvaro@e...> escreveu
> Bem, meu barbeiro me garante que não vou ficar careca, pois meu
cabelo "nasce reto". Coisa de índio. Já da barriga ninguém está livre,
mas ainda tenho esperanças de me livrar de uma leve adiposidade
abdominal que começou depois dos 30...
>
> Isso quer dizer que vou morrer pobre?
>
> [ ]s
>
> Alvaro Augusto






SUBJECT: Re: Uma mente brilhante - os carecas
FROM: Maria Natália <grasdic@hotmail.com>
TO: ciencialist@yahoogrupos.com.br
DATE: 13/02/2005 18:09


"murilo filo":

Tal como o dinheiro está mal distribuído e não dá para a investigação
ou bolsas de estudo de bons alunos em universidades
brasileiras/portuguesas. E o pior é que não há teoria económica que
consiga resolver o problema. Há sempre humanos se enchendo para outros
estarem á míngua.
Um abraço
Maria Natália, falando de Machholz ali na direcçãp de nossa galáxia




<avalanchedrive@h...> escreveu
> Vcs não sabem o que padece um careca ''conhecido'' meu...
> todas cabeças saudáveis, bem saudáveis e... sem o tal l'argent!
> Caso triste, o dêste cara... já que desoptou da solução gay!
> ( cabelo está até sobrando, o problema é o da sua distribuição )
> abr/M.
>
> >From: Maria Natália <grasdic@h...>
> >Reply-To: ciencialist@yahoogrupos.com.br
> >To: ciencialist@yahoogrupos.com.br
> >Subject: [ciencialist] Re: Uma mente brilhante - os carecas
>





SUBJECT: Re: Fw: ciências
FROM: Maria Natália <grasdic@hotmail.com>
TO: ciencialist@yahoogrupos.com.br
DATE: 13/02/2005 18:13


Leo:

Arqueologia, Astronomia (cosmologia), Medicina Legal (estuda o passado
de um corpo) e a vez agora a outros...

Aquele abraço, professor
Maria Natália


> ===========================
> Luiz Ferraz Netto [Léo]
> leobarretos@u...
> http://www.feiradeciencias.com.br
> ===========================
> -----Mensagem Original-----
> De: Lula Klautau
> Para: leobarretos@u...
> Enviada em: sábado, 12 de fevereiro de 2005 10:04
> Assunto: ciências
>
>
> Quero saber algumas ciências que estuda o passado
>
>
>
>
--------------------------------------------------------------------------------
>


>
> No virus found in this incoming message.
> Checked by AVG Anti-Virus.
> Version: 7.0.300 / Virus Database: 265.8.7 - Release Date: 10/02/2005
>
> ----------
>
> No virus found in this outgoing message.
> Checked by AVG Anti-Virus.
> Version: 7.0.300 / Virus Database: 265.8.7 - Release Date: 10/02/2005
>
>
> [As partes desta mensagem que não continham texto foram removidas]





SUBJECT: Re: Foucault / Luz ondas ou corpusculos.
FROM: Hélio Ricardo Carvalho <hrc@fis.puc-rio.br>
TO: ciencialist@yahoogrupos.com.br
DATE: 13/02/2005 18:50


Para Victor,

Primeiramente para tirar alguma má impressão quero te dizer que eu
gosto de falar de forma bem humorada também. Se eu pedi para você
falar numa linguagem mais técnica, foi para ter certeza de alguns
pontos da sua msg.

Eu escrevi:
> > >Se isto é verdade, refaço a pergunta original:
> > > Por que esta experiência descartou a luz corpuscular??????

Victor respondeu na msg 44272:

>...
>No que tange ao conhecido fenômeno da refração, o
>que se observa, experimentalmente, com um modelo corpuscular?
>Velocidade da luz no meio de maior índice, maior que a velocidade
>da luz no vácuo; explicação: as partículas de luz, ao colidirem
>com a superfície da água ficam submetidas a uma atração, uma força
>normal à
>superfície no ponto de contato, obrigando as dita cujas aproximarem-
>se da normal e esta seria a causa da refração, da mudança de direção
>ao sair de um meio e entrar em outro, diferente. Como resultado desta
>atração, as partículas sofreriam uma pequena aceleração na
>superfície,
>senda esta a causa de sua maior velocidade no meio mais denso. Isto
>é o
>que se obtém com um modelo corpuscular, para o fenômeno refração.
>...


Perfeito! É isto que eu queria saber: a linha de raciocínio que levou
aos cientistas da época (com os poucos conhecimentos que tinham) a
acharem que a dita experiência derrubava a luz corpuscular.
:-)
:-)

Mas, depois da física quântica (falo da FQ básica e não o esoterismo
de hoje) existe um modelo de como funciona a refração levando em
consideração as sucessivas absorções e reemissões dos corpúsculos de
luz.

Veja o final de minha msg 44263.


Só quando se leva em conta que a luz é absorvida e depois reemitida
pelos átomos do meio que se pode começar a entender a refração.
Seja ela corpúsculos ou onda.

É difícil para mim escrever isto, como algo pode SER onda. Para mim
onda não é um SER. :-)
Onda, para mim é apenas uma das formas da expressar um determinado
comportamento da matéria.



[ ]'s
Hélio

PS.: Eu também estou aqui para aprender e me aprimorar.






SUBJECT: Re: Dois relógios e a relatividade?
FROM: Hélio Ricardo Carvalho <hrc@fis.puc-rio.br>
TO: ciencialist@yahoogrupos.com.br
DATE: 13/02/2005 18:57


Rayfisica,

Faço minhas as suas palavras. Eu também gostaria de saber detalhes
desta experiência. Ela foi mesmo realizada ou é lenda.

Você sabia que o chamado "paradoxo dos gêmeos" está contido nela?


[ ]'s
Hélio



--- Em ciencialist@yahoogrupos.com.br, "rayfisica" <rayfisica@y...>
escreveu
>
> Alguém pode me dizer algo sobre uma experiência realizada com
> dois
> relógios muito precisos onde um foi colocado em um avião e o
> outro
> ficou em terra, e segundo dizem comprovou a relatividade.
> Não estou questionando a relatividade.
> Porem ainda que os relógios funcionassem a diversa velocidade,
> qualquer outro equipamento no avião estaria sofrendo o efeito da
> relatividade o mesmo acontecendo na terra, e qualquer tentativa, de
> igualar os sistemas, provocaria um ajuste.(por exemplo aterrizar o
> avião ou acelerar o relógio da terra)
> Pergunto lhes, como foi realizado essa experiência, uma câmara
> fotográfica japonesa ultrahipermegabeta,que estando na terra
> fotografou os dois relógios ao mesmo tempo (visse verso) ou essa
> é
> uma daquelas experiência onde vale o que disse quem realizou.






SUBJECT: Relatividade
FROM: "Hugo Santos" <urano@netvisao.pt>
TO: <ciencialist@yahoogrupos.com.br>
DATE: 13/02/2005 20:48

Caros colegas de lista,



Tenho uma questão a colocar. Julgo que a maioria das pessoas estará
familiarizada com as experiências teóricas que normalmente se usam para
exemplificar as diferenças na passagem do tempo relativamente a diferentes
observadores. Uma delas é aquela do relógio fotónico (julgo que lhe posso
chamar assim) em que se tem dois espelhos e um fotão a saltar entre eles.
Este relógio é metido num comboio que anda a muito altas velocidades e
depois chega-se ao movimento relativo do fotão para um observador externo,
em forma semelhante a “WWWW”. Até aqui, nada de novo. No entanto, a minha
pergunta prende-se para o seguinte caso: se se colocar um segundo relógio,
semelhante ao primeiro, com igual distância entre os espelhos, mas que
esteja perpendicular ao primeiro relógio, com o fotão viajando paralelamente
à direcção de deslocamento do comboio, como é que esse relógio bate o tempo,
relativamente ao primeiro, quando vistos em simultâneo por um observador
externo?



A meu ver, este segundo relógio nem sequer vai bater bem o tempo. Ou melhor,
o fotão demora mais tempo a percorrer a distância entre os espelhos quando
se desloca num sentido e menos tempo quando se desloca no sentido inverso.
Não tenho a certeza, mas a intuição diz-me que a cada duas batidas os
relógios estão certos. No entanto, isso não quer dizer que naquele segundo
relógio o tempo esteja constantemente a ser acelerado e desacelerado
dependendo do sentido de deslocamento do fotão...



Que me podem dizer sobre este caso? Este exemplo só funciona para relógios
perpendiculares ao movimento? Porque é que não funciona o outro?



Hugo Santos



[As partes desta mensagem que não continham texto foram removidas]



SUBJECT: RE: [ciencialist] Ainda há tempo
FROM: "murilo filo" <avalanchedrive@hotmail.com>
TO: ciencialist@yahoogrupos.com.br
CC: avalanchedrive@hotmail.com
DATE: 13/02/2005 20:51

Zé, oi.
Gostei muito da sua matéria.
Todos na lista deveriam lêr, *pelo menos*, a última frase.
Umas das sínteses mais verdadeiras que já lí e que tem muito que vêr como a
'minha' filosofia.
Deve ser muito duro não entender esta cosmologia.
Obr., abração. Murilo SP 13/fev

>From: José Renato <jrma@terra.com.br>
>Reply-To: ciencialist@yahoogrupos.com.br
>To: <ciencialist@yahoogrupos.com.br>
>Subject: [ciencialist] Ainda há tempo
>Date: Sun, 13 Feb 2005 12:00:23 -0300
>
>
>O alerta dado pelos ambientalistas de 340 ONGs da Rede Ação Clima - RAC, em
>Paris, durante a abertura de uma série de atos por conta da entrada em
>vigor do Protocolo de Kioto no dia 16 de fevereiro, fez-me lembrar do
>artigo que escrevi durante a reunião anual da SBPC, em Salvador, há dois
>anos, que colo abaixo para lembrança e divulgação.
>Abraços
>José Renato
>....................................................
>
>A PONTE
>
>José Renato M. de Almeida*
>
>
>Em uma região longínqua, havia uma ponte ligando as duas margens de um rio.
>
>Com o passar dos anos algumas pessoas começaram a percebê-la e,
>imediatamente, a questionar sobre ela. Inicialmente, essas questões vinham
>sob um sorriso compreensivo, tolerante ou uma boa risada de deboche. Ora -
>diziam os mais apressados - ela está aí porque está! Porque sempre esteve!
>
>E o que antes nem era percebido passou a ser a curiosidade e o assunto de
>muitas conversas, elucubrações e motivo de muitos 'causos'. Quem construiu
>essa bela ponte? De que ela é feita? Para que foi construída se só agora há
>habitantes nas margens do rio?
>
>
>
>Os mais antigos do lugar diziam que tinham ouvido seus avós contarem que,
>há muito tempo, chegou um exército numeroso de homens com cabeças
>resplandecentes, alguns empunhando espadas com o brilho de sol, escalando
>torres e escadas que alcançavam os céus. Foram eles que escavaram e
>cravaram grandes tubulões na terra. Durante dia e noite, construíram a
>ponte em meio a estranhos rugidos e estrondos de trovões.
>
>
>
>Outros diziam que ela fora trazida dos céus já pronta e instalada ali
>mesmo, por seres alados.
>
>Eram várias as lendas e histórias para explicar seu aparecimento, sua
>construção ou instalação. Lembramos mais algumas delas. Uns diziam que
>foram as intempéries climáticas, agindo nas rochas, que haviam moldado a
>ponte naquele formato, por mero acaso. Outros aventavam que uma enorme
>besta de carga havia sacudido a cangalha do lombo, que, atirada longe, foi
>se fincar justamente daquele jeito sobre o rio. E muitas outras
>explicações.
>
>
>
>A APM - Assembléia dos Pescadores a Molinete, proclamava aos seus
>seguidores que a ponte havia sido construída por um antigo senhor muito
>bondoso, para que eles pudessem pescar com todo o conforto de cima da
>ponte, apreciando a brisa suave e o sol cálido.
>
>Os da CPT - Congregação dos Pescadores a Tarrafa, afirmavam que ela fora
>construída para que eles pudessem pescar desfrutando a sombra e a água
>fresca sob a ponte.
>
>Muitos não entravam nessas questões metafísicas ou utilitárias de menor
>relevância, mas arranjavam um jeito de cobrar pedágio e licenças daqueles
>que a atravessavam ou a usavam para pescar.
>
>
>
>Todas essas controvérsias, porém, não bastava aos mais curiosos. Um grupo
>deles reuniu-se para procurar responder as principais questões ainda
>pendentes. Depois de pesquisar externamente as superfícies e as formas da
>ponte, decidiram investigar sua estrutura interna. Houve algum receio e até
>protestos dos mais precavidos. Alguns acreditavam até que, se a ponte fosse
>perscrutada - a fim de tirar amostras para análise dos materiais internos
>de que era constituída - ela poderia desabar, caso atingisse pontos
>críticos de sustentação. Convenhamos que isso seria uma grande catástrofe
>para os habitantes da região, já acostumados a andar pra lá e pra cá sobre
>ela. Mas eles não se intimidaram. Após muitas discussões decidiram remover,
>com equipamentos especiais projetados com esse objetivo, amostra do
>material de seu interior. Tiveram sucesso no projeto e divulgaram, com
>grande euforia, o resultado das análises a todos os habitantes.
>
>
>
>Foi uma época de sucesso e vitória do conhecimento. Enfim, as respostas
>estavam dadas! Os componentes usados na edificação da ponte, aqueles que
>compunham sua estrutura não eram mais nenhum mistério.
>
>
>
>Depois disso, fizeram vários corpos de prova na tentativa de obter
>compostos que proporcionassem resultados práticos semelhantes em
>resistência, flexibilidade e harmonia de formas. Muitos fracassos foram
>ocultados e as experiências de sucesso passaram a ser registradas e
>vendidas, proporcionando grandes somas aos seus 'descobridores'.
>
>
>
>Mesmo assim, ainda havia alguns pesquisadores insatisfeitos com todas
>aquelas maravilhosas descobertas. Esses, silenciosamente, passavam dias e
>dias pensando, refletindo e conjeturando na tentativa de redesenhar o
>projeto original da ponte. Investiam em intensas pesquisas com ultra-som,
>raios-X, ressonância magnética, cálculos complicadíssimos e muita, muita
>imaginação.
>
>
>
>A grande maioria dos habitantes achava perda de tempo essas tentativas de
>resgatar o projeto original ou pelo menos tentar elaborar um esboço dele.
>Afinal, diziam, a ponte já estava ali, totalmente pronta e utilizável. Para
>que então se preocupar com isso? Esses malucos! Será que não têm nada que
>fazer de mais útil!
>
>
>
>O que de fato preocupa a nós outros é que poucos estão empenhados em
>preservá-la. Poucos se dão conta de sua importância para a própria vida e à
>vida dos demais habitantes. Acreditamos que, se nada de concreto for
>acordado e algo de efetivo realizado, breve a ponte estará deteriorada,
>suja, contaminada e intransitável. A hora de agir já tarda.
>
>
>
>
>
>
>
>Nota: Este artigo foi inspirado nas idéias do filósofo Spinoza (1569-1630)
>de que Deus é igual à natureza - Deus sive natura. Afirma também, que a
>felicidade máxima do homem é o conhecimento de Deus e este é precisamente o
>fim ao qual a filosofia se propõe. Considera que Deus é a única substância
>existente. Como substância, Deus é a cadeia ou processo causal, a
>subjacente condição de todas as coisas, a lei e a estrutura do mundo. Esse
>concreto universo de modos e coisas está para Deus assim como uma ponte
>está para o seu projeto, sua estrutura, e as leis da matemática e da
>mecânica segundo as quais ela é construída; são estas as bases de
>sustentação, a condição subjacente, a substância da ponte; sem elas, ela
>cairia. (Tratado político-teológico)
>
>As idéias não têm dono, mas têm autoria.
>
>
>
>* José Renato M. de Almeida é engenheiro, aprendiz de filosofia e teologia.
>
>
>
>SSA, 1° de maio de 2003
>
>Rev. 14.06.2004
>
>
>
>[As partes desta mensagem que não continham texto foram removidas]
>
>
>
>##### ##### #####
>
>Para saber mais visite
>http://www.ciencialist.hpg.ig.com.br
>
>
>##### ##### ##### #####
>Links do Yahoo! Grupos
>
>
>
>
>
>
>
>




SUBJECT: Re: on topics
FROM: "rayfisica" <rayfisica@yahoo.com.br>
TO: ciencialist@yahoogrupos.com.br
DATE: 14/02/2005 04:18


--- Em ciencialist@yahoogrupos.com.br, "murilo filo"
<avalanchedrive@h...> escreveu
> Oi,tenho certeza de que êste link será de interesse de alguns.
> Já ví outros, mas êste me parece mais refinado e menos paranóico.
> abr/Murilo SP 11/fev
> >
> >http://www.suppressedscience.net/
> >No virus found in this outgoing message.
> >Checked by AVG Anti-Virus.
> >Version: 7.0.296 / Virus Database: 265.8.6 - Release Date: 7/2/2005

>>>>>>>>>>>>>>>>>>>>>>>>>
>>>>>>>>>>>>>>>>>>>>>>>>>>>>
>>>>>>>>>>>>>>>>>>>>>>>>>>>>>>>>

Peço permissão
Pode até ser refinado e (menos paranóico?).
Mas que esconde uma tentativa de controle eu não tenho nenhuma duvida
exatamente como o outro lado, parece política ou então os
protestantes que destruíram as obras de arte da idade media por onde
passaram apenas por serem de origem católica, bem... Cadê as obras
que eles criaram? Em nome de deus apenas destruíram.
Assim parecem esses, criticam o sistema que todos sabem ser uma droga
(xiiiiiii), mas o que eles tem posto no lugar destruir apenas por
destruir é terrorismo.
Eu particularmente não estou feliz com o que estão fazendo com o
conhecimento, mas eu não quero voltar às trevas, mostrar os (leaks)
do sistema como a dizer jogue todo o resto no lixo é paranóia sim.
O problema não são as falhas do sistema e sim aquelas pessoas que
apesar das falhas agem como se o sistema fosse perfeito (alguns
doutos arrogantes), ou então aqueles que devido às falhas querem
quebrar tudo, ora a luz é onda que apenas pode interagir como
partícula, então é partícula oras... (viva! José Newton).
Quanto ao neutrino, se tem massa não é o neutrino e sim uma outra
partícula, pois o neutrino original...
Isso tudo que eu escrevi é paranóico, pois eu sou paranóico.
Cadê a ciência pura e bela de Galileu, Copérnico e Newton, cadê...





SUBJECT: luz velocidade e energia
FROM: "rayfisica" <rayfisica@yahoo.com.br>
TO: ciencialist@yahoogrupos.com.br
DATE: 14/02/2005 07:29


Lendo e meditando sobre a questão iniciada por Sr. Helio, sobre a
velocidade da luz na água ser menor que no vácuo me ocorreu o
seguinte.
Algum tempo atrás o Sr. Taborda explicou-me que a luz interage com a
gravidade por ter energia, pergunto então:
Essa energia esta relacionada com a velocidade (C)?
Na água a luz tem mais, menos ou igual energia que no vácuo?
Se menos o que acontece coma energia extra quando o fóton entra na
água (alem de se molhar) ou um meio semelhante?
O que acelera o fóton quando sai desse meio? Ou de onde vem a energia
de aceleração?
O que acontece se colocarmos um corpo negro imerso no meio onde a luz
tem velocidade menor (pois se a luz tem menos energia que no vácuo
logo o mesmo quantum não pode excitar o elétron)?
Se a energia é a mesma na água, por exemplo, que no vácuo como se
explica isso? Velocidade diferente mesma energia, não ta faltando
alguma coisa?
A relação da luz na água é diferente do que no vácuo com relação à
gravidade (interage menos)?
Eu sei que são perguntas néscias porem sou um leigo, faze o que né.
Agradeço a atenção, se merecedor.






SUBJECT: Re: [ciencialist] Re: Fw: ciências
FROM: ÿffffcdtalo Rocha <imrochaguedes@yahoo.com.br>
TO: ciencialist@yahoogrupos.com.br
DATE: 14/02/2005 11:07

Paleontologia, paleobotânica, paleopedologia (estuda os solos do passado).
Ítalo Moraes Rocha Guedes.


---------------------------------
Yahoo! Acesso Grátis - Internet rápida e grátis. Instale o discador do Yahoo! agora.

[As partes desta mensagem que não continham texto foram removidas]



SUBJECT: Re: Fw: ciências
FROM: "rmtakata" <rmtakata@altavista.net>
TO: ciencialist@yahoogrupos.com.br
DATE: 14/02/2005 12:14


História.

[]s,

Roberto Takata

--- Em ciencialist@yahoogrupos.com.br, "Luiz Ferraz Netto"
> Sugestões?
> Luiz Ferraz Netto [Léo]

> -----Mensagem Original-----
> De: Lula Klautau
> Quero saber algumas ciências que estuda o passado





SUBJECT: Re: Fw: Eletrostatica
FROM: "rmtakata" <rmtakata@altavista.net>
TO: ciencialist@yahoogrupos.com.br
DATE: 14/02/2005 12:16


Não seria um boitatá? Vulgo: fogo-fátuo?

[]s,

Roberto Takata

--- Em ciencialist@yahoogrupos.com.br, "Luiz Ferraz Netto"
> Quem entende disso?
> []'
> ===========================
> Luiz Ferraz Netto [Léo]
> -----Mensagem Original-----
> De: lessa
> Assunto: Eletrostatica
>
> Meu Nome é antonio Lessa de Oliveira,
> REsido em em Sao Viocente - SP e meu email é aolessa@i...
>
> uma bolar de luz forte, com o som parecido com oleo quente
fritando..... e com cheiro de cabelo queimado e ozonio
>
> estavamos no mato em e um coordenado , e entao fez com que se
deita-se de barriga para baixo e uma bola de luz com as
caracteristicas acima apareceu de um ponto afastado e andou ate as
costas e se dividiu... e apagou
>
> esse fenomeno foi filmado e aconteu varias vezes coordenados por
pessoa X.....participei e o vipois era filmado....
> poderiam me ajudar? a esclarecer isto? como é feito





SUBJECT: Fw: Eletrostatica
FROM: "Luiz Ferraz Netto" <leobarretos@uol.com.br>
TO: "ciencialist" <ciencialist@yahoogrupos.com.br>
DATE: 14/02/2005 12:23

Quem entende disso?
[]'
===========================
Luiz Ferraz Netto [Léo]
leobarretos@uol.com.br
http://www.feiradeciencias.com.br
===========================
-----Mensagem Original-----
De: lessa
Para: leobarretos@uol.com.br
Enviada em: domingo, 13 de fevereiro de 2005 17:31
Assunto: Eletrostatica

Meu Nome é antonio Lessa de Oliveira,
REsido em em Sao Viocente - SP e meu email é aolessa@ig.com.br

oi estou procurando uma ersposta para o que vi, vou descrever o que aconteceu ... e assim poderiam me dizre como foi feito ou que fenomeno é esse:


uma bolar de luz forte, com o som parecido com oleo quente fritando..... e com cheiro de cabelo queimado e ozonio

estavamos no mato em e um coordenado , e entao fez com que se deita-se de barriga para baixo e uma bola de luz com as caracteristicas acima apareceu de um ponto afastado e andou ate as costas e se dividiu... e apagou

esse fenomeno foi filmado e aconteu varias vezes coordenados por pessoa X.....participei e o vipois era filmado....
poderiam me ajudar? a esclarecer isto? como é feito


--------------------------------------------------------------------------------


----------

No virus found in this outgoing message.
Checked by AVG Anti-Virus.
Version: 7.0.300 / Virus Database: 265.8.7 - Release Date: 10/02/2005


[As partes desta mensagem que não continham texto foram removidas]



SUBJECT: Re: Astrologia e Metereologia e top-posting (era : Zodiaco)
FROM: "Sergio M. M. Taborda" <sergiotaborda@terra.com.br>
TO: ciencialist@yahoogrupos.com.br
DATE: 14/02/2005 12:34


--- Em ciencialist@yahoogrupos.com.br, "rmtakata" <rmtakata@a...> escreveu
>
> --- Em ciencialist@yahoogrupos.com.br, "Sergio M. M. Taborda"
> > Ha dados para sustentar essa afirmação ou vc está chutando numeros ?
> > Se ha dados, por favor apresente as fontes.
>
> Hah dados.
>
> Nesta pagina a taxa eh de 80%:
>
> http://www.weather2000.com/hdd_0304_verif.html
>
> (Em amarelo os acertos)

Sendo que o erro é mais de 50% fica fcil que o valor observado fique
dentro dos limites. Previsões com essa margem a astrologia tb pode
fazer , pois seguindo o que os criticos dizem sobre a astrologia, ela
tem a mesma probabilidade do acaso, que sempre será de 50% para cada
hipotese (pois só ha 2 hipoteses - ou se tem a caracteristica, ou não
se tem).

>
> Vc pode fazer isso pegando as previsoes publicadas nos jornais e
> comparando.
>
> Em 1994, a taxa de acerto para 48 horas era de 70%:
> Are Forecasters Better Today? Probably. Deseret News, April 3, 1994,
> p.B-2
>
> Vc tb pode pedir dados para o pessoal do CPTec:

Poder, posso, mas a propria entrevista admite que prever o tempo é
muito dificil. A previsão de chuva com 80% de acerto nas primeiras 24h
e valida por 3 dias significa que - sendo bonzinhos e usando um
decaimento linear - no segundo dia a probabilidade é de apenas 40%.
E isso é para prever chuva. Agora prever a posição e data da formação
de um furacão é bem mais dificil.

Vc tinha apresentado valores de 95% de taxa de acerto para a
metereologia, mesmo combinando os valores das taxas apresentadas no
texto , chega apenas a 60% sem contar as previsões que realmente
estavam em causa no decorrer do assunto, que eram as previsões de
eventos catacliticos como furações - que fariam esse numero descer
mais ainda.


Sérgio Taborda






SUBJECT: Fw: F�sica (Acho q � termologia)
FROM: "Luiz Ferraz Netto" <leobarretos@uol.com.br>
TO: "ciencialist" <ciencialist@yahoogrupos.com.br>
DATE: 14/02/2005 12:36

Oi Roberto,

onde está o texto referente a isso? Agradeço,
[]'
===========================
Luiz Ferraz Netto [Léo]
leobarretos@uol.com.br
http://www.feiradeciencias.com.br
===========================
-----Mensagem Original-----
De: Karen Lopes de Freitas
Para: leobarretos@uol.com.br
Enviada em: domingo, 13 de fevereiro de 2005 21:28
Assunto: Física (Acho q é termologia)


Olá professor.
Me chamo Karen, tenho 14 anos e estou no segundo ano do ensino médio, técnico em processamento de dados.
Recentemente surgiu uma discussão em sala sobre: Por que água e cerveja congelam quando colocamos a mão no meio do recipiente em que estão guardados, após retirar do freezer ou congelador, mesmo que eles não estejam completamente endurecidos.
Já fiz várias pesquisas, mas ainda não consegui achar uma resposta satisfatória.
Conto com sua colaboração para desvendar esse 'enigma'.
Obrigada desde já.
Karen Lopes de Freitas


--------------------------------------------------------------------------------
Chegou o que faltava: MSN Acesso Grátis Instale Já!


--------------------------------------------------------------------------------


No virus found in this incoming message.
Checked by AVG Anti-Virus.
Version: 7.0.300 / Virus Database: 265.8.7 - Release Date: 10/02/2005

----------

No virus found in this outgoing message.
Checked by AVG Anti-Virus.
Version: 7.0.300 / Virus Database: 265.8.7 - Release Date: 10/02/2005


[As partes desta mensagem que não continham texto foram removidas]



SUBJECT: Fw: Sensor I�nico
FROM: "Luiz Ferraz Netto" <leobarretos@uol.com.br>
TO: "ciencialist" <ciencialist@yahoogrupos.com.br>
DATE: 14/02/2005 12:56

Alguém ajuda?
[]'
===========================
Luiz Ferraz Netto [Léo]
leobarretos@uol.com.br
http://www.feiradeciencias.com.br
===========================
-----Mensagem Original-----
De: Carlos Pedro
Para: leobarretos@uol.com.br
Cc: luis.penteado@instalarme.com.br
Enviada em: segunda-feira, 14 de fevereiro de 2005 10:59
Assunto: Sensor Iônico


Possuimos Sensor Iônico, cujo circuito eletrônico danificado por vazamento de pilha; aí o porém,este sensor possui um dispositivo de detecção que utiliza material segundo impresso no produto dizendo:Radioactive-Material Americium 241,
1.0 Mcrocurie.Qual empresa especializada faz coleta do material ?



Grato:Luís (19) 3543-7039


--------------------------------------------------------------------------------


No virus found in this incoming message.
Checked by AVG Anti-Virus.
Version: 7.0.300 / Virus Database: 265.8.7 - Release Date: 10/02/2005

----------

No virus found in this outgoing message.
Checked by AVG Anti-Virus.
Version: 7.0.300 / Virus Database: 265.8.7 - Release Date: 10/02/2005


[As partes desta mensagem que não continham texto foram removidas]



SUBJECT: Re: [ciencialist] Fw: Sensor Iônico
FROM: JVictor <jvoneto@uol.com.br>
TO: ciencialist@yahoogrupos.com.br
DATE: 14/02/2005 13:12

Procure a Comissão de Energia Nuclear aí de sua cidade ou na Capital de
seu estado.
Eles possuem todo um processo de recepção de materiais radioativos. Lá,
fornecem todos os dados, formulários
e instruções para embalagem e transporte dos produtos. Se tiver
dificuldade, um telefonema para o
Departamento de Energia Nuclear Univ.federal mais próxima pode lhe
indicar o caminho a seguir.

sds,

Victor




Luiz Ferraz Netto escreveu:

> Alguém ajuda?
> []'
> ===========================
> Luiz Ferraz Netto [Léo]
> leobarretos@uol.com.br
> http://www.feiradeciencias.com.br
> ===========================
> -----Mensagem Original-----
> De: Carlos Pedro
> Para: leobarretos@uol.com.br
> Cc: luis.penteado@instalarme.com.br
> Enviada em: segunda-feira, 14 de fevereiro de 2005 10:59
> Assunto: Sensor Iônico
>
>
> Possuimos Sensor Iônico, cujo circuito eletrônico danificado por
> vazamento de pilha; aí o porém,este sensor possui um dispositivo de
> detecção que utiliza material segundo impresso no produto
> dizendo:Radioactive-Material Americium 241,
> 1.0 Mcrocurie.Qual empresa especializada faz coleta do material ?
>
>
>
> Grato:Luís (19) 3543-7039
>
>
> --------------------------------------------------------------------------------
>
>
> No virus found in this incoming message.
> Checked by AVG Anti-Virus.
> Version: 7.0.300 / Virus Database: 265.8.7 - Release Date: 10/02/2005
>
> ----------
>
> No virus found in this outgoing message.
> Checked by AVG Anti-Virus.
> Version: 7.0.300 / Virus Database: 265.8.7 - Release Date: 10/02/2005
>
>
> [As partes desta mensagem que não continham texto foram removidas]
>
>
>
> ##### ##### #####
>
> Para saber mais visite
> http://www.ciencialist.hpg.ig.com.br
>
>
> ##### ##### ##### #####
>
>
> *Yahoo! Grupos, um serviço oferecido por:*
> PUBLICIDADE
> <http://br.rd.yahoo.com/SIG=12argensj/M=264379.5078783.6203979.1588051/D=brclubs/S=2137111528:HM/EXP=1108477970/A=2332652/R=0/id=noscript/SIG=119058f8i/*http://br.download.yahoo.com/messenger/>
>
>
>
> ------------------------------------------------------------------------
> *Links do Yahoo! Grupos*
>
> * Para visitar o site do seu grupo na web, acesse:
> http://br.groups.yahoo.com/group/ciencialist/
>
> * Para sair deste grupo, envie um e-mail para:
> ciencialist-unsubscribe@yahoogrupos.com.br
> <mailto:ciencialist-unsubscribe@yahoogrupos.com.br?subject=Unsubscribe>
>
> * O uso que você faz do Yahoo! Grupos está sujeito aos Termos do
> Serviço do Yahoo! <http://br.yahoo.com/info/utos.html>.
>
>
>
>
> __________ Informação do NOD32 1.998 (20050212) __________
>
> Esta mensagem foi verificada pelo NOD32 Sistema Antivírus
> http://www.nod32.com.br




SUBJECT: Re: Astrologia e Metereologia e top-posting (era : Zodiaco)
FROM: "rmtakata" <rmtakata@altavista.net>
TO: ciencialist@yahoogrupos.com.br
DATE: 14/02/2005 13:21


--- Em ciencialist@yahoogrupos.com.br, "Sergio M. M. Taborda"
> > Nesta pagina a taxa eh de 80%:
> >
> > http://www.weather2000.com/hdd_0304_verif.html

> Sendo que o erro é mais de 50% fica fcil que o valor observado fique
> dentro dos limites. Previsões com essa margem a astrologia tb pode
> fazer , pois seguindo o que os criticos dizem sobre a astrologia,
> ela tem a mesma probabilidade do acaso, que sempre será de 50% para
> cada hipotese (pois só ha 2 hipoteses - ou se tem a caracteristica,
> ou não se tem).

Se a astromancia eh capaz disso, nao sei por q. nao o faz. Isso
calaria a boca dos criticos. (Embora o erro da meteorologia nao seja
de mais de 50% no intervalo considerado. Ha' o erro padrao, mas nada
de absurdo. Por exemplo, em Asheville, NC, para dez e mai, a min
prevista foi de 3.180 e a max de 3.669, uma variacao de 7% da media.)

E nao eh boa estatistica dizer q. sempre se tem 50% de chances de
acerto: ou eh ou nao eh. De fato sao duas as possibilidades, mas
seriam de 50% de chances apenas se fossem equiprovaveis. Ou eu acerto
na loteria ou eu nao acerto na loteria, nao se segue q. eu tenha 50%
de chances de acertar na loteria.

> Poder, posso, mas a propria entrevista admite que prever o tempo é
> muito dificil. A previsão de chuva com 80% de acerto nas primeiras
> 24h e valida por 3 dias significa que - sendo bonzinhos e usando um
> decaimento linear - no segundo dia a probabilidade é de apenas 40%.
> E isso é para prever chuva. Agora prever a posição e data da
> formação de um furacão é bem mais dificil.

Quero ver a astromancia chegar perto dessas taxas. O segundo dia de
probabilidade nao eh de 40%, vc simplesmente nao pode fazer uma
extrapolacao linear. As taxas de acerto para dois dias - no caso de
chuva - ficam entre 70% e 80%: depende da qualidade dos dados e da
localidade.

------------------------------
What is 24 to 48 hours forecast for agricultural operations?
* Forecast emphasis is about:
o High and low temperature
o Wind velocity and direction
o Sunshine duration
o Time and amount of rainfall
o Relative Humidity
* Forecast accuracy is 70 to 80 %.
http://www.vusat.org/learning/weather/crop_weather/weather_forcasting/types.htm#1
------------------------------

> Vc tinha apresentado valores de 95% de taxa de acerto para a
> metereologia, mesmo combinando os valores das taxas apresentadas no

E eu boto feh q. ha' taxas de ateh 95%. Mas se quiser pegar os 40% q.
vc inventou para dois dias, otimo: quero ver a astromancia chegar
perto disso. Cacique Cobra Coral nao acrescenta nada ao conhecimento
meteorologico.

[]s,

Roberto Takata






SUBJECT: Re: [ciencialist] Zodiaco e Acupuntura..
FROM: "Esteban Moreno" <estebanmoreno@idhi.org.br>
TO: <ciencialist@yahoogrupos.com.br>
DATE: 14/02/2005 13:45

Ola Homero, como prefere ser chamado, não vou me alongar nessa discussão
pós-férias. Apenas vou dar mais um pitaco num trecho que merece algum
comentário:

"Assim, acusar os que pesqusiasm com ironias do tipo "santos céticos
imparcialistas" é bobagem e uma falacia "ad hominem".

Foi por certo colocado de uma forma bastante irônica, como é comum a quase
todos deste fórum, mas não é tão falacioso como supõe. A ciência é parcial,
a idéia de objetividade na ciência mudou drasticamente nos últimos tempos, e
diversos estudos corroboram neste sentido. Talvez mais uma vez lhe pareça
inverossímil, mas vários pesquisadores tentam compreender como a ciência
teria se desenvolvido sem a discriminação histórica das mulheres, por
exemplo, ou mesmo como o formato da organização do laboratório interfere nos
resultados colhidos em experimentos (sugiro buscar os trabalhos de BRUNO
LATOUR ou pesquisar no google sobre CULTURAL STUDIES SCIENCE). Concordo que
há uma tendência a um nivelamento ao longo do tempo, mas isso não torna a
ciência imune a uma série de erros históricos, ainda mais em um pleno estado
transiente sob o qual nos encontramos. A astrologia para mim é um desses
erros, o seus netos um dia te dirão, sob os receios de uma forte "bengalada"
do vovô ;-). De forma semelhante tem ocorrido com a acupuntura, como queira
compreender ou não a acupuntura. Pois concordo que a acupuntura cujo artigo
devas estar se referenciando não é a acupuntura aprovada como especialidade
médica em diversos paises do mundo e no próprio Brasil. Esta que me
referencio é essencialmente a mesma acupuntura do velhinho de mais de 3000
anos atrás.

Não quero dizer que a ciência seja imprópria, apenas tendenciosa. Como a
maioria de vocês, tb cultuo a crença (ops, desculpe, a visão) do Einstein de
que pouco sabemos do universo e a ciência é o que melhor temos. Que coisa
mais chata deve ser para os céticos perceber que a maioria das mais
importantes descobertas da ciência até os tempos mais recentes foram feitas
por pessoas que possuíam alguma visão "espiritualizada" do universo. Que
desagradável deve ser descobrir que a quantidade de cientistas religiosos
está aumentando no mundo (esta estatística está no almanaque abril).
Chacoalhem-se logo, criem algo original, vocês tem que manter a pálida vela
do falecido Carl acesa. ;-)

No aguardo de mais uma looooooooonga resposta.
Um abraço,
Esteban.





SUBJECT: Ensinando cálculo infinitesimal de maneira amena
FROM: "Esteban Moreno" <estebanmoreno@idhi.org.br>
TO: <ciencialist@yahoogrupos.com.br>, "Vivens Scientia" <VivensScientia@yahoogroups.com>
DATE: 14/02/2005 13:51

Sem comentários.
Esteban.


* Calculus Hotties - Ensinando cálculo infinitesimal de maneira amena (3587 kB)

http://media.hamncheez.com/videos/calculus_hotties.wmv

[As partes desta mensagem que não continham texto foram removidas]



SUBJECT: Re: Astrologia e Metereologia e top-posting (era : Zodiaco)
FROM: "Sergio M. M. Taborda" <sergiotaborda@terra.com.br>
TO: ciencialist@yahoogrupos.com.br
DATE: 14/02/2005 14:20


--- Em ciencialist@yahoogrupos.com.br, "rmtakata" <rmtakata@a...> escreveu
>
> --- Em ciencialist@yahoogrupos.com.br, "Sergio M. M. Taborda"
> > > Nesta pagina a taxa eh de 80%:
> > >
> > > http://www.weather2000.com/hdd_0304_verif.html
>
> > Sendo que o erro é mais de 50% fica fcil que o valor observado fique
> > dentro dos limites. Previsões com essa margem a astrologia tb pode
> > fazer , pois seguindo o que os criticos dizem sobre a astrologia,
> > ela tem a mesma probabilidade do acaso, que sempre será de 50% para
> > cada hipotese (pois só ha 2 hipoteses - ou se tem a caracteristica,
> > ou não se tem).
>
> Se a astromancia eh capaz disso, nao sei por q. nao o faz.

Simplesmente não está interessada. Mas qq um pode recolher os dados e
fazer isso ele mesmo. É dificil, mas não é impossivel.

> (Embora o erro da meteorologia nao seja
> de mais de 50% no intervalo considerado. Ha' o erro padrao, mas nada
> de absurdo. Por exemplo, em Asheville, NC, para dez e mai, a min
> prevista foi de 3.180 e a max de 3.669, uma variacao de 7% da media.)

Mas por exemplo ,para Miami vc tem um valor máximo estimado de 270, o
valor observado foi 112 e a média é de 119. A variação é de 68%.
((270-106 / 2) /119)




>
> E nao eh boa estatistica dizer q. sempre se tem 50% de chances de
> acerto: ou eh ou nao eh. De fato sao duas as possibilidades, mas
> seriam de 50% de chances apenas se fossem equiprovaveis.
>Ou eu acerto na loteria ou eu nao acerto na loteria, nao se segue q.
>eu tenha 50% de chances de acertar na loteria.

Sim, mas a minha previsão do seu acerto só tem 50% para cada uma.
(supondo, como vcs supoem que a astrologia é mero acaso)

> > Poder, posso, mas a propria entrevista admite que prever o tempo é
> > muito dificil. A previsão de chuva com 80% de acerto nas primeiras
> > 24h e valida por 3 dias significa que - sendo bonzinhos e usando um
> > decaimento linear - no segundo dia a probabilidade é de apenas 40%.
> > E isso é para prever chuva. Agora prever a posição e data da
> > formação de um furacão é bem mais dificil.
>
> Quero ver a astromancia chegar perto dessas taxas.

Faça mapas, interrogue as suas cobaias e faça as contas.

O segundo dia de
> probabilidade nao eh de 40%, vc simplesmente nao pode fazer uma
> extrapolacao linear.

Pois não. Eu disse, se assumirmos um decaimento linear.
Na realidade o decaimento é exponencial.

As taxas de acerto para dois dias - no caso de
> chuva - ficam entre 70% e 80%: depende da qualidade dos dados e da
> localidade.

Vc está falando dos numeros que o medelo dá, mas o modelo dá muitos
numeros mesmo para depois do 3° dia. A questão é que se no 1° dia tem
80% e no terceiro tem 0 pq já não é mais válida, no meio terá um
valor conforme um decaimento do modelo. Eu usei o lienar pq é o mais
simples.
O ponto, é que a probabilidade depende do tempo, a da astrologia não
depende.



> E eu boto feh q. ha' taxas de ateh 95%. Mas se quiser pegar os 40% q.
> vc inventou para dois dias, otimo: quero ver a astromancia chegar
> perto disso. Cacique Cobra Coral nao acrescenta nada ao conhecimento
> meteorologico.

Mas quem tá preocupado com o conhecimento metereologico são vcs. Eu
estou preocupado com a afirmação que a meteorologia é mais exacta que
a astrologia. Ela simplesmente não é, pq nem sequer tem previsões
exactas de todos os fenomenos.
Se quiser comparar o acerto da pressão e temperatura com o acerto do
signo lunar da astrologia, vai ver que o da astrologia é muito mais
fiável. Mas se quiser comparar o acerto da formação de um furacaõ com
o acerto do casamento da pessoa vc nem poderá , sendo que não existe
real previsão da formação de furacões mas apenas aviso previso da já
existencia deles.


Sérgio Taborda






SUBJECT: Re: Astrologia e Metereologia e top-posting (era : Zodiaco)
FROM: "rmtakata" <rmtakata@altavista.net>
TO: ciencialist@yahoogrupos.com.br
DATE: 14/02/2005 14:44


--- Em ciencialist@yahoogrupos.com.br, "Sergio M. M. Taborda"
> > Se a astromancia eh capaz disso, nao sei por q. nao o faz.
>
> Simplesmente não está interessada. Mas qq um pode recolher os dados

Nao interessada? Os astromantes estao avidos para ter uma comprovacao
cientifica. Como nao conseguem, apesar das diversas tentativas, ficam
como a raposa sobre as uvas.

A bem da verdade, muitos cientistas tb estao avidos para q. a
astromancia possa ser comprovada cientificamente. Isso traria uma nova
perspectiva para as ciencias.

Infelizmente, como dito desde o inicio, qdo devidamente testada, a
astromancia falha clamorosamente.

> Mas por exemplo ,para Miami vc tem um valor máximo estimado de 270,
> o valor observado foi 112 e a média é de 119. A variação é de 68%.
> ((270-106 / 2) /119)

E vc pega varios dados e faz uma analise estatistica: 80% de acerto.
Nada mal. Poderia ser melhor? Esperamos q. venha a ser no futuro. A
astromancia nao chega nem perto disso. No dia em q. chegar, de bom
grado ela terah algo a nos dizer.

> > E nao eh boa estatistica dizer q. sempre se tem 50% de chances de
> > acerto: ou eh ou nao eh. De fato sao duas as possibilidades, mas
> > seriam de 50% de chances apenas se fossem equiprovaveis.
> >Ou eu acerto na loteria ou eu nao acerto na loteria, nao se segue
> >q. eu tenha 50% de chances de acertar na loteria.
>
> Sim, mas a minha previsão do seu acerto só tem 50% para cada uma.
> (supondo, como vcs supoem que a astrologia é mero acaso)

Ser mero acaso nao eh sinonimo de 50% - nem qdo sao duas alternativas.
Apenas qdo as duas alternativas sao equiprovaveis.

> > Quero ver a astromancia chegar perto dessas taxas.
>
> Faça mapas, interrogue as suas cobaias e faça as contas.

Jah foi feito. Nao chegou nem perto. Eh isso q. eu falo q. qdo
devidamente testada a astromancia nega fogo.

> Pois não. Eu disse, se assumirmos um decaimento linear.
> Na realidade o decaimento é exponencial.

Tb nao. Segue uma curva sigmoidal.

> Vc está falando dos numeros que o medelo dá, mas o modelo dá muitos
> numeros mesmo para depois do 3° dia. A questão é que se no 1° dia
> tem 80% e no terceiro tem 0 pq já não é mais válida, no meio terá um

No terceiro dia nao eh 0. Ser invalido eh ficar na taxa do acaso.
Afinal se eh zero, quer dizer q. se erra sistematicamente - entao
bastaria inverter a previsao e teriamos 100% de acerto!

> O ponto, é que a probabilidade depende do tempo, a da astrologia não
> depende.

Nao depende. Mantem-se ao nivel casual - ou no maximo, a certas dicas
q. nada tEm a ver com o mapa natal e outras configuracoes celestes.

> Mas quem tá preocupado com o conhecimento metereologico são vcs. Eu
> estou preocupado com a afirmação que a meteorologia é mais exacta
> que a astrologia. Ela simplesmente não é, pq nem sequer tem
> previsões exactas de todos os fenomenos.

Nao ter previsoes exatas nao quer dizer q. nao seja mais exata do q. a
astromancia. Se a meteorologia nao tem previsao exata, a astromancia
tem menos ainda. A meteorologia consegue niveis de 95% de acerto para
dois dias - ou se vc quiser, podemos ficar com os seus 40% - a
astromancia nao chega nem perto disso.

> Se quiser comparar o acerto da pressão e temperatura com o acerto do
> signo lunar da astrologia, vai ver que o da astrologia é muito mais
> fiável.

Acertar o signo lunar ou acertar o q. acontece com a pessoa baseada em
seu signo lunar?

[]s,

Roberto Takata





SUBJECT: Re: [ciencialist] Fw: Sensor Iônico
FROM: "Marcos Borges" <maborges@ccard.com.br>
TO: <ciencialist@yahoogrupos.com.br>
DATE: 14/02/2005 15:01

Este sensor iônico possui uma fonte seleda e deve ser tratada como rejeito
radioativo. Aconselho a entrar em contato com CNEN (www.cnen.gov.br) e
notifica-la. A CNEN possui unidades no Rio, São Paulo e BH que fazem coleta
desse material.


Abraços
Marcos Borges
Biólogo - UERJ
----- Original Message -----
From: "Luiz Ferraz Netto" <leobarretos@uol.com.br>
To: "ciencialist" <ciencialist@yahoogrupos.com.br>
Sent: Monday, February 14, 2005 11:56 AM
Subject: [ciencialist] Fw: Sensor Iônico


>
> Alguém ajuda?
> []'
> ===========================
> Luiz Ferraz Netto [Léo]
> leobarretos@uol.com.br
> http://www.feiradeciencias.com.br
> ===========================
> -----Mensagem Original-----
> De: Carlos Pedro
> Para: leobarretos@uol.com.br
> Cc: luis.penteado@instalarme.com.br
> Enviada em: segunda-feira, 14 de fevereiro de 2005 10:59
> Assunto: Sensor Iônico
>
>
> Possuimos Sensor Iônico, cujo circuito eletrônico danificado por vazamento
> de pilha; aí o porém,este sensor possui um dispositivo de detecção que
> utiliza material segundo impresso no produto dizendo:Radioactive-Material
> Americium 241,
> 1.0 Mcrocurie.Qual empresa especializada faz coleta do material ?
>
>
>
> Grato:Luís (19) 3543-7039
>
>
> --------------------------------------------------------------------------------
>
>
> No virus found in this incoming message.
> Checked by AVG Anti-Virus.
> Version: 7.0.300 / Virus Database: 265.8.7 - Release Date: 10/02/2005
>
> ----------
>
> No virus found in this outgoing message.
> Checked by AVG Anti-Virus.
> Version: 7.0.300 / Virus Database: 265.8.7 - Release Date: 10/02/2005
>
>
> [As partes desta mensagem que não continham texto foram removidas]
>
>
>
> ##### ##### #####
>
> Para saber mais visite
> http://www.ciencialist.hpg.ig.com.br
>
>
> ##### ##### ##### #####
> Links do Yahoo! Grupos
>
>
>
>
>
>
>
>
>
>



SUBJECT: Re: Astrologia e Metereologia e top-posting (era : Zodiaco)
FROM: "rmtakata" <rmtakata@altavista.net>
TO: ciencialist@yahoogrupos.com.br
DATE: 14/02/2005 15:34


--- Em ciencialist@yahoogrupos.com.br, "rmtakata" <rmtakata@a...>
> Nao interessada? Os astromantes estao avidos para ter uma
> comprovacao cientifica.

Uma observacao: nem todos. Alguns astromantes nao estao nem um pouco
preocupado com isso. E ha' os q. admitem q. nada tem de cientifico.

[]s,

Roberto Takata





SUBJECT: Re:[ciencialist] Re: Astrologia e METEOROLOGIA e top-posting (era : Zodiaco)
FROM: "Anne" <anne_moraes@bol.com.br>
TO: "ciencialist" <ciencialist@yahoogrupos.com.br>
DATE: 14/02/2005 15:41

Vamos aproveitar que estamos numa lista de pessoas tão cultas para escrever certo ao menos o nome do tópico em discussão?
Vamos lá:
METEOROLOGIA!!!
Grata,
Anne Moraes.

> --- Em ciencialist@yahoogrupos.com.br, "rmtakata"
> > Nao interessada? Os astromantes estao avidos para ter uma
> > comprovacao cientifica.
>
> Uma observacao: nem todos. Alguns astromantes nao estao nem um pouco
> preocupado com isso. E ha' os q. admitem q. nada tem de cientifico.
>
> []s,
>
> Roberto Takata
>
>
>
>
>
> ##### ##### #####
>
> Para saber mais visite
> http://www.ciencialist.hpg.ig.com.br
>
>
> ##### ##### ##### #####
> Links do Yahoo! Grupos
>
>
>
>
>
>
>
>

__________________________________________________________________________
Acabe com aquelas janelinhas que pulam na sua tela.
AntiPop-up UOL - É grátis!
http://antipopup.uol.com.br/




[As partes desta mensagem que não continham texto foram removidas]



SUBJECT: P/ Esteban ( Zodiaco e Acupuntura..)
FROM: "Oraculo" <oraculo@atibaia.com.br>
TO: <ciencialist@yahoogrupos.com.br>
DATE: 14/02/2005 16:41

Olá Esteban

risos..:-) Vou tentar uma reposta mais curta, ok?..:-)

Primeiro, não há erro na avaliação que faz sobre a influencia de carater e cultura na ciência, ou melhor, nos cientistas. Nem vou discordar..:-) Todos concordam com isso, e eu mesmo já coloquei essa posição diversas vezes. Mas, é exatamente por saber disso , que muitas das ferramentas e rigores do métido surgiram. Por conhecer a enorme capacidade da mente humana de se enganar, distorcer ou influenciar análises, que se tornou necessário que diversas pessoas refaçam cada experimento, que muitos pesqusiadores cheguem a mesma conclusão, que os efeitos e previsões decorrentes sejam coerentes e sólidos, tudo no sentido de filtrar o problema.

E, claro, nunca é 100% de certeza..:-)

Mas, mesmo assim, mesmo sem 100%, é fácil perceber que é mais confiável e eficaz usar conhecimentos produzidos dessa forma. No passado, existiam diversos modos de explicar os fenomenos, milhares deles, e diferentes visões de mundo, diferentes formas de agir em relação a estes fenomenos e explicações. E todas tinham igual eficácia, quase nenhuma. Não importa se tratava uma doença dessa ou daquela forma, como demonio maligno ou miasma sobrenatural, como desequilibrio energético ou como punição divina, a taxa de cura era a mesma que não tratar. Isso fica claro na avaliação da espectativa média de vida, de menos de 30 anos (em alguns lugares, menos de 25 anos).

Hoje, com o uso do conhecimento cientifico sobre doenças, nossa capacidade de curar é enorme e a espectativa média de vida mais de 80 anos nos paises mais civilizados.

Posso aplicar essa análise para qualquer área do conhecimento humano, qualquer fenomeno ou explicação de fenomeno já produzida pelo homem. Se a Lua é uma cabaça pouco mais alta que o topo das montanhas ou um enorme satelite rochoso a orbitar a Terra a milhares de kilometros, cada explicação produz diferentes consequencias e podemos testar o conjunto de conhecimento sobre nosso satelite.

A ciência não é tendenciosa, os homens (e cientistas são homens) são, certamente. E sabem disso. Por isso o rigor atinge nào apenas a acupuntura, mas qualquer alegação, mesmo as produzidas dentro de laboratorios de ponta e por cientistas de renome. A fusão a frio é um excelente exemplo. Não foi aceita simplesmenete porque nào cumpriu os requisitos do método: nenhum outro laboratório conseguiu reproduzir seus resultados. E era ciência, criada em laboratorio e com bases fisicas.

E não incomoda descobrir que muitos pesquisadores tinham uma visão "espiritual" do mundo..:-) Seria tolo esperar que não tivessem, já que são homens, frutos de sua época. Mesmo hoje, onde mais da metade dos cientistas não professa religião ou crença, ainda se encontram quem tenha uma visão espiritual. O que não significa que levem essa visão para dentro do laboratorio, pelo menos na maioria das vezes..:-) Nenhum deles espera que um fenomeno fisico seja alterado ou afetado por sua crença ou pela ação do sobrenatural. Seus experimentos são construidos sem levar em conta essas variáveis.

Espiritual nem sempre significa o mesmo para diferentes pessoas. Eu vejo a dimensão espiritual humana, mas não como algo transcendente ou sobrenatural, mas como uma caracteristica de nossa espécie, algo abstrato, subjetivo, que nos dá a dimensão humana, a capacidade de criar conceitos como bem, mal, honra, generosidade, etc. Não preciso de um espírito, no sentido ectoplasmatico, para isso..:-)

Para você a astrologia é real, e um dia nós, pobres cientistas e céticos, perceberemos o engano..:-) Ótimo, isso significa que tem uma crença na astrologia. Evidentemente deve perceber que todas as outras crenças dizem a mesma coisa, que um dia poderão provar que estavam corretas e teremos de aceita-las..:-) O que nào parecem perceber é que será um prazer aceitar qualquer dessas alegações, desde que venham acompanhadas das evidencais e do rigor necessário, não algo a ser recebido com bengaladas..:-) Novas descobertas, por mais espantosas que sejam, sempre serão o motivo principal da ciência. Apenas o cuidado de embasar essas descobertas com dados e provas é que tem de ser tomado, sob pena de voltarmos ao passado, com milhares de diferentes explicações, todas incorretas e com a mesma ineficiencia de ação.

Esteban, coisas mais malucas que a astrologia são aceitas pela ciência, simplesmente porque podem apresentar evidencias sólidas e sobreviver a análise. Coisas como continentes que se movem e seres invisiveis que causam doenças (sem falar em relatividade e quantica..:-). Não é birra, é cuidado, extremo cuidado, que tem dado excelente resultado prático.

O aumento de cientistas religiosos, mesmo que fosse verdadeiro, em nada muda a ciência. Ela tem sobrevivido a coisas piores que isso e seu rigor é justamente para produzir dados e conclusões confiáveis, seja seu autor muçulmano, catolico, budista, shintoista ou ateu. Não há uma ciência católica (a igreja pensa que há, mas é um engano fácil de demonstrar..:-), nem muçulmana. Há cientistas catolicos e muçulmanos. Embora estejam diminuindo e não aumentando..:-)

Um abraço.

Homero

PS: Pode me chamar de Oraculo ou de Homero, não tenho preferencias..:-) E, sinto muito, acho que o texto acabou enorme mesmo..:-)





----- Original Message -----
From: Esteban Moreno
To: ciencialist@yahoogrupos.com.br
Sent: Monday, February 14, 2005 12:45 PM
Subject: Re: [ciencialist] Zodiaco e Acupuntura..


Ola Homero, como prefere ser chamado, não vou me alongar nessa discussão
pós-férias. Apenas vou dar mais um pitaco num trecho que merece algum
comentário:

"Assim, acusar os que pesqusiasm com ironias do tipo "santos céticos
imparcialistas" é bobagem e uma falacia "ad hominem".

Foi por certo colocado de uma forma bastante irônica, como é comum a quase
todos deste fórum, mas não é tão falacioso como supõe. A ciência é parcial,
a idéia de objetividade na ciência mudou drasticamente nos últimos tempos, e
diversos estudos corroboram neste sentido. Talvez mais uma vez lhe pareça
inverossímil, mas vários pesquisadores tentam compreender como a ciência
teria se desenvolvido sem a discriminação histórica das mulheres, por
exemplo, ou mesmo como o formato da organização do laboratório interfere nos
resultados colhidos em experimentos (sugiro buscar os trabalhos de BRUNO
LATOUR ou pesquisar no google sobre CULTURAL STUDIES SCIENCE). Concordo que
há uma tendência a um nivelamento ao longo do tempo, mas isso não torna a
ciência imune a uma série de erros históricos, ainda mais em um pleno estado
transiente sob o qual nos encontramos. A astrologia para mim é um desses
erros, o seus netos um dia te dirão, sob os receios de uma forte "bengalada"
do vovô ;-). De forma semelhante tem ocorrido com a acupuntura, como queira
compreender ou não a acupuntura. Pois concordo que a acupuntura cujo artigo
devas estar se referenciando não é a acupuntura aprovada como especialidade
médica em diversos paises do mundo e no próprio Brasil. Esta que me
referencio é essencialmente a mesma acupuntura do velhinho de mais de 3000
anos atrás.

Não quero dizer que a ciência seja imprópria, apenas tendenciosa. Como a
maioria de vocês, tb cultuo a crença (ops, desculpe, a visão) do Einstein de
que pouco sabemos do universo e a ciência é o que melhor temos. Que coisa
mais chata deve ser para os céticos perceber que a maioria das mais
importantes descobertas da ciência até os tempos mais recentes foram feitas
por pessoas que possuíam alguma visão "espiritualizada" do universo. Que
desagradável deve ser descobrir que a quantidade de cientistas religiosos
está aumentando no mundo (esta estatística está no almanaque abril).
Chacoalhem-se logo, criem algo original, vocês tem que manter a pálida vela
do falecido Carl acesa. ;-)

No aguardo de mais uma looooooooonga resposta.
Um abraço,
Esteban.





##### ##### #####

Para saber mais visite
http://www.ciencialist.hpg.ig.com.br


##### ##### ##### #####


Yahoo! Grupos, um serviço oferecido por:
PUBLICIDADE




------------------------------------------------------------------------------
Links do Yahoo! Grupos

a.. Para visitar o site do seu grupo na web, acesse:
http://br.groups.yahoo.com/group/ciencialist/

b.. Para sair deste grupo, envie um e-mail para:
ciencialist-unsubscribe@yahoogrupos.com.br

c.. O uso que você faz do Yahoo! Grupos está sujeito aos Termos do Serviço do Yahoo!.



[As partes desta mensagem que não continham texto foram removidas]



SUBJECT: P/ Anne (era Astrologia e METEOROLOGIA )
FROM: "Oraculo" <oraculo@atibaia.com.br>
TO: <ciencialist@yahoogrupos.com.br>
DATE: 14/02/2005 17:01

Olá Anne

A culpa é minha, eu escrevi errado logo na primeira mensagem (e no campo assunto, para não deixar duvidas..:-) e virou gozação (eu sabia que iria pagar pelo erro de escrever rapido e "de ouvido"..:-)

O problema é que agora vai se eternizar o meu "metereologia" no campo assunto, no lugar do correto meteorologia..:-) Sinto muito..:-)

Um abraço.

Homero



[As partes desta mensagem que não continham texto foram removidas]



SUBJECT: Re: [ciencialist] Re: Fw: ciências
FROM: "Luiz Ferraz Netto" <leobarretos@uol.com.br>
TO: <ciencialist@yahoogrupos.com.br>
DATE: 14/02/2005 17:59

Pergunta da Lula Klautau
> Quero saber algumas ciências que estuda o passado

Takata sugere:
>História.

Léo argumenta:

Ela pediu alguma Ciência ... História não é algo científico! Todos meus professores de História alteraram os fatos a bel prazer ... era um festival de ad hoc!

[]'


--
No virus found in this outgoing message.
Checked by AVG Anti-Virus.
Version: 7.0.300 / Virus Database: 265.8.7 - Release Date: 10/02/2005



SUBJECT: Re: [ciencialist] P/ Esteban ( Zodiaco e Acupuntura..)
FROM: "Esteban Moreno" <estebanmoreno@idhi.org.br>
TO: <ciencialist@yahoogrupos.com.br>
DATE: 14/02/2005 18:04

Bom, veio looooonga mesmo ;-)
Quanto ao trecho: "...isso significa que tem uma crença na astrologia.
Evidentemente deve perceber que todas as outras crenças dizem a mesma
coisa". São duas frases com duas fortes arbitrariedades, colocadas de forma
tendenciosa aos seus próprios propósitos, como quase todo o resto. A
principal exceção deve-se: "A ciência não é tendenciosa, os homens (e
cientistas são homens) são.." E de fato, expressei-me mal, você está
correto, só faltou substituir homens por seres humanos. Faz parte de nossa
linguagem preconceituosa.

Além do mais e principalmente: "não há qualquer trabalho verossímil que
prove o não funcionamento da astrologia". Vocês se apóiam em vazios, pura
especulação preconceituosa e mal formulada. Por quantas e quantas vezes
pendurarão em vossas telas artigos supostamente científicos que tentam
provar falhas da astrologia com argumentos das quais a astrologia não se
propõe ou, quando muito, com erros básicos, como basear-se apenas em um
planeta ou uma posição no mapa. Eu realmente não tenho a paciência do
Taborda ou do Mesquita para responder-te ao resto. Mesmo porque, diga-me, a
esta altura, há algum fruto nessa discussão?

Mas ficou uma curiosidade, quando afirmas: "Eu vejo a dimensão espiritual
humana, mas não como algo transcendente ou sobrenatural, mas como uma
característica de nossa espécie, algo abstrato, subjetivo, que nos dá a
dimensão humana, a capacidade de criar conceitos como bem, mal, honra,
generosidade, etc. Não preciso de um espírito, no sentido ectoplasmatico". E
esta dimensão espitirual, não sendo ectoplasmática, localiza-se em padrões
implicitos no cerébro, suponho? (não é uma pergunta capciosa, ao menos não
tanto, tb não acredito em ectoplasma e coisas do gênero).
Um abraço ao uni-dual homero-oráculo,
Esteban.



----- Original Message -----
From: Oraculo
To: ciencialist@yahoogrupos.com.br
Sent: Monday, February 14, 2005 3:41 PM
Subject: [ciencialist] P/ Esteban ( Zodiaco e Acupuntura..)


Olá Esteban

risos..:-) Vou tentar uma reposta mais curta, ok?..:-)

Primeiro, não há erro na avaliação que faz sobre a influencia de carater e
cultura na ciência, ou melhor, nos cientistas. Nem vou discordar..:-) Todos
concordam com isso, e eu mesmo já coloquei essa posição diversas vezes. Mas,
é exatamente por saber disso , que muitas das ferramentas e rigores do
métido surgiram. Por conhecer a enorme capacidade da mente humana de se
enganar, distorcer ou influenciar análises, que se tornou necessário que
diversas pessoas refaçam cada experimento, que muitos pesqusiadores cheguem
a mesma conclusão, que os efeitos e previsões decorrentes sejam coerentes e
sólidos, tudo no sentido de filtrar o problema.

E, claro, nunca é 100% de certeza..:-)

Mas, mesmo assim, mesmo sem 100%, é fácil perceber que é mais confiável e
eficaz usar conhecimentos produzidos dessa forma. No passado, existiam
diversos modos de explicar os fenomenos, milhares deles, e diferentes visões
de mundo, diferentes formas de agir em relação a estes fenomenos e
explicações. E todas tinham igual eficácia, quase nenhuma. Não importa se
tratava uma doença dessa ou daquela forma, como demonio maligno ou miasma
sobrenatural, como desequilibrio energético ou como punição divina, a taxa
de cura era a mesma que não tratar. Isso fica claro na avaliação da
espectativa média de vida, de menos de 30 anos (em alguns lugares, menos de
25 anos).

Hoje, com o uso do conhecimento cientifico sobre doenças, nossa capacidade
de curar é enorme e a espectativa média de vida mais de 80 anos nos paises
mais civilizados.

Posso aplicar essa análise para qualquer área do conhecimento humano,
qualquer fenomeno ou explicação de fenomeno já produzida pelo homem. Se a
Lua é uma cabaça pouco mais alta que o topo das montanhas ou um enorme
satelite rochoso a orbitar a Terra a milhares de kilometros, cada explicação
produz diferentes consequencias e podemos testar o conjunto de conhecimento
sobre nosso satelite.

A ciência não é tendenciosa, os homens (e cientistas são homens) são,
certamente. E sabem disso. Por isso o rigor atinge nào apenas a acupuntura,
mas qualquer alegação, mesmo as produzidas dentro de laboratorios de ponta e
por cientistas de renome. A fusão a frio é um excelente exemplo. Não foi
aceita simplesmenete porque nào cumpriu os requisitos do método: nenhum
outro laboratório conseguiu reproduzir seus resultados. E era ciência,
criada em laboratorio e com bases fisicas.

E não incomoda descobrir que muitos pesquisadores tinham uma visão
"espiritual" do mundo..:-) Seria tolo esperar que não tivessem, já que são
homens, frutos de sua época. Mesmo hoje, onde mais da metade dos cientistas
não professa religião ou crença, ainda se encontram quem tenha uma visão
espiritual. O que não significa que levem essa visão para dentro do
laboratorio, pelo menos na maioria das vezes..:-) Nenhum deles espera que um
fenomeno fisico seja alterado ou afetado por sua crença ou pela ação do
sobrenatural. Seus experimentos são construidos sem levar em conta essas
variáveis.

Espiritual nem sempre significa o mesmo para diferentes pessoas. Eu vejo a
dimensão espiritual humana, mas não como algo transcendente ou sobrenatural,
mas como uma caracteristica de nossa espécie, algo abstrato, subjetivo, que
nos dá a dimensão humana, a capacidade de criar conceitos como bem, mal,
honra, generosidade, etc. Não preciso de um espírito, no sentido
ectoplasmatico, para isso..:-)

Para você a astrologia é real, e um dia nós, pobres cientistas e céticos,
perceberemos o engano..:-) Ótimo, isso significa que tem uma crença na
astrologia. Evidentemente deve perceber que todas as outras crenças dizem a
mesma coisa, que um dia poderão provar que estavam corretas e teremos de
aceita-las..:-) O que nào parecem perceber é que será um prazer aceitar
qualquer dessas alegações, desde que venham acompanhadas das evidencais e do
rigor necessário, não algo a ser recebido com bengaladas..:-) Novas
descobertas, por mais espantosas que sejam, sempre serão o motivo principal
da ciência. Apenas o cuidado de embasar essas descobertas com dados e provas
é que tem de ser tomado, sob pena de voltarmos ao passado, com milhares de
diferentes explicações, todas incorretas e com a mesma ineficiencia de ação.

Esteban, coisas mais malucas que a astrologia são aceitas pela ciência,
simplesmente porque podem apresentar evidencias sólidas e sobreviver a
análise. Coisas como continentes que se movem e seres invisiveis que causam
doenças (sem falar em relatividade e quantica..:-). Não é birra, é cuidado,
extremo cuidado, que tem dado excelente resultado prático.

O aumento de cientistas religiosos, mesmo que fosse verdadeiro, em nada muda
a ciência. Ela tem sobrevivido a coisas piores que isso e seu rigor é
justamente para produzir dados e conclusões confiáveis, seja seu autor
muçulmano, catolico, budista, shintoista ou ateu. Não há uma ciência
católica (a igreja pensa que há, mas é um engano fácil de demonstrar..:-),
nem muçulmana. Há cientistas catolicos e muçulmanos. Embora estejam
diminuindo e não aumentando..:-)

Um abraço.

Homero

PS: Pode me chamar de Oraculo ou de Homero, não tenho preferencias..:-) E,
sinto muito, acho que o texto acabou enorme mesmo..:-)





----- Original Message -----
From: Esteban Moreno
To: ciencialist@yahoogrupos.com.br
Sent: Monday, February 14, 2005 12:45 PM
Subject: Re: [ciencialist] Zodiaco e Acupuntura..


Ola Homero, como prefere ser chamado, não vou me alongar nessa discussão
pós-férias. Apenas vou dar mais um pitaco num trecho que merece algum
comentário:

"Assim, acusar os que pesqusiasm com ironias do tipo "santos céticos
imparcialistas" é bobagem e uma falacia "ad hominem".

Foi por certo colocado de uma forma bastante irônica, como é comum a quase
todos deste fórum, mas não é tão falacioso como supõe. A ciência é
parcial,
a idéia de objetividade na ciência mudou drasticamente nos últimos tempos,
e
diversos estudos corroboram neste sentido. Talvez mais uma vez lhe pareça
inverossímil, mas vários pesquisadores tentam compreender como a ciência
teria se desenvolvido sem a discriminação histórica das mulheres, por
exemplo, ou mesmo como o formato da organização do laboratório interfere
nos
resultados colhidos em experimentos (sugiro buscar os trabalhos de BRUNO
LATOUR ou pesquisar no google sobre CULTURAL STUDIES SCIENCE). Concordo
que
há uma tendência a um nivelamento ao longo do tempo, mas isso não torna a
ciência imune a uma série de erros históricos, ainda mais em um pleno
estado
transiente sob o qual nos encontramos. A astrologia para mim é um desses
erros, o seus netos um dia te dirão, sob os receios de uma forte
"bengalada"
do vovô ;-). De forma semelhante tem ocorrido com a acupuntura, como
queira
compreender ou não a acupuntura. Pois concordo que a acupuntura cujo
artigo
devas estar se referenciando não é a acupuntura aprovada como
especialidade
médica em diversos paises do mundo e no próprio Brasil. Esta que me
referencio é essencialmente a mesma acupuntura do velhinho de mais de 3000
anos atrás.

Não quero dizer que a ciência seja imprópria, apenas tendenciosa. Como a
maioria de vocês, tb cultuo a crença (ops, desculpe, a visão) do Einstein
de
que pouco sabemos do universo e a ciência é o que melhor temos. Que coisa
mais chata deve ser para os céticos perceber que a maioria das mais
importantes descobertas da ciência até os tempos mais recentes foram
feitas
por pessoas que possuíam alguma visão "espiritualizada" do universo. Que
desagradável deve ser descobrir que a quantidade de cientistas religiosos
está aumentando no mundo (esta estatística está no almanaque abril).
Chacoalhem-se logo, criem algo original, vocês tem que manter a pálida
vela
do falecido Carl acesa. ;-)

No aguardo de mais uma looooooooonga resposta.
Um abraço,
Esteban.





##### ##### #####

Para saber mais visite
http://www.ciencialist.hpg.ig.com.br


##### ##### ##### #####


Yahoo! Grupos, um serviço oferecido por:
PUBLICIDADE




----------------------------------------------------------------------------
--
Links do Yahoo! Grupos

a.. Para visitar o site do seu grupo na web, acesse:
http://br.groups.yahoo.com/group/ciencialist/

b.. Para sair deste grupo, envie um e-mail para:
ciencialist-unsubscribe@yahoogrupos.com.br

c.. O uso que você faz do Yahoo! Grupos está sujeito aos Termos do
Serviço do Yahoo!.



[As partes desta mensagem que não continham texto foram removidas]



##### ##### #####

Para saber mais visite
http://www.ciencialist.hpg.ig.com.br


##### ##### ##### #####


Yahoo! Grupos, um serviço oferecido por:
PUBLICIDADE





Links do Yahoo! Grupos

Para visitar o site do seu grupo na web, acesse:
http://br.groups.yahoo.com/group/ciencialist/

Para sair deste grupo, envie um e-mail para:
ciencialist-unsubscribe@yahoogrupos.com.br

O uso que você faz do Yahoo! Grupos está sujeito aos Termos do Serviço do
Yahoo!.



SUBJECT: Re: [ciencialist] P/ Esteban ( Zodiaco e Acupuntura..)
FROM: "Oraculo" <oraculo@atibaia.com.br>
TO: <ciencialist@yahoogrupos.com.br>
DATE: 14/02/2005 18:40

Olá Esteban

Bem, tenho dificuldades de escrever pouco..:-) Isso se deve ao fato que assuntos complexos demenadam explicações compelxas..:-)

Agora quanto a estudos que comprovem que a astrologia nào funciona. O correto é o contrário, estudos que comprovem que ela funciona. É isso que se pede a qualquer alegação, nào o contrário. Não se faz uma alegação e se espera que seja refutada. O onus da prova é de quem afirma, ou só fariamos isso, refutar alegaçòes sem embasamento.

A astrologia, como concordam até mesmo os astrologs (e o Taborda), nào tem emcanismo de ação conhecido. Ou seja, nào se sabe por que mecanismo a posição de astros infleunciariam a personaldiade das pessoas ou eventos do mundo fisico. Assim, se eu criar um sistema de calculos, bem precisos, e decidir que estes permitem que eu saiba o futuro ou mesmo como é formado o soslo do planeta Urano, este "novo conehciemnto" terá a mesma base de ação que a astrologia e nào será possível determianr se estou dizendo a verdade ou nào.

Mesmo assim, algo pdoe nào ter mecanismo de ação conhecido e ainda funcioanr e ser real. Para determianr isso, temos de preparar estudos que mostrem sua eficácia, mesmoque nào indiquem seu mecanismo de ação.

A astrologia também nào tem isso, estudos que determinem sua real eficácia.

Por isso nào é aceita, não por birra, como parece pensar.

Estudos, os que foram feitos com rigor e controle, nada mostraram, estando sua eficácia dentro da chance estatistica de acertar ou errar algo. E isso pode ser modificado a qualqeur hora, basta que astrologos criem e apresentem estudos de eficácia dentro do rigor do método. Sem isso, repito, suas alegações são as mesmas de qualquer outro tipo de alegação sobrenatural..:-) Afinal, todas as outras crenças dizem a mesma coisa, que, mesmo sem ter mecanismo de ação conehcido (como a homeopatia e a acupuntura), elas funcionam, só nào conseguem passar em experimentos controlados com rigor.

Toda afirmação requer evidencais. Não requer evidencias contrárias não acreditar em alegações sem evidencias..:-) Isso é uma falácia lógica, não faz sentido. A astrologia é que deve apresentar estudos de aficácia, nào a ciência estudos de nào eficácia (embora ela possa apresentar alguns, foram feitos por quem realmetne acerditava que teria resultado positivo).

Pense, eu digo que seres intraterenos, que vivem no interior da Terra, e que vieram de Alpha Centauro, me deram a chave do conehcimento cosmico, sobre tudo, mas com o compromisso de nào contar apra mais ninguém. Eu sei tudo o que vai acontecer no futuro, só nào posso contar. Para provar que eu estou certo, que minha alegação é real, eu digo a você que "a ciência nào tem qualquer trabalho verossímel que mostre que eu estou errado e que nào existem extraterrestres em alpha centauri" , portanto, você deve acreditar em mim e tomar minha alegação como real.

Vê o problema com sua posição?..:-) Crer na astrologia simplesmente porque "nào há qualquer trabalho verossimel que prove que ela nào funciona", mesmo que fosse verdade, nào seria lógico, racional..:-) Tem de concluir que a astrologia é real a partir de estudos positivos, de eficácia, nào o contrário..:-)

"Esteban: E esta dimensão espitirual, não sendo ectoplasmática, localiza-se em padrões
implicitos no cerébro, suponho? (não é uma pergunta capciosa, ao menos não
tanto, tb não acredito em ectoplasma e coisas do gênero)."

Sim, tanto quanto sabemos no momento, seria um processo neurológico, um padrão específico de circuitos neuronais, ligações sinapticas e reações. Como qualquer outro procedimento cerebral, seja uma conta de somar, seja um raciocinio abstrato..:-) Caracteristicas derivadas da evolução, processos que aumentaram a chance de sobreviver de nossos antepassados, mas que hoje podem ser exoadaptadas para outros usos, outras formas de pensar, como qualquer mecanismo biologico, fisico ou subjetivo.

Exoadaptação é um conceito poderoso e muito interessante. A natureza toma o que está a mão, seja um osso, seja um processo mental. No urso panda, a pressão seletiva precisava de um poleghar, devido a mudança aliemntar e de ambiente dos seus antepassados. Mas os dedos dos ursos já estavam foramdos, e nào permitiam uma volta ao polegar opositor. Assim, um osso do punho, que todos temos, inclusive ursos, se modificou, pelas eras, a onto de permitir que o panda apresente um facsimile de polegar, um sexto dedo, derivado desse osso. Interessante notar que, como o gene que permite esse crescimento é uno para todo o osso, ele cresceu nào apeans na direção correta, que permite a pegada, mas também na direção contrária, uma inutilidade inevitável, mas necessária ao preocesso..:-)

O mesmo para mecanismos mentais, como o cuidado com a prole, que se tornou o atual amor parental dos seres humanos (e primatas diversos). Ou a solidariedade, mecanismo que garante a animais gregários enorme vantagem competitiva, permitindo que o grupo seja mais forte que os individuos (como lobos, chimpanzés e homo sapiens).

Se desejar mais informação sobre estes processos, os livros de Dawkins, em especial a Escalada do Monte Improvável e O Rio que Saia do Éden e o fabuloso Tabula Rasa de pinker são ótima leitura. E o livro de Gould, Os Dentes da Galinha pode ser encontrado nos arquivos da lista, digitalizado, e pode ler em seu computador, é divertido e claro.

Um abraço.

Homero









----- Original Message -----
From: Esteban Moreno
To: ciencialist@yahoogrupos.com.br
Sent: Monday, February 14, 2005 5:04 PM
Subject: Re: [ciencialist] P/ Esteban ( Zodiaco e Acupuntura..)


Bom, veio looooonga mesmo ;-)
Quanto ao trecho: "...isso significa que tem uma crença na astrologia.
Evidentemente deve perceber que todas as outras crenças dizem a mesma
coisa". São duas frases com duas fortes arbitrariedades, colocadas de forma
tendenciosa aos seus próprios propósitos, como quase todo o resto. A
principal exceção deve-se: "A ciência não é tendenciosa, os homens (e
cientistas são homens) são.." E de fato, expressei-me mal, você está
correto, só faltou substituir homens por seres humanos. Faz parte de nossa
linguagem preconceituosa.

Além do mais e principalmente: "não há qualquer trabalho verossímil que
prove o não funcionamento da astrologia". Vocês se apóiam em vazios, pura
especulação preconceituosa e mal formulada. Por quantas e quantas vezes
pendurarão em vossas telas artigos supostamente científicos que tentam
provar falhas da astrologia com argumentos das quais a astrologia não se
propõe ou, quando muito, com erros básicos, como basear-se apenas em um
planeta ou uma posição no mapa. Eu realmente não tenho a paciência do
Taborda ou do Mesquita para responder-te ao resto. Mesmo porque, diga-me, a
esta altura, há algum fruto nessa discussão?

Mas ficou uma curiosidade, quando afirmas: "Eu vejo a dimensão espiritual
humana, mas não como algo transcendente ou sobrenatural, mas como uma
característica de nossa espécie, algo abstrato, subjetivo, que nos dá a
dimensão humana, a capacidade de criar conceitos como bem, mal, honra,
generosidade, etc. Não preciso de um espírito, no sentido ectoplasmatico". E
esta dimensão espitirual, não sendo ectoplasmática, localiza-se em padrões
implicitos no cerébro, suponho? (não é uma pergunta capciosa, ao menos não
tanto, tb não acredito em ectoplasma e coisas do gênero).
Um abraço ao uni-dual homero-oráculo,
Esteban.



----- Original Message -----
From: Oraculo
To: ciencialist@yahoogrupos.com.br
Sent: Monday, February 14, 2005 3:41 PM
Subject: [ciencialist] P/ Esteban ( Zodiaco e Acupuntura..)


Olá Esteban

risos..:-) Vou tentar uma reposta mais curta, ok?..:-)

Primeiro, não há erro na avaliação que faz sobre a influencia de carater e
cultura na ciência, ou melhor, nos cientistas. Nem vou discordar..:-) Todos
concordam com isso, e eu mesmo já coloquei essa posição diversas vezes. Mas,
é exatamente por saber disso , que muitas das ferramentas e rigores do
métido surgiram. Por conhecer a enorme capacidade da mente humana de se
enganar, distorcer ou influenciar análises, que se tornou necessário que
diversas pessoas refaçam cada experimento, que muitos pesqusiadores cheguem
a mesma conclusão, que os efeitos e previsões decorrentes sejam coerentes e
sólidos, tudo no sentido de filtrar o problema.

E, claro, nunca é 100% de certeza..:-)

Mas, mesmo assim, mesmo sem 100%, é fácil perceber que é mais confiável e
eficaz usar conhecimentos produzidos dessa forma. No passado, existiam
diversos modos de explicar os fenomenos, milhares deles, e diferentes visões
de mundo, diferentes formas de agir em relação a estes fenomenos e
explicações. E todas tinham igual eficácia, quase nenhuma. Não importa se
tratava uma doença dessa ou daquela forma, como demonio maligno ou miasma
sobrenatural, como desequilibrio energético ou como punição divina, a taxa
de cura era a mesma que não tratar. Isso fica claro na avaliação da
espectativa média de vida, de menos de 30 anos (em alguns lugares, menos de
25 anos).

Hoje, com o uso do conhecimento cientifico sobre doenças, nossa capacidade
de curar é enorme e a espectativa média de vida mais de 80 anos nos paises
mais civilizados.

Posso aplicar essa análise para qualquer área do conhecimento humano,
qualquer fenomeno ou explicação de fenomeno já produzida pelo homem. Se a
Lua é uma cabaça pouco mais alta que o topo das montanhas ou um enorme
satelite rochoso a orbitar a Terra a milhares de kilometros, cada explicação
produz diferentes consequencias e podemos testar o conjunto de conhecimento
sobre nosso satelite.

A ciência não é tendenciosa, os homens (e cientistas são homens) são,
certamente. E sabem disso. Por isso o rigor atinge nào apenas a acupuntura,
mas qualquer alegação, mesmo as produzidas dentro de laboratorios de ponta e
por cientistas de renome. A fusão a frio é um excelente exemplo. Não foi
aceita simplesmenete porque nào cumpriu os requisitos do método: nenhum
outro laboratório conseguiu reproduzir seus resultados. E era ciência,
criada em laboratorio e com bases fisicas.

E não incomoda descobrir que muitos pesquisadores tinham uma visão
"espiritual" do mundo..:-) Seria tolo esperar que não tivessem, já que são
homens, frutos de sua época. Mesmo hoje, onde mais da metade dos cientistas
não professa religião ou crença, ainda se encontram quem tenha uma visão
espiritual. O que não significa que levem essa visão para dentro do
laboratorio, pelo menos na maioria das vezes..:-) Nenhum deles espera que um
fenomeno fisico seja alterado ou afetado por sua crença ou pela ação do
sobrenatural. Seus experimentos são construidos sem levar em conta essas
variáveis.

Espiritual nem sempre significa o mesmo para diferentes pessoas. Eu vejo a
dimensão espiritual humana, mas não como algo transcendente ou sobrenatural,
mas como uma caracteristica de nossa espécie, algo abstrato, subjetivo, que
nos dá a dimensão humana, a capacidade de criar conceitos como bem, mal,
honra, generosidade, etc. Não preciso de um espírito, no sentido
ectoplasmatico, para isso..:-)

Para você a astrologia é real, e um dia nós, pobres cientistas e céticos,
perceberemos o engano..:-) Ótimo, isso significa que tem uma crença na
astrologia. Evidentemente deve perceber que todas as outras crenças dizem a
mesma coisa, que um dia poderão provar que estavam corretas e teremos de
aceita-las..:-) O que nào parecem perceber é que será um prazer aceitar
qualquer dessas alegações, desde que venham acompanhadas das evidencais e do
rigor necessário, não algo a ser recebido com bengaladas..:-) Novas
descobertas, por mais espantosas que sejam, sempre serão o motivo principal
da ciência. Apenas o cuidado de embasar essas descobertas com dados e provas
é que tem de ser tomado, sob pena de voltarmos ao passado, com milhares de
diferentes explicações, todas incorretas e com a mesma ineficiencia de ação.

Esteban, coisas mais malucas que a astrologia são aceitas pela ciência,
simplesmente porque podem apresentar evidencias sólidas e sobreviver a
análise. Coisas como continentes que se movem e seres invisiveis que causam
doenças (sem falar em relatividade e quantica..:-). Não é birra, é cuidado,
extremo cuidado, que tem dado excelente resultado prático.

O aumento de cientistas religiosos, mesmo que fosse verdadeiro, em nada muda
a ciência. Ela tem sobrevivido a coisas piores que isso e seu rigor é
justamente para produzir dados e conclusões confiáveis, seja seu autor
muçulmano, catolico, budista, shintoista ou ateu. Não há uma ciência
católica (a igreja pensa que há, mas é um engano fácil de demonstrar..:-),
nem muçulmana. Há cientistas catolicos e muçulmanos. Embora estejam
diminuindo e não aumentando..:-)

Um abraço.

Homero

PS: Pode me chamar de Oraculo ou de Homero, não tenho preferencias..:-) E,
sinto muito, acho que o texto acabou enorme mesmo..:-)





----- Original Message -----
From: Esteban Moreno
To: ciencialist@yahoogrupos.com.br
Sent: Monday, February 14, 2005 12:45 PM
Subject: Re: [ciencialist] Zodiaco e Acupuntura..


Ola Homero, como prefere ser chamado, não vou me alongar nessa discussão
pós-férias. Apenas vou dar mais um pitaco num trecho que merece algum
comentário:

"Assim, acusar os que pesqusiasm com ironias do tipo "santos céticos
imparcialistas" é bobagem e uma falacia "ad hominem".

Foi por certo colocado de uma forma bastante irônica, como é comum a quase
todos deste fórum, mas não é tão falacioso como supõe. A ciência é
parcial,
a idéia de objetividade na ciência mudou drasticamente nos últimos tempos,
e
diversos estudos corroboram neste sentido. Talvez mais uma vez lhe pareça
inverossímil, mas vários pesquisadores tentam compreender como a ciência
teria se desenvolvido sem a discriminação histórica das mulheres, por
exemplo, ou mesmo como o formato da organização do laboratório interfere
nos
resultados colhidos em experimentos (sugiro buscar os trabalhos de BRUNO
LATOUR ou pesquisar no google sobre CULTURAL STUDIES SCIENCE). Concordo
que
há uma tendência a um nivelamento ao longo do tempo, mas isso não torna a
ciência imune a uma série de erros históricos, ainda mais em um pleno
estado
transiente sob o qual nos encontramos. A astrologia para mim é um desses
erros, o seus netos um dia te dirão, sob os receios de uma forte
"bengalada"
do vovô ;-). De forma semelhante tem ocorrido com a acupuntura, como
queira
compreender ou não a acupuntura. Pois concordo que a acupuntura cujo
artigo
devas estar se referenciando não é a acupuntura aprovada como
especialidade
médica em diversos paises do mundo e no próprio Brasil. Esta que me
referencio é essencialmente a mesma acupuntura do velhinho de mais de 3000
anos atrás.

Não quero dizer que a ciência seja imprópria, apenas tendenciosa. Como a
maioria de vocês, tb cultuo a crença (ops, desculpe, a visão) do Einstein
de
que pouco sabemos do universo e a ciência é o que melhor temos. Que coisa
mais chata deve ser para os céticos perceber que a maioria das mais
importantes descobertas da ciência até os tempos mais recentes foram
feitas
por pessoas que possuíam alguma visão "espiritualizada" do universo. Que
desagradável deve ser descobrir que a quantidade de cientistas religiosos
está aumentando no mundo (esta estatística está no almanaque abril).
Chacoalhem-se logo, criem algo original, vocês tem que manter a pálida
vela
do falecido Carl acesa. ;-)

No aguardo de mais uma looooooooonga resposta.
Um abraço,
Esteban.





##### ##### #####

Para saber mais visite
http://www.ciencialist.hpg.ig.com.br


##### ##### ##### #####


Yahoo! Grupos, um serviço oferecido por:
PUBLICIDADE




----------------------------------------------------------------------------
--
Links do Yahoo! Grupos

a.. Para visitar o site do seu grupo na web, acesse:
http://br.groups.yahoo.com/group/ciencialist/

b.. Para sair deste grupo, envie um e-mail para:
ciencialist-unsubscribe@yahoogrupos.com.br

c.. O uso que você faz do Yahoo! Grupos está sujeito aos Termos do
Serviço do Yahoo!.



[As partes desta mensagem que não continham texto foram removidas]



##### ##### #####

Para saber mais visite
http://www.ciencialist.hpg.ig.com.br


##### ##### ##### #####


Yahoo! Grupos, um serviço oferecido por:
PUBLICIDADE





Links do Yahoo! Grupos

Para visitar o site do seu grupo na web, acesse:
http://br.groups.yahoo.com/group/ciencialist/

Para sair deste grupo, envie um e-mail para:
ciencialist-unsubscribe@yahoogrupos.com.br

O uso que você faz do Yahoo! Grupos está sujeito aos Termos do Serviço do
Yahoo!.



##### ##### #####

Para saber mais visite
http://www.ciencialist.hpg.ig.com.br


##### ##### ##### #####


Yahoo! Grupos, um serviço oferecido por:







------------------------------------------------------------------------------
Links do Yahoo! Grupos

a.. Para visitar o site do seu grupo na web, acesse:
http://br.groups.yahoo.com/group/ciencialist/

b.. Para sair deste grupo, envie um e-mail para:
ciencialist-unsubscribe@yahoogrupos.com.br

c.. O uso que você faz do Yahoo! Grupos está sujeito aos Termos do Serviço do Yahoo!.



[As partes desta mensagem que não continham texto foram removidas]



SUBJECT: Corrigindo o texto anterior- era P/ Esteban ( Zodiaco e Acupuntura..)
FROM: "Oraculo" <oraculo@atibaia.com.br>
TO: <ciencialist@yahoogrupos.com.br>
DATE: 14/02/2005 18:50

O texto saiu tão truncado, que achei melhor mandar um corrigido..:-) É a mesma mensagem anterior, mas melhor escrita..:-)
_____________________________________________________________

Olá Esteban

Bem, tenho dificuldades de escrever pouco..:-) Isso se deve ao fato que assuntos complexos demandam explicações complexas..:-)

Agora quanto a estudos que comprovem que a astrologia não funciona. O correto é o contrário, estudos que comprovem que ela funciona. É isso que se pede a qualquer alegação, nào o contrário. Não se faz uma alegação e se espera que seja refutada. O onus da prova é de quem afirma, ou só fariamos isso, refutar alegações sem embasamento.

A astrologia, como concordam até mesmo os astrologos (e o Taborda), não tem mecanismo de ação conhecido. Ou seja, não se sabe por que mecanismo a posição de astros influenciariam a personaldiade das pessoas ou eventos do mundo fisico. Assim, se eu criar um sistema de cálculos, bem precisos, e decidir que estes permitem que eu saiba o futuro ou mesmo como é formado o solo do planeta Urano, este "novo conehcimento" terá a mesma base de ação que a astrologia e não será possível determinar se estou dizendo a verdade ou não.

Mesmo assim, algo pode não ter mecanismo de ação conhecido e ainda funcionar e ser real. Para determinar isso, temos de preparar estudos que mostrem sua eficácia, mesmo que não indiquem seu mecanismo de ação.

A astrologia também não tem isso, estudos que determinem sua real eficácia.

Por isso não é aceita, não por birra, como parece pensar.

Estudos, os que foram feitos com rigor e controle, nada mostraram, estando sua eficácia dentro da chance estatistica de acertar ou errar algo. E isso pode ser modificado a qualquer hora, basta que astrólogos criem e apresentem estudos de eficácia dentro do rigor do método. Sem isso, repito, suas alegações são as mesmas de qualquer outro tipo de alegação sobrenatural..:-) Afinal, todas as outras crenças dizem a mesma coisa, que, mesmo sem ter mecanismo de ação conhecido (como a homeopatia e a acupuntura), elas funcionam, só não conseguem passar em experimentos controlados com rigor.

Toda afirmação requer evidencias. Não requer evidencias contrárias não acreditar em alegações sem evidencias..:-) Isso é uma falácia lógica, não faz sentido. A astrologia é que deve apresentar estudos de eficácia, não a ciência estudos de "não eficácia" (embora ela possa apresentar alguns, foram feitos por quem realmente acerditava que teria resultado positivo).

Pense, eu digo que seres intraterrenos, que vivem no interior da Terra, e que vieram de Alpha Centauro, me deram a chave do conhecimento cósmico, sobre todas as coisas, mas com o compromisso de não contar para mais ninguém, o que se deve fazer? Eu sei tudo o que vai acontecer no futuro, só não posso contar. Para provar que eu estou certo, que minha alegação é real, eu digo a você que "a ciência não tem qualquer trabalho verossímel que mostre que eu estou errado e que não existem extraterrestres em Alpha Centauri" , portanto, você deve acreditar em mim e tomar minha alegação como real.

Vê o problema com sua posição?..:-) Crer na astrologia simplesmente porque "não há qualquer trabalho verossimel que prove que ela não funciona", mesmo que fosse verdade, não seria lógico, racional..:-) Tem de concluir que a astrologia é real a partir de estudos positivos, de eficácia, não o contrário..:-)

"Esteban: E esta dimensão espitirual, não sendo ectoplasmática, localiza-se em padrões
implicitos no cerébro, suponho? (não é uma pergunta capciosa, ao menos não
tanto, tb não acredito em ectoplasma e coisas do gênero)."

Sim, tanto quanto sabemos no momento, seria um processo neurológico, um padrão específico de circuitos neuronais, ligações sinápticas e reações. Como qualquer outro procedimento cerebral, seja uma conta de somar, seja um raciocinio abstrato..:-) Caracteristicas derivadas da evolução, processos que aumentaram a chance de sobreviver de nossos antepassados, mas que hoje podem ser exo-adaptadas para outros usos, outras formas de pensar, como qualquer mecanismo biologico, fisico ou subjetivo.

Exo-adaptação é um conceito poderoso e muito interessante. A natureza toma o que está a mão, seja um osso, seja um processo mental. No urso panda, a pressão seletiva precisava de um polegar, devido a mudança aliemntar e de ambiente dos seus antepassados. Mas os dedos dos ursos já estavam formados, e não permitiam uma volta ao polegar opositor. Assim, um osso do punho, que todos temos, inclusive ursos, se modificou, pelas eras, a ponto de permitir que o panda apresente um fac-simile de polegar, um sexto dedo, derivado desse osso. Interessante notar que, como o gene que permite esse crescimento é uno para todo o osso, ele cresceu não apenas na direção correta, que permite a pegada, mas também na direção contrária, uma inutilidade inevitável, mas necessária ao processo..:-)

O mesmo para mecanismos mentais, como o cuidado com a prole, que se tornou o atual amor parental dos seres humanos (e primatas diversos). Ou a solidariedade, mecanismo que garante a animais gregários enorme vantagem competitiva, permitindo que o grupo seja mais forte que os individuos (como lobos, chimpanzés e homo sapiens).

Se desejar mais informação sobre estes processos, os livros de Dawkins, em especial a Escalada do Monte Improvável e O Rio que Saia do Éden e o fabuloso Tabula Rasa de pinker são ótima leitura. E o livro de Gould, Os Dentes da Galinha pode ser encontrado nos arquivos da lista, digitalizado, e pode ler em seu computador, é divertido e claro.

Um abraço.

Homero








----- Original Message -----
From: Esteban Moreno
To: ciencialist@yahoogrupos.com.br
Sent: Monday, February 14, 2005 5:04 PM
Subject: Re: [ciencialist] P/ Esteban ( Zodiaco e Acupuntura..)


Bom, veio looooonga mesmo ;-)
Quanto ao trecho: "...isso significa que tem uma crença na astrologia.
Evidentemente deve perceber que todas as outras crenças dizem a mesma
coisa". São duas frases com duas fortes arbitrariedades, colocadas de forma
tendenciosa aos seus próprios propósitos, como quase todo o resto. A
principal exceção deve-se: "A ciência não é tendenciosa, os homens (e
cientistas são homens) são.." E de fato, expressei-me mal, você está
correto, só faltou substituir homens por seres humanos. Faz parte de nossa
linguagem preconceituosa.

Além do mais e principalmente: "não há qualquer trabalho verossímil que
prove o não funcionamento da astrologia". Vocês se apóiam em vazios, pura
especulação preconceituosa e mal formulada. Por quantas e quantas vezes
pendurarão em vossas telas artigos supostamente científicos que tentam
provar falhas da astrologia com argumentos das quais a astrologia não se
propõe ou, quando muito, com erros básicos, como basear-se apenas em um
planeta ou uma posição no mapa. Eu realmente não tenho a paciência do
Taborda ou do Mesquita para responder-te ao resto. Mesmo porque, diga-me, a
esta altura, há algum fruto nessa discussão?

Mas ficou uma curiosidade, quando afirmas: "Eu vejo a dimensão espiritual
humana, mas não como algo transcendente ou sobrenatural, mas como uma
característica de nossa espécie, algo abstrato, subjetivo, que nos dá a
dimensão humana, a capacidade de criar conceitos como bem, mal, honra,
generosidade, etc. Não preciso de um espírito, no sentido ectoplasmatico". E
esta dimensão espitirual, não sendo ectoplasmática, localiza-se em padrões
implicitos no cerébro, suponho? (não é uma pergunta capciosa, ao menos não
tanto, tb não acredito em ectoplasma e coisas do gênero).
Um abraço ao uni-dual homero-oráculo,
Esteban.



----- Original Message -----
From: Oraculo
To: ciencialist@yahoogrupos.com.br
Sent: Monday, February 14, 2005 3:41 PM
Subject: [ciencialist] P/ Esteban ( Zodiaco e Acupuntura..)


Olá Esteban

risos..:-) Vou tentar uma reposta mais curta, ok?..:-)

Primeiro, não há erro na avaliação que faz sobre a influencia de carater e
cultura na ciência, ou melhor, nos cientistas. Nem vou discordar..:-) Todos
concordam com isso, e eu mesmo já coloquei essa posição diversas vezes. Mas,
é exatamente por saber disso , que muitas das ferramentas e rigores do
métido surgiram. Por conhecer a enorme capacidade da mente humana de se
enganar, distorcer ou influenciar análises, que se tornou necessário que
diversas pessoas refaçam cada experimento, que muitos pesqusiadores cheguem
a mesma conclusão, que os efeitos e previsões decorrentes sejam coerentes e
sólidos, tudo no sentido de filtrar o problema.

E, claro, nunca é 100% de certeza..:-)

Mas, mesmo assim, mesmo sem 100%, é fácil perceber que é mais confiável e
eficaz usar conhecimentos produzidos dessa forma. No passado, existiam
diversos modos de explicar os fenomenos, milhares deles, e diferentes visões
de mundo, diferentes formas de agir em relação a estes fenomenos e
explicações. E todas tinham igual eficácia, quase nenhuma. Não importa se
tratava uma doença dessa ou daquela forma, como demonio maligno ou miasma
sobrenatural, como desequilibrio energético ou como punição divina, a taxa
de cura era a mesma que não tratar. Isso fica claro na avaliação da
espectativa média de vida, de menos de 30 anos (em alguns lugares, menos de
25 anos).

Hoje, com o uso do conhecimento cientifico sobre doenças, nossa capacidade
de curar é enorme e a espectativa média de vida mais de 80 anos nos paises
mais civilizados.

Posso aplicar essa análise para qualquer área do conhecimento humano,
qualquer fenomeno ou explicação de fenomeno já produzida pelo homem. Se a
Lua é uma cabaça pouco mais alta que o topo das montanhas ou um enorme
satelite rochoso a orbitar a Terra a milhares de kilometros, cada explicação
produz diferentes consequencias e podemos testar o conjunto de conhecimento
sobre nosso satelite.

A ciência não é tendenciosa, os homens (e cientistas são homens) são,
certamente. E sabem disso. Por isso o rigor atinge nào apenas a acupuntura,
mas qualquer alegação, mesmo as produzidas dentro de laboratorios de ponta e
por cientistas de renome. A fusão a frio é um excelente exemplo. Não foi
aceita simplesmenete porque nào cumpriu os requisitos do método: nenhum
outro laboratório conseguiu reproduzir seus resultados. E era ciência,
criada em laboratorio e com bases fisicas.

E não incomoda descobrir que muitos pesquisadores tinham uma visão
"espiritual" do mundo..:-) Seria tolo esperar que não tivessem, já que são
homens, frutos de sua época. Mesmo hoje, onde mais da metade dos cientistas
não professa religião ou crença, ainda se encontram quem tenha uma visão
espiritual. O que não significa que levem essa visão para dentro do
laboratorio, pelo menos na maioria das vezes..:-) Nenhum deles espera que um
fenomeno fisico seja alterado ou afetado por sua crença ou pela ação do
sobrenatural. Seus experimentos são construidos sem levar em conta essas
variáveis.

Espiritual nem sempre significa o mesmo para diferentes pessoas. Eu vejo a
dimensão espiritual humana, mas não como algo transcendente ou sobrenatural,
mas como uma caracteristica de nossa espécie, algo abstrato, subjetivo, que
nos dá a dimensão humana, a capacidade de criar conceitos como bem, mal,
honra, generosidade, etc. Não preciso de um espírito, no sentido
ectoplasmatico, para isso..:-)

Para você a astrologia é real, e um dia nós, pobres cientistas e céticos,
perceberemos o engano..:-) Ótimo, isso significa que tem uma crença na
astrologia. Evidentemente deve perceber que todas as outras crenças dizem a
mesma coisa, que um dia poderão provar que estavam corretas e teremos de
aceita-las..:-) O que nào parecem perceber é que será um prazer aceitar
qualquer dessas alegações, desde que venham acompanhadas das evidencais e do
rigor necessário, não algo a ser recebido com bengaladas..:-) Novas
descobertas, por mais espantosas que sejam, sempre serão o motivo principal
da ciência. Apenas o cuidado de embasar essas descobertas com dados e provas
é que tem de ser tomado, sob pena de voltarmos ao passado, com milhares de
diferentes explicações, todas incorretas e com a mesma ineficiencia de ação.

Esteban, coisas mais malucas que a astrologia são aceitas pela ciência,
simplesmente porque podem apresentar evidencias sólidas e sobreviver a
análise. Coisas como continentes que se movem e seres invisiveis que causam
doenças (sem falar em relatividade e quantica..:-). Não é birra, é cuidado,
extremo cuidado, que tem dado excelente resultado prático.

O aumento de cientistas religiosos, mesmo que fosse verdadeiro, em nada muda
a ciência. Ela tem sobrevivido a coisas piores que isso e seu rigor é
justamente para produzir dados e conclusões confiáveis, seja seu autor
muçulmano, catolico, budista, shintoista ou ateu. Não há uma ciência
católica (a igreja pensa que há, mas é um engano fácil de demonstrar..:-),
nem muçulmana. Há cientistas catolicos e muçulmanos. Embora estejam
diminuindo e não aumentando..:-)

Um abraço.

Homero

PS: Pode me chamar de Oraculo ou de Homero, não tenho preferencias..:-) E,
sinto muito, acho que o texto acabou enorme mesmo..:-)





----- Original Message -----
From: Esteban Moreno
To: ciencialist@yahoogrupos.com.br
Sent: Monday, February 14, 2005 12:45 PM
Subject: Re: [ciencialist] Zodiaco e Acupuntura..


Ola Homero, como prefere ser chamado, não vou me alongar nessa discussão
pós-férias. Apenas vou dar mais um pitaco num trecho que merece algum
comentário:

"Assim, acusar os que pesqusiasm com ironias do tipo "santos céticos
imparcialistas" é bobagem e uma falacia "ad hominem".

Foi por certo colocado de uma forma bastante irônica, como é comum a quase
todos deste fórum, mas não é tão falacioso como supõe. A ciência é
parcial,
a idéia de objetividade na ciência mudou drasticamente nos últimos tempos,
e
diversos estudos corroboram neste sentido. Talvez mais uma vez lhe pareça
inverossímil, mas vários pesquisadores tentam compreender como a ciência
teria se desenvolvido sem a discriminação histórica das mulheres, por
exemplo, ou mesmo como o formato da organização do laboratório interfere
nos
resultados colhidos em experimentos (sugiro buscar os trabalhos de BRUNO
LATOUR ou pesquisar no google sobre CULTURAL STUDIES SCIENCE). Concordo
que
há uma tendência a um nivelamento ao longo do tempo, mas isso não torna a
ciência imune a uma série de erros históricos, ainda mais em um pleno
estado
transiente sob o qual nos encontramos. A astrologia para mim é um desses
erros, o seus netos um dia te dirão, sob os receios de uma forte
"bengalada"
do vovô ;-). De forma semelhante tem ocorrido com a acupuntura, como
queira
compreender ou não a acupuntura. Pois concordo que a acupuntura cujo
artigo
devas estar se referenciando não é a acupuntura aprovada como
especialidade
médica em diversos paises do mundo e no próprio Brasil. Esta que me
referencio é essencialmente a mesma acupuntura do velhinho de mais de 3000
anos atrás.

Não quero dizer que a ciência seja imprópria, apenas tendenciosa. Como a
maioria de vocês, tb cultuo a crença (ops, desculpe, a visão) do Einstein
de
que pouco sabemos do universo e a ciência é o que melhor temos. Que coisa
mais chata deve ser para os céticos perceber que a maioria das mais
importantes descobertas da ciência até os tempos mais recentes foram
feitas
por pessoas que possuíam alguma visão "espiritualizada" do universo. Que
desagradável deve ser descobrir que a quantidade de cientistas religiosos
está aumentando no mundo (esta estatística está no almanaque abril).
Chacoalhem-se logo, criem algo original, vocês tem que manter a pálida
vela
do falecido Carl acesa. ;-)

No aguardo de mais uma looooooooonga resposta.
Um abraço,
Esteban.





##### ##### #####

Para saber mais visite
http://www.ciencialist.hpg.ig.com.br


##### ##### ##### #####


Yahoo! Grupos, um serviço oferecido por:
PUBLICIDADE




----------------------------------------------------------------------------
--
Links do Yahoo! Grupos

a.. Para visitar o site do seu grupo na web, acesse:
http://br.groups.yahoo.com/group/ciencialist/

b.. Para sair deste grupo, envie um e-mail para:
ciencialist-unsubscribe@yahoogrupos.com.br

c.. O uso que você faz do Yahoo! Grupos está sujeito aos Termos do
Serviço do Yahoo!.



[As partes desta mensagem que não continham texto foram removidas]



##### ##### #####

Para saber mais visite
http://www.ciencialist.hpg.ig.com.br


##### ##### ##### #####


Yahoo! Grupos, um serviço oferecido por:
PUBLICIDADE





Links do Yahoo! Grupos

Para visitar o site do seu grupo na web, acesse:
http://br.groups.yahoo.com/group/ciencialist/

Para sair deste grupo, envie um e-mail para:
ciencialist-unsubscribe@yahoogrupos.com.br

O uso que você faz do Yahoo! Grupos está sujeito aos Termos do Serviço do
Yahoo!.



##### ##### #####

Para saber mais visite
http://www.ciencialist.hpg.ig.com.br


##### ##### ##### #####


Yahoo! Grupos, um serviço oferecido por:







------------------------------------------------------------------------------
Links do Yahoo! Grupos

a.. Para visitar o site do seu grupo na web, acesse:
http://br.groups.yahoo.com/group/ciencialist/

b.. Para sair deste grupo, envie um e-mail para:
ciencialist-unsubscribe@yahoogrupos.com.br

c.. O uso que você faz do Yahoo! Grupos está sujeito aos Termos do Serviço do Yahoo!.



[As partes desta mensagem que não continham texto foram removidas]



SUBJECT: RE: [ciencialist] Fw: Eletrostatica
FROM: "murilo filo" <avalanchedrive@hotmail.com>
TO: ciencialist@yahoogrupos.com.br
DATE: 14/02/2005 19:12

Credo!
Eu, heim?
Isso só pode ser a ''mãe do ouro''!
Onde se encontrava essa equipe de pesquisadores? abr/M.

>From: "Luiz Ferraz Netto" <leobarretos@uol.com.br>
>Reply-To: ciencialist@yahoogrupos.com.br
>To: "ciencialist" <ciencialist@yahoogrupos.com.br>
>Subject: [ciencialist] Fw: Eletrostatica
>Date: Mon, 14 Feb 2005 11:23:42 -0300
>
>
>Quem entende disso?
>[]'
> ===========================
> Luiz Ferraz Netto [Léo]
> leobarretos@uol.com.br
> http://www.feiradeciencias.com.br
> ===========================
>-----Mensagem Original-----
>De: lessa
>Para: leobarretos@uol.com.br
>Enviada em: domingo, 13 de fevereiro de 2005 17:31
>Assunto: Eletrostatica
>
>Meu Nome é antonio Lessa de Oliveira,
>REsido em em Sao Viocente - SP e meu email é aolessa@ig.com.br
>
>oi estou procurando uma ersposta para o que vi, vou descrever o que
>aconteceu ... e assim poderiam me dizre como foi feito ou que fenomeno é
>esse:
>
>
>uma bolar de luz forte, com o som parecido com oleo quente fritando..... e
>com cheiro de cabelo queimado e ozonio
>
>estavamos no mato em e um coordenado , e entao fez com que se deita-se de
>barriga para baixo e uma bola de luz com as caracteristicas acima apareceu
>de um ponto afastado e andou ate as costas e se dividiu... e apagou
>
>esse fenomeno foi filmado e aconteu varias vezes coordenados por pessoa
>X.....participei e o vipois era filmado....
> poderiam me ajudar? a esclarecer isto? como é feito
>
>
>--------------------------------------------------------------------------------
>
>
> ----------
>
>No virus found in this outgoing message.
>Checked by AVG Anti-Virus.
>Version: 7.0.300 / Virus Database: 265.8.7 - Release Date: 10/02/2005
>
>
>[As partes desta mensagem que não continham texto foram removidas]
>
>
>
>##### ##### #####
>
>Para saber mais visite
>http://www.ciencialist.hpg.ig.com.br
>
>
>##### ##### ##### #####
>Links do Yahoo! Grupos
>
>
>
>
>
>
>
>




SUBJECT: Re: [ciencialist] P/ Esteban ( Zodiaco e Acupuntura..)
FROM: "Esteban Moreno" <estebanmoreno@idhi.org.br>
TO: <ciencialist@yahoogrupos.com.br>
DATE: 14/02/2005 19:23

Oracle:
Estudos, os que foram feitos com rigor e controle, nada mostraram, estando
sua eficácia dentro da chance estatística de acertar ou errar algo. E

Esteban: Por favor, me envie os tais estudos com rigor e controle, um por
vez, que eu mostro ao público as diversas falácias. Eu gostaria de receber
comentários semelhantes sobre o tal grupo da Univ de brasilia, que
demonstrou resultados bem promissores quanto a eficácia da astrologia.

Oracle:
mesmo sem ter mecanismo de ação conehcido (como a homeopatia e a
acupuntura), elas funcionam, só nào conseguem passar em experimentos
controlados com rigor.

Esteban
A astrologia tem um mecanismo de ação conhecido tanto quanto as leis da
termodinâmica, por exemplo, sabe-se que funciona e pronto, verificável por
infindáveis de experimentos. Ok, relatos pessoais não valem, mas isso é uma
condição transiente. O psiquismo é muito mais subjetivo que a grande
maioria dos temas estudados pela ciência, é natural que exista um atraso
neste campo. Assim como os antigos verificaram que existia um principio de
conservação da matéria e aos poucos desenvolveram sua própria cognição do
funcionamento, no futuro em que seremos os antigos, talvez seja possível
perceber que a astromancia tem o seu principio de conservação, muito menos
arbitrário de como supõe. Eu tenho uma proposta de funcionalidade da
astrologia, mas estou sem testículos para receber comentários infundados.
Acho que ainda estou a procurar pessoas afins, pobres vislumbrados como eu.


Oracle:
A astrologia é que deve apresentar estudos de aficácia, nào a ciência
estudos de nào eficácia (embora ela possa apresentar alguns, foram feitos
por quem realmetne acerditava que teria resultado positivo).

Esteban:
O fato é que você não pode afirmar que não funciona tanto quanto eu não
posso afirmar que funciona, em termos estritamente científicos. A sua crença
da não funcionalidade é tão ignorante quanto a minha crença da
funcionalidade, com a diferença que, além de ciência, eu estudei astrologia,
você não. Por isso minha crença é melhor que a sua! ;-) Infinitamente
melhor, pois eu mesmo a testei, e eu próprio fui cético, tão cético que era
merecidamente tido como um chato. Vê como as posições se invertem. Quanto a
astrologia ter que provar algo, isso infelizmente não passa pela cabeça da
maioria dos bons astrólogos. Quem tenta fazê-los não são os astrólogos, os
reconhecidos ao menos (curioso que reconhecido ou não para vocês é mera
arbitrariedade, sem qualquer valor prático).


Oracle:
Pense, eu digo que seres intraterenos, que vivem no interior da Terra, e que
vieram de Alpha Centauro, me deram a chave do conehcimento cosmico, sobre
tudo, mas com o compromisso de nào contar apra mais ninguém. Eu sei tudo o
que vai acontecer no futuro, só nào posso contar. Para provar que eu estou
certo, que minha alegação é real, eu digo a você que "a ciência nào tem
qualquer trabalho verossímel que mostre que eu estou errado e que nào
existem extraterrestres em alpha centauri" , portanto, você deve acreditar
em mim e tomar minha alegação como real. ...

Esteban:
Pura aberração e redundância filosófica. Não merece comentário.

Abraços quase finais,
Esteban.






SUBJECT: Re: [ciencialist] luz velocidade e energia
FROM: "Sergio M. M. Taborda" <sergiotaborda@terra.com.br>
TO: ciencialist@yahoogrupos.com.br
DATE: 14/02/2005 21:06

rayfisica wrote:

>
> Lendo e meditando sobre a questão iniciada por Sr. Helio, sobre a
> velocidade da luz na água ser menor que no vácuo me ocorreu o
> seguinte.
> Algum tempo atrás o Sr. Taborda explicou-me que a luz interage com a
> gravidade por ter energia, pergunto então:
> Essa energia esta relacionada com a velocidade (C)?

Não. A energia da luz é proporcional à sua frequencia e não à sua
velocidade. A energia de um fotão é hf sendo h a constante de plank e f
a frequencia.
a freqeuncia relaciona-se com o comprimento de onda e a velocidade da
luz c = a f , sendo a o comprimento de onda.

> Na água a luz tem mais, menos ou igual energia que no vácuo?


Tem a mesma. (Lei da Conservação da Energia + Lei da Conservação do
Momento)

> Se menos o que acontece coma energia extra quando o fóton entra na
> água (alem de se molhar) ou um meio semelhante?

A velocidade num meio qualquer é relacionada com a velocidade da luz no
vácuo pelo indice de refracção n. Que pode ser medido experimentalmente
para cada meio.
A velocidade da luz num meio qualquer, V,e relacionada coma a velocidade
no vácuo por v = c/n. n é um numero igual ou maior que 1.
Para o vácuo n=1 e v =c
Para a agua n > 1 e v < c.
A energia permanece constante, pelas leis de conservação. Então o que se
altera ?
O comprimento de onda. As contas.

Se v = a f , e v = c / n , c = a f n
Se E = hf , f = E /h

então c = a E n/ h
E , c e h são constantes. Então pela variação de n, temos a variação de a.
Por isso que ao passar no prisma a luz se divide em cores. (isto tb pode
ser intrepretado como um n diferente para cada cor)

> O que acelera o fóton quando sai desse meio? Ou de onde vem a energia
> de aceleração?

Não existe um processo de aceleração.
Para entender isto melhor é preciso o conceito de velocidade dde fase e
de grupo.
A velocidade de grupo é aquela que representa o "movimento" nos mesmo
moldes que aplicamos a objectos macroscopicos.
A velocidade de fase é apenas a relação entre o comprimento de onda e a
frequencia.
A velocidade que se altera de um meio para o outro é a velocidade de
fase. E não a de grupo.
Quando vc pensa que o aumento da velocidade aumenta a energia, vc está
pensando que o aumento da velocidade de grupo aumenta a energia cinetica.
Sim, isso seria verdade, se a energia cinetica da luz fosse váriável.
Ela não é . Ela é constante. ( E = hf )

> O que acontece se colocarmos um corpo negro imerso no meio onde a luz
> tem velocidade menor (pois se a luz tem menos energia que no vácuo
> logo o mesmo quantum não pode excitar o elétron)?

Não. A energia não depende da velocidade.

> Se a energia é a mesma na água, por exemplo, que no vácuo como se
> explica isso? Velocidade diferente mesma energia, não ta faltando
> alguma coisa?

Não. É que a velocidade muda, mas a energia não, pq a enegia não depende
da velocidade.
Essa é a grande sacada da luz. E por isso ela é tão especial.

> A relação da luz na água é diferente do que no vácuo com relação à
> gravidade (interage menos)?

Não conheço nenhuma teoria sobre isso.

Sergio Taborda


--
No virus found in this outgoing message.
Checked by AVG Anti-Virus.
Version: 7.0.300 / Virus Database: 265.8.8 - Release Date: 14-02-2005



SUBJECT: Re: [ciencialist] Fw: Física (Acho q é termologia)
FROM: Luis Brudna <luisbrudna@gmail.com>
TO: ciencialist@yahoogrupos.com.br
DATE: 14/02/2005 21:54

Estado metaestavel.

Acho que estes textos ajudam

http://www.if.ufrgs.br/fis183/textos/superaquec/sa.html

http://www.fisica.ufc.br/qsaber/respostas/qr0038.htm

Ateh
Luis Brudna


On Mon, 14 Feb 2005 11:36:42 -0300, Luiz Ferraz Netto
<leobarretos@uol.com.br> wrote:
>
> Oi Roberto,
>
> onde está o texto referente a isso? Agradeço,
> []'
> ===========================
> Luiz Ferraz Netto [Léo]
> leobarretos@uol.com.br
> http://www.feiradeciencias.com.br
> ===========================
> -----Mensagem Original-----
> De: Karen Lopes de Freitas
> Para: leobarretos@uol.com.br
> Enviada em: domingo, 13 de fevereiro de 2005 21:28
> Assunto: Física (Acho q é termologia)
>
> Olá professor.
> Me chamo Karen, tenho 14 anos e estou no segundo ano do ensino médio, técnico em processamento de dados.
> Recentemente surgiu uma discussão em sala sobre: Por que água e cerveja congelam quando colocamos a mão no meio do recipiente em que estão guardados, após retirar do freezer ou congelador, mesmo que eles não estejam completamente endurecidos.
> Já fiz várias pesquisas, mas ainda não consegui achar uma resposta satisfatória.
> Conto com sua colaboração para desvendar esse 'enigma'.
> Obrigada desde já.
> Karen Lopes de Freitas
>
> --------------------------------------------------------------------------------


SUBJECT: Foucault / Luz ondas ou corpusculos.
FROM: "murilo filo" <avalanchedrive@hotmail.com>
TO: ciencialist@yahoogrupos.com.br
DATE: 14/02/2005 22:17

Oi!
Não é por nada não, mas eu repito esta msg do dia 11, e só por uma razão!
Gostaria, honestamente, que algum sapiente da lista comentasse algo,
confrontado com o que tem sido o atual conhecimento.
Não é pegadinha, qualquer um pode experimentar.
Obrigado pela consideração. abr/M. SP 14/fev

>From: "murilo filo" <avalanchedrive@hotmail.com>
>Reply-To: ciencialist@yahoogrupos.com.br
>To: ciencialist@yahoogrupos.com.br
>Subject: Re: [ciencialist] Re: Foucault / Luz ondas ou corpusculos.
>Date: Fri, 11 Feb 2005 22:47:04 -0200
>
>Oi,
>só uma rapidinha.
>Há uns 45 anos, um tio meu, fotógrafo altamente profissional e de 2ª
>geração, contou-me que quando êle queria tirar uma foto CONTRA A LUZ DO
>SOL,
>ÊLE USAVA O FLASH. E o flash evitava a indesejada sensibilização do filme e
>a perturbação na lente. O flash, mais forte, ''empurrava'' a luz solar...
>coisa linda!
>Nunca mais ouví alguém comentar sobre isto, que aprendí nos meus 15 anos.
>Para mim, leigo, isto é uma prova de que a luz é *matéria*, queiram ou não!
>Alguém comenta? abr/Murilo SP 11/fev
>
> >From: "Alberto Mesquita Filho" <albmesq@uol.com.br>
> >Reply-To: ciencialist@yahoogrupos.com.br
> >To: <ciencialist@yahoogrupos.com.br>
> >Subject: Re: [ciencialist] Re: Foucault / Luz ondas ou corpusculos.
> >Date: Fri, 11 Feb 2005 20:13:02 -0300
> >
> >----- Original Message -----
> >From: "Hélio Ricardo Carvalho"
> >Sent: Friday, February 11, 2005 5:19 PM
> >Subject: [ciencialist] Re: Foucault / Luz ondas ou corpusculos.
> >
> > > Só para complementar a msg anterior.
> > > Acho que na época de FOUCAULT a teoria ondulatória da luz era ONDAS
> > > LONGITUDINAIS NO ETHER. Logo de mesmo tipo que as sonoras.
> > > Quem conhecer bem história da ciência me corrija.
> >
> >A "história" via de regra é contada pelos "vencedores" e a verdadeira
> >história costuma levar séculos para vir a tona e entrar no domínio
> >acadêmico
> >e, assim mesmo, quando passível de ser recuperada. Concordo plenamente
> >contigo ao dizer que a experiência de Foucault fala muito mais a favor de
> >uma luz corpuscular do que uma luz mecânico-ondulatória. O Victor fala em
> >tipos diferentes de ondas, mas lembro que na época em que a experiência
>foi
> >feita somente existiam ondas mecânicas, a menos que alguém quisesse
> >ressuscitar o "espírito da matéria" newtoniano. Enfim, entre esse
>"espírito
> >da matéria" e as ondas não mecânicas (eletromagnéticas, gravitacionais
> >etc.), eu sou mais adepto do "espírito da matéria", ainda que utilize uma
> >nomenclatura mais apropriada aos dias atuais (informação eletromagnética,
> >gravitacional etc.). A meu ver, luz não é campo. Luz é luz e campo é
>campo.
> >Luz é matéria e o campo é imaterial, se bem que tenha sua origem na
>matéria
> >e sua ação em outra matéria.
> >
> > >Se isto é verdade, refaço a pergunta original:
> > > Por que esta experiência descartou a luz corpuscular??????
> >
> >Eu diria que não descartou.
> >
> >Com respeito à luz ser "corpuscular" ou "ondulatória", vale a pena ler o
> >seguinte trecho, copiado de Einstein e Infeld (A Evolução da Física):
> >
> >"A história da busca de uma teoria da luz não está de modo algum
>concluída.
> >O veredicto do século XIX não foi final e definitivo. Todo o problema de
> >decidir entre corpúsculos e ondas ainda existe para a Física moderna,
>desta
> >vez de uma forma muito mais profunda e intrincada. Aceitemos a derrota da
> >teoria corpuscular da luz até reconhecermos a natureza problemática da
> >vitória da teoria ondulatória."
> >
> >Este texto, escrito há mais de 60 anos, ainda é válido em toda a sua
> >plenitude.
> >
> >[ ]´s
> >Alberto
> >http://ecientificocultural.com/indice.htm
> >Mas indiferentemente a tudo isso, o neutrino tem massa, o elétron não é
> >uma carga elétrica coulombiana e a Terra se move. E a história se
>repetirá.
> >
>
>




SUBJECT: Re: [ciencialist] P/ Esteban ( Zodiaco e Acupuntura..)
FROM: "Oraculo" <oraculo@atibaia.com.br>
TO: <ciencialist@yahoogrupos.com.br>
DATE: 15/02/2005 00:10

Olá Esteban

Precisamos dar um jeito de colocar este debate em alguma forma mais racional, ou nâo chegaremos a nenhuma conclusão lógica..:-) Fica dificil discutir com os argumentos escorregando para os lados a cada mensagem.:-)

Astrologia e termodinâmica

- Bem, esta é do mesmo nivel da astrologia e a meteorologia.:-) Não são nem ao menos parecidas, que dirá terem a mesma base ou mecanismo de ação. As leis da termodinâmica derivam de estudos, evidencias, experimentos, resultados, etc, de centenas de anos, com excelente demonstração de eficácia e de precisão. Seu automóvel, assim como fogão, satelites, foguetes, geladeira, etc, etc, etc, só funcionam porque as leis da termodinâmica tem eficácia real. Sua capacidade de prever e acertar nas previsões, com base em cálculos precisos, bate de longe a astrologia, por mais precisa que esta alegue ser..:-)

O que chama de infindáveis experimentos é a base da ciência e de seu método e tem dado bons resultados até o momento e é a mais confiável ferramente que já criamos. Abandona-la para aceitar as alegações da astrologia, realmente exige mais que relatos ou crença pessoal.

Sim, relatos pessoais não valem. Ou melhor, podem indicar um campo de estudo, mas não podem sozinhos embasar uma conclusão. Assim, depois de 3000 anos, é razoável supor que a astrologia falhou em apresentar mais que relatos pessoais.

Crer que funciona versus crer que não funciona

- Este é um engano comum, que é melhor compreendido nos termos dos teístas. Estes quase sempre alegam que ateus tem uma crença, identica a deles, apenas invertida, acreditando que deus não existe. Entretanto, é um engano. Ateus apenas não veem evidencias da existencia de uma divindade (ou necessidade delas para explicar o mundo). Concluir que não há evidencias de existencia real é diferente de "acreditar" que algo não tem existencia real.

Isso serve para a astrologia. Se, como você afirma, nem eu posso provar que ela não funciona e nem você pode provar que ela funciona, o racional, lógico e razoável é concluir que ela não tem evidencias de ser real ou de funcionar como alega..:-) Isso é bem diferente de crer que ela não é real..:-) Veja, nem eu posso porvar que um unicórnio existe, nem você pode provar que ele não existe, ainda assim, penso que nem eu nem você acreditamos que as duas possibilidades são identicas..:-)

Por isso usei o exemplo de seres intra-terrenos. Que você descartou com desprezo e pouco caso, mas que tem defensores sérios e firmes, que usam os mesmos argumentos que você contra a "ciência de mente fechada" ao defender seu ponto de vista. Pessoas que você descartou, usando, como seria de esperar, os mesmos argumentos que eu estou usando para concluir pela inexistencia de evidencias a favor da astrologia.

Não usei os seres-extraterrenos como gozação ou brincadeira, nem mesmo como exemplo de alegação "maluca". Não é maluca para os que a professam. Usei para demonstrar que, quando necessário, mesmo você usa os argumentos da razão e da lógica para refutar crenças alheias, apenas isso.

De novo, eu não tenho crenças a respeito da astrologia (alias, a respeito de nada..:-), eu conclui, e a ciência neste momento também, que ela não tem evidencias de eficácia, conclui que ela não apresenta mecanismo de ação que justifique seu funcionamento, e conclui que ela não apresenta informação real ou verificável. Isso não é crença. E é identico a sua conclusão quanto aos seres-extraterrrenos ou a Atlantida ou qualquer outra alegação que não tenha evidencias e nem seja sua crença pessoal.

Você diz que testou a astrologia. Ótimo. Mas se pretende que estes testes sirvam de evidencia, é preciso que sejam reproduziveis por outros pesquisadores, analisados por diferentes examinadores, criticados e testados até seu limite, como tem sido feito com as leis da termodinamica, por exemplo. E que sobrevivam a este rigor, como tem sobrevivido as leis da termodinâmica até o momento.

Sem isso, é apenas mais um relato pessoal, não uma evidencia científica.

Esteban, nada tenho contra sua crença na astrologia. É direito seu e, enquanto crença, está fora do ambito e dos limites da ciência e da análise. Mas, quando apresenta sua crença como conclusão baseada em evidencias, é permitido analisar essas evidencias, criticá-las e mensura-las. E a astrologia não passa neste crivo, neste rigor, por isso não é considerada real.

Quanto a estudos, eles existem, mas como você bem lembrou, sempre será possível criar "ad hocs" para justificar porque não funcionam.

"Esteban: que eu mostro ao público as diversas falácias".

Esteban, falácias são falhas em argumentos, não erros de experimentos. As conclusões, os argumentos que permitem as conclusões, podem ser falaciosos, o experimento não. Os estudos que foram feitos contaram, em sua maioria, com a consultoria de astrologos, que realmente pensavem que resultados positivos seriam encontrados. Eles realmente acreditavam que o processo gerava resultados reais. E falharam, não devido a falácias em argumentos, mas a incapacidade de produzir os resultados que mesmo os astrologos esperavam.

Este trecho demonstra bem a confusão a respeito do que se está discutindo aqui:

"Esteban: Quanto a astrologia ter que provar algo, isso infelizmente não passa pela cabeça da
maioria dos bons astrólogos. Quem tenta fazê-los não são os astrólogos, os
reconhecidos ao menos (curioso que reconhecido ou não para vocês é mera
arbitrariedade, sem qualquer valor prático)."

Se uma alegação deseja ser cientifica, TEM de provar algo. Não é falha de carater, nem exigencia de mentes fechadas, é pressuposto, parte do conceito e definição de ciência. E deve fazer isso, a prova, de forma precisa e rigorosa, pelo método cientifico. Se não deseja "provar algo", não é ciência e nem é passível de debate ou discussão. Se a questão fosse, desde o começo, uma crença pessoal do tipo "eu acredito e pronto" não haveria debate ou discussão e nem mesmo eu tentaria convence-lo do contrário..:-)

A ciência não se ocupa de alegações impossíveis de verificar ou de crenças pessoais que se embasam em revelações, relatos e experiencias pessoais, segredos herméticos, etc. Ela se ocupa apenas do que pode ser verificado e analisado de forma racional e lógica.

Veja, quem tenta provar algo são os que afirmam ser a astrologia eficaz. A ciência, os céticos se preferir, apenas dizem "não temos evidencias de eficácia ou realidade nas alegações astrologicas". Isso poderia ser deixado assim, pelos que crêem na astrologia, e pronto. Como faz a igreja católica, que assume o misterio e a falta de evidencias fisicas como parte de sua crença.

Mas, astrologos e defensores não se contentam com essa posição, eles precisam demonstrar que existem evidencias para concluir que a astrologia deveria ser aceita pelos céticos como real. E ficam furiosos quando estas são derrubadas, com base no rigor do método. Ora bolas, eles já sabiam que seria assim, que o rigor seria aplicado ele é aplicado a tudo, até as alegações de cientistas, porque seria diferente com a astrologia?..:-)

Esteban, tem razão quando pensa que a ciência não aceita (conclui que é real) alegações sem evidencias. Mas isso não é um defeito, mas sua virtude maior. Com essa postura, ela pode separar joio de trigo por alguns séculos, cada vez com mais precisão e com mais resultados, o que redundou nos avanços que mesmo você pode sentir e se beneficiar. Abandonar essa confiabilidade só seria razoável se algo mais eficaz pudesse ser colocado em seu lugar. Por enquanto, não existe algo assim (e eu duvido que surja tão cedo..:-) e toda alegação deve passar por este crivo se deseja ser aceita ou mesmo chamada de cientifica.

Se a alegação não deseja ser aceita nem chamada de cientifica, então tudo bem, pode se colocar ao lado das centenas de milhares de alegaçõres extraordinarias, todas com a mesma falta de evidencias, e esperar que as pessoas creiam nelas apenas por ser uma atitude confortavel ou consoladora.

Um abraço.

Homero











----- Original Message -----
From: Esteban Moreno
To: ciencialist@yahoogrupos.com.br
Sent: Monday, February 14, 2005 6:23 PM
Subject: Re: [ciencialist] P/ Esteban ( Zodiaco e Acupuntura..)


Oracle:
Estudos, os que foram feitos com rigor e controle, nada mostraram, estando
sua eficácia dentro da chance estatística de acertar ou errar algo. E

Esteban: Por favor, me envie os tais estudos com rigor e controle, um por
vez, que eu mostro ao público as diversas falácias. Eu gostaria de receber
comentários semelhantes sobre o tal grupo da Univ de brasilia, que
demonstrou resultados bem promissores quanto a eficácia da astrologia.

Oracle:
mesmo sem ter mecanismo de ação conehcido (como a homeopatia e a
acupuntura), elas funcionam, só nào conseguem passar em experimentos
controlados com rigor.

Esteban
A astrologia tem um mecanismo de ação conhecido tanto quanto as leis da
termodinâmica, por exemplo, sabe-se que funciona e pronto, verificável por
infindáveis de experimentos. Ok, relatos pessoais não valem, mas isso é uma
condição transiente. O psiquismo é muito mais subjetivo que a grande
maioria dos temas estudados pela ciência, é natural que exista um atraso
neste campo. Assim como os antigos verificaram que existia um principio de
conservação da matéria e aos poucos desenvolveram sua própria cognição do
funcionamento, no futuro em que seremos os antigos, talvez seja possível
perceber que a astromancia tem o seu principio de conservação, muito menos
arbitrário de como supõe. Eu tenho uma proposta de funcionalidade da
astrologia, mas estou sem testículos para receber comentários infundados.
Acho que ainda estou a procurar pessoas afins, pobres vislumbrados como eu.


Oracle:
A astrologia é que deve apresentar estudos de aficácia, nào a ciência
estudos de nào eficácia (embora ela possa apresentar alguns, foram feitos
por quem realmetne acerditava que teria resultado positivo).

Esteban:
O fato é que você não pode afirmar que não funciona tanto quanto eu não
posso afirmar que funciona, em termos estritamente científicos. A sua crença
da não funcionalidade é tão ignorante quanto a minha crença da
funcionalidade, com a diferença que, além de ciência, eu estudei astrologia,
você não. Por isso minha crença é melhor que a sua! ;-) Infinitamente
melhor, pois eu mesmo a testei, e eu próprio fui cético, tão cético que era
merecidamente tido como um chato. Vê como as posições se invertem. Quanto a
astrologia ter que provar algo, isso infelizmente não passa pela cabeça da
maioria dos bons astrólogos. Quem tenta fazê-los não são os astrólogos, os
reconhecidos ao menos (curioso que reconhecido ou não para vocês é mera
arbitrariedade, sem qualquer valor prático).


Oracle:
Pense, eu digo que seres intraterenos, que vivem no interior da Terra, e que
vieram de Alpha Centauro, me deram a chave do conehcimento cosmico, sobre
tudo, mas com o compromisso de nào contar apra mais ninguém. Eu sei tudo o
que vai acontecer no futuro, só nào posso contar. Para provar que eu estou
certo, que minha alegação é real, eu digo a você que "a ciência nào tem
qualquer trabalho verossímel que mostre que eu estou errado e que nào
existem extraterrestres em alpha centauri" , portanto, você deve acreditar
em mim e tomar minha alegação como real. ...

Esteban:
Pura aberração e redundância filosófica. Não merece comentário.

Abraços quase finais,
Esteban.






##### ##### #####

Para saber mais visite
http://www.ciencialist.hpg.ig.com.br


##### ##### ##### #####


Yahoo! Grupos, um serviço oferecido por:

São Paulo Rio de Janeiro Curitiba Porto Alegre Belo Horizonte Brasília




------------------------------------------------------------------------------
Links do Yahoo! Grupos

a.. Para visitar o site do seu grupo na web, acesse:
http://br.groups.yahoo.com/group/ciencialist/

b.. Para sair deste grupo, envie um e-mail para:
ciencialist-unsubscribe@yahoogrupos.com.br

c.. O uso que você faz do Yahoo! Grupos está sujeito aos Termos do Serviço do Yahoo!.



[As partes desta mensagem que não continham texto foram removidas]



SUBJECT: Astrologers rated these killers as good guys
FROM: "Oraculo" <oraculo@atibaia.com.br>
TO: <ciencialist@yahoogrupos.com.br>
DATE: 15/02/2005 00:18

Murder:

Astrologers rated these killers as good guys

An article from the now defunct Kansas City Committee for Skeptical Inquiry. The article was originally titled "Astrology on Death Row!" and was reprinted in the Indian Skeptic 1989, Vol 1 (11). The results of two similar studies have been added in an appendix.


How accurate is astrology? To find out, a member of the Kansas City Committee for Skeptical Inquiry (KCCSI) went to five astrologers posing as a man interested in working with young people. He gave each astrologers the birth date, birth time and place of John Gacy instead of his own, and a computerized natal chart from a company internationally recognized for accuracy (Neil F.Michelsen), and asked them for their advice. The astrologers unanimously encouraged him to pursue youth work and none could see any problem with this.
(John Gacy was born in Chicago on 17 March 1942 at 00:49 CST, or 05:49 GMT, CST being the time required by Illinois for birth registration even though War Time was then in effect. Some hospitals did not follow this rule, so the birth time might have been 06:49 GMT. Both times give a Sagittarius ascendant.)
At the time, the real John Gacy was being held on death row at Menard Correctional Center in Chicago. He received 12 death sentences and 21 life terms for the brutal torture and murder of 33 young men and boys. He was executed by lethal injection on 10 May 1994.
Gacy was selected for the test because his chart should portray a clear picture of a sadistic sexually motivated killer. If astrologers are able to spot personality traits and destinies in any chart, then this is the one they should have no trouble with.
The way in which Gacy murdered his victims was particularly repulsive. They were usually teenage boys whom he lured into his home and overpowered. He had a pair of trick hand cuffs, which he would demonstrate that he could slip out of. After immobilizing them by persuading them to put on the hand cuffs, he repeatedly raped and tortured them. Finally after they begged for their own deaths he would strangle them, often by shoving their own underwear down their throats. Each astrologer was asked to examine the computerized chart for as long as they wanted before giving their reading.

Rhe advice given by the astrologers

John Sandbach, a nationally known astrologer who has authored six books, advised him not to "become weighed down with regrets about how you could have done more in some past situation." He described a "plasticity or lack of aggression" in the chart and encouraged him to work with young people because he could "bring out their best qualities."
Local astrological talent, Randy Goodman, was reported by the Kansas City Star (April 10, 1985) to have investigated "mysterious flying hotdogs" at the Radisson Hotel Muehlebach. He told our substitute Gacy that he was "really born to serve people." He stated that "In the past you have used your energies very well so therefore in this life you have a lot to contribute, and... your life will be very, very positive."
Another local astrological celebrity, Norma Knight, was also given an opportunity to analyze the chart. She described him as "a very, very sensitive person." When asked about the youth ministry she replied, "I think that you can be very good with kids and that it might be a good medium for you to learn to be more trusting in the giving and receiving."
Beverly Farrel lays claim to being an "internationally recognized author, lecturer and teacher of religion, metaphysics, astrology and psychic awareness" with "30 years experience in the field of (the) paranormal". She also encouraged the man behind the chart to go into youth work. She stated that when you're working with young people you're not to have a lot of heavy-duty problems."
The results seemed to indicate that astrologers cannot read a persons character from the positions of the planets at the moment of birth nor can they see into anyone's past or future nor do they seem to possess any insights other than the ability to impress their clientele.

How well did the readings fit anyone?

KCCSI next passed out envelopes to students in several local college philosophy classes. The students were instructed to write their names, birthdates, birth times and places on the outside of the envelopes and return them. KCCSI put in each envelope an excerpt from the Gacy readings and handed them back to the students telling them that a professional astrologer had made up a personality description specifically for them. They were asked to grade how well each descriptin fitted them.
When the results were tabulated it was discovered that those students who believed in astrology showed a significant tendency to grade the readings as more accurate than did those students who did not believe. This supports the premise that the popularity of astrology is due to the predisposition of the believers to exaggerate its accuracy and thus make it appear to work. Also this would explain the persistence of many people to assert that astrology really works despite evidence to the contrary.

What the astrologers said about Gacy

"I think that you can be very good with kids and that might be a good medium for you to learn to be more trusting in the giving and receiving."

"...just your presence would be of a beneficial nature to other people, a real calming kind of effect... In the past you have used your energies well: so therefore, in this life you have a lot to contribute and you will have some problems but basically your life will be very, very positive."

"...a fairly well-rounded personality...you can offer a good role model...when you're working with young people you're not gonna have a lot of heavy-duty problems."

"Helpful, understanding of the needs of others. At times a sucker for anyone who needs help... Kind, gentle, considerate of others needs."

"At your best, you are very impressionable and radiate the unconditional love of a happy infant... You have an instinctive awareness and your uninhibited response to life can refresh and gladden whomever you encounter."

Appendix. Two similar tests using mass murderers

The first is from Gary F Posner, Skeptically Speaking March 1993, with further information from a videotape of the Stossel TV show. The second is from an article by Michel Gauquelin in Aquarian Agent May 1970.
Joyce Jillson, whose column is carried by the Tampa Tribune and who is Hollywood astrologer to the stars, unwittingly allowed ABC-TV reporter John Stossel an opportunity to demonstrate for his millions of Good Morning America viewers in November 1988 just how well astrology works in spite of it appearing not to be real.
Jillson (who normally charges $200 per chart) first prepared a detailed reading for a person unknown to her, whose birth information (which is all she requested) was supplied to her by Stossel. Stossel then distributed a copy of the completed horoscope to each of 20 students in an adult education class (each student had given Stossel their own birth information one week earlier).
The students, thinking they were each reading their own personalized horoscopes, marveled at how Jillson knew things about them that no one else could possibly know. Typical responses were "very accurate", "I am amazed", "hits points that nobody knows". Stossel asked one student "is it you?" (yes) "but is it different from other people?" (yes it is). But each student, male or female, was reading the same horoscope, that of someone described by Jillson as "enormously bright ... [with] sexual charisma ... great charm ... a sense of moral propriety ... [who] may know celebrities ..."
Stossel thought that Jillson may have incorrectly assumed the birth information to have been his own. But Jillson's one-size-fits-all unisex horoscope was actually based on the birth information of mass murderer Edward Kemper III who, in addition to many other "charming" deeds, had cut off his mother's head and used it as a dartboard! Ironically one statement fitted the authentic subject quite aptly, namely "takes life seriously." Concluded Stossel, while confronting Jillson on camera with the facts and watching her squirm, "I just think this shows it's a hustle, and you make money by writing general things that everybody believes is about them."
(All this had no noticeable eggect on Jillson's popularity. In 2003 she suffered what the Skeptical Inquirerul Nov/Dec 2003 page 9 called "the ultimate embarrassment for an astrologer". In her syndicated column of 12 September, Jillson had given a glowing horoscope of TV star John Ritter "having a Virgo sun sign helps keep his career tickng", even though he had died the day before.)
The same thing happened when Michel Gauquelin gave 150 people a 10-page computer interpretation of supposedly their own birth chart but which was actually that of the notorious mass murderer Marcel Petiot. He asked for their opinions, and 90% found the accuracy to be confirmed by their family and friends, and 94% found it to accurately describe their character, their personal problems, and the cycle of events in their life. It produced responses like "the work done by this machine is marvellous", and "on the whole, everyone who knows me found it accurate, especially my wife" (from Aquarian Agent May 1970).
Obviously perceived accuracy means little. Studies like these have repeatedly shown that people find astrology's insights just as convincing when unknown to them the wrong birth data are used. For a worked example using Petiot's chart see Perceived effect sizes under Tests & Snags.


[As partes desta mensagem que não continham texto foram removidas]



SUBJECT: Astrologers vs Journal of Consciousness Studies
FROM: "Oraculo" <oraculo@atibaia.com.br>
TO: <ciencialist@yahoogrupos.com.br>
DATE: 15/02/2005 00:23

Olá pessoal

O artigo abaixo é longo, mas vale a pena dar uma lida. Os apendices do texto não foram copiados aqui (ficaria grande demais para um email..:-) e podem ser lidos neste link:

http://www.rudolfhsmit.nl/d5-starwarsnew.htm

Um abraço.

Homero
___________________________________________________________
Star Wars


Astrologers vs Journal of Consciousness Studies


The special June/July 2003 issue of the prestigious Journal of Consciousness Studies was devoted to parapsychology, It contained twelve long articles, and the issue as a whole received praise for its balanced approach from New Scientist (13 September 2003 "authoritative and accessible"), and from Amazon ("serious and responsible"). But what caught media attention was the article entitled "Is Astrology Relevant to Consciousness and Psi?", a scholarly article of 24 pages and 85 references by Geoffrey Dean and Ivan W Kelly, which one astrology website later described as "Dean and Kelly rehashing old and flawed research hype". It led to the biggest media frenzy on astrology for 2003.







Two things make the frenzy of particular interest. First, its focus is not your everyday sun sign astrology but the serious astrology of conferences and consulting rooms. Second, the frenzy brought out some of astrology's top defensive guns, so you can judge how well they perform when confronted by scientific findings. (If you happen to be a True Believer you may prefer to stop reading now, or at least take a tranquilliser.)

Dean and Kelly start by quoting various astrologers on how a successful birth chart reading requires some kind of psychic ability, where the chart acts like a crystal ball. If this were found to be true it might require a re-assessment of present theories of consciousness, so it deserves study. But a large-scale test of 2101 persons born on average less than five minutes apart found no hint of the similarities in personality or behaviour predicted by astrology. So if astrologers (as opposed to astrology) can predict personality or behaviour better than chance, as they claim to do, it might be evidence for psi.

But meta-analysis of more than forty controlled studies found no evidence that astrologers perform even marginally better than chance, even on basic tasks such as predicting extraversion (basic because according to astrologers it is one of the easiest things to see in a birth chart). They did not even usefully agree on what the birth chart indicates. More to the point, astrologers who claimed to use psychic ability performed no better than those who did not. Dean and Kelly cautiously conclude "the possibility that astrology might be relevant to consciousness and psi is not denied, but such influences, if they exist in astrology, would seem to be very weak or very rare". See Appendix 1 for an abstract of their article.

Normally this cautious non-link between astrology and psi might have passed unnoticed. But it was picked up and distorted by the Sunday Telegraph London, 17 August 2003 ("Astrologers fail to predict proof they are wrong"), and was duly copied or quoted around the world from Brazil to Finland. It was distorted because Dean and Kelly's focus was psi and consciousness, not the merits of astrology (for example its merits include providing low-cost ego support, and astrologers are generally nice people), merits they had already covered in other articles such as their chapter in Paul Kurtz's Skeptical Odysseys, Prometheus 2001. The result was a frenzy of misleading headlines and reports such as "Is astrology bunk?" (Daily Mail London, 18 August), "Research paper rubbishes astrology" (Hindustan Times India, 17 August), and "Who will put their faith in the stars?" (Sunday Herald Sun Melbourne, 14 September).

Regardless of the distortion, astrologers were predictably outraged and uninformed. They declared that negative results are by definition due to ignorance or hostility. Especially outraged were Indian astrologers, one of whom said in emails "it is only a study by some crazy white b's. They do not have any brain". Another said "Most probably these two guys are unemployed". Another said "if you really want to test astrology ... the only place where the research can be justified is here in India" (where presumably men are Men and astrologers are Astrologers).

Perhaps the most notable response from India was "Astrology is Science, not Rubbish" (India Express 25 August), where Dr Raj Baldev "who is considered an authority on the subject of Astronomy, Astrology, Cosmo-Mathematics and Metaphysics" (he has a website www.occultastrology.com offering "The perfect gift. Occult Horoscope by post. Only $6.95") said Dean and Kelly had made "an abominable mistake that can never be pardoned". He explained that ancient Hindu astrology "is a complete science" where even one million billionth of a second "makes a lot of difference". So it is ridiculous to believe that people born a few minutes apart should be similar. (Measuring birth times to a million billionth of a second implies that the position of shadows cast on ancient sundials was routinely read to better than a hundred millionth of the diameter of an atom. Even at night. Should we believe it?)

Western astrologers did not hesitate to give opinions without having read the article. In a Melbourne radio interview, Brian Clarke from the Australian Federation of Astrologers explained how there was more to astrology than sun signs, so all was well (in fact the Dean and Kelly article had nothing to do with sun signs). In The Guardian London, 19 August, astrologer Neil Spencer noted how astrology can "send arch-rationalists into fits of self-righteous indignation" (like his?), how the article lacks details (not true), how it ignores the positive results of Vernon Clark and Gauquelin (not true), and how the "Magi Society [an international society of astrologers based in New York] ... still has to receive a riposte to its statistical challenges" (one is in Skeptical Inquirer March-April 1997). He ends with "Astrology is not a science but a symbolic, allusive language" (boo to Dr Baldev), as if that somehow excused its failure to deliver on testable claims, to which the sociology website www.butterfliesandwheels.com replied, "Oh that old ploy".

The most orchestrated response came from the AA (UK Astrological Association), which in 2000 had refused to publicly declare its position on sun signs despite overwhelming evidence for their invalidity (see Response to an invitation under Sun Signs). The AA president Roy Gillett accused Dean of seeking to "discredit astrology". The AA website www.astrologicalassociation.com accused Dean and Kelly of having a "tortured imagination" and "defensively closed mindsets" that "deny astrology an even-handed debate", to correct which it then gave "a balanced response" via two reports that together "comprehensively dismiss these outrageous and disingenuous claims".

The first report, which originally appeared in the Daily Mirror London, 18 August 2003, was by "world renowned astrologer Jonathan Cainer". Cainer has his own astrology phonelines, and according to the Sunday Times London his estimated income of 2.2 million pounds a year puts him among the top 150 UK earners. Cainer begins by complaining that it cost him 15 pounds to download the article, whereupon he was suspicious the moment he saw the authors' names (refutation by name-calling?), claiming that Dean deliberately misunderstands what astrologers do (Dean is a former astrologer and understands very well what astrologers do).

Cainer then drops a series of clangers -- personality tests are "dangerously unreliable" (not these ones), "most scientists hate astrology" (most have better things to do), the tests covered ages only up to 23 (so Cainer rejects phoneline callers under 23?). His most notable clanger was "Once again, it turns out to be an experiment rigged to make astrology look silly" (in fact it was the AA who proposed the experiment in the first place, not in a moment of weakness but in a well-researched eight-page proposal prepared when the subjects were seven years old). He ends with "And some scientists claim to have a truly open mind" (like his open-minded behaviour towards negative findings?). See Appendix 2 for Dean and Kelly's point-by-point response to Cainer.

The second report was by "leading author and academic Dr Frank McGillion", a consultant to the astrological research group at Southampton University, who is said to find the article "essentially flawed and well below expected academic standards". Dr McGillion begins by saying the article is too long, then it is too short. He quibbles about definitions without providing his own definitions, he dwells on side issues without explaining their relevance, he cites unspecified "evidence" without supporting references or arguments, and he generally declines to act the way he tells Dean and Kelly to act.He says a focus on consciousness and psi

"might seem less relevant ... than the authors appear to consider" (the call for papers required it), he dislikes "the citation of names of non-scientists in formal scientific papers" (so how to give astrologers' views without citing astrologers?), and he says editing such psi papers "is a demanding task for the non-specialist" (one editor was world expert James Alcock). At the end he generously allows that "there is much here that astrologers can learn from" (so Cainer was wrong to call it "a load of rubbish"?). Nevertheless conspicuously absent is a description of what Dean and Kelly did and a critique of their results (refutation by censorship?). See Appendix 3 for Dean and Kelly's point-by-point reponse to Dr McGillion.

Claims of credibility are clearly not helped when the AA, which calls itself "one of Europe's leading astrological organisations", can see these two reports as "a balanced response" and "even-handed debate".

Yet other astrology websites uncritically recycled the reports almost verbatim. For example www.astrology.co.uk said Dean and Kelly were "manipulating results ... using self-fulfilling personality tests ... selecting data to fit results", and dropped new clangers such as implying the average birth interval was one day (it was less than five minutes), and asking "who says astrologers are psychic?" (answer: the astrologers and surveys quoted). Another website said "Dean and Kelly are known to be opponents of astrology", as if seeking evidence was an act of heresy. Similarly, in The Mountain Astrologer (Dec/Jan 2003/2004 issue), the American astrologer Gloria Star (without reading the article) said "the research itself appears to be significantly flawed". No doubt her opinion will be repeated by other astrologers around the world, always without reading the article. Should we be surprised?

Without exception the astrologer responses boiled down to name-calling, getting it wrong, and never citing compelling evidence to support their claims. Even serious media outlets adopted this distortion. For example in the New Zealand Listener (4 October 2003), in an article subtitled "a recent study claims to debunk astrology once and for all" (not true), astrologers and skeptics are seen as being permanently locked in "trench wars" and "long feuds", serious studies are seen as attacks on astrology, researchers are seen as debunkers, and astrologer quotes such as "you will never get a correlation that is significant, because we are dealing with individuals" are seen as an adequate response to the negative meta-analysis even though it involved individuals. So "In the end, you either believe or you don't" (yes, why have tests when you can have shouting matches?). In the end readers receive only titillation for the hard of thinking. Such outlets never notice that the two sides might be talking about different things (facts versus benefits) which are not mutually exclusive. It seems that media interest in astrology generally destroys any hope of informed debate.

Later, in the January/February 2004 issue of The Astrological Journal, AA President Roy Gillett added "Of course it is easy to answer the Dean/Kelly/Randy/'CSI COPS' (and all other) criticisms", presumably as easy as misspelling Randi and CSICOP, but he conveniently omitted to say how. Most likely he meant more of the same getting it wrong.

And in the January 2004 issue of Correlation, the AA's journal of research in astrology, Dr McGillion presents an update of his report. In it he claims the JCS article ignores relevant literature, is wordy, vague, illogical, factually incorrect, poorly researched, poorly edited, with imprecise definitions, nonsensical statements, loose terminology, fallacious reasoning, much unnecessary speculation, and much unnecessary material. So he is "not convinced it makes any meaningful contribution to consciousness research". Evidently Dr McGillion thinks the article and the entire JCS issue is (or should be) about consciousness research, not parapsychology. His focus is consequently wrong from the start, which makes most of his comments less than relevant. His rule that one should be relevant and "get it right" is evidently not one that he himself observes.

Inexplicably, Dr McGillion again mentions neither the aim of Dean and Kelly's article, nor their results, nor their conclusion, nor their discussion of artifacts and hidden persuaders, nor even the word parapsychology. He quotes the article out of context, and then uses the lack of context to ridicule the quote. He gives few details of the points he refers to, so his comments tend to read like riddles. The result is like commenting on a restaurant menu without mentioning food. See Appendix 4 for Dean and Kelly's point-by-point response to Dr McGillion.

It will be obvious by now that astrologers do not like awkward facts. But the reports presented by their top guns as counter evidence are essentially arguments by distortion and innuendo. Whatever we may think of astrology, it deserves better than this.


[As partes desta mensagem que não continham texto foram removidas]



SUBJECT: Re: [ciencialist] Ainda há tempo
FROM: José Renato <jrma@terra.com.br>
TO: <ciencialist@yahoogrupos.com.br>
DATE: 15/02/2005 00:25

Olá, Murilo!
Grato por seu comentário e incentivo.
Abraços
José Renato
..................................
From: "murilo filo" <avalanchedrive@To: <ciencialist@yahoogrupos.com.br>
Cc: <avalanchedrive@hotmail.com>
Sent: Sunday, February 13, 2005 7:51 PM
Subject: RE: [ciencialist] Ainda há tempo


> Zé, oi.
> Gostei muito da sua matéria.
> Todos na lista deveriam lêr, *pelo menos*, a última frase.
> Umas das sínteses mais verdadeiras que já lí e que tem muito que vêr como
> a
> 'minha' filosofia.
> Deve ser muito duro não entender esta cosmologia.
> Obr., abração. Murilo SP 13/fev
>
>>From: José Renato <jrma@terra.com.br>
>>Reply-To: ciencialist@yahoogrupos.com.br
>>To: <ciencialist@yahoogrupos.com.br>
>>Subject: [ciencialist] Ainda há tempo
>>Date: Sun, 13 Feb 2005 12:00:23 -0300
>>
>>
>>O alerta dado pelos ambientalistas de 340 ONGs da Rede Ação Clima - RAC,
>>em
>>Paris, durante a abertura de uma série de atos por conta da entrada em
>>vigor do Protocolo de Kioto no dia 16 de fevereiro, fez-me lembrar do
>>artigo que escrevi durante a reunião anual da SBPC, em Salvador, há dois
>>anos, que colo abaixo para lembrança e divulgação.
>>Abraços
>>José Renato
>>....................................................
>>
>>A PONTE
>>
>>José Renato M. de Almeida*
>>
>>
>>Em uma região longínqua, havia uma ponte ligando as duas margens de um
>>rio.
>>
>>Com o passar dos anos algumas pessoas começaram a percebê-la e,
>>imediatamente, a questionar sobre ela. Inicialmente, essas questões vinham
>>sob um sorriso compreensivo, tolerante ou uma boa risada de deboche. Ora -
>>diziam os mais apressados - ela está aí porque está! Porque sempre esteve!
>>
>>E o que antes nem era percebido passou a ser a curiosidade e o assunto de
>>muitas conversas, elucubrações e motivo de muitos 'causos'. Quem construiu
>>essa bela ponte? De que ela é feita? Para que foi construída se só agora
>>há
>>habitantes nas margens do rio?
>>
>>
>>Os mais antigos do lugar diziam que tinham ouvido seus avós contarem que,
>>há muito tempo, chegou um exército numeroso de homens com cabeças
>>resplandecentes, alguns empunhando espadas com o brilho de sol, escalando
>>torres e escadas que alcançavam os céus. Foram eles que escavaram e
>>cravaram grandes tubulões na terra. Durante dia e noite, construíram a
>>ponte em meio a estranhos rugidos e estrondos de trovões.
>>
>>
>>Outros diziam que ela fora trazida dos céus já pronta e instalada ali
>>mesmo, por seres alados.
>>
>>Eram várias as lendas e histórias para explicar seu aparecimento, sua
>>construção ou instalação. Lembramos mais algumas delas. Uns diziam que
>>foram as intempéries climáticas, agindo nas rochas, que haviam moldado a
>>ponte naquele formato, por mero acaso. Outros aventavam que uma enorme
>>besta de carga havia sacudido a cangalha do lombo, que, atirada longe, foi
>>se fincar justamente daquele jeito sobre o rio. E muitas outras
>>explicações.
>>
>>
>>A APM - Assembléia dos Pescadores a Molinete, proclamava aos seus
>>seguidores que a ponte havia sido construída por um antigo senhor muito
>>bondoso, para que eles pudessem pescar com todo o conforto de cima da
>>ponte, apreciando a brisa suave e o sol cálido.
>>
>>Os da CPT - Congregação dos Pescadores a Tarrafa, afirmavam que ela fora
>>construída para que eles pudessem pescar desfrutando a sombra e a água
>>fresca sob a ponte.
>>
>>Muitos não entravam nessas questões metafísicas ou utilitárias de menor
>>relevância, mas arranjavam um jeito de cobrar pedágio e licenças daqueles
>>que a atravessavam ou a usavam para pescar.
>>
>>
>>Todas essas controvérsias, porém, não bastava aos mais curiosos. Um grupo
>>deles reuniu-se para procurar responder as principais questões ainda
>>pendentes. Depois de pesquisar externamente as superfícies e as formas da
>>ponte, decidiram investigar sua estrutura interna. Houve algum receio e
>>até
>>protestos dos mais precavidos. Alguns acreditavam até que, se a ponte
>>fosse
>>perscrutada - a fim de tirar amostras para análise dos materiais internos
>>de que era constituída - ela poderia desabar, caso atingisse pontos
>>críticos de sustentação. Convenhamos que isso seria uma grande catástrofe
>>para os habitantes da região, já acostumados a andar pra lá e pra cá sobre
>>ela. Mas eles não se intimidaram. Após muitas discussões decidiram
>>remover,
>>com equipamentos especiais projetados com esse objetivo, amostra do
>>material de seu interior. Tiveram sucesso no projeto e divulgaram, com
>>grande euforia, o resultado das análises a todos os habitantes.
>>
>>
>>Foi uma época de sucesso e vitória do conhecimento. Enfim, as respostas
>>estavam dadas! Os componentes usados na edificação da ponte, aqueles que
>>compunham sua estrutura não eram mais nenhum mistério.
>>
>>
>>Depois disso, fizeram vários corpos de prova na tentativa de obter
>>compostos que proporcionassem resultados práticos semelhantes em
>>resistência, flexibilidade e harmonia de formas. Muitos fracassos foram
>>ocultados e as experiências de sucesso passaram a ser registradas e
>>vendidas, proporcionando grandes somas aos seus 'descobridores'.
>>
>>
>>Mesmo assim, ainda havia alguns pesquisadores insatisfeitos com todas
>>aquelas maravilhosas descobertas. Esses, silenciosamente, passavam dias e
>>dias pensando, refletindo e conjeturando na tentativa de redesenhar o
>>projeto original da ponte. Investiam em intensas pesquisas com ultra-som,
>>raios-X, ressonância magnética, cálculos complicadíssimos e muita, muita
>>imaginação.
>>
>>
>>A grande maioria dos habitantes achava perda de tempo essas tentativas de
>>resgatar o projeto original ou pelo menos tentar elaborar um esboço dele.
>>Afinal, diziam, a ponte já estava ali, totalmente pronta e utilizável.
>>Para
>>que então se preocupar com isso? Esses malucos! Será que não têm nada que
>>fazer de mais útil!
>>
>>
>>O que de fato preocupa a nós outros é que poucos estão empenhados em
>>preservá-la. Poucos se dão conta de sua importância para a própria vida e
>>à
>>vida dos demais habitantes. Acreditamos que, se nada de concreto for
>>acordado e algo de efetivo realizado, breve a ponte estará deteriorada,
>>suja, contaminada e intransitável. A hora de agir já tarda.
>>
>>
>>
>>
>>Nota: Este artigo foi inspirado nas idéias do filósofo Spinoza (1569-1630)
>>de que Deus é igual à natureza - Deus sive natura. Afirma também, que a
>>felicidade máxima do homem é o conhecimento de Deus e este é precisamente
>>o
>>fim ao qual a filosofia se propõe. Considera que Deus é a única substância
>>existente. Como substância, Deus é a cadeia ou processo causal, a
>>subjacente condição de todas as coisas, a lei e a estrutura do mundo. Esse
>>concreto universo de modos e coisas está para Deus assim como uma ponte
>>está para o seu projeto, sua estrutura, e as leis da matemática e da
>>mecânica segundo as quais ela é construída; são estas as bases de
>>sustentação, a condição subjacente, a substância da ponte; sem elas, ela
>>cairia. (Tratado político-teológico)
>>
>>As idéias não têm dono, mas têm autoria.
>>
>>* José Renato M. de Almeida é engenheiro, aprendiz de filosofia e
>>teologia.
>>
>>SSA, 1° de maio de 2003
>>
>>Rev. 14.06.2004



SUBJECT: How to convince clients that astrology works
FROM: "Oraculo" <oraculo@atibaia.com.br>
TO: <ciencialist@yahoogrupos.com.br>
DATE: 15/02/2005 00:36

Chamada do artigo abaixo:
Hidden persuaders are systematic errors in our reasoning that explain why a totally invalid system can still seem to work. Explores over 30 hidden persuaders which convince clients that astrology works. All are in everyday use in astrology. None require that astrology be true.

Tradução (meia boca..:-)
Persuadores escondidos (de persuadir, convencer) são erros sistemáticos em nosso raciocínio que explicam porque um sistema totalmente inválido pode aparentar funcionar. Apresenta mais de 30 "hidden persuaders" que convencem clientes que a astrologia funciona. Todos são usados em astrologia todos os dias. Nenhum requer que a astrologia seja real.

_______________________________________________________
Hidden persuaders:
How to convince clients that astrology works
By Geoffrey Dean and Ivan W Kelly

Unaided human reasoning is subject to systematic errors (we call them hidden persuaders) that can explain why an experience-based astrology should seem to work even if it were totally invalid. For example hidden persuaders explain why tens of thousands of Western tropical astrologers can say that in their experience Scorpios really are intense, while hundreds of thousands of Eastern sidereal astrologers can look at the same piece of sky, which they call Libra, and agree that in their experience it is not intense but relaxed. The same applies to all the others factors that astrologers disagree about, which is most of them.

Caution. You will be led seriously astray if you learn about astrology without first learning about hidden persuaders. Unless you can be sure that hidden persuaders have been ruled out, don't believe what you read in astrology books. In particular the claim that astrologers proudly and repeatedly make, that astrology is unassailable because it is experience-based, is simply mistaken -- what they see as its strength is actually its weakness. If this sounds preposterous, consider the following quote from psychologist Ray Hyman, How not to test mediums, Skeptical Inquirer 27(1), 20-30, January-February 2003. Hyman has devoted more than half a century to the study of psychic and other readings, especially to why such readings can seem so compelling.

How not to test a reading

"As a way to earn extra income, I began reading palms when I was in my teens. At first, I was skeptical. I thought that people believed in palmistry and other divination procedures because they could easily fit very general statements to their particular situation. To establish creadibility with my clients, I read books on palmistry and gave readings according to the accepted interpretations for the lines, shape of the fingers, mounds, and other indicators. I was astonished by the reactions of my clients.

"My clients consistently praised me for my accuracy even when I told them very specific things about problems with their health and other personal matters. I even would get phone calls from clients telling me that a prediction that I had made for them had come true. Within months of my entry into palm reading, I became a staunch believer in its validity. My conviction was so strong that I convinced my skeptical high school English teacher by giving him readings and arguing with him. I later also convinced the head of the psychology department where I was an undergraduate." (page 22)

So far it could be any astrologer talking about astrology. They read charts and they become staunch believers in astrology's validity. As in Hyman's case, their experience seems totally compelling. But this is not the way to test a reading. Hyman then did something every astrologer should do but never does:

"When I was a sophomore, majoring in journalism, a well-known mentalist and trusted friend persuaded me to try an experiment in which I would deliberately read a client's hand opposite to what the signs in her hand indicated. I was shocked to discover that this client insisted that this was the most accurate reading she had ever experienced. As a result, I carried out more experiments with the same outcome. It dawned on me that something important was going on. Whatever it was, it had nothing to do with the lines in the hand. I changed my major from journalism to psychology so that I could learn why not only other people, but also I, could be so badly led astray." (page 22)

Hidden persuaders

The answer to Hyman's puzzle is hidden persuaders, factors that can make a vague reading seem so uncannily accurate that it becomes almost impossible not to believe in the system's validity. There are many hidden persuaders. The most important hidden persuader for astrology generally is the consider-only-confirming-cases artifact discussed in the Artifacts and Truth articles on this website, but for a client consultation it becomes just one among thirty-four as listed below. None of these hidden persuaders require that astrology be true, yet each will (wrongly) convince astrologers and clients that astrology works. All are in routine use in astrology consulting rooms. For convenience we have grouped them under seven descriptive strategies:

1. Select initial hurdles

a.. Preach to the converted (client predisposition)
b.. Ignore everything on this website (ignorance is bliss)
c.. The best things in life are not free (charging a fee)
2. Stifle chances of being wrong

a.. Appeal to birth chart complexity (nonfalsifiability)
b.. Avoid conflict, see what you believe (cognitive dissonance)
c.. Believe what you cannot prove (unavailable data)
d.. Remember the hits, forget the misses (selective memory)
e.. Ask only confirming questions (stacking the deck).
This is the consider-only-confirming-cases artifact.
f.. Ignore disconfirming evidence (confirmation bias)
g.. Deny that astrology can be tested (testability veto)
3. Use cues

a.. Let context give the game away (vital statistics)
b.. Let body language be your guide (cold reading)
4. Make astrology look good

a.. The importance of first impressions (halo effect)
b.. If it looks right then it is right (face validity)
c.. Style is more important than content (Dr Fox effect)
d.. Underestimate chance effects (chance baseline shift)
e.. More is better (Aunt Fanny effect)
5. Make clients feel good

a.. Use a kind heart to entice belief (tea and sympathy)
b.. The power of positive thinking (Pollyanna principle)
c.. It does us good if we think it does (placebo effect)
d.. Having control makes us feel better (misattribution)
e.. Just naming the unknown is enough (Rumpelstiltskin effect)
f.. Closeness is its own reward (rapport)
6. Make the chart fit

a.. Find meaning where none exists (faces in clouds)
b.. Read specifics into generalities (Barnum effect)
c.. See only what you want to see (illusory correlation)
d.. Accentuate the positive (social desirability)
e.. Be seduced by resemblance (magical thinking)
f.. Afterwards we knew it all along (hindsight bias)
g.. Sound arguments yes, sound data no (stereotypes)
7. Make the client fit

a.. Find something, anything, to match the chart (repertoire)
b.. Let client role-play their chart (self-fulfilling prophecy)
c.. Force client to fit their chart (Procrustean effect)
d.. Winter does not last forever (regression to mean)
Each hidden persuader reflects the systematic error in human reasoning shown in parentheses, for which we have used the accepted name if there is one, or a provisional name if not. In the early days it was usual for critics to explain why clients were satisfied with astrology readings in terms of the Barnum effect, the reading of specifics into generalities such as "you have problems with money", where sense appears to come from the reading when in fact it comes from our ability to make sense out of vagueness. Today, as shown above, many more hidden persuaders are known. They vary in effectiveness, and in a given situation some may be irrelevant, but all lead to client satisfaction and none require that astrology be true.


But if clients are going to be satisfied, astrologers can hardly fail to believe in astrology. In this way a vicious circle of reinforcement is established whereby astrologers and their clients become more and more convinced that astrology works. Note that there are no hidden persuaders to convince us that astrology does not work other than the informed critical mind, which of course is not an error as such but rather a defence against errors. An astrologer typically makes no effort to become informed about research or to acquire critical thinking skills, preferring instead to spend years learning to read charts, during which time they have ample chance to respond to the above reinforcement. And of course to build up a huge vested interest in continuing their ill-founded beliefs. If this still seems preposterous, consider the salutary case of phrenology.

The salutary case of phrenology

The same hidden persuaders explain how phrenology (head reading), once more popular and far more influential than astrology is today, could be accepted as totally valid even though it is now known to be totally invalid. As noted in 1985 by Dean and Mather (Astrological Journal 28(1), 23-30, Winter 1985):

"Astrologers are like phrenologists: their systems cover the same ground, they apply them to the same kinds of people, they turn the same blind eye to the same lack of experimental evidence, and they are convinced for precisely the same reasons that everything works. But the phrenologists were wrong. So why shouldn't critics conclude for precisely the same reasons that astrologers are wrong?" (page 25)

That was nearly 20 years ago, but no reply from astrologers has been forthcoming other than to dismiss phrenology as irrelevant. But perhaps no reply is required. After all, it could be argued that the existence of mutually incompatible systems throughout astrology (for example tropical and sidereal zodiacs), all of which are nevertheless seen as completely valid by their users, has already put this question to the test and given us convincing answers.

The bottom line

Thanks to hidden persuaders, the bottom line could hardly be simpler --astrology in the consulting room does not need to be true. This point was aired in a Skeptical Inquirer article published in 1986-1987, and in the later works listed below, where among other things readings that were the opposite of what the chart indicated were found to be as acceptable to clients as authentic readings, just as Hyman had found for palmistry. But the point has been generally ignored by astrologers. Readers interested in applying critical thinking to astrology readings will find much useful information in Gambrill (1990).

Further reading

Dean G (1992). Does astrology need to be true? In Frazier K (ed). The Hundredth Monkey and other paradigms of the paranormal. Prometheus Books, Amherst NY, pages 279-319 with 126 references. Update of a two-part article first published in Skeptical Inquirer 1986-1987. The answer to the title question is no.
Dean G, Mather A & Kelly IW (1996). Astrology. In Stein G (ed). Encyclopedia of the Paranormal. Prometheus Books, Amherst NY, pages 47-99 with 15 general references.
Dean G & Kelly IW (2000). Does astrology work? Astrology and skepticism 1975-2000. In Kurtz P (ed). Skepticism: A 25 Year Retrospective. Prometheus Books, Amherst NY, pages 191-207.
Gambrill E (1990). Critical Thinking in Clinical Practice: Improving the Accuracy of Judgements and Decisions about Clients. Jossey-Bass, San Francisco. 432 pages, 660 references. How to reduce reasoning errors in psychology, medicine and the helping professions. Well organised, packed with information, many examples, a few references to astrology (but only to illustrate reasoning errors), each chapter has a summary. Equally applicable to astrology. Should be read by every astrologer.


[As partes desta mensagem que não continham texto foram removidas]



SUBJECT: ASTROLOGY IS BIGOTRY
FROM: "Oraculo" <oraculo@atibaia.com.br>
TO: <ciencialist@yahoogrupos.com.br>
DATE: 15/02/2005 00:48

http://www.pac-c.org/astrology2.htm
_____________________________________________
Trecho:

All quotations in the remainder of this article are from Linda Goodman's Sun Signs; citations are page numbers.

ARIES the Ram, March 21st through April 20th.

The ram is conscious only of himself (4).
His needs come first (4).
Aries is concerned with the world only as it relates to himself (5).
You can look for a liberal attitude, lavish generosity with both time and material things (6).
There's little that's graceful about the ram (6).
The ram can also be the epitome of social grace (10).

Are you getting the idea that if you read enough you will run across virtually all facets of all behavior patterns? Bingo! The above contradictions are separated in Goodman's text, so that you will not stumble upon them without some effort. That is what attracts people to astrology in the first place: the idea that they can get simple and immediate answers to life's complex problems without effort.

Tradução (meia boca..:-)

Aries, março 21 até abril 20
O carneiro é consciente apenas de sí mesmo
Ele necessita vir em primeiro lugar
Aries só se refere ao mundo quando este está relacionado a sí mesmo
Você pode procurar por uma atitude liberal, generosidade pródiga com o tempo e com as coisas materiais.
Pouca coisa é graciosa no carneiro
O carneiro pode ser a epitome da graça social

Você está notando que, se ler o suficiente dos textos e afrimações, poderá encontrar virutalmente todas as facetas de todo tipo de comportamento? Bingo" As contradições acima são separadas no texto de Goodman´s, de modo que não se tropece nelas sem algum esforço. Isto é o que atrai pessoas para a astrologia em primeiro lugar: a idéia que eles podem encontrar respostas simples e imediatas para os complexos problemas da vida sem esforço.
______________________________________

Interessante texto sobre processos mentais na astrologia.

Um abraço.

Homero

[As partes desta mensagem que não continham texto foram removidas]



SUBJECT: A guide for students doing projects - guia para projetos de estudos sobre astrologia
FROM: "Oraculo" <oraculo@atibaia.com.br>
TO: <ciencialist@yahoogrupos.com.br>
DATE: 15/02/2005 00:51

Um guia para projetos de estudos sobre astrologia. No final tem vasta bibliografia sobre o assunto, estudos e desdobramentos. O texto é bem longo.

http://www.rudolfhsmit.nl/t5-tests.htm

What tests are easy?
A guide for students doing projects

By Geoffrey Dean

Can astrology be tested?

As soon as astrologers hear the word "test" they tend to raise objections such as the following:

a.. Astrology (soul stuff) cannot be tested by science (materialism).
b.. Psychology is too confused for its tests to be meaningful.
c.. Proper testing is not possible in the absence of funding.
d.. Testers are closed-minded and cannot be taken seriously.
Ignore such objections. First, if they were true then astrologers could never know anything about astrology. How could they know it works if they could not test it? Second, the objections are irrelevant to the questions that matter, such as are Leos more generous than non-Leos, or do astrologers agree on what a chart means. Third, yes, some tests are difficult but others are easy and can be made by anyone.

Helpful questions

The following questions, even if not part of your eventual project, may help in clarifying your approach to astrology.

To evaluate an idea such as "astrology works", try asking the following questions. The aim is not to win but to learn.

(1) Why do you believe in this idea? This puts the burden of proof on the claimant where it belongs.
(2) What evidence would you accept as proving your idea wrong? This is a potent question because it opposes the tendency to consider only confirming cases.
(3) Are there other explanations that would produce the same outcome? This too is a potent question because it looks at how informed the claimant is.

Avoid open ended questions. Ask not "how does astrology work?" but "what is meant by astrology?" so you know what the issues are (astrologers seldom agree on this or anything else, so the answer will depend on who you talk to). Not "is astrology true?" but "to what extent is it true?" or "does it need to be true?" The key question is "does astrology deliver benefits beyond those due to non-astrological factors?", but this cannot be answered until the other questions have been answered. Do not expect to find this question answered in astrology books.

Projects about sun signs

Signs are the most researched topic in astrology with well over one hnndred empirical studies. Most studies are simply counts of people born under various signs, but such counts are too contaminated by ordinary influences (astronomy, sampling, demography, age incidence) to mean anything. We can try correcting for such influences but in practice the uncertainty is too great. So forget counting sun signs. If you need convincing, read this website's critiques of Sachs's Astrology File.

The remaining studies, if adequately controlled against non-astrological influences, have invariably been negative. Signs are not only the most researched topic in astrology but are also the most disconfirmed. Signs are simply not valid, not even slightly. So forget validation, including validation of sun sign compatibility. Turn instead to looking at people's reactions towards an extremely popular belief now known (but not widely known) to be untrue.

Sun signs seem valid because they cannot fail to fit (everyone, not just Leos, will admit to being generous). They are also a good topic of conversation and are nice to have around. People tend to like them, but exactly why is not well understood despite many speculations. So one easy and useful project might be to ask people why they like or dislike sun signs, and to find reasons for any differences. For example men may view sun signs differently from women.

Sun signs let you study people's reactions to disconfirmation. Assemble a roomful of people, sit them in a circle according to sun sign (this adds intrigue and keeps the group focussed), and record each person's level of belief in sun signs (put a scale on display for them to choose from, such as strong, moderately strong, and so on). Ask for opinions why anyone should believe or disbelieve in sun signs, and get a debate going. Observe the arguments and record how each person reacts to views opposing their own. After a suitable period stop the debate and again record each person's level of belief. Ask why they changed or why they didn't change. Next, raise the question that nobody will have raised, namely what evidence would each person accept as disproving sun signs. Record their answers. Then summarise the evidence from this website about sun signs, and restart the debate. After a suitable period stop the debate and again record each person's level of belief. Rationally, everyone should end up as disbelievers, but most likely the believers will carry on believing. Ask them why. You now have the makings of an absorbing essay for your social science teacher.

Belief in sun signs is known to slightly affect a person's self-image and in some cases even their choice of job. The daily horoscope may well have an effect on daily mood. But measuring these things requires large samples, careful techniques, and lots of time, which puts them well beyond the resources of most projects. But there is a much easier way of getting useful insight into these things, namely by talking to people who take sun sign columns very seriously and cannot exist without them. Such people comprise about 1% of the general population and may be hard to find, but your newsagent may know a few. This is an unexplored area that deserves attention.

Finally two easy projects that have been done before but are still informative and fun. (1) See if people can pick their own newspaper horoscope after you have mounted each one separately and removed all cues (they may need retyping). If picking is difficult, have people rank their top three or four choices. (2) Count agreements and disagreements between horoscopes from different newspapers or women's magazines. If you want more mileage, do a repeat but mixing the days so the horoscopes being compared are not for the same day. Then get others to repeat the exercise to see if their counts agree with yours. In each case it is a good idea to have a trial run to make sure there are no snags.

Projects about serious astrology

The bad news is that many tests are difficult, time consuming, and have already been done. For example testing whether an astrologer can identify people from their charts has already been done in 45 studies, some of which took several years to complete. Overall 600 astrologers and more than 1100 charts were tested but the results showed no support for astrology. (The support shown by some early tests was later found to be an artifact of sampling.) Another 20 studies involving nearly 500 subjects tested whether people can pick their own chart reading but again the results showed no support for astrology.

In general the better the study the more negative the outcome. The problem is that the sheer weight of studies will not be overturned by adding one more study. The good news is that other studies are much easier, just as useful, and more fun. And anyone can do them.

For example if you change the topic from seeing if astrologers are accurate to seeing if astrologers agree, the test becomes almost ridiculously easy. Just sit astrologers together in the same room, put a succession of charts on a screen, and ask questions that are a mixture of easy (is this person outgoing?) and difficult (does this person own a cat?). Any charts will do, even invented ones, because it is irrelevant what the person is actually like. What matters is whether the astrologers agree on what the chart is saying, and whether their agreement varies with difficulty.

To be feasible the above test requires access to a local astrology group. But it is worth the bother. Notice how all the usual excuses for error (birth time not accurate enough, person does not know themself, and so on) no longer apply, since accuracy is not a concern. The point is, if astrologers cannot agree on what a chart is saying, then what price astrology? So far 25 such tests have been made involving a total of 500 astrologers and 700 charts, but their agreement was scarcely better than no agreement. Do not expect to find this result mentioned in astrology books.

A popular but interesting test is pretending to obtain chart readings for a roomful of people, who rate each statement in the reading for accuracy and then indicate to what extent the reading has increased their belief in astrology. Each reading is preceded by the person's name and birth details, so it looks genuine, but in fact each person gets the same fake reading. Alternatively a different fake reading can be provided for every 5-10 people, which allows you to test more statements without lengthening the individual readings. The aim is to show people how easy it is to read specifics into generalities (this is called the Barnum effect), how this affects their belief level, and to compile a reading that will fit as many people as possible.

Here the sense seems to come from the reading but in fact it comes from our ability to make sense out of vague data, as when we see faces in clouds. The most readily-accepted Barnum statements are favourable (you are forceful and well-liked by others), or vague (you enjoy a certain amount of change and variety), or two-headed (you are generally cheerful but get depressed at times). Not unexpectedly, the result of accepting Barnum statements is an increase in belief. The Barnum effect has attracted around 70 studies but it still makes a telling point.

The fallacy of personal validation

Interestingly, the most famous Barnum study, made in 1949, and many studies since then, used 13 statements taken mostly from an astrology book. The statements were typical of what might be heard during a chart reading. The studies found that when the statements are presented as a personal reading, as would of course apply during a chart reading, most people rate their accuracy as good or excellent. In case you are interested, the statements are as follows:

(1) You have a great need for other people to like and admire you.
(2) You have a tendency to be critical of yourself.
(3) You have a great deal of unused capacity which you have not turned to your advantage.
(4) While you have some personality weaknesses, you are generally able to compensate for them.
(5) Your sexual adjustment has presented problems for you.
(6) Disciplined and self-controlled outside, you tend to be worrisome and insecure inside.
(7) At times you have serious doubts as to whether you have made the right decision or done the right thing.
(8) You prefer a certain amount of change and variety and become dissatisfied when hemmed in by restrictions and limitations.
(9) You pride yourself as an independent thinker and do not accept others' statements without satisfactory proof.
(10) You have found it unwise to be too frank in revealing yourself to others.
(11) At times you are extroverted, affable, sociable, while at other times you are introverted, wary, reserved.
(12) Some of your aspirations tend to be pretty unrealistic.
(13) Security is one of your major goals in life.

Taken from Forer BR, The fallacy of personal validation: A classroom demonstration of gullibility. Journal of Abnormal and Social Psychology 1949, 44, 118-123. The "personal validation" in the title refers to the process of looking at a statement, seeing that it fits, and concluding that the system works. Just as astrologers do every day.

For example look at volume 10 pages 25-27 of Noel Tyl's Principles and Practice of Astrology, Llewellyn 1975, where he gives a reading to show how astrological counselling should be conducted. Tyl, a famous American astrologer and psychology graduate of Harvard University, is renowned among astrologers for the quality of his work, so we can take this as being representative of the best practice. Tyl is talking to Eric, a new client aged 24. Comments have been added in [ ].

"Basically, we should begin with a general statement to differentiate you from ... all other people. The horoscope shows that you are gracious, friendly, that you express yourself softly [all this was already obvious, now comes the favourable Barnum], and that you're very sensitive. [Now the vague Barnum] But, Eric, there are tensions here ... linked to self-esteem, how you feel about yourself. [Now the double-headed Barnum] The horoscope suggests that, on the one hand, you're pretty particular about whom you relate to. [Now repeat vague Barnum] This might be a defense because you're not too pleased with yourself. [Now back to double-headed Barnum] And, then on the other hand, you're friendly to the extreme in order to be accepted. Do you think that is accurate?" To which Eric replied "Yeah (smiling). That's it, right on the button. [But how could it be otherwise?] Gee, I know a little about Astrology; how do you see all that?"

Sources of information

If your project requires a review of the literature, remember that books which examine the truth about astrology can be hard to find. For every book there are dozens of articles that can be even harder to find. Selected book titles are given below. Most address astrology as a whole with sun signs on the side. Be aware that works earlier than 1990 tend to be dated.

Ankerberg J & Weldon J (1989). Astrology: Do the Heavens Rule Our Destiny? Harvest House, Eugene OR 97402, 334 pages with 356 references but no index. Powerful and very readable critique by Bible scholars who have done their homework (the Christian perspective does not intrude). Many useful quotes, all referenced to the exact page.

Blackmore S & Hart-Davis A (1995). Test Your Psychic Powers: Find Out the Truth for Yourself. Thorsons, London. Sets out simple experiments for testing sun sign columns and nine other paranormal areas (telepathy, crystals, dreams, dowsing, pendulums, premonitions, psychokinesis, ouija boards, and palmisty) that you might find more appealing! Blackmore was then a leading skeptic and investigator of the paranormal.

Culver RG & Ianna PA (1988). Astrology: True or False? A Scientific Evaluation. Prometheus Books, Buffalo NY, 228 pages with over 200 references. Minor update of the original 1979 version. A clear and very readable critique by astronomers, much useful information including test results, a classic survey of astrological prediction, and ten tests issued as a challenge to astrologers (no volunteers to date), but lacks the psychological perspective of Eysenck & Nias (1982). Concludes that astrology is neither scientifically sound nor scientifically useful.

Dean G, Mather A, & 52 others (1977). Recent Advances in Natal Astrology: A Critical Review 1900-1976. Analogic, Perth Western Australia, 608 pages with 1010 references. Out of print but several copies are held by the library of the Astrological Association. Much is now very dated, and most of the positive findings have subsequently been overturned. All the important studies are in Eysenck & Nias (1982).

Dean G (1992). Does astrology need to be true? In Frazier K (ed). The Hundredth Monkey and other paradigms of the paranormal. Prometheus Books, Buffalo NY, pages 279-319 with 126 references. The answer to the title question is no.

Dean G, Mather A & Kelly IW (1996). Astrology. In Stein G (ed). Encyclopedia of the Paranormal. Prometheus Books, Amherst NY, pages 47-99 with 15 general references. A comprehensive scientific survey covering history, popularity, arguments for and against, conceptual problems, controlled tests, effect size comparisons, problems of birth chart interpretation, how belief in astrology arises, role of human judgement biasses, and the future of astrology. But at USD149.95 the encyclopedia is affordable only by libraries.

Dean G & Kelly IW (2000). Does astrology work? Astrology and skepticism 1975-2000. In Kurtz P (ed). Skepticism: A 25 Year Retrospective. Prometheus Books, Amherst NY. Non-technical with minimum references. Reviews the progress of scientific investigations of astrology since 1975.

Dean M (1980). The Astrology Game. Beaufort, New York, 360 pages with nearly 200 references. About one fifth is a useful critique of the pop astrology industry. The rest, now dated, tries to establish evidence for astrology. Predicts (wrongly) that a new astrology is being born.

Eysenck HJ & Nias DKB (1982). Astrology: Science or Superstition? St Martin's Press, New York, 244 pages with 230 references. Also in Penguin but out of print. Best single source on research up to 1982. Clear, sympathetic and very readable critique by psychologists. Concludes that everything is negative except the Gauquelin results. Now somewhat dated but still valid, later research is even more negative.

Gauquelin M (1979). Dreams and Illusions of Astrology. Prometheus Books, Buffalo NY, 158 pages with 125 references and notes but no index. Originally published in French 1969. Mostly a survey of Gauquelin's early non-positive results. Concludes that "since the most painstaking studies have shown the inanity of horoscopes, there should be a strong rising up against this exploitation of public credulity" (p.158).

Gauquelin M (1983). The Truth about Astrology. Basil Blackwell, London, 202 pages. The title was imposed by the publisher. Published in the USA as Birthtimes by St Martin's Press, New York. The best single source on the Gauquelin work but includes little on non-Gauquelin work and of course nothing on the important later developments. Also in Hutchinson paperback 1984. Details of sources occupy 15 pages.

Hines T (1988). Pseudoscience and the Paranormal: A Critical Examination of the Evidence. Prometheus Books, Buffalo NY. Astrology (along with lunar effects and biorhythms) is on pages 141-164. Now in 2nd edition.

Martens R & Trachet T (1998). Making Sense of Astrology. Prometheus Books, Amherst NY, 276 pages with 140 references. A critique by two prominent Belgian skeptics. Looks in detail at each astrological idea (signs, houses, aspects, etc) and then at some of the empirical evidence. Includes important studies published since Eysenck & Nias (1982). Concludes that the final judgement "is without any doubt negative". Nevertheless astrology is unlikely to go away.

Phillipson G (2000). Astrology in the Year Zero. London: Flare Publications. See http://www.flareuk.com. Explores key issues via interviews with astrologers and researchers. Includes a review of recent research findings, a look at human judgement errors, and a critical bibliography, all with expanded versions on this website.

Pottenger M ed (1995). Astrological Research Methods Volume 1. ISAR, PO Box 38613, Los Angeles CA 90038-8613, 466 pages. Not a critique but included here because it is the only available anthology of articles on research methods. Unfortunately there are frequent opposing views that are left unresolved, and there is no mention of faulty reasoning. But the authors include scientists as well as astrologers, quite a few articles convey the scientific spirit, and in general the anthology represents a major advance in astrological thinking.

Rae AC (1992). Bluff Your Way in Astrology & Fortune Telling. Ravette Books, Horsham, West Sussex, 62 pages. A wonderfully cynical guide to being an expert without having to know or study anythng. Includes Chinese astrology, Tarot, I Ching, palmistry, and runes.

Sachs G (1998). The Astrology File: Scientific Proof of the Link between Star Signs and Human Behavior. London: Orion Books. Occasionally books appear that claim to validate sun signs. But in every case a critical examination has revealed mistakes and procedural blunders, of which this book is a best-selling example. The author uses samples of up to several millions of cases, which are so huge that even the most trivial of errors (for example, in matching the sample to controls) become enormously inflated in statistical significance. So his conclusions are misleading. Otherwise, the book is readable and well set out. Three critiques that expertly reveal its mistakes and procedural blunders are on this website.

Standen A (1977). Forget Your Sun Sign: An Outline of Antiastrology. Legacy Publishing, Baton Rouge LA, 135 pages with 33 references. Very readable but now very dated. The author is a scientist who studied astrology "very thoroughly, and came out an antiastrologer. This book is to explain the reasons why." His reasons apply even more today.

Stewart JV (1996). Astrology: What's really in the stars. Prometheus Books, Buffalo NY, 156 pages 194 references. First two-thirds looks at early sources including Babylonian omen tablets (many photographs), the ancient Greeks, and Ptolemy. Rest is sketchy and lacking in detail.

Van de moortel K (2002). Astro-Logics. Self-published, Ghent. 156 pages, 109 references. See http://www.astrovdm.com. A critical look at astrology by a leading writer of astrological software. Very readable and largely nontechnical. Reveals the disagreements and inconsistencies behind almost every kind of chart factor and argues that only research can get out of this mess. Gives a selection of research results, most of them negative, and some basic guidelines for doing research, but does not address faulty reasoning.




[As partes desta mensagem que não continham texto foram removidas]



SUBJECT: Re: [ciencialist] Foucault / Luz ondas ou corpusculos.
FROM: "Alberto Mesquita Filho" <albmesq@uol.com.br>
TO: <ciencialist@yahoogrupos.com.br>
DATE: 15/02/2005 06:47

----- Original Message -----
From: "murilo filo"
Sent: Monday, February 14, 2005 9:17 PM
Subject: [ciencialist] Foucault / Luz ondas ou corpusculos.

> Oi!
> Não é por nada não, mas eu repito esta msg do dia 11, e só por uma razão!
> Gostaria, honestamente, que algum sapiente da lista comentasse algo,
> confrontado com o que tem sido o atual conhecimento.

> >Há uns 45 anos, um tio meu, fotógrafo altamente profissional e de 2ª
> >geração, contou-me que quando êle queria tirar uma foto CONTRA A LUZ DO
> >SOL, ÊLE USAVA O FLASH. E o flash evitava a indesejada sensibilização do
> >filme e a perturbação na lente. O flash, mais forte, ''empurrava'' a luz
> >solar... coisa linda! Nunca mais ouví alguém comentar sobre isto, que
> >aprendí nos meus 15 anos. Para mim, leigo, isto é uma prova de que a luz
> >é *matéria*, queiram ou não!
> >Alguém comenta? abr/Murilo SP 11/fev

Sinceramente, nunca ouvi nada a esse respeito. Seria interessante que os
"físicos ondulatórios" comentassem alguma coisa a respeito dessa possível
"interferência" e/ou que reproduzissem a experiência, ainda que com a idéia
de falseá-la. De minha parte, não tenho dúvidas de que a luz é corpuscular
(corpúsculo clássico e não aquela coisa estratosférica que chamam por fóton
e que ninguém sabe o que é) mesmo sem nunca ter preciso lançar mão dessa
experiência que, por sinal, desconhecia.

[ ]´s
Alberto
http://ecientificocultural.com/indice.htm
Mas indiferentemente a tudo isso, o neutrino tem massa, o elétron não é
uma carga elétrica coulombiana e a Terra se move. E a história se repetirá.



SUBJECT: Fw: a menor part�cula
FROM: "Luiz Ferraz Netto" <leobarretos@uol.com.br>
TO: "ciencialist" <ciencialist@yahoogrupos.com.br>
DATE: 15/02/2005 07:41

cuímicos .......
[]'
===========================
Luiz Ferraz Netto [Léo]
leobarretos@uol.com.br
http://www.feiradeciencias.com.br
===========================
-----Mensagem Original-----
De: Bialas
Para: leobarretos@uol.com.br
Enviada em: segunda-feira, 14 de fevereiro de 2005 15:32
Assunto: a menor partícula


Oi, me chamo Guiomar e voltei a estudar Quimica. Estou confusa. No ano passado ouvi uma reportagem sobre uma particula menor que o átomo. Por gentileza, eu poderia receber mais informações. Obrigada.


--------------------------------------------------------------------------------


No virus found in this incoming message.
Checked by AVG Anti-Virus.
Version: 7.0.300 / Virus Database: 265.8.7 - Release Date: 10/02/2005

----------

No virus found in this outgoing message.
Checked by AVG Anti-Virus.
Version: 7.0.300 / Virus Database: 265.8.7 - Release Date: 10/02/2005


[As partes desta mensagem que não continham texto foram removidas]



SUBJECT: Re: [ciencialist] Fw: a menor partícula
FROM: "E m i l i a n o C h e m e l l o" <chemelloe@yahoo.com.br>
TO: <ciencialist@yahoogrupos.com.br>
DATE: 15/02/2005 09:11

Mapa Conceitual das partículas
http://www.ucs.br/ccet/defq/naeq/material_didatico/textos_interativos_24.htm

[ ] 's do Emiliano Chemello
emiliano@quimica.net
http://www.quimica.net/emiliano
http://www.ucs.br/ccet/defq/naeq

" Rien ne se perd, rien ne se crée,
tout se transforme."

Antoine Laurent de Lavoisier (químico francês, 1743 - 1794)

----- Original Message -----
From: Luiz Ferraz Netto
To: ciencialist
Sent: Tuesday, February 15, 2005 6:41 AM
Subject: [ciencialist] Fw: a menor partícula


cuímicos .......
[]'
===========================
Luiz Ferraz Netto [Léo]
leobarretos@uol.com.br
http://www.feiradeciencias.com.br
===========================
-----Mensagem Original-----
De: Bialas
Para: leobarretos@uol.com.br
Enviada em: segunda-feira, 14 de fevereiro de 2005 15:32
Assunto: a menor partícula


Oi, me chamo Guiomar e voltei a estudar Quimica. Estou confusa. No ano
passado ouvi uma reportagem sobre uma particula menor que o átomo. Por
gentileza, eu poderia receber mais informações. Obrigada.





SUBJECT: Re: luz velocidade e energia
FROM: "rayfisica" <rayfisica@yahoo.com.br>
TO: ciencialist@yahoogrupos.com.br
DATE: 15/02/2005 09:55


--- Em ciencialist@yahoogrupos.com.br, "Sergio M. M. Taborda"
<sergiotaborda@t...> escreveu
> rayfisica wrote:
>
> >
> > Lendo e meditando sobre a questão iniciada por Sr. Helio,
sobre a
> > velocidade da luz na água ser menor que no vácuo me ocorreu
o
> > seguinte.
> > Algum tempo atrás o Sr. Taborda explicou-me que a luz interage
com a
> > gravidade por ter energia, pergunto então:
> > Essa energia esta relacionada com a velocidade (C)?
>
> Não. A energia da luz é proporcional à sua frequencia e
não à sua
> velocidade. A energia de um fotão é hf sendo h a constante de
plank
e f
> a frequencia.
> a freqeuncia relaciona-se com o comprimento de onda e a velocidade
da
> luz c = a f , sendo a o comprimento de onda.
>
> > Na água a luz tem mais, menos ou igual energia que no vácuo?
>
>
> Tem a mesma. (Lei da Conservação da Energia + Lei da
Conservação do
> Momento)
>
> > Se menos o que acontece coma energia extra quando o fóton
entra na
> > água (alem de se molhar) ou um meio semelhante?
>
> A velocidade num meio qualquer é relacionada com a velocidade da
luz no
> vácuo pelo indice de refracção n. Que pode ser medido
experimentalmente
> para cada meio.
> A velocidade da luz num meio qualquer, V,e relacionada coma a
velocidade
> no vácuo por v = c/n. n é um numero igual ou maior que 1.
> Para o vácuo n=1 e v =c
> Para a agua n > 1 e v < c.
> A energia permanece constante, pelas leis de conservação.
Então o
que se
> altera ?
> O comprimento de onda. As contas.
>
> Se v = a f , e v = c / n , c = a f n
> Se E = hf , f = E /h
>
> então c = a E n/ h
> E , c e h são constantes. Então pela variação de n, temos
a
variação de a.
> Por isso que ao passar no prisma a luz se divide em cores. (isto tb
pode
> ser intrepretado como um n diferente para cada cor)
>
> > O que acelera o fóton quando sai desse meio? Ou de onde vem a
energia
> > de aceleração?
>
> Não existe um processo de aceleração.
> Para entender isto melhor é preciso o conceito de velocidade dde
fase e
> de grupo.
> A velocidade de grupo é aquela que representa o "movimento" nos
mesmo
> moldes que aplicamos a objectos macroscopicos.
> A velocidade de fase é apenas a relação entre o comprimento
de onda
e a
> frequencia.
> A velocidade que se altera de um meio para o outro é a
velocidade
de
> fase. E não a de grupo.
> Quando vc pensa que o aumento da velocidade aumenta a energia, vc
está
> pensando que o aumento da velocidade de grupo aumenta a energia
cinetica.
> Sim, isso seria verdade, se a energia cinetica da luz fosse
váriável.
> Ela não é . Ela é constante. ( E = hf )
>
> > O que acontece se colocarmos um corpo negro imerso no meio onde a
luz
> > tem velocidade menor (pois se a luz tem menos energia que no
vácuo
> > logo o mesmo quantum não pode excitar o elétron)?
>
> Não. A energia não depende da velocidade.
>
> > Se a energia é a mesma na água, por exemplo, que no
vácuo como se
> > explica isso? Velocidade diferente mesma energia, não ta
faltando
> > alguma coisa?
>
> Não. É que a velocidade muda, mas a energia não, pq a
enegia não
depende
> da velocidade.
> Essa é a grande sacada da luz. E por isso ela é tão
especial.
>
> > A relação da luz na água é diferente do que no vácuo
com relação à
> > gravidade (interage menos)?
>
> Não conheço nenhuma teoria sobre isso.
>
> Sergio Taborda
>
>
> --
> No virus found in this outgoing message.
> Checked by AVG Anti-Virus.
> Version: 7.0.300 / Virus Database: 265.8.8 - Release Date: 14-02-
2005

>>>>>>>>>>>>>>>>>>>>>>>>>>>>>>>>>
>>>>>>>>>>>>>>>>>>>>>>>>>>>>>>>>>>

Muitissimo obrigado.
Como sempre simples e objetivo.
Se eu entendi bem poderíamos dizer que para compensar a
diminuição da
velocidade, a natureza compensa com o aumento da freqüência.
Legal! Então é por isso que as leis das conservações são
tão
importantes.
Apenas para materializar o meu entendimento, se eu estivesse
representando a luz através da tela de um osciloscópio, tanto
no
vácuo quanto na água a velocidade do traço seria o mesmo,
porem no
vácuo eu teria menos cristas e vales logo o comprimento da
região
varrida seria maior, pelo mesmo período de tempo do que na água

que eu teria menos cristas e vales, mantendo se a amplitude, ok?
Mas aqui eu peço socorro. (talvez seja até outro assunto,
portanto
peço a caridade da paciência comigo).
É muito difícil pra eu entender isso, pois posso entender uma
onda
por ser uma alternância entre vales e cristas, por exemplo, se eu
estiver no mar sobre uma prancha é só contar as cristas durante
um
período de tempo que eu sei a freqüência.
Por exemplo, um imã em barra infinitamente longo (monopolo) no
espaço
movendo se para frente e para traz produz em qualquer outro monopolo
uma atração ou repulsão com intensidade que apenas depende da
distancia e do fato de serem semelhantes ou opostos (Coulomb).
Portanto se o segundo monopolo estiver relativamente imóvel em
relação ao primeiro, quando mais próximo produz uma crista e
quando
mais distante um vale (convenção) isso em função do tempo
teremos uma
freqüência, e eu no segundo monopolo poderia dizer a freqüência
do
primeiro ainda que não visse ele, porem manteve se a freqüência.
E se substituirmos o segundo monopolo por um elétron seria apenas
respeitar a regra da mão...
Porem com a luz através de um meio semelhante à água
parece-me que é
como se a fonte estivesse vibrando em uma frequencia diferente da
real, como entender isso? E porque isso ocorre?






SUBJECT: Re: Foucault / Luz ondas ou corpusculos.
FROM: Hélio Ricardo Carvalho <hrc@fis.puc-rio.br>
TO: ciencialist@yahoogrupos.com.br
DATE: 15/02/2005 14:47


Para Murilo,


"murilo filo" escreveu:
> Oi!
> Não é por nada não, mas eu repito esta msg do dia 11, e só por uma
razão!
> Gostaria, honestamente, que algum sapiente da lista comentasse
algo, ...
> >Há uns 45 anos, um tio meu, fotógrafo altamente profissional e de

> >geração, contou-me que quando êle queria tirar uma foto CONTRA A
LUZ DO
> >SOL,
> >ÊLE USAVA O FLASH. E o flash evitava a indesejada sensibilização
do filme e
> >a perturbação na lente. O flash, mais forte, ''empurrava'' a luz
solar...
> >coisa linda!
> >Nunca mais ouví alguém comentar sobre isto, que aprendí nos meus
15 anos.
> >Para mim, leigo, isto é uma prova de que a luz é *matéria*,
queiram ou não!
> >...

Apesar de ser defensor da luz corpuscular (acho que tudo é matéria),
creio que as coisas não acontecem do jeito que seu tio acha.
Não sou especialista em fotografia mas, na minha opinião, o que
acontece é diminuição do contraste da imagem.
Quando se torna mais claro as outras partes da cena a luz do sol
perde um pouco a importância.

Quando corpusculos de luz vão de encontro a outros, a chances de se
chocarem são ínfimas (desprezíveis) pois apesar de serem
tremendamente numerosos são muitíssimos pequenos. :-)
Este último parágrafo é apenas uma hipótese de minha autoria.
:-) :-)

[ ]'s
Hélio






SUBJECT: [RN] MATEMATICA INDEPENDE DE GRAMATICA
FROM: "rmtakata" <rmtakata@altavista.net>
TO: ciencialist@yahoogrupos.com.br
DATE: 15/02/2005 16:29


BBC BRASIL, 15-02-2005

PENSAMENTO NAO DEPENDE DE LINGUAGEM, DIZ PESQUISA

Uma equipe de pesquisadores da Universidade de Sheffield demonstrou
que pacientes que perderam a capacidade de compreender a gramática
ainda são capazes de realizar operações aritméticas complexas.

Os pesquisadores acompanharam três pacientes que sofriam de afasia
aguda - eles haviam perdido a capacidade de compreender ou de produzir
linguagem gramaticalmente correta.

Eles compreendiam, por exemplo, o significado das palavras "leão",
"caçou" e "homem", mas não eram capazes de diferenciar a frase "O leão
caçou o homem" da sentença "O homem caçou o leão".

Mas os pacientes não tiveram quaisquer problemas para realizar
operações matemáticas como 52 menos 11 e 11 menos 52. (e)

http://www.bbc.co.uk/portuguese/ciencia/story/2005/02/050215_linguagembg.shtml

[]s,

Roberto Takata





SUBJECT: Re: Foucault / Luz ondas ou corpusculos.
FROM: César A. K. Grossmann <cesarakg@bol.com.br>
TO: ciencialist@yahoogrupos.com.br
DATE: 15/02/2005 17:07


--- Em ciencialist@yahoogrupos.com.br, "Alberto Mesquita Filho" >
> > >Há uns 45 anos, um tio meu, fotógrafo altamente profissional e de 2ª
> > >geração, contou-me que quando êle queria tirar uma foto CONTRA A
LUZ DO
> > >SOL, ÊLE USAVA O FLASH. E o flash evitava a indesejada
sensibilização do
> > >filme e a perturbação na lente. O flash, mais forte,
''empurrava'' a luz
> > >solar... coisa linda! Nunca mais ouví alguém comentar sobre isto, que
> > >aprendí nos meus 15 anos. Para mim, leigo, isto é uma prova de
que a luz
> > >é *matéria*, queiram ou não!
> > >Alguém comenta? abr/Murilo SP 11/fev

Uma perguntinha para o fotógrafo: quando ele liga o flash, a regulagem
da máquina fotográfica permanece a mesma ou automaticamente o
obturador fica um pouco menor? Sei que para as máquinas comuns,
aquelas que os não profissionais usam, existe uma compensação
automática, que acontece quando você liga o flash. Em máquinas
profissionais, existem aquelas que tem várias velocidades para
obturador, e uma especial para quando se usa flash.

Seria interessante ter mais dados sobre a experiência: máquina
(fabricante e modelo), flash, regulagem, ASA do filme, e duas fotos
seguidas do mesmo objeto nas condições de 'contra a luz do Sol', uma
com o flash e outra sem o flash.

[]s
--
.O. Cesar A. K. Grossmann ICQ UIN: 35659423
..O http://www.LinuxByGrossmann.cjb.net/
OOO Timeo Danaos, et dona ferentes. (Virgilio)





SUBJECT: Re: Mais um hoax?..:-) (era: Foucault / Luz ondas ou corpusculos.)
FROM: "rayfisica" <rayfisica@yahoo.com.br>
TO: ciencialist@yahoogrupos.com.br
DATE: 15/02/2005 17:34


--- Em ciencialist@yahoogrupos.com.br, "Oraculo" <oraculo@a...>
escreveu
> Olá
>
medida do fotometro).
>
> Se fosse minimamente real, já teria sido feito ou pelo menso
percebido pelos que estudam, a décadas, a luz e sua física.
>
> Um abraço.
>
> Homero
>
>
>
>
>>>>>>>>>>>>>>>>>>>>>>>>>>>>>>>>>>>>>>>>
>>>>>>>>>>>>>>>>>>>>>>>>>>>>>>>>>>>>
O negocio é tão simples, mas tão simples que por isso mesmo não foi
nunca com certeza tentado.
Simplesmente por medo do ignobel.
Mas seria muito interessante que antes de abandonar a idéia como algo
tão tolo que não deva perder se tempo de verificar...
Ai alguém no futuro dirá:
&#61656; Se fosse minimamente real, já teria sido feito ou pelo menso
percebido pelos que estudam, a décadas, a luz e sua física.






SUBJECT: Mais um hoax?..:-) (era: Foucault / Luz ondas ou corpusculos.)
FROM: "Oraculo" <oraculo@atibaia.com.br>
TO: <ciencialist@yahoogrupos.com.br>
DATE: 15/02/2005 18:27

Olá

Essa estória do flash e da foto que "prova" a teoria corpuscular da luz parece algo destinado a se tornar mais uma lenda urbana. Como diversos mitos, surge sem informação suficiente, tem pouca possibilidade de ser real, quase não pode ser testada (a estória, não o experimento..:-), e ninguém ouviu falar dela no início.

Mas com algum tempo, vai começar a ser citada a partir das citações iniciais, até se tornar algo que "todo mundo sabe" ou "todo mundo ouviu falar".

O questionamento do César é perfeito, e todas essas perguntas deveriam ter sido feitas no momento da experiência. E as variáveis controladas antes de qualquer conclusão. Mesmo eu, que não sou fotógrafo, pensei que havia algo de errado no experimento, assim que li a mensagem, já que diversos outros fatores poderiam influenciar o resultado, inclusive a distorção perceptiva de seres humanos (do tipo, acho que ficou assim ou assado, mas pode ser apenas minha impressão..:-)

O experimento seria simples demais de ser feito, para se manter tão incónito. Um sensor de luz, como os dos fotógrafos profissionais, seria posicionado diretamente no alcance de um facho de luz. Uma lanterna (ou flash) seria posicionado de forma a "empurrar" esse facho de luz. E o sensor mediria a diminuição sem problemas (se o flash pode impedir que o Sol sensibilize o filme, pode diminuir a medida do fotometro).

Se fosse minimamente real, já teria sido feito ou pelo menso percebido pelos que estudam, a décadas, a luz e sua física.

Um abraço.

Homero




----- Original Message -----
From: César A. K. Grossmann
To: ciencialist@yahoogrupos.com.br
Sent: Tuesday, February 15, 2005 4:07 PM
Subject: [ciencialist] Re: Foucault / Luz ondas ou corpusculos.



--- Em ciencialist@yahoogrupos.com.br, "Alberto Mesquita Filho" >
> > >Há uns 45 anos, um tio meu, fotógrafo altamente profissional e de 2ª
> > >geração, contou-me que quando êle queria tirar uma foto CONTRA A
LUZ DO
> > >SOL, ÊLE USAVA O FLASH. E o flash evitava a indesejada
sensibilização do
> > >filme e a perturbação na lente. O flash, mais forte,
''empurrava'' a luz
> > >solar... coisa linda! Nunca mais ouví alguém comentar sobre isto, que
> > >aprendí nos meus 15 anos. Para mim, leigo, isto é uma prova de
que a luz
> > >é *matéria*, queiram ou não!
> > >Alguém comenta? abr/Murilo SP 11/fev

Uma perguntinha para o fotógrafo: quando ele liga o flash, a regulagem
da máquina fotográfica permanece a mesma ou automaticamente o
obturador fica um pouco menor? Sei que para as máquinas comuns,
aquelas que os não profissionais usam, existe uma compensação
automática, que acontece quando você liga o flash. Em máquinas
profissionais, existem aquelas que tem várias velocidades para
obturador, e uma especial para quando se usa flash.

Seria interessante ter mais dados sobre a experiência: máquina
(fabricante e modelo), flash, regulagem, ASA do filme, e duas fotos
seguidas do mesmo objeto nas condições de 'contra a luz do Sol', uma
com o flash e outra sem o flash.

[]s
--
.O. Cesar A. K. Grossmann ICQ UIN: 35659423
..O http://www.LinuxByGrossmann.cjb.net/
OOO Timeo Danaos, et dona ferentes. (Virgilio)





##### ##### #####

Para saber mais visite
http://www.ciencialist.hpg.ig.com.br


##### ##### ##### #####


Yahoo! Grupos, um serviço oferecido por:







------------------------------------------------------------------------------
Links do Yahoo! Grupos

a.. Para visitar o site do seu grupo na web, acesse:
http://br.groups.yahoo.com/group/ciencialist/

b.. Para sair deste grupo, envie um e-mail para:
ciencialist-unsubscribe@yahoogrupos.com.br

c.. O uso que você faz do Yahoo! Grupos está sujeito aos Termos do Serviço do Yahoo!.



[As partes desta mensagem que não continham texto foram removidas]



SUBJECT: Re: [ciencialist] P/ Esteban ( Zodiaco e Acupuntura..)
FROM: "Esteban Moreno" <estebanmoreno@idhi.org.br>
TO: <ciencialist@yahoogrupos.com.br>
DATE: 15/02/2005 19:39

Salve Oráculo!
Pegue está sua frase: "As leis da termodinâmica derivam de estudos,
evidencias, experimentos, resultados, etc, de centenas de anos, com
excelente demonstração de eficácia e de precisão. ", substitua
"termodinâmica" por "astrologia" e "centenas" por "milhares".
Um abraço,
Esteban.
.



SUBJECT: Re: [ciencialist] ASTROLOGY IS BIGOTRY
FROM: "Esteban Moreno" <estebanmoreno@idhi.org.br>
TO: <ciencialist@yahoogrupos.com.br>
DATE: 15/02/2005 19:41

O texto é bem conveniente aos seus propósitos. Por que não experimenta enviar algo que relacione o sol no signo + casa + aspectos.
Esteban.

----- Original Message -----
From: Oraculo
To: ciencialist@yahoogrupos.com.br
Sent: Monday, February 14, 2005 11:48 PM
Subject: [ciencialist] ASTROLOGY IS BIGOTRY


http://www.pac-c.org/astrology2.htm
_____________________________________________
Trecho:

All quotations in the remainder of this article are from Linda Goodman's Sun Signs; citations are page numbers.

ARIES the Ram, March 21st through April 20th.

The ram is conscious only of himself (4).
His needs come first (4).
Aries is concerned with the world only as it relates to himself (5).
You can look for a liberal attitude, lavish generosity with both time and material things (6).
There's little that's graceful about the ram (6).
The ram can also be the epitome of social grace (10).

Are you getting the idea that if you read enough you will run across virtually all facets of all behavior patterns? Bingo! The above contradictions are separated in Goodman's text, so that you will not stumble upon them without some effort. That is what attracts people to astrology in the first place: the idea that they can get simple and immediate answers to life's complex problems without effort.

Tradução (meia boca..:-)

Aries, março 21 até abril 20
O carneiro é consciente apenas de sí mesmo
Ele necessita vir em primeiro lugar
Aries só se refere ao mundo quando este está relacionado a sí mesmo
Você pode procurar por uma atitude liberal, generosidade pródiga com o tempo e com as coisas materiais.
Pouca coisa é graciosa no carneiro
O carneiro pode ser a epitome da graça social

Você está notando que, se ler o suficiente dos textos e afrimações, poderá encontrar virutalmente todas as facetas de todo tipo de comportamento? Bingo" As contradições acima são separadas no texto de Goodman´s, de modo que não se tropece nelas sem algum esforço. Isto é o que atrai pessoas para a astrologia em primeiro lugar: a idéia que eles podem encontrar respostas simples e imediatas para os complexos problemas da vida sem esforço.
______________________________________

Interessante texto sobre processos mentais na astrologia.

Um abraço.

Homero

[As partes desta mensagem que não continham texto foram removidas]



##### ##### #####

Para saber mais visite
http://www.ciencialist.hpg.ig.com.br


##### ##### ##### #####


Yahoo! Grupos, um serviço oferecido por:

São Paulo Rio de Janeiro Curitiba Porto Alegre Belo Horizonte Brasília




------------------------------------------------------------------------------
Links do Yahoo! Grupos

a.. Para visitar o site do seu grupo na web, acesse:
http://br.groups.yahoo.com/group/ciencialist/

b.. Para sair deste grupo, envie um e-mail para:
ciencialist-unsubscribe@yahoogrupos.com.br

c.. O uso que você faz do Yahoo! Grupos está sujeito aos Termos do Serviço do Yahoo!.



[As partes desta mensagem que não continham texto foram removidas]



SUBJECT: Re: [ciencialist] A guide for students doing projects - guia para projetos de estudos sobre astrologia
FROM: "Esteban Moreno" <estebanmoreno@idhi.org.br>
TO: <ciencialist@yahoogrupos.com.br>
DATE: 15/02/2005 19:44

Parodiando uma velha lenda sobre Galileu, "no entanto ela funciona".
E.
----- Original Message -----
From: Oraculo
To: ciencialist@yahoogrupos.com.br
Sent: Monday, February 14, 2005 11:51 PM
Subject: [ciencialist] A guide for students doing projects - guia para projetos de estudos sobre astrologia


Um guia para projetos de estudos sobre astrologia. No final tem vasta bibliografia sobre o assunto, estudos e desdobramentos. O texto é bem longo.

http://www.rudolfhsmit.nl/t5-tests.htm

What tests are easy?
A guide for students doing projects

By Geoffrey Dean

Can astrology be tested?

As soon as astrologers hear the word "test" they tend to raise objections such as the following:

a.. Astrology (soul stuff) cannot be tested by science (materialism).
b.. Psychology is too confused for its tests to be meaningful.
c.. Proper testing is not possible in the absence of funding.
d.. Testers are closed-minded and cannot be taken seriously.
Ignore such objections. First, if they were true then astrologers could never know anything about astrology. How could they know it works if they could not test it? Second, the objections are irrelevant to the questions that matter, such as are Leos more generous than non-Leos, or do astrologers agree on what a chart means. Third, yes, some tests are difficult but others are easy and can be made by anyone.

Helpful questions

The following questions, even if not part of your eventual project, may help in clarifying your approach to astrology.

To evaluate an idea such as "astrology works", try asking the following questions. The aim is not to win but to learn.

(1) Why do you believe in this idea? This puts the burden of proof on the claimant where it belongs.
(2) What evidence would you accept as proving your idea wrong? This is a potent question because it opposes the tendency to consider only confirming cases.
(3) Are there other explanations that would produce the same outcome? This too is a potent question because it looks at how informed the claimant is.

Avoid open ended questions. Ask not "how does astrology work?" but "what is meant by astrology?" so you know what the issues are (astrologers seldom agree on this or anything else, so the answer will depend on who you talk to). Not "is astrology true?" but "to what extent is it true?" or "does it need to be true?" The key question is "does astrology deliver benefits beyond those due to non-astrological factors?", but this cannot be answered until the other questions have been answered. Do not expect to find this question answered in astrology books.

Projects about sun signs

Signs are the most researched topic in astrology with well over one hnndred empirical studies. Most studies are simply counts of people born under various signs, but such counts are too contaminated by ordinary influences (astronomy, sampling, demography, age incidence) to mean anything. We can try correcting for such influences but in practice the uncertainty is too great. So forget counting sun signs. If you need convincing, read this website's critiques of Sachs's Astrology File.

The remaining studies, if adequately controlled against non-astrological influences, have invariably been negative. Signs are not only the most researched topic in astrology but are also the most disconfirmed. Signs are simply not valid, not even slightly. So forget validation, including validation of sun sign compatibility. Turn instead to looking at people's reactions towards an extremely popular belief now known (but not widely known) to be untrue.

Sun signs seem valid because they cannot fail to fit (everyone, not just Leos, will admit to being generous). They are also a good topic of conversation and are nice to have around. People tend to like them, but exactly why is not well understood despite many speculations. So one easy and useful project might be to ask people why they like or dislike sun signs, and to find reasons for any differences. For example men may view sun signs differently from women.

Sun signs let you study people's reactions to disconfirmation. Assemble a roomful of people, sit them in a circle according to sun sign (this adds intrigue and keeps the group focussed), and record each person's level of belief in sun signs (put a scale on display for them to choose from, such as strong, moderately strong, and so on). Ask for opinions why anyone should believe or disbelieve in sun signs, and get a debate going. Observe the arguments and record how each person reacts to views opposing their own. After a suitable period stop the debate and again record each person's level of belief. Ask why they changed or why they didn't change. Next, raise the question that nobody will have raised, namely what evidence would each person accept as disproving sun signs. Record their answers. Then summarise the evidence from this website about sun signs, and restart the debate. After a suitable period stop the debate and again record each person's level of belief. Rationally, everyone should end up as disbelievers, but most likely the believers will carry on believing. Ask them why. You now have the makings of an absorbing essay for your social science teacher.

Belief in sun signs is known to slightly affect a person's self-image and in some cases even their choice of job. The daily horoscope may well have an effect on daily mood. But measuring these things requires large samples, careful techniques, and lots of time, which puts them well beyond the resources of most projects. But there is a much easier way of getting useful insight into these things, namely by talking to people who take sun sign columns very seriously and cannot exist without them. Such people comprise about 1% of the general population and may be hard to find, but your newsagent may know a few. This is an unexplored area that deserves attention.

Finally two easy projects that have been done before but are still informative and fun. (1) See if people can pick their own newspaper horoscope after you have mounted each one separately and removed all cues (they may need retyping). If picking is difficult, have people rank their top three or four choices. (2) Count agreements and disagreements between horoscopes from different newspapers or women's magazines. If you want more mileage, do a repeat but mixing the days so the horoscopes being compared are not for the same day. Then get others to repeat the exercise to see if their counts agree with yours. In each case it is a good idea to have a trial run to make sure there are no snags.

Projects about serious astrology

The bad news is that many tests are difficult, time consuming, and have already been done. For example testing whether an astrologer can identify people from their charts has already been done in 45 studies, some of which took several years to complete. Overall 600 astrologers and more than 1100 charts were tested but the results showed no support for astrology. (The support shown by some early tests was later found to be an artifact of sampling.) Another 20 studies involving nearly 500 subjects tested whether people can pick their own chart reading but again the results showed no support for astrology.

In general the better the study the more negative the outcome. The problem is that the sheer weight of studies will not be overturned by adding one more study. The good news is that other studies are much easier, just as useful, and more fun. And anyone can do them.

For example if you change the topic from seeing if astrologers are accurate to seeing if astrologers agree, the test becomes almost ridiculously easy. Just sit astrologers together in the same room, put a succession of charts on a screen, and ask questions that are a mixture of easy (is this person outgoing?) and difficult (does this person own a cat?). Any charts will do, even invented ones, because it is irrelevant what the person is actually like. What matters is whether the astrologers agree on what the chart is saying, and whether their agreement varies with difficulty.

To be feasible the above test requires access to a local astrology group. But it is worth the bother. Notice how all the usual excuses for error (birth time not accurate enough, person does not know themself, and so on) no longer apply, since accuracy is not a concern. The point is, if astrologers cannot agree on what a chart is saying, then what price astrology? So far 25 such tests have been made involving a total of 500 astrologers and 700 charts, but their agreement was scarcely better than no agreement. Do not expect to find this result mentioned in astrology books.

A popular but interesting test is pretending to obtain chart readings for a roomful of people, who rate each statement in the reading for accuracy and then indicate to what extent the reading has increased their belief in astrology. Each reading is preceded by the person's name and birth details, so it looks genuine, but in fact each person gets the same fake reading. Alternatively a different fake reading can be provided for every 5-10 people, which allows you to test more statements without lengthening the individual readings. The aim is to show people how easy it is to read specifics into generalities (this is called the Barnum effect), how this affects their belief level, and to compile a reading that will fit as many people as possible.

Here the sense seems to come from the reading but in fact it comes from our ability to make sense out of vague data, as when we see faces in clouds. The most readily-accepted Barnum statements are favourable (you are forceful and well-liked by others), or vague (you enjoy a certain amount of change and variety), or two-headed (you are generally cheerful but get depressed at times). Not unexpectedly, the result of accepting Barnum statements is an increase in belief. The Barnum effect has attracted around 70 studies but it still makes a telling point.

The fallacy of personal validation

Interestingly, the most famous Barnum study, made in 1949, and many studies since then, used 13 statements taken mostly from an astrology book. The statements were typical of what might be heard during a chart reading. The studies found that when the statements are presented as a personal reading, as would of course apply during a chart reading, most people rate their accuracy as good or excellent. In case you are interested, the statements are as follows:

(1) You have a great need for other people to like and admire you.
(2) You have a tendency to be critical of yourself.
(3) You have a great deal of unused capacity which you have not turned to your advantage.
(4) While you have some personality weaknesses, you are generally able to compensate for them.
(5) Your sexual adjustment has presented problems for you.
(6) Disciplined and self-controlled outside, you tend to be worrisome and insecure inside.
(7) At times you have serious doubts as to whether you have made the right decision or done the right thing.
(8) You prefer a certain amount of change and variety and become dissatisfied when hemmed in by restrictions and limitations.
(9) You pride yourself as an independent thinker and do not accept others' statements without satisfactory proof.
(10) You have found it unwise to be too frank in revealing yourself to others.
(11) At times you are extroverted, affable, sociable, while at other times you are introverted, wary, reserved.
(12) Some of your aspirations tend to be pretty unrealistic.
(13) Security is one of your major goals in life.

Taken from Forer BR, The fallacy of personal validation: A classroom demonstration of gullibility. Journal of Abnormal and Social Psychology 1949, 44, 118-123. The "personal validation" in the title refers to the process of looking at a statement, seeing that it fits, and concluding that the system works. Just as astrologers do every day.

For example look at volume 10 pages 25-27 of Noel Tyl's Principles and Practice of Astrology, Llewellyn 1975, where he gives a reading to show how astrological counselling should be conducted. Tyl, a famous American astrologer and psychology graduate of Harvard University, is renowned among astrologers for the quality of his work, so we can take this as being representative of the best practice. Tyl is talking to Eric, a new client aged 24. Comments have been added in [ ].

"Basically, we should begin with a general statement to differentiate you from ... all other people. The horoscope shows that you are gracious, friendly, that you express yourself softly [all this was already obvious, now comes the favourable Barnum], and that you're very sensitive. [Now the vague Barnum] But, Eric, there are tensions here ... linked to self-esteem, how you feel about yourself. [Now the double-headed Barnum] The horoscope suggests that, on the one hand, you're pretty particular about whom you relate to. [Now repeat vague Barnum] This might be a defense because you're not too pleased with yourself. [Now back to double-headed Barnum] And, then on the other hand, you're friendly to the extreme in order to be accepted. Do you think that is accurate?" To which Eric replied "Yeah (smiling). That's it, right on the button. [But how could it be otherwise?] Gee, I know a little about Astrology; how do you see all that?"

Sources of information

If your project requires a review of the literature, remember that books which examine the truth about astrology can be hard to find. For every book there are dozens of articles that can be even harder to find. Selected book titles are given below. Most address astrology as a whole with sun signs on the side. Be aware that works earlier than 1990 tend to be dated.

Ankerberg J & Weldon J (1989). Astrology: Do the Heavens Rule Our Destiny? Harvest House, Eugene OR 97402, 334 pages with 356 references but no index. Powerful and very readable critique by Bible scholars who have done their homework (the Christian perspective does not intrude). Many useful quotes, all referenced to the exact page.

Blackmore S & Hart-Davis A (1995). Test Your Psychic Powers: Find Out the Truth for Yourself. Thorsons, London. Sets out simple experiments for testing sun sign columns and nine other paranormal areas (telepathy, crystals, dreams, dowsing, pendulums, premonitions, psychokinesis, ouija boards, and palmisty) that you might find more appealing! Blackmore was then a leading skeptic and investigator of the paranormal.

Culver RG & Ianna PA (1988). Astrology: True or False? A Scientific Evaluation. Prometheus Books, Buffalo NY, 228 pages with over 200 references. Minor update of the original 1979 version. A clear and very readable critique by astronomers, much useful information including test results, a classic survey of astrological prediction, and ten tests issued as a challenge to astrologers (no volunteers to date), but lacks the psychological perspective of Eysenck & Nias (1982). Concludes that astrology is neither scientifically sound nor scientifically useful.

Dean G, Mather A, & 52 others (1977). Recent Advances in Natal Astrology: A Critical Review 1900-1976. Analogic, Perth Western Australia, 608 pages with 1010 references. Out of print but several copies are held by the library of the Astrological Association. Much is now very dated, and most of the positive findings have subsequently been overturned. All the important studies are in Eysenck & Nias (1982).

Dean G (1992). Does astrology need to be true? In Frazier K (ed). The Hundredth Monkey and other paradigms of the paranormal. Prometheus Books, Buffalo NY, pages 279-319 with 126 references. The answer to the title question is no.

Dean G, Mather A & Kelly IW (1996). Astrology. In Stein G (ed). Encyclopedia of the Paranormal. Prometheus Books, Amherst NY, pages 47-99 with 15 general references. A comprehensive scientific survey covering history, popularity, arguments for and against, conceptual problems, controlled tests, effect size comparisons, problems of birth chart interpretation, how belief in astrology arises, role of human judgement biasses, and the future of astrology. But at USD149.95 the encyclopedia is affordable only by libraries.

Dean G & Kelly IW (2000). Does astrology work? Astrology and skepticism 1975-2000. In Kurtz P (ed). Skepticism: A 25 Year Retrospective. Prometheus Books, Amherst NY. Non-technical with minimum references. Reviews the progress of scientific investigations of astrology since 1975.

Dean M (1980). The Astrology Game. Beaufort, New York, 360 pages with nearly 200 references. About one fifth is a useful critique of the pop astrology industry. The rest, now dated, tries to establish evidence for astrology. Predicts (wrongly) that a new astrology is being born.

Eysenck HJ & Nias DKB (1982). Astrology: Science or Superstition? St Martin's Press, New York, 244 pages with 230 references. Also in Penguin but out of print. Best single source on research up to 1982. Clear, sympathetic and very readable critique by psychologists. Concludes that everything is negative except the Gauquelin results. Now somewhat dated but still valid, later research is even more negative.

Gauquelin M (1979). Dreams and Illusions of Astrology. Prometheus Books, Buffalo NY, 158 pages with 125 references and notes but no index. Originally published in French 1969. Mostly a survey of Gauquelin's early non-positive results. Concludes that "since the most painstaking studies have shown the inanity of horoscopes, there should be a strong rising up against this exploitation of public credulity" (p.158).

Gauquelin M (1983). The Truth about Astrology. Basil Blackwell, London, 202 pages. The title was imposed by the publisher. Published in the USA as Birthtimes by St Martin's Press, New York. The best single source on the Gauquelin work but includes little on non-Gauquelin work and of course nothing on the important later developments. Also in Hutchinson paperback 1984. Details of sources occupy 15 pages.

Hines T (1988). Pseudoscience and the Paranormal: A Critical Examination of the Evidence. Prometheus Books, Buffalo NY. Astrology (along with lunar effects and biorhythms) is on pages 141-164. Now in 2nd edition.

Martens R & Trachet T (1998). Making Sense of Astrology. Prometheus Books, Amherst NY, 276 pages with 140 references. A critique by two prominent Belgian skeptics. Looks in detail at each astrological idea (signs, houses, aspects, etc) and then at some of the empirical evidence. Includes important studies published since Eysenck & Nias (1982). Concludes that the final judgement "is without any doubt negative". Nevertheless astrology is unlikely to go away.

Phillipson G (2000). Astrology in the Year Zero. London: Flare Publications. See http://www.flareuk.com. Explores key issues via interviews with astrologers and researchers. Includes a review of recent research findings, a look at human judgement errors, and a critical bibliography, all with expanded versions on this website.

Pottenger M ed (1995). Astrological Research Methods Volume 1. ISAR, PO Box 38613, Los Angeles CA 90038-8613, 466 pages. Not a critique but included here because it is the only available anthology of articles on research methods. Unfortunately there are frequent opposing views that are left unresolved, and there is no mention of faulty reasoning. But the authors include scientists as well as astrologers, quite a few articles convey the scientific spirit, and in general the anthology represents a major advance in astrological thinking.

Rae AC (1992). Bluff Your Way in Astrology & Fortune Telling. Ravette Books, Horsham, West Sussex, 62 pages. A wonderfully cynical guide to being an expert without having to know or study anythng. Includes Chinese astrology, Tarot, I Ching, palmistry, and runes.

Sachs G (1998). The Astrology File: Scientific Proof of the Link between Star Signs and Human Behavior. London: Orion Books. Occasionally books appear that claim to validate sun signs. But in every case a critical examination has revealed mistakes and procedural blunders, of which this book is a best-selling example. The author uses samples of up to several millions of cases, which are so huge that even the most trivial of errors (for example, in matching the sample to controls) become enormously inflated in statistical significance. So his conclusions are misleading. Otherwise, the book is readable and well set out. Three critiques that expertly reveal its mistakes and procedural blunders are on this website.

Standen A (1977). Forget Your Sun Sign: An Outline of Antiastrology. Legacy Publishing, Baton Rouge LA, 135 pages with 33 references. Very readable but now very dated. The author is a scientist who studied astrology "very thoroughly, and came out an antiastrologer. This book is to explain the reasons why." His reasons apply even more today.

Stewart JV (1996). Astrology: What's really in the stars. Prometheus Books, Buffalo NY, 156 pages 194 references. First two-thirds looks at early sources including Babylonian omen tablets (many photographs), the ancient Greeks, and Ptolemy. Rest is sketchy and lacking in detail.

Van de moortel K (2002). Astro-Logics. Self-published, Ghent. 156 pages, 109 references. See http://www.astrovdm.com. A critical look at astrology by a leading writer of astrological software. Very readable and largely nontechnical. Reveals the disagreements and inconsistencies behind almost every kind of chart factor and argues that only research can get out of this mess. Gives a selection of research results, most of them negative, and some basic guidelines for doing research, but does not address faulty reasoning.




[As partes desta mensagem que não continham texto foram removidas]



##### ##### #####

Para saber mais visite
http://www.ciencialist.hpg.ig.com.br


##### ##### ##### #####


Yahoo! Grupos, um serviço oferecido por:

São Paulo Rio de Janeiro Curitiba Porto Alegre Belo Horizonte Brasília




------------------------------------------------------------------------------
Links do Yahoo! Grupos

a.. Para visitar o site do seu grupo na web, acesse:
http://br.groups.yahoo.com/group/ciencialist/

b.. Para sair deste grupo, envie um e-mail para:
ciencialist-unsubscribe@yahoogrupos.com.br

c.. O uso que você faz do Yahoo! Grupos está sujeito aos Termos do Serviço do Yahoo!.



[As partes desta mensagem que não continham texto foram removidas]



SUBJECT: Re: [ciencialist] How to convince clients that astrology works
FROM: "Esteban Moreno" <estebanmoreno@idhi.org.br>
TO: <ciencialist@yahoogrupos.com.br>
DATE: 15/02/2005 19:46

Dê um lida em:
Self-attribution, sun-sign traits, and the alleged role of favourableness as
a moderator variable:
long-term e.ect or artefact? Edgar Wunder* Gesellschaft fu¨r Anomalistik,
Postfach 1202, 69200 Sandhausen, Germany Personality and Individual
Differences 35 (2003) 1783-1789 www.elsevier.com/locate/paid

Desmente este pseudocientificismo que nos apresenta.Posso envia-lo em anexo.
E.


----- Original Message -----
From: Oraculo
To: ciencialist@yahoogrupos.com.br
Sent: Monday, February 14, 2005 11:36 PM
Subject: [ciencialist] How to convince clients that astrology works


Chamada do artigo abaixo:
Hidden persuaders are systematic errors in our reasoning that explain why a totally invalid system can still seem to work. Explores over 30 hidden persuaders which convince clients that astrology works. All are in everyday use in astrology. None require that astrology be true.

Tradução (meia boca..:-)
Persuadores escondidos (de persuadir, convencer) são erros sistemáticos em nosso raciocínio que explicam porque um sistema totalmente inválido pode aparentar funcionar. Apresenta mais de 30 "hidden persuaders" que convencem clientes que a astrologia funciona. Todos são usados em astrologia todos os dias. Nenhum requer que a astrologia seja real.

_______________________________________________________
Hidden persuaders:
How to convince clients that astrology works
By Geoffrey Dean and Ivan W Kelly

Unaided human reasoning is subject to systematic errors (we call them hidden persuaders) that can explain why an experience-based astrology should seem to work even if it were totally invalid. For example hidden persuaders explain why tens of thousands of Western tropical astrologers can say that in their experience Scorpios really are intense, while hundreds of thousands of Eastern sidereal astrologers can look at the same piece of sky, which they call Libra, and agree that in their experience it is not intense but relaxed. The same applies to all the others factors that astrologers disagree about, which is most of them.

Caution. You will be led seriously astray if you learn about astrology without first learning about hidden persuaders. Unless you can be sure that hidden persuaders have been ruled out, don't believe what you read in astrology books. In particular the claim that astrologers proudly and repeatedly make, that astrology is unassailable because it is experience-based, is simply mistaken -- what they see as its strength is actually its weakness. If this sounds preposterous, consider the following quote from psychologist Ray Hyman, How not to test mediums, Skeptical Inquirer 27(1), 20-30, January-February 2003. Hyman has devoted more than half a century to the study of psychic and other readings, especially to why such readings can seem so compelling.

How not to test a reading

"As a way to earn extra income, I began reading palms when I was in my teens. At first, I was skeptical. I thought that people believed in palmistry and other divination procedures because they could easily fit very general statements to their particular situation. To establish creadibility with my clients, I read books on palmistry and gave readings according to the accepted interpretations for the lines, shape of the fingers, mounds, and other indicators. I was astonished by the reactions of my clients.

"My clients consistently praised me for my accuracy even when I told them very specific things about problems with their health and other personal matters. I even would get phone calls from clients telling me that a prediction that I had made for them had come true. Within months of my entry into palm reading, I became a staunch believer in its validity. My conviction was so strong that I convinced my skeptical high school English teacher by giving him readings and arguing with him. I later also convinced the head of the psychology department where I was an undergraduate." (page 22)

So far it could be any astrologer talking about astrology. They read charts and they become staunch believers in astrology's validity. As in Hyman's case, their experience seems totally compelling. But this is not the way to test a reading. Hyman then did something every astrologer should do but never does:

"When I was a sophomore, majoring in journalism, a well-known mentalist and trusted friend persuaded me to try an experiment in which I would deliberately read a client's hand opposite to what the signs in her hand indicated. I was shocked to discover that this client insisted that this was the most accurate reading she had ever experienced. As a result, I carried out more experiments with the same outcome. It dawned on me that something important was going on. Whatever it was, it had nothing to do with the lines in the hand. I changed my major from journalism to psychology so that I could learn why not only other people, but also I, could be so badly led astray." (page 22)

Hidden persuaders

The answer to Hyman's puzzle is hidden persuaders, factors that can make a vague reading seem so uncannily accurate that it becomes almost impossible not to believe in the system's validity. There are many hidden persuaders. The most important hidden persuader for astrology generally is the consider-only-confirming-cases artifact discussed in the Artifacts and Truth articles on this website, but for a client consultation it becomes just one among thirty-four as listed below. None of these hidden persuaders require that astrology be true, yet each will (wrongly) convince astrologers and clients that astrology works. All are in routine use in astrology consulting rooms. For convenience we have grouped them under seven descriptive strategies:

1. Select initial hurdles

a.. Preach to the converted (client predisposition)
b.. Ignore everything on this website (ignorance is bliss)
c.. The best things in life are not free (charging a fee)
2. Stifle chances of being wrong

a.. Appeal to birth chart complexity (nonfalsifiability)
b.. Avoid conflict, see what you believe (cognitive dissonance)
c.. Believe what you cannot prove (unavailable data)
d.. Remember the hits, forget the misses (selective memory)
e.. Ask only confirming questions (stacking the deck).
This is the consider-only-confirming-cases artifact.
f.. Ignore disconfirming evidence (confirmation bias)
g.. Deny that astrology can be tested (testability veto)
3. Use cues

a.. Let context give the game away (vital statistics)
b.. Let body language be your guide (cold reading)
4. Make astrology look good

a.. The importance of first impressions (halo effect)
b.. If it looks right then it is right (face validity)
c.. Style is more important than content (Dr Fox effect)
d.. Underestimate chance effects (chance baseline shift)
e.. More is better (Aunt Fanny effect)
5. Make clients feel good

a.. Use a kind heart to entice belief (tea and sympathy)
b.. The power of positive thinking (Pollyanna principle)
c.. It does us good if we think it does (placebo effect)
d.. Having control makes us feel better (misattribution)
e.. Just naming the unknown is enough (Rumpelstiltskin effect)
f.. Closeness is its own reward (rapport)
6. Make the chart fit

a.. Find meaning where none exists (faces in clouds)
b.. Read specifics into generalities (Barnum effect)
c.. See only what you want to see (illusory correlation)
d.. Accentuate the positive (social desirability)
e.. Be seduced by resemblance (magical thinking)
f.. Afterwards we knew it all along (hindsight bias)
g.. Sound arguments yes, sound data no (stereotypes)
7. Make the client fit

a.. Find something, anything, to match the chart (repertoire)
b.. Let client role-play their chart (self-fulfilling prophecy)
c.. Force client to fit their chart (Procrustean effect)
d.. Winter does not last forever (regression to mean)
Each hidden persuader reflects the systematic error in human reasoning shown in parentheses, for which we have used the accepted name if there is one, or a provisional name if not. In the early days it was usual for critics to explain why clients were satisfied with astrology readings in terms of the Barnum effect, the reading of specifics into generalities such as "you have problems with money", where sense appears to come from the reading when in fact it comes from our ability to make sense out of vagueness. Today, as shown above, many more hidden persuaders are known. They vary in effectiveness, and in a given situation some may be irrelevant, but all lead to client satisfaction and none require that astrology be true.


But if clients are going to be satisfied, astrologers can hardly fail to believe in astrology. In this way a vicious circle of reinforcement is established whereby astrologers and their clients become more and more convinced that astrology works. Note that there are no hidden persuaders to convince us that astrology does not work other than the informed critical mind, which of course is not an error as such but rather a defence against errors. An astrologer typically makes no effort to become informed about research or to acquire critical thinking skills, preferring instead to spend years learning to read charts, during which time they have ample chance to respond to the above reinforcement. And of course to build up a huge vested interest in continuing their ill-founded beliefs. If this still seems preposterous, consider the salutary case of phrenology.

The salutary case of phrenology

The same hidden persuaders explain how phrenology (head reading), once more popular and far more influential than astrology is today, could be accepted as totally valid even though it is now known to be totally invalid. As noted in 1985 by Dean and Mather (Astrological Journal 28(1), 23-30, Winter 1985):

"Astrologers are like phrenologists: their systems cover the same ground, they apply them to the same kinds of people, they turn the same blind eye to the same lack of experimental evidence, and they are convinced for precisely the same reasons that everything works. But the phrenologists were wrong. So why shouldn't critics conclude for precisely the same reasons that astrologers are wrong?" (page 25)

That was nearly 20 years ago, but no reply from astrologers has been forthcoming other than to dismiss phrenology as irrelevant. But perhaps no reply is required. After all, it could be argued that the existence of mutually incompatible systems throughout astrology (for example tropical and sidereal zodiacs), all of which are nevertheless seen as completely valid by their users, has already put this question to the test and given us convincing answers.

The bottom line

Thanks to hidden persuaders, the bottom line could hardly be simpler --astrology in the consulting room does not need to be true. This point was aired in a Skeptical Inquirer article published in 1986-1987, and in the later works listed below, where among other things readings that were the opposite of what the chart indicated were found to be as acceptable to clients as authentic readings, just as Hyman had found for palmistry. But the point has been generally ignored by astrologers. Readers interested in applying critical thinking to astrology readings will find much useful information in Gambrill (1990).

Further reading

Dean G (1992). Does astrology need to be true? In Frazier K (ed). The Hundredth Monkey and other paradigms of the paranormal. Prometheus Books, Amherst NY, pages 279-319 with 126 references. Update of a two-part article first published in Skeptical Inquirer 1986-1987. The answer to the title question is no.
Dean G, Mather A & Kelly IW (1996). Astrology. In Stein G (ed). Encyclopedia of the Paranormal. Prometheus Books, Amherst NY, pages 47-99 with 15 general references.
Dean G & Kelly IW (2000). Does astrology work? Astrology and skepticism 1975-2000. In Kurtz P (ed). Skepticism: A 25 Year Retrospective. Prometheus Books, Amherst NY, pages 191-207.
Gambrill E (1990). Critical Thinking in Clinical Practice: Improving the Accuracy of Judgements and Decisions about Clients. Jossey-Bass, San Francisco. 432 pages, 660 references. How to reduce reasoning errors in psychology, medicine and the helping professions. Well organised, packed with information, many examples, a few references to astrology (but only to illustrate reasoning errors), each chapter has a summary. Equally applicable to astrology. Should be read by every astrologer.


[As partes desta mensagem que não continham texto foram removidas]



##### ##### #####

Para saber mais visite
http://www.ciencialist.hpg.ig.com.br


##### ##### ##### #####


Yahoo! Grupos, um serviço oferecido por:







------------------------------------------------------------------------------
Links do Yahoo! Grupos

a.. Para visitar o site do seu grupo na web, acesse:
http://br.groups.yahoo.com/group/ciencialist/

b.. Para sair deste grupo, envie um e-mail para:
ciencialist-unsubscribe@yahoogrupos.com.br

c.. O uso que você faz do Yahoo! Grupos está sujeito aos Termos do Serviço do Yahoo!.



[As partes desta mensagem que não continham texto foram removidas]



SUBJECT: Re: [ciencialist] Astrologers rated these killers as good guys
FROM: "Esteban Moreno" <estebanmoreno@idhi.org.br>
TO: <ciencialist@yahoogrupos.com.br>
DATE: 15/02/2005 20:00

Esse é um dos erros mais comuns, tentar generalizar um mapa natal para o comportamento de todo um conjunto. Não funciona ignorar a individualidade, cada pessoa é um microcosmos e tem caracteristicas de acordo com uma série de influências ambientais que não são tratadas no mapa e nunca pretendeu tratar. Assassinos podem ser tão sensíveis quanto um artista. Assassinos podem também gostar tanto de crianças e, em função de relações parentais muito mal resolvidas, ter que matá-las. Alias, os piores assassintos são muito sensíveis. Este estudo é um tipo de pegadinha, no fundo está claro a resposta que queriam colher. Simplesmente induziram ao erro. Se algum astrologo ler o mapa do Gandhi sem que ele esteja na sua frente para depor, dirá que é um ser muito perturbado pisiquicamente. E foi, tanto que batia na esposa, porém soube muito bem trabalhar com as inclinações apontadas pelo mapa. Como cada pessoa vai desenvolver os recursos reumidos no mapa é particular de cada um, não existe outra possibilidade. A linguagem astrológica apenas vai conduzir 'as tendências daquele individuo. É por isso que a máxima da astrologia, a que devem conhecer é: "os astros inclinam mas não determinam."
O único experimento cientifico légitimo que concebo para começar a demonstrar o funcionamento ou não da astrologia é através do que chamamos de engenharia reversa. A pessoa é que deve se descrever e o astrologo descobrirá COM FACILIDADE qual o melhor mapa que se adequa ao caso. etc.
Fica a dica.
Um abraço,
Esteban.


----- Original Message -----
From: Oraculo
To: ciencialist@yahoogrupos.com.br
Sent: Monday, February 14, 2005 11:18 PM
Subject: [ciencialist] Astrologers rated these killers as good guys


Murder:

Astrologers rated these killers as good guys

An article from the now defunct Kansas City Committee for Skeptical Inquiry. The article was originally titled "Astrology on Death Row!" and was reprinted in the Indian Skeptic 1989, Vol 1 (11). The results of two similar studies have been added in an appendix.


How accurate is astrology? To find out, a member of the Kansas City Committee for Skeptical Inquiry (KCCSI) went to five astrologers posing as a man interested in working with young people. He gave each astrologers the birth date, birth time and place of John Gacy instead of his own, and a computerized natal chart from a company internationally recognized for accuracy (Neil F.Michelsen), and asked them for their advice. The astrologers unanimously encouraged him to pursue youth work and none could see any problem with this.
(John Gacy was born in Chicago on 17 March 1942 at 00:49 CST, or 05:49 GMT, CST being the time required by Illinois for birth registration even though War Time was then in effect. Some hospitals did not follow this rule, so the birth time might have been 06:49 GMT. Both times give a Sagittarius ascendant.)
At the time, the real John Gacy was being held on death row at Menard Correctional Center in Chicago. He received 12 death sentences and 21 life terms for the brutal torture and murder of 33 young men and boys. He was executed by lethal injection on 10 May 1994.
Gacy was selected for the test because his chart should portray a clear picture of a sadistic sexually motivated killer. If astrologers are able to spot personality traits and destinies in any chart, then this is the one they should have no trouble with.
The way in which Gacy murdered his victims was particularly repulsive. They were usually teenage boys whom he lured into his home and overpowered. He had a pair of trick hand cuffs, which he would demonstrate that he could slip out of. After immobilizing them by persuading them to put on the hand cuffs, he repeatedly raped and tortured them. Finally after they begged for their own deaths he would strangle them, often by shoving their own underwear down their throats. Each astrologer was asked to examine the computerized chart for as long as they wanted before giving their reading.

Rhe advice given by the astrologers

John Sandbach, a nationally known astrologer who has authored six books, advised him not to "become weighed down with regrets about how you could have done more in some past situation." He described a "plasticity or lack of aggression" in the chart and encouraged him to work with young people because he could "bring out their best qualities."
Local astrological talent, Randy Goodman, was reported by the Kansas City Star (April 10, 1985) to have investigated "mysterious flying hotdogs" at the Radisson Hotel Muehlebach. He told our substitute Gacy that he was "really born to serve people." He stated that "In the past you have used your energies very well so therefore in this life you have a lot to contribute, and... your life will be very, very positive."
Another local astrological celebrity, Norma Knight, was also given an opportunity to analyze the chart. She described him as "a very, very sensitive person." When asked about the youth ministry she replied, "I think that you can be very good with kids and that it might be a good medium for you to learn to be more trusting in the giving and receiving."
Beverly Farrel lays claim to being an "internationally recognized author, lecturer and teacher of religion, metaphysics, astrology and psychic awareness" with "30 years experience in the field of (the) paranormal". She also encouraged the man behind the chart to go into youth work. She stated that when you're working with young people you're not to have a lot of heavy-duty problems."
The results seemed to indicate that astrologers cannot read a persons character from the positions of the planets at the moment of birth nor can they see into anyone's past or future nor do they seem to possess any insights other than the ability to impress their clientele.

How well did the readings fit anyone?

KCCSI next passed out envelopes to students in several local college philosophy classes. The students were instructed to write their names, birthdates, birth times and places on the outside of the envelopes and return them. KCCSI put in each envelope an excerpt from the Gacy readings and handed them back to the students telling them that a professional astrologer had made up a personality description specifically for them. They were asked to grade how well each descriptin fitted them.
When the results were tabulated it was discovered that those students who believed in astrology showed a significant tendency to grade the readings as more accurate than did those students who did not believe. This supports the premise that the popularity of astrology is due to the predisposition of the believers to exaggerate its accuracy and thus make it appear to work. Also this would explain the persistence of many people to assert that astrology really works despite evidence to the contrary.

What the astrologers said about Gacy

"I think that you can be very good with kids and that might be a good medium for you to learn to be more trusting in the giving and receiving."

"...just your presence would be of a beneficial nature to other people, a real calming kind of effect... In the past you have used your energies well: so therefore, in this life you have a lot to contribute and you will have some problems but basically your life will be very, very positive."

"...a fairly well-rounded personality...you can offer a good role model...when you're working with young people you're not gonna have a lot of heavy-duty problems."

"Helpful, understanding of the needs of others. At times a sucker for anyone who needs help... Kind, gentle, considerate of others needs."

"At your best, you are very impressionable and radiate the unconditional love of a happy infant... You have an instinctive awareness and your uninhibited response to life can refresh and gladden whomever you encounter."

Appendix. Two similar tests using mass murderers

The first is from Gary F Posner, Skeptically Speaking March 1993, with further information from a videotape of the Stossel TV show. The second is from an article by Michel Gauquelin in Aquarian Agent May 1970.
Joyce Jillson, whose column is carried by the Tampa Tribune and who is Hollywood astrologer to the stars, unwittingly allowed ABC-TV reporter John Stossel an opportunity to demonstrate for his millions of Good Morning America viewers in November 1988 just how well astrology works in spite of it appearing not to be real.
Jillson (who normally charges $200 per chart) first prepared a detailed reading for a person unknown to her, whose birth information (which is all she requested) was supplied to her by Stossel. Stossel then distributed a copy of the completed horoscope to each of 20 students in an adult education class (each student had given Stossel their own birth information one week earlier).
The students, thinking they were each reading their own personalized horoscopes, marveled at how Jillson knew things about them that no one else could possibly know. Typical responses were "very accurate", "I am amazed", "hits points that nobody knows". Stossel asked one student "is it you?" (yes) "but is it different from other people?" (yes it is). But each student, male or female, was reading the same horoscope, that of someone described by Jillson as "enormously bright ... [with] sexual charisma ... great charm ... a sense of moral propriety ... [who] may know celebrities ..."
Stossel thought that Jillson may have incorrectly assumed the birth information to have been his own. But Jillson's one-size-fits-all unisex horoscope was actually based on the birth information of mass murderer Edward Kemper III who, in addition to many other "charming" deeds, had cut off his mother's head and used it as a dartboard! Ironically one statement fitted the authentic subject quite aptly, namely "takes life seriously." Concluded Stossel, while confronting Jillson on camera with the facts and watching her squirm, "I just think this shows it's a hustle, and you make money by writing general things that everybody believes is about them."
(All this had no noticeable eggect on Jillson's popularity. In 2003 she suffered what the Skeptical Inquirerul Nov/Dec 2003 page 9 called "the ultimate embarrassment for an astrologer". In her syndicated column of 12 September, Jillson had given a glowing horoscope of TV star John Ritter "having a Virgo sun sign helps keep his career tickng", even though he had died the day before.)
The same thing happened when Michel Gauquelin gave 150 people a 10-page computer interpretation of supposedly their own birth chart but which was actually that of the notorious mass murderer Marcel Petiot. He asked for their opinions, and 90% found the accuracy to be confirmed by their family and friends, and 94% found it to accurately describe their character, their personal problems, and the cycle of events in their life. It produced responses like "the work done by this machine is marvellous", and "on the whole, everyone who knows me found it accurate, especially my wife" (from Aquarian Agent May 1970).
Obviously perceived accuracy means little. Studies like these have repeatedly shown that people find astrology's insights just as convincing when unknown to them the wrong birth data are used. For a worked example using Petiot's chart see Perceived effect sizes under Tests & Snags.


[As partes desta mensagem que não continham texto foram removidas]



##### ##### #####

Para saber mais visite
http://www.ciencialist.hpg.ig.com.br


##### ##### ##### #####


Yahoo! Grupos, um serviço oferecido por:







------------------------------------------------------------------------------
Links do Yahoo! Grupos

a.. Para visitar o site do seu grupo na web, acesse:
http://br.groups.yahoo.com/group/ciencialist/

b.. Para sair deste grupo, envie um e-mail para:
ciencialist-unsubscribe@yahoogrupos.com.br

c.. O uso que você faz do Yahoo! Grupos está sujeito aos Termos do Serviço do Yahoo!.



[As partes desta mensagem que não continham texto foram removidas]



SUBJECT: Re: [ciencialist] Astrologers vs Journal of Consciousness Studies
FROM: "Esteban Moreno" <estebanmoreno@idhi.org.br>
TO: <ciencialist@yahoogrupos.com.br>
DATE: 15/02/2005 20:04

Oracle, eu francamente não compreendo por que dispensa tanto tempo em tentar provar algo que não acredita sem qualquer evidencia para tal. Temas complexos envolvem respostas complexas, como você disse anteriormente. Estude antes!
Amplexos descomplexos,
Esteban.



----- Original Message -----
From: Oraculo
To: ciencialist@yahoogrupos.com.br
Sent: Monday, February 14, 2005 11:23 PM
Subject: [ciencialist] Astrologers vs Journal of Consciousness Studies


Olá pessoal

O artigo abaixo é longo, mas vale a pena dar uma lida. Os apendices do texto não foram copiados aqui (ficaria grande demais para um email..:-) e podem ser lidos neste link:

http://www.rudolfhsmit.nl/d5-starwarsnew.htm

Um abraço.

Homero
___________________________________________________________
Star Wars


Astrologers vs Journal of Consciousness Studies


The special June/July 2003 issue of the prestigious Journal of Consciousness Studies was devoted to parapsychology, It contained twelve long articles, and the issue as a whole received praise for its balanced approach from New Scientist (13 September 2003 "authoritative and accessible"), and from Amazon ("serious and responsible"). But what caught media attention was the article entitled "Is Astrology Relevant to Consciousness and Psi?", a scholarly article of 24 pages and 85 references by Geoffrey Dean and Ivan W Kelly, which one astrology website later described as "Dean and Kelly rehashing old and flawed research hype". It led to the biggest media frenzy on astrology for 2003.







Two things make the frenzy of particular interest. First, its focus is not your everyday sun sign astrology but the serious astrology of conferences and consulting rooms. Second, the frenzy brought out some of astrology's top defensive guns, so you can judge how well they perform when confronted by scientific findings. (If you happen to be a True Believer you may prefer to stop reading now, or at least take a tranquilliser.)

Dean and Kelly start by quoting various astrologers on how a successful birth chart reading requires some kind of psychic ability, where the chart acts like a crystal ball. If this were found to be true it might require a re-assessment of present theories of consciousness, so it deserves study. But a large-scale test of 2101 persons born on average less than five minutes apart found no hint of the similarities in personality or behaviour predicted by astrology. So if astrologers (as opposed to astrology) can predict personality or behaviour better than chance, as they claim to do, it might be evidence for psi.

But meta-analysis of more than forty controlled studies found no evidence that astrologers perform even marginally better than chance, even on basic tasks such as predicting extraversion (basic because according to astrologers it is one of the easiest things to see in a birth chart). They did not even usefully agree on what the birth chart indicates. More to the point, astrologers who claimed to use psychic ability performed no better than those who did not. Dean and Kelly cautiously conclude "the possibility that astrology might be relevant to consciousness and psi is not denied, but such influences, if they exist in astrology, would seem to be very weak or very rare". See Appendix 1 for an abstract of their article.

Normally this cautious non-link between astrology and psi might have passed unnoticed. But it was picked up and distorted by the Sunday Telegraph London, 17 August 2003 ("Astrologers fail to predict proof they are wrong"), and was duly copied or quoted around the world from Brazil to Finland. It was distorted because Dean and Kelly's focus was psi and consciousness, not the merits of astrology (for example its merits include providing low-cost ego support, and astrologers are generally nice people), merits they had already covered in other articles such as their chapter in Paul Kurtz's Skeptical Odysseys, Prometheus 2001. The result was a frenzy of misleading headlines and reports such as "Is astrology bunk?" (Daily Mail London, 18 August), "Research paper rubbishes astrology" (Hindustan Times India, 17 August), and "Who will put their faith in the stars?" (Sunday Herald Sun Melbourne, 14 September).

Regardless of the distortion, astrologers were predictably outraged and uninformed. They declared that negative results are by definition due to ignorance or hostility. Especially outraged were Indian astrologers, one of whom said in emails "it is only a study by some crazy white b's. They do not have any brain". Another said "Most probably these two guys are unemployed". Another said "if you really want to test astrology ... the only place where the research can be justified is here in India" (where presumably men are Men and astrologers are Astrologers).

Perhaps the most notable response from India was "Astrology is Science, not Rubbish" (India Express 25 August), where Dr Raj Baldev "who is considered an authority on the subject of Astronomy, Astrology, Cosmo-Mathematics and Metaphysics" (he has a website www.occultastrology.com offering "The perfect gift. Occult Horoscope by post. Only $6.95") said Dean and Kelly had made "an abominable mistake that can never be pardoned". He explained that ancient Hindu astrology "is a complete science" where even one million billionth of a second "makes a lot of difference". So it is ridiculous to believe that people born a few minutes apart should be similar. (Measuring birth times to a million billionth of a second implies that the position of shadows cast on ancient sundials was routinely read to better than a hundred millionth of the diameter of an atom. Even at night. Should we believe it?)

Western astrologers did not hesitate to give opinions without having read the article. In a Melbourne radio interview, Brian Clarke from the Australian Federation of Astrologers explained how there was more to astrology than sun signs, so all was well (in fact the Dean and Kelly article had nothing to do with sun signs). In The Guardian London, 19 August, astrologer Neil Spencer noted how astrology can "send arch-rationalists into fits of self-righteous indignation" (like his?), how the article lacks details (not true), how it ignores the positive results of Vernon Clark and Gauquelin (not true), and how the "Magi Society [an international society of astrologers based in New York] ... still has to receive a riposte to its statistical challenges" (one is in Skeptical Inquirer March-April 1997). He ends with "Astrology is not a science but a symbolic, allusive language" (boo to Dr Baldev), as if that somehow excused its failure to deliver on testable claims, to which the sociology website www.butterfliesandwheels.com replied, "Oh that old ploy".

The most orchestrated response came from the AA (UK Astrological Association), which in 2000 had refused to publicly declare its position on sun signs despite overwhelming evidence for their invalidity (see Response to an invitation under Sun Signs). The AA president Roy Gillett accused Dean of seeking to "discredit astrology". The AA website www.astrologicalassociation.com accused Dean and Kelly of having a "tortured imagination" and "defensively closed mindsets" that "deny astrology an even-handed debate", to correct which it then gave "a balanced response" via two reports that together "comprehensively dismiss these outrageous and disingenuous claims".

The first report, which originally appeared in the Daily Mirror London, 18 August 2003, was by "world renowned astrologer Jonathan Cainer". Cainer has his own astrology phonelines, and according to the Sunday Times London his estimated income of 2.2 million pounds a year puts him among the top 150 UK earners. Cainer begins by complaining that it cost him 15 pounds to download the article, whereupon he was suspicious the moment he saw the authors' names (refutation by name-calling?), claiming that Dean deliberately misunderstands what astrologers do (Dean is a former astrologer and understands very well what astrologers do).

Cainer then drops a series of clangers -- personality tests are "dangerously unreliable" (not these ones), "most scientists hate astrology" (most have better things to do), the tests covered ages only up to 23 (so Cainer rejects phoneline callers under 23?). His most notable clanger was "Once again, it turns out to be an experiment rigged to make astrology look silly" (in fact it was the AA who proposed the experiment in the first place, not in a moment of weakness but in a well-researched eight-page proposal prepared when the subjects were seven years old). He ends with "And some scientists claim to have a truly open mind" (like his open-minded behaviour towards negative findings?). See Appendix 2 for Dean and Kelly's point-by-point response to Cainer.

The second report was by "leading author and academic Dr Frank McGillion", a consultant to the astrological research group at Southampton University, who is said to find the article "essentially flawed and well below expected academic standards". Dr McGillion begins by saying the article is too long, then it is too short. He quibbles about definitions without providing his own definitions, he dwells on side issues without explaining their relevance, he cites unspecified "evidence" without supporting references or arguments, and he generally declines to act the way he tells Dean and Kelly to act.He says a focus on consciousness and psi

"might seem less relevant ... than the authors appear to consider" (the call for papers required it), he dislikes "the citation of names of non-scientists in formal scientific papers" (so how to give astrologers' views without citing astrologers?), and he says editing such psi papers "is a demanding task for the non-specialist" (one editor was world expert James Alcock). At the end he generously allows that "there is much here that astrologers can learn from" (so Cainer was wrong to call it "a load of rubbish"?). Nevertheless conspicuously absent is a description of what Dean and Kelly did and a critique of their results (refutation by censorship?). See Appendix 3 for Dean and Kelly's point-by-point reponse to Dr McGillion.

Claims of credibility are clearly not helped when the AA, which calls itself "one of Europe's leading astrological organisations", can see these two reports as "a balanced response" and "even-handed debate".

Yet other astrology websites uncritically recycled the reports almost verbatim. For example www.astrology.co.uk said Dean and Kelly were "manipulating results ... using self-fulfilling personality tests ... selecting data to fit results", and dropped new clangers such as implying the average birth interval was one day (it was less than five minutes), and asking "who says astrologers are psychic?" (answer: the astrologers and surveys quoted). Another website said "Dean and Kelly are known to be opponents of astrology", as if seeking evidence was an act of heresy. Similarly, in The Mountain Astrologer (Dec/Jan 2003/2004 issue), the American astrologer Gloria Star (without reading the article) said "the research itself appears to be significantly flawed". No doubt her opinion will be repeated by other astrologers around the world, always without reading the article. Should we be surprised?

Without exception the astrologer responses boiled down to name-calling, getting it wrong, and never citing compelling evidence to support their claims. Even serious media outlets adopted this distortion. For example in the New Zealand Listener (4 October 2003), in an article subtitled "a recent study claims to debunk astrology once and for all" (not true), astrologers and skeptics are seen as being permanently locked in "trench wars" and "long feuds", serious studies are seen as attacks on astrology, researchers are seen as debunkers, and astrologer quotes such as "you will never get a correlation that is significant, because we are dealing with individuals" are seen as an adequate response to the negative meta-analysis even though it involved individuals. So "In the end, you either believe or you don't" (yes, why have tests when you can have shouting matches?). In the end readers receive only titillation for the hard of thinking. Such outlets never notice that the two sides might be talking about different things (facts versus benefits) which are not mutually exclusive. It seems that media interest in astrology generally destroys any hope of informed debate.

Later, in the January/February 2004 issue of The Astrological Journal, AA President Roy Gillett added "Of course it is easy to answer the Dean/Kelly/Randy/'CSI COPS' (and all other) criticisms", presumably as easy as misspelling Randi and CSICOP, but he conveniently omitted to say how. Most likely he meant more of the same getting it wrong.

And in the January 2004 issue of Correlation, the AA's journal of research in astrology, Dr McGillion presents an update of his report. In it he claims the JCS article ignores relevant literature, is wordy, vague, illogical, factually incorrect, poorly researched, poorly edited, with imprecise definitions, nonsensical statements, loose terminology, fallacious reasoning, much unnecessary speculation, and much unnecessary material. So he is "not convinced it makes any meaningful contribution to consciousness research". Evidently Dr McGillion thinks the article and the entire JCS issue is (or should be) about consciousness research, not parapsychology. His focus is consequently wrong from the start, which makes most of his comments less than relevant. His rule that one should be relevant and "get it right" is evidently not one that he himself observes.

Inexplicably, Dr McGillion again mentions neither the aim of Dean and Kelly's article, nor their results, nor their conclusion, nor their discussion of artifacts and hidden persuaders, nor even the word parapsychology. He quotes the article out of context, and then uses the lack of context to ridicule the quote. He gives few details of the points he refers to, so his comments tend to read like riddles. The result is like commenting on a restaurant menu without mentioning food. See Appendix 4 for Dean and Kelly's point-by-point response to Dr McGillion.

It will be obvious by now that astrologers do not like awkward facts. But the reports presented by their top guns as counter evidence are essentially arguments by distortion and innuendo. Whatever we may think of astrology, it deserves better than this.


[As partes desta mensagem que não continham texto foram removidas]



##### ##### #####

Para saber mais visite
http://www.ciencialist.hpg.ig.com.br


##### ##### ##### #####


Yahoo! Grupos, um serviço oferecido por:







------------------------------------------------------------------------------
Links do Yahoo! Grupos

a.. Para visitar o site do seu grupo na web, acesse:
http://br.groups.yahoo.com/group/ciencialist/

b.. Para sair deste grupo, envie um e-mail para:
ciencialist-unsubscribe@yahoogrupos.com.br

c.. O uso que você faz do Yahoo! Grupos está sujeito aos Termos do Serviço do Yahoo!.



[As partes desta mensagem que não continham texto foram removidas]



SUBJECT: Ciência e conhecimento pré-científico (p/Homero e demais)
FROM: Manuel Bulcão <manuelbulcao@uol.com.br>
TO: ciencialist@yahoogrupos.com.br
DATE: 15/02/2005 20:52


Oi Homero, mais uma vez vou pegar no teu pé! :-)

Homero: Afinal, todas as outras crenças dizem a mesma coisa, que,
mesmo sem ter mecanismo de ação conhecido (como a homeopatia e a
acupuntura), elas funcionam, só não conseguem passar em experimentos
controlados com rigor.

Manuel: Não sei quanto à homeopatia, mas a acupuntura e a
fitoterapia não são meras crendices, isto é, crenças infundadas: são
conhecimentos empíricos -- embora não-científicos -- obtidos por
meio de tentativas e erros no decorrer de milênios. (Claro que a
explicação mágica que se dá à eficácia desses procedimentos, bem
como alguns rituais que os acompanham, são descartáveis. Óbvio,
também, que tanto a técnica da acupuntura quanto este e aquele
chazinho não são nenhuma panacéia, ao contrário, têm efeitos
curativos restritos)

Não é à toa que as grandes indústrias farmacêuticas têm amplo
interesse pelas ervas e tubérculos que compõem as infusões
medicinais dos ianomânis e outros povos da floresta. Não raro,
chegam mesmo a patenteá-los, a se apropriar furtiva e descaradamente
desse conhecimento empírico.

Ainda vale lembrar que muitos medicamentos alopáticos também são um
mistério no que diz respeito ao mecanismo de ação do seu princípio
ativo.

E mais uma vez repito: a ciência não é a única forma de conhecimento
capaz de embasar uma técnica eficiente. Existem outros, entre os
quais o conhecimento empírico assistemático, não metódico, e, diga-
se de passagem, durante a maior parte da sua existência na Terra, a
humanidade só contou com esse tipo de conhecimento pré-científico
para mitigar suas dores e sofrimentos.

Abraços,
Manuel Bulcão





SUBJECT: (não um hoax!) Foucault / Luz ondas ou corpusculos.
FROM: "murilo filo" <avalanchedrive@hotmail.com>
TO: ciencialist@yahoogrupos.com.br
DATE: 15/02/2005 23:00

Oráculo, Alberto e outros,
obrigado pelos comentários.
Todo mundo sabe e todo mundo ouviu falar que o Oráculo é um cético e
representante da obrigatória e útil dicotomia.
Estou falando de 1960. Meu tio se chamava Victor Gomes e era um profissional
da rede Manchete.
Seguramente seu equipamento era uma Leyka, ou Hasemblat, ou Rolleyflex, sei
lá, mas todas de 1ª e com zero de automatismos. Seu flash era aquele de
lâmpadas one-shot, muito fortes.
Eu já poderia ter testado esta coisa, mas não o fiz pq não duvido. Qual o
interesse dêle em mentir p/um moleque de 16 anos? Só vou dizer que êle não
era nenhum pé de chinelo, e sim um fodão, lá.
Quem duvida, que experimente. Vai ser interessante. Os cientistas são vcs...
:| Investiguem.
Trata-se de um treco tão bôbo de ser ensaiado...
Por falar nisto, vcs já repararam naquele guarda-chuva que serve como
difusor de luz nos fotos da vida, por aí. Vcs repararam que êle emite um
estalo quando o flash é disparado?
Será que o estalo se deve unicamente ao fator choque térmico??? Sei não...
Os cientistas são vcs... abr/M. SP 15/fev/2005

>From: "Oraculo" <oraculo@atibaia.com.br>
>Reply-To: ciencialist@yahoogrupos.com.br
>To: <ciencialist@yahoogrupos.com.br>
>Subject: [ciencialist] Mais um hoax?..:-) (era: Foucault / Luz ondas ou
>corpusculos.)
>Date: Tue, 15 Feb 2005 17:27:47 -0300
>
>
>Olá
>
>Essa estória do flash e da foto que "prova" a teoria corpuscular da luz
>parece algo destinado a se tornar mais uma lenda urbana. Como diversos
>mitos, surge sem informação suficiente, tem pouca possibilidade de ser
>real, quase não pode ser testada (a estória, não o experimento..:-), e
>ninguém ouviu falar dela no início.
>
>Mas com algum tempo, vai começar a ser citada a partir das citações
>iniciais, até se tornar algo que "todo mundo sabe" ou "todo mundo ouviu
>falar".
>
>O questionamento do César é perfeito, e todas essas perguntas deveriam ter
>sido feitas no momento da experiência. E as variáveis controladas antes de
>qualquer conclusão. Mesmo eu, que não sou fotógrafo, pensei que havia algo
>de errado no experimento, assim que li a mensagem, já que diversos outros
>fatores poderiam influenciar o resultado, inclusive a distorção perceptiva
>de seres humanos (do tipo, acho que ficou assim ou assado, mas pode ser
>apenas minha impressão..:-)
>
>O experimento seria simples demais de ser feito, para se manter tão
>incónito. Um sensor de luz, como os dos fotógrafos profissionais, seria
>posicionado diretamente no alcance de um facho de luz. Uma lanterna (ou
>flash) seria posicionado de forma a "empurrar" esse facho de luz. E o
>sensor mediria a diminuição sem problemas (se o flash pode impedir que o
>Sol sensibilize o filme, pode diminuir a medida do fotometro).
>
>Se fosse minimamente real, já teria sido feito ou pelo menso percebido
>pelos que estudam, a décadas, a luz e sua física.
>
>Um abraço.
>
>Homero
>
>
>
>
> ----- Original Message -----
> From: César A. K. Grossmann
> To: ciencialist@yahoogrupos.com.br
> Sent: Tuesday, February 15, 2005 4:07 PM
> Subject: [ciencialist] Re: Foucault / Luz ondas ou corpusculos.
>
>
>
> --- Em ciencialist@yahoogrupos.com.br, "Alberto Mesquita Filho" >
> > > >Há uns 45 anos, um tio meu, fotógrafo altamente profissional e de
>2ª
> > > >geração, contou-me que quando êle queria tirar uma foto CONTRA A
> LUZ DO
> > > >SOL, ÊLE USAVA O FLASH. E o flash evitava a indesejada
> sensibilização do
> > > >filme e a perturbação na lente. O flash, mais forte,
> ''empurrava'' a luz
> > > >solar... coisa linda! Nunca mais ouví alguém comentar sobre isto,
>que
> > > >aprendí nos meus 15 anos. Para mim, leigo, isto é uma prova de
> que a luz
> > > >é *matéria*, queiram ou não!
> > > >Alguém comenta? abr/Murilo SP 11/fev
>
> Uma perguntinha para o fotógrafo: quando ele liga o flash, a regulagem
> da máquina fotográfica permanece a mesma ou automaticamente o
> obturador fica um pouco menor? Sei que para as máquinas comuns,
> aquelas que os não profissionais usam, existe uma compensação
> automática, que acontece quando você liga o flash. Em máquinas
> profissionais, existem aquelas que tem várias velocidades para
> obturador, e uma especial para quando se usa flash.
>
> Seria interessante ter mais dados sobre a experiência: máquina
> (fabricante e modelo), flash, regulagem, ASA do filme, e duas fotos
> seguidas do mesmo objeto nas condições de 'contra a luz do Sol', uma
> com o flash e outra sem o flash.
>
> []s
> --
> .O. Cesar A. K. Grossmann ICQ UIN: 35659423
> ..O http://www.LinuxByGrossmann.cjb.net/
> OOO Timeo Danaos, et dona ferentes. (Virgilio)
>
>
>
>
>
> ##### ##### #####
>
> Para saber mais visite
> http://www.ciencialist.hpg.ig.com.br
>
>
> ##### ##### ##### #####
>
>
> Yahoo! Grupos, um serviço oferecido por:
>
>
>
>
>
>
>
>------------------------------------------------------------------------------
> Links do Yahoo! Grupos
>
> a.. Para visitar o site do seu grupo na web, acesse:
> http://br.groups.yahoo.com/group/ciencialist/
>
> b.. Para sair deste grupo, envie um e-mail para:
> ciencialist-unsubscribe@yahoogrupos.com.br
>
> c.. O uso que você faz do Yahoo! Grupos está sujeito aos Termos do
>Serviço do Yahoo!.
>
>
>
>[As partes desta mensagem que não continham texto foram removidas]
>
>
>
>##### ##### #####
>
>Para saber mais visite
>http://www.ciencialist.hpg.ig.com.br
>
>
>##### ##### ##### #####
>Links do Yahoo! Grupos
>
>
>
>
>
>
>
>




SUBJECT: Re: [ciencialist] Mais um hoax?..:-) (era: Foucault / Luz ondas ou corpusculos.)
FROM: José Renato <jrma@terra.com.br>
TO: <ciencialist@yahoogrupos.com.br>
DATE: 16/02/2005 00:10

É fato que qualquer fotógrafo usa flash quando é necessário iluminar o
objeto fotografado com luz ou sol por traz ou eliminar sombras provocadas
pelo sol entre 10 e 14 horas. Eu às vezes sou advertido ao usar flash em
plena praia em dia ensolarado, por estar gastando bateria à toa. O Murilo
ouviu o tio falar do uso do flash mas interpretou de forma fantasiosa a
questão.
[]s
JR
....................................................

From: "Oraculo" <oraculo@To: <ciencialist@yahoogrupos.com.br>
Sent: Tuesday, February 15, 2005 5:27 PM
Subject: [ciencialist] Mais um hoax?..:-) (era: Foucault / Luz ondas ou
corpusculos.)

Olá

Essa estória do flash e da foto que "prova" a teoria corpuscular da luz
parece algo destinado a se tornar mais uma lenda urbana. Como diversos
mitos, surge sem informação suficiente, tem pouca possibilidade de ser real,
quase não pode ser testada (a estória, não o experimento..:-), e ninguém
ouviu falar dela no início.

Mas com algum tempo, vai começar a ser citada a partir das citações
iniciais, até se tornar algo que "todo mundo sabe" ou "todo mundo ouviu
falar".

O questionamento do César é perfeito, e todas essas perguntas deveriam ter
sido feitas no momento da experiência. E as variáveis controladas antes de
qualquer conclusão. Mesmo eu, que não sou fotógrafo, pensei que havia algo
de errado no experimento, assim que li a mensagem, já que diversos outros
fatores poderiam influenciar o resultado, inclusive a distorção perceptiva
de seres humanos (do tipo, acho que ficou assim ou assado, mas pode ser
apenas minha impressão..:-)

O experimento seria simples demais de ser feito, para se manter tão
incónito. Um sensor de luz, como os dos fotógrafos profissionais, seria
posicionado diretamente no alcance de um facho de luz. Uma lanterna (ou
flash) seria posicionado de forma a "empurrar" esse facho de luz. E o sensor
mediria a diminuição sem problemas (se o flash pode impedir que o Sol
sensibilize o filme, pode diminuir a medida do fotometro).

Se fosse minimamente real, já teria sido feito ou pelo menso percebido pelos
que estudam, a décadas, a luz e sua física.

Um abraço.

Homero




----- Original Message -----
From: César A. K. Grossmann
To: ciencialist@yahoogrupos.com.br
Sent: Tuesday, February 15, 2005 4:07 PM
Subject: [ciencialist] Re: Foucault / Luz ondas ou corpusculos.



--- Em ciencialist@yahoogrupos.com.br, "Alberto Mesquita Filho" >
> > >Há uns 45 anos, um tio meu, fotógrafo altamente profissional e de 2ª
> > >geração, contou-me que quando êle queria tirar uma foto CONTRA A
LUZ DO
> > >SOL, ÊLE USAVA O FLASH. E o flash evitava a indesejada
sensibilização do
> > >filme e a perturbação na lente. O flash, mais forte,
''empurrava'' a luz
> > >solar... coisa linda! Nunca mais ouví alguém comentar sobre isto, que
> > >aprendí nos meus 15 anos. Para mim, leigo, isto é uma prova de
que a luz
> > >é *matéria*, queiram ou não!
> > >Alguém comenta? abr/Murilo SP 11/fev

Uma perguntinha para o fotógrafo: quando ele liga o flash, a regulagem
da máquina fotográfica permanece a mesma ou automaticamente o
obturador fica um pouco menor? Sei que para as máquinas comuns,
aquelas que os não profissionais usam, existe uma compensação
automática, que acontece quando você liga o flash. Em máquinas
profissionais, existem aquelas que tem várias velocidades para
obturador, e uma especial para quando se usa flash.

Seria interessante ter mais dados sobre a experiência: máquina
(fabricante e modelo), flash, regulagem, ASA do filme, e duas fotos
seguidas do mesmo objeto nas condições de 'contra a luz do Sol', uma
com o flash e outra sem o flash.

[]s
--
.O. Cesar A. K. Grossmann ICQ UIN: 35659423
..O http://www.LinuxByGrossmann.cjb.net/
OOO Timeo Danaos, et dona ferentes. (Virgilio)




SUBJECT: Re: [ciencialist] Ciência e conhecimento pré-científico (p/Homero e demais)
FROM: "Oraculo" <oraculo@atibaia.com.br>
TO: <ciencialist@yahoogrupos.com.br>
DATE: 16/02/2005 00:10

Olá Manuel

É sempre um prazer quando pega no meu pé (sem segundas intenções, por favor..:-)

Mas eu jamais colocaria a fitoterapia junto as outras duas "ciências"..:-) Não sei onde a encontrou (não em meu texto, espero..:-), mas é claro que principios ativos, de diversas moléculas e drogas, podem ser, e são, encontradas em plantas e a muto tempo usadas como tratamento. Tem mecanismo de ação, tem estudos de eficácia, tem desdobramentos e quase sempre é possível encontrar seu princípio ativo e refinar o medicamento.

Assim, nada contra a fitoterapia, embora em seu nome muitas charlatanices e trambiques tenham sido praticados..:-)

Mas a acupuntura, da forma definida pelo conhecimento ancestral é outra coisa. Não a atual acupuntura, que já discutimos, onde a agulhada pode interferir na sintese de prostaglandinas, mas a que reequilibra meridianos de enregia mistica não detectável, essa é que eu usei como exemplo..:-)

Também concordo com você, durante a maior parte de sua existencia a humanidade só contou com isso mesmo, mas temos de reconhecer que os resultados eram precarios, para dizer o mínimo..:-) E o que chama de conhecimento empírico sistematico, eu chamaria de proto-ciência, um arcabouço primitivo que lembra o método de testes da atual ciência, que pode produzir, com mais esforço e menos eficácia, dados suficietnes para conclusões razoavelmetne confiáveis. Mas, se colocar esse conhecimento na lente do método atual, teremos mais dados e mais confiabilidade. Como no caso de usar o conhecimento indigena como base de pesquisa, mas separar o que é mito do que é real, tirar das plantas e rituais o principio ativo verdadeiro e aplica-lo com mais eficiencia e adequaçào.

Afinal, plantas podem também matar ou causar grandes dano, e apenas um cuidado e estudo dirigido pode diminuir esse risco.

No mais, minha concordancia com quase todas suas colocações, lembrando, de novo, que meus exemplos foram usados apenas no que se refere a explicação mistica que possuem..:-)

Um abraço.

Homero

----- Original Message -----
From: Manuel Bulcão
To: ciencialist@yahoogrupos.com.br
Sent: Tuesday, February 15, 2005 7:52 PM
Subject: [ciencialist] Ciência e conhecimento pré-científico (p/Homero e demais)



Oi Homero, mais uma vez vou pegar no teu pé! :-)

Homero: Afinal, todas as outras crenças dizem a mesma coisa, que,
mesmo sem ter mecanismo de ação conhecido (como a homeopatia e a
acupuntura), elas funcionam, só não conseguem passar em experimentos
controlados com rigor.

Manuel: Não sei quanto à homeopatia, mas a acupuntura e a
fitoterapia não são meras crendices, isto é, crenças infundadas: são
conhecimentos empíricos -- embora não-científicos -- obtidos por
meio de tentativas e erros no decorrer de milênios. (Claro que a
explicação mágica que se dá à eficácia desses procedimentos, bem
como alguns rituais que os acompanham, são descartáveis. Óbvio,
também, que tanto a técnica da acupuntura quanto este e aquele
chazinho não são nenhuma panacéia, ao contrário, têm efeitos
curativos restritos)

Não é à toa que as grandes indústrias farmacêuticas têm amplo
interesse pelas ervas e tubérculos que compõem as infusões
medicinais dos ianomânis e outros povos da floresta. Não raro,
chegam mesmo a patenteá-los, a se apropriar furtiva e descaradamente
desse conhecimento empírico.

Ainda vale lembrar que muitos medicamentos alopáticos também são um
mistério no que diz respeito ao mecanismo de ação do seu princípio
ativo.

E mais uma vez repito: a ciência não é a única forma de conhecimento
capaz de embasar uma técnica eficiente. Existem outros, entre os
quais o conhecimento empírico assistemático, não metódico, e, diga-
se de passagem, durante a maior parte da sua existência na Terra, a
humanidade só contou com esse tipo de conhecimento pré-científico
para mitigar suas dores e sofrimentos.

Abraços,
Manuel Bulcão





##### ##### #####

Para saber mais visite
http://www.ciencialist.hpg.ig.com.br


##### ##### ##### #####


Yahoo! Grupos, um serviço oferecido por:
PUBLICIDADE




------------------------------------------------------------------------------
Links do Yahoo! Grupos

a.. Para visitar o site do seu grupo na web, acesse:
http://br.groups.yahoo.com/group/ciencialist/

b.. Para sair deste grupo, envie um e-mail para:
ciencialist-unsubscribe@yahoogrupos.com.br

c.. O uso que você faz do Yahoo! Grupos está sujeito aos Termos do Serviço do Yahoo!.



[As partes desta mensagem que não continham texto foram removidas]



SUBJECT: Re: [ciencialist] P/ Esteban ( Zodiaco e Acupuntura..)
FROM: "Oraculo" <oraculo@atibaia.com.br>
TO: <ciencialist@yahoogrupos.com.br>
DATE: 16/02/2005 00:19

Olá Esteban

Ops..:-) Não basta substituir a palavra, tem de apresentar os mesmos resultados. Vejamos, com as leis da termodinamica é possivel que seu automóvel funcione e o leve para muitos lugares, sempre de acordo com rigidos principios e cálculos. Novos motores pdoem ser projetados e previstos, sempre aumentando o conhecimento e sempre dentro das leis termodinamicas. Não se sabe de nenhum sistema que tenha sido construido desrespeitando essas leis..:-)

Também é possível fazer previsões fantasticas, como tempo de vida de estrelas ou sobre a troca de calor em processos industriais. E mesmo projetar diversos e diferentes dispositivos muito úteis a todos nós..:-)

O que a astrologia pode apresentar como exemplos de eficácia e confiabilidade de previsões?

Onde estão as centenas de anos de demonstração de eficácia? Você pode matar pessoas ou cura-las usando o conhecimento conhecido como leis da termodinamica, pode fazer isso com o conjunto de conhecimento conhecido como astrologia?

Pode, pelo menos, apresentar um único estudo rigoroso, dentro do método cientifico, de eficácia sobre a astrologia?

Se não pode (e nào sabemos de quem possa, afinal, astrologos nào estão interessados em provar nada, não é verdade?..:-), não basta trocar o termo termodinamica por astrologia..:-)

Esteban, o meu texto, onde a frase sobre centenas de anos está colocada, usa esse tempo, não como argumetno de autoridade ou de passagem de tempo, mas como exemplo de resultados sempre coerentes. Significa que, em centenas de anos, ninguém pode criar um experimento que violasse essas leis ou demonstrasse sua inobservancia. Não é o mesmo que dizer que a astrologia tem 3 mil anos, e por isso, apenas, deve ser verdadeira.

Um abraço.

Homero


----- Original Message -----
From: Esteban Moreno
To: ciencialist@yahoogrupos.com.br
Sent: Tuesday, February 15, 2005 6:39 PM
Subject: Re: [ciencialist] P/ Esteban ( Zodiaco e Acupuntura..)


Salve Oráculo!
Pegue está sua frase: "As leis da termodinâmica derivam de estudos,
evidencias, experimentos, resultados, etc, de centenas de anos, com
excelente demonstração de eficácia e de precisão. ", substitua
"termodinâmica" por "astrologia" e "centenas" por "milhares".
Um abraço,
Esteban.
.



##### ##### #####

Para saber mais visite
http://www.ciencialist.hpg.ig.com.br


##### ##### ##### #####


Yahoo! Grupos, um serviço oferecido por:
PUBLICIDADE




------------------------------------------------------------------------------
Links do Yahoo! Grupos

a.. Para visitar o site do seu grupo na web, acesse:
http://br.groups.yahoo.com/group/ciencialist/

b.. Para sair deste grupo, envie um e-mail para:
ciencialist-unsubscribe@yahoogrupos.com.br

c.. O uso que você faz do Yahoo! Grupos está sujeito aos Termos do Serviço do Yahoo!.



[As partes desta mensagem que não continham texto foram removidas]



SUBJECT: Re: (não um hoax!) Foucault / Luz ondas ou corpusculos.
FROM: "oraculo333" <oraculo@atibaia.com.br>
TO: ciencialist@yahoogrupos.com.br
DATE: 16/02/2005 00:24


Olá Murilo

Por favor, em nenhum momento eu duvidei da sinceridade de seu tio, ou
honestidade, nem penso que ele esteria mentindo para você. Também não
penso que ele não fosse hábil em sua profissão ou que seu equipamento
não fosse excelente.

O que ponderei é que, mesmo com tudo isso, ele pode simplesmente ter
mal interpretado os resultados, ter usado de avaliações subjetivas, ou
mesmo ter sincero convencimento de um fenomeno que é irreal. Nada
disso diminui sua posição, e pode ser um engano comu.

Apenas que, com milhares de pessoas estudando profundamente fenomenos
luminosos, notar que uma fonte de luz interfer dessa forma descrita em
outra fonte de luz, seria certamente algo simples e que teria ocorrido
diversas vezes em diferentes laboratórios.

Uma fonte de luz modifica nossa visão, muda a abertura da pupila,
assim como ajustes e registros de máquinas fotograficas (que são olhos
mecanicos afinal..:-), etc, etc, etc. Usar essa percepção subjetiva
para embasar uma alegação tão forte como a que apresentou, é arriscado
e prematuro..:-). Um bom fotometro poderia resolver a questão da
influencia de uma fonte de luz em outra rapidamente. E não há registro
de que algum instrumento tenha detectado essa influencia.

Um abraço.

Homero




--- Em ciencialist@yahoogrupos.com.br, "murilo filo"
<avalanchedrive@h...> escreveu
> Oráculo, Alberto e outros,
> obrigado pelos comentários.
> Todo mundo sabe e todo mundo ouviu falar que o Oráculo é um cético e
> representante da obrigatória e útil dicotomia.
> Estou falando de 1960. Meu tio se chamava Victor Gomes e era um
profissional
> da rede Manchete.
> Seguramente seu equipamento era uma Leyka, ou Hasemblat, ou
Rolleyflex, sei
> lá, mas todas de 1ª e com zero de automatismos. Seu flash era aquele de
> lâmpadas one-shot, muito fortes.
> Eu já poderia ter testado esta coisa, mas não o fiz pq não duvido.
Qual o
> interesse dêle em mentir p/um moleque de 16 anos? Só vou dizer que
êle não
> era nenhum pé de chinelo, e sim um fodão, lá.
> Quem duvida, que experimente. Vai ser interessante. Os cientistas
são vcs...
> :| Investiguem.
> Trata-se de um treco tão bôbo de ser ensaiado...
> Por falar nisto, vcs já repararam naquele guarda-chuva que serve como
> difusor de luz nos fotos da vida, por aí. Vcs repararam que êle
emite um
> estalo quando o flash é disparado?
> Será que o estalo se deve unicamente ao fator choque térmico??? Sei
não...
> Os cientistas são vcs... abr/M. SP 15/fev/2005
>
> >From: "Oraculo" <oraculo@a...>
> >Reply-To: ciencialist@yahoogrupos.com.br
> >To: <ciencialist@yahoogrupos.com.br>
> >Subject: [ciencialist] Mais um hoax?..:-) (era: Foucault / Luz
ondas ou
> >corpusculos.)
> >Date: Tue, 15 Feb 2005 17:27:47 -0300
> >
> >
> >Olá
> >
> >Essa estória do flash e da foto que "prova" a teoria corpuscular da
luz
> >parece algo destinado a se tornar mais uma lenda urbana. Como diversos
> >mitos, surge sem informação suficiente, tem pouca possibilidade de ser
> >real, quase não pode ser testada (a estória, não o experimento..:-), e
> >ninguém ouviu falar dela no início.
> >
> >Mas com algum tempo, vai começar a ser citada a partir das citações
> >iniciais, até se tornar algo que "todo mundo sabe" ou "todo mundo
ouviu
> >falar".
> >
> >O questionamento do César é perfeito, e todas essas perguntas
deveriam ter
> >sido feitas no momento da experiência. E as variáveis controladas
antes de
> >qualquer conclusão. Mesmo eu, que não sou fotógrafo, pensei que
havia algo
> >de errado no experimento, assim que li a mensagem, já que diversos
outros
> >fatores poderiam influenciar o resultado, inclusive a distorção
perceptiva
> >de seres humanos (do tipo, acho que ficou assim ou assado, mas pode
ser
> >apenas minha impressão..:-)
> >
> >O experimento seria simples demais de ser feito, para se manter tão
> >incónito. Um sensor de luz, como os dos fotógrafos profissionais,
seria
> >posicionado diretamente no alcance de um facho de luz. Uma lanterna
(ou
> >flash) seria posicionado de forma a "empurrar" esse facho de luz. E o
> >sensor mediria a diminuição sem problemas (se o flash pode impedir
que o
> >Sol sensibilize o filme, pode diminuir a medida do fotometro).
> >
> >Se fosse minimamente real, já teria sido feito ou pelo menso percebido
> >pelos que estudam, a décadas, a luz e sua física.
> >
> >Um abraço.
> >
> >Homero
> >
> >
> >
> >
> > ----- Original Message -----
> > From: César A. K. Grossmann
> > To: ciencialist@yahoogrupos.com.br
> > Sent: Tuesday, February 15, 2005 4:07 PM
> > Subject: [ciencialist] Re: Foucault / Luz ondas ou corpusculos.
> >
> >
> >
> > --- Em ciencialist@yahoogrupos.com.br, "Alberto Mesquita Filho" >
> > > > >Há uns 45 anos, um tio meu, fotógrafo altamente
profissional e de
> >2ª
> > > > >geração, contou-me que quando êle queria tirar uma foto
CONTRA A
> > LUZ DO
> > > > >SOL, ÊLE USAVA O FLASH. E o flash evitava a indesejada
> > sensibilização do
> > > > >filme e a perturbação na lente. O flash, mais forte,
> > ''empurrava'' a luz
> > > > >solar... coisa linda! Nunca mais ouví alguém comentar sobre
isto,
> >que
> > > > >aprendí nos meus 15 anos. Para mim, leigo, isto é uma prova de
> > que a luz
> > > > >é *matéria*, queiram ou não!
> > > > >Alguém comenta? abr/Murilo SP 11/fev
> >
> > Uma perguntinha para o fotógrafo: quando ele liga o flash, a
regulagem
> > da máquina fotográfica permanece a mesma ou automaticamente o
> > obturador fica um pouco menor? Sei que para as máquinas comuns,
> > aquelas que os não profissionais usam, existe uma compensação
> > automática, que acontece quando você liga o flash. Em máquinas
> > profissionais, existem aquelas que tem várias velocidades para
> > obturador, e uma especial para quando se usa flash.
> >
> > Seria interessante ter mais dados sobre a experiência: máquina
> > (fabricante e modelo), flash, regulagem, ASA do filme, e duas fotos
> > seguidas do mesmo objeto nas condições de 'contra a luz do Sol', uma
> > com o flash e outra sem o flash.
> >
> > []s
> > --
> > .O. Cesar A. K. Grossmann ICQ UIN: 35659423
> > ..O http://www.LinuxByGrossmann.cjb.net/
> > OOO Timeo Danaos, et dona ferentes. (Virgilio)
> >
> >
> >
> >
> >
> > ##### ##### #####
> >
> > Para saber mais visite
> > http://www.ciencialist.hpg.ig.com.br
> >
> >
> > ##### ##### ##### #####
> >
> >
> > Yahoo! Grupos, um serviço oferecido por:
> >
> >
> >
> >
> >
> >
> >
>
>------------------------------------------------------------------------------
> > Links do Yahoo! Grupos
> >
> > a.. Para visitar o site do seu grupo na web, acesse:
> > http://br.groups.yahoo.com/group/ciencialist/
> >
> > b.. Para sair deste grupo, envie um e-mail para:
> > ciencialist-unsubscribe@yahoogrupos.com.br
> >
> > c.. O uso que você faz do Yahoo! Grupos está sujeito aos
Termos do
> >Serviço do Yahoo!.
> >
> >
> >
> >[As partes desta mensagem que não continham texto foram removidas]
> >
> >
> >
> >##### ##### #####
> >
> >Para saber mais visite
> >http://www.ciencialist.hpg.ig.com.br
> >
> >
> >##### ##### ##### #####
> >Links do Yahoo! Grupos
> >
> >
> >
> >
> >
> >
> >
> >





SUBJECT: Re: [ciencialist] Astrologers vs Journal of Consciousness Studies
FROM: "Oraculo" <oraculo@atibaia.com.br>
TO: <ciencialist@yahoogrupos.com.br>
DATE: 16/02/2005 00:28

Olá Esteban

risos..:-) Bem, na verdade não despendo muito tempo, tudo já está pronto..:-) Esses estudos já foram feitos, os argumentos foram apresentados, as pesquisas lidas, etc. Apenas não encontramos nada que valide a alegação astrologica, embora ela sempre retorne, na mesma base e com os mesmos, e incorretos, argumentos e exemplos.

Não me custa nada, e é muito instrutivo para os que chegam agora no debate..:-)

Se a pessoa compreender que sua mente podoe engana-lo, se dispuser de dados sobre os processos mentais envolvidos, pode perceber que não há nada de real em um mapa astral, que não pudesse ser encontrado dentro de nossa mente mesmo e não fora.

Basta, como já foi feito antes, misturar mapas astrais de diversas pessoas, e notar que continuarão a dar grande grau de precisão e acerto aos mesmos..:-)

Deve ter lido a interessante parte dos textos que apresentei que trata do contato de Ryman com esse tipo de engano. Ele mesmo parecia ter descoberto que o sistema funcionava, mas um teste simples desmontou a ilusão..:-)

Esteban, os testes, inclusive o que propõe, já foram feitos. Nenhum astrólogo pode encontrar o mapa astral ou determinar quem é quem em um experimento controlado. E diversos foram feitos, com centenas de astrologos em diferentes paises. Confrontados com o teste que você mesmo propôs, falham.

Por que não faz um teste? Crie um mapa astral falso, com datas e horario de nascimento inventado, e apresente para alguém como real, se possível, ara alguém que sincerametne acredite em astrologia. Escute o que ele tem a dizer sobre o grau de acerto e nos conte como foi..:-) Será uma experiencia reveladora, no final..:-)

Um abraço.

Homero


----- Original Message -----
From: Esteban Moreno
To: ciencialist@yahoogrupos.com.br
Sent: Tuesday, February 15, 2005 7:04 PM
Subject: Re: [ciencialist] Astrologers vs Journal of Consciousness Studies


Oracle, eu francamente não compreendo por que dispensa tanto tempo em tentar provar algo que não acredita sem qualquer evidencia para tal. Temas complexos envolvem respostas complexas, como você disse anteriormente. Estude antes!
Amplexos descomplexos,
Esteban.



----- Original Message -----
From: Oraculo
To: ciencialist@yahoogrupos.com.br
Sent: Monday, February 14, 2005 11:23 PM
Subject: [ciencialist] Astrologers vs Journal of Consciousness Studies


Olá pessoal

O artigo abaixo é longo, mas vale a pena dar uma lida. Os apendices do texto não foram copiados aqui (ficaria grande demais para um email..:-) e podem ser lidos neste link:

http://www.rudolfhsmit.nl/d5-starwarsnew.htm

Um abraço.

Homero
___________________________________________________________
Star Wars


Astrologers vs Journal of Consciousness Studies


The special June/July 2003 issue of the prestigious Journal of Consciousness Studies was devoted to parapsychology, It contained twelve long articles, and the issue as a whole received praise for its balanced approach from New Scientist (13 September 2003 "authoritative and accessible"), and from Amazon ("serious and responsible"). But what caught media attention was the article entitled "Is Astrology Relevant to Consciousness and Psi?", a scholarly article of 24 pages and 85 references by Geoffrey Dean and Ivan W Kelly, which one astrology website later described as "Dean and Kelly rehashing old and flawed research hype". It led to the biggest media frenzy on astrology for 2003.







Two things make the frenzy of particular interest. First, its focus is not your everyday sun sign astrology but the serious astrology of conferences and consulting rooms. Second, the frenzy brought out some of astrology's top defensive guns, so you can judge how well they perform when confronted by scientific findings. (If you happen to be a True Believer you may prefer to stop reading now, or at least take a tranquilliser.)

Dean and Kelly start by quoting various astrologers on how a successful birth chart reading requires some kind of psychic ability, where the chart acts like a crystal ball. If this were found to be true it might require a re-assessment of present theories of consciousness, so it deserves study. But a large-scale test of 2101 persons born on average less than five minutes apart found no hint of the similarities in personality or behaviour predicted by astrology. So if astrologers (as opposed to astrology) can predict personality or behaviour better than chance, as they claim to do, it might be evidence for psi.

But meta-analysis of more than forty controlled studies found no evidence that astrologers perform even marginally better than chance, even on basic tasks such as predicting extraversion (basic because according to astrologers it is one of the easiest things to see in a birth chart). They did not even usefully agree on what the birth chart indicates. More to the point, astrologers who claimed to use psychic ability performed no better than those who did not. Dean and Kelly cautiously conclude "the possibility that astrology might be relevant to consciousness and psi is not denied, but such influences, if they exist in astrology, would seem to be very weak or very rare". See Appendix 1 for an abstract of their article.

Normally this cautious non-link between astrology and psi might have passed unnoticed. But it was picked up and distorted by the Sunday Telegraph London, 17 August 2003 ("Astrologers fail to predict proof they are wrong"), and was duly copied or quoted around the world from Brazil to Finland. It was distorted because Dean and Kelly's focus was psi and consciousness, not the merits of astrology (for example its merits include providing low-cost ego support, and astrologers are generally nice people), merits they had already covered in other articles such as their chapter in Paul Kurtz's Skeptical Odysseys, Prometheus 2001. The result was a frenzy of misleading headlines and reports such as "Is astrology bunk?" (Daily Mail London, 18 August), "Research paper rubbishes astrology" (Hindustan Times India, 17 August), and "Who will put their faith in the stars?" (Sunday Herald Sun Melbourne, 14 September).

Regardless of the distortion, astrologers were predictably outraged and uninformed. They declared that negative results are by definition due to ignorance or hostility. Especially outraged were Indian astrologers, one of whom said in emails "it is only a study by some crazy white b's. They do not have any brain". Another said "Most probably these two guys are unemployed". Another said "if you really want to test astrology ... the only place where the research can be justified is here in India" (where presumably men are Men and astrologers are Astrologers).

Perhaps the most notable response from India was "Astrology is Science, not Rubbish" (India Express 25 August), where Dr Raj Baldev "who is considered an authority on the subject of Astronomy, Astrology, Cosmo-Mathematics and Metaphysics" (he has a website www.occultastrology.com offering "The perfect gift. Occult Horoscope by post. Only $6.95") said Dean and Kelly had made "an abominable mistake that can never be pardoned". He explained that ancient Hindu astrology "is a complete science" where even one million billionth of a second "makes a lot of difference". So it is ridiculous to believe that people born a few minutes apart should be similar. (Measuring birth times to a million billionth of a second implies that the position of shadows cast on ancient sundials was routinely read to better than a hundred millionth of the diameter of an atom. Even at night. Should we believe it?)

Western astrologers did not hesitate to give opinions without having read the article. In a Melbourne radio interview, Brian Clarke from the Australian Federation of Astrologers explained how there was more to astrology than sun signs, so all was well (in fact the Dean and Kelly article had nothing to do with sun signs). In The Guardian London, 19 August, astrologer Neil Spencer noted how astrology can "send arch-rationalists into fits of self-righteous indignation" (like his?), how the article lacks details (not true), how it ignores the positive results of Vernon Clark and Gauquelin (not true), and how the "Magi Society [an international society of astrologers based in New York] ... still has to receive a riposte to its statistical challenges" (one is in Skeptical Inquirer March-April 1997). He ends with "Astrology is not a science but a symbolic, allusive language" (boo to Dr Baldev), as if that somehow excused its failure to deliver on testable claims, to which the sociology website www.butterfliesandwheels.com replied, "Oh that old ploy".

The most orchestrated response came from the AA (UK Astrological Association), which in 2000 had refused to publicly declare its position on sun signs despite overwhelming evidence for their invalidity (see Response to an invitation under Sun Signs). The AA president Roy Gillett accused Dean of seeking to "discredit astrology". The AA website www.astrologicalassociation.com accused Dean and Kelly of having a "tortured imagination" and "defensively closed mindsets" that "deny astrology an even-handed debate", to correct which it then gave "a balanced response" via two reports that together "comprehensively dismiss these outrageous and disingenuous claims".

The first report, which originally appeared in the Daily Mirror London, 18 August 2003, was by "world renowned astrologer Jonathan Cainer". Cainer has his own astrology phonelines, and according to the Sunday Times London his estimated income of 2.2 million pounds a year puts him among the top 150 UK earners. Cainer begins by complaining that it cost him 15 pounds to download the article, whereupon he was suspicious the moment he saw the authors' names (refutation by name-calling?), claiming that Dean deliberately misunderstands what astrologers do (Dean is a former astrologer and understands very well what astrologers do).

Cainer then drops a series of clangers -- personality tests are "dangerously unreliable" (not these ones), "most scientists hate astrology" (most have better things to do), the tests covered ages only up to 23 (so Cainer rejects phoneline callers under 23?). His most notable clanger was "Once again, it turns out to be an experiment rigged to make astrology look silly" (in fact it was the AA who proposed the experiment in the first place, not in a moment of weakness but in a well-researched eight-page proposal prepared when the subjects were seven years old). He ends with "And some scientists claim to have a truly open mind" (like his open-minded behaviour towards negative findings?). See Appendix 2 for Dean and Kelly's point-by-point response to Cainer.

The second report was by "leading author and academic Dr Frank McGillion", a consultant to the astrological research group at Southampton University, who is said to find the article "essentially flawed and well below expected academic standards". Dr McGillion begins by saying the article is too long, then it is too short. He quibbles about definitions without providing his own definitions, he dwells on side issues without explaining their relevance, he cites unspecified "evidence" without supporting references or arguments, and he generally declines to act the way he tells Dean and Kelly to act.He says a focus on consciousness and psi

"might seem less relevant ... than the authors appear to consider" (the call for papers required it), he dislikes "the citation of names of non-scientists in formal scientific papers" (so how to give astrologers' views without citing astrologers?), and he says editing such psi papers "is a demanding task for the non-specialist" (one editor was world expert James Alcock). At the end he generously allows that "there is much here that astrologers can learn from" (so Cainer was wrong to call it "a load of rubbish"?). Nevertheless conspicuously absent is a description of what Dean and Kelly did and a critique of their results (refutation by censorship?). See Appendix 3 for Dean and Kelly's point-by-point reponse to Dr McGillion.

Claims of credibility are clearly not helped when the AA, which calls itself "one of Europe's leading astrological organisations", can see these two reports as "a balanced response" and "even-handed debate".

Yet other astrology websites uncritically recycled the reports almost verbatim. For example www.astrology.co.uk said Dean and Kelly were "manipulating results ... using self-fulfilling personality tests ... selecting data to fit results", and dropped new clangers such as implying the average birth interval was one day (it was less than five minutes), and asking "who says astrologers are psychic?" (answer: the astrologers and surveys quoted). Another website said "Dean and Kelly are known to be opponents of astrology", as if seeking evidence was an act of heresy. Similarly, in The Mountain Astrologer (Dec/Jan 2003/2004 issue), the American astrologer Gloria Star (without reading the article) said "the research itself appears to be significantly flawed". No doubt her opinion will be repeated by other astrologers around the world, always without reading the article. Should we be surprised?

Without exception the astrologer responses boiled down to name-calling, getting it wrong, and never citing compelling evidence to support their claims. Even serious media outlets adopted this distortion. For example in the New Zealand Listener (4 October 2003), in an article subtitled "a recent study claims to debunk astrology once and for all" (not true), astrologers and skeptics are seen as being permanently locked in "trench wars" and "long feuds", serious studies are seen as attacks on astrology, researchers are seen as debunkers, and astrologer quotes such as "you will never get a correlation that is significant, because we are dealing with individuals" are seen as an adequate response to the negative meta-analysis even though it involved individuals. So "In the end, you either believe or you don't" (yes, why have tests when you can have shouting matches?). In the end readers receive only titillation for the hard of thinking. Such outlets never notice that the two sides might be talking about different things (facts versus benefits) which are not mutually exclusive. It seems that media interest in astrology generally destroys any hope of informed debate.

Later, in the January/February 2004 issue of The Astrological Journal, AA President Roy Gillett added "Of course it is easy to answer the Dean/Kelly/Randy/'CSI COPS' (and all other) criticisms", presumably as easy as misspelling Randi and CSICOP, but he conveniently omitted to say how. Most likely he meant more of the same getting it wrong.

And in the January 2004 issue of Correlation, the AA's journal of research in astrology, Dr McGillion presents an update of his report. In it he claims the JCS article ignores relevant literature, is wordy, vague, illogical, factually incorrect, poorly researched, poorly edited, with imprecise definitions, nonsensical statements, loose terminology, fallacious reasoning, much unnecessary speculation, and much unnecessary material. So he is "not convinced it makes any meaningful contribution to consciousness research". Evidently Dr McGillion thinks the article and the entire JCS issue is (or should be) about consciousness research, not parapsychology. His focus is consequently wrong from the start, which makes most of his comments less than relevant. His rule that one should be relevant and "get it right" is evidently not one that he himself observes.

Inexplicably, Dr McGillion again mentions neither the aim of Dean and Kelly's article, nor their results, nor their conclusion, nor their discussion of artifacts and hidden persuaders, nor even the word parapsychology. He quotes the article out of context, and then uses the lack of context to ridicule the quote. He gives few details of the points he refers to, so his comments tend to read like riddles. The result is like commenting on a restaurant menu without mentioning food. See Appendix 4 for Dean and Kelly's point-by-point response to Dr McGillion.

It will be obvious by now that astrologers do not like awkward facts. But the reports presented by their top guns as counter evidence are essentially arguments by distortion and innuendo. Whatever we may think of astrology, it deserves better than this.


[As partes desta mensagem que não continham texto foram removidas]



##### ##### #####

Para saber mais visite
http://www.ciencialist.hpg.ig.com.br


##### ##### ##### #####


Yahoo! Grupos, um serviço oferecido por:







------------------------------------------------------------------------------
Links do Yahoo! Grupos

a.. Para visitar o site do seu grupo na web, acesse:
http://br.groups.yahoo.com/group/ciencialist/

b.. Para sair deste grupo, envie um e-mail para:
ciencialist-unsubscribe@yahoogrupos.com.br

c.. O uso que você faz do Yahoo! Grupos está sujeito aos Termos do Serviço do Yahoo!.



[As partes desta mensagem que não continham texto foram removidas]



##### ##### #####

Para saber mais visite
http://www.ciencialist.hpg.ig.com.br


##### ##### ##### #####


Yahoo! Grupos, um serviço oferecido por:







------------------------------------------------------------------------------
Links do Yahoo! Grupos

a.. Para visitar o site do seu grupo na web, acesse:
http://br.groups.yahoo.com/group/ciencialist/

b.. Para sair deste grupo, envie um e-mail para:
ciencialist-unsubscribe@yahoogrupos.com.br

c.. O uso que você faz do Yahoo! Grupos está sujeito aos Termos do Serviço do Yahoo!.



[As partes desta mensagem que não continham texto foram removidas]



SUBJECT: Re: How to convince clients that astrology works
FROM: "oraculo333" <oraculo@atibaia.com.br>
TO: ciencialist@yahoogrupos.com.br
DATE: 16/02/2005 00:44


Olá Esteban

"Esteban: Desmente este pseudocientificismo que nos apresenta.Posso
envia-lo em anexo."

Não é necessário, Esteban, não é relevante..:-) Acho que não entendeu
ou não fui claro no uso destes argumentos com relação a astrologia.
Nada neles diz que é assim que TODA astrologia age, mas que, sendo
possível que seja assim, é preciso primeiro examinar esse ponto, antes
de concluir por sua realidade. E demonstra que relatos e posturas do
tipo "funciona para mim" não servem de evidencia, justametne por isso.

Assim, enviar o seu texto não muda a questão (além do que posso enviar
outro testo refutando este texto, ad infinitum..:-)

Vou tentar outro exemplo, sem o uso da astrologia, para ver se sou
mais claro..:-)

Imagine que eu presencie um fenomeno ou evento que considere espantoso
(ou qualquer outra pessoa, não importa).

Antes de decidir examina-lo a fundo, de criar estudos sobre ele, posso
dividi-lo em duas categorias:

A) O fenomeno não pode ser reproduzido de nenhuma forma conhecida, com
o uso apenas de conhecimento natural, científico, tecnologia, truques,
processos mentais ou outro qualquer deste tipo. Eu usaria como exemplo
deste tipo, um braço amputado que nasce novametne no paciente..:-)

B) O fenomeno, apesar de espantoso, pode ser reproduzido com o uso de
conhecimentos naturais, leis fisicas, truques, uso de processos
mentais, ou de qualquer forma sem necessidade do sobrenatural. Existem
muitos exemplos, sendo os poderes de leitura de mente de mágicos de
palco e telepatas diversos, levitação, mediunidade, etc.

Se o fenomeno é do primeiro tipo, é pertinente começar seu estudo
diretamente nas explicações sobrenaturais, de forma a tentar entender
seus processos, funcionamento ou até mesmo sua eficácia.

Mas, se o fenomeno é do segundo tipo, se existem formas de fazer com
que pareça real, sem que ele necessáriamente seja real, devemos
primeiro examinar essa hipótese e eliminar a possibilidade.

É apenas isso, não uma prova de irrealidade da astrologia..:-)

Os processos citados neste estudo, como está apresentado em sua
introdução, apresentam diversas formas de fazer com que a astrologia
pareça ser real, sem necessidade de que SEJA real. Todas essas formas
produzem, nos clientes e nos astrólogos, a mesma sensação e impressão
de funcionamento real, embora nenhuma delas exiga que a astrologia
seja real.

Isso demonstra que, relatos pessoais, centenas de milhares, não fazem
uma evidencia confiável para a astrologia. Demonstra ainda que, para
saber se a astrologia é mesmo real, é preciso criar estudos e
experimentos que separem esses mecanismos e garantam a realidade do
processo e das alegações da astrologia.

Claro que sabemos que a simples existencia desses processos e
mecanismos, que reproduzem os poderes alegados da astrologia, por sí
só não significa que toda a astrologia fuinciona assim, ou que é
irreal. Significa apenas que, existindo processos e formas de simular
a realidade de um alegação, é preciso mais cuidado e estudo antes de
concluir pela realidade ou veracidade da mesma.

Como você mesmo apresentou, se astrólogos estivessem interessados em
provar algo, poderiam submeter seus poderes e alegações a testes como
encontrar a pessoa a partir de um perfil psicologico conhecido.
Infelizmente, para a astrologia, todos os estudos dentro do rigor
criados para o propósito de separar joio de trigo em relação a
astrologia, só encontrou joio, com astrologos não tendo mais sucesso
em encontrar correspondencias entre pessoas e perfis do que uma pessoa
normal escolhendo a esmo.

Isso de forma nenhuma é pseudo-ceticismo, mas ceticismo legítimo, "do
bom", base do pensamento crítico e ferramenta eficaz na análise de
alegações diversas e do universo físico em que vivemos..:-)

Um abraço.

Homero










--- Em ciencialist@yahoogrupos.com.br, "Esteban Moreno"
<estebanmoreno@i...> escreveu
> Dê um lida em:
> Self-attribution, sun-sign traits, and the alleged role of
favourableness as
> a moderator variable:
> long-term e.ect or artefact? Edgar Wunder* Gesellschaft fu¨r
Anomalistik,
> Postfach 1202, 69200 Sandhausen, Germany Personality and Individual
> Differences 35 (2003) 1783-1789 www.elsevier.com/locate/paid
>
> Desmente este pseudocientificismo que nos apresenta.Posso envia-lo
em anexo.
> E.
>
>
> ----- Original Message -----
> From: Oraculo
> To: ciencialist@yahoogrupos.com.br
> Sent: Monday, February 14, 2005 11:36 PM
> Subject: [ciencialist] How to convince clients that astrology works
>
>
> Chamada do artigo abaixo:
> Hidden persuaders are systematic errors in our reasoning that
explain why a totally invalid system can still seem to work. Explores
over 30 hidden persuaders which convince clients that astrology works.
All are in everyday use in astrology. None require that astrology be true.
>
> Tradução (meia boca..:-)
> Persuadores escondidos (de persuadir, convencer) são erros
sistemáticos em nosso raciocínio que explicam porque um sistema
totalmente inválido pode aparentar funcionar. Apresenta mais de 30
"hidden persuaders" que convencem clientes que a astrologia funciona.
Todos são usados em astrologia todos os dias. Nenhum requer que a
astrologia seja real.
>
> _______________________________________________________
> Hidden persuaders:
> How to convince clients that astrology works
> By Geoffrey Dean and Ivan W Kelly
>
> Unaided human reasoning is subject to systematic errors (we call
them hidden persuaders) that can explain why an experience-based
astrology should seem to work even if it were totally invalid. For
example hidden persuaders explain why tens of thousands of Western
tropical astrologers can say that in their experience Scorpios really
are intense, while hundreds of thousands of Eastern sidereal
astrologers can look at the same piece of sky, which they call Libra,
and agree that in their experience it is not intense but relaxed. The
same applies to all the others factors that astrologers disagree
about, which is most of them.
>
> Caution. You will be led seriously astray if you learn about
astrology without first learning about hidden persuaders. Unless you
can be sure that hidden persuaders have been ruled out, don't believe
what you read in astrology books. In particular the claim that
astrologers proudly and repeatedly make, that astrology is
unassailable because it is experience-based, is simply mistaken --
what they see as its strength is actually its weakness. If this sounds
preposterous, consider the following quote from psychologist Ray
Hyman, How not to test mediums, Skeptical Inquirer 27(1), 20-30,
January-February 2003. Hyman has devoted more than half a century to
the study of psychic and other readings, especially to why such
readings can seem so compelling.
>
> How not to test a reading
>
> "As a way to earn extra income, I began reading palms when I was
in my teens. At first, I was skeptical. I thought that people believed
in palmistry and other divination procedures because they could easily
fit very general statements to their particular situation. To
establish creadibility with my clients, I read books on palmistry and
gave readings according to the accepted interpretations for the lines,
shape of the fingers, mounds, and other indicators. I was astonished
by the reactions of my clients.
>
> "My clients consistently praised me for my accuracy even when I
told them very specific things about problems with their health and
other personal matters. I even would get phone calls from clients
telling me that a prediction that I had made for them had come true.
Within months of my entry into palm reading, I became a staunch
believer in its validity. My conviction was so strong that I convinced
my skeptical high school English teacher by giving him readings and
arguing with him. I later also convinced the head of the psychology
department where I was an undergraduate." (page 22)
>
> So far it could be any astrologer talking about astrology. They
read charts and they become staunch believers in astrology's validity.
As in Hyman's case, their experience seems totally compelling. But
this is not the way to test a reading. Hyman then did something every
astrologer should do but never does:
>
> "When I was a sophomore, majoring in journalism, a well-known
mentalist and trusted friend persuaded me to try an experiment in
which I would deliberately read a client's hand opposite to what the
signs in her hand indicated. I was shocked to discover that this
client insisted that this was the most accurate reading she had ever
experienced. As a result, I carried out more experiments with the same
outcome. It dawned on me that something important was going on.
Whatever it was, it had nothing to do with the lines in the hand. I
changed my major from journalism to psychology so that I could learn
why not only other people, but also I, could be so badly led astray."
(page 22)
>
> Hidden persuaders
>
> The answer to Hyman's puzzle is hidden persuaders, factors that
can make a vague reading seem so uncannily accurate that it becomes
almost impossible not to believe in the system's validity. There are
many hidden persuaders. The most important hidden persuader for
astrology generally is the consider-only-confirming-cases artifact
discussed in the Artifacts and Truth articles on this website, but for
a client consultation it becomes just one among thirty-four as listed
below. None of these hidden persuaders require that astrology be true,
yet each will (wrongly) convince astrologers and clients that
astrology works. All are in routine use in astrology consulting rooms.
For convenience we have grouped them under seven descriptive strategies:
>
> 1. Select initial hurdles
>
> a.. Preach to the converted (client predisposition)
> b.. Ignore everything on this website (ignorance is bliss)
> c.. The best things in life are not free (charging a fee)
> 2. Stifle chances of being wrong
>
> a.. Appeal to birth chart complexity (nonfalsifiability)
> b.. Avoid conflict, see what you believe (cognitive dissonance)
> c.. Believe what you cannot prove (unavailable data)
> d.. Remember the hits, forget the misses (selective memory)
> e.. Ask only confirming questions (stacking the deck).
> This is the consider-only-confirming-cases artifact.
> f.. Ignore disconfirming evidence (confirmation bias)
> g.. Deny that astrology can be tested (testability veto)
> 3. Use cues
>
> a.. Let context give the game away (vital statistics)
> b.. Let body language be your guide (cold reading)
> 4. Make astrology look good
>
> a.. The importance of first impressions (halo effect)
> b.. If it looks right then it is right (face validity)
> c.. Style is more important than content (Dr Fox effect)
> d.. Underestimate chance effects (chance baseline shift)
> e.. More is better (Aunt Fanny effect)
> 5. Make clients feel good
>
> a.. Use a kind heart to entice belief (tea and sympathy)
> b.. The power of positive thinking (Pollyanna principle)
> c.. It does us good if we think it does (placebo effect)
> d.. Having control makes us feel better (misattribution)
> e.. Just naming the unknown is enough (Rumpelstiltskin effect)
> f.. Closeness is its own reward (rapport)
> 6. Make the chart fit
>
> a.. Find meaning where none exists (faces in clouds)
> b.. Read specifics into generalities (Barnum effect)
> c.. See only what you want to see (illusory correlation)
> d.. Accentuate the positive (social desirability)
> e.. Be seduced by resemblance (magical thinking)
> f.. Afterwards we knew it all along (hindsight bias)
> g.. Sound arguments yes, sound data no (stereotypes)
> 7. Make the client fit
>
> a.. Find something, anything, to match the chart (repertoire)
> b.. Let client role-play their chart (self-fulfilling prophecy)
> c.. Force client to fit their chart (Procrustean effect)
> d.. Winter does not last forever (regression to mean)
> Each hidden persuader reflects the systematic error in human
reasoning shown in parentheses, for which we have used the accepted
name if there is one, or a provisional name if not. In the early days
it was usual for critics to explain why clients were satisfied with
astrology readings in terms of the Barnum effect, the reading of
specifics into generalities such as "you have problems with money",
where sense appears to come from the reading when in fact it comes
from our ability to make sense out of vagueness. Today, as shown
above, many more hidden persuaders are known. They vary in
effectiveness, and in a given situation some may be irrelevant, but
all lead to client satisfaction and none require that astrology be true.
>
>
> But if clients are going to be satisfied, astrologers can hardly
fail to believe in astrology. In this way a vicious circle of
reinforcement is established whereby astrologers and their clients
become more and more convinced that astrology works. Note that there
are no hidden persuaders to convince us that astrology does not work
other than the informed critical mind, which of course is not an error
as such but rather a defence against errors. An astrologer typically
makes no effort to become informed about research or to acquire
critical thinking skills, preferring instead to spend years learning
to read charts, during which time they have ample chance to respond to
the above reinforcement. And of course to build up a huge vested
interest in continuing their ill-founded beliefs. If this still seems
preposterous, consider the salutary case of phrenology.
>
> The salutary case of phrenology
>
> The same hidden persuaders explain how phrenology (head reading),
once more popular and far more influential than astrology is today,
could be accepted as totally valid even though it is now known to be
totally invalid. As noted in 1985 by Dean and Mather (Astrological
Journal 28(1), 23-30, Winter 1985):
>
> "Astrologers are like phrenologists: their systems cover the same
ground, they apply them to the same kinds of people, they turn the
same blind eye to the same lack of experimental evidence, and they are
convinced for precisely the same reasons that everything works. But
the phrenologists were wrong. So why shouldn't critics conclude for
precisely the same reasons that astrologers are wrong?" (page 25)
>
> That was nearly 20 years ago, but no reply from astrologers has
been forthcoming other than to dismiss phrenology as irrelevant. But
perhaps no reply is required. After all, it could be argued that the
existence of mutually incompatible systems throughout astrology (for
example tropical and sidereal zodiacs), all of which are nevertheless
seen as completely valid by their users, has already put this question
to the test and given us convincing answers.
>
> The bottom line
>
> Thanks to hidden persuaders, the bottom line could hardly be
simpler --astrology in the consulting room does not need to be true.
This point was aired in a Skeptical Inquirer article published in
1986-1987, and in the later works listed below, where among other
things readings that were the opposite of what the chart indicated
were found to be as acceptable to clients as authentic readings, just
as Hyman had found for palmistry. But the point has been generally
ignored by astrologers. Readers interested in applying critical
thinking to astrology readings will find much useful information in
Gambrill (1990).
>
> Further reading
>
> Dean G (1992). Does astrology need to be true? In Frazier K (ed).
The Hundredth Monkey and other paradigms of the paranormal. Prometheus
Books, Amherst NY, pages 279-319 with 126 references. Update of a
two-part article first published in Skeptical Inquirer 1986-1987. The
answer to the title question is no.
> Dean G, Mather A & Kelly IW (1996). Astrology. In Stein G (ed).
Encyclopedia of the Paranormal. Prometheus Books, Amherst NY, pages
47-99 with 15 general references.
> Dean G & Kelly IW (2000). Does astrology work? Astrology and
skepticism 1975-2000. In Kurtz P (ed). Skepticism: A 25 Year
Retrospective. Prometheus Books, Amherst NY, pages 191-207.
> Gambrill E (1990). Critical Thinking in Clinical Practice:
Improving the Accuracy of Judgements and Decisions about Clients.
Jossey-Bass, San Francisco. 432 pages, 660 references. How to reduce
reasoning errors in psychology, medicine and the helping professions.
Well organised, packed with information, many examples, a few
references to astrology (but only to illustrate reasoning errors),
each chapter has a summary. Equally applicable to astrology. Should be
read by every astrologer.
>
>
> [As partes desta mensagem que não continham texto foram removidas]
>
>
>
> ##### ##### #####
>
> Para saber mais visite
> http://www.ciencialist.hpg.ig.com.br
>
>
> ##### ##### ##### #####
>
>
> Yahoo! Grupos, um serviço oferecido por:
>
>
>
>
>
>
>
>
------------------------------------------------------------------------------
> Links do Yahoo! Grupos
>
> a.. Para visitar o site do seu grupo na web, acesse:
> http://br.groups.yahoo.com/group/ciencialist/
>
> b.. Para sair deste grupo, envie um e-mail para:
> ciencialist-unsubscribe@yahoogrupos.com.br
>
> c.. O uso que você faz do Yahoo! Grupos está sujeito aos Termos
do Serviço do Yahoo!.
>
>
>
> [As partes desta mensagem que não continham texto foram removidas]





SUBJECT: Re: How to convince clients that astrology works
FROM: "oraculo333" <oraculo@atibaia.com.br>
TO: ciencialist@yahoogrupos.com.br
DATE: 16/02/2005 01:16



Olá Esteban

O texto abaixo é parte do estudo que enviou a lista, seu abstract,
como sendo capaz de desmentir o "pseudocientificismo" dos artigos que
postei.

Entretanto, ele faz justamente o contrário, pelo menos no
abstract..:-) Demonstra que em um abrangente estudo, a astrologia não
foi capaz de predizer nem mesmo quais signos (e as pessoas destes
signos) seriam mais propensos a acreditar em astrologia..:-)

Acho que vou aceitar seu oferecimento de enviar o artigo completo, já
que não consegui acessar pela página (eles cobram por artigo, e cobram
caro..:-)

Talvez possa entender melhor o que está pensando que o artigo defende
ou afirma, ok? Mande para os arquivos da lista, já que as mensagens
não aceitam anexos por email.

Um abraço.

Homero
_______________________________________

Self-attribution, sun-sign traits, and the alleged role of
favourableness as a moderator variable: long-term effect or artefact?
Edgar WunderCorresponding Author Contact Information, E-mail The
Corresponding Author
Gesellschaft für Anomalistik, Postfach 1202, 69200, Sandhausen, Germany
Received 17 June 2002; revised 12 November 2002; accepted 12
December 2002. ; Available online 1 April 2003.

Abstract

The process of self-attribution is an important factor in the
development of beliefs in the validity of presented personality
descriptions. Hamilton (2001) proposed that the relative
favourableness of astrologically derived personality descriptions is a
moderator variable for long-term self-attribution effects based on
knowledge of the astrological sun-sign symbolism. Because the sets of
traits associated with some sun-signs are thought to be more
favourable than those of other sun-signs, she predicts that natives
born under a more favourable sign should show a stronger belief in
astrology than subjects born under less favourable signs. To test this
hypothesis, we studied 1700 German subjects, to see if their belief in
astrology varied with respect to their sun-sign. But the mean belief
scores were almost exactly the same for all sign groups, providing
strong evidence against Hamilton's hypothesis. It is proposed that the
conflicting empirical findings of Hamilton (2001) are probably
artefacts of the experimental setting, and not long-term effects of
self-attribution.

Author Keywords: Self-attribution; Personality descriptions;
Confirmatory testing strategies; Astrology; Paranormal belief systems;
Favourableness






SUBJECT: Re: Foucault / Luz ondas ou corpusculos.
FROM: "Sergio M. M. Taborda" <sergiotaborda@terra.com.br>
TO: ciencialist@yahoogrupos.com.br
DATE: 16/02/2005 08:41


--- Em ciencialist@yahoogrupos.com.br, "murilo filo"
<avalanchedrive@h...> escreveu
> Oi!
> Não é por nada não, mas eu repito esta msg do dia 11, e só por uma
razão!
> Gostaria, honestamente, que algum sapiente da lista comentasse algo,
> confrontado com o que tem sido o atual conhecimento.
> Não é pegadinha, qualquer um pode experimentar.
> Obrigado pela consideração. abr/M. SP 14/fev

A explicação do fenomeno, embora eu não o conheça a fundo e possam
haver outras explicações melhores - e por isso eu não comentei o seu
texoto antes - é que a fotografia depende da luz recebida no filme.
A luz recebida no filme depende da interacção - no filme - da luz do
sol / ambiente, com o flash. Sendo que a luz do flash é muito mais
forte localmente que a do sol, o fenomeno de interferencia destrutiva
rparece rebater a luz do sol. Básicamente o efeito é o mesmo que
disparar contra o sol, pois a luz que inside no filme é menor que a
luz do sol. Mas o que é menor não é a quantidade de luz , mas a
intensidade da luz. E isso são duas coisas diferentes.
Embora a frase do seu familiar enha valor empirico, a explicação mais
aceite é muito mais complexa pois involve o concetio de interferencia,
e de que a quantidade da luz não é a mesma coisa que a intensidade da
luz.


Sérgio Taborda





SUBJECT: Re: ASTROLOGY IS BIGOTRY
FROM: "Sergio M. M. Taborda" <sergiotaborda@terra.com.br>
TO: ciencialist@yahoogrupos.com.br
DATE: 16/02/2005 09:35


--- Em ciencialist@yahoogrupos.com.br, "Oraculo" <oraculo@a...> escreveu

> The ram is conscious only of himself (4).
> His needs come first (4).
> Aries is concerned with the world only as it relates to himself (5).
> You can look for a liberal attitude, lavish generosity with both
time and material things (6).
> There's little that's graceful about the ram (6).
> The ram can also be the epitome of social grace (10).
>
> Are you getting the idea that if you read enough you will run across
virtually all facets of all behavior patterns? Bingo! The above
contradictions are separated in Goodman's text, so that you will not
stumble upon them without some effort. That is what attracts people to
astrology in the first place: the idea that they can get simple and
immediate answers to life's complex problems without effort.
>
> Tradução (meia boca..:-)
>
> Aries, março 21 até abril 20
> O carneiro é consciente apenas de sí mesmo
> Ele necessita vir em primeiro lugar
> Aries só se refere ao mundo quando este está relacionado a sí mesmo
> Você pode procurar por uma atitude liberal, generosidade pródiga com
o tempo e com as coisas materiais.
> Pouca coisa é graciosa no carneiro
> O carneiro pode ser a epitome da graça social
>
> Você está notando que, se ler o suficiente dos textos e afrimações,
poderá encontrar virutalmente todas as facetas de todo tipo de
comportamento?

Não. Não notei.
A consciencia apenas de si mesmo leva a que sempre virá em primeiro
lugar , pq não ha nada mais e da mesma forma, so se refere a aquilo
que diz respieto a sim proprio. Não é egoismo, o carneiro é
self-centered. (centrado em si mesmo)

"Você pode procurar por uma atitude liberal, generosidade prodiga para
com as coisas do tempo e as coisas materiais" ( a sua tradução
distorce o significado)
Vc pode procurar, mas será que encontra ?
O texto parece querer inferi que se o carneiro é egoista e portanto
ele não pode ser generoso. Ora, o problema é que ninguem disse que ele
é egoista.
Egoismo é a necessidade de apenas satisfazer a sua necessidade não
apenas ignorando a dos outros, mas querendo ignorar a dos outros.
O carneiro, embora self-centered não é egoista. Ele apenas gosta de
achar que o mundo deve ser como ele quer que seja. Isso não significa
que ele queira o mundo para si.
Por isso que ele é material e activo , pq se o mundo não for como ele
supõe e vai lá e muda-o. Ora, isso não é uma atitude egoista.
A sua necessidade de manipular o material através da acção (que é algo
relacionado ao tempo, uma coisa do tempo) implica em que ele conhece e
domina essas ferramentas, e por isso ele adota posições diferentes das
costumeiras devido à intimidade e uso repetido ( que leva à inovação)
que ele tem com essas coisas, e dai a sua actitude liberal. O
objectivo do carneiro é transformar o mundo naquilo que ele quer. É o
tipico : maome não vem À montanha, a montanha vem a maome
,literalmente. O carneiro procura resultados e não estilo (como o
touro). E por isso , de facto, suas acções não são graciosas (ou seja,
são deprovidas de beleza, estilo ou arte. São rudes, simples mas
eficazes.) Por isso, a falta de graciosidade.
O jogo de palavras gracefull e social grace parece querer dizer que
alguem que cai nas graças da sociedade tem que ser gracioso, ora isso
não é verdade.(veja-se Eisntein, por exemplo) Por isso, mesmo com a
sua rudesa, simplicidade e fala de colocar enfeites nas coisas, o
carneiro é visto bem socialmente pois ele está pronto a mudar o mundo
com as suas proprias mãos, coisa que o resto da sociedade não está.
Por isso ele é um elemento valioso, e todos dependem dele quando
querem o serviço feito. E por ele fazer esse serviço com gosto, ele é
repeitado na sociedade (social grace)

O cara que escreveu o site, nem sequer sabe ler. E o que ele está
fazendo é colectar partes do texto - FORA DE CONTEXTO (o que por si só
é um desmerito) e tentar que signifiquem o que ele quer (ou seja, que
pareçam contradições). Mas nem isso ele consegue.


É curioso notar que todos estes "gurus" do cepticismo são antigos
ilusionistas. É curioso, pq se pensarmos ma astrologia como uma forma
de ilusão, ela é concorrente directa do ilusionismo, pois, ao
contrario do ilusionismo - que ha truque e se sabe qual é - se na
astrologia houver um truque, ninguem sabe qual é. ( E o primeiro a
descobrir será venerado)
O mesmo é válido para as outras areas chamadas do paranormal.
Ou seja, é tudo dor de cotovelo por causa da concorencia.

Ha quem, desmitifique os falsos porque eles usam esses truques, ha
quem use esses truques para enganar dizendo que estão demonstrando que
as coisas não são reais. Bom, em toda a profissão ha enganação. E
quando menos a sociedade conhece a profissão, mais enganação ha. (quem
não ouvio falar de empreteiros que robam material ? Mas quem aqui sabe
fazer casas?)
Enquanto a tentativa de prova por esses senhores, de que não ha
fantasmas, nem ovnis (entenda-se : avistamento de naves alienigenas),
nem outras coisas que tais, trouxe ao conhecimento a diferença entre
uma assombração e uma esquisofrenia, entre um ovni e um relampago
globolar, entre um truque de luz e som e EVP... ou seja, enquanto
trouxe a separação do que é real e não explicável do que não é real,
ou é outro tipo de realidade (como a multipla personalidade é
diferente de fantasmas, mas existe). Enquanto tudo isso, o esforço é
válido e de merito. Mas quando esses senhores apenas fazem pouco
daquilo que nunca estudaram e não entendem, usando tecnicas brulescas
como a que vimos neste texto, é simplesmente uma palhaçada que não
merece consideração.
O problema é que essa palhaçada engana tanta gente, como a propria
coisa que se quer desmitificar. como eu disse, ha maus proficionais em
todas as areas... até no combate às fraudes.


Sérgio Taborda










SUBJECT: Re: Mais um hoax?..:-) (era: Foucault / Luz ondas ou corpusculos.)
FROM: César A. K. Grossmann <cesarakg@bol.com.br>
TO: ciencialist@yahoogrupos.com.br
DATE: 16/02/2005 10:28


--- Em ciencialist@yahoogrupos.com.br, "Oraculo" <oraculo@a...> escreveu
>
> O questionamento do César é perfeito, e todas essas perguntas
deveriam ter sido feitas no momento da experiência. E as variáveis
controladas antes de qualquer conclusão. Mesmo eu, que não sou
fotógrafo, pensei que havia algo de errado no experimento, assim que
li a mensagem, já que diversos outros fatores poderiam influenciar o
resultado, inclusive a distorção perceptiva de seres humanos (do tipo,
acho que ficou assim ou assado, mas pode ser apenas minha impressão..:-)

Tem mais uma que eu lembrei agora. Fiz uma foto contra a luz alguns
anos atrás, eu queria que o céu aparecesse queimado (andei estudando
fotografia alguns anos atrás, eu tinha uma câmera legal), só que na
hora de revelar o filme, o laboratório "corrigiu" a foto, para que o
céu não ficasse "queimado". Quer dizer, além de todos os fatores que
influenciam a qualidade da foto na hora de faze a mesma, ainda tem que
verificar com o laboratório como foi revelada a mesma.

[]s
--
.O. Cesar A. K. Grossmann ICQ UIN: 35659423
..O http://www.LinuxByGrossmann.cjb.net/
OOO Timeo Danaos, et dona ferentes. (Virgilio)





SUBJECT: Re: Astrologers rated these killers as good guys
FROM: César A. K. Grossmann <cesarakg@bol.com.br>
TO: ciencialist@yahoogrupos.com.br
DATE: 16/02/2005 10:32


--- Em ciencialist@yahoogrupos.com.br, "Esteban Moreno"
<estebanmoreno@i...> escreveu
> Se algum astrologo ler o mapa do Gandhi sem que ele esteja na sua
frente para depor, dirá que é um ser muito perturbado pisiquicamente.
E foi, tanto que batia na esposa, porém soube muito bem trabalhar com
as inclinações apontadas pelo mapa.

Gandhi batia na esposa?

[]s
--
.O. Cesar A. K. Grossmann ICQ UIN: 35659423
..O http://www.LinuxByGrossmann.cjb.net/
OOO Timeo Danaos, et dona ferentes. (Virgilio)





SUBJECT: Re: Ciência e conhecimento pré-científico (p/Homero e demais)
FROM: César A. K. Grossmann <cesarakg@bol.com.br>
TO: ciencialist@yahoogrupos.com.br
DATE: 16/02/2005 10:45


--- Em ciencialist@yahoogrupos.com.br, Manuel Bulcão
<manuelbulcao@u...> escreveu
>
> Homero: Afinal, todas as outras crenças dizem a mesma coisa, que,
> mesmo sem ter mecanismo de ação conhecido (como a homeopatia e a
> acupuntura), elas funcionam, só não conseguem passar em experimentos
> controlados com rigor.

Existe algum estudo sobre a acupuntura? Meu pai, quando jovem, fugiu
de um hospital de medo da cirurgia que faria: apendicectomia. Foi em
um acupunturista, e acabou curando-se. Claro, é uma "anedoctal
evidence", mas gostaria de saber o que há de sério por trás de enfiar
dezenas, quando não centenas, de agulhas em uma pessoa...

> Ainda vale lembrar que muitos medicamentos alopáticos também são um
> mistério no que diz respeito ao mecanismo de ação do seu princípio
> ativo.

É, só que nenhum homeopata vai dizer "não sabemos por que funciona",
enquanto você vai encontrar em bulas de remédio (como eu mesmo já
encontrei em alguns) "não se conhece o mecanismo de ação do
medicamento" ou parecido. Pelo menos são honestos ao confessar sua
ignorância...

[]s
--
.O. Cesar A. K. Grossmann ICQ UIN: 35659423
..O http://www.LinuxByGrossmann.cjb.net/
OOO Timeo Danaos, et dona ferentes. (Virgilio)





SUBJECT: Re: A guide for students doing projects - guia para projetos de estudos sobre a
FROM: "Sergio M. M. Taborda" <sergiotaborda@terra.com.br>
TO: ciencialist@yahoogrupos.com.br
DATE: 16/02/2005 10:46


--- Em ciencialist@yahoogrupos.com.br, "Oraculo" <oraculo@a...> escreveu
> O texto é bem longo.

Mas muito interessante. Ele reslata alguns erros básicos que os
proprios ceticos cometem mas incorre ele mesmo em erros.
Contudo, sendo que o objectivo é um projectos para Ciencias Sociais,
ou Estudos Sociais, como é melhor - vale o que vale. Mas vale
bastante. Vejamos:

É verdade que a maioria dos astrologos se nega ao teste levantando
vários argumentos. Esses argumentos derivam de efeitos sociais sobre
os astrologos. Ou seja, muitos são ensinados a pensar daquela formae
não têm conhecimento para pensar diferente. eles não dizem aquelas
coisas pq querem, mas pq não sabem melhor. Mas como o ppr texto diz,
não importa nada o que os astrologos acham. Importa que o estudo pode
ser feito mesmo assim. Portanto, não interessa nada a OPINIÂO dos
astrologos sobre a sua propria profissão/arte.


> Avoid open ended questions. Ask not "how does astrology work?" but
>"what is meant by astrology?" (...)

Gostei especialmente deste trexo, pq resume muito bem o que se passa
aqui na CienciaLista cada vez que o assunto é astrologia.
Os cepticos , descrentes e afins, insistem em perguntar "Como a
astrologya funciona" para concluir que não ha resposta a essa pergunta
e portanto ela não funciona. Isso é uma falacia.
Enquanto eu , e outros, nos deparamos com o problema de explicar que
não é essa a pergunta a fazer, não sem antes saber "o que se quer
dizer com 'Astrologia'".

> Projects about sun signs
>
> Signs are the most researched topic in astrology with well over one
hnndred empirical studies. Most studies are simply counts of people
born under various signs, but such counts are too contaminated by
ordinary influences (astronomy, sampling, demography, age incidence)
to mean anything. We can try correcting for such influences but in
practice the uncertainty is too great. So forget counting sun signs.

Como eu sempre disse. Estudos sobre signos solares não podem
representar estudos sérios sobre astrologia.


> The remaining studies, if adequately controlled against
non-astrological influences, have invariably been negative. Signs are
not only the most researched topic in astrology but are also the most
disconfirmed. Signs are simply not valid, not even slightly.

É curioso como se pode afirmar isto, depois de afirmar para
desconsiderar todo e qualquer estudo sobre signos. Mas pronto.
O ponto do texto é : esqueçam os signos solares pq não vai dar para
fazer um estudo serio. E isso é verdade.

> Sun signs seem valid because they cannot fail to fit (everyone, not
just Leos, will admit to being generous). They are also a good topic
of conversation and are nice to have around. People tend to like them,
but exactly why is not well understood despite many speculations.

Esta frase revela outra falacia normalmente usada pelos cepticos e
descrestes, e que está no amago do problema. Quem nunca estudou
astrologia, acha que uma caracteristica da personalidade tem uma
relação um para um com um determinado aspecto do mapa. Isso não é
verdadeiro. Existe mais do que uma forma da pessoa ter o mesmo traço
caracteristico. Isso é uma permissa teorica da satrologia que nunca é
considerada nos testes que se fazem. Os astrologos referem-se a isto
como a Complexidade do Mapa. Ou, Existem demasiadas variáveis.
E os cepticos entendem isto como desculpa de mau pagador.
Não é desculpa, é mesmo assim.
Qualquer estudo , que se diga, serio, tem que ter isto em consideração
ou não será válido Astrologicamente falando.
Um exemplo classico da fisica são os neutrinos. A teoria diz que
certas reacções nucleares emitem neutrinos, e portanto, logicamente, a
precença deles deveria significar que ocurreu aquela reação. Isso é
Errado fisicamente falando, pois uma das propriedades dos neutrinos é
serem fracamente absorvidos pela materia, e portanto , oneutrino
detectado no laboatorio pode não ter vindo daquele nucleo, mas do
nucleo do laboratorio ao lado, ou do sol, ou do outro lado do mundo.
Por isso que existem tecnicas nucleares especiais para lidar com o
facto de não podermos identificar de onde veio a particula.
Ora, em astrologia é a mesma coisa. Um traço da personalidade pode ter
várias origens. Aliás, para que ele possa ser considerada um traço de
personalidade ele TEM que vir de várias origens. É por isso que o
estudo apenas dos signos solares nunca vai dar em nada. Pois uma
pessoa de leão pode não parecer generosa, se um traço de personalidade
oposto for mais forte no seu mapa. (como saturno em touro em conjunção
com o regente da 2º casa)

Então, analizar um mapa é um proxesso complexo SIM e não vale tentar
fazer parecer que não.

> Belief in sun signs is known to slightly affect a person's
self-image and in some cases even their choice of job.

Qualquer pessoa que conheça astrologia minimanete sabe, que o singo
solar não tem um significado quanto à carreira. O regente (que é um
planeta e não um signo) da Casa 10 é muito mais relevante. Esse
regente pode ser o regente do signo solar do nativo, mas isso é
meramente conincidencia.
O leigo pode achar que o seu signo solar é tudo o que ha saber, alguns
mais evoluidos querem até saber o ascendente, mas isso é como so ler a
ultima linha do relatorio médico. E o resto ?

> The daily horoscope may well have an effect on daily mood. But
>measuring these things requires large samples, careful techniques,
>and lots of time, which puts them well beyond the resources of most
>projects.

Que é exactamente o que se quer dizer, quando se diz que um estudo
serio da astrologia é muito caro! não é invensão dos astrologos,
realmente é!

> Finally two easy projects that have been done before but are still
>informative and fun. (1) See if people can pick their own newspaper
>horoscope after you have mounted each one separately and removed all
>cues (they may need retyping). If picking is difficult, have people
>rank their top three or four choices. (2) Count agreements and
>disagreements between horoscopes from different newspapers or women's
>magazines.

Isto é baseado noutro argumento falacioso. A ideia aqui é que : SE
(hipotese) um mapa descreve uma pessoa, a pessoa deve ser capaz de o
identificar. Seria a mesma logica de identificar um suspeito num
alinhamento. SE (hipotese) a pessoa viu o sujeito, ela será capaz de
dizer qual é. Ou , SE (hipotese) a pessoa está doente, ela será capaz
de indentificar o relatorio medico da sua doença.
Pois bem. Se ela não souber identificar o sujeito, o que se conclui ?
Que nenhum dos caras no alinhamento é culpado ? Não. Apenas que ela
não soube identificar qual era. Se ela não souber identificar o
relatorio medico, o que se conclui? Que ela não está doente ? Não.
Da mesma forma, se a pessoa não sabe identificar o mapa, não se pode
concluir que a astrologia não funcioan. Apenas que as pessoas não
sabem identificar seus mapas.

Por outro lado ha pontos tecnicos aqui:
1) A astrologia não afirma que o nativo saiba identificar o seu mapa
de entre vários.
2) O que é apresentado a essas pessoas não é o mapa, mas a
intrepretação do mapa. E são duas coisas bem diferentes. (Mesmo que a
astrologia afirmasse que as pessoas devem saber identifcar seus maaps,
este passo suplementar invalidaria o teste)

a) O mapa é univoco. Cada pessoas tem o seu. A menos que a pessoa
tenha nascido exatamente ao mesmo tempo no mesmo lugar o que é
impossivel.
(um metro ou um segundo de difereça por ser computado no mapa. Não vai
alterar nada astronomicamente, mas os mapas podem ser astrologicamente
distintos pq ha conceitos de intervalos no meio desta historia a serem
considerados).

b) O significado de dois mapas para um certa caracteristica não é
univoco. Ou seja, dois mapas diferentes, podem resultar no mesmo
resultado, na mesma intrepretação, para uma caracteristica especifica.

c) A intrepretação escrita depende do seu autor. Embora os conceitos
sejam comuns a todos os astrologos,as suas formas de se expressarem
por escrito não o são. Embora o mapa do céu (o mapa propriamente dito,
aquela roda com um monte de riscos e simbolosque é calculado pela
astronomia) seja o mesmo para todos os astrologos, a forma como eles
expressam o significado não é.
A forma como alguem se expressa depende da sua cultura, dos seus
ideias e de factores que nada têm a haver com astrologia.

Aposto que se fosse mostrado mapa (a roda com simbolos) as pessoas não
conseguiriam saber qual é o seu, mesmo podendo ser calculado
facilmente. A menos que a pessoa usa-se um computador, ou levasse
todos os mapas para comparar com os calculos, que a pessoa - por ser
leiga- não saberá fazer.
Mas o que se pretende é que a pessoa descubra qual a intreprestação do
mapa que está de acordo com a sua visão de si mesma e da sua vida.
Ora, isso testa muitas coisas, mas não a astrologia. Isso é como o
teste das manchas de tinta. no fim, não interessa o que as mancaham
realmente são.

Faça-se então o mesmo teste com relatorios médicos. A pessoa tem que
saber escolher o seu de um monte deles. Os dados tecnicos são omitidos
, tal como o mapa é, e veremos o que acontece. Depois disso, a
validade da medicina terá que ser avaliada pelos resultados deste teste.

Quanto a mim, este tipo de testes são abusrdos. Os leigos não têm
obrigação de saber escolher nada. Por isso que eles são leigos, bolas!


Quanto aos astrologos, serem capazes de destingir pessoas é uma outro
ponto interessante.
O método astrologico é one-way-only. Ele não é injectivo, por isso
não pode ser revertido. Ou seja, se a intrepretação do mapa diz que a
pessoa tem a caracteristica A , e ela pode ser dado pelos aspectos X,Y
e Z , como escolher um mapa de entre os que têm esses aspectos ? É uma
sorte.

O erro dos ceticos e desconhecdores é sempre pensar que cada pessoa é
unica no universo, e que cada um tem o seu mapa, completamente
destinto do outro e que todo o provecco é univoco e injectável*.
Ora, não é nada disso. Em astrologia, cada pessoas não é unica. Ela
pertence a um sub-conjunto. Cada mapa não é destinto do outro. Podem
haver pontos em comum, e que isso seja verdade não é raro. Aliás é a
base da Sinastria. (estudo da compatibilidade astrologica). O processo
NÂO é univoco nem injectável.

---
*) injectável é um propriedade das funções que significa que se
F(x) = y , então existe G, tal que G(y)=x.
A função y = 2x é injectiva (para todo o x eu possa saber o y , e para
todo o y eu posso saber o x que lhe corresponde)
A função y = x*x não é injectiva. PAra cada x eu sei o y, mas para
cada y existem dois x possiveis e não tenho como saber qual foi o usado.
Um algoritmo matemático é reversivel se todas as funções do algoritmo
o são. O algoritmo da astrologia não é reversivel (ou seja, não ha
como chegar do resultado nos dados iniciais).
Univoco significa que para cada valor de x só ha um valor de y.
Existem funções que dado um x, resultam vários y. Por exemplo a raiz
quadrada. Para cada x existem dois y. A raiz quadrada de 4 é 2 e -2
Em matemática so usamos a raiz positiva (aquela operação representada
pelo racicial) mas sabemos que ha 2 raizes. Tecnicamente não é
correcto dizer que a raiz quadrade de 4 é 2. O correcto é dizer que a
Raiz quadrada POSITIVA de 4 é 2.
---

O texto é muito interessante, mas não tenho mais tempo. contudo já
disse o que mais importante, que é a resalva para a teoria astrologia
, pq certos testes nunca darão certo, e pq seus resultados nunca serão
aceites pela comunicade astrologica. E a desmistificação de algumas
ideias preconcebidas (pre-conceitos) sobre a teoria astrologica que
não estão de acordo com o que a teoria diz. Isto é fundamental para
analizar os vários experimentos apresentados aqui.

Sérgio taborda





SUBJECT: Re: Fw: a menor partícula
FROM: "Sergio M. M. Taborda" <sergiotaborda@terra.com.br>
TO: ciencialist@yahoogrupos.com.br
DATE: 16/02/2005 10:54


--- Em ciencialist@yahoogrupos.com.br, "Luiz Ferraz Netto"
<leobarretos@u...> escreveu

>
> Oi, me chamo Guiomar e voltei a estudar Quimica. Estou confusa. No
ano passado ouvi uma reportagem sobre uma particula menor que o átomo.
Por gentileza, eu poderia receber mais informações. Obrigada.

Embora existam muitas particulas menores que o atomo tais como
Electrões , Protões, Neutrões, Muões, e muitas mais... para a quimica
isso só tem importancia para explicar pq a Tabela Periodica é como é e
as propriedades dos elementos.

Contudo, é vital o conceito do Electrão e do Nucleo do Atomo, mesmo
sem saber que tipo coisas são essas, é fundamental entender que Toda a
quimica existe por causa do electrão. Que o estudo dos Iões é o estudo
do atomos que tem mais, ou menos, electrões do que deveriam ter. E que
as moleculas se formam devido ao electrões.
O grau de promenor em que vc pecisa estudar isto, depende do grau de
promenor que vc necessita para o seu uso da quimica.
Se vc necessitar de demasiado promenor pode ser necessário entender
fisica, e em particular Fisica Quântica.


Sérgio Taborda





SUBJECT: Re: A guide for students doing projects - guia para projetos de estudos sobre a
FROM: "rmtakata" <rmtakata@altavista.net>
TO: ciencialist@yahoogrupos.com.br
DATE: 16/02/2005 11:11


--- Em ciencialist@yahoogrupos.com.br, "Sergio M. M. Taborda"
> Gostei especialmente deste trexo, pq resume muito bem o que se passa
> aqui na CienciaLista cada vez que o assunto é astrologia.
> Os cepticos , descrentes e afins, insistem em perguntar "Como a
> astrologya funciona" para concluir que não ha resposta a essa

Pra mim pouco importa como a astromancia poderia funcionar. O q.
importa eh se ela funciona ou nao. Se funcionar entao iremos
investigar como. Mas nao funciona tto qto os dados dos experimentos
mostram.

Facam os astromantes as interpretacoes q. quiserem em cima dos mapas
natais, sua predicoes nao sao melhores do q. um bom chute.

Nao eh questao de ceticismo, mas tao somente q. os astromantes alegam
q. analisando mapas astrais sao capazes de indicar as melhores datas
para se realizar alguma coisa.

O resto eh conversa fiada.

[]s,

Roberto Takata





SUBJECT: Re: (não um hoax!) Foucault / Luz ondas ou corpusculos.
FROM: César A. K. Grossmann <cesarakg@bol.com.br>
TO: ciencialist@yahoogrupos.com.br
DATE: 16/02/2005 11:11


--- Em ciencialist@yahoogrupos.com.br, "murilo filo"
<avalanchedrive@h...> escreveu

> Estou falando de 1960. Meu tio se chamava Victor Gomes e era um
profissional
> da rede Manchete.
> Seguramente seu equipamento era uma Leyka, ou Hasemblat, ou
Rolleyflex, sei
> lá, mas todas de 1ª e com zero de automatismos. Seu flash era aquele de
> lâmpadas one-shot, muito fortes.

Não precisa de automatismos para a "regulagem automática" da máquina
para a mesma ter uma regulagem para fotos normais e outra para fotos
com flash. Tudo que você precisa fazer é mudar uma alavanca, ou girar
um botão, ou sei lá. Vamos dar uma olhada em uma Leica M3, de 1955?

http://www.photoethnography.com/ClassicCameras/index-frameset.html?LeicaM3.html~mainFrame

(ou http://tinyurl.com/6ccgf)

Parece bastante sofisticada, com todas as regulagens que um
profissional pode querer (ajuste de abertura e de velocidade), o que
não quer dizer que seja automatizada.

E ajuste "automático" nem sempre é "100% automático". Por exemplo, nas
câmeras comuns com flash integrado, quando você liga o flash, o mesmo
botão altera a regulagem do obturador e/ou da velocidade de exposição.
É automático, mas você tem que mover uma alavanca ou girar um botão...
As câmeras mais modernas são 100% automáticas, você aponta para onde
quer tirar a foto e elas já medem luminosidade e selecionam velocidade
e abertura, e ligam o flash se for o caso. Você só tem que apertar o
disparador...

Apenas para completar a coleção de links, a Hasselblad:

http://www.photoethnography.com/ClassicCameras/index-frameset.html?Hasselblad500.html~mainFrame
(ou http://tinyurl.com/5wnhg)

Não achei nada sobre a RolleyFlex. O nome está certo?

> Eu já poderia ter testado esta coisa, mas não o fiz pq não duvido.
Qual o
> interesse dêle em mentir p/um moleque de 16 anos? Só vou dizer que
êle não
> era nenhum pé de chinelo, e sim um fodão, lá.

Ele pode entender tudo de fotografia, mas isto dá a ele um diploma de
física?

> Trata-se de um treco tão bôbo de ser ensaiado...
> Por falar nisto, vcs já repararam naquele guarda-chuva que serve como
> difusor de luz nos fotos da vida, por aí. Vcs repararam que êle
emite um
> estalo quando o flash é disparado?
> Será que o estalo se deve unicamente ao fator choque térmico??? Sei
não...

Não pode ser uma carga de energia elétrica estática, provovada pelo
disparo do flash? Que eu saiba, estas lâmpadas trabalham com
capacitores e alta tensão...

> Os cientistas são vcs... abr/M. SP 15/fev/2005






SUBJECT: Re: Mais um hoax?..:-) (era: Foucault / Luz ondas ou corpusculos.)
FROM: César A. K. Grossmann <cesarakg@bol.com.br>
TO: ciencialist@yahoogrupos.com.br
DATE: 16/02/2005 11:16


--- Em ciencialist@yahoogrupos.com.br, José Renato <jrma@t...> escreveu
> É fato que qualquer fotógrafo usa flash quando é necessário iluminar o
> objeto fotografado com luz ou sol por traz ou eliminar sombras
provocadas
> pelo sol entre 10 e 14 horas. Eu às vezes sou advertido ao usar
flash em
> plena praia em dia ensolarado, por estar gastando bateria à toa. O
Murilo
> ouviu o tio falar do uso do flash mas interpretou de forma fantasiosa a
> questão.

Usa-se para diminuir o contraste luz/sombra, para deixar a sombra
menos escura. Será que era este tipo de fotografia que o Murilo estava
falando? Por que se for, então a explicação é absurda (fótons do flash
empurrando os fótons do sol...).

[]s
--
.O. Cesar A. K. Grossmann ICQ UIN: 35659423
..O http://www.LinuxByGrossmann.cjb.net/
OOO Timeo Danaos, et dona ferentes. (Virgilio)





SUBJECT: Re: Ciência e conhecimento pré-científico (p/Homero e demais)
FROM: Manuel Bulcão <manuelbulcao@uol.com.br>
TO: ciencialist@yahoogrupos.com.br
DATE: 16/02/2005 11:18


--- Em ciencialist@yahoogrupos.com.br, César A. K. Grossmann
<cesarakg@b...> escreveu
> É, só que nenhum homeopata vai dizer "não sabemos por que
funciona", enquanto você vai encontrar em bulas de remédio (como eu
mesmo já encontrei em alguns) "não se conhece o mecanismo de ação do
medicamento" ou parecido. Pelo menos são honestos ao confessar sua
ignorância...

Manuel: Decerto. No que diz respeito à homeopatia, também sou um
descrente. :-)

Abraços,
Manuel Bulcão





SUBJECT: Re: (não um hoax!) Foucault / Luz ondas ou corpusculos.
FROM: César A. K. Grossmann <cesarakg@bol.com.br>
TO: ciencialist@yahoogrupos.com.br
DATE: 16/02/2005 11:19


--- Em ciencialist@yahoogrupos.com.br, "oraculo333" <oraculo@a...>
escreveu
>
> Olá Murilo
>
> Por favor, em nenhum momento eu duvidei da sinceridade de seu tio, ou
> honestidade, nem penso que ele esteria mentindo para você. Também não
> penso que ele não fosse hábil em sua profissão ou que seu equipamento
> não fosse excelente.

Nem eu. Só acho que a explicação encontrada foi do tipo "fantástica".
Ou seja, fotógrafo, nota 100, cientista ou físico, nem tanto...

> Apenas que, com milhares de pessoas estudando profundamente fenomenos
> luminosos, notar que uma fonte de luz interfer dessa forma descrita em
> outra fonte de luz, seria certamente algo simples e que teria ocorrido
> diversas vezes em diferentes laboratórios.


Uma experiência bem simples é fazer o facho de luz de duas fontes de
luz diferente se cruzarem, e medir a distância que eles são desviados
um pelo outro...

[]s
--
.O. Cesar A. K. Grossmann ICQ UIN: 35659423
..O http://www.LinuxByGrossmann.cjb.net/
OOO Timeo Danaos, et dona ferentes. (Virgilio)





SUBJECT: Extraterrestre verossímil
FROM: Rodrigo Marques <rodmarq72@yahoo.com.br>
TO: Ceticismo Aberto <ceticismoaberto@yahoogrupos.com.br>, CienciaList <ciencialist@yahoogrupos.com.br>, Sociedade Brasileira de "C�ticos" e Racionalistas <sbcr@yahoogrupos.com.br>, Sociedade da Terra redonda <strbrasil@yahoogrupos.com.br>
DATE: 16/02/2005 11:23

Pessoal, eu queria saber as opiniões de vocês a respeito de um certo assunto:
- Caso existam realmente extraterrestres, como, de acordo com o conhecimento atual, poderiam ser a aparência destes seres?

abraços

__________________________________________________
Converse com seus amigos em tempo real com o Yahoo! Messenger
http://br.download.yahoo.com/messenger/

[As partes desta mensagem que não continham texto foram removidas]



SUBJECT: Re: Extraterrestre verossímil
FROM: César A. K. Grossmann <cesarakg@bol.com.br>
TO: ciencialist@yahoogrupos.com.br
DATE: 16/02/2005 11:37


--- Em ciencialist@yahoogrupos.com.br, Rodrigo Marques
<rodmarq72@y...> escreveu
> Pessoal, eu queria saber as opiniões de vocês a respeito de um certo
assunto:
> - Caso existam realmente extraterrestres, como, de acordo com o
conhecimento atual, poderiam ser a aparência destes seres?

Não sou exobiologista, mas aposto em seres unicelulares pró-cariontes,
popularmente conhecidos como BACTÉRIAS.

[]s
--
.O. Cesar A. K. Grossmann ICQ UIN: 35659423
..O http://www.LinuxByGrossmann.cjb.net/
OOO Timeo Danaos, et dona ferentes. (Virgilio)





SUBJECT: Re: Extraterrestre verossímil
FROM: "rmtakata" <rmtakata@altavista.net>
TO: ciencialist@yahoogrupos.com.br
DATE: 16/02/2005 11:38


--- Em ciencialist@yahoogrupos.com.br, Rodrigo Marques <rodmarq72@y...>
> - Caso existam realmente extraterrestres, como, de acordo com o
> conhecimento atual, poderiam ser a aparência destes seres?

Poderiam ser, em tese, de (quase) qq forma. Mas provavelmente nao
seriam nada parecidos conosco em aspecto externo - pela origem
independente e pela exposicao a regimes de selecao bem diversos.
Talvez se parecessem em aspectos basicos (resta saber o q. seriam tais
aspectos basicos: compartimentalizacao, baseados na quimica do
carbono, dependentes de agua liquida?).

Os astrobiologos imaginam q. formas de vida unicelulares -
bacteriformes - poderiam ser bem disseminadas. Formas mais complexas
seriam mais dificeis: alguns apostam em uma grande raridade.

Mas sao chutes um tto no escuro - soh conhecemos a vida da Terra e eh
dificil extrapolar a partir de um unico ponto.

[]s,

Roberto Takata





SUBJECT: Re: Extraterrestre verossímil
FROM: "rmtakata" <rmtakata@altavista.net>
TO: ciencialist@yahoogrupos.com.br
DATE: 16/02/2005 11:40


--- Em ciencialist@yahoogrupos.com.br, César A. K. Grossmann
> Não sou exobiologista, mas aposto em seres unicelulares pró-cariontes,
> popularmente conhecidos como BACTÉRIAS.

E por q. nao seres de plasma ou de campos eletromagneticos?

[]s,

Roberto Takata





SUBJECT: Re: Ciência e conhecimento pré-científico (p/Homero e demais)
FROM: Manuel Bulcão <manuelbulcao@uol.com.br>
TO: ciencialist@yahoogrupos.com.br
DATE: 16/02/2005 12:57


Oi César,

> Existe algum estudo sobre a acupuntura? Meu pai, quando jovem,
fugiu de um hospital de medo da cirurgia que faria: apendicectomia.
Foi em um acupunturista, e acabou curando-se. Claro, é uma "anedoctal
evidence", mas gostaria de saber o que há de sério por trás de enfiar
dezenas, quando não centenas, de agulhas em uma pessoa...

Manuel: Sobre o mecanismo de ação da acupuntura, o médico
acupunturista Fábio Pisani escreveu:

<<Este é um tema que durante todos estes séculos praticamente não
foi discutido, se sabia que a acupuntura funcionava e isto era o
suficiente. Nos últimos anos com a chegada da acupuntura ao
ocidente, começaram os questionamentos (que foi precedido pela
ignorância e pela ridicularização) e nas ultimas décadas começaram a
surgir as pesquisas cientificas. A teoria mais aceita hoje é a neuro-
imuno-endrócrina. A agulha estimula as terminações nervosas dos
músculos que mandam sinais para o sistema nervoso central que por
sua vez libera neurotransmissores (endorfinas). No sistema nervoso
central o estímulo da acupuntura se da em 3 níveis distintos: o
medular, o mesencefálico e o hipotalâmico. A estimulação do eixo
hipotálamo-hipofisário provoca a liberação de beta-endorfina. Esta
liberação por sua vez leva a produção da mesma quantidade de
cortisol, que atua em processos inflamatórios em geral. No nível do
mesencéfalo os neurônios são estimulados e liberam serotonina e
norepinefrina que inibem o impulso doloroso a nível medular. E
finalmente no nível medular os interneurônios da substância
gelatinosa liberam dinorfina e bloqueiam o impulso doloroso que se
propaga pelas fibras aferentes nociceptivas.>>

Ele ainda cita algo sobre a explicação "clássica":

<<Existe ainda um outro mecanismo pouco estudado pela ciência
ocidental que é a captação da energia do céu e da terra pelo ponto
de acupuntura, que na realidade é o que está na origem da teoria da
acupuntura.>>

Abraços,
Manuel Bulcão






SUBJECT: Re: Extraterrestre verossímil
FROM: Hélio Ricardo Carvalho <hrc@fis.puc-rio.br>
TO: ciencialist@yahoogrupos.com.br
DATE: 16/02/2005 13:01



Eu não acredito em extraterrestre VERO SÍMIO !!!!!

:-) :-) :-) :-)


Perco o amigo mas não perco o trocadilho.
(mesmo que infame)


[ ]'s

Hélio


--- Em ciencialist@yahoogrupos.com.br, "rmtakata" <rmtakata@a...>
escreveu
>
> --- Em ciencialist@yahoogrupos.com.br, César A. K. Grossmann
> > Não sou exobiologista, mas aposto em seres unicelulares pró-
cariontes,
> > popularmente conhecidos como BACTÉRIAS.
>
> E por q. nao seres de plasma ou de campos eletromagneticos?
>
> []s,
>
> Roberto Takata





SUBJECT: Re: Extraterrestre verossímil
FROM: Manuel Bulcão <manuelbulcao@uol.com.br>
TO: ciencialist@yahoogrupos.com.br
DATE: 16/02/2005 13:12


Oi,

Grossmann: Não sou exobiologista, mas aposto em seres unicelulares
pró-cariontes, popularmente conhecidos como BACTÉRIAS.

Takata: E por q. nao seres de plasma ou de campos eletromagneticos?

Manuel: Muito interessante a observação do Takata. Aliás, no
livro "Deus e a Nova Física" de Paul Davies, há a seguinte
passagem: "Começa a surgir entre os cientistas a noção de que nem a
mente nem a vida necessitam de se limitar à matéria orgânica. Num
livro recente e altamente especulativo, mas também mentalmente
estimulante -- 'Life Beyond Earth' -- as possibilidades de vida
extraterrestre são passadas em revista por Gerard Feinberg, físico,
e Robert Shapiro, bioquímico. Argumentam a favor da vida baseada em
plasmas, energia de campos eletromagnéticos, domínios magnéticos em
estrelas de neutrões e numa variedade de outros sistemas bizarros. A
consciência e a inteligência são conceitos de 'software': só o
padrão -- a organização -- conta, e não o meio de expressão." (Paul
Davies; Deus e a Nova Física; Edições 70; Lisboa - Portugal; 2000;
pág. 221)

Abraços,
Manuel Bulcão





SUBJECT: Re: Extraterrestre verossímil
FROM: César A. K. Grossmann <cesarakg@bol.com.br>
TO: ciencialist@yahoogrupos.com.br
DATE: 16/02/2005 13:48


--- Em ciencialist@yahoogrupos.com.br, "rmtakata" <rmtakata@a...> escreveu
>
> E por q. nao seres de plasma ou de campos eletromagneticos?

Boa pergunta. Eu não sei a resposta. Tenho um certo preconceito, sou
uma forma de vida baseada no carbono, tenho a tendência de achar que
vida é só o que for baseado no carbono, como eu.

Se existisse alguma forma de vida tão surreal (o Arthur Clarke gosta
de brincar com este tipo de possibilidade nos seus livros), será que
poderíamos detectá-la e reconhecê-la como tal? Nunca me esqueço da
sonda que os russos fizeram para detectar vida em outros planetas.
Antes de mandar ela para o espaço, fizeram ela testar o planeta Terra,
e a sonda chegou à conclusão que não há vida na Terra...

[]s
--
.O. Cesar A. K. Grossmann ICQ UIN: 35659423
..O http://www.LinuxByGrossmann.cjb.net/
OOO Timeo Danaos, et dona ferentes. (Virgilio)





SUBJECT: Re: Extraterrestre verossímil
FROM: "rmtakata" <rmtakata@altavista.net>
TO: ciencialist@yahoogrupos.com.br
DATE: 16/02/2005 14:02


--- Em ciencialist@yahoogrupos.com.br, César A. K. Grossmann
> Boa pergunta. Eu não sei a resposta. Tenho um certo preconceito, sou
> uma forma de vida baseada no carbono, tenho a tendência de achar que
> vida é só o que for baseado no carbono, como eu.

Bem, se for por tendencia, eu preferiria q. o Universo fosse habitado
por Cameron Diazes e Ana Paula Arósios.

> Antes de mandar ela para o espaço, fizeram ela testar o planeta Terra,
> e a sonda chegou à conclusão que não há vida na Terra...

Ué, a história q. eu conhecia era da sonda Clementine (acho q. era
americana) e detetou vida apenas por causa das ondas de rádio.

[]s,

Roberto Takata





SUBJECT: Astrologia - Pedido
FROM: rhalah_rikota <rhalah_rikota@ig.com.br>
TO: ciencialist@yahoogrupos.com.br
DATE: 16/02/2005 14:10

PessoALL,

Já há algum tempo que venho lendo as mensagens da Ciencialist, embora não
seja um contribuinte (tá bom. Podem me chamar de esponja) e me classificaria
como um cético.

Não seria melhor, dadas as discussões anteriores sobre a astrologia, que o
Sérgio Taborda e demais colegas "defensores" dissessem que tipo de previsão
é possivel se fazer (a mais simples possível), para, só então, fosse feita
uma pesquisa a respeito?

Se fosse o caso até poderíamos bolar um teste.

Da forma como o assunto vem sendo discutida, parece que o "ataque" está
falando de uma coisa e a "defesa" de outra.


[As partes desta mensagem que não continham texto foram removidas]



SUBJECT: Re: Astrologia - Pedido
FROM: "rayfisica" <rayfisica@yahoo.com.br>
TO: ciencialist@yahoogrupos.com.br
DATE: 16/02/2005 15:18


--- Em ciencialist@yahoogrupos.com.br, rhalah_rikota
<rhalah_rikota@i...> escreveu
> PessoALL,
>
> Já há algum tempo que venho lendo as mensagens da Ciencialist,
embora não
> seja um contribuinte (tá bom. Podem me chamar de esponja) e me
classificaria
> como um cético.
>
> Não seria melhor, dadas as discussões anteriores sobre a
astrologia, que o
> Sérgio Taborda e demais colegas "defensores" dissessem que tipo de
previsão
> é possivel se fazer (a mais simples possível), para, só então,
fosse feita
> uma pesquisa a respeito?
>
> Se fosse o caso até poderíamos bolar um teste.
>
> Da forma como o assunto vem sendo discutida, parece que o "ataque"
está
> falando de uma coisa e a "defesa" de outra.
>
>
> [As partes desta mensagem que não continham texto foram removidas]

>>>>>>>>>>>>>>>>>>>>>>>>>>>>>>>>>>>>>>>
>>>>>>>>>>>>>>>>>>>>>>>>>>>>>>>>>>>>>>>>>
ola, tudo bem
não se preocupe, isso é uma constante aqui
mas no fim tudo dá certo e ninguem se entende





SUBJECT: Re: [ciencialist] Extraterrestre verossímil
FROM: "Oraculo" <oraculo@atibaia.com.br>
TO: <ciencialist@yahoogrupos.com.br>
DATE: 16/02/2005 15:47

Olá Rodrigo

Como deve ter percebido pelas respostas dos co-listeiros, quase qualquer coisa seria possível, e o que podemos fazer, neste ponto, seria chutar mesmo..:-)

Só conhecemos vida a partir de carbono, com bases nitrogenadas, com apenas 4 nucleotideos (ou 5, contando o Uracil do RNA) e isso não dá margem a nenhuma certeza ou predição confiável.

Se encontrássemos vida em outro planeta ou corpo celeste, Marte ou Titã, por exemplo, e esta fosse de carbono, com bases nitrogenadas, bem parecida com a nossa na Terra, teriamos um dado a mais, a indicar que esse tipo de estrutura é comum no sistema solar. Embora não diga nada sobre outros sistemas, seria uma boa base para reforçar o palpite que outros seres vivos poderiam seguir o mesmo padrão.

Se seguirem, talvez fosse possível encontrar "formas básicas", como colocou o Takatra, onde um padrão como multicelular, DNA, sexo, estruturas simétricas, etc, pudessem ser esperados. Mesmo um golfinho e uma ave tem estruturas básicas parecidas..:-)

Mas mesmo assim é arriscado, insetos não tem estruturas básicas de vertebrados e podem apresentar muitas formas. Mesmo a simetria pode faltar, me parece, em alguns seres vivos.

Mas se encontrássemos vida totalmetne diferente, por exemplo, silicio no lugar do carbono, em Titã, isso levaria as coisas bem mais longe..:-) Se a vida pode surgir sem se parecer com a nossa no padrão mais básico, suas estruturas de informação hereditárias, isso muda todo o enfoque. Seres de outros locais do universo poderiam ser quase qualquer coisa, silicio, carbono ou outro elemento.

E isso sem falar no que nem reconehceriamos como vida ou ser vivo, baseados em campos eletromagneticos, eletricidade, e outras viajens da imaginação..:-)

Então, hoje, neste momento, com o que se sabe, podemos apenas fazer conjecturas (bem frágeis) sobre como seria a vida da forma como conhecemos, baseadas em carbono, multi ou unicelulares, DNA e outras pequenas variáveis. Mais que isso, é chute e dos grandes..:-)

Um abraço.

Homero


----- Original Message -----
From: Rodrigo Marques
To: Ceticismo Aberto ; CienciaList ; Sociedade Brasileira de Céticos e Racionalistas ; Sociedade da Terra redonda
Sent: Wednesday, February 16, 2005 10:23 AM
Subject: [ciencialist] Extraterrestre verossímil


Pessoal, eu queria saber as opiniões de vocês a respeito de um certo assunto:
- Caso existam realmente extraterrestres, como, de acordo com o conhecimento atual, poderiam ser a aparência destes seres?

abraços

__________________________________________________
Converse com seus amigos em tempo real com o Yahoo! Messenger
http://br.download.yahoo.com/messenger/

[As partes desta mensagem que não continham texto foram removidas]



##### ##### #####

Para saber mais visite
http://www.ciencialist.hpg.ig.com.br


##### ##### ##### #####


Yahoo! Grupos, um serviço oferecido por:

São Paulo Rio de Janeiro Curitiba Porto Alegre Belo Horizonte Brasília




------------------------------------------------------------------------------
Links do Yahoo! Grupos

a.. Para visitar o site do seu grupo na web, acesse:
http://br.groups.yahoo.com/group/ciencialist/

b.. Para sair deste grupo, envie um e-mail para:
ciencialist-unsubscribe@yahoogrupos.com.br

c.. O uso que você faz do Yahoo! Grupos está sujeito aos Termos do Serviço do Yahoo!.



[As partes desta mensagem que não continham texto foram removidas]



SUBJECT: Re: [ciencialist] Re: ASTROLOGY IS BIGOTRY
FROM: "Oraculo" <oraculo@atibaia.com.br>
TO: <ciencialist@yahoogrupos.com.br>
DATE: 16/02/2005 16:13

Olá Taborda

Você demonstra o famoso "conte os acertos, descarte os erros" com bastante propriedade..:-) Mas o argumento e as criticas a astrologia continuam incolumes, apenas quem acredita com muita fé pode manter essa postura por muito tempo..:-)

Texto:> Você está notando que, se ler o suficiente dos textos e afrimações,
poderá encontrar virutalmente todas as facetas de todo tipo de
comportamento?

Taborda: Não. Não notei. "


Bem, nào notou por implicancia, mas está bem claro..:-) Mesmo com minha péssima tradução (eu avisei que era meia boca, feita de madrugada sem muito cuidado..:-), fica evidente que o afirmado no artigo pode ser aplicado a qualquer caso, pela contrariedade de termos e oposição de aspectos.

Por exemplo, se quem lê o prognóstico não gosta da pessoa relatada, encontrará um claro "acerto" no egoísmo da avaliação. Já se quem lê gosta da pessoa, sua mãe por exemplo, pode encontrar o acerto na afirmação de generosidade (e ela certametne encontrará, como qualquer màe..:-) e pode aceitar que o egoismo é apenas "um forte desejo de mudar o mundo para melhor". Fugir desse argumento afirmando que "pode procurar, mas será que encontra" é ilegítimo. Se o astrólogo indica que pode procurar, é porque espera achar. Ou melhor, talvez seja mesmo para deixar a margem, o caminho da fuga, o uso de termos vagos o suficiente para permitir que, na falta de qualquer generosidade, sempre reste a explicaçào, "ei, procurar não significa achar"..:-)

É engraçado como os mesmos mecanismos que usam processos mentais e truques de semantica, apresentados no outro texto, sobre "persuaders hidden", possam ser encontrados em sua defesa da astrologia, com bastante clareza.

O que não se encontra é a apresentação de evidencias de eficácia do que defende. Como criacionistas, que tentam demonstrar que a evolução está errada, mas não tentam demonstrar que o criacionismo está correto (em termos de evidencias e provas, claro, eles sabem que é correto porque seu livro sagrado assim o diz..:-), este debate fica apenas tentando demonstrar que a crítica dos céticos está equivocada, mas não que a astrologia tem evidencias de ser real ou eficaz.

Nenhuma evidência ou demonstração, nenhum estudo ou experimento foi apresentado ou sugerido para isso. Apenas o interminavel desfile de acusações contra a "cegueira" dos céticos, quanto a "verdadeira astrologia" que ninguém conhece (nem os outros astrologos citados nas criticas céticas, aqueles pobres tolos charlatões..:-) só quem a defende, etc.

Apenas muito sobre como céticos não compreendem o que é a astrologia, mas NADA sobre o que é a astrologia..:-) De verdade..:-)

Brincar com palavras, definir egoismo como sendo isso ou aquilo, é divertido, mas é um truque semantico, não uma refutação do artigo. Se alguém diz que fulano é egoísta E generoso, ele preparou o palco, acendeu as luzes, jogou poeira nos olhos e espera que, não importa quem seja o alvo, encontre-se refletido de uma ou outra forma..:-) Mágica, luzes e espelhos, nada mais..:-)

Se a pessoa se vê egoista, não se incomodará com o termo. Se não se vê assim, tome jogo de palavras, autocentrado, generoso mas se o mundo não for como ele deseja, ele o mudará, etc, etc, etc.

Conversa para boi dormir, me parece (no bom sentido, por favor, se ofensa..:-)

Com essa abordaghem, é claro que a astrologia funciona ou parece funcionar. Brandindo termos e definições, ajustando seu significadao para se adequar a praticamente tudo, apresentando egoismo ora como força motivadora, ora como vício de caráter, tudo pode ser encaixado e um mapa astral, qualquer mapa astral, vai ser perfeitamente preciso. Mas, será preciso não importa a data e hora do nascimento, será preciso não importa para quem seja apresentado, será preciso não porque a astrologia funciona, mas porque os processos mentais e truques semanticos dão essa impressão.

Como os truques que usa nesta crítica ao artigo apresentado..:-)

Um abraço.

Homero



[As partes desta mensagem que não continham texto foram removidas]



SUBJECT: Re: ASTROLOGY IS BIGOTRY
FROM: "Sergio M. M. Taborda" <sergiotaborda@terra.com.br>
TO: ciencialist@yahoogrupos.com.br
DATE: 16/02/2005 17:24


--- Em ciencialist@yahoogrupos.com.br, "Oraculo" <oraculo@a...> escreveu
> Olá Taborda
>
> Você demonstra o famoso "conte os acertos, descarte os erros" com
bastante propriedade..:-) Mas o argumento e as criticas a astrologia
continuam incolumes, apenas quem acredita com muita fé pode manter
essa postura por muito tempo..:-)
>
> Texto:> Você está notando que, se ler o suficiente dos textos e
afrimações,
> poderá encontrar virutalmente todas as facetas de todo tipo de
> comportamento?
>
> Taborda: Não. Não notei. "
>
>
> Bem, nào notou por implicancia, mas está bem claro..:-) Mesmo com
minha péssima tradução (eu avisei que era meia boca, feita de
madrugada sem muito cuidado..:-), fica evidente que o afirmado no
artigo pode ser aplicado a qualquer caso, pela contrariedade de termos
e oposição de aspectos.
>
> Por exemplo, se quem lê o prognóstico não gosta da pessoa relatada,
encontrará um claro "acerto" no egoísmo da avaliação. Já se quem lê
gosta da pessoa, sua mãe por exemplo, pode encontrar o acerto na
afirmação de generosidade (e ela certametne encontrará, como qualquer
màe..:-) e pode aceitar que o egoismo é apenas "um forte desejo de
mudar o mundo para melhor". Fugir desse argumento afirmando que "pode
procurar, mas será que encontra" é ilegítimo. Se o astrólogo indica
que pode procurar, é porque espera achar. Ou melhor, talvez seja mesmo
para deixar a margem, o caminho da fuga, o uso de termos vagos o
suficiente para permitir que, na falta de qualquer generosidade,
sempre reste a explicaçào, "ei, procurar não significa achar"..:-)
>
> É engraçado como os mesmos mecanismos que usam processos mentais e
truques de semantica, apresentados no outro texto, sobre "persuaders
hidden", possam ser encontrados em sua defesa da astrologia, com
bastante clareza.
>
> O que não se encontra é a apresentação de evidencias de eficácia do
que defende. Como criacionistas, que tentam demonstrar que a evolução
está errada, mas não tentam demonstrar que o criacionismo está correto
(em termos de evidencias e provas, claro, eles sabem que é correto
porque seu livro sagrado assim o diz..:-), este debate fica apenas
tentando demonstrar que a crítica dos céticos está equivocada

Exactamente pq esse é meu unico, e fianl, objectivo.

Para chegar lá, passo por:
a) Mostrar que a critica dos ceticos não faz sentido pq desconsidera o
proprio modelo / teoria da astrologia.É baseado em informações
parciais e/ou erradas do modelo/teoria astrologica.
b) Que ao cometer a) os ceticos violam o primeiro principio do método
cientifico, sendo suas considerações não-cientificas também.
c) Que muitos dos argumentos dos ceticos tb são falacias, porque nem
sequer têm base cientifica, historica ou qq outra.
d) Que os ceticos cometem as mesmas falacias de argumentação que
apontam aos crentes.
e) Que eu não tenho opinião formada sobre a eficacia da astrologia.
f) que , sendo d) não me cabe apresentar provas seja de que tipo
forem, pois não é meu objectivo provar a astrologia, apenas desprovar
o que os ceticos dizem sobre ela. E para isso aporto as provas
quanto a mim necessárias.
g) que os ceticos são tão crentes no seu desacreditar como os crentes
no seu acreditar, pois nenhum dos dois tem provas do que afirma.


Sérgio Taborda








SUBJECT: Re: Extraterrestre verossímil
FROM: César A. K. Grossmann <cesarakg@bol.com.br>
TO: ciencialist@yahoogrupos.com.br
DATE: 16/02/2005 18:08


--- Em ciencialist@yahoogrupos.com.br, "rmtakata" <rmtakata@a...> escreveu
>
> Ué, a história q. eu conhecia era da sonda Clementine (acho q. era
> americana) e detetou vida apenas por causa das ondas de rádio.

Não faz muito eu vi um programa em que um dos "cabeções" dos antigos
programas espaciais soviéticos (venera?) contou sobre algumas
experiências deles, e alguns fracassos também...

Infelizmente não lembro quase nada, sequer o nome do projeto...

[]s
--
.O. Cesar A. K. Grossmann ICQ UIN: 35659423
..O http://www.LinuxByGrossmann.cjb.net/
OOO Timeo Danaos, et dona ferentes. (Virgilio)





SUBJECT: Re: Extraterrestre verossímil
FROM: Manuel Bulcão <manuelbulcao@uol.com.br>
TO: ciencialist@yahoogrupos.com.br
DATE: 16/02/2005 20:00


--- Em ciencialist@yahoogrupos.com.br, "Oraculo" <oraculo@a...>
escreveu
> Mas se encontrássemos vida totalmetne diferente, por exemplo,
silicio no lugar do carbono, em Titã, isso levaria as coisas bem
mais longe... Seres de outros locais do universo poderiam ser quase
qualquer coisa, silicio, carbono ou outro elemento.

Manuel: Mas o silício apresenta muitas desvantagens em relação ao
carbono, como as apontadas pelo Prof. Juan Oró, biofísico:

<<De fato, poderia em princípio existir vida a partir de outros
elementos que têm certa capacidade de formar polímeros, como, por
exemplo, o silício; inclusive muitos desses polímeros silícicos se
formam em combinação com carbono. Suponhamos, como hipótese de
trabalho, que o silício fosse um elemento comparável ao carbono do
ponto de vista da possível constituição dos seres vivos. Resultaria
que, ao medirmos a abundância de silício e de carbono no Universo,
verificaríamos que o silício é aproximadamente umas dez vezes menos
abundante que o carbono, de modo que aquela possibilidade de vida
fica reduzida em igual número de vezes. Ademais, e isto é
importantíssimo, para que haja vida é preciso que haja dissolventes
e os dissolventes mais abundantes são a água e o amoníaco; para que
ambos atuem como dissolventes têm de estar sob a forma líquida. A
água encontra-se em tal estado entre 10 e 100 graus centígrados,
aproximadamente, que é a exata gradação da escala de temperatura
normal; para liquefazer o amoníaco são necessárias temperaturas mais
baixas. A tais temperaturas, o silício é um sólido e as combinações
que forma com o oxigênio também são sólidas, ou seja, é difícil
transformá-lo em solução.
<<Outra desvantagem do silício em relação ao carbono é que quando
este se oxida forma óxidos, como são o monóxido e o dióxido de
carbono, gases que se encontram na atmosfera. Por exemplo, a maioria
dos vegetais utiliza este dióxido de carbono atmosférico para reduzi-
lo por meio da luz e transformá-lo em compostos constituintes dos
aminoácidos e dos açúcares das células. Tal não poderia ocorrer com
o silício, porque este, uma vez oxidado, toma a forma do dióxido que
conhecemos como pedra sílex, pederneira ou areia que não é gás.
<<Pode-se, pois, afirmar que se trata de três desvantagens
excessivas para que o silício possa comparar-se ao carbono em sua
capacidade de chegar a formar compostos poliméricos de interesse
para a formação dos seres vivos e, além disso, com qualquer outro
elemento as possibilidades são inferiores. Resumindo: o carbono é o
elemento mais rápido para formar complexos que depois possam se
transformar nos fundamentos das proteínas e dos ácidos nucleicos.>>
(A Origem da Vida; Biblioteca Salvat de Grandes Temas; Salvat
Editora do Brasil S/A; 1979; pp. 78/81)

[]s
Manuel Bulcão





SUBJECT: Re: [ciencialist] Mais um hoax?..:-) (era: Foucault / Luz ondas ou corpusculos.)
FROM: "murilo filo" <avalanchedrive@hotmail.com>
TO: ciencialist@yahoogrupos.com.br
DATE: 16/02/2005 21:42

Não senhor, não houve mal entendido.
O que não me pareceu necessário explicar é que NÃO estávamos falando de
fotos com o horizonte aberto, p/o infinito...
Tratávamos de fotos de objetivos e/ou pessoas a curta distância E CONTRA O
SOL. Um mínimo eu já sabia e até hoje há muitos fotógrafos profissionais na
família.
Vou acabar experimentando êsse treim. abr/M.

>From: José Renato <jrma@terra.com.br>
>Reply-To: ciencialist@yahoogrupos.com.br
>To: <ciencialist@yahoogrupos.com.br>
>Subject: Re: [ciencialist] Mais um hoax?..:-) (era: Foucault / Luz ondas ou
>corpusculos.)
>Date: Tue, 15 Feb 2005 23:10:37 -0300
>
>
>É fato que qualquer fotógrafo usa flash quando é necessário iluminar o
>objeto fotografado com luz ou sol por traz ou eliminar sombras provocadas
>pelo sol entre 10 e 14 horas. Eu às vezes sou advertido ao usar flash em
>plena praia em dia ensolarado, por estar gastando bateria à toa. O Murilo
>ouviu o tio falar do uso do flash mas interpretou de forma fantasiosa a
>questão.
>[]s
>JR
>....................................................
>
>From: "Oraculo" <oraculo@To: <ciencialist@yahoogrupos.com.br>
>Sent: Tuesday, February 15, 2005 5:27 PM
>Subject: [ciencialist] Mais um hoax?..:-) (era: Foucault / Luz ondas ou
>corpusculos.)
>
>Olá
>
>Essa estória do flash e da foto que "prova" a teoria corpuscular da luz
>parece algo destinado a se tornar mais uma lenda urbana. Como diversos
>mitos, surge sem informação suficiente, tem pouca possibilidade de ser
>real,
>quase não pode ser testada (a estória, não o experimento..:-), e ninguém
>ouviu falar dela no início.
>
>Mas com algum tempo, vai começar a ser citada a partir das citações
>iniciais, até se tornar algo que "todo mundo sabe" ou "todo mundo ouviu
>falar".
>
>O questionamento do César é perfeito, e todas essas perguntas deveriam ter
>sido feitas no momento da experiência. E as variáveis controladas antes de
>qualquer conclusão. Mesmo eu, que não sou fotógrafo, pensei que havia algo
>de errado no experimento, assim que li a mensagem, já que diversos outros
>fatores poderiam influenciar o resultado, inclusive a distorção perceptiva
>de seres humanos (do tipo, acho que ficou assim ou assado, mas pode ser
>apenas minha impressão..:-)
>
>O experimento seria simples demais de ser feito, para se manter tão
>incónito. Um sensor de luz, como os dos fotógrafos profissionais, seria
>posicionado diretamente no alcance de um facho de luz. Uma lanterna (ou
>flash) seria posicionado de forma a "empurrar" esse facho de luz. E o
>sensor
>mediria a diminuição sem problemas (se o flash pode impedir que o Sol
>sensibilize o filme, pode diminuir a medida do fotometro).
>
>Se fosse minimamente real, já teria sido feito ou pelo menso percebido
>pelos
>que estudam, a décadas, a luz e sua física.
>
>Um abraço.
>
>Homero
>
>
>
>
> ----- Original Message -----
> From: César A. K. Grossmann
> To: ciencialist@yahoogrupos.com.br
> Sent: Tuesday, February 15, 2005 4:07 PM
> Subject: [ciencialist] Re: Foucault / Luz ondas ou corpusculos.
>
>
>
> --- Em ciencialist@yahoogrupos.com.br, "Alberto Mesquita Filho" >
> > > >Há uns 45 anos, um tio meu, fotógrafo altamente profissional e de
>2ª
> > > >geração, contou-me que quando êle queria tirar uma foto CONTRA A
> LUZ DO
> > > >SOL, ÊLE USAVA O FLASH. E o flash evitava a indesejada
> sensibilização do
> > > >filme e a perturbação na lente. O flash, mais forte,
> ''empurrava'' a luz
> > > >solar... coisa linda! Nunca mais ouví alguém comentar sobre isto,
>que
> > > >aprendí nos meus 15 anos. Para mim, leigo, isto é uma prova de
> que a luz
> > > >é *matéria*, queiram ou não!
> > > >Alguém comenta? abr/Murilo SP 11/fev
>
> Uma perguntinha para o fotógrafo: quando ele liga o flash, a regulagem
> da máquina fotográfica permanece a mesma ou automaticamente o
> obturador fica um pouco menor? Sei que para as máquinas comuns,
> aquelas que os não profissionais usam, existe uma compensação
> automática, que acontece quando você liga o flash. Em máquinas
> profissionais, existem aquelas que tem várias velocidades para
> obturador, e uma especial para quando se usa flash.
>
> Seria interessante ter mais dados sobre a experiência: máquina
> (fabricante e modelo), flash, regulagem, ASA do filme, e duas fotos
> seguidas do mesmo objeto nas condições de 'contra a luz do Sol', uma
> com o flash e outra sem o flash.
>
> []s
> --
> .O. Cesar A. K. Grossmann ICQ UIN: 35659423
> ..O http://www.LinuxByGrossmann.cjb.net/
> OOO Timeo Danaos, et dona ferentes. (Virgilio)
>
>
>
>
>##### ##### #####
>
>Para saber mais visite
>http://www.ciencialist.hpg.ig.com.br
>
>
>##### ##### ##### #####
>Links do Yahoo! Grupos
>
>
>
>
>
>
>
>




SUBJECT: RE: [ciencialist] Re: (não um hoax!) Foucault / Luz ondas ou corpusculos.
FROM: "murilo filo" <avalanchedrive@hotmail.com>
TO: ciencialist@yahoogrupos.com.br
DATE: 16/02/2005 21:49

Concordo com vc.
Mas tenho visto tanta babacagem científica correr por aí, que nunca duvidei
que há muitas coisas que ficaram para trás, e talvez até para nunca.
Me parece que mesmo que isto seja comprovado (o que é fácil), o pessoal dos
fotons vai ainda conseguir qualquer outra nova justificativa. abr/M.

>From: "oraculo333" <oraculo@atibaia.com.br>
>Reply-To: ciencialist@yahoogrupos.com.br
>To: ciencialist@yahoogrupos.com.br
>Subject: [ciencialist] Re: (não um hoax!) Foucault / Luz ondas ou
>corpusculos.
>Date: Wed, 16 Feb 2005 02:24:57 -0000
>
>
>
>Olá Murilo
>
>Por favor, em nenhum momento eu duvidei da sinceridade de seu tio, ou
>honestidade, nem penso que ele esteria mentindo para você. Também não
>penso que ele não fosse hábil em sua profissão ou que seu equipamento
>não fosse excelente.
>
>O que ponderei é que, mesmo com tudo isso, ele pode simplesmente ter
>mal interpretado os resultados, ter usado de avaliações subjetivas, ou
>mesmo ter sincero convencimento de um fenomeno que é irreal. Nada
>disso diminui sua posição, e pode ser um engano comu.
>
>Apenas que, com milhares de pessoas estudando profundamente fenomenos
>luminosos, notar que uma fonte de luz interfer dessa forma descrita em
>outra fonte de luz, seria certamente algo simples e que teria ocorrido
>diversas vezes em diferentes laboratórios.
>
>Uma fonte de luz modifica nossa visão, muda a abertura da pupila,
>assim como ajustes e registros de máquinas fotograficas (que são olhos
>mecanicos afinal..:-), etc, etc, etc. Usar essa percepção subjetiva
>para embasar uma alegação tão forte como a que apresentou, é arriscado
>e prematuro..:-). Um bom fotometro poderia resolver a questão da
>influencia de uma fonte de luz em outra rapidamente. E não há registro
>de que algum instrumento tenha detectado essa influencia.
>
>Um abraço.
>
>Homero
>
>
>
>
>--- Em ciencialist@yahoogrupos.com.br, "murilo filo"
><avalanchedrive@h...> escreveu
> > Oráculo, Alberto e outros,
> > obrigado pelos comentários.
> > Todo mundo sabe e todo mundo ouviu falar que o Oráculo é um cético e
> > representante da obrigatória e útil dicotomia.
> > Estou falando de 1960. Meu tio se chamava Victor Gomes e era um
>profissional
> > da rede Manchete.
> > Seguramente seu equipamento era uma Leyka, ou Hasemblat, ou
>Rolleyflex, sei
> > lá, mas todas de 1ª e com zero de automatismos. Seu flash era aquele de
> > lâmpadas one-shot, muito fortes.
> > Eu já poderia ter testado esta coisa, mas não o fiz pq não duvido.
>Qual o
> > interesse dêle em mentir p/um moleque de 16 anos? Só vou dizer que
>êle não
> > era nenhum pé de chinelo, e sim um fodão, lá.
> > Quem duvida, que experimente. Vai ser interessante. Os cientistas
>são vcs...
> > :| Investiguem.
> > Trata-se de um treco tão bôbo de ser ensaiado...
> > Por falar nisto, vcs já repararam naquele guarda-chuva que serve como
> > difusor de luz nos fotos da vida, por aí. Vcs repararam que êle
>emite um
> > estalo quando o flash é disparado?
> > Será que o estalo se deve unicamente ao fator choque térmico??? Sei
>não...
> > Os cientistas são vcs... abr/M. SP 15/fev/2005
> >
> > >From: "Oraculo" <oraculo@a...>
> > >Reply-To: ciencialist@yahoogrupos.com.br
> > >To: <ciencialist@yahoogrupos.com.br>
> > >Subject: [ciencialist] Mais um hoax?..:-) (era: Foucault / Luz
>ondas ou
> > >corpusculos.)
> > >Date: Tue, 15 Feb 2005 17:27:47 -0300
> > >
> > >
> > >Olá
> > >
> > >Essa estória do flash e da foto que "prova" a teoria corpuscular da
>luz
> > >parece algo destinado a se tornar mais uma lenda urbana. Como diversos
> > >mitos, surge sem informação suficiente, tem pouca possibilidade de ser
> > >real, quase não pode ser testada (a estória, não o experimento..:-), e
> > >ninguém ouviu falar dela no início.
> > >
> > >Mas com algum tempo, vai começar a ser citada a partir das citações
> > >iniciais, até se tornar algo que "todo mundo sabe" ou "todo mundo
>ouviu
> > >falar".
> > >
> > >O questionamento do César é perfeito, e todas essas perguntas
>deveriam ter
> > >sido feitas no momento da experiência. E as variáveis controladas
>antes de
> > >qualquer conclusão. Mesmo eu, que não sou fotógrafo, pensei que
>havia algo
> > >de errado no experimento, assim que li a mensagem, já que diversos
>outros
> > >fatores poderiam influenciar o resultado, inclusive a distorção
>perceptiva
> > >de seres humanos (do tipo, acho que ficou assim ou assado, mas pode
>ser
> > >apenas minha impressão..:-)
> > >
> > >O experimento seria simples demais de ser feito, para se manter tão
> > >incónito. Um sensor de luz, como os dos fotógrafos profissionais,
>seria
> > >posicionado diretamente no alcance de um facho de luz. Uma lanterna
>(ou
> > >flash) seria posicionado de forma a "empurrar" esse facho de luz. E o
> > >sensor mediria a diminuição sem problemas (se o flash pode impedir
>que o
> > >Sol sensibilize o filme, pode diminuir a medida do fotometro).
> > >
> > >Se fosse minimamente real, já teria sido feito ou pelo menso percebido
> > >pelos que estudam, a décadas, a luz e sua física.
> > >
> > >Um abraço.
> > >
> > >Homero
> > >
> > >
> > >
> > >
> > > ----- Original Message -----
> > > From: César A. K. Grossmann
> > > To: ciencialist@yahoogrupos.com.br
> > > Sent: Tuesday, February 15, 2005 4:07 PM
> > > Subject: [ciencialist] Re: Foucault / Luz ondas ou corpusculos.
> > >
> > >
> > >
> > > --- Em ciencialist@yahoogrupos.com.br, "Alberto Mesquita Filho" >
> > > > > >Há uns 45 anos, um tio meu, fotógrafo altamente
>profissional e de
> > >2ª
> > > > > >geração, contou-me que quando êle queria tirar uma foto
>CONTRA A
> > > LUZ DO
> > > > > >SOL, ÊLE USAVA O FLASH. E o flash evitava a indesejada
> > > sensibilização do
> > > > > >filme e a perturbação na lente. O flash, mais forte,
> > > ''empurrava'' a luz
> > > > > >solar... coisa linda! Nunca mais ouví alguém comentar sobre
>isto,
> > >que
> > > > > >aprendí nos meus 15 anos. Para mim, leigo, isto é uma prova de
> > > que a luz
> > > > > >é *matéria*, queiram ou não!
> > > > > >Alguém comenta? abr/Murilo SP 11/fev
> > >
> > > Uma perguntinha para o fotógrafo: quando ele liga o flash, a
>regulagem
> > > da máquina fotográfica permanece a mesma ou automaticamente o
> > > obturador fica um pouco menor? Sei que para as máquinas comuns,
> > > aquelas que os não profissionais usam, existe uma compensação
> > > automática, que acontece quando você liga o flash. Em máquinas
> > > profissionais, existem aquelas que tem várias velocidades para
> > > obturador, e uma especial para quando se usa flash.
> > >
> > > Seria interessante ter mais dados sobre a experiência: máquina
> > > (fabricante e modelo), flash, regulagem, ASA do filme, e duas fotos
> > > seguidas do mesmo objeto nas condições de 'contra a luz do Sol', uma
> > > com o flash e outra sem o flash.
> > >
> > > []s
> > > --
> > > .O. Cesar A. K. Grossmann ICQ UIN: 35659423
> > > ..O http://www.LinuxByGrossmann.cjb.net/
> > > OOO Timeo Danaos, et dona ferentes. (Virgilio)
> > >
> > >
> > >
> > >
> > >
> > > ##### ##### #####
> > >
> > > Para saber mais visite
> > > http://www.ciencialist.hpg.ig.com.br
> > >
> > >
> > > ##### ##### ##### #####
> > >
> > >
> > > Yahoo! Grupos, um serviço oferecido por:
> > >
> > >
> > >
> > >
> > >
> > >
> > >
> >
> >------------------------------------------------------------------------------
> > > Links do Yahoo! Grupos
> > >
> > > a.. Para visitar o site do seu grupo na web, acesse:
> > > http://br.groups.yahoo.com/group/ciencialist/
> > >
> > > b.. Para sair deste grupo, envie um e-mail para:
> > > ciencialist-unsubscribe@yahoogrupos.com.br
> > >
> > > c.. O uso que você faz do Yahoo! Grupos está sujeito aos
>Termos do
> > >Serviço do Yahoo!.
> > >
> > >
> > >
> > >[As partes desta mensagem que não continham texto foram removidas]
> > >
> > >
> > >
> > >##### ##### #####
> > >
> > >Para saber mais visite
> > >http://www.ciencialist.hpg.ig.com.br
> > >
> > >
> > >##### ##### ##### #####
> > >Links do Yahoo! Grupos
> > >
> > >
> > >
> > >
> > >
> > >
> > >
> > >
>
>
>
>
>
>##### ##### #####
>
>Para saber mais visite
>http://www.ciencialist.hpg.ig.com.br
>
>
>##### ##### ##### #####
>Links do Yahoo! Grupos
>
>
>
>
>
>
>
>




SUBJECT: RE: [ciencialist] Re: Foucault / Luz ondas ou corpusculos.
FROM: "murilo filo" <avalanchedrive@hotmail.com>
TO: ciencialist@yahoogrupos.com.br
DATE: 16/02/2005 21:56

Obrigado, Serjão.
Mas continuo achando que havia só o efeito instantâneo de impedir a
contra-luz de atingir a objetiva. Algo pulsante, como uma frente de choque e
bem mais potente que o sol. abr/M.

>From: "Sergio M. M. Taborda" <sergiotaborda@terra.com.br>
>Reply-To: ciencialist@yahoogrupos.com.br
>To: ciencialist@yahoogrupos.com.br
>Subject: [ciencialist] Re: Foucault / Luz ondas ou corpusculos.
>Date: Wed, 16 Feb 2005 10:41:29 -0000
>
>
>
>--- Em ciencialist@yahoogrupos.com.br, "murilo filo"
><avalanchedrive@h...> escreveu
> > Oi!
> > Não é por nada não, mas eu repito esta msg do dia 11, e só por uma
>razão!
> > Gostaria, honestamente, que algum sapiente da lista comentasse algo,
> > confrontado com o que tem sido o atual conhecimento.
> > Não é pegadinha, qualquer um pode experimentar.
> > Obrigado pela consideração. abr/M. SP 14/fev
>
>A explicação do fenomeno, embora eu não o conheça a fundo e possam
>haver outras explicações melhores - e por isso eu não comentei o seu
>texoto antes - é que a fotografia depende da luz recebida no filme.
>A luz recebida no filme depende da interacção - no filme - da luz do
>sol / ambiente, com o flash. Sendo que a luz do flash é muito mais
>forte localmente que a do sol, o fenomeno de interferencia destrutiva
>rparece rebater a luz do sol. Básicamente o efeito é o mesmo que
>disparar contra o sol, pois a luz que inside no filme é menor que a
>luz do sol. Mas o que é menor não é a quantidade de luz , mas a
>intensidade da luz. E isso são duas coisas diferentes.
>Embora a frase do seu familiar enha valor empirico, a explicação mais
>aceite é muito mais complexa pois involve o concetio de interferencia,
>e de que a quantidade da luz não é a mesma coisa que a intensidade da
>luz.
>
>
>Sérgio Taborda
>
>
>
>
>
>##### ##### #####
>
>Para saber mais visite
>http://www.ciencialist.hpg.ig.com.br
>
>
>##### ##### ##### #####
>Links do Yahoo! Grupos
>
>
>
>
>
>
>
>




SUBJECT: RE: [ciencialist] Re: (não um hoax!) Foucault / Luz ondas ou corpusculos.
FROM: "murilo filo" <avalanchedrive@hotmail.com>
TO: ciencialist@yahoogrupos.com.br
DATE: 16/02/2005 22:12

Ok, ave Cesar.
Se eu ainda tivesse 16 anos iríamos discutir.
Não sei bem qual é a sua, mas está bem... vc é o bamba, ok?
Temos aquí um gênio da informação, mas que não lê, ou não entende, a exata
mensagem de alguém da lista. Quem é que disse que meu tio era um físico, ou
que assim se achava, meu?
P/que gastar tanto papo, se o que eu queria dizer (em resposta a mais
alguém) era justamente que as câmeras antigas não tinham automatismo algum,
e não davam perdão p/uma foto mal batida?
Elas tinham que ser comandadas e não havia 'edição' de fotos, além dos
clássicos retoques. M.

>From: César A. K. Grossmann <cesarakg@bol.com.br>
>Reply-To: ciencialist@yahoogrupos.com.br
>To: ciencialist@yahoogrupos.com.br
>Subject: [ciencialist] Re: (não um hoax!) Foucault / Luz ondas ou
>corpusculos.
>Date: Wed, 16 Feb 2005 13:11:43 -0000
>
>
>
>--- Em ciencialist@yahoogrupos.com.br, "murilo filo"
><avalanchedrive@h...> escreveu
>
> > Estou falando de 1960. Meu tio se chamava Victor Gomes e era um
>profissional
> > da rede Manchete.
> > Seguramente seu equipamento era uma Leyka, ou Hasemblat, ou
>Rolleyflex, sei
> > lá, mas todas de 1ª e com zero de automatismos. Seu flash era aquele de
> > lâmpadas one-shot, muito fortes.
>
>Não precisa de automatismos para a "regulagem automática" da máquina
>para a mesma ter uma regulagem para fotos normais e outra para fotos
>com flash. Tudo que você precisa fazer é mudar uma alavanca, ou girar
>um botão, ou sei lá. Vamos dar uma olhada em uma Leica M3, de 1955?
>
>http://www.photoethnography.com/ClassicCameras/index-frameset.html?LeicaM3.html~mainFrame
>
>(ou http://tinyurl.com/6ccgf)
>
>Parece bastante sofisticada, com todas as regulagens que um
>profissional pode querer (ajuste de abertura e de velocidade), o que
>não quer dizer que seja automatizada.
>
>E ajuste "automático" nem sempre é "100% automático". Por exemplo, nas
>câmeras comuns com flash integrado, quando você liga o flash, o mesmo
>botão altera a regulagem do obturador e/ou da velocidade de exposição.
>É automático, mas você tem que mover uma alavanca ou girar um botão...
>As câmeras mais modernas são 100% automáticas, você aponta para onde
>quer tirar a foto e elas já medem luminosidade e selecionam velocidade
>e abertura, e ligam o flash se for o caso. Você só tem que apertar o
>disparador...
>
>Apenas para completar a coleção de links, a Hasselblad:
>
>http://www.photoethnography.com/ClassicCameras/index-frameset.html?Hasselblad500.html~mainFrame
>(ou http://tinyurl.com/5wnhg)
>
>Não achei nada sobre a RolleyFlex. O nome está certo?
>
> > Eu já poderia ter testado esta coisa, mas não o fiz pq não duvido.
>Qual o
> > interesse dêle em mentir p/um moleque de 16 anos? Só vou dizer que
>êle não
> > era nenhum pé de chinelo, e sim um fodão, lá.
>
>Ele pode entender tudo de fotografia, mas isto dá a ele um diploma de
>física?
>
> > Trata-se de um treco tão bôbo de ser ensaiado...
> > Por falar nisto, vcs já repararam naquele guarda-chuva que serve como
> > difusor de luz nos fotos da vida, por aí. Vcs repararam que êle
>emite um
> > estalo quando o flash é disparado?
> > Será que o estalo se deve unicamente ao fator choque térmico??? Sei
>não...
>
>Não pode ser uma carga de energia elétrica estática, provovada pelo
>disparo do flash? Que eu saiba, estas lâmpadas trabalham com
>capacitores e alta tensão...
>
> > Os cientistas são vcs... abr/M. SP 15/fev/2005
>
>
>
>
>
>
>##### ##### #####
>
>Para saber mais visite
>http://www.ciencialist.hpg.ig.com.br
>
>
>##### ##### ##### #####
>Links do Yahoo! Grupos
>
>
>
>
>
>
>
>




SUBJECT: Re: [ciencialist] Re: Foucault / Luz ondas ou corpusculos.
FROM: "Luiz Ferraz Netto" <leobarretos@uol.com.br>
TO: <ciencialist@yahoogrupos.com.br>
DATE: 16/02/2005 23:08

No contexto (lá de baixo):

O espectro das luzes do flash e do sol (mesmo dentro da faixa visível) são muito diferentes. As parcelas com que cada cor participa num é diferente no outro; os tons, as matizes, são diferentes. Flash vem da 'queima' de filamento de magnésio (um tipo) ou resultado de corrente gasosa sob alta tensão, em gases ionizados. A interferência destrutiva de feixes tão díspares é improvável!
Por outro lado, interferência destrutiva( ou, parcialmente) só pode ocorrer com propagações de mesma direção --- coincidir justamente aquelas de mesma freqüência, defasadas de 180 graus, num mesmo eixo ... para o caso proposto ... bruxaria!

Se o fenômeno realmente ocorre (sem querer duvidar da descrição), a explicação não é essa de inferência da luz sobre a película sensível.

[]'

===========================
Luiz Ferraz Netto [Léo]
leobarretos@uol.com.br
http://www.feiradeciencias.com.br
===========================
-----Mensagem Original-----
De: "murilo filo" <avalanchedrive@hotmail.com>
Para: <ciencialist@yahoogrupos.com.br>
Enviada em: quarta-feira, 16 de fevereiro de 2005 20:56
Assunto: RE: [ciencialist] Re: Foucault / Luz ondas ou corpusculos.



Obrigado, Serjão.
Mas continuo achando que havia só o efeito instantâneo de impedir a
contra-luz de atingir a objetiva. Algo pulsante, como uma frente de choque e
bem mais potente que o sol. abr/M.

>From: "Sergio M. M. Taborda" <sergiotaborda@terra.com.br>
>Reply-To: ciencialist@yahoogrupos.com.br
>To: ciencialist@yahoogrupos.com.br
>Subject: [ciencialist] Re: Foucault / Luz ondas ou corpusculos.
>Date: Wed, 16 Feb 2005 10:41:29 -0000
>
>
>
>--- Em ciencialist@yahoogrupos.com.br, "murilo filo"
><avalanchedrive@h...> escreveu
> > Oi!
> > Não é por nada não, mas eu repito esta msg do dia 11, e só por uma
>razão!
> > Gostaria, honestamente, que algum sapiente da lista comentasse algo,
> > confrontado com o que tem sido o atual conhecimento.
> > Não é pegadinha, qualquer um pode experimentar.
> > Obrigado pela consideração. abr/M. SP 14/fev
>
>A explicação do fenomeno, embora eu não o conheça a fundo e possam
>haver outras explicações melhores - e por isso eu não comentei o seu
>texoto antes - é que a fotografia depende da luz recebida no filme.
>A luz recebida no filme depende da interacção - no filme - da luz do
>sol / ambiente, com o flash. Sendo que a luz do flash é muito mais
>forte localmente que a do sol, o fenomeno de interferencia destrutiva
>rparece rebater a luz do sol. Básicamente o efeito é o mesmo que
>disparar contra o sol, pois a luz que inside no filme é menor que a
>luz do sol. Mas o que é menor não é a quantidade de luz , mas a
>intensidade da luz. E isso são duas coisas diferentes.
>Embora a frase do seu familiar enha valor empirico, a explicação mais
>aceite é muito mais complexa pois involve o concetio de interferencia,
>e de que a quantidade da luz não é a mesma coisa que a intensidade da
>luz.
>
>
>Sérgio Taborda
>



--
No virus found in this outgoing message.
Checked by AVG Anti-Virus.
Version: 7.0.300 / Virus Database: 265.8.7 - Release Date: 10/02/2005



SUBJECT: Por 10 dias de graça e para saberem o que perdem...
FROM: Maria Natália <grasdic@hotmail.com>
TO: ciencialist@yahoogrupos.com.br
DATE: 16/02/2005 23:47


...por não terem céu em vossas cidades.
Aqui vai um pouco do que nos leva a estar na noite até às 8 da matina:
http://www.starrynight.com/
Mesmo não se dominando inglês se entende.
Corram já hoje.
Um abraço
Maria Natália





SUBJECT: "O único homem que está isento de erros, é aquele que...
FROM: Maria Natália <grasdic@hotmail.com>
TO: ciencialist@yahoogrupos.com.br
DATE: 17/02/2005 00:00


...não arrisca a acertar." Einstein o disse.
ARRISCA na Faculdade de Ciências da Universidae de Lisboa. ACERTA no
Futuro!
Este é o lema dos DIAS ABERTTOS , 14 e 15 de Março de 2005. Ano
Mundial da Física.
Só para alunos de ensino secundário e mediante marcação a FCUL estará
aberta com seus laboratórios, feira de cursos e ainda conferências.
São os departamentos de biologia, Geologia, matemática, Engenharia
Geográfica, Estatística e Investigação Operacional, Informática,
F´sica, química, Bioquímica e Educação a mostrar a sua investigação e
como fazer uma licentura de Ci~encia é aliciante.
Consulte www.fc.ul.pt/dias-abertos

Que os investigadores e a Universidade deixem vir as criançinhas até ela.
Senão os velhos morrem sem quem os substitua.
sds Maria Natália





SUBJECT: Re: Astrologers vs Journal of Consciousness Studies
FROM: "rmtakata" <rmtakata@altavista.net>
TO: ciencialist@yahoogrupos.com.br
DATE: 17/02/2005 10:23


--- Em ciencialist@yahoogrupos.com.br, "Esteban Moreno"
> Isso não é verdade. Se algum dia se dispuser a realizar um set com
> um astrólogo, poderia confirmar ou não sua hipótese.

Isso ja' foi feito. E o resultado eh o q. Homero/Oraculo nos relatou.

[]s,

Roberto Takata





SUBJECT: Re: [ciencialist] Re: Astrologers rated these killers as good guys
FROM: "Esteban Moreno" <estebanmoreno@idhi.org.br>
TO: <ciencialist@yahoogrupos.com.br>
DATE: 17/02/2005 10:25

Batia e muito, antes de pregar a "não violência".

Esteban.

----- Original Message -----
From: César A. K. Grossmann
To: ciencialist@yahoogrupos.com.br
Sent: Wednesday, February 16, 2005 9:32 AM
Subject: [ciencialist] Re: Astrologers rated these killers as good guys



--- Em ciencialist@yahoogrupos.com.br, "Esteban Moreno"
<estebanmoreno@i...> escreveu
> Se algum astrologo ler o mapa do Gandhi sem que ele esteja na sua
frente para depor, dirá que é um ser muito perturbado pisiquicamente.
E foi, tanto que batia na esposa, porém soube muito bem trabalhar com
as inclinações apontadas pelo mapa.

Gandhi batia na esposa?

[]s
--
.O. Cesar A. K. Grossmann ICQ UIN: 35659423
..O http://www.LinuxByGrossmann.cjb.net/
OOO Timeo Danaos, et dona ferentes. (Virgilio)





##### ##### #####

Para saber mais visite
http://www.ciencialist.hpg.ig.com.br


##### ##### ##### #####


Yahoo! Grupos, um serviço oferecido por:

São Paulo Rio de Janeiro Curitiba Porto Alegre Belo Horizonte Brasília




------------------------------------------------------------------------------
Links do Yahoo! Grupos

a.. Para visitar o site do seu grupo na web, acesse:
http://br.groups.yahoo.com/group/ciencialist/

b.. Para sair deste grupo, envie um e-mail para:
ciencialist-unsubscribe@yahoogrupos.com.br

c.. O uso que você faz do Yahoo! Grupos está sujeito aos Termos do Serviço do Yahoo!.



[As partes desta mensagem que não continham texto foram removidas]



SUBJECT: Re: [ciencialist] Astrologers vs Journal of Consciousness Studies
FROM: "Esteban Moreno" <estebanmoreno@idhi.org.br>
TO: <ciencialist@yahoogrupos.com.br>
DATE: 17/02/2005 10:44

Oráculo:
"Basta, como já foi feito antes, misturar mapas astrais de diversas pessoas,
e notar que continuarão a dar grande grau de precisão e acerto aos
mesmos..:-)"

Esteban:
Isso não é verdade. Se algum dia se dispuser a realizar um set com um
astrólogo, poderia confirmar ou não sua hipótese. Ou mesmo tentar utilizar a
acupuntura primitiva para se fazer adequado ao seu discurso contrário. Mas a
crença é muito forte e seria quase um sacrilégio (ops) fazê-lo agora. Por
isso acredito que seus netos o farão por ti.

Oráculo: "Esteban, os testes, inclusive o que propõe, já foram feitos.
Nenhum astrólogo pode encontrar o mapa astral ou determinar quem é quem em
um experimento controlado. E diversos foram feitos, com centenas de
astrologos em diferentes paises. Confrontados com o teste que você mesmo
propôs, falham."

Esteban:
Não, o que eu proponho e deixei infelizmente mal explicado, nunca foi feito.
Estou quase certo disso, pelo menos não há nada publicado em meios céticos e
revistas científicas.

Oráculo:
Por que não faz um teste? Crie um mapa astral falso, com datas e horario de
nascimento inventado, e apresente para alguém como real, se possível, ara
alguém que sincerametne acredite em astrologia. Escute o que ele tem a dizer
sobre o grau de acerto e nos conte como foi..:-) Será uma experiência
reveladora, no final..:-)

Esteban:
Então por que não fazemos este teste juntos ou com o outro método que venho
desenvolvido? Onde moras? Vamos levar um pouco mais adiante a sua descrença
sobre a astrologia. Tens coragem ou passará a vida a atirar ovos naquilo que
desconhece? ;-) Há, sim, desculpe, não pretende perder o seu tempo com
pseudoscientificismos. Vamos lá, é bem simples, pode convidar os da terra
arredondada. Juro que não mordo pessoalmente, ao menos que sejas uma bela
jovem disfarçada por uma cerebrização extrema. ;_))

Um forte abraço,
Esteban.







SUBJECT: Re: [ciencialist] Re: Astrologers vs Journal of Consciousness Studies
FROM: "Esteban Moreno" <estebanmoreno@idhi.org.br>
TO: <ciencialist@yahoogrupos.com.br>
DATE: 17/02/2005 11:38

Com quem, como foi a situação? Favor, expor.
[]s,
Esteban.

----- Original Message -----
From: rmtakata
To: ciencialist@yahoogrupos.com.br
Sent: Thursday, February 17, 2005 9:23 AM
Subject: [ciencialist] Re: Astrologers vs Journal of Consciousness Studies



--- Em ciencialist@yahoogrupos.com.br, "Esteban Moreno"
> Isso não é verdade. Se algum dia se dispuser a realizar um set com
> um astrólogo, poderia confirmar ou não sua hipótese.

Isso ja' foi feito. E o resultado eh o q. Homero/Oraculo nos relatou.

[]s,

Roberto Takata





##### ##### #####

Para saber mais visite
http://www.ciencialist.hpg.ig.com.br


##### ##### ##### #####


Yahoo! Grupos, um serviço oferecido por:

São Paulo Rio de Janeiro Curitiba Porto Alegre Belo Horizonte Brasília




------------------------------------------------------------------------------
Links do Yahoo! Grupos

a.. Para visitar o site do seu grupo na web, acesse:
http://br.groups.yahoo.com/group/ciencialist/

b.. Para sair deste grupo, envie um e-mail para:
ciencialist-unsubscribe@yahoogrupos.com.br

c.. O uso que você faz do Yahoo! Grupos está sujeito aos Termos do Serviço do Yahoo!.



[As partes desta mensagem que não continham texto foram removidas]



SUBJECT: Re: Astrologers vs Journal of Consciousness Studies
FROM: "rmtakata" <rmtakata@altavista.net>
TO: ciencialist@yahoogrupos.com.br
DATE: 17/02/2005 11:43


--- Em ciencialist@yahoogrupos.com.br, "Esteban Moreno"
> Com quem, como foi a situação? Favor, expor.

O experimento foi com estudantes universitarios. Cada um recebeu
descricoes de personalidade feitas por meios astromanticos. Um grupo
recebeu descricoes q. nao eram originadas de seus proprios mapas
natais. Nao houve diferenca entre os grupos q. receberam suas
avaliacoes baseadas em mapas verdadeiros e em mapas trocados qto 'a
percepcao de acuracia das descricoes.

Snyder, CR & Shenkel, RJ. "The P.T. Barnum Effect." Psychology Today.
Dec. 1975.

[]s,

Roberto Takata





SUBJECT: Re: (não um hoax!) Foucault / Luz ondas ou corpusculos.
FROM: César A. K. Grossmann <cesarakg@bol.com.br>
TO: ciencialist@yahoogrupos.com.br
DATE: 17/02/2005 11:45


--- Em ciencialist@yahoogrupos.com.br, "murilo filo"
<avalanchedrive@h...> escreveu
> Ok, ave Cesar.
> Se eu ainda tivesse 16 anos iríamos discutir.
> Não sei bem qual é a sua, mas está bem... vc é o bamba, ok?

Gentileza sua...

> Temos aquí um gênio da informação, mas que não lê, ou não entende, a
exata
> mensagem de alguém da lista. Quem é que disse que meu tio era um
físico, ou
> que assim se achava, meu?

Ninguém disse que ele era físico ou que se achava físico, só que a
explicação que ele deu é uma explicação *física*, ok? Quer dizer, se
eu disser que você pode fixar uma foto preto-e-branco usando urina, eu
vou estar dando uma joão-sem-braço, me metendo em coisas que não sei.
Foi o que seu tio fez: sem ter conhecimentos de física, meteu-se a dar
uma explicação física.

Não estou atacando você ou seu tio: faz parte da natureza humana
tentar explicar o que ocorre ao redor. Só que para contradizer uma
teoria científica, espera-se um pouco mais de conhecimento desta
teoria e também um pouco de rigorismo.

Não vou negar que a experiência que você descreve (muito
suscintamente, deixando margem a dúvidas e erros de interpretação,
como acho que pode muito bem ser o meu caso) é intrigante, mas antes
de apelar para uma explicação radical, "contradizendo" as teorias
físicas atuais sobre a luz, eu acho que seria interessante ver se as
teorias atuais tem alguma explicação convincente para o resultado da
experiência, você não acha?

Bom, entrando agora na física, particularmente na ótica (não sou
físico, sou formado em engenharia elétrica), eu lembro de algumas
propriedades da luz:

1. dois raios de luz que não viagem paralelos e na mesma direção não
interferem um no outro. Você pode pegar duas lanternas, e fazer o
facho de uma cruzar o de outra e ver se acontece algum desvio. Ou usar
duas ponteiras laser para ver se uma interfere na outra.

2. dois raios de luz que viagem paralelos ou quase paralelos, e na
mesma direção, apresentam interferência. São as franjas de luz e
sombra que aparecem na famosa experiência das fendas.

Agora, entrando na fotografia, onde eu também não sou fotógrafo, mas
já fiz alguns cursos e experiências.

1. Se você quer fotografar algum objeto que está no contra-luz, ou
seja, com o sol pelas costas, e ter uma imagem do que está na sombra,
vai ter que usar uma das técnicas abaixo:

a. usar um tempo de exposição maior, e/ou usar uma abertura de
obturador maior, de forma que a parte iluminada da foto fique
queimada, mas a parte que está nas sombras fique nítida.

b. usar um anteparo para refletir uma luz difusa sobre a parte
sombreada, deixando-a mais iluminada, impressionando assim mais o
filme (ou seja, ficando mais nítida).

c. usando um flash, para iluminar a região de sombra, causando o mesmo
efeito de "b", ali em cima...

> P/que gastar tanto papo, se o que eu queria dizer (em resposta a mais
> alguém) era justamente que as câmeras antigas não tinham automatismo
algum,
> e não davam perdão p/uma foto mal batida?

Do quê você está falando? Quando eu disse "automático" não quis dizer
que ela fazia a foto e as correções para ti, mas que quando você
selecionava "foto com flash", estava também selecionando tempo de
exposição e abertura de diafragma, sem precisar tocar no botão de
velocidades e de abertura, ao mesmo tempo, por isto o "automático".
Parece que "automático" tem significado diferente para nós dois...

> Elas tinham que ser comandadas e não havia 'edição' de fotos, além dos
> clássicos retoques. M.

Quer um retoque simples de foto? Expor o papel fotográfico por mais
tempo. Vai queimar a parte iluminada, e dar mais detalhes para a parte
escura. Geralmente...

O problema de ser muito lacônico é que muita informação fica
subentendida, e daí se criam os mal-entendidos. Você fica de mal
comigo por que eu falei A, pensando que você iria entender que
implicava B, mas você ouviu A e pensou C, e já não gostou, e me chamou
de ..., e...

Percebes?

[]s
--
.O. Cesar A. K. Grossmann ICQ UIN: 35659423
..O http://www.LinuxByGrossmann.cjb.net/
OOO Timeo Danaos, et dona ferentes. (Virgilio)





SUBJECT: Re: Astrologers rated these killers as good guys
FROM: César A. K. Grossmann <cesarakg@bol.com.br>
TO: ciencialist@yahoogrupos.com.br
DATE: 17/02/2005 11:47


--- Em ciencialist@yahoogrupos.com.br, "Esteban Moreno"
<estebanmoreno@i...> escreveu
> Batia e muito, antes de pregar a "não violência".

Tem alguma outra fonte, além da tua afirmação? Uma biografia, quem
sabe? De prefereência algo que esteja à distância de um clique...

[]s
--
.O. Cesar A. K. Grossmann ICQ UIN: 35659423
..O http://www.LinuxByGrossmann.cjb.net/
OOO Timeo Danaos, et dona ferentes. (Virgilio)





SUBJECT: Re: [ciencialist] Re: How to convince clients that astrology works
FROM: "Esteban Moreno" <estebanmoreno@idhi.org.br>
TO: <ciencialist@yahoogrupos.com.br>
DATE: 17/02/2005 11:55

Segue uma rápida tradução:
"O processo de auto-atribuição é um importante fator no desenvolvimento de crenças e na validação das descrições apresentadas da personalidade. Hamilton propôs que o favorecimento das descrições derivadas das descrições astrológicas é uma variável moderadora por um longo prazo de auto-atribuição e atua baseada no conhecimento do simbolismo do signo astrológico."

Ou seja, o autor tá dizendo que o Hamilton acha que as pessoas que se mostram favoráveis o são devido a uma cultura de auto-atribuição a longo prazo.


"Uma vez que o jogo de características associado com alguns signos solares são mais favoráveis que outros, ela (Hamilton) prediz que os nativos que nasceram sob um signo solar favorável mostram uma crença mais forte em astrologia do que os que não nasceram sob signo favorável."

Novamente, reforça o sentido de auto-atribuição em função da crença pessoal.


"Para testar esta hipótese, nos estudamos 1700 alemães para averiguar a sua crença na astrologia em função do seu signo solar. Mas a significância das crenças foram praticamente a mesma para todos os grupos de signos, demonstrando uma forte evidência contra a hipótese do Hamilton. É proposto que os resultados do Hamilton são provavelmente artefatos, e não efeitos de auto-atribuição a longo prazo."

Ou seja, a crença do valor pessoal de cada signo não foi suficiente para alterar o valor de auto-significação.

O que estou tentando trazer com este artigo é que não é o fato de se achar que o signo é mais legal que vai influenciar na aceitação do significado astrológico. É isso o que o artigo diz no resumo. O que está sendo questionado no artigo não se a pessoa concorda ou não com a interpretação, ao contrário do que escreveu o Oráculo, mas contra a tendência de auto-atribuição (um argumento muito utilizado pelos céticos) no desenvolvimento de um parecer favorável ou não sobre o efeito descrito pelo signo solar.
Segue abaixo o resto do texto. Vou colocar nos arquivos da ciencialist.

Abraço,
Esteban.






1. Introduction
An anomaly discovered serendipitously by Mayo, White, and Eysenck (1978), which was subsequently
explained by Pawlik and Buse (1979) in terms of attribution theory, managed to
become a prominent exemplar for the process of self-attribution: Subjects tended to describe their
0191-8869/03/$ - see front matter # 2003 Elsevier Science Ltd. All rights reserved.
doi:10.1016/S0191-8869(03)00002-3
* Tel.: +49-6224-922290; fax: +49-6224-922291.
E-mail address: wunder@anomalistik.de (E. Wunder).
ARTICLE IN PRESS

personality in terms of those traits that pertained to their astrological sun-signs, but only if they
had some astrological knowledge. The e.ect was replicated several times (Eysenck & Nias 1981,
1982; Fichten & Sunerton, 1983; Jackson, 1979; Kelly, 1982; Smithers & Cooper, 1978), even if
no reference to astrology was made until the debrie.ng of the subjects (Hamilton, 1995; Van
Rooij, 1994, 1999), or if the data were gathered originally for a purpose which has nothing to do
with astrology at all (Clarke, Gabriels, & Barnes, 1996; Van Rooij, Brak, & Commandeur, 1988),
but the e.ect is stronger when a cue is given to the subjects that the study is about astrology (Van
Rooij 1994). Early evidence for sun-sign derived self-attribution e.ects has already been reported
by Silverman (1971) and Delaney & Woodyard (1974). In studies with subjects unfamiliar with the
meaning of the astrological sun-sign symbolism, no e.ect was observed (Fourie, 1984; Jackson &
Fiebert, 1980; Kanekar & Mukherjee, 1972;Mohan, Bhandari, &Meena, 1982; Mohan and Gulati,
1986; Saklofske, Kelly, & McKerracher, 1982; Silverman & Whitmer, 1974; Veno & Pamment,
1979). Thus a real connection between personality and astrological sun-signs can be excluded,
in accordance with other studies refuting alleged astrological sun-sign e.ects on personality
(Dahlstrom, Hopkins, Dahlstrom, Jackson, & Cumella 1996; Gauquelin, 1982; Hentschel &
Kiessling, 1985). Furthermore, there are not the slightest e.ects if other astrological factors than
sun-signs are studied (Clarke et al., 1996; Russell & Wagsta., 1983; Startup, 1985), because
obviously only very few people know something about the interpretive pro.le of their horoscope
beyond sun-sign astrology. Accordingly, self-attribution e.ects cannot be expected to occur.
Questions remain regarding the concrete self-attribution mechanism responsible for creating
this internalisation of subjects' astrological knowledge. Lo Iacono (1989) speculates that ''astrological
symbolism may be attributed to the self in a manner that a.rms the existence of desired
traits, or by virtue of its inherent ambiguity, explains away aspects of the self that are perceived as
less attractive''. Using this line of reasoning, Hamilton (2001) claims an e.ect of favourableness
of astrology-derived personality descriptions as a moderator variable for the self-attribution
process under that condition, since Glick, Gottesman, and Jolton (1989) have found that
favourable, or socially desirable, personality descriptions of astrological origin are more likely to
be accepted as true than unfavourable ones. In particular, Hamilton (2001) proposes that the
characteristics of odd signs (Aries, Gemini, Leo, Libra, Sagittarius, Aquarius)-in the astrological
tradition also called ''positive'' or ''male'' signs-are perceived as more favourable than
those of even signs (Taurus, Cancer, Virgo, Scorpio, Capricorn, Pisces), in the astrological
tradition also called ''negative'' or ''female'' signs. In conclusion, she predicts that subjects born
under odd (''positive'') signs should express a higher degree of belief in astrology than natives of
even (''negative'') signs. If true, this moderator e.ect should be valid at least for subjects with
knowledge in sun-sign traits, and not only under experimental conditions but also, for example,
in surveys on belief in astrology, if we assume long-term e.ects of astrological self-attribution, as
Hamilton (1995) did.
Indeed, Hamilton (2001) was empirically successful in showing that the personality descriptions
of ''positive'' signs, excerpted from a book on astrology, were perceived as more favourable than
those of ''negative'' signs by two samples of (mostly) psychology students, who were not
informed about the astrological origin of the statements. In Hamilton's second sample the participants
were told immediately afterwards that the personality descriptions were those of astrological
signs, subsequently completing a questionnaire on knowledge and belief in astrology,
including a question on the sun sign of the participants (controlled by month and day of birth). It
2 E. Wunder / Personality and Individual Di.erences 34 (2003) 1-7
turned out, as predicted, that subjects born under a ''positive'' sign expressed signi.cantly
(P<0.05) more belief in astrology than did subjects born under a ''negative'' sign (N=92).
Hamilton (2001, p. 900) states: ''The present study suggests that the e.ect of favorableness is
likely to be at least fairly long-lasting since our participants had, presumably, been exposed to
their own sun sign descriptions well before entering the study.'' If this supposition is true, the
di.erence in belief in astrology between natives of odd vs. even sun-signs should be independent
of Hamilton's priming procedure (i.e. .rst to give subjects the task to rate personality descriptions,
and then debrief them that the descriptions were based on sun-sign astrology).
The following study is a more direct test of the alleged long-lasting e.ects of the relative
favourableness of odd vs. even sun-sign personality characteristics on the belief in astrology,
because it avoids this kind of priming, which has to be regarded as a possible source of artefacts.
2. Study
2.1. Method
One thousand seven-hundred subjects were recruited by a call for participants in a ''study of the
validity of astrology'' in German newspapers, radio and television programs. Of the subjects
64.4% were females. Ages ranged from 12to 83 years, with a mean of 38.3 years. The subjects'
backgrounds were representative of all segments of German society. Originally the data were not
gathered for the purpose of testing Hamilton's hypothesis, but for a matching experiment similar
to the one reported by Carlson (1985), see Wunder (in preparation). The subjects were motivated
to take part in this study by the promise of an individual interpretation of their horoscope given
by an astrologer.
In the .rst step of this complex experiment (Wunder, in preparation), all subjects had to complete
a short questionnaire, asking for date and time of birth, gender and ratings for two items on belief
in astrology (''If you know the horoscope of a stranger, you know a lot about his character'', ''The
sign or horoscope of a person has an in.uence on his/her course of life'') on .ve-point scales
(''strongly agree'', ''tend to agree'', ''uncertain'', ''tend to disagree'', ''strongly disagree'').
This questionnaire also asked for ratings for four items on experience and knowledge of
astrology (''I know the personality traits of my sign well'', ''I know my ascendant'', ''I have
already invested some time in learning more about astrology'', ''I have already visited an astrologer
for a personal consultation''), with only ''Yes'' or ''No'' being possible answers. Wunder
(2002) gives details of the frequencies, correlations, and factor analysis for the answers on these
six items on belief, knowledge and experience in astrology.
2.2. Results
Reliability analyses of the two items on belief in astrology elicited a Cronbach's a of 0.77, so
these two items were combined in a belief-in-astrology score ranging from 1 (=strong disbelief)
to 5 (=strong belief). A t-test revealed that there is no signi.cant di.erence in belief in astrology
between natives of odd signs (M=3.33) and natives of even signs [M=3.36; t(1698)=0.58;
P=0.57]. If we select only the 1224 subjects who agreed to the item ''I know the personality traits
E. Wunder / Personality and Individual Di.erences 34 (2003) 1-7 3

of my sign well'', there is also no signi.cant di.erence in belief in astrology between natives of
odd signs (M=3.60) and natives of even signs [M=3.61, t(1222)=0.29, P=0.77]. And even if
we select only those 273 subjects scoring highest on astrological experience and knowledge
(answering ''Yes'' to all four items), no signi.cant di.erence in belief in astrology between natives
of odd signs (M=3.99) and natives of even signs [M=4.05, t(1222)=0.57, P=0.57] emerges.
These results are obviously in strong contradiction to the hypothesis of Hamilton (2001).
Results of an ANOVA with the belief-in-astrology score as dependent variable reveals
that there are strong main e.ects of female gender (F=30.9; P<0.001) and knowledge of the
personality traits of the subjects' sun-sign (F=242.5; P<0.001), but there is no main e.ect
(F=1.6; P=0.20) of the so called polarity of sun-signs (odd vs. even signs). More importantly,
there are also no signi.cant interaction e.ects, either between knowledge and sign polarity
(F=0.08; P=0.78), between gender and sign polarity (F=0.04; P=0.85), or between gender
and knowledge (F=0.74; P=0.39). By the way, this picture does not change at all, if we use
a score of all three items of astrological knowledge instead of the one item on the knowledge of
the personality traits of the subjects' sun-sign (which seems the most appropriate one in this
context).
Even if we do not dichotomise the signs into ''positive'' and ''negative'' ones, but use all 12
signs as separate categories, there is no signi.cant e.ect of the natives' sign on the degree of belief
in astrology (F=1.49; P=0.13).
3. Discussion
The .ndings from our study raised serious doubts as to whether Hamilton's interpretation of
her results is correct. Maybe the e.ect of favourableness, shown by Hamilton (2001), is nothing
more than an artefact dependent on the experimental settings. If favourableness is really a moderator
variable for the long-term internalisation of sun-sign personality traits due to self-attribution
under non-experimental conditions, as Hamilton (2001) proposes, this e.ect should be
detectable in our database. But it is not. However, since our sample size is a large one (N=1700),
even very tiny e.ects had a chance of becoming signi.cant.
Germany has no independent tradition regarding the meaning of the sun-sign symbolism.
Indeed many popular books on astrology are translations of the works of American authors.
Nevertheless, it may be argued that the failure to replicate Hamilton's .nding could be due to
cross-cultural di.erences between Germany and the United States regarding the perceived
favourableness of certain personality traits. Individuals from a society in which ''masculine''
values are generally more respected than ''feminine'' ones will probably prefer the ''male'' or odd
signs, while individuals from a ''feminine'' society will tend to prefer the ''female'' or even signs.
Hofstede (1998) has done a lot of comparative empirical work on the dimension of masculinity/
femininity in di.erent cultures. However, his tabulation of the Masculinity Index (MAS) for
50 countries (Hofstede 1998, p. 9) shows that Germany and the United States are very close
on the dimension of masculinity (the range of MAS is from 0 to 100, Germany scores 66,
the United States 62; for comparison: Sweden scores 5, Japan 95). Therefore, it seems not to
be plausible to argue that the con.icting results of Hamilton (2001) and our study may be
due to cross-cultural di.erences between Germany and the United States on the perceived
4 E. Wunder / Personality and Individual Di.erences 34 (2003) 1-7
favourableness of ''masculine'' or ''feminine'' values. Also, note that our results show no
interaction e.ect with gender.
Another di.erence between the two samples is that the current participants, unlike Hamilton's,
knowingly volunteered to participate in a study of astrology in order to receive a personalized
horoscope. As a consequence, the sample comprised quite a lot of people highly interested in
astrology-perhaps this attenuated, one might argue, any di.erences between positive and negative
sun-sign natives. In this case we should expect di.erences in belief in astrology between
natives of odd signs and natives of even signs at least among those participants who scored lowest
on experience and knowledge of astrology. But in any case, i.e. independent of the degree of
experience and knowledge of astrology, there was no signi.cant e.ect. Therefore this attempt to
explain the di.erent results of the two studies also seems not to be too promising.
A crucial di.erence between Hamiltons' experimental setting and the typical non-experimental
conditions is the fact that the evaluation of the accuracy of one's sun-sign personality description
in everyday life is usually done by a veri.cation strategy concentrating only on one's own sunsign
and not by comparing the traits with those of other sun-signs. Van Rooij (1994) describes
this process in the following way:
A knowledgeable person knows which traits he is supposed to have, according to his or her
sun-sign. For example, an Aries-person learns from astrology that Aries-persons are said to
be 'impulsive'. He starts observing his behaviour and indeed notices that from time to time he
is impulsive. Because he is focused on the impulsive moments he does not pay equal attention
to the moments that he is not impulsive. Now, everybody is impulsive once in a while, but by
this process of selective self-observation the person might indeed get to think of himself as
impulsive.
To summarise, the belief in the accuracy of sun-sign personality descriptions arises from a
combination of fundamental attribution error, con.rmatory testing strategies, and selective selfobservation
and recall (see also Glick & Snyder, 1986). Since trait collections for astrological
signs can generally be described as ambiguous hodgepodges, where everyone can .nd something
suitable if he or she only wants to (Lillqvist & Lindeman, 1998), it may be no surprise that e.ects
of favourableness fail to turn up under non-experimental conditions. This is because the intravariance
of favourableness within one sign is much more pronounced than the inter-variance of
favourableness between signs, while comparisons between signs in any case do not take place or
are at least very unusual in everyday life, where interest is focussed on one's own sign only.
Hamilton (2001) herself wrote: ''Note that the negative astrological signs may have been 'negative'
only in comparison to the positive sign descriptions.'' This is an important point, because all
sun-signs have to o.er a lot of positive traits, which are true in at least some situations, easy to
accept as a desirable description of oneself (Lillqvist & Lindeman, 1998).
So what might have happened to the subjects under the experimental conditions introduced by
Hamilton? After being informed that the personality descriptions were based on astrological signs,
some of Hamiltons' subjects may have recognized that their own sign matched the personality
description they just rated as more unfavourable in comparison to another one. To escape this
annoying conclusion, those subjects may have reduced their committed belief in astrology immediately,
but this does not need to be a long-term e.ect. The possible counter-argument, that the
E. Wunder / Personality and Individual Di.erences 34 (2003) 1-7 5
subjects' own sun-sign was not always among the personality descriptions presented beforehand,
is not valid, because even in this case a subject may compare his knowledge about the traits of his
own sign with the personality descriptions of the other signs presented, which may be perceived as
more favourable in such a direct comparison, resulting in a reduction of the belief in astrology.
To sum up, in direct comparison the trait collections for odd signs may indeed sound more
favourable than those of even signs, but since those comparisons are not the way astrological
statements are evaluated under non-experimental conditions, this di.erence should be irrelevant
for the formation of belief in astrology outside the laboratory.
Whether or not this kind of reasoning is the right approach to reconcile the con.icting results
of Hamilton (2001) and our study, it illustrates the importance of context when collecting data to
investigate how processes of self-attribution works.
6 E. Wunder / Personality and Individual Di.erences 34 (2003) 1-7


It is proposed that the con.icting
empirical .ndings of Hamilton (2001) are probably artefacts of the experimental setting, and not long-term
e.ects of self-attribution.

----- Original Message -----
From: oraculo333
To: ciencialist@yahoogrupos.com.br
Sent: Wednesday, February 16, 2005 12:16 AM
Subject: [ciencialist] Re: How to convince clients that astrology works




Olá Esteban

O texto abaixo é parte do estudo que enviou a lista, seu abstract,
como sendo capaz de desmentir o "pseudocientificismo" dos artigos que
postei.

Entretanto, ele faz justamente o contrário, pelo menos no
abstract..:-) Demonstra que em um abrangente estudo, a astrologia não
foi capaz de predizer nem mesmo quais signos (e as pessoas destes
signos) seriam mais propensos a acreditar em astrologia..:-)

Acho que vou aceitar seu oferecimento de enviar o artigo completo, já
que não consegui acessar pela página (eles cobram por artigo, e cobram
caro..:-)

Talvez possa entender melhor o que está pensando que o artigo defende
ou afirma, ok? Mande para os arquivos da lista, já que as mensagens
não aceitam anexos por email.

Um abraço.

Homero
_______________________________________

Self-attribution, sun-sign traits, and the alleged role of
favourableness as a moderator variable: long-term effect or artefact?
Edgar WunderCorresponding Author Contact Information, E-mail The
Corresponding Author
Gesellschaft für Anomalistik, Postfach 1202, 69200, Sandhausen, Germany
Received 17 June 2002; revised 12 November 2002; accepted 12
December 2002. ; Available online 1 April 2003.

Abstract

The process of self-attribution is an important factor in the
development of beliefs in the validity of presented personality
descriptions. Hamilton (2001) proposed that the relative
favourableness of astrologically derived personality descriptions is a
moderator variable for long-term self-attribution effects based on
knowledge of the astrological sun-sign symbolism. Because the sets of
traits associated with some sun-signs are thought to be more
favourable than those of other sun-signs, she predicts that natives
born under a more favourable sign should show a stronger belief in
astrology than subjects born under less favourable signs. To test this
hypothesis, we studied 1700 German subjects, to see if their belief in
astrology varied with respect to their sun-sign. But the mean belief
scores were almost exactly the same for all sign groups, providing
strong evidence against Hamilton's hypothesis. It is proposed that the
conflicting empirical findings of Hamilton (2001) are probably
artefacts of the experimental setting, and not long-term effects of
self-attribution.

Author Keywords: Self-attribution; Personality descriptions;
Confirmatory testing strategies; Astrology; Paranormal belief systems;
Favourableness






##### ##### #####

Para saber mais visite
http://www.ciencialist.hpg.ig.com.br


##### ##### ##### #####


Yahoo! Grupos, um serviço oferecido por:
PUBLICIDADE





Links do Yahoo! Grupos

Para visitar o site do seu grupo na web, acesse:
http://br.groups.yahoo.com/group/ciencialist/

Para sair deste grupo, envie um e-mail para:
ciencialist-unsubscribe@yahoogrupos.com.br

O uso que você faz do Yahoo! Grupos está sujeito aos Termos do Serviço do Yahoo!.

[As partes desta mensagem que não continham texto foram removidas]



SUBJECT: Re: [ciencialist] Re: Astrologers rated these killers as good guys
FROM: "Esteban Moreno" <estebanmoreno@idhi.org.br>
TO: <ciencialist@yahoogrupos.com.br>
DATE: 17/02/2005 18:35

Não tenho nada que possa lhe servir como prova que esteja à distância de um clique...
[]´s
----- Original Message -----
From: César A. K. Grossmann
To: ciencialist@yahoogrupos.com.br
Sent: Thursday, February 17, 2005 10:47 AM
Subject: [ciencialist] Re: Astrologers rated these killers as good guys



--- Em ciencialist@yahoogrupos.com.br, "Esteban Moreno"
<estebanmoreno@i...> escreveu
> Batia e muito, antes de pregar a "não violência".

Tem alguma outra fonte, além da tua afirmação? Uma biografia, quem
sabe? De prefereência algo que esteja à distância de um clique...

[]s
--
.O. Cesar A. K. Grossmann ICQ UIN: 35659423
..O http://www.LinuxByGrossmann.cjb.net/
OOO Timeo Danaos, et dona ferentes. (Virgilio)





##### ##### #####

Para saber mais visite
http://www.ciencialist.hpg.ig.com.br


##### ##### ##### #####


Yahoo! Grupos, um serviço oferecido por:

São Paulo Rio de Janeiro Curitiba Porto Alegre Belo Horizonte Brasília




------------------------------------------------------------------------------
Links do Yahoo! Grupos

a.. Para visitar o site do seu grupo na web, acesse:
http://br.groups.yahoo.com/group/ciencialist/

b.. Para sair deste grupo, envie um e-mail para:
ciencialist-unsubscribe@yahoogrupos.com.br

c.. O uso que você faz do Yahoo! Grupos está sujeito aos Termos do Serviço do Yahoo!.



[As partes desta mensagem que não continham texto foram removidas]



SUBJECT: Re: Astrologers rated these killers as good guys
FROM: César A. K. Grossmann <cesarakg@bol.com.br>
TO: ciencialist@yahoogrupos.com.br
DATE: 17/02/2005 18:51


--- Em ciencialist@yahoogrupos.com.br, "Esteban Moreno"
<estebanmoreno@i...> escreveu
> Não tenho nada que possa lhe servir como prova que esteja à
distância de um clique...

Eu encontrei alguma coisa...

http://br.busca.yahoo.com/search?p=gandhi+wife&fr=my_top

Particularmente, http://eserver.org/history/ghandi-nobody-knows.txt
parece bastante perturbador, embora não aponte que Mohandas tenha
batido na esposa. Talvez a imagem que tenha sido vendida para o
ocidente é a de alguém não-humano nas sua bondade... Ou talvez o texto
contenha mentiras.

Veja o caso de um recente livro sobre a esposa de Gandhi:

http://www.amazon.com/exec/obidos/ASIN/1886940029/104-8594080-2455915

The Forgotten Woman: The Untold Story of Kastur Gandhi, Wife of
Mahatma Gandhi

De qualquer forma, não encontrei nada que sustente a sua afirmação...

[]s
--
.O. Cesar A. K. Grossmann ICQ UIN: 35659423
..O http://www.LinuxByGrossmann.cjb.net/
OOO Timeo Danaos, et dona ferentes. (Virgilio)





SUBJECT: RE: [ciencialist] Re: (não um hoax!) Foucault / Luz ondas ou corpusculos.
FROM: "murilo filo" <avalanchedrive@hotmail.com>
TO: ciencialist@yahoogrupos.com.br
DATE: 17/02/2005 22:28

Sol pelas costas... eu sempre me referí a fotos contra o sol... o sol pela
cara!
Cesar, vamos deixar as coisas assim, ok? M. SP 17/fev

>From: César A. K. Grossmann <cesarakg@bol.com.br>
>Reply-To: ciencialist@yahoogrupos.com.br
>To: ciencialist@yahoogrupos.com.br
>Subject: [ciencialist] Re: (não um hoax!) Foucault / Luz ondas ou
>corpusculos.
>Date: Thu, 17 Feb 2005 13:45:26 -0000
>
>
>
>--- Em ciencialist@yahoogrupos.com.br, "murilo filo"
><avalanchedrive@h...> escreveu
> > Ok, ave Cesar.
> > Se eu ainda tivesse 16 anos iríamos discutir.
> > Não sei bem qual é a sua, mas está bem... vc é o bamba, ok?
>
>Gentileza sua...
>
> > Temos aquí um gênio da informação, mas que não lê, ou não entende, a
>exata
> > mensagem de alguém da lista. Quem é que disse que meu tio era um
>físico, ou
> > que assim se achava, meu?
>
>Ninguém disse que ele era físico ou que se achava físico, só que a
>explicação que ele deu é uma explicação *física*, ok? Quer dizer, se
>eu disser que você pode fixar uma foto preto-e-branco usando urina, eu
>vou estar dando uma joão-sem-braço, me metendo em coisas que não sei.
>Foi o que seu tio fez: sem ter conhecimentos de física, meteu-se a dar
>uma explicação física.
>
>Não estou atacando você ou seu tio: faz parte da natureza humana
>tentar explicar o que ocorre ao redor. Só que para contradizer uma
>teoria científica, espera-se um pouco mais de conhecimento desta
>teoria e também um pouco de rigorismo.
>
>Não vou negar que a experiência que você descreve (muito
>suscintamente, deixando margem a dúvidas e erros de interpretação,
>como acho que pode muito bem ser o meu caso) é intrigante, mas antes
>de apelar para uma explicação radical, "contradizendo" as teorias
>físicas atuais sobre a luz, eu acho que seria interessante ver se as
>teorias atuais tem alguma explicação convincente para o resultado da
>experiência, você não acha?
>
>Bom, entrando agora na física, particularmente na ótica (não sou
>físico, sou formado em engenharia elétrica), eu lembro de algumas
>propriedades da luz:
>
>1. dois raios de luz que não viagem paralelos e na mesma direção não
>interferem um no outro. Você pode pegar duas lanternas, e fazer o
>facho de uma cruzar o de outra e ver se acontece algum desvio. Ou usar
>duas ponteiras laser para ver se uma interfere na outra.
>
>2. dois raios de luz que viagem paralelos ou quase paralelos, e na
>mesma direção, apresentam interferência. São as franjas de luz e
>sombra que aparecem na famosa experiência das fendas.
>
>Agora, entrando na fotografia, onde eu também não sou fotógrafo, mas
>já fiz alguns cursos e experiências.
>
>1. Se você quer fotografar algum objeto que está no contra-luz, ou
>seja, com o sol pelas costas, e ter uma imagem do que está na sombra,
>vai ter que usar uma das técnicas abaixo:
>
>a. usar um tempo de exposição maior, e/ou usar uma abertura de
>obturador maior, de forma que a parte iluminada da foto fique
>queimada, mas a parte que está nas sombras fique nítida.
>
>b. usar um anteparo para refletir uma luz difusa sobre a parte
>sombreada, deixando-a mais iluminada, impressionando assim mais o
>filme (ou seja, ficando mais nítida).
>
>c. usando um flash, para iluminar a região de sombra, causando o mesmo
>efeito de "b", ali em cima...
>
> > P/que gastar tanto papo, se o que eu queria dizer (em resposta a mais
> > alguém) era justamente que as câmeras antigas não tinham automatismo
>algum,
> > e não davam perdão p/uma foto mal batida?
>
>Do quê você está falando? Quando eu disse "automático" não quis dizer
>que ela fazia a foto e as correções para ti, mas que quando você
>selecionava "foto com flash", estava também selecionando tempo de
>exposição e abertura de diafragma, sem precisar tocar no botão de
>velocidades e de abertura, ao mesmo tempo, por isto o "automático".
>Parece que "automático" tem significado diferente para nós dois...
>
> > Elas tinham que ser comandadas e não havia 'edição' de fotos, além dos
> > clássicos retoques. M.
>
>Quer um retoque simples de foto? Expor o papel fotográfico por mais
>tempo. Vai queimar a parte iluminada, e dar mais detalhes para a parte
>escura. Geralmente...
>
>O problema de ser muito lacônico é que muita informação fica
>subentendida, e daí se criam os mal-entendidos. Você fica de mal
>comigo por que eu falei A, pensando que você iria entender que
>implicava B, mas você ouviu A e pensou C, e já não gostou, e me chamou
>de ..., e...
>
>Percebes?
>
>[]s
>--
>.O. Cesar A. K. Grossmann ICQ UIN: 35659423
>..O http://www.LinuxByGrossmann.cjb.net/
>OOO Timeo Danaos, et dona ferentes. (Virgilio)
>
>
>
>
>
>##### ##### #####
>
>Para saber mais visite
>http://www.ciencialist.hpg.ig.com.br
>
>
>##### ##### ##### #####
>Links do Yahoo! Grupos
>
>
>
>
>
>
>
>




SUBJECT: adeus.
FROM: "rayfisica" <rayfisica@yahoo.com.br>
TO: ciencialist@yahoogrupos.com.br
DATE: 18/02/2005 06:09


Quando eu entrei para essa lista foi para aprender e conviver ainda
que distante com pessoas sábias, inteligentes e que amassem a
ciência.
Encontrei exatamente isso em algumas pessoas aqui e a essas eu
agradeço, de verdade o tempo que dedicaram a mim.
Desprezo os arrogantes e intolerantes que contradizem se.
Desculpem-me, pois não tenho tempo a perder com gente assim.
Adeus.






SUBJECT: Re: adeus.
FROM: marcelo ferrari <emailferrari@yahoo.com.br>
TO: ciencialist@yahoogrupos.com.br
DATE: 18/02/2005 08:32

Rya,

Releve o lado ruim e fique só com o lado bom. Faça do limão uma limonada. Entendo o que vc sente, muita gente aqui já deu soco na mesma ponta de faca que vc, inclusive eu, por exemplo. Parece que a arrogancia faz parte do doutoramento em ciencias. Não tormam os cientistas mais sábios, mas, de alguma forma que não sei explicar, os fazem sentir mais cientistas. Vai entender. Mas ainda assim vale a pena participar desta lista, basta saber usá-la. Aliás, não é isto que importa em ciencias, saber como as coisas funcionam e usá-las para atingir um fim.

forte abraço.
marcelo ferrari





rayfisica <rayfisica@yahoo.com.br> wrote:

Quando eu entrei para essa lista foi para aprender e conviver ainda
que distante com pessoas sábias, inteligentes e que amassem a
ciência.
Encontrei exatamente isso em algumas pessoas aqui e a essas eu
agradeço, de verdade o tempo que dedicaram a mim.
Desprezo os arrogantes e intolerantes que contradizem se.
Desculpem-me, pois não tenho tempo a perder com gente assim.
Adeus.






##### ##### #####

Para saber mais visite
http://www.ciencialist.hpg.ig.com.br


##### ##### ##### #####


Yahoo! Grupos, um serviço oferecido por: São Paulo Rio de Janeiro Curitiba Porto Alegre Belo Horizonte Brasília

---------------------------------
Links do Yahoo! Grupos

Para visitar o site do seu grupo na web, acesse:
http://br.groups.yahoo.com/group/ciencialist/

Para sair deste grupo, envie um e-mail para:
ciencialist-unsubscribe@yahoogrupos.com.br

O uso que você faz do Yahoo! Grupos está sujeito aos Termos do Serviço do Yahoo!.



---------------------------------
Yahoo! Acesso Grátis - Internet rápida e grátis. Instale o discador do Yahoo! agora.

[As partes desta mensagem que não continham texto foram removidas]



SUBJECT: Re: adeus.
FROM: "rmtakata" <rmtakata@altavista.net>
TO: ciencialist@yahoogrupos.com.br
DATE: 18/02/2005 09:27


--- Em ciencialist@yahoogrupos.com.br, marcelo ferrari
> arrogancia faz parte do doutoramento em ciencias. Não tormam os

Doutoramento levando 'a arrogancia?

Como o unico q. sabidamente fez doutorado aqui eh o Brudna... Fio e
dou fé q. Luiz Robert Brudna Hozle pouco ou nada tem de arrogante,
sendo um ativo e interessado divulgador das ciencias, inclusive
mantendo esta lista para a participacao de todos os interessados em
ciencias - independente da fe', filiacao ideologica, sexo, raca, time
de futebol, estado civil, orientacao sexual, QI, escolaridade, crenca
politica, nivel socioeconomico, preferencia culinaria, numero de
dedos, razao cintura-quadril, humor, alfabetizacao digital, idade,
local de nascimento, tempo de servico, altura, peso, relacoes
filogeneticas ou qq outra forma de discriminacao.

[]s,

Roberto Takata






SUBJECT: Re: (não um hoax!) Foucault / Luz ondas ou corpusculos.
FROM: César A. K. Grossmann <cesarakg@bol.com.br>
TO: ciencialist@yahoogrupos.com.br
DATE: 18/02/2005 10:37


--- Em ciencialist@yahoogrupos.com.br, "murilo filo"
<avalanchedrive@h...> escreveu
> Sol pelas costas... eu sempre me referí a fotos contra o sol... o
sol pela
> cara!

Sol pelas costas de quem está sendo fotografado... Pensei que tinha
ficado claro...

> Cesar, vamos deixar as coisas assim, ok? M. SP 17/fev

Não dá. Sou muito chato, e quando este tipo de coisa cai nas minhas
fuças, eu vou até o fim...

[]s
--
.O. Cesar A. K. Grossmann ICQ UIN: 35659423
..O http://www.LinuxByGrossmann.cjb.net/
OOO Timeo Danaos, et dona ferentes. (Virgilio)





SUBJECT: Re: [ciencialist] adeus.
FROM: "Alvaro Augusto \(E\)" <alvaro@electraenergy.com.br>
TO: <ciencialist@yahoogrupos.com.br>
CC: <rayfisica@yahoo.com.br>
DATE: 18/02/2005 10:50

Caro Ray,

Eu faço parte de umas quinze listas de discussão. Entrei na Ciencialist nos primórdios, arranjei uma briga com um advogado e me retirei. Voltei algum tempo depois e não me lembro direito o motivo da briga... Acredite, essa aqui é uma das listas mais tolerantes e menos arrogantes que há. O problema é que as pessoas são as mesmas em todos os lugares. Aonde quer que você vá, encontrará pessoas do mesmo tipo que as que aqui estão.

Além disso, você tem que considerar que as pessoas ficam mais "corajosas" na versão escrita. Nós escrevemos coisas que nunca diríamos pessoalmente. Assim, seja lá o que tiver acontecido, "largue mão"...

[ ]s

Alvaro Augusto
----- Original Message -----
From: rayfisica
To: ciencialist@yahoogrupos.com.br
Sent: Friday, February 18, 2005 5:09 AM
Subject: [ciencialist] adeus.



Quando eu entrei para essa lista foi para aprender e conviver ainda
que distante com pessoas sábias, inteligentes e que amassem a
ciência.
Encontrei exatamente isso em algumas pessoas aqui e a essas eu
agradeço, de verdade o tempo que dedicaram a mim.
Desprezo os arrogantes e intolerantes que contradizem se.
Desculpem-me, pois não tenho tempo a perder com gente assim.
Adeus.


[As partes desta mensagem que não continham texto foram removidas]



SUBJECT: Re: [ciencialist] Re: adeus.
FROM: "Luiz Ferraz Netto" <leobarretos@uol.com.br>
TO: <ciencialist@yahoogrupos.com.br>
DATE: 18/02/2005 11:04

[ ... ] número de dedos, [ ... ]


he he he ... só faltou dizer que o tipo sangüíneo do dito cujo é: A-peritivo !

[]'


--
No virus found in this outgoing message.
Checked by AVG Anti-Virus.
Version: 7.0.300 / Virus Database: 265.8.8 - Release Date: 14/02/2005



SUBJECT: Aplicacao da Quantica
FROM: "Luiz Ferraz Netto" <leobarretos@uol.com.br>
TO: <ciencialist@yahoogrupos.com.br>
DATE: 18/02/2005 11:12

Sempre quis colocar no Imperdível um experimento de Física Quântica ao nível do Ensino Médio; finalmente consegui:

http://www.laundry-alternative.com/fateofmissingsocks.html

[]'
===========================
Luiz Ferraz Netto [Léo]
leobarretos@uol.com.br
http://www.feiradeciencias.com.br
===========================


--
No virus found in this outgoing message.
Checked by AVG Anti-Virus.
Version: 7.0.300 / Virus Database: 265.8.8 - Release Date: 14/02/2005



SUBJECT: Re: [ciencialist] adeus.
FROM: "Luiz Ferraz Netto" <leobarretos@uol.com.br>
TO: <ciencialist@yahoogrupos.com.br>
DATE: 18/02/2005 11:20

Deixa isso tudo prá lá; escreva uma nova mensagem apenas com:

" E Ray " --- e retorne,

[ para aqueles que nada entendem de língua alienígena (*), isso se pronuncia assim: E Rei ]

ao finalizar a mensagem, não escreva "Adeus", escreva "Ao Léo".

aquele abraço,

(*) como eu
===========================
Luiz Ferraz Netto [Léo]
leobarretos@uol.com.br
http://www.feiradeciencias.com.br
===========================

-----Mensagem Original-----
De: "rayfisica" <rayfisica@yahoo.com.br>
Para: <ciencialist@yahoogrupos.com.br>
Enviada em: sexta-feira, 18 de fevereiro de 2005 05:09
Assunto: [ciencialist] adeus.




Quando eu entrei para essa lista foi para aprender e conviver ainda
que distante com pessoas sábias, inteligentes e que amassem a
ciência.
Encontrei exatamente isso em algumas pessoas aqui e a essas eu
agradeço, de verdade o tempo que dedicaram a mim.
Desprezo os arrogantes e intolerantes que contradizem se.
Desculpem-me, pois não tenho tempo a perder com gente assim.
Adeus.




--
No virus found in this outgoing message.
Checked by AVG Anti-Virus.
Version: 7.0.300 / Virus Database: 265.8.8 - Release Date: 14/02/2005



SUBJECT: Re: [ciencialist] Re: adeus.
FROM: Luis Brudna <luisbrudna@gmail.com>
TO: ciencialist@yahoogrupos.com.br
DATE: 18/02/2005 14:43

Hehe... obrigado!

Seus puxa! :-)

Pois é. Vi que o clima esquentou aqui na lista, e pra variar parece
que foi por causa da astrologia. Fico ansioso quando esse assunto
aparece pq não tenho conhecimentos para debater. Quero meter colher
mas essa parte da culinária eu não domino.

Pelo que pude ver não apareceu nenhuma mensagem agressiva, não vejo
motivos para alguém sair da lista.

Até
Luís Brudna


On Fri, 18 Feb 2005 14:33:59 -0300, Oraculo <oraculo@atibaia.com.br> wrote:
>
> Olá
>
> Assino embaixo da declaração do Takata sobre o Brudna, uma exceletne pessoa e cientista.
>
> Homero
> ----- Original Message -----
> From: rmtakata
> To: ciencialist@yahoogrupos.com.br
> Sent: Friday, February 18, 2005 8:27 AM
> Subject: [ciencialist] Re: adeus.
>
> --- Em ciencialist@yahoogrupos.com.br, marcelo ferrari
> > arrogancia faz parte do doutoramento em ciencias. Não tormam os
>
> Doutoramento levando 'a arrogancia?
>
> Como o unico q. sabidamente fez doutorado aqui eh o Brudna... Fio e
> dou fé q. Luiz Robert Brudna Hozle pouco ou nada tem de arrogante,
> sendo um ativo e interessado divulgador das ciencias, inclusive
> mantendo esta lista para a participacao de todos os interessados em
> ciencias - independente da fe', filiacao ideologica, sexo, raca, time
> de futebol, estado civil, orientacao sexual, QI, escolaridade, crenca
> politica, nivel socioeconomico, preferencia culinaria, numero de
> dedos, razao cintura-quadril, humor, alfabetizacao digital, idade,
> local de nascimento, tempo de servico, altura, peso, relacoes
> filogeneticas ou qq outra forma de discriminacao.
>
> []s,
>
> Roberto Takata


SUBJECT: Re: [ciencialist] Re: adeus.
FROM: "Oraculo" <oraculo@atibaia.com.br>
TO: <ciencialist@yahoogrupos.com.br>
DATE: 18/02/2005 15:33

Olá

Assino embaixo da declaração do Takata sobre o Brudna, uma exceletne pessoa e cientista.

Homero
----- Original Message -----
From: rmtakata
To: ciencialist@yahoogrupos.com.br
Sent: Friday, February 18, 2005 8:27 AM
Subject: [ciencialist] Re: adeus.



--- Em ciencialist@yahoogrupos.com.br, marcelo ferrari
> arrogancia faz parte do doutoramento em ciencias. Não tormam os

Doutoramento levando 'a arrogancia?

Como o unico q. sabidamente fez doutorado aqui eh o Brudna... Fio e
dou fé q. Luiz Robert Brudna Hozle pouco ou nada tem de arrogante,
sendo um ativo e interessado divulgador das ciencias, inclusive
mantendo esta lista para a participacao de todos os interessados em
ciencias - independente da fe', filiacao ideologica, sexo, raca, time
de futebol, estado civil, orientacao sexual, QI, escolaridade, crenca
politica, nivel socioeconomico, preferencia culinaria, numero de
dedos, razao cintura-quadril, humor, alfabetizacao digital, idade,
local de nascimento, tempo de servico, altura, peso, relacoes
filogeneticas ou qq outra forma de discriminacao.

[]s,

Roberto Takata






##### ##### #####

Para saber mais visite
http://www.ciencialist.hpg.ig.com.br


##### ##### ##### #####


Yahoo! Grupos, um serviço oferecido por:







------------------------------------------------------------------------------
Links do Yahoo! Grupos

a.. Para visitar o site do seu grupo na web, acesse:
http://br.groups.yahoo.com/group/ciencialist/

b.. Para sair deste grupo, envie um e-mail para:
ciencialist-unsubscribe@yahoogrupos.com.br

c.. O uso que você faz do Yahoo! Grupos está sujeito aos Termos do Serviço do Yahoo!.



[As partes desta mensagem que não continham texto foram removidas]



SUBJECT: Re: adeus.
FROM: Manuel Bulcão <manuelbulcao@uol.com.br>
TO: ciencialist@yahoogrupos.com.br
DATE: 18/02/2005 17:29


--- Em ciencialist@yahoogrupos.com.br, Luis Brudna <luisbrudna@g...>
escreveu
> Hehe... obrigado!
>
> Seus puxa! :-)

Manuel: É pique, é pique... é pique, é pique, é pique... É hora, é
hora... é hora, é hora, é hora... Ra-tim-bum... Brudna! Brudna!
Brudna! Brudna!... :-)

[]s
Manuel Bulcão






SUBJECT: Re: adeus.
FROM: Manuel Bulcão <manuelbulcao@uol.com.br>
TO: ciencialist@yahoogrupos.com.br
DATE: 18/02/2005 17:46


--- Em ciencialist@yahoogrupos.com.br, "rmtakata" <rmtakata@a...>
escreveu

> Como o unico q. sabidamente fez doutorado aqui eh o Brudna... Fio
e dou fé q. Luiz Robert Brudna Hozle pouco ou nada tem de arrogante,
sendo um ativo e interessado divulgador das ciencias, inclusive
mantendo esta lista para a participacao de todos os interessados em
ciencias - independente da... orientacao sexual... relacoes
filogeneticas...

Manuel: É, aqui na Ciencialist existe um amplo espectro que vai da
Inteligência Artificial (máquina de Turing universal) até o macaco
ergótico.

Só fiquei curioso quanto ao quesito "orientação sexual". Quem será o
viado da lista? :-)

[]s
Manuel Bulcão






SUBJECT: Re: [ciencialist] Re: adeus.
FROM: "Alberto Mesquita Filho" <albmesq@uol.com.br>
TO: <ciencialist@yahoogrupos.com.br>
DATE: 18/02/2005 18:26

----- Original Message -----
From: "Luis Brudna"
Sent: Friday, February 18, 2005 1:43 PM
Subject: Re: [ciencialist] Re: adeus.


> Seus puxa! :-)

Não apoiado. Ou melhor, apóio o que eles disseram.

> Pelo que pude ver não apareceu nenhuma mensagem agressiva, não vejo
> motivos para alguém sair da lista.

Também não li nenhuma mensagem agressiva, se bem que "não tive saco" para
ler todas as mensagens. Também procurei não palpitar pois quando me disponho
a debater essas balelas acabo me indispondo com aqueles com os quais
concordo, se bem que discordando frontalmente. ;-) Com efeito, fiz meu mapa
astrológico e descobri que não estou num período favorável para discutir
astrologia e temas afins.

[ ]´s
Alberto
http://ecientificocultural.com/indice.htm
Mas indiferentemente a tudo isso, o neutrino tem massa, o elétron não é
uma carga elétrica coulombiana e a Terra se move. E a história se repetirá.



SUBJECT: Fw: hora planetaria
FROM: "Luiz Ferraz Netto" <leobarretos@uol.com.br>
TO: "ciencialist" <ciencialist@yahoogrupos.com.br>
DATE: 18/02/2005 19:40

Aos sábios da astrologia ........

[]'
===========================
Luiz Ferraz Netto [Léo]
leobarretos@uol.com.br
http://www.feiradeciencias.com.br
===========================
-----Mensagem Original-----
De: msark
Para: leobarretos
Enviada em: sexta-feira, 18 de fevereiro de 2005 15:00
Assunto: hora planetaria


Ola professor, estou fazendo um estudo sobre astrologia e tenho a seguinte duvida.
Como posso calcular a primeira hora e primeiro minuto de 20 de março no planeta e nao em um determinado lugar ou seja qual seria a primeira hora do planeta a 20 de março no equinocio? Seria ela a de grenwich ??? ou a meia noite ??? porque ?
Atenciosamente,
Miken Sark





--------------------------------------------------------------------------------


No virus found in this incoming message.
Checked by AVG Anti-Virus.
Version: 7.0.300 / Virus Database: 265.8.8 - Release Date: 14/02/2005

----------

No virus found in this outgoing message.
Checked by AVG Anti-Virus.
Version: 7.0.300 / Virus Database: 265.8.8 - Release Date: 14/02/2005


[As partes desta mensagem que não continham texto foram removidas]



SUBJECT: Re: [ciencialist] Re: adeus.
FROM: "Luiz Ferraz Netto" <leobarretos@uol.com.br>
TO: <ciencialist@yahoogrupos.com.br>
DATE: 18/02/2005 19:45

Não bastou que me colocassem no Crush Fixo, ainda, graças ao preconceito, mais uma vez fui excluído, desta vez da lista takatiana.

[]'
deus
===========================
-----Mensagem Original-----
De: "Manuel Bulcão" <manuelbulcao@uol.com.br>
Para: <ciencialist@yahoogrupos.com.br>
Enviada em: sexta-feira, 18 de fevereiro de 2005 16:46
Assunto: [ciencialist] Re: adeus.




--- Em ciencialist@yahoogrupos.com.br, "rmtakata" <rmtakata@a...>
escreveu

> Como o unico q. sabidamente fez doutorado aqui eh o Brudna... Fio
e dou fé q. Luiz Robert Brudna Hozle pouco ou nada tem de arrogante,
sendo um ativo e interessado divulgador das ciencias, inclusive
mantendo esta lista para a participacao de todos os interessados em
ciencias - independente da... orientacao sexual... relacoes
filogeneticas...

Manuel: É, aqui na Ciencialist existe um amplo espectro que vai da
Inteligência Artificial (máquina de Turing universal) até o macaco
ergótico.

Só fiquei curioso quanto ao quesito "orientação sexual". Quem será o
viado da lista? :-)

[]s
Manuel Bulcão




--
No virus found in this outgoing message.
Checked by AVG Anti-Virus.
Version: 7.0.300 / Virus Database: 265.8.8 - Release Date: 14/02/2005



SUBJECT: Re: ASTROLOGY IS BIGOTRY
FROM: "oraculo333" <oraculo@atibaia.com.br>
TO: ciencialist@yahoogrupos.com.br
DATE: 19/02/2005 01:14


Olá Taborda

he he he, então vamos lá..:-)

"Taborda: a) Mostrar que a critica dos ceticos não faz sentido pq
desconsidera o proprio modelo / teoria da astrologia.É baseado em
informações parciais e/ou erradas do modelo/teoria astrologica. "

Isso só seria verdade se a única astrologia existente fosse a sua ou
se sua noção de astrologia fosse a correta e todas as outras erradas.
Como boa parte dos astrólogos discorda de você e dessa afirmativa, ela
é incorreta.

O modelo, os diversos modelos, de astrologia, tem diversas críticas.
Os argumentos céticos se referem a estes modelos, bem como ao seu
modelo. Acusar críticas ou analises baseadas em outras formas ou
alegações astrologicas de não fazer sentido, não faz sentido..:-)

"Taborda: b) Que ao cometer a) os ceticos violam o primeiro principio
do método cientifico, sendo suas considerações não-cientificas também."

De forma alguma, todas as críticas aqui apresentadas envolvem o uso
rigoroso do método e boa parte da crítica a essa refutação surge
exatamente devido a esse rigor. Coisas como "mente fechada", "novos
paradigmas necessários" e mesmo o famoso "a forma atual da ciência e
seu método não é suficiente para lidar com esses novos conhecimentos
ou com essas outras formas de saber" demonstram claramente isso. O
método, aplicado a essas alegações, as desmente e as refuta, por isso
é recusado como ferramenta de análise por quem crê.

"Taborda: c) Que muitos dos argumentos dos ceticos tb são falacias,
porque nem sequer têm base cientifica, historica ou qq outra."

Falácias são erros em argumentos, erros específicos, que podem ser
facilmente demonstrados, coisa que você não fez. Alegar que algo é uma
falácia sem dizer que tipo de falácia, o que a caracteriza como e onde
ela falha, em suas premissas ou na conclusão decorrente, é
tergiversar, não argumentar.

Desmonte um argumento cético falacioso, não baseado em sua própria e
pessoal visão do que é astrologia, mas formalmente, apontando as
premissas incorretas, a indução não autorizada, ou a conclusão não
procedente, e veremos o que acontece..:-)

Em tempo, não concordar com você não é um tipo de falácia..:-)

"Taborda: d) Que os ceticos cometem as mesmas falacias de argumentação
que apontam aos crentes."

Idem acima..:-) Céticos afirmam que, sem evidencias sólidas, é
perfeitamente razoável concluir que não existem evidencias de algo ser
real. Apenas isso..:-) Pode demonstrar que existem evidencias sólidas
ou pode demonstrar que evidências solidas não são necessárias, mas
apenas acusar de falácias os céticos, é incorreto.

"Taborda: e) Que eu não tenho opinião formada sobre a eficacia da
astrologia. "

risos...:-) Essa é bem interessante, frente a toda essa discussão..:-)
Eu também não tenho opinião, tenho uma conclusão baseada na falta de
evidencias de eficácia, realidade, confiabilidade, etc, sobre a
astrologia. Sem evidencias, sem crença.

E se não tem opinião, deveria usar o mesmo ceticismo que aplica em
outras alegações extraordinárias, como o Pé Grande, duendes ou
unicórnios. Também inexistem evidencias de que existem, embora não
existam provas de que NÃO existam. Apenas suspender a opinião não é
razoável, concluir que faltam evidencias para afirmar a existencia, é..:-)

"Taborda: f) que , sendo d) não me cabe apresentar provas seja de que
tipo forem, pois não é meu objectivo provar a astrologia, apenas
desprovar o que os ceticos dizem sobre ela. E para isso aporto as
provas quanto a mim necessárias."

Excelente..:-) Uma vez que o conceito e definição de ciência é
justamente o contrário, apresentar provas e basear em evidencias
confirmáveis por terceiros, você acaba de concordar que a astrologia
não se encaixa e, portanto, não é ciência e não produz resultados, ao
se aplicar o rigido método cientifico..:-)

"Desprovar" o que os céticos dizem sobre a astrologia, é, provar o
que a astrologia alega. Isso é evidente, já que os céticos dizem que
a) ela não tem mecanismo de ação conhecido, b) ela não produz
resultados confirmáveis, c) ela falha em testes efetuados com rigor
científico e d) ela não é real..:-) Para provar que isso que os
céticos dizem sobre a astrologia, deve provar seu contrário. E isso
nenhum astrologo pode fazer até o momento.

O resto é um interessante (embora repetitivo) debate sem fim e com
muito jogos de palavras..:-)

Um abraço.

Homero









SUBJECT: Faltou um..:-) (era Re: ASTROLOGY IS BIGOTRY)
FROM: "oraculo333" <oraculo@atibaia.com.br>
TO: ciencialist@yahoogrupos.com.br
DATE: 19/02/2005 01:20


Ops, faltou a última questão proposta..:-)

"Taborda: g) que os ceticos são tão crentes no seu desacreditar como
os crentes no seu acreditar, pois nenhum dos dois tem provas do que
afirma. "

Este é o falso argumento padrão de toda crença, religião ou alegação
sem evidencias, de que é preciso tanto provas para crer como para não
crer..:-)

Esperava mais a esta altura...:-)

Sem evidencias, não é preciso provas para não acreditar. A falta de
evidencias permite concluir que ... faltam evidencias..:-) Ninguém
espera provas negativas para unicórnios, nem duendes, fada do dente ou
Pé Grande. Nem o Taborda, claro.

Descrer, pela absoluta falta de provas de existencia, de evidencias
confirmáveis, não exige crença, nem se compara a acreditar sem provas.
Isso é evidente..|:-)

Não acredito em astrologia, pela falta de evidencias e também não
acredito em seres intra-terrenos ou em unicórnios que habitam as luas
de Jupiter. Embora nenhum deles tenha provas de não existir, a todos
eles faltam evidencias, provas, de existir. E isso basta, para mim, ou
para o Taborda..:-)

Homero





--- Em ciencialist@yahoogrupos.com.br, "Sergio M. M. Taborda"
<sergiotaborda@t...> escreveu
>
> --- Em ciencialist@yahoogrupos.com.br, "Oraculo" <oraculo@a...> escreveu
> > Olá Taborda
> >
> > Você demonstra o famoso "conte os acertos, descarte os erros" com
> bastante propriedade..:-) Mas o argumento e as criticas a astrologia
> continuam incolumes, apenas quem acredita com muita fé pode manter
> essa postura por muito tempo..:-)
> >
> > Texto:> Você está notando que, se ler o suficiente dos textos e
> afrimações,
> > poderá encontrar virutalmente todas as facetas de todo tipo de
> > comportamento?
> >
> > Taborda: Não. Não notei. "
> >
> >
> > Bem, nào notou por implicancia, mas está bem claro..:-) Mesmo com
> minha péssima tradução (eu avisei que era meia boca, feita de
> madrugada sem muito cuidado..:-), fica evidente que o afirmado no
> artigo pode ser aplicado a qualquer caso, pela contrariedade de termos
> e oposição de aspectos.
> >
> > Por exemplo, se quem lê o prognóstico não gosta da pessoa relatada,
> encontrará um claro "acerto" no egoísmo da avaliação. Já se quem lê
> gosta da pessoa, sua mãe por exemplo, pode encontrar o acerto na
> afirmação de generosidade (e ela certametne encontrará, como qualquer
> màe..:-) e pode aceitar que o egoismo é apenas "um forte desejo de
> mudar o mundo para melhor". Fugir desse argumento afirmando que "pode
> procurar, mas será que encontra" é ilegítimo. Se o astrólogo indica
> que pode procurar, é porque espera achar. Ou melhor, talvez seja mesmo
> para deixar a margem, o caminho da fuga, o uso de termos vagos o
> suficiente para permitir que, na falta de qualquer generosidade,
> sempre reste a explicaçào, "ei, procurar não significa achar"..:-)
> >
> > É engraçado como os mesmos mecanismos que usam processos mentais e
> truques de semantica, apresentados no outro texto, sobre "persuaders
> hidden", possam ser encontrados em sua defesa da astrologia, com
> bastante clareza.
> >
> > O que não se encontra é a apresentação de evidencias de eficácia do
> que defende. Como criacionistas, que tentam demonstrar que a evolução
> está errada, mas não tentam demonstrar que o criacionismo está correto
> (em termos de evidencias e provas, claro, eles sabem que é correto
> porque seu livro sagrado assim o diz..:-), este debate fica apenas
> tentando demonstrar que a crítica dos céticos está equivocada
>
> Exactamente pq esse é meu unico, e fianl, objectivo.
>
> Para chegar lá, passo por:
> a) Mostrar que a critica dos ceticos não faz sentido pq desconsidera o
> proprio modelo / teoria da astrologia.É baseado em informações
> parciais e/ou erradas do modelo/teoria astrologica.
> b) Que ao cometer a) os ceticos violam o primeiro principio do método
> cientifico, sendo suas considerações não-cientificas também.
> c) Que muitos dos argumentos dos ceticos tb são falacias, porque nem
> sequer têm base cientifica, historica ou qq outra.
> d) Que os ceticos cometem as mesmas falacias de argumentação que
> apontam aos crentes.
> e) Que eu não tenho opinião formada sobre a eficacia da astrologia.
> f) que , sendo d) não me cabe apresentar provas seja de que tipo
> forem, pois não é meu objectivo provar a astrologia, apenas desprovar
> o que os ceticos dizem sobre ela. E para isso aporto as provas
> quanto a mim necessárias.
> g) que os ceticos são tão crentes no seu desacreditar como os crentes
> no seu acreditar, pois nenhum dos dois tem provas do que afirma.
>
>
> Sérgio Taborda





SUBJECT: Onde NÃO há química?
FROM: "Emiliano Chemello - Yahoo Grupos" <chemelloe@yahoo.com.br>
TO: <ciencialist@yahoogrupos.com.br>, <quimica-qaw@yahoogrupos.com.br>, <quimica@grupos.com.br>, <naeq-ucs@yahoogrupos.com.br>
DATE: 19/02/2005 01:43

Amigos (as),

Estava no MSN Messenger quando surgiu uma dúvida de uma menina de 15
anos (primeiro ano do ensino médio). Ela me perguntou:

"onde *não* há química?".

Ponderei antes de formular uma resposta. Sendo bastante direto, até
cometendo alguns equívocos propositais, consegui reduzir a 'genérica'
pergunta nestas frases.

"A química estuda as transformações da matéria. Se não há matéria, não há
transformação e, por consequência, não há química. A grande questão da
ciência (não só da química) é: será que existe o 'vácuo' da forma como o
concebemos?"

Alguém concorda? Alguém discorda? Alguém não está nem ai para isso? :-)

1 mol de [ ]'s do
Emiliano Chemello
emiliano@quimica.net
http://www.quimica.net/emiliano
http://www.ucs.br/ccet/defq/naeq
[ MSN ] chemelloe@hotmail.com
[ ICQ ] 145060604

"Rien ne se perd, rien ne se crée, tout se transforme"
Lavoisier, químico francês (1743-1794)




SUBJECT: Sticker Shock (o alerta colado em livros sobre evoluçaõ nos USA.:-)
FROM: "Oraculo" <oraculo@atibaia.com.br>
TO: <ciencialist@yahoogrupos.com.br>
DATE: 19/02/2005 03:41

http://www.scientificamerican.com/article.cfm?colID=15&articleID=00022DE1-0C15-11E6-B75283414B7F0000

Sticker Shock
In the beginning was the cautionary advisory
By Steve Mirsky

Brushfires are raging all across America over the teaching of evolution, as various antievolution interests attempt to give religiously based views equal footing in science classes. These fires are fueled by so-called creation scientists, who allege that they have scientific evidence against evolution. (They don't.) Their co-conspirators, the "intelligent design" crowd, go with the full-blown intellectual surrender strategy--they say that life on earth is so complex that the only way to explain it is through the intercession of an intelligent superbeing. (They don't mention you-know-who by name as the designer, but you know who you-know-who is, and it isn't Brahma.)

One little blaze can be found in Cobb County, Ga. As this issue of Scientific American went to press, a federal judge in Atlanta was in the process of deciding whether biology textbooks in the county could continue to sport a warning sticker that read: "This textbook contains material on evolution. Evolution is a theory, not a fact, regarding the origin of living things. This material should be approached with an open mind, studied carefully, and critically considered."

Maybe that last sentence should be stamped into every textbook (and some other books I can think of). And maybe they could rewrite the advisory so that it's accurate. Perhaps something like, "Variation coupled with natural selection is the most widely accepted theory that explains evolution. Evidence for evolution itself is so overwhelming that those who deny its reality can do so only through nonscientific arguments. They have every right to hold such views. They just can't teach them as science in this science class."

But why pick on evolution in the first place when there's so much to be offended by in virtually any science class? I propose that Cobb County-style stickers be placed in numerous other textbooks. Here are some suggestions:

Sticker in Introduction to Cosmology: "Astronomers estimate the age of the universe to be approximately 13 billion years. If evolution ticks you off because you believe that the earth is only 6,000 years old, cosmology should really make smoke come out of your ears. There's a fire extinguisher next to the telescope."

Sticker in Geography for Today: "Some people believe that the earth is flat. An ant probably thinks the beach ball he's walking on is flat, too. Anyway, this book says the earth is more like an oblate spheroid. Now go find Moldova on a map."

Sticker in Earth Science: "You are free to exercise your First Amendment rights in this class and to identify all stratigraphic layers as being 6,000 years old. We are free to flunk you."

Sticker in Collegiate Chemistry: "Electrons. They're like little tiny ball bearings that fly around the atomic nucleus like planets orbit the sun. Except that they're actually waves. Only what they really are are probability waves. But they do make your MP3 player run, seriously."

Sticker in Our Solar System: "Remember they said in chemistry class that electrons fly around the nucleus like planets orbit the sun? Some people think the sun and other planets go around the earth. You'll have a much easier time with the math if you just let everybody go around the sun, trust me."

Sticker in Physics for Freshmen: "We know that a lot of what's in this book is wrong, and with any luck they'll eventually find out that even more of it is wrong. But it's not so far off, it took some real geniuses to get us this close, and it's way better than nothing."

Sticker in Creationism for Dummies: "Religious belief rests on a foundation of faith. Seeking empirical evidence for support of one's faith-based beliefs therefore could be considered pointless. Or even blasphemous."

Sticker in Modern Optics: "CAUTION! Dark ages in mirror may be closer than they appear."

[As partes desta mensagem que não continham texto foram removidas]



SUBJECT: Re: Onde NÃO há química?
FROM: Maria Natália <grasdic@hotmail.com>
TO: ciencialist@yahoogrupos.com.br
DATE: 19/02/2005 04:31


Emiliano:
Para a menina dá por enquanto.
Mas sabes que o vácuo não existe. Tem enrgia e matéria negra...
Mas já agora para uma moça tão nova dizer isso...Será que alguém na
escolinha já a fez odiar química?
Ai ai...sem 50% de aulas de laboratório da totalidade de tempos
lectivos dedicado à Química também eu a odiaria.
Desculpa lá se calhar a menina até tem trabalho sempre em sua vbancada
e em grupo de 3 apenas. Mas tenho ouvido tão mal de colégios sem
laboratório para os alunos e total desprezo pelos tais 50% de hands on
dos alunos...
Espero ter-me enganado
[]'
Maria Natália
--- Em ciencialist@yahoogrupos.com.br, "Emiliano Chemello - Yahoo
Grupos" <chemelloe@y...> escreveu
> Amigos (as),
>
> Estava no MSN Messenger quando surgiu uma dúvida de uma menina de 15
> anos (primeiro ano do ensino médio). Ela me perguntou:
>
> "onde *não* há química?".
>
> Ponderei antes de formular uma resposta. Sendo bastante direto, até
> cometendo alguns equívocos propositais, consegui reduzir a 'genérica'
> pergunta nestas frases.
>
> "A química estuda as transformações da matéria. Se não há matéria,
não há
> transformação e, por consequência, não há química. A grande questão da
> ciência (não só da química) é: será que existe o 'vácuo' da forma como o
> concebemos?"
>
> Alguém concorda? Alguém discorda? Alguém não está nem ai para isso? :-)
>
> 1 mol de [ ]'s do
> Emiliano Chemello
> emiliano@q...
> http://www.quimica.net/emiliano
> http://www.ucs.br/ccet/defq/naeq
> [ MSN ] chemelloe@h...
> [ ICQ ] 145060604
>
> "Rien ne se perd, rien ne se crée, tout se transforme"
> Lavoisier, químico francês (1743-1794)





SUBJECT: Re: adeus.
FROM: Maria Natália <grasdic@hotmail.com>
TO: ciencialist@yahoogrupos.com.br
DATE: 19/02/2005 04:44


Ray:

Mas porquê!? É assim em todo o lado e não vale a pena ferver.
Qual pessoal de ciência qual quê*!! É vê-los no boteco e na Acropóle à
cacetada. É xenofobia, racismo, intolerância, tabus...TUDO vem ao de
cima. Tens de ir fazer estágio ao Orkut.
Que maçada não estar aqui para tentar apagar fogo.
E até é interessante que certas coisas aconteçam: aprende-se muito
sobre a educação do pessoal, do chá que tomaram em pequeninos...tens
de saber tirar proveito de tudo. E depois coleccionar os ódios de
estimação é f...rsrs.
Vou-te escrever em privado pois já devo estar a falar pró boneco
Um abraço de...solidariedade.
Maria Natália
* Ferrari, qual a tua,pá? Te fazes de lucas agora? E os de letras à
pancada!? Te porta bem menino. Cuspindo para o ar te cai em cima.
(te gramo no Orkut rsrs)

--- Em ciencialist@yahoogrupos.com.br, "rayfisica" <rayfisica@y...>
escreveu
>
> Quando eu entrei para essa lista foi para aprender e conviver ainda
> que distante com pessoas sábias, inteligentes e que amassem a
> ciência.
> Encontrei exatamente isso em algumas pessoas aqui e a essas eu
> agradeço, de verdade o tempo que dedicaram a mim.
> Desprezo os arrogantes e intolerantes que contradizem se.
> Desculpem-me, pois não tenho tempo a perder com gente assim.
> Adeus.





SUBJECT: Re: adeus.
FROM: Maria Natália <grasdic@hotmail.com>
TO: ciencialist@yahoogrupos.com.br
DATE: 19/02/2005 04:56



Viva a Astronomia!!!!!!!!!!!!!!!!!!!!!!!!!!!!!!!
Cada um que estude o que quiser. Não foi criado em Roma um curso para
exorcistas!?
Espero é que não venham aqui para perto de minha porta fazer estágio.
E agora que já se chegou a Titã há muito espaço para todos.
Me custa não poder ter ajudado, indo em pvt.
Brudna a lista devia ter bombeiro/a permanente...Mas que falta de
segurança.
Um abraço, colega professor
Maria Natália

--- Em ciencialist@yahoogrupos.com.br, Luis Brudna <luisbrudna@g...>
escreveu
> Hehe... obrigado!
>
> Seus puxa! :-)
>
> Pois é. Vi que o clima esquentou aqui na lista, e pra variar parece
> que foi por causa da astrologia. Fico ansioso quando esse assunto
> aparece pq não tenho conhecimentos para debater. Quero meter colher
> mas essa parte da culinária eu não domino.
>
> Pelo que pude ver não apareceu nenhuma mensagem agressiva, não vejo
> motivos para alguém sair da lista.
>
> Até
> Luís Brudna
>
>
> On Fri, 18 Feb 2005 14:33:59 -0300, Oraculo <oraculo@a...> wrote:
> >
> > Olá
> >
> > Assino embaixo da declaração do Takata sobre o Brudna, uma
exceletne pessoa e cientista.
> >
> > Homero
> > ----- Original Message -----
> > From: rmtakata
> > To: ciencialist@yahoogrupos.com.br
> > Sent: Friday, February 18, 2005 8:27 AM
> > Subject: [ciencialist] Re: adeus.
> >
> > --- Em ciencialist@yahoogrupos.com.br, marcelo ferrari
> > > arrogancia faz parte do doutoramento em ciencias. Não tormam os
> >
> > Doutoramento levando 'a arrogancia?
> >
> > Como o unico q. sabidamente fez doutorado aqui eh o Brudna... Fio e
> > dou fé q. Luiz Robert Brudna Hozle pouco ou nada tem de arrogante,
> > sendo um ativo e interessado divulgador das ciencias, inclusive
> > mantendo esta lista para a participacao de todos os interessados em
> > ciencias - independente da fe', filiacao ideologica, sexo, raca,
time
> > de futebol, estado civil, orientacao sexual, QI, escolaridade,
crenca
> > politica, nivel socioeconomico, preferencia culinaria, numero de
> > dedos, razao cintura-quadril, humor, alfabetizacao digital, idade,
> > local de nascimento, tempo de servico, altura, peso, relacoes
> > filogeneticas ou qq outra forma de discriminacao.
> >
> > []s,
> >
> > Roberto Takata





SUBJECT: Re: [ciencialist] adeus.
FROM: JVictor <jvoneto@uol.com.br>
TO: ciencialist@yahoogrupos.com.br
DATE: 19/02/2005 11:14

de JVictor para Rayfísica,


rayfisica escreveu:

>
> Quando eu entrei para essa lista foi para aprender e conviver ainda
> que distante com pessoas sábias, inteligentes e que amassem a
> ciência.
> Encontrei exatamente isso em algumas pessoas aqui e a essas eu
> agradeço, de verdade o tempo que dedicaram a mim.
> Desprezo os arrogantes e intolerantes que contradizem se.
> Desculpem-me, pois não tenho tempo a perder com gente assim.
> Adeus.
>
>
> Rayfisica.


Como ando muito sem tempo de ler todos os e-mails desta lista, chamou-me
a atenção especialmente o assunto: adeus. Não sei a razão que o levou a
afastar-se.
Mas, de antemão, eu lhe digo: se foi em razão de algum participante o
haver criticado ou coisas da espécie, releve, ou devolva a crítica da
maneira que achar melhor. Se foi em razão de não concordar com posturas
pessoais e/ou científicas de quem quer que seja, também releve, respire
fundo e aprenda com isso. Pois, tirar de tudo uma lição, aprendê-la, é
próprio de pessoas sábias(que não precisam ser cultas, saber física,
matemática, ou qualquer outra coisa. Conheço pessoas simples, sem muita
formação como a gente pensa que deva ser, que são sábias. Minha querida
sogra é uma dessas pessoas que, quase não fala, mas quando o faz,
paramos, escutamos e podemos aprender! Um empregado que meu pai tinha,
analfabeto, me ensinou muita coisa que, ainda hoje, guardo em mim.
Pessoas sábias são assim. O fluxo luminoso que emitem é único e divergem
dela, de seu interior, de maneira natural. Precisamente como ensinou um
gordinho chamado Buda). Ver que alguém por mais "cão chupando manga" que
seja, na área do conhecimento,e que, mesmo assim, tenha pouca coisa a
ensinar, não deve ser motivo de alvoroço. Você pode tirar lições
importantes, de algo assim: por exemplo, não ser como essa pessoa é uma
desses impórtantes aprendizados; pois, se alguém é desagrável no que
diz, de maneira sistemática, deve sê-lo também no que faz e no
relacionamento com os coitados de sua vizinhança. Ser um desgraçado a
menos, diz o velho e sábio ditado. Eis a questão, ser ou não, como dizia
meu amigo xeiquispiare, em algum botequim.
A questão básica é a seguinte. Do mesmo jeito que a gente não consegue
sair com vida da vida, morre mesmo, com certeza, quer Heisenberg queira
ou não, por igual forma você não se livrará de pessoas da espécie, onde
quer que você esteja, na face deste nosso velho e quase redondo planeta.
No Ciencialist, no Brasil, Rússia, Tibet, China, ou numa aldeia ainda
primitiva e não descoberta, lá pelos confins da África. As pessoas, em
todos esses lugares são movidas pelos mesmos impulsos inconscientes que
compõem nosso esqueleto psicológico. Outro dia falei sobre estes tais. É
nisso em que acredito, sobre comportamentos, qualquer que seja o
desgraçado.
E há outra razão mais importante ainda para você rever sua postura: é
que a gente aprende com você. Eu, pelo menos; acredito que outros
também. Seus questionamentos são pertinentes e inteligentes. E isto é
mais importante do que saber respondê-los. São questionamentos, boas
perguntas, que movem a ciência, que garantem sua evolução, sua
atualização. Você quer um exemplo de total despojamento, noutro
aspecto, o comportamental?. Eis então: o Brudna. É o idealizador(ou um
dos, não sei) desse magnifício fórum. Com certeza, fica alí, lê todos os
e-mails. Absorve com prazer alguns, engole seco ou quadrado outros,
opina pouco, pois respeita as diversas tendências; e quando opina é para
acrescentar, para somar, às vezes com opiniões pessoais, outras com
perguntas ou questionamentos, e outras com sites informativos, sempre
relacionado aos assuntos. Não sei onde ele arranja tempo, mas isso é
outra coisa. Mas tenho certeza de que ele já aprendeu um pouquinho mais
do que sabia, antes do Ciencialist. É ou não, vero?

Sds,

Victor.

>
>
>
> ##### ##### #####
>
> Para saber mais visite
> http://www.ciencialist.hpg.ig.com.br
>
>
> ##### ##### ##### #####
>
>
> *Yahoo! Grupos, um serviço oferecido por:*
> <http://br.rd.yahoo.com/SIG=12am4b5i9/M=264379.5078783.6203979.1588051/D=brclubs/S=2137111528:HM/EXP=1108800599/A=2191897/R=0/SIG=10vqa2grn/*http://br.diversao.yahoo.com/>
>
> <http://br.rd.yahoo.com/SIG=12am4b5i9/M=264379.5078783.6203979.1588051/D=brclubs/S=2137111528:HM/EXP=1108800599/A=2191897/R=1/SIG=10vqa2grn/*http://br.diversao.yahoo.com/>
>
>
>
> ------------------------------------------------------------------------
> *Links do Yahoo! Grupos*
>
> * Para visitar o site do seu grupo na web, acesse:
> http://br.groups.yahoo.com/group/ciencialist/
>
> * Para sair deste grupo, envie um e-mail para:
> ciencialist-unsubscribe@yahoogrupos.com.br
> <mailto:ciencialist-unsubscribe@yahoogrupos.com.br?subject=Unsubscribe>
>
> * O uso que você faz do Yahoo! Grupos está sujeito aos Termos do
> Serviço do Yahoo! <http://br.yahoo.com/info/utos.html>.
>
>
>
>
> __________ Informação do NOD32 1.1001 (20050217) __________
>
> Esta mensagem foi verificada pelo NOD32 Sistema Antivírus
> http://www.nod32.com.br




SUBJECT: Fw: f�sica t�rmica
FROM: "Luiz Ferraz Netto" <leobarretos@uol.com.br>
TO: "ciencialist" <ciencialist@yahoogrupos.com.br>
DATE: 19/02/2005 12:59

Ajudemos o guri?
[]'
===========================
Luiz Ferraz Netto [Léo]
leobarretos@uol.com.br
http://www.feiradeciencias.com.br
===========================
-----Mensagem Original-----
De: gatinhavfs
Para: leobarretos
Enviada em: sexta-feira, 18 de fevereiro de 2005 13:56
Assunto: física térmica


Viviane Freitas

Quais são as fontes ou materias naturais que produzem calor?

E no dia-a-dia estamos em contato permanente com coisas relacionadas com o calor. Cite aparelhos, máquinas, instrumentos ou dispositivos que tenham relação com o calor.

Daqui a poucos anos os combustiveis como por exemplo: gasolina, o oleo, a querosene e o gás obtidos do petróleo irão se esgotarem como será o mundo sem eles?


--------------------------------------------------------------------------------


No virus found in this incoming message.
Checked by AVG Anti-Virus.
Version: 7.0.300 / Virus Database: 265.8.8 - Release Date: 14/02/2005

----------

No virus found in this outgoing message.
Checked by AVG Anti-Virus.
Version: 7.0.300 / Virus Database: 265.8.8 - Release Date: 14/02/2005


[As partes desta mensagem que não continham texto foram removidas]



SUBJECT: A matemática e o sexo
FROM: "brudna" <lrb@iq.ufrgs.br>
TO: ciencialist@yahoogrupos.com.br
DATE: 19/02/2005 14:30



Para um sábado excitante. :-) Um pouco de matemática e sexo.

http://revistagalileu.globo.com/Galileu/0,6993,ECT900781-1719-1,00.html

(perceba a última pergunta da entrevista, bem no fim da 2a página.
Arghh! :-) )



Mais um pouco do assunto
http://www.humornaciencia.hpg.ig.com.br/matematica/sexo.htm
http://www.humornaciencia.hpg.ig.com.br/matematica/pai.htm

Se alguém achar mais coisas sobre o assunto envie pra lista, vou
enriquecer o ´post´ http://antiparticula.webcindario.com/index.php?p=187


Até
Luís Brudna







SUBJECT: Re: Onde NÃO há química?
FROM: Manuel Bulcão <manuelbulcao@uol.com.br>
TO: ciencialist@yahoogrupos.com.br
DATE: 19/02/2005 18:23


Oi Emiliano,

> "onde *não* há química?".

Manuel: Dentro do buraco profundo da minha ignorância, eu pensava
que o objeto da química fosse a estrutura, as transformações e as
propriedades macroscópicas das substâncias 'atômicas' simples e
compostas, de modo que, durante aquele período da história do
Universo em que tudo consistia num caldo homogêneo de fótons,
elétrons, neutrinos e hádrons extremamente instáveis, ainda não
existiam substâncias no sentido 'químico' do termo.

Avento que, antes da cisão da força eletrofraca em eletromagnetismo
e força nuclear fraca, inexistiam quaisquer processos químicos. Após
essa quebra de simetria, surge uma proto-química na medida em que, a
partir de então, aparecem radionuclídeos cuja vida dura um átimo.

Processos químicos propriamente ditos só advieram, penso eu, depois
que os fótons se separaram da matéria (fenômeno que deu origem à
radiação cósmica de fundo), permitindo a combinação dos elétrons com
os núcleos e, por conseguinte a formação de átomos estáveis.

Também não creio que, no núcleo de um buraco negro, ocorram
processos químicos.

Ainda se pode imaginar uma fase futura da história do Universo em
expansão (daqui a muitos, muitíssimos `eões') em que não haverá um
único buraco negro, sequer um bárion real, nenhum positrônio que não
seja virtual, pois todos eles há muito teriam evaporado, decaído ou
se desintegrado. Nesta remotíssima `era', restaria uma sopa
extremamente rala de fótons, neutrinos, uns poucos elétrons isolados
boiando e, ipso facto, total ausência de processos químicos.

Penso que a lógica vem antes da física, a física antecede a química,
a química precede a biologia, a biologia é anterior à consciência, a
consciência dá origem à sensação de absurdo (`Angst der Kreatur'),
da sensação de absurdo surge a idéia de deus e que este, talvez,
seja a própria lógica que está no começo da história.

Abraços,
Manuel Bulcão
PS.: Desculpem-me se falei besteira demais. :-)





SUBJECT: RE: [ciencialist] Re: (não um hoax!) Foucault / Luz ondas ou corpusculos.
FROM: "murilo filo" <avalanchedrive@hotmail.com>
TO: ciencialist@yahoogrupos.com.br
DATE: 19/02/2005 19:54

Faltam-me argumentos para contradizer sua afirmação: vc é realmente muito
chato...
Hasta la vista, pentelho... M.

>From: César A. K. Grossmann <cesarakg@bol.com.br>
>Reply-To: ciencialist@yahoogrupos.com.br
>To: ciencialist@yahoogrupos.com.br
>Subject: [ciencialist] Re: (não um hoax!) Foucault / Luz ondas ou
>corpusculos.
>Date: Fri, 18 Feb 2005 12:37:23 -0000
>
>
>
>--- Em ciencialist@yahoogrupos.com.br, "murilo filo"
><avalanchedrive@h...> escreveu
> > Sol pelas costas... eu sempre me referí a fotos contra o sol... o
>sol pela
> > cara!
>
>Sol pelas costas de quem está sendo fotografado... Pensei que tinha
>ficado claro...
>
> > Cesar, vamos deixar as coisas assim, ok? M. SP 17/fev
>
>Não dá. Sou muito chato, e quando este tipo de coisa cai nas minhas
>fuças, eu vou até o fim...
>
>[]s
>--
>.O. Cesar A. K. Grossmann ICQ UIN: 35659423
>..O http://www.LinuxByGrossmann.cjb.net/
>OOO Timeo Danaos, et dona ferentes. (Virgilio)
>
>
>
>
>
>##### ##### #####
>
>Para saber mais visite
>http://www.ciencialist.hpg.ig.com.br
>
>
>##### ##### ##### #####
>Links do Yahoo! Grupos
>
>
>
>
>
>
>
>




SUBJECT: Re: [ciencialist] Onde NÃO há química?
FROM: "Geraldo M. Barcelos" <gemaba@veloxmail.com.br>
TO: <ciencialist@yahoogrupos.com.br>, <quimica-qaw@yahoogrupos.com.br>, <quimica@grupos.com.br>, <naeq-ucs@yahoogrupos.com.br>
DATE: 19/02/2005 21:42

Olá Emiliano !!!

Parece que minha reclamação ao Oráculo deu resultado e estou recebendo
mensagens da ciencialist novamente,
que bom pois afinal tenho muitas lições ainda a fazer com o Takata, Homero,
etc. hehehe Agradeço ao Brudna
por isso.

Nós humanos temos a capacidade de conceptualizar coisas que não existem em
essencia formal e me parece
ser o vácuo uma delas.

Saudações !!!
gemaba
www.gemaba.cjb.net


----- Original Message -----
From: "Emiliano Chemello - Yahoo Grupos" <chemelloe@yahoo.com.br>
To: <ciencialist@yahoogrupos.com.br>; <quimica-qaw@yahoogrupos.com.br>;
<quimica@grupos.com.br>; <naeq-ucs@yahoogrupos.com.br>
Sent: Saturday, February 19, 2005 12:43 AM
Subject: [ciencialist] Onde NÃO há química?


>
> Amigos (as),
>
> Estava no MSN Messenger quando surgiu uma dúvida de uma menina de 15
> anos (primeiro ano do ensino médio). Ela me perguntou:
>
> "onde *não* há química?".
>
> Ponderei antes de formular uma resposta. Sendo bastante direto, até
> cometendo alguns equívocos propositais, consegui reduzir a 'genérica'
> pergunta nestas frases.
>
> "A química estuda as transformações da matéria. Se não há matéria, não há
> transformação e, por consequência, não há química. A grande questão da
> ciência (não só da química) é: será que existe o 'vácuo' da forma como o
> concebemos?"
>
> Alguém concorda? Alguém discorda? Alguém não está nem ai para isso? :-)
>
> 1 mol de [ ]'s do
> Emiliano Chemello
> emiliano@quimica.net
> http://www.quimica.net/emiliano
> http://www.ucs.br/ccet/defq/naeq
> [ MSN ] chemelloe@hotmail.com
> [ ICQ ] 145060604
>
> "Rien ne se perd, rien ne se crée, tout se transforme"
> Lavoisier, químico francês (1743-1794)
>
>
>
>
> ##### ##### #####
>
> Para saber mais visite
> http://www.ciencialist.hpg.ig.com.br
>
>
> ##### ##### ##### #####
> Links do Yahoo! Grupos
>
>
>
>
>
>
>
>
>



SUBJECT: Re: [ciencialist] Re: Onde NÃO há química?
FROM: "Emiliano Chemello - Yahoo Grupos" <chemelloe@yahoo.com.br>
TO: <ciencialist@yahoogrupos.com.br>
DATE: 19/02/2005 23:04

Olá Manuel,

Vamos ao Aurélio:
"Vácuo": Sm. Espaço, imaginário ou real, não ocupado por coisa alguma;
lacuna, vão, vazio

Pesquisas dizem que não é bem assim. Evidentemente, estando a moça no
primeiro ano do ensino médio, o conceito de vácuo deve assemelhar-se muito
ao do dicionário e o do senso comum.

Sugiro a leitura do artido da revista ciência hoje sobre o tema: "vácuo
quântico".
http://ich.unito.com.br/materia/resources/files/chmais/pass/ch146/vacuo.pdf

Au revoir

1 mol de [ ]'s do
Emiliano Chemello
emiliano@quimica.net
http://www.quimica.net/emiliano
http://www.ucs.br/ccet/defq/naeq
[ MSN ] chemelloe@hotmail.com
[ ICQ ] 145060604

"Rien ne se perd, rien ne se crée, tout se transforme"
Lavoisier, químico francês (1743-1794)

----- Original Message -----
From: Manuel Bulcão
To: ciencialist@yahoogrupos.com.br
Sent: Saturday, February 19, 2005 5:23 PM
Subject: [ciencialist] Re: Onde NÃO há química?



Oi Emiliano,

> "onde *não* há química?".

Manuel: Dentro do buraco profundo da minha ignorância, eu pensava
que o objeto da química fosse a estrutura, as transformações e as
propriedades macroscópicas das substâncias 'atômicas' simples e
compostas, de modo que, durante aquele período da história do
Universo em que tudo consistia num caldo homogêneo de fótons,
elétrons, neutrinos e hádrons extremamente instáveis, ainda não
existiam substâncias no sentido 'químico' do termo.

Avento que, antes da cisão da força eletrofraca em eletromagnetismo
e força nuclear fraca, inexistiam quaisquer processos químicos. Após
essa quebra de simetria, surge uma proto-química na medida em que, a
partir de então, aparecem radionuclídeos cuja vida dura um átimo.

Processos químicos propriamente ditos só advieram, penso eu, depois
que os fótons se separaram da matéria (fenômeno que deu origem à
radiação cósmica de fundo), permitindo a combinação dos elétrons com
os núcleos e, por conseguinte a formação de átomos estáveis.

Também não creio que, no núcleo de um buraco negro, ocorram
processos químicos.

Ainda se pode imaginar uma fase futura da história do Universo em
expansão (daqui a muitos, muitíssimos `eões') em que não haverá um
único buraco negro, sequer um bárion real, nenhum positrônio que não
seja virtual, pois todos eles há muito teriam evaporado, decaído ou
se desintegrado. Nesta remotíssima `era', restaria uma sopa
extremamente rala de fótons, neutrinos, uns poucos elétrons isolados
boiando e, ipso facto, total ausência de processos químicos.

Penso que a lógica vem antes da física, a física antecede a química,
a química precede a biologia, a biologia é anterior à consciência, a
consciência dá origem à sensação de absurdo (`Angst der Kreatur'),
da sensação de absurdo surge a idéia de deus e que este, talvez,
seja a própria lógica que está no começo da história.

Abraços,
Manuel Bulcão
PS.: Desculpem-me se falei besteira demais. :-)





##### ##### #####

Para saber mais visite
http://www.ciencialist.hpg.ig.com.br


##### ##### ##### #####


Yahoo! Grupos, um serviço oferecido por:








Links do Yahoo! Grupos

Para visitar o site do seu grupo na web, acesse:
http://br.groups.yahoo.com/group/ciencialist/

Para sair deste grupo, envie um e-mail para:
ciencialist-unsubscribe@yahoogrupos.com.br

O uso que você faz do Yahoo! Grupos está sujeito aos Termos do Serviço do
Yahoo!.




SUBJECT: Re: Onde NÃO há química?
FROM: "rmtakata" <rmtakata@altavista.net>
TO: ciencialist@yahoogrupos.com.br
DATE: 19/02/2005 23:05


--- Em ciencialist@yahoogrupos.com.br, "Emiliano Chemello -
> Vamos ao Aurélio:
> "Vácuo": Sm. Espaço, imaginário ou real, não ocupado por
> coisa alguma; lacuna, vão, vazio
>
> Pesquisas dizem que não é bem assim.

Eh q. o q. um chama de vacuo nao eh o q o outro chama de vacuo.

De todo modo, um exemplo q. considero acabado do perigo de seu usar os
dicionarios sem cuidados como fonte autorizada - em especial de
dicionarios genericos sobre assuntos espeficos:

============
Aurélio 2a. edição. (Ferreira, 1986)
Dinossauro. [Do gr. deinós, 'terrível' + -sauro.] S. m. Espécie fóssil
de réptil marinho da era mesozóica, de dimensões gigantescas.
==============

No Aurelio Sec. 21 finalmente arrumaram essa barbeiragem q. vem de
decadas. Donde tiraram q. dinossauros eram marinhos nao faco a menor
ideia (confundiram com plessiosauros, ictiossauros e mesossauros?.

[]s,

Roberto Takata





SUBJECT: Re: [ciencialist] Re: Onde NÃO há química?
FROM: "Alberto Mesquita Filho" <albmesq@uol.com.br>
TO: <ciencialist@yahoogrupos.com.br>
DATE: 20/02/2005 00:51

----- Original Message -----
From: "Emiliano Chemello - Yahoo Grupos"
Sent: Saturday, February 19, 2005 10:04 PM
Subject: Re: [ciencialist] Re: Onde NÃO há química?

> Vamos ao Aurélio:
> "Vácuo": Sm. Espaço, imaginário ou real, não ocupado por coisa alguma;
> lacuna, vão, vazio
> Pesquisas dizem que não é bem assim. Evidentemente, estando a moça no
> primeiro ano do ensino médio, o conceito de vácuo deve assemelhar-se muito
> ao do dicionário e o do senso comum.

Bem, se a moça está no primeiro ano do ensino médio, é de se esperar que
ainda não tenha passado pela lavagem cerebral produzida nos institutos de
física. Essa é a regra, embora eu já tenha constatado exceções. Supondo
então que a moça se enquadre na regra, vamos raciocinar classicamente,
deixando a quântica para os fanáticos quânticos.

Em primeiro lugar, pensemos no que diz o dicionário. Não há problema algum
em "imaginarmos" um espaço não ocupado por coisa alguma, logo o dicionário
não está cometendo nenhum absurdo clássico. Como o dicionário diz
"imaginário ou real", poderíamos sim contestá-lo. Afinal a física clássica
não só não comprovou a existência de um espaço não ocupado por nada como
também, se pensarmos sob o ponto de vista newtoniano, poderemos garantir que
essa idealidade não existe a não ser como produto da imaginação.

Classicamente sabe-se que a matéria com seu centro de massa em repouso gera
um campo [principalmente dos tipos gravitacional (massa) ou eletromagnético
(cargas)]. Sabe-se também que esse campo tem extensão infinita e que a sua
intensidade tende a zero, mas rigorosamente falando jamais se anula, a não
ser numa região situada num hipotético infinito. Logo, onde quer que
estejamos num universo hipotético dotado de apenas um objeto material,
certamente estaremos sob a ação do campo desse objeto. Conseqüentemente,
neste universo não há como se falar classicamente em espaço real não ocupado
por coisa alguma. Se raciocinarmos segundo Newton, poderemos dizer que
existem espaços livres de matéria, mas não livres de campo. Ora, o campo é
alguma coisa, mesmo raciocinando-se classicamente, queiram os "físicos
modernos" ou não. É bem verdade que muitos deles gostam de avacalhar com a
física clássica, mas isso justifica-se, pois eles não a conhecem
devidamente, logo podemos perdoá-los, pois eles não sabem o que falam.

> Sugiro a leitura do artido da revista ciência hoje sobre o tema: "vácuo
> quântico".
> http://ich.unito.com.br/materia/resources/files/chmais/pass/ch146/vacuo.pdf

Eu diria que a vaca quântica está indo para o brejo e sem retorno, logo
sugiro a leitura do artigo "O movimento absoluto e a física de Newton" que
está no Espaço Científico Cultural em
http://ecientificocultural.com/ECC2/artigos/movab00.htm

> Au revoir

Pra você também. ;-)

[ ]´s
Alberto
http://ecientificocultural.com/indice.htm
Mas indiferentemente a tudo isso, o neutrino tem massa, o elétron não é
uma carga elétrica coulombiana e a Terra se move. E a história se repetirá.



SUBJECT: Re: Onde NÃO há química?
FROM: Manuel Bulcão <manuelbulcao@uol.com.br>
TO: ciencialist@yahoogrupos.com.br
DATE: 20/02/2005 02:06


Olá Emiliano,

Emiliano: Pesquisas dizem que não é bem assim. Evidentemente,
estando a moça no primeiro ano do ensino médio, o conceito de vácuo
deve assemelhar-se muito ao do dicionário e o do senso comum.

Sugiro a leitura do artido da revista ciência hoje sobre o
tema: "vácuo quântico".

Manuel: Na minha mensagem anterior, há o seguinte trecho <<Ainda se
pode imaginar uma fase futura da história do Universo em expansão
(daqui a muitos, muitíssimos `eões') em que não haverá um único
buraco negro, sequer um bárion REAL, nenhum positrônio que não seja
VIRTUAL...>>

Significa que eu estava pressupondo a existência de elétrons,
posítrons e até mesmo bárions 'virtuais', ou seja, que o vácuo
é 'quântico' e não 'clássico'.

Ocorre que as partículas virtuais do vácuo quântico, ao contrário
das partículas 'reais', não interagem com coisa alguma a não ser com
o seu par de sinal oposto no instante em que surgem e no momento em
que se aniquilam mutuamente. Ou seja, sua breve existência
absolutamente dependente do princípio da indeterminação extingue-se
antes que elas venham a interagir seja com os instrumentos de medida
de um observador (por isso que essas partículas são 'inobserváveis'
e, portanto, 'virtuais') ou com quaisquer outros corpúsculos com os
quais possam formar elementos 'químicos', tais como radionuclídeos e
átomos estáveis.

Para que haja química, é necessário que existam partículas 'reais'
que 'duram' e que, interagindo com outras partículas 'reais',
integram estruturas atômicas. Sem isso, não há reações químicas de
qualquer tipo -- nem mesmo químico-nucleares --, mas apenas
processos físicos elementares.

A propósito, transcrevo aqui um texto de Murray Gell-Mann: "As
condições de baixa energia sob as quais as questões químicas surgem
não são universais. No centro do Sol, a uma temperatura de dezenas
de milhões de graus, os átomos de hidrogênio são decompostos em seus
elétrons e prótons constituintes. Lá, nem átomos nem moléculas têm
probabilidade significativa de estar presentes. Não há, por assim
dizer, química no centro do Sol." (O Quark e o Jaguar; Rocco; Rio de
Janeiro; 1996; pág. 126)

Abraços,
Manuel Bulcão





SUBJECT: refrigerantes...
FROM: "Emiliano Chemello - Yahoo Grupos" <chemelloe@yahoo.com.br>
TO: <ciencialist@yahoogrupos.com.br>, <naeq-ucs@yahoogrupos.com.br>, <quimica-qaw@yahoogrupos.com.br>
DATE: 20/02/2005 13:03

alguma referência sobre isso?

Amplexos

Emiliano Chemello
---
Contato Naeq:
Nome: Elizeu Batista
Email: zzu2300@hotmail.com.br
Assunto: Coca cola
Mensagem: Gostaria saber quais os efeitos dos refrigerantes no nosso
organismo, e em especial a coca cola. De todos que existem, há um pior ?
Sempre tenho ouvido bastante coisa, mas muitas vezes sem fundamento
científico . Obrigado




SUBJECT: Re: Onde NÃO há química?
FROM: "Emiliano Chemello - Yahoo Grupos" <chemelloe@yahoo.com.br>
TO: <ciencialist@yahoogrupos.com.br>, <quimica-qaw@yahoogrupos.com.br>, <quimica@grupos.com.br>, <naeq-ucs@yahoogrupos.com.br>
DATE: 20/02/2005 14:21

Olá Manuel,

Creio que, novamente, você matou a charada. O conceito de vácuo sendo um
local onde não há coisa alguma é muito improvável. Certas observações (vide
artigo que fiz referência na msg anterior) dizem que existem partículas
ditas 'virtuais', as quais o estudo pertence a Física (eu diria coisa de
Físico, eheehe). Moléculas, nem mesmo átomos estão presentes no vácuo.
Portanto, no vácuo não há química, mesmo num possível 'vácuo quântico'. A
frase o Murray Gell-Mann resume todo o pensamento. "Núcleo do Sol..." um
ótimo exemplo. Reescrevendo a frase que enviei a moça, diria:

Pergunta:
"onde não há química?"

Resposta:
"A química estuda a estrutura e as transformações da matéria. Porém, sua
ação não abrange a todo o tipo de matéria, mas sim a um nível mínimo de
agregação específico, ou seja, os átomos (admitindo a exceção do estudo da
radioativiade, a qual analisa partículas como neutrinos, pósitons, núcleos
de hélio e outras estruturas subatômicas emitidas por certos núcleos
instáveis). Um nível máximo de agregarção seriam as macromoléculas, as quais
são agregados numerosos de átomos. Portanto, procurar um local onde não há
química é uma tarefa um tanto complicada.

Um local onde não há química seria o vácuo. Estudos recentes conjecturam
que o vácuo não é destituido de qualquer espécie de matéria. Presume-se que
existam partículas ditas 'virtuais' que não tem energia necessária para
interagir com a matéria dita 'real', ou seja, suponhe-se a existência de um
vácuo quântico. Portanto, se não há estruturas significativas que estejam no
escopo de estudo da química, não há transformações da matéria e, por
consequência, não há química.".

Outro local onde se pode dizer que não a química é o núcleo do Sol, onde
a matéria está num grau de dissociação tão grande que conjectura-se não
existir estruturas como átomos e moléculas. Inclusive, o estado físico da
matéria no núcleo do sol é o 'plasma'."

Sei, sei... talvez o texto esteja um pouco pesado para quem está no
primeiro ano do ensino médio. Mas quem mandou o professor fazer uma pergunta
tão capciosa? :-)

Amplexos do

Emiliano Chemello
emiliano@quimica.net
http://www.quimica.net/emiliano
http://www.ucs.br/ccet/defq/naeq
[ MSN ] chemelloe@hotmail.com
[ ICQ ] 145060604

"Rien ne se perd, rien ne se crée, tout se transforme"
Lavoisier, químico francês (1743-1794)

----- Original Message -----
From: Manuel Bulcão
To: ciencialist@yahoogrupos.com.br
Sent: Sunday, February 20, 2005 2:06 AM
Subject: [ciencialist] Re: Onde NÃO há química?



Olá Emiliano,

Emiliano: Pesquisas dizem que não é bem assim. Evidentemente,
estando a moça no primeiro ano do ensino médio, o conceito de vácuo
deve assemelhar-se muito ao do dicionário e o do senso comum.

Sugiro a leitura do artido da revista ciência hoje sobre o
tema: "vácuo quântico".

Manuel: Na minha mensagem anterior, há o seguinte trecho <<Ainda se
pode imaginar uma fase futura da história do Universo em expansão
(daqui a muitos, muitíssimos `eões') em que não haverá um único
buraco negro, sequer um bárion REAL, nenhum positrônio que não seja
VIRTUAL...>>

Significa que eu estava pressupondo a existência de elétrons,
posítrons e até mesmo bárions 'virtuais', ou seja, que o vácuo
é 'quântico' e não 'clássico'.

Ocorre que as partículas virtuais do vácuo quântico, ao contrário
das partículas 'reais', não interagem com coisa alguma a não ser com
o seu par de sinal oposto no instante em que surgem e no momento em
que se aniquilam mutuamente. Ou seja, sua breve existência
absolutamente dependente do princípio da indeterminação extingue-se
antes que elas venham a interagir seja com os instrumentos de medida
de um observador (por isso que essas partículas são 'inobserváveis'
e, portanto, 'virtuais') ou com quaisquer outros corpúsculos com os
quais possam formar elementos 'químicos', tais como radionuclídeos e
átomos estáveis.

Para que haja química, é necessário que existam partículas 'reais'
que 'duram' e que, interagindo com outras partículas 'reais',
integram estruturas atômicas. Sem isso, não há reações químicas de
qualquer tipo -- nem mesmo químico-nucleares --, mas apenas
processos físicos elementares.

A propósito, transcrevo aqui um texto de Murray Gell-Mann: "As
condições de baixa energia sob as quais as questões químicas surgem
não são universais. No centro do Sol, a uma temperatura de dezenas
de milhões de graus, os átomos de hidrogênio são decompostos em seus
elétrons e prótons constituintes. Lá, nem átomos nem moléculas têm
probabilidade significativa de estar presentes. Não há, por assim
dizer, química no centro do Sol." (O Quark e o Jaguar; Rocco; Rio de
Janeiro; 1996; pág. 126)

Abraços,
Manuel Bulcão





SUBJECT: Re: Onde NÃO há química?
FROM: Manuel Bulcão <manuelbulcao@uol.com.br>
TO: ciencialist@yahoogrupos.com.br
DATE: 20/02/2005 14:35


Olá Emiliano,

Seu texto está excelente! :-)

Abraços,
Manuel Bulcão
___________

Pergunta:
"onde não há química?"

Resposta:
"A química estuda a estrutura e as transformações da matéria. Porém,
sua ação não abrange a todo o tipo de matéria, mas sim a um nível
mínimo de agregação específico, ou seja, os átomos (admitindo a
exceção do estudo da radioativiade, a qual analisa partículas como
neutrinos, pósitons, núcleos de hélio e outras estruturas
subatômicas emitidas por certos núcleos instáveis). Um nível máximo
de agregarção seriam as macromoléculas, as quais são agregados
numerosos de átomos. Portanto, procurar um local onde não há química
é uma tarefa um tanto complicada.

Um local onde não há química seria o vácuo. Estudos recentes
conjecturam que o vácuo não é destituido de qualquer espécie de
matéria. Presume-se que existam partículas ditas 'virtuais' que não
tem energia necessária para interagir com a matéria dita 'real', ou
seja, suponhe-se a existência de um vácuo quântico. Portanto, se não
há estruturas significativas que estejam no escopo de estudo da
química, não há transformações da matéria e, por consequência, não
há química.".

Outro local onde se pode dizer que não há química é o núcleo do Sol,
onde a matéria está num grau de dissociação tão grande que
conjectura-se não existir estruturas como átomos e moléculas.
Inclusive, o estado físico da matéria no núcleo do sol é o 'plasma'."





SUBJECT: Re: Onde NÃO há química?
FROM: Hélio Ricardo Carvalho <hrc@fis.puc-rio.br>
TO: ciencialist@yahoogrupos.com.br
DATE: 21/02/2005 10:14


Manuel,

Você escreveu:
> ...
> Significa que eu estava pressupondo a existência de elétrons,
> posítrons e até mesmo bárions 'virtuais', ou seja, que o vácuo
> é 'quântico' e não 'clássico'.
>
> Ocorre que as partículas virtuais do vácuo quântico, ao contrário
> das partículas 'reais', não interagem com coisa alguma a não ser
com
> o seu par de sinal oposto no instante em que surgem e no momento
em
> que se aniquilam mutuamente. Ou seja, sua breve existência
> absolutamente dependente do princípio da indeterminação
extingue-se
> antes que elas venham a interagir seja com os instrumentos de
medida
> de um observador (por isso que essas partículas são
'inobserváveis'
> e, portanto, 'virtuais') ou com quaisquer outros corpúsculos com
os
> quais possam formar elementos 'químicos', tais como radionuclídeos
e
> átomos estáveis.
> ...

Na minha garagem tenho um ectoplasma mas você não vai poder vê-lo
pois é composto de partículas 'inobserváveis' para um não iniciado.

:-) :-) :-)

[ ]'s
Hélio






SUBJECT: Olá, novo a lista
FROM: JVictor <jvoneto@uol.com.br>
TO: ciencialist@yahoogrupos.com.br
DATE: 21/02/2005 12:16

Olá, novo na lista.

Só testando, envio.

Miro


SUBJECT: << Teste nuclear >>
FROM: "Esteban Moreno" <estebanmoreno@idhi.org.br>
TO: <ciencialist@yahoogrupos.com.br>, "Vivens Scientia" <VivensScientia@yahoogroups.com>
DATE: 21/02/2005 13:26


20 segundos de vídeo contendo uma pequena compilação de um teste nuclear.
Obviamente a bomba foi detonada abaixo do solo.

1,37Mb

http://pya.cc/pyaimg/img200502/2005021215261BA.wmv




SUBJECT: Re: Onde NÃO há química?
FROM: Manuel Bulcão <manuelbulcao@uol.com.br>
TO: ciencialist@yahoogrupos.com.br
DATE: 21/02/2005 13:43


Olá Hélio,

Hélio: Na minha garagem tenho um ectoplasma mas você não vai poder
vê-lo pois é composto de partículas 'inobserváveis' para um não
iniciado.

Manuel: Uma partícula virtual considerada isoladamente não pode ser
observada devido ao veto do princípio da incerteza. No entanto, pode-
se perfeitamente observar as conseqüências da totalidade do mar de
partículas virtuais. A atividade do vácuo quântico (um fervilhar de
pares virtuais) tem um efeito sobre os níveis de energia dos átomos.
Em 1953, Willis Lamb mensurou esse efeito no átomo de hidrogênio,
fenômeno este que ficou conhecido como "desvio de Lamb". Por este
trabalho Lamb foi agraciado com o Prêmio Nobel.

Em suma, os pares de partículas individuais não podem ser
observados "diretamente". Sua existência, porém, pode ser inferida
do estado excitado de energia dos átomos.

Abraços,
Manuel Bulcão






SUBJECT: vírus no celular
FROM: "Sandra Rosario" <coffeacruda@hotmail.com>
TO: ciencialist@yahoogrupos.com.br
DATE: 21/02/2005 16:11

Como isso é possível?
Sandra


21/02/2005 - 11h38m
Primeiro vírus para celular do mundo chega aos EUA

Reuters

SÃO FRANCISCO - O primeiro vírus de celular do mundo chegou aos Estados
Unidos oito meses após ser descoberto em seu local de origem, as Filipinas,
segundo informou Mikko Hypponen, diretor da empresa finlandesa de software
para segurança de computadores, F-Secure.

O vírus, conhecido como Cabir, se difundiu lentamente por 12 países e marca
o começo da era dos vírus de telefonia móvel, que um dia podem vir a
perturbar
a vida de boa parte dos usuários de celulares no planeta, que somam cerca
de 1,5 bilhão.

O maior impacto do vírus relativamente inócuo, do qual até agora foram
descobertas
cerca de 15 variantes, é causar o esgotamento das baterias dos celulares,
disse o dirigente. Ainda de acordo com ele, o Cabir foi localizado nesta
segunda-feira, em uma loja de aparelhos eletrônicos em Santa Monica,
Califórnia,
quando um visitante percebeu um sinal característico na tela de um dos
celulares
exposto na loja.

O perigo não é grande no momento, em parte devido à gama de tecnologias
diferentes de comunicação móvel em uso. Não existe no mundo dos celulares
uma situação como a da computação pessoal, em que o sistema operacional
Windows, da Microsoft claramente domina o mercado. Além disso, muitos dos
fabricantes de aparelhos de mão recentemente lançaram modelos novos
equipados
com software antivírus.

_________________________________________________________________
Chegou o que faltava: MSN Acesso Grátis. Instale Já!
http://www.msn.com.br/discador



SUBJECT: Re: vírus no celular
FROM: "rmtakata" <rmtakata@altavista.net>
TO: ciencialist@yahoogrupos.com.br
DATE: 21/02/2005 16:24


--- Em ciencialist@yahoogrupos.com.br, "Sandra Rosario"
> Como isso é possível?
> Sandra

Os telefones celulares 'inteligentes' funcionam a base de um programa
q. gerencia seu hardware. Brechas na seguranca permitem a invasao de
virus e outros comandos mal-intencionados como em qq computador -
celulares modernos sao pequenos computadores especializados em
telefonia (e tb jogos eletronicos, musicas, navegacao em internet,
televisao, fotografia, filmagem...)

Vírus informáticos podem atacar tb computadores de bordo de carros:

http://www1.folha.uol.com.br/folha/informatica/ult124u17904.shtml

Sabe aquele sonho da casa automatica controlada via internet? Pois eh,
haverah o risco de um hacker invadir a central e mudar a senha para a
entrada e acionamento dos eletrodomesticos...

[]s,

Roberto Takata





SUBJECT: Re: [ciencialist] vírus no celular
FROM: "Oraculo" <oraculo@atibaia.com.br>
TO: <ciencialist@yahoogrupos.com.br>
DATE: 21/02/2005 16:36

Olá Sandra

Um virus de computador, apesar da fama de algo "misterioso" e "vivo" que ele tem (graças a mídia, que adora manchetes explosivas..:-), é apenas um programa de computador como qualquer outro, como seu Word, Outlook, jogos ou qualquer coisa que passe instruções para o processador de seu computador.

O que o diferencia é que seu criador, um programador como qualquer outro (com um tremendo espírito de porco..:-) determina suas ações sem consultar o usuário. E, em geral, com intenções e ações daninhas.

Como os celulares agora também possuem pequenos processadores e rodam (cumprem instruções codificadas) software, virus, instruções daninhas, e sem o conhecimento do usuário, podem infecta-los.

Veja um exemplo de tarefas padrão que você permite que seu PC execute: você verifica a data no relógio do sistema (o que indica que o PC sabe a data e hora), se for sexta feira, você abre o gerenciador de arquivos e encontra os documentos dessa semana (o que indica que existem, já em seu sistema, instruções que permitem encontrar arquivos), e escolhe os que não tem mais relevância e os apaga (o que mostra que apagar arquivos é parte integrante da capacidade de um PC). Depois, copia e cola os arquivos importantes em um disquete ou os envia por email para quem desejar.

Em cada uma dessas ações, é sua vontade que age e, nos casos onde existe algum risco, o sistema cuidadosamente vai perguntar "deseja mesmo deletar este arquivo?". Mas fará isso apenas porque o programador original desses softwares assim determinou.

Agora, imagine um programador que junta todas essa capacidades em um único arquivo, bem pequeno. E que elimina as perguntas e as condições de segurança.

Seu pequeno programa verifica, a cada dia, a data e hora. Se for determianda data, ele procura (ou acessa aleatoriamente) arquivos no seu HD, deleta os que encontrar (sem perguntar nada), se copia para um disquete ou email, e o envia para os contatos de sua lista. Nada que você mesmo não faria, mas em um formato daninho e incógnito.

O mesmo vai acontecer com qualquer equipamento que seja capaz de cumprir ordens em codigo de um software.

Observe que, como nota a reportagem, os virus de celulares são relativametne inócuos. Isso porque a capacidade de rodar softare, cumprir ordens e praticar ações, é também limitada. Mas com o aumento da capacidade de processamento, deve aumentar a capacidade de causar dano dos virus desse tipo (junto a capacidade de processar funções benéficas, como processar textos, calcular planilhas, enviar emails, etc).

Um abraço.

Homero




----- Original Message -----
From: Sandra Rosario
To: ciencialist@yahoogrupos.com.br
Sent: Monday, February 21, 2005 4:11 PM
Subject: [ciencialist] vírus no celular


Como isso é possível?
Sandra


21/02/2005 - 11h38m
Primeiro vírus para celular do mundo chega aos EUA

Reuters

SÃO FRANCISCO - O primeiro vírus de celular do mundo chegou aos Estados
Unidos oito meses após ser descoberto em seu local de origem, as Filipinas,
segundo informou Mikko Hypponen, diretor da empresa finlandesa de software
para segurança de computadores, F-Secure.

O vírus, conhecido como Cabir, se difundiu lentamente por 12 países e marca
o começo da era dos vírus de telefonia móvel, que um dia podem vir a
perturbar
a vida de boa parte dos usuários de celulares no planeta, que somam cerca
de 1,5 bilhão.

O maior impacto do vírus relativamente inócuo, do qual até agora foram
descobertas
cerca de 15 variantes, é causar o esgotamento das baterias dos celulares,
disse o dirigente. Ainda de acordo com ele, o Cabir foi localizado nesta
segunda-feira, em uma loja de aparelhos eletrônicos em Santa Monica,
Califórnia,
quando um visitante percebeu um sinal característico na tela de um dos
celulares
exposto na loja.

O perigo não é grande no momento, em parte devido à gama de tecnologias
diferentes de comunicação móvel em uso. Não existe no mundo dos celulares
uma situação como a da computação pessoal, em que o sistema operacional
Windows, da Microsoft claramente domina o mercado. Além disso, muitos dos
fabricantes de aparelhos de mão recentemente lançaram modelos novos
equipados
com software antivírus.

_________________________________________________________________
Chegou o que faltava: MSN Acesso Grátis. Instale Já!
http://www.msn.com.br/discador



##### ##### #####

Para saber mais visite
http://www.ciencialist.hpg.ig.com.br


##### ##### ##### #####


Yahoo! Grupos, um serviço oferecido por:







------------------------------------------------------------------------------
Links do Yahoo! Grupos

a.. Para visitar o site do seu grupo na web, acesse:
http://br.groups.yahoo.com/group/ciencialist/

b.. Para sair deste grupo, envie um e-mail para:
ciencialist-unsubscribe@yahoogrupos.com.br

c.. O uso que você faz do Yahoo! Grupos está sujeito aos Termos do Serviço do Yahoo!.



[As partes desta mensagem que não continham texto foram removidas]



SUBJECT: Vácuo quântico (era: Onde NÃO há química?)
FROM: Hélio Ricardo Carvalho <hrc@fis.puc-rio.br>
TO: ciencialist@yahoogrupos.com.br
DATE: 21/02/2005 18:05


Olá Manuel,

--- Em ciencialist@yahoogrupos.com.br, Manuel Bulcão
<manuelbulcao@u...> escreveu
>
> Olá Hélio,
>
> Hélio: Na minha garagem tenho um ectoplasma mas você não vai poder
> vê-lo pois é composto de partículas 'inobserváveis' para um não
> iniciado.
>
> Manuel: Uma partícula virtual considerada isoladamente não pode
ser
> observada devido ao veto do princípio da incerteza. No entanto,
pode-
> se perfeitamente observar as conseqüências da totalidade do mar de
> partículas virtuais. A atividade do vácuo quântico (um fervilhar
de
> pares virtuais) tem um efeito sobre os níveis de energia dos
átomos.
> Em 1953, Willis Lamb mensurou esse efeito no átomo de hidrogênio,
> fenômeno este que ficou conhecido como "desvio de Lamb". Por este
> trabalho Lamb foi agraciado com o Prêmio Nobel.
>
> Em suma, os pares de partículas individuais não podem ser
> observados "diretamente". Sua existência, porém, pode ser inferida
> do estado excitado de energia dos átomos.
>

Hélio:
Eu não consigo observar diretamente o ectoplasma da minha garagem
mas o fato de lá o meu pão cair "sempre"(~50%) com a manteiga para
baixo já é um bom indicativo que ele existe.

:-) :-) :-)

Falando sério.
Eu ainda não conheço detalhes da hipótese [:-)] do vácuo quântico.
Sei que a física procura (ou procurava) explicações para a
instabilidade dos estados excitados dos átomos.

Algumas perguntas: (para todos)

Qual a ligação do princípio da incerteza com o desvio de Lamb?

Ele é do tipo tempo/energia ou posição/momento?

Quando se fala de pares de partículas virtuais, estamos falando de
partículas e anti-partículas?

Na frase "A atividade do vácuo quântico tem um efeito sobre os
níveis de energia dos átomos" não significa uma interação?

Última pergunta:
Entre duas partículas virtuais do mesmo par ou entre dois pares
destas partículas que estão existindo neste instante, o que existe?
"VÁCUO CLÁSSICO" ??????????? :-) :-) :-)

[ ]'s
Hélio





SUBJECT: Re: [ciencialist] Re: Onde NÃO há química?
FROM: "Alberto Mesquita Filho" <albmesq@uol.com.br>
TO: <ciencialist@yahoogrupos.com.br>
DATE: 22/02/2005 00:41

----- Original Message -----
From: "Manuel Bulcão"
Sent: Monday, February 21, 2005 1:43 PM
Subject: [ciencialist] Re: Onde NÃO há química?

> Manuel: A atividade do vácuo quântico (um fervilhar de pares virtuais) tem
> um efeito sobre os níveis de energia dos átomos. Em 1953, Willis Lamb
> mensurou esse efeito no átomo de hidrogênio, fenômeno este que ficou
> conhecido como "desvio de Lamb". Por este trabalho Lamb foi agraciado com
> o Prêmio Nobel.

De fato isso aconteceu e está resumidamente relatado na "Nobel Lecture" de
Willis E. Lamb Jr que pode ser lida a partir de
http://nobelprize.org/physics/laureates/1955/lamb-lecture.pdf

É interessante perceber a habilidade de Lamb em conseguir contornar um fato
na época constatado experimentalmente, aquele de que o elétron não se
sujeitava à lei de Coulomb. Aliás, mutreta do mesmo gênero já havia sido
feita, em outro contexto, por Bohr em 1913, através de suas órbitas
permitidas, quais sejam, aquelas órbitas em que os elétrons estavam
autorizados, ainda hoje não se sabe por quem, a desobedecerem a teoria
eletromagnética de Maxwell-Lorentz.

Essas manobras, via de regra "ad hoc", locupletam toda a física moderna,
aquela que consegue prever com uma precisão incrível o número de fantasmas
que habitam um castelo que os físicos quânticos acreditam existir numa das
luas de Júpiter.

É interessante notar que Lamb demorou a se dar conta dessas conjecturas
estratosféricas que levaram-no à Suécia. Em seus primeiros trabalhos ele
quase chegou a perceber a existência de partículas que não obedeciam a lei
de Coulomb. Isso está escrito no trabalho acima indicado. Num dos parágrafos
ele chega a contestar a teoria de Dirac, outro dos gigantes da física
moderna e que teria caído também na mesma falácia de confundir um elétron
com uma carga elétrica em miniatura. Digno de nota é o seguinte texto de
Lamb, mostrando seu primeiro trabalho a respeito dessa incompatibilidade com
a lei de Coulomb: "My thesis'dealt with field theories of nucleons which
predicted a very small discrepancy from Coulomb's law about a proton."

Vale a pena ler o artigo e constatar uma verdade que eu adaptei de um outro
contexto, qual seja: "As leis da física moderna são como as salsichas,
melhor não saber como foram feitas."

[ ]´s
Alberto
http://ecientificocultural.com/indice.htm
Mas indiferentemente a tudo isso, o neutrino tem massa, o elétron não é
uma carga elétrica coulombiana e a Terra se move. E a história se repetirá.



SUBJECT: The Fossil Fallacy
FROM: "Oraculo" <oraculo@atibaia.com.br>
TO: <ciencialist@yahoogrupos.com.br>
DATE: 22/02/2005 00:44

The Fossil Fallacy
Creationists' demand for fossils that represent "missing links" reveals a deep misunderstanding of science
By Michael Shermer


Nineteenth-century English social scientist Herbert Spencer made this prescient observation: "Those who cavalierly reject the Theory of Evolution, as not adequately supported by facts, seem quite to forget that their own theory is supported by no facts at all." Well over a century later nothing has changed. When I debate creationists, they present not one fact in favor of creation and instead demand "just one transitional fossil" that proves evolution. When I do offer evidence (for example, Ambulocetus natans, a transitional fossil between ancient land mammals and modern whales), they respond that there are now two gaps in the fossil record.
This is a clever debate retort, but it reveals a profound error that I call the Fossil Fallacy: the belief that a "single fossil"--one bit of data--constitutes proof of a multifarious process or historical sequence. In fact, proof is derived through a convergence of evidence from numerous lines of inquiry--multiple, independent inductions, all of which point to an unmistakable conclusion.

We know evolution happened not because of transitional fossils such as A. natans but because of the convergence of evidence from such diverse fields as geology, paleontology, biogeography, comparative anatomy and physiology, molecular biology, genetics, and many more. No single discovery from any of these fields denotes proof of evolution, but together they reveal that life evolved in a certain sequence by a particular process.
One of the finest compilations of evolutionary data and theory since Charles Darwin's On the Origin of Species is Richard Dawkins's magnum opus, The Ancestor's Tale: A Pilgrimage to the Dawn of Evolution (Houghton Mifflin, 2004)--688 pages of convergent science recounted with literary elegance. Dawkins traces numerous transitional fossils (what he calls "concestors," the last common ancestor shared by a set of species) from Homo sapiens back four billion years to the origin of heredity and the emergence of evolution. No single concestor proves that evolution happened, but together they reveal a majestic story of process over time.

Consider the tale of the dog. With so many breeds of dogs popular for so many thousands of years, one would think there would be an abundance of transitional fossils providing paleontologists with copious data from which to reconstruct their evolutionary ancestry. In fact, according to Jennifer A. Leonard, an evolutionary biologist then at the Smithsonian Institution's National Museum of Natural History, "the fossil record from wolves to dogs is pretty sparse." Then how do we know whence dogs evolved? In the November 22, 2002, Science, Leonard and her colleagues report that mitochondrial DNA (mtDNA) data from early dog remains "strongly support the hypothesis that ancient American and Eurasian domestic dogs share a common origin from Old World gray wolves."
In the same issue, molecular biologist Peter Savolainen of the Royal Institute of Technology in Stockholm and his colleagues note that even though the fossil record is problematic, their study of mtDNA sequence variation among 654 domestic dogs from around the world "points to an origin of the domestic dog in East Asia" about 15,000 years before the present from a single gene pool of wolves.

Finally, anthropologist Brian Hare of Harvard University and his colleagues describe in this same issue the results of a study showing that domestic dogs are more skillful than wolves at using human signals to indicate the location of hidden food. Yet "dogs and wolves do not perform differently in a nonsocial memory task, ruling out the possibility that dogs outperform wolves in all human-guided tasks," they write. Therefore, "dogs' social-communicative skills with humans were acquired during the process of domestication."

No single fossil proves that dogs came from wolves, but archaeological, morphological, genetic and behavioral "fossils" converge to reveal the concestor of all dogs to be the East Asian wolf. The tale of human evolution is divulged in a similar manner (although here we do have an abundance of fossils), as it is for all concestors in the history of life. We know evolution happened because innumerable bits of data from myriad fields of science conjoin to paint a rich portrait of life's pilgrimage.


--------------------------------------------------------------------------------

Michael Shermer is publisher of Skeptic (www.skeptic.com) and author of The Science of Good and Evil.

http://www.scientificamerican.com/article.cfm?chanID=sa006&colID=13&articleID=0003EFE0-D68A-1212-8F3983414B7F0000



[As partes desta mensagem que não continham texto foram removidas]



SUBJECT: Câmera em óculos pode restaurar visão, dizem cientistas
FROM: "Oraculo" <oraculo@atibaia.com.br>
TO: <ciencialist@yahoogrupos.com.br>
DATE: 22/02/2005 00:49

Câmera em óculos pode restaurar visão, dizem cientistas

Uma pequena câmera montada em um óculos e conectada ao nervo óptico pode restaurar a visão de milhares de pessoas que sofrerem de deterioração da retina, afirmaram nesta segunda-feira especialistas europeus.
O avanço médico tem o potencial de ajudar 300 mil europeus cuja visão foi enfraquecida conforme a fina camada de tecido celular que envolve o olho e processa as imagens se deteriora, afirmou o professor belga Claude Veraart, em conferência. "Implantamos o dispositivo em dois pacientes até agora", disse Veraart, da Universidade Católica de Louvain-la-Neuve, próxima a Bruxelas.

Uma câmera montada nos óculos envia imagens para um aparelho eletrônico implantado atrás do olho e estimula o nervo óptico que passa a informação visual ao cérebro.

A tecnologia pode ajudar também pessoas com doenças na retina e outras degenerações, que podem levar à perda dos detalhes das imagens, uma das principais causas da deficiência visual nos Estados Unidos.

Baseado no preço de aparelhos auriculares conhecidos como Cochlear, que também envolve a inserção de componentes na cabeça, a nova tecnologia vai custar cerca de 20 mil euros, afirmou Veraart.

Cinquenta equipes de cientistas ao redor do mundo trabalham em tecnologia similar, mas o projeto belga, que coordena um esforço pan-europeu de pesquisadores na França e Alemanha, obteve os melhores resultados, acrescentou.

A Comissária da Sociedade da Informação, Viviane Reding, afirmou que o dispositivo pode começar a ser vendido no mercado entre 2008 e 2010.

A Comissão Européia, que vai gastar 3,6 bilhões de euros em projetos de tecnologia da informação e comunicação entre 2002 e 2006, garantiu 2,79 milhões de euros em pesquisa de tratamento de deficiência visual.



http://noticias.terra.com.br/ciencia/interna/0,,OI475901-EI298,00.html


[As partes desta mensagem que não continham texto foram removidas]



SUBJECT: Protocolo de Kyoto
FROM: "Camila" <camilasaki@yahoo.com.br>
TO: ciencialist@yahoogrupos.com.br
DATE: 22/02/2005 01:51


Bem comoo assunto está na moda eu gostaria de ouvir a opinião de
algumas pessoas:

*será possível com uma redução de 5,2% dos gases diminuir o efeito
estufa?

*a Terra n poderia estar se aquecendo por estarmos chegando a uma era
d glaciação? onde os niveis d co2 tendem a aumentar.

*o mercado verde (credito de carbono) poderia funcianar, realmente?


espero poder partilhar da opiniao d todos.

[]
Ana Camila





SUBJECT: Re: Vácuo quântico (era: Onde NÃO há química?)
FROM: Manuel Bulcão <manuelbulcao@uol.com.br>
TO: ciencialist@yahoogrupos.com.br
DATE: 22/02/2005 07:07


Olá Hélio,

Hélio: Algumas perguntas: (para todos)

Manuel: Por ora vou tentar responder uma.

Hélio: Quando se fala de pares de partículas virtuais, estamos
falando de partículas e anti-partículas?

Manuel: Sim, elétrons e posítrons 'virtuais', por exemplo. Ocorre
que, no caso dos bósons de gauge -- entre os quais os fótons --, as
antipartículas são as próprias partículas.

[]s
Manuel Bulcão





SUBJECT: Re: Protocolo de Kyoto
FROM: "Tipoalgo" <tipoalgo@bol.com.br>
TO: ciencialist@yahoogrupos.com.br
DATE: 22/02/2005 08:44


Olá Camila,

Este Protocolo é polêmico, mas sob a minha visão é um pouco melhor do
que nada, pelo menos tenta de forma muito tímida atacar o assunto.

Este Mercado de Carbono acho ser pouco feliz! Parece um artifício
para os países industrializados continuarem a emitir grandes
quantidades de carbono com aval internacional, ficando bem na foto.
Mas mesmo assim os EUA estão de fora.

Em http://www2.ipef.br/pipermail/bioenergia-l/ este e outros pontos
relacionados são abordados, acho que vale a pena conferir.

O Eng. Fendel defende um Efeito Geladeira contrário ao Efeito Estufa.

Neste contexto o Brasil está em posição privilegiada, mas a falta de
visão e/ou determinação política pode deixar o bonde passar.

Imitando o Sr Fendel, Bioabraços Sequestrantes!
(Em relação à retirada de carbono da atmosfera, Efeito Geladeira)

Tipoalgo


--- Em ciencialist@yahoogrupos.com.br, "Camila" <camilasaki@y...>
escreveu
>
> Bem comoo assunto está na moda eu gostaria de ouvir a opinião
de
> algumas pessoas:
>
> *será possível com uma redução de 5,2% dos gases diminuir
o efeito
> estufa?
>
> *a Terra n poderia estar se aquecendo por estarmos chegando a uma
era
> d glaciação? onde os niveis d co2 tendem a aumentar.
>
> *o mercado verde (credito de carbono) poderia funcianar, realmente?
>
>
> espero poder partilhar da opiniao d todos.
>
> []
> Ana Camila





SUBJECT: União Européia diz que cupuaçu é nosso
FROM: José Renato <jrma@terra.com.br>
TO: <ciencialist@yahoogrupos.com.br>
DATE: 22/02/2005 10:28

JC e-mail 2710, de 21 de Fevereiro de 2005.

UE reconhece direito do Brasil à marca cupuaçu

Entre os argumentos usados para recuperar o cupuaçu estão o fato de o nome ser um termo de domínio público e de o fruto ser típico de uma árvore da Amazônia

Vivian Oswald escreve de Bruxelas para 'O Globo':

Os japoneses que nos desculpem, mas o cupuaçu é nosso.

Se ainda havia alguma dúvida a esse respeito no mercado europeu, o Escritório de Harmonização para o Mercado Interno, instituição européia responsável por marcas e patentes, encarregou-se de desfazê-la: cancelou o registro da marca cupuaçu que havia sido feito pela empresa japonesa Asahi Foods.

A decisão põe fim a uma disputa de quase dois anos que vinha sendo travada entre o governo brasileiro e a companhia.

A novidade permite que os exportadores brasileiros do cupuaçu ou de doces feitos com a fruta não precisem mais pagar royalties aos japoneses para vender seus produtos na Europa.

Essa é a segunda derrota da Asahi Foods, que ano passado foi obrigada a retirar o pedido de patenteamento do produto feito no Japão.

Açaí e andiroba ainda estão com registros estrangeiros

A vitória do governo brasileiro abre caminho para outras decisões do mesmo gênero na disputa por nomes de frutos, sementes e árvores brasileiras das quais grandes empresas estrangeiras tentam se apropriar para garantir o seu mercado.

Ao registrar a marca, todos os direitos passam a ser do seu dono. Estão nesta situação produtos como o açaí e a andiroba.

'Dá satisfação saber que o princípio está sendo respeitado. É encorajador e coerente com as posturas européias de proteção da denominação de origem. A medida derruba um dos obstáculos às exportações dos pequenos produtores de cupuaçu brasileiros', disse o chefe da Missão do Brasil junto a União Européia, embaixador José Alfredo Graça Lima.

A ação contra a empresa japonesa foi aberta pela missão brasileira no Escritório de Harmonização em abril de 2003, mas a decisão final só saiu semana passada.

A Asahi Foods também terá de pagar as custas do processo. Segundo a instituição européia, ainda cabe recurso para os japoneses, que terão até dois meses para fazê-lo.

Mas a expectativa é que a causa já esteja ganha. Entre os argumentos usados para recuperar o cupuaçu estão o fato de o nome ser um termo de domínio público e de o fruto ser típico de uma árvore da Amazônia.

Para o embaixador Graça Lima, este episódio pode acabar levando as empresas brasileiras a se interessarem por este mercado.

(O Estado de SP, 19/2)
< http://www.jornaldaciencia.org.br/Detalhe.jsp?id=25616 >

[As partes desta mensagem que não continham texto foram removidas]



SUBJECT: New Tack Wins Prisoner's Dilemma
FROM: "Alvaro Augusto \(E\)" <alvaro@electraenergy.com.br>
TO: <ciencialist@yahoogrupos.com.br>
CC: "Clemente Nobrega" <clenobrega@uol.com.br>, "Edvaldo Alves de Santana" <Edvaldo@aneel.gov.br>
DATE: 22/02/2005 11:12

"Tit for tat" foi finalmente derrotado. Detalhes em http://www.wired.com/news/culture/0,1284,65317,00.html

[ ]s

Alvaro Augusto de Almeida
alvaro@electraenergy.com.br


[As partes desta mensagem que não continham texto foram removidas]



SUBJECT: Receita caseira para Injetar EE na Rede Elétrica Comercial
FROM: "Tipoalgo" <tipoalgo@bol.com.br>
TO: ciencialist@yahoogrupos.com.br
DATE: 22/02/2005 11:48


Olá a todos,

Ai vai mais uma receita caseira.
Caso se aventure neste intento, os riscos correm por sua própria
conta, mas os benefícios com certeza não serão apenas seus.
A legislação brasileira atual proíbe tal operação, mas se você quiser
ver o seu relógio de Energia Elétrica rodar ao contrário siga a
receita. É claro que você deve dispor dos ingredientes básicos da
receita!

Com a palavra o Sr Fendel:

"""""""""""""""""""""""""""""""""""""""""
Caro Abdhir

Vou tentar adivinhar qual a parte que vc não entendeu. Seria CC e CA?
Corrente contínua é a que obtemos por exemplo numa pilha, onde termos
definidos o polo positivo (+) e o polo negativo (-).
Corrente alternada é a existente na tomada de tua casa, em que as
polaridades se invertem 60 vezes por segundo (60 Herz) de forma
senoidal.
Senoidal significa que essa inversão ocorre arredondadamente e não
abruptamente.
Existem inversores de onda quadrada (mais baratos) e de ondas suaves
(mais
caros).
Além de terem ondas semelhantes, os inversores para injetar energia
na rede
elétrica, necessitam estar sincronizados.

Relendo a pergunta do Roberto, me veio que a dúvida era sobre injetar
energia proveniente de biogás.
Bem, isso é mais simples e barato.
Parece-me que ele já tem um motor a explosão a gás e um gerador
síncrono, e
falta ligá-lo na rede.
Para ligar o gerador síncrono na rede é necessário fazer o
sincronismo, com
um equipamento manual ou automático.
Se a idéia é funcionar o gerador só ligado na rede, então é melhor
trocar o
gerador síncrono por um assíncrono.
Um gerador assíncrono é qualquer barato motor elétrico de indução.
Basta então ligar o motor a biogás no motor elétrico ligado na rede,
com uma
correia ou acoplamento, de forma que a velocidade deste motor
elétrico seja
maior do que a velocidade síncrona.
Ou seja, se o motor é de 4 polos, sua velocidade síncrona seria 1800
rpm em
60 Hz, e ele trabalha nominalmente como motor com 1750 rpm e
nominalmente
como gerador a 1850 rpm.
A própria rede segura a tensão e a freqüência de geração.
O único problema continuará sendo a parte "legal", que aqui continua
como
impedimento hipócrita e corporativista.

Bioenergéticos abraços em rede
Eng. Thomas Renatus Fendel
www.fendel.com.br
(Divulgação autorizada e desejável)
"É mais fácil injetar energia de qualidade na rede elétrica, do que
injetar
água limpa nas tubulações das concessionárias." - Fendel

"""""""""""""""""""""""""""""""""""""""""

Espero que a receita seja útil de alguma forma a alguém.

E dando um palpite futurista, mais e mais pessoas farão o seu
contador de EE girar ao contrário, e a Lei atual será mudada para
seguir o costume do povo, e acabar com a "hipocrisia e o
corporativismo".

Obrigado Sr Fendel.

Tipoalgo







SUBJECT: vácuo quântico, fótons virtuais, etc.
FROM: "manuelbulcao" <manuelbulcao@uol.com.br>
TO: "ciencialist" <ciencialist@yahoogrupos.com.br>
DATE: 22/02/2005 12:15

Hélio, Mesquista e demais

Achei essas notas sobre vácuo quântico, partículas virtuais, etc.

http://www.fis.ufba.br/dfg/pice/ff/ff-06.htm#apresentacao

Abraços,
Manuel Bulcão

__________________________________________________________________________
Acabe com aquelas janelinhas que pulam na sua tela.
AntiPop-up UOL - É grátis!
http://antipopup.uol.com.br/




SUBJECT: Re: [ciencialist] Receita caseira para Injetar EE na Rede Elétrica Comercial
FROM: "Alvaro Augusto \(E\)" <alvaro@electraenergy.com.br>
TO: <ciencialist@yahoogrupos.com.br>
DATE: 22/02/2005 12:26

Não entendi a parte do "impedimento legal". Não vai ser fácil ser autorizado a injetar energia na rede, mas não há impedimento legal, de maneira alguma. Primeiro, a pessoa interessada deve constituir uma empresa de geração de energia, e requerer junto à Aneel o registro como Produtor Independente de Energia (PIE), por meio da exploração de uma usina qualquer. O interessado deve apresentar o projeto básico do sistema de geração, com sistema de medição de energia aprovado pelo ONS. Depois de outorgada a concessão (ou autorização, no caso de usinas térmicas), o interessado deve entrar em contato com a concessionária local e solicitar a conexão em paralelo com o sistema. A concessionária vai exigir uma dúzia de equipamentos de proteção, que evitem, por exemplo, que a rede seja religada em cima do gerador (o texto do nosso ilustre colega não esclarece, por exemplo, que, em caso de desligamentos da rede, mesmo que momentâneos, o gerador pode ser danificado quando do religamento, pois terá acelarado para mais de 60 Hz e será subitamente frenado pela rede).

O processo todo pode demorar uns dois anos, sem contar o prazo para modelagem do gerador junto à CCEE (Câmara de Comercialização de Energia Elétrica), fica muito mais viável com a assessoria de uma comercializadora de energia (podemos enviar proposta, se for o caso).

Novamente, insisto, não há restrição legal alguma quanto à figura do Produtor Independente de Energia, que foi criada pela Lei 9074/1995 ("Lei das Concessões"). Com uma boa negociação junto à concessionária, também não há restrição técnica, pois as distribuidoras são obrigadas a conceder o livre acesso a geradores legalmente constituídos. A única restrição será econômica. Para quem você vai vender a energia? Por que preço? Por que alguém compraria sua energia a R$ 120/MWh quando pode comprar no mercado livre a R$ 50/MWh? A única possibilidade seria conseguir autorização para o desconto de 50% da Tarifa de Uso do Sistema de Distribuição, o que permitiria vender a energia diretamente para consumidores do subgrupo A4, a preços mais atraentes. Mas isso fica para outro dia...

[ ]s

Alvaro Augusto



----- Original Message -----
From: Tipoalgo
To: ciencialist@yahoogrupos.com.br
Sent: Tuesday, February 22, 2005 11:48 AM
Subject: [ciencialist] Receita caseira para Injetar EE na Rede Elétrica Comercial



Olá a todos,

Ai vai mais uma receita caseira.
Caso se aventure neste intento, os riscos correm por sua própria
conta, mas os benefícios com certeza não serão apenas seus.
A legislação brasileira atual proíbe tal operação, mas se você quiser
ver o seu relógio de Energia Elétrica rodar ao contrário siga a
receita. É claro que você deve dispor dos ingredientes básicos da
receita!

Com a palavra o Sr Fendel:

"""""""""""""""""""""""""""""""""""""""""
Caro Abdhir

Vou tentar adivinhar qual a parte que vc não entendeu. Seria CC e CA?
Corrente contínua é a que obtemos por exemplo numa pilha, onde termos
definidos o polo positivo (+) e o polo negativo (-).
Corrente alternada é a existente na tomada de tua casa, em que as
polaridades se invertem 60 vezes por segundo (60 Herz) de forma
senoidal.
Senoidal significa que essa inversão ocorre arredondadamente e não
abruptamente.
Existem inversores de onda quadrada (mais baratos) e de ondas suaves
(mais
caros).
Além de terem ondas semelhantes, os inversores para injetar energia
na rede
elétrica, necessitam estar sincronizados.

Relendo a pergunta do Roberto, me veio que a dúvida era sobre injetar
energia proveniente de biogás.
Bem, isso é mais simples e barato.
Parece-me que ele já tem um motor a explosão a gás e um gerador
síncrono, e
falta ligá-lo na rede.
Para ligar o gerador síncrono na rede é necessário fazer o
sincronismo, com
um equipamento manual ou automático.
Se a idéia é funcionar o gerador só ligado na rede, então é melhor
trocar o
gerador síncrono por um assíncrono.
Um gerador assíncrono é qualquer barato motor elétrico de indução.
Basta então ligar o motor a biogás no motor elétrico ligado na rede,
com uma
correia ou acoplamento, de forma que a velocidade deste motor
elétrico seja
maior do que a velocidade síncrona.
Ou seja, se o motor é de 4 polos, sua velocidade síncrona seria 1800
rpm em
60 Hz, e ele trabalha nominalmente como motor com 1750 rpm e
nominalmente
como gerador a 1850 rpm.
A própria rede segura a tensão e a freqüência de geração.
O único problema continuará sendo a parte "legal", que aqui continua
como
impedimento hipócrita e corporativista.

Bioenergéticos abraços em rede
Eng. Thomas Renatus Fendel
www.fendel.com.br
(Divulgação autorizada e desejável)
"É mais fácil injetar energia de qualidade na rede elétrica, do que
injetar
água limpa nas tubulações das concessionárias." - Fendel

"""""""""""""""""""""""""""""""""""""""""

Espero que a receita seja útil de alguma forma a alguém.

E dando um palpite futurista, mais e mais pessoas farão o seu
contador de EE girar ao contrário, e a Lei atual será mudada para
seguir o costume do povo, e acabar com a "hipocrisia e o
corporativismo".

Obrigado Sr Fendel.

Tipoalgo







##### ##### #####

Para saber mais visite
http://www.ciencialist.hpg.ig.com.br


##### ##### ##### #####


Yahoo! Grupos, um serviço oferecido por:







------------------------------------------------------------------------------
Links do Yahoo! Grupos

a.. Para visitar o site do seu grupo na web, acesse:
http://br.groups.yahoo.com/group/ciencialist/

b.. Para sair deste grupo, envie um e-mail para:
ciencialist-unsubscribe@yahoogrupos.com.br

c.. O uso que você faz do Yahoo! Grupos está sujeito aos Termos do Serviço do Yahoo!.



[As partes desta mensagem que não continham texto foram removidas]



SUBJECT: Re: Protocolo de Kyoto
FROM: "rmtakata" <rmtakata@altavista.net>
TO: ciencialist@yahoogrupos.com.br
DATE: 22/02/2005 13:10


--- Em ciencialist@yahoogrupos.com.br, "Camila" <camilasaki@y...> >
> *será possível com uma redução de 5,2% dos gases diminuir o efeito
> estufa?

Na verdade sao 5,2% a menos do q. o nivel de 1990. Como de la' pra ca'
houve um aumento de emissao de gases, a reducao seria de uns 12%.

Eh o suficiente para causar algum efeito? Nao se sabe ao certo. Se o
fenomeno for linear - responder diretamente ao nivel de poluentes -
entao algum efeito devera' ter.

Mas o principal pagamento devera' ser a estimulacao de novas
tecnologias menos poluidoras.

> *a Terra n poderia estar se aquecendo por estarmos chegando a uma
> era d glaciação? onde os niveis d co2 tendem a aumentar.

Se estivermos chegando a uma nova glaciacao a temperatura nao deveria
diminuir?

> *o mercado verde (credito de carbono) poderia funcianar, realmente?

Poder pode. Vai funcionar? Nao se sabe. De todo modo, antes mesmo da
oficializacao do protocolo, jah havia o comercio de creditos de
carbono -- mercado futuro. Poderah ajudar a financiar projetos de
reflorestamento e recuperacao de areas degradadas.

[]s,

Roberto Takata





SUBJECT: Re: New Tack Wins Prisoner's Dilemma
FROM: "rmtakata" <rmtakata@altavista.net>
TO: ciencialist@yahoogrupos.com.br
DATE: 22/02/2005 13:19


--- Em ciencialist@yahoogrupos.com.br, "Alvaro Augusto \(E\)"
> "Tit for tat" foi finalmente derrotado.

Mas o tit for tat nao era derrotado tb por um altruista puro?

[]s,

Roberto Takata





SUBJECT: Re: Receita caseira para Injetar EE na Rede Elétrica Comercial
FROM: "Tipoalgo" <tipoalgo@bol.com.br>
TO: ciencialist@yahoogrupos.com.br
DATE: 22/02/2005 14:35


Olá Alvaro Augusto,

Os seus argumentos estão corretos.

Só que a escala a que a receita se refere é bem menor.

Agora fico sabendo por sua manifestação não haver "impedimento legal"
e sim "impedimento real", o que para o pequeno que deseja injetar EE
no Sistema de Distribuição tem o mesmo resultado, ou seja, ele está
fora.

Neste sentido acho oportuno o empenho do Sr Fendel em tentar
implantar a ENERNET (detalhes em www.fendel.com.br), onde os pequenos
teriam a chance de contribuir com o sistema e colocar uns trocados no
bolso. Não sou engenheiro, mas o conceito é muito simples, mais até
que a internet; se esta tem a corbetura que tem é em grande parte
devida à sua segmentação, onde a falha de um ou muitos não acarreta
necessariamente na falha do todo; ou o modelo atual de
imensos "blecautes" na distribuição de EE é mais inteligente? Tentar
inviabilizar a idéia da ENERNET alegando falha de segurança é em
última análise um desserviço, ou ainda defesa de quem está
comprometido até o pescoço com o sistema atual, técnica ou
burocraticamente falando. A ENERNET não inviabiliza a GIGANTENET
atual.

SOBRECARGA? O Sr fendel é também Engenheiro e veja o que ele comenta:

"Para evitar a sobrecarga da rede na eventualidade de desligamento de
um ou mais geradores ou consumidores, e para transportar esta teoria
para a realidade das redes de distribuição, basta adicionar
controladores com tecnologia Plug&Play nas redes e nos geradores."

Mecanismos técnicos estão a disposição, mas alguns acham que é melhor
esperar que os nossos irmãos do Norte nos venda esta tecnologia já
pronta. É mais chique comprar em dólar ou euros!

Abraços

Tipoalgo






SUBJECT: Re: [ciencialist] vírus no celular
FROM: Luis Brudna <luisbrudna@gmail.com>
TO: ciencialist@yahoogrupos.com.br
DATE: 22/02/2005 15:15

Agora um pouco de futurologia estranha

Virus warning: Cyborgs at risk
http://news.com.com/Virus+warning+Cyborgs+at+risk/2100-7337_3-5450753.html

But the academic, who has wired his nervous system up to a computer
and put an RFID chip in his arm, is also warning that the day will
come when computer viruses can infect humans as well as PCs.

:-)

Ateh
Luis Brudna



On Mon, 21 Feb 2005 16:36:27 -0300, Oraculo <oraculo@atibaia.com.br> wrote:
>
> Olá Sandra
>
> Um virus de computador, apesar da fama de algo "misterioso" e "vivo" que ele tem (graças a mídia, que adora manchetes explosivas..:-), é apenas um programa de computador como qualquer outro, como seu Word, Outlook, jogos ou qualquer coisa que passe instruções para o processador de seu computador.
>


SUBJECT: Fw: Hexocromia?
FROM: "Luiz Ferraz Netto" <leobarretos@uol.com.br>
TO: "ciencialist" <ciencialist@yahoogrupos.com.br>
DATE: 22/02/2005 16:38

Quem é hexocromo nessa lista? :-)

Manifeste-se .......

aquele abraço,
===========================
Luiz Ferraz Netto [Léo]
leobarretos@uol.com.br
http://www.feiradeciencias.com.br
===========================
-----Mensagem Original-----
De: Anderson Brandão
Para: leobarretos@uol.com.br
Enviada em: segunda-feira, 21 de fevereiro de 2005 19:43
Assunto: Hexocromia?


Olá Professor Léo, meu nome é Anderson, sou estudante em artes gráficas, o Senhor já ouviu falar em Hexocromia ? Qualquer declaração andfab_pg@yahoo.com.br, agradeço pela atenção,.
__________________________________________________
Converse com seus amigos em tempo real com o Yahoo! Messenger
http://br.download.yahoo.com/messenger/



--------------------------------------------------------------------------------


No virus found in this incoming message.
Checked by AVG Anti-Virus.
Version: 7.0.300 / Virus Database: 266.3.0 - Release Date: 21/02/2005

----------

No virus found in this outgoing message.
Checked by AVG Anti-Virus.
Version: 7.0.300 / Virus Database: 266.3.0 - Release Date: 21/02/2005


[As partes desta mensagem que não continham texto foram removidas]



SUBJECT: Admiss
FROM: Guilherme Figo <ct1azc@yahoo.com.br>
TO: ciencialist@yahoogrupos.com.br
DATE: 22/02/2005 16:49



---------------------------------
Yahoo! Acesso Grátis - Internet rápida e grátis. Instale o discador do Yahoo! agora.

[As partes desta mensagem que não continham texto foram removidas]



SUBJECT: Fw: informa��o
FROM: "Luiz Ferraz Netto" <leobarretos@uol.com.br>
TO: "ciencialist" <ciencialist@yahoogrupos.com.br>
DATE: 22/02/2005 16:56

Alguém conhece a técnica de extrair a prata das chapas de raios - X ?

[]'
===========================
Luiz Ferraz Netto [Léo]
leobarretos@uol.com.br
http://www.feiradeciencias.com.br
===========================
-----Mensagem Original-----
De: wagnerlata
Para: leobarretos
Enviada em: domingo, 20 de fevereiro de 2005 11:29
Assunto: informação


Sr. leo, acredito ser este o terceiro ou quarto e-mail que invio ao Sr. , acredito ser o meu computador que não está muito bom, mas agora vou tentar direto com o Terra, nos anteriores solicitei as seguintes informações: como extrair a prata de chapas de raio X, já consegui chegar só até na borra, outra, qual seria a voltagem maxima para se obter a eletrolese, e se possível o Sr. me enviar o esquema de um Dimmer para 12 volts contínua 10 A na entrada da linha de energia alternada no transformador. Disse em um e-mail que conheço um pouco Barretos, tenho parentes que residem ai, não sei se o Sr. conheceu Paschoal Pizzo, era casado com uma tia minha, bom Sr. Leo, acreditando que agora o Sr. irá receber este, antecipo meus agradecimentos. Wagner.


--------------------------------------------------------------------------------


No virus found in this incoming message.
Checked by AVG Anti-Virus.
Version: 7.0.300 / Virus Database: 265.8.8 - Release Date: 14/02/2005

----------

No virus found in this outgoing message.
Checked by AVG Anti-Virus.
Version: 7.0.300 / Virus Database: 266.3.0 - Release Date: 21/02/2005


[As partes desta mensagem que não continham texto foram removidas]



SUBJECT: Re: [ciencialist] Fw: Hexocromia?
FROM: "Oraculo" <oraculo@atibaia.com.br>
TO: <ciencialist@yahoogrupos.com.br>
DATE: 22/02/2005 17:07


hexacromia: impressão com cores suplementares à quadricromia. (6 cores - C, M, Y, K mais laranja e verde)






----- Original Message -----
From: Luiz Ferraz Netto
To: ciencialist
Sent: Tuesday, February 22, 2005 4:38 PM
Subject: [ciencialist] Fw: Hexocromia?


Quem é hexocromo nessa lista? :-)

Manifeste-se .......

aquele abraço,
===========================
Luiz Ferraz Netto [Léo]
leobarretos@uol.com.br
http://www.feiradeciencias.com.br
===========================
-----Mensagem Original-----
De: Anderson Brandão
Para: leobarretos@uol.com.br
Enviada em: segunda-feira, 21 de fevereiro de 2005 19:43
Assunto: Hexocromia?


Olá Professor Léo, meu nome é Anderson, sou estudante em artes gráficas, o Senhor já ouviu falar em Hexocromia ? Qualquer declaração andfab_pg@yahoo.com.br, agradeço pela atenção,.
__________________________________________________
Converse com seus amigos em tempo real com o Yahoo! Messenger
http://br.download.yahoo.com/messenger/



--------------------------------------------------------------------------------


No virus found in this incoming message.
Checked by AVG Anti-Virus.
Version: 7.0.300 / Virus Database: 266.3.0 - Release Date: 21/02/2005

----------

No virus found in this outgoing message.
Checked by AVG Anti-Virus.
Version: 7.0.300 / Virus Database: 266.3.0 - Release Date: 21/02/2005


[As partes desta mensagem que não continham texto foram removidas]



##### ##### #####

Para saber mais visite
http://www.ciencialist.hpg.ig.com.br


##### ##### ##### #####


Yahoo! Grupos, um serviço oferecido por:







------------------------------------------------------------------------------
Links do Yahoo! Grupos

a.. Para visitar o site do seu grupo na web, acesse:
http://br.groups.yahoo.com/group/ciencialist/

b.. Para sair deste grupo, envie um e-mail para:
ciencialist-unsubscribe@yahoogrupos.com.br

c.. O uso que você faz do Yahoo! Grupos está sujeito aos Termos do Serviço do Yahoo!.



[As partes desta mensagem que não continham texto foram removidas]



SUBJECT: Re: [ciencialist] vírus no celular
FROM: "Sandra Rosario" <coffeacruda@hotmail.com>
TO: ciencialist@yahoogrupos.com.br
DATE: 22/02/2005 17:49

Agradeço ao Takata e ao Homero pelas explicações.
Valeu mesmo!
Sandra

_________________________________________________________________
MSN Messenger: converse online com seus amigos .
http://messenger.msn.com.br



SUBJECT: Re: [ciencialist] Fw: informação
FROM: Luis Brudna <luisbrudna@gmail.com>
TO: ciencialist@yahoogrupos.com.br
DATE: 22/02/2005 18:07

Lembro que fiz uma pratica na graduacao.

Nao tenho a receita comigo e nao consigo achar no Google.

O que consigo lembrar eh que era utilizado agua sanitaria para
remover o ´gel´ da chapa e depois a prata era parcialmente isolada
usando acucar! Nao era um processo muito eficiente e seraim
necessarias mais etapas para purificar a prata.

Pena eu nao poder contribuir mais.

Ateh
Luis Brudna


On Tue, 22 Feb 2005 16:56:08 -0300, Luiz Ferraz Netto
<leobarretos@uol.com.br> wrote:
>
> Alguém conhece a técnica de extrair a prata das chapas de raios - X ?
>
> []'
> ===========================
> Luiz Ferraz Netto [Léo]
> leobarretos@uol.com.br
> http://www.feiradeciencias.com.br
> ===========================
> -----Mensagem Original-----
> De: wagnerlata
> Para: leobarretos
> Enviada em: domingo, 20 de fevereiro de 2005 11:29
> Assunto: informação
>
> Sr. leo, acredito ser este o terceiro ou quarto e-mail que invio ao Sr. , acredito ser o meu computador que não está muito bom, mas agora vou tentar direto com o Terra, nos anteriores solicitei as seguintes informações: como extrair a prata de chapas de raio X, já consegui chegar só até na borra, outra, qual seria a voltagem maxima para se obter a eletrolese, e se possível o Sr. me enviar o esquema de um Dimmer para 12 volts contínua 10 A na entrada da linha de energia alternada no transformador. Disse em um e-mail que conheço um pouco Barretos, tenho parentes que residem ai, não sei se o Sr. conheceu Paschoal Pizzo, era casado com uma tia minha, bom Sr. Leo, acreditando que agora o Sr. irá receber este, antecipo meus agradecimentos. Wagner.
>
> --------------------------------------------------------------------------------
>
> No virus found in this incoming message.
> Checked by AVG Anti-Virus.
> Version: 7.0.300 / Virus Database: 265.8.8 - Release Date: 14/02/2005
>
> ----------
>
> No virus found in this outgoing message.
> Checked by AVG Anti-Virus.
> Version: 7.0.300 / Virus Database: 266.3.0 - Release Date: 21/02/2005
>
> [As partes desta mensagem que não continham texto foram removidas]
>
> ##### ##### #####
>
> Para saber mais visite
> http://www.ciencialist.hpg.ig.com.br
>
> ##### ##### ##### #####
> Links do Yahoo! Grupos
>
>
>
>
>


SUBJECT: Re: [ciencialist] Fw: informação
FROM: "Luiz Ferraz Netto" <leobarretos@uol.com.br>
TO: <ciencialist@yahoogrupos.com.br>
DATE: 22/02/2005 18:51

Oi Brudna,

valeu! Me explica qual o papel do açúcar nessa primeira fase de separação da prata.

Continuo aguardando novas contribuições dos listeiros.
[]'
===========================
Luiz Ferraz Netto [Léo]
leobarretos@uol.com.br
http://www.feiradeciencias.com.br
===========================
-----Mensagem Original-----
De: "Luis Brudna" <luisbrudna@gmail.com>
Para: <ciencialist@yahoogrupos.com.br>
Enviada em: terça-feira, 22 de fevereiro de 2005 18:07
Assunto: Re: [ciencialist] Fw: informação



Lembro que fiz uma pratica na graduacao.

Nao tenho a receita comigo e nao consigo achar no Google.

O que consigo lembrar eh que era utilizado agua sanitaria para
remover o ´gel´ da chapa e depois a prata era parcialmente isolada
usando acucar! Nao era um processo muito eficiente e seraim
necessarias mais etapas para purificar a prata.

Pena eu nao poder contribuir mais.

Ateh
Luis Brudna


On Tue, 22 Feb 2005 16:56:08 -0300, Luiz Ferraz Netto
<leobarretos@uol.com.br> wrote:
>
> Alguém conhece a técnica de extrair a prata das chapas de raios - X ?
>
> []'
> ===========================
> Luiz Ferraz Netto [Léo]
> leobarretos@uol.com.br
> http://www.feiradeciencias.com.br
> ===========================
> -----Mensagem Original-----
> De: wagnerlata
> Para: leobarretos
> Enviada em: domingo, 20 de fevereiro de 2005 11:29
> Assunto: informação
>
> Sr. leo, acredito ser este o terceiro ou quarto e-mail que invio ao Sr. , acredito ser o meu computador que não está muito bom, mas agora vou tentar direto com o Terra, nos anteriores solicitei as seguintes informações: como extrair a prata de chapas de raio X, já consegui chegar só até na borra, outra, qual seria a voltagem maxima para se obter a eletrolese, e se possível o Sr. me enviar o esquema de um Dimmer para 12 volts contínua 10 A na entrada da linha de energia alternada no transformador. Disse em um e-mail que conheço um pouco Barretos, tenho parentes que residem ai, não sei se o Sr. conheceu Paschoal Pizzo, era casado com uma tia minha, bom Sr. Leo, acreditando que agora o Sr. irá receber este, antecipo meus agradecimentos. Wagner.
>
> --------------------------------------------------------------------------------
>
> No virus found in this incoming message.
> Checked by AVG Anti-Virus.
> Version: 7.0.300 / Virus Database: 265.8.8 - Release Date: 14/02/2005
>
> ----------
>
> No virus found in this outgoing message.
> Checked by AVG Anti-Virus.
> Version: 7.0.300 / Virus Database: 266.3.0 - Release Date: 21/02/2005
>
> [As partes desta mensagem que não continham texto foram removidas]
>
> ##### ##### #####
>
> Para saber mais visite
> http://www.ciencialist.hpg.ig.com.br
>
> ##### ##### ##### #####
> Links do Yahoo! Grupos
>
>
>
>
>


##### ##### #####

Para saber mais visite
http://www.ciencialist.hpg.ig.com.br


##### ##### ##### #####
Links do Yahoo! Grupos










--
No virus found in this incoming message.
Checked by AVG Anti-Virus.
Version: 7.0.300 / Virus Database: 266.3.0 - Release Date: 21/02/2005




--
No virus found in this outgoing message.
Checked by AVG Anti-Virus.
Version: 7.0.300 / Virus Database: 266.3.0 - Release Date: 21/02/2005



SUBJECT: Re: [ciencialist] Fw: informação
FROM: Luis Brudna <luisbrudna@gmail.com>
TO: ciencialist@yahoogrupos.com.br
DATE: 22/02/2005 19:23

Indo mais longe nas profundas memorias. Me parece que o açúcar agia
como agente redutor. O resultado era uma borra preta (açúcar todo
degradado) com uns nódulos pouco identificáveis de prata, que devia
ser purificada.

É bem provável que tem outros ingredientes nessa receita. Continuei
procurando na net, e nada. Deve ser uma daquelas experiências
´obscuras´ que os professores acham por ai. :-)

Até
Luís Brudna


On Tue, 22 Feb 2005 18:51:17 -0300, Luiz Ferraz Netto
<leobarretos@uol.com.br> wrote:
>
> Oi Brudna,
>
> valeu! Me explica qual o papel do açúcar nessa primeira fase de separação da prata.
>
> Continuo aguardando novas contribuições dos listeiros.
> []'
> ===========================
> Luiz Ferraz Netto [Léo]
> leobarretos@uol.com.br
> http://www.feiradeciencias.com.br
> ===========================
> -----Mensagem Original-----
> De: "Luis Brudna" <luisbrudna@gmail.com>
> Para: <ciencialist@yahoogrupos.com.br>
> Enviada em: terça-feira, 22 de fevereiro de 2005 18:07
> Assunto: Re: [ciencialist] Fw: informação
>
> Lembro que fiz uma pratica na graduacao.
>
> Nao tenho a receita comigo e nao consigo achar no Google.
>
> O que consigo lembrar eh que era utilizado agua sanitaria para
> remover o ´gel´ da chapa e depois a prata era parcialmente isolada
> usando acucar! Nao era um processo muito eficiente e seraim
> necessarias mais etapas para purificar a prata.
>
> Pena eu nao poder contribuir mais.
>
> Ateh
> Luis Brudna
>


SUBJECT: Re: [ciencialist] Re: Uma mente brilhante - o filme
FROM: "Emiliano Chemello - Yahoo Grupos" <chemelloe@yahoo.com.br>
TO: <ciencialist@yahoogrupos.com.br>
DATE: 22/02/2005 20:12

Marcos!

Eternamente grato!

Amplexos do
Emiliano Chemello
----- Original Message -----
From: Marcos Ludwig
To: ciencialist@yahoogrupos.com.br
Sent: Saturday, February 12, 2005 1:20 AM
Subject: Re: [ciencialist] Re: Uma mente brilhante - o filme


> Emiliano, aproveitando que vc está escrevendo este artigo, me diga: vc tem ai a transcrição desta cena do filme. Ou seja, vc tem escrito o raciocinio dele no bar. Se tiver, pode postar.

achei.

"Nash: If we all go for the blonde and block each other, not a single
one of us is going to get her. So then we go for her friends, but they
will all give us the cold shoulder because no on likes to be second
choice. But what if none of us goes for the blonde? We won't get in
each other's way and we won't insult the other girls. It's the only
way to win. It's the only way we all get laid."

tem a outra cena que antecede a esta, em que ele leva o tapa da garota
do bar que ele tenta pular as "atividades platônicas que antecedem a
cópula":

"Nash: I don't exactly know what I am required to say in order for you
to have intercourse with me. But could we assume that I said all that.
I mean essentially we are talking about fluid exchange right? So could
we go just straight to the sex. "

extraído daqui: http://www.imdb.com/title/tt0268978/quotes

[]!
--
marcos ludwig [ ICQ# 5967606 ] [ marcosludwig@gmail.com ]
rei nada - apologias: [ www.sentinelas.org/reinada ]


##### ##### #####

Para saber mais visite
http://www.ciencialist.hpg.ig.com.br


##### ##### ##### #####


Yahoo! Grupos, um serviço oferecido por:

São Paulo Rio de Janeiro Curitiba Porto Alegre Belo Horizonte Brasília




------------------------------------------------------------------------------
Links do Yahoo! Grupos

a.. Para visitar o site do seu grupo na web, acesse:
http://br.groups.yahoo.com/group/ciencialist/

b.. Para sair deste grupo, envie um e-mail para:
ciencialist-unsubscribe@yahoogrupos.com.br

c.. O uso que você faz do Yahoo! Grupos está sujeito aos Termos do Serviço do Yahoo!.



[As partes desta mensagem que não continham texto foram removidas]



SUBJECT: RE: [ciencialist] União Européia diz que cupuaçu é nosso
FROM: "murilo filo" <avalanchedrive@hotmail.com>
TO: ciencialist@yahoogrupos.com.br
DATE: 22/02/2005 22:04

O que não ví ninguém comentar, é que o nome ''Asahi Foods'' já tem muito que
ver com 'assaí', o fruto daquela palmeira da amazônia.
O suco dela tomado com arroz, feijão, ovo, guaraná e carne é um muito bom
energético... abr/M.

>From: José Renato <jrma@terra.com.br>
>Reply-To: ciencialist@yahoogrupos.com.br
>To: <ciencialist@yahoogrupos.com.br>
>Subject: [ciencialist] União Européia diz que cupuaçu é nosso
>Date: Tue, 22 Feb 2005 10:28:41 -0300
>
>
>JC e-mail 2710, de 21 de Fevereiro de 2005.
>
>UE reconhece direito do Brasil à marca cupuaçu
>
>Entre os argumentos usados para recuperar o cupuaçu estão o fato de o nome
>ser um termo de domínio público e de o fruto ser típico de uma árvore da
>Amazônia
>
>Vivian Oswald escreve de Bruxelas para 'O Globo':
>
>Os japoneses que nos desculpem, mas o cupuaçu é nosso.
>
>Se ainda havia alguma dúvida a esse respeito no mercado europeu, o
>Escritório de Harmonização para o Mercado Interno, instituição européia
>responsável por marcas e patentes, encarregou-se de desfazê-la: cancelou o
>registro da marca cupuaçu que havia sido feito pela empresa japonesa Asahi
>Foods.
>
>A decisão põe fim a uma disputa de quase dois anos que vinha sendo travada
>entre o governo brasileiro e a companhia.
>
>A novidade permite que os exportadores brasileiros do cupuaçu ou de doces
>feitos com a fruta não precisem mais pagar royalties aos japoneses para
>vender seus produtos na Europa.
>
>Essa é a segunda derrota da Asahi Foods, que ano passado foi obrigada a
>retirar o pedido de patenteamento do produto feito no Japão.
>
>Açaí e andiroba ainda estão com registros estrangeiros
>
>A vitória do governo brasileiro abre caminho para outras decisões do mesmo
>gênero na disputa por nomes de frutos, sementes e árvores brasileiras das
>quais grandes empresas estrangeiras tentam se apropriar para garantir o seu
>mercado.
>
>Ao registrar a marca, todos os direitos passam a ser do seu dono. Estão
>nesta situação produtos como o açaí e a andiroba.
>
>'Dá satisfação saber que o princípio está sendo respeitado. É encorajador e
>coerente com as posturas européias de proteção da denominação de origem. A
>medida derruba um dos obstáculos às exportações dos pequenos produtores de
>cupuaçu brasileiros', disse o chefe da Missão do Brasil junto a União
>Européia, embaixador José Alfredo Graça Lima.
>
>A ação contra a empresa japonesa foi aberta pela missão brasileira no
>Escritório de Harmonização em abril de 2003, mas a decisão final só saiu
>semana passada.
>
>A Asahi Foods também terá de pagar as custas do processo. Segundo a
>instituição européia, ainda cabe recurso para os japoneses, que terão até
>dois meses para fazê-lo.
>
>Mas a expectativa é que a causa já esteja ganha. Entre os argumentos usados
>para recuperar o cupuaçu estão o fato de o nome ser um termo de domínio
>público e de o fruto ser típico de uma árvore da Amazônia.
>
>Para o embaixador Graça Lima, este episódio pode acabar levando as empresas
>brasileiras a se interessarem por este mercado.
>
>(O Estado de SP, 19/2)
>< http://www.jornaldaciencia.org.br/Detalhe.jsp?id=25616 >
>
>[As partes desta mensagem que não continham texto foram removidas]
>
>
>
>##### ##### #####
>
>Para saber mais visite
>http://www.ciencialist.hpg.ig.com.br
>
>
>##### ##### ##### #####
>Links do Yahoo! Grupos
>
>
>
>
>
>
>
>




SUBJECT: Links interessantes
FROM: "Narumi Abe" <omnivisio@gmail.com>
TO: ciencialist@yahoogrupos.com.br
DATE: 23/02/2005 07:11


Olá a todos,

Encontrei dois sites muito interessantes, ambos em língua
alienígena:

Este lembra o Imperdível do Léo. Obviamente não tão completo:
http://www.scitoys.com/

Este outro simula um zoom que começa a alguns anos luz da
Via Láctea até chegar aos quarks de uma planta na Terra.
http://micro.magnet.fsu.edu/primer/java/scienceopticsu/powersof10/

[]s,
--
Narumi Abe





SUBJECT: Antineutrino
FROM: José Renato <jrma@terra.com.br>
TO: <ciencialist@yahoogrupos.com.br>
DATE: 23/02/2005 07:29

Em um artigo sobre a partícula neutrino o autor infere que o antineutrino tem algo diferente a ser detectado em instalações a serem instaladas próximo à usina nuclear Angra II, em convênio com outros países.
Dúvida: se a antipartícula tem o mesmo spin e carga igual mas de sinal contrário, no caso do nêutron - existentes nos núcleos atômicos - e da partícula neutrino, elas não seriam exatamente iguais?
Abraços
José Renato

[As partes desta mensagem que não continham texto foram removidas]



SUBJECT: Re: [ciencialist] Links interessantes
FROM: "Luiz Ferraz Netto" <leobarretos@uol.com.br>
TO: <ciencialist@yahoogrupos.com.br>
DATE: 23/02/2005 08:00

Dei uma olhada nesse link abaixo e,

pergunto,

o que é 'marshmallow'? É uma salsicha alienígena? uma lingüiça? um doce para assar na churrasqueira?

[]'
===========================
Luiz Ferraz Netto [Léo]
leobarretos@uol.com.br
http://www.feiradeciencias.com.br
===========================
-----Mensagem Original-----
De: "Narumi Abe" <omnivisio@gmail.com>
Para: <ciencialist@yahoogrupos.com.br>
Enviada em: quarta-feira, 23 de fevereiro de 2005 07:11
Assunto: [ciencialist] Links interessantes




Olá a todos,

Encontrei dois sites muito interessantes, ambos em língua
alienígena:

Este lembra o Imperdível do Léo. Obviamente não tão completo:
http://www.scitoys.com/

Este outro simula um zoom que começa a alguns anos luz da
Via Láctea até chegar aos quarks de uma planta na Terra.
http://micro.magnet.fsu.edu/primer/java/scienceopticsu/powersof10/

[]s,
--
Narumi Abe





##### ##### #####

Para saber mais visite
http://www.ciencialist.hpg.ig.com.br


##### ##### ##### #####
Links do Yahoo! Grupos










--
No virus found in this incoming message.
Checked by AVG Anti-Virus.
Version: 7.0.300 / Virus Database: 266.3.0 - Release Date: 21/02/2005




--
No virus found in this outgoing message.
Checked by AVG Anti-Virus.
Version: 7.0.300 / Virus Database: 266.3.0 - Release Date: 21/02/2005



SUBJECT: Re: Links interessantes
FROM: "Narumi Abe" <omnivisio@gmail.com>
TO: ciencialist@yahoogrupos.com.br
DATE: 23/02/2005 09:59


--- Em ciencialist@yahoogrupos.com.br, "Luiz Ferraz Netto"
<leobarretos@u...> escreveu
> o que é 'marshmallow'? É uma salsicha alienígena? uma lingüiça?
> um doce para assar na churrasqueira?

É uma maria-mole não tão mole em cubinhos. Você pode assar na
churrasqueira, colocar no chocolate quente ou comer puro.

[]s,
--
Narumi Abe





SUBJECT: Re: Fw: informação
FROM: César A. K. Grossmann <cesarakg@bol.com.br>
TO: ciencialist@yahoogrupos.com.br
DATE: 23/02/2005 11:12


--- Em ciencialist@yahoogrupos.com.br, Luis Brudna <luisbrudna@g...>
escreveu
>
> É bem provável que tem outros ingredientes nessa receita. Continuei
> procurando na net, e nada. Deve ser uma daquelas experiências
> ´obscuras´ que os professores acham por ai. :-)

Veja se ajuda:

http://tinyurl.com/3ktye (pesquisa no Google, em inglês)
http://www.p2pays.org/ref/05/04619.htm (artigo em inglês, falando por
alto de alguns processos para recuperar a prata de filmes).

[]s
--
Cesar A. K. Grossmann
http://www.LinuxByGrossmann.cjb.net/






SUBJECT: Os Canindés
FROM: "Dolarhide dos Anjos" <vurutia@yahoo.com.br>
TO: ciencialist@yahoogrupos.com.br
DATE: 23/02/2005 14:57


Sou um cara que quer viver plenamente, amo a sabedoria, a natureza, a
liberdade, poesia, filosofia, literatura, e tocar em uma banda de
rock faz parte de minha plenitude. Sou do Ceará, moro em Canindé, a
terra de Bastião raposa e do Cocote e o Messias,são pessoas bastante
conhecidas e amadas. Sei falar Inglês, se alguém quiser praticar
comigo, entre em contato. My life is full of happiness and madness
coz I want it to be.





SUBJECT: Fw: ENC: Fw: Como chamar a policia
FROM: "Paulo Henrique Lerbach Rodrigues" <phatleta@yahoo.com.br>
TO: <Undisclosed-Recipient:;>
DATE: 23/02/2005 15:26



Paulo Henrique Lerbach Rodrigues

Jesus Cristo é o Único que pode te levar para o céu, não se engane, conheça-O!

Futuro Piloto Privado de Avião
(Já solei o Zarapa galerinha)

lerbach@brturbo.com

55 61 354 2197 ou 55 61 9975 9188




----- Original Message -----
From: André Melo
To: comrec@terra.com.br
Sent: Wednesday, February 23, 2005 2:51 PM
Subject: FW: ENC: Fw: Como chamar a policia






>From: jvdaniel <jvdaniel@ig.com.br>
>To: riconayy@ig.com.br
>CC: jaquerev@bol.com.br,ajulia@pe.sesc.com.br,daniamanda@hotmail.com,comandanteandre@hotmail.com,bamvieira@hotmail.com,graca_carvalho@bol.com.br,monica.lu.carvalho@globo.com
>Subject: ENC: Fw: Como chamar a policia
>Date: Thu, 17 Feb 2005 14:09:16 -0200
>
>
>
>
> > >Subject: Como chamar a policia
> > >Date: Mon, 7 Feb 2005 13:43:21 -0300
> > >
> > >Guia pratico
> > > Eu tenho o sono muito leve, e numa noite dessas notei que havia algu?m
> > >andando sorrateiramente no quintal de casa.
> > > Levantei em sil?ncio e fiquei acompanhando os leves ru?dos que vinham
>l?
> > >de fora, at? ver uma silhueta passando pela janela do banheiro.
> > > Como minha casa ? muito segura, com grades nas janelas e trancas
> > >internas
> > >nas portas, n?o fiquei muito preocupado, mas ? claro que eu n?o ia deixar
> > >um
> > >ladr?o ali, espiando tranq?ilamente.
> > > Liguei baixinho para a pol?cia informei a situa??o e o meu endere?o.
> > > Perguntaram-me se o ladr?o estava armado ou se j? estava no interior
>da
> > >casa.
> > > Esclareci que n?o e disseram-me que n?o havia nenhuma viatura por
>perto
> > >para ajudar, mas que iriam mandar algu?m assim que fosse poss?vel.
> > > Um minuto depois liguei de novo, e disse com a voz calma:
> > > - Ol?, eu liguei h? pouco porque tinha algu?m no meu quintal. N?o
> > >precisa
> > >mais ter pressa. Eu j? matei o ladr?o com um tiro da escopeta calibre 12
> > >que
> > >tenho guardada em casa para estas situa??es. Putz, o tiro fez um
> > > estrago danado no cara!
> > > Passados menos de tr?s minutos, estavam na minha rua cinco carros da
> > >pol?cia, um helic?ptero, uma unidade do resgate e uma equipe de TV. Eles
> > >prenderam o ladr?o em flagrante, que ficava olhando tudo com cara de
> > >assombrado. Talvez ele estivesse pensando que aquela era a casa do
> > >Comandante da Pol?cia. No meio do tumulto, um tenente se aproximou de mim
>e
> > >disse:
> > > - Pensei que tivesse dito que tinha matado o ladr?o.
> > > Eu respondi:
> > > - Pensei que tivesse dito que n?o havia ningu?m dispon?vel...
> > >
> > >
> >
> > _________________________________________________________________
> > Chegou o que faltava: MSN Acesso Gr?tis. Instale J?!
> > http://www.msn.com.br/discador
> >
> >
> >
> >
> > --
> > Internal Virus Database is out-of-date.
> > Checked by AVG Anti-Virus.
> > Version: 7.0.300 / Virus Database: 265.8.5 - Release Date: 3/2/2005
> >
> >
>
>
>
>--
>No virus found in this outgoing message.
>Checked by AVG Anti-Virus.
>Version: 7.0.300 / Virus Database: 265.8.8 - Release Date: 14/2/2005


--------------------------------------------------------------------------------
MSN Busca: f?cil, r?pido, direto ao ponto. Encontre o que voc? quiser. Clique aqui.

[As partes desta mensagem que não continham texto foram removidas]



SUBJECT: Re: [ciencialist] Fw: ENC: Fw: Como chamar a policia
FROM: "Oraculo" <oraculo@atibaia.com.br>
TO: <ciencialist@yahoogrupos.com.br>
DATE: 23/02/2005 15:33

Olá Paulo

Bem, prefiro ir até o céu de avião mesmo, cientificamente..:-)

Homero

----- Original Message -----
From: Paulo Henrique Lerbach Rodrigues
To: Undisclosed-Recipient:;
Sent: Wednesday, February 23, 2005 3:26 PM
Subject: [ciencialist] Fw: ENC: Fw: Como chamar a policia




Paulo Henrique Lerbach Rodrigues

Jesus Cristo é o Único que pode te levar para o céu, não se engane, conheça-O!

Futuro Piloto Privado de Avião
(Já solei o Zarapa galerinha)

lerbach@brturbo.com

55 61 354 2197 ou 55 61 9975 9188




----- Original Message -----
From: André Melo
To: comrec@terra.com.br
Sent: Wednesday, February 23, 2005 2:51 PM
Subject: FW: ENC: Fw: Como chamar a policia






>From: jvdaniel <jvdaniel@ig.com.br>
>To: riconayy@ig.com.br
>CC: jaquerev@bol.com.br,ajulia@pe.sesc.com.br,daniamanda@hotmail.com,comandanteandre@hotmail.com,bamvieira@hotmail.com,graca_carvalho@bol.com.br,monica.lu.carvalho@globo.com
>Subject: ENC: Fw: Como chamar a policia
>Date: Thu, 17 Feb 2005 14:09:16 -0200
>
>
>
>
> > >Subject: Como chamar a policia
> > >Date: Mon, 7 Feb 2005 13:43:21 -0300
> > >
> > >Guia pratico
> > > Eu tenho o sono muito leve, e numa noite dessas notei que havia algu?m
> > >andando sorrateiramente no quintal de casa.
> > > Levantei em sil?ncio e fiquei acompanhando os leves ru?dos que vinham
>l?
> > >de fora, at? ver uma silhueta passando pela janela do banheiro.
> > > Como minha casa ? muito segura, com grades nas janelas e trancas
> > >internas
> > >nas portas, n?o fiquei muito preocupado, mas ? claro que eu n?o ia deixar
> > >um
> > >ladr?o ali, espiando tranq?ilamente.
> > > Liguei baixinho para a pol?cia informei a situa??o e o meu endere?o.
> > > Perguntaram-me se o ladr?o estava armado ou se j? estava no interior
>da
> > >casa.
> > > Esclareci que n?o e disseram-me que n?o havia nenhuma viatura por
>perto
> > >para ajudar, mas que iriam mandar algu?m assim que fosse poss?vel.
> > > Um minuto depois liguei de novo, e disse com a voz calma:
> > > - Ol?, eu liguei h? pouco porque tinha algu?m no meu quintal. N?o
> > >precisa
> > >mais ter pressa. Eu j? matei o ladr?o com um tiro da escopeta calibre 12
> > >que
> > >tenho guardada em casa para estas situa??es. Putz, o tiro fez um
> > > estrago danado no cara!
> > > Passados menos de tr?s minutos, estavam na minha rua cinco carros da
> > >pol?cia, um helic?ptero, uma unidade do resgate e uma equipe de TV. Eles
> > >prenderam o ladr?o em flagrante, que ficava olhando tudo com cara de
> > >assombrado. Talvez ele estivesse pensando que aquela era a casa do
> > >Comandante da Pol?cia. No meio do tumulto, um tenente se aproximou de mim
>e
> > >disse:
> > > - Pensei que tivesse dito que tinha matado o ladr?o.
> > > Eu respondi:
> > > - Pensei que tivesse dito que n?o havia ningu?m dispon?vel...
> > >
> > >
> >
> > _________________________________________________________________
> > Chegou o que faltava: MSN Acesso Gr?tis. Instale J?!
> > http://www.msn.com.br/discador
> >
> >
> >
> >
> > --
> > Internal Virus Database is out-of-date.
> > Checked by AVG Anti-Virus.
> > Version: 7.0.300 / Virus Database: 265.8.5 - Release Date: 3/2/2005
> >
> >
>
>
>
>--
>No virus found in this outgoing message.
>Checked by AVG Anti-Virus.
>Version: 7.0.300 / Virus Database: 265.8.8 - Release Date: 14/2/2005


--------------------------------------------------------------------------------
MSN Busca: f?cil, r?pido, direto ao ponto. Encontre o que voc? quiser. Clique aqui.

[As partes desta mensagem que não continham texto foram removidas]



##### ##### #####

Para saber mais visite
http://www.ciencialist.hpg.ig.com.br


##### ##### ##### #####


Yahoo! Grupos, um serviço oferecido por:







------------------------------------------------------------------------------
Links do Yahoo! Grupos

a.. Para visitar o site do seu grupo na web, acesse:
http://br.groups.yahoo.com/group/ciencialist/

b.. Para sair deste grupo, envie um e-mail para:
ciencialist-unsubscribe@yahoogrupos.com.br

c.. O uso que você faz do Yahoo! Grupos está sujeito aos Termos do Serviço do Yahoo!.



[As partes desta mensagem que não continham texto foram removidas]



SUBJECT: videos de la NASA
FROM: "Esteban Moreno" <estebanmoreno@idhi.org.br>
TO: "Vivens Scientia" <VivensScientia@yahoogroups.com>, <ciencialist@yahoogrupos.com.br>
DATE: 23/02/2005 15:34

Noticias Científicas de la NASA del 10 febrero, 2005

Divertidos videos de la NASA, de un minuto de duración, contestan las
preguntas acerca del espacio que no te atrevías a preguntar.

TODO EL REPORTAJE en

http://ciencia.msfc.nasa.gov/headlines/y2005/10feb_brainbites.htm?list811189


[As partes desta mensagem que não continham texto foram removidas]



SUBJECT: Re: [ciencialist] Os Canindés
FROM: "Alvaro Augusto \(E\)" <alvaro@electraenergy.com.br>
TO: <ciencialist@yahoogrupos.com.br>
DATE: 23/02/2005 16:00

OK, dude. Welcome to this mad group!

It's nice to have another "goat of the plague" here...:o)

[ ]s

Alvaro Augusto

----- Original Message -----
From: Dolarhide dos Anjos
To: ciencialist@yahoogrupos.com.br
Sent: Wednesday, February 23, 2005 2:57 PM
Subject: [ciencialist] Os Canindés



Sou um cara que quer viver plenamente, amo a sabedoria, a natureza, a
liberdade, poesia, filosofia, literatura, e tocar em uma banda de
rock faz parte de minha plenitude. Sou do Ceará, moro em Canindé, a
terra de Bastião raposa e do Cocote e o Messias,são pessoas bastante
conhecidas e amadas. Sei falar Inglês, se alguém quiser praticar
comigo, entre em contato. My life is full of happiness and madness
coz I want it to be.





##### ##### #####

Para saber mais visite
http://www.ciencialist.hpg.ig.com.br


##### ##### ##### #####


Yahoo! Grupos, um serviço oferecido por:







------------------------------------------------------------------------------
Links do Yahoo! Grupos

a.. Para visitar o site do seu grupo na web, acesse:
http://br.groups.yahoo.com/group/ciencialist/

b.. Para sair deste grupo, envie um e-mail para:
ciencialist-unsubscribe@yahoogrupos.com.br

c.. O uso que você faz do Yahoo! Grupos está sujeito aos Termos do Serviço do Yahoo!.



[As partes desta mensagem que não continham texto foram removidas]



SUBJECT: Re: [ciencialist] Fw: ENC: Fw: Como chamar a policia
FROM: "Alvaro Augusto \(E\)" <alvaro@electraenergy.com.br>
TO: <ciencialist@yahoogrupos.com.br>
DATE: 23/02/2005 16:08

Caro Paulo Henrique,

Não entendi como a sua mensagem se relaciona com o resto. De qualquer modo, uma mensagem dessas em um grupo como esse é, no mínimo inusitada. Com o passar do tempo, minha posição sobre religiões foi se amenizando, mas quando vejo alguém sacar uma afirmação monopolista direto das profundezas da baixa idade média... sei não. Você acredita mesmo que 4/5 da humanidade estão condenados ao inferno? E, nesse caso, o céu será composto somente de cristãos? Não é por nada não, mas esse céu deve ser um inferno!

[ ]s

Alvaro Augusto



----- Original Message -----
From: Paulo Henrique Lerbach Rodrigues
To: Undisclosed-Recipient:;
Sent: Wednesday, February 23, 2005 3:26 PM
Subject: [ciencialist] Fw: ENC: Fw: Como chamar a policia




Paulo Henrique Lerbach Rodrigues

Jesus Cristo é o Único que pode te levar para o céu, não se engane, conheça-O!

Futuro Piloto Privado de Avião
(Já solei o Zarapa galerinha)

lerbach@brturbo.com

55 61 354 2197 ou 55 61 9975 9188




----- Original Message -----
From: André Melo
To: comrec@terra.com.br
Sent: Wednesday, February 23, 2005 2:51 PM
Subject: FW: ENC: Fw: Como chamar a policia






>From: jvdaniel <jvdaniel@ig.com.br>
>To: riconayy@ig.com.br
>CC: jaquerev@bol.com.br,ajulia@pe.sesc.com.br,daniamanda@hotmail.com,comandanteandre@hotmail.com,bamvieira@hotmail.com,graca_carvalho@bol.com.br,monica.lu.carvalho@globo.com
>Subject: ENC: Fw: Como chamar a policia
>Date: Thu, 17 Feb 2005 14:09:16 -0200
>
>
>
>
> > >Subject: Como chamar a policia
> > >Date: Mon, 7 Feb 2005 13:43:21 -0300
> > >
> > >Guia pratico
> > > Eu tenho o sono muito leve, e numa noite dessas notei que havia algu?m
> > >andando sorrateiramente no quintal de casa.
> > > Levantei em sil?ncio e fiquei acompanhando os leves ru?dos que vinham
>l?
> > >de fora, at? ver uma silhueta passando pela janela do banheiro.
> > > Como minha casa ? muito segura, com grades nas janelas e trancas
> > >internas
> > >nas portas, n?o fiquei muito preocupado, mas ? claro que eu n?o ia deixar
> > >um
> > >ladr?o ali, espiando tranq?ilamente.
> > > Liguei baixinho para a pol?cia informei a situa??o e o meu endere?o.
> > > Perguntaram-me se o ladr?o estava armado ou se j? estava no interior
>da
> > >casa.
> > > Esclareci que n?o e disseram-me que n?o havia nenhuma viatura por
>perto
> > >para ajudar, mas que iriam mandar algu?m assim que fosse poss?vel.
> > > Um minuto depois liguei de novo, e disse com a voz calma:
> > > - Ol?, eu liguei h? pouco porque tinha algu?m no meu quintal. N?o
> > >precisa
> > >mais ter pressa. Eu j? matei o ladr?o com um tiro da escopeta calibre 12
> > >que
> > >tenho guardada em casa para estas situa??es. Putz, o tiro fez um
> > > estrago danado no cara!
> > > Passados menos de tr?s minutos, estavam na minha rua cinco carros da
> > >pol?cia, um helic?ptero, uma unidade do resgate e uma equipe de TV. Eles
> > >prenderam o ladr?o em flagrante, que ficava olhando tudo com cara de
> > >assombrado. Talvez ele estivesse pensando que aquela era a casa do
> > >Comandante da Pol?cia. No meio do tumulto, um tenente se aproximou de mim
>e
> > >disse:
> > > - Pensei que tivesse dito que tinha matado o ladr?o.
> > > Eu respondi:
> > > - Pensei que tivesse dito que n?o havia ningu?m dispon?vel...
> > >
> > >
> >
> > _________________________________________________________________
> > Chegou o que faltava: MSN Acesso Gr?tis. Instale J?!
> > http://www.msn.com.br/discador
> >
> >
> >
> >
> > --
> > Internal Virus Database is out-of-date.
> > Checked by AVG Anti-Virus.
> > Version: 7.0.300 / Virus Database: 265.8.5 - Release Date: 3/2/2005
> >
> >
>
>
>
>--
>No virus found in this outgoing message.
>Checked by AVG Anti-Virus.
>Version: 7.0.300 / Virus Database: 265.8.8 - Release Date: 14/2/2005


--------------------------------------------------------------------------------
MSN Busca: f?cil, r?pido, direto ao ponto. Encontre o que voc? quiser. Clique aqui.

[As partes desta mensagem que não continham texto foram removidas]



##### ##### #####

Para saber mais visite
http://www.ciencialist.hpg.ig.com.br


##### ##### ##### #####


Yahoo! Grupos, um serviço oferecido por:







------------------------------------------------------------------------------
Links do Yahoo! Grupos

a.. Para visitar o site do seu grupo na web, acesse:
http://br.groups.yahoo.com/group/ciencialist/

b.. Para sair deste grupo, envie um e-mail para:
ciencialist-unsubscribe@yahoogrupos.com.br

c.. O uso que você faz do Yahoo! Grupos está sujeito aos Termos do Serviço do Yahoo!.



[As partes desta mensagem que não continham texto foram removidas]



SUBJECT: Re: [ciencialist] Re: Uma mente brilhante - o filme
FROM: "Alvaro Augusto \(E\)" <alvaro@electraenergy.com.br>
TO: <ciencialist@yahoogrupos.com.br>
DATE: 23/02/2005 16:28

Caro Emiliano,

A cena do bar está descrita em http://www.keepmedia.com/pubs/Newsweek/2002/03/25/310086/?extID=10047&data=nash_equilibrium, que contém uma entrevista com Paul Milgorm, um especialista em teoria dos jogos.

[ ]s

Alvaro Augusto

----- Original Message -----
From: Emiliano Chemello - Yahoo Grupos
To: ciencialist@yahoogrupos.com.br
Sent: Tuesday, February 22, 2005 8:12 PM
Subject: Re: [ciencialist] Re: Uma mente brilhante - o filme


Marcos!

Eternamente grato!

Amplexos do
Emiliano Chemello
----- Original Message -----
From: Marcos Ludwig
To: ciencialist@yahoogrupos.com.br
Sent: Saturday, February 12, 2005 1:20 AM
Subject: Re: [ciencialist] Re: Uma mente brilhante - o filme


> Emiliano, aproveitando que vc está escrevendo este artigo, me diga: vc tem ai a transcrição desta cena do filme. Ou seja, vc tem escrito o raciocinio dele no bar. Se tiver, pode postar.

achei.

"Nash: If we all go for the blonde and block each other, not a single
one of us is going to get her. So then we go for her friends, but they
will all give us the cold shoulder because no on likes to be second
choice. But what if none of us goes for the blonde? We won't get in
each other's way and we won't insult the other girls. It's the only
way to win. It's the only way we all get laid."

tem a outra cena que antecede a esta, em que ele leva o tapa da garota
do bar que ele tenta pular as "atividades platônicas que antecedem a
cópula":

"Nash: I don't exactly know what I am required to say in order for you
to have intercourse with me. But could we assume that I said all that.
I mean essentially we are talking about fluid exchange right? So could
we go just straight to the sex. "

extraído daqui: http://www.imdb.com/title/tt0268978/quotes

[]!
--
marcos ludwig [ ICQ# 5967606 ] [ marcosludwig@gmail.com ]
rei nada - apologias: [ www.sentinelas.org/reinada ]


##### ##### #####

Para saber mais visite
http://www.ciencialist.hpg.ig.com.br


##### ##### ##### #####


Yahoo! Grupos, um serviço oferecido por:

São Paulo Rio de Janeiro Curitiba Porto Alegre Belo Horizonte Brasília




------------------------------------------------------------------------------
Links do Yahoo! Grupos

a.. Para visitar o site do seu grupo na web, acesse:
http://br.groups.yahoo.com/group/ciencialist/

b.. Para sair deste grupo, envie um e-mail para:
ciencialist-unsubscribe@yahoogrupos.com.br

c.. O uso que você faz do Yahoo! Grupos está sujeito aos Termos do Serviço do Yahoo!.



[As partes desta mensagem que não continham texto foram removidas]



##### ##### #####

Para saber mais visite
http://www.ciencialist.hpg.ig.com.br


##### ##### ##### #####


Yahoo! Grupos, um serviço oferecido por:

São Paulo Rio de Janeiro Curitiba Porto Alegre Belo Horizonte Brasília




------------------------------------------------------------------------------
Links do Yahoo! Grupos

a.. Para visitar o site do seu grupo na web, acesse:
http://br.groups.yahoo.com/group/ciencialist/

b.. Para sair deste grupo, envie um e-mail para:
ciencialist-unsubscribe@yahoogrupos.com.br

c.. O uso que você faz do Yahoo! Grupos está sujeito aos Termos do Serviço do Yahoo!.



[As partes desta mensagem que não continham texto foram removidas]



SUBJECT: FW: [F/E-BR] (Fwd) [Keelynet] laser shoots matter waves
FROM: "murilo filo" <avalanchedrive@hotmail.com>
TO: ciencialist@yahoogrupos.com.br
DATE: 23/02/2005 17:26

Arre... esta não foi mole!
Laser que dispara ''matéria''? Boa sorte. M. SP 23/fev/2005

>From: "Mark Jordan" <mark@cpovo.net>
>Reply-To: FreeEnergy-BR@yahoogrupos.com.br
>To: freeEnergy-BR@yahoogrupos.com.br
>Subject: [F/E-BR] (Fwd) [Keelynet] laser shoots matter waves
>Date: Tue, 22 Feb 2005 22:26:59 -0300
>
>
>
>------- Forwarded message follows -------
>Hola Folks!
>
>For fans of 'TRON', remember where the computer scanned Flynn with a
>special laser to take him into its digital world, then resent him using
>a reverse technique, check this out;
>
>http://abc.net.au/science/news/stories/s20105.htm
>
>New laser emits matter waves
>
>Thursday, 18 March 1999
>
>Researchers at the Max Planck Institute for Quantum Optics and the
>University of Munich have developed a laser which emits a continuous
>beam of matter waves instead of light.
>
>The matter-wave emitting laser becomes possible by the laws of quantum
>mechanics at temperatures close to absolute zero, where atoms reveal
>their wave nature.
>
>The matter-wave laser uses a gas of rubidium atoms. The gas is captured
>in a magnetic trap then cooled to near absolute zero to reach the
>Bose-Einstein condensation. The gas atoms then lose their identity and
>behave as a single entity, a sort of super atom.
>
>Researchers then use a radiofrequency field to flip the atomic spin so
>that atoms are allowed to escape from the magnetic trap. In a vacuum the
>atoms are accelerated by gravity and form a parallel beam of coherent
>matter waves.
>
>This type of laser will make it possible to accurately deposit atoms at
>a nanoscale level. The laser could spray an accurate coating of atoms
>onto surfaces - for example - to create nanostructure miniature computer
>circuits.
>--
> Jerry Decker - http://www.keelynet.com
>Public Archive http://www.escribe.com/science/keelynet
> Order out of Chaos - From an Art to a Science
>
>
>------- End of forwarded message -------
>
>
>
>
>Links do Yahoo! Grupos
>
>
>
>
>
>
>
>




SUBJECT: O Especialista (era Extraterrestre verossímil)
FROM: "Oraculo" <oraculo@atibaia.com.br>
TO: <ciencialist@yahoogrupos.com.br>
DATE: 23/02/2005 17:48

Olá pessoal

Dentro do assunto extraterrestre verossimel, um conto fascinante, de Robert Scheckley, O Especialista.

Leiam o começo, e acessem o link para ler o resto (não copiei e colei inteiro por ser meio longo para um email).

Um abraço.

Homero
_________________________________
http://www.softaplic.com.br/~edesio/myself/hobbies/impulse.html

O Especialista

A tempestade de fótons despencou sem um aviso, precipitando-se sôbre a Espaçonave, vinha de trás de um aglomerado de estrêlas vermelhas gigantescas. Ôlho mal teve tempo para enviar um aviso de última hora através de Falante antes que a tempestade os atingisse.

Era a terceira viagem de Falante no espaço profundo e sua primeira tempestade de pressão de luz. Sentiu um calafrio quando a Nave desviou-se violentamente, recebeu todo o impacto da onda frontal virou de cabeça para baixo. Depois o mêdo desapareceu, sendo substituído por uma forte pulsação de excitamento.

"Por que teria mêdo", perguntou a si mesmo... "não fôra treinado justamente para uma emergência desta ordem?"

Estava conversando com Alimentador quando a tempestade desabou e a conversa foi interrompida bruscamente. Esperava que Alimentador estivesse bem. Era a primeira viagem no espaço profundo do jovem.

Os filamentos que constituíam a maior parte do corpo de Falante estendiam-se por tôda a Nave. Rapidamente êle os recolheu todos, com exceção dos que o ligavam a Ôlho, Motor e Paredes. Agora o trabalho era exclusivamente dêles. O restante da Tripulação deveria contar consigo mesmo até que a tempestade passasse.

Ôlho achatara seu corpo em forma de disco contra uma Parede e tinha um órgão de visão do lado de fora da Nave. Para maior concentração, o restante dos seus órgãos visuais tinha sido retraído, reunido a seu corpo.

Através do órgão visual de Ôlho, Falante observava a tempestade. Traduzia a imagem puramente visual de Ôlho na direção de Motor, que dirigia a Nave ao encontro das ondas. Quase ao mesmo tempo, Falante traduzia a direção em velocidade para as Paredes que se enrijeciam para resistir aos choques.

A coordenação era rápida e segura: Ôlho medindo as ondas, Falante transmitindo as mensagens para Motor e Paredes, Motor dirigindo a Nave de frente ao encontro das ondas, e as Paredes se enrijecendo para enfrentar os choques.

Falante esqueceu todo o mêdo que podia sentir tal a rapidez de operação da equipe. Não tinha tempo para pensar. Enquanto fôsse o sistema de comunicação da Nave, deveria traduzir e expedir suas mensagens com a máxima rapidez, coordenando as informações e dirigindo a ação.

Em questão de minutos a tempestade cessou.

-- Bem -- disse Falante -- vamos ver se houve alguns danos! -- Seus filamentos haviam se embaraçado durante a tempestade; êle os desembaraçou e os estendeu ao longo da Nave, ligando os outros em circuito. -- Motor?

-- Estou bem -- respondeu Motor. O fantástico camarada umedecera suas placas durante a tempestade, diminuindo assim as explosões atômicas no seu estômago. Nenhuma tempestade apanharia de surprêsa um astronauta experiente como Motor.

-- Paredes.

As Paredes responderam uma por uma, o que levou bastante tempo. Eram quase mil criaturas magras e retangulares, constituindo tôda a superfície da Nave. Naturalmente haviam reforçado sua periferia durante a tempestade, dando resistência à Nave inteira. Uma ou duas, contudo, apresentavam alguns danos severos.

Doutor comunicou que estava perfeitamente bem. Removeu o filamento de Falante da sua cabeça, desligando-se do circuito, e foi atender às Paredes empipocadas. Feito quase todo de mãos, Doutor agarrara-se a um Acumulador durante a tempestade.

-- Vamos voar um pouco mais depressa agora! -- disse Falante, lembrando que havia ainda o problema de determinar onde se encontravam. Abriu o circuito dos quatro Acumuladores. -- Como estão vocês? -- perguntou.

Não houve resposta. Os Acumuladores dormiam. Haviam mantido seus receptores abertos durante a tempestade e estavam empanturrados de energia. Falante deu um puxão nos filamentos que os rodeavam, mas os Acumuladores não responderam. -- Deixe-me experimentar! -- pediu Alimentador. Alimentador passara por um mau bocado antes de plantar seus cilindros de sucção nas Paredes, mas sua "forma" estava intacta. Era o único elemento da tripulação que nunca necessitava dos cuidados do Doutor; seu corpo era capaz de se reparar a si mesmo.



[As partes desta mensagem que não continham texto foram removidas]



SUBJECT: Re: Antineutrino
FROM: Manuel Bulcão <manuelbulcao@uol.com.br>
TO: ciencialist@yahoogrupos.com.br
DATE: 23/02/2005 22:46


Oi Renato,

> Dúvida: se a antipartícula tem o mesmo spin e carga igual mas de
sinal contrário, no caso do nêutron - existentes nos núcleos
atômicos - e da partícula neutrino, elas não seriam exatamente
iguais?

Manuel: Apesar do nêutron e do antinêutron coincidirem quanto à
massa, ao spin e à carga elétrica (nula), o primeiro é constituído
por dois quarks 'down' e um quark 'up', ao passo que o antinêutron é
formado por dois antiquarks 'down' e um antiquark 'up'. Por isso
que, ao colidirem, ocorre uma aniquilação mútua.

No que diz respeito aos neutrinos e antineutrinos, embora possuam
massa, carga elétrica (nula) e spin idênticos, há entre eles uma
oposição fundamental relacionada com o 'sentido' do spin. No
artigo "Neutrinos, Partículas Onipresentes e Misteriosas" --
http://cienciahoje.uol.com.br/materia/
resources/files/chmais/pass/ch147/neutrino.rtf --, Adriano A Natale
e Marcelo M. Guzzo explicam nestes termos essa contrariedade:

<<Enquanto o neutrino era 'caçado', os chineses Tsung-Dao Lee (1926-
) e Chen N. Yang (1922- ), da Universidade de Columbia, em Nova
York, propuseram uma modificação na teoria de Fermi para explicar o
comportamento de outras partículas conhecidas. A idéia também
arranhava um dos pilares da física: a simetria de paridade ou
simetria entre direita e esquerda. Essa simetria diz que, em toda
equação da física, se a coordenada de posição (que podemos indicar
por 'x') for trocada por sua imagem especular ('-x'), a equação não
se altera. A mudança, unida às idéias sobre criação e aniquilação de
partículas, permitia deduzir a existência de neutrinos e
antineutrinos e determinava que os primeiros teriam 'mão-esquerda' e
os segundos 'mão-direita'.
<<Para explicar o conceito de 'mão', é preciso falar do 'spin',
outra propriedade das partículas elementares. Em definição
simplificada, o spin é um valor que indica como a partícula gira em
torno de si própria. Se a direção do spin coincide com a do
movimento da partícula, diz-se que esta é de mão direita. Se as
direções são opostas, a mão é esquerda. Para simplificar, pode-se
ver o neutrino como um parafuso com a rosca apontando na direção
contrária ao movimento, e o antineutrino como estando na situação
oposta.>>

Abraços,
Manuel Bulcão






SUBJECT: Re: Antineutrino (corrigida)
FROM: Manuel Bulcão <manuelbulcao@uol.com.br>
TO: ciencialist@yahoogrupos.com.br
DATE: 23/02/2005 22:50


Oi Renato,

> Dúvida: se a antipartícula tem o mesmo spin e carga igual mas de
sinal contrário, no caso do nêutron - existentes nos núcleos
atômicos - e da partícula neutrino, elas não seriam exatamente
iguais?

Manuel: Apesar do nêutron e do antinêutron coincidirem quanto à
massa, ao spin e à carga elétrica (nula), o primeiro é constituído
por dois quarks 'down' e um quark 'up', ao passo que o antinêutron é
formado por dois antiquarks 'down' e um antiquark 'up'. Por isso
que, ao colidirem, ocorre uma aniquilação mútua.

No que diz respeito aos neutrinos e antineutrinos, embora possuam
massa, carga elétrica (nula) e spin idênticos, há entre eles uma
oposição fundamental relacionada com o 'sentido' do spin e o
movimento da partícula. No artigo "Neutrinos, Partículas
Onipresentes e Misteriosas" --
http://cienciahoje.uol.com.br/materia/
resources/files/chmais/pass/ch147/neutrino.rtf --, Adriano A Natale
e Marcelo M. Guzzo explicam nestes termos essa contrariedade:

<<Enquanto o neutrino era 'caçado', os chineses Tsung-Dao Lee (1926-
) e Chen N. Yang (1922- ), da Universidade de Columbia, em Nova
York, propuseram uma modificação na teoria de Fermi para explicar o
comportamento de outras partículas conhecidas. A idéia também
arranhava um dos pilares da física: a simetria de paridade ou
simetria entre direita e esquerda. Essa simetria diz que, em toda
equação da física, se a coordenada de posição (que podemos indicar
por 'x') for trocada por sua imagem especular ('-x'), a equação não
se altera. A mudança, unida às idéias sobre criação e aniquilação de
partículas, permitia deduzir a existência de neutrinos e
antineutrinos e determinava que os primeiros teriam 'mão-esquerda' e
os segundos 'mão-direita'.
<<Para explicar o conceito de 'mão', é preciso falar do 'spin',
outra propriedade das partículas elementares. Em definição
simplificada, o spin é um valor que indica como a partícula gira em
torno de si própria. Se a direção do spin coincide com a do
movimento da partícula, diz-se que esta é de mão direita. Se as
direções são opostas, a mão é esquerda. Para simplificar, pode-se
ver o neutrino como um parafuso com a rosca apontando na direção
contrária ao movimento, e o antineutrino como estando na situação
oposta.>>

Abraços,
Manuel Bulcão





SUBJECT: Fw: Viscosidade dinâmica e Cinemática
FROM: "Luiz Ferraz Netto" <leobarretos@uol.com.br>
TO: "ciencialist" <ciencialist@yahoogrupos.com.br>
DATE: 24/02/2005 19:12

quem quer opinar não faça cerimônia!

[]'
===========================
Luiz Ferraz Netto [Léo]
leobarretos@uol.com.br
http://www.feiradeciencias.com.br
===========================
-----Mensagem Original-----
De: <pereiratavares@aeiou.pt>
Para: <leobarretos@uol.com.br>
Enviada em: quarta-feira, 23 de fevereiro de 2005 18:47
Assunto: Viscosidade dinâmica e Cinemática


Caro Professor

Desejaria saber se dois óleos dos quais conheço:

Óleo Um - Viscosidade Dinâmica 1300 mPa.s (a 20ºC)
Óleo Dois - Viscosidade Cinemática 300 mm2/s (a 20ºC)

É possível estabelecer alguma relação entre eles que me permita
afirmar se as suas viscosidades sãs iguais?

Solicito a sua conceituada resposta para:
rapereira@mail.emef.pt

Cumprimentos
Rui Alves Pereira

_________________________________________________________
CEAC Cursos de formação profissional - peça informações aqui.:
http://ceac.online.pt/



--
No virus found in this incoming message.
Checked by AVG Anti-Virus.
Version: 7.0.300 / Virus Database: 266.4.0 - Release Date: 22/02/2005




--
No virus found in this outgoing message.
Checked by AVG Anti-Virus.
Version: 7.0.300 / Virus Database: 266.4.0 - Release Date: 22/02/2005



SUBJECT: Fw: acido sulfurico,metano,acido cloridrico
FROM: "Luiz Ferraz Netto" <leobarretos@uol.com.br>
TO: "ciencialist" <ciencialist@yahoogrupos.com.br>
DATE: 24/02/2005 19:17

cuimicos.............

[]'
===========================
Luiz Ferraz Netto [Léo]
leobarretos@uol.com.br
http://www.feiradeciencias.com.br
===========================
-----Mensagem Original-----
De: rachel campestrini
Para: leobarretos@uol.com.br
Enviada em: quarta-feira, 23 de fevereiro de 2005 11:11
Assunto: acido sulfurico,metano,acido cloridrico


Olá,eu me chamo Rachel,tenho 14 anos e estou na 8ª serie,eu gostaria de saber para que serve o acido sulfurico,o metano e o acido cloridrico?

Obrigada.


--------------------------------------------------------------------------------
Yahoo! Acesso Grátis - Internet rápida e grátis. Instale o discador do Yahoo! agora.


--------------------------------------------------------------------------------


No virus found in this incoming message.
Checked by AVG Anti-Virus.
Version: 7.0.300 / Virus Database: 266.4.0 - Release Date: 22/02/2005

----------

No virus found in this outgoing message.
Checked by AVG Anti-Virus.
Version: 7.0.300 / Virus Database: 266.4.0 - Release Date: 22/02/2005


[As partes desta mensagem que não continham texto foram removidas]



SUBJECT: Fw: ajuda (fisica)
FROM: "Luiz Ferraz Netto" <leobarretos@uol.com.br>
TO: "ciencialist" <ciencialist@yahoogrupos.com.br>
DATE: 24/02/2005 19:17

Será dona de uma auto-escola?

[]'
===========================
Luiz Ferraz Netto [Léo]
leobarretos@uol.com.br
http://www.feiradeciencias.com.br
===========================
-----Mensagem Original-----
De: Eliana polo
Para: leobarretos@uol.com.br
Enviada em: quarta-feira, 23 de fevereiro de 2005 09:00
Assunto: ajuda (fisica)


Ola professor preciso respnder este questionario mas nao estou conseguindo entender a linguagem dos livros, se puder me auxiliar

1- Quais são os fatores que determinam a velocidade de um automóvel?



2- Como é feita a transmissão da rotação do motor para o movimento das rodas?



3- Qual a ligação entre, a velocidade giro do motor (RPM), a potencia e velocidade do carro?



4- Como a queima do combustível produz o movimento do motor?



5- Como funciona o sistema de direção de um carro?



6- Existem sistemas de direção que exigem menor forca? Como eles funcionam?



7- Como funciona o sistema de freios de um carro?



8- Existem sistemas de freios que exigem menor forca? Como eles funcionam?



9- Quais são os fatores que determinam a estabilidade de um automóvel? Como eles funcionam?



10- Quem é o um bom motorista?





--------------------------------------------------------------------------------
Yahoo! Acesso Grátis - Internet rápida e grátis. Instale o discador do Yahoo! agora.


--------------------------------------------------------------------------------


No virus found in this incoming message.
Checked by AVG Anti-Virus.
Version: 7.0.300 / Virus Database: 266.4.0 - Release Date: 22/02/2005

----------

No virus found in this outgoing message.
Checked by AVG Anti-Virus.
Version: 7.0.300 / Virus Database: 266.4.0 - Release Date: 22/02/2005


[As partes desta mensagem que não continham texto foram removidas]



SUBJECT: Fw: Pesquisa
FROM: "Luiz Ferraz Netto" <leobarretos@uol.com.br>
TO: "ciencialist" <ciencialist@yahoogrupos.com.br>
DATE: 24/02/2005 20:01

Profunda pesquisa .......

[]'
===========================
Luiz Ferraz Netto [Léo]
leobarretos@uol.com.br
http://www.feiradeciencias.com.br
===========================
-----Mensagem Original-----
De: Gisele
Para: leobarretos@uol.com.br
Enviada em: terça-feira, 22 de fevereiro de 2005 08:56
Assunto: Pesquisa



Nas dúvidas experimentais, por gentileza coloque aqui o endereço da página, isso facilita o confronto. Agradeço. Meu nome é LUIZ FERRAZ NETTO, meu apelido é LÉO e moro em BARRETOS; dai vem meu e-mail: leobarretos@uol.com.br.


Sou nova na Faculdade no RJ, gostaria que me ajudase a fazer um trabalho.

citar alguns fenomenos que são estudados em cada um dos seguintes ramos.

macanica
calor
optica
movimento ondulatorio
eletricidade
fisica moderna


--------------------------------------------------------------------------------


No virus found in this incoming message.
Checked by AVG Anti-Virus.
Version: 7.0.300 / Virus Database: 266.4.0 - Release Date: 22/02/2005

----------

No virus found in this outgoing message.
Checked by AVG Anti-Virus.
Version: 7.0.300 / Virus Database: 266.4.0 - Release Date: 22/02/2005


[As partes desta mensagem que não continham texto foram removidas]



SUBJECT: Fw: intensidade luminosa
FROM: "Luiz Ferraz Netto" <leobarretos@uol.com.br>
TO: "ciencialist" <ciencialist@yahoogrupos.com.br>
DATE: 24/02/2005 20:02

Quem é bom de luz por aqui ..........
[]'
===========================
Luiz Ferraz Netto [Léo]
leobarretos@uol.com.br
http://www.feiradeciencias.com.br
===========================
-----Mensagem Original-----
De: Alessandro de Souza
Para: leobarretos@uol.com.br
Cc: fleaarm@pop.com.br
Enviada em: terça-feira, 22 de fevereiro de 2005 09:46
Assunto: intensidade luminosa


bom dia ,gostaria de maiores esclarecimentos sobre o assunto.
desde já agradeço a atenção.
alexandre rafael

----------

No virus found in this outgoing message.
Checked by AVG Anti-Virus.
Version: 7.0.300 / Virus Database: 266.4.0 - Release Date: 22/02/2005


[As partes desta mensagem que não continham texto foram removidas]



SUBJECT: Fw: Contamina��o dos solos
FROM: "Luiz Ferraz Netto" <leobarretos@uol.com.br>
TO: "ciencialist" <ciencialist@yahoogrupos.com.br>
DATE: 24/02/2005 20:04

Essa é para o pessoal das biologias .........
[]'
===========================
Luiz Ferraz Netto [Léo]
leobarretos@uol.com.br
http://www.feiradeciencias.com.br
===========================
-----Mensagem Original-----
De: Luiza Paiva Ferrari
Para: leobarretos@uol.com.br ; daivemmeue-mailleobarretos@uol.com.br
Enviada em: segunda-feira, 21 de fevereiro de 2005 18:18
Assunto: Contaminação dos solos


Olá, Prof. Léo!
Resido em Petrópolis-RJ. Na cidade há um rio ( o Palatinato) onde vivem centenas de gatos abandonados, em um banco de areia.. Estou escrevendo uma monografia, abordando este assunto, como conclusão de um Curso de Pós-graduação em Perícia Ambiental.
Pergunto, em sua opinião: Este solo contaminado por degetos, pêlos e animais doentes e/ ou eventualmente mortos poderá transmitir doenças para as pessoas através da água do rio? ( O rio faz parte de uma importante bacia hidrográfica: Bacia do Piabanha ). Os vetores, em contato com este solo, podem ser transmissores? É possível algum tipo de contaminação do lençol freático da região do entorno?
A monografia está bem adiantada. Caso lhe interesse, posso enviar parte, para uma maior compreensão do assunto.
Agradeço antecipadamente pela atenção.
Cordialmente
Luiza Ferrari





--------------------------------------------------------------------------------


No virus found in this incoming message.
Checked by AVG Anti-Virus.
Version: 7.0.300 / Virus Database: 266.4.0 - Release Date: 22/02/2005



--------------------------------------------------------------------------------


No virus found in this incoming message.
Checked by AVG Anti-Virus.
Version: 7.0.300 / Virus Database: 266.4.0 - Release Date: 22/02/2005

----------

No virus found in this outgoing message.
Checked by AVG Anti-Virus.
Version: 7.0.300 / Virus Database: 266.4.0 - Release Date: 22/02/2005


[As partes desta mensagem que não continham texto foram removidas]



SUBJECT: ASTRONOMIA e ASTROFÍSICA para todos*
FROM: Maria Natália <grasdic@hotmail.com>
TO: ciencialist@yahoogrupos.com.br
DATE: 25/02/2005 01:26


1--Já pensou como será feita a colonização do Espaço? Só na FC? Aqui têm:
http://tinyurl.com/5agv5

2--E umas férias de luxo? Num lugar distante:
http://tinyurl.com/4pk2g

3--Uma fotos FANTÁSTICAS:
http://www.livescience.com/php/multimedia/imagegallery/

4--E o que há de mais IN em Astrofísica e céu profundo:
a)ESA'S COMET CHASER TO FLY BY EARTH, February 23
ESA's comet-chaser Rosetta will make a fly-by of planet Earth on 4
March 2005, and sky watchers should be able to see it with telescopes or
binoculars if the sky is clear! Rosetta is approaching Earth from an area
in the sky between the constellations Leo and Sextans, and should first
become visible to large amateur telescopes around 26 February 2005.
Full story at http://www.physorg.com/news3164.html

b)SCIENTISTS DISCOVER WHY IS THE NORTH POLE FROZEN, February 23
Ice has been building up in the Arctic for 2.7 million years. Until
now, no-one has been able to prove what mechanism brought about this
accumulation of ice. However, a team of international scientists led by
Antoni Rosell, a researcher for the Universitat Autònoma de Barcelona,
and
Gerald H. Haug of the Potsdam Institute for Climate Impact Research
(Germany) has discovered the mechanism that set off the accumulation of
ice.
Full story at http://www.physorg.com/news3161.html

c)NASA SATELLITE OBSERVES MYSTERIOUS EARTH ENERGY, February 23
Scientists using observations from NASA's Reuven Ramaty High Energy
Solar Spectroscopic Imager (RHESSI) satellite detected flashes of gamma
ray energy in Earth's upper atmosphere in greater detail than ever
before. RHESSI is part of NASA's Sun-Earth Connection program. It was
designed to study X-rays and gamma rays from solar flares. However,
RHESSI's
detectors pick up gamma rays from a variety of sources.
Full story at http://www.physorg.com/news3160.html

c)BLACK HOLES IN A RADAR TRAP, February 23
European astronomers succeeded for the first time to confirm the
signatures predicted near Black Holes by Albert Einstein's theory of
Relativity in the light of the cosmic X-ray background. The group of
scientists
led by Günther Hasinger, director at the Max-Planck-Institute for
extraterrestrial Physics in Garching near Munich could identify the
spectral
fingerprint of iron atoms. They observed a strong, relativistically
smeared iron line in the average spectrum of roughly 100 active galaxies,
whose X-ray light had been emitted when the Universe was less than half
of its current age (Astronomy & Astrophysics, vol. 432(2), March 2005).
Full story at http://www.physorg.com/news3157.html

d)SEEING THE INVISIBLE – FIRST DARK GALAXY DISCOVERED?, February 23
A British-led team of astronomers have discovered an object that
appears to be an invisible galaxy made almost entirely of dark matter
– the
first ever detected. A dark galaxy is an area in the universe containing
a large amount of mass that rotates like a galaxy, but contains no
stars. Without any stars to give light, it could only be found using
radio
telescopes. It was first seen with the University of Manchester's
Lovell Telescope in Cheshire, and the sighting was confirmed with the
Arecibo telescope in Puerto Rico. The unknown material that is thought to
hold these galaxies together is known as `dark matter', but scientists
still know very little about what that is.
Full story at http://www.physorg.com/news3154.html
5--Vistas de Titã a partir da Terra:
http://tinyurl.com/6bfel
E ainda: Parece que Titã é geologicamente activo, na Newscientist:
http://www.newscientist.com/article.ns?id=dn7043
e ainda em:
http://www.spaceref.com/news/viewpr.html?pid=16193
6--Respondendo a questões sobre asteróides. Haverá um impacto na
Terra? Entrevista a quem sabe e as respostas em:
http://tinyurl.com/65wtr

PARABÉNS ! Coinseguiu ler tudo! Amanhã lhe dou um coração.

E de Astronomia e Astrofísica aqui têm para uma semana. Não está dando
para traduzir mas vou colocar sites em português

AH QUER MAIS? Então porque não se inscreve na lista do GRASDIC de
Maria Natália?
http://uk.groups.yahoo.com/group/grasdic/
Como já vi aqui notícia a outra lista... (Vais-me bater Brudna?)
Um abraço
Maria Natália
* Excepto Leo pois vai muita língua estrangeira. rs rs





SUBJECT: ESPECIAL para Prof e Mestre Leo!!!!!!!!
FROM: Maria Natália <grasdic@hotmail.com>
TO: ciencialist@yahoogrupos.com.br
DATE: 25/02/2005 01:38


E aqui em Portugal:
""Se tens entre 6 e 12 anos, o @rrobinha desafia-te a passar as férias
da Páscoa com a Ciência, participando em fantásticas aventuras. Vem
fazer de pirata e navegar em vários oceanos até descobrires a arca do
tesouro. Podes também saber quanto tempo tem o tempo, enquanto
constróis um calendário eterno! Vem descobrir o que os dinossauros têm
a ver com a Matemática. E já agora, vem reconstituir um!Outros
desafios que te esperam nestas férias são fazer a tua própria banda
desenhada com o @rrobinha e participar de uma prova: o jogo do
@rrobinha. Queres saber o que é? Vem ter connosco ao Pavilhão do
Conhecimento - Ciência Viva!Datas / Preço:21 a 24 de Março - Preço:
140 Euros (sócios: 110 Euros)28 de Março a 1 de Abril - Preço: 160
Euros (sócios: 130 Euros)Preço por dia: 40 Euros (sócios: 35 Euros) -
sujeito a confirmação do Pavilhão quanto à data Horário:Das 9h00 às
18h00Idade: dos 6 aos 12 anosPara mais informações e marcações:Tel.
218 917 100 Email: info@pavconhecimento.ptwww.pavconhecimento.pt
PROGRAMA9.00 - 9.30 Recepção das crianças
9.30 - 10.00 Briefing / lanche (trazem de casa)
10.00 - 12.00 Actividades
12.00 - 13.00 Almoço (fornecido pelo Pavilhão)
13.00 - 16.00 Actividades
16.00 - 16.30 "
Lanche (fornecido pelo Pavilhão)
16.30 - 18.00 Actividades
18.00 Entrega aos familiares""

Estou desculpada das notícias em lingua de bárbaros?
Um abraço
Maria Natália





SUBJECT: Re: Os Canindés
FROM: Maria Natália <grasdic@hotmail.com>
TO: ciencialist@yahoogrupos.com.br
DATE: 25/02/2005 01:52


Me dá licença, seu Brudna?

Pois o amigo escolheu bem. Caiu numa comunidade de irmãos de sangue no
Senhor Dono de toda a hesitação e desconhecimento que é a Ciência
nossa que está na Terra mas se manifesta desde o BB* e com toda a sua
ética e tolerância pelo Cosmos e seus anjos montados em pedras e
demais orixás.
Entrai pois irmão que a sabedoria é basta.
Maria Natália, a Irmã Porteira de serviço esta noite.
* Não esse seu i....!! Ms o Big Bang

--- Em ciencialist@yahoogrupos.com.br, "Dolarhide dos Anjos"
<vurutia@y...> escreveu
>
> Sou um cara que quer viver plenamente, amo a sabedoria, a natureza, a
> liberdade, poesia, filosofia, literatura, e tocar em uma banda de
> rock faz parte de minha plenitude. Sou do Ceará, moro em Canindé, a
> terra de Bastião raposa e do Cocote e o Messias,são pessoas bastante
> conhecidas e amadas. Sei falar Inglês, se alguém quiser praticar
> comigo, entre em contato. My life is full of happiness and madness
> coz I want it to be.





SUBJECT: Re: Fw: acido sulfurico,metano,acido cloridrico
FROM: Maria Natália <grasdic@hotmail.com>
TO: ciencialist@yahoogrupos.com.br
DATE: 25/02/2005 02:02


Leo:

Em assunto ...é mistura explosiva. Coitada de menina que tem professor
que não quer trabalhar...Manda a menina se informar.
Começemos:
O ácido sulfúrico é importante na indústria dos adubos e produtos para
a agricultura (sulfato de cobre, sulfato de amónio e fosfatos). mas
falta aqui um: o amoníaco. A menina não deveria querer dizer metano
mas amoníaco.
O metano pode ser usado em queima ao ser obtido como gás a partir
dos...restos das vacarias, o dito, estrume. Será o biogás.
O ácido clorídrico é muito utilizado na indústria do papel.
Claro que há mais. Mas o Emiliano e o Brudna têm de trabalhar também,
não é?
Dizei para a pequenina.
Um abraço
Maria Natália
--- Em ciencialist@yahoogrupos.com.br, "Luiz Ferraz Netto"
<leobarretos@u...> escreveu
> cuimicos.............
>
> []'
> ===========================
> Luiz Ferraz Netto [Léo]
> leobarretos@u...
> http://www.feiradeciencias.com.br
> ===========================
> -----Mensagem Original-----
> De: rachel campestrini
> Para: leobarretos@u...
> Enviada em: quarta-feira, 23 de fevereiro de 2005 11:11
> Assunto: acido sulfurico,metano,acido cloridrico
>
>
> Olá,eu me chamo Rachel,tenho 14 anos e estou na 8ª serie,eu gostaria
de saber para que serve o acido sulfurico,o metano e o acido cloridrico?
>
> Obrigada.
>
>
>
--------------------------------------------------------------------------------
> Yahoo! Acesso Grátis - Internet rápida e grátis. Instale o discador
do Yahoo! agora.
>
>
>
--------------------------------------------------------------------------------
>
>
> No virus found in this incoming message.
> Checked by AVG Anti-Virus.
> Version: 7.0.300 / Virus Database: 266.4.0 - Release Date: 22/02/2005
>
> ----------
>
> No virus found in this outgoing message.
> Checked by AVG Anti-Virus.
> Version: 7.0.300 / Virus Database: 266.4.0 - Release Date: 22/02/2005
>
>
> [As partes desta mensagem que não continham texto foram removidas]





SUBJECT: Re: Fw: ajuda (fisica)
FROM: Maria Natália <grasdic@hotmail.com>
TO: ciencialist@yahoogrupos.com.br
DATE: 25/02/2005 02:47


Trata-se apenas do questionário de exame teórico de condução em país
europeu. Senhora está fazendo cábula* para estudar.
Respostas aqui vão intercaladas pois sou encartada há 60 anos!!!!!!
Logo posso bem atropelar uns tantos.

1-Quais são os fatores que determinam a velocidade de um automóvel?

O estado da estrada: tipo de cobertura alcatrão macadame areia, brira,
caminho da cabra;
O estado dos pneus e jantes: descalibradas, carecas ou vulcanizados;
o estado de tempo: boletim meteorológico:
O tipo de estrada: auto estrada, itenerário principal, secundário,
ruas atravessando povoações
O estado do carro e motor: nível de óleo, última ida à revisão
O estado do condutor: alcoolizado, sem dormir, a atender celular, ou
com determinada companhia distractora

2- Como é feita a transmissão da rotação do motor para o movimento das
rodas?
será uma biela e Cambota?



3- Qual a ligação entre, a velocidade giro do motor (RPM), a potencia e
velocidade do carro?
RPM rotações por minuto é a frequência mas tb se pode referir á
velocidade angular em unidades nada próprias. O resto é física e deixo
aos físicos ou a um livro de física a resposta



4- Como a queima do combustível produz o movimento do motor?
A gasolina tem energia potencial química e portanto quando se quebram
as ligações que t~em essa energia armazenda e já viu nos filemes o
resultado disso, né? Pois a reacção é exoenergética e lá vai energia
na forma de gases em expansão e que empurra o êmbolo (pense numa
seringa e que está a levar uma injecção...) e tal cambota e coiso...


5- Como funciona o sistema de direção de um carro?

A direcção costuma ter um desmultiplicador e há ...mas ora que
pergunta mais de engenheiro mecânico. Obrigam as senhoras a saber
destas coisas...

6- Existem sistemas de direção que exigem menor forca? Como eles
funcionam?
Claro que existe senão nunca uma senhora poderia conduzir camião TIR

7- Como funciona o sistema de freios de um carro?

Há vários tipos. De disco... Em último caso se põe pé de fora...Por
isso senhoras usam botas aqui na UE.

8- Existem sistemas de freios que exigem menor forca? Como eles funcionam?
Sim emcarros que se destinam nomeadamente a deficientes físicos...

9- Quais são os fatores que determinam a estabilidade de um automóvel?
Como eles
funcionam?
Aqui entra a aerodinãmica de que só percebo a de foguetões espaciais.

10- Quem é o um bom motorista?

Um motorista é um ser altamente eficiente, bem remunerado, abstémio,
consciente (não pode conduzir a dormir), bem educado (só faz gestos
mágicos...)com a coluna vertebral direita e que usa bom desodorizante.
Será conveniente estar em dia com as cotações da bolsa, o último
número do Sun e as casas mais recomendáveis para cavalheiros de posição.
Esta foi a mais fácil pois calhou-me no meu exame para tractorista de
caminho de cabras.

E foi gostoso estar aqui a recordar a altura em que "tirei"** a minha
carteira de profissional "pesados"
Uma abraço professor Leo.
Maria Natália
* cábula aquele papelinho pequenino que se leva dentro do soutien (e
se vai de decote para impressiionar examinador...que examina) para
recordar umas coisa no exame e enquanto professor vigilante lê o jornal
**A quem é que tieu/roubei eu a carteira? Já nem lembro




--- Em ciencialist@yahoogrupos.com.br, "Luiz Ferraz Netto"
<leobarretos@u...> escreveu
> Será dona de uma auto-escola?
>
> []'
> ===========================
> Luiz Ferraz Netto [Léo]
> leobarretos@u...
> http://www.feiradeciencias.com.br
> ===========================
> -----Mensagem Original-----
> De: Eliana polo
> Para: leobarretos@u...
> Enviada em: quarta-feira, 23 de fevereiro de 2005 09:00
> Assunto: ajuda (fisica)
>
>
> Ola professor preciso respnder este questionario mas nao estou
conseguindo entender a linguagem dos livros, se puder me auxiliar
>
> 1- Quais são os fatores que determinam a velocidade de um automóvel?
>
>
>
> 2- Como é feita a transmissão da rotação do motor para o movimento
das rodas?
>
>
>
> 3- Qual a ligação entre, a velocidade giro do motor (RPM), a
potencia e velocidade do carro?
>
>
>
> 4- Como a queima do combustível produz o movimento do motor?
>
>
>
> 5- Como funciona o sistema de direção de um carro?
>
>
>
> 6- Existem sistemas de direção que exigem menor forca? Como eles
funcionam?
>
>
>
> 7- Como funciona o sistema de freios de um carro?
>
>
>
> 8- Existem sistemas de freios que exigem menor forca? Como eles
funcionam?
>
>
>
> 9- Quais são os fatores que determinam a estabilidade de um
automóvel? Como eles funcionam?
>
>
>
> 10- Quem é o um bom motorista?
>
>
>
>
>
>
--------------------------------------------------------------------------------
> Yahoo! Acesso Grátis - Internet rápida e grátis. Instale o discador
do Yahoo! agora.
>
>
>
--------------------------------------------------------------------------------
>
>
> No virus found in this incoming message.
> Checked by AVG Anti-Virus.
> Version: 7.0.300 / Virus Database: 266.4.0 - Release Date: 22/02/2005
>
> ----------
>
> No virus found in this outgoing message.
> Checked by AVG Anti-Virus.
> Version: 7.0.300 / Virus Database: 266.4.0 - Release Date: 22/02/2005
>
>
> [As partes desta mensagem que não continham texto foram removidas]





SUBJECT: Re: Fw: ajuda (fisica)
FROM: Maria Natália <grasdic@hotmail.com>
TO: ciencialist@yahoogrupos.com.br
DATE: 25/02/2005 02:52


Desculpem as gralhas Mas já não me ria há tanto tempo...E recordar o
exame de condução é viver.
Maria Natália


--- Em ciencial
ist@yahoogrupos.com.br, Maria Natália <grasdic@h...> escreveu
>
> Trata-se apenas do questionário de exame teórico de condução em país
> europeu. Senhora está fazendo cábula* para estudar.
> Respostas aqui vão intercaladas pois sou encartada há 60 anos!!!!!!
> Logo posso bem atropelar uns tantos.
>
> 1-Quais são os fatores que determinam a velocidade de um automóvel?
>
> O





SUBJECT: Re: Links interessantes
FROM: Maria Natália <grasdic@hotmail.com>
TO: ciencialist@yahoogrupos.com.br
DATE: 25/02/2005 03:15


Há que tempos não te "ouvia".

Um abraço
Maria Natália


--- Em ciencialist@yahoogrupos.com.br, "Narumi Abe" <omnivisio@g...>
escreveu
>
> Olá a todos,
>
> Encontrei dois sites muito interessantes, ambos em língua
> alienígena:
>
> Este lembra o Imperdível do Léo. Obviamente não tão completo:
> http://www.scitoys.com/
>
> Este outro simula um zoom que começa a alguns anos luz da
> Via Láctea até chegar aos quarks de uma planta na Terra.
> http://micro.magnet.fsu.edu/primer/java/scienceopticsu/powersof10/
>
> []s,
> --
> Narumi Abe





SUBJECT: Re: Fw: Pesquisa
FROM: Maria Natália <grasdic@hotmail.com>
TO: ciencialist@yahoogrupos.com.br
DATE: 25/02/2005 03:46


Tenha paciência querido professor mas esta não passa. Então a menina
fez o vestibular ainda há 30 dias e já não sabe o que aprendeu?
Puxa vida. Já me estragaram a noite. Me valha Santo Onofre
Um abraço
Maria Natália

--- Em ciencialist@yahoogrupos.com.br, "Luiz Ferraz Netto"
<leobarretos@u...> escreveu
> Profunda pesquisa .......
>
> []'
> ===========================
> Luiz Ferraz Netto [Léo]
> leobarretos@u...
> http://www.feiradeciencias.com.br
> ===========================
> -----Mensagem Original-----
> De: Gisele
> Para: leobarretos@u...
> Enviada em: terça-feira, 22 de fevereiro de 2005 08:56
> Assunto: Pesquisa
>
>
>
> Nas dúvidas experimentais, por gentileza coloque aqui o endereço da
página, isso facilita o confronto. Agradeço. Meu nome é LUIZ FERRAZ
NETTO, meu apelido é LÉO e moro em BARRETOS; dai vem meu e-mail:
leobarretos@u...
>
>
> Sou nova na Faculdade no RJ, gostaria que me ajudase a fazer um
trabalho.
>
> citar alguns fenomenos que são estudados em cada um dos seguintes ramos.
>
> macanica
> calor
> optica
> movimento ondulatorio
> eletricidade
> fisica moderna
>
>
>
--------------------------------------------------------------------------------
>
>
> No virus found in this incoming message.
> Checked by AVG Anti-Virus.
> Version: 7.0.300 / Virus Database: 266.4.0 - Release Date: 22/02/2005
>
> ----------
>
> No virus found in this outgoing message.
> Checked by AVG Anti-Virus.
> Version: 7.0.300 / Virus Database: 266.4.0 - Release Date: 22/02/2005
>
>
> [As partes desta mensagem que não continham texto foram removidas]





SUBJECT: Re: Fw: ENC: Fw: Como chamar a policia
FROM: Maria Natália <grasdic@hotmail.com>
TO: ciencialist@yahoogrupos.com.br
DATE: 25/02/2005 03:47


Está escrito em português?
Maria



--- Em ciencialist@yahoogrupos.com.br, "Paulo Henrique Lerbach
Rodrigues" <phatleta@y...> escreveu
>
>
> Paulo Henrique Lerbach Rodrigues
>
> Jesus Cristo é o Único que pode te levar para o céu, não se
engane, conheça-O!
>
> Futuro Piloto Privado de Avião
> (Já solei o Zarapa galerinha)
>
> lerbach@b...
>
> 55 61 354 2197 ou 55 61 9975 9188
>
>
>
>
> ----- Original Message -----
> From: André Melo
> To: comrec@t...
> Sent: Wednesday, February 23, 2005 2:51 PM
> Subject: FW: ENC: Fw: Como chamar a policia
>
>
>
>
>
>
> >From: jvdaniel <jvdaniel@i...>
> >To: riconayy@i...
> >CC:
jaquerev@b...,ajulia@p...,daniamanda@h...,comandanteandre@h...,bamvieira@h...,graca_carvalho@b...,monica.lu.carvalho@g...

> >Subject: ENC: Fw: Como chamar a policia
> >Date: Thu, 17 Feb 2005 14:09:16 -0200
> >
> >
> >
> >
> > > >Subject: Como chamar a policia
> > > >Date: Mon, 7 Feb 2005 13:43:21 -0300
> > > >
> > > >Guia pratico
> > > > Eu tenho o sono muito leve, e numa noite dessas notei que
havia algu?m
> > > >andando sorrateiramente no quintal de casa.
> > > > Levantei em sil?ncio e fiquei acompanhando os leves ru?dos
que vinham
> >l?
> > > >de fora, at? ver uma silhueta passando pela janela do banheiro.
> > > > Como minha casa ? muito segura, com grades nas janelas e
trancas
> > > >internas
> > > >nas portas, n?o fiquei muito preocupado, mas ? claro que eu n?o
ia deixar
> > > >um
> > > >ladr?o ali, espiando tranq?ilamente.
> > > > Liguei baixinho para a pol?cia informei a situa??o e o meu
endere?o.
> > > > Perguntaram-me se o ladr?o estava armado ou se j? estava no
interior
> >da
> > > >casa.
> > > > Esclareci que n?o e disseram-me que n?o havia nenhuma
viatura por
> >perto
> > > >para ajudar, mas que iriam mandar algu?m assim que fosse poss?vel.
> > > > Um minuto depois liguei de novo, e disse com a voz calma:
> > > > - Ol?, eu liguei h? pouco porque tinha algu?m no meu
quintal. N?o
> > > >precisa
> > > >mais ter pressa. Eu j? matei o ladr?o com um tiro da escopeta
calibre 12
> > > >que
> > > >tenho guardada em casa para estas situa??es. Putz, o tiro fez um
> > > > estrago danado no cara!
> > > > Passados menos de tr?s minutos, estavam na minha rua cinco
carros da
> > > >pol?cia, um helic?ptero, uma unidade do resgate e uma equipe de
TV. Eles
> > > >prenderam o ladr?o em flagrante, que ficava olhando tudo com
cara de
> > > >assombrado. Talvez ele estivesse pensando que aquela era a casa do
> > > >Comandante da Pol?cia. No meio do tumulto, um tenente se
aproximou de mim
> >e
> > > >disse:
> > > > - Pensei que tivesse dito que tinha matado o ladr?o.
> > > > Eu respondi:
> > > > - Pensei que tivesse dito que n?o havia ningu?m dispon?vel...
> > > >
> > > >
> > >
> > > _________________________________________________________________
> > > Chegou o que faltava: MSN Acesso Gr?tis. Instale J?!
> > > http://www.msn.com.br/discador
> > >
> > >
> > >
> > >
> > > --
> > > Internal Virus Database is out-of-date.
> > > Checked by AVG Anti-Virus.
> > > Version: 7.0.300 / Virus Database: 265.8.5 - Release Date: 3/2/2005
> > >
> > >
> >
> >
> >
> >--
> >No virus found in this outgoing message.
> >Checked by AVG Anti-Virus.
> >Version: 7.0.300 / Virus Database: 265.8.8 - Release Date: 14/2/2005
>
>
>
--------------------------------------------------------------------------------
> MSN Busca: f?cil, r?pido, direto ao ponto. Encontre o que voc?
quiser. Clique aqui.
>
> [As partes desta mensagem que não continham texto foram removidas]





SUBJECT: Re: O Especialista (era Extraterrestre verossímil)
FROM: "Rodrigo Marques" <rodmarq72@yahoo.com.br>
TO: ciencialist@yahoogrupos.com.br
DATE: 25/02/2005 07:21


Gostei muito deste conto, onde posso encontrar outros (em português)?
--- Em ciencialist@yahoogrupos.com.br, "Oraculo" <oraculo@a...>
escreveu
> Olá pessoal
>
> Dentro do assunto extraterrestre verossimel, um conto fascinante,
de Robert Scheckley, O Especialista.
>
> Leiam o começo, e acessem o link para ler o resto (não copiei e
colei inteiro por ser meio longo para um email).
>
> Um abraço.
>
> Homero
> _________________________________
> http://www.softaplic.com.br/~edesio/myself/hobbies/impulse.html
>
> O Especialista
>
> A tempestade de fótons despencou sem um aviso, precipitando-se
sôbre a Espaçonave, vinha de trás de um aglomerado de estrêlas
vermelhas gigantescas. Ôlho mal teve tempo para enviar um aviso de
última hora através de Falante antes que a tempestade os atingisse.
>
> Era a terceira viagem de Falante no espaço profundo e sua primeira
tempestade de pressão de luz. Sentiu um calafrio quando a Nave
desviou-se violentamente, recebeu todo o impacto da onda frontal
virou de cabeça para baixo. Depois o mêdo desapareceu, sendo
substituído por uma forte pulsação de excitamento.
>
> "Por que teria mêdo", perguntou a si mesmo... "não fôra treinado
justamente para uma emergência desta ordem?"
>
> Estava conversando com Alimentador quando a tempestade desabou e a
conversa foi interrompida bruscamente. Esperava que Alimentador
estivesse bem. Era a primeira viagem no espaço profundo do jovem.
>
> Os filamentos que constituíam a maior parte do corpo de Falante
estendiam-se por tôda a Nave. Rapidamente êle os recolheu todos, com
exceção dos que o ligavam a Ôlho, Motor e Paredes. Agora o trabalho
era exclusivamente dêles. O restante da Tripulação deveria contar
consigo mesmo até que a tempestade passasse.
>
> Ôlho achatara seu corpo em forma de disco contra uma Parede e
tinha um órgão de visão do lado de fora da Nave. Para maior
concentração, o restante dos seus órgãos visuais tinha sido
retraído, reunido a seu corpo.
>
> Através do órgão visual de Ôlho, Falante observava a tempestade.
Traduzia a imagem puramente visual de Ôlho na direção de Motor, que
dirigia a Nave ao encontro das ondas. Quase ao mesmo tempo, Falante
traduzia a direção em velocidade para as Paredes que se enrijeciam
para resistir aos choques.
>
> A coordenação era rápida e segura: Ôlho medindo as ondas, Falante
transmitindo as mensagens para Motor e Paredes, Motor dirigindo a
Nave de frente ao encontro das ondas, e as Paredes se enrijecendo
para enfrentar os choques.
>
> Falante esqueceu todo o mêdo que podia sentir tal a rapidez de
operação da equipe. Não tinha tempo para pensar. Enquanto fôsse o
sistema de comunicação da Nave, deveria traduzir e expedir suas
mensagens com a máxima rapidez, coordenando as informações e
dirigindo a ação.
>
> Em questão de minutos a tempestade cessou.
>
> -- Bem -- disse Falante -- vamos ver se houve alguns danos! --
Seus filamentos haviam se embaraçado durante a tempestade; êle os
desembaraçou e os estendeu ao longo da Nave, ligando os outros em
circuito. -- Motor?
>
> -- Estou bem -- respondeu Motor. O fantástico camarada umedecera
suas placas durante a tempestade, diminuindo assim as explosões
atômicas no seu estômago. Nenhuma tempestade apanharia de surprêsa
um astronauta experiente como Motor.
>
> -- Paredes.
>
> As Paredes responderam uma por uma, o que levou bastante tempo.
Eram quase mil criaturas magras e retangulares, constituindo tôda a
superfície da Nave. Naturalmente haviam reforçado sua periferia
durante a tempestade, dando resistência à Nave inteira. Uma ou duas,
contudo, apresentavam alguns danos severos.
>
> Doutor comunicou que estava perfeitamente bem. Removeu o filamento
de Falante da sua cabeça, desligando-se do circuito, e foi atender
às Paredes empipocadas. Feito quase todo de mãos, Doutor agarrara-se
a um Acumulador durante a tempestade.
>
> -- Vamos voar um pouco mais depressa agora! -- disse Falante,
lembrando que havia ainda o problema de determinar onde se
encontravam. Abriu o circuito dos quatro Acumuladores. -- Como estão
vocês? -- perguntou.
>
> Não houve resposta. Os Acumuladores dormiam. Haviam mantido seus
receptores abertos durante a tempestade e estavam empanturrados de
energia. Falante deu um puxão nos filamentos que os rodeavam, mas os
Acumuladores não responderam. -- Deixe-me experimentar! -- pediu
Alimentador. Alimentador passara por um mau bocado antes de plantar
seus cilindros de sucção nas Paredes, mas sua "forma" estava
intacta. Era o único elemento da tripulação que nunca necessitava
dos cuidados do Doutor; seu corpo era capaz de se reparar a si mesmo.
>
>
>
> [As partes desta mensagem que não continham texto foram removidas]





SUBJECT: Fw: [naeq-ucs] C pri. sec. terc. quart.
FROM: "E m i l i a n o C h e m e l l o" <chemelloe@yahoo.com.br>
TO: <ciencialist@yahoogrupos.com.br>
DATE: 25/02/2005 08:43

primário, secundário... terciário... quaternário! Eterna dúvida.

[ ] 's do Emiliano Chemello
emiliano@quimica.net
http://www.quimica.net/emiliano
http://www.ucs.br/ccet/defq/naeq

" Rien ne se perd, rien ne se crée,
tout se transforme."

Antoine Laurent de Lavoisier (químico francês, 1743 - 1794)

----- Original Message -----
From: orley_alves
To: naeq-ucs@yahoogrupos.com.br
Sent: Thursday, February 24, 2005 6:26 PM
Subject: [naeq-ucs] Dúvida



Prof. Emiliano, surgiu uma polêmica entre alguns professores aqui de
BH, sobre a classificação do carbono em algumas cadeias. A dúvida é
sobre o carbono primário.Por exemplo, na estrutura da uréia (NH2)2CO,
o carbono poderia ser classificado como primário?
Segundo alguns professores e alguns autores poderia, pois os mesmos
definem como primário o carbono ligado a um ou a nenhum carbono.
Porém oconceito que encontrei na maioria da literatura é: primário é
aquele carbono ligado a um outro carbono.
Penso que esta classificação se refere a carbonos ligados entre si na
cadeia, se não há carbonos ligados entre si, como na uréia, então não
há sentido em classificar este carbono desta forma.
Qual a opinião do prof. sobre este assunto?
Um abraço




Yahoo! Grupos, um serviço oferecido por:







--------------------------------------------------------------------------------
Links do Yahoo! Grupos

a.. Para visitar o site do seu grupo na web, acesse:
http://br.groups.yahoo.com/group/naeq-ucs/

b.. Para sair deste grupo, envie um e-mail para:
naeq-ucs-unsubscribe@yahoogrupos.com.br

c.. O uso que você faz do Yahoo! Grupos está sujeito aos Termos do Serviço do Yahoo!.



[As partes desta mensagem que não continham texto foram removidas]



SUBJECT: Fw: massa de vidraceiro
FROM: "Luiz Ferraz Netto" <leobarretos@uol.com.br>
TO: "ciencialist" <ciencialist@yahoogrupos.com.br>
DATE: 25/02/2005 09:17

Quem sabe?
[]''
===========================
Luiz Ferraz Netto [Léo]
leobarretos@uol.com.br
http://www.feiradeciencias.com.br
===========================
-----Mensagem Original-----
De: "mauricio de faria corrêa stanley" <mauriciostanley@hotmail.com>
Para: <leobarretos@uol.com.br>
Enviada em: terça-feira, 22 de fevereiro de 2005 17:18
Assunto: massa de vidraceiro


| Leo:
| Boa tarde você poderia me dar uma orientação de como fazer uma massa de
| vidraceiro pois preciso colocar em prática na sala de aula?
| Desde já muito obrigado.
| Stanley
|
| _________________________________________________________________
| MSN Messenger: converse online com seus amigos .
| http://messenger.msn.com.br
|
|
|
| --
| No virus found in this incoming message.
| Checked by AVG Anti-Virus.
| Version: 7.0.300 / Virus Database: 266.4.0 - Release Date: 22/02/2005
|
|


--
No virus found in this outgoing message.
Checked by AVG Anti-Virus.
Version: 7.0.300 / Virus Database: 266.4.0 - Release Date: 22/02/2005



SUBJECT: Fw: feiradeciencias c�pia de material
FROM: "Luiz Ferraz Netto" <leobarretos@uol.com.br>
TO: "ciencialist" <ciencialist@yahoogrupos.com.br>
DATE: 25/02/2005 09:20

BLZ!!!!!!

[]'
===========================
Luiz Ferraz Netto [Léo]
leobarretos@uol.com.br
http://www.feiradeciencias.com.br
===========================
-----Mensagem Original-----
De: geraldo londe
Para: leobarretos@uol.com.br
Enviada em: quinta-feira, 24 de fevereiro de 2005 21:04
Assunto: feiradeciencias cópia de material


Encontrei seu material na internet, ele é ótimo; é perfeito para minha aulas no curso técnico de patologia clínica, que utiliza espectroscopia.

Gostaria de sua permissão para reproduzir seu material e distribui-lo para os aluno, mantendo é claro sua autoria e endereço na internet (conforme exemplo abaixo).

posso contar com sua colaboração?

Antecipadamente obrigado


O átomo de Bohr

Prof. Luiz Ferraz Netto
leobarretos@uol.com.br

http://www.feiradeciencias.com.br/sala23/23_MA01.asp
Teoria de Bohr para o átomo de hidrogênio
Segundo o que foi......................


--------------------------------------------------------------------------------
Yahoo! Acesso Grátis - Internet rápida e grátis. Instale o discador do Yahoo! agora.


--------------------------------------------------------------------------------


No virus found in this incoming message.
Checked by AVG Anti-Virus.
Version: 7.0.300 / Virus Database: 266.4.0 - Release Date: 22/02/2005

----------

No virus found in this outgoing message.
Checked by AVG Anti-Virus.
Version: 7.0.300 / Virus Database: 266.4.0 - Release Date: 22/02/2005


[As partes desta mensagem que não continham texto foram removidas]



SUBJECT: Fw: Informações e dúvidas
FROM: "Luiz Ferraz Netto" <leobarretos@uol.com.br>
TO: "ciencialist" <ciencialist@yahoogrupos.com.br>
DATE: 25/02/2005 09:20

Alguém sabe o nome do dito cujo?
[]'
===========================
Luiz Ferraz Netto [Léo]
leobarretos@uol.com.br
http://www.feiradeciencias.com.br
===========================
-----Mensagem Original-----
De: <lulaehrlich@globo.com>
Para: <leobarretos@uol.com.br>
Enviada em: quinta-feira, 24 de fevereiro de 2005 21:33
Assunto: Informações e dúvidas


Olá Professor Luiz Ferraz Netto. Ouvi dizer a tempo atrás que um músico
queria provar que o silêncio não existe e fez uma experiência entrando dentro
de uma câmara a prova de som.
Gostaria de saber quem foi esse músico, como chamava a câmara e onde posso
obter figuras e imformações sobre essa experiência.
Muito grato desde já
Grande abraço
Lula Ehrlich



--
No virus found in this incoming message.
Checked by AVG Anti-Virus.
Version: 7.0.300 / Virus Database: 266.4.0 - Release Date: 22/02/2005




--
No virus found in this outgoing message.
Checked by AVG Anti-Virus.
Version: 7.0.300 / Virus Database: 266.4.0 - Release Date: 22/02/2005



SUBJECT: Re: Links interessantes
FROM: "Narumi Abe" <omnivisio@gmail.com>
TO: ciencialist@yahoogrupos.com.br
DATE: 25/02/2005 09:53


Querida e Jovial Natália,

--- Em ciencialist@yahoogrupos.com.br, Maria Natália <grasdic@h...>
escreveu
>
> Há que tempos não te "ouvia".

Ando mesmo muito ocupado por cá. Tentarei participar mais da
lista.


> Um abraço

Abraços para você também,

--
Narumi Abe





SUBJECT: Re: Fw: ajuda (fisica)
FROM: Hélio Ricardo Carvalho <hrc@fis.puc-rio.br>
TO: ciencialist@yahoogrupos.com.br
DATE: 25/02/2005 10:27


Léo,

Isto está mais parecido com algum tipo de teste do DETRAN para tirar
carteira categoria "D".

Não sei se devemos mastigar para ela.

Responder não é difícil. O complicado é responder do jeito que o
DETRAN quer, com as mesmas palavras que estão no livrinho específico
e que deveria ter sido decorado.
:-(

[ ]'s
Hélio


--- Em ciencialist@yahoogrupos.com.br, "Luiz Ferraz Netto"
<leobarretos@u...> escreveu
> Será dona de uma auto-escola?
>
> []'
> ===========================
> Luiz Ferraz Netto [Léo]
> leobarretos@u...
> http://www.feiradeciencias.com.br
> ===========================
> -----Mensagem Original-----
> De: Eliana polo
> Para: leobarretos@u...
> Enviada em: quarta-feira, 23 de fevereiro de 2005 09:00
> Assunto: ajuda (fisica)
>
>
> Ola professor preciso respnder este questionario mas nao estou
conseguindo entender a linguagem dos livros, se puder me auxiliar
>
> 1- Quais são os fatores que determinam a velocidade de um
automóvel?
>
>
>
> 2- Como é feita a transmissão da rotação do motor para o movimento
das rodas?
>
> ... ... ... ...







SUBJECT: Feira de Ciencias Permanente de Barretos
FROM: "Luiz Ferraz Netto" <leobarretos@uol.com.br>
TO: <ciencialist@yahoogrupos.com.br>
DATE: 25/02/2005 10:27

Olá pessoal,

nessa última 2a feira a boa cúpula política de Barretos (recém empossada) esteve aqui em casa para uma reunião sobre a Feira Permanente em Barretos. Foi uma tarde produtiva, agradável e de total sucesso! A coisa tá engrenada!
Tiramos umas fotos na varanda dos fundos de minha casa (e nem precisou tirar a mesa de 'snooker' do lugar) --- dê uma espiada:

http://www.feiradeciencias.com.br/css/projeto_barretos.htm

aquele abraço,
===========================
Luiz Ferraz Netto [Léo]
leobarretos@uol.com.br
http://www.feiradeciencias.com.br
===========================


--
No virus found in this outgoing message.
Checked by AVG Anti-Virus.
Version: 7.0.300 / Virus Database: 266.4.0 - Release Date: 22/02/2005



SUBJECT: Re: [ciencialist] Feira de Ciencias Permanente de Barretos
FROM: "E m i l i a n o C h e m e l l o" <chemelloe@yahoo.com.br>
TO: <ciencialist@yahoogrupos.com.br>
DATE: 25/02/2005 10:55

Léo,

Não foi possível acessar a página indicada. Será que devo acessá-la quanticamente? Estará ela está em um vácuo quântico e não pressionei o botão do mouse com a energia correta para tornar as partículas do site reais? Ou então, mais provável. Será que você errou o endereço? :-)

[ ] 's do Emiliano Chemello
emiliano@quimica.net
http://www.quimica.net/emiliano
http://www.ucs.br/ccet/defq/naeq

" Rien ne se perd, rien ne se crée,
tout se transforme."

Antoine Laurent de Lavoisier (químico francês, 1743 - 1794)

----- Original Message -----
From: Luiz Ferraz Netto
To: ciencialist@yahoogrupos.com.br
Sent: Friday, February 25, 2005 10:27 AM
Subject: [ciencialist] Feira de Ciencias Permanente de Barretos


Olá pessoal,

nessa última 2a feira a boa cúpula política de Barretos (recém empossada) esteve aqui em casa para uma reunião sobre a Feira Permanente em Barretos. Foi uma tarde produtiva, agradável e de total sucesso! A coisa tá engrenada!
Tiramos umas fotos na varanda dos fundos de minha casa (e nem precisou tirar a mesa de 'snooker' do lugar) --- dê uma espiada:

http://www.feiradeciencias.com.br/css/projeto_barretos.htm

aquele abraço,
===========================
Luiz Ferraz Netto [Léo]
leobarretos@uol.com.br
http://www.feiradeciencias.com.br
===========================


--
No virus found in this outgoing message.
Checked by AVG Anti-Virus.
Version: 7.0.300 / Virus Database: 266.4.0 - Release Date: 22/02/2005



##### ##### #####

Para saber mais visite
http://www.ciencialist.hpg.ig.com.br


##### ##### ##### #####


Yahoo! Grupos, um serviço oferecido por:







------------------------------------------------------------------------------
Links do Yahoo! Grupos

a.. Para visitar o site do seu grupo na web, acesse:
http://br.groups.yahoo.com/group/ciencialist/

b.. Para sair deste grupo, envie um e-mail para:
ciencialist-unsubscribe@yahoogrupos.com.br

c.. O uso que você faz do Yahoo! Grupos está sujeito aos Termos do Serviço do Yahoo!.



[As partes desta mensagem que não continham texto foram removidas]



SUBJECT: lateral sciency
FROM: "murilo filo" <avalanchedrive@hotmail.com>
TO: ciencialist@yahoogrupos.com.br
DATE: 25/02/2005 11:15

Alô pessoal!
Há coisas imperdíveis no site abaixo.
Gosto muito do sub-mundo da ciência.

http://www.lateralscience.co.uk
>
>It's both informative and entertaining - what more could you want!

Divirtam-se, se é que não conheciam. abr/M. 25/fev




SUBJECT: Re: [ciencialist] Feira de Ciencias Permanente de Barretos
FROM: "Eurico Ferreira de Souza Jr." <caodejah@yahoo.com.br>
TO: ciencialist@yahoogrupos.com.br
DATE: 25/02/2005 11:17


[E]> ah, que muito loco! parabéns! adorei!
dá pra ver que as experiências são feitas com muito capricho
quando eu for em barretos, visitarei!

[]s

Eurico, nascido em Guaíra (pertinho de Barretos)



_\|/_

---------------------------------
Yahoo! Acesso Grátis - Internet rápida e grátis. Instale o discador do Yahoo! agora.

[As partes desta mensagem que não continham texto foram removidas]



SUBJECT: Re: [ciencialist] Fw: massa de vidraceiro
FROM: "Eurico Ferreira de Souza Jr." <caodejah@yahoo.com.br>
TO: ciencialist@yahoogrupos.com.br
DATE: 25/02/2005 11:26

[E]> gesso + óleo de linhaça (eu acho)


_\|/_

---------------------------------
Yahoo! Acesso Grátis - Internet rápida e grátis. Instale o discador do Yahoo! agora.

[As partes desta mensagem que não continham texto foram removidas]



SUBJECT: Re: Foucault / Luz ondas ou corpusculos.
FROM: Hélio Ricardo Carvalho <hrc@fis.puc-rio.br>
TO: ciencialist@yahoogrupos.com.br
DATE: 25/02/2005 11:48


Para Victor,

Gostaria de retornar a questão original desta discussão.

Luz: ondas ou cospúsculos?

Você escreveu na msg 44272:
> >...
> >No que tange ao conhecido fenômeno da refração, o
> >que se observa, experimentalmente, com um modelo corpuscular?
> >Velocidade da luz no meio de maior índice, maior que a velocidade
> >da luz no vácuo; explicação: as partículas de luz, ao colidirem
> >com a superfície da água ficam submetidas a uma atração, uma
força
> >normal à
> >superfície no ponto de contato, obrigando as dita cujas
aproximarem-
> >se da normal e esta seria a causa da refração, da mudança de
direção
> >ao sair de um meio e entrar em outro, diferente. Como resultado
desta
> >atração, as partículas sofreriam uma pequena aceleração na
> >superfície,
> >senda esta a causa de sua maior velocidade no meio mais denso.
Isto
> >é o
> >que se obtém com um modelo corpuscular, para o fenômeno refração.
> >...

Eu completaria o final assim:
... Isto é o que se obtém com O MODELO CORPUSCULAR DE NEWTON, para o
fenômeno refração.

A experiência foi contra este modelo (que nem era tão importante
assim para Newton, como nos falou Alberto Mesquita) e não foi contra
a teoria corpuscular em geral.

No modelo ondulatório em vigor na época a luz também deveria ser
mais rápida no meio mais denso.

Então vai minha OUTRA pergunta:
Por que a experiência de Foucault não descartou de vez a luz
ondulatória também????


Uma investigação histórica superficial me leva a reafirmar o que
disse em outra msg: Era moda derrubar Newton e tudo era pretexto.


Na msg anterior eu disse com uma certa ironia:
"
> Perfeito! É isto que eu queria saber: a linha de raciocínio que
levou
> aos cientistas da época (com os poucos conhecimentos que tinham) a
> acharem que a dita experiência derrubava a luz corpuscular.
> :-)
> :-)
>
"
Completando, eu digo, que para este "achismo" dos cientista da época
a moda de derrubar Newton contribuiu bastante!!!!



Se possível comente também o este final da msg anterior:

> Mas, depois da física quântica (falo da FQ básica e não o
esoterismo
> de hoje) existe um modelo de como funciona a refração levando em
> consideração as sucessivas absorções e reemissões dos corpúsculos
de
> luz.
>
> Veja o final de minha msg 44263.
>
>
> Só quando se leva em conta que a luz é absorvida e depois
reemitida
> pelos átomos do meio que se pode começar a entender a refração.
> Seja ela corpúsculos ou onda.
>
> É difícil para mim escrever isto, como algo pode SER onda. Para
mim
> onda não é um SER. :-)
> Onda, para mim é apenas uma das formas da expressar um determinado
> comportamento da matéria.


[ ]'s
Helio





SUBJECT: José Collucci
FROM: Fabíola Iszlay <fabiolaiszlay@yahoo.com.br>
TO: ciencialist@yahoogrupos.com.br
DATE: 25/02/2005 11:49

Procuro pelo J. Collucci que trabalhou na FUNBEC, se alguém puder me ajudar...

Obrigada,

Fabíola

__________________________________________________
Converse com seus amigos em tempo real com o Yahoo! Messenger
http://br.download.yahoo.com/messenger/

[As partes desta mensagem que não continham texto foram removidas]



SUBJECT: Re: Feira de Ciencias Permanente de Barretos
FROM: Maria Natália <grasdic@hotmail.com>
TO: ciencialist@yahoogrupos.com.br
DATE: 25/02/2005 14:04


Boa!
E político não sabia mesmo nada de física?
Posso retirar uma foto para colocar lá na lista e meninos o conhecerem?
Que essa visita dê bons frutos.
Um abraço
Maria Natália


--- Em ciencialist@yahoogrupos.com.br, "Luiz Ferraz Netto"
<leobarretos@u...> escreveu
> Olá pessoal,
>
> nessa última 2a feira a boa cúpula política de Barretos (recém
empossada) esteve aqui em casa para uma reunião sobre a Feira
Permanente em Barretos. Foi uma tarde produtiva, agradável e de total
sucesso! A coisa tá engrenada!
> Tiramos umas fotos na varanda dos fundos de minha casa (e nem
precisou tirar a mesa de 'snooker' do lugar) --- dê uma espiada:
>
> http://www.feiradeciencias.com.br/css/projeto_barretos.htm
>
> aquele abraço,
> ===========================
> Liz Ferraz Netto [Léo]
> leobarretos@u...
> http://www.feiradeciencias.com.br
> ===========================
>
>
> --
> No virus found in this outgoing message.
> Checked by AVG Anti-Virus.
> Version: 7.0.300 / Virus Database: 266.4.0 - Release Date: 22/02/2005





SUBJECT: NeBIOQ2--2º Encontro Nacional de Estudantes Bioquímicos
FROM: Maria Natália <grasdic@hotmail.com>
TO: ciencialist@yahoogrupos.com.br
DATE: 25/02/2005 14:33


NeBIOQ2 - 2º Encontro Nacional de Estudantes Bioquímicos "Bioquímica:
Para Além das Fronteiras""
Ver mais em:
http://agenda.fc.ul.pt/formulario.aspx?id=593&mes=2&ano=2005

sds
Maria Natália






SUBJECT: Re: Feira de Ciencias Permanente de Barretos
FROM: "rmtakata" <rmtakata@altavista.net>
TO: ciencialist@yahoogrupos.com.br
DATE: 25/02/2005 14:34


--- Em ciencialist@yahoogrupos.com.br, "Luiz Ferraz Netto"
> A coisa tá engrenada!

Enfim uma noticia boa na politica nacional. Ha' luz no fim do tunel -
e parece q. nao eh o trem vindo em direcao contraria.

[]s,

Roberto Takata





SUBJECT: Re: Fw: [naeq-ucs] C pri. sec. terc. quart.
FROM: Maria Natália <grasdic@hotmail.com>
TO: ciencialist@yahoogrupos.com.br
DATE: 25/02/2005 14:36


Saí pois não me é dirigida
Maria Natália



--- Em ciencialist@yahoogrupos.com.br, "E m i l i a n o C h e m e l
l o" <chemelloe@y...> escreveu
> primário, secundário... terciário... quaternário! Eterna dúvida.
>
> [ ] 's do Emiliano Chemello
> emiliano@q...
> http://www.quimica.net/emiliano
> http://www.ucs.br/ccet/defq/naeq
>
> " Rien ne se perd, rien ne se crée,
> tout se transforme."
>
> Antoine Laurent de Lavoisier (químico francês, 1743 - 1794)
>
> ----- Original Message -----
> From: orley_alves
> To: naeq-ucs@yahoogrupos.com.br
> Sent: Thursday, February 24, 2005 6:26 PM
> Subject: [naeq-ucs] Dúvida
>
>
>
> Prof. EmilianSSaí


aí.
o, surgiu uma polêmica entre alguns professores aqui de
> BH, sobre a classificação do carbono em algumas cadeias. A dúvida é
> sobre o carbono primário.Por exemplo, na estrutura da uréia (NH2)2CO,
> o carbono poderia ser classificado como primário?
> Segundo alguns professores e alguns autores poderia, pois os mesmos
> definem como primário o carbono ligado a um ou a nenhum carbono.
> Porém oconceito que encontrei na maioria da literatura é: primário é
> aquele carbono ligado a um outro carbono.
> Penso que esta classificação se refere a carbonos ligados entre si na
> cadeia, se não há carbonos ligados entre si, como na uréia, então não
> há sentido em classificar este carbono desta forma.
> Qual a opinião do prof. sobre este assunto?
> Um abraço
>
>
>
>
> Yahoo! Grupos, um serviço oferecido por:
>
>
>
>
>
>
>
>
--------------------------------------------------------------------------------
> Links do Yahoo! Grupos
>
> a.. Para visitar o site do seu grupo na web, acesse:
> http://br.groups.yahoo.com/group/naeq-ucs/
>
> b.. Para sair deste grupo, envie um e-mail para:
> naeq-ucs-unsubscribe@yahoogrupos.com.br
>
> c.. O uso que você faz do Yahoo! Grupos está sujeito aos Termos do
Serviço do Yahoo!.
>
>
>
> [As partes desta mensagem que não continham texto foram removidas]





SUBJECT: Re: Fw: Informações e dúvidas
FROM: "rmtakata" <rmtakata@altavista.net>
TO: ciencialist@yahoogrupos.com.br
DATE: 25/02/2005 14:52


Seria John Cage? Embora a historia seja um tto diferente:

http://www.classicalnotes.net/columns/silence.html
http://www.cobussen.com/proefschrift/300_john_cage/317_cage_and_noise/317b_music_noise_silence_sound/music_noise_silence_sound.htm

[]s,

Roberto Takata

--- Em ciencialist@yahoogrupos.com.br, "Luiz Ferraz Netto"
> Alguém sabe o nome do dito cujo?
> []'
> ===========================
> Luiz Ferraz Netto [Léo]
> leobarretos@u...
> http://www.feiradeciencias.com.br
> ===========================
> -----Mensagem Original-----
> De: <lulaehrlich@g...>
> Para: <leobarretos@u...>
> Enviada em: quinta-feira, 24 de fevereiro de 2005 21:33
> Assunto: Informações e dúvidas
>
>
> Olá Professor Luiz Ferraz Netto. Ouvi dizer a tempo atrás que um músico
> queria provar que o silêncio não existe e fez uma experiência
entrando dentro
> de uma câmara a prova de som.
> Gostaria de saber quem foi esse músico, como chamava a câmara e
onde posso
> obter figuras e imformações sobre essa experiência.
> Muito grato desde já
> Grande abraço
> Lula Ehrlich
>
>
>
> --
> No virus found in this incoming message.
> Checked by AVG Anti-Virus.
> Version: 7.0.300 / Virus Database: 266.4.0 - Release Date: 22/02/2005
>
>
>
>
> --
> No virus found in this outgoing message.
> Checked by AVG Anti-Virus.
> Version: 7.0.300 / Virus Database: 266.4.0 - Release Date: 22/02/2005





SUBJECT: Re: Fw: Contaminação dos solos
FROM: "rmtakata" <rmtakata@altavista.net>
TO: ciencialist@yahoogrupos.com.br
DATE: 25/02/2005 14:54


O ideal seria fazer uma analise da qualidade de agua.

Verificar se em diferentes amostras de agua existe a presenca de
microorganismos q. contaminam gatos.

[]s,

Roberto Takata

--- Em ciencialist@yahoogrupos.com.br, "Luiz Ferraz Netto"
> Essa é para o pessoal das biologias .........
> Luiz Ferraz Netto [Léo]
> -----Mensagem Original-----
> De: Luiza Paiva Ferrari
> Pergunto, em sua opinião: Este solo contaminado por degetos, pêlos
> e animais doentes e/ ou eventualmente mortos poderá transmitir
> doenças para as pessoas através da água do rio? ( O rio faz parte de





SUBJECT: Re: [ciencialist] Fw: Informações e dúvidas
FROM: "Oraculo" <oraculo@atibaia.com.br>
TO: <ciencialist@yahoogrupos.com.br>
DATE: 25/02/2005 14:55

Olá Leo

Não conheço essa história, mas está mais parecendo um hoax ou conto pitoresco da rede..:-) Ou má compreensão de uma experiencia real.

Silêncio é um conceito relativo, muitas vezes subjetivo, de falta de sons audíveis. E audíveis é o foco da questão, que a torna subjetiva..:-) Se eu me encontro em um ambiente onde meu sistema auditivo não percebe nenhuma manifestação, posso dizer que estou em silêncio, em um ambiente silencioso e que existe silencio aqui. O que não significa que não existam sons ou vibrações sonoras, tanto acima quanto abaixo de minha capacidade auditiva.

Surdos completos, por exemplo, estão sempre em um ambiente silencioso, não importa a festa que se faça em volta..:-)

E o que provaria um músico entrando em uma camara a prova de som? Uma camara assim não permite que o som entre ou saia, e pode também absorver sons produzidos (seria uma camara silenciosa, não a prova de sons, na verdade) mas nada diz sobre o som que é produzido lá dentro. Ele pode tossir ou respirar e produzirá vibrações sonoras. Mesmo o batimento de seu coração pdoe produzir vibrações desse tipo.

Uma história bem estranha, se são apenas estes os dados disponíveis..:-)

Um abraço.

Homero


----- Original Message -----
From: Luiz Ferraz Netto
To: ciencialist
Sent: Friday, February 25, 2005 9:20 AM
Subject: [ciencialist] Fw: Informações e dúvidas


Alguém sabe o nome do dito cujo?
[]'
===========================
Luiz Ferraz Netto [Léo]
leobarretos@uol.com.br
http://www.feiradeciencias.com.br
===========================
-----Mensagem Original-----
De: <lulaehrlich@globo.com>
Para: <leobarretos@uol.com.br>
Enviada em: quinta-feira, 24 de fevereiro de 2005 21:33
Assunto: Informações e dúvidas


Olá Professor Luiz Ferraz Netto. Ouvi dizer a tempo atrás que um músico
queria provar que o silêncio não existe e fez uma experiência entrando dentro
de uma câmara a prova de som.
Gostaria de saber quem foi esse músico, como chamava a câmara e onde posso
obter figuras e imformações sobre essa experiência.
Muito grato desde já
Grande abraço
Lula Ehrlich



--
No virus found in this incoming message.
Checked by AVG Anti-Virus.
Version: 7.0.300 / Virus Database: 266.4.0 - Release Date: 22/02/2005




--
No virus found in this outgoing message.
Checked by AVG Anti-Virus.
Version: 7.0.300 / Virus Database: 266.4.0 - Release Date: 22/02/2005



##### ##### #####

Para saber mais visite
http://www.ciencialist.hpg.ig.com.br


##### ##### ##### #####


Yahoo! Grupos, um serviço oferecido por:
PUBLICIDADE




------------------------------------------------------------------------------
Links do Yahoo! Grupos

a.. Para visitar o site do seu grupo na web, acesse:
http://br.groups.yahoo.com/group/ciencialist/

b.. Para sair deste grupo, envie um e-mail para:
ciencialist-unsubscribe@yahoogrupos.com.br

c.. O uso que você faz do Yahoo! Grupos está sujeito aos Termos do Serviço do Yahoo!.



[As partes desta mensagem que não continham texto foram removidas]



SUBJECT: Site esquisito ..:-)
FROM: "Oraculo" <oraculo@atibaia.com.br>
TO: <ciencialist@yahoogrupos.com.br>
DATE: 25/02/2005 16:38

Olá pessoal

Apesar desta lista tratar de ciências, e este assunto a seguir ser um tanto off-topic, gostaria de saber o que acham os participantes deste site que encontrei por acaso..:-) E, afinal, trata-se de nossas universidades, estudantes e futuros profissionais também de ciências..:-)

http://www.monografa.cnt.br/

Um sistema de vendas de monografias, trabalhos escolares, resumos, relatorios de estágios, etc. Não sei o que está acontecendo, mas sempre pensei que isso deveria ser um comércio subterraneo, underground, com cuidados e disfarces, e não nesse nível de cara de pau e propaganda..:-)

Será que já é legítimo pagar por um trabalho ou tese de mestrado e eu não percebi a "mudança de paradigma"?..:-) Estou ficando ultrapassado?..:-)

Um abraço.

Homero

[As partes desta mensagem que não continham texto foram removidas]



SUBJECT: Re: [ciencialist] José Collucci
FROM: "Luiz Ferraz Netto" <leobarretos@uol.com.br>
TO: <ciencialist@yahoogrupos.com.br>
DATE: 25/02/2005 16:40

Olá Fabíola,

vou perguntar para ele se ele quer ser encontrado ... depois retorno.......

aquele abraço,
===========================
Luiz Ferraz Netto [Léo]
leobarretos@uol.com.br
http://www.feiradeciencias.com.br
===========================
-----Mensagem Original-----
De: "Fabíola Iszlay" <fabiolaiszlay@yahoo.com.br>
Para: <ciencialist@yahoogrupos.com.br>
Enviada em: sexta-feira, 25 de fevereiro de 2005 11:49
Assunto: [ciencialist] José Collucci



Procuro pelo J. Collucci que trabalhou na FUNBEC, se alguém puder me ajudar...

Obrigada,

Fabíola

__________________________________________________
Converse com seus amigos em tempo real com o Yahoo! Messenger
http://br.download.yahoo.com/messenger/

[As partes desta mensagem que não continham texto foram removidas]



##### ##### #####

Para saber mais visite
http://www.ciencialist.hpg.ig.com.br


##### ##### ##### #####
Links do Yahoo! Grupos










--
No virus found in this incoming message.
Checked by AVG Anti-Virus.
Version: 7.0.300 / Virus Database: 266.4.0 - Release Date: 22/02/2005




--
No virus found in this outgoing message.
Checked by AVG Anti-Virus.
Version: 7.0.300 / Virus Database: 266.4.0 - Release Date: 22/02/2005



SUBJECT: Re: [ciencialist] Re: Feira de Ciencias Permanente de Barretos
FROM: "Luiz Ferraz Netto" <leobarretos@uol.com.br>
TO: <ciencialist@yahoogrupos.com.br>
DATE: 25/02/2005 16:54

Oi Takata,

espero que uma vez vingado o processo da Feira vc me permitirá convidá-lo para participar dele. Assim que eu tiver certeza da entrada dos devidos $$ para o projeto vou força-lo a se mudar para Barretos City --- ai vc vai ver o que é 'calor', independente do que diz a termodinâmica!

aquele abraço,
===========================
Luiz Ferraz Netto [Léo]
leobarretos@uol.com.br
http://www.feiradeciencias.com.br
===========================
-----Mensagem Original-----
De: "rmtakata" <rmtakata@altavista.net>
Para: <ciencialist@yahoogrupos.com.br>
Enviada em: sexta-feira, 25 de fevereiro de 2005 14:34
Assunto: [ciencialist] Re: Feira de Ciencias Permanente de Barretos




--- Em ciencialist@yahoogrupos.com.br, "Luiz Ferraz Netto"
> A coisa tá engrenada!

Enfim uma noticia boa na politica nacional. Ha' luz no fim do tunel -
e parece q. nao eh o trem vindo em direcao contraria.

[]s,

Roberto Takata





##### ##### #####

Para saber mais visite
http://www.ciencialist.hpg.ig.com.br


##### ##### ##### #####
Links do Yahoo! Grupos










--
No virus found in this incoming message.
Checked by AVG Anti-Virus.
Version: 7.0.300 / Virus Database: 266.4.0 - Release Date: 22/02/2005




--
No virus found in this outgoing message.
Checked by AVG Anti-Virus.
Version: 7.0.300 / Virus Database: 266.4.0 - Release Date: 22/02/2005



SUBJECT: Feira de Barretos e GADO
FROM: "L.E.R.de Carvalho" <lecarvalho@infolink.com.br>
TO: ciencialist@yahoogrupos.com.br
DATE: 25/02/2005 19:44

Leo,

Me parece que seria muito interessante se, em Barretos, essa Feira de
Ciencias pudesse estar associada com "Agropecuária e Alimentos".

Ou seja, apresentar experimentos e exemplos de aplicação de ciências
(Física, Química, Biologia, Microbiologia, Genética, Matemática...) na
produção e industrialização agropecuária, com ênfase nos produtos de
origem bovina.

L.E.



At 10:27 25/2/2005, you wrote:
>Olá pessoal,
>
>nessa última 2a feira a boa cúpula política de Barretos (recém empossada)
>esteve aqui em casa para uma reunião sobre a Feira Permanente em Barretos.
>Foi uma tarde produtiva, agradável e de total sucesso! A coisa tá engrenada!
>Tiramos umas fotos na varanda dos fundos de minha casa (e nem precisou
>tirar a mesa de 'snooker' do lugar) --- dê uma espiada:
>
><http://www.feiradeciencias.com.br/css/projeto_barretos.htm>http://www.feiradeciencias.com.br/css/projeto_barretos.htm
>
>aquele abraço,
> ===========================
> Luiz Ferraz Netto [Léo]
> leobarretos@uol.com.br
> <http://www.feiradeciencias.com.br>http://www.feiradeciencias.com.br
> ===========================


[As partes desta mensagem que não continham texto foram removidas]



SUBJECT: Re: [ciencialist] Feira de Barretos e GADO
FROM: "Luiz Ferraz Netto" <leobarretos@uol.com.br>
TO: <ciencialist@yahoogrupos.com.br>
DATE: 25/02/2005 21:01

Olá LER,

excelente idéia ......
e quem, devidamente gabaritado, colocarei nessa tarefa? Onde acharei a pessoa adequada para levar essa idéia avante? Percebe, vc está falando de temas que envolvem pelo menos 5 especialistas no assunto. Deixa eu tirar o chapeuzinho de falsa modéstia e dizer que sou capaz de levar a frente Física (e 1/2 matemática) prá frente ... e o restante?
Não digo que seja idéia impraticável, não, não é ... desde que seja desenvolvida progressivamente, com o assentamento da Feira Permanente. Esse semestre contrata-se um 'cidadão' adequado a tal coisa, depois outro para desenvolver tal tema ... e assim, de leve, ...
Chamo a atenção para um ligeiro detalhe: A Física vai começar com cerca de 500 aparelhos já prontos; curso de capacitação para todos os professores de ciências da cidade já estruturado (essa capacitação vai abranger física, química e matemática --- a maioria dos profs de ciências são 'formados' em biologia!) --- além disso temos que projetar o Parque Temático com estruturas como Parafuso de Arquimedes de 2 m de comprimento; Tubo de Torricelli com tubo de 10 m de altura+água+boneca; casa dos espelhos, casa maluca etc.......
Estamos aguardando sugestões .........

[]'
===========================
Luiz Ferraz Netto [Léo]
leobarretos@uol.com.br
http://www.feiradeciencias.com.br
===========================
-----Mensagem Original-----
De: "L.E.R.de Carvalho" <lecarvalho@infolink.com.br>
Para: <ciencialist@yahoogrupos.com.br>
Enviada em: sexta-feira, 25 de fevereiro de 2005 19:44
Assunto: [ciencialist] Feira de Barretos e GADO



Leo,

Me parece que seria muito interessante se, em Barretos, essa Feira de
Ciencias pudesse estar associada com "Agropecuária e Alimentos".

Ou seja, apresentar experimentos e exemplos de aplicação de ciências
(Física, Química, Biologia, Microbiologia, Genética, Matemática...) na
produção e industrialização agropecuária, com ênfase nos produtos de
origem bovina.

L.E.



At 10:27 25/2/2005, you wrote:
>Olá pessoal,
>
>nessa última 2a feira a boa cúpula política de Barretos (recém empossada)
>esteve aqui em casa para uma reunião sobre a Feira Permanente em Barretos.
>Foi uma tarde produtiva, agradável e de total sucesso! A coisa tá engrenada!
>Tiramos umas fotos na varanda dos fundos de minha casa (e nem precisou
>tirar a mesa de 'snooker' do lugar) --- dê uma espiada:
>
><http://www.feiradeciencias.com.br/css/projeto_barretos.htm>http://www.feiradeciencias.com.br/css/projeto_barretos.htm
>
>aquele abraço,
> ===========================
> Luiz Ferraz Netto [Léo]
> leobarretos@uol.com.br
> <http://www.feiradeciencias.com.br>http://www.feiradeciencias.com.br
> ===========================


[As partes desta mensagem que não continham texto foram removidas]



##### ##### #####

Para saber mais visite
http://www.ciencialist.hpg.ig.com.br


##### ##### ##### #####
Links do Yahoo! Grupos










--
No virus found in this incoming message.
Checked by AVG Anti-Virus.
Version: 7.0.300 / Virus Database: 266.4.0 - Release Date: 22/02/2005




--
No virus found in this outgoing message.
Checked by AVG Anti-Virus.
Version: 7.0.300 / Virus Database: 266.4.0 - Release Date: 22/02/2005



SUBJECT: Feira de Barretos e GADO
FROM: "L.E.R.de Carvalho" <lecarvalho@infolink.com.br>
TO: ciencialist@yahoogrupos.com.br
DATE: 25/02/2005 21:27

At 21:01 25/2/2005, you wrote:
>Olá LER,
>
>excelente idéia ......
>e quem, devidamente gabaritado, colocarei nessa tarefa? Onde acharei a
>pessoa adequada para levar essa idéia avante? Percebe, vc está falando de
>temas que envolvem pelo menos 5 especialistas no assunto. Deixa eu tirar o
>chapeuzinho de falsa modéstia e dizer que sou capaz de levar a frente
>Física (e 1/2 matemática) prá frente ... e o restante?
>Não digo que seja idéia impraticável, não, não é ... desde que seja
>desenvolvida progressivamente, com o assentamento da Feira Permanente.
>Esse semestre contrata-se um 'cidadão' adequado a tal coisa, depois outro
>para desenvolver tal tema ... e assim, de leve, ...
>Chamo a atenção para um ligeiro detalhe: A Física vai começar com cerca de
>500 aparelhos já prontos; curso de capacitação para todos os professores
>de ciências da cidade já estruturado (essa capacitação vai abranger
>física, química e matemática --- a maioria dos profs de ciências são
>'formados' em biologia!) --- além disso temos que projetar o Parque
>Temático com estruturas como Parafuso de Arquimedes de 2 m de comprimento;
>Tubo de Torricelli com tubo de 10 m de altura+água+boneca; casa dos
>espelhos, casa maluca etc.......
>Estamos aguardando sugestões .........
>
>[]'
> ===========================


Bom...
deixa eu ver.

O Museu de Ciência, de Londres, fez uma enorme exposição temporária sobre
ALIMENTOS.

Eu estive lá e trouxe muito material.
Vou tentar, daqui uns dias, escanear alguma coisinha e te mandar pra uma
ideia inicial.

Penso que seja possível obter um patrocínio de empresas como a SADIA, por
exemplo.

Só pra ficar na Física, vou te perguntar uma coisa:
você já pensou por que o formato da lata de sardinha é completamente
diferente da lata de pêssego em calda ?

Vamos indo devagar com isso, mas tenho a convicção que se a Feira lograsse
uma vinculação temática com o negócio de Peão Boiadeiro... ia fazer ainda
mais sucesso.

Outra opção seria ter um setor sobre agropecuária, um outro setor sobre
rodeios (deve ter negócio de força centrífuga e centrípeta, imagino, nos
atos dos peões e seus laços) e assim por diante.

Tem química, biologia e genética.

L.E.

[As partes desta mensagem que não continham texto foram removidas]



SUBJECT: Feira de Barretos e Pessoal
FROM: "L.E.R.de Carvalho" <lecarvalho@infolink.com.br>
TO: ciencialist@yahoogrupos.com.br
DATE: 25/02/2005 21:30

At 21:01 25/2/2005, you wrote:
>Olá LER,
>
>excelente idéia ......
>e quem, devidamente gabaritado, colocarei nessa tarefa? Onde acharei a
>pessoa adequada para levar essa idéia avante? Percebe, vc está falando de
>temas que envolvem pelo menos 5 especialistas no assunto. Deixa eu tirar o
>chapeuzinho de falsa modéstia e dizer que sou capaz de levar a frente
>Física (e 1/2 matemática) prá frente ... e o restante?
>Não digo que seja idéia impraticável, não, não é ... desde que seja
>desenvolvida progressivamente, com o assentamento da Feira Permanente.
>Esse semestre contrata-se um 'cidadão' adequado a tal coisa, depois outro
>para desenvolver tal tema ... e assim, de leve, ...
>[]'
> ===========================
> Luiz Ferraz Netto [Léo]



Talvez não seja o caso de se contratar especialistas temáticos.
Mas sim de especialistas na criação e manutenção de objetos.

Para criar a idéia básica de cada experimento poderia ser feita uma
parceria com laboratórios de diferentes universidades.

A gente aqui na UFRJ poderia talvez participar com a definição de alguns
experimentos.
Mas é obvio que falta gente capaz de construir o "objeto".

Luiz Eduardo

[As partes desta mensagem que não continham texto foram removidas]



SUBJECT: Lord Kelvin
FROM: Amauri Nolasco Sanches Jr <amaurijunior2@yahoo.com.br>
TO: patricia.ferrari@uol.com.br, Pingao <Conversa_de_Botequim@yahoogrupos.com.br>, regiavalen@fastrun.at, reginaldo.pereira.li@itelefonica.com.br, Lucimary Vargas <lucimaryvargas@yahoo.com.br>, veloz <isnarcarlos@ig.com.br>, vicente <vi.colarusso@bol.com.br>, Celina Victor <celinavictor94@yahoo.com.br>, Maria Palmira Minholi Dias <m_p_m_d@yahoo.com.br>, Fabiano Donato <fdonatoreis@yahoo.com.br>, Aline Caldas <alinecaldasc@yahoo.com.br>, ciencia.2005@grupos.com.br, ciencilist <ciencialist@yahoogrupos.com.br>, acropolis@yahoogrupos.com.br
DATE: 25/02/2005 21:34


Lord Kelvin



(1824 - 1907)

No século XIX, a ciência foi definitivamente
consagrada como fator de aprimoramento da vida humana.
Gradativamente, o trabalho científico assumiu seu
papel social, integrando-se melhor às outras
atividades do homem. O apelo utilitário da indústria,
por exemplo, abria à ciência possibilidades
revolucionárias, acelerando o desenvolvimento da
técnica.

Figura ativa nesse cenário, William Thomson - mais
tarde Lorde Kelvin foi um dos cientistas que, vivendo
intensamente sua época, superou os limites de um campo
de trabalho até então muito estrito e estendeu seu
talento à invenção e à tecnologia.

No ano de 1899, a universidade escocesa de Glasgow
perdia o mais célebre de seus docentes, ao mesmo tempo
que conquistava um novo estudante. Lorde Kelvin,
deixando a cátedra que ocupara por 53 anos, pedia para
ser admitido como pesquisador.

(Universidade de Glasgow)

William Thomson nascera 75 anos antes (26 de julho de
1824) em Belfast, Irlanda, de uma família de
agricultores escoceses. Havia completado oito anos
quando seu pai começou a lecionar matemática na
Universidade de Glasgow. Dois anos depois, certamente
em consideração a seus dotes excepcionais, o rapaz foi
admitido na universidade. Sua mente, porém, não se
limitava aos estudos curriculares. Em pouco tempo,
conquistou um vasto conhecimento sobre os clássicos
antigos e orientais, que conseguia ler na língua
original.

Deixando Giasgow sem se graduar, em 1841, entrou para
o Peterhouse College, em Cambridge. Contava então com
dezessete anos. Lá se fez notar pela seriedade no
estudo, pela amabilidade de caráter e por uma grande
paixão esportiva (era um ótimo remador). Em 1845
diplomou-se e mereceu o Smith's Prize. No mesmo ano
viajou para a França.

Na época, havia poucas facilidades na Grã-Bretanha
para o estudo das ciências experimentais, ao contrário
do que se observava em outros países ocidentais.
Apesar da fecundidade da tradição newtoniana - baseada
na experimentação, e que estava na origem dos êxitos
dos físicos britânicos -, o ensino científico no país
sofria um declínio, graças à influência aristocrática
e religiosa sobre as universidades e as concepções
imediatistas da indústria.

A vitória da Inglaterra sobre Napoleão, ao mesmo tempo
que consagra a supremacia industrial britânica,
assinala a decadência da filosofia racionalista
difundida pela Revolução Francesa e provoca um
sentimento hostil ao desenvolvimento acelerado da
ciência, possível fonte de ateísmo.

Afastando-se desse ambiente pouco propício, Kelvin foi
a Paris, onde estudou sob a orientação de Regnault,
então empenhado em suas clássicas pesquisas sobre as
propriedades térmicas do vapor. O tratado de Joseph
Fourier sobre o calor havia despertado em Kelvin,
cinco anos antes, intenso interesse pela
termodinâmica.

Em 1846, Kelvin aceitou a cadeira de filosofia natural
na Universidade de Glasgow. Um ano depois, conheceu
Joule, encontro que influenciou decisivamente a
evolução de sua carreira.

As propriedades do calor foram um dos temas preferidos
de Kelvin. Analisou com mais profundidade as
descobertas de Jacques Charles sobre a variação de
volume dos gases em função da variação de temperatura.
Charles concluíra, com base em experimentos e
cálculos, que à temperatura de -273'C todos os gases
teriam volume igual a zero. Kelvin propôs outra
conclusão: não era o volume da matéria que se anularia
nessa temperatura, mas sim a energia cinética de suas
moléculas. Sugeriu então que essa temperatura deveria
ser considerada a mais baixa possível e chamou-a de
zero absoluto. A partir dela, propôs uma nova escala
termométrica (que posteriormente recebeu o nome de
escala Kelvin), a qual permitiria maior simplicidade
para a expressão matemática das relações entre
grandezas termodinâmicas.

Em 1851 apresentou um trabalho sobre a teoria dinâmica
do calor. Esta reconciliava os estudos de Sadi Carnot
com as conclusões de Rumford, Davy, Mayer e Joule.
Neste trabalho foi, pela primeira vez, estabelecido o
princípio da dissipação da energia, posteriormente
sumarizado no segundo princípio da termodinâmica.

Kelvin, porém, não se limitava a formular teorias
sobre os princípios gerais da física, mas as
experimentava tenazmente, usando engenhosos aparelhos
por ele mesmo inventados.

Na época de sua juventude, o estudo da eletricidade e,
em particular, a teoria matemática da eletrostática
estavam apenas esboçados e ainda imprecisos. A
contribuição de Kelvin nestas áreas foi notável.
Encontrou meios de medir tensões e correntes nas
condições as mais diversas. Construiu delicados
instrumentos capazes de verificar as leis da
eletrostática. Em 1853, formulou a teoria dos
circuitos oscilantes e conseguiu comprová-la com seu
aparelhamento de concepção verdadeiramente moderna.
Por fim, sugeriu um processo para a medição da força
eletromotriz e da resistência ôhmica (1861) e
construiu um eletrômetro, com o qual era possível
determinar, com exatidão, a constante que relaciona a
unidade eletromagnética e a unidade eletrostática de
intensidade de corrente (1867).

(Kelvin e o cabo submarino)

Com sua habilidade em construir instrumentos, era
lógico que o cientista viesse a interessar-se pelo
telégrafo com fio, assunto que naquele momento
fascinava o mundo, mas que apresentava grandes
dificuldades técnicas. O problema a ser resolvido era
o da ligação, entre a Europa e a América, por meio de
cabo submarino. Em fins de 1854, Kelvin começou a
apaixonar-se pelo problema, que o interessava não só
por ser físico e engenheiro, mas também porque o mar o
atraía.

Inicialmente, concluiu que o cabo comporta-se como um
condensador e calculou sua capacidade. Depois,
serviu-se das fórmulas de Fourier para deduzir as leis
da propagação dos impulsos. Finalmente, através de uma
série de experiências, demonstrou a influência
negativa das impurezas do cobre e melhorou as
transmissões. Atingiu este último objetivo colocando
um condensador em cada extremidade do cabo.

Unindo a genialidade do inventor ao senso prático do
empresário, em 1856 tornou-se diretor da Atlantic
Telegraph Co. e lutou para ver suas idéias realizadas.
As primeiras tentativas não foram felizes. Um cabo foi
perdido em 1857, em decorrência de uma manobra errada.
Outro, colocado no ano seguinte entre a Irlanda e a
Terra Nova, talvez por sabotagem, inicialmente não
funcionou. Depois, quando o galvanômetro de espelho de
Kelvin permitiu remover o último obstáculo, o cabo, já
estragado, teve que ser definitivamente abandonado.

(Galvanômetro de espelho de Kelvin)

As tentativas só viriam a ser retomadas em 1865. O
galvanômetro de espelho, tão útil nas mais delicadas
experiências físicas, foi inventado por Kelvin
exatamente para revelar os fraquíssimos sinais
recebidos através dos cabos telegráficos (atualmente
os sinais são amplificados por meio de dispositivos
eletrônicos). Além dêste, Kelvin preparou (1867) um
galvanômetro que, ao perceber os sinais, gravava-os em
uma fita de papel (este instrumento foi chamado Syphon
recorder).

Neste ínterim, com o auxílio do barco a vapor Great
Eastern, foi instalado um novo cabo de 1 200 milhas de
extensão, que funcionou perfeitamente. Em 27 de julho
de 1866, os dois continentes estavam ligados por
telégrafo.

Juntamente com seus colaboradores, Kelvin foi feito
baronete e, em 1869, começou a auferir os primeiros
lucros de sua empresa, os quais aplicou em patrocinar
uma bolsa de estudos na Universidade de Glasgow.

Durante suas numerosas viagens marítimas, percebeu o
quanto eram imprecisas as bússolas, sobretudo em
conseqüência da ação magnética exercida pela própria
embarcação. Dedicou-se então, a partir de 1873, ao
aperfeiçoamento desse instrumento e à invenção de
outros. Entre eles destacam-se um aparelho capaz de
calcular a amplitude das marés e uma máquina para
resolver sistemas lineares algébricos.

A geologia e a cosmogonia foram também objeto de seus
estudos, assim como a estrutura da matéria e do
"éter". Sob a influência de Hermann von Helmholtz,
grande físico alemão e seu amigo, Kelvin aceitou a
idéia de que os átomos fossem vértices do "éter". Em
seus últimos anos, porém, adotou a hipótese da
natureza elétrica da matéria, embora sem muita
convicção.

Na época de Kelvin não se conhecia ainda a natureza da
energia irradiada pelo Sol. Baseando-se na teoria de
que essa energia resultava do resfriamento da matéria
primitiva, deduziu as idades mínimas de 500 milhões de
anos para a Terra e 100 milhões para o Sol. Estas
conclusões, que suscitaram as mais vivas polêmicas,
eram consideradas exageradas. O conhecimento moderno
revelou que, em verdade, o erro era por falta.

Essa intensa atividade foi acompanhada de honrarias
que culminaram, em 1892, com o título de barão Kelvin
de Largs, na Inglaterra. No mesmo ano recebeu a Ordem
do Mérito. Dois anos antes havia se tornado presidente
da Royal Society e dois anos depois foi eleito
chancellor da Universidade de Glasgow.

Em nenhum momento, porém, modificou seus hábitos,
sempre caracterizados pela modéstia. Proveniente de
uma família muito numerosa, Lorde Kelvin havia
desposado, em 1852, Margaret Grums. Brilhante e culta,
mas de saúde frágil, Margaret faleceu, ainda jovem, em
1870.

Tornando a viajar sem descanso, Lorde Kelvin encontrou
e desposou quatro anos depois, na ilha da Madeira,
Francis Blandy. Esta união se estendeu, serena, pelo
espaço de trinta anos.

Deixando mais de trezentos trabalhos publicados, o
infatigável cientista morreu no dia 17 de dezembro de
1907, em Netherall, na mesma Escócia da qual haviam
emigrado os seus antepassados. Seu sepultamento
deu-se, com todas as honras, na Abadia de Westminster.
Com ele, desaparecia o tipo de físico e engenheiro que
havia simbolizado o século XIX e que representava o
espírito otimista e empreendedor daquela época.

www.saladefisica.cjb.net

ÍNDICE BIOGRAFIAS

ÍNDICE GERAL

=====

Meu grupo sobre sexualidade: http://br.groups.yahoo.com/group/Sexualidadedodef/?yguid=94014452

" O dia mais importante não é quando conhecemos as pessoas mas sim o momento em que elas passam a existir dentro de nós. "

MSN: amaurijunior3@hotmail.com

icq:153134120

Blog:http://blogdomaster.weblogger.terra.com.br/index.htm





















_______________________________________________________
Yahoo! Acesso Grátis - Instale o discador do Yahoo! agora. http://br.acesso.yahoo.com/ - Internet rápida e grátis


SUBJECT: Feira, Sorvete e Física
FROM: "L.E.R.de Carvalho" <lecarvalho@infolink.com.br>
TO: ciencialist@yahoogrupos.com.br
DATE: 25/02/2005 21:55

No Museu da Tecnologia, de Washington, agora denominado Museu da História
Americana, houve uma exposição temporária sobre SORVETE.

Havia fotos das primeiras sorveterias, havia os primeiros carrinhos de
sorvete, fotos de filmes onde aparecia muito sorvete, muito sundae... mas
havia também uma montagem enorme do ciclo frigorifico, explicando como é
que se faz pra ficar frio dentro da geladeira.

Da invenção das pasteurização, passando pelo uso de autoclaves, água a mais
de 100º devido à pressão, até o sistema de compressor-condensador-valvula
de expansão... tem muita física aí dentro.

E tem tudo a ver com a industria frigorífica carnea.

L.E.




SUBJECT: Re: Lord Kelvin
FROM: "Yvanhoe" <yvanhoe@usa.com>
TO: ciencialist@yahoogrupos.com.br
DATE: 25/02/2005 22:46


Esse Lord kelvin(físico) foi uma pedra no sapato de Darwin, um dos
seus mais poderosos críticos. Ele havia estabelecido um limite para a
idade da terra em apenas uns poucos milhoes de anos. Era um
criacionista, e ainda apegagado as ideias bíblicas.
Darwin calou-se diante de suas pesquisas, mais tarde com o advento das
descobertas radioativas, uma nova idade da Terra foi estabelecida, e
as teorias de Darwin foram glorificadas com novas descobertas e
medições da idade da terra em bilhoes de anos

IVANHOE


--- Em ciencialist@yahoogrupos.com.br, Amauri Nolasco Sanches Jr
<amaurijunior2@y...> escreveu
>
> Lord Kelvin
>
>
>
> (1824 - 1907)
>
> No século XIX, a ciência foi definitivamente
> consagrada como fator de aprimoramento da vida humana.
> Gradativamente, o trabalho científico assumiu seu
> papel social, integrando-se melhor às outras
> atividades do homem. O apelo utilitário da indústria,
> por exemplo, abria à ciência possibilidades
> revolucionárias, acelerando o desenvolvimento da
> técnica.
>
> Figura ativa nesse cenário, William Thomson - mais
> tarde Lorde Kelvin foi um dos cientistas que, vivendo
> intensamente sua época, superou os limites de um campo
> de trabalho até então muito estrito e estendeu seu
> talento à invenção e à tecnologia.
>
> No ano de 1899, a universidade escocesa de Glasgow
> perdia o mais célebre de seus docentes, ao mesmo tempo
> que conquistava um novo estudante. Lorde Kelvin,
> deixando a cátedra que ocupara por 53 anos, pedia para
> ser admitido como pesquisador.
>
> (Universidade de Glasgow)
>
> William Thomson nascera 75 anos antes (26 de julho de
> 1824) em Belfast, Irlanda, de uma família de
> agricultores escoceses. Havia completado oito anos
> quando seu pai começou a lecionar matemática na
> Universidade de Glasgow. Dois anos depois, certamente
> em consideração a seus dotes excepcionais, o rapaz foi
> admitido na universidade. Sua mente, porém, não se
> limitava aos estudos curriculares. Em pouco tempo,
> conquistou um vasto conhecimento sobre os clássicos
> antigos e orientais, que conseguia ler na língua
> original.
>
> Deixando Giasgow sem se graduar, em 1841, entrou para
> o Peterhouse College, em Cambridge. Contava então com
> dezessete anos. Lá se fez notar pela seriedade no
> estudo, pela amabilidade de caráter e por uma grande
> paixão esportiva (era um ótimo remador). Em 1845
> diplomou-se e mereceu o Smith's Prize. No mesmo ano
> viajou para a França.
>
> Na época, havia poucas facilidades na Grã-Bretanha
> para o estudo das ciências experimentais, ao contrário
> do que se observava em outros países ocidentais.
> Apesar da fecundidade da tradição newtoniana - baseada
> na experimentação, e que estava na origem dos êxitos
> dos físicos britânicos -, o ensino científico no país
> sofria um declínio, graças à influência aristocrática
> e religiosa sobre as universidades e as concepções
> imediatistas da indústria.
>
> A vitória da Inglaterra sobre Napoleão, ao mesmo tempo
> que consagra a supremacia industrial britânica,
> assinala a decadência da filosofia racionalista
> difundida pela Revolução Francesa e provoca um
> sentimento hostil ao desenvolvimento acelerado da
> ciência, possível fonte de ateísmo.
>
> Afastando-se desse ambiente pouco propício, Kelvin foi
> a Paris, onde estudou sob a orientação de Regnault,
> então empenhado em suas clássicas pesquisas sobre as
> propriedades térmicas do vapor. O tratado de Joseph
> Fourier sobre o calor havia despertado em Kelvin,
> cinco anos antes, intenso interesse pela
> termodinâmica.
>
> Em 1846, Kelvin aceitou a cadeira de filosofia natural
> na Universidade de Glasgow. Um ano depois, conheceu
> Joule, encontro que influenciou decisivamente a
> evolução de sua carreira.
>
> As propriedades do calor foram um dos temas preferidos
> de Kelvin. Analisou com mais profundidade as
> descobertas de Jacques Charles sobre a variação de
> volume dos gases em função da variação de temperatura.
> Charles concluíra, com base em experimentos e
> cálculos, que à temperatura de -273'C todos os gases
> teriam volume igual a zero. Kelvin propôs outra
> conclusão: não era o volume da matéria que se anularia
> nessa temperatura, mas sim a energia cinética de suas
> moléculas. Sugeriu então que essa temperatura deveria
> ser considerada a mais baixa possível e chamou-a de
> zero absoluto. A partir dela, propôs uma nova escala
> termométrica (que posteriormente recebeu o nome de
> escala Kelvin), a qual permitiria maior simplicidade
> para a expressão matemática das relações entre
> grandezas termodinâmicas.
>
> Em 1851 apresentou um trabalho sobre a teoria dinâmica
> do calor. Esta reconciliava os estudos de Sadi Carnot
> com as conclusões de Rumford, Davy, Mayer e Joule.
> Neste trabalho foi, pela primeira vez, estabelecido o
> princípio da dissipação da energia, posteriormente
> sumarizado no segundo princípio da termodinâmica.
>
> Kelvin, porém, não se limitava a formular teorias
> sobre os princípios gerais da física, mas as
> experimentava tenazmente, usando engenhosos aparelhos
> por ele mesmo inventados.
>
> Na época de sua juventude, o estudo da eletricidade e,
> em particular, a teoria matemática da eletrostática
> estavam apenas esboçados e ainda imprecisos. A
> contribuição de Kelvin nestas áreas foi notável.
> Encontrou meios de medir tensões e correntes nas
> condições as mais diversas. Construiu delicados
> instrumentos capazes de verificar as leis da
> eletrostática. Em 1853, formulou a teoria dos
> circuitos oscilantes e conseguiu comprová-la com seu
> aparelhamento de concepção verdadeiramente moderna.
> Por fim, sugeriu um processo para a medição da força
> eletromotriz e da resistência ôhmica (1861) e
> construiu um eletrômetro, com o qual era possível
> determinar, com exatidão, a constante que relaciona a
> unidade eletromagnética e a unidade eletrostática de
> intensidade de corrente (1867).
>
> (Kelvin e o cabo submarino)
>
> Com sua habilidade em construir instrumentos, era
> lógico que o cientista viesse a interessar-se pelo
> telégrafo com fio, assunto que naquele momento
> fascinava o mundo, mas que apresentava grandes
> dificuldades técnicas. O problema a ser resolvido era
> o da ligação, entre a Europa e a América, por meio de
> cabo submarino. Em fins de 1854, Kelvin começou a
> apaixonar-se pelo problema, que o interessava não só
> por ser físico e engenheiro, mas também porque o mar o
> atraía.
>
> Inicialmente, concluiu que o cabo comporta-se como um
> condensador e calculou sua capacidade. Depois,
> serviu-se das fórmulas de Fourier para deduzir as leis
> da propagação dos impulsos. Finalmente, através de uma
> série de experiências, demonstrou a influência
> negativa das impurezas do cobre e melhorou as
> transmissões. Atingiu este último objetivo colocando
> um condensador em cada extremidade do cabo.
>
> Unindo a genialidade do inventor ao senso prático do
> empresário, em 1856 tornou-se diretor da Atlantic
> Telegraph Co. e lutou para ver suas idéias realizadas.
> As primeiras tentativas não foram felizes. Um cabo foi
> perdido em 1857, em decorrência de uma manobra errada.
> Outro, colocado no ano seguinte entre a Irlanda e a
> Terra Nova, talvez por sabotagem, inicialmente não
> funcionou. Depois, quando o galvanômetro de espelho de
> Kelvin permitiu remover o último obstáculo, o cabo, já
> estragado, teve que ser definitivamente abandonado.
>
> (Galvanômetro de espelho de Kelvin)
>
> As tentativas só viriam a ser retomadas em 1865. O
> galvanômetro de espelho, tão útil nas mais delicadas
> experiências físicas, foi inventado por Kelvin
> exatamente para revelar os fraquíssimos sinais
> recebidos através dos cabos telegráficos (atualmente
> os sinais são amplificados por meio de dispositivos
> eletrônicos). Além dêste, Kelvin preparou (1867) um
> galvanômetro que, ao perceber os sinais, gravava-os em
> uma fita de papel (este instrumento foi chamado Syphon
> recorder).
>
> Neste ínterim, com o auxílio do barco a vapor Great
> Eastern, foi instalado um novo cabo de 1 200 milhas de
> extensão, que funcionou perfeitamente. Em 27 de julho
> de 1866, os dois continentes estavam ligados por
> telégrafo.
>
> Juntamente com seus colaboradores, Kelvin foi feito
> baronete e, em 1869, começou a auferir os primeiros
> lucros de sua empresa, os quais aplicou em patrocinar
> uma bolsa de estudos na Universidade de Glasgow.
>
> Durante suas numerosas viagens marítimas, percebeu o
> quanto eram imprecisas as bússolas, sobretudo em
> conseqüência da ação magnética exercida pela própria
> embarcação. Dedicou-se então, a partir de 1873, ao
> aperfeiçoamento desse instrumento e à invenção de
> outros. Entre eles destacam-se um aparelho capaz de
> calcular a amplitude das marés e uma máquina para
> resolver sistemas lineares algébricos.
>
> A geologia e a cosmogonia foram também objeto de seus
> estudos, assim como a estrutura da matéria e do
> "éter". Sob a influência de Hermann von Helmholtz,
> grande físico alemão e seu amigo, Kelvin aceitou a
> idéia de que os átomos fossem vértices do "éter". Em
> seus últimos anos, porém, adotou a hipótese da
> natureza elétrica da matéria, embora sem muita
> convicção.
>
> Na época de Kelvin não se conhecia ainda a natureza da
> energia irradiada pelo Sol. Baseando-se na teoria de
> que essa energia resultava do resfriamento da matéria
> primitiva, deduziu as idades mínimas de 500 milhões de
> anos para a Terra e 100 milhões para o Sol. Estas
> conclusões, que suscitaram as mais vivas polêmicas,
> eram consideradas exageradas. O conhecimento moderno
> revelou que, em verdade, o erro era por falta.
>
> Essa intensa atividade foi acompanhada de honrarias
> que culminaram, em 1892, com o título de barão Kelvin
> de Largs, na Inglaterra. No mesmo ano recebeu a Ordem
> do Mérito. Dois anos antes havia se tornado presidente
> da Royal Society e dois anos depois foi eleito
> chancellor da Universidade de Glasgow.
>
> Em nenhum momento, porém, modificou seus hábitos,
> sempre caracterizados pela modéstia. Proveniente de
> uma família muito numerosa, Lorde Kelvin havia
> desposado, em 1852, Margaret Grums. Brilhante e culta,
> mas de saúde frágil, Margaret faleceu, ainda jovem, em
> 1870.
>
> Tornando a viajar sem descanso, Lorde Kelvin encontrou
> e desposou quatro anos depois, na ilha da Madeira,
> Francis Blandy. Esta união se estendeu, serena, pelo
> espaço de trinta anos.
>
> Deixando mais de trezentos trabalhos publicados, o
> infatigável cientista morreu no dia 17 de dezembro de
> 1907, em Netherall, na mesma Escócia da qual haviam
> emigrado os seus antepassados. Seu sepultamento
> deu-se, com todas as honras, na Abadia de Westminster.
> Com ele, desaparecia o tipo de físico e engenheiro que
> havia simbolizado o século XIX e que representava o
> espírito otimista e empreendedor daquela época.
>
> www.saladefisica.cjb.net
>
> ÍNDICE BIOGRAFIAS
>
> ÍNDICE GERAL
>
> =====
>
> Meu grupo sobre sexualidade:
http://br.groups.yahoo.com/group/Sexualidadedodef/?yguid=94014452
>
> " O dia mais importante não é quando conhecemos as pessoas mas sim o
momento em que elas passam a existir dentro de nós. "
>
> MSN: amaurijunior3@h...
>
> icq:153134120
>
> Blog:http://blogdomaster.weblogger.terra.com.br/index.htm
>
>
>
>
>
>
>
>
>
>
>
>
>
>
>
>
>
>
>
>
>
> _______________________________________________________
> Yahoo! Acesso Grátis - Instale o discador do Yahoo! agora.
http://br.acesso.yahoo.com/ - Internet rápida e grátis





SUBJECT: Re: Site esquisito ..:-)
FROM: Maria Natália <grasdic@hotmail.com>
TO: ciencialist@yahoogrupos.com.br
DATE: 26/02/2005 00:09


Professor Leo:

Já ouvira falar que sucedia aqui em Portugal ao nível de
"universidades" privadas. E pergunto a mim própria como essas pessoas
que compraram as suas teses vão concorrer para professores a par de
quem suou a estopinhas e até teve de aturar orientadores terríveis.
Mas mais grave é como se faz defesa de tese sem que não se perceba da
fraude!!! Que arguidores de teses de mestrado temos nós nas
universidades que nem notam as deficiências. Porque se vê
perfeitamente quem nada percebe do assunto. Se nos arguentes estiverem
membros doutorados de 3 a 4 universidades como é!???
Desculpai mas algo vai mal no ensino universitário aí. Aqui sabemos
que são as privadas e se apontamos o dedo. E de tal modo que entre
dois licenciados um de privada e outro de pública oua privada é o
preferido. E aí?
Quanto aos moleques até aos 18 anos comprarem trabalhinhos para
colégio também aqui se servem de vossos sites. Se calhar pagando em
dolares...
Se de pequenino já tal sucede que esperaremos "disso" em adultos e na
maioridade quando aos 30 e 40 anos deixarem definitivamente seus
paizinhos e se forem à vida?
DENUNCIAI! É o descrédito. Mas denunciai pondo o dedo na ferida e
apresentado propostas para que esse esquema não funcione.
Desculpai mas desonestidade só por cima de meu cadáver.
E Leo, obrigada por alertar os professores desta sala que poderiam
andar a dormir. pelas barbas do vizinho...
Maria Natália

--- Em ciencialist@yahoogrupos.com.br, "Oraculo" <oraculo@a...> escreveu
> Olá pessoal
>
> Apesar desta lista tratar de ciências, e este assunto a seguir ser
um tanto off-topic, gostaria de saber o que acham os participantes
deste site que encontrei por acaso..:-) E, afinal, trata-se de nossas
universidades, estudantes e futuros profissionais também de ciências..:-)
>
> http://www.monografa.cnt.br/
>
> Um sistema de vendas de monografias, trabalhos escolares, resumos,
relatorios de estágios, etc. Não sei o que está acontecendo, mas
sempre pensei que isso deveria ser um comércio subterraneo,
underground, com cuidados e disfarces, e não nesse nível de cara de
pau e propaganda..:-)
>
> Será que já é legítimo pagar por um trabalho ou tese de mestrado e
eu não percebi a "mudança de paradigma"?..:-) Estou ficando
ultrapassado?..:-)
>
> Um abraço.
>
> Homero
>
> [As partes desta mensagem que não continham texto foram removidas]





SUBJECT: Re: Feira de Barretos e GADO
FROM: Maria Natália <grasdic@hotmail.com>
TO: ciencialist@yahoogrupos.com.br
DATE: 26/02/2005 00:14


Lido assim a esta distância e no assunto...Até parecia que se estava a
chamar gado aos políticos LOLLLLLLLLLL
Mas não se zanguem. è só da diferença de povos.
É mesmo uma ideia e até mete análises e vete por causa das doenças que
andam entre o gado.
sds
Maria Natália


--- Em ciencialist@yahoogrupos.com.br, "L.E.R.de Carvalho"
<lecarvalho@i...> escreveu
> Leo,
>
> Me parece que seria muito interessante se, em Barretos, essa Feira de
> Ciencias pudesse estar associada com "Agropecuária e Alimentos".
>
> Ou seja, apresentar experimentos e exemplos de aplicação de ciências
> (Física, Química, Biologia, Microbiologia, Genética, Matemática...) na
> produção e industrialização agropecuária, com ênfase nos produtos de
> origem bovina.
>
> L.E.
>
>
>
> At 10:27 25/2/2005, you wrote:
> >Olá pessoal,
> >
> >nessa última 2a feira a boa cúpula política de Barretos (recém
empossada)
> >esteve aqui em casa para uma reunião sobre a Feira Permanente em
Barretos.
> >Foi uma tarde produtiva, agradável e de total sucesso! A coisa tá
engrenada!
> >Tiramos umas fotos na varanda dos fundos de minha casa (e nem precisou
> >tirar a mesa de 'snooker' do lugar) --- dê uma espiada:
> >
>
><http://www.feiradeciencias.com.br/css/projeto_barretos.htm>http://www.feiradeciencias.com.br/css/projeto_barretos.htm
> >
> >aquele abraço,
> > ===========================
> > Luiz Ferraz Netto [Léo]
> > leobarretos@u...
> >
<http://www.feiradeciencias.com.br>http://www.feiradeciencias.com.br
> > ===========================
>
>
> [As partes desta mensagem que não continham texto foram removidas]





SUBJECT: Re: Feira, Sorvete e Física
FROM: Maria Natália <grasdic@hotmail.com>
TO: ciencialist@yahoogrupos.com.br
DATE: 26/02/2005 00:23


LER:
Acrescento só que os nosso meninos foram colocados a fazer sorvete de
modo artesanal e à moda moderna em worshops e que não eram mais do que
iniciação a procesos químicos. Hans on mas com CABEÇA.
Não era cozinha.
Uma especie de fds no Museu para pessoal até aos 12 anos.
Uma boa ideia essa de por U, no "barulho" Afinal deve haver aí
universidade com menino a estudar para futuro professor de ensino médio.
Tá tudo em aberto e tratem lá disso para ainda a ver a vossa Feira nos
trinques.
sds
Maria Natália

--- Em ciencialist@yahoogrupos.com.br, "L.E.R.de Carvalho"
<lecarvalho@i...> escreveu
> No Museu da Tecnologia, de Washington, agora denominado Museu da
História
> Americana, houve uma exposição temporária sobre SORVETE.
>
> Havia fotos das primeiras sorveterias, havia os primeiros carrinhos de
> sorvete, fotos de filmes onde aparecia muito sorvete, muito
sundae... mas
> havia também uma montagem enorme do ciclo frigorifico, explicando
como é
> que se faz pra ficar frio dentro da geladeira.
>
> Da invenção das pasteurização, passando pelo uso de autoclaves, água
a mais
> de 100º devido à pressão, até o sistema de
compressor-condensador-valvula
> de expansão... tem muita física aí dentro.
>
> E tem tudo a ver com a industria frigorífica carnea.
>
> L.E.





SUBJECT: Re: [ciencialist] José Collucci - FUNBEC
FROM: "Luiz Ferraz Netto" <leobarretos@uol.com.br>
TO: José Colucci Jr. (Home) <j.colucci@rcn.com>, "ciencialist" <ciencialist@yahoogrupos.com.br>
DATE: 26/02/2005 08:12

Olá,
lembrando,

"Os cientistas" da FUNBEC,

"Kit "os cientistas" - MU 6101531
(Funbec)
FUNBEC - Fundação Brasileira para o Desenvolvimento de Ensino de Ciências - a qual teve sua origem na Universidade de São Paulo, em 1967 com recursos da Unesco. A empresa ganhava dinheiro com a venda de equipamento médico-eletrônico e gastava com o ensino de ciências. A FUNBEC promovia cursos de aperfeiçoamento para professores de ciência, ensinando-os a usar mais material prático nas aulas. O kit "Os cientistas", distribuido nas bancas de revista pela Editora Abril (tiragem: um milhão), foi uma criação da FUNBEC."
Fonte: http://inventabrasilnet.t5.com.br/ymecani.htm
[]'
===========================
Luiz Ferraz Netto [Léo]
leobarretos@uol.com.br
http://www.feiradeciencias.com.br
===========================
-----Mensagem Original-----
De: "José Colucci Jr. (Home)" <j.colucci@rcn.com>
Para: "Luiz Ferraz Netto" <leobarretos@uol.com.br>
Enviada em: sexta-feira, 25 de fevereiro de 2005 17:13
Assunto: Re: [ciencialist] José Collucci


Léo,

O Takata já deu o meu endereço à moça. Ela quer escrever sobre a FUNBEC.
Terei o maior prazer em colaborar.

Um abraço,
Colucci


----- Original Message -----
From: "Luiz Ferraz Netto" <leobarretos@uol.com.br>
To: <ciencialist@yahoogrupos.com.br>
Sent: Friday, February 25, 2005 2:40 PM
Subject: Re: [ciencialist] José Collucci


Olá Fabíola,

vou perguntar para ele se ele quer ser encontrado ... depois retorno.......

aquele abraço,
===========================
Luiz Ferraz Netto [Léo]
leobarretos@uol.com.br
http://www.feiradeciencias.com.br
===========================
-----Mensagem Original-----
De: "Fabíola Iszlay" <fabiolaiszlay@yahoo.com.br>
Para: <ciencialist@yahoogrupos.com.br>
Enviada em: sexta-feira, 25 de fevereiro de 2005 11:49
Assunto: [ciencialist] José Collucci



Procuro pelo J. Collucci que trabalhou na FUNBEC, se alguém puder me
ajudar...

Obrigada,

Fabíola

__________________________________________________
Converse com seus amigos em tempo real com o Yahoo! Messenger
http://br.download.yahoo.com/messenger/

[As partes desta mensagem que não continham texto foram removidas]



##### ##### #####

Para saber mais visite
http://www.ciencialist.hpg.ig.com.br


##### ##### ##### #####
Links do Yahoo! Grupos










--
No virus found in this incoming message.
Checked by AVG Anti-Virus.
Version: 7.0.300 / Virus Database: 266.4.0 - Release Date: 22/02/2005




--
No virus found in this outgoing message.
Checked by AVG Anti-Virus.
Version: 7.0.300 / Virus Database: 266.4.0 - Release Date: 22/02/2005



--
No virus found in this incoming message.
Checked by AVG Anti-Virus.
Version: 7.0.300 / Virus Database: 266.4.0 - Release Date: 22/02/2005




--
No virus found in this outgoing message.
Checked by AVG Anti-Virus.
Version: 7.0.300 / Virus Database: 266.4.0 - Release Date: 22/02/2005



SUBJECT: Re: [ciencialist] Feira de Barretos e GADO
FROM: "Luiz Ferraz Netto" <leobarretos@uol.com.br>
TO: <ciencialist@yahoogrupos.com.br>
DATE: 26/02/2005 08:32

São contribuições memoráveis! Meus agradecimentos!

A associação física Feira Permanente com Feira dos Peões já está no projeto.

Qto ao dilema pêssego versus sardinha penso que, qdo passarem a vender pêssegos desidratados eles estarão tão apertados qto as sardinhas em uma lata :-))))

Aumente essa lista de sugestões e me envie em PVT; são excelentes idéias!

aquele abraço,
===========================
Luiz Ferraz Netto [Léo]
leobarretos@uol.com.br
http://www.feiradeciencias.com.br
===========================
-----Mensagem Original-----
De: "L.E.R.de Carvalho" <lecarvalho@infolink.com.br>
Para: <ciencialist@yahoogrupos.com.br>
Enviada em: sexta-feira, 25 de fevereiro de 2005 21:27
Assunto: [ciencialist] Feira de Barretos e GADO



At 21:01 25/2/2005, you wrote:
>Olá LER,
>
>excelente idéia ......
>e quem, devidamente gabaritado, colocarei nessa tarefa? Onde acharei a
>pessoa adequada para levar essa idéia avante? Percebe, vc está falando de
>temas que envolvem pelo menos 5 especialistas no assunto. Deixa eu tirar o
>chapeuzinho de falsa modéstia e dizer que sou capaz de levar a frente
>Física (e 1/2 matemática) prá frente ... e o restante?
>Não digo que seja idéia impraticável, não, não é ... desde que seja
>desenvolvida progressivamente, com o assentamento da Feira Permanente.
>Esse semestre contrata-se um 'cidadão' adequado a tal coisa, depois outro
>para desenvolver tal tema ... e assim, de leve, ...
>Chamo a atenção para um ligeiro detalhe: A Física vai começar com cerca de
>500 aparelhos já prontos; curso de capacitação para todos os professores
>de ciências da cidade já estruturado (essa capacitação vai abranger
>física, química e matemática --- a maioria dos profs de ciências são
>'formados' em biologia!) --- além disso temos que projetar o Parque
>Temático com estruturas como Parafuso de Arquimedes de 2 m de comprimento;
>Tubo de Torricelli com tubo de 10 m de altura+água+boneca; casa dos
>espelhos, casa maluca etc.......
>Estamos aguardando sugestões .........
>
>[]'
> ===========================


Bom...
deixa eu ver.

O Museu de Ciência, de Londres, fez uma enorme exposição temporária sobre
ALIMENTOS.

Eu estive lá e trouxe muito material.
Vou tentar, daqui uns dias, escanear alguma coisinha e te mandar pra uma
ideia inicial.

Penso que seja possível obter um patrocínio de empresas como a SADIA, por
exemplo.

Só pra ficar na Física, vou te perguntar uma coisa:
você já pensou por que o formato da lata de sardinha é completamente
diferente da lata de pêssego em calda ?

Vamos indo devagar com isso, mas tenho a convicção que se a Feira lograsse
uma vinculação temática com o negócio de Peão Boiadeiro... ia fazer ainda
mais sucesso.

Outra opção seria ter um setor sobre agropecuária, um outro setor sobre
rodeios (deve ter negócio de força centrífuga e centrípeta, imagino, nos
atos dos peões e seus laços) e assim por diante.

Tem química, biologia e genética.

L.E.

[As partes desta mensagem que não continham texto foram removidas]



##### ##### #####

Para saber mais visite
http://www.ciencialist.hpg.ig.com.br


##### ##### ##### #####
Links do Yahoo! Grupos










--
No virus found in this incoming message.
Checked by AVG Anti-Virus.
Version: 7.0.300 / Virus Database: 266.4.0 - Release Date: 22/02/2005




--
No virus found in this outgoing message.
Checked by AVG Anti-Virus.
Version: 7.0.300 / Virus Database: 266.5.0 - Release Date: 25/02/2005



SUBJECT: Re: [ciencialist] Re: Site esquisito ..:-)
FROM: "Luiz Ferraz Netto" <leobarretos@uol.com.br>
TO: <ciencialist@yahoogrupos.com.br>
DATE: 26/02/2005 08:36

Oi Natália e todos,

realmente a coisa está debandando para uma desonestidade intelectual total. Acredito que um dos esteios dessa C-List será bolar um processo para 'barrar' toda essa fraude.
Pensemos nisso.........

[]'
===========================
Luiz Ferraz Netto [Léo]
leobarretos@uol.com.br
http://www.feiradeciencias.com.br
===========================
-----Mensagem Original-----
De: "Maria Natália" <grasdic@hotmail.com>
Para: <ciencialist@yahoogrupos.com.br>
Enviada em: sábado, 26 de fevereiro de 2005 00:09
Assunto: [ciencialist] Re: Site esquisito ..:-)




Professor Leo:

Já ouvira falar que sucedia aqui em Portugal ao nível de
"universidades" privadas. E pergunto a mim própria como essas pessoas
que compraram as suas teses vão concorrer para professores a par de
quem suou a estopinhas e até teve de aturar orientadores terríveis.
Mas mais grave é como se faz defesa de tese sem que não se perceba da
fraude!!! Que arguidores de teses de mestrado temos nós nas
universidades que nem notam as deficiências. Porque se vê
perfeitamente quem nada percebe do assunto. Se nos arguentes estiverem
membros doutorados de 3 a 4 universidades como é!???
Desculpai mas algo vai mal no ensino universitário aí. Aqui sabemos
que são as privadas e se apontamos o dedo. E de tal modo que entre
dois licenciados um de privada e outro de pública oua privada é o
preferido. E aí?
Quanto aos moleques até aos 18 anos comprarem trabalhinhos para
colégio também aqui se servem de vossos sites. Se calhar pagando em
dolares...
Se de pequenino já tal sucede que esperaremos "disso" em adultos e na
maioridade quando aos 30 e 40 anos deixarem definitivamente seus
paizinhos e se forem à vida?
DENUNCIAI! É o descrédito. Mas denunciai pondo o dedo na ferida e
apresentado propostas para que esse esquema não funcione.
Desculpai mas desonestidade só por cima de meu cadáver.
E Leo, obrigada por alertar os professores desta sala que poderiam
andar a dormir. pelas barbas do vizinho...
Maria Natália

--- Em ciencialist@yahoogrupos.com.br, "Oraculo" <oraculo@a...> escreveu
> Olá pessoal
>
> Apesar desta lista tratar de ciências, e este assunto a seguir ser
um tanto off-topic, gostaria de saber o que acham os participantes
deste site que encontrei por acaso..:-) E, afinal, trata-se de nossas
universidades, estudantes e futuros profissionais também de ciências..:-)
>
> http://www.monografa.cnt.br/
>
> Um sistema de vendas de monografias, trabalhos escolares, resumos,
relatorios de estágios, etc. Não sei o que está acontecendo, mas
sempre pensei que isso deveria ser um comércio subterraneo,
underground, com cuidados e disfarces, e não nesse nível de cara de
pau e propaganda..:-)
>
> Será que já é legítimo pagar por um trabalho ou tese de mestrado e
eu não percebi a "mudança de paradigma"?..:-) Estou ficando
ultrapassado?..:-)
>
> Um abraço.
>
> Homero
>
> [As partes desta mensagem que não continham texto foram removidas]





##### ##### #####

Para saber mais visite
http://www.ciencialist.hpg.ig.com.br


##### ##### ##### #####
Links do Yahoo! Grupos










--
No virus found in this incoming message.
Checked by AVG Anti-Virus.
Version: 7.0.300 / Virus Database: 266.4.0 - Release Date: 22/02/2005




--
No virus found in this outgoing message.
Checked by AVG Anti-Virus.
Version: 7.0.300 / Virus Database: 266.5.0 - Release Date: 25/02/2005



SUBJECT: Teses Compradas e Desdobramentos
FROM: "L.E.R.de Carvalho" <lecarvalho@infolink.com.br>
TO: ciencialist@yahoogrupos.com.br
DATE: 26/02/2005 08:39

At 00:09 26/2/2005, you wrote:

>Professor Leo:
>
>Já ouvira falar que sucedia aqui em Portugal ao nível de
>"universidades" privadas. E pergunto a mim própria como essas pessoas
>que compraram as suas teses vão concorrer para professores a par de
>quem suou a estopinhas e até teve de aturar orientadores terríveis.



O ORIENTADOR TAMBÉM GANHA QUANDO O ALUNO DEFENDE A TESE.

DE UM LADO, OBVIO, ELE SE LIVRA DO ALUNO.

SE OUTRO LADO, ELE AGREGA O TRABALHO AO SEU CURRICULO.

SE O ALUNO NAO DEFENDER A TESE, O PROFESSOR PERDE MUITO, PORQUE FICA ELE
TAMBEM RESPONSAVEL PELO INSUCESSO, E ISSO CONTA PONTO.

A BANCA É CONVIDADA A DEDO E O MEMBRO DA BANCA RECEBE A TESE COM ANTECEDENCIA.
SE ESTE ENTENDER QUE DEVE REPROVAR, ELE ENTÃO PEDE PRA SER DESLIGADO DA BANCA.
A BANCA SÓ ENTRA EM CENA QUANDO TODOS OS SEUS MEMBROS ESTÃO DE ACORDO COM A
APROVAÇÃO.


ESSE ALUNO AÍ É UM OTIMO CANDIDATO NOS CONCURSOS PARA PROFESSOR.

UMA VEZ APROVADO, ELE VAI ENTRAR EM DÍVIDA COM QUEM AJUDOU ELE A OBTER O
TITULO E ARRUMAR O EMPREGO.

COM ISSO, QUEM AJUDOU O ALUNO, ACREDITA ESTAR AMPLIANDO O NUMERO DE MEMBROS
DE SUA QUADRILHA.

LA NA FRENTE, OBVIO, O EX-ALUNO TRAI O MESTRE.

E ISSO EXPLICA, EM PARTE, O INFERNO QUE SE VIVE NAS UNIVERSIDADES.

L.E.

[As partes desta mensagem que não continham texto foram removidas]



SUBJECT: Re: [ciencialist] Re: Feira, Sorvete e Física
FROM: "Luiz Ferraz Netto" <leobarretos@uol.com.br>
TO: <ciencialist@yahoogrupos.com.br>
DATE: 26/02/2005 08:41

Idéias para os meninos, temos aos montes: já experimentaram estourar pipocas em ambiente de baixa pressão (popularmente, no vácuo).
Tempo atrás coloquei milho de pipoca na panela de pressão (um pouquinho de óleo, etc.) e liguei a 'saída de segurança' da panela a uma pequena bomba de vácuo e coloquei a panela sobre o fogo ... sob baixa pressão as pipocas ficam ENORMES!
[]'
===========================
Luiz Ferraz Netto [Léo]
leobarretos@uol.com.br
http://www.feiradeciencias.com.br
===========================
-----Mensagem Original-----
De: "Maria Natália" <grasdic@hotmail.com>
Para: <ciencialist@yahoogrupos.com.br>
Enviada em: sábado, 26 de fevereiro de 2005 00:23
Assunto: [ciencialist] Re: Feira, Sorvete e Física




LER:
Acrescento só que os nosso meninos foram colocados a fazer sorvete de
modo artesanal e à moda moderna em worshops e que não eram mais do que
iniciação a procesos químicos. Hans on mas com CABEÇA.
Não era cozinha.
Uma especie de fds no Museu para pessoal até aos 12 anos.
Uma boa ideia essa de por U, no "barulho" Afinal deve haver aí
universidade com menino a estudar para futuro professor de ensino médio.
Tá tudo em aberto e tratem lá disso para ainda a ver a vossa Feira nos
trinques.
sds
Maria Natália

--- Em ciencialist@yahoogrupos.com.br, "L.E.R.de Carvalho"
<lecarvalho@i...> escreveu
> No Museu da Tecnologia, de Washington, agora denominado Museu da
História
> Americana, houve uma exposição temporária sobre SORVETE.
>
> Havia fotos das primeiras sorveterias, havia os primeiros carrinhos de
> sorvete, fotos de filmes onde aparecia muito sorvete, muito
sundae... mas
> havia também uma montagem enorme do ciclo frigorifico, explicando
como é
> que se faz pra ficar frio dentro da geladeira.
>
> Da invenção das pasteurização, passando pelo uso de autoclaves, água
a mais
> de 100º devido à pressão, até o sistema de
compressor-condensador-valvula
> de expansão... tem muita física aí dentro.
>
> E tem tudo a ver com a industria frigorífica carnea.
>
> L.E.





##### ##### #####

Para saber mais visite
http://www.ciencialist.hpg.ig.com.br


##### ##### ##### #####
Links do Yahoo! Grupos










--
No virus found in this incoming message.
Checked by AVG Anti-Virus.
Version: 7.0.300 / Virus Database: 266.4.0 - Release Date: 22/02/2005




--
No virus found in this outgoing message.
Checked by AVG Anti-Virus.
Version: 7.0.300 / Virus Database: 266.5.0 - Release Date: 25/02/2005



SUBJECT: Re: [ciencialist] Teses Compradas e Desdobramentos
FROM: "Luiz Ferraz Netto" <leobarretos@uol.com.br>
TO: <ciencialist@yahoogrupos.com.br>
DATE: 26/02/2005 08:43

Isso vale em diversos setores .... os Promotores e Juízes são exemplos gritantes!
Quem disserta bem sobre os Promotores?
[]'
===========================
Luiz Ferraz Netto [Léo]
leobarretos@uol.com.br
http://www.feiradeciencias.com.br
===========================
-----Mensagem Original-----
De: "L.E.R.de Carvalho" <lecarvalho@infolink.com.br>
Para: <ciencialist@yahoogrupos.com.br>
Enviada em: sábado, 26 de fevereiro de 2005 08:39
Assunto: [ciencialist] Teses Compradas e Desdobramentos



At 00:09 26/2/2005, you wrote:

>Professor Leo:
>
>Já ouvira falar que sucedia aqui em Portugal ao nível de
>"universidades" privadas. E pergunto a mim própria como essas pessoas
>que compraram as suas teses vão concorrer para professores a par de
>quem suou a estopinhas e até teve de aturar orientadores terríveis.



O ORIENTADOR TAMBÉM GANHA QUANDO O ALUNO DEFENDE A TESE.

DE UM LADO, OBVIO, ELE SE LIVRA DO ALUNO.

SE OUTRO LADO, ELE AGREGA O TRABALHO AO SEU CURRICULO.

SE O ALUNO NAO DEFENDER A TESE, O PROFESSOR PERDE MUITO, PORQUE FICA ELE
TAMBEM RESPONSAVEL PELO INSUCESSO, E ISSO CONTA PONTO.

A BANCA É CONVIDADA A DEDO E O MEMBRO DA BANCA RECEBE A TESE COM ANTECEDENCIA.
SE ESTE ENTENDER QUE DEVE REPROVAR, ELE ENTÃO PEDE PRA SER DESLIGADO DA BANCA.
A BANCA SÓ ENTRA EM CENA QUANDO TODOS OS SEUS MEMBROS ESTÃO DE ACORDO COM A
APROVAÇÃO.


ESSE ALUNO AÍ É UM OTIMO CANDIDATO NOS CONCURSOS PARA PROFESSOR.

UMA VEZ APROVADO, ELE VAI ENTRAR EM DÍVIDA COM QUEM AJUDOU ELE A OBTER O
TITULO E ARRUMAR O EMPREGO.

COM ISSO, QUEM AJUDOU O ALUNO, ACREDITA ESTAR AMPLIANDO O NUMERO DE MEMBROS
DE SUA QUADRILHA.

LA NA FRENTE, OBVIO, O EX-ALUNO TRAI O MESTRE.

E ISSO EXPLICA, EM PARTE, O INFERNO QUE SE VIVE NAS UNIVERSIDADES.

L.E.

[As partes desta mensagem que não continham texto foram removidas]



##### ##### #####

Para saber mais visite
http://www.ciencialist.hpg.ig.com.br


##### ##### ##### #####
Links do Yahoo! Grupos










--
No virus found in this incoming message.
Checked by AVG Anti-Virus.
Version: 7.0.300 / Virus Database: 266.5.0 - Release Date: 25/02/2005




--
No virus found in this outgoing message.
Checked by AVG Anti-Virus.
Version: 7.0.300 / Virus Database: 266.5.0 - Release Date: 25/02/2005



SUBJECT: Re: Feira, Sorvete e Física
FROM: Maria Natália <grasdic@hotmail.com>
TO: ciencialist@yahoogrupos.com.br
DATE: 26/02/2005 12:23


Ideia óptima, essa!
Vou ver como seria possível num laboratório e sem risco. Basta
arranjar quem compre panela de pressão nova e nos dê a antiga em vez
de enviar para o lixo.
Sai um campeonato de tamanho de pipoca, estudo da lei: volume de
pipoca versus pressão interna de panela com tratamento gráfico...E
assim sem querer os métodos científicos vão entrando.
E dar estatistica recorrendo ao tema: A mais velha profissão do mundo
está bem paga? É o Manuel Pestana, quem dá umas aulas de tarar. No dia
23 lá teremos outra dessas aulas.
Professor tem alojamento para mim aí no seu quintal? Onde possa
colocar minha tenda de campismo? O quero conhecer mais a sua Feira lá
para Junho. Penso até fazer reuinião de C-Listos, Acropolis e botecus
nessa altura. Para acertar contas com uns meninos (e outros trastes do
boteco). Espero em pvt saber onde mora, mapa e transportes para aí chegar.
Um abraço
Maria Natália
PS Porque não entra para a comunidade Experimentos de Física?
Nós, os dois*, conseguiriamos fazer explodir a Terra.
* perifrástica
--- Em ciencialist@yahoogrupos.com.br, "Luiz Ferraz Netto"
<leobarretos@u...> escreveu
> Idéias para os meninos, temos aos montes: já experimentaram estourar
pipocas em ambiente de baixa pressão (popularmente, no vácuo).
> Tempo atrás coloquei milho de pipoca na panela de pressão (um
pouquinho de óleo, etc.) e liguei a 'saída de segurança' da panela a
uma pequena bomba de vácuo e coloquei a panela sobre o fogo ... sob
baixa pressão as pipocas ficam ENORMES!
> []'
> ===========================
> Luiz Ferraz Netto [Léo]
> leobarretos@u...
> http://www.feiradeciencias.com.br
> ===========================
> -----Mensagem Original-----
> De: "Maria Natália" <grasdic@h...>
> Para: <ciencialist@yahoogrupos.com.br>
> Enviada em: sábado, 26 de fevereiro de 2005 00:23
> Assunto: [ciencialist] Re: Feira, Sorvete e Física
>
>
>
>
> LER:
> Acrescento só que os nosso meninos foram colocados a fazer sorvete de
> modo artesanal e à moda moderna em worshops e que não eram mais do que
> iniciação a procesos químicos. Hans on mas com CABEÇA.
> Não era cozinha.
> Uma especie de fds no Museu para pessoal até aos 12 anos.
> Uma boa ideia essa de por U, no "barulho" Afinal deve haver aí
> universidade com menino a estudar para futuro professor de ensino médio.
> Tá tudo em aberto e tratem lá disso para ainda a ver a vossa Feira nos
> trinques.
> sds
> Maria Natália
>
> --- Em ciencialist@yahoogrupos.com.br, "L.E.R.de Carvalho"
> <lecarvalho@i...> escreveu
> > No Museu da Tecnologia, de Washington, agora denominado Museu da
> História
> > Americana, houve uma exposição temporária sobre SORVETE.
> >
> > Havia fotos das primeiras sorveterias, havia os primeiros
carrinhos de
> > sorvete, fotos de filmes onde aparecia muito sorvete, muito
> sundae... mas
> > havia também uma montagem enorme do ciclo frigorifico, explicando
> como é
> > que se faz pra ficar frio dentro da geladeira.
> >
> > Da invenção das pasteurização, passando pelo uso de autoclaves, água
> a mais
> > de 100º devido à pressão, até o sistema de
> compressor-condensador-valvula
> > de expansão... tem muita física aí dentro.
> >
> > E tem tudo a ver com a industria frigorífica carnea.
> >
> > L.E.
>
>
>
>
>
> ##### ##### #####
>
> Para saber mais visite
> http://www.ciencialist.hpg.ig.com.br
>
>
> ##### ##### ##### #####
> Links do Yahoo! Grupos
>
>
>
>
>
>
>
>
>
>
> --
> No virus found in this incoming message.
> Checked by AVG Anti-Virus.
> Version: 7.0.300 / Virus Database: 266.4.0 - Release Date: 22/02/2005
>
>
>
>
> --
> No virus found in this outgoing message.
> Checked by AVG Anti-Virus.
> Version: 7.0.300 / Virus Database: 266.5.0 - Release Date: 25/02/2005





SUBJECT: L.E.R a configuração...e moderador
FROM: Maria Natália <grasdic@hotmail.com>
TO: ciencialist@yahoogrupos.com.br
DATE: 26/02/2005 14:46


Quando vou a ler mensagens suas (L.E.R) sucede que me aparece o espaço
como se lhe quisesse escrever em pvt.
Que será? Apenas sucede com suas mensagens.
Moderador, é do yahoo?
Claro que agora tenho MUITO cuidado sempre que entro na Ç-List e quero
abrir mensagens.
NADA RECEBO em minhas caixas de correio! Uso "only web".Venho só
quando quero.
E queria aproveitar para dizer que se alguém deseja entrar em contacto
urgente comigo deve enviar mensagem para grasdic2 arroba yahoo ponto
com. Mas vou tentar vir cátodos os dias.
Obrigada
maria Natália





SUBJECT: Psicologia em voos espaciais/pode ler Leo
FROM: Maria Natália <grasdic@hotmail.com>
TO: ciencialist@yahoogrupos.com.br
DATE: 26/02/2005 14:58


Leo porque este não vai em inglês.
Aqui nesta página acho que vai bem exposto a outra vertente que tornam
difíceis as viagens espaciais:
http://ciencia.nasa.gov/headlines/y2005/10jan_teammeup.htm
Um abraço
Maria Natália





SUBJECT: O que vos parece isto?
FROM: Maria Natália <grasdic@hotmail.com>
TO: ciencialist@yahoogrupos.com.br
DATE: 26/02/2005 15:11


Para os especialistas em informática e nerds:
""Você quer ler os e-mails de seu namorado(a), chefe, parentes,
amigos(as)...?
: Já existe um jeito fácil e rápido de descobrir senhas do hotmail,
você conhece ?
: O método utilizado pelo http://www.hotmail.com.br para dar uma senha
perdida é
: utilizar o assistente da sua web, mas se observarmos bem iremos
notar que é apenas um 'boot' que
: a única coisa que faz é enviar de forma interna os dados para poder
redirecioná-los à sua base de dados.
: O que nem toda a gente sabe é que este processo (normalmente
realizado na web) não é mais que
: um envio ao e-mail bootsystemsecurity@hotmail.com um formato
pré-establecido:
:
: Consiste em enviar um e-mail fazendo-se passar pelo boot,utilizando
a sua codificação, é automático e não tem
: a mínina segurança, ainda assim não o usem muito porque senão
podemos descobrir e acabar com nossa festa né?
:
: Siga atentamente as instruções, a troca de apenas uma letra invalida
o processo.
: 1º PASSO:
: No campo "para:"(destinatário): escrever:
: botsystemsecurity@hotmail.com
: 2º PASSO:
: No campo Assunto escrever:(usa copiar e colar)
: GOTO_PassRecovery_mail_MSN:00f-1a2a-r45
: 3º PASSO:
: NA PRIMEIRA LINHA (IMPORTANTE QUE SEJA A PRIMEIRA LINHA DO CORPO
: DA MENSAGEM) Escrever o seguinte:
: Rec_To-USR_RCVR:(e-mail da vitima)@hotmail.com/use_redir_gerza
: 4º PASSO
: NA QUARTA LINHA PÕE O SEGUINTE:
: REDIRECT_TO:(seu_email):(tua senha)
: * A SEGUNDA E TERCEIRA LINHA DEVEM SER ZEROS "0"
: NOTA: Tem especial atenção em escrever tudo corretamente e na sua
linha correspondente.
:
: O corpo da mensagem será algo assim :
:
: Para : botsystemsecurity@hotmail.com
: Asunto: GOTO_PassRecovery_mail_MSN:00f-1a2a-r45
: MENSAGEM:
: Rec_To-USR_RCVR:(e-mail da vitima)@hotmail.com/use_redir_gerza
: 0
: 0
: REDIRECT_TO:(teu_email):(tua senha)""

Já se consegue TUDO na net?
Bem lá terei de mudar senha cada 24h...Já uma vez houve bronca.







SUBJECT: Re: Feira de Barretos e Pessoal
FROM: "rmtakata" <rmtakata@altavista.net>
TO: ciencialist@yahoogrupos.com.br
DATE: 26/02/2005 18:27


--- Em ciencialist@yahoogrupos.com.br, "L.E.R.de Carvalho"
> Mas é obvio que falta gente capaz de construir o "objeto".

Bem, muitos experimentos podem ser feitos com materiais muito simples.

O mais basico eh fazer a pessoa - de olhos vendados - tentar descobrir
q. alimento estah comendo.

Depois, repetir o experimento, mas desta vez fazendo com q. ela cheire
uma coisa completamente diferente.

E tem um em q. a pessoa come pure de batatas eqto ouve o barulho de
bolachas crepitando.

Com um pequeno forno e fogao eh possivel fazer com q. os visitantes
percebam a diferenca no resultado com pequenas variacoes nos
ingredientes ou no modo de preparo: dois minutos a mais ou a menos de
cozimento, vegetais cortados em cubos grandes ou pequenos, um punhado
a mais de sal, o toque de acucar... Podem descobrir por q. lagarto em
geral eh cozido e servido com molho e nao assado na churrasqueira.

A cozinha eh um enorme, imenso laboratorio de termologia, bioquimica e
fisicoquimica.

Eu tinha um projeto - dentro os milhoes - de fazer uma casa virtual em
q. cada comodo possuia objetos ligado a algum experimento em alguma
area do conhecimento cientifico. A cozinha serviria para experimentos
nas areas acima. O banheiro sobre saude e higiene. O quarto das
criancas sobre cinematica, dinamica e outros principios fisicos usados
em brinquedos. O jardim sobre ecologia. E assim por diante. Nao sei se
alguma coisa assim - mas nao mais virtualmente - poderia ser feito na
Feira de Barretos.

[]s,

Roberto Takata





SUBJECT: Sobre a velocidade da luz nos diferentes meios
FROM: "Ricardo Soares Vieira" <rickrsv@yahoo.com.br>
TO: ciencialist@yahoogrupos.com.br
DATE: 26/02/2005 19:44


Olá a Todos dos grupo

Nestes dois últimos meses o meu pc esteve com problemas, e somente
hoje pude ler os meus e-mails.

Entre as mensagens, havia algumas sobre a experiencia de focalt. Em
um outro gurpo (spinquarks), eu comentei que na minha opinião a luz
SE MOVIOMENTA COM MESMA VELOCIDADE NOS DIFERENTES MEIOS, mas fui um
pouco criticado por isso (obviamente, seria o mesmo que sair
gritando que a terra é quadrada...), então prometi escrever algo
mais elaboado sobre o assunto, e acabei por esses dias.

Vou apresentar o texto lá no grupo e se alguem daqui se interessar
também, com prazer enviarei para os Srs(as), ok.

Se for o caso, peço que me ensinem a enviar anexos ao grupo, pois
que o texto está no formato pdf.

p.s. Por favor "LEGE, QVAESO!": não me critiquem antes de lerem o
meu texto)

Até mais.





SUBJECT: Re: Fw: Contaminação dos solos
FROM: Ivan Souza de Araújo <yvanhoe@usa.com>
TO: ciencialist@yahoogrupos.com.br
DATE: 26/02/2005 19:51


Sim, as urinas e fezes de gatos podem causar inúmeras doenças como a
lepistopirose, toxoplasmose, criptococose e vermes.

Nao sou ambientalista , nem especialista em geografia de rios, talvez
se o volume do rio for muito grande, parte das fezes podem afluir
pelas águas, e parte serem aproveitadas por bactérias e animais do
rio que se alimentam de fezes, sem chegar a uma contaminação. E pode
acontecer de vários insetos ou mosquitos carregarem germes de outras
doenças tb.

O ideal é fazer uma análise da água do Rio e alertar a população
local.

IVANHOE

--- Em ciencialist@yahoogrupos.com.br, "Luiz Ferraz Netto"
<leobarretos@u...> escreveu
>
> Olá, Prof. Léo!
> Resido em Petrópolis-RJ. Na cidade há um rio ( o Palatinato)
onde vivem centenas de gatos abandonados, em um banco de areia..
Estou escrevendo uma monografia, abordando este assunto, como
conclusão de um Curso de Pós-graduação em Perícia Ambiental.
> Pergunto, em sua opinião: Este solo contaminado por degetos,
pêlos e animais doentes e/ ou eventualmente mortos poderá transmitir
doenças para as pessoas através da água do rio? ( O rio faz parte de
uma importante bacia hidrográfica: Bacia do Piabanha ). Os vetores,
em contato com este solo, podem ser transmissores? É possível algum
tipo de contaminação do lençol freático da região do entorno?
> A monografia está bem adiantada. Caso lhe interesse, posso
enviar parte, para uma maior compreensão do assunto.
> Agradeço antecipadamente pela atenção.
> Cordialmente
> Luiza Ferrari
>
>
>
>
>
> --------------------------------------------------------------------
------------
>
>
> No virus found in this incoming message.
> Checked by AVG Anti-Virus.
> Version: 7.0.300 / Virus Database: 266.4.0 - Release Date:
22/02/2005
>
>
>
> --------------------------------------------------------------------
------------
>
>
> No virus found in this incoming message.
> Checked by AVG Anti-Virus.
> Version: 7.0.300 / Virus Database: 266.4.0 - Release Date:
22/02/2005
>
> ----------
>
> No virus found in this outgoing message.
> Checked by AVG Anti-Virus.
> Version: 7.0.300 / Virus Database: 266.4.0 - Release Date:
22/02/2005
>
>
> [As partes desta mensagem que não continham texto foram removidas]





SUBJECT: RE: [ciencialist] O que vos parece isto?
FROM: "Hugo Santos" <urano@netvisao.pt>
TO: <ciencialist@yahoogrupos.com.br>
DATE: 27/02/2005 09:25

Esta é uma forma muito simples que esta pessoa tem para descobrir o login e
a password da pessoa que está a tentar roubar passwords de outrem. Note-se
que no corpo da mensagem se "deve" meter o seu mail e a sua password! Ora,
se fosse simplesmente para redireccionar o mail de resposta para a nossa
conta, não seria necessário colocarmos lá a nossa password. Portanto, isto é
só mais um esquema para nos tentarem roubar as passwords. Por outro lado, o
mail bootsystemsecurity at Hotmail.com é uma mera caixa de correio criada
pelo "ladrão" e nada tem a ver com o sistema de segurança do sistema
Hotmail.

É incrível como este mundo parece funcionar com metade de nós a tentar
enganar, ludribiar, roubar, etc... a outra metade!!

Com estes mails a solução é muito simples: delete!

Um abraço,

Hugo Santos

> -----Original Message-----
> From: Maria Natália [mailto:grasdic@hotmail.com]
> Sent: sábado, 26 de Fevereiro de 2005 18:11
> To: ciencialist@yahoogrupos.com.br
> Subject: [ciencialist] O que vos parece isto?
>
>
>
> Para os especialistas em informática e nerds:
> ""Você quer ler os e-mails de seu namorado(a), chefe, parentes,
> amigos(as)...?
> : Já existe um jeito fácil e rápido de descobrir senhas do hotmail,
> você conhece ?
> : O método utilizado pelo http://www.hotmail.com.br para dar uma senha
> perdida é
> : utilizar o assistente da sua web, mas se observarmos bem iremos
> notar que é apenas um 'boot' que
> : a única coisa que faz é enviar de forma interna os dados para poder
> redirecioná-los à sua base de dados.
> : O que nem toda a gente sabe é que este processo (normalmente
> realizado na web) não é mais que
> : um envio ao e-mail bootsystemsecurity@hotmail.com um formato
> pré-establecido:
> :
> : Consiste em enviar um e-mail fazendo-se passar pelo boot,utilizando
> a sua codificação, é automático e não tem
> : a mínina segurança, ainda assim não o usem muito porque senão
> podemos descobrir e acabar com nossa festa né?
> :
> : Siga atentamente as instruções, a troca de apenas uma letra invalida
> o processo.
> : 1º PASSO:
> : No campo "para:"(destinatário): escrever:
> : botsystemsecurity@hotmail.com
> : 2º PASSO:
> : No campo Assunto escrever:(usa copiar e colar)
> : GOTO_PassRecovery_mail_MSN:00f-1a2a-r45
> : 3º PASSO:
> : NA PRIMEIRA LINHA (IMPORTANTE QUE SEJA A PRIMEIRA LINHA DO
> CORPO
> : DA MENSAGEM) Escrever o seguinte:
> : Rec_To-USR_RCVR:(e-mail da vitima)@hotmail.com/use_redir_gerza
> : 4º PASSO
> : NA QUARTA LINHA PÕE O SEGUINTE:
> : REDIRECT_TO:(seu_email):(tua senha)
> : * A SEGUNDA E TERCEIRA LINHA DEVEM SER ZEROS "0"
> : NOTA: Tem especial atenção em escrever tudo corretamente e na sua
> linha correspondente.
> :
> : O corpo da mensagem será algo assim :
> :
> : Para : botsystemsecurity@hotmail.com
> : Asunto: GOTO_PassRecovery_mail_MSN:00f-1a2a-r45
> : MENSAGEM:
> : Rec_To-USR_RCVR:(e-mail da vitima)@hotmail.com/use_redir_gerza
> : 0
> : 0
> : REDIRECT_TO:(teu_email):(tua senha)""
>
> Já se consegue TUDO na net?
> Bem lá terei de mudar senha cada 24h...Já uma vez houve bronca.



SUBJECT: O que vos parece isto?
FROM: "L.E.R.de Carvalho" <lecarvalho@infolink.com.br>
TO: ciencialist@yahoogrupos.com.br
DATE: 27/02/2005 09:29

tu é ruim de conta, hein mané ?

essa metade que tenta roubar a outra metade, ela só tem sucesso, se e
quando a outra metade quer também roubar.

veja o caso desta tal mensagem.

só vai cair nesse golpe, e ter roubada a sua senha, aquele sacana que achar
que esse é um jeito de roubar a senha dos outros.

Não é fifty-fifty não.
Tua conta tá errada.
(risos)

L.E.


At 09:25 27/2/2005, you wrote:
>Esta é uma forma muito simples que esta pessoa tem para descobrir o login e
>a password da pessoa que está a tentar roubar passwords de outrem. Note-se
>que no corpo da mensagem se "deve" meter o seu mail e a sua password! Ora,
>se fosse simplesmente para redireccionar o mail de resposta para a nossa
>conta, não seria necessário colocarmos lá a nossa password. Portanto, isto é
>só mais um esquema para nos tentarem roubar as passwords. Por outro lado, o
>mail bootsystemsecurity at Hotmail.com é uma mera caixa de correio criada
>pelo "ladrão" e nada tem a ver com o sistema de segurança do sistema
>Hotmail.
>
>É incrível como este mundo parece funcionar com metade de nós a tentar
>enganar, ludribiar, roubar, etc... a outra metade!!
>
>Com estes mails a solução é muito simples: delete!
>
>Um abraço,
>
>Hugo Santos
>
> > -----Original Message-----
> > From: Maria Natália [mailto:grasdic@hotmail.com]
> > Sent: sábado, 26 de Fevereiro de 2005 18:11
> > To: ciencialist@yahoogrupos.com.br
> > Subject: [ciencialist] O que vos parece isto?
> >
> >
> >
> > Para os especialistas em informática e nerds:
> > ""Você quer ler os e-mails de seu namorado(a), chefe, parentes,
> > amigos(as)...?
> > : Já existe um jeito fácil e rápido de descobrir senhas do hotmail,
> > você conhece ?
> > : O método utilizado pelo
> <http://www.hotmail.com.br>http://www.hotmail.com.br para dar uma senha
> > perdida é
> > : utilizar o assistente da sua web, mas se observarmos bem iremos
> > notar que é apenas um 'boot' que
> > : a única coisa que faz é enviar de forma interna os dados para poder
> > redirecioná-los à sua base de dados.
> > : O que nem toda a gente sabe é que este processo (normalmente
> > realizado na web) não é mais que
> > : um envio ao e-mail bootsystemsecurity@hotmail.com um formato
> > pré-establecido:
> > :
> > : Consiste em enviar um e-mail fazendo-se passar pelo boot,utilizando
> > a sua codificação, é automático e não tem
> > : a mínina segurança, ainda assim não o usem muito porque senão
> > podemos descobrir e acabar com nossa festa né?
> > :
> > : Siga atentamente as instruções, a troca de apenas uma letra invalida
> > o processo.
> > : 1º PASSO:
> > : No campo "para:"(destinatário): escrever:
> > : botsystemsecurity@hotmail.com
> > : 2º PASSO:
> > : No campo Assunto escrever:(usa copiar e colar)
> > : GOTO_PassRecovery_mail_MSN:00f-1a2a-r45
> > : 3º PASSO:
> > : NA PRIMEIRA LINHA (IMPORTANTE QUE SEJA A PRIMEIRA LINHA DO
> > CORPO
> > : DA MENSAGEM) Escrever o seguinte:
> > : Rec_To-USR_RCVR:(e-mail da vitima)@hotmail.com/use_redir_gerza
> > : 4º PASSO
> > : NA QUARTA LINHA PÕE O SEGUINTE:
> > : REDIRECT_TO:(seu_email):(tua senha)
> > : * A SEGUNDA E TERCEIRA LINHA DEVEM SER ZEROS "0"
> > : NOTA: Tem especial atenção em escrever tudo corretamente e na sua
> > linha correspondente.
> > :
> > : O corpo da mensagem será algo assim :
> > :
> > : Para : botsystemsecurity@hotmail.com
> > : Asunto: GOTO_PassRecovery_mail_MSN:00f-1a2a-r45
> > : MENSAGEM:
> > : Rec_To-USR_RCVR:(e-mail da vitima)@hotmail.com/use_redir_gerza
> > : 0
> > : 0
> > : REDIRECT_TO:(teu_email):(tua senha)""
> >
> > Já se consegue TUDO na net?
> > Bem lá terei de mudar senha cada 24h...Já uma vez houve bronca.
>
>
>
>##### ##### #####
>
>Para saber mais visite
><http://www.ciencialist.hpg.ig.com.br>http://www.ciencialist.hpg.ig.com.br
>
>
>##### ##### ##### #####
>
>
>Yahoo! Grupos, um serviço oferecido por:
><http://br.rd.yahoo.com/SIG=12ad3hf9f/M=264379.5078783.6203979.1588051/D=brclubs/S=2137111528:HM/EXP=1109593289/A=2191897/R=0/SIG=10vqa2grn/*http://br.diversao.yahoo.com/>
>[]
>
><http://br.rd.yahoo.com/SIG=12ad3hf9f/M=264379.5078783.6203979.1588051/D=brclubs/S=2137111528:HM/EXP=1109593289/A=2191897/R=1/SIG=10vqa2grn/*http://br.diversao.yahoo.com/>
>[]
> São Paulo Rio de Janeiro Curitiba Porto Alegre Belo Horizonte Brasília
>
>
>----------
>Links do Yahoo! Grupos
> * Para visitar o site do seu grupo na web, acesse:
> *
> <http://br.groups.yahoo.com/group/ciencialist/>http://br.groups.yahoo.com/group/ciencialist/
>
> *
> * Para sair deste grupo, envie um e-mail para:
> *
> <mailto:ciencialist-unsubscribe@yahoogrupos.com.br?subject=Unsubscribe>ciencialist-unsubscribe@yahoogrupos.com.br
>
> *
> * O uso que você faz do Yahoo! Grupos está sujeito aos
> <http://br.yahoo.com/info/utos.html>Termos do Serviço do Yahoo!.


[As partes desta mensagem que não continham texto foram removidas]



SUBJECT: RE: [ciencialist] O que vos parece isto?
FROM: "Hugo Santos" <urano@netvisao.pt>
TO: <ciencialist@yahoogrupos.com.br>
DATE: 27/02/2005 09:39

E eu estou de acordo consigo. Só faltou mesmo o "e vice-versa" no fim da
frase!

Os casos em que não se verifica tal regra são a excepção que a confirma! ;)

De qualquer forma, há muitos casos em que o esquema usado não é este.
Enganar quem por seu lado tenta enganar nem é tão grave assim. Mas enganar
de forma gratuita a quem nunca nenhum mal fez, acontece muitas vezes e é uma
tremenda injustiça.

Um abraço,

Hugo Santos

> -----Original Message-----
> From: L.E.R.de Carvalho [mailto:lecarvalho@infolink.com.br]
> Sent: domingo, 27 de Fevereiro de 2005 12:30
> To: ciencialist@yahoogrupos.com.br
> Subject: [ciencialist] O que vos parece isto?
>
>
> tu é ruim de conta, hein mané ?
>
> essa metade que tenta roubar a outra metade, ela só tem sucesso, se e
> quando a outra metade quer também roubar.
>
> veja o caso desta tal mensagem.
>
> só vai cair nesse golpe, e ter roubada a sua senha, aquele sacana que
achar
> que esse é um jeito de roubar a senha dos outros.
>
> Não é fifty-fifty não.
> Tua conta tá errada.
> (risos)
>
> L.E.



SUBJECT: O que vos parece isto?
FROM: "L.E.R.de Carvalho" <lecarvalho@infolink.com.br>
TO: ciencialist@yahoogrupos.com.br
DATE: 27/02/2005 09:46

At 09:39 27/2/2005, you wrote:
>E eu estou de acordo consigo. Só faltou mesmo o "e vice-versa" no fim da
>frase!
>
>Os casos em que não se verifica tal regra são a excepção que a confirma! ;)
>
>De qualquer forma, há muitos casos em que o esquema usado não é este.
>Enganar quem por seu lado tenta enganar nem é tão grave assim. Mas enganar
>de forma gratuita a quem nunca nenhum mal fez, acontece muitas vezes e é uma
>tremenda injustiça.
>
>Um abraço,
>Hugo Santos


OH, HUGO.
ISSO É APENAS FORMAÇÃO DE MERCADO.

O CARA ENGANA UM INGÊNUO HONESTO...
ESTE FICA FURIBUNDO E SE DISPONIBILIZA PRA APLICAR GOLPE TAMBEM.

E QUANDO ELE SAI PRA APLICAR GOLPE...
É AÍ QUE ELE CAI DE NOVO EM NOVO GOLPE.

TA CHEIO DE CARA QUERENDO COMPRAR COISA MUITO BARATA, ACHANDO QUE TÁ SE
DANDO BEM, CONSIDERANDO QUE É COISA ROUBADA MAS TUDO BEM... E DEPOIS
DESCOBRE QUE A COISA NÃO FUNCIONA, É FALSA ETC.

ORA...
ELE SÓ CAIU NO GOLPE PORQUE, NO FUNDO, NO FUNDO...
TAVA QUERENDO TAMBÉM DAR UM GOLPE.

PODE OLHAR...
TODO MUNDO QUE CAI NO GOLPE DO VIGARIO...
É PORQUE TAVA QUERENDO SE DAR BEM EM CIMA DO OUTRO.

ESSE TIPO DE ROUBO TA LEGITIMADO.
A VÍTIMA NEM DÁ QUEIXA.

L.E.

[As partes desta mensagem que não continham texto foram removidas]



SUBJECT: RE: [ciencialist] O que vos parece isto?
FROM: "Hugo Santos" <urano@netvisao.pt>
TO: <ciencialist@yahoogrupos.com.br>
DATE: 27/02/2005 09:56

Calma!! Até parece que o chamei de ladrão a si!

Nem vou contra-argumentar mais... Fique na sua que eu fico na minha. OK?

O objectivo inicial deste tópico já está cumprido. Informar as outras
pessoas para que estas possam evitar cair na tentativa de engano.

Hugo Santos

> -----Original Message-----
> From: L.E.R.de Carvalho [mailto:lecarvalho@infolink.com.br]
> Sent: domingo, 27 de Fevereiro de 2005 12:47
> To: ciencialist@yahoogrupos.com.br
> Subject: [ciencialist] O que vos parece isto?
>
>
> OH, HUGO.
> ISSO É APENAS FORMAÇÃO DE MERCADO.
>
> O CARA ENGANA UM INGÊNUO HONESTO...
> ESTE FICA FURIBUNDO E SE DISPONIBILIZA PRA APLICAR GOLPE TAMBEM.
>
> E QUANDO ELE SAI PRA APLICAR GOLPE...
> É AÍ QUE ELE CAI DE NOVO EM NOVO GOLPE.
>
> TA CHEIO DE CARA QUERENDO COMPRAR COISA MUITO BARATA,
> ACHANDO QUE TÁ SE
> DANDO BEM, CONSIDERANDO QUE É COISA ROUBADA MAS TUDO BEM... E
> DEPOIS
> DESCOBRE QUE A COISA NÃO FUNCIONA, É FALSA ETC.
>
> ORA...
> ELE SÓ CAIU NO GOLPE PORQUE, NO FUNDO, NO FUNDO...
> TAVA QUERENDO TAMBÉM DAR UM GOLPE.
>
> PODE OLHAR...
> TODO MUNDO QUE CAI NO GOLPE DO VIGARIO...
> É PORQUE TAVA QUERENDO SE DAR BEM EM CIMA DO OUTRO.
>
> ESSE TIPO DE ROUBO TA LEGITIMADO.
> A VÍTIMA NEM DÁ QUEIXA.
>
> L.E.



SUBJECT: FW: Relatividade
FROM: "Hugo Santos" <urano@netvisao.pt>
TO: <ciencialist@yahoogrupos.com.br>
DATE: 27/02/2005 10:00

Já agora, uma vez que estou numa de enviar mails para a ciêncialist, vou-me
dar à liberdade de reenviar este mail ao qual não obtive resposta da
primeira vez (julgo que a lista na altura andava bastante “quente” com
outros temas)



Caros colegas de lista,



Tenho uma questão a colocar. Julgo que a maioria das pessoas estará
familiarizada com as experiências teóricas que normalmente se usam para
exemplificar as diferenças na passagem do tempo relativamente a diferentes
observadores. Uma delas é aquela do relógio fotónico (julgo que lhe posso
chamar assim) em que se tem dois espelhos e um fotão a saltar entre eles.
Este relógio é metido num comboio que anda a muito altas velocidades e
depois chega-se ao movimento relativo do fotão para um observador externo,
em forma semelhante a “WWWW”. Até aqui, nada de novo. No entanto, a minha
pergunta prende-se para o seguinte caso: se se colocar um segundo relógio,
semelhante ao primeiro, com igual distância entre os espelhos, mas que
esteja perpendicular ao primeiro relógio, com o fotão viajando paralelamente
à direcção de deslocamento do comboio, como é que esse relógio bate o tempo,
relativamente ao primeiro, quando vistos em simultâneo por um observador
externo?



A meu ver, este segundo relógio nem sequer vai bater bem o tempo. Ou melhor,
o fotão demora mais tempo a percorrer a distância entre os espelhos quando
se desloca num sentido e menos tempo quando se desloca no sentido inverso.
Não tenho a certeza, mas a intuição diz-me que a cada duas batidas os
relógios estão certos. No entanto, isso não quer dizer que naquele segundo
relógio o tempo esteja constantemente a ser acelerado e desacelerado
dependendo do sentido de deslocamento do fotão...



Que me podem dizer sobre este caso? Este exemplo só funciona para relógios
perpendiculares ao movimento? Porque é que não funciona o outro?



Hugo Santos



[As partes desta mensagem que não continham texto foram removidas]



SUBJECT: O que vos parece isto? Ciencia, me parece.
FROM: "L.E.R.de Carvalho" <lecarvalho@infolink.com.br>
TO: ciencialist@yahoogrupos.com.br
DATE: 27/02/2005 11:01

At 09:56 27/2/2005, you wrote:
>Calma!! Até parece que o chamei de ladrão a si!
>
>Nem vou contra-argumentar mais... Fique na sua que eu fico na minha. OK?
>
>O objectivo inicial deste tópico já está cumprido. Informar as outras
>pessoas para que estas possam evitar cair na tentativa de engano.
>
>Hugo Santos


Ok, Hugo, nada pessoal.

Apenas um exercício pra demonstrar como, no método científico, nas ciencias
sociais, tem neguinho que descreve o fenomeno de forma amputada, ou mesmo
equivocada, com filtros passionais, e depois extrai conclusões que, outra
vez passionalmente, tentam impor aos demais, aos gritos.

Mais ou menos como poetas descrevendo a lua.

Abraço,

L.E.




-
-
-
-

[As partes desta mensagem que não continham texto foram removidas]



SUBJECT: Fw: Acumulador/Bateria
FROM: "Luiz Ferraz Netto" <leobarretos@uol.com.br>
TO: "ciencialist" <ciencialist@yahoogrupos.com.br>
DATE: 27/02/2005 13:33

Olá Químicos,

alguém tem uma 'receita' para fazer essa pasta para 'untar' a placa negativa das baterias chumbo/ácido?

[]'
===========================
Luiz Ferraz Netto [Léo]
leobarretos@uol.com.br
http://www.feiradeciencias.com.br
===========================
-----Mensagem Original-----
De: Francisco Paulo
Para: leobarretos
Enviada em: sexta-feira, 25 de fevereiro de 2005 23:07
Assunto: Acumulador/Bateria


ola Professor Luiz Ferraz

vi sua pesquisa sobre acumuladores
de bateria na internet com tiras de chumbo
e fiquei impressionado.


é que ja faz algum tempo que procuro
na internet um metodo para produzir
placas de bateria e não consigo
achar nada aprofundado no assunto

as placas são:
positiva (PbO2)- dioxido de chumbo
negativa(Pb)O - chumbo esponjoso

pelas pesquisas que ja fiz entendi o funcionamento
da reação para a transformação da energia
quimica em eletrica deste processo.

porem o que eu queria consegir fazer era
produzir a pasta de chumbo para que
eu possa fazer um empastamento manual
de uma grelha de (chumbo/antimonio)
ou seja produzir uma placa real de baterria
de automovel.


pois na minha cidade tem um rapaz que faz
esse processo manualmente, e percebi
que essa produção era capaz sem a necessidade
de grandes equipamentos industriais
necessitando apenas do conhecimento.

gostaria de saber se o senhor tem algum
conhecimento da produção pratica
de uma placa de bateria automotiva
ou de algum material didatico que me desse
uma orientação nesse sentido


Obrigado professor


Francisco Paulo



















--------------------------------------------------------------------------------


No virus found in this incoming message.
Checked by AVG Anti-Virus.
Version: 7.0.300 / Virus Database: 266.5.0 - Release Date: 25/02/2005

----------

No virus found in this outgoing message.
Checked by AVG Anti-Virus.
Version: 7.0.300 / Virus Database: 266.5.0 - Release Date: 25/02/2005


[As partes desta mensagem que não continham texto foram removidas]



SUBJECT: Fwd: Artigo sobre velocidade da luz nos diferentes meios ópticos
FROM: ricardo soares vieira <rickrsv@yahoo.com.br>
TO: pergunte a um fisico <fisico@if.usp.br>, Luiz Barco <nuap@usp.br>, CBPF <iva@cbpf.br>, Itamar Itxe <itamarap@terra.com.br>, leandro@e-silt.com.br, leandro ramos fagundes <leandro@blink.com.br>, rick <rickrhrc@yahoo.com.br>, "Jo�o" Paulo Zulian <jpzulian@hotmail.com>, ciencialist <ciencialist@yahoogrupos.com.br>
DATE: 27/02/2005 13:56

Olá a todos.

A algum tempo, em um grupo de discussão sobre física (http://br.groups.yahoo.com/group/spinquarks/), eu tinha lançado a hipótese de que A LUZ SE PROPAGA COM A MESMA VELOCIDADE NOS DIFERENTES MEIOS ÓPTICOS, mas fui um pouco criticado por isso (obviamente, seria o mesmo que um louco chegar e dizer que o universo tem a forma de uma maçã), tentei argumentar de forma rápida mas não fui compreendido, assim prometi escrever algo mais elaborado sobre esse assunto, que resultou no artigo que eu estou eviando a vocês, meus amigos.

Eu adicionei os contatos automaticamente, pois não estou enviando da minha casa e o tempo não me permitiu colocar os endereços somente daqueles que se interessariam por essa mensagem, se for esse o seu caso, peço por favor que desconsidere essa menságem.

Um bom final de semana, até mais.





Ricardo Soares Vieira.

rickrsv@yahoo.com.br



---------------------------------
Yahoo! Messenger 6.0 - jogos, emoticons sonoros e muita diversão. Instale agora!

[As partes desta mensagem que não continham texto foram removidas]



SUBJECT: Re: [ciencialist] Fwd: Artigo sobre velocidade da luz nos diferentes meios ópticos
FROM: "Luiz Ferraz Netto" <leobarretos@uol.com.br>
TO: <ciencialist@yahoogrupos.com.br>, "pergunte a um fisico" <fisico@if.usp.br>, "Luiz Barco" <nuap@usp.br>, "CBPF" <iva@cbpf.br>, "Itamar Itxe" <itamarap@terra.com.br>, <leandro@e-silt.com.br>, "leandro ramos fagundes" <leandro@blink.com.br>, "rick" <rickrhrc@yahoo.com.br>, João Paulo Zulian <jpzulian@hotmail.com>
DATE: 27/02/2005 13:59

>>>A LUZ SE PROPAGA COM A MESMA VELOCIDADE NOS DIFERENTES MEIOS ÓPTICOS, mas fui um pouco criticado por isso ...<

Isso é uma verdade! Onde está o problema de se afirmar isso? Justificou as velocidades de grupo e de fase?

[]'
Léo


--
No virus found in this outgoing message.
Checked by AVG Anti-Virus.
Version: 7.0.300 / Virus Database: 266.5.0 - Release Date: 25/02/2005



SUBJECT: Fw: Duvidas
FROM: "Luiz Ferraz Netto" <leobarretos@uol.com.br>
TO: "ciencialist" <ciencialist@yahoogrupos.com.br>
DATE: 27/02/2005 14:32

Fósforo e suas traquinagens ....... vamos ajudar ao jovem?
[]'
===========================
Luiz Ferraz Netto [Léo]
leobarretos@uol.com.br
http://www.feiradeciencias.com.br
===========================
-----Mensagem Original-----
De: Macufo
Para: leobarretos@uol.com.br
Enviada em: sexta-feira, 26 de novembro de 2004 07:46
Assunto: Duvidas


Ola´

estou tentando fazer uma experiencia para uma feira de ciencias em minha cidade, mas não consegui resultados ainda.
tenho q tentar retirar a maior quantidade de fosforo(elemento quimico) da caixa de fosforo(Da parte onde raspamos os palitos), mas não consigo, na verdade quero mostrar as propriedades do fosforo que se ilumina em uma cor esverdeada no escuro, consigo retirar um pouco pegando o papel que fica do lado externo da caixa(Onde raspamos o palito), coloco sobre uma moeda com a face en cotato com a moeda, apos isso queimo o papel, forma-se em cima da moeda um solução oleosa e escura, que contem uma grande quantidade de fosforo, passo isso sobre os dedos e no escuro quando atrito os dedos isso se ilumina na cor verde, em meu caso, gostaria de usar um outro metodo para retirar essas propriedades do fosforo ou conseguir uma grande quantidade de fosforo de outra maneira.

Peço encarecidamente q me ajude nisso, para mim isso é muito importante, consegui realizar meu experimento para os outros seria uma realização pessoal.

att Mac

Araras sp




Nas dúvidas experimentais, por gentileza coloque aqui o endereço da página, isso facilita o confronto. Agradeço. Meu nome é LUIZ FERRAZ NETTO, meu apelido é LÉO e moro em BARRETOS; dai vem meu e-mail: leobarretos@uol.com.br.


--------------------------------------------------------------------------------


No virus found in this incoming message.
Checked by AVG Anti-Virus.
Version: 7.0.300 / Virus Database: 266.5.0 - Release Date: 25/02/2005

----------

No virus found in this outgoing message.
Checked by AVG Anti-Virus.
Version: 7.0.300 / Virus Database: 266.5.0 - Release Date: 25/02/2005


[As partes desta mensagem que não continham texto foram removidas]



SUBJECT: Fw: Conceitos b�sicos
FROM: "Luiz Ferraz Netto" <leobarretos@uol.com.br>
TO: "ciencialist" <ciencialist@yahoogrupos.com.br>
DATE: 27/02/2005 14:34

Essas coisas são meus testes de 'paciência'; sempre consegui contorná-los e responder com toda a atenção, didática e palavras certas. Sempre consegui .... sempre consegui .... sempre .... sem ....

[]'
===========================
Luiz Ferraz Netto [Léo]
leobarretos@uol.com.br
http://www.feiradeciencias.com.br
===========================
-----Mensagem Original-----
De: Luana
Para: leobarretos@uol.com.br
Cc: glcarioca@yahoo.com.br
Enviada em: segunda-feira, 21 de fevereiro de 2005 13:40
Assunto: Conceitos básicos


Gostaria de saber o conceito das seguintes palavras:

-Velocidade Média
-Velocidade Relativa
-Velocidade Instantânea
-Aceleração Média


Espero sua resposta! Obrigado!



Nas dúvidas experimentais, por gentileza coloque aqui o endereço da página, isso facilita o confronto. Agradeço. Meu nome é LUIZ FERRAZ NETTO, meu apelido é LÉO e moro em BARRETOS; dai vem meu e-mail: leobarretos@uol.com.br.


--------------------------------------------------------------------------------


No virus found in this incoming message.
Checked by AVG Anti-Virus.
Version: 7.0.300 / Virus Database: 266.5.0 - Release Date: 25/02/2005

----------

No virus found in this outgoing message.
Checked by AVG Anti-Virus.
Version: 7.0.300 / Virus Database: 266.5.0 - Release Date: 25/02/2005


[As partes desta mensagem que não continham texto foram removidas]



SUBJECT: Fw: feira de ciencias para ed. infantil
FROM: "Luiz Ferraz Netto" <leobarretos@uol.com.br>
TO: "ciencialist" <ciencialist@yahoogrupos.com.br>
DATE: 27/02/2005 17:30

Oi Roberto,

aguardo uma sugestão sua para a Profa. Vivian.
Algo de Botânica me parece o mais recomendável no caso ... mera sugestão.

[]'
===========================
Luiz Ferraz Netto [Léo]
leobarretos@uol.com.br
http://www.feiradeciencias.com.br
===========================
-----Mensagem Original-----
De: viviancolodo
Para: leobarretos
Enviada em: domingo, 27 de fevereiro de 2005 16:17
Assunto: feira de ciencias para ed. infantil


Olá

Gostaria de saber se vocês podem me ajudar........
Sou professora de ed. infantil e preciso fazer algo com minhas crianças que tem somente 5 anos para a feira de ciências que acontecerá no final de março........O assunto é globalização

Não tenho nenhuma idéia vocês poderiam me enviar idéias

Preciso fazer algo bem legal pois sou nova nesta escola e preciso mostrar serviço, pois estou em experiência
Aguardo

Vivian


--------------------------------------------------------------------------------


No virus found in this incoming message.
Checked by AVG Anti-Virus.
Version: 7.0.300 / Virus Database: 266.5.0 - Release Date: 25/02/2005

----------

No virus found in this outgoing message.
Checked by AVG Anti-Virus.
Version: 7.0.300 / Virus Database: 266.5.0 - Release Date: 25/02/2005


[As partes desta mensagem que não continham texto foram removidas]



SUBJECT: Maestro mostra dados reais e atuais de Marte coletados pela Spirit e Opportunity
FROM: "Marcos Borges" <maborges@ccard.com.br>
TO: <ciencialist@yahoogrupos.com.br>
DATE: 27/02/2005 18:49

Maestro is a scaled-down version of the program that NASA scientists use to operate Spirit and Opportunity. Updates are also available for Maestro that contain real data from Mars that you can add to your copy of Maestro.

http://mars.telascience.org/home


Marcos Borges
Biólogo
Lab de Fisiologia Endócrina
HUPE-UERJ


[As partes desta mensagem que não continham texto foram removidas]



SUBJECT: Re: [ciencialist] FW: Relatividade
FROM: "Alberto Mesquita Filho" <albmesq@uol.com.br>
TO: <ciencialist@yahoogrupos.com.br>
DATE: 27/02/2005 18:57

----- Original Message -----
From: "Hugo Santos"
Sent: Sunday, February 27, 2005 10:00 AM
Subject: [ciencialist] FW: Relatividade

> a minha pergunta prende-se para o seguinte caso: se se colocar um segundo
> relógio, semelhante ao primeiro, com igual distância entre os espelhos,
> mas que esteja perpendicular ao primeiro relógio, com o fotão viajando
> paralelamente à direcção de deslocamento do comboio, como é que esse
> relógio bate o tempo, relativamente ao primeiro, quando vistos em
> simultâneo por um observador externo?

Olá Hugo

O assunto que você apresenta é por demais interessante, mas acho muito
difícil que você consiga uma resposta satisfatória. A falha não é da
Ciencialist mas, ao que tudo indica, da própria física que anda por aí.

Respostas pela metade você poderá obter em alguns desses centros que
insistem em fazer a lavagem cerebral nos jovens estudantes de física, mas
acho muito difícil que alguém venha palpitar aqui na Ciencialist, pois os
físicos "modernos" que por aqui militam sabem que não deixo barato respostas
pela metade. Que eles venham com tudo ou então que continuem enfiando o rabo
entre as pernas, pois resposta de fato realmente não existe a menos que
concordemos em negar os dogmas que apóiam a física moderna, mas isso seria o
mesmo que decretar o fim dessa balela.

Em junho de 2000 apresentei um paradoxo também relacionado ao que chamei
"relatividade em duas dimensões" (vide
http://ecientificocultural.com/ECC2/Dialogos/rdd.htm ). Com base nas
discussões que surgiram, escrevi um artigo intitulado "O fenômeno luz e as
falácias relativas à relatividade" e que pode ser lido a partir de
http://ecientificocultural.com/Relat/luz01.htm

Isso não responde às suas perguntas mas serve para colocar mais lenha na
fogueira. ;-)

[ ]´s
Alberto
http://ecientificocultural.com/indice.htm
Mas indiferentemente a tudo isso, o neutrino tem massa, o elétron não é
uma carga elétrica coulombiana e a Terra se move. E a história se repetirá.



SUBJECT: A nova cruzada contra a ciência
FROM: "Oraculo" <oraculo@atibaia.com.br>
TO: <ciencialist@yahoogrupos.com.br>
DATE: 27/02/2005 21:43

A nova cruzada contra a ciência

MARCELO GLEISER

Em seu discurso inaugural de janeiro passado, o presidente americano George W. Bush
afirmou: "Somos feitos à imagem do Criador da Terra e do Céu". Ou seja, Deus -na
versão judaico-cristã da história, veja bem- criou os homens e o Universo. Somos
todos deuses, portanto, brincando nos jardins sagrados do cosmo.

Vivemos em tempos conturbados. Sei que sempre é possível fazer essa afirmação;
problemas sócio-econômicos existiram desde o início da história e não vão desaparecer
tão cedo. O que preocupa no momento atual é a infiltração de certas idéias religiosas
na política, contrariando os preceitos de uma sociedade democrática. Se o Estado
começa a agir em nome de uma determinada religião, passamos a ter uma teocracia.

No novo Orçamento da União, Bush cortou o a receita de todos os órgãos dedicados à
pesquisa, com exceção do Departamento de Defesa e do de Segurança Interna. Apenas a
Nasa se saiu relativamente bem; mas isso porque Bush quer que seus fundos sejam
direcionados a levar americanos à Lua e, quem sabe, a Marte. Os fundos da agência
espacial destinados à pesquisa básica foram sumariamente cortados, incluindo os que
poderiam salvar o Telescópio Espacial Hubble. Ele está com os dias contados.

Por que isso ocorre agora? Uma das razões, certamente não a única, é que com o fim da
Guerra Fria os EUA perderam sua maior razão para investir pesado em pesquisa básica.
Os soviéticos eram competidores sagazes e os EUA não podiam ficar para trás. Osama
Bin Laden e a Al-Qaeda não estão interessados em ciência; só querem saber de religião
e terrorismo.

Não é coincidência que os ataques de 2001 em Nova York e Washington tenham ocorrido
após a direita cristã ter subido ao poder nos Estados Unidos. A noção de que vivemos
um retorno das Cruzadas -o uso da religião para justificar combates políticos- não é
tão absurda assim.

Uma das vítimas disso é a ciência. Na medida em que a religião controla o poder
político, pesquisas passam a ser proibidas em certas áreas, fundos são cortados e
dirigidos apenas a interesses nacionais. Isso é exatamente o que está ocorrendo agora
nos EUA: proibição da pesquisa com células-tronco, corte de fomento à pesquisa
básica, quantias enormes direcionadas para sistemas de defesa por mísseis e radares
superpoderosos, projetos espaciais propagandistas, tais como o retorno à Lua etc.

Podemos aprender com o que está ocorrendo nos EUA e tentar evitar que o mesmo
aconteça no Brasil. Pesquisas nos EUA e no Brasil mostram que, em ambos os países, a
maioria da população concorda com a afirmação de Bush em seu discurso inaugural. Isso
é prova de que a educação científica, tanto lá quanto cá, deixa a desejar.

Note que não tenho uma postura antirreligiosa. Muito pelo contrário, acho inocentes
os cientistas que consideram que fé e ignorância andam de mãos dadas. A religião é
muito mais antiga do que a ciência e não irá (e nem deve) desaparecer. O problema
ocorre quando a religião passa a atuar fora de sua esfera, tentando evangelizar a
população por meio de seu poder político. Como no Rio de Janeiro, forçando o ensino
do criacionismo como uma alternativa a teorias científicas.

Quem perde com isso são nossos estudantes e, com eles, o país inteiro. Se ciência de
ponta não puder ser feita aqui, será feita em outro lugar.

--------------------------------------------------------------------------------
Marcelo Gleiser é professor de física teórica do Dartmouth College, em Hanover (EUA),
e autor do livro "O Fim da Terra e do Céu"


[As partes desta mensagem que não continham texto foram removidas]



SUBJECT: Re: O que vos parece isto?
FROM: Maria Natália <grasdic@hotmail.com>
TO: ciencialist@yahoogrupos.com.br
DATE: 28/02/2005 00:15


Galera:

Esta me faz lembrar: Ladrão que rouba ladrão tem 100 anos de perdão.
E agora vinha a política etc e todos roubam e daqui a 10 min era a
guerra total e o kaos.
Com esta minha mensagem apenas quis confirmar o que me parecera
desonesto e chamar a atenção dos mais incautos. Quer queiramos quer
não a internet é usada por muita gente ingénua (no sentido de apenas
saber "andar" em linha recta).
Quando todos sabemos que ter carta de condução não é sinal de saber
circular nas estradas (da informação). Tem de saber andar em cima de
óleo, areia e lençóis de água...armadilhas várias.
Aliás como em tudo.
Agradeço pois a ajuda
Maria Natália

--- Em ciencialist@yahoogrupos.com.br, "L.E.R.de Carvalho"
<lecarvalho@i...> escreveu
> At 09:39 27/2/2005, you wrote:
> >E eu estou de acordo consigo. Só faltou mesmo o "e vice-versa" no
fim da
> >frase!
> >
> >Os casos em que não se verifica tal regra são a excepção que a
confirma! ;)
> >
> >De qualquer forma, há muitos casos em que o esquema usado não é este.
> >Enganar quem por seu lado tenta enganar nem é tão grave assim. Mas
enganar
> >de forma gratuita a quem nunca nenhum mal fez, acontece muitas v





SUBJECT: Re: Fwd: Artigo sobre velocidade da luz nos diferentes meios ópticos
FROM: Maria Natália <grasdic@hotmail.com>
TO: ciencialist@yahoogrupos.com.br
DATE: 28/02/2005 01:33


Ricardo:
Mas para se ler tem de se associar e não dá Já estou em 20 listas...
Não podes escrever aqui, copiar pedindo autorização?
Aguardo
Maria Natália


--- Em ciencialist@yahoogrupos.com.br, ricardo soares vieira
<rickrsv@y...> escreveu
> Olá a todos.
>
> A algum tempo, em um grupo de discussão sobre física
(http://br.groups.yahoo.com/group/spinquarks/), eu tinha lançado a
hipótese de que A LUZ SE PROPAGA COM A MESMA VELOCIDADE NOS DIFERENTES
MEIOS ÓPTICOS, mas fui um pouco criticado por isso (obviamente, seria
o mesmo que um louco chegar e dizer que o universo tem a forma de uma
maçã), tentei argumentar de forma rápida mas não fui compreendido,
assim prometi escrever algo mais elaborado sobre esse assunto, que resuas]





SUBJECT: Re: Fw: Duvidas
FROM: Maria Natália <grasdic@hotmail.com>
TO: ciencialist@yahoogrupos.com.br
DATE: 28/02/2005 01:46


òi professor!
Mas como é? Ainda pensei que menina fosse portuguesa e estivesse a
falar de fósoforos (de pau, palito) e que está dentro de caixa para
acender o fogão. Mas NÃO!!
Não pode. Pegar com a mão e sorte a da mina se não é mesmo P...Mas
cadê professor de laboratório e segurança percebendo? Quem são vossos
professores de QUÍMICA, pás?
1--tem de ir ao fabricante saber a constituição da lixa. Entrar em
contacto pela net por exemplo;
2--ir a um Vogel ou King saber como se extrai;
3--consultar MSDS para seguir as normas de segurança: chaminé? luvas?
eliminação de resíduos?
4--"puxar" pela professora e pô-la ao lado para a ajudar nas técnicas
e regras de laboratório. Eu deveria escrever. encostar professor à parede.
Não deixem meninos aprendizes de química sózinhos. Por favor Físicos:
isto não é o plano inclinado. Esse nunca intoxica ninguém
(teoricamnete falando LOLLLL).
E já pensaram se esse P é o branco? Faz a necrose óssea. E em feira
que tem de se repetir...Ai Karamba galera!!
Usai aqui também "a camizinha" LOLLLLLLL
Boa noite
Maria Natália



--- Em ciencialist@yahoogrupos.com.br, "Luiz Ferraz Netto"
<leobarretos@u...> escreveu
> Fósforo e suas traquinagens ....... vamos ajudar ao jovem?
> []'
> ===========================
> Luiz Ferraz Netto [Léo]
> leobarretos@u...
> http://www.feiradeciencias.com.br
> ===========================
> -----Mensagem Original-----
> De: Macufo
> Para: leobarretos@u...
> Enviada em: sexta-feira, 26 de novembro de 2004 07:46
> Assunto: Duvidas
>
>
> Ola´
>
> estou tentando fazer uma experiencia para uma feira de ciencias em
minha cidade, mas não consegui resultados ainda.
> tenho q tentar retirar a maior quantidade de fosforo(elemento
quimico) da caixa de fosforo(Da parte onde raspamos os palitos), mas
não consigo, na verdade quero mostrar as propriedades do fosforo que
se ilumina em uma cor esverdeada no escuro, consigo retirar um pouco
pegando o papel que fica do lado externo da caixa(Onde raspamos o
palito), coloco sobre uma moeda com a face en cotato com a moeda, apos
isso queimo o papel, forma-se em cima da moeda um solução oleosa e
escura, que contem uma grande quantidade de fosforo, passo isso sobre
os dedos e no escuro quando atrito os dedos isso se ilumina na cor
verde, em meu caso, gostaria de usar um outro metodo para retirar
essas propriedades do fosforo ou conseguir uma grande quantidade de
fosforo de outra maneira.
>
> Peço encarecidamente q me ajude nisso, para mim isso é muito
importante, consegui realizar meu experimento para os outros seria uma
realização pessoal.
>
> att Mac
>
> Araras sp
>
>
>
>
> Nas dúvidas experimentais, por gentileza coloque aqui o endereço da
página, isso facilita o confronto. Agradeço. Meu nome é LUIZ FERRAZ
NETTO, meu apelido é LÉO e moro em BARRETOS; dai vem meu e-mail:
leobarretos@u...
>
>
>
--------------------------------------------------------------------------------
>
>
> No virus found in this incoming message.
> Checked by AVG Anti-Virus.
> Version: 7.0.300 / Virus Database: 266.5.0 - Release Date: 25/02/2005
>
> ----------
>
> No virus found in this outgoing message.
> Checked by AVG Anti-Virus.
> Version: 7.0.300 / Virus Database: 266.5.0 - Release Date: 25/02/2005
>
>
> [As partes desta mensagem que não continham texto foram removidas]





SUBJECT: Re: Fw: Conceitos básicos
FROM: Maria Natália <grasdic@hotmail.com>
TO: ciencialist@yahoogrupos.com.br
DATE: 28/02/2005 01:50


Paciência de Job, direi.
Pois está tudo na feiradeciencias. com.br
Mas posso perguntar? Que livros adoptam para os meninos nada
perceberem do que lá vem escrito?
A net já é livro? não sabia que era assim. Que é isso de literacia????
Fui
Maria Natália


-Leo um santo...
paciência...paciência...paciência....paci ...paz...pá!
>
> []'
> ===========================
> Luiz Ferraz Netto [Léo]
> leobarretos@u...
> http://www.feiradeciencias.com.br
> ===========================
> -----Mensagem Original-----
> De: Luana
> Para: leobarretos@u...
> Cc: glcarioca@y...
> Enviada em: segunda-feira, 21 de fevereiro de 2005 13:40
> Assunto: Conceitos básicos
>
>
> Gostaria de saber o conceito das seguintes palavras:
>
> -Velocidade Média
> -Velocidade Relativa
> -Velocidade Instantânea
> -Aceleração Média
>
>
> Espero sua resposta! Obrigado!
>
>
>
> Nas dúvidas experimentais, por gentileza coloque aqui o endereço da
página, isso facilita o confronto. Agradeço. Meu nome é LUIZ FERRAZ
NETTO, meu apelido é LÉO e moro em BARRETOS; dai vem meu e-mail:
leobarretos@u...
>
>
>
--------------------------------------------------------------------------------
>
>
> No virus found in this incoming message.
> Checked by AVG Anti-Virus.
> Version: 7.0.300 / Virus Database: 266.5.0 - Release Date: 25/02/2005
>
> ----------
>
> No virus found in this outgoing message.
> Checked by AVG Anti-Virus.
> Version: 7.0.300 / Virus Database: 266.5.0 - Release Date: 25/02/2005
>
>
> [As partes desta mensagem que não continham texto foram removidas]





SUBJECT: Re: A nova cruzada contra a ciência
FROM: Maria Natália <grasdic@hotmail.com>
TO: ciencialist@yahoogrupos.com.br
DATE: 28/02/2005 02:03


Gostei que tivesse colocado nesta lista este tema.
E sinto o mesmo que este professor pois estou acompanhando os
acontecimentos por causa da astronomia e ainda por ter ex-alunos nos
EUA e que me chamam a atençâo para certos acontecimentos.
Acho que todos nós "pelas barbas do vizinho a arder...devemos
acautelar as nossas"
Que fazer? O que vai mal no nosso ensino para a molecada fugir de
física, de química de ciência enfim? Porque existem cidades onde todo
o pessoal vai para história, administração e computação? Porque será
que aluno que quer saber astronomia tem de ir para fora de sua terra e
se dinheiro não tem como será? E mais haverá.
Por favor nos vosso ChemQuês,encontros de física de ensino de tratai
deste tema da "fuga à Ciência" Pensai aí nem que seja no penúltimo
dia. Que se passa?
Nós em Portugal pensamos e sabemos que não basta uma feira como a de
Barretos...
Um abraço e obrigada por nos alertares aqui
Maria Natália

--- Em ciencialist@yahoogrupos.com.br, "Oraculo" <oraculo@a...> escreveu
> A nova cruzada contra a ciência
>
> MARCELO GLEISER
>
> Em seu discurso inaugural de janeiro passado, o presidente americano
George W. Bush
> afirmou: "Somos feitos à imagem do Criador da Terra e do Céu". Ou
seja, Deus -na
> versão judaico-cristã da história, veja bem- criou os homens e o
Universo. Somos
> todos deuses, portanto, brincando nos jardins sagrados do cosmo.
>
> Vivemos em tempos conturbados. Sei que sempre é possível fazer essa
afirmação;
> problemas sócio-econômicos existiram desde o início da história e
não vão desaparecer
> tã





SUBJECT: Re: Maestro mostra dados reais e atuais de Marte coletados pela Spirit e Opportu
FROM: Maria Natália <grasdic@hotmail.com>
TO: ciencialist@yahoogrupos.com.br
DATE: 28/02/2005 02:08


Obrigada pois isto vai permitir fazer-se uma ou outra questão sobre
temas de Ciência para que alunos pensem aplicando conceitos já aprendidos.
sds
Maria Natália

--- Em ciencialist@yahoogrupos.com.br, "Marcos Borges" <maborges@c...>
escreveu
> Maestro is a scaled-down version of the program that NASA scientists
use to operate Spirit and Opportunity. Updates are also available for
Maestro that contain real data from Mars that you can add to your copy
of Maestro.
>
> http://mars.telascience.org/home
>
>
> Marcos Borges
> Biólogo
> Lab de Fisiologia Endócrina
> HUPE-UERJ
>
>
> [As partes desta mensagem que não continham texto foram removidas]





SUBJECT: Re: Fw: feira de ciencias para ed. infantil
FROM: "rmtakata" <rmtakata@altavista.net>
TO: ciencialist@yahoogrupos.com.br
DATE: 28/02/2005 08:41


Bem, aqui eh preciso atentar para o q. se quer dizer com globalizacao.
Em termos estritos, normalmente, globalizacao se refere 'a
interconexao dos mercados (em particular do mercado financeiro, mas tb
do mercado de bens e produtos).

Nesse sentido estrito, talvez as criancas possam desenvolver algumas
nocoes basicas de economia: comparar o sistema de escambo com o
sistema de troca de moedas (e sofisticando um pouco: com o sistema de
troca de titulos - q. eh o q. o papel-moeda eh, sem um valor
intrinseco, apenas com o valor da confianca da populacao em relacao ao
q. esta' escrito no papel - mas ela teria q. ser uma professora
espetacular para conseguir trabalhar essa nocao - nao eh impossivel,
mas eu nao faco a minima ideia de como conseguir isso: talvez com
dinheiro de brinquedo).

Assim: criar um pequeno mercado ficcional. Um objeto grande (um pouco
desajeitado para se transportar, mas seguro para criancas - poderia
ser um imenso bloco de EPS) representaria um produto para troca por
escambo (digamos, uma vaca); moedas de plastico representariam as
moedas de metal. Fazer as criancas verificarem o q. e' mais pratico
transportar: o imenso objeto grande ou algumas moedas. (Nao sei se
seria possivel no tempo exiguo q. se tem, passar 'as criancas a nocao
de q. eh mais facil transportar moedas porq. uma pequena moedinha tem
um valor intrinseco ou um valor convencional maior do q. uma
quantidade em peso igual de mercadoria: em geral, 1 g de moeda vale
muito mais do q. 1 g de mercadoria.)

Uma sofisticacao maior nesse modelo, seria as criancas terem a nocao
da distancia entre os centros de producao e o centro de consumo - elas
iriam se esforcar muito mais para transportar a moeda-mercadoria (o
bloco de EPS) se tivessem q. andar mais (leva-se o bloco para um canto
mais afastado da sala). E ter a nocao de q. um esforco maior pode
significar um preco maior - para valer o esforco extra: a recompensa
tem q. ser proporcional ao trabalho (se bem q. isso pode causar uma
revolucao na casa das criancas - elas poderao fazer greve para ter
aumento de mesada...)

Se a professora for muito, muito competente, podera' criar pequenos
grupos inicialmente separados, cada qual representando um mercado. E
depois fazendo esses grupos interagirem - conexao dos mercados - e
verificar algumas consequencias. (A competencia aqui sera' de
conseguir direcionar a energia das criancas para a atividade e evitar
sua dispersao.)

(Em vez de dinheiro, poderiam usar balas ou confeitos - assim elas
poderiam dar mais valor... Mas eh preciso evitar q. elas consumam tudo
antes de terminar a demonstracao.)

[]s,

Roberto Takata

--- Em ciencialist@yahoogrupos.com.br, "Luiz Ferraz Netto"
> ===========================
> Luiz Ferraz Netto [Léo]
> -----Mensagem Original-----
> De: viviancolodo
> crianças que tem somente 5 anos para a feira de ciências que
> acontecerá no final de março........O assunto é globalização






SUBJECT: Re: Fw: Conceitos básicos
FROM: "rmtakata" <rmtakata@altavista.net>
TO: ciencialist@yahoogrupos.com.br
DATE: 28/02/2005 08:46


http://www.feiradeciencias.com.br/sala04/04_RE_01.asp

[]s,

Roberto Takata

--- Em ciencialist@yahoogrupos.com.br, "Luiz Ferraz Netto"
> ===========================
> Luiz Ferraz Netto [Léo]
> -----Mensagem Original-----
> De: Luana
> Gostaria de saber o conceito das seguintes palavras:
>
> -Velocidade Média
> -Velocidade Relativa
> -Velocidade Instantânea
> -Aceleração Média





SUBJECT: feira de ciencias para ed. infantil
FROM: "L.E.R.de Carvalho" <lecarvalho@infolink.com.br>
TO: ciencialist@yahoogrupos.com.br
DATE: 28/02/2005 09:23

Sim, eu ia dizer isso, mas não quis escrever muito.

No Museu da Ciência de Londres, a Expo sobre Alimentos, tinha um setor
apenas sobre Economia.

Explicava como funcionava a bolsa de Chicago, a formação de preços, os
canais de comercialização.

Claro que a molecada faz daquilo tudo um play ground e zoneia e não aprende
quase nada.

Então a Expo fornece às professores, que vieram de longe, de bus, trazendo
milhares de estudantes, um rico e amplo material didático, cobrindo
literatura, matemática, física, química e biologia, para que trabalhos
sejam feitos pós-visita à Expo, já na escola, já noutras cidades.

Trabalho não falta.

L.E.



At 08:41 28/2/2005, you wrote:

>Bem, aqui eh preciso atentar para o q. se quer dizer com globalizacao.
>Em termos estritos, normalmente, globalizacao se refere 'a
>interconexao dos mercados (em particular do mercado financeiro, mas tb
>do mercado de bens e produtos).
>
>Nesse sentido estrito, talvez as criancas possam desenvolver algumas
>nocoes basicas de economia: comparar o sistema de escambo com o
>sistema de troca de moedas (e sofisticando um pouco: com o sistema de
>troca de titulos - q. eh o q. o papel-moeda eh, sem um valor
>intrinseco, apenas com o valor da confianca da populacao em relacao ao
>q. esta' escrito no papel - mas ela teria q. ser uma professora
>espetacular para conseguir trabalhar essa nocao - nao eh impossivel,
>mas eu nao faco a minima ideia de como conseguir isso: talvez com
>dinheiro de brinquedo).
>
>Assim: criar um pequeno mercado ficcional. Um objeto grande (um pouco
>desajeitado para se transportar, mas seguro para criancas - poderia
>ser um imenso bloco de EPS) representaria um produto para troca por
>escambo (digamos, uma vaca); moedas de plastico representariam as
>moedas de metal. Fazer as criancas verificarem o q. e' mais pratico
>transportar: o imenso objeto grande ou algumas moedas. (Nao sei se
>seria possivel no tempo exiguo q. se tem, passar 'as criancas a nocao
>de q. eh mais facil transportar moedas porq. uma pequena moedinha tem
>um valor intrinseco ou um valor convencional maior do q. uma
>quantidade em peso igual de mercadoria: em geral, 1 g de moeda vale
>muito mais do q. 1 g de mercadoria.)
>
>Uma sofisticacao maior nesse modelo, seria as criancas terem a nocao
>da distancia entre os centros de producao e o centro de consumo - elas
>iriam se esforcar muito mais para transportar a moeda-mercadoria (o
>bloco de EPS) se tivessem q. andar mais (leva-se o bloco para um canto
>mais afastado da sala). E ter a nocao de q. um esforco maior pode
>significar um preco maior - para valer o esforco extra: a recompensa
>tem q. ser proporcional ao trabalho (se bem q. isso pode causar uma
>revolucao na casa das criancas - elas poderao fazer greve para ter
>aumento de mesada...)
>
>Se a professora for muito, muito competente, podera' criar pequenos
>grupos inicialmente separados, cada qual representando um mercado. E
>depois fazendo esses grupos interagirem - conexao dos mercados - e
>verificar algumas consequencias. (A competencia aqui sera' de
>conseguir direcionar a energia das criancas para a atividade e evitar
>sua dispersao.)
>
>(Em vez de dinheiro, poderiam usar balas ou confeitos - assim elas
>poderiam dar mais valor... Mas eh preciso evitar q. elas consumam tudo
>antes de terminar a demonstracao.)
>
>[]s,
>
>Roberto Takata


[As partes desta mensagem que não continham texto foram removidas]



SUBJECT: Fw: Porque o oxigênio é invisivel ?
FROM: "Luiz Ferraz Netto" <leobarretos@uol.com.br>
TO: "ciencialist" <ciencialist@yahoogrupos.com.br>
DATE: 28/02/2005 10:59

Que tal essa pergunta?
Afinal, bem mais simples do que a cor do elétron!
[]'
===========================
Luiz Ferraz Netto [Léo]
leobarretos@uol.com.br
http://www.feiradeciencias.com.br
===========================
-----Mensagem Original-----
De: "Fornov Ik" <fornov_ik@yahoo.com.br>
Para: <leobarretos@uol.com.br>
Enviada em: segunda-feira, 28 de fevereiro de 2005 07:59
Assunto: Porque o oxigênio é invisivel ?



Olá professor existe alguma explicação cientifica
para a invisibilidade do oxigênio?


Alexsander Antunes. Magé - Rj





_______________________________________________________
Yahoo! Acesso Grátis - Instale o discador do Yahoo! agora. http://br.acesso.yahoo.com/ - Internet rápida e grátis


--
No virus found in this incoming message.
Checked by AVG Anti-Virus.
Version: 7.0.300 / Virus Database: 266.5.0 - Release Date: 25/02/2005




--
No virus found in this outgoing message.
Checked by AVG Anti-Virus.
Version: 7.0.300 / Virus Database: 266.5.0 - Release Date: 25/02/2005



SUBJECT: Re: [ciencialist] Re: Fw: Conceitos básicos
FROM: "Luiz Ferraz Netto" <leobarretos@uol.com.br>
TO: <ciencialist@yahoogrupos.com.br>
DATE: 28/02/2005 11:17

Oh.... Roberto,

sonhos de uma noite de verão... querer que essa meninada entenda 'aquela cinemática'!

Repare a pergunta da Luana:

"Gostaria de saber o conceito das seguintes palavras:"

[]'

===========================
Luiz Ferraz Netto [Léo]
leobarretos@uol.com.br
http://www.feiradeciencias.com.br
===========================
-----Mensagem Original-----
De: "rmtakata" <rmtakata@altavista.net>
Para: <ciencialist@yahoogrupos.com.br>
Enviada em: segunda-feira, 28 de fevereiro de 2005 08:46
Assunto: [ciencialist] Re: Fw: Conceitos básicos




http://www.feiradeciencias.com.br/sala04/04_RE_01.asp

[]s,

Roberto Takata

--- Em ciencialist@yahoogrupos.com.br, "Luiz Ferraz Netto"
> ===========================
> Luiz Ferraz Netto [Léo]
> -----Mensagem Original-----
> De: Luana
> Gostaria de saber o conceito das seguintes palavras:
>
> -Velocidade Média
> -Velocidade Relativa
> -Velocidade Instantânea
> -Aceleração Média





##### ##### #####

Para saber mais visite
http://www.ciencialist.hpg.ig.com.br


##### ##### ##### #####
Links do Yahoo! Grupos










--
No virus found in this incoming message.
Checked by AVG Anti-Virus.
Version: 7.0.300 / Virus Database: 266.5.0 - Release Date: 25/02/2005




--
No virus found in this outgoing message.
Checked by AVG Anti-Virus.
Version: 7.0.300 / Virus Database: 266.5.0 - Release Date: 25/02/2005



SUBJECT: Re: Fw: Porque o oxigênio é invisivel ?
FROM: "rmtakata" <rmtakata@altavista.net>
TO: ciencialist@yahoogrupos.com.br
DATE: 28/02/2005 11:24


Atomos de oxigenio sao muito pequenos para poderem ser vistos.

Uma amostra de gas oxigenio nao eh visivel a olho desarmado por ser
transparente 'a luz dentro do espectro visivel. Mas ele absorve um
pouco de luz ultravioleta - se fossemos capazes de enxergar na faixa
do ultravioleta, continuariamos a nao ver atomos individuais ou
moleculas diatomicas de oxigenio, mas poderiamos perceber a presenca
do gas. Sondas espaciais procuram pela presenca de oxigenio na
atmosfera de outros planetas por meio dessa propriedade.

[]s,

Roberto Takata

--- Em ciencialist@yahoogrupos.com.br, "Luiz Ferraz Netto"
> Que tal essa pergunta?
> Afinal, bem mais simples do que a cor do elétron!
> Luiz Ferraz Netto [Léo]
> -----Mensagem Original-----
> De: "Fornov Ik" <fornov_ik@y...>
> Olá professor existe alguma explicação cientifica
> para a invisibilidade do oxigênio?





SUBJECT: gimmick
FROM: "Luiz Ferraz Netto" <leobarretos@uol.com.br>
TO: <ciencialist@yahoogrupos.com.br>
DATE: 28/02/2005 12:23

Ois,

o que é "gimmick"?

O que é um "Jornal com gimmick " ?

[]'
===========================
Luiz Ferraz Netto [Léo]
leobarretos@uol.com.br
http://www.feiradeciencias.com.br
===========================


--
No virus found in this outgoing message.
Checked by AVG Anti-Virus.
Version: 7.0.300 / Virus Database: 266.5.0 - Release Date: 25/02/2005



SUBJECT: Re: FW: Relatividade
FROM: Hélio Ricardo Carvalho <hrc@fis.puc-rio.br>
TO: ciencialist@yahoogrupos.com.br
DATE: 28/02/2005 12:27


Valeu Alberto,

Só para completar de forma bem humorada:

Isto é apenas experiência de pensamento.

Numa experiência de pensamento podemos fazer o seguinte:
1- Faz de conta que a natureza funciona deste o daquele jeito.
2- Se for assim vai acontecer isto ou aquilo. :-)

Não tenho notícia se um relógio fotônico deste tipo já foi realmente
construído.

Quem quiser defender a "Teoria de Relatividade de Einstein" poderá
dizer: Um relógio destes colocado longitudinal com o movimento
produz mudanças da velocidade no momento da reflexão (aceleração).
Estas acelerações estão na mesma direção do "movimento
relativístico" logo para explicar isto só com a "Relatividade Geral"
que é muito complicado falar em poucas linhas. Mas basta por ora
você saber que a TRG explica tudo. Se não, a FQ explicará.

:-) :-) :-) :-) :-) :-) :-) :-) :-) :-)

[ ]'s
Hélio


--- Em ciencialist@yahoogrupos.com.br, "Alberto Mesquita Filho"
<albmesq@u...> escreveu
> ----- Original Message -----
> From: "Hugo Santos"
> Sent: Sunday, February 27, 2005 10:00 AM
> Subject: [ciencialist] FW: Relatividade
>
> > a minha pergunta prende-se para o seguinte caso: se se colocar
um segundo
> > relógio, semelhante ao primeiro, com igual distância entre os
espelhos,
> > mas que esteja perpendicular ao primeiro relógio, com o fotão
viajando
> > paralelamente à direcção de deslocamento do comboio, como é que
esse
> > relógio bate o tempo, relativamente ao primeiro, quando vistos
em
> > simultâneo por um observador externo?
>
> Olá Hugo
>
> O assunto que você apresenta é por demais interessante, mas acho
muito
> difícil que você consiga uma resposta satisfatória. A falha não é
da
> Ciencialist mas, ao que tudo indica, da própria física que anda
por aí.
>
> Respostas pela metade você poderá obter em alguns desses centros
que
> insistem em fazer a lavagem cerebral nos jovens estudantes de
física, mas
> acho muito difícil que alguém venha palpitar aqui na Ciencialist,
pois os
> físicos "modernos" que por aqui militam sabem que não deixo barato
respostas
> pela metade. Que eles venham com tudo ou então que continuem
enfiando o rabo
> entre as pernas, pois resposta de fato realmente não existe a
menos que
> concordemos em negar os dogmas que apóiam a física moderna, mas
isso seria o
> mesmo que decretar o fim dessa balela.
>
> Em junho de 2000 apresentei um paradoxo também relacionado ao que
chamei
> "relatividade em duas dimensões" (vide
> http://ecientificocultural.com/ECC2/Dialogos/rdd.htm ). Com base
nas
> discussões que surgiram, escrevi um artigo intitulado "O fenômeno
luz e as
> falácias relativas à relatividade" e que pode ser lido a partir de
> http://ecientificocultural.com/Relat/luz01.htm
>
> Isso não responde às suas perguntas mas serve para colocar mais
lenha na
> fogueira. ;-)
> [ ]´s
> Alberto
> http://ecientificocultural.com/indice.htm
> Mas indiferentemente a tudo isso, o neutrino tem massa, o elétron
não é
> uma carga elétrica coulombiana e a Terra se move. E a história se
repetirá.





SUBJECT: Re: gimmick
FROM: "rmtakata" <rmtakata@altavista.net>
TO: ciencialist@yahoogrupos.com.br
DATE: 28/02/2005 12:57


--- Em ciencialist@yahoogrupos.com.br, "Luiz Ferraz Netto"
> o que é "gimmick"?

Termo em ingles para: aparelho para fraudar; bugiganga; estratagema
para promover um projeto; uma pegadinha; coisa de cujo nome nao nos
lembramos na hora.

> O que é um "Jornal com gimmick " ?

Qual o contexto?

[]s,

Roberto Takata





SUBJECT: Re: [ciencialist] Fw: Porque o oxigênio é invisivel ?
FROM: TARCISIO BORGES <tbs97@fisica.ufpr.br>
TO: ciencialist <ciencialist@yahoogrupos.com.br>
DATE: 28/02/2005 12:59

Mas quem disse que oxigênio é invisível?

Pegue um tubo linear com uns 200km de comprimento cheio de O2. Use um
feixe de luz branca em uma abertura e veja o que sai na outra...

[]s
TARCISIO BORGES
tbs97@fisica.ufpr.br

On Mon, 28 Feb 2005, Luiz Ferraz Netto wrote:
> Que tal essa pergunta?
> Afinal, bem mais simples do que a cor do elétron!
> ===========================
> Luiz Ferraz Netto [Léo]
> -----Mensagem Original-----
> Olá professor existe alguma explicação cientifica
> para a invisibilidade do oxigênio?
>
> Alexsander Antunes. Magé - Rj



SUBJECT: Re: Fw: Porque o oxigênio é invisivel ?
FROM: "Luis" <luis.alcides@he.com.br>
TO: ciencialist@yahoogrupos.com.br
DATE: 28/02/2005 13:48


Eu fiquei muito impressionado com esta resposta, foi muito
esclarecedora. Todavia se eu não tiver disponível um tubo linear com
200km existe outra forma de comprovar se o O2 tem cor ou não?
[]s
De Boni.



--- Em ciencialist@yahoogrupos.com.br, TARCISIO BORGES <tbs97@f...>
escreveu
> Mas quem disse que oxigênio é invisível?
>
> Pegue um tubo linear com uns 200km de comprimento cheio de O2. Use um
> feixe de luz branca em uma abertura e veja o que sai na outra...
>
> []s
> TARCISIO BORGES
> tbs97@f...
>
> On Mon, 28 Feb 2005, Luiz Ferraz Netto wrote:
> > Que tal essa pergunta?
> > Afinal, bem mais simples do que a cor do elétron!
> > ===========================
> > Luiz Ferraz Netto [Léo]
> > -----Mensagem Original-----
> > Olá professor existe alguma explicação cientifica
> > para a invisibilidade do oxigênio?
> >
> > Alexsander Antunes. Magé - Rj





SUBJECT: No carro: com volt, ampère e ohm
FROM: "L.E.R.de Carvalho" <lecarvalho@infolink.com.br>
TO: ciencialist@yahoogrupos.com.br
DATE: 28/02/2005 15:02


Coluna Lingua Viva
Deonísio da Silva
JB 28Fev05

No carro: com volt, ampère e ohm


Os dicionários mais consultados do país não têm ''relê'', mas o seu carro
tem. A língua portuguesa foi buscar a palavra no francês ''relais'', do
verbo ''relayer'', revezar, alternar, substituir. Inicialmente foi aplicado
aos cavalos que, fatigados, eram substituídos, principalmente em carroças e
arados, por animais descansados, ensejando que os outros fossem
alimentados, para depois voltarem ao trabalho novamente, no moto perpétuo
que o homem lhes impôs com a domesticação.

Nossa língua acolheu o vocábulo como ''relé'', que está grafado ''relê'' no
Dicionário do carro, do carioca Bob Sharp, jornalista especializado em
automóveis, ex-piloto de competição com vários títulos nacionais e
regionais. Os lexicógrafos lhe devem 1.200 verbetes por ele reunidos,
movido pela paixão do conhecimento, tempero que apresenta diferencial de
qualidade em qualquer pesquisa. Fale, ou melhor, escreva Bob Sharp: ''relê:
dispositivo eletromecânico que, por meio de uma corrente de baixa
intensidade (amperagem), efetua ligação elétrica de outra, de alta
intensidade. Com isso, interruptores de bordo não precisam ser reforçados e
volumosos''.

Mas o que é amperagem? Designa intensidade de corrente elétrica, medida em
amperes, plural de ampere, que o Aurélio ainda não acolheu, preferindo
indexá-lo em sua versão francesa, ''ampère''. Em Portugal, desde 1929, foi
adotada a forma ''ampério'', recusada pelo Brasil por uma daquelas razões
que nos lembram o dito famoso de Bernard Shaw: ''Inglaterra e Estados
Unidos são dois países separados pela mesma língua'', o que pode ser uma
''boutade'' divertida, mas desprovida de base. Alias, ''boutade'', sinônimo
de dito espirituoso, de brincadeira verbal, deriva do verbo ''bouter'',
empurrar. Toda a comunidade lusófona se entende muito bem em vários
continentes.

Vejamos que curiosa a viagem das palavras. ''Ampère'', ''volt'' e ''ohm'',
entre outras palavras, entraram para o francês em 1881, por ocasião de um
congresso de eletricistas realizado naquele ano em Paris. Cientistas
europeus estão na origem das três palavras: o francês André-Marie Ampère
(1775-1836); o italiano Alessandro Volta (1745-1827) e o alemão Georg Simon
Ohm (1789-1854). A eletricidade é nossa companheira inseparável, no carro e
em casa. Explica Sharp: ''Um volt produz uma corrente elétrica de 1 ampère
através de uma resistência de 1 ohm''.

Não denominamos ''volta'', mas ''volt'', porque a palavra deu entrada pelo
francês, que omitiu a vogal final. No carro, falamos de seis volts, 12
volts, mas nas residências em 110 ou 220 volts.

Alessandro Volta, mudo até os 7 anos, foi considerado deficiente mental. Na
universidade onde ensinava, seus colegas achavam que ele era ''pouco
científico''. Já tinha descoberto a pilha quando foi nomeado senador do
reino da Itália por Napoleão Bonaparte. A Academia de Ciências da França
recusou-se a condecorá-lo, alegando que o regulamento vetava premiações a
estrangeiros. Não é de hoje... Napoleão revidou: acrescentou à medalha de
ouro por reconhecimento intelectual um prêmio de seis mil francos.

O cientista, que morreu aos 82 anos, quis aposentar-se quando chegou à
velhice, mas outra vez o imperador interveio: se estava cansado de ensinar,
que desse apenas uma lição por ano, ''pois os bons generais morrem no campo
de batalha''. Desde que não sejam presos e exilados numa ilha...

No Brasil atual, leis esdrúxulas teriam chutado o velho cientista para bem
longe de qualquer campus. Mas seu nome está presente em nossa vida
cotidiana: em casa, nos meios de transporte, no trabalho. E no automóvel, o
substituto do cavalo. Até a invenção do trem, no século 19, o cavalo foi o
meio de transporte mais rápido.



[As partes desta mensagem que não continham texto foram removidas]



SUBJECT: Re: Brilho metálico
FROM: Hélio Ricardo Carvalho <hrc@fis.puc-rio.br>
TO: ciencialist@yahoogrupos.com.br
DATE: 28/02/2005 15:08


Voltando para as mensagens de janeiro recuperei uma que me tinha
escapado naquela época.

"pedrolazaromoreira" escreveu:
>
> Olá a todos os membros do ciencialist.
>
> venha por este meio pedir ajuda para me ajudarem a explicar com
toda
> a clareza necessária o porque dos metais apresentarem um brilho
> caracteristico.
>
> Abraço.
> Pedro Moreira.
> ... ... ... ... ... ... ... ...


As respostas dadas não me pareceram muito completas então resolvi
"refletir e interferir". :-)

O "brilho metálico" é o efeito macroscópico da reflexão. Olhe ao seu
redor e você verá que a maioria das coisas tem esta propriedade.
Basta a superfície ser bem lisa. Com materiais orgânicos como
madeira ou papel isto é mais difícil mas nada que um verniz ou uma
plastificação não resolva. Num metal este efeito é mais acentuado. É
este o motivo dos espelhos serem feitos de metal (o metal é
depositado numa superfície de vidro para garantir um plano de metal
bem liso). Até ai tudo bem.

A beleza (e ao mesmo tempo polêmica) é a explicação microscópica
para o fenômeno da reflexão.
Eu acho que quanto mais de perto se olha mais a física se unifica.
A explicação da reflexão é a mesma que explica a refração, o
espalhamento etc.
Os elétrons (uma população deles no ponto considerado) absorvem a
luz e, DEPOIS DE UM PEQUENO TEMPO, emitem. dando como resultado em
média emissão a partir deste ponto para TODAS as direções.
A imagem da luz batendo e voltando como uma bola de bilhar é muito
boa para o segundo grau mas deveria ser retirada dos cursos de
graduação de física.

O que acontece é que dependendo da posição relativa entre objeto,
plano do espelho e observador os efeitos de contribuição construtiva
vão ocorrer.

Para explicar melhor vou exemplificar:
Imagine um lápis em pé na sua frente. A luz que sai da ponta do
lápis vai para todas as direções mas só uma parte dela que irá
atingir o seu olho (para simplificar olhe com um olho só). Esta
parte tem a forma um cone de luz com o vértice na ponta do lápis e a
base do cone é do diâmetro da sua pupila. De cada ponto do lápis sai
luz para todas as direções. Se pensarmos só nas partes de luz que
chegarão ao seu olho serão infinitos cones com base igual (pupila) e
vértices em cada ponto do lápis.

Imagine agora que vejo o mesmo lápis por um espelho de forma que
este esteja na metade do caminho óptico. Em cada ponto do espelho
chegará luz de todos os pontos da sala logo de todos os pontos do
lápis também. Note: a luz proveniente da ponta do lápis está em toda
superfície do espelho e não apenas no lugar que você a vê. E de cada
ponto do espalho ela vai para todas as direções e não somente para o
seu olho. (temos que deixar um pouco o egocentrismo de lado)
A parte da luz de UM ponto do espelho que vai ser vista por você é
aquela que chegou na superfície do espelho naquele ponto e espalhou
para todas as direções mas que na sua direção tem contribuições
construtivas. Para uma outra pessoa ao seu lado não serão mais estas
e sim outras que terão esta contribuição construtiva.

O que dá a ilusão de que funciona como bola da bilhar são as
contribuições construtivas que acontecem por ser a superfície lisa e
por ser aproximadamente iguais os tempos de absorção dos elétrons na
superfície do metal.

[ ]'s
Hélio








SUBJECT: Re: Brilho metálico
FROM: "rmtakata" <rmtakata@altavista.net>
TO: ciencialist@yahoogrupos.com.br
DATE: 28/02/2005 15:38


--- Em ciencialist@yahoogrupos.com.br, Hélio Ricardo Carvalho
> Os elétrons (uma população deles no ponto considerado) absorvem a
> luz e, DEPOIS DE UM PEQUENO TEMPO, emitem. dando como resultado em
> média emissão a partir deste ponto para TODAS as direções.
> A imagem da luz batendo e voltando como uma bola de bilhar é muito
> boa para o segundo grau mas deveria ser retirada dos cursos de
> graduação de física.

Eu discordo disso. Do contrario, ao apontar um feixe de luz para o
espelho a liz teria q se espalhar em todas as direcoes. Se for um
laser, vai se refletir em um ponto bem especifico, com uma relacao
geometrica bem especifica (o angulo do raio refletido eh igual, em
relacao 'a normal, ao angulo do raio incidente).

Sim, do lapis saem feixes de luz em todas as direcoes de um dos
pontos. Mas o lapis nao tem um brilho metalico. O q. caracteriza o
brilho metalico eh uma certa organizacao dos raios refletidos.

[]s,

Roberto Takata





SUBJECT: Re: [ciencialist] Re: gimmick
FROM: "Luiz Ferraz Netto" <leobarretos@uol.com.br>
TO: <ciencialist@yahoogrupos.com.br>
DATE: 28/02/2005 16:32

Trata-se de equipamento de mágica.
Esse que enviei é (deve ser) um baralho especial, com algum truque (talvez esse tal gimmick), para que a carta selecionada saia de dentro do baralho.
A intenção é comprar algum equipamento para mágica para o Guto (meu filho) enxergar como é feito o 'truque' e assim, toda vez que ver 'algo misterioso' despertar a busca pelo 'truque'. Ele já tem 6 anos... e é apenas 1/2 cético!
A página que está cheio desse termo "gimmick" é:
.... não achei! depois envio .....

[]'
===========================
Luiz Ferraz Netto [Léo]
leobarretos@uol.com.br
http://www.feiradeciencias.com.br
===========================
-----Mensagem Original-----
De: "rmtakata" <rmtakata@altavista.net>
Para: <ciencialist@yahoogrupos.com.br>
Enviada em: segunda-feira, 28 de fevereiro de 2005 12:57
Assunto: [ciencialist] Re: gimmick




--- Em ciencialist@yahoogrupos.com.br, "Luiz Ferraz Netto"
> o que é "gimmick"?

Termo em ingles para: aparelho para fraudar; bugiganga; estratagema
para promover um projeto; uma pegadinha; coisa de cujo nome nao nos
lembramos na hora.

> O que é um "Jornal com gimmick " ?

Qual o contexto?

[]s,

Roberto Takata





##### ##### #####

Para saber mais visite
http://www.ciencialist.hpg.ig.com.br


##### ##### ##### #####
Links do Yahoo! Grupos










--
No virus found in this incoming message.
Checked by AVG Anti-Virus.
Version: 7.0.300 / Virus Database: 266.5.0 - Release Date: 25/02/2005




--
No virus found in this outgoing message.
Checked by AVG Anti-Virus.
Version: 7.0.300 / Virus Database: 266.5.0 - Release Date: 25/02/2005



SUBJECT: Re: [ciencialist] No carro: com volt, ampère e ohm
FROM: "Luiz Ferraz Netto" <leobarretos@uol.com.br>
TO: <ciencialist@yahoogrupos.com.br>
DATE: 28/02/2005 16:50

Dentro do contexto do relê:

http://www.feiradeciencias.com.br/sala13/13_40.asp

[]'
===========================
Luiz Ferraz Netto [Léo]
leobarretos@uol.com.br
http://www.feiradeciencias.com.br
===========================
-----Mensagem Original-----
De: "L.E.R.de Carvalho" <lecarvalho@infolink.com.br>
Para: <ciencialist@yahoogrupos.com.br>
Enviada em: segunda-feira, 28 de fevereiro de 2005 15:02
Assunto: [ciencialist] No carro: com volt, ampère e ohm




Coluna Lingua Viva
Deonísio da Silva
JB 28Fev05

No carro: com volt, ampère e ohm


Os dicionários mais consultados do país não têm ''relê'', mas o seu carro
tem. A língua portuguesa foi buscar a palavra no francês ''relais'', do
verbo ''relayer'', revezar, alternar, substituir. Inicialmente foi aplicado
aos cavalos que, fatigados, eram substituídos, principalmente em carroças e
arados, por animais descansados, ensejando que os outros fossem
alimentados, para depois voltarem ao trabalho novamente, no moto perpétuo
que o homem lhes impôs com a domesticação.

Nossa língua acolheu o vocábulo como ''relé'', que está grafado ''relê'' no
Dicionário do carro, do carioca Bob Sharp, jornalista especializado em
automóveis, ex-piloto de competição com vários títulos nacionais e
regionais. Os lexicógrafos lhe devem 1.200 verbetes por ele reunidos,
movido pela paixão do conhecimento, tempero que apresenta diferencial de
qualidade em qualquer pesquisa. Fale, ou melhor, escreva Bob Sharp: ''relê:
dispositivo eletromecânico que, por meio de uma corrente de baixa
intensidade (amperagem), efetua ligação elétrica de outra, de alta
intensidade. Com isso, interruptores de bordo não precisam ser reforçados e
volumosos''.

Mas o que é amperagem? Designa intensidade de corrente elétrica, medida em
amperes, plural de ampere, que o Aurélio ainda não acolheu, preferindo
indexá-lo em sua versão francesa, ''ampère''. Em Portugal, desde 1929, foi
adotada a forma ''ampério'', recusada pelo Brasil por uma daquelas razões
que nos lembram o dito famoso de Bernard Shaw: ''Inglaterra e Estados
Unidos são dois países separados pela mesma língua'', o que pode ser uma
''boutade'' divertida, mas desprovida de base. Alias, ''boutade'', sinônimo
de dito espirituoso, de brincadeira verbal, deriva do verbo ''bouter'',
empurrar. Toda a comunidade lusófona se entende muito bem em vários
continentes.

Vejamos que curiosa a viagem das palavras. ''Ampère'', ''volt'' e ''ohm'',
entre outras palavras, entraram para o francês em 1881, por ocasião de um
congresso de eletricistas realizado naquele ano em Paris. Cientistas
europeus estão na origem das três palavras: o francês André-Marie Ampère
(1775-1836); o italiano Alessandro Volta (1745-1827) e o alemão Georg Simon
Ohm (1789-1854). A eletricidade é nossa companheira inseparável, no carro e
em casa. Explica Sharp: ''Um volt produz uma corrente elétrica de 1 ampère
através de uma resistência de 1 ohm''.

Não denominamos ''volta'', mas ''volt'', porque a palavra deu entrada pelo
francês, que omitiu a vogal final. No carro, falamos de seis volts, 12
volts, mas nas residências em 110 ou 220 volts.

Alessandro Volta, mudo até os 7 anos, foi considerado deficiente mental. Na
universidade onde ensinava, seus colegas achavam que ele era ''pouco
científico''. Já tinha descoberto a pilha quando foi nomeado senador do
reino da Itália por Napoleão Bonaparte. A Academia de Ciências da França
recusou-se a condecorá-lo, alegando que o regulamento vetava premiações a
estrangeiros. Não é de hoje... Napoleão revidou: acrescentou à medalha de
ouro por reconhecimento intelectual um prêmio de seis mil francos.

O cientista, que morreu aos 82 anos, quis aposentar-se quando chegou à
velhice, mas outra vez o imperador interveio: se estava cansado de ensinar,
que desse apenas uma lição por ano, ''pois os bons generais morrem no campo
de batalha''. Desde que não sejam presos e exilados numa ilha...

No Brasil atual, leis esdrúxulas teriam chutado o velho cientista para bem
longe de qualquer campus. Mas seu nome está presente em nossa vida
cotidiana: em casa, nos meios de transporte, no trabalho. E no automóvel, o
substituto do cavalo. Até a invenção do trem, no século 19, o cavalo foi o
meio de transporte mais rápido.



[As partes desta mensagem que não continham texto foram removidas]



##### ##### #####

Para saber mais visite
http://www.ciencialist.hpg.ig.com.br


##### ##### ##### #####
Links do Yahoo! Grupos










--
No virus found in this incoming message.
Checked by AVG Anti-Virus.
Version: 7.0.300 / Virus Database: 266.5.0 - Release Date: 25/02/2005




--
No virus found in this outgoing message.
Checked by AVG Anti-Virus.
Version: 7.0.300 / Virus Database: 266.5.0 - Release Date: 25/02/2005



SUBJECT: RN: Veja denuncia "medicina natural de A a Z" como farsa
FROM: André Roviralta Dias Baptista <andrediasbaptista@yahoo.com.br>
TO: ciencialist@yahoogrupos.com.br
DATE: 28/02/2005 16:55


Essa denúncia me parece inédita e auspiciosa. A partir dela, cria-se
um paradigma a ser seguido e cobrado da revista.

http://veja.abril.com.br/020305/p_112.html

"Um dos maiores best-sellers do país no momento, o livro Medicina
Alternativa de A a Z é uma fraude. Apanhado de crendices e terapias
não-avalizadas pelos médicos, ele ensina a tratar doenças
com "remédios" à base de ingredientes frugais. Para combater o
câncer, recomenda que se coma apenas um tipo de fruta nas refeições.
No caso do diabetes, indica a ingestão de suco de berinjela com
argila. E por aí afora. Com capa dura, fartamente ilustrado e preço
de 62 reais, o livro já atingiu a marca dos 2 milhões de exemplares
comercializados e apareceu no topo da lista de mais vendidos de VEJA
por quase seis meses. Suas vendas foram impulsionadas por uma
campanha de divulgação maciça na televisão."

"Uma picaretagem como Medicina Alternativa de A a Z representa um
risco para a saúde das pessoas. Os alimentos ajudam na manutenção de
uma vida saudável, mas não curam doenças graves. Para eximir-se de
responsabilidades, o livro volta e meia chama atenção para a
necessidade de procurar um médico. Ao mesmo tempo, entretanto, propõe
tratamentos mágicos (veja quadro). "Um livro como esse é um
desserviço à população. Não pode ser chamado de medicina", diz o
geriatra Clineu Almada Filho, da Escola Paulista de Medicina. "É uma
irresponsabilidade", afirma o clínico geral Milton Glezer, da
Universidade de São Paulo."

" a partir desta edição, Medicina Alternativa de A a Z será retirado
da lista de mais vendidos de VEJA. A cada semana, a lista oferece um
instantâneo do mercado editorial. Mas também é usada como um guia de
compras pelos leitores, razão pela qual a revista tomou essa decisão."







SUBJECT: Re: [ciencialist] RN: Veja denuncia "medicina natural de A a Z" como farsa
FROM: "Eurico Ferreira de Souza Jr." <caodejah@yahoo.com.br>
TO: ciencialist@yahoogrupos.com.br
DATE: 28/02/2005 17:10

[E]> a veja vai censurar a lista dos mais vendidos? vai ter que mudar o nome para "lista dos livros mais vendidos indicados pela veja". Não se combate mentira com mentira...

Percebam o interesse da população por qualquer "medicina alternativa". É indicativo do descontentamento com o monopólio da medicina.


_\|/_

---------------------------------
Yahoo! Acesso Grátis - Internet rápida e grátis. Instale o discador do Yahoo! agora.

[As partes desta mensagem que não continham texto foram removidas]



SUBJECT: Re: [ciencialist] RN: Veja denuncia "medicina natural de A a Z" como farsa
FROM: "E m i l i a n o C h e m e l l o" <chemelloe@yahoo.com.br>
TO: <ciencialist@yahoogrupos.com.br>
DATE: 28/02/2005 17:15

Puxa. Ainda bem que alguém falou sobre este livro. Já diz o velho ditado: "quando a esmola é demais..."

[ ] 's do Emiliano Chemello
emiliano@quimica.net
http://www.quimica.net/emiliano
http://www.ucs.br/ccet/defq/naeq

" Rien ne se perd, rien ne se crée,
tout se transforme."

Antoine Laurent de Lavoisier (químico francês, 1743 - 1794)

----- Original Message -----
From: André Roviralta Dias Baptista
To: ciencialist@yahoogrupos.com.br
Sent: Monday, February 28, 2005 4:55 PM
Subject: [ciencialist] RN: Veja denuncia "medicina natural de A a Z" como farsa



Essa denúncia me parece inédita e auspiciosa. A partir dela, cria-se
um paradigma a ser seguido e cobrado da revista.

http://veja.abril.com.br/020305/p_112.html

"Um dos maiores best-sellers do país no momento, o livro Medicina
Alternativa de A a Z é uma fraude. Apanhado de crendices e terapias
não-avalizadas pelos médicos, ele ensina a tratar doenças
com "remédios" à base de ingredientes frugais. Para combater o
câncer, recomenda que se coma apenas um tipo de fruta nas refeições.
No caso do diabetes, indica a ingestão de suco de berinjela com
argila. E por aí afora. Com capa dura, fartamente ilustrado e preço
de 62 reais, o livro já atingiu a marca dos 2 milhões de exemplares
comercializados e apareceu no topo da lista de mais vendidos de VEJA
por quase seis meses. Suas vendas foram impulsionadas por uma
campanha de divulgação maciça na televisão."

"Uma picaretagem como Medicina Alternativa de A a Z representa um
risco para a saúde das pessoas. Os alimentos ajudam na manutenção de
uma vida saudável, mas não curam doenças graves. Para eximir-se de
responsabilidades, o livro volta e meia chama atenção para a
necessidade de procurar um médico. Ao mesmo tempo, entretanto, propõe
tratamentos mágicos (veja quadro). "Um livro como esse é um
desserviço à população. Não pode ser chamado de medicina", diz o
geriatra Clineu Almada Filho, da Escola Paulista de Medicina. "É uma
irresponsabilidade", afirma o clínico geral Milton Glezer, da
Universidade de São Paulo."

" a partir desta edição, Medicina Alternativa de A a Z será retirado
da lista de mais vendidos de VEJA. A cada semana, a lista oferece um
instantâneo do mercado editorial. Mas também é usada como um guia de
compras pelos leitores, razão pela qual a revista tomou essa decisão."







##### ##### #####

Para saber mais visite
http://www.ciencialist.hpg.ig.com.br


##### ##### ##### #####


Yahoo! Grupos, um serviço oferecido por:

São Paulo Rio de Janeiro Curitiba Porto Alegre Belo Horizonte Brasília




------------------------------------------------------------------------------
Links do Yahoo! Grupos

a.. Para visitar o site do seu grupo na web, acesse:
http://br.groups.yahoo.com/group/ciencialist/

b.. Para sair deste grupo, envie um e-mail para:
ciencialist-unsubscribe@yahoogrupos.com.br

c.. O uso que você faz do Yahoo! Grupos está sujeito aos Termos do Serviço do Yahoo!.



[As partes desta mensagem que não continham texto foram removidas]



SUBJECT: Re: Brilho metálico
FROM: Hélio Ricardo Carvalho <hrc@fis.puc-rio.br>
TO: ciencialist@yahoogrupos.com.br
DATE: 28/02/2005 17:22


"rmtakata" escreveu

>
> --- Em ciencialist@yahoogrupos.com.br, Hélio Ricardo Carvalho
> > Os elétrons (uma população deles no ponto considerado) absorvem
a
> > luz e, DEPOIS DE UM PEQUENO TEMPO, emitem. dando como resultado
em
> > média emissão a partir deste ponto para TODAS as direções.
> >
> > ...
> Eu discordo disso. Do contrario, ao apontar um feixe de luz para o
> espelho a liz teria q se espalhar em todas as direcoes. Se for um
> laser, vai se refletir em um ponto bem especifico, com uma relacao
> geometrica bem especifica (o angulo do raio refletido eh igual, em
> relacao 'a normal, ao angulo do raio incidente).
>
> Sim, do lapis saem feixes de luz em todas as direcoes de um dos
> pontos. Mas o lapis nao tem um brilho metalico. O q. caracteriza o
> brilho metalico eh uma certa organizacao dos raios refletidos.
>

"Por construção" a luz do laser é direcional e coerente o que faz
ele ser especial de certa forma. Mas em linhas gerais o raciocínio
continua o mesmo.

"Só" aquela região do espelho será iluminada. (este só é uma
situação idealizada pois no caminho há espalhamentos devido as
moléculas do ar: absorção, tempo, reemissão em todas as direções).
Mas cada ponto do espelho desta região iluminada pelo laser irá
emitir em todas as direções. Você só verá (com óculos de proteção,
por favor) em uma direção específica pois só por este caminho que
está havendo contribuição construtiva.

[ ]'s
Hélio





SUBJECT: Re: RN: Veja denuncia "medicina natural de A a Z" como farsa
FROM: "rmtakata" <rmtakata@altavista.net>
TO: ciencialist@yahoogrupos.com.br
DATE: 28/02/2005 17:22


--- Em ciencialist@yahoogrupos.com.br, "Eurico Ferreira de Souza Jr."
> Não se combate mentira com mentira...

Ateh aih nao houve nenhuma mentira: a Veja avisou q. estava retirando
e o porq. de retirar.

> Percebam o interesse da população por qualquer "medicina alternativa".
> É indicativo do descontentamento com o monopólio da medicina.

Sim. O q. nao quer dizer q. a tal 'medicina alternativa' seja melhor
do q. a medicina (em termos de eficiencia de tratamento).

[]s,

Roberto Takata





SUBJECT: Re: Brilho metálico
FROM: "rmtakata" <rmtakata@altavista.net>
TO: ciencialist@yahoogrupos.com.br
DATE: 28/02/2005 17:27


--- Em ciencialist@yahoogrupos.com.br, Hélio Ricardo Carvalho
> Você só verá (com óculos de proteção, por favor) em uma direção
> específica pois só por este caminho que está havendo contribuição
> construtiva.

Isso apenas se considerarmos a explicacao quantica (como a de Fermi)
de q. cada particula viaja por todos os caminhos...

Mesmo com uma luz normal, nao ha' espalhamento, alem do q. ocorreria
se a luz percorresse um caminho de igual comprimento sem reflexao, ao
se refletir no espelho.

A reflexao especular eh organizada, por isso forma imagens. Do
contrario seria um borrao branco.

[]s,

Roberto Takata





SUBJECT: Re: [ciencialist] RN: Veja denuncia "medicina natural de A a Z" como farsa
FROM: "Oraculo" <oraculo@atibaia.com.br>
TO: <ciencialist@yahoogrupos.com.br>
DATE: 28/02/2005 17:27

Olá Eurico

A retirada de um livro claramente fraudulento, com recomendações que colocam em perigo a vida de quem o consulta, dificilmente se enquadraria na categoria de censura. Está mais para ação de responsabilidade social, que deveria ser seguida por todos os meios de comunicação, mesmo os que lucram com determinadas maluquices como medicinas alternativas.

O interesse da população por medicina alternativa (que em geral significa apenas medicina sem comprovação de eficácia) não se dá devido ao "monopolio" da medicina tradicional, mas sim devido a pouca informação do público em geral sobre mecanismos de ação, metabolismo e biologia, segurança de tratamento, aliada a curiosidade natural de seres humanos, busca por alternativas à falta de acesso a medicina tradicional (que qualquer pessoa preferiria, se fosse possível o acesso universal a esta), e diversos outros fatores.

Motoristas de terceiro mundo (e alguns no primeiro também) tem grande atração por comportamentos de risco, como dirigir a altas velocidades e ignorar leis de transito, e é dever tanto do estado quanto dos meios de comunicação lutar contra essa atitude. Da mesma forma, o interesse em tratar cancer com argila e sucos deve ser investigado dentro de rigorosos procedimentos de verificaçào, e não receitado em livros de procedencia prá lá de duvidosa (eu diria criminosa mesmo).

Um tratamento avançado, com medicina de ponta, se não cura 100% de uma doença (e nada cura ou afirma curar), tem o mérito de ter sido testado com niveis de segurança e produzido resultados, comprováveis, muitas vezes maior que as alternativas (não tratar ou tratar com medicina alternativa). E pode apresentar suas evidencias disso.

Ainda que tenha críticas a revista Veja, como de resto sobre a maioria das revistas de entretenimento em geral, e mesmo a mídia como um todo, a ação da revista neste caso foi legítma e digna de ser aplaudida. Livrarias que comercializam o livro deveriam disponibilizar o artigo para seus clientes, como forma de prevenir o gasto desnecessário e o risco que este tipo de "literatura" e de "medicina alternativa" pode causar.

Um abraço.

Homero







----- Original Message -----
From: Eurico Ferreira de Souza Jr.
To: ciencialist@yahoogrupos.com.br
Sent: Monday, February 28, 2005 5:10 PM
Subject: Re: [ciencialist] RN: Veja denuncia "medicina natural de A a Z" como farsa


[E]> a veja vai censurar a lista dos mais vendidos? vai ter que mudar o nome para "lista dos livros mais vendidos indicados pela veja". Não se combate mentira com mentira...

Percebam o interesse da população por qualquer "medicina alternativa". É indicativo do descontentamento com o monopólio da medicina.


_\|/_

---------------------------------
Yahoo! Acesso Grátis - Internet rápida e grátis. Instale o discador do Yahoo! agora.

[As partes desta mensagem que não continham texto foram removidas]



##### ##### #####

Para saber mais visite
http://www.ciencialist.hpg.ig.com.br


##### ##### ##### #####


Yahoo! Grupos, um serviço oferecido por:







------------------------------------------------------------------------------
Links do Yahoo! Grupos

a.. Para visitar o site do seu grupo na web, acesse:
http://br.groups.yahoo.com/group/ciencialist/

b.. Para sair deste grupo, envie um e-mail para:
ciencialist-unsubscribe@yahoogrupos.com.br

c.. O uso que você faz do Yahoo! Grupos está sujeito aos Termos do Serviço do Yahoo!.



[As partes desta mensagem que não continham texto foram removidas]



SUBJECT: A a Z
FROM: "L.E.R.de Carvalho" <lecarvalho@infolink.com.br>
TO: ciencialist@yahoogrupos.com.br
DATE: 28/02/2005 17:31

A VEJA não quer é concorrentes.




SUBJECT: Re: [ciencialist] RN: Veja denuncia "medicina natural de A a Z" como farsa
FROM: Amauri Nolasco Sanches Jr <amaurijunior2@yahoo.com.br>
TO: ciencialist@yahoogrupos.com.br
DATE: 28/02/2005 17:42

Oi Oraculo

Minha mãe tem e fez e deu certo, como uma coisa
natural pode fazer mal? Será que não é uma
armação dos laboratórios?

Amauri




--- Oraculo <oraculo@atibaia.com.br> escreveu:

---------------------------------
Olá Eurico

A retirada de um livro claramente fraudulento, com
recomendações que colocam em perigo a vida de quem o
consulta, dificilmente se enquadraria na categoria de
censura. Está mais para ação de responsabilidade
social, que deveria ser seguida por todos os meios de
comunicação, mesmo os que lucram com determinadas
maluquices como medicinas alternativas.

O interesse da população por medicina alternativa (que
em geral significa apenas medicina sem comprovação de
eficácia) não se dá devido ao "monopolio" da medicina
tradicional, mas sim devido a pouca informação do
público em geral sobre mecanismos de ação, metabolismo
e biologia, segurança de tratamento, aliada a
curiosidade natural de seres humanos, busca por
alternativas à falta de acesso a medicina tradicional
(que qualquer pessoa preferiria, se fosse possível o
acesso universal a esta), e diversos outros fatores.

Motoristas de terceiro mundo (e alguns no primeiro
também) tem grande atração por comportamentos de
risco, como dirigir a altas velocidades e ignorar leis
de transito, e é dever tanto do estado quanto dos
meios de comunicação lutar contra essa atitude. Da
mesma forma, o interesse em tratar cancer com argila e
sucos deve ser investigado dentro de rigorosos
procedimentos de verificaçào, e não receitado em
livros de procedencia prá lá de duvidosa (eu diria
criminosa mesmo).

Um tratamento avançado, com medicina de ponta, se não
cura 100% de uma doença (e nada cura ou afirma curar),
tem o mérito de ter sido testado com niveis de
segurança e produzido resultados, comprováveis, muitas
vezes maior que as alternativas (não tratar ou tratar
com medicina alternativa). E pode apresentar suas
evidencias disso.

Ainda que tenha críticas a revista Veja, como de resto
sobre a maioria das revistas de entretenimento em
geral, e mesmo a mídia como um todo, a ação da revista
neste caso foi legítma e digna de ser aplaudida.
Livrarias que comercializam o livro deveriam
disponibilizar o artigo para seus clientes, como forma
de prevenir o gasto desnecessário e o risco que este
tipo de "literatura" e de "medicina alternativa" pode
causar.

Um abraço.

Homero







----- Original Message -----
From: Eurico Ferreira de Souza Jr.
To: ciencialist@yahoogrupos.com.br
Sent: Monday, February 28, 2005 5:10 PM
Subject: Re: [ciencialist] RN: Veja denuncia
"medicina natural de A a Z" como farsa


[E]> a veja vai censurar a lista dos mais vendidos?
vai ter que mudar o nome para "lista dos livros mais
vendidos indicados pela veja". Não se combate mentira
com mentira...

Percebam o interesse da população por qualquer
"medicina alternativa". É indicativo do
descontentamento com o monopólio da medicina.


_\|/_

---------------------------------
Yahoo! Acesso Grátis - Internet rápida e grátis.
Instale o discador do Yahoo! agora.

[As partes desta mensagem que não continham texto
foram removidas]



##### ##### #####

Para saber mais visite
http://www.ciencialist.hpg.ig.com.br


##### ##### ##### #####


Yahoo! Grupos, um serviço oferecido por:







------------------------------------------------------------------------------
Links do Yahoo! Grupos

a.. Para visitar o site do seu grupo na web,
acesse:
http://br.groups.yahoo.com/group/ciencialist/

b.. Para sair deste grupo, envie um e-mail para:
ciencialist-unsubscribe@yahoogrupos.com.br

c.. O uso que você faz do Yahoo! Grupos está
sujeito aos Termos do Serviço do Yahoo!.



[As partes desta mensagem que não continham texto
foram removidas]



##### ##### #####

Para saber mais visite
http://www.ciencialist.hpg.ig.com.br


##### ##### ##### #####


Yahoo! Grupos, um serviço oferecido por:

function SearchComboBox() {
if (document.form_combo.keyword.value.length==0){
alert("Por favor, digite algo."); return false; }else
{ document.form_combo.action
="http://br.rd.yahoo.com/SIG=12anq1f40/M=264105.3931087.6562589.1588051/D=brclubs/S=2137111528:HM/EXP=1109708879/A=2361264/R=0/SIG=11uaou2jn/*http://www.bondfaro.com/bondfaro/in/combosearch_in.jsp?sk=11";
} return true;}
[input]
[input] [input]


---------------------------------
Links do Yahoo! Grupos

Para visitar o site do seu grupo na web, acesse:
http://br.groups.yahoo.com/group/ciencialist/

Para sair deste grupo, envie um e-mail para:
ciencialist-unsubscribe@yahoogrupos.com.br

O uso que você faz do Yahoo! Grupos está sujeito
aos Termos do Serviço do Yahoo!.


=====

Meu grupo sobre sexualidade: http://br.groups.yahoo.com/group/Sexualidadedodef/?yguid=94014452

" O dia mais importante não é quando conhecemos as pessoas mas sim o momento em que elas passam a existir dentro de nós. "

MSN: amaurijunior3@hotmail.com

icq:153134120

Blog:http://blogdomaster.weblogger.terra.com.br/index.htm





















_______________________________________________________
Yahoo! Acesso Grátis - Instale o discador do Yahoo! agora. http://br.acesso.yahoo.com/ - Internet rápida e grátis


SUBJECT: Re: Brilho metálico
FROM: "rmtakata" <rmtakata@altavista.net>
TO: ciencialist@yahoogrupos.com.br
DATE: 28/02/2005 17:54


--- Em ciencialist@yahoogrupos.com.br, "rmtakata" <rmtakata@a...>
> A reflexao especular eh organizada, por isso forma imagens. Do
> contrario seria um borrao branco.

Inseri na secao de arquivos da lista um gif ilustrando isso
(reflexo.gif): http://tinyurl.com/3odbx

Se o espelho refletisse para todos os lados um raio de luz incidente
teriamos o caso em q. a ponta do lapis iria aparecer na outra ponta na
imagem - linha em vermelho.

[]s,

Roberto Takata







SUBJECT: Re: gimmick
FROM: "rmtakata" <rmtakata@altavista.net>
TO: ciencialist@yahoogrupos.com.br
DATE: 28/02/2005 17:56


--- Em ciencialist@yahoogrupos.com.br, "Luiz Ferraz Netto"
> Trata-se de equipamento de mágica.

Entao esse gimmick eh um apetrecho especial q. permite a realizacao do
truque: seria, no caso, um jornal preparado (possivelmente para aquele
truque em q. dobramos uma folha de jornal e despejamos liquido nela
sem q. vaze ou molhe o jornal).

[]s,

Roberto Takata





SUBJECT: Re: RN: Veja denuncia "medicina natural de A a Z" como farsa
FROM: "rmtakata" <rmtakata@altavista.net>
TO: ciencialist@yahoogrupos.com.br
DATE: 28/02/2005 18:00


--- Em ciencialist@yahoogrupos.com.br, Amauri Nolasco Sanches Jr
> Minha mãe tem e fez e deu certo, como uma coisa natural pode fazer
> mal? Será que não é uma armação dos laboratórios?

Veneno de cobra eh natural, nao consta q. nas doses q. ela aplica faca
bem. Um raio tb eh natural, nao consta q. faca bem qdo cai na cabeca
de alguem. Um tronco de arvore eh natural, nao consta q. faca bem qdo
cai sobre alguem.

Se sua mae separar tres grupos de pessoas: uma em q. nao aplica nada,
outro em q. aplica o q. o livro recomenda e o terceiro em q. se aplica
o tratamento medico convencional - e o grupo em q. aplica o tratamento
recomendado pelo livro tiver uma media de recuperacao melhor do q. nos
outros grupos, entao pode ser uma armacao dos laboratorios.

[]s,

Roberto Takata





SUBJECT: Re: [ciencialist] RN: Veja denuncia "medicina natural de A a Z" como farsa
FROM: "Eurico Ferreira de Souza Jr." <caodejah@yahoo.com.br>
TO: ciencialist@yahoogrupos.com.br
DATE: 28/02/2005 18:03



Oraculo <oraculo@atibaia.com.br> wrote:

A retirada de um livro claramente fraudulento,

[E]> (através de ranking fraudolento.)

com recomendações que colocam em perigo a vida de quem o consulta, dificilmente se enquadraria na categoria de censura. Está mais para ação de responsabilidade social, que deveria ser seguida por todos os meios de comunicação, mesmo os que lucram com determinadas maluquices como medicinas alternativas.



[E]> responsabilidade social seria proibir o livro. mentir sobre números de vendas não é responsabilidade social.

O interesse da população por medicina alternativa (que em geral significa apenas medicina sem comprovação de eficácia)

[E]> p. ex. eu tinha um monte de verrugas no cotovelo e curei esfregando palitos de fósforos, que devolvi à caixa que joguei fora por cima do ombro sem olhar para trás (depois confirmei em livros de MEDICINA que se cura verrugas com sugestão)...

não se dá devido ao "monopolio" da medicina tradicional, mas sim devido a pouca informação do público em geral sobre mecanismos de ação, metabolismo e biologia, segurança de tratamento,

[E]> curiosidade nunca satisfeita. Os médicos querem seus pacientes totalmente ignorantes para
não terem que estudar mais nem serem contestados. Quase toda vez que tive que me consultar com médicos, fui tratado como retardado (tá, fui tratado como "cidadão normal"). Os médicos, em geral, não tem o menor respeito pelos pacientes
(aliás, o nome "paciente" já indica como eles encaram seus clientes)

aliada a curiosidade natural de seres humanos, busca por alternativas à falta de acesso a medicina tradicional (que qualquer pessoa preferiria, se fosse possível o acesso universal a esta), e diversos outros fatores.

[E]> É natural que as pessoas queiram saber se curar (e não uma curiosidade descabida) A humanidade subsistiu com essa curiosidade ...e vira e mexe descobrem que merthiolate não desinfeta, vioxx mata, etc. cadê a responsabilidade nesses casos?

Motoristas de terceiro mundo (e alguns no primeiro também) tem grande atração por comportamentos de risco, como dirigir a altas velocidades e ignorar leis de transito, e é dever tanto do estado quanto dos meios de comunicação lutar contra essa atitude. Da mesma forma, o interesse em tratar cancer com argila e sucos deve ser investigado dentro de rigorosos procedimentos de verificaçào, e não receitado em livros de procedencia prá lá de duvidosa (eu diria criminosa mesmo).

[E]> eu não tô defendendo o livro... mas sim a verdade (o livro vende!)

Um tratamento avançado, com medicina de ponta, se não cura 100% de uma doença (e nada cura ou afirma curar), tem o mérito de ter sido testado com niveis de segurança e produzido resultados, comprováveis, muitas vezes maior que as alternativas (não tratar ou tratar com medicina alternativa). E pode apresentar suas evidencias disso.

[E]> Medicina é artigo de luxo para a elite. Deixemos que a cangalha ao menos tenha seus placebos (que remédio vc receitaria pros indiozinhos que tão morrendo de desnutrição?)

Ainda que tenha críticas a revista Veja,

[E]> não tenho nenhuma birra a priori com a veja

como de resto sobre a maioria das revistas de entretenimento em geral, e mesmo a mídia como um todo, a ação da revista neste caso foi legítma e digna de ser aplaudida.

[E]> putz... (mentira nunca é legítima e digna de aplausos)

Livrarias que comercializam o livro deveriam disponibilizar o artigo para seus clientes, como forma de prevenir o gasto desnecessário e o risco que este tipo de "literatura" e de "medicina alternativa" pode causar.

[E]> que se responsabilize o autor pelas consequências...

[]s

Eurico


_\|/_
__________________________________________________
Converse com seus amigos em tempo real com o Yahoo! Messenger
http://br.download.yahoo.com/messenger/

[As partes desta mensagem que não continham texto foram removidas]



SUBJECT: Re: [ciencialist] Re: RN: Veja denuncia "medicina natural de A a Z" como farsa
FROM: "Eurico Ferreira de Souza Jr." <caodejah@yahoo.com.br>
TO: ciencialist@yahoogrupos.com.br
DATE: 28/02/2005 18:06



rmtakata <rmtakata@altavista.net> wrote:
(...)

> Percebam o interesse da população por qualquer "medicina alternativa".
> É indicativo do descontentamento com o monopólio da medicina.

Sim. O q. nao quer dizer q. a tal 'medicina alternativa' seja melhor
do q. a medicina (em termos de eficiencia de tratamento).

[E]> sim. eu não sugeri o contrário. mas também não afirmo que toda medicina alternativa é fraudolenta. (vira e mexe a medicina tem que "engolir" estrepolias como a acupuntura, homeopatia, etc)

[]s

Eurico



_\|/_
__________________________________________________
Converse com seus amigos em tempo real com o Yahoo! Messenger
http://br.download.yahoo.com/messenger/

[As partes desta mensagem que não continham texto foram removidas]



SUBJECT: Re: RN: Veja denuncia "medicina natural de A a Z" como farsa
FROM: "rmtakata" <rmtakata@altavista.net>
TO: ciencialist@yahoogrupos.com.br
DATE: 28/02/2005 18:11


--- Em ciencialist@yahoogrupos.com.br, "Eurico Ferreira de Souza Jr."
> [E]> sim. eu não sugeri o contrário. mas também não afirmo que toda
> medicina alternativa é fraudolenta. (vira e mexe a medicina tem que
> "engolir" estrepolias como a acupuntura, homeopatia, etc)

A homeopatia e a acupuntura foram engolidas apenas por determinacao
politica do CFM. Nao ha' nenhuma mencao maior a algum estudo q. tenha
comprovado (acima de qq duvida razoavel) a eficiencia dessas terapias.

Mas de fato pode ser q. haja alguma coisa q. se salve em meio a tta
proposta. O problema eh q. elas sao vendidas sem terem passado por
nenhum teste. Testes controlados sao o melhor meio de se separar o
joio do trigo sem por a saude de tta gente em risco.

Vc nao gostaria q. liberassem para vender como comida qq porcaria q. a
industria inventasse: tem q. passar por um teste pra ver se nao eh
perigoso para a saude.

Eu tb nao gostaria q. liberassem qq porcaria como medicamento ou
tratamento.

[]s,

Roberto Takata





SUBJECT: Relatividade e relógios de luz
FROM: "Ricardo Soares Vieira" <rickrsv@yahoo.com.br>
TO: ciencialist@yahoogrupos.com.br
DATE: 28/02/2005 18:24


Olá a todos do ciencialist,

Me interessei pelas mensagens que discutiram a experiência
do "relógio de luz", utilizados para deduzir a dilatação do tempo da
relatividade de uma forma fácil. Nestas mensagens, não me recordo
quais, perguntaram o que ocorreria se a luz fosse lançada na direção
do movimento da caixa; isso eu já tinha me perguntado também e
cheguei a seguinte conclusão depois de fazer umas continhas...

Vou discutir aqui o que eu encontrei:

1. Relógio-Luz na perpendicular ao movimento:

Suponham que numa caixa quadrada de lados L alguém meça o tempo por
raios de luz: primeiro ele lança um raio na direção vertical Y. O
tempo gasto para a luz ir até o teto será

t = L/c.

Mas para alguém que vê a caixa se movimentar na direção X com
velocidade v, a luz percorrerá um trajeto diagonal d, cujos catetos
serão respectivamente Y e v.t', onde t' é o tempo que queremos
encontrar (que não diremos nada a priori) assim, podemos escrever
por Pitágoras:

d'^2 = L^2 + (vt')^2

Mas sendo d' = c.t', e c.t = L, e assumindo que é constante a
velocidade da luz, temos:

(ct')^2 = (c.t)^2 + (vt')^2

(c^2 - v^2)t´^2 = (ct)^2

t'= t/(1 - v^2/c^2)^1/2

que é a equação da relatividade que conhecemos, do qual concluimos
que um relógio na vertical, visto em movimento, é mais lento...


2. Relógio-Luz na direção do movimento:


Mas o qual resultado encontraria este observador se o cientista da
caixa lançasse a luz na direção X?

Neste caso, o tempo de ida será diferente do de volta, obviamente
pois que no primeiro caso a luz caminhará no mesmo sentido da caixa,
e no segundo, no sentido oposto.

Antes de mais nada, temos que levar em consideração que o
comprimento da caixa na direção X estará contraída via Lorentz, pela
equação:

L" = L . (1 - v^2/c^2)^1/2

Assim, na ida, teremos a luz partindo do ponto zero, e o extremo da
caixa que parte do ponto L ". O tempo de ida corresponde ao tempo do
encontro da luz com esse extremo da caixa, logo temos:

ct' = L .[(1 - v^2/c^2)^1/2] + vt'

Mas sendo L = c.t, temos:

(c-v)t' =ct .[(1 - v^2/c^2)^1/2]

logo,

t' = [t . (1 - v^2/c^2)^1/2]/(1 - v/c)

Analogamente, na volta, teremos:

t' = [t . (1 - v^2/c^2)^1/2]/(1 + v/c)

Assim, o tempo total (T' = 2t'), ou seja, a soma do tempo da ida com
o da volta, depois de fazer os cálculos, vamos encontrar que:

t'= t/(1 - v^2/c^2)^1/2

Ou seja, igual ao anterior, onde o relógio-lus estava na vertical.
Assim vemos que, ao considerar o relógio na horizontal como um tique
e taque completo, ele funcionaria mais lento tal qual a
relatividade...

Ora considere que a nossa caixa seja esferica e espelhada por
dentro, uma luz emitida do seu centro atingirá a esfera
simultaneamente, e em determinado instante se encontrará novamente
no ponto de orígem. Mas quando vista em movimento, teremos uma
elipse, a luz atingirá primeiramente a parte de tras, depois o teto
e por ultimo a frente, mas ao voltar, se encontrar em um determinado
ponto de uma só vez...

Essa é conclusão que chegamos pela relativiade habitual. Eu estou
desenvolvendo outra (se me permitem), na qual considero que a altura
dos corpos é diminuida, nessa teoria minha, é importante além do
movimento, o fato se os corpos se aproximam ou se afastam do
observador. O problema aqui seria resolvido de forma muito mais
simples...

Até mais, espero ter explicado mais que complicado...

Rick






SUBJECT: Re: [ciencialist] RN: Veja denuncia "medicina natural de A a Z" como farsa
FROM: Victor Pimentel Nunes <victor2002@gmail.com>
TO: ciencialist@yahoogrupos.com.br
DATE: 28/02/2005 18:29

Na Seção "Os Mais Vendidos" deveria ter uma nota também falando da
ausência do livro no "ranking(???)"

Aquilo não é um guia porcaria nenhuma. Aquilo é simplesmente (deveria
ser) a lista de livros mais vendidos nas livrarias mencionadas.

Tratar uma lista de mais vendidos (o que é uma análise QUANTITATIVA)
como guia (o que no caso sugere uma análise qualitativa) parece
história para boi dormir.

CASO HIPOTÉTICO (Muito possível de ocorrer): Alguém em uma sala de
espera pega a revista VEJA. É muito comum em sala de espera, ler-se
essas partes da revistas (assim como aquelas de frases ou de notícias
curtas) e não ler-se as matérias ao longo da revista.
Então a pessoa que ler a seção "Os Mais Vendidos" de VEJA, aceitará
que os 10 mais vendidos na categoria são de fato os 10 mais vendidos
(consideremos aqui que o leitor não duvide da revista).
Ao ir ao bar com seus amigos e um falar que comprou um livro X só
porque ele é um dos 10 mais vendidos no país, o leitor da sala de
espera irá corrigí-lo.
Não só irá corrigí-lo como irá mostrar a fonte, a revista VEJA.

Caso a revista não queira compactuar com algo que considera
picaretagem, ela poderia mudar o nome do ranking para "Os livros
recomendados por VEJA mais vendidos". Ou então deixar como está e
publicar editoriais ou notas de rodapé dizendo que não recomenda o
livro, mas sem negar o fato que ele é um dos mais vendidos.

Saudações
Victor

PS: Tirando a palavra porcaria do 2º parágrafo, mandarei esse e-mail para VEJA






On Mon, 28 Feb 2005 18:03:39 -0300 (ART), Eurico Ferreira de Souza Jr.
<caodejah@yahoo.com.br> wrote:
>
>
> Oraculo <oraculo@atibaia.com.br> wrote:
>
> A retirada de um livro claramente fraudulento,
>
> [E]> (através de ranking fraudolento.)
>
> com recomendações que colocam em perigo a vida de quem o consulta,
> dificilmente se enquadraria na categoria de censura. Está mais para ação de
> responsabilidade social, que deveria ser seguida por todos os meios de
> comunicação, mesmo os que lucram com determinadas maluquices como medicinas
> alternativas.
>
>
>
> [E]> responsabilidade social seria proibir o livro. mentir sobre números de
> vendas não é responsabilidade social.
>
> O interesse da população por medicina alternativa (que em geral significa
> apenas medicina sem comprovação de eficácia)
>
> [E]> p. ex. eu tinha um monte de verrugas no cotovelo e curei esfregando
> palitos de fósforos, que devolvi à caixa que joguei fora por cima do ombro
> sem olhar para trás (depois confirmei em livros de MEDICINA que se cura
> verrugas com sugestão)...
>
> não se dá devido ao "monopolio" da medicina tradicional, mas sim devido a
> pouca informação do público em geral sobre mecanismos de ação, metabolismo e
> biologia, segurança de tratamento,
>
> [E]> curiosidade nunca satisfeita. Os médicos querem seus pacientes
> totalmente ignorantes para
> não terem que estudar mais nem serem contestados. Quase toda vez que tive
> que me consultar com médicos, fui tratado como retardado (tá, fui tratado
> como "cidadão normal"). Os médicos, em geral, não tem o menor respeito pelos
> pacientes
> (aliás, o nome "paciente" já indica como eles encaram seus clientes)
>
> aliada a curiosidade natural de seres humanos, busca por alternativas à
> falta de acesso a medicina tradicional (que qualquer pessoa preferiria, se
> fosse possível o acesso universal a esta), e diversos outros fatores.
>
> [E]> É natural que as pessoas queiram saber se curar (e não uma curiosidade
> descabida) A humanidade subsistiu com essa curiosidade ...e vira e mexe
> descobrem que merthiolate não desinfeta, vioxx mata, etc. cadê a
> responsabilidade nesses casos?
>
> Motoristas de terceiro mundo (e alguns no primeiro também) tem grande
> atração por comportamentos de risco, como dirigir a altas velocidades e
> ignorar leis de transito, e é dever tanto do estado quanto dos meios de
> comunicação lutar contra essa atitude. Da mesma forma, o interesse em tratar
> cancer com argila e sucos deve ser investigado dentro de rigorosos
> procedimentos de verificaçào, e não receitado em livros de procedencia prá
> lá de duvidosa (eu diria criminosa mesmo).
>
> [E]> eu não tô defendendo o livro... mas sim a verdade (o livro vende!)
>
> Um tratamento avançado, com medicina de ponta, se não cura 100% de uma
> doença (e nada cura ou afirma curar), tem o mérito de ter sido testado com
> niveis de segurança e produzido resultados, comprováveis, muitas vezes maior
> que as alternativas (não tratar ou tratar com medicina alternativa). E pode
> apresentar suas evidencias disso.
>
> [E]> Medicina é artigo de luxo para a elite. Deixemos que a cangalha ao
> menos tenha seus placebos (que remédio vc receitaria pros indiozinhos que
> tão morrendo de desnutrição?)
>
> Ainda que tenha críticas a revista Veja,
>
> [E]> não tenho nenhuma birra a priori com a veja
>
> como de resto sobre a maioria das revistas de entretenimento em geral, e
> mesmo a mídia como um todo, a ação da revista neste caso foi legítma e digna
> de ser aplaudida.
>
> [E]> putz... (mentira nunca é legítima e digna de aplausos)
>
> Livrarias que comercializam o livro deveriam disponibilizar o artigo para
> seus clientes, como forma de prevenir o gasto desnecessário e o risco que
> este tipo de "literatura" e de "medicina alternativa" pode causar.
>
> [E]> que se responsabilize o autor pelas consequências...
>
> []s
>
> Eurico
>
>
> _\|/_
> __________________________________________________
> Converse com seus amigos em tempo real com o Yahoo! Messenger
> http://br.download.yahoo.com/messenger/
>
> [As partes desta mensagem que não continham texto foram removidas]
>
>
>
> ##### ##### #####
>
> Para saber mais visite
> http://www.ciencialist.hpg.ig.com.br
>
>
> ##### ##### ##### #####
>
>
> Yahoo! Grupos, um serviço oferecido por:
> PUBLICIDADE
>
> Yahoo! Grupos, um serviço oferecido por:
> PUBLICIDADE
>
>
> ________________________________
> Links do Yahoo! Grupos
> Para visitar o site do seu grupo na web, acesse:
> http://br.groups.yahoo.com/group/ciencialist/
>
> Para sair deste grupo, envie um e-mail para:
> ciencialist-unsubscribe@yahoogrupos.com.br
>
> O uso que você faz do Yahoo! Grupos está sujeito aos Termos do Serviço do
> Yahoo!.


SUBJECT: Re: Brilho metálico
FROM: Hélio Ricardo Carvalho <hrc@fis.puc-rio.br>
TO: ciencialist@yahoogrupos.com.br
DATE: 28/02/2005 18:30


--- Em ciencialist@yahoogrupos.com.br, "rmtakata" <rmtakata@a...>
escreveu

> Isso apenas se considerarmos a explicacao quantica (como a de
Fermi)
> de q. cada particula viaja por todos os caminhos...

Não estou falando de esoterismo ainda!!! :-)
Esta "medicina alternativa" funciona mas como diz Alberto não se
sabe o porque. Minha pretensão é aprender este porque.

A luz emitida por cada elétron viaja por todos os caminhos, a
quantidade é enorme (não me pergunte quanto ainda -- Isto é só
qualitativo) mas a parte desta luz que chega ao olho são aqueles
caminhos dentro do cone que termina na pupila do observador.
Veja com cuidado minha msg 44596.
Este cone é a parte que lhe cabe neste latifúndio. Deixando o cone
dos outros para os outros. :-)


> Mesmo com uma luz normal, nao ha' espalhamento, alem do q.
ocorreria
> se a luz percorresse um caminho de igual comprimento sem reflexao,
ao
> se refletir no espelho.

Com "luz normal" você quer dizer não laser?
Há espalhamento durante todo o caminho pois existe ar. Note: não
estou falando de Huyghens (onde não era necessário o ar), estou
falando que a luz que atravessou um pequeno caminho de ar já não é
mais a mesma que saiu da fonte. Cada átomo absorve e depois de um
tempo reemite em todas as direções. As perdas ocorridas no espelho
não são tão maiores que as que já ocorrem no ar.
E agora, só para fundir seus semicundutores [:-)], para mim tudo
isto é matéria (ondas de matéria).

>
> A reflexao especular eh organizada, por isso forma imagens. Do
> contrario seria um borrao branco.
>

Usando uma linguagem que para mim é um pouco imprópria mas que é
normalmente usada:
Um "fóton" não formará imagem. Para se ter imagem é necessário que a
luz deste ponto passe por uma região (e não um só ponto) do espelho
antes de chegar ao olho do observador.

Vou procurar referências porém você que é bom nisto.

[ ]'s
Hélio






SUBJECT: RE: [ciencialist] Relatividade e relógios de luz
FROM: "Hugo Santos" <urano@netvisao.pt>
TO: <ciencialist@yahoogrupos.com.br>
DATE: 28/02/2005 18:48

Caro Ricardo,

Deixe-me desde já agradecer-lhe pela elucidativa resposta.

Tal como a minha intuição me indicava, os relógios só batem certo a cada
duas batidas (como você muito bem comprovou matematicamente).

Ora, aqui temos então um problema de simultaneidade de eventos. Para um
observador no interior do comboio, os dois relógios batem certos em todas as
batidas, mas para um observador externo, os relógios só batem certo de duas
em duas batidas. Ora, isto parece-me muito estranho. Compreendo a matemática
que apresentou, mas como é que poderemos justificar que a ordem dos
acontecimentos, vistos por cada observador, é diferente?

Veja-se, se os observadores quiserem usar um dos relógios-luz, que se
encontram dentro do comboio, como padrão para medir a hora de um certo
acontecimento que se passe também dentro do comboio (por exemplo: hora a que
uma gota de água cai no chão), qual é o relógio que eles devem usar? Se
assumirmos que a hora é expressa em número de batidas do fotão nos espelhos
desde um certo tempo inicial, então os observadores só vão estar de acordo
se usarem o relógio perpendicular ao movimento ou então se usarem qualquer
um dos relógios mas o evento acontecer exactamente no momento da tal
"segunda-batida" em que os relógios estão de acordo.

Acho este aspecto da relatividade simplesmente fascinante. Acho que a
maioria de nós discute a relatividade como se fosse uma mera brincadeira
matemática, mas é extraordinário quando se assume que o nosso mundo é mesmo
assim (ou algo parecido, pelo menos)!

Mais uma vez, obrigado pela resposta,

Um abraço,

Hugo Santos


> -----Original Message-----
> From: Ricardo Soares Vieira [mailto:rickrsv@yahoo.com.br]
> Sent: segunda-feira, 28 de Fevereiro de 2005 21:24
> To: ciencialist@yahoogrupos.com.br
> Subject: [ciencialist] Relatividade e relógios de luz
>
>
>
> Olá a todos do ciencialist,
>
> Me interessei pelas mensagens que discutiram a experiência
> do "relógio de luz", utilizados para deduzir a dilatação do tempo da
> relatividade de uma forma fácil. Nestas mensagens, não me recordo
> quais, perguntaram o que ocorreria se a luz fosse lançada na direção
> do movimento da caixa; isso eu já tinha me perguntado também e
> cheguei a seguinte conclusão depois de fazer umas continhas...
>
> Vou discutir aqui o que eu encontrei:
>
> 1. Relógio-Luz na perpendicular ao movimento:
>
> Suponham que numa caixa quadrada de lados L alguém meça o tempo por
> raios de luz: primeiro ele lança um raio na direção vertical Y. O
> tempo gasto para a luz ir até o teto será
>
> t = L/c.
>
> Mas para alguém que vê a caixa se movimentar na direção X com
> velocidade v, a luz percorrerá um trajeto diagonal d, cujos catetos
> serão respectivamente Y e v.t', onde t' é o tempo que queremos
> encontrar (que não diremos nada a priori) assim, podemos escrever
> por Pitágoras:
>
> d'^2 = L^2 + (vt')^2
>
> Mas sendo d' = c.t', e c.t = L, e assumindo que é constante a
> velocidade da luz, temos:
>
> (ct')^2 = (c.t)^2 + (vt')^2
>
> (c^2 - v^2)t´^2 = (ct)^2
>
> t'= t/(1 - v^2/c^2)^1/2
>
> que é a equação da relatividade que conhecemos, do qual concluimos
> que um relógio na vertical, visto em movimento, é mais lento...
>
>
> 2. Relógio-Luz na direção do movimento:
>
>
> Mas o qual resultado encontraria este observador se o cientista da
> caixa lançasse a luz na direção X?
>
> Neste caso, o tempo de ida será diferente do de volta, obviamente
> pois que no primeiro caso a luz caminhará no mesmo sentido da caixa,
> e no segundo, no sentido oposto.
>
> Antes de mais nada, temos que levar em consideração que o
> comprimento da caixa na direção X estará contraída via Lorentz, pela
> equação:
>
> L" = L . (1 - v^2/c^2)^1/2
>
> Assim, na ida, teremos a luz partindo do ponto zero, e o extremo da
> caixa que parte do ponto L ". O tempo de ida corresponde ao tempo do
> encontro da luz com esse extremo da caixa, logo temos:
>
> ct' = L .[(1 - v^2/c^2)^1/2] + vt'
>
> Mas sendo L = c.t, temos:
>
> (c-v)t' =ct .[(1 - v^2/c^2)^1/2]
>
> logo,
>
> t' = [t . (1 - v^2/c^2)^1/2]/(1 - v/c)
>
> Analogamente, na volta, teremos:
>
> t' = [t . (1 - v^2/c^2)^1/2]/(1 + v/c)
>
> Assim, o tempo total (T' = 2t'), ou seja, a soma do tempo da ida com
> o da volta, depois de fazer os cálculos, vamos encontrar que:
>
> t'= t/(1 - v^2/c^2)^1/2
>
> Ou seja, igual ao anterior, onde o relógio-lus estava na vertical.
> Assim vemos que, ao considerar o relógio na horizontal como um tique
> e taque completo, ele funcionaria mais lento tal qual a
> relatividade...
>
> Ora considere que a nossa caixa seja esferica e espelhada por
> dentro, uma luz emitida do seu centro atingirá a esfera
> simultaneamente, e em determinado instante se encontrará novamente
> no ponto de orígem. Mas quando vista em movimento, teremos uma
> elipse, a luz atingirá primeiramente a parte de tras, depois o teto
> e por ultimo a frente, mas ao voltar, se encontrar em um determinado
> ponto de uma só vez...
>
> Essa é conclusão que chegamos pela relativiade habitual. Eu estou
> desenvolvendo outra (se me permitem), na qual considero que a altura
> dos corpos é diminuida, nessa teoria minha, é importante além do
> movimento, o fato se os corpos se aproximam ou se afastam do
> observador. O problema aqui seria resolvido de forma muito mais
> simples...
>
> Até mais, espero ter explicado mais que complicado...
>
> Rick
>
>
>
>
>
>
> ##### ##### #####
>
> Para saber mais visite
> http://www.ciencialist.hpg.ig.com.br
>
>
> ##### ##### ##### #####
> Links do Yahoo! Grupos
>
>
>
>
>
>
>




SUBJECT: Re: Brilho metálico
FROM: Hélio Ricardo Carvalho <hrc@fis.puc-rio.br>
TO: ciencialist@yahoogrupos.com.br
DATE: 28/02/2005 18:52



.........
>
> Vou procurar referências porém você que é bom nisto.

Achei uma razoavel. Apesar de ser muito Huyghens para o meu gosto
mas nào tenho agora tempo para uma melhor.

http://tinyurl.com/4hbvk


[ ]'s
Hélio















>
> [ ]'s
> Hélio





SUBJECT: Re: Relatividade e relógios de luz
FROM: "Ricardo Soares Vieira" <rickrsv@yahoo.com.br>
TO: ciencialist@yahoogrupos.com.br
DATE: 28/02/2005 18:58




Realmente caro Hugo, é mesmo estranho... Não dá para ter uma noção
correta de tempo dessa forma. Será que Deus (com todo respeito) não
tinha tomado umas antes de construir o universo?...



--- Em ciencialist@yahoogrupos.com.br, "Hugo Santos" <urano@n...>
escreveu
> Caro Ricardo,
>
> Deixe-me desde já agradecer-lhe pela elucidativa resposta.
>
> Tal como a minha intuição me indicava, os relógios só batem certo
a cada
> duas batidas (como você muito bem comprovou matematicamente).
>
> Ora, aqui temos então um problema de simultaneidade de eventos.
Para um
> observador no interior do comboio, os dois relógios batem certos
em todas as
> batidas, mas para um observador externo, os relógios só batem
certo de duas
> em duas batidas. Ora, isto parece-me muito estranho. Compreendo a
matemática
> que apresentou, mas como é que poderemos justificar que a ordem dos
> acontecimentos, vistos por cada observador, é diferente?
>
> Veja-se, se os observadores quiserem usar um dos relógios-luz, que
se
> encontram dentro do comboio, como padrão para medir a hora de um
certo
> acontecimento que se passe também dentro do comboio (por exemplo:
hora a que
> uma gota de água cai no chão), qual é o relógio que eles devem
usar? Se
> assumirmos que a hora é expressa em número de batidas do fotão nos
espelhos
> desde um certo tempo inicial, então os observadores só vão estar
de acordo
> se usarem o relógio perpendicular ao movimento ou então se usarem
qualquer
> um dos relógios mas o evento acontecer exactamente no momento da
tal
> "segunda-batida" em que os relógios estão de acordo.
>
> Acho este aspecto da relatividade simplesmente fascinante. Acho
que a
> maioria de nós discute a relatividade como se fosse uma mera
brincadeira
> matemática, mas é extraordinário quando se assume que o nosso
mundo é mesmo
> assim (ou algo parecido, pelo menos)!
>
> Mais uma vez, obrigado pela resposta,
>
> Um abraço,
>
> Hugo Santos
>
>
> > -----Original Message-----
> > From: Ricardo Soares Vieira [mailto:rickrsv@y...]
> > Sent: segunda-feira, 28 de Fevereiro de 2005 21:24
> > To: ciencialist@yahoogrupos.com.br
> > Subject: [ciencialist] Relatividade e relógios de luz
> >
> >
> >
> > Olá a todos do ciencialist,
> >
> > Me interessei pelas mensagens que discutiram a experiência
> > do "relógio de luz", utilizados para deduzir a dilatação do
tempo da
> > relatividade de uma forma fácil. Nestas mensagens, não me recordo
> > quais, perguntaram o que ocorreria se a luz fosse lançada na
direção
> > do movimento da caixa; isso eu já tinha me perguntado também e
> > cheguei a seguinte conclusão depois de fazer umas continhas...
> >
> > Vou discutir aqui o que eu encontrei:
> >
> > 1. Relógio-Luz na perpendicular ao movimento:
> >
> > Suponham que numa caixa quadrada de lados L alguém meça o tempo
por
> > raios de luz: primeiro ele lança um raio na direção vertical Y. O
> > tempo gasto para a luz ir até o teto será
> >
> > t = L/c.
> >
> > Mas para alguém que vê a caixa se movimentar na direção X com
> > velocidade v, a luz percorrerá um trajeto diagonal d, cujos
catetos
> > serão respectivamente Y e v.t', onde t' é o tempo que queremos
> > encontrar (que não diremos nada a priori) assim, podemos escrever
> > por Pitágoras:
> >
> > d'^2 = L^2 + (vt')^2
> >
> > Mas sendo d' = c.t', e c.t = L, e assumindo que é constante a
> > velocidade da luz, temos:
> >
> > (ct')^2 = (c.t)^2 + (vt')^2
> >
> > (c^2 - v^2)t´^2 = (ct)^2
> >
> > t'= t/(1 - v^2/c^2)^1/2
> >
> > que é a equação da relatividade que conhecemos, do qual
concluimos
> > que um relógio na vertical, visto em movimento, é mais lento...
> >
> >
> > 2. Relógio-Luz na direção do movimento:
> >
> >
> > Mas o qual resultado encontraria este observador se o cientista
da
> > caixa lançasse a luz na direção X?
> >
> > Neste caso, o tempo de ida será diferente do de volta, obviamente
> > pois que no primeiro caso a luz caminhará no mesmo sentido da
caixa,
> > e no segundo, no sentido oposto.
> >
> > Antes de mais nada, temos que levar em consideração que o
> > comprimento da caixa na direção X estará contraída via Lorentz,
pela
> > equação:
> >
> > L" = L . (1 - v^2/c^2)^1/2
> >
> > Assim, na ida, teremos a luz partindo do ponto zero, e o extremo
da
> > caixa que parte do ponto L ". O tempo de ida corresponde ao
tempo do
> > encontro da luz com esse extremo da caixa, logo temos:
> >
> > ct' = L .[(1 - v^2/c^2)^1/2] + vt'
> >
> > Mas sendo L = c.t, temos:
> >
> > (c-v)t' =ct .[(1 - v^2/c^2)^1/2]
> >
> > logo,
> >
> > t' = [t . (1 - v^2/c^2)^1/2]/(1 - v/c)
> >
> > Analogamente, na volta, teremos:
> >
> > t' = [t . (1 - v^2/c^2)^1/2]/(1 + v/c)
> >
> > Assim, o tempo total (T' = 2t'), ou seja, a soma do tempo da ida
com
> > o da volta, depois de fazer os cálculos, vamos encontrar que:
> >
> > t'= t/(1 - v^2/c^2)^1/2
> >
> > Ou seja, igual ao anterior, onde o relógio-lus estava na
vertical.
> > Assim vemos que, ao considerar o relógio na horizontal como um
tique
> > e taque completo, ele funcionaria mais lento tal qual a
> > relatividade...
> >
> > Ora considere que a nossa caixa seja esferica e espelhada por
> > dentro, uma luz emitida do seu centro atingirá a esfera
> > simultaneamente, e em determinado instante se encontrará
novamente
> > no ponto de orígem. Mas quando vista em movimento, teremos uma
> > elipse, a luz atingirá primeiramente a parte de tras, depois o
teto
> > e por ultimo a frente, mas ao voltar, se encontrar em um
determinado
> > ponto de uma só vez...
> >
> > Essa é conclusão que chegamos pela relativiade habitual. Eu estou
> > desenvolvendo outra (se me permitem), na qual considero que a
altura
> > dos corpos é diminuida, nessa teoria minha, é importante além do
> > movimento, o fato se os corpos se aproximam ou se afastam do
> > observador. O problema aqui seria resolvido de forma muito mais
> > simples...
> >
> > Até mais, espero ter explicado mais que complicado...
> >
> > Rick
> >
> >
> >
> >
> >
> >
> > ##### ##### #####
> >
> > Para saber mais visite
> > http://www.ciencialist.hpg.ig.com.br
> >
> >
> > ##### ##### ##### #####
> > Links do Yahoo! Grupos
> >
> >
> >
> >
> >
> >
> >





SUBJECT: arquivo arrumado velocidade da luz
FROM: ricardo soares vieira <rickrsv@yahoo.com.br>
TO: pergunte a um fisico <fisico@if.usp.br>, Luiz Barco <nuap@usp.br>, CBPF <iva@cbpf.br>, ciencialist <ciencialist@yahoogrupos.com.br>, spinquark fisica grupo <spinquarks@yahoogrupos.com.br>, Itamar Itxe <itamarap@terra.com.br>, leandro@e-silt.com.br, leandro ramos fagundes <leandro@blink.com.br>, rick <rickrhrc@yahoo.com.br>, "Jo�o" Paulo Zulian <jpzulian@hotmail.com>
DATE: 28/02/2005 19:00

O arquivo que enviei estava com problemas de visualização. aqui vai o "arrumado".




Ricardo Soares Vieira.

rickrsv@yahoo.com.br



---------------------------------
Yahoo! Messenger 6.0 - jogos, emoticons sonoros e muita diversão. Instale agora!

[As partes desta mensagem que não continham texto foram removidas]



SUBJECT: AULA Sobre REFRIGERANTES
FROM: "Cyberlander" <mrcyberlander@i12.com>
TO: <Undisclosed-Recipient:;>
DATE: 28/02/2005 19:20



Repassando ..........
--------------------------------------------------------------------------------



AULA Sobre REFRIGERANTES

Na verdade, a fórmula "secreta" da Coca-Cola (CC) se desvenda em 18 segundos em
qualquer espectrômetro-ótico, e basicamente até os cachorros a conhecem. Só que
não dá para fabricar igual, a não ser que você tenha uns 10 bilhões de dólares
para brigar com a CC na justiça, porque eles vão cair
matando.

A fórmula da Pepsi tem uma diferença básica da CC e é proposital
exatamente para evitar processo judicial. Não é diferente porque não conseguiram
fazer igual não, é de propósito, mas próximo o suficiente para atrair o consumidor
da CC que quer um gostinho diferente com menos sal e açúcar.

Entre outras coisas, fui eu quem teve que aprender tudo sobre refrigerante
gaseificado para produzir o guaraná Golly aqui, que usa o concentrado Brahma. Está
no mercado até hoje, mas falhou terrivelmente em estratégia promocional e vende
só para o mercado local, tudo isso devido à cabeça dura
de alguns diretores.

Tive que aprender química, entender tudo sobre componentes de
refrigerantes, conservantes, sais, ácidos, cafeína, enlatamento,produção de
label de lata, permissões, aprovações e muito etc. e tal. Montei um
mini-laboratório de análise de produto, equipamento até para analisar quantidade
de sólidos, etc. Até desenvolvi programas para PC para cálculo da fórmula com base
nos volumes e tipo de envasamento (plástico ou alumínio), pois isso muda os
valores e o sabor. Tivemos até equipe de competição em stock- car ...

Tire a imensa quantidade de sal que a CC usa (50mg de sódio na lata) e você
verá que a CC fica igualzinha a qualquer outro refrigerante sem-vergonha e
porcaria, adocicado e enjoado.

É exatamente o Cloreto de Sódio em exagero (que eles dizem ser "very low sodium")
que refresca e ao mesmo tempo dá sede em dobro, pedindo outro refrigerante, e não
enjoa porque o tal sal mata literalmente a
sensibilidade ao doce, que também tem de montão: 39 gramas de "açúcar" (sacarose).

É ridículo, dos 350 gramas de produto líquido, mais de 10% é açúcar. Imagine numa
lata de CC, mais de 1 centímetro e meio da lata é açúcar puro... isso dá
aproximadamente umas 3 colheres de sopa CHEIAS DE AÇÚCAR POR LATA !...

- Fórmula da COCA-COLA ?...

Simples: Concentrado de Açúcar queimado - Caramelo - para dar cor escura e gosto;
ácido ortofosfórico (azedinho); sacarose - açúcar (HFCS- High Fructose Corn
Syrup - açúcar líquido da frutose do milho); extrato da folha
da planta COCA (África e Índia) e poucos outros aromatizantes naturais de outras
plantas, cafeína, e conservante que pode ser Benzoato de Sódio ou Benzoato de
Potássio, Dióxido de carbono de montão para fritar a língua quando você a toma e
junto com o sal dar a sensação de refrigeração.

O uso de ácido ortofosfórico e não o ácido cítrico como todos os outros usam, é
para dar a sensação de dentes e boca limpa ao beber, o fosfórico literalmente
frita tudo e em quantidade pode até causar decapamento do esmalte dos dentes,
coisa que o cítrico ataca com muito menor violência, pois o artofosfórico"chupa"
todo o cálcio do organismo, podendo causar até osteoporose, sem contar o
comprometimento na formação dos ossos e dentes das crianças em idade de formação
óssea, dos 2 aos 14 anos. Tente comprar ácido fosfórico para ver as mil
recomendações de segurança e manuseio (queima o cristalino do olho, queima a pele,
etc.).

Só como informação geral, é proibido usar ácido fosfórico em qualquer outro
refrigerante, só a CC tem permissão... (claro, se tirar, a CC ficará com gosto de
sabão).

O extrato da coca e outras folhas quase não mudam nada no sabor, é mais efeito
cosmético e mercadológico, assim como o guaraná, você não sente o gosto dele, nem
cheiro, (o verdadeiro guaraná tem gosto amargo) ele está lá
até porque legalmente tem que estar (qeustão de registro comercial), mas se
tirar você nem nota diferença no gosto.

O gosto é dado basicamente pelas quantidades diferentes de açúcar, açúcar
queimado, sais, ácidos e conservantes. Tem uma empresa química aqui em Bartow, sul
de Orlando. Já visitei os caras inúmeras vezes e eles
basicamente produzem aromatizantes e essências para sucos.

Sais concentrados e essências o dia inteiro, caminhão atrás de caminhão! Eles
produzem isso para fábricas de sorvete, refrigerantes, sucos, enlatados, até
comida colorida e aromatizada.

Visitando a fábrica, pedi para ver o depósito de concentrados das frutas, que
deveria ser imenso, cheio de reservatórios imensos de laranja, abacaxi,
morango, e tantos outros (comentei)... O sujeito olhou para mim, deu uma risadinha
e me levou para visitar os depósitos imensos de corantes e mais de 50 tipos de
componentes químicos. O refrigerante de laranja, o que menos
tem é laranja; morango, até os gominhos que ficam em suspensão são feitos de goma
(uma liga química que envolve um semi-polímero). Abacaxi é um festival de ácidos e
mais goma. Essência para sorvete de Abacate? Usam até peróxido de hidrogênio (água
oxigenada) para dar aquela sensação de arrasto espumoso no céu da boca ao comer,
típico do abacate.

O segundo refrigerante mais vendido aqui nos Estados Unidos é o Dr. Pepper, o mais
antigo de todos, mais antigo que a própria CC. Esse refrigerante era vendido
obviamente sem refrigeração e sem gaseificação em mil oitocentos e pedrada, em
garrafinhas com rolha como medicamento, nas carroças ambulantes que você vê em
filmes do velho oeste americano. Além de tirar dor de barriga e unha encravada,
também tirava mancha de ferrugem de cortina, além de ajudar a renovar a graxa dos
eixos das carroças. Para quem não sabe, Dr. Pepper tem um sabor horrível, e é muito
fácil de experimentar em casa: pegue GELOL spray, aquele que você usa quando leva
um chute na canela, e dê um bom spray na boca! Esse é o gosto do tal famoso
Dr.Pepper que vende muito por aqui.

- Refrigerante DIET

Quer saber a quantidade de lixo que tem em refrigerante diet? Não uso nem para
desentupir a pia, porque tenho pena da tubulação de pvc...

Olha, só para abrir os olhos dos cegos : os produtos adocicantes diet têm vida
muito curta.

O aspartame, por exemplo, após 3 semanas de molhado passa a ter gosto de pano
velho sujo.

Para evitar isso, soma-se uma infinidade de outros químicos, um para esticar a
vida do aspartame, outro para dar buffer (arredondar) o gosto do segundo químico,
outro para neutralizar a cor dos dois químicos juntos que deixam o líquido turvo,
outro para manter o terceiro químico em suspensão, senão o fundo do refrigerante
fica escuro, outro para evitar cristalização do aspartame, outro para realçar, dar
"edge" no ácido cítrico ou fosfórico que acaba sofrendo pela influência dos 4
produtos químicos iniciais, e assim vai... a lista é enorme.

Depois de toda essa minha experiência com produção e estudo de
refrigerantes, posso afirmar:

Sabe qual é o melhor refrigerante?

Água filtrada, de preferência duplamente filtrada, laranja ou limão espremido e
gelo, mais nada, nem açúcar nem sal.

(AUTOR: ANÔNIMO - por motivos óbvios)



CYBERLANDER

Ama a realidade que constróis,
que nem a morte deterá teu voo · ·




[As partes desta mensagem que não continham texto foram removidas]



SUBJECT: Re: Fwd: Artigo sobre velocidade da luz nos diferentes meios ópticos
FROM: "Ricardo Soares Vieira" <rickrsv@yahoo.com.br>
TO: ciencialist@yahoogrupos.com.br
DATE: 28/02/2005 19:56


Olá Maria Natália

Então, eu tenho um texto pronto mas não sei se tem como enviar ao
grupo porque ele está em pdf...

Se alguem puder me explicar como, eu envio com prazer ok.

Tchau, até mais.

--- Em ciencialist@yahoogrupos.com.br, Maria Natália <grasdic@h...>
escreveu
>
> Ricardo:
> Mas para se ler tem de se associar e não dá Já estou em 20
listas...
> Não podes escrever aqui, copiar pedindo autorização?
> Aguardo
> Maria Natália
>
>
> --- Em ciencialist@yahoogrupos.com.br, ricardo soares vieira
> <rickrsv@y...> escreveu
> > Olá a todos.
> >
> > A algum tempo, em um grupo de discussão sobre física
> (http://br.groups.yahoo.com/group/spinquarks/), eu tinha lançado a
> hipótese de que A LUZ SE PROPAGA COM A MESMA VELOCIDADE NOS
DIFERENTES
> MEIOS ÓPTICOS, mas fui um pouco criticado por isso (obviamente,
seria
> o mesmo que um louco chegar e dizer que o universo tem a forma de
uma maçã),(...)





SUBJECT: Novo arquivo carregado em ciencialist
FROM: ciencialist@yahoogrupos.com.br
TO: ciencialist@yahoogrupos.com.br
DATE: 28/02/2005 20:08



Olá,

Esta mensagem é uma notificação para informá-lo que um novo arquivo foi adicionado no grupo ciencialist.

Arquivo : /velocidade da luz.pdf
Carregado por : rickrsv <rickrsv@yahoo.com.br>
Descrição :

Você pode acessar o arquivo pela URL:

http://br.groups.yahoo.com/group/ciencialist/files/velocidade%20da%20luz.pdf

Para saber mais sobre compartilhamento de arquivos no grupo, leia:

http://help.yahoo.com/help/br/groups/files

Atenciosamente,

rickrsv <rickrsv@yahoo.com.br>









SUBJECT: Re: RN: Veja denuncia "medicina natural de A a Z" como farsa
FROM: André Roviralta Dias Baptista <andrediasbaptista@yahoo.com.br>
TO: ciencialist@yahoogrupos.com.br
DATE: 28/02/2005 21:36


--- Em ciencialist@yahoogrupos.com.br, Victor Pimentel Nunes
<victor2002@g...> escreveu
> Na Seção "Os Mais Vendidos" deveria ter uma nota também falando da
> ausência do livro no "ranking(???)"
>
> Aquilo não é um guia porcaria nenhuma. Aquilo é simplesmente
(deveria
> ser) a lista de livros mais vendidos nas livrarias mencionadas.
>
> Tratar uma lista de mais vendidos (o que é uma análise
QUANTITATIVA)
> como guia (o que no caso sugere uma análise qualitativa) parece
> história para boi dormir.
>

Isso é correto. A "Veja" não pode misturar as bolas. Estatística de
venda é estatística, ponto. O que ela pode é criar uma quarta seção
para enquadrar esse tipo de livro. Além das
categorias "Ficção", "Não-Ficção" e "Auto-ajuda", pode criar a
seção "pseudo-ciência e charlatanismo" ou "auto-engano".

[]´s

André





SUBJECT: A Física é a igreja católica da Ciência
FROM: Maria Natália <grasdic@hotmail.com>
TO: ciencialist@yahoogrupos.com.br
DATE: 28/02/2005 22:23


Tá dito!
Pois é. Não sabiam?
Vem no livro: " As calças de Pitágoras, Deus, a Física e a guerra dos
sexos" de Margaret Wertherm e claro é da GRADIVA.
Alguém já leu? Encontrei-o à venda na feira do livro de minha escola e
não resisti a provocar-vos.
Quem o ler primeiro põe aqui o seu comentério?
Uma abraço





SUBJECT: Re: AULA Sobre REFRIGERANTES
FROM: Maria Natália <grasdic@hotmail.com>
TO: ciencialist@yahoogrupos.com.br
DATE: 28/02/2005 22:28


Cyber:

E não dá para colocares lá no local? Ou queres que ponha eu?
Pode ser como anedota.
Agradeço
Um abraço (verm?)
Maria Natália


--- Em ciencialist@yahoogrupos.com.br, "Cyberlander"
<mrcyberlander@i...> escreveu
>
>
> Repassando ..........
>
--------------------------------------------------------------------------------
>
>
>
> AULA Sobre REFRIGERANTES
>
> Na verdade, a fórmula "secreta" da Coca-Cola (CC) se desvenda em
18 segundos em
> qualquer espectrômetro-ótico, e basicamente até os cachorros a
conhecem. Só que
> não dá para fabricar igual, a não ser que você tenha uns 10 bilhões
de dólares
> para brigar com a CC na justiça, porque eles vão cair
> matando.
>
> A fórmula da Pepsi tem uma diferença básica da CC e é proposital
> exatamente para evitar processo judicial. Não é diferente porque não
conseguiram
> fazer igual não, é de propósito, mas próximo o suficiente para
atrair o consumidor
> da CC que quer um gostinho diferente com menos sal e açúcar.
>
> Entre outras coisas, fui eu quem teve que aprender tudo sobre
refrigerante
> gaseificado para produzir o guaraná Golly aqui, que usa o
concentrado Brahma. Está
> no mercado até hoje, mas falhou terrivelmente em estratégia
promocional e vende
> só para o mercado local, tudo isso devido à cabeça dura
> de alguns diretores.
>
> Tive que aprender química, entender tudo sobre componentes de
> refrigerantes, conservantes, sais, ácidos, cafeína,
enlatamento,produção de
> label de lata, permissões, aprovações e muito etc. e tal. Montei um
> mini-laboratório de análise de produto, equipamento até para
analisar quantidade
> de sólidos, etc. Até desenvolvi programas para PC para cálculo da
fórmula com base
> nos volumes e tipo de envasamento (plástico ou alumínio), pois isso
muda os
> valores e o sabor. Tivemos até equipe de competição em stock- car ...
>





SUBJECT: Re: [ciencialist] RN: Veja denuncia "medicina natural de A a Z" como farsa
FROM: "Oraculo" <oraculo@atibaia.com.br>
TO: <ciencialist@yahoogrupos.com.br>
DATE: 28/02/2005 23:26

Oi Amaury

Bem, relatos do tipo " eu fiz e deu certo" ou "alguém que conheço fez e deu certo" não embasam conclusões confiáveis. Há uma enorme gama de motivos para ter "dado certo" com sua mãe, que não a simpatia alternativa escolhida.

A quase totalidade das doenças e dificuldades de saúde regridem naturalmente. É uma piada recorrente entre médicos que se você tiver uma gripe e tomar homeopatia estará curado em uma semana, mas se não tomar, vai levar 7 dias para sarar..:-)

Outro problema é o termo "natural". O que considera "natural"? Curare é natural, comigo-ninguém-pode é natural, diversos venenos poderosos são naturais, se por natural quiser dizer sem a modificação ou intervenção humana (ou mesmo se quiser dizer vindo de plantas). Diversas plantas são perigosas, e mandioca selvagem mata em algumas horas, se comer sua raiz.

Se alguém sugerir que uma determinada simpatia cura resfriado, e você se curar, depois de cumprir a determinação, isso não significa que a primeira tem relação com a segunda. Por exemplo, suponha que, antes de entrar na lotérica e apostar, você de uma topada. Se ganhar na MegaSena, vai relacionar a topada como causa de ter ganho? Claro que não..:-)

Da mesma forma, se fizer algo, e depois de algum tempo se curar ou se sentir melhor, isso não indica relação entre causa e efeito. Apenas relatos estatisticos confiaveis, no lugar de anedóticos, dão alguma confiabilidade sobre isso.

Se acompanhar 100 pessoas com cancer, 5% delas vai se curar expontaneamente, e 95% delas vai morrer, em alguns anos. Não precisa fazer nada, nenhum tratamento, a taxa de regressão é de 5% (variando em alguns tipos de cancer). Mas, se usar tratamentos medicos atuais em 100 outras pessoas com cancer, entre 85 e 95% destas vai se curar, e entre 15 e 5% vai morrer em alguns anos. O que isso diz a você, sobre causa e efeito nesse caso?

A única forma de saber se sua mãe se curou devido ao tratamento alternativo, ou se se curaria de toda forma, seria um estudo estatistico que acompanhasse diversas pessoas com o mesmo problema, o mesmo tratamento, e comparar as curas. Suponha que, de 100 pessoas com o mesmo problema que sua mãe, 15% se curem sozinhas em 2 meses. Imagine que em um estudo controlado, com outras 100 pessoas com o mesmo problema, o tratamento alternativo cure, digamos, 15% delas em dois meses. Poderemos dizer que é o tratamento que está fazendo efeito nestes casos, sua mãe incluida, ou ele é inócuo?

Quando um tratamento alternativo é submetido a esse tipo de estudo de rigor, ele em geral falha. E apenas por isso não é aceito, e continua alternativo. Pensa que se um tratamento alternativo, por exemplo curar cancer com suco de babosa, pudesse demonstrar que cura mais que a taxa de regressão natural da doença, seria mantido "alternativo"? Ou seria incorporado no arsenal médico para tratar canceres?

Quando tratamentos alternativos reclamam da "medicina tradicional" ou do FDA americano, que impede que sejam prescritos por médicos ou vendidos a população, eles estão na verdade tentando escapar do rigor de apresentar estudos dentro desses parametros. Se concordassem a se submeter a esse tipo de exigencia, que as vezes leva anos de estudos, poderiam demonstrar que são eficazes e que tem efeito real. Mas, e o risco de falhar?..:-)

Agora pense em algo sério, uma doença real e perigosa, por exemplo, tuberculose. Ficaria tranquilo se seu filho com tuberculose demorasse a ir ao médico "tradicional" e, no lugar, apenas tomasse os "remedios alternativos" recomendados por um livro desses? Dormiria tranquilo à noite, mesmo sabendo que essas receitas foram criadas por charlatões, que não tem a menor noção do que falam, não fizeram nenhum estudo e baseiam suas receitas em coisas como "fulano fez e ficou melhor"?

Acho que não..:-)

Um abraço.

Homero








----- Original Message -----
From: Amauri Nolasco Sanches Jr
To: ciencialist@yahoogrupos.com.br
Sent: Monday, February 28, 2005 5:42 PM
Subject: Re: [ciencialist] RN: Veja denuncia "medicina natural de A a Z" como farsa


Oi Oraculo

Minha mãe tem e fez e deu certo, como uma coisa
natural pode fazer mal? Será que não é uma
armação dos laboratórios?

Amauri




--- Oraculo <oraculo@atibaia.com.br> escreveu:

---------------------------------
Olá Eurico

A retirada de um livro claramente fraudulento, com
recomendações que colocam em perigo a vida de quem o
consulta, dificilmente se enquadraria na categoria de
censura. Está mais para ação de responsabilidade
social, que deveria ser seguida por todos os meios de
comunicação, mesmo os que lucram com determinadas
maluquices como medicinas alternativas.

O interesse da população por medicina alternativa (que
em geral significa apenas medicina sem comprovação de
eficácia) não se dá devido ao "monopolio" da medicina
tradicional, mas sim devido a pouca informação do
público em geral sobre mecanismos de ação, metabolismo
e biologia, segurança de tratamento, aliada a
curiosidade natural de seres humanos, busca por
alternativas à falta de acesso a medicina tradicional
(que qualquer pessoa preferiria, se fosse possível o
acesso universal a esta), e diversos outros fatores.

Motoristas de terceiro mundo (e alguns no primeiro
também) tem grande atração por comportamentos de
risco, como dirigir a altas velocidades e ignorar leis
de transito, e é dever tanto do estado quanto dos
meios de comunicação lutar contra essa atitude. Da
mesma forma, o interesse em tratar cancer com argila e
sucos deve ser investigado dentro de rigorosos
procedimentos de verificaçào, e não receitado em
livros de procedencia prá lá de duvidosa (eu diria
criminosa mesmo).

Um tratamento avançado, com medicina de ponta, se não
cura 100% de uma doença (e nada cura ou afirma curar),
tem o mérito de ter sido testado com niveis de
segurança e produzido resultados, comprováveis, muitas
vezes maior que as alternativas (não tratar ou tratar
com medicina alternativa). E pode apresentar suas
evidencias disso.

Ainda que tenha críticas a revista Veja, como de resto
sobre a maioria das revistas de entretenimento em
geral, e mesmo a mídia como um todo, a ação da revista
neste caso foi legítma e digna de ser aplaudida.
Livrarias que comercializam o livro deveriam
disponibilizar o artigo para seus clientes, como forma
de prevenir o gasto desnecessário e o risco que este
tipo de "literatura" e de "medicina alternativa" pode
causar.

Um abraço.

Homero







----- Original Message -----
From: Eurico Ferreira de Souza Jr.
To: ciencialist@yahoogrupos.com.br
Sent: Monday, February 28, 2005 5:10 PM
Subject: Re: [ciencialist] RN: Veja denuncia
"medicina natural de A a Z" como farsa


[E]> a veja vai censurar a lista dos mais vendidos?
vai ter que mudar o nome para "lista dos livros mais
vendidos indicados pela veja". Não se combate mentira
com mentira...

Percebam o interesse da população por qualquer
"medicina alternativa". É indicativo do
descontentamento com o monopólio da medicina.


_\|/_

---------------------------------
Yahoo! Acesso Grátis - Internet rápida e grátis.
Instale o discador do Yahoo! agora.

[As partes desta mensagem que não continham texto
foram removidas]



##### ##### #####

Para saber mais visite
http://www.ciencialist.hpg.ig.com.br


##### ##### ##### #####


Yahoo! Grupos, um serviço oferecido por:







------------------------------------------------------------------------------
Links do Yahoo! Grupos

a.. Para visitar o site do seu grupo na web,
acesse:
http://br.groups.yahoo.com/group/ciencialist/

b.. Para sair deste grupo, envie um e-mail para:
ciencialist-unsubscribe@yahoogrupos.com.br

c.. O uso que você faz do Yahoo! Grupos está
sujeito aos Termos do Serviço do Yahoo!.



[As partes desta mensagem que não continham texto
foram removidas]



##### ##### #####

Para saber mais visite
http://www.ciencialist.hpg.ig.com.br


##### ##### ##### #####


Yahoo! Grupos, um serviço oferecido por:

function SearchComboBox() {
if (document.form_combo.keyword.value.length==0){
alert("Por favor, digite algo."); return false; }else
{ document.form_combo.action
="http://br.rd.yahoo.com/SIG=12anq1f40/M=264105.3931087.6562589.1588051/D=brclubs/S=2137111528:HM/EXP=1109708879/A=2361264/R=0/SIG=11uaou2jn/*http://www.bondfaro.com/bondfaro/in/combosearch_in.jsp?sk=11";
} return true;}
[input]
[input] [input]


---------------------------------
Links do Yahoo! Grupos

Para visitar o site do seu grupo na web, acesse:
http://br.groups.yahoo.com/group/ciencialist/

Para sair deste grupo, envie um e-mail para:
ciencialist-unsubscribe@yahoogrupos.com.br

O uso que você faz do Yahoo! Grupos está sujeito
aos Termos do Serviço do Yahoo!.


=====

Meu grupo sobre sexualidade: http://br.groups.yahoo.com/group/Sexualidadedodef/?yguid=94014452

" O dia mais importante não é quando conhecemos as pessoas mas sim o momento em que elas passam a existir dentro de nós. "

MSN: amaurijunior3@hotmail.com

icq:153134120

Blog:http://blogdomaster.weblogger.terra.com.br/index.htm





















_______________________________________________________
Yahoo! Acesso Grátis - Instale o discador do Yahoo! agora. http://br.acesso.yahoo.com/ - Internet rápida e grátis


##### ##### #####

Para saber mais visite
http://www.ciencialist.hpg.ig.com.br


##### ##### ##### #####


Yahoo! Grupos, um serviço oferecido por:
PUBLICIDADE




------------------------------------------------------------------------------
Links do Yahoo! Grupos

a.. Para visitar o site do seu grupo na web, acesse:
http://br.groups.yahoo.com/group/ciencialist/

b.. Para sair deste grupo, envie um e-mail para:
ciencialist-unsubscribe@yahoogrupos.com.br

c.. O uso que você faz do Yahoo! Grupos está sujeito aos Termos do Serviço do Yahoo!.



[As partes desta mensagem que não continham texto foram removidas]



SUBJECT: Re: Fw: Porque o oxigênio é invisivel ?
FROM: Maria Natália <grasdic@hotmail.com>
TO: ciencialist@yahoogrupos.com.br
DATE: 01/03/2005 01:27


Mas não será mais simples apanhar um astrofísico distraído e
perguntar-le como detecta oxigénio num objecto cósmico? Usamos as
ocultações para sabermos se um dado objecto tem atmosférica e qual o
tipo de constuinte por espectroscopia. Takata tem razão.
Isso de ser transparente é relativo e depende do observador. Não sei
se o oxigénio seria transparente num planeta que estivesse em
translação em relação a uma outra estrela....
E já agora porque é que o hidrogénio e o azoto são incolores? O azoto
até parece estranho...aflige ser incolor...Se ele é o maior
constituinte da nossa atmosfera.
E o fluor já teria cor? A molécula é maior do que a do o oxigénio.
E a ligação na molécula de azoto é igual á do átomo de oxigénio? É que
são ambos transparentes...
Não será que há mais perguntas dificeis para atrapalhar físico-químico?
Isto é mesmo difícil...
E as partículas com sabores?
A alma tem cor? E os fantasmas?
E á água salgada quando se mete remo na água de noite tem brilho
esverdeado?
Porque a água do mar muda de cor?
Vou-me
Maria Natália


--- Em ciencialist@yahoogrupos.com.br, TARCISIO BORGES <tbs97@f...>
escreveu
> Mas quem disse que oxigênio é invisível?
>
> Pegue um tubo linear com uns 200km de comprimento cheio de O2. Use um
> feixe de luz branca em uma abertura e veja o que sai na outra...
>
> []s
> TARCISIO BORGES
> tbs97@f...
>
> On Mon, 28 Feb 2005, Luiz Ferraz Netto wrote:
> > Que tal essa pergunta?
> > Afinal, bem mais simples do que a cor do elétron!
> > ==========================
> > Luiz Ferraz Netto [Léo]
> > -----Mensagem Original-----
> > Olá professor existe alguma explicação cientifica
> > para a invisibilidade do oxigênio?
> >
> > r Antunes. Magé - Rj





SUBJECT: Re: [ciencialist] A Física é a igreja católica da Ciência
FROM: "Ivan Carlos" <icarlos@icarlos.net>
TO: <ciencialist@yahoogrupos.com.br>
DATE: 01/03/2005 01:58

a pequena grande diferença é que a física provamos, mostramos e repetimos os
fatos quantas vezes forem necessárias a quem quizer ver

...já a crença... só crendo mesmo rsss

Ivan "Doomer" Carlos
Social Engineering Specialist
-
Cell.: +55 (11) 8112-0666
icarlos@icarlos.net
www.icarlos.net
-
MSN: icarlos@icarlos.net
Y!M / Skype: ivandoomer
--------------------------------------------------


----- Original Message -----
From: "Maria Natália" <grasdic@hotmail.com>
To: <ciencialist@yahoogrupos.com.br>
Sent: Monday, February 28, 2005 10:23 PM
Subject: [ciencialist] A Física é a igreja católica da Ciência




Tá dito!
Pois é. Não sabiam?
Vem no livro: " As calças de Pitágoras, Deus, a Física e a guerra dos
sexos" de Margaret Wertherm e claro é da GRADIVA.
Alguém já leu? Encontrei-o à venda na feira do livro de minha escola e
não resisti a provocar-vos.
Quem o ler primeiro põe aqui o seu comentério?
Uma abraço





##### ##### #####

Para saber mais visite
http://www.ciencialist.hpg.ig.com.br


##### ##### ##### #####
Links do Yahoo! Grupos











SUBJECT: Professores nos EUA tomam posição: relatório em Ciência e Matemática
FROM: Maria Natália <grasdic@hotmail.com>
TO: ciencialist@yahoogrupos.com.br
DATE: 01/03/2005 02:51


Foi aqui falado pelo Homero sobre os problemas que se põem à Ciência
nos EUA e que podiam contagiar o Brasil. Nem mais: me chegou da NSTA a
seguinte notícia:
""If America is to sustain its international competitiveness, its
national security, and the quality of life for its citizens, then it
must move quickly to achieve significant improvements in the
participation of all students in mathematics and science," begins a
report issued on February 16 by the Business-Higher Education Forum
titled A Commitment to America's Future: Responding to the Crisis in
Mathematics and Science Education. The report calls for business,
higher education, and policy leaders to organize and implement a
nationwide plan that addresses the quality of the mathematics and
science education provided to all students, "in collaboration with
classroom teachers and school administrators and taking advantage of
the promising work they have already initiated." Read more at
http://science.nsta.org/nstaexpress/nstaexpress_2005_02_22_bhef.htm. ""

Vamos lá a ver o que eles pensam fazer.
Maria Natália







SUBJECT: Química de colóides e de superfície explica...
FROM: Maria Natália <grasdic@hotmail.com>
TO: ciencialist@yahoogrupos.com.br
DATE: 01/03/2005 02:57


o crescimento de bactérias de acordo com experiências realizadas no
departamento de Química na Universidade de Florença em Itália:

"Relatively simple phenomena in physical, colloid and surface
chemistry play a role in the growth of bacteria according to new
experiments. The results also suggest that a process called the
Hofmeister effect could influence the growth of cells (P Lo Nostro et
al. 2005 Phys. Biol. 2 1)".
http://physicsweb.org/article/news/9/2/16""
Maria Natália








SUBJECT: RES: RN: Veja denuncia "medicina natural de A a Z" como farsa
FROM: "Anderson Porto" <acp722003@tudosobreplantas.com.br>
TO: <ciencialist@yahoogrupos.com.br>
CC: <tudosobreplantas@yahoogrupos.com.br>
DATE: 01/03/2005 02:59

Olá Homero,


Faz tempo que não participo da lista ciencialist, mas não poderia deixar
de comentar umas coisas que li. Concordo em quase tudo que diz, mas...


- O FDA americano não é exemplo de verdade científica, pois já foi
dominado por empresas fabricantes de drogas e agrotóxicos, vulgos
remédios e venenos; Concorda?


- Eu não sabia que as informações nutricionais impressas em nossos
mantimentos é de acordo com o padrão americano, você sabia?


- Medicina natural, medicina alternativa, medicina holística, medicina
tradicional... São termos que não significam a mesma coisa, apesar de
invariavelmente jornalistas ou pessoas mal informadas usarem como se
fossem;


- A ciência procura informações empíricas para se orientar e há bastante
tempo. Através da etnobotânica eles descobrem determinados usos de
espécies de uso popular e depois pesquisam se são realmente eficazes;


- A população procura "medicina alternativa" porque os chás funcionam,
as compressas, o banho de assento, as pomadas... Funcionam! Enquanto a
"oficial" está cheia de médicos mal formados, mal estruturados, com
professores mal remunerados, mal humorados... E MUUUUUUITO CARA!


Já descobriram diversas, milhares de espécies com potencial para
pesquisa. Entretanto, estas custam caro e, por causa de patentes, as
informações raramente chegam ao público leigo.

Como você mesmo disse, às vezes a doença passa em 7 dias, mesmo sem o
doente ter ido buscar orientação em consultas médicas, comprado remédios
e repousado.

Ao meu ver, o correto é ir contra o objeto "causa única", da medicina de
faculdades, que chamo de reducionista. O médico, no meu entender, deve
procurar as causas psicológicas, o fator gerador de um estado de doença.
Daí, em vez de recomendar cirurgias, quimioterapias e sei lá mais quais
agressões, onde invariavelmente ganha $$$ o médico anestesista (por
fora), o convênio, o hospital, o médico cirurgião ou quimioterapeuta, o
instrumentador... Procurar agredir o mínimo possível um corpo já
debilitado... E o bolso.

É claro que dentre diversas soluções e, numa situação de falta de tempo
para fazer este tipo de levantamento do paciente, o médico deverá
recomendar aquilo que não possa ser processado depois. Se existe uam
recomendação "oficial" ele deve seguir, pois senão será criticado pelos
colegas de profissão. A decisão sobre qual caminho seguir, depois de bem
orientado, deve ser sempre do paciente.

Existem plantas que precisam ser pesquisadas e principalmente fazer com
que estes dados cheguem ao público leigo de forma idônea. A Aveloz
(Euphorbia tirucalli) é uma delas.

Estas são minhas opiniões. Não sou médico e gostaria muito de ouvir a
verdade "oficial".



Deixo uma pergunta para vocês, se puderem, me responderem:


- Como fazer para diminuir o custo de uma pesquisa científica?


Tudo de bom,

Anderson Porto
http://www.tudosobreplantas.com.br


-----Mensagem original-----
De: Oraculo [mailto:oraculo@atibaia.com.br]
Enviada em: segunda-feira, 28 de fevereiro de 2005 23:27
Para: ciencialist@yahoogrupos.com.br
Assunto: Re: [ciencialist] RN: Veja denuncia "medicina natural de A a Z"
como farsa



Oi Amaury

Bem, relatos do tipo " eu fiz e deu certo" ou "alguém que conheço fez e
deu certo" não embasam conclusões confiáveis. Há uma enorme gama de
motivos para ter "dado certo" com sua mãe, que não a simpatia
alternativa escolhida.

A quase totalidade das doenças e dificuldades de saúde regridem
naturalmente. É uma piada recorrente entre médicos que se você tiver uma
gripe e tomar homeopatia estará curado em uma semana, mas se não tomar,
vai levar 7 dias para sarar..:-)

Outro problema é o termo "natural". O que considera "natural"? Curare é
natural, comigo-ninguém-pode é natural, diversos venenos poderosos são
naturais, se por natural quiser dizer sem a modificação ou intervenção
humana (ou mesmo se quiser dizer vindo de plantas). Diversas plantas
são perigosas, e mandioca selvagem mata em algumas horas, se comer sua
raiz.

Se alguém sugerir que uma determinada simpatia cura resfriado, e você se
curar, depois de cumprir a determinação, isso não significa que a
primeira tem relação com a segunda. Por exemplo, suponha que, antes de
entrar na lotérica e apostar, você de uma topada. Se ganhar na MegaSena,
vai relacionar a topada como causa de ter ganho? Claro que não..:-)

Da mesma forma, se fizer algo, e depois de algum tempo se curar ou se
sentir melhor, isso não indica relação entre causa e efeito. Apenas
relatos estatisticos confiaveis, no lugar de anedóticos, dão alguma
confiabilidade sobre isso.

Se acompanhar 100 pessoas com cancer, 5% delas vai se curar
expontaneamente, e 95% delas vai morrer, em alguns anos. Não precisa
fazer nada, nenhum tratamento, a taxa de regressão é de 5% (variando em
alguns tipos de cancer). Mas, se usar tratamentos medicos atuais em 100
outras pessoas com cancer, entre 85 e 95% destas vai se curar, e entre
15 e 5% vai morrer em alguns anos. O que isso diz a você, sobre causa e
efeito nesse caso?

A única forma de saber se sua mãe se curou devido ao tratamento
alternativo, ou se se curaria de toda forma, seria um estudo estatistico
que acompanhasse diversas pessoas com o mesmo problema, o mesmo
tratamento, e comparar as curas. Suponha que, de 100 pessoas com o mesmo
problema que sua mãe, 15% se curem sozinhas em 2 meses. Imagine que em
um estudo controlado, com outras 100 pessoas com o mesmo problema, o
tratamento alternativo cure, digamos, 15% delas em dois meses. Poderemos
dizer que é o tratamento que está fazendo efeito nestes casos, sua mãe
incluida, ou ele é inócuo?

Quando um tratamento alternativo é submetido a esse tipo de estudo de
rigor, ele em geral falha. E apenas por isso não é aceito, e continua
alternativo. Pensa que se um tratamento alternativo, por exemplo curar
cancer com suco de babosa, pudesse demonstrar que cura mais que a taxa
de regressão natural da doença, seria mantido "alternativo"? Ou seria
incorporado no arsenal médico para tratar canceres?

Quando tratamentos alternativos reclamam da "medicina tradicional" ou
do FDA americano, que impede que sejam prescritos por médicos ou
vendidos a população, eles estão na verdade tentando escapar do rigor de
apresentar estudos dentro desses parametros. Se concordassem a se
submeter a esse tipo de exigencia, que as vezes leva anos de estudos,
poderiam demonstrar que são eficazes e que tem efeito real. Mas, e o
risco de falhar?..:-)

Agora pense em algo sério, uma doença real e perigosa, por exemplo,
tuberculose. Ficaria tranquilo se seu filho com tuberculose demorasse a
ir ao médico "tradicional" e, no lugar, apenas tomasse os "remedios
alternativos" recomendados por um livro desses? Dormiria tranquilo à
noite, mesmo sabendo que essas receitas foram criadas por charlatões,
que não tem a menor noção do que falam, não fizeram nenhum estudo e
baseiam suas receitas em coisas como "fulano fez e ficou melhor"?

Acho que não..:-)

Um abraço.

Homero








----- Original Message -----
From: Amauri Nolasco Sanches Jr
To: ciencialist@yahoogrupos.com.br
Sent: Monday, February 28, 2005 5:42 PM
Subject: Re: [ciencialist] RN: Veja denuncia "medicina natural de A a
Z" como farsa


Oi Oraculo

Minha mãe tem e fez e deu certo, como uma coisa
natural pode fazer mal? Será que não é uma
armação dos laboratórios?

Amauri




--- Oraculo <oraculo@atibaia.com.br> escreveu:

---------------------------------
Olá Eurico

A retirada de um livro claramente fraudulento, com
recomendações que colocam em perigo a vida de quem o
consulta, dificilmente se enquadraria na categoria de
censura. Está mais para ação de responsabilidade
social, que deveria ser seguida por todos os meios de
comunicação, mesmo os que lucram com determinadas
maluquices como medicinas alternativas.

O interesse da população por medicina alternativa (que
em geral significa apenas medicina sem comprovação de
eficácia) não se dá devido ao "monopolio" da medicina
tradicional, mas sim devido a pouca informação do
público em geral sobre mecanismos de ação, metabolismo
e biologia, segurança de tratamento, aliada a
curiosidade natural de seres humanos, busca por
alternativas à falta de acesso a medicina tradicional
(que qualquer pessoa preferiria, se fosse possível o
acesso universal a esta), e diversos outros fatores.

Motoristas de terceiro mundo (e alguns no primeiro
também) tem grande atração por comportamentos de
risco, como dirigir a altas velocidades e ignorar leis
de transito, e é dever tanto do estado quanto dos
meios de comunicação lutar contra essa atitude. Da
mesma forma, o interesse em tratar cancer com argila e
sucos deve ser investigado dentro de rigorosos
procedimentos de verificaçào, e não receitado em
livros de procedencia prá lá de duvidosa (eu diria
criminosa mesmo).

Um tratamento avançado, com medicina de ponta, se não
cura 100% de uma doença (e nada cura ou afirma curar),
tem o mérito de ter sido testado com niveis de
segurança e produzido resultados, comprováveis, muitas
vezes maior que as alternativas (não tratar ou tratar
com medicina alternativa). E pode apresentar suas
evidencias disso.

Ainda que tenha críticas a revista Veja, como de resto
sobre a maioria das revistas de entretenimento em
geral, e mesmo a mídia como um todo, a ação da revista
neste caso foi legítma e digna de ser aplaudida.
Livrarias que comercializam o livro deveriam
disponibilizar o artigo para seus clientes, como forma
de prevenir o gasto desnecessário e o risco que este
tipo de "literatura" e de "medicina alternativa" pode
causar.

Um abraço.

Homero







----- Original Message -----
From: Eurico Ferreira de Souza Jr.
To: ciencialist@yahoogrupos.com.br
Sent: Monday, February 28, 2005 5:10 PM
Subject: Re: [ciencialist] RN: Veja denuncia
"medicina natural de A a Z" como farsa


[E]> a veja vai censurar a lista dos mais vendidos?
vai ter que mudar o nome para "lista dos livros mais
vendidos indicados pela veja". Não se combate mentira
com mentira...

Percebam o interesse da população por qualquer
"medicina alternativa". É indicativo do
descontentamento com o monopólio da medicina.


_\|/_





SUBJECT: Regolamento di sicurezza, especial Emiliano Chemello
FROM: Maria Natália <grasdic@hotmail.com>
TO: ciencialist@yahoogrupos.com.br
DATE: 01/03/2005 03:00


Porque me pareces saber italiano aqui vai uma página especial do
departamento de química da Universidade de Floreça:
http://www.chim.unifi.it/
sds
Maria Natália





SUBJECT: Re: Brilho metálico
FROM: "rmtakata" <rmtakata@altavista.net>
TO: ciencialist@yahoogrupos.com.br
DATE: 01/03/2005 08:17


--- Em ciencialist@yahoogrupos.com.br, Hélio Ricardo Carvalho
> Não estou falando de esoterismo ainda!!! :-)
> Esta "medicina alternativa" funciona mas como diz Alberto não se
> sabe o porque. Minha pretensão é aprender este porque.

Essa eh a maior das pretensoes. (Eu considero ateh maior do q. a
hubris.) Mas, sim, funciona, porem diferentemente das "medicinas
alternativas" - foi testada e varias vezes.

Uma das explicacoes de por q. funciona, seria: porq. eh verdade ou tem
algo a ver com a realidade. Mas realmente nao sabemos se essa
explicacao esta' correta.

> A luz emitida por cada elétron viaja por todos os caminhos, a
> quantidade é enorme (não me pergunte quanto ainda -- Isto é só

Se pensarmos em termos de foton: o eletron aborve esse foton e o
"reemite" (ou emite outro foton correspondente) em uma direcao bem
especifica (em relacao 'a trajetoria original do eletron). Se vier
outro foton em outra direcao, entao o foton "reemitido" segue outra
direcao (mas tb bem especifica em relacao a essa direcao de incidencia).

> > Mesmo com uma luz normal, nao ha' espalhamento, alem do q.
> > ocorreria se a luz percorresse um caminho de igual comprimento sem
> > reflexao, ao se refletir no espelho.
>
> Com "luz normal" você quer dizer não laser?
> Há espalhamento durante todo o caminho pois existe ar. Note: não
> estou falando de Huyghens (onde não era necessário o ar), estou
> falando que a luz que atravessou um pequeno caminho de ar já não é
> mais a mesma que saiu da fonte.

Eu disse: *alem do [espalhamento] q. ocorreria se a luz percorresse um
caminho de igual comprimento sem reflexao* - a reflexao no espelho nao
aumenta o espalhamento.

> E agora, só para fundir seus semicundutores [:-)], para mim tudo
> isto é matéria (ondas de matéria).

Pode ser. Se considerarmos como fotons, por exemplo.

> > A reflexao especular eh organizada, por isso forma imagens. Do
> > contrario seria um borrao branco.
>
> Usando uma linguagem que para mim é um pouco imprópria mas que é
> normalmente usada:
> Um "fóton" não formará imagem. Para se ter imagem é necessário que a
> luz deste ponto passe por uma região (e não um só ponto) do espelho
> antes de chegar ao olho do observador.

Nao estou dizendo q. *um* foton formara' imagem. Estou dizendo q. a
reflexao organizada formara' a imagem (ou levara' 'a formacao da imagem).

[]s,

Roberto Takata





SUBJECT: Re: Relatividade e relógios de luz
FROM: Hélio Ricardo Carvalho <hrc@fis.puc-rio.br>
TO: ciencialist@yahoogrupos.com.br
DATE: 01/03/2005 09:52




"Ricardo Soares Vieira" escreveu
>
> Realmente caro Hugo, é mesmo estranho... Não dá para ter uma noção
> correta de tempo dessa forma. Será que Deus (com todo respeito)
não
> tinha tomado umas antes de construir o universo?...
>

Será que Einstein (com todo respeito) não tinha tomado umas antes de
construir esta teoria?...

[ ]'s
Hélio





SUBJECT: Re: Brilho metálico
FROM: "rmtakata" <rmtakata@altavista.net>
TO: ciencialist@yahoogrupos.com.br
DATE: 01/03/2005 10:19


--- Em ciencialist@yahoogrupos.com.br, Hélio Ricardo Carvalho
> Achei uma razoavel. Apesar de ser muito Huyghens para o meu gosto
> mas nào tenho agora tempo para uma melhor.
>
> http://tinyurl.com/4hbvk

Mas o q. ocorre com as ondas q. migram para a esquerda? Como elas
estao se anulando?

E se um feixe de luz eh espalhado em todas as direcoes, alem da
questao da formacao de imagem, como fica a conservacao da energia? A
energia incidente teria q. ser dividida entre os diversos feixes
refletidos.

[]s,

Roberto Takata





SUBJECT: Fw: LIVRO: A ALQUIMIA
FROM: "Luiz Ferraz Netto" <leobarretos@uol.com.br>
TO: "ciencialist" <ciencialist@yahoogrupos.com.br>
DATE: 01/03/2005 10:44

Quem tem o caminho das pedras para a aquisição do 'book'?
[]'
===========================
Luiz Ferraz Netto [Léo]
leobarretos@uol.com.br
http://www.feiradeciencias.com.br
===========================
-----Mensagem Original-----
De: Almeida, Paulo R **BRSPA
Para: leobarretos@uol.com.br
Enviada em: terça-feira, 1 de março de 2005 09:52
Assunto: LIVRO: A ALQUIMIA


Prof. Luiz, bom dia.



Gostaria de saber se conheces o livro A Alquimia ou O Alquimista, que conta a história de um alquimista. Eu tinha este livro mas por algum motivo ele se perdeu. É um livro originalmente em inglês traduzido para o português e a editora, se não me engano era a Hemus. Se tiver esta informação, favor passar-me para que possa adquiri-lo e lê-lo novamente.



Agradeço a atenção,



Engº Paulo Almeida

Technical Service - Arch Biocides

fone: 55 11 4028 8117

fax: 55 11 4028 8109

e-mail: pralmeida@archchemicals.com





--------------------------------------------------------------------------------


No virus found in this incoming message.
Checked by AVG Anti-Virus.
Version: 7.0.300 / Virus Database: 266.5.0 - Release Date: 25/02/2005

----------

No virus found in this outgoing message.
Checked by AVG Anti-Virus.
Version: 7.0.300 / Virus Database: 266.5.0 - Release Date: 25/02/2005


[As partes desta mensagem que não continham texto foram removidas]



SUBJECT: Fw: Oi professor !
FROM: "Luiz Ferraz Netto" <leobarretos@uol.com.br>
TO: "ciencialist" <ciencialist@yahoogrupos.com.br>
DATE: 01/03/2005 10:45

Traduzir em linguagem científica.

[]'
===========================
Luiz Ferraz Netto [Léo]
leobarretos@uol.com.br
http://www.feiradeciencias.com.br
===========================
-----Mensagem Original-----
De: Leonardo Capriles Moura
Para: leobarretos@uol.com.br
Enviada em: segunda-feira, 28 de fevereiro de 2005 23:56
Assunto: Oi professor !


Olá, meu nome é Leonardo C. Moura.

Tenho uma pergunta. Espero que possa me ajudar!

Estava no site Feira de Ciências, quando vi seu e-mail...rsrs:{O

Onde posso aprender, a relação entre o peso de uma pessoa ou objeto, quando esta está dentro da água?

Ou seja: existe uma formula?

Obs: Se um homem que pesa 100 kg entra na água (salgada ou doce), qual será o peso suportado por uma almofada flutuante de aeronave?

Diz-se que tal almofada boiará por 8 horas com um peso de 60 kg.

Espero que entenda o que lhe pergunto. Sei que é meio estranho e talvez confuso.

Abraço Saúde!



--------------------------------------------------------------------------------
Yahoo! Acesso Grátis - Internet rápida e grátis. Instale o discador do Yahoo! agora.


--------------------------------------------------------------------------------


No virus found in this incoming message.
Checked by AVG Anti-Virus.
Version: 7.0.300 / Virus Database: 266.5.0 - Release Date: 25/02/2005

----------

No virus found in this outgoing message.
Checked by AVG Anti-Virus.
Version: 7.0.300 / Virus Database: 266.5.0 - Release Date: 25/02/2005


[As partes desta mensagem que não continham texto foram removidas]



SUBJECT: Re: Fw: Oi professor !
FROM: "rmtakata" <rmtakata@altavista.net>
TO: ciencialist@yahoogrupos.com.br
DATE: 01/03/2005 11:02


Acho q. ele quer saber sobre o princípio de Arquimedes, o empuxo e a
flutuabilidade.

[]s,

Roberto Takata

--- Em ciencialist@yahoogrupos.com.br, "Luiz Ferraz Netto"
> Traduzir em linguagem científica.
> ===========================
> Luiz Ferraz Netto [Léo]
> -----Mensagem Original-----
> De: Leonardo Capriles Moura
> Onde posso aprender, a relação entre o peso de uma pessoa ou objeto,
> quando esta está dentro da água?
>
> Ou seja: existe uma formula?
>
> Obs: Se um homem que pesa 100 kg entra na água (salgada ou doce),
> qual será o peso suportado por uma almofada flutuante de aeronave?
>
> Diz-se que tal almofada boiará por 8 horas com um peso de 60 kg.
>
> Espero que entenda o que lhe pergunto. Sei que é meio estranho e
> talvez confuso.






SUBJECT: Re: [ciencialist] A Física é a igreja católica da Ciência
FROM: "Alvaro Augusto \(E\)" <alvaro@electraenergy.com.br>
TO: <ciencialist@yahoogrupos.com.br>
DATE: 01/03/2005 11:37

Hum, interessante... Em TODA a física? Isso inclui supertrings também?

E as ondas gravitacionais? E os monopólos magnéticos? São coisas compatíveis com as teorias mais modernas, mas fogem à verificação experimental...

[ ]s

Alvaro Augusto
----- Original Message -----
From: Ivan Carlos
To: ciencialist@yahoogrupos.com.br
Sent: Tuesday, March 01, 2005 1:58 AM
Subject: Re: [ciencialist] A Física é a igreja católica da Ciência


a pequena grande diferença é que a física provamos, mostramos e repetimos os
fatos quantas vezes forem necessárias a quem quizer ver

...já a crença... só crendo mesmo rsss

Ivan "Doomer" Carlos
Social Engineering Specialist
-
Cell.: +55 (11) 8112-0666
icarlos@icarlos.net
www.icarlos.net
-
MSN: icarlos@icarlos.net
Y!M / Skype: ivandoomer
--------------------------------------------------


----- Original Message -----
From: "Maria Natália" <grasdic@hotmail.com>
To: <ciencialist@yahoogrupos.com.br>
Sent: Monday, February 28, 2005 10:23 PM
Subject: [ciencialist] A Física é a igreja católica da Ciência




Tá dito!
Pois é. Não sabiam?
Vem no livro: " As calças de Pitágoras, Deus, a Física e a guerra dos
sexos" de Margaret Wertherm e claro é da GRADIVA.
Alguém já leu? Encontrei-o à venda na feira do livro de minha escola e
não resisti a provocar-vos.
Quem o ler primeiro põe aqui o seu comentério?
Uma abraço



[As partes desta mensagem que não continham texto foram removidas]



SUBJECT: Veja denuncia "medicina natural de A a Z" como farsa
FROM: "Silvio" <scordeiro@terra.com.br>
TO: <ciencialist@yahoogrupos.com.br>
DATE: 01/03/2005 12:02

Oráculo:

A Veja cria sério precedente: ou se censura tudo ou não se censura nada....
Assim, os donos na revista ficam sob a espada da Damocles: deverão, todas as
semanas julgar o que é farsa, mentira, !má leitura" e caem inexoravelmente
numa ditadura intelectual.
Se a lista é de "mais vendidos", não se discute o mérito do livro. Basta
trocar o título da lista: "Livros que julgados edificantes e comprovadamente
qualificados para a leitura das famílias de bons costumes"
Será que esse livro contém mais impropriedades que os do paulo coelho?

A censura é sempre perniciosa, injustificável, digna de cristãos medievais
e, no caso presente, por certo estimulada por lobby dos laboratórios....

sds.,
silvio.
"não há efeito sem causa"
-----Mensagem Original-----
De: "Oraculo" <oraculo@atibaia.com.br>
Para: <ciencialist@yahoogrupos.com.br>
Enviada em: segunda-feira, 28 de fevereiro de 2005 17:27
Assunto: Re: [ciencialist] RN: Veja denuncia "medicina natural de A a Z"
como farsa



Olá Eurico

A retirada de um livro claramente fraudulento, com recomendações que colocam
em perigo a vida de quem o consulta, dificilmente se enquadraria na
categoria de censura. Está mais para ação de responsabilidade social, que
deveria ser seguida por todos os meios de comunicação, mesmo os que lucram
com determinadas maluquices como medicinas alternativas.

O interesse da população por medicina alternativa (que em geral significa
apenas medicina sem comprovação de eficácia) não se dá devido ao "monopolio"
da medicina tradicional, mas sim devido a pouca informação do público em
geral sobre mecanismos de ação, metabolismo e biologia, segurança de
tratamento, aliada a curiosidade natural de seres humanos, busca por
alternativas à falta de acesso a medicina tradicional (que qualquer pessoa
preferiria, se fosse possível o acesso universal a esta), e diversos outros
fatores.

Motoristas de terceiro mundo (e alguns no primeiro também) tem grande
atração por comportamentos de risco, como dirigir a altas velocidades e
ignorar leis de transito, e é dever tanto do estado quanto dos meios de
comunicação lutar contra essa atitude. Da mesma forma, o interesse em tratar
cancer com argila e sucos deve ser investigado dentro de rigorosos
procedimentos de verificaçào, e não receitado em livros de procedencia prá
lá de duvidosa (eu diria criminosa mesmo).

Um tratamento avançado, com medicina de ponta, se não cura 100% de uma
doença (e nada cura ou afirma curar), tem o mérito de ter sido testado com
niveis de segurança e produzido resultados, comprováveis, muitas vezes maior
que as alternativas (não tratar ou tratar com medicina alternativa). E pode
apresentar suas evidencias disso.

Ainda que tenha críticas a revista Veja, como de resto sobre a maioria das
revistas de entretenimento em geral, e mesmo a mídia como um todo, a ação da
revista neste caso foi legítma e digna de ser aplaudida. Livrarias que
comercializam o livro deveriam disponibilizar o artigo para seus clientes,
como forma de prevenir o gasto desnecessário e o risco que este tipo de
"literatura" e de "medicina alternativa" pode causar.

Um abraço.

Homero







----- Original Message -----
From: Eurico Ferreira de Souza Jr.
To: ciencialist@yahoogrupos.com.br
Sent: Monday, February 28, 2005 5:10 PM
Subject: Re: [ciencialist] RN: Veja denuncia "medicina natural de A a Z"
como farsa


[E]> a veja vai censurar a lista dos mais vendidos? vai ter que mudar o
nome para "lista dos livros mais vendidos indicados pela veja". Não se
combate mentira com mentira...

Percebam o interesse da população por qualquer "medicina alternativa". É
indicativo do descontentamento com o monopólio da medicina.


_\|/_

---------------------------------
Yahoo! Acesso Grátis - Internet rápida e grátis. Instale o discador do
Yahoo! agora.

[As partes desta mensagem que não continham texto foram removidas]



##### ##### #####

Para saber mais visite
http://www.ciencialist.hpg.ig.com.br


##### ##### ##### #####


Yahoo! Grupos, um serviço oferecido por:







------------------------------------------------------------------------------
Links do Yahoo! Grupos

a.. Para visitar o site do seu grupo na web, acesse:
http://br.groups.yahoo.com/group/ciencialist/

b.. Para sair deste grupo, envie um e-mail para:
ciencialist-unsubscribe@yahoogrupos.com.br

c.. O uso que você faz do Yahoo! Grupos está sujeito aos Termos do
Serviço do Yahoo!.



[As partes desta mensagem que não continham texto foram removidas]



##### ##### #####

Para saber mais visite
http://www.ciencialist.hpg.ig.com.br


##### ##### ##### #####
Links do Yahoo! Grupos












SUBJECT: P/ Anderson (era Veja denuncia "medicina natural de A a Z" como farsa)
FROM: "Oraculo" <oraculo@atibaia.com.br>
TO: <ciencialist@yahoogrupos.com.br>
DATE: 01/03/2005 14:49

Olá Anderson

Anderson: - O FDA americano não é exemplo de verdade científica, pois já foi
dominado por empresas fabricantes de drogas e agrotóxicos, vulgos
remédios e venenos; Concorda?

Mais ou menos..:-) Há um forte "vies" antiamericano (ou anti-organizações) nessa colocação, já que pressupõe que toda organização pode ser dominada por forças ocultas, em geral, do capitalismo malvado..:-) Ser "dominado" é muito parecido com as Grandes Conspirações Para Esconder A Verdade, que, em geral, são irreais.

Claro que qualquer orgão administrativo, em especial os que determinam controle de grandes somas de dinheiro, podem ser eventualmente alvo de pressões e mesmo de alguma corrupção. É da natureza humana, e da natureza das organizações humanas.

Mas, para determinar que a totalidade de qualquer sistema é falho,é necessário muitas evidencias sólidas, e o FDA ainda se mantém relativamente integro em sua maior parte. Por exemplo, ainda é necesário, mesmo para grandes fabricantes, apresentar estudos de longo prazo, ainda é preciso licença especial para testes em humanos, ainda é efetuado rigoroso controle de casos, etc. O que não impede que sempre escape algo, como Viox, e outros. Nem impede que o próprio governo, principalmettne o Bush, distorça o foco, direcionando a preocupação do sistema de controle mais para drogas ilícitas que a segurança da populaçào.

Entretanto, a solução destes desvios passa, não por ignorar o FDA, mas aumentar sua eficácia, reverter as posições politicas e ideologicas e aumentar seus mecanismos de controle e segurança, principalmente a estrutura cientifica. E, se fosse possível, criar um órgão tão eficaz quanto o FDA no Brasil, para depender menos do americano..:-)

Anderson: - Eu não sabia que as informações nutricionais impressas em nossos
mantimentos é de acordo com o padrão americano, você sabia?"

Sim, mas ainda é melhor que nada..:-) Se nossa tradicional incompetencia de terceiro mundo não nos permite fazer nossos próprios estudos (as universidades seriam excelente local de pesquisa, não acha?..:-), pelo menos alguma informação nutricional está disponível.

Anderson: - Medicina natural, medicina alternativa, medicina holística, medicina
tradicional... São termos que não significam a mesma coisa, apesar de
invariavelmente jornalistas ou pessoas mal informadas usarem como se
fossem;"

Sim, o problema de definir os termos antes de um debete ou alegação é sempre trazido a tona. É preciso definir, para comunicar. Em todo caso, se jornalistas ou pessoas mal informadas usam esses termos, como vamos separar o correto, de incorreto, que ferramenta nos dará um grau de confiabilidade nesse processo? Exatamettne por isso que a ciência, e sua ferramenta mais confiável o método cientifico, são usados. Não importa o termo que um jornalista ou pessoa mal informada use, se o processo em sí funciona, o método irá demonstrar isso.

Anderson: - A ciência procura informações empíricas para se orientar e há bastante
tempo. Através da etnobotânica eles descobrem determinados usos de
espécies de uso popular e depois pesquisam se são realmente eficazes;"

Claro que sim. Boa parte dos conhecimentos sobre medicamentos se iniciou com o estudo de drogas presentes em plantas. Mas, uma vez detectado o elemento, a molécula, refina-la, descobrir seu mecanismo de ação, determinar dosagem ideal, separar elementos daninhos ou com efeitos colaterais, etc, é parte importante do processo de aumentar tanto a eficácia quanto a segurança desses usos. Nenhum cientista nega isso, que espécies de uso popular podem mostrar indicações importantes, mas é preciso separa-las das que nada fazem, são placebos ou francamente perigosas.

Anderson: - A população procura "medicina alternativa" porque os chás funcionam,
as compressas, o banho de assento, as pomadas... Funcionam! Enquanto a
"oficial" está cheia de médicos mal formados, mal estruturados, com
professores mal remunerados, mal humorados... E MUUUUUUITO CARA! "

Sim, chás funcionam, para a maioria das doenças que, de outra forma, sarariam sozinhas. Eliminam desconfortos que, de outra forma, seriam suportados pelas pessoas, mas sarariam também. Tomar um chá de erva cidreira é excelente, eu adoro, mas não cura nem cancer nem tuberculose. Analise a espectativa de vida de nossos antepassados, tratados apenas com chás e banhos de assento, com as suas. Deve haver um motivo para que aqueles tivessem a espectativa de vida de 30 e poucos anos (ou menos) e você mais de 80.

O motivo principal é que, embora seja possivel viver com desconfortos, doenças graves e sérias só podem ser efetivamente curadas com medicina de ponta, conhecimento cientifico real, e drogas mais eficazes.

Pense em uma apendicite. Antes da cirurgia e antibióticos, matava 100%. Depois, pode ser curada 100%. Tomaria chá ou ervas para isso?

Anderson, não sou contra chás. Não são medicina alternativa, são medicina tradicional, bem conhecida e aceita por médicos tradicionais. Acupuntura, urinoterapia, homeopatia, iridologia, florais de bach, etc, é que são medicina alternativa, sem evidencias e sem confiabilidade.

Anderson: Ao meu ver, o correto é ir contra o objeto "causa única", da medicina de
faculdades, que chamo de reducionista. "

Concordo..:-) Mas essa é uma caricatura do medico e da medicina vendida pelos alternativos..:-) Homeopatas devem conversar com o paciente e verificar o conjunto, nào apenas a doença. Mas é exatamente o que um bom médico faz, ou deve fazer. Existem maus médicos? Claro..:-) Existem mau homeopatas? Claro..:-) Chamar a medicina atual de reducionista é uma acusação falsa. Juntas médicas existem justamente para analisar todas as possibilidades, em geral gerenciadas por um clínico geral.

A massificação da medicina, em especial na seguridade social, pode realmente ter criado um grave problema de atendimento, de falta de cuidado, de maus médicos, com jornadas de trabalho enormes, sem tempo para pacientes. Mas não é uma falha da medicina ou da ciência, mas da administração, dos governos. Vá a um bom médico, particular, de sua confiança, e será atendido como se deve, com tempo, uma boa conversa, analise de seus dados, sua vida, etc. Ele não vai, claro, analisar sua "aura", mas fará o que for preciso para ajuda-lo..:-)

Anderson: - Como fazer para diminuir o custo de uma pesquisa científica?

Esta questão, embora tenha resposta simples, não é simples de ser implnatada, já que depende da compreensão do problema por quem governa o pais: investir na educação, desde escolas de base, até universidades. A pesquisa em universidades é a base de uma forte industria baseada na ciência e para medicina de ponta. Universidades que podem construir equipamentos, e sustentar pesquisadores, alunos e professores, minimizam o custo desse conhecimento de forma espantosa. Alunos pesquisam pelo prazer de descobrir, criam sem restrições de qualquer ordem (por exemplo, não temem ter sua carreira prejudicada se pesquisarem um assunto controverso ou pouco aceito), estão no auge da criatividade e vigor, além de muitas outras vantagens.

Agora, implantar essa solução simples, é outro (enorme) problema..:-)

Um abraço.

Homero



----- Original Message -----
From: Anderson Porto
To: ciencialist@yahoogrupos.com.br
Cc: tudosobreplantas@yahoogrupos.com.br
Sent: Tuesday, March 01, 2005 2:59 AM
Subject: [ciencialist] RES: RN: Veja denuncia "medicina natural de A a Z" como farsa


Olá Homero,


Faz tempo que não participo da lista ciencialist, mas não poderia deixar
de comentar umas coisas que li. Concordo em quase tudo que diz, mas...


- O FDA americano não é exemplo de verdade científica, pois já foi
dominado por empresas fabricantes de drogas e agrotóxicos, vulgos
remédios e venenos; Concorda?


- Eu não sabia que as informações nutricionais impressas em nossos
mantimentos é de acordo com o padrão americano, você sabia?


- Medicina natural, medicina alternativa, medicina holística, medicina
tradicional... São termos que não significam a mesma coisa, apesar de
invariavelmente jornalistas ou pessoas mal informadas usarem como se
fossem;


- A ciência procura informações empíricas para se orientar e há bastante
tempo. Através da etnobotânica eles descobrem determinados usos de
espécies de uso popular e depois pesquisam se são realmente eficazes;


- A população procura "medicina alternativa" porque os chás funcionam,
as compressas, o banho de assento, as pomadas... Funcionam! Enquanto a
"oficial" está cheia de médicos mal formados, mal estruturados, com
professores mal remunerados, mal humorados... E MUUUUUUITO CARA!


Já descobriram diversas, milhares de espécies com potencial para
pesquisa. Entretanto, estas custam caro e, por causa de patentes, as
informações raramente chegam ao público leigo.

Como você mesmo disse, às vezes a doença passa em 7 dias, mesmo sem o
doente ter ido buscar orientação em consultas médicas, comprado remédios
e repousado.

Ao meu ver, o correto é ir contra o objeto "causa única", da medicina de
faculdades, que chamo de reducionista. O médico, no meu entender, deve
procurar as causas psicológicas, o fator gerador de um estado de doença.
Daí, em vez de recomendar cirurgias, quimioterapias e sei lá mais quais
agressões, onde invariavelmente ganha $$$ o médico anestesista (por
fora), o convênio, o hospital, o médico cirurgião ou quimioterapeuta, o
instrumentador... Procurar agredir o mínimo possível um corpo já
debilitado... E o bolso.

É claro que dentre diversas soluções e, numa situação de falta de tempo
para fazer este tipo de levantamento do paciente, o médico deverá
recomendar aquilo que não possa ser processado depois. Se existe uam
recomendação "oficial" ele deve seguir, pois senão será criticado pelos
colegas de profissão. A decisão sobre qual caminho seguir, depois de bem
orientado, deve ser sempre do paciente.

Existem plantas que precisam ser pesquisadas e principalmente fazer com
que estes dados cheguem ao público leigo de forma idônea. A Aveloz
(Euphorbia tirucalli) é uma delas.

Estas são minhas opiniões. Não sou médico e gostaria muito de ouvir a
verdade "oficial".



Deixo uma pergunta para vocês, se puderem, me responderem:


- Como fazer para diminuir o custo de uma pesquisa científica?


Tudo de bom,

Anderson Porto
http://www.tudosobreplantas.com.br


-----Mensagem original-----
De: Oraculo [mailto:oraculo@atibaia.com.br]
Enviada em: segunda-feira, 28 de fevereiro de 2005 23:27
Para: ciencialist@yahoogrupos.com.br
Assunto: Re: [ciencialist] RN: Veja denuncia "medicina natural de A a Z"
como farsa



Oi Amaury

Bem, relatos do tipo " eu fiz e deu certo" ou "alguém que conheço fez e
deu certo" não embasam conclusões confiáveis. Há uma enorme gama de
motivos para ter "dado certo" com sua mãe, que não a simpatia
alternativa escolhida.

A quase totalidade das doenças e dificuldades de saúde regridem
naturalmente. É uma piada recorrente entre médicos que se você tiver uma
gripe e tomar homeopatia estará curado em uma semana, mas se não tomar,
vai levar 7 dias para sarar..:-)

Outro problema é o termo "natural". O que considera "natural"? Curare é
natural, comigo-ninguém-pode é natural, diversos venenos poderosos são
naturais, se por natural quiser dizer sem a modificação ou intervenção
humana (ou mesmo se quiser dizer vindo de plantas). Diversas plantas
são perigosas, e mandioca selvagem mata em algumas horas, se comer sua
raiz.

Se alguém sugerir que uma determinada simpatia cura resfriado, e você se
curar, depois de cumprir a determinação, isso não significa que a
primeira tem relação com a segunda. Por exemplo, suponha que, antes de
entrar na lotérica e apostar, você de uma topada. Se ganhar na MegaSena,
vai relacionar a topada como causa de ter ganho? Claro que não..:-)

Da mesma forma, se fizer algo, e depois de algum tempo se curar ou se
sentir melhor, isso não indica relação entre causa e efeito. Apenas
relatos estatisticos confiaveis, no lugar de anedóticos, dão alguma
confiabilidade sobre isso.

Se acompanhar 100 pessoas com cancer, 5% delas vai se curar
expontaneamente, e 95% delas vai morrer, em alguns anos. Não precisa
fazer nada, nenhum tratamento, a taxa de regressão é de 5% (variando em
alguns tipos de cancer). Mas, se usar tratamentos medicos atuais em 100
outras pessoas com cancer, entre 85 e 95% destas vai se curar, e entre
15 e 5% vai morrer em alguns anos. O que isso diz a você, sobre causa e
efeito nesse caso?

A única forma de saber se sua mãe se curou devido ao tratamento
alternativo, ou se se curaria de toda forma, seria um estudo estatistico
que acompanhasse diversas pessoas com o mesmo problema, o mesmo
tratamento, e comparar as curas. Suponha que, de 100 pessoas com o mesmo
problema que sua mãe, 15% se curem sozinhas em 2 meses. Imagine que em
um estudo controlado, com outras 100 pessoas com o mesmo problema, o
tratamento alternativo cure, digamos, 15% delas em dois meses. Poderemos
dizer que é o tratamento que está fazendo efeito nestes casos, sua mãe
incluida, ou ele é inócuo?

Quando um tratamento alternativo é submetido a esse tipo de estudo de
rigor, ele em geral falha. E apenas por isso não é aceito, e continua
alternativo. Pensa que se um tratamento alternativo, por exemplo curar
cancer com suco de babosa, pudesse demonstrar que cura mais que a taxa
de regressão natural da doença, seria mantido "alternativo"? Ou seria
incorporado no arsenal médico para tratar canceres?

Quando tratamentos alternativos reclamam da "medicina tradicional" ou
do FDA americano, que impede que sejam prescritos por médicos ou
vendidos a população, eles estão na verdade tentando escapar do rigor de
apresentar estudos dentro desses parametros. Se concordassem a se
submeter a esse tipo de exigencia, que as vezes leva anos de estudos,
poderiam demonstrar que são eficazes e que tem efeito real. Mas, e o
risco de falhar?..:-)

Agora pense em algo sério, uma doença real e perigosa, por exemplo,
tuberculose. Ficaria tranquilo se seu filho com tuberculose demorasse a
ir ao médico "tradicional" e, no lugar, apenas tomasse os "remedios
alternativos" recomendados por um livro desses? Dormiria tranquilo à
noite, mesmo sabendo que essas receitas foram criadas por charlatões,
que não tem a menor noção do que falam, não fizeram nenhum estudo e
baseiam suas receitas em coisas como "fulano fez e ficou melhor"?

Acho que não..:-)

Um abraço.

Homero








----- Original Message -----
From: Amauri Nolasco Sanches Jr
To: ciencialist@yahoogrupos.com.br
Sent: Monday, February 28, 2005 5:42 PM
Subject: Re: [ciencialist] RN: Veja denuncia "medicina natural de A a
Z" como farsa


Oi Oraculo

Minha mãe tem e fez e deu certo, como uma coisa
natural pode fazer mal? Será que não é uma
armação dos laboratórios?

Amauri




--- Oraculo <oraculo@atibaia.com.br> escreveu:

---------------------------------
Olá Eurico

A retirada de um livro claramente fraudulento, com
recomendações que colocam em perigo a vida de quem o
consulta, dificilmente se enquadraria na categoria de
censura. Está mais para ação de responsabilidade
social, que deveria ser seguida por todos os meios de
comunicação, mesmo os que lucram com determinadas
maluquices como medicinas alternativas.

O interesse da população por medicina alternativa (que
em geral significa apenas medicina sem comprovação de
eficácia) não se dá devido ao "monopolio" da medicina
tradicional, mas sim devido a pouca informação do
público em geral sobre mecanismos de ação, metabolismo
e biologia, segurança de tratamento, aliada a
curiosidade natural de seres humanos, busca por
alternativas à falta de acesso a medicina tradicional
(que qualquer pessoa preferiria, se fosse possível o
acesso universal a esta), e diversos outros fatores.

Motoristas de terceiro mundo (e alguns no primeiro
também) tem grande atração por comportamentos de
risco, como dirigir a altas velocidades e ignorar leis
de transito, e é dever tanto do estado quanto dos
meios de comunicação lutar contra essa atitude. Da
mesma forma, o interesse em tratar cancer com argila e
sucos deve ser investigado dentro de rigorosos
procedimentos de verificaçào, e não receitado em
livros de procedencia prá lá de duvidosa (eu diria
criminosa mesmo).

Um tratamento avançado, com medicina de ponta, se não
cura 100% de uma doença (e nada cura ou afirma curar),
tem o mérito de ter sido testado com niveis de
segurança e produzido resultados, comprováveis, muitas
vezes maior que as alternativas (não tratar ou tratar
com medicina alternativa). E pode apresentar suas
evidencias disso.

Ainda que tenha críticas a revista Veja, como de resto
sobre a maioria das revistas de entretenimento em
geral, e mesmo a mídia como um todo, a ação da revista
neste caso foi legítma e digna de ser aplaudida.
Livrarias que comercializam o livro deveriam
disponibilizar o artigo para seus clientes, como forma
de prevenir o gasto desnecessário e o risco que este
tipo de "literatura" e de "medicina alternativa" pode
causar.

Um abraço.

Homero







----- Original Message -----
From: Eurico Ferreira de Souza Jr.
To: ciencialist@yahoogrupos.com.br
Sent: Monday, February 28, 2005 5:10 PM
Subject: Re: [ciencialist] RN: Veja denuncia
"medicina natural de A a Z" como farsa


[E]> a veja vai censurar a lista dos mais vendidos?
vai ter que mudar o nome para "lista dos livros mais
vendidos indicados pela veja". Não se combate mentira
com mentira...

Percebam o interesse da população por qualquer
"medicina alternativa". É indicativo do
descontentamento com o monopólio da medicina.


_\|/_





##### ##### #####

Para saber mais visite
http://www.ciencialist.hpg.ig.com.br


##### ##### ##### #####


Yahoo! Grupos, um serviço oferecido por:

São Paulo Rio de Janeiro Curitiba Porto Alegre Belo Horizonte Brasília




------------------------------------------------------------------------------
Links do Yahoo! Grupos

a.. Para visitar o site do seu grupo na web, acesse:
http://br.groups.yahoo.com/group/ciencialist/

b.. Para sair deste grupo, envie um e-mail para:
ciencialist-unsubscribe@yahoogrupos.com.br

c.. O uso que você faz do Yahoo! Grupos está sujeito aos Termos do Serviço do Yahoo!.



[As partes desta mensagem que não continham texto foram removidas]



SUBJECT: Re: [ciencialist] A Física é a igreja católica da Ciência
FROM: "Oraculo" <oraculo@atibaia.com.br>
TO: <ciencialist@yahoogrupos.com.br>
DATE: 01/03/2005 15:00

Olá Alvaro

Sim, em toda a fisica ..:-) Mas os superstrings ainda não são fisica, nesse sentido. São hipoteses, que, embora tenham consistencia fisico/matematica, e permitam explicar diversos fenomenos, não foram testados e nem se conhece (ainda) uma forma de faze-lo..:-)

Assim, estão esperando para se tornar fisica, o que pode, alias, nem acontecer (basta que um experimento demonstre o engano da proposição ou equação relacionada..:-).

Nenhum fisico ou cientista dirá que superstrings É fisica, mas que é uma boa hipótese a ser ainda testada. Já as leis da gravitação, pode perfeitamente ser demonstrada, dentro de experimentos controlados, permite fazer previsões acertadas (como encontrar planetas antes que fiquem visíveis por telescópios) e permite criar tecnologia embasada nela. Isso é física. A afirmação de que , se pular de um prédio de 10 andares, irá acelerar a taxa de 9,8 metros por segundo ao quadrado, desprezado o atrito do ar, é física e pode ser demonstrada (seria voluntário para provar que está incorreta?..:-).

É isso que pode ser demonstrado por evidências..:-)

Ser compátível e ser física são coisas diferentes. Uma hipotese sobre mais de 3 dimensões e a existencia real de mais de 3 dimensões são coisas diferentes. Afirmar que "existem" 5 dimensões e afirmar que "podem existir" 5 dimensões, são coisas diferentes. Confundir essas afirmações é que dá munição a acusações desse tipo, de que a física é tão vaga ou improvável quanto, digamos, a Terra Média ou o paraiso católico..:-) Ou que a homeopatia e as leis da termodinamica tem a mesma confiabilidade ou eficácia..:-)

Um abraço.

Homero




----- Original Message -----
From: Alvaro Augusto (E)
To: ciencialist@yahoogrupos.com.br
Sent: Tuesday, March 01, 2005 11:37 AM
Subject: Re: [ciencialist] A Física é a igreja católica da Ciência


Hum, interessante... Em TODA a física? Isso inclui supertrings também?

E as ondas gravitacionais? E os monopólos magnéticos? São coisas compatíveis com as teorias mais modernas, mas fogem à verificação experimental...

[ ]s

Alvaro Augusto
----- Original Message -----
From: Ivan Carlos
To: ciencialist@yahoogrupos.com.br
Sent: Tuesday, March 01, 2005 1:58 AM
Subject: Re: [ciencialist] A Física é a igreja católica da Ciência


a pequena grande diferença é que a física provamos, mostramos e repetimos os
fatos quantas vezes forem necessárias a quem quizer ver

...já a crença... só crendo mesmo rsss

Ivan "Doomer" Carlos
Social Engineering Specialist
-
Cell.: +55 (11) 8112-0666
icarlos@icarlos.net
www.icarlos.net
-
MSN: icarlos@icarlos.net
Y!M / Skype: ivandoomer
--------------------------------------------------


----- Original Message -----
From: "Maria Natália" <grasdic@hotmail.com>
To: <ciencialist@yahoogrupos.com.br>
Sent: Monday, February 28, 2005 10:23 PM
Subject: [ciencialist] A Física é a igreja católica da Ciência




Tá dito!
Pois é. Não sabiam?
Vem no livro: " As calças de Pitágoras, Deus, a Física e a guerra dos
sexos" de Margaret Wertherm e claro é da GRADIVA.
Alguém já leu? Encontrei-o à venda na feira do livro de minha escola e
não resisti a provocar-vos.
Quem o ler primeiro põe aqui o seu comentério?
Uma abraço



[As partes desta mensagem que não continham texto foram removidas]



##### ##### #####

Para saber mais visite
http://www.ciencialist.hpg.ig.com.br


##### ##### ##### #####


Yahoo! Grupos, um serviço oferecido por:







------------------------------------------------------------------------------
Links do Yahoo! Grupos

a.. Para visitar o site do seu grupo na web, acesse:
http://br.groups.yahoo.com/group/ciencialist/

b.. Para sair deste grupo, envie um e-mail para:
ciencialist-unsubscribe@yahoogrupos.com.br

c.. O uso que você faz do Yahoo! Grupos está sujeito aos Termos do Serviço do Yahoo!.



[As partes desta mensagem que não continham texto foram removidas]



SUBJECT: Re: RES: RN: Veja denuncia "medicina natural de A a Z" como farsa
FROM: "Rodrigo Marques" <rodmarq72@yahoo.com.br>
TO: ciencialist@yahoogrupos.com.br
DATE: 01/03/2005 16:06


Ei pessoal, alguém consegue o texto da Veja a esse respeito que eu
quero mandar pra uns amigos? É que eu não sou assinante nem comprei
o exemplar em banca...(na lista da STRBrasil colocaram mas os
acentos não saíram).





SUBJECT: Lei Americana proibe discriminacao genetica
FROM: "Silvio" <scordeiro@terra.com.br>
TO: <Conversa_de_Botequim@yahoogrupos.com.br>, <acropolis@yahoogrupos.com.br>, <ciencialist@yahoogrupos.com.br>
DATE: 01/03/2005 16:26

Copio do Jocax:



Lei proíbe discriminação com base em informação genética nos EUA
da Agência Lusa

O Senado dos Estados Unidos aprovou por unanimidade um projeto de lei que proíbe a discriminação com base no conhecimento do patrimônio genético das pessoas.

O documento impede as empresas de despedir ou não admitir empregados com tendências genéticas para doenças e proíbe as seguradoras de recusar ou aumentar apólices de cobertura médica com base em informações de testes genéticos.

"As informações genéticas servem apenas para detectar um potencial de doenças ou perturbações genéticas, o que não equivale a um diagnóstico", disse a republicana Olympia Snowe durante a discussão da lei.

"As extraordinárias promessas da ciência para melhorar a saúde e aliviar o sofrimento estão em perigo se as nossas leis não garantirem proteção necessária contra os abusos e o mau uso das informações genéticas", afirmou o democrata Edward Kennedy.

"As pessoas têm medo do câncer, mas receiam muito mais perder o emprego ou o seguro de saúde", sublinhou Kennedy, membro da Comissão de Saúde do Senado.

O Senado já tinha aprovado um texto semelhante em outubro de 2003, mas este ficou obsoleto por falta de votação da Câmara dos Representantes, devido à oposição de setores empresariais e de alguns legisladores republicanos.

Desta vez, porém, a maioria republicana do Senado divulgou uma mensagem encorajante do governo Bush a favor do texto, esperando que seja agora definitivamente adotado.

O texto da Casa Branca diz que o potencial mau uso das informações genéticas "levanta questões graves do ponto de vista moral e jurídico".

=============================

Achados pontos do DNA que podem levar a doenças
Pesquisadores identificam falhas no DNA comuns a várias pessoas, indicando caminhos para novas terapias genéticas


São Paulo - Uma equipe de cientistas deu um grande passo na busca por genes que causam doenças, ao mapear as pequenas - porém poderosas - variações que formam o ser humano. Um rascunho foi apresentado no encontro anual da Associação Americana para o Avanço da Ciência (AAAS) e é publicado nesta sexta-feira na revista da organização, a Science (www.sciencemag.org).

Segundo o editor, Donald Kennedy, o mapa "fornecerá uma fonte inestimável para a pesquisa genética melhorar a saúde humana".

As pessoas compartilham 99,9% de seu DNA. É justamente neste 0,1% que moram as diferenças, da cor do cabelo à forma pela qual um organismo responde a determinado remédio ou sua predisposição a doenças.

O tipo de mutação genética mais comum pode ser comparado a um erro de soletração.

O DNA é formado por 3,2 bilhões de bases químicas identificadas pelas letras A, T, C e G. Quando uma das letras sai da ordem, é chamado de polimorfismo de nucleotídeo único (SNP) - ou somente snip. A mudança pode significar ter uma proteína defeituosa, que leva ou não ao surgimento de doenças.

Os pesquisadores, da companhia de biotecnologia Perlegen Sciences, identificaram 1,58 milhão de snips, a maioria comum entre americanos de ascendências européia, africana e chinesa. A descoberta foi feita com base na análise do DNA de 71 pessoas.

A pesquisa genética tem se concentrado em problemas de saúde decorrentes de uma grande mutação em um único gene. Mas doenças como cardiopatias e diabetes são resultado de interações entre mais de um gene e fatores comportamentais e ambientais, quando indicar um "culpado" genético é ainda mais penoso.

Os líderes do projeto, David Cox e David Hinds, não identificaram quais snips causam doenças, mas sustentam que o mapa indica qual caminho deve ser seguido dentro da pesquisa genética.

"Eles também mostram que os snips não variam de forma independente, então a tipagem poderá ser feita entre 200 mil, 300 mil snips, não entre os 1,58 milhão", diz o geneticista Sergio Danilo Pena, da Universidade Federal de Minas Gerais (UFMG).

Em 2005, o Projeto Internacional HapMap, que mapeia padrões de variações genéticas em 270 pessoas da Nigéria, Japão, China e Estados Unidos, deve apresentar mais resultados.
http://www.estadao.com.br/ciencia/noticias/2005/fev/18/68.htm


Cristina Amorim, com AP






--//--
"Nossos genes são nosso bem mais precioso"

[As partes desta mensagem que não continham texto foram removidas]



SUBJECT: Re: [ciencialist] Re: RES: RN: Veja denuncia "medicina natural de A a Z" como farsa
FROM: Luis Brudna <luisbrudna@gmail.com>
TO: ciencialist@yahoogrupos.com.br
DATE: 01/03/2005 16:28

O texto completo está disponível no blog do Mori

http://www.liquito.blogger.com.br/2005_02_01_archive.html#35393056

Até
Luís Brudna


On Tue, 01 Mar 2005 19:06:01 -0000, Rodrigo Marques
<rodmarq72@yahoo.com.br> wrote:
>
>
> Ei pessoal, alguém consegue o texto da Veja a esse respeito que eu
> quero mandar pra uns amigos? É que eu não sou assinante nem comprei
> o exemplar em banca...(na lista da STRBrasil colocaram mas os
> acentos não saíram).
>


SUBJECT: Trechos da matéria da Veja (Medicina Alternativa de A a Z)
FROM: "E m i l i a n o C h e m e l l o" <chemelloe@yahoo.com.br>
TO: <ciencialist@yahoogrupos.com.br>
DATE: 01/03/2005 16:57

========================
"... Hebe Camargo, Ratinho, Gugu Liberato faturaram milhões para tecer
elogios ao livro, assinado por um certo Carlos Nascimento Spethmann. O
problema é que esse autor não existe. Pior: a obra foi escrita sem a
assessoria de médicos."

"...Em outras palavras, a Editora Natureza teria copiado a editora
Missionária A Verdade Presente, que agora a estaria copiando de volta. Um
estranho caso de plágio do plágio."

" Os alimentos ajudam na manutenção de uma vida saudável, mas não curam
doenças graves. Para eximir-se de responsabilidades, o livro volta e meia
chama atenção para a necessidade de procurar um médico. Ao mesmo tempo,
propõe tratamentos mágicos (veja quadro)".

(quadro)

Alguns absurdos do livro Medicina Alternativa de A a Z

BRONCOPNEUMONIA
> Recomendações do livro: Tratamentos à base de mel de abelhas, suco de alho
e sopa de cebola. Se nada funcionar em 48 a 72 horas, deve-se procurar
auxílio.
> A palavra dos médicos: É uma doença grave. O atraso no tratamento com
remédios pode ser fatal.

ERISIPELA
> Recomendações do livro: Hortaliças, arroz e argila devem ser aplicados
sobre essas infecções de pele que acomete principalmente idosos e
diabéticos. A doença seria "altamente contagiosa".
> A palavra dos médicos: A erisipela não é contagiosa. S enão for tratada
com antibióticos, pode resultar em infecção generalizada.

DIABETES
> Recomendações do livro: Indica uma compressa de argila com cebola ralada
na região lombro-ventral e a ingestão de argila diluída em suco de
beringela, entre outros tratamentos.
> A palavra dos médicos: Tudo isso é inúcuo. O perigo é o paciente apostar
nessas soluções mágicas e se afastar do tratamento.

========================

No mais, a veja fica 'chorando', talvez com uma certa 'inveja', falando
sobre os rios de dinheiro que os produtores do livro faturaram.

A minha opinião é que o livro, talvez, tenha seus méritos, como
indicação de soluções caseiras para males 'menores', como uma dor de barriga
de origens conhecidas (feijoada, eheheh), por exemplo. Entretando, doenças
graves como a Diabetes requerem uma análise mais específica, ou seja, de
médicos da área. A matéria serviu como uma alerta para as pessoas 'com pouca
cultura', que talvez pudessem seguir a risca as indicações do livro e
promover danos inrreversíveis a sua saúde.

[ ] 's do Emiliano Chemello
emiliano@quimica.net
http://www.quimica.net/emiliano
http://www.ucs.br/ccet/defq/naeq

" Rien ne se perd, rien ne se crée,
tout se transforme."

Antoine Laurent de Lavoisier (químico francês, 1743 - 1794)




SUBJECT: Re: [ciencialist] Fw: feira de ciencias para ed. infantil
FROM: "Silvio" <scordeiro@terra.com.br>
TO: <ciencialist@yahoogrupos.com.br>
DATE: 01/03/2005 17:02


1. fazer uma estufa para chocar 6 ovos (calcular os 21 dias do choco para
que a eclosão ocorra na data da feira)

2. plantar feijão em jarro com água ou alpiste na cabeça de um boneco
(obviamente o topo da cabeça em forma de vaso e com terra fértil)

3. colocar um ovo inteiro dentro de um vidro com o diâmetro da boca menor
que o ovo - basta colocar um ovo bem cozido numa solução de bicarbonato de
sódio. Quando a casta estiver macia, coloque-o com cuidado dentro da
garrafa. o ar endurece a casca novamente.

4. Dependendo de onde a tia Vivi mora, conseguir produtos da região para que
um aluno mostre os diversos sub-produtos : arroz: a palha como adubo, a
casca como cama para aves e adubo, a cutícula como materia prima para
divsersos produtos, o grão comestível - idem para o o trigo, a aveia, o
milho......

sds.,

silvio


-----Mensagem Original-----
De: "Luiz Ferraz Netto" <leobarretos@uol.com.br>
Para: "ciencialist" <ciencialist@yahoogrupos.com.br>
Enviada em: domingo, 27 de fevereiro de 2005 17:30
Assunto: [ciencialist] Fw: feira de ciencias para ed. infantil



Oi Roberto,

aguardo uma sugestão sua para a Profa. Vivian.
Algo de Botânica me parece o mais recomendável no caso ... mera sugestão.

[]'
===========================
Luiz Ferraz Netto [Léo]
leobarretos@uol.com.br
http://www.feiradeciencias.com.br
===========================
-----Mensagem Original-----
De: viviancolodo
Para: leobarretos
Enviada em: domingo, 27 de fevereiro de 2005 16:17
Assunto: feira de ciencias para ed. infantil


Olá

Gostaria de saber se vocês podem me ajudar........
Sou professora de ed. infantil e preciso fazer algo com minhas crianças que
tem somente 5 anos para a feira de ciências que acontecerá no final de
março........O assunto é globalização

Não tenho nenhuma idéia vocês poderiam me enviar idéias

Preciso fazer algo bem legal pois sou nova nesta escola e preciso mostrar
serviço, pois estou em experiência
Aguardo

Vivian


--------------------------------------------------------------------------------


No virus found in this incoming message.
Checked by AVG Anti-Virus.
Version: 7.0.300 / Virus Database: 266.5.0 - Release Date: 25/02/2005

----------

No virus found in this outgoing message.
Checked by AVG Anti-Virus.
Version: 7.0.300 / Virus Database: 266.5.0 - Release Date: 25/02/2005


[As partes desta mensagem que não continham texto foram removidas]



##### ##### #####

Para saber mais visite
http://www.ciencialist.hpg.ig.com.br


##### ##### ##### #####
Links do Yahoo! Grupos












SUBJECT: RES: [ciencialist] P/ Anderson (era Veja denuncia "medicina natural de A a Z" como farsa)
FROM: "Anderson Porto" <acp722003@tudosobreplantas.com.br>
TO: <ciencialist@yahoogrupos.com.br>
DATE: 01/03/2005 17:34

Bem...

Achei o link para seu "staff":


Anderson Porto
http://www.tudosobreplantas.com.br


-----Mensagem original-----
De: Oraculo [mailto:oraculo@atibaia.com.br]
Enviada em: terça-feira, 1 de março de 2005 14:50
Para: ciencialist@yahoogrupos.com.br
Assunto: [ciencialist] P/ Anderson (era Veja denuncia "medicina natural
de A a Z" como farsa)



Olá Anderson

Anderson: - O FDA americano não é exemplo de verdade científica, pois já
foi dominado por empresas fabricantes de drogas e agrotóxicos, vulgos
remédios e venenos; Concorda?

Mais ou menos..:-) Há um forte "vies" antiamericano (ou
anti-organizações) nessa colocação, já que pressupõe que toda
organização pode ser dominada por forças ocultas, em geral, do
capitalismo malvado..:-) Ser "dominado" é muito parecido com as Grandes
Conspirações Para Esconder A Verdade, que, em geral, são irreais.

Claro que qualquer orgão administrativo, em especial os que determinam
controle de grandes somas de dinheiro, podem ser eventualmente alvo de
pressões e mesmo de alguma corrupção. É da natureza humana, e da
natureza das organizações humanas.

Mas, para determinar que a totalidade de qualquer sistema é falho,é
necessário muitas evidencias sólidas, e o FDA ainda se mantém
relativamente integro em sua maior parte. Por exemplo, ainda é
necesário, mesmo para grandes fabricantes, apresentar estudos de longo
prazo, ainda é preciso licença especial para testes em humanos, ainda é
efetuado rigoroso controle de casos, etc. O que não impede que sempre
escape algo, como Viox, e outros. Nem impede que o próprio governo,
principalmettne o Bush, distorça o foco, direcionando a preocupação do
sistema de controle mais para drogas ilícitas que a segurança da
populaçào.

Entretanto, a solução destes desvios passa, não por ignorar o FDA, mas
aumentar sua eficácia, reverter as posições politicas e ideologicas e
aumentar seus mecanismos de controle e segurança, principalmente a
estrutura cientifica. E, se fosse possível, criar um órgão tão eficaz
quanto o FDA no Brasil, para depender menos do americano..:-)

Anderson: - Eu não sabia que as informações nutricionais impressas em
nossos mantimentos é de acordo com o padrão americano, você sabia?"

Sim, mas ainda é melhor que nada..:-) Se nossa tradicional incompetencia
de terceiro mundo não nos permite fazer nossos próprios estudos (as
universidades seriam excelente local de pesquisa, não acha?..:-), pelo
menos alguma informação nutricional está disponível.

Anderson: - Medicina natural, medicina alternativa, medicina holística,
medicina tradicional... São termos que não significam a mesma coisa,
apesar de invariavelmente jornalistas ou pessoas mal informadas usarem
como se fossem;"

Sim, o problema de definir os termos antes de um debete ou alegação é
sempre trazido a tona. É preciso definir, para comunicar. Em todo caso,
se jornalistas ou pessoas mal informadas usam esses termos, como vamos
separar o correto, de incorreto, que ferramenta nos dará um grau de
confiabilidade nesse processo? Exatamettne por isso que a ciência, e sua
ferramenta mais confiável o método cientifico, são usados. Não importa o
termo que um jornalista ou pessoa mal informada use, se o processo em sí
funciona, o método irá demonstrar isso.

Anderson: - A ciência procura informações empíricas para se orientar e
há bastante tempo. Através da etnobotânica eles descobrem determinados
usos de espécies de uso popular e depois pesquisam se são realmente
eficazes;"

Claro que sim. Boa parte dos conhecimentos sobre medicamentos se iniciou
com o estudo de drogas presentes em plantas. Mas, uma vez detectado o
elemento, a molécula, refina-la, descobrir seu mecanismo de ação,
determinar dosagem ideal, separar elementos daninhos ou com efeitos
colaterais, etc, é parte importante do processo de aumentar tanto a
eficácia quanto a segurança desses usos. Nenhum cientista nega isso, que
espécies de uso popular podem mostrar indicações importantes, mas é
preciso separa-las das que nada fazem, são placebos ou francamente
perigosas.

Anderson: - A população procura "medicina alternativa" porque os chás
funcionam, as compressas, o banho de assento, as pomadas... Funcionam!
Enquanto a "oficial" está cheia de médicos mal formados, mal
estruturados, com professores mal remunerados, mal humorados... E
MUUUUUUITO CARA! "

Sim, chás funcionam, para a maioria das doenças que, de outra forma,
sarariam sozinhas. Eliminam desconfortos que, de outra forma, seriam
suportados pelas pessoas, mas sarariam também. Tomar um chá de erva
cidreira é excelente, eu adoro, mas não cura nem cancer nem tuberculose.
Analise a espectativa de vida de nossos antepassados, tratados apenas
com chás e banhos de assento, com as suas. Deve haver um motivo para que
aqueles tivessem a espectativa de vida de 30 e poucos anos (ou menos) e
você mais de 80.

O motivo principal é que, embora seja possivel viver com desconfortos,
doenças graves e sérias só podem ser efetivamente curadas com medicina
de ponta, conhecimento cientifico real, e drogas mais eficazes.

Pense em uma apendicite. Antes da cirurgia e antibióticos, matava 100%.
Depois, pode ser curada 100%. Tomaria chá ou ervas para isso?

Anderson, não sou contra chás. Não são medicina alternativa, são
medicina tradicional, bem conhecida e aceita por médicos tradicionais.
Acupuntura, urinoterapia, homeopatia, iridologia, florais de bach, etc,
é que são medicina alternativa, sem evidencias e sem confiabilidade.

Anderson: Ao meu ver, o correto é ir contra o objeto "causa única", da
medicina de faculdades, que chamo de reducionista. "

Concordo..:-) Mas essa é uma caricatura do medico e da medicina vendida
pelos alternativos..:-) Homeopatas devem conversar com o paciente e
verificar o conjunto, nào apenas a doença. Mas é exatamente o que um bom
médico faz, ou deve fazer. Existem maus médicos? Claro..:-) Existem mau
homeopatas? Claro..:-) Chamar a medicina atual de reducionista é uma
acusação falsa. Juntas médicas existem justamente para analisar todas as
possibilidades, em geral gerenciadas por um clínico geral.

A massificação da medicina, em especial na seguridade social, pode
realmente ter criado um grave problema de atendimento, de falta de
cuidado, de maus médicos, com jornadas de trabalho enormes, sem tempo
para pacientes. Mas não é uma falha da medicina ou da ciência, mas da
administração, dos governos. Vá a um bom médico, particular, de sua
confiança, e será atendido como se deve, com tempo, uma boa conversa,
analise de seus dados, sua vida, etc. Ele não vai, claro, analisar sua
"aura", mas fará o que for preciso para ajuda-lo..:-)

Anderson: - Como fazer para diminuir o custo de uma pesquisa científica?

Esta questão, embora tenha resposta simples, não é simples de ser
implnatada, já que depende da compreensão do problema por quem governa o
pais: investir na educação, desde escolas de base, até universidades. A
pesquisa em universidades é a base de uma forte industria baseada na
ciência e para medicina de ponta. Universidades que podem construir
equipamentos, e sustentar pesquisadores, alunos e professores, minimizam
o custo desse conhecimento de forma espantosa. Alunos pesquisam pelo
prazer de descobrir, criam sem restrições de qualquer ordem (por
exemplo, não temem ter sua carreira prejudicada se pesquisarem um
assunto controverso ou pouco aceito), estão no auge da criatividade e
vigor, além de muitas outras vantagens.

Agora, implantar essa solução simples, é outro (enorme) problema..:-)

Um abraço.

Homero



----- Original Message -----
From: Anderson Porto
To: ciencialist@yahoogrupos.com.br
Cc: tudosobreplantas@yahoogrupos.com.br
Sent: Tuesday, March 01, 2005 2:59 AM
Subject: [ciencialist] RES: RN: Veja denuncia "medicina natural de A a
Z" como farsa


Olá Homero,


Faz tempo que não participo da lista ciencialist, mas não poderia
deixar
de comentar umas coisas que li. Concordo em quase tudo que diz, mas...



- O FDA americano não é exemplo de verdade científica, pois já foi
dominado por empresas fabricantes de drogas e agrotóxicos, vulgos
remédios e venenos; Concorda?


- Eu não sabia que as informações nutricionais impressas em nossos
mantimentos é de acordo com o padrão americano, você sabia?


- Medicina natural, medicina alternativa, medicina holística, medicina
tradicional... São termos que não significam a mesma coisa, apesar de
invariavelmente jornalistas ou pessoas mal informadas usarem como se
fossem;


- A ciência procura informações empíricas para se orientar e há
bastante
tempo. Através da etnobotânica eles descobrem determinados usos de
espécies de uso popular e depois pesquisam se são realmente eficazes;


- A população procura "medicina alternativa" porque os chás funcionam,
as compressas, o banho de assento, as pomadas... Funcionam! Enquanto a
"oficial" está cheia de médicos mal formados, mal estruturados, com
professores mal remunerados, mal humorados... E MUUUUUUITO CARA!


Já descobriram diversas, milhares de espécies com potencial para
pesquisa. Entretanto, estas custam caro e, por causa de patentes, as
informações raramente chegam ao público leigo.

Como você mesmo disse, às vezes a doença passa em 7 dias, mesmo sem o
doente ter ido buscar orientação em consultas médicas, comprado
remédios
e repousado.

Ao meu ver, o correto é ir contra o objeto "causa única", da medicina
de
faculdades, que chamo de reducionista. O médico, no meu entender, deve
procurar as causas psicológicas, o fator gerador de um estado de
doença.
Daí, em vez de recomendar cirurgias, quimioterapias e sei lá mais
quais
agressões, onde invariavelmente ganha $$$ o médico anestesista (por
fora), o convênio, o hospital, o médico cirurgião ou quimioterapeuta,
o
instrumentador... Procurar agredir o mínimo possível um corpo já
debilitado... E o bolso.

É claro que dentre diversas soluções e, numa situação de falta de
tempo
para fazer este tipo de levantamento do paciente, o médico deverá
recomendar aquilo que não possa ser processado depois. Se existe uam
recomendação "oficial" ele deve seguir, pois senão será criticado
pelos
colegas de profissão. A decisão sobre qual caminho seguir, depois de
bem
orientado, deve ser sempre do paciente.

Existem plantas que precisam ser pesquisadas e principalmente fazer
com
que estes dados cheguem ao público leigo de forma idônea. A Aveloz
(Euphorbia tirucalli) é uma delas.

Estas são minhas opiniões. Não sou médico e gostaria muito de ouvir a
verdade "oficial".



Deixo uma pergunta para vocês, se puderem, me responderem:


- Como fazer para diminuir o custo de uma pesquisa científica?


Tudo de bom,

Anderson Porto
http://www.tudosobreplantas.com.br


-----Mensagem original-----
De: Oraculo [mailto:oraculo@atibaia.com.br]
Enviada em: segunda-feira, 28 de fevereiro de 2005 23:27
Para: ciencialist@yahoogrupos.com.br
Assunto: Re: [ciencialist] RN: Veja denuncia "medicina natural de A a
Z"
como farsa



Oi Amaury

Bem, relatos do tipo " eu fiz e deu certo" ou "alguém que conheço fez
e
deu certo" não embasam conclusões confiáveis. Há uma enorme gama de
motivos para ter "dado certo" com sua mãe, que não a simpatia
alternativa escolhida.

A quase totalidade das doenças e dificuldades de saúde regridem
naturalmente. É uma piada recorrente entre médicos que se você tiver
uma
gripe e tomar homeopatia estará curado em uma semana, mas se não
tomar,
vai levar 7 dias para sarar..:-)

Outro problema é o termo "natural". O que considera "natural"? Curare
é
natural, comigo-ninguém-pode é natural, diversos venenos poderosos são
naturais, se por natural quiser dizer sem a modificação ou intervenção
humana (ou mesmo se quiser dizer vindo de plantas). Diversas plantas
são perigosas, e mandioca selvagem mata em algumas horas, se comer sua
raiz.

Se alguém sugerir que uma determinada simpatia cura resfriado, e você
se
curar, depois de cumprir a determinação, isso não significa que a
primeira tem relação com a segunda. Por exemplo, suponha que, antes de
entrar na lotérica e apostar, você de uma topada. Se ganhar na
MegaSena,
vai relacionar a topada como causa de ter ganho? Claro que não..:-)

Da mesma forma, se fizer algo, e depois de algum tempo se curar ou se
sentir melhor, isso não indica relação entre causa e efeito. Apenas
relatos estatisticos confiaveis, no lugar de anedóticos, dão alguma
confiabilidade sobre isso.

Se acompanhar 100 pessoas com cancer, 5% delas vai se curar
expontaneamente, e 95% delas vai morrer, em alguns anos. Não precisa
fazer nada, nenhum tratamento, a taxa de regressão é de 5% (variando
em
alguns tipos de cancer). Mas, se usar tratamentos medicos atuais em
100
outras pessoas com cancer, entre 85 e 95% destas vai se curar, e entre
15 e 5% vai morrer em alguns anos. O que isso diz a você, sobre causa
e
efeito nesse caso?

A única forma de saber se sua mãe se curou devido ao tratamento
alternativo, ou se se curaria de toda forma, seria um estudo
estatistico
que acompanhasse diversas pessoas com o mesmo problema, o mesmo
tratamento, e comparar as curas. Suponha que, de 100 pessoas com o
mesmo
problema que sua mãe, 15% se curem sozinhas em 2 meses. Imagine que em
um estudo controlado, com outras 100 pessoas com o mesmo problema, o
tratamento alternativo cure, digamos, 15% delas em dois meses.
Poderemos
dizer que é o tratamento que está fazendo efeito nestes casos, sua mãe
incluida, ou ele é inócuo?

Quando um tratamento alternativo é submetido a esse tipo de estudo de
rigor, ele em geral falha. E apenas por isso não é aceito, e continua
alternativo. Pensa que se um tratamento alternativo, por exemplo curar
cancer com suco de babosa, pudesse demonstrar que cura mais que a taxa
de regressão natural da doença, seria mantido "alternativo"? Ou seria
incorporado no arsenal médico para tratar canceres?

Quando tratamentos alternativos reclamam da "medicina tradicional" ou
do FDA americano, que impede que sejam prescritos por médicos ou
vendidos a população, eles estão na verdade tentando escapar do rigor
de
apresentar estudos dentro desses parametros. Se concordassem a se
submeter a esse tipo de exigencia, que as vezes leva anos de estudos,
poderiam demonstrar que são eficazes e que tem efeito real. Mas, e o
risco de falhar?..:-)

Agora pense em algo sério, uma doença real e perigosa, por exemplo,
tuberculose. Ficaria tranquilo se seu filho com tuberculose demorasse
a
ir ao médico "tradicional" e, no lugar, apenas tomasse os "remedios
alternativos" recomendados por um livro desses? Dormiria tranquilo à
noite, mesmo sabendo que essas receitas foram criadas por charlatões,
que não tem a menor noção do que falam, não fizeram nenhum estudo e
baseiam suas receitas em coisas como "fulano fez e ficou melhor"?

Acho que não..:-)

Um abraço.

Homero








----- Original Message -----
From: Amauri Nolasco Sanches Jr
To: ciencialist@yahoogrupos.com.br
Sent: Monday, February 28, 2005 5:42 PM
Subject: Re: [ciencialist] RN: Veja denuncia "medicina natural de A
a
Z" como farsa


Oi Oraculo

Minha mãe tem e fez e deu certo, como uma coisa
natural pode fazer mal? Será que não é uma
armação dos laboratórios?

Amauri




--- Oraculo <oraculo@atibaia.com.br> escreveu:

---------------------------------
Olá Eurico

A retirada de um livro claramente fraudulento, com
recomendações que colocam em perigo a vida de quem o
consulta, dificilmente se enquadraria na categoria de
censura. Está mais para ação de responsabilidade
social, que deveria ser seguida por todos os meios de
comunicação, mesmo os que lucram com determinadas
maluquices como medicinas alternativas.

O interesse da população por medicina alternativa (que
em geral significa apenas medicina sem comprovação de
eficácia) não se dá devido ao "monopolio" da medicina
tradicional, mas sim devido a pouca informação do
público em geral sobre mecanismos de ação, metabolismo
e biologia, segurança de tratamento, aliada a
curiosidade natural de seres humanos, busca por
alternativas à falta de acesso a medicina tradicional
(que qualquer pessoa preferiria, se fosse possível o
acesso universal a esta), e diversos outros fatores.

Motoristas de terceiro mundo (e alguns no primeiro
também) tem grande atração por comportamentos de
risco, como dirigir a altas velocidades e ignorar leis
de transito, e é dever tanto do estado quanto dos
meios de comunicação lutar contra essa atitude. Da
mesma forma, o interesse em tratar cancer com argila e
sucos deve ser investigado dentro de rigorosos
procedimentos de verificaçào, e não receitado em
livros de procedencia prá lá de duvidosa (eu diria
criminosa mesmo).

Um tratamento avançado, com medicina de ponta, se não
cura 100% de uma doença (e nada cura ou afirma curar),
tem o mérito de ter sido testado com niveis de
segurança e produzido resultados, comprováveis, muitas
vezes maior que as alternativas (não tratar ou tratar
com medicina alternativa). E pode apresentar suas
evidencias disso.

Ainda que tenha críticas a revista Veja, como de resto
sobre a maioria das revistas de entretenimento em
geral, e mesmo a mídia como um todo, a ação da revista
neste caso foi legítma e digna de ser aplaudida.
Livrarias que comercializam o livro deveriam
disponibilizar o artigo para seus clientes, como forma
de prevenir o gasto desnecessário e o risco que este
tipo de "literatura" e de "medicina alternativa" pode
causar.

Um abraço.

Homero







----- Original Message -----
From: Eurico Ferreira de Souza Jr.
To: ciencialist@yahoogrupos.com.br
Sent: Monday, February 28, 2005 5:10 PM
Subject: Re: [ciencialist] RN: Veja denuncia
"medicina natural de A a Z" como farsa


[E]> a veja vai censurar a lista dos mais vendidos?
vai ter que mudar o nome para "lista dos livros mais
vendidos indicados pela veja". Não se combate mentira
com mentira...

Percebam o interesse da população por qualquer
"medicina alternativa". É indicativo do
descontentamento com o monopólio da medicina.


_\|/_





##### ##### #####

Para saber mais visite
http://www.ciencialist.hpg.ig.com.br


##### ##### ##### #####


Yahoo! Grupos, um serviço oferecido por:

São Paulo Rio de Janeiro Curitiba Porto Alegre Belo
Horizonte Brasília




------------------------------------------------------------------------
------
Links do Yahoo! Grupos

a.. Para visitar o site do seu grupo na web, acesse:
http://br.groups.yahoo.com/group/ciencialist/

b.. Para sair deste grupo, envie um e-mail para:
ciencialist-unsubscribe@yahoogrupos.com.br

c.. O uso que você faz do Yahoo! Grupos está sujeito aos Termos do
Serviço do Yahoo!.



[As partes desta mensagem que não continham texto foram removidas]



##### ##### #####

Para saber mais visite
http://www.ciencialist.hpg.ig.com.br


##### ##### ##### #####
Links do Yahoo! Grupos













SUBJECT: RES: P/ Anderson - Medicina tradicional - FDA - CIB
FROM: "Anderson Porto" <acp722003@tudosobreplantas.com.br>
TO: <ciencialist@yahoogrupos.com.br>
CC: <tudosobreplantas@yahoogrupos.com.br>
DATE: 01/03/2005 18:05

Bem...

Achei o link para o "staff" da FDA:
http://www.fda.gov/oc/orgcharts/orgchart.html

Estranhei o fato de no primeiro click que dei encontrar um quadro de 3
gerentes "VACANCY":
http://www.fda.gov/oc/orgcharts/nctr5om2opfit.pdf , mas...

Percebi que será deveras complicado pesquisar nome por nome até achar
quem trabalha ou trabalhou para empresas de agrotóxicos. Será que existe
esta informação na Internet? Vou perder tempo nisso não... A questão é:
ela seria semelhante a nossa ANVISA?


Sobre "... pressupõe que toda organização pode ser dominada por forças
ocultas, em geral, do capitalismo malvado ...",

Hehehehe... Em vez de falar de lobbies no congreso ou de maracutaias de
cascatas e cachoeiras, me diga, conhece a CIB?

>>>

Paraná - Agência Estadual de Notícias, 26/02/21005 - Transgênicos
http://celepar8cta.pr.gov.br/secs/Cnoti.nsf/nd/A5FBDA8FCCA5976383256FB30
0796448

Ong confirma que é patrocinada por multinacionais

A organização não-governamental Conselho de Informações sobre
Biotecnologia (CIB), em correspondência enviada a Agência Estadual de
Notícias, assinada por Antonio Celso Vilari, informa que não é apenas
financiada pela Monsanto, mas "patrocinada por um grupo de empresas
multinacionais, como: Syngenta Seeds, Bayercropsciencies, BASF,
DuPont/Pioner, Dow Agrosciences e Monsanto". E esclarece também que o
CIB conta com 70 conselheiros e que "nenhum deles é contratado pela
ONG".

A campanha a favor dos transgênicos no Brasil está sendo financiada pelo
CIB, que recebe recursos das empresas multinacionais, para promover a
difusão da cultura de produtos geneticamente modificados na agricultura
brasileira.

A ofensiva maior a favor dos transgênicos tem sido patrocinada pela
Monsanto, que recentemente deflagrou uma milionária campanha
publicitária defendendo os produtos geneticamente modificados. A
campanha foi considerada "enganosa" e acabou sendo suspensa.

A resistência do Governo do Paraná e dos produtores rurais paranaenses
em produzir soja transgênica levou as multinacionais de sementes,
através de sua ONG, o CIB, a investir elevados recursos em campanhas de
marketing.

No Paraná, onde mais de 116 mil produtores plantam soja convencional, o
número dos que optaram por soja transgênica não chega a 3 mil.

============

NOTA:

As empresas que bancam o CIB são as maiores do mundo em vendas globais
de agrotóxicos. A não ser para a DuPont (Pioneer), elas vendem muito
mais veneno do que sementes. [DH]

http://www.etcgroup.org/article.asp?newsid=493

>>>


Sobre a erva-cidreira, cuidado! ;o) Tem muita gente tomando analgésicos
para ressaca e boldo para unha encravada. Não existem panacéias nem
pílulas vermelhas.

Tudo de bom,

Anderson Porto
http://www.tudosobreplantas.com.br


-----Mensagem original-----
De: Oraculo [mailto:oraculo@atibaia.com.br]
Enviada em: terça-feira, 1 de março de 2005 14:50
Para: ciencialist@yahoogrupos.com.br
Assunto: [ciencialist] P/ Anderson (era Veja denuncia "medicina natural
de A a Z" como farsa)



Olá Anderson

Anderson: - O FDA americano não é exemplo de verdade científica, pois já
foi dominado por empresas fabricantes de drogas e agrotóxicos, vulgos
remédios e venenos; Concorda?

Mais ou menos..:-) Há um forte "vies" antiamericano (ou
anti-organizações) nessa colocação, já que pressupõe que toda
organização pode ser dominada por forças ocultas, em geral, do
capitalismo malvado..:-) Ser "dominado" é muito parecido com as Grandes
Conspirações Para Esconder A Verdade, que, em geral, são irreais.

Claro que qualquer orgão administrativo, em especial os que determinam
controle de grandes somas de dinheiro, podem ser eventualmente alvo de
pressões e mesmo de alguma corrupção. É da natureza humana, e da
natureza das organizações humanas.

Mas, para determinar que a totalidade de qualquer sistema é falho,é
necessário muitas evidencias sólidas, e o FDA ainda se mantém
relativamente integro em sua maior parte. Por exemplo, ainda é
necesário, mesmo para grandes fabricantes, apresentar estudos de longo
prazo, ainda é preciso licença especial para testes em humanos, ainda é
efetuado rigoroso controle de casos, etc. O que não impede que sempre
escape algo, como Viox, e outros. Nem impede que o próprio governo,
principalmettne o Bush, distorça o foco, direcionando a preocupação do
sistema de controle mais para drogas ilícitas que a segurança da
populaçào.

Entretanto, a solução destes desvios passa, não por ignorar o FDA, mas
aumentar sua eficácia, reverter as posições politicas e ideologicas e
aumentar seus mecanismos de controle e segurança, principalmente a
estrutura cientifica. E, se fosse possível, criar um órgão tão eficaz
quanto o FDA no Brasil, para depender menos do americano..:-)

Anderson: - Eu não sabia que as informações nutricionais impressas em
nossos mantimentos é de acordo com o padrão americano, você sabia?"

Sim, mas ainda é melhor que nada..:-) Se nossa tradicional incompetencia
de terceiro mundo não nos permite fazer nossos próprios estudos (as
universidades seriam excelente local de pesquisa, não acha?..:-), pelo
menos alguma informação nutricional está disponível.

Anderson: - Medicina natural, medicina alternativa, medicina holística,
medicina tradicional... São termos que não significam a mesma coisa,
apesar de invariavelmente jornalistas ou pessoas mal informadas usarem
como se fossem;"

Sim, o problema de definir os termos antes de um debete ou alegação é
sempre trazido a tona. É preciso definir, para comunicar. Em todo caso,
se jornalistas ou pessoas mal informadas usam esses termos, como vamos
separar o correto, de incorreto, que ferramenta nos dará um grau de
confiabilidade nesse processo? Exatamettne por isso que a ciência, e sua
ferramenta mais confiável o método cientifico, são usados. Não importa o
termo que um jornalista ou pessoa mal informada use, se o processo em sí
funciona, o método irá demonstrar isso.

Anderson: - A ciência procura informações empíricas para se orientar e
há bastante tempo. Através da etnobotânica eles descobrem determinados
usos de espécies de uso popular e depois pesquisam se são realmente
eficazes;"

Claro que sim. Boa parte dos conhecimentos sobre medicamentos se iniciou
com o estudo de drogas presentes em plantas. Mas, uma vez detectado o
elemento, a molécula, refina-la, descobrir seu mecanismo de ação,
determinar dosagem ideal, separar elementos daninhos ou com efeitos
colaterais, etc, é parte importante do processo de aumentar tanto a
eficácia quanto a segurança desses usos. Nenhum cientista nega isso, que
espécies de uso popular podem mostrar indicações importantes, mas é
preciso separa-las das que nada fazem, são placebos ou francamente
perigosas.

Anderson: - A população procura "medicina alternativa" porque os chás
funcionam, as compressas, o banho de assento, as pomadas... Funcionam!
Enquanto a "oficial" está cheia de médicos mal formados, mal
estruturados, com professores mal remunerados, mal humorados... E
MUUUUUUITO CARA! "

Sim, chás funcionam, para a maioria das doenças que, de outra forma,
sarariam sozinhas. Eliminam desconfortos que, de outra forma, seriam
suportados pelas pessoas, mas sarariam também. Tomar um chá de erva
cidreira é excelente, eu adoro, mas não cura nem cancer nem tuberculose.
Analise a espectativa de vida de nossos antepassados, tratados apenas
com chás e banhos de assento, com as suas. Deve haver um motivo para que
aqueles tivessem a espectativa de vida de 30 e poucos anos (ou menos) e
você mais de 80.

O motivo principal é que, embora seja possivel viver com desconfortos,
doenças graves e sérias só podem ser efetivamente curadas com medicina
de ponta, conhecimento cientifico real, e drogas mais eficazes.

Pense em uma apendicite. Antes da cirurgia e antibióticos, matava 100%.
Depois, pode ser curada 100%. Tomaria chá ou ervas para isso?

Anderson, não sou contra chás. Não são medicina alternativa, são
medicina tradicional, bem conhecida e aceita por médicos tradicionais.
Acupuntura, urinoterapia, homeopatia, iridologia, florais de bach, etc,
é que são medicina alternativa, sem evidencias e sem confiabilidade.

Anderson: Ao meu ver, o correto é ir contra o objeto "causa única", da
medicina de faculdades, que chamo de reducionista. "

Concordo..:-) Mas essa é uma caricatura do medico e da medicina vendida
pelos alternativos..:-) Homeopatas devem conversar com o paciente e
verificar o conjunto, nào apenas a doença. Mas é exatamente o que um bom
médico faz, ou deve fazer. Existem maus médicos? Claro..:-) Existem mau
homeopatas? Claro..:-) Chamar a medicina atual de reducionista é uma
acusação falsa. Juntas médicas existem justamente para analisar todas as
possibilidades, em geral gerenciadas por um clínico geral.

A massificação da medicina, em especial na seguridade social, pode
realmente ter criado um grave problema de atendimento, de falta de
cuidado, de maus médicos, com jornadas de trabalho enormes, sem tempo
para pacientes. Mas não é uma falha da medicina ou da ciência, mas da
administração, dos governos. Vá a um bom médico, particular, de sua
confiança, e será atendido como se deve, com tempo, uma boa conversa,
analise de seus dados, sua vida, etc. Ele não vai, claro, analisar sua
"aura", mas fará o que for preciso para ajuda-lo..:-)

Anderson: - Como fazer para diminuir o custo de uma pesquisa científica?

Esta questão, embora tenha resposta simples, não é simples de ser
implnatada, já que depende da compreensão do problema por quem governa o
pais: investir na educação, desde escolas de base, até universidades. A
pesquisa em universidades é a base de uma forte industria baseada na
ciência e para medicina de ponta. Universidades que podem construir
equipamentos, e sustentar pesquisadores, alunos e professores, minimizam
o custo desse conhecimento de forma espantosa. Alunos pesquisam pelo
prazer de descobrir, criam sem restrições de qualquer ordem (por
exemplo, não temem ter sua carreira prejudicada se pesquisarem um
assunto controverso ou pouco aceito), estão no auge da criatividade e
vigor, além de muitas outras vantagens.

Agora, implantar essa solução simples, é outro (enorme) problema..:-)

Um abraço.

Homero

>>>

http://br.groups.yahoo.com/group/ciencialist/




SUBJECT: Re: [ciencialist] No carro: com volt, ampère e ohm
FROM: "Sergio M. M. Taborda" <sergiotaborda@terra.com.br>
TO: ciencialist@yahoogrupos.com.br
DATE: 01/03/2005 19:45

L.E.R.de Carvalho wrote:

>
>
> Mas o que é amperagem? Designa intensidade de corrente elétrica,
> medida em
> amperes, plural de ampere, que o Aurélio ainda não acolheu, preferindo
> indexá-lo em sua versão francesa, ''ampère''. Em Portugal, desde 1929,
> foi
> adotada a forma ''ampério'',

Eu vivi em portugal , estudei electricidade, electronica e fisica em
geral e nunca ouvi , nem li o termo amperio.
Além disso esse termo não consta do dicionário de protugues de portugal
que tenho.
O nome ampère é o proprio nome do cara, mas na lingua portugesa o acento
não é necessário. Assim, o nome do cara é Ampère , mas o nome da unidade
é ampere.
Amperagem é uma palavra da giria electrica e portanto não deve ser usada
em texto ou peças de algum valor informativo (como reportagens
jornalisticas)


Sérgio Taborda



--
No virus found in this outgoing message.
Checked by AVG Anti-Virus.
Version: 7.0.300 / Virus Database: 266.5.7 - Release Date: 01-03-2005



SUBJECT: Re: [ciencialist] refrigerantes...
FROM: "Silvio" <scordeiro@terra.com.br>
TO: <ciencialist@yahoogrupos.com.br>, <naeq-ucs@yahoogrupos.com.br>, <quimica-qaw@yahoogrupos.com.br>
DATE: 01/03/2005 20:50

Só espero que a resposta seja feita por químicos especialidados e não
ideólogos de má-fé.....

silvio.
-----Mensagem Original-----
De: "Emiliano Chemello - Yahoo Grupos" <chemelloe@yahoo.com.br>
Para: <ciencialist@yahoogrupos.com.br>; <naeq-ucs@yahoogrupos.com.br>;
<quimica-qaw@yahoogrupos.com.br>
Enviada em: domingo, 20 de fevereiro de 2005 13:03
Assunto: [ciencialist] refrigerantes...


>
> alguma referência sobre isso?
>
> Amplexos
>
> Emiliano Chemello
> ---
> Contato Naeq:
> Nome: Elizeu Batista
> Email: zzu2300@hotmail.com.br
> Assunto: Coca cola
> Mensagem: Gostaria saber quais os efeitos dos refrigerantes no nosso
> organismo, e em especial a coca cola. De todos que existem, há um pior ?
> Sempre tenho ouvido bastante coisa, mas muitas vezes sem fundamento
> científico . Obrigado
>
>
>
>
> ##### ##### #####
>
> Para saber mais visite
> http://www.ciencialist.hpg.ig.com.br
>
>
> ##### ##### ##### #####
> Links do Yahoo! Grupos
>
>
>
>
>
>
>
>
>
>



SUBJECT: Re: [ciencialist] Veja denuncia "medicina natural de A a Z" como farsa
FROM: "Oraculo" <oraculo@atibaia.com.br>
TO: <ciencialist@yahoogrupos.com.br>
DATE: 02/03/2005 00:35

Olá Silvio

Sou obrigado a concordar com você em tudo, censura é sempre ruim e ninguém deve ser direcionado para ler ou deixar de ler o que for. Mas, por isso mesmo, aleguei que o que a Veja fez não é exatamente censura..:-)

Primeiro, a revista é particular. O que significa que o termo censura não se aplica, pelo menos não totalmente. Censura é o impedimento de publicar, imposto pelo poder administrativo, governo ou estado. Se um tenho um jornal e decido o que publicar, não é mais censura. Embora seja algo, a meu ver, incorreto, daninho, e até imoral, não é ilegal, nem é censura.

Segundo, não é preciso julgar todos os livros de minah lsita (ou da lista da Veja, neste caso), ainda mais se são apresentados em lista de mais vendidos. "Mais vendidos", como bem colocou, não é um julgamento de valor, mas exibição de numero de vendas (se são numeros legítimos ou pagos pelas editoras, é outra história) e o valor deles não está em questão. Mas, uma vez que uma reportagem investigou e descobriu uma fraude ou crime (ou mesmo um perigo) é legítimo retirar da lista de mais vendidos, como forma de proteger seu leitor. Equivale a não publicar informações de sequestros, já que poderiam prejudicar as vítimas. Isso não pode ser considerado censura, certo?

Terceiro, não acho que exista nada com mais impropriedades que os livros do Paulo Coelho..:-) Nada, mesmo..:-) Mas os livros de Paulo (arghhh) Coelho, são vendidos, em tese, como ficção. Embora muitos acreditem no Paulo (arghhh) Coelho, seria como acreditar em Tolkien e sair procurando a Terra Média. Já o livro em questão é vendido como realidade, coisas que podem curar doenças, charlatanismo perigoso.

Talvez exista um lobby de laboratórios contra medicina alternativa, apesar de eu achar (achismo, mas no bom sentido..:-) que não é necessário. O comprador de remedios e drogas alopáticas existe em número suficiente para manter o negócio, desde que não se quebrem patentes..:-) Mas isso não torna menos perigoso um livro que recomenda tratar diabetes com argila e sucos (criando demora no tratamento eficaz dessa perigosa doença).

Um abraço.

Homero




----- Original Message -----
From: Silvio
To: ciencialist@yahoogrupos.com.br
Sent: Tuesday, March 01, 2005 12:02 PM
Subject: [ciencialist] Veja denuncia "medicina natural de A a Z" como farsa


Oráculo:

A Veja cria sério precedente: ou se censura tudo ou não se censura nada....
Assim, os donos na revista ficam sob a espada da Damocles: deverão, todas as
semanas julgar o que é farsa, mentira, !má leitura" e caem inexoravelmente
numa ditadura intelectual.
Se a lista é de "mais vendidos", não se discute o mérito do livro. Basta
trocar o título da lista: "Livros que julgados edificantes e comprovadamente
qualificados para a leitura das famílias de bons costumes"
Será que esse livro contém mais impropriedades que os do paulo coelho?

A censura é sempre perniciosa, injustificável, digna de cristãos medievais
e, no caso presente, por certo estimulada por lobby dos laboratórios....

sds.,
silvio.
"não há efeito sem causa"
-----Mensagem Original-----
De: "Oraculo" <oraculo@atibaia.com.br>
Para: <ciencialist@yahoogrupos.com.br>
Enviada em: segunda-feira, 28 de fevereiro de 2005 17:27
Assunto: Re: [ciencialist] RN: Veja denuncia "medicina natural de A a Z"
como farsa



Olá Eurico

A retirada de um livro claramente fraudulento, com recomendações que colocam
em perigo a vida de quem o consulta, dificilmente se enquadraria na
categoria de censura. Está mais para ação de responsabilidade social, que
deveria ser seguida por todos os meios de comunicação, mesmo os que lucram
com determinadas maluquices como medicinas alternativas.

O interesse da população por medicina alternativa (que em geral significa
apenas medicina sem comprovação de eficácia) não se dá devido ao "monopolio"
da medicina tradicional, mas sim devido a pouca informação do público em
geral sobre mecanismos de ação, metabolismo e biologia, segurança de
tratamento, aliada a curiosidade natural de seres humanos, busca por
alternativas à falta de acesso a medicina tradicional (que qualquer pessoa
preferiria, se fosse possível o acesso universal a esta), e diversos outros
fatores.

Motoristas de terceiro mundo (e alguns no primeiro também) tem grande
atração por comportamentos de risco, como dirigir a altas velocidades e
ignorar leis de transito, e é dever tanto do estado quanto dos meios de
comunicação lutar contra essa atitude. Da mesma forma, o interesse em tratar
cancer com argila e sucos deve ser investigado dentro de rigorosos
procedimentos de verificaçào, e não receitado em livros de procedencia prá
lá de duvidosa (eu diria criminosa mesmo).

Um tratamento avançado, com medicina de ponta, se não cura 100% de uma
doença (e nada cura ou afirma curar), tem o mérito de ter sido testado com
niveis de segurança e produzido resultados, comprováveis, muitas vezes maior
que as alternativas (não tratar ou tratar com medicina alternativa). E pode
apresentar suas evidencias disso.

Ainda que tenha críticas a revista Veja, como de resto sobre a maioria das
revistas de entretenimento em geral, e mesmo a mídia como um todo, a ação da
revista neste caso foi legítma e digna de ser aplaudida. Livrarias que
comercializam o livro deveriam disponibilizar o artigo para seus clientes,
como forma de prevenir o gasto desnecessário e o risco que este tipo de
"literatura" e de "medicina alternativa" pode causar.

Um abraço.

Homero







----- Original Message -----
From: Eurico Ferreira de Souza Jr.
To: ciencialist@yahoogrupos.com.br
Sent: Monday, February 28, 2005 5:10 PM
Subject: Re: [ciencialist] RN: Veja denuncia "medicina natural de A a Z"
como farsa


[E]> a veja vai censurar a lista dos mais vendidos? vai ter que mudar o
nome para "lista dos livros mais vendidos indicados pela veja". Não se
combate mentira com mentira...

Percebam o interesse da população por qualquer "medicina alternativa". É
indicativo do descontentamento com o monopólio da medicina.


_\|/_

---------------------------------
Yahoo! Acesso Grátis - Internet rápida e grátis. Instale o discador do
Yahoo! agora.

[As partes desta mensagem que não continham texto foram removidas]



##### ##### #####

Para saber mais visite
http://www.ciencialist.hpg.ig.com.br


##### ##### ##### #####


Yahoo! Grupos, um serviço oferecido por:







------------------------------------------------------------------------------
Links do Yahoo! Grupos

a.. Para visitar o site do seu grupo na web, acesse:
http://br.groups.yahoo.com/group/ciencialist/

b.. Para sair deste grupo, envie um e-mail para:
ciencialist-unsubscribe@yahoogrupos.com.br

c.. O uso que você faz do Yahoo! Grupos está sujeito aos Termos do
Serviço do Yahoo!.



[As partes desta mensagem que não continham texto foram removidas]



##### ##### #####

Para saber mais visite
http://www.ciencialist.hpg.ig.com.br


##### ##### ##### #####
Links do Yahoo! Grupos












##### ##### #####

Para saber mais visite
http://www.ciencialist.hpg.ig.com.br


##### ##### ##### #####


Yahoo! Grupos, um serviço oferecido por:







------------------------------------------------------------------------------
Links do Yahoo! Grupos

a.. Para visitar o site do seu grupo na web, acesse:
http://br.groups.yahoo.com/group/ciencialist/

b.. Para sair deste grupo, envie um e-mail para:
ciencialist-unsubscribe@yahoogrupos.com.br

c.. O uso que você faz do Yahoo! Grupos está sujeito aos Termos do Serviço do Yahoo!.



[As partes desta mensagem que não continham texto foram removidas]



SUBJECT: Re: A Física é a igreja católica da Ciência
FROM: Maria Natália <grasdic@hotmail.com>
TO: ciencialist@yahoogrupos.com.br
DATE: 02/03/2005 03:32


Alvaro:
Só lendo om livro e eu ainda não o levantei.
Mas a expressão que postei foi de um célebre físico. Mas quando tiver
o livro na maõ virei escrever aqui mais sobre este livro.
abraços
Maria Natália


--- Em ciencialist@yahoogrupos.com.br, "Alvaro Augusto \(E\)"
<alvaro@e...> escreveu
> Hum, interessante... Em TODA a física? Isso inclui supertrings também?
>
> E as ondas gravitacionais? E os monopólos magnéticos? São coisas
compatíveis com as teorias mais modernas, mas fogem à verificação
experimental...
>
> [ ]s
>
> Alvaro Augusto
> ----- Original Message -----
> From: Ivan Carlos
> To: ciencialist@yahoogrupos.com.br
> Sent: Tuesday, March 01, 2005 1:58 AM
> Subject: Re: [ciencialist] A Física é a igreja católica da Ciência
>
>
> a pequena grande diferença é que a física provamos, mostramos e
repetimos os
> fatos quantas vezes forem necessárias a quem quizer ver
>
> ...já a crença... só crendo mesmo rsss
>
> Ivan "Doomer" Carlos
> Social Engineering Specialist
> -
> Cell.: +55 (11) 8112-0666
> icarlos@i...
> www.icarlos.net
> -
> MSN: icarlos@i...
> Y!M / Skype: ivandoomer
> --------------------------------------------------
>
>
> ----- Original Message -----
> From: "Maria Natália" <grasdic@h...>
> To: <ciencialist@yahoogrupos.com.br>
> Sent: Monday, February 28, 2005 10:23 PM
> Subject: [ciencialist] A Física é a igreja católica da Ciência
>
>
>
>
> Tá dito!
> Pois é. Não sabiam?
> Vem no livro: " As calças de Pitágoras, Deus, a Física e a guerra dos
> sexos" de Margaret Wertherm e claro é da GRADIVA.
> Alguém já leu? Encontrei-o à venda na feira do livro de minha escola e
> não resisti a provocar-vos.
> Quem o ler primeiro põe aqui o seu comentério?
> Uma abraço
>
>
>
> [As partes desta mensagem que não continham texto foram removidas]





SUBJECT: Re: No carro: com volt, ampère e ohm
FROM: Maria Natália <grasdic@hotmail.com>
TO: ciencialist@yahoogrupos.com.br
DATE: 02/03/2005 03:38


tem razão Sérgio:
aqui Ampério nem sei se pai poderá chamar seu filho recem nascido,
quando vai no registo.
ampério não existe e deve ser obra de menino que ligou duas
disciplinas diferentes: física e história. ampere e império romano por
exemplo. LOLLL
Tem crianças distraída que adormeçe na aula de física e acorda na
aula seguinte e que era História.
Um abraço
Maria Natália



--- Em ciencialist@yahoogrupos.com.br, "Sergio M. M. Taborda"
<sergiotaborda@t...> escreveu
> L.E.R.de Carvalho wrote:
>
> >
> >
> > Mas o que é amperagem? Designa intensidade de corrente elétrica,
> > medida em
> > amperes, plural de ampere, que o Aurélio ainda não acolheu, preferindo
> > indexá-lo em sua versão francesa, ''ampère''. Em Portugal, desde
1929,
> > foi
> > adotada a forma ''ampério'',
>
> Eu vivi em portugal , estudei electricidade, electronica e fisica em
> geral e nunca ouvi , nem li o termo amperio.
> Além disso esse termo não consta do dicionário de protugues de portugal
> que tenho.
> O nome ampère é o proprio nome do cara, mas na lingua portugesa o
acento
> não é necessário. Assim, o nome do cara é Ampère , mas o nome da
unidade
> é ampere.
> Amperagem é uma palavra da giria electrica e portanto não deve ser
usada
> em texto ou peças de algum valor informativo (como reportagens
> jornalisticas)
>
>
> Sérgio Taborda
>
>
>
> --
> No virus found in this outgoing message.
> Checked by AVG Anti-Virus.
> Version: 7.0.300 / Virus Database: 266.5.7 - Release Date: 01-03-2005





SUBJECT: Fw: Poderia me ajudar ? Por favor responda o quanto antes .
FROM: "Luiz Ferraz Netto" <leobarretos@uol.com.br>
TO: "ciencialist" <ciencialist@yahoogrupos.com.br>
DATE: 02/03/2005 07:05

Alguém tem mais referências sobre o experimento (ou fenômeno)?
Isso interessa tb a mim!

O experimento em questão está em:

http://www.feiradeciencias.com.br/sala08/08_23.asp

[]'
===========================
Luiz Ferraz Netto [Léo]
leobarretos@uol.com.br
http://www.feiradeciencias.com.br
===========================
-----Mensagem Original-----
De: Everton
Para: leobarretos@uol.com.br
Enviada em: segunda-feira, 28 de fevereiro de 2005 16:35
Assunto: Poderia me ajudar ? Por favor responda o quanto antes .




Olá , meu nome é Daniela Kunzendorff Aureliano , aluna de engenharia civil da universidade anhembi morumbi e gostaria de saber se o senhor poderia me ajudar pois estou fazendo um trabalho baseado em seu site , mais especificamente sobre a experiência do genio da garrafa (contexto de termologia) , e preciso de alguns links ou títulos de assuntos que envolvem essa tão interessante experiência . Desde já agradeço .


Daniela Kunzendorff Aureliano






--------------------------------------------------------------------------------


No virus found in this incoming message.
Checked by AVG Anti-Virus.
Version: 7.0.300 / Virus Database: 266.5.0 - Release Date: 25/02/2005

----------

No virus found in this outgoing message.
Checked by AVG Anti-Virus.
Version: 7.0.300 / Virus Database: 266.5.0 - Release Date: 25/02/2005


[As partes desta mensagem que não continham texto foram removidas]



SUBJECT: Fw: Instrumentos b�sicos de um laborat�rio de qu�mica
FROM: "Luiz Ferraz Netto" <leobarretos@uol.com.br>
TO: "ciencialist" <ciencialist@yahoogrupos.com.br>
DATE: 02/03/2005 07:07

Cuimicos e biocuimicos ........

[]'
Léo agradece a assistência .........
===========================
Luiz Ferraz Netto [Léo]
leobarretos@uol.com.br
http://www.feiradeciencias.com.br
===========================
-----Mensagem Original-----
De: Fernando
Para: leobarretos@uol.com.br
Enviada em: segunda-feira, 28 de fevereiro de 2005 09:23
Assunto: Instrumentos básicos de um laboratório de química


Nas dúvidas experimentais, por gentileza coloque aqui o endereço da página, isso facilita o confronto. Agradeço. Meu nome é LUIZ FERRAZ NETTO, meu apelido é LÉO e moro em BARRETOS; dai vem meu e-mail: leobarretos@uol.com.br.

Olá Léo...

Sou Silvia Costa, de São Paulo.

Estou fazendo técnico em nutrição e dietética, e preciso, para a aula de bioquímica, fazer um trabalho sobre todos os instrumentos (vidraria, porcelana, ferro e outros) de um laboratório de química.

No meu trabalho preciso colocar o nome, a função e se possível a foto ou desenho do instrumento.

Já procurei em alguns livros e pela net...mas, é muito difícil. Preciso de sua ajuda...

Meu e-mail de contato é: silviarechecosta@yahoo.com.br

Desde já te agradeço,

Um abraço

Silvia





--------------------------------------------------------------------------------


No virus found in this incoming message.
Checked by AVG Anti-Virus.
Version: 7.0.300 / Virus Database: 266.5.0 - Release Date: 25/02/2005

----------

No virus found in this outgoing message.
Checked by AVG Anti-Virus.
Version: 7.0.300 / Virus Database: 266.5.0 - Release Date: 25/02/2005


[As partes desta mensagem que não continham texto foram removidas]



SUBJECT: Re: RES: RN: Veja denuncia "medicina natural de A a Z" como farsa
FROM: "Rodrigo Marques" <rodmarq72@yahoo.com.br>
TO: ciencialist@yahoogrupos.com.br
DATE: 02/03/2005 08:15


Valeu pela dica!! Fui até lá e copiei o texto e já estou
divulgando...
--- Em ciencialist@yahoogrupos.com.br, Luis Brudna <luisbrudna@g...>
escreveu
> O texto completo está disponível no blog do Mori
>
> http://www.liquito.blogger.com.br/2005_02_01_archive.html#35393056
>
> Até
> Luís Brudna
>
>
> On Tue, 01 Mar 2005 19:06:01 -0000, Rodrigo Marques
> <rodmarq72@y...> wrote:
> >
> >
> > Ei pessoal, alguém consegue o texto da Veja a esse respeito que
eu
> > quero mandar pra uns amigos? É que eu não sou assinante nem
comprei
> > o exemplar em banca...(na lista da STRBrasil colocaram mas os
> > acentos não saíram).
> >





SUBJECT: Re: Veja denuncia "medicina natural de A a Z" como farsa
FROM: "rmtakata" <rmtakata@altavista.net>
TO: ciencialist@yahoogrupos.com.br
DATE: 02/03/2005 12:38


--- Em ciencialist@yahoogrupos.com.br, "Silvio" <scordeiro@t...>
> A censura é sempre perniciosa, injustificável, digna de cristãos

So' q. nao eh uma censura de um milico sentado na redacao e dizendo
pra revista o q. pode ou nao ser publicado. Eh a propria revista
decidindo por ela o q. vale ou nao vale a pena ela mesma publicar - e
isso eh feito todos os dias por todos os veiculos.

Digamos q. se chame esse autopoliciamento de censura tb. Nao devemos
confundir isso com a censura imposta por um orgao governamental sobre
a imprensa, apenas porq. damos o mesmo nome 'as duas coisas.

Seria algo equivalente a chamar de discriminacao o fato de reservarmos
vagas no estacionamento para pessoas paraplegicas e dizer q. toda
forma de discriminacao eh perniciosa e q. tal reserva nao deveria ser
permitida.

Ninguem eh perfeito
Eu sou um ninguem
--------------------
Logo sou perfeito

O erro de raciocinio eh rapidamente exposto pelo fato de notarmos q.
na premissa um 'ninguem' tem um significado diferente de 'ninguem' na
premissa dois. Assim a conclusao esta' errada.

Toda censura eh perniciosa
O ato da Veja eh censura
--------------------------
Logo o ato da Veja eh perniciosa

As censuras na premissa maior e na menor sao iguais?

Outro ponto: declive escorregadio. Dizer q. do ato de retirar a mencao
sobre o livro na lista dos mais vendidos vai para a ditadura
intelectual... Primeiro q. nao ha' nenhuma necessidade de haver, a
partir disso, um escrutinio semanal com o respectivo carimbo
'imprimatur'. Trata-se de um caso em q. claramente estamos diante de
um livro danoso 'a saude publica (nao ha' julgamento moral ou politico
necessario aqui) - alias, a Anvisa esta' marcando passo para retirar o
livro do mercado. (Paulo Coelho nao da' receitas de cura de cancer ou
de erisipela.) Segundo, q. mesmo q. fosse um habito semanal da
revista, nem de longe isso iria se caracterizar como ditadura
intelectual: Veja nao publica textos elogiosos ao governo Lula, eh seu
posicionamento editorial. Chato? Para os esquerdistas talvez. Mas
ditadura? Nem de longe. Eh o direito do veiculo de imprensa 'a sua
opiniao - dentro dos limites legais.

[]s,

Roberto Takata





SUBJECT: Veja denuncia "medicina natural de A a Z" como farsa
FROM: "L.E.R.de Carvalho" <lecarvalho@infolink.com.br>
TO: ciencialist@yahoogrupos.com.br
DATE: 02/03/2005 13:04


>So' q. nao eh uma censura de um milico sentado na redacao e dizendo
>pra revista o q. pode ou nao ser publicado. Eh a propria revista
>decidindo por ela o q. vale ou nao vale a pena ela mesma publicar - e
>isso eh feito todos os dias por todos os veiculos.




Negativo.
Ela pode não publicar a lista.

Mas não pode modificar a lista.

Então a gente também vai poder excluir a VEJA da lista das revistas mais
vendidas, porque ela não tem moral nenhuma, do meu ponto de vista.

Assim não dá.

Ela, VEJA, também publica um monte de imbecilidades sobre Nutrição e Saúde.

L.E.

[As partes desta mensagem que não continham texto foram removidas]



SUBJECT: Re: Veja denuncia "medicina natural de A a Z" como farsa
FROM: "rmtakata" <rmtakata@altavista.net>
TO: ciencialist@yahoogrupos.com.br
DATE: 02/03/2005 13:22


--- Em ciencialist@yahoogrupos.com.br, "L.E.R.de Carvalho"
> Negativo.
> Ela pode não publicar a lista.
>
> Mas não pode modificar a lista.

Ela avisou q. modificou a lista.

> Então a gente também vai poder excluir a VEJA da lista das revistas
> mais vendidas, porque ela não tem moral nenhuma, do meu ponto de
> vista.

Se vc fizer a sua lista com certeza podera'.

> Ela, VEJA, também publica um monte de imbecilidades sobre Nutrição e
> Saúde.

Sim, publica um monte de bobagens. Mas nada q. se compare 'a "Medicina
Alternativa de A a Z" ou "Cancer tem Cura" e literaturas do genero.

[]s,

Roberto Takata





SUBJECT: Veja denuncia "medicina natural de A a Z" como farsa
FROM: "L.E.R.de Carvalho" <lecarvalho@infolink.com.br>
TO: ciencialist@yahoogrupos.com.br
DATE: 02/03/2005 13:29

At 13:22 2/3/2005, you wrote:

>--- Em ciencialist@yahoogrupos.com.br, "L.E.R.de Carvalho"
> > Negativo.
> > Ela pode não publicar a lista.
> >
> > Mas não pode modificar a lista.
>
>Ela avisou q. modificou a lista.


Não basta avisar uma vez.
Ela tem que mudar o título da lista.
E destacar que não estão incluídos livros que ela JULGOU que não são bons.

E tem mais:
se ela acha que aquilo não é verdade...
que então inclua na lista de livros de FICÇAO.

L.E.










-
-
-
-
-
-
-

[As partes desta mensagem que não continham texto foram removidas]



SUBJECT: Re: Veja denuncia "medicina natural de A a Z" como farsa
FROM: "rmtakata" <rmtakata@altavista.net>
TO: ciencialist@yahoogrupos.com.br
DATE: 02/03/2005 13:45


--- Em ciencialist@yahoogrupos.com.br, "L.E.R.de Carvalho"
> >Ela avisou q. modificou a lista.
>
> Não basta avisar uma vez.

Talvez nao baste. Vc poderia pedir pra revista.

> E destacar que não estão incluídos livros que ela JULGOU que não são
> bons.

Por eqto seria livro no singular. Ela expurgou um titulo. Claro q. eh
um julgamento - bom e mau eh um julgamento.

A revista nao divulga titulos didaticos como os mais vendidos, tb nao
divulga a Biblia entre os mais vendidos. Mas por motivos diversos.

> E tem mais:
> se ela acha que aquilo não é verdade...
> que então inclua na lista de livros de FICÇAO.

Seria uma possibilidade, mas ficcao eh uma categoria quase
autodeclarativa.

[]s,

Roberto Takata





SUBJECT: Re: [ciencialist] Re: Veja denuncia "medicina natural de A a Z" como farsa
FROM: Victor Pimentel Nunes <victor2002@gmail.com>
TO: ciencialist@yahoogrupos.com.br
DATE: 02/03/2005 15:01

> > Não basta avisar uma vez.
>
> Talvez nao baste. Vc poderia pedir pra revista.


Eu pedi. Agora espero a resposta da revista.


> > E destacar que não estão incluídos livros que ela JULGOU que não são
> > bons.
>
> Por eqto seria livro no singular. Ela expurgou um titulo. Claro q. eh
> um julgamento - bom e mau eh um julgamento.


Um livro que você leu a nota. E as revistas anteriores que você não
tenha lido inteiras para saber se continha uma notinha no meio delas ?

Continuo achando que enquanto o livro fosse um dos mais vendidos, uma
notinha na página da listagem não iria mal.

Saudações
Victor

PS: Chegou uma resposta automática de VEJA. Assim que chegar a
resposta para valer eu posto aqui.


SUBJECT: Re: Veja denuncia "medicina natural de A a Z" como farsa
FROM: "rmtakata" <rmtakata@altavista.net>
TO: ciencialist@yahoogrupos.com.br
DATE: 02/03/2005 15:22


--- Em ciencialist@yahoogrupos.com.br, Victor Pimentel Nunes
> Um livro que você leu a nota. E as revistas anteriores que você não
> tenha lido inteiras para saber se continha uma notinha no meio
> delas?

Vc tem registro disso?

> Continuo achando que enquanto o livro fosse um dos mais vendidos,
> uma notinha na página da listagem não iria mal.

Vc acha q. nao. A Veja acha q. sim.

Algumas alternativas para q. sua opiniao prevalesca na Veja:
- convencer o staff editorial da revista em conversa direta;
- convencer um grande numero de leitores para pressionar a revista
para rever seu posicionamento;
- comprar a Ed. Abril;
- virar o editor-chefe (ou eh chefe de redacao?) da Veja;
- fazer um protesto-monstro (passeata? buzinaco?) na frente da Ed. Abril;
- virar amigo influente do Roberto Civita;
- virar ditador do Brasil e botar um milico na redacao para garantir
q. nao havera' censura;

[]s,

Roberto Takata





SUBJECT: carbono 14...
FROM: "E m i l i a n o C h e m e l l o" <chemelloe@yahoo.com.br>
TO: <ciencialist@yahoogrupos.com.br>, <quimica-qaw@yahoogrupos.com.br>
DATE: 02/03/2005 16:41

Caros colegas,

Quem pode dar um suporte a nossa amiga Rosi? Tem no site do NAEQ um artigo que fala sobre a técnica do carbono 14 (http://www.ucs.br/ccet/defq/naeq/material_didatico/textos_interativos_29.htm) e que pode servir como leitura base para responder as perguntas da moça.


Aguardo sugestões,

Abraços do
Emiliano Chemello
---
Olá!
Sou estudante do 4o ano de jornalismo do Cesumar - Centro Universitário de Maringá e estou fazendo uma matéria sobre o Carbono 14. Gostaria de fazer algumas perguntas bastante triviais, mas que precisam ser respondidas para o entendimento da matéria. A linguagem tem de ser simples, pois a revista que fazemos é destinada a um público geral.

O desconhecido carbono 14 é a peça chave para se descobrir a idade de muitos restos encontrados hoje na terra.
Que técnica é essa e como foi descoberto esse mecanismo e produto?
A que tipo de produtos é aplicada essa técnica?
Como é feito o procedimento?
Quanto tempo pode ser medido com essa técnica?
Quem são os profissionais que trabalham com isso e como são preparados?
Alguma técnica já entra para substituir o C-14?
E as amostras que tem mais de 70 mil anos como é medido o tempo?

Desde já agradeço a atenção.
Att. Rosi Ortega


[As partes desta mensagem que não continham texto foram removidas]



SUBJECT: Carbono 14 - respostas
FROM: "E m i l i a n o C h e m e l l o" <chemelloe@yahoo.com.br>
TO: <ciencialist@yahoogrupos.com.br>, <quimica-qaw@yahoogrupos.com.br>
DATE: 02/03/2005 17:20

Olá Pessoal,

Gostaria de vossas avaliações antes de mandar a msg para ao Rosi. Aguardo
comentários

============= reposta ==========================

Olá Rosi,

Abaixo, seguem as respostas das suas perguntas:

1) O desconhecido carbono 14 é a peça chave para se descobrir a idade de
muitos restos encontrados hoje na terra. Que técnica é essa e como foi
descoberto esse mecanismo e produto?
Imagine que um rapaz, aos 20 anos, dono de uma cabeleira invejável. E
que, para seu desespero, a partir daí, o destino comece a levar seus
cabelos. Aos 30 anos, resta metade dos fios dos 20. Aos 40, metade do que
tinha aos 30. E assim por diante. Quer dizer: observando seus cabelos,
sempre poderemos estimar sua idade. É mais ou menos assim que funciona a
datação por este método. O carbono 14 se forma na atmosfera da Terra
através da colisão entre raios cósmicos e átomos de nitrogênio 14. Os átomos
de carbono 14 se ligam ao oxigênio, formando o dióxido de carbono (14CO2)
que é absorvido pelos seres vivos. Uma vez que a planta ou animal morre, a
troca de carbono com a atmosfera é interrompida e, portanto, a quantidade de
carbono 14 só diminui com o tempo, já que ele é radiativo. Medindo a razão
entre o carbono 14 e o carbono 12 em um objeto, podemos então medir sua
idade.

2) A que tipo de produtos é aplicada essa técnica?
A datação por carbono 14 aplicaca-se principalmente na determinação de
idades de materiais orgânicos, como folhas e fósseis, mas também pode ser
utilizado para definir a data de outros materiais.

3) Como é feito o procedimento?
Em vez de medir a proporção de carbono-14, o método experimental se
baseia na atividade da amostra. Como vimos, ao se desintegrar, o carbono-14
emite uma partícula beta. Essa partícula beta, que não passa de nosso velho
conhecido, o elétron, pode ser detectada por um contador Geiger. Esse tipo
de contador todo mundo conhece do cinema: é aquele que emite um barulhinho
que fica intenso quando se aproxima de alguma coisa radioativa. Cada tique
daqueles é uma contagem. Na verdade, os contadores não precisam emitir
nenhum som, apenas mostram um número que indica quantas partículas foram
detectadas. Pois bem, verifica-se que 1 grama de carbono retirada de um ser
vivo ou da atmosfera provoca, em média, 13,6 contagens por minuto. É um
número pequeno de contagens mas suficiente para uma medida de boa precisão,
dentro de limites que veremos a seguir. Se uma amostra, tirada de um velho
pedaço de madeira, só dá 6,8 contagens por minuto, saberemos de imediato que
já se passaram 5730 anos (uma meia-vida t) desde que a árvore de onde veio
essa madeira foi cortada.

4) Quanto tempo pode ser medido com essa técnica?
Os dados são variados. Mas todos aproximam-se de idades iguais a 50 000
anos. A partir deste valor, a atividade é muito fraca, e o método de datação
por carbono 14 fica inviável.

5) Quem são os profissionais que trabalham com isso e como são preparados?
Sugiro que você entre em contato com quem trabalha na área: Grupo de
carbono-14 do Departamento de Física da UFC - Prof. Marlucia:
marlucia@fisica.ufc.br

6) Alguma técnica já entra para substituir o C-14?
O método do carbono-14 não serve para medir a idade de rochas pois se limita
a tempos de, no máximo, uns 50.000 anos. Para datar rochas os isótopos mais
convenientes são o urânio-238, o potássio-40 e e rubídio-87

7) E as amostras que tem mais de 70 mil anos como é medido o tempo?
São utilizados outros isótopos instáveis, como se pode ver na tabela abaixo:

ISÓTOPO INSTÁVEL ISÓTOPO ESTÁVEL MEIA-VIDA
Carbono-14 Nitrogênio-14
5730 anos
Potássio-40 Argônio-40
1,25 bilhões de anos
Urânio-238 Chumbo-206
4,5 bilhões de anos
Rubídio-87 Estrôncio-87
48,8 bilhões de anos

Até mesmo a idade do nosso planeta já foi calculada (método
urânio-238/chumbo-206): mais de 4,5 bilhões de anos.

================

Para compreender melhor o surgimento o carbono 14 na natureza, sugiro a
leitura do artigo publicado por mim no site do NAEQ:

Carbono 14, datando o passado
http://www.ucs.br/ccet/defq/naeq/material_didatico/textos_interativos_29.htm

Espero ter ajudado,

Abraço do
Emiliano Chemello
emiliano@quimica.net

============= reposta ==========================

Abraços do Emiliano Chemello




SUBJECT: Re: carbono 14...
FROM: "rmtakata" <rmtakata@altavista.net>
TO: ciencialist@yahoogrupos.com.br
DATE: 02/03/2005 17:22


Para uma explicacao basica:

http://science.howstuffworks.com/carbon-14.htm

[]s,

Roberto Takata





SUBJECT: Re: Carbono 14 - respostas
FROM: "rmtakata" <rmtakata@altavista.net>
TO: ciencialist@yahoogrupos.com.br
DATE: 02/03/2005 17:34


--- Em ciencialist@yahoogrupos.com.br, "E m i l i a n o C h e m e l
> conhecido, o elétron, pode ser detectada por um contador Geiger.

Antes disso tem um longo - e complicado - processo q. passa por
vaporizacao e uso de espectrometria de massa (por isso a datacao nao
eh um metodo trivial - embora seu principio seja muito simples - e eh
cara pra caramba).

http://www.llnl.gov/str/Knezovich.html

> 6) Alguma técnica já entra para substituir o C-14?
> Para datar rochas os isótopos mais
> convenientes são o urânio-238, o potássio-40 e e rubídio-87

Rochas igneas. As rochas sedimentares, e suas inclusoes, como os
fosseis, sao datados indiretamente.

> Até mesmo a idade do nosso planeta já foi calculada (método
> urânio-238/chumbo-206): mais de 4,5 bilhões de anos.

A idade da Terra foi calculada indiretamente: tto qto se sabe as
rochas mais antigas sao um pouco mais jovens do q. 4 bilhoes de anos
(devido ao retrabalhamento da crosta terrestre nos tempos anteriores
em funcao de imensos choques com bolidos celestes).

Mediu-se, por meio de isotopos instaveis, a idade de certos meteoritos
q. se acredita terem-se formado nos primordios do sistema solar. Sua
idade eh utilizada como a idade da Terra - e de outros corpos do sistema.

[]s,

Roberto Takata





SUBJECT: RE: Porque o oxigênio é invisível
FROM: "Ricardo Soares Vieira" <rickrsv@yahoo.com.br>
TO: ciencialist@yahoogrupos.com.br
DATE: 02/03/2005 19:51



Olá Maria Natália

Porque o oxigênio é invisível?

Eu entendi isso a pouco tempo... uma resposta aceitável, senão
conclusiva, sobre esse assunto eu encontrei em um livro antigo aqui
em casa:

Para responder, temos que apelar para teorias atômicas,
especificamente o modelo de Niels Bohr. Se você estuda física, deve
saber que apenas algumas órbitas são permitidas, e que os elétrons
só absolvem ou emitem radiação se a freqüência desta radiação
estiver em ressonância com a freqüência do elétron (calculada pela
equação de De Broglie - Einstein: f = h/E). Observe que quanto maior
for a freqüência da luz, maior é a energia de seu photon.

Podemos definir uma substancia transparente como aquela na qual a
luz à atravessa completamente, e uma opaca, àquela que absolve a
luz. Mas no nível atômico, um elemento só pode absolver luz se esta
tiver energia suficiente para levar o seu elétron do estado
fundamental (primeira órbita) à seguinte.

A par disso, encontramos o porque do oxigênio ser transparente: como
está no livro, a luz visível (cuja freqüência é relativamente baixa)
não tem energia suficiente para escitar o atomo de oxigenio, logo
ela o atravessa. Mas uma luz de freqüência maior será absolvida, e
logo, ela deixará de ser transparente para essa cor de luz. (vide
espectro do oxigênio).

Espero ter ajudado, acho que essa resposta dada naquele livro
(Physical Science Study Committee, parte 4 pg 213) explica o
fenômeno não?

Até Mais

Rick





SUBJECT: Re: [ciencialist] Re: Veja denuncia "medicina natural de A a Z" como farsa
FROM: Victor Pimentel Nunes <victor2002@gmail.com>
TO: ciencialist@yahoogrupos.com.br
DATE: 02/03/2005 19:54

"> Um livro que você leu a nota. E as revistas anteriores que você não
> tenha lido inteiras para saber se continha uma notinha no meio
> delas?

Vc tem registro disso?"


Takata, eu não quis dizer se você leu ou deixou de ler alguma VEJA.
Mas é no ponto do registro que é onde queria chegar.

Certamente quem começar a ler a VEJA a partir de semana que vem não
terá registro (e é bem provável que não faça idéia que o livro em
questão não consta na lista)

Outra alternativa foi a que fiz, enviei um e-mail a VEJA. Estou
aguardando a resposta (e a edição da próxima semana).

Saudações
Victor

PS: Sei que a alternativa que citei agora pode estar dentro da
alternativa citada por ti, de que bastante gente reclame. Mas quero
acreditar que o conteúdo da reclamação vá prevalecer sobre a
quantidade







On Wed, 02 Mar 2005 18:22:02 -0000, rmtakata <rmtakata@altavista.net> wrote:
>
> --- Em ciencialist@yahoogrupos.com.br, Victor Pimentel Nunes
> > Um livro que você leu a nota. E as revistas anteriores que você não
> > tenha lido inteiras para saber se continha uma notinha no meio
> > delas?
>
> Vc tem registro disso?
>
> > Continuo achando que enquanto o livro fosse um dos mais vendidos,
> > uma notinha na página da listagem não iria mal.
>
> Vc acha q. nao. A Veja acha q. sim.
>
> Algumas alternativas para q. sua opiniao prevalesca na Veja:
> - convencer o staff editorial da revista em conversa direta;
> - convencer um grande numero de leitores para pressionar a revista
> para rever seu posicionamento;
> - comprar a Ed. Abril;
> - virar o editor-chefe (ou eh chefe de redacao?) da Veja;
> - fazer um protesto-monstro (passeata? buzinaco?) na frente da Ed. Abril;
> - virar amigo influente do Roberto Civita;
> - virar ditador do Brasil e botar um milico na redacao para garantir
> q. nao havera' censura;
>
> []s,
>
> Roberto Takata
>
>
>
>
>
> ##### ##### #####
>
> Para saber mais visite
> http://www.ciencialist.hpg.ig.com.br
>
>
> ##### ##### ##### #####
>
>
> Yahoo! Grupos, um serviço oferecido por:
>
>
> Yahoo! Grupos, um serviço oferecido por:
>
>
>
> ________________________________
> Links do Yahoo! Grupos
> Para visitar o site do seu grupo na web, acesse:
> http://br.groups.yahoo.com/group/ciencialist/
>
> Para sair deste grupo, envie um e-mail para:
> ciencialist-unsubscribe@yahoogrupos.com.br
>
> O uso que você faz do Yahoo! Grupos está sujeito aos Termos do Serviço do
> Yahoo!.


SUBJECT: Folha Online: Estudo japonês desfaz ligação entre o autismo e vacina tríplice
FROM: Notícia por email <manuelbulcao@uol.com.br>
TO: ciencialist@yahoogrupos.com.br
CC: "Manuel Bulcão" <manuelbulcao@uol.com.br>
DATE: 02/03/2005 22:27

A seguinte notícia da Folha Online (www.folha.com.br) foi enviada para você por
Manuel Bulcão (manuelbulcao@uol.com.br).

Clique no link abaixo para ler o texto completo:

Estudo japonês desfaz ligação entre o autismo e vacina tríplice
http://www1.folha.uol.com.br/folha/ciencia/ult306u13025.shtml

Folha Online
http://www.folha.com.br/




SUBJECT: RES: [ciencialist] Re: Veja denuncia "medicina natural de A a Z" como farsa
FROM: "Anderson Porto" <acp722003@tudosobreplantas.com.br>
TO: <ciencialist@yahoogrupos.com.br>
DATE: 02/03/2005 23:10

Sinceramente?

Você quer mandar numa redação de uma editora? Aff... Dizer que você não
concorda que eles tirem ou coloquem uma porcaria de listagem fictícia?

Nós não mandamos nem na p. dos deputados e senadores, que foram eleitos
com nossos votos! Eles fazem o que querem, com o nosso dinheiro, e vocês
perdem tempo debatendo sobre uma "droga" de revista?

Anderson Porto
http://www.tudosobreplantas.com.br


-----Mensagem original-----
De: Victor Pimentel Nunes [mailto:victor2002@gmail.com]
Enviada em: quarta-feira, 2 de março de 2005 19:54
Para: ciencialist@yahoogrupos.com.br
Assunto: Re: [ciencialist] Re: Veja denuncia "medicina natural de A a Z"
como farsa



"> Um livro que você leu a nota. E as revistas anteriores que você não
> tenha lido inteiras para saber se continha uma notinha no meio
> delas?

Vc tem registro disso?"


Takata, eu não quis dizer se você leu ou deixou de ler alguma VEJA. Mas
é no ponto do registro que é onde queria chegar.

Certamente quem começar a ler a VEJA a partir de semana que vem não terá
registro (e é bem provável que não faça idéia que o livro em questão não
consta na lista)

Outra alternativa foi a que fiz, enviei um e-mail a VEJA. Estou
aguardando a resposta (e a edição da próxima semana).

Saudações
Victor

PS: Sei que a alternativa que citei agora pode estar dentro da
alternativa citada por ti, de que bastante gente reclame. Mas quero
acreditar que o conteúdo da reclamação vá prevalecer sobre a quantidade







On Wed, 02 Mar 2005 18:22:02 -0000, rmtakata <rmtakata@altavista.net>
wrote:
>
> --- Em ciencialist@yahoogrupos.com.br, Victor Pimentel Nunes
> > Um livro que você leu a nota. E as revistas anteriores que você não
> > tenha lido inteiras para saber se continha uma notinha no meio
> > delas?
>
> Vc tem registro disso?
>
> > Continuo achando que enquanto o livro fosse um dos mais vendidos,
> > uma notinha na página da listagem não iria mal.
>
> Vc acha q. nao. A Veja acha q. sim.
>
> Algumas alternativas para q. sua opiniao prevalesca na Veja:
> - convencer o staff editorial da revista em conversa direta;
> - convencer um grande numero de leitores para pressionar a revista
> para rever seu posicionamento;
> - comprar a Ed. Abril;
> - virar o editor-chefe (ou eh chefe de redacao?) da Veja;
> - fazer um protesto-monstro (passeata? buzinaco?) na frente da Ed.
> Abril;
> - virar amigo influente do Roberto Civita;
> - virar ditador do Brasil e botar um milico na redacao para garantir
> q. nao havera' censura;
>
> []s,
>
> Roberto Takata
>
>
>
>
>
> ##### ##### #####
>
> Para saber mais visite
> http://www.ciencialist.hpg.ig.com.br
>
>
> ##### ##### ##### #####
>
>
> Yahoo! Grupos, um serviço oferecido por:
>
>
> Yahoo! Grupos, um serviço oferecido por:
>
>
>
> ________________________________
> Links do Yahoo! Grupos
> Para visitar o site do seu grupo na web, acesse:
> http://br.groups.yahoo.com/group/ciencialist/
>
> Para sair deste grupo, envie um e-mail para:
> ciencialist-unsubscribe@yahoogrupos.com.br
>
> O uso que você faz do Yahoo! Grupos está sujeito aos Termos do Serviço

> do Yahoo!.


##### ##### #####

Para saber mais visite
http://www.ciencialist.hpg.ig.com.br


##### ##### ##### #####
Links do Yahoo! Grupos













SUBJECT: Re: Porque o oxigênio é invisível
FROM: Maria Natália <grasdic@hotmail.com>
TO: ciencialist@yahoogrupos.com.br
DATE: 03/03/2005 03:00



O PSSC!!!!!!!!!!!
Como foi possível esquecer essa bíblia? E tu a tens aí desde 196...
És português?
Tá bom para explicar a moçinho.
Obrigada
Maria Natália

--- Em ciencialist@yahoogrupos.com.br, "Ricardo Soares Vieira"
<rickrsv@y...> escreveu
>
>
> Olá Maria Natália
>
> Porque o oxigênio é invisível?
>
> Eu entendi isso a pouco tempo... uma resposta aceitável, senão
> conclusiva, sobre esse assunto eu encontrei em um livro antigo aqui
> em casa:
>
> Para responder, temos que apelar para teorias atômicas,
> especificamente o modelo de Niels Bohr. Se você estuda física, deve
> saber que apenas algumas órbitas são permitidas, e que os elétrons
> só absolvem ou emitem radiação se a freqüência desta radiação
> estiver em ressonância com a freqüência do elétron (calculada pela
> equação de De Broglie - Einstein: f = h/E). Observe que quanto maior
> for a freqüência da luz, maior é a energia de seu photon.
>
> Podemos definir uma substancia transparente como aquela na qual a
> luz à atravessa completamente, e uma opaca, àquela que absolve a
> luz. Mas no nível atômico, um elemento só pode absolver luz se esta
> tiver energia suficiente para levar o seu elétron do estado
> fundamental (primeira órbita) à seguinte.
>
> A par disso, encontramos o porque do oxigênio ser transparente: como
> está no livro, a luz visível (cuja freqüência é relativamente baixa)
> não tem energia suficiente para escitar o atomo de oxigenio, logo
> ela o atravessa. Mas uma luz de freqüência maior será absolvida, e
> logo, ela deixará de ser transparente para essa cor de luz. (vide
> espectro do oxigênio).
>
> Espero ter ajudado, acho que essa resposta dada naquele livro
> (Physical Science Study Committee, parte 4 pg 213) explica o
> fenômeno não?
>
> Até Mais
>
> Rick





SUBJECT: Re: Fw: Poderia me ajudar ? Por favor responda o quanto antes .
FROM: Maria Natália <grasdic@hotmail.com>
TO: ciencialist@yahoogrupos.com.br
DATE: 03/03/2005 03:04



Leo:
Génio na garrafa!? Não será antes génio na lâmpada de Aladino? Li já
qualquer coisa e devia ter sido na Física Divertida do Fiolhaias,
GRADIVA. Talvez seja fumaça que saia de um frasco.
Nã não tou com a magia neste instante. Tenho aqui uma par de claças...
Abraço, mestre (quase saía sua bençon)
Maria Natália

--- Em ciencialist@yahoogrupos.com.br, "Luiz Ferraz Netto"
<leobarretos@u...> escreveu
> Alguém tem mais referências sobre o experimento (ou fenômeno)?
> Isso interessa tb a mim!
>
> O experimento em questão está em:
>
> http://www.feiradeciencias.com.br/sala08/08_23.asp
>
> []'
> ===========================
> Luiz Ferraz Netto [Léo]
> leobarretos@u...
> http://www.feiradeciencias.com.br
> ===========================
> -----Mensagem Original-----
> De: Everton
> Para: leobarretos@u...
> Enviada em: segunda-feira, 28 de fevereiro de 2005 16:35
> Assunto: Poderia me ajudar ? Por favor responda o quanto antes .
>
>
>
>
> Olá , meu nome é Daniela Kunzendorff Aureliano , aluna de
engenharia civil da universidade anhembi morumbi e gostaria de saber
se o senhor poderia me ajudar pois estou fazendo um trabalho baseado
em seu site , mais especificamente sobre a experiência do genio da
garrafa (contexto de termologia) , e preciso de alguns links ou
títulos de assuntos que envolvem essa tão interessante experiência .
Desde já agradeço .
>
>
> Daniela Kunzendorff Aureliano
>
>
>
>
>
>
>
--------------------------------------------------------------------------------
>
>
> No virus found in this incoming message.
> Checked by AVG Anti-Virus.
> Version: 7.0.300 / Virus Database: 266.5.0 - Release Date: 25/02/2005
>
> ----------
>
> No virus found in this outgoing message.
> Checked by AVG Anti-Virus.
> Version: 7.0.300 / Virus Database: 266.5.0 - Release Date: 25/02/2005
>
>
> [As partes desta mensagem que não continham texto foram removidas]





SUBJECT: Já vos falei das calças do Pitágoras?
FROM: Maria Natália <grasdic@hotmail.com>
TO: ciencialist@yahoogrupos.com.br
DATE: 03/03/2005 03:12


Anda aqui a moda das calças desse homem tão parecido a jesus pois diz
a lenda que nasceu de uma virgem tb.
Mas queria só falar-vos do Livro da historiadora Margareth Wertheim
(252 páginas, 18 euros) GRADIVA

""Deus, as mulheres e a Nova Física

A INTERACÇÃO ENTRE A CIÊNCIA NEWTONIANA E A TEOLOGIA ANGLIANA
(Norbertaaaaaa?) NA primeira metade do século XVIII ocorreu a muitos
níveis. Até à sua morte em 1727, o próprio Newton foi uma força que
ajudoua formar e estabelecer estarelação. Da nossa perspectiva actual,
um dos aspectos mais bizarros dessa relação foi o grau ao qual a nova
física foi chamada a defender o literalismo biblíco. Aqui, Newton
liderou o caminho, argumentando que à luz da nova ciência, a história
dos seis dias da Criação podia tomar-se literalmente. Com o peso
imenso da sua autoridade, assinalou que se a Terra não tivesse
começado a girar até ao 3º dia, então os primeiros "dois"dias podiam
ser tão longos quanto o desejássemos, desse modo dando a Deus tempo de
sobra para fazer tudo quanto a bíblia afirmava. Em tais esforços não
estava de modo algum sozinho. O potencial da nova física paravfornecer
corroboração literal de eventos bíblicos foi também rapidamente
apreendido por William Whiston, sucessor deNewton na Cadeira Lucasiana
de Matemática em Cambridge (a posição agora detida por Stephen
Hawking).Whiston aplicou-se a mostrar que um cometa particular que
observara podia ter sido responsável pelo começo do grande dilúvio
bíblico. Usando as leis da gravidade e movimento de Newton, calculou a
traject´ria do cometa para trás no tempo e afirmou que ele teria
estado no lugar e altura certos para desencadear o dilúvio . Longe de
tornar a religião redundante, a nova física podia servir para
alimentar as chamas até da fé mais literal.""

E foi um bocado da obra das calças do dito cujo
Mas nesta obra também se fala na semelhança da vida de Pitágoras com
um tal jesus no capítulo qu explica o porquê das calças de Pitágoras e
não as de Lula ou Max

Estamos falando de mulheres pô e de dia 8 de Março

Vamos então ver o porquê da mulher: "Sendo a física a responsável pela
visão que temos do universo e do lugar que ocupamos nele, é urgente,
defende a autora, em as ditas cujas calças, que o Homeme Matemático
abdique de um dos derradeiros redutos da misoginia na sociedade
ocidental e abrace a parceria da Mulher Matemática."
Há ainda o capítulo 8 do Santo Cientíco onde se fala da Marie Curie e
do Pierre...portanto estás a perceber que a autora sendo historiadora
está a levar-nos até ao sec XX onde se dá a ascenção da Mulher Matemática
Repara neste posfacio" Porque razão há tantos livros sobre física com
a palavra "Deus" no tútulo? Contestando a ideia comum de que a
religião e a ciência são velhos inimigos, a historiadora argumenta em
as Calças...que a física moderna, mau grado a sua aparente
objectividade, sempre foi de facto uma actividade de inspiração
religiosa--uma ciência baseada na concepção de Deus como Criador
matemático divino"
"Procedendo a uma revisão feminista da história da física no
Ocidente--da sua incepção nos números transcendentes de Pitágoras à
actual obsessão dos seus praticantes com as "Teorias de Tudo"--a
autora, mostra como esta disciplina que transformaou o mundo tem sido
até hoje um feudo de homens professando variantes de fé no mnoteísmo
cristão, os quais criaram em redor da física uma cultura patriarcal
que ao longo dos séculos impediu as mulheres de participarem no
desenvolvimento científico"
E por hoje de calças basta até pq existem outras calças que já não
estão na lista e vão delirar com estes textículos.
Boa noite. Porque a diferença de horas agora é três

metafíscos de plantão que pensais deste livro de história da Física
assim por estes bocadillos? Já leram? Bem eu agora vou ter de parar
porque estamos em fim de 2º período
Maraia Natália






SUBJECT: Fw: Incidência do Raio Laser
FROM: "Luiz Ferraz Netto" <leobarretos@uol.com.br>
TO: "ciencialist" <ciencialist@yahoogrupos.com.br>
DATE: 03/03/2005 07:47

Quem se habilita?
[]'
===========================
Luiz Ferraz Netto [Léo]
leobarretos@uol.com.br
http://www.feiradeciencias.com.br
===========================
-----Mensagem Original-----
De: "José luiz de Andrade Maciel" <macielbrandao@pop.com.br>
Para: <leobarretos@uol.com.br>
Cc: <jlamaciel@uol.com.br>
Enviada em: terça-feira, 1 de março de 2005 16:13
Assunto: Incidência do Raio Laser




Prezado professor

Já que nos é dado a oportunidade de formular perguntas, venho por esse meio
solicitar-lhe a seguinte pergunta: qual o máximo de tempo que um paciente pode
ser submetido á exposição do Laser?
A pergunta tem razão de ser: sou Odontólogo e ao fazer aplicações com o
aparelho, me preocupa o tempo de aplicação pois quero atingir o limite sem,
evidentemente, prejudicar o cliente.
Sem mais e no aguardo de vossa providências,
Atenciosamente,
José Luiz de Andrade Maciel
macielbrandao@pop.com.br
jlamaciel@iol.com.br


--
No virus found in this incoming message.
Checked by AVG Anti-Virus.
Version: 7.0.300 / Virus Database: 266.5.0 - Release Date: 25/02/2005




--
No virus found in this outgoing message.
Checked by AVG Anti-Virus.
Version: 7.0.300 / Virus Database: 266.5.0 - Release Date: 25/02/2005



SUBJECT: Re: Fw: Incidência do Raio Laser
FROM: "rmtakata" <rmtakata@altavista.net>
TO: ciencialist@yahoogrupos.com.br
DATE: 03/03/2005 09:01


Depende do tecido e da intensidade do laser.

Essa informacoes deveriam estar no manual do aparelho.

Mas com certeza ele obtera' informacoes detalhadas com a Sociedade de
Odontologia a Laser:

http://www.odontolaser.com.br/

[]s,

Roberto Takata

--- Em ciencialist@yahoogrupos.com.br, "Luiz Ferraz Netto"
> Quem se habilita?
> ===========================
> Luiz Ferraz Netto [Léo]
> -----Mensagem Original-----
> De: "José luiz de Andrade Maciel" <macielbrandao@p...>
> A pergunta tem razão de ser: sou Odontólogo e ao fazer aplicações
> com o aparelho, me preocupa o tempo de aplicação pois quero atingir
> o limite sem, evidentemente, prejudicar o cliente.






SUBJECT: 91,5% das notas de R$ 1 têm cocaína
FROM: "E m i l i a n o C h e m e l l o" <chemelloe@yahoo.com.br>
TO: <ciencialist@yahoogrupos.com.br>
DATE: 03/03/2005 09:33

Pesquisa aponta que 91,5% das notas de R$ 1 têm cocaína.

Pesquisadores do Centro de Medicina Legal de Ribeirão Preto, ligados à
Faculdade de Medicina da USP, publicaram esta semana um estudo onde afirmam
que 91,5% das notas de R$ 1 em circulação no Brasil, apresentam vestígios de
cocaína.

O estudo analisou 47 cédulas em nove cidades: São Paulo, Santos (SP),
Ribeirão Preto (SP), Guaíra (SP), Rio de Janeiro, Brasília, Goiânia (GO),
Juiz de Fora (MG) e Londrina (PR). Desses municípios, apenas em Goiânia e
Londrina foram encontradas amostras sem contaminação, nas outras sete
cidades, o índice foi de 100%, sendo que Ribeirão Preto apresentou a maior
média de contaminação da pesquisa.

É claro que nem todas elas foram utilizadas para a inalação da droga, mas
quando uma nota utilizada entra em contato com outras, acaba
contaminando-as. Outras pesquisas semelhantes revelaram que nos Estados
Unidos, 92% das notas de US$ 1, apresentam resquícios de cocaína e no Reino
Unido, esse número cai para 60%.

A polícia brasileira e os especialistas em drogas estimam que de 2 a 4% da
população brasileira usa este entorpecente.

Texto: Cassiano Sampaio
Fonte: Redação Saúde em Movimento

Publicado em: 02/03/2005

Fonte:
http://www.saudeemmovimento.com.br/reportagem/noticia_frame.asp?cod_noticia=1748

[ ] 's do Emiliano Chemello
emiliano@quimica.net
http://www.quimica.net/emiliano
http://www.ucs.br/ccet/defq/naeq

" Rien ne se perd, rien ne se crée,
tout se transforme."

Antoine Laurent de Lavoisier (químico francês, 1743 - 1794)




SUBJECT: Novidades E Segunda chamada para revista / Call for papers
FROM: "Luis" <luis.alcides@he.com.br>
TO: ciencialist@yahoogrupos.com.br
DATE: 03/03/2005 10:08




Novidades
E
Segunda chamada para revista / Call for papers



Novidades

Seguindo as instruções do CNPq (disponíveis em:
<http://www.cnpq.br/areas/humanas_sociaisaplicadas/programa.htm>) o
Grupo Tchê Química esta instaurando o Conselho de Alto Nível do
Periódico Tchê Química, para melhor avaliar o conteúdo dos artigos
submetidos ao periódico.

O Conselho de Alto Nível terá a seguinte composição:
* Até 8 membros de nacionalidade Brasileira;
* Ao menos dois membros estrangeiros;

Quer se unir ao Conselho? Mais detalhes estão disponíveis na seção
"REVISTA" do site www.tchequimica.com ou no endereço a seguir:

http://www.deboni.he.com.br/tq/clics/clic.php3?url=www.deboni.he.com.br/contrato.pdf

Dúvidas? Escreva para nós! tchequimica@tchequimica.com



Segunda chamada para revista / Call for papers


O Grupo Tchê Química convida os Professores e Pesquisadores que
quiserem publicar seus artigos na quarta edição do Periódico Tchê
Química para enviar seus artigos para avaliação até 29 de Junho de 2005.

Os trabalhos enviados para avaliação devem obedecer as regras abaixo:

INSTRUÇÕES PARA PUBLICAÇÃO
INSTRUCTIONS FOR PUBLICATIONS

1. O "Periódico Tchê Química" publica artigos inéditos em todos os
campos da química e áreas afins; artigos de revisão bibliográfica de
assuntos específicos; comunicações científicas e resenhas de livros.

1. "Periódico Tchê Química" publishes original work in all areas of
chemistry; review papers on specific subjects; short communications
and book reviews.



2. Artigos originais incluem um resumo com preferência à língua
inglesa, sob título de "Abstract", para expor, de modo claro e
compreensível o conteúdo do trabalho, sem generalidades e detalhes de
metodologia.

2. Original papers should include an abstract with preference written
in English, under the title "Abstract", explaining clearly the
contents of the article. This abstract should not contain general and
experimental details.



3. Os trabalhos devem ser submetidos para o endereço eletrônico e
tchequimica@ tchequimica.com, em formato Sun Microsystems Inc.
OpenOffice.org® (preferencial) ou MS-Word for Windows®. O cabeçalho é
constituído pelo título do trabalho, nome(s) do(s) autor(es) e
respectivas instituições. As tabelas e figuras devem ser inseridas no
corpo do texto, em espaços adequados e perfeitamente visíveis. A
nomenclatura deve obedecer às normas da IUPAC.

3. Manuscript should be submetted to tchequimica@ tchequimica.com in
Sun Microsystems Inc. OpenOffice.org® (preference) or MS-Word for
Windows®. Manuscriptheadings must include
title, author name(s) and instution (s) where the work was done.
Tables and figures must be sent in the text body appropriate and
visible. Nomenclature should conform to IUPAC rules.



4. Citações bibliográficas no texto devem ser fietas por nome e ano
entre colchetes. Alista de referências bibliográficas, ao fim do
trabalho, deve ser organizada por ordem alfabética seqüencialmente
numerada, seguindo ao seguinte modelo:
1. CAVALCANTE, A. A. M., Rübensam, G., Picada, J. N., da Silva, E. G.,
Moreira, J. C. F., Henriques, J. A. P., Mutagenicity, antioxidant
potential and antimutagenic activitity against hydrogen peroxide of
cashew (Anacardium occidentale) apple juice and Cajuina, Envaronmental
and Molecular Mutagenesis 41:5 (2003) 360-369;

4. Bibliographic citations in the txt must be done by author name and
year between brackets. The bibliographic lists, at the endo of the
text, must be organized by alphabetical order and sequential
numeration, obeyng the following model: 1. CAVALCANTE, A. A. M.,
Rübensam, G., Picada, J. N., da Silva, E. G., Moreira, J. C. F.,
Henriques, J. A. P., Mutagenicity, antioxidant potential and
antimutagenic activitity against hydrogen peroxide of cashew
(Anacardium occidentale) apple juice and Cajuina, Envaronmental and
Molecular Mutagenesis 41:5 (2003) 360-369;



5. O Conselho Editorial poderá fazer alterações no texto do trabalho,
para adequá-lo a estas instruções.

5. The Editorial Board may perform modifications in the text to make
adequate to these rules.


Dúvidas? Escreva para ttchequimica@ tchequimica.com, no campo assunto
coloque o título "duvida sobre publicação". Por favor não envie
arquivos anexados nesta mensagem. Ou entre no nosso Yahoo Group em
http://br.groups.yahoo.com/group/tchequimica.


Gratos pela atenção, Grupo Tchê Química. www.tchequimica.com





SUBJECT: Re: [ciencialist] refrigerantes...
FROM: luiz rodrigo <baygonbr@yahoo.com.br>
TO: ciencialist@yahoogrupos.com.br
DATE: 03/03/2005 13:40

O CREA-PR tem um trabalho muito bom sobre os efeitos dos refrigerantes.

atenciosamente

Luiz

Silvio <scordeiro@terra.com.br> wrote:
Só espero que a resposta seja feita por químicos especialidados e não
ideólogos de má-fé.....

silvio.
-----Mensagem Original-----
De: "Emiliano Chemello - Yahoo Grupos" <chemelloe@yahoo.com.br>
Para: <ciencialist@yahoogrupos.com.br>; <naeq-ucs@yahoogrupos.com.br>;
<quimica-qaw@yahoogrupos.com.br>
Enviada em: domingo, 20 de fevereiro de 2005 13:03
Assunto: [ciencialist] refrigerantes...


>
> alguma referência sobre isso?
>
> Amplexos
>
> Emiliano Chemello
> ---
> Contato Naeq:
> Nome: Elizeu Batista
> Email: zzu2300@hotmail.com.br
> Assunto: Coca cola
> Mensagem: Gostaria saber quais os efeitos dos refrigerantes no nosso
> organismo, e em especial a coca cola. De todos que existem, há um pior ?
> Sempre tenho ouvido bastante coisa, mas muitas vezes sem fundamento
> científico . Obrigado
>
>
>
>
> ##### ##### #####
>
> Para saber mais visite
> http://www.ciencialist.hpg.ig.com.br
>
>
> ##### ##### ##### #####
> Links do Yahoo! Grupos
>
>
>
>
>
>
>
>
>
>



##### ##### #####

Para saber mais visite
http://www.ciencialist.hpg.ig.com.br


##### ##### ##### #####


Yahoo! Grupos, um serviço oferecido por:



















function SearchComboBox() { if (document.form_combo.keyword.value.length==0){ alert("Por favor, digite algo."); return false; }else { document.form_combo.action ="http://br.rd.yahoo.com/SIG=12ahh1kv8/M=264105.3931087.6562589.1588051/D=brclubs/S=2137111528:HM/EXP=1109818734/A=2361264/R=0/SIG=11uaou2jn/*http://www.bondfaro.com/bondfaro/in/combosearch_in.jsp?sk=11"; } return true;} [input] [input] [input]

---------------------------------
Links do Yahoo! Grupos

Para visitar o site do seu grupo na web, acesse:
http://br.groups.yahoo.com/group/ciencialist/

Para sair deste grupo, envie um e-mail para:
ciencialist-unsubscribe@yahoogrupos.com.br

O uso que você faz do Yahoo! Grupos está sujeito aos Termos do Serviço do Yahoo!.



---------------------------------
Yahoo! Acesso Grátis - Internet rápida e grátis. Instale o discador do Yahoo! agora.

[As partes desta mensagem que não continham texto foram removidas]



SUBJECT: Albert Einstein, um hipócrita???????????
FROM: "Cyberlander" <mrcyberlander@i12.com>
TO: <Undisclosed-Recipient:;>
DATE: 03/03/2005 15:15


Albert Einstein


Segundo aquele dito popular "Casa de ferreiro, espeto de pau", a
relação de Einstein com seus filhos não era condizente com suas
afirmações públicas, por exemplo, com aquela famosa citação:

"Es gibt weder große Entwicklungen noch wahre Fortschritte auf
dieser Erde, solange noch ein unglückliches Kind auf ihr lebt."

Tradução:
< Não existem grandes desenvolvimentos nem tão pouco verdadeiro
progresso neste mundo, enquanto ainda nele viver uma criança
infeliz. >

Seus filhos sofreram com a ausência de um relacionamento paterno de
sua parte. Nas cartas ao seu filho Hans Albert, o tema sempre era
fórmulas matemáticas ou assuntos correlatos, mas jamais temas
familiares e afetivos.

Einstein permaneceu um estranho a seus filhos. O contato com seu
filho Eduard foi totalmente cortado nos anos 20, quando ficou
constatado seu estado esquizofrênico. Ele escreveu
posteriormente: "TIVESSE EU CONHECIMENTO DE QUAL ESTADO ELE SE
ENCONTRARIA, ELE NUNCA TERIA VINDO A ESTE MUNDO."

O paradeiro de sua primeira filha, Lieserl, continua um mistério até
o presente. Einstein nunca se interessou em saber sobre seu bem-
estar.

A relação com suas enteadas, Ilse e Margot, filhas de sua segunda
esposa Elsa, é algo também bastante peculiar. Com Ilse, ele teve uma
atração física tão grande que chegou em 1918 a lhe pedir em
casamento, ao invés de sua mãe.

Aqui não posso deixar de traçar um paralelo com nosso conhecido
Woody Allen, aquele diretor de Hollywood que de fato se casou com a
enteada...
O que será que estes dois senhores têm em comum??

Já em relação às mulheres, vale aqui uma outra citação de Einstein:
"O comportamento ético de uma pessoa deveria se basear em
misericórdia, educação e relações sociais; uma base religiosa não é
necessária."

Mas parece que isto não valeu para sua esposa, pois de suas cartas
em alemão podemos ler as seguintes passagens:

"Não espere qualquer afeto de minha parte"

"Eu trato minha mulher como uma empregada que não pode ser demitida"

Em junho de 1914, Einstein escreveu a sua mulher Mileva Maric a fim
de colocar suas condições para permanecer com ela:

a. minhas roupas devem ser colocadas em ordem
b. sirva-me diariamente três refeições em meus aposentos
c. nosso relacionamento limita-se às obrigações sociais. Não existe
mais qualquer relacionamento pessoal entre nós.
d. Não espere qualquer afeto de minha parte
e. Assim que lhe ordenar a deixar o cômodo, seja meu quarto ou meu
escritório, você deve obedecer imediatamente. Sem protesto!

Um gênio? Um hipócrita? Tendo em vista que ele é admirado e
defendido por Oliveira, RM e coorte, a segunda pergunta tem mais
chance de se afirmar positivamente.

Alguém sabe algo sobre o italiano Olinto de Pretto e um tal caso de
plágio? :o)

W.


[ ]'s

D.C.



CYBERLANDER

Ama a realidade que constróis,
que nem a morte deterá teu voo · ·




[As partes desta mensagem que não continham texto foram removidas]



SUBJECT: Re: Albert Einstein, um hipócrita???????????
FROM: César A. K. Grossmann <cesarakg@bol.com.br>
TO: ciencialist@yahoogrupos.com.br
DATE: 03/03/2005 15:39


--- Em ciencialist@yahoogrupos.com.br, "Cyberlander"
<mrcyberlander@i...> escreveu
>
> Segundo aquele dito popular "Casa de ferreiro, espeto de pau", a
> relação de Einstein com seus filhos não era condizente com suas
> afirmações públicas, por exemplo, com aquela famosa citação:
>
> "Es gibt weder große Entwicklungen noch wahre Fortschritte auf
> dieser Erde, solange noch ein unglückliches Kind auf ihr lebt."
>
> Tradução:
> < Não existem grandes desenvolvimentos nem tão pouco verdadeiro
> progresso neste mundo, enquanto ainda nele viver uma criança
> infeliz. >

Não vejo relação entre uma coisa e outra, é possível explicar melhor?
A afirmativa dele refere-se a progresso e desenvolvimento, e, partindo
do ponto que nem o progresso nem o desenvolvimento contribuem para a
harmonia do lar, imagino que ele estava se referindo a miséria e
abandono de quem fica à margem do progresso e do desenvolvimento.

> Um gênio? Um hipócrita? Tendo em vista que ele é admirado e
> defendido por Oliveira, RM e coorte, a segunda pergunta tem mais
> chance de se afirmar positivamente.

Deixa eu ver, a prova de que ele era um hipócrita reside no fato de
que ele é admirado e defendido por um certo Oliveira, RM, etc. Isto
não parece muito lógico. Aliás, não parece nada lógico. "Olha só o
cara, ele tem que ser um hipócrita, já que beltrano está sempre
defendendo ele".

Ainda na mesma novela, suponhamos que Einstein fosse um hipócrita. Em
que isto afetaria a sua genialidade? Quer dizer, se fosse possível
provar que ele foi hipócrita (o que eu não estou convencido, a mim
parece apenas que ele não era do tipo expansivo e afetuoso), isto
implica que ele era um burro? O que uma coisa tem a ver com a outra?
Um gênio não pode ser hipócrita? São qualidades mutuamente exclusivas?

Apenas para finalizar, quer me parecer que toda a missiva se trata de
um 'ad hominem': uma maneira de tentar desacreditar as idéias de uma
pessoa atacando a pessoa, sem considerar os méritos das idéias. Entre
cientistas não deveria ser assim. Quando as teorias são conflitantes,
o conflito deve ser entregue ao campo das idéias. Pouco importa se o
proponente ou o atacante da teoria é um papa-hóstias ou um dissoluto
imoral.

[]s
--
César A. K. Grossmann
http://www.LinuxbyGrossmann.cjb.net/





SUBJECT: Re: [ciencialist] Re: Mulheres: Cozinha e na cama, já!!!!!!!!!!
FROM: "Silvio" <scordeiro@terra.com.br>
TO: <ciencialist@yahoogrupos.com.br>
DATE: 03/03/2005 15:57

E o capim em duas dezenass de variedades permitem que criemos o "boi ao
capim" o que é sanitáriamente correto embora devaste nossas florestas com
sua expansão descontrolada....

-----Mensagem Original-----
De: "Alvaro Augusto (E)" <alvaro@electraenergy.com.br>
Para: <ciencialist@yahoogrupos.com.br>
Enviada em: quarta-feira, 26 de janeiro de 2005 15:46
Assunto: Re: [ciencialist] Re: Mulheres: Cozinha e na cama, já!!!!!!!!!!



Bem, não sou profundo conhecedor do assunto, mas me lembro que as gramíneas
incluem a cana-de-açúcar, um produto que tem certa importância na economia
brasileira...

[ ]s

Alvaro Augusto

----- Original Message -----
From: Luiz Ferraz Netto
To: ciencialist@yahoogrupos.com.br
Sent: Wednesday, January 26, 2005 4:25 PM
Subject: Re: [ciencialist] Re: Mulheres: Cozinha e na cama, já!!!!!!!!!!


Tá bom, tá bom,

... vamos comer grama. :-)

>Não esquecer, no Brasil, a Johanna Dobereiner, agrônoma, responsável pela
descoberta de bactérias fixadoras de nitrogênio em gramíneas e uma das
responsáveis pelo fato de o Brasil ter o mais econômico plantio de soja do
mundo.

>>Rosalyn Franklin, Lynn Margulis, Caroline Herschell, Rosalyn Sussman
Yalow...





[As partes desta mensagem que não continham texto foram removidas]



##### ##### #####

Para saber mais visite
http://www.ciencialist.hpg.ig.com.br


##### ##### ##### #####
Links do Yahoo! Grupos











SUBJECT: Re: [ciencialist] O Manifesto de Porto Alegre
FROM: "Silvio" <scordeiro@terra.com.br>
TO: <ciencialist@yahoogrupos.com.br>
DATE: 03/03/2005 16:02

Lembro ao missivista que a lei da gravitação universal já foi abolida há
mais de 50 anos pelo Exmo. Sr. Carlos Thibau que tinha como objetivo de sua
campanha ligar a represa da Pampulha às praias de Guarapari. quando um
engenheiro e disse da existência dessa lei, que poderia imviabilizar o
projeto o Dr. Thibau imediatamente retrucou: " - Nós revoga ela!!" E assim
foi feito. E foi o oitavo dia.

sds.,

silvio.

-----Mensagem Original-----
De: "Alvaro Augusto (E)" <alvaro@electraenergy.com.br>
Para: <ciencialist@yahoogrupos.com.br>
Enviada em: terça-feira, 1 de fevereiro de 2005 13:59
Assunto: Re: [ciencialist] O Manifesto de Porto Alegre



Concordo com todos esses itens. Depois que eles forem implementados, sugiro
também o seguinte:

a) Revogar a lei da gravitação universal, pois facilitaria enormemente o
transporte de mercadorias e a construção civil;
b) Revogar a Lei de Joule, pois diminuiria drasticamente as perdas ôhmicas
em linhas de transmissão;
c) Revogar a lei da oferta e da procura, pois qualquer dispositivo que
permite o estabelecimento de preços é desumano;
d) Fazer com que a Jennifer Lopez venha morar aqui do meu lado (se não der
tempo, tudo bem...)

[ ]s

Alvaro Augusto

----- Original Message -----
From: Esteban Moreno
To: ciencialist@yahoogrupos.com.br
Sent: Tuesday, February 01, 2005 2:55 PM
Subject: [ciencialist] O Manifesto de Porto Alegre



Manifesto de Porto Alegre

Desde o primeiro Fórum Social Mundial realizado em Porto Alegre, em
janeiro
de 2001, o fenômeno dos fóruns sociais se estendeu a todos os
continentes,
inclusive nos níveis nacional e local. O Fórum favoreceu a emergência de
um
espaço público planetário da cidadania e de suas lutas, assim como
a
elaboração de propostas de políticas alternativas à tirania da
globalização
neoliberal impulsionada pelos mercados financeiros e as
transnacionais,
cujo braço armado é o poder imperial dos Estados Unidos.

Por sua diversidade, assim como pela solidariedade entre os atores e
os
movimentos sociais que o compõe, o movimento altermundista se
transformou
em uma força que já é levada muito em conta em todo o planeta. Entre
as
inumeráveis propostas que têm saído dos fóruns, um grande número
delas
conta sem dúvida com um amplo apoio junto aos movimentos sociais.
Nós,
signatários do Manifesto de Porto Alegre, que nos exprimimos a
título
estritamente pessoal, sem pretender, de modo algum, falar em nome do
Fórum,
identificamos doze destas propostas que, em conjunto, dão sentido
à
construção de outro mundo possível. Se fossem aplicadas, permitiriam que
a
cidadania começasse por fim a reapropriar-se de seu futuro.

Submetemos estes pontos fundamentais à apreciação dos atores e
movimentos
sociais de todos os países. São eles que, em todos os níveis -
mundial,
continental, nacional e local - poderão levar adiante os
combates
necessários para que se transformem em realidade. Nós não temos
nenhuma
ilusão sobre a real vontade dos governos e das instituições
internacionais
em aplicar espontaneamente estas propostas.



A) Outro Mundo Possível deve respeitar o direito à vida de todos os
seres
humanos, mediante novas regras econômicas. Para tanto, é necessário:


1) Anular a dívida pública dos países do Sul, que já foi paga várias
vezes
e que constitui, para os Estados credores, os estabelecimentos
financeiros
e as instituições financeiras internacionais, a melhor maneira de
submeter
a maior parte da humanidade à sua tutela e mantê-la na miséria;

2) Aplicar taxas internacionais às transações financeiras (especialmente
a
Taxa Tobin às transações especulativas de divisas); aos
investimentos
diretos no estrangeiro, aos lucros consolidados das transnacionais, à
venda
de armas e às atividades que emitem de forma substantiva gases que
produzem
o efeito estufa;

3) Desmantelar progressivamente todas as formas de paraísos
fiscais,
jurídicos e bancários, que nada mais são do que refúgios do
crime
organizado, da corrupção e de todos os tipos de tráficos, fraudes e
evasões
fiscais, operações delituosas de grandes empresas e inclusive de governos;

4) Cada habitante do planeta deve ter direito a um emprego, à
proteção
social e à aposentadoria, respeitando a igualdade entre homens e
mulheres,
sendo este um imperativo de políticas públicas nacionais e internacionais;

5) Promover todas as formas de comércio justo, rechaçando as regras
de
livre comércio da Organização Mundial do Comércio (OMC) e colocando
em
execução mecanismos que permitam, nos processos de produção de bens
e
serviços, dirigir-se progressivamente a um nivelamento por alto das
normas
sociais (tal como estão consignadas nas convenções da
Organização
Internacional do Trabalho, OIT) e ambientais. Excluir totalmente
a
educação, a saúde, os serviços sociais e a cultura do terreno de
aplicação
do Acordo Geral Sobre o Comércio e os Serviços (AGCS) da OMC. A
convenção
sobre a diversidade cultural, que atualmente está sendo negociada
na
Unesco, deve fazer prevalecer explicitamente o direito à cultura sobre
o
direito ao comércio;

6) Garantir o direito à soberania e segurança alimentar de cada
país,
mediante a promoção da agricultura campesina. Isso pressupõe a
eliminação
total dos subsídios à exportação dos produtos agrícolas, em primeiro
lugar
por parte dos Estados Unidos e da União Européia. Da mesma maneira,
cada
país ou conjunto de países deve poder decidir soberanamente sobre
a
proibição da produção e importação de organismos geneticamente
modificados
destinados à alimentação;

7) Proibir todo tipo de patenteamento do conhecimento e dos seres
vivos
(tanto humanos, como animais e vegetais) do mesmo modo que toda
a
privatização de bens comuns da humanidade, em particular a água;



B) Outro Mundo Possível deve encorajar a vida em comum em paz e
com
justiça, para toda a humanidade. Para tanto, é necessário:


8) Lutar, em primeiro lugar, por diferentes políticas públicas contra
todas
as formas de discriminação (sexismo, xenofobia, anti-semitismo e
racismo).
Reconhecer plenamente os direitos políticos, culturais e
ambientais
(incluindo o domínio de recursos naturais), dos povos indígenas;

9) Tomar medidas urgentes para colocar um fim à destruição do meio
ambiente
e à ameaça de mudanças climáticas graves devido ao efeito
estufa
resultante, em primeiro lugar, da proliferação do transporte individual
e
do uso excessivo de energias não renováveis. Começar a implementar
outro
modelo de desenvolvimento fundado na sobriedade energética e no
controle
democrático dos recursos naturais, em particular a água potável, em
uma
escala planetária;

10) Exigir o desmantelamento das bases militares estrangeiras e de
suas
tropas em todos os países, salvo quando estejam sob mandato expresso
da
Organização das Nações Unidas (ONU);



C) Outro Mundo Possível deve promover a democracia desde o plano local
até
o global. Para tanto, é necessário:


11) Garantir o direito à informação e o direito de informar dos
cidadãos
mediante legislações que: a) ponham fim à concentração de veículos
em
grupos de comunicação gigantes; b) garantam a autonomia dos
jornalistas
diante dos acionistas; c) favoreçam a imprensa sem fins lucrativos,
em
particular a dos meios alternativos e comunitários. O respeito
destes
direitos implica contra-poderes cidadãos, em particular na forma
de
observatórios nacionais e internacionais de meios de comunicação;

12) Reformar e democratizar em profundidade as organizações
internacionais,
entre elas a ONU, fazendo prevalecer nelas os direitos humanos,
econômicos,
sociais e culturais, em concordância com a Declaração Universal
dos
Direitos Humanos. Isso implica a incorporação do Banco Mundial, do
Fundo
Monetário Internacional e da Organização Mundial do Comércio ao sistema
das
Nações Unidas. Caso persistam as violações do direito internacional
por
parte dos Estados Unidos, transferir a sede da ONU de Nova York para
outro
país, preferencialmente do Sul.



Porto Alegre, 29 de janeiro de 2005



Aminata Traoré, Adolfo Pérez Esquivel, Eduardo Galeano, José
Saramago,
François Houtart, Boaventura de Sousa Santos, Armand Mattelart,
Roberto
Savio, Riccardo Petrella, Ignacio Ramonet, Bernard Cassen, Samir
Amin,
Atilio Boron, Samuel Ruiz Garcia, Tariq Ali, Frei Betto, Emir Sader,
Walden
Bello, Immanuel Wallerstein





[As partes desta mensagem que não continham texto foram removidas]



##### ##### #####

Para saber mais visite
http://www.ciencialist.hpg.ig.com.br


##### ##### ##### #####


Yahoo! Grupos, um serviço oferecido por:







------------------------------------------------------------------------------
Links do Yahoo! Grupos

a.. Para visitar o site do seu grupo na web, acesse:
http://br.groups.yahoo.com/group/ciencialist/

b.. Para sair deste grupo, envie um e-mail para:
ciencialist-unsubscribe@yahoogrupos.com.br

c.. O uso que você faz do Yahoo! Grupos está sujeito aos Termos do
Serviço do Yahoo!.



[As partes desta mensagem que não continham texto foram removidas]



##### ##### #####

Para saber mais visite
http://www.ciencialist.hpg.ig.com.br


##### ##### ##### #####
Links do Yahoo! Grupos












SUBJECT: Re: [ciencialist] Astrologia e Metereologia e top-posting (era : Zodiaco)
FROM: "Silvio" <scordeiro@terra.com.br>
TO: <ciencialist@yahoogrupos.com.br>
DATE: 03/03/2005 16:04

visite um dicionário ático....


-----Mensagem Original-----
De: "Alvaro Augusto (E)" <alvaro@electraenergy.com.br>
Para: <ciencialist@yahoogrupos.com.br>
Enviada em: quarta-feira, 9 de fevereiro de 2005 12:52
Assunto: Re: [ciencialist] Astrologia e Metereologia e top-posting (era :
Zodiaco)



Deve ser aquilo que os meteorologistas fazem quando estão de ressaca...

Bem, o próprio termo "meteorologia" é incorreto. Meteorologia deveria ser o
estudo dos meteoros.... Previsão do tempo deveria receber outro nome, tal
como "tempoprevisologia"...

[ ]s

Alvaro Augusto

----- Original Message -----
From: Luiz Ferraz Netto
To: ciencialist@yahoogrupos.com.br
Sent: Wednesday, February 09, 2005 6:43 AM
Subject: Re: [ciencialist] Astrologia e Metereologia e top-posting (era :
Zodiaco)


Bolas! O que é Metereologia?

[]'
Léo


--
No virus found in this outgoing message.
Checked by AVG Anti-Virus.
Version: 7.0.300 / Virus Database: 265.8.6 - Release Date: 07/02/2005



##### ##### #####

Para saber mais visite
http://www.ciencialist.hpg.ig.com.br


##### ##### ##### #####


Yahoo! Grupos, um serviço oferecido por:

São Paulo Rio de Janeiro Curitiba Porto Alegre Belo
Horizonte Brasília




------------------------------------------------------------------------------
Links do Yahoo! Grupos

a.. Para visitar o site do seu grupo na web, acesse:
http://br.groups.yahoo.com/group/ciencialist/

b.. Para sair deste grupo, envie um e-mail para:
ciencialist-unsubscribe@yahoogrupos.com.br

c.. O uso que você faz do Yahoo! Grupos está sujeito aos Termos do
Serviço do Yahoo!.



[As partes desta mensagem que não continham texto foram removidas]



##### ##### #####

Para saber mais visite
http://www.ciencialist.hpg.ig.com.br


##### ##### ##### #####
Links do Yahoo! Grupos












SUBJECT: Re: [ciencialist] Ainda há tempo
FROM: "Silvio" <scordeiro@terra.com.br>
TO: <ciencialist@yahoogrupos.com.br>
DATE: 03/03/2005 16:11

D. Álvaro.

Aposto que a vida humana hão resistirá à deveastação do bioma e não durará
1.000 anos o que não tem a menor importância para os outros seres mais bem
estruturados como as baratas, escorpiões, jacarés..... E, com o
desaparecimento dos humanos, a Mãe Natureza dará enorme suspiro de alívio,
arrependida de ter deixado vingar tamanho predador.....

sds.,
silvio.
-----Mensagem Original-----
De: "Alvaro Augusto (E)" <alvaro@electraenergy.com.br>
Para: <ciencialist@yahoogrupos.com.br>
Enviada em: domingo, 13 de fevereiro de 2005 12:23
Assunto: Re: [ciencialist] Ainda há tempo



Isso é exagero. A Terra já passou por coisa muita mais perturbadora que os
gases emitidos pela raça humana. Nem mesmo meteroros gigantes conseguiram
acabar com a vida por aqui. Assim, podemos dizer, com razoável grau de
certeza, que a vida ainda estará presente na Terra no próximo bilhão de
anos.

Mas nós não estaremos...

[ ]s

Alvaro Augusto

----- Original Message -----
From: José Renato
To: ciencialist@yahoogrupos.com.br
Sent: Friday, February 11, 2005 7:25 PM
Subject: [ciencialist] Ainda há tempo


Notícias Terra
Quinta, 10 de fevereiro de 2005, 14h45

ONG diz que restam de 10 a 20 anos para salvar a Terra

A comunidade internacional tem de 10 a 20 anos para tentar evitar uma
mudança climática devastadora na Terra, advertiu hoje a Rede Ação Clima
(RAC), integrada por 340 ONGs de proteção ao meio ambiente. A RAC fez o
alerta em Paris durante a abertura de uma série de atos por conta da entrada
em vigor do Protocolo de Kioto no dia 16 de fevereiro.
O acordo internacional, não assinado pelos Estados Unidos, prevê a redução
de 5,2% das emissões de gases que provocam o efeito estufa pelos países
industrializados até 2012. "Para impedir o aumento de dois graus da
temperatura média mundial até 2100 e estabelecer uma margem de segurança, é
preciso estabilizar a concentração de CO2 em 400 ppm (partes por milhão)
frente às 379 ppm atuais. Para alcançar esse objetivo, temos apenas de 10 a
20 anos", afirmou Laetitia Marez, porta-voz da RAC.

Segundo a RAC, as emissões mundiais de gases que provocam o efeito estufa
devem alcançar seu patamar mais elevado em 2020, no máximo e, a partir daí,
cair pela metade até 2050 para evitar efeitos devastadores para o planeta.

Segundo os especialistas, reunidos na semana passada em Exeter (Reino
Unido), um aumento de um grau nas temperaturas terá um impacto destruidor
nos corais e dois graus provocarão o derretimento da Groenlândia, o que
faria o nível do mar subir consideravelmente.

A curto prazo, são os países industrializados que devem fazer os maiores
esforços para adotar novas medidas restritivas a partir de 2012. A RAC
considera necessária a redução de 30% das emissões de CO2 até 2020 e de 75%
até 2050.

Para a ONG, países emergentes, como Arábia Saudita e Coréia do Sul,
deveriam reduzir suas emissões de gases a partir de 2013. Grandes países
como China e Índia deveriam, com a ajuda financeira e tecnológica dos países
do Norte, devem se comprometer desde já com a produção industrial e
energética limpa, segundo a RAC.

EFE

Agência Efe - Todos os direitos reservados. É proibido todo tipo de
reprodução sem autorização escrita da Agência Efe S/A.

< http://noticias.terra.com.br/ciencia/interna/0,,OI469817-EI299,00.html >


[As partes desta mensagem que não continham texto foram removidas]



[As partes desta mensagem que não continham texto foram removidas]



##### ##### #####

Para saber mais visite
http://www.ciencialist.hpg.ig.com.br


##### ##### ##### #####
Links do Yahoo! Grupos












SUBJECT: RE: [ciencialist] Re: Albert Einstein, um hipócrita???????????
FROM: "murilo filo" <avalanchedrive@hotmail.com>
TO: ciencialist@yahoogrupos.com.br
DATE: 03/03/2005 22:28

Concordo com o César ( e pela 1ª vez )!!!
Afinal êle - Einstein - não teria que apresentar nenhuma perfeição moral e
ética.
Não havia fundado nenhuma religião, como é o caso do Bispo Edir Macedo, que
muito teria me decepcionado se tivesse tal comportamento... :/
Se formos procurar, até o Gandhi seria algo decepcionante. É o ser humano,
rapazes! abr/M.

>From: César A. K. Grossmann <cesarakg@bol.com.br>
>Reply-To: ciencialist@yahoogrupos.com.br
>To: ciencialist@yahoogrupos.com.br
>Subject: [ciencialist] Re: Albert Einstein, um hipócrita???????????
>Date: Thu, 03 Mar 2005 18:39:00 -0000
>
>
>
>--- Em ciencialist@yahoogrupos.com.br, "Cyberlander"
><mrcyberlander@i...> escreveu
> >
> > Segundo aquele dito popular "Casa de ferreiro, espeto de pau", a
> > relação de Einstein com seus filhos não era condizente com suas
> > afirmações públicas, por exemplo, com aquela famosa citação:
> >
> > "Es gibt weder große Entwicklungen noch wahre Fortschritte auf
> > dieser Erde, solange noch ein unglückliches Kind auf ihr lebt."
> >
> > Tradução:
> > < Não existem grandes desenvolvimentos nem tão pouco verdadeiro
> > progresso neste mundo, enquanto ainda nele viver uma criança
> > infeliz. >
>
>Não vejo relação entre uma coisa e outra, é possível explicar melhor?
>A afirmativa dele refere-se a progresso e desenvolvimento, e, partindo
>do ponto que nem o progresso nem o desenvolvimento contribuem para a
>harmonia do lar, imagino que ele estava se referindo a miséria e
>abandono de quem fica à margem do progresso e do desenvolvimento.
>
> > Um gênio? Um hipócrita? Tendo em vista que ele é admirado e
> > defendido por Oliveira, RM e coorte, a segunda pergunta tem mais
> > chance de se afirmar positivamente.
>
>Deixa eu ver, a prova de que ele era um hipócrita reside no fato de
>que ele é admirado e defendido por um certo Oliveira, RM, etc. Isto
>não parece muito lógico. Aliás, não parece nada lógico. "Olha só o
>cara, ele tem que ser um hipócrita, já que beltrano está sempre
>defendendo ele".
>
>Ainda na mesma novela, suponhamos que Einstein fosse um hipócrita. Em
>que isto afetaria a sua genialidade? Quer dizer, se fosse possível
>provar que ele foi hipócrita (o que eu não estou convencido, a mim
>parece apenas que ele não era do tipo expansivo e afetuoso), isto
>implica que ele era um burro? O que uma coisa tem a ver com a outra?
>Um gênio não pode ser hipócrita? São qualidades mutuamente exclusivas?
>
>Apenas para finalizar, quer me parecer que toda a missiva se trata de
>um 'ad hominem': uma maneira de tentar desacreditar as idéias de uma
>pessoa atacando a pessoa, sem considerar os méritos das idéias. Entre
>cientistas não deveria ser assim. Quando as teorias são conflitantes,
>o conflito deve ser entregue ao campo das idéias. Pouco importa se o
>proponente ou o atacante da teoria é um papa-hóstias ou um dissoluto
>imoral.
>
>[]s
>--
>César A. K. Grossmann
>http://www.LinuxbyGrossmann.cjb.net/
>
>
>
>
>
>##### ##### #####
>
>Para saber mais visite
>http://www.ciencialist.hpg.ig.com.br
>
>
>##### ##### ##### #####
>Links do Yahoo! Grupos
>
>
>
>
>
>
>
>




SUBJECT: Re: [ciencialist] Albert Einstein, um hipócrita???????????
FROM: "Alvaro Augusto \(E\)" <alvaro@electraenergy.com.br>
TO: <ciencialist@yahoogrupos.com.br>
DATE: 03/03/2005 22:31

Caríssimos,

Albert Einstein foi o maior físico de sua época e possivelmente o maior físico de todos os tempos. Exigir que, além disso, ele tenha sido também um grande pai, um grande marido, um grande filho, é exigir demais de qualquer ser humano!

Eu me sinto mais confortável agora, depois da abertura dos arquivos de Einstein, quando sabemos que ele foi um homem normal em vários aspectos (considerando-se que nasceu no século XIX), do que me sentia na década de 80, quando toda biografia de Einstein que eu lia o considerava uma espécie de "grande santo judeu" (leiam, por exemplo, "Einstein - O Enigma do Universo", de Huberto Rohden).

Einstein era hipócrita? Bem, todos nós somos, pois, como mostrou outro judeu (Freud), não temos acesso a todo o conteúdo da nossa psique. Logo, não podemos conhecer toda a verdade sobre o que pensamos. Logo, não podemos agir sempre de acordo com o que pensamos...

[ ]s

Alvaro Augusto



----- Original Message -----
From: Cyberlander
To: Undisclosed-Recipient:;
Sent: Thursday, March 03, 2005 3:15 PM
Subject: [ciencialist] Albert Einstein, um hipócrita???????????



Albert Einstein


Segundo aquele dito popular "Casa de ferreiro, espeto de pau", a
relação de Einstein com seus filhos não era condizente com suas
afirmações públicas, por exemplo, com aquela famosa citação:

"Es gibt weder große Entwicklungen noch wahre Fortschritte auf
dieser Erde, solange noch ein unglückliches Kind auf ihr lebt."

Tradução:
< Não existem grandes desenvolvimentos nem tão pouco verdadeiro
progresso neste mundo, enquanto ainda nele viver uma criança
infeliz. >

Seus filhos sofreram com a ausência de um relacionamento paterno de
sua parte. Nas cartas ao seu filho Hans Albert, o tema sempre era
fórmulas matemáticas ou assuntos correlatos, mas jamais temas
familiares e afetivos.

Einstein permaneceu um estranho a seus filhos. O contato com seu
filho Eduard foi totalmente cortado nos anos 20, quando ficou
constatado seu estado esquizofrênico. Ele escreveu
posteriormente: "TIVESSE EU CONHECIMENTO DE QUAL ESTADO ELE SE
ENCONTRARIA, ELE NUNCA TERIA VINDO A ESTE MUNDO."

O paradeiro de sua primeira filha, Lieserl, continua um mistério até
o presente. Einstein nunca se interessou em saber sobre seu bem-
estar.

A relação com suas enteadas, Ilse e Margot, filhas de sua segunda
esposa Elsa, é algo também bastante peculiar. Com Ilse, ele teve uma
atração física tão grande que chegou em 1918 a lhe pedir em
casamento, ao invés de sua mãe.

Aqui não posso deixar de traçar um paralelo com nosso conhecido
Woody Allen, aquele diretor de Hollywood que de fato se casou com a
enteada...
O que será que estes dois senhores têm em comum??

Já em relação às mulheres, vale aqui uma outra citação de Einstein:
"O comportamento ético de uma pessoa deveria se basear em
misericórdia, educação e relações sociais; uma base religiosa não é
necessária."

Mas parece que isto não valeu para sua esposa, pois de suas cartas
em alemão podemos ler as seguintes passagens:

"Não espere qualquer afeto de minha parte"

"Eu trato minha mulher como uma empregada que não pode ser demitida"

Em junho de 1914, Einstein escreveu a sua mulher Mileva Maric a fim
de colocar suas condições para permanecer com ela:

a. minhas roupas devem ser colocadas em ordem
b. sirva-me diariamente três refeições em meus aposentos
c. nosso relacionamento limita-se às obrigações sociais. Não existe
mais qualquer relacionamento pessoal entre nós.
d. Não espere qualquer afeto de minha parte
e. Assim que lhe ordenar a deixar o cômodo, seja meu quarto ou meu
escritório, você deve obedecer imediatamente. Sem protesto!

Um gênio? Um hipócrita? Tendo em vista que ele é admirado e
defendido por Oliveira, RM e coorte, a segunda pergunta tem mais
chance de se afirmar positivamente.

Alguém sabe algo sobre o italiano Olinto de Pretto e um tal caso de
plágio? :o)

W.


[ ]'s

D.C.



CYBERLANDER

Ama a realidade que constróis,
que nem a morte deterá teu voo · ·




[As partes desta mensagem que não continham texto foram removidas]



##### ##### #####

Para saber mais visite
http://www.ciencialist.hpg.ig.com.br


##### ##### ##### #####


Yahoo! Grupos, um serviço oferecido por:







------------------------------------------------------------------------------
Links do Yahoo! Grupos

a.. Para visitar o site do seu grupo na web, acesse:
http://br.groups.yahoo.com/group/ciencialist/

b.. Para sair deste grupo, envie um e-mail para:
ciencialist-unsubscribe@yahoogrupos.com.br

c.. O uso que você faz do Yahoo! Grupos está sujeito aos Termos do Serviço do Yahoo!.



[As partes desta mensagem que não continham texto foram removidas]



SUBJECT: Re: [ciencialist] Re: Albert Einstein, um hipócrita???????????
FROM: JVictor <jvoneto@uol.com.br>
TO: ciencialist@yahoogrupos.com.br
DATE: 04/03/2005 07:32

Murilo,

Sei as motivações pessoais, e outras razões, para alguém ter
interêsses em mostrar certos aspectos meramente humanos de pessoas do
nível de Einstein. Mas não vou dar opiniões a respeito. Só digo o
seguinte: essas pessoas esquecem que quando apontam com um dedo, há
pelos 3 apontando, volta, para ele!
Hipocrisia das hipocrisias, diria um famosos hipócritas bíblicos, perto
da morte, que fez um gigante rir até morrer, e disseram que foi de uma
pedrada.
O César está correto. E você também, em concordar com ele.

Sds,

Victor.



murilo filo escreveu:

> Concordo com o César ( e pela 1ª vez )!!!
> Afinal êle - Einstein - não teria que apresentar nenhuma perfeição
> moral e
> ética.
> Não havia fundado nenhuma religião, como é o caso do Bispo Edir
> Macedo, que
> muito teria me decepcionado se tivesse tal comportamento... :/
> Se formos procurar, até o Gandhi seria algo decepcionante. É o ser
> humano,
> rapazes! abr/M.
>
> >From: César A. K. Grossmann <cesarakg@bol.com.br>
> >Reply-To: ciencialist@yahoogrupos.com.br
> >To: ciencialist@yahoogrupos.com.br
> >Subject: [ciencialist] Re: Albert Einstein, um hipócrita???????????
> >Date: Thu, 03 Mar 2005 18:39:00 -0000
> >
> >
> >
> >--- Em ciencialist@yahoogrupos.com.br, "Cyberlander"
> ><mrcyberlander@i...> escreveu
> > >
> > > Segundo aquele dito popular "Casa de ferreiro, espeto de pau", a
> > > relação de Einstein com seus filhos não era condizente com suas
> > > afirmações públicas, por exemplo, com aquela famosa citação:
> > >
> > > "Es gibt weder große Entwicklungen noch wahre Fortschritte auf
> > > dieser Erde, solange noch ein unglückliches Kind auf ihr lebt."
> > >
> > > Tradução:
> > > < Não existem grandes desenvolvimentos nem tão pouco verdadeiro
> > > progresso neste mundo, enquanto ainda nele viver uma criança
> > > infeliz. >
> >
> >Não vejo relação entre uma coisa e outra, é possível explicar melhor?
> >A afirmativa dele refere-se a progresso e desenvolvimento, e, partindo
> >do ponto que nem o progresso nem o desenvolvimento contribuem para a
> >harmonia do lar, imagino que ele estava se referindo a miséria e
> >abandono de quem fica à margem do progresso e do desenvolvimento.
> >
> > > Um gênio? Um hipócrita? Tendo em vista que ele é admirado e
> > > defendido por Oliveira, RM e coorte, a segunda pergunta tem mais
> > > chance de se afirmar positivamente.
> >
> >Deixa eu ver, a prova de que ele era um hipócrita reside no fato de
> >que ele é admirado e defendido por um certo Oliveira, RM, etc. Isto
> >não parece muito lógico. Aliás, não parece nada lógico. "Olha só o
> >cara, ele tem que ser um hipócrita, já que beltrano está sempre
> >defendendo ele".
> >
> >Ainda na mesma novela, suponhamos que Einstein fosse um hipócrita. Em
> >que isto afetaria a sua genialidade? Quer dizer, se fosse possível
> >provar que ele foi hipócrita (o que eu não estou convencido, a mim
> >parece apenas que ele não era do tipo expansivo e afetuoso), isto
> >implica que ele era um burro? O que uma coisa tem a ver com a outra?
> >Um gênio não pode ser hipócrita? São qualidades mutuamente exclusivas?
> >
> >Apenas para finalizar, quer me parecer que toda a missiva se trata de
> >um 'ad hominem': uma maneira de tentar desacreditar as idéias de uma
> >pessoa atacando a pessoa, sem considerar os méritos das idéias. Entre
> >cientistas não deveria ser assim. Quando as teorias são conflitantes,
> >o conflito deve ser entregue ao campo das idéias. Pouco importa se o
> >proponente ou o atacante da teoria é um papa-hóstias ou um dissoluto
> >imoral.
> >
> >[]s
> >--
> >César A. K. Grossmann
> >http://www.LinuxbyGrossmann.cjb.net/
> >
> >
> >
> >
> >
> >##### ##### #####
> >
> >Para saber mais visite
> >http://www.ciencialist.hpg.ig.com.br
> >
> >
> >##### ##### ##### #####
> >Links do Yahoo! Grupos
> >
> >
> >
> >
> >
> >
> >
> >
>
>
>
>
> ##### ##### #####
>
> Para saber mais visite
> http://www.ciencialist.hpg.ig.com.br
>
>
> ##### ##### ##### #####
>
>
> *Yahoo! Grupos, um serviço oferecido por:*
>
> *
> <http://br.rd.yahoo.com/SIG=12ama5o9l/M=264105.3931087.6562589.1588051/D=brclubs/S=2137111528:HM/EXP=1109986216/A=2361264/R=6/SIG=10v4acpp0/*http://br.shopping.yahoo.com/>*
>
>
>
> ------------------------------------------------------------------------
> *Links do Yahoo! Grupos*
>
> * Para visitar o site do seu grupo na web, acesse:
> http://br.groups.yahoo.com/group/ciencialist/
>
> * Para sair deste grupo, envie um e-mail para:
> ciencialist-unsubscribe@yahoogrupos.com.br
> <mailto:ciencialist-unsubscribe@yahoogrupos.com.br?subject=Unsubscribe>
>
> * O uso que você faz do Yahoo! Grupos está sujeito aos Termos do
> Serviço do Yahoo! <http://br.yahoo.com/info/utos.html>.
>
>
>
>
> __________ Informação do NOD32 1.1017 (20050302) __________
>
> Esta mensagem foi verificada pelo NOD32 Sistema Antivírus
> http://www.nod32.com.br




SUBJECT: Fw: �on H
FROM: "Luiz Ferraz Netto" <leobarretos@uol.com.br>
TO: "ciencialist" <ciencialist@yahoogrupos.com.br>
DATE: 04/03/2005 08:30

Não é química ... mas pode ser respondido por cuímicos :-))

[]'
===========================
Luiz Ferraz Netto [Léo]
leobarretos@uol.com.br
http://www.feiradeciencias.com.br
===========================
-----Mensagem Original-----
De: guiarte
Para: leobarretos
Enviada em: quinta-feira, 3 de março de 2005 20:46
Assunto: Íon H


Professor Leo,

Parabém por sua utilíssima página.

Gostaria que me esclarecesse uma dúvida:

Como se forma o íon H+, pois, como sabemos, ele apresenta apenas um próton e um elétron. Se perder seu único elétron, ficará apenas um próton; ou seja, uma partícula solta no espaço...

Será isso possível, ou a formação de tal íon é a partir do gás hidrogênio (H2).

Por favor, aguardo uma resposta que não tenho encontrado nos livros.

Grato,

Prof. Vilson


--------------------------------------------------------------------------------


Internal Virus Database is out-of-date.
Checked by AVG Anti-Virus.
Version: 7.0.300 / Virus Database: 266.5.0 - Release Date: 25/02/2005

----------

Internal Virus Database is out-of-date.
Checked by AVG Anti-Virus.
Version: 7.0.300 / Virus Database: 266.5.0 - Release Date: 25/02/2005


[As partes desta mensagem que não continham texto foram removidas]



SUBJECT: Fw: nascimento do atomo
FROM: "Luiz Ferraz Netto" <leobarretos@uol.com.br>
TO: "ciencialist" <ciencialist@yahoogrupos.com.br>
DATE: 04/03/2005 08:35

como nascem os átomos?
[]'
===========================
Luiz Ferraz Netto [Léo]
leobarretos@uol.com.br
http://www.feiradeciencias.com.br
===========================
-----Mensagem Original-----
De: Mauro Santiago
Para: leobarretos@uol.com.br
Enviada em: quinta-feira, 3 de março de 2005 20:21
Assunto: nascimento do atomo


procurei em muitos sites mas não encontrei uma resposta exata para a pergunta "Como nascem os diferentes tipos de átomos?"


--------------------------------------------------------------------------------


No virus found in this incoming message.
Checked by AVG Anti-Virus.
Version: 7.0.300 / Virus Database: 266.5.0 - Release Date: 25/02/2005

----------

Internal Virus Database is out-of-date.
Checked by AVG Anti-Virus.
Version: 7.0.300 / Virus Database: 266.5.0 - Release Date: 25/02/2005


[As partes desta mensagem que não continham texto foram removidas]



SUBJECT: Fw: Como surgiu a ci�ncia?
FROM: "Luiz Ferraz Netto" <leobarretos@uol.com.br>
TO: "ciencialist" <ciencialist@yahoogrupos.com.br>
DATE: 04/03/2005 08:35

Como começou a ciência?
[]'
===========================
Luiz Ferraz Netto [Léo]
leobarretos@uol.com.br
http://www.feiradeciencias.com.br
===========================
-----Mensagem Original-----
De: Talita
Para: leobarretos@uol.com.br
Enviada em: segunda-feira, 28 de fevereiro de 2005 08:31
Assunto: Como surgiu a ciência?


Olá meu nome é Ana,eu queria saber como começou a ciência?
Nas dúvidas experimentais, por gentileza coloque aqui o endereço da página, isso facilita o confronto. Agradeço. Meu nome é LUIZ FERRAZ NETTO, meu apelido é LÉO e moro em BARRETOS; dai vem meu e-mail: leobarretos@uol.com.br.


--------------------------------------------------------------------------------


No virus found in this incoming message.
Checked by AVG Anti-Virus.
Version: 7.0.300 / Virus Database: 266.5.0 - Release Date: 25/02/2005

----------

Internal Virus Database is out-of-date.
Checked by AVG Anti-Virus.
Version: 7.0.300 / Virus Database: 266.5.0 - Release Date: 25/02/2005


[As partes desta mensagem que não continham texto foram removidas]



SUBJECT: RE: [ciencialist] Fw: Como surgiu a ciência?
FROM: "Hugo Santos" <urano@netvisao.pt>
TO: <ciencialist@yahoogrupos.com.br>
DATE: 04/03/2005 09:23

Segundo a minha visão, a ciência começou no instante em que o homem obteve a
primeira centelha de consciência e se viu como parte íntegra do mundo que o
rodeava. Observação, definição e sistematização terão sido as primeiras
acções científicas que o primeiro hominídeo terá feito. Depois foi só
evoluir...

Hugo Santos

> -----Original Message-----
> From: Luiz Ferraz Netto [mailto:leobarretos@uol.com.br]
> Sent: sexta-feira, 4 de Março de 2005 11:36
> To: ciencialist
> Subject: [ciencialist] Fw: Como surgiu a ciência?
>
>
> Como começou a ciência?
> []'
> ===========================
> Luiz Ferraz Netto [Léo]
> leobarretos@uol.com.br
> http://www.feiradeciencias.com.br
> ===========================
> -----Mensagem Original-----
> De: Talita
> Para: leobarretos@uol.com.br
> Enviada em: segunda-feira, 28 de fevereiro de 2005 08:31
> Assunto: Como surgiu a ciência?
>
>
> Olá meu nome é Ana,eu queria saber como começou a ciência?
> Nas dúvidas experimentais, por gentileza coloque aqui o endereço da
página, isso
> facilita o confronto. Agradeço. Meu nome é LUIZ FERRAZ NETTO, meu apelido
é
> LÉO e moro em BARRETOS; dai vem meu e-mail: leobarretos@uol.com.br.
>
>
>
----------------------------------------------------------------------------
----
>
>
> No virus found in this incoming message.
> Checked by AVG Anti-Virus.
> Version: 7.0.300 / Virus Database: 266.5.0 - Release Date: 25/02/2005
>
> ----------
>
> Internal Virus Database is out-of-date.
> Checked by AVG Anti-Virus.
> Version: 7.0.300 / Virus Database: 266.5.0 - Release Date: 25/02/2005
>
>
> [As partes desta mensagem que não continham texto foram removidas]
>
>
>
> ##### ##### #####
>
> Para saber mais visite
> http://www.ciencialist.hpg.ig.com.br
>
>
> ##### ##### ##### #####
> Links do Yahoo! Grupos
>
>
>
>
>
>
>




SUBJECT: Re: Fw: nascimento do atomo
FROM: "rmtakata" <rmtakata@altavista.net>
TO: ciencialist@yahoogrupos.com.br
DATE: 04/03/2005 10:29


Atualmente no interior das estrelas por meio da fusao nuclear: nucleos
de hidrogenio e de helio sao fundidos produzindo elementos mais
pesados. Durante a explosao estelar nucleos ainda mais pesados (alem
de 12 u.a.) podem ser sintetizados.

Acredita-se q. o hidrogenio e o helio tenham se formado logo apos o
esfriamento do universo.

[]s,

Roberto Takata

--- Em ciencialist@yahoogrupos.com.br, "Luiz Ferraz Netto"
> como nascem os átomos?
> ===========================
> Luiz Ferraz Netto [Léo]
> -----Mensagem Original-----
> De: Mauro Santiago
> procurei em muitos sites mas não encontrei uma resposta exata para a
> pergunta "Como nascem os diferentes tipos de átomos?"





SUBJECT: Re: Fw: Como surgiu a ciência?
FROM: "rmtakata" <rmtakata@altavista.net>
TO: ciencialist@yahoogrupos.com.br
DATE: 04/03/2005 10:34


A Ciencia Experimental moderna eh tido como tendo comeco com os
experimentos de Galileu.

A Ciencia observacional pode ser tracada pelo menos ate' os gregos.
Embora possamos considerar a astronomia como tendo comecado pelo menos
desde os egipcios e babilonios (e independentemente em outros lugares
como a China e o Mexico).

Mas nao existe um limite nitido q. se pode tracar: a partir daqui
comecou a ciencia. Uma especie de protociencia deve ter precedido
mesmo o surgimento da humanidade: nossos ancestrais primatas devem ter
tido algum nivel de capacidade de observacao e raciocinio - por meio
de extrapolacoes deveriam ser capazes de fazer certas previsoes
(animais atuais sao capazes disso), isso eh uma das bases das
ciencias. Alguns deveriam ter a capacidade de transmitir esse
conhecimento para outros membros da especie por meio da comunicacao.

[]s,

Roberto Takata

--- Em ciencialist@yahoogrupos.com.br, "Luiz Ferraz Netto"
> Como começou a ciência?
> ===========================
> Luiz Ferraz Netto [Léo]
> -----Mensagem Original-----
> De: Talita
> Olá meu nome é Ana,eu queria saber como começou a ciência?





SUBJECT: Re: Fw: Íon H
FROM: "rmtakata" <rmtakata@altavista.net>
TO: ciencialist@yahoogrupos.com.br
DATE: 04/03/2005 10:39


Um ion H+ eh de fato um proton no mais das vezes. Particulas soltas no
espaco eh o q. mais temos. Mas o ion H+ ou proton em geral rapidamente
se combina com alguma molecula dando sopa por ai' - reduzindo-a: por
isso os acidos corroem muitas coisas - os ions H+ se combinam com,
digamos, moleculas de celulose do papel, quebrando as moleculas.

Um ion H+ pode nao ser um proton se se tratar de um ion formado a
partir de deuterio (nesse caso seria uma combinacao de um proton com
um neutro) ou de um tritio (um proton + 2 neutrons).

[]s,

Roberto Takata

--- Em ciencialist@yahoogrupos.com.br, "Luiz Ferraz Netto"
> Não é química ... mas pode ser respondido por cuímicos :-))
> Luiz Ferraz Netto [Léo]
> -----Mensagem Original-----
> De: guiarte
> Como se forma o íon H+, pois, como sabemos, ele apresenta apenas um
> próton e um elétron. Se perder seu único elétron, ficará apenas um
> próton; ou seja, uma partícula solta no espaço...






SUBJECT: Re: Brilho metálico
FROM: Hélio Ricardo Carvalho <hrc@fis.puc-rio.br>
TO: ciencialist@yahoogrupos.com.br
DATE: 04/03/2005 12:52


Oi Takata,

--- Em ciencialist, "rmtakata" escreveu
>
>Mas o q. ocorre com as ondas q. migram para a esquerda?
>Como elas estao se anulando?
>

Naquela página que passei:
http://tinyurl.com/4hbvk
mostra um pouco sobre isto.
Num cubo de lado igual ao comprimento de onda típico da luz visível
(500nm) temos ~10^6 moléculas.
Isto significa que todo aquela espalhamento acontece numa "crista"
da "onda" e quando a outra vem tudo já terminou. As que aparecem
migrando para esquerda tem comprimento de onda mais de 100 vezes
menor (100 = raiz cúbica de 10^6). E só podem ser de fótons
diferentes!!!

>
>E se um feixe de luz eh espalhado em todas as direcoes,
>alem da questao da formacao de imagem,...
>

Você fala em "feixe de luz". Esta expressão é melhor que raio de
luz.
Na verdade a formação da imagem acontece na retina.
Tanto as imagens ditas virtuais (atrás do espelho) como as ditas
"reais" (na frente do espelho) não estão lá.
Desculpe se sou repetitivo. De cada ponto do objeto sai luz para
todos as direções. Uma parte da luz de cada ponto do objeto (UM
CONE, não "um raio") atinge a pupila do observador. O cristalino a
converge para um ponto na retina. A posição na retina depende do
ângulo de entrada e não do ponto de entrada já que entra por todos
os pontos. Assim é com todos os pontos do objeto. Cada ponto do
objeto aparece em um ponto da retina pois entram com inclinações
diferentes. Pronto: imagem formada!!

Para que a retina seja sensibilizada é necessário que toda a pupila
seja preenchia com aquela "frente de onda" ao mesmo tempo. Se não
chegarem juntas não terão "força" para "promover elétrons para
camada de condução na retina".
Para que olho veja, é necessário uma contribuição construtiva numa
superfície móvel que pode ser chamada de "frente de onda".

>
> ... como fica a conservacao da energia? A energia
>incidente teria q. ser dividida entre os diversos
>feixes refletidos.
>

Aí eu jogo a toalha. Pois não sei bem o que é "energia". :-)
Energia, para mim, não é um SER. :-)
É apenas uma forma de quantificar o movimento da matéria. E como
todo movimento é relativo. A energia também o é.
Pensei, pensei e não consegui responder esta parte dos seus
questionamentos.
Mas, como é destas crises que nascem as grandes idéias, estou muito
agradecido.

Para ajudar nesta REFLEXÃO vou abrir outra thread onde serei mais
perguntador:
Qual o tamanho do fóton??????

[ ]'s
Hélio





SUBJECT: OBSERVATORIO GEMINI - NOVA GALERIA DE IMAGENS
FROM: "L.E.R.de Carvalho" <lecarvalho@infolink.com.br>
TO: ciencialist@yahoogrupos.com.br
DATE: 04/03/2005 12:57


>***************************************************************************
> BOLETIM ELETRÔNICO SAB N. 269 - 02/03/2005
>***************************************************************************
>SEÇÃO: INFORMES * * Responsável: Vera Ap.F. Martin (vmartin@uefs.br)
>***************************************************************************
>1. OBSERVATORIO GEMINI - NOVA GALERIA DE IMAGENS
> (comunicado por Mariangela Oliveira Abans)
>
>OBSERVATORIO GEMINI TEM NOVA GALERIA DE IMAGENS PARA A IMPRENSA
>
>Totalmente remodelado e mais atraente, o novo portal de divulgacao do
>Observatorio Gemini permite encontrar imagens astronomicas de media
>e alta resolucao mais facilmente. Sao imagens de alto impacto visual e
>sempre acompanhadas de legendas.
>
>A Galeria de Imagens encontra-se no endereco:
>
>http://www.gemini.edu/gallery/
>
>Jornalistas interessados em outros materias de divulgacao, incluindo press
>releases, podem visitar:
>
>http://www.gemini.edu/pio/
>
>..........................................
>Contatos:
>. Mariangela de Oliveira-Abans (mabans@lna.br)
>. Peter Michaud (pmichaud@gemini.edu)
>...........................................................................
>O Laboratorio Nacional de Astrofisica e´ um instituto de pesquisas do
>Ministerio da Ciencia e Tecnologia e tambem atua como Secretaria Nacional do
>Observatorio Gemini. O Observatorio Gemini é consorcio formado pelos Estados
>Unidos, Canada´, Reino Unido, Australia, Brasil, Chile e Argentina. O
>consorcio
>opera dois telescopios de 8m de diametro, um localizado na Havai´ e
>outro, no
>Chile. Para maiores detalhes, visite os sites www.lna.br e www.gemini.edu .
>*******************************************************************************
>




SUBJECT: Qual é o tamanho do fóton??????
FROM: Hélio Ricardo Carvalho <hrc@fis.puc-rio.br>
TO: ciencialist@yahoogrupos.com.br
DATE: 04/03/2005 13:01


Qual é o tamanho do fóton??????

Baseado no texto de:
http://br.groups.yahoo.com/group/ciencialist/message/44700

Se o fóton tem informação de freqüência e o comprimento de onda
típico para o visível é ~500nm (verde). O fóton "verde" deveria ter
um comprimento maior que 500nm.
Sim ou não?

Hélio





SUBJECT: meio OFF-TOPIC!: Lentes de contato, energéticos e afins
FROM: "Ivan Carlos" <icarlos@icarlos.net>
TO: "Yahoo - Grupo ciencialist" <ciencialist@yahoogrupos.com.br>
DATE: 04/03/2005 13:10

Alguém pode me ajudar com alguns probleminhas? somente sugestões... =)

Sou novato ao uso de lentes de contato (20 dias)

Em um infeliz dia, tirei as 2 tampas do estojo ao mesmo tempo, e não sei se
troquei-as de lugar ao recolocá-las

Uma lente é tórica e a outra não

Notei que uma lente é um pouco mais azulada que a outra, os graus de miopia
são próximos, mas numa delas tem 1º de astigmatismo a + (por isso uso tórica
:p )

Ao colocar no olho, aparentemente não vejo diferença nenhuma, sugiro que a
tórica seja a de tonalidade mais azulada, jah q ela tem q ter uma formação
mais espessa nas bordas para moldar a deformação e fazer o peso

Alguém tem algum conhecimento sobre isso? Se eu ficar com ela invertida
aparentemente não tem problema, jah q em 20 dias vou substutuí-la (é
descartável) ^^

Outro problema: (desculpe, se a moderação achar melhor apagar, desculpe o
incomodo)

Estou dormindo 3:30hs por dia (trabalho + facul)

OBVIAMENTE antes das 8 e após as 20 horas, me sinto totalmente esgotado,
depois de um tempo a sensação passa e retorna gradativamente

Preciso enfrentar esta maratona até 2007...

Não existe possibilidade de trocar horário de estudo, trabalho, morar mais
perto dos estabelecimentos, etc.. por mais q os médicos me xinguem ><

Consultei um clínico geral, ele sugeriu um complexo vitamínico, mas tenho
sérias dúvidas quanto a eficácia desse negócio (tanto pela metabolização qto
pelo aproveitamente deles pelo corpo), alguém tem alguma sugestão nesse caso
tbém? Energético a base de guaraná acho q naum vai ajudar tbém... rs

desculpe ficar atormentando vcs, voltemos a programação normal :)

[]'s!!!

Ivan "Doomer" Carlos
Social Engineering Specialist
Cell.: +55 (11) 8112-0666
icarlos@icarlos.net
www.icarlos.net
--------------------------------------------------




SUBJECT: Fw: milha maritima e milha terrestre
FROM: "Luiz Ferraz Netto" <leobarretos@uol.com.br>
TO: "ciencialist" <ciencialist@yahoogrupos.com.br>
DATE: 04/03/2005 13:39

MessagePara o consulente minha resposta seria: Vc já viu coisa de norte-americano ter lógica? O galão não é a medida da capacidade do pinico da rainha sei lá o nome?

Mas, vcs podem ter outras idéias .........

[]'
===========================
Luiz Ferraz Netto [Léo]
leobarretos@uol.com.br
http://www.feiradeciencias.com.br
===========================
-----Mensagem Original-----
De: Carlos Alberto Martins Ferreira
Para: leobarretos@uol.com.br
Enviada em: sexta-feira, 4 de março de 2005 09:22
Assunto: milha maritima e milha terrestre


Tenho uma duvida, e gostaria se possivel ser esclarecido.

Quala a razão de haver diferença de medida entre a milha maritima e amilha terrestre.

desde ja obrigado

carlos







Nas dúvidas experimentais, por gentileza coloque aqui o endereço da página, isso facilita o confronto. Agradeço. Meu nome é LUIZ FERRAZ NETTO, meu apelido é LÉO e moro em BARRETOS; dai vem meu e-mail: leobarretos@uol.com.br.



--------------------------------------------------------------------------------


Internal Virus Database is out-of-date.
Checked by AVG Anti-Virus.
Version: 7.0.300 / Virus Database: 266.5.0 - Release Date: 25/02/2005

----------

Internal Virus Database is out-of-date.
Checked by AVG Anti-Virus.
Version: 7.0.300 / Virus Database: 266.5.0 - Release Date: 25/02/2005


[As partes desta mensagem que não continham texto foram removidas]



SUBJECT: Re: [ciencialist] Fw: Como surgiu a ciência?
FROM: "Luiz Ferraz Netto" <leobarretos@uol.com.br>
TO: <ciencialist@yahoogrupos.com.br>
DATE: 04/03/2005 14:06

Hugo: >Segundo a minha visão, a ciência começou no instante em que o homem obteve a
primeira centelha de consciência e se viu como parte íntegra do mundo que o
rodeava. Observação, definição e sistematização terão sido as primeiras
acções científicas que o primeiro hominídeo terá feito. Depois foi só
evoluir... <

Léo: evoluir ou criar? No caso, evolução ou criação?

Dúvida: --- Por que o turma do criacionismo não inclui nesse papo tb as rochas. As rochas foram 'criadas' como são atualmente? Prá que serve a classificação:

- Rocha magmática. Geol.
A que resultou da consolidação devida a resfriamento de magma; rocha ígnea, rocha eruptiva.

- Rocha metamórfica. Geol.
Rocha que sofreu o processo do metamorfismo (Transformação que sofre uma rocha sob a ação de temperatura, pressão, gases e vapor de água, que produzem, isolada ou conjuntamente, uma recristalização parcial ou total, formando-se novos minerais e novas texturas sem ocorrer a fusão da rocha)

- Rocha sedimentar. Geol.
Rocha resultante da destruição, desagregação ou decomposição de outras rochas ou de outros materiais preexistentes e da posterior sedimentação, em camadas ou estratos, dos detritos provenientes dessa destruição; rocha estratificada.

.... só queria saber .......

[]'

Léo



--
Internal Virus Database is out-of-date.
Checked by AVG Anti-Virus.
Version: 7.0.300 / Virus Database: 266.5.0 - Release Date: 25/02/2005



SUBJECT: Re: [ciencialist] Qual é o tamanho do fóton??????
FROM: TARCISIO BORGES <tbs97@fisica.ufpr.br>
TO: ciencialist@yahoogrupos.com.br
DATE: 04/03/2005 14:09

È simples.

Basta multiplicar a velocidade da luz pelo tempo de emissão do fóton
quando o eletron muda seu estado de energia.

Se eu não me engano dá uns três metros.

[]s
TARCISIO BORGES
tbs97@fisica.ufpr.br

On Fri, 4 Mar 2005, Hélio Ricardo Carvalho wrote:
> Qual é o tamanho do fóton??????
>
> Baseado no texto de:
> http://br.groups.yahoo.com/group/ciencialist/message/44700
>
> Se o fóton tem informação de freqüência e o comprimento de onda
> típico para o visível é ~500nm (verde). O fóton "verde" deveria ter
> um comprimento maior que 500nm.
> Sim ou não?
>
> Hélio



SUBJECT: Lentes de contato, energéticos e afins
FROM: "L.E.R.de Carvalho" <lecarvalho@infolink.com.br>
TO: ciencialist@yahoogrupos.com.br
DATE: 04/03/2005 14:32

At 13:10 4/3/2005, you wrote:
>Estou dormindo 3:30hs por dia (trabalho + facul)
>
>OBVIAMENTE antes das 8 e após as 20 horas, me sinto totalmente esgotado,
>depois de um tempo a sensação passa e retorna gradativamente
>
>Preciso enfrentar esta maratona até 2007...
>
>Não existe possibilidade de trocar horário de estudo, trabalho, morar mais
>perto dos estabelecimentos, etc.. por mais q os médicos me xinguem ><
>
>Consultei um clínico geral, ele sugeriu um complexo vitamínico, mas tenho
>sérias dúvidas quanto a eficácia desse negócio (tanto pela metabolização qto
>pelo aproveitamente deles pelo corpo), alguém tem alguma sugestão nesse caso
>tbém? Energético a base de guaraná acho q naum vai ajudar tbém... rs
>
>desculpe ficar atormentando vcs, voltemos a programação normal :)
>
>[]'s!!!
>
>Ivan "Doomer" Carlos
>Social Engineering Specialist
>Cell.: +55 (11) 8112-0666
>icarlos@icarlos.net
>www.icarlos.net



Ivan, meu caro.

Pra esse problema a Biologia não pode te dar resposta.
Melhor você arrumar um jeito de dormir, pelo menos, sete horas por dia.

E buscar resposta na Física.
Veja por aí se alguém cria uma tecnologia pro planeta rodar mais lento e o
dia ter 32 horas.

Ou manda um projeto de lei pra Câmara dos Deputados.
Depois do show que eles deram, com crianças em cadeiras de roda, inventando
a terapia gênica e liberando células-tronco... periga a solução vir de
Brasília.

Tens tempo e grana pra fazer o lobby ?

L.E.








-
-
-
-
-
-





SUBJECT: Re: [ciencialist] Re: Albert Einstein, um hipócrita???????????
FROM: "murilo filo" <avalanchedrive@hotmail.com>
TO: ciencialist@yahoogrupos.com.br
DATE: 04/03/2005 16:15

Victor,
somos todos bons camaradas!
Esta estória da risadas x pedrada eu não conheço. É gozação? Vc tem mais
detalhes?
abr/M.

>From: JVictor <jvoneto@uol.com.br>
>Reply-To: ciencialist@yahoogrupos.com.br
>To: ciencialist@yahoogrupos.com.br
>Subject: Re: [ciencialist] Re: Albert Einstein, um hipócrita???????????
>Date: Fri, 04 Mar 2005 07:32:02 -0300
>
>
>Murilo,
>
>Sei as motivações pessoais, e outras razões, para alguém ter
>interêsses em mostrar certos aspectos meramente humanos de pessoas do
>nível de Einstein. Mas não vou dar opiniões a respeito. Só digo o
>seguinte: essas pessoas esquecem que quando apontam com um dedo, há
>pelos 3 apontando, volta, para ele!
>Hipocrisia das hipocrisias, diria um famosos hipócritas bíblicos, perto
>da morte, que fez um gigante rir até morrer, e disseram que foi de uma
>pedrada.
>O César está correto. E você também, em concordar com ele.
>
>Sds,
>
>Victor.
>
>
>
>murilo filo escreveu:
>
> > Concordo com o César ( e pela 1ª vez )!!!
> > Afinal êle - Einstein - não teria que apresentar nenhuma perfeição
> > moral e
> > ética.
> > Não havia fundado nenhuma religião, como é o caso do Bispo Edir
> > Macedo, que
> > muito teria me decepcionado se tivesse tal comportamento... :/
> > Se formos procurar, até o Gandhi seria algo decepcionante. É o ser
> > humano,
> > rapazes! abr/M.
> >
> > >From: César A. K. Grossmann <cesarakg@bol.com.br>
> > >Reply-To: ciencialist@yahoogrupos.com.br
> > >To: ciencialist@yahoogrupos.com.br
> > >Subject: [ciencialist] Re: Albert Einstein, um hipócrita???????????
> > >Date: Thu, 03 Mar 2005 18:39:00 -0000
> > >
> > >
> > >
> > >--- Em ciencialist@yahoogrupos.com.br, "Cyberlander"
> > ><mrcyberlander@i...> escreveu
> > > >
> > > > Segundo aquele dito popular "Casa de ferreiro, espeto de pau", a
> > > > relação de Einstein com seus filhos não era condizente com suas
> > > > afirmações públicas, por exemplo, com aquela famosa citação:
> > > >
> > > > "Es gibt weder große Entwicklungen noch wahre Fortschritte auf
> > > > dieser Erde, solange noch ein unglückliches Kind auf ihr lebt."
> > > >
> > > > Tradução:
> > > > < Não existem grandes desenvolvimentos nem tão pouco verdadeiro
> > > > progresso neste mundo, enquanto ainda nele viver uma criança
> > > > infeliz. >
> > >
> > >Não vejo relação entre uma coisa e outra, é possível explicar melhor?
> > >A afirmativa dele refere-se a progresso e desenvolvimento, e, partindo
> > >do ponto que nem o progresso nem o desenvolvimento contribuem para a
> > >harmonia do lar, imagino que ele estava se referindo a miséria e
> > >abandono de quem fica à margem do progresso e do desenvolvimento.
> > >
> > > > Um gênio? Um hipócrita? Tendo em vista que ele é admirado e
> > > > defendido por Oliveira, RM e coorte, a segunda pergunta tem mais
> > > > chance de se afirmar positivamente.
> > >
> > >Deixa eu ver, a prova de que ele era um hipócrita reside no fato de
> > >que ele é admirado e defendido por um certo Oliveira, RM, etc. Isto
> > >não parece muito lógico. Aliás, não parece nada lógico. "Olha só o
> > >cara, ele tem que ser um hipócrita, já que beltrano está sempre
> > >defendendo ele".
> > >
> > >Ainda na mesma novela, suponhamos que Einstein fosse um hipócrita. Em
> > >que isto afetaria a sua genialidade? Quer dizer, se fosse possível
> > >provar que ele foi hipócrita (o que eu não estou convencido, a mim
> > >parece apenas que ele não era do tipo expansivo e afetuoso), isto
> > >implica que ele era um burro? O que uma coisa tem a ver com a outra?
> > >Um gênio não pode ser hipócrita? São qualidades mutuamente exclusivas?
> > >
> > >Apenas para finalizar, quer me parecer que toda a missiva se trata de
> > >um 'ad hominem': uma maneira de tentar desacreditar as idéias de uma
> > >pessoa atacando a pessoa, sem considerar os méritos das idéias. Entre
> > >cientistas não deveria ser assim. Quando as teorias são conflitantes,
> > >o conflito deve ser entregue ao campo das idéias. Pouco importa se o
> > >proponente ou o atacante da teoria é um papa-hóstias ou um dissoluto
> > >imoral.
> > >
> > >[]s
> > >--
> > >César A. K. Grossmann
> > >http://www.LinuxbyGrossmann.cjb.net/
> > >
> > >
> > >
> > >
> > >
> > >##### ##### #####
> > >
> > >Para saber mais visite
> > >http://www.ciencialist.hpg.ig.com.br
> > >
> > >
> > >##### ##### ##### #####
> > >Links do Yahoo! Grupos
> > >
> > >
> > >
> > >
> > >
> > >
> > >
> > >
> >
> >
> >
> >
> > ##### ##### #####
> >
> > Para saber mais visite
> > http://www.ciencialist.hpg.ig.com.br
> >
> >
> > ##### ##### ##### #####
> >
> >
> > *Yahoo! Grupos, um serviço oferecido por:*
> >
> > *
> >
><http://br.rd.yahoo.com/SIG=12ama5o9l/M=264105.3931087.6562589.1588051/D=brclubs/S=2137111528:HM/EXP=1109986216/A=2361264/R=6/SIG=10v4acpp0/*http://br.shopping.yahoo.com/>*
> >
> >
> >
> > ------------------------------------------------------------------------
> > *Links do Yahoo! Grupos*
> >
> > * Para visitar o site do seu grupo na web, acesse:
> > http://br.groups.yahoo.com/group/ciencialist/
> >
> > * Para sair deste grupo, envie um e-mail para:
> > ciencialist-unsubscribe@yahoogrupos.com.br
> >
><mailto:ciencialist-unsubscribe@yahoogrupos.com.br?subject=Unsubscribe>
> >
> > * O uso que você faz do Yahoo! Grupos está sujeito aos Termos do
> > Serviço do Yahoo! <http://br.yahoo.com/info/utos.html>.
> >
> >
> >
> >
> > __________ Informação do NOD32 1.1017 (20050302) __________
> >
> > Esta mensagem foi verificada pelo NOD32 Sistema Antivírus
> > http://www.nod32.com.br
>
>
>
>
>##### ##### #####
>
>Para saber mais visite
>http://www.ciencialist.hpg.ig.com.br
>
>
>##### ##### ##### #####
>Links do Yahoo! Grupos
>
>
>
>
>
>
>
>




SUBJECT: "ENGENHARIA ALTERNATIVA" UMA PARÁBOLA PÓS-MODERNA
FROM: "Oraculo" <oraculo@atibaia.com.br>
TO: <ciencialist@yahoogrupos.com.br>
DATE: 04/03/2005 17:58


"ENGENHARIA ALTERNATIVA" UMA PARÁBOLA PÓS-MODERNA
de Steven Novella, M.D.

Nota: As pessoas mencionadas nesta história são fictícias, mas os perigos da pseudociência aplicada são reais.

Um novo fenômeno está invadido os EUA, ganhando a atenção tanto dos consumidores como dos fabricantes. Cada vez mais decepcionadas com o frio mundo metálico da tecnologia moderna, as pessoas estão olhando mais atentamente para alternativas mais naturais. Coletivamente chamada de Engenharia Alternativa ("Alt Eng"), uma grande variedade de métodos novos e antigos está ganhando respeitabilidade científica e jornalística.

Alec Waterstone é um desses supostos engenheiros alternativos. Ele não tem nenhum diploma ou formação acadêmica em engenharia, o que, ele explica, é uma vantagem. "Meu pensamento não é limitado pela matemática, lógica ou qualquer enfadonho antigo paradigma mecanicista. Não tenho que render homenagens às preferências de Newton ou de outros pedagogos ocidentais. A ausência completa de treinamento me deixa livre para pensar em soluções únicas e inovadoras para problemas de engenharia, liberto dos irritantes limites da 'realidade.'"

Pontes Baseadas em Energia

O último projeto de Alec é uma ponte não-pênsil de 460 m. Ele afirma que a ponte será capaz de transpor esta distância sem pilares ou suspensões, e será sustentada somente pela antiga arte do Feng Shui. "Esta sabedoria, que tem milhares de anos, é a arte de canalizar energia através do design e da forma. Esta energia pode ser usada para suportar uma ponte de 460 m, ou mesmo estruturas maiores." Os planejadores urbanos estão intrigados por estes projetos, porque tais pontes custarão menos que a metade das pontes projetadas pela maneira convencional.

Alec também mostra que antigos documentos chineses não revelam absolutamente nenhum relato de pontes suspensas desmoronando. Ele argumenta que seus registros de segurança da técnica são sem paralelos. "Como poderia ter sobrevivido todos estes anos se não funcionasse?"

Anthony Trellis, professor de engenharia na Universidade Estado-da-Arte, alega que os projetos de Alec correm na direção contrária aos princípios básicos da física e da ciência dos materiais. Irritado, Trellis comentou que "uma ponte baseada nos projetos de Waterstone simplesmente não poderia se sustentar. Seria insegura ao extremo."

Mas Alec não se perturba com esssas críticas. "É claro que o professor Trellis não gosta dos meus projetos, porque eles desafiam seu precioso status quo e põem seu mundo de cabeça para baixo. Mas o protecionismo da velha guarda está começando a desmoronar, como também seus prédios obsoletos", ele retrucou em simpósio recente para pensadores progressistas que concordaram que aqueles que perdem o bonde da história serão deixados para trás. Seu discurso para uma platéia lotada acusou a American Society of Civil Engineers (Sociedade Americana de Engenheiros Civis), a indústria do aço e outros "interesses ocultos" de tentarem suprimir seus pontos de vista.

Céticos têm sugerido que antes de gastar milhões de dólares do contribuinte nesses projetos, e sujeitar os motoristas americanos a riscos desconhecidos de dirigir por sobre uma ponte de Waterstone, os princípios básicos de Waterstone deveriam ao menos ser testados para ver se eles funcionam. Isto é especialmente correto uma vez que seus projetos parecem ir contra o senso comum. Mas Waterstorne responde:

Estou ocupado demais projetando pontes para ficar satisfazendo alguns céticos. Seja como for, eles nunca estarão satisfeitos. Os motoristas americanos deveriam ser livres para decidir se desejam dirigir por sobre uma de minhas pontes. Respeito sua inteligência e habilidade para tomarem decisões inteligentes sozinhos. Eles não precisam ser informados por algum burocrata, ou um professor em uma torre de marfim, sobre quais pontes são seguras e quais não.
Professor Trellis e outros opositores argumentam que os indivíduos não deveriam ter de ser cientistas ou engenheiros de modo a dirigir com segurança sobre nossas pontes. As regras não são feitas para limitar a liberdade, mas para oferecer um nível básico de segurança e proteção para o público. Esta atitude, todavia, está progressivamente sendo rejeitada como excessivamente paternalista e protecionista.
Carros Intuitivos

Engenheiros civis não são o únicos a serem atraídos pela antiga sabedoria das sociedades pré-tecnológicas. A indústria automobilística também está se contagiando. Natural Designs é uma nova companhia automobilística com sede no Kansas. Seus presidente, Andy Wily, recebeu de Harvard um título acadêmico em engenharia 20 anos atrás, mas foi demitido de seu cargo subseqüente de professor após uso excessivo de drogas a ponto de quase destruir sua vida. Agora ele retornou com uma nova companhia e uma nova filosofia para a qual muitos consumidores se sentem atraídos.

"Estou defendendo uma mistura do melhor da engenharia científica moderna com as idéias anticientíficas e supersticiosas dos tempos antigos," explica Wily. "Chamo esta abordagem de Engenharia Integrativa."

O que foi que esta nova abordagem criou? O mais novo modelo sedan da Natural Design, o Millennium 2000, não usa air bags, nem cintos de segurança. "Cintos de segurança são perigosos, e air bags matam crianças," queixa-se Wily. Então ele apareceu com algo melhor. O interior do Millennium 2000 é revestido com um material psicoativo patenteado, chamado Natural Safe. "Tudo que o motorista ou os passageiros precisam fazer é ter pensamentos seguros, e este material milagroso fará o resto. Em um acidente, o material delicadamente conterá qualquer pessoa com pensamentos seguros no veículo, deixando-as livres de ferimentos", assegura Wily.

Quando os céticos apontam as mortes ou invalidez dos passageiros do Millennium 2000, Wily responde que os passageiros claramente não estavam pensando tão "seguramente" como deveriam. "Além disso", acrescenta, "o Millennium 2000 corre a somente 80 km por hora em um bom dia deixando para trás um vento feliz. Se os motoristas que morreram estivessem dirigindo algo desenvolvido pelo Cartel Internacional dos Fabricantes de Automóveis como um Ford ou um Chevrolet, estariam indo muito mais rápido, com uma chance de morte ainda maior. Só quando a Ford parar de matar milhares de pessoas por ano em nossas estradas é que suas reclamações sobre nós parecerão algo além de proteção da sua fatia de mercado. Na verdade, temos aqui mesmo um estudo que mostra que se todos deixassem de dirigir amanhã, a taxa de morte cairia na América! Até convencermos o povo americano dos milhões mortos desnecessariamente pela moderna 'ciência automotiva', a Natural Safe permanece a escolha mais segura."

Muitos consumidores estão convencidos. Para não ficar para trás, a GM e a Ford começaram a colocar o revestimento da Natural Safe em seus carros. Amy Zinger, de Arkansas, sobreviveu a uma colisão frontal a 65 km/h em um destes veículos. "Estava usando meu cinto de segurança e o air bag abriu, mas eu sei que foi o Natural Safe que salvou minha vida", declarou recentemente. "Além disso", apontou, "se não funcionasse, sua venda não seria permitida." Motivados por testemunhos assim, cada vez mais consumidores estão insistindo em comprar somente carros com o Natural Safe.

Um problema enfrentado pela Natural Design, todavia, é que as antiquadas leis de segurança, como aquelas que exigem cintos de segurança, não mencionam estes novos projetos integrativos. Recentemente, entretanto, tudo isto tem mudado. O senador Hackem, de Iowa, estado natal da Natural Design, propôs a legislação que irá isentar os fabricantes que usam princípios alternativos ou integrativos de regulação planejada para proteger os consumidores. Isto foi aclamado como um grande passo à frente.

Contudo, os céticos teimosos não desaparecerão. "Tudo que estou pedindo é um simples teste de colisão" exclamou um cético famoso, Perry DeAngelis. "Se a coisa realmente funciona, diacho, eu compro." Os céticos vêm cada vez mais pedindo tais testes, argumentando que o teste deveria ser realizado antes da implementação, especialmente quando vidas humanas estão em jogo.

Mas Wily explica o motivo pelo qual estes testes não funcionam. "Bonecos de testes não são pessoas. O material psicoativo portanto não responderá a eles. O fato é que estes projetos inovadores não podem ser submetidos aos mesmos testes e princípios como a engenharia tradicional. Mas os consumidores que dirigem nossos carros se sentem mais seguros. Como você pode argumentar contra isso?"

Contudo, DeAngelis aponta para estudos recentes que parecem indicar que os motoristas dos carros de Wily têm uma probabilidade duas vezes maior de morrer em um acidente que os motoristas de veículos convencionais. Mas Wily somente zomba disso, "no que você vai acreditar, números em um pedaço de papel, ou em pessoas?"

Conquistas Políticas

Apesar dos céticos, a Engenharia Alternativa parece que veio para ficar. Wily acabou de ser nomeado chefe do novo Departamento de Engenharia Integrativa na Zones University, onde ele espera treinar a nova geração de engenheiros em sua filosofia. Enquanto isso, o senador Hackem apresentou ao Congresso um projeto de lei para criar o Centro de Engenharia Alternativa. Este novo departamento irá impedir que dinheiro seja desperdiçado na manutenção da infra-estrutura do país e seja usado para estudar e promover os princípios alternativos na engenharia.

Finalmente, no que é caracterizado como um marco na produção, o Canadian College of Rainbow-Coloured Integrative Engineering - após passar mais de 100 anos como um pária científico - está finalizando negociações para se tornar parte da prestigiosa Dork University. Apesar dos gritos de consternação da faculdade de ciência e matemática da Dork e de diversos laureados com o Nobel, o conselho de Dork avança com seus planos de afiliação. Os 25 milhões de dólares que os Engenheiros Integrativos prometeram doar á Universidade não têm, de acordo com o presidente da Dork, influenciado o acordo. O presidente caracterizou os críticos da Engenharia Integrativa como "chorões" que expõem "desinformação há muito refutada" sobre a Engenharia Alternativa.


[As partes desta mensagem que não continham texto foram removidas]



SUBJECT: dormindo 3:30h por dia...
FROM: "Eurico Ferreira de Souza Jr." <caodejah@yahoo.com.br>
TO: ciencialist@yahoogrupos.com.br
DATE: 04/03/2005 19:09



Ivan Carlos <icarlos@icarlos.net> wrote:

Estou dormindo 3:30hs por dia (trabalho + facul)

OBVIAMENTE antes das 8 e após as 20 horas, me sinto totalmente esgotado,
depois de um tempo a sensação passa e retorna gradativamente

Preciso enfrentar esta maratona até 2007...

Não existe possibilidade de trocar horário de estudo, trabalho, morar mais
perto dos estabelecimentos, etc.. por mais q os médicos me xinguem ><

Consultei um clínico geral, ele sugeriu um complexo vitamínico, mas tenho
sérias dúvidas quanto a eficácia desse negócio (tanto pela metabolização qto
pelo aproveitamente deles pelo corpo), alguém tem alguma sugestão nesse caso
tbém? Energético a base de guaraná acho q naum vai ajudar tbém... rs

desculpe ficar atormentando vcs, voltemos a programação normal :)

[E]> sem querer "gorar", mas acho que vc vai adoecer e envelhecer mais rápido, se não pirar...

fiquei um ano dormindo 4 horas e me zoou bastante e definitivamente...(dormir depois não vai tirar o atraso) tente mudar de vida, enquanto tem uma. boa sorte e um forte abraço.




_\|/_
__________________________________________________
Converse com seus amigos em tempo real com o Yahoo! Messenger
http://br.download.yahoo.com/messenger/

[As partes desta mensagem que não continham texto foram removidas]



SUBJECT: Re: [ciencialist] dormindo 3:30h por dia...
FROM: "Ivan Carlos" <icarlos@icarlos.net>
TO: <ciencialist@yahoogrupos.com.br>
DATE: 04/03/2005 19:36

se eu aprender a dormir no ônibus ajuda? =)

passo 1 hora dentro do onibus na ida, depois + 30 mins, depois + metro...

na volta mais 30mins + 1 hora de bus... ^^

Ivan "Doomer" Carlos
Social Engineering Specialist
Cell.: +55 (11) 8112-0666
icarlos@icarlos.net
www.icarlos.net
--------------------------------------------------

----- Original Message -----
From: "Eurico Ferreira de Souza Jr." <caodejah@yahoo.com.br>
To: <ciencialist@yahoogrupos.com.br>
Sent: Friday, March 04, 2005 7:09 PM
Subject: [ciencialist] dormindo 3:30h por dia...





Ivan Carlos <icarlos@icarlos.net> wrote:

Estou dormindo 3:30hs por dia (trabalho + facul)

OBVIAMENTE antes das 8 e após as 20 horas, me sinto totalmente esgotado,
depois de um tempo a sensação passa e retorna gradativamente

Preciso enfrentar esta maratona até 2007...

Não existe possibilidade de trocar horário de estudo, trabalho, morar mais
perto dos estabelecimentos, etc.. por mais q os médicos me xinguem ><

Consultei um clínico geral, ele sugeriu um complexo vitamínico, mas tenho
sérias dúvidas quanto a eficácia desse negócio (tanto pela metabolização qto
pelo aproveitamente deles pelo corpo), alguém tem alguma sugestão nesse caso
tbém? Energético a base de guaraná acho q naum vai ajudar tbém... rs

desculpe ficar atormentando vcs, voltemos a programação normal :)

[E]> sem querer "gorar", mas acho que vc vai adoecer e envelhecer mais
rápido, se não pirar...

fiquei um ano dormindo 4 horas e me zoou bastante e
definitivamente...(dormir depois não vai tirar o atraso) tente mudar de
vida, enquanto tem uma. boa sorte e um forte abraço.




_\|/_
__________________________________________________
Converse com seus amigos em tempo real com o Yahoo! Messenger
http://br.download.yahoo.com/messenger/

[As partes desta mensagem que não continham texto foram removidas]



##### ##### #####

Para saber mais visite
http://www.ciencialist.hpg.ig.com.br


##### ##### ##### #####
Links do Yahoo! Grupos











SUBJECT: O ÍNDICE CRACKPOT
FROM: "Oraculo" <oraculo@atibaia.com.br>
TO: <ciencialist@yahoogrupos.com.br>
DATE: 04/03/2005 20:21


O ÍNDICE CRACKPOT
de John Baez

Um método simples para classificar contribuições à física potencialmente revolucionárias.


1.. Crédito inicial de -5 pontos.
2.. 1 ponto por afirmação reconhecidamente falsa.
3.. 2 pontos por afirmação claramente insubstancial.
4.. 3 pontos por afirmação logicamente inconsistente.
5.. 5 pontos por afirmação desse tipo que é aceita apesar de cuidadosa correção.
6.. 5 pontos por usar um experimento de pensamento que contradiz os resultados de um experimento real bem aceito.
7.. 5 pontos por palavra toda em maiúsculas (a não ser para aqueles que possuem teclados com defeito).
8.. 5 pontos por menção a Einstein, Hawkins ou Feynmann.
9.. 10 pontos por alegação de que a mecânica quântica está essencialmente errada (sem boas evidências).
10.. 10 pontos por afirmar que tem escolaridade, como se isso fosse evidência de sanidade.
11.. 10 pontos por começar a descrição da teoria dizendo quanto tempo você gastou trabalhando nela.
12.. 10 pontos por enviar a teoria a alguém que você não conhece pessoalmente e pedir a ela para não contar para mais ninguém, por medo de que suas idéias sejam roubadas.
13.. 10 pontos por oferecer um prêmio em dinheiro para quem provar e/ou encontrar erros na sua teoria.
14.. 10 pontos por afirmação do tipo "não sou bom em matemática, mas minha teoria está certa em termos conceituais, então só preciso que alguém a expresse em equações".
15.. 10 pontos por argumentar que certa teoria bem estabelecida é "somente uma teoria", como se isso fosse ruim.
16.. 10 pontos por afirmar que apesar de uma teoria bem estabelecida prever fenômenos corretamente, ela não explica "por que" eles ocorrem, ou não fornece um "mecanismo".
17.. 10 pontos por comparação favorável com Einstein, ou por afirmar que a relatividade gerasl está fundamentalmente errada (sem boas evidências).
18.. 10 pontos por alegar que seu trabalho é a vanguarda de uma "mudança de paradigma".
19.. 20 pontos por sugerir que você merece um prêmio Nobel.
20.. 20 pontos por comparação favorável com Newton, ou por alegar que a mecância clássica está essencialmente errada (sem boas evidências).
21.. 20 pontos por uso de obras ou mitos de ficção científica como se fossem fatos.
22.. 20 pontos por se defender ao citar o ridículo (real ou imaginário) associado a suas teorias anteriores.
23.. 20 pontos por uso da expressão "reacionário enrustido".
24.. 20 pontos por uso da expressão "auto-definido representante da ortodoxia".
25.. 30 pontos por sugerir que alguma personalidade importante na verdade não acreditava em uma teoria que publicamente apoiava (por exemplo, dizer que Feynman era um adversário não declarado da relatividade epecial, como se deduziria lendo entre as linhas dos seus textos para primeiranistas de física.)
26.. 30 pontos por sugerir que Einstein, em seus últimos anos, estava se encaminhando para as idéias que você apóia.
27.. 30 pontos por afirmar que suas teorias foram desenvolvidas por civilizações extraterrestres (sem boas evidências).
28.. 40 pontos por comparar os críticos de sua teoria com nazistas, tropas de choque ou algum outro grupo racista.
29.. 40 pontos por afirmar que o "establishment científico" está "conspirando" para impedir que seu trabalho receba a merecida fama, ou coisa que o valha.
30.. 40 pontos por comparação favorável com Galileu, sugerindo que existe uma inquisição moderna a todo vapor no seu caso, e assim por diante.
31.. 40 pontos por afirmar que quando sua teoria for avaliada, se perceberá que a ciência atual não passa de uma grande bobagem (mais 30 pontos por fantasiar sobre julgamentos em que os cientistas que zombaram da sua teoria serão forçados a se retratar).
32.. 50 pontos por afirmar que você tem uma teoria revolucionária mas sem dar predições concretas testáveis.


[As partes desta mensagem que não continham texto foram removidas]



SUBJECT: Re: [ciencialist] Re: Albert Einstein, um hipócrita???????????
FROM: ZMIR <zmirsilva@uol.com.br>
TO: ciencialist@yahoogrupos.com.br
DATE: 04/03/2005 20:49

murilo filo escreveu:

>Victor,
>somos todos bons camaradas!
>Esta estória da risadas x pedrada eu não conheço. É gozação? Vc tem mais
>detalhes?
>abr/M.
>
>

>Olá Murilo,
>
>
O Davi da bíblia, quando pivete, defrontou-se com um guerreiro gigante
e, destemidamente,
tascou-lhe uma tijolada, que o homem caiu duro. Isso é que diz o livro
dos livros. Eu não acredito, conversa prá boi dormir,
daí a alusão em tom de brincadeira. Dr. o gorila realmente dançou, só
pode ter sido de tanto rir, ante a palhaçada. E não lembro a qual dos
dois, Davi ou Salomão, a Bíblia se refereao dizer que teve umas
setecentas mulheres, durante toda a vida. Já no final dela, pertinho de
ir se encontrar com o salvador, arrependeu-se, que não era besta, e
então passou criticar atitudes pecaminosas, do tipo e de outros.
O "vaidade das vaidades", que inicia uma série de coisas santificantes
refere-se a isso. É ou não é " hipocrisia das hipocrisias"?Não dá para
falar sério com
coisas do tipo. Acho que o assunto acabou ficando off-tópic. Páro por aquí.

Sds,

Victor.

>>From: JVictor <jvoneto@uol.com.br>
>>Reply-To: ciencialist@yahoogrupos.com.br
>>To: ciencialist@yahoogrupos.com.br
>>Subject: Re: [ciencialist] Re: Albert Einstein, um hipócrita???????????
>>Date: Fri, 04 Mar 2005 07:32:02 -0300
>>
>>
>>Murilo,
>>
>>Sei as motivações pessoais, e outras razões, para alguém ter
>>interêsses em mostrar certos aspectos meramente humanos de pessoas do
>>nível de Einstein. Mas não vou dar opiniões a respeito. Só digo o
>>seguinte: essas pessoas esquecem que quando apontam com um dedo, há
>>pelos 3 apontando, volta, para ele!
>>Hipocrisia das hipocrisias, diria um famosos hipócritas bíblicos, perto
>>da morte, que fez um gigante rir até morrer, e disseram que foi de uma
>>pedrada.
>>O César está correto. E você também, em concordar com ele.
>>
>>Sds,
>>
>>Victor.
>>
>>
>>
>>murilo filo escreveu:
>>
>>
>>
>>>Concordo com o César ( e pela 1ª vez )!!!
>>>Afinal êle - Einstein - não teria que apresentar nenhuma perfeição
>>>moral e
>>>ética.
>>>Não havia fundado nenhuma religião, como é o caso do Bispo Edir
>>>Macedo, que
>>>muito teria me decepcionado se tivesse tal comportamento... :/
>>>Se formos procurar, até o Gandhi seria algo decepcionante. É o ser
>>>humano,
>>>rapazes! abr/M.
>>>
>>>
>>>
>>>>From: César A. K. Grossmann <cesarakg@bol.com.br>
>>>>Reply-To: ciencialist@yahoogrupos.com.br
>>>>To: ciencialist@yahoogrupos.com.br
>>>>Subject: [ciencialist] Re: Albert Einstein, um hipócrita???????????
>>>>Date: Thu, 03 Mar 2005 18:39:00 -0000
>>>>
>>>>
>>>>
>>>>--- Em ciencialist@yahoogrupos.com.br, "Cyberlander"
>>>><mrcyberlander@i...> escreveu
>>>>
>>>>
>>>>>Segundo aquele dito popular "Casa de ferreiro, espeto de pau", a
>>>>>relação de Einstein com seus filhos não era condizente com suas
>>>>>afirmações públicas, por exemplo, com aquela famosa citação:
>>>>>
>>>>>"Es gibt weder große Entwicklungen noch wahre Fortschritte auf
>>>>>dieser Erde, solange noch ein unglückliches Kind auf ihr lebt."
>>>>>
>>>>>Tradução:
>>>>>< Não existem grandes desenvolvimentos nem tão pouco verdadeiro
>>>>>progresso neste mundo, enquanto ainda nele viver uma criança
>>>>>infeliz. >
>>>>>
>>>>>
>>>>Não vejo relação entre uma coisa e outra, é possível explicar melhor?
>>>>A afirmativa dele refere-se a progresso e desenvolvimento, e, partindo
>>>>do ponto que nem o progresso nem o desenvolvimento contribuem para a
>>>>harmonia do lar, imagino que ele estava se referindo a miséria e
>>>>abandono de quem fica à margem do progresso e do desenvolvimento.
>>>>
>>>>
>>>>
>>>>>Um gênio? Um hipócrita? Tendo em vista que ele é admirado e
>>>>>defendido por Oliveira, RM e coorte, a segunda pergunta tem mais
>>>>>chance de se afirmar positivamente.
>>>>>
>>>>>
>>>>Deixa eu ver, a prova de que ele era um hipócrita reside no fato de
>>>>que ele é admirado e defendido por um certo Oliveira, RM, etc. Isto
>>>>não parece muito lógico. Aliás, não parece nada lógico. "Olha só o
>>>>cara, ele tem que ser um hipócrita, já que beltrano está sempre
>>>>defendendo ele".
>>>>
>>>>Ainda na mesma novela, suponhamos que Einstein fosse um hipócrita. Em
>>>>que isto afetaria a sua genialidade? Quer dizer, se fosse possível
>>>>provar que ele foi hipócrita (o que eu não estou convencido, a mim
>>>>parece apenas que ele não era do tipo expansivo e afetuoso), isto
>>>>implica que ele era um burro? O que uma coisa tem a ver com a outra?
>>>>Um gênio não pode ser hipócrita? São qualidades mutuamente exclusivas?
>>>>
>>>>Apenas para finalizar, quer me parecer que toda a missiva se trata de
>>>>um 'ad hominem': uma maneira de tentar desacreditar as idéias de uma
>>>>pessoa atacando a pessoa, sem considerar os méritos das idéias. Entre
>>>>cientistas não deveria ser assim. Quando as teorias são conflitantes,
>>>>o conflito deve ser entregue ao campo das idéias. Pouco importa se o
>>>>proponente ou o atacante da teoria é um papa-hóstias ou um dissoluto
>>>>imoral.
>>>>
>>>>[]s
>>>>--
>>>>César A. K. Grossmann
>>>>http://www.LinuxbyGrossmann.cjb.net/
>>>>
>>>>
>>>>
>>>>
>>>>
>>>>##### ##### #####
>>>>
>>>>Para saber mais visite
>>>>http://www.ciencialist.hpg.ig.com.br
>>>>
>>>>
>>>>##### ##### ##### #####
>>>>Links do Yahoo! Grupos
>>>>
>>>>
>>>>
>>>>
>>>>
>>>>
>>>>
>>>>
>>>>
>>>>
>>>
>>>
>>>##### ##### #####
>>>
>>>Para saber mais visite
>>>http://www.ciencialist.hpg.ig.com.br
>>>
>>>
>>>##### ##### ##### #####
>>>
>>>
>>>*Yahoo! Grupos, um serviço oferecido por:*
>>>
>>>*
>>>
>>>
>>>
>><http://br.rd.yahoo.com/SIG=12ama5o9l/M=264105.3931087.6562589.1588051/D=brclubs/S=2137111528:HM/EXP=1109986216/A=2361264/R=6/SIG=10v4acpp0/*http://br.shopping.yahoo.com/>*
>>
>>
>>>
>>>------------------------------------------------------------------------
>>>*Links do Yahoo! Grupos*
>>>
>>> * Para visitar o site do seu grupo na web, acesse:
>>> http://br.groups.yahoo.com/group/ciencialist/
>>>
>>> * Para sair deste grupo, envie um e-mail para:
>>> ciencialist-unsubscribe@yahoogrupos.com.br
>>>
>>>
>>>
>><mailto:ciencialist-unsubscribe@yahoogrupos.com.br?subject=Unsubscribe>
>>
>>
>>> * O uso que você faz do Yahoo! Grupos está sujeito aos Termos do
>>> Serviço do Yahoo! <http://br.yahoo.com/info/utos.html>.
>>>
>>>
>>>
>>>
>>>__________ Informação do NOD32 1.1017 (20050302) __________
>>>
>>>Esta mensagem foi verificada pelo NOD32 Sistema Antivírus
>>>http://www.nod32.com.br
>>>
>>>
>>
>>
>>##### ##### #####
>>
>>Para saber mais visite
>>http://www.ciencialist.hpg.ig.com.br
>>
>>
>>##### ##### ##### #####
>>Links do Yahoo! Grupos
>>
>>
>>
>>
>>
>>
>>
>>
>>
>>
>
>
>
>
>##### ##### #####
>
>Para saber mais visite
>http://www.ciencialist.hpg.ig.com.br
>
>
>##### ##### ##### #####
>Links do Yahoo! Grupos
>
>
>
>
>
>
>
>
>
>__________ Informação do NOD32 1.1017 (20050302) __________
>
>Esta mensagem foi verificada pelo NOD32 Sistema Antivírus
>http://www.nod32.com.br
>
>
>
>
>



SUBJECT: Re: [ciencialist] Re: Albert Einstein, um hipócrita???????????
FROM: "Oraculo" <oraculo@atibaia.com.br>
TO: <ciencialist@yahoogrupos.com.br>
DATE: 04/03/2005 20:53

Olá ZMIR

Quem levou uma vida de dissolução e orgias foi Santo Agostinho, que se arrependeu no final, claro..:-) Se não pode mais dar maus exemplos, passe a dar bons conselhos.

Santo Agostinho rezava para deus que lhe desse "forças para resistir aos pecados... mas não agora, mais tarde"..:-)

Um abraço.

Homero

----- Original Message -----
From: ZMIR
To: ciencialist@yahoogrupos.com.br
Sent: Friday, March 04, 2005 8:49 PM
Subject: Re: [ciencialist] Re: Albert Einstein, um hipócrita???????????


murilo filo escreveu:

>Victor,
>somos todos bons camaradas!
>Esta estória da risadas x pedrada eu não conheço. É gozação? Vc tem mais
>detalhes?
>abr/M.
>
>

>Olá Murilo,
>
>
O Davi da bíblia, quando pivete, defrontou-se com um guerreiro gigante
e, destemidamente,
tascou-lhe uma tijolada, que o homem caiu duro. Isso é que diz o livro
dos livros. Eu não acredito, conversa prá boi dormir,
daí a alusão em tom de brincadeira. Dr. o gorila realmente dançou, só
pode ter sido de tanto rir, ante a palhaçada. E não lembro a qual dos
dois, Davi ou Salomão, a Bíblia se refereao dizer que teve umas
setecentas mulheres, durante toda a vida. Já no final dela, pertinho de
ir se encontrar com o salvador, arrependeu-se, que não era besta, e
então passou criticar atitudes pecaminosas, do tipo e de outros.
O "vaidade das vaidades", que inicia uma série de coisas santificantes
refere-se a isso. É ou não é " hipocrisia das hipocrisias"?Não dá para
falar sério com
coisas do tipo. Acho que o assunto acabou ficando off-tópic. Páro por aquí.

Sds,

Victor.

>>From: JVictor <jvoneto@uol.com.br>
>>Reply-To: ciencialist@yahoogrupos.com.br
>>To: ciencialist@yahoogrupos.com.br
>>Subject: Re: [ciencialist] Re: Albert Einstein, um hipócrita???????????
>>Date: Fri, 04 Mar 2005 07:32:02 -0300
>>
>>
>>Murilo,
>>
>>Sei as motivações pessoais, e outras razões, para alguém ter
>>interêsses em mostrar certos aspectos meramente humanos de pessoas do
>>nível de Einstein. Mas não vou dar opiniões a respeito. Só digo o
>>seguinte: essas pessoas esquecem que quando apontam com um dedo, há
>>pelos 3 apontando, volta, para ele!
>>Hipocrisia das hipocrisias, diria um famosos hipócritas bíblicos, perto
>>da morte, que fez um gigante rir até morrer, e disseram que foi de uma
>>pedrada.
>>O César está correto. E você também, em concordar com ele.
>>
>>Sds,
>>
>>Victor.
>>
>>
>>
>>murilo filo escreveu:
>>
>>
>>
>>>Concordo com o César ( e pela 1ª vez )!!!
>>>Afinal êle - Einstein - não teria que apresentar nenhuma perfeição
>>>moral e
>>>ética.
>>>Não havia fundado nenhuma religião, como é o caso do Bispo Edir
>>>Macedo, que
>>>muito teria me decepcionado se tivesse tal comportamento... :/
>>>Se formos procurar, até o Gandhi seria algo decepcionante. É o ser
>>>humano,
>>>rapazes! abr/M.
>>>
>>>
>>>
>>>>From: César A. K. Grossmann <cesarakg@bol.com.br>
>>>>Reply-To: ciencialist@yahoogrupos.com.br
>>>>To: ciencialist@yahoogrupos.com.br
>>>>Subject: [ciencialist] Re: Albert Einstein, um hipócrita???????????
>>>>Date: Thu, 03 Mar 2005 18:39:00 -0000
>>>>
>>>>
>>>>
>>>>--- Em ciencialist@yahoogrupos.com.br, "Cyberlander"
>>>><mrcyberlander@i...> escreveu
>>>>
>>>>
>>>>>Segundo aquele dito popular "Casa de ferreiro, espeto de pau", a
>>>>>relação de Einstein com seus filhos não era condizente com suas
>>>>>afirmações públicas, por exemplo, com aquela famosa citação:
>>>>>
>>>>>"Es gibt weder große Entwicklungen noch wahre Fortschritte auf
>>>>>dieser Erde, solange noch ein unglückliches Kind auf ihr lebt."
>>>>>
>>>>>Tradução:
>>>>>< Não existem grandes desenvolvimentos nem tão pouco verdadeiro
>>>>>progresso neste mundo, enquanto ainda nele viver uma criança
>>>>>infeliz. >
>>>>>
>>>>>
>>>>Não vejo relação entre uma coisa e outra, é possível explicar melhor?
>>>>A afirmativa dele refere-se a progresso e desenvolvimento, e, partindo
>>>>do ponto que nem o progresso nem o desenvolvimento contribuem para a
>>>>harmonia do lar, imagino que ele estava se referindo a miséria e
>>>>abandono de quem fica à margem do progresso e do desenvolvimento.
>>>>
>>>>
>>>>
>>>>>Um gênio? Um hipócrita? Tendo em vista que ele é admirado e
>>>>>defendido por Oliveira, RM e coorte, a segunda pergunta tem mais
>>>>>chance de se afirmar positivamente.
>>>>>
>>>>>
>>>>Deixa eu ver, a prova de que ele era um hipócrita reside no fato de
>>>>que ele é admirado e defendido por um certo Oliveira, RM, etc. Isto
>>>>não parece muito lógico. Aliás, não parece nada lógico. "Olha só o
>>>>cara, ele tem que ser um hipócrita, já que beltrano está sempre
>>>>defendendo ele".
>>>>
>>>>Ainda na mesma novela, suponhamos que Einstein fosse um hipócrita. Em
>>>>que isto afetaria a sua genialidade? Quer dizer, se fosse possível
>>>>provar que ele foi hipócrita (o que eu não estou convencido, a mim
>>>>parece apenas que ele não era do tipo expansivo e afetuoso), isto
>>>>implica que ele era um burro? O que uma coisa tem a ver com a outra?
>>>>Um gênio não pode ser hipócrita? São qualidades mutuamente exclusivas?
>>>>
>>>>Apenas para finalizar, quer me parecer que toda a missiva se trata de
>>>>um 'ad hominem': uma maneira de tentar desacreditar as idéias de uma
>>>>pessoa atacando a pessoa, sem considerar os méritos das idéias. Entre
>>>>cientistas não deveria ser assim. Quando as teorias são conflitantes,
>>>>o conflito deve ser entregue ao campo das idéias. Pouco importa se o
>>>>proponente ou o atacante da teoria é um papa-hóstias ou um dissoluto
>>>>imoral.
>>>>
>>>>[]s
>>>>--
>>>>César A. K. Grossmann
>>>>http://www.LinuxbyGrossmann.cjb.net/
>>>>
>>>>
>>>>
>>>>
>>>>
>>>>##### ##### #####
>>>>
>>>>Para saber mais visite
>>>>http://www.ciencialist.hpg.ig.com.br
>>>>
>>>>
>>>>##### ##### ##### #####
>>>>Links do Yahoo! Grupos
>>>>
>>>>
>>>>
>>>>
>>>>
>>>>
>>>>
>>>>
>>>>
>>>>
>>>
>>>
>>>##### ##### #####
>>>
>>>Para saber mais visite
>>>http://www.ciencialist.hpg.ig.com.br
>>>
>>>
>>>##### ##### ##### #####
>>>
>>>
>>>*Yahoo! Grupos, um serviço oferecido por:*
>>>
>>>*
>>>
>>>
>>>
>><http://br.rd.yahoo.com/SIG=12ama5o9l/M=264105.3931087.6562589.1588051/D=brclubs/S=2137111528:HM/EXP=1109986216/A=2361264/R=6/SIG=10v4acpp0/*http://br.shopping.yahoo.com/>*
>>
>>
>>>
>>>------------------------------------------------------------------------
>>>*Links do Yahoo! Grupos*
>>>
>>> * Para visitar o site do seu grupo na web, acesse:
>>> http://br.groups.yahoo.com/group/ciencialist/
>>>
>>> * Para sair deste grupo, envie um e-mail para:
>>> ciencialist-unsubscribe@yahoogrupos.com.br
>>>
>>>
>>>
>><mailto:ciencialist-unsubscribe@yahoogrupos.com.br?subject=Unsubscribe>
>>
>>
>>> * O uso que você faz do Yahoo! Grupos está sujeito aos Termos do
>>> Serviço do Yahoo! <http://br.yahoo.com/info/utos.html>.
>>>
>>>
>>>
>>>
>>>__________ Informação do NOD32 1.1017 (20050302) __________
>>>
>>>Esta mensagem foi verificada pelo NOD32 Sistema Antivírus
>>>http://www.nod32.com.br
>>>
>>>
>>
>>
>>##### ##### #####
>>
>>Para saber mais visite
>>http://www.ciencialist.hpg.ig.com.br
>>
>>
>>##### ##### ##### #####
>>Links do Yahoo! Grupos
>>
>>
>>
>>
>>
>>
>>
>>
>>
>>
>
>
>
>
>##### ##### #####
>
>Para saber mais visite
>http://www.ciencialist.hpg.ig.com.br
>
>
>##### ##### ##### #####
>Links do Yahoo! Grupos
>
>
>
>
>
>
>
>
>
>__________ Informação do NOD32 1.1017 (20050302) __________
>
>Esta mensagem foi verificada pelo NOD32 Sistema Antivírus
>http://www.nod32.com.br
>
>
>
>
>



##### ##### #####

Para saber mais visite
http://www.ciencialist.hpg.ig.com.br


##### ##### ##### #####


Yahoo! Grupos, um serviço oferecido por:







------------------------------------------------------------------------------
Links do Yahoo! Grupos

a.. Para visitar o site do seu grupo na web, acesse:
http://br.groups.yahoo.com/group/ciencialist/

b.. Para sair deste grupo, envie um e-mail para:
ciencialist-unsubscribe@yahoogrupos.com.br

c.. O uso que você faz do Yahoo! Grupos está sujeito aos Termos do Serviço do Yahoo!.



[As partes desta mensagem que não continham texto foram removidas]



SUBJECT: Re: Porque o oxigênio é invisível
FROM: "Ricardo Soares Vieira" <rickrsv@yahoo.com.br>
TO: ciencialist@yahoogrupos.com.br
DATE: 05/03/2005 00:53


Alow Terraqueos

PSSC?!

Em que mundo estou?, esse Livro é muito bom, mas eu nem sabia que
ele era famoso (hehe), vejam: eu comprei ele a 1 ano mais ou menos,
e adivinha quantos eu paqguei (.....)

R$ 1,00 em um brechó, lá tinha os outros volumes, mas eu só comprei
esse...

Realmente ele é muito bom, deveria ter comprado todos...

Ah!k, sim, ele é em português!!!

Até.

--- Em ciencialist@yahoogrupos.com.br, Maria Natália <grasdic@h...>
escreveu
>
>
> O PSSC!!!!!!!!!!!
> Como foi possível esquecer essa bíblia? E tu a tens aí desde 196...
> És português?
> Tá bom para explicar a moçinho.
> Obrigada
> Maria Natália
>
> --- Em ciencialist@yahoogrupos.com.br, "Ricardo Soares Vieira"
> <rickrsv@y...> escreveu
> >
> >
> > Olá Maria Natália
> >
> > Porque o oxigênio é invisível?
> >
> > Eu entendi isso a pouco tempo... uma resposta aceitável, senão
> > conclusiva, sobre esse assunto eu encontrei em um livro antigo
aqui
> > em casa:
> >
> > Para responder, temos que apelar para teorias atômicas,
> > especificamente o modelo de Niels Bohr. Se você estuda física,
deve
> > saber que apenas algumas órbitas são permitidas, e que os
elétrons
> > só absolvem ou emitem radiação se a freqüência desta radiação
> > estiver em ressonância com a freqüência do elétron (calculada
pela
> > equação de De Broglie - Einstein: f = h/E). Observe que quanto
maior
> > for a freqüência da luz, maior é a energia de seu photon.
> >
> > Podemos definir uma substancia transparente como aquela na qual
a
> > luz à atravessa completamente, e uma opaca, àquela que absolve
a
> > luz. Mas no nível atômico, um elemento só pode absolver luz se
esta
> > tiver energia suficiente para levar o seu elétron do estado
> > fundamental (primeira órbita) à seguinte.
> >
> > A par disso, encontramos o porque do oxigênio ser transparente:
como
> > está no livro, a luz visível (cuja freqüência é relativamente
baixa)
> > não tem energia suficiente para escitar o atomo de oxigenio,
logo
> > ela o atravessa. Mas uma luz de freqüência maior será absolvida,
e
> > logo, ela deixará de ser transparente para essa cor de luz.
(vide
> > espectro do oxigênio).
> >
> > Espero ter ajudado, acho que essa resposta dada naquele livro
> > (Physical Science Study Committee, parte 4 pg 213) explica o
> > fenômeno não?
> >
> > Até Mais
> >
> > Rick





SUBJECT: Re: dormindo 3:30h por dia...
FROM: Maria Natália <grasdic@hotmail.com>
TO: ciencialist@yahoogrupos.com.br
DATE: 05/03/2005 00:58


Ivan:
Estou quase como tu. Estou em escola em que tenho de apanhar três a
4 onibus para chegar. São 3 horas de transporte e professor e mulher
tem trabalho de casa. Me desforro ao fds em que durmo até meio
dia...mas está arrasando. E depois tem amigos brasileiros e nos EUA
com diferença de fuso horário, respectivamente, 3 e 5 h!!!
Vou ter que por isto em ordem por altura das férias da Páscoa. Mas
Eurico tem razão...
Dormir no bus até dá mas parece mal, pode passar local de paragem e
ainda ser assaltado.
Por ser astrónoma estou habituada e lá vou indo.Malhação tá falhando e
eu começo a ficar saturada.
Carrego pilhas uma vez por semana dormindo 10h!!! Mas nem TV já vejo.
E à propos: tenho de ir.
Boa noite pessoal.
Um abraço
Maria Natália
--- Em ciencialist@yahoogrupos.com.br, "Ivan Carlos" <icarlos@i...>
escreveu
> se eu aprender a dormir no ônibus ajuda? =)
>
> passo 1 hora dentro do onibus na ida, depois + 30 mins, depois +
metro...
>
> na volta mais 30mins + 1 hora de bus... ^^
>
> Ivan "Doomer" Carlos
> Social Engineering Specialist
> Cell.: +55 (11) 8112-0666
> icarlos@i...
> www.icarlos.net
> --------------------------------------------------
>
> ----- Original Message -----
> From: "Eurico Ferreira de Souza Jr." <caodejah@y...>
> To: <ciencialist@yahoogrupos.com.br>
> Sent: Friday, March 04, 2005 7:09 PM
> Subject: [ciencialist] dormindo 3:30h por dia...
>
>
>
>
>
> Ivan Carlos <icarlos@i...> wrote:
>
> Estou dormindo 3:30hs por dia (trabalho + facul)
>
> OBVIAMENTE antes das 8 e após as 20 horas, me sinto totalmente esgotado,
> depois de um tempo a sensação passa e retorna gradativamente
>
> Preciso enfrentar esta maratona até 2007...
>
> Não existe possibilidade de trocar horário de estudo, trabalho,
morar mais
> perto dos estabelecimentos, etc.. por mais q os médicos me xinguem ><
>
> Consultei um clínico geral, ele sugeriu um complexo vitamínico, mas
tenho
> sérias dúvidas quanto a eficácia desse negócio (tanto pela
metabolização qto
> pelo aproveitamente deles pelo corpo), alguém tem alguma sugestão
nesse caso
> tbém? Energético a base de guaraná acho q naum vai ajudar tbém... rs
>
> desculpe ficar atormentando vcs, voltemos a programação normal :)
>
> [E]> sem querer "gorar", mas acho que vc vai adoecer e envelhecer mais
> rápido, se não pirar...
>
> fiquei um ano dormindo 4 horas e me zoou bastante e
> definitivamente...(dormir depois não vai tirar o atraso) tente mudar de
> vida, enquanto tem uma. boa sorte e um forte abraço.
>
>
>
>
> _\|/_
> __________________________________________________
> Converse com seus amigos em tempo real com o Yahoo! Messenger
> http://br.download.yahoo.com/messenger/
>
> [As partes desta mensagem que não continham texto foram removidas]
>
>
>
> ##### ##### #####
>
> Para saber mais visite
> http://www.ciencialist.hpg.ig.com.br
>
>
> ##### ##### ##### #####
> Links do Yahoo! Grupos





SUBJECT: Re: [ciencialist] O ÍNDICE CRACKPOT
FROM: "Alberto Mesquita Filho" <albmesq@uol.com.br>
TO: <ciencialist@yahoogrupos.com.br>
DATE: 05/03/2005 00:59

----- Original Message -----
From: "Oraculo"
Sent: Friday, March 04, 2005 8:21 PM
Subject: [ciencialist] O ÍNDICE CRACKPOT

> O ÍNDICE CRACKPOT
> de John Baez

Esse assunto chegou a ser debatido aqui na Ciencialist em novembro de 2000 e
as mensagens (que foram poucas, por sinal) podem ser lidas a partir de
http://ecientificocultural.com/ECC2/Dialogos/pot.htm

Vou reproduzir apenas um trecho extraído da msg 6942:

********* início da citação **********

"Os crackpots existem para atrapalhar a vida dos teorizadores sérios. O
problema é que não há como reconhecer um crackpot genuíno de um ser criativo
e que acredita em seu potencial (talvez haja como reconhecer mas é muito
difícil e não é uma tabelinha feita por um humorista idiota como o Baez --a
esse respeito prefiro o Jô Soares-- que vai nos levar a alguma coisa). O
Baez parece-me ser americano e como todo americano tem suas idéias fixas: ou
matar o presidente, ou ingressar na lista dos dez mais alguma coisa ou então
criar a sua própria lista dos dez mais alguma coisa (estes últimos disputam
a entrada na lista dos dez mais idiotas). Apesar disso, muita coisa do que o
Baez afirma tem sua razão de ser (ele perdeu a chance de escrever um bom
texto a respeito, o que foi feito pelo Bernstein que é um físico de fato).

Logo, ou acabamos com a teorização ou aprendemos a conviver com os
crackpots. Esse é o meu ponto de vista. O texto do Bernstein talvez seja
útil para reconhecermos os mais típicos, aqueles que talvez estejam mesmo
precisando de um médico, pois não é impossível que o genuíno crackpotismo
seja a manifestação inicial de uma doença muito séria. Não concordo com toda
a visão do Bernstein (sou popperiano de carteirinha :-) ) mas acho que é um
bom texto."

********* fim da citação **********

Em tempo: O texto do Bernstein também pode ser lido a partir do link
indicado acima.

[ ]´s
Alberto
http://ecientificocultural.com/indice.htm
Mas indiferentemente a tudo isso, o neutrino tem massa, o elétron não é
uma carga elétrica coulombiana e a Terra se move. E a história se repetirá.



SUBJECT: Re: Albert Einstein, um hipócrita???????????
FROM: Maria Natália <grasdic@hotmail.com>
TO: ciencialist@yahoogrupos.com.br
DATE: 05/03/2005 01:07


Pois, pois. E o comportamento de Gandi com as meninas, velho pegajoso
de menina virgem.Era só luxúria... E vou começar a má língua:
Ou não? Pois claro que não. Quero lá saber que Newton era execrável
para as mulheres? E que Xantipa tratava mal um tal de filósofo...Em
ciência não me comporto como certas pessoas no futebol e que até de
cuecas de jogador falam.
Mas que até já chegou à escolinha esta do Eisntein chegou. E já há
menino nas PUCs a dizer que não estudará mais esse libertino e
desnaturado...Desculpas é o que é.
Um abraço pois Eurico disse que tenho de dormir fora do teclado.
Maria Natália, na fase alcoviteira


--- Em ciencialist@yahoogrupos.com.br, "murilo filo"
<avalanchedrive@h...> escreveu
> Concordo com o César ( e pela 1ª vez )!!!
> Afinal êle - Einstein - não teria que apresentar nenhuma perfeição
moral e
> ética.
> Não havia fundado nenhuma religião, como é o caso do Bispo Edir
Macedo, que
> muito teria me decepcionado se tivesse tal comportamento... :/
> Se formos procurar, até o Gandhi seria algo decepcionante. É o ser
humano,
> rapazes! abr/M.
>
> >From: César A. K. Grossmann <cesarakg@b...>
> >Reply-To: ciencialist@yahoogrupos.com.br
> >To: ciencialist@yahoogrupos.com.br
> >Subject: [ciencialist] Re: Albert Einstein, um hipócrita???????????
> >Date: Thu, 03 Mar 2005 18:39:00 -0000
> >
> >
> >
> >--- Em ciencialist@yahoogrupos.com.br, "Cyberlander"
> ><mrcyberlander@i...> escreveu
> > >
> > > Segundo aquele dito popular "Casa de ferreiro, espeto de pau", a
> > > relação de Einstein com seus filhos não era condizente com suas
> > > afirmações públicas, por exemplo, com aquela famosa citação:
> > >
> > > "Es gibt weder große Entwicklungen noch wahre Fortschritte auf
> > > dieser Erde, solange noch ein unglückliches Kind auf ihr lebt."
> > >
> > > Tradução:
> > > < Não existem grandes desenvolvimentos nem tão pouco verdadeiro
> > > progresso neste mundo, enquanto ainda nele viver uma criança
> > > infeliz. >
> >
> >Não vejo relação entre uma coisa e outra, é possível explicar melhor?
> >A afirmativa dele refere-se a progresso e desenvolvimento, e, partindo
> >do ponto que nem o progresso nem o desenvolvimento contribuem para a
> >harmonia do lar, imagino que ele estava se referindo a miséria e
> >abandono de quem fica à margem do progresso e do desenvolvimento.
> >
> > > Um gênio? Um hipócrita? Tendo em vista que ele é admirado e
> > > defendido por Oliveira, RM e coorte, a segunda pergunta tem mais
> > > chance de se afirmar positivamente.
> >
> >Deixa eu ver, a prova de que ele era um hipócrita reside no fato de
> >que ele é admirado e defendido por um certo Oliveira, RM, etc. Isto
> >não parece muito lógico. Aliás, não parece nada lógico. "Olha só o
> >cara, ele tem que ser um hipócrita, já que beltrano está sempre
> >defendendo ele".
> >
> >Ainda na mesma novela, suponhamos que Einstein fosse um hipócrita. Em
> >que isto afetaria a sua genialidade? Quer dizer, se fosse possível
> >provar que ele foi hipócrita (o que eu não estou convencido, a mim
> >parece apenas que ele não era do tipo expansivo e afetuoso), isto
> >implica que ele era um burro? O que uma coisa tem a ver com a outra?
> >Um gênio não pode ser hipócrita? São qualidades mutuamente exclusivas?
> >
> >Apenas para finalizar, quer me parecer que toda a missiva se trata de
> >um 'ad hominem': uma maneira de tentar desacreditar as idéias de uma
> >pessoa atacando a pessoa, sem considerar os méritos das idéias. Entre
> >cientistas não deveria ser assim. Quando as teorias são conflitantes,
> >o conflito deve ser entregue ao campo das idéias. Pouco importa se o
> >proponente ou o atacante da teoria é um papa-hóstias ou um dissoluto
> >imoral.
> >
> >[]s
> >--
> >César A. K. Grossmann
> >http://www.LinuxbyGrossmann.cjb.net/
> >
> >
> >
> >
> >
> >##### ##### #####
> >
> >Para saber mais visite
> >http://www.ciencialist.hpg.ig.com.br
> >
> >
> >##### ##### ##### #####
> >Links do Yahoo! Grupos
> >
> >
> >
> >
> >
> >
> >
> >





SUBJECT: Re: Qual é o tamanho do fóton??????
FROM: Maria Natália <grasdic@hotmail.com>
TO: ciencialist@yahoogrupos.com.br
DATE: 05/03/2005 01:12


Mas Hélio:
Frequência não é tamanho...rs rs rsssssssssss. E não se trata sequer
de ondas mecãnicas.
Ainda pensei que era bricadeira de menina de 12 anos com pergunta
crápula de professorinha.
Claro que te entendo só noto uma concepçâo alternativa aí.
Um abraço
Maria Natália



--- Em ciencialist@yahoogrupos.com.br, Hélio Ricardo Carvalho
<hrc@f...> escreveu
>
> Qual é o tamanho do fóton??????
>
> Baseado no texto de:
> http://br.groups.yahoo.com/group/ciencialist/message/44700
>
> Se o fóton tem informação de freqüência e o comprimento de onda
> típico para o visível é ~500nm (verde). O fóton "verde" deveria ter
> um comprimento maior que 500nm.
> Sim ou não?
>
> Hélio





SUBJECT: Re: Fw: Íon H
FROM: Maria Natália <grasdic@hotmail.com>
TO: ciencialist@yahoogrupos.com.br
DATE: 05/03/2005 01:24


Minha querida amiga:

A partícula H+ tem vida curta neste mundo tão cheio de iões. Sendo
assim ela se liga quase logo a moléculas de compostos polares. Temos
água na atmosfera, na pele, etc. E como molécula da água tem
polaridade este H+ se liga na parte electronegativa ficando H3O+.
Para compreende bem isto que lhe digo deverá ser um ás em ligação
química. O amoníaco NH3 também tem essa propriedade de se ligar a H+
dando o ião amóniO, NH4+ (não digo amóniA que é hidróxido de amónio).
Além disso tem de ver: a massa do protão (H+) é muito pequena logo
esta partícula não pára e nessa mobilidade não escapa a molécula polar
"gulosa" de H+. As más linguas até chamam às substancias que recebem
H+ de bases ou hidróxidos.
Como vê tudo termina em bem. Agora basta pegar num livro e refinar a
línguagem pois a que usei foi para compreender o porquê. Agora vamos
apenas aprimorar o discurso
Bom fds, com a agradável companhia da química
Maria Natália


--- Em ciencialist@yahoogrupos.com.br, "Luiz Ferraz Netto"
<leobarretos@u...> escreveu
> Não é química ... mas pode ser respondido por cuímicos :-))
>
> []'
> ===========================
> Luiz Ferraz Netto [Léo]
> leobarretos@u...
> http://www.feiradeciencias.com.br
> ===========================
> -----Mensagem Original-----
> De: guiarte
> Para: leobarretos
> Enviada em: quinta-feira, 3 de março de 2005 20:46
> Assunto: Íon H
>
>
> Professor Leo,
>
> Parabém por sua utilíssima página.
>
> Gostaria que me esclarecesse uma dúvida:
>
> Como se forma


o íon H+, pois, como sabemos, ele apresenta apenas um próton e um
elétron. Se perder seu único elétron, ficará apenas um próton; ou
seja, uma partícula solta no espaço...
>
> Será isso possível, ou a formação de tal íon é a partir do gás
hidrogênio (H2).
>
> Por favor, aguardo uma resposta que não tenho encontrado nos livros.
>
> Grato,
>
> Prof. Vilson
>
>
>
--------------------------------------------------------------------------------
>
>
> Internal Virus Database is out-of-date.
> Checked by AVG Anti-Virus.
> Version: 7.0.300 / Virus Database: 266.5.0 - Release Date: 25/02/2005
>
> ----------
>
> Internal Virus Database is out-of-date.
> Checked by AVG Anti-Virus.
> Version: 7.0.300 / Virus Database: 266.5.0 - Release Date: 25/02/2005
>
>
> [As partes desta mensagem que não continham texto foram removidas]





SUBJECT: Re: Fw: nascimento do atomo
FROM: Manuel Bulcão <manuelbulcao@uol.com.br>
TO: ciencialist@yahoogrupos.com.br
DATE: 05/03/2005 06:47


--- Em ciencialist@yahoogrupos.com.br, "Luiz Ferraz Netto"
<leobarretos@u...> escreveu
> como nascem os átomos?

Manuel: Por meio de três classes de nucleossínteses: a primordial, a
estelar (que se divide em nucleossínteses quiescente e explosiva) e
a interestelar.

Tendo como válida a teoria do big-bang, a nucleossíntese primordial
deu-se durante os mil primeiros segundos, sendo ela responsável pelo
surgimento dos átomos de hidrogênio, deutério (isótopo estável do
hidrogênio constituído de um próton e um nêutron) e, por meio de
fusão nuclear, hélio e uma pequena quantidade de lítio-7. O processo
parou por aí porque, após 1000 s, as condições de temperatura e
pressão já não eram mais propícias.

No interior das estrelas, através de reações termonucleares (fusão
dos átomos de hidrogênio e, em seguida, a queima do hélio, etc.)
foram e estão sendo sintetizados os elementos mais pesados até o
ferro, cujo número atômico é 26 (nucleossíntese quiescente).

Algumas estrelas com massa maior que a do Sol 10 vezes ou mais, após
extinguir-se o seu combustível nuclear, entram em colapso
gravitacional: seu núcleo transforma-se numa estrela de nêutrons ou
mesmo num buraco negro, enquanto que sua camada externa é ejetada no
espaço numa grande explosão, fenômeno este conhecido
como 'supernova'. A energia gerada por essas explosões é intensa o
suficiente para que se formem os elementos mais pesados que o ferro,
como o cobre, o zinco, o ouro... até o urânio, cujo número atômico é
92. É esta a nucleossíntese explosiva.

A nucleossíntese interestelar se dá através da interação dos raios
cósmicos (elétrons, prótons, núcleos de hélio e outras partículas
mais pesadas movimentando-se em grande velocidade) com a poeira ou
gás interestelar formado por hidrogênio, hélio e, em menor
proporção, por elementos pesados. Quando os raios cósmicos impactam
esses elementos pesados, estes se fragmentam e dão origem a
elementos mais leves como o lítio-6 (3 prótons + 3 nêutrons), o
berílio e o boro. (A energia gerada pela interação entre poeira e
raios cósmicos é insuficiente para desencadear processos de fusão
nuclear.)

Abraços,
Manuel Bulcão







SUBJECT: Re: Albert Einstein, um hipócrita???????????
FROM: Manuel Bulcão <manuelbulcao@uol.com.br>
TO: ciencialist@yahoogrupos.com.br
DATE: 05/03/2005 07:20


--- Em ciencialist@yahoogrupos.com.br, "Alvaro Augusto \(E\)"
<alvaro@e...> escreveu
> Einstein era hipócrita? Bem, todos nós somos, pois, como mostrou
outro judeu (Freud), não temos acesso a todo o conteúdo da nossa
psique. Logo, não podemos conhecer toda a verdade sobre o que
pensamos. Logo, não podemos agir sempre de acordo com o que
pensamos...

Manuel: Vale citar duas sentenças de Freud: "um pouco de
transgressão faz bem à saúde", e "poderíamos ser muito melhores se
não quiséssemos ser tão bons". (Estou citando de cabeça, não me
lembro agora os livros em que se encontram.)

Abraços,
Manuel Bulcão





SUBJECT: Re: Fw: nascimento do atomo
FROM: Manuel Bulcão <manuelbulcao@uol.com.br>
TO: ciencialist@yahoogrupos.com.br
DATE: 05/03/2005 07:48


Na nucleossíntese primordial também foram produzidos núcleos de
berílio (Z=4) por meio da seguinte reação:

3He + 4He > 7Be + y

Ocorre que o berílio-7 (4 prótons + 3 nêutrons) é radioativo, isto
é, instável, gerando, ao decair, um núcleo de lítio-7 (3 prótons + 4
nêutrons).

[]s
Manuel Bulcão





SUBJECT: Re: nascimento do atomo (adendo)
FROM: Manuel Bulcão <manuelbulcao@uol.com.br>
TO: ciencialist@yahoogrupos.com.br
DATE: 05/03/2005 08:02


Importante observar, ainda, que os átomos propriamente ditos só
vieram a se formar um milhão de anos após o big-bang, quando os
fótons se desacoplaram da matéria (a origem da radiação cósmica de
fundo), o que tornou possível a combinação dos elétrons com os
núcleos.

[]s
Manuel Bulcão






SUBJECT: Re: [ciencialist] dormindo 3:30h por dia...
FROM: "Eurico Ferreira de Souza Jr." <caodejah@yahoo.com.br>
TO: ciencialist@yahoogrupos.com.br
DATE: 05/03/2005 11:25



Ivan Carlos <icarlos@icarlos.net> wrote:
se eu aprender a dormir no ônibus ajuda? =)

passo 1 hora dentro do onibus na ida, depois + 30 mins, depois + metro...

na volta mais 30mins + 1 hora de bus... ^^

[E]> creio que sim... :(

Ivan "Doomer" Carlos
Social Engineering Specialist

[E]> o que um especialista em engenharia social faz? :D





_\|/_

---------------------------------
Yahoo! Acesso Grátis - Internet rápida e grátis. Instale o discador do Yahoo! agora.

[As partes desta mensagem que não continham texto foram removidas]



SUBJECT: Re: Qual é o tamanho do fóton??????
FROM: Hélio Ricardo Carvalho <hrc@fis.puc-rio.br>
TO: ciencialist@yahoogrupos.com.br
DATE: 05/03/2005 13:45


Em ciencialist Maria Natália escreveu
>
> Mas Hélio:
> Frequência não é tamanho...rs rs rsssssssssss.
>

É costume (ou era) a freqüência e o tamanho de algo que tenha
freqüência estarem relacionados pela velocidade.

>
> E não se trata sequer
> de ondas mecãnicas.
>

Realmente não se trata de ondas mecânicas. Apesar de a maioria dos
cietistas terem pensado nelas assim durante uns dois séculos.

>
> Ainda pensei que era bricadeira de menina de 12 anos com pergunta
> crápula de professorinha.
>

Pode ser mais ou menos isto mas tirando a palavra "crápula" pois ela
tem um peso muito negativo e, de longe, não foi esta a intenção.
Seria assim:
Uma pergunta aparentemente boba feita pelo(a) professor(a) para fazer
pensar.

>
> Claro que te entendo só noto uma concepçâo alternativa aí.
>

Na verdade existem várias concepções alternativas para o fóton.
Confesso que preciso aprender mais sobre afinal quais delas são as
mais "oficiais".
ATENÇÃO: Só estou levando em conta as alternativas que não são
contrariadas pelos experimentos.


>
> Um abraço
> Maria Natália
>

Outro abraço
Hélio





SUBJECT: Re: Qual é o tamanho do fóton??????
FROM: Maria Natália <grasdic@hotmail.com>
TO: ciencialist@yahoogrupos.com.br
DATE: 05/03/2005 15:06


Hélio:
Apesar de estar nas ONAs e cheia de trabalho quero dizer-te que o
crápula empregue por mim o foi pelo som. Gostei do acentuação tónica
no á. LOL
Vem a propósito de professorinhas que não querendo fazer elas a
pesquisa manda imberbes fazê-las. E isso tem um nome muito mais
feio...(se usa no LMON ali em Campinas) LOL
Já deve ter lido certas perguntas para trabalhos de casa de meninos de
12 e 14 anos. Não se faz e foi apenas nesse sentido. Devia ter dito
mercenárias, desculpe
Tudo de bom,
Maria Natália



--- Em ciencialist@yahoogrupos.com.br, Hélio Ricardo Carvalho
<hrc@f...> escreveu
>
> Em ciencialist Maria Natália escreveu
> >
> > Mas Hélio:
> > Frequência não é tamanho...rs rs rsssssssssss.
> >
>
> É costume (ou era) a freqüência e o tamanho de algo que tenha
> freqüência estarem relacionados pela velocidade.
>
> >
> > E não se trata sequer
> > de ondas mecãnicas.
> >
>
> Realmente não se trata de ondas mecânicas. Apesar de a maioria dos
> cietistas terem pensado nelas assim durante uns dois séculos.
>
> >
> > Ainda pensei que era bricadeira de menina de 12 anos com pergunta
> > crápula de

professorinha.
> >
>
> Pode ser mais ou menos isto mas tirando a palavra "crápula" pois ela
> tem um peso muito negativo e, de longe, não foi esta a intenção.
> Seria assim:
> Uma pergunta aparentemente boba feita pelo(a) professor(a) para fazer
> pensar.
>
> >
> > Claro que te entendo só noto uma concepçâo alternativa aí.
> >
>
> Na verdade existem várias concepções alternativas para o fóton.
> Confesso que preciso aprender mais sobre afinal quais delas são as
> mais "oficiais".
> ATENÇÃO: Só estou levando em conta as alternativas que não são
> contrariadas pelos experimentos.
>
>
> >
> > Um abraço
> > Maria Natália
> >
>
> Outro abraço
> Hélio





SUBJECT: Re: dormindo 3:30h por dia...
FROM: Maria Natália <grasdic@hotmail.com>
TO: ciencialist@yahoogrupos.com.br
DATE: 05/03/2005 15:15


Também quero saber o que é engenharia social? Trabalha também com
especialista em clonagem? LOL
A não ser que vá à Ordem dos engenheiros saber o que é qual a
universidade e que tipo de empregos tem. Mas mais lá para o Verão.
Ivan se puderes dizer, agradeço
Um abraço
Maria Natália



--- Em ciencialist@yahoogrupos.com.br, "Eurico Ferreira de Souza Jr."
<caodejah@y...> escreveu
>
>
> Ivan Carlos <icarlos@i...> wrote:
> se eu aprender a dormir no ônibus ajuda? =)
>
> passo 1 hora dentro do onibus na ida, depois + 30 mins, depois +
metro...
>
> na volta mais 30mins + 1 hora de bus... ^^
>
> [E]> creio que sim... :(
>
> Ivan "Doomer" Carlos
> Social Engineering Specialist
>
> [E]> o que um especialista em engenharia social faz? :D
>
>
>
>
> Também quero saber o que é engenharia social? Trabalha também com
especialista em clonagem? LOL


> _\|/_
>
> ---------------------------------
> Yahoo! Acesso Grátis - Internet rápida e grátis. Instale o discador
do Yahoo! agora.
>
> [As partes desta mensagem que não continham texto foram removidas]





SUBJECT: RE: [ciencialist] "ENGENHARIA ALTERNATIVA" UMA PARÁBOLA PÓS-MODERNA
FROM: "murilo filo" <avalanchedrive@hotmail.com>
TO: ciencialist@yahoogrupos.com.br
DATE: 05/03/2005 18:05

Ninguém comentou êsse treco do Oráculo / Waterstone... Êste último,
definitivamente um ser liberto e que não ''é ciência encarnada'', nem
professor de nada e nem ''reservatório do saber''.
Há uma frase em destaque abaixo e quase todos vão achar impossível êle não
ser pirado.
Esta é uma frase de quem usa a ciência como uma ferramenta em suas
concepções, e eu mesmo já disse coisa parecida, modestamente, sobre minha
pessoa.
Pena que não dá p/se ter só idéias aproveitáveis, sempre.
Como um pirado normal, acho que o tal Waterstone é doidão mesmo! abr/M.

>From: "Oraculo" <oraculo@atibaia.com.br>
>Reply-To: ciencialist@yahoogrupos.com.br
>To: <ciencialist@yahoogrupos.com.br>
>Subject: [ciencialist] "ENGENHARIA ALTERNATIVA" UMA PARÁBOLA PÓS-MODERNA
>Date: Fri, 4 Mar 2005 17:58:03 -0300
>
>
>
>"ENGENHARIA ALTERNATIVA" UMA PARÁBOLA PÓS-MODERNA
>de Steven Novella, M.D.
>
>Nota: As pessoas mencionadas nesta história são fictícias, mas os perigos
>da pseudociência aplicada são reais.
>
>Um novo fenômeno está invadido os EUA, ganhando a atenção tanto dos
>consumidores como dos fabricantes. Cada vez mais decepcionadas com o frio
>mundo metálico da tecnologia moderna, as pessoas estão olhando mais
>atentamente para alternativas mais naturais. Coletivamente chamada de
>Engenharia Alternativa ("Alt Eng"), uma grande variedade de métodos novos e
>antigos está ganhando respeitabilidade científica e jornalística.
>
>>>>>>>>>>>>Alec Waterstone é um desses supostos engenheiros alternativos.
>>>>>>>>>>>>Ele não tem nenhum diploma ou formação acadêmica em engenharia,
>>>>>>>>>>>>o que, ele explica, é uma vantagem. "Meu pensamento não é
>>>>>>>>>>>>limitado pela matemática, lógica ou qualquer enfadonho antigo
>>>>>>>>>>>>paradigma mecanicista. Não tenho que render homenagens às
>>>>>>>>>>>>preferências de Newton ou de outros pedagogos ocidentais. A
>>>>>>>>>>>>ausência completa de treinamento me deixa livre para pensar em
>>>>>>>>>>>>soluções únicas e inovadoras para problemas de engenharia,
>>>>>>>>>>>>liberto dos irritantes limites da
>>>>>>>>>>>>'realidade.'<<<<<<<<<<<<<<<<<<<!!!!!!!!!!!!
>
>Pontes Baseadas em Energia
>
>O último projeto de Alec é uma ponte não-pênsil de 460 m. Ele afirma que a
>ponte será capaz de




SUBJECT: Re: [ciencialist] "ENGENHARIA ALTERNATIVA" UMA PARÁBOLA PÓS-MODERNA
FROM: "Oraculo" <oraculo@atibaia.com.br>
TO: <ciencialist@yahoogrupos.com.br>
DATE: 05/03/2005 19:59

Olá Murilo

Espero que não fique ofendido, mas eu acho que talvez não tenha percebido o sentido ironico e sarcástico do texto. E talvez tenha deixado passar o alerta inicial: As pessoas são ficticias..:-) O Dr. Waterstone não existe, embora esse tipo de corrente alternativa esteja cada dia mais forte..:-)

O que o texto, humoristicametne, tentou apresentar, foi um tipo de "reductio ad absurdum", ao aplicar os conceitos tipo "new age" a questões sérias e reais, como a engenharia. Tudo bem usar esse tipo de bobagem para arrumar os móveis da sala de forma a usar a "energia cósmica" do universo, mas levar à sério em situações de risco ou que dependem de confiabilidade, é uma coisa perigosa..:-)

Um abraço.

Homero

----- Original Message -----
From: murilo filo
To: ciencialist@yahoogrupos.com.br
Sent: Saturday, March 05, 2005 6:05 PM
Subject: RE: [ciencialist] "ENGENHARIA ALTERNATIVA" UMA PARÁBOLA PÓS-MODERNA


Ninguém comentou êsse treco do Oráculo / Waterstone... Êste último,
definitivamente um ser liberto e que não ''é ciência encarnada'', nem
professor de nada e nem ''reservatório do saber''.
Há uma frase em destaque abaixo e quase todos vão achar impossível êle não
ser pirado.
Esta é uma frase de quem usa a ciência como uma ferramenta em suas
concepções, e eu mesmo já disse coisa parecida, modestamente, sobre minha
pessoa.
Pena que não dá p/se ter só idéias aproveitáveis, sempre.
Como um pirado normal, acho que o tal Waterstone é doidão mesmo! abr/M.

>From: "Oraculo" <oraculo@atibaia.com.br>
>Reply-To: ciencialist@yahoogrupos.com.br
>To: <ciencialist@yahoogrupos.com.br>
>Subject: [ciencialist] "ENGENHARIA ALTERNATIVA" UMA PARÁBOLA PÓS-MODERNA
>Date: Fri, 4 Mar 2005 17:58:03 -0300
>
>
>
>"ENGENHARIA ALTERNATIVA" UMA PARÁBOLA PÓS-MODERNA
>de Steven Novella, M.D.
>
>Nota: As pessoas mencionadas nesta história são fictícias, mas os perigos
>da pseudociência aplicada são reais.
>
>Um novo fenômeno está invadido os EUA, ganhando a atenção tanto dos
>consumidores como dos fabricantes. Cada vez mais decepcionadas com o frio
>mundo metálico da tecnologia moderna, as pessoas estão olhando mais
>atentamente para alternativas mais naturais. Coletivamente chamada de
>Engenharia Alternativa ("Alt Eng"), uma grande variedade de métodos novos e
>antigos está ganhando respeitabilidade científica e jornalística.
>
>>>>>>>>>>>>Alec Waterstone é um desses supostos engenheiros alternativos.
>>>>>>>>>>>>Ele não tem nenhum diploma ou formação acadêmica em engenharia,
>>>>>>>>>>>>o que, ele explica, é uma vantagem. "Meu pensamento não é
>>>>>>>>>>>>limitado pela matemática, lógica ou qualquer enfadonho antigo
>>>>>>>>>>>>paradigma mecanicista. Não tenho que render homenagens às
>>>>>>>>>>>>preferências de Newton ou de outros pedagogos ocidentais. A
>>>>>>>>>>>>ausência completa de treinamento me deixa livre para pensar em
>>>>>>>>>>>>soluções únicas e inovadoras para problemas de engenharia,
>>>>>>>>>>>>liberto dos irritantes limites da
>>>>>>>>>>>>'realidade.'<<<<<<<<<<<<<<<<<<<!!!!!!!!!!!!
>
>Pontes Baseadas em Energia
>
>O último projeto de Alec é uma ponte não-pênsil de 460 m. Ele afirma que a
>ponte será capaz de




##### ##### #####

Para saber mais visite
http://www.ciencialist.hpg.ig.com.br


##### ##### ##### #####


Yahoo! Grupos, um serviço oferecido por:







------------------------------------------------------------------------------
Links do Yahoo! Grupos

a.. Para visitar o site do seu grupo na web, acesse:
http://br.groups.yahoo.com/group/ciencialist/

b.. Para sair deste grupo, envie um e-mail para:
ciencialist-unsubscribe@yahoogrupos.com.br

c.. O uso que você faz do Yahoo! Grupos está sujeito aos Termos do Serviço do Yahoo!.



[As partes desta mensagem que não continham texto foram removidas]



SUBJECT: ''micobuster" ENGENHARIA ALTERNATIVA" UMA PARÁBOLA PÓS-MODERNA
FROM: "murilo filo" <avalanchedrive@hotmail.com>
TO: ciencialist@yahoogrupos.com.br
DATE: 06/03/2005 00:00

1 x 0 prá vc , Oráculo.... eu deveria ter tido mais cuidado. Ah. aaah...
Já pensou se eu achasse todo êste bull-shit uma ótima matéria?
Valeu... se foi vc, parabens pela imaginação.
Quanto a mim, aquela tal frase continua valendo... abr/M.

>From: "Oraculo" <oraculo@atibaia.com.br>
>Reply-To: ciencialist@yahoogrupos.com.br
>To: <ciencialist@yahoogrupos.com.br>
>Subject: Re: [ciencialist] "ENGENHARIA ALTERNATIVA" UMA PARÁBOLA
>PÓS-MODERNA
>Date: Sat, 5 Mar 2005 19:59:54 -0300
>
>
>Olá Murilo
>
>Espero que não fique ofendido, mas eu acho que talvez não tenha percebido o
>sentido ironico e sarcástico do texto. E talvez tenha deixado passar o
>alerta inicial: As pessoas são ficticias..:-) O Dr. Waterstone não existe,
>embora esse tipo de corrente alternativa esteja cada dia mais forte..:-)
>
>O que o texto, humoristicametne, tentou apresentar, foi um tipo de
>"reductio ad absurdum", ao aplicar os conceitos tipo "new age" a questões
>sérias e reais, como a engenharia. Tudo bem usar esse tipo de bobagem para
>arrumar os móveis da sala de forma a usar a "energia cósmica" do universo,
>mas levar à sério em situações de risco ou que dependem de confiabilidade,
>é uma coisa perigosa..:-)
>
>Um abraço.
>
>Homero
>
> ----- Original Message -----
> From: murilo filo
> To: ciencialist@yahoogrupos.com.br
> Sent: Saturday, March 05, 2005 6:05 PM
> Subject: RE: [ciencialist] "ENGENHARIA ALTERNATIVA" UMA PARÁBOLA
>PÓS-MODERNA
>
>
> Ninguém comentou êsse treco do Oráculo / Waterstone... Êste último,
> definitivamente um ser liberto e que não ''é ciência encarnada'', nem
> professor de nada e nem ''reservatório do saber''.
> Há uma frase em destaque abaixo e quase todos vão achar impossível êle
>não
> ser pirado.
> Esta é uma frase de quem usa a ciência como uma ferramenta em suas
> concepções, e eu mesmo já disse coisa parecida, modestamente, sobre
>minha
> pessoa.
> Pena que não dá p/se ter só idéias aproveitáveis, sempre.
> Como um pirado normal, acho que o tal Waterstone é doidão mesmo! abr/M.
>
> >From: "Oraculo" <oraculo@atibaia.com.br>
> >Reply-To: ciencialist@yahoogrupos.com.br
> >To: <ciencialist@yahoogrupos.com.br>
> >Subject: [ciencialist] "ENGENHARIA ALTERNATIVA" UMA PARÁBOLA
>PÓS-MODERNA
> >Date: Fri, 4 Mar 2005 17:58:03 -0300
> >
> >
> >
> >"ENGENHARIA ALTERNATIVA" UMA PARÁBOLA PÓS-MODERNA
> >de Steven Novella, M.D.
> >
> >Nota: As pessoas mencionadas nesta história são fictícias, mas os
>perigos
> >da pseudociência aplicada são reais.
> >
> >Um novo fenômeno está invadido os EUA, ganhando a atenção tanto dos
> >consumidores como dos fabricantes. Cada vez mais decepcionadas com o
>frio
> >mundo metálico da tecnologia moderna, as pessoas estão olhando mais
> >atentamente para alternativas mais naturais. Coletivamente chamada de
> >Engenharia Alternativa ("Alt Eng"), uma grande variedade de métodos
>novos e
> >antigos está ganhando respeitabilidade científica e jornalística.
> >
> >>>>>>>>>>>>Alec Waterstone é um desses supostos engenheiros
>alternativos.
> >>>>>>>>>>>>Ele não tem nenhum diploma ou formação acadêmica em
>engenharia,
> >>>>>>>>>>>>o que, ele explica, é uma vantagem. "Meu pensamento não é
> >>>>>>>>>>>>limitado pela matemática, lógica ou qualquer enfadonho
>antigo
> >>>>>>>>>>>>paradigma mecanicista. Não tenho que render homenagens às
> >>>>>>>>>>>>preferências de Newton ou de outros pedagogos ocidentais. A
> >>>>>>>>>>>>ausência completa de treinamento me deixa livre para pensar
>em
> >>>>>>>>>>>>soluções únicas e inovadoras para problemas de engenharia,
> >>>>>>>>>>>>liberto dos irritantes limites da
> >>>>>>>>>>>>'realidade.'<<<<<<<<<<<<<<<<<<<!!!!!!!!!!!!
> >
> >Pontes Baseadas em Energia
> >
> >O último projeto de Alec é uma ponte não-pênsil de 460 m. Ele afirma
>que a
> >ponte será capaz de
>
>
>
>
> ##### ##### #####
>
> Para saber mais visite
> http://www.ciencialist.hpg.ig.com.br
>
>
> ##### ##### ##### #####
>
>
> Yahoo! Grupos, um serviço oferecido por:
>
>
>
>
>
>
>
>------------------------------------------------------------------------------
> Links do Yahoo! Grupos
>
> a.. Para visitar o site do seu grupo na web, acesse:
> http://br.groups.yahoo.com/group/ciencialist/
>
> b.. Para sair deste grupo, envie um e-mail para:
> ciencialist-unsubscribe@yahoogrupos.com.br
>
> c.. O uso que você faz do Yahoo! Grupos está sujeito aos Termos do
>Serviço do Yahoo!.
>
>
>
>[As partes desta mensagem que não continham texto foram removidas]
>
>
>
>##### ##### #####
>
>Para saber mais visite
>http://www.ciencialist.hpg.ig.com.br
>
>
>##### ##### ##### #####
>Links do Yahoo! Grupos
>
>
>
>
>
>
>
>




SUBJECT: Re: Albert Einstein, um hipócrita???????????
FROM: César A. K. Grossmann <cesarakg@bol.com.br>
TO: ciencialist@yahoogrupos.com.br
DATE: 06/03/2005 00:48


--- Em ciencialist@yahoogrupos.com.br, ZMIR <zmirsilva@u...> escreveu
> O Davi da bíblia, quando pivete, defrontou-se com um guerreiro
gigante
> e, destemidamente,
> tascou-lhe uma tijolada, que o homem caiu duro. Isso é que diz o
livro
> dos livros. Eu não acredito, conversa prá boi dormir,

Bom, se houve ou não uma luta entre um gigante filisteu chamado Golias
e um jovem israelita Davi, eu não sei, mas o relato bíblico é bastante
verossímil: usando de uma funda (que é uma máquina bastante simples:
uma correia de couro, ou um disco de couro com duas correias, que é
usado girando-se o mesmo sobre a cabeça, com uma pedra na volta da
correia ou no disco - lembra bastante o lançamento de martelo, aquele
esporte olímpico), Davi impulsionou um seixo roliço (nada de tijolo),
destes de rio, acertando a testa do gigante (provavelmente um dos
poucos pontos não protegidos por capacete). O gigante caiu (nocaute,
desmaio, tontura, morte, sei lá), Davi lançou-se sobre ele, tomou da
espada do gigante, e com ela decepou a cabeça do mesmo. Se ele matou o
gigante com a pedrada, eu não sei, mas tenho certeza que ninguém
sobrevive a uma decapitação...

> daí a alusão em tom de brincadeira. Dr. o gorila realmente dançou,

> pode ter sido de tanto rir, ante a palhaçada. E não lembro a qual
dos
> dois, Davi ou Salomão, a Bíblia se refereao dizer que teve umas
> setecentas mulheres, durante toda a vida. Já no final dela, pertinho
de
> ir se encontrar com o salvador, arrependeu-se, que não era besta, e
> então passou criticar atitudes pecaminosas, do tipo e de outros.
> O "vaidade das vaidades", que inicia uma série de coisas
santificantes
> refere-se a isso. É ou não é " hipocrisia das hipocrisias"?Não dá
para
> falar sério com
> coisas do tipo. Acho que o assunto acabou ficando off-tópic. Páro
por aquí.

Davi nunca foi santo e admitiu isto no leito de morte. Salomão foi o
que andou reto até que as suas muitas esposas pagãs o fizeram
desviar-se da fé israelita. Então o Cântico dos Cânticos não pode ter
sido escrito no fim da vida de Salomão (se é que foi escrito por
Salomão).

Quanto ao off-topic, arqueologia bíblica e história é ciência. Fé,
não.

[]s
--
César A. K. Grossmann
http://www.LinuxByGrossmann.cjb.net/





SUBJECT: Re: [ciencialist] "ENGENHARIA ALTERNATIVA" UMA PARÁBOLA PÓS-MODERNA
FROM: "Alvaro Augusto \(E\)" <alvaro@electraenergy.com.br>
TO: <ciencialist@yahoogrupos.com.br>
DATE: 06/03/2005 16:27

Bem, felizmente a engenharia não deixe margem para erros. Qualquer um pode criar uma religião ou um culto baseado no Feng Shui, mas construir uma ponte baseada nisso somente exporá as limitações de tais superstições frente aos princípios da estática e da resistência dos materiais. Se construída, a ponte certamente ruirá, juntamente com os egos dos construtores. Ainda assim, duvido muito que os conselhos de engenharia norte-americanos deixem que milhares de pessoas se exponham ao perigo baseadas no "pensamento Poliana" de um lunático.

[ ]s

Alvaro Augusto





----- Original Message -----
From: Oraculo
To: ciencialist@yahoogrupos.com.br
Sent: Friday, March 04, 2005 5:58 PM
Subject: [ciencialist] "ENGENHARIA ALTERNATIVA" UMA PARÁBOLA PÓS-MODERNA



"ENGENHARIA ALTERNATIVA" UMA PARÁBOLA PÓS-MODERNA
de Steven Novella, M.D.

Nota: As pessoas mencionadas nesta história são fictícias, mas os perigos da pseudociência aplicada são reais.

Um novo fenômeno está invadido os EUA, ganhando a atenção tanto dos consumidores como dos fabricantes. Cada vez mais decepcionadas com o frio mundo metálico da tecnologia moderna, as pessoas estão olhando mais atentamente para alternativas mais naturais. Coletivamente chamada de Engenharia Alternativa ("Alt Eng"), uma grande variedade de métodos novos e antigos está ganhando respeitabilidade científica e jornalística.

Alec Waterstone é um desses supostos engenheiros alternativos. Ele não tem nenhum diploma ou formação acadêmica em engenharia, o que, ele explica, é uma vantagem. "Meu pensamento não é limitado pela matemática, lógica ou qualquer enfadonho antigo paradigma mecanicista. Não tenho que render homenagens às preferências de Newton ou de outros pedagogos ocidentais. A ausência completa de treinamento me deixa livre para pensar em soluções únicas e inovadoras para problemas de engenharia, liberto dos irritantes limites da 'realidade.'"

Pontes Baseadas em Energia

O último projeto de Alec é uma ponte não-pênsil de 460 m. Ele afirma que a ponte será capaz de transpor esta distância sem pilares ou suspensões, e será sustentada somente pela antiga arte do Feng Shui. "Esta sabedoria, que tem milhares de anos, é a arte de canalizar energia através do design e da forma. Esta energia pode ser usada para suportar uma ponte de 460 m, ou mesmo estruturas maiores." Os planejadores urbanos estão intrigados por estes projetos, porque tais pontes custarão menos que a metade das pontes projetadas pela maneira convencional.

Alec também mostra que antigos documentos chineses não revelam absolutamente nenhum relato de pontes suspensas desmoronando. Ele argumenta que seus registros de segurança da técnica são sem paralelos. "Como poderia ter sobrevivido todos estes anos se não funcionasse?"

Anthony Trellis, professor de engenharia na Universidade Estado-da-Arte, alega que os projetos de Alec correm na direção contrária aos princípios básicos da física e da ciência dos materiais. Irritado, Trellis comentou que "uma ponte baseada nos projetos de Waterstone simplesmente não poderia se sustentar. Seria insegura ao extremo."

Mas Alec não se perturba com esssas críticas. "É claro que o professor Trellis não gosta dos meus projetos, porque eles desafiam seu precioso status quo e põem seu mundo de cabeça para baixo. Mas o protecionismo da velha guarda está começando a desmoronar, como também seus prédios obsoletos", ele retrucou em simpósio recente para pensadores progressistas que concordaram que aqueles que perdem o bonde da história serão deixados para trás. Seu discurso para uma platéia lotada acusou a American Society of Civil Engineers (Sociedade Americana de Engenheiros Civis), a indústria do aço e outros "interesses ocultos" de tentarem suprimir seus pontos de vista.

Céticos têm sugerido que antes de gastar milhões de dólares do contribuinte nesses projetos, e sujeitar os motoristas americanos a riscos desconhecidos de dirigir por sobre uma ponte de Waterstone, os princípios básicos de Waterstone deveriam ao menos ser testados para ver se eles funcionam. Isto é especialmente correto uma vez que seus projetos parecem ir contra o senso comum. Mas Waterstorne responde:

Estou ocupado demais projetando pontes para ficar satisfazendo alguns céticos. Seja como for, eles nunca estarão satisfeitos. Os motoristas americanos deveriam ser livres para decidir se desejam dirigir por sobre uma de minhas pontes. Respeito sua inteligência e habilidade para tomarem decisões inteligentes sozinhos. Eles não precisam ser informados por algum burocrata, ou um professor em uma torre de marfim, sobre quais pontes são seguras e quais não.
Professor Trellis e outros opositores argumentam que os indivíduos não deveriam ter de ser cientistas ou engenheiros de modo a dirigir com segurança sobre nossas pontes. As regras não são feitas para limitar a liberdade, mas para oferecer um nível básico de segurança e proteção para o público. Esta atitude, todavia, está progressivamente sendo rejeitada como excessivamente paternalista e protecionista.
Carros Intuitivos

Engenheiros civis não são o únicos a serem atraídos pela antiga sabedoria das sociedades pré-tecnológicas. A indústria automobilística também está se contagiando. Natural Designs é uma nova companhia automobilística com sede no Kansas. Seus presidente, Andy Wily, recebeu de Harvard um título acadêmico em engenharia 20 anos atrás, mas foi demitido de seu cargo subseqüente de professor após uso excessivo de drogas a ponto de quase destruir sua vida. Agora ele retornou com uma nova companhia e uma nova filosofia para a qual muitos consumidores se sentem atraídos.

"Estou defendendo uma mistura do melhor da engenharia científica moderna com as idéias anticientíficas e supersticiosas dos tempos antigos," explica Wily. "Chamo esta abordagem de Engenharia Integrativa."

O que foi que esta nova abordagem criou? O mais novo modelo sedan da Natural Design, o Millennium 2000, não usa air bags, nem cintos de segurança. "Cintos de segurança são perigosos, e air bags matam crianças," queixa-se Wily. Então ele apareceu com algo melhor. O interior do Millennium 2000 é revestido com um material psicoativo patenteado, chamado Natural Safe. "Tudo que o motorista ou os passageiros precisam fazer é ter pensamentos seguros, e este material milagroso fará o resto. Em um acidente, o material delicadamente conterá qualquer pessoa com pensamentos seguros no veículo, deixando-as livres de ferimentos", assegura Wily.

Quando os céticos apontam as mortes ou invalidez dos passageiros do Millennium 2000, Wily responde que os passageiros claramente não estavam pensando tão "seguramente" como deveriam. "Além disso", acrescenta, "o Millennium 2000 corre a somente 80 km por hora em um bom dia deixando para trás um vento feliz. Se os motoristas que morreram estivessem dirigindo algo desenvolvido pelo Cartel Internacional dos Fabricantes de Automóveis como um Ford ou um Chevrolet, estariam indo muito mais rápido, com uma chance de morte ainda maior. Só quando a Ford parar de matar milhares de pessoas por ano em nossas estradas é que suas reclamações sobre nós parecerão algo além de proteção da sua fatia de mercado. Na verdade, temos aqui mesmo um estudo que mostra que se todos deixassem de dirigir amanhã, a taxa de morte cairia na América! Até convencermos o povo americano dos milhões mortos desnecessariamente pela moderna 'ciência automotiva', a Natural Safe permanece a escolha mais segura."

Muitos consumidores estão convencidos. Para não ficar para trás, a GM e a Ford começaram a colocar o revestimento da Natural Safe em seus carros. Amy Zinger, de Arkansas, sobreviveu a uma colisão frontal a 65 km/h em um destes veículos. "Estava usando meu cinto de segurança e o air bag abriu, mas eu sei que foi o Natural Safe que salvou minha vida", declarou recentemente. "Além disso", apontou, "se não funcionasse, sua venda não seria permitida." Motivados por testemunhos assim, cada vez mais consumidores estão insistindo em comprar somente carros com o Natural Safe.

Um problema enfrentado pela Natural Design, todavia, é que as antiquadas leis de segurança, como aquelas que exigem cintos de segurança, não mencionam estes novos projetos integrativos. Recentemente, entretanto, tudo isto tem mudado. O senador Hackem, de Iowa, estado natal da Natural Design, propôs a legislação que irá isentar os fabricantes que usam princípios alternativos ou integrativos de regulação planejada para proteger os consumidores. Isto foi aclamado como um grande passo à frente.

Contudo, os céticos teimosos não desaparecerão. "Tudo que estou pedindo é um simples teste de colisão" exclamou um cético famoso, Perry DeAngelis. "Se a coisa realmente funciona, diacho, eu compro." Os céticos vêm cada vez mais pedindo tais testes, argumentando que o teste deveria ser realizado antes da implementação, especialmente quando vidas humanas estão em jogo.

Mas Wily explica o motivo pelo qual estes testes não funcionam. "Bonecos de testes não são pessoas. O material psicoativo portanto não responderá a eles. O fato é que estes projetos inovadores não podem ser submetidos aos mesmos testes e princípios como a engenharia tradicional. Mas os consumidores que dirigem nossos carros se sentem mais seguros. Como você pode argumentar contra isso?"

Contudo, DeAngelis aponta para estudos recentes que parecem indicar que os motoristas dos carros de Wily têm uma probabilidade duas vezes maior de morrer em um acidente que os motoristas de veículos convencionais. Mas Wily somente zomba disso, "no que você vai acreditar, números em um pedaço de papel, ou em pessoas?"

Conquistas Políticas

Apesar dos céticos, a Engenharia Alternativa parece que veio para ficar. Wily acabou de ser nomeado chefe do novo Departamento de Engenharia Integrativa na Zones University, onde ele espera treinar a nova geração de engenheiros em sua filosofia. Enquanto isso, o senador Hackem apresentou ao Congresso um projeto de lei para criar o Centro de Engenharia Alternativa. Este novo departamento irá impedir que dinheiro seja desperdiçado na manutenção da infra-estrutura do país e seja usado para estudar e promover os princípios alternativos na engenharia.

Finalmente, no que é caracterizado como um marco na produção, o Canadian College of Rainbow-Coloured Integrative Engineering - após passar mais de 100 anos como um pária científico - está finalizando negociações para se tornar parte da prestigiosa Dork University. Apesar dos gritos de consternação da faculdade de ciência e matemática da Dork e de diversos laureados com o Nobel, o conselho de Dork avança com seus planos de afiliação. Os 25 milhões de dólares que os Engenheiros Integrativos prometeram doar á Universidade não têm, de acordo com o presidente da Dork, influenciado o acordo. O presidente caracterizou os críticos da Engenharia Integrativa como "chorões" que expõem "desinformação há muito refutada" sobre a Engenharia Alternativa.


[As partes desta mensagem que não continham texto foram removidas]



##### ##### #####

Para saber mais visite
http://www.ciencialist.hpg.ig.com.br


##### ##### ##### #####


Yahoo! Grupos, um serviço oferecido por:







------------------------------------------------------------------------------
Links do Yahoo! Grupos

a.. Para visitar o site do seu grupo na web, acesse:
http://br.groups.yahoo.com/group/ciencialist/

b.. Para sair deste grupo, envie um e-mail para:
ciencialist-unsubscribe@yahoogrupos.com.br

c.. O uso que você faz do Yahoo! Grupos está sujeito aos Termos do Serviço do Yahoo!.



[As partes desta mensagem que não continham texto foram removidas]



SUBJECT: Fw: sistema internacional de unidades
FROM: "Luiz Ferraz Netto" <leobarretos@uol.com.br>
TO: "ciencialist" <ciencialist@yahoogrupos.com.br>
DATE: 06/03/2005 18:34

Alguém tem detalhes desse naipe sobre o SI?

[]'
===========================
Luiz Ferraz Netto [Léo]
leobarretos@uol.com.br
http://www.feiradeciencias.com.br
===========================
-----Mensagem Original-----
De: João Sacharo
Para: leobarretos@uol.com.br
Enviada em: domingo, 6 de março de 2005 01:44
Assunto: sistema internacional de unidades




Oi professor

Gostaria de saber quais os países que pertencem ao SI, e onde eles fazem as suas reuniões

atenciosamente

joão roberto
Nas dúvidas experimentais, por gentileza coloque aqui o endereço da página, isso facilita o confronto. Agradeço. Meu nome é LUIZ FERRAZ NETTO, meu apelido é LÉO e moro em BARRETOS; dai vem meu e-mail: leobarretos@uol.com.br.


--------------------------------------------------------------------------------


Internal Virus Database is out-of-date.
Checked by AVG Anti-Virus.
Version: 7.0.300 / Virus Database: 266.5.0 - Release Date: 25/02/2005

----------

Internal Virus Database is out-of-date.
Checked by AVG Anti-Virus.
Version: 7.0.300 / Virus Database: 266.5.0 - Release Date: 25/02/2005


[As partes desta mensagem que não continham texto foram removidas]



SUBJECT: Fw: Cadeia alimentar
FROM: "Luiz Ferraz Netto" <leobarretos@uol.com.br>
TO: "ciencialist" <ciencialist@yahoogrupos.com.br>
DATE: 06/03/2005 19:33

To Takata ......

[]'
===========================
Luiz Ferraz Netto [Léo]
leobarretos@uol.com.br
http://www.feiradeciencias.com.br
===========================
-----Mensagem Original-----
De: Fátima Vilarinho
Para: leobarretos@uol.com.br
Enviada em: domingo, 6 de março de 2005 15:58
Assunto: Cadeia alimentar


Srº Leo Barreto

TENHO 33 ANOS E VOLTEI A ESTUDAR RECENTEMETE EM UM SUPLETIVO DA 5ª E 6ª SÉRIE.
ESTOU COM DIFICULDADES NA CADEIA ALIMENTAR.
O PROFESSOR PASSOU UM TRABALHO
PARA MONTAR UMA CADEIA ALIMENTAR AQUATICA E TERRESTRE. EM BUSCA NA INTERNET ENCONTREI O SEU SITE E SE FOSSE POSSIVEL O SENHOR MIM DA UMA ORIENTAÇÃO OU EXEMPLOS COMO MONTAR MA CADEIA ALIMENTAR ELE QUE COM ANIMAIS .

MORO EM RECIFE-PE
fatimavilarinho@bol.com.br

FICO MUITO GRATA ,




------------------------------------------------------------------------------

avast! Antivirus: Outbound message clean.

Virus Database (VPS): 0509-5, 04/03/2005
Tested on: 6/3/2005 15:58:58
avast! - copyright (c) 1988-2004 ALWIL Software.




--------------------------------------------------------------------------------


Internal Virus Database is out-of-date.
Checked by AVG Anti-Virus.
Version: 7.0.300 / Virus Database: 266.5.0 - Release Date: 25/02/2005

----------

Internal Virus Database is out-of-date.
Checked by AVG Anti-Virus.
Version: 7.0.300 / Virus Database: 266.5.0 - Release Date: 25/02/2005


[As partes desta mensagem que não continham texto foram removidas]



SUBJECT: Água e Metabolismo
FROM: gisele campos <giscampos@yahoo.com.br>
TO: ciencialist@yahoogrupos.com.br
DATE: 06/03/2005 21:30

06/03/2005 - 21h25

Olá pessoal .
Sou Gisele Campos , professora de Química no ensino médio e EJA.

Percebo que os conceitos de química, muitas vezes não convecem
às dúvidas de alguns alunos, principalmente os do EJA(ensino à
jovens e adultos- antigo Suplência). As principais questões são :

Primeiro : A água H20 , não é óxido , não é ácido , então como posso
vencer a questão , num nível bem "popular", sem tecnicismo:

Segundo : O metabolismo é uma ação do organismo mutável - que posso
realmente manusear - ou é genético ? Entendo que podemos , mas
eles precisam entender como isso ocorre no organismo , até quimicamente
falando.

Agradeço explicações nas duas esferas , no técnico e no leigo .
Muito Obrigada.

Gisele Campos - Química
Centro educacional de Formação do Magistério e Técnicos.
Depto. Pesquisa - Diadema- SP
email : giscampos@yahoo.com.br





---------------------------------
Yahoo! Acesso Grátis - Internet rápida e grátis. Instale o discador do Yahoo! agora.

[As partes desta mensagem que não continham texto foram removidas]



SUBJECT: Re: Fw: Cadeia alimentar
FROM: Manuel Bulcão <manuelbulcao@uol.com.br>
TO: ciencialist@yahoogrupos.com.br
DATE: 06/03/2005 23:02


'Grosso modo', e sem considerar os organismos autotróficos que não
dependem da energia luminosa, uma cadeia alimentar pode ser
representada da seguinte forma:

Organismo fotossintético -> herbívoro -> carnívoro -> organismo
necrófago.

Os organismos fotossintéticos são os produtores, dada a sua
capacidade de sintetizar substâncias orgânicas a partir das
inorgânicas.

Os herbívoros são consumidores primários.

Os carnívoros são consumidores secundários (os que se alimentam de
herbívoros), terciários ou quaternários (predadores de outros
carnívoros).

Quanto aos 'degradadores' (organismos necrófagos ou sapróbios),
estes se alimentam de cadáveres e matéria orgânica residual ou em
decomposição. Podem ser vermes, insetos (os escaravelhos, por
exemplo), fungos ou bactérias.

Um exemplo de cadeia alimentar aquática (existente numa lagoa):

Algas verdes (produtores) -> crustáceos copépodes (consumidores
primários) -> besouros d'água (consumidores secundários) -> peixes
grandes, como a perca (consumidores terciários) -> bactérias e
fungos aquáticos (degradadores).

Os carnívoros que não são predados estão sujeitos apenas à ação dos
parasitas e dos degradadores.

Abraços,
Manuel Bulcão
____________________

--- Em ciencialist@yahoogrupos.com.br, "Luiz Ferraz Netto"
<leobarretos@u...> escreveu
> To Takata ......
>
> []'
> ===========================
> Luiz Ferraz Netto [Léo]
> leobarretos@u...
> http://www.feiradeciencias.com.br
> ===========================
> -----Mensagem Original-----
> De: Fátima Vilarinho
> Para: leobarretos@u...
> Enviada em: domingo, 6 de março de 2005 15:58
> Assunto: Cadeia alimentar
>
>
> Srº Leo Barreto
>
> TENHO 33 ANOS E VOLTEI A ESTUDAR RECENTEMETE EM UM SUPLETIVO DA 5ª
E 6ª SÉRIE.
> ESTOU COM DIFICULDADES NA CADEIA ALIMENTAR.
> O PROFESSOR PASSOU UM TRABALHO
> PARA MONTAR UMA CADEIA ALIMENTAR AQUATICA E TERRESTRE. EM BUSCA NA
INTERNET ENCONTREI O SEU SITE E SE FOSSE POSSIVEL O SENHOR MIM DA
UMA ORIENTAÇÃO OU EXEMPLOS COMO MONTAR MA CADEIA ALIMENTAR ELE QUE
COM ANIMAIS .
>
> MORO EM RECIFE-PE
> fatimavilarinho@b...
>
> FICO MUITO GRATA ,





SUBJECT: Re: Água e Metabolismo
FROM: Manuel Bulcão <manuelbulcao@uol.com.br>
TO: ciencialist@yahoogrupos.com.br
DATE: 06/03/2005 23:45


--- Em ciencialist@yahoogrupos.com.br, gisele campos
<giscampos@y...> escreveu
> 06/03/2005 - 21h25
> Primeiro : A água H20 , não é óxido , não é ácido , então como
posso vencer a questão , num nível bem "popular", sem tecnicismo:

Manuel: A água não é um óxido neutro? óxido de diidrogênio?

Abraços,
Manuel Bulcão





SUBJECT: Cientistas recriam nervos ópticos em ratos
FROM: "Oraculo" <oraculo@atibaia.com.br>
TO: <ciencialist@yahoogrupos.com.br>
DATE: 07/03/2005 01:52

Cientistas recriam nervos ópticos em ratos

Cientistas dizem ter conseguido reconstruir em ratos nervos ópticos danificados em toda a extensão do percurso desde o olho até o cérebro.
Especialistas dizem que a experiência, feita pelo Instituto Schepens de Pesquisas do Olho, traz novas esperanças para tratar o glaucoma, uma enfermidade na qual um nível elevado de pressão destrói o nervo óptico.

A técnica também poderia ser usada para ajudar pessoas que sofrem de problemas na medula espinhal e em outras partes do sistema nervoso.

A experiência é detalhada na revista especializada Journal of Cell Science.

Cicatriz

"Isto é o mais próximo que a ciência já chegou de regenerar tantas fibras de nervos ópticos em uma distância tão grande para chegar ao seu objetivo", disse Dong Feng Chen, o líder da equipe de pesquisadores.

"E também de reparar um nervo que antes se achava que tinha se danificado de forma irreparável."

Vários tecidos celulares do corpo humano se reconstituem sozinhos em caso de lesão.

Mas não o nervo óptico e outros tecidos do sistema nervoso central, cujos danos são sempre permanentes.

Os cientistas do Instituto Schepens descobriram que a incapacidade do nervo óptico de se regenerar estava ligada ao não funcionamento de um gene chamado BCL-2.

Eles também descobriram que o processo de regeneração estava bloqueado pela criação, logo após o nascimento, de uma cicatriz no cérebro por células gliais especializadas.

Essas células desempenham várias funções no cérebro, entre as quais a criação dessa espécie de cicatriz.

A cicatriz coloca uma barreira ao mesmo tempo física e molecular para a regeneração.

Sempre ativo

Os cientistas criaram ratos nos quais o BCL-2 estava sempre ativado.

Eles descobriram que os animais conseguiam reparar os nervos ópticos com rapidez - mas só enquanto eram jovens e a cicatriz no cérebro ainda não havia se desenvolvido.

Em seguida eles criaram ratos com o BCL-2 funcionando e uma reduzida capacidade de criar as cicatrizes pelas células gliais.

Desta vez, mesmo os roedores mais velhos conseguiram regenerar as células ópticas.

O próximo passo agora será determinar se os nervos regenerados funcionam direito.




http://www.bbc.co.uk/portuguese/ciencia/story/2005/03/050302_nervoopticoro.shtml

[As partes desta mensagem que não continham texto foram removidas]



SUBJECT: Re: Água e Metabolismo
FROM: "rmtakata" <rmtakata@altavista.net>
TO: ciencialist@yahoogrupos.com.br
DATE: 07/03/2005 08:05


--- Em ciencialist@yahoogrupos.com.br, Manuel Bulcão
<manuelbulcao@u...> > --- Em ciencialist@yahoogrupos.com.br, gisele
campos
> > Primeiro : A água H20 , não é óxido , não é ácido , então como
> posso vencer a questão , num nível bem "popular", sem tecnicismo:
>
> Manuel: A água não é um óxido neutro? óxido de diidrogênio?

Ou peroxido de hidrogenio, hidróxido de hidrogênio ou acido hidroxilico.

A agua pode se comportar como base, como oxido ou como sal.

[]s,

Roberto Takata





SUBJECT: Re: Fw: Cadeia alimentar
FROM: "rmtakata" <rmtakata@altavista.net>
TO: ciencialist@yahoogrupos.com.br
DATE: 07/03/2005 08:14


Prezada Fatima,

A ideia por tras da cadeia alimentar eh muito, muito simples. Imagine
um grupo de organismos (animais, plantas, microorganismos...) q. vivem
em um local - e imagine quem come o que?

Uma cadeia aquatica pode ser em um lago, um rio, no mar... Uma
terrestre, no campo, na floresta, no deserto, em uma caverna...

Fala-se em uma cadeia porq. a representacao dela eh uma corrente
(cadeia tb significa corrente - tem o significado de prisao, porq.
sempre foi muito comum prender pessoas com correntes): isso significa
q. um organismo, na representacao, segue o outro - na frente vai o
organismo q. eh comido pelo organismos seguinte.

Na frente de todos vai o organismo q. nao come ninguem e produz seu
proprio alimento - em geral uma planta. Assim fica facil: comece com
uma planta q. tem no local, imagine um bicho do local q. come essa
planta, depois um outro bicho q. come esse bicho q. come a planta, ai'
um outro bicho q. come esse bicho q. come o bicho q. come a planta.

(Eu falei em bicho - isto eh, um animal - mas ha' alguns casos em q. a
planta come o animal: como as plantas carnivoras, q. alem de
produzirem seu proprio alimento, complementa sua dieta com moscas e
outros insetos, e ateh pequenos vertebrados como rãs.)

[]s,

Roberto Takata

--- Em ciencialist@yahoogrupos.com.br, "Luiz Ferraz Netto"
> To Takata ......
> Luiz Ferraz Netto [Léo]
> -----Mensagem Original-----
> De: Fátima Vilarinho
> PARA MONTAR UMA CADEIA ALIMENTAR AQUATICA E TERRESTRE. EM BUSCA NA
> INTERNET ENCONTREI O SEU SITE E SE FOSSE POSSIVEL O SENHOR MIM DA
> UMA ORIENTAÇÃO OU EXEMPLOS COMO MONTAR MA CADEIA ALIMENTAR ELE QUE
> COM ANIMAIS .






SUBJECT: Obrigado
FROM: "joaogaspar" <joaogasparilha@terra.com.br>
TO: <ciencialist@yahoogrupos.com.br>
DATE: 07/03/2005 11:17

Obrigado pela aprovação de minha entrada no grupo.
Aproveito e envio o endereço de meu site
www.joaogaspar.com.br



[As partes desta mensagem que não continham texto foram removidas]



SUBJECT: Re: Água e Metabolismo
FROM: "rmtakata" <rmtakata@altavista.net>
TO: ciencialist@yahoogrupos.com.br
DATE: 07/03/2005 11:19


--- Em ciencialist@yahoogrupos.com.br, gisele campos <giscampos@y...>
> Segundo : O metabolismo é uma ação do organismo mutável - que posso
> realmente manusear - ou é genético ? Entendo que podemos , mas
> eles precisam entender como isso ocorre no organismo , até
> quimicamente falando.

Metabolismo eh todo o conjunto de reacoes quimicas que ocorrem dentro
de um organismo.

(Agora nao entendi a dicotomia: mutavel ou genetico - algo genetico
pode mudar, oras.)

[]s,

Roberto Takata





SUBJECT: Aprovacoes para C-List
FROM: "Luiz Ferraz Netto" <leobarretos@uol.com.br>
TO: <ciencialist@yahoogrupos.com.br>
DATE: 07/03/2005 11:24

Vários foram as solicitações de pessoas interessadas em participar da C-List; a todas essas (solicitações) enviei resposta direta de aprovação, via Yahoo.
Apreciaria que tais novos 'C-listeiros' se apresentassem, com brevidade e concisão. A todos meus particulares agradecimentos por nos elegerem como lista de discussão de seus interesses e anseios científicos.
{}'
Léo, um dos n+1(oficial) moderadores da C-List. (por enquanto, n natural)

===========================
Luiz Ferraz Netto [Léo]
leobarretos@uol.com.br
http://www.feiradeciencias.com.br
===========================


--
Internal Virus Database is out-of-date.
Checked by AVG Anti-Virus.
Version: 7.0.300 / Virus Database: 266.5.0 - Release Date: 25/02/2005



SUBJECT: Re: [ciencialist] Fw: sistema internacional de unidades
FROM: "E m i l i a n o C h e m e l l o" <chemelloe@yahoo.com.br>
TO: <ciencialist@yahoogrupos.com.br>
DATE: 07/03/2005 11:39

São Google diz:

http://www.bipm.fr/fr/si/
http://www.chemkeys.com/bra/ag/uec_7/sidu_4/sidu_4.htm

[ ] 's do Emiliano Chemello
emiliano@quimica.net
http://www.quimica.net/emiliano
http://www.ucs.br/ccet/defq/naeq

" Rien ne se perd, rien ne se crée,
tout se transforme."

Antoine Laurent de Lavoisier (químico francês, 1743 - 1794)

----- Original Message -----
From: Luiz Ferraz Netto
To: ciencialist
Sent: Sunday, March 06, 2005 6:34 PM
Subject: [ciencialist] Fw: sistema internacional de unidades


Alguém tem detalhes desse naipe sobre o SI?

[]'
===========================
Luiz Ferraz Netto [Léo]
leobarretos@uol.com.br
http://www.feiradeciencias.com.br
===========================
-----Mensagem Original-----
De: João Sacharo
Para: leobarretos@uol.com.br
Enviada em: domingo, 6 de março de 2005 01:44
Assunto: sistema internacional de unidades




Oi professor

Gostaria de saber quais os países que pertencem ao SI, e onde eles fazem as suas reuniões

atenciosamente

joão roberto
Nas dúvidas experimentais, por gentileza coloque aqui o endereço da página, isso facilita o confronto. Agradeço. Meu nome é LUIZ FERRAZ NETTO, meu apelido é LÉO e moro em BARRETOS; dai vem meu e-mail: leobarretos@uol.com.br.


--------------------------------------------------------------------------------


Internal Virus Database is out-of-date.
Checked by AVG Anti-Virus.
Version: 7.0.300 / Virus Database: 266.5.0 - Release Date: 25/02/2005

----------

Internal Virus Database is out-of-date.
Checked by AVG Anti-Virus.
Version: 7.0.300 / Virus Database: 266.5.0 - Release Date: 25/02/2005


[As partes desta mensagem que não continham texto foram removidas]



##### ##### #####

Para saber mais visite
http://www.ciencialist.hpg.ig.com.br


##### ##### ##### #####


Yahoo! Grupos, um serviço oferecido por:
PUBLICIDADE




------------------------------------------------------------------------------
Links do Yahoo! Grupos

a.. Para visitar o site do seu grupo na web, acesse:
http://br.groups.yahoo.com/group/ciencialist/

b.. Para sair deste grupo, envie um e-mail para:
ciencialist-unsubscribe@yahoogrupos.com.br

c.. O uso que você faz do Yahoo! Grupos está sujeito aos Termos do Serviço do Yahoo!.



[As partes desta mensagem que não continham texto foram removidas]



SUBJECT: Re: dormindo 3:30h por dia...
FROM: "rmtakata" <rmtakata@altavista.net>
TO: ciencialist@yahoogrupos.com.br
DATE: 07/03/2005 11:55


--- Em ciencialist@yahoogrupos.com.br, "Eurico Ferreira de Souza Jr."
> [E]> o que um especialista em engenharia social faz? :D

Eh outro nome para cracker. Eles fazem contatos com as pessoas para
descobrir as fraquezas do sistema de seguranca de redes de computadores.

(E o jogo, em q. peh estah?)

[]s,

Roberto Takata





SUBJECT: Re: [ciencialist] Re: Água e Metabolismo
FROM: "E m i l i a n o C h e m e l l o" <chemelloe@yahoo.com.br>
TO: <ciencialist@yahoogrupos.com.br>
DATE: 07/03/2005 11:56

[Manuel]
A água não é um óxido neutro? óxido de diidrogênio?

[Emiliano]
Sim. É um óxido. Mais precisamente: Monóxido de Dihidrogênio.

Sugestão para a Gisele Campos
http://www.unificado.com.br/correcao_UFRGS05/qui_ing_esp/quimica.htm

por falar nisso em monóxido de dihidrogênio:
http://www.novomilenio.inf.br/humor/0205h006.htm

[ ] 's do Emiliano Chemello
emiliano@quimica.net
http://www.quimica.net/emiliano
http://www.ucs.br/ccet/defq/naeq

" Rien ne se perd, rien ne se crée,
tout se transforme."

----- Original Message -----
From: Manuel Bulcão
To: ciencialist@yahoogrupos.com.br
Sent: Sunday, March 06, 2005 11:45 PM
Subject: [ciencialist] Re: Água e Metabolismo



--- Em ciencialist@yahoogrupos.com.br, gisele campos
<giscampos@y...> escreveu
> 06/03/2005 - 21h25
> Primeiro : A água H20 , não é óxido , não é ácido , então como
posso vencer a questão , num nível bem "popular", sem tecnicismo:

Manuel: A água não é um óxido neutro? óxido de diidrogênio?

Abraços,
Manuel Bulcão





##### ##### #####

Para saber mais visite
http://www.ciencialist.hpg.ig.com.br


##### ##### ##### #####


Yahoo! Grupos, um serviço oferecido por:

São Paulo Rio de Janeiro Curitiba Porto Alegre Belo Horizonte Brasília





Links do Yahoo! Grupos

Para visitar o site do seu grupo na web, acesse:
http://br.groups.yahoo.com/group/ciencialist/

Para sair deste grupo, envie um e-mail para:
ciencialist-unsubscribe@yahoogrupos.com.br

O uso que você faz do Yahoo! Grupos está sujeito aos Termos do Serviço do
Yahoo!.




SUBJECT: Prêmio da Unesco p/ Belita Koiller
FROM: José Renato <jrma@terra.com.br>
TO: <ciencialist@yahoogrupos.com.br>
DATE: 07/03/2005 12:03

< http://www.ambientebrasil.com.br/noticias/index.php3?action=ler&id=18259 >

06/03/2005
Física brasileira recebe prêmio da Unesco por pesquisa sobre elétrons

A física brasileira Belita Koiller recebeu em Paris (França) um dos cinco prêmios concedidos pela Unesco - Organização das Nações Unidas para a Ciência, a Educação e a Cultura e pela empresa internacional de cosméticos L'Oréal para promover o papel da mulher na ciência. Professora titular de Física da Universidade Federal do Rio de Janeiro, ela recebeu US$ 100 mil por seus trabalhos de pesquisa teórica sobre os elétrons em meios desordenados como o vidro.

Belita Koiller é a terceira brasileira a receber o prêmio principal, depois de Mayana Zatz, especialista em genética (2001) e da bioquímica Lúcia Mendonça Previato (2004). Pesquisadora do CNPq desde 1985 e primeira física titular da Academia Brasileira de Ciências, Koiller é uma reconhecida orientadora de teses e em 2002 foi condecorada com a Ordem Nacional do Mérito Científico.

Também receberam o prêmio da Unesco a tunisiana Zohra Ben Lajdar, a americana Myriam P. Sarachik, a japonesa Fumiko Yonezawa e a francesa Dominique Langevin. A edição 2005 da premiação coincide com a celebração do Ano Internacional da Física, e por isso privilegiou trabalhos de pesquisas da matéria, um campo no qual "as mulheres estão insuficientemente representadas", segundo a Unesco.

De acordo com informações da Academia Brasileira de Ciências, os trabalhos premiados dizem respeito aos campos mais promissores da Física, como a nanociência ou a física quântica. Não se centram apenas em diferentes tecnologias como os semicondutores, a medição da poluição do ar ou a extração de petróleo, mas abrem perspectivas à construção de módulos para a Estação Espacial Internacional de Marte ou para a criação do computador quântico, que poderia revolucionar todos os métodos de trabalho atuais. (Agência Brasil)



[As partes desta mensagem que não continham texto foram removidas]



SUBJECT: OFF-TOPIC em que pé está o jogo
FROM: "Eurico Ferreira de Souza Jr." <caodejah@yahoo.com.br>
TO: ciencialist@yahoogrupos.com.br
DATE: 07/03/2005 12:39



rmtakata <rmtakata@altavista.net> wrote:

(...)

(E o jogo, em q. peh estah?)

[E]> tá no contrapé, esperando o resultado
da segunda fase do concurso.
estou concorrendo com minha equipe, num
total de 55 pré-projetos, com 29 propostas
diferentes, sendo três projetos concorrendo
com minha idéia original (inclusive o meu)
O resultado sai dia 15.
As outras duas equipes que desenvolvem
minha idéia original não entraram em contato
comigo.
Os "produtores" da minha equipe já querem
aparecer como "autor da idéia original"...
(mesmo eu tendo saído na tv, jornal, revista
e ganho certificado do Gil) pode?
desde o início eu avisei que toda a glória
seria minha e que sairia no meu nome. agora
eles já querem roer a corda, ficar com tudo
e me pagar um salário... pode? eu acho que não!
ou seja... tô f... :(

[]s


Eurico


[]s,

Roberto Takata





_\|/_
__________________________________________________
Converse com seus amigos em tempo real com o Yahoo! Messenger
http://br.download.yahoo.com/messenger/

[As partes desta mensagem que não continham texto foram removidas]



SUBJECT: Re: [ciencialist] Re: dormindo 3:30h por dia...
FROM: "Ivan Carlos" <icarlos@icarlos.net>
TO: <ciencialist@yahoogrupos.com.br>
DATE: 07/03/2005 12:51

Não é bem isso (não sou um vândalo, oras)

Para quem perguntou qual o meu trabalho, trabalho com adm. de redes de
tecnologia

A engenharia social é aplicada em diversas situações e setores, não somente
na área "hacking", é complicado julgar seu quesito desta forma, onde pode
ser usado para estudo, acesso, estelionato, violação de provacidade...

Ao contrário do q o rmtakata disse, esta tecnica não consiste em descobrir
as fraquezas do sistema de segurança de redes, mas sim as fraquesas das
pessoas que podem lhe fornecer alguma informação objetivada, não
necessariamente precisa ser da área de tecnologia

por ex, grande parte dos detetives utilizam de algum conhecimento de
engenharia social para obter as informações necessárias a seus clientes, sem
que cometa uma fraude...

[]'s

Ivan "Doomer" Carlos
Social Engineering Specialist
Cell.: +55 (11) 8112-0666
icarlos@icarlos.net
www.icarlos.net
--------------------------------------------------



----- Original Message -----
From: "rmtakata" <rmtakata@altavista.net>
To: <ciencialist@yahoogrupos.com.br>
Sent: Monday, March 07, 2005 11:55 AM
Subject: [ciencialist] Re: dormindo 3:30h por dia...




--- Em ciencialist@yahoogrupos.com.br, "Eurico Ferreira de Souza Jr."
> [E]> o que um especialista em engenharia social faz? :D

Eh outro nome para cracker. Eles fazem contatos com as pessoas para
descobrir as fraquezas do sistema de seguranca de redes de computadores.

(E o jogo, em q. peh estah?)

[]s,

Roberto Takata





##### ##### #####

Para saber mais visite
http://www.ciencialist.hpg.ig.com.br


##### ##### ##### #####
Links do Yahoo! Grupos











SUBJECT: Re: [ciencialist] Aprovacoes para C-List
FROM: Victor Pimentel Nunes <victor2002@gmail.com>
TO: ciencialist@yahoogrupos.com.br
DATE: 07/03/2005 13:00

> Vários foram as solicitações de pessoas interessadas em participar da
> C-List; a todas essas (solicitações) enviei resposta direta de aprovação,
> via Yahoo.
> Apreciaria que tais novos 'C-listeiros' se apresentassem, com brevidade e
> concisão. A todos meus particulares agradecimentos por nos elegerem como
> lista de discussão de seus interesses e anseios científicos.


Apresentando-me:

Victor Pimentel Nunes
Estudante de Engenharia Civil na UFRJ, com ênfase em Mecânica dos
Solos (ou Geotecnia) e niteroiense.

Saudações


SUBJECT: Re: dormindo 3:30h por dia...
FROM: "rmtakata" <rmtakata@altavista.net>
TO: ciencialist@yahoogrupos.com.br
DATE: 07/03/2005 15:32


--- Em ciencialist@yahoogrupos.com.br, "Ivan Carlos" <icarlos@i...>
> Ao contrário do q o rmtakata disse, esta tecnica não consiste em
> descobrir as fraquezas do sistema de segurança de redes, mas sim as
> fraquesas das pessoas que podem lhe fornecer alguma informação
> objetivada, não necessariamente precisa ser da área de tecnologia

Fraquezas das pessoas quem busca sao os chantagistas.

Os engenheiros sociais (ou crackers) contam com tres coisas: pessoas
tEm lingua grande, pessoas sao obvias, pessoas sao distraidas.

Sim, tecnicas de engenharia social podem ser usadas em outras areas,
mas do mesmo modo como aplicar calculos estatisticos em minha pesquisa
sobre transgenicos nao faz de mim um estatistico, aplicar uma ou outra
tecnica de engenharia social para conseguir informacoes para o marido
corno sobre a vida sexual de sua mulher nao faz do detetive um
engenheiro social. (Do contrario, deveriamos chamar engenheiros
sociais de espioes, jah q. usam tecnicas de espionagem tb.)

(Verdade q. nem todos os crackers sao engenheiros sociais, alguns
preferem ir na forca bruta ou explorar diretamente as fraquezas dos
sistemas de seguranca de redes.)

[]s,

Roberto Takata





SUBJECT: Re: [ciencialist] Re: dormindo 3:30h por dia...
FROM: "Ivan Carlos" <icarlos@icarlos.net>
TO: <ciencialist@yahoogrupos.com.br>
DATE: 07/03/2005 15:45

alguns crackers podem se utilizar de enhenharia social, assim como alguns
engenheiros não crackers, mas isso não quer dizer q engenheiros são crackers
nem que crackers são engenheiros, essa afirmação não tem qualquer
fundamento! :(

"(Do contrario, deveriamos chamar engenheiros sociais de espioes, jah q.
usam tecnicas de espionagem tb.)"
sim, devemos, pois a busca de informação não exposta ao público é a base
fundamentalista da engenharia social.

[]'s

Ivan "Doomer" Carlos
Social Engineering Specialist
Cell.: +55 (11) 8112-0666
icarlos@icarlos.net
www.icarlos.net
--------------------------------------------------



----- Original Message -----
From: "rmtakata" <rmtakata@altavista.net>
To: <ciencialist@yahoogrupos.com.br>
Sent: Monday, March 07, 2005 3:32 PM
Subject: [ciencialist] Re: dormindo 3:30h por dia...




--- Em ciencialist@yahoogrupos.com.br, "Ivan Carlos" <icarlos@i...>
> Ao contrário do q o rmtakata disse, esta tecnica não consiste em
> descobrir as fraquezas do sistema de segurança de redes, mas sim as
> fraquesas das pessoas que podem lhe fornecer alguma informação
> objetivada, não necessariamente precisa ser da área de tecnologia

Fraquezas das pessoas quem busca sao os chantagistas.

Os engenheiros sociais (ou crackers) contam com tres coisas: pessoas
tEm lingua grande, pessoas sao obvias, pessoas sao distraidas.

Sim, tecnicas de engenharia social podem ser usadas em outras areas,
mas do mesmo modo como aplicar calculos estatisticos em minha pesquisa
sobre transgenicos nao faz de mim um estatistico, aplicar uma ou outra
tecnica de engenharia social para conseguir informacoes para o marido
corno sobre a vida sexual de sua mulher nao faz do detetive um
engenheiro social. (Do contrario, deveriamos chamar engenheiros
sociais de espioes, jah q. usam tecnicas de espionagem tb.)

(Verdade q. nem todos os crackers sao engenheiros sociais, alguns
preferem ir na forca bruta ou explorar diretamente as fraquezas dos
sistemas de seguranca de redes.)

[]s,

Roberto Takata



SUBJECT: Porque isso acontece?
FROM: "Rodrigo Marques" <rodmarq72@yahoo.com.br>
TO: ciencialist@yahoogrupos.com.br
DATE: 07/03/2005 16:34


Estranho.....
Tire a prova você mesmo, para ver se você tem controle de seu pé
direito.

Quando você estiver sentado à sua mesa, faça círculos com o seu pé
direito
no sentido dos ponteiros de um relógio.
Enquanto estiver fazendo isso, desenhe no ar o número "6" com a sua
mão
direita.
O movimento do seu pé vai mudar de direção...
Vai circular contrário aos ponteiros de um relógio...

Recebi um e-mail com o texto acima, experimentei e realmente é
verdade. Alguém aí sabe porque?
Abraços






SUBJECT: Gabeira, Células-Tronco & OGM
FROM: "L.E.R.de Carvalho" <lecarvalho@infolink.com.br>
TO: ciencialist@yahoogrupos.com.br
DATE: 07/03/2005 16:52

Encaminhamento da votação do § 3º do art. 16 do Substitutivo do Senado ao
Projeto de Lei 2.401 de 2003 (Estabelece normas de segurança e mecanismo de
fiscalização de atividades que envolvam organismos geneticamente
modificados e seus derivados, cria o Conselho Nacional de Biossegurança -
CNBS, reestrutura a Comissão Técnica Nacional de Biossegurança - CTNBio,
dispõe sobre a Política Nacional de Biossegurança e dá outras
providências), que dá competência à CTNBio para deliberar , em última e
definitva instância, sobre os casos em que a atividade com organismos
geneticamente modificados -OGMs for potencial ou efetivamente causadora de
degradação ambiental, bem como sobre a necessidade de licenciamento ambiental.


CÂMARA DOS DEPUTADOS - DETAQ
Sessão: 013.3.52.O Hora: 20:04 Fase: OD
Orador: FERNANDO GABEIRA, PV-RJ Data: 02/03/2005

----------
O SR. FERNANDO GABEIRA (PV-RJ. Sem revisão do orador. ) - Sr. Presidente,
primeiramente, peço desculpas por insistir em falar. Sei que todos estão
cansados, mas meu compromisso não é só com as Sras. e com os Srs.
Deputados; meu compromisso é com o meu País.
Essa questão é muito importante, porque estamos concedendo poder definitivo
à CTNBio de decidir sobre a comercialização e a liberação de transgênicos
no Brasil.
O Deputado Onyx Lorenzoni argumentou que estávamos lutando a ciência contra
a política. Esse argumento é falso, não se trata de luta da ciência contra
a política, mas da ciência contra a ciência. Ora, Ecologia também é uma
ciência; e, segundo ela, num ecossistema todos os elementos já estão
inter-relacionados. Não podemos repetir a barbaridade da CTNBio de aprovar
um produto contando com a pesquisa feita em outro país, porque o Brasil tem
outro ecossistema. A prova disso é que perdemos no mundo 4 bilhões de
dólares por ano com microorganismos que vêm de ecossistemas diferentes e
que estão nas nossas praias, nos nossos portos, em todo lugar.
Portanto, é fundamental compreendermos também que nosso debate foi falso.
Pensamos que, em entregando à CTNBio, nós estaríamos entregando tudo à
Monsanto, ou que, entregando ao Ministério do Meio Ambiente, estaríamos
proibindo ou entregando essa decisão ao Fernando Gabeira. Não é verdade.
Falhamos porque não constituímos sistema de biossegurança no Brasil, onde
as vacas não têm vacina contra a febre aftosa. Vejam que o nosso País ainda
não está preparado para isso. Prova clara é que, desde o princípio,
decidimos rotular os transgênicos no Brasil. E pergunto: onde é que eles
estão rotulados? É claro que isso não interessa. Quem ainda hesita em
proibir trabalho escravo, não vai se interessar por isso. É claro que não.
Quem destruiu a Mata Atlântica, plantando café, não vai se interessar por
isso. (Muito bem!)
Os senhores estão destruindo o Brasil e sendo glorificados pela TV Globo!
Ainda bem que passou por aqui, nos anos 1930, um homem chamado Claude
Levy-Strauss, que, já naquele tempo, viu que o Brasil se destruía
progressivamente. Hoje não vemos mais decadência, mas decomposição mesmo.
Aprovar um projeto desse tipo no Brasil é aceitar a decomposição, porque
podemos fazer melhor, como fez o Canadá, como fazem os Estados Unidos, como
faz a Inglaterra, que gastou 9 milhões de dólares em pesquisas sobre um só
produto. Mas ao que parece não queremos isso; queremos, pura e
simplesmente, aprovar a matéria, achando que com isso estamos entrando no
Primeiro Mundo.
Votamos algo importante: a célula-tronco. Sr. Presidente, votei com orgulho
na célula-tronco, mas não partilho desse deslumbramento em relação à
ciência. Sou da geração que soube de Hiroshima e Nagasaki. Sei que a
ciência não é algo que se coloque num pedestal, pura e simplesmente. Ela
merece e tem de ter permanentemente a avaliação crítica da sociedade.
Muito obrigado! (Palmas prolongadas.)


[As partes desta mensagem que não continham texto foram removidas]



SUBJECT: Ciencia, tecnologia e inovacao - FSP - 07/03/05
FROM: "L.E.R.de Carvalho" <lecarvalho@infolink.com.br>
TO: ciencialist@yahoogrupos.com.br
DATE: 08/03/2005 00:54

Ciência, tecnologia e inovação

RUY ALTENFELDER

Ao longo da história da humanidade, a cultura grega
teve um significativo papel no desenvolvimento da
ciência. A ciência originou-se da fusão de duas
antigas tradições: a tradição do pensamento
filosófico, que iniciou-se na Grécia Antiga, e a
tradição dos artesãos, que começaram até mesmo mais
cedo, e mais tarde floresceram na Europa medieval.


--------------------------------------------------------------------------------
A observação imediata da natureza é relevante; contudo
o experimento específico é que promoveria respostas
--------------------------------------------------------------------------------



Tales, no século 6 a. C., considerava a água como a
origem de todas as coisas. A terra, a água, o ar e o
fogo eram os quatro elementos de Empédocles, que
ostentavam os nomes dos deuses olímpicos -de sua união
pela força do amor (Afrodite) surgiram todas as
coisas, o Sol, a Terra, as árvores e até mesmo os
"deuses eternos". Platão -cerca de 400 a. C.-
considerava o próprio universo visível um ser divino
-a imagem do Deus supremo, que é o reino das idéias;
enquanto Aristóteles -350 a. C.- admitia que o mundo
das idéias coincide com o mundo visível.
Todavia foi a ciência de Galileu, entre 1564 e 1630,
que estabeleceu os fundamentos da nova ciência e de um
novo estilo de explicação dos fatos observados.
Desde o século 16, o desenvolvimento da ciência
moderna passa a definir uma nova concepção do mundo. O
modelo de mundo como um organismo foi substituído por
um mundo como mecanismo. O cientista de hoje, ao
utilizar modelos, concepções mecanicistas ou de
qualquer tipo, considera-os apenas como meios de
descrição racional, e não como explicação da essência
do mundo.
Uma das condições indispensáveis ao avanço da ciência
moderna foi a aplicação constante do método
experimental. A observação imediata da natureza é
relevante; contudo o experimento específico é que
promoveria respostas às questões propostas.
A filosofia forneceu os conceitos para as ciências e
os artesãos forneceram as ferramentas. Foi no século
17 que se seguiram a Galileu e Newton muitos gênios
das ciências físicas, da matemática, na química e nas
ciências do corpo humano, os quais contribuíram com os
fundamentos da ciência moderna. Foi ainda nesse século
que vários institutos de pesquisa foram fundados,
entre eles o de Florença (1657), o de Londres (1660) e
o de Paris (1666). A partir dessa época as invenções
proliferaram.
Com a Revolução Industrial, na Inglaterra, nos
primórdios do século 18, surgem os melhoramentos
agrícolas, a criação de animais em fazendas, a
mineração do carvão, as eficientes máquinas a vapor
-já patenteadas em 1769-, as grandes fábricas têxteis
de algodão, em que mulheres e crianças constituíam boa
parte da força de trabalho, e dá-se o início das
exportações.
A história do Brasil é mais recente. As duas potências
marítimas Portugal e Espanha, seguindo a palavra de
ordem em busca das "novas terras", fizeram
descobrimentos que iriam alterar as cartas geográficas
nos séculos 15 e 16 e dar início à história do Brasil.
A partir daí, desenvolvem-se as capitanias
hereditárias, há as invasões francesas, a entrada das
bandeiras, a vinda da família real de Portugal para o
Brasil, a Independência, a nossa primeira
Constituição, estimulada pelo imperador em 1824. Foi
com a Proclamação da República, no final do século 19,
que nosso país iniciou seus primeiros passos no
desenvolvimento da ciência.
Entre outros fatos, a criação da Comissão Geográfica e
Geológica passa a nuclear a origem de vários dos
nossos institutos de pesquisa, de observatórios e de
hoje importantes institutos e escolas integradas a
órgãos dos governos federal e estadual, bem como a
universidades paulistas, em particular alguns
integrando a Universidade de São Paulo após sua
criação, em 1934.
Nestes nossos dias, em particular no que concerne ao
desenvolvimento da ciência, da técnica e da tecnologia
em nosso país, este é um momento marcante para a
ciência, a tecnologia e a inovação. É preciso que a
expressiva capacidade científica seja orientada para o
desenvolvimento tecnológico competitivo em prol da
produção de melhor qualidade de vida da população e da
redução das desigualdades sociais.

FONTE: www.folha.com.br - 07/03/05




SUBJECT: celulas-tronco, liberar a pesquisa - F. Gabeira
FROM: "L.E.R.de Carvalho" <lecarvalho@infolink.com.br>
TO: ciencialist@yahoogrupos.com.br
DATE: 08/03/2005 03:25


liberar a pesquisa


Começa hoje na Câmara um importante debate. Pode-se ou não fazer pesquisas
com célula tronco, com o objetivo de curar doenças e salvar a vida de
milhares de doentes?

Minha resposta é sim. Já houve um debate semelhante na Câmara e fomos
derrotados pelos evangélicos e católicos que argumentavam, segundo eles, em
favor da vida. Rigorosamente, quem argumenta a favor da vida são os
defensores da pesquisa com células troncos, pois estão pensando não em
embriões mas em seres humanos reais que precisam de uma nova terapia para
seus problemas.

Na verdade, não acredito que a genética vá resolver todos os casos graves.
Nem acredito que as doenças sejam todas de origem genética. Acredito que em
muitas delas, fatores ambientais estão presentes, como no câncer.

Essa alta expectativa acontece com todo processo científico que começa. A
própria psicanálise, cuja base científica é contestada, quando surgiu no
século passado prometia resolver mais problemas do que realmente se mostrou
capaz.

Mas essa esperança exagerada está presente em muitos processos de
descoberta. O importante é afirmar que as pesquisas com célula tronco podem
resolver alguns problemas, salvar algumas vidas, e isso já justifica a votação.

Os religiosos na Câmara precisam admitir que não se pode impor ao estado
laico uma visão de qualquer confissão, pois esta conquista, a separação do
estado e igreja já aconteceu em outro século. Por ela se verteu muito
sangue e não haverá retrocesso.

Muitos deputados evangélicos chegam aqui com a Bíblia na mão e dizem que
querem aplicar suas determinações em todas as decisões. É um erro. A Bíblia
é um livro maravilhoso mas existem outros textos também considerados
sagrados e, sobretudo, existe gente que não acredita em nenhum deles. O
Parlamento deve legislar para todos e não para uma facção religiosa. (F.G.)

<http://www.gabeira.com.br/diario//biblioteca/trabalhos/t_causas.asp?id=349>Leia
o discurso de Gabeira na Câmara.


Discurso - Biossegurança

Origem: Câmara dos Deputados

O SR. FERNANDO GABEIRA (PV-RJ. Sem revisão do orador. ) - Sr. Presidente,
peço desculpas, sei que todos estão cansados, mas meu compromissos não é só
com as Sras. e com os Srs. Deputados, meu compromisso é com meu País.

Essa questão é muito importante, porque estamos concedendo poder definitivo
à CTNBio de decidir sobre a comercialização e a liberação de transgênicos
no Brasil.

O Deputado Onyx Lorenzoni argumentou que estávamos lutando a ciência contra
a política. Esse argumento é falso, não se trata de luta da ciência contra
a política, mas da ciência contra a ciência. A Ecologia também é uma
ciência e, segundo ela, num ecossistema todos os elementos estão
inter-relacionados. Não podemos repetir a barbaridade da CTNBio de aprovar
um produto contando com a pesquisa feita em outro país, porque o Brasil tem
outro ecossistema. A prova disso é que perdemos no mundo 4 bilhões de
dólares por ano com microorganismos que vêm de ecossistemas diferentes e
que estão nas nossas praias, nos nossos portos, em todo o lugar.

Portanto, é fundamental compreendermos também que nosso debate foi falso.
Pensamos que entregando à CTNBio, nós estaríamos entregando à Monsanto, e
que entregando ao Ministério do Meio Ambiente, estaríamos proibindo ou
entregando ao Fernando Gabeira. Não é esta a verdade. Falhamos porque não
constituímos sistema de biossegurança no Brasil, onde as vacas não têm
vacina contra a febre aftosa, um país que não está preparado para isso.
Prova clara é que desde o princípio decidimos que rotular os transgênicos
no Brasil. E pergunto: onde eles estão rotulados? É claro que isso não
interessa. Quem ainda hesita em proibir o trabalho escravo, não vai se
interessar por isso. É claro que não. Quem destruiu a Mata Atlântica
plantando café, não vai se interessar por isso. (Palmas.) Os senhores estão
destruindo o Brasil e sendo glorificados pela TV Globo. É isso que acontece.

Ainda bem que passou por aqui em 1930 um homem chamado Claude Levi-Strauss
e viu que o país se destruía progressivamente. Hoje não vemos mais a
decadência, mas a decomposição.

Aprovar um projeto desse tipo no Brasil é aceitar a decomposição, porque
podemos fazer melhor, como fez o Canadá, como fazem os Estados Unidos, como
faz a Inglaterra, que gastou 9 milhões de dólares em pesquisa sobre um só
produto.

Não queremos isso, queremos pura e simplesmente aprovar. E achamos que
estamos entrando no mundo. Votamos algo importante, a célula-tronco. Sr.
Presidente, votei com orgulho na célula-tronco, mas não partilho desse
deslumbramento em relação à ciência. Sou da geração que soube de Hiroshima
e Nagasaki. Sei que a ciência não é algo que se coloque num pedestal, pura
e simplesmente. Ela merece e tem de ter permanentemente avaliação crítica
da sociedade.

Muito obrigado. (Palmas prolongadas.)


[As partes desta mensagem que não continham texto foram removidas]



SUBJECT: modelo nao-ondulatorio da difracao
FROM: "Luiz Ferraz Netto" <leobarretos@uol.com.br>
TO: <ciencialist@yahoogrupos.com.br>
DATE: 08/03/2005 06:06

Olá C-listeiros,
bom dia.

Vou começar afirmando gratuitamente que não existe nenhum modelo "não-ondulatório da difração".
Alguém tem mais informações/referências sobre esse assunto?
[]'
===========================
Luiz Ferraz Netto [Léo]
leobarretos@uol.com.br
http://www.feiradeciencias.com.br
===========================


--
Internal Virus Database is out-of-date.
Checked by AVG Anti-Virus.
Version: 7.0.300 / Virus Database: 266.5.0 - Release Date: 25/02/2005



SUBJECT: Re: [ciencialist] Obrigado
FROM: "Luiz Ferraz Netto" <leobarretos@uol.com.br>
TO: <ciencialist@yahoogrupos.com.br>
DATE: 08/03/2005 06:11

Olá João,

bem vindo.

Não consegui navegar em suas páginas --- problemas em seu javascript.

aquele abraço,
===========================
Luiz Ferraz Netto [Léo]
leobarretos@uol.com.br
http://www.feiradeciencias.com.br
===========================
-----Mensagem Original-----
De: "joaogaspar" <joaogasparilha@terra.com.br>
Para: <ciencialist@yahoogrupos.com.br>
Enviada em: segunda-feira, 7 de março de 2005 11:17
Assunto: [ciencialist] Obrigado



Obrigado pela aprovação de minha entrada no grupo.
Aproveito e envio o endereço de meu site
www.joaogaspar.com.br



[As partes desta mensagem que não continham texto foram removidas]



##### ##### #####

Para saber mais visite
http://www.ciencialist.hpg.ig.com.br


##### ##### ##### #####
Links do Yahoo! Grupos










--
Internal Virus Database is out-of-date.
Checked by AVG Anti-Virus.
Version: 7.0.300 / Virus Database: 266.5.0 - Release Date: 25/02/2005




--
Internal Virus Database is out-of-date.
Checked by AVG Anti-Virus.
Version: 7.0.300 / Virus Database: 266.5.0 - Release Date: 25/02/2005



SUBJECT: Re: [ciencialist] Aprovacoes para C-List
FROM: Lÿfffffacia Valois Leite <lvaloisleite@yahoo.com.br>
TO: ciencialist@yahoogrupos.com.br
DATE: 08/03/2005 07:15

Oi Luiz,
Meu nome é Lúcia Valois Leite, sou formada em Biologia e atuo na área de Educação.Ensino duas disciplinas na FACED-UFBA : Ensino de Ciências (EDC-270) e Projetos (EDC-200), no curso de Ciências Naturais.

Venho participando do PRONERA, desde 2004, como coordenadora pedagógica, projeto que tem por objetivo capacitar professores de alfabetização e de 1ª a 4ª séries (ensino fundamental I) no MST e da Escolarização de Jovens e Adultos de 5ª a 8ª série (ensino fundamental II) também no MST, no Recôncavo baiano e na Chapada Diamantina. Desenvolvo um trabalho de formação de professores em serviço (de Ciências Naturais), da rede municipal de ensino do município de Alagoinhas-BA e atuo como professora de Ciências Naturais da rede particular de Ensino , no Colégio Joan Miró, com crianças de 5ª e 6ª séries do ensino fundamental, onde desenvolvemos projetos de pesquisa sobre: biotecnologia, energia nuclear, tecnologias para o estudo do Universo, biopirataria.

Por conta dessas atividades educativas que sinto a necessidade de estar participando, mesmo como observadora(inicialmente), dessa lista, ok?
Se desejar mais alguma informação é só solicitar.
Grande abraço,
Lúcia.











Luiz Ferraz Netto <leobarretos@uol.com.br> wrote:
Vários foram as solicitações de pessoas interessadas em participar da C-List; a todas essas (solicitações) enviei resposta direta de aprovação, via Yahoo.
Apreciaria que tais novos 'C-listeiros' se apresentassem, com brevidade e concisão. A todos meus particulares agradecimentos por nos elegerem como lista de discussão de seus interesses e anseios científicos.
{}'
Léo, um dos n+1(oficial) moderadores da C-List. (por enquanto, n natural)

===========================
Luiz Ferraz Netto [Léo]
leobarretos@uol.com.br
http://www.feiradeciencias.com.br
===========================


--
Internal Virus Database is out-of-date.
Checked by AVG Anti-Virus.
Version: 7.0.300 / Virus Database: 266.5.0 - Release Date: 25/02/2005



##### ##### #####

Para saber mais visite
http://www.ciencialist.hpg.ig.com.br


##### ##### ##### #####


Yahoo! Grupos, um serviço oferecido por:



















function SearchComboBox() { if (document.form_combo.keyword.value.length==0){ alert("Por favor, digite algo."); return false; }else { document.form_combo.action ="http://br.rd.yahoo.com/SIG=12aamfnop/M=264105.3931087.6562589.1588051/D=brclubs/S=2137111528:HM/EXP=1110292076/A=2361264/R=0/SIG=11uaou2jn/*http://www.bondfaro.com/bondfaro/in/combosearch_in.jsp?sk=11"; } return true;} [input] [input] [input]

---------------------------------
Links do Yahoo! Grupos

Para visitar o site do seu grupo na web, acesse:
http://br.groups.yahoo.com/group/ciencialist/

Para sair deste grupo, envie um e-mail para:
ciencialist-unsubscribe@yahoogrupos.com.br

O uso que você faz do Yahoo! Grupos está sujeito aos Termos do Serviço do Yahoo!.



Lucia Valois Leite


---------------------------------
Yahoo! Acesso Grátis - Internet rápida e grátis. Instale o discador do Yahoo! agora.

[As partes desta mensagem que não continham texto foram removidas]



SUBJECT: Re: [ciencialist] Re: Água e Metabolismo
FROM: pablo@unidavi.edu.br
TO: ciencialist@yahoogrupos.com.br
DATE: 08/03/2005 07:50

e como ácido também...

Pablo

>
> A agua pode se comportar como base, como oxido ou como sal.
>
> []s,
>
> Roberto Takata
>
>
>
>
>
> ##### ##### #####
>
> Para saber mais visite
> http://www.ciencialist.hpg.ig.com.br
>
>
> ##### ##### ##### #####
> Links do Yahoo! Grupos
>
>
>
>
>
>
>
>




-------------------------------------------------
This mail sent through IMP: http://horde.org/imp/



SUBJECT: Re: Água e Metabolismo
FROM: "rmtakata" <rmtakata@altavista.net>
TO: ciencialist@yahoogrupos.com.br
DATE: 08/03/2005 08:28


Onde eu escrevi sal, leia-se acido. Agua nao se comporta como sal...
Valeu pela observacao.

[]s,

Roberto Takata

--- Em ciencialist@yahoogrupos.com.br, pablo@u... escreveu
> e como ácido também...
>
> Pablo
>
> >
> > A agua pode se comportar como base, como oxido ou como sal.
> >
> > []s,
> >
> > Roberto Takata





SUBJECT: Re: [ciencialist] Re: Água e Metabolismo
FROM: "E m i l i a n o C h e m e l l o" <chemelloe@yahoo.com.br>
TO: <ciencialist@yahoogrupos.com.br>
DATE: 08/03/2005 10:02

Que tipo de ácido? A luz de qual teoria? Existem algumas teorias que definem, dentro de seus critérios, que espécies tem comportamento ácido ou básico.

Vejam em
http://sbqensino.foco.fae.ufmg.br/uploads/540/historia.pdf

[ ] 's do Emiliano Chemello
emiliano@quimica.net
http://www.quimica.net/emiliano
http://www.ucs.br/ccet/defq/naeq

" Rien ne se perd, rien ne se crée,
tout se transforme."

Antoine Laurent de Lavoisier (químico francês, 1743 - 1794)

----- Original Message -----
From: pablo@unidavi.edu.br
To: ciencialist@yahoogrupos.com.br
Sent: Tuesday, March 08, 2005 7:50 AM
Subject: Re: [ciencialist] Re: Água e Metabolismo


e como ácido também...

Pablo

>
> A agua pode se comportar como base, como oxido ou como sal.
>
> []s,
>
> Roberto Takata
>
>
>
>
>
> ##### ##### #####
>
> Para saber mais visite
> http://www.ciencialist.hpg.ig.com.br
>
>
> ##### ##### ##### #####
> Links do Yahoo! Grupos
>
>
>
>
>
>
>
>




-------------------------------------------------
This mail sent through IMP: http://horde.org/imp/



##### ##### #####

Para saber mais visite
http://www.ciencialist.hpg.ig.com.br


##### ##### ##### #####


Yahoo! Grupos, um serviço oferecido por:







------------------------------------------------------------------------------
Links do Yahoo! Grupos

a.. Para visitar o site do seu grupo na web, acesse:
http://br.groups.yahoo.com/group/ciencialist/

b.. Para sair deste grupo, envie um e-mail para:
ciencialist-unsubscribe@yahoogrupos.com.br

c.. O uso que você faz do Yahoo! Grupos está sujeito aos Termos do Serviço do Yahoo!.



[As partes desta mensagem que não continham texto foram removidas]



SUBJECT: Re: Água e Metabolismo
FROM: "rmtakata" <rmtakata@altavista.net>
TO: ciencialist@yahoogrupos.com.br
DATE: 08/03/2005 10:17


--- Em ciencialist@yahoogrupos.com.br, "E m i l i a n o C h e m e l
> Que tipo de ácido? A luz de qual teoria? Existem algumas teorias que
> definem, dentro de seus critérios, que espécies tem comportamento
> ácido ou básico.

Como doador de protons. Ou como aceptor de eletrons.

[]s,

Roberto Takata





SUBJECT: IFUSP --- apreciem isso --- a nona de Bethoven em 1 segundo!
FROM: "Luiz Ferraz Netto" <leobarretos@uol.com.br>
TO: <ciencialist@yahoogrupos.com.br>
DATE: 08/03/2005 10:48

Velocidade inédita

10/11/2004

Agência FAPESP - O Brasil acaba de ganhar uma conexão acadêmica com velocidade inédita. Inaugurada no último dia 5 pela Universidade de São Paulo (USP) e pela Universidade Internacional da Flórida, a linha, que ligará a Rede Acadêmica de São Paulo (ANSP) com a Rede Abilene (Internet 2), dos Estados Unidos, será demonstrada pela primeira vez na quinta-feira (11/11).

O evento ocorre durante a SuperComputing 2004, conferência internacional em computação de alto desempenho, redes e armazenamento de dados que termina no próximo sábado na cidade norte-americana de Pittsburgh. A transmissão está prevista para às 14h (hora do Brasil), com demonstração no Instituto de Física da USP, e faz parte das comemorações dos 35 anos do instituto.

A iniciativa tem financiamento da FAPESP, para a parte brasileira, e da National Science Foundation (NSF), para o lado norte-americano. A nova conexão aumenta em cerca de 14 vezes a velocidade média da comunicação entre as redes acadêmicas do Estado de São Paulo e dos Estados Unidos, passando dos atuais 45 milhões de bits por segundo (Mbps) para 622 Mbps. A velocidade pode ser ampliada sempre que necessário para até 2,5 bilhões de bits por segundo (Gbps).

“A nova rede tem como missão garantir a infra-estrutura de comunicação necessária a todos os projetos de pesquisa científica e tecnológica do Estado de São Paulo que necessitem de comunicação em banda realmente larga”, explica Luis Fernandez Lopez, professor da Faculdade de Medicina da USP e coordenador da Rede ANSP.

A linha de fibra óptica pode atingir até 2,5 gigabits por segundo na transmissão de dados. É a primeira vez que os hemisférios Norte e Sul são ligados em tamanha velocidade.

“Para se ter uma idéia do significado dessa velocidade, podemos tomar como exemplo a 9ª Sinfonia de Beethoven, um paradigma de medida de informação digital depois que Akio Morita (1921-1999), presidente da Sony, durante o desenvolvimento do CD, determinou que os 74 minutos de duração da obra coubessem num único disco, de modo que ela pudesse ser ouvida sem interrupção”, disse Sérgio Novaes, do Instituto de Física Teórica da Universidade Estadual Paulista (Unesp). “Com o novo link de 2,5 Gbps, os 320 MB da gravação em alta fidelidade da Nona Sinfonia podem ser transmitidos entre São Paulo e Chicago em exatamente 1 segundo.”

A rede que está sendo inaugurada proporcionará também conectividade a projetos de pesquisa do Brasil e da América Latina, por meio da Rede Nacional de Ensino e Pesquisa (RNP) e da Rede Clara (Cooperação Latino-Americana de Redes Avançadas). “A capacidade da nova conexão será extremamente útil para projetos de pesquisa nas mais variadas áreas, como a física de partículas em altas energias, televisão digital, astronomia ou meteorologia”, conta Lopez.


Física de altas energias

As primeiras demonstrações da nova rede serão feitas na SuperComputing 2004 pela comunidade internacional de física de altas energias, que deverá também transferir dados entre diversos laboratórios e institutos ao redor do mundo, entre eles o Centro de Pesquisa em Computação Avançada do Instituto de Tecnologia da Califórnia (Caltech), o Laboratório Fermi e o Centro Europeu de Pesquisas Nucleares (Cern).

No evento em Pittsburgh, o Centro Regional de Análise de São Paulo (Sprace) deverá utilizar a conexão recém-inaugurada para transmitir dados. O Sprace foi implantado com apoio da FAPESP por meio do Projeto Temático “Física Experimental de Anéis de Colisão: SPRACE e HEPGrid−Brazil”, coordenado por Novaes. O centro vem sendo operado com a ajuda do Programa de Apoio a Jovens Pesquisadores, em projeto coordenado por Eduardo Gregores, também do Instituto de Física Teórica da Unesp.

O Sprace conta atualmente com um cluster (aglomerado) de computadores no Instituto de Física da Universidade de São Paulo que, na fase final de implantação, deverá operar com um total de 80 nós de processamento. O centro é o primeiro do HEPGrid−Brazil a entrar em operação. O HEPGrid−Brazil, iniciativa de Alberto Santoro, da Universidade do Estado do Rio de Janeiro, deverá contar em breve com outros centros de processamento.

A física de altas energias se caracteriza por grandes colaborações internacionais distribuídas pelo mundo e pela produção de uma enorme quantidade de dados. “Para se ter uma idéia dos números envolvidos, a colaboração Compact Muon Solenoid (CMS), do Cern, conta com 167 instituições de 38 países. Ela deverá produzir a cada segundo uma quantidade de dados equivalente ao de 10 mil Enciclopédias Britânicas”, explica Novaes. “Isso faz com que a área requeira a utilização de redes de alto desempenho e grande capacidade de processamento distribuído de dados entre as instituições participantes”.

Novaes conta que a demonstração durante a SuperComputing 2004 deverá envolver infra-estrutura de rede (WLAN) de última geração e serviços de computação em grade baseados na arquitetura a ser adotada pelos futuros experimentos do Large Hadron Collider (LHC), do Cern.


Parcerias

Para transformar o projeto inicial da nova rede em realidade, os professores Luis Fernandez Lopez, da USP, e Julio Ibarra, da Universidade Internacional da Flórida (FIU), pesquisadores principais de projetos financiados respectivamente pela FAPESP e pela NSF, contaram com o apoio de diversas instituições.

Além da Rede ANSP, participam o Center for Internet Augmented Research and Assessment (Ciara/FIU) e o Latin American Nautilus (Telecom Italia), que cedeu capacidade no cabo submarino da empresa entre São Paulo e Miami.

Terremark e Impsat forneceram espaço e suporte em seus datacenters, a Eletropaulo Telecom cedeu fibra óptica em São Paulo e a FPL Fibernet a fibra em Miami. A Qwest e a Internet2 entraram com a conectividade entre Miami e Pittsburgh e Cisco e Foundry supriram o equipamento ótico necessário para o funcionamento da conexão.

[]'
===========================
Luiz Ferraz Netto [Léo]
leobarretos@uol.com.br
http://www.feiradeciencias.com.br
===========================


--
Internal Virus Database is out-of-date.
Checked by AVG Anti-Virus.
Version: 7.0.300 / Virus Database: 266.5.0 - Release Date: 25/02/2005



SUBJECT: [RN] FEBRACE
FROM: "rmtakata" <rmtakata@altavista.net>
TO: ciencialist@yahoogrupos.com.br
DATE: 08/03/2005 10:49


FOLHA ONLINE, 08-03-2005

FEIRA REÚNE PEQUENOS CIENTISTAS NA USP
http://www1.folha.uol.com.br/folha/ciencia/ult306u13063.shtml

=====================
Visitação Aberta ao Público - Entrada Franca

Data: 08/03/2005 ~ 10/03/2005
Horário: 14:00 ~ 19:00
Local: Tenda FEBRACE
Endereço: Av. Professor Luciano Gualberto, travessa 3, nº380 - Cidade
Universitária


Maiores Informações e Agendamento de Visitas:
(11) 3091-5430
febrace@lsi.usp.br
http://www.lsi.usp.br/febrace/
======================

Mulherada da lista: parabens!

[]s,

Roberto Takata





SUBJECT: Re: [ciencialist] Re: dormindo 3:30h por dia...
FROM: "Ivan Carlos" <icarlos@icarlos.net>
TO: <ciencialist@yahoogrupos.com.br>
DATE: 08/03/2005 11:17

alguns crackers podem se utilizar de enhenharia social, assim como alguns
engenheiros não crackers, mas isso não quer dizer q engenheiros são crackers
nem que crackers são engenheiros, essa afirmação não tem qualquer
fundamento! :(

"(Do contrario, deveriamos chamar engenheiros sociais de espioes, jah q.
usam tecnicas de espionagem tb.)"
sim, devemos, pois a busca de informação não exposta ao público é a base
fundamentalista da engenharia social.

[]'s

Ivan "Doomer" Carlos
Social Engineering Specialist
Cell.: +55 (11) 8112-0666
icarlos@icarlos.net
www.icarlos.net
--------------------------------------------------

----- Original Message -----
From: "rmtakata" <rmtakata@altavista.net>
To: <ciencialist@yahoogrupos.com.br>
Sent: Monday, March 07, 2005 3:32 PM
Subject: [ciencialist] Re: dormindo 3:30h por dia...




--- Em ciencialist@yahoogrupos.com.br, "Ivan Carlos" <icarlos@i...>
> Ao contrário do q o rmtakata disse, esta tecnica não consiste em
> descobrir as fraquezas do sistema de segurança de redes, mas sim as
> fraquesas das pessoas que podem lhe fornecer alguma informação
> objetivada, não necessariamente precisa ser da área de tecnologia

Fraquezas das pessoas quem busca sao os chantagistas.

Os engenheiros sociais (ou crackers) contam com tres coisas: pessoas
tEm lingua grande, pessoas sao obvias, pessoas sao distraidas.

Sim, tecnicas de engenharia social podem ser usadas em outras areas,
mas do mesmo modo como aplicar calculos estatisticos em minha pesquisa
sobre transgenicos nao faz de mim um estatistico, aplicar uma ou outra
tecnica de engenharia social para conseguir informacoes para o marido
corno sobre a vida sexual de sua mulher nao faz do detetive um
engenheiro social. (Do contrario, deveriamos chamar engenheiros
sociais de espioes, jah q. usam tecnicas de espionagem tb.)

(Verdade q. nem todos os crackers sao engenheiros sociais, alguns
preferem ir na forca bruta ou explorar diretamente as fraquezas dos
sistemas de seguranca de redes.)

[]s,

Roberto Takata





##### ##### #####

Para saber mais visite
http://www.ciencialist.hpg.ig.com.br


##### ##### ##### #####
Links do Yahoo! Grupos











SUBJECT: Re: modelo nao-ondulatorio da difracao
FROM: Hélio Ricardo Carvalho <hrc@fis.puc-rio.br>
TO: ciencialist@yahoogrupos.com.br
DATE: 08/03/2005 11:28



--- Em ciencialist Leo escreveu
> Olá C-listeiros,
> bom dia.
>
> Vou começar afirmando gratuitamente que não existe nenhum modelo
"não-ondulatório da difração".
> Alguém tem mais informações/referências sobre esse assunto?
>

[Leo, você sabe, isto aí abaixo é apenas minha opinião. Acho que é
inadequado para um sítio sério como o Imperdível. :-)]


Existe modelo não-ondulatório sim!

O do Alberto Mesquita.

Não é "oficial" mas está publicado nas suas páginas e livros.

No futuro talvez tenha o meu modelo. :-)

Como a quântica "explica" tudo, temos a dualidade partícula-onda.
É mais ou menos assim: como a luz se comporta como onda em certas
experiências e como partícula em outras, e estou com preguiça para
resolver este paradoxo então ela É uma dualidade partícula-onda e
pronto.

Quanticamente falando: :-(

Imagine: Uma fonte de luz, um anteparo com um furo, e depois outro
anteparo com sensores.
Enquanto o fóton não é detectado ele é uma "onda" mas não uma onda
comum, uma "onda de probabilidade". Seria a probabilidade de ele ser
detectado ali. Isto não fecha questão sobre o que é o "fóton" neste
momento. Mostra apenas uma indeterminação. Onda de probabilidade não
existe é apenas um modelo matemático.
Mas com o tempo esta onda foi criando vida própria. Hoje ela é
considerada como um SER.
Só uma parte muito pequena desta "onda" passa pelo furo. Uma parte
muito pequena dos fótons que saem da fonte passam para o outro lado.
Isto significa perda de intensidade mas não de energia de cada
fóton. Mas agora que vem o paradoxo. Ao atravessar o furo esta onda
de probabilidade se comporta como uma onda comum ao sair de uma nova
fonte e se espalha. É mais ou menos isto aí.


Para uma tentativa de explicação mais completa (não quântica) veja
as páginas do Alberto Mesquita
http://www.ecientificocultural.com

ou

minhas msgs com o takata começando em:
http://br.groups.yahoo.com/group/ciencialist/message/44596

[ ]'s
Hélio





SUBJECT: Astonishing Amount Of Oil And Gas Off Louisiana]
FROM: "murilo filo" <avalanchedrive@hotmail.com>
TO: ciencialist@yahoogrupos.com.br
DATE: 08/03/2005 11:56

Oi, Lista.
Esta foi uma surpreza... Caramba!
Sei não... ainda assim, fico com o nosso etanol!
( Fiquei sabendo que a Embraer vai lançar o Ipanema a alcool no dia 15
prox.)
abr/M.

>From: <erickrieg@verizon.net>
>To: <free_energy@yahoogroups.com>
>Subject: [free_energy] [Fwd: Astonishing Amount Of Oil And Gas Off
>Louisiana]
>Date: Mon, 07 Mar 2005 20:41:19 -0500
>
>
>This came from another list. It would be great if we get a reprieve on
>oil.
>
>I still think even if there is enough oil in new places for another
>century, we should still look for cleaner energy sources and maybe on top
>of that, look for ways to sequester carbon.
>
>Even if the following isn't true or doesn't pan out (and assuming FE stays
>another 50 years as only rumors, lunacy and cons), I have hope that much of
>our energy needs can be met by the sun . . . particularly if real
>technology advances to greatly lower the cost of solar voltaics.
>
>Eric Krieg
>
> > Subject: Astonishing Amount Of Oil And Gas Off Louisiana
> >
> > Petroleum Geology - It's Raining Hydrocarbons In The Gulf
> > http://www.rense.com/general63/astson.htm
> >
> > By Lisa M. Pinsker
> > Geotimes
> > From 2003
> > 3-7-5
> >
> > Cathles and his team estimate that in a study area of about 9,600 square
> > miles off the coast of Louisiana, source rocks a dozen kilometers down
>have
> > generated as much as 184 billion tons of oil and gas. "That's 30 percent
> > more than we humans have consumed over the entire petroleum era,"
>Cathles
> > says. "And that's just this one little postage stamp area; if this is
>going
> > on worldwide, then there's a LOT of hydrocarbons venting out."
> >
> > Below the Gulf of Mexico, hydrocarbons flow upward through an intricate
> > network of conduits and reservoirs. They start in thin layers of source
> > rock and, from there, buoyantly rise to the surface. On their way up,
>the
> > hydrocarbons collect in little rivulets, and create temporary pockets
>like
> > rain filling a pond. Eventually most escape to the ocean. And, this is
>all
> > happening now, not millions and millions of years ago, says Larry
>Cathles,
> > a chemical geologist at Cornell University.
> >
> > "We're dealing with this giant flow-through system where the
>hydrocarbons
> > are generating now, moving through the overlying strata now, building
>the
> > reservoirs now and spilling out into the ocean now," Cathles says.
> >
> > He's bringing this new view of an active hydrocarbon cycle to industry,
> > hoping it will lead to larger oil and gas discoveries. By matching the
> > chemical signatures of the oil and gas with geologic models for the
> > structures below the seafloor, petroleum geologists could tap into
>reserves
> > larger than the North Sea, says Cathles, who presented his findings at
>the
> > meeting of the American Chemical Society in New Orleans on March 27.
> >
> > This canvas image of the study area shows the top of salt surface (salt
> > domes are spikes) in the Gas Research Institute study area and four
>areas
> > of detailed study (stratigraphic layers). The oil fields seen here are
> > Tiger Shoals, South Marsh Island 9 (SMI 9), the South Eugene Island
>Block
> > 330 area (SEI 330), and Green Canyon 184 area (Jolliet reservoirs). In
>this
> > area, 125 kilometers by 200 kilometers, Larry Cathles of Cornell
>University
> > and his team estimate hydrocarbon reserves larger than those of the
>North
> > Sea. Image by Larry Cathles.
> >
> > Cathles and his team estimate that in a study area of about 9,600 square
> > miles off the coast of Louisiana, source rocks a dozen kilometers down
>have
> > generated as much as 184 billion tons of oil and gas " about 1,000
>billion
> > barrels of oil and gas equivalent. "That's 30 percent more than we
>humans
> > have consumed over the entire petroleum era," Cathles says. "And that's
> > just this one little postage stamp area; if this is going on worldwide,
> > then there's a lot of hydrocarbons venting out."
> >
> > According to a 2000 assessment from the Minerals Management Service
>(MMS),
> > the mean undiscovered, conventionally recoverable resources in the Gulf
>of
> > Mexico offshore continental shelf are 71 billion barrels of oil
>equivalent.
> > But, says Richie Baud of MMS, not all those resources are economically
> > recoverable and they cannot be directly compared to Cathles' numbers,
> > because "our assessment only includes those hydrocarbon resources that
>are
> > conventionally recoverable whereas their study includes unconventionally
> > recoverable resources." Future MMS assessments, Baud says, may include
> > unconventionally recoverable resources, such as gas hydrates.
> >
> > Of that huge resource of naturally generated hydrocarbons, Cathles says,
> > more than 70 percent have made their way upward through the vast network
>of
> > streams and ponds, venting into the ocean, at a rate of about 0.1 ton
>per
> > year. The escaped hydrocarbons then become food for bacteria, helping to
> > fuel the oceanic food web. Another 10 percent of the Gulf's total
> > hydrocarbons are hidden in the subsurface, representing about 60 billion
> > barrels of oil and 374 trillion cubic feet of gas that could be
>extracted.
> > The remaining hydrocarbons, about 20 percent, stay trapped in the source
> > strata.
> >
> > Driving the venting process is the replacement of deep,
>carbonate-sourced
> > Jurassic hydrocarbons by shale-sourced, Eocene hydrocarbons. Determining
> > the ratio between the younger and older hydrocarbons, based on their
> > chemical signatures, is key to understanding the migration paths of the
>oil
> > and gas and the potential volume waiting to be tapped. "If the Eocene
> > source matures and its chemical signature is going to be seen near the
> > surface, it's got to displace all that earlier generated hydrocarbon "
> > that's the secret of getting a handle on this number," Cathles says.
> >
> > Another important key to understanding hydrocarbon migration is "gas
> > washing," Cathles adds. A relatively new process his research team
> > discovered in the Gulf work, gas washing refers to the regular
>interaction
> > of oil with large amounts of natural gas. In the northern area of
>Cathles'
> > study area, he estimates that gas carries off 90 percent of the oil.
> >
> > Ed Colling, senior staff geologist at ChevronTexaco, says that
>identifying
> > the depth at which gas washing occurs could be extremely useful in
>locating
> > deeper oil reserves. "If you make a discovery, by back tracking the
> > chemistry and seeing where the gas washing occurred, you have the
> > opportunity to find deeper oil," he says.
> >
> > Using such information in combination with the active hydrocarbon flow
> > model Cathles' team produced and already existing 3-D seismic analyses
> > could substantially improve accuracy in drilling for oil and gas,
>Colling
> > says. ChevronTexaco, which funds Cathles' work through the Global Basins
> > Research Network http://tinyurl.com/5829j , has been working to
>integrate
> > the technologies. (Additional funding comes from the Gas Research
>Institute.)
> >
> > "All the players are looking for bigger reserves than what's on shore,"
> > Colling says. And deep water changes the business plan. With each well a
> > multibillion dollar investment, the discovery must amount to at least
> > several hundred million barrels of oil and gas for the drilling to be
> > economic. Chemical signatures and detailed basin models are just more
>tools
> > to help them decide where to drill, he says.
> >
> > "A big part of the future of exploration is being able to effectively
>use
> > chemical information," Cathles says. Working in an area with more oil by
>at
> > least a factor of two than the North Sea, he says he hopes that his
>models
> > will help companies better allocate their resources. But equally
>important,
> > Cathles says, is that his work is shifting the way people think about
> > natural hydrocarbon vent systems " from the past to the present.
> >
> > Raining hydrocarbons in the Gulf
> > http://www.geotimes.org/june03/NN_gulf.html
> >
> > This email was cleaned by emailStripper, available for free from
> > http://www.papercut.biz/emailStripper.htm
> >
> >
> >
> >
>
>
>
>
>To drop of the list, send email to:
>free_energy-unsubscribe@yahoogroups.com
>Yahoo! Groups Links
>
>
>
>
>
>
>




SUBJECT: Fw: Informa��o
FROM: "Luiz Ferraz Netto" <leobarretos@uol.com.br>
TO: "ciencialist" <ciencialist@yahoogrupos.com.br>
DATE: 08/03/2005 12:35

Opiniões são bem vindas.

[]'
===========================
Luiz Ferraz Netto [Léo]
leobarretos@uol.com.br
http://www.feiradeciencias.com.br
===========================
-----Mensagem Original-----
De: Flávia Porto
Para: leobarretos@uol.com.br
Enviada em: domingo, 6 de março de 2005 12:23
Assunto: Informação


Prezado professor Luiz Netto,

Pesquisando na Internet, encontrei a página Feira de Ciências, no item que fala sobre a gaiola de Faraday. Gostaria que, se for possível, você esclarecesse uma dúvida minha.

Sou professora de Educação Física e pesquisadora na área de biomecânica. Recentemente, tive a oportunidade de coletar dados de eletromiografia e eletrogoniometria em uma gaiola de Faraday, numa universidade de Porto Alegre. Apenas tinha ouvido falar de tal dispositivo, conhecendo, superficialmente, na teoria, portanto. Percebi que, realmente, os sinais não apresentaram qualquer ruído (artefatos) e o processamento pôde ser muito mais rápido e fácil. Os sinais saíram "limpos" como costuma-se dizer nesta cidade.

Nesta universidade, a gaiola é utilizada como ambiente para o tratamento de pacientes de fisioterapia, no qual utilizam-se ondas curtas e de calor.

Minhas dúvidas:

o Como os sinais elétricos são protegidos pela gaiola?

o Interruptores e tomadas são colocados na parede, a alguns centímetros da gaiola. Essa distância interfere?

o Há algum efeito negativo da gaiola?

Agradeço qualquer auxílio e, desde já agradeço a atenção.

Flávia.



--------------------------------------------------------------------------------


Internal Virus Database is out-of-date.
Checked by AVG Anti-Virus.
Version: 7.0.300 / Virus Database: 266.5.0 - Release Date: 25/02/2005

----------

Internal Virus Database is out-of-date.
Checked by AVG Anti-Virus.
Version: 7.0.300 / Virus Database: 266.5.0 - Release Date: 25/02/2005


[As partes desta mensagem que não continham texto foram removidas]



SUBJECT: Re: [ciencialist] Re: Água e Metabolismo
FROM: pablo@unidavi.edu.br
TO: ciencialist@yahoogrupos.com.br
DATE: 08/03/2005 13:16

a primeira, é a velha teoria de de brosnted-lowry. A outra, segundo a n tão
jovem teoria de lewis.

>
>
> --- Em ciencialist@yahoogrupos.com.br, "E m i l i a n o C h e m e l
> > Que tipo de ácido? A luz de qual teoria? Existem algumas teorias que
> > definem, dentro de seus critérios, que espécies tem comportamento
> > ácido ou básico.
>
> Como doador de protons. Ou como aceptor de eletrons.
>
> []s,
>
> Roberto Takata
>
>
>
>
>
> ##### ##### #####
>
> Para saber mais visite
> http://www.ciencialist.hpg.ig.com.br
>
>
> ##### ##### ##### #####
> Links do Yahoo! Grupos
>
>
>
>
>
>
>
>




-------------------------------------------------
This mail sent through IMP: http://horde.org/imp/



SUBJECT: Re: [ciencialist] modelo nao-ondulatorio da difracao
FROM: "Alberto Mesquita Filho" <albmesq@uol.com.br>
TO: <ciencialist@yahoogrupos.com.br>
DATE: 08/03/2005 14:37

----- Original Message -----
From: "Luiz Ferraz Netto"
Sent: Tuesday, March 08, 2005 6:06 AM
Subject: [ciencialist] modelo nao-ondulatorio da difracao

> Vou começar afirmando gratuitamente que não existe nenhum modelo
> "não-ondulatório da difração". Alguém tem mais informações/referências
> sobre esse assunto?

Eu diria, procurando ser rigoroso, que ao falarmos em difração estamos
inconscientemente aceitando a natureza ondulatória da entidade que sofre o
processo. Com efeito, o dicionário eletrônico Houaiss afirma o seguinte:

********* início da citação **************

Difração = "Passagem de uma onda pela borda de uma barreira ou através de
uma abertura provocando, em geral, um alargamento do comprimento de onda e
interferência das frentes de onda que criam regiões de maior ou menor
intensidade."

Etimologia: lat.cien. diffractìo,ónis 'id.', criado em 1665 por Grimaldi, do
rad. de part.pas. diffractus do v.lat. diffringère 'quebrar, espedaçar,
fazer em pedaços', der. de dis- 'separar' + v. frangère 'quebrar', prov. por
intermédio do fr. diffraction (1666); ver frang- e -fring-; f.hist. 1858
difracção

********* fim da citação **************

Na "antigüidade clássica" alguns autores hoje considerados de segunda linha,
como por exemplo Sir Isaac Newton, descreveram o fenômeno com uma
denominação adequada a uma natureza corpuscular da luz, tema que caiu em
desuso depois que toda a sua Óptica foi colocada na gaveta. Newton refere-se
com freqüência a esse fenômeno denominando-o "inflexão dos raios de luz". A
luz não se espedaça, como sugere a etimologia do termo difração, os raios
simplesmente se separam, cada um deles conservando a sua integridade como
tal.

Na atualidade, em virtude do encantamento dos autores do século XIX pela luz
ondulatória, seguido do encantamento dos autores do século XX pela ficção
dualístico-quântica, a expressão difração perdeu suas raízes e passou a
representar um fenômeno que deve ter sua explicação, qualquer que seja a
teoria destinada a explicá-lo enquanto fenômeno. Neste sentido eu tenho
conservado a denominação difração, ainda que imprópria, e procuro explicá-lo
segundo minhas convicções corpusculares, conforme apontado pelo Hélio, e
aproveito o ensejo para agradecer a lembrança.

[ ]´s
Alberto
http://ecientificocultural.com/indice.htm
Mas indiferentemente a tudo isso, o neutrino tem massa, o elétron não é
uma carga elétrica coulombiana e a Terra se move. E a história se repetirá.



SUBJECT: Re: [ciencialist] Porque isso acontece?
FROM: Renato Pereira <rdpf2005@gmail.com>
TO: ciencialist@yahoogrupos.com.br
DATE: 08/03/2005 16:35

Isso ocorre pq a area do cerebro que controla os dois movimentos e a mesma...
E parece haver uma prioridade de tarefas concomitantes, nesse tipo de
sobreposicao...
[]s
Renato


On Mon, 07 Mar 2005 19:34:34 -0000, Rodrigo Marques
<rodmarq72@yahoo.com.br> wrote:
>
> Estranho.....
> Tire a prova você mesmo, para ver se você tem controle de seu pé
> direito.
>
> Quando você estiver sentado à sua mesa, faça círculos com o seu pé
> direito
> no sentido dos ponteiros de um relógio.
> Enquanto estiver fazendo isso, desenhe no ar o número "6" com a sua
> mão
> direita.
> O movimento do seu pé vai mudar de direção...
> Vai circular contrário aos ponteiros de um relógio...
>
> Recebi um e-mail com o texto acima, experimentei e realmente é
> verdade. Alguém aí sabe porque?
> Abraços
>
>
>
>
>
>
> ##### ##### #####
>
> Para saber mais visite
> http://www.ciencialist.hpg.ig.com.br
>
>
> ##### ##### ##### #####
>
>
>
> Yahoo! Grupos, um serviço oferecido por:
>
>
>
>
>
>
> ________________________________
> Links do Yahoo! Grupos
>
>
> Para visitar o site do seu grupo na web, acesse:
> http://br.groups.yahoo.com/group/ciencialist/
>
> Para sair deste grupo, envie um e-mail para:
> ciencialist-unsubscribe@yahoogrupos.com.br
>
> O uso que você faz do Yahoo! Grupos está sujeito aos Termos do Serviço do
> Yahoo!.


SUBJECT: Re: Brilho metálico
FROM: "rmtakata" <rmtakata@altavista.net>
TO: ciencialist@yahoogrupos.com.br
DATE: 08/03/2005 16:49


--- Em ciencialist@yahoogrupos.com.br, Hélio Ricardo Carvalho
> Isto significa que todo aquela espalhamento acontece numa "crista"
> da "onda" e quando a outra vem tudo já terminou. As que aparecem
> migrando para esquerda tem comprimento de onda mais de 100 vezes
> menor (100 = raiz cúbica de 10^6). E só podem ser de fótons
> diferentes!!!

Sorry, mas isso nao explica a anulacao das ondas migrando para a
esquerda: do contrario, elas seriam detetadas. Um comprimento de onda
menor, implica em maior energia - se valer o principio da conservacao
da energia, os fontos refletidos teriam entao q. ter energia menor - o
q. significaria q. teriam frequencia menor e assim os fotons
refletidos deveriam nos dar uma sensacao de cor diferente. Se
incidissemos um feixe de luz azul sobre um espelho, o reflexo teria q.
ser vermelho ou algo assim...

> > ... como fica a conservacao da energia? A energia
> >incidente teria q. ser dividida entre os diversos
> >feixes refletidos.
> >
>
> Aí eu jogo a toalha. Pois não sei bem o que é "energia". :-)
> Energia, para mim, não é um SER. :-)

De qq forma, energia pode ser medida. E suas consequencias
verificadas: por exemplo, a cor do raio refletido teria q. ser
diferente do raio incidente. Ao enxergarmos um objeto pelo espelho,
sua imagem teria q. ter cor diferente do objeto visto diretamente.

[]s,

Roberto Takata





SUBJECT: (mais) QUANTA bobagem
FROM: "brudna" <lrb@iq.ufrgs.br>
TO: ciencialist@yahoogrupos.com.br
DATE: 08/03/2005 17:00


Vou atacar de ´nao li tudo e nao gostei´.

Serah que este texto eh brincadeira?

----
A VONTADE, AS PAIXÕES E O LIVRE ARBÍTRIO QUÂNTICOS.

RESUMO

Este artigo pretende ser uma reflexão sobre o conceito de vontade, das
emoções e do livre arbítrio à luz da física clássica, em que os
objetos são claramente distintos e separados do sujeito e à luz da
teoria quântica, onde esta distinção não é mais tão clara. A vontade é
definida como força geratriz de uma representação do mundo a qual se
opõe a realidade. Deste choque, resultam as paixões(emoções). O
indeterminismo e o acaso, reinantes no microcosmos, são
reinterpretados como asilos provisórios de nossa ignorância
servindo-nos para diminuir a ânsia por controle, previsão e precisão
com o objetivo de aliviar-nos dessas incômodas - e segundo alguns
inevitáveis- manifestações que nos impregnam a alma.

http://www.pedagogia.pro.br/arbitrio-quantico.htm

---------

Seria um legitimo representante das bobagens quanticas?

Sokal neles?

Ateh
Luis Brudna






SUBJECT: O DESAFIO DA RELATIVIDADE PIRELLI
FROM: "rmtakata" <rmtakata@altavista.net>
TO: ciencialist@yahoogrupos.com.br
DATE: 08/03/2005 17:50


A Pirelli - aquela mesma, dos pneus (e outras coisas) - lançou um
concurso para premiar (com generosos 25 mil euros) quem apresentar em
um trabalho multimídia a Teoria da Relatividade do modo mais simples
(e correto).

As apresentações devem ser feitas em inglês ou em italiano. Inscrições
podem ser feitas por e-mail até 31 de março.

www.pirelliaward.com

Mãos à obra pessoal!

[]s,

Roberto Takata





SUBJECT: Re: [ciencialist] Porque isso acontece?
FROM: "Luiz Ferraz Netto" <leobarretos@uol.com.br>
TO: <ciencialist@yahoogrupos.com.br>
DATE: 08/03/2005 22:27

Renato
>Isso ocorre pq a area do cerebro que controla os dois movimentos e a mesma...
E parece haver uma prioridade de tarefas concomitantes, nesse tipo de
sobreposicao...<


Não é bem assim, a sobreposição quantica é realmente a explicação dos dois estados probabilísticos das sensações dos movimentos. Este pequeno texto pretende ser uma reflexão sobre o conceito de vontade, das emoções e do livre arbítrio à luz da física clássica, em que os
objetos são claramente distintos e separados do sujeito e à luz da teoria quântica, onde esta distinção não é mais tão clara.
A vontade de mover os pés num sentido é definida como força geratriz de uma representação do mundo a qual se opõe a realidade. Deste choque, resultam as paixões(emoções) e concomitantemente o oposto na outra perna. O indeterminismo e o acaso, reinantes no microcosmos, são reinterpretados como asilos provisórios de nossa ignorância servindo-nos para diminuir a ânsia por controle, previsão e precisão com o objetivo de aliviar-nos dessas incômodas - e segundo alguns inevitáveis- manifestações que nos impregnam a alma. Veja como é trivial a explicação desse fenômeno. Na verdade, essa é uma explicação dada por De Broglio.

[]'
Kit Walker, caixa postal 14 - Bangala
++++++++++++++++++++++++++++++++++++++++++++++
>
> Estranho.....
> Tire a prova você mesmo, para ver se você tem controle de seu pé
> direito.
>
> Quando você estiver sentado à sua mesa, faça círculos com o seu pé
> direito
> no sentido dos ponteiros de um relógio.
> Enquanto estiver fazendo isso, desenhe no ar o número "6" com a sua
> mão
> direita.
> O movimento do seu pé vai mudar de direção...
> Vai circular contrário aos ponteiros de um relógio...
>
> Recebi um e-mail com o texto acima, experimentei e realmente é
> verdade. Alguém aí sabe porque?
> Abraços



--
Internal Virus Database is out-of-date.
Checked by AVG Anti-Virus.
Version: 7.0.300 / Virus Database: 266.5.0 - Release Date: 25/02/2005



SUBJECT: Re: [ciencialist] O DESAFIO DA RELATIVIDADE PIRELLI
FROM: "Alvaro Augusto \(E\)" <alvaro@electraenergy.com.br>
TO: <ciencialist@yahoogrupos.com.br>
DATE: 08/03/2005 22:43

Parece que o prazo de inscrição foi adiado para 15 de setembro...

[ ]s

Alvaro Augusto

----- Original Message -----
From: rmtakata
To: ciencialist@yahoogrupos.com.br
Sent: Tuesday, March 08, 2005 5:50 PM
Subject: [ciencialist] O DESAFIO DA RELATIVIDADE PIRELLI



A Pirelli - aquela mesma, dos pneus (e outras coisas) - lançou um
concurso para premiar (com generosos 25 mil euros) quem apresentar em
um trabalho multimídia a Teoria da Relatividade do modo mais simples
(e correto).

As apresentações devem ser feitas em inglês ou em italiano. Inscrições
podem ser feitas por e-mail até 31 de março.

www.pirelliaward.com

Mãos à obra pessoal!

[]s,

Roberto Takata





##### ##### #####

Para saber mais visite
http://www.ciencialist.hpg.ig.com.br


##### ##### ##### #####


Yahoo! Grupos, um serviço oferecido por:







------------------------------------------------------------------------------
Links do Yahoo! Grupos

a.. Para visitar o site do seu grupo na web, acesse:
http://br.groups.yahoo.com/group/ciencialist/

b.. Para sair deste grupo, envie um e-mail para:
ciencialist-unsubscribe@yahoogrupos.com.br

c.. O uso que você faz do Yahoo! Grupos está sujeito aos Termos do Serviço do Yahoo!.



[As partes desta mensagem que não continham texto foram removidas]



SUBJECT: Lattes
FROM: "L.E.R.de Carvalho" <lecarvalho@infolink.com.br>
TO: ciencialist@yahoogrupos.com.br
DATE: 09/03/2005 00:57


>Faleceu às 15h40 desta terça-feira, no Hospital das Clínicas da Unicamp,
>aos 80 anos, o físico César Lattes, descobridor do méson pi e professor da
>Unicamp desde 1969, onde se aposentou em 1986. Lattes, cujo nome
>verdadeiro é Cesare Mansueto Giulio Lattes, nasceu em Curitiba em 11 de
>julho de 1924. Fez seus primeiros estudos em Curitiba e em São Paulo.
>Graduou-se em Física e Matemática pela Faculdade de Filosofia, Ciências e
>Letras da Universidade de São Paulo em 1943.
>FONTE:
>http://indice.uol.com.br/barra/link.htm?url=http://cienciaemdia.zip.net/




SUBJECT: Re: (mais) QUANTA bobagem
FROM: Manuel Bulcão <manuelbulcao@uol.com.br>
TO: ciencialist@yahoogrupos.com.br
DATE: 09/03/2005 03:09


Oi Brudna,

Passei os olhos rapidamente sobre o texto e a impressão que tive foi
diferente da sua.

O texto não está escrito em pós-modernês. Não me pareceu que o autor
abuse dos conceitos físicos.

Trata-se, penso eu, de um esboço de ensaio de psicofísica em que,
levando-se em conta a física moderna (Teoria da Relatividade e
mecânica quântica), tenta-se explicar o fenômeno psíquico da volição.

* Se, consoante a Teoria da Relatividade, passado, presente e futuro
jazem justapostos, isto é, se o tempo 'realmente' não flui, por que,
então, o sentimos fluir?

* Se o futuro, tanto quanto o passado, está determinado
(vide 'postscriptum'), então por que 'instintivamente' agimos como
se ele não o fosse? Isto é, se o que acontecerá já está dado, por
que então existem agentes físicos impulsionados pela 'vontade' de
que algo 'não' aconteça"?

Essas interrogações nos levam a uma questão psicofísica importante:
a sensação de passagem do tempo e a volição são meros epifenômenos
físicos ou, ao contrário, têm a capacidade de influenciar
significativamente a própria realidade física?

E esta outra: se o substrato material da mente são fluxos
eletroquímicos, então não haveria uma conexão direta desses
fenômenos mentais com a física 'indeterminista' dos 'quanta'?

Bem, existe uma físico-química, uma biofísica e até uma econofísica.
Então, por que não uma psicofísica cujo objetivo seja investigar
alguns fenômenos mentais à luz da matemática do caos, da teoria da
relatividade e da mecânica quântica?

Essa 'defesa' do autor do texto citado não implica concordância com
a sua tese.

Abraços,
Manuel Bulcão
PS: "Desde Einstein, os físicos têm geralmente rejeitado a noção de
que os eventos `acontecem', em oposição ao conceito de que meramente
existem no continuum de espaço-tempo quadridimensional" (Paul Davies)
__________

Vou atacar de ´nao li tudo e nao gostei´.

Serah que este texto eh brincadeira?

----
A VONTADE, AS PAIXÕES E O LIVRE ARBÍTRIO QUÂNTICOS.

RESUMO

Este artigo pretende ser uma reflexão sobre o conceito de vontade,
das emoções e do livre arbítrio à luz da física clássica, em que os
objetos são claramente distintos e separados do sujeito e à luz da
teoria quântica, onde esta distinção não é mais tão clara...

http://www.pedagogia.pro.br/arbitrio-quantico.htm
---------
Seria um legitimo representante das bobagens quanticas?

Sokal neles?

Ateh
Luis Brudna





SUBJECT: Re: Lattes
FROM: Manuel Bulcão <manuelbulcao@uol.com.br>
TO: ciencialist@yahoogrupos.com.br
DATE: 09/03/2005 03:39


Uma enorme perda.

Que a memória de suas importantes realizações como físico
experimental oblitere por completo a lembrança de seus ciúmes e
veleidades.

[]s
Manuel Bulcão

-------------

--- Em ciencialist@yahoogrupos.com.br, "L.E.R.de Carvalho"
<lecarvalho@i...> escreveu
>
> >Faleceu às 15h40 desta terça-feira, no Hospital das Clínicas da
Unicamp,
> >aos 80 anos, o físico César Lattes, descobridor do méson pi e
professor da
> >Unicamp desde 1969, onde se aposentou em 1986. Lattes, cujo nome
> >verdadeiro é Cesare Mansueto Giulio Lattes, nasceu em Curitiba em
11 de
> >julho de 1924. Fez seus primeiros estudos em Curitiba e em São
Paulo.
> >Graduou-se em Física e Matemática pela Faculdade de Filosofia,
Ciências e
> >Letras da Universidade de São Paulo em 1943.
> >FONTE:
> >http://indice.uol.com.br/barra/link.htm?
url=http://cienciaemdia.zip.net/





SUBJECT: Re: dormindo 3:30h por dia...
FROM: "rmtakata" <rmtakata@altavista.net>
TO: ciencialist@yahoogrupos.com.br
DATE: 09/03/2005 09:15


--- Em ciencialist@yahoogrupos.com.br, "Ivan Carlos" <icarlos@i...>
> alguns crackers podem se utilizar de enhenharia social, assim como
> alguns engenheiros não crackers, mas isso não quer dizer q
> engenheiros são crackers nem que crackers são engenheiros, essa
> afirmação não tem qualquer fundamento! :(

Tem fundamento na tradicao do uso da expressao.

http://www.seas.rochester.edu:8080/CNG/docs/Security/node9.html
http://en.wikipedia.org/wiki/Social_engineering_%28computer_security%29
http://www.securityfocus.com/infocus/1527
http://searchsecurity.techtarget.com/sDefinition/0,,sid14_gci531120,00.html
http://cybercrimes.net/Property/Hacking/Social%20Engineering/PsychSocEng/PsySocEng.html

Entre outros. (Numa versao mais amena, em vez de cracker, a engenharia
social seria uma atividade hacker.)

[]s,

Roberto Takata





SUBJECT: Quente, quente, quente
FROM: "E m i l i a n o C h e m e l l o" <chemelloe@yahoo.com.br>
TO: <ciencialist@yahoogrupos.com.br>
DATE: 09/03/2005 09:20

Quente, quente, quente

09/03/2005

Agência FAPESP - Quem acha que o mundo está ficando cada vez mais quente tem
um motivo a mais para ficar preocupado. De acordo com a Nasa, a agência
espacial norte-americana, 2004 foi o quarto ano mais quente em mais de um
século.

Os resultados da pesquisa conduzida por Makiko Sato e James Hansen, do
Instituto Goddard para Estudos Espaciais, mostraram que o ano passado teve
uma temperatura média 0,48 grau Celsius acima da média verificada entre 1951
e 1980. Outro indicador favorável à teoria do aquecimento global é que as
quatro maiores médias desde o final do século 19 ocorreram em anos recentes.
O ano mais quente foi 1998, seguido por 2002, 2003 e 2004.

Para determinar se a Terra está esquentando ou esfriando, os cientistas se
basearam na análise de temperaturas médias, obtidas primeiro diariamente e,
depois, anualmente, para um grande número de pontos em todo o planeta.
As temperaturas foram tomadas em terra e na superfície dos oceanos, por meio
de estações terrestres e satélites. Os pesquisadores trabalharam os dados em
programas de computador e converteram os resultados em mapas que mostram as
variações de temperaturas em cada região do planeta.

Os pesquisadores lembram que fenômenos naturais costumam provocar mudanças
climáticas no planeta, como as erupções vulcânicas em 1963, 1982 e 1991.
Outro evento natural com grande impacto no clima global é o El Niño, que
espalha águas quentes pelo Oceano Pacífico e ajudou a atingir a temperatura
recorde de 1998.

Mesmo levando em consideração os fenômenos naturais, os pesquisadores da
Nasa afirmam que a poluição provocada pelo homem tem ocupado um papel cada
vez mais importante nas alterações climáticas. Segundo a análise feita
agora, 2005 deverá ser mais quente do que 2004 e, provavelmente, ainda mais
quente do que 1998.

"Tem havido uma forte tendência de aquecimento nos últimos 30 anos, uma
tendência cujos motivos têm sido mostrados como derivados primordialmente do
aumento na atmosfera de gases que provocam o efeito estufa", disse Hansen em
comunicado da Nasa.

fonte: http://www.agencia.fapesp.br/boletim_dentro.php?id=3395
[ ] 's do Emiliano Chemello
emiliano@quimica.net
http://www.quimica.net/emiliano
http://www.ucs.br/ccet/defq/naeq
[ MSN ] chemelloe@hotmail.com
[ ICQ ] 145060604

" Rien ne se perd, rien ne se crée,
tout se transforme."

Antoine Laurent de Lavoisier (químico francês, 1743 - 1794)




SUBJECT: Re: Lattes
FROM: Hélio Ricardo Carvalho <hrc@fis.puc-rio.br>
TO: ciencialist@yahoogrupos.com.br
DATE: 09/03/2005 09:47


Manuel Bulcão escreveu
>
> Uma enorme perda.
>
> Que a memória de suas importantes realizações como físico
> experimental oblitere por completo a lembrança de seus ciúmes e
> veleidades.
>

E que muitos outros sigam o exemplo dele:
Não se calem diante de um erro mesmo correndo o risco de serem
chamados de ciumentos!!!!!!!!

Concordo que em alguma(s) entrevista(s) ele foi deselegante na forma
de criticar Einstein mas isto não tira a importância do conteúdo.

Hélio





SUBJECT: Re: [ciencialist] Re: Lattes
FROM: "Alvaro Augusto \(E\)" <alvaro@electraenergy.com.br>
TO: <ciencialist@yahoogrupos.com.br>
DATE: 09/03/2005 10:12

Uma ironia Cesar Lattes morrer justamente no centenário da relatividade. Aposto que foi só para chamar a atenção...

De qualquer forma, com a morte desse curitibano ilustre, agora só nos resta o Oil Man... e, claro, Newton da Costa ...

[ ]s

Alvaro Augusto


----- Original Message -----
From: Manuel Bulcão
To: ciencialist@yahoogrupos.com.br
Sent: Wednesday, March 09, 2005 3:39 AM
Subject: [ciencialist] Re: Lattes



Uma enorme perda.

Que a memória de suas importantes realizações como físico
experimental oblitere por completo a lembrança de seus ciúmes e
veleidades.

[]s
Manuel Bulcão

-------------

--- Em ciencialist@yahoogrupos.com.br, "L.E.R.de Carvalho"
<lecarvalho@i...> escreveu
>
> >Faleceu às 15h40 desta terça-feira, no Hospital das Clínicas da
Unicamp,
> >aos 80 anos, o físico César Lattes, descobridor do méson pi e
professor da
> >Unicamp desde 1969, onde se aposentou em 1986. Lattes, cujo nome
> >verdadeiro é Cesare Mansueto Giulio Lattes, nasceu em Curitiba em
11 de
> >julho de 1924. Fez seus primeiros estudos em Curitiba e em São
Paulo.
> >Graduou-se em Física e Matemática pela Faculdade de Filosofia,
Ciências e
> >Letras da Universidade de São Paulo em 1943.
> >FONTE:
> >http://indice.uol.com.br/barra/link.htm?
url=http://cienciaemdia.zip.net/





##### ##### #####

Para saber mais visite
http://www.ciencialist.hpg.ig.com.br


##### ##### ##### #####


Yahoo! Grupos, um serviço oferecido por:







------------------------------------------------------------------------------
Links do Yahoo! Grupos

a.. Para visitar o site do seu grupo na web, acesse:
http://br.groups.yahoo.com/group/ciencialist/

b.. Para sair deste grupo, envie um e-mail para:
ciencialist-unsubscribe@yahoogrupos.com.br

c.. O uso que você faz do Yahoo! Grupos está sujeito aos Termos do Serviço do Yahoo!.



[As partes desta mensagem que não continham texto foram removidas]



SUBJECT: Re: Lattes
FROM: "rmtakata" <rmtakata@altavista.net>
TO: ciencialist@yahoogrupos.com.br
DATE: 09/03/2005 10:28


--- Em ciencialist@yahoogrupos.com.br, Hélio Ricardo Carvalho <hrc@f...>
> E que muitos outros sigam o exemplo dele:
> Não se calem diante de um erro mesmo correndo o risco de serem
> chamados de ciumentos!!!!!!!!

Acho q. o exemplo melhor q. ele deu eh de fazer pesquisa de alta
qualidade.

[]s,

Roberto Takata





SUBJECT: Fw: <grandes mulheres cientistas>
FROM: "Luiz Ferraz Netto" <leobarretos@uol.com.br>
TO: "ciencialist" <ciencialist@yahoogrupos.com.br>
DATE: 09/03/2005 11:03

Oi Roberto,

vc não postou algo sobre ESSAS mulheres há pouco tempo? Onde está?

[]'
===========================
Luiz Ferraz Netto [Léo]
leobarretos@uol.com.br
http://www.feiradeciencias.com.br
===========================
-----Mensagem Original-----
De: Silvana
Para: leobarretos@uol.com.br
Enviada em: terça-feira, 8 de março de 2005 12:42
Assunto: <grandes mulheres cientistas>


presado senhor LEOBARRETOS venho por meio deste e-mail lhe pedir um esclarecimento
sobre: grandes mulheres cientistas que marcaram a história da ciencia , suas funções e o ano em que elas atuaram.
desde já fico agradecida por sua ajuda.

silvana.


--------------------------------------------------------------------------------




--------------------------------------------------------------------------------


Internal Virus Database is out-of-date.
Checked by AVG Anti-Virus.
Version: 7.0.300 / Virus Database: 266.5.0 - Release Date: 25/02/2005

----------

Internal Virus Database is out-of-date.
Checked by AVG Anti-Virus.
Version: 7.0.300 / Virus Database: 266.5.0 - Release Date: 25/02/2005


[As partes desta mensagem que não continham texto foram removidas]



SUBJECT: Re: Fw: <grandes mulheres cientistas>
FROM: "rmtakata" <rmtakata@altavista.net>
TO: ciencialist@yahoogrupos.com.br
DATE: 09/03/2005 11:33


Esta eh uma lista das ganhadoras de Nobel (de ciencias):

http://br.groups.yahoo.com/group/ciencialist/message/43932

[]s,

Roberto Takata

--- Em ciencialist@yahoogrupos.com.br, "Luiz Ferraz Netto"
> vc não postou algo sobre ESSAS mulheres há pouco tempo? Onde está?
> Luiz Ferraz Netto [Léo]
> -----Mensagem Original-----
> De: Silvana
> sobre: grandes mulheres cientistas que marcaram a história da
> ciencia , suas funções e o ano em que elas atuaram.
> desde já fico agradecida por sua ajuda.






SUBJECT: [RN] RANKING DE TI
FROM: "rmtakata" <rmtakata@altavista.net>
TO: ciencialist@yahoogrupos.com.br
DATE: 09/03/2005 14:21


FOLHA ONLINE, 09-03-2005

BRASIL CAI EM RANKING MUNDIAL DE TECNOLOGIA E FICA ATRÁS DE CHIPRE E
BAHREIN

Vinícius Albuquerque

O Brasil caiu sete posições no ranking dos países mais bem-sucedidos
na utilização e no desenvolvimento de tecnologia de comunicação e
informação (TIC), segundo o "Relatório sobre Tecnologia da Informação"
divulgado nesta quarta-feira pelo Fórum Econômico Mundial.

O Brasil passou de 39º para o 46º lugar no ranking para o período
2004-2005, ficando atrás do Chile --país latino-americano melhor
posicionado no ranking, em 35º lugar. Outros países que estão à frente
do Brasil são Malta (28º), Tunísia (31º), Eslovênia (32º), Bahrein
(33º), Chipre (37º), Hungria (38º) e Jordânia (44º).

"Com exceção do Chile, a região [da América Latina] como um todo sofre
com um arcabouço legal pobre para o desenvolvimento do setor de TIC,
pesados fardos administrativos, baixo nível de priorização
governamental para o desenvolvimento do setor, baixas taxas de
penetração da internet e fuga de especialistas, o que mina o potencial
para um crescimento mais rápido do setor", diz o documento. (e)

http://www1.folha.uol.com.br/folha/dinheiro/ult91u94157.shtml

[]s,

Roberto Takata





SUBJECT: Outra perda: Hans Bethe
FROM: "Alvaro Augusto \(E\)" <alvaro@electraenergy.com.br>
TO: <ciencialist@yahoogrupos.com.br>
DATE: 09/03/2005 15:28

Hans Bethe faleceu em 6 de março último, em Ithaca, aos 98 anos. Detalhes em http://en.wikinews.org/wiki/Nobel_Laureate_Hans_Bethe_passes_away_at_age_of_98

Para quem se interessar, há algumas palestras de Bethe em http://bethe.cornell.edu/

[ ]s

Alvaro Augusto


[As partes desta mensagem que não continham texto foram removidas]



SUBJECT: Reino Unido tem 1ª cura de diabete por transplante
FROM: "Oraculo" <oraculo@atibaia.com.br>
TO: <ciencialist@yahoogrupos.com.br>
DATE: 09/03/2005 16:30

http://noticias.terra.com.br/ciencia/interna/0,,OI484931-EI298,00.html

Reino Unido tem 1ª cura de diabete por transplante

Um paciente com diabete do tipo 1, a forma mais severa da doença, foi curado no Reino Unido com o transplante de células pancreáticas, anunciou hoje o hospital King's College de Londres. O paciente, um empresário de 61 anos, não precisa mais de injeções de insulina.
O transplante celular foi feito pela primeira vez por uma equipe canadense da Universidade de Edmonton. O procedimento consiste em retirar de um doador em estado de morte cerebral as células de Langerhans, a parte do pâncreas que contém as células que segregam a insulina. Depois são injetadas de 500 mil a um milhão dessas células em uma veia que vai até o fígado, com uma ou várias injeções. As células são fixadas no fígado do paciente, não em seu pâncreas, e começam rapidamente a segregar insulina.

O paciente do King's College recebeu três injeções. Os especialistas avaliam que, em 2004, mais de cem pessoas foram submetidas a esta terapia no mundo. Metade dos 60 enfermos tratados no Canadá não precisava mais de insulina depois de três anos.



[As partes desta mensagem que não continham texto foram removidas]



SUBJECT: Re: [ciencialist] Aprovacoes para C-List
FROM: Marcelo Cortimiglia <cortimiglia@gmail.com>
TO: ciencialist@yahoogrupos.com.br
DATE: 09/03/2005 16:46

Olá a todos,

meu nome é Marcelo Cortimiglia, e cheguei aqui através da SBCR.

Sou engenheiro civil, com mestrado em engenharia de produção pela
UFRGS. Atualmente estou preparando um projeto de doutorado. Meu
principal interesse de pesquisa reside na área de Sistemas de
Informação, particularmente o uso de TI para gestão do conhecimento e
desdobramentos no campo da aprendizagem organizacional.

Abraços,

Marcelo Cortimiglia


On Tue, 8 Mar 2005 07:15:42 -0300 (ART), Lÿfffffacia Valois Leite
<lvaloisleite@yahoo.com.br> wrote:
> Oi Luiz,
> Meu nome é Lúcia Valois Leite, sou formada em Biologia e atuo na área de
> Educação.Ensino duas disciplinas na FACED-UFBA : Ensino de Ciências
> (EDC-270) e Projetos (EDC-200), no curso de Ciências Naturais.
>
> Venho participando do PRONERA, desde 2004, como coordenadora pedagógica,
> projeto que tem por objetivo capacitar professores de alfabetização e de 1ª
> a 4ª séries (ensino fundamental I) no MST e da Escolarização de Jovens e
> Adultos de 5ª a 8ª série (ensino fundamental II) também no MST, no Recôncavo
> baiano e na Chapada Diamantina. Desenvolvo um trabalho de formação de
> professores em serviço (de Ciências Naturais), da rede municipal de ensino
> do município de Alagoinhas-BA e atuo como professora de Ciências Naturais da
> rede particular de Ensino , no Colégio Joan Miró, com crianças de 5ª e 6ª
> séries do ensino fundamental, onde desenvolvemos projetos de pesquisa sobre:
> biotecnologia, energia nuclear, tecnologias para o estudo do Universo,
> biopirataria.
>
> Por conta dessas atividades educativas que sinto a necessidade de estar
> participando, mesmo como observadora(inicialmente), dessa lista, ok?
> Se desejar mais alguma informação é só solicitar.
> Grande abraço,
> Lúcia.
>


SUBJECT: Fw: �ngulo
FROM: "Luiz Ferraz Netto" <leobarretos@uol.com.br>
TO: "ciencialist" <ciencialist@yahoogrupos.com.br>
DATE: 09/03/2005 19:38

? é, eu enfrento todas! ... quase .....

[]'
===========================
Luiz Ferraz Netto [Léo]
leobarretos@uol.com.br
http://www.feiradeciencias.com.br
===========================
-----Mensagem Original-----
De: José Walter da Costa - Grupo Etesco
Para: leobarretos@uol.com.br
Enviada em: terça-feira, 8 de março de 2005 18:35
Assunto: ângulo


Gostaria de saber quem descobriu o ângulo


----------

Internal Virus Database is out-of-date.
Checked by AVG Anti-Virus.
Version: 7.0.300 / Virus Database: 266.5.0 - Release Date: 25/02/2005


[As partes desta mensagem que não continham texto foram removidas]



SUBJECT: Re: [ciencialist] Aprovacoes para C-List
FROM: "Luiz Ferraz Netto" <leobarretos@uol.com.br>
TO: <ciencialist@yahoogrupos.com.br>
DATE: 09/03/2005 19:53

Novos participantes,
são todos bem vindos.

Apresentem-se e entrem na rodada, se o problema é falta de pergunta aqui vão algumas:

1- o que vc entende por 1 kg de algodão e 1 kg de chumbo? Temos mesma massa de ambos?
2- por que as varetas das antenas de TV (principalmente na região da praia) estragam rapidamente?
3- Se vc soprar as costas de sua mão, com a boca aberta, o ar sai quente; se soprar com a boca fazendo 'biquinho' o ar sai frio. Por que? Que eletrodoméstico usa isso?
4- que fazem aquelas bolas (vermelhas) que são atravessadas por fios pertencentes às redes de alta tensão, principalmente no cruzamento com rodovias?
5- à época dos mosquetes, como eram feitas as esferas de chumbo das balas?

et coetera ......

[]'
===========================
Luiz Ferraz Netto [Léo]
leobarretos@uol.com.br
http://www.feiradeciencias.com.br
===========================
-----Mensagem Original-----
De: "Marcelo Cortimiglia" <cortimiglia@gmail.com>
Para: <ciencialist@yahoogrupos.com.br>
Enviada em: quarta-feira, 9 de março de 2005 16:46
Assunto: Re: [ciencialist] Aprovacoes para C-List



Olá a todos,

meu nome é Marcelo Cortimiglia, e cheguei aqui através da SBCR.

Sou engenheiro civil, com mestrado em engenharia de produção pela
UFRGS. Atualmente estou preparando um projeto de doutorado. Meu
principal interesse de pesquisa reside na área de Sistemas de
Informação, particularmente o uso de TI para gestão do conhecimento e
desdobramentos no campo da aprendizagem organizacional.

Abraços,

Marcelo Cortimiglia


On Tue, 8 Mar 2005 07:15:42 -0300 (ART), Lÿfffffacia Valois Leite
<lvaloisleite@yahoo.com.br> wrote:
> Oi Luiz,
> Meu nome é Lúcia Valois Leite, sou formada em Biologia e atuo na área de
> Educação.Ensino duas disciplinas na FACED-UFBA : Ensino de Ciências
> (EDC-270) e Projetos (EDC-200), no curso de Ciências Naturais.
>
> Venho participando do PRONERA, desde 2004, como coordenadora pedagógica,
> projeto que tem por objetivo capacitar professores de alfabetização e de 1ª
> a 4ª séries (ensino fundamental I) no MST e da Escolarização de Jovens e
> Adultos de 5ª a 8ª série (ensino fundamental II) também no MST, no Recôncavo
> baiano e na Chapada Diamantina. Desenvolvo um trabalho de formação de
> professores em serviço (de Ciências Naturais), da rede municipal de ensino
> do município de Alagoinhas-BA e atuo como professora de Ciências Naturais da
> rede particular de Ensino , no Colégio Joan Miró, com crianças de 5ª e 6ª
> séries do ensino fundamental, onde desenvolvemos projetos de pesquisa sobre:
> biotecnologia, energia nuclear, tecnologias para o estudo do Universo,
> biopirataria.
>
> Por conta dessas atividades educativas que sinto a necessidade de estar
> participando, mesmo como observadora(inicialmente), dessa lista, ok?
> Se desejar mais alguma informação é só solicitar.
> Grande abraço,
> Lúcia.
>


##### ##### #####

Para saber mais visite
http://www.ciencialist.hpg.ig.com.br


##### ##### ##### #####
Links do Yahoo! Grupos










--
Internal Virus Database is out-of-date.
Checked by AVG Anti-Virus.
Version: 7.0.300 / Virus Database: 266.5.0 - Release Date: 25/02/2005




--
Internal Virus Database is out-of-date.
Checked by AVG Anti-Virus.
Version: 7.0.300 / Virus Database: 266.5.0 - Release Date: 25/02/2005



SUBJECT: Re: [ciencialist] Fw: ângulo
FROM: "Alberto Mesquita Filho" <albmesq@uol.com.br>
TO: <ciencialist@yahoogrupos.com.br>
DATE: 09/03/2005 20:41

----- Original Message -----
From: "Luiz Ferraz Netto"
Sent: Wednesday, March 09, 2005 7:38 PM
Subject: [ciencialist] Fw: ângulo

-----Mensagem Original-----
De: José Walter da Costa - Grupo Etesco
Para: leobarretos@uol.com.br
Enviada em: terça-feira, 8 de março de 2005 18:35
Assunto: ângulo

> Gostaria de saber quem descobriu o ângulo

Em http://users.hotlink.com.br/marielli/matematica/histomatica/histogeo.html
lê-se o seguinte:

********* início da citação ****************

Foi atribuído aos egípcios e aos caldeus, pelos historiadores, a criação da
geometria.

[...]

Os gregos herdaram dos babilônios e, habituados ao uso dos ângulos por sua
longa experiência astronômica, introduziram muito cedo, na matemática, a
idéia de ângulo, sabendo-se que na época clássica eram definidos apenas
ângulos inferiores a dois retos, ou seja, menores que 180º.

********* fim da citação ****************

[ ]´s
Alberto
http://ecientificocultural.com/indice.htm
Mas indiferentemente a tudo isso, o neutrino tem massa, o elétron não é
uma carga elétrica coulombiana e a Terra se move. E a história se repetirá.



SUBJECT: Re: [ciencialist] Cientistas recriam nervos ópticos em ratos
FROM: "Ivan Carlos" <icarlos@icarlos.net>
TO: <ciencialist@yahoogrupos.com.br>
DATE: 09/03/2005 21:26

acho engraçado pacas expressões do tipo "O próximo passo agora será
determinar se os nervos regenerados funcionam direito." =)))

Ivan "Doomer" Carlos
Social Engineering Specialist
Cell.: +55 (11) 8112-0666
icarlos@icarlos.net
www.icarlos.net
--------------------------------------------------


----- Original Message -----
From: "Oraculo" <oraculo@atibaia.com.br>
To: <ciencialist@yahoogrupos.com.br>
Sent: Monday, March 07, 2005 1:52 AM
Subject: [ciencialist] Cientistas recriam nervos ópticos em ratos



Cientistas recriam nervos ópticos em ratos

Cientistas dizem ter conseguido reconstruir em ratos nervos ópticos
danificados em toda a extensão do percurso desde o olho até o cérebro.
Especialistas dizem que a experiência, feita pelo Instituto Schepens de
Pesquisas do Olho, traz novas esperanças para tratar o glaucoma, uma
enfermidade na qual um nível elevado de pressão destrói o nervo óptico.

A técnica também poderia ser usada para ajudar pessoas que sofrem de
problemas na medula espinhal e em outras partes do sistema nervoso.

A experiência é detalhada na revista especializada Journal of Cell Science.

Cicatriz

"Isto é o mais próximo que a ciência já chegou de regenerar tantas fibras de
nervos ópticos em uma distância tão grande para chegar ao seu objetivo",
disse Dong Feng Chen, o líder da equipe de pesquisadores.

"E também de reparar um nervo que antes se achava que tinha se danificado de
forma irreparável."

Vários tecidos celulares do corpo humano se reconstituem sozinhos em caso de
lesão.

Mas não o nervo óptico e outros tecidos do sistema nervoso central, cujos
danos são sempre permanentes.

Os cientistas do Instituto Schepens descobriram que a incapacidade do nervo
óptico de se regenerar estava ligada ao não funcionamento de um gene chamado
BCL-2.

Eles também descobriram que o processo de regeneração estava bloqueado pela
criação, logo após o nascimento, de uma cicatriz no cérebro por células
gliais especializadas.

Essas células desempenham várias funções no cérebro, entre as quais a
criação dessa espécie de cicatriz.

A cicatriz coloca uma barreira ao mesmo tempo física e molecular para a
regeneração.

Sempre ativo

Os cientistas criaram ratos nos quais o BCL-2 estava sempre ativado.

Eles descobriram que os animais conseguiam reparar os nervos ópticos com
rapidez - mas só enquanto eram jovens e a cicatriz no cérebro ainda não
havia se desenvolvido.

Em seguida eles criaram ratos com o BCL-2 funcionando e uma reduzida
capacidade de criar as cicatrizes pelas células gliais.

Desta vez, mesmo os roedores mais velhos conseguiram regenerar as células
ópticas.

O próximo passo agora será determinar se os nervos regenerados funcionam
direito.




http://www.bbc.co.uk/portuguese/ciencia/story/2005/03/050302_nervoopticoro.shtml

[As partes desta mensagem que não continham texto foram removidas]



##### ##### #####

Para saber mais visite
http://www.ciencialist.hpg.ig.com.br


##### ##### ##### #####
Links do Yahoo! Grupos











SUBJECT: Re: [ciencialist] Aprovacoes para C-List
FROM: "Ivan Carlos" <icarlos@icarlos.net>
TO: <ciencialist@yahoogrupos.com.br>
DATE: 09/03/2005 21:27

> Vários foram as solicitações de pessoas interessadas em participar da
> C-List; a todas essas (solicitações) enviei resposta direta de aprovação,
> via Yahoo.
> Apreciaria que tais novos 'C-listeiros' se apresentassem, com brevidade e
> concisão. A todos meus particulares agradecimentos por nos elegerem como
> lista de discussão de seus interesses e anseios científicos.


Apresentando-me too:

Ivan Carlos

Coordenador TI, Adm. de Redes
Estudante de Gerenciamento de Redes
22 anos

[]'s

Ivan "Doomer" Carlos
Social Engineering Specialist
Cell.: +55 (11) 8112-0666
icarlos@icarlos.net
www.icarlos.net
--------------------------------------------------




SUBJECT: Re: Água e Metabolismo
FROM: gisele campos <giscampos@yahoo.com.br>
TO: ciencialist@yahoogrupos.com.br
DATE: 09/03/2005 23:49


Como base ???? , neste caso eu já não entendi , sendo o hidrogenio da água (H2O) , um cátion ....... a hidroxila, um ânion !

Como ácido , como sal e como óxido , é bem plausível ...

Carpe dien .......
Gisele


pablo@unidavi.edu.br wrote:
e como ácido também...

Pablo

>
> A agua pode se comportar como base, como oxido ou como sal.
>
> []s,
>
> Roberto Takata
>
>
>
>
>
> ##### ##### #####
>
> Para saber mais visite
> http://www.ciencialist.hpg.ig.com.br
>
>
> ##### ##### ##### #####
> Links do Yahoo! Grupos
>
>
>
>
>
>
>
>




-------------------------------------------------
This mail sent through IMP: http://horde.org/imp/



##### ##### #####

Para saber mais visite
http://www.ciencialist.hpg.ig.com.br


##### ##### ##### #####


Yahoo! Grupos, um serviço oferecido por:



















function SearchComboBox() { if (document.form_combo.keyword.value.length==0){ alert("Por favor, digite algo."); return false; }else { document.form_combo.action ="http://br.rd.yahoo.com/SIG=12a5ulmtc/M=264105.3931087.6562589.1588051/D=brclubs/S=2137111528:HM/EXP=1110366048/A=2361264/R=0/SIG=11uaou2jn/*http://www.bondfaro.com/bondfaro/in/combosearch_in.jsp?sk=11"; } return true;} [input] [input] [input]

---------------------------------
Links do Yahoo! Grupos

Para visitar o site do seu grupo na web, acesse:
http://br.groups.yahoo.com/group/ciencialist/

Para sair deste grupo, envie um e-mail para:
ciencialist-unsubscribe@yahoogrupos.com.br

O uso que você faz do Yahoo! Grupos está sujeito aos Termos do Serviço do Yahoo!.



---------------------------------
Yahoo! Acesso Grátis - Internet rápida e grátis. Instale o discador do Yahoo! agora.

[As partes desta mensagem que não continham texto foram removidas]



SUBJECT: Re: Fw: <grandes mulheres cientistas>
FROM: Maria Natália <grasdic@hotmail.com>
TO: ciencialist@yahoogrupos.com.br
DATE: 09/03/2005 23:55


Professor e Mestre Nosso, Leo:

Por mommentos pensei que um vento de modernidade estivesse passando
pela C-List.
Vã sensação esta que me assolou a alma...Chuifff...chuifff . Vinham os
agradecimentos dos men da lista às Mulheres que aqui têm colaborado...
FOI. Pois foi o dia da Mulher e nem uma rosa aqui ficou. Em pvt tive
algumas mensagens de Homens desta lista a que ainda nem agradeci...mas
nunca é tarde, né? Aqui vai para os homens da lista o meu
agradecimento por se lembrarem que aqui temos 365 dias de vossa atenção:
--------888888
_8888___88888888___8888
888888_8888888888_888888
888888888888888888888888
888888888888888888888888
_8888888888888888888888
__88888888888888888888
____8888888888888888
______888888888888
___________**
_###_______**______###
#######___**___#######
__######__**__######
____######**######
______####**####

Umas beijocas nas vossas bochechas rosadinhas e bem escanhoadas
Maria Natália






SUBJECT: Assoprar a mão e outras
FROM: Luis Brudna <luisbrudna@gmail.com>
TO: ciencialist@yahoogrupos.com.br
DATE: 10/03/2005 00:12

Posso?

> 2- por que as varetas das antenas de TV (principalmente na região da praia)
> estragam rapidamente?

Mas... mas... as que conheço são feitas de alumínio e não estragam.
Bom, ando meio afastado da praia.


> 3- Se vc soprar as costas de sua mão, com a boca aberta, o ar sai quente; se
> soprar com a boca fazendo 'biquinho' o ar sai frio. Por que? Que
> eletrodoméstico usa isso?

Han? Pega ratão? Seria uma tentativa de efeito Joule-Thomson
caseira? Não creio que essa seja a explicação mais sensata, apesar de
já ter ouvido.


> 4- que fazem aquelas bolas (vermelhas) que são atravessadas por fios
> pertencentes às redes de alta tensão, principalmente no cruzamento com
> rodovias?

Hehe... o Takata deve saber direitinho o número das mensagens onde
isso foi debatido na Ciencialist.


> 5- à época dos mosquetes, como eram feitas as esferas de chumbo das balas?

Cláaaassica. Deixando cair em uma coluna d'agua?


Até
Luís Brudna


SUBJECT: Os desastres de Sofia
FROM: Maria Natália <grasdic@hotmail.com>
TO: ciencialist@yahoogrupos.com.br
DATE: 10/03/2005 00:27


Vos apresento um dos nossos melhores professores de Estatística da
Universidade de Lisboa. Esta mensagem é uma chamada aos colegas de
Lisboa e com anteced~encia Venham preparados para ficar sentados no
chão se não vierem a horas decentes...Os que têm de vir a Lisboa podem
fazer o programa para estarem cá nesta ocasião. Vale a pena e se não
gostarem mo digam lá, que vos pago o jantar com todo o gosto.
A primeira aula dele há 5 anos foi: "A Mais velha Profissão do Mundo
está bem paga?" e teve os meus alunos de 15 anos atentos desde o
princípio ao fim. E ouvi "agora percebo o que é a estatística". Eram
200 alunos de ensino secundário!!!

""Os Desastres de Sofia e as Estruturas do Acaso Dinis Pestana no
Ciclo Despertar para a Ciência (23 Março)
Dinis Pestana é o próximo orador do Ciclo Despertar para a Ciência,
promovido pelo Serviço de Ciência da FCG. Dinis Pestana, autor e
co-autor de diversas obras na área da estatística, vai falar sobre a
estatísca e o modo como esta ciência nos ajuda a lidar com a incerteza.

Dinis Pestana licenciou-se em Matemática Pura (Univ. de Lisboa, 1972),
doutorou-se em Probabilidade e Estatística na Univ. Sheffield, UK
(1978) e prestou provas de Agregação, em Matemática Aplicada, na
Universidade de Lisboa (1983), sendo Professor Catedrático daquela
Universidade desde 1986. Dirige a linha de investigação em
Probabilidade e Análise Funcional do CEAUL — Centro de Estatística e
Aplicações da Universidade de Lisboa. Foi representante do Conselho de
Reitores das Universidades Portuguesas no Conselho Superior de
Estatística, participando nessa qualidade na comissão de preparação e
seguimento do Censo de 2001. Publicou cerca de quarenta artigos, em
livros da Wiley, Springer e North Holland, e em revistas
internacionais. A convite de João Caraça, Director de Ciência da FCG,
Dinis Pestana preparou um programa de televisão (O Acaso não Acontece
por Acaso) sobre Probabilidade. Dinis Pestana defende que é preciso
saber ler os dados, e (que) a montante disso, ainda (é preciso) saber
que nem todos os dados prestam, e por isso aprender a obter os dados
que prestam: "A estatística é uma espécie de canivete suiço, a que
vamos sempre acrescentando novas ferramentas para abordar um mundo
cada vez mais complexo. Todos lidamos com a incerteza e em menor ou
maior quota todos precisamos de saber utilizar ao menos parcialmente
esse canivete. Porque se não soubermos, como procuraremos exemplificar
nesta conversa, vamos contribuir para que ocorram mais alguns
desastres à Sofia."

O evento tem entrada livre e será transmitido em videodifusão no site
www.live.fccn.pt/fcg.

O ciclo Despertar para a Ciência é organizado pelo Serviço de Ciência
da Fundação Calouste Gulbenkian em colaboração com a Fundação para a
Ciência e Tecnologia.

23 de Março, quarta-feira
Auditório 2
18h00 ""
Um abraço
Maria Natália








SUBJECT: Re: [ciencialist] Cientistas recriam nervos ópticos em ratos
FROM: "Oraculo" <oraculo@atibaia.com.br>
TO: <ciencialist@yahoogrupos.com.br>
DATE: 10/03/2005 01:33

Olá Ivan

Por que?..:-) Quero dizer, o que há de engraçado em ser cuidadoso com descobertas e dados de pesquisa, como o correto funcionamento de um sistema nervoso (em um procedimento de recuperação que nunca foi possível antes, e do qual temos pouca informação)?..:-)

Toda pesquisa cientifica tem um próximo passo, significando que nenhum conhecimento é completo ou final, dependendo sempre de ajustes e de novas pesquisas. Por exemplo, os primeiros passos com células tronco determinam apenas resultados parciais, ainda sem que se saiba exatameten como essa melhora ocorre. Os próximos passos são, claro, descobrir como isso cocore, se pode ser melhrpoado, como induzir determinados padrões de tecidos, etc.

O mesmo para a (fantastica) nova e recém descoberta capacidade de regenerar nervos óticos. O próximo passo é determianr se esse novo nervo regenerado funciona da mesma forma que o nervo original, ou se, por exemplo, embaralha os sinais de forma a impedir que sejam corretamente interpretados pelo cérebro. Não há, sei que deve percebner isso, nenhuma chance de aprendermos a recuperar nervos óticos de seres humanso de um dia para o outro, sem antes passar por diversas (e as vezes frustantes) fases e passos cuidadosos..:-)

Esse discurso todo acima (o pessoal da lista pode garantir que eu escrevo demais mesmo..:-) é apenas para determinar o que achou engraçado na frase dos pesquisadores..:-) Em geral, esse tipo de graça é derivada da acusação sempre recorrente que "cientistas não sabem de nada e ficam contando vantagem" ou coisa parecida..:-) O que, me parece, não é exatamente verdade.:-)

Se a intenção não era essa, me desculpe o engano (e o tamanho desta mensagem..:-)

Um abraço.

Homero


----- Original Message -----
From: Ivan Carlos
To: ciencialist@yahoogrupos.com.br
Sent: Wednesday, March 09, 2005 9:26 PM
Subject: Re: [ciencialist] Cientistas recriam nervos ópticos em ratos


acho engraçado pacas expressões do tipo "O próximo passo agora será
determinar se os nervos regenerados funcionam direito." =)))

Ivan "Doomer" Carlos
Social Engineering Specialist
Cell.: +55 (11) 8112-0666
icarlos@icarlos.net
www.icarlos.net
--------------------------------------------------


----- Original Message -----
From: "Oraculo" <oraculo@atibaia.com.br>
To: <ciencialist@yahoogrupos.com.br>
Sent: Monday, March 07, 2005 1:52 AM
Subject: [ciencialist] Cientistas recriam nervos ópticos em ratos



Cientistas recriam nervos ópticos em ratos

Cientistas dizem ter conseguido reconstruir em ratos nervos ópticos
danificados em toda a extensão do percurso desde o olho até o cérebro.
Especialistas dizem que a experiência, feita pelo Instituto Schepens de
Pesquisas do Olho, traz novas esperanças para tratar o glaucoma, uma
enfermidade na qual um nível elevado de pressão destrói o nervo óptico.

A técnica também poderia ser usada para ajudar pessoas que sofrem de
problemas na medula espinhal e em outras partes do sistema nervoso.

A experiência é detalhada na revista especializada Journal of Cell Science.

Cicatriz

"Isto é o mais próximo que a ciência já chegou de regenerar tantas fibras de
nervos ópticos em uma distância tão grande para chegar ao seu objetivo",
disse Dong Feng Chen, o líder da equipe de pesquisadores.

"E também de reparar um nervo que antes se achava que tinha se danificado de
forma irreparável."

Vários tecidos celulares do corpo humano se reconstituem sozinhos em caso de
lesão.

Mas não o nervo óptico e outros tecidos do sistema nervoso central, cujos
danos são sempre permanentes.

Os cientistas do Instituto Schepens descobriram que a incapacidade do nervo
óptico de se regenerar estava ligada ao não funcionamento de um gene chamado
BCL-2.

Eles também descobriram que o processo de regeneração estava bloqueado pela
criação, logo após o nascimento, de uma cicatriz no cérebro por células
gliais especializadas.

Essas células desempenham várias funções no cérebro, entre as quais a
criação dessa espécie de cicatriz.

A cicatriz coloca uma barreira ao mesmo tempo física e molecular para a
regeneração.

Sempre ativo

Os cientistas criaram ratos nos quais o BCL-2 estava sempre ativado.

Eles descobriram que os animais conseguiam reparar os nervos ópticos com
rapidez - mas só enquanto eram jovens e a cicatriz no cérebro ainda não
havia se desenvolvido.

Em seguida eles criaram ratos com o BCL-2 funcionando e uma reduzida
capacidade de criar as cicatrizes pelas células gliais.

Desta vez, mesmo os roedores mais velhos conseguiram regenerar as células
ópticas.

O próximo passo agora será determinar se os nervos regenerados funcionam
direito.




http://www.bbc.co.uk/portuguese/ciencia/story/2005/03/050302_nervoopticoro.shtml

[As partes desta mensagem que não continham texto foram removidas]



##### ##### #####

Para saber mais visite
http://www.ciencialist.hpg.ig.com.br


##### ##### ##### #####
Links do Yahoo! Grupos











##### ##### #####

Para saber mais visite
http://www.ciencialist.hpg.ig.com.br


##### ##### ##### #####


Yahoo! Grupos, um serviço oferecido por:







------------------------------------------------------------------------------
Links do Yahoo! Grupos

a.. Para visitar o site do seu grupo na web, acesse:
http://br.groups.yahoo.com/group/ciencialist/

b.. Para sair deste grupo, envie um e-mail para:
ciencialist-unsubscribe@yahoogrupos.com.br

c.. O uso que você faz do Yahoo! Grupos está sujeito aos Termos do Serviço do Yahoo!.



[As partes desta mensagem que não continham texto foram removidas]



SUBJECT: Re: [ciencialist] Cientistas recriam nervos ópticos em ratos
FROM: "Ivan Carlos" <icarlos@icarlos.net>
TO: <ciencialist@yahoogrupos.com.br>
DATE: 10/03/2005 02:13

Nanananaum foi isso o que quiz dizer não!

Diferente de achar q quiz dizer que "cientistas não sabem de nada e ficam
contando vantagem", fico maravilhado com o avanço dos trabalhos, só achei
engraçado pq o texto passou e muito uma impressão de ironia, talvez tenha
sido descuido do autor, mas não que eu concorde com isso...!

Acompanho a lista sbcr, acompanho as pesquisas do campo da neurociência,
compreendo o que quiz dizer sobre os passos de desenvolvimento da pesquisa.

Continue escrevendo bastante, não sei me expressar bem, assim vc me completa
:)

[]!

Ivan "Doomer" Carlos
Social Engineering Specialist
Cell.: +55 (11) 8112-0666
icarlos@icarlos.net
www.icarlos.net
--------------------------------------------------


----- Original Message -----
From: "Oraculo" <oraculo@atibaia.com.br>
To: <ciencialist@yahoogrupos.com.br>
Sent: Thursday, March 10, 2005 1:33 AM
Subject: Re: [ciencialist] Cientistas recriam nervos ópticos em ratos



Olá Ivan

Por que?..:-) Quero dizer, o que há de engraçado em ser cuidadoso com
descobertas e dados de pesquisa, como o correto funcionamento de um sistema
nervoso (em um procedimento de recuperação que nunca foi possível antes, e
do qual temos pouca informação)?..:-)

Toda pesquisa cientifica tem um próximo passo, significando que nenhum
conhecimento é completo ou final, dependendo sempre de ajustes e de novas
pesquisas. Por exemplo, os primeiros passos com células tronco determinam
apenas resultados parciais, ainda sem que se saiba exatameten como essa
melhora ocorre. Os próximos passos são, claro, descobrir como isso cocore,
se pode ser melhrpoado, como induzir determinados padrões de tecidos, etc.

O mesmo para a (fantastica) nova e recém descoberta capacidade de regenerar
nervos óticos. O próximo passo é determianr se esse novo nervo regenerado
funciona da mesma forma que o nervo original, ou se, por exemplo, embaralha
os sinais de forma a impedir que sejam corretamente interpretados pelo
cérebro. Não há, sei que deve percebner isso, nenhuma chance de aprendermos
a recuperar nervos óticos de seres humanso de um dia para o outro, sem antes
passar por diversas (e as vezes frustantes) fases e passos cuidadosos..:-)

Esse discurso todo acima (o pessoal da lista pode garantir que eu escrevo
demais mesmo..:-) é apenas para determinar o que achou engraçado na frase
dos pesquisadores..:-) Em geral, esse tipo de graça é derivada da acusação
sempre recorrente que "cientistas não sabem de nada e ficam contando
vantagem" ou coisa parecida..:-) O que, me parece, não é exatamente
verdade.:-)

Se a intenção não era essa, me desculpe o engano (e o tamanho desta
mensagem..:-)

Um abraço.

Homero


----- Original Message -----
From: Ivan Carlos
To: ciencialist@yahoogrupos.com.br
Sent: Wednesday, March 09, 2005 9:26 PM
Subject: Re: [ciencialist] Cientistas recriam nervos ópticos em ratos


acho engraçado pacas expressões do tipo "O próximo passo agora será
determinar se os nervos regenerados funcionam direito." =)))

Ivan "Doomer" Carlos
Social Engineering Specialist
Cell.: +55 (11) 8112-0666
icarlos@icarlos.net
www.icarlos.net
--------------------------------------------------


----- Original Message -----
From: "Oraculo" <oraculo@atibaia.com.br>
To: <ciencialist@yahoogrupos.com.br>
Sent: Monday, March 07, 2005 1:52 AM
Subject: [ciencialist] Cientistas recriam nervos ópticos em ratos



Cientistas recriam nervos ópticos em ratos

Cientistas dizem ter conseguido reconstruir em ratos nervos ópticos
danificados em toda a extensão do percurso desde o olho até o cérebro.
Especialistas dizem que a experiência, feita pelo Instituto Schepens de
Pesquisas do Olho, traz novas esperanças para tratar o glaucoma, uma
enfermidade na qual um nível elevado de pressão destrói o nervo óptico.

A técnica também poderia ser usada para ajudar pessoas que sofrem de
problemas na medula espinhal e em outras partes do sistema nervoso.

A experiência é detalhada na revista especializada Journal of Cell
Science.

Cicatriz

"Isto é o mais próximo que a ciência já chegou de regenerar tantas fibras
de
nervos ópticos em uma distância tão grande para chegar ao seu objetivo",
disse Dong Feng Chen, o líder da equipe de pesquisadores.

"E também de reparar um nervo que antes se achava que tinha se danificado
de
forma irreparável."

Vários tecidos celulares do corpo humano se reconstituem sozinhos em caso
de
lesão.

Mas não o nervo óptico e outros tecidos do sistema nervoso central, cujos
danos são sempre permanentes.

Os cientistas do Instituto Schepens descobriram que a incapacidade do
nervo
óptico de se regenerar estava ligada ao não funcionamento de um gene
chamado
BCL-2.

Eles também descobriram que o processo de regeneração estava bloqueado
pela
criação, logo após o nascimento, de uma cicatriz no cérebro por células
gliais especializadas.

Essas células desempenham várias funções no cérebro, entre as quais a
criação dessa espécie de cicatriz.

A cicatriz coloca uma barreira ao mesmo tempo física e molecular para a
regeneração.

Sempre ativo

Os cientistas criaram ratos nos quais o BCL-2 estava sempre ativado.

Eles descobriram que os animais conseguiam reparar os nervos ópticos com
rapidez - mas só enquanto eram jovens e a cicatriz no cérebro ainda não
havia se desenvolvido.

Em seguida eles criaram ratos com o BCL-2 funcionando e uma reduzida
capacidade de criar as cicatrizes pelas células gliais.

Desta vez, mesmo os roedores mais velhos conseguiram regenerar as células
ópticas.

O próximo passo agora será determinar se os nervos regenerados funcionam
direito.




http://www.bbc.co.uk/portuguese/ciencia/story/2005/03/050302_nervoopticoro.shtml

[As partes desta mensagem que não continham texto foram removidas]



##### ##### #####

Para saber mais visite
http://www.ciencialist.hpg.ig.com.br


##### ##### ##### #####
Links do Yahoo! Grupos











##### ##### #####

Para saber mais visite
http://www.ciencialist.hpg.ig.com.br


##### ##### ##### #####


Yahoo! Grupos, um serviço oferecido por:







------------------------------------------------------------------------------
Links do Yahoo! Grupos

a.. Para visitar o site do seu grupo na web, acesse:
http://br.groups.yahoo.com/group/ciencialist/

b.. Para sair deste grupo, envie um e-mail para:
ciencialist-unsubscribe@yahoogrupos.com.br

c.. O uso que você faz do Yahoo! Grupos está sujeito aos Termos do
Serviço do Yahoo!.



[As partes desta mensagem que não continham texto foram removidas]



##### ##### #####

Para saber mais visite
http://www.ciencialist.hpg.ig.com.br


##### ##### ##### #####
Links do Yahoo! Grupos











SUBJECT: Físico César Lattes morre aos 80 anos
FROM: "Cyberlander" <cybernews@superig.com.br>
TO: <Undisclosed-Recipient:;>
DATE: 10/03/2005 03:12

Físico César Lattes morre aos 80 anos

Agência Anhangüera

Um autêntico gênio brasileiro. Assim era a definição mais comum dada ao físico Cesar Lattes, de 80 anos. Ele faleceu às 15h40 desta terça-feira em virtude de uma parada cardiorespiratória, no Hospital das Clínicas da Universidade Estadual de Campinas (Unicamp). Nascido em Curitiba (PR), no dia 11 de julho de 1924, ele vivia no distrito de Barão Geraldo há mais de três décadas.

O físico, batizado como Cesare Mansueto Giulio Lattes era viúvo há dois anos e deixou quatro filhas. O enterro está marcado para as 16h45 desta quarta-feira, no Cemitério Parque Flamboyant. Lattes marcou seu nome na história da Ciência exatamente no dia 24 de maio de 1947, quando a prestigiada revista científica inglesa Nature anunciava que o pesquisador - filho dos imigrantes italianos Giuseppe e Carolina - havia descoberto e comprovado a existência de uma partícula decisiva na exploração do átomo: a sub-atômica Meson-Pi.

Leia entrevista de César Lattes para o Diário do Povo

Aquela que significaria o mínimo pedaço de tudo o que costumamos chamar de vida, que parecia ser algo muito distante e incompreensível para o público em geral foi considerado como um grande passo pelos cientistas, que avançaram no estudo da matéria. Por sua descoberta, Lattes poderia ter recebido o Prêmio Nobel aos 24 anos de idade, coisa que inexplicavelmente não se concretizou.

Seus estudos primários foram sendo completados em diversos pontos do Brasil e do mundo, como Curitiba, Porto Alegre, Turim (Itália) e São Paulo. Na Capital, ele optou em fazer o curso de Física na Faculdade de Filosofia, Ciências e Letras, o embrião da futura Universidade de São Paulo (USP), aonde foi um precoce professor aos 19 anos. A amizade de Lattes com cientistas como Marcelo Souza Campos reforça o seu interesse pela radiação cósmica.

O caminho para a descoberta da revolucionária Meson-Pi estava sendo criado. Depois de passar pela USP, Lattes fundamenta seus estudos na Universidade de Bristol, na Inglaterra. Sua viagem para a Europa aconteceu no primeiro cargueiro que saiu do Brasil rumo ao velho continente depois da 2ª Guerra Mundial. Foram 40 dias de uma viagem torturante, onde o físico precisava dormir no porão, apenas sobre uma tábua no chão.

Em Bristol, a imagem do desespero, tudo havia sido bombardeado, apenas o laboratório se mantinha de pé. Todo o trabalho era coordenado pelo inglês Cecil Frank Powell, ganhador de um Nobel em 1950. Um dos companheiros de estudos de Lattes era o também físico Giuseppe Occhialini, de origem italiana. As pesquisas em busca do Meson-Pi se aprofundaram depois que chapas fotográficas levadas para os montes Pirineus, na Itália, acenaram que a busca pelas partículas sub-atômicas estava no caminho certo.

Obstinado, Lattes levou o material para a Bolívia, onde se deslocou para montanhas com até 5,6 mil metros de altura para conseguir resultados ainda mais esclarecedores. Acompanhado por uma equipe, Lattes espalhou placas fotográficas especiais de chumbo em locais como Cochabamba, Oruro e La Paz, além da superfície do Lago Titicaca. Tudo com a ajuda fundamental de índios bolivianos, conhecedores dos atalhos de suas matas.

Os resultados foram os esperados, a Meson-Pi era definitivamente uma realidade no mundo científico. O fato, aguardado há tempos, ganhou repercussão internacional. Esta descoberta já havia sido "prevista" pelo japonês Hideki Yukawa na década de 30, que ganhou um Nobel em 1949, mas sem conseguir comprovar na prática suas conclusões. Isto coube ao brasileiro morador de Campinas.

Em 1948, já conhecido como o descobridor da Meson-Pi, Lattes deixa Bristol e parte rumo à Universidade de Berkeley, nos EUA, onde trabalhou com o físico norte-americano Eugene Gardner. Com final de sua bolsa, o brasileiro foi convidado para se unir aos cientistas da prestigiada Harvard, também dos EUA, mas recusou o convite. "Naquele tempo, ninguém ia para lá com a idéia de fazer carreira. A gente pensava em melhorar o Brasil", contou o físico à revista Ciência Hoje, do Rio de Janeiro, em uma ocasião.

Celebridade no mundo da Ciência, Lattes é levado para uma série de conferências por Neils Bohr, um dos pais da energia nuclear, ao redor do mundo. Sobre este personagem existe um fato curioso e misterioso que só o tempo poderá solucionar. Diz a lenda que no Museu Neils Bohr, em Copenhague, na Dinamarca, existe uma carta cujo envelope está escrito: "Por que César Lattes não ganhou o Prêmio Nobel. Abrir depois de 50 anos da minha morte". Como Bohr morreu em 1962, o mistério só será desvendado em 2012.

Apesar de não ter trazido aquele que seria o primeiro Nobel brasileiro, Lattes foi homenageado muitas vezes pela comunidade científica internacional. Entre outras condecorações que enalteceram a sua capacidade como cientista estão os prêmios da Organização dos Estados da América (OEA) e da Academia de Ciências do Terceiro Mundo. Lattes era casado com Martha Lima, com quem teve quatro filhas.

Questionado pela reportagem da revista Super Interessante em maio de 1997 sobre o que ele mudaria em sua carreira, o físico foi direto. "Não mudaria nada, fui empurrado pela história e fiz o possível", resumiu o gênio verde e amarelo chamado César Lattes.

[ ]'s

D.C.




CYBERLANDER

Ama a realidade que constróis,
que nem a morte deterá teu voo · ·




[As partes desta mensagem que não continham texto foram removidas]



SUBJECT: Re: [ciencialist] Assoprar a mão e outras
FROM: "Luiz Ferraz Netto" <leobarretos@uol.com.br>
TO: <ciencialist@yahoogrupos.com.br>
DATE: 10/03/2005 07:13

Brudna pergunta ... e responde:

# Posso?

> 2- por que as varetas das antenas de TV (principalmente na região da praia)
> estragam rapidamente?

# Mas... mas... as que conheço são feitas de alumínio e não estragam.
Bom, ando meio afastado da praia.

Léo: Sim, as varetas são feitas de alumínio (claro que o melhor seria com cobertura de prata ... mas, vá lá) e duram bastante (lenta oxidação). O problema está na montagem da antena --- e na química ---!
Para montar a antena usam de parafusos de latão ou de ferro (menos usado que latão) e esses materiais (junto com a maresia) formam uma pilha em curto circuito. O desgaste do furo, por corrosão, por onde passa o parafuso é rápido, a vareta começa a balançar, permite maior área de contato, novo desgaste, o furo aumenta --- até que a vareta caia sobre o telhado!
A solução é comprar antenas com varetas fixadas com 'pop', tb de alumínio, para evitar a formação de pilha secundária. Mas, sempre tem o fio de cobre ligando no alumínio.......

Completando a pergunta original: Por que varetas (subentende-se tubos ocos) e não hastes maciças de alumínio para fazer antenas de TV?


> 3- Se vc soprar as costas de sua mão, com a boca aberta, o ar sai quente; se
> soprar com a boca fazendo 'biquinho' o ar sai frio. Por que? Que
> eletrodoméstico usa isso?

# Han? Pega ratão? Seria uma tentativa de efeito Joule-Thomson
caseira? Não creio que essa seja a explicação mais sensata, apesar de
já ter ouvido.

Léo: Mas, é! Trata-se do trabalho realizado na expansão brusca (quando o escoamento é em forma de jato) cuja consequencia é o resfriamento do gás. A geladeira comum funciona baseado nesse princípio.


> 4- que fazem aquelas bolas (vermelhas) que são atravessadas por fios
> pertencentes às redes de alta tensão, principalmente no cruzamento com
> rodovias?

# Hehe... o Takata deve saber direitinho o número das mensagens onde
isso foi debatido na Ciencialist.

Léo: Não recordo desse debate aqui ... e isso não quer dizer nada ... minhas ligações sinápticas ... esqueci o que ia dizer ...... quem? Lattes?


> 5- à época dos mosquetes, como eram feitas as esferas de chumbo das balas?

#Cláaaassica. Deixando cair em uma coluna d'agua?

Léo: Certo e, apenas dando o devido valor às vírgulas: deixando-se cair ['gotejar'] (chumbo fundido) do alto de uma coluna,(vírgula) em uma tina com água.

Bravos Brudna --- retornando nosso moderador ONE ?

aquele abraço,





--
Internal Virus Database is out-of-date.
Checked by AVG Anti-Virus.
Version: 7.0.300 / Virus Database: 266.5.0 - Release Date: 25/02/2005



SUBJECT: imas --- correntes --- Leo pergunta
FROM: "Luiz Ferraz Netto" <leobarretos@uol.com.br>
TO: <ciencialist@yahoogrupos.com.br>
DATE: 10/03/2005 07:53

Estou escrevendo mais um trabalho para a Sala 13. Trata-se de um resumão intitulado "Interações Magnéticas" (mais para o primeiro ano universitário, onde o trivialmente visto em livros-textos de Física não é comentado --- caso da experiência de Oersted abaixo, como exemplo). As Partes 1 e 2 já estão on-line (ainda não dei a leitura final!). Na parte 3 há um detalhe que apreciaria discutir. Vou copiar o parágrafo:
=============================================
3.1 - Corrente elétrica gera campo magnético
Há séculos, acreditava-se que o magnetismo fosse fenômeno análogo à eletricidade, mas independente desta. Em 1820, Oersted, descobriu um fato que desmente esta presunção: corrente elétrica gera campo magnético. Logo, magnetismo é manifestação de cargas elétricas em movimento.
Em ímã não há corrente elétrica livre (ou verdadeira), mas existem correntes elétricas "intrínsecas" (correntes ligadas, ou de magnetização) associadas à própria estrutura da matéria. Ímãs possantes são constituídos essencialmente por ferro ou por ligas tais como "alnico" e "permalloy"; são materiais ditos ferromagnéticos. Fracamente ferromagnéticos são o cobalto, o níquel e o gadolínio.
==============================================
A questão são essas correntes intrínsecas (que já encontrei tb sob a denominação de 'correntes ligadas' ou ainda 'correntes de magnetização'). Gostaria de discorrer melhor sobre elas.
O que as diferencia, substancialmente, das correntes triviais com as quais trabalhamos (e com elas podemos 'fazer ímãs'?
Tais correntes, em domínios magnéticos de Wheistrass (lá vem esse nome complicado novamente), não são simplesmente elétrons rodopiando ao redor do núcleo!
Quando os ímãs são usados para realizar trabalho (na produção de energia elétrica, por exemplo) essas correntes intrínsecas não deveriam sofrer alguma alteração? O maldito elétron continua firme e majestoso circulando ao redor do núcleo e mantendo (às custas sei lá de quem) o formidável campo magnético? Esses elétrons tb entram na categoria dos 'privilegiados' na física quântica?
É isso .........

[]'
===========================
Luiz Ferraz Netto [Léo]
leobarretos@uol.com.br
http://www.feiradeciencias.com.br
===========================


--
Internal Virus Database is out-of-date.
Checked by AVG Anti-Virus.
Version: 7.0.300 / Virus Database: 266.5.0 - Release Date: 25/02/2005



SUBJECT: Re: Aprovacoes para C-List
FROM: "rmtakata" <rmtakata@altavista.net>
TO: ciencialist@yahoogrupos.com.br
DATE: 10/03/2005 08:25


--- Em ciencialist@yahoogrupos.com.br, "Luiz Ferraz Netto"
> 1- o que vc entende por 1 kg de algodão e 1 kg de chumbo? Temos
> mesma massa de ambos?

Normalmente isso eh medido atraves da deformacao de uma mola -
dinamometro calibrado. Como existe um fraco, mas presente empuxo do ar
e o algodao sofre mais empuxo (devido 'a sua densidade), eh preciso
uma massa maior de algodao do q. de chumbo para se ter a mesma
marcacao no dinamometro. (Tb vale para a balanca de braco.)

> 4- que fazem aquelas bolas (vermelhas) que são atravessadas por fios
> pertencentes às redes de alta tensão, principalmente no cruzamento
> com rodovias?

"- As bolas alaranjadas e vermelhas que são colocadas nos fios de alta
tensão que cruzam as rodovias, servem para sinalização visual para os
pilotos das aeronaves, quando tiverem que utilizar as estradas para
pousos de emergência."
http://www.enersul.com.br/aescelsa/pesquisa-escolar/curio.asp

Parece q. nem sempre funciona:

http://www1.folha.uol.com.br/folha/cotidiano/ult95u73606.shtml
http://www1.folha.uol.com.br/folha/cotidiano/ult95u54606.shl

> 5- à época dos mosquetes, como eram feitas as esferas de chumbo das
> balas?

No campo de batalha - onde nao se poderia montar uma coluna de agua -
tb se usavam moldes.

[]s,

Roberto Takata





SUBJECT: Re: [ciencialist] Assoprar a mão e outras
FROM: "E m i l i a n o C h e m e l l o" <chemelloe@yahoo.com.br>
TO: <ciencialist@yahoogrupos.com.br>
DATE: 10/03/2005 08:47

[Léo]
Completando a pergunta original: Por que varetas (subentende-se tubos ocos) e não hastes maciças de alumínio para fazer antenas de TV?

[Emiliano]
A função da antena é captar o sinal. Este sinal é captado pela superfície da antena, e não pela estrutura interna dela. Ou seja, fazer uma antena maciças de alumínio seria jogar dinheiro (e alumínio) fora, pois a 'eficiência' seria a mesma.


[ ] 's do Emiliano Chemello
emiliano@quimica.net
http://www.quimica.net/emiliano
http://www.ucs.br/ccet/defq/naeq
[ MSN ] chemelloe@hotmail.com
[ ICQ ] 145060604

" Rien ne se perd, rien ne se crée,
tout se transforme."

Antoine Laurent de Lavoisier (químico francês, 1743 - 1794)

----- Original Message -----
From: Luiz Ferraz Netto
To: ciencialist@yahoogrupos.com.br
Sent: Thursday, March 10, 2005 7:13 AM
Subject: Re: [ciencialist] Assoprar a mão e outras


Brudna pergunta ... e responde:

# Posso?

> 2- por que as varetas das antenas de TV (principalmente na região da praia)
> estragam rapidamente?

# Mas... mas... as que conheço são feitas de alumínio e não estragam.
Bom, ando meio afastado da praia.

Léo: Sim, as varetas são feitas de alumínio (claro que o melhor seria com cobertura de prata ... mas, vá lá) e duram bastante (lenta oxidação). O problema está na montagem da antena --- e na química ---!
Para montar a antena usam de parafusos de latão ou de ferro (menos usado que latão) e esses materiais (junto com a maresia) formam uma pilha em curto circuito. O desgaste do furo, por corrosão, por onde passa o parafuso é rápido, a vareta começa a balançar, permite maior área de contato, novo desgaste, o furo aumenta --- até que a vareta caia sobre o telhado!
A solução é comprar antenas com varetas fixadas com 'pop', tb de alumínio, para evitar a formação de pilha secundária. Mas, sempre tem o fio de cobre ligando no alumínio.......

Completando a pergunta original: Por que varetas (subentende-se tubos ocos) e não hastes maciças de alumínio para fazer antenas de TV?


> 3- Se vc soprar as costas de sua mão, com a boca aberta, o ar sai quente; se
> soprar com a boca fazendo 'biquinho' o ar sai frio. Por que? Que
> eletrodoméstico usa isso?

# Han? Pega ratão? Seria uma tentativa de efeito Joule-Thomson
caseira? Não creio que essa seja a explicação mais sensata, apesar de
já ter ouvido.

Léo: Mas, é! Trata-se do trabalho realizado na expansão brusca (quando o escoamento é em forma de jato) cuja consequencia é o resfriamento do gás. A geladeira comum funciona baseado nesse princípio.


> 4- que fazem aquelas bolas (vermelhas) que são atravessadas por fios
> pertencentes às redes de alta tensão, principalmente no cruzamento com
> rodovias?

# Hehe... o Takata deve saber direitinho o número das mensagens onde
isso foi debatido na Ciencialist.

Léo: Não recordo desse debate aqui ... e isso não quer dizer nada ... minhas ligações sinápticas ... esqueci o que ia dizer ...... quem? Lattes?


> 5- à época dos mosquetes, como eram feitas as esferas de chumbo das balas?

#Cláaaassica. Deixando cair em uma coluna d'agua?

Léo: Certo e, apenas dando o devido valor às vírgulas: deixando-se cair ['gotejar'] (chumbo fundido) do alto de uma coluna,(vírgula) em uma tina com água.

Bravos Brudna --- retornando nosso moderador ONE ?

aquele abraço,





--
Internal Virus Database is out-of-date.
Checked by AVG Anti-Virus.
Version: 7.0.300 / Virus Database: 266.5.0 - Release Date: 25/02/2005



##### ##### #####

Para saber mais visite
http://www.ciencialist.hpg.ig.com.br


##### ##### ##### #####


Yahoo! Grupos, um serviço oferecido por:







------------------------------------------------------------------------------
Links do Yahoo! Grupos

a.. Para visitar o site do seu grupo na web, acesse:
http://br.groups.yahoo.com/group/ciencialist/

b.. Para sair deste grupo, envie um e-mail para:
ciencialist-unsubscribe@yahoogrupos.com.br

c.. O uso que você faz do Yahoo! Grupos está sujeito aos Termos do Serviço do Yahoo!.



[As partes desta mensagem que não continham texto foram removidas]



SUBJECT: Re: [ciencialist] imas --- correntes --- Leo pergunta
FROM: Leonardo Souza <leo_feynman@yahoo.com.br>
TO: ciencialist@yahoogrupos.com.br
DATE: 10/03/2005 09:25

A magnetizacao tambem tem alguma coisa a ver com
spins, certo?? "Orientacao" de spins e tal...

falou

leo

--- Luiz Ferraz Netto <leobarretos@uol.com.br> wrote:
---------------------------------
Estou escrevendo mais um trabalho para a Sala 13.
Trata-se de um resumão intitulado "Interações
Magnéticas" (mais para o primeiro ano universitário,
onde o trivialmente visto em livros-textos de Física
não é comentado --- caso da experiência de Oersted
abaixo, como exemplo). As Partes 1 e 2 já estão
on-line (ainda não dei a leitura final!). Na parte 3
há um detalhe que apreciaria discutir. Vou copiar o
parágrafo:
=============================================
3.1 - Corrente elétrica gera campo magnético
Há séculos, acreditava-se que o magnetismo fosse
fenômeno análogo à eletricidade, mas independente
desta. Em 1820, Oersted, descobriu um fato que
desmente esta presunção: corrente elétrica gera campo
magnético. Logo, magnetismo é manifestação de cargas
elétricas em movimento.
Em ímã não há corrente elétrica livre (ou verdadeira),
mas existem correntes elétricas "intrínsecas"
(correntes ligadas, ou de magnetização) associadas à
própria estrutura da matéria. Ímãs possantes são
constituídos essencialmente por ferro ou por ligas
tais como "alnico" e "permalloy"; são materiais ditos
ferromagnéticos. Fracamente ferromagnéticos são o
cobalto, o níquel e o gadolínio.
==============================================
A questão são essas correntes intrínsecas (que já
encontrei tb sob a denominação de 'correntes ligadas'
ou ainda 'correntes de magnetização'). Gostaria de
discorrer melhor sobre elas.
O que as diferencia, substancialmente, das correntes
triviais com as quais trabalhamos (e com elas podemos
'fazer ímãs'?
Tais correntes, em domínios magnéticos de Wheistrass
(lá vem esse nome complicado novamente), não são
simplesmente elétrons rodopiando ao redor do núcleo!
Quando os ímãs são usados para realizar trabalho (na
produção de energia elétrica, por exemplo) essas
correntes intrínsecas não deveriam sofrer alguma
alteração? O maldito elétron continua firme e
majestoso circulando ao redor do núcleo e mantendo (às
custas sei lá de quem) o formidável campo magnético?
Esses elétrons tb entram na categoria dos
'privilegiados' na física quântica?
É isso .........

[]'
===========================
Luiz Ferraz Netto [Léo]
leobarretos@uol.com.br
http://www.feiradeciencias.com.br
===========================


--
Internal Virus Database is out-of-date.
Checked by AVG Anti-Virus.
Version: 7.0.300 / Virus Database: 266.5.0 - Release
Date: 25/02/2005



##### ##### #####

Para saber mais visite
http://www.ciencialist.hpg.ig.com.br


##### ##### ##### #####


Yahoo! Grupos, um serviço oferecido por:

S&atilde;o Paulo
Rio de Janeiro
Curitiba Porto
Alegre Belo Horizonte
Bras&iacute;lia


---------------------------------
Links do Yahoo! Grupos

Para visitar o site do seu grupo na web, acesse:
http://br.groups.yahoo.com/group/ciencialist/

Para sair deste grupo, envie um e-mail para:
ciencialist-unsubscribe@yahoogrupos.com.br

O uso que você faz do Yahoo! Grupos está sujeito
aos Termos do Serviço do Yahoo!.

__________________________________________________
Converse com seus amigos em tempo real com o Yahoo! Messenger
http://br.download.yahoo.com/messenger/


SUBJECT: Re: [ciencialist] Aprovacoes para C-List
FROM: Leonardo Souza <leo_feynman@yahoo.com.br>
TO: ciencialist@yahoogrupos.com.br
DATE: 10/03/2005 09:29

Sou Leonardo, mestrando em fisica.

falou

leo


--- Marcelo Cortimiglia <cortimiglia@gmail.com> wrote:
---------------------------------
Olá a todos,

meu nome é Marcelo Cortimiglia, e cheguei aqui através
da SBCR.

Sou engenheiro civil, com mestrado em engenharia de
produção pela
UFRGS. Atualmente estou preparando um projeto de
doutorado. Meu
principal interesse de pesquisa reside na área de
Sistemas de
Informação, particularmente o uso de TI para gestão do
conhecimento e
desdobramentos no campo da aprendizagem
organizacional.

Abraços,

Marcelo Cortimiglia


On Tue, 8 Mar 2005 07:15:42 -0300 (ART), Lÿfffffacia
Valois Leite
<lvaloisleite@yahoo.com.br> wrote:
> Oi Luiz,
> Meu nome é Lúcia Valois Leite, sou formada em
Biologia e atuo na área de
> Educação.Ensino duas disciplinas na FACED-UFBA :
Ensino de Ciências
> (EDC-270) e Projetos (EDC-200), no curso de
Ciências Naturais.
>
> Venho participando do PRONERA, desde 2004, como
coordenadora pedagógica,
> projeto que tem por objetivo capacitar professores
de alfabetização e de 1ª
> a 4ª séries (ensino fundamental I) no MST e da
Escolarização de Jovens e
> Adultos de 5ª a 8ª série (ensino fundamental II)
também no MST, no Recôncavo
> baiano e na Chapada Diamantina. Desenvolvo um
trabalho de formação de
> professores em serviço (de Ciências Naturais), da
rede municipal de ensino
> do município de Alagoinhas-BA e atuo como professora
de Ciências Naturais da
> rede particular de Ensino , no Colégio Joan Miró,
com crianças de 5ª e 6ª
> séries do ensino fundamental, onde desenvolvemos
projetos de pesquisa sobre:
> biotecnologia, energia nuclear, tecnologias para o
estudo do Universo,
> biopirataria.
>
> Por conta dessas atividades educativas que sinto a
necessidade de estar
> participando, mesmo como observadora(inicialmente),
dessa lista, ok?
> Se desejar mais alguma informação é só solicitar.
> Grande abraço,
> Lúcia.
>


##### ##### #####

Para saber mais visite
http://www.ciencialist.hpg.ig.com.br


##### ##### ##### #####


Yahoo! Grupos, um serviço oferecido por:

function SearchComboBox() {
if (document.form_combo.keyword.value.length==0){
alert("Por favor, digite algo."); return false; }else
{ document.form_combo.action
="http://br.rd.yahoo.com/SIG=12a5412r6/M=264105.3931087.6562589.1588051/D=brclubs/S=2137111528:HM/EXP=1110484002/A=2361264/R=0/SIG=11uaou2jn/*http://www.bondfaro.com/bondfaro/in/combosearch_in.jsp?sk=11";
} return true;}
[input]
[input] [input]


---------------------------------
Links do Yahoo! Grupos

Para visitar o site do seu grupo na web, acesse:
http://br.groups.yahoo.com/group/ciencialist/

Para sair deste grupo, envie um e-mail para:
ciencialist-unsubscribe@yahoogrupos.com.br

O uso que você faz do Yahoo! Grupos está sujeito
aos Termos do Serviço do Yahoo!.

__________________________________________________
Converse com seus amigos em tempo real com o Yahoo! Messenger
http://br.download.yahoo.com/messenger/


SUBJECT: Re: Aprovacoes para C-List
FROM: Hélio Ricardo Carvalho <hrc@fis.puc-rio.br>
TO: ciencialist@yahoogrupos.com.br
DATE: 10/03/2005 09:33


Será que um veterano também pode perguntar??

Em ciencialist Leo escreveu
>
> Apresentem-se e entrem na rodada, se o problema
> é falta de pergunta aqui vão algumas:
>
> ...
> ...
> ...
>
> 4- que fazem aquelas bolas (vermelhas) que são
> atravessadas por fios pertencentes às redes de
> alta tensão, principalmente no cruzamento com rodovias?
>


Engraçado, um dia (muito tempo atrás) uma pessoa, sabendo que eu era
físico, me perguntou isto e não consegui responde-la com certeza.
Alguns anos depois um amigo meu que é da aeronáutica e estava
aprendendo pilotagem de helicóptero me disse que era simplesmente
para que, se algum piloto quiser usar a estrada como pista, saber
que ali tem alta tensão.
É isto mesmo?

Hélio






SUBJECT: Re: [ciencialist] Fw: <grandes mulheres cientistas>
FROM: Leonardo Souza <leo_feynman@yahoo.com.br>
TO: ciencialist@yahoogrupos.com.br
DATE: 10/03/2005 09:33

Uma das grandes foi a senhorita Noether, do teorrema
de Noether (sobre simetrias e leis de conservacao).
Nao sei detalhes, mas o google sabe! hehe

Falou

Leo


--- Luiz Ferraz Netto <leobarretos@uol.com.br> wrote:
---------------------------------
Oi Roberto,

vc não postou algo sobre ESSAS mulheres há pouco
tempo? Onde está?

[]'
===========================
Luiz Ferraz Netto [Léo]
leobarretos@uol.com.br
http://www.feiradeciencias.com.br
===========================
-----Mensagem Original-----
De: Silvana
Para: leobarretos@uol.com.br
Enviada em: terça-feira, 8 de março de 2005 12:42
Assunto: <grandes mulheres cientistas>


presado senhor LEOBARRETOS venho por meio deste e-mail
lhe pedir um esclarecimento
sobre: grandes mulheres cientistas que marcaram a
história da ciencia , suas funções e o ano em que elas
atuaram.
desde já fico agradecida por sua ajuda.

silvana.


--------------------------------------------------------------------------------




--------------------------------------------------------------------------------


Internal Virus Database is out-of-date.
Checked by AVG Anti-Virus.
Version: 7.0.300 / Virus Database: 266.5.0 - Release
Date: 25/02/2005

----------

Internal Virus Database is out-of-date.
Checked by AVG Anti-Virus.
Version: 7.0.300 / Virus Database: 266.5.0 - Release
Date: 25/02/2005


[As partes desta mensagem que não continham texto
foram removidas]



##### ##### #####

Para saber mais visite
http://www.ciencialist.hpg.ig.com.br


##### ##### ##### #####


Yahoo! Grupos, um serviço oferecido por:

S&atilde;o Paulo
Rio de Janeiro
Curitiba Porto
Alegre Belo Horizonte
Bras&iacute;lia


---------------------------------
Links do Yahoo! Grupos

Para visitar o site do seu grupo na web, acesse:
http://br.groups.yahoo.com/group/ciencialist/

Para sair deste grupo, envie um e-mail para:
ciencialist-unsubscribe@yahoogrupos.com.br

O uso que você faz do Yahoo! Grupos está sujeito
aos Termos do Serviço do Yahoo!.





_______________________________________________________
Yahoo! Acesso Grátis - Instale o discador do Yahoo! agora. http://br.acesso.yahoo.com/ - Internet rápida e grátis


SUBJECT: Re: Aprovacoes para C-List
FROM: Hélio Ricardo Carvalho <hrc@fis.puc-rio.br>
TO: ciencialist@yahoogrupos.com.br
DATE: 10/03/2005 09:49


Não tinha visto que o Takata já tinha respondido.

Valeu Takata.

Hélio



--- Em ciencialist@yahoogrupos.com.br, Hélio Ricardo Carvalho
<hrc@f...> escreveu
>
> Será que um veterano também pode perguntar??
>
> Em ciencialist Leo escreveu
> >
> > Apresentem-se e entrem na rodada, se o problema
> > é falta de pergunta aqui vão algumas:
> >
> > ...
> > ...
> > ...
> >
> > 4- que fazem aquelas bolas (vermelhas) que são
> > atravessadas por fios pertencentes às redes de
> > alta tensão, principalmente no cruzamento com rodovias?
> >
>
>
> Engraçado, um dia (muito tempo atrás) uma pessoa, sabendo que eu
era
> físico, me perguntou isto e não consegui responde-la com certeza.
> Alguns anos depois um amigo meu que é da aeronáutica e estava
> aprendendo pilotagem de helicóptero me disse que era simplesmente
> para que, se algum piloto quiser usar a estrada como pista, saber
> que ali tem alta tensão.
> É isto mesmo?
>
> Hélio





SUBJECT: Re: [ciencialist] Fw: <grandes mulheres cientistas>
FROM: "E m i l i a n o C h e m e l l o" <chemelloe@yahoo.com.br>
TO: <ciencialist@yahoogrupos.com.br>
DATE: 10/03/2005 09:52

Falando nisso...

Marie Curie e a Radioatividade
http://www.ucs.br/ccet/defq/naeq/material_didatico/e-museu_quimica_01.htm

[ ] 's do Emiliano Chemello
emiliano@quimica.net
http://www.quimica.net/emiliano
http://www.ucs.br/ccet/defq/naeq
[ MSN ] chemelloe@hotmail.com
[ ICQ ] 145060604

" Rien ne se perd, rien ne se crée,
tout se transforme."

Antoine Laurent de Lavoisier (químico francês, 1743 - 1794)

----- Original Message -----
From: Leonardo Souza
To: ciencialist@yahoogrupos.com.br
Sent: Thursday, March 10, 2005 9:33 AM
Subject: Re: [ciencialist] Fw: <grandes mulheres cientistas>


Uma das grandes foi a senhorita Noether, do teorrema
de Noether (sobre simetrias e leis de conservacao).
Nao sei detalhes, mas o google sabe! hehe

Falou

Leo


--- Luiz Ferraz Netto <leobarretos@uol.com.br> wrote:
---------------------------------
Oi Roberto,

vc não postou algo sobre ESSAS mulheres há pouco
tempo? Onde está?

[]'
===========================
Luiz Ferraz Netto [Léo]
leobarretos@uol.com.br
http://www.feiradeciencias.com.br
===========================
-----Mensagem Original-----
De: Silvana
Para: leobarretos@uol.com.br
Enviada em: terça-feira, 8 de março de 2005 12:42
Assunto: <grandes mulheres cientistas>


presado senhor LEOBARRETOS venho por meio deste e-mail
lhe pedir um esclarecimento
sobre: grandes mulheres cientistas que marcaram a
história da ciencia , suas funções e o ano em que elas
atuaram.
desde já fico agradecida por sua ajuda.

silvana.


--------------------------------------------------------------------------------




--------------------------------------------------------------------------------


Internal Virus Database is out-of-date.
Checked by AVG Anti-Virus.
Version: 7.0.300 / Virus Database: 266.5.0 - Release
Date: 25/02/2005

----------

Internal Virus Database is out-of-date.
Checked by AVG Anti-Virus.
Version: 7.0.300 / Virus Database: 266.5.0 - Release
Date: 25/02/2005


[As partes desta mensagem que não continham texto
foram removidas]



##### ##### #####

Para saber mais visite
http://www.ciencialist.hpg.ig.com.br


##### ##### ##### #####


Yahoo! Grupos, um serviço oferecido por:

S&atilde;o Paulo
Rio de Janeiro
Curitiba Porto
Alegre Belo Horizonte
Bras&iacute;lia


---------------------------------
Links do Yahoo! Grupos

Para visitar o site do seu grupo na web, acesse:
http://br.groups.yahoo.com/group/ciencialist/

Para sair deste grupo, envie um e-mail para:
ciencialist-unsubscribe@yahoogrupos.com.br

O uso que você faz do Yahoo! Grupos está sujeito
aos Termos do Serviço do Yahoo!.





_______________________________________________________
Yahoo! Acesso Grátis - Instale o discador do Yahoo! agora. http://br.acesso.yahoo.com/ - Internet rápida e grátis


##### ##### #####

Para saber mais visite
http://www.ciencialist.hpg.ig.com.br


##### ##### ##### #####


Yahoo! Grupos, um serviço oferecido por:







------------------------------------------------------------------------------
Links do Yahoo! Grupos

a.. Para visitar o site do seu grupo na web, acesse:
http://br.groups.yahoo.com/group/ciencialist/

b.. Para sair deste grupo, envie um e-mail para:
ciencialist-unsubscribe@yahoogrupos.com.br

c.. O uso que você faz do Yahoo! Grupos está sujeito aos Termos do Serviço do Yahoo!.



[As partes desta mensagem que não continham texto foram removidas]



SUBJECT: Re: imas --- correntes --- Leo pergunta
FROM: "rmtakata" <rmtakata@altavista.net>
TO: ciencialist@yahoogrupos.com.br
DATE: 10/03/2005 10:06


--- Em ciencialist@yahoogrupos.com.br, Leonardo Souza
> A magnetizacao tambem tem alguma coisa a ver com
> spins, certo??

Sim:

http://hyperphysics.phy-astr.gsu.edu/hbase/spin.html

> --- Luiz Ferraz Netto <leobarretos@u...> wrote:
> Quando os ímãs são usados para realizar trabalho (na
> produção de energia elétrica, por exemplo) essas
> correntes intrínsecas não deveriam sofrer alguma
> alteração?

Nao eh o alinhamento magnetico dos atomos q. sofre as consequencias?

> O maldito elétron continua firme e majestoso circulando ao redor do
> núcleo e mantendo (às custas sei lá de quem) o formidável campo
> magnético?

Acho q. ele continua a 'girar' ao redor do nucleo (embora a imagem
atual seja de uma nuvem de probabilidade), mas o alinhamento dos
(campos magneticos dos) atomos pode mudar.

> Esses elétrons tb entram na categoria dos 'privilegiados' na física
> quântica?

Imagino q. sim.

[]s,

Roberto Takata





SUBJECT: Re: Os desastres de Sofia
FROM: César A. K. Grossmann <cesarakg@bol.com.br>
TO: ciencialist@yahoogrupos.com.br
DATE: 10/03/2005 11:10


--- Em ciencialist@yahoogrupos.com.br, Maria Natália <grasdic@h...>
escreveu
>
> A primeira aula dele há 5 anos foi: "A Mais velha Profissão do Mundo
> está bem paga?"

Tem uma transcrição da aula na internet? Fiquei curioso. Aliás,
curiosíssimo...

> A convite de João Caraça, Director de Ciência da FCG,
> Dinis Pestana preparou um programa de televisão (O Acaso não Acontece
> por Acaso) sobre Probabilidade.

Para com isto! Vou me consumir de curiosidade! Isto não é justo,
provocar o cidadão e não dar condições para o mesmo saciar sua
curiosidade. Por falar nisto, acho que vou ler novamente o "Innumeracy
: Mathematical Illiteracy and Its Social Consequences"
(http://www.amazon.com/exec/obidos/ISBN%3D0679726012/102-4046700-0752911),
tá dando uma saudade...

[]s
--
César A. K. Grossmann





SUBJECT: Re: [ciencialist] Re: imas --- correntes --- Leo pergunta
FROM: "Alvaro Augusto \(E\)" <alvaro@electraenergy.com.br>
TO: <ciencialist@yahoogrupos.com.br>
DATE: 10/03/2005 11:16

A magnetização de elementos ferromagnéticos (ferro, níquel, cobalto, gadolínio, disprósio e espuma de carbono) é um fenônemo complicado. O spin orbital tem pouca influência, pois é um efeito paramagnético, que todas as substâncias apresentam, e não é permanente, pois quando o campo magnetizante é removido, a agitação térmica destróis os alinhamentos orbitais. Já os ferromagnéticos se magnetizam permanentemente devido a uma conjunção de três fatores:

- alinhamento de spins eletrônicos da penúltima camada, possibilitado pela interação entre átomos vizinhos;
- blindagem magnética da camada de valência;
- formação de domínios magnéticos.

A blindagem só funciona para elementos com uma certa distância interatômica. Se a distância for muito grande, como no caso do alumínio, a interação entre elétrons de átomos vizinhos é muito fraca, e o alinhamento não será mantido, por causa da fraca energia de troca. Se a distância for muito pequena, como no caso do chumbo, a interação entre elétrons de átomos vizinhos será muito forte, e o alinhamento será destruído.

A rigor, somente o ferro é ferromagnético, pois somente o ferro é magnético da forma que o ferro é magnético, apresentando seis elétrons na penúltima camada de valência, estrutura cristalina cúbica e outras propriedade. O cobalto, por exemplo, apresenta estrutura hexagonal. O níquel também apresenta estrutura cúbica, mas é mais facilmente magnetizado ao longo das diagonais das faces dos cristais, e não ao longo das aretas das faces, como no caso do ferro. Assim, devido à formação de cristais, o ferromagnetismo é uma propriedade do elemento, não do átomo em si.

[ ]s

Alvaro Augusto

----- Original Message -----
From: rmtakata
To: ciencialist@yahoogrupos.com.br
Sent: Thursday, March 10, 2005 10:06 AM
Subject: [ciencialist] Re: imas --- correntes --- Leo pergunta



--- Em ciencialist@yahoogrupos.com.br, Leonardo Souza
> A magnetizacao tambem tem alguma coisa a ver com
> spins, certo??

Sim:

http://hyperphysics.phy-astr.gsu.edu/hbase/spin.html

> --- Luiz Ferraz Netto <leobarretos@u...> wrote:
> Quando os ímãs são usados para realizar trabalho (na
> produção de energia elétrica, por exemplo) essas
> correntes intrínsecas não deveriam sofrer alguma
> alteração?

Nao eh o alinhamento magnetico dos atomos q. sofre as consequencias?

> O maldito elétron continua firme e majestoso circulando ao redor do
> núcleo e mantendo (às custas sei lá de quem) o formidável campo
> magnético?

Acho q. ele continua a 'girar' ao redor do nucleo (embora a imagem
atual seja de uma nuvem de probabilidade), mas o alinhamento dos
(campos magneticos dos) atomos pode mudar.

> Esses elétrons tb entram na categoria dos 'privilegiados' na física
> quântica?

Imagino q. sim.

[]s,

Roberto Takata





##### ##### #####

Para saber mais visite
http://www.ciencialist.hpg.ig.com.br


##### ##### ##### #####


Yahoo! Grupos, um serviço oferecido por:

São Paulo Rio de Janeiro Curitiba Porto Alegre Belo Horizonte Brasília




------------------------------------------------------------------------------
Links do Yahoo! Grupos

a.. Para visitar o site do seu grupo na web, acesse:
http://br.groups.yahoo.com/group/ciencialist/

b.. Para sair deste grupo, envie um e-mail para:
ciencialist-unsubscribe@yahoogrupos.com.br

c.. O uso que você faz do Yahoo! Grupos está sujeito aos Termos do Serviço do Yahoo!.



[As partes desta mensagem que não continham texto foram removidas]



SUBJECT: Re: Assoprar a mão e outras
FROM: César A. K. Grossmann <cesarakg@bol.com.br>
TO: ciencialist@yahoogrupos.com.br
DATE: 10/03/2005 11:17


--- Em ciencialist@yahoogrupos.com.br, Luis Brudna <luisbrudna@g...>
escreveu
>
> > 5- à época dos mosquetes, como eram feitas as esferas de chumbo
das balas?
>
> Cláaaassica. Deixando cair em uma coluna d'agua?

No filme "O Patriota", o personagem do Mel Gibson tem uma espécie de
alicate. Pouco antes das batalhas, ele senta ao lado de uma fogueira,
e derrete alguns soldadinhos de chumbo de seu filho assassinado, para
fazer balas para matar ingleses (justiça poética?).

Agora, será que esta parte do filme é autêntica? O alicate seria uma
forma com um pequeno furo por onde o chumbo fluía, depois era
esfriado, abria-se o alicate e ali estava a bala em formato esférico,
pronta a ser usada...

[]s
--
César A. K. Grossmann





SUBJECT: Re: [ciencialist] Re: Aprovacoes para C-List
FROM: "Luiz Ferraz Netto" <leobarretos@uol.com.br>
TO: <ciencialist@yahoogrupos.com.br>
DATE: 10/03/2005 12:46

É Roberto,

como se percebe

os três textos (três links) cometem o mesmo erro conceitual: existência de um "fio de alta tensão". Não têm a mínima idéia de d.d.p. ou tensão e potencial elétrico.

link 1- ... que são colocadas nos fios de alta tensão que cruzam as rodovias, ...

link 2- ... O avião, que partiu de Ubatuba (litoral norte) e se dirigia para São Paulo, voava em baixa altitude quando bateu em um fio de alta tensão e caiu no mar, segundo o Corpo de Bombeiros.

link 3- ... De acordo com testemunhas, o monomotor estava fazendo vôos rasantes sobre uma avenida quando caiu. Os moradores afirmaram, na oportunidade, que o avião começou a perder altitude e atingiu um fio de alta-tensão.

OK, sobre a resposta do algodão/chumbo. Só não 'gostei' do "fraco", no trecho:
".....Como existe um fraco, mas presente ....". Fraco?! Zzeeppellinn?!

[]'
===========================
Luiz Ferraz Netto [Léo]
leobarretos@uol.com.br
http://www.feiradeciencias.com.br
===========================
-----Mensagem Original-----
De: "rmtakata" <rmtakata@altavista.net>
Para: <ciencialist@yahoogrupos.com.br>
Enviada em: quinta-feira, 10 de março de 2005 08:25
Assunto: [ciencialist] Re: Aprovacoes para C-List




--- Em ciencialist@yahoogrupos.com.br, "Luiz Ferraz Netto"
> 1- o que vc entende por 1 kg de algodão e 1 kg de chumbo? Temos
> mesma massa de ambos?

Normalmente isso eh medido atraves da deformacao de uma mola -
dinamometro calibrado. Como existe um fraco, mas presente empuxo do ar
e o algodao sofre mais empuxo (devido 'a sua densidade), eh preciso
uma massa maior de algodao do q. de chumbo para se ter a mesma
marcacao no dinamometro. (Tb vale para a balanca de braco.)

> 4- que fazem aquelas bolas (vermelhas) que são atravessadas por fios
> pertencentes às redes de alta tensão, principalmente no cruzamento
> com rodovias?

"- As bolas alaranjadas e vermelhas que são colocadas nos fios de alta
tensão que cruzam as rodovias, servem para sinalização visual para os
pilotos das aeronaves, quando tiverem que utilizar as estradas para
pousos de emergência."
http://www.enersul.com.br/aescelsa/pesquisa-escolar/curio.asp

Parece q. nem sempre funciona:

http://www1.folha.uol.com.br/folha/cotidiano/ult95u73606.shtml
http://www1.folha.uol.com.br/folha/cotidiano/ult95u54606.shl

> 5- à época dos mosquetes, como eram feitas as esferas de chumbo das
> balas?

No campo de batalha - onde nao se poderia montar uma coluna de agua -
tb se usavam moldes.

[]s,

Roberto Takata



--
Internal Virus Database is out-of-date.
Checked by AVG Anti-Virus.
Version: 7.0.300 / Virus Database: 266.5.0 - Release Date: 25/02/2005



SUBJECT: Re: [ciencialist] Fw: <grandes mulheres cientistas>
FROM: "Luiz Ferraz Netto" <leobarretos@uol.com.br>
TO: <ciencialist@yahoogrupos.com.br>
DATE: 10/03/2005 12:52

#Uma das grandes foi a senhorita Noether, do teorrema
de Noether (sobre simetrias e leis de conservacao).
Nao sei detalhes, mas o google sabe! hehe

Falou

Leo #

Quem está usando o meu santo nome em vão?
Tá certo que Léo tem acento (eis porque não fico de pé o dia inteiro), e que 'em vão' pode significar 'em freta', 'em fenda', ...... mas .......

Léo - the first primeiro



--
Internal Virus Database is out-of-date.
Checked by AVG Anti-Virus.
Version: 7.0.300 / Virus Database: 266.5.0 - Release Date: 25/02/2005



SUBJECT: Insulina e Carambola
FROM: "Alvaro Augusto \(E\)" <alvaro@electraenergy.com.br>
TO: <ciencialist@yahoogrupos.com.br>
DATE: 10/03/2005 12:56

Alguém sabe se a informação procede? Substituição total da insulina para
diabéticos?

[ ]s

Alvaro Augusto de Almeida




INSULINA x CARAMBOLA





Você que é diabético, já pensou em aposentar a INSULINA ?



A E & G está comercializando um produto inédito composto do extrato das
folhas da Carambola, que é utilizada como coadjuvante no controle de
diabetes, colesterol e triglicerideos, chamado GLICOVITAE



Este produto se finaliza a substituir definitivamente a utilização da
Insulina, bem como é Hipoglicemiante Oral, propiciando ao organismo fontes
de energia, abatendo o cansaço, mal estar, fadiga e depressão.



O produto apresenta-se em forma líquida (50ml) a fim de ser ingerido em
doses de 35 gotas no café da manhã e refeições.



Não há contra-indicações, nem reações adversas quando utilizado na dose
recomendada.



Informações adicionais podem ser obtidas através de nosso email:
egcom@egcom.com.br, ou se preferir: 11 4614.0823 ou 4614.0824.



Vale a pena conferir !





SUBJECT: Re: [ciencialist] Re: imas --- correntes --- Leo pergunta
FROM: "Luiz Ferraz Netto" <leobarretos@uol.com.br>
TO: <ciencialist@yahoogrupos.com.br>
DATE: 10/03/2005 13:04

Gostei disso do Álvaro ...... vamos aprofundar a idéia?
Essa blindagem oferecida pela última camada ficou um tanto 'esquisito'; se a parte interna da blindagem é 'norte', a externa é 'sul'?
Como pode uma zona de probabilidade (em substituição ao elétron girando ao redor do núcleo) gerar um campo magnético? Se não há elétron efetivamente girando ao redor do núcleo, o que vem a ser elétron girando ao redor de si mesmo ... os spins emparelhados e não emparelhados?
Quem participa na formação dos domínios magnéticos? alguns átomos especiais? alguma estrutura cristalina?

e vamos em frente .........

[]'
===========================
Luiz Ferraz Netto [Léo]
leobarretos@uol.com.br
http://www.feiradeciencias.com.br
===========================
-----Mensagem Original-----
De: "Alvaro Augusto (E)" <alvaro@electraenergy.com.br>
Para: <ciencialist@yahoogrupos.com.br>
Enviada em: quinta-feira, 10 de março de 2005 11:16
Assunto: Re: [ciencialist] Re: imas --- correntes --- Leo pergunta



A magnetização de elementos ferromagnéticos (ferro, níquel, cobalto, gadolínio, disprósio e espuma de carbono) é um fenônemo complicado. O spin orbital tem pouca influência, pois é um efeito paramagnético, que todas as substâncias apresentam, e não é permanente, pois quando o campo magnetizante é removido, a agitação térmica destróis os alinhamentos orbitais. Já os ferromagnéticos se magnetizam permanentemente devido a uma conjunção de três fatores:

- alinhamento de spins eletrônicos da penúltima camada, possibilitado pela interação entre átomos vizinhos;
- blindagem magnética da camada de valência;
- formação de domínios magnéticos.

A blindagem só funciona para elementos com uma certa distância interatômica. Se a distância for muito grande, como no caso do alumínio, a interação entre elétrons de átomos vizinhos é muito fraca, e o alinhamento não será mantido, por causa da fraca energia de troca. Se a distância for muito pequena, como no caso do chumbo, a interação entre elétrons de átomos vizinhos será muito forte, e o alinhamento será destruído.

A rigor, somente o ferro é ferromagnético, pois somente o ferro é magnético da forma que o ferro é magnético, apresentando seis elétrons na penúltima camada de valência, estrutura cristalina cúbica e outras propriedade. O cobalto, por exemplo, apresenta estrutura hexagonal. O níquel também apresenta estrutura cúbica, mas é mais facilmente magnetizado ao longo das diagonais das faces dos cristais, e não ao longo das aretas das faces, como no caso do ferro. Assim, devido à formação de cristais, o ferromagnetismo é uma propriedade do elemento, não do átomo em si.

[ ]s

Alvaro Augusto

----- Original Message -----
From: rmtakata
To: ciencialist@yahoogrupos.com.br
Sent: Thursday, March 10, 2005 10:06 AM
Subject: [ciencialist] Re: imas --- correntes --- Leo pergunta



--- Em ciencialist@yahoogrupos.com.br, Leonardo Souza
> A magnetizacao tambem tem alguma coisa a ver com
> spins, certo??

Sim:

http://hyperphysics.phy-astr.gsu.edu/hbase/spin.html

> --- Luiz Ferraz Netto <leobarretos@u...> wrote:
> Quando os ímãs são usados para realizar trabalho (na
> produção de energia elétrica, por exemplo) essas
> correntes intrínsecas não deveriam sofrer alguma
> alteração?

Nao eh o alinhamento magnetico dos atomos q. sofre as consequencias?

> O maldito elétron continua firme e majestoso circulando ao redor do
> núcleo e mantendo (às custas sei lá de quem) o formidável campo
> magnético?

Acho q. ele continua a 'girar' ao redor do nucleo (embora a imagem
atual seja de uma nuvem de probabilidade), mas o alinhamento dos
(campos magneticos dos) atomos pode mudar.

> Esses elétrons tb entram na categoria dos 'privilegiados' na física
> quântica?

Imagino q. sim.

[]s,

Roberto Takata





##### ##### #####

Para saber mais visite
http://www.ciencialist.hpg.ig.com.br


##### ##### ##### #####


Yahoo! Grupos, um serviço oferecido por:

São Paulo Rio de Janeiro Curitiba Porto Alegre Belo Horizonte Brasília




------------------------------------------------------------------------------
Links do Yahoo! Grupos

a.. Para visitar o site do seu grupo na web, acesse:
http://br.groups.yahoo.com/group/ciencialist/

b.. Para sair deste grupo, envie um e-mail para:
ciencialist-unsubscribe@yahoogrupos.com.br

c.. O uso que você faz do Yahoo! Grupos está sujeito aos Termos do Serviço do Yahoo!.



[As partes desta mensagem que não continham texto foram removidas]



##### ##### #####

Para saber mais visite
http://www.ciencialist.hpg.ig.com.br


##### ##### ##### #####
Links do Yahoo! Grupos










--
Internal Virus Database is out-of-date.
Checked by AVG Anti-Virus.
Version: 7.0.300 / Virus Database: 266.5.0 - Release Date: 25/02/2005




--
Internal Virus Database is out-of-date.
Checked by AVG Anti-Virus.
Version: 7.0.300 / Virus Database: 266.5.0 - Release Date: 25/02/2005



SUBJECT: Re: Aprovacoes para C-List
FROM: "rmtakata" <rmtakata@altavista.net>
TO: ciencialist@yahoogrupos.com.br
DATE: 10/03/2005 13:31


--- Em ciencialist@yahoogrupos.com.br, "Luiz Ferraz Netto"
> ".....Como existe um fraco, mas presente ....". Fraco?!
> Zzeeppellinn?!

No caso do Zepelin (e dos aerostatos em geral) é forte - relativamente
ao peso do Zepelin e tb relativamente 'as forcas aplicadas pelo ser
humano cotidianamente.

No caso de 1 kg de algodao eh fraco - relativamente (o algodao nao
flutua) e tb 'as forcas aplicadas pelo ser humano (a diferenca deve
ser minuscula entre a massa real e a aferida no ar).

[]s,

Roberto Takata





SUBJECT: ano da física
FROM: TARCISIO BORGES <tbs97@fisica.ufpr.br>
TO: ciencialist@yahoogrupos.com.br
CC: Rudolf Lessmann <rudolf.lessmann@gmx.net>
DATE: 10/03/2005 13:39

Quem está fazendo sucesso no ano da física é a dona morte.

Depois de César Lattes e Hans Bethe eis que hoje morreu o físico Gilberto
Odo coordenador do curso de física da UFPR.

[]s
TARCISIO BORGES
tbs97@fisica.ufpr.br



SUBJECT: infecção Bacteriana Rumo ao Brasil
FROM: César A. K. Grossmann <cesarakg@bol.com.br>
TO: ciencialist@yahoogrupos.com.br
DATE: 10/03/2005 13:54


http://www2.uol.com.br/sciam/conteudo/noticia/noticia_103.html

Infecção bacteriana rumo ao Brasil
Março 2005

Endêmica dos Andes, bartonelose se alastra com abertura de estradas e
degradação ambiental.


Uma doença praticamente desconhecida dos profissionais de saúde
brasileiros está se aproximando de nossa fronteira, e seu impacto pode
ser agravado pelo modelo de desenvolvimento adotado na Amazônia. A
infecção em questão é a bartonelose, originária dos Andes, causada
pela bactéria Bartonella bacilliformis e transmitida pelos mesmos
mosquitos vetores da leishmaniose. Comum nos vales interandinos
peruanos, na última década a bactéria atingiu áreas mais extensas do
Peru, incluindo altitudes mais baixas. Em 2003, chegou ao Departamento
de Loreto (fronteira com o estado do Amazonas) e, em 2004, infectou 19
pessoas em Madre de Diós, que faz fronteira com o Acre e a Bolívia. O
receio é que, caso a bartonelose entre na Amazônia brasileira, sua
disseminação seja acelerada pela falta de treinamento específico dos
profissionais de saúde do país.

(continua na página...)

-o=O=o-

E agora, quem poderá nos defender?

[]s
--
César A. K. Grossmann





SUBJECT: Re: imas --- correntes --- Leo pergunta
FROM: "rmtakata" <rmtakata@altavista.net>
TO: ciencialist@yahoogrupos.com.br
DATE: 10/03/2005 14:01


--- Em ciencialist@yahoogrupos.com.br, "Luiz Ferraz Netto"
> Como pode uma zona de probabilidade (em substituição ao elétron
> girando ao redor do núcleo) gerar um campo magnético?

Chutando (e tb chutando o pau da barraca), talvez de um modo nao tao
diferente de uma carga eletrica em movimento. Mas nesse caso, o q.
iria se alterar no espaco seria a distribuicao da densidade de
probabilidade.

> Se não há elétron efetivamente girando ao redor do núcleo, o que vem
> a ser elétron girando ao redor de si mesmo

Essa eh complicada, bem cabeluda. Entre outras coisas nao existe um
analogo classico pleno para o spin quantico do eletron.

[]s,

Roberto Takata





SUBJECT: Re: imas --- correntes --- Leo pergunta
FROM: "rmtakata" <rmtakata@altavista.net>
TO: ciencialist@yahoogrupos.com.br
DATE: 10/03/2005 14:16


--- Em ciencialist@yahoogrupos.com.br, "rmtakata" <rmtakata@a...>
> Mas nesse caso, o q. iria se alterar no espaco seria a distribuicao da
> densidade de probabilidade.

Claro, isso dentro da interpretacao da escola de Copenhagen. Existem
outras interpretacoes alternativas possiveis dentro da quantica.

Há uma proposicao q. se assemelha - bem de longe - 'a proposta de
Mesquita: ao passar pela questao da informacao, um tal de Anton
Zeilinger da Universidade de Viena. (Mas nao sei se ele chegou a
desenvolver suas ideias.)

[]s,

Roberto Takata





SUBJECT: Memória da água é curta, afirma estudo
FROM: Manuel Bulcão <manuelbulcao@uol.com.br>
TO: ciencialist@yahoogrupos.com.br
DATE: 10/03/2005 15:55


Uma pesquisa recente indica que a homeopatia pode até funcionar, mas
que qualquer remédio homeopático já está com a sua validade vencida
50 femtosegundos após a sua fabricação (1 femtosegundo = 1
bilionésimo de milionésimo de segundo):

"Memória" da água é curta, afirma estudo
SALVADOR NOGUEIRA
da Folha de S.Paulo

Uma boa e uma má notícia para os entusiastas da homeopatia.
Cientistas acabam de demonstrar que a água de fato tem memória --a
capacidade de armazenar de algum modo propriedades de substâncias que
já estiveram diluídas nela, mas não estão mais lá. Os pesquisadores
também constataram que a "lembrança" dura no máximo 50 femtosegundos.

http://www1.folha.uol.com.br/folha/ciencia/ult306u13087.shtml

[]s
Manuel Bulcão






SUBJECT: Re: [ciencialist] Fw: <grandes mulheres cientistas> OFF - leo the first
FROM: Leonardo Souza <leo_feynman@yahoo.com.br>
TO: ciencialist@yahoogrupos.com.br
DATE: 10/03/2005 16:14

hehhe

Nao uso seu santo nome em vao, uso Leo... e o linux
nao ta aceitando acento (eu tinha que escrever
leho???) hehehe

falou

Leo
--- Luiz Ferraz Netto <leobarretos@uol.com.br> wrote:
---------------------------------
#Uma das grandes foi a senhorita Noether, do teorrema
de Noether (sobre simetrias e leis de conservacao).
Nao sei detalhes, mas o google sabe! hehe

Falou

Leo #

Quem está usando o meu santo nome em vão?
Tá certo que Léo tem acento (eis porque não fico de pé
o dia inteiro), e que 'em vão' pode significar 'em
freta', 'em fenda', ...... mas .......

Léo - the first primeiro



--
Internal Virus Database is out-of-date.
Checked by AVG Anti-Virus.
Version: 7.0.300 / Virus Database: 266.5.0 - Release
Date: 25/02/2005



##### ##### #####

Para saber mais visite
http://www.ciencialist.hpg.ig.com.br


##### ##### ##### #####


Yahoo! Grupos, um serviço oferecido por:PUBLICIDADE


---------------------------------
Links do Yahoo! Grupos

Para visitar o site do seu grupo na web, acesse:
http://br.groups.yahoo.com/group/ciencialist/

Para sair deste grupo, envie um e-mail para:
ciencialist-unsubscribe@yahoogrupos.com.br

O uso que você faz do Yahoo! Grupos está sujeito
aos Termos do Serviço do Yahoo!.

__________________________________________________
Converse com seus amigos em tempo real com o Yahoo! Messenger
http://br.download.yahoo.com/messenger/


SUBJECT: Fw: Cinza Queimada
FROM: "Luiz Ferraz Netto" <leobarretos@uol.com.br>
TO: "ciencialist" <ciencialist@yahoogrupos.com.br>
DATE: 10/03/2005 16:44

Hoje a correspondência está enorme ..........

[]'
===========================
Luiz Ferraz Netto [Léo]
leobarretos@uol.com.br
http://www.feiradeciencias.com.br
===========================
-----Mensagem Original-----
De: "adrianorvalle" <adrianorvalle@ig.com.br>
Para: <leobarretos@uol.com.br>
Enviada em: quinta-feira, 10 de março de 2005 16:36
Assunto: Cinza Queimada


Boa tarde
Professor

Gostaria que o professor me tira-se uma dúvida a respeito que quais
elementos químicos que resultam da queima do carvão de churrasqueira.

Antecipadamente agradeço.

Adriano.



--------------------------------------------------------------------------------


Internal Virus Database is out-of-date.
Checked by AVG Anti-Virus.
Version: 7.0.300 / Virus Database: 266.5.0 - Release Date: 25/02/2005



--
Internal Virus Database is out-of-date.
Checked by AVG Anti-Virus.
Version: 7.0.300 / Virus Database: 266.5.0 - Release Date: 25/02/2005



SUBJECT: NO STRESS
FROM: José Renato <jrma@terra.com.br>
TO: <ciencialist@yahoogrupos.com.br>
DATE: 10/03/2005 16:52

O Diretor de Produção ganhou um convite para assistir a um concerto da
"Sinfonia Inacabada", de Franz Schubert. Como estava impossibilitado de
comparecer, passou o convite para o seu gerente de Organização, Sistemas
e Métodos. Na manhã seguinte o administrador, perguntou de passagem, se ele tinha
gostado do concerto. Ao invés de comentários, ele emitiu o seguinte relatório:

De: Gerencia de Organização, Sistemas e Métodos
Para: Diretoria
Ref: Sinfonia Inacabada

1- Por um período considerável de tempo, os músicos com oboé não tinham
o que fazer. Sua quantidade deveria ser reduzida e seu trabalho
redistribuído pela orquestra, evitando esses picos de inatividade;

2- Todos os doze violinos da primeira seção tocavam notas idênticas.
Isso parece ser uma duplicidade desnecessária de
esforços e o contingente nessa seção deveria ser drasticamente cortado.
Se um alto volume de som fosse requerido, isso poderia ser obtido
através de uso de amplificador;

3- Muito esforço foi envolvido em tocar semitons. Isso parece ser um
preciosismo desnecessário e seria recomendável que as notas fossem
executadas no tom mais próximo. Se isso fosse feito, poder-se-ia
utilizar estagiários em vez de profissionais;

4- Não havia utilidade prática em repetir com os metais a mesma passagem
já tocada pelas cordas. Se toda essa redundância fosse eliminada, o
concerto poderia ser reduzido de duas horas para apenas 20 minutos;

5- Enfim, sumarizando as observações anteriores, podemos concluir que:
Se Schubert tivesse dado um pouco de atenção a esses pontos, talvez
tivesse tido tempo de acabar sua sinfonia.

A Gerência.



[As partes desta mensagem que não continham texto foram removidas]



SUBJECT: Re: [ciencialist] Re: imas --- correntes --- Leo pergunta
FROM: "Alvaro Augusto \(E\)" <alvaro@electraenergy.com.br>
TO: <ciencialist@yahoogrupos.com.br>
DATE: 10/03/2005 16:53

Bem, a blindagem aparece porque os elétrons da camada de valência dos átomos de ferro estão com seus spins alinhados anti-paralelamente. Quando aparece algum efeito vindo de átomos vizinhos, são esses elétrons que respondem, e não os elétrons alinhados da penúltima camada (que tem seis elétrons, quatro alinhados e dois antiparalelos). A magnetização do átomo de ferro é produzida por esses quatro elétrons.

Quanto à zona de probabilidade, ela não é o elétron em si, mas sim uma medida da probabilidade de se encontrar o elétron em uma determinada região. O elétron, bem, ninguém sabe quem é! Logo, não é correto dizer que o spin é o "giro" do elétron em torno de seu eixo. Até onde se sabe o spin é somente uma propriedade do elétron que serve para produzir o campo magnético!

A formação dos domínios magnéticos está ligada à minimização de energia da rede cristalina. Um monocristal, uma vez magnetizado, produziria um campo magnético uniforme, mas muito intenso. Na região externa ao material, esse campo tenderá a inverter a orientação dos spins, desmagnetizando pedaços do material. Assim, o monocristal inicial acabará se "fracionando" magneticamente, até que a energia magnética se torne mínima. Quando o processo de fracionamento terminar, os pedaços resultantes (domínios) estarão uniformemente magnetizados, embora alinhados aleatoriamente uns em relação aos outros.

Mas todos os átomos participam dos domínios magnéticos.

[ ]s

Alvaro Augusto



----- Original Message -----
From: Luiz Ferraz Netto
To: ciencialist@yahoogrupos.com.br
Sent: Thursday, March 10, 2005 1:04 PM
Subject: Re: [ciencialist] Re: imas --- correntes --- Leo pergunta


Gostei disso do Álvaro ...... vamos aprofundar a idéia?
Essa blindagem oferecida pela última camada ficou um tanto 'esquisito'; se a parte interna da blindagem é 'norte', a externa é 'sul'?
Como pode uma zona de probabilidade (em substituição ao elétron girando ao redor do núcleo) gerar um campo magnético? Se não há elétron efetivamente girando ao redor do núcleo, o que vem a ser elétron girando ao redor de si mesmo ... os spins emparelhados e não emparelhados?
Quem participa na formação dos domínios magnéticos? alguns átomos especiais? alguma estrutura cristalina?

e vamos em frente .........

[]'
===========================
Luiz Ferraz Netto [Léo]
leobarretos@uol.com.br
http://www.feiradeciencias.com.br
===========================
-----Mensagem Original-----
De: "Alvaro Augusto (E)" <alvaro@electraenergy.com.br>
Para: <ciencialist@yahoogrupos.com.br>
Enviada em: quinta-feira, 10 de março de 2005 11:16
Assunto: Re: [ciencialist] Re: imas --- correntes --- Leo pergunta



A magnetização de elementos ferromagnéticos (ferro, níquel, cobalto, gadolínio, disprósio e espuma de carbono) é um fenônemo complicado. O spin orbital tem pouca influência, pois é um efeito paramagnético, que todas as substâncias apresentam, e não é permanente, pois quando o campo magnetizante é removido, a agitação térmica destróis os alinhamentos orbitais. Já os ferromagnéticos se magnetizam permanentemente devido a uma conjunção de três fatores:

- alinhamento de spins eletrônicos da penúltima camada, possibilitado pela interação entre átomos vizinhos;
- blindagem magnética da camada de valência;
- formação de domínios magnéticos.

A blindagem só funciona para elementos com uma certa distância interatômica. Se a distância for muito grande, como no caso do alumínio, a interação entre elétrons de átomos vizinhos é muito fraca, e o alinhamento não será mantido, por causa da fraca energia de troca. Se a distância for muito pequena, como no caso do chumbo, a interação entre elétrons de átomos vizinhos será muito forte, e o alinhamento será destruído.

A rigor, somente o ferro é ferromagnético, pois somente o ferro é magnético da forma que o ferro é magnético, apresentando seis elétrons na penúltima camada de valência, estrutura cristalina cúbica e outras propriedade. O cobalto, por exemplo, apresenta estrutura hexagonal. O níquel também apresenta estrutura cúbica, mas é mais facilmente magnetizado ao longo das diagonais das faces dos cristais, e não ao longo das aretas das faces, como no caso do ferro. Assim, devido à formação de cristais, o ferromagnetismo é uma propriedade do elemento, não do átomo em si.

[ ]s

Alvaro Augusto

----- Original Message -----
From: rmtakata
To: ciencialist@yahoogrupos.com.br
Sent: Thursday, March 10, 2005 10:06 AM
Subject: [ciencialist] Re: imas --- correntes --- Leo pergunta



--- Em ciencialist@yahoogrupos.com.br, Leonardo Souza
> A magnetizacao tambem tem alguma coisa a ver com
> spins, certo??

Sim:

http://hyperphysics.phy-astr.gsu.edu/hbase/spin.html

> --- Luiz Ferraz Netto <leobarretos@u...> wrote:
> Quando os ímãs são usados para realizar trabalho (na
> produção de energia elétrica, por exemplo) essas
> correntes intrínsecas não deveriam sofrer alguma
> alteração?

Nao eh o alinhamento magnetico dos atomos q. sofre as consequencias?

> O maldito elétron continua firme e majestoso circulando ao redor do
> núcleo e mantendo (às custas sei lá de quem) o formidável campo
> magnético?

Acho q. ele continua a 'girar' ao redor do nucleo (embora a imagem
atual seja de uma nuvem de probabilidade), mas o alinhamento dos
(campos magneticos dos) atomos pode mudar.

> Esses elétrons tb entram na categoria dos 'privilegiados' na física
> quântica?

Imagino q. sim.

[]s,

Roberto Takata





##### ##### #####

Para saber mais visite
http://www.ciencialist.hpg.ig.com.br


##### ##### ##### #####


Yahoo! Grupos, um serviço oferecido por:

São Paulo Rio de Janeiro Curitiba Porto Alegre Belo Horizonte Brasília




------------------------------------------------------------------------------
Links do Yahoo! Grupos

a.. Para visitar o site do seu grupo na web, acesse:
http://br.groups.yahoo.com/group/ciencialist/

b.. Para sair deste grupo, envie um e-mail para:
ciencialist-unsubscribe@yahoogrupos.com.br

c.. O uso que você faz do Yahoo! Grupos está sujeito aos Termos do Serviço do Yahoo!.



[As partes desta mensagem que não continham texto foram removidas]



##### ##### #####

Para saber mais visite
http://www.ciencialist.hpg.ig.com.br


##### ##### ##### #####
Links do Yahoo! Grupos










--
Internal Virus Database is out-of-date.
Checked by AVG Anti-Virus.
Version: 7.0.300 / Virus Database: 266.5.0 - Release Date: 25/02/2005




--
Internal Virus Database is out-of-date.
Checked by AVG Anti-Virus.
Version: 7.0.300 / Virus Database: 266.5.0 - Release Date: 25/02/2005



##### ##### #####

Para saber mais visite
http://www.ciencialist.hpg.ig.com.br


##### ##### ##### #####


Yahoo! Grupos, um serviço oferecido por:
PUBLICIDADE




------------------------------------------------------------------------------
Links do Yahoo! Grupos

a.. Para visitar o site do seu grupo na web, acesse:
http://br.groups.yahoo.com/group/ciencialist/

b.. Para sair deste grupo, envie um e-mail para:
ciencialist-unsubscribe@yahoogrupos.com.br

c.. O uso que você faz do Yahoo! Grupos está sujeito aos Termos do Serviço do Yahoo!.



[As partes desta mensagem que não continham texto foram removidas]



SUBJECT: P/ Ivan (era Cientistas recriam nervos ópticos em ratos
FROM: "Oraculo" <oraculo@atibaia.com.br>
TO: <ciencialist@yahoogrupos.com.br>
DATE: 10/03/2005 17:17

Olá Ivan

Peço desculpas novamente, acho que me preciptei..:-) E você se expressa bem, eu é que li demais em poucas linhas..:-)

Terei mais atenção de agora em diante, com textos curtos.:-)

Um abraço.

Homero

----- Original Message -----
From: Ivan Carlos
To: ciencialist@yahoogrupos.com.br
Sent: Thursday, March 10, 2005 2:13 AM
Subject: Re: [ciencialist] Cientistas recriam nervos ópticos em ratos


Nanananaum foi isso o que quiz dizer não!

Diferente de achar q quiz dizer que "cientistas não sabem de nada e ficam
contando vantagem", fico maravilhado com o avanço dos trabalhos, só achei
engraçado pq o texto passou e muito uma impressão de ironia, talvez tenha
sido descuido do autor, mas não que eu concorde com isso...!

Acompanho a lista sbcr, acompanho as pesquisas do campo da neurociência,
compreendo o que quiz dizer sobre os passos de desenvolvimento da pesquisa.

Continue escrevendo bastante, não sei me expressar bem, assim vc me completa
:)

[]!

Ivan "Doomer" Carlos
Social Engineering Specialist
Cell.: +55 (11) 8112-0666
icarlos@icarlos.net
www.icarlos.net
--------------------------------------------------


----- Original Message -----
From: "Oraculo" <oraculo@atibaia.com.br>
To: <ciencialist@yahoogrupos.com.br>
Sent: Thursday, March 10, 2005 1:33 AM
Subject: Re: [ciencialist] Cientistas recriam nervos ópticos em ratos



Olá Ivan

Por que?..:-) Quero dizer, o que há de engraçado em ser cuidadoso com
descobertas e dados de pesquisa, como o correto funcionamento de um sistema
nervoso (em um procedimento de recuperação que nunca foi possível antes, e
do qual temos pouca informação)?..:-)

Toda pesquisa cientifica tem um próximo passo, significando que nenhum
conhecimento é completo ou final, dependendo sempre de ajustes e de novas
pesquisas. Por exemplo, os primeiros passos com células tronco determinam
apenas resultados parciais, ainda sem que se saiba exatameten como essa
melhora ocorre. Os próximos passos são, claro, descobrir como isso cocore,
se pode ser melhrpoado, como induzir determinados padrões de tecidos, etc.

O mesmo para a (fantastica) nova e recém descoberta capacidade de regenerar
nervos óticos. O próximo passo é determianr se esse novo nervo regenerado
funciona da mesma forma que o nervo original, ou se, por exemplo, embaralha
os sinais de forma a impedir que sejam corretamente interpretados pelo
cérebro. Não há, sei que deve percebner isso, nenhuma chance de aprendermos
a recuperar nervos óticos de seres humanso de um dia para o outro, sem antes
passar por diversas (e as vezes frustantes) fases e passos cuidadosos..:-)

Esse discurso todo acima (o pessoal da lista pode garantir que eu escrevo
demais mesmo..:-) é apenas para determinar o que achou engraçado na frase
dos pesquisadores..:-) Em geral, esse tipo de graça é derivada da acusação
sempre recorrente que "cientistas não sabem de nada e ficam contando
vantagem" ou coisa parecida..:-) O que, me parece, não é exatamente
verdade.:-)

Se a intenção não era essa, me desculpe o engano (e o tamanho desta
mensagem..:-)

Um abraço.

Homero


----- Original Message -----
From: Ivan Carlos
To: ciencialist@yahoogrupos.com.br
Sent: Wednesday, March 09, 2005 9:26 PM
Subject: Re: [ciencialist] Cientistas recriam nervos ópticos em ratos


acho engraçado pacas expressões do tipo "O próximo passo agora será
determinar se os nervos regenerados funcionam direito." =)))

Ivan "Doomer" Carlos
Social Engineering Specialist
Cell.: +55 (11) 8112-0666
icarlos@icarlos.net
www.icarlos.net
--------------------------------------------------


----- Original Message -----
From: "Oraculo" <oraculo@atibaia.com.br>
To: <ciencialist@yahoogrupos.com.br>
Sent: Monday, March 07, 2005 1:52 AM
Subject: [ciencialist] Cientistas recriam nervos ópticos em ratos



Cientistas recriam nervos ópticos em ratos

Cientistas dizem ter conseguido reconstruir em ratos nervos ópticos
danificados em toda a extensão do percurso desde o olho até o cérebro.
Especialistas dizem que a experiência, feita pelo Instituto Schepens de
Pesquisas do Olho, traz novas esperanças para tratar o glaucoma, uma
enfermidade na qual um nível elevado de pressão destrói o nervo óptico.

A técnica também poderia ser usada para ajudar pessoas que sofrem de
problemas na medula espinhal e em outras partes do sistema nervoso.

A experiência é detalhada na revista especializada Journal of Cell
Science.

Cicatriz

"Isto é o mais próximo que a ciência já chegou de regenerar tantas fibras
de
nervos ópticos em uma distância tão grande para chegar ao seu objetivo",
disse Dong Feng Chen, o líder da equipe de pesquisadores.

"E também de reparar um nervo que antes se achava que tinha se danificado
de
forma irreparável."

Vários tecidos celulares do corpo humano se reconstituem sozinhos em caso
de
lesão.

Mas não o nervo óptico e outros tecidos do sistema nervoso central, cujos
danos são sempre permanentes.

Os cientistas do Instituto Schepens descobriram que a incapacidade do
nervo
óptico de se regenerar estava ligada ao não funcionamento de um gene
chamado
BCL-2.

Eles também descobriram que o processo de regeneração estava bloqueado
pela
criação, logo após o nascimento, de uma cicatriz no cérebro por células
gliais especializadas.

Essas células desempenham várias funções no cérebro, entre as quais a
criação dessa espécie de cicatriz.

A cicatriz coloca uma barreira ao mesmo tempo física e molecular para a
regeneração.

Sempre ativo

Os cientistas criaram ratos nos quais o BCL-2 estava sempre ativado.

Eles descobriram que os animais conseguiam reparar os nervos ópticos com
rapidez - mas só enquanto eram jovens e a cicatriz no cérebro ainda não
havia se desenvolvido.

Em seguida eles criaram ratos com o BCL-2 funcionando e uma reduzida
capacidade de criar as cicatrizes pelas células gliais.

Desta vez, mesmo os roedores mais velhos conseguiram regenerar as células
ópticas.

O próximo passo agora será determinar se os nervos regenerados funcionam
direito.




http://www.bbc.co.uk/portuguese/ciencia/story/2005/03/050302_nervoopticoro.shtml

[As partes desta mensagem que não continham texto foram removidas]



##### ##### #####

Para saber mais visite
http://www.ciencialist.hpg.ig.com.br


##### ##### ##### #####
Links do Yahoo! Grupos











##### ##### #####

Para saber mais visite
http://www.ciencialist.hpg.ig.com.br


##### ##### ##### #####


Yahoo! Grupos, um serviço oferecido por:







------------------------------------------------------------------------------
Links do Yahoo! Grupos

a.. Para visitar o site do seu grupo na web, acesse:
http://br.groups.yahoo.com/group/ciencialist/

b.. Para sair deste grupo, envie um e-mail para:
ciencialist-unsubscribe@yahoogrupos.com.br

c.. O uso que você faz do Yahoo! Grupos está sujeito aos Termos do
Serviço do Yahoo!.



[As partes desta mensagem que não continham texto foram removidas]



##### ##### #####

Para saber mais visite
http://www.ciencialist.hpg.ig.com.br


##### ##### ##### #####
Links do Yahoo! Grupos











##### ##### #####

Para saber mais visite
http://www.ciencialist.hpg.ig.com.br


##### ##### ##### #####


Yahoo! Grupos, um serviço oferecido por:







------------------------------------------------------------------------------
Links do Yahoo! Grupos

a.. Para visitar o site do seu grupo na web, acesse:
http://br.groups.yahoo.com/group/ciencialist/

b.. Para sair deste grupo, envie um e-mail para:
ciencialist-unsubscribe@yahoogrupos.com.br

c.. O uso que você faz do Yahoo! Grupos está sujeito aos Termos do Serviço do Yahoo!.



[As partes desta mensagem que não continham texto foram removidas]



SUBJECT: Fw: Feira de Ci�ncias - PARAB�NS !
FROM: "Luiz Ferraz Netto" <leobarretos@uol.com.br>
TO: "ciencialist" <ciencialist@yahoogrupos.com.br>
DATE: 10/03/2005 20:20

É ... em cada montão sempre tem um ou outro .........

Estendo a todos .........

[]'
===========================
Luiz Ferraz Netto [Léo]
leobarretos@uol.com.br
http://www.feiradeciencias.com.br
===========================
-----Mensagem Original-----
De: Celio
Para: leobarretos@uol.com.br
Enviada em: quinta-feira, 10 de março de 2005 17:41
Assunto: Feira de Ciências - PARABÉNS !


Prezado professor Luiz Ferraz Netto,

ao navegar na rede, hoje, encontrei seu site Feira de Ciências.
Tenho 63 anos, aposentado da IBM e nas horas disponíveis gosto de estudar cosmologia, astronomia e física.
Fiquei surpreso pela qualidade fantástica deste site. Ao ler o artigo Física Quantica para os misticos, esotéricos etc, não me contive; interrompí a leitura para parabeniza-lo pelo seu belíssimo trabalho. Parabéns pelo seu fantástico trabalho para melhorar o nível de conhecimento do brasileiro, infelizmente arraigado em misticismos, falsas crenças, fundamentalismo religioso.
A governadora Rosinha pretende propor a inclusão do estudo do criacionismo nas escolas. Estamos regredindo ao fundamentalismo !
Mas retornando ao seu trabalho, aceite meus cumprimentos e admiração. Já incluí seu site na relação de favoritos e vou divulga-lo para amigos. Este assunto interessa não apenas aos jovens, mas a todos nós que não perdemos uma oportunidade para expandir os horizontes.
saudações,
Celio Bastos


--------------------------------------------------------------------------------


Internal Virus Database is out-of-date.
Checked by AVG Anti-Virus.
Version: 7.0.300 / Virus Database: 266.5.0 - Release Date: 25/02/2005

----------

Internal Virus Database is out-of-date.
Checked by AVG Anti-Virus.
Version: 7.0.300 / Virus Database: 266.5.0 - Release Date: 25/02/2005


[As partes desta mensagem que não continham texto foram removidas]



SUBJECT: Re: [ciencialist] Re: imas --- correntes --- Leo pergunta
FROM: "Alberto Mesquita Filho" <albmesq@uol.com.br>
TO: <ciencialist@yahoogrupos.com.br>
DATE: 10/03/2005 20:33

----- Original Message -----
From: "rmtakata"
Sent: Thursday, March 10, 2005 2:01 PM
Subject: [ciencialist] Re: imas --- correntes --- Leo pergunta

> Entre outras coisas nao existe um
> analogo classico pleno para o spin quantico do eletron.

Eu diria que esta afirmativa, a apoiar-se nas idéias de Bohr e referendada
por Pauli, está dia a dia mostrando-se totalmente errada e, em decorrência,
a comprometer tremendamente o dogmatismo ficcionista quântico não apenas da
escola de Copenhage mas também.

Segundo Pauli (1958), "it is impossible to observe the spin of the electron,
separated fully from its orbital momentum, by means of experiments based on
the concept of classical particle trajectories". Não obstante, segundo
Batellaan e al. (1997) a experimentação demonstrou exatamente o contrário.
Em suas palavras: "Recently, however, the range of validity of Bohr's
assertion has been rendered uncertain by the beautiful experiments of
Dehmelt and his colleagues, in which electrons of a given spin are isolated
in a modified Penning trap, allowing a determination of their magnetic
moment".

Sem dúvida, os autores tentam de todas as maneiras salvar o comprometimento
quântico através das já famosas hipóteses "ad hoc" associadas aos não menos
famosos malabarismos teóricos que outra coisa não fazem senão adiar o
apocalipse quântico. No caso apresentado, propõem uma "análise semiclássica"
e que, sob certos aspectos, concordaria com a visão do Takata, pois não
seria um análogo clássico "pleno".

As citações acima poderão ser encontradas no artigo
http://www.physics.unl.edu/~hbatelaan/pdf/stern%20girlach.pdf , mas existem
inúmeros outros artigos na literatura a tratarem do mesmo assunto, como
aquele citado pelo Takata (Anton Zeilinger).

Eu não tenho dúvidas que, enquanto os físicos aceitarem gratuitamente (sem
comprovação laboratorial) que classicamente um elétron gera um campo
coulombiano, a exemplo de uma carga elétrica coulombiana clássica, o impasse
persistirá e nem a física quântica, nem essa física clássica deturpada que
está por aí, conseguirão ir além da ficção que tem contaminado a física há
cerca de um século.

Com respeito a minha visão sobre o assunto eu já cheguei a expor aqui por
várias vezes, e coloco-me à disposição para as críticas e/ou retomada da
discussão. Um resumo das minhas idéias pode ser encontrado em
http://ecientificocultural.com/Eletron2/parep.htm

[ ]´s
Alberto
http://ecientificocultural.com/indice.htm
Mas indiferentemente a tudo isso, o neutrino tem massa, o elétron não é
uma carga elétrica coulombiana e a Terra se move. E a história se repetirá.



SUBJECT: Re: Os desastres de Sofia
FROM: Maria Natália <grasdic@hotmail.com>
TO: ciencialist@yahoogrupos.com.br
DATE: 10/03/2005 23:12


César:

Pois naquele tempo e no anfiteatro da Faculdade de Ciências não
havia sequer equipamento para se fazer vídeo conferência.
Ando para pedir a este Professor que venha a minha escola fazer essa
primeira conferência mas os alunos já não são o que eram e os colegas
de matemática andam numa de preparar para exame e nem perceberam ainda
a importãncia da estatística e como todos estamos dependentes dela.
Já pensaram em como se determinam os prémios de seguro automóvel ou
outro!?
Não tenho nada gravado.Contudo... e atendendo ao jeitinho...com que o
pedido é feito vou falar com ele pessoalmente e até saber se faz
tenções de ir aí ou se tem essa conferência gravada...
Pense assim: Conhece o LMON ali para Campinas? Pois suponha que fez
entrevista ás meninas da zona, tá vendo o castiço e o folclore?
Depois dos resultados e com alguma pimenta os conceitos entraram sem
dor. E ver os moços de 15 a 18 anos a acompanharem aquele voo e que
até equações matemáticas tinha...Era Matemática divertida e apimentada.
Obrigada pela indicação do livro o vou passar para a outra lista.
Sabe que coloquei a mensagem não só para os portugueses da lista mas
para vos fazer invejinhas? LOLLLL
Um abraço
Maria Natália

--- Em ciencialist@yahoogrupos.com.br, César A. K. Grossmann
<cesarakg@b...> escreveu
>
> --- Em ciencialist@yahoogrupos.com.br, Maria Natália <grasdic@h...>
> escreveu
> >
> > A primeira aula dele há 5 anos foi: "A Mais velha Profissão do Mundo
> > está bem paga?"
>
> Tem uma transcrição da aula na internet? Fiquei curioso. Aliás,
> curiosíssimo...
>
> > A convite de João Caraça, Director de Ciência da FCG,
> > Dinis Pestana preparou um programa de televisão (O Acaso não Acontece
> > po





SUBJECT: Re: [ciencialist] Re: Fw: <grandes mulheres cientistas>
FROM: Franco <dfranco@pop.com.br>
TO: ciencialist@yahoogrupos.com.br
DATE: 11/03/2005 02:05

Aproveitando o tema, em Novembro de 2004 ocorreu no Rio a Conferência
Mulheres Latino Americanas em Ciências Exatas e da Vida
<http://www.cbpf.br/%7Emulher/>, congregando pesquisadoras pesquisadoras
nas áreas de física, química, matemática e biologia de diferentes países
da América Latina. Só descobri o site essa semana, por acaso. Vale a
pena uma visita na sessão Material para Leitura
<http://www.cbpf.br/~mulher/leitura.html> com textos pertinentes ao tema.
E o site está em <http://www.cbpf.br/~mulher/>

Franco.

rmtakata wrote:

>
> Esta eh uma lista das ganhadoras de Nobel (de ciencias):
>
> http://br.groups.yahoo.com/group/ciencialist/message/43932
>
> []s,
>
> Roberto Takata




SUBJECT: mulé do Einstein, sexta na TV
FROM: "L.E.R.de Carvalho" <lecarvalho@infolink.com.br>
TO: ciencialist@yahoogrupos.com.br
DATE: 11/03/2005 02:30


[]
Por trás de todo homem...
<http://sender.mailsender.com.br/Redir?0459f8ebb506c02718d4b29b11f586a0&423127752DA5856159EF1867DD277375>“A
Mulher de Einstein” (GNT, 22h45) conta a história de Mileva Maric, primeira
esposa do cientista. O documentário mostra como a contribuição de Mileva
para o desenvolvimento da teoria da relatividade e do efeito fotoelétrico
foi apagada pela história.


[As partes desta mensagem que não continham texto foram removidas]



SUBJECT: Re: imas --- correntes --- Leo pergunta
FROM: "rmtakata" <rmtakata@altavista.net>
TO: ciencialist@yahoogrupos.com.br
DATE: 11/03/2005 08:17


--- Em ciencialist@yahoogrupos.com.br, "Alberto Mesquita Filho"
> Eu diria que esta afirmativa, a apoiar-se nas idéias de Bohr e

De todo modo, o simples giro do eletron criaria uma dificuldade. O
spin eh quantizado, ao contrario do giro.

[]s,

Roberto Takata





SUBJECT: Re: [ciencialist] Re: imas --- correntes --- Leo pergunta
FROM: "Alberto Mesquita Filho" <albmesq@uol.com.br>
TO: <ciencialist@yahoogrupos.com.br>
DATE: 11/03/2005 09:20

----- Original Message -----
From: "rmtakata"
Sent: Friday, March 11, 2005 8:17 AM
Subject: [ciencialist] Re: imas --- correntes --- Leo pergunta

> De todo modo, o simples giro do eletron criaria uma dificuldade. O spin eh
> quantizado, ao contrario do giro.

Digamos que o spin é uma entidade abstrata criada pelos estudiosos da física
quântica com a finalidade de justificar alguns fenômenos que, por si só, não
são nem quânticos nem clássicos, mas simplesmente constatações
experimentais. Por exemplo, a duplicação (separação) de um feixe de átomos
de hidrogênio ao passar por um campo magnético ou a duplicação de
determinadas raias do espectro da luz emitida, por exemplo, por átomos
contendo um único elétron. Ou seja, existe um fenômeno a necessitar de uma
explicação, e existe uma abstração chamada spin (palavra que significa giro,
até prova em contrário) que estaria coerente com a ficção quântica, e neste
caso sem significar giro graças a um impedimento desta própria física
ficcionista. O termo spin foi então criado para justificar um algoritmo
quântico a supostamente explicar como poderiam ser encaixados os fenômenos
físicos acima descritos numa física que, a rigor, não explica nada. Nada
mais é senão um nome a justificar um algoritmo que dá certo sem que se saiba
o porquê. Neste sentido concordo contigo: "O spin eh quantizado, ao
contrario do giro." Mas é quantizado por definição de spin, e isso não
significa dizer que o fenômeno seja quantizado e/ou que não possa ser
explicado por uma física clássica genuinamente newtoniana.

Obviamente, ao pensarmos em física clássica chegaremos à conclusão que o
fenômeno não pode ser apenas um giro. Qualquer um que conheça um pouquinho
de física clássica perceberá que esta suposição seria por demais ingênua
(não sei se os "físicos quânticos" já se deram conta disso, pois eles são
por demais ingênuos em termos de física clássica). Ou seja, o fenômeno
descoberto na experiência de Stern Gerlach ou na análise espectral de
Uhlenbeck e Gouldsmit, para que tenha uma explicação clássica, deve ser um
pouquinho mais complexo, como já cheguei a demonstrar aqui na Ciencialist.
Mas, apesar disso, é bem mais simples do que aquele estratosférico algoritmo
que dá certo sem que se saiba o porquê.

[ ]´s
Alberto
http://ecientificocultural.com/indice.htm
Mas indiferentemente a tudo isso, o neutrino tem massa, o elétron não é
uma carga elétrica coulombiana e a Terra se move. E a história se repetirá.



SUBJECT: Re: imas --- correntes --- Leo pergunta
FROM: "rmtakata" <rmtakata@altavista.net>
TO: ciencialist@yahoogrupos.com.br
DATE: 11/03/2005 10:01


--- Em ciencialist@yahoogrupos.com.br, "Alberto Mesquita Filho"
> ----- Original Message -----
> From: "rmtakata"
> Digamos que o spin é uma entidade abstrata criada pelos estudiosos
> da física quântica com a finalidade de justificar alguns fenômenos
> que, por si só, não são nem quânticos nem clássicos, mas
> simplesmente constatações experimentais.

Sim, como ocorre com outras coisas atribuidas 'a materia: massa, carga
eletrica...

> explicação, e existe uma abstração chamada spin (palavra que
> significa giro, até prova em contrário) que estaria coerente com a
> ficção quântica, e neste

Giro foi a acepcao original. Um sabor de um quark certamente nao eh
nada do q. entendamos como o sabor do sal (alias, quark tb nao eh
queijo). Por outro lado, eletron nao eh ambar; rato-toupeira nao eh
rato nem toupeira; massa nao eh sovado (bem, a de pao eh); uma
corrente de prender cachorro nao corre; orelha de livro nao escuta;
perna de mesa nao anda; agua doce nao tem acucar; dureza da agua nao
se mede em Mohs... e vivemos.

[]s,

Roberto Takata





SUBJECT: Re: [ciencialist] Re: imas --- correntes --- Leo pergunta
FROM: "Alvaro Augusto \(E\)" <alvaro@electraenergy.com.br>
TO: <ciencialist@yahoogrupos.com.br>
DATE: 11/03/2005 13:34


----- Original Message -----
From: Alberto Mesquita Filho
To: ciencialist@yahoogrupos.com.br
Sent: Friday, March 11, 2005 9:20 AM
Subject: Re: [ciencialist] Re: imas --- correntes --- Leo pergunta


----- Original Message -----
From: "rmtakata"
Sent: Friday, March 11, 2005 8:17 AM
Subject: [ciencialist] Re: imas --- correntes --- Leo pergunta

> > De todo modo, o simples giro do eletron criaria uma dificuldade. O spin
> > eh
> > quantizado, ao contrario do giro.

> Obviamente, ao pensarmos em física clássica chegaremos à conclusão que o
> fenômeno não pode ser apenas um giro. Qualquer um que conheça um pouquinho
> de física clássica perceberá que esta suposição seria por demais ingênua
> (não sei se os "físicos quânticos" já se deram conta disso, pois eles são
> por demais ingênuos em termos de física clássica). Ou seja, o fenômeno
> descoberto na experiência de Stern Gerlach ou na análise espectral de
> Uhlenbeck e Gouldsmit, para que tenha uma explicação clássica, deve ser um
> pouquinho mais complexo, como já cheguei a demonstrar aqui na Ciencialist.
> Mas, apesar disso, é bem mais simples do que aquele estratosférico
> algoritmo
> que dá certo sem que se saiba o porquê.

Assim que o spin foi descoberto, e que foi proposto ser análogo ao giro do
elétron em torno de si mesmo, um dos grandes físicos clássicos (acho que foi
Lorentz) correu para calcular a velocidade de giro, e chegou à conclusão que
esta deveria ser superior à velocidade da luz. Assim, o fato de que o spin
não é exatamente rotação já era conhecido desde os primórdios...

O spin é só uma grandeza a mais. Se os físicos clássicos não gostam dele,
fazer o quê!? Os adeptos da mecânica de Hertz também não gostavam do
conceito newtoniano de força, pois o entendiam como antropomórfico demais.
Finalmente, com auxílio do teorema de Padoa, foi demonstrado que o conceito
de força não pode ser removido das axiomáticas existentes para a mecânica de
Newton, nem mesmo por definição.

O spin existe e, a despeito das críticas, "continua girando"...

[ ]s

Alvaro Augusto





SUBJECT: POR QUE????
FROM: Leonardo Souza <leo_feynman@yahoo.com.br>
TO: ciencialist@yahoogrupos.com.br
DATE: 11/03/2005 15:02


Por quê tanto ódio com relação à mecânica quântica neste grupo???

Leo


__________________________________________________
Converse com seus amigos em tempo real com o Yahoo! Messenger
http://br.download.yahoo.com/messenger/

[As partes desta mensagem que não continham texto foram removidas]



SUBJECT: Re: POR QUE????
FROM: "rmtakata" <rmtakata@altavista.net>
TO: ciencialist@yahoogrupos.com.br
DATE: 11/03/2005 15:19


--- Em ciencialist@yahoogrupos.com.br, Leonardo Souza <leo_feynman@y...>
> Por quê tanto ódio com relação à mecânica quântica neste grupo???

Não vejo ódio. Mas, sim, há bastante questionamento. De todo modo há
os admiradores tb.

[]s,

Roberto Takata





SUBJECT: Re: [ciencialist] POR QUE????
FROM: "E m i l i a n o C h e m e l l o" <chemelloe@yahoo.com.br>
TO: <ciencialist@yahoogrupos.com.br>
DATE: 11/03/2005 15:28

Olá Leonardo,

Você é novo no grupo? Creio que sim. E não fico surpreso com sua
primeira impressão. Confesso que também fiquei 'surpreso' com as
'espinafradas' do Alberto em relação Mecânica Quântica (lembra Alberto
quando a gente utilizava os códigos Espinafrar e SPINafrar?, eheheh) .

Mas é difícil separar a emoção da razão daquilo que se gosta ou
acredita. Não quero dizer com isso que o Alberto e outros estejam deixando
de lado argumentos e se baseando em devaneios ou crendices próprias. Mas
cada um tem seu estilo. Coloco aqui, para quem quiser ler, um pequeno trecho
do livro "Física em 6 lições" do Richard Feynman. O texto não é do Feynman,
mas do Paul Davies, o qual fez a introdução do livro.

--- início da citação ---

O público tem uma idéia equivocada de que a ciência é um empreendimento
impessoal, desapaixonado e totalmente objetivo. Enquanto a maioria das
outras atividades humanas é dominada por modas e personalidades, supõe-se
que a ciência seja restringida por regras de procedimento consagradas e
testes rigorosos. São os resultados que contam, não as pessoas que os
produzem.

Trata-se, é claro, de um disparate. A ciência é uma atividade baseada em
pessoas, como todo empreendimento humano, e igualmente sujeita à moda e ao
capricho. Neste caso, a moda é ditada menos pela escolha do assunto do que
pela forma de os cientistas pensarem sobre o mundo. Cada época adota sua
abordagem particular dos problemas cientificos, geralmente seguindo a trilha
aberta por certas figuras dominantes que definem a agenda e os melhores
métodos de atacá-la. Ocasionalmente, cientistas adquirem uma estatura
suficiente para serem notados pelo público em geral, e, quando dotado de um
talento excepcional, um cientista pode se tornar um ícone para toda a
comunidade científica. Em séculos anteriores, Isaac Newton foi um ícone.
Newton personificou o cientista cavalheiro - bem relacionado, devotadamente
religioso, calmo e metódico em seu trabalho. Seu estilo de fazer ciência
fixou o padrão por duzentos anos. Na primeira metade do século XX, Albert
Einstein substituiu Newton com o ícone cientista popular. Excêntrico,
descabelado, germânico, distraído, totalmente absorvido em seu trabalho e um
pensador abstrato arquetípico, Einstein mudou a forma de se fazer física,
questionando os próprios conceitos que definem o assunto.

--- final da citação ---

Enfim... era isso que eu tinha para te dizer :-)

[ ] 's do Emiliano Chemello
emiliano@quimica.net
http://www.quimica.net/emiliano
http://www.ucs.br/ccet/defq/naeq
[ MSN ] chemelloe@hotmail.com
[ ICQ ] 145060604

" Rien ne se perd, rien ne se crée,
tout se transforme."

Antoine Laurent de Lavoisier (químico francês, 1743 - 1794)

----- Original Message -----
From: Leonardo Souza
To: ciencialist@yahoogrupos.com.br
Sent: Friday, March 11, 2005 3:02 PM
Subject: [ciencialist] POR QUE????



Por quê tanto ódio com relação à mecânica quântica neste grupo???

Leo


__________________________________________________
Converse com seus amigos em tempo real com o Yahoo! Messenger
http://br.download.yahoo.com/messenger/

[As partes desta mensagem que não continham texto foram removidas]



##### ##### #####

Para saber mais visite
http://www.ciencialist.hpg.ig.com.br


##### ##### ##### #####


Yahoo! Grupos, um serviço oferecido por:
PUBLICIDADE





Links do Yahoo! Grupos

Para visitar o site do seu grupo na web, acesse:
http://br.groups.yahoo.com/group/ciencialist/

Para sair deste grupo, envie um e-mail para:
ciencialist-unsubscribe@yahoogrupos.com.br

O uso que você faz do Yahoo! Grupos está sujeito aos Termos do Serviço do
Yahoo!.




SUBJECT: Re: [ciencialist] POR QUE????
FROM: "Oraculo" <oraculo@atibaia.com.br>
TO: <ciencialist@yahoogrupos.com.br>
DATE: 11/03/2005 16:18

Olá Leo

Não é ódio, é a justa e saudável dúvida..:-) Toda ciência, todo conhecimento cientifico digno do nome deve suportar ataques vigorosos, questionamentos profundos e toda sorte de crítica e análise, Se for real, vai sobreviver, se não for, deve ser deixado para trás.

A quantica é muito nova (em termos, já que suas bases podem ser encontradas em trabalhos antigos) e por isso ainda tem muito a explicar e a ser testado. E, embora eu pessoalmente a considere bem estabelecida, isso não significa nada e ela deve ser objeto de críticas se quiser um dia ser aceita como conhecimento confiável (eu já a acho confiável, e muitos dos físicos atuais também pensam assim, mas isso, como disse antes, não basta..:-)

Pense que, no lugar de ódio, na verdade estas são demonstrações de interesse e legítimo exercício da dúvida..:-)

E veja pelo lado positivo, se o Alberto conseguir demonstrar além de toda dúvida seus pontos de vista, teremos o primeiro premio Nobel brasileiro, e justo em física, que nem o Lates conseguiu..:-) E eu torço por isso, mesmo discordando de suas críticas a mecanica quantica..:-)

Um abraço.

Homero

----- Original Message -----
From: Leonardo Souza
To: ciencialist@yahoogrupos.com.br
Sent: Friday, March 11, 2005 3:02 PM
Subject: [ciencialist] POR QUE????



Por quê tanto ódio com relação à mecânica quântica neste grupo???

Leo


__________________________________________________
Converse com seus amigos em tempo real com o Yahoo! Messenger
http://br.download.yahoo.com/messenger/

[As partes desta mensagem que não continham texto foram removidas]



##### ##### #####

Para saber mais visite
http://www.ciencialist.hpg.ig.com.br


##### ##### ##### #####


Yahoo! Grupos, um serviço oferecido por:
PUBLICIDADE




------------------------------------------------------------------------------
Links do Yahoo! Grupos

a.. Para visitar o site do seu grupo na web, acesse:
http://br.groups.yahoo.com/group/ciencialist/

b.. Para sair deste grupo, envie um e-mail para:
ciencialist-unsubscribe@yahoogrupos.com.br

c.. O uso que você faz do Yahoo! Grupos está sujeito aos Termos do Serviço do Yahoo!.



[As partes desta mensagem que não continham texto foram removidas]



SUBJECT: Re: POR QUE????
FROM: Hélio Ricardo Carvalho <hrc@fis.puc-rio.br>
TO: ciencialist@yahoogrupos.com.br
DATE: 11/03/2005 16:32


Em ciencialist Leonardo Souza escreveu
>
> Por quê tanto ódio com relação à mecânica quântica neste grupo???
>

Eu já vejo de forma totalmente diferente de você.

Ninguém aqui tem "ódio" da "mecânica quântica" (uma mecânica se
graxa).
A maioria esmagadora do grupo é FÃ dela.
Por mais incrível que isto possa parecer.

:-) :-) :-)
E quanto a mim acho que física quântica é menos pior que a
relatividade.
:-) :-) :-)

[ ]'s
Hélio






SUBJECT: Re: Brilho metálico
FROM: Hélio Ricardo Carvalho <hrc@fis.puc-rio.br>
TO: ciencialist@yahoogrupos.com.br
DATE: 11/03/2005 16:50


Takata:

Eu identifico uma divergência clara entre nossas formas de ver a
reflexão num espelho. E outras não tão claras assim que podem ser
apenas questões semânticas.

Veja se você concorda com esta classificação.

Para Takata:
[A luz (aqui não vou entrar no mérito se é onda, fóton ou partículas
menores) ao chegar nos elétrons de uma superfície metálica é
absorvida e depois reemitida e esta reemissão de cada elétron tem
uma direção e sentido bem definida que dependente da direção de
entrada e da configuração dos elétrons vizinhos. Eles guardam a
memória de como absorveram a luz.
Numa superfície de interface entre meios de índice de refração
diferentes, alguns saem, voltando para o mesmo meio, com uma direção
que forma um ângulo com a normal ao plano (conjunto dos elétrons)
igual ao ângulo de entrada com a mesma normal. Outros avançam no
outro meio mas formando um ângulo com a normal que obedece a lei de
Snell.]

Para Hélio:
[A luz (aqui também não vou entrar no mérito do que ela é) ao chegar
nos elétrons de uma superfície metálica é absorvida e depois
reemitida e esta reemissão de cada elétron é em todas as direções
(ou de forma aleatória para qualquer direção que na média sob várias
emissões dá algo parecido com "todas as direções") Eles NÃO guardam
a memória de como absorveram a luz.
Numa superfície de interface entre meios de índice de refração
diferentes, acontece o mesmo que acima mas os que avançam no outro
meio (digamos que com n maior) terão uma velocidade MÉDIA menor
devido a mais paradas a a menor distâncias entre as paradas (entre
uma parada e outra a velocidade é sempre c, seja qual for o meio).]


Explicando melhor a minha opinião:

O que determina as leis de reflexão e de Snell para a refração não é
mais um principio "imposto" a natureza (princípio de Fermat) mas
algo bem mais lógico relacionado com o local geométrico onde há
contribuições construtivas das várias frentes de onda (ou de
partículas). É um efeito de conjunto e nunca de uma só emissão ou de
emissões de um só elétron. É devido ao fato de estarem na mesma
"fase" somente naquele local geométrico a luz que bate naquela
região do espelho sob aquele ângulo.
Veja novamente:
http://tinyurl.com/4hbvk e aperte várias vezes o botão "Slower".
O ângulo de reflexão é igual ao de entrada em relação a normal ao
plano porque é aí que a fase se iguala.
No caso da refração o mecanismo é o mesmo só que a velocidade MËDIA
do meio é menor (se n for maior). Isto desloca o ângulo de acordo
com a lei de Snell.

Vamos as contas (usando a linguagem de fóton) [:-(]

Interface plana entre dois meios.
(não estou com tempo para desenhos então use sua imaginação :-), se
for necessário eu faço desenho depois)

Meio 1 --> velocidade MÉDIA v1
Meio 2 --> velocidade MÉDIA v2

Digamos que v2 < v1

Imagine dois fótons caminhando lado a lado. Quando o primeiro chega
a superfície o segundo ainda vai levar um tempo T para percorrer os
v1T de comprimento que faltam para também chegar à superfície. Então
pense num triângulo retângulo em que a distância entre os fótons é
um cateto, v1T é o outro e a distância entre os pontos de chegada na
superfície é a hipotenusa (vamos chama-la "D"). Quando o primeiro
chegou ele foi reemitido para uma direção aleatória e se foi dentro
do meio 2 saiu com velocidade v2. e quando o segundo chegar na
superfície o primeiro já terá percorrido uma distância v2T. Agora
trace uma semi circunferência de raio v2T dentro do meio 2 e com
centro no ponto de chegada do primeiro. Depois uma reta tangente a
esta circunferência que passe pelo segundo ponto de chegada. Se um
terceiro fóton veio entre os outros dois, é fácil ver que esta reta
também iria tangenciar a sua circunferência no meio 2.
Bem, o ângulo que linha dos fótons no meio 1 ("frente de onda")
forma com a superfície é o mesmo que a trajetória deles forma com a
normal ao plano, vamos chama-lo de theta1. Logo sen(theta1)=v1T/D.
De forma análoga a tangente descrita acima será como a "frente de
onda" no meio 2. Chamando de theta2 o ângulo entre a trajetória dos
fótons "refratados" e a normal ao plano temos: sen(theta2)=v2T/D.

T/D = sen(theta1)/v1 = sen(theta2)/v2 (lei de Snell)

Para ficar mais familiar v1=c/n1 e v2=c/n2

n1.sen(theta1) = n2.sen(theta2)

------------------------------------------------

Agora comentando sua última msg.

Em ciencialista Takata escreveu:

>Sorry, mas isso nao explica a anulacao das ondas migrando
>para a esquerda: do contrario, elas seriam detetadas. Um
>comprimento de onda menor, implica em maior energia
> - se valer o principio da conservacao da energia, os
>fontos refletidos teriam entao q. ter energia menor - o q.
>significaria q. teriam frequencia menor e assim os fotons
> refletidos deveriam nos dar uma sensacao de cor diferente.
>Se incidissemos um feixe de luz azul sobre um espelho, o
>reflexo teria q. ser vermelho ou algo assim...

Que confusão!!!
Não são do mesmo fóton!!!!
Não temos onda com maior freqüência. Temos fótons um pouco defasados
se anulando.
Se é que se pode falar assim quando trato de fótons.
Estes (vendo como fótons) ou estas (vendo como onda) se anulam numa
média temporal. Principalmente quando se encontram com outros vindos
de outras regiões do espelho pois não estão em fase. A questão de
energia deixo para comentar abaixo.

>>> ... como fica a conservacao da energia? A energia
>>>incidente teria q. ser dividida entre os diversos
>> >feixes refletidos.
>>
>> Aí eu jogo a toalha. Pois não sei bem o que é "energia".:)
>> Energia, para mim, não é um SER. :-)
>
> De qq forma, energia pode ser medida. E suas consequencias
> verificadas: por exemplo, a cor do raio refletido teria q.
> ser diferente do raio incidente. Ao enxergarmos um objeto
> pelo espelho, sua imagem teria q. ter cor diferente do
> objeto visto diretamente.
>

Quando digo que jogo a toalha quero dizer que ainda não tenho uma
explicação para esta "não perda de energia por espalhamento" fora da
"explicação" da física Quântica (note a palavra "explicação" entre
aspas).

Mas com FQ tudo pode ser "explicado".
Os fótons que vão "para a esquerda" não vão colapsar lá pois as
ondas de probabilidade chamadas de função de onda se anulam
mutuamente. LOLLLLLL

A 100 anos atrás um tal de Einstein falou:
"A teoria ondulatória da luz, que opera com funções espaciais
contínuas, provou-se sobremaneira adequada na descrição de fenômenos
puramente ópticos, e provavelmente nunca será substituída por outra
teoria. Deve-se ter em mente, porém, que as observações ópticas
referem-se a médias temporais, e não a valores instantâneos; e é
bastante concebível, A DESPEITO DA CONFIRMAÇÃO EXPERIMENTAL COMPLETA
DA DIFRAÇÃO, REFRAÇÃO, DISPERSÃO ETC, que a teoria da luz, operando
com funções espaciais contínuas, leve a contradições quando aplicada
aos fenômenos de emissão e transformação da luz."
Trecho de: "Sobre um ponto de vista heurístico a respeito da
produção e transformação da luz", Albert Einstein, Annalen der
Physik 17 [1905], p. 132-148, Tradução: Alexandre Carlos Torr.

Este artigo inventou o fóton (ainda não com este nome, o batismo foi
depois no nascimento) e a quântica.

Os destaques em letras capitais são meus.


[ ]'s
Hélio






SUBJECT: Re: POR QUE????
FROM: Hélio Ricardo Carvalho <hrc@fis.puc-rio.br>
TO: ciencialist@yahoogrupos.com.br
DATE: 11/03/2005 17:01


Onde se le "uma mecânica se graxa"
leia-se "uma mecânica sem graxa"

Puxa, com correção perdeu toda a graça. :-)

[ ]'s
Hélio

PS.:
O título desta thread é sujestivo.
POR QUE???
Não faça uma pergunta destas para a FQ. :-)




Em ciencialist@yahoogrupos.com.br, Hélio Ricardo Carvalho <hrc@f...>
escreveu
>
> Em ciencialist Leonardo Souza escreveu
> >
> > Por quê tanto ódio com relação à mecânica quântica neste
grupo???
> >
>
> Eu já vejo de forma totalmente diferente de você.
>
> Ninguém aqui tem "ódio" da "mecânica quântica" (uma mecânica se
> graxa).
> A maioria esmagadora do grupo é FÃ dela.
> Por mais incrível que isto possa parecer.
>
> :-) :-) :-)
> E quanto a mim acho que física quântica é menos pior que a
> relatividade.
> :-) :-) :-)
>
> [ ]'s
> Hélio





SUBJECT: Re: Brilho metálico
FROM: "rmtakata" <rmtakata@altavista.net>
TO: ciencialist@yahoogrupos.com.br
DATE: 11/03/2005 17:12


--- Em ciencialist@yahoogrupos.com.br, Hélio Ricardo Carvalho
> Eles guardam a memória de como absorveram a luz.

Não sei se esse acréscimo é necessário. Tudo o q. parece necessário é
q. os elétrons do material entrem em ressonância com a frequência da
luz incidente.

> Numa superfície de interface entre meios de índice de refração
> diferentes, acontece o mesmo que acima mas os que avançam no outro

Estamos falando de reflexão, não de refração.

> O que determina as leis de reflexão e de Snell para a refração não é
> mais um principio "imposto" a natureza (princípio de Fermat) mas

Não se trata de impor a natureza, mas de aceitar a imposição da
natureza no domínio dos fenômenos naturais.

> algo bem mais lógico relacionado com o local geométrico onde há
> contribuições construtivas das várias frentes de onda (ou de
> partículas).

Se pensarmos em termos de fótons: poderemos enviar um único fóton de
cada vez - sem frentes de partículas.

> É um efeito de conjunto e nunca de uma só emissão ou de
> emissões de um só elétron. É devido ao fato de estarem na mesma
> "fase" somente naquele local geométrico a luz que bate naquela
> região do espelho sob aquele ângulo.
> Veja novamente:
> http://tinyurl.com/4hbvk e aperte várias vezes o botão "Slower".

Nesse site, as ondas q. vão para a esquerda são concêntricas - não há
como elas se anularem.

> Meio 1 --> velocidade MÉDIA v1
> Meio 2 --> velocidade MÉDIA v2
>
> Digamos que v2 < v1
>
> Imagine dois fótons caminhando lado a lado. Quando o primeiro chega

Imagino/e dois fotons incidindo *perpendicularmente* 'a superficie. E
ai'? Nao ha' reflexao? E como se cancelam as ondas q. se espalham? No way.

[]s,

Roberto Takata





SUBJECT: Re: Brilho metálico
FROM: "rmtakata" <rmtakata@altavista.net>
TO: ciencialist@yahoogrupos.com.br
DATE: 11/03/2005 17:23


--- Em ciencialist@yahoogrupos.com.br, "rmtakata" <rmtakata@a...>
> Nesse site, as ondas q. vão para a esquerda são concêntricas - não há
> como elas se anularem.

Concentricas nao. Nao se interceptam 'a esquerda. A primeira onda q.
se forma, nao tem como ser anulada pela segunda onda q. se forma - ela
esta' sempre mais 'a esquerda.

[]s,

Roberto Takata





SUBJECT: Re: Brilho metálico
FROM: Hélio Ricardo Carvalho <hrc@fis.puc-rio.br>
TO: ciencialist@yahoogrupos.com.br
DATE: 11/03/2005 17:38


Takata,

É de impressionar a rapidez com que você responde. Infelizmente sou
apenas um humano talvez volte a falar sobre isto semana que vem.

:-) :-) :-)

Você, como sempre, está de parabéns.

[ ]'s
Hélio




--- Em ciencialist@yahoogrupos.com.br, "rmtakata" <rmtakata@a...>
escreveu
>
> --- Em ciencialist@yahoogrupos.com.br, Hélio Ricardo Carvalho
> > Eles guardam a memória de como absorveram a luz.
>
> Não sei se esse acréscimo é necessário. Tudo o q. parece
necessário é
> q. os elétrons do material entrem em ressonância com a frequência
da
> luz incidente.
>
> > Numa superfície de interface entre meios de índice de refração
> > diferentes, acontece o mesmo que acima mas os que avançam no
outro
>
> Estamos falando de reflexão, não de refração.
>
> > O que determina as leis de reflexão e de Snell para a refração
não é
> > mais um principio "imposto" a natureza (princípio de Fermat) mas
>
> Não se trata de impor a natureza, mas de aceitar a imposição da
> natureza no domínio dos fenômenos naturais.
>
> > algo bem mais lógico relacionado com o local geométrico onde há
> > contribuições construtivas das várias frentes de onda (ou de
> > partículas).
>
> Se pensarmos em termos de fótons: poderemos enviar um único fóton
de
> cada vez - sem frentes de partículas.
>
> > É um efeito de conjunto e nunca de uma só emissão ou de
> > emissões de um só elétron. É devido ao fato de estarem na mesma
> > "fase" somente naquele local geométrico a luz que bate naquela
> > região do espelho sob aquele ângulo.
> > Veja novamente:
> > http://tinyurl.com/4hbvk e aperte várias vezes o botão "Slower".
>
> Nesse site, as ondas q. vão para a esquerda são concêntricas - não

> como elas se anularem.
>
> > Meio 1 --> velocidade MÉDIA v1
> > Meio 2 --> velocidade MÉDIA v2
> >
> > Digamos que v2 < v1
> >
> > Imagine dois fótons caminhando lado a lado. Quando o primeiro
chega
>
> Imagino/e dois fotons incidindo *perpendicularmente* 'a
superficie. E
> ai'? Nao ha' reflexao? E como se cancelam as ondas q. se espalham?
No way.
>
> []s,
>
> Roberto Takata





SUBJECT: Re: [ciencialist] POR QUE????
FROM: Luis Brudna <luisbrudna@gmail.com>
TO: ciencialist@yahoogrupos.com.br
DATE: 11/03/2005 19:07

Eu não tenho ódio da quântica. Tenho do mau uso dela.

Tenho asco aos ´frasistas de banheiro´ que usam a quântica em seus
devaneios. :-)

Até
Luís Brudna


On Fri, 11 Mar 2005 15:02:35 -0300 (ART), Leonardo Souza
<leo_feynman@yahoo.com.br> wrote:
>
>
> Por quê tanto ódio com relação à mecânica quântica neste grupo???
>
> Leo
>


SUBJECT: Re: [ciencialist] POR QUE????
FROM: "Alberto Mesquita Filho" <albmesq@uol.com.br>
TO: <ciencialist@yahoogrupos.com.br>
DATE: 11/03/2005 23:21

----- Original Message -----
From: "Leonardo Souza"
Sent: Friday, March 11, 2005 3:02 PM
Subject: [ciencialist] POR QUE????

> Por quê tanto ódio com relação à mecânica quântica neste grupo???

Prezado Leonardo

Já dizia Popper: "As teorias científicas distinguem-se dos mitos unicamente
por serem criticáveis e por estarem abertas a modificações à luz da
crítica." Sob esse aspecto quero crer que ninguém aqui tem respeitado mais a
mecânica quântica, enquanto teoria, quanto eu. Mas a verdade é que para cada
cientista como o Belisário que tenho encontrado pela frente, e a enobrecer o
meu posicionamento crítico (vide o diálogo Teorias Realistas Atuais e o
Paradoxo EPR em http://ecientificocultural.com/ECC2/Dialogos/tra.htm ), têm
também surgido aqui milhares de outros supostos conhecedores da teoria
quântica. Estes últimos, sem contestarem nenhum, repito, nenhum de meus
argumentos, denunciam um certo ar de descaso à repugnância que demonstro não
pela teoria propriamente dita, mas por essa mitificação que se apossou do
Olimpo Acadêmico. É neste sentido que digo que nos dias atuais não existe
mais uma física quântica no estilo proposto por Bohr, Einstein e tantos
outros cientistas destemidos e admiráveis. Muitos físicos dos dias atuais
têm verdadeiro pavor de ver suas idéias desabarem e ficarem sem nada para
colocar no lugar (posto que os grandes teorizadores do século XX já estão
mortos). Têm verdadeira ojeriza por qualquer crítica e/ou tentativa de que
se modifique a receita de bolo que aprenderam na universidade e num
verdadeiro regime de lavagem cerebral. Ora, que dizer desses falsos
cientistas? Isso que propugnam não é ciência, é misticismo, é ficção pura.
Nada contra o misticismo, nada contra a ficção, mesmo porque não sou um
"cético de carteirinha" a "odiar" tudo o que não satisfaça minhas crenças
pessoais. Digo apenas que isso não condiz com um academicismo sadio. Espero
que você não esteja caindo nesse engôdo, apesar de sua mensagem, curta e
grossa, ser bastante sugestiva.

[ ]´s
Alberto
http://ecientificocultural.com/indice.htm
Mas indiferentemente a tudo isso, o neutrino tem massa, o elétron não é
uma carga elétrica coulombiana e a Terra se move. E a história se repetirá.



SUBJECT: Re: O Paradoxo do bombom.
FROM: "anderson_a5" <anderson_a5@yahoo.com.br>
TO: ciencialist@yahoogrupos.com.br
DATE: 12/03/2005 17:52


--- Em ciencialist@yahoogrupos.com.br, Joao Carlos Holland de
Barcellos <jocax@u...> escreveu
>
>
> João Mário Miranda wrote:
> >
> > Sobre a Natureza das Probabilidades
> > As probabilidades em física são frequentemente mal
> > compreendidas. O que queremos dizer quando dizemos que
> > há 50% de probabilidades de um determinado acontecimento
> > ocorrer ? Para se perceber o que quero dizer proponho ao
> > meu leitor um pequeno jogo.
> >
> > http://www.nonio.com/article.php3?sid=75
>
>
> Estava lendo o artigo do Monio ( em que nao concordei ), sobre este
> assunto q sempre me interessou, e lembrei-me de um problema-
paradoxo
> deveras interessante que lhes proponho para divertirem-se.
>
>
> Tres pessoas (P1,P2,P3) fizeram um jogo no qual apenas um deles
> poderia ganhar.
>
> Pediram para P4 ( q nao entrou no jogo ) sortear um numero
> e verificar quem ganhou.
>
> Nenhum dos 3 sabia quem era o ganhador.
> Apenas o sorteador P4 sabia quem era.
>
> P1 pensou: "eu tenho 1/3 (=0,33) de ser o ganhador ja q todos tem
> iguais chances de serem sorteados. Isso significa que as chances
> de eu ganhar sao menores q as de eu perder. Preciso mudar isso !"
>
> Antes que P4 dissesse quem foi o vencedor, P1 diz a P4 :
> "Vc poderia entregar este bombom de consolo a P2 ou a P3 qualquer
> um deles que nao tenha ganhado?"
>
> Entao, apos P4 entregar o bombom a P3 ele pensa :
> "Ahh! agora aumentei minhas chances de ganhar !!!
> Como P3 nao ganhou, o vencedor esta entre eu e P2 e, neste caso,
> como temos a mesma chance de ganhar, eu terei 50% de chances de
vencer.
> Isso eh bem melhor do que os 33% q eu tinha antes de dar o
bombom!! "
>
>
> Pergunta :
> P1 aumentou de fato suas chances de ganhar ( de 1/3 para 1/2 )
> depois que deu o bombom ?
>
> []s
> jocax

>>Bem a rsposta é bastante simples. Como as probabilidades em
matemática podem ser vistas como uma razão entre o número de chances
onde há o "gahno", e o número total de possibilidades, pode-se dizer
que as chances de vitória para P1 aumentaram, pois antes havia uma
chance em três e depois da enrega do bombom as possibilidades totais
se reduziram para uma chance em duas. É como se o participante P3 não
estivesse no jogo desde o começo.





SUBJECT: O Paradoxo do bombom.
FROM: "Oraculo" <oraculo@atibaia.com.br>
TO: <ciencialist@yahoogrupos.com.br>
DATE: 12/03/2005 18:59

Olá Anderson

Vira e mexe, as probabilidades, estatisticas e enigmas como este voltam a lista..:-) E sempre, sempre, causam confusão e discussões acaloradas. A probabilidade, a ciência da estatística, realmente, não é fácil de compreender, já que lidam com conceitos para os quais a evolução não achou necessário nos dotar de compreensão intuitiva (ou mesmo a partir de raciocínios..:-)

Esse problema do sorteio inclusive provocou muita discussão na forma do Problema dos Condenados. Tres prisioneiros condenados seriam alvo de um sorteio, e o ganhador seria libertado, os outros dois mortos. Depois que o Rei sorteou e antes de contar quem seria libertado, um dos prisioneiros ouviu do carcereiro que um dos outros dois prisioneiros não tinha sido o sorteado.

O prisioneiro que ouviu, passou a ter 1/2 de chance ter ganho? Ou continuava com o 1/3 original?

Anderson: >>Bem a rsposta é bastante simples. Como as probabilidades em
matemática podem ser vistas como uma razão entre o número de chances
onde há o "gahno", e o número total de possibilidades, pode-se dizer
que as chances de vitória para P1 aumentaram, pois antes havia uma
chance em três e depois da enrega do bombom as possibilidades totais
se reduziram para uma chance em duas. É como se o participante P3 não
estivesse no jogo desde o começo."

risos..:-) Se a resposta fosse simples, por que toda a confusão?..:-) Porque não é simples. Intuitivamente, parece que agora o P1 tem mais chances. Mas isso só seria verdade se o sorteio não tivesse acontecido. Como no caso dos prisioneiros, o Rei já decidiu, e a chance de qualquer prisioneiro, no momento da decisão, foi (e não é) de 1/3. Saber disso não muda nada.

Pense, se o prisioneiro soubesse pelo carcereiro que ele, o prisioneiro em questão, foi escolhido para ser solto, sua probabilidade seria de 100%? Claro que não..:-) Probabilidades se referem ao momento do sorteio, do evento e da ocorrência, não se referem ao conhecimento que se tem sobre os eventos "a posteriori".

Para reforçar, P1 teve sua probabilidade definida no momento do sorteio. Tudo o que aconteceu depois não importa mais.

O que parece ser a causa desta confusão, é que ao ler a historia de P1, pensamos, se eu fosse apostar, agora sabendo que P3 não ganhou, que chances eu teria de ganhar escolhendo P1? Claro, nesse NOVO jogo, eu teria 1/2 de chance. Assim como P1, se fosse apostar em sí mesmo, depois de saber que P3 não ganhou, teria 1/2 de chance. Mas essa é uma NOVA aposta, um NOVO jogo. No jogo original, vale a chance, a probabilidade estatística, que existe no momento do sorteio, 3 pessoas e 1/3 de chance para cada uma.

Pensemos no prisioneiro que passou a saber que um dos outros dois não foi sorteado. Sua chance de ter sido o escolhido não muda. Mas sua chance de ser solto, que equivale a formular uma NOVA aposta, é de 1/2. Nesse NOVO JOGO, nessa nova aposta, ele está concorrendo apenas com o outro prisioneiro. Mas no momento do sorteio, estava concorrendo com os dois, e portanto sua chance de ganhar sempre foi de 1/3.

Mas sou capaz de apostar que teremos muitas mensagens discutindo este problema antes que todos compreendam a questão corretamente..:-)

Outro problema bem interessante e que causou muita polemica aqui (procure nas mensagens da lista para ter uma ideia de como o pessoal discutiu isso..:-) é parecido. Trata-se do sorteio na TV, com o participante tendo de escolher uma em 3 portas. Uma delas tem um premio alto, as outras duas, nada. Ele escolhe uma delas. Agora, o apresentador abre uma das outras duas restantes, mas sem premio. E pergunta ao participante, você quer mudar ou manter a escolha?

O que será mais vantajoso, mudar ou manter?

O que será mais vantajoso, mudar SEMPRE ou manter SEMPRE?

Embora pareçam a mesma pergunta, não são. A segunda é o principio que baseia o ganho no jogo de poquer. Não se pode ganhar sempre (esqueça os filmes de cinema, onde o mocinho usa sua habilidade no poquer para ganhar a última e mais importante rodada..:-), mas o bom jogador ganha MAIS vezes que perde..:-) E faz isso examinando as chances a longo prazo, e escolher que maximizará suas chances estatísticas.

Para o participante do game, se ele joga todo fim de semana no programa de TV, deve trocar SEMPRE. A longo prazo, ganhará mais vezes que perderá, se fizer isso. Veja se consegue descobrir por que..:-)

Um abraço.

Homero






----- Original Message -----
From: anderson_a5
To: ciencialist@yahoogrupos.com.br
Sent: Saturday, March 12, 2005 5:52 PM
Subject: [ciencialist] Re: O Paradoxo do bombom.



--- Em ciencialist@yahoogrupos.com.br, Joao Carlos Holland de
Barcellos <jocax@u...> escreveu
>
>
> João Mário Miranda wrote:
> >
> > Sobre a Natureza das Probabilidades
> > As probabilidades em física são frequentemente mal
> > compreendidas. O que queremos dizer quando dizemos que
> > há 50% de probabilidades de um determinado acontecimento
> > ocorrer ? Para se perceber o que quero dizer proponho ao
> > meu leitor um pequeno jogo.
> >
> > http://www.nonio.com/article.php3?sid=75
>
>
> Estava lendo o artigo do Monio ( em que nao concordei ), sobre este
> assunto q sempre me interessou, e lembrei-me de um problema-
paradoxo
> deveras interessante que lhes proponho para divertirem-se.
>
>
> Tres pessoas (P1,P2,P3) fizeram um jogo no qual apenas um deles
> poderia ganhar.
>
> Pediram para P4 ( q nao entrou no jogo ) sortear um numero
> e verificar quem ganhou.
>
> Nenhum dos 3 sabia quem era o ganhador.
> Apenas o sorteador P4 sabia quem era.
>
> P1 pensou: "eu tenho 1/3 (=0,33) de ser o ganhador ja q todos tem
> iguais chances de serem sorteados. Isso significa que as chances
> de eu ganhar sao menores q as de eu perder. Preciso mudar isso !"
>
> Antes que P4 dissesse quem foi o vencedor, P1 diz a P4 :
> "Vc poderia entregar este bombom de consolo a P2 ou a P3 qualquer
> um deles que nao tenha ganhado?"
>
> Entao, apos P4 entregar o bombom a P3 ele pensa :
> "Ahh! agora aumentei minhas chances de ganhar !!!
> Como P3 nao ganhou, o vencedor esta entre eu e P2 e, neste caso,
> como temos a mesma chance de ganhar, eu terei 50% de chances de
vencer.
> Isso eh bem melhor do que os 33% q eu tinha antes de dar o
bombom!! "
>
>
> Pergunta :
> P1 aumentou de fato suas chances de ganhar ( de 1/3 para 1/2 )
> depois que deu o bombom ?
>
> []s
> jocax

>>Bem a rsposta é bastante simples. Como as probabilidades em
matemática podem ser vistas como uma razão entre o número de chances
onde há o "gahno", e o número total de possibilidades, pode-se dizer
que as chances de vitória para P1 aumentaram, pois antes havia uma
chance em três e depois da enrega do bombom as possibilidades totais
se reduziram para uma chance em duas. É como se o participante P3 não
estivesse no jogo desde o começo.





##### ##### #####

Para saber mais visite
http://www.ciencialist.hpg.ig.com.br


##### ##### ##### #####


Yahoo! Grupos, um serviço oferecido por:

São Paulo Rio de Janeiro Curitiba Porto Alegre Belo Horizonte Brasília




------------------------------------------------------------------------------
Links do Yahoo! Grupos

a.. Para visitar o site do seu grupo na web, acesse:
http://br.groups.yahoo.com/group/ciencialist/

b.. Para sair deste grupo, envie um e-mail para:
ciencialist-unsubscribe@yahoogrupos.com.br

c.. O uso que você faz do Yahoo! Grupos está sujeito aos Termos do Serviço do Yahoo!.



[As partes desta mensagem que não continham texto foram removidas]



SUBJECT: Bate-papo com Gleiser
FROM: "rmtakata" <rmtakata@altavista.net>
TO: ciencialist@yahoogrupos.com.br
DATE: 12/03/2005 21:00


O UOL promovera' na segunda-feira (14/03), a partir das 19h (horario
de Brasilia), um bate-papo com o fisico Marcelo Gleiser - o mote da
conversa sera' os 100 anos da TRR.

[]s,

Roberto Takata





SUBJECT: Louis de Broglie versus Erwin Schrödinger
FROM: "Ricardo Soares Vieira" <rickrsv@yahoo.com.br>
TO: ciencialist@yahoogrupos.com.br
DATE: 12/03/2005 22:00


Olá a todos do ciencialist!

A mecanica quantica atual provove bastante o nome de Schrödinger, ao
passo que o nome de Louis de Broglie é apenas comentado dizendo que
ele foi o percursor...

Desse modo, dá a entender que o mérito é 90% de Schrödinger e 10% de
L. de Broglie, mas gostaria de oferecer a seguinte leitura a todos
os interessados:



"DISSERTAÇÃO DE MESTRADO Louis de Broglie e as ondas de matéria" de
Pedro-Sergio-Rosa, texto em pdf.

http://ghtc.ifi.unicamp.br/Teses/Pedro-Sergio-Rosa.pdf

Encontrei essa tese na net, procurando no google por "tese louis de
Broglie".

Nela, o autor demonstra que a teoria de L. de Broglie é desde o
inicio relativistica, além de ser determinista e não probabilistica
como na concepção de Schrödinger. Também comenta-se que o autor
sabia que haveria o princípio da incertesa, atribuido atualmente à
Heinsenberg, pois que ele conhecia o fato de que o comprimento de um
grupo de ondas fica sujeito a determinadas "condições de
incertesas"...

Enfim, é um texto muito interessante, e abre uma boa discução: "Qual
vocês acham a teoria que melhor descreve a MQ, a de Schrodinger ou a
de Louis de Broglie?"

p.s. Também vale comentar as formulações de Heisenberg e de
Feynman...


Até mais.

Rick





SUBJECT: Re: [ciencialist] POR QUE????
FROM: JVictor <jvoneto@uol.com.br>
TO: ciencialist@yahoogrupos.com.br
DATE: 13/03/2005 00:52


Muito tempo sem acessar, hoje acessei e tive um esperado susto. Nem digo
a quantidade de e-mails para ler. Mas vou começar pelo seguinte:

Emiliano escreveu: Enfim... era isso que eu tinha para te dizer :-)

Victor: E o disse muito bem, Emiliano. P. Davies sintetizou
coerentemente, mas destaco o seguinte: "São os resultados que contam,
não as pessoas que os produzem." Para argumentos contra fatos, é
necessário fatos para tais argumentos. Fatos experimentais, ancoramento
matemático. Só isso.
Um certo "caro fraco, desdentado e feio, pele e osso simplesmente, quase
sem recheio(C.Veloso)", além de chato e genialmente irônico, disse:
"Posso não concordar com uma só das palavras que dizeis, mas defenderei
até à morte o vosso direito de dizê-las." Bem, como não sou muito
machão, defenderei, sim, mas não até à morte. Besta foi G.Bruno que
virou churrasco mas não arredou de suas idéias e heroi o G.Galilei, que
deu um inteligente "arriê" quando a coisa apertou.

Victor.




E m i l i a n o C h e m e l l o escreveu:

> Olá Leonardo,
>
> Você é novo no grupo? Creio que sim. E não fico surpreso com sua
> primeira impressão. Confesso que também fiquei 'surpreso' com as
> 'espinafradas' do Alberto em relação Mecânica Quântica (lembra Alberto
> quando a gente utilizava os códigos Espinafrar e SPINafrar?, eheheh) .
>
> Mas é difícil separar a emoção da razão daquilo que se gosta ou
> acredita. Não quero dizer com isso que o Alberto e outros estejam deixando
> de lado argumentos e se baseando em devaneios ou crendices próprias. Mas
> cada um tem seu estilo. Coloco aqui, para quem quiser ler, um pequeno
> trecho
> do livro "Física em 6 lições" do Richard Feynman. O texto não é do
> Feynman,
> mas do Paul Davies, o qual fez a introdução do livro.
>
> --- início da citação ---
>
> O público tem uma idéia equivocada de que a ciência é um
> empreendimento
> impessoal, desapaixonado e totalmente objetivo. Enquanto a maioria das
> outras atividades humanas é dominada por modas e personalidades, supõe-se
> que a ciência seja restringida por regras de procedimento consagradas e
> testes rigorosos. São os resultados que contam, não as pessoas que os
> produzem.
>
> Trata-se, é claro, de um disparate. A ciência é uma atividade
> baseada em
> pessoas, como todo empreendimento humano, e igualmente sujeita à moda e ao
> capricho. Neste caso, a moda é ditada menos pela escolha do assunto do que
> pela forma de os cientistas pensarem sobre o mundo. Cada época adota sua
> abordagem particular dos problemas cientificos, geralmente seguindo a
> trilha
> aberta por certas figuras dominantes que definem a agenda e os melhores
> métodos de atacá-la. Ocasionalmente, cientistas adquirem uma estatura
> suficiente para serem notados pelo público em geral, e, quando dotado
> de um
> talento excepcional, um cientista pode se tornar um ícone para toda a
> comunidade científica. Em séculos anteriores, Isaac Newton foi um ícone.
> Newton personificou o cientista cavalheiro - bem relacionado,
> devotadamente
> religioso, calmo e metódico em seu trabalho. Seu estilo de fazer ciência
> fixou o padrão por duzentos anos. Na primeira metade do século XX, Albert
> Einstein substituiu Newton com o ícone cientista popular. Excêntrico,
> descabelado, germânico, distraído, totalmente absorvido em seu
> trabalho e um
> pensador abstrato arquetípico, Einstein mudou a forma de se fazer física,
> questionando os próprios conceitos que definem o assunto.
>
> --- final da citação ---
>
> Enfim... era isso que eu tinha para te dizer :-)
>
> [ ] 's do Emiliano Chemello
> emiliano@quimica.net
> http://www.quimica.net/emiliano
> http://www.ucs.br/ccet/defq/naeq
> [ MSN ] chemelloe@hotmail.com
> [ ICQ ] 145060604
>
> " Rien ne se perd, rien ne se crée,
> tout se transforme."
>
> Antoine Laurent de Lavoisier (químico francês, 1743 - 1794)
>
> ----- Original Message -----
> From: Leonardo Souza
> To: ciencialist@yahoogrupos.com.br
> Sent: Friday, March 11, 2005 3:02 PM
> Subject: [ciencialist] POR QUE????
>
>
>
> Por quê tanto ódio com relação à mecânica quântica neste grupo???
>
> Leo
>
>
> __________________________________________________
> Converse com seus amigos em tempo real com o Yahoo! Messenger
> http://br.download.yahoo.com/messenger/
>
> [As partes desta mensagem que não continham texto foram removidas]
>
>
>
> ##### ##### #####
>
> Para saber mais visite
> http://www.ciencialist.hpg.ig.com.br
>
>
> ##### ##### ##### #####
>
>
> Yahoo! Grupos, um serviço oferecido por:
> PUBLICIDADE
>
>
>
>
>
> Links do Yahoo! Grupos
>
> Para visitar o site do seu grupo na web, acesse:
> http://br.groups.yahoo.com/group/ciencialist/
>
> Para sair deste grupo, envie um e-mail para:
> ciencialist-unsubscribe@yahoogrupos.com.br
>
> O uso que você faz do Yahoo! Grupos está sujeito aos Termos do Serviço do
> Yahoo!.
>
>
>
>
> ##### ##### #####
>
> Para saber mais visite
> http://www.ciencialist.hpg.ig.com.br
>
>
> ##### ##### ##### #####
>
>
> *Yahoo! Grupos, um serviço oferecido por:*
>
> *
> <http://br.rd.yahoo.com/SIG=12aam640b/M=264105.3931087.6562589.1588051/D=brclubs/S=2137111528:HM/EXP=1110652055/A=2361264/R=6/SIG=10v4acpp0/*http://br.shopping.yahoo.com/>*
>
>
>
> ------------------------------------------------------------------------
> *Links do Yahoo! Grupos*
>
> * Para visitar o site do seu grupo na web, acesse:
> http://br.groups.yahoo.com/group/ciencialist/
>
> * Para sair deste grupo, envie um e-mail para:
> ciencialist-unsubscribe@yahoogrupos.com.br
> <mailto:ciencialist-unsubscribe@yahoogrupos.com.br?subject=Unsubscribe>
>
> * O uso que você faz do Yahoo! Grupos está sujeito aos Termos do
> Serviço do Yahoo! <http://br.yahoo.com/info/utos.html>.
>
>
>
>
> __________ Informação do NOD32 1.1024 (20050311) __________
>
> Esta mensagem foi verificada pelo NOD32 Sistema Antivírus
> http://www.nod32.com.br




SUBJECT: Re: POR QUE????
FROM: Maria Natália <grasdic@hotmail.com>
TO: ciencialist@yahoogrupos.com.br
DATE: 13/03/2005 01:37


Leo:

Bem por não me meter nessas discussões quânticas* não quer dizer que
despreze tal área da ciência mas apenas que sendo química me preocupam
outros temas.
Acho que aqui não há ódio. Há discussão saudável mesmo quando às vezes
se vai para cartão amarelo e até "morte súbita". Esta rapaziada da
C-List tem tudo coração de manteiga... e depois Homem de ciência que
se preze tem estômago que até digere pedras.
Foram mais umas metáforas para chatear os que "adoram" a "lusitana" flor.
Ao menos na Química volta e meia há um desatre laboratorial e
desaparecem uns tantos cientistas malucos...na física não se morre
tanto em laboratório
Um abraço, amigo
Maria Natália

*Não sei porquê me lembro de mulheres...Emiliano, ainda tás nessa?


--- Em ciencialist@yahoogrupos.com.br, Leonardo Souza
<leo_feynman@y...> escreveu
>
> Por quê tanto ódio com relação à mecânica quântica neste grupo???
>
> Leo
>
>
> __________________________________________________
> Converse com seus amigos em tempo real com o Yahoo! Messenger
> http://br.download.yahoo.com/messenger/
>
> [As partes desta mensagem que não continham texto foram removidas]





SUBJECT: Re: mulé do Einstein, sexta na TV
FROM: Maria Natália <grasdic@hotmail.com>
TO: ciencialist@yahoogrupos.com.br
DATE: 13/03/2005 01:41


...há pelo menos duas mulhé...a mãe e a mulher, propriamente dita.
Neste caso eram 3.
Pena não ser cá. Mas creio que já vimos há um ano ou dois. Foi falta
de pontaria: deveria ter sido a 8 de Março...
Maria Natália


--- Em ciencialist@yahoogrupos.com.br, "L.E.R.de Carvalho"
<lecarvalho@i...> escreveu
>
> []
> Por trás de todo homem...
>
<http://sender.mailsender.com.br/Redir?0459f8ebb506c02718d4b29b11f586a0&423127752DA5856159EF1867DD277375>"A

> Mulher de Einstein" (GNT, 22h45) conta a história de Mileva Maric,
primeira
> esposa do cientista. O documentário mostra como a contribuição de
Mileva
> para o desenvolvimento da teoria da relatividade e do efeito
fotoelétrico
> foi apagada pela história.
>
>
> [As partes desta mensagem que não continham texto foram removidas]





SUBJECT: Curso de Bruce Lewenstein, sobre Comunicação públi ca da ciência e da tecnologia
FROM: "L.E.R.de Carvalho" <lecarvalho@infolink.com.br>
TO: ciencialist@yahoogrupos.com.br
DATE: 13/03/2005 03:17


>From: "Centro de Estudos do Museu da Vida" <cestudos@coc.fiocruz.br>
>
>
>Curso sobre Comunicação pública da ciência e da tecnologia
>
>Professor: Bruce Lewenstein, Associate Professor of Science Communication,
>Department of Communication Department of Science & Technology Studies,
>Cornell University (Estados Unidos)
>
>Dr. Lewenstein é jornalista e vem se destacando ao longo das últimas três
>décadas no estudo da divulgação científica. Nos Estados Unidos e no campo
>internacional, é considerado um das grandes nomes desse campo e vem se
>dedicando a questões histórica e contemporâneas da divulgação científica.
>Suas reflexões incluem a forma como a ciência é apresentada ao público e
>como o público entende questões controversas.
>
>Mais informações sobre Lewenstein:
><http://www.people.cornell.edu/pages/bvl1>www.people.cornell.edu/pages/bvl1
> e http://www.comm.cornell.edu/directory/lewenstein2.htm
>
>Público-alvo: Destinado a pessoas interessadas no campo de divulgação
>científica. Ministrada pelo Centro de Estudos do Museu da Vida/Casa de
>Oswaldo Cruz/Fiocruz e pelo curso de pós-graduação em Ensino de
>Biociências e Saúde, vale também como crédito para alunos de mestrado e
>doutorado.
>
>Nome da disciplina: Public Communication of Science and Technology
>(Comunicação pública da ciência e da tecnologia)
>
>Ementa da disciplina: Explora a estrutura, os significados e as
>implicações da Comunicação pública da ciência e da tecnologia (PCST, do
>inglês “public communication of science and technology”). Examina os
>contextos em que ocorre a PCST, analisa as motivações e limitações das
>pessoas envolvidas na produção de informações sobre temas de ciência para
>audiências leigas e analisa as funções da PCST.
>
>Carga horária: 15 horas
>
>Dias: 18 e 19 de abril (segunda e terça-feira)
>
>Horário: (a confirmar em breve)
>
>Local: Fiocruz - RJ (auditório a confirmar em breve)
>
>Observação: O curso será ministrado em inglês
>
>Informações: <mailto:cestudos@coc.fiocruz.br>cestudos@coc.fiocruz.br


[As partes desta mensagem que não continham texto foram removidas]



SUBJECT: Re: [ciencialist] Re: mulé do Einstein, sexta na TV
FROM: "Alvaro Augusto \(E\)" <alvaro@electraenergy.com.br>
TO: <ciencialist@yahoogrupos.com.br>
DATE: 13/03/2005 12:50

Três? Que nada! Foi muito mais que isso! Dizem que a casa de Eistein em Caput era praticamente um abatedouro. Ele se engraçou até mesmo com a enteada...

[ ]s

Alvaro Augusto


----- Original Message -----
From: Maria Natália
To: ciencialist@yahoogrupos.com.br
Sent: Sunday, March 13, 2005 1:41 AM
Subject: [ciencialist] Re: mulé do Einstein, sexta na TV



...há pelo menos duas mulhé...a mãe e a mulher, propriamente dita.
Neste caso eram 3.
Pena não ser cá. Mas creio que já vimos há um ano ou dois. Foi falta
de pontaria: deveria ter sido a 8 de Março...
Maria Natália


--- Em ciencialist@yahoogrupos.com.br, "L.E.R.de Carvalho"
<lecarvalho@i...> escreveu
>
> []
> Por trás de todo homem...
>
<http://sender.mailsender.com.br/Redir?0459f8ebb506c02718d4b29b11f586a0&423127752DA5856159EF1867DD277375>"A

> Mulher de Einstein" (GNT, 22h45) conta a história de Mileva Maric,
primeira
> esposa do cientista. O documentário mostra como a contribuição de
Mileva
> para o desenvolvimento da teoria da relatividade e do efeito
fotoelétrico
> foi apagada pela história.
>
>
> [As partes desta mensagem que não continham texto foram removidas]




[As partes desta mensagem que não continham texto foram removidas]



SUBJECT: Re: Foucault / Luz ondas ou corpusculos.
FROM: "Ricardo Soares Vieira" <rickrsv@yahoo.com.br>
TO: ciencialist@yahoogrupos.com.br
DATE: 13/03/2005 13:28


Olá a todos!

Eu pude ver que no memento discutem sobre a óptica de Huygens e de
Newton e achei oportuno dizer que eu desenvolvi uma HIPÓTESE de que
a luz se propaga com mesma velocidade independente do meio de
propagação. Neste trabalho, eu demonstro que os fenômenos da óptica
também podem ser deduzidosa através dessa hipótese e apresento uma
lei que complementa a lei de Snell-Descartes. Quem se interessar, o
texto em pdf está lá no arquivo do grupo, com o nome "velocidade da
luz. pdf"...

Mas vejam, não sei se essa hipótese resiste a experiência, e não
discuto isso, no meu artigo eu apenas quiz demonstrar que a teoria é
matematicamente consistente.

Eu estou escrevendo alguns outros capítulos desse artigo na qual
comento uma formulação ondulatória dessa teoria, que não está
presente nesse texto, assim que eu acabar eu atualizo...

Observem: não quero fazer sensacionalismo entendem, eu apenas levei
em frente uma hipótese teoricamente consistente e que não tinha
ainda sido desenvolvida por ninguém, assim eu peço Quaeso Lege!, ou
seja, não me critiquem antes de lerem o meu trabalho (pois a
primeira vista parece ser uma hipótese de louco né...)

Até mais, quem se interessar fiquem a vontadade para comentar e
criticar, depois de ler, claro...






SUBJECT: Fw: Ebuli��o/press�o
FROM: "Luiz Ferraz Netto" <leobarretos@uol.com.br>
TO: "ciencialist" <ciencialist@yahoogrupos.com.br>
DATE: 13/03/2005 19:44

Quem pode me arrumar isso?

[]'
===========================
Luiz Ferraz Netto [Léo]
leobarretos@uol.com.br
http://www.feiradeciencias.com.br
===========================
-----Mensagem Original-----
De: US BRASIL
Para: leobarretos@uol.com.br
Enviada em: sábado, 12 de março de 2005 11:54
Assunto: Ebulição/pressão


Bom dia!

Meu nome é Altair e vendo produtos para arrefecimento, tenho os seguintes interesses!

Gostaria de obter:

Tabela de ponto de ebulição da agua em relação a pressão em Bar / kg

Tabela de ponto de ebulição do monoetilenoglicol em relação da pressão Bar / kg.


Grato,


Altair

Grato


Nas dúvidas experimentais, por gentileza coloque aqui o endereço da página, isso facilita o confronto. Agradeço. Meu nome é LUIZ FERRAZ NETTO, meu apelido é LÉO e moro em BARRETOS; dai vem meu e-mail: leobarretos@uol.com.br.


--------------------------------------------------------------------------------


Internal Virus Database is out-of-date.
Checked by AVG Anti-Virus.
Version: 7.0.300 / Virus Database: 266.5.0 - Release Date: 25/02/2005

----------

Internal Virus Database is out-of-date.
Checked by AVG Anti-Virus.
Version: 7.0.300 / Virus Database: 266.5.0 - Release Date: 25/02/2005


[As partes desta mensagem que não continham texto foram removidas]



SUBJECT: [ciencialist] Re: Foucault / Luz ondas ou corpusculos.
FROM: "JVictor" <jvoneto@uol.com.br>
TO: <ciencialist@yahoogrupos.com.br>
DATE: 13/03/2005 21:28


Hélio,

Fiquei uns tempos fora. Apesar do atraso, vou tentar responder alguma coisa. Ví que as questões que você coloca são importantes.
Para Victor,

Gostaria de retornar a questão original desta discussão.

Luz: ondas ou cospúsculos?

Você escreveu na msg 44272:
> >...
> >No que tange ao conhecido fenômeno da refração, o
> >que se observa, experimentalmente, com um modelo corpuscular?
> >Velocidade da luz no meio de maior índice, maior que a velocidade
> >da luz no vácuo; explicação: as partículas de luz, ao colidirem
> >com a superfície da água ficam submetidas a uma atração, uma
força
> >normal à
> >superfície no ponto de contato, obrigando as dita cujas
aproximarem-
> >se da normal e esta seria a causa da refração, da mudança de
direção
> >ao sair de um meio e entrar em outro, diferente. Como resultado
desta
> >atração, as partículas sofreriam uma pequena aceleração na
> >superfície,
> >senda esta a causa de sua maior velocidade no meio mais denso.
Isto
> >é o
> >que se obtém com um modelo corpuscular, para o fenômeno refração.
> >...

Hélio: Eu completaria o final assim:
... Isto é o que se obtém com O MODELO CORPUSCULAR DE NEWTON, para o
fenômeno refração.

Victor: Ok. Faltou esse complemento. Tá certo!.

Hélio: A experiência foi contra este modelo (que nem era tão importante
assim para Newton, como nos falou Alberto Mesquita

Victor: Discordo. Era importante, e muito, sim. Pertencia ao cerne de seu pensamento. A teoria corpuscular tinha uma importância capital para Newton.

Hélio: e não foi contra
a teoria corpuscular em geral.

Victor: Concordo. Foi feita com espírito genuinamente científico(o genuínamente é do Mesquita, que tomo emprestado), até em razão das tendências e hipóteses de alguns cientista ao longo das épocas,alguns dos quais já começavam a ver diferenças quanto à maneira de abordar certos eventos luminosos. Com o advento das lentes, telescópios, microscópios, espectográfos, novos efeitos foram sendo observados, que a teoria corpuscular não conseguia explicar. Os experimentos de Arago, Young, Foucault não deixavam dúvidas de que a luz era um fenômeno que apresentava comportamento ondulatório, já com justificativa matematica.

Hélio: No modelo ondulatório em vigor na época a luz também deveria ser mais rápida no meio mais denso.

Victor: Honestamente, não encontrei, ainda, nenhuma referência à informação acima. de qualquer maneira, se alguém chegou a essa conclusão, interpretou alguma coisa de maneira errada.

Hélio: Então vai minha OUTRA pergunta:
Por que a experiência de Foucault não descartou de vez a luz
ondulatória também????

Victor: Não descartou por uma razão que julgo muito forte. A autoridade de Newton era muito grande não só entre pessoas comuns como entre as grandes mentes como Poisson, Biot, Malus(descobridor da polarização por reflexão), Brewster(aquele, do ângulo crtítico, que foi também biógrafo de Newton), só para citar alguns de peso. Com uma caracterísitica. Os dois primeiros eram defensores radicais da teoria corpuscular, anti-éticos até, pois se valiam de suas autoridades para aterrorizar todos quanto não falassem em sua cartilha newtoniana. Há um fato registrado que diz bem do comportamento daquelas pessoas. A Academia de Ciências de Paris naqueles tempos reunia as melhores mentes científicas de então, na França. Havia reuniões periódicas, onde os assuntos novos e velhos era discutidas, com paixão até. Em 1816, um dos membros, Arago, apresentou à Academia um trabalho de Agustin Fresnel sobre difração. Poisson reagiu e afirmou que, "segundo, SEUS CÁLCULOS, SE FRESNEL TIVESSE RAZÃO, aconteceria uma coisa tão abusurda como esta: no centro da sombra projetada de um pequeno disco circular deveria surgir um ponto brilhante." Mas Arago, já convencido de que a luz era um fenômeno ondulatório, e grande de amigo do jovem Fresnel, declarou a Poisson que não sessão seguinte ele iria mostrar se Fresnel tinha ou não razão. Elaborou e montou um aparato experimental com o qual demonstrou que o "resultado absurdo" previsto por Poisson realmente acontecia!. Ante esse juiz implacável e frio, que é um resultado experimental, Poisson inteligentemente reconheceu e sucumbiu às teses de Fresnel. Mas Biot, mesmo vendo o resultado experimental, confirmando as teses de Fresnel, ainda assim, não aceitou. Continou engangado nas teoria de Newton. Paixão assim, só sexual! Nem ante fatos o cara retrocedeu. Putz.
Então fica difícil se mudar algo que está impregnado na mente das pessoas, no insconciente coletivo. De qualquer maneira, creio que, com os conhecimentos que temos hoje, fica difícil imaginarmos como nos comportaríamos na época deles.Os conhecimentos e os quadros de referências da época eram bastante diferentes. Ao menos para mim, não é fácil. Acho que eu, se estivesse na época e fosse um cientista com C maiúsclo, acho que não descartaria. Mas essa resposta é subjetiva e é o máximo que posso fazer. Hoje sabemos que nem uma nem outra é completa em si. O que uma não consegue descrever a outra consegue e vice-versa; como os números obtidos "batem" com as predições das duas, como tudo isto é empregado na tecnologia com extraordinária margem de acerto, há razões para acreditar que, enquanto Mesquita não encontrar outra para substituir as duas, elas vão servindo muito bem.

Hélio: Uma investigação histórica superficial me leva a reafirmar o que
disse em outra msg: Era moda derrubar Newton e tudo era pretexto.
Victor: Não, não concordo com você nesse ponto, pelo menos no que concerne a pessoas como Foucault, Young, Fizeau, cujos trabalhos realmente foram decisivos, e que não se dariam ao "luxo" de ninharias do tipo, pelo que já lí deles. Evidentemente, um ou outro, de capacidade intelectual e moral discutíveis, possa ter tido a motivação "Era moda derrubar Newton e tudo era pretexto". Mas, até onde conheço de história de ciência - que também não é muito - não posso afirmar isso. Se você fizer um acompanhamento de como as coisas aconteceram, certamente não deverá encontrar referências confiáveis a esse respeito. Contudo, paradoxalmente, isto existe hoje, no que se refere às teorias de Einstein, Hawking e outros,pelas mais diversas motivações. É difícil explicar coisas assim. Coisas do homem, suas limitações, suas tradições, ou, o que é o mesmo, sua visão (mental) fechada pelos preconceitos, e pelo que vai lá pelo seus quadros de referências, suas estruturas mentais mesmo. Sabe-se que Newton rejeitava qualquer coisa que desconfiasse tivesse vindo de Hooke ou Huygens, ainda que reconhecesse que estava correta!. Acho que esse vírus propagou-se até os dias de Poisson, Biot e outros.

Hélio:Na msg anterior eu disse com uma certa ironia:
"
> Perfeito! É isto que eu queria saber: a linha de raciocínio que
levou
> aos cientistas da época (com os poucos conhecimentos que tinham) a
> acharem que a dita experiência derrubava a luz corpuscular.
> :-)
> :-)
>
"
Completando, eu digo, que para este "achismo" dos cientista da época
a moda de derrubar Newton contribuiu bastante!!!!


Victor: Como disse, as teses de Fresnel, Young, Foucault não podem ser reduzidas a "achismos". Eram teses robustas e coerentes, pois estribadas numa estruturação matemática que até hoje usamos(Fresnel) e em resultados experimentais incontestáveis, como sabemos. Sabe equação de onda: D^2 S/Dt^2 = v^2 Lap S, com s=s(x,y,z,t)? Resulta da representação matematica de Fresnel! Isto deve ter deixado alguns pensadores algo confusos, na época. Eu digo que Poisson foi um grande matemático, mas um matemático sem convicções, do contrário seus resultados a respeito do desafio que fez, e Arago o desmontou, deveria ter colocado um pulga atrás da orelha. Mas não. Prevaleceu a autoridade de Newton, acima de tudo. Alguns estudiosos afirmam que Newton atrasou o desenvolvimento da ótica por 100 anos. Em concordo. Aliás, sua ótica é um tanto confusa e incoerente em diversos pontos. O contrário acontece com sua mecânica, que continua poderosa e soberba.

Hélio: Se possível comente também o este final da msg anterior:
Mas, depois da física quântica (falo da FQ básica e não o
esoterismo
> de hoje) existe um modelo de como funciona a refração levando em
> consideração as sucessivas absorções e reemissões dos corpúsculos
de
> luz.

Victor: É, Hélio, a teoria ondulatória em si apenas descreve como as ondas se comportam e explicitam ferramentas matemáticas adequadas, que permitem fazer previsões, sem entrar no mérito do porquê. A Teoria quântica já trabalha com outros conhecimentos que permitem uma tentativa de explicação, a nível atômico como os que você referiu acima. Ela chega mais perto do que a gente vive exigindo. Mas, ainda falta muito, para se chegar às verdades essenciais por trás dessa história toda. No caso de ondas sonoras, o mecanismo de funcionamento, grosso modo, é na base da transferência de energia para "camadas" à frente, que empurram a próxima, e assim sucessivamente. A recepção, onde que que ocorra, tímpanos ou um alto-falante, recebe tal energia mecânica e vai prá frente e prá trás do mesmo jeito, que depois da decodificação, tem-se a informação presente no modo de vibração da onda. Num sólido, o movimento não é macroscópico, visível. É feito átomo a átomo. Por isso que a velocidade do som nos sólidos rígidos tende a ser maior que no ar. Que dizer, a própria estrutura do meio facilita o movimento do som. Já para a luz, ou qualquer outra OEM, o meio age sempre "freiando". O modo de vibração de uma OEM é determinante para tal comportamento. Mas, o processo em geral envolve interações atômicas. No caso da da reflexão e da refração, e suas variantes, os processos são precisamente o que você apontou, absorções e reemissões, como os efeitos quânticos que espantam e causam ojeriza em tantas pessoas.
>
Hélio: Veja o final de minha msg 44263.
>
>
> Só quando se leva em conta que a luz é absorvida e depois
reemitida
> pelos átomos do meio que se pode começar a entender a refração.
> Seja ela corpúsculos ou onda.

Victor: É verdade. E isto só corrobora o que se sabe, com embasamento teórico e experimental. A velocidade da luz na água é menor do que no ar. Sempre. Quanto maior for a refringência do meio, menor será essa velocidade. Não há dúvidas, hoje, a esse respeito. Mas é como eu disse acima. Os dois modelos estão corretos, e são complementares. É estranho isso? É. Mas até que fatos novos surjam, temos de ir andando assim mesmo, à espera de mentes iluminadas que vejam o que não se pode enxergar até hoje, embora tudo possa estar a um milésimo de milimetro diante de nosso nariz. Não se fazem mais einsteins como antigamente.


Hélio: É difícil para mim escrever isto, como algo pode SER onda. Para
mim
> onda não é um SER. :-)
Victor: A onda não é um ser. Uma das confusões é associar uma onda(e esse não é o seu caso) a, digamos, uma senoíde, um tobogã. Essas formas apenas indicam como a grandeza associada varia no tempo. Nem mais nem menos que isso. Vejamos um exemplo banal. Uma tensão em uma tomada, só para garantir que vamos ver a coisa em um ponto fixo no espaço. Por causa das caracteríscas físicas dos modos de transmissão da energia elétrica, o que acontece naquela tomada, naquele fim de linha, fixo?
Num dado instante, um osciloscópio indica que o valor da tensão alí é, digamos, zero. Um instante depois, no mesmo ponto, a tensão tem um valor maior, depois maior ainda, atinge um máximo, vai diminuindo, chega a zero novamente, e, a partir daí, começa crescer em sentido contrário, vai a um máximo e volta zero, do mesmo jeito anterior. E daí tudo se repete. O ponto em questão é fixo, é uma tomada de sua casa. Alí não tem onda coisa nenhuma. Mas, se você pegar todos os valores de tensão que você mediu, desde o primeiro valor zero inicial até o segundo valor zero, quando tudo recomeça, e plotar isso num gráfico, o conjunto de pontos vai se dispor numa forma senoidal. Na tomada seguinte, é a mesma coisa, bem como nas tomadas aquí de casa. É como se você pegasse aquela sucessão de tensões(em volts) ocorrendo em um ponto fixo do espaço,mas distintas no tempo, e, "espichando" o tempo, anotasse num eixo vertical valores proporcionais às quantidades medidas para cada instante observado. Quando se confundem essas duas coisas,as atrapalhações no juízo vêm por acréscimo. Em mecânica, pode-se raciocinar do mesmo jeito.

Hélio: Onda, para mim é apenas uma das formas da expressar um determinado
> comportamento da matéria.
Victor: Acho muito vaga a definição acima. Talvez não tenha entendido bem. O que sei (ou penso saber) é que tudo está centrado no conceito de energia, que não sabemos o que é, e nada podemos fazer quanto a isso, a não ser especular, medir, anotar resultados e comparar com a as previsões do aparato teórico. Sabemos sim, como manusear energia, usá-la, transformá-la a contento, para tocar nossa vidinha, construir, destruir, e tudo o mais a que temos direito. Só isso. Há diversas formas de energia, tantas que fica difícil listar. Mas a energia é uma só, apenas se transforma de um tipo em outro. E se conserva. As leis de conservação de energia, falam de energia, mas não do tipo. Pois, segundo essas leis, a democracia prevalece para todas as formas! E o que acontece é que um corpo material, em sí, já uma forma de energia; e reaje de maneiras diferentes a diferentes formas de outras energias. E a energia pode ser transferida de diversas maneiras e modos. O comportamento de um corpo sujeito a isso reflete o tipo de energia que recebe(ou que pode repassar), inclusive "parecendo" tornar-se uma onda, o próprio corpo, como o caso daquela ponte lá nos isteites que até matou gente e causou enormes prejuízos, como todos sabem. Enfim, há muito para se dizer e especular. E também pouco a acrescentar ao que já se conhece.
Na FQ, por exemplo, com respeito ao hélio fluído, o comportamento deste foge, como o diabo foge da cruz, da saia não tão justa desenhada acima. É objeto macroscópico, porém quântico, que não sobe e desce quando uma onda passa, como as ondas na água, mas ele, o próprio hélio ai, sim, é a própria onda!..., sem nadinha daquele exemplo das tensões que dei acima. É estranho, muito estranho, de doido ficar bom. Mas é o que a experiência fornece, como resultado!. Explicações? Nihil.
Mas isso é papo para o papa de FQ, eterno apaixonado, nosso amigo Mesquita. Passo. Com ele a palavra.

Sds,

Victor.


Helio





##### ##### #####

Para saber mais visite
http://www.ciencialist.hpg.ig.com.br


##### ##### ##### #####



Yahoo! Grupos, um serviço oferecido por:

São Paulo Rio de Janeiro Curitiba Porto Alegre Belo Horizonte Brasília




------------------------------------------------------------------------------
Links do Yahoo! Grupos

a.. Para visitar o site do seu grupo na web, acesse:
http://br.groups.yahoo.com/group/ciencialist/

b.. Para sair deste grupo, envie um e-mail para:
ciencialist-unsubscribe@yahoogrupos.com.br

c.. O uso que você faz do Yahoo! Grupos está sujeito aos Termos do Serviço do Yahoo!.



---
Outgoing mail is certified Virus Free.
Checked by AVG anti-virus system (http://www.grisoft.com).
Version: 6.0.859 / Virus Database: 585 - Release Date: 14/02/05


__________ Informação do NOD32 1.1009 (20050226) __________

Esta mensagem foi verificada pelo NOD32 Sistema Antivírus
http://www.nod32.com.br


[As partes desta mensagem que não continham texto foram removidas]



SUBJECT: Re: [ciencialist] Re: Foucault / Luz ondas ou corpusculos.
FROM: "Alberto Mesquita Filho" <albmesq@uol.com.br>
TO: <ciencialist@yahoogrupos.com.br>
DATE: 13/03/2005 22:48

----- Original Message -----
From: "JVictor"
Sent: Sunday, March 13, 2005 9:28 PM
Subject: [ciencialist] Re: Foucault / Luz ondas ou corpusculos.

> > Hélio: A experiência foi contra este modelo (que nem era tão importante
> > assim para Newton, como nos falou Alberto Mesquita

> Victor: Discordo. Era importante, e muito, sim. Pertencia ao cerne de
> seu pensamento. A teoria corpuscular tinha uma importância capital para
> Newton.

A teoria corpuscular sim, o modelo não. Segundo Newton seus corpúsculos
seriam acelerados antes de entrarem no meio denso e portanto ganhariam em
velocidade e concomitantemente mudariam de direção (isso ocorre antes da
entrada no meio denso, e não durante a entrada como supõe a teoria
ondulatória). Daí surgiu a interpretação, por demais ingênua, de que a luz
deveria ser mais veloz no meio denso, como se não houvesse outros fatores a
modificar a velocidade da luz [esse modelo ingênuo é que não é tão
importante para se justificar hoje a Óptica newtoniana -- nos tempos de
Newton ninguém chegava a comentar sobre a possível existência de outros
fatores, mesmo porque não havia como se estimar a velocidade da luz nos
meios densos). Hoje sabe-se que o que diminui a velocidade da luz não é a
sua natureza ondulatória, mas o fato de a luz ser reabsorvida e reemitida
sucessivamente enquanto atravessa o meio denso. Não obstante, entre uma
molécula e outra do meio a luz viaja numa velocidade a contrariar a teoria
do seu ídolo Einstein (ou seja, maior que c) e a compatibilizar-se com a
teoria do meu ídolo Newton.

O interessante é que hoje sabe-se como a luz se propaga num meio denso, mas
continua-se assumindo-se aquela babosela de que o desvio na direção seria
devido à natureza ondulatória. Ou seja, explica-se a redução de velocidade
por uma teoria e a mudança na direção por outra. Essa seria mais uma das
razões a justificar a ficção dualística corpúsculo-onda.

> Os experimentos de Arago, Young, Foucault não deixavam dúvidas de que a
> luz era um fenômeno que apresentava comportamento ondulatório, já com
> justificativa matematica.

Não é o que dizia o seu ídolo: "A história da busca de uma teoria da luz não
está de modo algum concluída. O veredicto do século XIX não foi final e
definitivo. Todo o problema de decidir entre corpúsculos e ondas ainda
existe para a Física moderna, desta vez de uma forma muito mais profunda e
intrincada. Aceitemos a derrota da teoria corpuscular da luz até
reconhecermos a natureza problemática da vitória da teoria ondulatória."
[Einstein e Infeld (A Evolução da Física)]

> > Hélio: Se possível comente também o este final da msg anterior: Mas,
> > depois da física quântica (falo da FQ básica e não o esoterismo de hoje)
> > existe um modelo de como funciona a refração levando em consideração as
> > sucessivas absorções e reemissões dos corpúsculos de luz.

> Victor: É, Hélio, a teoria ondulatória em si apenas descreve como as
> ondas se comportam e explicitam ferramentas matemáticas adequadas, que
> permitem fazer previsões, sem entrar no mérito do porquê. A Teoria
> quântica já trabalha com outros conhecimentos que permitem uma tentativa
> de explicação, a nível atômico como os que você referiu acima. Ela chega
> mais perto do que a gente vive exigindo. Mas, ainda falta muito, para se
> chegar às verdades essenciais por trás dessa história toda. No caso de
> ondas sonoras, o mecanismo de funcionamento, grosso modo, é na base da
> transferência de energia para "camadas" à frente, que empurram a próxima,
> e assim sucessivamente. A recepção, onde que que ocorra, tímpanos ou um
> alto-falante, recebe tal energia mecânica e vai prá frente e prá trás do
> mesmo jeito, que depois da decodificação, tem-se a informação presente no
> modo de vibração da onda. Num sólido, o movimento não é macroscópico,
> visível. É feito átomo a átomo. Por isso que a velocidade do som nos
> sólidos rígidos tende a ser maior que no ar. Que dizer, a própria
> estrutura do meio facilita o movimento do som. Já para a luz, ou qualquer
> outra OEM, o meio age sempre "freiando". O modo de vibração de uma OEM é
> determinante para tal comportamento. Mas, o processo em geral envolve
> interações atômicas. No caso da da reflexão e da refração, e suas
> variantes, os processos são precisamente o que você apontou, absorções e
> reemissões, como os efeitos quânticos que espantam e causam ojeriza em
> tantas pessoas.

Neste caso fica faltando explicar como ocorre, segundo a teoria ondulatória,
a mudança de direção dos raios de luz. De duas, uma: ou a mudança de
velocidade ocorre simultaneamente à passagem de um meio para o outro, como
assumiram os físicos ondulatórios do século XIX, ou então seria devida ao
fato de "o meio agindo sempre freiando", como você afirmou. Ou eu muito me
engano, ou este segundo caso incompatibiliza-se com a teoria ondulatória
aceita no século XIX. Sem dúvida alguma, aceitar gratuitamente esse tipo de
dualidade me espanta e me causa ojeriza, assim como qualquer outro tipo de
ficção que se pretenda encarar como de valor científico insofismável.

> > Hélio: Veja o final de minha msg 44263. Só quando se leva em conta que a
> > luz é absorvida e depois reemitida pelos átomos do meio que se pode
> > começar a entender a refração. Seja ela corpúsculos ou onda.

Falou e disse, mas não creio que você encontrará uma resposta satisfatória
em meio aos livros de ficção, digo, de física "moderna".

> Victor: Na FQ, por exemplo, com respeito ao hélio fluído, o comportamento
> deste foge, como o diabo foge da cruz, da saia não tão justa desenhada
> acima. É objeto macroscópico, porém quântico, que não sobe e desce quando
> uma onda passa, como as ondas na água, mas ele, o próprio hélio ai, sim, é
> a própria onda!..., sem nadinha daquele exemplo das tensões que dei acima.
> É estranho, muito estranho, de doido ficar bom. Mas é o que a experiência
> fornece, como resultado!. Explicações? Nihil. Mas isso é papo para o papa
> de FQ, eterno apaixonado, nosso amigo Mesquita. Passo. Com ele a palavra.

Pô, mas a ficção quântica é tão simples, tão elementar, tão lógica!!! Não
sei nem como começar a explicar uma coisa tão banal. É só introduzir umas
duas ou três hipóteses "ad hoc" e tudo se esclarecerá. Pô! Será que você
andou cabulando as aulas de ficção quântica no tempo de universidade?

[ ]´s
Alberto
http://ecientificocultural.com/indice.htm
Mas indiferentemente a tudo isso, o neutrino tem massa, o elétron não é
uma carga elétrica coulombiana e a Terra se move. E a história se repetirá.



SUBJECT: A polêmica do cordão umbilical
FROM: "Cyberlander" <cybernews@superig.com.br>
TO: <Undisclosed-Recipient:;>
DATE: 14/03/2005 02:42

CIÊNCIA

A polêmica do cordão

O Ministério da Saúde alerta para a ilusão de guardar cordão umbilical para uso da própria criança

VALÉRIA BLANC - revista Época


Raphael Falavigna/ÉPOCA

PROCURA
Tanque de nitrogênio no CordVida tem 500 cordões
Os avanços científicos que envolvem terapias para o uso de células-tronco levaram ao surgimento de um negócio que é novo no país, mas já causa polêmica: os bancos privados de armazenamento de sangue de cordão umbilical. O sangue do cordão possui células-tronco, que podem vir a ter várias utilidades no futuro, mas atualmente têm uma única aplicação principal - produzir material para substituir o transplante da medula óssea. A polêmica é sobre a estratégia de armazenamento: em bancos particulares, onde só quem pode ter acesso ao material é o usuário ou um parente, ou públicos, onde é doado a quem necessitar. O Ministério da Saúde adverte que guardar o cordão do bebê em laboratórios particulares é praticamente inútil. ''A possibilidade de usar o cordão para a própria criança é mínima: de 1 para 20 mil'', avisa João Paulo Baccara, coordenador de Política Nacional de Sangue e Hemoderivados. ''A célula-tronco da criança traz todos os eventuais problemas que aquele adulto desejará tratar no futuro.'' Baccara defende a doação do cordão para os bancos públicos da rede BrasilCord, lançada em setembro. Ali, qualquer paciente na fila de transplante que achar uma amostra compatível poderá usar o sangue, sem pagar nada. No momento, ele serve para tratar leucemias. É possível ainda o uso em pacientes com doenças hematológicas e imunológicas.



Sandra Lopes/Ed. Globo

''A Ciência avança rápido e qualquer chance que se tem à mão vale usar por um filho. Sou um incentivador''

MARCELO SERRADO, ator, sobre a decisão tomada com a mulher, Rafaella Mandelli, de congelar o cordão da filha, Catarina, em banco privado

A discussão é acalorada entre representantes de lado a lado. Os bancos particulares têm seus argumentos. ''Deve-se pensar que os pais estarão beneficiando não só a criança, mas a família. Se for descoberto um câncer hereditário, a criança não poderá usar seu cordão. Mas, se um irmão o tiver armazenado, será um doador com maior chance de compatibilidade'', diz Lygia da Veiga Pereira, coordenadora de um banco privado. A demanda nos cinco bancos particulares em funcionamento no Brasil tem aumentado. Nos últimos meses, o movimento no paulista CordVida cresceu 25%. Lá, a armazenagem custa R$ 4 mil e a manutenção R$ 699 por ano. A procura pelo banco carioca Cryopraxis aumenta 30% ao mês. Pagam-se R$ 4.435 pelo armazenamento e R$ 584 pela manutenção.

Fora da disputa entre público e privado, Patrícia Pranke, médica do Laboratório de Células-Tronco e Banco Público de Sangue de Cordão Umbilical do New York Blood Center, em Nova York, opina: ''Deve-se entender um banco de sangue de cordão umbilical público como se fosse um banco de sangue normal. Se alguém sofre um acidente hoje, não é preciso que tenha guardado seu sangue na semana passada, porque poderá usar o de um doador'', diz. Para ter idéia, até 2003 apenas cinco transplantes foram realizados no mundo com sangue do cordão do próprio bebê. Há ainda outro forte argumento. ''O volume de sangue de um cordão é suficiente apenas para um paciente de até 50 quilos, ou seja, pouquíssimos adultos. Nos bancos públicos, podem-se juntar amostras de dois doadores compatíveis'', acrescenta Patrícia.



Marcelo Correa/ÉPOCA

EM NOME DE TODOS
Bouzas, coordenador do banco público do Inca, anuncia novos cinco transplantes com sangue do instituto
A rede pública tem duas unidades em operação: no Instituto Nacional do Câncer (Inca), no Rio de Janeiro, e no Hospital Albert Einstein, em São Paulo. No ano passado, o Inca realizou um transplante de medula óssea com células de cordão guardadas em seu banco. ''A genética diz que são de 25% as chances de encontrar doador compatível nas famílias. Daí a importância de ter um banco com células de outras pessoas'', argumenta Luis Fernando Bouzas, diretor do Centro de Transplantes de Medula Óssea do Inca. O instituto está prestes a realizar outros cinco transplantes a partir de material dos cordões estocados. No exterior, essa polêmica começou mais cedo e produziu veredictos. Em 2004, o comitê de ética europeu declarou achar desnecessário o armazenamento do cordão umbilical dos filhos para uso próprio. A França, por exemplo, proibiu bancos privados de cordão umbilical por considerá-los improdutivos. A Itália e a Bélgica tomaram a mesma decisão.

Como no Brasil os dois bancos existem paralelamente, os laboratórios privados ''vendem'' o negócio como uma espécie de seguro de vida, uma aposta nas futuras pesquisas. Eles têm clientes como o apresentador César Filho e a modelo Joanna Prado, que congelaram ali, em nitrogênio líquido, o cordão umbilical dos filhos. O ator Marcelo Serrado e a mulher, Rafaela Mandelli, fizeram o mesmo no CordVida. ''Fomos avisados da baixa probabilidade de uso. Mas a Ciência avança rápido e qualquer chance que se tem à mão vale usar por um filho'', declara Marcelo. De qualquer maneira, a aplicação de novos resultados de pesquisas com células-tronco no Brasil depende de sua evolução, agora que foi aprovada na Câmara dos Deputados a Lei de Biossegurança.



[ ]'s

D.C.




CYBERLANDER

Ama a realidade que constróis,
que nem a morte deterá teu voo · ·




[As partes desta mensagem que não continham texto foram removidas]



SUBJECT: Fw: duvidas
FROM: "Luiz Ferraz Netto" <leobarretos@uol.com.br>
TO: "ciencialist" <ciencialist@yahoogrupos.com.br>
DATE: 14/03/2005 07:12

Quem tem esse procedimento pronto para ser usado numa piscina?
[]'
===========================
Luiz Ferraz Netto [Léo]
leobarretos@uol.com.br
http://www.feiradeciencias.com.br
===========================
-----Mensagem Original-----
De: Clinica
Para: leobarretos@uol.com.br
Enviada em: quarta-feira, 9 de março de 2005 10:44
Assunto: duvidas


Professor


Tenho uma loja de piscinas, e gostaria de saber como faço para calcular a evaporação de agua de uma piscina.....

Moro em Cascavel-Pr.

agradeceria se pudesse me ajudar...

Paulo Cesar Justino
(45)223-4222









--------------------------------------------------------------------------------


Internal Virus Database is out-of-date.
Checked by AVG Anti-Virus.
Version: 7.0.300 / Virus Database: 266.5.0 - Release Date: 25/02/2005

----------

Internal Virus Database is out-of-date.
Checked by AVG Anti-Virus.
Version: 7.0.300 / Virus Database: 266.5.0 - Release Date: 25/02/2005


[As partes desta mensagem que não continham texto foram removidas]



SUBJECT: Fw: batatas e lampada
FROM: "Luiz Ferraz Netto" <leobarretos@uol.com.br>
TO: "ciencialist" <ciencialist@yahoogrupos.com.br>
DATE: 14/03/2005 07:48

Que tal essa?
[]'
===========================
Luiz Ferraz Netto [Léo]
leobarretos@uol.com.br
http://www.feiradeciencias.com.br
===========================
-----Mensagem Original-----
De: "laurie kuniyoshi" <meb_12_3_04@hotmail.com>
Para: <leobarretos@uol.com.br>
Enviada em: quarta-feira, 9 de março de 2005 22:56


| Oi, eu estava vendo sua pagina, onde tem a experiencia :Uma bateria elétrica
| de batatas. Então queria te fazer uam pergunta, quantas meias batatas seriam
| necessarias para acender uma lampada 40 / 127???
| me responda por favor
|
| _________________________________________________________________
| Chegou o que faltava: MSN Acesso Grátis. Instale Já!
| http://www.msn.com.br/discador
|
|
|
| --
| Internal Virus Database is out-of-date.
| Checked by AVG Anti-Virus.
| Version: 7.0.300 / Virus Database: 266.5.0 - Release Date: 25/02/2005
|
|


--
Internal Virus Database is out-of-date.
Checked by AVG Anti-Virus.
Version: 7.0.300 / Virus Database: 266.5.0 - Release Date: 25/02/2005



SUBJECT: Fw: batatas e lampada
FROM: "Luiz Ferraz Netto" <leobarretos@uol.com.br>
TO: "ciencialist" <ciencialist@yahoogrupos.com.br>
DATE: 14/03/2005 07:56

Enviei uma resposta para a Laurie ... muito a grosso modo...:
======================
Olá Laurie,

para acender uma lâmpada de 40W sob tensão de 127VDC usando as batatas, precisaríamos de; vejamos:

Supondo que cada meia-batata forneça 0,63 V, serão necessárias duas meias-batatas ligadas em série para obter 1,27 VDC.
Como queremos 127 VDC, precisaremos de 200 meias-batatas.
200 meias-batatas ligadas em série resolvem o problema da tensão; temos os 127 V ... mas a lâmpada não acenderá porque ela precisa de corrente de, pelo menos 40W/127V = 0,32 A, coisa de 320 mA ... e como cada meia batata só fornece uns míseros 32microampère, vc deverá colocar associações em paralelo para reforçar a corrente. Cada associação em série teria 200 meias-batatas ou 100 batatas e será preciso pelo menos 1000 dessas series associadas em paralelo, ou seja, umas 100.000 batatas!

|----||-----||------||-------||--- 200 meias-batatas---||-----||----|
|----||-----||------||-------||--- 200 meias-batatas---||-----||----|
|----||-----||------||-------||--- 200 meias-batatas---||-----||----|
|..... |
|.....
|......... 1000 dessas séries em paralelo .....
|.....
|.....
|----||-----||------||-------||--- 200 meias-batatas---||-----||----|
|----||-----||------||-------||--- 200 meias-batatas---||-----||----|
|----||-----||------||-------||--- 200 meias-batatas---||-----||----|
|----||-----||------||-------||--- 200 meias-batatas---||-----||----|
| |
|------------ aqui liga-se a lâmpada de 40W---------------|

NOTA: Com isso, a lâmpada de 40W permanecerá acesa por .... uns 2 minutos!

aquele abraço,|

===========================
Luiz Ferraz Netto [Léo]
leobarretos@uol.com.br
http://www.feiradeciencias.com.br
===========================
-----Mensagem Original-----
De: "laurie kuniyoshi" <meb_12_3_04@hotmail.com>
Para: <leobarretos@uol.com.br>
Enviada em: quarta-feira, 9 de março de 2005 22:56


| Oi, eu estava vendo sua pagina, onde tem a experiencia :Uma bateria elétrica
| de batatas. Então queria te fazer uam pergunta, quantas meias batatas seriam
| necessarias para acender uma lampada 40 / 127???
| me responda por favor
|
| _________________________________________________________________
| Chegou o que faltava: MSN Acesso Grátis. Instale Já!
| http://www.msn.com.br/discador
|
|
|
| --
| Internal Virus Database is out-of-date.
| Checked by AVG Anti-Virus.
| Version: 7.0.300 / Virus Database: 266.5.0 - Release Date: 25/02/2005
|
|


--
Internal Virus Database is out-of-date.
Checked by AVG Anti-Virus.
Version: 7.0.300 / Virus Database: 266.5.0 - Release Date: 25/02/2005



SUBJECT: Fw: du.alva
FROM: "Luiz Ferraz Netto" <leobarretos@uol.com.br>
TO: "ciencialist" <ciencialist@yahoogrupos.com.br>
DATE: 14/03/2005 08:05

urgência!
[]'
===========================
Luiz Ferraz Netto [Léo]
leobarretos@uol.com.br
http://www.feiradeciencias.com.br
===========================
-----Mensagem Original-----
De: "du.alva" <du.alva@ig.com.br>
Para: <leobarretos@uol.com.br>
Enviada em: quarta-feira, 9 de março de 2005 23:58


Em 09/03/2005, Dulcilene escreve.

Leo ou Prof.Luiz Ferraz Netto?

Entrei no site da feira de ciencias e la vi seu email.
Minha filha esta fazendo um trabalho escolar sobre medidas.Ela tem 6 item
que deverá pesquisar ou fazer ela mesmo.

Alguns,ela fez, mas três itens está difícil resolver, e eu já não me lembro
mais como fazer. Gostaria que me ajudasse.

São eles:

1-A massa de uma gota de água;
2-O tempo que uma formiga leva para percorrer em linha reta a distância de 1
metro;
3-O volume de uma lata de leite condensado.

Obrigada por sua atenção e espero que possa me ajudar.

O trabalho é para a semana que vem.

DULCILENE

Cheguei a imprimir o desenho da representação esquemática de uma gota de
água.



--------------------------------------------------------------------------------


Internal Virus Database is out-of-date.
Checked by AVG Anti-Virus.
Version: 7.0.300 / Virus Database: 266.5.0 - Release Date: 25/02/2005



--
Internal Virus Database is out-of-date.
Checked by AVG Anti-Virus.
Version: 7.0.300 / Virus Database: 266.5.0 - Release Date: 25/02/2005



SUBJECT: Fw: press�o
FROM: "Luiz Ferraz Netto" <leobarretos@uol.com.br>
TO: "ciencialist" <ciencialist@yahoogrupos.com.br>
DATE: 14/03/2005 08:24

Decifrar isso ..........
[]'
===========================
Luiz Ferraz Netto [Léo]
leobarretos@uol.com.br
http://www.feiradeciencias.com.br
===========================
-----Mensagem Original-----
De: Hermes Andrade
Para: leobarretos@uol.com.br
Enviada em: quinta-feira, 10 de março de 2005 08:40
Assunto: pressão


Prezado professor:
Ante de mais nada, parabenizo-lhe pelo belo trabalho.

Minha pergunta:

se eu tenho um recipiente que pesa 20Kg e com saídas controláveis, quanto de pressão interna eu preciso para elevar este recipiente a 3 metros de altura ?

Desde já agradeço pela atenção;

Hermes


--------------------------------------------------------------------------------
Yahoo! Acesso Grátis - Internet rápida e grátis. Instale o discador do Yahoo! agora.


--------------------------------------------------------------------------------


Internal Virus Database is out-of-date.
Checked by AVG Anti-Virus.
Version: 7.0.300 / Virus Database: 266.5.0 - Release Date: 25/02/2005

----------

Internal Virus Database is out-of-date.
Checked by AVG Anti-Virus.
Version: 7.0.300 / Virus Database: 266.5.0 - Release Date: 25/02/2005


[As partes desta mensagem que não continham texto foram removidas]



SUBJECT: Re: [ciencialist] Re: Foucault / Luz ondas ou corpusculos.
FROM: JVictor <jvoneto@uol.com.br>
TO: ciencialist@yahoogrupos.com.br
DATE: 14/03/2005 10:31

Correção: onde se lê "...acho que NÃO descartaria", leia-se: "...*acho*
que descartaria".

Victor.





JVictor escreveu:

>
> Hélio,
>
> Fiquei uns tempos fora. Apesar do atraso, vou tentar responder alguma
> coisa. Ví que as questões que você coloca são importantes.
> Para Victor,
>
> Gostaria de retornar a questão original desta discussão.
>
> Luz: ondas ou cospúsculos?
>
> Você escreveu na msg 44272:
> > >...
> > >No que tange ao conhecido fenômeno da refração, o
> > >que se observa, experimentalmente, com um modelo corpuscular?
> > >Velocidade da luz no meio de maior índice, maior que a velocidade
> > >da luz no vácuo; explicação: as partículas de luz, ao colidirem
> > >com a superfície da água ficam submetidas a uma atração, uma
> força
> > >normal à
> > >superfície no ponto de contato, obrigando as dita cujas
> aproximarem-
> > >se da normal e esta seria a causa da refração, da mudança de
> direção
> > >ao sair de um meio e entrar em outro, diferente. Como resultado
> desta
> > >atração, as partículas sofreriam uma pequena aceleração na
> > >superfície,
> > >senda esta a causa de sua maior velocidade no meio mais denso.
> Isto
> > >é o
> > >que se obtém com um modelo corpuscular, para o fenômeno refração.
> > >...
>
> Hélio: Eu completaria o final assim:
> ... Isto é o que se obtém com O MODELO CORPUSCULAR DE NEWTON, para o
> fenômeno refração.
>
> Victor: Ok. Faltou esse complemento. Tá certo!.
>
> Hélio: A experiência foi contra este modelo (que nem era tão importante
> assim para Newton, como nos falou Alberto Mesquita
>
> Victor: Discordo. Era importante, e muito, sim. Pertencia ao cerne
> de seu pensamento. A teoria corpuscular tinha uma importância capital
> para Newton.
>
> Hélio: e não foi contra
> a teoria corpuscular em geral.
>
> Victor: Concordo. Foi feita com espírito genuinamente científico(o
> genuínamente é do Mesquita, que tomo emprestado), até em razão das
> tendências e hipóteses de alguns cientista ao longo das épocas,alguns
> dos quais já começavam a ver diferenças quanto à maneira de abordar
> certos eventos luminosos. Com o advento das lentes, telescópios,
> microscópios, espectográfos, novos efeitos foram sendo observados, que
> a teoria corpuscular não conseguia explicar. Os experimentos de Arago,
> Young, Foucault não deixavam dúvidas de que a luz era um fenômeno que
> apresentava comportamento ondulatório, já com justificativa
> matematica.
>
> Hélio: No modelo ondulatório em vigor na época a luz também deveria
> ser mais rápida no meio mais denso.
>
> Victor: Honestamente, não encontrei, ainda, nenhuma referência à
> informação acima. de qualquer maneira, se alguém chegou a essa
> conclusão, interpretou alguma coisa de maneira errada.
>
> Hélio: Então vai minha OUTRA pergunta:
> Por que a experiência de Foucault não descartou de vez a luz
> ondulatória também????
>
> Victor: Não descartou por uma razão que julgo muito forte. A
> autoridade de Newton era muito grande não só entre pessoas comuns como
> entre as grandes mentes como Poisson, Biot, Malus(descobridor da
> polarização por reflexão), Brewster(aquele, do ângulo crtítico, que
> foi também biógrafo de Newton), só para citar alguns de peso. Com uma
> caracterísitica. Os dois primeiros eram defensores radicais da teoria
> corpuscular, anti-éticos até, pois se valiam de suas autoridades para
> aterrorizar todos quanto não falassem em sua cartilha newtoniana. Há
> um fato registrado que diz bem do comportamento daquelas pessoas. A
> Academia de Ciências de Paris naqueles tempos reunia as melhores
> mentes científicas de então, na França. Havia reuniões periódicas,
> onde os assuntos novos e velhos era discutidas, com paixão até. Em
> 1816, um dos membros, Arago, apresentou à Academia um trabalho de
> Agustin Fresnel sobre difração. Poisson reagiu e afirmou que,
> "segundo, SEUS CÁLCULOS, SE FRESNEL TIVESSE RAZÃO, aconteceria uma
> coisa tão abusurda como esta: no centro da sombra projetada de um
> pequeno disco circular deveria surgir um ponto brilhante." Mas Arago,
> já convencido de que a luz era um fenômeno ondulatório, e grande de
> amigo do jovem Fresnel, declarou a Poisson que não sessão seguinte ele
> iria mostrar se Fresnel tinha ou não razão. Elaborou e montou um
> aparato experimental com o qual demonstrou que o "resultado absurdo"
> previsto por Poisson realmente acontecia!. Ante esse juiz implacável e
> frio, que é um resultado experimental, Poisson inteligentemente
> reconheceu e sucumbiu às teses de Fresnel. Mas Biot, mesmo vendo o
> resultado experimental, confirmando as teses de Fresnel, ainda assim,
> não aceitou. Continou engangado nas teoria de Newton. Paixão assim, só
> sexual! Nem ante fatos o cara retrocedeu. Putz.
> Então fica difícil se mudar algo que está impregnado na mente das
> pessoas, no insconciente coletivo. De qualquer maneira, creio que,
> com os conhecimentos que temos hoje, fica difícil imaginarmos como
> nos comportaríamos na época deles.Os conhecimentos e os quadros de
> referências da época eram bastante diferentes. Ao menos para mim, não
> é fácil. Acho que eu, se estivesse na época e fosse um cientista com C
> maiúsclo, acho que não descartaria. Mas essa resposta é subjetiva e é
> o máximo que posso fazer. Hoje sabemos que nem uma nem outra é
> completa em si. O que uma não consegue descrever a outra consegue e
> vice-versa; como os números obtidos "batem" com as predições das duas,
> como tudo isto é empregado na tecnologia com extraordinária margem de
> acerto, há razões para acreditar que, enquanto Mesquita não encontrar
> outra para substituir as duas, elas vão servindo muito bem.
>
> Hélio: Uma investigação histórica superficial me leva a reafirmar o que
> disse em outra msg: Era moda derrubar Newton e tudo era pretexto.
> Victor: Não, não concordo com você nesse ponto, pelo menos no que
> concerne a pessoas como Foucault, Young, Fizeau, cujos trabalhos
> realmente foram decisivos, e que não se dariam ao "luxo" de ninharias
> do tipo, pelo que já lí deles. Evidentemente, um ou outro, de
> capacidade intelectual e moral discutíveis, possa ter tido a motivação
> "Era moda derrubar Newton e tudo era pretexto". Mas, até onde conheço
> de história de ciência - que também não é muito - não posso afirmar
> isso. Se você fizer um acompanhamento de como as coisas aconteceram,
> certamente não deverá encontrar referências confiáveis a esse
> respeito. Contudo, paradoxalmente, isto existe hoje, no que se refere
> às teorias de Einstein, Hawking e outros,pelas mais diversas
> motivações. É difícil explicar coisas assim. Coisas do homem, suas
> limitações, suas tradições, ou, o que é o mesmo, sua visão (mental)
> fechada pelos preconceitos, e pelo que vai lá pelo seus quadros de
> referências, suas estruturas mentais mesmo. Sabe-se que Newton
> rejeitava qualquer coisa que desconfiasse tivesse vindo de Hooke ou
> Huygens, ainda que reconhecesse que estava correta!. Acho que esse
> vírus propagou-se até os dias de Poisson, Biot e outros.
>
> Hélio:Na msg anterior eu disse com uma certa ironia:
> "
> > Perfeito! É isto que eu queria saber: a linha de raciocínio que
> levou
> > aos cientistas da época (com os poucos conhecimentos que tinham) a
> > acharem que a dita experiência derrubava a luz corpuscular.
> > :-)
> > :-)
> >
> "
> Completando, eu digo, que para este "achismo" dos cientista da época
> a moda de derrubar Newton contribuiu bastante!!!!
>
>
> Victor: Como disse, as teses de Fresnel, Young, Foucault não podem
> ser reduzidas a "achismos". Eram teses robustas e coerentes, pois
> estribadas numa estruturação matemática que até hoje usamos(Fresnel) e
> em resultados experimentais incontestáveis, como sabemos. Sabe equação
> de onda: D^2 S/Dt^2 = v^2 Lap S, com s=s(x,y,z,t)? Resulta da
> representação matematica de Fresnel! Isto deve ter deixado alguns
> pensadores algo confusos, na época. Eu digo que Poisson foi um grande
> matemático, mas um matemático sem convicções, do contrário seus
> resultados a respeito do desafio que fez, e Arago o desmontou, deveria
> ter colocado um pulga atrás da orelha. Mas não. Prevaleceu a
> autoridade de Newton, acima de tudo. Alguns estudiosos afirmam que
> Newton atrasou o desenvolvimento da ótica por 100 anos. Em concordo.
> Aliás, sua ótica é um tanto confusa e incoerente em diversos pontos. O
> contrário acontece com sua mecânica, que continua poderosa e soberba.
>
> Hélio: Se possível comente também o este final da msg anterior:
> Mas, depois da física quântica (falo da FQ básica e não o
> esoterismo
> > de hoje) existe um modelo de como funciona a refração levando em
> > consideração as sucessivas absorções e reemissões dos corpúsculos
> de
> > luz.
>
> Victor: É, Hélio, a teoria ondulatória em si apenas descreve como
> as ondas se comportam e explicitam ferramentas matemáticas adequadas,
> que permitem fazer previsões, sem entrar no mérito do porquê. A Teoria
> quântica já trabalha com outros conhecimentos que permitem uma
> tentativa de explicação, a nível atômico como os que você referiu
> acima. Ela chega mais perto do que a gente vive exigindo. Mas, ainda
> falta muito, para se chegar às verdades essenciais por trás dessa
> história toda. No caso de ondas sonoras, o mecanismo de funcionamento,
> grosso modo, é na base da transferência de energia para "camadas" à
> frente, que empurram a próxima, e assim sucessivamente. A recepção,
> onde que que ocorra, tímpanos ou um alto-falante, recebe tal energia
> mecânica e vai prá frente e prá trás do mesmo jeito, que depois da
> decodificação, tem-se a informação presente no modo de vibração da
> onda. Num sólido, o movimento não é macroscópico, visível. É feito
> átomo a átomo. Por isso que a velocidade do som nos sólidos rígidos
> tende a ser maior que no ar. Que dizer, a própria estrutura do meio
> facilita o movimento do som. Já para a luz, ou qualquer outra OEM, o
> meio age sempre "freiando". O modo de vibração de uma OEM é
> determinante para tal comportamento. Mas, o processo em geral envolve
> interações atômicas. No caso da da reflexão e da refração, e suas
> variantes, os processos são precisamente o que você apontou, absorções
> e reemissões, como os efeitos quânticos que espantam e causam ojeriza
> em tantas pessoas.
> >
> Hélio: Veja o final de minha msg 44263.
> >
> >
> > Só quando se leva em conta que a luz é absorvida e depois
> reemitida
> > pelos átomos do meio que se pode começar a entender a refração.
> > Seja ela corpúsculos ou onda.
>
> Victor: É verdade. E isto só corrobora o que se sabe, com
> embasamento teórico e experimental. A velocidade da luz na água é
> menor do que no ar. Sempre. Quanto maior for a refringência do meio,
> menor será essa velocidade. Não há dúvidas, hoje, a esse respeito. Mas
> é como eu disse acima. Os dois modelos estão corretos, e são
> complementares. É estranho isso? É. Mas até que fatos novos surjam,
> temos de ir andando assim mesmo, à espera de mentes iluminadas que
> vejam o que não se pode enxergar até hoje, embora tudo possa estar a
> um milésimo de milimetro diante de nosso nariz. Não se fazem mais
> einsteins como antigamente.
>
>
> Hélio: É difícil para mim escrever isto, como algo pode SER onda. Para
> mim
> > onda não é um SER. :-)
> Victor: A onda não é um ser. Uma das confusões é associar uma onda(e
> esse não é o seu caso) a, digamos, uma senoíde, um tobogã. Essas
> formas apenas indicam como a grandeza associada varia no tempo. Nem
> mais nem menos que isso. Vejamos um exemplo banal. Uma tensão em uma
> tomada, só para garantir que vamos ver a coisa em um ponto fixo no
> espaço. Por causa das caracteríscas físicas dos modos de transmissão
> da energia elétrica, o que acontece naquela tomada, naquele fim de
> linha, fixo?
> Num dado instante, um osciloscópio indica que o valor da tensão alí
> é, digamos, zero. Um instante depois, no mesmo ponto, a tensão tem um
> valor maior, depois maior ainda, atinge um máximo, vai diminuindo,
> chega a zero novamente, e, a partir daí, começa crescer em sentido
> contrário, vai a um máximo e volta zero, do mesmo jeito anterior. E
> daí tudo se repete. O ponto em questão é fixo, é uma tomada de sua
> casa. Alí não tem onda coisa nenhuma. Mas, se você pegar todos os
> valores de tensão que você mediu, desde o primeiro valor zero inicial
> até o segundo valor zero, quando tudo recomeça, e plotar isso num
> gráfico, o conjunto de pontos vai se dispor numa forma senoidal. Na
> tomada seguinte, é a mesma coisa, bem como nas tomadas aquí de casa. É
> como se você pegasse aquela sucessão de tensões(em volts) ocorrendo em
> um ponto fixo do espaço,mas distintas no tempo, e, "espichando" o
> tempo, anotasse num eixo vertical valores proporcionais às quantidades
> medidas para cada instante observado. Quando se confundem essas duas
> coisas,as atrapalhações no juízo vêm por acréscimo. Em mecânica,
> pode-se raciocinar do mesmo jeito.
>
> Hélio: Onda, para mim é apenas uma das formas da expressar um
> determinado
> > comportamento da matéria.
> Victor: Acho muito vaga a definição acima. Talvez não tenha
> entendido bem. O que sei (ou penso saber) é que tudo está centrado no
> conceito de energia, que não sabemos o que é, e nada podemos fazer
> quanto a isso, a não ser especular, medir, anotar resultados e
> comparar com a as previsões do aparato teórico. Sabemos sim, como
> manusear energia, usá-la, transformá-la a contento, para tocar nossa
> vidinha, construir, destruir, e tudo o mais a que temos direito. Só
> isso. Há diversas formas de energia, tantas que fica difícil listar.
> Mas a energia é uma só, apenas se transforma de um tipo em outro. E se
> conserva. As leis de conservação de energia, falam de energia, mas não
> do tipo. Pois, segundo essas leis, a democracia prevalece para todas
> as formas! E o que acontece é que um corpo material, em sí, já uma
> forma de energia; e reaje de maneiras diferentes a diferentes formas
> de outras energias. E a energia pode ser transferida de diversas
> maneiras e modos. O comportamento de um corpo sujeito a isso reflete
> o tipo de energia que recebe(ou que pode repassar), inclusive
> "parecendo" tornar-se uma onda, o próprio corpo, como o caso daquela
> ponte lá nos isteites que até matou gente e causou enormes prejuízos,
> como todos sabem. Enfim, há muito para se dizer e especular. E também
> pouco a acrescentar ao que já se conhece.
> Na FQ, por exemplo, com respeito ao hélio fluído, o comportamento
> deste foge, como o diabo foge da cruz, da saia não tão justa
> desenhada acima. É objeto macroscópico, porém quântico, que não sobe e
> desce quando uma onda passa, como as ondas na água, mas ele, o próprio
> hélio ai, sim, é a própria onda!..., sem nadinha daquele exemplo das
> tensões que dei acima. É estranho, muito estranho, de doido ficar bom.
> Mas é o que a experiência fornece, como resultado!. Explicações? Nihil.
> Mas isso é papo para o papa de FQ, eterno apaixonado, nosso amigo
> Mesquita. Passo. Com ele a palavra.
>
> Sds,
>
> Victor.
>
>
> Helio
>
>
>
>
>
> ##### ##### #####
>
> Para saber mais visite
> http://www.ciencialist.hpg.ig.com.br
>
>
> ##### ##### ##### #####
>
>
>
> Yahoo! Grupos, um serviço oferecido por:
>
> São Paulo Rio de Janeiro Curitiba Porto Alegre
> Belo Horizonte Brasília
>
>
>
>
> ------------------------------------------------------------------------------
> Links do Yahoo! Grupos
>
> a.. Para visitar o site do seu grupo na web, acesse:
> http://br.groups.yahoo.com/group/ciencialist/
>
> b.. Para sair deste grupo, envie um e-mail para:
> ciencialist-unsubscribe@yahoogrupos.com.br
>
> c.. O uso que você faz do Yahoo! Grupos está sujeito aos Termos do
> Serviço do Yahoo!.
>
>
>
> ---
> Outgoing mail is certified Virus Free.
> Checked by AVG anti-virus system (http://www.grisoft.com).
> <http://www.grisoft.com%29.>
> Version: 6.0.859 / Virus Database: 585 - Release Date: 14/02/05
>
>
> __________ Informação do NOD32 1.1009 (20050226) __________
>
> Esta mensagem foi verificada pelo NOD32 Sistema Antivírus
> http://www.nod32.com.br
>
>
> [As partes desta mensagem que não continham texto foram removidas]
>
>
>
> ##### ##### #####
>
> Para saber mais visite
> http://www.ciencialist.hpg.ig.com.br
>
>
> ##### ##### ##### #####
>
>
> *Yahoo! Grupos, um serviço oferecido por:*
>
> *
> <http://br.rd.yahoo.com/SIG=12alass94/M=264105.3931087.6562589.1588051/D=brclubs/S=2137111528:HM/EXP=1110804237/A=2361264/R=6/SIG=10v4acpp0/*http://br.shopping.yahoo.com/>*
>
>
>
> ------------------------------------------------------------------------
> *Links do Yahoo! Grupos*
>
> * Para visitar o site do seu grupo na web, acesse:
> http://br.groups.yahoo.com/group/ciencialist/
>
> * Para sair deste grupo, envie um e-mail para:
> ciencialist-unsubscribe@yahoogrupos.com.br
> <mailto:ciencialist-unsubscribe@yahoogrupos.com.br?subject=Unsubscribe>
>
> * O uso que você faz do Yahoo! Grupos está sujeito aos Termos do
> Serviço do Yahoo! <http://br.yahoo.com/info/utos.html>.
>
>
>
>
> __________ Informação do NOD32 1.1024 (20050311) __________
>
> Esta mensagem foi verificada pelo NOD32 Sistema Antivírus
> http://www.nod32.com.br




SUBJECT: Re: [ciencialist] Fw: du.alva
FROM: "Alvaro Augusto \(E\)" <alvaro@electraenergy.com.br>
TO: <ciencialist@yahoogrupos.com.br>
DATE: 14/03/2005 10:34

1. Basta medir a massa da gota! Como, geralmente, as pessoas normais não têm balanças com essa precisão, dá para fazer o seguinte: (a) encha um frasco de conta-gotas com água; (b) conte quantas gotas couberam no conta-gotas; (c) meça a massa da quantidade de água que coube no frasco; (d) divida (c) por (b);

2. Depende do tamanho da formiga e da fome!

3. V=2*r*A*pi, onde r é o raio da base da lata, A é a altura e pi é pi.

[ ]s

Alvaro Augusto

----- Original Message -----
From: Luiz Ferraz Netto
To: ciencialist
Sent: Monday, March 14, 2005 8:05 AM
Subject: [ciencialist] Fw: du.alva


urgência!
[]'
===========================
Luiz Ferraz Netto [Léo]
leobarretos@uol.com.br
http://www.feiradeciencias.com.br
===========================
-----Mensagem Original-----
De: "du.alva" <du.alva@ig.com.br>
Para: <leobarretos@uol.com.br>
Enviada em: quarta-feira, 9 de março de 2005 23:58


Em 09/03/2005, Dulcilene escreve.

Leo ou Prof.Luiz Ferraz Netto?

Entrei no site da feira de ciencias e la vi seu email.
Minha filha esta fazendo um trabalho escolar sobre medidas.Ela tem 6 item
que deverá pesquisar ou fazer ela mesmo.

Alguns,ela fez, mas três itens está difícil resolver, e eu já não me lembro
mais como fazer. Gostaria que me ajudasse.

São eles:

1-A massa de uma gota de água;
2-O tempo que uma formiga leva para percorrer em linha reta a distância de 1
metro;
3-O volume de uma lata de leite condensado.

Obrigada por sua atenção e espero que possa me ajudar.

O trabalho é para a semana que vem.

DULCILENE

Cheguei a imprimir o desenho da representação esquemática de uma gota de
água.



--------------------------------------------------------------------------------


Internal Virus Database is out-of-date.
Checked by AVG Anti-Virus.
Version: 7.0.300 / Virus Database: 266.5.0 - Release Date: 25/02/2005



--
Internal Virus Database is out-of-date.
Checked by AVG Anti-Virus.
Version: 7.0.300 / Virus Database: 266.5.0 - Release Date: 25/02/2005



##### ##### #####

Para saber mais visite
http://www.ciencialist.hpg.ig.com.br


##### ##### ##### #####


Yahoo! Grupos, um serviço oferecido por:







------------------------------------------------------------------------------
Links do Yahoo! Grupos

a.. Para visitar o site do seu grupo na web, acesse:
http://br.groups.yahoo.com/group/ciencialist/

b.. Para sair deste grupo, envie um e-mail para:
ciencialist-unsubscribe@yahoogrupos.com.br

c.. O uso que você faz do Yahoo! Grupos está sujeito aos Termos do Serviço do Yahoo!.



[As partes desta mensagem que não continham texto foram removidas]



SUBJECT: Re: [ciencialist] Fw: du.alva
FROM: "Alberto Mesquita Filho" <albmesq@uol.com.br>
TO: <ciencialist@yahoogrupos.com.br>
DATE: 14/03/2005 11:12

----- Original Message -----
From: "Alvaro Augusto (E)"
Sent: Monday, March 14, 2005 10:34 AM
Subject: Re: [ciencialist] Fw: du.alva

> > 2-O tempo que uma formiga leva para percorrer em linha reta a distância
> > de 1 metro;

> 2. Depende do tamanho da formiga e da fome!

Na década de 80 do século passado o físico Cerqueira Leite dizia que "as
formigas só desenvolvem três velocidades específicas, além do repouso". É
bem provável que ele estivesse se referindo a formigas quânticas e neste
caso devemos pensar num repouso virtual, com energia do ponto zero diferente
de zero.

Cheguei a brincar com esse tema em um dos meus livros, e o item relativo a
essa brincadeira pode ser lido em "Uma corrida de formigas", que está em no
site da 51a turma da fmusp em
http://fmusp51a.ecientificocultural.com/textos/formigas.htm .

[ ]´s
Alberto
http://ecientificocultural.com/indice.htm
Mas indiferentemente a tudo isso, o neutrino tem massa, o elétron não é
uma carga elétrica coulombiana e a Terra se move. E a história se repetirá.




SUBJECT: doenças que curam doenças
FROM: Manuel Bulcão <manuelbulcao@uol.com.br>
TO: ciencialist@yahoogrupos.com.br
DATE: 14/03/2005 11:52


Oi,

Sabe-se que a paresia generalizada ou demência paralítica (uma
doença psiquiátrica causada pela neurosífilis avançada) pode ser
curada inoculando-se no paciente sangue malárico. A febre induzida
pelo parasita da malária é capaz de matar as espiroquetas que causam
a sífilis.

Sabe-se também que muitos psicóticos apresentam uma melhora
significativa após sobreviverem à febre tifóide, erisipela ou
tuberculose.

Alguém aqui na lista conhece alguma outra doença capaz de curar
doenças fatais?

Agradeço qualquer informação.

Abraços,
Manuel Bulcão







SUBJECT: Re: [ciencialist] Fw: pressão
FROM: "Oraculo" <oraculo@atibaia.com.br>
TO: <ciencialist@yahoogrupos.com.br>
DATE: 14/03/2005 13:00

Olá Professor

he he he...:-) O Hermes foi muito "hermético" em sua pergunta..:-) (desculpe, não resisti a piada sem graça..:-)

Homero

----- Original Message -----
From: Luiz Ferraz Netto
To: ciencialist
Sent: Monday, March 14, 2005 8:24 AM
Subject: [ciencialist] Fw: pressão


Decifrar isso ..........
[]'
===========================
Luiz Ferraz Netto [Léo]
leobarretos@uol.com.br
http://www.feiradeciencias.com.br
===========================
-----Mensagem Original-----
De: Hermes Andrade
Para: leobarretos@uol.com.br
Enviada em: quinta-feira, 10 de março de 2005 08:40
Assunto: pressão


Prezado professor:
Ante de mais nada, parabenizo-lhe pelo belo trabalho.

Minha pergunta:

se eu tenho um recipiente que pesa 20Kg e com saídas controláveis, quanto de pressão interna eu preciso para elevar este recipiente a 3 metros de altura ?

Desde já agradeço pela atenção;

Hermes


--------------------------------------------------------------------------------
Yahoo! Acesso Grátis - Internet rápida e grátis. Instale o discador do Yahoo! agora.


--------------------------------------------------------------------------------


Internal Virus Database is out-of-date.
Checked by AVG Anti-Virus.
Version: 7.0.300 / Virus Database: 266.5.0 - Release Date: 25/02/2005

----------

Internal Virus Database is out-of-date.
Checked by AVG Anti-Virus.
Version: 7.0.300 / Virus Database: 266.5.0 - Release Date: 25/02/2005


[As partes desta mensagem que não continham texto foram removidas]



##### ##### #####

Para saber mais visite
http://www.ciencialist.hpg.ig.com.br


##### ##### ##### #####


Yahoo! Grupos, um serviço oferecido por:







------------------------------------------------------------------------------
Links do Yahoo! Grupos

a.. Para visitar o site do seu grupo na web, acesse:
http://br.groups.yahoo.com/group/ciencialist/

b.. Para sair deste grupo, envie um e-mail para:
ciencialist-unsubscribe@yahoogrupos.com.br

c.. O uso que você faz do Yahoo! Grupos está sujeito aos Termos do Serviço do Yahoo!.



[As partes desta mensagem que não continham texto foram removidas]



SUBJECT: Re: [ciencialist] Fw: du.alva
FROM: "Oraculo" <oraculo@atibaia.com.br>
TO: <ciencialist@yahoogrupos.com.br>
DATE: 14/03/2005 13:04

Olá Alberto

Acho que o Cerqueira Leite se referia a biologia mesmo..:-) Formigas "ligam e desligam" suas velocidades, não tem um sistema de comutação gradual, como botões de volume de aparelhos de som, mas mecanismos de liga e desliga como chaves de luz de paredes..:-)

Assim, ela passa do caminhar para a quase corrida e da quase corrida para a corrida, sem transição. Não pode escolher em que velocidade vai andar entre cada uma das velocidades, tem apenas 3 possibilidades (seu sistema nervoso é bem simplificado nesse sentido).

Um abraço.

Homero

----- Original Message -----
From: Alberto Mesquita Filho
To: ciencialist@yahoogrupos.com.br
Sent: Monday, March 14, 2005 11:12 AM
Subject: Re: [ciencialist] Fw: du.alva


----- Original Message -----
From: "Alvaro Augusto (E)"
Sent: Monday, March 14, 2005 10:34 AM
Subject: Re: [ciencialist] Fw: du.alva

> > 2-O tempo que uma formiga leva para percorrer em linha reta a distância
> > de 1 metro;

> 2. Depende do tamanho da formiga e da fome!

Na década de 80 do século passado o físico Cerqueira Leite dizia que "as
formigas só desenvolvem três velocidades específicas, além do repouso". É
bem provável que ele estivesse se referindo a formigas quânticas e neste
caso devemos pensar num repouso virtual, com energia do ponto zero diferente
de zero.

Cheguei a brincar com esse tema em um dos meus livros, e o item relativo a
essa brincadeira pode ser lido em "Uma corrida de formigas", que está em no
site da 51a turma da fmusp em
http://fmusp51a.ecientificocultural.com/textos/formigas.htm .

[ ]´s
Alberto
http://ecientificocultural.com/indice.htm
Mas indiferentemente a tudo isso, o neutrino tem massa, o elétron não é
uma carga elétrica coulombiana e a Terra se move. E a história se repetirá.




##### ##### #####

Para saber mais visite
http://www.ciencialist.hpg.ig.com.br


##### ##### ##### #####


Yahoo! Grupos, um serviço oferecido por:

São Paulo Rio de Janeiro Curitiba Porto Alegre Belo Horizonte Brasília




------------------------------------------------------------------------------
Links do Yahoo! Grupos

a.. Para visitar o site do seu grupo na web, acesse:
http://br.groups.yahoo.com/group/ciencialist/

b.. Para sair deste grupo, envie um e-mail para:
ciencialist-unsubscribe@yahoogrupos.com.br

c.. O uso que você faz do Yahoo! Grupos está sujeito aos Termos do Serviço do Yahoo!.



[As partes desta mensagem que não continham texto foram removidas]



SUBJECT: Re: [ciencialist] Louis de Broglie versus Erwin Schrödinger
FROM: JVictor <jvoneto@uol.com.br>
TO: ciencialist@yahoogrupos.com.br
DATE: 14/03/2005 15:12

Rick,


Muito o trabalho de Pedro Sergio. Valeu a aprovação. Lí, gostei, está
bem escrito, e ancorado numa fata bibliografia histórica.
Nota 10, com louvor.

Sds,

Victor.




Ricardo Soares Vieira escreveu:

>
> Olá a todos do ciencialist!
>
> A mecanica quantica atual provove bastante o nome de Schrödinger, ao
> passo que o nome de Louis de Broglie é apenas comentado dizendo que
> ele foi o percursor...
>
> Desse modo, dá a entender que o mérito é 90% de Schrödinger e 10% de
> L. de Broglie, mas gostaria de oferecer a seguinte leitura a todos
> os interessados:
>
>
>
> "DISSERTAÇÃO DE MESTRADO Louis de Broglie e as ondas de matéria" de
> Pedro-Sergio-Rosa, texto em pdf.
>
> http://ghtc.ifi.unicamp.br/Teses/Pedro-Sergio-Rosa.pdf
>
> Encontrei essa tese na net, procurando no google por "tese louis de
> Broglie".
>
> Nela, o autor demonstra que a teoria de L. de Broglie é desde o
> inicio relativistica, além de ser determinista e não probabilistica
> como na concepção de Schrödinger. Também comenta-se que o autor
> sabia que haveria o princípio da incertesa, atribuido atualmente à
> Heinsenberg, pois que ele conhecia o fato de que o comprimento de um
> grupo de ondas fica sujeito a determinadas "condições de
> incertesas"...
>
> Enfim, é um texto muito interessante, e abre uma boa discução: "Qual
> vocês acham a teoria que melhor descreve a MQ, a de Schrodinger ou a
> de Louis de Broglie?"
>
> p.s. Também vale comentar as formulações de Heisenberg e de
> Feynman...
>
>
> Até mais.
>
> Rick
>
>
>
>
>
> ##### ##### #####
>
> Para saber mais visite
> http://www.ciencialist.hpg.ig.com.br
>
>
> ##### ##### ##### #####
>
>
> *Yahoo! Grupos, um serviço oferecido por:*
>
> *
> <http://br.rd.yahoo.com/SIG=12ak73s7m/M=264105.3931087.6562589.1588051/D=brclubs/S=2137111528:HM/EXP=1110762047/A=2361264/R=6/SIG=10v4acpp0/*http://br.shopping.yahoo.com/>*
>
>
>
> ------------------------------------------------------------------------
> *Links do Yahoo! Grupos*
>
> * Para visitar o site do seu grupo na web, acesse:
> http://br.groups.yahoo.com/group/ciencialist/
>
> * Para sair deste grupo, envie um e-mail para:
> ciencialist-unsubscribe@yahoogrupos.com.br
> <mailto:ciencialist-unsubscribe@yahoogrupos.com.br?subject=Unsubscribe>
>
> * O uso que você faz do Yahoo! Grupos está sujeito aos Termos do
> Serviço do Yahoo! <http://br.yahoo.com/info/utos.html>.
>
>
>
>
> __________ Informação do NOD32 1.1024 (20050311) __________
>
> Esta mensagem foi verificada pelo NOD32 Sistema Antivírus
> http://www.nod32.com.br




SUBJECT: Fw: Curso de fisica
FROM: "Luiz Ferraz Netto" <leobarretos@uol.com.br>
TO: "ciencialist" <ciencialist@yahoogrupos.com.br>
DATE: 14/03/2005 18:47

A todos os Físicos .......

[]'
===========================
Luiz Ferraz Netto [Léo]
leobarretos@uol.com.br
http://www.feiradeciencias.com.br
===========================
-----Mensagem Original-----
De: "Caio Ferrari de Oliveira" <caioferrari@yahoo.com.br>
Para: <leobarretos@uol.com.br>
Enviada em: domingo, 13 de março de 2005 23:14
Assunto: Curso de fisica


Olá luiz, como vai?

Meu nome é Caio, sou um vestibulando e estou com um pouco de duvida em
relação a minha faculdade.

Desde pequeno, sempre despertei interesse por eletrônica e sempre quis
fazer engenharia elétrica e trabalhar com isso na área de projetos na
área. Posteriormente, desenvolvi um interesse maior por
áudio(amplificadores, préamps, etc), mas sempre no ramo da eletrônica.

Porém, depois que entrei para o colegial, passei a desenvolver um
interesse imenso por física, principalmente quando tive uma aula com
nocões de física quântica. Percebi que um engenheiro com conhecimento
de física teria possibilidades incríveis. Por exemplo em um
amplificador de áudio, um físico pode projetar muito adequadamente um
sistema de dissipação de calor, escolhendo metais corretos,
calculando/testando coolers com diferentes vazões, etc. Fora outros
inúmeros exemplos.

Entretanto, o que me desestimula a seguir em frente nesse curso é o meu
aprendizado em eletrônica, ciência que eu gosto muito. Além disso,
percebo que o mercado de trabalho aqui no brasil para fisicos é muito
restrito.

Sei que em física se aprende muito sobre materiais, estudando
estrutura de diodo, transistor, etc.
Porém, gostaria de saber se um físico, projetaria por exemplo um
amplificador, uma TV, um motor de um ventilador, etc. Se não, existe
alguma especialização em eletrônica que vc faz durante ou após o
bacharelado?

Caso haja, como ficaria um físico no mercado de trabalho. Se uma
empresa como a GE, a Gradiente ou qualquer outra abrisse uma vaga para
engenheiro elétrico, um físico especializado em eletronica, poderia
preencher essa vaga?

Gostaria muito de cursar minha faculdade e fazer um estágio, algo do
tipo. Como ficaria.

Desculpe pelas zilhões de perguntas. Espero não ter incomodado

Um abraço
Caio Ferrari



--
Internal Virus Database is out-of-date.
Checked by AVG Anti-Virus.
Version: 7.0.300 / Virus Database: 266.5.0 - Release Date: 25/02/2005




--
Internal Virus Database is out-of-date.
Checked by AVG Anti-Virus.
Version: 7.0.300 / Virus Database: 266.5.0 - Release Date: 25/02/2005



SUBJECT: Fw: Resposta reduzida
FROM: "Luiz Ferraz Netto" <leobarretos@uol.com.br>
TO: "ciencialist" <ciencialist@yahoogrupos.com.br>
DATE: 14/03/2005 18:48

To Takata ........ bem resumido!
[]'
===========================
Luiz Ferraz Netto [Léo]
leobarretos@uol.com.br
http://www.feiradeciencias.com.br
===========================
-----Mensagem Original-----
De: Janaina Ramos
Para: leobarretos@uol.com.br
Enviada em: segunda-feira, 14 de março de 2005 12:46
Assunto: Resposta reduzida


Boa Tarde professor,
Sexta feira minha professora de biologia, mandou respondermos umas perguntas para revizar matéria. Procurei respostas rezumidas por toda internet mas não consigo acessar um site mais direto. Preciso saber oque são?
1- Célula
2- Átomo
3- Moléculas
4- Substâncias
5- Tecidos
6- Órgãos
7- Sistemas
8- Aparelhos

Por favor, me responda assim que possível
Grata

Nas dúvidas experimentais, por gentileza coloque aqui o endereço da página, isso facilita o confronto. Agradeço. Meu nome é LUIZ FERRAZ NETTO, meu apelido é LÉO e moro em BARRETOS; dai vem meu e-mail: leobarretos@uol.com.br.



--------------------------------------------------------------------------------


Internal Virus Database is out-of-date.
Checked by AVG Anti-Virus.
Version: 7.0.300 / Virus Database: 266.5.0 - Release Date: 25/02/2005

----------

Internal Virus Database is out-of-date.
Checked by AVG Anti-Virus.
Version: 7.0.300 / Virus Database: 266.5.0 - Release Date: 25/02/2005


[As partes desta mensagem que não continham texto foram removidas]



SUBJECT: Fw: duvida - fossas
FROM: "Luiz Ferraz Netto" <leobarretos@uol.com.br>
TO: "ciencialist" <ciencialist@yahoogrupos.com.br>
DATE: 14/03/2005 18:53

Quem entende de fossa?
[]'
===========================
Luiz Ferraz Netto [Léo]
leobarretos@uol.com.br
http://www.feiradeciencias.com.br
===========================
-----Mensagem Original-----
De: Sandra da Silva Correa
Para: leobarretos@uol.com.br
Enviada em: segunda-feira, 14 de março de 2005 14:41
Assunto: duvida


Meu nome é Sandra, sou da Unespar-Fecilcam, estou cursando Geografia (1º ano )e moro em Campo Mourão-PR, e gostaría de obter informações sobre Fossas oceanicas, em especial a Fossa das Filipinas.
Grata
Sandra



__________________________________________________
Converse com seus amigos em tempo real com o Yahoo! Messenger
http://br.download.yahoo.com/messenger/



--------------------------------------------------------------------------------


Internal Virus Database is out-of-date.
Checked by AVG Anti-Virus.
Version: 7.0.300 / Virus Database: 266.5.0 - Release Date: 25/02/2005

----------

Internal Virus Database is out-of-date.
Checked by AVG Anti-Virus.
Version: 7.0.300 / Virus Database: 266.5.0 - Release Date: 25/02/2005


[As partes desta mensagem que não continham texto foram removidas]



SUBJECT: Fw: aluno curioso -- do barometro
FROM: "Luiz Ferraz Netto" <leobarretos@uol.com.br>
TO: "ciencialist" <ciencialist@yahoogrupos.com.br>
DATE: 14/03/2005 19:05

Onde está aquele antigo texto do bate-papo do barômetro para medir a altura do prédio. O consulente aqui deve ter ouvido cantar o galo e .......

[]'
===========================
Luiz Ferraz Netto [Léo]
leobarretos@uol.com.br
http://www.feiradeciencias.com.br
===========================
-----Mensagem Original-----
De: santana957@hotmail.com
Para: leobarretos@uol.com.br
Enviada em: domingo, 13 de março de 2005 15:28
Assunto: aluno curioso


caro prof. vc poderia m ajudar a entender funcionamento de um barômetro e com ele pod medir a altura d um predio
Gostaria muito s vc m resp a esta pergunta o mais possivel.
Muito obrigado


--------------------------------------------------------------------------------


Internal Virus Database is out-of-date.
Checked by AVG Anti-Virus.
Version: 7.0.300 / Virus Database: 266.5.0 - Release Date: 25/02/2005

----------

Internal Virus Database is out-of-date.
Checked by AVG Anti-Virus.
Version: 7.0.300 / Virus Database: 266.5.0 - Release Date: 25/02/2005


[As partes desta mensagem que não continham texto foram removidas]



SUBJECT: Fw: Calculadoras
FROM: "Luiz Ferraz Netto" <leobarretos@uol.com.br>
TO: "ciencialist" <ciencialist@yahoogrupos.com.br>
DATE: 14/03/2005 19:06

Calculadoras antes do ábaco? Será que estou vivendo do avesso?

[]'
===========================
Luiz Ferraz Netto [Léo]
leobarretos@uol.com.br
http://www.feiradeciencias.com.br
===========================
-----Mensagem Original-----
De: "pimentaomucholoco" <pimentaomucholoco@ig.com.br>
Para: <leobarretos@uol.com.br>
Enviada em: domingo, 13 de março de 2005 13:05
Assunto: Calculadoras


Eu queria saber quais as calculadoras que existiram dês de o ábaco, se
possível me mande uma lista, muito obrigado.



--------------------------------------------------------------------------------


Internal Virus Database is out-of-date.
Checked by AVG Anti-Virus.
Version: 7.0.300 / Virus Database: 266.5.0 - Release Date: 25/02/2005



--
Internal Virus Database is out-of-date.
Checked by AVG Anti-Virus.
Version: 7.0.300 / Virus Database: 266.5.0 - Release Date: 25/02/2005



SUBJECT: Re: Foucault / Luz ondas ou corpusculos.
FROM: Hélio Ricardo Carvalho <hrc@fis.puc-rio.br>
TO: ciencialist@yahoogrupos.com.br
DATE: 14/03/2005 19:32


Victor,

Responder uma MSG grande como esta é um pouco difícil mas vamos lá.

Algumas partes em que eu concordo com a resposta do Alberto (msg:
44868), vou pular.

Victor:
>Com o advento das lentes, telescópios, microscópios,...

Tudo isto é refração.

>...espectográfos, ...

Isto é obtido com prismas ou rede de difração. Se for prisma no fundo
é o mesmo mecanismo (absorção, tempo, reemissão), muda apenas o número
de absorções (e talvez o tempo) para cada freqüência.
No caso de rede de difração fica mais claro que devemos sempre admitir
que as emissões são em todas as direções (ou aleotoriamente em qq
direção o que dá em média temporal "todas as direções". Mas isto é do
debate com o Takata)

>...novos efeitos foram sendo observados, que a teoria
>corpuscular não conseguia explicar.

Novamente, "aquela teoria corpuscular"

>Os experimentos de Arago, Young, Foucault não deixavam
>dúvidas de que a luz era um fenômeno que apresentava
>comportamento ondulatório, já com justificativa matematica.

Você, hoje fala isto com um certo cuidado.
Eles, provavelmente diziam que estas experiências confirmavam que a
luz É ondas e não simplesmente tem "comportamento ondulatório".
Naquela época isto se justificava porque "existia" o ether.
Quem acreditava em luz ondulatória acreditava no ether e quem não
acreditava no ether tinha outra teoria para a luz (teoria de emissão).
E as ondas eram longitudinais como as do som e com mecanismo de
transmissão semelhante.

> Hélio: No modelo ondulatório em vigor na época a luz
> também deveria ser mais rápida no meio mais denso.
>
> Victor: Honestamente, não encontrei, ainda, nenhuma
> referência à informação acima. de qualquer maneira,
> se alguém chegou a essa conclusão, interpretou alguma
> coisa de maneira errada.

Errada entre aspas, né. Hoje interpretamos várias coisas de maneira
errada mas achamos que é certa. Eles teriam seus motivos para
interpretar assim pois o som é assim e se a luz se propaga como o som
deveria ser também.

>Hélio: Então vai minha OUTRA pergunta:
> Por que a experiência de Foucault não descartou de vez a luz
> ondulatória também????

A sua resposta abaixo me parece que não corresponde muito à esta
pergunta acima.
No início eu achei que você estava respondendo uma pergunta do tipo.
"Por que a experiência de Foucault não descartou de vez a luz
CORPUSCULAR de Newton?" que eu não fiz.

> Victor: Não descartou por uma razão que julgo muito forte. A
> autoridade de Newton era muito grande não só entre pessoas
> comuns como entre as grandes mentes como Poisson, Biot,
> Malus(descobridor da polarização por reflexão), Brewster (aquele,
> do ângulo crtítico, que foi também biógrafo de Newton), só para
> citar alguns de peso. Com uma caracterísitica. Os dois primeiros
> eram defensores radicais da teoria corpuscular, anti-éticos até,
> pois se valiam de suas autoridades para aterrorizar todos quanto
> não falassem em sua cartilha newtoniana. Há um fato registrado
> que diz bem do comportamento daquelas pessoas. ...

A estória da luz no centro da sombra teve algum "quem conta um conto
aumenta um ponto". Não deve ter acontecido exatamente assim mas é esta
que eu tinha como A experiência que descartou a teoria corpuscular da
época e não a de Foucault que, para este fim é mais dúbia.

No final da estória você diz:
> Ante esse juiz implacável e frio, que é um resultado experimental,
> Poisson inteligentemente reconheceu e sucumbiu às teses de Fresnel.
> Mas Biot, mesmo vendo o resultado experimental, confirmando as
> teses de Fresnel, ainda assim, não aceitou. Continou engangado nas
> teoria de Newton. Paixão assim, só sexual!
> Nem ante fatos o cara retrocedeu. Putz.

Eu tenho que comentar esta parte:
Não sei se Biot agiu assim por "paixão sexual" a Newton. Se foi, foi
errado. Mas se não foi muito bem pois ele já antecipava a física que
viria depois, pois o que não se pode negar é o resultado experimental
(o fato!).
A interpretação que se faz dele pode ser sempre questionada.

Voltando ao trecho em que você diz:
>Os experimentos de Arago, Young, Foucault não deixavam
>dúvidas de que a luz era um fenômeno que apresentava
>comportamento ondulatório.

A rigor, eles não poderiam dizer: "Estes experimentos dão a certeza
que a luz É onda". Eles pode dizer: "... tem comportamento ondulatório".
(este rigorismo é só ilustrativo do que eu quero te transmitir, não
precisamos ser sempre assim, as vezes isto fica subentendido)

No trecho abaixo já fiz a correção indicada por você na outra MSG:
> ...*Acho* que eu, se estivesse na época e fosse um cientista
> com C maiúsclo, acho que descartaria.

Depois de tudo isto eu acho que você esta falando que descartaria o
modelo corpuscular de Newton. Certo?

Mas minha pergunta foi:
>Hélio: Então vai minha OUTRA pergunta:
> Por que a experiência de Foucault não descartou de vez a luz
> ondulatória também????

Claro que alguém poderia dizer: o som usa o ar e a matéria sólida e a
luz usa o ether logo onde tem matéria mais densa não sobra muito
espaço para o ether. [:-)] A luz é mais lenta na matéria mais densa
porque aí tem menos ether. [:-)]

Vou parar por aqui mas a outra parte de sua MSG também é muito
interessante. Depois eu respondo.

Hélio





SUBJECT: Gravitons
FROM: "Luiz Ferraz Netto" <leobarretos@uol.com.br>
TO: <ciencialist@yahoogrupos.com.br>
DATE: 14/03/2005 19:36

Apreciaria comentários sobre o seguinte texto:

http://rolfguthmann.sites.uol.com.br/TQG/tqg.html

[]'
===========================
Luiz Ferraz Netto [Léo]
leobarretos@uol.com.br
http://www.feiradeciencias.com.br
===========================


--
Internal Virus Database is out-of-date.
Checked by AVG Anti-Virus.
Version: 7.0.300 / Virus Database: 266.5.0 - Release Date: 25/02/2005



SUBJECT: Re: [ciencialist] Fw: batatas e lampada
FROM: Luis Brudna <luisbrudna@gmail.com>
TO: ciencialist@yahoogrupos.com.br
DATE: 14/03/2005 20:00

Jah ligaram um aparelho de som. :-)

http://www.humornaciencia.blogger.com.br/2004_05_01_archive.html#26319216

Ateh
Luis Brudna


On Mon, 14 Mar 2005 07:56:38 -0300, Luiz Ferraz Netto
<leobarretos@uol.com.br> wrote:
>
> Enviei uma resposta para a Laurie ... muito a grosso modo...:
> ======================
> Olá Laurie,
>
> para acender uma lâmpada de 40W sob tensão de 127VDC usando as batatas, precisaríamos de; vejamos:
>
> Supondo que cada meia-batata forneça 0,63 V, serão necessárias duas meias-batatas ligadas em série para obter 1,27 VDC.


SUBJECT: Re: O DESAFIO DA RELATIVIDADE PIRELLI
FROM: "Marcella" <marcellairocha@yahoo.com.br>
TO: ciencialist@yahoogrupos.com.br
DATE: 14/03/2005 20:02


olá
Estou empenhada fazendo um projeto para o desafio Pirelli.
Queria saber se alguém pode me ajudar nas revisões,pois ainda não
aprendi essa matéria na escola.
Onde vocês souberam que as inscrições foram prorrogadas?
Espero respostas
marcellairocha@yahoo.com.br

--- Em ciencialist@yahoogrupos.com.br, "Alvaro Augusto \(E\)"
<alvaro@e...> escreveu
> Parece que o prazo de inscrição foi adiado para 15 de setembro...
>
> [ ]s
>
> Alvaro Augusto
>
> ----- Original Message -----
> From: rmtakata
> To: ciencialist@yahoogrupos.com.br
> Sent: Tuesday, March 08, 2005 5:50 PM
> Subject: [ciencialist] O DESAFIO DA RELATIVIDADE PIRELLI
>
>
>
> A Pirelli - aquela mesma, dos pneus (e outras coisas) - lançou um
> concurso para premiar (com generosos 25 mil euros) quem
apresentar em
> um trabalho multimídia a Teoria da Relatividade do modo mais
simples
> (e correto).
>
> As apresentações devem ser feitas em inglês ou em italiano.
Inscrições
> podem ser feitas por e-mail até 31 de março.
>
> www.pirelliaward.com
>
> Mãos à obra pessoal!
>
> []s,
>
> Roberto Takata
>
>
>
>
>
> ##### ##### #####
>
> Para saber mais visite
> http://www.ciencialist.hpg.ig.com.br
>
>
> ##### ##### ##### #####
>
>
> Yahoo! Grupos, um serviço oferecido por:
>
>
>
>
>
>
>
> --------------------------------------------------------------------
----------
> Links do Yahoo! Grupos
>
> a.. Para visitar o site do seu grupo na web, acesse:
> http://br.groups.yahoo.com/group/ciencialist/
>
> b.. Para sair deste grupo, envie um e-mail para:
> ciencialist-unsubscribe@yahoogrupos.com.br
>
> c.. O uso que você faz do Yahoo! Grupos está sujeito aos Termos
do Serviço do Yahoo!.
>
>
>
> [As partes desta mensagem que não continham texto foram removidas]





SUBJECT: Re: [ciencialist] Louis de Broglie versus Erwin Schrödinger
FROM: "Luiz Ferraz Netto" <leobarretos@uol.com.br>
TO: <ciencialist@yahoogrupos.com.br>
DATE: 14/03/2005 20:06

Tenho algo sobre De Broglie em :

http://www.feiradeciencias.com.br/sala19/texto64.asp

que continua em:

http://www.feiradeciencias.com.br/sala19/texto65.asp

[]'
===========================
Luiz Ferraz Netto [Léo]
leobarretos@uol.com.br
http://www.feiradeciencias.com.br
===========================
-----Mensagem Original-----
De: "Ricardo Soares Vieira" <rickrsv@yahoo.com.br>
Para: <ciencialist@yahoogrupos.com.br>
Enviada em: sábado, 12 de março de 2005 22:00
Assunto: [ciencialist] Louis de Broglie versus Erwin Schrödinger




Olá a todos do ciencialist!

A mecanica quantica atual provove bastante o nome de Schrödinger, ao
passo que o nome de Louis de Broglie é apenas comentado dizendo que
ele foi o percursor...

Desse modo, dá a entender que o mérito é 90% de Schrödinger e 10% de
L. de Broglie, mas gostaria de oferecer a seguinte leitura a todos
os interessados:



"DISSERTAÇÃO DE MESTRADO Louis de Broglie e as ondas de matéria" de
Pedro-Sergio-Rosa, texto em pdf.

http://ghtc.ifi.unicamp.br/Teses/Pedro-Sergio-Rosa.pdf

Encontrei essa tese na net, procurando no google por "tese louis de
Broglie".

Nela, o autor demonstra que a teoria de L. de Broglie é desde o
inicio relativistica, além de ser determinista e não probabilistica
como na concepção de Schrödinger. Também comenta-se que o autor
sabia que haveria o princípio da incertesa, atribuido atualmente à
Heinsenberg, pois que ele conhecia o fato de que o comprimento de um
grupo de ondas fica sujeito a determinadas "condições de
incertesas"...

Enfim, é um texto muito interessante, e abre uma boa discução: "Qual
vocês acham a teoria que melhor descreve a MQ, a de Schrodinger ou a
de Louis de Broglie?"

p.s. Também vale comentar as formulações de Heisenberg e de
Feynman...


Até mais.

Rick





##### ##### #####

Para saber mais visite
http://www.ciencialist.hpg.ig.com.br


##### ##### ##### #####
Links do Yahoo! Grupos










--
Internal Virus Database is out-of-date.
Checked by AVG Anti-Virus.
Version: 7.0.300 / Virus Database: 266.5.0 - Release Date: 25/02/2005




--
Internal Virus Database is out-of-date.
Checked by AVG Anti-Virus.
Version: 7.0.300 / Virus Database: 266.5.0 - Release Date: 25/02/2005



SUBJECT: Re: [ciencialist] Re: O DESAFIO DA RELATIVIDADE PIRELLI
FROM: "Luiz Ferraz Netto" <leobarretos@uol.com.br>
TO: <ciencialist@yahoogrupos.com.br>
DATE: 14/03/2005 20:15

Oi Marcela,

tenho um trabalho incompleto e ainda não explorado que é o Modelo Mecânico para o Estudo da Relatividade Restrita. É uma montagem super-simples que permite uma boa visualização dos fenômenos básicos da RR, em câmara lenta!
É preciso burilar a coisa ... não terminei o trabalho (não estou interessado em ganhar o prêmio Nobel --- pode ficar para vc).
Está em:
http://www.feiradeciencias.com.br/sala23/23_R00.asp

aquele prêmio, digo, abraço,
===========================
Luiz Ferraz Netto [Léo]
leobarretos@uol.com.br
http://www.feiradeciencias.com.br
===========================
-----Mensagem Original-----
De: "Marcella" <marcellairocha@yahoo.com.br>
Para: <ciencialist@yahoogrupos.com.br>
Enviada em: segunda-feira, 14 de março de 2005 20:02
Assunto: [ciencialist] Re: O DESAFIO DA RELATIVIDADE PIRELLI




olá
Estou empenhada fazendo um projeto para o desafio Pirelli.
Queria saber se alguém pode me ajudar nas revisões,pois ainda não
aprendi essa matéria na escola.
Onde vocês souberam que as inscrições foram prorrogadas?
Espero respostas
marcellairocha@yahoo.com.br

--- Em ciencialist@yahoogrupos.com.br, "Alvaro Augusto \(E\)"
<alvaro@e...> escreveu
> Parece que o prazo de inscrição foi adiado para 15 de setembro...
>
> [ ]s
>
> Alvaro Augusto
>
> ----- Original Message -----
> From: rmtakata
> To: ciencialist@yahoogrupos.com.br
> Sent: Tuesday, March 08, 2005 5:50 PM
> Subject: [ciencialist] O DESAFIO DA RELATIVIDADE PIRELLI
>
>
>
> A Pirelli - aquela mesma, dos pneus (e outras coisas) - lançou um
> concurso para premiar (com generosos 25 mil euros) quem
apresentar em
> um trabalho multimídia a Teoria da Relatividade do modo mais
simples
> (e correto).
>
> As apresentações devem ser feitas em inglês ou em italiano.
Inscrições
> podem ser feitas por e-mail até 31 de março.
>
> www.pirelliaward.com
>
> Mãos à obra pessoal!
>
> []s,
>
> Roberto Takata
>
>
>
>
>
> ##### ##### #####
>
> Para saber mais visite
> http://www.ciencialist.hpg.ig.com.br
>
>
> ##### ##### ##### #####
>
>
> Yahoo! Grupos, um serviço oferecido por:
>
>
>
>
>
>
>
> --------------------------------------------------------------------
----------
> Links do Yahoo! Grupos
>
> a.. Para visitar o site do seu grupo na web, acesse:
> http://br.groups.yahoo.com/group/ciencialist/
>
> b.. Para sair deste grupo, envie um e-mail para:
> ciencialist-unsubscribe@yahoogrupos.com.br
>
> c.. O uso que você faz do Yahoo! Grupos está sujeito aos Termos
do Serviço do Yahoo!.
>
>
>
> [As partes desta mensagem que não continham texto foram removidas]





##### ##### #####

Para saber mais visite
http://www.ciencialist.hpg.ig.com.br


##### ##### ##### #####
Links do Yahoo! Grupos










--
Internal Virus Database is out-of-date.
Checked by AVG Anti-Virus.
Version: 7.0.300 / Virus Database: 266.5.0 - Release Date: 25/02/2005




--
Internal Virus Database is out-of-date.
Checked by AVG Anti-Virus.
Version: 7.0.300 / Virus Database: 266.5.0 - Release Date: 25/02/2005



SUBJECT: Re: [ciencialist] Fw: Resposta reduzida
FROM: "Alvaro Augusto \(E\)" <alvaro@electraenergy.com.br>
TO: <ciencialist@yahoogrupos.com.br>
DATE: 15/03/2005 00:39

O que mais me impressiona é que o menino diz ter procurado por respostas por TODA A INTERNET! Nem o Takata é páreo para ele...

[ ]s

Alvaro Augusto

----- Original Message -----
From: Luiz Ferraz Netto
To: ciencialist
Sent: Monday, March 14, 2005 6:48 PM
Subject: [ciencialist] Fw: Resposta reduzida


To Takata ........ bem resumido!
[]'
===========================
Luiz Ferraz Netto [Léo]
leobarretos@uol.com.br
http://www.feiradeciencias.com.br
===========================
-----Mensagem Original-----
De: Janaina Ramos
Para: leobarretos@uol.com.br
Enviada em: segunda-feira, 14 de março de 2005 12:46
Assunto: Resposta reduzida


Boa Tarde professor,
Sexta feira minha professora de biologia, mandou respondermos umas perguntas para revizar matéria. Procurei respostas rezumidas por toda internet mas não consigo acessar um site mais direto. Preciso saber oque são?
1- Célula
2- Átomo
3- Moléculas
4- Substâncias
5- Tecidos
6- Órgãos
7- Sistemas
8- Aparelhos

Por favor, me responda assim que possível
Grata

Nas dúvidas experimentais, por gentileza coloque aqui o endereço da página, isso facilita o confronto. Agradeço. Meu nome é LUIZ FERRAZ NETTO, meu apelido é LÉO e moro em BARRETOS; dai vem meu e-mail: leobarretos@uol.com.br.



--------------------------------------------------------------------------------


[As partes desta mensagem que não continham texto foram removidas]



SUBJECT: Re: [ciencialist] Fw: Calculadoras
FROM: "Alvaro Augusto \(E\)" <alvaro@electraenergy.com.br>
TO: <ciencialist@yahoogrupos.com.br>
DATE: 15/03/2005 00:45

Caro Léo,

Não é "antes" do ábaco, e sim "dês de" o ábaco. No dialeto falado pelos adolecentes, "dês de" provavelmente significa "desde"...

[ ]s

Alvaro Augusto

----- Original Message -----
From: Luiz Ferraz Netto
To: ciencialist
Sent: Monday, March 14, 2005 7:06 PM
Subject: [ciencialist] Fw: Calculadoras


Calculadoras antes do ábaco? Será que estou vivendo do avesso?

[]'
===========================
Luiz Ferraz Netto [Léo]
leobarretos@uol.com.br
http://www.feiradeciencias.com.br
===========================
-----Mensagem Original-----
De: "pimentaomucholoco" <pimentaomucholoco@ig.com.br>
Para: <leobarretos@uol.com.br>
Enviada em: domingo, 13 de março de 2005 13:05
Assunto: Calculadoras


Eu queria saber quais as calculadoras que existiram dês de o ábaco, se
possível me mande uma lista, muito obrigado.



[As partes desta mensagem que não continham texto foram removidas]



SUBJECT: Re: [ciencialist] Gravitons
FROM: "Alvaro Augusto \(E\)" <alvaro@electraenergy.com.br>
TO: <ciencialist@yahoogrupos.com.br>
DATE: 15/03/2005 00:55

Eu gostaria de ter metade da criatividade e um quarto da cara-de-pau desse moço. Já no segundo parágrafo do resumo ele se auto-detona:

"Veremos que a gravidade é o resultado do desequilíbrio entre a força coulombiana e a força centrípeta nos átomos."

Bem, se existisse esse desequilíbrio, o átomo (de Bohr) se desintegraria, não é?

[ ]s

Alvaro Augusto

----- Original Message -----
From: Luiz Ferraz Netto
To: ciencialist@yahoogrupos.com.br
Sent: Monday, March 14, 2005 7:36 PM
Subject: [ciencialist] Gravitons


Apreciaria comentários sobre o seguinte texto:

http://rolfguthmann.sites.uol.com.br/TQG/tqg.html

[]'
===========================
Luiz Ferraz Netto [Léo]
leobarretos@uol.com.br
http://www.feiradeciencias.com.br
===========================




[As partes desta mensagem que não continham texto foram removidas]



SUBJECT: 'Theory of everything' tying researchers up in knots
FROM: "marcelomjr" <marcelomjr@hotmail.com>
TO: ciencialist@yahoogrupos.com.br
DATE: 15/03/2005 01:50


http://sfgate.com/cgi-bin/article.cgi?
f=/c/a/2005/03/14/MNGRMBOURE1.DTL

'Theory of everything' tying researchers up in knots
Keay Davidson, Chronicle Science Writer

Monday, March 14, 2005


Printable Version
Email This Article


The most celebrated theory in modern physics faces increasing attacks
from skeptics who fear it has lured a generation of researchers down
an intellectual dead end.

In its original, simplified form, circa the mid-1980s, string theory
held that reality consists of infinitesimally small, wiggling objects
called strings, which vibrate in ways that yield the different
subatomic particles that comprise the cosmos. An analogy is the
vibrations on a violin string, which yield different musical notes.

Advocates claimed that string theory would smooth out the conflicts
between Einsteinian relativity and quantum mechanics. The result,
they said, would be a grand unifying "theory of everything," which
could explain everything from the nature of matter to the Big Bang to
the fate of the cosmos.

Over the years, string theory has simultaneously become more
frustrating and fabulous. On the one hand, the original theory has
become mind-bogglingly complex, one that posits an 11-dimensional
universe (far more than the four- dimensional universe of Einstein).
The modified theory is so mathematically dense that many Ph.D.-
bearing physicists haven't a clue what their string- theorist
colleagues are talking about.

On the other hand, new versions of the theory suggest our universe is
just one of zillions of alternate, invisible -- perhaps even
inhabited -- universes where the laws of physics are radically
different. String buffs claim this bizarre hypothesis might help to
explain various cosmic mysteries.

Untestable theory

But skeptics suggest it's the latest sign of how string theorists,
sometimes called "superstringers," try to colorfully camouflage the
theory's flaws, like "a 50-year-old woman wearing way too much
lipstick," jokes Robert B. Laughlin, a Nobel Prize-winning physicist
at Stanford. "People have been changing string theory in wild ways
because it has never worked."

Already, the split over string theory has caused tensions at some of
the nation's university physics departments. "The physics department
at Stanford effectively fissioned over this issue," said Laughlin,
now on sabbatical in South Korea. "I think string theory is
textbook 'post-modernism' (and) fueled by irresponsible expenditures
of money."

The dispute could become explosive this year, with the publication of
contrarily minded books by two of the best-known and most eloquent
scientific popularizers of physics, string theorist Michio Kaku of
City University of New York and astrophysicist-particle theorist
Lawrence Krauss of Case Western Reserve University in Cleveland.

Skeptics have long mocked string theory as untestable, because
experimental studies of it would require machines of huge scale,
perhaps even as big as the solar system. In his new book "Parallel
Worlds" (Doubleday), Kaku disagrees and argues that the first
experimental evidence for string theory might begin to emerge within
several years from experiments with scientific instruments such as a
new particle accelerator, the Large Hadron Collider, which opens for
business near Geneva in 2007.

Wormhole travel

Kaku, whose previous books include the acclaimed "Hyperspace" and
equation-packed textbooks on string theory, also suggests that humans
might eventually travel to those alternate universes, perhaps via
hypothetical portals in space called wormholes.

Such claims dismay Krauss, a leading expert on cosmic dark matter and
dark energy who is popularly known as author of a best-seller, "The
Physics of Star Trek." In his book "Hiding in the Mirror: The
Mysterious Allure of Extra Dimensions," to be published by Viking in
September, Krauss argues that string theorists have produced no
satisfactory explanations for anything.

Krauss believes continued research is worthwhile just in case it pans
out. But he said that so far, string theorists have promised far more
than they have delivered and have fostered the false impression that
string theory is the only feasible way to explain cosmic mysteries.

Those who dabble in alternate-universe speculations might be just
modern versions of "16th century theologians (who) speculated that
spirits and angels emerge from the extra-dimensional universe," says
Krauss, who is also an outspoken foe of creationist teaching in
schools.

A great deal is at stake. Over the last two decades, a generation of
brilliant young physicists -- the kinds of proto-Einsteins who
historically have led intellectual revolution after revolution -- has
flocked to string theory because their professors told them that's
where the action was. Now many of them are reaching middle age and
have gained tenured posts on prestigious campuses. They're also
educating a whole new generation of fresh- faced wannabe string
theorists who are thrilled by the publicity that string theory
attracts, which has included several best-selling books and a special
effects-packed TV extravaganza on PBS.

The dispute has split partly along subdisciplinary lines, and mirrors
a timeless squabble in the philosophy of science: Which is more
important for scientific innovation -- theoretical daring or
empirical observations and experiments?

"Superstringers have now created a culture in physics departments
that is openly disdainful of experiments. ... There is an
intellectual struggle going on for the very soul of theoretical
physics, and for the hearts and minds of young scientists entering
our field," says physicist Zlatko Tesanovic of Johns Hopkins
University.

String theorists and their foes can't even agree on what constitutes
success or failure. For example, the most unexpected and
counterintuitive discovery of recent science occurred in the 1990s,
when astrophysicists at Berkeley and elsewhere realized the universe
is expanding faster with time. The apparent reason: a mysterious dark
energy pervades space and drives the accelerated expansion.

Critics mock superstringers because their so-called theory of
everything failed to predict this colossal discovery. String
theorists fire back that no one else predicted it, either, and
besides, "string theory is the only approach that has the potential
for explaining dark energy" based on pure theory, says John Schwarz,
a pioneering string theorist at Caltech.

That's because string theory is the only existing hypothesis that
holds serious promise of merging the two grandest branches of
physics -- the theory of gravity, the basis of cosmological theory;
and quantum mechanics, the science of the subatomic realm, Schwarz
says.

Even so, "it's my impression that more and more physicists are
starting to join Krauss as 'skeptical agnostics' about string
theory," said mathematician Peter Woit of Columbia University, who
offers comments on string- theory developments at his blog:
www.math.columbia.edu/~woit/blog/.

One possible reason for the sea change is that string theory's
ambitions have radically changed since the 1980s. Back then,
theorists hoped to develop a string theory that would predict the
existence of one universe -- ours, of course -- with its given
physical forces and constants, such as the known intensity of gravity
and the known electrical charge on the electron.

In later years, though, string theorists realized their theory
predicted innumerable possible universes with widely varying physical
forces and constants. As usual, superstringers and their critics
viewed this development differently.

To critics, like Woit, it is a disaster for string theory because the
sheer number of estimated universes -- equal to the number one
followed by 500 zeroes -- is unimaginably large.

If true, it means that string theory is so flexible that it can be
used to predict almost any kind of universe you want, no matter how
crazy, and hence it predicts nothing specific enough to be
scientifically interesting.

"A theory that can't predict anything is not a scientific theory,"
Woit says.

But what if the universe is unimaginably complex and as jammed with
diverse universes as the seas are jammed with diverse fish? That's
the thesis of Kaku, who compares the history of string research
to "wandering around the desert and then stumbling on a tiny pebble.
But when we examine it carefully, we find that it's actually the tip
of a gigantic pyramid."

"But just as we are about to open the door," Kaku says, "some critics
say that it's taking too much time, that the writings are too hard to
understand, that (it) is draining resources from other projects, that
it's getting too much publicity, that the script seems to be mutating
as we go from floor to floor, et cetera, et cetera."

Opinions on the theory

In an informal Chronicle e-mail survey, the world's physicists
expressed widely differing, sometimes emotional, opinions on the
dispute over string theory:

-- "String theory is anything but a futile effort," said an e-mail
from David Gross of UC Santa Barbara, who shared the Nobel Prize in
physics last year. Among other accomplishments, it has enabled
physicists "to understand, finally, many of the mysteries of black
holes. ... I am convinced that string theory, as presently
understood, is on the right path, but that this path is quite long,
and (perhaps many) further breakthroughs are required."

-- "I agree entirely with Larry Krauss," says Nobel Prize-winning
physicist Philip Anderson of Princeton University. In academia, "we
from outside the (string) field are disturbed by our colleagues'
insistence that every new semi-adolescent who has done something in
string theory is the greatest genius since Einstein and therefore
must occupy yet another tenure track. ... Our sciences are becoming
increasingly infected with quasi-theology, a tendency which needs to
be openly debated."

-- "To the considerable extent that string theory has been developed,
it has turned out to be a logically consistent quantum theory of
gravity," says string theorist Raphael Bousso of Lawrence Berkeley
National Laboratory. "This is a very significant achievement utterly
unmatched by any other approach to this problem -- and many have been
tried over the past several decades."

-- "There has been, in recent years, a pernicious, uncritical hype of
string theory," says Carlo Rovelli of the Centre de Physique
Theorique in Marseille, France. While the theory is worth developing
and is a "very interesting attempt to address the fundamental open
problems of physics," he says, "so far it is only an attempt, (one)
that has delivered less than what was expected some years ago,"
and "its uncritical promotion is damaging to science."

-- Krauss' charge that string theory "has probably been the least
successful 'great' idea in physics" in a century is unfair and
premature, replies string physicist Brian Greene of Columbia
University, author of two acclaimed books on the topic,
including "The Elegant Universe." "That's like someone going into
Antonio Stradivari's workshop and complaining about the sound
produced by one of his as yet unfinished violins."

E-mail Keay Davidson at kdavidson@sfchronicle.com.





SUBJECT: Linking Life After Death To Subatomic Physics
FROM: "marcelomjr" <marcelomjr@hotmail.com>
TO: ciencialist@yahoogrupos.com.br
DATE: 15/03/2005 01:57


http://www.vnn.org/editorials/ET0212/ET26-7687.html

Linking Life After Death To Subatomic Physics

SUBMITTED BY PARAGATI DAS

EDITORIAL, Dec 26 (VNN) — Sir Oliver Lodge FRS: The Mode of Future
Existence. 1933 Lecture

Linking survival after death with sub atomic physics is censored in
Great Britain.

Introduction by Michael Roll.

This article by Sir Oliver Lodge was published in The Queen's
Hospital Annual in 1933 (Birmingham). It is because this great
scientist wrote articles and published books along these lines that
he has been vilified by obscurantists who have taken control of
scientific teaching throughout the world.

Sir Oliver Lodge was the first person to send a radio message, one
year before Marconi! His great contribution to science has been
deliberately played down solely because the powerful materialists are
terrified that millions may find out he was correct in saying that we
all survive death.

The same treatment has been meted out to Sir William Crookes who
actually proved by repeatable experiments under laboratory conditions
that the subject of survival after death is a branch of physics -
natural philosophy. Crookes was a President of the Royal Society,
inventor of the Cathode-ray tube. Look up x-rays in the
encyclopaedia, Crookes was the pioneer of subatomic physics - proving
that reality exists beyond our five physical senses.
(...)






SUBJECT: NETHER: Lew Price atualizou site...
FROM: "marcelomjr" <marcelomjr@hotmail.com>
TO: ciencialist@yahoogrupos.com.br
DATE: 15/03/2005 02:06


Olá,

Lew Paxton Price, criador da Nether (Dynamic Ether) Theory [
http://www.softcom.net/users/greebo/price.htm#main ] atualizou
recentemente seu site com algumas mais ilustraçoes e explicaçoes.
Veja em:

Magnetism:
http://www.softcom.net/users/greebo/magnet.htm

Time:
http://www.softcom.net/users/greebo/time.htm

Electron:
http://www.softcom.net/users/greebo/electron.htm

http://www.softcom.net/users/greebo/plncklen.htm


"Updated: March 12, 2005
Added more explanation and illustrations to the parts on
magnetism, the electron, and time. Also added information
on the new E-book version of Behind Light's Illusion."

http://www.softcom.net/users/greebo/updates.htm
----------------------

Eis alguns recentes comentários de Lew Paxton Price sobre a exatidão
e irrefutabilidade de sua "Nether Theory":

http://www.softcom.net/users/greebo/confirm.htm

"Comments as of February 23, 2005

There have been so many articles written on confirmations of what
theoretical physicists call "dark energy" and "dark matter" (two
separate things) that I am no longer keeping track. To date, the
mainline physicists have no idea what either is and are placing more
and more band-aids on their theories. Nothing has been found that
refutes the theory on this website - in fact, everything to date
merely provides more proof that this theory is correct.

According to articles written on the latest experiments regarding the
equations of relativity the equations of relativity still work. These
same equations have been derived from the foundations of the theory
found on this website. Therefore, the new data confirms what is here
as well as relativity, although relativity has been found to be
incorrect in some instances while nether theory is still intact."

Um abraço,
Marcelo M. Jr.
"Authority should always be questioned." - Lew Price





SUBJECT: THE *TROUBLE* WITH RELATIVITY
FROM: "marcelomjr" <marcelomjr@hotmail.com>
TO: ciencialist@yahoogrupos.com.br
DATE: 15/03/2005 02:21


Leia o texto na integra em:

http://www.wbabin.net/physics/marcus.htm

THE *TROUBLE* WITH RELATIVITY

Marcus Coleman

PROLOGUE -

In October '96 the following was published worldwide in the news
media:

GRAVITY LENS ADDS FOCUS TO UNIVERSE

Such was one Australian headline reporting an Australian research
group's announcement of having discovered the strongest incidence yet
of the "gravity lens system", which they hope to use to measure the
scale of the universe, the "gravity lens" effect of course being
based on Einstein's Relativity.

Yet it was also reported that although the new gravity lens was said
to be 10 times stronger than any other (and at radio
wavelengths "incredibly bright"), it greatly complicated matters,
with one researcher (J. Lovell) reported as saying "We thought we
understood what was involved, but now it's all uncertain again"
and "Really, this poses more questions than it answers."

(...)

Um abraço,
Marcelo M. Jr.





SUBJECT: Re: [ciencialist] Re: mulé do Einstein, sexta na TV
FROM: JVictor <jvoneto@uol.com.br>
TO: ciencialist@yahoogrupos.com.br
DATE: 15/03/2005 07:31




Victor: Ufa! Ainda bem que ele era normal! Palmas para prá ele. Já
pensaram se ele, além da genialidade incomparável, ainda fosse
"bonzinho", certinho, dentro do contexto das concepções dos
"puritanos". Aí, meu caro, deveria ser um cara muito chato, intragável,
deveria ser um porre!

Sds,.

Victor.




Alvaro Augusto (E) escreveu:

> Três? Que nada! Foi muito mais que isso! Dizem que a casa de Eistein
> em Caput era praticamente um abatedouro. Ele se engraçou até mesmo com
> a enteada...
>
> [ ]s
>
> Alvaro Augusto




>
>
> ----- Original Message -----
> From: Maria Natália
> To: ciencialist@yahoogrupos.com.br
> Sent: Sunday, March 13, 2005 1:41 AM
> Subject: [ciencialist] Re: mulé do Einstein, sexta na TV
>
>
>
> ...há pelo menos duas mulhé...a mãe e a mulher, propriamente dita.
> Neste caso eram 3.
> Pena não ser cá. Mas creio que já vimos há um ano ou dois. Foi falta
> de pontaria: deveria ter sido a 8 de Março...
> Maria Natália
>
>
> --- Em ciencialist@yahoogrupos.com.br, "L.E.R.de Carvalho"
> <lecarvalho@i...> escreveu
> >
> > []
> > Por trás de todo homem...
> >
>
> <http://sender.mailsender.com.br/Redir?0459f8ebb506c02718d4b29b11f586a0&423127752DA5856159EF1867DD277375
> <http://sender.mailsender.com.br/Redir?0459f8ebb506c02718d4b29b11f586a0&423127752DA5856159EF1867DD277375>>"A
>
> > Mulher de Einstein" (GNT, 22h45) conta a história de Mileva Maric,
> primeira
> > esposa do cientista. O documentário mostra como a contribuição de
> Mileva
> > para o desenvolvimento da teoria da relatividade e do efeito
> fotoelétrico
> > foi apagada pela história.
> >
> >
> > [As partes desta mensagem que não continham texto foram removidas]
>
>
>
>
> [As partes desta mensagem que não continham texto foram removidas]
>
>
>
> ##### ##### #####
>
> Para saber mais visite
> http://www.ciencialist.hpg.ig.com.br
>
>
> ##### ##### ##### #####
>
>
> *Yahoo! Grupos, um serviço oferecido por:*
>
> *
> <http://br.rd.yahoo.com/SIG=12a5kclp5/M=264105.3931087.6562589.1588051/D=brclubs/S=2137111528:HM/EXP=1110857233/A=2361264/R=6/SIG=10v4acpp0/*http://br.shopping.yahoo.com/>*
>
>
>
> ------------------------------------------------------------------------
> *Links do Yahoo! Grupos*
>
> * Para visitar o site do seu grupo na web, acesse:
> http://br.groups.yahoo.com/group/ciencialist/
>
> * Para sair deste grupo, envie um e-mail para:
> ciencialist-unsubscribe@yahoogrupos.com.br
> <mailto:ciencialist-unsubscribe@yahoogrupos.com.br?subject=Unsubscribe>
>
> * O uso que você faz do Yahoo! Grupos está sujeito aos Termos do
> Serviço do Yahoo! <http://br.yahoo.com/info/utos.html>.
>
>
>
>
> __________ Informação do NOD32 1.1024 (20050311) __________
>
> Esta mensagem foi verificada pelo NOD32 Sistema Antivírus
> http://www.nod32.com.br




SUBJECT: Re: [ciencialist] Fw: Calculadoras
FROM: TARCISIO BORGES <tbs97@fisica.ufpr.br>
TO: ciencialist <ciencialist@yahoogrupos.com.br>
DATE: 15/03/2005 07:51

Acho que ele quer saber quais as calculadoras que existiram depois do
ábaco.

Antes do advento do transistor existiam as calculadoras mecânicas. A mais
célebre é a CURTA.

[]s
TARCISIO BORGES
tbs97@fisica.ufpr.br

On Mon, 14 Mar 2005, Luiz Ferraz Netto wrote:
> Calculadoras antes do ábaco? Será que estou vivendo do avesso?
> Luiz Ferraz Netto [Léo]

> -----Mensagem Original-----
> De: "pimentaomucholoco" <pimentaomucholoco@ig.com.br>
> Eu queria saber quais as calculadoras que existiram dês de o ábaco, se
> possível me mande uma lista, muito obrigado.



SUBJECT: Re: [ciencialist] Linking Life After Death To Subatomic Physics
FROM: Leonardo Souza <leo_feynman@yahoo.com.br>
TO: ciencialist@yahoogrupos.com.br
DATE: 15/03/2005 09:48

Vaishnava News Network????? Esse site é de ciência ou de hare krishna???
Acho q nao é confiavel.....

Leo

marcelomjr <marcelomjr@hotmail.com> wrote:

http://www.vnn.org/editorials/ET0212/ET26-7687.html

Linking Life After Death To Subatomic Physics

SUBMITTED BY PARAGATI DAS

EDITORIAL, Dec 26 (VNN) — Sir Oliver Lodge FRS: The Mode of Future
Existence. 1933 Lecture

Linking survival after death with sub atomic physics is censored in
Great Britain.

Introduction by Michael Roll.

This article by Sir Oliver Lodge was published in The Queen's
Hospital Annual in 1933 (Birmingham). It is because this great
scientist wrote articles and published books along these lines that
he has been vilified by obscurantists who have taken control of
scientific teaching throughout the world.

Sir Oliver Lodge was the first person to send a radio message, one
year before Marconi! His great contribution to science has been
deliberately played down solely because the powerful materialists are
terrified that millions may find out he was correct in saying that we
all survive death.

The same treatment has been meted out to Sir William Crookes who
actually proved by repeatable experiments under laboratory conditions
that the subject of survival after death is a branch of physics -
natural philosophy. Crookes was a President of the Royal Society,
inventor of the Cathode-ray tube. Look up x-rays in the
encyclopaedia, Crookes was the pioneer of subatomic physics - proving
that reality exists beyond our five physical senses.
(...)






##### ##### #####

Para saber mais visite
http://www.ciencialist.hpg.ig.com.br


##### ##### ##### #####


Yahoo! Grupos, um serviço oferecido por:



















function SearchComboBox() { if (document.form_combo.keyword.value.length==0){ alert("Por favor, digite algo."); return false; }else { document.form_combo.action ="http://br.rd.yahoo.com/SIG=12a6c8spl/M=264105.3931087.6562589.1588051/D=brclubs/S=2137111528:HM/EXP=1110949083/A=2361264/R=0/SIG=11uaou2jn/*http://www.bondfaro.com/bondfaro/in/combosearch_in.jsp?sk=11"; } return true;} [input] [input] [input]

---------------------------------
Links do Yahoo! Grupos

Para visitar o site do seu grupo na web, acesse:
http://br.groups.yahoo.com/group/ciencialist/

Para sair deste grupo, envie um e-mail para:
ciencialist-unsubscribe@yahoogrupos.com.br

O uso que você faz do Yahoo! Grupos está sujeito aos Termos do Serviço do Yahoo!.



---------------------------------
Yahoo! Mail - Com 250MB de espaço. Abra sua conta!

[As partes desta mensagem que não continham texto foram removidas]



SUBJECT: Re: Fw: Resposta reduzida
FROM: "rmtakata" <rmtakata@altavista.net>
TO: ciencialist@yahoogrupos.com.br
DATE: 15/03/2005 10:09


--- Em ciencialist@yahoogrupos.com.br, "Luiz Ferraz Netto"
> ===========================
> Luiz Ferraz Netto [Léo]
> -----Mensagem Original-----
> De: Janaina Ramos
> Sexta feira minha professora de biologia, mandou respondermos umas
> perguntas para revizar matéria. Procurei respostas rezumidas por
> toda internet mas não consigo acessar um site mais direto. Preciso
> saber oque são?
> 1- Célula
> 2- Átomo
> 3- Moléculas
> 4- Substâncias
> 5- Tecidos
> 6- Órgãos
> 7- Sistemas
> 8- Aparelhos

Se eh pra *revisar* a materia, suponho q. a professora tenha jah
explicado isso. Entao nao eh preciso procurar na internet, mas dar uma
lida nas anotacoes de aula e no livro-texto.

[]s,

Roberto Takata





SUBJECT: Re: Fw: duvida - fossas
FROM: "rmtakata" <rmtakata@altavista.net>
TO: ciencialist@yahoogrupos.com.br
DATE: 15/03/2005 10:13


http://www.absoluteastronomy.com/encyclopedia/o/oc/oceanic_trench.htm
http://en.wikipedia.org/wiki/Oceanic_trench
http://home.arcor.de/be/bethge/philippinetrench.htm
http://www.gsajournals.org/gsaonline/?request=get-abstract&doi=10.1130%2F0091-7613(1991)019%3C0032:PFAKFP%3E2.3.CO%3B2

[]s,

Roberto Takata

--- Em ciencialist@yahoogrupos.com.br, "Luiz Ferraz Netto"
> Quem entende de fossa?
> Luiz Ferraz Netto [Léo]
> -----Mensagem Original-----
> De: Sandra da Silva Correa
> Meu nome é Sandra, sou da Unespar-Fecilcam, estou cursando Geografia
> (1º ano )e moro em Campo Mourão-PR, e gostaría de obter informações
> sobre Fossas oceanicas, em especial a Fossa das Filipinas.
> Grata
> Sandra





SUBJECT: Re: O DESAFIO DA RELATIVIDADE PIRELLI
FROM: "rmtakata" <rmtakata@altavista.net>
TO: ciencialist@yahoogrupos.com.br
DATE: 15/03/2005 10:16


--- Em ciencialist@yahoogrupos.com.br, "Marcella"
> Estou empenhada fazendo um projeto para o desafio Pirelli.
> Queria saber se alguém pode me ajudar nas revisões,pois ainda não
> aprendi essa matéria na escola.

Se quiser, posso sim lhe dar uma ajuda - ainda q nao especializada.

> Onde vocês souberam que as inscrições foram prorrogadas?

No proprio site do desafio:

"2. Submissions must be sent by FTP or by an e-mail attachment before
September 15, 2005 to the adresses the respective links."
http://www.pirelliaward.com/einstein.html

[]s,

Roberto Takata





SUBJECT: Re: [ciencialist] Re: Fw: Resposta reduzida
FROM: "Itabajara Vaz - UFRGS" <ita@cbiot.ufrgs.br>
TO: ciencialist@yahoogrupos.com.br
DATE: 15/03/2005 10:20

parece que a gurizada acha que pesquisar eh faze perguntas para os outros..


On 15 Mar 2005 at 13:09, rmtakata wrote:

>
>
> --- Em ciencialist@yahoogrupos.com.br, "Luiz Ferraz Netto"
> > ===========================
> > Luiz Ferraz Netto [Léo]
> > -----Mensagem Original-----
> > De: Janaina Ramos
> > Sexta feira minha professora de biologia, mandou respondermos umas
> > perguntas para revizar matéria. Procurei respostas rezumidas por
> > toda internet mas não consigo acessar um site mais direto. Preciso
> > saber oque são?
> > 1- Célula
> > 2- Átomo
> > 3- Moléculas
> > 4- Substâncias
> > 5- Tecidos
> > 6- Órgãos
> > 7- Sistemas
> > 8- Aparelhos
>
> Se eh pra *revisar* a materia, suponho q. a professora tenha jah
> explicado isso. Entao nao eh preciso procurar na internet, mas dar uma
> lida nas anotacoes de aula e no livro-texto.
>
> []s,
>
> Roberto Takata
>
>
>
>
>
> ##### ##### #####
>
> Para saber mais visite
> http://www.ciencialist.hpg.ig.com.br
>
>
> ##### ##### ##### #####
> Links do Yahoo! Grupos
>
>
>
>
>
>
>
>




SUBJECT: off-topic: Fwd: Dia 19 de março no É de Lei
FROM: "Eurico Ferreira de Souza Jr." <caodejah@yahoo.com.br>
TO: acropolis@yahoogrupos.com.br
CC: ciencialist@yahoogrupos.com.br
DATE: 15/03/2005 11:38



Naíme Silva <noreply-orkut@google.com> wrote:Para: "Eurico (cão)_\|/_" <caodejah@yahoo.com.br>
De: "Naíme Silva" <noreply-orkut@google.com>
Data: Mon, 14 Mar 2005 21:09:17 -0800 (PST)
Assunto: Dia 19 de março no É de Lei

Galera da Fumaça,

Venho lembrar o que ja sabem pelo email que mandei par Gabeira e outras mensagens que dia 19 de março, sabado, as 14 horas, haverá na sede do É de Lei a primeira reunião de organização e formação Politica sobre a Legalização da Maconha no Brasil e a Passeta Verde 2005.
Pretendemos convidar pessoas da academia e de outros movimentos sociais para nos ajudarem neste debate.
Gostaria de destacar e solicitar a presença das pessoas abaixo:
1. Sonia Francine - Vereadora
2. Carlos Gianazzi- Vereador
5.Regina Facchini- Associação da Parada GLBT-Espaço B
6.Dinalva Tavares- União Brasileira de Mulheres
7.Regina Figueiredo- Rede CLAE
8.Fabiana Brites - Advogada do Centro de Defesa Faça o que tu queres pois é tudo da LEI/É de Lei/SP
9. Centros Academicos das Universidades
10.Kleber Santana- Educadores Municipais de SP/SINPEEM
11. Rafael Adaime- Pesquisador PUC/SP Naip
12.Monica Gorgulho- Psicologa, Dinamo/IHRA
13.Luis Paulo Guanabara- Psicotropicos-RJ

Pauta:
1. Filme da Passeta Verde 2004 e Avaliação
2. Eleição de Comissão Orgainizadora e Divisão de Tarefas
3. Articulação entre Movimentos Sociais
4. Qual o Cenário Nacional e MUndial da Politica de DRogas no Brasil

Esperamos que todos que recebam este email o divulguem para o maximo possivel de pessoas interessadas numa discussão seria e estrategica da Legalização da Canabis.

Então não esqueçam:
Primeira Reunião de Organização e Reflexão sobre a Legalização da Canabis no Brasil
Data: 19 de março de 2005(sabado)
horario: 14 horas
local: Centro de Convivencia É de Lei Rua 24 de Maio, 116 Galeria Presidente, 4º andar, salas 36 e 37
Republica - tel: (11) 3337 6049 email: edelei@terra.com.br

Aguardamos vcs,

Bjos enfumaçados

Naíme Silva- Presidente do É de Lei

This message was sent by Naíme Silva to friends.

* * *

http://www.orkut.com/


To control notification emails, access your Account Settings:

http://www.orkut.com/Settings.aspx



_\|/_

---------------------------------
Yahoo! Messenger 6.0 - jogos, emoticons sonoros e muita diversão. Instale agora!

[As partes desta mensagem que não continham texto foram removidas]



SUBJECT: Dia 19 de março no É d e Lei
FROM: "L.E.R.de Carvalho" <lecarvalho@infolink.com.br>
TO: ciencialist@yahoogrupos.com.br
DATE: 15/03/2005 11:43

Me parece lamentavel que pessoas queiram defender o uso de substancias para
alterar estados de consciencia.

A vida propicia essas mesmas substancias de forma natural.

Morpheus quer falar contigo.

L.E.


At 11:38 15/3/2005, you wrote:


>Naíme Silva <noreply-orkut@google.com> wrote:Para: "Eurico (cão)_\|/_"
><caodejah@yahoo.com.br>
>De: "Naíme Silva" <noreply-orkut@google.com>
>Data: Mon, 14 Mar 2005 21:09:17 -0800 (PST)
>Assunto: Dia 19 de março no É de Lei
>
>Galera da Fumaça,
>
>Venho lembrar o que ja sabem pelo email que mandei par Gabeira e outras
>mensagens que dia 19 de março, sabado, as 14 horas, haverá na sede do É de
>Lei a primeira reunião de organização e formação Politica sobre a
>Legalização da Maconha no Brasil e a Passeta Verde 2005.
>Pretendemos convidar pessoas da academia e de outros movimentos sociais
>para nos ajudarem neste debate.
>Gostaria de destacar e solicitar a presença das pessoas abaixo:
>1. Sonia Francine - Vereadora
>2. Carlos Gianazzi- Vereador
>5.Regina Facchini- Associação da Parada GLBT-Espaço B
>6.Dinalva Tavares- União Brasileira de Mulheres
>7.Regina Figueiredo- Rede CLAE
>8.Fabiana Brites - Advogada do Centro de Defesa Faça o que tu queres pois
>é tudo da LEI/É de Lei/SP
>9. Centros Academicos das Universidades
>10.Kleber Santana- Educadores Municipais de SP/SINPEEM
>11. Rafael Adaime- Pesquisador PUC/SP Naip
>12.Monica Gorgulho- Psicologa, Dinamo/IHRA
>13.Luis Paulo Guanabara- Psicotropicos-RJ
>
>Pauta:
>1. Filme da Passeta Verde 2004 e Avaliação
>2. Eleição de Comissão Orgainizadora e Divisão de Tarefas
>3. Articulação entre Movimentos Sociais
>4. Qual o Cenário Nacional e MUndial da Politica de DRogas no Brasil
>
>Esperamos que todos que recebam este email o divulguem para o maximo
>possivel de pessoas interessadas numa discussão seria e estrategica da
>Legalização da Canabis.
>
>Então não esqueçam:
>Primeira Reunião de Organização e Reflexão sobre a Legalização da Canabis
>no Brasil
>Data: 19 de março de 2005(sabado)
>horario: 14 horas
>local: Centro de Convivencia É de Lei Rua 24 de Maio, 116 Galeria
>Presidente, 4º andar, salas 36 e 37
>Republica - tel: (11) 3337 6049 email: edelei@terra.com.br
>
>Aguardamos vcs,
>
>Bjos enfumaçados
>
>Naíme Silva- Presidente do É de Lei
>
>This message was sent by Naíme Silva to friends.
>
>* * *
>
><http://www.orkut.com/>http://www.orkut.com/
>
>
>To control notification emails, access your Account Settings:
>
><http://www.orkut.com/Settings.aspx>http://www.orkut.com/Settings.aspx
>
>
>
>_\|/_
>
>---------------------------------
>Yahoo! Messenger 6.0 - jogos, emoticons sonoros e muita diversão. Instale
>agora!
>
>[As partes desta mensagem que não continham texto foram removidas]
>
>
>
>##### ##### #####
>
>Para saber mais visite
><http://www.ciencialist.hpg.ig.com.br>http://www.ciencialist.hpg.ig.com.br
>
>
>##### ##### ##### #####
>
>
>Yahoo! Grupos, um serviço oferecido por:
>[]
>
><http://br.rd.yahoo.com/SIG=12agekuvo/M=264105.3931087.6562589.1588051/D=brclubs/S=2137111528:HM/EXP=1110983932/A=2361264/R=6/SIG=10v4acpp0/*http://br.shopping.yahoo.com/>
>[]
>
>
>
>----------
>Links do Yahoo! Grupos
> * Para visitar o site do seu grupo na web, acesse:
> *
> <http://br.groups.yahoo.com/group/ciencialist/>http://br.groups.yahoo.com/group/ciencialist/
>
> *
> * Para sair deste grupo, envie um e-mail para:
> *
> <mailto:ciencialist-unsubscribe@yahoogrupos.com.br?subject=Unsubscribe>ciencialist-unsubscribe@yahoogrupos.com.br
>
> *
> * O uso que você faz do Yahoo! Grupos está sujeito aos
> <http://br.yahoo.com/info/utos.html>Termos do Serviço do Yahoo!.


[As partes desta mensagem que não continham texto foram removidas]



SUBJECT: Resposta reduzida
FROM: "L.E.R.de Carvalho" <lecarvalho@infolink.com.br>
TO: ciencialist@yahoogrupos.com.br
DATE: 15/03/2005 11:45

Ainda com relação às empresas que vendem teses e monografias...

Eu só queria saber a diferença entre...
pedir respostas pra essas perguntas, na internet...

e pagar pra alguém fazer isso pro aluno.


Eu só vejo uma diferença:
é que um paga e o outro, pior ainda, mais burro ainda, acha que existem
otários pra oferecer isso de graça.

Internet não é pra isso.

Sites de Ciencia não são pra isso.

L.E.



At 10:20 15/3/2005, you wrote:
>parece que a gurizada acha que pesquisar eh faze perguntas para os outros..
>
>
>On 15 Mar 2005 at 13:09, rmtakata wrote:
>
> >
> >
> > --- Em ciencialist@yahoogrupos.com.br, "Luiz Ferraz Netto"
> > > ===========================
> > > Luiz Ferraz Netto [Léo]
> > > -----Mensagem Original-----
> > > De: Janaina Ramos
> > > Sexta feira minha professora de biologia, mandou respondermos umas
> > > perguntas para revizar matéria. Procurei respostas rezumidas por
> > > toda internet mas não consigo acessar um site mais direto. Preciso
> > > saber oque são?
> > > 1- Célula
> > > 2- Átomo
> > > 3- Moléculas
> > > 4- Substâncias
> > > 5- Tecidos
> > > 6- Órgãos
> > > 7- Sistemas
> > > 8- Aparelhos
> >
> > Se eh pra *revisar* a materia, suponho q. a professora tenha jah
> > explicado isso. Entao nao eh preciso procurar na internet, mas dar uma
> > lida nas anotacoes de aula e no livro-texto.
> >
> > []s,
> >
> > Roberto Takata
> >
> >
> >
> >
> >
> > ##### ##### #####
> >
> > Para saber mais visite
> > <http://www.ciencialist.hpg.ig.com.br>http://www.ciencialist.hpg.ig.com.br
> >
> >
> > ##### ##### ##### #####
> > Links do Yahoo! Grupos
> >
> >
> >
> >
> >
> >
> >
> >
>
>
>
>
>##### ##### #####
>
>Para saber mais visite
><http://www.ciencialist.hpg.ig.com.br>http://www.ciencialist.hpg.ig.com.br
>
>
>##### ##### ##### #####
>
>
>Yahoo! Grupos, um serviço oferecido por:
>[]
>
><http://br.rd.yahoo.com/SIG=12asb5llo/M=264105.3931087.6562589.1588051/D=brclubs/S=2137111528:HM/EXP=1110978982/A=2361264/R=6/SIG=10v4acpp0/*http://br.shopping.yahoo.com/>
>[]
>
>
>
>----------
>Links do Yahoo! Grupos
> * Para visitar o site do seu grupo na web, acesse:
> *
> <http://br.groups.yahoo.com/group/ciencialist/>http://br.groups.yahoo.com/group/ciencialist/
>
> *
> * Para sair deste grupo, envie um e-mail para:
> *
> <mailto:ciencialist-unsubscribe@yahoogrupos.com.br?subject=Unsubscribe>ciencialist-unsubscribe@yahoogrupos.com.br
>
> *
> * O uso que você faz do Yahoo! Grupos está sujeito aos
> <http://br.yahoo.com/info/utos.html>Termos do Serviço do Yahoo!.


[As partes desta mensagem que não continham texto foram removidas]



SUBJECT: Re: Dia 19 de março no É d e Lei
FROM: "rmtakata" <rmtakata@altavista.net>
TO: ciencialist@yahoogrupos.com.br
DATE: 15/03/2005 11:54


Nao sei o q. isso tem exatamente a ver com ciencia. Mas enfim,
direitos elas tEm para discutir o tema e procurar mudar a lei sobre
drogas.

[]s,

Roberto Takata

--- Em ciencialist@yahoogrupos.com.br, "L.E.R.de Carvalho"
> Me parece lamentavel que pessoas queiram defender o uso de
> substancias para alterar estados de consciencia.
>
> A vida propicia essas mesmas substancias de forma natural.
>
> Morpheus quer falar contigo.
>
> L.E.
>
>
> At 11:38 15/3/2005, you wrote:
> >Naíme Silva <noreply-orkut@g...> wrote:Para: "Eurico (cão)_\|/_"
> >Venho lembrar o que ja sabem pelo email que mandei par Gabeira e
outras
> >mensagens que dia 19 de março, sabado, as 14 horas, haverá na sede
do É de
> >Lei a primeira reunião de organização e formação Politica sobre a
> >Legalização da Maconha no Brasil e a Passeta Verde 2005.





SUBJECT: Geração "Ctrl C; Ctrl V"
FROM: "E m i l i a n o C h e m e l l o" <chemelloe@yahoo.com.br>
TO: <ciencialist@yahoogrupos.com.br>
DATE: 15/03/2005 12:12

Olá L.E.R, Olá Takata, Olá Léo, Olá a todos!

Como existe gente 'folgada' neste país! Eu recebo, diariamente, através
do NAEQ e do SAVEQ (respectivamente: www.ucs.br/ccet/defq/naeq e
www.ucs.br/ccet/defq/naeq/saveq ) várias mensagens neste estilo. Sugiro ao
Léo e aos demais que recebem este tipo de 'piada' que eliminem sem dar
respostas. Desta forma, talvez eles se 'toquem' que não é por ai que eles
irão aprender. Acho que o objetivo destas nossas respostas é dar um
'suporte' ao aluno para que ele, sozinho, consiga chegar ao objetivo
principal: "aprender".

[ ] 's do Emiliano Chemello
emiliano@quimica.net
http://www.quimica.net/emiliano
http://www.ucs.br/ccet/defq/naeq
[ MSN ] chemelloe@hotmail.com
[ ICQ ] 145060604

" Rien ne se perd, rien ne se crée,
tout se transforme."

Antoine Laurent de Lavoisier (químico francês, 1743 - 1794)

----- Original Message -----
From: L.E.R.de Carvalho
To: ciencialist@yahoogrupos.com.br
Sent: Tuesday, March 15, 2005 11:45 AM
Subject: [ciencialist] Resposta reduzida


Ainda com relação às empresas que vendem teses e monografias...

Eu só queria saber a diferença entre...
pedir respostas pra essas perguntas, na internet...

e pagar pra alguém fazer isso pro aluno.


Eu só vejo uma diferença:
é que um paga e o outro, pior ainda, mais burro ainda, acha que existem
otários pra oferecer isso de graça.

Internet não é pra isso.

Sites de Ciencia não são pra isso.

L.E.



At 10:20 15/3/2005, you wrote:
>parece que a gurizada acha que pesquisar eh faze perguntas para os outros..
>
>
>On 15 Mar 2005 at 13:09, rmtakata wrote:
>
> >
> >
> > --- Em ciencialist@yahoogrupos.com.br, "Luiz Ferraz Netto"
> > > ===========================
> > > Luiz Ferraz Netto [Léo]
> > > -----Mensagem Original-----
> > > De: Janaina Ramos
> > > Sexta feira minha professora de biologia, mandou respondermos umas
> > > perguntas para revizar matéria. Procurei respostas rezumidas por
> > > toda internet mas não consigo acessar um site mais direto. Preciso
> > > saber oque são?
> > > 1- Célula
> > > 2- Átomo
> > > 3- Moléculas
> > > 4- Substâncias
> > > 5- Tecidos
> > > 6- Órgãos
> > > 7- Sistemas
> > > 8- Aparelhos
> >
> > Se eh pra *revisar* a materia, suponho q. a professora tenha jah
> > explicado isso. Entao nao eh preciso procurar na internet, mas dar uma
> > lida nas anotacoes de aula e no livro-texto.
> >
> > []s,
> >
> > Roberto Takata




SUBJECT: Fw: bomba de carboneto
FROM: "Luiz Ferraz Netto" <leobarretos@uol.com.br>
TO: "ciencialist" <ciencialist@yahoogrupos.com.br>
DATE: 15/03/2005 13:33

Cuimicos ..... ação!
[]'
===========================
Luiz Ferraz Netto [Léo]
leobarretos@uol.com.br
http://www.feiradeciencias.com.br
===========================
-----Mensagem Original-----
De: fabiana campos pereira
Para: leobarretos@uol.com.br
Enviada em: terça-feira, 15 de março de 2005 00:07
Assunto: bomba de carboneto


Olá professor! Meu nome é Fabiana Campos e sou de Belo Horizonte.
Estou fazendo um trabalho de Química e preciso explicar a reação da bomba de carboneto; e eu não consegui nenhum site que possa me fornecer essas informações.
Por gentileza, o senhor poderia me enviar a explicação no meu e-mail?!
Caso possa, desde já, fico agradecida.
Obrigada,
Fabiana Campos.


--------------------------------------------------------------------------------
Yahoo! Mail - Com 250MB de espaço. Abra sua conta!


--------------------------------------------------------------------------------


Internal Virus Database is out-of-date.
Checked by AVG Anti-Virus.
Version: 7.0.300 / Virus Database: 266.5.0 - Release Date: 25/02/2005

----------

Internal Virus Database is out-of-date.
Checked by AVG Anti-Virus.
Version: 7.0.300 / Virus Database: 266.5.0 - Release Date: 25/02/2005


[As partes desta mensagem que não continham texto foram removidas]



SUBJECT: Fw: Magnetismo
FROM: "Luiz Ferraz Netto" <leobarretos@uol.com.br>
TO: "ciencialist" <ciencialist@yahoogrupos.com.br>
DATE: 15/03/2005 13:39

Dou um prêmio (muito parecido com o Nobel) para quem responder adequadamente essas perguntas.
[]'
===========================
Luiz Ferraz Netto [Léo]
leobarretos@uol.com.br
http://www.feiradeciencias.com.br
===========================
-----Mensagem Original-----
De: lucasbittar
Para: leobarretos
Enviada em: domingo, 13 de março de 2005 20:31
Assunto: Re: Magnetismo


Prezado Leo

Tenho mais algumas duvidas sobre imas: Se eu juntar um ima com outro obterei um ima mais forte?;Quanto tempo leva para um ima NdfeB e para um ima SmCo perderem seu poder de magnetizacao em condicoes normais e temperatura ambiente?;e por ultimo, como faco para saber o alcance do campo magnetico de um ima? Por exemplo, existem imas que so atraem clips quando estao bem proximos enquanto outros atraem de distancias maiores. Qual e o termo cientifico para medir este alcance?Preciso atrair uma esfera metalica de 0,6cm. de diametro por uma rampa de 7cm inclinada em 30 graus.

Grato

Lucas


--------------------------------------------------------------------------------


Internal Virus Database is out-of-date.
Checked by AVG Anti-Virus.
Version: 7.0.300 / Virus Database: 266.5.0 - Release Date: 25/02/2005

----------

Internal Virus Database is out-of-date.
Checked by AVG Anti-Virus.
Version: 7.0.300 / Virus Database: 266.5.0 - Release Date: 25/02/2005


[As partes desta mensagem que não continham texto foram removidas]



SUBJECT: Fw: Pergunta! Uma boa pergunta
FROM: "Luiz Ferraz Netto" <leobarretos@uol.com.br>
TO: "ciencialist" <ciencialist@yahoogrupos.com.br>
DATE: 15/03/2005 13:45

Realmente, gostei da pergunta: "Aqui na alface da Terra qual o equipamento (aparelho, máquina, processo .... ) experimenta a maior aceleração (mesmo que seja por pequeno intervalo de tempo)?
É aquele carrinho esquisito que atinge 500 km/h? É a bala de fuzil?

[]'
===========================
Luiz Ferraz Netto [Léo]
leobarretos@uol.com.br
http://www.feiradeciencias.com.br
===========================
-----Mensagem Original-----
De: "Saulo Fulanete Trombini" <sauloft@pop.com.br>
Para: <leobarretos@uol.com.br>
Enviada em: sábado, 12 de março de 2005 10:53
Assunto: Pergunta!


Prezado Professor,
Baseado no fato de que o senhor tem muito mais comnhecimento, e recursos do
que eu, venho através desta, lhe perguntar: Qual a maior aceleração mecânica
terrestre? Parabenizo-o pelo seu excelente site, pois através dele estou
melhorando meus conhecimentos, e adquirindo novas idéias que posteriormente
levarei a Feira de Ciências da minha escola.

Agradecendo, despeço-me;

Saulo Fulanete Trombini



--
Internal Virus Database is out-of-date.
Checked by AVG Anti-Virus.
Version: 7.0.300 / Virus Database: 266.5.0 - Release Date: 25/02/2005




--
Internal Virus Database is out-of-date.
Checked by AVG Anti-Virus.
Version: 7.0.300 / Virus Database: 266.5.0 - Release Date: 25/02/2005



SUBJECT: Re: Fw: bomba de carboneto
FROM: "rmtakata" <rmtakata@altavista.net>
TO: ciencialist@yahoogrupos.com.br
DATE: 15/03/2005 13:53


Carboneto ou carbureto?

O carbureto de calcio reage com a agua e libera acetileno - o gas
aumenta a pressao do recipiente ate' arrebenta'-lo. Se houver uma
fonte de calor por perto, o acetileno pode entrar em ignicao e
provocar uma explosao ainda mais violenta.

O q. eh q. esse pessoal anda aprendendo? Estao fazendo o curso da Al
Qaeda?

[]s,

Roberto Takata

--- Em ciencialist@yahoogrupos.com.br, "Luiz Ferraz Netto"
> Cuimicos ..... ação!
> []'
> ===========================
> Luiz Ferraz Netto [Léo]
> -----Mensagem Original-----
> De: fabiana campos pereira
> Olá professor! Meu nome é Fabiana Campos e sou de Belo Horizonte.
> Estou fazendo um trabalho de Química e preciso explicar a reação da
> bomba de carboneto; e eu não consegui nenhum site que possa me





SUBJECT: Re: Foucault / Luz ondas ou corpusculos.
FROM: Hélio Ricardo Carvalho <hrc@fis.puc-rio.br>
TO: ciencialist@yahoogrupos.com.br
DATE: 15/03/2005 13:57


Victor,

Creio que você ainda não entendeu as minhas motivações ao começar esta
thread.
vou aqui tentar esclarecer.
Você disse:
>Victor: O modêlo ondulatório conduz diretamente a que a velocidada
>da luz num meio mais refringente é menor que essa velocidade em
>um meio menos refrigente. Tanto experimental como
>teoricamente! O contrário para um modelo corpuscular.
>Honestamente, não sei como chegar a resultados diferentes.

Experimental sim, "teoricamente" NÃO.
A luz (seja o que ela for) é mais lenta em meios mais refringentes
(~mais densos), isto é o FATO. A experiência de Foucault mostrou isto.
Mas que teoria usar para explicar este fato? Usando só teorias da
época, acho que este experimento sozinho reforça a luz corpuscular e
descarta a ondulatória.
(Explico este raciocínio mais abaixo)
Claro que, APESAR deste experimento (Foucault) ter dito isto, outros,
como aquele de Fresnel/Arago, diziam o contrário.
Meu objetivo nesta thread é descobrir porque um experimento como o de
Foucault que tinha tudo para descartar a teoria ondulatória pode ser
apontado como O que a consagrou????

Explicando:
Quais eram os fenômenos tidos como ondulatórios na época?
Creio que todos eles mostravam uma velocidade maior em meios mais
densos. Isto fazia sentido pois quanto mais matéria mais fácil e
rápido a transmissão de movimentos (energia) entre porções de matéria
adjacentes.
No caso de corpusculos deveria acontecer ao contrário.
Alguma coisa do tipo "quanto mais denso maior a secção de choque" ou
algo do gênero.

Este é o motivo da minha pergunta:

"Por que a experiência de Foucault não descartou de vez a luz
ondulatória também????"

Existe aquela possibilidade que escrevi na MSG anterior com uma certa
ironia:
[[[
"Claro que alguém poderia dizer: o som usa o ar e a matéria sólida e a
luz usa o ether logo onde tem matéria mais densa não sobra muito
espaço para o ether. [:-)] A luz é mais lenta na matéria mais densa
porque aí tem menos ether. [:-)]"
]]]

Será que foi por aí???????

Quando você diz:
>Bem, não acho que os resultados de Foulcault sejam dúbios. Mas é o
>que eu acho, em função do que sei.

Você acha isto em função do que você sabe HOJE. Eu também não acho
HOJE que os resultados sejam dúbios para a escolha de modelo para luz.

Mas eu estou sempre me reportando à época!!!

Só para terminar.
Se NA ÉPOCA eu fosse um cientista com C maiúsculo e só visse o
resultado de Foucault, eu iria dizer que a luz NÃO era ondulatória

Hélio.






SUBJECT: Fw: Quero uma ajuda
FROM: "Luiz Ferraz Netto" <leobarretos@uol.com.br>
TO: "ciencialist" <ciencialist@yahoogrupos.com.br>
DATE: 15/03/2005 14:13

Primeiro, preciso interpretar ....... depois .......

[]'
===========================
Luiz Ferraz Netto [Léo]
leobarretos@uol.com.br
http://www.feiradeciencias.com.br
===========================
-----Mensagem Original-----
De: Morquimica RJ
Para: leobarretos@uol.com.br
Enviada em: sexta-feira, 11 de março de 2005 18:27
Assunto: Quero uma ajuda



Nas dúvidas experimentais, por gentileza coloque aqui o endereço da página, isso facilita o confronto. Agradeço. Meu nome é LUIZ FERRAZ NETTO, meu apelido é LÉO e moro em BARRETOS; dai vem meu e-mail: leobarretos@uol.com.br.

Oi Sr Luiz, Boa Noite,

Eu estou fazendo um trabalho na minha escola e um profº, me fez uma pergunta para que eu desse uma resposta a ele sobre a Força Motriz a pergunta é a seguinte:


Porque o arroz conzinha com a força motriz e não entendi,

pois até que eu saiba a força motriz seria duas substâncias em contato uma com a outra existindo uma interação formando uma ligação, seria isso força motriz.

Por favor tenta me ajudar

agradeço

Luiz Carlos de Araujo.
email - araujo_454@hotmail.com


--------------------------------------------------------------------------------


Internal Virus Database is out-of-date.
Checked by AVG Anti-Virus.
Version: 7.0.300 / Virus Database: 266.5.0 - Release Date: 25/02/2005

----------

Internal Virus Database is out-of-date.
Checked by AVG Anti-Virus.
Version: 7.0.300 / Virus Database: 266.5.0 - Release Date: 25/02/2005


[As partes desta mensagem que não continham texto foram removidas]



SUBJECT: Re: [ciencialist] Fw: Pergunta! Uma boa pergunta
FROM: "E m i l i a n o C h e m e l l o" <chemelloe@yahoo.com.br>
TO: <ciencialist@yahoogrupos.com.br>
DATE: 15/03/2005 14:18

Não fiz muita pesquisa... mas acho que uma boa referência é o site do guinness:
http://www.guinnessworldrecords.com/gwr5/content_pages/record.asp?recordid=43540

[ ] 's do Emiliano Chemello
emiliano@quimica.net
http://www.quimica.net/emiliano
http://www.ucs.br/ccet/defq/naeq
[ MSN ] chemelloe@hotmail.com
[ ICQ ] 145060604

" Rien ne se perd, rien ne se crée,
tout se transforme."

Antoine Laurent de Lavoisier (químico francês, 1743 - 1794)

----- Original Message -----
From: Luiz Ferraz Netto
To: ciencialist
Sent: Tuesday, March 15, 2005 1:45 PM
Subject: [ciencialist] Fw: Pergunta! Uma boa pergunta


Realmente, gostei da pergunta: "Aqui na alface da Terra qual o equipamento (aparelho, máquina, processo .... ) experimenta a maior aceleração (mesmo que seja por pequeno intervalo de tempo)?
É aquele carrinho esquisito que atinge 500 km/h? É a bala de fuzil?

[]'
===========================
Luiz Ferraz Netto [Léo]
leobarretos@uol.com.br
http://www.feiradeciencias.com.br
===========================
-----Mensagem Original-----
De: "Saulo Fulanete Trombini" <sauloft@pop.com.br>
Para: <leobarretos@uol.com.br>
Enviada em: sábado, 12 de março de 2005 10:53
Assunto: Pergunta!


Prezado Professor,
Baseado no fato de que o senhor tem muito mais comnhecimento, e recursos do
que eu, venho através desta, lhe perguntar: Qual a maior aceleração mecânica
terrestre? Parabenizo-o pelo seu excelente site, pois através dele estou
melhorando meus conhecimentos, e adquirindo novas idéias que posteriormente
levarei a Feira de Ciências da minha escola.

Agradecendo, despeço-me;

Saulo Fulanete Trombini



--
Internal Virus Database is out-of-date.
Checked by AVG Anti-Virus.
Version: 7.0.300 / Virus Database: 266.5.0 - Release Date: 25/02/2005




--
Internal Virus Database is out-of-date.
Checked by AVG Anti-Virus.
Version: 7.0.300 / Virus Database: 266.5.0 - Release Date: 25/02/2005



##### ##### #####

Para saber mais visite
http://www.ciencialist.hpg.ig.com.br


##### ##### ##### #####


Yahoo! Grupos, um serviço oferecido por:







------------------------------------------------------------------------------
Links do Yahoo! Grupos

a.. Para visitar o site do seu grupo na web, acesse:
http://br.groups.yahoo.com/group/ciencialist/

b.. Para sair deste grupo, envie um e-mail para:
ciencialist-unsubscribe@yahoogrupos.com.br

c.. O uso que você faz do Yahoo! Grupos está sujeito aos Termos do Serviço do Yahoo!.



[As partes desta mensagem que não continham texto foram removidas]



SUBJECT: Re: [ciencialist] Fw: Pergunta! Uma boa pergunta
FROM: "Luiz Ferraz Netto" <leobarretos@uol.com.br>
TO: <ciencialist@yahoogrupos.com.br>
DATE: 15/03/2005 14:36

Oi Emiliano,

o interesse recai na aceleração e não na velocidade.
Mas, gostei do carrinho --- aqui em Barretos seria um furor. Vou ver em quantos segundos eu atravessaria a cidade.
[]'
===========================
Luiz Ferraz Netto [Léo]
leobarretos@uol.com.br
http://www.feiradeciencias.com.br
===========================
-----Mensagem Original-----
De: "E m i l i a n o C h e m e l l o" <chemelloe@yahoo.com.br>
Para: <ciencialist@yahoogrupos.com.br>
Enviada em: terça-feira, 15 de março de 2005 14:18
Assunto: Re: [ciencialist] Fw: Pergunta! Uma boa pergunta



Não fiz muita pesquisa... mas acho que uma boa referência é o site do guinness:
http://www.guinnessworldrecords.com/gwr5/content_pages/record.asp?recordid=43540

[ ] 's do Emiliano Chemello
emiliano@quimica.net
http://www.quimica.net/emiliano
http://www.ucs.br/ccet/defq/naeq
[ MSN ] chemelloe@hotmail.com
[ ICQ ] 145060604

" Rien ne se perd, rien ne se crée,
tout se transforme."

Antoine Laurent de Lavoisier (químico francês, 1743 - 1794)

----- Original Message -----
From: Luiz Ferraz Netto
To: ciencialist
Sent: Tuesday, March 15, 2005 1:45 PM
Subject: [ciencialist] Fw: Pergunta! Uma boa pergunta


Realmente, gostei da pergunta: "Aqui na alface da Terra qual o equipamento (aparelho, máquina, processo .... ) experimenta a maior aceleração (mesmo que seja por pequeno intervalo de tempo)?
É aquele carrinho esquisito que atinge 500 km/h? É a bala de fuzil?

[]'
===========================
Luiz Ferraz Netto [Léo]
leobarretos@uol.com.br
http://www.feiradeciencias.com.br
===========================
-----Mensagem Original-----
De: "Saulo Fulanete Trombini" <sauloft@pop.com.br>
Para: <leobarretos@uol.com.br>
Enviada em: sábado, 12 de março de 2005 10:53
Assunto: Pergunta!


Prezado Professor,
Baseado no fato de que o senhor tem muito mais comnhecimento, e recursos do
que eu, venho através desta, lhe perguntar: Qual a maior aceleração mecânica
terrestre? Parabenizo-o pelo seu excelente site, pois através dele estou
melhorando meus conhecimentos, e adquirindo novas idéias que posteriormente
levarei a Feira de Ciências da minha escola.

Agradecendo, despeço-me;

Saulo Fulanete Trombini



--
Internal Virus Database is out-of-date.
Checked by AVG Anti-Virus.
Version: 7.0.300 / Virus Database: 266.5.0 - Release Date: 25/02/2005




--
Internal Virus Database is out-of-date.
Checked by AVG Anti-Virus.
Version: 7.0.300 / Virus Database: 266.5.0 - Release Date: 25/02/2005



##### ##### #####

Para saber mais visite
http://www.ciencialist.hpg.ig.com.br


##### ##### ##### #####


Yahoo! Grupos, um serviço oferecido por:







------------------------------------------------------------------------------
Links do Yahoo! Grupos

a.. Para visitar o site do seu grupo na web, acesse:
http://br.groups.yahoo.com/group/ciencialist/

b.. Para sair deste grupo, envie um e-mail para:
ciencialist-unsubscribe@yahoogrupos.com.br

c.. O uso que você faz do Yahoo! Grupos está sujeito aos Termos do Serviço do Yahoo!.



[As partes desta mensagem que não continham texto foram removidas]



##### ##### #####

Para saber mais visite
http://www.ciencialist.hpg.ig.com.br


##### ##### ##### #####
Links do Yahoo! Grupos










--
Internal Virus Database is out-of-date.
Checked by AVG Anti-Virus.
Version: 7.0.300 / Virus Database: 266.5.0 - Release Date: 25/02/2005




--
Internal Virus Database is out-of-date.
Checked by AVG Anti-Virus.
Version: 7.0.300 / Virus Database: 266.5.0 - Release Date: 25/02/2005



SUBJECT: Re: [ciencialist] Fw: Pergunta! Uma boa pergunta
FROM: "E m i l i a n o C h e m e l l o" <chemelloe@yahoo.com.br>
TO: <ciencialist@yahoogrupos.com.br>
DATE: 15/03/2005 15:52

ops...

Fastest Acceleration.
The fastest road-tested acceleration on record is 0-60mph in 3.07 sec's by a
Ford RS2000 Evolution at the Millbrook Proving Ground UK May 1994.
fonte: http://www.ukcar.com/guinness/
---

Considerando que 60 mph (milhas por hora) equivalem a (arredondando) 100
km/h.

Já outra fonte diz que este é o carro mais rápido do mundo
"... Acceleration: 0-100 km/h (0-62 mph) 3.2 seconds..."
fonte: http://www.autoemirates.com/MiddleEast/2005/0127KoenigseggCCRME.asp

No site do guinness eu não achei referência sobre 'aceleração'. Eu sugiro
que se faça uma especificação de que tipo de "mecânica terrestre" se quer.

[ ] 's do Emiliano Chemello
emiliano@quimica.net
http://www.quimica.net/emiliano
http://www.ucs.br/ccet/defq/naeq
[ MSN ] chemelloe@hotmail.com
[ ICQ ] 145060604

" Rien ne se perd, rien ne se crée,
tout se transforme."

Antoine Laurent de Lavoisier (químico francês, 1743 - 1794)

----- Original Message -----
From: Luiz Ferraz Netto
To: ciencialist@yahoogrupos.com.br
Sent: Tuesday, March 15, 2005 2:36 PM
Subject: Re: [ciencialist] Fw: Pergunta! Uma boa pergunta


Oi Emiliano,

o interesse recai na aceleração e não na velocidade.
Mas, gostei do carrinho --- aqui em Barretos seria um furor. Vou ver em
quantos segundos eu atravessaria a cidade.
[]'
===========================
Luiz Ferraz Netto [Léo]
leobarretos@uol.com.br
http://www.feiradeciencias.com.br
===========================
-----Mensagem Original-----
De: "E m i l i a n o C h e m e l l o" <chemelloe@yahoo.com.br>
Para: <ciencialist@yahoogrupos.com.br>
Enviada em: terça-feira, 15 de março de 2005 14:18
Assunto: Re: [ciencialist] Fw: Pergunta! Uma boa pergunta



Não fiz muita pesquisa... mas acho que uma boa referência é o site do
guinness:
http://www.guinnessworldrecords.com/gwr5/content_pages/record.asp?recordid=43540

[ ] 's do Emiliano Chemello
emiliano@quimica.net
http://www.quimica.net/emiliano
http://www.ucs.br/ccet/defq/naeq
[ MSN ] chemelloe@hotmail.com
[ ICQ ] 145060604

" Rien ne se perd, rien ne se crée,
tout se transforme."

Antoine Laurent de Lavoisier (químico francês, 1743 - 1794)

----- Original Message -----
From: Luiz Ferraz Netto
To: ciencialist
Sent: Tuesday, March 15, 2005 1:45 PM
Subject: [ciencialist] Fw: Pergunta! Uma boa pergunta


Realmente, gostei da pergunta: "Aqui na alface da Terra qual o equipamento
(aparelho, máquina, processo .... ) experimenta a maior aceleração (mesmo
que seja por pequeno intervalo de tempo)?
É aquele carrinho esquisito que atinge 500 km/h? É a bala de fuzil?

[]'
===========================
Luiz Ferraz Netto [Léo]
leobarretos@uol.com.br
http://www.feiradeciencias.com.br
===========================
-----Mensagem Original-----
De: "Saulo Fulanete Trombini" <sauloft@pop.com.br>
Para: <leobarretos@uol.com.br>
Enviada em: sábado, 12 de março de 2005 10:53
Assunto: Pergunta!


Prezado Professor,
Baseado no fato de que o senhor tem muito mais comnhecimento, e
recursos do
que eu, venho através desta, lhe perguntar: Qual a maior aceleração
mecânica
terrestre? Parabenizo-o pelo seu excelente site, pois através dele estou
melhorando meus conhecimentos, e adquirindo novas idéias que
posteriormente
levarei a Feira de Ciências da minha escola.

Agradecendo, despeço-me;

Saulo Fulanete Trombini



--
Internal Virus Database is out-of-date.
Checked by AVG Anti-Virus.
Version: 7.0.300 / Virus Database: 266.5.0 - Release Date: 25/02/2005




--
Internal Virus Database is out-of-date.
Checked by AVG Anti-Virus.
Version: 7.0.300 / Virus Database: 266.5.0 - Release Date: 25/02/2005



##### ##### #####

Para saber mais visite
http://www.ciencialist.hpg.ig.com.br


##### ##### ##### #####


Yahoo! Grupos, um serviço oferecido por:







----------------------------------------------------------------------------
--
Links do Yahoo! Grupos

a.. Para visitar o site do seu grupo na web, acesse:
http://br.groups.yahoo.com/group/ciencialist/

b.. Para sair deste grupo, envie um e-mail para:
ciencialist-unsubscribe@yahoogrupos.com.br

c.. O uso que você faz do Yahoo! Grupos está sujeito aos Termos do
Serviço do Yahoo!.



[As partes desta mensagem que não continham texto foram removidas]



##### ##### #####

Para saber mais visite
http://www.ciencialist.hpg.ig.com.br


##### ##### ##### #####
Links do Yahoo! Grupos










--
Internal Virus Database is out-of-date.
Checked by AVG Anti-Virus.
Version: 7.0.300 / Virus Database: 266.5.0 - Release Date: 25/02/2005




--
Internal Virus Database is out-of-date.
Checked by AVG Anti-Virus.
Version: 7.0.300 / Virus Database: 266.5.0 - Release Date: 25/02/2005



##### ##### #####

Para saber mais visite
http://www.ciencialist.hpg.ig.com.br


##### ##### ##### #####


Yahoo! Grupos, um serviço oferecido por:

São Paulo Rio de Janeiro Curitiba Porto Alegre Belo Horizonte Brasília





Links do Yahoo! Grupos

Para visitar o site do seu grupo na web, acesse:
http://br.groups.yahoo.com/group/ciencialist/

Para sair deste grupo, envie um e-mail para:
ciencialist-unsubscribe@yahoogrupos.com.br

O uso que você faz do Yahoo! Grupos está sujeito aos Termos do Serviço do
Yahoo!.




SUBJECT: Fw: Pilha.
FROM: "Luiz Ferraz Netto" <leobarretos@uol.com.br>
TO: "ciencialist" <ciencialist@yahoogrupos.com.br>
DATE: 15/03/2005 16:08

Oi Roberto,

veja-me os links de detalhes construtivos das pilhas comuns de lanterna (pois são as mais usadas).
agradeço,
===========================
Luiz Ferraz Netto [Léo]
leobarretos@uol.com.br
http://www.feiradeciencias.com.br
===========================
-----Mensagem Original-----
De: "Edwards Martins" <aterrasistema@gmail.com>
Para: <leobarretos@uol.com.br>
Enviada em: terça-feira, 15 de março de 2005 11:01
Assunto: Pilha.


Prof: Luiz Ferraz Netto.

Meu nome: Edwards Martins Vieira
Residente: Goiânia-Go. Rua R-13 Qd. 12 Lt. 24 - Conjunto Itatiaia
CEP: 74690-260.
Fone/Fax: (62) 205.45.62 - Celular: (62) 8412-9950
Profissão: Inventor.
Equipamento Inventado: ATERRASISTEMA - ATERRASISTER - APARALAMA.


Eu preciso de ajuda e gostária de saber se suber e diferem exbosto sob
como e feito uma pilha.
Tenho uma mudança de elementos para complementar a utilização desta
pilha, não afetando o meio ambiente.
Me perdoe vou explicar melhor:
Quero mudar os elemento de uma pilha, para que a mesma não possa
poluir o meio ambiente, podendo a pessoa que estiver utilizando esta
pilha ter contato com os elemento dentro e não prejudicando a sua
saude.
Portanto necessito da base completa desta produção.
Creio que posso contar com a ajuda de você, aguardo uma resposta.

Edwards Martins Vieira.


--
Internal Virus Database is out-of-date.
Checked by AVG Anti-Virus.
Version: 7.0.300 / Virus Database: 266.5.0 - Release Date: 25/02/2005




--
Internal Virus Database is out-of-date.
Checked by AVG Anti-Virus.
Version: 7.0.300 / Virus Database: 266.5.0 - Release Date: 25/02/2005



SUBJECT: Re: [ciencialist] Fw: Pergunta! Uma boa pergunta
FROM: "Luiz Ferraz Netto" <leobarretos@uol.com.br>
TO: <ciencialist@yahoogrupos.com.br>
DATE: 15/03/2005 16:17

Oi Emiliano,

a pergunta do garoto (e que tb me interessou) é essa mesmo: nas coisas da tecnologia humana, onde se observa a maior aceleração (mesmo por breve intervalos de tempo).
Serão nas partículas nos aceleradores? na bala de fuzil? no supercarro que vc apontou? no motor à explosão?

[]'
===========================
Luiz Ferraz Netto [Léo]
leobarretos@uol.com.br
http://www.feiradeciencias.com.br
===========================
-----Mensagem Original-----
De: "E m i l i a n o C h e m e l l o" <chemelloe@yahoo.com.br>
Para: <ciencialist@yahoogrupos.com.br>
Enviada em: terça-feira, 15 de março de 2005 15:52
Assunto: Re: [ciencialist] Fw: Pergunta! Uma boa pergunta



ops...

Fastest Acceleration.
The fastest road-tested acceleration on record is 0-60mph in 3.07 sec's by a
Ford RS2000 Evolution at the Millbrook Proving Ground UK May 1994.
fonte: http://www.ukcar.com/guinness/
---

Considerando que 60 mph (milhas por hora) equivalem a (arredondando) 100
km/h.

Já outra fonte diz que este é o carro mais rápido do mundo
"... Acceleration: 0-100 km/h (0-62 mph) 3.2 seconds..."
fonte: http://www.autoemirates.com/MiddleEast/2005/0127KoenigseggCCRME.asp

No site do guinness eu não achei referência sobre 'aceleração'. Eu sugiro
que se faça uma especificação de que tipo de "mecânica terrestre" se quer.

[ ] 's do Emiliano Chemello
emiliano@quimica.net
http://www.quimica.net/emiliano
http://www.ucs.br/ccet/defq/naeq
[ MSN ] chemelloe@hotmail.com
[ ICQ ] 145060604

" Rien ne se perd, rien ne se crée,
tout se transforme."

Antoine Laurent de Lavoisier (químico francês, 1743 - 1794)

----- Original Message -----
From: Luiz Ferraz Netto
To: ciencialist@yahoogrupos.com.br
Sent: Tuesday, March 15, 2005 2:36 PM
Subject: Re: [ciencialist] Fw: Pergunta! Uma boa pergunta


Oi Emiliano,

o interesse recai na aceleração e não na velocidade.
Mas, gostei do carrinho --- aqui em Barretos seria um furor. Vou ver em
quantos segundos eu atravessaria a cidade.
[]'
===========================
Luiz Ferraz Netto [Léo]
leobarretos@uol.com.br
http://www.feiradeciencias.com.br
===========================
-----Mensagem Original-----
De: "E m i l i a n o C h e m e l l o" <chemelloe@yahoo.com.br>
Para: <ciencialist@yahoogrupos.com.br>
Enviada em: terça-feira, 15 de março de 2005 14:18
Assunto: Re: [ciencialist] Fw: Pergunta! Uma boa pergunta



Não fiz muita pesquisa... mas acho que uma boa referência é o site do
guinness:
http://www.guinnessworldrecords.com/gwr5/content_pages/record.asp?recordid=43540

[ ] 's do Emiliano Chemello
emiliano@quimica.net
http://www.quimica.net/emiliano
http://www.ucs.br/ccet/defq/naeq
[ MSN ] chemelloe@hotmail.com
[ ICQ ] 145060604

" Rien ne se perd, rien ne se crée,
tout se transforme."

Antoine Laurent de Lavoisier (químico francês, 1743 - 1794)

----- Original Message -----
From: Luiz Ferraz Netto
To: ciencialist
Sent: Tuesday, March 15, 2005 1:45 PM
Subject: [ciencialist] Fw: Pergunta! Uma boa pergunta


Realmente, gostei da pergunta: "Aqui na alface da Terra qual o equipamento
(aparelho, máquina, processo .... ) experimenta a maior aceleração (mesmo
que seja por pequeno intervalo de tempo)?
É aquele carrinho esquisito que atinge 500 km/h? É a bala de fuzil?

[]'
===========================
Luiz Ferraz Netto [Léo]
leobarretos@uol.com.br
http://www.feiradeciencias.com.br
===========================
-----Mensagem Original-----
De: "Saulo Fulanete Trombini" <sauloft@pop.com.br>
Para: <leobarretos@uol.com.br>
Enviada em: sábado, 12 de março de 2005 10:53
Assunto: Pergunta!


Prezado Professor,
Baseado no fato de que o senhor tem muito mais comnhecimento, e
recursos do
que eu, venho através desta, lhe perguntar: Qual a maior aceleração
mecânica
terrestre? Parabenizo-o pelo seu excelente site, pois através dele estou
melhorando meus conhecimentos, e adquirindo novas idéias que
posteriormente
levarei a Feira de Ciências da minha escola.

Agradecendo, despeço-me;

Saulo Fulanete Trombini



--
Internal Virus Database is out-of-date.
Checked by AVG Anti-Virus.
Version: 7.0.300 / Virus Database: 266.5.0 - Release Date: 25/02/2005




--
Internal Virus Database is out-of-date.
Checked by AVG Anti-Virus.
Version: 7.0.300 / Virus Database: 266.5.0 - Release Date: 25/02/2005



##### ##### #####

Para saber mais visite
http://www.ciencialist.hpg.ig.com.br


##### ##### ##### #####


Yahoo! Grupos, um serviço oferecido por:







----------------------------------------------------------------------------
--
Links do Yahoo! Grupos

a.. Para visitar o site do seu grupo na web, acesse:
http://br.groups.yahoo.com/group/ciencialist/

b.. Para sair deste grupo, envie um e-mail para:
ciencialist-unsubscribe@yahoogrupos.com.br

c.. O uso que você faz do Yahoo! Grupos está sujeito aos Termos do
Serviço do Yahoo!.



[As partes desta mensagem que não continham texto foram removidas]



##### ##### #####

Para saber mais visite
http://www.ciencialist.hpg.ig.com.br


##### ##### ##### #####
Links do Yahoo! Grupos










--
Internal Virus Database is out-of-date.
Checked by AVG Anti-Virus.
Version: 7.0.300 / Virus Database: 266.5.0 - Release Date: 25/02/2005




--
Internal Virus Database is out-of-date.
Checked by AVG Anti-Virus.
Version: 7.0.300 / Virus Database: 266.5.0 - Release Date: 25/02/2005



##### ##### #####

Para saber mais visite
http://www.ciencialist.hpg.ig.com.br


##### ##### ##### #####


Yahoo! Grupos, um serviço oferecido por:

São Paulo Rio de Janeiro Curitiba Porto Alegre Belo Horizonte Brasília





Links do Yahoo! Grupos

Para visitar o site do seu grupo na web, acesse:
http://br.groups.yahoo.com/group/ciencialist/

Para sair deste grupo, envie um e-mail para:
ciencialist-unsubscribe@yahoogrupos.com.br

O uso que você faz do Yahoo! Grupos está sujeito aos Termos do Serviço do
Yahoo!.




##### ##### #####

Para saber mais visite
http://www.ciencialist.hpg.ig.com.br


##### ##### ##### #####
Links do Yahoo! Grupos










--
Internal Virus Database is out-of-date.
Checked by AVG Anti-Virus.
Version: 7.0.300 / Virus Database: 266.5.0 - Release Date: 25/02/2005




--
Internal Virus Database is out-of-date.
Checked by AVG Anti-Virus.
Version: 7.0.300 / Virus Database: 266.5.0 - Release Date: 25/02/2005



SUBJECT: RE: [ciencialist] Fw: Pergunta! Uma boa pergunta
FROM: "Hugo Santos" <urano@netvisao.pt>
TO: <ciencialist@yahoogrupos.com.br>
DATE: 15/03/2005 16:38

De que é que vocês estão falando??? Isso não é nada!! Nunca ninguém viu
corridas de dragsters?
Isso sim é que são acelerações mecânicas decentes (cerca de 8G):

http://www.capecodcorvetteclub.com/acceleration.htm

Isso que vocês estão aí a falar são só carros de estrada modificados.

Por outro lado, os aviões mais recentes conseguem atingir acelerações
superiores a 10G em curva, não em recta, mas não deixa de ser uma
aceleração. Por isso é que os pilotos têm de usar fatos especiais para
"empurrar" o sangue para o cérebro nessas circunstâncias.

Quanto ao do record do guiness, aquilo usa duas turbinas mas a aceleração é
bastante baixa inicialmente e mesmo com o afterburner não deve ser muito
alta. A vantagem é que a aceleração é praticamente constante até serem
atingidas velocidades muito altas. É um pouco como o motor espacial iónico,
apesar de ser o mais fraquinho de todos os motores espaciais, tem a vantagem
de produzir uma aceleração constante, praticamente independente da
velocidade.

A meu ver, deverá haver certamente qualquer dispositivo mecânico que produza
acelerações bem mais altas do que qualquer das que foi referida por mim ou
pelos outros. O disparo de uma bala ou uma explosão, presumo que não sejam
consideradas acelerações mecânicas mas sim químicas ou físico-químicas,
porque senão, a aceleração de uma das placas que envolvem uma bomba atómica,
assumindo que elas não se desintegram totalmente, deverá ser
incomparavelmente superior a qualquer outra que se consiga fazer na terra.

Abraços,

Hugo Santos


> -----Original Message-----
> From: E m i l i a n o C h e m e l l o [mailto:chemelloe@yahoo.com.br]
> Sent: terça-feira, 15 de Março de 2005 18:53
> To: ciencialist@yahoogrupos.com.br
> Subject: Re: [ciencialist] Fw: Pergunta! Uma boa pergunta
>
>
> ops...
>
> Fastest Acceleration.
> The fastest road-tested acceleration on record is 0-60mph in 3.07 sec's by
a
> Ford RS2000 Evolution at the Millbrook Proving Ground UK May 1994.
> fonte: http://www.ukcar.com/guinness/
> ---
>
> Considerando que 60 mph (milhas por hora) equivalem a (arredondando) 100
> km/h.
>
> Já outra fonte diz que este é o carro mais rápido do mundo
> "... Acceleration: 0-100 km/h (0-62 mph) 3.2 seconds..."
> fonte: http://www.autoemirates.com/MiddleEast/2005/0127KoenigseggCCRME.asp
>
> No site do guinness eu não achei referência sobre 'aceleração'. Eu sugiro
> que se faça uma especificação de que tipo de "mecânica terrestre" se quer.
>
> [ ] 's do Emiliano Chemello
> emiliano@quimica.net
> http://www.quimica.net/emiliano
> http://www.ucs.br/ccet/defq/naeq
> [ MSN ] chemelloe@hotmail.com
> [ ICQ ] 145060604
>
> " Rien ne se perd, rien ne se crée,
> tout se transforme."
>
> Antoine Laurent de Lavoisier (químico francês, 1743 - 1794)
>
> ----- Original Message -----
> From: Luiz Ferraz Netto
> To: ciencialist@yahoogrupos.com.br
> Sent: Tuesday, March 15, 2005 2:36 PM
> Subject: Re: [ciencialist] Fw: Pergunta! Uma boa pergunta
>
>
> Oi Emiliano,
>
> o interesse recai na aceleração e não na velocidade.
> Mas, gostei do carrinho --- aqui em Barretos seria um furor. Vou ver em
> quantos segundos eu atravessaria a cidade.
> []'
> ===========================
> Luiz Ferraz Netto [Léo]
> leobarretos@uol.com.br
> http://www.feiradeciencias.com.br
> ===========================
> -----Mensagem Original-----
> De: "E m i l i a n o C h e m e l l o" <chemelloe@yahoo.com.br>
> Para: <ciencialist@yahoogrupos.com.br>
> Enviada em: terça-feira, 15 de março de 2005 14:18
> Assunto: Re: [ciencialist] Fw: Pergunta! Uma boa pergunta
>
>
>
> Não fiz muita pesquisa... mas acho que uma boa referência é o site do
> guinness:
>
http://www.guinnessworldrecords.com/gwr5/content_pages/record.asp?recordid=4
3
> 540
>
> [ ] 's do Emiliano Chemello
> emiliano@quimica.net
> http://www.quimica.net/emiliano
> http://www.ucs.br/ccet/defq/naeq
> [ MSN ] chemelloe@hotmail.com
> [ ICQ ] 145060604
>
> " Rien ne se perd, rien ne se crée,
> tout se transforme."
>
> Antoine Laurent de Lavoisier (químico francês, 1743 - 1794)
>
> ----- Original Message -----
> From: Luiz Ferraz Netto
> To: ciencialist
> Sent: Tuesday, March 15, 2005 1:45 PM
> Subject: [ciencialist] Fw: Pergunta! Uma boa pergunta
>
>
> Realmente, gostei da pergunta: "Aqui na alface da Terra qual o
equipamento
> (aparelho, máquina, processo .... ) experimenta a maior aceleração (mesmo
> que seja por pequeno intervalo de tempo)?
> É aquele carrinho esquisito que atinge 500 km/h? É a bala de fuzil?
>
> []'
> ===========================
> Luiz Ferraz Netto [Léo]
> leobarretos@uol.com.br
> http://www.feiradeciencias.com.br
> ===========================
> -----Mensagem Original-----
> De: "Saulo Fulanete Trombini" <sauloft@pop.com.br>
> Para: <leobarretos@uol.com.br>
> Enviada em: sábado, 12 de março de 2005 10:53
> Assunto: Pergunta!
>
>
> Prezado Professor,
> Baseado no fato de que o senhor tem muito mais comnhecimento, e
> recursos do
> que eu, venho através desta, lhe perguntar: Qual a maior aceleração
> mecânica
> terrestre? Parabenizo-o pelo seu excelente site, pois através dele estou
> melhorando meus conhecimentos, e adquirindo novas idéias que
> posteriormente
> levarei a Feira de Ciências da minha escola.
>
> Agradecendo, despeço-me;
>
> Saulo Fulanete Trombini
>
>
>
> --
> Internal Virus Database is out-of-date.
> Checked by AVG Anti-Virus.
> Version: 7.0.300 / Virus Database: 266.5.0 - Release Date: 25/02/2005
>
>
>
>
> --
> Internal Virus Database is out-of-date.
> Checked by AVG Anti-Virus.
> Version: 7.0.300 / Virus Database: 266.5.0 - Release Date: 25/02/2005
>
>
>
> ##### ##### #####
>
> Para saber mais visite
> http://www.ciencialist.hpg.ig.com.br
>
>
> ##### ##### ##### #####
>
>
> Yahoo! Grupos, um serviço oferecido por:
>
>
>
>
>
>
>
>
----------------------------------------------------------------------------
> --
> Links do Yahoo! Grupos
>
> a.. Para visitar o site do seu grupo na web, acesse:
> http://br.groups.yahoo.com/group/ciencialist/
>
> b.. Para sair deste grupo, envie um e-mail para:
> ciencialist-unsubscribe@yahoogrupos.com.br
>
> c.. O uso que você faz do Yahoo! Grupos está sujeito aos Termos do
> Serviço do Yahoo!.
>
>
>
> [As partes desta mensagem que não continham texto foram removidas]
>
>
>
> ##### ##### #####
>
> Para saber mais visite
> http://www.ciencialist.hpg.ig.com.br
>
>
> ##### ##### ##### #####
> Links do Yahoo! Grupos
>
>
>
>
>
>
>
>
>
>
> --
> Internal Virus Database is out-of-date.
> Checked by AVG Anti-Virus.
> Version: 7.0.300 / Virus Database: 266.5.0 - Release Date: 25/02/2005
>
>
>
>
> --
> Internal Virus Database is out-of-date.
> Checked by AVG Anti-Virus.
> Version: 7.0.300 / Virus Database: 266.5.0 - Release Date: 25/02/2005
>
>
>
> ##### ##### #####
>
> Para saber mais visite
> http://www.ciencialist.hpg.ig.com.br
>
>
> ##### ##### ##### #####
>
>
> Yahoo! Grupos, um serviço oferecido por:
>
> São Paulo Rio de Janeiro Curitiba Porto Alegre Belo Horizonte Brasília
>
>
>
>
>
> Links do Yahoo! Grupos
>
> Para visitar o site do seu grupo na web, acesse:
> http://br.groups.yahoo.com/group/ciencialist/
>
> Para sair deste grupo, envie um e-mail para:
> ciencialist-unsubscribe@yahoogrupos.com.br
>
> O uso que você faz do Yahoo! Grupos está sujeito aos Termos do Serviço do
> Yahoo!.
>
>
>
>
> ##### ##### #####
>
> Para saber mais visite
> http://www.ciencialist.hpg.ig.com.br
>
>
> ##### ##### ##### #####
> Links do Yahoo! Grupos
>
>
>
>
>
>
>




SUBJECT: Re: [ciencialist] Fw: Pergunta! Uma boa pergunta
FROM: "E m i l i a n o C h e m e l l o" <chemelloe@yahoo.com.br>
TO: <ciencialist@yahoogrupos.com.br>
DATE: 15/03/2005 16:59

Olá Hugo,

[Hugo]
"... O disparo de uma bala ou uma explosão, presumo que não sejam
consideradas acelerações mecânicas mas sim químicas ou físico-químicas,
porque senão, a aceleração de uma das placas que envolvem uma bomba atómica,
assumindo que elas não se desintegram totalmente, deverá ser
incomparavelmente superior a qualquer outra que se consiga fazer na terra."

[Emiliano]
Então, seguindo este critério, um carro, um caminhão ou motocicleta com
motor a explosão, em movimento, não podem ser considerado como exemplo de
'aceleração mecânica', pois o que promove o movimento do veículo é o
combustível, o qual sofre uma reação química (combustão).

O fato da pergunta ser genérica demais dificulta darmos uma resposta direta.

[ ] 's do Emiliano Chemello
emiliano@quimica.net
http://www.quimica.net/emiliano
http://www.ucs.br/ccet/defq/naeq
[ MSN ] chemelloe@hotmail.com
[ ICQ ] 145060604

" Rien ne se perd, rien ne se crée,
tout se transforme."

Antoine Laurent de Lavoisier (químico francês, 1743 - 1794)

----- Original Message -----
From: Hugo Santos
To: ciencialist@yahoogrupos.com.br
Sent: Tuesday, March 15, 2005 4:38 PM
Subject: RE: [ciencialist] Fw: Pergunta! Uma boa pergunta


De que é que vocês estão falando??? Isso não é nada!! Nunca ninguém viu
corridas de dragsters?
Isso sim é que são acelerações mecânicas decentes (cerca de 8G):

http://www.capecodcorvetteclub.com/acceleration.htm

Isso que vocês estão aí a falar são só carros de estrada modificados.

Por outro lado, os aviões mais recentes conseguem atingir acelerações
superiores a 10G em curva, não em recta, mas não deixa de ser uma
aceleração. Por isso é que os pilotos têm de usar fatos especiais para
"empurrar" o sangue para o cérebro nessas circunstâncias.

Quanto ao do record do guiness, aquilo usa duas turbinas mas a aceleração é
bastante baixa inicialmente e mesmo com o afterburner não deve ser muito
alta. A vantagem é que a aceleração é praticamente constante até serem
atingidas velocidades muito altas. É um pouco como o motor espacial iónico,
apesar de ser o mais fraquinho de todos os motores espaciais, tem a vantagem
de produzir uma aceleração constante, praticamente independente da
velocidade.

A meu ver, deverá haver certamente qualquer dispositivo mecânico que produza
acelerações bem mais altas do que qualquer das que foi referida por mim ou
pelos outros. O disparo de uma bala ou uma explosão, presumo que não sejam
consideradas acelerações mecânicas mas sim químicas ou físico-químicas,
porque senão, a aceleração de uma das placas que envolvem uma bomba atómica,
assumindo que elas não se desintegram totalmente, deverá ser
incomparavelmente superior a qualquer outra que se consiga fazer na terra.

Abraços,

Hugo Santos


> -----Original Message-----
> From: E m i l i a n o C h e m e l l o [mailto:chemelloe@yahoo.com.br]
> Sent: terça-feira, 15 de Março de 2005 18:53
> To: ciencialist@yahoogrupos.com.br
> Subject: Re: [ciencialist] Fw: Pergunta! Uma boa pergunta
>
>
> ops...
>
> Fastest Acceleration.
> The fastest road-tested acceleration on record is 0-60mph in 3.07 sec's by
a
> Ford RS2000 Evolution at the Millbrook Proving Ground UK May 1994.
> fonte: http://www.ukcar.com/guinness/
> ---
>
> Considerando que 60 mph (milhas por hora) equivalem a (arredondando) 100
> km/h.
>
> Já outra fonte diz que este é o carro mais rápido do mundo
> "... Acceleration: 0-100 km/h (0-62 mph) 3.2 seconds..."
> fonte: http://www.autoemirates.com/MiddleEast/2005/0127KoenigseggCCRME.asp
>
> No site do guinness eu não achei referência sobre 'aceleração'. Eu sugiro
> que se faça uma especificação de que tipo de "mecânica terrestre" se quer.
>
> [ ] 's do Emiliano Chemello
> emiliano@quimica.net
> http://www.quimica.net/emiliano
> http://www.ucs.br/ccet/defq/naeq
> [ MSN ] chemelloe@hotmail.com
> [ ICQ ] 145060604
>
> " Rien ne se perd, rien ne se crée,
> tout se transforme."
>
> Antoine Laurent de Lavoisier (químico francês, 1743 - 1794)
>
> ----- Original Message -----
> From: Luiz Ferraz Netto
> To: ciencialist@yahoogrupos.com.br
> Sent: Tuesday, March 15, 2005 2:36 PM
> Subject: Re: [ciencialist] Fw: Pergunta! Uma boa pergunta
>
>
> Oi Emiliano,
>
> o interesse recai na aceleração e não na velocidade.
> Mas, gostei do carrinho --- aqui em Barretos seria um furor. Vou ver em
> quantos segundos eu atravessaria a cidade.
> []'
> ===========================
> Luiz Ferraz Netto [Léo]
> leobarretos@uol.com.br
> http://www.feiradeciencias.com.br
> ===========================
> -----Mensagem Original-----
> De: "E m i l i a n o C h e m e l l o" <chemelloe@yahoo.com.br>
> Para: <ciencialist@yahoogrupos.com.br>
> Enviada em: terça-feira, 15 de março de 2005 14:18
> Assunto: Re: [ciencialist] Fw: Pergunta! Uma boa pergunta
>
>
>
> Não fiz muita pesquisa... mas acho que uma boa referência é o site do
> guinness:
>
http://www.guinnessworldrecords.com/gwr5/content_pages/record.asp?recordid=4
3
> 540
>
> [ ] 's do Emiliano Chemello
> emiliano@quimica.net
> http://www.quimica.net/emiliano
> http://www.ucs.br/ccet/defq/naeq
> [ MSN ] chemelloe@hotmail.com
> [ ICQ ] 145060604
>
> " Rien ne se perd, rien ne se crée,
> tout se transforme."
>
> Antoine Laurent de Lavoisier (químico francês, 1743 - 1794)
>
> ----- Original Message -----
> From: Luiz Ferraz Netto
> To: ciencialist
> Sent: Tuesday, March 15, 2005 1:45 PM
> Subject: [ciencialist] Fw: Pergunta! Uma boa pergunta
>
>
> Realmente, gostei da pergunta: "Aqui na alface da Terra qual o
equipamento
> (aparelho, máquina, processo .... ) experimenta a maior aceleração (mesmo
> que seja por pequeno intervalo de tempo)?
> É aquele carrinho esquisito que atinge 500 km/h? É a bala de fuzil?
>
> []'
> ===========================
> Luiz Ferraz Netto [Léo]
> leobarretos@uol.com.br
> http://www.feiradeciencias.com.br
> ===========================
> -----Mensagem Original-----
> De: "Saulo Fulanete Trombini" <sauloft@pop.com.br>
> Para: <leobarretos@uol.com.br>
> Enviada em: sábado, 12 de março de 2005 10:53
> Assunto: Pergunta!
>
>
> Prezado Professor,
> Baseado no fato de que o senhor tem muito mais comnhecimento, e
> recursos do
> que eu, venho através desta, lhe perguntar: Qual a maior aceleração
> mecânica
> terrestre? Parabenizo-o pelo seu excelente site, pois através dele estou
> melhorando meus conhecimentos, e adquirindo novas idéias que
> posteriormente
> levarei a Feira de Ciências da minha escola.
>
> Agradecendo, despeço-me;
>
> Saulo Fulanete Trombini
>
>
>
> --
> Internal Virus Database is out-of-date.
> Checked by AVG Anti-Virus.
> Version: 7.0.300 / Virus Database: 266.5.0 - Release Date: 25/02/2005
>
>
>
>
> --
> Internal Virus Database is out-of-date.
> Checked by AVG Anti-Virus.
> Version: 7.0.300 / Virus Database: 266.5.0 - Release Date: 25/02/2005
>
>
>
> ##### ##### #####
>
> Para saber mais visite
> http://www.ciencialist.hpg.ig.com.br
>
>
> ##### ##### ##### #####
>
>
> Yahoo! Grupos, um serviço oferecido por:
>
>
>
>
>
>
>
>
----------------------------------------------------------------------------
> --
> Links do Yahoo! Grupos
>
> a.. Para visitar o site do seu grupo na web, acesse:
> http://br.groups.yahoo.com/group/ciencialist/
>
> b.. Para sair deste grupo, envie um e-mail para:
> ciencialist-unsubscribe@yahoogrupos.com.br
>
> c.. O uso que você faz do Yahoo! Grupos está sujeito aos Termos do
> Serviço do Yahoo!.
>
>
>
> [As partes desta mensagem que não continham texto foram removidas]
>
>
>
> ##### ##### #####
>
> Para saber mais visite
> http://www.ciencialist.hpg.ig.com.br
>
>
> ##### ##### ##### #####
> Links do Yahoo! Grupos
>
>
>
>
>
>
>
>
>
>
> --
> Internal Virus Database is out-of-date.
> Checked by AVG Anti-Virus.
> Version: 7.0.300 / Virus Database: 266.5.0 - Release Date: 25/02/2005
>
>
>
>
> --
> Internal Virus Database is out-of-date.
> Checked by AVG Anti-Virus.
> Version: 7.0.300 / Virus Database: 266.5.0 - Release Date: 25/02/2005
>
>
>
> ##### ##### #####
>
> Para saber mais visite
> http://www.ciencialist.hpg.ig.com.br
>
>
> ##### ##### ##### #####
>
>
> Yahoo! Grupos, um serviço oferecido por:
>
> São Paulo Rio de Janeiro Curitiba Porto Alegre Belo Horizonte Brasília
>
>
>
>
>
> Links do Yahoo! Grupos
>
> Para visitar o site do seu grupo na web, acesse:
> http://br.groups.yahoo.com/group/ciencialist/
>
> Para sair deste grupo, envie um e-mail para:
> ciencialist-unsubscribe@yahoogrupos.com.br
>
> O uso que você faz do Yahoo! Grupos está sujeito aos Termos do Serviço do
> Yahoo!.
>
>
>
>
> ##### ##### #####
>
> Para saber mais visite
> http://www.ciencialist.hpg.ig.com.br
>
>
> ##### ##### ##### #####
> Links do Yahoo! Grupos
>
>
>
>
>
>
>




##### ##### #####

Para saber mais visite
http://www.ciencialist.hpg.ig.com.br


##### ##### ##### #####


Yahoo! Grupos, um serviço oferecido por:








Links do Yahoo! Grupos

Para visitar o site do seu grupo na web, acesse:
http://br.groups.yahoo.com/group/ciencialist/

Para sair deste grupo, envie um e-mail para:
ciencialist-unsubscribe@yahoogrupos.com.br

O uso que você faz do Yahoo! Grupos está sujeito aos Termos do Serviço do
Yahoo!.




SUBJECT: Marie Curie
FROM: "E m i l i a n o C h e m e l l o" <chemelloe@yahoo.com.br>
TO: <ciencialist@yahoogrupos.com.br>, <quimica-qaw@yahoogrupos.com.br>, <naeq-ucs@yahoogrupos.com.br>, <quimica@grupos.com.br>
DATE: 15/03/2005 17:10

O artigo que a moça faz referência é este:

Marie Curie e a radioatividade
http://www.ucs.br/ccet/defq/naeq/material_didatico/e-museu_quimica_01.htm
---
Contato Naeq:
Nome: Adriana
Email: adriana_guimaraes01@yahoo.com.br
Assunto: Marie Curie
Mensagem: A algum tempo li a biografia de Marie Curie inicialmente movida
meramente pela coincidencia de termos nascido no mesmo dia, mas a medida q
ia lendo sua hist ria crescia em mim uma grande admiracao, nessa poca
despertou em mim uma paixao por radioatividade, mas nao s isso despertou em
mim uma ansia pelo saber, pela descoberta.Sou estudante de
odontologia(pretendo prestar vestibular para radiologia) e acima de tudo uma
idealista.Se todos estudantes pudessem ter a oportunidades de conhecer a
historia de pessoas tao persistentes, que com forca de vontade mudaram a
humanidade por meio da ciencia muita gente acordaria para o saber.Obrigada
pelo artigo,pois cada vez que vejo citado o nome dessa grande mulher me
emociono e me alegro!!!
---

[ ] 's do Emiliano Chemello
emiliano@quimica.net
http://www.quimica.net/emiliano
http://www.ucs.br/ccet/defq/naeq
[ MSN ] chemelloe@hotmail.com
[ ICQ ] 145060604

" Rien ne se perd, rien ne se crée,
tout se transforme."

Antoine Laurent de Lavoisier (químico francês, 1743 - 1794)




SUBJECT: Re: [ciencialist] Fw: Quero uma ajuda
FROM: "Oraculo" <oraculo@atibaia.com.br>
TO: <ciencialist@yahoogrupos.com.br>
DATE: 15/03/2005 17:35

Olá Professor

Se conseguir interpretar, nos conte o que descobriu..:-) Um maior esforço em aprender portugues para melhorar a comunicação, seria um bom conselho ao consulente..:-) "Conzinha"??? Me fez uma pergunta para que eu desse uma resposta??? Duas "substâncias" em contato para gerar força motrix???

Vai ter de interpretar um bocado para ajudar o rapaz..:-)

Um abraço.

Homero

----- Original Message -----
From: Luiz Ferraz Netto
To: ciencialist
Sent: Tuesday, March 15, 2005 2:13 PM
Subject: [ciencialist] Fw: Quero uma ajuda


Primeiro, preciso interpretar ....... depois .......

[]'
===========================
Luiz Ferraz Netto [Léo]
leobarretos@uol.com.br
http://www.feiradeciencias.com.br
===========================
-----Mensagem Original-----
De: Morquimica RJ
Para: leobarretos@uol.com.br
Enviada em: sexta-feira, 11 de março de 2005 18:27
Assunto: Quero uma ajuda



Nas dúvidas experimentais, por gentileza coloque aqui o endereço da página, isso facilita o confronto. Agradeço. Meu nome é LUIZ FERRAZ NETTO, meu apelido é LÉO e moro em BARRETOS; dai vem meu e-mail: leobarretos@uol.com.br.

Oi Sr Luiz, Boa Noite,

Eu estou fazendo um trabalho na minha escola e um profº, me fez uma pergunta para que eu desse uma resposta a ele sobre a Força Motriz a pergunta é a seguinte:


Porque o arroz conzinha com a força motriz e não entendi,

pois até que eu saiba a força motriz seria duas substâncias em contato uma com a outra existindo uma interação formando uma ligação, seria isso força motriz.

Por favor tenta me ajudar

agradeço

Luiz Carlos de Araujo.
email - araujo_454@hotmail.com


--------------------------------------------------------------------------------


Internal Virus Database is out-of-date.
Checked by AVG Anti-Virus.
Version: 7.0.300 / Virus Database: 266.5.0 - Release Date: 25/02/2005

----------

Internal Virus Database is out-of-date.
Checked by AVG Anti-Virus.
Version: 7.0.300 / Virus Database: 266.5.0 - Release Date: 25/02/2005


[As partes desta mensagem que não continham texto foram removidas]



##### ##### #####

Para saber mais visite
http://www.ciencialist.hpg.ig.com.br


##### ##### ##### #####


Yahoo! Grupos, um serviço oferecido por:







------------------------------------------------------------------------------
Links do Yahoo! Grupos

a.. Para visitar o site do seu grupo na web, acesse:
http://br.groups.yahoo.com/group/ciencialist/

b.. Para sair deste grupo, envie um e-mail para:
ciencialist-unsubscribe@yahoogrupos.com.br

c.. O uso que você faz do Yahoo! Grupos está sujeito aos Termos do Serviço do Yahoo!.



[As partes desta mensagem que não continham texto foram removidas]



SUBJECT: Re: [ciencialist] 'Theory of everything' tying researchers up in knots
FROM: "Sergio M. M. Taborda" <sergiotaborda@terra.com.br>
TO: ciencialist@yahoogrupos.com.br
DATE: 15/03/2005 19:50

marcelomjr wrote:

>
> http://sfgate.com/cgi-bin/article.cgi?
> f=/c/a/2005/03/14/MNGRMBOURE1.DTL
>
> 'Theory of everything' tying researchers up in knots
> Keay Davidson, Chronicle Science Writer
>
> Monday, March 14, 2005
>
já todos aqui deveriam saber o que está escrito no texto. Que a teoria
das superstrings não passa de uma teoria, inacabada, que nunca mais
acaba, e que acaba por não significar nada para a ciencia e a fisica em
particular. Que é apenas uma forma de muitos terem empregos.
Fico contente por finalmente alguem por o dedo na ferida. Esta
"quasi-teologia" - como chamam no texto , e acho muito apropriado - já
chateia e tem que ser extinta.

Sérgio Taborda


SUBJECT: Re: [ciencialist] Re: Foucault / Luz ondas ou corpusculos.
FROM: "Alberto Mesquita Filho" <albmesq@uol.com.br>
TO: <ciencialist@yahoogrupos.com.br>
DATE: 15/03/2005 20:05

----- Original Message -----
From: "JVictor"
Sent: Wednesday, March 16, 2005 6:35 PM
Subject: Re: [ciencialist] Re: Foucault / Luz ondas ou corpusculos.

> Já Huygnes descobriu entre um montão de coisas, a polarização da luz,...

Há quem atribua essa descoberta a Erasmus Bartholine em 1670. Sabe-se, não
obstante, que essa descoberta preocupou tremendamente Huyghens que quase
chegou a visualizar o desabamento de todos os seus argumentos ondulatórios.

> ...que também não foi capaz de associar à sua transversalidade, pois a
> polarização exige que a propagação seja transversal.

A polarização constituiu-se num impecilho terrível para as idéias de
Huyghens, o que foi reconhecido pelo próprio autor. Com efeito, isso sugeria
uma dualidade do éter e nessa época não ficava bem para um cientista de
respeito ficar por aí inventando hipóteses "ad hoc" adoidado, como acontece
nos dias atuais. A esse respeito existe uma frase célebre de Newton, no
Escólio Geral dos Principia: "Não INVENTO hipóteses" e que durante muito
tempo foi mal traduzida por "não faço hipóteses". Grande parte dos físicos
modernos que conheço ainda conserva essa má tradução e pregam, atribuindo a
paternidade da idéia a Newton, o não fazer hipóteses. De fato eles não fazem
hipóteses, simplesmente as inventam adoidado, como me referi acima.

> Newton empurrou para debaixo do tapete tudo isso!

Protesto veementemente. Newton não só aceitou a polarização como deu mostra
que somente através de uma teoria corpuscular a polarização poderia ser
explicada sem o recurso a hipóteses "ad hoc" e a despersonalizarem o
conceito do que seria uma onda mecânica. Se você estiver com a Óptica de
Newton em mãos sugiro que não procure pela palavra polarização, que não era
muito utilizada na época. Procure por "refração do cristal-da-islândia",
pois foi a dupla refração desse cristal o fenômeno descoberto por Erasmus
Bartholine. O termo dupla refração também parece que ainda não era
utilizado, e Newton refere-se a refração normal e refração extraordinária.
Na Óptica III, questão 25, você poderá encontrar esse fenômeno descrito de
forma magistral por Newton, e nas questões seguintes Newton conclui que em
virtude do que hoje chamamos polarização, os raios de luz teriam "lados
dotados de várias propriedades originais". Isso significa que seus
corpúsculos não seriam bolinhas simples e/ou idênticas sob quaisquer ângulos
em que fossem observadas. São páginas e páginas destinadas à discussão do
processo, páginas essas que os físicos "modernos" empurraram para debaixo do
tapete. Expus uma versão moderna desse assunto, conservando a metodologia
clássica, em http://ecientificocultural.com/ECC2/artigos/polar00.htm

Obviamente o que você fala faz sentido, pois certamente deve ter ouvido isso
em alguma das melhores universidades da atualidade. Os físicos da atualidade
sim, colocaram debaixo do tapete não apenas a Óptica de Newton, mas tudo o
que foi escrito por Newton e a não condizer com essa absurda física que está
por aí. Não só colocaram debaixo do tapete como também, e de maneira sob
certos aspectos criminosa (ainda que, via de regra, por ignorância --> crime
doloso) chegam até mesmo a atribuir a Newton afirmações que ele, de próprio
punho, chegou a negar. Já ouvi ou li isso centenas de vezes de cientistas
"eméritos", e essas frases são repetidas em uníssono por milhares de jovens
que não freqüentam a Ciencialist, pois se repetirem aqui eu não deixarei
barato. Um exemplo típico (mas não é o único) é o dogma de Côtes (um dos
prefaciadores dos Principia), conforme amplamente comentado por Maxwell e
que ainda hoje é assumido como sendo devido a Newton. A impressão que me
fica é que muitos dentre os físicos "modernos" que andam por aí nunca leram
Newton, nunca leram Maxwell e às vezes fica-me também a impressão de que
nunca leram Einstein nem Bohr, mas apenas algum catecismo escrito por alguns
de seus seguidores mais fanáticos e que, como tais, não devem ter entendido
suas obras (pois ciência não combina com dogmas e/ou fanatismos, e tanto
Einsten quanto Bohr demonstraram estar cientes disso).

Em tempo: Ciência não combina com dogma e/ou fanatismo, mas combina com bom
humor. Quando digo Einstei é seu ídolo, ou que Newton é o meu ídolo, estou
evoluindo nessa direção, pois acho que idolatria também não combina com
ciência.

[ ]´s
Alberto
http://ecientificocultural.com/indice.htm
Mas indiferentemente a tudo isso, o neutrino tem massa, o elétron não é
uma carga elétrica coulombiana e a Terra se move. E a história se repetirá.



SUBJECT: Re: 'Theory of everything' tying researchers up in knots
FROM: Manuel Bulcão <manuelbulcao@uol.com.br>
TO: ciencialist@yahoogrupos.com.br
DATE: 15/03/2005 20:40


--- Em ciencialist@yahoogrupos.com.br, "Sergio M. M. Taborda"
<sergiotaborda@t...> escreveu
> já todos aqui deveriam saber o que está escrito no texto. Que a
teoria das superstrings não passa de uma teoria, inacabada, que
nunca mais acaba, e que acaba por não significar nada para a ciencia
e a fisica em particular. Que é apenas uma forma de muitos terem
empregos. Fico contente por finalmente alguem por o dedo na ferida.
Esta "quasi-teologia" - como chamam no texto , e acho muito
apropriado - já chateia e tem que ser extinta.

Manuel: Pois eu concordo com a assertiva de Popper segundo a qual "é
preferível uma explicação metafísica a nenhuma explicação".

De resto, não há conhecimento "acabado".

Abraços,
Manuel Bulcão





SUBJECT: Re: [ciencialist] Re: Foucault / Luz ondas ou corpusculos.
FROM: JVictor <jvoneto@uol.com.br>
TO: ciencialist@yahoogrupos.com.br
DATE: 16/03/2005 00:38

Hélio Ricardo Carvalho escreveu:

>
> Victor,
>
> Responder uma MSG grande como esta é um pouco difícil mas vamos lá.
>
> Algumas partes em que eu concordo com a resposta do Alberto (msg:
> 44868), vou pular.
>
> Victor:
> Victor escreveu:>Com o advento das lentes, telescópios, microscópios,...
>
> Hélio1 : Tudo isto é refração.
>
> Victor escreveu: >...espectográfos, ...
>
> Hélio2: Isto é obtido com prismas ou rede de difração. Se for prisma
> no fundo
> é o mesmo mecanismo (absorção, tempo, reemissão), muda apenas o número
> de absorções (e talvez o tempo) para cada freqüência.
> No caso de rede de difração fica mais claro que devemos sempre admitir
> que as emissões são em todas as direções (ou aleotoriamente em qq
> direção o que dá em média temporal "todas as direções". Mas isto é do
> debate com o Takata)
>
> Victor escreveu:>...novos efeitos foram sendo observados, que a teoria
> >corpuscular não conseguia explicar.
>
> Hélio3: Novamente, "aquela teoria corpuscular"

Victor: Ref.: Hélio1,2,3: Até fins do século XVI, era precário o nível
de conhecimento a respeito da luz.
A partir dessa época, algumas de suas propriedades fundamentais vieram à
tona e algumas descobertas fundamentais foram feitas
com o advento dos instrumentos que citei. Foi nesse sentido, a colocação.

> Victor escreveu:>Os experimentos de Arago, Young, Foucault não deixavam
> >dúvidas de que a luz era um fenômeno que apresentava
> >comportamento ondulatório, já com justificativa matematica.
>
> Hélio: Você, hoje fala isto com um certo cuidado.
> Eles, provavelmente diziam que estas experiências confirmavam que a
> luz É ondas e não simplesmente tem "comportamento ondulatório".

Victor: É verdade. Realmente é assim que penso. E qual a razão?
Esta: hoje, temos duas teorias
uma ondulatória, que explica certos fenênomenos que a corpuscular não
consegue; outra, corupuscular, que explica
outros que a ondulatória também não consegue. As duas simplesmente se
complementam. Eu n ão vejo dificuldades ou ficção nisso.
Pode não ser bom, mas é o que temos. Em resumo, a natureza da luz ainda
não foi desvendada em suas verdades essenciais.
Mas o que se tem para descrever seu comportamento, ante as diversas
interações é o que disse acima. Na época, nenhum dos três
moços tinha sequer noção, por mínima que fôsse, desse comportamento
dualístico, mas o é de maneira on/off. Ou seja, de cada vez,
a luz ou se comporta de um jeito ou de outro.
Por mim, entendo que a teoria do comportamento da luz é uma só; se
comporta como ondas quando está sendo transmitida;
e o faz como corpúsculo quando é absorvida. Isto é o que mostram as
experiências, os ensaios em laboratórios. Pode haver uma
outra interpretação, mais ao gosto de todos? Pode.
Só que ninguém tem nada, ainda. E desconfio que não terá.
E veja você que desde que o homem é homem, que essa busca pela natureza
da luz vem acontecendo, até chegar aos dias de hoje. E os conceitos de
partículas e ondas, associada à luz tem se alternado. Essas duvidas e
questionamentos ancestrais estão presentes aquí, e agora.

> Hélio: No modelo ondulatório em vigor na época a luz

> > também deveria ser mais rápida no meio mais denso.
> >
> Victor escreveu:: Honestamente, não encontrei, ainda, nenhuma
> > referência à informação acima. de qualquer maneira,
> > se alguém chegou a essa conclusão, interpretou alguma
> > coisa de maneira errada.
>
> Hélio: Errada entre aspas, né. Hoje interpretamos várias coisas de maneira
> errada mas achamos que é certa. Eles teriam seus motivos para
> interpretar assim pois o som é assim e se a luz se propaga como o som
> deveria ser também.

Victor: O modêlo ondulatório conduz diretamente a que a velocidadade da
luz num meio mais refringente é menor que essa
velocidade em um meio menos refrigente. Tanto experimental como
teoricamente! O contrário para um modelo corpuscular.
Honestamente, não sei como chegar a resultados diferentes.

>Hélio: Então vai minha OUTRA pergunta:

> > Por que a experiência de Foucault não descartou de vez a luz
> > ondulatória também????
>
> Hélio:A sua resposta abaixo me parece que não corresponde muito à esta
> pergunta acima.
> No início eu achei que você estava respondendo uma pergunta do tipo.
> "Por que a experiência de Foucault não descartou de vez a luz
> CORPUSCULAR de Newton?" que eu não fiz.
>
> > Victor: Não descartou por uma razão que julgo muito forte. A
> > autoridade de Newton era muito grande não só entre pessoas
> > comuns como entre as grandes mentes como Poisson, Biot,
> > Malus(descobridor da polarização por reflexão), Brewster (aquele,
> > do ângulo crtítico, que foi também biógrafo de Newton), só para
> > citar alguns de peso. Com uma caracterísitica. Os dois primeiros
> > eram defensores radicais da teoria corpuscular, anti-éticos até,
> > pois se valiam de suas autoridades para aterrorizar todos quanto
> > não falassem em sua cartilha newtoniana. Há um fato registrado
> > que diz bem do comportamento daquelas pessoas. ...
>
Victor: Realmente, falha minha. Você tem razão. A resposta acima
refere-se à corpuscular! É que eu não relí, após escrever, como é um
desgraçado costume que que eu tenho.
Quanto à descartar a ondulatória, qual a razão para fazerem isso, se
tais eram as conclusões a que seus experimentos conduziam?

> Hélio: A estória da luz no centro da sombra teve algum "quem conta um
> conto
> aumenta um ponto". Não deve ter acontecido exatamente assim mas é esta
> que eu tinha como A experiência que descartou a teoria corpuscular da
> época e não a de Foucault que, para este fim é mais dúbia.

Victor:
Veja que Poisson rendeu-se às evidências. Mas não foi apenas Biot que
não aceitou.
Mas trata-se de história e a gente tem que se ater ela. Supõe-se que os
historiadores são íntegros e competentes e que tiveram
acesso às informações. Ideal seria termos acesso aos originais, por
exemplo, à ata da reunião que relata o que contei. Na falta disso, é
buscar aqueles
autores cujas "histórias" coincidam o mais das vezes com as de outros.
Bem, não acho que os resultados de Foulcault sejam dúbios. Mas é o que e
acho, em função do que sei. Como fiquei curioso, e se não for muito
trabalhoso, gostaria de ler seus comentários a esse respeito. Talvez
esclareça eventuais pontos obscuros nos que já coletei.

> No final da estória você diz:
> Victor escreveu: > Ante esse juiz implacável e frio, que é um
> resultado experimental,
> > Poisson inteligentemente reconheceu e sucumbiu às teses de Fresnel.
> > Mas Biot, mesmo vendo o resultado experimental, confirmando as
> > teses de Fresnel, ainda assim, não aceitou. Continou engangado nas
> > teoria de Newton. Paixão assim, só sexual!
> > Nem ante fatos o cara retrocedeu. Putz.
>
> Hélio: Eu tenho que comentar esta parte:
> Não sei se Biot agiu assim por "paixão sexual" a Newton.

Victor: É evidente que essa "estória" (... sexual) é exagero desopilador
de minha parte. Mas o exagêro é só nisso. No restante, chequei em outra
fonte. Se você tiver outra versão, gostaria de saber.

Hélio: Voltando ao trecho em que você diz:

> >Os experimentos de Arago, Young, Foucault não deixavam
> >dúvidas de que a luz era um fenômeno que apresentava
> >comportamento ondulatório.
>
> A rigor, eles não poderiam dizer: "Estes experimentos dão a certeza
> que a luz É onda". Eles pode dizer: "... tem comportamento ondulatório".
> (este rigorismo é só ilustrativo do que eu quero te transmitir, não
> precisamos ser sempre assim, as vezes isto fica subentendido)
>
> No trecho abaixo já fiz a correção indicada por você na outra MSG:
> > ...*Acho* que eu, se estivesse na época e fosse um cientista
> > com C maiúsclo, acho que descartaria.
>
> Depois de tudo isto eu acho que você esta falando que descartaria o
> modelo corpuscular de Newton. Certo?

Victor: Certo. Hoje sabermos que as duas concepções coexistem de
maneira a que uma complementa a outra, na descrição do comportamento da luz.
Naquela época não. Além do mais, mesmo hoje, particularmante acho os
experimentos a que nos referimos nesse bate-e-mail são tão claros, tão
óbvios que, acho, mesmo naquela época eu não teria dúvidas a respeito.
Mas veja bem, estou dizendo o que poderia fazer a naquela época, mas
evidentemente não o faria hoje! Pois os dados são outros, as evidências
são outras.


Sds,

Victor.

>
>
>
>
>
>
>
>
>
> ##### ##### #####
>
>
>
> ------------------------------------------------------------------------
> *Links do Yahoo! Grupos*
>
> * Para visitar o site do seu grupo na web, acesse:
> http://br.groups.yahoo.com/group/ciencialist/
>
> * Para sair deste grupo, envie um e-mail para:
> ciencialist-unsubscribe@yahoogrupos.com.br
> <mailto:ciencialist-unsubscribe@yahoogrupos.com.br?subject=Unsubscribe>
>
> * O uso que você faz do Yahoo! Grupos está sujeito aos Termos do
> Serviço do Yahoo! <http://br.yahoo.com/info/utos.html>.
>
>
>
>
> __________ Informação do NOD32 1.1025 (20050314) __________
>
> Esta mensagem foi verificada pelo NOD32 Sistema Antivírus
> http://www.nod32.com.br




SUBJECT: Fw: Quem inventou o Balan�o do parque?
FROM: "Luiz Ferraz Netto" <leobarretos@uol.com.br>
TO: "ciencialist" <ciencialist@yahoogrupos.com.br>
DATE: 16/03/2005 00:42

Foi o mesmo que inventou o pêndulo? :-))

[]'
===========================
Luiz Ferraz Netto [Léo]
leobarretos@uol.com.br
http://www.feiradeciencias.com.br
===========================
-----Mensagem Original-----
De: silvia
Para: leobarretos@uol.com.br
Enviada em: segunda-feira, 14 de março de 2005 21:13
Assunto: Quem inventou o Balanço do parque?


Caro professor,
meu filho está fazendo uma pesquisa para a escola e coube-lhe a tarefa de descobrir quem inventou o balanço?
Procurei em enciclopédia, na internet, no dicionário e nada!!!!! Será que o sr. consegue uma resposta para tal questão.
Desde já lhe agradeço, abraços, Roberto.


--------------------------------------------------------------------------------


Internal Virus Database is out-of-date.
Checked by AVG Anti-Virus.
Version: 7.0.300 / Virus Database: 266.5.0 - Release Date: 25/02/2005

----------

Internal Virus Database is out-of-date.
Checked by AVG Anti-Virus.
Version: 7.0.300 / Virus Database: 266.5.0 - Release Date: 25/02/2005


[As partes desta mensagem que não continham texto foram removidas]



SUBJECT: Fw: 258 anos de falecimento de Isaac Newton
FROM: "Luiz Ferraz Netto" <leobarretos@uol.com.br>
TO: "ciencialist" <ciencialist@yahoogrupos.com.br>
DATE: 16/03/2005 00:43

Foi ontem!
alguém sabe de um link para o consulente?
[]'
===========================
Luiz Ferraz Netto [Léo]
leobarretos@uol.com.br
http://www.feiradeciencias.com.br
===========================
-----Mensagem Original-----
De: Alexandre César aZEVEDO
Para: leobarretos@uol.com.br
Cc: acesar-azevedo@uol.com.br
Enviada em: terça-feira, 15 de março de 2005 13:55
Assunto: 258 anos de falecimento de Isaac Newton


Gostaria de saber se estará disponível no site alguma coisa sobre a data de falecimento de Newton (20/03/1727).
Sem mais, um abraço.
__________________________________________________
Converse com seus amigos em tempo real com o Yahoo! Messenger
http://br.download.yahoo.com/messenger/



--------------------------------------------------------------------------------


Internal Virus Database is out-of-date.
Checked by AVG Anti-Virus.
Version: 7.0.300 / Virus Database: 266.5.0 - Release Date: 25/02/2005

----------

Internal Virus Database is out-of-date.
Checked by AVG Anti-Virus.
Version: 7.0.300 / Virus Database: 266.5.0 - Release Date: 25/02/2005


[As partes desta mensagem que não continham texto foram removidas]



SUBJECT: Fw: quest�o a responder
FROM: "Luiz Ferraz Netto" <leobarretos@uol.com.br>
TO: "ciencialist" <ciencialist@yahoogrupos.com.br>
DATE: 16/03/2005 00:45

?
===========================
Luiz Ferraz Netto [Léo]
leobarretos@uol.com.br
http://www.feiradeciencias.com.br
===========================
-----Mensagem Original-----
De: duarterosa@clix.pt
Para: leobarretos@uol.com.br
Enviada em: terça-feira, 15 de março de 2005 22:43
Assunto: questão a responder


Imaginemos um automóvel que seu motor gira a 3000rpm constantes, trocando-lhe suas rodas de 14 para 16 polegadas e mantendo a mesma altura dos pneus (imaginemos), o perimetro da roda aumenta. E agora pergunto eu, será que o veiculo se desloca mais, ou menos?
Fico a aguardar uma explicação em breve, se possivel, sr. professor,
sem mais obrigado
moises duarte delgadinho
-----------------------------------
Chegou o Clix Turbo ADSL a 8 Megas
Internet + Telefone sem assinatura telefónica
Saiba mais em http://adsl.clix.pt






--------------------------------------------------------------------------------


Internal Virus Database is out-of-date.
Checked by AVG Anti-Virus.
Version: 7.0.300 / Virus Database: 266.5.0 - Release Date: 25/02/2005

----------

Internal Virus Database is out-of-date.
Checked by AVG Anti-Virus.
Version: 7.0.300 / Virus Database: 266.5.0 - Release Date: 25/02/2005


[As partes desta mensagem que não continham texto foram removidas]



SUBJECT: Re: [ciencialist] Fw: 258 anos de falecimento de Isaac Newton
FROM: "Alberto Mesquita Filho" <albmesq@uol.com.br>
TO: <ciencialist@yahoogrupos.com.br>
DATE: 16/03/2005 01:19

----- Original Message -----
From: "Luiz Ferraz Netto"
Sent: Wednesday, March 16, 2005 12:43 AM
Subject: [ciencialist] Fw: 258 anos de falecimento de Isaac Newton

> Foi ontem!

Não! O ontem referido era dia 15, pois sua mensagem está datada de 16 de
março. Se bem que para o deus Léo o tempo não significa nada. Mas vamos ao
que interessa.

> alguém sabe de um link para o consulente?

Respondo após reproduzir a msg do Alexandre.

> -----Mensagem Original-----
> De: Alexandre César aZEVEDO
> Enviada em: terça-feira, 15 de março de 2005 13:55
> Assunto: 258 anos de falecimento de Isaac Newton

> > Gostaria de saber se estará disponível no site alguma coisa sobre a data
> > de falecimento de Newton (20/03/1727).

A data está correta. Vou reproduzir aqui o último parágrafo de "A brief
biographical scketch" encontrado no livro"The Cambridge Companion to
Newton", editado por I. Bernard Cohen e George E. Smith, Cambridge
University Press, Cambridge, United Kingdom, 2002 (a bibliografia está nas
páginas 9-14).

********* início da citação **************

Newton remained intellectually engaged during the last ten years of his
life, though less in science and mathematics than in theology, chronology,
and prophecy. Further editions of his "Opticks" appeared in 1717/18 [and
posthumously in 1730]. Newton also produced a third edition of the
"Principia", appearing in 1726, when he was 83 years old. It does not differ
in essentials from the second edition, the main change was some new text
based on recent data. Thoug his theory of gravity remained still largely
unaccepted on the Continent, there can be no question but that Newton had
himself achieved the status of legend throughout the educated world. He died
on 20 March 1727.

********* final da citação **************

[ ]´s
Alberto
http://ecientificocultural.com/indice.htm



SUBJECT: Re: Fw: questão a responder
FROM: Maria Natália <grasdic@hotmail.com>
TO: ciencialist@yahoogrupos.com.br
DATE: 16/03/2005 03:05


Luiz Ferraz Netto:

Desculpai mestre mas não percebi se era Léo ou Leo? e pelo sim pelo
não estando cheia de trabalho...
Esta questão é de trabalho prático e vai ser resolvida no Robótica
2005 que decorrerá em Coimbra. Vamos substituir as rodas de nosso
Transparente por dois CD-Roms em prinípio mas depois temos de ver o
perigo das derrapagens na pista... Claro que primeiro é de lápis e
papel e depois se experimenta.
Aguardo a resposta dos físicos. Aliás me parece que esta questão já
foi colocada aqui
Um abraço insigne professor
Um abraço
Maria Natália

--- Em ciencialist@yahoogrupos.com.br, "Luiz Ferraz Netto"
<leobarretos@u...> escreveu
> ?
> ===========================
> Luiz Ferraz Netto [Léo]
> leobarretos@u...
> http://www.feiradeciencias.com.br
> ===========================
> -----Mensagem Original-----
> De: duarterosa@c...
> Para: leobarretos@u...
> Enviada em: terça-feira, 15 de março de 2005 22:43
> Assunto: questão a responder
>
>
> Imaginemos um automóvel que seu motor gira a 3000rpm constantes,
trocando-lhe suas rodas de 14 para 16 polegadas e mantendo a mesma
altura dos pneus (imaginemos), o perimetro da roda aumenta. E agora
pergunto eu, será que o veiculo se desloca mais, ou menos?
> Fico a aguardar uma explicação em breve, se possivel, sr. professor,
> sem mais obrigado
> moises duarte delgadinho
> -----------------------------------
> Chegou o Clix Turbo ADSL a 8 Megas
> Internet + Telefone sem assinatura telefónica
> Saiba mais em http://adsl.clix.pt
>
>
>
>
>
>
>
--------------------------------------------------------------------------------
>
>
> Internal Virus Database is out-of-date.
> Checked by AVG Anti-Virus.
> Version: 7.0.300 / Virus Database: 266.5.0 - Release Date: 25/02/2005
>
> ----------
>
> Internal Virus Database is out-of-date.
> Checked by AVG Anti-Virus.
> Version: 7.0.300 / Virus Database: 266.5.0 - Release Date: 25/02/2005
>
>
> [As partes desta mensagem que não continham texto foram removidas]





SUBJECT: FÍSICA SOBRE RODAS, em Portugal
FROM: Maria Natália <grasdic@hotmail.com>
TO: ciencialist@yahoogrupos.com.br
DATE: 16/03/2005 03:09


O Pavilhão do Conhecimento-Ciência Viva acolhe a Física sobre Rodas
nos próximos dias 17, 18 e 19 de Março.
Trata-se da 2ª edição de uma iniciativa que o Núcleo de Física do
Instituto Superior Técnico (NFIST) tem vindo a organizar,
proporcionando um conjunto de experiências interactivas de Física que
percorrem o país de norte a sul durante uma semana, com grande adesão
das escolas do ensino básico e secundário. Física sobre Rodas viaja
num camião cheio de experiências que serão animadas pelos elementos do
NFIST, estudantes de Engenharia Física e Tecnológica do IST, que têm
aliado o seu trabalho académico à divulgação da Física junto dos
colegas mais novos.
Nos mesmos dias, as crianças mais jovens poderão realizar um conjunto
de actividades lúdicas e pedagógicas no autocarro BiblioCiência, uma
iniciativa da Câmara Municipal de Lisboa que contou também com a
colaboração do NFIST e da Ciência Viva.Horário:5ª e 6ª feira: das 10h
às 18hSábado: das 11h às 19h Haverá também sessões de Astronomia com
um pequeno planetário no Foyer do Pavilhão.Entrada livre
Participe.

Espreitai aqui:
http://fsr2.nfist.ist.utl.pt/

Só para fazer "raivinhas" aos senhores futuros engenheiros da lista.
Um abraço
Maria Natália





SUBJECT: Dia Mundial da Árvore e da Floresta
FROM: Maria Natália <grasdic@hotmail.com>
TO: ciencialist@yahoogrupos.com.br
DATE: 16/03/2005 03:13


No dia 21 de Março comemora-se o Dia Mundial da Árvore e da Floresta.
O FAPAS antecipa a comemoração para o dia 20 de Março,domingo, dia
mais adequado para actividades colectivas.

Comemoração do Dia da Árvore, dia 20 de Março (Domingo), 14h30-17h30,
com as seguintes actividades no Jardim Botânico da Universidade de Lisboa.

1.- Sementeira de Carvalhos no viveiro do FAPAS, no Jardim Botânico da
Universidade de Lisboa (Rua da Escola Politécnica)

2.- Breve percurso pelo Jardim, para escolha de árvores ou arbustos
nativos a adoptar pelo FAPAS.


Descrição das actividades:
Local - Jardim Botânico na Rua da Escola Politécnica, 58, em Lisboa
(nas traseiras do Museu da Ciência e do Museu de História Natural)
Data: 20 de Março de 2005
Horário: início às 14h30, final pelas 17h30. Pode participar o tempo
que quiser, dentro deste horário. Aproveite para visitar um dos
jardins mais bonitos de Lisboa.
Ponto de Encontro: às 14h30 junto ao guichet na entrada do Jardim.
Entre pelo portão de ferro da rua da Escola Politécnica, e percorra a
"avenida" das palmeiras, ao lado do Museu da Ciência, até atingir o
portão do Jardim, ao fundo da "avenida".

Para chegar ao Jardim: a rua da Escola Politécnica fica entre o Largo
do Rato e o Largo do Príncipe Real.
Metro: linha amarela, estação Rato. Autocarros: 58 (paragem em frente
à entrada do jardim), 100 (largo do Príncipe Real), 6, 9, 20, 27, 38,
49 (paragens no Largo do Rato).


1.- Sementeira de Carvalhos no viveiro do FAPAS, no Jardim Botânico da
Univ.Lisboa

Desde a fundação do FAPAS, há cerca de 12 anos, que a associação tem
organizado sementeiras e plantações de árvores nativas, numa tentativa
de auxiliar a regeneração das nossas matas nativas, quase
desaparecidas. O Núcleo de Lisboa tem organizado recolhas de sementes,
sementeiras e plantações de árvores (sobreiros, carvalhos-cerquinhos e
carvalhos-negrais) desde 2002, tendo semeado dezenas de milhares de
árvores nas serras de Sintra, Montejunto e Arrábida.

Infelizmente, a seca deste inverno motivou a realização de um número
de sementeiras inferior ao previsto, pelo que vamos tentar aproveitar
os milhares de sementes de carvalho-cerquinho que ficaram em stock,
para produzir pequenas árvores em viveiro. Estas árvores serão
plantadas por voluntários em Áreas Protegidas nos próximos meses de
Novembro e Dezembro.

Associe-se a nós no próximo dia 20 de Março, e semeie uma dúzia de
carvalhos (ou mais) no nosso viveiro. A sementeira consiste em
colocar em recipientes com terra sementes (bolotas) germinadas de
carvalho-cerquinho, árvore nativa da região Centro-litoral do nosso
país. A tarefa é muito simples e rápida, sendo acessível para crianças.
Material aconselhado: luvas de cozinha, avental.
Agradecemos se trouxer consigo alguns pacotes (vazios) de litro de
sumos ou leite, tipo tetrapack ou Vigor. Se possível, passe-os por
água e corte a parte de cima, para formar uma espécie de vaso.

2. Percurso pelo Jardim para adopção de árvores e arbustos.

Em solidariedade com o Jardim Botânico da Universidade de Lisboa, o
FAPAS associa-se à campanha de apadrinhamento de árvores e arbustos
lançada pela Liga dos Amigos do Jardim Botânico. Pelo preço de 1 euro
por metro quadrado, pode-se adoptar durante um ano uma árvore ou
arbusto à nossa escolha. O FAPAS vai efectuar um pequeno percurso para
visitar algumas árvores e arbustos nativos, e seleccionar os
exemplares a adoptar. Os participantes podem-nos acompanhar no
percurso e ajudar-nos a escolher os afilhados, e se pretenderem podem
também apadrinhar um exemplar.
O nome dos padrinhos fica registado on-line no site do Jardim, e dá
direito a um cartão de adopção, que inclui 4 entradas gratuitas no Jardim.

-----------------
INSCRIÇÕES nas actividades:
Envie uma mensagem com o seu nome para fapas.lisboa@clix.pt ,
indicando o número de pessoas. .
Mais informações pelo tm 93 8491355 (João Morais).
----------------

Preço das entradas no jardim:
Adultos com menos de 65 anos- 1,50 euros. Crianças até 6 anos -
grátis. Estudantes e funcionários da Universidade de Lisboa - grátis.
Estudantes e portadores do cartão jovem - 0,60 euros.
Adultos com mais de 65 anos - 0,60 euros


PARTICIPE! POR UMA NOVA FLORESTA, EM DEFESA DA NATUREZA!

-------------------------------------------------
Colabore connosco na Conservação da Natureza: Ajude o FAPAS a
divulgar estas iniciativas.
------------------------------------------------
O FAPAS é uma Associação de defesa da Vida Selvagem com 11 anos de
existência, e uma vasta actividade na defesa e estudo da Natureza e
Vida Selvagem em Portugal. Sede: Rua Alexandre Herculano, 371-4º Dtº,
4000-055 PORTO (telef.22 200 2472).
Publicações: o FAPAS edita trimestralmente a revista Tribuna da
Natureza, e publicou, entre outros, uma série de Guias de Campo de
fauna e flora: Aves de Portugal e Europa, Mamíferos de Portugal e
Europa, Árvores de Portugal e Europa, Anfíbios e Répteis de Portugal.
Para ser sócio do FAPAS: consulte www.fapas.pt , ou contacte-nos por
email ou telefone
Quotas anuais - adultos: 16,50 Euros (inclui a assinatura de 4 números
da revista Tribuna da Natureza); ou 7,50 Euros (sem a assinatura).
juvenis (até 15 anos de idade): 11,50 euros (com assinatura da TN), ou
2,50 euros (sem assinatura).
--------------------------

Saudações Ecologistas

FAPAS - Lisboa
tm: 93 849 1355
fapas.lisboa@clix.pt

E aí é o começo do Outono, né?
Um abraço
Maria Natália







SUBJECT: Re: Fw: bomba de carboneto
FROM: Maria Natália <grasdic@hotmail.com>
TO: ciencialist@yahoogrupos.com.br
DATE: 16/03/2005 03:20


Mas isto é que vai UM PERIGO!!
Uma BOMBA!???
Mas que é isto? Aula para terrorista? LOLLL
Bem agora a sério o carbeto? Será o carbureto e com o qual se faz
acetileno? Deixa-se cair água gota a gota sobre a carbite e depois
como o acetileno é danadinhos para as com...bustões 8largo busto) vai
chegar lume e Kabum!!!!
Será isto? é que a diferença de linguagem pode dar "lapsus linguae"
Passo a bola ao Emiliano...cuidado é explosiva!
Um abraço
Maria Natália

--- Em ciencialist@yahoogrupos.com.br, "Luiz Ferraz Netto"
<leobarretos@u...> escreveu
> Cuimicos ..... ação!
> []'
> ===========================
> Luiz Ferraz Netto [Léo]
> leobarretos@u...
> http://www.feiradeciencias.com.br
> ===========================
> -----Mensagem Original-----
> De: fabiana campos pereira
> Para: leobarretos@u...
> Enviada em: terça-feira, 15 de março de 2005 00:07
> Assunto: bomba de carboneto
>
>
> Olá professor! Meu nome é Fabiana Campos e sou de Belo Horizonte.
> Estou fazendo um trabalho de Química e preciso explicar a reação da
bomba de carboneto; e eu não consegui nenhum site que possa me
fornecer essas informações.
> Por gentileza, o senhor poderia me enviar a explicação no meu e-mail?!
> Caso possa, desde já, fico agradecida.
> Obrigada,
> Fabiana Campos.
>
>
>
--------------------------------------------------------------------------------
> Yahoo! Mail - Com 250MB de espaço. Abra sua conta!
>
>
>
--------------------------------------------------------------------------------
>
>
> Internal Virus Database is out-of-date.
> Checked by AVG Anti-Virus.
> Version: 7.0.300 / Virus Database: 266.5.0 - Release Date: 25/02/2005
>
> ----------
>
> Internal Virus Database is out-of-date.
> Checked by AVG Anti-Virus.
> Version: 7.0.300 / Virus Database: 266.5.0 - Release Date: 25/02/2005
>
>
> [As partes desta mensagem que não continham texto foram removidas]





SUBJECT: Re: Fw: Pergunta! Uma boa pergunta
FROM: Maria Natália <grasdic@hotmail.com>
TO: ciencialist@yahoogrupos.com.br
DATE: 16/03/2005 03:30


E já agora para aula prática de estatística: quantos peões atropelava?
Mas teria de repetir a viagem pelo menos umas 10 vezes...Acho a
amostra pequena...Duplique, duplique.
É que depressa e bem não faz ninguém
Bom dia
Maria Natália


--- Em ciencialist@yahoogrupos.com.br, "Luiz Ferraz Netto"
<leobarretos@u...> escreveu
> Oi Emiliano,
>
> o interesse recai na aceleração e não na velocidade.
> Mas, gostei do carrinho --- aqui em Barretos seria um furor. Vou ver
em quantos segundos eu atravessaria a cidade.
> []'
> ===========================
> Luiz Ferraz Netto [Léo]
> leobarretos@u...
> http://www.feiradeciencias.com.br
> ===========================
> -----Mensagem Original-----
> De: "E m i l i a n o C h e m e l l o" <chemelloe@y...>
> Para: <ciencialist@yahoogrupos.com.br>
> Enviada em: terça-feira, 15 de março de 2005 14:18
> Assunto: Re: [ciencialist] Fw: Pergunta! Uma boa pergunta
>
>
>
> Não fiz muita pesquisa... mas acho que uma boa referência é o site
do guinness:
>
http://www.guinnessworldrecords.com/gwr5/content_pages/record.asp?recordid=43540
>
> [ ] 's do Emiliano Chemello
> emiliano@q...
> http://www.quimica.net/emiliano
> http://www.ucs.br/ccet/defq/naeq
> [ MSN ] chemelloe@h...
> [ ICQ ] 145060604
>
> " Rien ne se perd, rien ne se crée,
> tout se transforme."
>
> Antoine Laurent de Lavoisier (químico francês, 1743 - 1794)
>
> ----- Original Message -----
> From: Luiz Ferraz Netto
> To: ciencialist
> Sent: Tuesday, March 15, 2005 1:45 PM
> Subject: [ciencialist] Fw: Pergunta! Uma boa pergunta
>
>
> Realmente, gostei da pergunta: "Aqui na alface da Terra qual o
equipamento (aparelho, máquina, processo .... ) experimenta a maior
aceleração (mesmo que seja por pequeno intervalo de tempo)?
> É aquele carrinho esquisito que atinge 500 km/h? É a bala de fuzil?
>
> []'
> ===========================
> Luiz Ferraz Netto [Léo]
> leobarretos@u...
> http://www.feiradeciencias.com.br
> ===========================
> -----Mensagem Original-----
> De: "Saulo Fulanete Trombini" <sauloft@p...>
> Para: <leobarretos@u...>
> Enviada em: sábado, 12 de março de 2005 10:53
> Assunto: Pergunta!
>
>
> Prezado Professor,
> Baseado no fato de que o senhor tem muito mais
comnhecimento, e recursos do
> que eu, venho através desta, lhe perguntar: Qual a maior
aceleração mecânica
> terrestre? Parabenizo-o pelo seu excelente site, pois através dele
estou
> melhorando meus conhecimentos, e adquirindo novas idéias que
posteriormente
> levarei a Feira de Ciências da minha escola.
>
> Agradecendo, despeço-me;
>
> Saulo Fulanete Trombini
>
>
>
> --
> Internal Virus Database is out-of-date.
> Checked by AVG Anti-Virus.
> Version: 7.0.300 / Virus Database: 266.5.0 - Release Date: 25/02/2005
>
>
>
>
> --
> Internal Virus Database is out-of-date.
> Checked by AVG Anti-Virus.
> Version: 7.0.300 / Virus Database: 266.5.0 - Release Date: 25/02/2005
>
>
>
> ##### ##### #####
>
> Para saber mais visite
> http://www.ciencialist.hpg.ig.com.br
>
>
> ##### ##### ##### #####
>
>
> Yahoo! Grupos, um serviço oferecido por:
>
>
>
>
>
>
>
>
------------------------------------------------------------------------------
> Links do Yahoo! Grupos
>
> a.. Para visitar o site do seu grupo na web, acesse:
> http://br.groups.yahoo.com/group/ciencialist/
>
> b.. Para sair deste grupo, envie um e-mail para:
> ciencialist-unsubscribe@yahoogrupos.com.br
>
> c.. O uso que você faz do Yahoo! Grupos está sujeito aos Termos
do Serviço do Yahoo!.
>
>
>
> [As partes desta mensagem que não continham texto foram removidas]
>
>
>
> ##### ##### #####
>
> Para saber mais visite
> http://www.ciencialist.hpg.ig.com.br
>
>
> ##### ##### ##### #####
> Links do Yahoo! Grupos
>
>
>
>
>
>
>
>
>
>
> --
> Internal Virus Database is out-of-date.
> Checked by AVG Anti-Virus.
> Version: 7.0.300 / Virus Database: 266.5.0 - Release Date: 25/02/2005
>
>
>
>
> --
> Internal Virus Database is out-of-date.
> Checked by AVG Anti-Virus.
> Version: 7.0.300 / Virus Database: 266.5.0 - Release Date: 25/02/2005





SUBJECT: Re: [ciencialist] Fw: 258 anos de falecimento de Isaac Newton
FROM: "Alberto Mesquita Filho" <albmesq@uol.com.br>
TO: <ciencialist@yahoogrupos.com.br>
DATE: 16/03/2005 06:06

----- Original Message -----
From: "Alberto Mesquita Filho"
Sent: Wednesday, March 16, 2005 1:19 AM
Subject: Re: [ciencialist] Fw: 258 anos de falecimento de Isaac Newton

Na mensagem acima, ONDE SE LÊ:

> (a bibliografia está nas páginas 9-14).

LEIA-SE:

(a biografia está nas páginas 9-14).

[ ]´s
Alberto
http://ecientificocultural.com/indice.htm
Mas indiferentemente a tudo isso, o neutrino tem massa, o elétron não é
uma carga elétrica coulombiana e a Terra se move. E a história se repetirá.



SUBJECT: Re: [ciencialist] Re: Fw: Pergunta! Uma boa pergunta
FROM: "E m i l i a n o C h e m e l l o" <chemelloe@yahoo.com.br>
TO: <ciencialist@yahoogrupos.com.br>
DATE: 16/03/2005 08:56

Maria Natália,

Sinceramente, não entendi nada do que você escreveu :-)

[ ] 's do Emiliano Chemello
emiliano@quimica.net
http://www.quimica.net/emiliano
http://www.ucs.br/ccet/defq/naeq
[ MSN ] chemelloe@hotmail.com
[ ICQ ] 145060604

" Rien ne se perd, rien ne se crée,
tout se transforme."

Antoine Laurent de Lavoisier (químico francês, 1743 - 1794)

----- Original Message -----
From: Maria Natália
To: ciencialist@yahoogrupos.com.br
Sent: Wednesday, March 16, 2005 3:30 AM
Subject: [ciencialist] Re: Fw: Pergunta! Uma boa pergunta



E já agora para aula prática de estatística: quantos peões atropelava?
Mas teria de repetir a viagem pelo menos umas 10 vezes...Acho a
amostra pequena...Duplique, duplique.
É que depressa e bem não faz ninguém
Bom dia
Maria Natália


--- Em ciencialist@yahoogrupos.com.br, "Luiz Ferraz Netto"
<leobarretos@u...> escreveu
> Oi Emiliano,
>
> o interesse recai na aceleração e não na velocidade.
> Mas, gostei do carrinho --- aqui em Barretos seria um furor. Vou ver
em quantos segundos eu atravessaria a cidade.
> []'
> ===========================
> Luiz Ferraz Netto [Léo]
> leobarretos@u...
> http://www.feiradeciencias.com.br
> ===========================
> -----Mensagem Original-----
> De: "E m i l i a n o C h e m e l l o" <chemelloe@y...>
> Para: <ciencialist@yahoogrupos.com.br>
> Enviada em: terça-feira, 15 de março de 2005 14:18
> Assunto: Re: [ciencialist] Fw: Pergunta! Uma boa pergunta
>
>
>
> Não fiz muita pesquisa... mas acho que uma boa referência é o site
do guinness:
>
http://www.guinnessworldrecords.com/gwr5/content_pages/record.asp?recordid=43540
>
> [ ] 's do Emiliano Chemello
> emiliano@q...
> http://www.quimica.net/emiliano
> http://www.ucs.br/ccet/defq/naeq
> [ MSN ] chemelloe@h...
> [ ICQ ] 145060604
>
> " Rien ne se perd, rien ne se crée,
> tout se transforme."
>
> Antoine Laurent de Lavoisier (químico francês, 1743 - 1794)
>
> ----- Original Message -----
> From: Luiz Ferraz Netto
> To: ciencialist
> Sent: Tuesday, March 15, 2005 1:45 PM
> Subject: [ciencialist] Fw: Pergunta! Uma boa pergunta
>
>
> Realmente, gostei da pergunta: "Aqui na alface da Terra qual o
equipamento (aparelho, máquina, processo .... ) experimenta a maior
aceleração (mesmo que seja por pequeno intervalo de tempo)?
> É aquele carrinho esquisito que atinge 500 km/h? É a bala de fuzil?
>
> []'
> ===========================
> Luiz Ferraz Netto [Léo]
> leobarretos@u...
> http://www.feiradeciencias.com.br
> ===========================
> -----Mensagem Original-----
> De: "Saulo Fulanete Trombini" <sauloft@p...>
> Para: <leobarretos@u...>
> Enviada em: sábado, 12 de março de 2005 10:53
> Assunto: Pergunta!
>
>
> Prezado Professor,
> Baseado no fato de que o senhor tem muito mais
comnhecimento, e recursos do
> que eu, venho através desta, lhe perguntar: Qual a maior
aceleração mecânica
> terrestre? Parabenizo-o pelo seu excelente site, pois através dele
estou
> melhorando meus conhecimentos, e adquirindo novas idéias que
posteriormente
> levarei a Feira de Ciências da minha escola.
>
> Agradecendo, despeço-me;
>
> Saulo Fulanete Trombini
>
>
>
> --
> Internal Virus Database is out-of-date.
> Checked by AVG Anti-Virus.
> Version: 7.0.300 / Virus Database: 266.5.0 - Release Date: 25/02/2005
>
>
>
>
> --
> Internal Virus Database is out-of-date.
> Checked by AVG Anti-Virus.
> Version: 7.0.300 / Virus Database: 266.5.0 - Release Date: 25/02/2005
>
>
>
> ##### ##### #####
>
> Para saber mais visite
> http://www.ciencialist.hpg.ig.com.br
>
>
> ##### ##### ##### #####
>
>
> Yahoo! Grupos, um serviço oferecido por:
>
>
>
>
>
>
>
>
------------------------------------------------------------------------------
> Links do Yahoo! Grupos
>
> a.. Para visitar o site do seu grupo na web, acesse:
> http://br.groups.yahoo.com/group/ciencialist/
>
> b.. Para sair deste grupo, envie um e-mail para:
> ciencialist-unsubscribe@yahoogrupos.com.br
>
> c.. O uso que você faz do Yahoo! Grupos está sujeito aos Termos
do Serviço do Yahoo!.
>
>
>
> [As partes desta mensagem que não continham texto foram removidas]
>
>
>
> ##### ##### #####
>
> Para saber mais visite
> http://www.ciencialist.hpg.ig.com.br
>
>
> ##### ##### ##### #####
> Links do Yahoo! Grupos
>
>
>
>
>
>
>
>
>
>
> --
> Internal Virus Database is out-of-date.
> Checked by AVG Anti-Virus.
> Version: 7.0.300 / Virus Database: 266.5.0 - Release Date: 25/02/2005
>
>
>
>
> --
> Internal Virus Database is out-of-date.
> Checked by AVG Anti-Virus.
> Version: 7.0.300 / Virus Database: 266.5.0 - Release Date: 25/02/2005





##### ##### #####

Para saber mais visite
http://www.ciencialist.hpg.ig.com.br


##### ##### ##### #####


Yahoo! Grupos, um serviço oferecido por:

São Paulo Rio de Janeiro Curitiba Porto Alegre Belo Horizonte Brasília




------------------------------------------------------------------------------
Links do Yahoo! Grupos

a.. Para visitar o site do seu grupo na web, acesse:
http://br.groups.yahoo.com/group/ciencialist/

b.. Para sair deste grupo, envie um e-mail para:
ciencialist-unsubscribe@yahoogrupos.com.br

c.. O uso que você faz do Yahoo! Grupos está sujeito aos Termos do Serviço do Yahoo!.



[As partes desta mensagem que não continham texto foram removidas]



SUBJECT: Re: [ciencialist] Re: Foucault / Luz ondas ou corpusculos.
FROM: "Alberto Mesquita Filho" <albmesq@uol.com.br>
TO: <ciencialist@yahoogrupos.com.br>
DATE: 16/03/2005 09:39

----- Original Message -----
From: "JVictor"
Sent: Thursday, March 17, 2005 8:00 AM
Subject: Re: [ciencialist] Re: Foucault / Luz ondas ou corpusculos.

> Mostrei a minha sobrinha a sua foto(ela o conhece pois sempre lê seus
> trabalhos na minha cx. de entrada). E disse: pôxa, é elegante e jovem,
> ainda(êsse ainda é de lascar...). E não está de jogar fora!.. Piveta
> atirada..

Bem, depois dessa nem sei o que dizer. Mas antes que a turma fique ouriçada,
diria que as fotos estão no site http://fmusp51a.ecientificocultural.com/ .
Chegando nesta página cliquem no link "Encontro de Avaré - 35 anos" e após
procurarem as minhas fotos na página indicada percebam que o vovô da
Ciencialist "não é de se jogar fora", apesar de seus 62 anos. ;-)

> Bem, só para registrar que vimos a alegria estampada naquelas fotos.

Por falar em foto, andei pesquisando o meu site e encontrei uma foto do Max
Planck em http://ecientificocultural.com/ECC2/figuras/planck.jpg . Comparem
com a foto do outro vovô da Ciencialist, o Léo, e que está em
http://www.feiradeciencias.com.br/image00/fcluiz.gif . Imaginem o Planck
olhando para a foto do Léo e dizendo: "Eu sou você amanhã".
;-)))))))))

> Quero lhe dizer que minha admiração por Newton é profunda. Sua
> importância para a ciência, toda ela, inclusive para a linguagem da
> natureza, coloca-o num pedestal que, para mim, chegar superar Einstein e
> Poincaré.

Concordo contigo, mas ainda assim, e por incrível que pareça, a minha
admiração por Einstein é maior do que por Newton. Como cientista, Newton foi
insuperável, mas aprende-se muito mais a ser cientista lendo-se Einstein do
que lendo-se Newton. Não sei se dá para entender, mas através de seus
escritos Einstein nos mostra como um cientista deve ser, e são pouquíssimos
os cientistas que têm esse pendor. Quanto ao mais, teorias são teorias e
estão aí para serem derrubadas. Acredito não obstante que as idéias de
Newton permanecerão e muitas até mesmo ressurgirão das cinzas, onde
pretenderam colocá-las, mas a teoria da relatividade de Einstein se
esfacelará no decorrer da primeira metade deste século.

> > Alberto: Pô, mas a ficção quântica é tão simples, tão elementar, tão
> > lógica!!! Não sei nem como começar a explicar uma coisa tão banal. É só
> > introduzir umas duas ou três hipóteses "ad hoc" e tudo se esclarecerá.
> > Pô! Será que você andou cabulando as aulas de ficção quântica no tempo
> > de universidade?

> Victor: Mas e quem disse que a quântica é simples, fácil e lógica? Se o
> fiz alguma vez, foi em tom de ironia.

Não, fui eu quem disse isso. É muito simples e fácil criar uma hipótese "ad
hoc" a remendar um achado experimental incompatível com as hipóteses básicas
de uma teoria. O difícil é manter-se fiel ao método científico, como fizeram
Galileu, Newton, Maxwell e, até mesmo, se bem que sob certas reservas, os
fundadores da física "moderna" (com algumas exceções). Por outro lado, ela é
tão lógica quanto qualquer boa peça teatral de ficção. Logicidade não é tudo
em ciência, ainda que seja fundamental. Mas vou parar por aqui, pois todos
já sabem o que penso a respeito.

[ ]´s
Alberto
http://ecientificocultural.com/indice.htm
Mas indiferentemente a tudo isso, o neutrino tem massa, o elétron não é
uma carga elétrica coulombiana e a Terra se move. E a história se repetirá.



SUBJECT: Re: [ciencialist] Re: 'Theory of everything' tying researchers up in knots
FROM: "Sergio M. M. Taborda" <sergiotaborda@terra.com.br>
TO: ciencialist@yahoogrupos.com.br
DATE: 16/03/2005 10:04

Manuel Bulcão wrote:

>
> --- Em ciencialist@yahoogrupos.com.br, "Sergio M. M. Taborda"
> <sergiotaborda@t...> escreveu
> > já todos aqui deveriam saber o que está escrito no texto. Que a
> teoria das superstrings não passa de uma teoria, inacabada, que
> nunca mais acaba, e que acaba por não significar nada para a ciencia
> e a fisica em particular. Que é apenas uma forma de muitos terem
> empregos. Fico contente por finalmente alguem por o dedo na ferida.
> Esta "quasi-teologia" - como chamam no texto , e acho muito
> apropriado - já chateia e tem que ser extinta.
>
> Manuel: Pois eu concordo com a assertiva de Popper segundo a qual "é
> preferível uma explicação metafísica a nenhuma explicação".
>
> De resto, não há conhecimento "acabado".

Pois eu prefiro a máxima : " É preferivel nenhuma explicação que uma
enganação".
Quando a mainstream se der conta da enganação em que estão metidos
procuraram novas teorias. Por enquanto, não fazem isso, pq continuam
pensando com vc, que uma explicação sem utilidade cientifica é melhor
que nada. Se fisica fosse filosofia seria verdade, mas fisica é uma
ciencia. O minimo que se pede é que produza teorias cientificas, e não
metafisicas.

Sergio Taborda


SUBJECT: Re: [ciencialist] Fw: Quem inventou o Balanço do parque?
FROM: Marcelo Cortimiglia <cortimiglia@gmail.com>
TO: ciencialist@yahoogrupos.com.br
DATE: 16/03/2005 11:47

Ué...
Porque o pai está fazendo a pesquisa do filho?!

Abraços,

Marcelo Cortimiglia


On Wed, 16 Mar 2005 00:42:15 -0300, Luiz Ferraz Netto
<leobarretos@uol.com.br> wrote:
> Foi o mesmo que inventou o pêndulo? :-))
>
> []'
> ===========================
> Luiz Ferraz Netto [Léo]
> leobarretos@uol.com.br
> http://www.feiradeciencias.com.br
> ===========================
> -----Mensagem Original-----
> De: silvia
> Para: leobarretos@uol.com.br
> Enviada em: segunda-feira, 14 de março de 2005 21:13
> Assunto: Quem inventou o Balanço do parque?
>
>
> Caro professor,
> meu filho está fazendo uma pesquisa para a escola e coube-lhe a tarefa de
> descobrir quem inventou o balanço?
> Procurei em enciclopédia, na internet, no dicionário e nada!!!!! Será que o
> sr. consegue uma resposta para tal questão.
> Desde já lhe agradeço, abraços, Roberto.
>


SUBJECT: off-topic: Minha idéia vai ser desenvolvida + dúvida sobre direitos autorais
FROM: "Eurico Ferreira de Souza Jr." <caodejah@yahoo.com.br>
TO: acropolis@yahoogrupos.com.br
CC: ciencialist@yahoogrupos.com.br
DATE: 16/03/2005 12:01

Salve Galera,

Minha idéia de fazer um videogame de Capoeira vai ser financiada
pelo Minc. Uma empresa de Curitiba vai desenvolver o game.
Eu tava concorrendo também, mas fico feliz que pelo menos
a coisa tá acontecendo... Entrei em contato com chefe do projeto
e vou colaborar, afinal é meu interesse que o jogo fique perfeito, e
tenho certeza que vou aprender bastante nessa empreitada.
Vou oferecer todas as idéias que tenho e o apoio de todos
meus amigos que estão na minha equipe.
PERGUNTO: alguém tem alguma dica de como vou receber
meus direitos autorais? como se calcula? é um índice fixo (tipo
imposto) ou tenho que pleitear e negociar para receber?


_\|/_
__________________________________________________
Converse com seus amigos em tempo real com o Yahoo! Messenger
http://br.download.yahoo.com/messenger/

[As partes desta mensagem que não continham texto foram removidas]



SUBJECT: Livre-docência
FROM: "E m i l i a n o C h e m e l l o" <chemelloe@yahoo.com.br>
TO: <ciencialist@yahoogrupos.com.br>
DATE: 16/03/2005 13:22

Caros amigos,

Na análise de alguns currículos (Lattes), me deparei com a seguinte
formação: "Livre-docência". O que isto significa exatamente? Como se
consegue?

[ ] 's do Emiliano Chemello
emiliano@quimica.net
http://www.quimica.net/emiliano
http://www.ucs.br/ccet/defq/naeq
[ MSN ] chemelloe@hotmail.com
[ ICQ ] 145060604

" Rien ne se perd, rien ne se crée,
tout se transforme."

Antoine Laurent de Lavoisier (químico francês, 1743 - 1794)




SUBJECT: Fw: História da química analítica
FROM: "E m i l i a n o C h e m e l l o" <chemelloe@yahoo.com.br>
TO: <ciencialist@yahoogrupos.com.br>, <quimica-qaw@yahoogrupos.com.br>, <quimica@grupos.com.br>
DATE: 16/03/2005 16:53

Alguém conhece alguém site para indicar para Joelma?
(veja mensagem original abaixo)

---
[ ] 's do Emiliano Chemello
emiliano@quimica.net
http://www.quimica.net/emiliano

----- Original Message -----
From: JJFadigas
To: emiliano@quimica.net
Sent: Wednesday, March 16, 2005 4:38 PM
Subject: História da química analítica


Boa tarde!

Sou professora de química e estou precisando de ajuda para encontrar sites
que falem da história da química analítica.
Agradeço desde já a colaboração de todos.
Um abraço,
Joelma




SUBJECT: Re: [ciencialist] Re: Foucault / Luz ondas ou corpusculos.
FROM: JVictor <jvoneto@uol.com.br>
TO: ciencialist@yahoogrupos.com.br
DATE: 16/03/2005 18:35

Hélio Ricardo Carvalho escreveu:

>
>
>
> Hélio: Creio que você ainda não entendeu as minhas motivações ao
> começar esta
> thread.

Victor: Um dia eu entenderei, um dia. Mas vamos lá, vamos ver se me
salvo, ao menos pelo gongo!.

> Hélio:vou aqui tentar esclarecer.
> Você disse:
> >Victor: O modêlo ondulatório conduz diretamente a que a velocidada
> >da luz num meio mais refringente é menor que essa velocidade em
> >um meio menos refrigente. Tanto experimental como
> >teoricamente! O contrário para um modelo corpuscular.
> >Honestamente, não sei como chegar a resultados diferentes

>
> Hélio: Experimental sim, "teoricamente" NÃO.

Victor: A lei da refração já era conhecida desde a época de
Descartes(1596-1650), a quem devemos o enunciado atual da lei da refração
e a explicação correta do fenônemo do arco-iris, como já disse
anteriormente. Entre 1801 e 1803, Thomas Young estudou o fenênomeno da
interferência da
interferência luminosa. E os resultados obtidos apontavam para algo
ondulatório, não corpuscular. Estes estudos foram refeitos por outros
cientistas. Young, assim, pode fazer a associação correta com a teoria
ondulatória de Huygnes. Por volta de 1819 Fresnel apresenta à Acedemia
de Ciências seus trabalhos sobre difração. E mais, dá uma estrutura
matemática poderosa à teoria ondulatória, conforme já disse também.
Basicamente, é a estrutura que se estuda e se usa hoje

> em dia. A maior parte do trabalho de Fresnel, aliás, foi matemático,
> que chegou a ser acusado de fazer mágicas para justificar suas teses
> ondulatórias, referência essa feita pelos adeptos da teoria
> newtoniana. Mas não era assim, rigorosamente falando. A teoria
> corpuscular leva a que a lei da refração seja o inverso do que
> expressa a lei da refração obtida com a teoria ondulatória. E a que a
> velocidade da luz num meio mais refrigente seja maior que num menos
> refringente. Teoricamente, pode-se facilmente se chegar a tal
> resultado. É só um pouco de geometria e a explicação de Newton para o
> fenômeno, que é assim: " seja um raio de luz incidindo em um ponto O
> na superfície de um recipiente com água. Se um corpúsculo luminoso
> avança ao longo daquele raio com velocidade Vo, Newton admite que,
> muito próximo do ponto de incidência, o corpúsculo INFLETE a sua
> trajetória, em consequência de uma " atração" das partículas da
> água(mais "pesadas" que as do ar). O ângulo de refração R é menor que
> o ângulo de incidência I de acordo com a experiência e, também, com
> esta hipótese. Ora a "atração" referida tem de ser entendida como um
> força, normal à superfície de separação ar-água, que vai pois alterar
> a componente vertical da velocidade da luz; e, supondo, como é
> natural, que a componente horizontal não se modifique, uma simples
> construção geométrica mostra que Vo/Vi = senR/SenI=No/Na, onde No é o
> indice de refração do meio menos refringente onde a velocidade da luz
> é Vo. As demais variáveis referem-se ao mais refringente. Ora, se uma
> força é aplicada, segue-se que no meio Na a velocidade é maior, mas
> isto faz o ângulo de refração ser menor que o de incidência. Logo,
> pela relação acima, resulta fatalmente que Va>Vo. Usando o principio
> de Huygnes, chega-se a um resultado contrário. Utilizando-se a Lei de
> Descartes(que Newton fazia questão de atribuí-la a Snell também) e
> mais o princípio de Huygnes, chega-se a um resultado oposto.
> Então, antes do experimento de Foulcault, em 1862, isto tudo já era
> conhecido, sobretudo o aparato matemático de Fresnel, para representar
> um onda! O papel de Fresnel e de Arago nessa história foi cruciall.
> Fresnel morreu, bem jovem, mas Arago continuou a desenvolver suas
> idéias, mas não foi capaz de seguir adiante. Mas, como referi acima,
> em 1862 Foucault mostrou experimentalmente que a velocidade da luz na
> água era menor na água que no ar, contrariando a previsão newtoniana e
> justificando a previsão da teoria ondulatória!. Daí, minha afirmação,
> com a qual você só concordou em parte. Só para terminar, acrescento a
> Fresnel é atribuída a glória de ter descoberto o caráter
> transversal das vibrações luminosas, o que aconteceu ontem, em 1819!
> Há muito, muito, a falar a esse respeito.
> Ou seja, por isso que eu disse o que disse naquele trecho em que você
> concordou 50% (pelo menos fiquei na média...).

Hélio: A luz (seja o que ela for) é mais lenta em meios mais refringentes
(~mais densos), isto é o FATO. A experiência de Foucault mostrou isto.

> Mas que teoria usar para explicar este fato? Usando só teorias da
> época, acho que este experimento sozinho reforça a luz corpuscular e
> descarta a ondulatória.

Victor: Não consigo enxergar assim.

> Hélio:(Explico este raciocínio mais abaixo)
> Claro que, APESAR deste experimento (Foucault) ter dito isto, outros,
> como aquele de Fresnel/Arago, diziam o contrário.
> Meu objetivo nesta thread é descobrir porque um experimento como o de
> Foucault que tinha tudo para descartar a teoria ondulatória pode ser
> apontado como O que a consagrou????

Victor: Acho que expliquei as razões! Não estou entendendo, ainda.

>
> Hélio: Explicando:
> Quais eram os fenômenos tidos como ondulatórios na época?

Victor: Já eram conhecidos: difração, refração, dupla refração,
polarização, o caráter transversal das ondas luminosas(aliás, desde
Hooke, em 1675: o movimento da luz, quando é produzido num meio
homogênio, propaga-se por impulsos ou ondas simples e de forma
constante, PERPENDICULARES à linha de propagação. Essa proposta
continha, já naquela época uma propriedade fundamental da luz. No
entanto, ninguém ligou! Já Huygnes descobriu entre um montão de coisas,
a polarização da luz, que também não foi capaz de associar à sua
transversalidade, pois a polarização exige que a propagação seja
transversal. Por outro lado, também Hooke não atinou para a polarização!
Newton empurrou para debaixo do tapete tudo isso! Mas eu acho que Newton
fazia isso por pirraça, acho que sabia que a teoria ondulatória também
estava correta, pois não explicava tanta coisa que a dele explicava?.
Mas, mesmo que que reconhecesse que Huygnes ou Hooke tivessem razão, ele
batia o pé e não aceitava, nem no tapa...). De qualquer maneira, deixo
claro que esse assunto, comportamento da luz, nunca foi algo fácil de
compreender. Veja, até hoje estamos aquí discutindo a respeito. Outra
informação da qual me lembrei agora: Fresnel não cogitava da natureza da
luz! Apenas se interessou fortemente em descrever seu comportamento,
como lhe parecia através de experimentos.

> Hélio: Creio que todos eles mostravam uma velocidade maior em meios mais
> densos. Isto fazia sentido pois quanto mais matéria mais fácil e
> rápido a transmissão de movimentos (energia) entre porções de matéria
> adjacentes.
> No caso de corpusculos deveria acontecer ao contrário.
> Alguma coisa do tipo "quanto mais denso maior a secção de choque" ou
> algo do gênero.


Victor: É como já lhe disse. Não encontrei nada a esse respeito. Os
seguidores de cada teoria tinham consciência das consequências de suas
teorias quanto aos diversos fenômenos.

>
> Hélio:Este é o motivo da minha pergunta:
>
> "Por que a experiência de Foucault não descartou de vez a luz
> ondulatória também????"

Victor: Acho que continuo sem entender sua questão. Mas acho que disse
tudo que podia dizer a respeito, nas respostas anteriores e em outros
e-mails

>
>
>
> Hélio: Quando você diz:
> >Bem, não acho que os resultados de Foulcault sejam dúbios. Mas é o
> >que eu acho, em função do que sei.


Victor: Confirmo a postura acima. O papo anterior é claro a esse respeito.

>
> Hélio: Você acha isto em função do que você sabe HOJE. Eu também não acho
> HOJE que os resultados sejam dúbios para a escolha de modelo para luz.
>
> Mas eu estou sempre me reportando à época!!!

Victor: Mesmo assim, Hélio. Aliás, principalmente assim, em vista do
escrevi acima.

>
> Hélio: Só para terminar.
> Se NA ÉPOCA eu fosse um cientista com C maiúsculo e só visse o
> resultado de Foucault, eu iria dizer que a luz NÃO era ondulatória

Victor: Eu não, tô fora!.
achei muito boa a discussão. Suas questões realmente forçam a quem se
digne responder a se esforçar, não dizer besteiras(pelo menos tentar
não dizê-las..., que é o meu caso) e ir direto ao ponto.
Acabei azeitando um pouco o juízo.
Parabéns prá você.

> Sds,


Victor.


>
>
>
>
>
>
> ##### ##### #####
>
> Para saber mais visite
> http://www.ciencialist.hpg.ig.com.br
>
>
> ##### ##### ##### #####
>
>
> *Yahoo! Grupos, um serviço oferecido por:*
> PUBLICIDADE
> <http://br.rd.yahoo.com/SIG=12agq5j32/M=264379.5078783.6203979.1588051/D=brclubs/S=2137111528:HM/EXP=1110992282/A=2332652/R=0/id=noscript/SIG=119058f8i/*http://br.download.yahoo.com/messenger/>
>
>
>
> ------------------------------------------------------------------------
> *Links do Yahoo! Grupos*
>
> * Para visitar o site do seu grupo na web, acesse:
> http://br.groups.yahoo.com/group/ciencialist/
>
> * Para sair deste grupo, envie um e-mail para:
> ciencialist-unsubscribe@yahoogrupos.com.br
> <mailto:ciencialist-unsubscribe@yahoogrupos.com.br?subject=Unsubscribe>
>
> * O uso que você faz do Yahoo! Grupos está sujeito aos Termos do
> Serviço do Yahoo! <http://br.yahoo.com/info/utos.html>.
>
>
>
>
> __________ Informação do NOD32 1.1026 (20050314) __________
>
> Esta mensagem foi verificada pelo NOD32 Sistema Antivírus
> http://www.nod32.com.br




SUBJECT: Re: [ciencialist] Fw: História da química analítica
FROM: "Alberto Mesquita Filho" <albmesq@uol.com.br>
TO: <ciencialist@yahoogrupos.com.br>
DATE: 16/03/2005 18:40

----- Original Message -----
From: "E m i l i a n o C h e m e l l o"
Sent: Wednesday, March 16, 2005 4:53 PM
Subject: [ciencialist] Fw: História da química analítica

> Alguém conhece alguém site para indicar para Joelma?
> (veja mensagem original abaixo)

> ----- Original Message -----
> From: JJFadigas
> Sent: Wednesday, March 16, 2005 4:38 PM
> Subject: História da química analítica

> > Sou professora de química e estou precisando de ajuda para encontrar
> > sites que falem da história da química analítica.

Vide:

1) Selected Classic Papers from the History of Chemistry
http://web.lemoyne.edu/~giunta/papers.html

2) Classic Papers from the History of Chemistry
http://dbhs.wvusd.k12.ca.us/webdocs/Chem-History/Classic-Papers-Menu.html

[ ]´s
Alberto
http://ecientificocultural.com/indice.htm
Mas indiferentemente a tudo isso, o neutrino tem massa, o elétron não é
uma carga elétrica coulombiana e a Terra se move. E a história se repetirá.



SUBJECT: Fw: QU�MICA
FROM: "Luiz Ferraz Netto" <leobarretos@uol.com.br>
TO: "ciencialist" <ciencialist@yahoogrupos.com.br>
DATE: 16/03/2005 19:15

Cuímicos ..... ?

[]'
===========================
Luiz Ferraz Netto [Léo]
leobarretos@uol.com.br
http://www.feiradeciencias.com.br
===========================
-----Mensagem Original-----
De: gapv
Para: leobarretos
Enviada em: quarta-feira, 16 de março de 2005 15:04
Assunto: QUÍMICA


Caro professor sou aluno universitário da UFPB (Universidade Federal da Paraíba) do curso de ciencias agrárias, eutou com dificuldades de encontrar em livros e até mesmo na internet as questões que lhe mando por meio deste e-mail, peço por gentileza que me ajude com o seguinte:

1 Por que quando misturamos mesma quantidade de Alcool Etilico e de Água, e molhamos um pedaço de tecido nessa mistura e ateamos fogo no final quando o fogo se apaga o tecido não queima?

2 Se jogarmos Hidróxido de Amônio e Fenolftalina misturados em um tecido branco ficará roseo, com o tempo desaparecerar a mancha roseo, pOr que?

3 Que substancia se forma depois da desidratação da sacarose, usando acido sufurico para desidrata-lá?

4 Usando iodo cristalizado e aquecendo-o até ele evaporar e colocando um papel tocado por alguém acima da evaporação do mesmo, aparecerar a impresão digital de quem o tocou. Por que?


Desde Já agradeço
Gilberto Azevedo

João Câmara-RN.


--------------------------------------------------------------------------------


Internal Virus Database is out-of-date.
Checked by AVG Anti-Virus.
Version: 7.0.300 / Virus Database: 266.5.0 - Release Date: 25/02/2005

----------

Internal Virus Database is out-of-date.
Checked by AVG Anti-Virus.
Version: 7.0.300 / Virus Database: 266.5.0 - Release Date: 25/02/2005


[As partes desta mensagem que não continham texto foram removidas]



SUBJECT: Re: [ciencialist] 'Theory of everything' tying researchers up in knots
FROM: JVictor <jvoneto@uol.com.br>
TO: ciencialist@yahoogrupos.com.br
DATE: 16/03/2005 21:00

Sergio M. M. Taborda escreveu:

> >
> > 'Theory of everything' tying researchers up in knots
> > Keay Davidson, Chronicle Science Writer
> >
> > Monday, March 14, 2005
> >
> já todos aqui deveriam saber o que está escrito no texto. Que a teoria
> das superstrings não passa de uma teoria, inacabada, que nunca mais
> acaba, e que acaba por não significar nada para a ciencia e a fisica em
> particular. Que é apenas uma forma de muitos terem empregos.
> Fico contente por finalmente alguem por o dedo na ferida. Esta
> "quasi-teologia" - como chamam no texto , e acho muito apropriado - já
> chateia e tem que ser extinta.
>
> Victor: Concordo em gênero, número e grau. Nunca acreditei que uma
> teoria do tudo pudesse ter algum sucesso; não é muito difícil
> chegar-se a essa conclusão. E não compreendo como liberam verbas,
> quando estas deveriam ser destinadas a coisas úteis(não digo mais
> úteis, pois a utilidade dessas teoria das cordas é zero). Parece que
> os chefes de departamento o fazem intimidados pela eventual autoridade
> de quem solicita. Os pesquisadores que enveredam por essas áreas
> realmente não estão com nada. Estão jogando fora o dinheiro do
> contribuinte e se aproveitando do efeito de suas falácias junto a
> pessoas alheias à ciência. Posso até concordar que alguém pesquise
> esse assunto inútil. Tudo bem. É um direito de cada um. Mas que pague
> as contas de seu próprio bolso.


Sds,

Victor.

>
>
> ##### ##### #####
>
> Para saber mais visite
> http://www.ciencialist.hpg.ig.com.br
>
>
> ##### ##### ##### #####
>
>
> *Yahoo! Grupos, um serviço oferecido por:*
> PUBLICIDADE
> <http://br.rd.yahoo.com/SIG=12a5nbg60/M=264379.5078783.6203979.1588051/D=brclubs/S=2137111528:HM/EXP=1111013427/A=2332652/R=0/id=noscript/SIG=119058f8i/*http://br.download.yahoo.com/messenger/>
>
>
>
> ------------------------------------------------------------------------
> *Links do Yahoo! Grupos*
>
> * Para visitar o site do seu grupo na web, acesse:
> http://br.groups.yahoo.com/group/ciencialist/
>
> * Para sair deste grupo, envie um e-mail para:
> ciencialist-unsubscribe@yahoogrupos.com.br
> <mailto:ciencialist-unsubscribe@yahoogrupos.com.br?subject=Unsubscribe>
>
> * O uso que você faz do Yahoo! Grupos está sujeito aos Termos do
> Serviço do Yahoo! <http://br.yahoo.com/info/utos.html>.
>
>
>
>
> __________ Informação do NOD32 1.1026 (20050314) __________
>
> Esta mensagem foi verificada pelo NOD32 Sistema Antivírus
> http://www.nod32.com.br




SUBJECT: Re: Foucault / Luz ondas ou corpusculos.
FROM: Hélio Ricardo Carvalho <hrc@fis.puc-rio.br>
TO: ciencialist@yahoogrupos.com.br
DATE: 16/03/2005 22:20


--- Em ciencialist JVictor escreveu

>
>Achei muito boa a discussão. Suas questões realmente forçam
>a quem se digne responder a se esforçar, não dizer besteiras
>(pelo menos tentar não dizê-las..., que é o meu caso) e ir direto
> ao ponto. Acabei azeitando um pouco o juízo.
>Parabéns prá você.
>

Não sabia que já estava na hora de os debatedores fazerem suas
considerações finais. Que pena!

Farei também as minhas.
Aprendi muito com esta thread. Confesso que história da ciência não é
o meu forte. É uma pena que você, até ao final, não entendeu direito o
meu raciocínio. Mas tudo bem, num grupo de discussão como este, agente
corre este risco, mas as compensações valem muito a pena.
Parabéns para você também!


Hélio






SUBJECT: Lattes e o Nobel
FROM: Manuel Bulcão <manuelbulcao@uol.com.br>
TO: ciencialist@yahoogrupos.com.br
DATE: 17/03/2005 04:54


<<A Real Academia Sueca já incorreu em mais de um deslize na outorga
do Prêmio Nobel. Mas poucos tão infelizes quanto o cometido em
relação ao físico brasileiro Cesar Lattes. A omissão de seu nome na
premiação que celebrou a descoberta do méson pi até hoje espanta os
historiadores da ciência.
(...)
Lattes tinha apenas 22 anos quando comunicou, na edição de 25 de
maio de 1947 da revista Nature, a descoberta de Pic du Midi. E 23
quando divulgou, em outubro de 1948, os achados de Chacaltaya. A
façanha rendeu a Powell o Prêmio Nobel de Física de 1950. Lattes e
Occhialini, seus verdadeiros autores, ficaram a ver navios. "Powell
ganhou o Nobel por um trabalho assinado por Lattes, Occhialini e
Powell. Eu fiz a experimentação e as medidas. Ele apenas ajudou a
redigir, porque possuía maior domínio da língua inglesa. Mas o
médico brasileiro Carlos Chagas foi ainda mais injustiçado. Ele
merecia ter ganho não um, mas quatro nobéis!", consola-se hoje o
físico.>>

http://www2.uol.com.br/sciam/conteudo/noticia/noticia_104.html

[]s
Manuel Bulcão





SUBJECT: Re: [ciencialist] Fw: História da química analítica
FROM: "Alberto Mesquita Filho" <albmesq@uol.com.br>
TO: <ciencialist@yahoogrupos.com.br>
DATE: 17/03/2005 07:00

----- Original Message -----
From: "Alberto Mesquita Filho"
Sent: Wednesday, March 16, 2005 6:40 PM
Subject: Re: [ciencialist] Fw: História da química analítica

Mais alguns sites que talvez sejam úteis e a complementar a msg anterior
http://br.groups.yahoo.com/group/ciencialist/message/44950:

1) A História da Química. O terceiro tópico tem como título "O surgimento da
química analítica". (português)
http://www.profcupido.hpg.ig.com.br/a%20historia%20quimparteC.htm

2) Wikipedia. Apenas um ligeiro esboço do assunto. (português)
http://pt.wikipedia.org/wiki/Qu%C3%ADmica_anal%C3%ADtica

3) The History of Chemistry (inglês)
http://www.woodrow.org/teachers/ci/1992/

4) Gesellschaft Deutscher Chemiker (alemão)
http://www.gdch.de/gdch.htm

5) Textos clasicos de la historia de la quimica (espanhol)
http://www.uv.es/~bertomeu/material/clasico/index.html

6) El nascimiento de las especialidades de la quimica (espanhol)
http://www.uv.es/~bertomeu/material/museo/GUIA6.html

7) Orígenes históricos de la química (espanhol)
http://www.uv.es/~bertomeu/material/museo/INDEX.html

8) The Nobel Prize in Chemistry - Laureates
Artigos apresentados pelos laureados por ocasião da premiação.
Clique em qualquer dos nomes e a seguir em Nobel Lectures para ler o artigo
correspondente (geralmente em pdf e em inglês).
http://nobelprize.org/chemistry/laureates/index.html (inglês)

[ ]´s
Alberto
http://ecientificocultural.com/indice.htm
Mas indiferentemente a tudo isso, o neutrino tem massa, o elétron não é
uma carga elétrica coulombiana e a Terra se move. E a história se repetirá.



SUBJECT: Re: [ciencialist] Re: Foucault / Luz ondas ou corpusculos.
FROM: JVictor <jvoneto@uol.com.br>
TO: ciencialist@yahoogrupos.com.br
DATE: 17/03/2005 08:00

Alberto,

Outro dia entrei num site onde você disponibilizava fotos de encontros
com companheiros dos tempos de universidade.
Mostrei a minha sobrinha a sua foto(ela o conhece pois sempre lê seus
trabalhos na minha cx. de entrada). E disse: pôxa, é elegante e jovem,
ainda(êsse ainda é de lascar...). E não está de jogar fora!.. Piveta
atirada..
Bem, só para registrar que vimos a alegria estampada naquelas fotos.
Sobre essa thread, tenho pouco coisa a acrescentar. Trata-se de
informações baseadas em coisas escritas e levantadas por outros ou pelos
próprios personagens, com as consequentes interpretações que nós somos
tentados a dar, seja em razão do que a documentação examinada nos conduz
ou, o que não é raro, e até inconsciente, em razão de nossas
preferências e identificações pessoais, nem sempre claras para nós
mesmos, e por mais imparciais que queiramos ser. Digo isto em razão de
ter obersavdo frequentemente comportamentos assim em diversos autores,
segundo minhas percepções, que também podem ser traídas por alguma
parafuso frouxo ou um pano preto na frente. Quando a gente futuca muito,
faz comparações, percebe essas tendências. Que é humana. Que ninguém é
de ferro.
Quero lhe dizer que minha admiração por Newton é profunda. Sua
importância para a ciência, toda ela, inclusive para a linguagem da
natureza, coloca-o num pedestal que, para mim, chegar superar Einstein e
Poincaré. É difícil fazer comparações e também não sei se é apropriado.
Viveram em épocas diferentes, conceitos e costumes difrentes, concepções
das coisas em geral diferentes, sendo que os recursos intelectuais
disponíveis para Newton eram quase nenhuns. Em dado momento, as
concepções físicas de Newton careciam de um suporte matemático para que
as descrições pudessem ser expressas de maneira sólida e sistematizada.
Não tinha isso. Pois bem , ele criou um aparato matemático adequado!.
Chato, não é? Só isso, digo neste momento, o torna praticamente
insuperável.
Mas, cada um, com os recursos de que dispunham nas respectivas
épocas deram esse tamanho e maravilhoso colorido à ciência. Mas todos
todos acertaram e erraram, mas não sei se o contrário poderia ser
aventado, pois antes de chegaram às suas conclusões finais devem ter
ralado muito, varado noites a dentro, gastado um bocado de papel e
lápis. Pois apenas o produto final veio até nós.




>
> Alberto:> Os experimentos de Arago, Young, Foucault não deixavam
> dúvidas de que a
> > luz era um fenômeno que apresentava comportamento ondulatório, já com
> > justificativa matematica.
>
> Não é o que dizia o seu ídolo: "A história da busca de uma teoria da
> luz não
> está de modo algum concluída. O veredicto do século XIX não foi final e
> definitivo. Todo o problema de decidir entre corpúsculos e ondas ainda
> existe para a Física moderna, desta vez de uma forma muito mais profunda e
> intrincada. Aceitemos a derrota da teoria corpuscular da luz até
> reconhecermos a natureza problemática da vitória da teoria ondulatória."
> [Einstein e Infeld (A Evolução da Física)]


Victor: E ele o disse corretamente. Não estava concluída e ainda não não
se chegou a uma conclusão definitiva.
E desconfio que ainda estamos bastante longe disso. Einstein
apenasmente percebeu que o pensamento unilateral dos
pesquisadores, sejam os "ondulateiros" ou " corpusculeiros"(estou
concorrendo com Houaiss, por isso inventei essas palavras agora..) não
era de jeito nenhum definitivo, coisa que estes não percebiam, mesmo
ante as evidências de ambos os lados. Até nisto o seu pensamento foi
coerente.

>
>
> > Victor: Na FQ, por exemplo, com respeito ao hélio fluído, o
> comportamento
> > deste foge, como o diabo foge da cruz, da saia não tão justa desenhada
> > acima. É objeto macroscópico, porém quântico, que não sobe e desce
> quando
> > uma onda passa, como as ondas na água, mas ele, o próprio hélio ai,
> sim, é
> > a própria onda!..., sem nadinha daquele exemplo das tensões que dei
> acima.
> > É estranho, muito estranho, de doido ficar bom. Mas é o que a
> experiência
> > fornece, como resultado!. Explicações? Nihil. Mas isso é papo para o
> papa
> > de FQ, eterno apaixonado, nosso amigo Mesquita. Passo. Com ele a
> palavra.
>
> Alberto: Pô, mas a ficção quântica é tão simples, tão elementar, tão
> lógica!!! Não
> sei nem como começar a explicar uma coisa tão banal. É só introduzir umas
> duas ou três hipóteses "ad hoc" e tudo se esclarecerá. Pô! Será que você
> andou cabulando as aulas de ficção quântica no tempo de universidade?

Victor: Mas e quem disse que a quântica é simples, fácil e lógica? Se o
fiz alguma vez, foi em tom de ironia. O que eu acho é que
a FQ está correta, explica todos os fenônemos que a MC não consegue, e
são muitos( não há demérito algum para a MC, pois trata-se de domínios
de atuação), e está muito bem estruturada teoricamente. Suas estranhezas
existem sim. Mas, quer moleza, é?
E é inegável que basicamente todo o acréscimo de tecnologia e e avanço
cientifico dos últimos70/80 anos deve-se à FQ e suas descobertas.
Quer concordemos com ela ou não. Particulamente, do alto de minha
ignorância, minha percepção é que ela veio para ficar.

Abraços.

Victor.

>
> [ ]´s
> Alberto
> http://ecientificocultural.com/indice.htm
> Mas indiferentemente a tudo isso, o neutrino tem massa, o elétron não é
> uma carga elétrica coulombiana e a Terra se move. E a história se
> repetirá.
>
>
>
> ##### ##### #####
>
> Para saber mais visite
> http://www.ciencialist.hpg.ig.com.br
>
>
> ##### ##### ##### #####
>
>
> *Yahoo! Grupos, um serviço oferecido por:*
>
> *
> <http://br.rd.yahoo.com/SIG=12a4gvah7/M=264105.3931087.6562589.1588051/D=brclubs/S=2137111528:HM/EXP=1110851395/A=2361264/R=6/SIG=10v4acpp0/*http://br.shopping.yahoo.com/>*
>
>
>
> ------------------------------------------------------------------------
> *Links do Yahoo! Grupos*
>
> * Para visitar o site do seu grupo na web, acesse:
> http://br.groups.yahoo.com/group/ciencialist/
>
> * Para sair deste grupo, envie um e-mail para:
> ciencialist-unsubscribe@yahoogrupos.com.br
> <mailto:ciencialist-unsubscribe@yahoogrupos.com.br?subject=Unsubscribe>
>
> * O uso que você faz do Yahoo! Grupos está sujeito aos Termos do
> Serviço do Yahoo! <http://br.yahoo.com/info/utos.html>.
>
>
>
>
> __________ Informação do NOD32 1.1024 (20050311) __________
>
> Esta mensagem foi verificada pelo NOD32 Sistema Antivírus
> http://www.nod32.com.br




SUBJECT: Experiências macabras com células tronco.
FROM: "Geraldo M. Barcelos" <gemaba@veloxmail.com.br>
TO: <ciencialist@yahoogrupos.com.br>
DATE: 17/03/2005 08:19

Olá todos !!!

Se pegarmos uma célula tronco produzida com material genético, diriámos, de
um rato e a inserirmos em sua corrente
sanguínea, o que acontecerá?

Ela será considerada estranha ao organismo?

Ela se reproduzirá?

Se assim ocorrer qual o pontencial de duplicação dessa célula?

Ela se especificará nas diversas células do indivíduo?

Teoricamente o indivíduo, no caso o rato, teria um tempo mais de vida?

Se essa experiência fosse em humanos seria considerada aborto?

Poderíamos dizer que nossos filhos cresceriam dentro de nós?

Ps.: Alô Takata, cê sumiu !!!

Saudações !!!
gemaba
www.egmaba.cjb.net




SUBJECT: Re: [ciencialist] Experiências macabras com células tronco.
FROM: "Itabajara Vaz - UFRGS" <ita@cbiot.ufrgs.br>
TO: ciencialist@yahoogrupos.com.br
DATE: 17/03/2005 08:45

ola,
imagino que quando disse "produziada com material genetico" significa que eh uma
celula de rato.
sera reconhecida como estarnho e destruida e acabou a historia..
ita


> Se pegarmos uma célula tronco produzida com material genético
diriámos, de
> um rato e a inserirmos em sua corrente
> sanguínea, o que acontecerá?
>
> Ela será considerada estranha ao organismo?

>
> Ela se reproduzirá?
>
> Se assim ocorrer qual o pontencial de duplicação dessa célula?
>
> Ela se especificará nas diversas células do indivíduo?
>
> Teoricamente o indivíduo, no caso o rato, teria um tempo mais de vida?
>
> Se essa experiência fosse em humanos seria considerada aborto?
>
> Poderíamos dizer que nossos filhos cresceriam dentro de nós?
>
> Ps.: Alô Takata, cê sumiu !!!
>
> Saudações !!!
> gemaba
> www.egmaba.cjb.net
>
>
>
>
> ##### ##### #####
>
> Para saber mais visite
> http://www.ciencialist.hpg.ig.com.br
>
>
> ##### ##### ##### #####
> Links do Yahoo! Grupos
>
>
>
>
>
>
>




SUBJECT: Re: [ciencialist] Fw: QUÍMICA
FROM: "E m i l i a n o C h e m e l l o" <chemelloe@yahoo.com.br>
TO: <ciencialist@yahoogrupos.com.br>
DATE: 17/03/2005 10:16

Olá Léo!

Vou responder somente algumas... quer moleza? Coma sopa de minhoca! :-)


2 Se jogarmos Hidróxido de Amônio e Fenolftalina misturados em um tecido
branco ficará roseo, com o tempo desaparecerar a mancha roseo, pOr que?
Emiliano: Porque o hidróxido de amônio é muito volátil. Volatilizando, o
pano volta a sua cor original.

3 Que substancia se forma depois da desidratação da sacarose, usando acido
sufurico para desidrata-lá?
Emiliano: Carbono e água.

---
Quanto as questões 1 e 4, eu perguntaria, de volta, para fazer o consulente
refletir:

1 Por que quando misturamos mesma quantidade de Alcool Etilico e de Água, e
molhamos um pedaço de tecido nessa mistura e ateamos fogo no final quando o
fogo se apaga o tecido não queima?
Emiliano: Haverá alguma interação intermolecular entre o álcool e a água que
explique, de forma parcial, este fenômeno?

4) Usando iodo cristalizado e aquecendo-o até ele evaporar e colocando um
papel tocado por alguém acima da evaporação do mesmo, aparecerar a impresão
digital de quem o tocou. Por que?
Emiliano: Evapora? Não seria uma sublimação?

[ ] 's do Emiliano Chemello
emiliano@quimica.net
http://www.quimica.net/emiliano
http://www.ucs.br/ccet/defq/naeq
[ MSN ] chemelloe@hotmail.com
[ ICQ ] 145060604

" Rien ne se perd, rien ne se crée,
tout se transforme."

Antoine Laurent de Lavoisier (químico francês, 1743 - 1794)

----- Original Message -----
From: Luiz Ferraz Netto
To: ciencialist
Sent: Wednesday, March 16, 2005 7:15 PM
Subject: [ciencialist] Fw: QUÍMICA


Cuímicos ..... ?

[]'
===========================
Luiz Ferraz Netto [Léo]
leobarretos@uol.com.br
http://www.feiradeciencias.com.br
===========================
-----Mensagem Original-----
De: gapv
Para: leobarretos
Enviada em: quarta-feira, 16 de março de 2005 15:04
Assunto: QUÍMICA


Caro professor sou aluno universitário da UFPB (Universidade Federal da
Paraíba) do curso de ciencias agrárias, eutou com dificuldades de encontrar
em livros e até mesmo na internet as questões que lhe mando por meio deste
e-mail, peço por gentileza que me ajude com o seguinte:

1 Por que quando misturamos mesma quantidade de Alcool Etilico e de Água, e
molhamos um pedaço de tecido nessa mistura e ateamos fogo no final quando o
fogo se apaga o tecido não queima?

2 Se jogarmos Hidróxido de Amônio e Fenolftalina misturados em um tecido
branco ficará roseo, com o tempo desaparecerar a mancha roseo, pOr que?

3 Que substancia se forma depois da desidratação da sacarose, usando acido
sufurico para desidrata-lá?

4 Usando iodo cristalizado e aquecendo-o até ele evaporar e colocando um
papel tocado por alguém acima da evaporação do mesmo, aparecerar a impresão
digital de quem o tocou. Por que?


Desde Já agradeço
Gilberto Azevedo

João Câmara-RN.


----------------------------------------------------------------------------
----


Internal Virus Database is out-of-date.
Checked by AVG Anti-Virus.
Version: 7.0.300 / Virus Database: 266.5.0 - Release Date: 25/02/2005

----------

Internal Virus Database is out-of-date.
Checked by AVG Anti-Virus.
Version: 7.0.300 / Virus Database: 266.5.0 - Release Date: 25/02/2005


[As partes desta mensagem que não continham texto foram removidas]



##### ##### #####

Para saber mais visite
http://www.ciencialist.hpg.ig.com.br


##### ##### ##### #####


Yahoo! Grupos, um serviço oferecido por:








Links do Yahoo! Grupos

Para visitar o site do seu grupo na web, acesse:
http://br.groups.yahoo.com/group/ciencialist/

Para sair deste grupo, envie um e-mail para:
ciencialist-unsubscribe@yahoogrupos.com.br

O uso que você faz do Yahoo! Grupos está sujeito aos Termos do Serviço do
Yahoo!.




SUBJECT: off-topic/Dia Meteorológico Mundial!
FROM: "cumullosnimbos" <cumullosnimbos@bol.com.br>
TO: "ciencialist" <ciencialist@yahoogrupos.com.br>
DATE: 17/03/2005 11:04

Daniel Carlos de Menezes
==========
Divulgação

Em celebração ao dia meteorológico mundial que é comemorado no dia 23 de março, serão realizadas palestras e debates sobre os seguintes temas:

21/03-Mercado de cotas de carbono

Dr. Marcos Otávio Bezerra Prates - Ministério do Desenvolvimento, Indústria e Comércio Exterior
Carolina Burle Schmidt Dubeaux – COPPE/UFRJ
Dr. Carlos Nobre - INPE
Dr. Verdi Monteiro - BM&F
Prof. Luiz Francisco Pires Guimarães Maia - UFRJ (mediador)

22/03- A Transposição do Rio São Francisco

Dr. Rômulo de Macedo Vieira - Ministério da Integração Nacional
Dr. Marcelo Caus Asfora – Comitê da Bacia do Rio São Francisco
Dr. Arno Oliveira Maschmann – UFES
Dr. Luiz Carlos Baldicero Molion – UFAL
Dr. Otto Rotunno Correa Filho - UFRJ (mediador)


Horário: A partir das 14:00h
Endereço - CREA - Rua Buenos aires, 40, Centro - RJ.

__________________________________________________________________________
Acabe com aquelas janelinhas que pulam na sua tela.
AntiPop-up UOL - É grátis!
http://antipopup.uol.com.br/




[As partes desta mensagem que não continham texto foram removidas]



SUBJECT: Re: Experiências macabras com células tronco.
FROM: César A. K. Grossmann <cesarakg@bol.com.br>
TO: ciencialist@yahoogrupos.com.br
DATE: 17/03/2005 14:28


--- Em ciencialist@yahoogrupos.com.br, "Itabajara Vaz - UFRGS"
<ita@c...> escreveu
> ola,
> imagino que quando disse "produziada com material genetico"
significa que eh uma
> celula de rato.
> sera reconhecida como estarnho e destruida e acabou a historia..
>

Acho que não. Parece-me ter visto uma reportagem em que um homem
recebeu células-tronco suínas em uma reconstituição cerebral, para
lidar com uma doença degenerativa. As mesmas diferenciaram-se
produzindo neurônios e não foram rejeitadas. O diacho é que não lembro
dos detalhes da reportagem...

[]s
--
César A. K. Grossmann





SUBJECT: Re: Experiências macabras com células tronco.
FROM: César A. K. Grossmann <cesarakg@bol.com.br>
TO: ciencialist@yahoogrupos.com.br
DATE: 17/03/2005 14:31


--- Em ciencialist@yahoogrupos.com.br, César A. K. Grossmann
>
> Acho que não. Parece-me ter visto uma reportagem em que um homem
> recebeu células-tronco suínas em uma reconstituição cerebral, para
> lidar com uma doença degenerativa. As mesmas diferenciaram-se
> produzindo neurônios e não foram rejeitadas. O diacho é que não lembro
> dos detalhes da reportagem...

Único lugar em que eu encontrei alguma coisa em inglês sobre o assunto
(não fiz a persquisa em outras línguas):

http://www.quackwatch.org/01QuackeryRelatedTopics/Cancer/cellular.html

[]s
--
César A. K. Grossmann





SUBJECT: Fw: Colega de Física (Portugal)
FROM: "Luiz Ferraz Netto" <leobarretos@uol.com.br>
TO: "ciencialist" <ciencialist@yahoogrupos.com.br>
DATE: 17/03/2005 16:43

Imperdível .... International ... ou seja, Portugal! BLZ!

a todos,

aquele abraço,
===========================
Luiz Ferraz Netto [Léo]
leobarretos@uol.com.br
http://www.feiradeciencias.com.br
===========================
-----Mensagem Original-----
De: <carlos.saraiva1@sapo.pt>
Para: <leobarretos@uol.com.br>
Enviada em: quarta-feira, 16 de março de 2005 11:45
Assunto: Colega de Física (Portugal)


Caro colega
venho dar-lhe os parabéns pela sua "Feira de Ciências" da Net. Sou professor
de ciências Físico-químicas em Portugal e nos nossos livros escolares já
aparece o seu Site como referência o que significa que o seu trabalho é
apreciado em portugal.

cumprimentos,
carlos saraiva




Fale com MSN, ICQ, SAPO, Telepac e Netcabo, envie e receba SMS, bocas animadas e muito mais. Consulte http://messenger.sapo.pt para conhecer e instalar totalmente GRÁTIS o novo SAPO Messenger.


--
Internal Virus Database is out-of-date.
Checked by AVG Anti-Virus.
Version: 7.0.300 / Virus Database: 266.5.0 - Release Date: 25/02/2005




--
Internal Virus Database is out-of-date.
Checked by AVG Anti-Virus.
Version: 7.0.300 / Virus Database: 266.5.0 - Release Date: 25/02/2005



SUBJECT: Será que os animais pensam???
FROM: "neurophys" <jbrasil@unb.br>
TO: ciencialist@yahoogrupos.com.br
DATE: 17/03/2005 22:18


Será que os animais pensam?
Vejam a opinião do Dr. Renato Sabattini no site da Neurofisiologia e
Fisiologia Endócrina: http://neurofisiologia.cjb.net

Joaquim





SUBJECT: Re: [ciencialist] ninguem é inocente.
FROM: "Silvio" <scordeiro@terra.com.br>
TO: <ciencialist@yahoogrupos.com.br>
DATE: 17/03/2005 22:29

Rays:

Já foi aventada a possibilidade de se ter o DNA do Sr. J. Cristo no sangue
(...) do sudário?

Aposto que dá pra clonar o bicho....

sds.,

silvio.



SUBJECT: Algodão transgênico ameaça cerrado
FROM: "Anderson Porto" <acp722003@tudosobreplantas.com.br>
TO: <ciencialist@yahoogrupos.com.br>
DATE: 18/03/2005 01:20

Existe realmente alguma ameaça?

Anderson Porto
http://www.tudosobreplantas.com.br <http://www.tudosobreplantas.com.br/>




-----Mensagem original-----
De: David Hathaway [mailto:hathaway@infoporto.eti.br]
Enviada em: quinta-feira, 17 de março de 2005 20:54
Assunto: [TSP] Algodão transgênico ameaça cerrado


Greenpeace Brasil, 17-03-2005 - São Paulo (SP)
http://www.greenpeace.org.br/transgenicos/?conteudo_id=1961
<http://www.greenpeace.org.br/transgenicos/?conteudo_id=1961&sub_campanh
a=0> &sub_campanha=0

Cerrado está ameaçado por decisão ilegal e irresponsável da CTNBio

Comissão Técnica Nacional de Biossegurança (CTNBio) libera
apressadamente algodão transgênico, com gene de resistência a
antibiótico, sem estudos de efeitos sobre o meio ambiente e coloca em
risco biodiversidade do Cerrado

A CTNBio liberou hoje a variedade de algodão geneticamente modificado
Bollgard, da multinacional Monsanto, sem aguardar a sanção presidencial
à Lei de Biossegurança, que regulamenta o processo de liberação dos
transgênicos.

O algodão é uma planta de polinização cruzada, ou seja, o pólen pode
fecundar outras plantas distantes. A região do cerrado é centro de
origem do algodão e as variedades selvagens podem ser contaminadas com o
pólen de plantas transgênicas, gerando a perda das espécies nativas.

“Os atuais integrantes da CTNBio estão aproveitando os seus últimos dias
de poder para liberar apressadamente todos os pedidos que beneficiam as
grandes corporações de biotecnologia”, disse Ventura Barbeiro,
engenheiro agrônomo do Greenpeace. A aprovação antes da sanção
presidencial à Lei de Biossegurança é um ato ilegal e irresponsável (1).
A motivação da CTNBio em apressar a liberação é parte de uma estratégia
de garantir a liberação desse cultivar assim que o presidente sancione a
Lei (2). “Isso é um escândalo, um ato imoral, que a sociedade brasileira
não pode tolerar, principalmente a liberação de uma planta com dois
genes que podem conferir resistência a antibióticos”, completou
Barbeiro.

A Comissão Técnica de Biossegurança sofrerá grandes modificações com a
nova lei de biossegurança, passando de 18 para 27 integrantes, o que
ocasionará um tempo maior para a composição da nova equipe e
conseqüentemente impedindo liberações comerciais de alimentos
transgênicos para a próxima safra. Por isso, a pressa nessa aprovação. A
Lei de Biossegurança aguarda uma decisão presidencial sobre o veto de
dois polêmicos artigos que atribuem poder total à CTNBio sobre o tema
transgênicos e tornam facultativa a realização de estudos de impacto
ambiental e dos efeitos dos transgênicos sobre a saúde humana e animal.

A variedade liberada é uma planta inseticida. O algodão Bollgard também
é chamado de Bt por receber o gene Cry1Ac da bactéria Bacilus
thuringiensis, que codifica proteínas tóxicas, fazendo o papel de
agrotóxico. A planta também recebe dois genes da bactéria Escherichia
coli que confere resistência aos antibióticos espectinomicina e
estreptomicina. Esses genes, o nptII e o aad podem ser incorporados por
bactérias, transferindo a esses microorganismos resistência a
antibióticos. Um gene do vírus do mosaico da couve-flor também é
inserido nesse pacote.

Essa variedade de algodão inseticida produz proteínas tóxicas e pode
comprometer toda a cadeia ecológica do Cerrado. A flor do algodoeiro
atrai muitas abelhas e vespas selvagens devido à grande quantidade de
néctar e estes insetos podem desaparecer pelo efeito da proteína tóxica.
“No cerrado, 35% das plantas silvestres dependem de abelhas e vespas
para a polinização. O desaparecimento desses agentes polinizadores pode
causar a extinção de muitas plantas”, disse Barbeiro, “o algodão
transgênico inseticida é uma ameaça muito séria à nossa biodiversidade”,
completou.

Notas

(1) A liberação traduz-se em ato ilegal e arbitrário, que viola decisão
judicial em vigor. A decisão judicial vigente é aquela proferida nos
autos da Ação Civil Pública (processo no. 1998.34.00.027682-0), que
proibe a liberação de qualquer variedade de semente transgênica.
Permanecem preservadas as competências do Ministérios incumbidos da
aplicação da legislação vigente de exigir e conduzir prévio Estudo de
Impacto Ambiental - EIA/RIMA e licenciamento ambiental.

(2) A nova Lei de Biossegurança, pretende em seus artigos 30, 32 e 34
"convalidar" os atos ilegais e inconstitucionais praticados pela CTNBio
tornando permanente os registros provisórios concedido pela mesma. Com a
sanção presidencial da Lei, as decisões tomadas antes do dia da sanção
tornam-se definitivas.

Mais informações com Greenpeace:

- Ventura Barbeiro, campanha de engenharia genética, (11) 3035-1168,
(11) 8245-2248
- Gabriela Couto, campanha de engenharia genética, (11) 3035-1184, (11)
8245-2249
- Caroline Donatti, assessoria de imprensa, (11) 3035-1196, (11)
8272-6926

www.greenpeace.org.br <http://www.greenpeace.org.br/>




[As partes desta mensagem que não continham texto foram removidas]



SUBJECT: Re: [ciencialist] Re: Foucault / Luz ondas ou corpusculos.
FROM: JVictor <jvoneto@uol.com.br>
TO: ciencialist@yahoogrupos.com.br
DATE: 18/03/2005 02:16

Caro Hélio,

hbonestamente, esperei ter sido claro em muitas das minhas
considerações. Oportunamente, vou reler tudo e rever suas questões e
meus enfoques. Se achar mesmo que não entendí, voltarei a comentar. Não
pretendí encerrar o assunto, de qualquer maneira. Fiz apenas um
parêtensis devido à empolgação de momento.
Estarei à disposição, dentro de minhas disponibilidades de tempo.

Aproveito o ensejo para recomendar a todos a leitura do Scientific
American de março. Entre outros assuntos de nível, está o artigo Chips
de Átomos.

Sds,

Victor.



Hélio Ricardo Carvalho escreveu:

>
> --- Em ciencialist JVictor escreveu
>
> >
> >Achei muito boa a discussão. Suas questões realmente forçam
> >a quem se digne responder a se esforçar, não dizer besteiras
> >(pelo menos tentar não dizê-las..., que é o meu caso) e ir direto
> > ao ponto. Acabei azeitando um pouco o juízo.
> >Parabéns prá você.
> >
>
> Não sabia que já estava na hora de os debatedores fazerem suas
> considerações finais. Que pena!
>
> Farei também as minhas.
> Aprendi muito com esta thread. Confesso que história da ciência não é
> o meu forte. É uma pena que você, até ao final, não entendeu direito o
> meu raciocínio. Mas tudo bem, num grupo de discussão como este, agente
> corre este risco, mas as compensações valem muito a pena.
> Parabéns para você também!
>
>
> Hélio
>
>
>
>
>
>
> ##### ##### #####
>
> Para saber mais visite
> http://www.ciencialist.hpg.ig.com.br
>
>
> ##### ##### ##### #####
>
>
> *Yahoo! Grupos, um serviço oferecido por:*
> <http://br.rd.yahoo.com/SIG=12a8bebhi/M=264379.5078783.6203979.1588051/D=brclubs/S=2137111528:HM/EXP=1111108849/A=2191897/R=0/SIG=10vqa2grn/*http://br.diversao.yahoo.com/>
>
> <http://br.rd.yahoo.com/SIG=12a8bebhi/M=264379.5078783.6203979.1588051/D=brclubs/S=2137111528:HM/EXP=1111108849/A=2191897/R=1/SIG=10vqa2grn/*http://br.diversao.yahoo.com/>
>
>
>
> ------------------------------------------------------------------------
> *Links do Yahoo! Grupos*
>
> * Para visitar o site do seu grupo na web, acesse:
> http://br.groups.yahoo.com/group/ciencialist/
>
> * Para sair deste grupo, envie um e-mail para:
> ciencialist-unsubscribe@yahoogrupos.com.br
> <mailto:ciencialist-unsubscribe@yahoogrupos.com.br?subject=Unsubscribe>
>
> * O uso que você faz do Yahoo! Grupos está sujeito aos Termos do
> Serviço do Yahoo! <http://br.yahoo.com/info/utos.html>.
>
>
>
>
> __________ Informação do NOD32 1.1027 (20050316) __________
>
> Esta mensagem foi verificada pelo NOD32 Sistema Antivírus
> http://www.nod32.com.br




SUBJECT: Re: off-topic: Minha idéia vai ser desenvolvida + dúvida sobre direitos autorais
FROM: "rmtakata" <rmtakata@altavista.net>
TO: ciencialist@yahoogrupos.com.br
DATE: 18/03/2005 02:36


--- Em ciencialist@yahoogrupos.com.br, "Eurico Ferreira de
> PERGUNTO: alguém tem alguma dica de como vou receber
> meus direitos autorais? como se calcula? é um índice fixo
> (tipo imposto) ou tenho que pleitear e negociar para receber?

Tem q. negociar.

A media para livros fica em torno de 5 a 10% do preco de capa -
autores de grande vendagem podem conseguir ateh 20%.

No caso de games talvez o pessoal do curso de design de games da
Anhembi Morumbi possa dar as informacoes de como funciona.

[]s,

Roberto Takata





SUBJECT: César Lattes
FROM: JVictor <jvoneto@uol.com.br>
TO: "ciencialist@yahoogrupos.com.br" <ciencialist@yahoogrupos.com.br>
DATE: 18/03/2005 07:09



Lista,


Sobre César Lattes,



*O brasileiro que o Nobel esqueceu*
/Março 2005/

A aventura de Cesar Lattes, que revolucionou nossa visão do mundo
subatômico.

por José Tadeu Arantes

A Real Academia Sueca já incorreu em mais de um deslize na outorga do
Prêmio Nobel. Mas poucos tão infelizes quanto o cometido em relação ao
físico brasileiro Cesar Lattes. A omissão de seu nome na premiação que
celebrou a descoberta do méson pi até hoje espanta os historiadores da
ciência. Lattes é um dos grandes responsáveis pelo desbravamento do
mundo subatômico, demonstrando experimentalmente que a matéria não se
resume a simples arranjos de prótons, elétrons e nêutrons. Neto e filho
de banqueiros italianos, Cesare Mansueto Giulio Lattes nasceu em
Curitiba, Paraná, em 11 de julho de 1924. Iniciou o curso primário em
Porto Alegre, Rio Grande do Sul, e concluiu o secundário no Colégio
Dante Alighieri, em São Paulo. Aos 16 anos, pelas mãos do pai,
encontrou-se com o físico ucraniano (naturalizado italiano) Gleb
Wataghin, introdutor da física moderna no Brasil. Wataghin o aconselhou
a aproveitar uma portaria governamental, pular os anos que faltavam e
ingressar imediatamente na então Faculdade de Filosofia, Ciências e
Letras da Universidade de São Paulo (USP). Com 19 anos, idade na qual
muitos estudantes ainda estão se preparando para entrar na Universidade,
Lattes ganhava seu primeiro salário como professor-assistente da USP.
"Nunca mais fiz curso algum.

Daí para frente, o que eu aprendi, aprendi fazendo", orgulha-se.
Marcello Damy de Souza Santos, que foi seu professor de física geral e
experimental, registrou a notável intuição de Lattes para os fenômenos
físicos e sua habilidade como experimentador. Tanta que, ainda aluno,
Wataghin e o italiano Giuseppe Occhialini o convidaram a participar de
pesquisas teóricas e experimentais. "Meu primeiro trabalho foi com
Wataghin", recorda-se. "Construímos o modelo teórico de um gás com
temperatura da ordem de trilhões de graus e pressão 10 milhões de vezes
maior que a da água. Nesse meio hipotético, semelhante ao que deve ter
existido numa fase inicial da vida do universo, verificamos que a
abundância na formação de núcleos atômicos correspondia à seqüência real
dos elementos situados entre o carbono e o argônio na tabela periódica.
Supõe-se que os elementos mais leves tenham sido formados antes, em
pressões e temperaturas ainda maiores, e os elementos mais pesados, depois."

O trabalho seguinte, Lattes realizou sob a orientação do grande físico
brasileiro Mário Schenberg. "Era um modelo teórico do elétron, supondo
uma partícula puntiforme dotada de momento angular", relata. "Uma
'calculeira' infernal! A equação do movimento tinha várias dezenas de
termos. Depois que consegui acabar, nunca mais me dediquei à física
teórica. Decidi virar experimentador." Em 1946, apenas três anos depois
de formado, o jovem partia para a Inglaterra, unindo-se ao grupo de
pesquisadores liderados por Cecil Powell, na Universidade de Bristol.
Tinha uma bolsa paga pela companhia de cigarros Wills: "foi aí que
comecei a fumar, aos 22 anos", diz o físico, até hoje um tabagista
inveterado.

O inglês vinha estudando os traços produzidos por partículas subatômicas
em certas chapas especiais, espessas e sensíveis, chamadas "emulsões
fotográficas". Pela análise dos rastros deixados por essas partículas
era possível determinar sua massa, energia e outras propriedades
físicas. O trabalho se arrastava há 10 anos, sem resultados importantes.
A entrada de Occhialini e Lattes reoxigenou o grupo. Occhialini tratou
de conseguir emulsões mais densas, capazes de registrar maior número de
eventos. Logo que chegou, Lattes foi encarregado de calibrar as novas
emulsões e descobriu que, quando carregadas com um composto de boro, o
bórax, elas eram capazes de reter o traço das partículas por muito mais
tempo.

O jovem não se conformava com as baixas energias das partículas então
produzidas em laboratório e esperava utilizar as chapas para detectar as
partículas muito mais energéticas presentes nos raios cósmicos, que
permanentemente chegam à Terra, vindos do espaço exterior.

Aproveitando uma viagem de férias de Occhialini ao Pic du Midi, nos
Pirineus, convenceu-o a levar dois kits de chapas, um com bórax, outro
sem, e expô-los ao intenso bombardeio de raios cósmicos existente
naquela altitude. Quando revelaram as chapas, os pesquisadores ficaram
boquiabertos com a quantidade de fenômenos registrados nas emulsões
carregadas com bórax. Um evento os impressionou em especial. Era o traço
de um méson, que diminuía a velocidade e parava, dando origem a um novo
traço.

A existência do méson fora proposta teoricamente pelo japonês Hideki
Yukawa, em 1935, como forma de explicar a força que mantinha o núcleo
atômico coeso. Essa partícula hipotética, com massa aproximadamente 200
vezes maior do que a do elétron, seria incessantemente emitida e
absorvida por prótons e nêutrons. E a troca de mésons entre os
constituintes do núcleo atômico produziria, entre eles, uma atração de
curto alcance, responsável pela estabilidade nuclear. Fora do núcleo
atômico, os mésons sobreviveriam por apenas um bilionésimo de segundo,
desintegrando-se em seguida.

"Em 1937-38, Carl Anderson e Seth Neddermeyer encontraram nos raios
cósmicos os sinais de algo que parecia ser o méson de Yukawa", informa o
físico e historiador da ciência Roberto Martins, do Departamento de
Raios Cósmicos do Instituto de Física da Universidade Estadual de
Campinas (Unicamp-SP). "Verificou-se, porém, que essa partícula não
tinha o comportamento previsto. Para poderem explicar as forças
nucleares, os mésons deviam ser fortemente absorvidos por prótons e
nêutrons e, portanto, capturados com facilidade pela matéria. Mas os
mésons de Anderson e Neddermeyer podiam atravessar centenas de núcleos
atômicos sem sofrer nenhuma alteração".

A descoberta de Lattes e Occhialini poderia solucionar o enigma se o
estranho traço na emulsão resultasse da transformação de um méson em
outro - a parte inicial da trajetória correspondendo ao méson de Yukawa
e a parte seguinte ao méson de Anderson e Neddermeyer. O primeiro, mais
pesado, seria responsável pela força nuclear; porém, fora do núcleo,
logo se desintegraria, produzindo o segundo, que atravessava a matéria
quase sem interagir.

Lattes estava convencido disso. Mas os dados obtidos em Pic du Midi, a
2800 metros acima do nível do mar, eram insuficientes para corroborar
essa interpretação. O pesquisador precisava de raios cósmicos ainda mais
abundantes e energéticos. E descobriu um lugar onde poderia
encontrá-los: uma precária estação andina, localizada em Chacaltaya,
Bolívia, a 5500 metros de altitude. "Lá, o número de partículas cósmicas
é 100 mil vezes maior", explica.

Com pequeno suporte financeiro do laboratório de Bristol e escapando por
pouco de um infortunado vôo da British Airways, que caiu, Lattes voltou
ao Brasil e, daqui, dirigiu-se à Bolívia. Num ar rarefeito, com metade
da pressão atmosférica existente ao nível do mar, ele realizou o
experimento que o imortalizou. As emulsões expostas em Chacaltaya
revelaram cerca de 30 rastros de mésons duplos. Feitos os cálculos,
confirmou-se a existência de dois tipos de partículas com massas
diferentes: o corpúsculo secundário correspondia àquele de Anderson e
Neddermeyer e foi batizado como méson mi (ou míon); o corpúsculo
primário, 30 a 40% mais pesado, era algo novo e recebeu o nome de méson
pi (ou píon).

Estudos posteriores mostraram que ele tinha uma forte interação com o
núcleo atômico, possuindo todas as características exigidas pela teoria
de Yukawa.

Lattes tinha apenas 22 anos quando comunicou, na edição de 25 de maio de
1947 da revista Nature, a descoberta de Pic du Midi. E 23 quando
divulgou, em outubro de 1948, os achados de Chacaltaya. A façanha rendeu
a Powell o Prêmio Nobel de Física de 1950. Lattes e Occhialini, seus
verdadeiros autores, ficaram a ver navios. "Powell ganhou o Nobel por um
trabalho assinado por Lattes, Occhialini e Powell. Eu fiz a
experimentação e as medidas. Ele apenas ajudou a redigir, porque possuía
maior domínio da língua inglesa. Mas o médico brasileiro Carlos Chagas
foi ainda mais injustiçado. Ele merecia ter ganho não um, mas quatro
nobéis!", consola-se hoje o físico. Na época, porém, no auge de sua
produtividade científica, ele só estava interessado em tocar as
pesquisas para frente. Quando soube que havia sido construído um
poderoso acelerador cíclotron em Berkeley, na Califórnia, decidiu deixar
Bristol e ir para lá. Não sem antes passar pelo Brasil, onde se casou
com sua inseparável companheira, Martha Siqueira Neto Lattes, com quem
tem quatro filhas e nove netos.

Em apenas duas semanas de Califórnia, conseguiu produzir mésons pi em
laboratório. Poderia ter feito uma brilhante e muito bem-remunerada
carreira nos EUA. Preferiu voltar ao Brasil para dar sua contribuição ao
desenvolvimento científico do país. Graças a seus esforços, surgiram o
Centro Brasileiro de Pesquisas Físicas (CBPF) e o Conselho Nacional de
Pesquisas (CNPq). Mais tarde, teve também importante participação na
implantação da Universidade Estadual de Campinas (Unicamp-SP), uma das
mais importantes instituições de ensino e pesquisa do Brasil.

As difíceis condições de trabalho num país subdesenvolvido e as severas
crises de depressão que passaram a açoitá-lo desde 1955 não o afastaram
da física. Sua descoberta do méson pi abriu uma página nova no
conhecimento da natureza. Pode-se dizer que o atual modelo padrão de
partículas, baseado na teoria dos quarks, é seu herdeiro direto. Mas,
para um experimentalista empedernido como Lattes, trata-se de um
herdeiro indesejado: "Não acredito no modelo padrão", diz. "Ele é
completamente ad hoc. O pessoal está descobrindo o que prevê. O
importante é descobrir o que não está previsto."

Cesar Lattes: descobridor do méson pi

 Nascido em Curitiba, Paraná, em 1924; casado com Martha Siqueira Neto
Lattes, pai de quatro filhas, avô de nove netos.
 Sua descoberta, realizada aos 22 anos, abriu caminho para a nova
física de partículas, mostrando que há muito mais coisas nas entranhas
do átomo do que prótons, elétrons e nêutrons.
 "Não gosto dos quarks. Eles jamais foram detectados. São pura
idealização. A gente aprende o que é natureza experimentando, não
idealizando"

(Matéria publicada em junho de 2002)


Sds,

Victor.



SUBJECT: Re: [ciencialist] Novo no grupo
FROM: "Luiz Ferraz Netto" <leobarretos@uol.com.br>
TO: <ciencialist@yahoogrupos.com.br>
DATE: 18/03/2005 07:32

P.Sávio escreve:

#Apresentando-me:
Mestrando de Física
Tive acesso, através de um amigo, a algumas mensagens envolvendo matemática avançada, física, história da física, questionamentos, novas propostas outros assuntos e gostei. Achei o grupo de alto nível. Espero poder aprender aqquí enriquecer minha tese.#


Essa é uma boa hipótese!

Seja bem vindo
===========================
Luiz Ferraz Netto [Léo]
leobarretos@uol.com.br
http://www.feiradeciencias.com.br
===========================



--
Internal Virus Database is out-of-date.
Checked by AVG Anti-Virus.
Version: 7.0.300 / Virus Database: 266.5.0 - Release Date: 25/02/2005



SUBJECT: Re: [ciencialist] Será que os animais pensam???
FROM: "Luiz Ferraz Netto" <leobarretos@uol.com.br>
TO: <ciencialist@yahoogrupos.com.br>
DATE: 18/03/2005 07:45

Eu não tenho a menor dúvida de que os animais pensam.
Pelo menos é o que penso!
[]'
===========================
Luiz Ferraz Netto [Léo]
leobarretos@uol.com.br
http://www.feiradeciencias.com.br
===========================
Joaquim

#Será que os animais pensam?
Vejam a opinião do Dr. Renato Sabattini no site da Neurofisiologia e
Fisiologia Endócrina: http://neurofisiologia.cjb.net #




--
Internal Virus Database is out-of-date.
Checked by AVG Anti-Virus.
Version: 7.0.300 / Virus Database: 266.5.0 - Release Date: 25/02/2005



SUBJECT: Re: [ciencialist] ninguem é inocente.
FROM: TARCISIO BORGES <tbs97@fisica.ufpr.br>
TO: ciencialist@yahoogrupos.com.br
DATE: 18/03/2005 08:53

Já foi sim. Mas a possibilidade de achar sangue lá, de quem quer que seja,
é pequena.

Mesmo porque o sudário data de +/- 1300 DC.

[]s
TARCISIO BORGES
tbs97@fisica.ufpr.br

On Thu, 17 Mar 2005, Silvio wrote:
> Já foi aventada a possibilidade de se ter o DNA do Sr. J. Cristo no sangue
> (...) do sudário?
>
> Aposto que dá pra clonar o bicho....
> silvio.



SUBJECT: Re: ninguem é inocente.
FROM: "pubmed2005" <pubmed2005@yahoo.com.br>
TO: ciencialist@yahoogrupos.com.br
DATE: 18/03/2005 11:57


De acordo com nova pesquisa o Sudário tem aproximadamente 3.000 anos,
isto é, podendo ser anterior a Cristo

--- Em ciencialist@yahoogrupos.com.br, TARCISIO BORGES <tbs97@f...>
escreveu
> Já foi sim. Mas a possibilidade de achar sangue lá, de quem quer
que seja,
> é pequena.
>
> Mesmo porque o sudário data de +/- 1300 DC.
>
> []s
> TARCISIO BORGES
> tbs97@f...






SUBJECT: Re: [ciencialist] Será que os animais pensam???
FROM: Aline Santos <haline_santos@yahoo.com.br>
TO: ciencialist@yahoogrupos.com.br
DATE: 18/03/2005 12:20

Aninmais!!!! Todos somos um, porém cada qual pensa da sua maneira, certo???!!!!!


Luiz Ferraz Netto <leobarretos@uol.com.br> wrote: Eu não tenho a menor dúvida de que os animais pensam.
Pelo menos é o que penso!
[]'
===========================
Luiz Ferraz Netto [Léo]
leobarretos@uol.com.br
http://www.feiradeciencias.com.br
===========================
Joaquim

#Será que os animais pensam?
Vejam a opinião do Dr. Renato Sabattini no site da Neurofisiologia e
Fisiologia Endócrina: http://neurofisiologia.cjb.net #




--
Internal Virus Database is out-of-date.
Checked by AVG Anti-Virus.
Version: 7.0.300 / Virus Database: 266.5.0 - Release Date: 25/02/2005



##### ##### #####

Para saber mais visite
http://www.ciencialist.hpg.ig.com.br


##### ##### ##### #####


Yahoo! Grupos, um serviço oferecido por:

















function SearchComboBox() { if (document.form_combo.keyword.value.length==0){ alert("Por favor, digite algo."); return false; }else { document.form_combo.action ="http://br.rd.yahoo.com/SIG=12al7s3cp/M=264105.3931087.6562589.1588051/D=brclubs/S=2137111528:HM/EXP=1111229323/A=2361264/R=0/SIG=11uaou2jn/*http://www.bondfaro.com/bondfaro/in/combosearch_in.jsp?sk=11"; } return true;} [input] [input] [input]

---------------------------------
Links do Yahoo! Grupos

Para visitar o site do seu grupo na web, acesse:
http://br.groups.yahoo.com/group/ciencialist/

Para sair deste grupo, envie um e-mail para:
ciencialist-unsubscribe@yahoogrupos.com.br

O uso que você faz do Yahoo! Grupos está sujeito aos Termos do Serviço do Yahoo!.




---------------------------------
Yahoo! Mail - Com 250MB de espaço. Abra sua conta!

[As partes desta mensagem que não continham texto foram removidas]



SUBJECT: Re: [ciencialist] Livre-docência
FROM: "Alvaro Augusto \(E\)" <alvaro@electraenergy.com.br>
TO: <ciencialist@yahoogrupos.com.br>
DATE: 18/03/2005 12:27

Caro Emiliano,

Os livres-docentes são os Grandes Mestres Jedy do mundo acadêmico. Eles são capazes de controlar os elementos, andar sobre as águas, multiplicar os pães, rescussitar os mortos, etc. Após fazer mestrado, doutorado, pós-doutorado, ter vários anos de profissão e publicar vários trabalhos releventes, o candidato pode fazer concurso para livre-docência, quando houver vagas. O concurso envolve, além de prova escrita, a preparação de um resumo biográfico (memorial) e a defesa de uma tese em frente a banca examinadora. Algumas faculdades também submetem o memorial, que deve comprovar a obra do pesquisador, à argüição pública. É mole?

[ ]s

Alvaro Augusto
----- Original Message -----
From: E m i l i a n o C h e m e l l o
To: ciencialist@yahoogrupos.com.br
Sent: Wednesday, March 16, 2005 1:22 PM
Subject: [ciencialist] Livre-docência


Caros amigos,

Na análise de alguns currículos (Lattes), me deparei com a seguinte
formação: "Livre-docência". O que isto significa exatamente? Como se
consegue?

[ ] 's do Emiliano Chemello
emiliano@quimica.net
http://www.quimica.net/emiliano
http://www.ucs.br/ccet/defq/naeq
[ MSN ] chemelloe@hotmail.com
[ ICQ ] 145060604

" Rien ne se perd, rien ne se crée,
tout se transforme."

Antoine Laurent de Lavoisier (químico francês, 1743 - 1794)




##### ##### #####

Para saber mais visite
http://www.ciencialist.hpg.ig.com.br


##### ##### ##### #####


Yahoo! Grupos, um serviço oferecido por:







------------------------------------------------------------------------------
Links do Yahoo! Grupos

a.. Para visitar o site do seu grupo na web, acesse:
http://br.groups.yahoo.com/group/ciencialist/

b.. Para sair deste grupo, envie um e-mail para:
ciencialist-unsubscribe@yahoogrupos.com.br

c.. O uso que você faz do Yahoo! Grupos está sujeito aos Termos do Serviço do Yahoo!.



[As partes desta mensagem que não continham texto foram removidas]



SUBJECT: 13 Coisas Que Não Fazem Sentido
FROM: César A. K. Grossmann <cesarakg@bol.com.br>
TO: ciencialist@yahoogrupos.com.br
DATE: 18/03/2005 14:01


http://www.newscientist.com/channel/space/mg18524911.600

O artigo (em inglês) tem uma breve descrição de cada um, só vou citar
os mesmos.

1 The placebo effect
2 The horizon problem
3 Ultra-energetic cosmic rays
4 Belfast homeopathy results
5 Dark matter
6 Viking's methane
7 Tetraneutrons
8 The Pioneer anomaly
9 Dark energy
10 The Kuiper cliff
11 The Wow signal
12 Not-so-constant constants
13 Cold fusion

E, para não dizer que não falei de flores, vai o link para os
"Millenium Problems", de matemática (também na língua de Shakespeare):

http://www.claymath.org/millennium/

[]s
--
César A. K. Grossmann





SUBJECT: Re: [ciencialist] Livre-docência
FROM: "Alberto Mesquita Filho" <albmesq@uol.com.br>
TO: <ciencialist@yahoogrupos.com.br>
DATE: 18/03/2005 14:01

----- Original Message -----
From: "Alvaro Augusto (E)"
Sent: Friday, March 18, 2005 12:27 PM
Subject: Re: [ciencialist] Livre-docência

> Os livres-docentes são os Grandes Mestres Jedy do mundo acadêmico. Eles
> são capazes de controlar os elementos, andar sobre as águas, multiplicar
> os pães, rescussitar os mortos, etc. Após fazer mestrado, doutorado,
> pós-doutorado, ter vários anos de profissão e publicar vários trabalhos
> releventes, o candidato pode fazer concurso para livre-docência, quando
> houver vagas. O concurso envolve, além de prova escrita, a preparação de
> um resumo biográfico (memorial) e a defesa de uma tese em frente a banca
> examinadora. Algumas faculdades também submetem o memorial, que deve
> comprovar a obra do pesquisador, à argüição pública. É mole?

No meu tempo [Putz, acho que estou ficando velho! :-)] o candidato a
livre-docente devia também demonstrar a sua didática através de uma aula de
uma hora de duração e sobre um tema sorteado com 24 horas de antecedência.
Mas isso foi em meados do século passado, não sei se ainda está em voga.

[ ]´s
Alberto
http://ecientificocultural.com/indice.htm
Mas indiferentemente a tudo isso, o neutrino tem massa, o elétron não é
uma carga elétrica coulombiana e a Terra se move. E a história se repetirá.



SUBJECT: Re: [ciencialist] Re: ninguem é inocente.
FROM: TARCISIO BORGES <tbs97@fisica.ufpr.br>
TO: ciencialist@yahoogrupos.com.br
DATE: 18/03/2005 14:11

Mmmm (pensamento bovino)... que interessante.

Poderia me indicar a fonte desta informação?

[]s
TARCISIO BORGES
tbs97@fisica.ufpr.br

On Fri, 18 Mar 2005, pubmed2005 wrote:

> De acordo com nova pesquisa o Sudário tem aproximadamente 3.000 anos,
> isto é, podendo ser anterior a Cristo



SUBJECT: Re: [ciencialist] Novo no grupo
FROM: "Alberto Mesquita Filho" <albmesq@uol.com.br>
TO: <ciencialist@yahoogrupos.com.br>
DATE: 18/03/2005 14:24

----- Original Message -----
From: "PSavio"
Sent: Saturday, March 19, 2005 6:31 AM
Subject: [ciencialist] Novo no grupo

> Apresentando-me:

Bem-vindo à Ciencialist, o mundo do Léo.

> Mestrando de Física

Sem problemas. Ser físico não chega a ser um grande defeito. ;-)

> Espero poder aprender aqquí enriquecer minha tese.

Como disse o Léo, não deixa de ser uma boa hipótese, mas cuidado com os
doidos e com os defensores do escolasticismo, principalmente se a sua tese
for "moderna".

[ ]´s
Alberto
Nobre e Excelso Cavaleiro do Apocalipse Quântico
http://ecientificocultural.com/indice.htm
Mas indiferentemente a tudo isso, o neutrino tem massa, o elétron não é
uma carga elétrica coulombiana e a Terra se move. E a história se repetirá.



SUBJECT: Esverdeando o Currículo de Química
FROM: "Esteban" <estebanmoreno@idhi.org.br>
TO: "Vivens Scientia" <VivensScientia@yahoogroups.com>, <ciencialist@yahoogrupos.com.br>
DATE: 18/03/2005 14:31

Esverdeando o Currículo de Química
**********************************************************************

Já está no ar a tradução para o português de "Greening Across the
Chemistry Curriculum"
(http://academic.scranton.edu/faculty/CANNM1/dreyfusmodulesport.html),
site desenvolvido por professores da Universidade de Scranton (EUA),
sob coordenação do Prof. Michael Cann.

Os tradutores esperam que este material educacional em português
contribua para a introdução da química verde nas diferentes
disciplinas lecionadas nos cursos de graduação em química do país.

Além de uma introdução à química verde, o site apresenta nove módulos
abordando a química verde em disciplinas como química geral, química
inorgânica, bioquímica, química orgânica, química de polímeros,
toxicologia, etc. O acesso é livre e gratuito. A tradução foi feita
pelo grupo de síntese orgânica da Universidade Federal de Pelotas,
que também mantém a WWVerde, a página de divulgação da química verde
no Brasil (http://iqg.ufpel.edu.br/wwverde).




[As partes desta mensagem que não continham texto foram removidas]



SUBJECT: Re: ninguem é inocente.
FROM: "pubmed2005" <pubmed2005@yahoo.com.br>
TO: ciencialist@yahoogrupos.com.br
DATE: 18/03/2005 15:01


Procura no Google que voce encontra rapidinho


--- Em ciencialist@yahoogrupos.com.br, TARCISIO BORGES <tbs97@f...>
escreveu
> Mmmm (pensamento bovino)... que interessante.
>
> Poderia me indicar a fonte desta informação?
>
> []s
> TARCISIO BORGES
> tbs97@f...
>
> On Fri, 18 Mar 2005, pubmed2005 wrote:
>
> > De acordo com nova pesquisa o Sudário tem aproximadamente 3.000
anos,
> > isto é, podendo ser anterior a Cristo





SUBJECT: Uma bola de fogo, um ET?
FROM: Rodrigo Marques <rodmarq72@yahoo.com.br>
TO: Ceticismo Aberto <ceticismoaberto@yahoogrupos.com.br>, CienciaList <ciencialist@yahoogrupos.com.br>, Perry Rhodan Brasil <perry-rhodan-brazil@yahoogrupos.com.br>, Sociedade Brasileira de "C�ticos" e Racionalistas <sbcr@yahoogrupos.com.br>, Sociedade da Terra redonda <strbrasil@yahoogrupos.com.br>
DATE: 18/03/2005 15:18

http://odia.ig.com.br/brasil/br180302.htm
Uma bola de fogo, um ET?

Moradores de Aracruz levam ‘alien’ para hospital e descobrem que objeto era um boneco

‘Moradores do bairro Limão entraram em contato com a emissora comunicando que após a queda de uma bola de fogo, um objeto não identificado foi encontrado no bairro". A notícia era transmitida por uma rádio do município de Aracruz, interior do Espírito Santo, enquanto moradores e policiais do 5º BPM chegavam ao hospital municipal São Camilo com partes de um suposto extraterrestre dentro de uma caixa de sapatos. O material chegou a ser submetido a exames a pedido da população que acreditava se tratar de um ET. O exame do material no microscópio constatou: um boneco, feito de borracha e material esponjoso.

A história começou às 18h de terça-feira quando a dona de casa Adriana Silva Francisco, 34 anos, encontrou em seu quintal, no bairro Vila Nova, um objeto, para ela, não identificado, após ter visto uma bola de fogo no céu. Segundo o policial Balan, Adriana estava assustada e chamou os vizinhos achando que fosse um animal ou até mesmo uma criança deformada. "Nunca vi algo parecido", disse. A polícia recebeu mais 50 ligações de pessoas que perguntavam como se defender de uma invasão alienígena. ‘ET’ foi espancado por moradores com madeira

Segundo moradores, antes de ser levado para o hospital, o suposto ET "se mexeu", o que levou as pessoas a surrarem o objeto com paus e pontapés. A gerente do hospital, Bernardete Braz, explicou que a princípio os médicos ficaram preocupados porque já receberam fetos em caixas de sapato. "Foi uma brincadeira de mau-gosto", avaliou. Muitos não quiseram sair do hospital antes do "diagnóstico médico", que detectou apenas borracha e coliformes fecais.

O mistério só foi finalmente concluído quando constataram se tratar de um brinquedo, vendido na cidade vizinha, Ibiraçu. O motorista, Marcelo Sfalsin, não se convenceu e levou uma perna de recordação.

Para a polícia, uma brincadeira

O município de Aracruz tem 70 mil habitantes e fica a 50 km da capital Vitória. Segundo o gerente da rádio Nova Onda, Evilásio Oliveira Costa, a central telefônica da estação ficou sobrecarregada. "Diziam que estávamos escondendo informação. Quando informamos que o suposto alien foi levado para o hospital, um ouvinte disse que era mentira e que a NASA (agência espacial americana) o tinha levado", relatou. Ele explicou que os repórteres não identificaram o brinquedo porque já estava em pedaços.

A polícia considerou o episódio como "um mal-entendido" e uma "brincadeira", por isso não foi registrada ocorrência. Como o número de pessoas que afirmaram ter visto um extraterrestre foi grande, os policiais não conseguiram identificar os responsáveis.

Boneco vendido em lojas do Rio

O que é motivo de preocupação em Limão, nas lojas do Saara é pura chacota. A universitária Renata Emille comprou ontem na Carla Brinquedos, no Centro, o boneco ET igual ao encontrado em Aracruz para enganar os amigos. "Quando meus amigos estiverem bêbados, vou colocar o ET na mesa e assustá-los", brinca.

O brinquedo mede 5 centímetros, quando submerso na água ele pode chegar a 20. "Se deixar fora d’água ele volta a ficar pequeno", explica , dono da loja, Felippe Kyrillos.



---------------------------------
Yahoo! Mail - Com 250MB de espaço. Abra sua conta!

[As partes desta mensagem que não continham texto foram removidas]



SUBJECT: Re: [ciencialist] Uma bola de fogo, um ET?
FROM: "Ivan Carlos" <icarlos@icarlos.net>
TO: <ciencialist@yahoogrupos.com.br>, "Ceticismo Aberto" <ceticismoaberto@yahoogrupos.com.br>, "Perry Rhodan Brasil" <perry-rhodan-brazil@yahoogrupos.com.br>, Sociedade Brasileira de Céticos e Racionalistas <sbcr@yahoogrupos.com.br>, "Sociedade da Terra redonda" <strbrasil@yahoogrupos.com.br>
DATE: 18/03/2005 15:26

HUAHUAHUAHUAHUAHUAHUAH q viagem..!!!!

não é a toa q por qqer coisa meio mundo coloca a culpa em "alguém"
suportamente "maior" q eles mesmos =)))

Ivan "Doomer" Carlos
Social Engineering Specialist
Cell.: +55 (11) 8112-0666
icarlos@icarlos.net
www.icarlos.net
--------------------------------------------------


----- Original Message -----
From: "Rodrigo Marques" <rodmarq72@yahoo.com.br>
To: "Ceticismo Aberto" <ceticismoaberto@yahoogrupos.com.br>; "CienciaList"
<ciencialist@yahoogrupos.com.br>; "Perry Rhodan Brasil"
<perry-rhodan-brazil@yahoogrupos.com.br>; "Sociedade Brasileira de Céticos e
Racionalistas" <sbcr@yahoogrupos.com.br>; "Sociedade da Terra redonda"
<strbrasil@yahoogrupos.com.br>
Sent: Friday, March 18, 2005 3:18 PM
Subject: [ciencialist] Uma bola de fogo, um ET?



http://odia.ig.com.br/brasil/br180302.htm
Uma bola de fogo, um ET?

Moradores de Aracruz levam 'alien' para hospital e descobrem que objeto era
um boneco

'Moradores do bairro Limão entraram em contato com a emissora comunicando
que após a queda de uma bola de fogo, um objeto não identificado foi
encontrado no bairro". A notícia era transmitida por uma rádio do município
de Aracruz, interior do Espírito Santo, enquanto moradores e policiais do 5º
BPM chegavam ao hospital municipal São Camilo com partes de um suposto
extraterrestre dentro de uma caixa de sapatos. O material chegou a ser
submetido a exames a pedido da população que acreditava se tratar de um ET.
O exame do material no microscópio constatou: um boneco, feito de borracha e
material esponjoso.

A história começou às 18h de terça-feira quando a dona de casa Adriana Silva
Francisco, 34 anos, encontrou em seu quintal, no bairro Vila Nova, um
objeto, para ela, não identificado, após ter visto uma bola de fogo no céu.
Segundo o policial Balan, Adriana estava assustada e chamou os vizinhos
achando que fosse um animal ou até mesmo uma criança deformada. "Nunca vi
algo parecido", disse. A polícia recebeu mais 50 ligações de pessoas que
perguntavam como se defender de uma invasão alienígena. 'ET' foi espancado
por moradores com madeira

Segundo moradores, antes de ser levado para o hospital, o suposto ET "se
mexeu", o que levou as pessoas a surrarem o objeto com paus e pontapés. A
gerente do hospital, Bernardete Braz, explicou que a princípio os médicos
ficaram preocupados porque já receberam fetos em caixas de sapato. "Foi uma
brincadeira de mau-gosto", avaliou. Muitos não quiseram sair do hospital
antes do "diagnóstico médico", que detectou apenas borracha e coliformes
fecais.

O mistério só foi finalmente concluído quando constataram se tratar de um
brinquedo, vendido na cidade vizinha, Ibiraçu. O motorista, Marcelo Sfalsin,
não se convenceu e levou uma perna de recordação.

Para a polícia, uma brincadeira

O município de Aracruz tem 70 mil habitantes e fica a 50 km da capital
Vitória. Segundo o gerente da rádio Nova Onda, Evilásio Oliveira Costa, a
central telefônica da estação ficou sobrecarregada. "Diziam que estávamos
escondendo informação. Quando informamos que o suposto alien foi levado para
o hospital, um ouvinte disse que era mentira e que a NASA (agência espacial
americana) o tinha levado", relatou. Ele explicou que os repórteres não
identificaram o brinquedo porque já estava em pedaços.

A polícia considerou o episódio como "um mal-entendido" e uma "brincadeira",
por isso não foi registrada ocorrência. Como o número de pessoas que
afirmaram ter visto um extraterrestre foi grande, os policiais não
conseguiram identificar os responsáveis.

Boneco vendido em lojas do Rio

O que é motivo de preocupação em Limão, nas lojas do Saara é pura chacota. A
universitária Renata Emille comprou ontem na Carla Brinquedos, no Centro, o
boneco ET igual ao encontrado em Aracruz para enganar os amigos. "Quando
meus amigos estiverem bêbados, vou colocar o ET na mesa e assustá-los",
brinca.

O brinquedo mede 5 centímetros, quando submerso na água ele pode chegar a
20. "Se deixar fora d'água ele volta a ficar pequeno", explica , dono da
loja, Felippe Kyrillos.



---------------------------------
Yahoo! Mail - Com 250MB de espaço. Abra sua conta!

[As partes desta mensagem que não continham texto foram removidas]



##### ##### #####

Para saber mais visite
http://www.ciencialist.hpg.ig.com.br


##### ##### ##### #####
Links do Yahoo! Grupos











SUBJECT: Re: [ciencialist] Re: ninguem é inocente.
FROM: "Oraculo" <oraculo@atibaia.com.br>
TO: <ciencialist@yahoogrupos.com.br>
DATE: 18/03/2005 15:53

Olá Tarciso

De uma olhada neste texto publicado no Observatorio da Imprensa, de autoria do nosso colega Takata, para saber mais sobre o caso do Sudário..:-)

http://observatorio.ultimosegundo.ig.com.br/ofjor/ofc201099.htm

O texto está no meio da página, role um pouco e vai encontrar o titulo Santo Sudário. É exceletne leitura.

Um abraço.

Homero


----- Original Message -----
From: TARCISIO BORGES
To: ciencialist@yahoogrupos.com.br
Sent: Friday, March 18, 2005 2:11 PM
Subject: Re: [ciencialist] Re: ninguem é inocente.


Mmmm (pensamento bovino)... que interessante.

Poderia me indicar a fonte desta informação?

[]s
TARCISIO BORGES
tbs97@fisica.ufpr.br

On Fri, 18 Mar 2005, pubmed2005 wrote:

> De acordo com nova pesquisa o Sudário tem aproximadamente 3.000 anos,
> isto é, podendo ser anterior a Cristo



##### ##### #####

Para saber mais visite
http://www.ciencialist.hpg.ig.com.br


##### ##### ##### #####


Yahoo! Grupos, um serviço oferecido por:
PUBLICIDADE




------------------------------------------------------------------------------
Links do Yahoo! Grupos

a.. Para visitar o site do seu grupo na web, acesse:
http://br.groups.yahoo.com/group/ciencialist/

b.. Para sair deste grupo, envie um e-mail para:
ciencialist-unsubscribe@yahoogrupos.com.br

c.. O uso que você faz do Yahoo! Grupos está sujeito aos Termos do Serviço do Yahoo!.



[As partes desta mensagem que não continham texto foram removidas]



SUBJECT: Livre-docência
FROM: "L.E.R.de Carvalho" <lecarvalho@infolink.com.br>
TO: ciencialist@yahoogrupos.com.br
DATE: 18/03/2005 16:14

At 12:27 18/3/2005, you wrote:
>Caro Emiliano,
>
>Os livres-docentes são os Grandes Mestres Jedy do mundo acadêmico. Eles
>são capazes de controlar os elementos, andar sobre as águas, multiplicar
>os pães, rescussitar os mortos, etc. Após fazer mestrado, doutorado,
>pós-doutorado, ter vários anos de profissão e publicar vários trabalhos
>releventes, o candidato pode fazer concurso para livre-docência, quando
>houver vagas. O concurso envolve, além de prova escrita, a preparação de
>um resumo biográfico (memorial) e a defesa de uma tese em frente a banca
>examinadora. Algumas faculdades também submetem o memorial, que deve
>comprovar a obra do pesquisador, à argüição pública. É mole?
>
>[ ]s
>
>Alvaro Augusto



Acho que não, Alvaro.
Me parece que em cada universidade isso tem uma definição diferente.


Em alguns lugares, acho, isso serve pra quem nao fez doutorado.
Substitui o título pra permitir promoção funcional.

L.E.

[As partes desta mensagem que não continham texto foram removidas]



SUBJECT: Re: ninguem é inocente.
FROM: "pubmed2005" <pubmed2005@yahoo.com.br>
TO: ciencialist@yahoogrupos.com.br
DATE: 18/03/2005 16:19


La vai

http://diariodigital.sapo.pt/news_history.asp?
section_id=13&id_news=156054


--- Em ciencialist@yahoogrupos.com.br, "pubmed2005" <pubmed2005@y...>
escreveu
>
> Procura no Google que voce encontra rapidinho
>
>
> --- Em ciencialist@yahoogrupos.com.br, TARCISIO BORGES <tbs97@f...>
> escreveu
> > Mmmm (pensamento bovino)... que interessante.
> >
> > Poderia me indicar a fonte desta informação?
> >
> > []s
> > TARCISIO BORGES
> > tbs97@f...






SUBJECT: Re: ninguem é inocente.
FROM: "pubmed2005" <pubmed2005@yahoo.com.br>
TO: ciencialist@yahoogrupos.com.br
DATE: 18/03/2005 16:24


É o link nao entrou, lá vai

Santo Sudário é mais antigo do que se pensava

O Santo Sudário é muito mais antigo do que o que o revelado pela
datação com carbono-14 feita nos anos 80, de acordo com um novo
estudo. A investigação, publicada na Thermochimica Acta, aponta para
que a relíquia tenha entre 1300 e 3 mil anos.



Para o autor do estudo, Raymond Rogers, a datação feita com carbono-
14 feitas em 1988, que tinham concluído que o sudário não passava de
uma falsificação medieval, está errada. Isto porque, defende, o
material usado na análise de então provinha de um remendo feito à
relíquia na Idade Média, depois de um incêndio a ter danificado.
No seu estudo, Rogers analisou e comparou a amostra usada em 1988 com
outras retiradas do pano, submetendo-os a testes a nível
microquímico, e concluiu que existem diferenças na tinta. Além disso,
as análises revelaram a presença de um químico denominado vanilina na
primeira amostra, que não pode ser encontrada no resto do sudário.
Este químico existe normalmente em tecidos de origem vegetal, mas
desaparece à medida que o material envelhece.

«O facto de não ser possível detectar vanilina nas fibras do sudário,
nos Pergaminhos do Mar Morto e em outros tecidos muito antigos indica
que o sudário é muito antigo», adianta Rogers, acrescentando que a
deterioração da composição deste químico sugere que a relíquia «tem
entre 1300 e três mil anos».


--- Em ciencialist@yahoogrupos.com.br, "pubmed2005" <pubmed2005@y...>
escreveu
>
> La vai
>
> http://diariodigital.sapo.pt/news_history.asp?
> section_id=13&id_news=156054






SUBJECT: Experimento mostra que efeito placebo é bioquímico
FROM: "pubmed2005" <pubmed2005@yahoo.com.br>
TO: ciencialist@yahoogrupos.com.br
DATE: 18/03/2005 16:27


Experimento mostra que efeito placebo é bioquímico

Um experimento realizado na Universidade de Turim, na Itália,
descobriu que o
efeito placebo, quando uma substância provoca efeito apesar de não
possuir um
ingrediente ativo, é uma reação bioquímica.
O placebo, uma substância inerte ou cirurgia ou terapia "de mentira",
é usada
como controle em uma experiência ou dada a um paciente afirmando que
ela possui
um efeito benéfico. Muitos acreditam que o efeito placebo seja
psicológico
devido à crença de que o remédio ajudará.

Os cientistas costumam realizar experimento em que, por vários dias,
a dor é
controlada com morfina. No último dia da experiência, a morfina é
trocada por
uma substância salina (plaecebo), mas o paciente continua sem dor. O
pesquisador
italiano Fabrizio Benedetti resolveu adicionar a essa substância
salina a
naloxone, uma droga que bloqueia os efeitos da morfina.

O resultado surpreendeu: o poder de aliviar a dor da solução salina
foi cortado.
A experiência italiana provou que se trata de uma reação bioquímica.
Os
cientistas conhecem há anos o efeito placebo, mas ainda não tinha
obtido nenhuma
explicação. Os pesquisadores ainda precisam identificar quando e onde
o placebo
age.

"Nós temos muito a aprender sobre o que acontece, mas uma coisa está
clara, a
mente pode afetar a química do corpo", disse Benedetti à revista New
Scientist.


Redação Terra







SUBJECT: Re: [ciencialist] Re: ninguem é inocente.
FROM: TARCISIO BORGES <tbs97@fisica.ufpr.br>
TO: ciencialist@yahoogrupos.com.br
DATE: 18/03/2005 17:08

Ok. Mas onde está a fonte de que o sudorífero pano tem 3000 anos???

[]s
TARCISIO BORGES
tbs97@fisica.ufpr.br

On Fri, 18 Mar 2005, Oraculo wrote:
> Olá Tarciso
>
> De uma olhada neste texto publicado no Observatorio da Imprensa, de
> autoria do nosso colega Takata, para saber mais sobre o caso do
> Sudário..:-)
>
> http://observatorio.ultimosegundo.ig.com.br/ofjor/ofc201099.htm
>
> O texto está no meio da página, role um pouco e vai encontrar o titulo
> Santo Sudário. É exceletne leitura.
>
> Um abraço.
>
> Homero
>
> On Fri, 18 Mar 2005, pubmed2005 wrote:
>
> > De acordo com nova pesquisa o Sudário tem aproximadamente 3.000 anos,
> > isto é, podendo ser anterior a Cristo



SUBJECT: Re: [ciencialist] Re: ninguem é inocente.
FROM: TARCISIO BORGES <tbs97@fisica.ufpr.br>
TO: ciencialist@yahoogrupos.com.br
DATE: 18/03/2005 17:18

http://brazil.skepdic.com/sudario.html

Encontrei algo aqui.

A metodologia do Sr. Rogers parece muito suspeita.

[]s
TARCISIO BORGES
tbs97@fisica.ufpr.br

On Fri, 18 Mar 2005, pubmed2005 wrote:
> La vai
>
> http://diariodigital.sapo.pt/news_history.asp?
> section_id=13&id_news=156054



SUBJECT: Re: ninguem é inocente.
FROM: "pubmed2005" <pubmed2005@yahoo.com.br>
TO: ciencialist@yahoogrupos.com.br
DATE: 18/03/2005 17:26


O correto seria refazer todos os testes a respeito do Sudário,
inclusive a do carbono-14 em diferentes partes do tecido, senão tb se
poderia argumentar que o teste nao foi completo e tb apresenta
falhas . Depois, se realizar os testes químicos para comparar.

O parecer atual da ciencia ainda continua meio escorregativo - pois a
parte analisada do carbono 14 pode ter sido muito bem a remendada que
possuía vinila(uma sibstancia que nao resiste ao envelhecimento)

Acho muito pertinente a nova dewcoberta do Rogers e em se tratando de
ciencia, fico com ela ate o momento.



--- Em ciencialist@yahoogrupos.com.br, TARCISIO BORGES <tbs97@f...>
escreveu
> http://brazil.skepdic.com/sudario.html
>
> Encontrei algo aqui.
>
> A metodologia do Sr. Rogers parece muito suspeita.
>
> []s
> TARCISIO BORGES
> tbs97@f...
>
> On Fri, 18 Mar 2005, pubmed2005 wrote:
> > La vai
> >
> > http://diariodigital.sapo.pt/news_history.asp?
> > section_id=13&id_news=156054





SUBJECT: Re: ninguem é inocente.
FROM: "pubmed2005" <pubmed2005@yahoo.com.br>
TO: ciencialist@yahoogrupos.com.br
DATE: 18/03/2005 17:32


Nao me parece.

Rogers nao afirmou que o tecido é o manto de Jesus, ele apenas
realizou testes químicos em diferentes partes do tecido.
Agora se os "céticos" nao acreditam que a pesquisa é séria é outros
quinhentos.
Então, quem nunca viu a pesquisa do carbono-14 pode especular que os
cientistas nunca puseram a mao no manto, o que seria um contrasenso.

Ainda mais esse negócio de ceticismo pela internet virou religiao.
Hoje sou cético aos céticos de plantão

--- Em ciencialist@yahoogrupos.com.br, TARCISIO BORGES <tbs97@f...>
escreveu
> http://brazil.skepdic.com/sudario.html
>
> Encontrei algo aqui.
>
> A metodologia do Sr. Rogers parece muito suspeita.
>
> []s
> TARCISIO BORGES
> tbs97@f...






SUBJECT: Re: [ciencialist] Re: ninguem é inocente.
FROM: "Oraculo" <oraculo@atibaia.com.br>
TO: <ciencialist@yahoogrupos.com.br>
DATE: 18/03/2005 17:40

Olá Pubmed

Bem mais escorregadio é o comportamento da detentora do objeto em questão..:-) Afinal, para ser possível uma análise precisa e confiável de um artefato arqueológico, de forma cientifica e racional, é preciso o maior número de testes possível, total colaboração do proprietário, acesso a todos os componentes, e a todas as partes do objeto, etc.

E isso, é fácil perceber, nào será obtido tão cedo pelos cientistas..:-)

Assim, se a hipotese cientifica tem problemas, eles não são nem de perto tão grandes quanto a hipótese milagrosa..:-) Com o estudo de apenas alguns fios e pedaços mínimos de tecido foi possível levantar diversas hipoteses (que só podem ser comprovadas se a igreja permitir o exame minuciosos e completo do artefato), mas a hipótese milagrosa se baseia em nada mais que a afirmação da igreja e em lendas diversas. Como a impressão da imagem pelo "calor de uma explosão nuclear enquanto o corpo flutuava sobre o pano" e outras mais malucas.

O problema é sempre esse, não se pode usar do conhecimento cientifico para analisar o objeto, apenas um contato mínimo e misterioso com partes minimas e em condições menos que ideais. Pesquisadores não iriam destruir o artefato, na verdade, quando é necessário descobrir formas de proteger reliquias sagradas, a igreja sempre pede a ajuda de cientistas. Assim, porque não se permite estudar a fundo o sudario, e descobrir todo o possível sobre ele, com confiabilidade?

O que o teste carbono efetuado determinou não foi nada mais que uma data. Para um milagre, nem isso importa, já que a divindade milagrosa poderia facilmente ter modificado a taxa de carbono do pano ao imprimir sua imagem mágicamente, e não há nada que possa refutar essa afirmação..:-) Por que o medo?

E o mesmo sobre o novo estudo, ainda não confirmado por pesquisadores independentes (coisa que o primeiro já foi mais de uma vez), ele apenas modifica uma data possível (e com enorme amplitude, de 1300 a 3000 anos). Não implica em que é real ou comprova que foi usado em jesus. Aliás, a argumentação que sendo de 3000 anos, ele poderia ter sido usado na época de jesus, guardado até o momento de ser necessário, pode ser usada da mesma forma se for datado com, digamos, 2000 anos. Sim, pode ter 2000 anos, mas pode ter sido guardado até o século 12 e só então usado para criar a fraude.

Para saber com mais certeza e um mínimo de confiabilidade, basta que a igreja permita um estudo profundo e cuidadoso por cientistas. Mas, eu não apostaria nisso..:-)

Um abraço.

Homero





----- Original Message -----
From: pubmed2005
To: ciencialist@yahoogrupos.com.br
Sent: Friday, March 18, 2005 5:26 PM
Subject: [ciencialist] Re: ninguem é inocente.



O correto seria refazer todos os testes a respeito do Sudário,
inclusive a do carbono-14 em diferentes partes do tecido, senão tb se
poderia argumentar que o teste nao foi completo e tb apresenta
falhas . Depois, se realizar os testes químicos para comparar.

O parecer atual da ciencia ainda continua meio escorregativo - pois a
parte analisada do carbono 14 pode ter sido muito bem a remendada que
possuía vinila(uma sibstancia que nao resiste ao envelhecimento)

Acho muito pertinente a nova dewcoberta do Rogers e em se tratando de
ciencia, fico com ela ate o momento.



--- Em ciencialist@yahoogrupos.com.br, TARCISIO BORGES <tbs97@f...>
escreveu
> http://brazil.skepdic.com/sudario.html
>
> Encontrei algo aqui.
>
> A metodologia do Sr. Rogers parece muito suspeita.
>
> []s
> TARCISIO BORGES
> tbs97@f...
>
> On Fri, 18 Mar 2005, pubmed2005 wrote:
> > La vai
> >
> > http://diariodigital.sapo.pt/news_history.asp?
> > section_id=13&id_news=156054





##### ##### #####

Para saber mais visite
http://www.ciencialist.hpg.ig.com.br


##### ##### ##### #####


Yahoo! Grupos, um serviço oferecido por:







------------------------------------------------------------------------------
Links do Yahoo! Grupos

a.. Para visitar o site do seu grupo na web, acesse:
http://br.groups.yahoo.com/group/ciencialist/

b.. Para sair deste grupo, envie um e-mail para:
ciencialist-unsubscribe@yahoogrupos.com.br

c.. O uso que você faz do Yahoo! Grupos está sujeito aos Termos do Serviço do Yahoo!.



[As partes desta mensagem que não continham texto foram removidas]



SUBJECT: Re: ninguem é inocente.
FROM: "pubmed2005" <pubmed2005@yahoo.com.br>
TO: ciencialist@yahoogrupos.com.br
DATE: 18/03/2005 17:48


Mas o Rogers nao afirmou que o manto era de Jesus . Apenas realizou
testes químicos e nao encontrando vanila no resto do manto, deduziu
cientificamente que o manto "deve" tem mais idade do que o previsto.

Não sei de onde voces tiraram que o Rogers disse que o manto tinha
aproximadamente 3000 anos e portanto deveria ser de Jesus.

Ninguém disse isso...


--- Em ciencialist@yahoogrupos.com.br, "Oraculo" <oraculo@a...>
escreveu
> Olá Pubmed
>
> Bem mais escorregadio é o comportamento da detentora do objeto em
questão..:-) Afinal, para ser possível uma análise precisa e
confiável de um artefato arqueológico, de forma cientifica e
racional, é preciso o maior número de testes possível, total
colaboração do proprietário, acesso a todos os componentes, e a todas
as partes do objeto, etc.
>
> E isso, é fácil perceber, nào será obtido tão cedo pelos
cientistas..:-)
>
> Assim, se a hipotese cientifica tem problemas, eles não são nem de
perto tão grandes quanto a hipótese milagrosa..:-) Com o estudo de
apenas alguns fios e pedaços mínimos de tecido foi possível levantar
diversas hipoteses (que só podem ser comprovadas se a igreja permitir
o exame minuciosos e completo do artefato), mas a hipótese milagrosa
se baseia em nada mais que a afirmação da igreja e em lendas
diversas. Como a impressão da imagem pelo "calor de uma explosão
nuclear enquanto o corpo flutuava sobre o pano" e outras mais malucas.
>
> O problema é sempre esse, não se pode usar do conhecimento
cientifico para analisar o objeto, apenas um contato mínimo e
misterioso com partes minimas e em condições menos que ideais.
Pesquisadores não iriam destruir o artefato, na verdade, quando é
necessário descobrir formas de proteger reliquias sagradas, a igreja
sempre pede a ajuda de cientistas. Assim, porque não se permite
estudar a fundo o sudario, e descobrir todo o possível sobre ele, com
confiabilidade?
>
> O que o teste carbono efetuado determinou não foi nada mais que uma
data. Para um milagre, nem isso importa, já que a divindade milagrosa
poderia facilmente ter modificado a taxa de carbono do pano ao
imprimir sua imagem mágicamente, e não há nada que possa refutar essa
afirmação..:-) Por que o medo?
>
> E o mesmo sobre o novo estudo, ainda não confirmado por
pesquisadores independentes (coisa que o primeiro já foi mais de uma
vez), ele apenas modifica uma data possível (e com enorme amplitude,
de 1300 a 3000 anos). Não implica em que é real ou comprova que foi
usado em jesus. Aliás, a argumentação que sendo de 3000 anos, ele
poderia ter sido usado na época de jesus, guardado até o momento de
ser necessário, pode ser usada da mesma forma se for datado com,
digamos, 2000 anos. Sim, pode ter 2000 anos, mas pode ter sido
guardado até o século 12 e só então usado para criar a fraude.
>
> Para saber com mais certeza e um mínimo de confiabilidade, basta
que a igreja permita um estudo profundo e cuidadoso por cientistas.
Mas, eu não apostaria nisso..:-)
>
> Um abraço.
>
> Homero
>
>
>
>
>
> ----- Original Message -----
> From: pubmed2005
> To: ciencialist@yahoogrupos.com.br
> Sent: Friday, March 18, 2005 5:26 PM
> Subject: [ciencialist] Re: ninguem é inocente.
>
>
>
> O correto seria refazer todos os testes a respeito do Sudário,
> inclusive a do carbono-14 em diferentes partes do tecido, senão
tb se
> poderia argumentar que o teste nao foi completo e tb apresenta
> falhas . Depois, se realizar os testes químicos para comparar.
>
> O parecer atual da ciencia ainda continua meio escorregativo -
pois a
> parte analisada do carbono 14 pode ter sido muito bem a remendada
que
> possuía vinila(uma sibstancia que nao resiste ao envelhecimento)
>
> Acho muito pertinente a nova dewcoberta do Rogers e em se
tratando de
> ciencia, fico com ela ate o momento.
>
>
>
> --- Em ciencialist@yahoogrupos.com.br, TARCISIO BORGES
<tbs97@f...>
> escreveu
> > http://brazil.skepdic.com/sudario.html
> >
> > Encontrei algo aqui.
> >
> > A metodologia do Sr. Rogers parece muito suspeita.
> >
> > []s
> > TARCISIO BORGES
> > tbs97@f...
> >
> > On Fri, 18 Mar 2005, pubmed2005 wrote:
> > > La vai
> > >
> > > http://diariodigital.sapo.pt/news_history.asp?
> > > section_id=13&id_news=156054
>
>
>
>
>
> ##### ##### #####
>
> Para saber mais visite
> http://www.ciencialist.hpg.ig.com.br
>
>
> ##### ##### ##### #####
>
>
> Yahoo! Grupos, um serviço oferecido por:
>
>
>
>
>
>
>
> --------------------------------------------------------------------
----------
> Links do Yahoo! Grupos
>
> a.. Para visitar o site do seu grupo na web, acesse:
> http://br.groups.yahoo.com/group/ciencialist/
>
> b.. Para sair deste grupo, envie um e-mail para:
> ciencialist-unsubscribe@yahoogrupos.com.br
>
> c.. O uso que você faz do Yahoo! Grupos está sujeito aos Termos
do Serviço do Yahoo!.
>
>
>
> [As partes desta mensagem que não continham texto foram removidas]





SUBJECT: Re: [ciencialist] Re: ninguem é inocente.
FROM: "Oraculo" <oraculo@atibaia.com.br>
TO: <ciencialist@yahoogrupos.com.br>
DATE: 18/03/2005 17:50

Olá Pubmed

Suspeita toda pesquisa é, até que confirmação independente apareça. Essa é a base da confiabilidade do método científico e da ciência. Uma pesquisa, qualquer pesquisa, seja sobre a data do sudario, seja sobre fusão a frio, seja sobre a gravitação universal, deve ser repetida e confirmada de forma independente. Ou será apenas um relato anedótico.

A gravitação tem sido confirmada por diversas pesquisas, estudos e previsões confirmadas por muito tempo. É um conhecimento confiável e científico. A fusão a frio foi refutada, por ser impossível repetir seus dados e experimentos, o que invalida suas conclusões. Não é, neste momento e na forma como foi proposta pelos cientistas que a apresentaram, confiável ou científica. Já o novo estudo do Sr. Rogers ainda carece de repetição ou confirmação independente. Por enquanto, é uma pesquisa em compasso de espera, precisa de mais elementos e estudos que a refutem ou confirmem.

O autor do primeiro estudo com datação de carbono, que foi confirmada por outros estudos, tem reservas e críticas ao novo estudo. Críticas bem embasadas e pertinentes, que esperam por uma refutação ou contestação. Até lá, ser cético é uma boa idéia, um cuidado racional, não uma demonstração de "mente fechada"..:-)

Cetisismo não é religiào, não precisa de crenças, é uma ferramenta de avaliação, que dá a seu usuário uma grande confiabilidade e segurança de resultados. Não é uma ferramenta perfeita, nem pretende ser, mas é a melhor que temos, a que mais confiabilidade apresenta para essa função: pensar sobre o universo.

Um abraço.

Homero

----- Original Message -----
From: pubmed2005
To: ciencialist@yahoogrupos.com.br
Sent: Friday, March 18, 2005 5:32 PM
Subject: [ciencialist] Re: ninguem é inocente.



Nao me parece.

Rogers nao afirmou que o tecido é o manto de Jesus, ele apenas
realizou testes químicos em diferentes partes do tecido.
Agora se os "céticos" nao acreditam que a pesquisa é séria é outros
quinhentos.
Então, quem nunca viu a pesquisa do carbono-14 pode especular que os
cientistas nunca puseram a mao no manto, o que seria um contrasenso.

Ainda mais esse negócio de ceticismo pela internet virou religiao.
Hoje sou cético aos céticos de plantão

--- Em ciencialist@yahoogrupos.com.br, TARCISIO BORGES <tbs97@f...>
escreveu
> http://brazil.skepdic.com/sudario.html
>
> Encontrei algo aqui.
>
> A metodologia do Sr. Rogers parece muito suspeita.
>
> []s
> TARCISIO BORGES
> tbs97@f...






##### ##### #####

Para saber mais visite
http://www.ciencialist.hpg.ig.com.br


##### ##### ##### #####


Yahoo! Grupos, um serviço oferecido por:

São Paulo Rio de Janeiro Curitiba Porto Alegre Belo Horizonte Brasília




------------------------------------------------------------------------------
Links do Yahoo! Grupos

a.. Para visitar o site do seu grupo na web, acesse:
http://br.groups.yahoo.com/group/ciencialist/

b.. Para sair deste grupo, envie um e-mail para:
ciencialist-unsubscribe@yahoogrupos.com.br

c.. O uso que você faz do Yahoo! Grupos está sujeito aos Termos do Serviço do Yahoo!.



[As partes desta mensagem que não continham texto foram removidas]



SUBJECT: Vanilina - santo sudario
FROM: "pubmed2005" <pubmed2005@yahoo.com.br>
TO: ciencialist@yahoogrupos.com.br
DATE: 18/03/2005 17:51


O nome correto da substancia é vanilina.

Desculpem-me





SUBJECT: Re: [ciencialist] Re: ninguem é inocente.
FROM: "Oraculo" <oraculo@atibaia.com.br>
TO: <ciencialist@yahoogrupos.com.br>
DATE: 18/03/2005 18:00

Olá Pubmed

Não disse mesmo, nem nós estamos dizendo..:-) Mas é claro que se a data for de 1200 depois de cristo, não pode ter sido usado por jesus, mas se for de mais de 3000 anos atras (veja, 3000 anos significa 2000 depois de cristo e 1000 antes, que é a margem que ele encontrou na pesquisa), ele mantém a esperança..:-) De uma pesquisada nos sites catolicos, e veja como a argumentação sobre as evidencais "científicas" sobre o sudário "provam" que ele foi usado por jesus..:-).

Como eu disse antes, esses estudos apenas determinam uma data, nada indicam a respeito de ter sido usado por quem quer que seja..:-) Mas, o que incomoda aos crentes é a data recente, que impede seu uso por jesus. O que, se pensar bem, é uma demonstração de falta de fé, uma vez que nada impede que o ser milagroso tenha modificado o carbono do pano, só para tirar um sarro da cara dos cientistas..:-)

Um abraço.

Homero


----- Original Message -----
From: pubmed2005
To: ciencialist@yahoogrupos.com.br
Sent: Friday, March 18, 2005 5:48 PM
Subject: [ciencialist] Re: ninguem é inocente.



Mas o Rogers nao afirmou que o manto era de Jesus . Apenas realizou
testes químicos e nao encontrando vanila no resto do manto, deduziu
cientificamente que o manto "deve" tem mais idade do que o previsto.

Não sei de onde voces tiraram que o Rogers disse que o manto tinha
aproximadamente 3000 anos e portanto deveria ser de Jesus.

Ninguém disse isso...


--- Em ciencialist@yahoogrupos.com.br, "Oraculo" <oraculo@a...>
escreveu
> Olá Pubmed
>
> Bem mais escorregadio é o comportamento da detentora do objeto em
questão..:-) Afinal, para ser possível uma análise precisa e
confiável de um artefato arqueológico, de forma cientifica e
racional, é preciso o maior número de testes possível, total
colaboração do proprietário, acesso a todos os componentes, e a todas
as partes do objeto, etc.
>
> E isso, é fácil perceber, nào será obtido tão cedo pelos
cientistas..:-)
>
> Assim, se a hipotese cientifica tem problemas, eles não são nem de
perto tão grandes quanto a hipótese milagrosa..:-) Com o estudo de
apenas alguns fios e pedaços mínimos de tecido foi possível levantar
diversas hipoteses (que só podem ser comprovadas se a igreja permitir
o exame minuciosos e completo do artefato), mas a hipótese milagrosa
se baseia em nada mais que a afirmação da igreja e em lendas
diversas. Como a impressão da imagem pelo "calor de uma explosão
nuclear enquanto o corpo flutuava sobre o pano" e outras mais malucas.
>
> O problema é sempre esse, não se pode usar do conhecimento
cientifico para analisar o objeto, apenas um contato mínimo e
misterioso com partes minimas e em condições menos que ideais.
Pesquisadores não iriam destruir o artefato, na verdade, quando é
necessário descobrir formas de proteger reliquias sagradas, a igreja
sempre pede a ajuda de cientistas. Assim, porque não se permite
estudar a fundo o sudario, e descobrir todo o possível sobre ele, com
confiabilidade?
>
> O que o teste carbono efetuado determinou não foi nada mais que uma
data. Para um milagre, nem isso importa, já que a divindade milagrosa
poderia facilmente ter modificado a taxa de carbono do pano ao
imprimir sua imagem mágicamente, e não há nada que possa refutar essa
afirmação..:-) Por que o medo?
>
> E o mesmo sobre o novo estudo, ainda não confirmado por
pesquisadores independentes (coisa que o primeiro já foi mais de uma
vez), ele apenas modifica uma data possível (e com enorme amplitude,
de 1300 a 3000 anos). Não implica em que é real ou comprova que foi
usado em jesus. Aliás, a argumentação que sendo de 3000 anos, ele
poderia ter sido usado na época de jesus, guardado até o momento de
ser necessário, pode ser usada da mesma forma se for datado com,
digamos, 2000 anos. Sim, pode ter 2000 anos, mas pode ter sido
guardado até o século 12 e só então usado para criar a fraude.
>
> Para saber com mais certeza e um mínimo de confiabilidade, basta
que a igreja permita um estudo profundo e cuidadoso por cientistas.
Mas, eu não apostaria nisso..:-)
>
> Um abraço.
>
> Homero
>
>
>
>
>
> ----- Original Message -----
> From: pubmed2005
> To: ciencialist@yahoogrupos.com.br
> Sent: Friday, March 18, 2005 5:26 PM
> Subject: [ciencialist] Re: ninguem é inocente.
>
>
>
> O correto seria refazer todos os testes a respeito do Sudário,
> inclusive a do carbono-14 em diferentes partes do tecido, senão
tb se
> poderia argumentar que o teste nao foi completo e tb apresenta
> falhas . Depois, se realizar os testes químicos para comparar.
>
> O parecer atual da ciencia ainda continua meio escorregativo -
pois a
> parte analisada do carbono 14 pode ter sido muito bem a remendada
que
> possuía vinila(uma sibstancia que nao resiste ao envelhecimento)
>
> Acho muito pertinente a nova dewcoberta do Rogers e em se
tratando de
> ciencia, fico com ela ate o momento.
>
>
>
> --- Em ciencialist@yahoogrupos.com.br, TARCISIO BORGES
<tbs97@f...>
> escreveu
> > http://brazil.skepdic.com/sudario.html
> >
> > Encontrei algo aqui.
> >
> > A metodologia do Sr. Rogers parece muito suspeita.
> >
> > []s
> > TARCISIO BORGES
> > tbs97@f...
> >
> > On Fri, 18 Mar 2005, pubmed2005 wrote:
> > > La vai
> > >
> > > http://diariodigital.sapo.pt/news_history.asp?
> > > section_id=13&id_news=156054
>
>
>
>
>
> ##### ##### #####
>
> Para saber mais visite
> http://www.ciencialist.hpg.ig.com.br
>
>
> ##### ##### ##### #####
>
>
> Yahoo! Grupos, um serviço oferecido por:
>
>
>
>
>
>
>
> --------------------------------------------------------------------
----------
> Links do Yahoo! Grupos
>
> a.. Para visitar o site do seu grupo na web, acesse:
> http://br.groups.yahoo.com/group/ciencialist/
>
> b.. Para sair deste grupo, envie um e-mail para:
> ciencialist-unsubscribe@yahoogrupos.com.br
>
> c.. O uso que você faz do Yahoo! Grupos está sujeito aos Termos
do Serviço do Yahoo!.
>
>
>
> [As partes desta mensagem que não continham texto foram removidas]





##### ##### #####

Para saber mais visite
http://www.ciencialist.hpg.ig.com.br


##### ##### ##### #####


Yahoo! Grupos, um serviço oferecido por:







------------------------------------------------------------------------------
Links do Yahoo! Grupos

a.. Para visitar o site do seu grupo na web, acesse:
http://br.groups.yahoo.com/group/ciencialist/

b.. Para sair deste grupo, envie um e-mail para:
ciencialist-unsubscribe@yahoogrupos.com.br

c.. O uso que você faz do Yahoo! Grupos está sujeito aos Termos do Serviço do Yahoo!.



[As partes desta mensagem que não continham texto foram removidas]



SUBJECT: Re: ninguem é inocente.
FROM: "pubmed2005" <pubmed2005@yahoo.com.br>
TO: ciencialist@yahoogrupos.com.br
DATE: 18/03/2005 18:03


Ué, o autor do segundo estudo está fazendo ciencia ao questionar o
primeiro estudo. E tb nunca ninguém disse qe o primeiro estudo tinha
sido refutado, e nem essa foi a intenção da pesquisa

Acho que Rogers levantou questoes bem interessantes com bases
científicas que merecem uma investigação. A questão da análise de
outras partes do tecido e da nao-presença de vanilina em grande parte
do tecido que nao foi analisada

O fato de o primeiro estudo ter sido bem sucedido nao invalida o
segundo estudo, que pode ser muito bem um complemento e até mesmo um
chamado para falhas do primeiro estudo.

A ciencia nao para...o fato que descobertas novas sempre demoram a
ser aceitas porque os cientistas se apegam ao dogma antigo.

Voce deve ter me mente que a intenção do segundo estudo é trazer mais
elementos a pesquisa, e nao concluir inexiravelmente que o manto tem
realmente 1300 anos, quando pode muito bem ter muito mais.

O segundo estudo foi muito pertinente,e merece atençao científica. O
fato de a pesquisa ter sido feita por um químico aposentado envolve
muitos preconceitos tb da comunidade científica, etc e tal...

--- Em ciencialist@yahoogrupos.com.br, "Oraculo" <oraculo@a...>
escreveu

> O autor do primeiro estudo com datação de carbono, que foi
confirmada por outros estudos, tem reservas e críticas ao novo
estudo. Críticas bem embasadas e pertinentes, que esperam por uma
refutação ou contestação. Até lá, ser cético é uma boa idéia, um
cuidado racional, não uma demonstração de "mente fechada"..:-)






SUBJECT: Re: ninguem é inocente.
FROM: "pubmed2005" <pubmed2005@yahoo.com.br>
TO: ciencialist@yahoogrupos.com.br
DATE: 18/03/2005 18:07


E incomoda tb aos "crentes céticos" ao fato de ele porventura ter
aprximadamente ter 3000 anos de idade, pelo simples fato, de ele
poder ter sido de Cristo de fato, não que seja, apenas uma suposição

Aliás a cristofobia é uma constante entre os céticos.
Porque se estava falando de alhos, e vieram com bugalhos


--- Em ciencialist@yahoogrupos.com.br, "Oraculo" <oraculo@a...>
escreveu
> Olá Pubmed
>
> Não disse mesmo, nem nós estamos dizendo..:-) Mas é claro que se a
data for de 1200 depois de cristo, não pode ter sido usado por jesus,
mas se for de mais de 3000 anos atras (veja, 3000 anos significa 2000
depois de cristo e 1000 antes, que é a margem que ele encontrou na
pesquisa), ele mantém a esperança..:-) De uma pesquisada nos sites
catolicos, e veja como a argumentação sobre as
evidencais "científicas" sobre o sudário "provam" que ele foi usado
por jesus..:-).
>
> Como eu disse antes, esses estudos apenas determinam uma data, nada
indicam a respeito de ter sido usado por quem quer que seja..:-) Mas,
o que incomoda aos crentes é a data recente, que impede seu uso por
jesus. O que, se pensar bem, é uma demonstração de falta de fé, uma
vez que nada impede que o ser milagroso tenha modificado o carbono do
pano, só para tirar um sarro da cara dos cientistas..:-)
>
> Um abraço.
>
> Homero
>
>
> ----- Original Message -----
> From: pubmed2005
> To: ciencialist@yahoogrupos.com.br
> Sent: Friday, March 18, 2005 5:48 PM
> Subject: [ciencialist] Re: ninguem é inocente.
>
>
>
> Mas o Rogers nao afirmou que o manto era de Jesus . Apenas
realizou
> testes químicos e nao encontrando vanila no resto do manto,
deduziu
> cientificamente que o manto "deve" tem mais idade do que o
previsto.
>
> Não sei de onde voces tiraram que o Rogers disse que o manto
tinha
> aproximadamente 3000 anos e portanto deveria ser de Jesus.
>
> Ninguém disse isso...
>
>
> --- Em ciencialist@yahoogrupos.com.br, "Oraculo" <oraculo@a...>
> escreveu
> > Olá Pubmed
> >
> > Bem mais escorregadio é o comportamento da detentora do objeto
em
> questão..:-) Afinal, para ser possível uma análise precisa e
> confiável de um artefato arqueológico, de forma cientifica e
> racional, é preciso o maior número de testes possível, total
> colaboração do proprietário, acesso a todos os componentes, e a
todas
> as partes do objeto, etc.
> >
> > E isso, é fácil perceber, nào será obtido tão cedo pelos
> cientistas..:-)
> >
> > Assim, se a hipotese cientifica tem problemas, eles não são nem
de
> perto tão grandes quanto a hipótese milagrosa..:-) Com o estudo
de
> apenas alguns fios e pedaços mínimos de tecido foi possível
levantar
> diversas hipoteses (que só podem ser comprovadas se a igreja
permitir
> o exame minuciosos e completo do artefato), mas a hipótese
milagrosa
> se baseia em nada mais que a afirmação da igreja e em lendas
> diversas. Como a impressão da imagem pelo "calor de uma explosão
> nuclear enquanto o corpo flutuava sobre o pano" e outras mais
malucas.
> >
> > O problema é sempre esse, não se pode usar do conhecimento
> cientifico para analisar o objeto, apenas um contato mínimo e
> misterioso com partes minimas e em condições menos que ideais.
> Pesquisadores não iriam destruir o artefato, na verdade, quando é
> necessário descobrir formas de proteger reliquias sagradas, a
igreja
> sempre pede a ajuda de cientistas. Assim, porque não se permite
> estudar a fundo o sudario, e descobrir todo o possível sobre ele,
com
> confiabilidade?
> >
> > O que o teste carbono efetuado determinou não foi nada mais que
uma
> data. Para um milagre, nem isso importa, já que a divindade
milagrosa
> poderia facilmente ter modificado a taxa de carbono do pano ao
> imprimir sua imagem mágicamente, e não há nada que possa refutar
essa
> afirmação..:-) Por que o medo?
> >
> > E o mesmo sobre o novo estudo, ainda não confirmado por
> pesquisadores independentes (coisa que o primeiro já foi mais de
uma
> vez), ele apenas modifica uma data possível (e com enorme
amplitude,
> de 1300 a 3000 anos). Não implica em que é real ou comprova que
foi
> usado em jesus. Aliás, a argumentação que sendo de 3000 anos, ele
> poderia ter sido usado na época de jesus, guardado até o momento
de
> ser necessário, pode ser usada da mesma forma se for datado com,
> digamos, 2000 anos. Sim, pode ter 2000 anos, mas pode ter sido
> guardado até o século 12 e só então usado para criar a fraude.
> >
> > Para saber com mais certeza e um mínimo de confiabilidade,
basta
> que a igreja permita um estudo profundo e cuidadoso por
cientistas.
> Mas, eu não apostaria nisso..:-)
> >
> > Um abraço.
> >
> > Homero
> >
> >
> >
> >
> >
> > ----- Original Message -----
> > From: pubmed2005
> > To: ciencialist@yahoogrupos.com.br
> > Sent: Friday, March 18, 2005 5:26 PM
> > Subject: [ciencialist] Re: ninguem é inocente.
> >
> >
> >
> > O correto seria refazer todos os testes a respeito do
Sudário,
> > inclusive a do carbono-14 em diferentes partes do tecido,
senão
> tb se
> > poderia argumentar que o teste nao foi completo e tb
apresenta
> > falhas . Depois, se realizar os testes químicos para comparar.
> >
> > O parecer atual da ciencia ainda continua meio escorregativo -

> pois a
> > parte analisada do carbono 14 pode ter sido muito bem a
remendada
> que
> > possuía vinila(uma sibstancia que nao resiste ao
envelhecimento)
> >
> > Acho muito pertinente a nova dewcoberta do Rogers e em se
> tratando de
> > ciencia, fico com ela ate o momento.
> >
> >
> >
> > --- Em ciencialist@yahoogrupos.com.br, TARCISIO BORGES
> <tbs97@f...>
> > escreveu
> > > http://brazil.skepdic.com/sudario.html
> > >
> > > Encontrei algo aqui.
> > >
> > > A metodologia do Sr. Rogers parece muito suspeita.
> > >
> > > []s
> > > TARCISIO BORGES
> > > tbs97@f...
> > >
> > > On Fri, 18 Mar 2005, pubmed2005 wrote:
> > > > La vai
> > > >
> > > > http://diariodigital.sapo.pt/news_history.asp?
> > > > section_id=13&id_news=156054
> >
> >
> >
> >
> >
> > ##### ##### #####
> >
> > Para saber mais visite
> > http://www.ciencialist.hpg.ig.com.br
> >
> >
> > ##### ##### ##### #####
> >
> >
> > Yahoo! Grupos, um serviço oferecido por:
> >
> >
> >
> >
> >
> >
> >
> > ----------------------------------------------------------------
----
> ----------
> > Links do Yahoo! Grupos
> >
> > a.. Para visitar o site do seu grupo na web, acesse:
> > http://br.groups.yahoo.com/group/ciencialist/
> >
> > b.. Para sair deste grupo, envie um e-mail para:
> > ciencialist-unsubscribe@yahoogrupos.com.br
> >
> > c.. O uso que você faz do Yahoo! Grupos está sujeito aos
Termos
> do Serviço do Yahoo!.
> >
> >
> >
> > [As partes desta mensagem que não continham texto foram
removidas]
>
>
>
>
>
> ##### ##### #####
>
> Para saber mais visite
> http://www.ciencialist.hpg.ig.com.br
>
>
> ##### ##### ##### #####
>
>
> Yahoo! Grupos, um serviço oferecido por:
>
>
>
>
>
>
>
> --------------------------------------------------------------------
----------
> Links do Yahoo! Grupos
>
> a.. Para visitar o site do seu grupo na web, acesse:
> http://br.groups.yahoo.com/group/ciencialist/
>
> b.. Para sair deste grupo, envie um e-mail para:
> ciencialist-unsubscribe@yahoogrupos.com.br
>
> c.. O uso que você faz do Yahoo! Grupos está sujeito aos Termos
do Serviço do Yahoo!.
>
>
>
> [As partes desta mensagem que não continham texto foram removidas]





SUBJECT: Re: ninguem é inocente.
FROM: "pubmed2005" <pubmed2005@yahoo.com.br>
TO: ciencialist@yahoogrupos.com.br
DATE: 18/03/2005 18:12


Ah é, é assim que voce vê? Quer dizer que se analizo que um artefato
arqueológico tem 2500 anos isso insinua que deve ter passado pelas
maos de Cristo?

Se a Igreja utiliza isso ou mão, nao tem nada ver com Rogers, é um
fato a parte - estão fazendo de uma descoberta cientifica uma
validação de seus argumentos religiosos

Mas isso é outra história....

-- Em ciencialist@yahoogrupos.com.br, "Oraculo" <oraculo@a...>
escreveu
> Olá Pubmed
>
> Não disse mesmo, nem nós estamos dizendo..:-) Mas é claro que se a
data for de 1200 depois de cristo, não pode ter sido usado por jesus,
mas se for de mais de 3000 anos atras (veja, 3000 anos significa 2000
depois de cristo e 1000 antes, que é a margem que ele encontrou na
pesquisa), ele mantém a esperança..:-) De uma pesquisada nos sites
catolicos, e veja como a argumentação sobre as
evidencais "científicas" sobre o sudário "provam" que ele foi usado
por jesus..:-).
>
> Como eu disse antes, esses estudos apenas determinam uma data, nada
indicam a respeito de ter sido usado por quem quer que seja..:-) Mas,
o que incomoda aos crentes é a data recente, que impede seu uso por
jesus. O que, se pensar bem, é uma demonstração de falta de fé, uma
vez que nada impede que o ser milagroso tenha modificado o carbono do
pano, só para tirar um sarro da cara dos cientistas..:-)
>
> Um abraço.
>
> Homero
>
>
> ----- Original Message -----
> From: pubmed2005
> To: ciencialist@yahoogrupos.com.br
> Sent: Friday, March 18, 2005 5:48 PM
> Subject: [ciencialist] Re: ninguem é inocente.
>
>
>
> Mas o Rogers nao afirmou que o manto era de Jesus . Apenas
realizou
> testes químicos e nao encontrando vanila no resto do manto,
deduziu
> cientificamente que o manto "deve" tem mais idade do que o
previsto.
>
> Não sei de onde voces tiraram que o Rogers disse que o manto
tinha
> aproximadamente 3000 anos e portanto deveria ser de Jesus.
>
> Ninguém disse isso...
>
>
> --- Em ciencialist@yahoogrupos.com.br, "Oraculo" <oraculo@a...>
> escreveu
> > Olá Pubmed
> >
> > Bem mais escorregadio é o comportamento da detentora do objeto
em
> questão..:-) Afinal, para ser possível uma análise precisa e
> confiável de um artefato arqueológico, de forma cientifica e
> racional, é preciso o maior número de testes possível, total
> colaboração do proprietário, acesso a todos os componentes, e a
todas
> as partes do objeto, etc.
> >
> > E isso, é fácil perceber, nào será obtido tão cedo pelos
> cientistas..:-)
> >
> > Assim, se a hipotese cientifica tem problemas, eles não são nem
de
> perto tão grandes quanto a hipótese milagrosa..:-) Com o estudo
de
> apenas alguns fios e pedaços mínimos de tecido foi possível
levantar
> diversas hipoteses (que só podem ser comprovadas se a igreja
permitir
> o exame minuciosos e completo do artefato), mas a hipótese
milagrosa
> se baseia em nada mais que a afirmação da igreja e em lendas
> diversas. Como a impressão da imagem pelo "calor de uma explosão
> nuclear enquanto o corpo flutuava sobre o pano" e outras mais
malucas.
> >
> > O problema é sempre esse, não se pode usar do conhecimento
> cientifico para analisar o objeto, apenas um contato mínimo e
> misterioso com partes minimas e em condições menos que ideais.
> Pesquisadores não iriam destruir o artefato, na verdade, quando é
> necessário descobrir formas de proteger reliquias sagradas, a
igreja
> sempre pede a ajuda de cientistas. Assim, porque não se permite
> estudar a fundo o sudario, e descobrir todo o possível sobre ele,
com
> confiabilidade?
> >
> > O que o teste carbono efetuado determinou não foi nada mais que
uma
> data. Para um milagre, nem isso importa, já que a divindade
milagrosa
> poderia facilmente ter modificado a taxa de carbono do pano ao
> imprimir sua imagem mágicamente, e não há nada que possa refutar
essa
> afirmação..:-) Por que o medo?
> >
> > E o mesmo sobre o novo estudo, ainda não confirmado por
> pesquisadores independentes (coisa que o primeiro já foi mais de
uma
> vez), ele apenas modifica uma data possível (e com enorme
amplitude,
> de 1300 a 3000 anos). Não implica em que é real ou comprova que
foi
> usado em jesus. Aliás, a argumentação que sendo de 3000 anos, ele
> poderia ter sido usado na época de jesus, guardado até o momento
de
> ser necessário, pode ser usada da mesma forma se for datado com,
> digamos, 2000 anos. Sim, pode ter 2000 anos, mas pode ter sido
> guardado até o século 12 e só então usado para criar a fraude.
> >
> > Para saber com mais certeza e um mínimo de confiabilidade,
basta
> que a igreja permita um estudo profundo e cuidadoso por
cientistas.
> Mas, eu não apostaria nisso..:-)
> >
> > Um abraço.
> >
> > Homero
> >
> >
> >
> >
> >
> > ----- Original Message -----
> > From: pubmed2005
> > To: ciencialist@yahoogrupos.com.br
> > Sent: Friday, March 18, 2005 5:26 PM
> > Subject: [ciencialist] Re: ninguem é inocente.
> >
> >
> >
> > O correto seria refazer todos os testes a respeito do
Sudário,
> > inclusive a do carbono-14 em diferentes partes do tecido,
senão
> tb se
> > poderia argumentar que o teste nao foi completo e tb
apresenta
> > falhas . Depois, se realizar os testes químicos para comparar.
> >
> > O parecer atual da ciencia ainda continua meio escorregativo -

> pois a
> > parte analisada do carbono 14 pode ter sido muito bem a
remendada
> que
> > possuía vinila(uma sibstancia que nao resiste ao
envelhecimento)
> >
> > Acho muito pertinente a nova dewcoberta do Rogers e em se
> tratando de
> > ciencia, fico com ela ate o momento.
> >
> >
> >
> > --- Em ciencialist@yahoogrupos.com.br, TARCISIO BORGES
> <tbs97@f...>
> > escreveu
> > > http://brazil.skepdic.com/sudario.html
> > >
> > > Encontrei algo aqui.
> > >
> > > A metodologia do Sr. Rogers parece muito suspeita.
> > >
> > > []s
> > > TARCISIO BORGES
> > > tbs97@f...
> > >
> > > On Fri, 18 Mar 2005, pubmed2005 wrote:
> > > > La vai
> > > >
> > > > http://diariodigital.sapo.pt/news_history.asp?
> > > > section_id=13&id_news=156054
> >
> >
> >
> >
> >
> > ##### ##### #####
> >
> > Para saber mais visite
> > http://www.ciencialist.hpg.ig.com.br
> >
> >
> > ##### ##### ##### #####
> >
> >
> > Yahoo! Grupos, um serviço oferecido por:
> >
> >
> >
> >
> >
> >
> >
> > ----------------------------------------------------------------
----
> ----------
> > Links do Yahoo! Grupos
> >
> > a.. Para visitar o site do seu grupo na web, acesse:
> > http://br.groups.yahoo.com/group/ciencialist/
> >
> > b.. Para sair deste grupo, envie um e-mail para:
> > ciencialist-unsubscribe@yahoogrupos.com.br
> >
> > c.. O uso que você faz do Yahoo! Grupos está sujeito aos
Termos
> do Serviço do Yahoo!.
> >
> >
> >
> > [As partes desta mensagem que não continham texto foram
removidas]
>
>
>
>
>
> ##### ##### #####
>
> Para saber mais visite
> http://www.ciencialist.hpg.ig.com.br
>
>
> ##### ##### ##### #####
>
>
> Yahoo! Grupos, um serviço oferecido por:
>
>
>
>
>
>
>
> --------------------------------------------------------------------
----------
> Links do Yahoo! Grupos
>
> a.. Para visitar o site do seu grupo na web, acesse:
> http://br.groups.yahoo.com/group/ciencialist/
>
> b.. Para sair deste grupo, envie um e-mail para:
> ciencialist-unsubscribe@yahoogrupos.com.br
>
> c.. O uso que você faz do Yahoo! Grupos está sujeito aos Termos
do Serviço do Yahoo!.
>
>
>
> [As partes desta mensagem que não continham texto foram removidas]





SUBJECT: Re: [ciencialist] Novo no grupo
FROM: Anderson Almeida da Silveira <anderson_a5@yahoo.com.br>
TO: ciencialist@yahoogrupos.com.br
DATE: 18/03/2005 19:00



Alberto Mesquita Filho <albmesq@uol.com.br> wrote:
----- Original Message -----
From: "PSavio"
Sent: Saturday, March 19, 2005 6:31 AM
Subject: [ciencialist] Novo no grupo

> Apresentando-me:

Bem-vindo à Ciencialist, o mundo do Léo.

> Mestrando de Física

Sem problemas. Ser físico não chega a ser um grande defeito. ;-)

> Espero poder aprender aqquí enriquecer minha tese.

Como disse o Léo, não deixa de ser uma boa hipótese, mas cuidado com os
doidos e com os defensores do escolasticismo, principalmente se a sua tese
for "moderna".

[ ]´s
Alberto
Nobre e Excelso Cavaleiro do Apocalipse Quântico
http://ecientificocultural.com/indice.htm
Mas indiferentemente a tudo isso, o neutrino tem massa, o elétron não é
uma carga elétrica coulombiana e a Terra se move. E a história se repetirá.


Olá Alberto, acho que você encontrou alguém que não vai totalmente contra as suas idéias. Caso algum defensor da "física moderna" termine com sua carreira e suas idéias, eu posso continuar elas para você. Faço ainda o 2° ano no CEFET-MA, mas aqui há defensores da "física moderna" que já ma ameaçaram quando falei sobre suas idéias. Tanto os secundaristas, quanto o pessoal da universidade.comecei a ler o seu site agora mais tarde poderei fazer algum comentário mais detalhado sobre ele.




---------------------------------
Yahoo! Mail - Com 250MB de espaço. Abra sua conta!

[As partes desta mensagem que não continham texto foram removidas]



SUBJECT: Re: [ciencialist] Re: ninguem é inocente.
FROM: Anderson Almeida da Silveira <anderson_a5@yahoo.com.br>
TO: ciencialist@yahoogrupos.com.br
DATE: 18/03/2005 19:19



pubmed2005 <pubmed2005@yahoo.com.br> wrote:

Ah é, é assim que voce vê? Quer dizer que se analizo que um artefato
arqueológico tem 2500 anos isso insinua que deve ter passado pelas
maos de Cristo?

Se a Igreja utiliza isso ou mão, nao tem nada ver com Rogers, é um
fato a parte - estão fazendo de uma descoberta cientifica uma
validação de seus argumentos religiosos

Mas isso é outra história....

-- Em ciencialist@yahoogrupos.com.br, "Oraculo" <oraculo@a...>
escreveu
> Olá Pubmed
>
> Não disse mesmo, nem nós estamos dizendo..:-) Mas é claro que se a
data for de 1200 depois de cristo, não pode ter sido usado por jesus,
mas se for de mais de 3000 anos atras (veja, 3000 anos significa 2000
depois de cristo e 1000 antes, que é a margem que ele encontrou na
pesquisa), ele mantém a esperança..:-) De uma pesquisada nos sites
catolicos, e veja como a argumentação sobre as
evidencais "científicas" sobre o sudário "provam" que ele foi usado
por jesus..:-).
>
> Como eu disse antes, esses estudos apenas determinam uma data, nada
indicam a respeito de ter sido usado por quem quer que seja..:-) Mas,
o que incomoda aos crentes é a data recente, que impede seu uso por
jesus. O que, se pensar bem, é uma demonstração de falta de fé, uma
vez que nada impede que o ser milagroso tenha modificado o carbono do
pano, só para tirar um sarro da cara dos cientistas..:-)
>
> Um abraço.
>
> Homero
>
>
> ----- Original Message -----
> From: pubmed2005
> To: ciencialist@yahoogrupos.com.br
> Sent: Friday, March 18, 2005 5:48 PM
> Subject: [ciencialist] Re: ninguem é inocente.
>
>
>
> Mas o Rogers nao afirmou que o manto era de Jesus . Apenas
realizou
> testes químicos e nao encontrando vanila no resto do manto,
deduziu
> cientificamente que o manto "deve" tem mais idade do que o
previsto.
>
> Não sei de onde voces tiraram que o Rogers disse que o manto
tinha
> aproximadamente 3000 anos e portanto deveria ser de Jesus.
>
> Ninguém disse isso...
>
>
> --- Em ciencialist@yahoogrupos.com.br, "Oraculo" <oraculo@a...>
> escreveu
> > Olá Pubmed
> >
> > Bem mais escorregadio é o comportamento da detentora do objeto
em
> questão..:-) Afinal, para ser possível uma análise precisa e
> confiável de um artefato arqueológico, de forma cientifica e
> racional, é preciso o maior número de testes possível, total
> colaboração do proprietário, acesso a todos os componentes, e a
todas
> as partes do objeto, etc.
> >
> > E isso, é fácil perceber, nào será obtido tão cedo pelos
> cientistas..:-)
> >
> > Assim, se a hipotese cientifica tem problemas, eles não são nem
de
> perto tão grandes quanto a hipótese milagrosa..:-) Com o estudo
de
> apenas alguns fios e pedaços mínimos de tecido foi possível
levantar
> diversas hipoteses (que só podem ser comprovadas se a igreja
permitir
> o exame minuciosos e completo do artefato), mas a hipótese
milagrosa
> se baseia em nada mais que a afirmação da igreja e em lendas
> diversas. Como a impressão da imagem pelo "calor de uma explosão
> nuclear enquanto o corpo flutuava sobre o pano" e outras mais
malucas.
> >
> > O problema é sempre esse, não se pode usar do conhecimento
> cientifico para analisar o objeto, apenas um contato mínimo e
> misterioso com partes minimas e em condições menos que ideais.
> Pesquisadores não iriam destruir o artefato, na verdade, quando é
> necessário descobrir formas de proteger reliquias sagradas, a
igreja
> sempre pede a ajuda de cientistas. Assim, porque não se permite
> estudar a fundo o sudario, e descobrir todo o possível sobre ele,
com
> confiabilidade?
> >
> > O que o teste carbono efetuado determinou não foi nada mais que
uma
> data. Para um milagre, nem isso importa, já que a divindade
milagrosa
> poderia facilmente ter modificado a taxa de carbono do pano ao
> imprimir sua imagem mágicamente, e não há nada que possa refutar
essa
> afirmação..:-) Por que o medo?
> >
> > E o mesmo sobre o novo estudo, ainda não confirmado por
> pesquisadores independentes (coisa que o primeiro já foi mais de
uma
> vez), ele apenas modifica uma data possível (e com enorme
amplitude,
> de 1300 a 3000 anos). Não implica em que é real ou comprova que
foi
> usado em jesus. Aliás, a argumentação que sendo de 3000 anos, ele
> poderia ter sido usado na época de jesus, guardado até o momento
de
> ser necessário, pode ser usada da mesma forma se for datado com,
> digamos, 2000 anos. Sim, pode ter 2000 anos, mas pode ter sido
> guardado até o século 12 e só então usado para criar a fraude.
> >
> > Para saber com mais certeza e um mínimo de confiabilidade,
basta
> que a igreja permita um estudo profundo e cuidadoso por
cientistas.
> Mas, eu não apostaria nisso..:-)
> >
> > Um abraço.
> >
> > Homero
> >
> >
> >
> >
> >
> > ----- Original Message -----
> > From: pubmed2005
> > To: ciencialist@yahoogrupos.com.br
> > Sent: Friday, March 18, 2005 5:26 PM
> > Subject: [ciencialist] Re: ninguem é inocente.
> >
> >
> >
> > O correto seria refazer todos os testes a respeito do
Sudário,
> > inclusive a do carbono-14 em diferentes partes do tecido,
senão
> tb se
> > poderia argumentar que o teste nao foi completo e tb
apresenta
> > falhas . Depois, se realizar os testes químicos para comparar.
> >
> > O parecer atual da ciencia ainda continua meio escorregativo -

> pois a
> > parte analisada do carbono 14 pode ter sido muito bem a
remendada
> que
> > possuía vinila(uma sibstancia que nao resiste ao
envelhecimento)
> >
> > Acho muito pertinente a nova dewcoberta do Rogers e em se
> tratando de
> > ciencia, fico com ela ate o momento.
> >
> >
> >
> > --- Em ciencialist@yahoogrupos.com.br, TARCISIO BORGES
> <tbs97@f...>
> > escreveu
> > > http://brazil.skepdic.com/sudario.html
> > >
> > > Encontrei algo aqui.
> > >
> > > A metodologia do Sr. Rogers parece muito suspeita.
> > >
> > > []s
> > > TARCISIO BORGES
> > > tbs97@f...
> > >
> > > On Fri, 18 Mar 2005, pubmed2005 wrote:
> > > > La vai
> > > >
> > > > http://diariodigital.sapo.pt/news_history.asp?
> > > > section_id=13&id_news=156054
> >
> >
> >
> >
> >
> > ##### ##### #####
> >
> > Para saber mais visite
> > http://www.ciencialist.hpg.ig.com.br
> >
> > >
> >
> >
> >
>

Bem, mesmo que provem que o tal sudário seja da época de Jesus, nada garante que ele o tenha usado. Se inventarem uma máquina do tempo ou coisa parecida aí, sim poderemos dar uma resposta segura. Pesquisas desse tipo não levam a lugar nenhum mesmo porque ninguém que está vivo hoje sabe como era o rosto de Jesus.




---------------------------------
Yahoo! Mail - Com 250MB de espaço. Abra sua conta!

[As partes desta mensagem que não continham texto foram removidas]



SUBJECT: Re: [ciencialist] Uma bola de fogo, um ET?
FROM: Anderson Almeida da Silveira <anderson_a5@yahoo.com.br>
TO: ciencialist@yahoogrupos.com.br
DATE: 18/03/2005 19:35



Ivan Carlos <icarlos@icarlos.net> wrote:
HUAHUAHUAHUAHUAHUAHUAH q viagem..!!!!

não é a toa q por qqer coisa meio mundo coloca a culpa em "alguém"
suportamente "maior" q eles mesmos =)))

Ivan "Doomer" Carlos
Social Engineering Specialist
Cell.: +55 (11) 8112-0666
icarlos@icarlos.net
www.icarlos.net
--------------------------------------------------


----- Original Message -----
From: "Rodrigo Marques" <rodmarq72@yahoo.com.br>
To: "Ceticismo Aberto" <ceticismoaberto@yahoogrupos.com.br>; "CienciaList"
<ciencialist@yahoogrupos.com.br>; "Perry Rhodan Brasil"
<perry-rhodan-brazil@yahoogrupos.com.br>; "Sociedade Brasileira de Céticos e
Racionalistas" <sbcr@yahoogrupos.com.br>; "Sociedade da Terra redonda"
<strbrasil@yahoogrupos.com.br>
Sent: Friday, March 18, 2005 3:18 PM
Subject: [ciencialist] Uma bola de fogo, um ET?



http://odia.ig.com.br/brasil/br180302.htm
Uma bola de fogo, um ET?

Moradores de Aracruz levam 'alien' para hospital e descobrem que objeto era
um boneco

'Moradores do bairro Limão entraram em contato com a emissora comunicando
que após a queda de uma bola de fogo, um objeto não identificado foi
encontrado no bairro". A notícia era transmitida por uma rádio do município
de Aracruz, interior do Espírito Santo, enquanto moradores e policiais do 5º
BPM chegavam ao hospital municipal São Camilo com partes de um suposto
extraterrestre dentro de uma caixa de sapatos. O material chegou a ser
submetido a exames a pedido da população que acreditava se tratar de um ET.
O exame do material no microscópio constatou: um boneco, feito de borracha e
material esponjoso.

A história começou às 18h de terça-feira quando a dona de casa Adriana Silva
Francisco, 34 anos, encontrou em seu quintal, no bairro Vila Nova, um
objeto, para ela, não identificado, após ter visto uma bola de fogo no céu.
Segundo o policial Balan, Adriana estava assustada e chamou os vizinhos
achando que fosse um animal ou até mesmo uma criança deformada. "Nunca vi
algo parecido", disse. A polícia recebeu mais 50 ligações de pessoas que
perguntavam como se defender de uma invasão alienígena. 'ET' foi espancado
por moradores com madeira

Segundo moradores, antes de ser levado para o hospital, o suposto ET "se
mexeu", o que levou as pessoas a surrarem o objeto com paus e pontapés. A
gerente do hospital, Bernardete Braz, explicou que a princípio os médicos
ficaram preocupados porque já receberam fetos em caixas de sapato. "Foi uma
brincadeira de mau-gosto", avaliou. Muitos não quiseram sair do hospital
antes do "diagnóstico médico", que detectou apenas borracha e coliformes
fecais.

O mistério só foi finalmente concluído quando constataram se tratar de um
brinquedo, vendido na cidade vizinha, Ibiraçu. O motorista, Marcelo Sfalsin,
não se convenceu e levou uma perna de recordação.

Para a polícia, uma brincadeira

O município de Aracruz tem 70 mil habitantes e fica a 50 km da capital
Vitória. Segundo o gerente da rádio Nova Onda, Evilásio Oliveira Costa, a
central telefônica da estação ficou sobrecarregada. "Diziam que estávamos
escondendo informação. Quando informamos que o suposto alien foi levado para
o hospital, um ouvinte disse que era mentira e que a NASA (agência espacial
americana) o tinha levado", relatou. Ele explicou que os repórteres não
identificaram o brinquedo porque já estava em pedaços.

A polícia considerou o episódio como "um mal-entendido" e uma "brincadeira",
por isso não foi registrada ocorrência. Como o número de pessoas que
afirmaram ter visto um extraterrestre foi grande, os policiais não
conseguiram identificar os responsáveis.

Boneco vendido em lojas do Rio

O que é motivo de preocupação em Limão, nas lojas do Saara é pura chacota. A
universitária Renata Emille comprou ontem na Carla Brinquedos, no Centro, o
boneco ET igual ao encontrado em Aracruz para enganar os amigos. "Quando
meus amigos estiverem bêbados, vou colocar o ET na mesa e assustá-los",
brinca.

O brinquedo mede 5 centímetros, quando submerso na água ele pode chegar a
20. "Se deixar fora d'água ele volta a ficar pequeno", explica , dono da
loja, Felippe Kyrillos.

>É, pois é. Se meu pai ouvisse essa diria que é coisa de gente do mato.

Mas não podemos cupá-los, vai ver um filme sobre ETs estava em cartaz na cidade, o pessoal ficou impressionado com o filme( coisa de gente do mato) e se eles olhassem um vaga-lume iriam dizer que era uma nave extra-terrestre ou coisa do tipo.



---------------------------------
Yahoo! Mail - Com 250MB de espaço. Abra sua conta!

[As partes desta mensagem que não continham texto foram removidas]



SUBJECT: Re: ninguem é inocente.
FROM: "pubmed2005" <pubmed2005@yahoo.com.br>
TO: ciencialist@yahoogrupos.com.br
DATE: 18/03/2005 20:08


E ainda tem mais...os antigos costumavam envolver o corpo dos mortos
com tecidos, linho, etc...

O fato de o tecido ter 3000 ou anos (ou menos) pode indicar uma prova
arqueológica que os antigos- no caso os judeus(ou hebreus) tinham o
hábito de envolver os corpos dos mortos após a morte - ou para
preserva-lo ou por puro costume

O problemas desses céticos positivistas da internet é que eles
costumavam ver tudo sobre o prisma de Jesus, e que um tal descoberta
por dar esperança aos religiosos que Jesus de fato, existiu.

São uns obcecados e falsos céticos....


--- Em ciencialist@yahoogrupos.com.br, Anderson Almeida da Silveira
<anderson_a5@y...> escreveu
> Bem, mesmo que provem que o tal sudário seja da época de Jesus,
nada garante que ele o tenha usado. Se inventarem uma máquina do
tempo ou coisa parecida aí, sim poderemos dar uma resposta segura.
Pesquisas desse tipo não levam a lugar nenhum mesmo porque ninguém
que está vivo hoje sabe como era o rosto de Jesus.






SUBJECT: Re: [ciencialist] Novo no grupo
FROM: "Alberto Mesquita Filho" <albmesq@uol.com.br>
TO: <ciencialist@yahoogrupos.com.br>
DATE: 18/03/2005 21:12

----- Original Message -----
From: "Anderson Almeida da Silveira"
Sent: Friday, March 18, 2005 7:00 PM
Subject: Re: [ciencialist] Novo no grupo

> Olá Alberto, acho que você encontrou alguém que não vai totalmente contra
> as suas idéias.

Fico feliz com isso.

> Caso algum defensor da "física moderna" termine com sua
> carreira e suas idéias, eu posso continuar elas para você.

Use e abuse das minhas idéias. Nem que seja para derrubá-las decentemente,
pois isso é o mínimo que um cientista deveria esperar de seus semelhantes.
Por outro lado, se você resolver aceitá-las e/ou acrescentar alguma idéia
nova, o único conselho que posso lhe dar é que tome muito cuidado. Nos dias
atuais seria muito difícil alguém conseguir "terminar com a minha carreira",
mesmo porque, e a esse respeito, eu já estou "no fim da picada", obviamente
no bom sentido do que isso possa representar, qual seja: não dependo mais do
academicismo para sobreviver, e posso muito bem "enfrentar os leões que
insistem em me apontar um caminho que todos sabem terminar num precipício".
Quanto a você, e supondo que persista com esse ideal revolucionário, só me
resta desejar que consiga sobreviver frente à "Inquisição dos Tempos
Modernos".

> Faço ainda o 2°
> ano no CEFET-MA, mas aqui há defensores da "física moderna" que já ma
> ameaçaram quando falei sobre suas idéias.

Interessante (se bem que lamentável) esse seu depoimento. Mas é bom que isso
fique registrado, pois tem muita gente aqui na Ciencialist que acha que eu
exagero em minhas críticas ao "academicismo selvagem". E perceba não cheguei
a relatar nem um décimo das desilusões por que já passei. Ora, se alguém é
ameaçado tão somente por falar sobre as minhas idéias... Que mais dizer?!!!

> comecei a ler o seu site agora mais tarde poderei
> fazer algum comentário mais detalhado sobre ele.

Sinta-se à vontade.

[ ]´s
Alberto
http://ecientificocultural.com/indice.htm
Mas indiferentemente a tudo isso, o neutrino tem massa, o elétron não é
uma carga elétrica coulombiana e a Terra se move. E a história se repetirá.



SUBJECT: Re: [ciencialist] Re: ninguem é inocente.
FROM: "Oraculo" <oraculo@atibaia.com.br>
TO: <ciencialist@yahoogrupos.com.br>
DATE: 18/03/2005 21:55

Olá Pubmed

risos..:-) Está confundindo um pouco as coisas..:-) Veja, se um artefato que, a priori, é apresentado como sendo o manto que cobriu o corpo de cristo após sua morte, tem sua data definida para 1200 depois de cristo, isso automaticamente implica que não pode ser verdadeiro. Mas, se a data muda para mais de 2000 anos, se torna possível a alegação.

Não é insinuação, mas conclusões lógicas, decorrentes dos dados apresentados pelas pequisas, qualquer delas.:-) A Igreja é que inicialmente apresentou o caso, o sudário, como reliquia autentica e com as afirmações sobre sua origem e utilização. A mesma igreja que, brincando de esconde-esconde, as vezes permite, as vezes nega, o estudo confiável ao artefato.

As conclusões nada tem a ver com as opiniões ou afirmações da Igreja, ou de qualquer religião.

O estudo de Rogers é valido, como qualquer estudo que use o método cientifico, mas precisa ser confirmado, validado, analisado. As críticas, que são feitas a qualquer alegação cientifica ou não, não pretender ser "contra cristo", mas refletem o natural e necessário rigor do conhecimento científico. Sem confirmação, o estudo de Roger tem apenas validade parcial, enquanto os anteriores tem maior confiabilidade. Mas isso pode mudar, basta que estudos complementares confirmem os dados de Rogers. De novo, nada contra cristo (nem mesmo temos evidencias de que tenha existido, como ter algo contra ele?..:-)

Mas em um ponto temos realmente um problema:

"Pubmed: Voce deve ter me mente que a intenção do segundo estudo é trazer mais
elementos a pesquisa, e nao concluir inexiravelmente que o manto tem
realmente 1300 anos, quando pode muito bem ter muito mais."

Na verdade, o estudo tinha a intenção declarada, desde o início, de demonstrar que o sudario poderia ser de jesus. Seu autor tinha esse objetivo em mente ao preparar o estudo. Não é preconceito da comunidade cientifica, mas um viés que precisa ser considerado, principalmente na produção de estudos de confirmação. Que devem ser feitos de preferencia por pesquisadores sem esse objetivo inicial. Não apenas com relação a este estudo sobre o sudário, mas em relação a todos os estudos e experimentos científicos. Foi assim que a fusão a frio foi refutada, quando, sem o viés inicial, novos estudos não encontraram o mesmo resultado.

Pubmed, todo estudo é sempre olhado com cuidado e crítica quando surge. É parte do rigor científico, não perseguição anti-cristã. Acontece com o sudário e acontece com qualquer alegação científica. É preciso que o rigor filtre e confirme (ou refute) o estudo e os dados apra que estes adquiram confiabilidade. Sem isso, é temerário concluir apenas sobre um único estudo. Ainda mais quando diversos outros discordam de seus resultados. Quando o primeiro estudo com carbono-14 foi apresentado, a mesma crítica, o mesmo cuidado, a mesma consideração que seriam precisos mais estudos de confirmação foi feita (inclusive pela igreja). E eles foram feitos e confirmaram os dados do primeiro.

Cientistas não se apegam ao dogma antigo, como afirmou, eles apenas são muito cuidadosos com novas descobertas, porque aprenderam que boa parte delas não se confirma. Com esse cuidado, que chama de apego, evitam erros e enganos, e, se o que foi alegado for real, acabará sendo confirmado (como a teoria da deriva dos continentes, aceita apenas quando evidencias de sua realidade puderam ser produzidas).

Um abraço.

Homero




----- Original Message -----
From: pubmed2005
To: ciencialist@yahoogrupos.com.br
Sent: Friday, March 18, 2005 6:12 PM
Subject: [ciencialist] Re: ninguem é inocente.



Ah é, é assim que voce vê? Quer dizer que se analizo que um artefato
arqueológico tem 2500 anos isso insinua que deve ter passado pelas
maos de Cristo?

Se a Igreja utiliza isso ou mão, nao tem nada ver com Rogers, é um
fato a parte - estão fazendo de uma descoberta cientifica uma
validação de seus argumentos religiosos

Mas isso é outra história....

-- Em ciencialist@yahoogrupos.com.br, "Oraculo" <oraculo@a...>
escreveu
> Olá Pubmed
>
> Não disse mesmo, nem nós estamos dizendo..:-) Mas é claro que se a
data for de 1200 depois de cristo, não pode ter sido usado por jesus,
mas se for de mais de 3000 anos atras (veja, 3000 anos significa 2000
depois de cristo e 1000 antes, que é a margem que ele encontrou na
pesquisa), ele mantém a esperança..:-) De uma pesquisada nos sites
catolicos, e veja como a argumentação sobre as
evidencais "científicas" sobre o sudário "provam" que ele foi usado
por jesus..:-).
>
> Como eu disse antes, esses estudos apenas determinam uma data, nada
indicam a respeito de ter sido usado por quem quer que seja..:-) Mas,
o que incomoda aos crentes é a data recente, que impede seu uso por
jesus. O que, se pensar bem, é uma demonstração de falta de fé, uma
vez que nada impede que o ser milagroso tenha modificado o carbono do
pano, só para tirar um sarro da cara dos cientistas..:-)
>
> Um abraço.
>
> Homero
>
>
> ----- Original Message -----
> From: pubmed2005
> To: ciencialist@yahoogrupos.com.br
> Sent: Friday, March 18, 2005 5:48 PM
> Subject: [ciencialist] Re: ninguem é inocente.
>
>
>
> Mas o Rogers nao afirmou que o manto era de Jesus . Apenas
realizou
> testes químicos e nao encontrando vanila no resto do manto,
deduziu
> cientificamente que o manto "deve" tem mais idade do que o
previsto.
>
> Não sei de onde voces tiraram que o Rogers disse que o manto
tinha
> aproximadamente 3000 anos e portanto deveria ser de Jesus.
>
> Ninguém disse isso...
>
>
> --- Em ciencialist@yahoogrupos.com.br, "Oraculo" <oraculo@a...>
> escreveu
> > Olá Pubmed
> >
> > Bem mais escorregadio é o comportamento da detentora do objeto
em
> questão..:-) Afinal, para ser possível uma análise precisa e
> confiável de um artefato arqueológico, de forma cientifica e
> racional, é preciso o maior número de testes possível, total
> colaboração do proprietário, acesso a todos os componentes, e a
todas
> as partes do objeto, etc.
> >
> > E isso, é fácil perceber, nào será obtido tão cedo pelos
> cientistas..:-)
> >
> > Assim, se a hipotese cientifica tem problemas, eles não são nem
de
> perto tão grandes quanto a hipótese milagrosa..:-) Com o estudo
de
> apenas alguns fios e pedaços mínimos de tecido foi possível
levantar
> diversas hipoteses (que só podem ser comprovadas se a igreja
permitir
> o exame minuciosos e completo do artefato), mas a hipótese
milagrosa
> se baseia em nada mais que a afirmação da igreja e em lendas
> diversas. Como a impressão da imagem pelo "calor de uma explosão
> nuclear enquanto o corpo flutuava sobre o pano" e outras mais
malucas.
> >
> > O problema é sempre esse, não se pode usar do conhecimento
> cientifico para analisar o objeto, apenas um contato mínimo e
> misterioso com partes minimas e em condições menos que ideais.
> Pesquisadores não iriam destruir o artefato, na verdade, quando é
> necessário descobrir formas de proteger reliquias sagradas, a
igreja
> sempre pede a ajuda de cientistas. Assim, porque não se permite
> estudar a fundo o sudario, e descobrir todo o possível sobre ele,
com
> confiabilidade?
> >
> > O que o teste carbono efetuado determinou não foi nada mais que
uma
> data. Para um milagre, nem isso importa, já que a divindade
milagrosa
> poderia facilmente ter modificado a taxa de carbono do pano ao
> imprimir sua imagem mágicamente, e não há nada que possa refutar
essa
> afirmação..:-) Por que o medo?
> >
> > E o mesmo sobre o novo estudo, ainda não confirmado por
> pesquisadores independentes (coisa que o primeiro já foi mais de
uma
> vez), ele apenas modifica uma data possível (e com enorme
amplitude,
> de 1300 a 3000 anos). Não implica em que é real ou comprova que
foi
> usado em jesus. Aliás, a argumentação que sendo de 3000 anos, ele
> poderia ter sido usado na época de jesus, guardado até o momento
de
> ser necessário, pode ser usada da mesma forma se for datado com,
> digamos, 2000 anos. Sim, pode ter 2000 anos, mas pode ter sido
> guardado até o século 12 e só então usado para criar a fraude.
> >
> > Para saber com mais certeza e um mínimo de confiabilidade,
basta
> que a igreja permita um estudo profundo e cuidadoso por
cientistas.
> Mas, eu não apostaria nisso..:-)
> >
> > Um abraço.
> >
> > Homero
> >
> >
> >
> >
> >
> > ----- Original Message -----
> > From: pubmed2005
> > To: ciencialist@yahoogrupos.com.br
> > Sent: Friday, March 18, 2005 5:26 PM
> > Subject: [ciencialist] Re: ninguem é inocente.
> >
> >
> >
> > O correto seria refazer todos os testes a respeito do
Sudário,
> > inclusive a do carbono-14 em diferentes partes do tecido,
senão
> tb se
> > poderia argumentar que o teste nao foi completo e tb
apresenta
> > falhas . Depois, se realizar os testes químicos para comparar.
> >
> > O parecer atual da ciencia ainda continua meio escorregativo -

> pois a
> > parte analisada do carbono 14 pode ter sido muito bem a
remendada
> que
> > possuía vinila(uma sibstancia que nao resiste ao
envelhecimento)
> >
> > Acho muito pertinente a nova dewcoberta do Rogers e em se
> tratando de
> > ciencia, fico com ela ate o momento.
> >
> >
> >
> > --- Em ciencialist@yahoogrupos.com.br, TARCISIO BORGES
> <tbs97@f...>
> > escreveu
> > > http://brazil.skepdic.com/sudario.html
> > >
> > > Encontrei algo aqui.
> > >
> > > A metodologia do Sr. Rogers parece muito suspeita.
> > >
> > > []s
> > > TARCISIO BORGES
> > > tbs97@f...
> > >
> > > On Fri, 18 Mar 2005, pubmed2005 wrote:
> > > > La vai
> > > >
> > > > http://diariodigital.sapo.pt/news_history.asp?
> > > > section_id=13&id_news=156054
> >
> >
> >
> >
> >
> > ##### ##### #####
> >
> > Para saber mais visite
> > http://www.ciencialist.hpg.ig.com.br
> >
> >
> > ##### ##### ##### #####
> >
> >
> > Yahoo! Grupos, um serviço oferecido por:
> >
> >
> >
> >
> >
> >
> >
> > ----------------------------------------------------------------
----
> ----------
> > Links do Yahoo! Grupos
> >
> > a.. Para visitar o site do seu grupo na web, acesse:
> > http://br.groups.yahoo.com/group/ciencialist/
> >
> > b.. Para sair deste grupo, envie um e-mail para:
> > ciencialist-unsubscribe@yahoogrupos.com.br
> >
> > c.. O uso que você faz do Yahoo! Grupos está sujeito aos
Termos
> do Serviço do Yahoo!.
> >
> >
> >
> > [As partes desta mensagem que não continham texto foram
removidas]
>
>
>
>
>
> ##### ##### #####
>
> Para saber mais visite
> http://www.ciencialist.hpg.ig.com.br
>
>
> ##### ##### ##### #####
>
>
> Yahoo! Grupos, um serviço oferecido por:
>
>
>
>
>
>
>
> --------------------------------------------------------------------
----------
> Links do Yahoo! Grupos
>
> a.. Para visitar o site do seu grupo na web, acesse:
> http://br.groups.yahoo.com/group/ciencialist/
>
> b.. Para sair deste grupo, envie um e-mail para:
> ciencialist-unsubscribe@yahoogrupos.com.br
>
> c.. O uso que você faz do Yahoo! Grupos está sujeito aos Termos
do Serviço do Yahoo!.
>
>
>
> [As partes desta mensagem que não continham texto foram removidas]





##### ##### #####

Para saber mais visite
http://www.ciencialist.hpg.ig.com.br


##### ##### ##### #####


Yahoo! Grupos, um serviço oferecido por:







------------------------------------------------------------------------------
Links do Yahoo! Grupos

a.. Para visitar o site do seu grupo na web, acesse:
http://br.groups.yahoo.com/group/ciencialist/

b.. Para sair deste grupo, envie um e-mail para:
ciencialist-unsubscribe@yahoogrupos.com.br

c.. O uso que você faz do Yahoo! Grupos está sujeito aos Termos do Serviço do Yahoo!.



[As partes desta mensagem que não continham texto foram removidas]



SUBJECT: Re: [ciencialist] Re: ninguem é inocente.
FROM: "Oraculo" <oraculo@atibaia.com.br>
TO: <ciencialist@yahoogrupos.com.br>
DATE: 18/03/2005 22:02

Olá Pubmed

Bem, é preciso cuidado com os argumentos, eles podem se virar contra você..:-)

Sim, os antigos costumavam fazer isso. Mas se vai usar os hábitos dos antigos para validar a possibilidade que defende, tem de fazer o serviço completo. Por exemplo, o costume na época de jesus era usar diversos panos, e não apenas um, além de um pano pequeno, especial para cobrir apenas o rosto, o que impediria a formação da imagem que está no sudário.

Ou seja, se os costumes antigos servem para defender sua tese, também servem para refuta-la. Outro exemplo, o tipo de tecido existente na época era diferente, com trama simples, e não a complexa trama do sudário, que só se tornaria comum quase mil anos depois de jesus.

Além disso, é evidente que, se o sudário é falso, seu falsificador deve ter estudado bem os critérios para construir o mesmo..:-)

Não sou obcecado nem falso cético, sou um cético de verdade usando a ferramenta da razão para pensar sobre as alegações e afirmativas que são apresentadas..:-) Tem todo o direito de, se for possível, refutar meus argumentos, mas é uma falácia "ad hominem" apenas me acusar de ser falso cético obcecado..:-)

Um abraço.

Homero




----- Original Message -----
From: pubmed2005
To: ciencialist@yahoogrupos.com.br
Sent: Friday, March 18, 2005 8:08 PM
Subject: [ciencialist] Re: ninguem é inocente.



E ainda tem mais...os antigos costumavam envolver o corpo dos mortos
com tecidos, linho, etc...

O fato de o tecido ter 3000 ou anos (ou menos) pode indicar uma prova
arqueológica que os antigos- no caso os judeus(ou hebreus) tinham o
hábito de envolver os corpos dos mortos após a morte - ou para
preserva-lo ou por puro costume

O problemas desses céticos positivistas da internet é que eles
costumavam ver tudo sobre o prisma de Jesus, e que um tal descoberta
por dar esperança aos religiosos que Jesus de fato, existiu.

São uns obcecados e falsos céticos....


--- Em ciencialist@yahoogrupos.com.br, Anderson Almeida da Silveira
<anderson_a5@y...> escreveu
> Bem, mesmo que provem que o tal sudário seja da época de Jesus,
nada garante que ele o tenha usado. Se inventarem uma máquina do
tempo ou coisa parecida aí, sim poderemos dar uma resposta segura.
Pesquisas desse tipo não levam a lugar nenhum mesmo porque ninguém
que está vivo hoje sabe como era o rosto de Jesus.






##### ##### #####

Para saber mais visite
http://www.ciencialist.hpg.ig.com.br


##### ##### ##### #####


Yahoo! Grupos, um serviço oferecido por:

São Paulo Rio de Janeiro Curitiba Porto Alegre Belo Horizonte Brasília




------------------------------------------------------------------------------
Links do Yahoo! Grupos

a.. Para visitar o site do seu grupo na web, acesse:
http://br.groups.yahoo.com/group/ciencialist/

b.. Para sair deste grupo, envie um e-mail para:
ciencialist-unsubscribe@yahoogrupos.com.br

c.. O uso que você faz do Yahoo! Grupos está sujeito aos Termos do Serviço do Yahoo!.



[As partes desta mensagem que não continham texto foram removidas]



SUBJECT: Re: [ciencialist] Re: ninguem é inocente.
FROM: "Alberto Mesquita Filho" <albmesq@uol.com.br>
TO: <ciencialist@yahoogrupos.com.br>
DATE: 18/03/2005 22:18

----- Original Message -----
From: "Oraculo"
Sent: Friday, March 18, 2005 10:02 PM
Subject: Re: [ciencialist] Re: ninguem é inocente.

> sou um cético de verdade usando a ferramenta da razão para pensar sobre as
> alegações e afirmativas que são apresentadas..:-)

Existem céticos de verdade? Afinal, o que é ceticismo? Ou então: O que é
verdade?

Não precisa responder. Eu só queria perguntar. ;-))

[ ]´s
Alberto
http://ecientificocultural.com/indice.htm
Mas indiferentemente a tudo isso, o neutrino tem massa, o elétron não é
uma carga elétrica coulombiana e a Terra se move. E a história se repetirá.



SUBJECT: Re: ninguem é inocente.
FROM: "pubmed2005" <pubmed2005@yahoo.com.br>
TO: ciencialist@yahoogrupos.com.br
DATE: 18/03/2005 22:27


Eu só queria separar alhos de bugalhos. Se a carapuça serviu...

O estudo de Rogers nao tem nada haver com Jesus ou qualquer treco que
valide insinuações religiosas, isso é que tem se deixar bem claro. É
um estudo científico.

E seus céticos querem refutar podem muito bem faze-lo. Assim como
qualquer um tb poderia dizer que o trabalho do carbono 14 tb foi
insuficiente, pois nao se tem um acesso direto e satisfatório ao manto
(creio que só seje por isso). Não se teve tempo de analisar toa a
parte do tecido, verificar o nível de sibstancias no tecido, etc e tal

Isso nao invalida o fato do manto ser um artefato arqueológico mesmo
nao se comprove que foi de Cristo. E isto tb não foi a intenção de
Rogers ao fazer a pesquisa.

Um pessoa para ser cetica tem que no mínimo ter algum conhecimento
científico para nao se dar a margem a bobagens. O que vejo por ai é o
cultivo de um ceticismo sem nenhum base de conhecimento para tal.
Ceticismo nao é nilismo, nao é cientistifista, nem tampouco é
profissão.



--- Em ciencialist@yahoogrupos.com.br, "Oraculo" <oraculo@a...>
escreveu
> Olá Pubmed

>
> Não sou obcecado nem falso cético, sou um cético de verdade usando
a ferramenta da razão para pensar sobre as alegações e afirmativas
que são apresentadas..:-) Tem todo o direito de, se for possível,
refutar meus argumentos, mas é uma falácia "ad hominem" apenas me
acusar de ser falso cético obcecado..:-)
>
> Um abraço.
>
> Homero
>
>
>
>
> ----- Original Message -----
> From: pubmed2005
> To: ciencialist@yahoogrupos.com.br
> Sent: Friday, March 18, 2005 8:08 PM
> Subject: [ciencialist] Re: ninguem é inocente.
>
>
>
> E ainda tem mais...os antigos costumavam envolver o corpo dos
mortos
> com tecidos, linho, etc...
>
> O fato de o tecido ter 3000 ou anos (ou menos) pode indicar uma
prova
> arqueológica que os antigos- no caso os judeus(ou hebreus) tinham
o
> hábito de envolver os corpos dos mortos após a morte - ou para
> preserva-lo ou por puro costume
>
> O problemas desses céticos positivistas da internet é que eles
> costumavam ver tudo sobre o prisma de Jesus, e que um tal
descoberta
> por dar esperança aos religiosos que Jesus de fato, existiu.
>
> São uns obcecados e falsos céticos....
>
>
> --- Em ciencialist@yahoogrupos.com.br, Anderson Almeida da
Silveira
> <anderson_a5@y...> escreveu
> > Bem, mesmo que provem que o tal sudário seja da época de Jesus,
> nada garante que ele o tenha usado. Se inventarem uma máquina do
> tempo ou coisa parecida aí, sim poderemos dar uma resposta
segura.
> Pesquisas desse tipo não levam a lugar nenhum mesmo porque
ninguém
> que está vivo hoje sabe como era o rosto de Jesus.
>
>
>
>
>
>
> ##### ##### #####
>
> Para saber mais visite
> http://www.ciencialist.hpg.ig.com.br
>
>
> ##### ##### ##### #####
>
>
> Yahoo! Grupos, um serviço oferecido por:
>
> São Paulo Rio de Janeiro Curitiba Porto Alegre
Belo Horizonte Brasília
>
>
>
>
> --------------------------------------------------------------------
----------
> Links do Yahoo! Grupos
>
> a.. Para visitar o site do seu grupo na web, acesse:
> http://br.groups.yahoo.com/group/ciencialist/
>
> b.. Para sair deste grupo, envie um e-mail para:
> ciencialist-unsubscribe@yahoogrupos.com.br
>
> c.. O uso que você faz do Yahoo! Grupos está sujeito aos Termos
do Serviço do Yahoo!.
>
>
>
> [As partes desta mensagem que não continham texto foram removidas]





SUBJECT: Re: ninguem é inocente.
FROM: "pubmed2005" <pubmed2005@yahoo.com.br>
TO: ciencialist@yahoogrupos.com.br
DATE: 18/03/2005 22:35


Põe uma coisa na sua cabeça
Ninguem aqui disse (pelo menos eu não disse) que o santo sudário tem
que ser o manto de Jesus.
Pode ter sido um manto qualquer que cobriu um corpo qualquer. E mesmo
que tenha sido uma falsificação, isso pode confirmar que os antigos
tinham o hábito de enrolarem os corpos em tecidos....pode ser até que
faziam isso com pessoas "especiais" que representavam um determinado
contexto daquela época

É dificil entender?

Ai, ai, ai...tem que ter paciencia



--- Em ciencialist@yahoogrupos.com.br, "Oraculo" <oraculo@a...>
escreveu
> Olá Pubmed
>
> risos..:-) Está confundindo um pouco as coisas..:-) Veja, se um
artefato que, a priori, é apresentado como sendo o manto que cobriu o
corpo de cristo após sua morte, tem sua data definida para 1200
depois de cristo, isso automaticamente implica que não pode ser
verdadeiro. Mas, se a data muda para mais de 2000 anos, se torna
possível a alegação.
]





SUBJECT: Re: ninguem é inocente.
FROM: "pubmed2005" <pubmed2005@yahoo.com.br>
TO: ciencialist@yahoogrupos.com.br
DATE: 18/03/2005 22:50


Voce esqueceu de completar sua retórica perguntando o que é
ferramenta da razão para pensar...
Será que é o cérebro?...ah esqueci de dizer que só os "céticos tem
cérebros" e "que só eles podem pensar"


--- Em ciencialist@yahoogrupos.com.br, "Alberto Mesquita Filho"
<albmesq@u...> escreveu
> ----- Original Message -----
> From: "Oraculo"
> Sent: Friday, March 18, 2005 10:02 PM
> Subject: Re: [ciencialist] Re: ninguem é inocente.
>
> > sou um cético de verdade usando a ferramenta da razão para pensar
sobre as
> > alegações e afirmativas que são apresentadas..:-)
>
> Existem céticos de verdade? Afinal, o que é ceticismo? Ou então: O
que é
> verdade?
>
> Não precisa responder. Eu só queria perguntar. ;-))






SUBJECT: Re: [ciencialist] Re: ninguem é inocente.
FROM: "Oraculo" <oraculo@atibaia.com.br>
TO: <ciencialist@yahoogrupos.com.br>
DATE: 18/03/2005 22:54

Olá Alberto

he he he..:-) Sempre provocativo..:-) Acho que você é um excelente exemplo de um cético..:-)

Mas, para melhor esclarecer a questão do ceticismo, talvez seja interessante dar uma lida neste artigo sobre a ferramenta do ceticismo, é bem legal ..:-)

O Ceticismo como técnica de auto-defesa intelectual
http://www.ceticismoaberto.com/ceticismo/cetadi.htm

Um abraço.

Homero
----- Original Message -----
From: Alberto Mesquita Filho
To: ciencialist@yahoogrupos.com.br
Sent: Friday, March 18, 2005 10:18 PM
Subject: Re: [ciencialist] Re: ninguem é inocente.


----- Original Message -----
From: "Oraculo"
Sent: Friday, March 18, 2005 10:02 PM
Subject: Re: [ciencialist] Re: ninguem é inocente.

> sou um cético de verdade usando a ferramenta da razão para pensar sobre as
> alegações e afirmativas que são apresentadas..:-)

Existem céticos de verdade? Afinal, o que é ceticismo? Ou então: O que é
verdade?

Não precisa responder. Eu só queria perguntar. ;-))

[ ]´s
Alberto
http://ecientificocultural.com/indice.htm
Mas indiferentemente a tudo isso, o neutrino tem massa, o elétron não é
uma carga elétrica coulombiana e a Terra se move. E a história se repetirá.



##### ##### #####

Para saber mais visite
http://www.ciencialist.hpg.ig.com.br


##### ##### ##### #####


Yahoo! Grupos, um serviço oferecido por:







------------------------------------------------------------------------------
Links do Yahoo! Grupos

a.. Para visitar o site do seu grupo na web, acesse:
http://br.groups.yahoo.com/group/ciencialist/

b.. Para sair deste grupo, envie um e-mail para:
ciencialist-unsubscribe@yahoogrupos.com.br

c.. O uso que você faz do Yahoo! Grupos está sujeito aos Termos do Serviço do Yahoo!.



[As partes desta mensagem que não continham texto foram removidas]



SUBJECT: Re: ninguem é inocente.
FROM: "pubmed2005" <pubmed2005@yahoo.com.br>
TO: ciencialist@yahoogrupos.com.br
DATE: 18/03/2005 23:07


karatê cerebral? heheheh

Foi só uma brincadeira.

--- Em ciencialist@yahoogrupos.com.br, "Oraculo" <oraculo@a...>
escreveu
> Olá Alberto
>
> O Ceticismo como técnica de auto-defesa intelectual
> http://www.ceticismoaberto.com/ceticismo/cetadi.htm
>
> Um abraço.
>
> Homero
> ----- Original Message -----
> From: Alberto Mesquita Filho
> To: ciencialist@yahoogrupos.com.br
> Sent: Friday, March 18, 2005 10:18 PM
> Subject: Re: [ciencialist] Re: ninguem é inocente.
>
>
> ----- Original Message -----
> From: "Oraculo"
> Sent: Friday, March 18, 2005 10:02 PM
> Subject: Re: [ciencialist] Re: ninguem é inocente.
>
> > sou um cético de verdade usando a ferramenta da razão para
pensar sobre as
> > alegações e afirmativas que são apresentadas..:-)
>
> Existem céticos de verdade? Afinal, o que é ceticismo? Ou então:
O que é
> verdade?
>
> Não precisa responder. Eu só queria perguntar. ;-))
>
> [ ]´s
> Alberto
> http://ecientificocultural.com/indice.htm
> Mas indiferentemente a tudo isso, o neutrino tem massa, o elétron
não é
> uma carga elétrica coulombiana e a Terra se move. E a história se
repetirá.
>
>
>
> ##### ##### #####
>
> Para saber mais visite
> http://www.ciencialist.hpg.ig.com.br
>
>
> ##### ##### ##### #####
>
>
> Yahoo! Grupos, um serviço oferecido por:
>
>
>
>
>
>
>
> --------------------------------------------------------------------
----------
> Links do Yahoo! Grupos
>
> a.. Para visitar o site do seu grupo na web, acesse:
> http://br.groups.yahoo.com/group/ciencialist/
>
> b.. Para sair deste grupo, envie um e-mail para:
> ciencialist-unsubscribe@yahoogrupos.com.br
>
> c.. O uso que você faz do Yahoo! Grupos está sujeito aos Termos
do Serviço do Yahoo!.
>
>
>
> [As partes desta mensagem que não continham texto foram removidas]





SUBJECT: Re: [ciencialist] Re: ninguem é inocente.
FROM: "Oraculo" <oraculo@atibaia.com.br>
TO: <ciencialist@yahoogrupos.com.br>
DATE: 18/03/2005 23:23

Olá Pubmed

Bem, em algo tem razão, tem que ter paciencia..:-) É disso que é feita a ciência, paciência, esforço, análise, tempo, cuidado, etc..:-) Não a perca por tão pouco..:-)

"Ninguém disse" é exagero, a igreja católica, dona do manto e sua maior defensora, disse. Mudou de ideia recentemente, afirmando que não importa, que ele apenas reforça a fé dos catolicos, que é um simbolo, mas foi quase obrigada a fazer essa colocação..:-) Inicialmente, todo o debate se resumia a ter ou não pertencido a jesus, embrulhado seu corpo. E mesmo hoje, é essa a força do sudário, fornecer evidencias empiricas de uma crença ou superstição ancestral (procure no Google por santro sudario, e verá que tenho razão..:-).

Assim, é totalmente pertinente discutir as implicações de cada estudo apresentado a seu respeito.

De todo modo, não, não é dificil de entender, embora isso não signifique aceitar seu argumento. Minha posição ainda é a mesma, mantenho a pertinencia de debater no mesmo foco que o apresentador do objeto, a igreja, deu a ele: ter pertencido a jesus. Se tem apenas 640 anos, não pode ter sido, se tem mais de 2000, pode ter sido.

Um abraço.

Homero

----- Original Message -----
From: pubmed2005
To: ciencialist@yahoogrupos.com.br
Sent: Friday, March 18, 2005 10:35 PM
Subject: [ciencialist] Re: ninguem é inocente.



Põe uma coisa na sua cabeça
Ninguem aqui disse (pelo menos eu não disse) que o santo sudário tem
que ser o manto de Jesus.
Pode ter sido um manto qualquer que cobriu um corpo qualquer. E mesmo
que tenha sido uma falsificação, isso pode confirmar que os antigos
tinham o hábito de enrolarem os corpos em tecidos....pode ser até que
faziam isso com pessoas "especiais" que representavam um determinado
contexto daquela época

É dificil entender?

Ai, ai, ai...tem que ter paciencia



--- Em ciencialist@yahoogrupos.com.br, "Oraculo" <oraculo@a...>
escreveu
> Olá Pubmed
>
> risos..:-) Está confundindo um pouco as coisas..:-) Veja, se um
artefato que, a priori, é apresentado como sendo o manto que cobriu o
corpo de cristo após sua morte, tem sua data definida para 1200
depois de cristo, isso automaticamente implica que não pode ser
verdadeiro. Mas, se a data muda para mais de 2000 anos, se torna
possível a alegação.
]





##### ##### #####

Para saber mais visite
http://www.ciencialist.hpg.ig.com.br


##### ##### ##### #####


Yahoo! Grupos, um serviço oferecido por:







------------------------------------------------------------------------------
Links do Yahoo! Grupos

a.. Para visitar o site do seu grupo na web, acesse:
http://br.groups.yahoo.com/group/ciencialist/

b.. Para sair deste grupo, envie um e-mail para:
ciencialist-unsubscribe@yahoogrupos.com.br

c.. O uso que você faz do Yahoo! Grupos está sujeito aos Termos do Serviço do Yahoo!.



[As partes desta mensagem que não continham texto foram removidas]



SUBJECT: Re: [ciencialist] Re: ninguem é inocente.
FROM: "Oraculo" <oraculo@atibaia.com.br>
TO: <ciencialist@yahoogrupos.com.br>
DATE: 18/03/2005 23:27

Olá Pubmed

Desculpe, onde foi que eu, ou qualquer outro, disse ou escreveu que "só ceticos tem cérebro" ou que "só ceticos podem pensar"? Não é, claro, um argumento, apenas um ataque do tipo "ad hominem", mas gostaria de saber se algo que eu escrevi tenha dado essa ideia ou poderia ser interpretado dessa forma..:-)

Em todo caso, se eu achasse isso, não estaria debatendo com você e com a lista, com cuidado e educação, expondo meus argumentos e lendo atentamente os seus..:-) É um debate, não uma guerra, e não pretendo faze-lo mudar de ideia ou aceitar o que quer que seja. Apenas apresento meu ponto de vista, meus argumentos e minhas evidencias, esperando que essa troca traga algo de útil a nós dois.

Um abraço.

Homero

----- Original Message -----
From: pubmed2005
To: ciencialist@yahoogrupos.com.br
Sent: Friday, March 18, 2005 10:50 PM
Subject: [ciencialist] Re: ninguem é inocente.



Voce esqueceu de completar sua retórica perguntando o que é
ferramenta da razão para pensar...
Será que é o cérebro?...ah esqueci de dizer que só os "céticos tem
cérebros" e "que só eles podem pensar"


--- Em ciencialist@yahoogrupos.com.br, "Alberto Mesquita Filho"
<albmesq@u...> escreveu
> ----- Original Message -----
> From: "Oraculo"
> Sent: Friday, March 18, 2005 10:02 PM
> Subject: Re: [ciencialist] Re: ninguem é inocente.
>
> > sou um cético de verdade usando a ferramenta da razão para pensar
sobre as
> > alegações e afirmativas que são apresentadas..:-)
>
> Existem céticos de verdade? Afinal, o que é ceticismo? Ou então: O
que é
> verdade?
>
> Não precisa responder. Eu só queria perguntar. ;-))






##### ##### #####

Para saber mais visite
http://www.ciencialist.hpg.ig.com.br


##### ##### ##### #####


Yahoo! Grupos, um serviço oferecido por:

São Paulo Rio de Janeiro Curitiba Porto Alegre Belo Horizonte Brasília




------------------------------------------------------------------------------
Links do Yahoo! Grupos

a.. Para visitar o site do seu grupo na web, acesse:
http://br.groups.yahoo.com/group/ciencialist/

b.. Para sair deste grupo, envie um e-mail para:
ciencialist-unsubscribe@yahoogrupos.com.br

c.. O uso que você faz do Yahoo! Grupos está sujeito aos Termos do Serviço do Yahoo!.



[As partes desta mensagem que não continham texto foram removidas]



SUBJECT: O Ceticismo como técnica de auto-defesa intelectual (era: ninguem é inocente.)
FROM: "Oraculo" <oraculo@atibaia.com.br>
TO: <ciencialist@yahoogrupos.com.br>
DATE: 18/03/2005 23:29

Olá Pubmed

risos..:-) A ideia é mais ou menos essa..:-) Permitir que tenhamos uma ferramenta que faça pela mente, argumentos e escolhas, o mesmo que as técnicas de auto-defesa fazem pelo corpo..:-) É um artigo interessante, me diga o que achou dele..:-)

Um abraço.

Homero


----- Original Message -----
From: pubmed2005
To: ciencialist@yahoogrupos.com.br
Sent: Friday, March 18, 2005 11:07 PM
Subject: [ciencialist] Re: ninguem é inocente.



karatê cerebral? heheheh

Foi só uma brincadeira.

--- Em ciencialist@yahoogrupos.com.br, "Oraculo" <oraculo@a...>
escreveu
> Olá Alberto
>
> O Ceticismo como técnica de auto-defesa intelectual
> http://www.ceticismoaberto.com/ceticismo/cetadi.htm
>
> Um abraço.
>
> Homero
> ----- Original Message -----
> From: Alberto Mesquita Filho
> To: ciencialist@yahoogrupos.com.br
> Sent: Friday, March 18, 2005 10:18 PM
> Subject: Re: [ciencialist] Re: ninguem é inocente.
>
>
> ----- Original Message -----
> From: "Oraculo"
> Sent: Friday, March 18, 2005 10:02 PM
> Subject: Re: [ciencialist] Re: ninguem é inocente.
>
> > sou um cético de verdade usando a ferramenta da razão para
pensar sobre as
> > alegações e afirmativas que são apresentadas..:-)
>
> Existem céticos de verdade? Afinal, o que é ceticismo? Ou então:
O que é
> verdade?
>
> Não precisa responder. Eu só queria perguntar. ;-))
>
> [ ]´s
> Alberto
> http://ecientificocultural.com/indice.htm
> Mas indiferentemente a tudo isso, o neutrino tem massa, o elétron
não é
> uma carga elétrica coulombiana e a Terra se move. E a história se
repetirá.
>
>
>
> ##### ##### #####
>
> Para saber mais visite
> http://www.ciencialist.hpg.ig.com.br
>
>
> ##### ##### ##### #####
>
>
> Yahoo! Grupos, um serviço oferecido por:
>
>
>
>
>
>
>
> --------------------------------------------------------------------
----------
> Links do Yahoo! Grupos
>
> a.. Para visitar o site do seu grupo na web, acesse:
> http://br.groups.yahoo.com/group/ciencialist/
>
> b.. Para sair deste grupo, envie um e-mail para:
> ciencialist-unsubscribe@yahoogrupos.com.br
>
> c.. O uso que você faz do Yahoo! Grupos está sujeito aos Termos
do Serviço do Yahoo!.
>
>
>
> [As partes desta mensagem que não continham texto foram removidas]





##### ##### #####

Para saber mais visite
http://www.ciencialist.hpg.ig.com.br


##### ##### ##### #####


Yahoo! Grupos, um serviço oferecido por:

São Paulo Rio de Janeiro Curitiba Porto Alegre Belo Horizonte Brasília




------------------------------------------------------------------------------
Links do Yahoo! Grupos

a.. Para visitar o site do seu grupo na web, acesse:
http://br.groups.yahoo.com/group/ciencialist/

b.. Para sair deste grupo, envie um e-mail para:
ciencialist-unsubscribe@yahoogrupos.com.br

c.. O uso que você faz do Yahoo! Grupos está sujeito aos Termos do Serviço do Yahoo!.



[As partes desta mensagem que não continham texto foram removidas]



SUBJECT: Re: O Ceticismo como técnica de auto-defesa intelectual (era: ninguem é inocente.)
FROM: "pubmed2005" <pubmed2005@yahoo.com.br>
TO: ciencialist@yahoogrupos.com.br
DATE: 18/03/2005 23:42


O inicio do texto até que é bom...mas depois vai virando uma lavagem
cerebral danada...hehehe
Ele poderia simplesmente dizer que o ceticismo é um
questionamento...e nao uma posição afirmativa, nem detentora da
verdade

Nao me leve a mal por favor...

--- Em ciencialist@yahoogrupos.com.br, "Oraculo" <oraculo@a...>
escreveu
> Olá Pubmed
>
> risos..:-) A ideia é mais ou menos essa..:-) Permitir que tenhamos
uma ferramenta que faça pela mente, argumentos e escolhas, o mesmo
que as técnicas de auto-defesa fazem pelo corpo..:-) É um artigo
interessante, me diga o que achou dele..:-)
>
> Um abraço.
>
> Homero






SUBJECT: Re: ninguem é inocente.
FROM: "pubmed2005" <pubmed2005@yahoo.com.br>
TO: ciencialist@yahoogrupos.com.br
DATE: 18/03/2005 23:49


Que eu me lembre, estávamos falando do santo sudário, e ninguem na
lista havia feito afirmações de que esse manto cobria o manto de
jesus...se os católicos e os céticos fazem é outro esquema.

Portanto, nao vejo sentido nesta discussão e nos seus argumentos que
insiste na mesma coisa. A única coisa lógica que tenho a fazer é
evitar de discutir o que eu nao disse.

E ainda mais as pesquisas giram em torno da idade do tecido, do que é
feito, as substancias que ele contém, etc e tal. Se é uma
falsificação , ela pode ser uma falsificação antiga - bem mais antiga
que pensavamos ou apenas um tecido que foi remexido e remendado
através do tempo por pertecencer a algum importante(mesmo que tenha
sido um rei ou algum monarca do reino antigo, ou até mesmo a alguem
comum) - isso tem valor histórico e arqueológico

--- Em ciencialist@yahoogrupos.com.br, "Oraculo" <oraculo@a...>
escreveu
> Olá Pubmed
>
> Bem, em algo tem razão, tem que ter paciencia..:-) É disso que é
feita a ciência, paciência, esforço, análise, tempo, cuidado, etc..:-
) Não a perca por tão pouco..:-)
>
> "Ninguém disse" é exagero, a igreja católica, dona do manto e sua
maior defensora, disse. Mudou de ideia recentemente, afirmando que
não importa, que ele apenas reforça a fé dos catolicos, que é um
simbolo, mas foi quase obrigada a fazer essa colocação..:-)
Inicialmente, todo o debate se resumia a ter ou não pertencido a
jesus, embrulhado seu corpo. E mesmo hoje, é essa a força do sudário,
fornecer evidencias empiricas de uma crença ou superstição ancestral
(procure no Google por santro sudario, e verá que tenho razão..:-).
>
> Assim, é totalmente pertinente discutir as implicações de cada
estudo apresentado a seu respeito.
>
> De todo modo, não, não é dificil de entender, embora isso não
signifique aceitar seu argumento. Minha posição ainda é a mesma,
mantenho a pertinencia de debater no mesmo foco que o apresentador do
objeto, a igreja, deu a ele: ter pertencido a jesus. Se tem apenas
640 anos, não pode ter sido, se tem mais de 2000, pode ter sido.
>
> Um abraço.
>
> Homero
>
> ----- Original Message -----
> From: pubmed2005
> To: ciencialist@yahoogrupos.com.br
> Sent: Friday, March 18, 2005 10:35 PM
> Subject: [ciencialist] Re: ninguem é inocente.
>
>
>
> Põe uma coisa na sua cabeça
> Ninguem aqui disse (pelo menos eu não disse) que o santo sudário
tem
> que ser o manto de Jesus.
> Pode ter sido um manto qualquer que cobriu um corpo qualquer. E
mesmo
> que tenha sido uma falsificação, isso pode confirmar que os
antigos
> tinham o hábito de enrolarem os corpos em tecidos....pode ser até
que
> faziam isso com pessoas "especiais" que representavam um
determinado
> contexto daquela época
>
> É dificil entender?
>
> Ai, ai, ai...tem que ter paciencia
>
>
>
> --- Em ciencialist@yahoogrupos.com.br, "Oraculo" <oraculo@a...>
> escreveu
> > Olá Pubmed
> >
> > risos..:-) Está confundindo um pouco as coisas..:-) Veja, se um
> artefato que, a priori, é apresentado como sendo o manto que
cobriu o
> corpo de cristo após sua morte, tem sua data definida para 1200
> depois de cristo, isso automaticamente implica que não pode ser
> verdadeiro. Mas, se a data muda para mais de 2000 anos, se torna
> possível a alegação.
> ]
>
>
>
>
>
> ##### ##### #####
>
> Para saber mais visite
> http://www.ciencialist.hpg.ig.com.br
>
>
> ##### ##### ##### #####
>
>
> Yahoo! Grupos, um serviço oferecido por:
>
>
>
>
>
>
>
> --------------------------------------------------------------------
----------
> Links do Yahoo! Grupos
>
> a.. Para visitar o site do seu grupo na web, acesse:
> http://br.groups.yahoo.com/group/ciencialist/
>
> b.. Para sair deste grupo, envie um e-mail para:
> ciencialist-unsubscribe@yahoogrupos.com.br
>
> c.. O uso que você faz do Yahoo! Grupos está sujeito aos Termos
do Serviço do Yahoo!.
>
>
>
> [As partes desta mensagem que não continham texto foram removidas]





SUBJECT: Fw: A verdade sobre os melhores e dos piores alunos
FROM: "Alvaro Augusto \(E\)" <alvaro@electraenergy.com.br>
TO: <ciencialist@yahoogrupos.com.br>
DATE: 18/03/2005 23:50

Segue mensagem enviada por um aluno meu desse semestre, do curso de Engenharia Elétrica. O que vocês acham disso? Há alguma correlação entre "alunos do fundão" e "futuros diretores de empresas"? A falta de ética vale mesmo a pena?

[ ]s

Alvaro Augusto

__________________________


Prezados,

Segue abaixo uma reportagem sobre MBA, que contém o resultado de uma pesquisa onde analisou-se o destino dos melhores e dos piores alunos de 20 anos atrás, nos EUA.

Segundo o Bill Gates os melhores alunos serão os os futuros diretores de amanhã, mas na prática verifica-se que os alunos do fundão por serem mais flexíveis são os melhores em negociação e gestão de pessoas ( pois incentivavam os CDFs e depois copiavam os trabalhos prontos, além de organizar um grupo de cola na sala) e serão chefes dos antigos colegas cdf´s da turma.

Logo segue abaixo o texto para analise e reflexão.

-------------------------------------------------------------------------------------
VOCÊ S/A DE Fevereiro/2002 - Edição 44
"ESSA HISTÓRIA DE MBA É UM HORROR!"

A frase acima é de Vicky Bloch e foi endossada por boa parte dos membros da mesa-redonda. Trata-se de uma mudança importante sobre o que se tem pregado nos últimos anos. Detalhe: diante da afirmação de que os headhunters só olham para MBA, Assumpção, que dirige uma das maiores firmas de recrutamento de executivos do país, respondeu enfático:


- Não mais!


Opa! Estamos diante de uma mudança importante. Às vezes, diz Assumpção, o excesso de informação e de formação é prejudicial, pois pode fazer o profissional se distanciar do mundo real e dos resultados. Que rufem os tambores para o recado de Assumpção: "Você pode ter inúmeros diplomas e ser uma tartaruga, ser um cara que não realiza, que vive no mundo acadêmico".


Quando as regras nas salas dos headhunters mudaram, ninguém sabe. Talvez desde que surgiram indícios de que MBA é apenas mais um diploma. Importante, mas inócuo nas mãos de quem não produz resultados, não tem talento interpessoal, não sabe o valor das equipes. Descobriu-se também que boas notas na escola não significam sucesso profissional. Assumpção lembra uma pesquisa feita com ex-alunos de uma tradicional universidade americana. Analisou-se o destino dos melhores e dos piores alunos de 20 anos atrás. A conclusão foi a seguinte: grande parte da turma do fundão estava bem posicionada na carreira. O inverso aconteceu com a turma do gargalo. "Aqueles que se sentam na frente aceitam o modelo previamente estabelecido, não criticam, entregam e acabou. A turma do fundão, no mínimo, tem de interagir para colar, negociar, inventar desculpas para a bagunça", diz Assumpção.


O que aconteceu, então, com a grande onda de valorização dos diplomas? Max Gehringer compara esse momento com o do menino que era o dono da bola. Ele era o titular do time o ano inteiro, mesmo não sabendo jogar. Os outros meninos entenderam a mensagem. Para entrar no jogo era preciso ser dono da bola. "O que dissemos para as gerações seguintes foi: tenha um diploma que você será titular do time", diz Max. O problema agora é que um monte de gente comprou bola. Só que não dá para todo mundo ser titular. "Às vezes, a gente corre o risco de ficar dizendo 'estude, faça MBA, aprenda inglês' quando, na verdade, é muito mais que isso", diz Max. "Você precisa ter um diferencial em relação aos outros." Caso contrário, não importa quanto conhecimento teórico acumular na vida, será, infelizmente, uma pessoa frustrada. Traduzindo: quando apenas um garoto tinha a bola (ou o diploma de MBA), seu peso no time era enorme. Hoje, quando todo mundo tem a chance de fazer MBA, só vai aparecer quem tiver um diferencial.


Ao procurar um curso - seja ele qual for -, tenha em mente a reflexão de Pochmann: "A educação deve ensinar maneiras de ser mais eficiente no trabalho, mas também deve ensinar para a vida". Não é o que os bancos escolares têm oferecido no Brasil. "A impressão que eu tenho é que a escola brasileira passa informação demais, mas não ensina a analisar essa informação, a relacionar uma com a outra", diz Pochmann. Segundo ele, nesse sentido, a educação é muito simplista - não nos prepara para a complexidade da vida.


Pochmann lembra que estamos vivendo uma época em que o trabalho não é tudo na vida. Há 100 anos, a expectativa média de vida do brasileiro era de 40 anos (hoje é de 68), a sociedade era agrária e a pessoa começava a trabalhar com 5 ou 6 anos de idade. Trabalhava até morrer. Não tinha aposentadoria e a jornada de trabalho era intensa. Hoje, o trabalho representa uma pequena parcela da nossa vida, cerca de um terço. Estude, portanto, maneiras de aproveitar os dois terços do tempo em que está fora do trabalho. Se for fazer MBA, não jogue fora a oportunidade de convivência com os colegas, de aprender com a diversidade de pensamentos, de expandir horizontes. Isso é aprender para a vida.


[As partes desta mensagem que não continham texto foram removidas]



SUBJECT: O Ceticismo como técnica de auto-defesa intelectual (era: ninguem é inocente.)
FROM: "Oraculo" <oraculo@atibaia.com.br>
TO: <ciencialist@yahoogrupos.com.br>
DATE: 18/03/2005 23:50

Olá Pubmed

Não se preocupe, jamais levaria alguém a mal por expressar uma opinião..:-) Mesmo que não concorde comigo, isso não significa que esteja errado ou que seja má pessoa..:-)

Claro que eu penso diferente, inclusive sobre o texto sobre o ceticismo, mas isso é de se esperar, pessoas diferentes pensam de forma diferente afinal..:-)

E acho que ele diz em diversos pontos do artigo exatamente isso, é uma ferramenta, não uma verdade verdadeira, nem mesmo o melhor, sempre. Mas, se a ação que se pretende é pensar de forma racional sobre qualquer coisa, é uma ferramenta útil, e a mais confiável que dispomos no momento. Usamos martelos e chaves de fenda, e nenhuma das duas ferramentas é melhor que a outra de forma absoluta. São melhores em termos relativos, uma para bater pregos (dá para fazer isso com a chave de fenda, mas não é exatamente eficiente..:-) e outra para prender parafusos (martelar parafusos também não é o mais indicado..:-).

Da mesma forma, o ceticismo evita enganos quando se trata de definir a confiabilidade de alegações sobre o mundo fisico, apenas isso..:-)

Um abraço.

Homero

----- Original Message -----
From: pubmed2005
To: ciencialist@yahoogrupos.com.br
Sent: Friday, March 18, 2005 11:42 PM
Subject: [ciencialist] Re: O Ceticismo como técnica de auto-defesa intelectual (era: ninguem é inocente.)



O inicio do texto até que é bom...mas depois vai virando uma lavagem
cerebral danada...hehehe
Ele poderia simplesmente dizer que o ceticismo é um
questionamento...e nao uma posição afirmativa, nem detentora da
verdade

Nao me leve a mal por favor...

--- Em ciencialist@yahoogrupos.com.br, "Oraculo" <oraculo@a...>
escreveu
> Olá Pubmed
>
> risos..:-) A ideia é mais ou menos essa..:-) Permitir que tenhamos
uma ferramenta que faça pela mente, argumentos e escolhas, o mesmo
que as técnicas de auto-defesa fazem pelo corpo..:-) É um artigo
interessante, me diga o que achou dele..:-)
>
> Um abraço.
>
> Homero






##### ##### #####

Para saber mais visite
http://www.ciencialist.hpg.ig.com.br


##### ##### ##### #####


Yahoo! Grupos, um serviço oferecido por:







------------------------------------------------------------------------------
Links do Yahoo! Grupos

a.. Para visitar o site do seu grupo na web, acesse:
http://br.groups.yahoo.com/group/ciencialist/

b.. Para sair deste grupo, envie um e-mail para:
ciencialist-unsubscribe@yahoogrupos.com.br

c.. O uso que você faz do Yahoo! Grupos está sujeito aos Termos do Serviço do Yahoo!.



[As partes desta mensagem que não continham texto foram removidas]



SUBJECT: Re: [ciencialist] Re: ninguem é inocente.
FROM: "Oraculo" <oraculo@atibaia.com.br>
TO: <ciencialist@yahoogrupos.com.br>
DATE: 18/03/2005 23:54

Olá Pubmed

Bem, porque o sudário é "santo"?..:-)

Um abraço.

Homero
----- Original Message -----
From: pubmed2005
To: ciencialist@yahoogrupos.com.br
Sent: Friday, March 18, 2005 11:49 PM
Subject: [ciencialist] Re: ninguem é inocente.



Que eu me lembre, estávamos falando do santo sudário, e ninguem na
lista havia feito afirmações de que esse manto cobria o manto de
jesus...se os católicos e os céticos fazem é outro esquema.

Portanto, nao vejo sentido nesta discussão e nos seus argumentos que
insiste na mesma coisa. A única coisa lógica que tenho a fazer é
evitar de discutir o que eu nao disse.

E ainda mais as pesquisas giram em torno da idade do tecido, do que é
feito, as substancias que ele contém, etc e tal. Se é uma
falsificação , ela pode ser uma falsificação antiga - bem mais antiga
que pensavamos ou apenas um tecido que foi remexido e remendado
através do tempo por pertecencer a algum importante(mesmo que tenha
sido um rei ou algum monarca do reino antigo, ou até mesmo a alguem
comum) - isso tem valor histórico e arqueológico

--- Em ciencialist@yahoogrupos.com.br, "Oraculo" <oraculo@a...>
escreveu
> Olá Pubmed
>
> Bem, em algo tem razão, tem que ter paciencia..:-) É disso que é
feita a ciência, paciência, esforço, análise, tempo, cuidado, etc..:-
) Não a perca por tão pouco..:-)
>
> "Ninguém disse" é exagero, a igreja católica, dona do manto e sua
maior defensora, disse. Mudou de ideia recentemente, afirmando que
não importa, que ele apenas reforça a fé dos catolicos, que é um
simbolo, mas foi quase obrigada a fazer essa colocação..:-)
Inicialmente, todo o debate se resumia a ter ou não pertencido a
jesus, embrulhado seu corpo. E mesmo hoje, é essa a força do sudário,
fornecer evidencias empiricas de uma crença ou superstição ancestral
(procure no Google por santro sudario, e verá que tenho razão..:-).
>
> Assim, é totalmente pertinente discutir as implicações de cada
estudo apresentado a seu respeito.
>
> De todo modo, não, não é dificil de entender, embora isso não
signifique aceitar seu argumento. Minha posição ainda é a mesma,
mantenho a pertinencia de debater no mesmo foco que o apresentador do
objeto, a igreja, deu a ele: ter pertencido a jesus. Se tem apenas
640 anos, não pode ter sido, se tem mais de 2000, pode ter sido.
>
> Um abraço.
>
> Homero
>
> ----- Original Message -----
> From: pubmed2005
> To: ciencialist@yahoogrupos.com.br
> Sent: Friday, March 18, 2005 10:35 PM
> Subject: [ciencialist] Re: ninguem é inocente.
>
>
>
> Põe uma coisa na sua cabeça
> Ninguem aqui disse (pelo menos eu não disse) que o santo sudário
tem
> que ser o manto de Jesus.
> Pode ter sido um manto qualquer que cobriu um corpo qualquer. E
mesmo
> que tenha sido uma falsificação, isso pode confirmar que os
antigos
> tinham o hábito de enrolarem os corpos em tecidos....pode ser até
que
> faziam isso com pessoas "especiais" que representavam um
determinado
> contexto daquela época
>
> É dificil entender?
>
> Ai, ai, ai...tem que ter paciencia
>
>
>
> --- Em ciencialist@yahoogrupos.com.br, "Oraculo" <oraculo@a...>
> escreveu
> > Olá Pubmed
> >
> > risos..:-) Está confundindo um pouco as coisas..:-) Veja, se um
> artefato que, a priori, é apresentado como sendo o manto que
cobriu o
> corpo de cristo após sua morte, tem sua data definida para 1200
> depois de cristo, isso automaticamente implica que não pode ser
> verdadeiro. Mas, se a data muda para mais de 2000 anos, se torna
> possível a alegação.
> ]
>
>
>
>
>
> ##### ##### #####
>
> Para saber mais visite
> http://www.ciencialist.hpg.ig.com.br
>
>
> ##### ##### ##### #####
>
>
> Yahoo! Grupos, um serviço oferecido por:
>
>
>
>
>
>
>
> --------------------------------------------------------------------
----------
> Links do Yahoo! Grupos
>
> a.. Para visitar o site do seu grupo na web, acesse:
> http://br.groups.yahoo.com/group/ciencialist/
>
> b.. Para sair deste grupo, envie um e-mail para:
> ciencialist-unsubscribe@yahoogrupos.com.br
>
> c.. O uso que você faz do Yahoo! Grupos está sujeito aos Termos
do Serviço do Yahoo!.
>
>
>
> [As partes desta mensagem que não continham texto foram removidas]





##### ##### #####

Para saber mais visite
http://www.ciencialist.hpg.ig.com.br


##### ##### ##### #####


Yahoo! Grupos, um serviço oferecido por:







------------------------------------------------------------------------------
Links do Yahoo! Grupos

a.. Para visitar o site do seu grupo na web, acesse:
http://br.groups.yahoo.com/group/ciencialist/

b.. Para sair deste grupo, envie um e-mail para:
ciencialist-unsubscribe@yahoogrupos.com.br

c.. O uso que você faz do Yahoo! Grupos está sujeito aos Termos do Serviço do Yahoo!.



[As partes desta mensagem que não continham texto foram removidas]



SUBJECT: Re: ninguem é inocente.
FROM: "pubmed2005" <pubmed2005@yahoo.com.br>
TO: ciencialist@yahoogrupos.com.br
DATE: 19/03/2005 00:01


Ok oráculo , tudo bem

Mas deixa eu perguntar uma coisa. No fundo , no fundo os céticos tem
um pouquinho de cagaço que o manto seja mesmo de Jesus né?

Não vejo outra explicação quando alguém do meio cético rejeita( ou
faz uma analise tão critica de uma pesquisa tão importante. Apostaria
que se fosse uma pesquisa que validasse apenas os 1300 anos ou até
menos que isso, os céticos aplaudiriam e seriam menos críticos

O exame do carbono 14 foi muito importante para a ciencia, mas
podemos ser bastates críticos e afirmar que ele foi insuficiente pelo
pouco tempo que os cientistas tiveram o manto disponíveis para a
pesquisa. e poderíamos testar e retestar as pésquisas do queímico e
as diferentes análises de outras partes do tecido pelo carbono-14

Outra coisa que gostaria de explicar aos pertecentes a esta lista,
quando uso o termo cético nao é no sentido pejorativo de ceticismo
como uma coisa má e ruim, mas uma análise sobre este ceticismo
ilógico que reina pela internet


--- Em ciencialist@yahoogrupos.com.br, "Oraculo" <oraculo@a...>
escreveu
> Olá Pubmed
>
> Desculpe, onde foi que eu, ou qualquer outro, disse ou escreveu
que "só ceticos tem cérebro" ou que "só ceticos podem pensar"? Não é,
claro, um argumento, apenas um ataque do tipo "ad hominem", mas
gostaria de saber se algo que eu escrevi tenha dado essa ideia ou
poderia ser interpretado dessa forma..:-)
>
> Em todo caso, se eu achasse isso, não estaria debatendo com você e
com a lista, com cuidado e educação, expondo meus argumentos e lendo
atentamente os seus..:-) É um debate, não uma guerra, e não pretendo
faze-lo mudar de ideia ou aceitar o que quer que seja. Apenas
apresento meu ponto de vista, meus argumentos e minhas evidencias,
esperando que essa troca traga algo de útil a nós dois.
>
> Um abraço.
>
> Homero
>
> ----- Original Message -----
> From: pubmed2005
> To: ciencialist@yahoogrupos.com.br
> Sent: Friday, March 18, 2005 10:50 PM
> Subject: [ciencialist] Re: ninguem é inocente.
>
>
>
> Voce esqueceu de completar sua retórica perguntando o que é
> ferramenta da razão para pensar...
> Será que é o cérebro?...ah esqueci de dizer que só os "céticos
tem
> cérebros" e "que só eles podem pensar"
>
>
> --- Em ciencialist@yahoogrupos.com.br, "Alberto Mesquita Filho"
> <albmesq@u...> escreveu
> > ----- Original Message -----
> > From: "Oraculo"
> > Sent: Friday, March 18, 2005 10:02 PM
> > Subject: Re: [ciencialist] Re: ninguem é inocente.
> >
> > > sou um cético de verdade usando a ferramenta da razão para
pensar
> sobre as
> > > alegações e afirmativas que são apresentadas..:-)
> >
> > Existem céticos de verdade? Afinal, o que é ceticismo? Ou
então: O
> que é
> > verdade?
> >
> > Não precisa responder. Eu só queria perguntar. ;-))
>
>
>
>
>
>
> ##### ##### #####
>
> Para saber mais visite
> http://www.ciencialist.hpg.ig.com.br
>
>
> ##### ##### ##### #####
>
>
> Yahoo! Grupos, um serviço oferecido por:
>
> São Paulo Rio de Janeiro Curitiba Porto Alegre
Belo Horizonte Brasília
>
>
>
>
> --------------------------------------------------------------------
----------
> Links do Yahoo! Grupos
>
> a.. Para visitar o site do seu grupo na web, acesse:
> http://br.groups.yahoo.com/group/ciencialist/
>
> b.. Para sair deste grupo, envie um e-mail para:
> ciencialist-unsubscribe@yahoogrupos.com.br
>
> c.. O uso que você faz do Yahoo! Grupos está sujeito aos Termos
do Serviço do Yahoo!.
>
>
>
> [As partes desta mensagem que não continham texto foram removidas]





SUBJECT: Re: ninguem é inocente.
FROM: "pubmed2005" <pubmed2005@yahoo.com.br>
TO: ciencialist@yahoogrupos.com.br
DATE: 19/03/2005 00:15


Não sei.
Mas, nao sei o que isso tem haver com as pesquisas realizadas.
Talvez a denominação de santo pode ter sido dada pelos católicos, ou
algo que tenha sido feita pelos cristãos...ou até mesmo por pertencer
a alguém importate em que se acreditava que ia ressussitar.

Mas isso é relevante, o que é relevante é que o manto é historico ,
independente do seu poder mágico ou ter pertencido a um homem chamado
Jesus. Mesmo que tenha sido uma falsifica da idade édia nao deixa de
ter valor histórico. E nao um simples tecido sem utilidade como
versam os céticos

A cultura antiga está sempre dando pores mágicos a objetos. Os
proprios egípcios tinham esse costume. E os mortos nobres eram
preservados com tecidos e outros materiais

Qualquer tecido antigo, objeto ou artefato tem que ser considerado e
tratado pela ciencia como um objeto histórico e arqueológico sujeito
a estudos.
O mesmo poderia ser com uma moeda, a toalha de um egício, ou até
mesmo o papel que limpou a bunda de napoleão(se um dia for achado).

Aos cientistas cabe o estudo do objeto histórico em questão, explicar
origem, constituição, de onde veio, etc e tal

Os que alegam que um determinado papel foi o que limpou a bunda de
napoleão nao é importante, o que é importante é o que a ciencia apura
dos fatos e o acesso e consenso sobre origem, idade, constituição
do papel E se foi realmente o papel que limpou a bunda do Napoleão(
feliz seja Napoleão que limpava a bunda com papel. Se nao foi o de
Napoleão, mas foi o papel que limpou a bunda da rainha Nefertini. Que
assim seja!!!
Mas, o fato é que o papel nao deixa de ser um papel que tem alguma
importancia histórica




--- Em ciencialist@yahoogrupos.com.br, "Oraculo" <oraculo@a...>
escreveu
> Olá Pubmed
>
> Bem, porque o sudário é "santo"?..:-)






SUBJECT: Re: ninguem é inocente.
FROM: "rmtakata" <rmtakata@altavista.net>
TO: ciencialist@yahoogrupos.com.br
DATE: 19/03/2005 00:19


--- Em ciencialist@yahoogrupos.com.br, "pubmed2005"
> céticos tem um pouquinho de cagaço que o manto seja mesmo de
> Jesus né?

Usando de seu termo chulo, parece q. ocorre o contrario. Os crentes
continuam procurando 'justificativas' para considerar o manto como
verdadeiro.

> O exame do carbono 14 foi muito importante para a ciencia,
> mas podemos ser bastates críticos e afirmar que ele foi
> insuficiente pelo pouco tempo que os cientistas tiveram o
> manto disponíveis para a pesquisa.

Esse tempo nao interfere no resultado do carbono-14.

> e poderíamos testar e retestar as pésquisas do queímico e
> as diferentes análises de outras partes do tecido pelo
> carbono-14

Sim e aih acabaria-se o manto. Lembre-se de q. o teste foi feito por
*tres* laboratorios independentes e os resultados foram semelhantes.

Ainda estao pra provar q. analise quimica seja uma metodologia de
datacao mais confiavel do q. radiodatacao.

[]s,

Roberto Takata





SUBJECT: Re: [ciencialist] Re: ninguem é inocente.
FROM: "Oraculo" <oraculo@atibaia.com.br>
TO: <ciencialist@yahoogrupos.com.br>
DATE: 19/03/2005 00:20

Olá Pubmed

"Pubmed: No fundo , no fundo os céticos tem um pouquinho de cagaço que o manto seja mesmo de Jesus né?"

Não, não temos..:-) Eu pelo menos não tenho nenhum..:-) Veja, se for de jesus, isso significa que existiu um cara chamado jesus, apenas isso. Não significa muito mais que isso, e isso apenas é algo a ser considerado e estudado, não significa que todos nós devemos nos tornar católicos ou cristãos..:-)

E eu tentei explicar, e talvez não tenha sido claro, porque céticos rejeitam estudos. É porque rejeitar estudos é parte fundamental do método cientifico..;-) Apenas se sobreviver a todas as rejeições, a todas as analises críticas, a todas as tentativas de refutação, é que um conhecimento, uma alegação, uma conclusão pode ser chamada de científica. Não é uma rejeição especial, contra estudos sobre o manto, é uma rejeição geral, parte da ferramenta conhecida como método científico.

Como eu disse antes, foi aplicada até mesmo contra o primeiro estudo, de carbono-14. Apenas com as confirmações subsequentes, que validaram os dados obtidos no primeiro estudo, é que foram aceitos. Por que acha que foram feitos novos estudos com o mesmo processo, se o primeiro teria sido aceito sem contestação e apenas com aplausos?..:-) Ainda assim, podem ser refutados ou descartados, se novos estudos modificarem as conclusões.

Como este novo que você defende. Mas, para fazer isso, para ser capaz de refutar o anterior, este novo estudo tem de ser tão forte quanto, pelo menos, o que pretende refutar. Tem de sobreviver a tantas críticas quanto o anterior, tem de ter tantos estudos de confirmação quanto o primeiro. Só assim será aceito como ciência legítima.

Isso aconteceu com todo conhecimento cientifico, não apeaans com alegações religiosas. Com a fusão a frio, que foi refutada, e com a Deriva dos Continentes, que foi finalmente aceita. As duas, alegações cientificas, não sobrenaturais ou esotericas.

Pubmed: Apostaria que se fosse uma pesquisa que validasse apenas os 1300 anos ou até
menos que isso, os céticos aplaudiriam e seriam menos críticos "

risos..:-) Não perca seu dinheiro assim tão fácil..:-) Se apostasse, perderia. Céticos não apenas não aplaudiriam como fariam os mesmos questionamentos, que é preciso esperar que estudos comprovem a conclusão, que esta possa ser repetida, etc. É parte do método, sem ele não valeria nada uma alegação. Está confundindo céticos com torcedores de futebol, que, não importa o quão mal seus times joguem, sempre é culpa do juiz..:-) Claro que existem os que são contra a igreja, ou mesmo de outra religião, e torcem para que ela se de mal, para que o manto seja uma fraude. Estes, embora as vezes se disfarcem de céticos, na verdade creem, mas creem em outras crenças. É como no antigo ditado, o inimigo de meu inimigo, meu amigo é..:-)

Como católicos (Padre Quevedo, conhece?) que usam argumentos céticos contra alegações espíritas..:-) Não são céticos de verdade, já que evitam usar os mesmos argumentos contra suas próprias crenças, mas um tipo de torcedor..:-) Somos todos céticos com as crenças alheias..:-)

Um abraço.

Homero



----- Original Message -----
From: pubmed2005
To: ciencialist@yahoogrupos.com.br
Sent: Saturday, March 19, 2005 12:01 AM
Subject: [ciencialist] Re: ninguem é inocente.



Ok oráculo , tudo bem

Mas deixa eu perguntar uma coisa. No fundo , no fundo os céticos tem
um pouquinho de cagaço que o manto seja mesmo de Jesus né?

Não vejo outra explicação quando alguém do meio cético rejeita( ou
faz uma analise tão critica de uma pesquisa tão importante. Apostaria
que se fosse uma pesquisa que validasse apenas os 1300 anos ou até
menos que isso, os céticos aplaudiriam e seriam menos críticos

O exame do carbono 14 foi muito importante para a ciencia, mas
podemos ser bastates críticos e afirmar que ele foi insuficiente pelo
pouco tempo que os cientistas tiveram o manto disponíveis para a
pesquisa. e poderíamos testar e retestar as pésquisas do queímico e
as diferentes análises de outras partes do tecido pelo carbono-14

Outra coisa que gostaria de explicar aos pertecentes a esta lista,
quando uso o termo cético nao é no sentido pejorativo de ceticismo
como uma coisa má e ruim, mas uma análise sobre este ceticismo
ilógico que reina pela internet


--- Em ciencialist@yahoogrupos.com.br, "Oraculo" <oraculo@a...>
escreveu
> Olá Pubmed
>
> Desculpe, onde foi que eu, ou qualquer outro, disse ou escreveu
que "só ceticos tem cérebro" ou que "só ceticos podem pensar"? Não é,
claro, um argumento, apenas um ataque do tipo "ad hominem", mas
gostaria de saber se algo que eu escrevi tenha dado essa ideia ou
poderia ser interpretado dessa forma..:-)
>
> Em todo caso, se eu achasse isso, não estaria debatendo com você e
com a lista, com cuidado e educação, expondo meus argumentos e lendo
atentamente os seus..:-) É um debate, não uma guerra, e não pretendo
faze-lo mudar de ideia ou aceitar o que quer que seja. Apenas
apresento meu ponto de vista, meus argumentos e minhas evidencias,
esperando que essa troca traga algo de útil a nós dois.
>
> Um abraço.
>
> Homero
>
> ----- Original Message -----
> From: pubmed2005
> To: ciencialist@yahoogrupos.com.br
> Sent: Friday, March 18, 2005 10:50 PM
> Subject: [ciencialist] Re: ninguem é inocente.
>
>
>
> Voce esqueceu de completar sua retórica perguntando o que é
> ferramenta da razão para pensar...
> Será que é o cérebro?...ah esqueci de dizer que só os "céticos
tem
> cérebros" e "que só eles podem pensar"
>
>
> --- Em ciencialist@yahoogrupos.com.br, "Alberto Mesquita Filho"
> <albmesq@u...> escreveu
> > ----- Original Message -----
> > From: "Oraculo"
> > Sent: Friday, March 18, 2005 10:02 PM
> > Subject: Re: [ciencialist] Re: ninguem é inocente.
> >
> > > sou um cético de verdade usando a ferramenta da razão para
pensar
> sobre as
> > > alegações e afirmativas que são apresentadas..:-)
> >
> > Existem céticos de verdade? Afinal, o que é ceticismo? Ou
então: O
> que é
> > verdade?
> >
> > Não precisa responder. Eu só queria perguntar. ;-))
>
>
>
>
>
>
> ##### ##### #####
>
> Para saber mais visite
> http://www.ciencialist.hpg.ig.com.br
>
>
> ##### ##### ##### #####
>
>
> Yahoo! Grupos, um serviço oferecido por:
>
> São Paulo Rio de Janeiro Curitiba Porto Alegre
Belo Horizonte Brasília
>
>
>
>
> --------------------------------------------------------------------
----------
> Links do Yahoo! Grupos
>
> a.. Para visitar o site do seu grupo na web, acesse:
> http://br.groups.yahoo.com/group/ciencialist/
>
> b.. Para sair deste grupo, envie um e-mail para:
> ciencialist-unsubscribe@yahoogrupos.com.br
>
> c.. O uso que você faz do Yahoo! Grupos está sujeito aos Termos
do Serviço do Yahoo!.
>
>
>
> [As partes desta mensagem que não continham texto foram removidas]





##### ##### #####

Para saber mais visite
http://www.ciencialist.hpg.ig.com.br


##### ##### ##### #####


Yahoo! Grupos, um serviço oferecido por:
PUBLICIDADE




------------------------------------------------------------------------------
Links do Yahoo! Grupos

a.. Para visitar o site do seu grupo na web, acesse:
http://br.groups.yahoo.com/group/ciencialist/

b.. Para sair deste grupo, envie um e-mail para:
ciencialist-unsubscribe@yahoogrupos.com.br

c.. O uso que você faz do Yahoo! Grupos está sujeito aos Termos do Serviço do Yahoo!.



[As partes desta mensagem que não continham texto foram removidas]



SUBJECT: Sudario
FROM: Eduardo Gueron <edgueron@yahoo.com>
TO: ciencialist@yahoogrupos.com.br
DATE: 19/03/2005 00:21

Oi a todos,

Voltei a ciencialist motivado pela morte de Cesar
Lattes, certamente o fmsico mais bem sucedido de todos
os tempos no Brasil e que, por algumas de suas
crmticas ferozes, despertou certa antipatia no meio
medmocre dos fmsicos atuais, principalmente da mafia
USPiana. A despeito de discordar de algumas das
crmticas que ele fez a Rel. Geral ou certas paixues
que nutriu por alguns picaretas, sempre o admirei pela
sua "falta de mediocridade". Comecei a estudar fmsica
em 1990 e conheci muitos fmsicos muito bem sucedidos
(no conceito predominante que se tem de carreira
cientmfica) que transbordavam ignorbncia. Fui ` casa
de Lattes em Campinas duas vezes. Na primeira, entre
outras coisas, me perguntou se havia lido Voltaire -
por sorte lera Cbndido - e, depois de falar de alguns
cientistas brilhantes, afirmou que nada se compara `s
artes... Acho que isso o resume bem. Uma pena que apss
a morte de sua grande companheira, a professora de
histsria Martha em 2003, Lattes simplesmente se deixou
morrer.

Bem, agora fazendo jus ao tmtulo, afirmo sem o mmnimo
rigor que o Sudario i uma farsa. Argumentos
cientmficos sco varios, vejam por exemplo o smtio:
http://brazil.skepdic.com/sudario.html
Mas podemos usar outras vias lsgicas de raciocmnio
para chegar a essa conclusco. Se a Igreja acreditasse
piamente que o Sudario i fruto de um milagre, nco
teria ela interesse em que a comunidade cientmfica
fizesse testes a fim de comprovar sua autenticidade?
Por que entco depois do primeiro resultado negativo,
procura impedir outros testes e liberar apenas um
teste oficial feito por um Qummico aposentado da
California?

Acho, sinceramente, que muito tempo se perde com essa
instituigco nefasta que i a Igreja catslica (minha
crmtica i contra a instituigco nco contra os
catslicos) . Uma igreja que apresenta estudos dando
conta de que a camisinha nco i eficiente no combate a
AIDS e cujo Papa condena no mesmo texto o terrorismo e
o casamento homossexual. E que se calou perante a
regimes autoritarios e violentos na Amirica Latina?
(Com honrosas excegues como o saudoso D. Helder
Cbmara)

Sou citico e creio (crer?) que o ceticismo i
necessario para o cientista. Tentar desqualifica-lo
por causa de bobagens `s vezes faladas em fsruns
citicos i, no mmnimo, covardia. Lembra a atitude da
rede Globo que se aproveitou de um idiota sadico para
desqualificar os movimentos de defesa dos animais que
protestam contra rodeios, tema da novela das 8
atual...(nada a ver a comparagco, mas queria falar
disso em algum lugar)

I isso,

Um abrago, Eduardo.


__________________________________________________
Do You Yahoo!?
Tired of spam? Yahoo! Mail has the best spam protection around
http://mail.yahoo.com


SUBJECT: Re: [ciencialist] Re: ninguem é inocente.
FROM: "Oraculo" <oraculo@atibaia.com.br>
TO: <ciencialist@yahoogrupos.com.br>
DATE: 19/03/2005 00:31

Olá Pumed

Punbmed: E nao um simples tecido sem utilidade como
versam os céticos"

Desculpe, mas agora é você que esta lendo demais, onde não existe nada..:-) De onde tirou que céticos afirmam isso sobre o manto???? Céticos apenas duvidam de sua veracidade como cobertura de jesus e da alegação que a imagem é milagrosa, não discutem a importância como artefato antropologico ou histórico. Ele, o manto, é um objeto fascinante, mesmo sem o pretenso milagre que lhe é relacionado ..:-)

Eu acho o manto e sua história algo digno de ser estudado, analisado, compreendido, de toda forma possível. Em nenhum momento isso foi diferente ou eu afirmei o contrário. Duvidar, ou melhor, afirmar que é preciso esperar que outros estudos validem o que o Rogers apresentou nada modifica isso, nem está de alguma forma desmerecendo a importancia do manto e de seu estudo. Está confundindo as coisas..:-)

Pubmed: qualquer tecido antigo, objeto ou artefato tem que ser considerado e
tratado pela ciencia como um objeto histórico e arqueológico sujeito
a estudos.

Nenhum cético, cientista ou pesqusiador discordaria dessas palavras, nem isso foi afirmado em nenhum momento, nem por mim, nem pelos que criticam o estudo de Rogers. Todo objeto deve ser estudado e todo obejto antigo tem seu interesse e fascinio. E não seria diferente com o manto. Até mesmo o sangue de são genaro, que se liquefaz a cada ano, deve ser estudado (se fosse possível..:-), por ser interessante e passivel de ensinar algo sobre os que criaram o brinquedo.

Pubmed: Os que alegam que um determinado papel foi o que limpou a bunda de
napoleão nao é importante, o que é importante é o que a ciencia apura
dos fatos e o acesso e consenso sobre origem, idade, constituição
do papel E se foi realmente o papel que limpou a bunda do Napoleão(
feliz seja Napoleão que limpava a bunda com papel.

Apesar da evidente intenção de ser uma brincaderia, seu texto é correto.:-) Se encontrassem um papel que tivesse o destino que apresenta acima, centenas de pesquisadores lutariam pelo priovilégio de estuda-lo..:-) É possivel descobrir coisas fantásticas em residuos animais, inclusive humanos, como envenenamento (será que Napoleão foi envenenado ou morreu de causas naturais?), doenças diversas (dizem que ele tinha uma úlcera terrível), etc. O estudo de coprólitos (fezes fósseis) é uma das áreas mais interessantes da paleontologia.:-)

Um abraço.

Homero

----- Original Message -----
From: pubmed2005
To: ciencialist@yahoogrupos.com.br
Sent: Saturday, March 19, 2005 12:15 AM
Subject: [ciencialist] Re: ninguem é inocente.



Não sei.
Mas, nao sei o que isso tem haver com as pesquisas realizadas.
Talvez a denominação de santo pode ter sido dada pelos católicos, ou
algo que tenha sido feita pelos cristãos...ou até mesmo por pertencer
a alguém importate em que se acreditava que ia ressussitar.

Mas isso é relevante, o que é relevante é que o manto é historico ,
independente do seu poder mágico ou ter pertencido a um homem chamado
Jesus. Mesmo que tenha sido uma falsifica da idade édia nao deixa de
ter valor histórico. E nao um simples tecido sem utilidade como
versam os céticos

A cultura antiga está sempre dando pores mágicos a objetos. Os
proprios egípcios tinham esse costume. E os mortos nobres eram
preservados com tecidos e outros materiais

Qualquer tecido antigo, objeto ou artefato tem que ser considerado e
tratado pela ciencia como um objeto histórico e arqueológico sujeito
a estudos.
O mesmo poderia ser com uma moeda, a toalha de um egício, ou até
mesmo o papel que limpou a bunda de napoleão(se um dia for achado).

Aos cientistas cabe o estudo do objeto histórico em questão, explicar
origem, constituição, de onde veio, etc e tal

Os que alegam que um determinado papel foi o que limpou a bunda de
napoleão nao é importante, o que é importante é o que a ciencia apura
dos fatos e o acesso e consenso sobre origem, idade, constituição
do papel E se foi realmente o papel que limpou a bunda do Napoleão(
feliz seja Napoleão que limpava a bunda com papel. Se nao foi o de
Napoleão, mas foi o papel que limpou a bunda da rainha Nefertini. Que
assim seja!!!
Mas, o fato é que o papel nao deixa de ser um papel que tem alguma
importancia histórica




--- Em ciencialist@yahoogrupos.com.br, "Oraculo" <oraculo@a...>
escreveu
> Olá Pubmed
>
> Bem, porque o sudário é "santo"?..:-)






##### ##### #####

Para saber mais visite
http://www.ciencialist.hpg.ig.com.br


##### ##### ##### #####


Yahoo! Grupos, um serviço oferecido por:
PUBLICIDADE




------------------------------------------------------------------------------
Links do Yahoo! Grupos

a.. Para visitar o site do seu grupo na web, acesse:
http://br.groups.yahoo.com/group/ciencialist/

b.. Para sair deste grupo, envie um e-mail para:
ciencialist-unsubscribe@yahoogrupos.com.br

c.. O uso que você faz do Yahoo! Grupos está sujeito aos Termos do Serviço do Yahoo!.



[As partes desta mensagem que não continham texto foram removidas]



SUBJECT: Re: ninguem é inocente.
FROM: "pubmed2005" <pubmed2005@yahoo.com.br>
TO: ciencialist@yahoogrupos.com.br
DATE: 19/03/2005 00:33


E quem disse que o novo estudo tem a intenção de refutar o primeiro
ou pelo menos derruba-lo? Ele apenas levantou a questão com bases em
premissas científicas que o manto deveria ter aproximadamente 3000
anos, e nao apenas os quase exatos 1300 anos do carbono-14

Tenho a impressão que o manto nao foi exaustivamente verificado como
deveria. Portanto o primeiro estudo pode estar cheio de erros e
imprecisões.

E segundo, a análise química do manto é tb muito importante, e
portanto a segunda pesquisa nao seria uma rival da primeira, e sim
uma complementação , ou um questionamento necessário a primeira
posição.

de qualquer forma precisaríamos fazer uma aanálise mais completa do
tecido, antes de chegar uma conclusão, lou fazer afirmações que o
manto tenha que ter inexoravelmente 1300 anos

Voce se apegando ao primeiro estudo, e fazendo pouco caso do segundo
nao está agindo como um cético, e sim como aqueles céticos da
internet como eu havia falado, sinto muito.

Eu tb nunca falei que o primeiro estudo estava errado...apenas eu
disse que o segundo levantou questões muito interessantes que podem
mudar todo o aparato dogmático que envolve as afirmações
experimentais do primeiro estudo

E outra coisita, quem disse que os céticos estão isento de crenças e
que estão isentos de criticar suas proprias crenças com o mesmo rigor
que criticam as alheias? Mesmo que as crenças dos céticos sejam de
nartureza diferentes das místicas, acho isso muito suspeito de se
dizer, e até mesmo uma posição nao-cética a de dizer que nao se tem
crenças e que dedicam o mesmo tempo de exposição a que se dedicam as
crenças alheias.


--- Em ciencialist@yahoogrupos.com.br, "Oraculo" <oraculo@a...>
escreveu
> Olá Pubmed
>
> "Pubmed: No fundo , no fundo os céticos tem um pouquinho de cagaço
que o manto seja mesmo de Jesus né?"
>
> Não, não temos..:-) Eu pelo menos não tenho nenhum..:-) Veja, se
for de jesus, isso significa que existiu um cara chamado jesus,
apenas isso. Não significa muito mais que isso, e isso apenas é algo
a ser considerado e estudado, não significa que todos nós devemos nos
tornar católicos ou cristãos..:-)
>
> E eu tentei explicar, e talvez não tenha sido claro, porque céticos
rejeitam estudos. É porque rejeitar estudos é parte fundamental do
método cientifico..;-) Apenas se sobreviver a todas as rejeições, a
todas as analises críticas, a todas as tentativas de refutação, é que
um conhecimento, uma alegação, uma conclusão pode ser chamada de
científica. Não é uma rejeição especial, contra estudos sobre o
manto, é uma rejeição geral, parte da ferramenta conhecida como
método científico.
>
> Como eu disse antes, foi aplicada até mesmo contra o primeiro
estudo, de carbono-14. Apenas com as confirmações subsequentes, que
validaram os dados obtidos no primeiro estudo, é que foram aceitos.
Por que acha que foram feitos novos estudos com o mesmo processo, se
o primeiro teria sido aceito sem contestação e apenas com
aplausos?..:-) Ainda assim, podem ser refutados ou descartados, se
novos estudos modificarem as conclusões.
>
> Como este novo que você defende. Mas, para fazer isso, para ser
capaz de refutar o anterior, este novo estudo tem de ser tão forte
quanto, pelo menos, o que pretende refutar. Tem de sobreviver a
tantas críticas quanto o anterior, tem de ter tantos estudos de
confirmação quanto o primeiro. Só assim será aceito como ciência
legítima.
>
> Isso aconteceu com todo conhecimento cientifico, não apeaans com
alegações religiosas. Com a fusão a frio, que foi refutada, e com a
Deriva dos Continentes, que foi finalmente aceita. As duas, alegações
cientificas, não sobrenaturais ou esotericas.
>
> Pubmed: Apostaria que se fosse uma pesquisa que validasse apenas os
1300 anos ou até
> menos que isso, os céticos aplaudiriam e seriam menos críticos "
>
> risos..:-) Não perca seu dinheiro assim tão fácil..:-) Se
apostasse, perderia. Céticos não apenas não aplaudiriam como fariam
os mesmos questionamentos, que é preciso esperar que estudos
comprovem a conclusão, que esta possa ser repetida, etc. É parte do
método, sem ele não valeria nada uma alegação. Está confundindo
céticos com torcedores de futebol, que, não importa o quão mal seus
times joguem, sempre é culpa do juiz..:-) Claro que existem os que
são contra a igreja, ou mesmo de outra religião, e torcem para que
ela se de mal, para que o manto seja uma fraude. Estes, embora as
vezes se disfarcem de céticos, na verdade creem, mas creem em outras
crenças. É como no antigo ditado, o inimigo de meu inimigo, meu amigo
é..:-)
>
> Como católicos (Padre Quevedo, conhece?) que usam argumentos
céticos contra alegações espíritas..:-) Não são céticos de verdade,
já que evitam usar os mesmos argumentos contra suas próprias
crenças, mas um tipo de torcedor..:-) Somos todos céticos com as
crenças alheias..:-)
>
> Um abraço.
>
> Homero
>
>
>
> ----- Original Message -----
> From: pubmed2005
> To: ciencialist@yahoogrupos.com.br
> Sent: Saturday, March 19, 2005 12:01 AM
> Subject: [ciencialist] Re: ninguem é inocente.
>
>
>
> Ok oráculo , tudo bem
>
> Mas deixa eu perguntar uma coisa. No fundo , no fundo os céticos
tem
> um pouquinho de cagaço que o manto seja mesmo de Jesus né?
>
> Não vejo outra explicação quando alguém do meio cético rejeita(
ou
> faz uma analise tão critica de uma pesquisa tão importante.
Apostaria
> que se fosse uma pesquisa que validasse apenas os 1300 anos ou
até
> menos que isso, os céticos aplaudiriam e seriam menos críticos
>
> O exame do carbono 14 foi muito importante para a ciencia, mas
> podemos ser bastates críticos e afirmar que ele foi insuficiente
pelo
> pouco tempo que os cientistas tiveram o manto disponíveis para a
> pesquisa. e poderíamos testar e retestar as pésquisas do queímico
e
> as diferentes análises de outras partes do tecido pelo carbono-14
>
> Outra coisa que gostaria de explicar aos pertecentes a esta
lista,
> quando uso o termo cético nao é no sentido pejorativo de
ceticismo
> como uma coisa má e ruim, mas uma análise sobre este ceticismo
> ilógico que reina pela internet
>
>
> --- Em ciencialist@yahoogrupos.com.br, "Oraculo" <oraculo@a...>
> escreveu
> > Olá Pubmed
> >
> > Desculpe, onde foi que eu, ou qualquer outro, disse ou escreveu
> que "só ceticos tem cérebro" ou que "só ceticos podem pensar"?
Não é,
> claro, um argumento, apenas um ataque do tipo "ad hominem", mas
> gostaria de saber se algo que eu escrevi tenha dado essa ideia ou
> poderia ser interpretado dessa forma..:-)
> >
> > Em todo caso, se eu achasse isso, não estaria debatendo com
você e
> com a lista, com cuidado e educação, expondo meus argumentos e
lendo
> atentamente os seus..:-) É um debate, não uma guerra, e não
pretendo
> faze-lo mudar de ideia ou aceitar o que quer que seja. Apenas
> apresento meu ponto de vista, meus argumentos e minhas
evidencias,
> esperando que essa troca traga algo de útil a nós dois.
> >
> > Um abraço.
> >
> > Homero
> >
> > ----- Original Message -----
> > From: pubmed2005
> > To: ciencialist@yahoogrupos.com.br
> > Sent: Friday, March 18, 2005 10:50 PM
> > Subject: [ciencialist] Re: ninguem é inocente.
> >
> >
> >
> > Voce esqueceu de completar sua retórica perguntando o que é
> > ferramenta da razão para pensar...
> > Será que é o cérebro?...ah esqueci de dizer que só
os "céticos
> tem
> > cérebros" e "que só eles podem pensar"
> >
> >
> > --- Em ciencialist@yahoogrupos.com.br, "Alberto Mesquita
Filho"
> > <albmesq@u...> escreveu
> > > ----- Original Message -----
> > > From: "Oraculo"
> > > Sent: Friday, March 18, 2005 10:02 PM
> > > Subject: Re: [ciencialist] Re: ninguem é inocente.
> > >
> > > > sou um cético de verdade usando a ferramenta da razão
para
> pensar
> > sobre as
> > > > alegações e afirmativas que são apresentadas..:-)
> > >
> > > Existem céticos de verdade? Afinal, o que é ceticismo? Ou
> então: O
> > que é
> > > verdade?
> > >
> > > Não precisa responder. Eu só queria perguntar. ;-))
> >
> >
> >
> >
> >
> >
> > ##### ##### #####
> >
> > Para saber mais visite
> > http://www.ciencialist.hpg.ig.com.br
> >
> >
> > ##### ##### ##### #####
> >
> >
> > Yahoo! Grupos, um serviço oferecido por:
> >
> > São Paulo Rio de Janeiro Curitiba Porto
Alegre
> Belo Horizonte Brasília
> >
> >
> >
> >
> > ----------------------------------------------------------------
----
> ----------
> > Links do Yahoo! Grupos
> >
> > a.. Para visitar o site do seu grupo na web, acesse:
> > http://br.groups.yahoo.com/group/ciencialist/
> >
> > b.. Para sair deste grupo, envie um e-mail para:
> > ciencialist-unsubscribe@yahoogrupos.com.br
> >
> > c.. O uso que você faz do Yahoo! Grupos está sujeito aos
Termos
> do Serviço do Yahoo!.
> >
> >
> >
> > [As partes desta mensagem que não continham texto foram
removidas]
>
>
>
>
>
> ##### ##### #####
>
> Para saber mais visite
> http://www.ciencialist.hpg.ig.com.br
>
>
> ##### ##### ##### #####
>
>
> Yahoo! Grupos, um serviço oferecido por:
> PUBLICIDADE
>
>
>
>
> --------------------------------------------------------------------
----------
> Links do Yahoo! Grupos
>
> a.. Para visitar o site do seu grupo na web, acesse:
> http://br.groups.yahoo.com/group/ciencialist/
>
> b.. Para sair deste grupo, envie um e-mail para:
> ciencialist-unsubscribe@yahoogrupos.com.br
>
> c.. O uso que você faz do Yahoo! Grupos está sujeito aos Termos
do Serviço do Yahoo!.
>
>
>
> [As partes desta mensagem que não continham texto foram removidas]





SUBJECT: Re: ninguem é inocente.
FROM: "pubmed2005" <pubmed2005@yahoo.com.br>
TO: ciencialist@yahoogrupos.com.br
DATE: 19/03/2005 00:51


SSegue comentarios

--- Em ciencialist@yahoogrupos.com.br, "rmtakata" <rmtakata@a...>
escreveu
>
> --- Em ciencialist@yahoogrupos.com.br, "pubmed2005"
> > céticos tem um pouquinho de cagaço que o manto seja mesmo de
> > Jesus né?
>
> Usando de seu termo chulo, parece q. ocorre o contrario. Os crentes
> continuam procurando 'justificativas' para considerar o manto como
> verdadeiro.

Chulo eu acho essas terminologias: Crentes e céticos. Céticos podem
muito bem serem crentes de suas posições afirmações e crentes serem
céticos com relação a algumas coisas. Portanto a siginificação e
definição de crença podem um teor bem mais profundo e abranger tb
aos "céticos de plantão"

>
> > O exame do carbono 14 foi muito importante para a ciencia,
> > mas podemos ser bastates críticos e afirmar que ele foi
> > insuficiente pelo pouco tempo que os cientistas tiveram o
> > manto disponíveis para a pesquisa.
>
> Esse tempo nao interfere no resultado do carbono-14.

Voce fala como o carbono -14 fosse infalível. Ele pode ser
instrumento relativamente confiável, mas nao infalível. E é claro que
um tempo de estudo maior interferia nos resultados sim. A
quantificação do trabalho poderia fornecer outros indícios e
contradizer alguns resultados. Lembrando que a ciencia é
imprevisível, e nao tao previsível como voce parece afirmar
positivamente

>
> > e poderíamos testar e retestar as pésquisas do queímico e
> > as diferentes análises de outras partes do tecido pelo
> > carbono-14
>
> Sim e aih acabaria-se o manto. Lembre-se de q. o teste foi feito por
> *tres* laboratorios independentes e os resultados foram semelhantes.

Claro que nao precisaria acabar com o manto. Mais intensificar as
pesquisas. Arqueólogos e paleoentologos nao destroem o material, mas
trabalham sobre ele realizando testes mais concisos e precisos. O
manto ja nao tem essa liberação

>
> Ainda estao pra provar q. analise quimica seja uma metodologia de
> datacao mais confiavel do q. radiodatacao.


Sua informação é muito limitada. Os indicios químicos podem fornecer
analises tão importantes quanto a datação. Elas podem nao datar com
precisão, mais podem fornecer elementos chaves de completar os
estudos, ou até mesmo descobrir informações que possam colidir com
outras.

Lembrando que a intenção do segundo estudo nao é refutar o primeiro.
E sim fornecer elementos-chaves que podem explicar a origem do manto.
Se bem que isso pode acabar com a refutação
E ninguem pode garantir com exatidão que o carbono -14 seja realmente
um instrumento tão preciso a ponto de acertar com precisão a data
correta e verdadeira de um objeto no tempo. Pode ser o melhor
instrumento até agora descoberto, mas nao o mais preciso.






SUBJECT: Re: [ciencialist] Re: ninguem é inocente.
FROM: "Oraculo" <oraculo@atibaia.com.br>
TO: <ciencialist@yahoogrupos.com.br>
DATE: 19/03/2005 00:59

Olá Pubmed

"Pubmed:E quem disse que o novo estudo tem a intenção de refutar o primeiro
ou pelo menos derruba-lo? "

Sejamos razoáveis..:-) Não é possível que os dois esrtudos estejam corretos, assim, se um deles estiver correto, o outro será refutado. Não é possível fugir disso..:-)

Pubmed: Tenho a impressão que o manto nao foi exaustivamente verificado como
deveria."

Tem razão, mas isso é responsabilidade exclusiva de seu proprietário, a igreja catolica, que deveria ser, se fosse real, a maior interessada emd escobrir formas de provar isso..:-)

Pubmed:Voce se apegando ao primeiro estudo, e fazendo pouco caso do segundo
nao está agindo como um cético, e sim como aqueles céticos da
internet como eu havia falado, sinto muito."

Na verdade não, eu estou apenas dando maior confiabilidade as conclusões que se embasam em estudos já reproduzidos, que a um estudo ainda único, com metodologia diferente, que alega ter encontrado resulttados diferentes do primeiro estudo. Isso é ser cético da forma correta e razoável, não como "os da internet"..:-)

Não é pouco caso, é cuidado necessário comm um estudo único, que contradiz uma conclusão anterior mais bem embasada. Quando, e "se", este novo estudo for reproduzido, e se os novos experimentos confirmarem os dados, isso pode mudar. Até lá. é bom se manter cuidadoso e aceitar como mais confiável os estudos que tem maior confiabilidade.

Pubmed: Eu tb nunca falei que o primeiro estudo estava errado...apenas eu
disse que o segundo levantou questões muito interessantes que podem
mudar todo o aparato dogmático que envolve as afirmações
experimentais do primeiro estudo

Não há muita escolha, se o segundo está correto, e o manto tem até 3000 anos, o primeiro, e mesmo os estudos que o reproduziram, estão errados. Não precisa afirmar isso explicitamente, é uma conclusão lógica decorrente dos fatos.

E claro que concordo com você, o novo estudo apresenta um novo angulo, quimico, mas este tem de ser aidna confirmado. Sem isso, a datação por carbono, repetida mais de uma vez, tem prioridade e confiabilidade maior.

Pubmed: E outra coisita, quem disse que os céticos estão isento de crenças e
que estão isentos de criticar suas proprias crenças com o mesmo rigor
que criticam as alheias?

Não estão..:-) Por isso insistem tanto em que se reproduzam os estudos, para garantir que crenças pessoais, mesmo as nossas, não interfiram nos resultados..:-) É a compreensão que todos, seres humanos falhos, temos filtros e podemos distorcer os resultados, que resultou no rigoroso método chamado de cientifico. É justametne essa compreensão que exige que se seja tão cuidadoso e crítico, que se repita tudo em laboratorios independentes, que pesquisadores neutros refaçam os experimentos, etc.

É exatamente por saber que isso que afirmou acima é verdade, que insistimos tanto no método, no rigor e no cuidado com conclusões..:-) Não é, na verdade, uma crítica que está fazendo, é apenas a base do ceticismo, uma virtude, não um defeito, que as falhas de seres humanos sejam levadas em conta ao analisar qualquer alegação ou conclusão.

Pubmed: Mesmo que as crenças dos céticos sejam de
nartureza diferentes das místicas, acho isso muito suspeito de se
dizer, e até mesmo uma posição nao-cética a de dizer que nao se tem
crenças e que dedicam o mesmo tempo de exposição a que se dedicam as
crenças alheias."

Aqui temos um engano comum, derivado das diferentes definições do termo "acreditar"..:-) A crença mistica e a crença cética são coisas diferentes, embora usem o mesmo termo.

Um exemplo prático, quando um catolico afirma que "crê" em anjos, ele não quer dizer o mesmo que um engenheiro que afirma que "acredita" que o predio cairá corretamente com a implosão. No primeiro caso, é uma crença derivada de fé, que significa "acreditar sem ter motivo para tanto". No segundo caso, é uma forma resumida de dizer algo do tipo "eu, com base no conhecimento de física, resistencia de materiais, colocação dos explosivos, quimica de explosivos, fundação e concretagem do edificio, leis da fisica, etc, CONCLUO que o edifício cairá corretamente com a implosão." .

É uma conclusão, não uma crença. E como tal, pode ser modificada, alterada, revista, refutada ou outra coisa qualquer, sem dor ou dano. Baseia-se em dados e evidências. No caso do catolico, nada mudará sua crença, ela não vai se modificar, alterar, ser refutada ou ajustada. Deriva da fé e de mais nada. Nem evidencias nem argumentos tem qualquer efeito na mesma.

Assim, crenças e conclusões são diferentes, e não podem ser comparadas ou equiparadas.

Um abraço.

Homero


----- Original Message -----
From: pubmed2005
To: ciencialist@yahoogrupos.com.br
Sent: Saturday, March 19, 2005 12:33 AM
Subject: [ciencialist] Re: ninguem é inocente.



E quem disse que o novo estudo tem a intenção de refutar o primeiro
ou pelo menos derruba-lo? Ele apenas levantou a questão com bases em
premissas científicas que o manto deveria ter aproximadamente 3000
anos, e nao apenas os quase exatos 1300 anos do carbono-14

Tenho a impressão que o manto nao foi exaustivamente verificado como
deveria. Portanto o primeiro estudo pode estar cheio de erros e
imprecisões.

E segundo, a análise química do manto é tb muito importante, e
portanto a segunda pesquisa nao seria uma rival da primeira, e sim
uma complementação , ou um questionamento necessário a primeira
posição.

de qualquer forma precisaríamos fazer uma aanálise mais completa do
tecido, antes de chegar uma conclusão, lou fazer afirmações que o
manto tenha que ter inexoravelmente 1300 anos

Voce se apegando ao primeiro estudo, e fazendo pouco caso do segundo
nao está agindo como um cético, e sim como aqueles céticos da
internet como eu havia falado, sinto muito.

Eu tb nunca falei que o primeiro estudo estava errado...apenas eu
disse que o segundo levantou questões muito interessantes que podem
mudar todo o aparato dogmático que envolve as afirmações
experimentais do primeiro estudo

E outra coisita, quem disse que os céticos estão isento de crenças e
que estão isentos de criticar suas proprias crenças com o mesmo rigor
que criticam as alheias? Mesmo que as crenças dos céticos sejam de
nartureza diferentes das místicas, acho isso muito suspeito de se
dizer, e até mesmo uma posição nao-cética a de dizer que nao se tem
crenças e que dedicam o mesmo tempo de exposição a que se dedicam as
crenças alheias.


--- Em ciencialist@yahoogrupos.com.br, "Oraculo" <oraculo@a...>
escreveu
> Olá Pubmed
>
> "Pubmed: No fundo , no fundo os céticos tem um pouquinho de cagaço
que o manto seja mesmo de Jesus né?"
>
> Não, não temos..:-) Eu pelo menos não tenho nenhum..:-) Veja, se
for de jesus, isso significa que existiu um cara chamado jesus,
apenas isso. Não significa muito mais que isso, e isso apenas é algo
a ser considerado e estudado, não significa que todos nós devemos nos
tornar católicos ou cristãos..:-)
>
> E eu tentei explicar, e talvez não tenha sido claro, porque céticos
rejeitam estudos. É porque rejeitar estudos é parte fundamental do
método cientifico..;-) Apenas se sobreviver a todas as rejeições, a
todas as analises críticas, a todas as tentativas de refutação, é que
um conhecimento, uma alegação, uma conclusão pode ser chamada de
científica. Não é uma rejeição especial, contra estudos sobre o
manto, é uma rejeição geral, parte da ferramenta conhecida como
método científico.
>
> Como eu disse antes, foi aplicada até mesmo contra o primeiro
estudo, de carbono-14. Apenas com as confirmações subsequentes, que
validaram os dados obtidos no primeiro estudo, é que foram aceitos.
Por que acha que foram feitos novos estudos com o mesmo processo, se
o primeiro teria sido aceito sem contestação e apenas com
aplausos?..:-) Ainda assim, podem ser refutados ou descartados, se
novos estudos modificarem as conclusões.
>
> Como este novo que você defende. Mas, para fazer isso, para ser
capaz de refutar o anterior, este novo estudo tem de ser tão forte
quanto, pelo menos, o que pretende refutar. Tem de sobreviver a
tantas críticas quanto o anterior, tem de ter tantos estudos de
confirmação quanto o primeiro. Só assim será aceito como ciência
legítima.
>
> Isso aconteceu com todo conhecimento cientifico, não apeaans com
alegações religiosas. Com a fusão a frio, que foi refutada, e com a
Deriva dos Continentes, que foi finalmente aceita. As duas, alegações
cientificas, não sobrenaturais ou esotericas.
>
> Pubmed: Apostaria que se fosse uma pesquisa que validasse apenas os
1300 anos ou até
> menos que isso, os céticos aplaudiriam e seriam menos críticos "
>
> risos..:-) Não perca seu dinheiro assim tão fácil..:-) Se
apostasse, perderia. Céticos não apenas não aplaudiriam como fariam
os mesmos questionamentos, que é preciso esperar que estudos
comprovem a conclusão, que esta possa ser repetida, etc. É parte do
método, sem ele não valeria nada uma alegação. Está confundindo
céticos com torcedores de futebol, que, não importa o quão mal seus
times joguem, sempre é culpa do juiz..:-) Claro que existem os que
são contra a igreja, ou mesmo de outra religião, e torcem para que
ela se de mal, para que o manto seja uma fraude. Estes, embora as
vezes se disfarcem de céticos, na verdade creem, mas creem em outras
crenças. É como no antigo ditado, o inimigo de meu inimigo, meu amigo
é..:-)
>
> Como católicos (Padre Quevedo, conhece?) que usam argumentos
céticos contra alegações espíritas..:-) Não são céticos de verdade,
já que evitam usar os mesmos argumentos contra suas próprias
crenças, mas um tipo de torcedor..:-) Somos todos céticos com as
crenças alheias..:-)
>
> Um abraço.
>
> Homero
>
>
>
> ----- Original Message -----
> From: pubmed2005
> To: ciencialist@yahoogrupos.com.br
> Sent: Saturday, March 19, 2005 12:01 AM
> Subject: [ciencialist] Re: ninguem é inocente.
>
>
>
> Ok oráculo , tudo bem
>
> Mas deixa eu perguntar uma coisa. No fundo , no fundo os céticos
tem
> um pouquinho de cagaço que o manto seja mesmo de Jesus né?
>
> Não vejo outra explicação quando alguém do meio cético rejeita(
ou
> faz uma analise tão critica de uma pesquisa tão importante.
Apostaria
> que se fosse uma pesquisa que validasse apenas os 1300 anos ou
até
> menos que isso, os céticos aplaudiriam e seriam menos críticos
>
> O exame do carbono 14 foi muito importante para a ciencia, mas
> podemos ser bastates críticos e afirmar que ele foi insuficiente
pelo
> pouco tempo que os cientistas tiveram o manto disponíveis para a
> pesquisa. e poderíamos testar e retestar as pésquisas do queímico
e
> as diferentes análises de outras partes do tecido pelo carbono-14
>
> Outra coisa que gostaria de explicar aos pertecentes a esta
lista,
> quando uso o termo cético nao é no sentido pejorativo de
ceticismo
> como uma coisa má e ruim, mas uma análise sobre este ceticismo
> ilógico que reina pela internet
>
>
> --- Em ciencialist@yahoogrupos.com.br, "Oraculo" <oraculo@a...>
> escreveu
> > Olá Pubmed
> >
> > Desculpe, onde foi que eu, ou qualquer outro, disse ou escreveu
> que "só ceticos tem cérebro" ou que "só ceticos podem pensar"?
Não é,
> claro, um argumento, apenas um ataque do tipo "ad hominem", mas
> gostaria de saber se algo que eu escrevi tenha dado essa ideia ou
> poderia ser interpretado dessa forma..:-)
> >
> > Em todo caso, se eu achasse isso, não estaria debatendo com
você e
> com a lista, com cuidado e educação, expondo meus argumentos e
lendo
> atentamente os seus..:-) É um debate, não uma guerra, e não
pretendo
> faze-lo mudar de ideia ou aceitar o que quer que seja. Apenas
> apresento meu ponto de vista, meus argumentos e minhas
evidencias,
> esperando que essa troca traga algo de útil a nós dois.
> >
> > Um abraço.
> >
> > Homero
> >
> > ----- Original Message -----
> > From: pubmed2005
> > To: ciencialist@yahoogrupos.com.br
> > Sent: Friday, March 18, 2005 10:50 PM
> > Subject: [ciencialist] Re: ninguem é inocente.
> >
> >
> >
> > Voce esqueceu de completar sua retórica perguntando o que é
> > ferramenta da razão para pensar...
> > Será que é o cérebro?...ah esqueci de dizer que só
os "céticos
> tem
> > cérebros" e "que só eles podem pensar"
> >
> >
> > --- Em ciencialist@yahoogrupos.com.br, "Alberto Mesquita
Filho"
> > <albmesq@u...> escreveu
> > > ----- Original Message -----
> > > From: "Oraculo"
> > > Sent: Friday, March 18, 2005 10:02 PM
> > > Subject: Re: [ciencialist] Re: ninguem é inocente.
> > >
> > > > sou um cético de verdade usando a ferramenta da razão
para
> pensar
> > sobre as
> > > > alegações e afirmativas que são apresentadas..:-)
> > >
> > > Existem céticos de verdade? Afinal, o que é ceticismo? Ou
> então: O
> > que é
> > > verdade?
> > >
> > > Não precisa responder. Eu só queria perguntar. ;-))
> >
> >
> >
> >
> >
> >
> > ##### ##### #####
> >
> > Para saber mais visite
> > http://www.ciencialist.hpg.ig.com.br
> >
> >
> > ##### ##### ##### #####
> >
> >
> > Yahoo! Grupos, um serviço oferecido por:
> >
> > São Paulo Rio de Janeiro Curitiba Porto
Alegre
> Belo Horizonte Brasília
> >
> >
> >
> >
> > ----------------------------------------------------------------
----
> ----------
> > Links do Yahoo! Grupos
> >
> > a.. Para visitar o site do seu grupo na web, acesse:
> > http://br.groups.yahoo.com/group/ciencialist/
> >
> > b.. Para sair deste grupo, envie um e-mail para:
> > ciencialist-unsubscribe@yahoogrupos.com.br
> >
> > c.. O uso que você faz do Yahoo! Grupos está sujeito aos
Termos
> do Serviço do Yahoo!.
> >
> >
> >
> > [As partes desta mensagem que não continham texto foram
removidas]
>
>
>
>
>
> ##### ##### #####
>
> Para saber mais visite
> http://www.ciencialist.hpg.ig.com.br
>
>
> ##### ##### ##### #####
>
>
> Yahoo! Grupos, um serviço oferecido por:
> PUBLICIDADE
>
>
>
>
> --------------------------------------------------------------------
----------
> Links do Yahoo! Grupos
>
> a.. Para visitar o site do seu grupo na web, acesse:
> http://br.groups.yahoo.com/group/ciencialist/
>
> b.. Para sair deste grupo, envie um e-mail para:
> ciencialist-unsubscribe@yahoogrupos.com.br
>
> c.. O uso que você faz do Yahoo! Grupos está sujeito aos Termos
do Serviço do Yahoo!.
>
>
>
> [As partes desta mensagem que não continham texto foram removidas]





##### ##### #####

Para saber mais visite
http://www.ciencialist.hpg.ig.com.br


##### ##### ##### #####


Yahoo! Grupos, um serviço oferecido por:

São Paulo Rio de Janeiro Curitiba Porto Alegre Belo Horizonte Brasília




------------------------------------------------------------------------------
Links do Yahoo! Grupos

a.. Para visitar o site do seu grupo na web, acesse:
http://br.groups.yahoo.com/group/ciencialist/

b.. Para sair deste grupo, envie um e-mail para:
ciencialist-unsubscribe@yahoogrupos.com.br

c.. O uso que você faz do Yahoo! Grupos está sujeito aos Termos do Serviço do Yahoo!.



[As partes desta mensagem que não continham texto foram removidas]



SUBJECT: Re: ninguem é inocente.
FROM: "rmtakata" <rmtakata@altavista.net>
TO: ciencialist@yahoogrupos.com.br
DATE: 19/03/2005 01:38


Salve, Julio,

--- Em ciencialist@yahoogrupos.com.br, "pubmed2005"
> Chulo eu acho essas terminologias: Crentes e céticos.

Ueh, vc comecou usando o termo cetico. Enfim...

> > Esse tempo nao interfere no resultado do carbono-14.
>
> Voce fala como o carbono -14 fosse infalível. Ele pode ser
> instrumento relativamente confiável, mas nao infalível.

Claro q. pode falhar. Tudo pode falhar.

> Claro que nao precisaria acabar com o manto. Mais
> intensificar as pesquisas. Arqueólogos e paleoentologos nao
> destroem o material, mas trabalham sobre ele realizando
> testes mais concisos e precisos.

Arqueologos e paleontologos tb destroem material. A datacao por C-14
eh um metodo destrutivo. Em paleontologia existe um processo de
fatiamento de fosseis q. o destroi, mas revela a anatomia fina
interna. O q. se faz eh usar amostras pequenas e geralmente em
material q. possui outros especimes - o tal sudario eh um especime
unico e daih o xodo' com q. a Igreja o trata.

> Sua informação é muito limitada. Os indicios químicos podem
> fornecer analises tão importantes quanto a datação.

Estou falando de datacao. Se a data eh de 1300 analises complementares
podem enriquecer com outras informacoes - q. tecnica foi usada por
exemplo. Mas isso nao faria com q. o manto pudesse ser original.

> ou até mesmo descobrir informações que possam colidir com
> outras.

Se colidir, como a tal analise da vanilina colide, como eu disse,
ainda estah para se provar q. essa metodologia eh mais confiavel do q.
a datacao por C-14.

> Pode ser o melhor instrumento até agora descoberto, mas nao
> o mais preciso.

Eh isto q. estou falando Siqueira: ainda estah para se provar q. a
datacao quimica eh mais confiavel do q. a datacao por radioisotopos.

[]s,

Roberto Takata






SUBJECT: Re: ninguem é inocente.
FROM: "pubmed2005" <pubmed2005@yahoo.com.br>
TO: ciencialist@yahoogrupos.com.br
DATE: 19/03/2005 01:50


Comentarios se seguem

--- Em ciencialist@yahoogrupos.com.br, "Oraculo" <oraculo@a...>
escreveu
> Olá Pubmed
>
> "Pubmed:E quem disse que o novo estudo tem a intenção de refutar o
primeiro
> ou pelo menos derruba-lo? "
>
> Sejamos razoáveis..:-) Não é possível que os dois esrtudos estejam
corretos, assim, se um deles estiver correto, o outro será refutado.
Não é possível fugir disso..:-)

Os dois nao podem estar corretos e nao podem se coauxiliarem? Nao
vejo impedimento para isso. Se bem que Rogers se baseou na idade
minima para o manto como os 1300 anos datados na primeira pesquisa,
mas nao se limitou a isso, vai em busca de mais dados que foram
encontrados, que poderiam ampliar a margem de anos. A ausensia de
vanalina na maior parte dos tecidos é uma evidencia e tanta...já que
ela só encontradas em tecidos mais recentes(Só nao sei estipular o
limite e a margem de anos, talvez quem saiba um químico queira se
manifestar)
Acho dificil um estudo que foi uma fraude publicado pela nature, mas
em todo o caso....


> Não é pouco caso, é cuidado necessário comm um estudo único, que
contradiz uma conclusão anterior mais bem embasada. Quando, e "se",
este novo estudo for reproduzido, e se os novos experimentos
confirmarem os dados, isso pode mudar. Até lá. é bom se manter
cuidadoso e aceitar como mais confiável os estudos que tem maior
confiabilidade.

Nao contradiz no todo, tanto que a idade minima estabelecida por
Rogers é de 1300 anos. Ele apenas amplia pesquisa com as novas
evidencias de substancias e a ausencia delas, e deduz que a margem de
anos pode ser aumentada com base em evidencias químicas

>
> Não há muita escolha, se o segundo está correto, e o manto tem até
3000 anos, o primeiro, e mesmo os estudos que o reproduziram, estão
errados. Não precisa afirmar isso explicitamente, é uma conclusão
lógica decorrente dos fatos.

Mas o segundo nao diz enfatica e inexoravelmente que o manto tem 3000
anos, isto pode ser uma interpretação mal dada feita pelos
jornalistsas. Ele deu uma data aproximada entre 1300 e 3000 anos

>
> E claro que concordo com você, o novo estudo apresenta um novo
angulo, quimico, mas este tem de ser aidna confirmado. Sem isso, a
datação por carbono, repetida mais de uma vez, tem prioridade e
confiabilidade maior.

Todos os estudos tem de ser repetidos se possível até a exaustao.
Ciencia nao deve ser encarada como uma uma balança comparativa que
diz o primeiro é superior ao segundo estudo. Isso seria uma
estratégia positivista com medidas cientifistas. Mas claro que os
resultados podem ser comparados



>
> Aqui temos um engano comum, derivado das diferentes definições do
termo "acreditar"..:-) A crença mistica e a crença cética são coisas
diferentes, embora usem o mesmo termo.
ovidas]


Crença é crença, nao importa se misticas ou não. Se voce acredita que
é Superhomem ou o Tio Patinhas isso soa tão ridículo que voce achar
que é o caboclo das 7 encruzilhadas





SUBJECT: Re: [ciencialist] Re: ninguem é inocente.
FROM: "Oraculo" <oraculo@atibaia.com.br>
TO: <ciencialist@yahoogrupos.com.br>
DATE: 19/03/2005 02:05

Olá Pubmed

Pubmed: Acho dificil um estudo que foi uma fraude publicado pela nature, mas
em todo o caso...."

Um estudo que se descobre errado e uma fraude são coisas diferentes. Ninguém disse que o primeiro estudo de fusão a frio foi uma fraude, disse que não pode ser repetido e portando, deveria ser refeito para se descobrir o erro. O mesmo para este estudo sobre o manto. Se novos estudos com o mesmo método quimico encontrarem os mesmos dados, este será confirmado. Se não descobrirem, será refutado. Mas, em ambos os casos, não se trata de fraude, mas de engano legitimo, comum na ciência, e que deve ser analisado para aprendermos mais um pouco.

Pubmed: Os dois nao podem estar corretos e nao podem se coauxiliarem?"

Não, porque suas conclusões são contraditórias. A datação indica que não pode ser mais velho que 640 anos, o estudo novo afirma que deve ter entre 1300 e 3000 anos. Não podem ser ambos corretos.

"Pubmed: Ele apenas amplia pesquisa com as novas
evidencias de substancias e a ausencia delas, e deduz que a margem de
anos pode ser aumentada com base em evidencias químicas"

De novo não. A datação é limitante, não se pode ter um artefato com datação carbono 14 mais antiga que o medido. Se leu o artigo do Takata sobre o método de datação, deve compreender o porque dessa limitação.

Pubmed: Ciencia nao deve ser encarada como uma uma balança comparativa que
diz o primeiro é superior ao segundo estudo."

Pode, se usar os parametros corretos para isso. Não superior, mas mais confiável. Um estudo que já é bem conhecido, com uma técinica que tem se mostrado eficaz, e que tem diversos estudos confirmatórios é mais confiável, neste momento, que outro sem nada disso, com uma tecnica recem desenvolvida e sem nenhum estudo de confirmação. É perfeitamente legítimo tomar o primeiro como mais confiável (e não superior, conceito muito subjetivo).

Pubmed: Crença é crença, nao importa se misticas ou não. Se voce acredita que
é Superhomem ou o Tio Patinhas isso soa tão ridículo que voce achar
que é o caboclo das 7 encruzilhadas"

Nos casos citados, tem toda razão..:-) São identicos..:-) Mas, a questão era sobre crenças misticas e céticas. Céticos dificilmente acreditariam ser o super homem ou o caboclo..:-) Mas eles acreditam que a Terra é redonda.

Neste caso, é um termo diferente, com uso e conceito diferente. "Acredito" que a Terra é redonda é uma forma de resumir uma conclusão. Na verdade, céticos e cientistas concluem que a Terra é redonda, baseados em evidencias diversas, todas bem documentadas. O que difere de forma gritante de acreditar ser o Tio Patinhas..:-)

Um abraço.

Homero



----- Original Message -----
From: pubmed2005
To: ciencialist@yahoogrupos.com.br
Sent: Saturday, March 19, 2005 1:50 AM
Subject: [ciencialist] Re: ninguem é inocente.



Comentarios se seguem

--- Em ciencialist@yahoogrupos.com.br, "Oraculo" <oraculo@a...>
escreveu
> Olá Pubmed
>
> "Pubmed:E quem disse que o novo estudo tem a intenção de refutar o
primeiro
> ou pelo menos derruba-lo? "
>
> Sejamos razoáveis..:-) Não é possível que os dois esrtudos estejam
corretos, assim, se um deles estiver correto, o outro será refutado.
Não é possível fugir disso..:-)

Os dois nao podem estar corretos e nao podem se coauxiliarem? Nao
vejo impedimento para isso. Se bem que Rogers se baseou na idade
minima para o manto como os 1300 anos datados na primeira pesquisa,
mas nao se limitou a isso, vai em busca de mais dados que foram
encontrados, que poderiam ampliar a margem de anos. A ausensia de
vanalina na maior parte dos tecidos é uma evidencia e tanta...já que
ela só encontradas em tecidos mais recentes(Só nao sei estipular o
limite e a margem de anos, talvez quem saiba um químico queira se
manifestar)
Acho dificil um estudo que foi uma fraude publicado pela nature, mas
em todo o caso....


> Não é pouco caso, é cuidado necessário comm um estudo único, que
contradiz uma conclusão anterior mais bem embasada. Quando, e "se",
este novo estudo for reproduzido, e se os novos experimentos
confirmarem os dados, isso pode mudar. Até lá. é bom se manter
cuidadoso e aceitar como mais confiável os estudos que tem maior
confiabilidade.

Nao contradiz no todo, tanto que a idade minima estabelecida por
Rogers é de 1300 anos. Ele apenas amplia pesquisa com as novas
evidencias de substancias e a ausencia delas, e deduz que a margem de
anos pode ser aumentada com base em evidencias químicas

>
> Não há muita escolha, se o segundo está correto, e o manto tem até
3000 anos, o primeiro, e mesmo os estudos que o reproduziram, estão
errados. Não precisa afirmar isso explicitamente, é uma conclusão
lógica decorrente dos fatos.

Mas o segundo nao diz enfatica e inexoravelmente que o manto tem 3000
anos, isto pode ser uma interpretação mal dada feita pelos
jornalistsas. Ele deu uma data aproximada entre 1300 e 3000 anos

>
> E claro que concordo com você, o novo estudo apresenta um novo
angulo, quimico, mas este tem de ser aidna confirmado. Sem isso, a
datação por carbono, repetida mais de uma vez, tem prioridade e
confiabilidade maior.

Todos os estudos tem de ser repetidos se possível até a exaustao.
Ciencia nao deve ser encarada como uma uma balança comparativa que
diz o primeiro é superior ao segundo estudo. Isso seria uma
estratégia positivista com medidas cientifistas. Mas claro que os
resultados podem ser comparados



>
> Aqui temos um engano comum, derivado das diferentes definições do
termo "acreditar"..:-) A crença mistica e a crença cética são coisas
diferentes, embora usem o mesmo termo.
ovidas]


Crença é crença, nao importa se misticas ou não. Se voce acredita que
é Superhomem ou o Tio Patinhas isso soa tão ridículo que voce achar
que é o caboclo das 7 encruzilhadas





##### ##### #####

Para saber mais visite
http://www.ciencialist.hpg.ig.com.br


##### ##### ##### #####


Yahoo! Grupos, um serviço oferecido por:
PUBLICIDADE




------------------------------------------------------------------------------
Links do Yahoo! Grupos

a.. Para visitar o site do seu grupo na web, acesse:
http://br.groups.yahoo.com/group/ciencialist/

b.. Para sair deste grupo, envie um e-mail para:
ciencialist-unsubscribe@yahoogrupos.com.br

c.. O uso que você faz do Yahoo! Grupos está sujeito aos Termos do Serviço do Yahoo!.



[As partes desta mensagem que não continham texto foram removidas]



SUBJECT: Re: ninguem é inocente.
FROM: "pubmed2005" <pubmed2005@yahoo.com.br>
TO: ciencialist@yahoogrupos.com.br
DATE: 19/03/2005 02:12


Po Takata...Júlio é sacanagem

Se voce tivesse falando Bradock sua datação talvez estivesse mais
proxima dos parametros corretos

Se bem que até creio que voce em seu discernimento concorde que entre
o Bradock e o Júlio há um diferença ideológica bem siginificativa.

Mais, quanto aos estudos...nao acho que os estudos químicos sejam
mais irrelevantes que os radioisotópicos
Eu só não me identifiquei porque acho que no fundo alguém iria acabar
descobrindo minha verdadeira identidade, ou até mesmo barrar minha
entrada na lista. Mas nao resisti minha indignação ao ser chamado de
Julio...hehehe Mas imitando Sagan, eu estava enganado. Mas vamos ao
que interessa:


--- Em ciencialist@yahoogrupos.com.br, "rmtakata" <rmtakata@a...>
escreveu
>
> Salve, Julio,
>
> --- Em ciencialist@yahoogrupos.com.br, "pubmed2005"
> > Chulo eu acho essas terminologias: Crentes e céticos.
>
> Ueh, vc comecou usando o termo cetico. Enfim...

Pois é , cada qual com sua obsessão. O fato de eu falar mal
dos "céticos" nao quer dizer que eu nao seja um deles, ou que eu
esteja fazendo proselitismo ao crentinismo


>
> > Claro que nao precisaria acabar com o manto. Mais
> > intensificar as pesquisas. Arqueólogos e paleoentologos nao
> > destroem o material, mas trabalham sobre ele realizando
> > testes mais concisos e precisos.
>
> Arqueologos e paleontologos tb destroem material. A datacao por C-14
> eh um metodo destrutivo. Em paleontologia existe um processo de
> fatiamento de fosseis q. o destroi, mas revela a anatomia fina
> interna. O q. se faz eh usar amostras pequenas e geralmente em
> material q. possui outros especimes - o tal sudario eh um especime
> unico e daih o xodo' com q. a Igreja o trata.

Sim , mais isso é muito diferente de acabar com o manto. O problemA é
que o manto agora está sendo usado para tapar a traqueostomia do Papa

>
> > Sua informação é muito limitada. Os indicios químicos podem
> > fornecer analises tão importantes quanto a datação.
>
> Estou falando de datacao. Se a data eh de 1300 analises
complementares
> podem enriquecer com outras informacoes - q. tecnica foi usada por
> exemplo. Mas isso nao faria com q. o manto pudesse ser original.

Nao entendi. O que eu queria dizer é se a substancia vanalina é
encontrada apenas em tecidos recentes o manto pode ter mesmo a
datação verificada. O problema é que o Rogers diz que o resto do
manto nao tem essa substancia, o que pode fazer com que o manto tenha
muito mais idade. A informação química colide com a datação. No caso
seria plausível dizer que a informação radioisotópica é superior a
química? Nem é isso que está em questão acho eu

>
> Se colidir, como a tal analise da vanilina colide, como eu disse,
> ainda estah para se provar q. essa metodologia eh mais confiavel do
q.
> a datacao por C-14.

Nao é quesçao de ser mais confiável. A ausencia de vanilina no resto
do manto demostra que o manto é muito mais antigo que parece. Essa
substanci nao resiste a ação do tempo. Só queria saber qual a
informação que os químicos tem sobre esta substancia na forma de uma
detecção temporal

>
> > Pode ser o melhor instrumento até agora descoberto, mas nao
> > o mais preciso.
>
> Eh isto q. estou falando Siqueira: ainda estah para se provar q. a
> datacao quimica eh mais confiavel do q. a datacao por radioisotopos.


De novo nao, Takata. pel primeira vez o computador Takara falhou.
Ufa!!! Nao sou o Siqueira. Sou o Chuk Norris.
Mas indo ao que interessa nao é questao de mais confiável no caso,
mas de que uma informação colidir com a outra.
O tecido por inteiro teria que apresentar vanilina para que pudesse
pertencer a Idade Média, o que naso parece ser o caso. Mas em todo
caso, nuncaa quis invalidar o primeiro estudo, apenas questiona-lo em
partes e ultrapassar seu limite inexorável(nem tanto) de datação





SUBJECT: Re: ninguem é inocente.
FROM: "pubmed2005" <pubmed2005@yahoo.com.br>
TO: ciencialist@yahoogrupos.com.br
DATE: 19/03/2005 02:27


Vamos lá

--- Em ciencialist@yahoogrupos.com.br, "Oraculo" <oraculo@a...>
escreveu
> Olá Pubmed
>
> Pubmed: Acho dificil um estudo que foi uma fraude publicado pela
nature, mas
> em todo o caso...."
>
> Um estudo que se descobre errado e uma fraude são coisas
diferentes. >


O mesmo deve valer para o teste do carbono-14. Nada é definitivo em
Ciencia

> Pubmed: Os dois nao podem estar corretos e nao podem se
coauxiliarem?"
>
> Não, porque suas conclusões são contraditórias. A datação indica
que não pode ser mais velho que 640 anos, o estudo novo afirma que
deve ter entre 1300 e 3000 anos. Não podem ser ambos corretos.

Quer dizer que o primeiro estudo impoe um limite que nao pode ser
ultrapassado? Ué, se ele coloca idade entre 1330 a 3000, é bem
possível que o manto possa ter realmente 1300 anos ou 3000 anos. se
tiver entre as duaas datas ele nao paga feio. Mas se o limite do
primeiro for provado errado essa verificação cai por terra sem perdão.
A nao ser que o químico tava de sacanagem e colocou esotericamente
esses limites só para nao pagar feio...o que nao acho

> De novo não. A datação é limitante, não se pode ter um artefato com
datação carbono 14 mais antiga que o medido. Se leu o artigo do
Takata sobre o método de datação, deve compreender o porque dessa
limitação.

Pois é, por isso o método caarbono-14 pode ser falho tb. Nada garante
que ele tenha mesmo 1300 anos no tempo. Nao há garantias em ciencia.
Tudo sao datas aproximadas


>
> Pode, se usar os parametros corretos para isso. Não superior, mas
mais confiável. Um estudo que já é bem conhecido, com uma técinica
que tem se mostrado eficaz, e que tem diversos estudos confirmatórios
é mais confiável, neste momento, que outro sem nada disso, com uma
tecnica recem desenvolvida e sem nenhum estudo de confirmação. É
perfeitamente legítimo tomar o primeiro como mais confiável (e não
superior, conceito muito subjetivo).


O conceito de "mais" confiável é um tanto positivista nao acha?
Porque voce acha que um analise química é menos confiável que uma
radioisotópica? Talvez para datação seja inadequada, mas pode ser um
poderoso auxiliar, e até mesmo um poderoso inimigo na hora de avaliar
os resultados.

> Nos casos citados, tem toda razão..:-) São identicos..:-) Mas, a
questão era sobre crenças misticas e céticas. Céticos dificilmente
acreditariam ser o super homem ou o caboclo..:-) Mas eles acreditam
que a Terra é redonda.

Ué qual a diferença de crença entre achar que o Éter luminífero é um
meio universal presente no todo e achar que o todo universal é a
única realidade última chamada bramahn?
Além do mais , tem céticos que tem a crença que estão pensando e
usando a razão, mas essa é outra história....








SUBJECT: Re: Sudario
FROM: "pubmed2005" <pubmed2005@yahoo.com.br>
TO: ciencialist@yahoogrupos.com.br
DATE: 19/03/2005 02:38


Olá Eduardo eu só gostaria de fazer um breve comentário sobre su
colocação: "Por que então depois do primeiro resultado negativo,
procura impedir outros testes e liberar apenas um
teste oficial feito por um Químico aposentado da
California?"

1- Bem, acho que o fato do químico ser aposentado nao interfere no
resultado da pesquisa.

2-Talvez o fato de ele ser aposentado contribua muito pelo descrédito
de outros cientistas, o que nao invalida o resultado de sua pesquisa.
Muitos cientistas torceram o nariz p/ novas descobertas que foram
confirmadas mais tarde.

3- Esses sites céticos costumama ser tendenciosos, e puxar a farinha
para o saco deles. Portanto devemos ser céticos e criticos para muito
deles. Vamos lá que o Rogers esteja certo mesmo, que micão seria pra
um site desses do Dr. Randi!!!

4- Ser aposentado nao é o mesmo que ser capado para a ciencia. Há
muita fertilidaade nos aposentados. Meu pai que o diga!!!

5- Químicos e biólogos costumam ser preconceituados pelos seus amigos
físicos; Os físicos tem uma ilusão secular que a física é uma ciencia
por excelência, e nao é bem assim




--- Em ciencialist@yahoogrupos.com.br, Eduardo Gueron <edgueron@y...>
escreveu
> Oi a todos,
>
> Voltei a ciencialist motivado pela morte de Cesar
> Lattes, certamente o físico mais bem sucedido de todos
> os tempos no Brasil e que, por algumas de suas
> críticas ferozes, despertou certa antipatia no meio
> medíocre dos físicos atuais, principalmente da máfia
> USPiana. A despeito de discordar de algumas das
> críticas que ele fez a Rel. Geral ou certas paixões
> que nutriu por alguns picaretas, sempre o admirei pela
> sua "falta de mediocridade". Comecei a estudar física
> em 1990 e conheci muitos físicos muito bem sucedidos
> (no conceito predominante que se tem de carreira
> científica) que transbordavam ignorância. Fui à casa
> de Lattes em Campinas duas vezes. Na primeira, entre
> outras coisas, me perguntou se havia lido Voltaire -
> por sorte lera Cândido - e, depois de falar de alguns
> cientistas brilhantes, afirmou que nada se compara às
> artes... Acho que isso o resume bem. Uma pena que após
> a morte de sua grande companheira, a professora de
> história Martha em 2003, Lattes simplesmente se deixou
> morrer.
>
> Bem, agora fazendo jus ao título, afirmo sem o mínimo
> rigor que o Sudário é uma farsa. Argumentos
> científicos são vários, vejam por exemplo o sítio:
> http://brazil.skepdic.com/sudario.html
> Mas podemos usar outras vias lógicas de raciocínio
> para chegar a essa conclusão. Se a Igreja acreditasse
> piamente que o Sudário é fruto de um milagre, não
> teria ela interesse em que a comunidade científica
> fizesse testes a fim de comprovar sua autenticidade?
> Por que então depois do primeiro resultado negativo,
> procura impedir outros testes e liberar apenas um
> teste oficial feito por um Químico aposentado da
> California?
>
> Acho, sinceramente, que muito tempo se perde com essa
> instituição nefasta que é a Igreja católica (minha
> crítica é contra a instituição não contra os
> católicos) . Uma igreja que apresenta estudos dando
> conta de que a camisinha não é eficiente no combate a
> AIDS e cujo Papa condena no mesmo texto o terrorismo e
> o casamento homossexual. E que se calou perante a
> regimes autoritários e violentos na América Latina?
> (Com honrosas exceções como o saudoso D. Helder
> Câmara)
>
> Sou cético e creio (crer?) que o ceticismo é
> necessário para o cientista. Tentar desqualificá-lo
> por causa de bobagens às vezes faladas em fóruns
> céticos é, no mínimo, covardia. Lembra a atitude da
> rede Globo que se aproveitou de um idiota sádico para
> desqualificar os movimentos de defesa dos animais que
> protestam contra rodeios, tema da novela das 8
> atual...(nada a ver a comparação, mas queria falar
> disso em algum lugar)
>
> É isso,
>
> Um abraço, Eduardo.
>
>
> __________________________________________________
> Do You Yahoo!?
> Tired of spam? Yahoo! Mail has the best spam protection around
> http://mail.yahoo.com





SUBJECT: Re: ninguem é inocente.
FROM: "rmtakata" <rmtakata@altavista.net>
TO: ciencialist@yahoogrupos.com.br
DATE: 19/03/2005 02:42


--- Em ciencialist@yahoogrupos.com.br, "pubmed2005"
> o Bradock e o Júlio há um diferença ideológica bem
> siginificativa.

Tem?

> Nao é quesçao de ser mais confiável.

Eh questao de ser mais confiavel. Se duas fontes se contradizem pelo
menos uma delas estah errada.

> Ufa!!! Nao sou o Siqueira. Sou o Chuk Norris.

Ainda preciso descobrir a diferenca.

[]s,

Roberto Takata





SUBJECT: Re: [ciencialist] Re: ninguem é inocente.
FROM: "Oraculo" <oraculo@atibaia.com.br>
TO: <ciencialist@yahoogrupos.com.br>
DATE: 19/03/2005 02:50

Olá Pubmed

Pubmed, esta conversa está a um passo de se tornar sem sentido..:-) Eu estou argumentando sobre suas colocações, mudar de enfoque não ajuda para chegar a uma conclusão..:-)

Vejamos, você diz que o novo estudo não pretende contestar o anterior, que podem ser complementares. Eu digo que não, que um deles refuta o outro e que se um está correto, o outro está errado. E você muda tudo, sem nem tentar refutar o argumento..:-)

Veja:

Pubmed: Quer dizer que o primeiro estudo impoe um limite que nao pode ser
ultrapassado?

Não, quer dizer apenas que se este limite for ultrapassado, o primeiro estudo está incorreto e foi refutado pelo segundo. Se não for ultrapassado, o primeiro é que é o correto e o segundo não. Ou seja, NÃO PODEM COEXISTIR NEM SER COMPLEMENTARES, COMO VOCÊ AFIRMA INICIALMENTE..:-)

Pubmed: Pois é, por isso o método caarbono-14 pode ser falho tb. Nada garante
que ele tenha mesmo 1300 anos no tempo. Nao há garantias em ciencia.
Tudo sao datas aproximadas"

E dai? Se o metodo carbono-14 falhou na determinaão da data, está refutado. Se não falhou, refuta o segundo estudo, quimico. Onde está a evidencia que podem ser complementares, ou seja, o que tem isso a ver com o debate neste ponto?

Lógica. Se o sudario, como determinou o carbono-14, tem não mais que 660 anos, significa que foi criado por volta de 1340 depois de cristo (DC), nunca antes. Se tem entre 1300 e 3000 anos, foi criado entre o ano de 2000 antes de cristo e 700 depois de cristo (DC). Um dos dois estudos tem de estar errado, para que o outro esteja certo. O que está incorreto neste raciocínio??? Um mesmo artefato não pode ter sido criado ANTES do ano de 700 e DEPOIS do ano 1340 ao mesmo tempo!!!!!!!!!

Pubmed: O conceito de "mais" confiável é um tanto positivista nao acha?
Porque voce acha que um analise química é menos confiável que uma
radioisotópica? "

Vamos tentar mais uma vez..:-) Um estudo, qualquer que seja, que foi REPETIDO e encontrou os mesmos resultados, é mais confiável que um que NÃO FOI REPETIDO e portanto não tem confirmação independente. Não importa se é carbono-14 ou datação quimica.

Segundo, o carbono-14 é considerado confiável por ter sido exaustivamente testado e produzido resultados consistentes. A datação quimica, em especial a usada neste segundo estudo, é nova, pouco documentada e ainda a ser validada. Assim, das duas formas, a primeira, carbono-14 é mais confiável.

Dificil de compreender meu argumento? Acho que não..:-)

"Pubmed: Ué qual a diferença de crença entre achar que o Éter luminífero é um
meio universal presente no todo e achar que o todo universal é a
única realidade última chamada bramahn?"

risos..:-) É enorme..:-) No primeiro caso, a conclusão, e não crença, foi abandonada devido a evidencias posteriores que refutaram a teoria do éter. No segundo, nada vai mudar e quem crê nisso vai continuar crendo, sem que evidencias ou provas ou aspectos materiais ou racionais tenham o poder de interferir oiu mudar essa crença. A diferença, como pode notar é enorme..:-) A primeira, uma teoria cientifica, esperava por dados e evidencias que a confirmassem, e se conformou com a refutação por falta de provas. A segunda, criada da imaginação humana, vai se manter indefinidamente, exatamente como foi criada, dependendo apenas da crença subjetiva de seres humanos. Enorme diferença..:-)

Homero




----- Original Message -----
From: pubmed2005
To: ciencialist@yahoogrupos.com.br
Sent: Saturday, March 19, 2005 2:27 AM
Subject: [ciencialist] Re: ninguem é inocente.



Vamos lá

--- Em ciencialist@yahoogrupos.com.br, "Oraculo" <oraculo@a...>
escreveu
> Olá Pubmed
>
> Pubmed: Acho dificil um estudo que foi uma fraude publicado pela
nature, mas
> em todo o caso...."
>
> Um estudo que se descobre errado e uma fraude são coisas
diferentes. >


O mesmo deve valer para o teste do carbono-14. Nada é definitivo em
Ciencia

> Pubmed: Os dois nao podem estar corretos e nao podem se
coauxiliarem?"
>
> Não, porque suas conclusões são contraditórias. A datação indica
que não pode ser mais velho que 640 anos, o estudo novo afirma que
deve ter entre 1300 e 3000 anos. Não podem ser ambos corretos.

Quer dizer que o primeiro estudo impoe um limite que nao pode ser
ultrapassado? Ué, se ele coloca idade entre 1330 a 3000, é bem
possível que o manto possa ter realmente 1300 anos ou 3000 anos. se
tiver entre as duaas datas ele nao paga feio. Mas se o limite do
primeiro for provado errado essa verificação cai por terra sem perdão.
A nao ser que o químico tava de sacanagem e colocou esotericamente
esses limites só para nao pagar feio...o que nao acho

> De novo não. A datação é limitante, não se pode ter um artefato com
datação carbono 14 mais antiga que o medido. Se leu o artigo do
Takata sobre o método de datação, deve compreender o porque dessa
limitação.

Pois é, por isso o método caarbono-14 pode ser falho tb. Nada garante
que ele tenha mesmo 1300 anos no tempo. Nao há garantias em ciencia.
Tudo sao datas aproximadas


>
> Pode, se usar os parametros corretos para isso. Não superior, mas
mais confiável. Um estudo que já é bem conhecido, com uma técinica
que tem se mostrado eficaz, e que tem diversos estudos confirmatórios
é mais confiável, neste momento, que outro sem nada disso, com uma
tecnica recem desenvolvida e sem nenhum estudo de confirmação. É
perfeitamente legítimo tomar o primeiro como mais confiável (e não
superior, conceito muito subjetivo).


O conceito de "mais" confiável é um tanto positivista nao acha?
Porque voce acha que um analise química é menos confiável que uma
radioisotópica? Talvez para datação seja inadequada, mas pode ser um
poderoso auxiliar, e até mesmo um poderoso inimigo na hora de avaliar
os resultados.

> Nos casos citados, tem toda razão..:-) São identicos..:-) Mas, a
questão era sobre crenças misticas e céticas. Céticos dificilmente
acreditariam ser o super homem ou o caboclo..:-) Mas eles acreditam
que a Terra é redonda.

Ué qual a diferença de crença entre achar que o Éter luminífero é um
meio universal presente no todo e achar que o todo universal é a
única realidade última chamada bramahn?
Além do mais , tem céticos que tem a crença que estão pensando e
usando a razão, mas essa é outra história....








##### ##### #####

Para saber mais visite
http://www.ciencialist.hpg.ig.com.br


##### ##### ##### #####


Yahoo! Grupos, um serviço oferecido por:







------------------------------------------------------------------------------
Links do Yahoo! Grupos

a.. Para visitar o site do seu grupo na web, acesse:
http://br.groups.yahoo.com/group/ciencialist/

b.. Para sair deste grupo, envie um e-mail para:
ciencialist-unsubscribe@yahoogrupos.com.br

c.. O uso que você faz do Yahoo! Grupos está sujeito aos Termos do Serviço do Yahoo!.



[As partes desta mensagem que não continham texto foram removidas]



SUBJECT: Re: ninguem é inocente.
FROM: "pubmed2005" <pubmed2005@yahoo.com.br>
TO: ciencialist@yahoogrupos.com.br
DATE: 19/03/2005 02:53


Se eu nao me engano o Siqueira é espirita e espíritualista, eu nao
tenho religião. Pode ser até que combater céticos equivocados seja
minha religiao atual, o que faça eu parecer um pouco com o Julio. Mas
ele combate o ceticismo defendendo a religiao dele, e eu faço
defendendo o equivoco filosófico de alguns céticos de plantão, e
alguns tons pretenciosos de suas faças e escritos.
Mas há de convir que é até uma religião nobre combater o lado negro
do ceticismo")- o ceticismo que é outra forma de religião
E ainda tem o lado das falácias. O Julio costuma escorregar pacas,
mas essa é outra questão, o cara é legal e nao é lícito julga-lo
nesta lista

Os problema é que os falsos céticos sempre recebem mal essas
observações, e não recebem as criticas com o mesmo humor com que as
fazem. Creio que os verdadeiros céticos pouco se importam com minhas
críticas....


Mas isso nao interessa, vamos aos comentários



--- Em ciencialist@yahoogrupos.com.br, "rmtakata" <rmtakata@a...>
escreveu
>
> --- Em ciencialist@yahoogrupos.com.br, "pubmed2005"
> > o Bradock e o Júlio há um diferença ideológica bem
> > siginificativa.
>
> Tem?
>
> > Nao é quesçao de ser mais confiável.
>
> Eh questao de ser mais confiavel. Se duas fontes se contradizem pelo
> menos uma delas estah errada.

O método radioisotópíco nao é mais confiável que o químico. Pode até
ser que eles tenham natureza investigativas distintas, mas um método
nao é mais confiável que outros, apenas tem limites em alguns
setores. O químico pode nao ser datativo, mas pode fornecer
informções importantes sobre outras questões e até mesmo fazer com
que elas colidam nas informações essenciais






SUBJECT: Re: [ciencialist] Re: Sudario
FROM: "Oraculo" <oraculo@atibaia.com.br>
TO: <ciencialist@yahoogrupos.com.br>
DATE: 19/03/2005 02:55

Olá Pubmed

Pubmed: Vamos lá que o Rogers esteja certo mesmo, que micão seria pra
um site desses do Dr. Randi!!!"

Visão estereotipada da ciência e de cientistas. Descobrir que um conhecimento cientifico está incorreto ou que precisa ser ajustado, não é um "mico" para a ciência e para cientistas, pelo contrário. Em geral, quanto maior a descoberta, quanto maior o erro descoberto, maior o prestígio e aplauso que se recebe. Diferente de crenças e religiões, mostrar um erro em ciência dá direito a ser premiado, e não repreendido. Até um premio Nobel é possível ganhar, se sua descoberta for realmente revolucionária.

Descobrir que novos métodos de datação são mais precisos e confiáveis pode garantir um Nobel para o Rogers e aplausos do Randi, não um "mico". É uma critica comum e totalmente erronea a que afirma isso..:-)

Homero


----- Original Message -----
From: pubmed2005
To: ciencialist@yahoogrupos.com.br
Sent: Saturday, March 19, 2005 2:38 AM
Subject: [ciencialist] Re: Sudario



Olá Eduardo eu só gostaria de fazer um breve comentário sobre su
colocação: "Por que então depois do primeiro resultado negativo,
procura impedir outros testes e liberar apenas um
teste oficial feito por um Químico aposentado da
California?"

1- Bem, acho que o fato do químico ser aposentado nao interfere no
resultado da pesquisa.

2-Talvez o fato de ele ser aposentado contribua muito pelo descrédito
de outros cientistas, o que nao invalida o resultado de sua pesquisa.
Muitos cientistas torceram o nariz p/ novas descobertas que foram
confirmadas mais tarde.

3- Esses sites céticos costumama ser tendenciosos, e puxar a farinha
para o saco deles. Portanto devemos ser céticos e criticos para muito
deles. Vamos lá que o Rogers esteja certo mesmo, que micão seria pra
um site desses do Dr. Randi!!!

4- Ser aposentado nao é o mesmo que ser capado para a ciencia. Há
muita fertilidaade nos aposentados. Meu pai que o diga!!!

5- Químicos e biólogos costumam ser preconceituados pelos seus amigos
físicos; Os físicos tem uma ilusão secular que a física é uma ciencia
por excelência, e nao é bem assim




--- Em ciencialist@yahoogrupos.com.br, Eduardo Gueron <edgueron@y...>
escreveu
> Oi a todos,
>
> Voltei a ciencialist motivado pela morte de Cesar
> Lattes, certamente o físico mais bem sucedido de todos
> os tempos no Brasil e que, por algumas de suas
> críticas ferozes, despertou certa antipatia no meio
> medíocre dos físicos atuais, principalmente da máfia
> USPiana. A despeito de discordar de algumas das
> críticas que ele fez a Rel. Geral ou certas paixões
> que nutriu por alguns picaretas, sempre o admirei pela
> sua "falta de mediocridade". Comecei a estudar física
> em 1990 e conheci muitos físicos muito bem sucedidos
> (no conceito predominante que se tem de carreira
> científica) que transbordavam ignorância. Fui à casa
> de Lattes em Campinas duas vezes. Na primeira, entre
> outras coisas, me perguntou se havia lido Voltaire -
> por sorte lera Cândido - e, depois de falar de alguns
> cientistas brilhantes, afirmou que nada se compara às
> artes... Acho que isso o resume bem. Uma pena que após
> a morte de sua grande companheira, a professora de
> história Martha em 2003, Lattes simplesmente se deixou
> morrer.
>
> Bem, agora fazendo jus ao título, afirmo sem o mínimo
> rigor que o Sudário é uma farsa. Argumentos
> científicos são vários, vejam por exemplo o sítio:
> http://brazil.skepdic.com/sudario.html
> Mas podemos usar outras vias lógicas de raciocínio
> para chegar a essa conclusão. Se a Igreja acreditasse
> piamente que o Sudário é fruto de um milagre, não
> teria ela interesse em que a comunidade científica
> fizesse testes a fim de comprovar sua autenticidade?
> Por que então depois do primeiro resultado negativo,
> procura impedir outros testes e liberar apenas um
> teste oficial feito por um Químico aposentado da
> California?
>
> Acho, sinceramente, que muito tempo se perde com essa
> instituição nefasta que é a Igreja católica (minha
> crítica é contra a instituição não contra os
> católicos) . Uma igreja que apresenta estudos dando
> conta de que a camisinha não é eficiente no combate a
> AIDS e cujo Papa condena no mesmo texto o terrorismo e
> o casamento homossexual. E que se calou perante a
> regimes autoritários e violentos na América Latina?
> (Com honrosas exceções como o saudoso D. Helder
> Câmara)
>
> Sou cético e creio (crer?) que o ceticismo é
> necessário para o cientista. Tentar desqualificá-lo
> por causa de bobagens às vezes faladas em fóruns
> céticos é, no mínimo, covardia. Lembra a atitude da
> rede Globo que se aproveitou de um idiota sádico para
> desqualificar os movimentos de defesa dos animais que
> protestam contra rodeios, tema da novela das 8
> atual...(nada a ver a comparação, mas queria falar
> disso em algum lugar)
>
> É isso,
>
> Um abraço, Eduardo.
>
>
> __________________________________________________
> Do You Yahoo!?
> Tired of spam? Yahoo! Mail has the best spam protection around
> http://mail.yahoo.com





##### ##### #####

Para saber mais visite
http://www.ciencialist.hpg.ig.com.br


##### ##### ##### #####


Yahoo! Grupos, um serviço oferecido por:
PUBLICIDADE




------------------------------------------------------------------------------
Links do Yahoo! Grupos

a.. Para visitar o site do seu grupo na web, acesse:
http://br.groups.yahoo.com/group/ciencialist/

b.. Para sair deste grupo, envie um e-mail para:
ciencialist-unsubscribe@yahoogrupos.com.br

c.. O uso que você faz do Yahoo! Grupos está sujeito aos Termos do Serviço do Yahoo!.



[As partes desta mensagem que não continham texto foram removidas]



SUBJECT: Re: Sudario
FROM: "pubmed2005" <pubmed2005@yahoo.com.br>
TO: ciencialist@yahoogrupos.com.br
DATE: 19/03/2005 02:59


Vai ser o Randi que vai dizer que o Rogers está errado? O Randi
entende muito de mágica, mas de ciencia desconfio....

Se bem que o Randi nao está metido nesta parada. Homoeopatia é muito
diferente de química.


--- Em ciencialist@yahoogrupos.com.br, "Oraculo" <oraculo@a...>
escreveu
> Olá Pubmed
>
> Pubmed: Vamos lá que o Rogers esteja certo mesmo, que micão seria
pra
> um site desses do Dr. Randi!!!"
>
> Visão estereotipada da ciência e de cientistas. Descobrir que um
conhecimento cientifico está incorreto ou que precisa ser ajustado,
não é um "mico" para a ciência e para cientistas, pelo contrário. Em
geral, quanto maior a descoberta, quanto maior o erro descoberto,
maior o prestígio e aplauso que se recebe. Diferente de crenças e
religiões, mostrar um erro em ciência dá direito a ser premiado, e
não repreendido. Até um premio Nobel é possível ganhar, se sua
descoberta for realmente revolucionária.
>
> Descobrir que novos métodos de datação são mais precisos e
confiáveis pode garantir um Nobel para o Rogers e aplausos do Randi,
não um "mico". É uma critica comum e totalmente erronea a que afirma
isso..:-)
>
> Homero
>
>
> ----- Original Message -----
> From: pubmed2005
> To: ciencialist@yahoogrupos.com.br
> Sent: Saturday, March 19, 2005 2:38 AM
> Subject: [ciencialist] Re: Sudario
>
>
>
> Olá Eduardo eu só gostaria de fazer um breve comentário sobre su
> colocação: "Por que então depois do primeiro resultado negativo,
> procura impedir outros testes e liberar apenas um
> teste oficial feito por um Químico aposentado da
> California?"
>
> 1- Bem, acho que o fato do químico ser aposentado nao interfere
no
> resultado da pesquisa.
>
> 2-Talvez o fato de ele ser aposentado contribua muito pelo
descrédito
> de outros cientistas, o que nao invalida o resultado de sua
pesquisa.
> Muitos cientistas torceram o nariz p/ novas descobertas que
foram
> confirmadas mais tarde.
>
> 3- Esses sites céticos costumama ser tendenciosos, e puxar a
farinha
> para o saco deles. Portanto devemos ser céticos e criticos para
muito
> deles. Vamos lá que o Rogers esteja certo mesmo, que micão seria
pra
> um site desses do Dr. Randi!!!
>
> 4- Ser aposentado nao é o mesmo que ser capado para a ciencia. Há
> muita fertilidaade nos aposentados. Meu pai que o diga!!!
>
> 5- Químicos e biólogos costumam ser preconceituados pelos seus
amigos
> físicos; Os físicos tem uma ilusão secular que a física é uma
ciencia
> por excelência, e nao é bem assim
>
>
>
>
> --- Em ciencialist@yahoogrupos.com.br, Eduardo Gueron
<edgueron@y...>
> escreveu
> > Oi a todos,
> >
> > Voltei a ciencialist motivado pela morte de Cesar
> > Lattes, certamente o físico mais bem sucedido de todos
> > os tempos no Brasil e que, por algumas de suas
> > críticas ferozes, despertou certa antipatia no meio
> > medíocre dos físicos atuais, principalmente da máfia
> > USPiana. A despeito de discordar de algumas das
> > críticas que ele fez a Rel. Geral ou certas paixões
> > que nutriu por alguns picaretas, sempre o admirei pela
> > sua "falta de mediocridade". Comecei a estudar física
> > em 1990 e conheci muitos físicos muito bem sucedidos
> > (no conceito predominante que se tem de carreira
> > científica) que transbordavam ignorância. Fui à casa
> > de Lattes em Campinas duas vezes. Na primeira, entre
> > outras coisas, me perguntou se havia lido Voltaire -
> > por sorte lera Cândido - e, depois de falar de alguns
> > cientistas brilhantes, afirmou que nada se compara às
> > artes... Acho que isso o resume bem. Uma pena que após
> > a morte de sua grande companheira, a professora de
> > história Martha em 2003, Lattes simplesmente se deixou
> > morrer.
> >
> > Bem, agora fazendo jus ao título, afirmo sem o mínimo
> > rigor que o Sudário é uma farsa. Argumentos
> > científicos são vários, vejam por exemplo o sítio:
> > http://brazil.skepdic.com/sudario.html
> > Mas podemos usar outras vias lógicas de raciocínio
> > para chegar a essa conclusão. Se a Igreja acreditasse
> > piamente que o Sudário é fruto de um milagre, não
> > teria ela interesse em que a comunidade científica
> > fizesse testes a fim de comprovar sua autenticidade?
> > Por que então depois do primeiro resultado negativo,
> > procura impedir outros testes e liberar apenas um
> > teste oficial feito por um Químico aposentado da
> > California?
> >
> > Acho, sinceramente, que muito tempo se perde com essa
> > instituição nefasta que é a Igreja católica (minha
> > crítica é contra a instituição não contra os
> > católicos) . Uma igreja que apresenta estudos dando
> > conta de que a camisinha não é eficiente no combate a
> > AIDS e cujo Papa condena no mesmo texto o terrorismo e
> > o casamento homossexual. E que se calou perante a
> > regimes autoritários e violentos na América Latina?
> > (Com honrosas exceções como o saudoso D. Helder
> > Câmara)
> >
> > Sou cético e creio (crer?) que o ceticismo é
> > necessário para o cientista. Tentar desqualificá-lo
> > por causa de bobagens às vezes faladas em fóruns
> > céticos é, no mínimo, covardia. Lembra a atitude da
> > rede Globo que se aproveitou de um idiota sádico para
> > desqualificar os movimentos de defesa dos animais que
> > protestam contra rodeios, tema da novela das 8
> > atual...(nada a ver a comparação, mas queria falar
> > disso em algum lugar)
> >
> > É isso,
> >
> > Um abraço, Eduardo.
> >
> >
> > __________________________________________________
> > Do You Yahoo!?
> > Tired of spam? Yahoo! Mail has the best spam protection around
> > http://mail.yahoo.com
>
>
>
>
>
> ##### ##### #####
>
> Para saber mais visite
> http://www.ciencialist.hpg.ig.com.br
>
>
> ##### ##### ##### #####
>
>
> Yahoo! Grupos, um serviço oferecido por:
> PUBLICIDADE
>
>
>
>
> --------------------------------------------------------------------
----------
> Links do Yahoo! Grupos
>
> a.. Para visitar o site do seu grupo na web, acesse:
> http://br.groups.yahoo.com/group/ciencialist/
>
> b.. Para sair deste grupo, envie um e-mail para:
> ciencialist-unsubscribe@yahoogrupos.com.br
>
> c.. O uso que você faz do Yahoo! Grupos está sujeito aos Termos
do Serviço do Yahoo!.
>
>
>
> [As partes desta mensagem que não continham texto foram removidas]





SUBJECT: Re: [ciencialist] Re: ninguem é inocente.
FROM: "Oraculo" <oraculo@atibaia.com.br>
TO: <ciencialist@yahoogrupos.com.br>
DATE: 19/03/2005 03:00

Olá Pubmed

"Pubmed: O método radioisotópíco nao é mais confiável que o químico"

Prove. Aceito estudos de comparação, experimentos controlados com a datação quimica de artefatos de data conhecida, papers de quimicos e fisicos sobre o assunto, bibliografia sobre carbono-14 e datação quimica, e qualquer estudo publicado em revista cientifica..:-) Até mesmo a palavra de especialistas nos dois métodos, publicados em livros ou artigos relacionados...:-)

Já que fez uma afirmação prerremptória (que palavrinha maneira..:-) fica a seu ônus provar o que disse..:-)

Mas, se não puder, deve aceitar a alegação que o carbono-14, que tem tudo isso bem documentado, é mais confiável que a datação quimica apresentada no novo estudo..:-)

Homero

----- Original Message -----
From: pubmed2005
To: ciencialist@yahoogrupos.com.br
Sent: Saturday, March 19, 2005 2:53 AM
Subject: [ciencialist] Re: ninguem é inocente.



Se eu nao me engano o Siqueira é espirita e espíritualista, eu nao
tenho religião. Pode ser até que combater céticos equivocados seja
minha religiao atual, o que faça eu parecer um pouco com o Julio. Mas
ele combate o ceticismo defendendo a religiao dele, e eu faço
defendendo o equivoco filosófico de alguns céticos de plantão, e
alguns tons pretenciosos de suas faças e escritos.
Mas há de convir que é até uma religião nobre combater o lado negro
do ceticismo")- o ceticismo que é outra forma de religião
E ainda tem o lado das falácias. O Julio costuma escorregar pacas,
mas essa é outra questão, o cara é legal e nao é lícito julga-lo
nesta lista

Os problema é que os falsos céticos sempre recebem mal essas
observações, e não recebem as criticas com o mesmo humor com que as
fazem. Creio que os verdadeiros céticos pouco se importam com minhas
críticas....


Mas isso nao interessa, vamos aos comentários



--- Em ciencialist@yahoogrupos.com.br, "rmtakata" <rmtakata@a...>
escreveu
>
> --- Em ciencialist@yahoogrupos.com.br, "pubmed2005"
> > o Bradock e o Júlio há um diferença ideológica bem
> > siginificativa.
>
> Tem?
>
> > Nao é quesçao de ser mais confiável.
>
> Eh questao de ser mais confiavel. Se duas fontes se contradizem pelo
> menos uma delas estah errada.

O método radioisotópíco nao é mais confiável que o químico. Pode até
ser que eles tenham natureza investigativas distintas, mas um método
nao é mais confiável que outros, apenas tem limites em alguns
setores. O químico pode nao ser datativo, mas pode fornecer
informções importantes sobre outras questões e até mesmo fazer com
que elas colidam nas informações essenciais






##### ##### #####

Para saber mais visite
http://www.ciencialist.hpg.ig.com.br


##### ##### ##### #####


Yahoo! Grupos, um serviço oferecido por:







------------------------------------------------------------------------------
Links do Yahoo! Grupos

a.. Para visitar o site do seu grupo na web, acesse:
http://br.groups.yahoo.com/group/ciencialist/

b.. Para sair deste grupo, envie um e-mail para:
ciencialist-unsubscribe@yahoogrupos.com.br

c.. O uso que você faz do Yahoo! Grupos está sujeito aos Termos do Serviço do Yahoo!.



[As partes desta mensagem que não continham texto foram removidas]



SUBJECT: Re: ninguem é inocente.
FROM: "pubmed2005" <pubmed2005@yahoo.com.br>
TO: ciencialist@yahoogrupos.com.br
DATE: 19/03/2005 03:07


Naõ começa tropeçar nos seus proprios argumentos, o Éter luminífero é
um mito que foi derrubado...uma teoria absurda que foi demonstrada
falsa, portanto nao passa de um mito

Se descobrirem que o Bing bang nao é teoria mais proxima da origem do
universo, isso vai soar tão absurdo como a crença no boi tatá. Se o
boi tatá nao existe, se foi comprovado a sua inexistencia, isso vai
refutar que o boi tatá só existe na imaginação criativa das pessoas.
Assim como o Éter luminífero preencheu as mentes científicas dos
séculos anteriores. Tudo nao se passou de uma ilusão

Quanto ao Sudário, já está mais que esclarecida minha posição, nao
tenho que ficar repetindo os mesmos arguimentos de forma circular
como voce vem fazendo



--- Em ciencialist@yahoogrupos.com.br, "Oraculo" <oraculo@a...>
escreveu
> Olá Pubmed

>
> risos..:-) É enorme..:-) No primeiro caso, a conclusão, e não
crença, foi abandonada devido a evidencias posteriores que refutaram
a teoria do éter. No segundo, nada vai mudar e quem crê nisso vai
continuar crendo, sem que evidencias ou provas ou aspectos materiais
ou racionais tenham o poder de interferir oiu mudar essa crença. A
diferença, como pode notar é enorme..:-) A primeira, uma teoria
cientifica, esperava por dados e evidencias que a confirmassem, e se
conformou com a refutação por falta de provas. A segunda, criada da
imaginação humana, vai se manter indefinidamente, exatamente como foi
criada, dependendo apenas da crença subjetiva de seres humanos.
Enorme diferença..:-)
>






SUBJECT: Re: ninguem é inocente.
FROM: "pubmed2005" <pubmed2005@yahoo.com.br>
TO: ciencialist@yahoogrupos.com.br
DATE: 19/03/2005 03:13


Antes de cortar pedacinhos de minha mensagem, leia todo o conteúdo,
que voce vai entender. Voce nao leu e quer fazer comentários
absurdos. Vai lá e le de novo, ou então coloque o texto na integra e
comente, senão voce destroi o sentido do que está dizendo que a
analise química no é so utilizada paraa datação copmo para muitos
outros resultados como constituição do tecido, das substancia nele
contidas até que ponto uma substancia química pode durar secularmente
A que nível ela pode ser destruída e conservada durante opassar do
tempo

Voce está vendo a coisa unilateralmente validando excessivamente o
primeiro estudo. Isso a meu ver é um estratégia cientifista. Não
quero invalidar o primeiro estudo, mas antes poder complementa-lo com
o segundo, comparar as análises e chegar uma conclusão ou apenas
parte dela. Ver os pontos colisivos.

Voce só está interessando em afirmar ou provar que o manto nao foi o
de cristo, eu quero ir bem mais além


--- Em ciencialist@yahoogrupos.com.br, "Oraculo" <oraculo@a...>
escreveu
> Olá Pubmed
>
> "Pubmed: O método radioisotópíco nao é mais confiável que o químico"
>
> Prove. Aceito estudos de comparação, experimentos controlados com a
datação quimica de artefatos de data conhecida, papers de quimicos e
fisicos sobre o assunto, bibliografia sobre carbono-14 e datação
quimica, e qualquer estudo publicado em revista cientifica..:-) Até
mesmo a palavra de especialistas nos dois métodos, publicados em
livros ou artigos relacionados...:-)
>
> Já que fez uma afirmação prerremptória (que palavrinha maneira..:-)
fica a seu ônus provar o que disse..:-)
>
> Mas, se não puder, deve aceitar a alegação que o carbono-14, que
tem tudo isso bem documentado, é mais confiável que a datação quimica
apresentada no novo estudo..:-)
>
> Homero
>
> ----- Original Message -----
> From: pubmed2005
> To: ciencialist@yahoogrupos.com.br
> Sent: Saturday, March 19, 2005 2:53 AM
> Subject: [ciencialist] Re: ninguem é inocente.
>
>
>
> Se eu nao me engano o Siqueira é espirita e espíritualista, eu
nao
> tenho religião. Pode ser até que combater céticos equivocados
seja
> minha religiao atual, o que faça eu parecer um pouco com o Julio.
Mas
> ele combate o ceticismo defendendo a religiao dele, e eu faço
> defendendo o equivoco filosófico de alguns céticos de plantão, e
> alguns tons pretenciosos de suas faças e escritos.
> Mas há de convir que é até uma religião nobre combater o lado
negro
> do ceticismo")- o ceticismo que é outra forma de religião
> E ainda tem o lado das falácias. O Julio costuma escorregar
pacas,
> mas essa é outra questão, o cara é legal e nao é lícito julga-lo
> nesta lista
>
> Os problema é que os falsos céticos sempre recebem mal essas
> observações, e não recebem as criticas com o mesmo humor com que
as
> fazem. Creio que os verdadeiros céticos pouco se importam com
minhas
> críticas....
>
>
> Mas isso nao interessa, vamos aos comentários
>
>
>
> --- Em ciencialist@yahoogrupos.com.br, "rmtakata" <rmtakata@a...>
> escreveu
> >
> > --- Em ciencialist@yahoogrupos.com.br, "pubmed2005"
> > > o Bradock e o Júlio há um diferença ideológica bem
> > > siginificativa.
> >
> > Tem?
> >
> > > Nao é quesçao de ser mais confiável.
> >
> > Eh questao de ser mais confiavel. Se duas fontes se contradizem
pelo
> > menos uma delas estah errada.
>
> O método radioisotópíco nao é mais confiável que o químico. Pode
até
> ser que eles tenham natureza investigativas distintas, mas um
método
> nao é mais confiável que outros, apenas tem limites em alguns
> setores. O químico pode nao ser datativo, mas pode fornecer
> informções importantes sobre outras questões e até mesmo fazer
com
> que elas colidam nas informações essenciais
>
>
>
>
>
>
> ##### ##### #####
>
> Para saber mais visite
> http://www.ciencialist.hpg.ig.com.br
>
>
> ##### ##### ##### #####
>
>
> Yahoo! Grupos, um serviço oferecido por:
>
>
>
>
>
>
>
> --------------------------------------------------------------------
----------
> Links do Yahoo! Grupos
>
> a.. Para visitar o site do seu grupo na web, acesse:
> http://br.groups.yahoo.com/group/ciencialist/
>
> b.. Para sair deste grupo, envie um e-mail para:
> ciencialist-unsubscribe@yahoogrupos.com.br
>
> c.. O uso que você faz do Yahoo! Grupos está sujeito aos Termos
do Serviço do Yahoo!.
>
>
>
> [As partes desta mensagem que não continham texto foram removidas]





SUBJECT: Re: Sudario
FROM: "pubmed2005" <pubmed2005@yahoo.com.br>
TO: ciencialist@yahoogrupos.com.br
DATE: 19/03/2005 04:23


Segue abaixo

--- Em ciencialist@yahoogrupos.com.br, "Oraculo" <oraculo@a...>
escreveu
> Visão estereotipada da ciência e de cientistas. Descobrir que um
conhecimento cientifico está incorreto ou que precisa ser ajustado,
não é um "mico" para a ciência e para cientistas, pelo contrário.


Então me diz no que o conhecimento cientifico do químico está errado
e no que ele está sendo incorreto. Não acho que o trabalho de Rogers
seja tão ruim. Ele levtou questoes muito interessantes que merecem
uma verificação. Se voce tem coisa melhor em mente apresente a
comunidade científica


ORACULO: Em geral, quanto maior a descoberta, quanto maior o erro
descoberto, maior o prestígio e aplauso que se recebe. Diferente de
crenças e religiões, mostrar um erro em ciência dá direito a ser
premiado, e não repreendido. Até um premio Nobel é possível ganhar,
se sua descoberta for realmente revolucionária.

Um religioso pode ganhar o premio nobel da paz , ate aí voce nao
disse nada. Pode ficar famoso, e ter méritos reconhecidos. E um
religioso cientista pode ter seu trabalho publicado e vangloriado nos
meios cientificos

> Descobrir que novos métodos de datação são mais precisos e
confiáveis pode garantir um Nobel para o Rogers e aplausos do Randi,
não um "mico". É uma critica comum e totalmente erronea a que afirma
isso..:-)

Mas Randi nao vai se meter com um químico, ele só saberia lidar bem
com homeopatas, espiritas e mágicos.

Como ele poderia criticar e lidar c/ um trabalho feito por um
cientista do porte como o físico Einstein? Ele só ia meter a colher
dele aonde nao deveria meter. Ser um grande palpiteiro sem maior
conhecimento. O mundo está cheio de palpiteiros
Veja, criticar o cientifismo nao é erroneo, é sim um grande serviço
prestadoa comuninidade





SUBJECT: Novo no grupo
FROM: "PSavio" <psdmo@uol.com.br>
TO: <ciencialist@yahoogrupos.com.br>
DATE: 19/03/2005 06:31

Apresentando-me:

Mestrando de Física

Tive acesso, através de um amigo, a algumas mensagens envolvendo matemática avançada, física, história da física, questionamentos, novas propostas outros assuntos e gostei. Achei o grupo de alto nível. Espero poder aprender aqquí enriquecer minha tese.

P.Sávio



[As partes desta mensagem que não continham texto foram removidas]



SUBJECT: Re: [ciencialist] Re: ninguem é inocente.
FROM: "Sergio M. M. Taborda" <sergiotaborda@terra.com.br>
TO: ciencialist@yahoogrupos.com.br
DATE: 19/03/2005 11:17

Oraculo escreveu:

Mas escreveu , como de costume , cheio de incoerencias cientificas. Na
sua busca pelo conhecimento imaculado livre de crença, como de costume,
é forçado a acreditar em algo para suportar os seus argumentos
anti-cientificos. O que, é normal nos céticos. (e eu uso o termo no
sentido prejorativo sim. Faz muito tempo que esse termo significou
alguma coisa para a ciencia. Quem duvida sistemáticamente,não está
fazendo nenhum bem à ciencia, pq não aporta mais nada. Um cientista de
verdade duvida sistemática, mas levanta hipoteses para os problemas, faz
testes, eleabora estruturas de pensamento que possam explicar o
ocurrido. Em primeiro lugar, testa de existe realmente um fenomeno a
esudar. Os ceticos não fazem isso, simplesmente duvidam e são contra qq
hipote de demonstracção da verdade cientifica.

> Olá Pubmed
>
> "Pubmed: No fundo , no fundo os céticos tem um pouquinho de cagaço que
> o manto seja mesmo de Jesus né?"
>
> Não, não temos..:-) Eu pelo menos não tenho nenhum..:-) Veja, se for
> de jesus, isso significa que existiu um cara chamado jesus, apenas
> isso.Não significa muito mais que isso, e isso apenas é algo a ser
> considerado e estudado, não significa que todos nós devemos nos tornar
> católicos ou cristãos..:-)

Vc está enganado, como de costume. É improvável que se possa provar que
o manto pertenceu , ou melhor, esteve em contacto com, Jesus. O mais
que se pode provar é que é possivel aceitar q o manto tenha estado no
local onde jesus foi morto, no tempo em que ele foi morto.
Por outro lado, se isso for provado, levantam-se várias questões. A
principal é: pq guardar o manto que envolveu alguem, se esse alguem não
foi ninguem inportante. Isso sim não faria sentido. As pessoas guardam
reliquias pelos mais diversos motivos, mas esses motivos existem.
Pessoas guardam pedras do muro de brelim. Tento provar que essas pedras
pertenceram a esse muro daqui a 2000 anos.
Ninguem acreditará em vc se não souberem da existencia historica de dito
muro. Ora, sabemos que os judeus tinham o cosutme de envolver os mortos
em panos, tal como os egipcios. Alias foi no egipto que aprenderam isso.
Mas tal costume desapreceu com o tempo. Porteriormente a 1300 DC não
existia mais esse costume. Razão pela qual não é credivel que o manto
tenha apenas essa idade.
Por outro lado, o manto viajou muito. Foram encontrados vestigios de
plantas que são especificas de certos locais geográficos, entre eles, o
local onde jusus terá sido supultado. Isso coloca o manto na região
geografica em causa, mas isso não significa que tenha estado lá quando
Jesus morreu.
O facto dele ter viajado, dele ter estado guardado durante a ideade
média - epoca em que as casas eram de madeira mais do que de pedra -num
local que posteriormente ardeu ( foi possivelmente contaminado com
cinzas ) , facto que o mutilou e que fez com que seus guardiões o
tentassem remendar devido ao efeito moral que quasaria vê-lo danificado
, explica perfeitamente pq os testes do carbono 14 foram inconclusivos.
Esta é uma palavra que os ceticos desocnhecem. Os testes são feitos,
tudo bem. Mas os resultados não são sagrados , como os ceticos
agreditam. Tudo bem que os testes podem até afirmar que o manto tem 1300
anos, mas o que isso significa para o estudo historico do manto ?
Comprova apenas que a historia do fogo que aconteceu no local onde
estava guardado pode ser veridica.

Facto muito mais interessante e que ninguem conseguiu explicar até agora
é a formação da imagem no manto. A imagem não é formada por sangue ou
outro produto , mas pelo que parece ser um especie de quimadura por
radiação. Como explicar a existencia dessa radiação , ainda para mais
quando ela prece ser emitida de dentro para fora ? Só o facto da
radiaçao existir já é um enigma. Como explicar a existencia daquela
radiação num tumulo de pedra solida ? Naquele tempo não existia nada que
podesse emitir radiação daquela, os raios-X são uma invensão modera e o
efeito tridimencional da imagem é um fenomeno da holografia apenas usado
a alguns anos. Explicar esse ponto é muito mais interessante do que
saber de quando é o manto. Seja de quando for ele é do passado, tempo no
qual não existia forma de produzir aquela radiação com aquela "assinatura".
Esse é o ponto onde os crentes podem entender que o manto pertenceu a
Jesus, já que suportamente ele se evadiu do tumulo por intermédio de um
anjo (ser de luz) transformando-se ele mesmo num ser do mesmo tipo, até
que apareceu depois aos apostulos. Isso não significa que ele seja filho
de Deus , mas dependendo de como se intrepreta "Deus" isso pode pelo
menos desacreditar o facto da resurreição ter sido um milagre. Notar,
que provar que aconteceu um facto tecnologico à 2000 anos, é provar que
algo importante aconteceu, independentemente de quem o fez, ou porquê.
Tudo isto para dizer que a sua frase "Veja, se for de jesus, isso
significa que existiu um cara chamado jesus, apenas isso." não está nem
perto da verdade.

>
> E eu tentei explicar, e talvez não tenha sido claro, porque céticos
> rejeitam estudos. É porque rejeitar estudos é parte fundamental do
> método cientifico..;-)

Apenas outros estudos podem refutar estudos preliminares e não a razão,
ou a imaginação das pessoas. É isso que os ceticos fazem constantemente,
e que sinceramente, já encheu o saco.

Sérgio Taborda


SUBJECT: Re: [ciencialist] Sudario
FROM: "Sergio M. M. Taborda" <sergiotaborda@terra.com.br>
TO: ciencialist@yahoogrupos.com.br
DATE: 19/03/2005 11:35

Eduardo Gueron wrote:

>
>Bem, agora fazendo jus ao título, afirmo sem o mínimo
>rigor que o Sudário é uma farsa. Argumentos
>científicos são vários, vejam por exemplo o sítio:
>http://brazil.skepdic.com/sudario.html
>Mas podemos usar outras vias lógicas de raciocínio
>para chegar a essa conclusão. Se a Igreja acreditasse
>piamente que o Sudário é fruto de um milagre, não
>teria ela interesse em que a comunidade científica
>fizesse testes a fim de comprovar sua autenticidade?
>
>

Não necessáriamente. Este é o tipo de racionio simplista e longe da
verdade que os cepticos usam para se convenser, e com esperam convencer
os outros.
Se o sudário fosse realmente o manto que cobrio Jesus. Se se puder
provar que era realemtne de jusus e de ninguem mais, que ao mesmo tempo
a informação que o manto contém poder ser explicada pela ciencia actual,
mas não no tempo de Jesus isso cria um problema. Isso prova que Jesus
era realmente especial e que a Igreja tem toda a razão em o adorar como
a um Deus. Isso invevitávelmente levaria ao fortalecimento das crenças
em Jesus como um ser especial. E isso não é necessáriamente bom para a
Igreja. Lembresse que existem dezenas de igrejas protestantes. Quanto
mais a Igreja for desafiada pior.
Por isso não 100% certeza que a igreja quisesses que o manto fosse
investigado, mesmo que eles tivessem a certeza absoluta - que não têm -
de que é verdadeiro.

>Por que então depois do primeiro resultado negativo,
>procura impedir outros testes e liberar apenas um
>teste oficial feito por um Químico aposentado da
>California?
>
>
>
Hoje em dia isso não existe mais. Muitas pessoas estudam o sudário.
Apenas a recolha de novas informações é dificil, mas um cientista não
tem que estar cosntamente tirando dados.

> Sou cético e creio (crer?) que o ceticismo é
>necessário para o cientista.
>
Quando se acredita demasiado na duvida, nada mais faz sentido.
Uma coisa que nunca entendi é pq o cetico não duvida do que diz. Pq ele
nunca acha que está enganado ?

Sérgio Taborda


SUBJECT: Re: [ciencialist] Re: Sudario
FROM: "Sergio M. M. Taborda" <sergiotaborda@terra.com.br>
TO: ciencialist@yahoogrupos.com.br
DATE: 19/03/2005 11:38

Oraculo wrote:

> Olá Pubmed
>
> Pubmed: Vamos lá que o Rogers esteja certo mesmo, que micão seria pra
> um site desses do Dr. Randi!!!"
>
> Visão estereotipada da ciência e de cientistas. Descobrir que um
> conhecimento cientifico está incorreto ou que precisa ser ajustado,
> não é um "mico" para a ciência e para cientistas, pelo contrário.

Pois não é. Mas como o Randi não é nada disso, é um mico para ele sim.
E para todos os que acreditam como ele.

Sérgio Taborda


SUBJECT: Santo sudário e a vinilina
FROM: "pubmed2005" <pubmed2005@yahoo.com.br>
TO: ciencialist@yahoogrupos.com.br
DATE: 19/03/2005 12:55


Oráculo e Takata voces simplesmente nao podem dizer que outros testes
químicos nao foram realizados no tecido.

Propriedades químicas

Foram também efectuados testes químicos ao sudário por especialistas
das Universidades de Milão e da Califórnia. Os métodos utilizados
foram a análise espectral e fotografia ultra-violeta. Os resultados
mostram que a porção amostrada para datação radiométrica é distinta
do resto do tecido, nomeadamente pela presença de pigmentos e
fixantes. Este resultado sugere que esta porção do tecido seja na
realidade um remendo posterior. Outra diferença consiste na presença
de vanilina, um produto da decomposição térmica da linhina (um
composto das fibras naturais). A vanilina é um composto habitual na
análise a tecidos da Idade Média, mas que se encontra ausente em
amostras mais antigas, uma vez que sofre decaímento.

Como as amostras do Sudário anlisada pelos Dr. Rogers nao traziam
restos de vinilina no restante do manto, ele supos que o manto
deveria ter entre 1300 e 3000 anos.
Lembrando que a vinilina suporta datas até aproximadamente a Idade
Média como explicitado no texto acima


Tirando alguns comentários meus o resto do texto pode ser encontrado
na Wikipedia

http://pt.wikipedia.org/wiki/Santo_Sudario










SUBJECT: Santo sudário e a vanilina(correção)
FROM: "pubmed2005" <pubmed2005@yahoo.com.br>
TO: ciencialist@yahoogrupos.com.br
DATE: 19/03/2005 12:58


Oráculo e Takata voces simplesmente nao podem dizer que outros testes
químicos nao foram realizados no tecido.

Propriedades químicas

Foram também efectuados testes químicos ao sudário por especialistas
das Universidades de Milão e da Califórnia. Os métodos utilizados
foram a análise espectral e fotografia ultra-violeta. Os resultados
mostram que a porção amostrada para datação radiométrica é distinta
do resto do tecido, nomeadamente pela presença de pigmentos e
fixantes. Este resultado sugere que esta porção do tecido seja na
realidade um remendo posterior. Outra diferença consiste na presença
de vanilina, um produto da decomposição térmica da linhina (um
composto das fibras naturais). A vanilina é um composto habitual na
análise a tecidos da Idade Média, mas que se encontra ausente em
amostras mais antigas, uma vez que sofre decaímento.

Como as amostras do Sudário anlisada pelos Dr. Rogers nao traziam
restos de vanilina no restante do manto, ele supos que o manto
deveria ter entre 1300 e 3000 anos.
Lembrando que a vinalina suporta datas até aproximadamente a Idade
Média como explicitado no texto acima


Tirando alguns comentários meus o resto do texto pode ser encontrado
na Wikipedia

http://pt.wikipedia.org/wiki/Santo_Sudario

















SUBJECT: Re: Sudario
FROM: "pubmed2005" <pubmed2005@yahoo.com.br>
TO: ciencialist@yahoogrupos.com.br
DATE: 19/03/2005 13:24


Excelente Taborda, e quando o fazem dão a desculpa que a ciencia
propoe correções, os paradigmas mudam e aí eles livram a cara deles.
Se voce observar direitinho o cético é sempre imune. Aquele argumento
tipo: "Se for comprovado que está errado, temos de mudar de opinião,
mas se estiver certo permaneceremos nisso". Aí o cético fica numa
situação de imunidade. Quer dizer, ele nunca está certo , nem errado.
Vai depender das descobertas científicas . Então onde fica a razão
nisso?

Só que o que muda é a ciencia, os céticos vivem ao sabor disso. A
ciencia contém o ceticismo, mas o ceticismo nao abrange a ciencia(ele
é apenas parte dela) É preciso muito cuidado e pensamento acurado
para nao se cair nesta armadilha

O que é mais irritante nos céticos é o fato de que quando aparecem um
trbalho que os favorecem é muito bem recebido e acatado, mas quando
surge algo que nao corresponde as expectativas céticas a novidade
cientifica é acolhida com ceticismo. Sinceramente meu ver, isso nao
pode ser chamado de ciencia. A ciencia é algo neutro, independente
das opiniões pessoais, e das técnicas de auto-defesa intelectual


--- Em ciencialist@yahoogrupos.com.br, "Sergio M. M. Taborda"
<sergiotaborda@t...> escreveu
> Quando se acredita demasiado na duvida, nada mais faz sentido.
> Uma coisa que nunca entendi é pq o cetico não duvida do que diz. Pq
ele
> nunca acha que está enganado ?
>
> Sérgio Taborda





SUBJECT: Re: [ciencialist] Santo sudário e a vanilina(correção)
FROM: "Alberto Mesquita Filho" <albmesq@uol.com.br>
TO: <ciencialist@yahoogrupos.com.br>
DATE: 19/03/2005 13:37

----- Original Message -----
From: "pubmed2005"
Sent: Saturday, March 19, 2005 12:58 PM
Subject: [ciencialist] Santo sudário e a vanilina(correção)

> Oráculo e Takata voces simplesmente nao podem dizer que outros testes
> químicos nao foram realizados no tecido.

Poder dizer eles podem. O que eles não podem é tentar nos convencer que
estão dizendo a verdade, sob pena de serem taxados como "céticos de
verdade", aqueles que seguem um cartesianismo às avessas: Penso, logo não
existo.

PS: Essa nem eu entendi, mas eu chego lá ;-))

[ ]´s
Alberto
http://ecientificocultural.com/indice.htm
Mas indiferentemente a tudo isso, o neutrino tem massa, o elétron não é
uma carga elétrica coulombiana e a Terra se move. E a história se repetirá.



SUBJECT: Re: [ciencialist] Sudario
FROM: "Sergio M. M. Taborda" <sergiotaborda@terra.com.br>
TO: ciencialist@yahoogrupos.com.br
DATE: 19/03/2005 14:04

Eduardo Gueron wrote:

> Sou cético e creio (crer?) que o ceticismo é
>necessário para o cientista. Tentar desqualificá-lo
>por causa de bobagens às vezes faladas em fóruns
>céticos é, no mínimo, covardia.
>
Covardia é isto aqui:
" É também alegado que a imagem não é uma pintura, mas uma imagem
milagrosamente <http://brazil.skepdic.com/milagres.html> transposta.
Os céticos discordam e argumentam que o sudário é uma pintura e uma
falsificação".
in http://brazil.skepdic.com/sudario.html

Eles argumentam muito, mas não provam nada do que dizem. Se provassem,
não existiria discussão.

Sérgio Taborda



SUBJECT: Re: Santo sudário e a vanilina(correção)
FROM: "pubmed2005" <pubmed2005@yahoo.com.br>
TO: ciencialist@yahoogrupos.com.br
DATE: 19/03/2005 14:10


Alberto,

Não seria: "Não penso, mas existo" ?



--- Em ciencialist@yahoogrupos.com.br, "Alberto Mesquita Filho"
<albmesq@u...> escreveu
> ----- Original Message -----
> From: "pubmed2005"
> Sent: Saturday, March 19, 2005 12:58 PM
> Subject: [ciencialist] Santo sudário e a vanilina(correção)
>
> > Oráculo e Takata voces simplesmente nao podem dizer que outros
testes
> > químicos nao foram realizados no tecido.
>
> Poder dizer eles podem. O que eles não podem é tentar nos convencer
que
> estão dizendo a verdade, sob pena de serem taxados como "céticos de
> verdade", aqueles que seguem um cartesianismo às avessas: Penso,
logo não
> existo.
>
> PS: Essa nem eu entendi, mas eu chego lá ;-))
>
> [ ]´s
> Alberto
> http://ecientificocultural.com/indice.htm
> Mas indiferentemente a tudo isso, o neutrino tem massa, o elétron
não é
> uma carga elétrica coulombiana e a Terra se move. E a história se
repetirá.





SUBJECT: p/ os engenheiros e os que estão construindo
FROM: "pubmed2005" <pubmed2005@yahoo.com.br>
TO: ciencialist@yahoogrupos.com.br
DATE: 19/03/2005 14:28


Existe uma terrível bactéria, a T. ferooxidans que tem o nome
alternativo de T. concretivorans. Ela tem uma predileção especial por
concretos de baixa qualidade, com alto teor de enxofre,
particularmente se reforçado com hastes de ferro. Para a consternação
dos engenheiros , essa bactéria é capaz de produzir tanto ácido
sulfúrico que apodrece o concreto, fazendo pontes e viadutos
desabarem e edifíciios altos a desmoronar. Levou-se algum tempo para
reconhecer que o apodrecimento do concreto se deve a uma infecção por
bactérias, pois a densidade microbiana é muito baixa - uma bacteria
precisa de 50 vezes seu proprio peso em ferro para uma única divisão
celular.

Extraído do livro da fisiolista Frances Ashcroft. "A Vida no Limite"
página 276 - Editora Jorge Zahar


NOTA: Sempre que fizerem construções ou reformas sempre de
preferencia a materiais de alta qualidade e resistencia. Vale a pena
gastar um pouquinho mais para se ter menos dor de cabeça






SUBJECT: Re: [ciencialist] Sudario
FROM: Anderson Almeida da Silveira <anderson_a5@yahoo.com.br>
TO: ciencialist@yahoogrupos.com.br
DATE: 19/03/2005 14:54



"Sergio M. M. Taborda" <sergiotaborda@terra.com.br> wrote:Eduardo Gueron wrote:

> Sou cético e creio (crer?) que o ceticismo é
>necessário para o cientista. Tentar desqualificá-lo
>por causa de bobagens às vezes faladas em fóruns
>céticos é, no mínimo, covardia.
>
Covardia é isto aqui:
" É também alegado que a imagem não é uma pintura, mas uma imagem
milagrosamente <http://brazil.skepdic.com/milagres.html> transposta.
Os céticos discordam e argumentam que o sudário é uma pintura e uma
falsificação".
in http://brazil.skepdic.com/sudario.html

Eles argumentam muito, mas não provam nada do que dizem. Se provassem,
não existiria discussão.

Sérgio Taborda



##### ##### #####

Para saber mais visite
http://www.ciencialist.hpg.ig.com.br

PRA MIM A DISCURÇÃO SOBRE O SUDÁRIO É INÚTIL. NIGUÉM VIU O ROSTO DE JESUS E PORTANTO NÃO HÁ COMO SABER SE ELE O USOU OU NÃO.

__________________________________________________
Converse com seus amigos em tempo real com o Yahoo! Messenger
http://br.download.yahoo.com/messenger/

[As partes desta mensagem que não continham texto foram removidas]



SUBJECT: Re: Sudario
FROM: "pubmed2005" <pubmed2005@yahoo.com.br>
TO: ciencialist@yahoogrupos.com.br
DATE: 19/03/2005 15:00


Oi anderson, ainda nao vi nenhuma posição religiosa por aqui. O que
se está discutindo são posições científicas.
De um lado os céticos que se dizem adeptos da ciencia que alegam que
só o santo carbono-14 é infalível e de outro uns( eu ou alguns) que
defende a apenas a posição da ciencia que outros testes tb são
importantes e nao precisam necessariamente invalidar o outro. Tanto
que no estudo químico a idade *mínima* para o manto ainda se manteve
a detectadas pelo carbono-14, embora estipula uma idade máxima de
3000 anos. Ela não anula necessariamente o primeiro estudo, mas dá
uma dimensão maior a descoberta. E abre caminhos para novos estudos


-- Em ciencialist@yahoogrupos.com.br, Anderson Almeida da Silveira
<anderson_a5@y...> escreveu
>
> PRA MIM A DISCURÇÃO SOBRE O SUDÁRIO É INÚTIL. NIGUÉM VIU O ROSTO DE
JESUS E PORTANTO NÃO HÁ COMO SABER SE ELE O USOU OU NÃO.
>
> __________________________________________________
> Converse com seus amigos em tempo real com o Yahoo! Messenger
> http://br.download.yahoo.com/messenger/
>
> [As partes desta mensagem que não continham texto foram removidas]





SUBJECT: Re: Sudario
FROM: "pubmed2005" <pubmed2005@yahoo.com.br>
TO: ciencialist@yahoogrupos.com.br
DATE: 19/03/2005 16:08


O que é interessante nestes sites céticos como o
http://brazil.skepdic.com/milagres.html é como eles mudam o discurso
após uma descoberta científica, mesmo continuando a questiona-la

Vejamos, eles dizem: <<<"Naturalmente, o tecido poderia ter 3.000 ou
2.000 anos de idade, como especula Rogers, mas a imagem nele poderia
datar de um período muito posterior. Qualquer que seja a data
correta, tanto para o pano quando para a figura, não prova em nenhum
grau de probabilidade razoável que o tecido seja o sudário em que
Jesus foi envolvido e que a imagem seja de alguma forma milagrosa.
Acreditar nisso sempre será uma questão de fé, não de provas
científicas.">>>

ORA BOLAS, QUEM ESTÁ FALANDO DA IMAGEM DE JESUS?
Estamos aventando apenas a possibilidade do tecido ter 3000 anos...
dane-se a imagem
Não entendo como os céticos só se dirigem aos religiosos como na
mensagem acima. Nem todo mundo tem posições religiosas, muitos tem
interesse arqueológico e historico pela peça de tecido chamado "santo
sudário"




-- Em ciencialist@yahoogrupos.com.br, Anderson Almeida da Silveira
<anderson_a5@y...> escreveu
<http://brazil.skepdic.com/milagres.html> transposta.
> Os céticos discordam e argumentam que o sudário é uma pintura e uma
> falsificação".
]





SUBJECT: Re: Santo sudário e a vanilina(correção)
FROM: "rmtakata" <rmtakata@altavista.net>
TO: ciencialist@yahoogrupos.com.br
DATE: 19/03/2005 17:40


--- Em ciencialist@yahoogrupos.com.br, "Alberto Mesquita
> From: "pubmed2005"
> > Oráculo e Takata voces simplesmente nao podem dizer que
> > outros testes químicos nao foram realizados no tecido.
>
> Poder dizer eles podem. O que eles não podem é tentar nos
> convencer que estão dizendo a verdade

Sim, poderiamos dizer. Mas eu nao disse (e ao q. eu saiba nem o
Oraculo/Homero). O q. eu disse e digo novamente eh q. ainda estah para
se demonstrar q. datacao quimica eh mais confiavel do q. datacao por
radioisotopos.

[]s,

Roberto Takata





SUBJECT: Re: [ciencialist] Livre-docência
FROM: Franco <dfranco@pop.com.br>
TO: ciencialist@yahoogrupos.com.br
DATE: 19/03/2005 17:42

Talvez dependa da instituição, Alberto. Até ler a resposta do Álvaro eu
não entendia perfeitamente sobre a "importância" da livre-docência. Mas
há dois anos atrás tive aulas com uma professora, digo, com uma
livre-docente que vivia nos lembrando, em toda aula, que ela havia
passado no concurso de livre-docência; em toda a aula ela comentava ou
repetia algo já comentado a respeito de sua árdua luta para conseguir o
título. E, segundo a mesma, em uma das etapas do processo, sortearam um
tema para ser apresentado - se não me engano, o tema foi apresentado no
mesmo dia do sorteio.
Ah, sim, e a aula dela era terrível, considerando-se a sua falta de
didática... Durante muito tempo fiquei intrigado (e ainda continuo)
sobre a necessidade/utilidade de se fazer um curso de livre-docência.
Acredito que seja apenas por meras formalidades acadêmicas.

Franco.


Alberto Mesquita Filho wrote:

> ----- Original Message -----
> From: "Alvaro Augusto (E)"
> Sent: Friday, March 18, 2005 12:27 PM
> Subject: Re: [ciencialist] Livre-docência
>
> > Os livres-docentes são os Grandes Mestres Jedy do mundo acadêmico. Eles
> > são capazes de controlar os elementos, andar sobre as águas, multiplicar
> > os pães, rescussitar os mortos, etc. Após fazer mestrado, doutorado,
> > pós-doutorado, ter vários anos de profissão e publicar vários trabalhos
> > releventes, o candidato pode fazer concurso para livre-docência, quando
> > houver vagas. O concurso envolve, além de prova escrita, a preparação de
> > um resumo biográfico (memorial) e a defesa de uma tese em frente a banca
> > examinadora. Algumas faculdades também submetem o memorial, que deve
> > comprovar a obra do pesquisador, à argüição pública. É mole?
>
> No meu tempo [Putz, acho que estou ficando velho! :-)] o candidato a
> livre-docente devia também demonstrar a sua didática através de uma
> aula de
> uma hora de duração e sobre um tema sorteado com 24 horas de
> antecedência.
> Mas isso foi em meados do século passado, não sei se ainda está em voga.
>
> [ ]´s





SUBJECT: Livre-docência
FROM: "L.E.R.de Carvalho" <lecarvalho@infolink.com.br>
TO: ciencialist@yahoogrupos.com.br
DATE: 19/03/2005 18:42

At 17:42 19/3/2005, you wrote:
>Talvez dependa da instituição, Alberto. Até ler a resposta do Álvaro eu
>não entendia perfeitamente sobre a "importância" da livre-docência. Mas
>há dois anos atrás tive aulas com uma professora, digo, com uma
>livre-docente que vivia nos lembrando, em toda aula, que ela havia
>passado no concurso de livre-docência; em toda a aula ela comentava ou
>repetia algo já comentado a respeito de sua árdua luta para conseguir o
>título. E, segundo a mesma, em uma das etapas do processo, sortearam um
>tema para ser apresentado - se não me engano, o tema foi apresentado no
>mesmo dia do sorteio.
>Ah, sim, e a aula dela era terrível, considerando-se a sua falta de
>didática... Durante muito tempo fiquei intrigado (e ainda continuo)
>sobre a necessidade/utilidade de se fazer um curso de livre-docência.
>Acredito que seja apenas por meras formalidades acadêmicas.
>
>Franco.



Pois é.
Quem não tem o que mostrar...
mostra título.

L.E.

[As partes desta mensagem que não continham texto foram removidas]



SUBJECT: Re: Santo sudário e a vanilina(correção)
FROM: "pubmed2005" <pubmed2005@yahoo.com.br>
TO: ciencialist@yahoogrupos.com.br
DATE: 19/03/2005 18:43


Takata, voce realmente pode nao ter dito, mas o Oráculo disse mais de
uma vez que o teste químico deveria ser repetido pra pudesse ser
aceita a nova pesquisa...e no entanto este teste foi realizado mais
de uma vez
O problema é que voce persiste neste "mais confiáavel", e nem é
disto que está se tratando. Mas que novas descobertas de peso foram
surgiram que podem mudar o padrão estipulado anteriormente. Se não há
traços de vanilina nos tecidos nao analisados pelo radioisótopo, isso
lança uma outra possibilidade, já que vanilina só permanece em
tecidos até a idade média aproximadamente , desaparecendo depois
deste tempo.
Nisso os cientistas podem concluir evidentemente que o tecido
era muito mais antigo. Mas, se voces não se querem aparentemente
aceitar as novas pesquisas tudo bem...essa é uma outra história.

--- Em ciencialist@yahoogrupos.com.br, "rmtakata" <rmtakata@a...>
escreveu
>
> --- Em ciencialist@yahoogrupos.com.br, "Alberto Mesquita
> > From: "pubmed2005"
> > > Oráculo e Takata voces simplesmente nao podem dizer que
> > > outros testes químicos nao foram realizados no tecido.
> >
> > Poder dizer eles podem. O que eles não podem é tentar nos
> > convencer que estão dizendo a verdade
>
> Sim, poderiamos dizer. Mas eu nao disse (e ao q. eu saiba nem o
> Oraculo/Homero). O q. eu disse e digo novamente eh q. ainda estah
para
> se demonstrar q. datacao quimica eh mais confiavel do q. datacao por
> radioisotopos.






SUBJECT: Re: [ciencialist] Livre-docência
FROM: "Alvaro Augusto \(E\)" <alvaro@electraenergy.com.br>
TO: <ciencialist@yahoogrupos.com.br>
DATE: 19/03/2005 18:57

Caro Franco,

Não são só formalidades acadêmicas. O salário e a reputação aumentam com a livre-docência. De qualquer forma, nem livre-docência, nem doutorado, nem mestrado, nem o prêmio Nobel são credenciais de desempenho em sala de aula. Na faculdade onde eu leciono, o CEFET-PR, os melhores professores geralmente têm apenas graduação ou uma pós lato senso. Claro que sempre há os "pontos fora da curva", de um lado e do outro, mas, para ser bom professor, não há curso nem concurso que dê conta. A receita é a mesma daquela para ser bom em qualquer coisa: paixão.

[ ]s

Alvaro Augusto





----- Original Message -----
From: Franco
To: ciencialist@yahoogrupos.com.br
Sent: Saturday, March 19, 2005 5:42 PM
Subject: Re: [ciencialist] Livre-docência


Talvez dependa da instituição, Alberto. Até ler a resposta do Álvaro eu
não entendia perfeitamente sobre a "importância" da livre-docência. Mas
há dois anos atrás tive aulas com uma professora, digo, com uma
livre-docente que vivia nos lembrando, em toda aula, que ela havia
passado no concurso de livre-docência; em toda a aula ela comentava ou
repetia algo já comentado a respeito de sua árdua luta para conseguir o
título. E, segundo a mesma, em uma das etapas do processo, sortearam um
tema para ser apresentado - se não me engano, o tema foi apresentado no
mesmo dia do sorteio.
Ah, sim, e a aula dela era terrível, considerando-se a sua falta de
didática... Durante muito tempo fiquei intrigado (e ainda continuo)
sobre a necessidade/utilidade de se fazer um curso de livre-docência.
Acredito que seja apenas por meras formalidades acadêmicas.

Franco.


Alberto Mesquita Filho wrote:

> ----- Original Message -----
> From: "Alvaro Augusto (E)"
> Sent: Friday, March 18, 2005 12:27 PM
> Subject: Re: [ciencialist] Livre-docência
>
> > Os livres-docentes são os Grandes Mestres Jedy do mundo acadêmico. Eles
> > são capazes de controlar os elementos, andar sobre as águas, multiplicar
> > os pães, rescussitar os mortos, etc. Após fazer mestrado, doutorado,
> > pós-doutorado, ter vários anos de profissão e publicar vários trabalhos
> > releventes, o candidato pode fazer concurso para livre-docência, quando
> > houver vagas. O concurso envolve, além de prova escrita, a preparação de
> > um resumo biográfico (memorial) e a defesa de uma tese em frente a banca
> > examinadora. Algumas faculdades também submetem o memorial, que deve
> > comprovar a obra do pesquisador, à argüição pública. É mole?
>
> No meu tempo [Putz, acho que estou ficando velho! :-)] o candidato a
> livre-docente devia também demonstrar a sua didática através de uma
> aula de
> uma hora de duração e sobre um tema sorteado com 24 horas de
> antecedência.
> Mas isso foi em meados do século passado, não sei se ainda está em voga.
>
> [ ]´s




[As partes desta mensagem que não continham texto foram removidas]



SUBJECT: Re: [ciencialist] Livre-docência
FROM: Luis Brudna <luisbrudna@gmail.com>
TO: ciencialist@yahoogrupos.com.br
DATE: 19/03/2005 19:22

Fazem essas coisas para contratar professores.

Normalmente sao 4 etapas.
- analise de curriculo
- prova escrita
- prova didatica (dar uma aula para uma banca, com assunto escolhido
com 24 horas de antecedencia)
- entrevista

Vou participar de uma dessas logo logo. To quase entrando em parafuso.

Neste exato momento termino de escrever minha tese... ainda faltam as
conclusoes (!), mas estou exausto. :-)

O mais importante ficou pro final e o gas acabou.

Ateh
Luis Brudna -

On Fri, 18 Mar 2005 14:01:44 -0300, Alberto Mesquita Filho
<albmesq@uol.com.br> wrote:
>
> ----- Original Message -----
> From: "Alvaro Augusto (E)"
> Sent: Friday, March 18, 2005 12:27 PM
> Subject: Re: [ciencialist] Livre-docência
>
> > Os livres-docentes são os Grandes Mestres Jedy do mundo acadêmico. Eles
> > são capazes de controlar os elementos, andar sobre as águas, multiplicar
> > os pães, rescussitar os mortos, etc. Após fazer mestrado, doutorado,
> > pós-doutorado, ter vários anos de profissão e publicar vários trabalhos
> > releventes, o candidato pode fazer concurso para livre-docência, quando
> > houver vagas. O concurso envolve, além de prova escrita, a preparação de
> > um resumo biográfico (memorial) e a defesa de uma tese em frente a banca
> > examinadora. Algumas faculdades também submetem o memorial, que deve
> > comprovar a obra do pesquisador, à argüição pública. É mole?
>
> No meu tempo [Putz, acho que estou ficando velho! :-)] o candidato a
> livre-docente devia também demonstrar a sua didática através de uma aula de
> uma hora de duração e sobre um tema sorteado com 24 horas de antecedência.
> Mas isso foi em meados do século passado, não sei se ainda está em voga.
>
> [ ]´s
> Alberto
> http://ecientificocultural.com/indice.htm
> Mas indiferentemente a tudo isso, o neutrino tem massa, o elétron não é
> uma carga elétrica coulombiana e a Terra se move. E a história se repetirá.
>
>


SUBJECT: CeticismoAberto.com
FROM: "Kentaro Mori" <kentaro.mori@itelefonica.com.br>
TO: ciencialist@yahoogrupos.com.br
DATE: 19/03/2005 19:42


Agora divido por temas, incluindo Ciência:

http://ceticismoaberto.com/ciencia/

Mori





SUBJECT: Re: Santo sudário e a vanilina(correção)
FROM: "rmtakata" <rmtakata@altavista.net>
TO: ciencialist@yahoogrupos.com.br
DATE: 19/03/2005 21:13


--- Em ciencialist@yahoogrupos.com.br, "pubmed2005"
> O problema é que voce persiste neste "mais confiáavel", e
> nem é disto que está se tratando.

Eh disso q se trata. O metodo A diz q. o tecido eh de 1300, o metodo B
diz q. o tecido eh bem anterior a isso - talvez de 40. Os metodos se
contradizem. Entao eh questao de ver qual o mais confiavel.

> Mas que novas descobertas de peso foram surgiram que podem
> mudar o padrão estipulado anteriormente.

Claro q. se houver novas descobertas de peso q. contradigam a idade
apontada pelo C-14 serah preciso rever.

> Se não há traços de vanilina nos tecidos nao analisados pelo
> radioisótopo, isso lança uma outra possibilidade

Lanca a possibilidade do metodo da analise quimica nao ser tao
confiavel. Sabemos q. o decaimento radioativo nao eh alterado por
pressao, temperatura ou qq outro fator ambiental conhecido. Jah a taxa
de reacoes quimicas variam enormente de acordo com a temperatura (e
sabemos q. o tecido em questao pegou fogo), presenca de umidade,
exposicao a luz, presenca de bacterias e fungos...

> Mas, se voces não se querem aparentemente aceitar as novas
> pesquisas tudo bem...essa é uma outra história.

Como eu disse a questao eh de se mostrar q. a metodologia de datacao
quimica eh mais confiavel do q. o de radioisotopos.

(Poderiamos analisar a questao sob a optica inversa: se vcs *querem*
aceitar essas pesquisas...)

[]s,

Roberto Takata





SUBJECT: Re: Santo sudário e a vanilina(correção)
FROM: "pubmed2005" <pubmed2005@yahoo.com.br>
TO: ciencialist@yahoogrupos.com.br
DATE: 19/03/2005 21:34


Lá vai

--- Em ciencialist@yahoogrupos.com.br, "rmtakata" <rmtakata@a...>
escreveu
>
> Eh disso q se trata. O metodo A diz q. o tecido eh de 1300, o
metodo B
> diz q. o tecido eh bem anterior a isso - talvez de 40. Os metodos se
> contradizem. Entao eh questao de ver qual o mais confiavel.

*Não se contradizem, o mínimo estipulado do método B é de 1300*. O
máximo que estipulado que é 3000

>
> > Mas que novas descobertas de peso foram surgiram que podem
> > mudar o padrão estipulado anteriormente.
>
> Claro q. se houver novas descobertas de peso q. contradigam a idade
> apontada pelo C-14 serah preciso rever.

Já está havendo revisão. A nova pesquisa foi publicada na Nature

>
> Lanca a possibilidade do metodo da analise quimica nao ser tao
> confiavel. Sabemos q. o decaimento radioativo nao eh alterado por
> pressao, temperatura ou qq outro fator ambiental conhecido. Jah a
taxa
> de reacoes quimicas variam enormente de acordo com a temperatura (e
> sabemos q. o tecido em questao pegou fogo), presenca de umidade,
> exposicao a luz, presenca de bacterias e fungos...

Não seja tão certo da confiabilidade. de uma lida:

O Dr Leôncio Garza Valdes, médico e professor da Universidade do
Texas, pesquisou e descobriu resíduos orgânicos gerados por bactérias
em múmias. Ao tentar data-las pelo processo de radiocarbono descobriu
que esses resíduos, que formavam um revestimento bio-plástico sobre
os fios do tecido, alteravam o resultado da datação, devido à
impossibilidade de se limpá-los de modo adequado.

Em 1993, O Dr.Valdes conseguiu algumas amostras do Sudário que haviam
sido cortadas na mesma ocasião do teste de rádiocarbono de 1988,
junto ao Professor Giovani Riggi, o microanalista de Turim já
mencionado, guardião das amostras, e homem de confiança do Cardeal
Ballestrero.

Ao analisá-las descobriu a presença do revestimento bio-plástico
(Lichenothelia Varnish) sobre os fios de linho, assim como a presença
desses organismos (Lichenothelia) vivos.



Aliás, o Dr Valdes não conseguiu fazer o teste de rádiocarbono
exatamente pela dificuldade em se fazer a limpeza. Ele chegou a
utilizar exatamente o mesmo processo de limpeza que foi utilizado
pelos laboratórios do teste de 1988, e constatou que o revestimento
bio-plástico não saia.

Segundo o Dr. Valdes: &#8220;A pátina causada pelos fungos é um acréscimo
natural nas superfícies antigas estáveis. São necessárias centenas de
anos para que os fungos criem um &#8216;verniz&#8217; contínuo. As fibras do
Sudário de Turim têm um depósito espesso devido às bactérias. A data
obtida mediante o C14 em 1988 deve-se na realidade, a uma mistura do
C14 do Tecido do Sudário com o C14 da pátina de bactérias. Este seria
de até 60%&#8221;.

À luz desses estudos, o inventor do processo de datação por
rádiocarbono pelo processo usado no Sudário, Harry Gove, teve de
declarar em 27 de Janeiro de 1995: &#8220;A técnica que se usou em 1988
para a datação do Sudário de Turim por meio do C14 foi inventada em
meu laboratório, na Universidade de Rochester, em 1977. Depois dessa
datação, estive convencido do resultado durante anos. Recentemente,
porém, o doutor Garza Valdés, de San Antonio, Texas, apresentou
provas consistentes a respeito de um tipo de contaminação por carbono
recente produzida nos fios do Sudário por bactérias que os processos
de limpeza usados pelos três laboratórios podem não ter removido.
Essa contaminação, de acordo com sua espessura, pode fazer com que a
data fornecida pelos três laboratórios seja mais recente&#8221;.

3.5- A contestação de Kouznetsov

O diretor dos Laboratórios E.A. Sedoo de Pesquisa de Bio-Polímeros de
Moscou e premio Lênin de Ciência, Dr. A. Kouznetsov, participou de um
congresso em Londres onde foi convidado a testar seus métodos de
datação por radiocarbono pelo Geólogo Dr Guy Berthault, para comparar
com os resultados do exame do Sudário.

Analisando a história do Sudário, descobriu que ele passou por um
incêndio em 1532. Resolveu recriar a condição em laboratório, e
descobriu que o linho absorveu C14 recente, principalmente por ter se
submetido a altas temperaturas por tempos suficientemente longos na
presença de produtos da combustão (água, anidrido carbônico e óxido
de carbono) e íons de prata, pois a urna era de prata, capazes de
agir como catalisadores.

Para comprovar isso, Kouznetsov pegou um tecido de linho do séc. I,
procedente de Em Gedi (Israel), e enviou ao Laboratório de Tucson,
Arizona, onde fizeram a datação.

Mais tarde, o professor russo queimou outro pedaço do mesmo lençol,
simulando a condição do Sudário no incêndio, e enviou novamente para
Tucson. O resultado desta datação foi de 1300 anos mais recente.

.6- Conclusão

Conforme o Dr Valdés e o Dr Kouznetsov, não se aplica testes de
rádiocarbono a matérias têxteis como o linho, devido aos seus altos
níveis de contaminação sem possibilidade prática de remoção.

> Como eu disse a questao eh de se mostrar q. a metodologia de datacao
> quimica eh mais confiavel do q. o de radioisotopos.

Se encontro uma prova nova, e ela me fornece evidencias novas o
quadro tem de mudar, e lá se vai a confiabilidade. E é isso que está
acontecendo. O que parece que é, nao é

>
> (Poderiamos analisar a questao sob a optica inversa: se vcs *querem*
> aceitar essas pesquisas...)


Takata , algumas coisa temos que aceitar: assim como aceitamos a
datação do carbono-14. Nao podemos ter o santo sudário nas maos para
testá-los
Só que a ciencia avança , e o teste é bem antiguinho e provavelmente
deve estar bem defasado. De qualquer maneira muitos cientistas tem de
acreditar nisso - ter confiança peloe menos em algum grau E tem
aqueles que questionam e apresentam provas que colidem com as
anteriores, enfim o mundo não para.















SUBJECT: Re: Santo sudário e a vanilina(correção)
FROM: "pubmed2005" <pubmed2005@yahoo.com.br>
TO: ciencialist@yahoogrupos.com.br
DATE: 19/03/2005 21:39


Takata nao esteja tão certo. Leia o texto

Revestimento bio-plástico

O Dr Leôncio Garza Valdes, médico e professor da Universidade do
Texas, pesquisou e descobriu resíduos orgânicos gerados por bactérias
em múmias. Ao tentar data-las pelo processo de radiocarbono descobriu
que esses resíduos, que formavam um revestimento bio-plástico sobre
os fios do tecido, alteravam o resultado da datação, devido à
impossibilidade de se limpá-los de modo adequado.

Em 1993, O Dr.Valdes conseguiu algumas amostras do Sudário que haviam
sido cortadas na mesma ocasião do teste de rádiocarbono de 1988,
junto ao Professor Giovani Riggi, o microanalista de Turim já
mencionado, guardião das amostras, e homem de confiança do Cardeal
Ballestrero.

Ao analisá-las descobriu a presença do revestimento bio-plástico
(Lichenothelia Varnish) sobre os fios de linho, assim como a presença
desses organismos (Lichenothelia) vivos.



Aliás, o Dr Valdes não conseguiu fazer o teste de rádiocarbono
exatamente pela dificuldade em se fazer a limpeza. Ele chegou a
utilizar exatamente o mesmo processo de limpeza que foi utilizado
pelos laboratórios do teste de 1988, e constatou que o revestimento
bio-plástico não saia.

Segundo o Dr. Valdes: &#8220;A pátina causada pelos fungos é um acréscimo
natural nas superfícies antigas estáveis. São necessárias centenas de
anos para que os fungos criem um &#8216;verniz&#8217; contínuo. As fibras do
Sudário de Turim têm um depósito espesso devido às bactérias. A data
obtida mediante o C14 em 1988 deve-se na realidade, a uma mistura do
C14 do Tecido do Sudário com o C14 da pátina de bactérias. Este seria
de até 60%&#8221;.

À luz desses estudos, o inventor do processo de datação por
rádiocarbono pelo processo usado no Sudário, Harry Gove, teve de
declarar em 27 de Janeiro de 1995: &#8220;A técnica que se usou em 1988
para a datação do Sudário de Turim por meio do C14 foi inventada em
meu laboratório, na Universidade de Rochester, em 1977. Depois dessa
datação, estive convencido do resultado durante anos. Recentemente,
porém, o doutor Garza Valdés, de San Antonio, Texas, apresentou
provas consistentes a respeito de um tipo de contaminação por carbono
recente produzida nos fios do Sudário por bactérias que os processos
de limpeza usados pelos três laboratórios podem não ter removido.
Essa contaminação, de acordo com sua espessura, pode fazer com que a
data fornecida pelos três laboratórios seja mais recente&#8221;.


A contestação de Kouznetsov

O diretor dos Laboratórios E.A. Sedoo de Pesquisa de Bio-Polímeros de
Moscou e premio Lênin de Ciência, Dr. A. Kouznetsov, participou de um
congresso em Londres onde foi convidado a testar seus métodos de
datação por radiocarbono pelo Geólogo Dr Guy Berthault, para comparar
com os resultados do exame do Sudário.

Analisando a história do Sudário, descobriu que ele passou por um
incêndio em 1532. Resolveu recriar a condição em laboratório, e
descobriu que o linho absorveu C14 recente, principalmente por ter se
submetido a altas temperaturas por tempos suficientemente longos na
presença de produtos da combustão (água, anidrido carbônico e óxido
de carbono) e íons de prata, pois a urna era de prata, capazes de
agir como catalisadores.

Para comprovar isso, Kouznetsov pegou um tecido de linho do séc. I,
procedente de Em Gedi (Israel), e enviou ao Laboratório de Tucson,
Arizona, onde fizeram a datação.

Mais tarde, o professor russo queimou outro pedaço do mesmo lençol,
simulando a condição do Sudário no incêndio, e enviou novamente para
Tucson. O resultado desta datação foi de 1300 anos mais recente.


Conclusão

Conforme o Dr Valdés e o Dr Kouznetsov, não se aplica testes de
rádiocarbono a matérias têxteis como o linho, devido aos seus altos
níveis de contaminação sem possibilidade prática de remoção.



--- Em ciencialist@yahoogrupos.com.br, "rmtakata" <rmtakata@a...>
escreveu
>
> Lanca a possibilidade do metodo da analise quimica nao ser tao
> confiavel. Sabemos q. o decaimento radioativo nao eh alterado por
> pressao, temperatura ou qq outro fator ambiental conhecido. Jah a
taxa
> de reacoes quimicas variam enormente de acordo com a temperatura (e
> sabemos q. o tecido em questao pegou fogo), presenca de umidade,
> exposicao a luz, presenca de bacterias e fungos...






SUBJECT: Re: Santo sudário e a vanilina(correção)
FROM: "rmtakata" <rmtakata@altavista.net>
TO: ciencialist@yahoogrupos.com.br
DATE: 19/03/2005 22:07


--- Em ciencialist@yahoogrupos.com.br, "pubmed2005"
> Revestimento bio-plástico

Bem, pra q. a datacao tenha sido influenciada pelo revestimento
bioplastico e q. o tecido tenha idade compativel com a epoca de XPTO,
seria preciso q. houvesse a mesma massa do tecido em bacteria. Eu
mostro os detalhes do calculo em:

http://observatorio.ultimosegundo.ig.com.br/ofjor/ofc051299.htm

[]s,

Roberto Takata





SUBJECT: Re: Santo sudário e a vanilina(correção)
FROM: "rmtakata" <rmtakata@altavista.net>
TO: ciencialist@yahoogrupos.com.br
DATE: 19/03/2005 22:09


--- Em ciencialist@yahoogrupos.com.br, "pubmed2005"
> *Não se contradizem, o mínimo estipulado do método B é de
> 1300*. O máximo que estipulado que é 3000

Se nao se contradizem pegue-se a area de sobreposicao: o tecido seria
de cerca de 1.300, portanto nao seria original.

Enfim, nada novo sob o sol.

[]s,

Roberto Takata





SUBJECT: Re: Santo sudário e a vanilina(correção)
FROM: "pubmed2005" <pubmed2005@yahoo.com.br>
TO: ciencialist@yahoogrupos.com.br
DATE: 19/03/2005 22:11


É, mas isso foi testado na prática pelo Dr. Valdez do Texas, e o
resultado foi alterado. Os cálculos nao correspodem muito bem na
prática o que relmente acontece.
O problema todo é que voce teria que comprovar isso
experimentalmente. Já ´que temos resultamos experimentais de que o
revestimento realmente interfere no resultado da datação como o
apresentado tb pelo cientista soviético Kouznetsov.



--- Em ciencialist@yahoogrupos.com.br, "rmtakata" <rmtakata@a...>
escreveu
>
> --- Em ciencialist@yahoogrupos.com.br, "pubmed2005"
> > Revestimento bio-plástico
>
> Bem, pra q. a datacao tenha sido influenciada pelo revestimento
> bioplastico e q. o tecido tenha idade compativel com a epoca de
XPTO,
> seria preciso q. houvesse a mesma massa do tecido em bacteria. Eu
> mostro os detalhes do calculo em:
>
> http://observatorio.ultimosegundo.ig.com.br/ofjor/ofc051299.htm
>
> []s,
>
> Roberto Takata





SUBJECT: Re: Santo sudário e a vanilina(correção)
FROM: "rmtakata" <rmtakata@altavista.net>
TO: ciencialist@yahoogrupos.com.br
DATE: 19/03/2005 22:17


--- Em ciencialist@yahoogrupos.com.br, "pubmed2005"
> É, mas isso foi testado na prática pelo Dr. Valdez do Texas,

Soh o q. a magnitude de alteracao nao eh o suficiente para
compatibilizar a idade do tecido com a data da vida de XPTO.

Eh o mesmo q. querer provar q. uma descarga de banheiro pode destruir
uma cidade porq. pode ser registrada por um sismografo.

[]s,

Roberto Takata





SUBJECT: Re: Santo sudário e a vanilina(correção)
FROM: "pubmed2005" <pubmed2005@yahoo.com.br>
TO: ciencialist@yahoogrupos.com.br
DATE: 19/03/2005 22:30


Bem , isso é o que voce supõe. Os trabalhos experimentais foram
apresentados. O interessante seria voce apresentar algo experimental
neste sentido.




--- Em ciencialist@yahoogrupos.com.br, "rmtakata" <rmtakata@a...>
escreveu
>
> --- Em ciencialist@yahoogrupos.com.br, "pubmed2005"
> > É, mas isso foi testado na prática pelo Dr. Valdez do Texas,
>
> Soh o q. a magnitude de alteracao nao eh o suficiente para
> compatibilizar a idade do tecido com a data da vida de XPTO.
>
> Eh o mesmo q. querer provar q. uma descarga de banheiro pode
destruir
> uma cidade porq. pode ser registrada por um sismografo.
>
> []s,
>
> Roberto Takata





SUBJECT: Re: Santo sudário e a vanilina(correção)
FROM: "pubmed2005" <pubmed2005@yahoo.com.br>
TO: ciencialist@yahoogrupos.com.br
DATE: 19/03/2005 22:54


Além do mais ainda tem o problema dos testes de carbono terem sido
aplicados somente nos remendos medievais, somado a bioplastia e o
linho com carbono-14 recente.... dá uma boa diferença

Enfim, os teste de 1988 pode ter tido falhas .


--- Em ciencialist@yahoogrupos.com.br, "rmtakata" <rmtakata@a...>
escreveu
>
> --- Em ciencialist@yahoogrupos.com.br, "pubmed2005"
> > É, mas isso foi testado na prática pelo Dr. Valdez do Texas,
>
> Soh o q. a magnitude de alteracao nao eh o suficiente para
> compatibilizar a idade do tecido com a data da vida de XPTO.
>
> Eh o mesmo q. querer provar q. uma descarga de banheiro pode
destruir
> uma cidade porq. pode ser registrada por um sismografo.
>
> []s,
>
> Roberto Takata





SUBJECT: Re: [ciencialist] Re: Santo sudário e a vanilina(correção)
FROM: "Alberto Mesquita Filho" <albmesq@uol.com.br>
TO: <ciencialist@yahoogrupos.com.br>
DATE: 19/03/2005 22:55

----- Original Message -----
From: "rmtakata"
Sent: Saturday, March 19, 2005 5:40 PM
Subject: [ciencialist] Re: Santo sudário e a vanilina(correção)

> > Poder dizer eles podem. O que eles não podem é tentar nos
> > convencer que estão dizendo a verdade

> Sim, poderiamos dizer. Mas eu nao disse (e ao q. eu saiba nem o
> Oraculo/Homero). O q. eu disse e digo novamente eh q. ainda estah para se
> demonstrar q. datacao quimica eh mais confiavel do q. datacao por
> radioisotopos.

Mas... Com esse corte a piada perdeu a graça. Então vou entrar na sua,
vejamos:

1) O que significa ser mais confiável?
2) Há como quantificar essa confiabilidade?
Se sim:
3) Seria possível traduzir em termos probabilísticos?
4) Eu poderia dizer que o método X é Z% mais confiável que o método Y?
Se não:
5) Este ser confiável seria do tipo ter sido mais vezes corroborado?
Ou então:
6) Seria o método X potencialmente gerador de hipóteses falseáveis em maior
número que o método Y?
7) Haveria como objetivar essa diferença?

Perceba que ser cético é muito fácil. O difícil é ser cientista e ter que
aturar esse "ceticismo insosso" que contamina a Internet. Nada contra um
ceticismo sadio, mesmo porque não há como assumir uma postura científica
deixando-se o ceticismo totalmente de lado.

PS: Insosso está sendo aqui empregado no sentido de algo que não leva a
nada. A esse respeito concordo plenamente com as queixas do Sérgio em
especial com a conclusão final, a de que a postura de vocês "já encheu o
saco".

[ ]´s
Alberto
http://ecientificocultural.com/indice.htm
Mas indiferentemente a tudo isso, o neutrino tem massa, o elétron não é
uma carga elétrica coulombiana e a Terra se move. E a história se repetirá.



SUBJECT: Re: Santo sudário e a vanilina(correção)
FROM: "pubmed2005" <pubmed2005@yahoo.com.br>
TO: ciencialist@yahoogrupos.com.br
DATE: 19/03/2005 23:40


O impressionante é que todos esses processos que voce citou, tanto os
quimicos, físicos e biológicos no interferiu em nada na detecção da
*ausencia de vanilina* nas partes dos tecidos onde nao existiam os
remendos. Por então essa substancia só apareceu nos remendos mais
recentes feitos na idade média? Como voce me explica isso então?


--- Em ciencialist@yahoogrupos.com.br, "rmtakata" <rmtakata@a...>
escreveu
> Lanca a possibilidade do metodo da analise quimica nao ser tao
> confiavel. Sabemos q. o decaimento radioativo nao eh alterado por
> pressao, temperatura ou qq outro fator ambiental conhecido. Jah a
taxa
> de reacoes quimicas variam enormente de acordo com a temperatura (e
> sabemos q. o tecido em questao pegou fogo), presenca de umidade,
> exposicao a luz, presenca de bacterias e fungos...






SUBJECT: Re: Santo sudário e a vanilina(correção)
FROM: "rmtakata" <rmtakata@altavista.net>
TO: ciencialist@yahoogrupos.com.br
DATE: 19/03/2005 23:58


--- Em ciencialist@yahoogrupos.com.br, "Alberto Mesquita
> 1) O que significa ser mais confiável?
> 2) Há como quantificar essa confiabilidade?

Basta pegar uma serie de objetos de idades conhecidas e aplicar ambas
as metodologias. Comparam-se os intervalos do resultados com as idades
dos objetos.

[]s,

Roberto Takata





SUBJECT: Re: Santo sudário e a vanilina(correção)
FROM: "rmtakata" <rmtakata@altavista.net>
TO: ciencialist@yahoogrupos.com.br
DATE: 20/03/2005 00:00


--- Em ciencialist@yahoogrupos.com.br, "pubmed2005"
> O impressionante é que todos esses processos que voce citou,
> tanto os quimicos, físicos e biológicos no interferiu em
> nada na detecção da *ausencia de vanilina* nas partes dos
> tecidos onde nao existiam os remendos.

Ueh, o fogo apenas acelera a degradacao da vanilina.

[]s,

Roberto Takata





SUBJECT: Nova teoria propõe como tornar objetos invisíveis
FROM: "pubmed2005" <pubmed2005@yahoo.com.br>
TO: ciencialist@yahoogrupos.com.br
DATE: 20/03/2005 00:00


Nova teoria propõe como tornar objetos invisíveis
02/03/2005


Os fãs da série Jornada nas Estrelas estão acostumados a ver as naves
Klingon se camuflarem, ficando totalmente invisíveis, reaparecendo
apenas para disparar suas armas. Ainda estamos muito longe de ver
algo parecido acontecer na prática.

Mas agora dois cientistas da Universidade da Pensilvânia, Nader
Engheta e Andrea Alu, demonstraram que é possível fazer com que um
objeto fique totalmente invisível. Por enquanto, eles conseguem
tornar invisível qualquer objeto desde que ele seja uma esfera
microscópica feita de ouro ou prata.

Para fazer o objeto ficar invisível, os cientistas alteraram a forma
como ele reflete a luz, utilizando plasmons - vibrações dos elétrons
que acontecem na superfície de alguns metais. Os plasmons foram
descobertos quando a equipe do Dr. Thomas Ebbesen incidiu luz sobre
uma placa de ouro contendo furos menores do que o comprimento de onda
dessa luz. Eles não esperavam ver nada do outro lado mas, para sua
surpresa, saía mais luz do outro lado do que entrava.

"Um design adequado pode induzir uma queda dramática na superfície de
reflexão, tornando o objeto praticamente invisível a um observador,"
escrevem os cientistas em seu artigo, disponível no site da
Universidade (veja quadro Para Navegar).

Os objetos são visíveis aos nossos olhos porque eles refletem a luz.
As cores dos objetos variam conforme o comprimento de onda da luz que
eles refletem. A parcela da luz não refletida é absorvida. O que os
cientistas fizeram foi alterar essa capacidade de reflexão, fazendo
com que praticamente toda a luz fosse absorvida. Fazendo a energia
dos plasmons coincidir com a energia da luz incidente, eles
conseguiram que toda a luz incidente fosse absorvida de um lado e
emitida do outro lado do objeto.

Mas a descoberta poderá ter aplicações em outros comprimentos de
onda, que não a luz visível. Por exemplo, provavelmente nunca será
possível fazer com que um homem se torne invisível aos olhos de
outros humanos, mas ele poderá se tornar invisível às microondas, por
exemplo, que têm um comprimento de onda maior.

A descoberta tem grande interesse científico e poderá mesmo, um dia,
permitir aplicações práticas. Mas os cientistas avisam que ainda não
se pode pensar em nenhum objeto do dia-a-dia tornando-se invisível.
Objetos grandes têm geometria complexa e necessitariam ser revestidos
com camadas de materiais adequados, com características detalhadas em
nível nanométrico. Além disso, eles precisariam ser construídos de
modo a refletir todos os comprimentos de onda da luz visível, o que
ainda não foi conseguido.

A revista Nature também publicou uma reportagem baseada no artigo
divulgado pelos cientistas.



Cientistas da Universidade de Ohio (Estados Unidos) descobriram uma
forma de ampliar a absorção de luz por uma rede metálica em até 1.000
vezes, o que poderá abrir caminho para a fabricação de sensores
químicos e instrumentos de laboratórios muito mais eficientes. A
descoberta consiste em uma nova técnica de revestimento que permite
que a tela capture e transmita mais luz através de seus poros
microscópicos do que seria possivel normalmente.

James V. Coe e seus colegas também descobriram que, se a malha
metálica for recoberta com moléculas de gordura, pode-se utilizar o
calor para controlar a quantidade de luz que passa pela malha. Os
resultados foram apresentados por Coe e seu aluno Kenneth R.
Rodriguez na reunião anual da American Chemical.

"Com o revestimento correto, nós descobrimos que podemos controlar
precisamente o diâmetro dos buracos e a quantidade de luz
transmitida. De fato, a malha age como uma chave ótica," explica Coe,
referindo-se a dispositivos que controlam sinais de luz em
optoeletrônica. "Com a adição de controles de calor, você pode chamá-
la de chave termo-ótica."

A Universidade registrou dois pedidos de patente para a nova
tecnologia e está procurando parceiros comerciais para seu
aprimoramento. Uma das possibilidades de utilização é o estudo da
interação entre colesterol e outras células, ou ainda o efeito do
calor sobre o DNA.

O fenômeno de ampliação da luz foi descoberto em 1.998 nos
laboratórios da empresa NEC. Os pesquisadores perceberam que fileiras
nanométricas de átomos de prata podem transmitir uma quantidade
enorme de luz na forma de pacotes de energia chamados plasmons de
superfície. A palavra plasmon é uma junção e plasma e fóton. A luz é
excitada e reaparece do outro lado da placa, em quantidade superior à
prevista.

Os pesquisadores da Universidade de Ohio queriam saber se poderiam
criar plasmons em outros metais, utilizando luz infravermelha ao
invés de ultravioleta, como feito nos laboratórios da NEC. Para suas
experiências, eles utilizaram uma malha metálica de níquel,
disponível comercialmente. A olho nu, a malha metálica parece-se com
uma fita metálica flexível. Suas perfurações têm menos do que 13
micrômetros de diâmetro.

Os pesquisadores tiveram que criar um método de cobrir a malha de
níquel com uma camada de átomos de cobre. Como resultado dessa
pesquisa, eles descobriram que poderiam afinar o processo para
preencher as bordas das perfurações e estreitar os buracos para
qualquer tamanho que eles desejassem. Quando o tamanho dos buracos
atingiu uma dimensão comparável àquele do comprimento de onda da luz
que atingia a malha metálica, apareceram os plasmons.

Como os buracos normalmente cobrem 25 por cento da superfície, a
malha deveria deixar passar apenas 25 por cento da luz que a atingia.
Mas, nos testes, a placa transmitiu 75 por cento da luz, o que
sugeria que a luz incidente sobre a malha estava sendo transmitida
para o outro lado. É como se a placa perfurada sumisse com a luz de
um lado e a fizesse reaparecer do outro lado, uma vez que não há área
suficiente para que tanta luz atinja o outro lado.

Quando os pesquisadores adicionaram uma camada de gordura, mais
especificamente moléculas obtidas da soja, a superfície absorveu
ainda mais luz, atingindo até 1.000 vezes mais do que em qualquer
outra experiência com plasmons.

As moléculas de gordura vegetal podem ser utilizadas para controlar a
quantidade de luz passante devido ao seu formato. Em temperatura
ambiente elas formam longas cadeias que permanecem retas sobre a
superfície. Mas, à medida em que a temperatura sobe, as cadeias se
quebram, alterando a polarização da luz. Em um processo ainda não
totalmente entendido pelos pesquisadores, o processo funciona
seguidamente desde que se mantenha as moléculas hidratadas, ou seja,
desde que a temperatura não ultrapasse os 100º C.







SUBJECT: Re: Santo sudário e a vanilina(correção)
FROM: "rmtakata" <rmtakata@altavista.net>
TO: ciencialist@yahoogrupos.com.br
DATE: 20/03/2005 00:07


--- Em ciencialist@yahoogrupos.com.br, "pubmed2005"
> Bem , isso é o que voce supõe. Os trabalhos experimentais
> foram apresentados.

Exatamente, foram apresentados. E os resultados apresentam uma
magnitude de alteracao q. nao permite compatibilizar com a idade do
inicio da era crista. Nao eh suposicao, eh apenas a leitura dos
resultados.

> O interessante seria voce apresentar algo experimental
> neste sentido.

Oras, os proprios experimentos realizados mostram isso. O trabalho de
Kouznetsov detetou apenas 25% de troca *maxima* pela fumaca. Isso nao
eh o suficiente para rejuvenescer um tecido de 2000 anos para um de
500 anos.

Para as bacterias, como dito, seria necessario q. houvesse a mesma
quantidade em massa de bacterias do q. de tecido.

[]s,

Roberto Takata





SUBJECT: Re: Santo sudário e a vanilina(correção)
FROM: "pubmed2005" <pubmed2005@yahoo.com.br>
TO: ciencialist@yahoogrupos.com.br
DATE: 20/03/2005 00:09


Ai Takata

O processo microquimico nao é para determinar a data , apenas para
investigar que determinados tipos de substancias dura até
determinados períodos históricos, o que pode ser uma ferramente
utilizada para precisar , corroborar ou autenticar ainda mais a data.
Com base em experiencias relizadas com artefatos históricos e
arqueologicos, os teste microquímicos é um poderosos auxiliar, e uma
ferramenta importante para detecção de substancia que nao resistiriam
a uma quantidade X de anos e séculos. Já se tem um experiencia
pratica bem convicente deste método praticado arqueólogos em
artefatos e objetos antigos

Foi o que aconteceu a ausencia de vanilina não corrobora o primeiro
estudo, mas colide com ele. Isso demonstra que o primeiro estudo pode
ter falhas.


--- Em ciencialist@yahoogrupos.com.br, "rmtakata" <rmtakata@a...>
escreveu
>
> --- Em ciencialist@yahoogrupos.com.br, "Alberto Mesquita
> > 1) O que significa ser mais confiável?
> > 2) Há como quantificar essa confiabilidade?
>
> Basta pegar uma serie de objetos de idades conhecidas e aplicar
ambas
> as metodologias. Comparam-se os intervalos do resultados com as
idades
> dos objetos.
>
> []s,
>
> Roberto Takata





SUBJECT: Re: Santo sudário e a vanilina(correção)
FROM: "pubmed2005" <pubmed2005@yahoo.com.br>
TO: ciencialist@yahoogrupos.com.br
DATE: 20/03/2005 00:13


Mas tem um porém, o manto nao pegou fogo, apenas parte dele que foram
remendadas. Mas de qualquer forma o fogo alteraria o resultado do
carbono-14, porque aceleraria a queima das moléculas de carbono já
existente no tecido

--- Em ciencialist@yahoogrupos.com.br, "rmtakata" <rmtakata@a...>
escreveu
>
> --- Em ciencialist@yahoogrupos.com.br, "pubmed2005"
> > O impressionante é que todos esses processos que voce citou,
> > tanto os quimicos, físicos e biológicos no interferiu em
> > nada na detecção da *ausencia de vanilina* nas partes dos
> > tecidos onde nao existiam os remendos.
>
> Ueh, o fogo apenas acelera a degradacao da vanilina.
>
> []s,
>
> Roberto Takata





SUBJECT: Re: Santo sudário e a vanilina(correção)
FROM: "pubmed2005" <pubmed2005@yahoo.com.br>
TO: ciencialist@yahoogrupos.com.br
DATE: 20/03/2005 00:18


Quem fez os calculos foi voce, e isso tb nao inspira muita
confiabilidade - j á que voce não apresentou nenhum experimento
testável que comprova o que voce diz.

Uma diferença de 500 anos já traria um impacto siginificativo.O ideal
seria agora analisar as partes nao remendadas do manto





--- Em ciencialist@yahoogrupos.com.br, "rmtakata" <rmtakata@a...>
escreveu
>
> --- Em ciencialist@yahoogrupos.com.br, "pubmed2005"
> > Bem , isso é o que voce supõe. Os trabalhos experimentais
> > foram apresentados.
>
> Exatamente, foram apresentados. E os resultados apresentam uma
> magnitude de alteracao q. nao permite compatibilizar com a idade do
> inicio da era crista. Nao eh suposicao, eh apenas a leitura dos
> resultados.
>
> > O interessante seria voce apresentar algo experimental
> > neste sentido.
>
> Oras, os proprios experimentos realizados mostram isso. O trabalho
de
> Kouznetsov detetou apenas 25% de troca *maxima* pela fumaca. Isso
nao
> eh o suficiente para rejuvenescer um tecido de 2000 anos para um de
> 500 anos.
>
> Para as bacterias, como dito, seria necessario q. houvesse a mesma
> quantidade em massa de bacterias do q. de tecido.
>
> []s,
>
> Roberto Takata





SUBJECT: Re: Santo sudário e a vanilina(correção)
FROM: "rmtakata" <rmtakata@altavista.net>
TO: ciencialist@yahoogrupos.com.br
DATE: 20/03/2005 00:25


--- Em ciencialist@yahoogrupos.com.br, "pubmed2005"
> O processo microquimico nao é para determinar a data ,
> apenas para investigar que determinados tipos de substancias
> dura até determinados períodos históricos, o que pode ser
> uma ferramente utilizada para precisar , corroborar ou
> autenticar ainda mais a data.

Ou seja nao eh pra determinar a data, mas eh pra determinar a data...

> Foi o que aconteceu a ausencia de vanilina não corrobora o
> primeiro estudo, mas colide com ele. Isso demonstra que o
> primeiro estudo pode ter falhas.

Ou q. a ausencia de vanilina nao eh bom indicador da idade de um objeto.

(Melhor vc decidir se vc acha q. o estudo colide ou nao. Na outra
mensagem vc achava q. nao se contradiziam.)

"*Não se contradizem, o mínimo estipulado do método B é de 1300*. O
máximo que estipulado que é 3000."
http://br.groups.yahoo.com/group/ciencialist/message/45066

[]s,

Roberto Takata





SUBJECT: Re: Santo sudário e a vanilina(correção)
FROM: "rmtakata" <rmtakata@altavista.net>
TO: ciencialist@yahoogrupos.com.br
DATE: 20/03/2005 00:28


--- Em ciencialist@yahoogrupos.com.br, "pubmed2005"
> Mas tem um porém, o manto nao pegou fogo, apenas parte dele
> que foram remendadas.

O manto pegou fogo - parte do manto foi destruido. De todo modo as
partes q. nao queimaram estiveram expostas a temperaturas razoaveis
pelo jeito.

> Mas de qualquer forma o fogo alteraria o resultado do
> carbono-14, porque aceleraria a queima das moléculas de
> carbono já existente no tecido

Bem a parte queimada desapareceu e nao se mede essa parte. A parte
restante seria alterada apenas se o fogo tivesse preferencia por C-12,
coisa de q. nao se tem noticia.

[]s,


Roberto Takata





SUBJECT: Re: Santo sudário e a vanilina(correção)
FROM: "rmtakata" <rmtakata@altavista.net>
TO: ciencialist@yahoogrupos.com.br
DATE: 20/03/2005 00:34


--- Em ciencialist@yahoogrupos.com.br, "pubmed2005"
> Quem fez os calculos foi voce, e isso tb nao inspira muita
> confiabilidade - j á que voce não apresentou nenhum
> experimento testável que comprova o que voce diz.

Os calculos foram verificados. Eles se baseiam em formulas obtidas
experimentalmente - alias, os cientistas usam essas formulas para
fazer a datacao por radioisotopos.

Se acha q. tem um erro nos calculos pode refaze-los e indicar onde
houve erro.

Outros cientistas tb fizeram calculos semelhantes aos meus e chegaram
aos mesmos resultados.

> Uma diferença de 500 anos já traria um impacto
> siginificativo.O ideal seria agora analisar as partes nao
> remendadas do manto

Sim, o ideal seria analisar outras partes - embora seja apenas
suposicao Rogers de q. as amostras datadas por radioisotopos fossem
retiradas da area de remendo (ou o remendo era tao bom q. os
especialistas q. retiraram a amostra nao notaram a diferenca ou os
especialistas eram incompententes o suficiente para retirar amostras
do remendo... claro nao se pode descartar nenhuma dessas hipoteses).

Uma diferenca de 500 anos nao eh o suficiente para compatibilizar a
idade do manto com o inicio da era crista.

[]s,

Roberto Takata





SUBJECT: Re: Santo sudário e a vanilina(correção)
FROM: "pubmed2005" <pubmed2005@yahoo.com.br>
TO: ciencialist@yahoogrupos.com.br
DATE: 20/03/2005 00:36


--- Em ciencialist@yahoogrupos.com.br, "rmtakata" <rmtakata@a...>
escreveu
>
> --- Em ciencialist@yahoogrupos.com.br, "pubmed2005"
> > O processo microquimico nao é para determinar a data ,
> > apenas para investigar que determinados tipos de substancias
> > dura até determinados períodos históricos, o que pode ser
> > uma ferramente utilizada para precisar , corroborar ou
> > autenticar ainda mais a data.
>
> Ou seja nao eh pra determinar a data, mas eh pra determinar a
data...

Se voce levar em consideração que um pode complementar o resultado do
outro outro, pode sim ser um auxiliar.


>
> Ou q. a ausencia de vanilina nao eh bom indicador da idade de um
objeto.

Nem essa é a intenção.

>
> (Melhor vc decidir se vc acha q. o estudo colide ou nao. Na outra
> mensagem vc achava q. nao se contradiziam.)
>
> "*Não se contradizem, o mínimo estipulado do método B é de 1300*. O
> máximo que estipulado que é 3000."
> http://br.groups.yahoo.com/group/ciencialist/message/45066
>

Colide no sentido de que o manto é mais antigo do que se pensava. O
prmeiro método se prende apenas ao medieval. Mais é estipulada uma
data mínima em que a vanilina começa a escassear com base nos estudos
praticos de artefatos de arqueologia, e uma boa margem especulativa
da antiguidade do tecido.









SUBJECT: Re: Santo sudário e a vanilina(correção)
FROM: "pubmed2005" <pubmed2005@yahoo.com.br>
TO: ciencialist@yahoogrupos.com.br
DATE: 20/03/2005 00:39


Mas a outra parte ficou exposta ao calor e alterou as moléculas do
carbono presente no tecido. O tecido pode em parte ter ficado
chamuscado sem queimar. O que altera o resultado do carbono-14

--- Em ciencialist@yahoogrupos.com.br, "rmtakata" <rmtakata@a...>
escreveu
> Bem a parte queimada desapareceu e nao se mede essa parte. A parte
> restante seria alterada apenas se o fogo tivesse preferencia por C-
12,
> coisa de q. nao se tem noticia.
>
> []s,
>
>
> Roberto Takata





SUBJECT: Re: [ciencialist] Re: Santo sudário e a vanilina(correção)
FROM: "Emiliano Chemello - Yahoo Grupos" <chemelloe@yahoo.com.br>
TO: <ciencialist@yahoogrupos.com.br>
DATE: 20/03/2005 00:39

Caros amigos,

Peguei esta 'super' discussão pelo caminho. Mas percebi que há algumas
referência ao fato de manto ter pego fogo e a técnica de datação por carbono
14 ser 'prejudicada' ou 'inviabilizada'. Pelos meus parcos conhecimentos,
digo que mesmo que o manto tenha queimado, os carbonos radioativos NÃO irão
perder sua atividade radioativa, visto que o fenômeno se dá a nível nuclear.

Au revoir

1 mol de [ ]'s do

Emiliano Chemello
emiliano@quimica.net
http://www.quimica.net/emiliano
http://www.ucs.br/ccet/defq/naeq
[ MSN ] chemelloe@hotmail.com
[ ICQ ] 145060604

"Rien ne se perd, rien ne se crée, tout se transforme"
Lavoisier, químico francês (1743-1794)
----- Original Message -----
From: rmtakata
To: ciencialist@yahoogrupos.com.br
Sent: Sunday, March 20, 2005 12:28 AM
Subject: [ciencialist] Re: Santo sudário e a vanilina(correção)



--- Em ciencialist@yahoogrupos.com.br, "pubmed2005"
> Mas tem um porém, o manto nao pegou fogo, apenas parte dele
> que foram remendadas.

O manto pegou fogo - parte do manto foi destruido. De todo modo as
partes q. nao queimaram estiveram expostas a temperaturas razoaveis
pelo jeito.

> Mas de qualquer forma o fogo alteraria o resultado do
> carbono-14, porque aceleraria a queima das moléculas de
> carbono já existente no tecido

Bem a parte queimada desapareceu e nao se mede essa parte. A parte
restante seria alterada apenas se o fogo tivesse preferencia por C-12,
coisa de q. nao se tem noticia.

[]s,


Roberto Takata





##### ##### #####

Para saber mais visite
http://www.ciencialist.hpg.ig.com.br


##### ##### ##### #####


Yahoo! Grupos, um serviço oferecido por:

São Paulo Rio de Janeiro Curitiba Porto Alegre Belo Horizonte Brasília





Links do Yahoo! Grupos

Para visitar o site do seu grupo na web, acesse:
http://br.groups.yahoo.com/group/ciencialist/

Para sair deste grupo, envie um e-mail para:
ciencialist-unsubscribe@yahoogrupos.com.br

O uso que você faz do Yahoo! Grupos está sujeito aos Termos do Serviço do
Yahoo!.




SUBJECT: Re: Santo sudário e a vanilina(correção)
FROM: "pubmed2005" <pubmed2005@yahoo.com.br>
TO: ciencialist@yahoogrupos.com.br
DATE: 20/03/2005 00:48


--- Em ciencialist@yahoogrupos.com.br, "rmtakata" <rmtakata@a...>
escreveu
>
> --- Em ciencialist@yahoogrupos.com.br, "pubmed2005"
> > Quem fez os calculos foi voce, e isso tb nao inspira muita
> > confiabilidade - j á que voce não apresentou nenhum
> > experimento testável que comprova o que voce diz.
>
> Os calculos foram verificados. Eles se baseiam em formulas obtidas
> experimentalmente - alias, os cientistas usam essas formulas para
> fazer a datacao por radioisotopos.

Nada disso. Voce apresentou calculos de como funciona a datação do
carbono-14. Mas nao com aqueles diferenciais que apresentei. Refaça
aos calculos com os novos diferenciais procados por umidade , fungos,
bactérias, fogo, calor, bioplastia sofrida pelo tecido e aprensente-o
a uma publicação digna


>
> Se acha q. tem um erro nos calculos pode refaze-los e indicar onde
> houve erro.

Voce já pegou calculos prontos baseado em pesquisas do Google. Não
foi voce que criou os calculos. E nao empurre pra mim, refaça os
clculos com os diferenciais e publique-o

>
> Outros cientistas tb fizeram calculos semelhantes aos meus e
chegaram
> aos mesmos resultados.

Repito aqueles calculos foram copiados da internet, ou recriados com
bases em pesquisaas pela mesma sobre o carbono-14. São facilmente
encontráveis, eu mesmo posso te fornecer vários deles pelos sites de
quimica e fisica da internet

>
> Sim, o ideal seria analisar outras partes - embora seja apenas
> suposicao Rogers de q. as amostras datadas por radioisotopos fossem
> retiradas da area de remendo (ou o remendo era tao bom q. os
> especialistas q. retiraram a amostra nao notaram a diferenca ou os
> especialistas eram incompententes o suficiente para retirar amostras
> do remendo... claro nao se pode descartar nenhuma dessas hipoteses).
>
> Uma diferenca de 500 anos nao eh o suficiente para compatibilizar a
> idade do manto com o inicio da era crista.

Lá vem voce com essa mania religiosa de que o manto era de Cristo.
Os dados do Rogers já havia sido detectados em outras pesquisas,
inclusive a de Milão e da Califórnia. Portanto nao foram suposições.





SUBJECT: Re: Santo sudário e a vanilina(correção)
FROM: "rmtakata" <rmtakata@altavista.net>
TO: ciencialist@yahoogrupos.com.br
DATE: 20/03/2005 00:49


--- Em ciencialist@yahoogrupos.com.br, "pubmed2005"
> Colide no sentido de que o manto é mais antigo do que se
> pensava.

Pra falar q, eh mais antigo eh preciso ser um metodo q. determine
datas - ainda q. bem vagamente: tipo, 'mais velho do q. 500 anos'.

Tbm como dito antes: metodo A fala idade xis (uns 500 anos), metodo B
fala idade ipsolon (entre 700 e 2000 anos). Entao eh questao de qual
metodo eh mais confiavel.

Ainda estah pra se provar q. a datacao quimica seja mais confiavel do
q. a datacao por radioisotopos, volto a repetir.

[]s,

Roberto Takata





SUBJECT: Re: [ciencialist] Nova teoria propõe como tornar objetos invisíveis
FROM: "Alberto Mesquita Filho" <albmesq@uol.com.br>
TO: <ciencialist@yahoogrupos.com.br>
DATE: 20/03/2005 00:53

----- Original Message -----
From: "pubmed2005"
Sent: Sunday, March 20, 2005 12:00 AM
Subject: [ciencialist] Nova teoria propõe como tornar objetos invisíveis

> Para fazer o objeto ficar invisível, os cientistas alteraram a forma como
> ele reflete a luz, utilizando plasmons - vibrações dos elétrons que
> acontecem na superfície de alguns metais.

Mas... Que vibrações misteriosas seriam essas? Essa idéia faria sentido
frente à teoria de elétron de Lorentz, ou frente à teoria de átomos em pudim
de passas de Thompsom, dos princípios do século passado. Matematicamente
Planck demonstrou que isso pode ser equacionável, mas... Existem de fato
essas vibrações? Elétrons "livres" vibram? Existem os elétrons livres de
Lorentz nos metais?

Os físicos "modernos" colocaram esses questionamentos debaixo do tapete e
substituiram essas idéias por uma matemática que dá certo. Não obstante,
quando querem se comunicar entre si, ou então com os leigos, e haja vista
que não sabem como e nem porquê essa matemática dá certo, conservam a
terminologia antiga e falam em vibrações de elétrons. Sem dúvida estão
frente a um fenômeno dotado de periodicidade, mas falar em vibrações soa-me
de maneira tão absurda quanto se disséssemos que a Terra vibra segundo um
eixo que passa pelo Sol. Poderíamos, não obstante, construir um modelo
matemático a dar certo. Seria suficientemente projetarmos o movimento da
Terra num dos diâmetros de sua órbita ao redor do Sol, e esta imagem sim,
vibraria segundo um eixo que passa pelo Sol.

> Eles não esperavam ver nada do outro lado mas, para sua surpresa, saía
> mais luz do outro lado do que entrava.
> "Um design adequado pode induzir uma queda dramática na superfície de
> reflexão, tornando o objeto praticamente invisível a um observador,"

Taí um desafio para uma das minhas teorias, assumindo que esse fenômeno seja
mesmo observável. Não acho muito difícil explicar esse fenômeno a partir do
que comento em http://ecientificocultural.com/ECC2/artigos/polar05.htm#V42
(item V.4.2, "Reflexão e refração da luz"). A figura 22 é bastante
sugestiva. Se conseguirmos de alguma maneira rotacionar o que chamei por
agente de reflexão (b), de tal forma que todos assumam a configuração (a)
[agente de propagação], não seria impossível chegarmos ao efeito que
surpreendeu os pesquisadores. Acho apenas que o trecho acima está mal
traduzido e/ou a revelar uma certa ambigüidade (saía mais luz do outro lado
do que entrava), pois do contrário o material estaria agindo como fonte
primária de luz, e não como um simples transmissor. Ou seja, a luz que entra
por um lado sairá pelo outro e vice-versa, e graças a isso o objeto ficaria
invisível.

[ ]´s
Alberto
http://ecientificocultural.com/indice.htm
Mas indiferentemente a tudo isso, o neutrino tem massa, o elétron não é
uma carga elétrica coulombiana e a Terra se move. E a história se repetirá.



SUBJECT: Re: Santo sudário e a vanilina(correção)
FROM: "pubmed2005" <pubmed2005@yahoo.com.br>
TO: ciencialist@yahoogrupos.com.br
DATE: 20/03/2005 00:53


Ué, por experiencias praticas em tecidos arqueológicos que apresentam
vanilina até a faixa da idade média, a partir disso só é possível
especular que tal tecido se nao possuir essa substancia deve ter uma
idade mais avançada

Como eu disse o ideal seria repetir de novo a experiencia do carbono-
14 no tecido novamente nas partes onde nao foram feitas, ou seja
aonde a vnilina nao estava presente. Daí que vejo ncessidade de um
estudo poder ajudar o outro

--- Em ciencialist@yahoogrupos.com.br, "rmtakata" <rmtakata@a...>
escreveu
>
> --- Em ciencialist@yahoogrupos.com.br, "pubmed2005"
> > Colide no sentido de que o manto é mais antigo do que se
> > pensava.
>
> Pra falar q, eh mais antigo eh preciso ser um metodo q. determine
> datas - ainda q. bem vagamente: tipo, 'mais velho do q. 500 anos'.
>
> Tbm como dito antes: metodo A fala idade xis (uns 500 anos), metodo
B
> fala idade ipsolon (entre 700 e 2000 anos). Entao eh questao de qual
> metodo eh mais confiavel.
>
> Ainda estah pra se provar q. a datacao quimica seja mais confiavel
do
> q. a datacao por radioisotopos, volto a repetir.
>
> []s,
>
> Roberto Takata





SUBJECT: Re: Santo sudário e a vanilina(correção)
FROM: "rmtakata" <rmtakata@altavista.net>
TO: ciencialist@yahoogrupos.com.br
DATE: 20/03/2005 00:57


--- Em ciencialist@yahoogrupos.com.br, "pubmed2005"
> Mas a outra parte ficou exposta ao calor e alterou as
> moléculas do carbono presente no tecido. O tecido pode em
> parte ter ficado chamuscado sem queimar. O que altera o
> resultado do carbono-14

Como eu disse, apenas se o fogo tiver preferencia por C-12.

A datacao eh feita pela quantidade *relativa* de C-14.

Pegue um copo e encha de agua. Coloque um pouco de corante na agua.
Misture bem. Se vc jogar a metade da agua colorida fora, a
concentracao da agua restante nao muda. Eh preciso um processo q.
tenha preferencia por moleculas de agua ou de corante - por exemplo,
deixar evaporar (a agua vai, mas o corante fica, a solucao fica mais
concentrada).

O fogo precisaria levar mais C-12 (proporcionalmente) do q. C-14 para
aumentar a concentracao de C-14 e rejuvenescer a amostra.

Os atomos de C-12 e C-14 em si nao sao alterados com o fogo. Se vc nao
sabe, no processo de datacao a amostra eh *queimada* para liberar os
atomos na forma de gas. (Processos nucleares tto qto se sabe nao sao
alterados por temperatura e pressao - por isso o metodo de datacao por
radioisotopos, qdo aplicavel, eh preferido pelos cientistas. Ja'
processos quimicos - q. envolvem a troca de eletrons, particularmente
das camadas mais externas - sao alterados bastante pela temperatura e
pressao.)

[]s,

Roberto Takata





SUBJECT: Re: Santo sudário e a vanilina(correção)
FROM: "pubmed2005" <pubmed2005@yahoo.com.br>
TO: ciencialist@yahoogrupos.com.br
DATE: 20/03/2005 00:59


Tudo bem Emiliano, nao sou especilista no assunto e posso ter falado
bobagem, mas e o revestimento bioplastico da qual se tem resultados
experimentais que comprovaram que há erros na datação em outros
objetos , inclusive palontológicos?

--- Em ciencialist@yahoogrupos.com.br, "Emiliano Chemello - Yahoo
Grupos" <chemelloe@y...> escreveu
> Caros amigos,
>
> Peguei esta 'super' discussão pelo caminho. Mas percebi que há
algumas
> referência ao fato de manto ter pego fogo e a técnica de datação
por carbono
> 14 ser 'prejudicada' ou 'inviabilizada'. Pelos meus parcos
conhecimentos,
> digo que mesmo que o manto tenha queimado, os carbonos radioativos
NÃO irão
> perder sua atividade radioativa, visto que o fenômeno se dá a nível
nuclear.
>
> Au revoir
>
> 1 mol de [ ]'s do
>
> Emiliano Chemello
> emiliano@q...
> http://www.quimica.net/emiliano
> http://www.ucs.br/ccet/defq/naeq
> [ MSN ] chemelloe@h...
> [ ICQ ] 145060604
>
> "Rien ne se perd, rien ne se crée, tout se transforme"
> Lavoisier, químico francês (1743-1794)
> ----- Original Message -----
> From: rmtakata
> To: ciencialist@yahoogrupos.com.br
> Sent: Sunday, March 20, 2005 12:28 AM
> Subject: [ciencialist] Re: Santo sudário e a vanilina(correção)
>
>
>
> --- Em ciencialist@yahoogrupos.com.br, "pubmed2005"
> > Mas tem um porém, o manto nao pegou fogo, apenas parte dele
> > que foram remendadas.
>
> O manto pegou fogo - parte do manto foi destruido. De todo modo as
> partes q. nao queimaram estiveram expostas a temperaturas razoaveis
> pelo jeito.
>
> > Mas de qualquer forma o fogo alteraria o resultado do
> > carbono-14, porque aceleraria a queima das moléculas de
> > carbono já existente no tecido
>
> Bem a parte queimada desapareceu e nao se mede essa parte. A parte
> restante seria alterada apenas se o fogo tivesse preferencia por C-
12,
> coisa de q. nao se tem noticia.
>
> []s,
>
>
> Roberto Takata
>
>
>
>
>
> ##### ##### #####
>
> Para saber mais visite
> http://www.ciencialist.hpg.ig.com.br
>
>
> ##### ##### ##### #####
>
>
> Yahoo! Grupos, um serviço oferecido por:
>
> São Paulo Rio de Janeiro Curitiba Porto Alegre Belo Horizonte
Brasília
>
>
>
>
>
> Links do Yahoo! Grupos
>
> Para visitar o site do seu grupo na web, acesse:
> http://br.groups.yahoo.com/group/ciencialist/
>
> Para sair deste grupo, envie um e-mail para:
> ciencialist-unsubscribe@yahoogrupos.com.br
>
> O uso que você faz do Yahoo! Grupos está sujeito aos Termos do
Serviço do
> Yahoo!.





SUBJECT: Re: Santo sudário e a vanilina(correção)
FROM: "pubmed2005" <pubmed2005@yahoo.com.br>
TO: ciencialist@yahoogrupos.com.br
DATE: 20/03/2005 01:16


Vamos supor que sim, mas quando a coisa é feita direita né? Pois está
comprovado que análise que foi feita foi nas partes remendadas como
foi comprovada pela análise dos fios dos tecidos retirados pelas
pesquisas realizadas em Milão e na Califórnia

Bater o martelo e dizer que tudo ocorreu perfeito e que essa é a data
real do tecido é no mínimo questionável. Datar tecido remendado é
dizer que o manto é da idade média é no mínimo muito questionável

-- Em ciencialist@yahoogrupos.com.br, "rmtakata" <rmtakata@a...>
escreveu
>
> --- Em ciencialist@yahoogrupos.com.br, "pubmed2005"
> > Colide no sentido de que o manto é mais antigo do que se
> > pensava.
>
> Ainda estah pra se provar q. a datacao quimica seja mais confiavel
do
> q. a datacao por radioisotopos, volto a repetir.
>
> []s,
>
> Roberto Takata





SUBJECT: Re: Santo sudário e a vanilina(correção)
FROM: "rmtakata" <rmtakata@altavista.net>
TO: ciencialist@yahoogrupos.com.br
DATE: 20/03/2005 01:27


--- Em ciencialist@yahoogrupos.com.br, "pubmed2005"
> Refaça aos calculos com os novos diferenciais procados por
> umidade , fungos, bactérias, fogo, calor, bioplastia sofrida
> pelo tecido e aprensente-o a uma publicação digna

Se vc ler o artigo em q. fiz os calculos verah q. tais fatores foram
analisados - no caso da fumaca, teria q. ter havido uma troca de quase
80% com a fumaca; no caso de bacterias, seria necessario uma quantida
de bacteria igual ao de tecido - em vez de sudario de Turim, teriamos
q. falar de placa bacteriana de Turim.

Eu nao preciso publicar em uma revista cientifica. Alias, nao devo.
Isso porq. jah foi feito. Se vc nao leu - no artigo em q. apresentei
os calculos ha' referencias bibliograficas - passo alguns trabalhos q.
contestam Kouznetsov.

Hedges R. E. M. & C. B. Ramsey 1998 An experiment to refute the
likelihood of cellulose carboxylation. Radiocarbon 40(1): 59-60.

Hedges, R. E. M. 1989 Shroud irradiated with neutrons? Hedges replies.
Nature 337: 594.

> > Se acha q. tem um erro nos calculos pode refaze-los e
> > indicar onde houve erro.
>
> Voce já pegou calculos prontos baseado em pesquisas do
> Google. Não foi voce que criou os calculos. E nao empurre
> pra mim, refaça os clculos com os diferenciais e publique-o

Q. pouca fe' na minha diligencia. Refaco o convite, se acha q. tem
erro nas contas, pode refaze-los e indicar onde houve o erro.

> Repito aqueles calculos foram copiados da internet, ou
> recriados com bases em pesquisaas pela mesma sobre o
> carbono-14.

Na verdade eu tive o trabalho de fazer. Fiz durante uma viagem de
onibus para matar o tempo: e houve uma extensa discussao nesta mesma
lista (o Taborda participou da critica aos calculos). Mas, sim, ha'
varios calculos do genero publicado por ai' - o q. eh bom, pessoas
chegando independentemente ao mesmo resultado. E mesmo q. eu tivesse
copiado - reitero q. nao o fiz, mas nao estou nem um pouco preocupado
em convence-lo a respeito disso - nao diz nada contra as contas.

> Lá vem voce com essa mania religiosa de que o manto era de
> Cristo.

Ueh, nao sou eu, sao os adoradores do sudario q afirmam isso. E se nao
eh do inicio da era crista, o sudario eh uma fraude.

> Os dados do Rogers já havia sido detectados em outras
> pesquisas, inclusive a de Milão e da Califórnia. Portanto
> nao foram suposições.

Eh suposicao de q. as amostras usadas na datacao sao de areas do
remendo. (Vc estah ciente de q. as *outras* pesquisas a q. vc se
refere *nao* sao independentes?)

http://www.csicop.org/specialarticles/shroud-printable.html

[]s,

Roberto Takata





SUBJECT: Re: Santo sudário e a vanilina(correção)
FROM: "junior_br2001" <kdelinux@usa.com>
TO: ciencialist@yahoogrupos.com.br
DATE: 20/03/2005 01:32


Voce apresentou calculos s/ os experimentos, eu te apresentei
experimentos dos especialistas soviéticos e dos mexicanos , e
portanto voce fica me devendo essa.

OFF_-TOPIC:Ah e sobre a outra lista, acabei sendo expulso
praticamente sem motivo algum, simplesmente por respoder uma mensagem
sua. Essa é lista de céticos que a gente por aí. Mas tudo bem. Nao
vai me fazer falta não.

--- Em ciencialist@yahoogrupos.com.br, "rmtakata" <rmtakata@a...>
escreveu
>
> --- Em ciencialist@yahoogrupos.com.br, "pubmed2005"
> > Refaça aos calculos com os novos diferenciais procados por
> > umidade , fungos, bactérias, fogo, calor, bioplastia sofrida
> > pelo tecido e aprensente-o a uma publicação digna
>
> Se vc ler o artigo em q. fiz os calculos verah q. tais fatores foram
> analisados - no caso da fumaca, teria q. ter havido uma troca de
quase
> 80% com a fumaca; no caso de bacterias, seria necessario uma
quantida
> de bacteria igual ao de tecido - em vez de sudario de Turim,
teriamos
> q. falar de placa bacteriana de Turim.
>
> Eu nao preciso publicar em uma revista cientifica. Alias, nao devo.
> Isso porq. jah foi feito. Se vc nao leu - no artigo em q. apresentei
> os calculos ha' referencias bibliograficas - passo alguns trabalhos
q.
> contestam Kouznetsov.
>
> Hedges R. E. M. & C. B. Ramsey 1998 An experiment to refute the
> likelihood of cellulose carboxylation. Radiocarbon 40(1): 59-60.
>
> Hedges, R. E. M. 1989 Shroud irradiated with neutrons? Hedges
replies.
> Nature 337: 594.
>
> > > Se acha q. tem um erro nos calculos pode refaze-los e
> > > indicar onde houve erro.
> >
> > Voce já pegou calculos prontos baseado em pesquisas do
> > Google. Não foi voce que criou os calculos. E nao empurre
> > pra mim, refaça os clculos com os diferenciais e publique-o
>
> Q. pouca fe' na minha diligencia. Refaco o convite, se acha q. tem
> erro nas contas, pode refaze-los e indicar onde houve o erro.
>
> > Repito aqueles calculos foram copiados da internet, ou
> > recriados com bases em pesquisaas pela mesma sobre o
> > carbono-14.
>
> Na verdade eu tive o trabalho de fazer. Fiz durante uma viagem de
> onibus para matar o tempo: e houve uma extensa discussao nesta mesma
> lista (o Taborda participou da critica aos calculos). Mas, sim, ha'
> varios calculos do genero publicado por ai' - o q. eh bom, pessoas
> chegando independentemente ao mesmo resultado. E mesmo q. eu tivesse
> copiado - reitero q. nao o fiz, mas nao estou nem um pouco
preocupado
> em convence-lo a respeito disso - nao diz nada contra as contas.
>
> > Lá vem voce com essa mania religiosa de que o manto era de
> > Cristo.
>
> Ueh, nao sou eu, sao os adoradores do sudario q afirmam isso. E se
nao
> eh do inicio da era crista, o sudario eh uma fraude.
>
> > Os dados do Rogers já havia sido detectados em outras
> > pesquisas, inclusive a de Milão e da Califórnia. Portanto
> > nao foram suposições.
>
> Eh suposicao de q. as amostras usadas na datacao sao de areas do
> remendo. (Vc estah ciente de q. as *outras* pesquisas a q. vc se
> refere *nao* sao independentes?)
>
> http://www.csicop.org/specialarticles/shroud-printable.html
>
> []s,
>
> Roberto Takata





SUBJECT: Re: Santo sudário e a vanilina(correção)
FROM: "rmtakata" <rmtakata@altavista.net>
TO: ciencialist@yahoogrupos.com.br
DATE: 20/03/2005 01:34


--- Em ciencialist@yahoogrupos.com.br, "pubmed2005"
> Ué, por experiencias praticas em tecidos arqueológicos que
> apresentam vanilina até a faixa da idade média, a partir
> disso só é possível especular que tal tecido se nao possuir
> essa substancia deve ter uma idade mais avançada

Como dito: metodo A -> idade de cerca de 500 (ou 700) anos; metodo B
-> idade maior do q. 1000.

Questao de ver qual metodo eh o mais confiavel.

> Como eu disse o ideal seria repetir de novo a experiencia do
> carbono-14 no tecido novamente nas partes onde nao foram
> feitas, ou seja aonde a vnilina nao estava presente.

Sim, pelo menos neste ponto concordo com vc: o ideal seria repetir de
novo.

[]s,

Roberto Takata





SUBJECT: Re: Santo sudário e a vanilina(correção)
FROM: "rmtakata" <rmtakata@altavista.net>
TO: ciencialist@yahoogrupos.com.br
DATE: 20/03/2005 01:38


--- Em ciencialist@yahoogrupos.com.br, "pubmed2005"
> Pois está comprovado que análise que foi feita foi nas
> partes remendadas como foi comprovada pela análise dos fios
> dos tecidos retirados pelas pesquisas realizadas em Milão e
> na Califórnia

Nao estah comprovado. Rogers defende q a amostra eh de remendo, apenas
isso.

> Bater o martelo e dizer que tudo ocorreu perfeito e que essa
> é a data real do tecido é no mínimo questionável.

Td pode ser questionado. Inclusive o ato de dizer q. a amostra era de
um remendo.

[]s,

Roberto Takata





SUBJECT: Re: Santo sudário e a vanilina(correção)
FROM: "rmtakata" <rmtakata@altavista.net>
TO: ciencialist@yahoogrupos.com.br
DATE: 20/03/2005 01:49


--- Em ciencialist@yahoogrupos.com.br, "junior_br2001"
> Voce apresentou calculos s/ os experimentos, eu te
> apresentei experimentos dos especialistas soviéticos e dos
> mexicanos , e portanto voce fica me devendo essa.

Nao estou devendo nada.

Em primeirissimo lugar: no texto em q. apresento as contas, ha' toda
uma bibliografia (faco questao de apresenta'-la mesmo em um texto q.
nao tem finalidade cientifica). Nessa bibliografia ha' trabalhos q.
contestam as alegacoes feitas sobre a fumaca e o biofilme.

Em segundo lugar: os calculos foram feitos com base em formulas usadas
pelos cientistas - eh como dizer q o engenheiro nao apresentou um
trabalho cientifico mostrando q. os calculos dele estao corretos - e
os dados usados foram dos proprios estudos q. vc defende.

> OFF_-TOPIC:Ah e sobre a outra lista, acabei sendo expulso
> praticamente sem motivo algum, simplesmente por respoder uma
> mensagem sua.

Responder a mensagens minhas falando *reiteradamente* q. o q. eu
escrevo eh babaquice, lixo, baboseira, futilidades e coisas do tipo.
Bem, eu nao expulsaria. Mas eh decisao do moderador. Embora vc tenha o
direito de achar q. isso foi "praticamente sem motivo algum".

[]s,

Roberto Takata





SUBJECT: Re: [ciencialist] Santo Sudario
FROM: Eduardo Gueron <edgueron@yahoo.com>
TO: ciencialist@yahoogrupos.com.br
DATE: 20/03/2005 01:57

Ola,

Sinceramente nco domino ticnica de datagco alguma,
acho, entretanto, que o resultado do qummico
californiano de Los Alamos (antes que falem qq coisa,
nada contra qummicos, bislogos ou aposentado, ati
porque minha mce i bisloga aposentada) i no mmnimo
fraco, uma margem de erro de 2300 anos em um universo
de 3000 i um pouco grande, nco acham? Outra coisa a
ser notada i a quantidade de gente envolvida na
analise anterior. Pelo menos 15 pessoas de 6 centros
de pesquisas diferentes em um artigo publicado na
Nature. NATURE 337 (6208): 611-615 FEB 16 1989 (se
alguim quiser, posso mandar em particular a versco
pdf). O do R. Rogers, publicado em uma revista um
pouco duvidosa, o tem como znico autor.THERMOCHIMICA
ACTA 425 (1-2): 189-194 JAN 20 2005.

Concordo que isso i quase um argumento de autoridade
e, portanto, nco se deve levar muito em conta mas i um
elemento a ser considerado. Podemos interpretar ou que
os resultados da equipe de 89 sco mais confiaveis ou
que ha um complt dos cientistas e editores contra a
autenticidade do Sudario...

Finalmente, acho um pouco estranho nessa discussco
que o tnus da prova esteja cabendo aos citicos. Ora,
se vc afirma fazer um cachorro voar com uma piscadela,
i seu papel provar isso, nco de um citico que duvida
de sua palavra. Nesse caso, ha uma dzvida nos mitodos
do Dr. Rogers e nos argumentos que usou para
"desconstruir" a datagco anterior. Tudo bem, vamos
assumir que todos tjm suas falhas... o que devemos
fazer entco? Simples, optar pelo conservadorismo, que
nessa situagco i a nco autenticidade. Se nco i
interesse da igreja provar a autenticidade do Sudario,
como disse o Taborda, entco, nco ha cijncia a ser
feita, vamos tratar de pesquisar algo de ztil ou, ao
menos, interessante.

Gostaria de observar, ainda que, nessa discussco, esta
clara a religiosidade de cada um. Posso estar
enganado, mas chutaria que todos os que defendem a
autenticidade do Sudario sco cristcos (e acusam os
citicos de dogmaticos). I justo, portanto, que cada um
assuma a sua religico para que a briga fique mais
honesta. Lembro que desde o inmcio, apesar de estar
meio ` margem da briga, disse que sou um citico
(agnsstico e ateu, depende do contexto e do meu
humor).

I isso que tinha para falar.


Abragos, Eduardo.



__________________________________
Do you Yahoo!?
Yahoo! Small Business - Try our new resources site!
http://smallbusiness.yahoo.com/resources/


SUBJECT: Re: Santo sudário e a vanilina(correção)
FROM: "junior_br2001" <kdelinux@usa.com>
TO: ciencialist@yahoogrupos.com.br
DATE: 20/03/2005 02:08


--- Em ciencialist@yahoogrupos.com.br, "rmtakata" <rmtakata@a...>
escreveu
>
> Em segundo lugar: os calculos foram feitos com base em formulas
usadas
> pelos cientistas - eh como dizer q o engenheiro nao apresentou um
> trabalho cientifico mostrando q. os calculos dele estao corretos - e
> os dados usados foram dos proprios estudos q. vc defende.


Sim, mas faltou o experimento(s). Eu tb realizar uma porção de
cálculo. O problema é testar

>
> > OFF_-TOPIC:Ah e sobre a outra lista, acabei sendo expulso
> > praticamente sem motivo algum, simplesmente por respoder uma
> > mensagem sua.
>
> Responder a mensagens minhas falando *reiteradamente* q. o q. eu
> escrevo eh babaquice, lixo, baboseira, futilidades e coisas do tipo.
> Bem, eu nao expulsaria. Mas eh decisao do moderador. Embora vc
tenha o
> direito de achar q. isso foi "praticamente sem motivo algum".


Pior que era tudo futilidades mesmo. Levantar questões sobre a vida
as pessoas,inclusive coisas que elas nao fizereram de fato, é no
mínimo falta de respeito, e ainda ficar responder agressões c/ outras
formas de ofensas e gracinhas, é falta de zelo e respeito com a lista
e c/os partipantes. Não que eu tb nao tenha errado, mas não fui tão
longe assim - o meu erro foi só de responder as piadinhas e os
deboches proferidos. Mas isso pra mim já é passado....
Afinal , eles nunca expulsariam "alguém" que esta do lado do
movimento *cético"....ainda mais alguém tão conhecido de lá...





SUBJECT: Re: Santo sudário e a vanilina(correção)
FROM: "rmtakata" <rmtakata@altavista.net>
TO: ciencialist@yahoogrupos.com.br
DATE: 20/03/2005 02:14


--- Em ciencialist@yahoogrupos.com.br, "junior_br2001"
> Sim, mas faltou o experimento(s). Eu tb realizar uma porção
> de cálculo. O problema é testar

A formula jah foi testada diversas vezes. Estou apenas aplicando a
formula. A menos q. vc tenha algum indicio de q. a formula esteja errada.

E o problema nao eh testar, pois como dito antes, ha' trabalhos
experimentais contestando as alegacoes sobre o biofilme e a fumaca -
eu jah passei algumas referencias em outra mensagem.

> Pior que era tudo futilidades mesmo.

Direito seu 'a opiniao.

[]s,

Roberto Takata





SUBJECT: Re: Santo sudário e a vanilina(correção)
FROM: "junior_br2001" <kdelinux@usa.com>
TO: ciencialist@yahoogrupos.com.br
DATE: 20/03/2005 02:20


--- Em ciencialist@yahoogrupos.com.br, "rmtakata" <rmtakata@a...>
escreveu
>
> --- Em ciencialist@yahoogrupos.com.br, "junior_br2001"
> > Sim, mas faltou o experimento(s). Eu tb realizar uma porção
> > de cálculo. O problema é testar
>
> A formula jah foi testada diversas vezes. Estou apenas aplicando a
> formula. A menos q. vc tenha algum indicio de q. a formula esteja
errada.
>
> E o problema nao eh testar, pois como dito antes, ha' trabalhos
> experimentais contestando as alegacoes sobre o biofilme e a fumaca -
> eu jah passei algumas referencias em outra mensagem.


Pois é, estes trabalhos que estão faltando aqui para serem analisados.
Mas já estou encerrando esta discussão, acho que ela já deu tudo que
tinha que dá, e o mistério a respeito do manto vai permanecer, até o
dia que a Igreja pretenda libera-lo para uma análise mais acurada.






SUBJECT: Re: Santo sudário e a vanilina(correção)
FROM: "rmtakata" <rmtakata@altavista.net>
TO: ciencialist@yahoogrupos.com.br
DATE: 20/03/2005 02:31


--- Em ciencialist@yahoogrupos.com.br, "junior_br2001"
> tinha que dá, e o mistério a respeito do manto vai permanecer,

Misterio pra mim eh saber por q. tem gente q. acredita nessa historia.

[]s,

Roberto Takata





SUBJECT: Re: Santo Sudario
FROM: "pubmed2005" <pubmed2005@yahoo.com.br>
TO: ciencialist@yahoogrupos.com.br
DATE: 20/03/2005 02:32


Não. É até o contrário, eu tenho mais apreço pela filosofia
oriental, da harmonização interior com o universo. E no máximo posso
ser considerado um panteísta intelectual nos moldes de Spinoza sem
conotações religiosas, e no entanto nao-seguidor de nenhuma filosofia
e religiao.

Mas gosto de história, e sou fascinado pela *peça* manto sudário,
pouco me importanto se ela pertenceu a Cristo ou nao, e mesmo que
seja uma falsificação medieval é algo que me fascina, não pela
religiosidade, mas pela historicidade da mesma.

Aliás outros artefatos, tecidos, moedas antigas, vasos, etc. me
encantam de igual modo

Jr(Ex-Pubmed)

--- Em ciencialist@yahoogrupos.com.br, Eduardo Gueron <edgueron@y...>
escreveu
> Gostaria de observar, ainda que, nessa discussão, está
> clara a religiosidade de cada um. Posso estar
> enganado, mas chutaria que todos os que defendem a
> autenticidade do Sudário são cristãos (e acusam os
> céticos de dogmáticos). É justo, portanto, que cada um
> assuma a sua religião para que a briga fique mais
> honesta. Lembro que desde o início, apesar de estar
> meio à margem da briga, disse que sou um cético
> (agnóstico e ateu, depende do contexto e do meu
> humor).
>
> É isso que tinha para falar.
>
>
> Abraços, Eduardo.
>
>
>
> __________________________________
> Do you Yahoo!?
> Yahoo! Small Business - Try our new resources site!
> http://smallbusiness.yahoo.com/resources/





SUBJECT: Re: Santo sudário e a vanilina(correção)
FROM: "junior_br2001" <junior_br2001@yahoo.com.br>
TO: ciencialist@yahoogrupos.com.br
DATE: 20/03/2005 02:49


Mas o manto é de fato um mistério, nao pela suposta religiosidade,
mas porque ele é guardado a sete chaves, longe do alcance da ciencia
para uma investigação mais sólida. É uma peça praticamente intocável,
diferente dos outros artefatos arqueológicos e históricos que estão a
disposição dos cientistas a todo momento.

É e foi somente neste sentido que foi empregado a palavra mistério.

--- Em ciencialist@yahoogrupos.com.br, "rmtakata" <rmtakata@a...>
escreveu
>
> --- Em ciencialist@yahoogrupos.com.br, "junior_br2001"
> > tinha que dá, e o mistério a respeito do manto vai permanecer,
>
> Misterio pra mim eh saber por q. tem gente q. acredita nessa
historia.
>
> []s,
>
> Roberto Takata





SUBJECT: Re: [ciencialist] Santo Sudario
FROM: "Alberto Mesquita Filho" <albmesq@uol.com.br>
TO: <ciencialist@yahoogrupos.com.br>
DATE: 20/03/2005 03:17

----- Original Message -----
From: "Eduardo Gueron"
Sent: Sunday, March 20, 2005 1:57 AM
Subject: Re: [ciencialist] Santo Sudario

> Gostaria de observar, ainda que, nessa discussão, está clara a
> religiosidade de cada um. Posso estar enganado, mas chutaria que todos os
> que defendem a autenticidade do Sudário são cristãos (e acusam os céticos
> de dogmáticos). É justo, portanto, que cada um assuma a sua religião para
> que a briga fique mais honesta. Lembro que desde o início, apesar de estar
> meio à margem da briga, disse que sou um cético (agnóstico e ateu, depende
> do contexto e do meu humor).

Caro Eduardo

Sinceramente não sei dizer se a coisa seria tão simples assim e por vários
aspectos. Em primeiro lugar, diria que esse "todos" refere-se a uma
amostragem bastante pobre. Ou muito me engano, ou poderia contar 2 de cada
lado. Por um lado, vejo o Júnior e o Taborda, cada um com idéias totalmente
diversas e a abordarem prismas totalmente diversos e nem sei até que ponto
eles entrariam em acordo sobre o que escreveram. De outro, o Homero e o
Takata, esses sim a comutarem freqüências próximas, para usar uma linguagem
metafórica, e a traduzirem o que aprenderam naquele catecismo que pulula
pela Internet, aquele mesmo que faz considerações sobre os "ad hominem",
"onus da prova" e outras tacanhices nem sei repetir. E digo tacanhice não
por que seja contrário aos conceitos que estão embutidos nessas palavras,
mas para me referir aos contextos em que elas têm sido utilizadas.

Eu não entrei na briga a não ser para tirar o sarro, ora do Homero, ora do
Takata, e mais no sentido de demonstrar que ser cético, pelo menos em
ciência, não significa "levantar uma bandeira", mas principalmente adotar
uma postura que nos permita levar à produção de novos conhecimentos. No
mais, que se deixe o ceticismo aos filófosos pois eles dão de 1000 a zero em
qualquer desses "céticos de carteirinha" que outra coisa não fazem a não ser
iludir os jovens e denegrir um ceticismo sadio.

Se lhe interessa saber, participo de um grupo sério de discussão de
ceticismo entre filósofos, e neste grupo o único com pendores voltados
exclusivamente à ciência sou eu. Via de regra durante essas reuniões
permaneço calado e assumo a minha insignificância associada à vontade de
aprender. Após as reuniões costumamos ir a uma pizzaria e aí sim animo-me a
dizer alguma coisa a esses filósosos (alguns expoentes nacionais e com
currículo Lates a ser invejado pelo mais culto dentre os freqüentadores da
Ciencialist, e que certamente não sou eu) e chego mesmo a brincar com eles,
referindo-me a essa nossa segunda reunião, regada a chope e coca-cola
(alguns não bebem), como destinada a que possamos praticar uma filosofia de
botequim.

Não sei até que ponto a religiosidade entraria em jogo, ainda que esses
falsos céticos insistam em misturar as coisas. De qualquer forma, e se você
insistir em saber a quantas anda o meu agnosticismo, pois é assim que me
considero, sugiro que acompanhe um debate que travei numa lista já extinta,
mas que pode ser lido, sob o título "Ciência, materialismo e
espiritualismo", a partir de
http://ecientificocultural.com/ECC2/Dialogos/cmee.htm

[ ]´s
Alberto
http://ecientificocultural.com/indice.htm
Mas indiferentemente a tudo isso, o neutrino tem massa, o elétron não é
uma carga elétrica coulombiana e a Terra se move. E a história se repetirá.



SUBJECT: Conseqüências práticas do ceticismo filosófico
FROM: Manuel Bulcão <manuelbulcao@uol.com.br>
TO: ciencialist@yahoogrupos.com.br
DATE: 20/03/2005 07:44


CONSEQUÊNCAS PRÁTICAS DO CETICISMO FILOSÓFICO


"não devemos temer a dúvida, mas antes devemos acolhê-la como a
possibilidade de um novo potencial para os seres humanos."
(Richard Feynman)


Não é fácil conviver com a dúvida. Se tenho dúvidas, sou obrigado a
escolher, a exercitar a minha liberdade, a me responsabilizar pela
escolha feita e pelos meus atos, a admitir que "posso estar errado".
A dúvida é a sombra da liberdade e tem por corolário a angústia e a
responsabilidade moral.

Penso que a certeza, "a crença na precisão absoluta da minha
crença", é o mais velho antídoto contra a ansiedade que sempre
acompanha o ato da escolha. Pois, se não tenho dúvidas acerca do
caminho a trilhar, não estou escolhendo, não estou a agir
livremente, mas tão-somente obedecendo a um imperativo, que pode ser
a palavra de Deus, o Destino ou mesmo a Razão. A certeza absoluta
tranqüiliza a consciência, nos faz agir sem remorsos e sem ela os
genocidas em nome de Deus ou da Razão não dormiriam em paz. É porque
temem a liberdade, é por não quererem assumir responsabilidade moral
que os homens costumam se deixar levar por certezas. Se um dia
descobrem que o que motivou a sua ação era um erro, amaldiçoam Deus,
o Destino ou a Razão, lamentam o fato de terem sido "enganados"
(colocam-se sempre numa situação passiva) e tranqüilos permanecem
com suas outras certezas.

Ao contrário do que muitos pensam, o cético não é um frívolo
diletante que tem por hobby questionar tudo e que faz da filosofia
uma disputa entre egos, um joguinho de salão. Ao contrário, há mesmo
um propósito moral no ceticismo filosófico. Ao fazer da incerteza um
dos princípios do pensamento humano, o que o filósofo cético também
pretende é destruir esse escudo da covardia, essa negação da
liberdade, essa arma da intolerância, esse fundamento de todas as
tiranias, isso que é a categoria central de todos os sistemas
filosóficos fechados imunes à discussão e ao questionamento e que
apenas servem de ideologia para os inimigos da sociedade aberta: o
mito da certeza absoluta.

Penso que nós, céticos, tivemos sucesso em nossa empreitada. Sim, ao
menos no front filosófico, destruímos o encouraçado do dogmatismo.
Deste só resta destroços e alguns náufragos que, para não se
afogarem, fazem das tautologias — como, por exemplo, "o existente
existe" — e do cogito cartesiano sua tábua de salvação.

Claro, existem muitos não-céticos que estão absolutamente certos
das "virtudes" da sociedade aberta. Estes costumam criticar o cético
pelo fato de ele não saber com certeza se assassinar uma criança
indefesa é moralmente justificável ou não, ou porque "diz não saber
com certeza que uma ditadura sanguinolenta é defensável ou não. Não
percebem eles que o que ameaça a sociedade aberta não é a incerteza
quanto a verdade dos seus princípios, mas a certeza compartilhada
por muitos, inclusive por supostos paladinos da democracia (Nixon,
Kissinger, Sharon, Bush, etc.) de que assassinar crianças é muitas
vezes moralmente justificável; essa mesma certeza ou convicção
inabalável tão característica dos que se arvoram os guardiões da
Razão, do Pensamento Correto ou da Verdade e que, por isso, não
estão muito dispostos a perder tempo com discussão, com a busca do
consenso mediante o diálogo e outras "masturbações" democráticas,
motivo pelo qual chegam mesmo a defender ditaduras sanguinolentas
sempre que lhes convém. Como, por exemplo, o "racionalista"
e "libertário" de direita Milton Friedman, que certa vez teceu
elogios rasgados a Pinochet e justificou sua tirania.

É investindo contra o mito da certeza absoluta ou reduzindo-a ao
ponto de jamais se transformar em instrumento de dominação política
que se garante "quase certamente" a sociedade aberta.

Ao admitir que "posso estar errado e você certo", não estou senão
aceitando que: a) é razoável — ou seja, é uma atitude racional — que
eu considere os seus argumentos; por conseguinte, b) não é razoável —
isto é, não é uma atitude racionalmente justificável — que eu venha
a eliminá-lo, a não ser, obviamente, que você queira e tente me
eliminar antes. O caráter duvidoso tanto da minha quanto da sua
crença é o que nos torna iguais apesar de sermos diferentes. A
incerteza é, também, a justificação racional da minha e da sua
liberdade de questionar. Ora, o reconhecimento do outro (do
diferente), a igualdade entre os interlocutores e a liberdade de
questionar são tanto os princípios da razão quanto da democracia.

E nem é preciso ter certeza absoluta da verdade desses princípios
para defendê-los, pois todos eles têm por pressuposto o princípio
mais fundamental de que "tudo é incerto, inclusive esta afirmação".


Manuel Bulcão
17/02/2002





SUBJECT: Re: [ciencialist] Re: Santo sudário e a vanilina(correção)
FROM: "Sergio M. M. Taborda" <sergiotaborda@terra.com.br>
TO: ciencialist@yahoogrupos.com.br
DATE: 20/03/2005 08:49

Emiliano Chemello - Yahoo Grupos wrote:

> Caros amigos,
>
> Peguei esta 'super' discussão pelo caminho. Mas percebi que há algumas
> referência ao fato de manto ter pego fogo e a técnica de datação por
> carbono
> 14 ser 'prejudicada' ou 'inviabilizada'. Pelos meus parcos conhecimentos,
> digo que mesmo que o manto tenha queimado, os carbonos radioativos NÃO
> irão
> perder sua atividade radioativa, visto que o fenômeno se dá a nível
> nuclear.
>
Pois não irão. Mas o problema não é esse. O problema, ao contrário do
que se pensa, não a datação por caborno, mas a origem do carbono datado.
O problema é que o fogo e outros factores contaminam o tecido ao longo
do anos com carbono "mais recente" do que aquele presente na época
verdadeira do tecido..
Portanto, os carbonos presentes no tecido têm "várias datas" e isso
contamina o resultado final.
Mas isso não é descontado no exame. Nem isso nem outras coisas. O exame
de 88 foi demasiado simplista e portanto seus resultados
são inconclusivos.
O exame do carbono 14 não é milagroso. Existem outros factores, outros
exames, quimicos, biologicos e historicos que têm que se levados em
consideração para a datação do objecto.
O pior cientista é aquele que só vê os dados que lhe interessam.

Sérgio Taborda


SUBJECT: Re: [ciencialist] Santo Sudario
FROM: "Sergio M. M. Taborda" <sergiotaborda@terra.com.br>
TO: ciencialist@yahoogrupos.com.br
DATE: 20/03/2005 09:16

Eduardo Gueron wrote:

>Olá,
>
>Sinceramente não domino técnica de datação alguma,
>acho, entretanto, que o resultado do químico
>californiano de Los Alamos (antes que falem qq coisa,
>nada contra químicos, biólogos ou aposentado, até
>porque minha mãe é bióloga aposentada) é no mínimo
>fraco, uma margem de erro de 2300 anos em um universo
>de 3000 é um pouco grande, não acham? Outra coisa a
>ser notada é a quantidade de gente envolvida na
>análise anterior. Pelo menos 15 pessoas de 6 centros
>de pesquisas diferentes em um artigo publicado na
>Nature. NATURE 337 (6208): 611-615 FEB 16 1989 (se
>alguém quiser, posso mandar em particular a versão
>pdf). O do R. Rogers, publicado em uma revista um
>pouco duvidosa, o tem como único autor.THERMOCHIMICA
>ACTA 425 (1-2): 189-194 JAN 20 2005.
>
> Concordo que isso é quase um argumento de autoridade
>e, portanto, não se deve levar muito em conta mas é um
>elemento a ser considerado.
>
Não é quase, é mesmo , um argumento de autoridade.
Repare que a Nature, sendo uma revista "cientifica" possuiu entre seus
quadros muito mais cepticos que pessoas normais, já que para ser cetico
não é preciso ser cientista.
Por isso, não admira que uma trabalho contrariando o de 89 não tenha
aparecido na Nature. E vários foram feitos, e muitos por pessoas tão
creditadas como os que fizeram o de 89.
A questão aqui é que os ceticos querem por força que acreditemos que o
teste do C14 de 89 é à prova de bala, e que nada nem ninguem pode ir
contra os resultados.
Ora, isso é pedir de mais e revela até uma certa ingenuidade cientifica.
Nenhum teste é À prova de bala. Ainda para mais quando o teste foi feito
sob condições de stress, quando a tecnica foi inventada no momento e
quando outros testes posteriores relutam em comprovar esse.
Ao ocntrário do que os ceticos pensam, a conprovação de um teste não se
faz repetindo o teste. Faz-se tentando obter o mesmo resultados por
métodos diferentes. Por isso que as equipas de testes trabalham em
separado e apenas os resultados são mostrados. Quando eles usam o mesmo
procedimento o que está em teste é o proprio procedimento.
O teste do C14 não é suficiente contra outros tipos de testes que dão
resultados destintos.

>Podemos interpretar ou que
>os resultados da equipe de 89 são mais confiáveis ou
>que há um complô dos cientistas e editores contra a
>autenticidade do Sudário...
>
>
PQ os de 89 são mais confiáveis? Vamos lá a ver se alguem explica isso
direitinho.
O facto de ser publicado na nature não os torna mais confiáveis.

>Finalmente, acho um pouco estranho nessa discussão
>que o ônus da prova esteja cabendo aos céticos.
>
Claro. Pq neste caso os cepticos estão dizendo que o unico teste
confiável é o de 88.
Mas não demonstram pq é o mais confiável. Nem o que "confiável" significa.
Com tantos testes ,de diversos tipos, que foram feitos, pq temos que
aceitar o mais antigo deles ,
quando para mais nem sequer conincide com os outros ?

>Ora,
>se vc afirma fazer um cachorro voar com uma piscadela,
>é seu papel provar isso, não de um cético que duvida
>de sua palavra.
>
Exacto. Mas o problema é que são os ceticos no papel de quem afirma que
o cachorro voa.
Afirmam que o teste de 88 é mais confiável. Cadê as provas de que ele é
mais confiável ?
A afirmação de que ele é mass confiável é dos ceticos, não das pessoas
normais, que acham que todos os testes têm igual peso.

>Nesse caso, há uma dúvida nos métodos
>do Dr. Rogers e nos argumentos que usou para
>"desconstruir" a datação anterior. Tudo bem, vamos
>assumir que todos têm suas falhas... o que devemos
>fazer então?
>
Não escolher à priopri sem ter todos os dados em maos.

> Simples, optar pelo conservadorismo
>
Não. Isso não é uma actitude cientifica!
Vc pode fazer isso pessoalmente, em casa , ou com os amigos. Mas não é
admissivel que defenda essa psotura no lista de ciencia.
A natureza da ciencia não é conservadora, é dinamica.

>, que
>nessa situação é a não autenticidade. Se não é
>interesse da igreja provar a autenticidade do Sudário,
>como disse o Taborda, então, não há ciência a ser
>feita(...)
>
Ora, que palermice. Então não temos o direto de saber a verdade ?
Só pq a santa madre igreja não deseja que invertiguemos o pano, não
ficamos parados ?
O que é isto, é a inquisião disfarçada ?
Os cientistas vão continuar a estudar o pano, atté que existam certezas.
A posse do pano não pode ser atribuida a Jesus, pois não temos nenhum
dado dele que possa ser testado (como DNA ou impressões digitais, ou
representações da época)
O mais que podemos fazer é provar que pertenceu a uma pessoa , algures
no hoje israel, em algum tempo entre o ano X e Y.
Provado isso, teremos feito a obrigação cientifica. Tudo o resto é
inferencia e religião.

>
>Gostaria de observar, ainda que, nessa discussão, está
>clara a religiosidade de cada um.
>
Se vc acha. ... Mas tb, um cetico acha o que for preciso para suster o
seu insustentável argumento .
E isso cheira-me a ataque moral ...
Toda a gente - exceto os ceticos, claro - tem interesse no manto.
Para já é uma forma de testar e desenvolver tecnicas de datação
importantes em arquilogia.
Por outro lado se for verdadeiro , tem valor historico como pova que os
corpos dos mortos eram envolvidos , tal como sabemos documentalmente.
Poderemos não só saber que eram, mas que como eram.
Se for falso, teremos uma importante peça para provar que o ser humano é
capaz de tudo para manter o controle sobre as massas,
sobre os grupos e sobre si mesmo.
Portanto é tão interessante para crentes em Jesus, como para descrentes
e agnosticos pois o seu valor é muito mais que religioso, é historico e
sociologico tb.

>Posso estar
>enganado, mas chutaria que todos os que defendem a
>autenticidade do Sudário são cristãos (e acusam os
>céticos de dogmáticos). É justo, portanto, que cada um
>assuma a sua religião para que a briga fique mais
>honesta.
>
Não. Não é justo. A ciencia é imparcial. E não queremos aqui ataques ad
hominem, ou ataques morais.
Se tem algo a dizer , diga-o e demonstre seu ponto. Se não é capaz ,
fique calado.
As experiencias cientificas valem pelo que valem e não por quem as fez,
da mesma forma as argumentações.

>Lembro que desde o início, apesar de estar
>meio à margem da briga, disse que sou um cético
>(agnóstico e ateu, depende do contexto e do meu
>humor).
>
>
Toda a gente sabe que os ceticos querem é briga. É ficar martelano
inutilidades em discussões futeis, sem aportar nenhuma ideia concreta ou
que faça sentido.
Mas entrar nesse jogo é perder tempo precioso pq os ceticos não tão
preocupados em obter coisa alguma, só perder tempo.

Sérgio Taborda


SUBJECT: Re: [ciencialist] Conseqüências práticas do ceticismo filosófico
FROM: "Alberto Mesquita Filho" <albmesq@uol.com.br>
TO: <ciencialist@yahoogrupos.com.br>
DATE: 20/03/2005 11:03

----- Original Message -----
From: "Manuel Bulcão"
Sent: Sunday, March 20, 2005 7:44 AM
Subject: [ciencialist] Conseqüências práticas do ceticismo filosófico

> CONSEQUÊNCAS PRÁTICAS DO CETICISMO FILOSÓFICO

Olá Manuel Bulcão

Texto muito bom. Não, mais que isso. Eu diria excelente. Estou mesmo
pensando em indicar a leitura no meu site, e nem preciso reproduzi-lo, pois
a Ciencialist é uma lista aberta e posso utilizar o link
http://br.groups.yahoo.com/group/ciencialist/message/45109 . Não estranhe
pois se este link for por mim utilizado nos próximos dias em alguma de
minhas páginas web.

Apesar de tudo --e não obstante ter sido atacado por um sentimento de
invídia, ao ler algo que gostaria de ter escrito-- durante a leitura fui
acometido por uma série de dúvidas dentre as quais destaco a seguinte, e que
já me persegue há vários anos: Quem é mais cético no sentido filosófico do
termo? 1) Aqueles que têm vindo aqui na Ciencialist para desfraldar uma
suposta bandeira do ceticismo, como se "estivessem tão-somente obedecendo um
imperativo, que poderia ser a palavra de Deus, o Destino ou mesmo a Razão";
ou 2) aqueles que por ignorância do que seja um ceticismo filosófico e
sadio --ou então, quiçá, por se sentirem expropriados de um título que a
rigor seria deles, mas que está sendo utilizado por pessoas que agem
exatamente ao contrário do que pregam, seja por ignorância, seja por má fé--
vêm aqui na Ciencialist combater o falso ceticismo que pulula na Internet,
deixando-se passar por anticéticos.

Sinceramente não sei e enquanto isso continuarei lançando bordoadas, ora
nestes, ora naqueles, mais nos primeiros do que nos segundos, posto que
entre a hipocrisia e a ignorância não há muito o que escolher. Enquanto isso
continuarei no aguardo de suas intervenções, sempre oportunas e no tamanho
apropriado. Parabens pelo texto.

[ ]´s
Alberto
http://ecientificocultural.com/indice.htm
Mas indiferentemente a tudo isso, o neutrino tem massa, o elétron não é
uma carga elétrica coulombiana e a Terra se move. E a história se repetirá.



SUBJECT: Re: Conseqüências práticas do ceticismo filosófico
FROM: Manuel Bulcão <manuelbulcao@uol.com.br>
TO: ciencialist@yahoogrupos.com.br
DATE: 20/03/2005 11:52


--- Em ciencialist@yahoogrupos.com.br, "Alberto Mesquita Filho"
<albmesq@u...> escreveu
> Texto muito bom. Não, mais que isso. Eu diria excelente. Estou
mesmo pensando em indicar a leitura no meu site, e nem preciso
reproduzi-lo, pois a Ciencialist é uma lista aberta e posso utilizar
o link http://br.groups.yahoo.com/group/ciencialist/message/45109 .
Não estranhe pois se este link for por mim utilizado nos próximos
dias em alguma de minhas páginas web.

Salve, Mesquita!

Manuel: Suas críticas não são vãs. Como já disse alhures, antes de
conhecê-las eu apostava 1000 dólares na "verdade" da física
quântica. Hoje, depois de escarafunchar o seu site, reduzi minha
aposta a apenas $300,00. :-)

Pode usar o meu texto como bem lhe aprouver, é de domínio público.

Grato pelos elogios.

Um abraço,
Manuel Bulcão





SUBJECT: Re: Santo sudário e a vanilina(correção)
FROM: "junior_br2001" <junior_br2001@yahoo.com.br>
TO: ciencialist@yahoogrupos.com.br
DATE: 20/03/2005 11:57


Bem, então eu estava certo, em partes no que eu disse.

Jr


--- Em ciencialist@yahoogrupos.com.br, "Sergio M. M. Taborda"
<sergiotaborda@t...> escreveu
> Emiliano Chemello - Yahoo Grupos wrote:
>
> > Caros amigos,
> >
> > Peguei esta 'super' discussão pelo caminho. Mas percebi que
há algumas
> > referência ao fato de manto ter pego fogo e a técnica de datação
por
> > carbono
> > 14 ser 'prejudicada' ou 'inviabilizada'. Pelos meus parcos
conhecimentos,
> > digo que mesmo que o manto tenha queimado, os carbonos
radioativos NÃO
> > irão
> > perder sua atividade radioativa, visto que o fenômeno se dá a
nível
> > nuclear.
> >
> Pois não irão. Mas o problema não é esse. O problema, ao contrário
do
> que se pensa, não a datação por caborno, mas a origem do carbono
datado.
> O problema é que o fogo e outros factores contaminam o tecido ao
longo
> do anos com carbono "mais recente" do que aquele presente na época
> verdadeira do tecido..
> Portanto, os carbonos presentes no tecido têm "várias datas" e isso
> contamina o resultado final.
> Mas isso não é descontado no exame. Nem isso nem outras coisas. O
exame
> de 88 foi demasiado simplista e portanto seus resultados
> são inconclusivos.
> O exame do carbono 14 não é milagroso. Existem outros factores,
outros
> exames, quimicos, biologicos e historicos que têm que se levados em
> consideração para a datação do objecto.
> O pior cientista é aquele que só vê os dados que lhe interessam.
>
> Sérgio Taborda





SUBJECT: Re: Santo Sudario
FROM: "junior_br2001" <junior_br2001@yahoo.com.br>
TO: ciencialist@yahoogrupos.com.br
DATE: 20/03/2005 12:02


--- Em ciencialist@yahoogrupos.com.br, "Sergio M. M. Taborda"
<sergiotaborda@t...> escreveu
> Eduardo Gueron wrote:

Taborda disse: A ciencia é imparcial.

Essa é uma grande verdade. Para a ciencia nao importa o *ceticismo* ,
nem a *religiosidade*







SUBJECT: Re: Santo Sudario
FROM: "junior_br2001" <junior_br2001@yahoo.com.br>
TO: ciencialist@yahoogrupos.com.br
DATE: 20/03/2005 12:11


Alberto,

Tem um ditado que criei agora que diz: " A dúvida é uma excelente
arma da ciência, mas, sem *acreditar* a ciencia nao chega a lugar
algum"

Jr(filósofo de botequim)


--- Em ciencialist@yahoogrupos.com.br, "Alberto Mesquita Filho"
<albmesq@u...> escreveu
> ----- Original Message -----
> From: "Eduardo Gueron"
> Sent: Sunday, March 20, 2005 1:57 AM
> Subject: Re: [ciencialist] Santo Sudario
>
> > Gostaria de observar, ainda que, nessa discussão, está clara a
> > religiosidade de cada um. Posso estar enganado, mas chutaria que
todos os
> > que defendem a autenticidade do Sudário são cristãos (e acusam os
céticos
> > de dogmáticos). É justo, portanto, que cada um assuma a sua
religião para
> > que a briga fique mais honesta. Lembro que desde o início, apesar
de estar
> > meio à margem da briga, disse que sou um cético (agnóstico e
ateu, depende
> > do contexto e do meu humor).
>
> Caro Eduardo
>
> Sinceramente não sei dizer se a coisa seria tão simples assim e por
vários
> aspectos. Em primeiro lugar, diria que esse "todos" refere-se a uma
> amostragem bastante pobre. Ou muito me engano, ou poderia contar 2
de cada
> lado. Por um lado, vejo o Júnior e o Taborda, cada um com idéias
totalmente
> diversas e a abordarem prismas totalmente diversos e nem sei até
que ponto
> eles entrariam em acordo sobre o que escreveram. De outro, o Homero
e o
> Takata, esses sim a comutarem freqüências próximas, para usar uma
linguagem
> metafórica, e a traduzirem o que aprenderam naquele catecismo que
pulula
> pela Internet, aquele mesmo que faz considerações sobre os "ad
hominem",
> "onus da prova" e outras tacanhices nem sei repetir. E digo
tacanhice não
> por que seja contrário aos conceitos que estão embutidos nessas
palavras,
> mas para me referir aos contextos em que elas têm sido utilizadas.
>
> Eu não entrei na briga a não ser para tirar o sarro, ora do Homero,
ora do
> Takata, e mais no sentido de demonstrar que ser cético, pelo menos
em
> ciência, não significa "levantar uma bandeira", mas principalmente
adotar
> uma postura que nos permita levar à produção de novos
conhecimentos. No
> mais, que se deixe o ceticismo aos filófosos pois eles dão de 1000
a zero em
> qualquer desses "céticos de carteirinha" que outra coisa não fazem
a não ser
> iludir os jovens e denegrir um ceticismo sadio.
>
> Se lhe interessa saber, participo de um grupo sério de discussão de
> ceticismo entre filósofos, e neste grupo o único com pendores
voltados
> exclusivamente à ciência sou eu. Via de regra durante essas reuniões
> permaneço calado e assumo a minha insignificância associada à
vontade de
> aprender. Após as reuniões costumamos ir a uma pizzaria e aí sim
animo-me a
> dizer alguma coisa a esses filósosos (alguns expoentes nacionais e
com
> currículo Lates a ser invejado pelo mais culto dentre os
freqüentadores da
> Ciencialist, e que certamente não sou eu) e chego mesmo a brincar
com eles,
> referindo-me a essa nossa segunda reunião, regada a chope e coca-
cola
> (alguns não bebem), como destinada a que possamos praticar uma
filosofia de
> botequim.
>
> Não sei até que ponto a religiosidade entraria em jogo, ainda que
esses
> falsos céticos insistam em misturar as coisas. De qualquer forma, e
se você
> insistir em saber a quantas anda o meu agnosticismo, pois é assim
que me
> considero, sugiro que acompanhe um debate que travei numa lista já
extinta,
> mas que pode ser lido, sob o título "Ciência, materialismo e
> espiritualismo", a partir de
> http://ecientificocultural.com/ECC2/Dialogos/cmee.htm
>
> [ ]´s
> Alberto
> http://ecientificocultural.com/indice.htm
> Mas indiferentemente a tudo isso, o neutrino tem massa, o elétron
não é
> uma carga elétrica coulombiana e a Terra se move. E a história se
repetirá.





SUBJECT: Re: Conseqüências práticas do ceticismo filosófico
FROM: "junior_br2001" <junior_br2001@yahoo.com.br>
TO: ciencialist@yahoogrupos.com.br
DATE: 20/03/2005 12:14


Excelente texto Bulcão

--- Em ciencialist@yahoogrupos.com.br, Manuel Bulcão
<manuelbulcao@u...> escreveu
>
> CONSEQUÊNCAS PRÁTICAS DO CETICISMO FILOSÓFICO
>
>
> "não devemos temer a dúvida, mas antes devemos acolhê-la como a
> possibilidade de um novo potencial para os seres humanos."
> (Richard Feynman)
>
>
> Não é fácil conviver com a dúvida. Se tenho dúvidas, sou obrigado a
> escolher, a exercitar a minha liberdade, a me responsabilizar pela
> escolha feita e pelos meus atos, a admitir que "posso estar errado".
> A dúvida é a sombra da liberdade e tem por corolário a angústia e a
> responsabilidade moral.
>
> Penso que a certeza, "a crença na precisão absoluta da minha
> crença", é o mais velho antídoto contra a ansiedade que sempre
> acompanha o ato da escolha. Pois, se não tenho dúvidas acerca do
> caminho a trilhar, não estou escolhendo, não estou a agir
> livremente, mas tão-somente obedecendo a um imperativo, que pode
ser
> a palavra de Deus, o Destino ou mesmo a Razão. A certeza absoluta
> tranqüiliza a consciência, nos faz agir sem remorsos e sem ela os
> genocidas em nome de Deus ou da Razão não dormiriam em paz. É
porque
> temem a liberdade, é por não quererem assumir responsabilidade
moral
> que os homens costumam se deixar levar por certezas. Se um dia
> descobrem que o que motivou a sua ação era um erro, amaldiçoam
Deus,
> o Destino ou a Razão, lamentam o fato de terem sido "enganados"
> (colocam-se sempre numa situação passiva) e tranqüilos permanecem
> com suas outras certezas.
>
> Ao contrário do que muitos pensam, o cético não é um frívolo
> diletante que tem por hobby questionar tudo e que faz da filosofia
> uma disputa entre egos, um joguinho de salão. Ao contrário, há
mesmo
> um propósito moral no ceticismo filosófico. Ao fazer da incerteza
um
> dos princípios do pensamento humano, o que o filósofo cético também
> pretende é destruir esse escudo da covardia, essa negação da
> liberdade, essa arma da intolerância, esse fundamento de todas as
> tiranias, isso que é a categoria central de todos os sistemas
> filosóficos fechados imunes à discussão e ao questionamento e que
> apenas servem de ideologia para os inimigos da sociedade aberta: o
> mito da certeza absoluta.
>
> Penso que nós, céticos, tivemos sucesso em nossa empreitada. Sim,
ao
> menos no front filosófico, destruímos o encouraçado do dogmatismo.
> Deste só resta destroços e alguns náufragos que, para não se
> afogarem, fazem das tautologias &#8212; como, por exemplo, "o existente
> existe" &#8212; e do cogito cartesiano sua tábua de salvação.
>
> Claro, existem muitos não-céticos que estão absolutamente certos
> das "virtudes" da sociedade aberta. Estes costumam criticar o cético
> pelo fato de ele não saber com certeza se assassinar uma criança
> indefesa é moralmente justificável ou não, ou porque "diz não saber
> com certeza que uma ditadura sanguinolenta é defensável ou não. Não
> percebem eles que o que ameaça a sociedade aberta não é a incerteza
> quanto a verdade dos seus princípios, mas a certeza compartilhada
> por muitos, inclusive por supostos paladinos da democracia (Nixon,
> Kissinger, Sharon, Bush, etc.) de que assassinar crianças é muitas
> vezes moralmente justificável; essa mesma certeza ou convicção
> inabalável tão característica dos que se arvoram os guardiões da
> Razão, do Pensamento Correto ou da Verdade e que, por isso, não
> estão muito dispostos a perder tempo com discussão, com a busca do
> consenso mediante o diálogo e outras "masturbações" democráticas,
> motivo pelo qual chegam mesmo a defender ditaduras sanguinolentas
> sempre que lhes convém. Como, por exemplo, o "racionalista"
> e "libertário" de direita Milton Friedman, que certa vez teceu
> elogios rasgados a Pinochet e justificou sua tirania.
>
> É investindo contra o mito da certeza absoluta ou reduzindo-a ao
> ponto de jamais se transformar em instrumento de dominação política
> que se garante "quase certamente" a sociedade aberta.
>
> Ao admitir que "posso estar errado e você certo", não estou senão
> aceitando que: a) é razoável &#8212; ou seja, é uma atitude racional &#8212;
que
> eu considere os seus argumentos; por conseguinte, b) não é
razoável &#8212;
> isto é, não é uma atitude racionalmente justificável &#8212; que eu
venha
> a eliminá-lo, a não ser, obviamente, que você queira e tente me
> eliminar antes. O caráter duvidoso tanto da minha quanto da sua
> crença é o que nos torna iguais apesar de sermos diferentes. A
> incerteza é, também, a justificação racional da minha e da sua
> liberdade de questionar. Ora, o reconhecimento do outro (do
> diferente), a igualdade entre os interlocutores e a liberdade de
> questionar são tanto os princípios da razão quanto da democracia.
>
> E nem é preciso ter certeza absoluta da verdade desses princípios
> para defendê-los, pois todos eles têm por pressuposto o princípio
> mais fundamental de que "tudo é incerto, inclusive esta afirmação".
>
>
> Manuel Bulcão
> 17/02/2002





SUBJECT: Re: Santo Sudario
FROM: "junior_br2001" <junior_br2001@yahoo.com.br>
TO: ciencialist@yahoogrupos.com.br
DATE: 20/03/2005 12:46


Olá Alberto,
Suas observsações merecem comentários

--- Em ciencialist@yahoogrupos.com.br, "Alberto Mesquita Filho"
<albmesq@u...> escreveu
> Subject: Re: [ciencialist] Santo Sudario
> Caro Eduardo
>
> Sinceramente não sei dizer se a coisa seria tão simples assim e por
vários
> aspectos. Em primeiro lugar, diria que esse "todos" refere-se a uma
> amostragem bastante pobre. Ou muito me engano, ou poderia contar 2
de cada
> lado. Por um lado, vejo o Júnior e o Taborda, cada um com idéias
totalmente
> diversas e a abordarem prismas totalmente diversos e nem sei até
que ponto
> eles entrariam em acordo sobre o que escreveram.

JR: Pois é, até que mesmo parecendo concordar, o Taborda já discordou
muitas vezes de mim, embora nossos contatos foram bem poucos. E pelo
que eu leio aqui, apesar de nao entender bulufas de física, me parece
ser o físico mais inteligente e capaz desta lista. Ele tem uma
postura a respeito da astrologia e eu tenho outra completamente
diferente, que apesar de eu ser cético a astrologia, acho que
qualquer um tem o direito de pratica-la. E acho essa perseguição a
ela uma tremenda balela. Enfim, o Taborda versa mais sobre um lado
que eu não entendo, e portanto nossos confrontos foram quase de
aprendizado a suas explicações.

De outro, o Homero e o
> Takata, esses sim a comutarem freqüências próximas, para usar uma
linguagem
> metafórica, e a traduzirem o que aprenderam naquele catecismo que
pulula
> pela Internet, aquele mesmo que faz considerações sobre os "ad
hominem",
> "onus da prova" e outras tacanhices nem sei repetir. E digo
tacanhice não
> por que seja contrário aos conceitos que estão embutidos nessas
palavras,
> mas para me referir aos contextos em que elas têm sido utilizadas.

JR: Pois é como eu afirmei em outra mensgem, os céticos são imunes,
eles sempre dão um jeitinho de tirarem o deles da reta. Eles afirmam
que quase nao praticam ad hominem e não fazem nenhuma alegação
absurda. Enfim, há na verdade um perfeicionismo embutido, e uma
autoproteção psicológica que só os psicólogos saberiam explicar. É
como criar um escudo ao redor de si mesmo para nao se machucarem, e
quando estão errados, as justitificativas entram em cena. O que mais
um cético sabe fazer é se justificar diante de seus erros e
imperfeições.
É muito difícil realmente perceber diferenças individuais entre eles,
quase todos procedem com a mesma metodologia, são fiéis aos mesmos
padrões, enfim a razão parece que é algo treinado e robotizado, e não
advinda do pensamento criativo de fato

>
> Eu não entrei na briga a não ser para tirar o sarro, ora do Homero,
ora do
> Takata, e mais no sentido de demonstrar que ser cético, pelo menos
em
> ciência, não significa "levantar uma bandeira", mas principalmente
adotar
> uma postura que nos permita levar à produção de novos conhecimentos.


JR: Pois é, temos antes de tudo sermos cientistas antes de sermos
céticos




> mais, que se deixe o ceticismo aos filófosos pois eles dão de 1000
a zero em
> qualquer desses "céticos de carteirinha" que outra coisa não fazem
a não ser
> iludir os jovens e denegrir um ceticismo sadio.


JR: Pois é o ceticismo é uma postura filosófica antes de tudo. Elas
nasceram com eles, com os filósofos. Os céticos de carterinha quase
nao tem conhecimento da filosofia, pois estão demasiados presos e
atarracados com aquilo que eles chamam de *ciencia*

>






SUBJECT: Software para estudo de Geometria de Moleculas
FROM: Paulo Sérgio Dias <psdlistdisc@terra.com.br>
TO: <ciencialist@yahoogrupos.com.br>
DATE: 20/03/2005 13:04

Pessoal:

a) Alguem conhece um software gratuito que permita visualizar
Geometria de Moleculas ? Nao e' necessario nada muito sofisticado...
Comecei a estudar esse assunto na aula de Quimica Geral.

Obs.: Encontrei o "Pintar VirtuaLab Molecule", mas e' pago.
Sua versao "trial" e' muito limitada.

b) Solicito tambem o endereco de alguma lista de discussao
sobre Quimica


Grato pela atencao de todos
Paulo

[As partes desta mensagem que não continham texto foram removidas]



SUBJECT: Re: [ciencialist] Software para estudo de Geometria de Moleculas
FROM: "Alberto Mesquita Filho" <albmesq@uol.com.br>
TO: <ciencialist@yahoogrupos.com.br>
DATE: 20/03/2005 13:39

----- Original Message -----
From: "Paulo Sérgio Dias"
Sent: Sunday, March 20, 2005 1:04 PM
Subject: [ciencialist] Software para estudo de Geometria de Moleculas

> a) Alguem conhece um software gratuito que permita visualizar Geometria de
> Moleculas? Nao e' necessario nada muito sofisticado... Comecei a estudar
> esse assunto na aula de Quimica Geral.

Olá Paulo

Eu não sou químico mas acho que você poderá encontrar isso no ChemSketch. A
versão 8.0 já está disponível para download (freeware) em
http://www.acdlabs.com/download/

Creio que o Emiliano poderá dizer se o software atende a suas exigências.
Aliás, antes de fazer o download você poderia ler o "Manual do Programa
ACD/ChemsKetch versão 5.0" em português, escrito pelo próprio Emiliano e que
está disponível no meu site em http://ecientificocultural.com/ftp/manual.pdf

> b) Solicito tambem o endereco de alguma lista de discussao sobre Quimica

As únicas que conheço são a NAEQ-UCS e a Tchê Química, que também me foram
indicadas pelo Emiliano. Elas estão respectivamente em
http://br.groups.yahoo.com/group/naeq-ucs/ e
http://br.groups.yahoo.com/group/tchequimica/

[ ]´s
Alberto
http://ecientificocultural.com/indice.htm
Mas indiferentemente a tudo isso, o neutrino tem massa, o elétron não é
uma carga elétrica coulombiana e a Terra se move. E a história se repetirá.



SUBJECT: Re: [ciencialist] Re: Santo Sudario
FROM: "Alberto Mesquita Filho" <albmesq@uol.com.br>
TO: <ciencialist@yahoogrupos.com.br>
DATE: 20/03/2005 14:00

----- Original Message -----
From: "junior_br2001"
Sent: Sunday, March 20, 2005 12:46 PM
Subject: [ciencialist] Re: Santo Sudario

> Olá Alberto,
> Suas observsações merecem comentários

Olá Júnior

Grato pelos comentários.

Com respeito a outra mensagem que enviei em resposta ao Bulcão, espero que
nem você nem os demais tenham ficado melindrados com a possível taxação de
ignorantes, mesmo porque eu me considero o ignorante mor da Ciencialist.
Aliás, a esse respeito escrevi o seguinte em 1995:

************ início da citaçao *************

A ignorância é um tema pouco estudado. O Novo Dicionário Aurélio, por
exemplo, que, de forma imparcial e não proposital, retrata o resultado de
uma pesquisa sobre nossa cultura literária, dedica 84 linhas para se referir
a saber e sabedoria e apenas 21 linhas, ou seja, um quarto do total
anterior, para discorrer sobre ignorar e ignorância. Perscrutando outras
fontes de conhecimento como, por exemplo, livrarias, bibliotecas, jornais,
revistas, programas de rádio ou televisão (excluindo-se os humorísticos)
concluiremos que nossa ignorância sobre a ignorância é muito grande. Não
obstante, não foram poucos os sábios dos séculos passados que, a exemplo de
Rousseau, a cultuaram como a geradora da insatisfação que, aliada à fé em si
próprios, atuou como o agente motriz a condicioná-los a procurar pela
verdade.

************ final da citaçao *************

Já quanto à hipocrisia eu não digo nada.

O texto citado é parte do artigo "A síndrome dos três i's" que pode ser lido
a partir de http://ecientificocultural.com/ECC2/artigos/tres_is.htm

[ ]´s
Alberto
http://ecientificocultural.com/indice.htm
Mas indiferentemente a tudo isso, o neutrino tem massa, o elétron não é
uma carga elétrica coulombiana e a Terra se move. E a história se repetirá.



SUBJECT: Re: [ciencialist] Software para estudo de Geometria de Moleculas
FROM: Paulo Sérgio Dias <psdlistdisc@terra.com.br>
TO: <ciencialist@yahoogrupos.com.br>
DATE: 20/03/2005 14:14

Alberto:

Muito obrigado pelas informacoes !

Paulo


[As partes desta mensagem que não continham texto foram removidas]



SUBJECT: Re: Santo Sudario
FROM: "junior_br2001" <junior_br2001@yahoo.com.br>
TO: ciencialist@yahoogrupos.com.br
DATE: 20/03/2005 14:22


Ola Taborda, suaas considerações merecem comentários

--- Em ciencialist@yahoogrupos.com.br, "Sergio M. M. Taborda"
<sergiotaborda@t...> escreveu
> >
> > Concordo que isso é quase um argumento de autoridade
> >e, portanto, não se deve levar muito em conta mas é um
> >elemento a ser considerado.
> >
> Não é quase, é mesmo , um argumento de autoridade.
> Repare que a Nature, sendo uma revista "cientifica" possuiu entre
seus
> quadros muito mais cepticos que pessoas normais, já que para ser
cetico
> não é preciso ser cientista.

JR: É o que sempre falei. Quem são essa turminha de céticos? Muitos
nao são cientistas, e nem sequer tem qualquer embasamento em ciência.
No lance da revista, a natureza da revista em que é publicada as
descobertas relamente nao interessa, o que´interessa é que o
experimento ou a nova idéia possa ser reproduzida por todos sem
problemas. Muitos cientistas bons, comuns, capazes e super-
inteligentes são impedidos de publicar por causa de uma elitizinha de
merda financiada pelos americanos.

> Por isso, não admira que uma trabalho contrariando o de 89 não
tenha
> aparecido na Nature. E vários foram feitos, e muitos por pessoas
tão
> creditadas como os que fizeram o de 89.


JR: Pois é, isso que só vale os testes de 88(ou 89)já ficou meio
cansativo. Virou um repeteco dogmático. Muitas pesquisas foram feitas
e elas nao são consideradas válidas por ceticos que tem o cagaço que
o manto seja realmente de Jesus. Claro que eles nao admitem, mas isso
óbvio


> A questão aqui é que os ceticos querem por força que acreditemos
que o
> teste do C14 de 89 é à prova de bala, e que nada nem ninguem pode
ir
> contra os resultados.


JR: Isso, os popperianos positivistas são bem contraditórios, sempre
afirma que nada no mundo científico é infalível, que as hipóteses e
as teorias são falseáveis; ma defendem com unhas e dentes a perfeição
do teste crbono-14 feitos naquela época de forma tão simples,
precária, insuficiente e até mesmo imatura por causas das técnicas
utilizadas naquela época


> Ora, isso é pedir de mais e revela até uma certa ingenuidade
cientifica.
> Nenhum teste é À prova de bala. Ainda para mais quando o teste foi
feito
> sob condições de stress, quando a tecnica foi inventada no momento
e
> quando outros testes posteriores relutam em comprovar esse.


JR: isso tá mais que óbvio


> Ao ocntrário do que os ceticos pensam, a conprovação de um teste
não se
> faz repetindo o teste. Faz-se tentando obter o mesmo resultados por
> métodos diferentes. Por isso que as equipas de testes trabalham em
> separado e apenas os resultados são mostrados. Quando eles usam o
mesmo
> procedimento o que está em teste é o proprio procedimento.
> O teste do C14 não é suficiente contra outros tipos de testes que
dão
> resultados destintos.

Jr: Pois é. Os céticos nao conseguem entender esse fato tão óbvio que
é fazer ciencia, dar enfase o pensamento criador, ser cientista antes
de tudo, até mesmo antes de ser cético

> >
> >
> PQ os de 89 são mais confiáveis? Vamos lá a ver se alguem explica
isso
> direitinho.
> O facto de ser publicado na nature não os torna mais confiáveis.


Jr: Pois é, e já foram apontadas as falhas do teste.


>
> >Finalmente, acho um pouco estranho nessa discussão
> >que o ônus da prova esteja cabendo aos céticos.
> >
> Claro. Pq neste caso os cepticos estão dizendo que o unico teste
> confiável é o de 88.
> Mas não demonstram pq é o mais confiável. Nem o que "confiável"
significa.
> Com tantos testes ,de diversos tipos, que foram feitos, pq temos
que
> aceitar o mais antigo deles ,
> quando para mais nem sequer conincide com os outros ?

JR: Os céticos estão mudados. Até a Popper eles deixaram de render
homenagem. Esse negócio de mais confiáveis, mais isso e aquilo me
cheira a cientifismo barato, um positivismo imiscuído nas idéias
científicas

>
> >Ora,
> >se vc afirma fazer um cachorro voar com uma piscadela,
> >é seu papel provar isso, não de um cético que duvida
> >de sua palavra.
> >
> Exacto. Mas o problema é que são os ceticos no papel de quem afirma
que
> o cachorro voa.
> Afirmam que o teste de 88 é mais confiável. Cadê as provas de que
ele é
> mais confiável ?
> A afirmação de que ele é mass confiável é dos ceticos, não das
pessoas
> normais, que acham que todos os testes têm igual peso.

JR: Pois, eles custam a entender isso. Computadores(não estou falando
do Takata não) tem igual deficiencia para entender uma coisa tão
óbvia, a não ser que temos a todo momento que programa-los. Todos os
testes científicos que seguem uma metodologia deveriam se inspirar
em confiabilidade. Negar isso é cair no nilismo, que é a forma de
ceticismo atual que pulula na internt e nas listas de discussões
céticas.

>
> >Nesse caso, há uma dúvida nos métodos
> >do Dr. Rogers e nos argumentos que usou para
> >"desconstruir" a datação anterior. Tudo bem, vamos
> >assumir que todos têm suas falhas... o que devemos
> >fazer então?
> >
> Não escolher à priopri sem ter todos os dados em maos.
>
> > Simples, optar pelo conservadorismo
> >
> Não. Isso não é uma actitude cientifica!
> Vc pode fazer isso pessoalmente, em casa , ou com os amigos. Mas
não é
> admissivel que defenda essa psotura no lista de ciencia.
> A natureza da ciencia não é conservadora, é dinamica.

Jr: E bota dinamica nisso. Se o teste do Dr. Rogers tem falhas ,
imaginem um teste relizado em 88, com as antigas técnicas de
utilização do carbono 14, num tecido remendado e pouco investigado?


>
> >, que
> >nessa situação é a não autenticidade. Se não é
> >interesse da igreja provar a autenticidade do Sudário,
> >como disse o Taborda, então, não há ciência a ser
> >feita(...)
> >
> Ora, que palermice. Então não temos o direto de saber a verdade ?
> Só pq a santa madre igreja não deseja que invertiguemos o pano, não
> ficamos parados ?
> O que é isto, é a inquisião disfarçada ?
> Os cientistas vão continuar a estudar o pano, atté que existam
certezas.
> A posse do pano não pode ser atribuida a Jesus, pois não temos
nenhum
> dado dele que possa ser testado (como DNA ou impressões digitais,
ou
> representações da época)
> O mais que podemos fazer é provar que pertenceu a uma pessoa ,
algures
> no hoje israel, em algum tempo entre o ano X e Y.
> Provado isso, teremos feito a obrigação cientifica. Tudo o resto é
> inferencia e religião.


JR: A ciencia tem de fazer parte dela. Mas tenho espernças que um
dia o manto seja definitivamente liberado.


>
> >
> >Gostaria de observar, ainda que, nessa discussão, está
> >clara a religiosidade de cada um.
> >
> Se vc acha. ... Mas tb, um cetico acha o que for preciso para
suster o
> seu insustentável argumento .
> E isso cheira-me a ataque moral ...
> Toda a gente - exceto os ceticos, claro - tem interesse no manto.
> Para já é uma forma de testar e desenvolver tecnicas de datação
> importantes em arquilogia.
> Por outro lado se for verdadeiro , tem valor historico como pova
que os
> corpos dos mortos eram envolvidos , tal como sabemos
documentalmente.


Jr: Os céticos tem medo que o manto seja de fato de Jesus, coisa que
os cientistas tão nem aí para isso



>
> >Posso estar
> >enganado, mas chutaria que todos os que defendem a
> >autenticidade do Sudário são cristãos (e acusam os
> >céticos de dogmáticos). É justo, portanto, que cada um
> >assuma a sua religião para que a briga fique mais
> >honesta.
> >
> Não. Não é justo. A ciencia é imparcial. E não queremos aqui
ataques ad
> fique calado.
> As experiencias cientificas valem pelo que valem e não por quem as
fez,
> da mesma forma as argumentações.


Jr : Perfeito. A ciencia pouco tá se lixando pro ceticismo
organizado e a religião.


>
> >Lembro que desde o início, apesar de estar
> >meio à margem da briga, disse que sou um cético
> >(agnóstico e ateu, depende do contexto e do meu
> >humor).
> >
> >
> Toda a gente sabe que os ceticos querem é briga. É ficar martelano
> inutilidades em discussões futeis, sem aportar nenhuma ideia
concreta ou
> que faça sentido.
> Mas entrar nesse jogo é perder tempo precioso pq os ceticos não tão
> preocupados em obter coisa alguma, só perder tempo.


Jr: Poxa, já perdi demais meu tempo com eles, e quando mais penso em
não perder, mais eu perco.





SUBJECT: Re: Santo Sudario
FROM: "junior_br2001" <junior_br2001@yahoo.com.br>
TO: ciencialist@yahoogrupos.com.br
DATE: 20/03/2005 14:37


Imagina se eu me sentiria ofendido, eu entendi muito bem o que voce
escreveu.
Mas na verdade, eu mesmo me considero um ignorante tb...afinal senão
fôssemos ignorantes como nos sentiríamos no direito de nos darmos ao
luxo de estarmos sempre aprendendo?

Jr(O filósofo de trombeta)


--- Em ciencialist@yahoogrupos.com.br, "Alberto Mesquita Filho"
<albmesq@u...> escreveu
> ----- Original Message -----
> From: "junior_br2001"
> Sent: Sunday, March 20, 2005 12:46 PM
> Subject: [ciencialist] Re: Santo Sudario
>
> > Olá Alberto,
> > Suas observsações merecem comentários
>
> Olá Júnior
>
> Grato pelos comentários.
>
> Com respeito a outra mensagem que enviei em resposta ao Bulcão,
espero que
> nem você nem os demais tenham ficado melindrados com a possível
taxação de
> ignorantes, mesmo porque eu me considero o ignorante mor da
Ciencialist.
> Aliás, a esse respeito escrevi o seguinte em 1995:
>
> ************ início da citaçao *************
>
> A ignorância é um tema pouco estudado. O Novo Dicionário Aurélio,
por
> exemplo, que, de forma imparcial e não proposital, retrata o
resultado de
> uma pesquisa sobre nossa cultura literária, dedica 84 linhas para
se referir
> a saber e sabedoria e apenas 21 linhas, ou seja, um quarto do total
> anterior, para discorrer sobre ignorar e ignorância. Perscrutando
outras
> fontes de conhecimento como, por exemplo, livrarias, bibliotecas,
jornais,
> revistas, programas de rádio ou televisão (excluindo-se os
humorísticos)
> concluiremos que nossa ignorância sobre a ignorância é muito
grande. Não
> obstante, não foram poucos os sábios dos séculos passados que, a
exemplo de
> Rousseau, a cultuaram como a geradora da insatisfação que, aliada à
fé em si
> próprios, atuou como o agente motriz a condicioná-los a procurar
pela
> verdade.
>
> ************ final da citaçao *************
>
> Já quanto à hipocrisia eu não digo nada.
>
> O texto citado é parte do artigo "A síndrome dos três i's" que pode
ser lido
> a partir de http://ecientificocultural.com/ECC2/artigos/tres_is.htm
>
> [ ]´s
> Alberto
> http://ecientificocultural.com/indice.htm
> Mas indiferentemente a tudo isso, o neutrino tem massa, o elétron
não é
> uma carga elétrica coulombiana e a Terra se move. E a história se
repetirá.





SUBJECT: Re: [ciencialist] Sudario
FROM: Eduardo Gueron <edgueron@yahoo.com>
TO: ciencialist@yahoogrupos.com.br
DATE: 20/03/2005 16:19

Caro Taborda,

Creio que vc nco entendeu ou quis distorcer o que
falei. Primeiro, em boa parte das mensagens
prs-Rogers, estco sendo atacados os citicos como uma
entidade maligna de um bando de idiotas que odeiam
tudo. Quem esta fazendo julgamento moral de quem?

De tudo o que li nessa discussco e fora dela a
respeito do tema, posso apenas dizer que os argumentos
usados por Rogers para desconstruir o trabalho de 89
sco fracos. Presumo que as ticnicas hoje, quase 20
anos depois, estejam melhores e questionamentos sirios
ao trabalho de 89 devem ser feitos. Infelizmente, nco
considero sirio o trabalho de ROgers, ss isso...

>Nco i quase, i mesmo , um argumento de autoridade.
>Repare que a Nature, sendo uma revista "cientifica"
>possuiu entre seus
>quadros muito mais cepticos que pessoas normais, ja
>que para ser cetico
>nco i preciso ser cientista.
>Por isso, nco admira que uma trabalho contrariando o
>de 89 nco tenha
>aparecido na Nature. E varios foram feitos, e muitos
>por pessoas tco
>creditadas como os que fizeram o de 89.
>A questco aqui i que os ceticos querem por forga que
>acreditemos que o
>teste do C14 de 89 i ` prova de bala, e que nada nem
>ninguem pode ir
>contra os resultados.
>Ora, isso i pedir de mais e revela ati uma certa
>ingenuidade
>cientifica.
>Nenhum teste i @ prova de bala. Ainda para mais
quando >o teste foi
>feito
>sob condigues de stress, quando a tecnica foi
>inventada no momento e
>quando outros testes posteriores relutam em comprovar
>esse.
>Ao ocntrario do que os ceticos pensam, a conprovagco
>de um teste nco se
>faz repetindo o teste. Faz-se tentando obter o mesmo
>resultados por
>mitodos diferentes. Por isso que as equipas de testes
>trabalham em
>separado e apenas os resultados sco mostrados.
Com que autoridade vocj fala "ao contrario do que os
citicos pensam"? Desde o inmcio insisto para que sejam
feitos varios testes por varias equipes diferentes e,
principalmente, que a ticnica seja explmcita. Em
situagues conflitantes como essa nco bastam os
resultados, i necessario que os mitodos sejam abertos
a outros grupos independentes para que qq um possa
repetir o resultado obtido. Isso foi feito, por
exemplo, no caso da fusco a frio (foi uma condigco do
perisdico em que os resultados foram publicados, acho
que foi a Nature mesmo). Varios grupos repetiram o
experimento e nco obtiveram nada, a partir dam
passou-se a nco acreditar nos autores do trabalho. Nco
vejo a mesma disposigco do Rogers e muito menos da
igreja de permitir isso.

Quando >eles usam o mesmo
>procedimento o que esta em teste i o proprio
>procedimento.
>O teste do C14 nco i suficiente contra outros tipos
de >testes que dco
>resultados destintos.


>PQ os de 89 sco mais confiaveis? Vamos la a ver se
>alguem explica isso
>direitinho.
>O facto de ser publicado na nature nco os torna mais
>confiaveis.

>Claro. Pq neste caso os cepticos estco dizendo que o
>unico teste
>confiavel i o de 88.
>Mas nco demonstram pq i o mais confiavel. Nem o que
>"confiavel"
>significa.
>Com tantos testes ,de diversos tipos, que foram
>feitos, pq temos que
>aceitar o mais antigo deles ,
>quando para mais nem sequer conincide com os outros ?

Bem, o teste do Roger diz que o Sudario tem mais de
705 e menos de 3000 anos. Esse resultado i,
provavelmente, muito confiavel na mesma proporgco que
nco significa quase nada. A sua principal tentativa
foi desconstruir o resultado de 89 e, pelo que li,
cada um de seus argumentos foi rebatido.

>Exacto. Mas o problema i que sco os ceticos no papel
>de quem afirma que
>o cachorro voa.
>Afirmam que o teste de 88 i mais confiavel. Cadj as
>provas de que ele i
>mais confiavel ?
>A afirmagco de que ele i mass confiavel i dos
ceticos, >nco das pessoas
>normais, que acham que todos os testes tjm igual
peso.


> Simples, optar pelo conservadorismo
>
>Nco. Isso nco i uma actitude cientifica!
>Vc pode fazer isso pessoalmente, em casa , ou com os
>amigos. Mas nco i
>admissivel que defenda essa psotura no lista de
>ciencia.
>A natureza da ciencia nco i conservadora, i dinamica.

A cijncia i dinbmica e, ao mesmo tempo, conservadora.
Se uma nova teoria nada acrescenta a anterior, os
cientistas tendem a manter a original. I obrigagco do
cientista ser citico ao novo para que aquele que
inovou se sinta obrigado a mostrar exaustivamente a
validade e importbncia de seus novos resultados. Sendo
bem sucedido nisso, se torna pai de algo novo...
(Claro que isso tudo i um pouco utspico ja que ha
outros tipos de critirios nco muito cientmficos
envolvidos)


>Ora, que palermice. Entco nco temos o direto de saber
>a verdade ?
>Ss pq a santa madre igreja nco deseja que
>invertiguemos o pano, nco
>ficamos parados ?
>O que i isto, i a inquisico disfargada ?
>Os cientistas vco continuar a estudar o pano, atti
que >existam
>certezas.
>A posse do pano nco pode ser atribuida a Jesus, pois
>nco temos nenhum
>dado dele que possa ser testado (como DNA ou
>impressues digitais, ou
>representagues da ipoca)
>O mais que podemos fazer i provar que pertenceu a uma
>pessoa , algures
>no hoje israel, em algum tempo entre o ano X e Y.
>Provado isso, teremos feito a obrigagco cientifica.
>Tudo o resto i
>inferencia e religico.

O znico problema disso tudo i que o pano pertence a
igreja e ela libera seu estudo quando e para quem
quiser. QUem disse que nco vivemos em um tipo de
inquisigco mais light? Veja a posigco do Vaticano em
relagco a Argentina, aos gays, a AIDS, ao governo
espanhol...

>
>Se vc acha. ... Mas tb, um cetico acha o que for
>preciso para suster o
>seu insustentavel argumento .
>E isso cheira-me a ataque moral ...
>Toda a gente - exceto os ceticos, claro - tem
>interesse no manto.
>Para ja i uma forma de testar e desenvolver tecnicas
>de datagco
>importantes em arquilogia.
>Por outro lado se for verdadeiro , tem valor
historico >como pova que os
>corpos dos mortos eram envolvidos , tal como sabemos
>documentalmente.
>Poderemos nco ss saber que eram, mas que como eram.
>Se for falso, teremos uma importante pega para provar
>que o ser humano
>i
>capaz de tudo para manter o controle sobre as massas,
>sobre os grupos e sobre si mesmo.
>Portanto i tco interessante para crentes em Jesus,
como >para descrentes
>e agnosticos pois o seu valor i muito mais que
>religioso, i historico e
>sociologico tb.

>
>Nco. Nco i justo. A ciencia i imparcial. E nco
>queremos aqui ataques ad
>hominem, ou ataques morais.
>Se tem algo a dizer , diga-o e demonstre seu ponto.
Se >nco i capaz ,
>fique calado.
>As experiencias cientificas valem pelo que valem e
nco >por quem as fez,
>da mesma forma as argumentagues.

>
>
>Toda a gente sabe que os ceticos querem i briga. I
>ficar martelano
>inutilidades em discussues futeis, sem aportar
nenhuma >ideia concreta
>ou
>que faga sentido.
>Mas entrar nesse jogo i perder tempo precioso pq os
>ceticos nco tco
>preocupados em obter coisa alguma, ss perder tempo.

Realmente, essa i uma bela linha argumentativa. Quem
sou eu para refutar?

Abraco, Eduardo.





__________________________________
Do you Yahoo!?
Yahoo! Small Business - Try our new resources site!
http://smallbusiness.yahoo.com/resources/


SUBJECT: Re: Santo sudário e a vanilina(correção)
FROM: "rmtakata" <rmtakata@altavista.net>
TO: ciencialist@yahoogrupos.com.br
DATE: 20/03/2005 18:04


--- Em ciencialist@yahoogrupos.com.br, "Sergio M. M. Taborda"
> O problema é que o fogo e outros factores contaminam o
> tecido ao longo do anos com carbono "mais recente" do que
> aquele presente na época verdadeira do tecido..

Seria problema apenas se esse efeito fosse significativo. A troca
maxima do carbono chega a 25%. Para q. houvesse compatibilidade com o
inicio da era crista a troca teria q. ser de quase 100% na epoca do
incendio.

[]s,

Roberto Takata





SUBJECT: Re: Santo sudário e a vanilina(correção)
FROM: "junior_br2001" <junior_br2001@yahoo.com.br>
TO: ciencialist@yahoogrupos.com.br
DATE: 20/03/2005 18:13


Takata ninguém aqui está sopondo que o manto é de Cristo, portanto
descarte essa opção.
25% já seria uma margem boa para invalidar o teste do carbono-14
realizado em 88

JR

--- Em ciencialist@yahoogrupos.com.br, "rmtakata" <rmtakata@a...>
escreveu
>
> --- Em ciencialist@yahoogrupos.com.br, "Sergio M. M. Taborda"
> > O problema é que o fogo e outros factores contaminam o
> > tecido ao longo do anos com carbono "mais recente" do que
> > aquele presente na época verdadeira do tecido..
>
> Seria problema apenas se esse efeito fosse significativo. A troca
> maxima do carbono chega a 25%. Para q. houvesse compatibilidade com
o
> inicio da era crista a troca teria q. ser de quase 100% na epoca do
> incendio.
>
> []s,
>
> Roberto Takata





SUBJECT: Re: Sudario
FROM: "junior_br2001" <junior_br2001@yahoo.com.br>
TO: ciencialist@yahoogrupos.com.br
DATE: 20/03/2005 19:42


--- Em ciencialist@yahoogrupos.com.br, Eduardo Gueron <edgueron@y...>
escreveu
> Caro Taborda,
> A ciência é dinâmica e, ao mesmo tempo, conservadora.
> Se uma nova teoria nada acrescenta a anterior, os
> cientistas tendem a manter a original. É obrigação do
> cientista ser cético ao novo para que aquele que
> inovou se sinta obrigado a mostrar exaustivamente a
> validade e importância de seus novos resultados. Sendo
> bem sucedido nisso, se torna pai de algo novo...
> (Claro que isso tudo é um pouco utópico já que há
> outros tipos de critérios não muito científicos
> envolvidos)
/


JR: Hein, não é bem assim, ciencia nao é uma ditadura. Se teoria e a
hipótese for absurda sim. Mas me parece que o trabalho de Rogers nao
foi nenhum absurdo homeopatico. Ele apresentou a evidencia de uma
substancia no manto que indica que o manto é mais velho que poderiam
imaginar ; metodo de estudo que *pode* , eu disse pode(necessário
mais estudos) falsear o primeiro estudo.

Não se pode ser cético a um fato ou diante de uma evidencia
importante que foi bem testada exaustivamente. Claro que ainda nao é
o caso do experimento do Rogers; nem portanto , é o caso do
experimento do carbono-14-algo vago e imcompleto





SUBJECT: Re: Gravitons/p/Victor)
FROM: "junior_br2001" <junior_br2001@yahoo.com.br>
TO: ciencialist@yahoogrupos.com.br
DATE: 20/03/2005 19:53


Fala aí, Victor, estou retornando meus estudos de física que ficaram
parados por causa de problemas pessoais ano passado.
Qualquer coisa vou encher o seu saquinho aí...hehehe

Jr

--- Em ciencialist@yahoogrupos.com.br, JVictor <jvoneto@u...> escreveu
> Luiz Ferraz Netto escreveu:
>
> Apreciaria comentários sobre o seguinte texto:
>
> http://rolfguthmann.sites.uol.com.br/TQG/tqg.html
>
> Victor.





SUBJECT: Enrolamentos de Motores Elétricos
FROM: "Tipoalgo" <tipoalgo@bol.com.br>
TO: ciencialist@yahoogrupos.com.br
DATE: 20/03/2005 19:57


Olá a todos

Estou estudando e praticando como enrolar, ou bobinar, motores
elétricos.
Tenho um livro, Cálculo de Enrolamentos de Máquinas Elétricas e
Sistemas de Alarme, que trata bem o assunto, mas gostaria de auxílio
mais moderno para o assunto.
Para facilitar meus estudos procurei na internet um programa livre, e
achei em http://www.md.cefetpr.br/tocadofisch/dgex101.zip um bom
ajudante.
Entretanto este programa ainda está em desenvolvimento.
Aproveito a oportunidade para agradecer ao autor por disponibilizá-lo
na internet de forma livre.
Alguém conhece um programa livre semelhante que pudesse nos
apresentar?

Agradecimentos antecipados.

Tipoalgo





SUBJECT: Re: Enrolamentos de Motores Elétricos
FROM: "junior_br2001" <junior_br2001@yahoo.com.br>
TO: ciencialist@yahoogrupos.com.br
DATE: 20/03/2005 19:59


Programa nao sei de nenhum não, mas sei que o Instituro Monitor tem
um cursos profissionalizantes e técnicos por correspondencia sobre
enrolamento de motores - É muito interessante, e nao é tão caro
porque voce paga parcelado

Jr

--- Em ciencialist@yahoogrupos.com.br, "Tipoalgo" <tipoalgo@b...>
escreveu
>
> Olá a todos
>
> Estou estudando e praticando como enrolar, ou bobinar, motores
> elétricos.
> Tenho um livro, Cálculo de Enrolamentos de Máquinas Elétricas e
> Sistemas de Alarme, que trata bem o assunto, mas gostaria de
auxílio
> mais moderno para o assunto.
> Para facilitar meus estudos procurei na internet um programa livre,
e
> achei em http://www.md.cefetpr.br/tocadofisch/dgex101.zip um bom
> ajudante.
> Entretanto este programa ainda está em desenvolvimento.
> Aproveito a oportunidade para agradecer ao autor por disponibilizá-
lo
> na internet de forma livre.
> Alguém conhece um programa livre semelhante que pudesse nos
> apresentar?
>
> Agradecimentos antecipados.
>
> Tipoalgo





SUBJECT: Eter Luminifero (era: ninguem é inocente.)
FROM: "Oraculo" <oraculo@atibaia.com.br>
TO: <ciencialist@yahoogrupos.com.br>
DATE: 20/03/2005 22:28

Olá Pubmed

Pubmed: Naõ começa tropeçar nos seus proprios argumentos, o Éter luminífero é
um mito que foi derrubado...uma teoria absurda que foi demonstrada
falsa, portanto nao passa de um mito"

Está enganado. O éter luminífero foi uma teoria que se embasava nos dados e conhecimentos da época. Explicava eventos e fenomenos do mundo físico, através de observações e experimentos. Ainda que tenha se mostrado irreal depois, com mais dados, nada nele lembra um mito, como duendes ou seres divinos de religiões. Sua estrutura, função e principalmente sua posterior refutação o tornam totalmente diferente de mitos e lendas.

Os erros e enganos que as teorias cientificas as vezes apresentam fazem parte do método de criar repostas e depois testá-las rigorosamente, ajustando se necessário e abandonando o que for refutado. Mitos, por sua vez, se mantém indefinidamente, sem mudança ou ajuste. Não faz parte de sua natureza ser refutado, nme precisam de dados e testes para se manter.

Se descobrirem que o Big Bang é incorreto, e não absurdo como parece pensar que pode acontecer, será uma ação legitimamente científica e os cientistas, tanto os que criaram a teoria quanto os que a refutaram, ficarão felizes e aceitarão as novas conclusões. O Big Bang é uma teoria que, neste momento, é mais capaz de explicar o surgimento do universo que qualquer outra, e perdurará até que mais dados a comprovem ou refutem. Não é nem de longe um mito, nem mesmo se descobrirmos que está incorreto (de novo, a teoria será, não absurda, mas incorreta).

Veja, hoje a Big Bang explica porque o céu não é totalmente claro, mesmo durante a noite, explica como podem as galáxias estarem se afastando, explica a formação de estrelas, nuvens de gás estelar, materia escura, elementos quimicos, etc. Para substitui-la, é preciso uma teoria que, além de explicar tudo isso, ainda explique coisas que escapam a teoria do BB. Se o fizer, o BB será ultrapassado, você (ou quem for responsável) ganhará um premio Nobel, e a ciência terá novos dados e hipóteses para testar. Tudo com o rigor necessário.

O Éter Luminífero explicava a propagação da luz e dezenas de outros fenomenos observados, que depois foram melhor explicados pela teoria eletromagnética. Apenas isso, não foi a substituição de um mito por outro.

Precisa compreender isso, ou jamais compreenderá o que é a ciência e o que a torna diferente de crenças e mitos.

Homero

----- Original Message -----
From: pubmed2005
To: ciencialist@yahoogrupos.com.br
Sent: Saturday, March 19, 2005 3:07 AM
Subject: [ciencialist] Re: ninguem é inocente.



Naõ começa tropeçar nos seus proprios argumentos, o Éter luminífero é
um mito que foi derrubado...uma teoria absurda que foi demonstrada
falsa, portanto nao passa de um mito

Se descobrirem que o Bing bang nao é teoria mais proxima da origem do
universo, isso vai soar tão absurdo como a crença no boi tatá. Se o
boi tatá nao existe, se foi comprovado a sua inexistencia, isso vai
refutar que o boi tatá só existe na imaginação criativa das pessoas.
Assim como o Éter luminífero preencheu as mentes científicas dos
séculos anteriores. Tudo nao se passou de uma ilusão

Quanto ao Sudário, já está mais que esclarecida minha posição, nao
tenho que ficar repetindo os mesmos arguimentos de forma circular
como voce vem fazendo



--- Em ciencialist@yahoogrupos.com.br, "Oraculo" <oraculo@a...>
escreveu
> Olá Pubmed

>
> risos..:-) É enorme..:-) No primeiro caso, a conclusão, e não
crença, foi abandonada devido a evidencias posteriores que refutaram
a teoria do éter. No segundo, nada vai mudar e quem crê nisso vai
continuar crendo, sem que evidencias ou provas ou aspectos materiais
ou racionais tenham o poder de interferir oiu mudar essa crença. A
diferença, como pode notar é enorme..:-) A primeira, uma teoria
cientifica, esperava por dados e evidencias que a confirmassem, e se
conformou com a refutação por falta de provas. A segunda, criada da
imaginação humana, vai se manter indefinidamente, exatamente como foi
criada, dependendo apenas da crença subjetiva de seres humanos.
Enorme diferença..:-)
>






##### ##### #####

Para saber mais visite
http://www.ciencialist.hpg.ig.com.br


##### ##### ##### #####


Yahoo! Grupos, um serviço oferecido por:







------------------------------------------------------------------------------
Links do Yahoo! Grupos

a.. Para visitar o site do seu grupo na web, acesse:
http://br.groups.yahoo.com/group/ciencialist/

b.. Para sair deste grupo, envie um e-mail para:
ciencialist-unsubscribe@yahoogrupos.com.br

c.. O uso que você faz do Yahoo! Grupos está sujeito aos Termos do Serviço do Yahoo!.



[As partes desta mensagem que não continham texto foram removidas]



SUBJECT: Re: [ciencialist] Re: ninguem é inocente.
FROM: "Oraculo" <oraculo@atibaia.com.br>
TO: <ciencialist@yahoogrupos.com.br>
DATE: 20/03/2005 22:33

Olá Pubmed

Pubmed: Voce está vendo a coisa unilateralmente validando excessivamente o
primeiro estudo. Isso a meu ver é um estratégia cientifista. Não
quero invalidar o primeiro estudo,..."

Tolice. Desculpe o termo, mas é tolice. Eu dou maior valor ao primeiro estudo sim, e expliquei exatamente o porque. E não apenas eu, mas toda a comunidade cientifica e ela o faz pelas mesmas razões: neste momento os primeiros estudos são mais confiáveis. Estes tem confirmação independente, mais de uma vez, o segundo ainda precisa ser confirmado. E isso não é cientificismo, mesmo que cientificismo fosse isso que você pensa que é.

E não importa se você quer ou deixa de querer invalidar o primeiro estudo, você simplesmente não importa. Importa que o segundo estudo encontrou, e afirma serem corretas, datas que contradizem os primeiros estudos. Se o segundo está correto, os primeiros estão incorretos. Você nada tem a ver com isso, nem o debate.

Esta diuscussão está fora de controle, com esse tipo de abordagem "ad hominem" cosntante. Se atenha aos argumentos, se puder refuta-los, ótimo, se não puder, acabamos a discussão.

Homero



----- Original Message -----
From: pubmed2005
To: ciencialist@yahoogrupos.com.br
Sent: Saturday, March 19, 2005 3:13 AM
Subject: [ciencialist] Re: ninguem é inocente.



Antes de cortar pedacinhos de minha mensagem, leia todo o conteúdo,
que voce vai entender. Voce nao leu e quer fazer comentários
absurdos. Vai lá e le de novo, ou então coloque o texto na integra e
comente, senão voce destroi o sentido do que está dizendo que a
analise química no é so utilizada paraa datação copmo para muitos
outros resultados como constituição do tecido, das substancia nele
contidas até que ponto uma substancia química pode durar secularmente
A que nível ela pode ser destruída e conservada durante opassar do
tempo

Voce está vendo a coisa unilateralmente validando excessivamente o
primeiro estudo. Isso a meu ver é um estratégia cientifista. Não
quero invalidar o primeiro estudo, mas antes poder complementa-lo com
o segundo, comparar as análises e chegar uma conclusão ou apenas
parte dela. Ver os pontos colisivos.

Voce só está interessando em afirmar ou provar que o manto nao foi o
de cristo, eu quero ir bem mais além


--- Em ciencialist@yahoogrupos.com.br, "Oraculo" <oraculo@a...>
escreveu
> Olá Pubmed
>
> "Pubmed: O método radioisotópíco nao é mais confiável que o químico"
>
> Prove. Aceito estudos de comparação, experimentos controlados com a
datação quimica de artefatos de data conhecida, papers de quimicos e
fisicos sobre o assunto, bibliografia sobre carbono-14 e datação
quimica, e qualquer estudo publicado em revista cientifica..:-) Até
mesmo a palavra de especialistas nos dois métodos, publicados em
livros ou artigos relacionados...:-)
>
> Já que fez uma afirmação prerremptória (que palavrinha maneira..:-)
fica a seu ônus provar o que disse..:-)
>
> Mas, se não puder, deve aceitar a alegação que o carbono-14, que
tem tudo isso bem documentado, é mais confiável que a datação quimica
apresentada no novo estudo..:-)
>
> Homero
>
> ----- Original Message -----
> From: pubmed2005
> To: ciencialist@yahoogrupos.com.br
> Sent: Saturday, March 19, 2005 2:53 AM
> Subject: [ciencialist] Re: ninguem é inocente.
>
>
>
> Se eu nao me engano o Siqueira é espirita e espíritualista, eu
nao
> tenho religião. Pode ser até que combater céticos equivocados
seja
> minha religiao atual, o que faça eu parecer um pouco com o Julio.
Mas
> ele combate o ceticismo defendendo a religiao dele, e eu faço
> defendendo o equivoco filosófico de alguns céticos de plantão, e
> alguns tons pretenciosos de suas faças e escritos.
> Mas há de convir que é até uma religião nobre combater o lado
negro
> do ceticismo")- o ceticismo que é outra forma de religião
> E ainda tem o lado das falácias. O Julio costuma escorregar
pacas,
> mas essa é outra questão, o cara é legal e nao é lícito julga-lo
> nesta lista
>
> Os problema é que os falsos céticos sempre recebem mal essas
> observações, e não recebem as criticas com o mesmo humor com que
as
> fazem. Creio que os verdadeiros céticos pouco se importam com
minhas
> críticas....
>
>
> Mas isso nao interessa, vamos aos comentários
>
>
>
> --- Em ciencialist@yahoogrupos.com.br, "rmtakata" <rmtakata@a...>
> escreveu
> >
> > --- Em ciencialist@yahoogrupos.com.br, "pubmed2005"
> > > o Bradock e o Júlio há um diferença ideológica bem
> > > siginificativa.
> >
> > Tem?
> >
> > > Nao é quesçao de ser mais confiável.
> >
> > Eh questao de ser mais confiavel. Se duas fontes se contradizem
pelo
> > menos uma delas estah errada.
>
> O método radioisotópíco nao é mais confiável que o químico. Pode
até
> ser que eles tenham natureza investigativas distintas, mas um
método
> nao é mais confiável que outros, apenas tem limites em alguns
> setores. O químico pode nao ser datativo, mas pode fornecer
> informções importantes sobre outras questões e até mesmo fazer
com
> que elas colidam nas informações essenciais
>
>
>
>
>
>
> ##### ##### #####
>
> Para saber mais visite
> http://www.ciencialist.hpg.ig.com.br
>
>
> ##### ##### ##### #####
>
>
> Yahoo! Grupos, um serviço oferecido por:
>
>
>
>
>
>
>
> --------------------------------------------------------------------
----------
> Links do Yahoo! Grupos
>
> a.. Para visitar o site do seu grupo na web, acesse:
> http://br.groups.yahoo.com/group/ciencialist/
>
> b.. Para sair deste grupo, envie um e-mail para:
> ciencialist-unsubscribe@yahoogrupos.com.br
>
> c.. O uso que você faz do Yahoo! Grupos está sujeito aos Termos
do Serviço do Yahoo!.
>
>
>
> [As partes desta mensagem que não continham texto foram removidas]





##### ##### #####

Para saber mais visite
http://www.ciencialist.hpg.ig.com.br


##### ##### ##### #####


Yahoo! Grupos, um serviço oferecido por:

São Paulo Rio de Janeiro Curitiba Porto Alegre Belo Horizonte Brasília




------------------------------------------------------------------------------
Links do Yahoo! Grupos

a.. Para visitar o site do seu grupo na web, acesse:
http://br.groups.yahoo.com/group/ciencialist/

b.. Para sair deste grupo, envie um e-mail para:
ciencialist-unsubscribe@yahoogrupos.com.br

c.. O uso que você faz do Yahoo! Grupos está sujeito aos Termos do Serviço do Yahoo!.



[As partes desta mensagem que não continham texto foram removidas]



SUBJECT: Re: [ciencialist] Re: Sudario
FROM: "Oraculo" <oraculo@atibaia.com.br>
TO: <ciencialist@yahoogrupos.com.br>
DATE: 20/03/2005 22:35

Olá Pubmed

Pubmed: Então me diz no que o conhecimento cientifico do químico está errado
e no que ele está sendo incorreto. Não acho que o trabalho de Rogers
seja tão ruim. Ele levtou questoes muito interessantes que merecem
uma verificação. Se voce tem coisa melhor em mente apresente a
comunidade científica"

Embora não pense que vai adiantar, vou tentar de novo: este estudo precisa ser confirmado por pesquisadores independentes. Isso é absolutametne necessário, devido ao rigor do método, para dar confiabilidade aos resultados. Até lá, os estudos com confirmação independente, tem preferencia.

Homero



----- Original Message -----
From: pubmed2005
To: ciencialist@yahoogrupos.com.br
Sent: Saturday, March 19, 2005 4:23 AM
Subject: [ciencialist] Re: Sudario



Segue abaixo

--- Em ciencialist@yahoogrupos.com.br, "Oraculo" <oraculo@a...>
escreveu
> Visão estereotipada da ciência e de cientistas. Descobrir que um
conhecimento cientifico está incorreto ou que precisa ser ajustado,
não é um "mico" para a ciência e para cientistas, pelo contrário.


Então me diz no que o conhecimento cientifico do químico está errado
e no que ele está sendo incorreto. Não acho que o trabalho de Rogers
seja tão ruim. Ele levtou questoes muito interessantes que merecem
uma verificação. Se voce tem coisa melhor em mente apresente a
comunidade científica


ORACULO: Em geral, quanto maior a descoberta, quanto maior o erro
descoberto, maior o prestígio e aplauso que se recebe. Diferente de
crenças e religiões, mostrar um erro em ciência dá direito a ser
premiado, e não repreendido. Até um premio Nobel é possível ganhar,
se sua descoberta for realmente revolucionária.

Um religioso pode ganhar o premio nobel da paz , ate aí voce nao
disse nada. Pode ficar famoso, e ter méritos reconhecidos. E um
religioso cientista pode ter seu trabalho publicado e vangloriado nos
meios cientificos

> Descobrir que novos métodos de datação são mais precisos e
confiáveis pode garantir um Nobel para o Rogers e aplausos do Randi,
não um "mico". É uma critica comum e totalmente erronea a que afirma
isso..:-)

Mas Randi nao vai se meter com um químico, ele só saberia lidar bem
com homeopatas, espiritas e mágicos.

Como ele poderia criticar e lidar c/ um trabalho feito por um
cientista do porte como o físico Einstein? Ele só ia meter a colher
dele aonde nao deveria meter. Ser um grande palpiteiro sem maior
conhecimento. O mundo está cheio de palpiteiros
Veja, criticar o cientifismo nao é erroneo, é sim um grande serviço
prestadoa comuninidade





##### ##### #####

Para saber mais visite
http://www.ciencialist.hpg.ig.com.br


##### ##### ##### #####


Yahoo! Grupos, um serviço oferecido por:







------------------------------------------------------------------------------
Links do Yahoo! Grupos

a.. Para visitar o site do seu grupo na web, acesse:
http://br.groups.yahoo.com/group/ciencialist/

b.. Para sair deste grupo, envie um e-mail para:
ciencialist-unsubscribe@yahoogrupos.com.br

c.. O uso que você faz do Yahoo! Grupos está sujeito aos Termos do Serviço do Yahoo!.



[As partes desta mensagem que não continham texto foram removidas]



SUBJECT: Re: [ciencialist] Sudario
FROM: "Sergio M. M. Taborda" <sergiotaborda@terra.com.br>
TO: ciencialist@yahoogrupos.com.br
DATE: 20/03/2005 22:50

Eduardo Gueron wrote:

>Caro Taborda,
>
>Creio que vc não entendeu ou quis distorcer o que
>falei. Primeiro, em boa parte das mensagens
>pró-Rogers, estão sendo atacados os céticos como uma
>entidade maligna de um bando de idiotas que odeiam
>tudo. Quem está fazendo julgamento moral de quem?
>
>
Os ceticos não estão sendo atacados. Eles estão se sentido atacados.
O que está sendo atacado é o ceticismo. Mas não ha ceticismo sem ceticos.
De facto os ceticos são uma entidade maligna , na medida que não aportam
nada de nada a coisa nenhuma.
A sua unica razão de existir é ser do contra. Isso não é uma atitude
cientifica.
Desse ponto de vista eles são uma epidemia para a comunidade cientifica.

O ponto que doi é que os ceticos entitulam-se criticos, e dificiceis de
convencer, mas ninguem crticia
muito do se anda passando por ai não só nas ciencias , mas em outras
areas da vida humana.
Quem é cetico de verdade não duvida apenas de bruxas , fantasmas, e
mantos de turim. Duvida tb de novas leis,
novas práticas, novas ideias em geral.
Não vejo a comunidade cetica duvidar de deputados, do senado , do
presidente. Nem dos artigos da Nature.
Nem dos livros de Hawkings, Miko e outros da mesma estripe.
E por eles não fazerem isso, conclui-se que seu unico objectivo é ser do
contra no que diz respeito a aquilo que não pode ser comprovado.
É facil ser contra a existencia de aliens, fantasmas e outras coisas
dais quias não existem provas taxativas da sua existencia.
Mas quando a responsabilidade aumenta, os ceticos calam-se, Isso no meu
dicionário é ser idiota. Então, não é um ataque ad hominen, nem um
ataque moral.
É uma comprovação dos factos que a comunicade cetica apresenta nos seus
sites na web e nos argumentos dos seus advogados (advogado é aquele que
apoia a causa
e tenta convencer outros)

>De tudo o que li nessa discussão e fora dela a
>respeito do tema, posso apenas dizer que os argumentos
>usados por Rogers para desconstruir o trabalho de 89
>são fracos. Presumo que as técnicas hoje, quase 20
>anos depois, estejam melhores e questionamentos sérios
>ao trabalho de 89 devem ser feitos. Infelizmente, não
>considero sério o trabalho de ROgers, só isso...
>
>
Então justifique pq. Não diga apenas que é do contra. Explique-se!
Afinal , isto é uma lista de discussão de ciencia. Se vc é contra,
apresente seus argumentos.
Tenha uma atitude cientifica.

Sérgio Taborda

P.S. Eu sei que é complicao explicar os argumentos em poucas linhas, mas
somos pacientes, temos todo o tempo.
O que interessa é que as explicações apareçam.


SUBJECT: Re: [ciencialist] Re: ninguem é inocente.
FROM: "Oraculo" <oraculo@atibaia.com.br>
TO: <ciencialist@yahoogrupos.com.br>
DATE: 20/03/2005 23:01

Olá Sergio

Embora quase todo o texto seja apenas um ataque "ad hominem", eu não fico chateado..:-) Todos tem direito a opinião, afinal..:-)

Sergio: Os ceticos não fazem isso, simplesmente duvidam e são contra qq
hipote de demonstracção da verdade cientifica."

Bullshit..:-) Um bom termo em ingles, para definir a coisa toda..:-) Céticos duvidam sim, mas se a comprovação for rigorossa e bem suportada, é perfeitamente aceitável..:-) Dificilmente se encontrará qualquer cético ou cientista que concorde com sua afirmação, e mesmo qualquer exemplo que demonstre a validade dessa afirmação. É apenas "ad homine" e um espantalho, claramente falso..:-)

Céticos aceitam milhares de afirmações, bastante espantosas, que a ciencia já produziu, mas todas elas muito bem embasadas por evidencais e estudos. Se céticos deixam de lado outras afirmações, é simplesmente porque elas tentam " escapar" desse rigor, e desejam ser "científicas" sem passar pela crítica e análise cientifica.

Sergio: Vc está enganado, como de costume. É improvável que se possa provar que
o manto pertenceu , ou melhor, esteve em contacto com, Jesus. "

Apenas se estiver usando jesus significando o mesmo jesus milagroso da igreja catolica. Se usar como eu, apenas como o nome de uma pessoa que viveu naquela época, como Julio César ou Marco Antonio, é perfeitamente razoável e possível, embora improvável, determinar isso. Quanto ao jesus milagroso da igreja, é problema dela provar sua existencia, e nada tem a ver com ciência, história ou qualquer forma de conhecimento verificável.

Você está enganado, como de costume..:-)

Sergio: explica perfeitamente pq os testes do carbono 14 foram inconclusivos.
Esta é uma palavra que os ceticos desocnhecem. "

Bullshit..:-) (gostei do termo..:-) Céticos inssistem nessa palavra, talvez mais que crentes, que tem certezas diversas, e tendem a tomar qualquer fiapo de esperança como prova conclusiva de suas crenças. Testes inconclusivos, como o proprio nome diz, não significam que qualquer explicação seja correta, muito menos uma sobrenatural e sem evidencais como costumam ser as explicações religiosas. Crendices não servem de alternativa a testes inconclusivos.

E, afinal, toda a discussão desta thread é justamente sobre o segundo teste ser inconclusivo até que se tenham estudos confirmatórios e a repetição dos experimentos e resultados. Eu poderia usar sua frase para refutar toda argumentação sobre este segundo estudo até agora..:-)

Sergio: Facto muito mais interessante e que ninguem conseguiu explicar até agora
é a formação da imagem no manto. "

Sim, muito interessante, e deveria ser investigado. Com acesso pleno ao manto, com toda a capacidade da ciência em desvendar mistérios, com cientistas de diversas áreas trabalhando nisso, e não um manto secreto, guardado a sete chaves, com fiapos de tecido apenas disponível, e um outro manto de histórias fantasticas e sobrenaturais toldando a visão. Concordo com você, deveria , mesmo, ser investigado, mas vá dizer isso a igreja catolica e a santa sé, para ver o que é bom para a tosse..:-)

De novo, a falta de explicação por sí só não valida qualquer outra explicação sem evidencias, ainda mais sobre um artefato mitico e escondido, sobre o qual há mais historias falsas que informação confiável.

Sergio: Tudo isto para dizer que a sua frase "Veja, se for de jesus, isso
significa que existiu um cara chamado jesus, apenas isso." não está nem
perto da verdade."

Minha frase é precisa, no alcance que eu dei a ela. Provar que o manto pertenceu a um sujeito chamado jesus, prova apenas que o manto pertenceu a um sujeito chamado jesus. Como provar que um túmulo pertenceu a Heitor, prova apenas que o tumulo pertenceu a Heitor, não que este lutou com Aquiles, que Aquiles era filho de uma ninfa e invulnerável, que podia ser ferido no calcanhar, que Ulisses foi ao Hades, ou que Circe se apaixonou por ele e transformou seus homens em animais.

Da mesma forma, encontrar ruinas de um castelo onde teria morado um comandante ingles chamado Arthur, prova que existiu um castelo onde morou um Arthur. Mesmo que no periodo correto da lenda de Pendragon, não valida a existencia de Merlin, Avalon ou feiticeiros.

Minha frase é perfeitametne aceitável e racional, no alcance que eu dei a ela. Sua implicancia para comigo (talvez devido aos debates sobre astrologia) não está deixando você ver corretamente o argumento, e produzindo diatribes no lugar de refutações razoáveis.

Sergio: Apenas outros estudos podem refutar estudos preliminares e não a razão,
ou a imaginação das pessoas. É isso que os ceticos fazem constantemente,
e que sinceramente, já encheu o saco."

Como eu disse, sua irritação comigo está toldando seu, em geral excelente, raciocínio. Releia o texto e verá que a afirmação sobre refutar foi do Pubmed. Minha resposta apenas queria demonstar que um estudo único fica sempre em compasso de espera que outros estudos confirmem ou refutem seus resultados e conclusões. Que é exatamente o que você afirma na mensagem acima, embora pense estar discordando de mim.

E tem razão, já encheu o saco, mas não há nada que se possa fazer a respeito. Mesmo que seja um saco, conferir, analisar, testar e retestar cada estudo, cada alegação, cada proposição, esta é a única maneira de dar confiabilidade ao conhecimento. Abandonar esse rigor, por que isso "enche o saco", apenas nos levaria de volta a um tempo onde o conhecimento era pouco confiável, e eramos incapazes de separar real de irreal.

Mesmo enchendo o saco, a ciência, e os cientistas e céticos, continuarão a usar o rigor e a análise crítica a toda informação e alegação apresentada como real.

Um abraço.

Homero



----- Original Message -----
From: Sergio M. M. Taborda
To: ciencialist@yahoogrupos.com.br
Sent: Saturday, March 19, 2005 11:17 AM
Subject: Re: [ciencialist] Re: ninguem é inocente.


Oraculo escreveu:

Mas escreveu , como de costume , cheio de incoerencias cientificas. Na
sua busca pelo conhecimento imaculado livre de crença, como de costume,
é forçado a acreditar em algo para suportar os seus argumentos
anti-cientificos. O que, é normal nos céticos. (e eu uso o termo no
sentido prejorativo sim. Faz muito tempo que esse termo significou
alguma coisa para a ciencia. Quem duvida sistemáticamente,não está
fazendo nenhum bem à ciencia, pq não aporta mais nada. Um cientista de
verdade duvida sistemática, mas levanta hipoteses para os problemas, faz
testes, eleabora estruturas de pensamento que possam explicar o
ocurrido. Em primeiro lugar, testa de existe realmente um fenomeno a
esudar. Os ceticos não fazem isso, simplesmente duvidam e são contra qq
hipote de demonstracção da verdade cientifica.

> Olá Pubmed
>
> "Pubmed: No fundo , no fundo os céticos tem um pouquinho de cagaço que
> o manto seja mesmo de Jesus né?"
>
> Não, não temos..:-) Eu pelo menos não tenho nenhum..:-) Veja, se for
> de jesus, isso significa que existiu um cara chamado jesus, apenas
> isso.Não significa muito mais que isso, e isso apenas é algo a ser
> considerado e estudado, não significa que todos nós devemos nos tornar
> católicos ou cristãos..:-)

Vc está enganado, como de costume. É improvável que se possa provar que
o manto pertenceu , ou melhor, esteve em contacto com, Jesus. O mais
que se pode provar é que é possivel aceitar q o manto tenha estado no
local onde jesus foi morto, no tempo em que ele foi morto.
Por outro lado, se isso for provado, levantam-se várias questões. A
principal é: pq guardar o manto que envolveu alguem, se esse alguem não
foi ninguem inportante. Isso sim não faria sentido. As pessoas guardam
reliquias pelos mais diversos motivos, mas esses motivos existem.
Pessoas guardam pedras do muro de brelim. Tento provar que essas pedras
pertenceram a esse muro daqui a 2000 anos.
Ninguem acreditará em vc se não souberem da existencia historica de dito
muro. Ora, sabemos que os judeus tinham o cosutme de envolver os mortos
em panos, tal como os egipcios. Alias foi no egipto que aprenderam isso.
Mas tal costume desapreceu com o tempo. Porteriormente a 1300 DC não
existia mais esse costume. Razão pela qual não é credivel que o manto
tenha apenas essa idade.
Por outro lado, o manto viajou muito. Foram encontrados vestigios de
plantas que são especificas de certos locais geográficos, entre eles, o
local onde jusus terá sido supultado. Isso coloca o manto na região
geografica em causa, mas isso não significa que tenha estado lá quando
Jesus morreu.
O facto dele ter viajado, dele ter estado guardado durante a ideade
média - epoca em que as casas eram de madeira mais do que de pedra -num
local que posteriormente ardeu ( foi possivelmente contaminado com
cinzas ) , facto que o mutilou e que fez com que seus guardiões o
tentassem remendar devido ao efeito moral que quasaria vê-lo danificado
, explica perfeitamente pq os testes do carbono 14 foram inconclusivos.
Esta é uma palavra que os ceticos desocnhecem. Os testes são feitos,
tudo bem. Mas os resultados não são sagrados , como os ceticos
agreditam. Tudo bem que os testes podem até afirmar que o manto tem 1300
anos, mas o que isso significa para o estudo historico do manto ?
Comprova apenas que a historia do fogo que aconteceu no local onde
estava guardado pode ser veridica.

Facto muito mais interessante e que ninguem conseguiu explicar até agora
é a formação da imagem no manto. A imagem não é formada por sangue ou
outro produto , mas pelo que parece ser um especie de quimadura por
radiação. Como explicar a existencia dessa radiação , ainda para mais
quando ela prece ser emitida de dentro para fora ? Só o facto da
radiaçao existir já é um enigma. Como explicar a existencia daquela
radiação num tumulo de pedra solida ? Naquele tempo não existia nada que
podesse emitir radiação daquela, os raios-X são uma invensão modera e o
efeito tridimencional da imagem é um fenomeno da holografia apenas usado
a alguns anos. Explicar esse ponto é muito mais interessante do que
saber de quando é o manto. Seja de quando for ele é do passado, tempo no
qual não existia forma de produzir aquela radiação com aquela "assinatura".
Esse é o ponto onde os crentes podem entender que o manto pertenceu a
Jesus, já que suportamente ele se evadiu do tumulo por intermédio de um
anjo (ser de luz) transformando-se ele mesmo num ser do mesmo tipo, até
que apareceu depois aos apostulos. Isso não significa que ele seja filho
de Deus , mas dependendo de como se intrepreta "Deus" isso pode pelo
menos desacreditar o facto da resurreição ter sido um milagre. Notar,
que provar que aconteceu um facto tecnologico à 2000 anos, é provar que
algo importante aconteceu, independentemente de quem o fez, ou porquê.
Tudo isto para dizer que a sua frase "Veja, se for de jesus, isso
significa que existiu um cara chamado jesus, apenas isso." não está nem
perto da verdade.

>
> E eu tentei explicar, e talvez não tenha sido claro, porque céticos
> rejeitam estudos. É porque rejeitar estudos é parte fundamental do
> método cientifico..;-)

Apenas outros estudos podem refutar estudos preliminares e não a razão,
ou a imaginação das pessoas. É isso que os ceticos fazem constantemente,
e que sinceramente, já encheu o saco.

Sérgio Taborda


##### ##### #####

Para saber mais visite
http://www.ciencialist.hpg.ig.com.br


##### ##### ##### #####


Yahoo! Grupos, um serviço oferecido por:

São Paulo Rio de Janeiro Curitiba Porto Alegre Belo Horizonte Brasília




------------------------------------------------------------------------------
Links do Yahoo! Grupos

a.. Para visitar o site do seu grupo na web, acesse:
http://br.groups.yahoo.com/group/ciencialist/

b.. Para sair deste grupo, envie um e-mail para:
ciencialist-unsubscribe@yahoogrupos.com.br

c.. O uso que você faz do Yahoo! Grupos está sujeito aos Termos do Serviço do Yahoo!.



[As partes desta mensagem que não continham texto foram removidas]



SUBJECT: Se for comprovado que está errado, temos de mudar de opinião,
FROM: "Oraculo" <oraculo@atibaia.com.br>
TO: <ciencialist@yahoogrupos.com.br>
DATE: 20/03/2005 23:18

Olá Pubmed

Isso não é um jogo. Não é um Big Brother da Globo onde se procura imunidade. Isso é sobre a confiabilidade da informação produzida. Não é para "ganhar ou perder". Sua posição é infantil, de times de futebol, "céticos versus crentes".

Esta frase "Se for comprovado que está errado, temos de mudar de opinião,
mas se estiver certo permaneceremos nisso". , que você pensa ser uma fuga, uma forma de "escapar", é a sintese da ciência e do conhecimento cientifico. É um elogio, não uma crítica.

Pare de pensar como criança, e preste atenção na frase. Sem ela, ficamos todos como torcedores de futebol fanaticos, que mesmo quando seu time joga mal e perde, a culpa é do juiz, o time é o melhor do mundo e pronto.

Esqueça. Se uma afirmação, uma teoria, for refutada, tem de ser abandonada. Não se deve lealdade a ela, não é um time de futebol. Se for comprovada, se tiver mais e mais evidencais, deve ser mantida. Mesmo que não se goste dela, mesmo que demosntre algo que discorda de nós. A culpa não é do juiz.

A razão é justamente separar conhecimento confiável, real, da imaginação humana e erros e enganos. Isso é razão, e a ciência tem demonstrado sua eficácia nessa tarefa. E talvez seja essa eficácia que irrite tanto os que lutam contra ela, os "do outro time". Se apegar a fiapos de esperança, vibrar com as poucas oportunidades em que fica demonstrado um erro cientifico, esquecendo todos os incontáveis acertos e conquistas, tudo demonstra uma forte tendencia ao rancor.

Vive-se em um mundo cientifico, usa-se as conquistas da ciencia, se debate seobre ela em um ambiente virtual, global, só possivel a partir de conhecimento confiável, vive-se mais e melhor que todos os antepassados, os filhos já não morrem mais como antes, e , aidna assim, a ciência "é bobagem".

Engraçado..:-) Seria, inclusive, fácil abandonar toda essa "bobagem", ir viver na mata, ou em tribos afastadas, abdicar dessa tolice de ciência. Mas ninguém vai, ficam, vivendo com o melhor que ela produz, com a segurança que ela produz, mas continuam atacando o rigor, como sendo de "mentes fechadas".

Pubmed: O que é mais irritante nos céticos é o fato de que quando aparecem um
trbalho que os favorecem é muito bem recebido e acatado, mas quando
surge algo que nao corresponde as expectativas céticas a novidade
cientifica é acolhida com ceticismo. "

Mentira. Talvez não consciente, mas é uma mentira..:-) E está baseada em uma má utilização do termo "favorecer". O que seria "favorecer" a ciência? Que tipo de estudo "favorece" a ciência? Na verdade, todo estudo dentro do rigor e que apresenta resultados consistentes, favorece a ciência, como conhecimento.

Uma breve olhada na historia do conhecimento, demonstra que mesmo teorias estranhas, espantosas, francamente contra a crença de cada época, acabou aceita quando as evidencias e os estudos se somaram. Micróbios, deriva dos continentes, relatividade, e dezenas de outros exemplos, se estudados no contexto histórico de sua época, demonstram claramente isso. Foram recusados, como qualquer conhecimento novo, tiveram de se esforçar em produzir evidências, tiveram de resistir a críticas e ataques, os mais violentos possíveis, mas resistiram e acabaram sendo conhecimento confiável.

Toda afirmação, favoreça ou não, é recebida com ceticismo. Precisa ser, ou o rigor necessário desaparece. Essa afirmação é claramente incorreta, portanto.

Pubmed: Sinceramente meu ver, isso nao
pode ser chamado de ciencia. A ciencia é algo neutro, independente
das opiniões pessoais, e das técnicas de auto-defesa intelectual"

A seu ver. Uma opinião, todos tem direto a te-la. Mesmo as sem sentido. A ciência é o conjunto de conhecimento confiável produzido através do método e do rigor. O ceticismo, o cuidado antes de aceitar uma afirmação, é parte integrante do método. Não compreender isso, é não compreender a ciência.

Homero







----- Original Message -----
From: pubmed2005
To: ciencialist@yahoogrupos.com.br
Sent: Saturday, March 19, 2005 1:24 PM
Subject: [ciencialist] Re: Sudario



Excelente Taborda, e quando o fazem dão a desculpa que a ciencia
propoe correções, os paradigmas mudam e aí eles livram a cara deles.
Se voce observar direitinho o cético é sempre imune. Aquele argumento
tipo: "Se for comprovado que está errado, temos de mudar de opinião,
mas se estiver certo permaneceremos nisso". Aí o cético fica numa
situação de imunidade. Quer dizer, ele nunca está certo , nem errado.
Vai depender das descobertas científicas . Então onde fica a razão
nisso?

Só que o que muda é a ciencia, os céticos vivem ao sabor disso. A
ciencia contém o ceticismo, mas o ceticismo nao abrange a ciencia(ele
é apenas parte dela) É preciso muito cuidado e pensamento acurado
para nao se cair nesta armadilha

O que é mais irritante nos céticos é o fato de que quando aparecem um
trbalho que os favorecem é muito bem recebido e acatado, mas quando
surge algo que nao corresponde as expectativas céticas a novidade
cientifica é acolhida com ceticismo. Sinceramente meu ver, isso nao
pode ser chamado de ciencia. A ciencia é algo neutro, independente
das opiniões pessoais, e das técnicas de auto-defesa intelectual


--- Em ciencialist@yahoogrupos.com.br, "Sergio M. M. Taborda"
<sergiotaborda@t...> escreveu
> Quando se acredita demasiado na duvida, nada mais faz sentido.
> Uma coisa que nunca entendi é pq o cetico não duvida do que diz. Pq
ele
> nunca acha que está enganado ?
>
> Sérgio Taborda





##### ##### #####

Para saber mais visite
http://www.ciencialist.hpg.ig.com.br


##### ##### ##### #####


Yahoo! Grupos, um serviço oferecido por:
PUBLICIDADE




------------------------------------------------------------------------------
Links do Yahoo! Grupos

a.. Para visitar o site do seu grupo na web, acesse:
http://br.groups.yahoo.com/group/ciencialist/

b.. Para sair deste grupo, envie um e-mail para:
ciencialist-unsubscribe@yahoogrupos.com.br

c.. O uso que você faz do Yahoo! Grupos está sujeito aos Termos do Serviço do Yahoo!.



[As partes desta mensagem que não continham texto foram removidas]



SUBJECT: Re: Se for comprovado que está errado, temos de mudar de opinião,
FROM: "junior_br2001" <junior_br2001@yahoo.com.br>
TO: ciencialist@yahoogrupos.com.br
DATE: 20/03/2005 23:22


--- Em ciencialist@yahoogrupos.com.br, "Oraculo" <oraculo@a...>
escreveu
> Olá Pubmed
>> Pare de pensar como criança, e preste atenção na frase


JR: Creio que está frase sim é que um ad hominem.





SUBJECT: Onus da prova (era Sudario)
FROM: "Oraculo" <oraculo@atibaia.com.br>
TO: <ciencialist@yahoogrupos.com.br>
DATE: 20/03/2005 23:26

Olá Sergio

Eles não precisam provar, quem precisa é quem alega. Apesar disso, até poderiam tentar provar sua alegação, se a igreja, proprietária, não estivesse tão decidida a manter o misterio e impedir o estudo do artefato em questão.

Imaginemos um antrolpologo que afirme ter encontrado um esqueleto de "homo sapiens" em camadas geológicas do periodo cretáceo. Isso demoliria um edifício de conhecimento arqueológico, revolucionaria a ciência, seria uma descoberta fantastica, seria como descobrir o coelho cambriano, que os criacionistas tem procuradao tão ardentemente..:-). Mas, o descobridor não parmite exames. Afirma que é como diz, mas só é possível ver poucos elementos, fiapos de evidencia, pequenos e minúsculos pedaços, talvez fotos desfocadas.

E a maioria dos que tem autorzação para examinar, são defensores da alegação deste antropologo.

Seria necessário provar que os ossos são fraudes? Deveriamos acreditar no descobridor, apenas por que ele assim afirma? E, o principal, o que esse caso hipotético difere do alegado santo sudário, guardado pela igreja, misteriosamente criado, com imagens fantasticamente imprimidadas em seu tecido, que a ciência e laboratórios especializados não podem estudar?

Covardia é pensar que é obrigação de cada cético ou cientista passar a vida a demolir afirmações fantasticas, sem nem sequer ter a chance de analisar os artefatos envolvidos.

Se a igreja provasse a autenticidade do santo sudário, nem haveria esta discussão.

Homero


----- Original Message -----
From: Sergio M. M. Taborda
To: ciencialist@yahoogrupos.com.br
Sent: Saturday, March 19, 2005 2:04 PM
Subject: Re: [ciencialist] Sudario


Eduardo Gueron wrote:

> Sou cético e creio (crer?) que o ceticismo é
>necessário para o cientista. Tentar desqualificá-lo
>por causa de bobagens às vezes faladas em fóruns
>céticos é, no mínimo, covardia.
>
Covardia é isto aqui:
" É também alegado que a imagem não é uma pintura, mas uma imagem
milagrosamente <http://brazil.skepdic.com/milagres.html> transposta.
Os céticos discordam e argumentam que o sudário é uma pintura e uma
falsificação".
in http://brazil.skepdic.com/sudario.html

Eles argumentam muito, mas não provam nada do que dizem. Se provassem,
não existiria discussão.

Sérgio Taborda



##### ##### #####

Para saber mais visite
http://www.ciencialist.hpg.ig.com.br


##### ##### ##### #####


Yahoo! Grupos, um serviço oferecido por:







------------------------------------------------------------------------------
Links do Yahoo! Grupos

a.. Para visitar o site do seu grupo na web, acesse:
http://br.groups.yahoo.com/group/ciencialist/

b.. Para sair deste grupo, envie um e-mail para:
ciencialist-unsubscribe@yahoogrupos.com.br

c.. O uso que você faz do Yahoo! Grupos está sujeito aos Termos do Serviço do Yahoo!.



[As partes desta mensagem que não continham texto foram removidas]



SUBJECT: Re: Sudario
FROM: "junior_br2001" <junior_br2001@yahoo.com.br>
TO: ciencialist@yahoogrupos.com.br
DATE: 20/03/2005 23:32


Essa discussão já evoluiu(voce nao deve ter lido as mensagens
anteriores) e já foi mostrado exames químicos que foram feitos em
universidades diferentes. Nem vou mais adentrar nisso

Além temos a confirmação de um físico como o Taborda a respeito da
falibilidade e falhas do carbono-14 com as antigas técnicas de 88
aplicadas sobre remendos do manto.

Além, disso apresentei os experimentos do Valdez e dos sovieticos
com datação do carbono-14 e suas possíveis falhas caso haja vestígios
de bioplastia, e contaminação de carbono-14 recente no linho.
Se quiser leia todas as mensagens anteriores e localize que elas
foram postadas por mim
O Takata apresentou um estudo dele, mas que foi criticado pelo
Taborda. Mas o ele só mostrou o cálculo e nao o teste pratico-
experimental. Além do mais, ele nao é a pessoa mais adequada para
isso, porque nao é a especialidade dele(nem em datação). Mas é claro,
isso nao impede que eu, ele o outros realize os calculos.

O problema é que o lado cético nao apresentou os experimentos que foi
pedido

Jr



--- Em ciencialist@yahoogrupos.com.br, "Oraculo" <oraculo@a...>
escreveu
> Olá Pubmed
>
> Pubmed: Então me diz no que o conhecimento cientifico do químico
está errado
> e no que ele está sendo incorreto. Não acho que o trabalho de
Rogers
> seja tão ruim. Ele levtou questoes muito interessantes que merecem
> uma verificação. Se voce tem coisa melhor em mente apresente a
> comunidade científica"
>
> Embora não pense que vai adiantar, vou tentar de novo: este estudo
precisa ser confirmado por pesquisadores independentes. Isso é
absolutametne necessário, devido ao rigor do método, para dar
confiabilidade aos resultados. Até lá, os estudos com confirmação
independente, tem preferencia.






SUBJECT: Espantalho (era Sudario
FROM: "Oraculo" <oraculo@atibaia.com.br>
TO: <ciencialist@yahoogrupos.com.br>
DATE: 20/03/2005 23:32

Olá Pubmed

Pubmed:De um lado os céticos que se dizem adeptos da ciencia que alegam que
só o santo carbono-14 é infalível e de outro uns( eu ou alguns) que
defende a apenas a posição da ciencia que outros testes tb são

Falácia do espantalho. Se o Anderson se der ao trabalho de reler as mensagens, em nenhuma delas vai encontrar algo sequer parecido com essa afirmação. O carbono-14 é um bom e confiável método de datação, apenas isso. Não é santo, não é infalível, nem é a base da discussão.

Tanto assim, que foram necessários mais estudos de datação por carbono-14, por pesquisadores independentes, para que fosse aceito como confiável no caso do sudário. Isso desmente claramente a afirmação (espantalho) que fez acima.

A discussão é, e sempre foi, que o novo estudo, que contesta os anteriores, precisa esperar estudos de confirmação, como qualquer estudo cientifico.

Tentar transformar a discussão em gozação pode ser divertido, mas fica feio se o leitor resolver reler as mensagens e perceber a tentativa de induzir a erro.

Você não é um paladino da verdade defendendo , heroicamente, que testes além do carbono-14 devem ser feitos. Está apenas tentando fugir da questão principal do debate, argumento irrespondivel, que foi apresentado: até que estudos independentes confirmem este novo estudo, os anteriores tem maior confiabilidade. Responde a esse argumento, ao invés de tentar confundir que nos lê.

Homero


----- Original Message -----
From: pubmed2005
To: ciencialist@yahoogrupos.com.br
Sent: Saturday, March 19, 2005 3:00 PM
Subject: [ciencialist] Re: Sudario



Oi anderson, ainda nao vi nenhuma posição religiosa por aqui. O que
se está discutindo são posições científicas.
De um lado os céticos que se dizem adeptos da ciencia que alegam que
só o santo carbono-14 é infalível e de outro uns( eu ou alguns) que
defende a apenas a posição da ciencia que outros testes tb são
importantes e nao precisam necessariamente invalidar o outro. Tanto
que no estudo químico a idade *mínima* para o manto ainda se manteve
a detectadas pelo carbono-14, embora estipula uma idade máxima de
3000 anos. Ela não anula necessariamente o primeiro estudo, mas dá
uma dimensão maior a descoberta. E abre caminhos para novos estudos


-- Em ciencialist@yahoogrupos.com.br, Anderson Almeida da Silveira
<anderson_a5@y...> escreveu
>
> PRA MIM A DISCURÇÃO SOBRE O SUDÁRIO É INÚTIL. NIGUÉM VIU O ROSTO DE
JESUS E PORTANTO NÃO HÁ COMO SABER SE ELE O USOU OU NÃO.
>
> __________________________________________________
> Converse com seus amigos em tempo real com o Yahoo! Messenger
> http://br.download.yahoo.com/messenger/
>
> [As partes desta mensagem que não continham texto foram removidas]





##### ##### #####

Para saber mais visite
http://www.ciencialist.hpg.ig.com.br


##### ##### ##### #####


Yahoo! Grupos, um serviço oferecido por:







------------------------------------------------------------------------------
Links do Yahoo! Grupos

a.. Para visitar o site do seu grupo na web, acesse:
http://br.groups.yahoo.com/group/ciencialist/

b.. Para sair deste grupo, envie um e-mail para:
ciencialist-unsubscribe@yahoogrupos.com.br

c.. O uso que você faz do Yahoo! Grupos está sujeito aos Termos do Serviço do Yahoo!.



[As partes desta mensagem que não continham texto foram removidas]



SUBJECT: Re: [ciencialist] Re: Sudario
FROM: "Oraculo" <oraculo@atibaia.com.br>
TO: <ciencialist@yahoogrupos.com.br>
DATE: 20/03/2005 23:39

Olá Pubmed

Pubmed: ORA BOLAS, QUEM ESTÁ FALANDO DA IMAGEM DE JESUS?

Está por fora mesmo. A igreja, detentora do santo sudário, está falando de jesus. Toda a questão está falando de jesus. A imagem, milagrosa, deve ser de jesus. O sudário é santo, porque pertenceu e cobriu o corpo de jesus.

Por isso se chama de santo esse pano, e de sudário. E por isso em toda a rede, nos livros e nas revistas, cristãos e catolicos usam o santo saudario como evidencia de sua existencia real (que, afinal, tem poucos e inconclusivos sinais de ser real).

A imagem, de acordo com os defensores da santidade do sudario, só poderia ter sido feita de forma milagrosa, por não existirem técnicas, nem hoje em dia, que permitissem que se formasse.

Tudo isso é parte integrante do santo sudário e da discussão sobre ele. Sua data de origem, que é um aspecto acessório, deveria apenas decidir se seria possível que fosse contemporâneo de jesus. Sua data de fabricação tem interessa justamente por esse motivo, e o carbono-14 é tão contestado por catolicosa e cristãos exatametne por impedir que essa contemporaneidade seja real. Refutar os testes de carbono-14 feitos no sudário é uma das maiores esperanças da cristandade, principalmente a que precisa de evidencias para manter sua crença em cristo.

Homero


----- Original Message -----
From: pubmed2005
To: ciencialist@yahoogrupos.com.br
Sent: Saturday, March 19, 2005 4:08 PM
Subject: [ciencialist] Re: Sudario



O que é interessante nestes sites céticos como o
http://brazil.skepdic.com/milagres.html é como eles mudam o discurso
após uma descoberta científica, mesmo continuando a questiona-la

Vejamos, eles dizem: <<<"Naturalmente, o tecido poderia ter 3.000 ou
2.000 anos de idade, como especula Rogers, mas a imagem nele poderia
datar de um período muito posterior. Qualquer que seja a data
correta, tanto para o pano quando para a figura, não prova em nenhum
grau de probabilidade razoável que o tecido seja o sudário em que
Jesus foi envolvido e que a imagem seja de alguma forma milagrosa.
Acreditar nisso sempre será uma questão de fé, não de provas
científicas.">>>

ORA BOLAS, QUEM ESTÁ FALANDO DA IMAGEM DE JESUS?
Estamos aventando apenas a possibilidade do tecido ter 3000 anos...
dane-se a imagem
Não entendo como os céticos só se dirigem aos religiosos como na
mensagem acima. Nem todo mundo tem posições religiosas, muitos tem
interesse arqueológico e historico pela peça de tecido chamado "santo
sudário"




-- Em ciencialist@yahoogrupos.com.br, Anderson Almeida da Silveira
<anderson_a5@y...> escreveu
<http://brazil.skepdic.com/milagres.html> transposta.
> Os céticos discordam e argumentam que o sudário é uma pintura e uma
> falsificação".
]





##### ##### #####

Para saber mais visite
http://www.ciencialist.hpg.ig.com.br


##### ##### ##### #####


Yahoo! Grupos, um serviço oferecido por:

São Paulo Rio de Janeiro Curitiba Porto Alegre Belo Horizonte Brasília




------------------------------------------------------------------------------
Links do Yahoo! Grupos

a.. Para visitar o site do seu grupo na web, acesse:
http://br.groups.yahoo.com/group/ciencialist/

b.. Para sair deste grupo, envie um e-mail para:
ciencialist-unsubscribe@yahoogrupos.com.br

c.. O uso que você faz do Yahoo! Grupos está sujeito aos Termos do Serviço do Yahoo!.



[As partes desta mensagem que não continham texto foram removidas]



SUBJECT: Re: [ciencialist] Re: Santo sudário e a vanilina(correção)
FROM: "Oraculo" <oraculo@atibaia.com.br>
TO: <ciencialist@yahoogrupos.com.br>
DATE: 20/03/2005 23:44

Olá Pubmed

Pubmed: *Não se contradizem, o mínimo estipulado do método B é de 1300*. O
máximo que estipulado que é 3000

Faça as contas. Talvez esteja confundino (e talvez não tenha lido a explicação nas outras mensagens) com a data de existencia e a data de criação. O carbono diz que existe a 660 anos, o que significa que foi criado em 1340. A quimica diz que tem entre 1300 e 3000 anos, o que significa que foi craido entre 700 DC e 2000 AC.

Faça as contas. Ou apenas compare as datas. Um tecido criado no máximo até o ano 700 depois de cristo não pode ter sido criado depois de 1340 depois de cristo. É matematica simples.

O 1300 do metodo B não é o ano de criação, mas o tempo de existencia. Ele existe a pelo menos 1300 anos, ou seja, foi criado até o ano 700, não até o ano 1340.

Homero


----- Original Message -----
From: pubmed2005
To: ciencialist@yahoogrupos.com.br
Sent: Saturday, March 19, 2005 9:34 PM
Subject: [ciencialist] Re: Santo sudário e a vanilina(correção)



Lá vai

--- Em ciencialist@yahoogrupos.com.br, "rmtakata" <rmtakata@a...>
escreveu
>
> Eh disso q se trata. O metodo A diz q. o tecido eh de 1300, o
metodo B
> diz q. o tecido eh bem anterior a isso - talvez de 40. Os metodos se
> contradizem. Entao eh questao de ver qual o mais confiavel.

*Não se contradizem, o mínimo estipulado do método B é de 1300*. O
máximo que estipulado que é 3000

>
> > Mas que novas descobertas de peso foram surgiram que podem
> > mudar o padrão estipulado anteriormente.
>
> Claro q. se houver novas descobertas de peso q. contradigam a idade
> apontada pelo C-14 serah preciso rever.

Já está havendo revisão. A nova pesquisa foi publicada na Nature

>
> Lanca a possibilidade do metodo da analise quimica nao ser tao
> confiavel. Sabemos q. o decaimento radioativo nao eh alterado por
> pressao, temperatura ou qq outro fator ambiental conhecido. Jah a
taxa
> de reacoes quimicas variam enormente de acordo com a temperatura (e
> sabemos q. o tecido em questao pegou fogo), presenca de umidade,
> exposicao a luz, presenca de bacterias e fungos...

Não seja tão certo da confiabilidade. de uma lida:

O Dr Leôncio Garza Valdes, médico e professor da Universidade do
Texas, pesquisou e descobriu resíduos orgânicos gerados por bactérias
em múmias. Ao tentar data-las pelo processo de radiocarbono descobriu
que esses resíduos, que formavam um revestimento bio-plástico sobre
os fios do tecido, alteravam o resultado da datação, devido à
impossibilidade de se limpá-los de modo adequado.

Em 1993, O Dr.Valdes conseguiu algumas amostras do Sudário que haviam
sido cortadas na mesma ocasião do teste de rádiocarbono de 1988,
junto ao Professor Giovani Riggi, o microanalista de Turim já
mencionado, guardião das amostras, e homem de confiança do Cardeal
Ballestrero.

Ao analisá-las descobriu a presença do revestimento bio-plástico
(Lichenothelia Varnish) sobre os fios de linho, assim como a presença
desses organismos (Lichenothelia) vivos.



Aliás, o Dr Valdes não conseguiu fazer o teste de rádiocarbono
exatamente pela dificuldade em se fazer a limpeza. Ele chegou a
utilizar exatamente o mesmo processo de limpeza que foi utilizado
pelos laboratórios do teste de 1988, e constatou que o revestimento
bio-plástico não saia.

Segundo o Dr. Valdes: &#8220;A pátina causada pelos fungos é um acréscimo
natural nas superfícies antigas estáveis. São necessárias centenas de
anos para que os fungos criem um &#8216;verniz&#8217; contínuo. As fibras do
Sudário de Turim têm um depósito espesso devido às bactérias. A data
obtida mediante o C14 em 1988 deve-se na realidade, a uma mistura do
C14 do Tecido do Sudário com o C14 da pátina de bactérias. Este seria
de até 60%&#8221;.

À luz desses estudos, o inventor do processo de datação por
rádiocarbono pelo processo usado no Sudário, Harry Gove, teve de
declarar em 27 de Janeiro de 1995: &#8220;A técnica que se usou em 1988
para a datação do Sudário de Turim por meio do C14 foi inventada em
meu laboratório, na Universidade de Rochester, em 1977. Depois dessa
datação, estive convencido do resultado durante anos. Recentemente,
porém, o doutor Garza Valdés, de San Antonio, Texas, apresentou
provas consistentes a respeito de um tipo de contaminação por carbono
recente produzida nos fios do Sudário por bactérias que os processos
de limpeza usados pelos três laboratórios podem não ter removido.
Essa contaminação, de acordo com sua espessura, pode fazer com que a
data fornecida pelos três laboratórios seja mais recente&#8221;.

3.5- A contestação de Kouznetsov

O diretor dos Laboratórios E.A. Sedoo de Pesquisa de Bio-Polímeros de
Moscou e premio Lênin de Ciência, Dr. A. Kouznetsov, participou de um
congresso em Londres onde foi convidado a testar seus métodos de
datação por radiocarbono pelo Geólogo Dr Guy Berthault, para comparar
com os resultados do exame do Sudário.

Analisando a história do Sudário, descobriu que ele passou por um
incêndio em 1532. Resolveu recriar a condição em laboratório, e
descobriu que o linho absorveu C14 recente, principalmente por ter se
submetido a altas temperaturas por tempos suficientemente longos na
presença de produtos da combustão (água, anidrido carbônico e óxido
de carbono) e íons de prata, pois a urna era de prata, capazes de
agir como catalisadores.

Para comprovar isso, Kouznetsov pegou um tecido de linho do séc. I,
procedente de Em Gedi (Israel), e enviou ao Laboratório de Tucson,
Arizona, onde fizeram a datação.

Mais tarde, o professor russo queimou outro pedaço do mesmo lençol,
simulando a condição do Sudário no incêndio, e enviou novamente para
Tucson. O resultado desta datação foi de 1300 anos mais recente.

.6- Conclusão

Conforme o Dr Valdés e o Dr Kouznetsov, não se aplica testes de
rádiocarbono a matérias têxteis como o linho, devido aos seus altos
níveis de contaminação sem possibilidade prática de remoção.

> Como eu disse a questao eh de se mostrar q. a metodologia de datacao
> quimica eh mais confiavel do q. o de radioisotopos.

Se encontro uma prova nova, e ela me fornece evidencias novas o
quadro tem de mudar, e lá se vai a confiabilidade. E é isso que está
acontecendo. O que parece que é, nao é

>
> (Poderiamos analisar a questao sob a optica inversa: se vcs *querem*
> aceitar essas pesquisas...)


Takata , algumas coisa temos que aceitar: assim como aceitamos a
datação do carbono-14. Nao podemos ter o santo sudário nas maos para
testá-los
Só que a ciencia avança , e o teste é bem antiguinho e provavelmente
deve estar bem defasado. De qualquer maneira muitos cientistas tem de
acreditar nisso - ter confiança peloe menos em algum grau E tem
aqueles que questionam e apresentam provas que colidem com as
anteriores, enfim o mundo não para.















##### ##### #####

Para saber mais visite
http://www.ciencialist.hpg.ig.com.br


##### ##### ##### #####


Yahoo! Grupos, um serviço oferecido por:
PUBLICIDADE




------------------------------------------------------------------------------
Links do Yahoo! Grupos

a.. Para visitar o site do seu grupo na web, acesse:
http://br.groups.yahoo.com/group/ciencialist/

b.. Para sair deste grupo, envie um e-mail para:
ciencialist-unsubscribe@yahoogrupos.com.br

c.. O uso que você faz do Yahoo! Grupos está sujeito aos Termos do Serviço do Yahoo!.



[As partes desta mensagem que não continham texto foram removidas]



SUBJECT: Re: Espantalho (era Sudario
FROM: "junior_br2001" <junior_br2001@yahoo.com.br>
TO: ciencialist@yahoogrupos.com.br
DATE: 20/03/2005 23:48


Falácia espantalho? De onde tirou esse palavrão? Dos livros de Sagan?

A datação de carbono-14 feita em 88 nem se compara com as técnicas
atuais. Portanto, se isso deve serve então de *espantalho*(para
espantar logo de uma vez) essa idéia equivocada de que o carbono-14 é
um método confiável(pelo menos naquela época), e ainda tem os
impedimentos que falei anteriormente(revestimento bioplastico,
contaminação do tecido e teste realizado em remendos recentes
conforme as confirmações laboratoriais realizadas nas fibras dos
tecidos mostraram).

Mesmo que voce esteja falando da época atual, nao existe um método de
datação 100%% infalível. Aliás voce que é cético deveria saber que a
ciencia , seus métodos, hipóteses e teoriais nao são tão confiáveis
como parece ser

JR


--- Em ciencialist@yahoogrupos.com.br, "Oraculo" <oraculo@a...>
escreveu
> Olá Pubmed
>
> Pubmed:De um lado os céticos que se dizem adeptos da ciencia que
alegam que
> só o santo carbono-14 é infalível e de outro uns( eu ou alguns) que
> defende a apenas a posição da ciencia que outros testes tb são
>
> Falácia do espantalho. Se o Anderson se der ao trabalho de reler as
mensagens, em nenhuma delas vai encontrar algo sequer parecido com
essa afirmação. O carbono-14 é um bom e confiável método de datação,
apenas isso. Não é santo, não é infalível, nem é a base da discussão.
>





SUBJECT: Re: Sudario
FROM: "junior_br2001" <junior_br2001@yahoo.com.br>
TO: ciencialist@yahoogrupos.com.br
DATE: 20/03/2005 23:50


É, mais a Igreja católica e nem seus representantes nao está nesta
discussão aqui.

JR

--- Em ciencialist@yahoogrupos.com.br, "Oraculo" <oraculo@a...>
escreveu
> Olá Pubmed
>
> Pubmed: ORA BOLAS, QUEM ESTÁ FALANDO DA IMAGEM DE JESUS?
>
> Está por fora mesmo. A igreja, detentora do santo sudário, está
falando de jesus.





SUBJECT: Confiabilidade (era Santo sudário e a vanilina(correção)
FROM: "Oraculo" <oraculo@atibaia.com.br>
TO: <ciencialist@yahoogrupos.com.br>
DATE: 20/03/2005 23:51

Olá Alberto

Alberto:mesmo porque não há como assumir uma postura científica
deixando-se o ceticismo totalmente de lado."

Tente explicar isso a seu discipulo, o Pubmed..:-)

E a confiabilidade neste caso é simplesmente uma questão de possibilidade de ser repetido e encontrar os mesmos resultados. Os testes de carbono-14, mais de um, passaram nesta exigencia e, por enquanto, são confiáveis, até onde um conhecimento cientifico pode ser confiável. Isto é, não uma certeza de 100%, mas uma probabilidade forte, e a espera por refutações pelo menos tão solidas quanto o estudo em questão.

O segundo estudo espera por confirmação independente, estudos que, utilizando a mesma metodologia, (ou mais rigorosa, se necessário) apresentem os mesmos resultados. Sem isso, um dos estudos tem mais confiabilidade que o outro. E isso serve para qualquer situação, e não apenas, como acusou o Pubmed, para coisas que "favorecem" a ciência (o que quer que isso signifique).

Por exemplo, entre um medicamente que, após passar por diversos estudos de controle e repetição, recebeu aval para ser aplicado contre determinada patologia, e um novo medicamente que, apesar de ter apresentado efeitos melhores que o anterior em um estudo de pesquisa, ainda espera por estudos de repetição, seria mais seguro e "confiável", receitar o primeiro medicamento. Ou não?

Um abraço.

Homero



----- Original Message -----
From: Alberto Mesquita Filho
To: ciencialist@yahoogrupos.com.br
Sent: Saturday, March 19, 2005 10:55 PM
Subject: Re: [ciencialist] Re: Santo sudário e a vanilina(correção)


----- Original Message -----
From: "rmtakata"
Sent: Saturday, March 19, 2005 5:40 PM
Subject: [ciencialist] Re: Santo sudário e a vanilina(correção)

> > Poder dizer eles podem. O que eles não podem é tentar nos
> > convencer que estão dizendo a verdade

> Sim, poderiamos dizer. Mas eu nao disse (e ao q. eu saiba nem o
> Oraculo/Homero). O q. eu disse e digo novamente eh q. ainda estah para se
> demonstrar q. datacao quimica eh mais confiavel do q. datacao por
> radioisotopos.

Mas... Com esse corte a piada perdeu a graça. Então vou entrar na sua,
vejamos:

1) O que significa ser mais confiável?
2) Há como quantificar essa confiabilidade?
Se sim:
3) Seria possível traduzir em termos probabilísticos?
4) Eu poderia dizer que o método X é Z% mais confiável que o método Y?
Se não:
5) Este ser confiável seria do tipo ter sido mais vezes corroborado?
Ou então:
6) Seria o método X potencialmente gerador de hipóteses falseáveis em maior
número que o método Y?
7) Haveria como objetivar essa diferença?

Perceba que ser cético é muito fácil. O difícil é ser cientista e ter que
aturar esse "ceticismo insosso" que contamina a Internet. Nada contra um
ceticismo sadio, mesmo porque não há como assumir uma postura científica
deixando-se o ceticismo totalmente de lado.

PS: Insosso está sendo aqui empregado no sentido de algo que não leva a
nada. A esse respeito concordo plenamente com as queixas do Sérgio em
especial com a conclusão final, a de que a postura de vocês "já encheu o
saco".

[ ]´s
Alberto
http://ecientificocultural.com/indice.htm
Mas indiferentemente a tudo isso, o neutrino tem massa, o elétron não é
uma carga elétrica coulombiana e a Terra se move. E a história se repetirá.



##### ##### #####

Para saber mais visite
http://www.ciencialist.hpg.ig.com.br


##### ##### ##### #####


Yahoo! Grupos, um serviço oferecido por:







------------------------------------------------------------------------------
Links do Yahoo! Grupos

a.. Para visitar o site do seu grupo na web, acesse:
http://br.groups.yahoo.com/group/ciencialist/

b.. Para sair deste grupo, envie um e-mail para:
ciencialist-unsubscribe@yahoogrupos.com.br

c.. O uso que você faz do Yahoo! Grupos está sujeito aos Termos do Serviço do Yahoo!.



[As partes desta mensagem que não continham texto foram removidas]



SUBJECT: Re: Santo sudário e a vanilina(correção)
FROM: "junior_br2001" <junior_br2001@yahoo.com.br>
TO: ciencialist@yahoogrupos.com.br
DATE: 20/03/2005 23:52


Deixo esses cálculos pro Taborda responder(se ele quiser é claro),
ele deve te explicar isso melhor que eu

JR

--- Em ciencialist@yahoogrupos.com.br, "Oraculo" <oraculo@a...>
escreveu
> Olá Pubmed
>
> Pubmed: *Não se contradizem, o mínimo estipulado do método B é de
1300*. O
> máximo que estipulado que é 3000
>
> Faça as contas. Talvez esteja confundino (e talvez não tenha lido a
explicação nas outras mensagens) com a data de existencia e a data de
criação. O carbono diz que existe a 660 anos, o que significa que foi
criado em 1340. A quimica diz que tem entre 1300 e 3000 anos, o que
significa que foi craido entre 700 DC e 2000 AC.
>
> Faça as contas. Ou apenas compare as datas. Um tecido criado no
máximo até o ano 700 depois de cristo não pode ter sido criado depois
de 1340 depois de cristo. É matematica simples.
>
> O 1300 do metodo B não é o ano de criação, mas o tempo de
existencia. Ele existe a pelo menos 1300 anos, ou seja, foi criado
até o ano 700, não até o ano 1340.
>
> Homero
>
>
> ----- Original Message -----
> From: pubmed2005
> To: ciencialist@yahoogrupos.com.br
> Sent: Saturday, March 19, 2005 9:34 PM
> Subject: [ciencialist] Re: Santo sudário e a vanilina(correção)
>
>
>
> Lá vai
>
> --- Em ciencialist@yahoogrupos.com.br, "rmtakata" <rmtakata@a...>
> escreveu
> >
> > Eh disso q se trata. O metodo A diz q. o tecido eh de 1300, o
> metodo B
> > diz q. o tecido eh bem anterior a isso - talvez de 40. Os
metodos se
> > contradizem. Entao eh questao de ver qual o mais confiavel.
>
> *Não se contradizem, o mínimo estipulado do método B é de 1300*.
O
> máximo que estipulado que é 3000
>
> >
> > > Mas que novas descobertas de peso foram surgiram que podem
> > > mudar o padrão estipulado anteriormente.
> >
> > Claro q. se houver novas descobertas de peso q. contradigam a
idade
> > apontada pelo C-14 serah preciso rever.
>
> Já está havendo revisão. A nova pesquisa foi publicada na Nature
>
> >
> > Lanca a possibilidade do metodo da analise quimica nao ser tao
> > confiavel. Sabemos q. o decaimento radioativo nao eh alterado
por
> > pressao, temperatura ou qq outro fator ambiental conhecido. Jah
a
> taxa
> > de reacoes quimicas variam enormente de acordo com a
temperatura (e
> > sabemos q. o tecido em questao pegou fogo), presenca de umidade,
> > exposicao a luz, presenca de bacterias e fungos...
>
> Não seja tão certo da confiabilidade. de uma lida:
>
> O Dr Leôncio Garza Valdes, médico e professor da Universidade do
> Texas, pesquisou e descobriu resíduos orgânicos gerados por
bactérias
> em múmias. Ao tentar data-las pelo processo de radiocarbono
descobriu
> que esses resíduos, que formavam um revestimento bio-plástico
sobre
> os fios do tecido, alteravam o resultado da datação, devido à
> impossibilidade de se limpá-los de modo adequado.
>
> Em 1993, O Dr.Valdes conseguiu algumas amostras do Sudário que
haviam
> sido cortadas na mesma ocasião do teste de rádiocarbono de 1988,
> junto ao Professor Giovani Riggi, o microanalista de Turim já
> mencionado, guardião das amostras, e homem de confiança do
Cardeal
> Ballestrero.
>
> Ao analisá-las descobriu a presença do revestimento bio-plástico
> (Lichenothelia Varnish) sobre os fios de linho, assim como a
presença
> desses organismos (Lichenothelia) vivos.
>
>
>
> Aliás, o Dr Valdes não conseguiu fazer o teste de rádiocarbono
> exatamente pela dificuldade em se fazer a limpeza. Ele chegou a
> utilizar exatamente o mesmo processo de limpeza que foi utilizado
> pelos laboratórios do teste de 1988, e constatou que o
revestimento
> bio-plástico não saia.
>
> Segundo o Dr. Valdes: &#8220;A pátina causada pelos fungos é um
acréscimo
> natural nas superfícies antigas estáveis. São necessárias
centenas de
> anos para que os fungos criem um &#8216;verniz&#8217; contínuo.
As fibras do
> Sudário de Turim têm um depósito espesso devido às bactérias. A
data
> obtida mediante o C14 em 1988 deve-se na realidade, a uma mistura
do
> C14 do Tecido do Sudário com o C14 da pátina de bactérias. Este
seria
> de até 60%&#8221;.
>
> À luz desses estudos, o inventor do processo de datação por
> rádiocarbono pelo processo usado no Sudário, Harry Gove, teve de
> declarar em 27 de Janeiro de 1995: &#8220;A técnica que se usou
em 1988
> para a datação do Sudário de Turim por meio do C14 foi inventada
em
> meu laboratório, na Universidade de Rochester, em 1977. Depois
dessa
> datação, estive convencido do resultado durante anos.
Recentemente,
> porém, o doutor Garza Valdés, de San Antonio, Texas, apresentou
> provas consistentes a respeito de um tipo de contaminação por
carbono
> recente produzida nos fios do Sudário por bactérias que os
processos
> de limpeza usados pelos três laboratórios podem não ter removido.
> Essa contaminação, de acordo com sua espessura, pode fazer com
que a
> data fornecida pelos três laboratórios seja mais recente&#8221;.
>
> 3.5- A contestação de Kouznetsov
>
> O diretor dos Laboratórios E.A. Sedoo de Pesquisa de Bio-
Polímeros de
> Moscou e premio Lênin de Ciência, Dr. A. Kouznetsov, participou
de um
> congresso em Londres onde foi convidado a testar seus métodos de
> datação por radiocarbono pelo Geólogo Dr Guy Berthault, para
comparar
> com os resultados do exame do Sudário.
>
> Analisando a história do Sudário, descobriu que ele passou por um
> incêndio em 1532. Resolveu recriar a condição em laboratório, e
> descobriu que o linho absorveu C14 recente, principalmente por
ter se
> submetido a altas temperaturas por tempos suficientemente longos
na
> presença de produtos da combustão (água, anidrido carbônico e
óxido
> de carbono) e íons de prata, pois a urna era de prata, capazes de
> agir como catalisadores.
>
> Para comprovar isso, Kouznetsov pegou um tecido de linho do séc.
I,
> procedente de Em Gedi (Israel), e enviou ao Laboratório de
Tucson,
> Arizona, onde fizeram a datação.
>
> Mais tarde, o professor russo queimou outro pedaço do mesmo
lençol,
> simulando a condição do Sudário no incêndio, e enviou novamente
para
> Tucson. O resultado desta datação foi de 1300 anos mais recente.
>
> .6- Conclusão
>
> Conforme o Dr Valdés e o Dr Kouznetsov, não se aplica testes de
> rádiocarbono a matérias têxteis como o linho, devido aos seus
altos
> níveis de contaminação sem possibilidade prática de remoção.
>
> > Como eu disse a questao eh de se mostrar q. a metodologia de
datacao
> > quimica eh mais confiavel do q. o de radioisotopos.
>
> Se encontro uma prova nova, e ela me fornece evidencias novas o
> quadro tem de mudar, e lá se vai a confiabilidade. E é isso que
está
> acontecendo. O que parece que é, nao é
>
> >
> > (Poderiamos analisar a questao sob a optica inversa: se vcs
*querem*
> > aceitar essas pesquisas...)
>
>
> Takata , algumas coisa temos que aceitar: assim como aceitamos a
> datação do carbono-14. Nao podemos ter o santo sudário nas maos
para
> testá-los
> Só que a ciencia avança , e o teste é bem antiguinho e
provavelmente
> deve estar bem defasado. De qualquer maneira muitos cientistas
tem de
> acreditar nisso - ter confiança peloe menos em algum grau E tem
> aqueles que questionam e apresentam provas que colidem com as
> anteriores, enfim o mundo não para.
>
>
>
>
>
>
>
>
>
>
>
>
>
>
>
> ##### ##### #####
>
> Para saber mais visite
> http://www.ciencialist.hpg.ig.com.br
>
>
> ##### ##### ##### #####
>
>
> Yahoo! Grupos, um serviço oferecido por:
> PUBLICIDADE
>
>
>
>
> --------------------------------------------------------------------
----------
> Links do Yahoo! Grupos
>
> a.. Para visitar o site do seu grupo na web, acesse:
> http://br.groups.yahoo.com/group/ciencialist/
>
> b.. Para sair deste grupo, envie um e-mail para:
> ciencialist-unsubscribe@yahoogrupos.com.br
>
> c.. O uso que você faz do Yahoo! Grupos está sujeito aos Termos
do Serviço do Yahoo!.
>
>
>
> [As partes desta mensagem que não continham texto foram removidas]





SUBJECT: Re: Confiabilidade (era Santo sudário e a vanilina(correção)
FROM: "junior_br2001" <junior_br2001@yahoo.com.br>
TO: ciencialist@yahoogrupos.com.br
DATE: 20/03/2005 23:55


--- Em ciencialist@yahoogrupos.com.br, "Oraculo" <oraculo@a...>
escreveu

>
> Alberto:mesmo porque não há como assumir uma postura científica
> deixando-se o ceticismo totalmente de lado."
>
> Tente explicar isso a seu discipulo, o Pubmed..:-)
>

JR: Seria uma honra ser aluno do Alberto. Se a sua intenção foi
diminui-lo ou um ataque a minha pessoa, não colou.





SUBJECT: Re: [ciencialist] Re: Santo sudário e a vanilina(correção)
FROM: "Oraculo" <oraculo@atibaia.com.br>
TO: <ciencialist@yahoogrupos.com.br>
DATE: 21/03/2005 00:01

Olá Pubmed

Pubmed: Bater o martelo e dizer que tudo ocorreu perfeito e que essa é a data
real do tecido é no mínimo questionável. Datar tecido remendado é
dizer que o manto é da idade média é no mínimo muito questionável"

É por isso que a discussão é interminável, circular. NINGUÉM bateu o martelo, NINGUÉM disse que tudo correu perfeito, NINGUÉM (e é só ler as mensagens anteriores) disse que carbono-14 não falha, nem nada disso. E a alegação que os estudos foram sobre remendos foi feita pelo aturo do segundo estudo e nunca comprovada. É uma especulação que, embora possível, tem a palavra dos autores, e seus experimentos e protocolos, publicados para embasa-los, de que coletaram tecido de forma correta.

A questão é apenas se, entre estudos que foram repetidos, com um método reconhecidamente seguro (até o limite de segurança de qualquer conhecimento cientifico), é, neste momento, mais confiável que um estudo que usa métodos quimicos para analise de data e que ainda não tem estudos de confirmação.

Essa a questão, unica e clara, que, não podendo ser refutada por você, gera essa interminável troca de mensagens circulares, com ad hominens e espantalhos no lugar de razão e argumentos.

Homero


----- Original Message -----
From: pubmed2005
To: ciencialist@yahoogrupos.com.br
Sent: Sunday, March 20, 2005 1:16 AM
Subject: [ciencialist] Re: Santo sudário e a vanilina(correção)



Vamos supor que sim, mas quando a coisa é feita direita né? Pois está
comprovado que análise que foi feita foi nas partes remendadas como
foi comprovada pela análise dos fios dos tecidos retirados pelas
pesquisas realizadas em Milão e na Califórnia

Bater o martelo e dizer que tudo ocorreu perfeito e que essa é a data
real do tecido é no mínimo questionável. Datar tecido remendado é
dizer que o manto é da idade média é no mínimo muito questionável

-- Em ciencialist@yahoogrupos.com.br, "rmtakata" <rmtakata@a...>
escreveu
>
> --- Em ciencialist@yahoogrupos.com.br, "pubmed2005"
> > Colide no sentido de que o manto é mais antigo do que se
> > pensava.
>
> Ainda estah pra se provar q. a datacao quimica seja mais confiavel
do
> q. a datacao por radioisotopos, volto a repetir.
>
> []s,
>
> Roberto Takata





##### ##### #####

Para saber mais visite
http://www.ciencialist.hpg.ig.com.br


##### ##### ##### #####


Yahoo! Grupos, um serviço oferecido por:

São Paulo Rio de Janeiro Curitiba Porto Alegre Belo Horizonte Brasília




------------------------------------------------------------------------------
Links do Yahoo! Grupos

a.. Para visitar o site do seu grupo na web, acesse:
http://br.groups.yahoo.com/group/ciencialist/

b.. Para sair deste grupo, envie um e-mail para:
ciencialist-unsubscribe@yahoogrupos.com.br

c.. O uso que você faz do Yahoo! Grupos está sujeito aos Termos do Serviço do Yahoo!.



[As partes desta mensagem que não continham texto foram removidas]



SUBJECT: Re: Sudario
FROM: "junior_br2001" <junior_br2001@yahoo.com.br>
TO: ciencialist@yahoogrupos.com.br
DATE: 21/03/2005 00:02


--- Em ciencialist@yahoogrupos.com.br, "Sergio M. M. Taborda"
<sergiotaborda@t...> escreveu
> >tudo. Quem está fazendo julgamento moral de quem?
> >
> >
> Os ceticos não estão sendo atacados. Eles estão se sentido atacados.
> O que está sendo atacado é o ceticismo. Mas não ha ceticismo sem
ceticos.
> De facto os ceticos são uma entidade maligna , na medida que não
aportam
> nada de nada a coisa nenhuma.



JR: Pois é Taborda. A ciencia no precisa de céticos, ela já é cética
por ntureza.





SUBJECT: Calculos e carbono-14 (era: Santo sudário e a vanilina(correção)
FROM: "Oraculo" <oraculo@atibaia.com.br>
TO: <ciencialist@yahoogrupos.com.br>
DATE: 21/03/2005 00:10

Olá Pubmed

Pubmed: Sim, mas faltou o experimento(s). Eu tb realizar uma porção de
cálculo. O problema é testar "

Isso é como testar se 2+2=4 é mesmo 4. Cálculos não precisam de testes, as conclusões sobre eles é que precisam. Se existem cálculos de datação por carbono-14, que foram testados com artefatos de datas conhecidas, e estas corresponderam, estes calculos são validos. "Testa-los" seria o mesmo que eu apresentar um calculo de soma de 2 mais 2, dizer que é 4 e você pedir para que eu "teste", colocando duas laranjas ao lado de duas laranjas para ver se dá realmente 4. Não faz o menor sentido.

Se os calculos que o Takata apresentou estão matematicamente corretos, a única coisa que pode tentar discutir é se o método de datação, o carbono-14, é valido ou prciso. E este métoido tem dezenas de estudos de demonstram sua capacidade de determianr corretametne uma data a partir de isotopos de C14. O resto é tergiversação e fugas da discussão.

Faça o seguinte, apresente calculos, com base na datação de carbono-14, que invalidem os calculos do Takata, por exemplo, indicando outra data final. Seria interessante de saber como fará isso.

Ora bolas, o problema é testar procedimentos, não calculos matemáticos.

Homero

----- Original Message -----
From: junior_br2001
To: ciencialist@yahoogrupos.com.br
Sent: Sunday, March 20, 2005 2:08 AM
Subject: [ciencialist] Re: Santo sudário e a vanilina(correção)



--- Em ciencialist@yahoogrupos.com.br, "rmtakata" <rmtakata@a...>
escreveu
>
> Em segundo lugar: os calculos foram feitos com base em formulas
usadas
> pelos cientistas - eh como dizer q o engenheiro nao apresentou um
> trabalho cientifico mostrando q. os calculos dele estao corretos - e
> os dados usados foram dos proprios estudos q. vc defende.


Sim, mas faltou o experimento(s). Eu tb realizar uma porção de
cálculo. O problema é testar

>
> > OFF_-TOPIC:Ah e sobre a outra lista, acabei sendo expulso
> > praticamente sem motivo algum, simplesmente por respoder uma
> > mensagem sua.
>
> Responder a mensagens minhas falando *reiteradamente* q. o q. eu
> escrevo eh babaquice, lixo, baboseira, futilidades e coisas do tipo.
> Bem, eu nao expulsaria. Mas eh decisao do moderador. Embora vc
tenha o
> direito de achar q. isso foi "praticamente sem motivo algum".


Pior que era tudo futilidades mesmo. Levantar questões sobre a vida
as pessoas,inclusive coisas que elas nao fizereram de fato, é no
mínimo falta de respeito, e ainda ficar responder agressões c/ outras
formas de ofensas e gracinhas, é falta de zelo e respeito com a lista
e c/os partipantes. Não que eu tb nao tenha errado, mas não fui tão
longe assim - o meu erro foi só de responder as piadinhas e os
deboches proferidos. Mas isso pra mim já é passado....
Afinal , eles nunca expulsariam "alguém" que esta do lado do
movimento *cético"....ainda mais alguém tão conhecido de lá...





##### ##### #####

Para saber mais visite
http://www.ciencialist.hpg.ig.com.br


##### ##### ##### #####


Yahoo! Grupos, um serviço oferecido por:







------------------------------------------------------------------------------
Links do Yahoo! Grupos

a.. Para visitar o site do seu grupo na web, acesse:
http://br.groups.yahoo.com/group/ciencialist/

b.. Para sair deste grupo, envie um e-mail para:
ciencialist-unsubscribe@yahoogrupos.com.br

c.. O uso que você faz do Yahoo! Grupos está sujeito aos Termos do Serviço do Yahoo!.



[As partes desta mensagem que não continham texto foram removidas]



SUBJECT: Re: [ciencialist] Re: Santo sudário e a vanilina(correção)
FROM: "Oraculo" <oraculo@atibaia.com.br>
TO: <ciencialist@yahoogrupos.com.br>
DATE: 21/03/2005 00:17

Olá Pubmed

Pubmed: Bem, então eu estava certo, em partes no que eu disse.

Do ponto de vista do Sergio, sim. Do meu, e de diversos criticos e cientistas (todos, talvez, tolos e ignorantes, talvez hipócritas, como eu e o Takata..:-) , não. Afirmar que o fogo contaminou o tecido examinado exige provas. O autor, ou autores, dos estudos discordam. E apresentam evidencias e argumetnos. Para refuta-los, é preciso mais que raiva a frustação em uma lista de internet. É preciso que estudos demonstrem que o fogo contaminou os estudos de datação de carbono-14.

Podem ter contaminado? É possível, ninguem diria o contrario. Isso acontecveu com os estudos do sudário? Não existem evidencias e os pesqusiadores afirmam que levaram em conta. O Sergio afirma que não.

Escolha sua crença, com base no que achar melhor. E não se esqueça do complo das "revistas cientificas" como a Nature contra o mundo dos que são céticos de verdade..:-)

Homero

----- Original Message -----
From: junior_br2001
To: ciencialist@yahoogrupos.com.br
Sent: Sunday, March 20, 2005 11:57 AM
Subject: [ciencialist] Re: Santo sudário e a vanilina(correção)



Bem, então eu estava certo, em partes no que eu disse.

Jr


--- Em ciencialist@yahoogrupos.com.br, "Sergio M. M. Taborda"
<sergiotaborda@t...> escreveu
> Emiliano Chemello - Yahoo Grupos wrote:
>
> > Caros amigos,
> >
> > Peguei esta 'super' discussão pelo caminho. Mas percebi que
há algumas
> > referência ao fato de manto ter pego fogo e a técnica de datação
por
> > carbono
> > 14 ser 'prejudicada' ou 'inviabilizada'. Pelos meus parcos
conhecimentos,
> > digo que mesmo que o manto tenha queimado, os carbonos
radioativos NÃO
> > irão
> > perder sua atividade radioativa, visto que o fenômeno se dá a
nível
> > nuclear.
> >
> Pois não irão. Mas o problema não é esse. O problema, ao contrário
do
> que se pensa, não a datação por caborno, mas a origem do carbono
datado.
> O problema é que o fogo e outros factores contaminam o tecido ao
longo
> do anos com carbono "mais recente" do que aquele presente na época
> verdadeira do tecido..
> Portanto, os carbonos presentes no tecido têm "várias datas" e isso
> contamina o resultado final.
> Mas isso não é descontado no exame. Nem isso nem outras coisas. O
exame
> de 88 foi demasiado simplista e portanto seus resultados
> são inconclusivos.
> O exame do carbono 14 não é milagroso. Existem outros factores,
outros
> exames, quimicos, biologicos e historicos que têm que se levados em
> consideração para a datação do objecto.
> O pior cientista é aquele que só vê os dados que lhe interessam.
>
> Sérgio Taborda





##### ##### #####

Para saber mais visite
http://www.ciencialist.hpg.ig.com.br


##### ##### ##### #####


Yahoo! Grupos, um serviço oferecido por:







------------------------------------------------------------------------------
Links do Yahoo! Grupos

a.. Para visitar o site do seu grupo na web, acesse:
http://br.groups.yahoo.com/group/ciencialist/

b.. Para sair deste grupo, envie um e-mail para:
ciencialist-unsubscribe@yahoogrupos.com.br

c.. O uso que você faz do Yahoo! Grupos está sujeito aos Termos do Serviço do Yahoo!.



[As partes desta mensagem que não continham texto foram removidas]



SUBJECT: Ad Hominem..:-)
FROM: "Oraculo" <oraculo@atibaia.com.br>
TO: <ciencialist@yahoogrupos.com.br>
DATE: 21/03/2005 00:22

Olá Pubmed

Talvez, embora não tenha chamado você de criança mas seu modo de pensar de infantil. Do tipo partidário, eles e nós, o time deles e o nosso time. Assim, é apenas meio "ad hominem.".:-) Depois de todos que endereçou a mim, meio até que está de bom tamanho..:-)

Homero


----- Original Message -----
From: junior_br2001
To: ciencialist@yahoogrupos.com.br
Sent: Sunday, March 20, 2005 11:22 PM
Subject: [ciencialist] Re: Se for comprovado que está errado, temos de mudar de opinião,



--- Em ciencialist@yahoogrupos.com.br, "Oraculo" <oraculo@a...>
escreveu
> Olá Pubmed
>> Pare de pensar como criança, e preste atenção na frase


JR: Creio que está frase sim é que um ad hominem.





##### ##### #####

Para saber mais visite
http://www.ciencialist.hpg.ig.com.br


##### ##### ##### #####


Yahoo! Grupos, um serviço oferecido por:
PUBLICIDADE




------------------------------------------------------------------------------
Links do Yahoo! Grupos

a.. Para visitar o site do seu grupo na web, acesse:
http://br.groups.yahoo.com/group/ciencialist/

b.. Para sair deste grupo, envie um e-mail para:
ciencialist-unsubscribe@yahoogrupos.com.br

c.. O uso que você faz do Yahoo! Grupos está sujeito aos Termos do Serviço do Yahoo!.



[As partes desta mensagem que não continham texto foram removidas]



SUBJECT: Carbono-14 (era: Sudario)
FROM: "Oraculo" <oraculo@atibaia.com.br>
TO: <ciencialist@yahoogrupos.com.br>
DATE: 21/03/2005 00:24

Olá Pubmed

Pubmed: Além temos a confirmação de um físico como o Taborda a respeito da
falibilidade e falhas do carbono-14 com as antigas técnicas de 88
aplicadas sobre remendos do manto.

E temos a resposta/refutação do Takata demonstando que é um erro e que não alteraria a datação mais que alguns anos.

Elas por elas, com vantagem para as primeiras, confirmadas por fisicos diversos e revistas cientificas (aquelas que tem um complo contra o Sérgio e os verdadeiros céticos..:-)

Homero


----- Original Message -----
From: junior_br2001
To: ciencialist@yahoogrupos.com.br
Sent: Sunday, March 20, 2005 11:32 PM
Subject: [ciencialist] Re: Sudario



Essa discussão já evoluiu(voce nao deve ter lido as mensagens
anteriores) e já foi mostrado exames químicos que foram feitos em
universidades diferentes. Nem vou mais adentrar nisso

Além temos a confirmação de um físico como o Taborda a respeito da
falibilidade e falhas do carbono-14 com as antigas técnicas de 88
aplicadas sobre remendos do manto.

Além, disso apresentei os experimentos do Valdez e dos sovieticos
com datação do carbono-14 e suas possíveis falhas caso haja vestígios
de bioplastia, e contaminação de carbono-14 recente no linho.
Se quiser leia todas as mensagens anteriores e localize que elas
foram postadas por mim
O Takata apresentou um estudo dele, mas que foi criticado pelo
Taborda. Mas o ele só mostrou o cálculo e nao o teste pratico-
experimental. Além do mais, ele nao é a pessoa mais adequada para
isso, porque nao é a especialidade dele(nem em datação). Mas é claro,
isso nao impede que eu, ele o outros realize os calculos.

O problema é que o lado cético nao apresentou os experimentos que foi
pedido

Jr



--- Em ciencialist@yahoogrupos.com.br, "Oraculo" <oraculo@a...>
escreveu
> Olá Pubmed
>
> Pubmed: Então me diz no que o conhecimento cientifico do químico
está errado
> e no que ele está sendo incorreto. Não acho que o trabalho de
Rogers
> seja tão ruim. Ele levtou questoes muito interessantes que merecem
> uma verificação. Se voce tem coisa melhor em mente apresente a
> comunidade científica"
>
> Embora não pense que vai adiantar, vou tentar de novo: este estudo
precisa ser confirmado por pesquisadores independentes. Isso é
absolutametne necessário, devido ao rigor do método, para dar
confiabilidade aos resultados. Até lá, os estudos com confirmação
independente, tem preferencia.






##### ##### #####

Para saber mais visite
http://www.ciencialist.hpg.ig.com.br


##### ##### ##### #####


Yahoo! Grupos, um serviço oferecido por:
PUBLICIDADE




------------------------------------------------------------------------------
Links do Yahoo! Grupos

a.. Para visitar o site do seu grupo na web, acesse:
http://br.groups.yahoo.com/group/ciencialist/

b.. Para sair deste grupo, envie um e-mail para:
ciencialist-unsubscribe@yahoogrupos.com.br

c.. O uso que você faz do Yahoo! Grupos está sujeito aos Termos do Serviço do Yahoo!.



[As partes desta mensagem que não continham texto foram removidas]



SUBJECT: Espantalho (era Sudario
FROM: "Oraculo" <oraculo@atibaia.com.br>
TO: <ciencialist@yahoogrupos.com.br>
DATE: 21/03/2005 00:29

Olá Pubmed

Falácia do espantalho é um erro de lógica que usa a construção de uma imagem incorreta, mas facilmente identificavel como tal, para tentar destruir o argumento do adversário. Como afirmar que céticos tem um método como "santo e sagrado" sabendo que será tomado como falso, mesmo que céticos em nenhum momento tenham tomado como santo e sagrado nada disso..:-)

Pubmed: Mesmo que voce esteja falando da época atual, nao existe um método de
datação 100%% infalível. Aliás voce que é cético deveria saber que a
ciencia , seus métodos, hipóteses e teoriais nao são tão confiáveis
como parece ser

Outro espantalho. Em todas as mensagens vai encontrtar frases minhas afirmando que não há nada 100% infalivel ou verdadeiro em ciência, ainda assim tenta alegar que é uma posição minha..:-) Percebeu o espantalho? Eu digo que entre dois estudos, um deles é mais confiável, você afirma que estou defendendo que um deles é 100% correto e sagrado..:-) Espantalho..:-)

Não, não são 100% confiáveis. Mas são, certamente, o mais confiável que podemos dispor no momento (e nos últimos séculos). Tão confiáveis que mesmo você recorrerá a eles se sua saúde ou segurança estiver em risco.

E depois voltará a lista para desancar essa tolice de céticos..:-)

Homero


----- Original Message -----
From: junior_br2001
To: ciencialist@yahoogrupos.com.br
Sent: Sunday, March 20, 2005 11:48 PM
Subject: [ciencialist] Re: Espantalho (era Sudario



Falácia espantalho? De onde tirou esse palavrão? Dos livros de Sagan?

A datação de carbono-14 feita em 88 nem se compara com as técnicas
atuais. Portanto, se isso deve serve então de *espantalho*(para
espantar logo de uma vez) essa idéia equivocada de que o carbono-14 é
um método confiável(pelo menos naquela época), e ainda tem os
impedimentos que falei anteriormente(revestimento bioplastico,
contaminação do tecido e teste realizado em remendos recentes
conforme as confirmações laboratoriais realizadas nas fibras dos
tecidos mostraram).

Mesmo que voce esteja falando da época atual, nao existe um método de
datação 100%% infalível. Aliás voce que é cético deveria saber que a
ciencia , seus métodos, hipóteses e teoriais nao são tão confiáveis
como parece ser

JR


--- Em ciencialist@yahoogrupos.com.br, "Oraculo" <oraculo@a...>
escreveu
> Olá Pubmed
>
> Pubmed:De um lado os céticos que se dizem adeptos da ciencia que
alegam que
> só o santo carbono-14 é infalível e de outro uns( eu ou alguns) que
> defende a apenas a posição da ciencia que outros testes tb são
>
> Falácia do espantalho. Se o Anderson se der ao trabalho de reler as
mensagens, em nenhuma delas vai encontrar algo sequer parecido com
essa afirmação. O carbono-14 é um bom e confiável método de datação,
apenas isso. Não é santo, não é infalível, nem é a base da discussão.
>





##### ##### #####

Para saber mais visite
http://www.ciencialist.hpg.ig.com.br


##### ##### ##### #####


Yahoo! Grupos, um serviço oferecido por:

São Paulo Rio de Janeiro Curitiba Porto Alegre Belo Horizonte Brasília




------------------------------------------------------------------------------
Links do Yahoo! Grupos

a.. Para visitar o site do seu grupo na web, acesse:
http://br.groups.yahoo.com/group/ciencialist/

b.. Para sair deste grupo, envie um e-mail para:
ciencialist-unsubscribe@yahoogrupos.com.br

c.. O uso que você faz do Yahoo! Grupos está sujeito aos Termos do Serviço do Yahoo!.



[As partes desta mensagem que não continham texto foram removidas]



SUBJECT: Contraditórios ou complementares.
FROM: "Oraculo" <oraculo@atibaia.com.br>
TO: <ciencialist@yahoogrupos.com.br>
DATE: 21/03/2005 00:34

Olá Pubmed

Pubmed: Deixo esses cálculos pro Taborda responder(se ele quiser é claro),
ele deve te explicar isso melhor que eu"

Não faça isso, por favor. Os cálculos são apenas soma e subtração simples. Se eu digo que nasci, no máximo, em 1930 e outra pessoa diz que eu nasci em 1960, estas datas são clarametne contraditórias.

Um objeto é tido como tendo 500 anos. Significa que foi criado em 1500, nunca antes. Se o mesmo objeto tem sua data determinada que foi criado não depois do ano 600, estas duas afirmações são clarametne contraditórias. Não podem ser complementares de nenhuma forma lógica.

Se o primeiro estudo data o sudario como tendo 660 anos, significa que foi criado não antes do ano 1340 DC. Se outro estudo diz que ele foi criado, no máximo, até o ano 700 DC, estas duas afirmações são claramente contraditórias e não podem ser complementares. É bem simples.

Homero


----- Original Message -----
From: junior_br2001
To: ciencialist@yahoogrupos.com.br
Sent: Sunday, March 20, 2005 11:52 PM
Subject: [ciencialist] Re: Santo sudário e a vanilina(correção)



Deixo esses cálculos pro Taborda responder(se ele quiser é claro),
ele deve te explicar isso melhor que eu

JR

--- Em ciencialist@yahoogrupos.com.br, "Oraculo" <oraculo@a...>
escreveu
> Olá Pubmed
>
> Pubmed: *Não se contradizem, o mínimo estipulado do método B é de
1300*. O
> máximo que estipulado que é 3000
>
> Faça as contas. Talvez esteja confundino (e talvez não tenha lido a
explicação nas outras mensagens) com a data de existencia e a data de
criação. O carbono diz que existe a 660 anos, o que significa que foi
criado em 1340. A quimica diz que tem entre 1300 e 3000 anos, o que
significa que foi craido entre 700 DC e 2000 AC.
>
> Faça as contas. Ou apenas compare as datas. Um tecido criado no
máximo até o ano 700 depois de cristo não pode ter sido criado depois
de 1340 depois de cristo. É matematica simples.
>
> O 1300 do metodo B não é o ano de criação, mas o tempo de
existencia. Ele existe a pelo menos 1300 anos, ou seja, foi criado
até o ano 700, não até o ano 1340.
>
> Homero
>
>
> ----- Original Message -----
> From: pubmed2005
> To: ciencialist@yahoogrupos.com.br
> Sent: Saturday, March 19, 2005 9:34 PM
> Subject: [ciencialist] Re: Santo sudário e a vanilina(correção)
>
>
>
> Lá vai
>
> --- Em ciencialist@yahoogrupos.com.br, "rmtakata" <rmtakata@a...>
> escreveu
> >
> > Eh disso q se trata. O metodo A diz q. o tecido eh de 1300, o
> metodo B
> > diz q. o tecido eh bem anterior a isso - talvez de 40. Os
metodos se
> > contradizem. Entao eh questao de ver qual o mais confiavel.
>
> *Não se contradizem, o mínimo estipulado do método B é de 1300*.
O
> máximo que estipulado que é 3000
>
> >
> > > Mas que novas descobertas de peso foram surgiram que podem
> > > mudar o padrão estipulado anteriormente.
> >
> > Claro q. se houver novas descobertas de peso q. contradigam a
idade
> > apontada pelo C-14 serah preciso rever.
>
> Já está havendo revisão. A nova pesquisa foi publicada na Nature
>
> >
> > Lanca a possibilidade do metodo da analise quimica nao ser tao
> > confiavel. Sabemos q. o decaimento radioativo nao eh alterado
por
> > pressao, temperatura ou qq outro fator ambiental conhecido. Jah
a
> taxa
> > de reacoes quimicas variam enormente de acordo com a
temperatura (e
> > sabemos q. o tecido em questao pegou fogo), presenca de umidade,
> > exposicao a luz, presenca de bacterias e fungos...
>
> Não seja tão certo da confiabilidade. de uma lida:
>
> O Dr Leôncio Garza Valdes, médico e professor da Universidade do
> Texas, pesquisou e descobriu resíduos orgânicos gerados por
bactérias
> em múmias. Ao tentar data-las pelo processo de radiocarbono
descobriu
> que esses resíduos, que formavam um revestimento bio-plástico
sobre
> os fios do tecido, alteravam o resultado da datação, devido à
> impossibilidade de se limpá-los de modo adequado.
>
> Em 1993, O Dr.Valdes conseguiu algumas amostras do Sudário que
haviam
> sido cortadas na mesma ocasião do teste de rádiocarbono de 1988,
> junto ao Professor Giovani Riggi, o microanalista de Turim já
> mencionado, guardião das amostras, e homem de confiança do
Cardeal
> Ballestrero.
>
> Ao analisá-las descobriu a presença do revestimento bio-plástico
> (Lichenothelia Varnish) sobre os fios de linho, assim como a
presença
> desses organismos (Lichenothelia) vivos.
>
>
>
> Aliás, o Dr Valdes não conseguiu fazer o teste de rádiocarbono
> exatamente pela dificuldade em se fazer a limpeza. Ele chegou a
> utilizar exatamente o mesmo processo de limpeza que foi utilizado
> pelos laboratórios do teste de 1988, e constatou que o
revestimento
> bio-plástico não saia.
>
> Segundo o Dr. Valdes: &#8220;A pátina causada pelos fungos é um
acréscimo
> natural nas superfícies antigas estáveis. São necessárias
centenas de
> anos para que os fungos criem um &#8216;verniz&#8217; contínuo.
As fibras do
> Sudário de Turim têm um depósito espesso devido às bactérias. A
data
> obtida mediante o C14 em 1988 deve-se na realidade, a uma mistura
do
> C14 do Tecido do Sudário com o C14 da pátina de bactérias. Este
seria
> de até 60%&#8221;.
>
> À luz desses estudos, o inventor do processo de datação por
> rádiocarbono pelo processo usado no Sudário, Harry Gove, teve de
> declarar em 27 de Janeiro de 1995: &#8220;A técnica que se usou
em 1988
> para a datação do Sudário de Turim por meio do C14 foi inventada
em
> meu laboratório, na Universidade de Rochester, em 1977. Depois
dessa
> datação, estive convencido do resultado durante anos.
Recentemente,
> porém, o doutor Garza Valdés, de San Antonio, Texas, apresentou
> provas consistentes a respeito de um tipo de contaminação por
carbono
> recente produzida nos fios do Sudário por bactérias que os
processos
> de limpeza usados pelos três laboratórios podem não ter removido.
> Essa contaminação, de acordo com sua espessura, pode fazer com
que a
> data fornecida pelos três laboratórios seja mais recente&#8221;.
>
> 3.5- A contestação de Kouznetsov
>
> O diretor dos Laboratórios E.A. Sedoo de Pesquisa de Bio-
Polímeros de
> Moscou e premio Lênin de Ciência, Dr. A. Kouznetsov, participou
de um
> congresso em Londres onde foi convidado a testar seus métodos de
> datação por radiocarbono pelo Geólogo Dr Guy Berthault, para
comparar
> com os resultados do exame do Sudário.
>
> Analisando a história do Sudário, descobriu que ele passou por um
> incêndio em 1532. Resolveu recriar a condição em laboratório, e
> descobriu que o linho absorveu C14 recente, principalmente por
ter se
> submetido a altas temperaturas por tempos suficientemente longos
na
> presença de produtos da combustão (água, anidrido carbônico e
óxido
> de carbono) e íons de prata, pois a urna era de prata, capazes de
> agir como catalisadores.
>
> Para comprovar isso, Kouznetsov pegou um tecido de linho do séc.
I,
> procedente de Em Gedi (Israel), e enviou ao Laboratório de
Tucson,
> Arizona, onde fizeram a datação.
>
> Mais tarde, o professor russo queimou outro pedaço do mesmo
lençol,
> simulando a condição do Sudário no incêndio, e enviou novamente
para
> Tucson. O resultado desta datação foi de 1300 anos mais recente.
>
> .6- Conclusão
>
> Conforme o Dr Valdés e o Dr Kouznetsov, não se aplica testes de
> rádiocarbono a matérias têxteis como o linho, devido aos seus
altos
> níveis de contaminação sem possibilidade prática de remoção.
>
> > Como eu disse a questao eh de se mostrar q. a metodologia de
datacao
> > quimica eh mais confiavel do q. o de radioisotopos.
>
> Se encontro uma prova nova, e ela me fornece evidencias novas o
> quadro tem de mudar, e lá se vai a confiabilidade. E é isso que
está
> acontecendo. O que parece que é, nao é
>
> >
> > (Poderiamos analisar a questao sob a optica inversa: se vcs
*querem*
> > aceitar essas pesquisas...)
>
>
> Takata , algumas coisa temos que aceitar: assim como aceitamos a
> datação do carbono-14. Nao podemos ter o santo sudário nas maos
para
> testá-los
> Só que a ciencia avança , e o teste é bem antiguinho e
provavelmente
> deve estar bem defasado. De qualquer maneira muitos cientistas
tem de
> acreditar nisso - ter confiança peloe menos em algum grau E tem
> aqueles que questionam e apresentam provas que colidem com as
> anteriores, enfim o mundo não para.
>
>
>
>
>
>
>
>
>
>
>
>
>
>
>
> ##### ##### #####
>
> Para saber mais visite
> http://www.ciencialist.hpg.ig.com.br
>
>
> ##### ##### ##### #####
>
>
> Yahoo! Grupos, um serviço oferecido por:
> PUBLICIDADE
>
>
>
>
> --------------------------------------------------------------------
----------
> Links do Yahoo! Grupos
>
> a.. Para visitar o site do seu grupo na web, acesse:
> http://br.groups.yahoo.com/group/ciencialist/
>
> b.. Para sair deste grupo, envie um e-mail para:
> ciencialist-unsubscribe@yahoogrupos.com.br
>
> c.. O uso que você faz do Yahoo! Grupos está sujeito aos Termos
do Serviço do Yahoo!.
>
>
>
> [As partes desta mensagem que não continham texto foram removidas]





##### ##### #####

Para saber mais visite
http://www.ciencialist.hpg.ig.com.br


##### ##### ##### #####


Yahoo! Grupos, um serviço oferecido por:







------------------------------------------------------------------------------
Links do Yahoo! Grupos

a.. Para visitar o site do seu grupo na web, acesse:
http://br.groups.yahoo.com/group/ciencialist/

b.. Para sair deste grupo, envie um e-mail para:
ciencialist-unsubscribe@yahoogrupos.com.br

c.. O uso que você faz do Yahoo! Grupos está sujeito aos Termos do Serviço do Yahoo!.



[As partes desta mensagem que não continham texto foram removidas]



SUBJECT: Confiabilidade (era Santo sudário e a vanilina(correção)
FROM: "Oraculo" <oraculo@atibaia.com.br>
TO: <ciencialist@yahoogrupos.com.br>
DATE: 21/03/2005 00:36

Ola Pubmed

Pubmed: Seria uma honra ser aluno do Alberto. Se a sua intenção foi
diminui-lo ou um ataque a minha pessoa, não colou.

risos..:-) Não, pubmed, não tentei diminuir nem você nem ele..:-) Foi uma brincadeira, com o fato de ele ter colocado que ceticismo (mesmo que seja de um tipo especial, do Alberto) é importante, e você, mesmo que usando a mensagem dele como apoio a sua posição, afirmou ser inútil e que só atrapalha a ciência..:-)

Uma brincadeira, nada mais..:-)

Homero




----- Original Message -----
From: junior_br2001
To: ciencialist@yahoogrupos.com.br
Sent: Sunday, March 20, 2005 11:55 PM
Subject: [ciencialist] Re: Confiabilidade (era Santo sudário e a vanilina(correção)



--- Em ciencialist@yahoogrupos.com.br, "Oraculo" <oraculo@a...>
escreveu

>
> Alberto:mesmo porque não há como assumir uma postura científica
> deixando-se o ceticismo totalmente de lado."
>
> Tente explicar isso a seu discipulo, o Pubmed..:-)
>

JR: Seria uma honra ser aluno do Alberto. Se a sua intenção foi
diminui-lo ou um ataque a minha pessoa, não colou.





##### ##### #####

Para saber mais visite
http://www.ciencialist.hpg.ig.com.br


##### ##### ##### #####


Yahoo! Grupos, um serviço oferecido por:

São Paulo Rio de Janeiro Curitiba Porto Alegre Belo Horizonte Brasília




------------------------------------------------------------------------------
Links do Yahoo! Grupos

a.. Para visitar o site do seu grupo na web, acesse:
http://br.groups.yahoo.com/group/ciencialist/

b.. Para sair deste grupo, envie um e-mail para:
ciencialist-unsubscribe@yahoogrupos.com.br

c.. O uso que você faz do Yahoo! Grupos está sujeito aos Termos do Serviço do Yahoo!.



[As partes desta mensagem que não continham texto foram removidas]



SUBJECT: Re: Ad Hominem..:-)
FROM: "junior_br2001" <junior_br2001@yahoo.com.br>
TO: ciencialist@yahoogrupos.com.br
DATE: 21/03/2005 00:53


Epa, uma falha aqui, não dirige nenhuma crítica a sua pessoa...falei
dos céticos de modo geral. Se a carapuça serviu é outros
quinhentos....

Mas voce continua praticando ad hominem contra minha pessoa, continua
a me chamar de criança , infantil, coisas que nem levantei sobre sua
personalidade.
E além do mais , mesmo se eu o tivesse feito, não justifioca voce me
atacar por isso. Mass não foi feito nenhum ataque a sua pessoa, não
mesmo. Critiquei o ceticismo e os céticos da internet. Se voce se
enquadra nisso é outros quinhentos. Afinal não temos como evitar
isso. Tipo:
Se critico o Palmeiras, e um jogador se sente ressentido, isso não é
culpa minha. Eu critiquei o Palmeiras, falei que ele joga mal, é um
péssimo time, e não deve ganhar o campeonato brasileiro. Foi um
crítica dirigida ao Palmeiras e não a um jogador. Se critico o
Governo americano, nao estou criticando nenhum americano em
particular. Portanto voce nao pode me acusar de ter me dirigido a
voce de forma agressiva.

JR

--- Em ciencialist@yahoogrupos.com.br, "Oraculo" <oraculo@a...>
escreveu
> Olá Pubmed
>
> Talvez, embora não tenha chamado você de criança mas seu modo de
pensar de infantil. Do tipo partidário, eles e nós, o time deles e o
nosso time. Assim, é apenas meio "ad hominem.".:-) Depois de todos
que endereçou a mim, meio até que está de bom tamanho..:-)
>
> Homero






SUBJECT: Re: Carbono-14 (era: Sudario)
FROM: "junior_br2001" <junior_br2001@yahoo.com.br>
TO: ciencialist@yahoogrupos.com.br
DATE: 21/03/2005 01:06




--- Em ciencialist@yahoogrupos.com.br, "Oraculo" <oraculo@a...>
escreveu
> Olá Pubmed
>
>
> E temos a resposta/refutação do Takata demonstando que é um erro e
que não alteraria a datação mais que alguns anos.
>



JR: A confiabilidade e o crédito é mais pela resposta do Taborda, o
Taborda é físico e engenheiro. Embora nao querendo menosprezar o
Takata ele nao é um especialista nisso, nao é físico, nem químico.
Nem especialista em datação. Claro que isso nao impede que ele faça
calculos. Mas a comunidade científica iria dar mais crédito e valor a
palavra de um físico.
Claro que isso não é um argumento de autoridade, cada qual com sua
função, seria um argumento de autoridade se eu tivesse comparando
dois físicos(ou químicos) especialistas, e desse preferencia a um
mais famoso só porque publicou na Nature poe exemplo e o outro numa
revista vagabunda. Se o Takata é biólogo temos de creditar ele por
trabalhos neste campo. Claro que ele tb pode ser um fisico amador,
mas nao sei até onde a comunidade científica aceitaria uma publicação
dele neste sentido - pode até ser que abram uma exceção, mas nao sei
se funciona assim.
Portanto, o Takata pode questionar na condição de cético , mas não na
de cientista físico-químico no caso.

Além do mais apresentei trabalhos que foram realizadas pela
comunidade científica, testados, publicados e analisados com as
devidas referencias. Voce nao deve ter lido

JR






SUBJECT: Re: Confiabilidade (era Santo sudário e a vanilina(correção)
FROM: "junior_br2001" <junior_br2001@yahoo.com.br>
TO: ciencialist@yahoogrupos.com.br
DATE: 21/03/2005 01:10


O ceticismo organizado é inútil, pois a ciencia é cética por
natureza. Se assim não fosse não seria ciencia.

JR

--- Em ciencialist@yahoogrupos.com.br, "Oraculo" <oraculo@a...>
escreveu
> Ola Pubmed
>
> Pubmed: Seria uma honra ser aluno do Alberto. Se a sua intenção foi
> diminui-lo ou um ataque a minha pessoa, não colou.
>
> risos..:-) Não, pubmed, não tentei diminuir nem você nem ele..:-)
Foi uma brincadeira, com o fato de ele ter colocado que ceticismo
(mesmo que seja de um tipo especial, do Alberto) é importante, e
você, mesmo que usando a mensagem dele como apoio a sua posição,
afirmou ser inútil e que só atrapalha a ciência..:-)
>
> Uma brincadeira, nada mais..:-)
>
> Homero
>
>
>
>
> ----- Original Message -----
> From: junior_br2001
> To: ciencialist@yahoogrupos.com.br
> Sent: Sunday, March 20, 2005 11:55 PM
> Subject: [ciencialist] Re: Confiabilidade (era Santo sudário e a
vanilina(correção)
>
>
>
> --- Em ciencialist@yahoogrupos.com.br, "Oraculo" <oraculo@a...>
> escreveu
>
> >
> > Alberto:mesmo porque não há como assumir uma postura científica
> > deixando-se o ceticismo totalmente de lado."
> >
> > Tente explicar isso a seu discipulo, o Pubmed..:-)
> >
>
> JR: Seria uma honra ser aluno do Alberto. Se a sua intenção foi
> diminui-lo ou um ataque a minha pessoa, não colou.
>
>
>
>
>
> ##### ##### #####
>
> Para saber mais visite
> http://www.ciencialist.hpg.ig.com.br
>
>
> ##### ##### ##### #####
>
>
> Yahoo! Grupos, um serviço oferecido por:
>
> São Paulo Rio de Janeiro Curitiba Porto Alegre
Belo Horizonte Brasília
>
>
>
>
> --------------------------------------------------------------------
----------
> Links do Yahoo! Grupos
>
> a.. Para visitar o site do seu grupo na web, acesse:
> http://br.groups.yahoo.com/group/ciencialist/
>
> b.. Para sair deste grupo, envie um e-mail para:
> ciencialist-unsubscribe@yahoogrupos.com.br
>
> c.. O uso que você faz do Yahoo! Grupos está sujeito aos Termos
do Serviço do Yahoo!.
>
>
>
> [As partes desta mensagem que não continham texto foram removidas]





SUBJECT: Re: [ciencialist] Re: Ad Hominem..:-)
FROM: "Oraculo" <oraculo@atibaia.com.br>
TO: <ciencialist@yahoogrupos.com.br>
DATE: 21/03/2005 01:18

Olá Pubmed

Por favor, a forma de pensar, o argumento "nós contra eles" é infantil, não necessariamente você. Não confunda uma afirmação a respeito de uma forma de pensar com outra sobre você. Mesmo que pense dessa forma neste caso, não significa que possa pensar diferente, com outro ponto de vista, em outra situação. Apenas nesta especifica é que está colocando as coisas dessa forma, futebolistica..:-)

E sou cético, e claro que a crítica a céticos tolos e que tais se refere a mim..:-) Não ligo nem fico chateado, é direito seu, mas continua sendo "ad hominem" no que importa, não serve como argumento válido..:-)

E reconheci ser a critica sobre a infantilidade da forma de pensar de time de futebol, nós e eles, um "meio ad hominem", e sinto muito..:-) Mas preciso criticar essa postura de alguma forma, já que ela impede que se chegue a alguma conclusão (nem que seja a falta de uma conclusão conjunta).

Assim, insisto, nós e eles não é uma forma razoável de debater ou de compreender uma questão.

Homero

----- Original Message -----
From: junior_br2001
To: ciencialist@yahoogrupos.com.br
Sent: Monday, March 21, 2005 12:53 AM
Subject: [ciencialist] Re: Ad Hominem..:-)



Epa, uma falha aqui, não dirige nenhuma crítica a sua pessoa...falei
dos céticos de modo geral. Se a carapuça serviu é outros
quinhentos....

Mas voce continua praticando ad hominem contra minha pessoa, continua
a me chamar de criança , infantil, coisas que nem levantei sobre sua
personalidade.
E além do mais , mesmo se eu o tivesse feito, não justifioca voce me
atacar por isso. Mass não foi feito nenhum ataque a sua pessoa, não
mesmo. Critiquei o ceticismo e os céticos da internet. Se voce se
enquadra nisso é outros quinhentos. Afinal não temos como evitar
isso. Tipo:
Se critico o Palmeiras, e um jogador se sente ressentido, isso não é
culpa minha. Eu critiquei o Palmeiras, falei que ele joga mal, é um
péssimo time, e não deve ganhar o campeonato brasileiro. Foi um
crítica dirigida ao Palmeiras e não a um jogador. Se critico o
Governo americano, nao estou criticando nenhum americano em
particular. Portanto voce nao pode me acusar de ter me dirigido a
voce de forma agressiva.

JR

--- Em ciencialist@yahoogrupos.com.br, "Oraculo" <oraculo@a...>
escreveu
> Olá Pubmed
>
> Talvez, embora não tenha chamado você de criança mas seu modo de
pensar de infantil. Do tipo partidário, eles e nós, o time deles e o
nosso time. Assim, é apenas meio "ad hominem.".:-) Depois de todos
que endereçou a mim, meio até que está de bom tamanho..:-)
>
> Homero






##### ##### #####

Para saber mais visite
http://www.ciencialist.hpg.ig.com.br


##### ##### ##### #####


Yahoo! Grupos, um serviço oferecido por:

São Paulo Rio de Janeiro Curitiba Porto Alegre Belo Horizonte Brasília




------------------------------------------------------------------------------
Links do Yahoo! Grupos

a.. Para visitar o site do seu grupo na web, acesse:
http://br.groups.yahoo.com/group/ciencialist/

b.. Para sair deste grupo, envie um e-mail para:
ciencialist-unsubscribe@yahoogrupos.com.br

c.. O uso que você faz do Yahoo! Grupos está sujeito aos Termos do Serviço do Yahoo!.



[As partes desta mensagem que não continham texto foram removidas]



SUBJECT: Carbono-14 (era: Sudario)
FROM: "Oraculo" <oraculo@atibaia.com.br>
TO: <ciencialist@yahoogrupos.com.br>
DATE: 21/03/2005 01:24

Olá Pubmed

Pubmed: A confiabilidade e o crédito é mais pela resposta do Taborda, o
Taborda é físico e engenheiro."

Então, temos aqui mais um caso de palavra sagrada..:-) (não resisti a brincadeira..:-)

A confiabilidade, para mim, deriva de muitos fatores, a maioria deles determinados pelo método cientifico, o que inclui estudos publicados, pesquisas sólidas e aceitação da comunidade cientifica, etc. Mas se deseja escolher o Taborda como fonte única de sua avaliação de confiabilidade, é direito seu, não discutirei isso..:-)

Justamente por nenhum dos dois, Takata e Taborda, estarem diretametne envolvidos com os estudos, e ambos estarem relatando pesqusias e conhecimento produzido por terceiros (e isso serve para a maioria do conehcimento cientifico), que coloco minha maior confiança (ainda que, mais uma vez, não de 100% e nem como um torcedor..:-) na posição de maior segurança, pesquisas publicadas, reproduzidas, aceitação da comunidade cientifica, maior numero de pesquisadores, maior numero de cientistas a defender a posição, etc. Se isso mudar, como não estou torcendo nem é um time, mude de idéia e de conclusão (ainda que você ache isso vexatório, esse "vira casaca" da ciência todo o tempo. Porque não podem ser fieis a seus credos como catolicos ou islamicos, esses cientistas, não?..:-)

Mas, de novo, pode ficar do lado que desejar, é direito seu..:-) Mas não torça tanto, é chato quando o time da gente perde..:-)

Homero

----- Original Message -----
From: junior_br2001
To: ciencialist@yahoogrupos.com.br
Sent: Monday, March 21, 2005 1:06 AM
Subject: [ciencialist] Re: Carbono-14 (era: Sudario)





--- Em ciencialist@yahoogrupos.com.br, "Oraculo" <oraculo@a...>
escreveu
> Olá Pubmed
>
>
> E temos a resposta/refutação do Takata demonstando que é um erro e
que não alteraria a datação mais que alguns anos.
>



JR: A confiabilidade e o crédito é mais pela resposta do Taborda, o
Taborda é físico e engenheiro. Embora nao querendo menosprezar o
Takata ele nao é um especialista nisso, nao é físico, nem químico.
Nem especialista em datação. Claro que isso nao impede que ele faça
calculos. Mas a comunidade científica iria dar mais crédito e valor a
palavra de um físico.
Claro que isso não é um argumento de autoridade, cada qual com sua
função, seria um argumento de autoridade se eu tivesse comparando
dois físicos(ou químicos) especialistas, e desse preferencia a um
mais famoso só porque publicou na Nature poe exemplo e o outro numa
revista vagabunda. Se o Takata é biólogo temos de creditar ele por
trabalhos neste campo. Claro que ele tb pode ser um fisico amador,
mas nao sei até onde a comunidade científica aceitaria uma publicação
dele neste sentido - pode até ser que abram uma exceção, mas nao sei
se funciona assim.
Portanto, o Takata pode questionar na condição de cético , mas não na
de cientista físico-químico no caso.

Além do mais apresentei trabalhos que foram realizadas pela
comunidade científica, testados, publicados e analisados com as
devidas referencias. Voce nao deve ter lido

JR






##### ##### #####

Para saber mais visite
http://www.ciencialist.hpg.ig.com.br


##### ##### ##### #####


Yahoo! Grupos, um serviço oferecido por:
PUBLICIDADE




------------------------------------------------------------------------------
Links do Yahoo! Grupos

a.. Para visitar o site do seu grupo na web, acesse:
http://br.groups.yahoo.com/group/ciencialist/

b.. Para sair deste grupo, envie um e-mail para:
ciencialist-unsubscribe@yahoogrupos.com.br

c.. O uso que você faz do Yahoo! Grupos está sujeito aos Termos do Serviço do Yahoo!.



[As partes desta mensagem que não continham texto foram removidas]



SUBJECT: Re: Ad Hominem..:-)
FROM: "junior_br2001" <junior_br2001@yahoo.com.br>
TO: ciencialist@yahoogrupos.com.br
DATE: 21/03/2005 01:29


--- Em ciencialist@yahoogrupos.com.br, "Oraculo" <oraculo@a...>
escreveu
> Olá Pubmed
>
>> E sou cético, e claro que a crítica a céticos tolos e que tais se
refere a mim..:-) Não ligo nem fico chateado, é direito seu, mas
continua sendo "ad hominem" no que importa, não serve como argumento
válido..:-)
>


JR: Mas de qualquer forma não houve ataque dirigido a voce. Voce
sentiu que foi com voce, o que é muito diferente.

Quando falo céticos tolos e voce acha que foi com voce... fazer o que?
De qualquer forma, quando falei em céticos tolos pela primeira vez,
voce foi o primeiro a responder, eu ainda nem havia entrado em
discussão com voce ainda, eu estava falando em santo sudário e os
pseudo-ceticos da internet. Isso corrobora que voce respondeu como
uma reação ao que eu disse. E o ataque crítico aos céticos nem era
com voce. Depois que voce respondeu meu ataque crítico, voce acabou
se envolvendo neurologicamente(ou psicologicamente) com a idéia, e
achou que era com voce pelo fato de voce ser um cético.

JR





SUBJECT: Confiabilidade (era Santo sudário e a vanilina(correção)
FROM: "Oraculo" <oraculo@atibaia.com.br>
TO: <ciencialist@yahoogrupos.com.br>
DATE: 21/03/2005 01:31

Olá Pubmed

Pubmed: O ceticismo organizado é inútil, pois a ciencia é cética por
natureza. Se assim não fosse não seria ciencia.

Depois não quer que eu brinque com suas frases..:-) A ciência é cética, mas não precisa de céticos..:-) Sei, será feita, claro, por crédulos, embora o resultado e os estudos sejam céticos. Deve não aceitar uma afirmação sem estudos de comprovação, mas, como não devem ser céticos, os cientistas devem aceitar sem estudos de confirmação, para não parecer que são céticos..:-) E nem é preciso ceticismo organizado, para ensinar os mecanismos e formas de pensar críticas necessárias ao exercício de ceticismo a estudantes e futuros cientistas, porque, ao se tornar cientistas, automaticamente farão ciência cética..:-) Uma graça..:-)

Sei, faz mesmo muito sentido..:-)

Não seria ciência, se não fosse cética, mas o ceticismo é prejudicial a ciência. Frases de efeito, embora nada signifiquem..:-)

O ceticismo é uma ferramenta. Implica em ser cuidadoso ao extremo antes de levar uma alegação em conta, antes de decidir que uma afirmação ou conclusão tem validade ou é real. Apenas isso. Ser cuidadoso e esperar por evidencias, cada vez mais fortes. Implica também em não se apegar a certezas e mudar de conclusão, se as evidencias apresentarem motivo e embasamento.

Qualquer coisa além disso é bobagem, acusações sem fundamento.

Se o probelma em questão aqui, o segundo estudo sobre a data do santo sudário, passar por isso, será aceito de bom grado. Enquanto isso, ficamos, céticos e cientistas, na espera. Como sempre..:-)

Homero


----- Original Message -----
From: junior_br2001
To: ciencialist@yahoogrupos.com.br
Sent: Monday, March 21, 2005 1:10 AM
Subject: [ciencialist] Re: Confiabilidade (era Santo sudário e a vanilina(correção)



O ceticismo organizado é inútil, pois a ciencia é cética por
natureza. Se assim não fosse não seria ciencia.

JR

--- Em ciencialist@yahoogrupos.com.br, "Oraculo" <oraculo@a...>
escreveu
> Ola Pubmed
>
> Pubmed: Seria uma honra ser aluno do Alberto. Se a sua intenção foi
> diminui-lo ou um ataque a minha pessoa, não colou.
>
> risos..:-) Não, pubmed, não tentei diminuir nem você nem ele..:-)
Foi uma brincadeira, com o fato de ele ter colocado que ceticismo
(mesmo que seja de um tipo especial, do Alberto) é importante, e
você, mesmo que usando a mensagem dele como apoio a sua posição,
afirmou ser inútil e que só atrapalha a ciência..:-)
>
> Uma brincadeira, nada mais..:-)
>
> Homero
>
>
>
>
> ----- Original Message -----
> From: junior_br2001
> To: ciencialist@yahoogrupos.com.br
> Sent: Sunday, March 20, 2005 11:55 PM
> Subject: [ciencialist] Re: Confiabilidade (era Santo sudário e a
vanilina(correção)
>
>
>
> --- Em ciencialist@yahoogrupos.com.br, "Oraculo" <oraculo@a...>
> escreveu
>
> >
> > Alberto:mesmo porque não há como assumir uma postura científica
> > deixando-se o ceticismo totalmente de lado."
> >
> > Tente explicar isso a seu discipulo, o Pubmed..:-)
> >
>
> JR: Seria uma honra ser aluno do Alberto. Se a sua intenção foi
> diminui-lo ou um ataque a minha pessoa, não colou.
>
>
>
>
>
> ##### ##### #####
>
> Para saber mais visite
> http://www.ciencialist.hpg.ig.com.br
>
>
> ##### ##### ##### #####
>
>
> Yahoo! Grupos, um serviço oferecido por:
>
> São Paulo Rio de Janeiro Curitiba Porto Alegre
Belo Horizonte Brasília
>
>
>
>
> --------------------------------------------------------------------
----------
> Links do Yahoo! Grupos
>
> a.. Para visitar o site do seu grupo na web, acesse:
> http://br.groups.yahoo.com/group/ciencialist/
>
> b.. Para sair deste grupo, envie um e-mail para:
> ciencialist-unsubscribe@yahoogrupos.com.br
>
> c.. O uso que você faz do Yahoo! Grupos está sujeito aos Termos
do Serviço do Yahoo!.
>
>
>
> [As partes desta mensagem que não continham texto foram removidas]





##### ##### #####

Para saber mais visite
http://www.ciencialist.hpg.ig.com.br


##### ##### ##### #####


Yahoo! Grupos, um serviço oferecido por:
PUBLICIDADE




------------------------------------------------------------------------------
Links do Yahoo! Grupos

a.. Para visitar o site do seu grupo na web, acesse:
http://br.groups.yahoo.com/group/ciencialist/

b.. Para sair deste grupo, envie um e-mail para:
ciencialist-unsubscribe@yahoogrupos.com.br

c.. O uso que você faz do Yahoo! Grupos está sujeito aos Termos do Serviço do Yahoo!.



[As partes desta mensagem que não continham texto foram removidas]



SUBJECT: Re: Carbono-14 (era: Sudario)
FROM: "junior_br2001" <junior_br2001@yahoo.com.br>
TO: ciencialist@yahoogrupos.com.br
DATE: 21/03/2005 01:37


Nao é questão disso, para se ter um parecer técnico sobre datação é
preciso ser um técnico, ou ser um químico ou fisico no nínimo,
ninguem iria dar crédito a um biólogo.
Apesar de que um biologo pode ser um fisico amador por conta dele. O
problema é que se ele quiser publicar algo e tentar refutar algo ele
estará que estar tecnicamente credenciado para isso.

Cada qual em seu campo...Ninguém nasce sabendo tudo. temos de dar o
trabalho a pessoa que está capacitada.

Na área de astronomia me parece que amadores podem reltar suas
descobertas, mas no campo da fisica, queimica e biologia, nao sei se
um pode publicar trabalhos fora de suas áreas respectivas na condição
de amadores.

Eu mesmo sou um amador..nao sou nada...mas sou um eterno aprendiz.Tb
nao tenho intenção de publicar nada, e nem refutar oficialmente
ninguém. mesmo porque não posso, e nem estou credenciado para isso

JR

--- Em ciencialist@yahoogrupos.com.br, "Oraculo" <oraculo@a...>
escreveu
."
>
> Então, temos aqui mais um caso de palavra sagrada..:-) (não resisti
a brincadeira..:-)






SUBJECT: Re: [ciencialist] Re: Ad Hominem..:-)
FROM: "Oraculo" <oraculo@atibaia.com.br>
TO: <ciencialist@yahoogrupos.com.br>
DATE: 21/03/2005 01:42

Olá Pubmed

Pubmed: Quando falo céticos tolos e voce acha que foi com voce... fazer o que?
De qualquer forma, quando falei em céticos tolos pela primeira vez,
voce foi o primeiro a responder, eu ainda nem havia entrado em
discussão com voce ainda, eu estava falando em santo sudário e os
pseudo-ceticos da internet."

Sim, disse tudo isso e eu discordo de você em tudo isso..:-) E apresentei argumentos, para refutar a alegação de que pseudo-céticos estavam "contra" o segundo estudo por serem tolos. Estão, como sempre, cuidadosamente avaliando um estudo que ainda não foi repetido nem confirmado. Como fariam com qualquer estudo, de fusão a frio ou com a teoria da deriva dos continentes. O primeiro recusado por não poder ser repetido, o segundo aceito por apresentar evidencias fortes. Simples assim..:-)

Não ligo que ache que céticos são tolos, ou que façam isso ou aquilo, nem que me ache um pseudo-cético..:-) Se fosse apenas isso, sua opinião, ficaria por aqui..:-) Estou debatendo argumentos e evidencias, não opiniões. Opinião todos tem direito. Mas argumentos podem, e devem, ser confrontados.

O estudo que contesta a datação do santo sudário pode, e deve, ser discutido e debatido. E refutado, ou, como não me canso de dizer (mentira, eu me canso sim..:-), ser tomado com a cautela que todo novo estudo que contesta estudos anteriores bem documentados merece. Esperar por repetição e comprovação, qualquer que seja o assunto do estudo.

O que acontece é que, se o estudo, que aguarda confirmação e é tomado com cautela, for sobre um assunto que não interessa a crentes, ele será ignorado. O cuidado com que será recebido passará desapercebido do grande publico. Mas, se o assunto for como este do santo sudario, esse mesmo cuidado será objeto de escandalo pelos que já creem ter a verdade sobre o assunto.

Estudos de novos medicamentos podem levar décadas antes de serem aceitos. A diminuição desse tempo de espera e cuidado tem recebido criticas acidas da comunidade cientifica, por causar problemas como o de recente Viox. Tempo e cautela são necessários nesse caso, mas não se vê (a não ser nos representantes dos grandes laboratorios) uma gritaria para que se aceite logo os resultados dos primeiros estudos, antes mesmo de confirmação independente, sob o argumento de que estão sendo "ceticos demais" ou, como gosta de dizer, "pseudo-céticos".

O cuidado da ciência com novos conhecimentos é geral, mas só provoca espanto quando trata de fenomenos como o santo sudario e afins.

Homero



----- Original Message -----
From: junior_br2001
To: ciencialist@yahoogrupos.com.br
Sent: Monday, March 21, 2005 1:29 AM
Subject: [ciencialist] Re: Ad Hominem..:-)



--- Em ciencialist@yahoogrupos.com.br, "Oraculo" <oraculo@a...>
escreveu
> Olá Pubmed
>
>> E sou cético, e claro que a crítica a céticos tolos e que tais se
refere a mim..:-) Não ligo nem fico chateado, é direito seu, mas
continua sendo "ad hominem" no que importa, não serve como argumento
válido..:-)
>


JR: Mas de qualquer forma não houve ataque dirigido a voce. Voce
sentiu que foi com voce, o que é muito diferente.

Quando falo céticos tolos e voce acha que foi com voce... fazer o que?
De qualquer forma, quando falei em céticos tolos pela primeira vez,
voce foi o primeiro a responder, eu ainda nem havia entrado em
discussão com voce ainda, eu estava falando em santo sudário e os
pseudo-ceticos da internet. Isso corrobora que voce respondeu como
uma reação ao que eu disse. E o ataque crítico aos céticos nem era
com voce. Depois que voce respondeu meu ataque crítico, voce acabou
se envolvendo neurologicamente(ou psicologicamente) com a idéia, e
achou que era com voce pelo fato de voce ser um cético.

JR





##### ##### #####

Para saber mais visite
http://www.ciencialist.hpg.ig.com.br


##### ##### ##### #####


Yahoo! Grupos, um serviço oferecido por:







------------------------------------------------------------------------------
Links do Yahoo! Grupos

a.. Para visitar o site do seu grupo na web, acesse:
http://br.groups.yahoo.com/group/ciencialist/

b.. Para sair deste grupo, envie um e-mail para:
ciencialist-unsubscribe@yahoogrupos.com.br

c.. O uso que você faz do Yahoo! Grupos está sujeito aos Termos do Serviço do Yahoo!.



[As partes desta mensagem que não continham texto foram removidas]



SUBJECT: Re: [ciencialist] Re: Carbono-14 (era: Sudario)
FROM: "Oraculo" <oraculo@atibaia.com.br>
TO: <ciencialist@yahoogrupos.com.br>
DATE: 21/03/2005 01:50

Olá Pubmed

Está desviando do assunto (porque não me espanto com isso?..:-) Não estamos confrontando a opinião do Takata com a opinião do Taborda. O Takata apresentou diversas referencias bibliograficas, estudos de confirmação, pesquisas com carbono-14, etc, etc, etc, todas solenemente ignoradas por você. A opinião do Taborda e a do Takata nada tem a ver com isso.

A questão é que a datação com carbono-14 , bem testada, bem conhecida, bem embasada, bem documentada, etc, foi contraposta ao estudo de Rogers, sem comprovação independente. Até que isso surja, comprovação independente, o método originalmente usado, carbono-14, tem maior confiabilidade, não para mim ou o Takata, mas para a comunidade cientifica, que o método usado no novo estudo. Isso pode mudar? Claro, todo conhecimento pode mudar, mas neste momento não há evidencias que suportem essa mudança.

A datação com carbono-14 é uma forma de determinar datas. A técnica usada por Rogers não, a data é uma consequencia interpretativa dos resultados, que depende de diversos fatores. Pode estar correta? Claro, mas as evidencias são fracas neste momento. Como apontou um co-listeiro, até mesmo a amplitude dos resultados, de 3000 anos, indica pouca precisão na técnica.

Escolhe a opinião do Taborda? Direito seu, inalienável..:-) Mas isso nada tem a ver com o debate.

Homero


----- Original Message -----
From: junior_br2001
To: ciencialist@yahoogrupos.com.br
Sent: Monday, March 21, 2005 1:37 AM
Subject: [ciencialist] Re: Carbono-14 (era: Sudario)



Nao é questão disso, para se ter um parecer técnico sobre datação é
preciso ser um técnico, ou ser um químico ou fisico no nínimo,
ninguem iria dar crédito a um biólogo.
Apesar de que um biologo pode ser um fisico amador por conta dele. O
problema é que se ele quiser publicar algo e tentar refutar algo ele
estará que estar tecnicamente credenciado para isso.

Cada qual em seu campo...Ninguém nasce sabendo tudo. temos de dar o
trabalho a pessoa que está capacitada.

Na área de astronomia me parece que amadores podem reltar suas
descobertas, mas no campo da fisica, queimica e biologia, nao sei se
um pode publicar trabalhos fora de suas áreas respectivas na condição
de amadores.

Eu mesmo sou um amador..nao sou nada...mas sou um eterno aprendiz.Tb
nao tenho intenção de publicar nada, e nem refutar oficialmente
ninguém. mesmo porque não posso, e nem estou credenciado para isso

JR

--- Em ciencialist@yahoogrupos.com.br, "Oraculo" <oraculo@a...>
escreveu
."
>
> Então, temos aqui mais um caso de palavra sagrada..:-) (não resisti
a brincadeira..:-)






##### ##### #####

Para saber mais visite
http://www.ciencialist.hpg.ig.com.br


##### ##### ##### #####


Yahoo! Grupos, um serviço oferecido por:







------------------------------------------------------------------------------
Links do Yahoo! Grupos

a.. Para visitar o site do seu grupo na web, acesse:
http://br.groups.yahoo.com/group/ciencialist/

b.. Para sair deste grupo, envie um e-mail para:
ciencialist-unsubscribe@yahoogrupos.com.br

c.. O uso que você faz do Yahoo! Grupos está sujeito aos Termos do Serviço do Yahoo!.



[As partes desta mensagem que não continham texto foram removidas]



SUBJECT: Ad Hominem e FAMOSOS..:-)
FROM: "L.E.R.de Carvalho" <lecarvalho@infolink.com.br>
TO: ciencialist@yahoogrupos.com.br
DATE: 21/03/2005 01:58

At 01:42 21/3/2005, you wrote:
>Olá Pubmed
>
>Pubmed: Quando falo céticos tolos e voce acha que foi com voce... fazer o que?



Eu tinha certeza que vocês iriam acabar brigando.

Isso é científico.

Dá até pra estabelecer uma Lei.

Enfim... causa e efeito nas ciencias sociais.

Voces ainda serão internacionalmente famosos.

L.E.

[As partes desta mensagem que não continham texto foram removidas]



SUBJECT: Re: Carbono-14 (era: Sudario)
FROM: "junior_br2001" <junior_br2001@yahoo.com.br>
TO: ciencialist@yahoogrupos.com.br
DATE: 21/03/2005 02:02


--- Em ciencialist@yahoogrupos.com.br, "Oraculo" <oraculo@a...>
escreveu
> Olá Pubmed
> A questão é que a datação com carbono-14 , bem testada, bem
conhecida, bem embasada, bem documentada, etc, foi contraposta ao
estudo de Rogers, sem comprovação independente.> Escolhe a opinião do
Taborda? Direito seu, inalienável..:-) Mas isso nada tem a ver com o
debate.
>


JR: Não entendo essa sua posição cientifista de grandeza. Por favor,
nao compare uma datação de 88 com uma datação dos dias atuais. As
técnicas sofreram mudanças. E o que está bem documentado? Que o
Sudário nao passou todos os testes que deveria?

Jr







SUBJECT: Re: [ciencialist] Re: Carbono-14 (era: Sudario)
FROM: "Oraculo" <oraculo@atibaia.com.br>
TO: <ciencialist@yahoogrupos.com.br>
DATE: 21/03/2005 02:15

Olá Pubmed

Pubmed: JR: Não entendo essa sua posição cientifista de grandeza. Por favor,
nao compare uma datação de 88 com uma datação dos dias atuais. As
técnicas sofreram mudanças. E o que está bem documentado? Que o
Sudário nao passou todos os testes que deveria?"

Não, não passou e nem vai passar. Sua proprietária, maior interessada em manter o mistério (e esconder a verdade, digasse..:-) não vai permitir. Nunca. Então temos de escolher, entre estudos disponíveis, os que apresentam maior confiabilidade e segurança. O fato de que o estudo foi feito em 88 não significa, necessáriamente, que é incorreto ou menos preciso. O método sofreu poucas mudanças justamente por ser simples e confiável.

O novo estudo talvez seja mais eficiente e se torne um padrão de datação a partir de agora. É possível. Mas, para que assim seja, tem de ser repetido e confimado. Antes disso, é só mais um teste. Só isso. Nada de cientificista (o que quer que pense, como já disse, que isso signifique, além de um "ataque" a ciência..:-)

Em medos dos anos 60 fotos da Terra foram tiradas do espaço, confimando empiricamente que esta é redonda. Diria que essas evidencias são menos válidas porque foram produzidas na década de 60? Que novas decobertas e novas tecnologias superam essa evidencia em confiabilidade? Que podemos descobrir, com um novo método, que a Terra é quadrada? Claro que não..:-) O mesmo para carbono -14, não vai ser substituido pelo tempo simplesmente, mas por métodos que demonstrem, sem (muitas) dúvidas, ser mais eficiente. Ainda esperamos por isso quanto ao novo estudo, apenas isso.

Não entendo essa sua posição obscurantista e anti-cientifica..:-)

Homero



----- Original Message -----
From: junior_br2001
To: ciencialist@yahoogrupos.com.br
Sent: Monday, March 21, 2005 2:02 AM
Subject: [ciencialist] Re: Carbono-14 (era: Sudario)



--- Em ciencialist@yahoogrupos.com.br, "Oraculo" <oraculo@a...>
escreveu
> Olá Pubmed
> A questão é que a datação com carbono-14 , bem testada, bem
conhecida, bem embasada, bem documentada, etc, foi contraposta ao
estudo de Rogers, sem comprovação independente.> Escolhe a opinião do
Taborda? Direito seu, inalienável..:-) Mas isso nada tem a ver com o
debate.
>


JR: Não entendo essa sua posição cientifista de grandeza. Por favor,
nao compare uma datação de 88 com uma datação dos dias atuais. As
técnicas sofreram mudanças. E o que está bem documentado? Que o
Sudário nao passou todos os testes que deveria?

Jr







##### ##### #####

Para saber mais visite
http://www.ciencialist.hpg.ig.com.br


##### ##### ##### #####


Yahoo! Grupos, um serviço oferecido por:

São Paulo Rio de Janeiro Curitiba Porto Alegre Belo Horizonte Brasília




------------------------------------------------------------------------------
Links do Yahoo! Grupos

a.. Para visitar o site do seu grupo na web, acesse:
http://br.groups.yahoo.com/group/ciencialist/

b.. Para sair deste grupo, envie um e-mail para:
ciencialist-unsubscribe@yahoogrupos.com.br

c.. O uso que você faz do Yahoo! Grupos está sujeito aos Termos do Serviço do Yahoo!.



[As partes desta mensagem que não continham texto foram removidas]



SUBJECT: Re: [ciencialist] Confiabilidade (era Santo sudário e a vanilina(correção)
FROM: "Alberto Mesquita Filho" <albmesq@uol.com.br>
TO: <ciencialist@yahoogrupos.com.br>
DATE: 21/03/2005 02:47

----- Original Message -----
From: "Oraculo"
Sent: Sunday, March 20, 2005 11:51 PM
Subject: [ciencialist] Confiabilidade (era Santo sudário e a
vanilina(correção)

Olá Homero

> > Alberto:mesmo porque não há como assumir uma postura científica
> > deixando-se o ceticismo totalmente de lado."

> Tente explicar isso a seu discipulo, o Pubmed..:-)

Seria uma honra ser professor do Júnior assim como, e a depender do assunto,
seria também uma honra ser aluno do Júnior. Mas, como não estamos numa
escola tradicional onde uns sempre ensinam e outros sempre aprendem, mas na
escola da vida, onde até mesmo o relacionamento professor/aluno é relativo,
o máximo que posso dizer é que sinto-me honrado em poder contar com
debatedores de elevado nível como você, o Júnior, o Takata, o Taborda, o
Eduardo e tantos outros. Há sempre o que aprender até mesmo num diálogo
insosso como esse e com grande probabilidade de não nos levar a mais nada
além desse aprendizado, o que já é muito.

> E a confiabilidade neste caso é simplesmente uma questão de possibilidade
> de ser repetido e encontrar os mesmos resultados.

Eu não chamaria isso de confiabilidade, mas de critério de cientificidade,
mas nem todos concordariam com isso. Popper, por exemplo, estabelece outro
critério de cientificidade, a meu ver bastante criticável, ainda que a
apoiar-se num conceito que, por si só, está *quase* isento de críticas, a
falseabilidade (em ciência, assim como em filosofia, esse quase é
fundamental e nem é preciso se dizer cético para se dar conta disso).

> Os testes de carbono-14,
> mais de um, passaram nesta exigencia e, por enquanto, são confiáveis, até
> onde um conhecimento cientifico pode ser confiável.

Pois é, então creio que você deve ter entendido o que procurei dizer no
parágrafo anterior, se bem que sem utilizar nenhum teste específico, mesmo
porque não entendo nada do teste referido (carbono-14) e também não sei
quase nada sobre o objeto de estudo que vocês tanto falam (sudário).

> Isto é, não uma
> certeza de 100%, mas uma probabilidade forte, e a espera por refutações
> pelo menos tão solidas quanto o estudo em questão.

Mas se o teste já passou pelo critério da repetibilidade, não há mais o que
ser dito a respeito. O teste será válido sim, e não há porque objetivá-lo
com um número. Ou trata-se de um teste científico (neste caso o índice
poderia ser 1 ou 100) ou não se trata de um teste científico (e neste caso o
índice seria zero). Isso a meu ver não tem nada a ver com o que chamei
confiabilidade. O termo confiabilidade parece-me ter sido primeiramente
expresso aqui pelo Takata, e eu entrei na discussão justamente procurando
saber o que ele queria dizer com isso.

Quero crer, mas pode ser que esteja enganado, que a confiabilidade no caso
está a se referir a uma comparação entre dois métodos diversos, e nesse caso
a dúvida seria saber se um dos dois é mais ou menos confiável do que o
outro, obviamente aceitando-se a premissa de que os dois sejam científicos.
Perceba que não estou dizendo que precisariam ser 100% científicos, pois
isso soaria como uma redundância. Por outro lado, ser científico não
significa exprimir uma verdade absoluta, e quero crer que todos aqui da
Ciencialist, ou pelo menos os mais veteranos, concordam com isso.

Seria possível objetivar essa maior confiabilidade em um dos métodos do que
em relação ao outro? Seria possível atribuir um peso a essa confiabilidade?
Em outras palavras: Poderiamos estabelecer uma escala de valores, por
exemplo, de 0 a 100? Ora, como diria Popper, "Não existem observações puras:
elas estão impregnadas pelas teorias e são orientadas pelos problemas e
acompanhadas pelas teorias." Essa idéia não é de Popper, e acredito que os
filósofos seiscentistas já se depararam com esse problema. Esse problema
surge sempre que tentamos analisar em maior profundidade o chamado "problema
da indução" e/ou o "problema da corroboração" (que é o nosso caso atual). Se
um método A reproduz um resultado em 90% das vezes em que me utilizo dele,
enquanto outro B o faz em apenas 60% dos casos, isso pode me convencer (a
despeito de poder estar errado) de que A é melhor do que B, mas isso não me
autoriza dar a nota 90 (ou 9) a A e 60 (ou 6) a B. Isso porque as teorias
que impregnam tais metodologias via de regra (quase com certeza) não são as
mesmas e até mesmo os critérios que utilizamos para chegar nesses valores,
seja o 90%, seja o 60%, estão contaminados pelas teorias utilizadas.

Espero com isso ter mostrado *uma das razões* porque acho que não é fácil
arcar com o ônus do ceticismo, e é por isso que não me aventuro a dizer-me
cético com respeito àquilo que não entendo, pois isso não é postura digna de
um cético, mas sim de um ignorante. A medida em que essa ignorância, seja
ela por ingenuidade (ignorância sócio-disfuncional, dentre outras), seja ela
do tipo reflexivo (a ignorância dos arrogantes, dentre outras) adquire o
status de sabedoria (no extremo o yin se transforma no yang, para usar um
ditado oriental), acaba por nos levar a um consensualismo via de regra
irracional, posto que movido mais pela apatia, pelo conformismo, pelo
modismo ou pelo comodismo (é mais fácil concordar do que subverter). É nesse
sentido que eu digo que ou se estuda, e muito, o ceticismo, ou é melhor não
dizer nada, sob pena de sermos confundidos com os aproveitadores da
ingenuidade popular. Sinceramente, é assim que vejo a grande maioria desses
falsos movimentos céticos que pululam pela Internet. Não me parece que nem
você nem o Takata, nem muitos outros com quem tenho debatido esse assunto
aqui na Ciencialist, estejam enquadrados na categoria dessa corja a que me
referi acima (creio já ter cruzado com alguns) mas não acho impossível que
vocês estejam alimentando um monstro.

> Por exemplo, entre um medicamente que, após passar por diversos estudos de
> controle e repetição, recebeu aval para ser aplicado contre determinada
> patologia, e um novo medicamente que, apesar de ter apresentado efeitos
> melhores que o anterior em um estudo de pesquisa, ainda espera por estudos
> de repetição, seria mais seguro e "confiável", receitar o primeiro
> medicamento. Ou não?

Quero crer que neste caso não estaríamos mais discutindo a produção de
conhecimentos científicos, mas sim estaríamos frente a uma aplicação da
ciência e neste caso precisaríamos pensar não mais na "ciência pela
ciência", mas na "ciência como um bem social". Esse assunto é bastante
complexo, ainda que não tão complicado que não possa ser discutido aqui, mas
neste caso teríamos que fugir da temática atual, pois não haveria como
deixar de lado certos fatores importantes, como a ética profissional e até
mesmo aspectos regionais (medicina de guerra, por exemplo). Comecei minha
vida acadêmica nesta praia (aplicação da ciência, exatamente na medicina
clínica e bastante em terapia intensiva, que na década de 70 assemelhava-se
muito com a medicina de frente de batalha) e vejo uma distância enorme entre
esses dois campos, ainda que de uma maneira ou de outra sejam campos
interligados e indissociáveis. Em menor grau, parece-me que a "briguinha"
entre vocês relaciona-se também a essa diferença. O Júnior e o Taborda estão
preocupados com a discussão dos métodos em si (ciência pela ciência), e você
e o Takata estão mais preocupados com a maneira como os métodos têm sido
utilizados para satisfazer vaidades extracientíficas (no caso, de índole
religiosa ou contra-religiosa). Aliás, esse é um outro assunto importante e
a denotar uma postura típica e a retratar a preocupação maior daqueles que
se engajam nos movimentos céticos acima referidos. Vejo nesses movimentos
muito de vaidade e muito pouco de ciência.

É o que tinha a dizer e perdoe-me se fui por demais sincero (o que não
significa que tenha dito somente verdades), mas não sei discutir de outra
maneira.

[ ]´s
Alberto
http://ecientificocultural.com/indice.htm
Mas indiferentemente a tudo isso, o neutrino tem massa, o elétron não é
uma carga elétrica coulombiana e a Terra se move. E a história se repetirá.



SUBJECT: Confiabilidade (era Santo sudário e a vanilina(correção)
FROM: "Oraculo" <oraculo@atibaia.com.br>
TO: <ciencialist@yahoogrupos.com.br>
DATE: 21/03/2005 03:08

Olá Alberto

Não é o caso de se desculpar, nada do que disse é ofensivo ou minimamente descortez..:-) E acho impressionante como é possível termos tantas discordancias e ao mesmo tempo, eu concordar com suas colocações em quase tudo..:-)

Sua mensagem está perfeitamente clara e é perfeitamente aceitável, inclusive quanto a preocupação dos debatedores deste santo sudário. O que não significa, é claro, que eu não pense de forma diferente sobre algumas conclusões que tira a partir dessas colocações. Mas tem razão, muitos dos problemas que aponta seriam motivo para longas e diferentes discussões, com desdobramentos que iriam bem longe desta questão do sudário.

Sobre a confiabilidade, neste caso é uma questão mais pragmática que filosofica. O novo estudo pretende refutar os anteriores, determinando uma nova data de origem para um artefato que, sem sombra de duvida, tem um componente subjetivo, religioso, importante. Para mim, ou para a ciência, não importa quem está correto, desse ponto de vista. Vivo bem com um sudário criado em 1340, quanto com um sudário de 3000 anos..:-)

Mas, ao ter de escolher um conjunto de dados e conclusões sobre essa data, vou ser conservador e cuidadoso, e ficar com estudos e métodos bem conhecidos, no lugar da novidade. Que, embora possa estar correta, ninguém disse o contrário, deve esperar por mais confirmações e evidencias (até pela imprecisão dos resultados, 3000 anos de possibilidades).

Escolha cética e pragmática, como seria de se esperar de um cético..:-)

Um abraço.

Homero


----- Original Message -----
From: Alberto Mesquita Filho
To: ciencialist@yahoogrupos.com.br
Sent: Monday, March 21, 2005 2:47 AM
Subject: Re: [ciencialist] Confiabilidade (era Santo sudário e a vanilina(correção)


----- Original Message -----
From: "Oraculo"
Sent: Sunday, March 20, 2005 11:51 PM
Subject: [ciencialist] Confiabilidade (era Santo sudário e a
vanilina(correção)

Olá Homero

> > Alberto:mesmo porque não há como assumir uma postura científica
> > deixando-se o ceticismo totalmente de lado."

> Tente explicar isso a seu discipulo, o Pubmed..:-)

Seria uma honra ser professor do Júnior assim como, e a depender do assunto,
seria também uma honra ser aluno do Júnior. Mas, como não estamos numa
escola tradicional onde uns sempre ensinam e outros sempre aprendem, mas na
escola da vida, onde até mesmo o relacionamento professor/aluno é relativo,
o máximo que posso dizer é que sinto-me honrado em poder contar com
debatedores de elevado nível como você, o Júnior, o Takata, o Taborda, o
Eduardo e tantos outros. Há sempre o que aprender até mesmo num diálogo
insosso como esse e com grande probabilidade de não nos levar a mais nada
além desse aprendizado, o que já é muito.

> E a confiabilidade neste caso é simplesmente uma questão de possibilidade
> de ser repetido e encontrar os mesmos resultados.

Eu não chamaria isso de confiabilidade, mas de critério de cientificidade,
mas nem todos concordariam com isso. Popper, por exemplo, estabelece outro
critério de cientificidade, a meu ver bastante criticável, ainda que a
apoiar-se num conceito que, por si só, está *quase* isento de críticas, a
falseabilidade (em ciência, assim como em filosofia, esse quase é
fundamental e nem é preciso se dizer cético para se dar conta disso).

> Os testes de carbono-14,
> mais de um, passaram nesta exigencia e, por enquanto, são confiáveis, até
> onde um conhecimento cientifico pode ser confiável.

Pois é, então creio que você deve ter entendido o que procurei dizer no
parágrafo anterior, se bem que sem utilizar nenhum teste específico, mesmo
porque não entendo nada do teste referido (carbono-14) e também não sei
quase nada sobre o objeto de estudo que vocês tanto falam (sudário).

> Isto é, não uma
> certeza de 100%, mas uma probabilidade forte, e a espera por refutações
> pelo menos tão solidas quanto o estudo em questão.

Mas se o teste já passou pelo critério da repetibilidade, não há mais o que
ser dito a respeito. O teste será válido sim, e não há porque objetivá-lo
com um número. Ou trata-se de um teste científico (neste caso o índice
poderia ser 1 ou 100) ou não se trata de um teste científico (e neste caso o
índice seria zero). Isso a meu ver não tem nada a ver com o que chamei
confiabilidade. O termo confiabilidade parece-me ter sido primeiramente
expresso aqui pelo Takata, e eu entrei na discussão justamente procurando
saber o que ele queria dizer com isso.

Quero crer, mas pode ser que esteja enganado, que a confiabilidade no caso
está a se referir a uma comparação entre dois métodos diversos, e nesse caso
a dúvida seria saber se um dos dois é mais ou menos confiável do que o
outro, obviamente aceitando-se a premissa de que os dois sejam científicos.
Perceba que não estou dizendo que precisariam ser 100% científicos, pois
isso soaria como uma redundância. Por outro lado, ser científico não
significa exprimir uma verdade absoluta, e quero crer que todos aqui da
Ciencialist, ou pelo menos os mais veteranos, concordam com isso.

Seria possível objetivar essa maior confiabilidade em um dos métodos do que
em relação ao outro? Seria possível atribuir um peso a essa confiabilidade?
Em outras palavras: Poderiamos estabelecer uma escala de valores, por
exemplo, de 0 a 100? Ora, como diria Popper, "Não existem observações puras:
elas estão impregnadas pelas teorias e são orientadas pelos problemas e
acompanhadas pelas teorias." Essa idéia não é de Popper, e acredito que os
filósofos seiscentistas já se depararam com esse problema. Esse problema
surge sempre que tentamos analisar em maior profundidade o chamado "problema
da indução" e/ou o "problema da corroboração" (que é o nosso caso atual). Se
um método A reproduz um resultado em 90% das vezes em que me utilizo dele,
enquanto outro B o faz em apenas 60% dos casos, isso pode me convencer (a
despeito de poder estar errado) de que A é melhor do que B, mas isso não me
autoriza dar a nota 90 (ou 9) a A e 60 (ou 6) a B. Isso porque as teorias
que impregnam tais metodologias via de regra (quase com certeza) não são as
mesmas e até mesmo os critérios que utilizamos para chegar nesses valores,
seja o 90%, seja o 60%, estão contaminados pelas teorias utilizadas.

Espero com isso ter mostrado *uma das razões* porque acho que não é fácil
arcar com o ônus do ceticismo, e é por isso que não me aventuro a dizer-me
cético com respeito àquilo que não entendo, pois isso não é postura digna de
um cético, mas sim de um ignorante. A medida em que essa ignorância, seja
ela por ingenuidade (ignorância sócio-disfuncional, dentre outras), seja ela
do tipo reflexivo (a ignorância dos arrogantes, dentre outras) adquire o
status de sabedoria (no extremo o yin se transforma no yang, para usar um
ditado oriental), acaba por nos levar a um consensualismo via de regra
irracional, posto que movido mais pela apatia, pelo conformismo, pelo
modismo ou pelo comodismo (é mais fácil concordar do que subverter). É nesse
sentido que eu digo que ou se estuda, e muito, o ceticismo, ou é melhor não
dizer nada, sob pena de sermos confundidos com os aproveitadores da
ingenuidade popular. Sinceramente, é assim que vejo a grande maioria desses
falsos movimentos céticos que pululam pela Internet. Não me parece que nem
você nem o Takata, nem muitos outros com quem tenho debatido esse assunto
aqui na Ciencialist, estejam enquadrados na categoria dessa corja a que me
referi acima (creio já ter cruzado com alguns) mas não acho impossível que
vocês estejam alimentando um monstro.

> Por exemplo, entre um medicamente que, após passar por diversos estudos de
> controle e repetição, recebeu aval para ser aplicado contre determinada
> patologia, e um novo medicamente que, apesar de ter apresentado efeitos
> melhores que o anterior em um estudo de pesquisa, ainda espera por estudos
> de repetição, seria mais seguro e "confiável", receitar o primeiro
> medicamento. Ou não?

Quero crer que neste caso não estaríamos mais discutindo a produção de
conhecimentos científicos, mas sim estaríamos frente a uma aplicação da
ciência e neste caso precisaríamos pensar não mais na "ciência pela
ciência", mas na "ciência como um bem social". Esse assunto é bastante
complexo, ainda que não tão complicado que não possa ser discutido aqui, mas
neste caso teríamos que fugir da temática atual, pois não haveria como
deixar de lado certos fatores importantes, como a ética profissional e até
mesmo aspectos regionais (medicina de guerra, por exemplo). Comecei minha
vida acadêmica nesta praia (aplicação da ciência, exatamente na medicina
clínica e bastante em terapia intensiva, que na década de 70 assemelhava-se
muito com a medicina de frente de batalha) e vejo uma distância enorme entre
esses dois campos, ainda que de uma maneira ou de outra sejam campos
interligados e indissociáveis. Em menor grau, parece-me que a "briguinha"
entre vocês relaciona-se também a essa diferença. O Júnior e o Taborda estão
preocupados com a discussão dos métodos em si (ciência pela ciência), e você
e o Takata estão mais preocupados com a maneira como os métodos têm sido
utilizados para satisfazer vaidades extracientíficas (no caso, de índole
religiosa ou contra-religiosa). Aliás, esse é um outro assunto importante e
a denotar uma postura típica e a retratar a preocupação maior daqueles que
se engajam nos movimentos céticos acima referidos. Vejo nesses movimentos
muito de vaidade e muito pouco de ciência.

É o que tinha a dizer e perdoe-me se fui por demais sincero (o que não
significa que tenha dito somente verdades), mas não sei discutir de outra
maneira.

[ ]´s
Alberto
http://ecientificocultural.com/indice.htm
Mas indiferentemente a tudo isso, o neutrino tem massa, o elétron não é
uma carga elétrica coulombiana e a Terra se move. E a história se repetirá.



##### ##### #####

Para saber mais visite
http://www.ciencialist.hpg.ig.com.br


##### ##### ##### #####


Yahoo! Grupos, um serviço oferecido por:







------------------------------------------------------------------------------
Links do Yahoo! Grupos

a.. Para visitar o site do seu grupo na web, acesse:
http://br.groups.yahoo.com/group/ciencialist/

b.. Para sair deste grupo, envie um e-mail para:
ciencialist-unsubscribe@yahoogrupos.com.br

c.. O uso que você faz do Yahoo! Grupos está sujeito aos Termos do Serviço do Yahoo!.



[As partes desta mensagem que não continham texto foram removidas]



SUBJECT: Re: Carbono-14 (era: Sudario)
FROM: "junior_br2001" <junior_br2001@yahoo.com.br>
TO: ciencialist@yahoogrupos.com.br
DATE: 21/03/2005 03:10


--- Em ciencialist@yahoogrupos.com.br, "Oraculo" <oraculo@a...>
escreveu
>> Em medos dos anos 60 fotos da Terra foram tiradas do espaço,
confimando empiricamente que esta é redonda. Diria que essas
evidencias são menos válidas porque foram produzidas na década de 60?
Que novas decobertas e novas tecnologias superam essa evidencia em
confiabilidade? Que podemos descobrir, com um novo método, que a
Terra é quadrada? Claro que não..:-) O mesmo para carbono -14, não
vai ser substituido pelo tempo simplesmente, mas por métodos que
demonstrem, sem (muitas) dúvidas, ser mais eficiente. Ainda esperamos
por isso quanto ao novo estudo, apenas isso.


JR: Mas , hoje em dia temos fotos a cores mais detalhadas, temos um
dominio maior das técnicas de fotografia que pode mostrar detalhes
que a técnica anterior de fotografia nao revelou

A mesma coisa, em 88 me parece que a técnica do carbono-14 inda
estava no inicio. Hoje em dia se o teste for relizado ele será bem
mais preciso.
Assim como os 3 estudos independentes mostraram diferenças, imagine
na época de hoje como seria se o manto pudesse ser submetido a novo
teste, principalmente nas partes onde não há remendos.

Portanto teste realizado em 88 já deve estar ultrapassado, levando em
consideraçãao o que poderíamos descobrir com esse manto nos dias de
hoje. Este tipo de coisa é previsível

Voce nao vai querer comparar um motor de carro de 1920 com os motores
atuais. O mesmo vale para as técnicaas e dominio de datação pelo
carbono-14 e hoje já temos técnicas com radioisótopos bem mais
potentes para datar além de 50.000 anos, superios a dez meias-vidas,
como a do argonio-potássio e outras. Claro que nao é o caso do manto,
mas outros objetos e ossadas paleontológicas etc...

Portanto é melhor nao ficar preso a esta data do manto, que
certamente um novo teste indicará outro resultado
Isso é perfeitamente previsível

JR





SUBJECT: Com amor para os amigos
FROM: Maria Natália <grasdic@hotmail.com>
TO: ciencialist@yahoogrupos.com.br
DATE: 21/03/2005 04:28


http://science.nasa.gov/headlines/y2005/images/moonfirst/iss010e18585.
jpg
Perceberam porque MoonFirst?

Um abraço
Maria Natália






SUBJECT: Re: [ciencialist] Re: ninguem é inocente.
FROM: TARCISIO BORGES <tbs97@fisica.ufpr.br>
TO: ciencialist@yahoogrupos.com.br
DATE: 21/03/2005 08:09

Mas para o teste de C14 a peça é queimada. De onde ele tirou a idéia de
que a peça queimada possuía vinila?

Não importa quantos testes de façam, sempre vai existir alguém que vai
contestar, mesmo que se queime o sudário inteiro.

[]s
TARCISIO BORGES
tbs97@fisica.ufpr.br

On Fri, 18 Mar 2005, pubmed2005 wrote:

>
>
> O correto seria refazer todos os testes a respeito do Sudário,
> inclusive a do carbono-14 em diferentes partes do tecido, senão tb se
> poderia argumentar que o teste nao foi completo e tb apresenta
> falhas . Depois, se realizar os testes químicos para comparar.
>
> O parecer atual da ciencia ainda continua meio escorregativo - pois a
> parte analisada do carbono 14 pode ter sido muito bem a remendada que
> possuía vinila(uma sibstancia que nao resiste ao envelhecimento)
>
> Acho muito pertinente a nova dewcoberta do Rogers e em se tratando de
> ciencia, fico com ela ate o momento.
>



SUBJECT: Re: [ciencialist] Re: ninguem é inocente.
FROM: TARCISIO BORGES <tbs97@fisica.ufpr.br>
TO: ciencialist@yahoogrupos.com.br
DATE: 21/03/2005 08:24

On Fri, 18 Mar 2005, pubmed2005 wrote:
> O fato de o primeiro estudo ter sido bem sucedido nao invalida o
> segundo estudo, que pode ser muito bem um complemento e até mesmo um
> chamado para falhas do primeiro estudo.

Invalida sim. O primeiro estudo [C14] deu uma data de 700 anos +/- 60
anos. O segundo estudo deu uma data entre 1300 e 3000 anos.

Pelo que pude perceber as duas datas não possuem intercecção e sendo ambos
os métodos aceitos, um dos dois está errado.


> A ciencia nao para...o fato que descobertas novas sempre demoram a
> ser aceitas porque os cientistas se apegam ao dogma antigo.

Na verdade os cientistas se apegam ao que funciona. Se o antigo funciona,
mas o novo funciona melhor, os cientistas vão ter uma certa resistência em
aceitar o novo, pois o antigo ainda vai muito bem.


> O segundo estudo foi muito pertinente,e merece atençao científica. O
> fato de a pesquisa ter sido feita por um químico aposentado envolve
> muitos preconceitos tb da comunidade científica, etc e tal...

Não consigo enxergar estes preconceitos. O Stephen William Hawking possui
a idade do meu pai e nem por isso é desacreditado, mesmo admitindo que
errou em sua teoria.

Como já disse em outro email, o preconceito está na metodologia usada por
Rogers.

[]s
TARCISIO BORGES
tbs97@fisica.ufpr.br



SUBJECT: Re: [ciencialist] Re: ninguem é inocente.
FROM: TARCISIO BORGES <tbs97@fisica.ufpr.br>
TO: ciencialist@yahoogrupos.com.br
DATE: 21/03/2005 08:32

On Fri, 18 Mar 2005, Oraculo wrote:
> A Igreja é que inicialmente apresentou o caso, o sudário, como
> reliquia autentica e com as afirmações sobre sua origem e utilização.
> A mesma igreja que, brincando de esconde-esconde, as vezes permite, as
> vezes nega, o estudo confiável ao artefato.

O Vaticano não assume a autenticidade do artefato. Quem defende sua
autenticidade são alguns bispos e padres.


[]s
TARCISIO BORGES
tbs97@fisica.ufpr.br



SUBJECT: Atomistica: Ainda se espera por melhores teorias ?
FROM: Paulo Sérgio Dias <psdlistdisc@terra.com.br>
TO: <ciencialist@yahoogrupos.com.br>
DATE: 21/03/2005 08:33


Pessoal:

Sou praticamente analfabeto em Quimica (nao
sei se eu posso dizer que ja' conheco o a-e-i-o-u).

A pergunta que vou fazer e' menos para aprender o assunto
em si, e mais para tentar entender como a ciencia atual
esta' seguindo seu caminho (embora suas dicas sejam desejaveis,
vou postar minhas duvidas em uma lista especifica de Quimica)

La' vai:

Estou cursando a disciplina de Quimica Geral...
(no meu curso de licenciatura em Fisica).

a) Excecoes `a regra do octeto
--------------------------------------------------------

Ao pesquisar sobre "ligacoes ionicas" e "ligacoes covalentes",
percebi que na Quimica parecem existir muitas excecoes,
que as "regras" gerais nao abrangem.

Por exemplo, no site http://inorgan221.iq.unesp.br/quimgeral/respostas/dativa.html
e' mostrada uma situacao em que o Boro (familia 3A ou 13) faz ligacoes covalentes
com o cloro, mas ainda assim nao consegue completar 8 eletrons na sua ultima camada:

Cl
|
Cl - B - Cl

No entanto, segundo o autor do site, o atomo fica "estavel". Isso entao e' uma excecao
`a regra do octeto ? O Boro nao poderia (segundo a teoria) formar ligacoes apenas
ligacoes ionicas, "cedendo" seus eletrons a atomos carentes de eletrons ?
Isto realmente e' uma excecao, ou em algum topico mais aprofundado de atomistica
esse comportamento do Boro ja' e' satisfatoriamente explicado ?


b) Outra topico que apresenta diversas excecoes e' a "distribuicao" eletronica
em subniveis (spdf). Pelo que li nos livros de Quimica, os elementos do
grupo "d" da tabela periodica nao seguem o padrao da distribuicoes dos grupos
s e p. As principais excecoes seriam o Cobre, Cromo, Prata e Ouro.
Essas excecoes possuem alguma explicacao, em algum topico mais avancado
da atomistica ? Ou ainda se espera por uma teoria que explique essas excecoes ?

c) No mesmo site, ha' um paragrafo que considerei estranho. O autor diz que ao se colocar o BCl3 junto com
o NH3, o Nitrogenio "doa" seus dois eletrons nao-ligantes ao Boro (que estava com falta de dois eletrons).
O termo "doar" significa que houve a chamada "ligacao covalente dativa" ? Ou realmente o N transfere seus
dois atomos para o B ? (como ocorre em ligacoes ionicas) ???
O paragrafo e' este:

"A natureza é sábia: colocando os dois compostos em contato, eles farão o que podemos chamar de "aduto": o nitrogênio "doa" o seu
par não utilizado par o boro, que assim completa o seu octeto Graficamente": (e vem a representacao da ligacao, logo abaixo).


d) Finalmente... que outros topicos da atomistica apresenta muitas excecoes `a espera
de uma teoria mais abrangente ?

Grato pela atencao de todos.

Paulo

[As partes desta mensagem que não continham texto foram removidas]



SUBJECT: Re: [ciencialist] Re: ninguem é inocente.
FROM: TARCISIO BORGES <tbs97@fisica.ufpr.br>
TO: ciencialist@yahoogrupos.com.br
DATE: 21/03/2005 08:35

Talvez Buda o fosse, pois em determinado momento diz aos seus discípulos:

Não acredite em seus professores.

:-)

[]s
TARCISIO BORGES
tbs97@fisica.ufpr.br

On Fri, 18 Mar 2005, Alberto Mesquita Filho wrote:
> Existem céticos de verdade? Afinal, o que é ceticismo? Ou então: O que é
> verdade?
> Não precisa responder. Eu só queria perguntar. ;-))
> Alberto



SUBJECT: Re: [ciencialist] Re: ninguem é inocente.
FROM: TARCISIO BORGES <tbs97@fisica.ufpr.br>
TO: ciencialist@yahoogrupos.com.br
DATE: 21/03/2005 08:38

On Sat, 19 Mar 2005, pubmed2005 wrote:
> Põe uma coisa na sua cabeça
> Ninguem aqui disse (pelo menos eu não disse) que o santo sudário tem
> que ser o manto de Jesus.

O que vc afirma é verdade. Quem insistiu que o sudário não tinha nada a
ver com cristo foi vc mesmo, quando ninguém até então tinha feito tal
afirmação ou a negado.

[]s
TARCISIO BORGES
tbs97@fisica.ufpr.br



SUBJECT: Re: [ciencialist] Re: ninguem é inocente.
FROM: TARCISIO BORGES <tbs97@fisica.ufpr.br>
TO: ciencialist@yahoogrupos.com.br
DATE: 21/03/2005 08:46

É tão triste quando o questionado não entende a pergunta...

O que o oráculo quiz dizer foi que todos o chamam de sudário e apenas vc o
chama de santo sudário.

[]s
TARCISIO BORGES
tbs97@fisica.ufpr.br

On Sat, 19 Mar 2005, pubmed2005 wrote:
> Não sei.
> Mas, nao sei o que isso tem haver com as pesquisas realizadas.

> --- Em ciencialist@yahoogrupos.com.br, "Oraculo" <oraculo@a...>
> > Bem, porque o sudário é "santo"?..:-)



SUBJECT: Fwd:Palestra - Estratégia no Mercado de Ações
FROM: Mario Goncalves <mario.jor@gmail.com>
TO:
DATE: 21/03/2005 09:22

From: Rmac3 - Mercado Financeiro <info_mf@rmac3.com.br>
Date: Fri, 18 Mar 2005 22:41:45 -0300
Subject: Palestra - Estratégia no Mercado de Ações

PALESTRA - ESTRATÉGIA NO MERCADO DE AÇÕES
Rodrigo Côrtes - Rmac3.com.br

ASSUNTOS: Como investir em ações (Home Broker e Corretor); A
importância do desenvolvimento de uma estratégia; Disciplina; Análise
Técnica (Alguns Conceitos e Exemplos).
DATA: 29/03/05
HORÁRIO: 18 às 19 horas
LOCAL: Faculdade São Camilo - Rua Dr. Satamini, 145 - Tijuca (perto da
Estação do Metrô Afonso Pena)
Rio de Janeiro
INSCRIÇÃO: 2568-9350 ou extensaoadm@saocamilo-rj.br


NÃO DEIXE DE LEVAR 1 Kg DE ALIMENTO!


[As partes desta mensagem que não continham texto foram removidas]



SUBJECT: no lugar do lactopurga...
FROM: "E m i l i a n o C h e m e l l o" <chemelloe@yahoo.com.br>
TO: <quimica-qaw@yahoogrupos.com.br>, <quimica@grupos.com.br>, <naeq-ucs@yahoogrupos.com.br>, <ciencialist@yahoogrupos.com.br>
CC: <tchequimica@yahoogrupos.com.br>
DATE: 21/03/2005 09:51

Alguma sugestão?

[ ] 's do Emiliano Chemello
emiliano@quimica.net
http://www.quimica.net/emiliano
http://www.ucs.br/ccet/defq/naeq
---
Contato Naeq:
Nome: Neide Domingues
Email: domingues@vialavras.com.br
Assunto: Química
Mensagem: Que substância pode ser usada no lugar da fenolftaleína (usada nas
experiências ácidos/bases)?
Eu sempre usei o comprimido de lactopurga. Depois que a fenolftaleína foi
retirada do mercado, não tem mais fenolftaleína neste comprimido e eu não
sei qual substância a substitui.
Poderiam me ajudar?
Obrigada
Neide
[ MSN ] chemelloe@hotmail.com
[ ICQ ] 145060604





SUBJECT: Re: [ciencialist] no lugar do lactopurga...
FROM: Aline Santos <haline_santos@yahoo.com.br>
TO: ciencialist@yahoogrupos.com.br
DATE: 21/03/2005 13:47


Pode ser usada uma solução de beterraba. (Corte uma beterraba em fatias e ferva em uma vazilha somente com água, a "água vermelha" será a sua nova solução, ela deve ser estocada na geladeira em um frasco fechado).

Como foi a muito tempo que eu fiz esta experiência, não me lembro qual é a cor de identificação de um ácido/base, pode ser feito um teste rápido, colocando algumas gotas da solução em um pouco de suco de limão (feito da fruta), certo!!!











E m i l i a n o C h e m e l l o <chemelloe@yahoo.com.br> wrote: Alguma sugestão?

[ ] 's do Emiliano Chemello
emiliano@quimica.net
http://www.quimica.net/emiliano
http://www.ucs.br/ccet/defq/naeq
---
Contato Naeq:
Nome: Neide Domingues
Email: domingues@vialavras.com.br
Assunto: Química
Mensagem: Que substância pode ser usada no lugar da fenolftaleína (usada nas
experiências ácidos/bases)?
Eu sempre usei o comprimido de lactopurga. Depois que a fenolftaleína foi
retirada do mercado, não tem mais fenolftaleína neste comprimido e eu não
sei qual substância a substitui.
Poderiam me ajudar?
Obrigada
Neide
[ MSN ] chemelloe@hotmail.com
[ ICQ ] 145060604





##### ##### #####

Para saber mais visite
http://www.ciencialist.hpg.ig.com.br


##### ##### ##### #####


Yahoo! Grupos, um serviço oferecido por: São Paulo Rio de Janeiro Curitiba Porto Alegre Belo Horizonte Brasília

---------------------------------
Links do Yahoo! Grupos

Para visitar o site do seu grupo na web, acesse:
http://br.groups.yahoo.com/group/ciencialist/

Para sair deste grupo, envie um e-mail para:
ciencialist-unsubscribe@yahoogrupos.com.br

O uso que você faz do Yahoo! Grupos está sujeito aos Termos do Serviço do Yahoo!.




---------------------------------
Yahoo! Mail - Com 250MB de espaço. Abra sua conta!

[As partes desta mensagem que não continham texto foram removidas]



SUBJECT: 13 things that do not make sense
FROM: "E m i l i a n o C h e m e l l o" <chemelloe@yahoo.com.br>
TO: <ciencialist@yahoogrupos.com.br>, <naeq-ucs@yahoogrupos.com.br>, <quimica-qaw@yahoogrupos.com.br>, <quimica@grupos.com.br>
DATE: 21/03/2005 13:57

Caros amigos,

Uma dica de leitura:

"13 things that do not make sense" é um artigo muito interessante publicado
na New Scientist sobre observações que não parecem fazer muito sentido em
face das teorias actuais

veja completo em:
http://www.newscientist.com/channel/space/mg18524911.600

[ ] 's do Emiliano Chemello
emiliano@quimica.net
http://www.quimica.net/emiliano
http://www.ucs.br/ccet/defq/naeq
[ MSN ] chemelloe@hotmail.com
[ ICQ ] 145060604

" Rien ne se perd, rien ne se crée,
tout se transforme."

Antoine Laurent de Lavoisier (químico francês, 1743 - 1794)




SUBJECT: Re: ninguem é inocente.
FROM: "junior_br2001" <junior_br2001@yahoo.com.br>
TO: ciencialist@yahoogrupos.com.br
DATE: 21/03/2005 14:05


Não a peça queimada, mas as fibras do remendo que foi testado pelo
carbono-14 possuia vanilina, e não vinila

JR

--- Em ciencialist@yahoogrupos.com.br, TARCISIO BORGES <tbs97@f...>
escreveu
> Mas para o teste de C14 a peça é queimada. De onde ele tirou a
idéia de
> que a peça queimada possuía vinila?
>
> Não importa quantos testes de façam, sempre vai existir alguém que
vai
> contestar, mesmo que se queime o sudário inteiro.
>






SUBJECT: Re: Carbono-14 (era: Sudario)
FROM: "rmtakata" <rmtakata@altavista.net>
TO: ciencialist@yahoogrupos.com.br
DATE: 21/03/2005 14:07


--- Em ciencialist@yahoogrupos.com.br, "junior_br2001"
> Portanto, o Takata pode questionar na condição de cético , mas não
> na de cientista físico-químico no caso.

Ou seja, nao devemos dar credito a vc tampouco...

[]s,

Roberto Takata





SUBJECT: Re: ninguem é inocente.
FROM: "junior_br2001" <junior_br2001@yahoo.com.br>
TO: ciencialist@yahoogrupos.com.br
DATE: 21/03/2005 14:17


Segue comentários

--- Em ciencialist@yahoogrupos.com.br, TARCISIO BORGES <tbs97@f...>
escreveu
> On Fri, 18 Mar 2005, pubmed2005 wrote:
> > O fato de o primeiro estudo ter sido bem sucedido nao invalida o
> > segundo estudo, que pode ser muito bem um complemento e até mesmo
um
> > chamado para falhas do primeiro estudo.
>
> Invalida sim. O primeiro estudo [C14] deu uma data de 700 anos +/-
60
> anos. O segundo estudo deu uma data entre 1300 e 3000 anos.
>
> Pelo que pude perceber as duas datas não possuem intercecção e
sendo ambos
> os métodos aceitos, um dos dois está errado.


JR: Voce tem certeza disso? Eu li 1340 anos, inclusive li isso no
site que voce me apontou, dos céticos( a nao ser que o valor de 600
seja o minimo do primeiro estudo), . Se o Rogers estipulou um
mínimo , ele coincide *mais ou menos* com esse valor de 1340. O que
pode invalidar é margem especulada em 3000 anos

> Na verdade os cientistas se apegam ao que funciona. Se o antigo
funciona,
> mas o novo funciona melhor, os cientistas vão ter uma certa
resistência em
> aceitar o novo, pois o antigo ainda vai muito bem.

JR: As técnicas do carbono-14 hoje em dia estão muito melhores que as
de 88. Além do mais voce nao prestou atenção ao que foi dito sobre
contaminação do tecido com carbono-14 recente, revestimento
bioplástico e teste relizado nos remendos medievais e não no tecido
propriamente dito. Se isso é o que voce diz que funciona, a ciencia
pode estar então bem equivocada. se fosse pelo menos um estudo bem
seguro sobre o manto poderíamos aplicar o que voce disse. Mas foi um
estudo insuficiente com falhas.

>
>
> > O segundo estudo foi muito pertinente,e merece atençao
científica. O
> > fato de a pesquisa ter sido feita por um químico aposentado
envolve
> > muitos preconceitos tb da comunidade científica, etc e tal...
>
> Não consigo enxergar estes preconceitos. O Stephen William Hawking
possui
> a idade do meu pai e nem por isso é desacreditado, mesmo admitindo
que
> errou em sua teoria.
>
> Como já disse em outro email, o preconceito está na metodologia
usada por
> Rogers.


JR: Foi uma metologia científica, e não um teste homeopático, ele
encontrou uma substancia química no remendo, e no resto do tecido a
vanilina já não estava presente. Isso é uma evidencia, não um estudo
mal-feito.
O Stephen William Hawking só admitiu que estava errado, quando sua
teoria já tinha sido refutada por outro físico...se ele não admitisse
pegaria mal. Muitos físicos não gostam dele, nao pela sua
deficiencia, mas por que ele é um especulador sonhador





SUBJECT: Re: Carbono-14 (era: Sudario)
FROM: "junior_br2001" <junior_br2001@yahoo.com.br>
TO: ciencialist@yahoogrupos.com.br
DATE: 21/03/2005 14:22


Hein, mas eu nao fiz nenhum estudo, nem pretendo apresentar qualquer
trabalho. Eu posso tecer comentários, refutar aqui na lista, mas
nunca numa comunidade científica. A não ser que eu descubra algo
muito importante, registre isso legalmente e depois apresente a
comunidade científica. Mas se claramente os créditos de físico eu nao
posso ter, posso pelo menos ser o autor de uma descoberta, assim como
voce pode tb.
Não sei como é isso, mas deve existir um meio legal de a descoberta
de uma pessoa ser aceita. Agora, se a comunidade científica vai
aceitar ou não é outros quinhentos. Bem se for retestada e houver
evidencias , eles vão ter que aceitar de qualquer maneira, apesar das
críticas.
Me parcve que astronomos amadores podem relatar suas descobertas, e
serem o autores do que descobrirem, mas quanto a outras áreas nao sei
como funciona.

JR

--- Em ciencialist@yahoogrupos.com.br, "rmtakata" <rmtakata@a...>
escreveu
>
> --- Em ciencialist@yahoogrupos.com.br, "junior_br2001"
> > Portanto, o Takata pode questionar na condição de cético , mas
não
> > na de cientista físico-químico no caso.
>
> Ou seja, nao devemos dar credito a vc tampouco...
>
> []s,
>
> Roberto Takata





SUBJECT: Re: ninguem é inocente.
FROM: "junior_br2001" <junior_br2001@yahoo.com.br>
TO: ciencialist@yahoogrupos.com.br
DATE: 21/03/2005 14:25


Buda disso algo diferente:

"Não acredite em algo simplesmente porque ouviu. Não acredite em algo
simplesmente porque todos falam a respeito. Não acredite em algo
simplesmente porque esta escrito em seus livros religiosos. Não
acredite em algo só porque seus professores e mestres dizem que é
verdade. Não acredite em tradições só porque foram passadas de
geração em geração. Mas depois de muita análise e observação, se você
vê que algo concorda com a razão, e que conduz ao bem e beneficio de
todos, aceite-o e viva-o."

JR





--- Em ciencialist@yahoogrupos.com.br, TARCISIO BORGES <tbs97@f...>
escreveu
> Talvez Buda o fosse, pois em determinado momento diz aos seus
discípulos:
>
> Não acredite em seus professores.
>
> :-)






SUBJECT: Re: ninguem é inocente.
FROM: "junior_br2001" <junior_br2001@yahoo.com.br>
TO: ciencialist@yahoogrupos.com.br
DATE: 21/03/2005 14:27


Para mim é irrelevante se o as pessoas dizem que o manto é religioso,
eu o chamo manto sudário, santo sudário, sudário . Pra mim tanto faz
qualquer uma dessas opções.

Não sei porque tanta tristeza... Bem pelo menos eu não estou tão
triste por isso

JR

--- Em ciencialist@yahoogrupos.com.br, TARCISIO BORGES <tbs97@f...>
escreveu
> É tão triste quando o questionado não entende a pergunta...
>
> O que o oráculo quiz dizer foi que todos o chamam de sudário e
apenas vc o
> chama de santo sudário.






SUBJECT: Re: [ciencialist] Ad Hominem e FAMOSOS..:-)
FROM: "Oraculo" <oraculo@atibaia.com.br>
TO: <ciencialist@yahoogrupos.com.br>
DATE: 21/03/2005 14:31

Olá LER

he he he..:-) Não se preocupe, eu nunca brigo com ninguém..:-) Mesmo eu sendo tolo e cético..:-)

Um abraço.

Homero

----- Original Message -----
From: L.E.R.de Carvalho
To: ciencialist@yahoogrupos.com.br
Sent: Monday, March 21, 2005 1:58 AM
Subject: [ciencialist] Ad Hominem e FAMOSOS..:-)


At 01:42 21/3/2005, you wrote:
>Olá Pubmed
>
>Pubmed: Quando falo céticos tolos e voce acha que foi com voce... fazer o que?



Eu tinha certeza que vocês iriam acabar brigando.

Isso é científico.

Dá até pra estabelecer uma Lei.

Enfim... causa e efeito nas ciencias sociais.

Voces ainda serão internacionalmente famosos.

L.E.

[As partes desta mensagem que não continham texto foram removidas]



##### ##### #####

Para saber mais visite
http://www.ciencialist.hpg.ig.com.br


##### ##### ##### #####


Yahoo! Grupos, um serviço oferecido por:







------------------------------------------------------------------------------
Links do Yahoo! Grupos

a.. Para visitar o site do seu grupo na web, acesse:
http://br.groups.yahoo.com/group/ciencialist/

b.. Para sair deste grupo, envie um e-mail para:
ciencialist-unsubscribe@yahoogrupos.com.br

c.. O uso que você faz do Yahoo! Grupos está sujeito aos Termos do Serviço do Yahoo!.



[As partes desta mensagem que não continham texto foram removidas]



SUBJECT: Re: ninguem é inocente.
FROM: "junior_br2001" <junior_br2001@yahoo.com.br>
TO: ciencialist@yahoogrupos.com.br
DATE: 21/03/2005 14:32


Bem, diante disso não se pode dizer que os gregos foram os detentores
do pensamento racional. Talvez no sentido de uma explicação física
para a Natureza sim, mas nao foram os primeiros a usar a razão, há
relatos no hinduísmo , nos Upanishads que os antigos hindus
questionaram os antigos deuses em prol de uma realidade única(não
confundir com monoteísmo)

JR

--- Em ciencialist@yahoogrupos.com.br, "junior_br2001"
<junior_br2001@y...> escreveu
>
> Buda disso algo diferente:
>
> "Não acredite em algo simplesmente porque ouviu. Não acredite em
algo
> simplesmente porque todos falam a respeito. Não acredite em algo
> simplesmente porque esta escrito em seus livros religiosos. Não
> acredite em algo só porque seus professores e mestres dizem que é
> verdade. Não acredite em tradições só porque foram passadas de
> geração em geração. Mas depois de muita análise e observação, se
você
> vê que algo concorda com a razão, e que conduz ao bem e beneficio
de
> todos, aceite-o e viva-o."
>
> JR
>
>
>
>
>
> --- Em ciencialist@yahoogrupos.com.br, TARCISIO BORGES <tbs97@f...>
> escreveu
> > Talvez Buda o fosse, pois em determinado momento diz aos seus
> discípulos:
> >
> > Não acredite em seus professores.
> >
> > :-)





SUBJECT: Re: [ciencialist] Conseqüências práticas do ceticismo filosófico
FROM: JVictor <jvoneto@uol.com.br>
TO: ciencialist@yahoogrupos.com.br
DATE: 21/03/2005 14:33

Manoel,

Excelente texto, nada a acrescentar ou tirar. Vai para minha pasta
especial, de artigos a serem usados como referência.
Ia até trecer algumas considerações sobre o assunto tratado, até que lí
seu trabalho. Então, dei ré...

Se me pedissem nota, numa escala de 0 a 10, daria 11!.

Parabéns.

Sds,

Victor.



Manuel Bulcão escreveu:

>
> CONSEQUÊNCAS PRÁTICAS DO CETICISMO FILOSÓFICO
>
>
> "não devemos temer a dúvida, mas antes devemos acolhê-la como a
> possibilidade de um novo potencial para os seres humanos."
> (Richard Feynman)
>
>
> Não é fácil conviver com a dúvida. Se tenho dúvidas, sou obrigado a
> escolher, a exercitar a minha liberdade, a me responsabilizar pela
> escolha feita e pelos meus atos, a admitir que "posso estar errado".
> A dúvida é a sombra da liberdade e tem por corolário a angústia e a
> responsabilidade moral.
>
> Penso que a certeza, "a crença na precisão absoluta da minha
> crença", é o mais velho antídoto contra a ansiedade que sempre
> acompanha o ato da escolha. Pois, se não tenho dúvidas acerca do
> caminho a trilhar, não estou escolhendo, não estou a agir
> livremente, mas tão-somente obedecendo a um imperativo, que pode ser
> a palavra de Deus, o Destino ou mesmo a Razão. A certeza absoluta
> tranqüiliza a consciência, nos faz agir sem remorsos e sem ela os
> genocidas em nome de Deus ou da Razão não dormiriam em paz. É porque
> temem a liberdade, é por não quererem assumir responsabilidade moral
> que os homens costumam se deixar levar por certezas. Se um dia
> descobrem que o que motivou a sua ação era um erro, amaldiçoam Deus,
> o Destino ou a Razão, lamentam o fato de terem sido "enganados"
> (colocam-se sempre numa situação passiva) e tranqüilos permanecem
> com suas outras certezas.
>
> Ao contrário do que muitos pensam, o cético não é um frívolo
> diletante que tem por hobby questionar tudo e que faz da filosofia
> uma disputa entre egos, um joguinho de salão. Ao contrário, há mesmo
> um propósito moral no ceticismo filosófico. Ao fazer da incerteza um
> dos princípios do pensamento humano, o que o filósofo cético também
> pretende é destruir esse escudo da covardia, essa negação da
> liberdade, essa arma da intolerância, esse fundamento de todas as
> tiranias, isso que é a categoria central de todos os sistemas
> filosóficos fechados imunes à discussão e ao questionamento e que
> apenas servem de ideologia para os inimigos da sociedade aberta: o
> mito da certeza absoluta.
>
> Penso que nós, céticos, tivemos sucesso em nossa empreitada. Sim, ao
> menos no front filosófico, destruímos o encouraçado do dogmatismo.
> Deste só resta destroços e alguns náufragos que, para não se
> afogarem, fazem das tautologias — como, por exemplo, "o existente
> existe" — e do cogito cartesiano sua tábua de salvação.
>
> Claro, existem muitos não-céticos que estão absolutamente certos
> das "virtudes" da sociedade aberta. Estes costumam criticar o cético
> pelo fato de ele não saber com certeza se assassinar uma criança
> indefesa é moralmente justificável ou não, ou porque "diz não saber
> com certeza que uma ditadura sanguinolenta é defensável ou não. Não
> percebem eles que o que ameaça a sociedade aberta não é a incerteza
> quanto a verdade dos seus princípios, mas a certeza compartilhada
> por muitos, inclusive por supostos paladinos da democracia (Nixon,
> Kissinger, Sharon, Bush, etc.) de que assassinar crianças é muitas
> vezes moralmente justificável; essa mesma certeza ou convicção
> inabalável tão característica dos que se arvoram os guardiões da
> Razão, do Pensamento Correto ou da Verdade e que, por isso, não
> estão muito dispostos a perder tempo com discussão, com a busca do
> consenso mediante o diálogo e outras "masturbações" democráticas,
> motivo pelo qual chegam mesmo a defender ditaduras sanguinolentas
> sempre que lhes convém. Como, por exemplo, o "racionalista"
> e "libertário" de direita Milton Friedman, que certa vez teceu
> elogios rasgados a Pinochet e justificou sua tirania.
>
> É investindo contra o mito da certeza absoluta ou reduzindo-a ao
> ponto de jamais se transformar em instrumento de dominação política
> que se garante "quase certamente" a sociedade aberta.
>
> Ao admitir que "posso estar errado e você certo", não estou senão
> aceitando que: a) é razoável — ou seja, é uma atitude racional — que
> eu considere os seus argumentos; por conseguinte, b) não é razoável —
> isto é, não é uma atitude racionalmente justificável — que eu venha
> a eliminá-lo, a não ser, obviamente, que você queira e tente me
> eliminar antes. O caráter duvidoso tanto da minha quanto da sua
> crença é o que nos torna iguais apesar de sermos diferentes. A
> incerteza é, também, a justificação racional da minha e da sua
> liberdade de questionar. Ora, o reconhecimento do outro (do
> diferente), a igualdade entre os interlocutores e a liberdade de
> questionar são tanto os princípios da razão quanto da democracia.
>
> E nem é preciso ter certeza absoluta da verdade desses princípios
> para defendê-los, pois todos eles têm por pressuposto o princípio
> mais fundamental de que "tudo é incerto, inclusive esta afirmação".
>
>
> Manuel Bulcão
> 17/02/2002
>
>
>
>
>
> ##### ##### #####
>
> Para saber mais visite
> http://www.ciencialist.hpg.ig.com.br
>
>
> ##### ##### ##### #####
>
>
> *Yahoo! Grupos, um serviço oferecido por:*
> PUBLICIDADE
> <http://br.rd.yahoo.com/SIG=12aj62lk6/M=264379.5078783.6203979.1588051/D=brclubs/S=2137111528:HM/EXP=1111401869/A=2332652/R=0/id=noscript/SIG=119058f8i/*http://br.download.yahoo.com/messenger/>
>
>
>
> ------------------------------------------------------------------------
> *Links do Yahoo! Grupos*
>
> * Para visitar o site do seu grupo na web, acesse:
> http://br.groups.yahoo.com/group/ciencialist/
> * Para sair deste grupo, envie um e-mail para:
> ciencialist-unsubscribe@yahoogrupos.com.br
> <mailto:ciencialist-unsubscribe@yahoogrupos.com.br?subject=Unsubscribe>
> * O uso que você faz do Yahoo! Grupos está sujeito aos Termos do
> Serviço do Yahoo! <http://br.yahoo.com/info/utos.html>.
>
>
>
>
> __________ Informação do NOD32 1.1030 (20050319) __________
>
> Esta mensagem foi verificada pelo NOD32 Sistema Antivírus
> http://www.nod32.com.br




SUBJECT: Re: [ciencialist] Re: ninguem é inocente.
FROM: "Oraculo" <oraculo@atibaia.com.br>
TO: <ciencialist@yahoogrupos.com.br>
DATE: 21/03/2005 14:35

Olá Tarciso

Sim, hoje em dia. Mas historicamente, o sudario já foi reliquia real e inconteste. Se hoje apenas alguns bispos e padres (e alguns dos mais influentes e poderosos..:-) é porque a igreja sempre soube se manter com um dos pés em cada lado de uma questão, principalmente depois do fiasco de Galileu..:-)

Ela hoje toma bem mais cuidado com suas afirmações (veja o trabalho que dá hoje reconhecer um milagre, e como era bem mais rápido e comum no passado), para não ficar (muito) desacreditada.

De qualquer forma, não é dificil perceber que a igreja gostaria demais que o sudario tivesse a data correta, para ser real (no sentido de ter coberto jesus)..:-)

Um abraço.

Homero

----- Original Message -----
From: TARCISIO BORGES
To: ciencialist@yahoogrupos.com.br
Sent: Monday, March 21, 2005 8:32 AM
Subject: Re: [ciencialist] Re: ninguem é inocente.


On Fri, 18 Mar 2005, Oraculo wrote:
> A Igreja é que inicialmente apresentou o caso, o sudário, como
> reliquia autentica e com as afirmações sobre sua origem e utilização.
> A mesma igreja que, brincando de esconde-esconde, as vezes permite, as
> vezes nega, o estudo confiável ao artefato.

O Vaticano não assume a autenticidade do artefato. Quem defende sua
autenticidade são alguns bispos e padres.


[]s
TARCISIO BORGES
tbs97@fisica.ufpr.br



##### ##### #####

Para saber mais visite
http://www.ciencialist.hpg.ig.com.br


##### ##### ##### #####


Yahoo! Grupos, um serviço oferecido por:
PUBLICIDADE




------------------------------------------------------------------------------
Links do Yahoo! Grupos

a.. Para visitar o site do seu grupo na web, acesse:
http://br.groups.yahoo.com/group/ciencialist/

b.. Para sair deste grupo, envie um e-mail para:
ciencialist-unsubscribe@yahoogrupos.com.br

c.. O uso que você faz do Yahoo! Grupos está sujeito aos Termos do Serviço do Yahoo!.



[As partes desta mensagem que não continham texto foram removidas]



SUBJECT: Re: ninguem é inocente.
FROM: "junior_br2001" <junior_br2001@yahoo.com.br>
TO: ciencialist@yahoogrupos.com.br
DATE: 21/03/2005 14:42


Mas, o Vaticano é um Estado(ou cidade sei lá). Os padres e bispos que
pertence a Igreja Catolica Romana da Cidade do Vaticano é que podem
defender a autenticidade. Se bem que no sei, ainda nao procurei saber
qual é a opinião geral dos católicos praticantes a respeito disso, e
depois dos testes.

JR

--- Em ciencialist@yahoogrupos.com.br, "Oraculo" <oraculo@a...>
escreveu
>
> O Vaticano não assume a autenticidade do artefato. Quem defende
sua
> autenticidade são alguns bispos e padres.
]





SUBJECT: Re: [ciencialist] Re: ninguem é inocente.
FROM: "Oraculo" <oraculo@atibaia.com.br>
TO: <ciencialist@yahoogrupos.com.br>
DATE: 21/03/2005 14:51

Olá Pubmed

"Pubmed: Se bem que no sei, ainda nao procurei saber qual é a opinião geral dos católicos praticantes a respeito disso, e depois dos testes."

Católicos praticantes, e cristãos de modo geral (basta digitar no Google para descobrir..:-) acreditam que o sudário cobriu o corpo de jesus, antes dos testes e depois dos testes, qualquer teste, carbono-14 ou outro. Sua crença se baseia na fé, e esta não precisa de evidências e não se abala ou modifica a partir delas.

Para quem crê em milagres, nada impede que o mesmo milagre que imprimiu a imagem do messias no pano tenha modificado suas propriedades quimicas e fisicas, de forma a não ser possível a ciência saber sua data e origem. É um milagre, e milagres aceitam qualquer "ad hoc" para se manter, já que começam com a suspensão das leis fisicas deste universo. Uma vez suspensas, por força do poder absoluto da divindade e do sobrenatural, nada mais é impedimento ou capaz de dificultar a ocorrencia de qualquer fenômeno.

Claro que não serão coisas mundanas como evidências e conhecimento científico que vão atrapalhar a capacidade do sobrenatural de agir como lhe bem aprouver..:-)

Homero


----- Original Message -----
From: junior_br2001
To: ciencialist@yahoogrupos.com.br
Sent: Monday, March 21, 2005 2:42 PM
Subject: [ciencialist] Re: ninguem é inocente.



Mas, o Vaticano é um Estado(ou cidade sei lá). Os padres e bispos que
pertence a Igreja Catolica Romana da Cidade do Vaticano é que podem
defender a autenticidade. Se bem que no sei, ainda nao procurei saber
qual é a opinião geral dos católicos praticantes a respeito disso, e
depois dos testes.

JR

--- Em ciencialist@yahoogrupos.com.br, "Oraculo" <oraculo@a...>
escreveu
>
> O Vaticano não assume a autenticidade do artefato. Quem defende
sua
> autenticidade são alguns bispos e padres.
]





##### ##### #####

Para saber mais visite
http://www.ciencialist.hpg.ig.com.br


##### ##### ##### #####


Yahoo! Grupos, um serviço oferecido por:







------------------------------------------------------------------------------
Links do Yahoo! Grupos

a.. Para visitar o site do seu grupo na web, acesse:
http://br.groups.yahoo.com/group/ciencialist/

b.. Para sair deste grupo, envie um e-mail para:
ciencialist-unsubscribe@yahoogrupos.com.br

c.. O uso que você faz do Yahoo! Grupos está sujeito aos Termos do Serviço do Yahoo!.



[As partes desta mensagem que não continham texto foram removidas]



SUBJECT: Re: Confiabilidade (era Santo sudário e a vanilina(correção)
FROM: "junior_br2001" <junior_br2001@yahoo.com.br>
TO: ciencialist@yahoogrupos.com.br
DATE: 21/03/2005 14:59


Alberto
Vale comentários:

--- Em ciencialist@yahoogrupos.com.br, "Alberto Mesquita Filho"
<albmesq@u...> escreveu

>> Seria uma honra ser professor do Júnior assim como, e a depender
do assunto,
> seria também uma honra ser aluno do Júnior. Mas, como não estamos
numa
> escola tradicional onde uns sempre ensinam e outros sempre
aprendem, mas na
> escola da vida, onde até mesmo o relacionamento professor/aluno é
relativo,
> o máximo que posso dizer é que sinto-me honrado em poder contar com
> debatedores de elevado nível como você, o Júnior, o Takata, o
Taborda, o
> Eduardo e tantos outros. Há sempre o que aprender até mesmo num
diálogo
> insosso como esse e com grande probabilidade de não nos levar a
mais nada
> além desse aprendizado, o que já é muito.

JR: Realmente o assunto no é dos melhores, mas realmente o debate não
desancou para futilidades.
Quanto a eu ser seu mestre, tenho muito a ensinar da vida, sou
volúvel pacas...hehehehe. Mas devo ser uma cara legal.

>
> > E a confiabilidade neste caso é simplesmente uma questão de
possibilidade
> > de ser repetido e encontrar os mesmos resultados.

JR: perfeito, em ciencia nao teste que um teste é mais confiável que
outro, apareceu evidencias, a missão é investigar.

>>
> > Os testes de carbono-14,
> > mais de um, passaram nesta exigencia e, por enquanto, são
confiáveis, até
> > onde um conhecimento cientifico pode ser confiável.

JR: De fato, apesar de que, confiabilidade nao é exatidão. As
técnicas evoluiram muito desde 88. Portanto aquele resultado merece
um outr verificação com as técnicas atuais que estão mais precisas
que algumas décadas

>
> Pois é, então creio que você deve ter entendido o que procurei
dizer no
> parágrafo anterior, se bem que sem utilizar nenhum teste
específico, mesmo
> porque não entendo nada do teste referido (carbono-14) e também não
sei
> quase nada sobre o objeto de estudo que vocês tanto falam (sudário).

JR: Pois é, Creio tb que nesse assunto há uma ignorancia geral,
inclusive a minha.

>
> > Isto é, não uma
> > certeza de 100%, mas uma probabilidade forte, e a espera por
refutações
> > pelo menos tão solidas quanto o estudo em questão.

JR: Isso que estou defendendo tb.

>
> Mas se o teste já passou pelo critério da repetibilidade, não há
mais o que
> ser dito a respeito. O teste será válido sim, e não há porque
objetivá-lo
> com um número. Ou trata-se de um teste científico (neste caso o
índice
> poderia ser 1 ou 100) ou não se trata de um teste científico (e
neste caso o
> índice seria zero). Isso a meu ver não tem nada a ver com o que
chamei
> confiabilidade. O termo confiabilidade parece-me ter sido
primeiramente
> expresso aqui pelo Takata, e eu entrei na discussão justamente
procurando
> saber o que ele queria dizer com isso.


JR: Pois é, o teste deveria ser repetido novamente com os novos
conhecimento e as novas técnicas

>
> Quero crer, mas pode ser que esteja enganado, que a confiabilidade
no caso
> está a se referir a uma comparação entre dois métodos diversos, e
nesse caso
> a dúvida seria saber se um dos dois é mais ou menos confiável do
que o
> outro, obviamente aceitando-se a premissa de que os dois sejam
científicos.
> Perceba que não estou dizendo que precisariam ser 100% científicos,
pois
> isso soaria como uma redundância. Por outro lado, ser científico não
> significa exprimir uma verdade absoluta, e quero crer que todos
aqui da
> Ciencialist, ou pelo menos os mais veteranos, concordam com isso.

JR: POis é. Acho que cada método tem suas vantagens e suas
desvantagens, mas nao se pode dizer que uym é mais confiável que
outro, nao ser naquilo em que o teste é específico.

>
> Seria possível objetivar essa maior confiabilidade em um dos
métodos do que
> em relação ao outro? Seria possível atribuir um peso a essa
confiabilidade?
> Em outras palavras: Poderiamos estabelecer uma escala de valores,
por
> exemplo, de 0 a 100? Ora, como diria Popper, "Não existem
observações puras:
> elas estão impregnadas pelas teorias e são orientadas pelos
problemas e
> acompanhadas pelas teorias." Essa idéia não é de Popper, e acredito
que os
> filósofos seiscentistas já se depararam com esse problema. Esse
problema
> surge sempre que tentamos analisar em maior profundidade o
chamado "problema
> da indução" e/ou o "problema da corroboração" (que é o nosso caso
atual). Se
> um método A reproduz um resultado em 90% das vezes em que me
utilizo dele,
> enquanto outro B o faz em apenas 60% dos casos, isso pode me
convencer (a
> despeito de poder estar errado) de que A é melhor do que B, mas
isso não me
> autoriza dar a nota 90 (ou 9) a A e 60 (ou 6) a B. Isso porque as
teorias
> que impregnam tais metodologias via de regra (quase com certeza)
não são as
> mesmas e até mesmo os critérios que utilizamos para chegar nesses
valores,
> seja o 90%, seja o 60%, estão contaminados pelas teorias utilizadas.

JR: Perfeito


>
> Espero com isso ter mostrado *uma das razões* porque acho que não é
fácil
> arcar com o ônus do ceticismo, e é por isso que não me aventuro a
dizer-me
> cético com respeito àquilo que não entendo, pois isso não é postura
digna de
> um cético, mas sim de um ignorante. A medida em que essa
ignorância, seja
> ela por ingenuidade (ignorância sócio-disfuncional, dentre outras),
seja ela
> do tipo reflexivo (a ignorância dos arrogantes, dentre outras)
adquire o
> status de sabedoria (no extremo o yin se transforma no yang, para
usar um
> ditado oriental), acaba por nos levar a um consensualismo via de
regra
> irracional, posto que movido mais pela apatia, pelo conformismo,
pelo
> modismo ou pelo comodismo (é mais fácil concordar do que
subverter). É nesse
> sentido que eu digo que ou se estuda, e muito, o ceticismo, ou é
melhor não
> dizer nada, sob pena de sermos confundidos com os aproveitadores da
> ingenuidade popular. Sinceramente, é assim que vejo a grande
maioria desses
> falsos movimentos céticos que pululam pela Internet. Não me parece
que nem
> você nem o Takata, nem muitos outros com quem tenho debatido esse
assunto
> aqui na Ciencialist, estejam enquadrados na categoria dessa corja a
que me
> referi acima (creio já ter cruzado com alguns) mas não acho
impossível que
> vocês estejam alimentando um monstro.

JR: Se ceticismo organizado fosse bom mesmo, o Drauzio Varella, o
Marcelo Gleiser, Attico Chassot, O falecido César Lattes e outros
renomados, estava tudo lá na STR ou no SBCR








SUBJECT: Re: [ciencialist] Confiabilidade (era Santo sudário e a vanilina(correção)
FROM: "Alberto Mesquita Filho" <albmesq@uol.com.br>
TO: <ciencialist@yahoogrupos.com.br>
DATE: 21/03/2005 15:01

----- Original Message -----
From: "Oraculo"
Sent: Monday, March 21, 2005 3:08 AM
Subject: [ciencialist] Confiabilidade (era Santo sudário e a
vanilina(correção)

> E acho impressionante como é possível termos tantas discordancias e ao
> mesmo tempo, eu concordar com suas colocações em quase tudo..:-)

Eu diria que essa é uma das *magias* associadas à dialética. Não havendo
contradições não há dialética. Em as havendo, não é impossível que evoluam
para a síntese, quando os contendores entram naquela fase de "discordâncias
concordantes". ;-) Creio que tanto Popper quanto David Bohm chegam a
comentar alguma coisa sobre essa forma de aprendizado, mas são poucos
aqueles que conseguem assimilar essa magia. Você há de se lembrar que não é
a primeira vez que "quebramos o pau" e evoluímos para essa síntese em que
nenhum dos dois se despersonaliza, mas acabamos assimilando alguma coisa do
pensamento do outro.

O dicionário Houaiss diz o seguinte sobre a dialética de Platão: "No
platonismo, processo de diálogo, debate entre interlocutores comprometidos
profundamente com a busca da verdade, através do qual a alma se eleva,
gradativamente, das aparências sensíveis às realidades inteligíveis ou
idéias". Popper, por outro lado, faz o seguinte comentário sobre a arte da
argumentação: "A arte da argumentação é uma forma peculiar da arte de
combater - com palavras em vez de espadas -, uma arte inspirada pelo
interesse em nos aproximarmos da verdade acerca do Mundo." Entre um
pensamento e outro a Terra rodou ao redor do Sol mais de duas mil vezes e,
não obstante, a "busca da verdade" permaneceu como o objetivo principal do
"verdadeiro" diálogo.

Seria bom que os demais percebessem esse jogo dialético, em especial aqueles
que se desesperam antes de que se possa chegar numa síntese e estes, sem
dúvida, constituem a maioria (pelo menos no relacionamento comigo). Lembro
também que o processo não pode se manter indefinidamente, pois a cada vez
fica mais difícil nós conseguirmos "quebrar o pau" :-)). Sob esse aspecto
acho muito mais fácil "quebrar o pau" com o Takata, se bem que tudo o que é
fácil acaba se transformando em algo improdutivo. Ou seja, não consigo
estabelecer um diálogo dialético com ele. Nada contra o Takata, digo apenas
que essa é uma questão de empatia e não há como dizer que ele ou eu
estamos errados, mas a verdade é que nenhum dos dois entra no jogo do outro
e o diálogo acaba se esvaziando antes de chegar ao término. Felizmente o
"quebra-pau" acaba por se desfazer, mas de uma forma artificial (cansaço de
uma das partes) e raramente se traduzindo em aprendizado para qualquer um
dos dois.

[ ]´s
Alberto
http://ecientificocultural.com/indice.htm
Mas indiferentemente a tudo isso, o neutrino tem massa, o elétron não é
uma carga elétrica coulombiana e a Terra se move. E a história se repetirá.



SUBJECT: Re: Carbono-14 (era: Sudario)
FROM: "rmtakata" <rmtakata@altavista.net>
TO: ciencialist@yahoogrupos.com.br
DATE: 21/03/2005 15:06


--- Em ciencialist@yahoogrupos.com.br, "junior_br2001"
> Hein, mas eu nao fiz nenhum estudo, nem pretendo apresentar
> qualquer trabalho. Eu posso tecer comentários, refutar aqui na
> lista, mas nunca numa comunidade científica.

Se isso for verdade, entao o seu comentario original nao foi de muita
utilidade, afinal nao tenho pretensao alguma de publicar as minhas
consideracoes sobre o tema em revistas especializadas - estou tao
somente rebatendo alguns argumentos q. vc tem postado nesta lista.

O q. tb torna inutil o seu comentario de q. eu deveria publicar as
minhas contas.

> Não sei como é isso, mas deve existir um meio legal de a descoberta
> de uma pessoa ser aceita.

Qq pessoa pode submeter seus estudos a uma revista cientifica. Em
geral seu trabalho passa por uma serie de triagens e eh enviado para
a analise de especialistas. Se seu trabalho nao tiver nenhum
problema, ele poderah ser publicado.

[]s,

Roberto Takata





SUBJECT: Tempo de existencia e data de criação, um engano (era ninguem é inocente.)
FROM: "Oraculo" <oraculo@atibaia.com.br>
TO: <ciencialist@yahoogrupos.com.br>
DATE: 21/03/2005 15:14

Olá Pubmed

Pubmed: Voce tem certeza disso? Eu li 1340 anos, "

Eu já havia percebido isso, você confundiu tempo de existencia com data do calendário. O tempo de existencia é de 660 anos, o que significa que foi criado por volta do ANO de 1340 DC. Não significa que ele tem 1340 anos de existencia, mas que foi criado no ano domini de 1340. Se o segundo estudo diz que ele foi criado até no máximo o ano domini de 700, evidentemente não podem ser complementares, não importa a data inicial (2000 antes de cristo).

Eu expliquei diversas vezes essa confusão, em diversas mensagens anteriores, de uma relida nelas..:-)

Homero


----- Original Message -----
From: junior_br2001
To: ciencialist@yahoogrupos.com.br
Sent: Monday, March 21, 2005 2:17 PM
Subject: [ciencialist] Re: ninguem é inocente.



Segue comentários

--- Em ciencialist@yahoogrupos.com.br, TARCISIO BORGES <tbs97@f...>
escreveu
> On Fri, 18 Mar 2005, pubmed2005 wrote:
> > O fato de o primeiro estudo ter sido bem sucedido nao invalida o
> > segundo estudo, que pode ser muito bem um complemento e até mesmo
um
> > chamado para falhas do primeiro estudo.
>
> Invalida sim. O primeiro estudo [C14] deu uma data de 700 anos +/-
60
> anos. O segundo estudo deu uma data entre 1300 e 3000 anos.
>
> Pelo que pude perceber as duas datas não possuem intercecção e
sendo ambos
> os métodos aceitos, um dos dois está errado.


JR: Voce tem certeza disso? Eu li 1340 anos, inclusive li isso no
site que voce me apontou, dos céticos( a nao ser que o valor de 600
seja o minimo do primeiro estudo), . Se o Rogers estipulou um
mínimo , ele coincide *mais ou menos* com esse valor de 1340. O que
pode invalidar é margem especulada em 3000 anos

> Na verdade os cientistas se apegam ao que funciona. Se o antigo
funciona,
> mas o novo funciona melhor, os cientistas vão ter uma certa
resistência em
> aceitar o novo, pois o antigo ainda vai muito bem.

JR: As técnicas do carbono-14 hoje em dia estão muito melhores que as
de 88. Além do mais voce nao prestou atenção ao que foi dito sobre
contaminação do tecido com carbono-14 recente, revestimento
bioplástico e teste relizado nos remendos medievais e não no tecido
propriamente dito. Se isso é o que voce diz que funciona, a ciencia
pode estar então bem equivocada. se fosse pelo menos um estudo bem
seguro sobre o manto poderíamos aplicar o que voce disse. Mas foi um
estudo insuficiente com falhas.

>
>
> > O segundo estudo foi muito pertinente,e merece atençao
científica. O
> > fato de a pesquisa ter sido feita por um químico aposentado
envolve
> > muitos preconceitos tb da comunidade científica, etc e tal...
>
> Não consigo enxergar estes preconceitos. O Stephen William Hawking
possui
> a idade do meu pai e nem por isso é desacreditado, mesmo admitindo
que
> errou em sua teoria.
>
> Como já disse em outro email, o preconceito está na metodologia
usada por
> Rogers.


JR: Foi uma metologia científica, e não um teste homeopático, ele
encontrou uma substancia química no remendo, e no resto do tecido a
vanilina já não estava presente. Isso é uma evidencia, não um estudo
mal-feito.
O Stephen William Hawking só admitiu que estava errado, quando sua
teoria já tinha sido refutada por outro físico...se ele não admitisse
pegaria mal. Muitos físicos não gostam dele, nao pela sua
deficiencia, mas por que ele é um especulador sonhador





##### ##### #####

Para saber mais visite
http://www.ciencialist.hpg.ig.com.br


##### ##### ##### #####


Yahoo! Grupos, um serviço oferecido por:







------------------------------------------------------------------------------
Links do Yahoo! Grupos

a.. Para visitar o site do seu grupo na web, acesse:
http://br.groups.yahoo.com/group/ciencialist/

b.. Para sair deste grupo, envie um e-mail para:
ciencialist-unsubscribe@yahoogrupos.com.br

c.. O uso que você faz do Yahoo! Grupos está sujeito aos Termos do Serviço do Yahoo!.



[As partes desta mensagem que não continham texto foram removidas]



SUBJECT: Re: Carbono-14 (era: Sudario)
FROM: "junior_br2001" <junior_br2001@yahoo.com.br>
TO: ciencialist@yahoogrupos.com.br
DATE: 21/03/2005 15:15


Oi Takata,

Nao se sinta ressentido, nao foi por mal que falei aquilo. É que o
Oráculo estava usando seus estudos como uma prova cabal. Aí entrei na
nesta questão. Mas a intenção nao foi menosprezar sua pessoa, OK?
Sendo que no fundo, no fundo por mais títulos que tenhamos somos
todos ignorantes em busca do saber

JR

--- Em ciencialist@yahoogrupos.com.br, "rmtakata" <rmtakata@a...>
escreveu
>
> --- Em ciencialist@yahoogrupos.com.br, "junior_br2001"
> > Hein, mas eu nao fiz nenhum estudo, nem pretendo apresentar
> > qualquer trabalho. Eu posso tecer comentários, refutar aqui na
> > lista, mas nunca numa comunidade científica.
>
> Se isso for verdade, entao o seu comentario original nao foi de
muita
> utilidade, afinal nao tenho pretensao alguma de publicar as minhas
> consideracoes sobre o tema em revistas especializadas - estou tao
> somente rebatendo alguns argumentos q. vc tem postado nesta lista.
>
> O q. tb torna inutil o seu comentario de q. eu deveria publicar as
> minhas contas.
>
> > Não sei como é isso, mas deve existir um meio legal de a
descoberta
> > de uma pessoa ser aceita.
>
> Qq pessoa pode submeter seus estudos a uma revista cientifica. Em
> geral seu trabalho passa por uma serie de triagens e eh enviado
para
> a analise de especialistas. Se seu trabalho nao tiver nenhum
> problema, ele poderah ser publicado.
>
> []s,
>
> Roberto Takata





SUBJECT: Re: Confiabilidade (era Santo sudário e a vanilina(correção)
FROM: "rmtakata" <rmtakata@altavista.net>
TO: ciencialist@yahoogrupos.com.br
DATE: 21/03/2005 15:19


--- Em ciencialist@yahoogrupos.com.br, "junior_br2001"
> JR: perfeito, em ciencia nao teste que um teste é mais confiável
> que outro, apareceu evidencias, a missão é investigar.

Claro q. tem. Por exemplo, o teste de amplificacao por PCR para
detetar a presenca de HIV eh muito mais confiavel do q. o ELISA.

[]s,

Roberto Takata





SUBJECT: Re: Carbono-14 (era: Sudario)
FROM: "rmtakata" <rmtakata@altavista.net>
TO: ciencialist@yahoogrupos.com.br
DATE: 21/03/2005 15:23


--- Em ciencialist@yahoogrupos.com.br, "junior_br2001"
> Nao se sinta ressentido, nao foi por mal que falei aquilo.

Nao sei por q. eu me sentiria ressentido.

[]s,

Roberto Takata





SUBJECT: Mecânico de Venâncio Aires tem um Corcel movido a ar
FROM: César A. K. Grossmann <cesarakg@bol.com.br>
TO: ciencialist@yahoogrupos.com.br
DATE: 21/03/2005 15:23


http://gazeta.viavale.com.br/default.php?arquivo=_noticia.php&intIdConteudo=29268&intIdEdicao=490

Mecânico de Venâncio Aires tem um Corcel movido a ar

Mecânico de 73 anos desenvolveu sistema de bombas injetoras adaptado a
motor convencional, que não precisa de gasolina e se auto-alimenta
quando não é acelerado

Você já imaginou sair de casa, passar em um posto, colocar algumas
libras de ar no tanque do carro e sair rodando sem se preocupar com o
marcador de combustível e muito menos com o seu bolso? Em Venâncio
Aires, um Ford Corcel ano 77, inteiraço, com 97 mil quilômetros e todo
original – menos o motor, é claro –, dispensa gasolina ou álcool para
andar: é movido a ar comprimido, se auto-alimenta enquanto não está
sendo acelerado, dispensa água para refrigerar o motor e não
precisaria nem de bateria não fossem as luzes.

A invenção é o xodó do mecânico Percival de Freitas, 73 anos. Há quase
20 anos ele trabalha no projeto, que só não saiu de sua oficina
desativada, no centro da cidade, porque precisa de avaliação e
ajustes, os quais só podem ser feitos por especialistas, de
preferência um engenheiro. "Fui pensando e fazendo, mas como não sou
engenheiro, acho que o sistema precisa de ajustes. Gostaria que isso
fosse feito por alguém da Ford", salientou. Freitas conta que em uma
das poucas saídas que deu com o carro movido a ar, o motor parou de
funcionar a poucos metros da garagem devido ao rompimento de uma peça
malplanejada.

Embora estranho, o motor desenvolvido pelo aposentado é bem simples e
funciona mesmo. Como o carro dispensa o uso da gasolina ou qualquer
outro derivado de petróleo, o primeiro passo foi tirar o tanque
debaixo do porta-malas. No compartimento foi instalado um cilindro de
ar com capacidade para 200 litros, que é abastecido de duas formas:
por um ventil de câmara de pneu de caminhão, soldado na ponta do
tanque, e por um cano de ferro que vem do motor. Quando a máquina está
funcionando, mas não é acelerada, como em uma descida, por exemplo,
serve também de compressor, mandando ar de volta para o cilindro.

Para ligar o motor de dois tempos, basta abrir a válvula de ar,
instalada dentro do cilindro. O ar chega a quatro bombas injetoras –
parecidas com as utilizadas em motores a diesel –, que o empurra com
força para os pistões, acionando o motor. Quando o carro está sendo
acelerado, o ar é liberado e, quando está parado ou andando "na
banguela", o combustível volta para o cilindro. "É a coisa mais
ecológica que já vi na vida. Até mesmo o ar é reutilizado e não
precisa nem de água para refrigerar o sistema", defendeu o aposentado.

Afora a mão-de-obra de quase duas décadas, Percival de Freitas calcula
já ter investido R$ 50 mil na invenção. "Sou o segundo dono do Corcel,
que foi comprado especialmente para isso", conta, apontando para o
motor a gasolina do veículo, guardado em um canto da oficina. Na lista
de custos estão ainda a compra de um torno, no qual foram fabricados
quase todos os componentes do motor ecológico e a aquisição de peças.
O mecânico gasta também com luz, que movimenta o compressor usado para
abastecer o Corcel.

Cansado de mexer na engenhoca e de andar com o carro pra frente e pra
trás dentro de sua oficina, Freitas está disposto a vender o motor
adaptado. Ele ainda não conseguiu descobrir a potência nem a autonomia
do dois tempos ecológico, mas está aberto a propostas. "Posso vender
só o motor ou então o Corcel junto. Se o motor for primeiro, fico com
o carro para aproveitá-lo um pouco", planeja o aposentado, que se diz
um apaixonado pela marca Ford. Caso Freitas não consiga negociar o
invento, vai desmontar todo o motor para que ninguém se aproveite dele
no futuro. "Acho que até o governo federal deveria se interessar pelo
projeto", completou.

[]s
--
.O. Cesar A. K. Grossmann ICQ UIN: 35659423
..O http://www.LinuxByGrossmann.cjb.net/
OOO Timeo Danaos, et dona ferentes. (Virgilio)





SUBJECT: O manto de Turin
FROM: "Sergio M. M. Taborda" <sergiotaborda@terra.com.br>
TO: ciencialist@yahoogrupos.com.br
DATE: 21/03/2005 15:29

O estudo do manto, é antes de mais nada um estudo arqueologico.
A fisica , a quimica e a biologia, são para a arquologia meras
ferramentas. E o resultado final é a conjunção de todos os factores.
Ou seja, para quem pensa que testes quimicos ou fisicos tem a ultima
palavra desengane-se.Serão os arquologos que terão a ultima palavra.

O teste do caborno 14 não é confiável:
http://www.shroud.com/meacham.htm
http://www.historicaljesusquest.com/
*"* the carbon 14 testing has been so challenged by modern forensic
science (as recently reported by /National Geographic News/ and /PBS)
/that is fails the test of reasonable doubt. The honest CSI is left with
no option but to admit that there is no evidence from forensic science
that the cloth is medieval. " in http://www.historicaljesusquest.com/


A falta de confiança dos arqueologos neste tesde deve à experiencia
deles em campo e não a considerações teoricas e metafisicas do assunto.
Portanto, mesmo que o teste esteja fisicamente correcto ele pode não
estar arquologicamente correcto. Este é o primeiro ponto que os céticos
esquecem.

A imagem não é uma pintura
http://www.shroudforum.com/
http://www.shroudstory.com/faq/

Ao contrário do que todos os sites ceticos afirmam.

Ha muito mais a dizer, mas acho que esses artigos são suficientes para
desacreditar as opiniões que os ceticos proferiram em relação a este
assunto.
Para mais informação visitem o site http://www.shroudstory.com/faq/

Sérgio Taborda


SUBJECT: Re: Confiabilidade (era Santo sudário e a vanilina(correção)
FROM: "junior_br2001" <junior_br2001@yahoo.com.br>
TO: ciencialist@yahoogrupos.com.br
DATE: 21/03/2005 15:44


Mas voce nao leu a minha conclusão abaixo , em que eu disse que ele é
superior dentro de sua especialidade. No caso voce está comparando
dois testes específicos.
Os teste químico oferece outro indício que nao é bem o específico
para datação. A presença de vanilina pode oferecer indícios que o
tecido seja da idade média . Simplesmente porque não há indícios
dessa susbstancia em nenhum objeto que ultrapasse esssa margem
especulada, isso não deixa de ser uma evidencia de que o objeto de
estudo em questão é mais antigo que isso. A datação pelo carbono-a4
pode ser mais preciso para datar no tempo, mais o segundo pode
forencer indicios ou para confirmar ou p/refutar o primeiro estudo.
sde elas conflitam, é porque tem algo errado. A vanilina não pode
durar muito tempo no tecido

JR

--- Em ciencialist@yahoogrupos.com.br, "rmtakata" <rmtakata@a...>
escreveu
>
> --- Em ciencialist@yahoogrupos.com.br, "junior_br2001"
> > JR: perfeito, em ciencia nao teste que um teste é mais confiável
> > que outro, apareceu evidencias, a missão é investigar.
>
> Claro q. tem. Por exemplo, o teste de amplificacao por PCR para
> detetar a presenca de HIV eh muito mais confiavel do q. o ELISA.
>
> []s,
>
> Roberto Takata





SUBJECT: Re: Tempo de existencia e data de criação, um engano (era ninguem é inocente.)
FROM: "junior_br2001" <junior_br2001@yahoo.com.br>
TO: ciencialist@yahoogrupos.com.br
DATE: 21/03/2005 15:50


OK Oráculo percebi.
Então o segundo estudo é realmente uma tentativa de refutação do
primeiro. Já que ele afirma que realmente o santo sudario é mais
antigo que se pensava.

Mesmo que minha conclusão nas datas foram equivocadas, isso nao
invalida segundo estudo , que é tb um estudo cientifico, e se o
primeiro tb tem falhas, obvimente o segundo tb pode ter. Mas voces
nao podem dizer que o Dr Rogers nao é cientista, e que o estudo dele
nao é valido. Se ele seguiu uma metologia e chegou a conclusoes e
evidencias , o estudo dele tb é válido. Mais essa história de que é
mais ou menos confiável nao é muito pertinente. O que importa é que
os dois procedimentos sao cientificos.
Agora o parecer tecnico final só poderemos ter quando a Igreja
liberar de novo o maldito manto.

JR


--- Em ciencialist@yahoogrupos.com.br, "Oraculo" <oraculo@a...>
escreveu
> Olá Pubmed
>
> Pubmed: Voce tem certeza disso? Eu li 1340 anos, "
>
> Eu já havia percebido isso, você confundiu tempo de existencia com
data do calendário. O tempo de existencia é de 660 anos, o que
significa que foi criado por volta do ANO de 1340 DC. Não significa
que ele tem 1340 anos de existencia, mas que foi criado no ano domini
de 1340. Se o segundo estudo diz que ele foi criado até no máximo o
ano domini de 700, evidentemente não podem ser complementares, não
importa a data inicial (2000 antes de cristo).
>
> Eu expliquei diversas vezes essa confusão, em diversas mensagens
anteriores, de uma relida nelas..:-)
>
> Homero
>
>
> ----- Original Message -----
> From: junior_br2001
> To: ciencialist@yahoogrupos.com.br
> Sent: Monday, March 21, 2005 2:17 PM
> Subject: [ciencialist] Re: ninguem é inocente.
>
>
>
> Segue comentários
>
> --- Em ciencialist@yahoogrupos.com.br, TARCISIO BORGES
<tbs97@f...>
> escreveu
> > On Fri, 18 Mar 2005, pubmed2005 wrote:
> > > O fato de o primeiro estudo ter sido bem sucedido nao
invalida o
> > > segundo estudo, que pode ser muito bem um complemento e até
mesmo
> um
> > > chamado para falhas do primeiro estudo.
> >
> > Invalida sim. O primeiro estudo [C14] deu uma data de 700 anos
+/-
> 60
> > anos. O segundo estudo deu uma data entre 1300 e 3000 anos.
> >
> > Pelo que pude perceber as duas datas não possuem intercecção e
> sendo ambos
> > os métodos aceitos, um dos dois está errado.
>
>
> JR: Voce tem certeza disso? Eu li 1340 anos, inclusive li isso no
> site que voce me apontou, dos céticos( a nao ser que o valor de
600
> seja o minimo do primeiro estudo), . Se o Rogers estipulou um
> mínimo , ele coincide *mais ou menos* com esse valor de 1340. O
que
> pode invalidar é margem especulada em 3000 anos
>
> > Na verdade os cientistas se apegam ao que funciona. Se o antigo
> funciona,
> > mas o novo funciona melhor, os cientistas vão ter uma certa
> resistência em
> > aceitar o novo, pois o antigo ainda vai muito bem.
>
> JR: As técnicas do carbono-14 hoje em dia estão muito melhores
que as
> de 88. Além do mais voce nao prestou atenção ao que foi dito
sobre
> contaminação do tecido com carbono-14 recente, revestimento
> bioplástico e teste relizado nos remendos medievais e não no
tecido
> propriamente dito. Se isso é o que voce diz que funciona, a
ciencia
> pode estar então bem equivocada. se fosse pelo menos um estudo
bem
> seguro sobre o manto poderíamos aplicar o que voce disse. Mas foi
um
> estudo insuficiente com falhas.
>
> >
> >
> > > O segundo estudo foi muito pertinente,e merece atençao
> científica. O
> > > fato de a pesquisa ter sido feita por um químico aposentado
> envolve
> > > muitos preconceitos tb da comunidade científica, etc e tal...
> >
> > Não consigo enxergar estes preconceitos. O Stephen William
Hawking
> possui
> > a idade do meu pai e nem por isso é desacreditado, mesmo
admitindo
> que
> > errou em sua teoria.
> >
> > Como já disse em outro email, o preconceito está na metodologia
> usada por
> > Rogers.
>
>
> JR: Foi uma metologia científica, e não um teste homeopático, ele
> encontrou uma substancia química no remendo, e no resto do tecido
a
> vanilina já não estava presente. Isso é uma evidencia, não um
estudo
> mal-feito.
> O Stephen William Hawking só admitiu que estava errado, quando
sua
> teoria já tinha sido refutada por outro físico...se ele não
admitisse
> pegaria mal. Muitos físicos não gostam dele, nao pela sua
> deficiencia, mas por que ele é um especulador sonhador
>
>
>
>
>
> ##### ##### #####
>
> Para saber mais visite
> http://www.ciencialist.hpg.ig.com.br
>
>
> ##### ##### ##### #####
>
>
> Yahoo! Grupos, um serviço oferecido por:
>
>
>
>
>
>
>
> --------------------------------------------------------------------
----------
> Links do Yahoo! Grupos
>
> a.. Para visitar o site do seu grupo na web, acesse:
> http://br.groups.yahoo.com/group/ciencialist/
>
> b.. Para sair deste grupo, envie um e-mail para:
> ciencialist-unsubscribe@yahoogrupos.com.br
>
> c.. O uso que você faz do Yahoo! Grupos está sujeito aos Termos
do Serviço do Yahoo!.
>
>
>
> [As partes desta mensagem que não continham texto foram removidas]





SUBJECT: Re: [ciencialist] O manto de Turin
FROM: "Oraculo" <oraculo@atibaia.com.br>
TO: <ciencialist@yahoogrupos.com.br>
DATE: 21/03/2005 16:11

Olá Taborda

"Taborda: Ha muito mais a dizer, mas acho que esses artigos são suficientes para
desacreditar as opiniões que os ceticos proferiram em relação a este
assunto."

Evidentemente, não são. Sáo apenas a opinião contrária, defendida por quem acredita. Náo é assim que se decide uma questão científica, e obviamente que seria o mesmo que perguntar ao Papa ou aos bispos que defendem a autenticidade do sudario o que eles acham.

A falta de confiabilidade do carbono-14 é relativa, deve ser tomada com o mesmo cuidado que qualquer outra metodologia. Se fosse SEMPRE pouco confiável, teria sido abandonado e pronto.

Estes artigos ajudam, mas não são a palavra final sobre o assunto. E desde o começo o debate é sobre evidencias e necessiadade de novos estudos, não sobre certezas. Em que pese a tentativa constante de alegar isso, que estamos apresentando certezas baseadas no carbono-14 (e em nossa "cegueira" cética) isso simplesmente não é verdade. Estamos, como você, procurando dados e evidencias, e os estudos que se contradizem tem apenas valor relativo, não absoluto.

Seria espantoso que apenas você tivesse conehcimento dos estudos corretos, que PROVAM seu ponto, enquanto o resto do mundo, céticos em especial, se afoga nos enganos dos testes contrários, não? Um pouco de arrogancia, talvez?

A questão, insisto, não é desacreditar as opiniões de quem quer que seja. A questão é avaliar evidências. Sempre foi essa a questão, e é isso que estamos propondo. Apresentar estudos e dizer que "desacreditam"o que quer que seja é bem religioso, mas pouco científico.

Temos, na verdade, estudos pro e contra. Os pró em geral produzidos por quem já acredita e apenas procura confirmação para crenças. Os mais equilibrados, por pesquisadores idnependentes. Isso deveria bastar, não para desacreditar quem quer que seja, mas para produzir um cuidado maior com a aceitação de "provas absolutas e finais" do que quer que seja.

A não ser para religiosos em geral, claro..:-)

Homero

----- Original Message -----
From: Sergio M. M. Taborda
To: ciencialist@yahoogrupos.com.br
Sent: Monday, March 21, 2005 3:29 PM
Subject: [ciencialist] O manto de Turin


O estudo do manto, é antes de mais nada um estudo arqueologico.
A fisica , a quimica e a biologia, são para a arquologia meras
ferramentas. E o resultado final é a conjunção de todos os factores.
Ou seja, para quem pensa que testes quimicos ou fisicos tem a ultima
palavra desengane-se.Serão os arquologos que terão a ultima palavra.

O teste do caborno 14 não é confiável:
http://www.shroud.com/meacham.htm
http://www.historicaljesusquest.com/
*"* the carbon 14 testing has been so challenged by modern forensic
science (as recently reported by /National Geographic News/ and /PBS)
/that is fails the test of reasonable doubt. The honest CSI is left with
no option but to admit that there is no evidence from forensic science
that the cloth is medieval. " in http://www.historicaljesusquest.com/


A falta de confiança dos arqueologos neste tesde deve à experiencia
deles em campo e não a considerações teoricas e metafisicas do assunto.
Portanto, mesmo que o teste esteja fisicamente correcto ele pode não
estar arquologicamente correcto. Este é o primeiro ponto que os céticos
esquecem.

A imagem não é uma pintura
http://www.shroudforum.com/
http://www.shroudstory.com/faq/

Ao contrário do que todos os sites ceticos afirmam.

Ha muito mais a dizer, mas acho que esses artigos são suficientes para
desacreditar as opiniões que os ceticos proferiram em relação a este
assunto.
Para mais informação visitem o site http://www.shroudstory.com/faq/

Sérgio Taborda


##### ##### #####

Para saber mais visite
http://www.ciencialist.hpg.ig.com.br


##### ##### ##### #####


Yahoo! Grupos, um serviço oferecido por:







------------------------------------------------------------------------------
Links do Yahoo! Grupos

a.. Para visitar o site do seu grupo na web, acesse:
http://br.groups.yahoo.com/group/ciencialist/

b.. Para sair deste grupo, envie um e-mail para:
ciencialist-unsubscribe@yahoogrupos.com.br

c.. O uso que você faz do Yahoo! Grupos está sujeito aos Termos do Serviço do Yahoo!.



[As partes desta mensagem que não continham texto foram removidas]



SUBJECT: Re: O manto de Turin
FROM: "junior_br2001" <junior_br2001@yahoo.com.br>
TO: ciencialist@yahoogrupos.com.br
DATE: 21/03/2005 16:12


--- Em ciencialist@yahoogrupos.com.br, "Oraculo" <oraculo@a...>
escreveu
> Estes artigos ajudam, mas não são a palavra final sobre o assunto.


JR: Ora bolas, nem a palavra dos céticos é a palavra final. Será que
voce tb vai dizer que os arqueólogos tb nao são cientistas e que nao
conhecem os meandros da datação?







SUBJECT: Re: O manto de Turin
FROM: "junior_br2001" <junior_br2001@yahoo.com.br>
TO: ciencialist@yahoogrupos.com.br
DATE: 21/03/2005 16:13


Aliás, os céticos que duvidam dos céticos sao tb céticos

JR

--- Em ciencialist@yahoogrupos.com.br, "junior_br2001"
<junior_br2001@y...> escreveu
>
> --- Em ciencialist@yahoogrupos.com.br, "Oraculo" <oraculo@a...>
> escreveu
> > Estes artigos ajudam, mas não são a palavra final sobre o
assunto.
>
>
> JR: Ora bolas, nem a palavra dos céticos é a palavra final. Será
que
> voce tb vai dizer que os arqueólogos tb nao são cientistas e que
nao
> conhecem os meandros da datação?





SUBJECT: Re: O manto de Turin
FROM: "junior_br2001" <junior_br2001@yahoo.com.br>
TO: ciencialist@yahoogrupos.com.br
DATE: 21/03/2005 16:29


Segue comentários

--- Em ciencialist@yahoogrupos.com.br, "Oraculo" <oraculo@a...>
escreveu
>
> Evidentemente, não são. Sáo apenas a opinião contrária, defendida
por quem acredita. Náo é assim que se decide uma questão científica,
e obviamente que seria o mesmo que perguntar ao Papa ou aos bispos
que defendem a autenticidade do sudario o que eles acham.

JR: A dos céticos tb nao seriam uma opnião contrária, e no entanto
tão escorregadia como as demais?. Por que só as opnioes dos céticos
tem de ser as melhores?


>
> A falta de confiabilidade do carbono-14 é relativa, deve ser tomada
com o mesmo cuidado que qualquer outra metodologia. Se fosse SEMPRE
pouco confiável, teria sido abandonado e pronto.

JR: Não se trata de abandonar . Tudo tem seu tempo. Naquele tempo
pensava-se que com aquelas técnicias do carbono-14 estava-se certo.
Hoje em dias as proprias técnicas do carbono-14 já são bem mais
precisas e mais bem elaboradas do que as de 88.


>
> Estes artigos ajudam, mas não são a palavra final sobre o assunto.
E desde o começo o debate é sobre evidencias e necessiadade de novos
estudos, não sobre certezas. Em que pese a tentativa constante de
alegar isso, que estamos apresentando certezas baseadas no carbono-14
(e em nossa "cegueira" cética) isso simplesmente não é verdade.
Estamos, como você, procurando dados e evidencias, e os estudos que
se contradizem tem apenas valor relativo, não absoluto.

JR: Os estudos de Rogers, dos químicos de Milão, e da California, e
dos soviéticos não foram feitas nos mesmos laboratórios e todas
apresentaram evidencias, e não tão somente opiniões.
No entnato, a palavra dos céticos tb nao são a palavra final. Nõ se
deve esquecer que quem refuta os céticos tb são céticos.


>
> Seria espantoso que apenas você tivesse conehcimento dos estudos
corretos, que PROVAM seu ponto, enquanto o resto do mundo, céticos em
especial, se afoga nos enganos dos testes contrários, não? Um pouco
de arrogancia, talvez?

JR: os céticos tb podem estar esbarrando em arrogancia, por que? Não
são humanos? Aliás quem está questionando que o primeiro estudos tem
falhas tb são céticos

>
> A questão, insisto, não é desacreditar as opiniões de quem quer que
seja. A questão é avaliar evidências. Sempre foi essa a questão, e é
isso que estamos propondo. Apresentar estudos e dizer
que "desacreditam"o que quer que seja é bem religioso, mas pouco
científico.

JR: mas as evidencias já form apresentadas. O que voce quer?

>
> Temos, na verdade, estudos pro e contra. Os pró em geral produzidos
por quem já acredita e apenas procura confirmação para crenças. Os
mais equilibrados, por pesquisadores idnependentes. Isso deveria
bastar, não para desacreditar quem quer que seja, mas para produzir
um cuidado maior com a aceitação de "provas absolutas e finais" do
que quer que seja.

JR: Não, não, nõ me venha com essa. Ninguem apresentou estudos de
religiosos aqui. Os religiosos podem ser se aproveitar destes
estudos, mas eles não foram feitos por religiosos e sim cientistas,
químicos, físicos e arqueólogos, principalmente de gente desvinculada
do espirito religioso






SUBJECT: Re: [ciencialist] Re: O manto de Turin
FROM: "Oraculo" <oraculo@atibaia.com.br>
TO: <ciencialist@yahoogrupos.com.br>
DATE: 21/03/2005 16:31

Olá Pubmed


"Pubmed: Ora bolas, nem a palavra dos céticos é a palavra final."

risos... Muitos risos, embora já um pouco histéricos de cansaço..:-) Para evitar que este debate se torne infinitamente circular e sem sentido, só vou responder a esta mensagem se você encontrar uma mensagem, qualquer mensagem, onde eu ou qualquer outro cético tenha afirmado que a palavra dos céticos é a palavra final..:-)

Sem essa ação, toda discussão volta ao início, e terei de começar tudo de novo, sempre lembrando que são probabilidades e possibilidades, evidencias e confiabilidade, nunca, nunca, a "verdade verdadeira 100%" (que é coisa de religiosos em geral).

Homero

----- Original Message -----
From: junior_br2001
To: ciencialist@yahoogrupos.com.br
Sent: Monday, March 21, 2005 4:12 PM
Subject: [ciencialist] Re: O manto de Turin



--- Em ciencialist@yahoogrupos.com.br, "Oraculo" <oraculo@a...>
escreveu
> Estes artigos ajudam, mas não são a palavra final sobre o assunto.


JR: Ora bolas, nem a palavra dos céticos é a palavra final. Será que
voce tb vai dizer que os arqueólogos tb nao são cientistas e que nao
conhecem os meandros da datação?







##### ##### #####

Para saber mais visite
http://www.ciencialist.hpg.ig.com.br


##### ##### ##### #####


Yahoo! Grupos, um serviço oferecido por:
PUBLICIDADE




------------------------------------------------------------------------------
Links do Yahoo! Grupos

a.. Para visitar o site do seu grupo na web, acesse:
http://br.groups.yahoo.com/group/ciencialist/

b.. Para sair deste grupo, envie um e-mail para:
ciencialist-unsubscribe@yahoogrupos.com.br

c.. O uso que você faz do Yahoo! Grupos está sujeito aos Termos do Serviço do Yahoo!.



[As partes desta mensagem que não continham texto foram removidas]



SUBJECT: Re: O manto de Turin
FROM: "junior_br2001" <junior_br2001@yahoo.com.br>
TO: ciencialist@yahoogrupos.com.br
DATE: 21/03/2005 16:33


Mas voces tb vire e mexe vem com os mesmo assuntos, principalmente
quando a discussão evoluiu e os assuntos já estavam por demais
amadurecidos

JR

--- Em ciencialist@yahoogrupos.com.br, "Oraculo" <oraculo@a...>
escreveu
> Olá Pubmed
>
>
> "Pubmed: Ora bolas, nem a palavra dos céticos é a palavra final."
>
> risos... Muitos risos, embora já um pouco histéricos de cansaço..:-
) Para evitar que este debate se torne infinitamente circular e sem
sentido, só vou responder a esta mensagem se você encontrar uma
mensagem, qualquer mensagem, onde eu ou qualquer outro cético tenha
afirmado que a palavra dos céticos é a palavra final..:-)
>
> Sem essa ação, toda discussão volta ao início, e terei de começar
tudo de novo, sempre lembrando que são probabilidades e
possibilidades, evidencias e confiabilidade, nunca, nunca, a "verdade
verdadeira 100%" (que é coisa de religiosos em geral).
>
> Homero
>
]





SUBJECT: Re: Confiabilidade (era Santo sudário e a vanilina(correção)
FROM: "rmtakata" <rmtakata@altavista.net>
TO: ciencialist@yahoogrupos.com.br
DATE: 21/03/2005 16:36


--- Em ciencialist@yahoogrupos.com.br, "junior_br2001"
> Os teste químico oferece outro indício que nao é bem o específico
> para datação. A presença de vanilina pode oferecer indícios que o
> tecido seja da idade média .

Como eu disse e repito mais uma vez: questao de ver qual eh o mais
confiavel, afinal o metodo A diz q. o pano tem uns 500 a 700 anos; o
metodo B diz q. tem mais de 700 anos - pelo menos um dos dois metodos
estah errado, coisa com q. vc concorda a se valer do trecho abaixo:

> sde elas conflitam, é porque tem algo errado.

Sim, tem algo errado. Entao eh questao de ver qual eh o mais
confiavel. Ainda estah pra se provar q. o metodo quimico eh mais
confiavel do q. o metodo de radioisotopos.

Vc pode medir a altura de uma montanha por GPS e pode medir com a
ajuda de um barometro. Digamos q. o GPS dah a altura como de 356 m
(mais ou menos 1 metro); o barometro dah uma altura de 348 metros
(mais ou menos 5 metros). Eh o caso em questao.

[]s,

Roberto Takata





SUBJECT: Provas cabais e alegações inconsequentes..:-) (era: Carbono-14 )
FROM: "Oraculo" <oraculo@atibaia.com.br>
TO: <ciencialist@yahoogrupos.com.br>
DATE: 21/03/2005 16:42

Olá Pubmed

"Pubmed: É que o Oráculo estava usando seus estudos como uma prova cabal."

Prove isso. É uma afirmação que eu considerarei mentirosa (sem ofensa, claro..:-) se não puder prova-la. Encontre, entre as inúmeras mensagens que trocamos, uma, apenas uma, em que eu apresento uma prova cabal, ou faço qualquer alegação que possa minimamente ser confundida com isso.

É preciso, para que quem lê tenha confiança em suas afirmações, que uma alegação como esta, que pode facilmente ser comprovada com o recurso as mensagens guardadas na lista, que você prove o que afirma.

Caso contrário, a perda de credibilidade, para alegações simples de provar, será imensa. E, claro, bem maior para alegações mais difíceis de comprovar..:-)

Por favor, lembre-se que a lista mantém as mensagens guardadas e que se afirma que eu disse ou aleguei algo, deve estar preparado para demonstar isso..:-)

Em todas as mensagens, praticamente todas, eu pondero que é uma questão de confiabilidade, que estudos podem ser refutados (inclusive o carbono-14) e que o segundo estudo tem de aguardar confirmações independentes para ser aceito com mais confiança. E que se isso ocorrer, não há problema em mudar de conclusão quanto a data do sudário. Se conseguir encvontrar qualquer mensagem que seja diferente, será bem interessante..:-)

Homero


----- Original Message -----
From: junior_br2001
To: ciencialist@yahoogrupos.com.br
Sent: Monday, March 21, 2005 3:15 PM
Subject: [ciencialist] Re: Carbono-14 (era: Sudario)



Oi Takata,

Nao se sinta ressentido, nao foi por mal que falei aquilo. É que o
Oráculo estava usando seus estudos como uma prova cabal. Aí entrei na
nesta questão. Mas a intenção nao foi menosprezar sua pessoa, OK?
Sendo que no fundo, no fundo por mais títulos que tenhamos somos
todos ignorantes em busca do saber

JR

--- Em ciencialist@yahoogrupos.com.br, "rmtakata" <rmtakata@a...>
escreveu
>
> --- Em ciencialist@yahoogrupos.com.br, "junior_br2001"
> > Hein, mas eu nao fiz nenhum estudo, nem pretendo apresentar
> > qualquer trabalho. Eu posso tecer comentários, refutar aqui na
> > lista, mas nunca numa comunidade científica.
>
> Se isso for verdade, entao o seu comentario original nao foi de
muita
> utilidade, afinal nao tenho pretensao alguma de publicar as minhas
> consideracoes sobre o tema em revistas especializadas - estou tao
> somente rebatendo alguns argumentos q. vc tem postado nesta lista.
>
> O q. tb torna inutil o seu comentario de q. eu deveria publicar as
> minhas contas.
>
> > Não sei como é isso, mas deve existir um meio legal de a
descoberta
> > de uma pessoa ser aceita.
>
> Qq pessoa pode submeter seus estudos a uma revista cientifica. Em
> geral seu trabalho passa por uma serie de triagens e eh enviado
para
> a analise de especialistas. Se seu trabalho nao tiver nenhum
> problema, ele poderah ser publicado.
>
> []s,
>
> Roberto Takata





##### ##### #####

Para saber mais visite
http://www.ciencialist.hpg.ig.com.br


##### ##### ##### #####


Yahoo! Grupos, um serviço oferecido por:
PUBLICIDADE




------------------------------------------------------------------------------
Links do Yahoo! Grupos

a.. Para visitar o site do seu grupo na web, acesse:
http://br.groups.yahoo.com/group/ciencialist/

b.. Para sair deste grupo, envie um e-mail para:
ciencialist-unsubscribe@yahoogrupos.com.br

c.. O uso que você faz do Yahoo! Grupos está sujeito aos Termos do Serviço do Yahoo!.



[As partes desta mensagem que não continham texto foram removidas]



SUBJECT: Re: Confiabilidade (era Santo sudário e a vanilina(correção)
FROM: "junior_br2001" <junior_br2001@yahoo.com.br>
TO: ciencialist@yahoogrupos.com.br
DATE: 21/03/2005 16:50


--- Em ciencialist@yahoogrupos.com.br, "rmtakata" <rmtakata@a...>
escreveu
> Sim, tem algo errado. Entao eh questao de ver qual eh o mais
> confiavel. Ainda estah pra se provar q. o metodo quimico eh mais
> confiavel do q. o metodo de radioisotopos.


JR: Bem pelo que eu siba, ainda nao foi realizado um teste
experimental que diga que o segundo estudo esteja errado. Portanto
mesmo que radiosótopo seja o método de dataçao mais confiável não
quer dizer que o segundo esteja errado por ser supostamente menos
confiável

Mesmo que atualmente o radioisotopos seja realmente o método de
dataação mais confiável, isso não siginica que as tecnicas e os
procedimentos de 88 foram realizados corretamente e nas partes dos
tecidos que não sofreram o remendo, e que estavam contaminadas

Portanto se todos os estudos fossem repetidos voce poderia chegar
essa conclusão.





SUBJECT: Re: [ciencialist] Re: O manto de Turin
FROM: "Oraculo" <oraculo@atibaia.com.br>
TO: <ciencialist@yahoogrupos.com.br>
DATE: 21/03/2005 16:57

Olá Pubmed

Pubmed: JR: A dos céticos tb nao seriam uma opnião contrária, e no entanto
tão escorregadia como as demais?. Por que só as opnioes dos céticos
tem de ser as melhores?

Ai meu saquinho, como diriam crianças no parque infantil..:-) Não melhores, no sentido subjetivo, mas mais confiáveis, neste momento. Apenas isso, é difícil compreender?

Neste momento, as evidências mais fortes, mais bem comprovadas e mais confiáveis, pelos parametros científicos aceitos pela comunidade, são estes. Podem mudar? Claro, a qualquer momento. Devem mudar. Mas para mudar não basta a opinião ou o desejo e muito menos a crença de quem quer que seja, cético ou crente.

Para mudar precisa de elementos e evidencias pelo menos tão fortes quanto as anteriores. A datação quimica e o segundo estudo, que contestam os primeiros com carbono, ainda não são. Podem vir a ser? Claro..:-) Mas ainda não são, e como tal devem esperar por confirmação independente.

Apenas isso, simples assim (e repetido "ad neuseum" neste debate..:-)

Pubmed: Não se trata de abandonar . Tudo tem seu tempo. Naquele tempo
pensava-se que com aquelas técnicias do carbono-14 estava-se certo. "

Está enganado. Claro que não vamos concordar nisso, mas sem compreender a técnica e o método de datação, nunca vai comprender porque está enganado. E porque importa pouco se foi feito em 1988 ou agora.

Mas isso só pode ser consertado se se dispuser a aprender sobre o método para ter base suficiente, e isso não parece disposto, como disse algumas mensagens atras, a fazer..:-) Embora o Takata tenha tentado explicar diversas vezes também..:-)

O resto é o mesmo, de novo, repetição dos mesmos argumentos, sem cessar, como "mas os céticos também não são humanos, arrogantes?" etc..:-) Sim, são, eu rerererepetiria, por isso foi desenvolvido um sistema, um método que se dedica a impedir ou diminuir essas falhas humanas, conhecido como método cientifico, etc, etc, etc.. Tudo de novo..:-)

Homero


----- Original Message -----
From: junior_br2001
To: ciencialist@yahoogrupos.com.br
Sent: Monday, March 21, 2005 4:29 PM
Subject: [ciencialist] Re: O manto de Turin



Segue comentários

--- Em ciencialist@yahoogrupos.com.br, "Oraculo" <oraculo@a...>
escreveu
>
> Evidentemente, não são. Sáo apenas a opinião contrária, defendida
por quem acredita. Náo é assim que se decide uma questão científica,
e obviamente que seria o mesmo que perguntar ao Papa ou aos bispos
que defendem a autenticidade do sudario o que eles acham.

JR: A dos céticos tb nao seriam uma opnião contrária, e no entanto
tão escorregadia como as demais?. Por que só as opnioes dos céticos
tem de ser as melhores?


>
> A falta de confiabilidade do carbono-14 é relativa, deve ser tomada
com o mesmo cuidado que qualquer outra metodologia. Se fosse SEMPRE
pouco confiável, teria sido abandonado e pronto.

JR: Não se trata de abandonar . Tudo tem seu tempo. Naquele tempo
pensava-se que com aquelas técnicias do carbono-14 estava-se certo.
Hoje em dias as proprias técnicas do carbono-14 já são bem mais
precisas e mais bem elaboradas do que as de 88.


>
> Estes artigos ajudam, mas não são a palavra final sobre o assunto.
E desde o começo o debate é sobre evidencias e necessiadade de novos
estudos, não sobre certezas. Em que pese a tentativa constante de
alegar isso, que estamos apresentando certezas baseadas no carbono-14
(e em nossa "cegueira" cética) isso simplesmente não é verdade.
Estamos, como você, procurando dados e evidencias, e os estudos que
se contradizem tem apenas valor relativo, não absoluto.

JR: Os estudos de Rogers, dos químicos de Milão, e da California, e
dos soviéticos não foram feitas nos mesmos laboratórios e todas
apresentaram evidencias, e não tão somente opiniões.
No entnato, a palavra dos céticos tb nao são a palavra final. Nõ se
deve esquecer que quem refuta os céticos tb são céticos.


>
> Seria espantoso que apenas você tivesse conehcimento dos estudos
corretos, que PROVAM seu ponto, enquanto o resto do mundo, céticos em
especial, se afoga nos enganos dos testes contrários, não? Um pouco
de arrogancia, talvez?

JR: os céticos tb podem estar esbarrando em arrogancia, por que? Não
são humanos? Aliás quem está questionando que o primeiro estudos tem
falhas tb são céticos

>
> A questão, insisto, não é desacreditar as opiniões de quem quer que
seja. A questão é avaliar evidências. Sempre foi essa a questão, e é
isso que estamos propondo. Apresentar estudos e dizer
que "desacreditam"o que quer que seja é bem religioso, mas pouco
científico.

JR: mas as evidencias já form apresentadas. O que voce quer?

>
> Temos, na verdade, estudos pro e contra. Os pró em geral produzidos
por quem já acredita e apenas procura confirmação para crenças. Os
mais equilibrados, por pesquisadores idnependentes. Isso deveria
bastar, não para desacreditar quem quer que seja, mas para produzir
um cuidado maior com a aceitação de "provas absolutas e finais" do
que quer que seja.

JR: Não, não, nõ me venha com essa. Ninguem apresentou estudos de
religiosos aqui. Os religiosos podem ser se aproveitar destes
estudos, mas eles não foram feitos por religiosos e sim cientistas,
químicos, físicos e arqueólogos, principalmente de gente desvinculada
do espirito religioso






##### ##### #####

Para saber mais visite
http://www.ciencialist.hpg.ig.com.br


##### ##### ##### #####


Yahoo! Grupos, um serviço oferecido por:

São Paulo Rio de Janeiro Curitiba Porto Alegre Belo Horizonte Brasília




------------------------------------------------------------------------------
Links do Yahoo! Grupos

a.. Para visitar o site do seu grupo na web, acesse:
http://br.groups.yahoo.com/group/ciencialist/

b.. Para sair deste grupo, envie um e-mail para:
ciencialist-unsubscribe@yahoogrupos.com.br

c.. O uso que você faz do Yahoo! Grupos está sujeito aos Termos do Serviço do Yahoo!.



[As partes desta mensagem que não continham texto foram removidas]



SUBJECT: Re: [ciencialist] Re: Confiabilidade (era Santo sudário e a vanilina(correção)
FROM: "E m i l i a n o C h e m e l l o" <chemelloe@yahoo.com.br>
TO: <ciencialist@yahoogrupos.com.br>
DATE: 21/03/2005 16:57

Pessoas,

Afinal de contas, sobre o que se está discutindo mesmo? :-)

[ ] 's do Emiliano Chemello
emiliano@quimica.net
http://www.quimica.net/emiliano
http://www.ucs.br/ccet/defq/naeq
[ MSN ] chemelloe@hotmail.com
[ ICQ ] 145060604

" Rien ne se perd, rien ne se crée,
tout se transforme."

Antoine Laurent de Lavoisier (químico francês, 1743 - 1794)

----- Original Message -----
From: junior_br2001
To: ciencialist@yahoogrupos.com.br
Sent: Monday, March 21, 2005 4:50 PM
Subject: [ciencialist] Re: Confiabilidade (era Santo sudário e a vanilina(correção)



--- Em ciencialist@yahoogrupos.com.br, "rmtakata" <rmtakata@a...>
escreveu
> Sim, tem algo errado. Entao eh questao de ver qual eh o mais
> confiavel. Ainda estah pra se provar q. o metodo quimico eh mais
> confiavel do q. o metodo de radioisotopos.


JR: Bem pelo que eu siba, ainda nao foi realizado um teste
experimental que diga que o segundo estudo esteja errado. Portanto
mesmo que radiosótopo seja o método de dataçao mais confiável não
quer dizer que o segundo esteja errado por ser supostamente menos
confiável

Mesmo que atualmente o radioisotopos seja realmente o método de
dataação mais confiável, isso não siginica que as tecnicas e os
procedimentos de 88 foram realizados corretamente e nas partes dos
tecidos que não sofreram o remendo, e que estavam contaminadas

Portanto se todos os estudos fossem repetidos voce poderia chegar
essa conclusão.





##### ##### #####

Para saber mais visite
http://www.ciencialist.hpg.ig.com.br


##### ##### ##### #####


Yahoo! Grupos, um serviço oferecido por:
PUBLICIDADE




------------------------------------------------------------------------------
Links do Yahoo! Grupos

a.. Para visitar o site do seu grupo na web, acesse:
http://br.groups.yahoo.com/group/ciencialist/

b.. Para sair deste grupo, envie um e-mail para:
ciencialist-unsubscribe@yahoogrupos.com.br

c.. O uso que você faz do Yahoo! Grupos está sujeito aos Termos do Serviço do Yahoo!.



[As partes desta mensagem que não continham texto foram removidas]



SUBJECT: Re: O manto de Turin
FROM: "rmtakata" <rmtakata@altavista.net>
TO: ciencialist@yahoogrupos.com.br
DATE: 21/03/2005 16:58


--- Em ciencialist@yahoogrupos.com.br, "Sergio M. M. Taborda"
> O teste do caborno 14 não é confiável:
> http://www.shroud.com/meacham.htm
> http://www.historicaljesusquest.com/

Claaaaro. A teoria da evolucao tb eh uma farsa:

http://www.icr.org/
http://www.creationscience.com/

[]s,

Roberto Takata





SUBJECT: Re: [ciencialist] Atomistica: Ainda se espera por melhores teorias ?
FROM: "Alberto Mesquita Filho" <albmesq@uol.com.br>
TO: <ciencialist@yahoogrupos.com.br>
DATE: 21/03/2005 16:59

----- Original Message -----
From: "Paulo Sérgio Dias"
Sent: Monday, March 21, 2005 8:33 AM
Subject: [ciencialist] Atomistica: Ainda se espera por melhores teorias?

Olá Paulo

> No entanto, segundo o autor do site, o atomo fica "estavel".

Como já disse em msg anterior, não sou químico e portanto não estou em
condições de comentar sobre todas as suas dúvidas. Percebo, no entanto, que
por estar fazendo licenciatura em Física e, no momento, cursando Química
Geral, você deve estar enfrentando algum problema de natureza semântica
(significado das palavras). A palavra chave para a resolução das suas
dúvidas me parece ser "estável" ou "estabilidade". Com efeito, embora nenhum
dos dois esteja errado, parece-me que físicos e químicos dão preferência
para a utilização da palavra com conotações ligeiramente diferentes.

Quando um físico diz que uma partícula é estável, quase sempre ele está
pretendendo dizer que ela provavelmente não irá se degradar no decorrer do
tempo e/ou que a probabilidade para que isso ocorra é diminuta. Via de regra
ele está pensando na partícula como tal, e raramente como componente de um
complexo maior e/ou como algo capaz de se combinar com outra partícula para
formar esse complexo. Formando ou não o complexo, se a partícula for
instável, não é impossível que continue instável, a menos que a união, por
motivos outros, dê uma estabilidade ao conjunto.

A visão do químico já me parece ser um pouquinho diversa. Quando ele fala em
estabilidade de um átomo, molécula ou íon, ele pensa não somente na
partícula em si, mas também no meio em que ela se encontra. Via de regra ele
não pensa em degradação da partícula (a menos que esteja estudando processos
nas áreas de fronteira com a física -- emissões radioativas, por ex.), mas
tão somente em reações químicas, e daí o seu interesse pelo meio em que a
partícula se encontra. Ora, uma partícula que não se modifica no decorrer do
tempo não deixa de ser estável, no sentido encontrado nos dicionários. Sob
esse aspecto, o composto a que você se referiu, o BCl3, será estável num
meio onde não existam elétrons a serem compartilhados com o Boro além
daqueles que ele já está compartilhando com o cloro. Isso não quer dizer que
o BCl3 está "satisfeito" com a sua natureza estrutural, simplesmente ele
está sem opções para adquirir aquela estabilidade teórica que você deve ter
aprendido nos capítulos preliminares da química. Não obstante, o BCl3 não se
desfaz, e nesse aspecto pode-se dizer que neste meio ele permanece estável.
Seria como um homem perdido numa ilha deserta. Estando num período fértil,
dificilmente ele estará satisfeito com a sua solidão, mas nem por isso irá
se desfazer e/ou se suicidar, não é mesmo?

Ao ser colocado em outro meio, por exemplo, em contato com outros elementos
químicos possuidores de elétrons não compartilhados, os químicos percebem
que o BCl3 não permanece como tal, mas combina-se com esses outros
elementos. Ora, se neste novo meio a partícula não permanece como tal, não
vejo incongruência nenhuma em dizermos que neste meio ela não é mais
estável. Haverá agora um processo de tal forma que tanto as partículas
simples como os complexos irão agora coexistir num equilíbrio dinâmico. O
sistema tenderá a uma estabilidade dinâmica (probabilística -- equilíbrio
químico), mas cada partícula estará sempre sujeita a se modificar no
decorrer do tempo.

Bem, é o que eu tinha a dizer a respeito e espero que algum químico
complemente a resposta com dados mais satisfatórios. Quanto a sua dúvida
sobre "uma teoria que explique satisfatoriamente algumas exceções"...

> Ou ainda se espera por uma teoria que explique essas excecoes?

...eu sou suspeito para falar, pois acho que as que temos por aí sequer
servem para que se explique o que de fato acontece com um próton ou um
elétron isolados. Não creio que exista uma teoria, apenas um modelo
matemático que dá certo sem que saibamos exatamente onde, como, quando e nem
porquê, mas esse é um outro problema. A maioria dos químicos que conheço,
talvez por comodismo, aceita isso numa boa e vêem sendo enrolados pelos
"físicos modernos" há bastante tempo.

> O termo "doar" significa que houve a chamada "ligacao covalente dativa"?

Sim, o sentido da expressão "doar" é esse. Na realidade forma-se um complexo
molecular a que o autor está dando o nome de "aduto". No meu tempo de estudo
de Bioquímica e Biofísica creio que o nome que utilizávamos era "Complexo de
Coordenação" (mas não estou muito certo disso). O par de elétrons é
compartilhado pelos dois átomos, ainda que mais próximo daquele que "recebe"
do que daquele que "doa". Eu diria que há uma interposição de órbitas (algo
que algum dia talvez seja possível explicar através das minhas teorias) de
tal maneira que os elétrons consigam se parear dinamicamente com os prótons
dos dois átomos. Mas que isso fique entre nós, pois os seus professores não
irão aceitar essa idéia, pois que para tal precisaríamos pensar numa
natureza não coulombiana dos campos de elétrons e prótons. Uma outra opção
seria pensarmos nas famigeradas "órbitas permitidas", mas deixo isso a
critério daqueles que ainda se lembram da teoria quântica primitiva de Bohr
(1913). A terceira opção seria engolir a função psi de Schrödinger, não sem
antes tomar muito antiácido. Tome cuidado também para não se engasgar com o
tridente da função psi ;-)), algo que nem mesmo o pai da teoria da
relatividade moderna conseguiu evitar. Creio que até concluir a licenciatura
em física você será apresentado a essa matemática, se é que ainda não foi.

> d) Finalmente... que outros topicos da atomistica apresenta muitas
> excecoes `a espera de uma teoria mais abrangente?

Eu diria que todos, mas não repita isso por aí, pelo menos enquanto não
"garfar" o diploma. ;-) Depois disso, cuidado apenas com a "Inquisição dos
Tempos Modernos", caso pretenda fazer carreira universitária.

[ ]´s
Alberto
http://ecientificocultural.com/indice.htm
Mas indiferentemente a tudo isso, o neutrino tem massa, o elétron não é
uma carga elétrica coulombiana e a Terra se move. E a história se repetirá.



SUBJECT: Re: Provas cabais e alegações inconsequentes..:-) (era: Carbono-14 )
FROM: "junior_br2001" <junior_br2001@yahoo.com.br>
TO: ciencialist@yahoogrupos.com.br
DATE: 21/03/2005 16:59


Uá, qualquer coisa voce coloca a autoridade do Takata no meio da
história. Voce faz uso da autoridade. Aí coloquei outra pessoa da
lista que tem mais credenciais no assunto que o Takata no tocante ao
assunto que é o físico Taborda.
Se o takata apresentou os calculos deles, eu tb apresentei vários
estudos feitos em laboratórios, assim como o Taborda apresentou os
dele.

Aliás, mesmo nas discussões anterioeres quando eu era o Bradock,
lembra? Voce sempre colocava o Takata no meio da história. Ué mesma
oportunidade que o Takata tem de aprender eu tb tenho, assim como
voce ou qualquer outro. Mesmo que o Takata seja biólogo e eu
simplesmente nada, isso nao quer dizer que ele tem de ser
necessariamente mais inteligente que os outros.

Mas, nao voce tem necessidade de ficar tão irritado, isso aqui é um
mera discussão boba.

JR

--- Em ciencialist@yahoogrupos.com.br, "Oraculo" <oraculo@a...>
escreveu
> Olá Pubmed
>
> "Pubmed: É que o Oráculo estava usando seus estudos como uma prova
cabal."
>
> Prove isso. É uma afirmação que eu considerarei mentirosa (sem
ofensa, claro..:-) se não puder prova-la. Encontre, entre as inúmeras
mensagens que trocamos, uma, apenas uma, em que eu apresento uma
prova cabal, ou faço qualquer alegação que possa minimamente ser
confundida com isso.
>
> É preciso, para que quem lê tenha confiança em suas afirmações, que
uma alegação como esta, que pode facilmente ser comprovada com o
recurso as mensagens guardadas na lista, que você prove o que afirma.
>
> Caso contrário, a perda de credibilidade, para alegações simples de
provar, será imensa. E, claro, bem maior para alegações mais difíceis
de comprovar..:-)
>
> Por favor, lembre-se que a lista mantém as mensagens guardadas e
que se afirma que eu disse ou aleguei algo, deve estar preparado para
demonstar isso..:-)
>
> Em todas as mensagens, praticamente todas, eu pondero que é uma
questão de confiabilidade, que estudos podem ser refutados (inclusive
o carbono-14) e que o segundo estudo tem de aguardar confirmações
independentes para ser aceito com mais confiança. E que se isso
ocorrer, não há problema em mudar de conclusão quanto a data do
sudário. Se conseguir encvontrar qualquer mensagem que seja
diferente, será bem interessante..:-)
>
> Homero
>
>
> ----- Original Message -----
> From: junior_br2001
> To: ciencialist@yahoogrupos.com.br
> Sent: Monday, March 21, 2005 3:15 PM
> Subject: [ciencialist] Re: Carbono-14 (era: Sudario)
>
>
>
> Oi Takata,
>
> Nao se sinta ressentido, nao foi por mal que falei aquilo. É que
o
> Oráculo estava usando seus estudos como uma prova cabal. Aí
entrei na
> nesta questão. Mas a intenção nao foi menosprezar sua pessoa, OK?
> Sendo que no fundo, no fundo por mais títulos que tenhamos somos
> todos ignorantes em busca do saber
>
> JR
>
> --- Em ciencialist@yahoogrupos.com.br, "rmtakata" <rmtakata@a...>
> escreveu
> >
> > --- Em ciencialist@yahoogrupos.com.br, "junior_br2001"
> > > Hein, mas eu nao fiz nenhum estudo, nem pretendo apresentar
> > > qualquer trabalho. Eu posso tecer comentários, refutar aqui
na
> > > lista, mas nunca numa comunidade científica.
> >
> > Se isso for verdade, entao o seu comentario original nao foi de
> muita
> > utilidade, afinal nao tenho pretensao alguma de publicar as
minhas
> > consideracoes sobre o tema em revistas especializadas - estou
tao
> > somente rebatendo alguns argumentos q. vc tem postado nesta
lista.
> >
> > O q. tb torna inutil o seu comentario de q. eu deveria publicar
as
> > minhas contas.
> >
> > > Não sei como é isso, mas deve existir um meio legal de a
> descoberta
> > > de uma pessoa ser aceita.
> >
> > Qq pessoa pode submeter seus estudos a uma revista cientifica.
Em
> > geral seu trabalho passa por uma serie de triagens e eh enviado
> para
> > a analise de especialistas. Se seu trabalho nao tiver nenhum
> > problema, ele poderah ser publicado.
> >
> > []s,
> >
> > Roberto Takata
>
>
>
>
>
> ##### ##### #####
>
> Para saber mais visite
> http://www.ciencialist.hpg.ig.com.br
>
>
> ##### ##### ##### #####
>
>
> Yahoo! Grupos, um serviço oferecido por:
> PUBLICIDADE
>
>
>
>
> --------------------------------------------------------------------
----------
> Links do Yahoo! Grupos
>
> a.. Para visitar o site do seu grupo na web, acesse:
> http://br.groups.yahoo.com/group/ciencialist/
>
> b.. Para sair deste grupo, envie um e-mail para:
> ciencialist-unsubscribe@yahoogrupos.com.br
>
> c.. O uso que você faz do Yahoo! Grupos está sujeito aos Termos
do Serviço do Yahoo!.
>
>
>
> [As partes desta mensagem que não continham texto foram removidas]





SUBJECT: Re: O manto de Turin
FROM: "junior_br2001" <junior_br2001@yahoo.com.br>
TO: ciencialist@yahoogrupos.com.br
DATE: 21/03/2005 17:03


Voce está flando dos céticos e dos cientistas? Me parece existir uma
diferença básica: Cientistas podem ser céticos, mas nem todo cético
um cientista

O fato é que tem cientistas que são céticos aos primeiro teste, assim
como tem para o segundo. E ponto final

JR

-- Em ciencialist@yahoogrupos.com.br, "Oraculo" <oraculo@a...>
escreveu
> Olá Pubmed
>
> Pubmed: JR: A dos céticos tb nao seriam uma opnião contrária, e no
entanto
> tão escorregadia como as demais?. Por que só as opnioes dos céticos
> tem de ser as melhores?
>
> Ai meu saquinho, como diriam crianças no parque infantil..:-) Não
melhores, no sentido subjetivo, mas mais confiáveis, neste momento.
Apenas isso, é difícil compreender?
>
>]





SUBJECT: Re: Confiabilidade (era Santo sudário e a vanilina(correção)
FROM: "rmtakata" <rmtakata@altavista.net>
TO: ciencialist@yahoogrupos.com.br
DATE: 21/03/2005 17:08


--- Em ciencialist@yahoogrupos.com.br, "junior_br2001"
> JR: Bem pelo que eu siba, ainda nao foi realizado um teste
> experimental que diga que o segundo estudo esteja errado. Portanto
> mesmo que radiosótopo seja o método de dataçao mais confiável não
> quer dizer que o segundo esteja errado por ser supostamente menos
> confiável

Eh, Bradoque, vc a cada dia se supera... Se o metodo eh menos
confiavel, entao o estudo anterior ja' funciona como uma demonstracao
de q. ele estah errado.

Senao eh soh fazer um teste menos eficiente nas pessoas para detetar
HIV, assim, por mais q. elas tenham sido apontadas como soropositivas
no teste anterior mais confiavel, podemos considera'-las
soronegativas com o novo teste. Brilhante.

> tecidos que não sofreram o remendo, e que estavam contaminadas

Ainda estah para se demonstrar q. a amostra era um remendo (Rogers
sugere isso, mas tEm sido contestado). Ainda estah para se demonstrar
q. houve contaminacao na magnitude necessaria para invalidar a
datacao.

> Portanto se todos os estudos fossem repetidos voce poderia chegar
> essa conclusão.

Ou 'a outra. A questao eh, no momento, tao somente qual o metodo mais
confiavel posto q. conflitam nos resultados.

[]s,

Roberto Takata





SUBJECT: Re: Confiabilidade (era Santo sudário e a vanilina(correção)
FROM: "junior_br2001" <junior_br2001@yahoo.com.br>
TO: ciencialist@yahoogrupos.com.br
DATE: 21/03/2005 17:09


Pois é, Emiliano, me parece que esta discussão nao tem fim, e eles me
obrigam a repetir sempre as mesmas coisas

JR

--- Em ciencialist@yahoogrupos.com.br, "E m i l i a n o C h e m e
l l o" <chemelloe@y...> escreveu
> Pessoas,
>
> Afinal de contas, sobre o que se está discutindo mesmo? :-)
>
> [ ] 's do Emiliano Chemello
> emiliano@q...
> http://www.quimica.net/emiliano
> http://www.ucs.br/ccet/defq/naeq
> [ MSN ] chemelloe@h...
> [ ICQ ] 145060604
>
> " Rien ne se perd, rien ne se crée,
> tout se transforme."
>
> Antoine Laurent de Lavoisier (químico francês, 1743 - 1794)
>
> ----- Original Message -----
> From: junior_br2001
> To: ciencialist@yahoogrupos.com.br
> Sent: Monday, March 21, 2005 4:50 PM
> Subject: [ciencialist] Re: Confiabilidade (era Santo sudário e a
vanilina(correção)
>
>
>
> --- Em ciencialist@yahoogrupos.com.br, "rmtakata" <rmtakata@a...>
> escreveu
> > Sim, tem algo errado. Entao eh questao de ver qual eh o mais
> > confiavel. Ainda estah pra se provar q. o metodo quimico eh
mais
> > confiavel do q. o metodo de radioisotopos.
>
>
> JR: Bem pelo que eu siba, ainda nao foi realizado um teste
> experimental que diga que o segundo estudo esteja errado.
Portanto
> mesmo que radiosótopo seja o método de dataçao mais confiável não
> quer dizer que o segundo esteja errado por ser supostamente menos
> confiável
>
> Mesmo que atualmente o radioisotopos seja realmente o método de
> dataação mais confiável, isso não siginica que as tecnicas e os
> procedimentos de 88 foram realizados corretamente e nas partes
dos
> tecidos que não sofreram o remendo, e que estavam contaminadas
>
> Portanto se todos os estudos fossem repetidos voce poderia chegar
> essa conclusão.
>
>
>
>
>
> ##### ##### #####
>
> Para saber mais visite
> http://www.ciencialist.hpg.ig.com.br
>
>
> ##### ##### ##### #####
>
>
> Yahoo! Grupos, um serviço oferecido por:
> PUBLICIDADE
>
>
>
>
> --------------------------------------------------------------------
----------
> Links do Yahoo! Grupos
>
> a.. Para visitar o site do seu grupo na web, acesse:
> http://br.groups.yahoo.com/group/ciencialist/
>
> b.. Para sair deste grupo, envie um e-mail para:
> ciencialist-unsubscribe@yahoogrupos.com.br
>
> c.. O uso que você faz do Yahoo! Grupos está sujeito aos Termos
do Serviço do Yahoo!.
>
>
>
> [As partes desta mensagem que não continham texto foram removidas]





SUBJECT: Re: Confiabilidade (era Santo sudário e a vanilina(correção)
FROM: "junior_br2001" <junior_br2001@yahoo.com.br>
TO: ciencialist@yahoogrupos.com.br
DATE: 21/03/2005 17:11


--- Em ciencialist@yahoogrupos.com.br, "rmtakata" <rmtakata@a...>
escreveu
>
> --- Em ciencialist@yahoogrupos.com.br, "junior_br2001"
> > JR: Bem pelo que eu siba, ainda nao foi realizado um teste
> > experimental que diga que o segundo estudo esteja errado.
Portanto
> > mesmo que radiosótopo seja o método de dataçao mais confiável não
> > quer dizer que o segundo esteja errado por ser supostamente menos
> > confiável
>
> Eh, Bradoque, vc a cada dia se supera...

JR: Isso não deixa de ser um ataque pessoal.





SUBJECT: Re: [ciencialist] Re: Confiabilidade (era Santo sudário e a vanilina(correção)
FROM: "Oraculo" <oraculo@atibaia.com.br>
TO: <ciencialist@yahoogrupos.com.br>
DATE: 21/03/2005 17:13

Olá Pubmed

Pubmed: JR: Bem pelo que eu siba, ainda nao foi realizado um teste
experimental que diga que o segundo estudo esteja errado.

É isso!!!! Talvez agora comprenda o problema. Não foi publicado estudo que diga que o segundo estudo está correto em seus resultados ou estudo que diga que está incorreto. Mas, para o primeiro estudo, de carbono-14, já foram publicados estudos que confirmam seus resultados.

É isso!!! Vê como é simples?..:-)

Homero


----- Original Message -----
From: junior_br2001
To: ciencialist@yahoogrupos.com.br
Sent: Monday, March 21, 2005 4:50 PM
Subject: [ciencialist] Re: Confiabilidade (era Santo sudário e a vanilina(correção)



--- Em ciencialist@yahoogrupos.com.br, "rmtakata" <rmtakata@a...>
escreveu
> Sim, tem algo errado. Entao eh questao de ver qual eh o mais
> confiavel. Ainda estah pra se provar q. o metodo quimico eh mais
> confiavel do q. o metodo de radioisotopos.


JR: Bem pelo que eu siba, ainda nao foi realizado um teste
experimental que diga que o segundo estudo esteja errado. Portanto
mesmo que radiosótopo seja o método de dataçao mais confiável não
quer dizer que o segundo esteja errado por ser supostamente menos
confiável

Mesmo que atualmente o radioisotopos seja realmente o método de
dataação mais confiável, isso não siginica que as tecnicas e os
procedimentos de 88 foram realizados corretamente e nas partes dos
tecidos que não sofreram o remendo, e que estavam contaminadas

Portanto se todos os estudos fossem repetidos voce poderia chegar
essa conclusão.





##### ##### #####

Para saber mais visite
http://www.ciencialist.hpg.ig.com.br


##### ##### ##### #####


Yahoo! Grupos, um serviço oferecido por:
PUBLICIDADE




------------------------------------------------------------------------------
Links do Yahoo! Grupos

a.. Para visitar o site do seu grupo na web, acesse:
http://br.groups.yahoo.com/group/ciencialist/

b.. Para sair deste grupo, envie um e-mail para:
ciencialist-unsubscribe@yahoogrupos.com.br

c.. O uso que você faz do Yahoo! Grupos está sujeito aos Termos do Serviço do Yahoo!.



[As partes desta mensagem que não continham texto foram removidas]



SUBJECT: Re: [ciencialist] Re: O manto de Turin
FROM: "Oraculo" <oraculo@atibaia.com.br>
TO: <ciencialist@yahoogrupos.com.br>
DATE: 21/03/2005 17:15

Olá Pubmed

Pubmed: Aliás, os céticos que duvidam dos céticos sao tb céticos"

Se eles duvidam como métdo de conhecimento, sim, são céticos também. Se duvidam por "acreditar" em outra coisa, que a alegação cética inicial contradiz, não são, são apenas crentes exercendo seu legítimo direito de crer no que desejarem.

É uma diferença importante.

Homero


----- Original Message -----
From: junior_br2001
To: ciencialist@yahoogrupos.com.br
Sent: Monday, March 21, 2005 4:13 PM
Subject: [ciencialist] Re: O manto de Turin



Aliás, os céticos que duvidam dos céticos sao tb céticos

JR

--- Em ciencialist@yahoogrupos.com.br, "junior_br2001"
<junior_br2001@y...> escreveu
>
> --- Em ciencialist@yahoogrupos.com.br, "Oraculo" <oraculo@a...>
> escreveu
> > Estes artigos ajudam, mas não são a palavra final sobre o
assunto.
>
>
> JR: Ora bolas, nem a palavra dos céticos é a palavra final. Será
que
> voce tb vai dizer que os arqueólogos tb nao são cientistas e que
nao
> conhecem os meandros da datação?





##### ##### #####

Para saber mais visite
http://www.ciencialist.hpg.ig.com.br


##### ##### ##### #####


Yahoo! Grupos, um serviço oferecido por:







------------------------------------------------------------------------------
Links do Yahoo! Grupos

a.. Para visitar o site do seu grupo na web, acesse:
http://br.groups.yahoo.com/group/ciencialist/

b.. Para sair deste grupo, envie um e-mail para:
ciencialist-unsubscribe@yahoogrupos.com.br

c.. O uso que você faz do Yahoo! Grupos está sujeito aos Termos do Serviço do Yahoo!.



[As partes desta mensagem que não continham texto foram removidas]



SUBJECT: Re: Confiabilidade (era Santo sudário e a vanilina(correção)
FROM: "junior_br2001" <junior_br2001@yahoo.com.br>
TO: ciencialist@yahoogrupos.com.br
DATE: 21/03/2005 17:15


--- Em ciencialist@yahoogrupos.com.br, "rmtakata" <rmtakata@a...>
escreveu
>
> --- Em ciencialist@yahoogrupos.com.br, "junior_br2001"
> > JR: Bem pelo que eu siba, ainda nao foi realizado um teste
> > experimental que diga que o segundo estudo esteja errado.
Portanto
> > mesmo que radiosótopo seja o método de dataçao mais confiável não
> > quer dizer que o segundo esteja errado por ser supostamente menos
> > confiável
>
> Eh, Bradoque, vc a cada dia se supera... Se o metodo eh menos
> confiavel, entao o estudo anterior ja' funciona como uma
demonstracao
> de q. ele estah errado.



JR: Forneça as provas experimentais que o segundo estudo esteja
errado. Criticar qulquer palpiteiro pode fazer. Aliás há uma
diferença entre crítica e questionamento

Aliás, seria mais adequado a um popperiano dizer *supostamente ou
aparentemente mais confiável* do que *mais confiável*

Seria lícito voce rever algumas consideraççoes do Alberto sobre
ceticismo e confiabilidade

JR





SUBJECT: Re: Provas cabais e alegações inconsequentes..:-) (era: Carbono-14 )
FROM: "rmtakata" <rmtakata@altavista.net>
TO: ciencialist@yahoogrupos.com.br
DATE: 21/03/2005 17:16


--- Em ciencialist@yahoogrupos.com.br, "junior_br2001"
> Aí coloquei outra pessoa da lista que tem mais credenciais no
> assunto que o Takata no tocante ao assunto que é o físico Taborda.

Ueh o Taborda largou engenharia fisica para cursar fisica?

Se for questao de fisica eh soh perguntar para o Prof. Leo ou Prof.
Zeca.

> isso nao quer dizer que ele tem de ser necessariamente mais
> inteligente que os outros.

Nunca, jamais, pus-me como mais inteligente do q. alguem (ainda mais
nesta lista). Se for por falta de ter dito o contrario, digo agora:
sou o mais burro desta lista, sou o menos inteligente da internet,
sou o mais ignorante da Terra, sou o menos esperto da Galaxia.

[]s,

Roberto Takata





SUBJECT: Re: Confiabilidade (era Santo sudário e a vanilina(correção)
FROM: "junior_br2001" <junior_br2001@yahoo.com.br>
TO: ciencialist@yahoogrupos.com.br
DATE: 21/03/2005 17:18


Como não, apresentei trabalhos realizados em laboratórios sobre as
falhas do carbono-14, o Taborda tb apresentou. Mas nao foi
apresentado nenhum trabalho experimental que mostre que as evidencias
encontradas no segundo estudo não sao evidencias. Só palpites de
céticos sem base experimental. Ali, no botequim da esquina tb tem um
bocado de palpiteiro

JR

--- Em ciencialist@yahoogrupos.com.br, "Oraculo" <oraculo@a...>
escreveu
> Olá Pubmed
>
> Pubmed: JR: Bem pelo que eu siba, ainda nao foi realizado um teste
> experimental que diga que o segundo estudo esteja errado.
>
> É isso!!!! Talvez agora comprenda o problema. Não foi publicado
estudo que diga que o segundo estudo está correto em seus resultados
ou estudo que diga que está incorreto. Mas, para o primeiro estudo,
de carbono-14, já foram publicados estudos que confirmam seus
resultados.
>
> É isso!!! Vê como é simples?..:-)






SUBJECT: Re: [ciencialist] Re: Provas cabais e alegações inconsequentes..:-) (era: Carbono-14 )
FROM: "E m i l i a n o C h e m e l l o" <chemelloe@yahoo.com.br>
TO: <ciencialist@yahoogrupos.com.br>
DATE: 21/03/2005 17:26

ahaha... já que o clima é de descontração:

[Takata]
Nunca, jamais, pus-me como mais inteligente do q. alguem (ainda mais
nesta lista). Se for por falta de ter dito o contrario, digo agora:
sou o mais burro desta lista, sou o menos inteligente da internet,
sou o mais ignorante da Terra, sou o menos esperto da Galaxia.

[Emiliano]
O Takata é esperto. Disse que era o mais burro desta lista, menos
inteligente da Internet, mais ignorante da Terra, menos esperto da Galaxia,
porém, acredito eu, pensa ser o mais genial do Universo! :-) (brincadeira)

Falando sério. Acho que as discussões estão derivando muito. Está se
discutindo uma coisa... que deriva para a coisa 2, que por sua vez deriva
para a coisa 3 e assim por diante. E no final, estaremos (não duvido) que
irá sair do BBB5.


Cadê o Brudna!? :-)

Abraços coletivos,
Emiliano Chemello

----- Original Message -----
From: rmtakata
To: ciencialist@yahoogrupos.com.br
Sent: Monday, March 21, 2005 5:16 PM
Subject: [ciencialist] Re: Provas cabais e alegações inconsequentes..:-)
(era: Carbono-14 )



--- Em ciencialist@yahoogrupos.com.br, "junior_br2001"
> Aí coloquei outra pessoa da lista que tem mais credenciais no
> assunto que o Takata no tocante ao assunto que é o físico Taborda.

Ueh o Taborda largou engenharia fisica para cursar fisica?

Se for questao de fisica eh soh perguntar para o Prof. Leo ou Prof.
Zeca.

> isso nao quer dizer que ele tem de ser necessariamente mais
> inteligente que os outros.

Nunca, jamais, pus-me como mais inteligente do q. alguem (ainda mais
nesta lista). Se for por falta de ter dito o contrario, digo agora:
sou o mais burro desta lista, sou o menos inteligente da internet,
sou o mais ignorante da Terra, sou o menos esperto da Galaxia.

[]s,

Roberto Takata





##### ##### #####

Para saber mais visite
http://www.ciencialist.hpg.ig.com.br


##### ##### ##### #####


Yahoo! Grupos, um serviço oferecido por:

São Paulo Rio de Janeiro Curitiba Porto Alegre Belo Horizonte Brasília





Links do Yahoo! Grupos

Para visitar o site do seu grupo na web, acesse:
http://br.groups.yahoo.com/group/ciencialist/

Para sair deste grupo, envie um e-mail para:
ciencialist-unsubscribe@yahoogrupos.com.br

O uso que você faz do Yahoo! Grupos está sujeito aos Termos do Serviço do
Yahoo!.




SUBJECT: Re: Provas cabais e alegações inconsequentes..:-) (era: Carbono-14 )
FROM: "junior_br2001" <junior_br2001@yahoo.com.br>
TO: ciencialist@yahoogrupos.com.br
DATE: 21/03/2005 17:28


--- Em ciencialist@yahoogrupos.com.br, "rmtakata" <rmtakata@a...>
escreveu
> Nunca, jamais, pus-me como mais inteligente do q. alguem (ainda
mais
> nesta lista). Se for por falta de ter dito o contrario, digo agora:
> sou o mais burro desta lista, sou o menos inteligente da internet,
> sou o mais ignorante da Terra, sou o menos esperto da Galaxia.
>



JR: Voce nao disse de fato. Mas usam seu nome nos assuntos, uma forma
de apelo a autoridade. Se voce nao se considera autoridade nenhuma ja
é outra historia.

Se usarem voce como argumento, eu tenho o direito de rebater. Afinal,
é mesma coisa de utilizarem o Junior como sendo biólogo numa
discussão de física. A outra pessoa do outro lado da discussão tem o
direito de dizer que nao sou a pessoa mais adequada para opiniar
sobre o tema. Aliás, quando o assunto é física , eu nunca me meto.
Sempre falo que nao entendo do assunto. Aliás,quando o assunto é
física eu sempre sou o aluno.
Minhas particpação tem sido como cético(claro que nao cético de
carteirinha) e palpiteiro sem maiores pretensões numa discussao sobre
o Sudário

Nada pessoal contra voce de modo algum





SUBJECT: Re: Provas cabais e alegações inconsequentes..:-) (era: Carbono-14 )
FROM: "junior_br2001" <junior_br2001@yahoo.com.br>
TO: ciencialist@yahoogrupos.com.br
DATE: 21/03/2005 17:40




--- Em ciencialist@yahoogrupos.com.br, "E m i l i a n o C h e m e
l l o" <chemelloe@y...> escreveu
> >
> Ueh o Taborda largou engenharia fisica para cursar fisica?
>

JR: E a grade curricular dos que estudam engenharia física nao conta?
Pelo que eu saiba , eles podem dar aulas de física









SUBJECT: Re: [ciencialist] Gravitons
FROM: Daniel Moser <dmoser@pop.com.br>
TO: ciencialist@yahoogrupos.com.br
DATE: 21/03/2005 18:11

Prof. Luiz,

Minha opinião a respeito desta teoria é: ela é simplesmente SURPREENDENTE!
Achei ela muito interessante, mas ainda não tive tempo de fazer uma
análise cuidadosa da parte "quantitativa". Mas a parte "qualitativa", me
pareceu extraordinária!!!

Sob os pontos levantados pelo Victor:
1. Para Einstein, a gravidade cria um espaço tempo curvo, e não o
contrário (o texto está ok)
2. Da maneira que estão escritas, AO PÉ DA LETRA, são erros conceituais.
Mas o contexto e as conclusões tiradas pelo autor demonstram um correto
conhecimento de todos os conceitos envolvidos no trecho.
As alegações a) e b) não me parecem de alguém que está realmente
interessado em conhecer todas as possibilidades da natureza e da
ciência. Por acaso o autor destes comentários está TOTALMENTE a par dos
mecanismos que formam os átomos? Como os elétrons se "fixam " e se
mantém estáveis em suas órbitas em torno do núcleo, e como realizam seu
movimento continuamente? Pois eu posso dizer que nenhum cientista hoje
possui tal conhecimento, o portanto não podemos desconsiderar a hipótese
do texto "TQG".

Atenciosamente,
Daniel Moser
Aluno de 3º ano - Bacharelo em Física - USP


JVictor escreveu:

> Luiz Ferraz Netto escreveu:
>
> Apreciaria comentários sobre o seguinte texto:
>
> http://rolfguthmann.sites.uol.com.br/TQG/tqg.html
>
>
> Professor Luiz Ferraz,
>
> > Lí o artigo até o quinto parágrafo da seção 2. E parei. Até chegar
> > ao item 2 encontramos hipóteses absolutamente estranhas e sem
> > significado físico, pelo que já peço ao autor que seja mais claro..
> >
> > Com relação ao item 2- O que é gravidade?, transcrevo dois conceitos
> > absolutamente equivocados, que comento rapidamente logo após o destaque.
>
> 1. " Para Einstein, a gravidade causava uma deformação no espaço tempo
> contínuo e, com esta idéia, ele desenvolveu uma álgebra muito complexa
> que a descreve apenas geometricamente."
>
> Comentário: 1) É o contrário: a gravidade não é causa; é efeito da
> curvatura do espaço. 2) Einstein NÃO desenvolveu nenhuma álgebra muito
> complexa. Ele usou um formalismo matemático, novo na época, criado por
> outros matemáticos, baseado em geometria diferencial, formalismo esse
> de jeito nenhum complexo. Desculpe-me o autor, mas esse é o tipo do
> erro conceitual inadmissível para alguém que se propõe a elaborar uma
> nova teoria.
>
> > 2. - " Uma outra evidência se reporta a teoria da relatividade de
> > Einstein, que está embasada no princípio da equivalência o qual
> > estabelece que a massa, ou a masssa inercial e o peso, ou massa
> > gravitacional de materiais diferentes sofrem a mesma aceleração... "
>
> Comentários: Os erros conceituais aquí são para ninguém botar
> defeito!. Massa e peso são coisas diferentes!. E esse princípio de
> equivalência é devido a Galileu e é chamado Princípio de Equivalência
> fraco. Einstein o estendeu a toda a física, e o PE de Einstein tem tem
> a seguinte forma:
>
> " Um laboratório local, não girante, em queda livre, é equivalente, para
> a realização de qualquer experiência, a um referencial inercial numa
> região do espaço sem gravidade." Esta é a forma forte do Princípio de
> Equivalência, devido a Einstein. É o mesmo que dizer: se você estiver em
> uma nave, no espaço, e esta estiver sob uma aceleração, digamos, de
> 10m/s/s, você será empurrado contra o piso da nave e não saberá dizer
> se essa aceleração é devida aos motores que a provocam ou se é devida
> aos efeitos gravitacionais de alguma massa grande nas proximidades,
> como, por exemplo, a terra(cujos efeitos gravitacionais, que todos
> conhecemos, expressam-se por uma aceleração, o g= aprox. a 10m/s/s e nos
> pregados no chão, do mesmo jeito que o astronauta em relação ao piso da
> nave, com tudo que tem direito!) ou outro planeta. Ou seja, essas
> aceleraçãos são equivalentes!. Na nave, uma balança dirá que o seu peso
> será, digamos, 700kg.m/s/s, se for um gordinho, como eu, de 70 Kg de
> massa. Se os motores aumentarem a aceleração, a balança vai acusar um
> peso maior. Na terra, se consideramos que aceleração gravitacional tem
> o mesmo valor, aprx. 10m/s/s, uma balança indicará o mesmo peso para
> você, ou seja 700kg.m/s/s! Refletindo sobre coisas assim, num momento
> de inspiração, Einstein chegou ao seu racicínio mais feliz, segundo
> disse: concluiu que uma pessoa em queda livre não sente seu próprio
> peso! Ou seja, para alguém que cai em queda livre, não existe, ao menos
> nas vizinhanças, qualquer campo gravitacional. Com esse conceito,
> Einstein mostra também que a gravidade é uma coisa cuja existência é
> relativa. Independe da constituição dos corpos. Aliás, o PE fraco já diz
> a mesma coisa. É esta a base da TRG! Compare, então, o que expressei
> acima, com suas afirmações abaixo e decida o autor se ainda mantém:
>
> a) sua hipótese, item 1, que não me animei a comentar(idem, outras, de
> mesma qualidade):
> "Veremos que a a gravidade é o resultado do desequilíbrio entre força
> coloumbiana e força centrípeta",
>
> b) ou com esta, quinto parágrafo do item 2:
> " ...podemos afirmar que a gravidade é uma inércia, e é causada por uma
> força eletromagnética de origem nuclear, estes indícios( (??),
> estupefação minha)) ainda apontam, que a diferença relativa das forças
> eletrostática e centrífuga nos átomos, é a fonte ou causa da
> gravidade..."
> Comentário: não era centrípeta?. Só lembro uma coisinha adicional: força
> centrípeta e força centrífuga são aplicadas em corpos diferentes,
> portanto, uma não é reação à outra, e a centrífuga surge em alguns
> sistemas de referência e outras não! Logo..." Pode esclarecer o
> seguinte enunciado, de vossa lavra?: " ...podemos afirmar que gravidade
> é uma inércia, e é causada por uma força eletromagnética de origem
> nuclear".
>
> c) todo o resto que não lí...
>
> Não podendo ir mais além, por conta do " impacto" das coisas acima, e
> logo no início do trabalho(!), sugiro fortemente que o autor, se
> estiver com vontade e for corajoso, como parece que o é, publicando
> esta, desenvolva SUA OUTRA TEORIA. Quem nem lí ainda!
>
> Sds,
>
> Victor.




--
No virus found in this outgoing message.
Checked by AVG Anti-Virus.
Version: 7.0.308 / Virus Database: 266.7.4 - Release Date: 18/3/2005



SUBJECT: Re: Eter Luminifero (era: ninguem é inocente.)
FROM: "marcelomjr" <marcelomjr@hotmail.com>
TO: ciencialist@yahoogrupos.com.br
DATE: 21/03/2005 19:17


Olá,

Já que citaram a questao da (suposta) inexistência do Ether (ou
aether)de forma tão conclusiva e dogmatica, gostaria apenas relembrar
que nem todos os cientistas do mundo pensam assim e/ou creem que foi
dada a ultima palavra sobre este assunto. Muito pelo contrario, um
pouco mais de pesquisa sobre este tema na rede mundial de
computadores, mostrará que a questão da existencia ou da inexistencia
de um Ether ainda está longe (e muito longe) de acabar na comunidade
cientifica (ao contrário do que pessoas mal informadas tem promulgado
abundantemente aos leigos por aí), aliás como sugere o excerto
abaixo, retirado do site da "Natural Philosophy Alliance (NPA)"
[http://mywebpages.comcast.net/Deneb/]:

http://mywebpages.comcast.net/Deneb/Steps.htm
"(...)
7. TAKE THE SAGNAC EXPERIMENT SERIOUSLY. In this case, the "infrared
film" needed was provided by Sagnac in 1913, when he looked for the
aether with an interferometer that rotated, instead of translating in
a near-straight line. Something caused his fringes to shift as viewed
on the rotating platform, and these shifts meant that the velocity of
light was remaining constant relative to the laboratory. Sagnac
advanced this as experimental proof against the second postulate of
SR, which it actually was. His method has been modified and repeated
many times since his day, and currently is being tested constantly
among the satellites of the Global Positioning System (GPS). Every
single time, when rotation of a light path within a surrounding
dominant coordinate system occurs, fringes are shifted, light
velocities are altered, and the existence of a luminiferous aether is
strongly inferred--all contrary to SR.

Establishment physicists have usually ignored the Sagnac effect, or
once in a while they have attempted to explain it in terms of special
or general relativity--but all of these attempts have fallen short.
(...)"


Um abraço,
Marcelo M. Jr.

--- Em ciencialist@yahoogrupos.com.br, "Oraculo" <oraculo@a...>
escreveu
> Olá Pubmed
>
> Pubmed: Naõ começa tropeçar nos seus proprios argumentos, o Éter
luminífero é
> um mito que foi derrubado...uma teoria absurda que foi demonstrada
> falsa, portanto nao passa de um mito"
>
> Está enganado. O éter luminífero foi uma teoria que se embasava nos
dados e conhecimentos da época. Explicava eventos e fenomenos do
mundo físico, através de observações e experimentos. Ainda que tenha
se mostrado irreal depois, com mais dados, nada nele lembra um mito,
como duendes ou seres divinos de religiões. Sua estrutura, função e
principalmente sua posterior refutação o tornam totalmente diferente
de mitos e lendas.
>
> Os erros e enganos que as teorias cientificas as vezes apresentam
fazem parte do método de criar repostas e depois testá-las
rigorosamente, ajustando se necessário e abandonando o que for
refutado. Mitos, por sua vez, se mantém indefinidamente, sem mudança
ou ajuste. Não faz parte de sua natureza ser refutado, nme precisam
de dados e testes para se manter.
>
> Se descobrirem que o Big Bang é incorreto, e não absurdo como
parece pensar que pode acontecer, será uma ação legitimamente
científica e os cientistas, tanto os que criaram a teoria quanto os
que a refutaram, ficarão felizes e aceitarão as novas conclusões. O
Big Bang é uma teoria que, neste momento, é mais capaz de explicar o
surgimento do universo que qualquer outra, e perdurará até que mais
dados a comprovem ou refutem. Não é nem de longe um mito, nem mesmo
se descobrirmos que está incorreto (de novo, a teoria será, não
absurda, mas incorreta).
>
> Veja, hoje a Big Bang explica porque o céu não é totalmente claro,
mesmo durante a noite, explica como podem as galáxias estarem se
afastando, explica a formação de estrelas, nuvens de gás estelar,
materia escura, elementos quimicos, etc. Para substitui-la, é preciso
uma teoria que, além de explicar tudo isso, ainda explique coisas que
escapam a teoria do BB. Se o fizer, o BB será ultrapassado, você (ou
quem for responsável) ganhará um premio Nobel, e a ciência terá novos
dados e hipóteses para testar. Tudo com o rigor necessário.
>
> O Éter Luminífero explicava a propagação da luz e dezenas de outros
fenomenos observados, que depois foram melhor explicados pela teoria
eletromagnética. Apenas isso, não foi a substituição de um mito por
outro.
>
> Precisa compreender isso, ou jamais compreenderá o que é a ciência
e o que a torna diferente de crenças e mitos.
>
> Homero
>
> ----- Original Message -----
> From: pubmed2005
> To: ciencialist@yahoogrupos.com.br
> Sent: Saturday, March 19, 2005 3:07 AM
> Subject: [ciencialist] Re: ninguem é inocente.
>
>
>
> Naõ começa tropeçar nos seus proprios argumentos, o Éter
luminífero é
> um mito que foi derrubado...uma teoria absurda que foi
demonstrada
> falsa, portanto nao passa de um mito
>
> Se descobrirem que o Bing bang nao é teoria mais proxima da
origem do
> universo, isso vai soar tão absurdo como a crença no boi tatá. Se
o
> boi tatá nao existe, se foi comprovado a sua inexistencia, isso
vai
> refutar que o boi tatá só existe na imaginação criativa das
pessoas.
> Assim como o Éter luminífero preencheu as mentes científicas dos
> séculos anteriores. Tudo nao se passou de uma ilusão
>
> Quanto ao Sudário, já está mais que esclarecida minha posição,
nao
> tenho que ficar repetindo os mesmos arguimentos de forma circular
> como voce vem fazendo
>
>
>
> --- Em ciencialist@yahoogrupos.com.br, "Oraculo" <oraculo@a...>
> escreveu
> > Olá Pubmed
>
> >
> > risos..:-) É enorme..:-) No primeiro caso, a conclusão, e não
> crença, foi abandonada devido a evidencias posteriores que
refutaram
> a teoria do éter. No segundo, nada vai mudar e quem crê nisso vai
> continuar crendo, sem que evidencias ou provas ou aspectos
materiais
> ou racionais tenham o poder de interferir oiu mudar essa crença.
A
> diferença, como pode notar é enorme..:-) A primeira, uma teoria
> cientifica, esperava por dados e evidencias que a confirmassem, e
se
> conformou com a refutação por falta de provas. A segunda, criada
da
> imaginação humana, vai se manter indefinidamente, exatamente como
foi
> criada, dependendo apenas da crença subjetiva de seres humanos.
> Enorme diferença..:-)
> >
>
>
>
>
>
>
> ##### ##### #####
>
> Para saber mais visite
> http://www.ciencialist.hpg.ig.com.br
>
>
> ##### ##### ##### #####
>
>
> Yahoo! Grupos, um serviço oferecido por:
>
>
>
>
>
>
>
> --------------------------------------------------------------------
----------
> Links do Yahoo! Grupos
>
> a.. Para visitar o site do seu grupo na web, acesse:
> http://br.groups.yahoo.com/group/ciencialist/
>
> b.. Para sair deste grupo, envie um e-mail para:
> ciencialist-unsubscribe@yahoogrupos.com.br
>
> c.. O uso que você faz do Yahoo! Grupos está sujeito aos Termos
do Serviço do Yahoo!.
>
>
>
> [As partes desta mensagem que não continham texto foram removidas]





SUBJECT: Gravitons
FROM: JVictor <jvoneto@uol.com.br>
TO: "ciencialist@yahoogrupos.com.br" <ciencialist@yahoogrupos.com.br>
DATE: 21/03/2005 19:44

Luiz Ferraz Netto escreveu:

Apreciaria comentários sobre o seguinte texto:

http://rolfguthmann.sites.uol.com.br/TQG/tqg.html


Professor Luiz Ferraz,

> Lí o artigo até o quinto parágrafo da seção 2. E parei. Até chegar
> ao item 2 encontramos hipóteses absolutamente estranhas e sem
> significado físico, pelo que já peço ao autor que seja mais claro..
>
> Com relação ao item 2- O que é gravidade?, transcrevo dois conceitos
> absolutamente equivocados, que comento rapidamente logo após o destaque.

1. " Para Einstein, a gravidade causava uma deformação no espaço tempo
contínuo e, com esta idéia, ele desenvolveu uma álgebra muito complexa
que a descreve apenas geometricamente."

Comentário: 1) É o contrário: a gravidade não é causa; é efeito da
curvatura do espaço. 2) Einstein NÃO desenvolveu nenhuma álgebra muito
complexa. Ele usou um formalismo matemático, novo na época, criado por
outros matemáticos, baseado em geometria diferencial, formalismo esse
de jeito nenhum complexo. Desculpe-me o autor, mas esse é o tipo do
erro conceitual inadmissível para alguém que se propõe a elaborar uma
nova teoria.

> 2. - " Uma outra evidência se reporta a teoria da relatividade de
> Einstein, que está embasada no princípio da equivalência o qual
> estabelece que a massa, ou a masssa inercial e o peso, ou massa
> gravitacional de materiais diferentes sofrem a mesma aceleração... "

Comentários: Os erros conceituais aquí são para ninguém botar
defeito!. Massa e peso são coisas diferentes!. E esse princípio de
equivalência é devido a Galileu e é chamado Princípio de Equivalência
fraco. Einstein o estendeu a toda a física, e o PE de Einstein tem tem
a seguinte forma:

" Um laboratório local, não girante, em queda livre, é equivalente, para
a realização de qualquer experiência, a um referencial inercial numa
região do espaço sem gravidade." Esta é a forma forte do Princípio de
Equivalência, devido a Einstein. É o mesmo que dizer: se você estiver em
uma nave, no espaço, e esta estiver sob uma aceleração, digamos, de
10m/s/s, você será empurrado contra o piso da nave e não saberá dizer
se essa aceleração é devida aos motores que a provocam ou se é devida
aos efeitos gravitacionais de alguma massa grande nas proximidades,
como, por exemplo, a terra(cujos efeitos gravitacionais, que todos
conhecemos, expressam-se por uma aceleração, o g= aprox. a 10m/s/s e nos
pregados no chão, do mesmo jeito que o astronauta em relação ao piso da
nave, com tudo que tem direito!) ou outro planeta. Ou seja, essas
aceleraçãos são equivalentes!. Na nave, uma balança dirá que o seu peso
será, digamos, 700kg.m/s/s, se for um gordinho, como eu, de 70 Kg de
massa. Se os motores aumentarem a aceleração, a balança vai acusar um
peso maior. Na terra, se consideramos que aceleração gravitacional tem
o mesmo valor, aprx. 10m/s/s, uma balança indicará o mesmo peso para
você, ou seja 700kg.m/s/s! Refletindo sobre coisas assim, num momento
de inspiração, Einstein chegou ao seu racicínio mais feliz, segundo
disse: concluiu que uma pessoa em queda livre não sente seu próprio
peso! Ou seja, para alguém que cai em queda livre, não existe, ao menos
nas vizinhanças, qualquer campo gravitacional. Com esse conceito,
Einstein mostra também que a gravidade é uma coisa cuja existência é
relativa. Independe da constituição dos corpos. Aliás, o PE fraco já diz
a mesma coisa. É esta a base da TRG! Compare, então, o que expressei
acima, com suas afirmações abaixo e decida o autor se ainda mantém:

a) sua hipótese, item 1, que não me animei a comentar(idem, outras, de
mesma qualidade):
"Veremos que a a gravidade é o resultado do desequilíbrio entre força
coloumbiana e força centrípeta",

b) ou com esta, quinto parágrafo do item 2:
" ...podemos afirmar que a gravidade é uma inércia, e é causada por uma
força eletromagnética de origem nuclear, estes indícios( (??),
estupefação minha)) ainda apontam, que a diferença relativa das forças
eletrostática e centrífuga nos átomos, é a fonte ou causa da
gravidade..."
Comentário: não era centrípeta?. Só lembro uma coisinha adicional: força
centrípeta e força centrífuga são aplicadas em corpos diferentes,
portanto, uma não é reação à outra, e a centrífuga surge em alguns
sistemas de referência e outras não! Logo..." Pode esclarecer o
seguinte enunciado, de vossa lavra?: " ...podemos afirmar que gravidade
é uma inércia, e é causada por uma força eletromagnética de origem
nuclear".

c) todo o resto que não lí...

Não podendo ir mais além, por conta do " impacto" das coisas acima, e
logo no início do trabalho(!), sugiro fortemente que o autor, se
estiver com vontade e for corajoso, como parece que o é, publicando
esta, desenvolva SUA OUTRA TEORIA. Quem nem lí ainda!

Sds,

Victor.







SUBJECT: Re: O manto de Turin
FROM: "junior_br2001" <junior_br2001@yahoo.com.br>
TO: ciencialist@yahoogrupos.com.br
DATE: 21/03/2005 20:07


Oracúlo , o problemas desses grupinhos céticos brasileiros é o
fanatismo bem claro e a falta de maturidade. Veja bem, uma CSICOP da
vida aceita deístas, um dos proprios fundadores dela o matemático
Martin Garder, um dos escritos pioneiros de livros céticos e
inspirador de Sagan é um deísta filosófico. Existem vários deístas na
ortganização e provavelmente deve ter alguns teístas não-religiosos
tb, e é lógico agnosticos e ateistas, além de humanistas e
secularistas que nao querem se denominarem com "ismos" limitativos
E além do mais isso de crer é muito relativo, um cético pode exercer
seu poder de crença quando acha que está bem certo e convicto no que
diz, e quando atendem ao imperativo de se fixarem nos seus
pressupostos ou possíveis dogmas científicos. Não pode existir coisa
mais enganosa que essa, se auto-iludir. Ele pode nao acreditar em
deus, não acreditar em mitos, mas tem crenças comportamentais e
egocentricas tb. Enfim nao se pode ser cético 100% pra tudo.
O Padre Quevedo pode ser um cético que acredita em dogmas catolicos;
e tem os céticos puristas que acreditam que eles mesmos não tem
defeitos comportamentais ou crenças baseadas em seu ego ou na dita
ciência(não se pode esquecer dos cientifistas), mas que no fundo está
cheio delas. Por isso nao existe 100% de ceticismo. Voce as vezes tem
de crer em algo para garantir sua sobrevivencia como pessoa. Voce
mesmo e ninguém está isento de crenças, voce pode questionar crenças
absurdas, mas no fundo voce ainda mantém certas crenças
Portanto esse conceito de ser cético e crente é muito relativo.

O problema todo é que o fanatismo é óbvio nestes grupinhos céticos,
mas é claro que tem muita gente boa e consciente do seu lado cético e
de buscador da ciencia. Quando me refiro a esses pseudoceticos falo
destes que aaúnica coisa que tem em mente é duvidar e cair num
nilismo insensato

JR

--- Em ciencialist@yahoogrupos.com.br, "Oraculo" <oraculo@a...>
escreveu
> Olá Pubmed
>
> Pubmed: Aliás, os céticos que duvidam dos céticos sao tb céticos"
>
> Se eles duvidam como métdo de conhecimento, sim, são céticos
também. Se duvidam por "acreditar" em outra coisa, que a alegação
cética inicial contradiz, não são, são apenas crentes exercendo seu
legítimo direito de crer no que desejarem.
>
> É uma diferença importante.
>
> Homero
>





SUBJECT: Santo sudário (p/ Sérgio Taborda e demais)
FROM: "Emiliano Chemello - Yahoo Grupos" <chemelloe@yahoo.com.br>
TO: <ciencialist@yahoogrupos.com.br>
DATE: 21/03/2005 20:09

Sérgio,

Retomando a discussão.

[Sérgio]
O problema, ao contrário do que se pensa, não a datação por caborno, mas a
origem do carbono datado.O problema é que o fogo e outros factores
contaminam o tecido ao longo do anos com carbono "mais recente" do que
aquele presente na época verdadeira do tecido.. Portanto, os carbonos
presentes no tecido têm "várias datas" e isso contamina o resultado final.

[Emiliano]
Sérgio, ainda não consegui 'visualizar' *como* ocorre a contaminação. Por
acaso haveria uma 'substituição' dos carbonos da amostra por 'outros
carbonos' durante a queima do tecido? Este carbono-14 sairia na forma de CO2
durante a queima, alterando os resultados caso não houvesse a queima? Creio
que estes questionamentos são importantes para considerarmos ou não a
questão da contaminação.

Emiliano Chemello

----- Original Message -----
From: Sergio M. M. Taborda
To: ciencialist@yahoogrupos.com.br
Sent: Sunday, March 20, 2005 8:49 AM
Subject: Re: [ciencialist] Re: Santo sudário e a vanilina(correção)


Emiliano Chemello - Yahoo Grupos wrote:

> Caros amigos,
>
> Peguei esta 'super' discussão pelo caminho. Mas percebi que há algumas
> referência ao fato de manto ter pego fogo e a técnica de datação por
> carbono
> 14 ser 'prejudicada' ou 'inviabilizada'. Pelos meus parcos conhecimentos,
> digo que mesmo que o manto tenha queimado, os carbonos radioativos NÃO
> irão
> perder sua atividade radioativa, visto que o fenômeno se dá a nível
> nuclear.
>
Pois não irão. Mas o problema não é esse. O problema, ao contrário do
que se pensa, não a datação por caborno, mas a origem do carbono datado.
O problema é que o fogo e outros factores contaminam o tecido ao longo
do anos com carbono "mais recente" do que aquele presente na época
verdadeira do tecido..
Portanto, os carbonos presentes no tecido têm "várias datas" e isso
contamina o resultado final.
Mas isso não é descontado no exame. Nem isso nem outras coisas. O exame
de 88 foi demasiado simplista e portanto seus resultados
são inconclusivos.
O exame do carbono 14 não é milagroso. Existem outros factores, outros
exames, quimicos, biologicos e historicos que têm que se levados em
consideração para a datação do objecto.
O pior cientista é aquele que só vê os dados que lhe interessam.

Sérgio Taborda




SUBJECT: Editor de calculos e formulas
FROM: "junior_br2001" <junior_br2001@yahoo.com.br>
TO: ciencialist@yahoogrupos.com.br
DATE: 21/03/2005 20:27


Pessoal, gostaria de saber onde encontro um editor para calculos e
fórmulas matemáticas e gráficos para se utilizados nos meus estudos
de física e pra tirarem minhas dúvidas nos grupos de discussão, já
que o modo texto do grupo nao suporta as formulas e calculos

JR





SUBJECT: Re: [ciencialist] Re: O manto de Turin
FROM: "Sergio M. M. Taborda" <sergiotaborda@terra.com.br>
TO: ciencialist@yahoogrupos.com.br
DATE: 21/03/2005 20:34

rmtakata wrote:

>
> --- Em ciencialist@yahoogrupos.com.br, "Sergio M. M. Taborda"
> > O teste do caborno 14 não é confiável:
> > http://www.shroud.com/meacham.htm
> > http://www.historicaljesusquest.com/
>
> Claaaaro. A teoria da evolucao tb eh uma farsa:
>
> http://www.icr.org/
> http://www.creationscience.com/

Exactamente onde, nesse sites, existem trabalhos cientificos que
demonstrem que a evolução é uma farsa ?
E já agora, onde se pode ler que a datatação por C14 é infalivel ?

Ségio Taborda

P.S. Vc conseguiu ler todos os sites em 1h30 !?! fantástico!


SUBJECT: Datação por Carbono 14 (curiosidade)
FROM: "Emiliano Chemello - Yahoo Grupos" <chemelloe@yahoo.com.br>
TO: <ciencialist@yahoogrupos.com.br>
DATE: 21/03/2005 20:36

Curiosidade:

A radioatividade específica da amostra, em qualquer caso, é medida a um
padrão mundial de referência, em geral o "ácido oxálico NBS" fornecido pelo
"National Bureau of Standards" (Estados Unidos), cuja radioatividade é
devidamente corrigida a do CO2 atmosférico de 1950. Isto porque, a partir de
1950, houve importante aumento da radioatividade específica de CO2 do ar
devido à produção de carbono 14 artificial, ligado aos ensaios
termonucleares atmosféricos.

Geologia do Quartenário e Mudanças Ambientais - Kenitiro Suguio - Paulo's
Editora, p. 159


Emiliano Chemello
emiliano@quimica.net
http://www.quimica.net/emiliano
http://www.ucs.br/ccet/defq/naeq
[ MSN ] chemelloe@hotmail.com
[ ICQ ] 145060604

"Rien ne se perd, rien ne se crée, tout se transforme"
Lavoisier, químico francês (1743-1794)




SUBJECT: Re: O manto de Turin
FROM: "junior_br2001" <junior_br2001@yahoo.com.br>
TO: ciencialist@yahoogrupos.com.br
DATE: 21/03/2005 20:38


Pelo que eu li(ou o pouco que eu li com meu deficiente ingles) trata-
se sites de cientistas e historiadores

Já os sites que o Takata nao tem nada haver , são sites de
criacionistas.

JR


--- Em ciencialist@yahoogrupos.com.br, "Sergio M. M. Taborda"
<sergiotaborda@t...> escreveu
> rmtakata wrote:
>
> >
> > --- Em ciencialist@yahoogrupos.com.br, "Sergio M. M. Taborda"
> > > O teste do caborno 14 não é confiável:
> > > http://www.shroud.com/meacham.htm
> > > http://www.historicaljesusquest.com/
> >
> > Claaaaro. A teoria da evolucao tb eh uma farsa:
> >
> > http://www.icr.org/
> > http://www.creationscience.com/
>
> Exactamente onde, nesse sites, existem trabalhos cientificos que
> demonstrem que a evolução é uma farsa ?
> E já agora, onde se pode ler que a datatação por C14 é infalivel ?
>
> Ségio Taborda
>
> P.S. Vc conseguiu ler todos os sites em 1h30 !?! fantástico!





SUBJECT: Re: [ciencialist] Re: Santo Sudario
FROM: "Silvio" <scordeiro@terra.com.br>
TO: <ciencialist@yahoogrupos.com.br>
DATE: 21/03/2005 21:06

Jr.:
não entrei nessa confusão porque perder tempo com esse trapo é trabalho de
Ocno: não leva a nada e não se convence quem "crê" de alguma coisa que
conteste os postulados das igrejas.
São irracionais.
Há, entre os católicos, pedaços de ossos, ampolas com sangue e outros restos
que são "sagrados".
Mas isso não é nada: os budistas vendiam o cocô do dalai - lama como coisa
sagrada. É preciso, sempre, dar além do pão, um pouco de circo para o
respeitável público....

Sempre pergunto, ninguém responde, se o senhor J. Cristo tinha o cromosomo
"Y"....

Minha bênção apostólica,

silvius, cardinal.do santo ofício.



Alberto,

Tem um ditado que criei agora que diz: " A dúvida é uma excelente
arma da ciência, mas, sem *acreditar* a ciencia nao chega a lugar
algum"

Jr(filósofo de botequim)


--- Em ciencialist@yahoogrupos.com.br, "Alberto Mesquita Filho"
<albmesq@u...> escreveu
> ----- Original Message -----
> From: "Eduardo Gueron"
> Sent: Sunday, March 20, 2005 1:57 AM
> Subject: Re: [ciencialist] Santo Sudario
>
> > Gostaria de observar, ainda que, nessa discussão, está clara a
> > religiosidade de cada um. Posso estar enganado, mas chutaria que
todos os
> > que defendem a autenticidade do Sudário são cristãos (e acusam os
céticos
> > de dogmáticos). É justo, portanto, que cada um assuma a sua
religião para
> > que a briga fique mais honesta. Lembro que desde o início, apesar
de estar
> > meio à margem da briga, disse que sou um cético (agnóstico e
ateu, depende
> > do contexto e do meu humor).
>
> Caro Eduardo
>
> Sinceramente não sei dizer se a coisa seria tão simples assim e por
vários
> aspectos. Em primeiro lugar, diria que esse "todos" refere-se a uma
> amostragem bastante pobre. Ou muito me engano, ou poderia contar 2
de cada
> lado. Por um lado, vejo o Júnior e o Taborda, cada um com idéias
totalmente
> diversas e a abordarem prismas totalmente diversos e nem sei até
que ponto
> eles entrariam em acordo sobre o que escreveram. De outro, o Homero
e o
> Takata, esses sim a comutarem freqüências próximas, para usar uma
linguagem
> metafórica, e a traduzirem o que aprenderam naquele catecismo que
pulula
> pela Internet, aquele mesmo que faz considerações sobre os "ad
hominem",
> "onus da prova" e outras tacanhices nem sei repetir. E digo
tacanhice não
> por que seja contrário aos conceitos que estão embutidos nessas
palavras,
> mas para me referir aos contextos em que elas têm sido utilizadas.
>
> Eu não entrei na briga a não ser para tirar o sarro, ora do Homero,
ora do
> Takata, e mais no sentido de demonstrar que ser cético, pelo menos
em
> ciência, não significa "levantar uma bandeira", mas principalmente
adotar
> uma postura que nos permita levar à produção de novos
conhecimentos. No
> mais, que se deixe o ceticismo aos filófosos pois eles dão de 1000
a zero em
> qualquer desses "céticos de carteirinha" que outra coisa não fazem
a não ser
> iludir os jovens e denegrir um ceticismo sadio.
>
> Se lhe interessa saber, participo de um grupo sério de discussão de
> ceticismo entre filósofos, e neste grupo o único com pendores
voltados
> exclusivamente à ciência sou eu. Via de regra durante essas reuniões
> permaneço calado e assumo a minha insignificância associada à
vontade de
> aprender. Após as reuniões costumamos ir a uma pizzaria e aí sim
animo-me a
> dizer alguma coisa a esses filósosos (alguns expoentes nacionais e
com
> currículo Lates a ser invejado pelo mais culto dentre os
freqüentadores da
> Ciencialist, e que certamente não sou eu) e chego mesmo a brincar
com eles,
> referindo-me a essa nossa segunda reunião, regada a chope e coca-
cola
> (alguns não bebem), como destinada a que possamos praticar uma
filosofia de
> botequim.
>
> Não sei até que ponto a religiosidade entraria em jogo, ainda que
esses
> falsos céticos insistam em misturar as coisas. De qualquer forma, e
se você
> insistir em saber a quantas anda o meu agnosticismo, pois é assim
que me
> considero, sugiro que acompanhe um debate que travei numa lista já
extinta,
> mas que pode ser lido, sob o título "Ciência, materialismo e
> espiritualismo", a partir de
> http://ecientificocultural.com/ECC2/Dialogos/cmee.htm
>
> [ ]´s
> Alberto
> http://ecientificocultural.com/indice.htm
> Mas indiferentemente a tudo isso, o neutrino tem massa, o elétron
não é
> uma carga elétrica coulombiana e a Terra se move. E a história se
repetirá.





##### ##### #####

Para saber mais visite
http://www.ciencialist.hpg.ig.com.br


##### ##### ##### #####
Links do Yahoo! Grupos











SUBJECT: Re: [ciencialist] Gravitons
FROM: "Sergio M. M. Taborda" <sergiotaborda@terra.com.br>
TO: ciencialist@yahoogrupos.com.br
DATE: 21/03/2005 21:55

Eu não sei qual é a relevancia desde texto ,mas...

Parece que toda a querela é à roda de uma pequena coisa. insignificante,
que nem sequer importa para a teoria.
Cito
"Para Einstein, a gravidade causava uma deformação no espaço-tempo
contínuo e com esta idéia,
ele desenvolveu uma álgebra muito complexa que a descreve apenas
geometricamente."

Afinal parece que nem sequer sabem intrepretar esta frase.
De facto a frase está certa embora o portugues podesse ser melhor.
De facto a gravidade causa a deformação do espaço-tempo. Ha quem diga
que a gravidade é essa mesma deformação.
Mas isso não é rigorosamente correcto, é uma abuso de linguagem, já que
a deformação do espaço-tempo é o campo gravitico e não a gravidade (a
força gravitica)

A massa causa gravidade que causa a deformação.
É preciso lembrar que o espaço tempo não deformado é aquele da TRR, nem
a gravidade, nem a massa criam o espaço-tempo. Ele pre-existe. É um
axioma da fisica,

De facto ele desenvolveu uma algebra complexa. Alguem disse que
matemática foi criada antes e ele apenas a utilizou.
É verdade que alguem criou a matemática antes, mas foi Eisntein que
entendeu que o seu modelo fisico era dscrito por aquele modelo
matemático, e portanto ele desenvolveur uma algebra: A equação de
Eisntein.
O autor poderia ser mais explicito. E de facto isto demonstra uma certa
falta de conhecimento ou falha no discurso do autor, mas não é errado.
Muito menos é motivo para desacreditar a teoria.

É tb verdade que a Teoria da Relatividade é uma descrição puramente
geométrica (é como o teorema de pitagoras). A teoria não explica pq é
assim.
Comparado a eq de Eisntein é para a fisica de hoje o que F=ma foi para o
tempo pos Newton. Tb F=ma é um postulado que não explica pq.
E o F=ma é como o U = RI , todas elas seguem o mesmo principio básico.
A/ B = k
Então, ao menos, o autor consegue entender que ha um vácuo teorico no
assunto em questão.

Vcs não comentaram sobre a parte quantica, que a afinal é o que mais
interessa aqui.
O promenor relativistico mais importante é que onde a TRG é válida a TRR
não é , e na realidade o que é usadao é TRR. (eq 12)
A teoria baseia-se no modelo de Borh que já um pouco velho, mas válido e
serve para um primeiro estudo e apresentação da teoria .

O autor parece tb não ter conhecimento de algarismos cientificos pois
apresenta demasiados algarismos nos resultados das contas.

Se algo está errado no documento é algo muito mais fundo que meros
problemas de portugues ou de conceito. Os conceitos parecem estar
entendidos, mas o que não parece estar entendio são as fronteira de
aplicabilidade das teorias. Mas quebrar essas fronteiras é exatamente a
proposta.

Um verdadeiro cientista não se recusa a ler documentos só pq a
introdução não tem um portugues perfeito. Se isso fosse assim, nenhum
paper passaria nos reviews, nenhum aluno conseguiria entregar um
relatório prático. Não se pode criticar um trabalho sem ler o que é.

Portanto leiam , digam menos sandices, e depois veremos qual é o
problema FISICO que o texto cria.
Básciamente o texto defende que a gavidade é um efeito de observação que
apenas diz respeito à dimensão temporal. É como se existisse gravidade,
pq os tempos proprios dos objetos no universos estão desfazados. Isso é
interessante.
Lembrando que o Principio da Equivalencia iguala o campo gravitico a um
campo de acelerações, o que significa que , fisicamente, é apenas um
campo cinemático.
Portanto, em principio, não ha nada contra a teoria. Agora, falta
analizar o texto com olhos de ver e saber se consegue resolver alguma
coisa.

Mas qual é mesmo o objectivo de analizar este texto ?

Sérgio Taborda


SUBJECT: Re: [ciencialist] manto de turim e c�ticos
FROM: Eduardo Gueron <edgueron@yahoo.com>
TO: ciencialist@yahoogrupos.com.br
DATE: 22/03/2005 01:52

Oi,

Creio ser consenso que a discussco perdeu o sentido.
Nco seria difmcil de prever esse final em uma debate
desta natureza.

Na realidade, nco ha interesse da proprietaria do
objeto a ser estudado em que o objeto seja
minunciosamente analisado e, portanto, sempre faltarco
argumentos aos dois lados. Espero que todos aqui
concordem que, em situagues como essa, o ideal seriam
repetidos e testes e exaustivos debates sobre os
resultados. Debates daqueles que tiveram acesso `s
amostras e que entendessem realmente de datagco -
coisa que ninguim aqui realmente entende embora alguns
tenham clara nogco. Independentemente disso, mantenho
minha opinico sobre o assunto que, no fundo, ss pode
ser uma opinico ja que a grande pinimba i se as
amostras utilizadas pelo grupo de 88 sco ou nco
confiaveis, isso esta na introdugco do artigo do
Rogers. (Reitero que possuo as versues em pdf dos 2
artigos em disputa, quem quiser posso mandar uma cspia
por email).

Sobre os citicos: Se ha citicos demstas ou Padres
Quevedos, pouco importa, certamente ha citicos
assassinos, maus carater e burros (me enquadro nessa
zltima categoria ao lado do Takata). Mas o ceticismo
rasteiro (sem maiores rigores filossficos) pode ser
simplorio, tem como ideal que a pessoa que propue os
resultados heterodoxos se esmere em prova-los. E o
caso do Sudario i, certamente, um dos exemplos mais
simples e ao mesmo tempo nco satisfatsrio. Ha
situagues mais complicadas como na medicina em que o
mitodo cientmfico passa por restrigues iticas, nesse
caso, as alternativas se tornam mais complexas.

ISto posto, acho que nco tenho nada a acrescentar, ati
porque, ha pessoas mais cuidadosas e pacientes do que
eu nessa lista e particularmente bem articuladas como
o caso do Homero que defende pontos de vistas
semelhantes aos meus de maneira bem mais brilhante.

Mudando de assunto: Na configuragco que uso, recebo
resumos da lista ao invis de mensagens isoladas, nisso
acho que me perdi na discussco sobre curvatura e Rel
Geral. Alguim poderia me dizer onde comegou?

[]'s

Eduardo.



__________________________________
Do you Yahoo!?
Read only the mail you want - Yahoo! Mail SpamGuard.
http://promotions.yahoo.com/new_mail


SUBJECT: Re: manto de turim e céticos
FROM: "junior_br2001" <junior_br2001@yahoo.com.br>
TO: ciencialist@yahoogrupos.com.br
DATE: 22/03/2005 02:03


Comentários abaixo:

--- Em ciencialist@yahoogrupos.com.br, Eduardo Gueron <edgueron@y...>
escreveu
> Oi,
>
> Creio ser consenso que a discussão perdeu o sentido.
> Não seria difícil de prever esse final em uma debate
> desta natureza.


JR: Na verdade o lado cético defende um fixismo e isso é muito ruim
no debate e em ciencia


>
> Na realidade, não há interesse da proprietária do
> objeto a ser estudado em que o objeto seja
> minunciosamente analisado e, portanto, sempre faltarão
> argumentos aos dois lados.

JR: Isso é uma verdade. O que falta sao mais experimentos

Espero que todos aqui
> concordem que, em situações como essa, o ideal seriam
> repetidos e testes e exaustivos debates sobre os
> resultados. Debates daqueles que tiveram acesso às
> amostras e que entendessem realmente de datação -
> coisa que ninguém aqui realmente entende embora alguns
> tenham clara noção.

JR: Bem a gente tá se baseando em especialistas, por isso citei os
estudos. Inclusive o Taborda mandou os ultimos links interessantes,
se voce tiver calma para ler.

Independentemente disso, mantenho
> minha opinião sobre o assunto que, no fundo, só pode
> ser uma opinião já que a grande pinimba é se as
> amostras utilizadas pelo grupo de 88 são ou não
> confiáveis, isso está na introdução do artigo do
> Rogers. (Reitero que possuo as versões em pdf dos 2
> artigos em disputa, quem quiser posso mandar uma cópia
> por email).

JR: Certo

>
> Sobre os céticos: Se há céticos deístas ou Padres
> Quevedos, pouco importa, certamente há céticos
> assassinos, maus caráter e burros (me enquadro nessa
> última categoria ao lado do Takata). Mas o ceticismo
> rasteiro (sem maiores rigores filosóficos) pode ser
> simplorio, tem como ideal que a pessoa que propõe os
> resultados heterodoxos se esmere em prová-los.


JR: A ciencia é mais interessante que o ceticismo organizado.

E o
> caso do Sudário é, certamente, um dos exemplos mais
> simples e ao mesmo tempo não satisfatório. Há
> situações mais complicadas como na medicina em que o
> método científico passa por restrições éticas, nesse
> caso, as alternativas se tornam mais complexas.

JR: Nao entendi direito, mas em todo caso , fica assim mesmo.

>
> ISto posto, acho que não tenho nada a acrescentar, até
> porque, há pessoas mais cuidadosas e pacientes do que
> eu nessa lista e particularmente bem articuladas como
> o caso do Homero que defende pontos de vistas
> semelhantes aos meus de maneira bem mais brilhante.

JR: Isso é um ponto de vista seu, se ele defende um lado cético meio
equivocado, claro que será sempre pessoas afins pra acompanha-lo
nesta empreitada. Embora outros simpatize com outras idéias. Se bem
que verborragias não é sinal de inteligencia, tampouco de saber.







SUBJECT: Re: [ciencialist] manto de turim e céticos
FROM: "Alberto Mesquita Filho" <albmesq@uol.com.br>
TO: <ciencialist@yahoogrupos.com.br>
DATE: 22/03/2005 02:20

----- Original Message -----
From: "Eduardo Gueron"
Sent: Tuesday, March 22, 2005 1:52 AM
Subject: Re: [ciencialist] manto de turim e céticos

> Na configuração que uso, recebo resumos da lista ao invés de mensagens
> isoladas, nisso acho que me perdi na discussão sobre curvatura e Rel
> Geral. Alguém poderia me dizer onde começou?

Começou em 14 de março com o Léo. Abaixo, e para quem interessar, todas as
mensagens da thread (até o momento):

Léo: http://br.groups.yahoo.com/group/ciencialist/message/44888
Álvaro: http://br.groups.yahoo.com/group/ciencialist/message/44896
JVictor: http://br.groups.yahoo.com/group/ciencialist/message/45129
Júnior: http://br.groups.yahoo.com/group/ciencialist/message/45131
Daniel Moser: http://br.groups.yahoo.com/group/ciencialist/message/45236
PSavio: http://br.groups.yahoo.com/group/ciencialist/message/45238
PSavio: http://br.groups.yahoo.com/group/ciencialist/message/45246
Taborda: http://br.groups.yahoo.com/group/ciencialist/message/45247

[ ]´s
Alberto
http://ecientificocultural.com/indice.htm
Mas indiferentemente a tudo isso, o neutrino tem massa, o elétron não é
uma carga elétrica coulombiana e a Terra se move. E a história se repetirá.



SUBJECT: Fixismo - nova alegação insustentável..:-) (era manto de turim e céticos)
FROM: "Oraculo" <oraculo@atibaia.com.br>
TO: <ciencialist@yahoogrupos.com.br>
DATE: 22/03/2005 02:29

Olá Pubmed

Pubmed: JR: Na verdade o lado cético defende um fixismo e isso é muito ruim
no debate e em ciencia"

Não, não defende. Você continua a levantar alegações que não pode sustentar. Isso não é legítimo, nem ajuda o debate. Procure nas mensagens anteriores e tente encontrar uma mensagem, qualquer uma, que defenda o fixismo. Está distorcendo, me parece conscientemente, o debate, na falta de argumentos ou evidencias que sejam válidas para sustentar seu ponto de vista.

Em todas as mensagens, céticas, encontrará a posição exatamente contrária, de não-fixismo, de mudança, apenas coma condição do rigor e cuidado ao aceitar evidencias e conclusões. Nunca, em nenhuma mensagem, fixismo, até porque tudo o que a ciência tem de melhor é a mudança e a adaptação a novos conhecimentos e dados.

Alias, foi justametne você que criticou essa posição cética, de "mudar conforme as evidencias mudam", alegando que era um exemplo da falta de firmeza dos céticos..:-) Como pode agora acusar os céticos exatamente do contrário???

Homero



----- Original Message -----
From: junior_br2001
To: ciencialist@yahoogrupos.com.br
Sent: Tuesday, March 22, 2005 2:03 AM
Subject: [ciencialist] Re: manto de turim e céticos



Comentários abaixo:

--- Em ciencialist@yahoogrupos.com.br, Eduardo Gueron <edgueron@y...>
escreveu
> Oi,
>
> Creio ser consenso que a discussão perdeu o sentido.
> Não seria difícil de prever esse final em uma debate
> desta natureza.


JR: Na verdade o lado cético defende um fixismo e isso é muito ruim
no debate e em ciencia


>
> Na realidade, não há interesse da proprietária do
> objeto a ser estudado em que o objeto seja
> minunciosamente analisado e, portanto, sempre faltarão
> argumentos aos dois lados.

JR: Isso é uma verdade. O que falta sao mais experimentos

Espero que todos aqui
> concordem que, em situações como essa, o ideal seriam
> repetidos e testes e exaustivos debates sobre os
> resultados. Debates daqueles que tiveram acesso às
> amostras e que entendessem realmente de datação -
> coisa que ninguém aqui realmente entende embora alguns
> tenham clara noção.

JR: Bem a gente tá se baseando em especialistas, por isso citei os
estudos. Inclusive o Taborda mandou os ultimos links interessantes,
se voce tiver calma para ler.

Independentemente disso, mantenho
> minha opinião sobre o assunto que, no fundo, só pode
> ser uma opinião já que a grande pinimba é se as
> amostras utilizadas pelo grupo de 88 são ou não
> confiáveis, isso está na introdução do artigo do
> Rogers. (Reitero que possuo as versões em pdf dos 2
> artigos em disputa, quem quiser posso mandar uma cópia
> por email).

JR: Certo

>
> Sobre os céticos: Se há céticos deístas ou Padres
> Quevedos, pouco importa, certamente há céticos
> assassinos, maus caráter e burros (me enquadro nessa
> última categoria ao lado do Takata). Mas o ceticismo
> rasteiro (sem maiores rigores filosóficos) pode ser
> simplorio, tem como ideal que a pessoa que propõe os
> resultados heterodoxos se esmere em prová-los.


JR: A ciencia é mais interessante que o ceticismo organizado.

E o
> caso do Sudário é, certamente, um dos exemplos mais
> simples e ao mesmo tempo não satisfatório. Há
> situações mais complicadas como na medicina em que o
> método científico passa por restrições éticas, nesse
> caso, as alternativas se tornam mais complexas.

JR: Nao entendi direito, mas em todo caso , fica assim mesmo.

>
> ISto posto, acho que não tenho nada a acrescentar, até
> porque, há pessoas mais cuidadosas e pacientes do que
> eu nessa lista e particularmente bem articuladas como
> o caso do Homero que defende pontos de vistas
> semelhantes aos meus de maneira bem mais brilhante.

JR: Isso é um ponto de vista seu, se ele defende um lado cético meio
equivocado, claro que será sempre pessoas afins pra acompanha-lo
nesta empreitada. Embora outros simpatize com outras idéias. Se bem
que verborragias não é sinal de inteligencia, tampouco de saber.







##### ##### #####

Para saber mais visite
http://www.ciencialist.hpg.ig.com.br


##### ##### ##### #####


Yahoo! Grupos, um serviço oferecido por:







------------------------------------------------------------------------------
Links do Yahoo! Grupos

a.. Para visitar o site do seu grupo na web, acesse:
http://br.groups.yahoo.com/group/ciencialist/

b.. Para sair deste grupo, envie um e-mail para:
ciencialist-unsubscribe@yahoogrupos.com.br

c.. O uso que você faz do Yahoo! Grupos está sujeito aos Termos do Serviço do Yahoo!.



[As partes desta mensagem que não continham texto foram removidas]



SUBJECT: Re: [ciencialist] Re: ninguem é inocente.
FROM: TARCISIO BORGES <tbs97@fisica.ufpr.br>
TO: ciencialist@yahoogrupos.com.br
DATE: 22/03/2005 08:01

Mas se estas fibras foram queimadas, como ele sabe que elas possuíam
vanilina?

[]s
TARCISIO BORGES
tbs97@fisica.ufpr.br

On Mon, 21 Mar 2005, junior_br2001 wrote:
> Não a peça queimada, mas as fibras do remendo que foi testado pelo
> carbono-14 possuia vanilina, e não vinila
>
> JR


SUBJECT: Re: [ciencialist] Re: ninguem é inocente.
FROM: TARCISIO BORGES <tbs97@fisica.ufpr.br>
TO: ciencialist@yahoogrupos.com.br
DATE: 22/03/2005 08:18

On Mon, 21 Mar 2005, junior_br2001 wrote:
> JR: Voce tem certeza disso? Eu li 1340 anos, inclusive li isso no
> site que voce me apontou, dos céticos( a nao ser que o valor de 600
> seja o minimo do primeiro estudo), . Se o Rogers estipulou um
> mínimo , ele coincide *mais ou menos* com esse valor de 1340. O que
> pode invalidar é margem especulada em 3000 anos

Você tem razão. Fui ler novamente:

"O Dr. Rogers estimou a data real do sudário entre cerca de 1000
AEC e 1700 EC."

Neste caso a segunda datação não invalida a primeira, ou vice versa. A
minha primeira interpretação do assunto é que a peça teria de 1000 a 3000
anos e portanto incompatível com a primeira datação.


> JR: As técnicas do carbono-14 hoje em dia estão muito melhores que as
> de 88. Além do mais voce nao prestou atenção ao que foi dito sobre
> contaminação do tecido com carbono-14 recente, revestimento
> bioplástico e teste relizado nos remendos medievais e não no tecido
> propriamente dito.

Como você diz, eu não vi nada disso sendo divulgado.


> JR: Foi uma metologia científica, e não um teste homeopático, ele
> encontrou uma substancia química no remendo, e no resto do tecido a
> vanilina já não estava presente. Isso é uma evidencia, não um estudo
> mal-feito.

Mas que remendo? Aquele que foi queimado no teste de C14?


> O Stephen William Hawking só admitiu que estava errado, quando sua
> teoria já tinha sido refutada por outro físico...se ele não admitisse
> pegaria mal. Muitos físicos não gostam dele, nao pela sua
> deficiencia, mas por que ele é um especulador sonhador

Todas as teorias são refutadas por outros cientistas e precisam ser. A
questão é convencer o autor da teoria que ela está errada.

O mérito de Hawking está em aceitar o erro.

Quando os bispos e padres irão aceitar que a relíquia é uma fraude?

Não estou muito confiante de que aconteça.

[]s
TARCISIO BORGES
tbs97@fisica.ufpr.br


SUBJECT: Fw: Problema basico de volumes
FROM: "Luiz Ferraz Netto" <leobarretos@uol.com.br>
TO: "ciencialist" <ciencialist@yahoogrupos.com.br>
DATE: 22/03/2005 09:06

Quem quer fazer a primeira redação da resposta a essa pergunta?

[]'
===========================
Luiz Ferraz Netto [Léo]
leobarretos@uol.com.br
http://www.feiradeciencias.com.br
===========================
-----Mensagem Original-----
De: nunofigueiredo
Para: leobarretos
Enviada em: terça-feira, 22 de março de 2005 02:00
Assunto: Problema basico de volumes


Prezado professor:
Achei o seu site muitop interessante. Assim, estoui a lhe escrever um função de uma duvida.
O meu cunhado tem uma fabrica de cosmeticos e, surgiu uma discussão entre nós: Ele insiste, que em determiado vasilhame que tem a capacidade para 4 litros, recebe 4 litros de água, mas uma quantidade diferente (menor) de shampoo.
Por mais que eu tentasse explicar que o litro é uima unidade universal(não varia com o tipo de produto) por se tratar de uma medida de volume, ele insiste no erro.
Eu expliquei-lhe que o peso sim, varia conforme o produto por causa da densidade, mas o volume jamais muda. Ele aceitou que o peso varia, mas o volume também.
Eu argumentei entre outras, que não poderiamos ter duas variaveis para o mesmo produto, pois tornar-se-ia impossivel calcular uma em função da outra... Temtei explicar que um litro de agua ou um litro de chumbo derretido sao um litro. Apenas um pesa aproximadamente um Kg e o outro vários Kg.
Ele insiste que está certo e que já foi autoado pelo inmetro mais de uma vês por isso. Eu, irritado disse que não era possível e, que se o inmetro multou porque um litro não era um litro, o inmetro estava errado. É obvio que ele foi autoado por diferenças no calculo do peso liquido e nao no volume em l ou ml.
Bem, resumindo, fui incapaz de lhe explicar isso, apesar da minha formação superior em física (incompleta).
Achei qe o tema é interessante o suficiente para o expor ao senhor. Se não for pedir demais, gostaria de uma opinião para a apresentar.
Talvez o problema seja apresentar de forma compreensivel a definiç]ao de litro. Sem se levar em consideração os gases por serem fluidos compressiveis, e claro.
Agradeço de antemão a sua resposta,
Atenciosamente,

Jose Nuno Figueiredo
nunofigueiredo@terra.com.br


--------------------------------------------------------------------------------


Internal Virus Database is out-of-date.
Checked by AVG Anti-Virus.
Version: 7.0.308 / Virus Database: 266.7.3 - Release Date: 15/03/2005


[As partes desta mensagem que não continham texto foram removidas]



SUBJECT: Re: [ciencialist] Fw: Problema basico de volumes
FROM: "E m i l i a n o C h e m e l l o" <chemelloe@yahoo.com.br>
TO: <ciencialist@yahoogrupos.com.br>
DATE: 22/03/2005 09:28

. 3ª CGPM, 1901, (CR, 38-39): DECLARAÇÃO RELATIVA À DEFINIÇÃO DO LITRO*
A Conferência declara:
1º) A unidade de volume, para determinações de alta precisão, é o volume
ocupado pela massa de 1 quilograma de água pura em sua densidade máxima e
sob pressão atmosférica normal; este volume é chamado de "litro".

* Definição revogada pela 12ª CGPM (1964, Resolução 6).

---
. 12ª CGPM, 1964, RESOLUÇÃO 6 (CR, 93): LITRO
A Décima Segunda Conferência Geral de Pesos e Medidas considerando a
Resolução 13 adotada pela Décima Primeira Conferência Geral, em 1960, e a
Recomendação adotada pelo Comitê Internacional de Pesos e Medidas na sua
sessão de 1961,

1º) abole a definição do litro dada em 1901 pela Terceira Conferência Geral
de Pesos e Medidas;

2º) declara que a palavra litro pode ser utilizada como nome especial
aplicado ao decímetro cúbico;

3º) recomenda que o nome litro não seja utilizado para exprimir resultados
de medidas de volume de alta precisão.

Fonte: Inmetro.

Eu faria uma análise matemática, envolvendo os conceitos de densidade,
massa e volume, mostrando, matematicamente que:

d = m/v

d = densidade
m = massa
v = volume

[ ] 's do Emiliano Chemello
emiliano@quimica.net
http://www.quimica.net/emiliano
http://www.ucs.br/ccet/defq/naeq
[ MSN ] chemelloe@hotmail.com
[ ICQ ] 145060604

" Rien ne se perd, rien ne se crée,
tout se transforme."

Antoine Laurent de Lavoisier (químico francês, 1743 - 1794)

----- Original Message -----
From: Luiz Ferraz Netto
To: ciencialist
Sent: Tuesday, March 22, 2005 9:06 AM
Subject: [ciencialist] Fw: Problema basico de volumes


Quem quer fazer a primeira redação da resposta a essa pergunta?

[]'
===========================
Luiz Ferraz Netto [Léo]
leobarretos@uol.com.br
http://www.feiradeciencias.com.br
===========================
-----Mensagem Original-----
De: nunofigueiredo
Para: leobarretos
Enviada em: terça-feira, 22 de março de 2005 02:00
Assunto: Problema basico de volumes


Prezado professor:
Achei o seu site muitop interessante. Assim, estoui a lhe escrever um função
de uma duvida.
O meu cunhado tem uma fabrica de cosmeticos e, surgiu uma discussão entre
nós: Ele insiste, que em determiado vasilhame que tem a capacidade para 4
litros, recebe 4 litros de água, mas uma quantidade diferente (menor) de
shampoo.
Por mais que eu tentasse explicar que o litro é uima unidade universal(não
varia com o tipo de produto) por se tratar de uma medida de volume, ele
insiste no erro.
Eu expliquei-lhe que o peso sim, varia conforme o produto por causa da
densidade, mas o volume jamais muda. Ele aceitou que o peso varia, mas o
volume também.
Eu argumentei entre outras, que não poderiamos ter duas variaveis para o
mesmo produto, pois tornar-se-ia impossivel calcular uma em função da
outra... Temtei explicar que um litro de agua ou um litro de chumbo
derretido sao um litro. Apenas um pesa aproximadamente um Kg e o outro
vários Kg.
Ele insiste que está certo e que já foi autoado pelo inmetro mais de uma vês
por isso. Eu, irritado disse que não era possível e, que se o inmetro multou
porque um litro não era um litro, o inmetro estava errado. É obvio que ele
foi autoado por diferenças no calculo do peso liquido e nao no volume em l
ou ml.
Bem, resumindo, fui incapaz de lhe explicar isso, apesar da minha formação
superior em física (incompleta).
Achei qe o tema é interessante o suficiente para o expor ao senhor. Se não
for pedir demais, gostaria de uma opinião para a apresentar.
Talvez o problema seja apresentar de forma compreensivel a definiç]ao de
litro. Sem se levar em consideração os gases por serem fluidos
compressiveis, e claro.
Agradeço de antemão a sua resposta,
Atenciosamente,

Jose Nuno Figueiredo
nunofigueiredo@terra.com.br


----------------------------------------------------------------------------
----


Internal Virus Database is out-of-date.
Checked by AVG Anti-Virus.
Version: 7.0.308 / Virus Database: 266.7.3 - Release Date: 15/03/2005


[As partes desta mensagem que não continham texto foram removidas]



##### ##### #####

Para saber mais visite
http://www.ciencialist.hpg.ig.com.br


##### ##### ##### #####


Yahoo! Grupos, um serviço oferecido por:

São Paulo Rio de Janeiro Curitiba Porto Alegre Belo Horizonte Brasília





Links do Yahoo! Grupos

Para visitar o site do seu grupo na web, acesse:
http://br.groups.yahoo.com/group/ciencialist/

Para sair deste grupo, envie um e-mail para:
ciencialist-unsubscribe@yahoogrupos.com.br

O uso que você faz do Yahoo! Grupos está sujeito aos Termos do Serviço do
Yahoo!.




SUBJECT: Re: [ciencialist] Fw: Problema basico de volumes
FROM: Lourenço <listaslou@yahoo.com.br>
TO: <ciencialist@yahoogrupos.com.br>
DATE: 22/03/2005 11:08

Não poderia ser por causa de bolhas de ar, que depois seriam expulsas por
acomodação do shampoo?

Até.

Lourenço.

listaslou@yahoo.com.br
----- Original Message -----
From: "Luiz Ferraz Netto" <leobarretos@uol.com.br>
To: "ciencialist" <ciencialist@yahoogrupos.com.br>
Sent: Tuesday, March 22, 2005 9:06 AM
Subject: [ciencialist] Fw: Problema basico de volumes



Quem quer fazer a primeira redação da resposta a essa pergunta?

[]'
===========================
Luiz Ferraz Netto [Léo]
leobarretos@uol.com.br
http://www.feiradeciencias.com.br
===========================
-----Mensagem Original-----
De: nunofigueiredo
Para: leobarretos
Enviada em: terça-feira, 22 de março de 2005 02:00
Assunto: Problema basico de volumes


Prezado professor:
Achei o seu site muitop interessante. Assim, estoui a lhe escrever um função
de uma duvida.
O meu cunhado tem uma fabrica de cosmeticos e, surgiu uma discussão entre
nós: Ele insiste, que em determiado vasilhame que tem a capacidade para 4
litros, recebe 4 litros de água, mas uma quantidade diferente (menor) de
shampoo.
Por mais que eu tentasse explicar que o litro é uima unidade universal(não
varia com o tipo de produto) por se tratar de uma medida de volume, ele
insiste no erro.
Eu expliquei-lhe que o peso sim, varia conforme o produto por causa da
densidade, mas o volume jamais muda. Ele aceitou que o peso varia, mas o
volume também.
Eu argumentei entre outras, que não poderiamos ter duas variaveis para o
mesmo produto, pois tornar-se-ia impossivel calcular uma em função da
outra... Temtei explicar que um litro de agua ou um litro de chumbo
derretido sao um litro. Apenas um pesa aproximadamente um Kg e o outro
vários Kg.
Ele insiste que está certo e que já foi autoado pelo inmetro mais de uma vês
por isso. Eu, irritado disse que não era possível e, que se o inmetro multou
porque um litro não era um litro, o inmetro estava errado. É obvio que ele
foi autoado por diferenças no calculo do peso liquido e nao no volume em l
ou ml.
Bem, resumindo, fui incapaz de lhe explicar isso, apesar da minha formação
superior em física (incompleta).
Achei qe o tema é interessante o suficiente para o expor ao senhor. Se não
for pedir demais, gostaria de uma opinião para a apresentar.
Talvez o problema seja apresentar de forma compreensivel a definiç]ao de
litro. Sem se levar em consideração os gases por serem fluidos
compressiveis, e claro.
Agradeço de antemão a sua resposta,
Atenciosamente,

Jose Nuno Figueiredo
nunofigueiredo@terra.com.br


----------------------------------------------------------------------------
----


Internal Virus Database is out-of-date.
Checked by AVG Anti-Virus.
Version: 7.0.308 / Virus Database: 266.7.3 - Release Date: 15/03/2005


[As partes desta mensagem que não continham texto foram removidas]



##### ##### #####

Para saber mais visite
http://www.ciencialist.hpg.ig.com.br


##### ##### ##### #####
Links do Yahoo! Grupos











SUBJECT: Re: Fixismo - nova alegação insustentável..:-) (era manto de turim e céticos)
FROM: "junior_br2001" <junior_br2001@yahoo.com.br>
TO: ciencialist@yahoogrupos.com.br
DATE: 22/03/2005 13:49


Segue comentários:

--- Em ciencialist@yahoogrupos.com.br, "Oraculo" <oraculo@a...>
escreveu
> Olá Pubmed
>
> Pubmed: JR: Na verdade o lado cético defende um fixismo e isso é
muito ruim
> no debate e em ciencia"
>
> Não, não defende. Você continua a levantar alegações que não pode
sustentar. Isso não é legítimo, nem ajuda o debate. Procure nas
mensagens anteriores e tente encontrar uma mensagem, qualquer uma,
que defenda o fixismo. Está distorcendo, me parece conscientemente, o
debate, na falta de argumentos ou evidencias que sejam válidas para
sustentar seu ponto de vista.

JR: Isso é fácil levantar, basta ver a postura dogmática em que voces
defendem o primeiro teste. Se fossem céticos de verdade,
questionariam as condições do testes, as técnicas de 88, as possíveis
falhas que o teste possa ter com contaminações de carbono-14 recente
e revestimento bioplástico, as condições das fibras em que foram
realizados os testes(se foram realmente os remendos ou não), e
aceitariam as novas descobertas até que outraas descobertas refutem
com dados experimentais( e claaro que como contece no primeiro
estudo, o segundo tb pode ser questionado. Isso sim é agir
cientificamente, o resto é balela do ceticismo organizado

>
> Em todas as mensagens, céticas, encontrará a posição exatamente
contrária, de não-fixismo, de mudança, apenas coma condição do rigor
e cuidado ao aceitar evidencias e conclusões. Nunca, em nenhuma
mensagem, fixismo, até porque tudo o que a ciência tem de melhor é a
mudança e a adaptação a novos conhecimentos e dados.

JR: Exato. Mas voces não tomam esse cuidado. Saem questionando logo
de cara o segundo estudo sem dados experimentais e atuais que possam
contradize-lo. E se voces tivessem todo esse rigor em aceitar as
evidencias e conclusões aceitariam o segundo estudo, pois ele tb se
baseia em evidencias e dados reais. Se dão valor excessivo ao
primeiro estudo e que o segundo é balela, voce está fazendo
exatamente o oposto do diz em relação a mudnças e aceitação de novos
conhecimentos. para que haja mudanças todos os estudos tem de ser
questionados, tanto o primeiro como o segundo.

>
> Alias, foi justametne você que criticou essa posição cética,
de "mudar conforme as evidencias mudam", alegando que era um exemplo
da falta de firmeza dos céticos..:-) Como pode agora acusar os
céticos exatamente do contrário???
>

JR: Então mude a partir de agora, pois o segundo estudo apresenta
traços de evidenciais de uma substancia na parte pesquisada no
primeiro estudo, e no restante da fibras do tecido essa substancia
naõ é encontrada. Pelo menos voces céticos nao podem argumentar que
não há evidencias. Segundo dados empíricos de historiadores,
arqueólogos e especialistas em datação a substancia vanilina só é
encontrada em objetos e artefatos que pertencem a Idade média. Os que
objetos se encontram além não contém essa substancia. Como foi
comprovado em laboratórios distintos , as análises químicas
realizadas em Milão, califórnia, no Texas e na União soviética as
fibras do tecido que foram analisadas quimicamente e foi-se
constatado que pertenciam aos remendos medievais, mas as encontradas
no restate do tecido nao era dessa natureza, onde não foi encontrado
vanilina.

E por favor nao me obrigue sempre a repetir as mesmas coisas.







SUBJECT: Re: ninguem é inocente.
FROM: "junior_br2001" <junior_br2001@yahoo.com.br>
TO: ciencialist@yahoogrupos.com.br
DATE: 22/03/2005 13:52


Tarcisio,

De fato eu me equivoquei com as datas. Me basei no site cético e nao
conferi outros sites oficiais da ciencia a respeito do manto.
Na realidade o segundo estudo é uma *tentativa* de refutar o primeiro
sim e nao um estudo auxiliar como eu havia dito.

Mas mesmo assim, o segundo estudo traz evidencias que merecem
investigação( mas é claro, tudo em ciencia é questionável)

JR


--- Em ciencialist@yahoogrupos.com.br, TARCISIO BORGES <tbs97@f...>
escreveu
> On Mon, 21 Mar 2005, junior_br2001 wrote:
> > JR: Voce tem certeza disso? Eu li 1340 anos, inclusive li isso no
> > site que voce me apontou, dos céticos( a nao ser que o valor de
600
> > seja o minimo do primeiro estudo), . Se o Rogers estipulou um
> > mínimo , ele coincide *mais ou menos* com esse valor de 1340. O
que
> > pode invalidar é margem especulada em 3000 anos
>
> Você tem razão. Fui ler novamente:
>
> "O Dr. Rogers estimou a data real do sudário entre cerca de
1000
> AEC e 1700 EC."
>
> Neste caso a segunda datação não invalida a primeira, ou vice
versa. A
> minha primeira interpretação do assunto é que a peça teria de 1000
a 3000
> anos e portanto incompatível com a primeira datação.
>
>
> > JR: As técnicas do carbono-14 hoje em dia estão muito melhores
que as
> > de 88. Além do mais voce nao prestou atenção ao que foi dito sobre
> > contaminação do tecido com carbono-14 recente, revestimento
> > bioplástico e teste relizado nos remendos medievais e não no
tecido
> > propriamente dito.
>
> Como você diz, eu não vi nada disso sendo divulgado.
>
>
> > JR: Foi uma metologia científica, e não um teste homeopático, ele
> > encontrou uma substancia química no remendo, e no resto do tecido
a
> > vanilina já não estava presente. Isso é uma evidencia, não um
estudo
> > mal-feito.
>
> Mas que remendo? Aquele que foi queimado no teste de C14?
>
>
> > O Stephen William Hawking só admitiu que estava errado, quando
sua
> > teoria já tinha sido refutada por outro físico...se ele não
admitisse
> > pegaria mal. Muitos físicos não gostam dele, nao pela sua
> > deficiencia, mas por que ele é um especulador sonhador
>
> Todas as teorias são refutadas por outros cientistas e precisam
ser. A
> questão é convencer o autor da teoria que ela está errada.
>
> O mérito de Hawking está em aceitar o erro.
>
> Quando os bispos e padres irão aceitar que a relíquia é uma fraude?
>
> Não estou muito confiante de que aconteça.
>
> []s
> TARCISIO BORGES
> tbs97@f...





SUBJECT: Re: ninguem é inocente.
FROM: "junior_br2001" <junior_br2001@yahoo.com.br>
TO: ciencialist@yahoogrupos.com.br
DATE: 22/03/2005 14:09


--- Em ciencialist@yahoogrupos.com.br, TARCISIO BORGES <tbs97@f...>
escreveu
> Mas se estas fibras foram queimadas, como ele sabe que elas possuíam
> vanilina?


JR: Foram retiradas amostras de fibras exatamente do local em que
foram retiraddas para o primeiro estudo, mas não foi exatamente
aquelas que foram utilizadas o primeiro estudo.
Me parece que o Eduardo tem todo o trabalho do Rogers em .pdf. Não
sei exatamente se foram retiradas essas amostras do tecido(Isso teria
que ter autorização do Vaticano) ou se as amostras estudadas foram
sobras de fibras do primeiro estudo que foram guardadas e analisadas
em laboratórios de queimica posteriormente. Aí tem que pesquisar
direito.

JR







SUBJECT: Re: manto de turim (pessoas bem articuladas)
FROM: "junior_br2001" <junior_br2001@yahoo.com.br>
TO: ciencialist@yahoogrupos.com.br
DATE: 22/03/2005 14:24


--- Em ciencialist@yahoogrupos.com.br, Eduardo Gueron <edgueron@y...>
escreveu
>
> ISto posto, acho que não tenho nada a acrescentar, até
> porque, há pessoas mais cuidadosas e pacientes do que
> eu nessa lista e particularmente bem articuladas como
> o caso do Homero que defende pontos de vistas
> semelhantes aos meus de maneira bem mais brilhante.

JR: Isso é uma questão de opinião, eu particularmente gosto muito das
coisas que o Victor, o Alberto, o Manuel Bulcão escrevem. São textos
bem embasados com teorias da ciencia e apresentando uma certa
coerencia, razão, conhecimentos, saber, experiencia, prática e
maturidade. E não textos que contém repetecos exaustivos de slogans e
frases céticas.








SUBJECT: Fixismo - nova alegação insustentável..:-) (era manto de turim e céticos)
FROM: "Oraculo" <oraculo@atibaia.com.br>
TO: <ciencialist@yahoogrupos.com.br>
DATE: 22/03/2005 14:43

Olá Pubmed

Pubmed: Isso é fácil levantar, basta ver a postura dogmática em que voces
defendem o primeiro teste."

Isso é uma opinião, não uma prova ou evidencia. Copie e cole (ou apresente a mensagem) em que uma afirmação corrobore a acusação de fixismo.

Observe que não ligo se pensa assim ou assado, ou se acredita que é verdade o que afirmou. Estou cuidando que, quem leia as discussões, note o problema aqui. E o problema aqui é, quem lê percebe que, se você pode afirmar algo que não é verdade, que não tem evidencias nas mensagens anteriores, sobre céticos, nada impede que alege coisas que também não são verdade sobre outros pontos e em outros debates.

Assim, você precisa perceber que deve ter cuidado com o que afirma, principalmente se não é verdade. Mostre que a alegação de fixismo é real apresentando uma mensagem (basta uma) em que eu, ou qualquer outro que discordou de você até agora, defenda um fixismo, a não modificação do conhecimento, a aceitação dogmatica de qualquer alegação. Ou se desculpe pelo erro..:-) É simples assim.

Opiniões não são evidencias.

Pubmed: Se fossem céticos de verdade, questionariam as condições do testes, as técnicas de 88,"

Foram questionadas, por isso foram repetidas e confirmadas. Por isso são aceitas no momento, até que novos estudos, repetidos e confirmados, venham a mudar esse quadro. Tente outra fuga..:-)

Este trecho é um primor de absurdo lógico..:-)

Trecho absurdo:

Homero: Em todas as mensagens, céticas, encontrará a posição exatamente
contrária, de não-fixismo, de mudança, apenas coma condição do rigor
e cuidado ao aceitar evidencias e conclusões. Nunca, em nenhuma
mensagem, fixismo, até porque tudo o que a ciência tem de melhor é a
mudança e a adaptação a novos conhecimentos e dados.

Pubmed: JR: Exato.


O que está acontecendo? Não lê o texto que responde? Estou dizendo que em todos os textos que enviamos, céticos dizem exatamente o contrário que nos acusa e você concorda como EXATO? E em seguida diz que não tomamos esse cuidado??? A contradição entre suas frase é tão espantosa que vou me abster de responder..:-)

Sobre aceitar novos estudos, deixe-me ver se dessa vez consigo, embora já esteja acreditando que está fazendo isso de propósito e para disfarçar a falta de argumentos:

Pubmed: E se voces tivessem todo esse rigor em aceitar as
evidencias e conclusões aceitariam o segundo estudo, pois ele tb se
baseia em evidencias e dados reais.

Um segundo estudo QUE NÃO FOI AINDA REPRODUZIDO E CUJAS CONCLUSÕES ESPERAM CONFIRMAÇÃO. O que há de dificil de compreender nisso? NENHUM estudo é aceito sem isso, nem os primeiros com o carbono-14, nem esse novo estudo quimico. NENHUM. Pare de repetir a mesma tolice e tente um argumento novo, as pesoas que leem estão achando que tem alguma limitação em compreender o obvio..:-)

Pubmed: > Alias, foi justametne você que criticou essa posição cética,
de "mudar conforme as evidencias mudam", alegando que era um exemplo
da falta de firmeza dos céticos..:-) Como pode agora acusar os
céticos exatamente do contrário???
>

JR: Então mude a partir de agora,

Não vou mudar porque não tenho motivo para mudar. Muda-se com motivo, não porque sim e pronto. E estou REFUTANDO seu argumento, não discutindo uma mudança. Se defenda ou aceite o erro, "saida pela direita" é uma fuga sem sentido. Você afirmou duas coisas que se contradizem, que céticos como eu são fixistas, que tem dogmas que nunca mudam e AO MESMO TEMPO que mudamos a toa, que basta as evidencias mudarem que mudamos de conclusão e que isso é um defeito, uma falha que demonstra que somos fracos e mutáveis. AS DUAS COISAS NÃO PODEM SER CORRETAS AO MESMO TEMPO, DECIDA-SE. Ou assuma o erro..:-)

Pubmed, isso é só teimosia de sua parte, por não ter mais o que argumentar. As evidencias e argumentos já foram expostos. Está tudo claro para quem leu e acompanhou o debate. Você o iniciou alegando que céticos eram tolos por não aceitar um novo estudo sobre o sudario que mudava a data de criação. Nós apresentamos evidencias que o novo estudo, por falta de confirmação independente e reprodução, não era tão confiável quanto os anteriores, bem documentados e já confirmados e reproduzidos. Iniciou-se então uma enorme troca de mensagens com toda uma discussão, mas o cerne do debate é este, que estudo tem mais confiabilidade, neste momento.

Escolha o que quiser, mas não invente acusações ou alegações que não pode manter. E não pense que estou chateado, não é por isso..:-) Eu realmente não ligo para isso, apenas gosto de honestidade intelectual e clareza ao debater..:-)

Um abraço.

Homero


----- Original Message -----
From: junior_br2001
To: ciencialist@yahoogrupos.com.br
Sent: Tuesday, March 22, 2005 1:49 PM
Subject: [ciencialist] Re: Fixismo - nova alegação insustentável..:-) (era manto de turim e céticos)



Segue comentários:

--- Em ciencialist@yahoogrupos.com.br, "Oraculo" <oraculo@a...>
escreveu
> Olá Pubmed
>
> Pubmed: JR: Na verdade o lado cético defende um fixismo e isso é
muito ruim
> no debate e em ciencia"
>
> Não, não defende. Você continua a levantar alegações que não pode
sustentar. Isso não é legítimo, nem ajuda o debate. Procure nas
mensagens anteriores e tente encontrar uma mensagem, qualquer uma,
que defenda o fixismo. Está distorcendo, me parece conscientemente, o
debate, na falta de argumentos ou evidencias que sejam válidas para
sustentar seu ponto de vista.

JR: Isso é fácil levantar, basta ver a postura dogmática em que voces
defendem o primeiro teste. Se fossem céticos de verdade,
questionariam as condições do testes, as técnicas de 88, as possíveis
falhas que o teste possa ter com contaminações de carbono-14 recente
e revestimento bioplástico, as condições das fibras em que foram
realizados os testes(se foram realmente os remendos ou não), e
aceitariam as novas descobertas até que outraas descobertas refutem
com dados experimentais( e claaro que como contece no primeiro
estudo, o segundo tb pode ser questionado. Isso sim é agir
cientificamente, o resto é balela do ceticismo organizado

>
> Em todas as mensagens, céticas, encontrará a posição exatamente
contrária, de não-fixismo, de mudança, apenas coma condição do rigor
e cuidado ao aceitar evidencias e conclusões. Nunca, em nenhuma
mensagem, fixismo, até porque tudo o que a ciência tem de melhor é a
mudança e a adaptação a novos conhecimentos e dados.

JR: Exato. Mas voces não tomam esse cuidado. Saem questionando logo
de cara o segundo estudo sem dados experimentais e atuais que possam
contradize-lo. E se voces tivessem todo esse rigor em aceitar as
evidencias e conclusões aceitariam o segundo estudo, pois ele tb se
baseia em evidencias e dados reais. Se dão valor excessivo ao
primeiro estudo e que o segundo é balela, voce está fazendo
exatamente o oposto do diz em relação a mudnças e aceitação de novos
conhecimentos. para que haja mudanças todos os estudos tem de ser
questionados, tanto o primeiro como o segundo.

>
> Alias, foi justametne você que criticou essa posição cética,
de "mudar conforme as evidencias mudam", alegando que era um exemplo
da falta de firmeza dos céticos..:-) Como pode agora acusar os
céticos exatamente do contrário???
>

JR: Então mude a partir de agora, pois o segundo estudo apresenta
traços de evidenciais de uma substancia na parte pesquisada no
primeiro estudo, e no restante da fibras do tecido essa substancia
naõ é encontrada. Pelo menos voces céticos nao podem argumentar que
não há evidencias. Segundo dados empíricos de historiadores,
arqueólogos e especialistas em datação a substancia vanilina só é
encontrada em objetos e artefatos que pertencem a Idade média. Os que
objetos se encontram além não contém essa substancia. Como foi
comprovado em laboratórios distintos , as análises químicas
realizadas em Milão, califórnia, no Texas e na União soviética as
fibras do tecido que foram analisadas quimicamente e foi-se
constatado que pertenciam aos remendos medievais, mas as encontradas
no restate do tecido nao era dessa natureza, onde não foi encontrado
vanilina.

E por favor nao me obrigue sempre a repetir as mesmas coisas.







##### ##### #####

Para saber mais visite
http://www.ciencialist.hpg.ig.com.br


##### ##### ##### #####


Yahoo! Grupos, um serviço oferecido por:







------------------------------------------------------------------------------
Links do Yahoo! Grupos

a.. Para visitar o site do seu grupo na web, acesse:
http://br.groups.yahoo.com/group/ciencialist/

b.. Para sair deste grupo, envie um e-mail para:
ciencialist-unsubscribe@yahoogrupos.com.br

c.. O uso que você faz do Yahoo! Grupos está sujeito aos Termos do Serviço do Yahoo!.



[As partes desta mensagem que não continham texto foram removidas]



SUBJECT: Pedido de desculpas (meu)..:-) (era: Fixismo )
FROM: "Oraculo" <oraculo@atibaia.com.br>
TO: <ciencialist@yahoogrupos.com.br>
DATE: 22/03/2005 15:08

Olá Pubmed

Embora minha argumentação na mensagem anterior continue pertinente, eu acho que me empolguei e acabei usando a expressão "pare de dizer tolices" que pode ser bastante ofensiva de alguma forma. Assim, peço desculpas antecipadas por ela e espero que a desconsidere na mensagem anterior. Fui, devido a emoção da escrita, deselegante e infeliz no uso do termo.

Toda alegação, em uma lista de discussão civilizada, deve ser levada em conta e toda manifestação, mesmo as mais exasperantes, deve ser considerada e recebida com cuidado.

De novo, me desculpe pelo excesso e afirmo que não penso que em nenhum momento você diga tolices ou coisa do tipo, mesmo que discordemos em muitas coisas..:-)

Um abraço.

Homero

[As partes desta mensagem que não continham texto foram removidas]



SUBJECT: Re: [ciencialist] Re: ninguem é inocente.
FROM: "Silvio" <scordeiro@terra.com.br>
TO: <ciencialist@yahoogrupos.com.br>
DATE: 22/03/2005 15:17

Júnior!!!!!

Curiosamente, nenhum dos "iluminados" ( Buda, J. Cristo,) deixou qualquer
vestígio de sua passagem por esse vale de lágrimas. Buda pode até ter
falado: mas quem, em sã consciência,quem pode repetir ipsum litera suas
palavras que rolam em tradições e traduções monásticas há 2.000 anos?

Não vá no papo dessa turma, prefira ler Freud. Vamos lutar pela canonização
do Paulo Coelho.

sds.,

silvio


-----Mensagem Original-----
De: "junior_br2001" <junior_br2001@yahoo.com.br>
Para: <ciencialist@yahoogrupos.com.br>
Enviada em: segunda-feira, 21 de março de 2005 14:25
Assunto: [ciencialist] Re: ninguem é inocente.




Buda disso algo diferente:

"Não acredite em algo simplesmente porque ouviu. Não acredite em algo
simplesmente porque todos falam a respeito. Não acredite em algo
simplesmente porque esta escrito em seus livros religiosos. Não
acredite em algo só porque seus professores e mestres dizem que é
verdade. Não acredite em tradições só porque foram passadas de
geração em geração. Mas depois de muita análise e observação, se você
vê que algo concorda com a razão, e que conduz ao bem e beneficio de
todos, aceite-o e viva-o."

JR





--- Em ciencialist@yahoogrupos.com.br, TARCISIO BORGES <tbs97@f...>
escreveu
> Talvez Buda o fosse, pois em determinado momento diz aos seus
discípulos:
>
> Não acredite em seus professores.
>
> :-)






##### ##### #####

Para saber mais visite
http://www.ciencialist.hpg.ig.com.br


##### ##### ##### #####
Links do Yahoo! Grupos












SUBJECT: Re: [ciencialist] Re: ninguem é inocente.
FROM: "Silvio" <scordeiro@terra.com.br>
TO: <ciencialist@yahoogrupos.com.br>
DATE: 22/03/2005 15:26

Homero:

Milagres não existem. E a igreja só tem crédito em sua cáfila de fiéis.

Inclua na lista, não só Galileu que não foi queimado porque era sobrinho do
papa, as milhares de feiticeiras anônimas, os judeus ricos, Giordano Bruno,
Miguel Servet (esse mandado assar em fogo brando por calvino, fundador de
uma igreja....)

Tudo em nome do Pai, amém.

sds.,

silvio.

-----Mensagem Original-----
De: "Oraculo" <oraculo@atibaia.com.br>
Para: <ciencialist@yahoogrupos.com.br>
Enviada em: segunda-feira, 21 de março de 2005 14:35
Assunto: Re: [ciencialist] Re: ninguem é inocente.



Olá Tarciso

Sim, hoje em dia. Mas historicamente, o sudario já foi reliquia real e
inconteste. Se hoje apenas alguns bispos e padres (e alguns dos mais
influentes e poderosos..:-) é porque a igreja sempre soube se manter com um
dos pés em cada lado de uma questão, principalmente depois do fiasco de
Galileu..:-)

Ela hoje toma bem mais cuidado com suas afirmações (veja o trabalho que dá
hoje reconhecer um milagre, e como era bem mais rápido e comum no passado),
para não ficar (muito) desacreditada.

De qualquer forma, não é dificil perceber que a igreja gostaria demais que o
sudario tivesse a data correta, para ser real (no sentido de ter coberto
jesus)..:-)

Um abraço.

Homero

----- Original Message -----
From: TARCISIO BORGES
To: ciencialist@yahoogrupos.com.br
Sent: Monday, March 21, 2005 8:32 AM
Subject: Re: [ciencialist] Re: ninguem é inocente.


On Fri, 18 Mar 2005, Oraculo wrote:
> A Igreja é que inicialmente apresentou o caso, o sudário, como
> reliquia autentica e com as afirmações sobre sua origem e utilização.
> A mesma igreja que, brincando de esconde-esconde, as vezes permite, as
> vezes nega, o estudo confiável ao artefato.

O Vaticano não assume a autenticidade do artefato. Quem defende sua
autenticidade são alguns bispos e padres.


[]s
TARCISIO BORGES
tbs97@fisica.ufpr.br



##### ##### #####

Para saber mais visite
http://www.ciencialist.hpg.ig.com.br


##### ##### ##### #####


Yahoo! Grupos, um serviço oferecido por:
PUBLICIDADE




------------------------------------------------------------------------------
Links do Yahoo! Grupos

a.. Para visitar o site do seu grupo na web, acesse:
http://br.groups.yahoo.com/group/ciencialist/

b.. Para sair deste grupo, envie um e-mail para:
ciencialist-unsubscribe@yahoogrupos.com.br

c.. O uso que você faz do Yahoo! Grupos está sujeito aos Termos do
Serviço do Yahoo!.



[As partes desta mensagem que não continham texto foram removidas]



##### ##### #####

Para saber mais visite
http://www.ciencialist.hpg.ig.com.br


##### ##### ##### #####
Links do Yahoo! Grupos












SUBJECT: Re: Fixismo - nova alegação insustentável..:-) (era manto de turim e céticos)
FROM: "junior_br2001" <junior_br2001@yahoo.com.br>
TO: ciencialist@yahoogrupos.com.br
DATE: 22/03/2005 15:27


Segue comentários:


--- Em ciencialist@yahoogrupos.com.br, "Oraculo" <oraculo@a...>
escreveu
> Olá Pubmed
>
> Pubmed: Isso é fácil levantar, basta ver a postura dogmática em que
voces
> defendem o primeiro teste."
>
> Isso é uma opinião, não uma prova ou evidencia. Copie e cole (ou
apresente a mensagem) em que uma afirmação corrobore a acusação de
fixismo.

JR: Quer dizer que temos tb de ter uma evidencia e realizar um teste
em relaçao a frase que eu disse acima? O que eu disse são argumentos,
não são hipóteses , nem teorias que precisam de testes. Quer fazer um
experimento de minhas fraes? É por aí que falo que o fanatismo dos
céticos vão muito além. O máximo que se pode fazer é rebater meus
argumentos

>
> Observe que não ligo se pensa assim ou assado, ou se acredita que é
verdade o que afirmou. Estou cuidando que, quem leia as discussões,
note o problema aqui. E o problema aqui é, quem lê percebe que, se
você pode afirmar algo que não é verdade, que não tem evidencias nas
mensagens anteriores, sobre céticos, nada impede que alege coisas que
também não são verdade sobre outros pontos e em outros debates.

JR: Me baseio na minha razão para dizer que existe fanatismo no meio
cetico. Não preciso de evidencias. As atitudes dogmáticas dos céticos
dizem por elas mesmos e serve como evidencias

>
> Assim, você precisa perceber que deve ter cuidado com o que afirma,
principalmente se não é verdade.

JR: Voce diz que nao é verdade. Mas voce tb não é o dono da verdade.
E pode estar errado em seus julgamentos

Mostre que a alegação de fixismo é real apresentando uma mensagem
(basta uma) em que eu, ou qualquer outro que discordou de você até
agora, defenda um fixismo, a não modificação do conhecimento, a
aceitação dogmatica de qualquer alegação. Ou se desculpe pelo erro..:-
) É simples assim.

JR: Basta ler várias de suas mensaagens.

>
> Opiniões não são evidencias.

JR: Opniões nos argumentos de minhas mensagens, ou voce está falando
dos estudos que apresentei que nao foram meras opniões. Veja o
absurdo dessa afirmação acima. Minhas opniãoes nao precisam ter
evidencias, nem ser passadas por testes científicos - isso a meu ver
soa fanatismo. Alías o que mai apresentei foram estudos, e no
opinioes minhas.

>>
> Foram questionadas, por isso foram repetidas e confirmadas. Por
isso são aceitas no momento, até que novos estudos, repetidos e
confirmados, venham a mudar esse quadro. Tente outra fuga..:-)


JR: Repetidas e confirmadas? A Igreja católica nem liberou o manto
novamente para que fossem feitas novas pesquisas. Tudo o que temos
são 3 testes independentes do carbono-14 realizados em 88, e vários
testes químicos que foram repetidos em laboratórios de vários países
tb


>
> Pubmed: JR: Exato.
>
>
> O que está acontecendo? Não lê o texto que responde? Estou dizendo
que em todos os textos que enviamos, céticos dizem exatamente o
contrário que nos acusa e você concorda como EXATO? E em seguida diz
que não tomamos esse cuidado??? A contradição entre suas frase é tão
espantosa que vou me abster de responder..:-)


JR: Voce afirma teoricamente uma coisa , mas nao é fiel a ela na
pratica. Se voces defendem que deve haver mudanças quando há
evidencias, e quando elas surgem voces ficam parados; estão agindo
como fixistas. Então, quem se contradiz são os céticos.


>
> Sobre aceitar novos estudos, deixe-me ver se dessa vez consigo,
embora já esteja acreditando que está fazendo isso de propósito e
para disfarçar a falta de argumentos:

JR: Nada está feito de propósito por mim. É muito óbvio a sua
incoerencia nos textos que envia.

>> Um segundo estudo QUE NÃO FOI AINDA REPRODUZIDO E CUJAS CONCLUSÕES
ESPERAM CONFIRMAÇÃO. O que há de dificil de compreender nisso? NENHUM
estudo é aceito sem isso, nem os primeiros com o carbono-14, nem esse
novo estudo quimico. NENHUM. Pare de repetir a mesma tolice e tente
um argumento novo, as pesoas que leem estão achando que tem alguma
limitação em compreender o obvio..:-)

JR: Retorne as mensagens anteriores, e reveja os testes químicos que
form realizados antes de me acusar. Inclusive o Rogers fez a pesquisa
para confirmar alguns destes estudos.

> Não vou mudar porque não tenho motivo para mudar. Muda-se com
motivo, não porque sim e pronto. E estou REFUTANDO seu argumento, não
discutindo uma mudança. Se defenda ou aceite o erro, "saida pela
direita" é uma fuga sem sentido. Você afirmou duas coisas que se
contradizem, que céticos como eu são fixistas, que tem dogmas que
nunca mudam e AO MESMO TEMPO que mudamos a toa, que basta as
evidencias mudarem que mudamos de conclusão e que isso é um defeito,
uma falha que demonstra que somos fracos e mutáveis. AS DUAS COISAS
NÃO PODEM SER CORRETAS AO MESMO TEMPO, DECIDA-SE. Ou assuma o erro..:-

JR: Então continue fixista. Afinal é um direito seu

>
> Pubmed, isso é só teimosia de sua parte, por não ter mais o que
argumentar. As evidencias e argumentos já foram expostos. Está tudo
claro para quem leu e acompanhou o debate. Você o iniciou alegando
que céticos eram tolos por não aceitar um novo estudo sobre o sudario
que mudava a data de criação. Nós apresentamos evidencias que o novo
estudo, por falta de confirmação independente e reprodução, não era
tão confiável quanto os anteriores, bem documentados e já confirmados
e reproduzidos. Iniciou-se então uma enorme troca de mensagens com
toda uma discussão, mas o cerne do debate é este, que estudo tem mais
confiabilidade, neste momento.

JR: Se voce acompanhasse a discussão e lesse as mensagens veria que
os estudos químicos foram apresentados. Inclusive o de Rogers é uma
continução e constatação dos anteriores, senão a Nature nao teria
publicado, lembre-se que qualquer matéria publicada é analisada por
uma junta de cientistas e especialistas no assunto, antes de ser
enviadas ao conhecimento público. Mas o seu ceticismo é tao
exacerbado que voce nega isso

>
> Escolha o que quiser, mas não invente acusações ou alegações que
não pode manter. E não pense que estou chateado, não é por isso..:-)
Eu realmente não ligo para isso, apenas gosto de honestidade
intelectual e clareza ao debater..:-)

JR: È claro que voce está chateado, isso é nítido. Senão voce nao
viria até aqui com justificativas e omitiria uma resposta. Mas voce
não pode me acusar de invenção e alegações falsas diantes de estudos
tão verdadeiros.
Se quer chocoalhar , crie resistencias a junta científica e de
especialistas que analisou o trabalho de Rogers , antes de aprova-lo
a Nature.


JR






SUBJECT: Re: Pedido de desculpas (meu)..:-) (era: Fixismo )
FROM: "junior_br2001" <junior_br2001@yahoo.com.br>
TO: ciencialist@yahoogrupos.com.br
DATE: 22/03/2005 15:32


OK. Aceito suas desculpas e lhe peço tb caso lhe tenha ofendido em
alguma ocasião. Mas lembre-se quando falo de céticos, falo de modo
geral, principalmente os pseudoceticos.
Mas, na verdade a minha critica se estende mais aos grupinhos
braisleiros que nao aceitam outros pessoas a nao ser ateus e
agnosticos
Como te falei a CSICOP, tem até religiosos, existem judeus e
católicos que são céticos por nao seguir uma interpretação literal da
bíblia. O proprio Steven Jay Gould era judeu, ele tem até um livro
que defende a religião convivendo em respeito mútuo com a ciencia,
mas cada área com sus respectivas verdades e atuações.
Recentemente o Marcelo Gleiser envou um texto em que ele fala sobre
religiao, que ela não deveria desparecer da face da Terra.

JR

--- Em ciencialist@yahoogrupos.com.br, "Oraculo" <oraculo@a...>
escreveu
> Olá Pubmed
>
> Embora minha argumentação na mensagem anterior continue pertinente,
eu acho que me empolguei e acabei usando a expressão "pare de dizer
tolices" que pode ser bastante ofensiva de alguma forma. Assim, peço
desculpas antecipadas por ela e espero que a desconsidere na mensagem
anterior. Fui, devido a emoção da escrita, deselegante e infeliz no
uso do termo.
>
> Toda alegação, em uma lista de discussão civilizada, deve ser
levada em conta e toda manifestação, mesmo as mais exasperantes, deve
ser considerada e recebida com cuidado.
>
> De novo, me desculpe pelo excesso e afirmo que não penso que em
nenhum momento você diga tolices ou coisa do tipo, mesmo que
discordemos em muitas coisas..:-)
>
> Um abraço.
>
> Homero
>
> [As partes desta mensagem que não continham texto foram removidas]





SUBJECT: Re: [ciencialist] Fw: Problema basico de volumes
FROM: "Sergio M. M. Taborda" <sergiotaborda@terra.com.br>
TO: ciencialist@yahoogrupos.com.br
DATE: 22/03/2005 15:48

Luiz Ferraz Netto wrote:

> Quem quer fazer a primeira redação da resposta a essa pergunta?
>
> []'
> ===========================
> Luiz Ferraz Netto [Léo]
> leobarretos@uol.com.br
> http://www.feiradeciencias.com.br
> ===========================
> -----Mensagem Original-----
> De: nunofigueiredo
> Para: leobarretos
> Enviada em: terça-feira, 22 de março de 2005 02:00
> Assunto: Problema basico de volumes
>
>
> Prezado professor:
> Achei o seu site muitop interessante. Assim, estoui a lhe escrever um
> função de uma duvida.
> O meu cunhado tem uma fabrica de cosmeticos e, surgiu uma discussão
> entre nós: Ele insiste, que em determiado vasilhame que tem a
> capacidade para 4 litros, recebe 4 litros de água, mas uma quantidade
> diferente (menor) de shampoo.
> Por mais que eu tentasse explicar que o litro é uima unidade
> universal(não varia com o tipo de produto) por se tratar de uma medida
> de volume, ele insiste no erro.
> Eu expliquei-lhe que o peso sim, varia conforme o produto por causa da
> densidade, mas o volume jamais muda. Ele aceitou que o peso varia, mas
> o volume também.
> Eu argumentei entre outras, que não poderiamos ter duas variaveis para
> o mesmo produto, pois tornar-se-ia impossivel calcular uma em função
> da outra... Temtei explicar que um litro de agua ou um litro de chumbo
> derretido sao um litro. Apenas um pesa aproximadamente um Kg e o outro
> vários Kg.
> Ele insiste que está certo e que já foi autoado pelo inmetro mais de
> uma vês por isso. Eu, irritado disse que não era possível e, que se o
> inmetro multou porque um litro não era um litro, o inmetro estava
> errado. É obvio que ele foi autoado por diferenças no calculo do peso
> liquido e nao no volume em l ou ml.
> Bem, resumindo, fui incapaz de lhe explicar isso, apesar da minha
> formação superior em física (incompleta).
> Achei qe o tema é interessante o suficiente para o expor ao senhor. Se
> não for pedir demais, gostaria de uma opinião para a apresentar.
> Talvez o problema seja apresentar de forma compreensivel a definiçao
> de litro. Sem se levar em consideração os gases por serem fluidos
> compressiveis, e claro.
> Agradeço de antemão a sua resposta,
>

O volume de um recipiente (vazilhanme) é constante. O recipiente suporta
o mesmo volume de qualquer substancia. Parece que é isto que o seu
cunhado falha em entender. O volume do recipiente não depende do liquido
que ele contém, pois o volume é sempre o mesmo, mesmo que o recipiente
estiver vazio.
O volume do recipiente é calculado geometricamente. Existem formulas
matemáticas para o calcular para recipientes simples. Por exemplo, um
cilindro tem um volume igual ao produto da altura pela area da base.
Esta formula é a base dos medidores de volume. Que são normalmente
cilindros transparentes com escalas do lado de fora.
Se a forma do recipiente não é simples de corresponder com solidos
geometricos simples - como cilindros, cubos e esfera - então existe um
truque, que é encher o recipiente com água e despejar essa água num
recepiente do qual sabemos o volume. (se possivel um recepiente calibrado).
Sendo que o volume do recipiente depende apenas da forma do recipiente e
não do que ele contém, o seu volume é constante e não varia conforme o
liquido que se coloca nele.


Sérgio Taborda








SUBJECT: Re: [ciencialist] Re: Fixismo - nova alegação insustentável..:-) (era manto de turim e céticos)
FROM: "Oraculo" <oraculo@atibaia.com.br>
TO: <ciencialist@yahoogrupos.com.br>
DATE: 22/03/2005 15:57

Olá Pubmed

Pubmed: Basta ler várias de suas mensaagens.

Não, o onus é seu, você afirmou que eu disse, deve apresentar as mensagens ou frases correspondentes..:-) Ou ficaremos todos, na lista, dizendo que fulano disse, que cicrano falou, sem que ninguém mais saiba se é verdade ou invenção. Se eu disse, se basta ler as mensagens, apresente as mesmas e prove sua afirmação..:-)

Não precisa de um estudo publicado ou teste, basta dar uma olhada nos arquivos da lista e apresentar as que eu, segundo você, disse isso ou aquilo..:-)

Homero



----- Original Message -----
From: junior_br2001
To: ciencialist@yahoogrupos.com.br
Sent: Tuesday, March 22, 2005 3:27 PM
Subject: [ciencialist] Re: Fixismo - nova alegação insustentável..:-) (era manto de turim e céticos)



Segue comentários:


--- Em ciencialist@yahoogrupos.com.br, "Oraculo" <oraculo@a...>
escreveu
> Olá Pubmed
>
> Pubmed: Isso é fácil levantar, basta ver a postura dogmática em que
voces
> defendem o primeiro teste."
>
> Isso é uma opinião, não uma prova ou evidencia. Copie e cole (ou
apresente a mensagem) em que uma afirmação corrobore a acusação de
fixismo.

JR: Quer dizer que temos tb de ter uma evidencia e realizar um teste
em relaçao a frase que eu disse acima? O que eu disse são argumentos,
não são hipóteses , nem teorias que precisam de testes. Quer fazer um
experimento de minhas fraes? É por aí que falo que o fanatismo dos
céticos vão muito além. O máximo que se pode fazer é rebater meus
argumentos

>
> Observe que não ligo se pensa assim ou assado, ou se acredita que é
verdade o que afirmou. Estou cuidando que, quem leia as discussões,
note o problema aqui. E o problema aqui é, quem lê percebe que, se
você pode afirmar algo que não é verdade, que não tem evidencias nas
mensagens anteriores, sobre céticos, nada impede que alege coisas que
também não são verdade sobre outros pontos e em outros debates.

JR: Me baseio na minha razão para dizer que existe fanatismo no meio
cetico. Não preciso de evidencias. As atitudes dogmáticas dos céticos
dizem por elas mesmos e serve como evidencias

>
> Assim, você precisa perceber que deve ter cuidado com o que afirma,
principalmente se não é verdade.

JR: Voce diz que nao é verdade. Mas voce tb não é o dono da verdade.
E pode estar errado em seus julgamentos

Mostre que a alegação de fixismo é real apresentando uma mensagem
(basta uma) em que eu, ou qualquer outro que discordou de você até
agora, defenda um fixismo, a não modificação do conhecimento, a
aceitação dogmatica de qualquer alegação. Ou se desculpe pelo erro..:-
) É simples assim.

JR: Basta ler várias de suas mensaagens.

>
> Opiniões não são evidencias.

JR: Opniões nos argumentos de minhas mensagens, ou voce está falando
dos estudos que apresentei que nao foram meras opniões. Veja o
absurdo dessa afirmação acima. Minhas opniãoes nao precisam ter
evidencias, nem ser passadas por testes científicos - isso a meu ver
soa fanatismo. Alías o que mai apresentei foram estudos, e no
opinioes minhas.

>>
> Foram questionadas, por isso foram repetidas e confirmadas. Por
isso são aceitas no momento, até que novos estudos, repetidos e
confirmados, venham a mudar esse quadro. Tente outra fuga..:-)


JR: Repetidas e confirmadas? A Igreja católica nem liberou o manto
novamente para que fossem feitas novas pesquisas. Tudo o que temos
são 3 testes independentes do carbono-14 realizados em 88, e vários
testes químicos que foram repetidos em laboratórios de vários países
tb


>
> Pubmed: JR: Exato.
>
>
> O que está acontecendo? Não lê o texto que responde? Estou dizendo
que em todos os textos que enviamos, céticos dizem exatamente o
contrário que nos acusa e você concorda como EXATO? E em seguida diz
que não tomamos esse cuidado??? A contradição entre suas frase é tão
espantosa que vou me abster de responder..:-)


JR: Voce afirma teoricamente uma coisa , mas nao é fiel a ela na
pratica. Se voces defendem que deve haver mudanças quando há
evidencias, e quando elas surgem voces ficam parados; estão agindo
como fixistas. Então, quem se contradiz são os céticos.


>
> Sobre aceitar novos estudos, deixe-me ver se dessa vez consigo,
embora já esteja acreditando que está fazendo isso de propósito e
para disfarçar a falta de argumentos:

JR: Nada está feito de propósito por mim. É muito óbvio a sua
incoerencia nos textos que envia.

>> Um segundo estudo QUE NÃO FOI AINDA REPRODUZIDO E CUJAS CONCLUSÕES
ESPERAM CONFIRMAÇÃO. O que há de dificil de compreender nisso? NENHUM
estudo é aceito sem isso, nem os primeiros com o carbono-14, nem esse
novo estudo quimico. NENHUM. Pare de repetir a mesma tolice e tente
um argumento novo, as pesoas que leem estão achando que tem alguma
limitação em compreender o obvio..:-)

JR: Retorne as mensagens anteriores, e reveja os testes químicos que
form realizados antes de me acusar. Inclusive o Rogers fez a pesquisa
para confirmar alguns destes estudos.

> Não vou mudar porque não tenho motivo para mudar. Muda-se com
motivo, não porque sim e pronto. E estou REFUTANDO seu argumento, não
discutindo uma mudança. Se defenda ou aceite o erro, "saida pela
direita" é uma fuga sem sentido. Você afirmou duas coisas que se
contradizem, que céticos como eu são fixistas, que tem dogmas que
nunca mudam e AO MESMO TEMPO que mudamos a toa, que basta as
evidencias mudarem que mudamos de conclusão e que isso é um defeito,
uma falha que demonstra que somos fracos e mutáveis. AS DUAS COISAS
NÃO PODEM SER CORRETAS AO MESMO TEMPO, DECIDA-SE. Ou assuma o erro..:-

JR: Então continue fixista. Afinal é um direito seu

>
> Pubmed, isso é só teimosia de sua parte, por não ter mais o que
argumentar. As evidencias e argumentos já foram expostos. Está tudo
claro para quem leu e acompanhou o debate. Você o iniciou alegando
que céticos eram tolos por não aceitar um novo estudo sobre o sudario
que mudava a data de criação. Nós apresentamos evidencias que o novo
estudo, por falta de confirmação independente e reprodução, não era
tão confiável quanto os anteriores, bem documentados e já confirmados
e reproduzidos. Iniciou-se então uma enorme troca de mensagens com
toda uma discussão, mas o cerne do debate é este, que estudo tem mais
confiabilidade, neste momento.

JR: Se voce acompanhasse a discussão e lesse as mensagens veria que
os estudos químicos foram apresentados. Inclusive o de Rogers é uma
continução e constatação dos anteriores, senão a Nature nao teria
publicado, lembre-se que qualquer matéria publicada é analisada por
uma junta de cientistas e especialistas no assunto, antes de ser
enviadas ao conhecimento público. Mas o seu ceticismo é tao
exacerbado que voce nega isso

>
> Escolha o que quiser, mas não invente acusações ou alegações que
não pode manter. E não pense que estou chateado, não é por isso..:-)
Eu realmente não ligo para isso, apenas gosto de honestidade
intelectual e clareza ao debater..:-)

JR: È claro que voce está chateado, isso é nítido. Senão voce nao
viria até aqui com justificativas e omitiria uma resposta. Mas voce
não pode me acusar de invenção e alegações falsas diantes de estudos
tão verdadeiros.
Se quer chocoalhar , crie resistencias a junta científica e de
especialistas que analisou o trabalho de Rogers , antes de aprova-lo
a Nature.


JR






##### ##### #####

Para saber mais visite
http://www.ciencialist.hpg.ig.com.br


##### ##### ##### #####


Yahoo! Grupos, um serviço oferecido por:

São Paulo Rio de Janeiro Curitiba Porto Alegre Belo Horizonte Brasília




------------------------------------------------------------------------------
Links do Yahoo! Grupos

a.. Para visitar o site do seu grupo na web, acesse:
http://br.groups.yahoo.com/group/ciencialist/

b.. Para sair deste grupo, envie um e-mail para:
ciencialist-unsubscribe@yahoogrupos.com.br

c.. O uso que você faz do Yahoo! Grupos está sujeito aos Termos do Serviço do Yahoo!.



[As partes desta mensagem que não continham texto foram removidas]



SUBJECT: Re: [ciencialist] Re: Provas cabais e alegações inconsequentes..:-) (era: Carbono-14 )
FROM: "Silvio" <scordeiro@terra.com.br>
TO: <ciencialist@yahoogrupos.com.br>
DATE: 22/03/2005 16:03

E o mais modesto, né Takata?


-----Mensagem Original-----
De: "E m i l i a n o C h e m e l l o" <chemelloe@yahoo.com.br>
Para: <ciencialist@yahoogrupos.com.br>
Enviada em: segunda-feira, 21 de março de 2005 17:26
Assunto: Re: [ciencialist] Re: Provas cabais e alegações inconsequentes..:-)
(era: Carbono-14 )


>
> ahaha... já que o clima é de descontração:
>
> [Takata]
> Nunca, jamais, pus-me como mais inteligente do q. alguem (ainda mais
> nesta lista). Se for por falta de ter dito o contrario, digo agora:
> sou o mais burro desta lista, sou o menos inteligente da internet,
> sou o mais ignorante da Terra, sou o menos esperto da Galaxia.
>
> [Emiliano]
> O Takata é esperto. Disse que era o mais burro desta lista, menos
> inteligente da Internet, mais ignorante da Terra, menos esperto da
> Galaxia,
> porém, acredito eu, pensa ser o mais genial do Universo! :-) (brincadeira)
>



SUBJECT: Fixismo - nova alegação insustentável..:-)
FROM: "Oraculo" <oraculo@atibaia.com.br>
TO: <ciencialist@yahoogrupos.com.br>
DATE: 22/03/2005 16:03

Olá Pubmed

Pubmed: JR: Opniões nos argumentos de minhas mensagens, ou voce está falando
dos estudos que apresentei que nao foram meras opniões. Veja o
absurdo dessa afirmação acima. Minhas opniãoes nao precisam ter
evidencias, nem ser passadas por testes científicos - isso a meu ver
soa fanatismo. Alías o que mai apresentei foram estudos, e no
opinioes minhas."

Um certo rigor formal, lógico, é fundamental para os debates. Nesta thread, em especial, estou refutando sua afirmação a respeito de minhas mensagens e de meu modo de pensar e de me expressar. Nada tem a ver com o sudario, com estudos ou outra coisa. Se relaciona unicamente com sua afirmação a meu respeito, a respeito da forma de pensar cética e pode ser demonstrada a partir das mensagens da lista. Basta que as apresente e comprove sua alegação.

Os estudos, sobre o sudario e outros, nada tem a ver com isso e nada modificam neste debate (veja que até o subject das mensagens mudou, é agora fixismo).

Portanto, sem as mensagens que comprovem meu fixismo, é apenas uma opinião sua, nada mais. Não importa quantos estudos tenha citado sobre o sudário, continua sendo apenas sua opinião sobre o fixismo de minhas posiçòes.

Sejamos claros e lóicos, ou o debate nào vai produzir anda de útil.

Homero


----- Original Message -----
From: junior_br2001
To: ciencialist@yahoogrupos.com.br
Sent: Tuesday, March 22, 2005 3:27 PM
Subject: [ciencialist] Re: Fixismo - nova alegação insustentável..:-) (era manto de turim e céticos)



Segue comentários:


--- Em ciencialist@yahoogrupos.com.br, "Oraculo" <oraculo@a...>
escreveu
> Olá Pubmed
>
> Pubmed: Isso é fácil levantar, basta ver a postura dogmática em que
voces
> defendem o primeiro teste."
>
> Isso é uma opinião, não uma prova ou evidencia. Copie e cole (ou
apresente a mensagem) em que uma afirmação corrobore a acusação de
fixismo.

JR: Quer dizer que temos tb de ter uma evidencia e realizar um teste
em relaçao a frase que eu disse acima? O que eu disse são argumentos,
não são hipóteses , nem teorias que precisam de testes. Quer fazer um
experimento de minhas fraes? É por aí que falo que o fanatismo dos
céticos vão muito além. O máximo que se pode fazer é rebater meus
argumentos

>
> Observe que não ligo se pensa assim ou assado, ou se acredita que é
verdade o que afirmou. Estou cuidando que, quem leia as discussões,
note o problema aqui. E o problema aqui é, quem lê percebe que, se
você pode afirmar algo que não é verdade, que não tem evidencias nas
mensagens anteriores, sobre céticos, nada impede que alege coisas que
também não são verdade sobre outros pontos e em outros debates.

JR: Me baseio na minha razão para dizer que existe fanatismo no meio
cetico. Não preciso de evidencias. As atitudes dogmáticas dos céticos
dizem por elas mesmos e serve como evidencias

>
> Assim, você precisa perceber que deve ter cuidado com o que afirma,
principalmente se não é verdade.

JR: Voce diz que nao é verdade. Mas voce tb não é o dono da verdade.
E pode estar errado em seus julgamentos

Mostre que a alegação de fixismo é real apresentando uma mensagem
(basta uma) em que eu, ou qualquer outro que discordou de você até
agora, defenda um fixismo, a não modificação do conhecimento, a
aceitação dogmatica de qualquer alegação. Ou se desculpe pelo erro..:-
) É simples assim.

JR: Basta ler várias de suas mensaagens.

>
> Opiniões não são evidencias.

JR: Opniões nos argumentos de minhas mensagens, ou voce está falando
dos estudos que apresentei que nao foram meras opniões. Veja o
absurdo dessa afirmação acima. Minhas opniãoes nao precisam ter
evidencias, nem ser passadas por testes científicos - isso a meu ver
soa fanatismo. Alías o que mai apresentei foram estudos, e no
opinioes minhas.

>>
> Foram questionadas, por isso foram repetidas e confirmadas. Por
isso são aceitas no momento, até que novos estudos, repetidos e
confirmados, venham a mudar esse quadro. Tente outra fuga..:-)


JR: Repetidas e confirmadas? A Igreja católica nem liberou o manto
novamente para que fossem feitas novas pesquisas. Tudo o que temos
são 3 testes independentes do carbono-14 realizados em 88, e vários
testes químicos que foram repetidos em laboratórios de vários países
tb


>
> Pubmed: JR: Exato.
>
>
> O que está acontecendo? Não lê o texto que responde? Estou dizendo
que em todos os textos que enviamos, céticos dizem exatamente o
contrário que nos acusa e você concorda como EXATO? E em seguida diz
que não tomamos esse cuidado??? A contradição entre suas frase é tão
espantosa que vou me abster de responder..:-)


JR: Voce afirma teoricamente uma coisa , mas nao é fiel a ela na
pratica. Se voces defendem que deve haver mudanças quando há
evidencias, e quando elas surgem voces ficam parados; estão agindo
como fixistas. Então, quem se contradiz são os céticos.


>
> Sobre aceitar novos estudos, deixe-me ver se dessa vez consigo,
embora já esteja acreditando que está fazendo isso de propósito e
para disfarçar a falta de argumentos:

JR: Nada está feito de propósito por mim. É muito óbvio a sua
incoerencia nos textos que envia.

>> Um segundo estudo QUE NÃO FOI AINDA REPRODUZIDO E CUJAS CONCLUSÕES
ESPERAM CONFIRMAÇÃO. O que há de dificil de compreender nisso? NENHUM
estudo é aceito sem isso, nem os primeiros com o carbono-14, nem esse
novo estudo quimico. NENHUM. Pare de repetir a mesma tolice e tente
um argumento novo, as pesoas que leem estão achando que tem alguma
limitação em compreender o obvio..:-)

JR: Retorne as mensagens anteriores, e reveja os testes químicos que
form realizados antes de me acusar. Inclusive o Rogers fez a pesquisa
para confirmar alguns destes estudos.

> Não vou mudar porque não tenho motivo para mudar. Muda-se com
motivo, não porque sim e pronto. E estou REFUTANDO seu argumento, não
discutindo uma mudança. Se defenda ou aceite o erro, "saida pela
direita" é uma fuga sem sentido. Você afirmou duas coisas que se
contradizem, que céticos como eu são fixistas, que tem dogmas que
nunca mudam e AO MESMO TEMPO que mudamos a toa, que basta as
evidencias mudarem que mudamos de conclusão e que isso é um defeito,
uma falha que demonstra que somos fracos e mutáveis. AS DUAS COISAS
NÃO PODEM SER CORRETAS AO MESMO TEMPO, DECIDA-SE. Ou assuma o erro..:-

JR: Então continue fixista. Afinal é um direito seu

>
> Pubmed, isso é só teimosia de sua parte, por não ter mais o que
argumentar. As evidencias e argumentos já foram expostos. Está tudo
claro para quem leu e acompanhou o debate. Você o iniciou alegando
que céticos eram tolos por não aceitar um novo estudo sobre o sudario
que mudava a data de criação. Nós apresentamos evidencias que o novo
estudo, por falta de confirmação independente e reprodução, não era
tão confiável quanto os anteriores, bem documentados e já confirmados
e reproduzidos. Iniciou-se então uma enorme troca de mensagens com
toda uma discussão, mas o cerne do debate é este, que estudo tem mais
confiabilidade, neste momento.

JR: Se voce acompanhasse a discussão e lesse as mensagens veria que
os estudos químicos foram apresentados. Inclusive o de Rogers é uma
continução e constatação dos anteriores, senão a Nature nao teria
publicado, lembre-se que qualquer matéria publicada é analisada por
uma junta de cientistas e especialistas no assunto, antes de ser
enviadas ao conhecimento público. Mas o seu ceticismo é tao
exacerbado que voce nega isso

>
> Escolha o que quiser, mas não invente acusações ou alegações que
não pode manter. E não pense que estou chateado, não é por isso..:-)
Eu realmente não ligo para isso, apenas gosto de honestidade
intelectual e clareza ao debater..:-)

JR: È claro que voce está chateado, isso é nítido. Senão voce nao
viria até aqui com justificativas e omitiria uma resposta. Mas voce
não pode me acusar de invenção e alegações falsas diantes de estudos
tão verdadeiros.
Se quer chocoalhar , crie resistencias a junta científica e de
especialistas que analisou o trabalho de Rogers , antes de aprova-lo
a Nature.


JR






##### ##### #####

Para saber mais visite
http://www.ciencialist.hpg.ig.com.br


##### ##### ##### #####


Yahoo! Grupos, um serviço oferecido por:

São Paulo Rio de Janeiro Curitiba Porto Alegre Belo Horizonte Brasília




------------------------------------------------------------------------------
Links do Yahoo! Grupos

a.. Para visitar o site do seu grupo na web, acesse:
http://br.groups.yahoo.com/group/ciencialist/

b.. Para sair deste grupo, envie um e-mail para:
ciencialist-unsubscribe@yahoogrupos.com.br

c.. O uso que você faz do Yahoo! Grupos está sujeito aos Termos do Serviço do Yahoo!.



[As partes desta mensagem que não continham texto foram removidas]



SUBJECT: Re: [ciencialist] Fw: Problema basico de volumes
FROM: "Oraculo" <oraculo@atibaia.com.br>
TO: <ciencialist@yahoogrupos.com.br>
DATE: 22/03/2005 16:15

Olá Luiz

Acho que o enfoque do Taborda será a única forma de convencer cunhado em questão..:-) Se ele puder perceber que o volume do recipiente, mesmo vazio (e mesmo no vácuo) ainda será de um litro, talvez perceba a incongruencia de afirmar que tem um litro de água e menos de shampoo. Me parece que, apenas com calculos e definições, será mais dificil, já que deve faltar a base de conhecimento necessária para absorver esse tipo de evidencia ou argumento.

Talvez exista também um problema extra, que é a verificaçào que organismos de controle fazem com produtos destinados ao consumo público. A água, ao preencher o recipiente, ocupará todo o volume do mesmo. Mas outros tipos de produto podem deixar espaços ou mesmo conter ar, como no caso de shampoo, ao ser inserido com maquinas de pressão automáticas.

É possível que os problemas do cunhado com o instituto de medição e controle se deva a metodologia, por exemplo, processar o shampoo contido na embalagem de forma a retirar todo o ar (bolhas) e só então verificar o volume. Em alguns casos, principalmente com aparelhos de preenchimento da industria descalibrados ou fora de ajuste, as bolhas podem representar um volume significativo, e causar a multa do fabricante.

Isso pode ter sido interpretado pelo cunhado como sendo uma diferença real de volume entre água e o shampoo na embalagem.

A abordagem do Taborda pode faze-lo compreender que um litro de água e um litro de shampoo contém ambos o mesmo volume, mas de qualquer coisa que estiver dentro do recipiente, seja apenas agua, apenas shampoo ou shampoo e ar (em bolhas ou diluido) e que foi isso que causou seus problemas com a medição.

Um abraço.

Homero




----- Original Message -----
From: Sergio M. M. Taborda
To: ciencialist@yahoogrupos.com.br
Sent: Tuesday, March 22, 2005 3:48 PM
Subject: Re: [ciencialist] Fw: Problema basico de volumes


Luiz Ferraz Netto wrote:

> Quem quer fazer a primeira redação da resposta a essa pergunta?
>
> []'
> ===========================
> Luiz Ferraz Netto [Léo]
> leobarretos@uol.com.br
> http://www.feiradeciencias.com.br
> ===========================
> -----Mensagem Original-----
> De: nunofigueiredo
> Para: leobarretos
> Enviada em: terça-feira, 22 de março de 2005 02:00
> Assunto: Problema basico de volumes
>
>
> Prezado professor:
> Achei o seu site muitop interessante. Assim, estoui a lhe escrever um
> função de uma duvida.
> O meu cunhado tem uma fabrica de cosmeticos e, surgiu uma discussão
> entre nós: Ele insiste, que em determiado vasilhame que tem a
> capacidade para 4 litros, recebe 4 litros de água, mas uma quantidade
> diferente (menor) de shampoo.
> Por mais que eu tentasse explicar que o litro é uima unidade
> universal(não varia com o tipo de produto) por se tratar de uma medida
> de volume, ele insiste no erro.
> Eu expliquei-lhe que o peso sim, varia conforme o produto por causa da
> densidade, mas o volume jamais muda. Ele aceitou que o peso varia, mas
> o volume também.
> Eu argumentei entre outras, que não poderiamos ter duas variaveis para
> o mesmo produto, pois tornar-se-ia impossivel calcular uma em função
> da outra... Temtei explicar que um litro de agua ou um litro de chumbo
> derretido sao um litro. Apenas um pesa aproximadamente um Kg e o outro
> vários Kg.
> Ele insiste que está certo e que já foi autoado pelo inmetro mais de
> uma vês por isso. Eu, irritado disse que não era possível e, que se o
> inmetro multou porque um litro não era um litro, o inmetro estava
> errado. É obvio que ele foi autoado por diferenças no calculo do peso
> liquido e nao no volume em l ou ml.
> Bem, resumindo, fui incapaz de lhe explicar isso, apesar da minha
> formação superior em física (incompleta).
> Achei qe o tema é interessante o suficiente para o expor ao senhor. Se
> não for pedir demais, gostaria de uma opinião para a apresentar.
> Talvez o problema seja apresentar de forma compreensivel a definiçao
> de litro. Sem se levar em consideração os gases por serem fluidos
> compressiveis, e claro.
> Agradeço de antemão a sua resposta,
>

O volume de um recipiente (vazilhanme) é constante. O recipiente suporta
o mesmo volume de qualquer substancia. Parece que é isto que o seu
cunhado falha em entender. O volume do recipiente não depende do liquido
que ele contém, pois o volume é sempre o mesmo, mesmo que o recipiente
estiver vazio.
O volume do recipiente é calculado geometricamente. Existem formulas
matemáticas para o calcular para recipientes simples. Por exemplo, um
cilindro tem um volume igual ao produto da altura pela area da base.
Esta formula é a base dos medidores de volume. Que são normalmente
cilindros transparentes com escalas do lado de fora.
Se a forma do recipiente não é simples de corresponder com solidos
geometricos simples - como cilindros, cubos e esfera - então existe um
truque, que é encher o recipiente com água e despejar essa água num
recepiente do qual sabemos o volume. (se possivel um recepiente calibrado).
Sendo que o volume do recipiente depende apenas da forma do recipiente e
não do que ele contém, o seu volume é constante e não varia conforme o
liquido que se coloca nele.


Sérgio Taborda








##### ##### #####

Para saber mais visite
http://www.ciencialist.hpg.ig.com.br


##### ##### ##### #####


Yahoo! Grupos, um serviço oferecido por:







------------------------------------------------------------------------------
Links do Yahoo! Grupos

a.. Para visitar o site do seu grupo na web, acesse:
http://br.groups.yahoo.com/group/ciencialist/

b.. Para sair deste grupo, envie um e-mail para:
ciencialist-unsubscribe@yahoogrupos.com.br

c.. O uso que você faz do Yahoo! Grupos está sujeito aos Termos do Serviço do Yahoo!.



[As partes desta mensagem que não continham texto foram removidas]



SUBJECT: Um pedido - mais uma vez
FROM: "junior_br2001" <junior_br2001@yahoo.com.br>
TO: ciencialist@yahoogrupos.com.br
DATE: 22/03/2005 16:24


Comom faz, qual editor uso para poder passar calculos e simbolos
matemáticos para esta lista?

JR





SUBJECT: Re: [ciencialist] Um pedido - mais uma vez
FROM: TARCISIO BORGES <tbs97@fisica.ufpr.br>
TO: ciencialist@yahoogrupos.com.br
DATE: 22/03/2005 16:26

Use apenas o formato ASCII. [texto puro]

[]s
TARCISIO BORGES
tbs97@fisica.ufpr.br

On Tue, 22 Mar 2005, junior_br2001 wrote:
> Comom faz, qual editor uso para poder passar calculos e simbolos
> matemáticos para esta lista?


SUBJECT: Re: Um pedido - mais uma vez
FROM: "junior_br2001" <junior_br2001@yahoo.com.br>
TO: ciencialist@yahoogrupos.com.br
DATE: 22/03/2005 16:27


Como faço, onde copio isso? Aonde encontro isso?

JR

--- Em ciencialist@yahoogrupos.com.br, TARCISIO BORGES <tbs97@f...>
escreveu
> Use apenas o formato ASCII. [texto puro]
>
> []s
> TARCISIO BORGES
> tbs97@f...
>
> On Tue, 22 Mar 2005, junior_br2001 wrote:
> > Comom faz, qual editor uso para poder passar calculos e simbolos
> > matemáticos para esta lista?





SUBJECT: Re: [ciencialist] Exercitando a Dialética (era Fixismo...)
FROM: "Alberto Mesquita Filho" <albmesq@uol.com.br>
TO: <ciencialist@yahoogrupos.com.br>
DATE: 22/03/2005 16:32

----- Original Message -----
From: "Oraculo"
Sent: Tuesday, March 22, 2005 2:29 AM
Subject: [ciencialist] Fixismo - nova alegação insustentável..:-) (era manto
de turim e céticos)

Caríssimos Homero e Júnior

Na msg acima citada, que pode ser lida em
http://br.groups.yahoo.com/group/ciencialist/message/45252 flagrei um
aspecto em que vocês concordam, em meio a muitas discordâncias que foram
propositalmente cortadas. Trata-se do seguinte:

Júnior: [...] fixismo [...] é muito ruim [...] em ciencia.

Homero: [...] tudo o que a ciência tem de melhor é a mudança e a adaptação a
novos conhecimentos e dados.

Em condições normais eu concordaria com ambos e nem iria me dar conta sobre
a mais remota possibilidade de vir a discordar de ambos. Não obstante, em
meio a tantas outras discordâncias misturadas a esse que seria, a meu ver, o
argumento principal de ambos, captei algo que em condições normais eu não
teria dado conta. Vocês dois estão generalizando, e isso não costuma ser
muito bom em ciência. Perceba, Júnior, como construi esta última frase e
compare-a com a sua ("não costuma ser muito bom" no lugar de "é muito ruim).
Repare, Homero, no "tudo" de sua frase. Será que essas generalizações se
sustentam? Talvez sim, talvez não, talvez... quem sabe? Espero que vocês não
fiquem chateados pelo fato de eu estar aqui fazendo o papel de advogado do
diabo e/ou exercitando a minha dialética de botequim.

Eu costumo dizer que a ciência é eminentemente realista. Há quem não
concorde com isso, mas a grande maioria dos cientistas, dentre os mais
respeitáveis, age como se isso fosse uma verdade e alguns filósofos da
ciência, também dentre os mais respeitáveis, concordariam comigo. Quanto aos
primeiros, o exemplo típico é Einstein, e quanto aos últimos eu citaria Karl
Popper e Mario Bunge. Em meados do século passado surgiu uma polêmica muito
intensa sobre o realismo da física, pois assumiam alguns que a teoria
quântica não comportava esse realismo. Não obstante, muitos dentre os
respeitáveis físicos quânticos (sim, sou eu que estou escrevendo isso, por
incrível que possa parecer) perceberam que poderiam assimilar o realismo sem
despersonalisar a teoria quântica. Fala-se hoje sem o menor ressentimento,
em realismo quântico, no qual os "universais" seriam os pares conjugados. Ou
seja, até mesmo a fantasmagórica dualidade pode ser encarada
realisticamente, bastando que se considere aquele esquisito "dual" como
sendo uma unidade, mas isso é o que sempre foi feito, pois pelo princípio da
complementaridade de Bohr os dois membros do par não podem ser isolados num
mesmo contexto experimental. Logo não existem como "dois", mas como "um" que
se manifesta ora de uma maneira, ora de outra, mas sempre a caracterizar "um
universal".

Pois bem. O que tem isso a ver com o fixismo? Eu diria que tem tudo. É a
partir de uma postura realista que podemos estabelecer uma regra fundamental
para a ciência e a se apoiar na afirmação milenar a dizer que "os princípios
fundamentais são universais". Ou seja, os princípios fundamentais hoje serão
os mesmos amanhã, e os princípios universais verificados aqui na Terra serão
os mesmos verificados em Saturno. Será que isso é verdade? Talvez sim,
talvez não, mas a verdade é que se isso não for verdade, a regra da
repetibilidade se desfaz, e a ciência experimental não mais se justificará
como tal. Vocês já imaginaram? Hoje eu faço uma experiência e obtenho um
resultado, amanhã eu repito a mesma experiência, procurando observar o mesmo
cenário, e obtenho um resultado a contradizer o primeiro!!! Mas isso não
seria concordar que a ciência apóia-se num fantasmagórico fixismo? (Nota:
não encontrei essa palavra "fixismo" no dicionário, mas estou mantendo a
forma pois entendi seu significado). Sem dúvida estamos frente a um "a
priori" e/ou um elemento transcendental no estilo Kant, daí eu ter empregado
o termo "fantasmagórico", pois os cientistas via de regra não gostam muito
desses "a priori". Mas, gostando ou não, não há como se falar em ciência sem
se dar conta do terreno pantanoso em que seus alicerces foram fincados.
Alguns chamam a esse terreno de metafísica, enquanto outros abominam a
metafísica sem colocar nada no lugar. Não sou contra nem a favor da
metafísica, simplesmente procuro fazer a minha ciência a partir dessa
postura realista a justificar a regra científica fundamental. Esse assunto é
discutido em maior profundidade no meu artigo "A ciência e seus princípios",
que está em http://ecientificocultural.com/ECC2/FilCien/cap01.htm

Vamos complicar um pouquinho mais. Falou-se muito durante os debates
ocorridos nesta thread (por sinal, com várias subdivisões) em método
científico. Pergunto: o método científico de hoje é diferente do método
científico dos tempos de Galileu? E este último é diferente do método
científico dos tempos de Eratóstenes, Zenão ou Arquimedes? Muitos pensarão
que estou confundindo as coisas, pois o primeiro método científico teria
sido descrito por Descartes ou por Francis Bacon, logo não há como falar em
método científico nos tempos de Arquimedes. Há muita gente boa que pensa
assim, mas há também muita gente boa que pensa o contrário: Que o método não
evolui no decorrer do tempo, o que evolui não é o método, mas tão somente a
nossa concepção do que seja esse método. Em outras palavras eu costumo dizer
que embora o método seja importante para a ciência, "a ciência enquanto tal
evolui independentemente da existência do método, da mesma maneira que os
planetas circulam ao redor do Sol, indiferentes à existência ou não das leis
da mecânica ou da gravitação." Ou então: "o cientista, consciente ou
inconscientemente, quer queira, quer não queira, ao produzir conhecimentos
científicos utiliza-se do método científico, cabendo ao estudioso em teoria
do conhecimento decodificar ou decifrar esse método". Essa postura não deixa
de ser uma extensão do realismo acima exposto, se bem que, e como disse
acima, tem muita gente boa que não aceita esse extensionismo. De qualquer
forma, deixo à reflexão de vocês mais este "fixismo" relacionado a um tema
importantíssimo e fundamental para a ciência, qual seja, o método
científico.

Nada contra a evolução. Nada contra o caráter mutável da ciência. Nada
contra a dialética de Heráclito. Digo apenas, parafraseando Heráclito, que
"um rio nunca é o mesmo de um dia para outro", mas vou além: o leito do rio
tem muito de perene e os princípios e leis que fazem a água correr *parecem
ser* os mesmos, desde que o Universo existe como tal.

Pergunto agora: 1) O fixismo é muito ruim para a ciência? 2) Tudo o que a
ciência tem de melhor é a mudança?

Pela primeira vez flagrei vocês concordando com alguma coisa... Sobre a qual
discordo ;-)))))))))))))), pelo menos num contexto generalizante.

[ ]´s
Alberto
http://ecientificocultural.com/indice.htm
Mas indiferentemente a tudo isso, o neutrino tem massa, o elétron não é
uma carga elétrica coulombiana e a Terra se move. E a história se repetirá.



SUBJECT: Re: Um pedido - mais uma vez
FROM: "junior_br2001" <junior_br2001@yahoo.com.br>
TO: ciencialist@yahoogrupos.com.br
DATE: 22/03/2005 16:50


--- Em ciencialist@yahoogrupos.com.br, TARCISIO BORGES <tbs97@f...>
escreveu
> Use apenas o formato ASCII. [texto puro]
>
> []s
> TARCISIO BORGES
> tbs97@f...
>
> On Tue, 22 Mar 2005, junior_br2001 wrote:
> > Comom faz, qual editor uso para poder passar calculos e simbolos
> > matemáticos para esta lista?


JR Nao sei se voce entendeu. Primeiro eu gostaria de saber o que é
ASCII(desculpe minh ignorancia). Depois na verdade o que eu quero é
algum editor em que eu posso colocar os símbolos e os caracteres de
matemática, tipo uma raiz quadrada, um equação, frações, algebra e
simbolos gemométricos para depois envia-los a lista copiando e
colando.

JR





SUBJECT: Re: [ciencialist] Re: Um pedido - mais uma vez
FROM: TARCISIO BORGES <tbs97@fisica.ufpr.br>
TO: ciencialist@yahoogrupos.com.br
DATE: 22/03/2005 17:29

O formato ASCII é o texto que estamos usando.

Para dizer dois mais dois é igual a cinco, digitamos:

2+2=5

Mas se vc quiser usar equações complicadas, então use o latex ou equation
editor e salve como imagem. Coloque a imagem em um site e nos mande o
link.

http://www.hostsrv.com/webmab/app1/MSP?MSPStoreID=MSPStore200559410_2&MSPStoreType=image/gif

[]s
TARCISIO BORGES
tbs97@fisica.ufpr.br

On Tue, 22 Mar 2005, junior_br2001 wrote:

>
>
> Como faço, onde copio isso? Aonde encontro isso?
>
> JR
>
> --- Em ciencialist@yahoogrupos.com.br, TARCISIO BORGES <tbs97@f...>
> escreveu
> > Use apenas o formato ASCII. [texto puro]
> >
> > []s
> > TARCISIO BORGES
> > tbs97@f...
> >
> > On Tue, 22 Mar 2005, junior_br2001 wrote:
> > > Comom faz, qual editor uso para poder passar calculos e simbolos
> > > matemáticos para esta lista?


SUBJECT: ASCII (era Um pedido - mais uma vez)
FROM: "Oraculo" <oraculo@atibaia.com.br>
TO: <ciencialist@yahoogrupos.com.br>
DATE: 22/03/2005 17:38

Olá Pubmed

ASCII é American Standard Code for Information Interchange, o código que define os caracteres que usamos para escrever nos computadores. Por exemplo, se pressionar a tecla Alt e ao mesmo tempo digitar os numeros 167, terá o caractere de numeração, assim º. Se digitar 166, terá o "a" pequeno, para usar em primeira, assim 1ª. Todas as letras podem ser codificadas assim.

São codigos ASCII, padrão em todos os sistemas de computadores. Fórmulas e caracteres especiais tem codificação especial e não podem ser exibidos aqui na lista, que só aceita o formato "texto", ou como disse o Tarciso, ASCII.

Assim, não é posível escrever com caracteres especias na lista.

Um abraço.

Homero

Tabela ASCII
http://www.numaboa.com.br/informatica/oiciliS/tabelas/ascii.php

http://www.e-kit.com.br/tabela_ascii_print.asp?i=0&f=255

----- Original Message -----
From: junior_br2001
To: ciencialist@yahoogrupos.com.br
Sent: Tuesday, March 22, 2005 4:50 PM
Subject: [ciencialist] Re: Um pedido - mais uma vez



--- Em ciencialist@yahoogrupos.com.br, TARCISIO BORGES <tbs97@f...>
escreveu
> Use apenas o formato ASCII. [texto puro]
>
> []s
> TARCISIO BORGES
> tbs97@f...
>
> On Tue, 22 Mar 2005, junior_br2001 wrote:
> > Comom faz, qual editor uso para poder passar calculos e simbolos
> > matemáticos para esta lista?


JR Nao sei se voce entendeu. Primeiro eu gostaria de saber o que é
ASCII(desculpe minh ignorancia). Depois na verdade o que eu quero é
algum editor em que eu posso colocar os símbolos e os caracteres de
matemática, tipo uma raiz quadrada, um equação, frações, algebra e
simbolos gemométricos para depois envia-los a lista copiando e
colando.

JR





##### ##### #####

Para saber mais visite
http://www.ciencialist.hpg.ig.com.br


##### ##### ##### #####


Yahoo! Grupos, um serviço oferecido por:
PUBLICIDADE




------------------------------------------------------------------------------
Links do Yahoo! Grupos

a.. Para visitar o site do seu grupo na web, acesse:
http://br.groups.yahoo.com/group/ciencialist/

b.. Para sair deste grupo, envie um e-mail para:
ciencialist-unsubscribe@yahoogrupos.com.br

c.. O uso que você faz do Yahoo! Grupos está sujeito aos Termos do Serviço do Yahoo!.



[As partes desta mensagem que não continham texto foram removidas]



SUBJECT: equações
FROM: TARCISIO BORGES <tbs97@fisica.ufpr.br>
TO: ciencialist@yahoogrupos.com.br
DATE: 22/03/2005 17:39

O link não funcionou... quem sabe estes:

http://www.soft4science.com/pics/MathMLControl_2.gif

http://web.uvic.ca/~webct/modules/modules38/equation/equation02.jpg

Ou se vc tiver uma câmera digital:

http://www.hallogram.com/science/mathtype/equation.jpg

[]s
TARCISIO BORGES
tbs97@fisica.ufpr.br


SUBJECT: Re: Um pedido - mais uma vez
FROM: "junior_br2001" <junior_br2001@yahoo.com.br>
TO: ciencialist@yahoogrupos.com.br
DATE: 22/03/2005 17:39


Tarcisio . Obrigado

JR

--- Em ciencialist@yahoogrupos.com.br, TARCISIO BORGES <tbs97@f...>
escreveu
> O formato ASCII é o texto que estamos usando.
>
> Para dizer dois mais dois é igual a cinco, digitamos:
>
> 2+2=5
>
> Mas se vc quiser usar equações complicadas, então use o latex ou
equation
> editor e salve como imagem. Coloque a imagem em um site e nos mande
o
> link.
>
> http://www.hostsrv.com/webmab/app1/MSP?
MSPStoreID=MSPStore200559410_2&MSPStoreType=image/gif
>
> []s
> TARCISIO BORGES
> tbs97@f...
>
> On Tue, 22 Mar 2005, junior_br2001 wrote:
>
> >
> >
> > Como faço, onde copio isso? Aonde encontro isso?
> >
> > JR
> >
> > --- Em ciencialist@yahoogrupos.com.br, TARCISIO BORGES
<tbs97@f...>
> > escreveu
> > > Use apenas o formato ASCII. [texto puro]
> > >
> > > []s
> > > TARCISIO BORGES
> > > tbs97@f...
> > >
> > > On Tue, 22 Mar 2005, junior_br2001 wrote:
> > > > Comom faz, qual editor uso para poder passar calculos e
simbolos
> > > > matemáticos para esta lista?





SUBJECT: Re: equações
FROM: "junior_br2001" <junior_br2001@yahoo.com.br>
TO: ciencialist@yahoogrupos.com.br
DATE: 22/03/2005 17:45


OK. Eu localizei o Latex e o Equation Editor para o Windows no
GOOGLE. Agora resta eu saber onde encontro pro Linux, vou perguntar
na comunidade linux, valeuz..

JR


--- Em ciencialist@yahoogrupos.com.br, TARCISIO BORGES <tbs97@f...>
escreveu
> O link não funcionou... quem sabe estes:
>
> http://www.soft4science.com/pics/MathMLControl_2.gif
>
> http://web.uvic.ca/~webct/modules/modules38/equation/equation02.jpg
>
> Ou se vc tiver uma câmera digital:
>
> http://www.hallogram.com/science/mathtype/equation.jpg
>
> []s
> TARCISIO BORGES
> tbs97@f...





SUBJECT: Re: manto de turim e c�ticos
FROM: Eduardo Gueron <edgueron@yahoo.com>
TO: ciencialist@yahoogrupos.com.br
DATE: 22/03/2005 18:09

Oi Alberto,

Obrigado pela gentileza.

Abragos, Eduardo.

----- Original Message -----
From: "Eduardo Gueron"
Sent: Tuesday, March 22, 2005 1:52 AM
Subject: Re: [ciencialist] manto de turim e citicos

> Na configuragco que uso, recebo resumos da lista ao
invis de
mensagens
> isoladas, nisso acho que me perdi na discussco sobre
curvatura e Rel
> Geral. Alguim poderia me dizer onde comegou?

Comegou em 14 de margo com o Lio. Abaixo, e para quem
interessar, todas
as
mensagens da thread (ati o momento):

Lio:
http://br.groups.yahoo.com/group/ciencialist/message/44888
Alvaro:
http://br.groups.yahoo.com/group/ciencialist/message/44896
JVictor:
http://br.groups.yahoo.com/group/ciencialist/message/45129
Jznior:
http://br.groups.yahoo.com/group/ciencialist/message/45131
Daniel Moser:
http://br.groups.yahoo.com/group/ciencialist/message/45236
PSavio:
http://br.groups.yahoo.com/group/ciencialist/message/45238
PSavio:
http://br.groups.yahoo.com/group/ciencialist/message/45246
Taborda:
http://br.groups.yahoo.com/group/ciencialist/message/45247

[ ]4s
Alberto
--- ciencialist@yahoogrupos.com.br wrote:




__________________________________
Do you Yahoo!?
Yahoo! Mail - You care about security. So do we.
http://promotions.yahoo.com/new_mail


SUBJECT: Re: [ciencialist] Fw: Problema basico de volumes
FROM: "Luiz Ferraz Netto" <leobarretos@uol.com.br>
TO: <ciencialist@yahoogrupos.com.br>
DATE: 22/03/2005 18:27

Olá Oráculo, Taborda e amigos,
li as suas apresentações --- e conclui o que segue --- digam-me se procede:
==================================================
Olá José Nuno,
A "falha" de seu cunhado talvez possa ser sanada se vc apresentar a ele definições e conceitos rigorosamente científicos. Todavia, devo advertir de imediato: seu cunhado pode não estar falhando!
Usar dos termos corretos e não tentar extrapolar tais termos para o uso popular. Serve de exemplo o "trabalho", um conceito físico definido para forças, mas é, sem dúvida, uma palavra que leva em si um forte componente psicológico. É bem possível que seja necessário um esforço maior para disputar uma partida de tênis do que para colocar em ordem alfabética um conjunto de fichas de arquivo, mas nós consideramos a última atividade como um 'trabalho' e a primeira como 'divertimento', 'competição' ou 'exercício', mas nunca como trabalho!. O 'trabalho' da física, pouco ou nada tem a ver com o 'trabalho' da telefonista, do ascensorista etc.
Então, a primeira coisa a ser lapidada, é o conceito de "volume" e suas unidades (oficial e legal). "Volume" é o resultado da medição de um determinado 'espaço' (propriedade geométrica de apresentar três dimensões). Esse 'espaço' pode ser, por exemplo, aquele limitado pelas 4 paredes, teto e chão de uma simples sala. Para a medição desse espaço torna-se necessário que se defina um 'espaço unitário' ao qual se associe a 'unidade de volume'; um cubo de 1 m de aresta tem, por definição, o volume unitário, ou seja, 1 metro cúbico.
Além do 'metro cúbico', que é a unidade oficial de volume, aceita-se, legalmente, a unidade denominada "litro", que corresponde à milésima parte do metro cúbico, ou 1 metro cúbico = 1 000 litros.
Certos depósitos/vasilhames têm capacidade de armazenamento de líquidos, são assim os tanques de gasolina dos postos, os bujões de água, as caixas d'águas domiciliares, as garrafas PETs etc. Essas capacidades de armazenamento serão dadas pelas medidas dos volumes disponíveis desses depósitos/vasilhames. Tem garrafa PET, por exemplo, cuja capacidade de armazenamento (espaço geométrico) é de 2 litros. Tem lata de mantimento que apresentam volume útil de exato 1 litro. Qualquer coisa que vc coloque lá dentro e preencha totalmente o espaço disponível ocupará "por definição" o volume de 1 litro.
Mas, isso não significa que lá exista "1 litro" daquele material!
Vou exemplificar:
Aqui na minha cidade vende-se jabuticabas soltas nas feiras livres; a 'unidade' de comercialização é uma lata de capacidade 1 litro. Qdo vc pede 2 litros de jabuticabas o feirante enche duas vezes aquela lata --- eis "2 litros de jabuticaba".
Você concordará que ali não há "realmente" 2 litros de jabuticaba, uma vez que entre elas tem enorme quantidade de espaço vazio. Seu cunhado tem, sobre o shampoo, a mesma idéia que vc está tendo agora do 'litro de jabuticaba. O 'litro de shampoo' não tem realmente "1 litro de shampoo"!!!!!!

Um shampoo é um produto cujo fórmula e fabricação implica num resultado pouco denso devido à minúsculas gotículas de ar (gases) e isso o coloca em similaridade com a jabuticaba.

Como resolver o problema?

Numa solução 'ideal'o shampoo deveria ser 'envazado' no vácuo (para eliminação das bolhas de ar)--- obviamente teríamos um shampoo mais denso que aquele normalmente vendido por ai. Esse seria o shampoo 'padrão' --- todos os demais shampoos teriam um adjetivo extra: shampoo a 10%, shampoo 30 ... ou sei lá o que.

Então, o dilema deixa de ser dilema entendendo-se que:

(a) "volume" é a especificação de um espaço; é um invariante, independente do que está preenchendo esse espaço. Assim, quer o espaço contenha água, shampoo ou jabuticabas --- o volume é de 1 litro (no exemplo).

(b) "volume" de substâncias porosas é um 'volume aparente'; não há realmente o 'tanto' de substância que se espera. Para tais casos, "apenas" volume, não é grandeza suficiente para especificação da substância.

Aquele abraço,
=================================


===========================
Luiz Ferraz Netto [Léo]
leobarretos@uol.com.br
http://www.feiradeciencias.com.br
===========================


SUBJECT: Re: [ciencialist] Um pedido - mais uma vez
FROM: "Sergio M. M. Taborda" <sergiotaborda@terra.com.br>
TO: ciencialist@yahoogrupos.com.br
DATE: 22/03/2005 18:48

junior_br2001 wrote:

>
> Comom faz, qual editor uso para poder passar calculos e simbolos
> matemáticos para esta lista?

Por que quer fazer isso ? Se a formula é conhecida , deve existir uma
página na net que a tenha.
Se ela é criada por si, pode descrevê-la normalmente com texto.
Quanto a simbolos matemáticos em texto normal, eis alguns

soma, multiplicação, divisão, subtracção , potenciação + * / - ^
exemplo
2^3 = 8
2^(-1) = 0.5

Letras gregas são representadas por seus nomes epsilon, alfa, etc...
exemplo

c ^2 = 1 / (espilon*miu) O quadrado da velocidade da luz no vácuo é
o inverso do produto da permitividade pela permissividade

indices são representados por _ por exemplo x indice 2 seria x_2

epsilon_0 que é a permitividade do vacuo

derivadas

da/dx para derivadas parciais e Da/Dx para derivadas totais.

operadores de campo

divergencia, rotacional e gradiente : div, rot , grad

exemplo, leis de maxwell

div E = rho/epsilon
div B =0
rot E = - dB/dt
rot B = miu(j + dE/dt)


As letras que representam grandezas fisicas são covencionais, por isso
não deve haver problema.

Isso cobre o mais usual.

Mas qual é a formula que pretende mostrar-nos ?




SUBJECT: Re: Um pedido - mais uma vez
FROM: "junior_br2001" <junior_br2001@yahoo.com.br>
TO: ciencialist@yahoogrupos.com.br
DATE: 22/03/2005 18:57


Sérgio, é que eu queria retomar meus estudos de física e preciso de
um editor para expor algums dúvidas por aqui e pelo ORKUT. de
qualquer forma estou com um editor aqui que copiei.
E muito obrigado pelas suas dicas, elas me serão muito úteis. Já
salvei as suas dicas numa pasta separado. OK

Muito obrigado

JR


--- Em ciencialist@yahoogrupos.com.br, "Sergio M. M. Taborda"
<sergiotaborda@t...> escreveu
> junior_br2001 wrote:
>
> >
> > Comom faz, qual editor uso para poder passar calculos e simbolos
> > matemáticos para esta lista?
>
> Por que quer fazer isso ? Se a formula é conhecida , deve existir
uma
> página na net que a tenha.
> Se ela é criada por si, pode descrevê-la normalmente com texto.
> Quanto a simbolos matemáticos em texto normal, eis alguns
>
> soma, multiplicação, divisão, subtracção , potenciação + * / - ^
> exemplo
> 2^3 = 8
> 2^(-1) = 0.5
>
> Letras gregas são representadas por seus nomes epsilon, alfa, etc...
> exemplo
>
> c ^2 = 1 / (espilon*miu) O quadrado da velocidade da luz no
vácuo é
> o inverso do produto da permitividade pela permissividade
>
> indices são representados por _ por exemplo x indice 2 seria x_2
>
> epsilon_0 que é a permitividade do vacuo
>
> derivadas
>
> da/dx para derivadas parciais e Da/Dx para derivadas totais.
>
> operadores de campo
>
> divergencia, rotacional e gradiente : div, rot , grad
>
> exemplo, leis de maxwell
>
> div E = rho/epsilon
> div B =0
> rot E = - dB/dt
> rot B = miu(j + dE/dt)
>
>
> As letras que representam grandezas fisicas são covencionais, por
isso
> não deve haver problema.
>
> Isso cobre o mais usual.
>
> Mas qual é a formula que pretende mostrar-nos ?





SUBJECT: Re: [ciencialist] Gravitons
FROM: "Luiz Ferraz Netto" <leobarretos@uol.com.br>
TO: <ciencialist@yahoogrupos.com.br>
DATE: 22/03/2005 19:15

Vichi!!! Essa mensagem do JVictor escapou-me de ser lida --- ??? ---, mas, em tempo, já vou avisando que não concordo com o princípio da equivalência como está apresentado.

Notoriamente ele fura para os líquidos.

Escrevi algo sobre isso (e os mais antigos de C-list já participaram disso) na Sala 07, em:

http://www.feiradeciencias.com.br/sala07/07_T04.asp

O empuxo de Newton nos referenciais acelerados.

[]'
===========================
Luiz Ferraz Netto [Léo]
leobarretos@uol.com.br
http://www.feiradeciencias.com.br
===========================
-----Mensagem Original-----
De: "JVictor" <jvoneto@uol.com.br>
Para: <ciencialist@yahoogrupos.com.br>
Enviada em: segunda-feira, 21 de março de 2005 19:44
Assunto: [ciencialist] Gravitons



Luiz Ferraz Netto escreveu:

Apreciaria comentários sobre o seguinte texto:

http://rolfguthmann.sites.uol.com.br/TQG/tqg.html


Professor Luiz Ferraz,

> Lí o artigo até o quinto parágrafo da seção 2. E parei. Até chegar
> ao item 2 encontramos hipóteses absolutamente estranhas e sem
> significado físico, pelo que já peço ao autor que seja mais claro..
>
> Com relação ao item 2- O que é gravidade?, transcrevo dois conceitos
> absolutamente equivocados, que comento rapidamente logo após o destaque.

1. " Para Einstein, a gravidade causava uma deformação no espaço tempo
contínuo e, com esta idéia, ele desenvolveu uma álgebra muito complexa
que a descreve apenas geometricamente."

Comentário: 1) É o contrário: a gravidade não é causa; é efeito da
curvatura do espaço. 2) Einstein NÃO desenvolveu nenhuma álgebra muito
complexa. Ele usou um formalismo matemático, novo na época, criado por
outros matemáticos, baseado em geometria diferencial, formalismo esse
de jeito nenhum complexo. Desculpe-me o autor, mas esse é o tipo do
erro conceitual inadmissível para alguém que se propõe a elaborar uma
nova teoria.

> 2. - " Uma outra evidência se reporta a teoria da relatividade de
> Einstein, que está embasada no princípio da equivalência o qual
> estabelece que a massa, ou a masssa inercial e o peso, ou massa
> gravitacional de materiais diferentes sofrem a mesma aceleração... "

Comentários: Os erros conceituais aquí são para ninguém botar
defeito!. Massa e peso são coisas diferentes!. E esse princípio de
equivalência é devido a Galileu e é chamado Princípio de Equivalência
fraco. Einstein o estendeu a toda a física, e o PE de Einstein tem tem
a seguinte forma:

" Um laboratório local, não girante, em queda livre, é equivalente, para
a realização de qualquer experiência, a um referencial inercial numa
região do espaço sem gravidade." Esta é a forma forte do Princípio de
Equivalência, devido a Einstein. É o mesmo que dizer: se você estiver em
uma nave, no espaço, e esta estiver sob uma aceleração, digamos, de
10m/s/s, você será empurrado contra o piso da nave e não saberá dizer
se essa aceleração é devida aos motores que a provocam ou se é devida
aos efeitos gravitacionais de alguma massa grande nas proximidades,
como, por exemplo, a terra(cujos efeitos gravitacionais, que todos
conhecemos, expressam-se por uma aceleração, o g= aprox. a 10m/s/s e nos
pregados no chão, do mesmo jeito que o astronauta em relação ao piso da
nave, com tudo que tem direito!) ou outro planeta. Ou seja, essas
aceleraçãos são equivalentes!. Na nave, uma balança dirá que o seu peso
será, digamos, 700kg.m/s/s, se for um gordinho, como eu, de 70 Kg de
massa. Se os motores aumentarem a aceleração, a balança vai acusar um
peso maior. Na terra, se consideramos que aceleração gravitacional tem
o mesmo valor, aprx. 10m/s/s, uma balança indicará o mesmo peso para
você, ou seja 700kg.m/s/s! Refletindo sobre coisas assim, num momento
de inspiração, Einstein chegou ao seu racicínio mais feliz, segundo
disse: concluiu que uma pessoa em queda livre não sente seu próprio
peso! Ou seja, para alguém que cai em queda livre, não existe, ao menos
nas vizinhanças, qualquer campo gravitacional. Com esse conceito,
Einstein mostra também que a gravidade é uma coisa cuja existência é
relativa. Independe da constituição dos corpos. Aliás, o PE fraco já diz
a mesma coisa. É esta a base da TRG! Compare, então, o que expressei
acima, com suas afirmações abaixo e decida o autor se ainda mantém:

a) sua hipótese, item 1, que não me animei a comentar(idem, outras, de
mesma qualidade):
"Veremos que a a gravidade é o resultado do desequilíbrio entre força
coloumbiana e força centrípeta",

b) ou com esta, quinto parágrafo do item 2:
" ...podemos afirmar que a gravidade é uma inércia, e é causada por uma
força eletromagnética de origem nuclear, estes indícios( (??),
estupefação minha)) ainda apontam, que a diferença relativa das forças
eletrostática e centrífuga nos átomos, é a fonte ou causa da
gravidade..."
Comentário: não era centrípeta?. Só lembro uma coisinha adicional: força
centrípeta e força centrífuga são aplicadas em corpos diferentes,
portanto, uma não é reação à outra, e a centrífuga surge em alguns
sistemas de referência e outras não! Logo..." Pode esclarecer o
seguinte enunciado, de vossa lavra?: " ...podemos afirmar que gravidade
é uma inércia, e é causada por uma força eletromagnética de origem
nuclear".

c) todo o resto que não lí...

Não podendo ir mais além, por conta do " impacto" das coisas acima, e
logo no início do trabalho(!), sugiro fortemente que o autor, se
estiver com vontade e for corajoso, como parece que o é, publicando
esta, desenvolva SUA OUTRA TEORIA. Quem nem lí ainda!

Sds,

Victor.







##### ##### #####

Para saber mais visite
http://www.ciencialist.hpg.ig.com.br


##### ##### ##### #####
Links do Yahoo! Grupos










--
No virus found in this incoming message.
Checked by AVG Anti-Virus.
Version: 7.0.308 / Virus Database: 266.7.3 - Release Date: 15/03/2005




SUBJECT: Fw: Minutos para hora
FROM: "Luiz Ferraz Netto" <leobarretos@uol.com.br>
TO: "ciencialist" <ciencialist@yahoogrupos.com.br>
DATE: 22/03/2005 19:27

Não uso Excel, nem sei o que é isso, então .......

[]'
===========================
Luiz Ferraz Netto [Léo]
leobarretos@uol.com.br
http://www.feiradeciencias.com.br
===========================
-----Mensagem Original-----
De: Carlos Moraes
Para: leobarretos@uol.com.br
Enviada em: terça-feira, 22 de março de 2005 17:21
Assunto: Minutos para hora


Professor,
não estou conseguindo converter em uma planilha excel minutos para hora.
O Sr tem alguma sugestão?

Carlos

Santa Maria RS

carlos.pauta@terra.com.br
msn carlosmoraes_sm@hotmail.com

Nas dúvidas experimentais, por gentileza coloque aqui o endereço da página, isso facilita o confronto. Agradeço. Meu nome é LUIZ FERRAZ NETTO, meu apelido é LÉO e moro em BARRETOS; dai vem meu e-mail: leobarretos@uol.com.br.


--------------------------------------------------------------------------------


Internal Virus Database is out-of-date.
Checked by AVG Anti-Virus.
Version: 7.0.308 / Virus Database: 266.7.3 - Release Date: 15/03/2005


[As partes desta mensagem que não continham texto foram removidas]



SUBJECT: Re: Exercitando a Dialética (era Fixismo...)
FROM: "junior_br2001" <junior_br2001@yahoo.com.br>
TO: ciencialist@yahoogrupos.com.br
DATE: 22/03/2005 19:49


Oi Alberto. É óbvio que eu nao estava falando dos principios
universais que sempre serão a mesma a ontem, hoje e amanhã - embora
possa-se mudar o paradigma. Não se pode ir contra os fatos

O fixismo a que eu referia é a negação dos novos experimentos, mesmo
que o primeiro no caso do manto seja relativamente confiável. Não é o
caso de se rejeitar outros experimentos só porque o primeiro ainda
oferece um "certo conforto ou segurança" por ser "mais confiável"(o
que nao é o mesmo que infalível e imunes a dúvidas).
Se existem outras evidencias elas devem ser levadas em consideração,
mesmo porque elas pode servir de base a outros experimentos, e não
serem simplesmente refutadas.

Mas mesmo, acho que voce fez em belo texto em cima de toda essa
discussão. Não há como negar.

Como essa discussão parece estar encerrada. devo abster-me de dizer
mais sobre o Sudário, abrindo espaço para outras discussões

JR


--- Em ciencialist@yahoogrupos.com.br, "Alberto Mesquita Filho"
<albmesq@u...> escreveu
> ----- Original Message -----
> From: "Oraculo"
> Sent: Tuesday, March 22, 2005 2:29 AM
> Subject: [ciencialist] Fixismo - nova alegação insustentável..:-)
(era manto
> de turim e céticos)
>
> Caríssimos Homero e Júnior
>
> Na msg acima citada, que pode ser lida em
> http://br.groups.yahoo.com/group/ciencialist/message/45252 flagrei
um
> aspecto em que vocês concordam, em meio a muitas discordâncias que
foram
> propositalmente cortadas. Trata-se do seguinte:
>
> Júnior: [...] fixismo [...] é muito ruim [...] em ciencia.
>
> Homero: [...] tudo o que a ciência tem de melhor é a mudança e a
adaptação a
> novos conhecimentos e dados.
>
> Em condições normais eu concordaria com ambos e nem iria me dar
conta sobre
> a mais remota possibilidade de vir a discordar de ambos. Não
obstante, em
> meio a tantas outras discordâncias misturadas a esse que seria, a
meu ver, o
> argumento principal de ambos, captei algo que em condições normais
eu não
> teria dado conta. Vocês dois estão generalizando, e isso não
costuma ser
> muito bom em ciência. Perceba, Júnior, como construi esta última
frase e
> compare-a com a sua ("não costuma ser muito bom" no lugar de "é
muito ruim).
> Repare, Homero, no "tudo" de sua frase. Será que essas
generalizações se
> sustentam? Talvez sim, talvez não, talvez... quem sabe? Espero que
vocês não
> fiquem chateados pelo fato de eu estar aqui fazendo o papel de
advogado do
> diabo e/ou exercitando a minha dialética de botequim.
>
> Eu costumo dizer que a ciência é eminentemente realista. Há quem não
> concorde com isso, mas a grande maioria dos cientistas, dentre os
mais
> respeitáveis, age como se isso fosse uma verdade e alguns filósofos
da
> ciência, também dentre os mais respeitáveis, concordariam comigo.
Quanto aos
> primeiros, o exemplo típico é Einstein, e quanto aos últimos eu
citaria Karl
> Popper e Mario Bunge. Em meados do século passado surgiu uma
polêmica muito
> intensa sobre o realismo da física, pois assumiam alguns que a
teoria
> quântica não comportava esse realismo. Não obstante, muitos dentre
os
> respeitáveis físicos quânticos (sim, sou eu que estou escrevendo
isso, por
> incrível que possa parecer) perceberam que poderiam assimilar o
realismo sem
> despersonalisar a teoria quântica. Fala-se hoje sem o menor
ressentimento,
> em realismo quântico, no qual os "universais" seriam os pares
conjugados. Ou
> seja, até mesmo a fantasmagórica dualidade pode ser encarada
> realisticamente, bastando que se considere aquele esquisito "dual"
como
> sendo uma unidade, mas isso é o que sempre foi feito, pois pelo
princípio da
> complementaridade de Bohr os dois membros do par não podem ser
isolados num
> mesmo contexto experimental. Logo não existem como "dois", mas
como "um" que
> se manifesta ora de uma maneira, ora de outra, mas sempre a
caracterizar "um
> universal".
>
> Pois bem. O que tem isso a ver com o fixismo? Eu diria que tem
tudo. É a
> partir de uma postura realista que podemos estabelecer uma regra
fundamental
> para a ciência e a se apoiar na afirmação milenar a dizer que "os
princípios
> fundamentais são universais". Ou seja, os princípios fundamentais
hoje serão
> os mesmos amanhã, e os princípios universais verificados aqui na
Terra serão
> os mesmos verificados em Saturno. Será que isso é verdade? Talvez
sim,
> talvez não, mas a verdade é que se isso não for verdade, a regra da
> repetibilidade se desfaz, e a ciência experimental não mais se
justificará
> como tal. Vocês já imaginaram? Hoje eu faço uma experiência e
obtenho um
> resultado, amanhã eu repito a mesma experiência, procurando
observar o mesmo
> cenário, e obtenho um resultado a contradizer o primeiro!!! Mas
isso não
> seria concordar que a ciência apóia-se num fantasmagórico fixismo?
(Nota:
> não encontrei essa palavra "fixismo" no dicionário, mas estou
mantendo a
> forma pois entendi seu significado). Sem dúvida estamos frente a
um "a
> priori" e/ou um elemento transcendental no estilo Kant, daí eu ter
empregado
> o termo "fantasmagórico", pois os cientistas via de regra não
gostam muito
> desses "a priori". Mas, gostando ou não, não há como se falar em
ciência sem
> se dar conta do terreno pantanoso em que seus alicerces foram
fincados.
> Alguns chamam a esse terreno de metafísica, enquanto outros
abominam a
> metafísica sem colocar nada no lugar. Não sou contra nem a favor da
> metafísica, simplesmente procuro fazer a minha ciência a partir
dessa
> postura realista a justificar a regra científica fundamental. Esse
assunto é
> discutido em maior profundidade no meu artigo "A ciência e seus
princípios",
> que está em http://ecientificocultural.com/ECC2/FilCien/cap01.htm
>
> Vamos complicar um pouquinho mais. Falou-se muito durante os debates
> ocorridos nesta thread (por sinal, com várias subdivisões) em método
> científico. Pergunto: o método científico de hoje é diferente do
método
> científico dos tempos de Galileu? E este último é diferente do
método
> científico dos tempos de Eratóstenes, Zenão ou Arquimedes? Muitos
pensarão
> que estou confundindo as coisas, pois o primeiro método científico
teria
> sido descrito por Descartes ou por Francis Bacon, logo não há como
falar em
> método científico nos tempos de Arquimedes. Há muita gente boa que
pensa
> assim, mas há também muita gente boa que pensa o contrário: Que o
método não
> evolui no decorrer do tempo, o que evolui não é o método, mas tão
somente a
> nossa concepção do que seja esse método. Em outras palavras eu
costumo dizer
> que embora o método seja importante para a ciência, "a ciência
enquanto tal
> evolui independentemente da existência do método, da mesma maneira
que os
> planetas circulam ao redor do Sol, indiferentes à existência ou não
das leis
> da mecânica ou da gravitação." Ou então: "o cientista, consciente ou
> inconscientemente, quer queira, quer não queira, ao produzir
conhecimentos
> científicos utiliza-se do método científico, cabendo ao estudioso
em teoria
> do conhecimento decodificar ou decifrar esse método". Essa postura
não deixa
> de ser uma extensão do realismo acima exposto, se bem que, e como
disse
> acima, tem muita gente boa que não aceita esse extensionismo. De
qualquer
> forma, deixo à reflexão de vocês mais este "fixismo" relacionado a
um tema
> importantíssimo e fundamental para a ciência, qual seja, o método
> científico.
>
> Nada contra a evolução. Nada contra o caráter mutável da ciência.
Nada
> contra a dialética de Heráclito. Digo apenas, parafraseando
Heráclito, que
> "um rio nunca é o mesmo de um dia para outro", mas vou além: o
leito do rio
> tem muito de perene e os princípios e leis que fazem a água correr
*parecem
> ser* os mesmos, desde que o Universo existe como tal.
>
> Pergunto agora: 1) O fixismo é muito ruim para a ciência? 2) Tudo o
que a
> ciência tem de melhor é a mudança?
>
> Pela primeira vez flagrei vocês concordando com alguma coisa...
Sobre a qual
> discordo ;-)))))))))))))), pelo menos num contexto generalizante.
>
> [ ]´s
> Alberto
> http://ecientificocultural.com/indice.htm
> Mas indiferentemente a tudo isso, o neutrino tem massa, o elétron
não é
> uma carga elétrica coulombiana e a Terra se move. E a história se
repetirá.





SUBJECT: Sobre a Evidencia
FROM: "junior_br2001" <junior_br2001@yahoo.com.br>
TO: ciencialist@yahoogrupos.com.br
DATE: 22/03/2005 19:56


O Luis Fernando Waib criou um grupo de discussão chamado Sobre a
Evidencia

Um grupo destinado à discussão aberta de temas como as religiões, o
obscurantismo, as grandes manipulações mundiais sob a ótica da
evidência científica e do ceticismo crítico.

Sem restrições de credo (importante!), cor, raça ou pseudônimo. Todos
são bem-vindos, a única ressalva se refere ao comportamento dentro do
fórum, que deve ser sempre cordial, sob pena de moderação.

No mais, sejam bem-vindos!

http://br.groups.yahoo.com/group/sobre_a_evidencia








SUBJECT: Fw: [ciencialist] Gravitons
FROM: "PSavio" <psdmo@uol.com.br>
TO: <ciencialist@yahoogrupos.com.br>
DATE: 22/03/2005 20:03


----- Original Message -----
Daniel Moser wrote:

Prof. Luiz,

Minha opinião a respeito desta teoria é: ela é simplesmente SURPREENDENTE!
Achei ela muito interessante, mas ainda não tive tempo de fazer uma
análise cuidadosa da parte "quantitativa". Mas a parte "qualitativa", me
pareceu extraordinária!!!

>Sob os pontos levantados pelo Victor:
1. Para Einstein, a gravidade cria um espaço tempo curvo, e não o
contrário (o texto está ok)

sávio: Não. O Victor está certo. A gravidade é uma manifestação da curvatura do espaçotempo.
O conceito expresso pelo autor da TGQ é errado

>2. Da maneira que estão escritas, AO PÉ DA LETRA, são erros conceituais.
Mas o contexto e as conclusões tiradas pelo autor demonstram um correto
conhecimento de todos os conceitos envolvidos no trecho.

sávio: você leu e refletiu sobre tudo que foi escrito? Vou esperar que você leia e analise todo o trabalho do autor.
Depois, você volta aquí e comenta a respeito.

>As alegações a) e b) não me parecem de alguém que está realmente
interessado em conhecer todas as possibilidades da natureza e da
ciência.

sávio: Eu não acho que alguém do nível científico e intelectual do articulador ora criticado se enquadre nisso.

>Por acaso o autor destes comentários está TOTALMENTE a par dos
mecanismos que formam os átomos? Como os elétrons se "fixam " e se
mantém estáveis em suas órbitas em torno do núcleo, e como realizam seu
movimento continuamente?

sávio: E se não souber? Isto importaria? Afinal, os efeitos que observamos contrariam fortemente a alegação de que a a gravidade o resultado de um desequilíbrio entre força eletrostática, ou eletromagnética ou nucler e as forças centrípeta e centrífuga do átomo. Onde estão as bases para isso? Victor mostrou de maneira simples e educada

>Pois eu posso dizer que nenhum cientista hoje
possui tal conhecimento, o portanto não podemos desconsiderar a hipótese
do texto "TQG".

sávio: Está certo. Mas concordo com o Victor sobre o autor publicar o outro trabalho de sua autoria, qualquer que seja!... Os dois primeiros capítulos, ou seções têm "conteúdo" suficiente para que eu particularmente não perca com o resto.
Por fim, digo o seguinte: nada de pessoal. São os argumentos.


Pedro Sávio


JVictor escreveu:

> Luiz Ferraz Netto escreveu:
>
> Apreciaria comentários sobre o seguinte texto:
>
> http://rolfguthmann.sites.uol.com.br/TQG/tqg.html
>
>
> Professor Luiz Ferraz,
>
> > Lí o artigo até o quinto parágrafo da seção 2. E parei. Até chegar
> > ao item 2 encontramos hipóteses absolutamente estranhas e sem
> > significado físico, pelo que já peço ao autor que seja mais claro..
> >
> > Com relação ao item 2- O que é gravidade?, transcrevo dois conceitos
> > absolutamente equivocados, que comento rapidamente logo após o destaque.
>
> 1. " Para Einstein, a gravidade causava uma deformação no espaço tempo
> contínuo e, com esta idéia, ele desenvolveu uma álgebra muito complexa
> que a descreve apenas geometricamente."
>
> Comentário: 1) É o contrário: a gravidade não é causa; é efeito da
> curvatura do espaço. 2) Einstein NÃO desenvolveu nenhuma álgebra muito
> complexa. Ele usou um formalismo matemático, novo na época, criado por
> outros matemáticos, baseado em geometria diferencial, formalismo esse
> de jeito nenhum complexo. Desculpe-me o autor, mas esse é o tipo do
> erro conceitual inadmissível para alguém que se propõe a elaborar uma
> nova teoria.
>
> > 2. - " Uma outra evidência se reporta a teoria da relatividade de
> > Einstein, que está embasada no princípio da equivalência o qual
> > estabelece que a massa, ou a masssa inercial e o peso, ou massa
> > gravitacional de materiais diferentes sofrem a mesma aceleração... "
>
> Comentários: Os erros conceituais aquí são para ninguém botar
> defeito!. Massa e peso são coisas diferentes!. E esse princípio de
> equivalência é devido a Galileu e é chamado Princípio de Equivalência
> fraco. Einstein o estendeu a toda a física, e o PE de Einstein tem tem
> a seguinte forma:
>
> " Um laboratório local, não girante, em queda livre, é equivalente, para
> a realização de qualquer experiência, a um referencial inercial numa
> região do espaço sem gravidade." Esta é a forma forte do Princípio de
> Equivalência, devido a Einstein. É o mesmo que dizer: se você estiver em
> uma nave, no espaço, e esta estiver sob uma aceleração, digamos, de
> 10m/s/s, você será empurrado contra o piso da nave e não saberá dizer
> se essa aceleração é devida aos motores que a provocam ou se é devida
> aos efeitos gravitacionais de alguma massa grande nas proximidades,
> como, por exemplo, a terra(cujos efeitos gravitacionais, que todos
> conhecemos, expressam-se por uma aceleração, o g= aprox. a 10m/s/s e nos
> pregados no chão, do mesmo jeito que o astronauta em relação ao piso da
> nave, com tudo que tem direito!) ou outro planeta. Ou seja, essas
> aceleraçãos são equivalentes!. Na nave, uma balança dirá que o seu peso
> será, digamos, 700kg.m/s/s, se for um gordinho, como eu, de 70 Kg de
> massa. Se os motores aumentarem a aceleração, a balança vai acusar um
> peso maior. Na terra, se consideramos que aceleração gravitacional tem
> o mesmo valor, aprx. 10m/s/s, uma balança indicará o mesmo peso para
> você, ou seja 700kg.m/s/s! Refletindo sobre coisas assim, num momento
> de inspiração, Einstein chegou ao seu racicínio mais feliz, segundo
> disse: concluiu que uma pessoa em queda livre não sente seu próprio
> peso! Ou seja, para alguém que cai em queda livre, não existe, ao menos
> nas vizinhanças, qualquer campo gravitacional. Com esse conceito,
> Einstein mostra também que a gravidade é uma coisa cuja existência é
> relativa. Independe da constituição dos corpos. Aliás, o PE fraco já diz
> a mesma coisa. É esta a base da TRG! Compare, então, o que expressei
> acima, com suas afirmações abaixo e decida o autor se ainda mantém:
>
> a) sua hipótese, item 1, que não me animei a comentar(idem, outras, de
> mesma qualidade):
> "Veremos que a a gravidade é o resultado do desequilíbrio entre força
> coloumbiana e força centrípeta",
>
> b) ou com esta, quinto parágrafo do item 2:
> " ...podemos afirmar que a gravidade é uma inércia, e é causada por uma
> força eletromagnética de origem nuclear, estes indícios( (??),
> estupefação minha)) ainda apontam, que a diferença relativa das forças
> eletrostática e centrífuga nos átomos, é a fonte ou causa da
> gravidade..."
> Comentário: não era centrípeta?. Só lembro uma coisinha adicional: força
> centrípeta e força centrífuga são aplicadas em corpos diferentes,
> portanto, uma não é reação à outra, e a centrífuga surge em alguns
> sistemas de referência e outras não! Logo..." Pode esclarecer o
> seguinte enunciado, de vossa lavra?: " ...podemos afirmar que gravidade
> é uma inércia, e é causada por uma força eletromagnética de origem
> nuclear".
>
> c) todo o resto que não lí...
>
> Não podendo ir mais além, por conta do " impacto" das coisas acima, e
> logo no início do trabalho(!), sugiro fortemente que o autor, se
> estiver com vontade e for corajoso, como parece que o é, publicando
> esta, desenvolva SUA OUTRA TEORIA. Quem nem lí ainda!
>
> Sds,
>
> Victor.




--
No virus found in this outgoing message.
Checked by AVG Anti-Virus.
Version: 7.0.308 / Virus Database: 266.7.4 - Release Date: 18/3/2005



##### ##### #####

Para saber mais visite
http://www.ciencialist.hpg.ig.com.br


##### ##### ##### #####



Yahoo! Grupos, um serviço oferecido por:

São Paulo Rio de Janeiro Curitiba Porto Alegre Belo Horizonte Brasília




--------------------------------------------------------------------------------
Links do Yahoo! Grupos

a.. Para visitar o site do seu grupo na web, acesse:
http://br.groups.yahoo.com/group/ciencialist/

b.. Para sair deste grupo, envie um e-mail para:
ciencialist-unsubscribe@yahoogrupos.com.br

c.. O uso que você faz do Yahoo! Grupos está sujeito aos Termos do Serviço do Yahoo!.



[As partes desta mensagem que não continham texto foram removidas]



SUBJECT: Re: [ciencialist] Re: Eter Luminifero (era: ninguem é inocente.)
FROM: JVictor <jvoneto@uol.com.br>
TO: ciencialist@yahoogrupos.com.br
DATE: 22/03/2005 20:26


Marcelo,

Para a TRE a existência do éter nem fede nem cheira. Einstein não
descartou o éter. Ele apenas mostrou que o éter é supérfluo, não influe
em sua teoria, em vista de sua estrutura conceitual.. Creio que o autor
se refere ao éter inventado por Descartes e assumido pelos que vieram
depois, para servir de suporte às ondas eletromagnéticas. Veja, por
exemplo, como o próprio Lorentz se refere ao Éter! E sua teoria
eletromagnética é totalmente baseada nisso!(Aliás, Lorentz morreu e não
se afastou um pingo dos conceitos etéricos). Esse éter milagroso,
rígido, imponderável, indetectável, que penetra tudo, e está em tudo e
em em todos, e tem poderes milagrosos, simplesmente pega a 3a. lei de
Newton e créu nela!
Esse éter não existe mesmo. Agora(bem, ainda vou ler, se tiver tempo,
essas "descobertas"), existem outras coisas, expostas pela física
quântica, que permeiam e preenchem do espaço. Creio estar havendo alguma
confusão.
Mas, vou procurar saber do que se trata realmente. Particularmente, não
tenho mais dúvidas de que o éter a que me referi acima não existe.

Sds,

Victor.

marcelomjr escreveu:

>
> Olá,
>
> Já que citaram a questao da (suposta) inexistência do Ether (ou
> aether)de forma tão conclusiva e dogmatica, gostaria apenas relembrar
> que nem todos os cientistas do mundo pensam assim e/ou creem que foi
> dada a ultima palavra sobre este assunto. Muito pelo contrario, um
> pouco mais de pesquisa sobre este tema na rede mundial de
> computadores, mostrará que a questão da existencia ou da inexistencia
> de um Ether ainda está longe (e muito longe) de acabar na comunidade
> cientifica (ao contrário do que pessoas mal informadas tem promulgado
> abundantemente aos leigos por aí), aliás como sugere o excerto
> abaixo, retirado do site da "Natural Philosophy Alliance (NPA)"
> [http://mywebpages.comcast.net/Deneb/]:
> <http://mywebpages.comcast.net/Deneb/%5D:>
>
> http://mywebpages.comcast.net/Deneb/Steps.htm
> "(...)
> 7. TAKE THE SAGNAC EXPERIMENT SERIOUSLY. In this case, the "infrared
> film" needed was provided by Sagnac in 1913, when he looked for the
> aether with an interferometer that rotated, instead of translating in
> a near-straight line. Something caused his fringes to shift as viewed
> on the rotating platform, and these shifts meant that the velocity of
> light was remaining constant relative to the laboratory. Sagnac
> advanced this as experimental proof against the second postulate of
> SR, which it actually was. His method has been modified and repeated
> many times since his day, and currently is being tested constantly
> among the satellites of the Global Positioning System (GPS). Every
> single time, when rotation of a light path within a surrounding
> dominant coordinate system occurs, fringes are shifted, light
> velocities are altered, and the existence of a luminiferous aether is
> strongly inferred--all contrary to SR.
>
> Establishment physicists have usually ignored the Sagnac effect, or
> once in a while they have attempted to explain it in terms of special
> or general relativity--but all of these attempts have fallen short.
> (...)"
>
>
> Um abraço,
> Marcelo M. Jr.
>
> --- Em ciencialist@yahoogrupos.com.br, "Oraculo" <oraculo@a...>
> escreveu
> > Olá Pubmed
> >
> > Pubmed: Naõ começa tropeçar nos seus proprios argumentos, o Éter
> luminífero é
> > um mito que foi derrubado...uma teoria absurda que foi demonstrada
> > falsa, portanto nao passa de um mito"
> >
> > Está enganado. O éter luminífero foi uma teoria que se embasava nos
> dados e conhecimentos da época. Explicava eventos e fenomenos do
> mundo físico, através de observações e experimentos. Ainda que tenha
> se mostrado irreal depois, com mais dados, nada nele lembra um mito,
> como duendes ou seres divinos de religiões. Sua estrutura, função e
> principalmente sua posterior refutação o tornam totalmente diferente
> de mitos e lendas.
> >
> > Os erros e enganos que as teorias cientificas as vezes apresentam
> fazem parte do método de criar repostas e depois testá-las
> rigorosamente, ajustando se necessário e abandonando o que for
> refutado. Mitos, por sua vez, se mantém indefinidamente, sem mudança
> ou ajuste. Não faz parte de sua natureza ser refutado, nme precisam
> de dados e testes para se manter.
> >
> > Se descobrirem que o Big Bang é incorreto, e não absurdo como
> parece pensar que pode acontecer, será uma ação legitimamente
> científica e os cientistas, tanto os que criaram a teoria quanto os
> que a refutaram, ficarão felizes e aceitarão as novas conclusões. O
> Big Bang é uma teoria que, neste momento, é mais capaz de explicar o
> surgimento do universo que qualquer outra, e perdurará até que mais
> dados a comprovem ou refutem. Não é nem de longe um mito, nem mesmo
> se descobrirmos que está incorreto (de novo, a teoria será, não
> absurda, mas incorreta).
> >
> > Veja, hoje a Big Bang explica porque o céu não é totalmente claro,
> mesmo durante a noite, explica como podem as galáxias estarem se
> afastando, explica a formação de estrelas, nuvens de gás estelar,
> materia escura, elementos quimicos, etc. Para substitui-la, é preciso
> uma teoria que, além de explicar tudo isso, ainda explique coisas que
> escapam a teoria do BB. Se o fizer, o BB será ultrapassado, você (ou
> quem for responsável) ganhará um premio Nobel, e a ciência terá novos
> dados e hipóteses para testar. Tudo com o rigor necessário.
> >
> > O Éter Luminífero explicava a propagação da luz e dezenas de outros
> fenomenos observados, que depois foram melhor explicados pela teoria
> eletromagnética. Apenas isso, não foi a substituição de um mito por
> outro.
> >
> > Precisa compreender isso, ou jamais compreenderá o que é a ciência
> e o que a torna diferente de crenças e mitos.
> >
> > Homero
> >
> > ----- Original Message -----
> > From: pubmed2005
> > To: ciencialist@yahoogrupos.com.br
> > Sent: Saturday, March 19, 2005 3:07 AM
> > Subject: [ciencialist] Re: ninguem é inocente.
> >
> >
> >
> > Naõ começa tropeçar nos seus proprios argumentos, o Éter
> luminífero é
> > um mito que foi derrubado...uma teoria absurda que foi
> demonstrada
> > falsa, portanto nao passa de um mito
> >
> > Se descobrirem que o Bing bang nao é teoria mais proxima da
> origem do
> > universo, isso vai soar tão absurdo como a crença no boi tatá. Se
> o
> > boi tatá nao existe, se foi comprovado a sua inexistencia, isso
> vai
> > refutar que o boi tatá só existe na imaginação criativa das
> pessoas.
> > Assim como o Éter luminífero preencheu as mentes científicas dos
> > séculos anteriores. Tudo nao se passou de uma ilusão
> >
> > Quanto ao Sudário, já está mais que esclarecida minha posição,
> nao
> > tenho que ficar repetindo os mesmos arguimentos de forma circular
> > como voce vem fazendo
> >
> >
> >
> > --- Em ciencialist@yahoogrupos.com.br, "Oraculo" <oraculo@a...>
> > escreveu
> > > Olá Pubmed
> >
> > >
> > > risos..:-) É enorme..:-) No primeiro caso, a conclusão, e não
> > crença, foi abandonada devido a evidencias posteriores que
> refutaram
> > a teoria do éter. No segundo, nada vai mudar e quem crê nisso vai
> > continuar crendo, sem que evidencias ou provas ou aspectos
> materiais
> > ou racionais tenham o poder de interferir oiu mudar essa crença.
> A
> > diferença, como pode notar é enorme..:-) A primeira, uma teoria
> > cientifica, esperava por dados e evidencias que a confirmassem, e
> se
> > conformou com a refutação por falta de provas. A segunda, criada
> da
> > imaginação humana, vai se manter indefinidamente, exatamente como
> foi
> > criada, dependendo apenas da crença subjetiva de seres humanos.
> > Enorme diferença..:-)
> > >
> >
> >
> >
> >
> >
> >
> > ##### ##### #####
> >
> > Para saber mais visite
> > http://www.ciencialist.hpg.ig.com.br
> >
> >
> > ##### ##### ##### #####
> >
> >
> > Yahoo! Grupos, um serviço oferecido por:
> >
> >
> >
> >
> >
> >
> >
> > --------------------------------------------------------------------
> ----------
> > Links do Yahoo! Grupos
> >
> > a.. Para visitar o site do seu grupo na web, acesse:
> > http://br.groups.yahoo.com/group/ciencialist/
> >
> > b.. Para sair deste grupo, envie um e-mail para:
> > ciencialist-unsubscribe@yahoogrupos.com.br
> >
> > c.. O uso que você faz do Yahoo! Grupos está sujeito aos Termos
> do Serviço do Yahoo!.
> >
> >
> >
> > [As partes desta mensagem que não continham texto foram removidas]
>
>
>
>
>
> ##### ##### #####
>
> Para saber mais visite
> http://www.ciencialist.hpg.ig.com.br
>
>
> ##### ##### ##### #####
>
>
> *Yahoo! Grupos, um serviço oferecido por:*
>
> *
> <http://br.rd.yahoo.com/SIG=12a402nmr/M=264105.3931087.6562589.1588051/D=brclubs/S=2137111528:HM/EXP=1111529884/A=2361264/R=6/SIG=10v4acpp0/*http://br.shopping.yahoo.com/>*
>
>
>
> ------------------------------------------------------------------------
> *Links do Yahoo! Grupos*
>
> * Para visitar o site do seu grupo na web, acesse:
> http://br.groups.yahoo.com/group/ciencialist/
>
> * Para sair deste grupo, envie um e-mail para:
> ciencialist-unsubscribe@yahoogrupos.com.br
> <mailto:ciencialist-unsubscribe@yahoogrupos.com.br?subject=Unsubscribe>
>
> * O uso que você faz do Yahoo! Grupos está sujeito aos Termos do
> Serviço do Yahoo! <http://br.yahoo.com/info/utos.html>.
>
>
>
>
> __________ Informação do NOD32 1.1030 (20050319) __________
>
> Esta mensagem foi verificada pelo NOD32 Sistema Antivírus
> http://www.nod32.com.br




SUBJECT: Gravitons
FROM: "PSavio" <psdmo@uol.com.br>
TO: <ciencialist@yahoogrupos.com.br>
DATE: 22/03/2005 21:13

Daniel Moser escreveu:


>Prof. Luiz,

Minha opinião a respeito desta teoria é: ela é simplesmente SURPREENDENTE!
Achei ela muito interessante, mas ainda não tive tempo de fazer uma
análise cuidadosa da parte "quantitativa". Mas a parte "qualitativa", me
pareceu extraordinária!!!

>Sob os pontos levantados pelo Victor:
1. Para Einstein, a gravidade cria um espaço tempo curvo, e não o
contrário (o texto está ok)

sávio: Então, escrevo o mesmo que o Victor escreveu de outro jeito:

"gravidade é uma manifestação da curvatura do espaçotempo, e não o contrário". É disto, precisamente,

que a TRG trata. Se você diz que o texto do autor está correto, paciência! Só reflita e estude mais. E isto é elementar na TRG! Elementar.

>2. Da maneira que estão escritas, AO PÉ DA LETRA, são erros conceituais.
Mas o contexto e as conclusões tiradas pelo autor demonstram um correto
conhecimento de todos os conceitos envolvidos no trecho.
As alegações a) e b) não me parecem de alguém que está realmente
interessado em conhecer todas as possibilidades da natureza e da
ciência. Por acaso o autor destes comentários está TOTALMENTE a par dos
mecanismos que formam os átomos? Como os elétrons se "fixam " e se
mantém estáveis em suas órbitas em torno do núcleo, e como realizam seu
movimento continuamente? Pois eu posso dizer que nenhum cientista hoje
possui tal conhecimento, o portanto não podemos desconsiderar a hipótese
do texto "TQG".

sávio: não defendo ninguém. Mas não dá sequer para comentar o texto acima. O Victor está corretíssimo, até na maneira elegante onde mostra as incoerências, que pululam em todo o trabalho! Faça o seguinte; o estude a teoria, analise, reflita, e depois volte para cá, com algo substancial e provas concretas. Nada pessoal. Só contra argumentos.

Atenciosamente,

Pedro Sávio.


Atenciosamente,
Daniel Moser
Aluno de 3º ano - Bacharelo em Física - USP


JVictor escreveu:

> Luiz Ferraz Netto escreveu:
>
> Apreciaria comentários sobre o seguinte texto:
>
> http://rolfguthmann.sites.uol.com.br/TQG/tqg.html
>
>
> Professor Luiz Ferraz,
>
> > Lí o artigo até o quinto parágrafo da seção 2. E parei. Até chegar
> > ao item 2 encontramos hipóteses absolutamente estranhas e sem
> > significado físico, pelo que já peço ao autor que seja mais claro..
> >
> > Com relação ao item 2- O que é gravidade?, transcrevo dois conceitos
> > absolutamente equivocados, que comento rapidamente logo após o destaque.
>
> 1. " Para Einstein, a gravidade causava uma deformação no espaço tempo
> contínuo e, com esta idéia, ele desenvolveu uma álgebra muito complexa
> que a descreve apenas geometricamente."
>
> Comentário: 1) É o contrário: a gravidade não é causa; é efeito da
> curvatura do espaço. 2) Einstein NÃO desenvolveu nenhuma álgebra muito
> complexa. Ele usou um formalismo matemático, novo na época, criado por
> outros matemáticos, baseado em geometria diferencial, formalismo esse
> de jeito nenhum complexo. Desculpe-me o autor, mas esse é o tipo do
> erro conceitual inadmissível para alguém que se propõe a elaborar uma
> nova teoria.
>
> > 2. - " Uma outra evidência se reporta a teoria da relatividade de
> > Einstein, que está embasada no princípio da equivalência o qual
> > estabelece que a massa, ou a masssa inercial e o peso, ou massa
> > gravitacional de materiais diferentes sofrem a mesma aceleração... "
>
> Comentários: Os erros conceituais aquí são para ninguém botar
> defeito!. Massa e peso são coisas diferentes!. E esse princípio de
> equivalência é devido a Galileu e é chamado Princípio de Equivalência
> fraco. Einstein o estendeu a toda a física, e o PE de Einstein tem tem
> a seguinte forma:
>
> " Um laboratório local, não girante, em queda livre, é equivalente, para
> a realização de qualquer experiência, a um referencial inercial numa
> região do espaço sem gravidade." Esta é a forma forte do Princípio de
> Equivalência, devido a Einstein. É o mesmo que dizer: se você estiver em
> uma nave, no espaço, e esta estiver sob uma aceleração, digamos, de
> 10m/s/s, você será empurrado contra o piso da nave e não saberá dizer
> se essa aceleração é devida aos motores que a provocam ou se é devida
> aos efeitos gravitacionais de alguma massa grande nas proximidades,
> como, por exemplo, a terra(cujos efeitos gravitacionais, que todos
> conhecemos, expressam-se por uma aceleração, o g= aprox. a 10m/s/s e nos
> pregados no chão, do mesmo jeito que o astronauta em relação ao piso da
> nave, com tudo que tem direito!) ou outro planeta. Ou seja, essas
> aceleraçãos são equivalentes!. Na nave, uma balança dirá que o seu peso
> será, digamos, 700kg.m/s/s, se for um gordinho, como eu, de 70 Kg de
> massa. Se os motores aumentarem a aceleração, a balança vai acusar um
> peso maior. Na terra, se consideramos que aceleração gravitacional tem
> o mesmo valor, aprx. 10m/s/s, uma balança indicará o mesmo peso para
> você, ou seja 700kg.m/s/s! Refletindo sobre coisas assim, num momento
> de inspiração, Einstein chegou ao seu racicínio mais feliz, segundo
> disse: concluiu que uma pessoa em queda livre não sente seu próprio
> peso! Ou seja, para alguém que cai em queda livre, não existe, ao menos
> nas vizinhanças, qualquer campo gravitacional. Com esse conceito,
> Einstein mostra também que a gravidade é uma coisa cuja existência é
> relativa. Independe da constituição dos corpos. Aliás, o PE fraco já diz
> a mesma coisa. É esta a base da TRG! Compare, então, o que expressei
> acima, com suas afirmações abaixo e decida o autor se ainda mantém:
>
> a) sua hipótese, item 1, que não me animei a comentar(idem, outras, de
> mesma qualidade):
> "Veremos que a a gravidade é o resultado do desequilíbrio entre força
> coloumbiana e força centrípeta",
>
> b) ou com esta, quinto parágrafo do item 2:
> " ...podemos afirmar que a gravidade é uma inércia, e é causada por uma
> força eletromagnética de origem nuclear, estes indícios( (??),
> estupefação minha)) ainda apontam, que a diferença relativa das forças
> eletrostática e centrífuga nos átomos, é a fonte ou causa da
> gravidade..."
> Comentário: não era centrípeta?. Só lembro uma coisinha adicional: força
> centrípeta e força centrífuga são aplicadas em corpos diferentes,
> portanto, uma não é reação à outra, e a centrífuga surge em alguns
> sistemas de referência e outras não! Logo..." Pode esclarecer o
> seguinte enunciado, de vossa lavra?: " ...podemos afirmar que gravidade
> é uma inércia, e é causada por uma força eletromagnética de origem
> nuclear".
>
> c) todo o resto que não lí...
>
> Não podendo ir mais além, por conta do " impacto" das coisas acima, e
> logo no início do trabalho(!), sugiro fortemente que o autor, se
> estiver com vontade e for corajoso, como parece que o é, publicando
> esta, desenvolva SUA OUTRA TEORIA. Quem nem lí ainda!
>
> Sds,
>
> Victor.


[As partes desta mensagem que não continham texto foram removidas]



SUBJECT: Radiocarbon Dating The Shroud -A Critical Statistical Analysis
FROM: "junior_br2001" <junior_br2001@yahoo.com.br>
TO: ciencialist@yahoogrupos.com.br
DATE: 22/03/2005 21:15


Radiocarbon Dating The Shroud -A Critical Statistical Analysis

http://www.shroud.com/vanhels3.htm


JR










SUBJECT: Science & the shroud
FROM: "junior_br2001" <junior_br2001@yahoo.com.br>
TO: ciencialist@yahoogrupos.com.br
DATE: 22/03/2005 21:24


Science & the shroud
Microbiology meets archaeology in a renewed quest for answers

http://www.uthscsa.edu/mission/spring96/shroud.htm


JR





SUBJECT: Scientific Papers and Articles
FROM: "junior_br2001" <junior_br2001@yahoo.com.br>
TO: ciencialist@yahoogrupos.com.br
DATE: 22/03/2005 21:27


Eis aqui uma série de artigos científicos e arquelógicos sobre o
Sudário


http://www.shroud.com/papers.htm

JR





SUBJECT: Re: ninguem é inocente.
FROM: "pubmed2005" <pubmed2005@yahoo.com.br>
TO: ciencialist@yahoogrupos.com.br
DATE: 23/03/2005 00:09


Silvio, segundo seu raciocínio então nenhum personagem histórico
existiu então.
Tudo o que sabemos dos povos antigos foram transmitidos por outros e
não por eles mesmos.

JR

--- Em ciencialist@yahoogrupos.com.br, "Silvio" <scordeiro@t...>
escreveu
> Júnior!!!!!
>
> Curiosamente, nenhum dos "iluminados" ( Buda, J. Cristo,) deixou
qualquer
> vestígio de sua passagem por esse vale de lágrimas. Buda pode até
ter
> falado: mas quem, em sã consciência,quem pode repetir ipsum litera
suas
> palavras que rolam em tradições e traduções monásticas há 2.000
anos?
>
> Não vá no papo dessa turma, prefira ler Freud. Vamos lutar pela
canonização
> do Paulo Coelho.
>
> sds.,
>
> silvio
>
>
> -----Mensagem Original-----
> De: "junior_br2001" <junior_br2001@y...>
> Para: <ciencialist@yahoogrupos.com.br>
> Enviada em: segunda-feira, 21 de março de 2005 14:25
> Assunto: [ciencialist] Re: ninguem é inocente.
>
>
>
>
> Buda disso algo diferente:
>
> "Não acredite em algo simplesmente porque ouviu. Não acredite em
algo
> simplesmente porque todos falam a respeito. Não acredite em algo
> simplesmente porque esta escrito em seus livros religiosos. Não
> acredite em algo só porque seus professores e mestres dizem que é
> verdade. Não acredite em tradições só porque foram passadas de
> geração em geração. Mas depois de muita análise e observação, se
você
> vê que algo concorda com a razão, e que conduz ao bem e beneficio de
> todos, aceite-o e viva-o."
>
> JR
>
>
>
>
>
> --- Em ciencialist@yahoogrupos.com.br, TARCISIO BORGES <tbs97@f...>
> escreveu
> > Talvez Buda o fosse, pois em determinado momento diz aos seus
> discípulos:
> >
> > Não acredite em seus professores.
> >
> > :-)
>
>
>
>
>
>
> ##### ##### #####
>
> Para saber mais visite
> http://www.ciencialist.hpg.ig.com.br
>
>
> ##### ##### ##### #####
> Links do Yahoo! Grupos





SUBJECT: Re: [ciencialist] Re: Provas cabais e alegações inconsequentes..:-) (era: Carbono-14 )
FROM: "Luiz Ferraz Netto" <leobarretos@uol.com.br>
TO: <ciencialist@yahoogrupos.com.br>
DATE: 23/03/2005 06:41

Acredito que, nesses papos todos, vcs esteja, falando de mim, não?

[]'

deus, the general and arquibancad
=======================



E o mais modesto, né Takata?


Emilian ......

>
> ahaha... já que o clima é de descontração:
>
> [Takata]
> Nunca, jamais, pus-me como mais inteligente do q. alguem (ainda mais
> nesta lista). Se for por falta de ter dito o contrario, digo agora:
> sou o mais burro desta lista, sou o menos inteligente da internet,
> sou o mais ignorante da Terra, sou o menos esperto da Galaxia.
>
> [Emiliano]
> O Takata é esperto. Disse que era o mais burro desta lista, menos
> inteligente da Internet, mais ignorante da Terra, menos esperto da
> Galaxia,
> porém, acredito eu, pensa ser o mais genial do Universo! :-) (brincadeira)
>




SUBJECT: Re: [ciencialist] Re: Provas cabais e alegações inconsequentes..:-) (era: Carbono-14 )
FROM: "Luiz Ferraz Netto" <leobarretos@uol.com.br>
TO: <ciencialist@yahoogrupos.com.br>
DATE: 23/03/2005 06:56

on the mensagem anterior, onde se lê 'esteja', leia-se 'estejam':

"Acredito que, nesses papos todos, vcs esteja, falando de mim, não?"

[]' abstract


SUBJECT: Re: [ciencialist] Re: equações
FROM: TARCISIO BORGES <tbs97@fisica.ufpr.br>
TO: ciencialist@yahoogrupos.com.br
DATE: 23/03/2005 09:07

Pra Linux tem o OpenOffice, mas não está bem implementado.

O Latex é nativo no linux.

[]s
TARCISIO BORGES
tbs97@fisica.ufpr.br

On Tue, 22 Mar 2005, junior_br2001 wrote:
> OK. Eu localizei o Latex e o Equation Editor para o Windows no
> GOOGLE. Agora resta eu saber onde encontro pro Linux, vou perguntar
> na comunidade linux, valeuz..
>
> JR


SUBJECT: Re: Um pedido - mais uma vez
FROM: César A. K. Grossmann <cesarakg@bol.com.br>
TO: ciencialist@yahoogrupos.com.br
DATE: 23/03/2005 10:21


--- Em ciencialist@yahoogrupos.com.br, "junior_br2001"
<junior_br2001@y...> escreveu
>
> Comom faz, qual editor uso para poder passar calculos e simbolos
> matemáticos para esta lista?

Tem uma discussão e artigo sobre o assunto em
http://mathforum.org/typesetting/

[]s
--
César A. K. Grossmann





SUBJECT: Re: [ciencialist] Gravitons
FROM: "Alberto Mesquita Filho" <albmesq@uol.com.br>
TO: <ciencialist@yahoogrupos.com.br>
DATE: 23/03/2005 11:00

----- Original Message -----
From: "JVictor"
Sent: Thursday, March 24, 2005 9:25 AM
Subject: Re: [ciencialist] Gravitons

Olá JVictor

> 5) Quanto ao furo do PE no estudo que você fez, ainda não ví como. Mas
> chego lá, garanto.

Em novembro de 2000 nós chegamos a discutir aqui na Ciencialist alguns
aspectos desta magnífica idéia do Léo. Creio que ainda falta muito para ser
debatido a respeito, mas não deixa de ser um começo. Seria interessante que
mais físicos palpitassem pois o assunto é interessantíssimo e o paradoxo
ainda está com sua resolução pendente. Deixá-lo na gaveta e/ou debaixo do
tapete não me parece ser a atitude mais sábia, mesmo porque acredito que o
Léo, pelo que tem feito pela física de nosso país, mereceria bem mais do que
isso.

As mensagens podem ser recuperadas através do "algoritmo indexatório"
montado na página
http://ecientificocultural.com/ECC2/Dialogos/dsfcvm4.htm que tem por título
"O empuxo de Newton que, por sinal, é do Léo".

Para os que gostam do princípio da relatividade, outro tema interessante e
que chegou a ser discutido aqui na Ciencialist (dezembro de 2000) poderá ser
lido através da página
http://ecientificocultural.com/ECC2/Dialogos/rgpca1.htm que tem por título
"Resolução Clássica do Paradoxo da Carga Acelerada". O debate é apresentado
em duas páginas web. Neste caso as mensagens foram reproduzidas nestas duas
páginas do meu site e o algoritmo indexatório está aí apenas para auxiliar
aqueles que quiserem pular uma ou mais mensagens.

[ ]´s
Alberto
http://ecientificocultural.com/indice.htm
Mas indiferentemente a tudo isso, o neutrino tem massa, o elétron não é
uma carga elétrica coulombiana e a Terra se move. E a história se repetirá.



SUBJECT: Re: [ciencialist] Gravitons - ERRATA
FROM: "Alberto Mesquita Filho" <albmesq@uol.com.br>
TO: <ciencialist@yahoogrupos.com.br>
DATE: 23/03/2005 11:05


----- Original Message -----
From: "Alberto Mesquita Filho" <albmesq@uol.com.br>
To: <ciencialist@yahoogrupos.com.br>
Sent: Wednesday, March 23, 2005 11:00 AM
Subject: Re: [ciencialist] Gravitons


ONDE SE LÊ:

Para os que gostam do princípio da RELATIVIDADE, outro tema interessante e

LEIA-SE

Para os que gostam do princípio da EQUIVALÊNCIA, outro tema interessante e

[ ]´s
Alberto
http://ecientificocultural.com/indice.htm
Mas indiferentemente a tudo isso, o neutrino tem massa, o elétron não é
uma carga elétrica coulombiana e a Terra se move. E a história se repetirá.



SUBJECT: [ciencialist] Gravitons
FROM: JVictor <jvoneto@uol.com.br>
TO: ciencialist@yahoogrupos.com.br
DATE: 23/03/2005 12:21

Alberto,

É por isso que fiz o e-mail com aquelas sugestões. Pois acho esse
trabalho do Professor simplesmente magnifíco. E que não deve ficar
apenas exposto virtualmente. Acho que deve ir para o lugar que merece.
Estantes de professores, alunos, pesquisadores, laboratórios,
educadores, adotados como livro texto.

Voltando ao PE, vou me dedicar um pouco para ver se consigo encontrar o
que o Professor viu. Certamente, como você disse, será um bom motivo
para pesquisas.
Sds,

Victor


Alberto Mesquita Filho escreveu:

> ----- Original Message -----
> From: "JVictor"
> Sent: Thursday, March 24, 2005 9:25 AM
> Subject: Re: [ciencialist] Gravitons
>
> Olá JVictor
>
> > 5) Quanto ao furo do PE no estudo que você fez, ainda não ví como. Mas
> > chego lá, garanto.
>
> Em novembro de 2000 nós chegamos a discutir aqui na Ciencialist alguns
> aspectos desta magnífica idéia do Léo. Creio que ainda falta muito
> para ser
> debatido a respeito, mas não deixa de ser um começo. Seria
> interessante que
> mais físicos palpitassem pois o assunto é interessantíssimo e o paradoxo
> ainda está com sua resolução pendente. Deixá-lo na gaveta e/ou debaixo do
> tapete não me parece ser a atitude mais sábia, mesmo porque acredito que o
> Léo, pelo que tem feito pela física de nosso país, mereceria bem mais
> do que
> isso.
>
> As mensagens podem ser recuperadas através do "algoritmo indexatório"
> montado na página
> http://ecientificocultural.com/ECC2/Dialogos/dsfcvm4.htm que tem por
> título
> "O empuxo de Newton que, por sinal, é do Léo".
>
> Para os que gostam do princípio da relatividade, outro tema interessante e
> que chegou a ser discutido aqui na Ciencialist (dezembro de 2000)
> poderá ser
> lido através da página
> http://ecientificocultural.com/ECC2/Dialogos/rgpca1.htm que tem por título
> "Resolução Clássica do Paradoxo da Carga Acelerada". O debate é
> apresentado
> em duas páginas web. Neste caso as mensagens foram reproduzidas nestas
> duas
> páginas do meu site e o algoritmo indexatório está aí apenas para auxiliar
> aqueles que quiserem pular uma ou mais mensagens.
>
> [ ]´s
> Alberto
> http://ecientificocultural.com/indice.htm
> Mas indiferentemente a tudo isso, o neutrino tem massa, o elétron não é
> uma carga elétrica coulombiana e a Terra se move. E a história se
> repetirá.
>
>
>
> ##### ##### #####
>
> Para saber mais visite
> http://www.ciencialist.hpg.ig.com.br
>
>
> ##### ##### ##### #####
>
>
> *Yahoo! Grupos, um serviço oferecido por:*
>
> *
> <http://br.rd.yahoo.com/SIG=12a4sdj8a/M=264105.3931087.6562589.1588051/D=brclubs/S=2137111528:HM/EXP=1111672990/A=2361264/R=6/SIG=10v4acpp0/*http://br.shopping.yahoo.com/>*
>
>
>
> *Links do Yahoo! Grupos*
>
> * Para visitar o site do seu grupo na web, acesse:
> http://br.groups.yahoo.com/group/ciencialist/
>
> * Para sair deste grupo, envie um e-mail para:
> ciencialist-unsubscribe@yahoogrupos.com.br
> <mailto:ciencialist-unsubscribe@yahoogrupos.com.br?subject=Unsubscribe>
>
> * O uso que você faz do Yahoo! Grupos está sujeito aos Termos do
> Serviço do Yahoo! <http://br.yahoo.com/info/utos.html>.
>
>



SUBJECT: Re: [ciencialist] Re: Um pedido - mais uma vez
FROM: JVictor <jvoneto@uol.com.br>
TO: ciencialist@yahoogrupos.com.br
DATE: 23/03/2005 12:57

Alguém, parece que o Sílvio, já deu uma solução para o assunto. De
qualquer maneira, vou falar do que aprendí a respeito.
Símbolos matemáticos são objetos, não aceitos pelo outlouco ou outro
gerenciador de imagens. Se você quer comunicar algo via e-mail, o único
jeito-que conheço- é tratar tudo como texto mesmo. Ou seja, escrevendo
mesmo os nomes do símbolos representativos, onde não for possivel usar
os simbolos ASCII. Outra alternativa é você fazer convenções, como o
exemplo abaixo:
Integral de f(x) de a até b = I_a^b f(x)dx. É uma alaternativa,
trabalhosa, mas viável. Para publicar na internet você poder usar os
códigos HTML, que são fáceis e reconhecidos pelos browser's I.E e
Firefox. Agora, para escrever qualquer equação, de quaslquer e jeito,
há o Látex, que estou usando. Aprendí-o não há muito tempo. É
espetacular, mas você tem que estudar e aprender os muitos comandos e
nuances dos tais. Mas é gratificante. Uso o Miktex 2.4, que é gratuito.
A página a seguir é como uma "receita de bolo" pois você encontrará
todas as dicas de como instalar, de como baixar o Miktex, de como baixar
e instalar outros programas necessários e, sobretudo, em língua pátria,
um tutorial que, inclusive, lhe ensinar a escrever o primeiro trabalho.
Daí por diante é relaxar e mandar ver.
Estou escrevendo alguns ensaiors num programa que comprei, o WSP 5.0. É
do tipo "o que você vê é o que você terá."Mas é que eu sou preguiçoso
mesmo. Continuo usando o látex, mas uso mais este outro que, também,
É um poderoso programa de matemática, tipo MathCad, que também
tenho(registrado, também), mas que é um editor matemático sofrível.
Sugiro, então, que você aprenda o Látex. Os trabalhos saem num num nível
de apresentação muito profissional.
Você pode usar também, diretamente, o word com o EditEquation do
Windows, símbolos tipo objeto, para escrever seus trabalhos. É muito
bom, mas é limitado. A maioria, aliás, graaande maioria, das formatações
você tem que fazer na munheca mesmo, via comandos do word! É abusado
por isso. Mas funciona perfeitamente.
Depois dos trabalhos feitos, mesmo com Edit Equation, você pode
convertê-lo para PDF usando um programa brasileiro, free, chamado
PDFlivre, que você encontra através do xereta e detetive mor da
internet, o google.(O pdfLivre só é chatinho para instalar. Mas como
você é esperto, isto não será óbice de vergonha. Depois, o céu é o limite.
Pronto, quero ler a primeira peça da arte.Vamos lá.

Sds,

Victor.






César A. K. Grossmann escreveu:

>
> --- Em ciencialist@yahoogrupos.com.br, "junior_br2001"
> <junior_br2001@y...> escreveu
> >
> > Comom faz, qual editor uso para poder passar calculos e simbolos
> > matemáticos para esta lista?
>
> Tem uma discussão e artigo sobre o assunto em
> http://mathforum.org/typesetting/
>
> []s
> --
> César A. K. Grossmann
>
>
>
>
>
> ##### ##### #####
>
> Para saber mais visite
> http://www.ciencialist.hpg.ig.com.br
>
>
> ##### ##### ##### #####
>
>
> *Yahoo! Grupos, um serviço oferecido por:*
> <http://br.rd.yahoo.com/SIG=12augmlmn/M=264379.5078783.6203979.1588051/D=brclubs/S=2137111528:HM/EXP=1111670556/A=2191897/R=0/SIG=10vqa2grn/*http://br.diversao.yahoo.com/>
>
> <http://br.rd.yahoo.com/SIG=12augmlmn/M=264379.5078783.6203979.1588051/D=brclubs/S=2137111528:HM/EXP=1111670556/A=2191897/R=1/SIG=10vqa2grn/*http://br.diversao.yahoo.com/>
>
>
>
> ------------------------------------------------------------------------
> *Links do Yahoo! Grupos*
>
> * Para visitar o site do seu grupo na web, acesse:
> http://br.groups.yahoo.com/group/ciencialist/
>
> * Para sair deste grupo, envie um e-mail para:
> ciencialist-unsubscribe@yahoogrupos.com.br
> <mailto:ciencialist-unsubscribe@yahoogrupos.com.br?subject=Unsubscribe>
>
> * O uso que você faz do Yahoo! Grupos está sujeito aos Termos do
> Serviço do Yahoo! <http://br.yahoo.com/info/utos.html>.
>
>



SUBJECT: Teoria inglesa é contestada
FROM: Rodrigo Marques <rodmarq72@yahoo.com.br>
TO: Ceticismo Aberto <ceticismoaberto@yahoogrupos.com.br>, CienciaList <ciencialist@yahoogrupos.com.br>, CuritibaCetica <curitibacetica@yahoogroups.com>, Sociedade Brasileira de "C�ticos" e Racionalistas <sbcr@yahoogrupos.com.br>, Sociedade da Terra redonda <strbrasil@yahoogrupos.com.br>
DATE: 23/03/2005 13:46


http://www.investnews.com.br/ultimasnoticias/default.asp?id_noticia=490902&id_editoria=2239

Teoria inglesa é contestada
27/02 - 11:00



Curitiba, 27 de fevereiro de 2005 - O cientista curitibano Marcelo Samuel Berman, 59, foi convidado a escrever sobre sua teoria da antigravidade em um capítulo do livro "Black Holes" da editora Nova Science Publishers, de Nova York, que estará nas livrarias do mundo todo em final de 2005.
Samuel Berman vai expor a tese "Energy of Black Holes and Hawking's Universe", onde ele questiona a afirmação do célebre cientista britânico Stephen Hawking, da Universidade de Cambridge, de que o Universo está dentro de uma casca de noz. Berman diz que essa casca só pode ser a membrana exterior do buraco negro e não uma superficie material qualquer, como Hawking sugere. O cientista curitibano concorda com Hawking quanto à afirmação de que a energia total é nula no universo, quando computadas todas as formas de energia, inclusive a do campo gravitacional, que é negativa.
Berman acredita que em um buraco negro há energia negativa que poderia ser usada, no futuro, para mover naves espaciais pelo efeito do movimento antigravitacional. O mesmo processo de produção de energia negativa poderia ser desenvolvida em modelos fechados para movimentar motores, aqui no planeta. A descrição de Hawking, segundo Berman, implica que o universo é machiano, ou seja, obedece ao princípio de Mach, que foi introduzido por Albert Einstein na Teoria da Relatividade Geral, como um dos seus fundamentos. O físico americano John Archibald Wheeler, diz que a distribuição das massas gravitacionais nos pontos mais distantes do universo é que geram a inércia local. Inércia é a propriedade dos corpos de resistirem às acelerações e é medida pela massa. Berman afirma que o Universo é machiano, pois obedece a uma relação obtida por vários cosmólogos nos últimos 50 anos, dentre eles Robert Dicke e Carl Brans. Os dois sustentam que o produto da constante de gravitação universal de Newton
vezes a massa do universo, dividido pelo produto do quadrado da velocidade da luz no vácuo, multiplicado pelo raio do universo, é um número puro, aproximadamente igual a uma unidade. Segundo Berman, alguns outros modelos do universo não obedece a esta relação de Brans-Dickeb, porém exprimentalmente parece ser verificada.
Para o cientista curitibano, que já publicou 50 trabalhos de pesquisa no Japão, Europa e Estados Unidos, o universo descrito por Hawking só pode ter energia total zero se a constante cosmológica decrescer com o inverso do quadrado do raio do universo. A constante cosmológica foi introduzida na Teoria da Relatividade Geral por Albert Einstein quando ele percebeu que sem essa constante o universo estaria em expansão ou em contração, enquanto este julgava, à época, que o Universo era estático. Desta forma Einstein cometeu o que ele chamou de "maior erro de sua carreira", já que, se tivesse acreditado nas suas equações, ele poderia ter feito a previsão de que o universo não era estático. Foi somente em 1929 que o astrônomo americano Edwin Hubble verificou experimentalmente que o Universo estava em expansão. Esquecida pelos cosmólogos, a constante cosmológica retornou ao cenário da cosmologia por volta de 1974, quando o físico russo Yakov Zeldovich anunciou que o vácuo possuía energia
dentro da Teoria da Relatividade Geral, e que essa energia revelava uma constante cosmológica não nula. Por volta de 1980, o cientista americano Alan Guth, pai do modelo inflacionário do universo, mostrava que, dentro das modernas teorias da física das partículas elementares, era preciso que no universo primordial a constante cosmológica fosse muito grande, cento e vinte ordens de grandeza maiores do que o máximo de valor que ela poderia ter hoje em dia. Berman desenvolveu cálculos concluindo que uma "constante" cosmológica tem que ser um termo variável que decai com o quadrado do raio do universo.
Outros cosmólogos também aventaram esta hipótese mas o que diferencia, segundo Berman, é o fato de seus estudos mostrarem que o Universo de Hawking obrigatoriamente tem que estar dotado desta propriedade. Berman conclui ainda que o Universo de Hawking está dentro da membrana exterior de um buraco negro.
Berman disse que a constante cosmológica, na verdade, tem duas facetas: a primeira é a de que a constante tem um valor muito pequeno hoje em dia, quando comparado com seu valor no Universo primordial. Mas aí, diz o cientista , é preciso explicar de onde vem a matéria escura do universo, já que a densidade de energia prevista pelo modelo inflacionário é 100 vezes maior do que a densidade medida experimentalmente para matéria do universo. Embora a questão não tenha sido definitivamente elucidada, muitos cientistas acreditam, segundo Berman, que a densidade de energia do vácuo preenche aquela densidade de energia faltante na matéria visível para alcançar o valor previsto pela teoria inflacionaria, também chamada de densidade critica. Por este entendimento, se a matéria visível no universo representa apenas 1% da energia crítica, os outros 99% têm que ser preenchidos com a densidade de energia do vácuo ou de matéria escura. Isso leva à suposição de que o Universo atual está sendo
acelerado. "É o seguinte. Trata-se de uma aceleração da taxa de expansão do Universo medida pela constante de Hubble, que, na verdade, é um parâmetro variável com a idade do universo", disse Berman, doutor em física teórica pela Universidade Federal do Rio de Janeiro(UFRJ), membro vitalício da Sociedade Internacional da Relatividade Geral e Grativação, com sede na Suíça.
(Gazeta Mercantil) (Ivanir José Bortot)



__________________________________________________
Converse com seus amigos em tempo real com o Yahoo! Messenger
http://br.download.yahoo.com/messenger/

[As partes desta mensagem que não continham texto foram removidas]



SUBJECT: Re: Um pedido - mais uma vez
FROM: César A. K. Grossmann <cesarakg@bol.com.br>
TO: ciencialist@yahoogrupos.com.br
DATE: 23/03/2005 14:28


--- Em ciencialist@yahoogrupos.com.br, César A. K. Grossmann
<cesarakg@b...> escreveu
>

Um aplicativo para desenhar equações em caracteres ASCII:

http://fuse.superglue.se/aamath/

Só que para ler precisa configurar seu programa de correio para usar
fontes fixas na apresentação da mensagem, em vez de fontes
proporcionais. Uma fonte fixa bastante apropriada é "Courier New",
para quem usa o Windows.

[]s
--
César A. K. Grossmann





SUBJECT: Re: Um pedido - mais uma vez
FROM: César A. K. Grossmann <cesarakg@bol.com.br>
TO: ciencialist@yahoogrupos.com.br
DATE: 23/03/2005 14:34


--- Em ciencialist@yahoogrupos.com.br, JVictor <jvoneto@u...> escreveu
> Você pode usar também, diretamente, o word com o EditEquation do
> Windows, símbolos tipo objeto, para escrever seus trabalhos. É muito
> bom, mas é limitado. A maioria, aliás, graaande maioria, das
formatações
> você tem que fazer na munheca mesmo, via comandos do word! É abusado
> por isso. Mas funciona perfeitamente.

O Editor de Equações do Word tem dois inconvenientes pequenos: 1. Só
está disponível na plataforma (sistema operacional) Windows, e 2. Até
onde eu lembro, só para quem tem o MS-Office.

P.S.: Quem está querendo colocar equações em mensagens de email não
sou eu. Atenção para a thread!
[]s
--
César A. K. Grossmann





SUBJECT: Re: [ciencialist] Editor de calculos e formulas
FROM: Franco <dfranco@pop.com.br>
TO: ciencialist@yahoogrupos.com.br
DATE: 23/03/2005 14:43

Para Windows ou GNU/Linux?
Se não me engano, nos arquivos desta lista há um artigo em pdf
intitulado "Introdução a programas físico-matemáticos livres", sugerindo
alguns softwares - se não encontrar posso lhe enviar em pvt.

Uma maneira de usar fórmulas e/ou gráficos em grupos de discussão, seria
editá-los em um programa e salvá-los como documento de imagem para
enviá-los às listas; porém, deve-se verificar se determinada lista
aceitará anexos, caso contrário, não adiantará nada.
Nesse caso, para Linux, há o Math que vem com a suíte OpenOffice, mas
não há como salvar as equações diretamente em formato *gif (figura) -
teria de fazer algumas gambiarras para isso e o resultado não é muito bom.
Existe o Scilabus, também, mas não sei quais seus recursos.
Para Windows, existe o MathType que salva diretamente suas equações no
formato .gif e outros tantos para gráficos.

Ainda, e esse achei interessante, é um fórum de matemática (acho que o
site fica em http://www.teorema.mat.br - ou procure por forum + teorema
no Google); em suma, você pode usar a notação do LATex para escrever
fórmulas matemáticas nesse fórum e o servidor as converte para "fórmulas
visíveis" - acho que entendeu o que queria dizer :) - é que fugiram-me
os termos corretos aqui.

Franco.

junior_br2001 escreveu:

>
> Pessoal, gostaria de saber onde encontro um editor para calculos e
> fórmulas matemáticas e gráficos para se utilizados nos meus estudos
> de física e pra tirarem minhas dúvidas nos grupos de discussão, já
> que o modo texto do grupo nao suporta as formulas e calculos
>
> JR
>
>




SUBJECT: Re: Editor de calculos e formulas
FROM: César A. K. Grossmann <cesarakg@bol.com.br>
TO: ciencialist@yahoogrupos.com.br
DATE: 23/03/2005 14:44


--- Em ciencialist@yahoogrupos.com.br, Franco <dfranco@p...> escreveu
>
> Uma maneira de usar fórmulas e/ou gráficos em grupos de discussão,
seria
> editá-los em um programa e salvá-los como documento de imagem para
> enviá-los às listas; porém, deve-se verificar se determinada lista
> aceitará anexos, caso contrário, não adiantará nada.

É o caso desta lista. Que filtra também o HTML (eu leio a lista a
partir do navegador internet, no endereço
http://br.groups.yahoo.com/group/ciencialist, e não vejo nada de HTML
nem figuras).

[]s
--
César A. K. Grossmann





SUBJECT: Re: Editor de calculos e formulas
FROM: "junior_br2001" <junior_br2001@yahoo.com.br>
TO: ciencialist@yahoogrupos.com.br
DATE: 23/03/2005 14:53


--- Em ciencialist@yahoogrupos.com.br, César A. K. Grossmann
<cesarakg@b...> escreveu
>
> É o caso desta lista. Que filtra também o HTML (eu leio a lista a
> partir do navegador internet, no endereço
> http://br.groups.yahoo.com/group/ciencialist, e não vejo nada de
HTML
> nem figuras).


Mas isso é configurável pelo moderador, fica a cargo e opção dele.
Numa lista de ciencia acho que deveria se abrir uma concessão para
aceitar os anexos. Pois se tivesse html ficaria ainda mais fácil que
texto puro. O problema que é muitos moderadores e usuários de lista
nao gostam por que a lista fica mais pesada. Nem tanto, já que nao é
todo mundo que manda coisas em html.

PS: Aproveitando a deixa par agradecer a todos pela ajuda, ao César,
ao Oráculo, ao Victor, Ao franco, ao Tarcísio, ao Leo, etc...

JR





SUBJECT: Re: Editor de calculos e formulas
FROM: "rmtakata" <rmtakata@altavista.net>
TO: ciencialist@yahoogrupos.com.br
DATE: 23/03/2005 14:58


--- Em ciencialist@yahoogrupos.com.br, "junior_br2001"
> Mas isso é configurável pelo moderador, fica a cargo e opção dele.
> Numa lista de ciencia acho que deveria se abrir uma concessão para
> aceitar os anexos.

Anexos nem pensar. Com a proliferacao de virus, anexos sao uma festa.

HTML nao trara' muitas vantagens, alem de criar mais riscos.

Se tiver algo importante, basta ou inserir figuras (ou outros
arquivos) na secao de arquivos da lista - ou carregar uma pagina
propria e indicar o caminho.

[]s,

Roberto Takata





SUBJECT: Re: Editor de calculos e formulas
FROM: "junior_br2001" <junior_br2001@yahoo.com.br>
TO: ciencialist@yahoogrupos.com.br
DATE: 23/03/2005 15:11


--- Em ciencialist@yahoogrupos.com.br, "rmtakata" <rmtakata@a...>
escreveu

>
> Anexos nem pensar. Com a proliferacao de virus, anexos sao uma
festa.



Com as novas mudanças do yahoo nao tem como transmitir mais vírus
pelo Yahoo. Os vírus são filtrados

JR





SUBJECT: Re: Editor de calculos e formulas
FROM: "rmtakata" <rmtakata@altavista.net>
TO: ciencialist@yahoogrupos.com.br
DATE: 23/03/2005 15:14


--- Em ciencialist@yahoogrupos.com.br, "junior_br2001"
> Com as novas mudanças do yahoo nao tem como transmitir mais vírus
> pelo Yahoo. Os vírus são filtrados

Vai nessa. Virus sao criados a cada segundo.

Anexos sao desnecessarios. Ha' a secao de arquivos da lista.

[]s,

Roberto Takata





SUBJECT: Re: Editor de calculos e formulas
FROM: "junior_br2001" <junior_br2001@yahoo.com.br>
TO: ciencialist@yahoogrupos.com.br
DATE: 23/03/2005 15:20


--- Em ciencialist@yahoogrupos.com.br, "rmtakata" <rmtakata@a...>
escreveu
>
> --- Em ciencialist@yahoogrupos.com.br, "junior_br2001"
> > Com as novas mudanças do yahoo nao tem como transmitir mais vírus
> > pelo Yahoo. Os vírus são filtrados
>
> Vai nessa. Virus sao criados a cada segundo.
>
>


Faz um teste , crie uma lista, depois tente mandar um vírus, ou crie
um e mande.
Os filtros barram todos os arquvos com as extensões .exe, .src e por
aí vai.

Agora se é desncessário é uma questão de opinião sua

JR





SUBJECT: Re: Editor de calculos e formulas
FROM: "junior_br2001" <junior_br2001@yahoo.com.br>
TO: ciencialist@yahoogrupos.com.br
DATE: 23/03/2005 15:23


--- Em ciencialist@yahoogrupos.com.br, "rmtakata" <rmtakata@a...>
escreveu

>
> Anexos sao desnecessarios. Ha' a secao de arquivos da lista.
>


E voce no entendeu tb...se uma pessoa programa em html e quer mandar
os dados matemáticos, ela faz e mandar pelo outlook, a lista recebe
em html, em vez de receber em texto. Facilita o trbalho de quem usa
Html





SUBJECT: Re: Editor de calculos e formulas
FROM: "rmtakata" <rmtakata@altavista.net>
TO: ciencialist@yahoogrupos.com.br
DATE: 23/03/2005 15:28


--- Em ciencialist@yahoogrupos.com.br, "junior_br2001"
> Faz um teste , crie uma lista, depois tente mandar um vírus, ou
> crie um e mande.
> Os filtros barram todos os arquvos com as extensões .exe, .src e
> por aí vai.

Virus nao se escondem apenas sob extensoes .exe, .src. Existem virus
de macro q. se escondem em .doc e ha' ate' comandos maliciosos
em .jpg. Os antivirus nao podem detetar todas as novas variantes -
eles sao atualizados exatamente *depois* q. os primeiros casos
comecam a ser relatados. Os virus estao sempre um passo na frente.

> Agora se é desncessário é uma questão de opinião sua

Claro q. eh opiniao minha.

[]s,

Roberto Takata





SUBJECT: Re: Editor de calculos e formulas
FROM: "junior_br2001" <junior_br2001@yahoo.com.br>
TO: ciencialist@yahoogrupos.com.br
DATE: 23/03/2005 15:34


--- Em ciencialist@yahoogrupos.com.br, "rmtakata" <rmtakata@a...>
escreveu
>
> --- Em ciencialist@yahoogrupos.com.br, "junior_br2001"
> > Faz um teste , crie uma lista, depois tente mandar um vírus, ou
> > crie um e mande.
> > Os filtros barram todos os arquvos com as extensões .exe, .src e
> > por aí vai.
>
> Virus nao se escondem apenas sob extensoes .exe, .src. Existem
virus
> de macro q. se escondem em .doc e ha' ate' comandos maliciosos
> em .jpg. Os antivirus nao podem detetar todas as novas variantes -
> eles sao atualizados exatamente *depois* q. os primeiros casos
> comecam a ser relatados. Os virus estao sempre um passo na frente.
>


Se voce nao reparou eu coloquei etc... etc.. apos citar as duas
extensões, mais se for vírus em .doc, eles vao ter que ter comandos
que o filtro detecta e barra, o mesmo vle pra .jpeg e outros. O que
pode aontecer é a pessoa mandr um link que contenha um executável que
dispar instanteneamente(hoje já tem vírus assim). Mas não custa nada
instslar um antiviruszinho free para deteecção dos maliciosos, além
de usar o spybot

Mas o que eu estava querendo dizer que é que para a pessoa que
programa em html facilita mandar os calculos de matemática e física,
pois ela insere diretamente no código html e manda pelo outlook e a
lista recebe maravilhosamente os calculos em html. Facilita muito





SUBJECT: Re: [ciencialist] Re: Editor de calculos e formulas
FROM: "E m i l i a n o C h e m e l l o" <chemelloe@yahoo.com.br>
TO: <ciencialist@yahoogrupos.com.br>
DATE: 23/03/2005 15:46

Júnior,

Mais uma opinião. Sou administrador de 'algumas' listas e lhe digo por experiência. O benefício de permitir anexos e formato 'html' não vale o risco.


[ ] 's do Emiliano Chemello
emiliano@quimica.net
http://www.quimica.net/emiliano
http://www.ucs.br/ccet/defq/naeq
[ MSN ] chemelloe@hotmail.com
[ ICQ ] 145060604

" Rien ne se perd, rien ne se crée,
tout se transforme."

Antoine Laurent de Lavoisier (químico francês, 1743 - 1794)

----- Original Message -----
From: junior_br2001
To: ciencialist@yahoogrupos.com.br
Sent: Wednesday, March 23, 2005 3:34 PM
Subject: [ciencialist] Re: Editor de calculos e formulas



--- Em ciencialist@yahoogrupos.com.br, "rmtakata" <rmtakata@a...>
escreveu
>
> --- Em ciencialist@yahoogrupos.com.br, "junior_br2001"
> > Faz um teste , crie uma lista, depois tente mandar um vírus, ou
> > crie um e mande.
> > Os filtros barram todos os arquvos com as extensões .exe, .src e
> > por aí vai.
>
> Virus nao se escondem apenas sob extensoes .exe, .src. Existem
virus
> de macro q. se escondem em .doc e ha' ate' comandos maliciosos
> em .jpg. Os antivirus nao podem detetar todas as novas variantes -
> eles sao atualizados exatamente *depois* q. os primeiros casos
> comecam a ser relatados. Os virus estao sempre um passo na frente.
>


Se voce nao reparou eu coloquei etc... etc.. apos citar as duas
extensões, mais se for vírus em .doc, eles vao ter que ter comandos
que o filtro detecta e barra, o mesmo vle pra .jpeg e outros. O que
pode aontecer é a pessoa mandr um link que contenha um executável que
dispar instanteneamente(hoje já tem vírus assim). Mas não custa nada
instslar um antiviruszinho free para deteecção dos maliciosos, além
de usar o spybot

Mas o que eu estava querendo dizer que é que para a pessoa que
programa em html facilita mandar os calculos de matemática e física,
pois ela insere diretamente no código html e manda pelo outlook e a
lista recebe maravilhosamente os calculos em html. Facilita muito





##### ##### #####

Para saber mais visite
http://www.ciencialist.hpg.ig.com.br


##### ##### ##### #####


Yahoo! Grupos, um serviço oferecido por:

São Paulo Rio de Janeiro Curitiba Porto Alegre Belo Horizonte Brasília




------------------------------------------------------------------------------
Links do Yahoo! Grupos

a.. Para visitar o site do seu grupo na web, acesse:
http://br.groups.yahoo.com/group/ciencialist/

b.. Para sair deste grupo, envie um e-mail para:
ciencialist-unsubscribe@yahoogrupos.com.br

c.. O uso que você faz do Yahoo! Grupos está sujeito aos Termos do Serviço do Yahoo!.



[As partes desta mensagem que não continham texto foram removidas]



SUBJECT: Re: Editor de calculos e formulas
FROM: "junior_br2001" <junior_br2001@yahoo.com.br>
TO: ciencialist@yahoogrupos.com.br
DATE: 23/03/2005 15:48


--- Em ciencialist@yahoogrupos.com.br, "rmtakata" <rmtakata@a...>
escreveu
>> Vai nessa. Virus sao criados a cada segundo.
>
> Anexos sao desnecessarios. Ha' a secao de arquivos da lista.



Ah, mas tb tem um porém , como eu leio as mensagens no site da Web do
grupo. O yahoo so protege que lê as mensagens pelo site como está
registrado a seguir:

Mensagens e anexos:
O Yahoo! Grupos continuará permitindo o envio de arquivos anexados a
mensagens de e-mail, no entanto, estes arquivos não serão mais
armazenados na área "Mensagens" no site do grupo na web. Você poderá
continuar a carregar e compartilhar arquivos e imagens de seus grupos
nas áreas "Arquivos" e "Fotos" normalmente. Solicitamos que você
transfira todos os arquivos que estejam armazenados atualmente na
área "Mensagens" para as áreas "Arquivos" ou "Fotos" do grupo. No dia
17 de setembro de 2003, todos os arquivos que estiverem armazenados
na área "Mensagens" serão excluídos

Isto quer dizer, por mais vírus ou anexos que voce mande, eles nao
poderão ser armazenados na área de mensagens. Mas os que leem pelo
Outloock correm este risco. Mas veja, o mesmo risco se corre ao se
fazer download na área de arquivos do grupo, não? Só se abrir anexos
de pessoas conhecidas, ou passar o antivírus

JR





SUBJECT: Re: [ciencialist] Re: Um pedido - mais uma vez
FROM: JVictor <jvoneto@uol.com.br>
TO: ciencialist@yahoogrupos.com.br
DATE: 23/03/2005 15:53

César A. K. Grossmann escreveu:

>
>
> P.S.: Quem está querendo colocar equações em mensagens de email não
> sou eu. Atenção para a thread!
> []s
> --
> César A. K. Grossmann

Victor: Tem razão, a mensagem é dirigida ao Junior.
Minhas desculpas.

Victor.

>
>
>
>
>
> ##### ##### #####
>
> Para saber mais visite
> http://www.ciencialist.hpg.ig.com.br
>
>
> ##### ##### ##### #####
>
>
> *Yahoo! Grupos, um serviço oferecido por:*
>
> *
> <http://br.rd.yahoo.com/SIG=12a0o09ev/M=264105.3931087.6562589.1588051/D=brclubs/S=2137111528:HM/EXP=1111685723/A=2361264/R=6/SIG=10v4acpp0/*http://br.shopping.yahoo.com/>*
>
>
>
> ------------------------------------------------------------------------
> *Links do Yahoo! Grupos*
>
> * Para visitar o site do seu grupo na web, acesse:
> http://br.groups.yahoo.com/group/ciencialist/
>
> * Para sair deste grupo, envie um e-mail para:
> ciencialist-unsubscribe@yahoogrupos.com.br
> <mailto:ciencialist-unsubscribe@yahoogrupos.com.br?subject=Unsubscribe>
>
> * O uso que você faz do Yahoo! Grupos está sujeito aos Termos do
> Serviço do Yahoo! <http://br.yahoo.com/info/utos.html>.
>
>



SUBJECT: Re: [ciencialist] Re: Editor de calculos e formulas
FROM: "Oraculo" <oraculo@atibaia.com.br>
TO: <ciencialist@yahoogrupos.com.br>
DATE: 23/03/2005 15:55

Pubmed

Mais uma opinião. Administro redes e conexões a rede e sevidores e, acredite, não vale o risco permitir anexos. Virus se modificam (ou são modificados por pentelhos com muito tempo livre..:-) a toda hora e os anti-virus simplesmente não podem acompanhar a variação

As novas versões do relativamente inofensivo StartPage tem deixado o pessoal de cabelo em pé e como única solução formatar cada micro infectado.

Por maior que seja a boa vontade dos participantes, por mais que o pessoal se cuide, anexos serão sempre perigosos em listas, devido a grande quantidade de pessoas e a replicação imediata das mensagens em todos os emails.

Melhor um pouco de trabalho, arquivo zipado na área de arquivos e links que correr mais esse risco..:-)

Um abraço.

Homero

----- Original Message -----
From: E m i l i a n o C h e m e l l o
To: ciencialist@yahoogrupos.com.br
Sent: Wednesday, March 23, 2005 3:46 PM
Subject: Re: [ciencialist] Re: Editor de calculos e formulas


Júnior,

Mais uma opinião. Sou administrador de 'algumas' listas e lhe digo por experiência. O benefício de permitir anexos e formato 'html' não vale o risco.


[ ] 's do Emiliano Chemello
emiliano@quimica.net
http://www.quimica.net/emiliano
http://www.ucs.br/ccet/defq/naeq
[ MSN ] chemelloe@hotmail.com
[ ICQ ] 145060604

" Rien ne se perd, rien ne se crée,
tout se transforme."

Antoine Laurent de Lavoisier (químico francês, 1743 - 1794)

----- Original Message -----
From: junior_br2001
To: ciencialist@yahoogrupos.com.br
Sent: Wednesday, March 23, 2005 3:34 PM
Subject: [ciencialist] Re: Editor de calculos e formulas



--- Em ciencialist@yahoogrupos.com.br, "rmtakata" <rmtakata@a...>
escreveu
>
> --- Em ciencialist@yahoogrupos.com.br, "junior_br2001"
> > Faz um teste , crie uma lista, depois tente mandar um vírus, ou
> > crie um e mande.
> > Os filtros barram todos os arquvos com as extensões .exe, .src e
> > por aí vai.
>
> Virus nao se escondem apenas sob extensoes .exe, .src. Existem
virus
> de macro q. se escondem em .doc e ha' ate' comandos maliciosos
> em .jpg. Os antivirus nao podem detetar todas as novas variantes -
> eles sao atualizados exatamente *depois* q. os primeiros casos
> comecam a ser relatados. Os virus estao sempre um passo na frente.
>


Se voce nao reparou eu coloquei etc... etc.. apos citar as duas
extensões, mais se for vírus em .doc, eles vao ter que ter comandos
que o filtro detecta e barra, o mesmo vle pra .jpeg e outros. O que
pode aontecer é a pessoa mandr um link que contenha um executável que
dispar instanteneamente(hoje já tem vírus assim). Mas não custa nada
instslar um antiviruszinho free para deteecção dos maliciosos, além
de usar o spybot

Mas o que eu estava querendo dizer que é que para a pessoa que
programa em html facilita mandar os calculos de matemática e física,
pois ela insere diretamente no código html e manda pelo outlook e a
lista recebe maravilhosamente os calculos em html. Facilita muito





##### ##### #####

Para saber mais visite
http://www.ciencialist.hpg.ig.com.br


##### ##### ##### #####


Yahoo! Grupos, um serviço oferecido por:

São Paulo Rio de Janeiro Curitiba Porto Alegre Belo Horizonte Brasília




------------------------------------------------------------------------------
Links do Yahoo! Grupos

a.. Para visitar o site do seu grupo na web, acesse:
http://br.groups.yahoo.com/group/ciencialist/

b.. Para sair deste grupo, envie um e-mail para:
ciencialist-unsubscribe@yahoogrupos.com.br

c.. O uso que você faz do Yahoo! Grupos está sujeito aos Termos do Serviço do Yahoo!.



[As partes desta mensagem que não continham texto foram removidas]



##### ##### #####

Para saber mais visite
http://www.ciencialist.hpg.ig.com.br


##### ##### ##### #####


Yahoo! Grupos, um serviço oferecido por:







------------------------------------------------------------------------------
Links do Yahoo! Grupos

a.. Para visitar o site do seu grupo na web, acesse:
http://br.groups.yahoo.com/group/ciencialist/

b.. Para sair deste grupo, envie um e-mail para:
ciencialist-unsubscribe@yahoogrupos.com.br

c.. O uso que você faz do Yahoo! Grupos está sujeito aos Termos do Serviço do Yahoo!.



[As partes desta mensagem que não continham texto foram removidas]



SUBJECT: Re: Editor de calculos e formulas
FROM: "rmtakata" <rmtakata@altavista.net>
TO: ciencialist@yahoogrupos.com.br
DATE: 23/03/2005 15:56


--- Em ciencialist@yahoogrupos.com.br, "junior_br2001"
> Se voce nao reparou eu coloquei etc... etc.. apos citar as duas
> extensões, mais se for vírus em .doc, eles vao ter que ter comandos
> que o filtro detecta e barra, o mesmo vle pra .jpeg e outros.

Vc disse "Os filtros barram todos os arquvos com as
extensões .exe, .src e por aí vai." - barram *arquivos*. Não irá
barrar aquivos jpg.

> Mas não custa nada instslar um antiviruszinho free para deteecção
> dos maliciosos, além de usar o spybot

As pessoas tEm q. usar antivirus, mas isso nao faz com q. se possa
liberar geral o envio de anexos por listas de discussao. Como eu
disse antes, antivirus estao sempre um passo atras do virus. Tb nao
podemos obrigar os usuarios menos habilidosos a se virarem com as
atualizacoes de virus.

> Mas o que eu estava querendo dizer que é que para a pessoa que
> programa em html facilita mandar os calculos de matemática e
> física

O q. ela ira' inserir, no maximo, eh uma figura. Em html em si, pode-
se fazer uma exponenciacao e so'. Isso pode ser feito com texto puro
usando-se de alguns truques.

2^3 eh dois elevado 'a terceira potencia.
2^0,5 eh raiz quadrada de dois (dois elevado 'a potencia fracionaria
1/2)

Sum(x/i=n,p) ou Som(x/i=n,p) - somatorio de xi com indice i de valor
n ateh valor p.

Jah temos diversos casos, nesta lista, de virus barrados porq. esta
lista nao aceita anexo. E jah foi feito anteriormente uma enquete
entre os participantes, votaram para q os anexos fossem barrados.

[]s,

Roberto Takata





SUBJECT: Re: Editor de calculos e formulas
FROM: "junior_br2001" <junior_br2001@yahoo.com.br>
TO: ciencialist@yahoogrupos.com.br
DATE: 23/03/2005 16:01


Concordo em partees, mas não inteiramente. Pode ser perigoso para
quem lê as mensagens pelo Outloock , mas pelo site da lista(como um
grande numeros de pessoas vem fazendo nao tem perigo nenhum:

Mensagens e anexos:
O Yahoo! Grupos continuará permitindo o envio de arquivos anexados a
mensagens de e-mail, no entanto, estes arquivos não serão mais
armazenados na área "Mensagens" no site do grupo na web. Você poderá
continuar a carregar e compartilhar arquivos e imagens de seus grupos
nas áreas "Arquivos" e "Fotos" normalmente. Solicitamos que você
transfira todos os arquivos que estejam armazenados atualmente na
área "Mensagens" para as áreas "Arquivos" ou "Fotos" do grupo. No dia
17 de setembro de 2003, todos os arquivos que estiverem armazenados
na área "Mensagens" serão excluídos

JR



--- Em ciencialist@yahoogrupos.com.br, "Oraculo" <oraculo@a...>
escreveu
> Pubmed
>
> Mais uma opinião. Administro redes e conexões a rede e sevidores e,
acredite, não vale o risco permitir anexos. Virus se modificam (ou
são modificados por pentelhos com muito tempo livre..:-) a toda hora
e os anti-virus simplesmente não podem acompanhar a variação
>
> As novas versões do relativamente inofensivo StartPage tem deixado
o pessoal de cabelo em pé e como única solução formatar cada micro
infectado.
>
> Por maior que seja a boa vontade dos participantes, por mais que o
pessoal se cuide, anexos serão sempre perigosos em listas, devido a
grande quantidade de pessoas e a replicação imediata das mensagens em
todos os emails.
>
> Melhor um pouco de trabalho, arquivo zipado na área de arquivos e
links que correr mais esse risco..:-)
>
> Um abraço.
>
> Homero
>
> ----- Original Message -----
> From: E m i l i a n o C h e m e l l o
> To: ciencialist@yahoogrupos.com.br
> Sent: Wednesday, March 23, 2005 3:46 PM
> Subject: Re: [ciencialist] Re: Editor de calculos e formulas
>
>
> Júnior,
>
> Mais uma opinião. Sou administrador de 'algumas' listas e lhe
digo por experiência. O benefício de permitir anexos e formato 'html'
não vale o risco.
>
>
> [ ] 's do Emiliano Chemello
> emiliano@q...
> http://www.quimica.net/emiliano
> http://www.ucs.br/ccet/defq/naeq
> [ MSN ] chemelloe@h...
> [ ICQ ] 145060604
>
> " Rien ne se perd, rien ne se crée,
> tout se transforme."
>
> Antoine Laurent de Lavoisier (químico francês, 1743 - 1794)
>
> ----- Original Message -----
> From: junior_br2001
> To: ciencialist@yahoogrupos.com.br
> Sent: Wednesday, March 23, 2005 3:34 PM
> Subject: [ciencialist] Re: Editor de calculos e formulas
>
>
>
> --- Em ciencialist@yahoogrupos.com.br, "rmtakata"
<rmtakata@a...>
> escreveu
> >
> > --- Em ciencialist@yahoogrupos.com.br, "junior_br2001"
> > > Faz um teste , crie uma lista, depois tente mandar um
vírus, ou
> > > crie um e mande.
> > > Os filtros barram todos os arquvos com as
extensões .exe, .src e
> > > por aí vai.
> >
> > Virus nao se escondem apenas sob extensoes .exe, .src.
Existem
> virus
> > de macro q. se escondem em .doc e ha' ate' comandos
maliciosos
> > em .jpg. Os antivirus nao podem detetar todas as novas
variantes -
> > eles sao atualizados exatamente *depois* q. os primeiros
casos
> > comecam a ser relatados. Os virus estao sempre um passo na
frente.
> >
>
>
> Se voce nao reparou eu coloquei etc... etc.. apos citar as duas
> extensões, mais se for vírus em .doc, eles vao ter que ter
comandos
> que o filtro detecta e barra, o mesmo vle pra .jpeg e outros. O
que
> pode aontecer é a pessoa mandr um link que contenha um
executável que
> dispar instanteneamente(hoje já tem vírus assim). Mas não custa
nada
> instslar um antiviruszinho free para deteecção dos maliciosos,
além
> de usar o spybot
>
> Mas o que eu estava querendo dizer que é que para a pessoa que
> programa em html facilita mandar os calculos de matemática e
física,
> pois ela insere diretamente no código html e manda pelo outlook
e a
> lista recebe maravilhosamente os calculos em html. Facilita
muito
>
>
>
>
>
> ##### ##### #####
>
> Para saber mais visite
> http://www.ciencialist.hpg.ig.com.br
>
>
> ##### ##### ##### #####
>
>
> Yahoo! Grupos, um serviço oferecido por:
>
> São Paulo Rio de Janeiro Curitiba Porto
Alegre Belo Horizonte Brasília
>
>
>
>
> ------------------------------------------------------------------
------------
> Links do Yahoo! Grupos
>
> a.. Para visitar o site do seu grupo na web, acesse:
> http://br.groups.yahoo.com/group/ciencialist/
>
> b.. Para sair deste grupo, envie um e-mail para:
> ciencialist-unsubscribe@yahoogrupos.com.br
>
> c.. O uso que você faz do Yahoo! Grupos está sujeito aos
Termos do Serviço do Yahoo!.
>
>
>
> [As partes desta mensagem que não continham texto foram removidas]
>
>
>
> ##### ##### #####
>
> Para saber mais visite
> http://www.ciencialist.hpg.ig.com.br
>
>
> ##### ##### ##### #####
>
>
> Yahoo! Grupos, um serviço oferecido por:
>
>
>
>
>
>
>
> --------------------------------------------------------------------
----------
> Links do Yahoo! Grupos
>
> a.. Para visitar o site do seu grupo na web, acesse:
> http://br.groups.yahoo.com/group/ciencialist/
>
> b.. Para sair deste grupo, envie um e-mail para:
> ciencialist-unsubscribe@yahoogrupos.com.br
>
> c.. O uso que você faz do Yahoo! Grupos está sujeito aos Termos
do Serviço do Yahoo!.
>
>
>
> [As partes desta mensagem que não continham texto foram removidas]





SUBJECT: Re: Editor de calculos e formulas
FROM: "rmtakata" <rmtakata@altavista.net>
TO: ciencialist@yahoogrupos.com.br
DATE: 23/03/2005 16:03


--- Em ciencialist@yahoogrupos.com.br, "junior_br2001"
> O yahoo so protege que lê as mensagens pelo site como está
> registrado a seguir:
>
> Mensagens e anexos:
> O Yahoo! Grupos continuará permitindo o envio de arquivos anexados
> a mensagens de e-mail, no entanto, estes arquivos não serão mais
> armazenados na área "Mensagens" no site do grupo na web.

Isso para os grupos q. permitem o envio de anexos. Grupos q. nao
permitem envio de anexos nao distribuem para quem usa a leitura por
email.

> Isto quer dizer, por mais vírus ou anexos que voce mande, eles nao
> poderão ser armazenados na área de mensagens. Mas os que leem pelo
> Outloock correm este risco. Mas veja, o mesmo risco se corre ao se
> fazer download na área de arquivos do grupo, não?

Nao. Na area de arquivos sao armazenados por upload - quem coloca o
arquivo la' o faz propositadamente. Arquivos anexos ao email podem
seguir sem q. os donos do endereco de email saibam q. isso estah
acontecendo.

(Claro, ainda assim sempre tem q. se passar um antivirus se for abrir
os documentos na area de arquivos.)

[]s,

Roberto Takata





SUBJECT: Re: Editor de calculos e formulas
FROM: "junior_br2001" <junior_br2001@yahoo.com.br>
TO: ciencialist@yahoogrupos.com.br
DATE: 23/03/2005 16:12


--- Em ciencialist@yahoogrupos.com.br, "rmtakata" <rmtakata@a...>
escreveu
>>
> Isso para os grupos q. permitem o envio de anexos. Grupos q. nao
> permitem envio de anexos nao distribuem para quem usa a leitura por
> email.


Tá me chamando de burro Takata? hehehe
Isso é muito óbvio , nem sei para que voce escreveu isso.

JR






SUBJECT: Re: Editor de calculos e formulas
FROM: "rmtakata" <rmtakata@altavista.net>
TO: ciencialist@yahoogrupos.com.br
DATE: 23/03/2005 16:12


--- Em ciencialist@yahoogrupos.com.br, "junior_br2001"
> Concordo em partees, mas não inteiramente. Pode ser perigoso para
> quem lê as mensagens pelo Outloock , mas pelo site da lista(como um
> grande numeros de pessoas vem fazendo nao tem perigo nenhum:

Nao iremos obrigar (espero falar em nome dos moderadores) q. todos os
membros acessem a lista pela web. Uma parte bem significativa* dos
membros acessam a lista por leitura de email - nao eh razoavel expor
esses usuarios a riscos de outro modo facilmente contornaveis.

*Em uma rapida amostragem:
13 usam o modo leitura de emails individuais
3 usam o resumo diario
1 aviso especial
13 usam a leitura web

[]s,

Roberto Takata





SUBJECT: A variável “Constante Universal da Gravidade”
FROM: "Rolf Guthmann" <rolfguthmann@uol.com.br>
TO: ciencialist@yahoogrupos.com.br
DATE: 23/03/2005 16:13


A inconstância da "Constante Universal da Gravidade"

A mais antiga constante da física, a constante universal da
gravidade ("G"), que oficialmente possui o valor de 6,6726 x 10-11 e
as curiosas unidades de m3.kg-1.s-2(o cubo da distância dividida
pela massa multiplicada pelo quadrado do tempo), demonstrou ser a
mais difícil de determinar. A maioria das constantes físicas possui
precisão de mais de 8 casas decimais, enquanto, valores encontrados
para a "G" apresentam diferenças logo depois da terceira casa
decimal (às vezes até antes). Até hoje não se sabe com precisão o
seu valor, cada vez que alguma nova equipe de pesquisadores aventura-
se com novos e modernos sistemas para determiná-la, são encontrados
valores diferentes.
A seguir, veremos que diversos resultados experimentais, ou
anomalias gravitacionais, questionam a validade da lei universal da
gravidade, conforme Newton. Esta lei estabelece que a força
gravitacional entre dois corpos é proporcional a multiplicação de
suas massas e inversamente proporcional ao quadrado da distância
entre elas, para se obter a força de atração é necessário ainda
multiplicar o resultado pela "G".
Um trabalho intitulado de `Geophysical evidence for non-newtonian
gravity (Nature, v. 292, 1981, pp. 230-232.)', publicado em 1981 por
F.D. Stacey e G.J. Tuck, mostra que medições realizadas abaixo do
nível do mar, no fundo de minas, etc.., apresentavam valores até 1%
superiores ao oficial, sendo que, quanto mais funda a medição, maior
era o valor da "G".
Sabemos da TQG , que quanto maior for à densidade da matéria em
torno do local da experiência, maior definição teremos para a
portadora do tempo ou do presente, como vimos no cap. 2. Átomos
pesados possuem mais elétrons, sabemos que para cada elétron, temos
um vetor de aceleração ou de gravidade que o acompanha, como se
fosse uma sombra, estes são os que geram o campo gravitacional, como
vimos no cap. 8. Como átomos muito ligados temporalmente geram menos
gravidade (exatamente o contrário do que foi visto, na TQG no cap.
7, para as sondas que se afastam do sistema solar), para compensar a
força faltante, devemos aumentar o valor da "G".
O trabalho publicado em 1924 por Charles F. Brush (`Some new
experiments in gravitation', Proceedings of the American Philosophy
Society, v. 63, pp. 57-61.) com uma minuciosa análise fotográfica,
nos mostra que corpos metálicos com átomos mais pesados e densos
tendem a cair mais rápido ou a ter maior força de atração
gravitacional do que corpos com a mesma massa, porém, menos densos
ou com menor número atômico.
Pela TQG (cap. 8), verificamos que átomos com muitos
elétrons geram um campo gravitacional mais uniforme, isto lhes
confere, também, maior força de atração, porque, quanto mais leve
for o átomo, menor a uniformidade do campo gravitacional gerado
pelos vetores de inércia atrelados aos elétrons. No átomo de
hidrogênio esta geração e unilateral, não temos um equilíbrio
geométrico, talvez, este seja um dos motivos da sua difícil
manipulação. Com certeza, neste caso tivemos uma diferença
gravitacional muito sutil, muito pequena, mas suficiente para que
Charles conseguisse detectá-la com fótons.

Em 1798, Henry Cavendish realizou uma experiência, publicada no
Philosophical Transactions, que vem desafiando a física há mais de
200 anos. Utilizando a balança de torção para determinar a constante
da gravidade, ele verificou que quando aquecia as esferas, a força
de atração entre elas aumentava consideravelmente ( veja Stephen
Mooney, `From the cause of gravity to the revolution of science',
Apeiron, 1999, pp. 138-141). Desde então, esta experiência foi
repetida inúmeras vezes, inclusive, em câmaras de alto vácuo,
utilizando-se os mais modernos equipamentos de medição. Mesmo com
todo este poderoso arsenal da física atual, os esforços para
explicar este fenômeno foram em vão ou não muito convincentes.

Sabemos que materiais ou metais aquecidos irradiam ondas
eletromagnéticas ou fótons e, quando isto acontece, temos elétrons
trocando de órbitas ou um aumento de elétrons livres que se alojam
na estrutura cristalina do metal, imaginemos por exemplo, um
filamento de tungstênio conduzindo uma corrente elétrica. Cada vez
que um elétron ligado se espalha, conforme visto na TQG cap. 8, ele
deixa uma lacuna no átomo, aumentando o desequilíbrio entre as
forças nucleares. O elétron permanece nas imediações mantendo a
neutralidade elétrica do material, mas, neste curto período de
tempo, temos um aumento da força de atração coulombiana pelo núcleo
atômico, o que gera mais gravidade. Lembramos, como estabelece a
TQG, que neste caso, temos que considerar o tempo referenciado ao
núcleo atômico.

A cada ano aparecem mais exemplos de anomalias gravitacionais que
desafiam as leis de Newton, principalmente na astronomia, como as
complexas órbitas dos planetas externos ou os polêmicos milhares de
anéis de saturno, etc. Sempre que alguma experiência acusava alguma
pequena ou ínfima disparidade com o valor esperado, a comunidade
científica atribuía isto a erros experimentais, para assim não
comprometer a consagrada e conservadora teoria de Newton.

As experiências para a medição da "G"são realizadas sobre a
superfície de uma esfera com bilhões e bilhões de átomos, que
chamamos de Terra, esta possui dimensões gigantescas se comparado
com os pequenos corpos de prova utilizados, os quais, possuem uma
quantidade muito menor de átomos, consequentemente isto deixa
pouquíssima margem para que se perceba experimentalmente a gravidade
gerada por eles. Estes, simplesmente, sofrem a implacável ação deste
gigantesco campo gravitacional.

Esta desproporção de dimensões planetárias dificultou por mais de
200 anos perceber-se a real natureza da gravidade. Constatamos que
os efeitos gravitacionais entre a teoria de Newton e a nova teoria
da qravidade, a TQG, são muito semelhantes, porque proporcionalmente
a diferença torna-se imperceptível. Numa, a gravidade é conseqüência
da simples presença de massa, enquanto na outra, a gravidade resulta
da diferença relativa das forças nucleares pelos elétrons ligados
nos átomos. Podemos constatar que no final das contas a quantidade
de elétrons envolvidos pela TQG e proporcional a quantidade de massa
considerada por Newton. Considerando isto só encontraremos alguma
diferença se considerarmos as pequenas sutilezas encontradas em
experiências muito precisas. Por outro lado, vimos na TQG (cap. 7),
que estas diferenças se acentuam quando estão envolvidas grandes
distâncias onde o referencial temporal faz a diferença.

veja detalhes em
http://rolfguthmann.sites.uol.com.br/

Rolf Guthmann






SUBJECT: Re: Editor de calculos e formulas
FROM: "junior_br2001" <junior_br2001@yahoo.com.br>
TO: ciencialist@yahoogrupos.com.br
DATE: 23/03/2005 16:14


Ninguém falou em obrigar né?
Foi só uma sugestão, ok?
Se nao der, tudo bem

JR


--- Em ciencialist@yahoogrupos.com.br, "rmtakata" <rmtakata@a...>
escreveu
>
> --- Em ciencialist@yahoogrupos.com.br, "junior_br2001"
> > Concordo em partees, mas não inteiramente. Pode ser perigoso para
> > quem lê as mensagens pelo Outloock , mas pelo site da lista(como
um
> > grande numeros de pessoas vem fazendo nao tem perigo nenhum:
>
> Nao iremos obrigar (espero falar em nome dos moderadores) q. todos
os
> membros acessem a lista pela web. Uma parte bem significativa* dos
> membros acessam a lista por leitura de email - nao eh razoavel
expor
> esses usuarios a riscos de outro modo facilmente contornaveis.
>
> *Em uma rapida amostragem:
> 13 usam o modo leitura de emails individuais
> 3 usam o resumo diario
> 1 aviso especial
> 13 usam a leitura web
>
> []s,
>
> Roberto Takata





SUBJECT: Re: Editor de calculos e formulas
FROM: "rmtakata" <rmtakata@altavista.net>
TO: ciencialist@yahoogrupos.com.br
DATE: 23/03/2005 16:20


--- Em ciencialist@yahoogrupos.com.br, "junior_br2001"
> Tá me chamando de burro Takata? hehehe

O roto nao mofa do rasgado. Eu jah disse q. eu sou o ser mais burro
da Galaxia (do Universo, Chemello, sempre tem um Bush da vida - e
quem perguntar entao: ueh, mas por q. ele, o Bush, nao seria o mais
burro da Terra? alguem acredita q. Bush eh da Terra?), nao cacoaria
de ninguem nessas condicoes.

> Isso é muito óbvio , nem sei para que voce escreveu isso.

Justamente para mostrar a obviedade. Vc disse q. os q. tEm leitura
web estariam protegidos dos anexos; estou apenas dizendo q. sem os
anexos, nao apenas os q. lEem pela web, mas tb os q. lEem por email
(e q. compoem uma fracao consideravel da lista) estariam menos
expostos.

[]s,

Roberto Takata





SUBJECT: Re: Editor de calculos e formulas
FROM: "junior_br2001" <junior_br2001@yahoo.com.br>
TO: ciencialist@yahoogrupos.com.br
DATE: 23/03/2005 16:31


--- Em ciencialist@yahoogrupos.com.br, "rmtakata" <rmtakata@a...>
escreveu
> Justamente para mostrar a obviedade. Vc disse q. os q. tEm leitura
> web estariam protegidos dos anexos; estou apenas dizendo q. sem os
> anexos, nao apenas os q. lEem pela web, mas tb os q. lEem por email
> (e q. compoem uma fracao consideravel da lista) estariam menos
> expostos.


De qualquer forma a leitura por email está com seus dias contato.
Muitos Portais estão migrando par a forma web definitivamente. Sai
muito mais barato para eles e incomodo por usuário que recebem um
número x de mensagens lotando suas caixas de correio. E é justamente
por isso que a opçao web do yahoo vem ganhando mais usuários a cada
dia - o pessoa que frequenta o Yhoo ainda tem uma quedinha pela
leitur por email.
Pode não acontecer aqui no ciencialist, mas a maioria dos grupos do
yahoo estão bem fraquinhos por causa disso.
Aliás os sistemas web lota bem mais por serem mais comodos e praticos
paraa os usuários. Basta ver o ORKUT que tem comunidades científicas
de até 14 mil usuários ou mais, e ainda possibilita o contato com
diversos e um numero muito maior de usuários , enquanto numa lista
como a ciencialist com 550 membros aproximadamente só uns 10
participam ativamente da lista, e ocasionalmente alguns outros.
Isso sem contar com diversos grupos que ficam parados, com uma
afluencia de mensagens de vez ou outra

JR





SUBJECT: Disponibilizar arquivos
FROM: "brudna" <lrb@iq.ufrgs.br>
TO: ciencialist@yahoogrupos.com.br
DATE: 23/03/2005 16:57


Quem deseja disponibilizar algum arquivo (temporario) para o
pessoal da lista pode usar o sistema de ´Arquivos´ do proprio
YahooGrupos ou entao o

YouSendIt

http://www.yousendit.com/

No campo "Recipient's Email Address", indico colocar o proprio mail e
depois copiar o link gerado pelo sistema para dentro do mail que vai
para a lista.

O arquivo fica disponivel por um certo tempo.

Outro sistema semelhante - http://dropload.com/


Ateh
Luis Brudna







SUBJECT: Santo Sudário - Wikipédia
FROM: "E m i l i a n o C h e m e l l o" <chemelloe@yahoo.com.br>
TO: <ciencialist@yahoogrupos.com.br>
DATE: 23/03/2005 17:00

Caros amigos,

Para quem quiser ler um 'bom' material sobre o assunto "Santo Sudário", aqui vai um link da Enciclopédia Livre Wilipédia.
http://pt.wikipedia.org/wiki/Sud%C3%A1rio_de_Turim#An.C3.A1lise_cient.C3.ADfica

Boa leitura

[ ] 's do Emiliano Chemello
emiliano@quimica.net
http://www.quimica.net/emiliano
http://www.ucs.br/ccet/defq/naeq
[ MSN ] chemelloe@hotmail.com
[ ICQ ] 145060604

" Rien ne se perd, rien ne se crée,
tout se transforme."

Antoine Laurent de Lavoisier (químico francês, 1743 - 1794)


[As partes desta mensagem que não continham texto foram removidas]



SUBJECT: Re: Editor de calculos e formulas
FROM: "junior_br2001" <junior_br2001@yahoo.com.br>
TO: ciencialist@yahoogrupos.com.br
DATE: 23/03/2005 17:03


--- Em ciencialist@yahoogrupos.com.br, "rmtakata" <rmtakata@a...>
escreveu
>
> Nao. Na area de arquivos sao armazenados por upload - quem coloca o
> arquivo la' o faz propositadamente. Arquivos anexos ao email podem
> seguir sem q. os donos do endereco de email saibam q. isso estah
> acontecendo.
>
> (Claro, ainda assim sempre tem q. se passar um antivirus se for
abrir
> os documentos na area de arquivos.)


Mas, seria muito inocencia voce abrir um executável de uma pessoa
desconhecida. Uma das mais relevantes lições para usuários iniciantes
em internet, é usar passar o antivírus atualizado ou deletar
mensagens de anexos de desconhecidos. Claro que tem vírus que se
autoexecutam(os servidores estão detectando estes com facilitade),
por isso estes estão meio desaparecidos.
Além disso a mesma dificuldade que um usuário iniciante tem de
utilizar o antivírus, tem tb de inscrever, no Yahoo, em seus grupos e
utiliza-los corretamente, aprendendo a enviar e responder as
mensagens, muitos se confundem nisso, até muitos não sabem nem sair
de uma lista.
O mesmo pode ocorrer eu abrir um executável numa área de arqwuivo de
um desconhecido que coloca um link para lá. Não me parece haver tanta
diferença assim.
E como se recebe as mensagens por email, o mesmo risco se corre com
mensagens que estão vindo de fora da lista de outros lugares, estõ
potencialmentes perigosas em que os usuários *iniciantes* devem ser
bem orientados(os próprios provedores tem alerta e instruções deste
tipo).
Mas em todo o caso , voce falou...então tá falado

JR





SUBJECT: Re: Santo Sudário - Wikipédia
FROM: "junior_br2001" <junior_br2001@yahoo.com.br>
TO: ciencialist@yahoogrupos.com.br
DATE: 23/03/2005 17:09


Encontrei uma falha bem grave neste texto. segue:

"A datação radiométrica por carbono-14 é uma metodologia bastante
precisa quando aplicada a materiais com menos de 2000 anos de idade.
Existem, no entanto, várias fontes de erro que podem induzir
resultados duvidosos. Muita da polémica alimentada pelos defensores e
opositores da autenticidade do sudário foca as possíveis fontes de
erros da datação. Um sumário dos argumentos contra e a favor é
apresentado em baixo, na secção dedicada à controvérsia."

A datação pelo carbono-14 é relativamente precisa em materiais até
50.000 anos e nao em materiais com menos de 2000 anos de idde. Nao
teria sido o autor do texto um cristão?

JR




--- Em ciencialist@yahoogrupos.com.br, "E m i l i a n o C h e m e
l l o" <chemelloe@y...> escreveu
> Caros amigos,
>
> Para quem quiser ler um 'bom' material sobre o assunto "Santo
Sudário", aqui vai um link da Enciclopédia Livre Wilipédia.
> http://pt.wikipedia.org/wiki/Sud%C3%
A1rio_de_Turim#An.C3.A1lise_cient.C3.ADfica
>
> Boa leitura
>
> [ ] 's do Emiliano Chemello
> emiliano@q...
> http://www.quimica.net/emiliano
> http://www.ucs.br/ccet/defq/naeq
> [ MSN ] chemelloe@h...
> [ ICQ ] 145060604
>
> " Rien ne se perd, rien ne se crée,
> tout se transforme."
>
> Antoine Laurent de Lavoisier (químico francês, 1743 - 1794)
>
>
> [As partes desta mensagem que não continham texto foram removidas]





SUBJECT: Re: [ciencialist] Re: Um pedido - mais uma vez
FROM: TARCISIO BORGES <tbs97@fisica.ufpr.br>
TO: ciencialist@yahoogrupos.com.br
DATE: 23/03/2005 17:14

On Wed, 23 Mar 2005, César A. K. Grossmann wrote:
> O Editor de Equações do Word tem dois inconvenientes pequenos: 1. Só
> está disponível na plataforma (sistema operacional) Windows, e 2. Até
> onde eu lembro, só para quem tem o MS-Office.
> César A. K. Grossmann

O Editor de Equações vem com o pacote M$Office, mas roda em vários
programas como PageMaker e Origin.

[]s
TARCISIO BORGES
tbs97@fisica.ufpr.br


SUBJECT: Re: [ciencialist] Re: Editor de calculos e formulas
FROM: TARCISIO BORGES <tbs97@fisica.ufpr.br>
TO: ciencialist@yahoogrupos.com.br
DATE: 23/03/2005 17:40

On Wed, 23 Mar 2005, junior_br2001 wrote:
> De qualquer forma a leitura por email está com seus dias contato.
> Muitos Portais estão migrando par a forma web definitivamente. Sai
> muito mais barato para eles e incomodo por usuário que recebem um
> número x de mensagens lotando suas caixas de correio. E é justamente
> por isso que a opçao web do yahoo vem ganhando mais usuários a cada
> dia - o pessoa que frequenta o Yhoo ainda tem uma quedinha pela
> leitur por email.

Não está com os dias contados não. Faz 10 anos que trabalho em ambiente
Unix e o padrão é o uso do Pine, o qual agora estou usando.

Mas para o uso de listas de discussão o email particular tem a vantagem de
que vc baixa tudo de uma vez e responde posteriormente economizando banda
ou ligação telefônica.

O uso de foruns e semelhantes surgiu para satisfazer um nicho que o email
não cobria. A catalogação de uma quantidade avassaladora de informações
sem a necessidade de que o treco seja totalmente tranferido antes da
consulta. Pelo mesmo motivo o gopher é quase desconhecido por quem usa a
internet a pouco tempo.

[]s
TARCISIO BORGES
tbs97@fisica.ufpr.br


SUBJECT: Re: Editor de calculos e formulas
FROM: "junior_br2001" <junior_br2001@yahoo.com.br>
TO: ciencialist@yahoogrupos.com.br
DATE: 23/03/2005 17:47


--- Em ciencialist@yahoogrupos.com.br, TARCISIO BORGES <tbs97@f...>
escreveu
> On Wed, 23 Mar 2005, junior_br2001 wrote:
> > De qualquer forma a leitura por email está com seus dias contato.
> > Muitos Portais estão migrando par a forma web definitivamente. Sai
> > muito mais barato para eles e incomodo por usuário que recebem um
> > número x de mensagens lotando suas caixas de correio. E é
justamente
> > por isso que a opçao web do yahoo vem ganhando mais usuários a
cada
> > dia - o pessoa que frequenta o Yhoo ainda tem uma quedinha pela
> > leitur por email.
>
> Não está com os dias contados não. Faz 10 anos que trabalho em
ambiente
> Unix e o padrão é o uso do Pine, o qual agora estou usando.
>
> Mas para o uso de listas de discussão o email particular tem a
vantagem de
> que vc baixa tudo de uma vez e responde posteriormente economizando
banda
> ou ligação telefônica.
>
> O uso de foruns e semelhantes surgiu para satisfazer um nicho que o
email
> não cobria. A catalogação de uma quantidade avassaladora de
informações
> sem a necessidade de que o treco seja totalmente tranferido antes da
> consulta. Pelo mesmo motivo o gopher é quase desconhecido por quem
usa a
> internet a pouco tempo.


Não, basta ver a popularização de fóruns como o ORKUT, com comunidade
de 5000 a 25.000 membros num valor aproximado e fóruns como o Yahoo,
e os portais que estão migrando seus sistemas para o sistem web.
Sobre o Pine, e o Unix nao é um sistema para usuários finais. Voce
pode estar fgalando de uma vantaggem pessoal sua. Mas nao para os
esmagadores usuários de Windows.
Basta ver os números, tanto de usuários Windows como os de fóruns WEB
como esses que estão cada vez mais populares. O YahooGrupos para não
morrer(caiu muito os yahoogrupos ultimamente) vai ter que tomar mais
cedo ou mais tarde a mesm medida.

JR






SUBJECT: Re: [ciencialist] Re: Santo Sudário - Wikipédia
FROM: Marcelo Cortimiglia <cortimiglia@gmail.com>
TO: ciencialist@yahoogrupos.com.br
DATE: 23/03/2005 17:57

On Wed, 23 Mar 2005 20:09:20 -0000, junior_br2001
<junior_br2001@yahoo.com.br> wrote:
>
> Encontrei uma falha bem grave neste texto. segue:
>
> "A datação radiométrica por carbono-14 é uma metodologia bastante
> precisa quando aplicada a materiais com menos de 2000 anos de idade.
> Existem, no entanto, várias fontes de erro que podem induzir
> resultados duvidosos. Muita da polémica alimentada pelos defensores e
> opositores da autenticidade do sudário foca as possíveis fontes de
> erros da datação. Um sumário dos argumentos contra e a favor é
> apresentado em baixo, na secção dedicada à controvérsia."
>
> A datação pelo carbono-14 é relativamente precisa em materiais até
> 50.000 anos e nao em materiais com menos de 2000 anos de idde. Nao
> teria sido o autor do texto um cristão?
>

Não vou discutir se o texto citado está certo ou não, pois meus
conhecimentos sobre métodos de datação são quase nulos, mas vou
aproveitar o ensejo para criticar a adoção dos tais "wiki" (entre os
quais a Wikipedia é o mais famoso) como ferramentas de gerenciamento
do conhecimento. Como vejo, o fato de que praticamente qualquer um
pode editar o conteúdo ali publicado é muito positivo por um lado,
pois possibilita a multiplicidade de pontos de vista sobre determinado
tema, bem como contribuições de usuários das mais diversas formações e
especialidades. Por outro lado, se tanta gente tem acesso, é maior a
probabilidade de que alguém acabe escrevendo algo equivocado ou
não-factual sobre determinado tema, intencionalmente ou não. Por isso,
costumo usar os wikis apenas como ponto de partida de uma pesquisa,
nunca como referência confiável.

Abraços,

Marcelo Cortimiglia

> JR
>


SUBJECT: Re: [ciencialist] Re: Editor de calculos e formulas
FROM: Victor Pimentel Nunes <victor2002@gmail.com>
TO: ciencialist@yahoogrupos.com.br
DATE: 23/03/2005 19:39

Júnior escreveu:

"..., enquanto numa lista
como a ciencialist com 550 membros aproximadamente só uns 10
participam ativamente da lista, e ocasionalmente alguns outros."


----------------------------------------------

Sutileza na estatística:

10 (também não sei se esse é o número correto) participam ativamente
da lista POSTANDO (desculpem se parece grito, mas não consigo
sublinhar ou deixar em negrito). Não esqueça de levar em conta nas
estatísticas os que apenas lêem.

Saudações Tricolores
Victor (leitor assíduo)


SUBJECT: Re: Editor de calculos e formulas
FROM: "junior_br2001" <junior_br2001@yahoo.com.br>
TO: ciencialist@yahoogrupos.com.br
DATE: 23/03/2005 20:45



--- Em ciencialist@yahoogrupos.com.br, Victor Pimentel Nunes
<victor2002@g...> escreveu
>>> 10 (também não sei se esse é o número correto) participam
ativamente
> da lista POSTANDO (desculpem se parece grito, mas não consigo
> sublinhar ou deixar em negrito). Não esqueça de levar em conta nas
> estatísticas os que apenas lêem.
>
> Saudações Tricolores
> Victor (leitor assíduo)

Verdade. Muitos participantes são silenciosos, e preferem a leitura,
falta de tempo de muitos tb, eu to qwusse aposentado...hehehe. Mas eu
só quis fazer uma compração sobre a falencia do sistemas de listas
por email em relação aos fóruns Web. Se bem que não são todas as
listas do Yahoo, mas a minoria funcionam legal, enquanto muitas
outras estão às moscas. Eu fiz um levantamento, me inscrevi em 572
listas, umas 10 vingaram. As listas do yahoo tem caído muito
ultimamente com a explosão do ORKUT.
Tb não vou dizer que o sistema ORKUT é melhor que o YAHOO. O Yahoo é
bem melhor, mas o ORKUT tem feito mais sucesso. Eu burro velho de uns
42 anos prefiro mais um Yahoo do que o ORKUT< mas o que eu prefiro é
irrelevante diante da popularidade do ORKUT.

Por coincidencia eu tb sou tricolor... Fluzão do Rio que nunca mais
ganhou nada

Inté
JR






SUBJECT: Re: [ciencialist] Re: Zodiaco - 0ff toppic
FROM: "Silvio" <scordeiro@terra.com.br>
TO: <ciencialist@yahoogrupos.com.br>
DATE: 23/03/2005 21:16

Tenho uns 30 livros sobre astromancia.
Troco pelo "O Ser e o Nada" de Sartre, e o interessado paga o sedex.
sds.,
silvio.

-----Mensagem Original-----
De: "Sergio M. M. Taborda" <sergiotaborda@terra.com.br>
Para: <ciencialist@yahoogrupos.com.br>
Enviada em: sexta-feira, 4 de fevereiro de 2005 22:31
Assunto: Re: [ciencialist] Re: Zodiaco


>
> Oraculo wrote:
>
>> Olá Taborda
>>
>> A respsta é sempre a mesma: prove isso tudo. Demonstre que leoninos
>> são bons pais, que tem relação com seu signo, etc.
>
> Eu não disse que os leoninos são bons pais. Não me obrigue a provar o
> que eu não afirmei , nem é verdade.
>
>>
>>
>> Sem isso (e continuamos sem isso..:-),
>
> E vai continuar sem isso, pq vc quer que eu prove mentiras.
>
>> O caso dos setores é evidente. Se a astrologia não existisse, mas
>> fosse inventada (inventada é um termo preciso..:-)
>
>
> Acaso vc não saiba, a astrologia foi inventada sim. Tal como a fisica, a
> quimica, a electronia , a politica a economia .. . alias como qq
> actividade intelectual do ser humano.
>
>> hoje, teriamos 13 signos, e 13 setores.
>
>
> Prove que sim. Em que se baseia para dizer isso ?
> Ah , ja sei, aquela asneira sobre as constelações. Otimo. Divida o ceu
> em 13 partes, quantos graus tem cada uma ?
> Vc esqueceu que 13 é um numero primo ? Acaso o que lhe ensinaram na
> escola não serve para nada ? Vc não sabe que não se pode divir um numero
> por um numero primo e obter um numero inteiro, a menos que esse numero
> seja um multiplo dele ? Ah ! afinal vc diz-se muito seguidor do
> pensamento logico, mas falha nestes pequeno promenores. Que raios vc
> pensa afinal ?
>
> Vc esquece o elementar. A astrologia indu não usa graus. Foram os
> gregos que intruziram toda a matematica da coisa.
> Se originalmente tivessem escolhido um signo para cada constelação
> teriam escolhido 13. Ou vc acha que a 13º constelação aprecer do dia
> para a noite ? Acha que eles não a viram ?
> Se assim fosse o circulo teria hoje 520 graus (13x4) e não 360.
> Mas não, escolheram 12. que os gregos multiplicaram por 4. O pq do 4 é
> outra historia.
>
> Se a sua teoria sobre as constalções fosse certa, explique pq eles
> ignoraram a 13º.
>
> Tlv isto ajude, vc sabia que o periodo sinodico da lua (29 dias e alguma
> coisa) cai 12, e não 13, vezes no do sol
>
> 365 / 29 = 12
>
> O que sobra é a precessão do zodiaco , e por isso que o zodiaco
> astrologico não é mais sincronizado com o zodiaco celeste.
> Matemática, meu caro, não é magia. É Matemática. E lembre-se que eles
> fizeram o calculo sem computadores e sem medições de periodos siderais
> apenas pelo bom e velho metodo que aprendemos na escola primária.. .bom,
> pelo menos eu aprendi.
>
> Vc imagina que a astrologia foi inventada como ? Baseada em quê ?
> Palhaçada ?
> Prove-se que faz sentido logico-amtematico-astronomico escolher o numero
> 13 (que é um numero primo) e darei o braço a torcer.
>
>> Como poderia alguém que estivesse criando a astrologia hoje determinar
>> que um dos setores não deveria ser utilizado? Como ele teria acesso a
>> essa informação, se existem 13 constelações a considerar ?
>
> Se vc soubesse mais de astronoia básica, saberia explicar pq, tal como
> eu fiz acima.
>
>>
>> Como o astrologo nos informou, valem os setores criados pelos
>> assirios, com base em seu céu visivel na época.
>
> E como eles os criaram ? Por magia ? Vamos, lá., vc - o ceptico -
> acreditando em magia ?
>
>> Se criada hoje, a astrologia seria de 13 signos e totalmente diferente
>> em seus significados e interpretações e os mapas astrais diferentes
>> nos resultados.
>
> Vc continua repetindo isso mas sem explicar pq . Qual a razão , a regra,
> que faria isso acontecer ? Pq 13 ?
>
>> Como pode isso ser uma fonte de conhecimento confiável ou mesmo real?
>
> A sua ou a minha ?
>
>> Conhecimento que determinava que o Sol e a Lua eram planetas e agiam
>> como tal, inclusive girando em torno da Terra (a Lua até que gira,
>> embora não como os planetas, mas o Sol..:-).
>
> Se vc soubesse mais de fisica, saberia que o sol sim gira em torno da
> terra , tal como a lua, tal tudo o resto, para quem está na terra. Para
> o sistema do observador solidário com a terra, tudo o resto anda À volta
> dela. É por falta deste conhecimento que pouca gente entende
> Relatividade (Galiena mesmo, para não falar da outra). A Primeira regra
> é que o que vc observa depende do referencial que esolhe.
> A astrologia, não nega que o seu referencial é a Terra. E como tal, é
> correcto, observatorialmente e até fisicamente, dizer que é o sol que se
> move em torno da terra.
>
> Toda a discussão sobre se o sol era o centro ou não era uma discussão
> astrologia. Tlv vc não soubesse disto, e pensasse que era uma discussão
> fisica, ou filosofica... .
> Mas claro, vc não acredita nisso.
>
> Planeta é uma palavra grega , femenina , que significa Estrela Errante.
> Qq fisico sabe que aquilo que não se move, não contem informação. É no
> movimento, na mudança das coisas, que está a informação. Então é nos
> Planetas, Estrelas que se movem, e não nas Estrelas fixas que deveriamos
> concentrar o estudo. É por isso que em astrologia chamamos a lua e o sol
> de planetas. Não é para chatear os astronomos. É para ser fiel ao
> significado das palavras. Pq eles são estrelas que se movem (são astros
> luminosos)
> Mas pq Astrologia e não Planetalogia ? Pq o que a astrologia realmente
> estuda não são os planetas em si, suas posições etc... isso é apenas uma
> ferramenta. O que interessa são os arranjos que eles fazem uns com os
> outros no ceu. A forma como eles se agrupam. A forma como as estrelas
> errantes de dispoem relativamente umas às outras.Aquilo que hoje
> chamamos Aspectos. Mas como se chamava a disposição de estrelas umas em
> relação às outras no tempo dos gregos?
> O mesmo que hoje :Constelação. Que em grego se diz, Astron e dai
> Astron+Logia = Astrologia = Estudo/Catalogo das Constelações , dos
> arranjos das estrelas.
> Mas entenda de que estrelas estamos falando aqui!
> Estamos falando das estrelas que se movem, as Planetas e não das
> Estrelas fixas. UAU! Agora vc ficou confuso , sim ? Otimo.
>
> A astrologia é o estudo das estrelas e das constelações, mas não
> "estrela" e "constelaçao" no sentido vulgar que vc dá a essas palavras
> hoje , e sim, a que era dada à milhares de anos atrás. Eles não faziam
> destinção ente Estrela e Planeta como os astornomos. Tudo eram estrelas,
> mas umas moviam-se e outras não. (Tudo isto do ponto de vista de quem
> olha da Terra) . As que não se movem não contém informação e server
> apenas como referencia visual do referencial asbtracto que estamos
> usando. As que se movem, as Planetas, forma configurações diversas no
> ceu, e são essas configurações que estavam interessados em estudar.
> Esses arranjos, essas constelações de estrelas moveis, e não as
> constelações de estrelas fixas que não têm nenhum importancia para o
> fenomeno. (Um referencial, nunca tem)
>
> Deu para entender pq a astrologia é aquilo que é e pq os argumentos como
> os seus e os do takata são ocos de sentido logico ? Pq eles são mais
> bestas do que os argumentos dos astrologos em si ?
> Se não deu é pura má fá da sua parte. Acho que o texto é bem explicativo.
>
>> Sempre mais do mesmo. Apenas prove o que afirma, e nenhuma discussão
>> seria necessária.
>
> Acabei de o fazer.
> Claro que vc não está convencido, pq vc escolhe não entender, mas não
> pode negar que a logia está ali. A historia, a metamática, a fisica e
> até a semântica, confirmam.
> Que mais provas vc quer ?
>
> Sérgio Taborda
>
>
> --
> No virus found in this outgoing message.
> Checked by AVG Anti-Virus.
> Version: 7.0.300 / Virus Database: 265.8.5 - Release Date: 03-02-2005
>
>
>
> ##### ##### #####
>
> Para saber mais visite
> http://www.ciencialist.hpg.ig.com.br
>
>
> ##### ##### ##### #####
> Links do Yahoo! Grupos
>
>
>
>
>
>
>
>
>
>



SUBJECT: Re: [ciencialist] Re: Zodiaco e Acupuntura..:-)
FROM: "Silvio" <scordeiro@terra.com.br>
TO: <ciencialist@yahoogrupos.com.br>
DATE: 23/03/2005 21:29

Mestre Manoel:

Tive uma hérnia de disco, tratei durante anos com médicos ortopedistas e
massagens e acupuntura aplicadas por dois mestres japoneses (trabalhava na
Usiminas).
Adiantou bulhufas: só depois de cirurgia houve melhora significativa.

Os tratamentos alternativos são bons para quem os aplica e fatura dos
pacientes: para casos reais nada melhor que a medicina tradicional desde que
praticada por especialistas. Na oncologia então, nem pensar: trate um câncer
com homeopatia e acupuntura e ganha logo um pijama de madeira.
Quem contestar, dê um pulo num hospital especializado em câncer, converse
com os pacientes.

um abraço,

silvio.

Manuel Bulcão pontifica:

Salve, Homero!

Homero escreveu: Recentemente, detectou-se algum efeito na sintese
de prostaglandinas ao se aplicar agulhas na pele de pacientes. Como
as prostaglandinas estão envolvidas nas sensações dolorosas, isso
pode causar algum efeito do tipo anestésico leve pelas agulhas.

Manuel: Talvez o efeito das agulhas seja um pouco mais potente que a
de um analgésico leve. Parece que a técnica da "acupuntura" (as
aspas são para lhe agradar :-)) é eficaz como tratamento
complementar de dores "crônicas", como a fibromialgia e até mesmo
dores oncológicas.

Homero: Mas, isso NÃO É MAIS ACUPUNTURA. (...) Não exige que os
meridianos sejam identificados, nào exige que pontos especificos
sejam encontrados, e pode ser substituída por beliscões, em seu
efeito nas prostaglandinas.

Manuel: Foi questionando e testando a técnica da "acupuntura" que a
medicina ocidental chegou à conclusão de que, "realmente", a
estimulação de certos pontos do corpo produz como resposta -- via
secreção de determinados hormônios e neurotransmissores -- analgesia
e sedação. A "acupuntura" PREVÊ esse efeito sensível, previsão esta
que foi confirmada pelos testes empíricos. Significa dizer que a
práxis científico-experimental corroborou a "acupuntura" em alguma
medida.

Além disso, parece que, consoante esses testes, e malgrado a
explicação mágica da medicina sino-nipônica arcaica, muitos pontos
corporais que correspondem aos "meridianos" não são arbitrários.

Quanto ao desequlíbrio de "energia" que causa a doença, bem, pelo
menos no que diz respeito à supressão ou amenização dos sintomas
decorrente das agulhadas, isso se deve a processos "eletroquímicos"
(portanto, "energéticos") que começa com um "choque". Não é a
energia que a turma do new age & Cia. sonha, mas é energia, sem
dúvida.

Acho que o problema está mais na forma da explicação do que no poder
de predição (não quero com isso dizer que o poder de predição de
todas as fórmulas da "acupuntura" sejam supimpas, frise-se). Mas,
será que a explicação dos fenômenos naturais EM LINGUAGEM NÃO-
MATEMÁTICA é unívoca? Há quem sustente que qualquer lei física pode
ser enunciada VALIDAMENTE até mesmo na forma de uma lenda ou conto
de fadas em que as forças da natureza figuram como sujeitos
intencionais. Por exemplo, numa dessas estórias da Carochinha, os
glúons seriam tutores liberais que só intervêm duramente quando os
tutelados (quarks) estão passando dos limites (liberdade
assintótica).

No que tange ao termo designativo dessa técnica terapêutica,
pergunto: há uma palavra melhor para nomeá-la que não
seja "acupuntura"? Será que os fatos acima mencionados não a
legitimam hoje em dia? ô, Homero, meu amigo, você que foi tão firme
ao afirmar "ISSO NÃO É MAIS ACUPUNTURA', apresente uma alternativa!

A ciência deve tanto ao conhecimento que a precedeu (ao conhecimento
empírico rasteiro e à metafísica) que tomou para si muitos das suas
imagens, entre as quais a palavra "átomo": termo cuja etimologia o
torna impróprio para designar qualquer coisa que seja composta,
divisível, quebrável, redutível ou desintegrável -- como o é
um "átomo" de urânio.

Então, por que não preservar também o termo "acupuntura"?

Confetes e serpentinas,
Manuel Bulcão





##### ##### #####

Para saber mais visite
http://www.ciencialist.hpg.ig.com.br


##### ##### ##### #####
Links do Yahoo! Grupos











SUBJECT: nesta ponte não passo.
FROM: "Silvio" <scordeiro@terra.com.br>
TO: <ciencialist@yahoogrupos.com.br>, <Conversa_de_Botequim@yahoogrupos.com.br>, <acropolis@yahoogrupos.com.br>
DATE: 23/03/2005 21:36



A ponte entre a Ciência e a Religião



Transcrição completa da entrevista concedida pelo físico Amit Goswami ao programa "Roda Viva" da TV Cultura.



O Roda Viva entrevista o físico nuclear indiano AMIT GOSWAMI. Considerado um importante cientista da atualidade ele tem instigado os meios acadêmicos com sua busca de uma ponte entre a ciência e a espiritualidade. Amit Goswami vive nos Estados Unidos. É PHD em física quântica e professor titular de física da Universidade de Oregon. Há mais de quinze anos está envolvido em estudos que buscam construir o ponto de união entre a física quântica e a espiritualidade. Já foi rotulado de místico, pela comunidade científica, e acabou acalmando os críticos através de várias publicações técnicas a respeito de suas idéias. Em seu livro O UNIVERSO AUTOCONSCIENTE - publicado no Brasil - ele procura demonstrar que o Universo é matematicamente inconsistente sem a existência de um conjunto superior - no caso, DEUS. E diz que, se esses estudos se desenvolverem, logo no início do terceiro milênio Deus será objeto de ciência e não mais de religião.



A bancada de entrevistadores será formada por Mário Sérgio Cortella, filósofo e dir.em educação, prof. do Depto. Teologia e ciências religião da Puc SP; Cláudio Renato Weber Abramo, jornalista e mestre em filosofia da ciência; Pierre Weil, educador e reitor da Universidade Holísitica Internacional de Brasília; Rose Marie Muraro, escritora e editora; Leonor Lia Beatriz Diskin Pawlowicz, jornalista e Pres.da Assoc. Palas Athena; Joel Sales Giglio, psiquiatra, ex chefe do Depto.de Psic. Médica e psquiatria da Unicamp, analista junguiano da Assoc. Junguiana do Brasil e membro da International Assossiation for Analitical Psychology; Carlos Ziller Camenietzki, físico, dr. em filosofia e pesquisador do Museu de Astronomia do Min. da Ciência e Tecnologia.



Heródoto Barbeiro: Dr. Amit Goswami, Boa Noite. Inicialmente eu gostaria que o senhor dissesse aos telespectadores da TV Cultura, que ao longo do século XX os cientistas estiveram ligados muito mais ao materialismo do que à religiosidade. A impressão que eu tenho é que nessa virada para o século XXI, essas coisas estão mudando. O senhor poderia nos explicar o porque dessa aproximação entre a ciência e a espiritualidade?



Amit Goswami: Com prazer. Esta mudança da ciência, de uma visão materialista para uma visão espiritualista, foi quase totalmente devida ao advento da Física Quântica. Ao mesmo tempo, houve algumas mudanças em Psicologia transpessoal, em Biologia evolucionista, e em medicina. Mas acho que é correto dizer que a revolução que a Física Quântica causou na Física, na virada do século, seria baseada nessas transições contínuas, não apenas movimento contínuo, mas também descontínuo. Não localidade. Não apenas transferência local de informações, mas transferência não-local de informações. E, finalmente, o conceito de causalidade descendente. É um conceito interessante, pois os físicos sempre acreditaram que a causalidade subia a partir da base: partículas elementares, átomos, para moléculas, para células, para cérebro. E o cérebro é tudo. O cérebro nos dá consciência, inteligência, todas essas coisas. Mas descobrimos, na Física Quântica que a consciência é necessária, o observador é necessário. É o observador que converte as ondas de possibilidades, os objetos quânticos, em eventos e objetos reais. Essa idéia de que a consciência é um produto do cérebro nos cria paradoxos. Em vez disso, cresceu a idéia de que é a consciência que também é causal. Assim, cresceu a idéia da causalidade descendente. Eu diria que a revolução que a Física Quântica trouxe, com três conceitos revolucionários, movimento descontínuo, interconectividade não-localizada e, finalmente, somando-se ao conceito de causalidade ascendente da ciência newtoniana normal, o conceito de causalidade descendente, a consciência escolhendo entre as possibilidades, o evento real. Esses são os três conceitos revolucionários. Então, se houver causalidade descendente, se pudermos identificar essa causalidade descendente como algo que está acima da visão materialista do mundo, então Deus tem um ponto de entrada. Agora sabemos como Deus, se quiser, a consciência, interage com o mundo: através da escolha das possibilidades quânticas.



Rose Marie Muraro: O que mais me espanta na Física é o problema da medição quântica de Heisemberg, que voce, realmente, acha que deve ter um observador olhando e que modifica a realidade, por exemplo, transforma a onda em partícula. Eu gostaria de saber... isso aí houve uma grande briga de Einstein com Niels Bohr. Eu gostaria de saber, em escala cósmica, onde não há observadores, se há um observador supremo, na sua opinião, e se ele cria matéria ou como se faz esse fenômeno?



Amit Goswami: Essa é a questão fundamental, Rose Marie, porque.. qual é o papel do observador? É a pergunta que abre a integração entre Física e espiritualidade. Na Física Quântica, por sete décadas, tentou-se negar o observador. De alguma forma, achava-se que a Física deveria ser objetiva. Se dessem um papel ao observador, a Física não seria mais objetiva. A famosa disputa entre Böhr e Einstein, a que se refere essa disputa, basicamente, sempre terminava com Bohr ganhando a discussão, mostrando que não há fenômeno no mundo a menos que ele seja registrado. Bohr não usou a consciência.. mas atualmente, vem crescendo o consenso, muito lentamente, de que a Física Quântica não está completa, a menos que concordemos que nenhum fenômeno é um fenômeno, a menos que seja registrado por um observador, na consciência de um observador. E isso se tornou a base da nova ciência. É a ciência que, aos poucos, mas com certeza, vem integrando os conceitos científicos e espirituais.



Cláudio Abramo: Em sua fala inicial, o senhor mencionou, deu como fato, que teria crescido a idéia de que haveria uma causalidade no sentido inverso àqueles do tradicional que se considera, e daí saltou para a afirmação de que isso abriria a porta para a entrada de Deus. A minha pergunta se divide em duas. Em primeiro lugar, essa idéia cresceu aonde? Quem, além do senhor, defende esse tipo de visão de mundo? E... dois, o porque Deus entrou aí nessa equação?



Amit Goswami: Na Física Quântica há um movimento contínuo. A Física Quântica prevê isso. Não há dúvida que a Matemática Quântica é muito capaz, muito competente, e ela prevê o desenvolvimento de ondas de possibilidades, a matéria é retratada como ondas de possibilidades. O modo como elas se espalham é totalmente previsto pela Física Quântica. Mas agora temos probabilidades de possibilidades. Nenhum evento real é previsto pela Física Quântica. Para conectar a Física Quântica a observações reais, embora não vejamos possibilidades e probabilidades, na verdade vemos realidades. Esse é o problema das medições quânticas. E luta-se com esse problema há décadas, como eu já disse, mas nenhuma solução materialista, uma solução mantida dentro da primazia da matéria foi bem sucedida. Por outro lado, se considerarmos que é a consciência que escolhe entre as possibilidades, teremos uma resposta, mas a resposta não é matemática. Teremos de sair da matemática. Não existe Matemática Quântica para este evento de mudança de possibilidades em eventos reais, que os físicos chamam de 'colapso da onda de possibilidade em realidade'. É essa descontinuidade do colapso que nos obriga a buscar uma resposta fora da Física. O que é interessante é que se postularmos que a consciência, o observador, causa o colapso da onda de possibilidades, escolhendo a realidade que está ocorrendo, podemos fazer a pergunta: qual é a natureza da consciência? E encontraremos uma resposta surpreendente. Essa consciência que escolhe e causa o colapso da onda de possibilidades não é a consciência individual do observador. Em vez disso, é uma consciência cósmica. O observador não causa o colapso em um estado de consciência normal, mas em um estado de consciência anormal, no qual ele é parte da consciência cósmica. Isso é muito interessante. O que é a consciência cósmica diante do conceito de Deus, do qual os místicos e teólogos falam?



Mário Cortella: Uma questão para o doutor Amit que é a seguinte: o senhor é originado de uma cultura, que é a cultura da Índia, onde o hinduísmo, como religião, tem uma profusão de deuses ou de divindades, ou de deidades. Alguns chegam a falar em 300 milhões de deidades dentro da religião hindu. De outro lado, seu pai foi um guru brâmane, o senhor tem um irmão que é filósofo. Esta mescla de situações induziu no senhor uma compreensão em relação a um ponto de chegada, na religião, partindo da Física, ou o senhor já partiu da religião e, por isso, chegou até a Física e supõe que a Física Quântica é uma das formas de praticar teologia?



Amit Goswami: Obrigado pela pergunta, porque costumam me perguntar se minha formação como indiano hindu afeta o modo como pratico a Física. Na verdade, fui materialista por um bom tempo. Fui físico materialista dos 14 anos de idade até cerca de 45 anos. O materialismo foi importante para mim. Eu trabalhei com ele, filosofei nele, cresci nele. Eu obtive sucesso em Física dentro da Física materialista. Mas quando comecei a trabalhar no problema da medição quântica, eu realmente tentei resolvê-lo dentro do materialismo. Enquanto todos nós trabalhávamos, falei com muitos físicos que trabalhavam no problema (este é o problema mais estudado da Física, um dos mais estudados). E todos tentávamos resolver este paradoxo: se a consciência é um fenômeno cerebral, obedece à Física Quântica, como a observação consciente de um evento pode causar o colapso da onda de possibilidades levando ao evento real que estamos vendo? A consciência em si é uma possibilidade. Possibilidade não pode causar um colapso na possibilidade. Assim, eu tive de abandonar esse pensamento materialista. Embora fosse interessante, em minha vida pessoal eu sentia necessidade de mudar. Alguns consideraram uma transição de meia-idade, e os dois problemas, crescimento na vida pessoal e o problema da medição quântica, se confundiram, e eu comecei a ver a consciência não apenas como um problema físico, mas também como um problema pessoal. O que é que deixa alguém feliz? Qual é a natureza da consciência, da qual as pessoas falam quando se pensa além do materialismo? Então, comecei a meditar e a me aproximar de alguns místicos, e isso ajudou. E um dia, quando falava com um místico, e ele me dava a tradicional visão mística do mundo, que eu já ouvira muitas vezes antes, mas, de algum modo, essa conversa causou uma nova impressão em mim. Eu pude ver, eu realmente vi além do pensamento, tive a percepção de que a consciência é a base do ser, e essa percepção soluciona o problema da medição quântica. Não só isso: pode ser usada como base para a ciência. Normalmente, os cientistas presumem que a ciência deve ser objetiva, etc, mas eu vi, naquele momento, que a ciência deve ser objetiva até um certo ponto. Eu chamo de objetividade fraca, mas isso pode ser alcançado nessa nova Metafísica. Consciência é a base de todos os seres. Então, para mim, foi o contrário, eu fui da Física para a espiritualidade, sob o aspecto da Física. Porque minha formação espiritual, embora em retrospecto, eu possa dizer que foi saudável, deve ter sido, como Freud diria, no subconsciente. Mas conscientemente foi o oposto. Eu vim de uma questão muito inquietante, de como resolver um problema físico, um problema do mundo, pois esse é o problema mais importante do século XX. E a partir disso, esse salto conceitual, esse salto quântico perceptivo me fez reconhecer que o modo como espiritualistas vêem a consciência é o modo certo de ver a consciência. E esse modo de ver a consciência resolve o problema da medição quântica. Ele nos dá a base para uma nova ciência.



Carlos Ziller: Eu gostaria de fazer uma pergunta, dando um passo mais atrás no sentido da própria Física clássica. Porque nós sabemos, hoje em dia, que os fundadores da Física clássica, Newton, Déscartes e outros grandes cientistas do século XVII, para eles, para os projetos científicos que propunham, Deus era uma parte constitutiva inseparável do mundo que eles imaginavam, seja como sendo quem garantia a eficácia, eficiência, o funcionamento das leis do mundo, seja como alguém que operava os próprios fenômenos naturais. Bom, isso foi sendo afastado, expulso do mundo da ciência ao longo do século XVIII, século XIX, ou século XX, talvez, até os anos 50 tenha sido o ápice dessa questão, os cientistas, os físicos, sobretudo, não gostavam totalmente nada de falar sobre esse assunto. Deus era um problema. Talvez o seu estudo e a sua reflexão esteja tentando recolocar no seu próprio lugar, pelo menos foi assim que eu interpretei, algumas idéias do próprio século XVII, dos fundadores da ciência moderna. Eu gostaria de saber se essa aproximação do Deus do Newton, o que garantia que as leis naturais funcionavam, se esse Deus tem algum paralelo com a consciência, supra-consciência que o senhor propõe como sendo o princípio a partir do qual os fenômenos do mundo, a realidade estaria constituída?



Amit Goswami: É uma pergunta muito boa. Os conceitos da Física clássica, no início, não separavam Deus, como disse, mas então, aos poucos, descobriu-se que Deus não era necessário. Depois que Deus estabeleceu o movimento do mundo, ele passou a ser guardião de seu jardim, e isso é o que a maioria dos físicos clássicos pode fazer. Mas na Física Quântica, há o problema da medição. Como as possibilidades tornam-se eventos reais, temos espaço para uma consciência, e ela deve ser uma consciência cósmica. Há uma semelhança com o modo como Deus é retratado, pelo menos na subespiritualidade tradicional, não na mente popular. A mente popular considera Deus um imperador, um super-humano sentado no céu. Essa imagem de Deus não é científica, e espero que esteja claro que não estamos falando em Deus dessa forma, mas Deus nessa consciência mais cósmica, nessa forma mais estrutural. Esse tipo de Deus está retornando porque, se voce se recorda, o debate entre teólogos e cientistas sempre foi: Deus é o guardião ou Deus intervém? Teólogos afirmam que Deus intervém nos seres biológicos. E então surgiu Darwin. Foi um grande golpe nos teólogos, porque antes, apesar de Newton, os teólogos podiam citar o exemplo da Biologia, cujo propósito é muito óbvio, pelo menos, óbvio para a maioria. Mas a teoria de Darwin foi um golpe porque se dizia que a evolução ocorria... mas ela era natural? Darwin disse que ela era natural. Oportunidade e necessidade. Não há necessidade de Deus na evolução e não há necessidade de Deus na biologia. Então, no século XX, surgiu o behaviorismo e a idéia de que temos livre-arbítrio subjetivo. Essa idéia também foi superada, porque experimentos mostraram que somos muito condicionados, não há livre-arbítrio. Contra tudo isso, vejam só, a Física Quântica também cresceu ao mesmo tempo que o behaviorismo, e a Física Quântica tem uma coisa peculiar: o princípio da incerteza. O mundo não está determinado como imaginamos. Deus não é o guardião. O princípio da incerteza levou à onda de possibilidades, depois o colapso da onda de possibilidades para a introdução da idéia do colapso da consciência. Paradoxalmente, fomos criados contra essa idéia, mas nos anos 90, eu, Henry Stab, Fred Allan Wolf, Nick Herbert, todos mostramos que esse paradoxo pode ser resolvido. Não há paradoxo se presumirmos que a consciência que causa o colapso da onda de possibilidades em eventos reais é uma consciência cósmica. E o evento do colapso em si nos dá a separação matéria-objeto do mundo. Assim, não só resolvemos o problema da medição quântica como também demos uma nova resposta de como a consciência de um torna-se várias. Como ela se divide em matérias e objetos, para poder ver a si mesma. E essa idéia de que o mundo é um jogo da consciência, um jogo de Deus, que é uma idéia muito mística, voltou à tona. Então, podemos voltar à biologia. Deus intervém na biologia? Deus intervém na vida das pessoas? Essas perguntas continuam tendo respostas muito positivas. Vi, em um jornal sobre Biologia evolucionista, que há muitos furos conhecidos na teoria darwiniana. Esses furos são chamados sinais de pontuação. A teoria da evolução de Darwin explica alguns estágios homeostáticos da evolução, ou seja, como as espécies adaptam-se a mudanças ambientais. Mas não explica como uma espécie torna-se outra. Essa especiação, mudança de uma espécie em outra, é uma nova mudança na evolução, não está na teoria de Darwin. Experimentalmente, isso é demonstrado em lacunas de fósseis. Não temos uma continuidade de fósseis mostrando como um réptil tornou-se um pássaro. A idéia é que sejam sinais de pontuação, estágios muito rápidos de evolução. Eu sugiro que isto seja um salto quântico, um salto quântico na evolução. Nesse salto quântico, a consciência interveio, não de um modo subjetivo, de um modo caprichoso, mas de um modo muito objetivo.. muito objetivo, e essas idéias objetivas ficam claras com o trabalho de Rupert Sheldrake e outros, o modo como isso pode ser objetivo. Mas, sem dúvida alguma, há uma intervenção da causalidade descendente. Não se pode explicar a Biologia evolucionista só com a causalidade ascendente. Essa é a coisa mais interessante, a partir do pensamento original dos físicos de que Deus deve ser o guardião, pois tudo pode ser explicado e tudo é determinado, que não precisamos de Deus. Agora, estamos fechando o círculo, e vemos que não só precisamos de Deus: há movimentos descontínuos no mundo para os quais não existe explicação matemática ou lógica. Ainda assim, é totalmente objetivo, não é arbitrário. Deus age de forma objetiva, bem definida. A consciência cósmica não é subjetiva, não é a consciência individual que afeta o mundo. Isso ocorre de forma cósmica, podemos discutir objetivamente. A ciência detém seu poder, sua objetividade e, ainda assim, temos agora a descontinuidade, temos a interconectividade e podemos falar sobre vários assuntos dos quais os místicos tradicionalmente falam.



Pierre Weil: Durante essa discussão eu me coloquei como educador do ponto de vista do telespectador, e estou um pouco com medo de que alguns já desligaram o aparelho diante do alto nível científico do debate, que é necessário e indispensável. Eu queria ressaltar a importância da sua presença aqui em termos mais simples. Para o telespectador... tem telespectadores que acreditam em Deus, acreditam em espiritualidade e tem outros que não acreditam em Deus, não acreditam... são os materialistas versus os espiritualistas. Entre os dois têm os que não sabem ou os que nem se interessem para isto. Nestas três categorias, a sua presença aqui tem uma importância muito grande. Ela tem uma importância porque nesse século que passou, nós estivemos assistindo a três grandes movimentos: o primeiro movimento, em que muitos espiritualistas, muitas pessoas que acreditavam em Deus, abalados pelas "provas", pelas evidências da ciência, largaram a religião e só acreditaram na matéria. E nisso foram até muitos sacerdotes de várias religiões. Largaram a batina, largaram a sua fé e se transformaram em protagonistas do materialismo. Estamos assistindo, atualmente, a um movimento contrário. Eu tenho, por exemplo, dois amigos meus. Um, Matew, grande biólogo francês, largou a biologia e hoje ele é monge budista tibetano. O outro era astrofísico, colega seu, largou a astrofísica e hoje ele é rabino. Então estamos assistindo a um movimento contrário. A sua presença aqui apresenta uma terceira saída, e que me parece a mais conveniente e a mais razoável, a mais holística, que é a minha também. A sua, como Física Quântica, fez com que, vindo do materialismo, não caísse no extremo do espiritualismo, mas integrou os dois. Eu fiz isso também como psicólogo, através da psicologia transpessoal... o senhor através da Física Quântica, eu, através da Psicologia Transpessoal... e nos encontramos muito bem e nos abraçamos o tempo todo. A minha pergunta é uma pergunta pessoal: poderia contar para os telespectadores, em termos mais simples, o que fez com que Amit Goswami ficasse no meio do caminho e fizesse um encontro dentro dele, da razão da Física, da razão materialista, e do outro lado, da Intuição? Falou nos seus amigos místicos, mas pela minha experiência eu sei que a segurança pela qual eu falo, não é apenas racional, ela é baseada numa experiência chamada interior, chamada subjetiva, chame como quiser, de luz, e de saber mais ou menos como que é esse mundo espiritual. Qual é a sua experiência que fez com que unisse, na sua pessoa, o lado masculino, racional, e o lado feminino, intuitivo, sentimental? O que aconteceu com a sua pessoa? Eu acho que isso nos vai reconciliar com os telespectadores.



Amit Goswami: Sim, obrigado. Esta é a questão fundamental. Às vezes, eu digo que todos nós, todas as pessoas, espectadores, cientistas, o orador, todos aqui, todos nós temos dois lados. Um é semelhante a Newton, que quer entender tudo em termos de objetividades, ciências e matemática, e o outro é William Blake, que é místico e ouve diretamente, intuitivamente, e desenvolve seu retrato do mundo baseado nessa percepção intuitiva. O que ocorre nessa integração, o que ocorreu por um tempo, mesmo antes de essa integração começar, é que começamos a entender a natureza da criatividade. E a falsa idéia de que cientistas só trabalham com idéias racionais e matemáticas, está, aos poucos, caindo. Einstein disse isso muito claramente: "Não descobri a Teoria da Relatividade apenas com o pensamento racional". As pessoas não levam a sério tais declarações. Mas Einstein falou sério. Ele sabia que a criatividade era importante. Agora, quase cem anos de pesquisas sobre criatividade estão mostrando que os cientistas também dependem da intuição. Eles também dependem de visões criativas para desenvolver sua ciência. Nem tudo é racional, matemático; nem tudo é pensamento racional. Voce perguntou sobre minha experiência pessoal. Eu já compartilhei a experiência fundamental pessoal que tive quando troquei... nem devo dizer que troquei, eu tive uma percepção. Não posso descrevê-la em termos de espaço-tempo. Eu estava fora do espaço-tempo, experimentando diretamente a consciência como a base do ser. É esse tipo de experiência que dá a base para ficarmos convencidos, para termos certeza de que a realidade é algo mais do que o espaço-tempo no mundo em movimento faz parecer. Este é o escopo fundamental para o ponto de encontro dos cientistas e espiritualistas. Porque os espiritualistas ouviram esse chamado, essa intuição, muito antes. Os cientistas também a ouviram. Mas por eles sempre expressarem suas percepções em termos de lógica, em termos de razão, isso ocorre mais tarde. Eles esquecem a origem de seu trabalho, a origem de sua percepção. Já para os espiritualistas, a percepção leva à transformação do modo de vida. Assim, eles nunca esquecem que foi a intuição que trouxe a felicidade, foi ela que os fez quem são. Essa é a diferença. Cientistas usam a intuição para desenvolver sistemas que estão fora deles, o que chamo de criatividade externa. E isso torna-se uma camuflagem dos verdadeiros mecanismos do mundo para eles. Enquanto espiritualistas mantêm-se com a percepção, mudam suas vidas, e incidentalmente, mudam o mundo externo. Mas eles sabem que aquela percepção que tiveram é a coisa fundamental que gere o mundo. Para eles, a consciência é cósmica, isto é algo determinado. Para os cientistas, a mesma descoberta é possível, mas eles ignoram o chamado e prestam mais atenção ao que ocorre no cenário externo. Acho que, se todos nós compartilharmos isso, o mundo poderá mudar. Agradeço pela pergunta. Estou disposto a compartilhar: escrevi um livro sobre criatividade, no qual conto minhas histórias pessoais. Em todos os meus livros conto minhas histórias pessoais. É importante compartilharmos nossas histórias pessoais, e acabar com o mito de que os cientistas são apenas pensadores racionais. Eles também têm percepções que vão muito além do pensamento racional.



Heródoto Barbeiro: Doutor Goswami, o senhor falou muito em Deus durante a primeira parte deste programa, e aqui no ocidente, quando se fala em Deus, se imagina que exista o seu contraponto. E aqui no ocidente se dá uma série de nomes a ele. Eu gostaria de saber como é que o senhor explica essa... se o senhor concebe a existência desse contraponto, dessas outras forças que não são necessariamente Deus.



Amit Goswami: Essa questão de Deus contra o Mal é interessante. Segundo a visão da Física Quântica, existem as forças da criatividade e as forças do condicionamento. Não falamos muito sobre isso, mas eu defendo a idéia que a Física Quântica nos dá, de que é a consciência cósmica que escolhe entre as possibilidades para trazer à realidade o evento real que ocorre. A questão é: então temos de entrar nesse estado incomum de consciência, no qual somos cósmicos, no qual escolhemos e, então... como entrar nessa consciência individual na qual somos uma pessoa? Na qual temos personalidade e caráter? Ao trabalharmos com a matemática disso, descobrimos que essa condição ocorre porque todas as nossas experiências aparecem após serem refletidas no espelho da nossa memória, muitas vezes. É essa memória que causa o condicionamento. Uma propensão a agir do modo como já agi antes. Uma propensão para responder a estímulos do modo como já respondi antes. Todas as pessoas sabem disso. Elas passam a manhã no cabeleireiro e o marido volta para casa e diz: "O que há para o almoço?", sem notar o novo penteado da esposa, o que é muito irritante, tenho certeza. Mas esse condicionamento é o que nos torna indivíduos. Então, a questão é que, na Física Quântica, vemos claramente o papel da consciência cósmica, que eu chamo de "ser quântico", no qual há criatividade, há forças criativas. E então perdemos essa criatividade, ficamos condicionados. E o condicionamento nos faz parecidos com máquinas. Assim, o mal maior que a nova ciência nos traz é o condicionamento. Pois é ele que nos faz esquecer a divindade que temos, o poder criativo que temos, a força criativa que realmente representa o que buscamos quando invocamos Deus. Mas isso também está incompleto. Essa questão pode ser estudada mais a fundo e há um escopo maior, trazendo idéias como emoções negativas e positivas. Assim, teremos uma exposição maior do Bem contra o Mal. Mas, de fato, a consciência cósmica inclui tudo. Esse é o conceito esotérico, não tanto exotérico, mas esotérico, por trás de todas as religiões, de que há apenas Deus, e que o Bem e o Mal são uma divisão, uma necessidade da criação, mas não é fundamental, ou seja, o diabo não é igual a Deus; o diabo é uma criação subsequente. É útil pensarmos em termos de Bem e Mal mas, às vezes, é preciso transcender isso, é preciso perceber que Deus é tudo. Esse é o cenário que a Física Quântica defende.



Joel Giglio: Doutro Amit, eu sou psiquiatra, analista Junguiano, formado pela Associação Junguiana do Brasil, e tenho muitas perguntas a fazer ao senhor. Mas em vista do tempo e dos objetivos desse programa, vou me centrar numa delas. Eu pensei muito, quando li seu livro, em questões que ainda são incógnitas à nossa prática psicoterápica. A questão do 'insight'... O 'insight' nós não sabemos, em psicoterapia, quando ele vai acontecer, como vai acontecer. Ele simplesmente aparece e quase que do nada, embora a gente intua que o 'insight' vá aparecer. A questão da criatividade... a questão da sincronicidade... mas eu gostaria de fazer uma questão sobre os arquétipos. O senhor menciona no seu livro, idéias de arquétipos de objetos mentais. Cita Platão e cita Jung, que é o criador da psicologia analítica, setor da psicologia onde eu me situo. A questão que tem me perturbado muito é: os arquétipos evoluem, embora eles estejam fora do eixo espaço-tempo? Alguns autores dizem que está havendo uma evolução dos arquétipos. Quem fala isso, por exemplo, é Sheldrake, que o senhor mencionou há pouco e que não é psicólogo, é biólogo, mas que tem uma visão diferente dentro do campo da biologia. Como é que a teoria da Física Quântica explicaria, supondo que os arquétipos evoluem, a evolução dos próprios pensamentos arquetípicos, por exemplo, a evolução do arquétipo de Deus, se é que ele está evoluindo ou não. Essa questão... e muitos outros arquétipos, nós supomos que estejam evoluindo sem anularem os arquétipos anteriores.



Amit Goswami: Obrigado pela pergunta. Sou um grande seguidor de Jung. Acho que Jung foi dos precursores da integração que está ocorrendo agora. Nos meus primeiros textos, eu citava muito a afirmação de Jung de que, um dia, a Física Nuclear e a Psicologia se unirão. E acho que Jung ficaria satisfeito com esta conversa e, em geral, com a integração da Física e da Psicologia transpessoal que vemos hoje. Isto posto, acredito no conceito de arquétipo de Jung, e acho que o modo como Jung o apresentou, e Platão o apresentou, de que são aspectos eternos da consciência, contextos eternos da consciência... a consciência tem um corpo contextual no qual os arquétipos são definidos e, então, eles governam o movimento do nosso pensamento. Acho que é um conceito muito poderoso. Mas, ao mesmo tempo, na Física Quântica, existe a idéia de que todos os corpos de consciência, tudo o que pertence à consciência, inconsciência, são possibilidades. E por causa disso, por tudo ser possibilidade, surge a questão: alguém pode ir além de arquétipos fixos e considerar arquétipos evolucionistas? Não se pode descartar o que Rupert tenta dizer. Houve uma idéia semelhante, de Brian Josephson, um físico que publicou um trabalho na Physical Review Letters, revista de grande prestígio, dizendo que as leis da Física podem estar evoluindo. Da mesma forma, outras pessoas, cientistas muito sérios, sugeriram que, talvez, forças gravitacionais mudem com o tempo. Essa idéia de arquétipos fixos é uma idéia muito importante. Eu a apóio totalmente. Mas também vejo que na Física Quântica há espaço para a evolução dos arquétipos. Não devemos descartar totalmente idéias que dizem que arquétipos evoluíram. Ainda seremos capazes de determinar isso experimentalmente. Obrigado pela pergunta.



Lia Diskin: O senhor manifesta certo interesse pelas questões éticas, grande parte do final de sua obra se dedica a essa questão. O senhor nos disse que há necessidade da participação da ambiguidade para dar garantias de criatividade no campo ético. Entretanto, no mesmo contexto, nos fala imediatamente das linhas e instruções éticas numa obra monumental da tradição indiana que se chama "Bhagavad Gitâ". E a "Bhagavad Gitâ" se inicia pelo pressuposto da instrução do mestre para um discípulo, de que ele deve agir, de que ele deve entrar no combate, que ele deve assumir sua parte de ação, porque pertence a uma casta, a uma tradição de guerreiros, em que há ação da própria. Como fica o livre-arbítrio, como fica a ambiguidade como necessidade da criatividade dentro de um contexto de que existe um pressuposto, obviamente não-ambíguo e não-escolhível, que não pôde escolher? O que fazer... mas se está cominado a fazer, está cominado a agir? Como será isso, Professor?



Amit Goswami: Acho que essa também é uma pergunta muito difícil, muito sutil. Realmente, se considerarmos a ética compulsória, não parece haver escolha. Mas a ética não é tão definida: é muito ambígua. Lembro de uma história que o grande filósofo Jean-Paul Sartre contava. Suponha que voce vá em uma expedição de natação, ou melhor, de barco, e o barco afunde. Voce está com um amigo, voce sabe nadar, mas ele não. Mas voce não é muito forte. Se tentar salvá-lo, os dois podem morrer. Voce tem uma boa chance de se salvar, mas ama seu amigo e seu dever ético com ele está muito claro. O que fazer? Casos assim mostram claramente que há ambiguidade mesmo em decisões éticas, em decisões morais. Na Física Quântica, é muito claro que devemos esperar, e esperar pela intuição, ver se há um salto quântico, uma resposta criativa como voce a chama, se uma resposta criativa irá surgir. E é essa resposta criativa que é a resposta correta para solucionar essa ambigüidade em questões éticas. Quando a moralidade ou a ética são apresentadas como um conjunto de regras, e as pessoas seguem essas regras, elas perdem essa parte ambígua e, por causa disso, as regras perdem o sentido. Passa a ser um conjunto de regras inútil, sem vida. Mas, se considerarmos a ética com vida, e reconhecermos que temos um papel a desempenhar em todas as situações éticas, temos um papel a desempenhar em termos de irmos para dentro de nós, como as pessoas criativas fazem, combatendo isso, combatendo a ambiguidade. Então, o salto quântico da percepção virá e vai-nos permitir tomar a ação correta. É nisso que a Física Quântica está nos ajudando, é nessa conclusão que ela está nos ajudando. E acho que Sartre também buscava essa resposta porque a ética fixa é uma coisa impossível de se seguir.



Cláudio Abramo: Eu vou, infelizmente, ter que me estender ligeiramente na minha pergunta. Ela é precedida de uma declaração... Vou fazer uma interpretação do que foi declarado até agora, que eu acho que deve ser útil para os telespectadores. Não estou fazendo isso para me expor, mas para esclarecer o que me parece ser algumas questões importantes nesse debate para o telespectador. O entrevistado faz menção a fenômenos inexplicados, a fenômenos desviantes, entre diversas disciplinas. Começa com a Física, passa pela Biologia, faz referência a problemas seculares com respeito à consciência humana, ao livre-arbítrio, ao modo como raciocinamos, ao modo como chegamos a conclusões, menciona casos como, por exemplo, Einstein declarando, como tantos outros cientistas, que não sabe muito bem como chegou a uma conclusão. Poincaré, antes dele, havia escrito muito sobre isso... Poincaré era uma matemático, o último grande matemático universalista francês... ele morreu no começo desse século (XX). Bom, esse tipo de anedota é completamente comum na ciência. Não há nenhuma originalidade nisso. Esse gênero de anedota, repito, fenômenos inexplicados que são característicos da ciência... a ciência quanto mais sabe, menos sabe... quanto mais a ciência sabe, quanto mais fenômenos são explicados, mais avenidas de desconhecimento se abrem. Um cientista diz "não sei" o tempo todo. Um não-cientista explica tudo, porque sempre tem uma resposta do tipo "todo abrangente" como é esta resposta. O fato de se ter isso, para os telespectadores entenderem, o fato de se formular uma pseudo-explicação a respeito de como o universo funciona não dá a essa explicação, foros de verdade. Simplesmente declarar coisas não confere verdade ao que se declara. Agora, no que o senhor declara existe uma característica que eu acho bastante preocupante, ou pelo menos intrigante, vinda de alguém conhecido aqui como Físico, como o senhor declarou... o senhor foi.. o senhor foi Físico. O senhor diz, em primeiro lugar, que aquilo que seria essa intervenção de uma consciência cósmica, não é matematizável, quer dizer, isto não é introdutível dentro da teoria física na forma como a teoria física aceita as suas idéias. Não existe outra maneira de introduzir na Física idéias senão a matemática. Não existe.. não é possível, não é Física... se é não-matematizável, não é Física. Muito bem, então esta idéia de consciência cósmica não é Física, quer dizer, certamente nenhum Físico aceitará isso. Em segundo lugar, ela também, já que se está falando de alguma coisa que existe no mundo, que é uma consciência cósmica que se reflete na consciência das pessoas e faz as pessoas fazerem saltos quânticos... o senhor não vai usar esse termo, mas saltos quânticos em direção à solução de problemas. onde é que estão as evidências empíricas disso? Onde estão as experiências que levam a esse tipo de conclusão? Porque ou a gente pode ter conhecimento do mundo que seja muito estruturado, como no caso da Física, ou conhecimento do mundo pouco estruturado. Não existe uma teoria, não existe um conjunto de idéias muito organizado por trás, mas sabemos empiricamente que são verdadeiras, ou parecem verdadeiras. Onde é que estão as evidências empíricas e onde está o raciocínio, eu diria, desculpe a palavra, científico, que o leva a declarar que existiria uma consciência cósmica que estaria governando tudo e resolvendo todos os problemas aqui, da Biologia, da Psicologia?... O senhor afirma que estas suas idéias explicariam o problema da biologia evolucionista dos 'gaps' na criação de espécies, por exemplo. O senhor não acha ambicioso demais e, repito, onde é que estão as evidências empíricas disso?



Amit Goswami: Boa pergunta. Pergunta muito boa. Precisamos sempre fazer esta pergunta: onde está a evidência? Falarei da evidência mais tarde. Antes, responderei à pergunta: a Física é matemática? Ela deve ser totalmente matemática? Essa é uma crença que cresceu gradualmente na Física, por causa do sucesso da matemática para expressar a Física. Há duas coisas que devemos lembrar. Primeiro: não há motivo para a Física ser matemática. Às vezes os filósofos levantam essa questão. Nancy Cartwright escreveu um livro: Why do laws of Physics lie. Ela estava argumentando que não há provas dentro da filosofia materialista de que a Matemática deve governar as leis da Física. De onde vem a Matemática? Pessoas como Richard Feynman, grande físico, Eugene Bigner, todos estudaram a questão. E não há resposta dentro da filosofia materialista. Platão tem uma resposta: a matemática molda a Física porque surgiu antes da Física, faz parte do mundo arquetípico que discutimos. Assim, o idealismo de Platão é fundamental para entender o papel da Matemática na Física, em primeiro lugar. A Física em si precisa de algo além da matéria, ou seja, da matemática e de arquétipos para ser uma ciência consistente. É preciso se lembrar disso. O segundo aspecto da questão é o mais importante. Na Física Quântica, procuramos insistentemente uma forma matemática de encerrar a Mecânica Quântica. Uma forma matemática para entender a medição quântica. Não fomos capazes. Niels Bohr demonstrou para Erwin Schrödinger, há muito tempo, quando a Mecânica Quântica estava sendo desenvolvida. Schrödinger achou que tinha obtido a continuidade e Bohr provou o contrário e o convenceu disso. E Schrödinger disse: "Se eu soubesse que essa descontinuidade, saltos quânticos, iriam permanecer, eu nunca teria descoberto a Mecânica Quântica". Bohr disse: "Estamos felizes que tenha descoberto". Essas descontinuidades vão continuar existindo, não há explicação matemática, e por não haver explicação matemática, portanto, há espaço para o livre-arbítrio. O livre-arbítrio, Deus, consciência, colapso, tudo isso entrou para a Física porque atingimos o conhecimento, a sabedoria, de que existe o princípio da incerteza, existem a probabilidade e possibilidades. E por existirem probabilidade e possibilidades, deve haver um agente que causa o colapso das possibilidades em eventos reais. E esse agente não pode ser matemático porque, se for, não poderá haver livre-arbítrio: seria determinista. Mas não é determinista. O princípio da incerteza é fundamental. Assim, nós chegamos à conclusão, após décadas de lutas nós conseguimos...



Cláudio Abramo: Quem é "nós"?



Amit Goswami: "Nós" quer dizer que há um consenso entre cientistas...



Cláudio Abramo: Há um consenso a respeito de suas idéias?



Amit Goswami: Não a respeito de minhas idéias. Esqueça as minhas idéias. Mas há um consenso de que não há solução matemática para o problema da medição quântica. Nisso, chegamos a um consenso. E por não haver uma solução matemática para isso, e por haver uma solução consistente em termos de consciência causando colapso de possibilidades quânticas em realidade, podemos falar sobre essas idéias publicamente. Quanto à segunda pergunta: Há evidência empírica? Acontece que os dois aspectos fundamentais da nova física, a consciência causa o colapso da possibilidade em realidade, e o segundo, que essa consciência é uma consciência cósmica, os dois aspectos foram confirmados por dados empíricos. Antes, darei os dados para o segundo, porque é o mais simples para o espectador. O primeiro é um pouco difícil. Talvez possamos incluir os dois. O primeiro experimento é muito importante porque já foi aplicado. Em 1993 e 1994, o neurofisiologista mexicano Jacobo Greenberg Silberman, ele e seus colaboradores fizeram um experimento, no qual havia dois observadores meditando por 20 minutos, com o propósito de terem comunicação direta. Comunicação direta no estilo de não-localidade. Sinais não-locais ocorrendo entre eles, e ainda assim eles teriam comunicação. Certo, eles meditaram juntos. Pediu-se que mantivessem o estado meditativo durante o resto do experimento. Mas então, um deles é levado para outro recinto. Eles ficam em câmaras de Faraday, onde não é possível a comunicação eletromagnética. Os cérebros deles são monitorados. Uma das pessoas vê uma série de 'flashes' brilhantes, o cérebro dele responde com atividade elétrica, obtém-se o potencial de resposta muito claro, picos muito claros, fases muito claras. O cérebro da outra pessoa mostra atividade, a partir da qual obtém-se um potencial de transferência que é muito semelhante em força e 70% idêntico em fases ao potencial de resposta da primeira pessoa. O mais interessante é que, se voce pegar duas outras pessoas, duas pessoas que não meditaram juntas, ou pessoas que não tinham a intenção de se comunicar, para elas, não há potencial de transferência. Mas para pessoas que meditam juntas, invariavelmente, muitas vezes, um em cada quatro casos, obtemos o fenômeno de potencial de transferência. E Peter Fenwick, na Inglaterra, há dois anos, confirmou isso, repetindo o experimento. Assim, temos evidência empírica. Se tivéssemos tempo, e voce tivesse paciência, eu poderia lhe dar inúmeros dados. Outro dado que é muito interessante: considere o caso de geradores de números aleatórios. Eles são realmente aparelhos quânticos, pois eles pegam eventos radiativos, que são aleatórios, e os convertem em seqüências de números, seqüências de zeros e uns. Em uma longa cadeia, deve haver número igual de zeros e uns. É o que se espera da sequência aleatória. Helmut Schmidt, um físico que pesquisa parapsicologia, tenta há quase 20 anos, fazer com que médiuns influenciem os geradores de números aleatórios para gerarem sequências não-aleatórias, mais zeros que uns. E ao longo dos anos ele conseguiu boas evidências de que, até certo ponto, os médiuns conseguem fazer isso. Um resultado com um grande desvio. Isso ainda não tem nada a ver com Física Quântica, mas recentemente, em um trabalho publicado em 1993, Schmidt retratou uma modificação revolucionária desses dados. O que ele fez, recentemente, é que o gerador de números aleatórios, os dados do gerador de números, a sequência, é armazenada num computador, ela é impressa, mas ninguém olha. Os dados impressos são fechados num envelope e enviados para um observador independente. Três meses depois, o observador, sem abrir o envelope, escolhe o que quer ver, mais zeros ou mais uns. Tudo segue um critério. Então ele liga para o pesquisador, o pesquisador diz ao médium para olhar os dados, e pede a ele para mudar os resultados, influenciá-los, se puder. E o médium tenta produzir mais zeros, se esse for o desejo do observador. E então, o observador abre o envelope e verifica se o médium conseguiu. E a incrível conclusão é (é um resultado sério, não é fácil contestá-lo) que o médium, em 4 de cada 5 tentativas, consegue mudar os números aleatórios gerados pelo aparelho, mesmo após três meses. Este mito de que o pensamento causa o colapso de si mesmo, que o colapso é objetivo, sem que o observador consciente as veja, é apenas um mito. Nada acontece, tudo é uma possibilidade até que o observador consciente veja. Numa experiência controlada, as pessoas intervieram. As pessoas viram, sem contar a ninguém, viram os dados, a impressão. Nesses casos, o médium não influenciou os dados. Está claro que a consciência exerce um efeito, exatamente como Bohr suspeitava, como Newman suspeitava. Agora estamos fazendo teorias mais completas e experimentos mais completos baseados nessas teorias. Henry Stab colaborou com todas essas idéias que apresentei, consciência causando o colapso de funções quânticas em eventos reais. Ele participou do experimento com Schmidt. Então, estamos vendo uma mudança revolucionária na Física, não menos revolucionária do que a acontecida com Copérnico. Claro que haverá reações, como a que apresentou, e temos de ser muito pacientes, calmos, e trabalharmos juntos para superar essas tendências contrárias. Mas temos a certeza de que existe algo que todos devemos olhar. Isso é revolucionário, é novo e pode mudar, como já discutimos, as dificuldades com valores que a sociedade vem enfrentando. Não vamos nos preocupar em como pode ser, mas vamos olhar os dados, olhar a teoria e perguntar: pode ser? Se pode, que oportunidade fantástica temos para integrar todos esses movimentos díspares de consciência que nos separaram por tanto tempo.



Heródoto Barbeiro: Ele é autor também do livro "O universo autoconsciente -Como a consciência cria o mundo material". Dr. Goswami, dentro dessas explicações que o senhor nos deu até agora, como fica a questão da reencarnação e da preservação dessa consciência dos seres humanos?



Amit Goswami: A questão da reencarnação, provavelmente, é a pergunta mais radical que pode ser feita. E é impressionante que a Física Quântica nos permita dar uma resposta afirmativa. Eu mesmo fiquei tão surpreso quanto qualquer um, com isto.. No início, quando me perguntavam isso, eu me recusava a discutir. Mas então, eu acordei de um sonho, e, basicamente, o sonho me dizia... eu ouvi isso no sonho: "O Livro Tibetano dos Mortos está certo e seu trabalho é provar". Após acordar desse sonho, eu passei a encarar reencarnação com seriedade. Basicamente, o problema com a reencarnação é este: o corpo físico morre, e o que resta? Se a consciência é a base do ser, vem a idéia de que o que resta é a consciência. É a primeira pista. A segunda pista é que tudo é possibilidade, no modo quântico de ver as coisas. Então, não é irrelevante dizer que as possibilidades podem viver. Algumas possibilidades morrem com o corpo material e o cérebro, mas pode haver outras possibilidades, outras possibilidades que se modificam ao longo da nossa vida, e essas modificações das probabilidades das possibilidades podem formar uma confluência que possa viver mais tarde na vida de outra pessoa. É essa idéia que pude desenvolver de forma mais completa, num livro que será lançado no ano que vem, e fico feliz em dizer que podemos lidar com essa questão. A vantagem de se fazerem essas perguntas é que podemos ver imediatamente a utilidade das novas ciências que virão. Porque são essas coisas que preocupam as pessoas. As pessoas são fundamentalmente incomodadas por perguntas como "o que acontecerá quando eu morrer?". E se a nova Física puder responder essas perguntas, a despeito da importância da Psicologia transpessoal, e da Psicologia junguiana, em que a nova ciência ajuda, e também da medicina alternativa, que nem discutimos ainda, acho que tocaremos o coração das pessoas quando pudermos dizer: "Finalmente, a Ciência pode ajudar a entender essa pergunta". Até agora, apenas o padre, o teólogo pode dar qualquer resposta para a pessoa. E se pudermos dizer a ela: "Faz sentido fazer essa pergunta, e voce pode fazer algo para ajudar voce com o que acontecerá após a morte". Não seria um progresso maravilhoso na ciência?



Joel Giglio: Professor Amit, eu vou fazer uma pergunta baseado no trabalho de um ex-orientando de tese de doutoramento que eu orientei na Universidade de Campinas, e que fez a primeira tese, pelo menos na Unicamp, e talvez em qualquer universidade estadual ou federal do Brasil, sobre parapsicologia. Ele fez uma tese sobre clarividência e eu não vou, naturalmente, falar da metodologia do trabalho que seria bastante extensa, mas resumir pelo menos os resultados principais. Várias pessoas, vários sujeitos tentavam adivinhar as cartas de um baralho de símbolos geométricos, baralho de Zener muito usado em pesquisa e parapsicologia, e tentava adivinhar as cartas de um baralho Tarô, que é baseado em imagens arquetípicas, o Rei, a Rainha, etc. Nos resultados que foram feitos seguindo uma metodologia tradicional, estatística, as pessoas acertaram, no baralho de Zener, um pouquinho acima do que era esperado ao acaso e 10% acima no baralho de Tarô, comparando com o de Zener. A explicação dada pelo meu orientando foi dentro da teoria da Psicologia Analítica, em relação aos arquétipos emergentes que, de uma certa forma, estariam mobilizados mais no baralho de Tarô do que simplesmente no baralho de símbolos geométricos. Mas essa explicação, embora nos satisfaça um pouco, ainda deixa muito a desejar. Eu perguntaria se o senhor teria alguma explicação a mais baseada na Teoria Quântica sobre essa maior adivinhação das cartas do baralho do Tarô, que são símbolos arquetípicos em relação ao baralho comum de Zener , que são cinco símbolos geométricos, quadrado, cículo, etc.?



Amit Goswami: Sim. Obrigado pela pergunta. Na verdade, somente no Brasil alguém pensaria em fazer um experimento tão brilhante. Tenho visitado o Brasil nos últimos 5 anos e o futuro parece promissor. Eu fico entusiasmado com a mente do brasileiro. Qual é a diferença entre o experimento original de adivinhação de cartas e as cartas de Tarô? A idéia que proponho, acho que voce pensa da mesma forma, é que quando o objeto que usamos na telepatia é significativo, ele é um objeto melhor. Os cientistas, os parapsicólogos anteriores preocupavam-se demais com a objetividade e ignoravam esse aspecto. Agora, nos novos experimentos parapsicológicos, espero usarmos cada vez mais objetos significativos na transferência telepática. E voce tem razão, a explicação completa tem de usar a palavra "telepatia", tem de usar a transferência não-local de informações, neste caso, transferência não-local de informações significativas, arquetípicas. E esse é o motivo para os melhores resultados. Mas a não-localidade, a não-localidade quântica, tem de ser evocada para se ter uma explicação completa do que ocorreu. Obrigado.



Mario Cortella: Doutor Amit, eu juntei algumas questões nisso que eu não vou tratar delas como perguntas, porque eu acho que na sua obra, pelo menos no que eu pude ler, há um aprofundamento disso e uma leitura mais detalhada ofereceria mais questões. Por exemplo, no campo da psicanálise essa idéia de que o universo é quando é percebido e até interferido, será que não seria uma postura um pouco ego-narcísica da nossa parte, um pouco antropocêntrica em relação ao próprio universo que dificulta a idéia de um cosmo, invertendo Dostoievsky. Dostoievsky disse que se Deus não existe, tudo é permitido. Nessa compreensão, parece que se Deus existe, aí é que tudo é permitido, porque existe aí uma probabilidade que pode ser interferida. E uma outra questão, que eu acho que está na sua obra mas acho que vale aprofundamento, é o ateísmo metodológico, sendo que foi tão caro para a ciência para poder buscar explicações, mas ele não é mais necessário. Mais aí a questão de fundo: eu tenho lido, não sei se é verdade, que a Física Quântica mostra que hoje o tempo é uma ilusão. Alguns têm dito que não se fala mais em universo, mas em multiverso, porque haveria vários universos paralelos. Isso traria um problema: a possibilidade de viajar no tempo. A maior explicação que achei até hoje contra a viagem no tempo, foi do Físico inglês, Stephen Hawking, que usou um argumento lógico. Ele disse: "É impossível viajar no tempo porque se um dia for possível isso, os homens do futuro já teriam voltado". Mas a Física Quântica ao falar em universos paralelos levanta a possibilidade de se ter o tempo como uma mera ilusão humana. Isso me coloca a seguinte pergunta aí para o senhor: será que nós chegaremos, com a Física Quântica, a voltar à origem do cosmos e, aí sim, encontrar o princípio explicativo?



Amit Goswami: Bem, suas duas colocações são muito boas, e a pergunta é extremamente fascinante. A primeira coisa que quero dizer é que 'não dizemos que tudo é possível' apenas por termos incluído aí a consciência em nossas teorias, porque ainda estamos concordando totalmente com a Física Quântica que a causalidade ascendente molda a forma das possibilidades, a partir da qual a consciência escolhe. Tanto a causalidade descendente, quanto a ascendente têm papel fundamental na nova Física, na nova Ciência. Essa é uma das virtudes que temos. A nova Ciência absorve a velha Ciência nos limites do princípio da correspondência, no limite de que poderíamos falar apenas em termos de probabilidades para um grande número de coisas e eventos. A velha Ciência não desaparece. Não poderia. É solidamente baseada em dados experimentais. A nova Ciência expande a velha Ciência em arenas com as quais a velha Ciência não pode lidar. Como eventos singulares de criação, criatividade. Esse é o primeiro ponto. Sobre voltar no tempo, há experimentos quânticos. O mais famoso é o experimento de Le Choice, mas é muito longo para explicar, e muito complicado para os espectadores realmente apreciarem. Embora, se alguém estiver interessado nele, há livros sobre ele. Leiam, por favor, é fascinante. Há algo acontecendo. Essa idéia de voltar no tempo é real na Física Quântica. Podemos ser afetados por coisas no futuro, assim como somos afetados por coisas no passado. Na Física Quântica, o tempo é não-linear. Isto posto, claro que experimentos recentes são tão impressionantes, tão surpreendentes, que muitos físicos convencionais, conservadores, procuram formas de viajar no tempo. Mas acho que o consenso é que a viagem no tempo envolve muito mais do que esta observação da Mecânica Quântica. Não podemos mais descartá-la, mas ela envolve muito mais pois ainda temos sérios problemas de como trazer os efeitos quânticos aos macrocorpos. Pois os efeitos quânticos são muito destacados apenas em objetos microscópicos, e não tão destacados em macro-objetos. A situação da medição é uma exceção. Mas normalmente descobrimos apenas raios 'laser', supercondutores, poucas coisas, poucos macro-objetos em que os efeitos quânticos persistem. Então temos de resolver esse problema de como macrocorpos podem ser transportados pelo tempo, e isso levará um tempo. Se a consciência voltar a essa equação, e ela precisa voltar, em algum ponto, então, outra dimensão de pensamento se abrirá e isso pode nos dar novas respostas, novas visões sobre isso. Mas é muito prematuro falar sobre isso, acho.



Rose Marie: Eu sou muito interessada em história da tecnologia, porque eu acho que através da tecnologia é que os sistemas econômicos se desenvolvem, que cresce uma dominação de potências hegemônicas. Isso vai muito na linha da pergunta do Cláudio Abramo. Eu sei que o senhor está trabalhando na construção do primeiro computador quântico. Eu quero perguntar uma coisa: o computador quântico dá saltos quânticos, ele cria? Qual a diferença dele do computador determinístico?



Amit Goswami: Essa é uma pergunta muito interessante. O que é um computador quântico? Um computador quântico em vez de usar um algoritmo específico, usa um algoritmo ambíguo. No computador quântico é usada a superposição de possibilidades e, dessa forma, espera-se que seja muito mais rápido que o computador convencional. Desde que o computador quântico opere apenas nesse nível, eu não espero que ele seja uma novidade tão grande, a não ser o fato de ele ser mais rápido. É isso que interessa aos cientistas da computação. Mas eu tenho um interesse diferente nesse computador. Se o computador for construído, por ter um processador quântico, por processar superpondo possibilidades...



Rose Marie: É tão realista.



Amit Goswami: Isso mesmo. Assim como o ser humano faz. O cérebro humano, de forma semelhante, processa de forma quântica as possibilidades, em vez de trabalhar diretamente, de maneira algoritmica, sem ambiguidade. Então, alguém pode fazer um computador que tenha todos os outros aspectos da medição quântica? A situação da medição quântica envolve um mecanismo que chamo de hierarquia embaraçada. É um pouco difícil de entender, mas um exemplo é a frase: "Eu sou mentiroso". Se pensar nela, verá que a relação hierárquica entre sujeito e predicado é recíproca. "Eu" qualifica mentiroso, e vice-versa. Um qualifica o outro. É o que chamo de hierarquia embaraçada. A medição quântica no cérebro é assim. A questão intrigante para mim é que: suponha que no futuro encontremos um computador com hierarquia embaraçada. O interessante é que a hierarquia embaraçada dá margem à auto-referência. Então, este computador quântico terá auto-referência? A consciência cooperará na criação de um aparelho feito por humanos, que não seguiu uma evolução, mas desenvolvido pela inteligência humana? A consciência cooperará? A consciência cósmica cooperará e o tornará um ser consciente? Eu não sei a resposta. Mas esta será uma verificação fundamental, uma das mais fantásticas, das idéias que discutimos hoje. Acho que essa pesquisa deve ser encorajada. Obrigado pela pergunta.



Lia Diskin: Tentando fazer uma síntese dentro das idéias da biologia, dentro das idéias da psicologia e, logicamente, de toda a Física que o senhor coloca, o que hoje sabemos é que apenas 2% de nosso cérebro utiliza vias neuro-cerebrais para entrada e saída de informação. E é a partir disso, que nós construímos o que chamamos "os objetos ideais e universais" que constituem a ciência. 98% restante pertence a um universo interno, nebuloso, no qual existe a fantasia, a ilusão, logicamente a irracionalidade e também a probabilidade. Até que ponto podemos dizer que é possível um verdadeiro diálogo com essa disparidade de porcentagens? Até que ponto podemos dizer que é possível uma cientificação das idéias, de Deus, ou das idéias internas, humanas, divinizadas, como queira chamá-las?



Amit Goswami: Em outras palavras, deixe-me ver se entendi a pergunta, há muitas coisas que são fantasias e há muitas coisas que envolvem Deus. É possível transformar esses aspectos fantasiosos em científicos? É uma pergunta interessante. Claro, na criatividade, transformamos fantasias, transformamos algumas fantasias em algo científico. Porque algumas delas são fantasias criativas. Em outras palavras, a imaginação, a parte mental de nossas vidas, a parte interna de nossa vida, é fundamental no que fazemos no mundo externo. Na nova Ciência, por estarmos igualmente envolvidos com o mundo externo e o interno, pelo fato de a subjetividade ter voltado à ciência, estamos validando o conceito de que, talvez, devamos levar algumas de nossas fantasias a sério. Porque a idéia contrária também pode ser positiva, ou seja, de que tudo é uma fantasia. Fantasia da mente, fantasia da consciência. Porque a consciência é a base do ser, e o que pensávamos ser material e real, e o que pensávamos ser fantasia e irreal, esta distinção não é muito clara, agora. São todas possibilidades da consciência. Portanto, é a consciência que as valida, que escolhe entre elas, que lhes dá substancialidade. Então, qual delas será substancial depende totalmente da escolha, do contexto no qual a consciência as vê. Isso vai revolucionar a sociedade, como voce antecipou com sua pergunta. Em outras palavras, vamos levar nosso mundo interno muito mais a sério. Eu costumo dizer às pessoas que, se elas estudarem seus sonhos, o preconceito que costumamos ter é de que o sonho não é contínuo, portanto, de que adianta estudá-los? Há evidências de que os sonhos são contínuos, mas é preciso olhá-los sob o ponto de vista significativo. Alguns ficariam felizes com essa descoberta científica, de que os sonhos dão um relatório sobre a parte significativa das nossas vidas. Então, há outros aspectos da vida com os quais a ciência materialista não pode lidar e com os quais podemos lidar agora por colocar a consciência de volta, por exemplo, o pensamento. E, quando fazemos isso, nossa vida interna adquire uma enorme importância. Sim, a vida interna lida com o pensamento, a beleza, os arquétipos, de uma forma diferente que a vida externa, materialista, pode. E, focalizando na vida interna, não só podemos nos transformar, essa é a parte mística, mas também podemos ter enormes visões sobre o que criar, como criar, sobre nossas artes, sobre nossa música, até sobre a ciência.



Pierre Weil: Eu queria primeiro felicitar esse programa, Roda Viva, pelas iniciativas que está tomando. Eu quero dizer que é a primeira vez que eu vejo na televisão, problemas tratados no nível que merecem, na altitude que merecem, problemas como a parapsicologia, a psicologia transpessoal. Isso é feito graças a uma mudança de paradigma. E eu queria realçar de novo para o público telespectador que o que estamos tratando aqui tem uma influência muito grande sobre a destruição da vida no planeta e a grande crise de violência que está assolando atualmente o mundo, não é só o Brasil. Eu queria, já que estamos no fim do programa, deixar a oportunidade a Amit Goswami, que nós convidamos na nossa Universidade da Paz em Brasília, justamente porque ele representa um novo paradigma, como que o antigo paradigma é responsável pela violência atual do mundo, antiga visão que está responsável pela destruição da vida no planeta, e como o novo paradigma pode nos ajudar a nos tirar dessa crise, além de medidas policiais e de mudança de lei que são necessárias, mas são absolutamente insuficientes?



Amit Goswami: Obrigado. Acho muito importante dizer que, sem reconhecer a consciência e sem reconhecer o valor da nossa vida interna, sem reconhecer o valor da transformação, nunca mudaremos a violência na sociedade. Então, é muito importante ver que apenas pensando em não-violência, apenas falando dela, não deixaremos a violência. É preciso passar por todo o processo criativo. A nova Ciência, o novo paradigma, é extremamente importante porque sempre enfatiza a criatividade. Na velha Ciência, o determinismo e behaviorismo, essa idéia de que o condicionamento prevalece, nos cegou tanto quanto à transformação, nos cegou tanto que desistimos. Basicamente, os valores não eram necessários. Steve Weinberg disse que não há significado no universo, não há valores se o consenso é o julgamento dos cientistas materialistas, e isso ocorre dentro da sociedade, e o behaviorismo diz: "Não podemos fazer nada. Somos seres comportamentais, somos condicionados". E a nova Ciência diz: "Não. Também há forças criativas dentro de nós. Basta aprender a agir a partir desse estado de consciência não-ordinário no qual voce tem escolhas". E o meu novo lema, em vez do cartesiano "eu penso, logo existo", e pensamento é uma condição behaviorista, meu novo lema é: "escolho, logo existo". Se é "escolho, logo existo", posso escolher a não-violência. Mas tenho de aprender como escolher, e isso exige criatividade. Essa é, realmente... a nova confiança do novo paradigma: em vez de escolher a metade condicionada do mundo, vamos dividir o mundo em condicionamento e criatividade. Forças do Bem e do Mal, das quais falamos antes. Podemos ser muito otimistas. Se essa mudança para o novo paradigma vier logo, talvez possamos realmente lidar com a violência de uma forma realmente prática, em vez de apenas verbalmente, como fazemos.



Carlos Ziller: Quando eu estava fazendo a minha leitura dos seus trabalhos, percebi um sentimento que eu compartilho, de um incômodo profundo com relação a algumas conclusões que emergem de determinados meios científicos. Vou dar só um exemplo, acho que o telespectador vai se lembrar, certamente. Há algum tempo atrás apareceu um resultado de um laboratório do EUA que falava da descoberta do gene da homossexualidade. Mais recentemente falou-se no gene da obesidade, e há toda uma série de conclusões desse tipo que não deixam de produzir, nos homens de bom senso, uma certa surpresa, e, contudo, mesmo em homens que são materialistas e bem convencidos, que não aceitam, rejeitam esse determinismo radical que emerge de alguns ambientes científicos, sobretudo norte-americanos. Há um materialismo que convive muito bem com o livre-arbítrio. Há um realismo filosófico que convive muito bem, sem muito inconveniente, com paradoxos, com contradições. Isso não é, digamos, o todo, do que se poderia chamar de "atividade científica". Por fim, eu gostaria de fazer uma pergunta, e é a questão mais importante que eu teria a colocar, que emerge também de uma sensação que eu tive ao ler "O universo auto-consciente". Eu tive a sensação de retornar ao passado, aí sim uma viagem ao passado. Eu vi ali, arrumados, organizados de uma forma muito particular por voce, idéias e proposições que eu já havia conhecido em leituras, por exemplo, da obra do cardeal Nicolau de Cusa , grande pensador do século XV, que propôs que o universo era resultado de uma contração de Deus, e essa contração, enfim, não é o caso aqui de eu explanar essa filosofia. Mas esse tipo de pensamento, produziu, interagiu com concepções científicas do século XVI, do século XVII, com concepções que propunham que a divindade organizasse, ou enfim, propunha uma visão bastante parecida com essa, um projeto científico bastante parecido com esse que voce está propondo nesse seu livro. A humanidade passou por um processo muito longo, muito duro, para conseguir, digamos, não eliminar Deus da Ciência, mas pelo menos reduzir um pouco seu papel, esse processo foi longo e lento. Para concluir, como o senhor acredita poder convencer os cientistas desse seu projeto, depois de tanto esforço para conseguir criar uma noção de objetividade, de realidade, de realismo, com todos os exageros em alguns momentos, mas convencer esses homens depois de tanto esforço? O senhor imagina conseguir isso usando que gênero de recursos?



Amit Goswami: Eu acredito que as idéias se verificarão por si mesmas, serão confirmadas nos laboratórios e serão úteis. A ciência tem dois critérios fundamentais. Por isso Galileu é chamado de pai da ciência moderna, pois ele enunciou claramente esses dois critérios. Um é que a ciência deve ser verificável. Ela deve ser verificada experimentalmente. E a segunda idéia é que a ciência deve ser útil. No aspecto da verificação, já apresentei alguns experimentos a voces, pois o tempo é curto, não entrarei em outros experimentos, mas digo que há um número enorme de experimentos sendo realizados, graças à Parapsicologia e interessados em Parapsicologia. Mas também em Biologia, e a Medicina é uma grande área de verificação experimental de algumas de nossas idéias. Mas a questão da utilidade é a mais importante. Deepak Chopra ficou famoso por um livro que escreveu, chamado Cura Quântica, lançado há 10 anos. Ele começou a revolucionar a Medicina, de certa forma, pois há um fenômeno chamado "efeito placebo" para o qual os cientistas não têm explicação. E esse trabalho, que é muito semelhante à minha forma de pensar, e eu tenho lido trabalhos citando a conexão entre as nossas idéias... Mas veja as implicações disso. Se, de fato, houver cura quântica, se houver Medicina mental, o efeito da mente sobre a cura, então as pessoas serão de fato ajudadas, não apenas no campo da Psicologia, mas no campo da verdadeira saúde física. A saúde física real, que importa para muito mais pessoas do que a saúde mental, ainda não estamos esclarecidos o bastante para levar a saúde mental tão a sério. Mas todos se preocupam com a saúde física, levam muito a sério. É a aplicação da nova Ciência a essas áreas, especialmente na área da saúde, que vai trazer a revolução de que Deus é importante, a consciência é importante, a criatividade é importante, observar o livre-arbítrio e responsabilidade é importante, que temos um paradigma científico que pode unir todas essas coisas, trazê-las para junto da velha ciência e ter formas objetivas de proceder e prever. Será uma ciência previsível, poderá ser verificada e também será útil. Isso é o que mudará a percepção do público. A percepção dos cientistas, também. Obrigado.



Heródoto Barbeiro: Doutor Goswami, muito obrigado por vir.



Amit Goswami: Muito obrigado. Foi um prazer estar aqui.






[As partes desta mensagem que não continham texto foram removidas]



SUBJECT: Re: [ciencialist] Sobre a TQG e Amit Goswani
FROM: Eduardo Gueron <edgueron@yahoo.com>
TO: ciencialist@yahoogrupos.com.br
DATE: 24/03/2005 00:53

Ola,

Sou especialista em dar opiniues nco solicitadas nessa
lista mas vamos la: Ha inzmeros erros conceituais na
TQG bem mais relevantes do que se matiria deforma o
espago-tempo ou vice-versa

. Primeiro, o princmpio da Incerteza ss existe com 2
grandezas a serem medidas, no caso do princmpio de
Heinsenberg, o momento e a posigco. Estendendo para
outras grandezas, qualquer par de variaveis nco
comutantes, por ex:Pares das projegues do momento
angular (Lx e Ly, ou Lx e Lz, ou Ly e Lz), Energia e
frequjncia...etc. Portanto, nco faz o mmnimo sentido
restringir o princmpio a simplesmente a posigco. Como
este passo i necessario para obtengco de algumas
fsrmulas, por si ss, a teoria popde ser descartada
(eq2 e eq3).

A inequagco DxDp>h implica que a precisco das duas
medidas i sempre maior que um valor mmnimo e nco que o
valor das medidas i maior que um mmnimo. Quando se usa
que x i o raio de Bohr, esta se escolhendo um valor
exato para x, ou seja, Dx=0 (Por D leia-se delta no
texto). Portanto, Dp deveria ser infinito. Outros
inzmeros erros estco sendo cometidos, para citar
alguns: A variagco da posigco e da velocidade e da
Forga em Rel. Especial (ou Restrita) depende da
diregco do movimento do outro referencial. A Forga
centrmpeta i, essencialmente, radial e o movimento
assumido pelo autor seria circular. Ora, se o
movimento do referencial for no plano do mov. do
elitron, o cmrculo vira uma elipse e a Forga
centrmpeta passa a ter componentes longitudinais. Se
for perpendicular a esse plano, o raio do cmrculo nco
se altera... Ademais, mnzmeras questues do
eletromagnetismo classico como a radiagco do elitron
(cargas aceleradas emitem radiagco) nco estco sendo
consideradas. Note que esse i o problema fundamental
resolvido por Bohr com seus postulados mas, o autor da
TQG esta tratando o atomo classicamente.

Enfim, nco ha outro conselho a dar ao autor da teoria
se nco, estudar a fundo fundamentos basicos da Mec
Qubntica, Rel Restrita (nem precisa chegar na Rel
Geral) e Eletromagnetismo classico.
.....


Mudando de pato para ganso: Esse Amit Goswami (citado
em alguma das mensagens) i um picareta de marca maior.
Ele confunde todos os princmpios basicos da Mec.
QUbntica e colabora para o mau entendimento desta
teoria. Como assim a conscijncia do observador i
importante para o resultado do experimento? Se vc
fizer a experijncia da fenda dupla com alguim pondo o
dedo em uma das fendas e vendo o aparato onde a figura
de interferjncia i ou nco formada ou ptr um robt para
tapar uma das fendas e filmar o aparato, o resultado i
exatamente o mesmo. O problema i que da muito mais
dinheiro escrever livros e dar palestras dizendo que a
Fmsica Qubntica espiritualiza a cijncia do que
trata-la como uma mera teoria fmsica repleta de
sucessos experimentais e carente de alguns ajustes
finos.

I isso, um abrago Eduardo.



__________________________________
Do you Yahoo!?
Yahoo! Small Business - Try our new resources site!
http://smallbusiness.yahoo.com/resources/


SUBJECT: Re: [ciencialist] Re: Editor de calculos e formulas
FROM: Franco <dfranco@pop.com.br>
TO: ciencialist@yahoogrupos.com.br
DATE: 24/03/2005 01:29

Não me lembro mais quem enviou a dúvida, pois já deletei a mensagem.
Mas, veja bem, expressar fórmulas em forma de texto nem é tão complicado
assim, aliás é até mais rápido do que usar um editor como o MathType que
funciona ao estilo "clica aqui - clica ali".
Depois retorne seus resultados caso tenha optado por algum editor de
fórmulas e gráficos.

Agora, se desejar/precisar mesmo incuir alguma figura (fórmula/gráfico)
em sua mensagem, há opções:
- coloque o arquivo de imagem na sessão arquivos do grupo e indique o
link do mesmo ao enviar sua mensagem a esse grupo;
ou
- vá até http://www.photobucket.com/ e faça lá um cadastro gratuito. O
site permite que vc armazene imagens (até 250 kB); da mesma maneira,
quando enviar sua mensagem a alguma lista, basta indicar o link de sua
figura no photobucker para o pessoal.

Franco.

>



SUBJECT: Re: [ciencialist] Um pedido - mais uma vez
FROM: Franco <dfranco@pop.com.br>
TO: ciencialist@yahoogrupos.com.br
DATE: 24/03/2005 02:40

Use o mesmo formato de fórmulas do Excel, por exemplo - consulte o help
dele.

Use os símbolos:
+ para adição
- para subtração
/ para divisão
* para multiplicação
^ para exponenciação

Exemplos:
F = m*a (da lei de Newton)

Módulo da força elétrica que uma carga puniforme exerce sobre a outra:
E = [1/(4*Pi*e0)]*(Q1*Q2)/(r^2)

E por aí vai...

Franco.


TARCISIO BORGES wrote:

> Use apenas o formato ASCII. [texto puro]
>
> []s
> TARCISIO BORGES
> tbs97@fisica.ufpr.br
>
> On Tue, 22 Mar 2005, junior_br2001 wrote:
> > Comom faz, qual editor uso para poder passar calculos e simbolos
> > matemáticos para esta lista?
>



SUBJECT: Dúvidas
FROM: "junior_br2001" <junior_br2001@yahoo.com.br>
TO: ciencialist@yahoogrupos.com.br
DATE: 24/03/2005 02:45


Tem como provar experimentalmente que matéria pode ser criada do
nada? Tem como testar isso?

A teoria do Big bang como sendo a *origem do universo* pode ser uma
teoria científica do ponto de vista de Popper? Pois se não podemos
testar a origem do universo, o ponto de partida de tudo, como isso
pode ser considerado científico? Sei que tem algumas evidencias como
radiação de fundo e expansão do universo, mas nao é disso que estou
falando, estou falando se podemos testar a origem do universo c/ a
matéria sendo criada do nada. Isso pode ser reproduzido
experimentalmente?

JR







SUBJECT: Re: nesta ponte não passo.
FROM: "Tipoalgo" <tipoalgo@bol.com.br>
TO: ciencialist@yahoogrupos.com.br
DATE: 24/03/2005 03:20


> A ponte entre a Ciência e a Religião
>
> Transcrição completa da entrevista concedida pelo físico Amit
Goswami ao programa "Roda Viva" da TV Cultura.

Olá a Sílvio e a todos,

Já vi a opnião do Eduardo Gueron e a sua, porque você não passa nesta
ponte?

O Amit Goswami parece abordar um ponto semelhante ao Arte na Ciência
e não vi má fé na entrevista. Acho que um cientista pode ser humano
também. Uso no dia-a-dia alguns conceitos de Jung, e tenho me dado
bem com eles. Sei que alguns aqui não vão muito com a tal da
quântica, e se esta serve de base para unir ciência e religião aí que
a coisa fica mais feia ainda pro lado dela.

Abraços

Tipoalgo





SUBJECT: Atualização em CeticismoAberto.com 24/03
FROM: "Kentaro Mori" <kentaro.mori@itelefonica.com.br>
TO: ciencialist@yahoogrupos.com.br
DATE: 24/03/2005 03:21


UFOLOGIA

- O Ovo do ET e a Bola de Cristal
"De repente, aquelas três tochas de fogo compridonas passaram e ficou
todo mundo aperreado, gritando, meu Deus, meu Deus, já vai se acabar o
mundo agora".
http://www.ceticismoaberto.com/ufologia/ovo_et.htm

- Terror! Extraterrestres invadem festa de aniversário!
Mulheres gritando em pânico, discos voadores e extraterrestres
malignos: poderia ser um Plano 10 do Espaço Exterior. Mas é um vídeo
no estilo "Bruxa de Blair", com extraterrestres Greys.
http://www.ceticismoaberto.com/ufologia/ets_aniversario.htm

PARANORMAL

- As Irmãs Fox – O Que os Céticos Não Contam
Céticos usam unicamente a confissão de fraude de Margareth para dizer
que o Espiritualismo começou com uma fraude, mas se esquecem de dizer
que Margareth desmentiu a confissão e que a própria confissão possui
erros crassos.
http://www.ceticismoaberto.com/paranormal/mediumfox.htm

- As Irmãs Fox - Pancadas no Espiritualismo
Em retrospecto, é desolador notar que crentes e céticos não produziram
nenhuma avaliação satisfatória. Talvez a única utilidade póstuma do
caso seja exatamente a de recapitular os eventos em seu contexto para
uma olhada sobre a época.
http://www.ceticismoaberto.com/paranormal/irmasfox.htm

CIÊNCIA

- A Terra Oca na Ciência
A Terra oca de fato cruzou pelo menos duas vezes o reino da ciência. O
primeiro encontro é através dos trabalhos dos filósofos naturais dos
séculos 17 e 18. O segundo aconteceu quase trezentos anos depois da
introdução por Halley da idéia.
http://www.ceticismoaberto.com/ciencia/terraoca_ciencia.htm

CETICISMO

- Ciência, Fortianos e Céticos
Este ensaio descreve e discute o modo pela qual a ciência e os
cientistas são representados nas comunidades Fortiana e Cética.
http://www.ceticismoaberto.com/ceticismo/fort_gardner.htm

FORTIANISMO

- Hominídeos Elusivos
Dos aborígenes da Austrália aos indígenas da Amazônia, lendas e
relatos de seres humanóides enormes e primitivos estão espalhados por
todo o mundo. Há realmente um Pé-Grande por trás do mito?
http://www.ceticismoaberto.com/fortianismo/pegrande.htm

- Estatuetas Dogu: Jomon Jedi?
Nosso interesse aqui é algo que liga a pré-história japonesa à
fantasia espacial moderna. Os artefatos de cerâmica Jomon Jidai
incluem estatuetas Dogu. Elas logo foram entendidas por alguns como
"escafandros espaciais".
http://www.ceticismoaberto.com/fortianismo/estatuetas_dogu.htm

REFERÊNCIAS

- Símios de Bondo
Um grupo de símios está intrigando os cientistas: podem ser uma nova
espécie de macaco gigante desconhecida até hoje.
http://www.ceticismoaberto.com/referencias/simios_bondo.htm

- O Garadiávolo
A imagem certamente não é das melhores, e a estranha criatura tem
aparência tão horrível que foi batizada de Garadiávolo em Porto Rico e
outros países latinos.
http://www.ceticismoaberto.com/referencias/garadiavolo.htm

FOTOS DE FANTASMAS
Quatro novas imagens comentadas:
http://www.ceticismoaberto.com/fotos/fotofantasma42.htm

FOTOS DE OVNIS
Cinco novas imagens comentadas:
http://www.ceticismoaberto.com/fotos/fotosovnis14.htm


SOBRE O WEBSITE
Esta deve ser a última atualização de CeticismoAberto.com por pelo
menos alguns meses. O novo sítio contudo deve estar online por pelo
menos mais um ano, graças à colaboração de diversas pessoas, algumas
das quais listadas em:
http://www.ceticismoaberto.com/sobre.htm

Obrigado a todos, boa leitura e até mais!

Mori
http://www.ceticismoaberto.com





SUBJECT: Re: [ciencialist] Sobre a TQG e Amit Goswani
FROM: "Alberto Mesquita Filho" <albmesq@uol.com.br>
TO: <ciencialist@yahoogrupos.com.br>
DATE: 24/03/2005 05:21

----- Original Message -----
From: "Eduardo Gueron"
Sent: Thursday, March 24, 2005 12:53 AM
Subject: Re: [ciencialist] Sobre a TQG e Amit Goswani

> Esse Amit Goswami (citado em alguma das mensagens) é um picareta de marca
> maior. Ele confunde todos os princípios básicos da Mec. QUântica e
> colabora para o mau entendimento desta teoria. Como assim a consciência
> do observador é importante para o resultado do experimento?

Eu diria que o Goswami está valorizando uma das primeiras interpretações
quânticas (se bem que muitos ainda a aceitem) freqüentemente citada como
interpretação ortodoxa ou de Copenhague. Isso fica bem claro na entrevista
do Goswami, em especial quando ele diz:

********* início da citação ************
A famosa disputa entre Böhr e Einstein [...] basicamente, *sempre*
terminava com Bohr ganhando a discussão, mostrando que não há fenômeno no
mundo a menos que ele seja registrado.
********* final da citação ************

Vejo um pouco de fanatismo neste *sempre* e/ou na pretensão de que o debate
teria tido um vencedor, mas não é só o Goswani que vende essa imagem por aí.
Tem muito físico, dentre aqueles considerados bons pelos "referees" das
melhores revistas, que diz a mesma coisa com outras palavras.

Estou lendo um artigo a respeito da Realidade Quântica, escrito por um
adepto da teoria quântica não ortodoxa, J.R. Croca:
http://cfcul.fc.ul.pt/Equipa/Croca/A%20Realidade%20na%20Fisica%20Quantica.doc
Quando o autor se refere à interpretação ortodoxa ele diz o seguinte:

********* início da citação ************
Esta situação levou os físicos ortodoxos a afirmar que o colapso da função
de onda, a redução das múltiplas possibilidades não é, em última análise,
devida a uma interacção física mas sim obra da consciência do observador.
********* final da citação ************

Perceba que o Goswani tomou carona nessa interpretação. Em uma página mais
para a frente o Croca começa a expor o que pensa a respeito:

********* início da citação ************
Apesar dos seus êxitos o paradigma indeterminista de Copenhague foi sempre
combatido por alguns dos mais importantes físicos do século XX, como por
exemplo Alberto Einstein, Luís de Broglie, Schrödinger, Max Planck e muitos
outros.
********* final da citação ************

E após rediscutir a experiência da dupla fenda, conclui seu trabalho com as
seguintes palavras:

********* início da citação ************
Estes factos mostram as limitações e fraquezas da teoria indeterminista e a
necessidade da sua substituição pela nova teoria causal mais geral contém,
naturalmente, do ponto de vista formal e preditivo a teoria indeterminista
da Escola de Copenhague como um caso particular. Para terminar só quero
acrescentar que os progressos recentes da ciência conduzem novamente, apesar
da forte oposição das correntes conservadoras, à reposição da causalidade e
por conseguinte à recuperação da existência de uma Realidade física
objectiva independente do observador.
********* final da citação ************

É importante assinalar, haja vista que estamos frente a uma interpretação a
confundir ciência com religião, que o artigo acima citado foi apresentado no
Colóquio "A Mente, a Religião e a Ciência", que aconteceu na Universidade de
Lisboa em 30 de novembro de 2001.

> Se vc
> fizer a experiência da fenda dupla com alguém pondo o
> dedo em uma das fendas e vendo o aparato onde a figura
> de interferência é ou não formada ou pôr um robô para
> tapar uma das fendas e filmar o aparato, o resultado é
> exatamente o mesmo.

Eu tenho a impressão que para a interpretação de Copenhague não importa o
fator consciência em si, mas sim a cognoscibilidade. Se você filmou, não há
necessidade de você visualizar o filme para confirmar (ou conhecer) a
medida, isso segundo o realismo clássico. Com respeito à ortodoxia de
Copenhague não posso dizer nada, mas não acho impossível que alguns dentre
eles interpretem o filme não visto como equivalente a uma experiência não
feita (isso caracterizaria o não-realismo quântico da escola de Copenhague).
Isso sem dúvida deu origem a muita confusão e muitos picaretas se
aproveitaram dessa confusão para vender ilusões. Quanto ao Goswani não digo
nada pois pouco o conheço. Acho apenas que se ele for honesto, certamente
deve ter embarcado num navio que não chegará a destino algum. Noto, não
obstante, que ainda tem muito físico bom neste mesmo navio que saiu de
Copenhague há cerca de 70 anos e ao que tudo indica já está a deriva, mas
isso só o tempo confirmará.

[ ]´s
Alberto
http://ecientificocultural.com/indice.htm
Mas indiferentemente a tudo isso, o neutrino tem massa, o elétron não é
uma carga elétrica coulombiana e a Terra se move. E a história se repetirá.



SUBJECT: Problema basico de volumes - comentarios
FROM: "Luiz Ferraz Netto" <leobarretos@uol.com.br>
TO: <ciencialist@yahoogrupos.com.br>
DATE: 24/03/2005 07:14

Problema original apresentado por Nuno Figueiredo em 22/03/05:
===================================================
> Prezado professor:
> Achei o seu site muito interessante. Assim, estou a lhe escrever um
> função de uma duvida.
> O meu cunhado tem uma fabrica de cosméticos e, surgiu uma discussão
> entre nós: Ele insiste, que em determinado vasilhame que tem a
> capacidade para 4 litros, recebe 4 litros de água, mas uma quantidade
> diferente (menor) de shampoo.
> Por mais que eu tentasse explicar que o litro é uma unidade
> universal(não varia com o tipo de produto) por se tratar de uma medida
> de volume, ele insiste no erro.
> Eu expliquei-lhe que o peso sim, varia conforme o produto por causa da
> densidade, mas o volume jamais muda. Ele aceitou que o peso varia, mas
> o volume também.
> Eu argumentei entre outras, que não poderíamos ter duas variáveis para
> o mesmo produto, pois tornar-se-ia impossível calcular uma em função
> da outra... Tentei explicar que um litro de água ou um litro de chumbo
> derretido são um litro. Apenas um pesa aproximadamente um Kg e o outro
> vários Kg.
> Ele insiste que está certo e que já foi autoado pelo inmetro mais de
> uma vês por isso. Eu, irritado disse que não era possível e, que se o
> inmetro multou porque um litro não era um litro, o inmetro estava
> errado. É obvio que ele foi autoado por diferenças no calculo do peso
> liquido e não no volume em l ou ml.
> Bem, resumindo, fui incapaz de lhe explicar isso, apesar da minha
> formação superior em física (incompleta).
> Achei que o tema é interessante o suficiente para o expor ao senhor. Se
> não for pedir demais, gostaria de uma opinião para a apresentar.
> Talvez o problema seja apresentar de forma compreensível a definição
> de litro. Sem se levar em consideração os gases por serem fluidos
> compressíveis, e claro.
> Agradeço de antemão a sua resposta,
==========================================================
Primeira proposta de resposta, por S. Taborda, em 22/03/05:
==========================================================
O volume de um recipiente (vasilhame) é constante. O recipiente suporta
o mesmo volume de qualquer substancia. Parece que é isto que o seu
cunhado falha em entender. O volume do recipiente não depende do liquido
que ele contém, pois o volume é sempre o mesmo, mesmo que o recipiente
estiver vazio.
O volume do recipiente é calculado geometricamente. Existem formulas
matemáticas para o calcular para recipientes simples. Por exemplo, um
cilindro tem um volume igual ao produto da altura pela área da base.
Esta formula é a base dos medidores de volume. Que são normalmente
cilindros transparentes com escalas do lado de fora.
Se a forma do recipiente não é simples de corresponder com sólidos
geométricos simples - como cilindros, cubos e esfera - então existe um
truque, que é encher o recipiente com água e despejar essa água num
recipiente do qual sabemos o volume. (se possível um recipiente calibrado).
Sendo que o volume do recipiente depende apenas da forma do recipiente e
não do que ele contém, o seu volume é constante e não varia conforme o
liquido que se coloca nele.
=================================================================
Comentário do Oráculo, postado em 22/03/05:
=================================================================
Olá Luiz
Acho que o enfoque do Taborda será a única forma de convencer cunhado em questão..:-) Se ele puder perceber que o volume do recipiente, mesmo vazio (e mesmo no vácuo) ainda será de um litro, talvez perceba a incongruência de afirmar que tem um litro de água e menos de shampoo. Me parece que, apenas com cálculos e definições, será mais difícil, já que deve faltar a base de conhecimento necessária para absorver esse tipo de evidencia ou argumento.

Talvez exista também um problema extra, que é a verificação que organismos de controle fazem com produtos destinados ao consumo público. A água, ao preencher o recipiente, ocupará todo o volume do mesmo. Mas outros tipos de produto podem deixar espaços ou mesmo conter ar, como no caso de shampoo, ao ser inserido com maquinas de pressão automáticas.

É possível que os problemas do cunhado com o instituto de medição e controle se deva a metodologia, por exemplo, processar o shampoo contido na embalagem de forma a retirar todo o ar (bolhas) e só então verificar o volume. Em alguns casos, principalmente com aparelhos de preenchimento da industria descalibrados ou fora de ajuste, as bolhas podem representar um volume significativo, e causar a multa do fabricante.

Isso pode ter sido interpretado pelo cunhado como sendo uma diferença real de volume entre água e o shampoo na embalagem.

A abordagem do Taborda pode faze-lo compreender que um litro de água e um litro de shampoo contém ambos o mesmo volume, mas de qualquer coisa que estiver dentro do recipiente, seja apenas água, apenas shampoo ou shampoo e ar (em bolhas ou diluído) e que foi isso que causou seus problemas com a medição.
==============================================================
Segunda proposta de resposta, postado por Léo, redação final, em 24/03/05:
==============================================================
Olá José Nuno,
A "falha" de seu cunhado talvez possa ser sanada se vc apresentar a ele definições e conceitos rigorosamente científicos. Devemos usar os termos/palavras corretamente e não tentar extrapolar tais termos para o uso popular. Serve de exemplo o "trabalho", um conceito físico definido para forças, mas é, sem dúvida, uma palavra que leva em si um forte componente psicológico. É bem possível que seja necessário um esforço maior para disputar uma partida de tênis do que para colocar em ordem alfabética um conjunto de fichas de arquivo, mas nós consideramos a última atividade como um 'trabalho' e a primeira como 'divertimento', 'competição' ou 'exercício', mas nunca como trabalho!. O 'trabalho' da física, pouco ou nada tem a ver com o 'trabalho' da telefonista, do ascensorista etc.

Todavia, devo advertir de imediato: seu cunhado pode não estar falhando!

Então, a primeira coisa a ser lapidada, é o conceito de "volume" e suas unidades (oficial e legal). "Volume" é o resultado da medição de um determinado 'espaço' (propriedade geométrica de apresentar três dimensões). Esse 'espaço' pode ser, por exemplo, aquele limitado pelas 4 paredes, teto e chão de uma simples sala. Para a medição desse espaço torna-se necessário que se defina um 'espaço unitário' ao qual se associe a 'unidade de volume'; um cubo de 1 m de aresta limita um espaço que tem, por definição, o volume unitário, ou seja, 1 metro cúbico.
Além do 'metro cúbico', que é a unidade oficial de volume, aceita-se, legalmente, a unidade denominada "litro", que corresponde à milésima parte do metro cúbico, ou 1 metro cúbico = 1 000 litros --- há outra definição para o litro; não vem ao caso.

Certos depósitos/vasilhames têm capacidade de armazenamento de líquidos, são assim os tanques de gasolina dos postos, os bujões de água, as caixas d'águas domiciliares, as garrafas PETs etc. Essas capacidades de armazenamento serão dadas pelas medidas dos espaços (essas medidas serão os volumes) disponíveis nesses depósitos/vasilhames. Tem garrafa PET, por exemplo, cuja capacidade de armazenamento (isso é um espaço geométrico) é de 2 litros (isso é um volume). Tem lata de mantimento que disponibiliza um volume útil de, exato, 1 litro. Qualquer coisa que vc coloque lá dentro e preencha totalmente o espaço disponível ocupará "por definição" o volume de 1 litro.

Mas, isso não significa que lá dentro exista "1 litro" daquele material!

Vou exemplificar:
Aqui na minha cidade vendem-se jabuticabas soltas nas feiras livres; a 'unidade' de comercialização é "uma lata de capacidade 1 litro". Qdo vc pede 2 litros de jabuticabas o feirante enche duas vezes aquela lata --- eis "2 litros de jabuticaba".
Você concordará que ali não há "realmente" 2 litros de jabuticaba, uma vez que entre elas, dentro da lata, tem enorme quantidade de espaço vazio. Seu cunhado tem, sobre o shampoo, a mesma idéia que vc está tendo agora do 'litro de jabuticaba. O 'litro de shampoo' não tem realmente "1 litro de shampoo"!!!!!!

Um shampoo é um produto cujo fórmula e fabricação implica num resultado pouco denso devido à minúsculas gotículas de ar (gases) e isso o coloca em similaridade com a jabuticaba.

Como resolver o problema?

Numa solução 'ideal'o shampoo deveria ser 'envazado' no vácuo (para eliminação das bolhas de ar)--- obviamente teríamos um shampoo mais denso que aquele normalmente vendido por ai. Esse seria o shampoo 'padrão' --- todos os demais shampoos teriam um adjetivo extra: shampoo a 10%, shampoo 30 ... ou sei lá o que.

Então, o dilema deixa de ser dilema entendendo-se que:

(a) "volume" é a especificação de um espaço; é um invariante, independente do que está preenchendo esse espaço. Assim, quer o espaço contenha água, shampoo ou jabuticabas --- o volume é de 1 litro (no exemplo).

(b) "volume" de substâncias porosas é um 'volume aparente'; não há realmente o 'tanto' de substância que se espera. Para tais casos, "apenas" volume, não é grandeza suficiente para especificação da substância.

PS: Vc percebe o erro conceitual ao dizer : Vamos medir o volume dessa caixa? Volume já não é uma medida?

Aquele abraço,
===============================================================
Pronto .... fica registrado o pedido do Nuno e as propostas. Fico no aguardo das sugestões finais para poder enviar o definitivo ao Nuno.

Aquele abraço a todos que, permanentemente, têm contribuído para o assentamento da Ciência na cuca de nossos jovens e 'outros'.

aquele abraço,

===========================
Luiz Ferraz Netto [Léo]
leobarretos@uol.com.br
http://www.feiradeciencias.com.br
===========================


SUBJECT: Re: [ciencialist] Editor de calculos e formulas
FROM: "Luiz Ferraz Netto" <leobarretos@uol.com.br>
TO: <ciencialist@yahoogrupos.com.br>
DATE: 24/03/2005 07:32

Olás .......

minha colaboração nesse tema: em todas minhas páginas do Imperdível que requerem as 'formuletas' etc. eu uso do Equation Editor (da microsoft) e o Paint Bush (da microsoft). Como exemplo dentre as mais de 2000 páginas veja

http://www.feiradeciencias.com.br/sala04/04_QC05.asp#Solução%2008

Primeiro escrevo a fórmula, desenvolvimento algébrico etc. no Equation; seleciono tudo que escrevi e 'control-C'; abro o Paint e 'control-V'. No Paint, ajusto a formatação, o texto bem alinhado etc. e salvo com sufixo .bmp, 16 cores. Feito isso, coloco a figura .bmp na página do FrontPage onde desenvolvo o trabalho; clico sobre a figura e passo a ferramenta de transparente, a figura será salva em .gif, transparente no local que vc escolher.
Isso, tudo é feito numa espetacular rapidez.
[]'
===========================
Luiz Ferraz Netto [Léo]
leobarretos@uol.com.br
http://www.feiradeciencias.com.br
===========================
-----Mensagem Original-----
De: "Franco" <dfranco@pop.com.br>
Para: <ciencialist@yahoogrupos.com.br>
Enviada em: quarta-feira, 23 de março de 2005 14:43
Assunto: Re: [ciencialist] Editor de calculos e formulas



Para Windows ou GNU/Linux?
Se não me engano, nos arquivos desta lista há um artigo em pdf
intitulado "Introdução a programas físico-matemáticos livres", sugerindo
alguns softwares - se não encontrar posso lhe enviar em pvt.

Uma maneira de usar fórmulas e/ou gráficos em grupos de discussão, seria
editá-los em um programa e salvá-los como documento de imagem para
enviá-los às listas; porém, deve-se verificar se determinada lista
aceitará anexos, caso contrário, não adiantará nada.
[...]


SUBJECT: Re: [ciencialist] Re: Editor de calculos e formulas
FROM: "Luiz Ferraz Netto" <leobarretos@uol.com.br>
TO: <ciencialist@yahoogrupos.com.br>
DATE: 24/03/2005 07:44


--- Em ciencialist@yahoogrupos.com.br, Franco <dfranco@p...> escreveu
>
> Uma maneira de usar fórmulas e/ou gráficos em grupos de discussão,
seria
> editá-los em um programa e salvá-los como documento de imagem para
> enviá-los às listas; porém, deve-se verificar se determinada lista
> aceitará anexos, caso contrário, não adiantará nada.

Cesar informa:

É o caso desta lista. Que filtra também o HTML (eu leio a lista a
partir do navegador internet, no endereço
http://br.groups.yahoo.com/group/ciencialist, e não vejo nada de HTML
nem figuras).

Léo complementa:

Há muito tempo, no nascimento do C-List, houve uma 'votação' sobre qual seria a 'norma' da lista qto à apresentação --- democraticamente venceu o 'modo texto', sem anexo, seco, curto e grosso, sem enfeites, sem segundas intenções; uma iniciativa séria de participantes sérios. [fica explicado, assim, a relutância em permitirem minha entrada na lista]
Não confundir 'sérios' com 'sisudos' etc.
[]'
Léo


SUBJECT: Re: [ciencialist] O cético que encontrou um bom astrologo..:-) (era Zodiaco)
FROM: "Silvio" <scordeiro@terra.com.br>
TO: <ciencialist@yahoogrupos.com.br>
DATE: 24/03/2005 08:14

Murilo:
Permita-me um pitaco:
você diz: "tudo que ainda não aconteceu pode ser mudado". Ora, isso é uma
falácia pois se não aconteceu e não podemos prever o futuro lógicamente esse
raciocínio é pueril.

A tábua de marés é fornecida pelo Minist. da Marinha e feita segundo as
técnicas internacionais para esse evento. São absolutamente corretas pois
são científicas. Todas as embarcações dependem delas para a navegação.

Peço-lhe, já que acredita em intuição, dar uma dica para a Megasena (em PVD)

sds.,
silvio.
-----Mensagem Original-----
De: "murilo filo" <avalanchedrive@hotmail.com>
Para: <ciencialist@yahoogrupos.com.br>
Enviada em: sábado, 5 de fevereiro de 2005 18:44
Assunto: RE: [ciencialist] O cético que encontrou um bom astrologo..:-) (era
Zodiaco)


>
> Oi, Oraculo.
> Estava me referindo à recepção e medição de resultados, e sempre usando
> êste
> estado mental comum que nos suporta por êste mundão velho sem porteira.
> A inteligência normalmente treinada e uma estrutura psicológica estável
> conseguem razoável cognição entre o que é furado e o tal ''epa, aquí tem
> alguma coisa...''
> A astrologia é um outro mundo onde se encontram todas aquelas mumunhas
> normais e comuns aos outros ramos, inclusive os sacanas e aquêles que
> prometem o impossível.
> Aprecio muito a intuição.
> Previsões? Acho meio difícil, mas já ví. Encaro esta coisa mais como uma
> tábua das marés, sempre conforme a pessoa e um cacetão de variáveis,
> inclusive as mais prosáicas. Tratam-se de tendências, informações de prá
> onde e como correrá, ou corria, a maré. Uma ferramenta a mais.
> Quando a indicação de uma 'maré' é muito forte, a coisa é quase
> inescapável.
> Não pense que espero receber de qualquer outra coisa externa um refresco
> p/meus pepinos! Estamos todos aquí só para aprender, e o êrro ensina mais
> do
> que o acêrto.
> Tudo que ainda não aconteceu pode ser mudado e qualquer tendência poderá
> ser
> contrariada pela vontade, pela ação e até pela ignorância humanas. abr.
> M. sp 05/fev
>
>
>>From: "Oraculo" <oraculo@atibaia.com.br>
>>Reply-To: ciencialist@yahoogrupos.com.br
>>To: <ciencialist@yahoogrupos.com.br>
>>Subject: [ciencialist] O cético que encontrou um bom astrologo..:-) (era
>>Zodiaco)
>>Date: Sat, 5 Feb 2005 19:01:03 -0200
>>
>>Olá Murilo
>>
>>Infelizmente, essa previsão vai falhar.:-)
>>
>>E o motivo é simples, mesmo que um cético ou cientista se encaixe no caso
>>relatado por você, encontrar um astrologo que faça seu mapa astral com
>>enorme relevância de dados, ainda assim ele saberá que é apenas um caso
>>anedotico (não estatistico) e que isso não representa evidencia confiável.
>>
>>Veja, não é porque conheço o efeito da percepção seletiva, da pareidolia,
>>do famoso "conte os acertos e ignore os erros" que estou imune a
>>eles..:-)
>>Sei perfeitamente que posso ser enganado por minha mente ou por um bom
>>astrologo (ou qualquer outra forma de alegação sem evidencias) tanto
>>quanto
>>sou enganado por mágicos de palco (que repetem exatamente os mesmos
>>fenomenos sobrenaturais)..
>>
>>Assim, mesmo que eu encontre um astrologo que faça meu mapa astral, de
>>confiança, competente, e que eu me veja refletido no mapa, ainda assim vou
>>manter o ceticismo natural e pedir uma avaliação de outra pessoa, outra
>>analise, uma significancai estatistca no acerto do astrolog, etc, etc,
>>etc.
>>Eu posso me enganar (e minha mente tanto quanto eu..:-) como qualquer
>>pessoa.
>>
>>E tem de perceber que a classificação "competente", "consciente", "sério",
>>etc, é um julgamento de valor. Como escolher o astrologo que se encaixe?
>>Como decidir se é competente, ou honesto? Como, engim, filtrar os dados e
>>analisar essa questão? Bem, se posso sugerir algo, devemos, de novo, usar
>>o
>>sistema rigoroso da ciência apra faze-lo..:-) Ou teremos, de novo, de
>>acreditar na palavra de quem nos recomenda o astrologo. Se você decidir
>>analisar o trabalho de um astrologo, apra decidir se ele se encaixa nos
>>termos que propos, o que usará como ferramenta de análise?
>>
>>É isso que parece dificil de explicar e de entender: não é por falta de
>>experiencia pessoal que cientistas e céticos não acreditam, é apenas por
>>saber que é necessário mais que isso para uma conclusão confiável.
>>
>>Se eu tomar uma erva, receitada por um curandeiro, e minha doença se
>>curar,
>>ainda assim eu vou manter o ceticismo a respeito do poder real de cura da
>>mesma. E estarei apenas sendo cuidadoso com a conclusão, já que para
>>determinar a eficácia de uma droga ou medicamente são necessários testes
>>rigorosos, duplo cego, estudos longos e muita reprodutibilidade e análise.
>>E mesmo você deve concordar com isso, e se recursar a tomar drogas ou
>>ervas
>>(que podem matar) sem essas precauções, não importa o testemunho de quem
>>quer que seja que tenha se curado ao toma-la, não?
>>
>>Parece que quem crê em algo pensa que céticos tem falta de experiencias
>>pessoais..:-) Que, como você previu, se encontrarem um "bom astrologo"
>>mudarão de opinião e passarão a acreditar. Não é assim que funciona.:-)
>>Ciência é mais que relatos anedoticos, mesmo os pessoais, mesmo as
>>experiencias que eu experimento. É um conhecimento que só se tornou
>>confiável quando passou a ser produzido dentro do rigor necessário.
>>
>>Um abraço.
>>
>>Homero
>>
>> ----- Original Message -----
>> From: murilo filo
>> To: ciencialist@yahoogrupos.com.br
>> Sent: Friday, February 04, 2005 7:30 PM
>> Subject: RE: [ciencialist] Re: Zodiaco
>>
>>
>> Pois eu tenho uma previsão para alguns de vcs.
>> ( e nada tenho a ganhar com isto! )
>> Alguns felizardos, e privilegiados, terão, algum dia, a oportunidade de
>> conhecer a um astrólogo ( a ) SÉRIO, COMPETENTE E C O N S C I E N T
>>E...
>> quando então ficarão bem desconfiados a respeito de uma arte tão antiga
>> quanto a humanidade. Ficarão bem encafifados e também verão o quanto a
>>coisa
>> é diferente daquilo que sai nos jornais.
>> Todo aquêle cascão da má informação e da superstição irá cair fora.
>> Está nos astros... aguardem e/ou procurem. Alguns ficarão menores, mas
>>de pé
>> no chão (e ainda assim seguirão sem qualquer explicação cartesiana,
>> tipo
>> batatolina!).
>> A gente não pode gastar a vida só duvidando da inteligência dos outros,
>>por
>> todo o tempo, pô!
>> Sinceramente, M. SP 04/fev
>>
>> >From: Maria Natália <grasdic@hotmail.com>
>> >Reply-To: ciencialist@yahoogrupos.com.br
>> >To: ciencialist@yahoogrupos.com.br
>> >Subject: [ciencialist] Re: Zodiaco
>> >Date: Fri, 04 Feb 2005 04:05:36 -0000
>> >
>> >
>> >Sérgio:
>> >
>> >Agradeço.
>> >A astrologia se usa aqui para fazer estatísitica a brincar. Pega-se
>> >num jornal e se vê qual o signo que diz " vais cair do escadote". Só o
>> >professor sabe qual o jornal escolhido e todos respondem ao
>> >questionário. Depois os que eram do signo da previsão "má" sofrem
>> >comparação.Mas todos preenchem o inquérito. As perguntas também
>> >permitem tirar conclusões sobre supertições, crenças e mitos. Quanto
>> >mais turmas mais científico será (amostra maior)
>> >Ao Ofiuco também se chama Serpentário, me lembrei agora.
>> >Na escola costumamos apenas nos referirmos às constelações mais
>> >visíveis em cidades com poluição luminosa. Ora se se souber que aqui
>> >onde vivo da Ursa Menor apenas se vê a Polar e mesmo meia "safada" nem
>> >capricónio nos interessa para meninos de 11 e 12 anos.
>> >Quando os alunos fazem a "fita" do Zodíaco na parede da sala de aula,
>> >cúbica ou paralelipipédica, se representam todas para se perceber essa
>> >do "sol está na constelação..." sendo o Sol um aluno que anda junto às
>> >paredes simulando o movimento aparente e a Terra os restantes alunos
>> >num foco. De resto as constelações só começa a ter muito valor quando
>> >queremos ver cometa ou asteróide
>> >Não gostamos quando, a meio da noite, nos aparecem adultos a perguntar
>> >se pelo facto de Vénus estar em Virgem eles vão ter mais sorte no
>> >Totoloto.
>> >Daí as alergias com a astrologia.
>> >E este ano tive até um pai engenheiro a perguntar-me se achava que o
>> >professor X iria faltar muito...O home estava a confundir astrónomo
>> >com astrólogo. Aceitamos bem melhor sermos gastrólogos e gastrónomos.
>> >Mas a astrologia dá para muita brincadeira. É que este pessoal que nos
>> >chega à mão aos 11 anos já tem pelo menos 3 anos de laboratórios e
>> >trabalhos de campo. Vem com a mania de apanhar a alma na ponta de um
>> >bisturi. Vamos ver o que esta geração dará na universidade dentro de 4
>> >anos. Pois se trata da reforma do ensino em curso.
>> >Entretanto já lera o artigo do Galileu noutra lista, o CdA
>> >
>> >Abraço
>> >Maria Natália
>> >PS Pois como agora estou a dar aulas longe de casa o tempo vai
>> >falhando para acompanhar diariamente a lista...
>> >
>> >--- Em ciencialist@yahoogrupos.com.br, "Sergio M. M. Taborda"
>> ><sergiotaborda@t...> escreveu
>> > > Maria Natália wrote:
>> > >
>> > > >
>> > > > Estebam:
>> > > > O zodíaco dos astrólogos é o de há 2000 anos e não tem em conta
>> os
>> > > > movimentos do eixo da Terra. O zodíaco tem 13 constelações: a
>>seguir
>> > > > ao capricónio tem o Ofiuco...São 13 os tais signos. E agora que
>> me
>> > > > dizeis acerca do Ofiuco e que de acordo com o zodíaco actual é o
>>meu
>> > > > signo: poderei amanhã comprar lotaria para me sair prémio?
>> > >
>> > > Eu ja respondi a essa pergunta. E a resposta é : Esse argumento
>>faria
>> > > sentido SE a astrologia fosse baseada nas constelações.
>> > > Como não é , insistir nesse argumento é pura besteira (=coisas que
>>os
>> > > bestas fazem)
>> > > Jà lhe expliquei pq so ha 12 signos, o que já foi aqui explicado
>> por
>> > > outros. É uma razão matemática para que seja assim. É baseada no
>> > > conceito de Periodo Sinódico.
>> > > Ha uma logica , séria, matemática, fisica, _astronomica_ para o
>>numero
>> > > 12. Só que aos astronomos - com problemas de falta de atenção -
>> > > inventaram esse argumento sobre Ofucio.
>> > > fazer o quê ? Mas se vc acredita nesses mentirosos, não é diferente
>>de
>> > > quem acredita em astrologos fajutos.
>> > >
>> > > > Desculpe, por acaso ouvi dizer que os astrónomos e astrofísicos
>>falam
>> > > > besteira? Ou entendi mal?
>> > >
>> > > Ouvio bem. Eles dizem mesmo. E ha muitas mais do que a de Ofucio.
>> > >
>> > >
>> > > Sérgio Taborda
>> > >
>> > >
>> > >
>> > > --
>> > > No virus found in this outgoing message.
>> > > Checked by AVG Anti-Virus.
>> > > Version: 7.0.300 / Virus Database: 265.8.5 - Release Date:
>>03-02-2005
>> >
>> >
>> >
>>
>>
>>
>>
>> ##### ##### #####
>>
>> Para saber mais visite
>> http://www.ciencialist.hpg.ig.com.br
>>
>>
>> ##### ##### ##### #####
>>
>>
>> Yahoo! Grupos, um serviço oferecido por:
>> PUBLICIDADE
>>
>>
>>
>>
>>------------------------------------------------------------------------------
>> Links do Yahoo! Grupos
>>
>> a.. Para visitar o site do seu grupo na web, acesse:
>> http://br.groups.yahoo.com/group/ciencialist/
>>
>> b.. Para sair deste grupo, envie um e-mail para:
>> ciencialist-unsubscribe@yahoogrupos.com.br
>>
>> c.. O uso que você faz do Yahoo! Grupos está sujeito aos Termos do
>>Serviço do Yahoo!.
>>
>>
>>
>>[As partes desta mensagem que não continham texto foram removidas]
>>
>
>
>
>
> ##### ##### #####
>
> Para saber mais visite
> http://www.ciencialist.hpg.ig.com.br
>
>
> ##### ##### ##### #####
> Links do Yahoo! Grupos
>
>
>
>
>
>
>
>
>
>



SUBJECT: Terremoto em ortugal.
FROM: "Silvio" <scordeiro@terra.com.br>
TO: <acropolis@yahoogrupos.com.br>, <ciencialist@yahoogrupos.com.br>, <Conversa_de_Botequim@yahoogrupos.com.br>
DATE: 24/03/2005 08:27



Movimento sísmico em Portugal ...


Depois dos problemas acontecidos na Ásia, o governo de Portugal resolveu instalar um medidor de abalos, que cobre todo o país.
Então, foi enviado, pelo Centro Sísmico Nacional ao quartel da polícia da cidade de Vilamoura, no norte de Portugal, um telegrama, que dizia:
" Possível movimento sísmico na zona. Ponto. Muito perigoso, superior Richter 7. Ponto. Epicentro a 3 km do povoado. Ponto. Tomem medidas. Ponto. Informem resultados com urgência. Ponto."
Bem, os dias se passaram, e só depois de mais de uma semana é que foi recebido no Centro Sísmico Nacional um telegrama que dizia:
" Aqui é do Quartel da Polícia de Vilamoura. Ponto. Movimento sísmico totalmente desarticulado. Ponto.


O tal Ritchter tentou fugir, mas foi abatido a tiros. Ponto, desativamos as zonas Ponto, as putas estão todas prezas a trabalhar na lavanderia dos presidios feminino Ponto. Epicentro, Epifânio e três cupinchas detidos. Ponto.


Não respondemos antes, porque aqui houve um terremoto do caralho. Ponto.

Abraços e beijos sem tremer.


[As partes desta mensagem que não continham texto foram removidas]



SUBJECT: Re: [ciencialist] Gravitons
FROM: JVictor <jvoneto@uol.com.br>
TO: ciencialist@yahoogrupos.com.br
DATE: 24/03/2005 09:25

Professor Ferraz,

1) Tenho lido alguns trabalhos de seus tabalhos lá no imperdível. Estou
fazendo as lições de casa direitinho. E comecei a estudar(estudar, não
ler, meramente) este que você indicou abaixo, no e-amail que começa com
"vichi", que não entendí.
2) Seus trabalhos têm qualidade e conteúdo suficientes para provocar em
estudantes de nível superior exclamações como: puxa, e porque não me
disseram que era(refiro-me a qualquer dos trabalhos que lí) tão
profundo e ao mesmo tempo tão simples?
3) Você não pensa em transformar tudo em um livro, que seria separado em
volumes? Apesar de se ter hoje livros didáticos de nível superior e
segundo grau muito bons, não tenho dúvidas de que o conjunto de seu
trabalho superaria todos, em muitos aspectos.
4) Finalmente, esclareço que a "declaração" acima não tem outro
objetivo senão prestar ums erviço à causa do ensino da física, e de
maneira alguma, o que alguns podem estar pensando, até com um sorrisinho
maroto!

5) Quanto ao furo do PE no estudo que você fez, ainda não ví como. Mas
chego lá, garanto.

Sds,

Victor.






Luiz Ferraz Netto escreveu:

> Vichi!!! Essa mensagem do JVictor escapou-me de ser lida --- ??? ---,
> mas, em tempo, já vou avisando que não concordo com o princípio da
> equivalência como está apresentado.
>
> Notoriamente ele fura para os líquidos.
>
> Escrevi algo sobre isso (e os mais antigos de C-list já participaram
> disso) na Sala 07, em:
>
> http://www.feiradeciencias.com.br/sala07/07_T04.asp
>
> O empuxo de Newton nos referenciais acelerados.
>
> []'
> ===========================
> Luiz Ferraz Netto [Léo]
> leobarretos@uol.com.br
> http://www.feiradeciencias.com.br
> ===========================
> -----Mensagem Original-----
> De: "JVictor" <jvoneto@uol.com.br>
> Para: <ciencialist@yahoogrupos.com.br>
> Enviada em: segunda-feira, 21 de março de 2005 19:44
> Assunto: [ciencialist] Gravitons
>
>
>
> Luiz Ferraz Netto escreveu:
>
> Apreciaria comentários sobre o seguinte texto:
>
> http://rolfguthmann.sites.uol.com.br/TQG/tqg.html
>
>
> Professor Luiz Ferraz,
>
> > Lí o artigo até o quinto parágrafo da seção 2. E parei. Até chegar
> > ao item 2 encontramos hipóteses absolutamente estranhas e sem
> > significado físico, pelo que já peço ao autor que seja mais claro..
> >
> > Com relação ao item 2- O que é gravidade?, transcrevo dois conceitos
> > absolutamente equivocados, que comento rapidamente logo após o destaque.
>
> 1. " Para Einstein, a gravidade causava uma deformação no espaço tempo
> contínuo e, com esta idéia, ele desenvolveu uma álgebra muito complexa
> que a descreve apenas geometricamente."
>
> Comentário: 1) É o contrário: a gravidade não é causa; é efeito da
> curvatura do espaço. 2) Einstein NÃO desenvolveu nenhuma álgebra muito
> complexa. Ele usou um formalismo matemático, novo na época, criado por
> outros matemáticos, baseado em geometria diferencial, formalismo esse
> de jeito nenhum complexo. Desculpe-me o autor, mas esse é o tipo do
> erro conceitual inadmissível para alguém que se propõe a elaborar uma
> nova teoria.
>
> > 2. - " Uma outra evidência se reporta a teoria da relatividade de
> > Einstein, que está embasada no princípio da equivalência o qual
> > estabelece que a massa, ou a masssa inercial e o peso, ou massa
> > gravitacional de materiais diferentes sofrem a mesma aceleração... "
>
> Comentários: Os erros conceituais aquí são para ninguém botar
> defeito!. Massa e peso são coisas diferentes!. E esse princípio de
> equivalência é devido a Galileu e é chamado Princípio de Equivalência
> fraco. Einstein o estendeu a toda a física, e o PE de Einstein tem tem
> a seguinte forma:
>
> " Um laboratório local, não girante, em queda livre, é equivalente, para
> a realização de qualquer experiência, a um referencial inercial numa
> região do espaço sem gravidade." Esta é a forma forte do Princípio de
> Equivalência, devido a Einstein. É o mesmo que dizer: se você estiver em
> uma nave, no espaço, e esta estiver sob uma aceleração, digamos, de
> 10m/s/s, você será empurrado contra o piso da nave e não saberá dizer
> se essa aceleração é devida aos motores que a provocam ou se é devida
> aos efeitos gravitacionais de alguma massa grande nas proximidades,
> como, por exemplo, a terra(cujos efeitos gravitacionais, que todos
> conhecemos, expressam-se por uma aceleração, o g= aprox. a 10m/s/s e nos
> pregados no chão, do mesmo jeito que o astronauta em relação ao piso da
> nave, com tudo que tem direito!) ou outro planeta. Ou seja, essas
> aceleraçãos são equivalentes!. Na nave, uma balança dirá que o seu peso
> será, digamos, 700kg.m/s/s, se for um gordinho, como eu, de 70 Kg de
> massa. Se os motores aumentarem a aceleração, a balança vai acusar um
> peso maior. Na terra, se consideramos que aceleração gravitacional tem
> o mesmo valor, aprx. 10m/s/s, uma balança indicará o mesmo peso para
> você, ou seja 700kg.m/s/s! Refletindo sobre coisas assim, num momento
> de inspiração, Einstein chegou ao seu racicínio mais feliz, segundo
> disse: concluiu que uma pessoa em queda livre não sente seu próprio
> peso! Ou seja, para alguém que cai em queda livre, não existe, ao menos
> nas vizinhanças, qualquer campo gravitacional. Com esse conceito,
> Einstein mostra também que a gravidade é uma coisa cuja existência é
> relativa. Independe da constituição dos corpos. Aliás, o PE fraco já diz
> a mesma coisa. É esta a base da TRG! Compare, então, o que expressei
> acima, com suas afirmações abaixo e decida o autor se ainda mantém:
>
> a) sua hipótese, item 1, que não me animei a comentar(idem, outras, de
> mesma qualidade):
> "Veremos que a a gravidade é o resultado do desequilíbrio entre força
> coloumbiana e força centrípeta",
>
> b) ou com esta, quinto parágrafo do item 2:
> " ...podemos afirmar que a gravidade é uma inércia, e é causada por uma
> força eletromagnética de origem nuclear, estes indícios( (??),
> estupefação minha)) ainda apontam, que a diferença relativa das forças
> eletrostática e centrífuga nos átomos, é a fonte ou causa da
> gravidade..."
> Comentário: não era centrípeta?. Só lembro uma coisinha adicional: força
> centrípeta e força centrífuga são aplicadas em corpos diferentes,
> portanto, uma não é reação à outra, e a centrífuga surge em alguns
> sistemas de referência e outras não! Logo..." Pode esclarecer o
> seguinte enunciado, de vossa lavra?: " ...podemos afirmar que gravidade
> é uma inércia, e é causada por uma força eletromagnética de origem
> nuclear".
>
> c) todo o resto que não lí...
>
> Não podendo ir mais além, por conta do " impacto" das coisas acima, e
> logo no início do trabalho(!), sugiro fortemente que o autor, se
> estiver com vontade e for corajoso, como parece que o é, publicando
> esta, desenvolva SUA OUTRA TEORIA. Quem nem lí ainda!
>
> Sds,
>
> Victor.
>
>
>
>
>
>
>
> ##### ##### #####
>
> Para saber mais visite
> http://www.ciencialist.hpg.ig.com.br
>
>
> ##### ##### ##### #####
> Links do Yahoo! Grupos
>
>
>
>
>
>
>
>
>
>
> --
> No virus found in this incoming message.
> Checked by AVG Anti-Virus.
> Version: 7.0.308 / Virus Database: 266.7.3 - Release Date: 15/03/2005
>
>
>
>
> ##### ##### #####
>
> Para saber mais visite
> http://www.ciencialist.hpg.ig.com.br
>
>
> ##### ##### ##### #####
>
>
> *Yahoo! Grupos, um serviço oferecido por:*
>
> *
> <http://br.rd.yahoo.com/SIG=12aiicui1/M=264105.3931087.6562589.1588051/D=brclubs/S=2137111528:HM/EXP=1111616395/A=2361264/R=6/SIG=10v4acpp0/*http://br.shopping.yahoo.com/>*
>
>
>
> ------------------------------------------------------------------------
> *Links do Yahoo! Grupos*
>
> * Para visitar o site do seu grupo na web, acesse:
> http://br.groups.yahoo.com/group/ciencialist/
>
> * Para sair deste grupo, envie um e-mail para:
> ciencialist-unsubscribe@yahoogrupos.com.br
> <mailto:ciencialist-unsubscribe@yahoogrupos.com.br?subject=Unsubscribe>
>
> * O uso que você faz do Yahoo! Grupos está sujeito aos Termos do
> Serviço do Yahoo! <http://br.yahoo.com/info/utos.html>.
>
>
>
>
> __________ Informação do NOD32 1.1031 (20050321) __________
>
> Esta mensagem foi verificada pelo NOD32 Sistema Antivírus
> http://www.nod32.com.br




SUBJECT: Re: [ciencialist] Dúvidas
FROM: TARCISIO BORGES <tbs97@fisica.ufpr.br>
TO: ciencialist@yahoogrupos.com.br
DATE: 24/03/2005 09:26

Se o nada inclui a ausência de energia, então NÃO. A matéria não pode ser
criada do nada.

As teorias de criação do universo são quase que puramente metafísicas,
dizer que alguma coisa interferiu no nada e gerou o universo também é
metafísica.

Se você não está contente com as teorias atuais sinta-se a vontade para
criar uma, se fizer sentido será tão boa quanto as outras :-) e
possivelmente quase tão impossível de provar que é verdadeira.

[]s
TARCISIO BORGES
tbs97@fisica.ufpr.br

On Thu, 24 Mar 2005, junior_br2001 wrote:
> Tem como provar experimentalmente que matéria pode ser criada do
> nada? Tem como testar isso?
>
> A teoria do Big bang como sendo a *origem do universo* pode ser uma
> teoria científica do ponto de vista de Popper? Pois se não podemos
> testar a origem do universo, o ponto de partida de tudo, como isso
> pode ser considerado científico? Sei que tem algumas evidencias como
> radiação de fundo e expansão do universo, mas nao é disso que estou
> falando, estou falando se podemos testar a origem do universo c/ a
> matéria sendo criada do nada. Isso pode ser reproduzido
> experimentalmente?
>
> JR


SUBJECT: Re: [ciencialist] Santo Sudário - Wikipédia
FROM: Aline Santos <haline_santos@yahoo.com.br>
TO: ciencialist@yahoogrupos.com.br
DATE: 24/03/2005 09:48

Não sei muito bem qual é o assunto em que o Santo Sudário está envolvido pois peguei o barco já andando, mas na minha opinião, não convém para a igreja católica sair por aí dizendo que o Sudário é falso, ou coisa do tipo devido aos seus fiéis. Então, acredito que a igreja enquanto puder dizer que o Sudário é verdadeiro e existirem pessoas que concordem com isso, eles não arredarão o pé. Primeiro que apesar de não ser mais tão implicíto, a igreja ainda possui forte poder perante as ações governamentais e isto pode complicar para o lado científico... Fica aqui a minha opinião.
Abs
Aline


__________________________________________________
Converse com seus amigos em tempo real com o Yahoo! Messenger
http://br.download.yahoo.com/messenger/

[As partes desta mensagem que não continham texto foram removidas]



SUBJECT: folder Regata
FROM: Mario Goncalves <mario.jor@gmail.com>
TO:
DATE: 24/03/2005 10:05

I Regata a Remo em Escaler Almirante Gast�o Motta

S�bado � 02 de abril de 2005 - a partir das 09 horas

SEQU�NCIA DE EVENTOS:
08:30 Chegada das equipes
08:45 Reuni�o preparat�ria
09:15 Cerim�nia de abertura
09:30 Concentra��o das embarca��es
09:40 Largada da prova feminina
09:50 Troca de tripula��es
10:00 Largada da prova veterana


Local: Complexo do Comando do Primeiro Distrito Naval - Pra�a do
Acanto � Centro = Rio de Janeiro � RJ

OBSERVA��O: folder em anexo a este e-mail


[As partes desta mensagem que n�o continham texto foram removidas]



SUBJECT: Fw: K L M N O P Q pq?
FROM: "Luiz Ferraz Netto" <leobarretos@uol.com.br>
TO: "ciencialist" <ciencialist@yahoogrupos.com.br>
DATE: 24/03/2005 10:14

Cuímicos em ação .........

[]'
===========================
Luiz Ferraz Netto [Léo]
leobarretos@uol.com.br
http://www.feiradeciencias.com.br
===========================
-----Mensagem Original-----
De: Luciano
Para: leobarretos@uol.com.br
Enviada em: quinta-feira, 24 de março de 2005 01:36
Assunto: K L M N O P Q pq?


Minha duvida é mais uma curiosidade...
Sobre o Modelo Atomico de Bohr...tem as camadas com os nomes: K L M N O P Q gostaria de saber o motivo dessa sequencia, o por que, que começou do K e não do A...
Sou estudante, e ate agora nenhum professor consiguiu me explicar o por que disso...

[As partes desta mensagem que não continham texto foram removidas]



SUBJECT: Re: [ciencialist] folder Regata
FROM: "E m i l i a n o C h e m e l l o" <chemelloe@yahoo.com.br>
TO: <ciencialist@yahoogrupos.com.br>
DATE: 24/03/2005 10:15

Olha só! Para quem nunca viu, ISSO É UM SPAM. Brudna, Takata et al
moderadores... solicito ações imediatas.

Emiliano Chemello

----- Original Message -----
From: "Mario Goncalves" <mario.jor@gmail.com>
Sent: Thursday, March 24, 2005 10:05 AM
Subject: [ciencialist] folder Regata



I Regata a Remo em Escaler Almirante Gastão Motta

Sábado - 02 de abril de 2005 - a partir das 09 horas

SEQUÊNCIA DE EVENTOS:
08:30 Chegada das equipes
08:45 Reunião preparatória
09:15 Cerimônia de abertura
09:30 Concentração das embarcações
09:40 Largada da prova feminina
09:50 Troca de tripulações
10:00 Largada da prova veterana


Local: Complexo do Comando do Primeiro Distrito Naval - Praça do
Acanto - Centro = Rio de Janeiro - RJ

OBSERVAÇÃO: folder em anexo a este e-mail


[As partes desta mensagem que não continham texto foram removidas]



##### ##### #####

Para saber mais visite
http://www.ciencialist.hpg.ig.com.br


##### ##### ##### #####
Links do Yahoo! Grupos











SUBJECT: Fw: Por favor
FROM: "Luiz Ferraz Netto" <leobarretos@uol.com.br>
TO: "ciencialist" <ciencialist@yahoogrupos.com.br>
DATE: 24/03/2005 10:17

Takata and bio's .......

[]'
===========================
Luiz Ferraz Netto [Léo]
leobarretos@uol.com.br
http://www.feiradeciencias.com.br
===========================
-----Mensagem Original-----
De: "cacauyahoo" <cacauyahoo@ig.com.br>
Assunto: Por favor


Gostaria de saber se todas as células são compostas por bomba de Na e K e
carregadores? Se não quais são?
Grata.
Carline Alves da Silva




SUBJECT: Re: [ciencialist] Fw: K L M N O P Q pq?
FROM: "E m i l i a n o C h e m e l l o" <chemelloe@yahoo.com.br>
TO: <ciencialist@yahoogrupos.com.br>
DATE: 24/03/2005 10:36

Olá Léo,

Veja em: http://cienciahoje.uol.com.br/materia/resources/files/revista-ch/2004/outubro/olp.pdf

[ ] 's do Emiliano Chemello
emiliano@quimica.net
http://www.quimica.net/emiliano
http://www.ucs.br/ccet/defq/naeq
[ MSN ] chemelloe@hotmail.com
[ ICQ ] 145060604

" Rien ne se perd, rien ne se crée,
tout se transforme."

Antoine Laurent de Lavoisier (químico francês, 1743 - 1794)

----- Original Message -----
From: Luiz Ferraz Netto
To: ciencialist
Sent: Thursday, March 24, 2005 10:14 AM
Subject: [ciencialist] Fw: K L M N O P Q pq?


Cuímicos em ação .........

[]'
===========================
Luiz Ferraz Netto [Léo]
leobarretos@uol.com.br
http://www.feiradeciencias.com.br
===========================
-----Mensagem Original-----
De: Luciano
Para: leobarretos@uol.com.br
Enviada em: quinta-feira, 24 de março de 2005 01:36
Assunto: K L M N O P Q pq?


Minha duvida é mais uma curiosidade...
Sobre o Modelo Atomico de Bohr...tem as camadas com os nomes: K L M N O P Q gostaria de saber o motivo dessa sequencia, o por que, que começou do K e não do A...
Sou estudante, e ate agora nenhum professor consiguiu me explicar o por que disso...

[As partes desta mensagem que não continham texto foram removidas]



##### ##### #####

Para saber mais visite
http://www.ciencialist.hpg.ig.com.br


##### ##### ##### #####


Yahoo! Grupos, um serviço oferecido por:

São Paulo Rio de Janeiro Curitiba Porto Alegre Belo Horizonte Brasília




------------------------------------------------------------------------------
Links do Yahoo! Grupos

a.. Para visitar o site do seu grupo na web, acesse:
http://br.groups.yahoo.com/group/ciencialist/

b.. Para sair deste grupo, envie um e-mail para:
ciencialist-unsubscribe@yahoogrupos.com.br

c.. O uso que você faz do Yahoo! Grupos está sujeito aos Termos do Serviço do Yahoo!.



[As partes desta mensagem que não continham texto foram removidas]



SUBJECT: Re: folder Regata
FROM: "Mario Jorge" <mario_dodge@yahoo.com.br>
TO: ciencialist@yahoogrupos.com.br
DATE: 24/03/2005 11:26


Não era intenção magoar ninguem.

Só pretendia divulgar aos interessados.

Peço desculpas, caso tenha quebrado alguma regra.

MARIO

--- Em ciencialist@yahoogrupos.com.br, "E m i l i a n o C h e m e
l l o" <chemelloe@y...> escreveu
> Olha só! Para quem nunca viu, ISSO É UM SPAM. Brudna, Takata et al
> moderadores... solicito ações imediatas.
>
> Emiliano Chemello
>
> ----- Original Message -----
> From: "Mario Goncalves" <mario.jor@g...>
> Sent: Thursday, March 24, 2005 10:05 AM
> Subject: [ciencialist] folder Regata
>
>
>
> I Regata a Remo em Escaler Almirante Gastão Motta
>
> Sábado - 02 de abril de 2005 - a partir das 09 horas
>
> SEQUÊNCIA DE EVENTOS:
> 08:30 Chegada das equipes
> 08:45 Reunião preparatória
> 09:15 Cerimônia de abertura
> 09:30 Concentração das embarcações
> 09:40 Largada da prova feminina
> 09:50 Troca de tripulações
> 10:00 Largada da prova veterana
>
>
> Local: Complexo do Comando do Primeiro Distrito Naval - Praça do
> Acanto - Centro = Rio de Janeiro - RJ
>
> OBSERVAÇÃO: folder em anexo a este e-mail
>
>
> [As partes desta mensagem que não continham texto foram removidas]
>
>
>
> ##### ##### #####
>
> Para saber mais visite
> http://www.ciencialist.hpg.ig.com.br
>
>
> ##### ##### ##### #####
> Links do Yahoo! Grupos





SUBJECT: Re: desculpas
FROM: "E m i l i a n o C h e m e l l o" <chemelloe@yahoo.com.br>
TO: "Mario Jorge" <mario_dodge@yahoo.com.br>
CC: <ciencialist@yahoogrupos.com.br>
DATE: 24/03/2005 11:44

Olá Mario,

Eu também peço desculpas. Pensei que você fosse um Spamer. Mas sugiro
que, caso desejar divulgar algum evento que não esteja relacionado
diretamente a "Ciência", entre em contato com os administradores da lista
para realizar tal ação. Mandei uma cópia desta mensagem para a lista a fim
de esclarecer o mal entendido.

[ ] 's do Emiliano Chemello
emiliano@quimica.net
http://www.quimica.net/emiliano

----- Original Message -----
From: "Mario Jorge" <mario_dodge@yahoo.com.br>
To: <chemelloe@yahoo.com.br>
Sent: Thursday, March 24, 2005 11:25 AM
Subject: desculpas



Não é um spam. Só queria divulgar aos interessados. Caso tenha ferido
qualquer regra, peço desculpas.

Mario






SUBJECT: Re: Dúvidas
FROM: "junior_br2001" <junior_br2001@yahoo.com.br>
TO: ciencialist@yahoogrupos.com.br
DATE: 24/03/2005 13:43


--- Em ciencialist@yahoogrupos.com.br, TARCISIO BORGES <tbs97@f...>
escreveu
> Se o nada inclui a ausência de energia, então NÃO. A matéria não
pode ser
> criada do nada.
>
> As teorias de criação do universo são quase que puramente
metafísicas,
> dizer que alguma coisa interferiu no nada e gerou o universo também
é
> metafísica.
>
> Se você não está contente com as teorias atuais sinta-se a vontade
para
> criar uma, se fizer sentido será tão boa quanto as outras :-) e
> possivelmente quase tão impossível de provar que é verdadeira.


JR: Tarcisio eu não estou contente mesmo. Mas nao tenho intenção de
criar uma. Me afinizo mais com a teoria cíclica que tb inclui o Big-
Bang, mas que não coloca como o ponto de partida de tudo.

Na minha opinião(que não deve valer de nada) para haver um explosão
deveria existir materia pre-existente e obviamente energia. Não dá
para engolir que o nada(o nada mesmo) cria matéria, por mais que os
maiores cientistas do mundo postulem que sim. Mas em todo o caso,
eles estão fazendo o trabalho deles

Valeu
JR





SUBJECT: Re: [ciencialist] Big Bang
FROM: Eduardo Gueron <edgueron@yahoo.com>
TO: ciencialist@yahoogrupos.com.br
DATE: 24/03/2005 14:50


> > Se o nada inclui a ausjncia de energia, entco NCO.
> A matiria nco
> pode ser
> > criada do nada.
> >
> > As teorias de criagco do universo sco quase que
> puramente
> metafmsicas,
> > dizer que alguma coisa interferiu no nada e gerou
> o universo tambim
> i
> > metafmsica.
> >
> > Se vocj nco esta contente com as teorias atuais
> sinta-se a vontade
> para
> > criar uma, se fizer sentido sera tco boa quanto as
> outras :-) e
> > possivelmente quase tco impossmvel de provar que i
> verdadeira.
>
>
> JR: Tarcisio eu nco estou contente mesmo. Mas nao
> tenho intengco de
> criar uma. Me afinizo mais com a teoria cmclica que
> tb inclui o Big-
> Bang, mas que nco coloca como o ponto de partida de
> tudo.
>
> Na minha opinico(que nco deve valer de nada) para
> haver um explosco
> deveria existir materia pre-existente e obviamente
> energia. Nco da
> para engolir que o nada(o nada mesmo) cria matiria,
> por mais que os
> maiores cientistas do mundo postulem que sim. Mas em
> todo o caso,
> eles estco fazendo o trabalho deles
>
> Valeu
> JR

Oi,

Sv para esclarecer. O Big Bang em si, a explosco, i
uma extrapolagco, nco exatamente uma teoria fmsica com
bases experimentais. O que ha de fato: Inzmeras
evidjncias observacionais de que o Universo esta se
expandindo e que ha bilhues de anos era muito pequeno
e muito quente. Das principais evidjncias desta teoria
cito a radiagco cssmica de fundo (temperatura por
volta de 3K) e a distribuigco uniforme de elementos
leves como Litium, Helio 3 e outros... Nisso, a
maioria dos fmsicos acredita e acabam chamando, por
abuso de linguagem, de "teoria do Big Bang". Quanto a
explosco, a origem do Universo, nenhum fmsico sirio
pode afirmar nada que nco sejam especulagues ou
crengas.

Antes de uma determinada ipoca do Universo nco ha
qualquer vestmgio observavel porque a reagco que leva
um fston em um par de partmcula anti-partmcula era
reversmvel e, portanto, a luz emitida ` ipoca nco i
acessmvel.

[]s

Eduardo



__________________________________
Do you Yahoo!?
Yahoo! Small Business - Try our new resources site!
http://smallbusiness.yahoo.com/resources/


SUBJECT: Re: Big Bang
FROM: "junior_br2001" <junior_br2001@yahoo.com.br>
TO: ciencialist@yahoogrupos.com.br
DATE: 24/03/2005 15:07


--- Em ciencialist@yahoogrupos.com.br, Eduardo Gueron <edgueron@y...>
escreveu
> Oi,
>
> Sö para esclarecer. O Big Bang em si, a explosão, é
> uma extrapolação, não exatamente uma teoria física com
> bases experimentais. O que há de fato: Inúmeras
> evidências observacionais de que o Universo está se
> expandindo e que há bilhões de anos era muito pequeno
> e muito quente. Das principais evidências desta teoria
> cito a radiação cósmica de fundo (temperatura por
> volta de 3K) e a distribuição uniforme de elementos
> leves como Litium, Helio 3 e outros... Nisso, a
> maioria dos físicos acredita e acabam chamando, por
> abuso de linguagem, de "teoria do Big Bang". Quanto a
> explosão, a origem do Universo, nenhum físico sério
> pode afirmar nada que não sejam especulações ou
> crenças.
>
> Antes de uma determinada época do Universo não há
> qualquer vestígio observável porque a reação que leva
> um fóton em um par de partícula anti-partícula era
> reversível e, portanto, a luz emitida à época não é
> acessível.



JR: Boa explicação Eduardo. Obrigado





SUBJECT: Artigo apresenta os trabalhos revolucionários de Einstein
FROM: "junior_br2001" <junior_br2001@yahoo.com.br>
TO: ciencialist@yahoogrupos.com.br
DATE: 24/03/2005 15:53


Para o Victor e outros historiadores do ciencialist

http://cienciahoje.uol.com.br/controlPanel/materia/view/3242


PS: Lembrando que no final do texto no site tem o link para o *texto*
completo em .pdf





SUBJECT: Criacionismo
FROM: "Mr. Vain" <mr.vain@gmail.com>
TO: ciencialist@yahoogrupos.com.br
DATE: 24/03/2005 16:51


Parece-me que temos um criacionista disfarçado na lista.

[ ]'s

Dr. Vain
Humanitate Magis
Quam Religione
Nobis Opus Est





SUBJECT: Re: Terremoto em ortugal.
FROM: Maria Natália <grasdic@hotmail.com>
TO: ciencialist@yahoogrupos.com.br
DATE: 24/03/2005 17:01


Só pode partir de um facho mesmo
E chegas atrasado ó velhinho. O governo de direita já era.
Vê lá se te cai o barraco em cima.
Tu agora estás em geofísica ó botequeiro?
Te invadi a choça?
Ai queres?
Espera que já te atendo
Maria Natália

--- Em ciencialist@yahoogrupos.com.br, "Silvio" <scordeiro@t...> escreveu
>
>
> Movimento sísmico em Portugal ...
>
>
> Depois dos problemas acontecidos na Ásia, o governo de Portugal
resolveu instalar um medidor de abalos, que cobre todo o país.
> Então, foi enviado, pelo Centro Sísmico Nacional ao quartel da
polícia da cidade de Vilamoura, no norte de Portugal, um telegrama,
que dizia:
> " Possível movimento sísmico na zona. Ponto. Muito perigoso,
superior Richter 7. Ponto. Epicentro a 3 km do povoado. Ponto. Tomem
medidas. Ponto. Informem resultados com urgência. Ponto."
> Bem, os dias se passaram, e só depois de mais de uma semana é que
foi recebido no Centro Sísmico Nacional um telegrama que dizia:
> " Aqui é do Quartel da Polícia de Vilamoura. Ponto. Movimento
sísmico totalmente desarticulado. Ponto.
>
>
> O tal Ritchter tentou fugir, mas foi abatido a tiros. Ponto,
desativamos as zonas Ponto, as putas estão todas prezas a trabalhar na
lavanderia dos presidios feminino Ponto. Epicentro, Epifânio e três
cupinchas detidos. Ponto.
>
>
> Não respondemos antes, porque aqui houve um terremoto do caralho.
Ponto.
>
> Abraços e beijos sem tremer.
>
>
> [As partes desta mensagem que não continham texto foram removidas]





SUBJECT: Re: Criacionismo
FROM: "junior_br2001" <junior_br2001@yahoo.com.br>
TO: ciencialist@yahoogrupos.com.br
DATE: 24/03/2005 17:14


Não sei até onde aceitar a teoria cíclica sobre a origem do universo,
seja criacionismo.
Eu respondi porque senti que a indireta foi para mim, e porque eu
tava defendendo a possibilidade do Sudário ter 3000 anos.

Mas tem gente que me conhece aqui e pode responder por mim. Sempre
afirmei que gosto das filosofias orientais. Se eu tivesse que ser um
religiosos eu seria um budista, mas nunca um cristão, mesmo porque
acho o cristianismo muito ridículo

Até,
JR


-- Em ciencialist@yahoogrupos.com.br, "Mr. Vain" <mr.vain@g...>
escreveu
>
> Parece-me que temos um criacionista disfarçado na lista.
>
> [ ]'s
>
> Dr. Vain
> Humanitate Magis
> Quam Religione
> Nobis Opus Est





SUBJECT: Re: [ciencialist] Re: Criacionismo
FROM: "Alvaro Augusto \(E\)" <alvaro@electraenergy.com.br>
TO: <ciencialist@yahoogrupos.com.br>
DATE: 24/03/2005 18:46

Caro Junior,

O cristianismo nasceu na Judéia, que ficava bem pra lá do meridiano de Greenwitch. Logo, o cristianismo também é uma filosofia oriental...

[ ]s

Alvaro Augusto

----- Original Message -----
From: junior_br2001
To: ciencialist@yahoogrupos.com.br
Sent: Thursday, March 24, 2005 5:14 PM
Subject: [ciencialist] Re: Criacionismo



Não sei até onde aceitar a teoria cíclica sobre a origem do universo,
seja criacionismo.
Eu respondi porque senti que a indireta foi para mim, e porque eu
tava defendendo a possibilidade do Sudário ter 3000 anos.

Mas tem gente que me conhece aqui e pode responder por mim. Sempre
afirmei que gosto das filosofias orientais. Se eu tivesse que ser um
religiosos eu seria um budista, mas nunca um cristão, mesmo porque
acho o cristianismo muito ridículo

Até,
JR


-- Em ciencialist@yahoogrupos.com.br, "Mr. Vain" <mr.vain@g...>
escreveu
>
> Parece-me que temos um criacionista disfarçado na lista.
>
> [ ]'s
>
> Dr. Vain
> Humanitate Magis
> Quam Religione
> Nobis Opus Est



[As partes desta mensagem que não continham texto foram removidas]



SUBJECT: Outra dúvida(Origem do universo)
FROM: "junior_br2001" <junior_br2001@yahoo.com.br>
TO: ciencialist@yahoogrupos.com.br
DATE: 24/03/2005 18:53


No caso na teoria do Big argumentam-se que existe radiação de fundo e
expansãopar corrobora-la

Um átomo de hidrogenio, feito de um próton e um elétron tem uma
energia de ligação. Ambos podem mover-se independente um do outro. O
que Bethe e outros descobriram é que as estrelas são poderosos
*tranformadores*, capazes de transformar partículas mais leves em
partículas mais pesadas, liberando uma quntidade imensa de energias
durante um processo de fusão. Essa fusão de progressiva de nucleos
mais pesados. No entanto este sistema que gera tanta energia será
fadado mais tarde ao esgotamente de energia.Um estrela se autoconsome
para existir. Enquanto a intensa atração gravitacinal de uma estrela
tende a implosão, a liberação de energia térmica faz com ela tenda a
explodir e parte de seu material se expanda para outros pontos do
universo, emitindo tb radiação

Gente sou super leigo no assunto e quero entender sobre isso que
expus, aguardo alguma luz sobre o assunto. Tem alguma razão o que eu
disse sobre a radiaç~~ao ter vindo da explosão de uma estrela ou sol?

Fico no aguardo

JR







SUBJECT: Re: Criacionismo
FROM: "junior_br2001" <junior_br2001@yahoo.com.br>
TO: ciencialist@yahoogrupos.com.br
DATE: 24/03/2005 18:55


--- Em ciencialist@yahoogrupos.com.br, "Alvaro Augusto \(E\)"
<alvaro@e...> escreveu
> Caro Junior,
>
> O cristianismo nasceu na Judéia, que ficava bem pra lá do meridiano
de Greenwitch. Logo, o cristianismo também é uma filosofia oriental...
>
>


JR: hhaha Alvaro, é mesmo. Mas eu tava me referindo mesmo as dos
extremo oriente como o budismo e o taoismo.Tenho simpatia por elas





SUBJECT: Re: Criacionismo
FROM: "junior_br2001" <junior_br2001@yahoo.com.br>
TO: ciencialist@yahoogrupos.com.br
DATE: 24/03/2005 18:58


Assim como podemos dizer que a ciencia nao é essencialmente grega e
ocidental. Mesmo se tratando que a Grécia foi invadida por
Macedonicos e outros povos do oriente próximo, e muito de seus
pupilos terem estudado e bebido seu saber no Egito, trazendo parte da
matemática e a astronomia para a grécia, e até mesmo principios de um
pensamentio racional

JR

--- Em ciencialist@yahoogrupos.com.br, "Alvaro Augusto \(E\)"
<alvaro@e...> escreveu
> Caro Junior,
>
> O cristianismo nasceu na Judéia, que ficava bem pra lá do meridiano
de Greenwitch. Logo, o cristianismo também é uma filosofia oriental...
>






SUBJECT: Re: [ciencialist] Problema basico de volumes - comentarios
FROM: "Ramon Salvan Fernandes" <ramonsf@pop.com.br>
TO: <ciencialist@yahoogrupos.com.br>
DATE: 24/03/2005 19:03

Saudações a todos

Não sei se este seria o caso, mas a confusão do cunhado do Nuno pode fazer
sentido, pois já vi ocorrer algo semelhante durante um estagio que fiz na
graduação em uma empresa que fabricava frascos plásticos para industrias de
alimentos, cosméticos e limpeza.

Certa vez, tivemos um problema com um cliente que havia sido notificado pela
segunda vez pelo INMETRO porque a quantidade de liquido (amaciante de
roupas) dentro da embalagem era menor que a indicada no rotulo. Já era a
terceira ou quarta vez que este cliente reclamava que as embalagens estavam
com o volume baixo, neste caso, 2 litros. Nos testes realizados na empresa,
o volume total da embalagem era de 2020 ml, neste caso, o 20 ml
corresponde aquele espaço vazio no gargalo. O que eles não sabiam, é que os
materiais plásticos sofrem um fenômeno chamado relaxação, onde as moléculas
(macromoléculas) dos polímeros que são fortemente esticadas durante a
fabricação das embalagens, tende com o tempo voltar a sua forma original,
provocando assim, uma redução de volume que pode chegar a até 2,5%. Como
este fenômeno pode levar dias para se estabilizar, no momento da produção
elas realmente tinham 2020 ml, no entanto, até serem postas na linha de
produção do cliente, o volume se reduzia em cerca de 40 ml.

Outro caso, foi o "aumento" do volume durante o envase de um desinfetante.
Nós produzíamos um frasco de 5 litros com um formato semelhante aqueles
utilizados no armazenamento de combustíveis em veículos (especialmente
jipes), ou seja, um paralelepípedo. No frasco original, eram utilizados
cerda de 120 ml de material para fabrica-lo, mas por uma questão de redução
de custo exigido pelo cliente, seu peso foi reduzido para menos de 90g.
Consequentemente, as paredes deste frasco tornaram-se mas finas e flexíveis,
assim, quando este era cheio com o liquido, a embalagem deformava-se,
tornando-se mais "esférica", e portanto com um volume maior. No caso do
cunhado do Nuno, o problema pode ser que a embalagem realmente esteja com
um volume menor, que quando é testada com água acaba sendo mascarado devido
a sua alta densidade, quando comparada a do shampoo.

Sei que estes casos não solucionam o problema básico, que é a noção de
volume, mas pode explicar o porquê da insistência do nosso amigo em que a
água ocupa um volume maior que o shampoo, afinal, ele "comprovou"
pessoalmente esse fenômeno.


Abraço a todos

Ramon

----- Original Message -----
From: "Luiz Ferraz Netto" <leobarretos@uol.com.br>
To: <ciencialist@yahoogrupos.com.br>
Sent: Thursday, March 24, 2005 7:14 AM
Subject: [ciencialist] Problema basico de volumes - comentarios



Problema original apresentado por Nuno Figueiredo em 22/03/05:
===================================================
> Prezado professor:
> Achei o seu site muito interessante. Assim, estou a lhe escrever um
> função de uma duvida.
> O meu cunhado tem uma fabrica de cosméticos e, surgiu uma discussão
> entre nós: Ele insiste, que em determinado vasilhame que tem a
> capacidade para 4 litros, recebe 4 litros de água, mas uma quantidade
> diferente (menor) de shampoo.
> Por mais que eu tentasse explicar que o litro é uma unidade
> universal(não varia com o tipo de produto) por se tratar de uma medida
> de volume, ele insiste no erro.
> Eu expliquei-lhe que o peso sim, varia conforme o produto por causa da
> densidade, mas o volume jamais muda. Ele aceitou que o peso varia, mas
> o volume também.
> Eu argumentei entre outras, que não poderíamos ter duas variáveis para
> o mesmo produto, pois tornar-se-ia impossível calcular uma em função
> da outra... Tentei explicar que um litro de água ou um litro de chumbo
> derretido são um litro. Apenas um pesa aproximadamente um Kg e o outro
> vários Kg.
> Ele insiste que está certo e que já foi autoado pelo inmetro mais de
> uma vês por isso. Eu, irritado disse que não era possível e, que se o
> inmetro multou porque um litro não era um litro, o inmetro estava
> errado. É obvio que ele foi autoado por diferenças no calculo do peso
> liquido e não no volume em l ou ml.
> Bem, resumindo, fui incapaz de lhe explicar isso, apesar da minha
> formação superior em física (incompleta).
> Achei que o tema é interessante o suficiente para o expor ao senhor. Se
> não for pedir demais, gostaria de uma opinião para a apresentar.
> Talvez o problema seja apresentar de forma compreensível a definição
> de litro. Sem se levar em consideração os gases por serem fluidos
> compressíveis, e claro.
> Agradeço de antemão a sua resposta,
==========================================================
Primeira proposta de resposta, por S. Taborda, em 22/03/05:
==========================================================
O volume de um recipiente (vasilhame) é constante. O recipiente suporta
o mesmo volume de qualquer substancia. Parece que é isto que o seu
cunhado falha em entender. O volume do recipiente não depende do liquido
que ele contém, pois o volume é sempre o mesmo, mesmo que o recipiente
estiver vazio.
O volume do recipiente é calculado geometricamente. Existem formulas
matemáticas para o calcular para recipientes simples. Por exemplo, um
cilindro tem um volume igual ao produto da altura pela área da base.
Esta formula é a base dos medidores de volume. Que são normalmente
cilindros transparentes com escalas do lado de fora.
Se a forma do recipiente não é simples de corresponder com sólidos
geométricos simples - como cilindros, cubos e esfera - então existe um
truque, que é encher o recipiente com água e despejar essa água num
recipiente do qual sabemos o volume. (se possível um recipiente calibrado).
Sendo que o volume do recipiente depende apenas da forma do recipiente e
não do que ele contém, o seu volume é constante e não varia conforme o
liquido que se coloca nele.
=================================================================
Comentário do Oráculo, postado em 22/03/05:
=================================================================
Olá Luiz
Acho que o enfoque do Taborda será a única forma de convencer cunhado em
questão..:-) Se ele puder perceber que o volume do recipiente, mesmo vazio
(e mesmo no vácuo) ainda será de um litro, talvez perceba a incongruência de
afirmar que tem um litro de água e menos de shampoo. Me parece que, apenas
com cálculos e definições, será mais difícil, já que deve faltar a base de
conhecimento necessária para absorver esse tipo de evidencia ou argumento.

Talvez exista também um problema extra, que é a verificação que organismos
de controle fazem com produtos destinados ao consumo público. A água, ao
preencher o recipiente, ocupará todo o volume do mesmo. Mas outros tipos de
produto podem deixar espaços ou mesmo conter ar, como no caso de shampoo, ao
ser inserido com maquinas de pressão automáticas.

É possível que os problemas do cunhado com o instituto de medição e controle
se deva a metodologia, por exemplo, processar o shampoo contido na embalagem
de forma a retirar todo o ar (bolhas) e só então verificar o volume. Em
alguns casos, principalmente com aparelhos de preenchimento da industria
descalibrados ou fora de ajuste, as bolhas podem representar um volume
significativo, e causar a multa do fabricante.

Isso pode ter sido interpretado pelo cunhado como sendo uma diferença real
de volume entre água e o shampoo na embalagem.

A abordagem do Taborda pode faze-lo compreender que um litro de água e um
litro de shampoo contém ambos o mesmo volume, mas de qualquer coisa que
estiver dentro do recipiente, seja apenas água, apenas shampoo ou shampoo e
ar (em bolhas ou diluído) e que foi isso que causou seus problemas com a
medição.
==============================================================
Segunda proposta de resposta, postado por Léo, redação final, em 24/03/05:
==============================================================
Olá José Nuno,
A "falha" de seu cunhado talvez possa ser sanada se vc apresentar a ele
definições e conceitos rigorosamente científicos. Devemos usar os
termos/palavras corretamente e não tentar extrapolar tais termos para o uso
popular. Serve de exemplo o "trabalho", um conceito físico definido para
forças, mas é, sem dúvida, uma palavra que leva em si um forte componente
psicológico. É bem possível que seja necessário um esforço maior para
disputar uma partida de tênis do que para colocar em ordem alfabética um
conjunto de fichas de arquivo, mas nós consideramos a última atividade como
um 'trabalho' e a primeira como 'divertimento', 'competição' ou 'exercício',
mas nunca como trabalho!. O 'trabalho' da física, pouco ou nada tem a ver
com o 'trabalho' da telefonista, do ascensorista etc.

Todavia, devo advertir de imediato: seu cunhado pode não estar falhando!

Então, a primeira coisa a ser lapidada, é o conceito de "volume" e suas
unidades (oficial e legal). "Volume" é o resultado da medição de um
determinado 'espaço' (propriedade geométrica de apresentar três dimensões).
Esse 'espaço' pode ser, por exemplo, aquele limitado pelas 4 paredes, teto e
chão de uma simples sala. Para a medição desse espaço torna-se necessário
que se defina um 'espaço unitário' ao qual se associe a 'unidade de volume';
um cubo de 1 m de aresta limita um espaço que tem, por definição, o volume
unitário, ou seja, 1 metro cúbico.
Além do 'metro cúbico', que é a unidade oficial de volume, aceita-se,
legalmente, a unidade denominada "litro", que corresponde à milésima parte
do metro cúbico, ou 1 metro cúbico = 1 000 litros --- há outra definição
para o litro; não vem ao caso.

Certos depósitos/vasilhames têm capacidade de armazenamento de líquidos, são
assim os tanques de gasolina dos postos, os bujões de água, as caixas
d'águas domiciliares, as garrafas PETs etc. Essas capacidades de
armazenamento serão dadas pelas medidas dos espaços (essas medidas serão os
volumes) disponíveis nesses depósitos/vasilhames. Tem garrafa PET, por
exemplo, cuja capacidade de armazenamento (isso é um espaço geométrico) é de
2 litros (isso é um volume). Tem lata de mantimento que disponibiliza um
volume útil de, exato, 1 litro. Qualquer coisa que vc coloque lá dentro e
preencha totalmente o espaço disponível ocupará "por definição" o volume de
1 litro.

Mas, isso não significa que lá dentro exista "1 litro" daquele material!

Vou exemplificar:
Aqui na minha cidade vendem-se jabuticabas soltas nas feiras livres; a
'unidade' de comercialização é "uma lata de capacidade 1 litro". Qdo vc pede
2 litros de jabuticabas o feirante enche duas vezes aquela lata --- eis "2
litros de jabuticaba".
Você concordará que ali não há "realmente" 2 litros de jabuticaba, uma vez
que entre elas, dentro da lata, tem enorme quantidade de espaço vazio. Seu
cunhado tem, sobre o shampoo, a mesma idéia que vc está tendo agora do
'litro de jabuticaba. O 'litro de shampoo' não tem realmente "1 litro de
shampoo"!!!!!!

Um shampoo é um produto cujo fórmula e fabricação implica num resultado
pouco denso devido à minúsculas gotículas de ar (gases) e isso o coloca em
similaridade com a jabuticaba.

Como resolver o problema?

Numa solução 'ideal'o shampoo deveria ser 'envazado' no vácuo (para
eliminação das bolhas de ar)--- obviamente teríamos um shampoo mais denso
que aquele normalmente vendido por ai. Esse seria o shampoo 'padrão' ---
todos os demais shampoos teriam um adjetivo extra: shampoo a 10%, shampoo 30
... ou sei lá o que.

Então, o dilema deixa de ser dilema entendendo-se que:

(a) "volume" é a especificação de um espaço; é um invariante, independente
do que está preenchendo esse espaço. Assim, quer o espaço contenha água,
shampoo ou jabuticabas --- o volume é de 1 litro (no exemplo).

(b) "volume" de substâncias porosas é um 'volume aparente'; não há realmente
o 'tanto' de substância que se espera. Para tais casos, "apenas" volume, não
é grandeza suficiente para especificação da substância.

PS: Vc percebe o erro conceitual ao dizer : Vamos medir o volume dessa
caixa? Volume já não é uma medida?

Aquele abraço,
===============================================================
Pronto .... fica registrado o pedido do Nuno e as propostas. Fico no aguardo
das sugestões finais para poder enviar o definitivo ao Nuno.

Aquele abraço a todos que, permanentemente, têm contribuído para o
assentamento da Ciência na cuca de nossos jovens e 'outros'.

aquele abraço,

===========================
Luiz Ferraz Netto [Léo]
leobarretos@uol.com.br
http://www.feiradeciencias.com.br
===========================


##### ##### #####

Para saber mais visite
http://www.ciencialist.hpg.ig.com.br


##### ##### ##### #####
Links do Yahoo! Grupos












SUBJECT: Re: Criacionismo
FROM: "junior_br2001" <junior_br2001@yahoo.com.br>
TO: ciencialist@yahoogrupos.com.br
DATE: 24/03/2005 21:08


--- Em ciencialist@yahoogrupos.com.br, JVictor <jvoneto@u...> escreveu
> Só queria saber uma coisa, liás, two:
>
> se ao invés de Barrabás, os mui amigos judeus tivessem livrado
Jesus da
> crucificação:
>
> 1-como seria a Igreja e seus seguidores, hoje?
>
> 2 - e nós, seríamos cristãos ou o quê?
>
> Por nada não, por nada não...


JR: Acho que nem seríamos cristãos, porque o termo cristão não foi
nem criado por Jesus, aliás ele não criou religião nem nenhuma seita,
apewar de ter feita uma referencia sobre uma Igreja que nao tinha
nada haver com poder, ou igreja nos moldes católicos ou evangélicos.

Bem, mas se barrabás fosse crucificado certamente existiria uma seita
chamada barrabãos. E talvez eu seria uma barrabão, tomando vinho o
dia inteiro e pegando as mulheres mais bonitas que estivesse por lá,
além é claro de conspirar contra Roma

JR





SUBJECT: Re: [ciencialist] Re: ninguem é inocente.
FROM: "Silvio" <scordeiro@terra.com.br>
TO: <ciencialist@yahoogrupos.com.br>
DATE: 24/03/2005 21:51

Júnior:

Exatamente.
Excluindo alguns papeis religiosos como os da Índia e as epopéias Olímpicas
pelos primeiros poetas gregos, creio que foi Herôdotos o autor da primeira
obra em prosa da literatura grega preservada até nossos dias. Ele nasceu em
484 a.c. No livro "História" narra suas viagens pelo mundo então dominado
pelos gregos. A passagem da tomada do trono Persa por Dario, o que modificou
toda a história moderna, citada no livro III números 80-88 daquele livro é
um clássico exemplo para os políticos e deve ter influenciado Maquiavel.

Raros personagens da história que viveram a Idade Média escreveram
biografia: apenas após o iluminismo (Renascimento) os registros começaram a
ser mais claros. Era tudo "por ouvir dizer" ou presenciado pelo escritor que
colore o tema com suas próprias tintas, se me entende.

Sua pergunta tá mal colocada pois os personagens realmente existiram. apenas
o relato de suas vidas é que precisam ser lidos com extremo cuidado
observando a fonte.

Sds.,
silvio.

Nota: Conto com seu voto para 2006: SEVERINO PARA PRESIDENTE.PAULO COELHO,
VICE.

pubmed pergunta:

Silvio, segundo seu raciocínio então nenhum personagem histórico
existiu então.
Tudo o que sabemos dos povos antigos foram transmitidos por outros e
não por eles mesmos.

JR

--- Em ciencialist@yahoogrupos.com.br, "Silvio" <scordeiro@t...>
escreveu
> Júnior!!!!!
>
> Curiosamente, nenhum dos "iluminados" ( Buda, J. Cristo,) deixou
qualquer
> vestígio de sua passagem por esse vale de lágrimas. Buda pode até
ter
> falado: mas quem, em sã consciência,quem pode repetir ipsum litera
suas
> palavras que rolam em tradições e traduções monásticas há 2.000
anos?
>
> Não vá no papo dessa turma, prefira ler Freud. Vamos lutar pela
canonização
> do Paulo Coelho.
>
> sds.,
>
> silvio



SUBJECT: Dúvida
FROM: Daniela Yoshikawa <danieleakemi@yahoo.com.br>
TO: ciencialist@yahoogrupos.com.br
DATE: 24/03/2005 21:57

Olá!

Alguém poderia me explicar com mais detalhes a 1ª Lei de Newton?

Obrigada.



__________________________________________________
Converse com seus amigos em tempo real com o Yahoo! Messenger
http://br.download.yahoo.com/messenger/

[As partes desta mensagem que não continham texto foram removidas]



SUBJECT: Re: [ciencialist] Re: Criacionismo
FROM: "Alvaro Augusto \(E\)" <alvaro@electraenergy.com.br>
TO: <ciencialist@yahoogrupos.com.br>
DATE: 24/03/2005 22:06

Se Barrabás tivesse sido crucificado, provavelmente nenhuma religião teria surgido disso. Barrabás era apenas um sicário, mais precisamente um zelote, uma espécie de terrorista judeu, cuja luta era meramente terrestre e se resumia a expulsar os romanos e reestabelecer a suposta glória do reino de Salomão. Como ele, havia centenas, muitos dos quais foram crucificados.

Mas há um conto fascinante, de Poul Anderson, chamado "Esperando os Olimpianos", no qual Jesus não foi crucificado, a igreja católica nunca existiu e o império romano dura até os dias de hoje...

[ ]s

Alvaro Augusto

----- Original Message -----
From: JVictor
To: ciencialist@yahoogrupos.com.br
Sent: Friday, March 25, 2005 9:06 PM
Subject: Re: [ciencialist] Re: Criacionismo


Só queria saber uma coisa, liás, two:

se ao invés de Barrabás, os mui amigos judeus tivessem livrado Jesus da
crucificação:

1-como seria a Igreja e seus seguidores, hoje?

2 - e nós, seríamos cristãos ou o quê?

Por nada não, por nada não...

Sds,

Victor.






Alvaro Augusto (E) escreveu:

> Caro Junior,
>
> O cristianismo nasceu na Judéia, que ficava bem pra lá do meridiano de
> Greenwitch. Logo, o cristianismo também é uma filosofia oriental...
>
> [ ]s
>
> Alvaro Augusto
>
> ----- Original Message -----
> From: junior_br2001
> To: ciencialist@yahoogrupos.com.br
> Sent: Thursday, March 24, 2005 5:14 PM
> Subject: [ciencialist] Re: Criacionismo
>
>
>
> Não sei até onde aceitar a teoria cíclica sobre a origem do universo,
> seja criacionismo.
> Eu respondi porque senti que a indireta foi para mim, e porque eu
> tava defendendo a possibilidade do Sudário ter 3000 anos.
>
> Mas tem gente que me conhece aqui e pode responder por mim. Sempre
> afirmei que gosto das filosofias orientais. Se eu tivesse que ser um
> religiosos eu seria um budista, mas nunca um cristão, mesmo porque
> acho o cristianismo muito ridículo
>
> Até,
> JR
>
>
> -- Em ciencialist@yahoogrupos.com.br, "Mr. Vain" <mr.vain@g...>
> escreveu
> >
> > Parece-me que temos um criacionista disfarçado na lista.
> >
> > [ ]'s
> >
> > Dr. Vain
> > Humanitate Magis
> > Quam Religione
> > Nobis Opus Est


[As partes desta mensagem que não continham texto foram removidas]



SUBJECT: Re: Criacionismo
FROM: "junior_br2001" <junior_br2001@yahoo.com.br>
TO: ciencialist@yahoogrupos.com.br
DATE: 24/03/2005 22:32


Álvaro, mesmo se não existisse a religião alguma outra forma de
ideologia política e imperial iria dominar as massas e deter o poder.
Quanto ao referido livro é do estilo de O Código da Vinci?

Até,
JR

--- Em ciencialist@yahoogrupos.com.br, "Alvaro Augusto \(E\)"
<alvaro@e...> escreveu
> Se Barrabás tivesse sido crucificado, provavelmente nenhuma
religião teria surgido disso. Barrabás era apenas um sicário, mais
precisamente um zelote, uma espécie de terrorista judeu, cuja luta
era meramente terrestre e se resumia a expulsar os romanos e
reestabelecer a suposta glória do reino de Salomão. Como ele, havia
centenas, muitos dos quais foram crucificados.
>
> Mas há um conto fascinante, de Poul Anderson, chamado "Esperando os
Olimpianos", no qual Jesus não foi crucificado, a igreja católica
nunca existiu e o império romano dura até os dias de hoje...
>
> [ ]s
>
> Alvaro Augusto
>
> ----- Original Message -----
> From: JVictor
> To: ciencialist@yahoogrupos.com.br
> Sent: Friday, March 25, 2005 9:06 PM
> Subject: Re: [ciencialist] Re: Criacionismo
>
>
> Só queria saber uma coisa, liás, two:
>
> se ao invés de Barrabás, os mui amigos judeus tivessem livrado
Jesus da
> crucificação:
>
> 1-como seria a Igreja e seus seguidores, hoje?
>
> 2 - e nós, seríamos cristãos ou o quê?
>
> Por nada não, por nada não...
>
> Sds,
>
> Victor.
>
>
>
>
>
>
> Alvaro Augusto (E) escreveu:
>
> > Caro Junior,
> >
> > O cristianismo nasceu na Judéia, que ficava bem pra lá do
meridiano de
> > Greenwitch. Logo, o cristianismo também é uma filosofia
oriental...
> >
> > [ ]s
> >
> > Alvaro Augusto
> >
> > ----- Original Message -----
> > From: junior_br2001
> > To: ciencialist@yahoogrupos.com.br
> > Sent: Thursday, March 24, 2005 5:14 PM
> > Subject: [ciencialist] Re: Criacionismo
> >
> >
> >
> > Não sei até onde aceitar a teoria cíclica sobre a origem do
universo,
> > seja criacionismo.
> > Eu respondi porque senti que a indireta foi para mim, e
porque eu
> > tava defendendo a possibilidade do Sudário ter 3000 anos.
> >
> > Mas tem gente que me conhece aqui e pode responder por mim.
Sempre
> > afirmei que gosto das filosofias orientais. Se eu tivesse que
ser um
> > religiosos eu seria um budista, mas nunca um cristão, mesmo
porque
> > acho o cristianismo muito ridículo
> >
> > Até,
> > JR
> >
> >
> > -- Em ciencialist@yahoogrupos.com.br, "Mr. Vain"
<mr.vain@g...>
> > escreveu
> > >
> > > Parece-me que temos um criacionista disfarçado na lista.
> > >
> > > [ ]'s
> > >
> > > Dr. Vain
> > > Humanitate Magis
> > > Quam Religione
> > > Nobis Opus Est
>
>
> [As partes desta mensagem que não continham texto foram removidas]





SUBJECT: Fw: Capitalismo
FROM: "Silvio" <scordeiro@terra.com.br>
TO: "(REDELIBERAL3" <redeliberal3@yahoogrupos.com.br>, <ciencialist@yahoogrupos.com.br>, <acropolis@yahoogrupos.com.br>
DATE: 24/03/2005 23:09




CAPITALISMO IDEAL
Você tem duas vacas.

Vende uma e compra um touro.

Eles se multiplicam, e a economia cresce.

Você vende o rebanho e aposenta-se, rico!

CAPITALISMO AMERICANO

Você tem duas vacas.

Vende uma e força a outra a produzir o leite de quatro vacas.

Fica surpreso quando ela morre.

CAPITALISMO JAPONÊS

Você tem duas vacas.

Redesenha-as para que tenham um décimo do tamanho de uma vaca normal e produzam 20 vezes mais leite.

Depois cria desenhinhos de vacas chamados Vaquimon e os vende para o mundo inteiro.

CAPITALISMO BRITÂNICO

Você tem duas vacas.

As duas são loucas.

CAPITALISMO HOLANDÊS

Você tem duas vacas.

Elas vivem juntas, não gostam de touros e tudo bem.

CAPITALISMO ALEMÃO

Você tem duas vacas.

Elas produzem leite regularmente, segundo padrões de quantidade e horário previamente estabelecido, de forma precisa e lucrativa.

Mas o que você queria mesmo era criar porcos.

CAPITALISMO RUSSO

Você tem duas vacas.

Conta-as e vê que tem cinco.

Conta de novo e vê que tem 42.

Conta de novo e vê que tem 12 vacas.

Você pára de contar e abre outra garrafa de vodca.

CAPITALISMO SUÍÇO

Você tem 500 vacas, mas nenhuma é sua.

Você cobra para guardar a vaca dos outros.

CAPITALISMO ESPANHOL

Você tem muito orgulho de ter duas vacas.

CAPITALISMO PORTUGUÊS

Você tem duas vacas.

E reclama porque seu rebanho não cresce...

CAPITALISMO HINDU

Você tem duas vacas.

Ai de quem tocar nelas.

CAPITALISMO ARGENTINO

Você tem duas vacas.

Você se esforça para ensinar as vacas a mugirem em inglês... As vacas morrem. Você entrega a carne delas para o churrasco de fim de ano ao FMI.

CAPITALISMO BRASILEIRO

Você tem duas vacas.

Uma delas é roubada.

O governo cria a CCPV- Contribuição Compulsória pela Posse de Vaca.

Um fiscal vem e autua você, porque embora você tenha recolhido corretamente a CCPV, o valor era pelo número de vacas presumidas e não pelo de vacas reais. A Receita Federal, por meio de dados também presumidos do seu consumo de leite, queijo, sapatos de couro, botões, presume que você tenha 200 vacas e para se livrar da encrenca, você dá a vaca restante para o fiscal deixar por isso mesmo.




--------------------------------------------------------------------------------




--------------------------------------------------------------------------------


No virus found in this outgoing message.
Checked by AVG Anti-Virus.
Version: 7.0.308 / Virus Database: 266.8.1 - Release Date: 23/3/2005


[As partes desta mensagem que não continham texto foram removidas]



SUBJECT: Leis de newton (p/ Daniela)
FROM: "junior_br2001" <junior_br2001@yahoo.com.br>
TO: ciencialist@yahoogrupos.com.br
DATE: 24/03/2005 23:25


Outros podem complementar com uma explicação, mas voce pode encontrar
algo bem simples neste site

http://luisperna.com.sapo.pt/leis_newton.htm

JR


--- Em ciencialist@yahoogrupos.com.br, Daniela Yoshikawa
<danieleakemi@y...> escreveu
> Olá!
>
> Alguém poderia me explicar com mais detalhes a 1ª Lei de Newton?
>
> Obrigada.
>





SUBJECT: Re: Eter Luminifero (era: ninguem é inocente.)
FROM: "marcelomjr" <marcelomjr@hotmail.com>
TO: ciencialist@yahoogrupos.com.br
DATE: 24/03/2005 23:32


Vitor,

Como eu disse antes, nem todos os cientistas do mundo crêem na
inexistência do Ether, e Einstein certamente estava entre estes. Da
mesma maneira que Hendrik Antoon Lorentz (vide "Lorentz Ether
Theory"), Einstein nunca abandonou completamente a noção básica de um
*meio (medium) físico permeando o Universo inteiro, dotando o Espaço
de qualidades físicas mensuráveis*. Einstein prontamente reconheceu
isto quando, de uma reunião de 5 de Maio de 1920, em homenagem a
Lorentz, na "University of Leyden", ele discursou (enfases minhas):


"(...) we may say that according to the general theory of relativity
SPACE IS ENDOWED WITH PHYSICAL QUALITIES; in this sense, therefore,
THERE EXISTS AN ETHER. According to the general theory of relativity
SPACE WITHOUT ETHER IS UNTHINKABLE; for in such space there not only
WOULD BE NO PROPAGATION OF LIGHT, but also no possibility of
existence for standards of space and time (measuring-rods and
clocks), nor therefore any space-time intervals in the physical
sense. But this ether may not be thought of as endowed with the
quality characteristic of ponderable media, as consisting of parts
which may be tracked through time. The idea of motion may not be
applied to it."

(Ether and the Theory of Relativity. Albert Einstein, May 5th, 1920,
University of Leyden <http://www.tu-harburg.de/rzt/rzt/it/Ether.html>)

(nota: contrariamente as palavras de Einstein e ao modelo etérico
estacionário predominante em fins do séc. XIX, a idéia de movimento
pode ser, sim, aplicada ao Ether; isto é: um Ether Dinâmico.)

(nota: na época do experimento de Michelson-Morley, o Ether era
geralmente considerado estacionário e distinto dos corpos materiais -
embora coexistisse com estes. Ainda não se cogitava de que a própria
matéria poderia ser feita de Ether. O Ether era o meio através do
qual os corpos materiais, como a Terra, moviam-se.)


Semelhantemente, num artigo de Abril de 1950, da revista "Scientific
American" (Volume 182, No. 4) Einstein revela sua real posição em
relação à hipótese de um meio físico - ou Ether - preenchendo o
Espaço nos seguintes termos (enfase minha):

"According to general relativity, THE CONCEPT OF SPACE DETACHED FROM
ANY PHYSICAL CONTENT DOES NOT EXIST. The physical reality of space is
represented by a field whose components are continuous functions of
four independent variables - the coordinates of space and time."

(nota: portanto, segundo Einstein, o Espaço universal não é, nunca
foi, nem jamais poderia ser um ente vazio, um vácuo [vacuum], ou um
nada, ao contrário do que é irrefletida e freqüentemente propalado
aos quatro ventos. Existiria, isto sim, um meio [medium] físico
permeando o Universo inteiro. "A Natureza tem aversão ao Vácuo" é um
princípio conhecido.)

Além do mais, e não menos importante, em seu segundo princípio da
Teoria Especial da Relatividade (O Princípio da Constância da
Velocidade da Luz), Einstein declara que: "the velocity of light is a
constant independent of its source and of the Galilean system with
respect to which it is measured." O princípio da constância da
velocidade da Luz no Espaço parece estar mais do que cientificamente
estabelecido atualmente. Agora, e curiosamente, esta característica
da Luz de ter velocidade constante independentemente de sua fonte ou
de qualquer observador em qualquer sistema de referência, também é
EXATAMENTE aquela de uma onda movendo-se através de um *meio*, muito
embora esta característica da Luz não seja expressa desta forma (em
relação às ondas) na Teoria Especial da Relatividade. Ou seja, isto é
equivalente a dizer que a Luz é uma forma de onda, porque *"somente
ondas" possuem uma velocidade constante independentemente de sua
fonte ou de qualquer observador em qualquer sistema de referência*.
De qualquer modo, a teoria corpuscular da Luz, que se saiba, não
corresponde a este quesito/princípio do segundo postulado da
Relatividade Especial, ao contrário da teoria ondulatória para a Luz
num meio [Ether]. É interessante observar que, na Natureza, as
partículas em geral não apresentam propriedades de onda, mas pode-se
encontrar ondas com atributos de partículas... O único exemplo
conhecido de partículas apresentando simultaneamente propriedade de
ondas existe tão-somente na mente dos físicos quânticos.

Para arrematar: de acordo com as notas autobiográficas de Einstein
aos 67 anos de idade (Volume I de "Albert Einstein, Philosopher –
Scientist", editado por Paul Arthur Schilpp) parece que Einstein foi
influenciado pela teoria do Ether de James Clerk Maxwell (as equações
de Maxwell foram pensadas somente para expressar o eletromagnetismo
propagando-se no Ether) para desenvolver sua Teoria Especial da
Relatividade:


"The special theory of relativity owes its origin to Maxwell's
equations for the electromagnetic field. Inversely the latter can be
grasped formally in satisfactory fashion only by way of the special
theory of relativity. Maxwell's equations are the simplest Lorentz-
invariant field equations which can be postulated for an anti-
symmetric tensor derived from a vector field. This in itself would be
satisfactory, if we did not know from quantum phenomena that
Maxwell's theory does not do justice to the energetic properties of
radiation. But how Maxwell's theory would have to be modified in a
natural fashion, for this even the special theory of relativity
offers no adequate foothold..."

E não apenas isto: ainda com base na leitura destas notas de Einstein
há quem afirme categoricamente que Einstein - desde antes da
publicação de suas teorias até o fim de sua vida física - estava na
verdade trabalhando na hipótese de um meio (Ether) no qual a luz se
propagaria na forma de onda, usando da Teoria Tensorial para analisar
as propriedades físicas (tensão, compressão) desse meio universal. Em
outras palavras, Einstein estava, desde o início, trabalhando em um
modo de explicar a luz como sendo uma forma de onda movendo-se
através de um meio (Ether) que existiria por todo o Universo. Ao que
parece, Einstein nunca acreditou que a luz era transmitida por
partículas ou que a luz era uma dualidade. Quem leu estas notas
revela que há muitas outras porções destas notas em que Einstein faz
observações semelhantes. Seja como for, isto ainda é discutível e só
nos resta ler por nós mesmos.

http://www.softcom.net/users/greebo/price.htm#main


Um abraço,
Marcelo M. Jr.
"The important thing is not to stop questioning. Curiosity has its
own reason for existing. One cannot help but be in awe when he
contemplates the mysteries of eternity, of life, of the marvelous
structure of reality. It is enough if one tries merely to comprehend
a little of this mystery every day. Never lose a holy curiosity."
(Albert Einstein, May 2, 1955.)


--- Em ciencialist@yahoogrupos.com.br, JVictor <jvoneto@u...> escreveu
>
> Marcelo,
>
> Para a TRE a existência do éter nem fede nem cheira. Einstein não
> descartou o éter. Ele apenas mostrou que o éter é supérfluo, não
influe
> em sua teoria, em vista de sua estrutura conceitual.. Creio que o
autor
> se refere ao éter inventado por Descartes e assumido pelos que
vieram
> depois, para servir de suporte às ondas eletromagnéticas. Veja, por
> exemplo, como o próprio Lorentz se refere ao Éter! E sua teoria
> eletromagnética é totalmente baseada nisso!(Aliás, Lorentz morreu e
não
> se afastou um pingo dos conceitos etéricos). Esse éter milagroso,
> rígido, imponderável, indetectável, que penetra tudo, e está em
tudo e
> em em todos, e tem poderes milagrosos, simplesmente pega a 3a. lei
de
> Newton e créu nela!
> Esse éter não existe mesmo. Agora(bem, ainda vou ler, se tiver
tempo,
> essas "descobertas"), existem outras coisas, expostas pela física
> quântica, que permeiam e preenchem do espaço. Creio estar havendo
alguma
> confusão.
> Mas, vou procurar saber do que se trata realmente. Particularmente,
não
> tenho mais dúvidas de que o éter a que me referi acima não existe.
>
> Sds,
>
> Victor.
>
> marcelomjr escreveu:
>
> >
> > Olá,
> >
> > Já que citaram a questao da (suposta) inexistência do Ether (ou
> > aether)de forma tão conclusiva e dogmatica, gostaria apenas
relembrar
> > que nem todos os cientistas do mundo pensam assim e/ou creem que
foi
> > dada a ultima palavra sobre este assunto. Muito pelo contrario, um
> > pouco mais de pesquisa sobre este tema na rede mundial de
> > computadores, mostrará que a questão da existencia ou da
inexistencia
> > de um Ether ainda está longe (e muito longe) de acabar na
comunidade
> > cientifica (ao contrário do que pessoas mal informadas tem
promulgado
> > abundantemente aos leigos por aí), aliás como sugere o excerto
> > abaixo, retirado do site da "Natural Philosophy Alliance (NPA)"
> > [http://mywebpages.comcast.net/Deneb/]:
> > <http://mywebpages.comcast.net/Deneb/%5D:>
> >
> > http://mywebpages.comcast.net/Deneb/Steps.htm
> > "(...)
> > 7. TAKE THE SAGNAC EXPERIMENT SERIOUSLY. In this case,
the "infrared
> > film" needed was provided by Sagnac in 1913, when he looked for
the
> > aether with an interferometer that rotated, instead of
translating in
> > a near-straight line. Something caused his fringes to shift as
viewed
> > on the rotating platform, and these shifts meant that the
velocity of
> > light was remaining constant relative to the laboratory. Sagnac
> > advanced this as experimental proof against the second postulate
of
> > SR, which it actually was. His method has been modified and
repeated
> > many times since his day, and currently is being tested constantly
> > among the satellites of the Global Positioning System (GPS). Every
> > single time, when rotation of a light path within a surrounding
> > dominant coordinate system occurs, fringes are shifted, light
> > velocities are altered, and the existence of a luminiferous
aether is
> > strongly inferred--all contrary to SR.
> >
> > Establishment physicists have usually ignored the Sagnac effect,
or
> > once in a while they have attempted to explain it in terms of
special
> > or general relativity--but all of these attempts have fallen
short.
> > (...)"
> >
> >
> > Um abraço,
> > Marcelo M. Jr.
> >
> > --- Em ciencialist@yahoogrupos.com.br, "Oraculo" <oraculo@a...>
> > escreveu
> > > Olá Pubmed
> > >
> > > Pubmed: Naõ começa tropeçar nos seus proprios argumentos, o Éter
> > luminífero é
> > > um mito que foi derrubado...uma teoria absurda que foi
demonstrada
> > > falsa, portanto nao passa de um mito"
> > >
> > > Está enganado. O éter luminífero foi uma teoria que se embasava
nos
> > dados e conhecimentos da época. Explicava eventos e fenomenos do
> > mundo físico, através de observações e experimentos. Ainda que
tenha
> > se mostrado irreal depois, com mais dados, nada nele lembra um
mito,
> > como duendes ou seres divinos de religiões. Sua estrutura, função
e
> > principalmente sua posterior refutação o tornam totalmente
diferente
> > de mitos e lendas.
> > >
> > > Os erros e enganos que as teorias cientificas as vezes
apresentam
> > fazem parte do método de criar repostas e depois testá-las
> > rigorosamente, ajustando se necessário e abandonando o que for
> > refutado. Mitos, por sua vez, se mantém indefinidamente, sem
mudança
> > ou ajuste. Não faz parte de sua natureza ser refutado, nme
precisam
> > de dados e testes para se manter.
> > >
> > > Se descobrirem que o Big Bang é incorreto, e não absurdo como
> > parece pensar que pode acontecer, será uma ação legitimamente
> > científica e os cientistas, tanto os que criaram a teoria quanto
os
> > que a refutaram, ficarão felizes e aceitarão as novas conclusões.
O
> > Big Bang é uma teoria que, neste momento, é mais capaz de
explicar o
> > surgimento do universo que qualquer outra, e perdurará até que
mais
> > dados a comprovem ou refutem. Não é nem de longe um mito, nem
mesmo
> > se descobrirmos que está incorreto (de novo, a teoria será, não
> > absurda, mas incorreta).
> > >
> > > Veja, hoje a Big Bang explica porque o céu não é totalmente
claro,
> > mesmo durante a noite, explica como podem as galáxias estarem se
> > afastando, explica a formação de estrelas, nuvens de gás estelar,
> > materia escura, elementos quimicos, etc. Para substitui-la, é
preciso
> > uma teoria que, além de explicar tudo isso, ainda explique coisas
que
> > escapam a teoria do BB. Se o fizer, o BB será ultrapassado, você
(ou
> > quem for responsável) ganhará um premio Nobel, e a ciência terá
novos
> > dados e hipóteses para testar. Tudo com o rigor necessário.
> > >
> > > O Éter Luminífero explicava a propagação da luz e dezenas de
outros
> > fenomenos observados, que depois foram melhor explicados pela
teoria
> > eletromagnética. Apenas isso, não foi a substituição de um mito
por
> > outro.
> > >
> > > Precisa compreender isso, ou jamais compreenderá o que é a
ciência
> > e o que a torna diferente de crenças e mitos.
> > >
> > > Homero
> > >
> > > ----- Original Message -----
> > > From: pubmed2005
> > > To: ciencialist@yahoogrupos.com.br
> > > Sent: Saturday, March 19, 2005 3:07 AM
> > > Subject: [ciencialist] Re: ninguem é inocente.
> > >
> > >
> > >
> > > Naõ começa tropeçar nos seus proprios argumentos, o Éter
> > luminífero é
> > > um mito que foi derrubado...uma teoria absurda que foi
> > demonstrada
> > > falsa, portanto nao passa de um mito
> > >
> > > Se descobrirem que o Bing bang nao é teoria mais proxima da
> > origem do
> > > universo, isso vai soar tão absurdo como a crença no boi
tatá. Se
> > o
> > > boi tatá nao existe, se foi comprovado a sua inexistencia,
isso
> > vai
> > > refutar que o boi tatá só existe na imaginação criativa das
> > pessoas.
> > > Assim como o Éter luminífero preencheu as mentes científicas
dos
> > > séculos anteriores. Tudo nao se passou de uma ilusão
> > >
> > > Quanto ao Sudário, já está mais que esclarecida minha posição,
> > nao
> > > tenho que ficar repetindo os mesmos arguimentos de forma
circular
> > > como voce vem fazendo
> > >
> > >
> > >
> > > --- Em ciencialist@yahoogrupos.com.br, "Oraculo"
<oraculo@a...>
> > > escreveu
> > > > Olá Pubmed
> > >
> > > >
> > > > risos..:-) É enorme..:-) No primeiro caso, a conclusão, e
não
> > > crença, foi abandonada devido a evidencias posteriores que
> > refutaram
> > > a teoria do éter. No segundo, nada vai mudar e quem crê nisso
vai
> > > continuar crendo, sem que evidencias ou provas ou aspectos
> > materiais
> > > ou racionais tenham o poder de interferir oiu mudar essa
crença.
> > A
> > > diferença, como pode notar é enorme..:-) A primeira, uma
teoria
> > > cientifica, esperava por dados e evidencias que a
confirmassem, e
> > se
> > > conformou com a refutação por falta de provas. A segunda,
criada
> > da
> > > imaginação humana, vai se manter indefinidamente, exatamente
como
> > foi
> > > criada, dependendo apenas da crença subjetiva de seres
humanos.
> > > Enorme diferença..:-)
> > > >
> > >
> > >
> > >
> > >
> > >
> > >
> > > ##### ##### #####
> > >
> > > Para saber mais visite
> > > http://www.ciencialist.hpg.ig.com.br
> > >
> > >
> > > ##### ##### ##### #####
> > >
> > >
> > > Yahoo! Grupos, um serviço oferecido por:
> > >
> > >
> > >
> > >
> > >
> > >
> > >
> > > ----------------------------------------------------------------
----
> > ----------
> > > Links do Yahoo! Grupos
> > >
> > > a.. Para visitar o site do seu grupo na web, acesse:
> > > http://br.groups.yahoo.com/group/ciencialist/
> > >
> > > b.. Para sair deste grupo, envie um e-mail para:
> > > ciencialist-unsubscribe@yahoogrupos.com.br
> > >
> > > c.. O uso que você faz do Yahoo! Grupos está sujeito aos
Termos
> > do Serviço do Yahoo!.
> > >
> > >
> > >
> > > [As partes desta mensagem que não continham texto foram
removidas]
> >
> >
> >
> >
> >
> > ##### ##### #####
> >
> > Para saber mais visite
> > http://www.ciencialist.hpg.ig.com.br
> >
> >
> > ##### ##### ##### #####
> >
> >
> > *Yahoo! Grupos, um serviço oferecido por:*
> >
> > *
> >
<http://br.rd.yahoo.com/SIG=12a402nmr/M=264105.3931087.6562589.1588051
/D=brclubs/S=2137111528:HM/EXP=1111529884/A=2361264/R=6/SIG=10v4acpp0/
*http://br.shopping.yahoo.com/>*
> >
> >
> >
> > ------------------------------------------------------------------
------
> > *Links do Yahoo! Grupos*
> >
> > * Para visitar o site do seu grupo na web, acesse:
> > http://br.groups.yahoo.com/group/ciencialist/
> >
> > * Para sair deste grupo, envie um e-mail para:
> > ciencialist-unsubscribe@yahoogrupos.com.br
> > <mailto:ciencialist-unsubscribe@yahoogrupos.com.br?
subject=Unsubscribe>
> >
> > * O uso que você faz do Yahoo! Grupos está sujeito aos Termos
do
> > Serviço do Yahoo! <http://br.yahoo.com/info/utos.html>.
> >
> >
> >
> >
> > __________ Informação do NOD32 1.1030 (20050319) __________
> >
> > Esta mensagem foi verificada pelo NOD32 Sistema Antivírus
> > http://www.nod32.com.br





SUBJECT: Re: [ciencialist] Re: Terremoto em ortugal.
FROM: "Silvio" <scordeiro@terra.com.br>
TO: <ciencialist@yahoogrupos.com.br>
DATE: 25/03/2005 00:09

Natália: não enche o saco que não coloquei na notícia que a Senhora é que
era a chefe de polícia de Vilamoura..... preservei sua posição e ainda me
velhinho? tenho 37 anos.

silvio, pegureiro.

-----Mensagem Original-----
De: "Maria Natália" <grasdic@hotmail.com>
Para: <ciencialist@yahoogrupos.com.br>
Enviada em: quinta-feira, 24 de março de 2005 17:01
Assunto: [ciencialist] Re: Terremoto em ortugal.




Só pode partir de um facho mesmo
E chegas atrasado ó velhinho. O governo de direita já era.
Vê lá se te cai o barraco em cima.
Tu agora estás em geofísica ó botequeiro?
Te invadi a choça?
Ai queres?
Espera que já te atendo
Maria Natália

--- Em ciencialist@yahoogrupos.com.br, "Silvio" <scordeiro@t...> escreveu
>
>
> Movimento sísmico em Portugal ...
>
>
> Depois dos problemas acontecidos na Ásia, o governo de Portugal
resolveu instalar um medidor de abalos, que cobre todo o país.
> Então, foi enviado, pelo Centro Sísmico Nacional ao quartel da
polícia da cidade de Vilamoura, no norte de Portugal, um telegrama,
que dizia:
> " Possível movimento sísmico na zona. Ponto. Muito perigoso,
superior Richter 7. Ponto. Epicentro a 3 km do povoado. Ponto. Tomem
medidas. Ponto. Informem resultados com urgência. Ponto."
> Bem, os dias se passaram, e só depois de mais de uma semana é que
foi recebido no Centro Sísmico Nacional um telegrama que dizia:
> " Aqui é do Quartel da Polícia de Vilamoura. Ponto. Movimento
sísmico totalmente desarticulado. Ponto.
>
>
> O tal Ritchter tentou fugir, mas foi abatido a tiros. Ponto,
desativamos as zonas Ponto, as putas estão todas prezas a trabalhar na
lavanderia dos presidios feminino Ponto. Epicentro, Epifânio e três
cupinchas detidos. Ponto.
>
>
> Não respondemos antes, porque aqui houve um terremoto do caralho.
Ponto.
>
> Abraços e beijos sem tremer.
>
>
> [As partes desta mensagem que não continham texto foram removidas]





##### ##### #####

Para saber mais visite
http://www.ciencialist.hpg.ig.com.br


##### ##### ##### #####
Links do Yahoo! Grupos












SUBJECT: Re: ninguem é inocente.
FROM: "junior_br2001" <junior_br2001@yahoo.com.br>
TO: ciencialist@yahoogrupos.com.br
DATE: 25/03/2005 00:19


Silvio, o historiadores contam com a autenticidade de fontes
históricas pela análise crítica pela hermeneutica e heurística.
Existem outras ciencias auxiliares que ajudam neste exame crítico
como Numismática, Heráldica, Arqueologia, filologia, etc e tal

As fontes históricas são divididas, em fontes não-intencionais que
são a arqueologia, e aas provas arqueológicas sobre atividade humana
em relação a materiais manipulados, e fontes intencionais que eram a
literatura que tinha a intenção de informar mas nao de comprovar; as
fontes de arquivos, que são registro dessa pessoa, suas atividades,
onde trabalhou, como viveu, registro no Estado, seu trabalho,
profissões e arquivos de historiadores para fins de vivencia da
realidade e relatos da época, e fontes orais que tem a intenção de
informar mais não de comprovar(essa é mais tradicional delas)

Não sei em relação a vida de Jesus, mas hoje é existe um certo
consenso que Jesus foi um personagem da história de fato. Claro que
deveria ter um rosto diferente, um nome proprio sem o pseudonimo
Jesus cristo, , etc e tal. A mesma coisa vale para o Buda, em que se
tem registros de monastérios da India, manutenção da tradição oral e
escrita pelos monges, registro de documentos em papiros, pedras,etc e
tal. Mas, o Buda fica devendo foto dele para a posteridade, naquela
época nao existia fotografia, mas era costume antigo e medieval se
fazerem-se pinturas dos rostos dos mestres tradicionais e figuras
públicas e políticas(já não se fazem artesão como antes...hehe),
mantendo assim a forma aproximada deles.

Enfim, dá para se ter uma relativa certeza de muitos personagens
históricos.

JR

--- Em ciencialist@yahoogrupos.com.br, "Silvio" <scordeiro@t...>
escreveu
> Júnior:
>
> Exatamente.
> Excluindo alguns papeis religiosos como os da Índia e as epopéias
Olímpicas
> pelos primeiros poetas gregos, creio que foi Herôdotos o autor da
primeira
> obra em prosa da literatura grega preservada até nossos dias. Ele
nasceu em
> 484 a.c. No livro "História" narra suas viagens pelo mundo então
dominado
> pelos gregos. A passagem da tomada do trono Persa por Dario, o que
modificou
> toda a história moderna, citada no livro III números 80-88 daquele
livro é
> um clássico exemplo para os políticos e deve ter influenciado
Maquiavel.
>
> Raros personagens da história que viveram a Idade Média escreveram
> biografia: apenas após o iluminismo (Renascimento) os registros
começaram a
> ser mais claros. Era tudo "por ouvir dizer" ou presenciado pelo
escritor que
> colore o tema com suas próprias tintas, se me entende.
>
> Sua pergunta tá mal colocada pois os personagens realmente
existiram. apenas
> o relato de suas vidas é que precisam ser lidos com extremo cuidado
> observando a fonte.
>
> Sds.,
> silvio.
>
> Nota: Conto com seu voto para 2006: SEVERINO PARA PRESIDENTE.PAULO
COELHO,
> VICE.
>
> pubmed pergunta:
>
> Silvio, segundo seu raciocínio então nenhum personagem histórico
> existiu então.
> Tudo o que sabemos dos povos antigos foram transmitidos por outros e
> não por eles mesmos.
>
> JR
>
> --- Em ciencialist@yahoogrupos.com.br, "Silvio" <scordeiro@t...>
> escreveu
> > Júnior!!!!!
> >
> > Curiosamente, nenhum dos "iluminados" ( Buda, J. Cristo,) deixou
> qualquer
> > vestígio de sua passagem por esse vale de lágrimas. Buda pode até
> ter
> > falado: mas quem, em sã consciência,quem pode repetir ipsum
litera
> suas
> > palavras que rolam em tradições e traduções monásticas há 2.000
> anos?
> >
> > Não vá no papo dessa turma, prefira ler Freud. Vamos lutar pela
> canonização
> > do Paulo Coelho.
> >
> > sds.,
> >
> > silvio





SUBJECT: Não acredito em homeopatia, mas...
FROM: "junior_br2001" <junior_br2001@yahoo.com.br>
TO: ciencialist@yahoogrupos.com.br
DATE: 25/03/2005 02:18


No século 19 a homoeopatia chegou a ser usada com sucesso em casos de
epidemias de cóleras e outras epidemias(representou um paradigma para
a ciencia médica), o que mais me assustou foram os números que
probalisticamente foram muitos melhores do que eu esperaria de um
tratamento homeopatico. De qualquer forma eu não chequei as
informações, podendo ser algo manipulado. Mas acredito que não, pois
me lembro de visto uma filme americano sobre epidemias de cóleras e o
uso da homeopatia, e mais recentemente na Record ela sendo usada por
um médico numa índia doente de cólera, e ela havi ficado boa. Mas
claro concordo piamente que hoje as formas de tratamento avaançaram e
a industria de medicamente sem dúvida bem mais sofisticadas

Veja os números:

Em 1799, Hahnemann controlou uma epidemia de Escarlatina com o
medicamento Belladona. Em 1813 tratou e epidemia de Tifo em
Leipizig , curando 178 de 180 casos com apenas uma fatalidade. Na
grande epidemia de cólera de 1831, a Homeopatia perdeu apenas 6 de
154 pacientes. A medicina convencional da época perdeu 821 de 1501
casos (55%). Nesta época Hahnemann também deu um espantoso
ensinamento de Saúde Pública para época, publicando conselhos sobre
ventilação, higiene, esterelização, infecção e quarentena. Há que se
lembrar que o bacilo do cólera, foi somente descoberto por Koch em
1883, e isto valoriza a inteligência privilegiada e o poder de
observação do Hahnemann. O Dr. Hahnemann não era um teórico, mas um
magnífico praticante da Arte de curar, e trouxe para nós não apenas
mais uma Doutrina Terapêutica, mas um novo conceito de Medicina .

Em 1854, na Inglaterra,onze anos após a morte do Hahnemann, a
Homeopatia mostrou sua eficiência também em uma epidemia de cólera.
Nos hospitais homeopáticos, a taxa de mortalidade foi de 16,4% ,
muito baixa se comparada à taxa nos hospitais que utilizavam a
medicina convencional, que foi de 51.8%. A Homeopatia e o princípio
dos semelhantes foram reconhecidos pelo parlamento britânico e
posteriormente pelo congresso dos EUA como um método terapêutico
médico válido e eficiente.

O texto acima foi escrito por uma médica veterinária que usa
homeopatia como terapia em animais
http://homeopatiaonline.com/ver_textoh.asp?id=11
delete







SUBJECT: Solar Death Ray
FROM: Franco <dfranco@pop.com.br>
TO: ciencialist@yahoogrupos.com.br
DATE: 25/03/2005 02:21

Arma feita com raios solares. Galeria dos alvos usados exterminados por
ela. Pode-se, inclusive, sugerir alvos.

Fonte: Portal Terra

http://www.solardeathray.com/ <http://ueba.com.br/go/25482>

A galeria de fotos está em http://www.solardeathray.com/gallery_p4.html

E aqui, a descrição de seu funcionamento:
http://www.solardeathray.com/about.html



SUBJECT: El interior del cuerpo humano
FROM: Franco <dfranco@pop.com.br>
TO: ciencialist@yahoogrupos.com.br
DATE: 25/03/2005 02:43

link:
<http://elmundosalud.elmundo.es/elmundosalud/especiales/2005/03/galeria_cuerpo/index.html>

Não sei me lembro se já foi postado aqui.





SUBJECT: Re: [ciencialist] Re: Eter Luminifero (era: ninguem é inocente.)
FROM: "Alberto Mesquita Filho" <albmesq@uol.com.br>
TO: <ciencialist@yahoogrupos.com.br>
DATE: 25/03/2005 04:07

----- Original Message -----
From: "marcelomjr"
Sent: Thursday, March 24, 2005 11:32 PM
Subject: [ciencialist] Re: Eter Luminifero (era: ninguem é inocente.)

Caro Marcelo

Sua mensagem é bastante oportuna e esclarecedora, em especial no que diz
respeito a concepção de ether segundo Einstein. Deixo a critério do JVictor
maiores comentários, mesmo porque minha preocupação maior refere-se a um
problema bem mais antigo, qual seja: a concepção de ether segundo Newton.
Sob esse aspecto, achei interessante a seguinte observação em sua mensagem:

> (nota: contrariamente as palavras de Einstein e ao modelo etérico
> estacionário predominante em fins do séc. XIX, a idéia de movimento pode
> ser, sim, aplicada ao Ether; isto é: um Ether Dinâmico.)

Recentemente escrevi um artigo intitulado "O Movimento Absoluto e a Física
de Newton" e no final do capítulo 10 (o artigo tem 11 capítulos
relativamente curtos) concluo que Newton, no final do século XVII, já
demonstra estar ciente dessa possibilidade. Isso a que você chama "ether
dinâmico", Newton chamou por "espírito da matéria". A conclusão de meu
artigo é a seguinte:

********* início da citação ************

É interessante perceber que se dermos uma conotação mais ampla ao vocábulo
éter, poderemos dizer que ao optarmos por um espírito da matéria (e,
portanto, algo de natureza imaterial) estaremos, de alguma maneira, firmando
a idéia da existência de um éter imaterial não fixo (ou seja, não exercendo
o papel de meio) mas emitido e a se propagar pelo espaço. Por outro lado, ao
optarmos pelas vibrações citadas por Newton estaremos firmando a idéia da
existência de um éter também imaterial, mas fixo, maleável (tal e qual uma
gelatina) e incapaz de opor resistência aos corpos materiais. O ponto em
comum das duas idéias seria tão somente a imaterialidade desses supostos
éteres.

Ainda que seja possível, através da aceitação de qualquer dessas duas
condutas acima apontadas, pensarmos numa recuperação total da física
genuinamente newtoniana, decorrente de uma reformulação das interpretações
do observado em experiências efetuadas nos séculos XIX e XX, quero crer que
ainda não há um número suficiente de experimentos a resolver, de uma vez por
todas, o dilema vivenciado por Newton e, curiosamente, deixado de lado pela
grande maioria de seus mais obstinados seguidores.

********* final da citação ************

O artigo pode ser lido a partir de
http://ecientificocultural.com/ECC2/artigos/movab00.htm e já convidei os
amigos físicos da Ciencialist a palpitarem sobre o mesmo
( http://br.groups.yahoo.com/group/ciencialist/message/40983 ). Apenas o Léo
palpitou e até mesmo corrigiu alguns tópicos iniciais, mas acredito que os
demais não gostaram da minha irreverência ao concluir o trabalho com as
seguintes palavras:

********* início da citação ************

À parte essas dificuldades, encaradas por muitos como intransponíveis, quero
crer que ainda assim vale a pena tentarmos recuperar a genuína física
newtoniana, sob pena de nos perpetuarmos na condição de filósofos da
incerteza e/ou, como diria o poeta Carlos Drummond, de nos contentarmos com
a constatação de que no meio do caminho tinha uma pedra --e, não obstante,
nada fizemos para removê-la.

********* final da citação ************

Parece-me que os "céticos" da Ciencialist ainda estão se questionando se
existiria ou não um caminho além dessa pedra. É neste sentido que digo que é
muito fácil ser "cético" (as aspas aqui são importantes), o difícil é ser
quebrador de pedras.

[ ]´s
Alberto
http://ecientificocultural.com/indice.htm
Mas indiferentemente a tudo isso, o neutrino tem massa, o elétron não é
uma carga elétrica coulombiana e a Terra se move. E a história se repetirá.



SUBJECT: Re: [ciencialist] Leis de newton (p/ Daniela)
FROM: "Luiz Ferraz Netto" <leobarretos@uol.com.br>
TO: <ciencialist@yahoogrupos.com.br>
DATE: 25/03/2005 09:23

JR

Outros podem complementar com uma explicação, mas voce pode encontrar
algo bem simples neste site

http://luisperna.com.sapo.pt/leis_newton.htm


Léo: Sinto desapontá-los, mas o texto no site indicado 'tem problemas'; veja:

"Quando o cavalo freia subitamente, o cavaleiro é projectado.

Veja o exemplo do cavalo e do cavaleiro. Quando o cavalo pára subitamente, o cavaleiro que estava em movimento tende a continuar em movimento, logo este é lançado para a frente. O exemplo, ilustra bem a importância do uso do cinto de segurança quando andamos de automóvel. Se os passageiros estiverem soltos no interior do automóvel, qualquer movimento brusco, como o de uma travagem ou um choque acidental, o automóvel irá parar subitamente, e os passageiros serão projectados, ....."

Ninguém é projetado --- não há força alguma que arranque o cavaleiro de cima do cavalo --- não há força alguma que 'lança' para a frente --- os pasageiros não são projetados ..........etc.

A primeira lei é 'bem filosófica' --- pois nada diz sobre o 'porque a matéria quer continuar do jeito que está' --- apenas conclui um fato. Ocorre disso tb na rotação!

[]'
Léo




SUBJECT: Matemática
FROM: Daniela Yoshikawa <danieleakemi@yahoo.com.br>
TO: ciencialist@yahoogrupos.com.br
DATE: 25/03/2005 09:44

No quadrilátero convexo ABCD, as diagonais AC e BD são perpendiculares e os lados opostos AB e DC não são paralelos. Suponha que o ponto P, onde as mediatrizes de AB e DC se encontram, é interior a ABCD. Mostre que ABCD é um quadrilátero inscritível se, e somente se os triângulos ABP e CDP têm áreas iguais.

Obrigada.



---------------------------------
Yahoo! Acesso Grátis: Internet rápida e grátis. Instale o discador agora!

[As partes desta mensagem que não continham texto foram removidas]



SUBJECT: Re: [ciencialist] El interior del cuerpo humano
FROM: "Silvio" <scordeiro@terra.com.br>
TO: <ciencialist@yahoogrupos.com.br>
DATE: 25/03/2005 10:14

Franco:

Maravilha!!! Tem mais?? Sempre tem gente nova que quer aprender.

Parabéns,

silvio.
Franco"

>
> link:
> <http://elmundosalud.elmundo.es/elmundosalud/especiales/2005/03/galeria_cuerpo/index.html>
>

>
>
>
>
>
> ##### ##### #####
>
> Para saber mais visite
> http://www.ciencialist.hpg.ig.com.br
>
>
> ##### ##### ##### #####
> Links do Yahoo! Grupos
>
>
>
>
>
>
>
>
>
>



SUBJECT: Re: [ciencialist] Re: Criacionismo
FROM: "Alvaro Augusto \(E\)" <alvaro@electraenergy.com.br>
TO: <ciencialist@yahoogrupos.com.br>
DATE: 25/03/2005 11:07

Caro Junior,

A história do surgimento do cristianismo é muito mais complexa do que parece. Supor que o "barrabismo" teria surgido da crucificação de Barrabás é simplificar demais a coisa.

Quanto a "Esperando os Olimpianos", trata-se apenas de um conto de 40 páginas, não de um livro. Foi publicado no terceiro número da finada Isaac Asimov Magazine brasileira, e é um conto no estilo "realidade alternativa". Acabo de verificar que o autor é Frederick Pohl, não de Poul Anderson.

[ ]s

Alvaro Augusto

----- Original Message -----
From: junior_br2001
To: ciencialist@yahoogrupos.com.br
Sent: Thursday, March 24, 2005 10:32 PM
Subject: [ciencialist] Re: Criacionismo



Álvaro, mesmo se não existisse a religião alguma outra forma de
ideologia política e imperial iria dominar as massas e deter o poder.
Quanto ao referido livro é do estilo de O Código da Vinci?

Até,
JR

--- Em ciencialist@yahoogrupos.com.br, "Alvaro Augusto \(E\)"
<alvaro@e...> escreveu
> Se Barrabás tivesse sido crucificado, provavelmente nenhuma
religião teria surgido disso. Barrabás era apenas um sicário, mais
precisamente um zelote, uma espécie de terrorista judeu, cuja luta
era meramente terrestre e se resumia a expulsar os romanos e
reestabelecer a suposta glória do reino de Salomão. Como ele, havia
centenas, muitos dos quais foram crucificados.
>
> Mas há um conto fascinante, de Poul Anderson, chamado "Esperando os
Olimpianos", no qual Jesus não foi crucificado, a igreja católica
nunca existiu e o império romano dura até os dias de hoje...
>
> [ ]s
>
> Alvaro Augusto
>
> ----- Original Message -----
> From: JVictor
> To: ciencialist@yahoogrupos.com.br
> Sent: Friday, March 25, 2005 9:06 PM
> Subject: Re: [ciencialist] Re: Criacionismo
>
>
> Só queria saber uma coisa, liás, two:
>
> se ao invés de Barrabás, os mui amigos judeus tivessem livrado
Jesus da
> crucificação:
>
> 1-como seria a Igreja e seus seguidores, hoje?
>
> 2 - e nós, seríamos cristãos ou o quê?
>
> Por nada não, por nada não...
>
> Sds,
>
> Victor.
>
>
>
>
>
>
> Alvaro Augusto (E) escreveu:
>
> > Caro Junior,
> >
> > O cristianismo nasceu na Judéia, que ficava bem pra lá do
meridiano de
> > Greenwitch. Logo, o cristianismo também é uma filosofia
oriental...
> >
> > [ ]s
> >
> > Alvaro Augusto
> >
> > ----- Original Message -----
> > From: junior_br2001
> > To: ciencialist@yahoogrupos.com.br
> > Sent: Thursday, March 24, 2005 5:14 PM
> > Subject: [ciencialist] Re: Criacionismo
> >
> >
> >
> > Não sei até onde aceitar a teoria cíclica sobre a origem do
universo,
> > seja criacionismo.
> > Eu respondi porque senti que a indireta foi para mim, e
porque eu
> > tava defendendo a possibilidade do Sudário ter 3000 anos.
> >
> > Mas tem gente que me conhece aqui e pode responder por mim.
Sempre
> > afirmei que gosto das filosofias orientais. Se eu tivesse que
ser um
> > religiosos eu seria um budista, mas nunca um cristão, mesmo
porque
> > acho o cristianismo muito ridículo
> >
> > Até,
> > JR
> >
> >
> > -- Em ciencialist@yahoogrupos.com.br, "Mr. Vain"
<mr.vain@g...>
> > escreveu
> > >
> > > Parece-me que temos um criacionista disfarçado na lista.
> > >
> > > [ ]'s
> > >
> > > Dr. Vain
> > > Humanitate Magis
> > > Quam Religione
> > > Nobis Opus Est
>
>
> [As partes desta mensagem que não continham texto foram removidas]





##### ##### #####

Para saber mais visite
http://www.ciencialist.hpg.ig.com.br


##### ##### ##### #####


Yahoo! Grupos, um serviço oferecido por:







------------------------------------------------------------------------------
Links do Yahoo! Grupos

a.. Para visitar o site do seu grupo na web, acesse:
http://br.groups.yahoo.com/group/ciencialist/

b.. Para sair deste grupo, envie um e-mail para:
ciencialist-unsubscribe@yahoogrupos.com.br

c.. O uso que você faz do Yahoo! Grupos está sujeito aos Termos do Serviço do Yahoo!.



[As partes desta mensagem que não continham texto foram removidas]



SUBJECT: Re: Criacionismo
FROM: Maria Natália <grasdic@hotmail.com>
TO: ciencialist@yahoogrupos.com.br
DATE: 25/03/2005 11:08


Com calma:
Não é a religião , o facto de os católicos acreditarem e terem a fé
deles é sim as normas da organização dita católica serem rídiculas. É
preciso cuidado senão teremos aqui uma guerra santa e vai haver mortos
feridos e estropiados.
Para se discutir temas como estes de análise destrutiva de
organizações é na Acropolis. Ali todos os incréus, infieis, agnósticos
ateus, anti clericais e outros afins, podem viver, sobreviver e vegetar.
E depois por sermos democratas e não andarmos aqui a achincalhar
pessoas e muto menos povos admitimos criacionistas pois em nossa
lista. A Ciência tem destas coisas: até a lista é aberta e temos aqui
uns moderadores bacanas. Não falei de bacanais. Isso é...noutro lado e
em pvt direi onde a quem quiser saber. Temos aqui crinças menores de
40 anos.
E de ciência: a teoria ciclica é assim a modos que um big bang seguido
de um big crash etc e tal?
Só estive...deitando petróleo para apagar a fogueira.
Maria Natália

--- Em ciencialist@yahoogrupos.com.br, "junior_br2001"
<junior_br2001@y...> escreveu
>
> Não sei até onde aceitar a teoria cíclica sobre a origem do universo,
> seja criacionismo.
> Eu respondi porque senti que a indireta foi para mim, e porque eu
> tava defendendo a possibilidade do Sudário ter 3000 anos.
>
> Mas tem gente que me conhece aqui e pode responder por mim. Sempre
> afirmei que gosto das filosofias orientais. Se eu tivesse que ser um
> religiosos eu seria um budista, mas nunca um cristão, mesmo porque
> acho o cristianismo muito ridículo
>
> Até,
> JR
>
>
> -- Em ciencialist@yahoogrupos.com.br, "Mr. Vain" <mr.vain@g...>
> escreveu
> >
> > Parece-me que temos um criacionista disfarçado na lista.
> >
> > [ ]'s
> >
> > Dr. Vain
> > Humanitate Magis
> > Quam Religione
> > Nobis Opus Est





SUBJECT: Re: [ciencialist] Re: Criacionismo
FROM: "Alberto Mesquita Filho" <albmesq@uol.com.br>
TO: <ciencialist@yahoogrupos.com.br>
DATE: 25/03/2005 12:21

----- Original Message -----
From: "Maria Natália"
Sent: Friday, March 25, 2005 11:08 AM
Subject: [ciencialist] Re: Criacionismo

> É preciso cuidado senão teremos aqui uma guerra santa e vai haver mortos
> feridos e estropiados.

Apoiada. "Quebrar o pau" sobre religião ou temas afins, aqui na Ciencialist,
eu só aceito se o meu oponente for o Jocax. Mas pelo visto, depois que ele
criou uma seita religiosa a se opor à nossa, ele não nos visita mais. Se
alguém quiser falar com o Jocax terá que ser no seu território sagrado
localizado em http://groups.yahoo.com/group/Genismo/

Vale a pena ler um artigo que o sumo sacerdote do Genismo, o Jocax,
recomendou para seus súditos:
http://www1.folha.uol.com.br/folha/ciencia/ult306u13127.shtml (se não
conseguirem acessar a FSP leiam a msg 3721 do grupo Genismo, de
22/mar/2005).

> Temos aqui crianças menores de 40 anos.

Bem lembrado. Precisamos pensar na formação dessa gurizada, para o bem da
Ciencialist. Do contrário eles irão debandar para criar novas seitas
religiosas, a exemplo do Jocax. O Júnior, por ex., vira e mexe tenta nos
trair criando outras seitas científicas a concorreram com a nossa. Estou até
com vontade de subir ao Léo (ou até ouranos, território do Brudna) para
pedir a Ele(s) que tome(m) uma providência mais drástica.

Ouranos, para a gurizada que não sabe nada, é o "supremo acima de
Aristóteles", ou a última camada dos céus.

> Só estive...deitando petróleo para apagar a fogueira.

Mui amiga!!! Essa é a Natália que eu conheço. ;-)

[ ]´s
Alberto
http://ecientificocultural.com/indice.htm
Mas indiferentemente a tudo isso, o neutrino tem massa, o elétron não é
uma carga elétrica coulombiana e a Terra se move. E a história se repetirá.



SUBJECT: Re: Criacionismo
FROM: "junior_br2001" <junior_br2001@yahoo.com.br>
TO: ciencialist@yahoogrupos.com.br
DATE: 25/03/2005 12:34


--- Em ciencialist@yahoogrupos.com.br, "Alvaro Augusto \(E\)"
<alvaro@e...> escreveu
> Se Barrabás tivesse sido crucificado, provavelmente nenhuma
religião teria surgido disso.


JR: Não sei se seria assim exatamente. Jesus era bastante conhecido,
e mesmo se tivesse morto depois, ele deixaria seguidores. Buda nao
morreu na cruz, mas deixou seguidores. Não gosto de ficar falando
como história deveria ser, pois o que importa é se focar como devem
ter acontecido os fatos
Quanto aos barrabãos , eu estava mesmo de brincadeira.
Mas, mesmo que o cristianismo, não dominasse as massas, outra
ideologia o faria, porque o poder não provinha de religião exatamente
e sim do domínio dos meios de produção, acúmulos de riquezas que
conferia poder e controle ao Estado, as Igreja e classe previlegiada.
Certamente seria um poder imperial tipo o de Roma que dominaria, Roma
só adotou a ideologia cristã porque estava em decadencia, mas se
tivesse qualquer outra ideologia(politica, religiosa, administrativa,
filosofica) na época que satisfizesse as massas(ou que fossem
impostas) serveria da mesma forma.

Sobre a história do cristianismo , eu tenho um texto na lista do
Waib, aquela que divulguei. Só nao passo o texto pra cá , porque essa
é uma lista de ciencia. Apesar de a história ser uma ciencia, ainda
não sei com certeza, se esta lista é apenas de ciencias naturais ou
de todas as ciencias em geral, isso só o Brudna pode confirmar pra
mim.

Quanto ao livro indicado, quando eu tiver cacife vou compra-lo.

Agradeço de antemão
JR







SUBJECT: Re: Criacionismo
FROM: "junior_br2001" <junior_br2001@yahoo.com.br>
TO: ciencialist@yahoogrupos.com.br
DATE: 25/03/2005 12:41


--- Em ciencialist@yahoogrupos.com.br, "Alberto Mesquita Filho"
<albmesq@u...> escreveu
O Júnior, por ex., vira e mexe tenta nos
> trair criando outras seitas científicas a concorreram com a nossa.
Estou até
> com vontade de subir ao Léo (ou até ouranos, território do Brudna)
para
> pedir a Ele(s) que tome(m) uma providência mais drástica.



JR: hahahah. Alberto eu nao tenho tido muita sorte ultimamente, tenho
sido expulso de várias listas ceticas, e se voce insistir dessa
empreitada acabarei sendo expulso da ciencialist. O SR. Deus Léo não
negaria um pedido seu, mesmo porque voce é um espécie de João Batista
por aqui.
Minhas seitas científicas tb nao tem feito muito sucesso, porque em
todo lugar que estudo sempre a Autoridade Maior do Consenso Geral da
Republica Científica Csicopista do Reino Maior sempre tem uma opinião
esmagadora , sendo que quase sempre vou a julgamento, e é me
preterida uma pena de expulsão com banimentos eternos.

heheheh

JR





SUBJECT: Re: Criacionismo
FROM: "junior_br2001" <junior_br2001@yahoo.com.br>
TO: ciencialist@yahoogrupos.com.br
DATE: 25/03/2005 12:46


Ops, onde escrevi proterida, o certo seria proferida


--- Em ciencialist@yahoogrupos.com.br, "junior_br2001"
<junior_br2001@y...> escreveu
> JR: hahahah. Alberto eu nao tenho tido muita sorte ultimamente,
tenho
> sido expulso de várias listas ceticas, e se voce insistir dessa
> empreitada acabarei sendo expulso da ciencialist. O SR. Deus Léo
não
> negaria um pedido seu, mesmo porque voce é um espécie de João
Batista
> por aqui.
> Minhas seitas científicas tb nao tem feito muito sucesso, porque em
> todo lugar que estudo sempre a Autoridade Maior do Consenso Geral
da
> Republica Científica Csicopista do Reino Maior sempre tem uma
opinião
> esmagadora , sendo que quase sempre vou a julgamento, e é me
> preterida uma pena de expulsão com banimentos eternos.
>
> heheheh






SUBJECT: Re: [ciencialist] Re: Criacionismo
FROM: "Alberto Mesquita Filho" <albmesq@uol.com.br>
TO: <ciencialist@yahoogrupos.com.br>
DATE: 25/03/2005 13:06

----- Original Message -----
From: "junior_br2001"
Sent: Friday, March 25, 2005 12:41 PM
Subject: [ciencialist] Re: Criacionismo

> se voce insistir dessa empreitada acabarei sendo expulso da ciencialist. O
> SR. Deus Léo não negaria um pedido seu, mesmo porque voce é um espécie de
> João Batista por aqui.

Nem pensar! O máximo que pode acontecer é lhe impormos o castigo de escrever
1000 vezes: "Prometo não abrir novas listas de discussão sobre ciência".

Mas isso você tira de letra, é só usar o Excel e o Léo nem vai perceber que
você escreveu uma vez só. Se duvida consulte a mensagem
http://br.groups.yahoo.com/group/ciencialist/message/45284

[ ]´s
Alberto
O Arcanjo Subversivo do Reino do Léo
http://ecientificocultural.com/indice.htm
Mas indiferentemente a tudo isso, o neutrino tem massa, o elétron não é
uma carga elétrica coulombiana e a Terra se move. E a história se repetirá.



SUBJECT: Re: [ciencialist] Re: Enrolamentos de Motores Elétricos
FROM: jose marks <realqualidadevida@yahoo.com.br>
TO: ciencialist@yahoogrupos.com.br
DATE: 25/03/2005 13:29

Boa tarde Junior .

Quanto a enrolar transformador, não posso lhe dizer nada, mas tenho um bobinadeira profissional, que enrola transformadores de diversos tamanhos.

Caso haja interesse, por favor entere em contato via e-mail jamarks@ig.com.br ou fone 0xx15-3222-6170 Sorocaba SP

Esse trabalho posso lhe informar alguma coisa.

Jose



junior_br2001 <junior_br2001@yahoo.com.br> wrote:

Programa nao sei de nenhum não, mas sei que o Instituro Monitor tem
um cursos profissionalizantes e técnicos por correspondencia sobre
enrolamento de motores - É muito interessante, e nao é tão caro
porque voce paga parcelado

Jr

--- Em ciencialist@yahoogrupos.com.br, "Tipoalgo" <tipoalgo@b...>
escreveu
>
> Olá a todos
>
> Estou estudando e praticando como enrolar, ou bobinar, motores
> elétricos.
> Tenho um livro, Cálculo de Enrolamentos de Máquinas Elétricas e
> Sistemas de Alarme, que trata bem o assunto, mas gostaria de
auxílio
> mais moderno para o assunto.
> Para facilitar meus estudos procurei na internet um programa livre,
e
> achei em http://www.md.cefetpr.br/tocadofisch/dgex101.zip um bom
> ajudante.
> Entretanto este programa ainda está em desenvolvimento.
> Aproveito a oportunidade para agradecer ao autor por disponibilizá-
lo
> na internet de forma livre.
> Alguém conhece um programa livre semelhante que pudesse nos
> apresentar?
>
> Agradecimentos antecipados.
>
> Tipoalgo

__________________________________________________
Converse com seus amigos em tempo real com o Yahoo! Messenger
http://br.download.yahoo.com/messenger/

[As partes desta mensagem que não continham texto foram removidas]



SUBJECT: Re: [ciencialist] Reflexão inversão em esp planos
FROM: "Eurico Ferreira de Souza Jr." <caodejah@yahoo.com.br>
TO: ciencialist@yahoogrupos.com.br
DATE: 25/03/2005 13:32



PSavio <psdmo@uol.com.br> wrote:
Prof.Léo e demais participantes desta Lista,


Uma imagem num espelho plano apresenta simetira esquerda direita.

[E]> nosso corpo possui simetria bilateral, logo a imagem do nosso corpo refletida no espelho, também...

mas n apresenta simetria superior-inferior.

[E]> nós também não. coloque uma bola (p.ex.) na frente do espelho e a simetria será total.

Ainda estou estudando para uma resposta satisfatória. Alguém tem alguma idéia?

[E]> pare de imaginar o espelho como um mundo do outro lado do vidro. é apenas um reflexo...






_\|/_

---------------------------------
Yahoo! Acesso Grátis: Internet rápida e grátis. Instale o discador agora!

[As partes desta mensagem que não continham texto foram removidas]



SUBJECT: Re: [ciencialist] Re: Enrolamentos de Motores Elétricos
FROM: jose marks <realqualidadevida@yahoo.com.br>
TO: ciencialist@yahoogrupos.com.br
DATE: 25/03/2005 13:36

Corrigindo escrita errada.
Desculpe, quanto a enrolar motor não posso lhe dizer nada, ok?
Então no lugar do transformador, leia sim "motor".
Desculpe,
José.

jose marks <realqualidadevida@yahoo.com.br> wrote:
Boa tarde Junior .

Quanto a enrolar transformador, não posso lhe dizer nada, mas tenho um bobinadeira profissional, que enrola transformadores de diversos tamanhos.

Caso haja interesse, por favor entere em contato via e-mail jamarks@ig.com.br ou fone 0xx15-3222-6170 Sorocaba SP

Esse trabalho posso lhe informar alguma coisa.

Jose



junior_br2001 <junior_br2001@yahoo.com.br> wrote:

Programa nao sei de nenhum não, mas sei que o Instituro Monitor tem
um cursos profissionalizantes e técnicos por correspondencia sobre
enrolamento de motores - É muito interessante, e nao é tão caro
porque voce paga parcelado

Jr

--- Em ciencialist@yahoogrupos.com.br, "Tipoalgo" <tipoalgo@b...>
escreveu
>
> Olá a todos
>
> Estou estudando e praticando como enrolar, ou bobinar, motores
> elétricos.
> Tenho um livro, Cálculo de Enrolamentos de Máquinas Elétricas e
> Sistemas de Alarme, que trata bem o assunto, mas gostaria de
auxílio
> mais moderno para o assunto.
> Para facilitar meus estudos procurei na internet um programa livre,
e
> achei em http://www.md.cefetpr.br/tocadofisch/dgex101.zip um bom
> ajudante.
> Entretanto este programa ainda está em desenvolvimento.
> Aproveito a oportunidade para agradecer ao autor por disponibilizá-
lo
> na internet de forma livre.
> Alguém conhece um programa livre semelhante que pudesse nos
> apresentar?
>
> Agradecimentos antecipados.
>
> Tipoalgo

__________________________________________________
Converse com seus amigos em tempo real com o Yahoo! Messenger
http://br.download.yahoo.com/messenger/

[As partes desta mensagem que não continham texto foram removidas]



##### ##### #####

Para saber mais visite
http://www.ciencialist.hpg.ig.com.br


##### ##### ##### #####


Yahoo! Grupos, um serviço oferecido por:PUBLICIDADE


---------------------------------
Links do Yahoo! Grupos

Para visitar o site do seu grupo na web, acesse:
http://br.groups.yahoo.com/group/ciencialist/

Para sair deste grupo, envie um e-mail para:
ciencialist-unsubscribe@yahoogrupos.com.br

O uso que você faz do Yahoo! Grupos está sujeito aos Termos do Serviço do Yahoo!.


__________________________________________________
Converse com seus amigos em tempo real com o Yahoo! Messenger
http://br.download.yahoo.com/messenger/

[As partes desta mensagem que não continham texto foram removidas]



SUBJECT: Re: Criacionismo
FROM: "junior_br2001" <junior_br2001@yahoo.com.br>
TO: ciencialist@yahoogrupos.com.br
DATE: 25/03/2005 13:42


--- Em ciencialist@yahoogrupos.com.br, "Alberto Mesquita Filho"
<albmesq@u...> escreveu
> ----- Original Message -----
> From: "Maria Natália"
> Sent: Friday, March 25, 2005 11:08 AM
> Subject: [ciencialist] Re: Criacionismo
>
> Estou até
> com vontade de subir ao Léo (ou até ouranos, território do Brudna)
para
> pedir a Ele(s) que tome(m) uma providência mais drástica.



JR: falando em moderadores, o Takata anda meio sumido. Tem alguma
coisa errada, acho que ele é o Sammy do Big Brother. Quando a Globo
está editando os programas o Takata aproveita p/ teclar no computador
da Globo e enviar suas mensagens para a lista.
Bem, pelo mneos já sabemos agora que o Takata está com dinheiro, ele
ganhou uma franquia comercial de salgadinhos, e ainda uma passagem
para Nova York, será que ele fugiu de lá e está novamente em Nova
York? Acho que não, pois senão ele provavelmente estaria respondendo
as mensagens do ciencialist no Word Trad Center... hehehe

Vamos esperar pra ver falta uns 4 para acabar o Big Brother, se ele
aparecer por aqui de novo, saberemos que ele estava de fato lá, e era
o Sammy. Po, o cara atacou logo de duas - Carla e a Pink. Que menino
guloso!!!

JR





SUBJECT: Re: [ciencialist] Re: Criacionismo
FROM: "Alberto Mesquita Filho" <albmesq@uol.com.br>
TO: <ciencialist@yahoogrupos.com.br>
DATE: 25/03/2005 14:05

----- Original Message -----
From: "junior_br2001"
Sent: Friday, March 25, 2005 1:42 PM
Subject: [ciencialist] Re: Criacionismo

> o Takata anda meio sumido. Tem alguma
> coisa errada, acho que ele é o Sammy do Big Brother.

Que ninguém nos ouça, mas os "céticos de carteirinha" (Takata, Oráculo e Cia
Ltda) estão no Retiro Espiritual. Afinal estamos na Semana Santa e eles
estão se penitenciando dos pecados que cometeram durante o ano aqui na
Ciencialist. Favor não espalhar. Que isso fique entre nós. Eu sei porque o
pedido de licença que ele encaminhou para o Brudna acabou, por engano,
passando pelo meu computador. Sacumé, o sofware que foi instalado no Takata
é da Microsoft e em virtude disso ele vive cometendo essas imprudências.

[ ]´s
Alberto
http://ecientificocultural.com/indice.htm
Mas indiferentemente a tudo isso, o neutrino tem massa, o elétron não é
uma carga elétrica coulombiana e a Terra se move. E a história se repetirá.



SUBJECT: Re: Reflexão inversão em esp planos
FROM: "rmtakata" <rmtakata@altavista.net>
TO: ciencialist@yahoogrupos.com.br
DATE: 25/03/2005 14:22


--- Em ciencialist@yahoogrupos.com.br, "PSavio" <psdmo@u...>
> Uma imagem num espelho plano apresenta simetira esquerda
> direita. mas n apresenta simetria superior-inferior. Ainda
> estou estudando para uma resposta satisfatória. Alguém tem
> alguma idéia?

Isso eh apenas uma confusao gerada pelo fato de, como disse Eurico,
nosso plano corporal de simetria dividir em metades esquerda e direita.

Vista uma camisa com uma frase qualquer e fique em frente ao espelho -
a frase nao terah uma simetria esquerda-direita.

Talvez pense: mas ela ficou invertida da direita para a esquerda.

Bem, se vc colocar um espelho no teto a frase - e sua imagem - ficarao
invertida de cima para baixo.

Tudo eh consequencia de dois fatos: raios de luz viajam em linha reta
(ao menos bem aproximadamente reta), espelho refletem raios de luz com
angulo equivalente ao angulo de incidencia.

[]s,

Roberto Takata





SUBJECT: Abundancia de hélio no universo
FROM: "junior_br2001" <junior_br2001@yahoo.com.br>
TO: ciencialist@yahoogrupos.com.br
DATE: 25/03/2005 14:29


Existe algum modelo que pode explicar a abundncia de hélio no
universo que no seja o evento primordial(Big bang)?

Pelo que pude perceber a explosão de uma estrela aumenta a proporção
de material mais denso que o hélio. Só o Big Bang explica essa
abundancia de hélio no universo? Cientista não identificaram outro
evento? Ou outra maneira de esse hélio ter sido produzido?

Quem encara?

JR





SUBJECT: Re: [ciencialist] Re: Reflexão inversão em esp planos
FROM: "Alberto Mesquita Filho" <albmesq@uol.com.br>
TO: <ciencialist@yahoogrupos.com.br>
DATE: 25/03/2005 14:54

----- Original Message -----
From: "rmtakata"
Sent: Friday, March 25, 2005 2:22 PM
Subject: [ciencialist] Re: Reflexão inversão em esp planos

Olá Takata, tudo bem.

Acabei de falar de você para o Júnior. Eu estava elogiando :-) o fato de
você ter ido para um retiro espiritual com o Homero, mas pelo visto você já
voltou. Bom retorno. Rezou bastante?

Quanto ao problema da imagem é como você e o Eurico disseram.

Eu estive pensando, por exemplo, num indivíduo deitado com a mão direita
para baixo e a esquerda para cima e olhando-se num espelho perpendicular ao
chão. A imagem terá o que parecerá ser a mão direita (na realidade é a
imagem da mão esquerda) para cima, e a outra para baixo. Talvez isso ajude o
solicitante a entender o processo.

[ ]´s
Alberto
http://ecientificocultural.com/indice.htm
Mas indiferentemente a tudo isso, o neutrino tem massa, o elétron não é
uma carga elétrica coulombiana e a Terra se move. E a história se repetirá.



SUBJECT: RE: [ciencialist] Dia Mundial da Árvore e da Floresta
FROM: "murilo filo" <avalanchedrive@hotmail.com>
TO: ciencialist@yahoogrupos.com.br
DATE: 25/03/2005 15:01

Natália, oi.
Gostei da sua campanha por ''uma nova floresta'', já que a original
funfou...
De vez em quando dou uma saída por aí e justamente acabo de chegar.
Estive por duas semanas em Mato Grosso e gostaria de propor um novo e mais
próprio nome para êste grande estado brasileiro.
Troquemos seu nome de Mato Grosso para *Fazenda Grossa*.
Esta é uma prova terminal e agonizante de que o progresso pode ser um crime
irresponsável.
Em alguns séculos estarão por lá fazendo suas campanhas e no meio do
deserto. abr/M.

>From: Maria Natália <grasdic@hotmail.com>
>Reply-To: ciencialist@yahoogrupos.com.br
>To: ciencialist@yahoogrupos.com.br
>Subject: [ciencialist] Dia Mundial da Árvore e da Floresta
>Date: Wed, 16 Mar 2005 06:13:06 -0000
>
>
>
>No dia 21 de Março comemora-se o Dia Mundial da Árvore e da Floresta.
> O FAPAS antecipa a comemoração para o dia 20 de Março,domingo, dia
>mais adequado para actividades colectivas.
>
>Comemoração do Dia da Árvore, dia 20 de Março (Domingo), 14h30-17h30,
>com as seguintes actividades no Jardim Botânico da Universidade de Lisboa.
>
>1.- Sementeira de Carvalhos no viveiro do FAPAS, no Jardim Botânico da
>Universidade de Lisboa (Rua da Escola Politécnica)
>
>2.- Breve percurso pelo Jardim, para escolha de árvores ou arbustos
>nativos a adoptar pelo FAPAS.
>
>
>Descrição das actividades:
>Local - Jardim Botânico na Rua da Escola Politécnica, 58, em Lisboa
>(nas traseiras do Museu da Ciência e do Museu de História Natural)
>Data: 20 de Março de 2005
>Horário: início às 14h30, final pelas 17h30. Pode participar o tempo
>que quiser, dentro deste horário. Aproveite para visitar um dos
>jardins mais bonitos de Lisboa.
>Ponto de Encontro: às 14h30 junto ao guichet na entrada do Jardim.
>Entre pelo portão de ferro da rua da Escola Politécnica, e percorra a
>"avenida" das palmeiras, ao lado do Museu da Ciência, até atingir o
>portão do Jardim, ao fundo da "avenida".
>
>Para chegar ao Jardim: a rua da Escola Politécnica fica entre o Largo
>do Rato e o Largo do Príncipe Real.
>Metro: linha amarela, estação Rato. Autocarros: 58 (paragem em frente
>à entrada do jardim), 100 (largo do Príncipe Real), 6, 9, 20, 27, 38,
>49 (paragens no Largo do Rato).
>
>
>1.- Sementeira de Carvalhos no viveiro do FAPAS, no Jardim Botânico da
>Univ.Lisboa
>
>Desde a fundação do FAPAS, há cerca de 12 anos, que a associação tem
>organizado sementeiras e plantações de árvores nativas, numa tentativa
>de auxiliar a regeneração das nossas matas nativas, quase
>desaparecidas. O Núcleo de Lisboa tem organizado recolhas de sementes,
>sementeiras e plantações de árvores (sobreiros, carvalhos-cerquinhos e
>carvalhos-negrais) desde 2002, tendo semeado dezenas de milhares de
>árvores nas serras de Sintra, Montejunto e Arrábida.
>
>Infelizmente, a seca deste inverno motivou a realização de um número
>de sementeiras inferior ao previsto, pelo que vamos tentar aproveitar
>os milhares de sementes de carvalho-cerquinho que ficaram em stock,
>para produzir pequenas árvores em viveiro. Estas árvores serão
>plantadas por voluntários em Áreas Protegidas nos próximos meses de
>Novembro e Dezembro.
>
>Associe-se a nós no próximo dia 20 de Março, e semeie uma dúzia de
>carvalhos (ou mais) no nosso viveiro. A sementeira consiste em
>colocar em recipientes com terra sementes (bolotas) germinadas de
>carvalho-cerquinho, árvore nativa da região Centro-litoral do nosso
>país. A tarefa é muito simples e rápida, sendo acessível para crianças.
>Material aconselhado: luvas de cozinha, avental.
>Agradecemos se trouxer consigo alguns pacotes (vazios) de litro de
>sumos ou leite, tipo tetrapack ou Vigor. Se possível, passe-os por
>água e corte a parte de cima, para formar uma espécie de vaso.
>
>2. Percurso pelo Jardim para adopção de árvores e arbustos.
>
>Em solidariedade com o Jardim Botânico da Universidade de Lisboa, o
>FAPAS associa-se à campanha de apadrinhamento de árvores e arbustos
>lançada pela Liga dos Amigos do Jardim Botânico. Pelo preço de 1 euro
>por metro quadrado, pode-se adoptar durante um ano uma árvore ou
>arbusto à nossa escolha. O FAPAS vai efectuar um pequeno percurso para
>visitar algumas árvores e arbustos nativos, e seleccionar os
>exemplares a adoptar. Os participantes podem-nos acompanhar no
>percurso e ajudar-nos a escolher os afilhados, e se pretenderem podem
>também apadrinhar um exemplar.
>O nome dos padrinhos fica registado on-line no site do Jardim, e dá
>direito a um cartão de adopção, que inclui 4 entradas gratuitas no Jardim.
>
>-----------------
>INSCRIÇÕES nas actividades:
>Envie uma mensagem com o seu nome para fapas.lisboa@clix.pt ,
>indicando o número de pessoas. .
>Mais informações pelo tm 93 8491355 (João Morais).
>----------------
>
>Preço das entradas no jardim:
>Adultos com menos de 65 anos- 1,50 euros. Crianças até 6 anos -
>grátis. Estudantes e funcionários da Universidade de Lisboa - grátis.
>Estudantes e portadores do cartão jovem - 0,60 euros.
>Adultos com mais de 65 anos - 0,60 euros
>
>
>PARTICIPE! POR UMA NOVA FLORESTA, EM DEFESA DA NATUREZA!
>
>-------------------------------------------------
>Colabore connosco na Conservação da Natureza: Ajude o FAPAS a
>divulgar estas iniciativas.
> ------------------------------------------------
>O FAPAS é uma Associação de defesa da Vida Selvagem com 11 anos de
>existência, e uma vasta actividade na defesa e estudo da Natureza e
>Vida Selvagem em Portugal. Sede: Rua Alexandre Herculano, 371-4º Dtº,
>4000-055 PORTO (telef.22 200 2472).
>Publicações: o FAPAS edita trimestralmente a revista Tribuna da
>Natureza, e publicou, entre outros, uma série de Guias de Campo de
>fauna e flora: Aves de Portugal e Europa, Mamíferos de Portugal e
>Europa, Árvores de Portugal e Europa, Anfíbios e Répteis de Portugal.
>Para ser sócio do FAPAS: consulte www.fapas.pt , ou contacte-nos por
>email ou telefone
>Quotas anuais - adultos: 16,50 Euros (inclui a assinatura de 4 números
>da revista Tribuna da Natureza); ou 7,50 Euros (sem a assinatura).
>juvenis (até 15 anos de idade): 11,50 euros (com assinatura da TN), ou
>2,50 euros (sem assinatura).
>--------------------------
>
>Saudações Ecologistas
>
>FAPAS - Lisboa
>tm: 93 849 1355
>fapas.lisboa@clix.pt
>
>E aí é o começo do Outono, né?
>Um abraço
>Maria Natália
>
>
>
>
>
>
>
>##### ##### #####
>
>Para saber mais visite
>http://www.ciencialist.hpg.ig.com.br
>
>
>##### ##### ##### #####
>Links do Yahoo! Grupos
>
>
>
>
>
>
>
>




SUBJECT: Re: Reflexão inversão em esp planos
FROM: "rmtakata" <rmtakata@altavista.net>
TO: ciencialist@yahoogrupos.com.br
DATE: 25/03/2005 15:29


--- Em ciencialist@yahoogrupos.com.br, "Alberto Mesquita
> Rezou bastante?

Acho q. sim a se basear na quantidade de fantasmas q. tem me
aparecido. Re re...

> Eu estive pensando, por exemplo, num indivíduo deitado com a
> mão direita para baixo e a esquerda para cima e olhando-se
> num espelho perpendicular ao chão. A imagem terá o que
> parecerá ser a mão direita (na realidade é a imagem da mão
> esquerda) para cima, e a outra para baixo. Talvez isso ajude
> o solicitante a entender o processo.

Pode ser. E se ele colocar o relogio no pulso esquerdo acho q.
facilitarah o individuo a identificar a correspondencia entre o
proprio braco e o braco do espelho.

[]s,

Roberto Takata





SUBJECT: Re: Reflexão inversão em esp planos
FROM: "junior_br2001" <junior_br2001@yahoo.com.br>
TO: ciencialist@yahoogrupos.com.br
DATE: 25/03/2005 15:34


--- Em ciencialist@yahoogrupos.com.br, "rmtakata" <rmtakata@a...>
escreveu
>
> --- Em ciencialist@yahoogrupos.com.br, "Alberto Mesquita
> > Rezou bastante?
>
> Acho q. sim a se basear na quantidade de fantasmas q. tem me
> aparecido. Re re...
> []s,



JR: Takata fala a verdade voce é o Sammy. Quem está lá no Big Brother
é um holograma feito por uma de suas filiais de computadores
dissidentes do Google. A não ser que voce esteja usando o notebook do
Bial





SUBJECT: Dúvida sobre o objetivo da lista
FROM: "junior_br2001" <junior_br2001@yahoo.com.br>
TO: ciencialist@yahoogrupos.com.br
DATE: 25/03/2005 16:49


Olá gente,

Esta é uma lista de ciencia. No entanto, aparentemente parece não ter
nenhuma restrição no aspecto de discussão do que é considerado
ciencia ou qualquer área do conhecimento humano. Hoje o conceito de
ciencia evoluiu, ela não é mais restrita somente ciencias naturais,
visto a dimensão de natureza investigativa que outras áreas do
conhecimento alcançaram como a História, a Sociologia, a Arqueologia,
a Psicologia, Antropologia, Ciência da Computação, etc ....e tl

Apesar de já ter pertencido uns 2 anos a esta lista no passado com
participação ativa e depois de estaar a aalguns meses afastado, a ela
novamente retornei , as vezes, fico meio sem graça de discutir outro
assunto está fora do escopo das ciencias naturais. Eu posso parecer
bastante desinibido na minha participação, mas é só aparencias, eu
tenho na verdde este receio e timidez.

Portanto gostaria uma opinião dos moderadores e de outros
participntes tb , se é lícito discutir outras áreras do conhecimento
humano que é considerado ciencia como a matemática, a história, a
sociologia,a arqueologia, a psicologia, a sociologia, ciencia da
computação, além é claro de todas as ciencias naturais : quimica ,
fisica , biologia, etc e tal

Digo isto, porque tenho interesse pelas outras áress de conhecimento
científico, e tb porque no gostaria de estar pagando micos, como
quando eu mando algumas mensagens referentes a história e a
arqueologia, assuntos bsastantes atraentes para mim

Portanto, opinião dos moderadores e principalmente do criador da
lista tem uma certa relevancia para mim, além é claro de
democraticamente , a opinião de todos os participante desta lista

Um grande abraço a todos
JR







SUBJECT: Re: Dúvida sobre o objetivo da lista
FROM: Luis Brudna <luisbrudna@gmail.com>
TO: ciencialist@yahoogrupos.com.br
DATE: 25/03/2005 18:10

On Fri, 25 Mar 2005 19:49:46 -0000, junior_br2001
<junior_br2001@yahoo.com.br> wrote:> Olá gente,> Esta é uma lista de
ciencia. No entanto, aparentemente parece não ter > nenhuma restrição
no aspecto de discussão do que é considerado
> ciencia ou qualquer área do conhecimento humano. Hoje o conceito de
> ciencia evoluiu, ela não é mais restrita somente ciencias naturais,
> visto a dimensão de natureza investigativa que outras áreas do
> conhecimento alcançaram como a História, a Sociologia, a Arqueologia,
> a Psicologia, Antropologia, Ciência da Computação, etc ....e tl

Vai depender do assunto debatido.

A minha preferencia eh por uma lista focada nas 'ciencias naturais'. :-)



> Portanto gostaria uma opinião dos moderadores e de outros
> participntes tb , se é lícito discutir outras áreras do conhecimento
> humano que é considerado ciencia como a matemática, a história, a
> sociologia,a arqueologia, a psicologia, a sociologia, ciencia da
> computação, além é claro de todas as ciencias naturais : quimica ,
> fisica , biologia, etc e tal

A lista tem esse tipo de debate. Nao eh sempre que algum assunto
interessante aparece nestas areas (mais sociais).


> Digo isto, porque tenho interesse pelas outras áress de conhecimento
> científico, e tb porque no gostaria de estar pagando micos, como
> quando eu mando algumas mensagens referentes a história e a
> arqueologia, assuntos bsastantes atraentes para mim
> Portanto, opinião dos moderadores e principalmente do criador da
> lista tem uma certa relevancia para mim, além é claro de
> democraticamente , a opinião de todos os participante desta lista
> Um grande abraço a todos
> JR

O que indico evitar na lista eh assuntos que envolvam religiao e
politica partidaria.


Ateh
Luis Brudna
ps: espero voltar para os debates dentro de alguns dias


SUBJECT: Re: [ciencialist] Abundancia de h�lio no universo
FROM: Eduardo Gueron <edgueron@yahoo.com>
TO: ciencialist@yahoogrupos.com.br
DATE: 25/03/2005 18:12

Oi,

Veja, nco i a explosco em si que gera a abundbncia de
Helio no Universo porque, em teoria, os nzcleos
attmicos se formam em uma determinada etapa da
expansco do Universo (a nucleossmntese). De fato, na
maioria das estrelas i produzido muito hilio por meio
de fusco nuclear (no Sol por exemplo). Outros
elementos sco produzidos em menor escala e alguns ss
surgem quando a estrela explode.

O que interessa, no entanto, i a distribuigco de Helio
e outros materiais leves que i quase uniforme ao longo
do universo (note que como o He 4 i muito produzido
nas estrelas, geralmente se usa a distribuigco de
outros elementos como Deutirio, He 3 e Litio). Para
que tais elementos fossem produzidos i necessario que,
apss a nucleossmntese (basicamente quando se formaram
nzcleos de H) o Universo tenha sido muito quente para
que houvesse fusues nucleares (supondo ser impossmvel
a fusco a frio) em um ambiente semelhante ao interior
de uma estrela. E por que a distribuigco desses
elementos i importante? Simples, porque se mostra que,
mesmo em regiues ausentes de estrelas (a maior parte
do Universo) ha mais ou menos 24% de He4 e uma alguns
porcentos (nco sei de cabega) de cada um dos elementos
acima citados. Ha alternativas ao big-bang mas em
todas que eu conhego, se assume que o Universo foi
muito quente e pequeno em uma determinada ipoca.

Para uma boa referjncia, veja o livro do Kolb (The
Early Universe). Procure na internet por Histsria
tirmica do Universo (Thermal History of the Universe)
que vc tera dados mais precisos. Os dados sco
impressionantemente precisos, vc prevj que a tal
temperatura com tal concentragco de H dever-se-ia
formar X de Litio. O resultado medido i algo que
difere de X na ordem de X/100.

Abrago, Eduardo.

--- junior_br2001 <junior_br2001@yahoo.com.br> wrote:
>
>
> Existe algum modelo que pode explicar a abundncia de
> hilio no
> universo que no seja o evento primordial(Big bang)?
>
> Pelo que pude perceber a explosco de uma estrela
> aumenta a proporgco
> de material mais denso que o hilio. Ss o Big Bang
> explica essa
> abundancia de hilio no universo? Cientista nco
> identificaram outro
> evento? Ou outra maneira de esse hilio ter sido
> produzido?
>
> Quem encara?
>
> JR
>
>
>
>
>
> ##### ##### #####
>
> Para saber mais visite
> http://www.ciencialist.hpg.ig.com.br
>
>
> ##### ##### ##### #####
> Links do Yahoo! Grupos
>
>
>
> http://br.yahoo.com/info/utos.html
>
>
>
>
>
>



__________________________________
Do you Yahoo!?
Yahoo! Small Business - Try our new resources site!
http://smallbusiness.yahoo.com/resources/


SUBJECT: Re: [ciencialist] El interior del cuerpo humano
FROM: "Luiz Ferraz Netto" <leobarretos@uol.com.br>
TO: <ciencialist@yahoogrupos.com.br>
DATE: 25/03/2005 18:18

To Silvio & JVictor,

Como ilustração do corpo humano, a partir do interior, prefiro essa demonstração do artista russo.

http://fcmx.net/vec/v.php?i=003702

É só aguardar um pouco (local certo da página) que o desenho do artista se inicia.

[]'
===========================
Luiz Ferraz Netto [Léo]
leobarretos@uol.com.br
http://www.feiradeciencias.com.br
===========================


Franco escreveu:

> link:
> <http://elmundosalud.elmundo.es/elmundosalud/especiales/2005/03/galeria_cuerpo/index.html>
>
> Não sei me lembro se já foi postado aqui.
>
> Victor: Muito bom, excelente. Serve para todos já começarem a se
> preocupar com a saude.

Victor



SUBJECT: Re: Dúvida sobre o objetivo da lista
FROM: "junior_br2001" <junior_br2001@yahoo.com.br>
TO: ciencialist@yahoogrupos.com.br
DATE: 25/03/2005 18:23


--- Em ciencialist@yahoogrupos.com.br, Luis Brudna <luisbrudna@g...>
escreveu
>
> O que indico evitar na lista eh assuntos que envolvam religiao e
> politica partidaria.
>


JR: Ok Brudna fico grato com sua resposta.
A história por exemplo tem um parte da história do cristianismo, que
é o exatamente estava se levantando por aqui, como tb a história das
religiões e filosofias. É inevitável as vezes falarmos de história
sem que os mitos, as filosofiaas nao estejam envolvidos, história de
Roma e da Grécia estão repletas de deuses e do cristianismo, isso sem
falar daquelas do Egito ntigo e mesopotmia.
Então fica claro e evidente para voce que os debates que envolvam
religioes, como origem delas , repercussões, consequencias não sejam
de caráter religioso. É isso? ou esses debates podem ser
contraproducentes para a lista?

JR





SUBJECT: Re: Abundancia de hélio no universo
FROM: "junior_br2001" <junior_br2001@yahoo.com.br>
TO: ciencialist@yahoogrupos.com.br
DATE: 25/03/2005 18:31


É possível que o hélio já existisse em grandes proporções, bem antes
do Big Bang, ou até mesmo independente dele? Pelo que eu pude
entender para que haja hélio é preciso uma grande reação
termonuclear, como no sol em que na coroa fica praticamente formada
de gases leves como o hidrogenio e e hélio. Não seria possível uma
grande reação nuclear descohhecida tenha formado gases de hélio e
hidrogenio suficientes para eles serem encontrados em grandes
proporções no universo, mesmo que isso não signifique uma criação a
partir deste ponto?

Bem, eu defendo um universo que sempre existiu, não vejo motivos para
uma criação seja ela de natureza religiosa ou científica. Mas
gostaria de mais respostas, tipo mais opiniões e o conhecimento de
novas teorias

E obrigado pela indicação do livro
Abraços
JR



--- Em ciencialist@yahoogrupos.com.br, Eduardo Gueron <edgueron@y...>
escreveu
> Oi,
>
> Veja, não é a explosão em si que gera a abundância de
> Helio no Universo porque, em teoria, os núcleos
> atômicos se formam em uma determinada etapa da
> expansão do Universo (a nucleossíntese). De fato, na
> maioria das estrelas é produzido muito hélio por meio
> de fusão nuclear (no Sol por exemplo). Outros
> elementos são produzidos em menor escala e alguns só
> surgem quando a estrela explode.
>
> O que interessa, no entanto, é a distribuição de Helio
> e outros materiais leves que é quase uniforme ao longo
> do universo (note que como o He 4 é muito produzido
> nas estrelas, geralmente se usa a distribuição de
> outros elementos como Deutério, He 3 e Litio). Para
> que tais elementos fossem produzidos é necessário que,
> após a nucleossíntese (basicamente quando se formaram
> núcleos de H) o Universo tenha sido muito quente para
> que houvesse fusões nucleares (supondo ser impossível
> a fusão a frio) em um ambiente semelhante ao interior
> de uma estrela. E por que a distribuição desses
> elementos é importante? Simples, porque se mostra que,
> mesmo em regiões ausentes de estrelas (a maior parte
> do Universo) há mais ou menos 24% de He4 e uma alguns
> porcentos (não sei de cabeça) de cada um dos elementos
> acima citados. Há alternativas ao big-bang mas em
> todas que eu conheço, se assume que o Universo foi
> muito quente e pequeno em uma determinada época.
>
> Para uma boa referência, veja o livro do Kolb (The
> Early Universe). Procure na internet por História
> térmica do Universo (Thermal History of the Universe)
> que vc terá dados mais precisos. Os dados são
> impressionantemente precisos, vc prevê que a tal
> temperatura com tal concentração de H dever-se-ia
> formar X de Litio. O resultado medido é algo que
> difere de X na ordem de X/100.
>
> Abraço, Eduardo.
>
> --- junior_br2001 <junior_br2001@y...> wrote:
> >
> >
> > Existe algum modelo que pode explicar a abundncia de
> > hélio no
> > universo que no seja o evento primordial(Big bang)?
> >
> > Pelo que pude perceber a explosão de uma estrela
> > aumenta a proporção
> > de material mais denso que o hélio. Só o Big Bang
> > explica essa
> > abundancia de hélio no universo? Cientista não
> > identificaram outro
> > evento? Ou outra maneira de esse hélio ter sido
> > produzido?
> >
> > Quem encara?
> >
> > JR
> >
> >
> >
> >
> >
> > ##### ##### #####
> >
> > Para saber mais visite
> > http://www.ciencialist.hpg.ig.com.br
> >
> >
> > ##### ##### ##### #####
> > Links do Yahoo! Grupos
> >
> >
> >
> > http://br.yahoo.com/info/utos.html
> >
> >
> >
> >
> >
> >
>
>
>
> __________________________________
> Do you Yahoo!?
> Yahoo! Small Business - Try our new resources site!
> http://smallbusiness.yahoo.com/resources/





SUBJECT: Re: [ciencialist] Dúvida sobre o objetivo da lista
FROM: "Luiz Ferraz Netto" <leobarretos@uol.com.br>
TO: <ciencialist@yahoogrupos.com.br>
DATE: 25/03/2005 19:15

Olá Júnior,

Sou mais radical que o Brudna qto ao escopo da C-List. Para começar, as áreas de conhecimento humana que vc cita nada tem de Ciência (especialmente hard).
O que pode haver de científico na história do 'descobrimento do Brasil'? Científico é a análise da carta enviada pelo P.V.C. mediante as técnicas modernas (o PVC tb é moderno). Que há de científico numa lei jurídica? As leis jurídicas são 'meros textos de interpretação' (com volumosos livros --- de palavras de autoridade --- cada um dando sua 'doutoral' interpretação --- e, é por isso que subsistem os advogados (que tb são 'doutores' interpretativos) e nada é refutável.
Bolas! Psicologia virou Ciência? Ciência da Computação é Ciência?
Por que não Ciência da Religião? Por que não estudar as leis do criacionismo (dentro de um laboratório, com vários Adões (adãos ?) e Evas, após alguns anos já teremos uma boa clientela para passar a sacolinha um para os outros!).

Qdo vc diz "Esta é uma lista de ciencia. No entanto, aparentemente parece não ter
nenhuma restrição no aspecto de discussão do que é considerado
ciencia ou qualquer área do conhecimento humano.", fica claro que a lista foi 'criada' especificamente para discussões científicas. Todavia, como os participantes da lista AINDA são humanos, transportando enorme carga psicológica e emocional é de se esperar certos 'desvios' do tema básico. Certos humorismos, por exemplo, não pertencem ao campo científico, mas não deixa de ser humano e aceito por todos. Do mesmo modo, ninguém vai fazer restrição se por acaso, aqui, alguém postar uma receita de salgadinho que desça redondo junto com a skoll --- somos perfeitos, mais que perfeitos, quase um subjuntivo!
Alguns assuntos não científico, esporadicamente, são discutidos, mas não chegam a se tornarem 'cotidianos'. Duendes, florais, misticismos, história do Brasil, acupuntura, homeopatia, sudários e quimeras já passaram (rapidamente) por aqui. Nada restou que se aproveitasse, de concreto (argamassa, reboque ...).

Afinal, ao fim e ao cabo, aprendendo e desenvolvendo ciência podemos prestar algum auxílio à humanidade, mesmo que seja através da tecnologia. O misticismo nos dá água magnetizada (mesmo que seja por 50 ns), espíritos batedores, ectoplasmas com data de validade já vencida etc.

Viu????? Humorismo ninguém contesta!

aquele abraço,
===========================
Luiz Ferraz Netto [Léo]
leobarretos@uol.com.br
http://www.feiradeciencias.com.br
===========================
De: "junior_br2001"
Olá gente,

Esta é uma lista de ciencia. No entanto, aparentemente parece não ter
nenhuma restrição no aspecto de discussão do que é considerado
ciencia ou qualquer área do conhecimento humano. Hoje o conceito de
ciencia evoluiu, ela não é mais restrita somente ciencias naturais,
visto a dimensão de natureza investigativa que outras áreas do
conhecimento alcançaram como a História, a Sociologia, a Arqueologia,
a Psicologia, Antropologia, Ciência da Computação, etc ....e tl

Apesar de já ter pertencido uns 2 anos a esta lista no passado com
participação ativa e depois de estaar a aalguns meses afastado, a ela
novamente retornei , as vezes, fico meio sem graça de discutir outro
assunto está fora do escopo das ciencias naturais. Eu posso parecer
bastante desinibido na minha participação, mas é só aparencias, eu
tenho na verdde este receio e timidez.

Portanto gostaria uma opinião dos moderadores e de outros
participntes tb , se é lícito discutir outras áreras do conhecimento
humano que é considerado ciencia como a matemática, a história, a
sociologia,a arqueologia, a psicologia, a sociologia, ciencia da
computação, além é claro de todas as ciencias naturais : quimica ,
fisica , biologia, etc e tal

Digo isto, porque tenho interesse pelas outras áress de conhecimento
científico, e tb porque no gostaria de estar pagando micos, como
quando eu mando algumas mensagens referentes a história e a
arqueologia, assuntos bsastantes atraentes para mim

Portanto, opinião dos moderadores e principalmente do criador da
lista tem uma certa relevancia para mim, além é claro de
democraticamente , a opinião de todos os participante desta lista

Um grande abraço a todos
JR




SUBJECT: Re: Reflexão inversão em esp planos
FROM: Hélio Ricardo Carvalho <hrc@fis.puc-rio.br>
TO: ciencialist@yahoogrupos.com.br
DATE: 25/03/2005 19:59


Oi,

Estive em off por alguns dias e a quantidade de coisas interessantes
(e outras nem tanto) que apareceram aqui me deixam feliz de participar
desta lista. Ainda vou ler com mais atenção para ver se tenho alguma
contribuição a dar.

Mas, por ora, vou responder esta MSG:

--- Em ciencialist "PSavio" escreveu
> Prof.Léo e demais participantes desta Lista,
>
>
> Uma imagem num espelho plano apresenta simetira esquerda direita.
mas n apresenta simetria superior-inferior. Ainda estou estudando para
uma resposta satisfatória. Alguém tem alguma idéia?
>
>
> Pedro

Isto já deve ter aparecido aqui várias vezes. Durante este um ano e
meio que acompanho a C-list já apareceu pelo menos uma vez.
Eu já respondi mas não encontrei para dar a referência, então
responderei novamente.

Se você quer falar de SIMETRIA mesmo, fique a resposta do Eurico em:
http://br.groups.yahoo.com/group/ciencialist/message/45404
(só com um detalhe: meu cabelo é repartido de lado)

Mas se você fala em INVERSÃO:
Não há inversão horizontal nem vertical na imagem do espelho em
relação ao objeto. A única inversão é perpendicular ao plano do espelho.

Se você está dentro de uma sala de aula olhando para um espelho que
está entre você e o "quadro negro" (tb conhecido por lousa), o seu
braço sem relógio está apontando para a porta e o com relógio está
para janela, como fica a imagem?

Sua cabeça está para cima, a cabeça da imagem também.
Seus pés estão para baixo, os pés da imagem também.
Seu braço sem relógio está para a porta, o braço sem relógio da imagem
também.
Seu braço com relógio está para a janela, o braço com relógio da
imagem também.

MAS:
Seu nariz está para o quadro negro, o nariz da imagem está para a
parede de trás.
Seu traseiro está para a parede oposta ao quadro, O traseiro da imagem
está para o quadro.

Esta é a única inversão do espelho.

[ ]'s
Hélio.





SUBJECT: Re: Dúvida sobre o objetivo da lista
FROM: "junior_br2001" <junior_br2001@yahoo.com.br>
TO: ciencialist@yahoogrupos.com.br
DATE: 25/03/2005 21:03


Oi Leo fico grato com os comentários. Segue abaixo considerações

--- Em ciencialist@yahoogrupos.com.br, "Luiz Ferraz Netto"
<leobarretos@u...> escreveu
> Olá Júnior,
>
> Sou mais radical que o Brudna qto ao escopo da C-List. Para
começar, as áreas de conhecimento humana que vc cita nada tem de
Ciência (especialmente hard).
> O que pode haver de científico na história do 'descobrimento do
Brasil'?

JR: Creio que história é muito mais abrangente que a história do
Brasil. Ele é tb o registro de todo o cohecimento da humanidade,
inclusive o científico, e hoje talvez voce nao seria um professor se
não fosse a transmissão dos conhecimentos científicos através dos
tempo, o registro que temos da ciencia, e a autenticidade dos fatos e
dados. A história estuda as contribuições e a herança cultural da
humanidade no palco onde tb desenvolveu-se a ciencia. Parte do
computador que voce usa é física, mas tb é o esforço de outras formas
de conhecimento para criar o hardware e os softwares. Hoje ciencia e
tecnologia se misturam.
A história do Brasil tb contém a história de nossa ciencia que nao
pode ser esquecida, os trabalhos dos físicos , dos biólogos
brasileiros, seus métodos, fracasso. A investigação de todo esse
processo histórico brasileiro nos ajuda a entender os erros, os
fracassos , os sucessos e elaborar uma análise crítica que nos guirá
para rumos melhores, melhores métodos, acrescimos e correções dos
conhecimentos.


> Bolas! Psicologia virou Ciência? Ciência da Computação é Ciência?

JR: A psicologia é uma ciencia humana, e parte dessa ciencia é
estruturada pela neurociencia que é o entendimento dos processos
neurológicos e o impacto sobre a psique humna, e a psicologia estuda
o comportamento e o impacto que psique tem sobre os mecanismos
neurológicos. Existem, a etologia animal é uma parte da psicologia
evolucionista e da biologia que é o estudo do comportamento animal no
meio ambiente em que ele vive.
A ciencia da computação é ciencia, pois ela conta com programadores
que se utilizam de conhecimentos específicos, matemáticos e técnicos,
da mesma maneiera que é preciso saber física quantica, matemática e
eletronica para se criar o hardware







SUBJECT: Re: [ciencialist] Dúvida sobre o objetivo da lista
FROM: "Alvaro Augusto \(E\)" <alvaro@electraenergy.com.br>
TO: <ciencialist@yahoogrupos.com.br>
DATE: 25/03/2005 21:05

Caro Léo,

Sempre tenho a impressão que você entende por "ciência" somente aqueles ramos do conhecimento semelhantes à física. Mas nenhuma ciência é científica da maneira como a física é científica.

Psicologia é ciência, sim, pois é capaz de criar enunciados que podem ser verificados experimentalmente. Ocorre que tais enunciados não podem ser verificados com a mesma precisão com que se mede o momento magnético do elétron!

[ ]s

Alvaro Augusto


----- Original Message -----
From: Luiz Ferraz Netto
To: ciencialist@yahoogrupos.com.br
Sent: Friday, March 25, 2005 7:15 PM
Subject: Re: [ciencialist] Dúvida sobre o objetivo da lista


Olá Júnior,

Sou mais radical que o Brudna qto ao escopo da C-List. Para começar, as áreas de conhecimento humana que vc cita nada tem de Ciência (especialmente hard).
O que pode haver de científico na história do 'descobrimento do Brasil'? Científico é a análise da carta enviada pelo P.V.C. mediante as técnicas modernas (o PVC tb é moderno). Que há de científico numa lei jurídica? As leis jurídicas são 'meros textos de interpretação' (com volumosos livros --- de palavras de autoridade --- cada um dando sua 'doutoral' interpretação --- e, é por isso que subsistem os advogados (que tb são 'doutores' interpretativos) e nada é refutável.
Bolas! Psicologia virou Ciência? Ciência da Computação é Ciência?
Por que não Ciência da Religião? Por que não estudar as leis do criacionismo (dentro de um laboratório, com vários Adões (adãos ?) e Evas, após alguns anos já teremos uma boa clientela para passar a sacolinha um para os outros!).

Qdo vc diz "Esta é uma lista de ciencia. No entanto, aparentemente parece não ter
nenhuma restrição no aspecto de discussão do que é considerado
ciencia ou qualquer área do conhecimento humano.", fica claro que a lista foi 'criada' especificamente para discussões científicas. Todavia, como os participantes da lista AINDA são humanos, transportando enorme carga psicológica e emocional é de se esperar certos 'desvios' do tema básico. Certos humorismos, por exemplo, não pertencem ao campo científico, mas não deixa de ser humano e aceito por todos. Do mesmo modo, ninguém vai fazer restrição se por acaso, aqui, alguém postar uma receita de salgadinho que desça redondo junto com a skoll --- somos perfeitos, mais que perfeitos, quase um subjuntivo!
Alguns assuntos não científico, esporadicamente, são discutidos, mas não chegam a se tornarem 'cotidianos'. Duendes, florais, misticismos, história do Brasil, acupuntura, homeopatia, sudários e quimeras já passaram (rapidamente) por aqui. Nada restou que se aproveitasse, de concreto (argamassa, reboque ...).

Afinal, ao fim e ao cabo, aprendendo e desenvolvendo ciência podemos prestar algum auxílio à humanidade, mesmo que seja através da tecnologia. O misticismo nos dá água magnetizada (mesmo que seja por 50 ns), espíritos batedores, ectoplasmas com data de validade já vencida etc.

Viu????? Humorismo ninguém contesta!

aquele abraço,
===========================
Luiz Ferraz Netto [Léo]
leobarretos@uol.com.br
http://www.feiradeciencias.com.br
===========================
De: "junior_br2001"
Olá gente,

Esta é uma lista de ciencia. No entanto, aparentemente parece não ter
nenhuma restrição no aspecto de discussão do que é considerado
ciencia ou qualquer área do conhecimento humano. Hoje o conceito de
ciencia evoluiu, ela não é mais restrita somente ciencias naturais,
visto a dimensão de natureza investigativa que outras áreas do
conhecimento alcançaram como a História, a Sociologia, a Arqueologia,
a Psicologia, Antropologia, Ciência da Computação, etc ....e tl

Apesar de já ter pertencido uns 2 anos a esta lista no passado com
participação ativa e depois de estaar a aalguns meses afastado, a ela
novamente retornei , as vezes, fico meio sem graça de discutir outro
assunto está fora do escopo das ciencias naturais. Eu posso parecer
bastante desinibido na minha participação, mas é só aparencias, eu
tenho na verdde este receio e timidez.

Portanto gostaria uma opinião dos moderadores e de outros
participntes tb , se é lícito discutir outras áreras do conhecimento
humano que é considerado ciencia como a matemática, a história, a
sociologia,a arqueologia, a psicologia, a sociologia, ciencia da
computação, além é claro de todas as ciencias naturais : quimica ,
fisica , biologia, etc e tal

Digo isto, porque tenho interesse pelas outras áress de conhecimento
científico, e tb porque no gostaria de estar pagando micos, como
quando eu mando algumas mensagens referentes a história e a
arqueologia, assuntos bsastantes atraentes para mim

Portanto, opinião dos moderadores e principalmente do criador da
lista tem uma certa relevancia para mim, além é claro de
democraticamente , a opinião de todos os participante desta lista

Um grande abraço a todos
JR



[As partes desta mensagem que não continham texto foram removidas]



SUBJECT: Re: [ciencialist] Re: Criacionismo
FROM: JVictor <jvoneto@uol.com.br>
TO: ciencialist@yahoogrupos.com.br
DATE: 25/03/2005 21:06

Só queria saber uma coisa, liás, two:

se ao invés de Barrabás, os mui amigos judeus tivessem livrado Jesus da
crucificação:

1-como seria a Igreja e seus seguidores, hoje?

2 - e nós, seríamos cristãos ou o quê?

Por nada não, por nada não...

Sds,

Victor.






Alvaro Augusto (E) escreveu:

> Caro Junior,
>
> O cristianismo nasceu na Judéia, que ficava bem pra lá do meridiano de
> Greenwitch. Logo, o cristianismo também é uma filosofia oriental...
>
> [ ]s
>
> Alvaro Augusto
>
> ----- Original Message -----
> From: junior_br2001
> To: ciencialist@yahoogrupos.com.br
> Sent: Thursday, March 24, 2005 5:14 PM
> Subject: [ciencialist] Re: Criacionismo
>
>
>
> Não sei até onde aceitar a teoria cíclica sobre a origem do universo,
> seja criacionismo.
> Eu respondi porque senti que a indireta foi para mim, e porque eu
> tava defendendo a possibilidade do Sudário ter 3000 anos.
>
> Mas tem gente que me conhece aqui e pode responder por mim. Sempre
> afirmei que gosto das filosofias orientais. Se eu tivesse que ser um
> religiosos eu seria um budista, mas nunca um cristão, mesmo porque
> acho o cristianismo muito ridículo
>
> Até,
> JR
>
>
> -- Em ciencialist@yahoogrupos.com.br, "Mr. Vain" <mr.vain@g...>
> escreveu
> >
> > Parece-me que temos um criacionista disfarçado na lista.
> >
> > [ ]'s
> >
> > Dr. Vain
> > Humanitate Magis
> > Quam Religione
> > Nobis Opus Est
>
>
>
> [As partes desta mensagem que não continham texto foram removidas]
>
>
>
> ##### ##### #####
>
> Para saber mais visite
> http://www.ciencialist.hpg.ig.com.br
>
>
> ##### ##### ##### #####
>
>
> *Yahoo! Grupos, um serviço oferecido por:*
>
> *
> <http://br.rd.yahoo.com/SIG=12a4r7kve/M=264105.3931087.6562589.1588051/D=brclubs/S=2137111528:HM/EXP=1111787186/A=2361264/R=6/SIG=10v4acpp0/*http://br.shopping.yahoo.com/>*
>
>
>
> ------------------------------------------------------------------------
> *Links do Yahoo! Grupos*
>
> * Para visitar o site do seu grupo na web, acesse:
> http://br.groups.yahoo.com/group/ciencialist/
>
> * Para sair deste grupo, envie um e-mail para:
> ciencialist-unsubscribe@yahoogrupos.com.br
> <mailto:ciencialist-unsubscribe@yahoogrupos.com.br?subject=Unsubscribe>
>
> * O uso que você faz do Yahoo! Grupos está sujeito aos Termos do
> Serviço do Yahoo! <http://br.yahoo.com/info/utos.html>.
>
>
>
>
> __________ Informação do NOD32 1.1033 (20050323) __________
>
> Esta mensagem foi verificada pelo NOD32 Sistema Antivírus
> http://www.nod32.com.br




SUBJECT: Dúvida sobre o objetivo da lista
FROM: "L.E.R.de Carvalho" <lecarvalho@infolink.com.br>
TO: ciencialist@yahoogrupos.com.br
DATE: 25/03/2005 21:32

At 21:03 25/3/2005, you wrote:

>Oi Leo fico grato com os comentários. Segue abaixo considerações
>
>--- Em ciencialist@yahoogrupos.com.br, "Luiz Ferraz Netto"
><leobarretos@u...> escreveu
> > Olá Júnior,
> >
> > Sou mais radical que o Brudna qto ao escopo da C-List. Para
>começar, as áreas de conhecimento humana que vc cita nada tem de
>Ciência (especialmente hard).
> > O que pode haver de científico na história do 'descobrimento do
>Brasil'?




E´...
deve ser História Quantica.

L.E.










-
-
-
-
-
-
-

[As partes desta mensagem que não continham texto foram removidas]



SUBJECT: Re: Dúvida sobre o objetivo da lista
FROM: Luis Brudna <luisbrudna@gmail.com>
TO: ciencialist@yahoogrupos.com.br
DATE: 25/03/2005 21:43

No mail que editei ateh pensei em colocar ressalvas quanto a debates
religiosos, mas acabei eliminando. Estava sem paciencia para detalhes.
:-)

Agora os detalhes.

O debate religioso, que nao eh indicado na Ciencialist, eh o que
envolve elementos de feh, de crenca em divindades.

As consequencias das religioes em uma sociedade podem ser sim objeto
de debates na lista. Desde que mantenham uma distancia dos dogmas de
cada religiao.

Ateh
Luis Brudna


> --- Em ciencialist@yahoogrupos.com.br, Luis Brudna <luisbrudna@g...>
> escreveu
> >
> > O que indico evitar na lista eh assuntos que envolvam religiao e
> > politica partidaria.
> >
>
>
> JR: Ok Brudna fico grato com sua resposta.
> A história por exemplo tem um parte da história do cristianismo, que
> é o exatamente estava se levantando por aqui, como tb a história das
> religiões e filosofias. É inevitável as vezes falarmos de história
> sem que os mitos, as filosofiaas nao estejam envolvidos, história de
> Roma e da Grécia estão repletas de deuses e do cristianismo, isso sem
> falar daquelas do Egito ntigo e mesopotmia.
> Então fica claro e evidente para voce que os debates que envolvam
> religioes, como origem delas , repercussões, consequencias não sejam
> de caráter religioso. É isso? ou esses debates podem ser
> contraproducentes para a lista?
>
> JR


SUBJECT: Re: Abundancia de hélio no universo
FROM: Luis Brudna <luisbrudna@gmail.com>
TO: ciencialist@yahoogrupos.com.br
DATE: 25/03/2005 21:49

Serah que interpretacao do Big Bang como 'criacao do universo sob
otica cientifica' eh saudavel?

Penso no Big Bang como um evento que deu origem ao universo como
conhecemos, nao como algo "onde tudo teve inicio".

Serah que Big Bang necessariamente elimina o 'sempre-existencia'?

Ateh
Luis Brudna


On Fri, 25 Mar 2005 21:31:51 -0000, junior_br2001
<junior_br2001@yahoo.com.br> wrote:
> Bem, eu defendo um universo que sempre existiu, não vejo motivos para
> uma criação seja ela de natureza religiosa ou científica. Mas
> gostaria de mais respostas, tipo mais opiniões e o conhecimento de
> novas teorias
> E obrigado pela indicação do livro
> Abraços
> JR


SUBJECT: Re: [ciencialist] Re: Abundancia de hélio no universo
FROM: "Ivan Carlos" <icarlos@icarlos.net>
TO: <ciencialist@yahoogrupos.com.br>
DATE: 25/03/2005 21:57

não elimina, simplesmente pq o big bang está sendo re-colocado em sua teoria
como "Big Crunch" justamente por poder se tratar de um ciclo, de infinitas
agregações e expansões

alias.. o universo É o nada, a duvida mor é a sempre existência do conteúdo
existente neste universo...

Ivan "Doomer" Carlos
Social Engineering Specialist
Cell.: +55 (11) 8112-0666
icarlos@icarlos.net
www.icarlos.net
--------------------------------------------------


----- Original Message -----
From: "Luis Brudna" <luisbrudna@gmail.com>
To: <ciencialist@yahoogrupos.com.br>
Sent: Friday, March 25, 2005 9:49 PM
Subject: [ciencialist] Re: Abundancia de hélio no universo



Serah que interpretacao do Big Bang como 'criacao do universo sob
otica cientifica' eh saudavel?

Penso no Big Bang como um evento que deu origem ao universo como
conhecemos, nao como algo "onde tudo teve inicio".

Serah que Big Bang necessariamente elimina o 'sempre-existencia'?

Ateh
Luis Brudna


On Fri, 25 Mar 2005 21:31:51 -0000, junior_br2001
<junior_br2001@yahoo.com.br> wrote:
> Bem, eu defendo um universo que sempre existiu, não vejo motivos para
> uma criação seja ela de natureza religiosa ou científica. Mas
> gostaria de mais respostas, tipo mais opiniões e o conhecimento de
> novas teorias
> E obrigado pela indicação do livro
> Abraços
> JR


##### ##### #####

Para saber mais visite
http://www.ciencialist.hpg.ig.com.br


##### ##### ##### #####
Links do Yahoo! Grupos




SUBJECT: Re: [ciencialist] El interior del cuerpo humano
FROM: JVictor <jvoneto@uol.com.br>
TO: ciencialist@yahoogrupos.com.br
DATE: 26/03/2005 11:04

Franco escreveu:

> link:
> <http://elmundosalud.elmundo.es/elmundosalud/especiales/2005/03/galeria_cuerpo/index.html>
>
> Não sei me lembro se já foi postado aqui.
>
> Victor: Muito bom, excelente. Serve para todos já começarem a se
> preocupar com a saude.

Victor

>
>
>
> ##### ##### #####
>
> Para saber mais visite
> http://www.ciencialist.hpg.ig.com.br
>
>
> ##### ##### ##### #####
>
>
> *Yahoo! Grupos, um serviço oferecido por:*
>
> *
> <http://br.rd.yahoo.com/SIG=12a7rq7qc/M=264105.3931087.6562589.1588051/D=brclubs/S=2137111528:HM/EXP=1111815385/A=2361264/R=6/SIG=10v4acpp0/*http://br.shopping.yahoo.com/>*
>
>
>
> ------------------------------------------------------------------------
> *Links do Yahoo! Grupos*
>
> * Para visitar o site do seu grupo na web, acesse:
> http://br.groups.yahoo.com/group/ciencialist/
>
> * Para sair deste grupo, envie um e-mail para:
> ciencialist-unsubscribe@yahoogrupos.com.br
> <mailto:ciencialist-unsubscribe@yahoogrupos.com.br?subject=Unsubscribe>
>
> * O uso que você faz do Yahoo! Grupos está sujeito aos Termos do
> Serviço do Yahoo! <http://br.yahoo.com/info/utos.html>.
>
>
>
>
> __________ Informação do NOD32 1.1035 (20050325) __________
>
> Esta mensagem foi verificada pelo NOD32 Sistema Antivírus
> http://www.nod32.com.br




SUBJECT: Reflexão inversão em esp planos
FROM: "PSavio" <psdmo@uol.com.br>
TO: <ciencialist@yahoogrupos.com.br>
DATE: 26/03/2005 12:52

Prof.Léo e demais participantes desta Lista,


Uma imagem num espelho plano apresenta simetira esquerda direita. mas n apresenta simetria superior-inferior. Ainda estou estudando para uma resposta satisfatória. Alguém tem alguma idéia?


Pedro

[As partes desta mensagem que não continham texto foram removidas]



SUBJECT: [Re: [ciencialist] Reflexão inversão em esp planos]
FROM: JVictor <jvoneto@uol.com.br>
TO: "ciencialist@yahoogrupos.com.br" <ciencialist@yahoogrupos.com.br>
DATE: 26/03/2005 16:36


PSavio wrote:

>Prof.Léo e demais participantes desta Lista,
Uma imagem num espelho plano apresenta simetira esquerda direita. mas n
apresenta simetria superior-inferior. Ainda estou estudando para uma
resposta satisfatória. Alguém tem alguma idéia?

Victor: O espelho não apresenta simetria lateral, seja para esquerda,
seja para a direita, ou para cima e para baixo. A inversão é do tipo
front-back, frente-trás ou frente-verso.
Se você simular o sistema de eixos tri-ortogonal com os dedos, e
apresentá-lo ao espelho, verá que os eixos paralelos ao plano do espelho
continuam sem inversão, a passo que o outro eixo, o perpendicular ao
plano do espelho, este sim, você o verá invertidinho da silva.
Outra experiência, para dissipar as dúvidas. Vá para um espelho, e, ao
invés de ficar se narcisando, faça assim: ponha as costas de sua mão
direita para você e a palma dessa mão para o espelho; para tornar a
experiência mais interessante, dobre os dedos um pouco para dentro, como
se fosse fechar a mão, mas sem o fazer totalamente. Nessa situação, o
seu polegar estará virado para sua esquerda, sentido negativo dos eixo
dos x. O que você verá? Claro, a palma da mão, dedos dobrados e o
polegar no mesmo sentido. Mantenha essa mão assim, e faça outro seguinte
com a mão esquerda: dobre os dedos da mesma maneira, e ponha-a diante
do espelho, com as costas para ele e a palma dessa mão, dobrada como a
direita, para você. Observe agora: a imagem de sua mão direita, que é
a palma de sua mão direita, que é igualzinha à palma de sua mão
esquerda, do lado de cá!. Ou seja, os dois polegares lá e cá têm o
mesmo sentido. E você vê que a imagem da palma de sua mão direita é
igual à de sua mão esquerda. Mas, você está com a mão direita com as
costas viradas para você e está vendo a palma da mão! Logo, não há
qualquer simetria lateral, nem dum lado nem do outro. A simetria é
frente-verso! Para esclarecer isso, lembre que se o meio reflexivo for
mais refringente, a parcela de luz que se reflete sempre terá sua fase
invertida(180graus), portanto apresenta inversão de amplitude.

É por aí.

Sds,

Victor.




[As partes desta mensagem que não continham texto foram removidas]